{"question": "\ud504\ub79c\uc2dc\uc2a4\uc640 \uad00\uacc4\uac00 \uc88b\uc9c0 \uc54a\uc558\ub358 \ub85c\ud0a4\uce20\uc758 \ucf54\uce58 \uc774\ub984\uc740?", "paragraph": "\ud504\ub79c\uc2dc\uc2a4\ub294 \uc57c\uc624 \ubc0d\uacfc \uac19\uc774 \ud504\ub808\uc774\uc624\ud504\uc9c4\ucd9c\uc5d0 \uc131\uacf5\uc744\ud588\uc9c0\ub9cc \ub2f9\uc2dc \ub85c\ud0a4\uce20\uc758 \ucf54\uce58\uc600\ub358 \uc81c\ud504 \ubc34 \uac74\ub514\ub791\uc740 \uc0ac\uc774\uac00 \uc548 \uc88b\uc558\ub2e4. \ubc34 \uac74\ub514 \ucf54\uce58\ub294 \uc57c\uc624 \ubc0d\uc744 \uacf5\uaca9\uc758 \uc911\uc2ec\uc73c\ub85c \uc7a1\uace0 \uc2f6\uc5c8\uace0 \ud504\ub79c\uc2dc\uc2a4\ub294 \ud300\uc758 \ub9ac\ub354\ub85c\uc11c \uc790\uae30\uac00 \ud300\uc744 \uc774\ub04c\uc5b4\uc57c \ub41c\ub2e4\uace0 \uc0dd\uac01\ud588\ub2e4.\uc774\ub7f0 \uc758\uacac \ucda9\ub3cc\uacfc \ud504\ub79c\uc2dc\uc2a4\uc758 \uc800\uc870\ud55c \uc2e4\uc801\uc740 \ub610 \ub2e4\ub978 \ud2b8\ub808\uc774\ub4dc\ub85c \uacb0\ub860\ub0ac\ub2e4. \ub85c\ud0a4\uce20\ub294 \uc62c\ub79c\ub3c4 \ub9e4\uc9c1\uc758 \uc288\ud37c\uc2a4\ud0c0 \ud2b8\ub808\uc774\uc2dc \ub9e5\uadf8\ub798\ub514\ub97c \uc601\uc785\ud558\uae30 \uc704\ud558\uc5ec \ud504\ub79c\uc2dc\uc2a4\uc640 \uadf8 \uc678 2\uba85\uc758 \uc120\uc218\ub97c \ud2b8\ub808\uc774\ub4dc \ud588\ub2e4. \ud504\ub79c\uc2dc\uc2a4\ub294 \uc774 \ud2b8\ub808\uc774\ub4dc\ub97c \ubd88\ub9cc\uc871\uc2a4\ub7fd\uac8c \uc5ec\uacbc\uc9c0\ub9cc \ube68\ub9ac \uc801\uc751\ud588\ub2e4. \uadf8\ub294 \ud3c9\uade0 21\uc810, 5.8\ub9ac\ubc14\uc6b4\ub4dc, 7\ub3c4\uc6c0\uc744 \uae30\ub85d\ud558\uba70 \ub9e4\uc9c1\uc5d0\uc11c\uc758 \uccab \uc2dc\uc98c\uc744 \ud654\ub824\ud558\uac8c \uc2dc\uc791\ud588\ub2e4. \ud558\uc9c0\ub9cc \ub9e4\uc9c1\uc740 \ud50c\ub808\uc774\uc624\ud504 \uc9c4\ucd9c\uc5d0 \uc2e4\ud328\ud558\uc600\ub2e4. \ub2e4\uc2dc \ud55c\ubc88 \ud504\ub79c\uc2dc\uc2a4\ub294 \uc790\uae30\uc640 \ub9e4\uc9c1\uacfc\uc758 \uc758\uacac \ucc28\uc758\uac00 \uc788\ub2e4\uba70 \ubd88\ub9cc\uc871\ud558\ub2e4\ub294 \ud0dc\ub3c4\ub97c \ubcf4\uc600\uace0 \uc774\uc5d0 \uc218\ub9ce\uc740 \ud2b8\ub808\uc774\ub4dc \uc18c\ubb38\uc5d0 \uc911\uc2ec\uc774 \ub410\ub2e4.", "answer": "\uc81c\ud504 \ubc34 \uac74\ub514\ub791", "sentence": "\ub2f9\uc2dc \ub85c\ud0a4\uce20\uc758 \ucf54\uce58\uc600\ub358 \uc81c\ud504 \ubc34 \uac74\ub514\ub791 \uc740 \uc0ac\uc774\uac00 \uc548 \uc88b\uc558\ub2e4.", "paragraph_sentence": "\ud504\ub79c\uc2dc\uc2a4\ub294 \uc57c\uc624 \ubc0d\uacfc \uac19\uc774 \ud504\ub808\uc774\uc624\ud504\uc9c4\ucd9c\uc5d0 \uc131\uacf5\uc744\ud588\uc9c0\ub9cc \ub2f9\uc2dc \ub85c\ud0a4\uce20\uc758 \ucf54\uce58\uc600\ub358 \uc81c\ud504 \ubc34 \uac74\ub514\ub791 \uc740 \uc0ac\uc774\uac00 \uc548 \uc88b\uc558\ub2e4. \ubc34 \uac74\ub514 \ucf54\uce58\ub294 \uc57c\uc624 \ubc0d\uc744 \uacf5\uaca9\uc758 \uc911\uc2ec\uc73c\ub85c \uc7a1\uace0 \uc2f6\uc5c8\uace0 \ud504\ub79c\uc2dc\uc2a4\ub294 \ud300\uc758 \ub9ac\ub354\ub85c\uc11c \uc790\uae30\uac00 \ud300\uc744 \uc774\ub04c\uc5b4\uc57c \ub41c\ub2e4\uace0 \uc0dd\uac01\ud588\ub2e4.\uc774\ub7f0 \uc758\uacac \ucda9\ub3cc\uacfc \ud504\ub79c\uc2dc\uc2a4\uc758 \uc800\uc870\ud55c \uc2e4\uc801\uc740 \ub610 \ub2e4\ub978 \ud2b8\ub808\uc774\ub4dc\ub85c \uacb0\ub860\ub0ac\ub2e4. \ub85c\ud0a4\uce20\ub294 \uc62c\ub79c\ub3c4 \ub9e4\uc9c1\uc758 \uc288\ud37c\uc2a4\ud0c0 \ud2b8\ub808\uc774\uc2dc \ub9e5\uadf8\ub798\ub514\ub97c \uc601\uc785\ud558\uae30 \uc704\ud558\uc5ec \ud504\ub79c\uc2dc\uc2a4\uc640 \uadf8 \uc678 2\uba85\uc758 \uc120\uc218\ub97c \ud2b8\ub808\uc774\ub4dc \ud588\ub2e4. \ud504\ub79c\uc2dc\uc2a4\ub294 \uc774 \ud2b8\ub808\uc774\ub4dc\ub97c \ubd88\ub9cc\uc871\uc2a4\ub7fd\uac8c \uc5ec\uacbc\uc9c0\ub9cc \ube68\ub9ac \uc801\uc751\ud588\ub2e4. \uadf8\ub294 \ud3c9\uade0 21\uc810, 5.8\ub9ac\ubc14\uc6b4\ub4dc, 7\ub3c4\uc6c0\uc744 \uae30\ub85d\ud558\uba70 \ub9e4\uc9c1\uc5d0\uc11c\uc758 \uccab \uc2dc\uc98c\uc744 \ud654\ub824\ud558\uac8c \uc2dc\uc791\ud588\ub2e4. \ud558\uc9c0\ub9cc \ub9e4\uc9c1\uc740 \ud50c\ub808\uc774\uc624\ud504 \uc9c4\ucd9c\uc5d0 \uc2e4\ud328\ud558\uc600\ub2e4. \ub2e4\uc2dc \ud55c\ubc88 \ud504\ub79c\uc2dc\uc2a4\ub294 \uc790\uae30\uc640 \ub9e4\uc9c1\uacfc\uc758 \uc758\uacac \ucc28\uc758\uac00 \uc788\ub2e4\uba70 \ubd88\ub9cc\uc871\ud558\ub2e4\ub294 \ud0dc\ub3c4\ub97c \ubcf4\uc600\uace0 \uc774\uc5d0 \uc218\ub9ce\uc740 \ud2b8\ub808\uc774\ub4dc \uc18c\ubb38\uc5d0 \uc911\uc2ec\uc774 \ub410\ub2e4.", "paragraph_answer": "\ud504\ub79c\uc2dc\uc2a4\ub294 \uc57c\uc624 \ubc0d\uacfc \uac19\uc774 \ud504\ub808\uc774\uc624\ud504\uc9c4\ucd9c\uc5d0 \uc131\uacf5\uc744\ud588\uc9c0\ub9cc \ub2f9\uc2dc \ub85c\ud0a4\uce20\uc758 \ucf54\uce58\uc600\ub358 \uc81c\ud504 \ubc34 \uac74\ub514\ub791 \uc740 \uc0ac\uc774\uac00 \uc548 \uc88b\uc558\ub2e4. \ubc34 \uac74\ub514 \ucf54\uce58\ub294 \uc57c\uc624 \ubc0d\uc744 \uacf5\uaca9\uc758 \uc911\uc2ec\uc73c\ub85c \uc7a1\uace0 \uc2f6\uc5c8\uace0 \ud504\ub79c\uc2dc\uc2a4\ub294 \ud300\uc758 \ub9ac\ub354\ub85c\uc11c \uc790\uae30\uac00 \ud300\uc744 \uc774\ub04c\uc5b4\uc57c \ub41c\ub2e4\uace0 \uc0dd\uac01\ud588\ub2e4.\uc774\ub7f0 \uc758\uacac \ucda9\ub3cc\uacfc \ud504\ub79c\uc2dc\uc2a4\uc758 \uc800\uc870\ud55c \uc2e4\uc801\uc740 \ub610 \ub2e4\ub978 \ud2b8\ub808\uc774\ub4dc\ub85c \uacb0\ub860\ub0ac\ub2e4. \ub85c\ud0a4\uce20\ub294 \uc62c\ub79c\ub3c4 \ub9e4\uc9c1\uc758 \uc288\ud37c\uc2a4\ud0c0 \ud2b8\ub808\uc774\uc2dc \ub9e5\uadf8\ub798\ub514\ub97c \uc601\uc785\ud558\uae30 \uc704\ud558\uc5ec \ud504\ub79c\uc2dc\uc2a4\uc640 \uadf8 \uc678 2\uba85\uc758 \uc120\uc218\ub97c \ud2b8\ub808\uc774\ub4dc \ud588\ub2e4. \ud504\ub79c\uc2dc\uc2a4\ub294 \uc774 \ud2b8\ub808\uc774\ub4dc\ub97c \ubd88\ub9cc\uc871\uc2a4\ub7fd\uac8c \uc5ec\uacbc\uc9c0\ub9cc \ube68\ub9ac \uc801\uc751\ud588\ub2e4. \uadf8\ub294 \ud3c9\uade0 21\uc810, 5.8\ub9ac\ubc14\uc6b4\ub4dc, 7\ub3c4\uc6c0\uc744 \uae30\ub85d\ud558\uba70 \ub9e4\uc9c1\uc5d0\uc11c\uc758 \uccab \uc2dc\uc98c\uc744 \ud654\ub824\ud558\uac8c \uc2dc\uc791\ud588\ub2e4. \ud558\uc9c0\ub9cc \ub9e4\uc9c1\uc740 \ud50c\ub808\uc774\uc624\ud504 \uc9c4\ucd9c\uc5d0 \uc2e4\ud328\ud558\uc600\ub2e4. \ub2e4\uc2dc \ud55c\ubc88 \ud504\ub79c\uc2dc\uc2a4\ub294 \uc790\uae30\uc640 \ub9e4\uc9c1\uacfc\uc758 \uc758\uacac \ucc28\uc758\uac00 \uc788\ub2e4\uba70 \ubd88\ub9cc\uc871\ud558\ub2e4\ub294 \ud0dc\ub3c4\ub97c \ubcf4\uc600\uace0 \uc774\uc5d0 \uc218\ub9ce\uc740 \ud2b8\ub808\uc774\ub4dc \uc18c\ubb38\uc5d0 \uc911\uc2ec\uc774 \ub410\ub2e4.", "sentence_answer": "\ub2f9\uc2dc \ub85c\ud0a4\uce20\uc758 \ucf54\uce58\uc600\ub358 \uc81c\ud504 \ubc34 \uac74\ub514\ub791 \uc740 \uc0ac\uc774\uac00 \uc548 \uc88b\uc558\ub2e4.", "paragraph_id": "6519214-1-0", "paragraph_question": "question: \ud504\ub79c\uc2dc\uc2a4\uc640 \uad00\uacc4\uac00 \uc88b\uc9c0 \uc54a\uc558\ub358 \ub85c\ud0a4\uce20\uc758 \ucf54\uce58 \uc774\ub984\uc740?, context: \ud504\ub79c\uc2dc\uc2a4\ub294 \uc57c\uc624 \ubc0d\uacfc \uac19\uc774 \ud504\ub808\uc774\uc624\ud504\uc9c4\ucd9c\uc5d0 \uc131\uacf5\uc744\ud588\uc9c0\ub9cc \ub2f9\uc2dc \ub85c\ud0a4\uce20\uc758 \ucf54\uce58\uc600\ub358 \uc81c\ud504 \ubc34 \uac74\ub514\ub791\uc740 \uc0ac\uc774\uac00 \uc548 \uc88b\uc558\ub2e4. \ubc34 \uac74\ub514 \ucf54\uce58\ub294 \uc57c\uc624 \ubc0d\uc744 \uacf5\uaca9\uc758 \uc911\uc2ec\uc73c\ub85c \uc7a1\uace0 \uc2f6\uc5c8\uace0 \ud504\ub79c\uc2dc\uc2a4\ub294 \ud300\uc758 \ub9ac\ub354\ub85c\uc11c \uc790\uae30\uac00 \ud300\uc744 \uc774\ub04c\uc5b4\uc57c \ub41c\ub2e4\uace0 \uc0dd\uac01\ud588\ub2e4.\uc774\ub7f0 \uc758\uacac \ucda9\ub3cc\uacfc \ud504\ub79c\uc2dc\uc2a4\uc758 \uc800\uc870\ud55c \uc2e4\uc801\uc740 \ub610 \ub2e4\ub978 \ud2b8\ub808\uc774\ub4dc\ub85c \uacb0\ub860\ub0ac\ub2e4. \ub85c\ud0a4\uce20\ub294 \uc62c\ub79c\ub3c4 \ub9e4\uc9c1\uc758 \uc288\ud37c\uc2a4\ud0c0 \ud2b8\ub808\uc774\uc2dc \ub9e5\uadf8\ub798\ub514\ub97c \uc601\uc785\ud558\uae30 \uc704\ud558\uc5ec \ud504\ub79c\uc2dc\uc2a4\uc640 \uadf8 \uc678 2\uba85\uc758 \uc120\uc218\ub97c \ud2b8\ub808\uc774\ub4dc \ud588\ub2e4. \ud504\ub79c\uc2dc\uc2a4\ub294 \uc774 \ud2b8\ub808\uc774\ub4dc\ub97c \ubd88\ub9cc\uc871\uc2a4\ub7fd\uac8c \uc5ec\uacbc\uc9c0\ub9cc \ube68\ub9ac \uc801\uc751\ud588\ub2e4. \uadf8\ub294 \ud3c9\uade0 21\uc810, 5.8\ub9ac\ubc14\uc6b4\ub4dc, 7\ub3c4\uc6c0\uc744 \uae30\ub85d\ud558\uba70 \ub9e4\uc9c1\uc5d0\uc11c\uc758 \uccab \uc2dc\uc98c\uc744 \ud654\ub824\ud558\uac8c \uc2dc\uc791\ud588\ub2e4. \ud558\uc9c0\ub9cc \ub9e4\uc9c1\uc740 \ud50c\ub808\uc774\uc624\ud504 \uc9c4\ucd9c\uc5d0 \uc2e4\ud328\ud558\uc600\ub2e4. \ub2e4\uc2dc \ud55c\ubc88 \ud504\ub79c\uc2dc\uc2a4\ub294 \uc790\uae30\uc640 \ub9e4\uc9c1\uacfc\uc758 \uc758\uacac \ucc28\uc758\uac00 \uc788\ub2e4\uba70 \ubd88\ub9cc\uc871\ud558\ub2e4\ub294 \ud0dc\ub3c4\ub97c \ubcf4\uc600\uace0 \uc774\uc5d0 \uc218\ub9ce\uc740 \ud2b8\ub808\uc774\ub4dc \uc18c\ubb38\uc5d0 \uc911\uc2ec\uc774 \ub410\ub2e4."} {"question": "\ubc00\ub9ac\uc5b8 \ub2ec\ub7ec \ud648\ud398\uc774\uc9c0\uc758 100\uac1c \ud654\uc18c\ube14\ub85d\ub2e8\uc704 \ucd5c\uc18c\ud310\ub9e4 \uac00\uaca9\uc740?", "paragraph": "\uac01 \ud654\uc18c\ub294 \ub108\ubb34 \uc791\uc544\uc11c \uc27d\uac8c \ubcf4\uc774\uc9c0 \uc54a\uc558\ub2e4. \uc774 \uae4c\ub2ed\uc5d0 \ud654\uc18c\ub4e4\uc740 10 \u00d7 10 \ud654\uc18c \ub2e8\uc704\uc778 100\uac1c\uc758 \ud654\uc18c \ube14\ub85d \ub2e8\uc704\ub85c \ud310\ub9e4\ub418\uc5c8\ub2e4. \uadf8\ub7ec\ubbc0\ub85c \ucd5c\uc18c \ud310\ub9e4 \uac00\uaca9\uc740 $100\uc774\ub2e4. \uc0ac\uc774\ud2b8 \uc6b4\uc601\uc744 \uc2dc\uc791\ud55c\uc9c0 \uc0ac\ud758\ub9cc\uc5d0 \ud29c\uc758 \uce5c\uad6c\uac00 \uc6b4\uc601\ud558\ub294 \uc628\ub77c\uc778 \uc74c\uc545 \uc6f9\uc0ac\uc774\ud2b8\uc5d0 \uc5f0\uacb0\ub418\uba74\uc11c \ud310\ub9e4\uac00 \ucc98\uc74c\uc73c\ub85c \uc2dc\uc791\ub418\uc5c8\ub2e4. \uadf8\ub294 400\uac1c\uc758 \ud654\uc18c(20 \u00d7 20 \ube14\ub85d)\ub97c \uc0c0\ub2e4. 2\uc8fc\uac00 \uc9c0\ub098 \ud29c\uc758 \uce5c\uad6c\ub4e4\uacfc \uac00\uc871\ub4e4\uc740 \ubaa8\ub450 4,700\uac1c\uc758 \ud654\uc18c\ub97c \uad6c\ub9e4\ud558\uc600\ub2e4. \uc774 \uc0ac\uc774\ud2b8\ub294 \ucd08\uae30\uc5d0 \uad6c\uc804 \ub9c8\ucf00\ud305\uc73c\ub85c\ub9cc\uc73c\ub85c \ubc8c\uc5ec\ub098\uac14\ub2e4. \uadf8\ub7ec\ub098 \uc774 \uc0ac\uc774\ud2b8\uac00 $1,000\ub97c \ubc8c\uc5b4\ub4e4\uc774\uc790 \uc5b8\ub860 \uae30\uc0ac\uac00 BBC\ub97c \ud1b5\ud574 \ubcf4\ub3c4\ub418\uc5c8\ub2e4. \uae30\uc220 \ub274\uc2a4 \uc6f9\uc0ac\uc774\ud2b8 \ub354 \ub808\uc9c0\uc2a4\ud130\ub294 9\uc6d4\uc5d0 \ubc00\ub9ac\uc5b8 \ub2ec\ub7ec \ud648\ud398\uc774\uc9c0\uc5d0 \ub300\ud55c \ub450 \uac1c\uc758 \uae30\uc0ac\ub97c \uc7a5\uc2dd\ud558\uc600\ub2e4. \uc774 \ub2ec\uc774 \ub05d\ub0a0 \ubb34\ub835 \ubc00\ub9ac\uc5b8 \ub2ec\ub7ec \ud648\ud398\uc774\uc9c0\ub294 $250,000\ub97c \ubc8c\uc5b4\ub4e4\uc5ec \uc54c\ub809\uc0ac \uc778\ud130\ub137\uc758 \ubb34\ubc84\uc2a4 \uc564\ub4dc \uc170\uc774\ucee4\uc2a4(Movers and Shakers) \ubaa9\ub85d\uc5d0 \ube0c\ub9ac\ud2b8\ub2c8 \uc2a4\ud53c\uc5b4\uc2a4\uc640 \ud3ec\ud1a0 \ub514\uc2a4\ud305\ud2b8 \ub274\uc2a4\uc5d0 \uc774\uc5b4 3\uc704\uc5d0 \ub4f1\uc7ac\ub418\uc5c8\ub2e4. 10\uc6d4 6\uc77c\uc5d0 \ud29c\ub294 65,000\uba85\uc758 \uc21c\uc218 \ubc29\ubb38\uc790 \uc218\ub97c \uae30\ub85d\ud558\uc600\ub2e4\uace0 \ubcf4\uace0\ud558\uc600\ub2e4. \ub514\uadf8(Digg) \uc218\uac00 1465\uac1c\ub97c \ub118\uc73c\uba74\uc11c \uadf8 \uc8fc\uc758 \uac00\uc7a5 \ub9ce\uc740 \ub354\uadf8(Dugg) \ub9c1\ud06c \uac00\uc6b4\ub370 \ud558\ub098\uac00 \ub418\uc5c8\ub2e4 11\uc77c\uc774 \uc9c0\ub098 \uc21c\uc218 \ubc29\ubb38\uac1d \uc218\uac00 100,000\uba85\uc5d0 \uc774\ub974\ub800\ub2e4. \ubc00\ub9ac\uc5b8 \ub2ec\ub7ec \ud648\ud398\uc774\uc9c0\ub97c \uc2dc\uc791\ud55c \ub4a4\ub85c \ub450 \ub2ec \ub4a4\uc778 10\uc6d4 26\uc77c\uc5d0\ub294 500,900 \uac1c\uac00 \ub118\ub294 \ud654\uc18c\uac00 1,400\uba85\uc758 \uace0\uac1d\uc5d0\uac8c \ud310\ub9e4\ub418\uc5c8\ub2e4. \uc0c8\ud574 \uc804\ub0a0\uc5d0 \ud29c\ub294 \uc774 \uc0ac\uc774\ud2b8\uac00 \ub9e4 \uc2dc\uac04 25,000\uba85\uc758 \uc21c\uc218 \ubc29\ubb38\uc790\ub97c \uae30\ub85d\ud558\uc600\uace0 \uc54c\ub809\uc0ac \uc21c\uc704\ub85c 127\uc704\ub97c \uae30\ub85d\ud558\uc600\uc73c\uba70 1,000,000\uac1c \ud654\uc18c \uac00\uc6b4\ub370 999,000\uac1c\ub97c \ud310\ub9e4\ud558\uc600\ub2e4\uace0 \ubcf4\uace0\ud558\uc600\ub2e4.", "answer": "$100", "sentence": "\ucd5c\uc18c \ud310\ub9e4 \uac00\uaca9\uc740 $100 \uc774\ub2e4.", "paragraph_sentence": "\uac01 \ud654\uc18c\ub294 \ub108\ubb34 \uc791\uc544\uc11c \uc27d\uac8c \ubcf4\uc774\uc9c0 \uc54a\uc558\ub2e4. \uc774 \uae4c\ub2ed\uc5d0 \ud654\uc18c\ub4e4\uc740 10 \u00d7 10 \ud654\uc18c \ub2e8\uc704\uc778 100\uac1c\uc758 \ud654\uc18c \ube14\ub85d \ub2e8\uc704\ub85c \ud310\ub9e4\ub418\uc5c8\ub2e4. \uadf8\ub7ec\ubbc0\ub85c \ucd5c\uc18c \ud310\ub9e4 \uac00\uaca9\uc740 $100 \uc774\ub2e4. \uc0ac\uc774\ud2b8 \uc6b4\uc601\uc744 \uc2dc\uc791\ud55c\uc9c0 \uc0ac\ud758\ub9cc\uc5d0 \ud29c\uc758 \uce5c\uad6c\uac00 \uc6b4\uc601\ud558\ub294 \uc628\ub77c\uc778 \uc74c\uc545 \uc6f9\uc0ac\uc774\ud2b8\uc5d0 \uc5f0\uacb0\ub418\uba74\uc11c \ud310\ub9e4\uac00 \ucc98\uc74c\uc73c\ub85c \uc2dc\uc791\ub418\uc5c8\ub2e4. \uadf8\ub294 400\uac1c\uc758 \ud654\uc18c(20 \u00d7 20 \ube14\ub85d)\ub97c \uc0c0\ub2e4. 2\uc8fc\uac00 \uc9c0\ub098 \ud29c\uc758 \uce5c\uad6c\ub4e4\uacfc \uac00\uc871\ub4e4\uc740 \ubaa8\ub450 4,700\uac1c\uc758 \ud654\uc18c\ub97c \uad6c\ub9e4\ud558\uc600\ub2e4. \uc774 \uc0ac\uc774\ud2b8\ub294 \ucd08\uae30\uc5d0 \uad6c\uc804 \ub9c8\ucf00\ud305\uc73c\ub85c\ub9cc\uc73c\ub85c \ubc8c\uc5ec\ub098\uac14\ub2e4. \uadf8\ub7ec\ub098 \uc774 \uc0ac\uc774\ud2b8\uac00 $1,000\ub97c \ubc8c\uc5b4\ub4e4\uc774\uc790 \uc5b8\ub860 \uae30\uc0ac\uac00 BBC\ub97c \ud1b5\ud574 \ubcf4\ub3c4\ub418\uc5c8\ub2e4. \uae30\uc220 \ub274\uc2a4 \uc6f9\uc0ac\uc774\ud2b8 \ub354 \ub808\uc9c0\uc2a4\ud130\ub294 9\uc6d4\uc5d0 \ubc00\ub9ac\uc5b8 \ub2ec\ub7ec \ud648\ud398\uc774\uc9c0\uc5d0 \ub300\ud55c \ub450 \uac1c\uc758 \uae30\uc0ac\ub97c \uc7a5\uc2dd\ud558\uc600\ub2e4. \uc774 \ub2ec\uc774 \ub05d\ub0a0 \ubb34\ub835 \ubc00\ub9ac\uc5b8 \ub2ec\ub7ec \ud648\ud398\uc774\uc9c0\ub294 $250,000\ub97c \ubc8c\uc5b4\ub4e4\uc5ec \uc54c\ub809\uc0ac \uc778\ud130\ub137\uc758 \ubb34\ubc84\uc2a4 \uc564\ub4dc \uc170\uc774\ucee4\uc2a4(Movers and Shakers) \ubaa9\ub85d\uc5d0 \ube0c\ub9ac\ud2b8\ub2c8 \uc2a4\ud53c\uc5b4\uc2a4\uc640 \ud3ec\ud1a0 \ub514\uc2a4\ud305\ud2b8 \ub274\uc2a4\uc5d0 \uc774\uc5b4 3\uc704\uc5d0 \ub4f1\uc7ac\ub418\uc5c8\ub2e4. 10\uc6d4 6\uc77c\uc5d0 \ud29c\ub294 65,000\uba85\uc758 \uc21c\uc218 \ubc29\ubb38\uc790 \uc218\ub97c \uae30\ub85d\ud558\uc600\ub2e4\uace0 \ubcf4\uace0\ud558\uc600\ub2e4. \ub514\uadf8(Digg) \uc218\uac00 1465\uac1c\ub97c \ub118\uc73c\uba74\uc11c \uadf8 \uc8fc\uc758 \uac00\uc7a5 \ub9ce\uc740 \ub354\uadf8(Dugg) \ub9c1\ud06c \uac00\uc6b4\ub370 \ud558\ub098\uac00 \ub418\uc5c8\ub2e4 11\uc77c\uc774 \uc9c0\ub098 \uc21c\uc218 \ubc29\ubb38\uac1d \uc218\uac00 100,000\uba85\uc5d0 \uc774\ub974\ub800\ub2e4. \ubc00\ub9ac\uc5b8 \ub2ec\ub7ec \ud648\ud398\uc774\uc9c0\ub97c \uc2dc\uc791\ud55c \ub4a4\ub85c \ub450 \ub2ec \ub4a4\uc778 10\uc6d4 26\uc77c\uc5d0\ub294 500,900 \uac1c\uac00 \ub118\ub294 \ud654\uc18c\uac00 1,400\uba85\uc758 \uace0\uac1d\uc5d0\uac8c \ud310\ub9e4\ub418\uc5c8\ub2e4. \uc0c8\ud574 \uc804\ub0a0\uc5d0 \ud29c\ub294 \uc774 \uc0ac\uc774\ud2b8\uac00 \ub9e4 \uc2dc\uac04 25,000\uba85\uc758 \uc21c\uc218 \ubc29\ubb38\uc790\ub97c \uae30\ub85d\ud558\uc600\uace0 \uc54c\ub809\uc0ac \uc21c\uc704\ub85c 127\uc704\ub97c \uae30\ub85d\ud558\uc600\uc73c\uba70 1,000,000\uac1c \ud654\uc18c \uac00\uc6b4\ub370 999,000\uac1c\ub97c \ud310\ub9e4\ud558\uc600\ub2e4\uace0 \ubcf4\uace0\ud558\uc600\ub2e4.", "paragraph_answer": "\uac01 \ud654\uc18c\ub294 \ub108\ubb34 \uc791\uc544\uc11c \uc27d\uac8c \ubcf4\uc774\uc9c0 \uc54a\uc558\ub2e4. \uc774 \uae4c\ub2ed\uc5d0 \ud654\uc18c\ub4e4\uc740 10 \u00d7 10 \ud654\uc18c \ub2e8\uc704\uc778 100\uac1c\uc758 \ud654\uc18c \ube14\ub85d \ub2e8\uc704\ub85c \ud310\ub9e4\ub418\uc5c8\ub2e4. \uadf8\ub7ec\ubbc0\ub85c \ucd5c\uc18c \ud310\ub9e4 \uac00\uaca9\uc740 $100 \uc774\ub2e4. \uc0ac\uc774\ud2b8 \uc6b4\uc601\uc744 \uc2dc\uc791\ud55c\uc9c0 \uc0ac\ud758\ub9cc\uc5d0 \ud29c\uc758 \uce5c\uad6c\uac00 \uc6b4\uc601\ud558\ub294 \uc628\ub77c\uc778 \uc74c\uc545 \uc6f9\uc0ac\uc774\ud2b8\uc5d0 \uc5f0\uacb0\ub418\uba74\uc11c \ud310\ub9e4\uac00 \ucc98\uc74c\uc73c\ub85c \uc2dc\uc791\ub418\uc5c8\ub2e4. \uadf8\ub294 400\uac1c\uc758 \ud654\uc18c(20 \u00d7 20 \ube14\ub85d)\ub97c \uc0c0\ub2e4. 2\uc8fc\uac00 \uc9c0\ub098 \ud29c\uc758 \uce5c\uad6c\ub4e4\uacfc \uac00\uc871\ub4e4\uc740 \ubaa8\ub450 4,700\uac1c\uc758 \ud654\uc18c\ub97c \uad6c\ub9e4\ud558\uc600\ub2e4. \uc774 \uc0ac\uc774\ud2b8\ub294 \ucd08\uae30\uc5d0 \uad6c\uc804 \ub9c8\ucf00\ud305\uc73c\ub85c\ub9cc\uc73c\ub85c \ubc8c\uc5ec\ub098\uac14\ub2e4. \uadf8\ub7ec\ub098 \uc774 \uc0ac\uc774\ud2b8\uac00 $1,000\ub97c \ubc8c\uc5b4\ub4e4\uc774\uc790 \uc5b8\ub860 \uae30\uc0ac\uac00 BBC\ub97c \ud1b5\ud574 \ubcf4\ub3c4\ub418\uc5c8\ub2e4. \uae30\uc220 \ub274\uc2a4 \uc6f9\uc0ac\uc774\ud2b8 \ub354 \ub808\uc9c0\uc2a4\ud130\ub294 9\uc6d4\uc5d0 \ubc00\ub9ac\uc5b8 \ub2ec\ub7ec \ud648\ud398\uc774\uc9c0\uc5d0 \ub300\ud55c \ub450 \uac1c\uc758 \uae30\uc0ac\ub97c \uc7a5\uc2dd\ud558\uc600\ub2e4. \uc774 \ub2ec\uc774 \ub05d\ub0a0 \ubb34\ub835 \ubc00\ub9ac\uc5b8 \ub2ec\ub7ec \ud648\ud398\uc774\uc9c0\ub294 $250,000\ub97c \ubc8c\uc5b4\ub4e4\uc5ec \uc54c\ub809\uc0ac \uc778\ud130\ub137\uc758 \ubb34\ubc84\uc2a4 \uc564\ub4dc \uc170\uc774\ucee4\uc2a4(Movers and Shakers) \ubaa9\ub85d\uc5d0 \ube0c\ub9ac\ud2b8\ub2c8 \uc2a4\ud53c\uc5b4\uc2a4\uc640 \ud3ec\ud1a0 \ub514\uc2a4\ud305\ud2b8 \ub274\uc2a4\uc5d0 \uc774\uc5b4 3\uc704\uc5d0 \ub4f1\uc7ac\ub418\uc5c8\ub2e4. 10\uc6d4 6\uc77c\uc5d0 \ud29c\ub294 65,000\uba85\uc758 \uc21c\uc218 \ubc29\ubb38\uc790 \uc218\ub97c \uae30\ub85d\ud558\uc600\ub2e4\uace0 \ubcf4\uace0\ud558\uc600\ub2e4. \ub514\uadf8(Digg) \uc218\uac00 1465\uac1c\ub97c \ub118\uc73c\uba74\uc11c \uadf8 \uc8fc\uc758 \uac00\uc7a5 \ub9ce\uc740 \ub354\uadf8(Dugg) \ub9c1\ud06c \uac00\uc6b4\ub370 \ud558\ub098\uac00 \ub418\uc5c8\ub2e4 11\uc77c\uc774 \uc9c0\ub098 \uc21c\uc218 \ubc29\ubb38\uac1d \uc218\uac00 100,000\uba85\uc5d0 \uc774\ub974\ub800\ub2e4. \ubc00\ub9ac\uc5b8 \ub2ec\ub7ec \ud648\ud398\uc774\uc9c0\ub97c \uc2dc\uc791\ud55c \ub4a4\ub85c \ub450 \ub2ec \ub4a4\uc778 10\uc6d4 26\uc77c\uc5d0\ub294 500,900 \uac1c\uac00 \ub118\ub294 \ud654\uc18c\uac00 1,400\uba85\uc758 \uace0\uac1d\uc5d0\uac8c \ud310\ub9e4\ub418\uc5c8\ub2e4. \uc0c8\ud574 \uc804\ub0a0\uc5d0 \ud29c\ub294 \uc774 \uc0ac\uc774\ud2b8\uac00 \ub9e4 \uc2dc\uac04 25,000\uba85\uc758 \uc21c\uc218 \ubc29\ubb38\uc790\ub97c \uae30\ub85d\ud558\uc600\uace0 \uc54c\ub809\uc0ac \uc21c\uc704\ub85c 127\uc704\ub97c \uae30\ub85d\ud558\uc600\uc73c\uba70 1,000,000\uac1c \ud654\uc18c \uac00\uc6b4\ub370 999,000\uac1c\ub97c \ud310\ub9e4\ud558\uc600\ub2e4\uace0 \ubcf4\uace0\ud558\uc600\ub2e4.", "sentence_answer": "\ucd5c\uc18c \ud310\ub9e4 \uac00\uaca9\uc740 $100 \uc774\ub2e4.", "paragraph_id": "6534686-2-0", "paragraph_question": "question: \ubc00\ub9ac\uc5b8 \ub2ec\ub7ec \ud648\ud398\uc774\uc9c0\uc758 100\uac1c \ud654\uc18c\ube14\ub85d\ub2e8\uc704 \ucd5c\uc18c\ud310\ub9e4 \uac00\uaca9\uc740?, context: \uac01 \ud654\uc18c\ub294 \ub108\ubb34 \uc791\uc544\uc11c \uc27d\uac8c \ubcf4\uc774\uc9c0 \uc54a\uc558\ub2e4. \uc774 \uae4c\ub2ed\uc5d0 \ud654\uc18c\ub4e4\uc740 10 \u00d7 10 \ud654\uc18c \ub2e8\uc704\uc778 100\uac1c\uc758 \ud654\uc18c \ube14\ub85d \ub2e8\uc704\ub85c \ud310\ub9e4\ub418\uc5c8\ub2e4. \uadf8\ub7ec\ubbc0\ub85c \ucd5c\uc18c \ud310\ub9e4 \uac00\uaca9\uc740 $100\uc774\ub2e4. \uc0ac\uc774\ud2b8 \uc6b4\uc601\uc744 \uc2dc\uc791\ud55c\uc9c0 \uc0ac\ud758\ub9cc\uc5d0 \ud29c\uc758 \uce5c\uad6c\uac00 \uc6b4\uc601\ud558\ub294 \uc628\ub77c\uc778 \uc74c\uc545 \uc6f9\uc0ac\uc774\ud2b8\uc5d0 \uc5f0\uacb0\ub418\uba74\uc11c \ud310\ub9e4\uac00 \ucc98\uc74c\uc73c\ub85c \uc2dc\uc791\ub418\uc5c8\ub2e4. \uadf8\ub294 400\uac1c\uc758 \ud654\uc18c(20 \u00d7 20 \ube14\ub85d)\ub97c \uc0c0\ub2e4. 2\uc8fc\uac00 \uc9c0\ub098 \ud29c\uc758 \uce5c\uad6c\ub4e4\uacfc \uac00\uc871\ub4e4\uc740 \ubaa8\ub450 4,700\uac1c\uc758 \ud654\uc18c\ub97c \uad6c\ub9e4\ud558\uc600\ub2e4. \uc774 \uc0ac\uc774\ud2b8\ub294 \ucd08\uae30\uc5d0 \uad6c\uc804 \ub9c8\ucf00\ud305\uc73c\ub85c\ub9cc\uc73c\ub85c \ubc8c\uc5ec\ub098\uac14\ub2e4. \uadf8\ub7ec\ub098 \uc774 \uc0ac\uc774\ud2b8\uac00 $1,000\ub97c \ubc8c\uc5b4\ub4e4\uc774\uc790 \uc5b8\ub860 \uae30\uc0ac\uac00 BBC\ub97c \ud1b5\ud574 \ubcf4\ub3c4\ub418\uc5c8\ub2e4. \uae30\uc220 \ub274\uc2a4 \uc6f9\uc0ac\uc774\ud2b8 \ub354 \ub808\uc9c0\uc2a4\ud130\ub294 9\uc6d4\uc5d0 \ubc00\ub9ac\uc5b8 \ub2ec\ub7ec \ud648\ud398\uc774\uc9c0\uc5d0 \ub300\ud55c \ub450 \uac1c\uc758 \uae30\uc0ac\ub97c \uc7a5\uc2dd\ud558\uc600\ub2e4. \uc774 \ub2ec\uc774 \ub05d\ub0a0 \ubb34\ub835 \ubc00\ub9ac\uc5b8 \ub2ec\ub7ec \ud648\ud398\uc774\uc9c0\ub294 $250,000\ub97c \ubc8c\uc5b4\ub4e4\uc5ec \uc54c\ub809\uc0ac \uc778\ud130\ub137\uc758 \ubb34\ubc84\uc2a4 \uc564\ub4dc \uc170\uc774\ucee4\uc2a4(Movers and Shakers) \ubaa9\ub85d\uc5d0 \ube0c\ub9ac\ud2b8\ub2c8 \uc2a4\ud53c\uc5b4\uc2a4\uc640 \ud3ec\ud1a0 \ub514\uc2a4\ud305\ud2b8 \ub274\uc2a4\uc5d0 \uc774\uc5b4 3\uc704\uc5d0 \ub4f1\uc7ac\ub418\uc5c8\ub2e4. 10\uc6d4 6\uc77c\uc5d0 \ud29c\ub294 65,000\uba85\uc758 \uc21c\uc218 \ubc29\ubb38\uc790 \uc218\ub97c \uae30\ub85d\ud558\uc600\ub2e4\uace0 \ubcf4\uace0\ud558\uc600\ub2e4. \ub514\uadf8(Digg) \uc218\uac00 1465\uac1c\ub97c \ub118\uc73c\uba74\uc11c \uadf8 \uc8fc\uc758 \uac00\uc7a5 \ub9ce\uc740 \ub354\uadf8(Dugg) \ub9c1\ud06c \uac00\uc6b4\ub370 \ud558\ub098\uac00 \ub418\uc5c8\ub2e4 11\uc77c\uc774 \uc9c0\ub098 \uc21c\uc218 \ubc29\ubb38\uac1d \uc218\uac00 100,000\uba85\uc5d0 \uc774\ub974\ub800\ub2e4. \ubc00\ub9ac\uc5b8 \ub2ec\ub7ec \ud648\ud398\uc774\uc9c0\ub97c \uc2dc\uc791\ud55c \ub4a4\ub85c \ub450 \ub2ec \ub4a4\uc778 10\uc6d4 26\uc77c\uc5d0\ub294 500,900 \uac1c\uac00 \ub118\ub294 \ud654\uc18c\uac00 1,400\uba85\uc758 \uace0\uac1d\uc5d0\uac8c \ud310\ub9e4\ub418\uc5c8\ub2e4. \uc0c8\ud574 \uc804\ub0a0\uc5d0 \ud29c\ub294 \uc774 \uc0ac\uc774\ud2b8\uac00 \ub9e4 \uc2dc\uac04 25,000\uba85\uc758 \uc21c\uc218 \ubc29\ubb38\uc790\ub97c \uae30\ub85d\ud558\uc600\uace0 \uc54c\ub809\uc0ac \uc21c\uc704\ub85c 127\uc704\ub97c \uae30\ub85d\ud558\uc600\uc73c\uba70 1,000,000\uac1c \ud654\uc18c \uac00\uc6b4\ub370 999,000\uac1c\ub97c \ud310\ub9e4\ud558\uc600\ub2e4\uace0 \ubcf4\uace0\ud558\uc600\ub2e4."} {"question": "\uacfc\ud14c\ub9d0\ub77c\uc758 \uad6d\ud1a0\ub97c \uba87 \uac1c\uc758 \uc9c0\ubc29\uc73c\ub85c \ub098\ub20c \uc218 \uc788\ub294\uac00?", "paragraph": "\ub3d9\uc11c\uac00 \uc0b0\ub9e5\uc73c\ub85c \uc774\ub8e8\uc5b4\uc9c0\uba74\uc11c \uad6d\ud1a0\ub97c \uc138 \uac1c\uc758 \uc9c0\ubc29\uc73c\ub85c \ub098\ub208\ub2e4. \ud0dc\ud3c9\uc591\ucabd\uc758 \uc881\uc740 \ud574\uc548\ud3c9\uc57c\uc640 \ubd81\ubd80\uc758 \ub113\uc740 \uc800\uc2b5\uc9c0\ub294 \ub365\uace0 \uac74\uac15\uc0c1 \uc88b\uc9c0 \uc54a\uc740 \uc9c0\ubc29\uc774\ub2e4. \uc774\ub97c \ube7c\uace0\uc11c\ub294 \uc0b0\uc545\uc9c0\ub300\uac00 \ub9ce\uc740\ub370, \uc911\uc559\uc758 \uc0b0\uc9c0\ub098 \uace0\uc6d0\uc740 \uc0c1\ub2f9\ud788 \uc2dc\uc6d0\ud558\uace0 \ucf8c\uc801\ud558\uba70, \uac74\uae30\uc640 \uc6b0\uae30 \uc911\uc5d0\uc11c \ud2b9\ud788 \uac74\uae30\uc5d0 \uc9c0\ub0b4\uae30 \uc88b\ub2e4. \ub530\ub77c\uc11c \uc778\uad6c\uc758 \ud0dc\ubc18\uc774 \uc911\uc559\uace0\uc9c0\uc5d0 \uc9d1\uc911\ud574 \uc788\uc73c\uba70, \uc218\ub3c4\uc778 \uacfc\ud14c\ub9d0\ub77c \uc2dc\ub3c4 \ud574\ubc1c \uc57d 1,500m\uc758 \uace0\uc9c0\uc5d0 \uc788\ub2e4. \uc911\uc559\uc0b0\uc9c0\ub294 \uba55\uc2dc\ucf54\uc5d0\uc11c \uacc4\uc18d\ub418\ub294 \ud5d8\uc900\ud55c \uc2b5\uace1 \uc0b0\ub9e5\uc774\uba70, \ud0dc\ud3c9\uc591 \uc5f0\uc548\uc744 \ub530\ub77c \ud654\uc0b0\uc774 \uc990\ube44\ud558\ub2e4. \ud654\uc0b0\uc131 \ud1a0\uc591\uc740 \ube44\uc625\ud558\ubbc0\ub85c \uc774 \uc9c0\uc5ed\uc5d0\ub294 \ub18d\uc5c5\uc774 \ubc1c\ub2ec\ud574 \uc788\ub2e4. 2018\ub144 \ud478\uc5d0\uace0\uc0b0 \ubd84\ud654\uac00 \uc77c\uc5b4\ub0a0 \ub9cc\ud07c \ud65c\ud654\uc0b0\uc774\ub098 \uc9c0\uc9c4\uc758 \ud65c\ub3d9\ub3c4 \uaca9\uc2ec\ud558\ub2e4. \uc57d 50\ub144 \uc804\uc758 \ub300\uc9c0\uc9c4 \ub54c\uc5d0\ub294 \uc218\ub3c4\uac00 \uac70\uc758 \ud30c\uad34\ub418\uc5c8\ub2e4.", "answer": "\uc138 \uac1c", "sentence": "\uad6d\ud1a0\ub97c \uc138 \uac1c \uc758 \uc9c0\ubc29\uc73c\ub85c \ub098\ub208\ub2e4.", "paragraph_sentence": "\ub3d9\uc11c\uac00 \uc0b0\ub9e5\uc73c\ub85c \uc774\ub8e8\uc5b4\uc9c0\uba74\uc11c \uad6d\ud1a0\ub97c \uc138 \uac1c \uc758 \uc9c0\ubc29\uc73c\ub85c \ub098\ub208\ub2e4. \ud0dc\ud3c9\uc591\ucabd\uc758 \uc881\uc740 \ud574\uc548\ud3c9\uc57c\uc640 \ubd81\ubd80\uc758 \ub113\uc740 \uc800\uc2b5\uc9c0\ub294 \ub365\uace0 \uac74\uac15\uc0c1 \uc88b\uc9c0 \uc54a\uc740 \uc9c0\ubc29\uc774\ub2e4. \uc774\ub97c \ube7c\uace0\uc11c\ub294 \uc0b0\uc545\uc9c0\ub300\uac00 \ub9ce\uc740\ub370, \uc911\uc559\uc758 \uc0b0\uc9c0\ub098 \uace0\uc6d0\uc740 \uc0c1\ub2f9\ud788 \uc2dc\uc6d0\ud558\uace0 \ucf8c\uc801\ud558\uba70, \uac74\uae30\uc640 \uc6b0\uae30 \uc911\uc5d0\uc11c \ud2b9\ud788 \uac74\uae30\uc5d0 \uc9c0\ub0b4\uae30 \uc88b\ub2e4. \ub530\ub77c\uc11c \uc778\uad6c\uc758 \ud0dc\ubc18\uc774 \uc911\uc559\uace0\uc9c0\uc5d0 \uc9d1\uc911\ud574 \uc788\uc73c\uba70, \uc218\ub3c4\uc778 \uacfc\ud14c\ub9d0\ub77c \uc2dc\ub3c4 \ud574\ubc1c \uc57d 1,500m\uc758 \uace0\uc9c0\uc5d0 \uc788\ub2e4. \uc911\uc559\uc0b0\uc9c0\ub294 \uba55\uc2dc\ucf54\uc5d0\uc11c \uacc4\uc18d\ub418\ub294 \ud5d8\uc900\ud55c \uc2b5\uace1 \uc0b0\ub9e5\uc774\uba70, \ud0dc\ud3c9\uc591 \uc5f0\uc548\uc744 \ub530\ub77c \ud654\uc0b0\uc774 \uc990\ube44\ud558\ub2e4. \ud654\uc0b0\uc131 \ud1a0\uc591\uc740 \ube44\uc625\ud558\ubbc0\ub85c \uc774 \uc9c0\uc5ed\uc5d0\ub294 \ub18d\uc5c5\uc774 \ubc1c\ub2ec\ud574 \uc788\ub2e4. 2018\ub144 \ud478\uc5d0\uace0\uc0b0 \ubd84\ud654\uac00 \uc77c\uc5b4\ub0a0 \ub9cc\ud07c \ud65c\ud654\uc0b0\uc774\ub098 \uc9c0\uc9c4\uc758 \ud65c\ub3d9\ub3c4 \uaca9\uc2ec\ud558\ub2e4. \uc57d 50\ub144 \uc804\uc758 \ub300\uc9c0\uc9c4 \ub54c\uc5d0\ub294 \uc218\ub3c4\uac00 \uac70\uc758 \ud30c\uad34\ub418\uc5c8\ub2e4.", "paragraph_answer": "\ub3d9\uc11c\uac00 \uc0b0\ub9e5\uc73c\ub85c \uc774\ub8e8\uc5b4\uc9c0\uba74\uc11c \uad6d\ud1a0\ub97c \uc138 \uac1c \uc758 \uc9c0\ubc29\uc73c\ub85c \ub098\ub208\ub2e4. \ud0dc\ud3c9\uc591\ucabd\uc758 \uc881\uc740 \ud574\uc548\ud3c9\uc57c\uc640 \ubd81\ubd80\uc758 \ub113\uc740 \uc800\uc2b5\uc9c0\ub294 \ub365\uace0 \uac74\uac15\uc0c1 \uc88b\uc9c0 \uc54a\uc740 \uc9c0\ubc29\uc774\ub2e4. \uc774\ub97c \ube7c\uace0\uc11c\ub294 \uc0b0\uc545\uc9c0\ub300\uac00 \ub9ce\uc740\ub370, \uc911\uc559\uc758 \uc0b0\uc9c0\ub098 \uace0\uc6d0\uc740 \uc0c1\ub2f9\ud788 \uc2dc\uc6d0\ud558\uace0 \ucf8c\uc801\ud558\uba70, \uac74\uae30\uc640 \uc6b0\uae30 \uc911\uc5d0\uc11c \ud2b9\ud788 \uac74\uae30\uc5d0 \uc9c0\ub0b4\uae30 \uc88b\ub2e4. \ub530\ub77c\uc11c \uc778\uad6c\uc758 \ud0dc\ubc18\uc774 \uc911\uc559\uace0\uc9c0\uc5d0 \uc9d1\uc911\ud574 \uc788\uc73c\uba70, \uc218\ub3c4\uc778 \uacfc\ud14c\ub9d0\ub77c \uc2dc\ub3c4 \ud574\ubc1c \uc57d 1,500m\uc758 \uace0\uc9c0\uc5d0 \uc788\ub2e4. \uc911\uc559\uc0b0\uc9c0\ub294 \uba55\uc2dc\ucf54\uc5d0\uc11c \uacc4\uc18d\ub418\ub294 \ud5d8\uc900\ud55c \uc2b5\uace1 \uc0b0\ub9e5\uc774\uba70, \ud0dc\ud3c9\uc591 \uc5f0\uc548\uc744 \ub530\ub77c \ud654\uc0b0\uc774 \uc990\ube44\ud558\ub2e4. \ud654\uc0b0\uc131 \ud1a0\uc591\uc740 \ube44\uc625\ud558\ubbc0\ub85c \uc774 \uc9c0\uc5ed\uc5d0\ub294 \ub18d\uc5c5\uc774 \ubc1c\ub2ec\ud574 \uc788\ub2e4. 2018\ub144 \ud478\uc5d0\uace0\uc0b0 \ubd84\ud654\uac00 \uc77c\uc5b4\ub0a0 \ub9cc\ud07c \ud65c\ud654\uc0b0\uc774\ub098 \uc9c0\uc9c4\uc758 \ud65c\ub3d9\ub3c4 \uaca9\uc2ec\ud558\ub2e4. \uc57d 50\ub144 \uc804\uc758 \ub300\uc9c0\uc9c4 \ub54c\uc5d0\ub294 \uc218\ub3c4\uac00 \uac70\uc758 \ud30c\uad34\ub418\uc5c8\ub2e4.", "sentence_answer": "\uad6d\ud1a0\ub97c \uc138 \uac1c \uc758 \uc9c0\ubc29\uc73c\ub85c \ub098\ub208\ub2e4.", "paragraph_id": "6471893-2-0", "paragraph_question": "question: \uacfc\ud14c\ub9d0\ub77c\uc758 \uad6d\ud1a0\ub97c \uba87 \uac1c\uc758 \uc9c0\ubc29\uc73c\ub85c \ub098\ub20c \uc218 \uc788\ub294\uac00?, context: \ub3d9\uc11c\uac00 \uc0b0\ub9e5\uc73c\ub85c \uc774\ub8e8\uc5b4\uc9c0\uba74\uc11c \uad6d\ud1a0\ub97c \uc138 \uac1c\uc758 \uc9c0\ubc29\uc73c\ub85c \ub098\ub208\ub2e4. \ud0dc\ud3c9\uc591\ucabd\uc758 \uc881\uc740 \ud574\uc548\ud3c9\uc57c\uc640 \ubd81\ubd80\uc758 \ub113\uc740 \uc800\uc2b5\uc9c0\ub294 \ub365\uace0 \uac74\uac15\uc0c1 \uc88b\uc9c0 \uc54a\uc740 \uc9c0\ubc29\uc774\ub2e4. \uc774\ub97c \ube7c\uace0\uc11c\ub294 \uc0b0\uc545\uc9c0\ub300\uac00 \ub9ce\uc740\ub370, \uc911\uc559\uc758 \uc0b0\uc9c0\ub098 \uace0\uc6d0\uc740 \uc0c1\ub2f9\ud788 \uc2dc\uc6d0\ud558\uace0 \ucf8c\uc801\ud558\uba70, \uac74\uae30\uc640 \uc6b0\uae30 \uc911\uc5d0\uc11c \ud2b9\ud788 \uac74\uae30\uc5d0 \uc9c0\ub0b4\uae30 \uc88b\ub2e4. \ub530\ub77c\uc11c \uc778\uad6c\uc758 \ud0dc\ubc18\uc774 \uc911\uc559\uace0\uc9c0\uc5d0 \uc9d1\uc911\ud574 \uc788\uc73c\uba70, \uc218\ub3c4\uc778 \uacfc\ud14c\ub9d0\ub77c \uc2dc\ub3c4 \ud574\ubc1c \uc57d 1,500m\uc758 \uace0\uc9c0\uc5d0 \uc788\ub2e4. \uc911\uc559\uc0b0\uc9c0\ub294 \uba55\uc2dc\ucf54\uc5d0\uc11c \uacc4\uc18d\ub418\ub294 \ud5d8\uc900\ud55c \uc2b5\uace1 \uc0b0\ub9e5\uc774\uba70, \ud0dc\ud3c9\uc591 \uc5f0\uc548\uc744 \ub530\ub77c \ud654\uc0b0\uc774 \uc990\ube44\ud558\ub2e4. \ud654\uc0b0\uc131 \ud1a0\uc591\uc740 \ube44\uc625\ud558\ubbc0\ub85c \uc774 \uc9c0\uc5ed\uc5d0\ub294 \ub18d\uc5c5\uc774 \ubc1c\ub2ec\ud574 \uc788\ub2e4. 2018\ub144 \ud478\uc5d0\uace0\uc0b0 \ubd84\ud654\uac00 \uc77c\uc5b4\ub0a0 \ub9cc\ud07c \ud65c\ud654\uc0b0\uc774\ub098 \uc9c0\uc9c4\uc758 \ud65c\ub3d9\ub3c4 \uaca9\uc2ec\ud558\ub2e4. \uc57d 50\ub144 \uc804\uc758 \ub300\uc9c0\uc9c4 \ub54c\uc5d0\ub294 \uc218\ub3c4\uac00 \uac70\uc758 \ud30c\uad34\ub418\uc5c8\ub2e4."} {"question": "\ub098\ud314\ubc8c\ub808\uc640 \uc885\ubc8c\ub808\uac00 \ubab8\uc744 \uc218\ucd95\ud560 \uc218 \uc788\ub3c4\ub85d \ud558\ub294 \uc6d0\ud615\uc9c8 \ub0b4\ubd80\uc758 \uae30\uad6c\ub294?", "paragraph": "\ud55c\ud3b8, \uadfc\uc721 \uc870\uc9c1\uc774 \uc5c6\ub294 \ud558\ub4f1 \ub3d9\ubb3c\uc5d0\ub294 \uadfc\uc721\uacfc \ube44\uc2b7\ud55c \uc218\ucd95\uc744 \ud558\ub294 \uc7a5\uce58\uac00 \uac16\ucd94\uc5b4\uc838 \uc788\ub2e4. \uc774\uac83\uc740 \uc6d0\uc2dc\uc801\uc778 \uc6d0\uc0dd\ub3d9\ubb3c\uc5d0\uc11c \ub9ce\uc774 \ubcfc \uc218 \uc788\ub2e4. \uc608\ub97c \ub4e4\uba74, \uc12c\ubaa8\ucda9\ub958\uc5d0 \uc18d\ud558\ub294 \ub098\ud314\ubc8c\ub808\ub294 \uc678\ubd80\ub85c\ubd80\ud130 \uc790\uadf9\uc774 \uac00\ud574\uc9c0\uba74, \ubab8\uc744 \uc218\ucd95\uc2dc\ucf1c \uadf8 \ubab8\uae38\uc774\ub97c \ubc14\uafbc\ub2e4. \uc885\ubc8c\ub808\ub3c4 \uc790\uadf9\uc774 \uac00\ud574\uc9c0\uba74, \uadf8 \uc790\ub8e8 \ubd80\ubd84\uc774 \ub098\uc120 \ubaa8\uc591\uc73c\ub85c \uc218\ucd95\ub41c\ub2e4. \uc5ec\uae30\uc5d0\ub294 \ub458\ub2e4 \uc6d0\ud615\uc9c8 \uc548\uc5d0 \ubbf8\uc624\ub134\uc774\ub77c\uace0 \ud558\ub294, \uadfc\uc721\uacfc \ud761\uc0ac\ud55c \uc12c\uc720 \ubaa8\uc591\uc73c\ub85c \ubd84\ud654\ub41c \uae30\uad6c\uac00 \ub4e4\uc5b4 \uc788\ub2e4. \uc885\ubc8c\ub808\uc758 \uc790\ub8e8 \ubd80\ubd84\uc740 \ub450 \uc885\ub958\uc758 \uce35\uc73c\ub85c \ub098\ub258\uc5b4 \uc788\ub294\ub370, \uadf8 \uc548\ucabd\uc740 \ud0a4\ub178\ud50c\ub77c\uc988\ub9c8\ub85c \uc774\ub8e8\uc5b4\uc838 \uc788\ub2e4. \uc774 \ud0a4\ub178\ud50c\ub77c\uc988\ub9c8\ub294 \uc218\ucd95\uc131\uc774 \uc788\uc73c\ubbc0\ub85c, \uc774 \ubd80\ubd84\uc774 \ud30c\uad34\ub418\uba74 \uc790\ub8e8 \ubd80\ubd84\uc758 \uc218\ucd95 \uc6b4\ub3d9\uc740 \uc77c\uc5b4\ub098\uc9c0 \uc54a\ub294\ub2e4. \ub610 \uac15\uc7a5\ub3d9\ubb3c\uc778 \ud788\ub4dc\ub77c\ub098 \ub9d0\ubbf8\uc798 \ub4f1\uc740 \uadfc\uc721 \uc0c1\ud53c\uc138\ud3ec\uac00 \uccb4\ubcbd \uc548\uc5d0 \uc0b0\uc7ac\ud558\uace0 \uc788\uc5b4, \uadf8 \uc12c\uc720 \ubaa8\uc591\uc73c\ub85c \ubd84\ud654\ud558\uace0 \uc788\ub294 \uc6d0\ud615\uc9c8\uc758 \uc77c\ubd80\uac00 \uc218\ucd95\ud558\uc5ec \uccb4\ubcbd\uc758 \uc6b4\ub3d9\uc774 \uc77c\uc5b4\ub09c\ub2e4.", "answer": "\ubbf8\uc624\ub134", "sentence": "\uc5ec\uae30\uc5d0\ub294 \ub458\ub2e4 \uc6d0\ud615\uc9c8 \uc548\uc5d0 \ubbf8\uc624\ub134 \uc774\ub77c\uace0 \ud558\ub294, \uadfc\uc721\uacfc \ud761\uc0ac\ud55c \uc12c\uc720 \ubaa8\uc591\uc73c\ub85c \ubd84\ud654\ub41c \uae30\uad6c\uac00 \ub4e4\uc5b4 \uc788\ub2e4.", "paragraph_sentence": "\ud55c\ud3b8, \uadfc\uc721 \uc870\uc9c1\uc774 \uc5c6\ub294 \ud558\ub4f1 \ub3d9\ubb3c\uc5d0\ub294 \uadfc\uc721\uacfc \ube44\uc2b7\ud55c \uc218\ucd95\uc744 \ud558\ub294 \uc7a5\uce58\uac00 \uac16\ucd94\uc5b4\uc838 \uc788\ub2e4. \uc774\uac83\uc740 \uc6d0\uc2dc\uc801\uc778 \uc6d0\uc0dd\ub3d9\ubb3c\uc5d0\uc11c \ub9ce\uc774 \ubcfc \uc218 \uc788\ub2e4. \uc608\ub97c \ub4e4\uba74, \uc12c\ubaa8\ucda9\ub958\uc5d0 \uc18d\ud558\ub294 \ub098\ud314\ubc8c\ub808\ub294 \uc678\ubd80\ub85c\ubd80\ud130 \uc790\uadf9\uc774 \uac00\ud574\uc9c0\uba74, \ubab8\uc744 \uc218\ucd95\uc2dc\ucf1c \uadf8 \ubab8\uae38\uc774\ub97c \ubc14\uafbc\ub2e4. \uc885\ubc8c\ub808\ub3c4 \uc790\uadf9\uc774 \uac00\ud574\uc9c0\uba74, \uadf8 \uc790\ub8e8 \ubd80\ubd84\uc774 \ub098\uc120 \ubaa8\uc591\uc73c\ub85c \uc218\ucd95\ub41c\ub2e4. \uc5ec\uae30\uc5d0\ub294 \ub458\ub2e4 \uc6d0\ud615\uc9c8 \uc548\uc5d0 \ubbf8\uc624\ub134 \uc774\ub77c\uace0 \ud558\ub294, \uadfc\uc721\uacfc \ud761\uc0ac\ud55c \uc12c\uc720 \ubaa8\uc591\uc73c\ub85c \ubd84\ud654\ub41c \uae30\uad6c\uac00 \ub4e4\uc5b4 \uc788\ub2e4. \uc885\ubc8c\ub808\uc758 \uc790\ub8e8 \ubd80\ubd84\uc740 \ub450 \uc885\ub958\uc758 \uce35\uc73c\ub85c \ub098\ub258\uc5b4 \uc788\ub294\ub370, \uadf8 \uc548\ucabd\uc740 \ud0a4\ub178\ud50c\ub77c\uc988\ub9c8\ub85c \uc774\ub8e8\uc5b4\uc838 \uc788\ub2e4. \uc774 \ud0a4\ub178\ud50c\ub77c\uc988\ub9c8\ub294 \uc218\ucd95\uc131\uc774 \uc788\uc73c\ubbc0\ub85c, \uc774 \ubd80\ubd84\uc774 \ud30c\uad34\ub418\uba74 \uc790\ub8e8 \ubd80\ubd84\uc758 \uc218\ucd95 \uc6b4\ub3d9\uc740 \uc77c\uc5b4\ub098\uc9c0 \uc54a\ub294\ub2e4. \ub610 \uac15\uc7a5\ub3d9\ubb3c\uc778 \ud788\ub4dc\ub77c\ub098 \ub9d0\ubbf8\uc798 \ub4f1\uc740 \uadfc\uc721 \uc0c1\ud53c\uc138\ud3ec\uac00 \uccb4\ubcbd \uc548\uc5d0 \uc0b0\uc7ac\ud558\uace0 \uc788\uc5b4, \uadf8 \uc12c\uc720 \ubaa8\uc591\uc73c\ub85c \ubd84\ud654\ud558\uace0 \uc788\ub294 \uc6d0\ud615\uc9c8\uc758 \uc77c\ubd80\uac00 \uc218\ucd95\ud558\uc5ec \uccb4\ubcbd\uc758 \uc6b4\ub3d9\uc774 \uc77c\uc5b4\ub09c\ub2e4.", "paragraph_answer": "\ud55c\ud3b8, \uadfc\uc721 \uc870\uc9c1\uc774 \uc5c6\ub294 \ud558\ub4f1 \ub3d9\ubb3c\uc5d0\ub294 \uadfc\uc721\uacfc \ube44\uc2b7\ud55c \uc218\ucd95\uc744 \ud558\ub294 \uc7a5\uce58\uac00 \uac16\ucd94\uc5b4\uc838 \uc788\ub2e4. \uc774\uac83\uc740 \uc6d0\uc2dc\uc801\uc778 \uc6d0\uc0dd\ub3d9\ubb3c\uc5d0\uc11c \ub9ce\uc774 \ubcfc \uc218 \uc788\ub2e4. \uc608\ub97c \ub4e4\uba74, \uc12c\ubaa8\ucda9\ub958\uc5d0 \uc18d\ud558\ub294 \ub098\ud314\ubc8c\ub808\ub294 \uc678\ubd80\ub85c\ubd80\ud130 \uc790\uadf9\uc774 \uac00\ud574\uc9c0\uba74, \ubab8\uc744 \uc218\ucd95\uc2dc\ucf1c \uadf8 \ubab8\uae38\uc774\ub97c \ubc14\uafbc\ub2e4. \uc885\ubc8c\ub808\ub3c4 \uc790\uadf9\uc774 \uac00\ud574\uc9c0\uba74, \uadf8 \uc790\ub8e8 \ubd80\ubd84\uc774 \ub098\uc120 \ubaa8\uc591\uc73c\ub85c \uc218\ucd95\ub41c\ub2e4. \uc5ec\uae30\uc5d0\ub294 \ub458\ub2e4 \uc6d0\ud615\uc9c8 \uc548\uc5d0 \ubbf8\uc624\ub134 \uc774\ub77c\uace0 \ud558\ub294, \uadfc\uc721\uacfc \ud761\uc0ac\ud55c \uc12c\uc720 \ubaa8\uc591\uc73c\ub85c \ubd84\ud654\ub41c \uae30\uad6c\uac00 \ub4e4\uc5b4 \uc788\ub2e4. \uc885\ubc8c\ub808\uc758 \uc790\ub8e8 \ubd80\ubd84\uc740 \ub450 \uc885\ub958\uc758 \uce35\uc73c\ub85c \ub098\ub258\uc5b4 \uc788\ub294\ub370, \uadf8 \uc548\ucabd\uc740 \ud0a4\ub178\ud50c\ub77c\uc988\ub9c8\ub85c \uc774\ub8e8\uc5b4\uc838 \uc788\ub2e4. \uc774 \ud0a4\ub178\ud50c\ub77c\uc988\ub9c8\ub294 \uc218\ucd95\uc131\uc774 \uc788\uc73c\ubbc0\ub85c, \uc774 \ubd80\ubd84\uc774 \ud30c\uad34\ub418\uba74 \uc790\ub8e8 \ubd80\ubd84\uc758 \uc218\ucd95 \uc6b4\ub3d9\uc740 \uc77c\uc5b4\ub098\uc9c0 \uc54a\ub294\ub2e4. \ub610 \uac15\uc7a5\ub3d9\ubb3c\uc778 \ud788\ub4dc\ub77c\ub098 \ub9d0\ubbf8\uc798 \ub4f1\uc740 \uadfc\uc721 \uc0c1\ud53c\uc138\ud3ec\uac00 \uccb4\ubcbd \uc548\uc5d0 \uc0b0\uc7ac\ud558\uace0 \uc788\uc5b4, \uadf8 \uc12c\uc720 \ubaa8\uc591\uc73c\ub85c \ubd84\ud654\ud558\uace0 \uc788\ub294 \uc6d0\ud615\uc9c8\uc758 \uc77c\ubd80\uac00 \uc218\ucd95\ud558\uc5ec \uccb4\ubcbd\uc758 \uc6b4\ub3d9\uc774 \uc77c\uc5b4\ub09c\ub2e4.", "sentence_answer": "\uc5ec\uae30\uc5d0\ub294 \ub458\ub2e4 \uc6d0\ud615\uc9c8 \uc548\uc5d0 \ubbf8\uc624\ub134 \uc774\ub77c\uace0 \ud558\ub294, \uadfc\uc721\uacfc \ud761\uc0ac\ud55c \uc12c\uc720 \ubaa8\uc591\uc73c\ub85c \ubd84\ud654\ub41c \uae30\uad6c\uac00 \ub4e4\uc5b4 \uc788\ub2e4.", "paragraph_id": "6487962-6-0", "paragraph_question": "question: \ub098\ud314\ubc8c\ub808\uc640 \uc885\ubc8c\ub808\uac00 \ubab8\uc744 \uc218\ucd95\ud560 \uc218 \uc788\ub3c4\ub85d \ud558\ub294 \uc6d0\ud615\uc9c8 \ub0b4\ubd80\uc758 \uae30\uad6c\ub294?, context: \ud55c\ud3b8, \uadfc\uc721 \uc870\uc9c1\uc774 \uc5c6\ub294 \ud558\ub4f1 \ub3d9\ubb3c\uc5d0\ub294 \uadfc\uc721\uacfc \ube44\uc2b7\ud55c \uc218\ucd95\uc744 \ud558\ub294 \uc7a5\uce58\uac00 \uac16\ucd94\uc5b4\uc838 \uc788\ub2e4. \uc774\uac83\uc740 \uc6d0\uc2dc\uc801\uc778 \uc6d0\uc0dd\ub3d9\ubb3c\uc5d0\uc11c \ub9ce\uc774 \ubcfc \uc218 \uc788\ub2e4. \uc608\ub97c \ub4e4\uba74, \uc12c\ubaa8\ucda9\ub958\uc5d0 \uc18d\ud558\ub294 \ub098\ud314\ubc8c\ub808\ub294 \uc678\ubd80\ub85c\ubd80\ud130 \uc790\uadf9\uc774 \uac00\ud574\uc9c0\uba74, \ubab8\uc744 \uc218\ucd95\uc2dc\ucf1c \uadf8 \ubab8\uae38\uc774\ub97c \ubc14\uafbc\ub2e4. \uc885\ubc8c\ub808\ub3c4 \uc790\uadf9\uc774 \uac00\ud574\uc9c0\uba74, \uadf8 \uc790\ub8e8 \ubd80\ubd84\uc774 \ub098\uc120 \ubaa8\uc591\uc73c\ub85c \uc218\ucd95\ub41c\ub2e4. \uc5ec\uae30\uc5d0\ub294 \ub458\ub2e4 \uc6d0\ud615\uc9c8 \uc548\uc5d0 \ubbf8\uc624\ub134\uc774\ub77c\uace0 \ud558\ub294, \uadfc\uc721\uacfc \ud761\uc0ac\ud55c \uc12c\uc720 \ubaa8\uc591\uc73c\ub85c \ubd84\ud654\ub41c \uae30\uad6c\uac00 \ub4e4\uc5b4 \uc788\ub2e4. \uc885\ubc8c\ub808\uc758 \uc790\ub8e8 \ubd80\ubd84\uc740 \ub450 \uc885\ub958\uc758 \uce35\uc73c\ub85c \ub098\ub258\uc5b4 \uc788\ub294\ub370, \uadf8 \uc548\ucabd\uc740 \ud0a4\ub178\ud50c\ub77c\uc988\ub9c8\ub85c \uc774\ub8e8\uc5b4\uc838 \uc788\ub2e4. \uc774 \ud0a4\ub178\ud50c\ub77c\uc988\ub9c8\ub294 \uc218\ucd95\uc131\uc774 \uc788\uc73c\ubbc0\ub85c, \uc774 \ubd80\ubd84\uc774 \ud30c\uad34\ub418\uba74 \uc790\ub8e8 \ubd80\ubd84\uc758 \uc218\ucd95 \uc6b4\ub3d9\uc740 \uc77c\uc5b4\ub098\uc9c0 \uc54a\ub294\ub2e4. \ub610 \uac15\uc7a5\ub3d9\ubb3c\uc778 \ud788\ub4dc\ub77c\ub098 \ub9d0\ubbf8\uc798 \ub4f1\uc740 \uadfc\uc721 \uc0c1\ud53c\uc138\ud3ec\uac00 \uccb4\ubcbd \uc548\uc5d0 \uc0b0\uc7ac\ud558\uace0 \uc788\uc5b4, \uadf8 \uc12c\uc720 \ubaa8\uc591\uc73c\ub85c \ubd84\ud654\ud558\uace0 \uc788\ub294 \uc6d0\ud615\uc9c8\uc758 \uc77c\ubd80\uac00 \uc218\ucd95\ud558\uc5ec \uccb4\ubcbd\uc758 \uc6b4\ub3d9\uc774 \uc77c\uc5b4\ub09c\ub2e4."} {"question": "\uc774\ud574\ucc2c\uc774 \uace0\ud5a5\uc5d0 \ub0b4\ub824\uac14\ub2e4\uac00 \ub2e4\uc2dc \uc0c1\uacbd\ud558\uc5ec \ubc14\ub85c \ud55c \uc77c\uc740 \ubb34\uc5c7\uc778\uac00?", "paragraph": "1972\ub144 10\uc6d4 17\uc77c \uc720\uc2e0 \uc120\ud3ec\ub97c \uacc4\uae30\ub85c \ud559\uc0dd\uc6b4\ub3d9\uc5d0 \ub6f0\uc5b4\ub4e4\uc5c8\ub2e4. \uadf8\ub294 \uc720\uc2e0 \uc120\ud3ec\ub85c \ud734\uad50\ub839\uc774 \ub0b4\ub824\uc9c0\uc790 \uace0\ud5a5 \uccad\uc591\uc5d0 \ub0b4\ub824\uc654\ub294\ub370 \uc544\ubc84\uc9c0\uac00 \"\ub098\ub77c\uac00 \uc774\ubaa8\uc591\uc778\ub370 \ud559\uc0dd\ub4e4\uc774 \ub370\ubaa8\ub3c4 \ud558\uc9c0 \uc54a\ub290\ub0d0\"\uba70 \uc9c8\ucc45\uc744 \ubc1b\uace0 \ubc14\ub85c \uc0c1\uacbd\ud558\uc5ec \ud559\uc0dd\uc6b4\ub3d9 \uc368\ud074\uc5d0 \uac00\uc785\ud588\ub2e4\uace0 \ud55c\ub2e4. \uc5b4\ub824\uc6b4 \ud658\uacbd\uc5d0\uc11c \uadf8\ub294 \uc9c1\uc811 \ub9c9\ub178\ub3d9\uc744 \ud558\uba70 \uc0dd\uacc4\uc640 \ud559\ube44\ub97c \ub9c8\ub828\ud574 \ub098\uac14\uc73c\ub098 1974\ub144 \ubbfc\uccad\ud559\ub828 \uc0ac\uac74\uc73c\ub85c \ud22c\uc625\ub418\uc5b4 1\ub144\uc744 \ubcf5\uc5ed\ud558\uace0 \ucd9c\uc18c\ud588\ub2e4. \uc774\ud6c4 \uc774\ud574\ucc2c\uc740 \uc0dd\uacc4\ub97c \uc704\ud574 \ubb34\uc5ed\ud68c\uc0ac\uc5d0 \ub2e4\ub2c8\uae30\ub3c4 \ud558\uace0 \ub3d9\uc544\uc77c\ubcf4\uc5d0\uc11c \ud574\uc9c1\ub41c \uae30\uc790\ub4e4\uc774 \ucc28\ub9b0 \ubc88\uc5ed\uc2e4\uc5d0\uc11c \ubc88\uc5ed\uc744 \ud558\uae30\ub3c4 \ud588\uc73c\uba70 \uc5e0\ub124\uc2a4\ud2f0 \ud55c\uad6d\uc9c0\ubd80 \uc0c1\uadfc\uc790\ub85c \uc77c\ud558\ub2e4 \ud3c9\uc18c \uad00\uc2ec\uc774 \ub9ce\ub358 \ucd9c\ud310\uc77c\uc744 \uc775\ud788\ub824\uace0 \ubc94\uc6b0\uc0ac\uc5d0\uc11c \uc77c\ud558\uae30\ub3c4 \ud588\ub2e4. 1978\ub144 \uc0ac\ud68c\ud559\uacfc \ud559\uc220\ubaa8\uc784\uc5d0\uc11c \ub9cc\ub098 \uc0ac\uadc0\uc5b4 \uc624\ub358 \uae40\uc815\uc625\uacfc \uacb0\ud63c\ud558\uace0 \uad11\uc7a5\uc11c\uc801\uc744 \uc124\ub9bd\ud558\uc600\ub2e4. \ucd9c\ud310\uc0ac '\ud55c\ub9c8\ub2f9'\uacfc '\ud3c9\ubbfc\uc11c\ub2f9'\uc744 \uc124\ub9bd\ud588\uc73c\ub098 \ubd88\uc628\uc11c\uc801\uc744 \ucd9c\ud310\ud588\ub2e4\ub294 \uc774\uc720\ub85c \ub4f1\ub85d\uc744 \ucde8\uc18c\ub2f9\ud558\uc5ec \ub3cc\ubca0\uac1c \ucd9c\ud310\uc0ac\ub97c \uc124\ub9bd\ud558\uace0 \uc0ac\ud68c\uacfc\ud559 \uc11c\uc801\uc744 \uc8fc\ub85c \ucd9c\ud310\ud588\ub2e4.", "answer": "\ud559\uc0dd\uc6b4\ub3d9 \uc368\ud074\uc5d0 \uac00\uc785", "sentence": "\uadf8\ub294 \uc720\uc2e0 \uc120\ud3ec\ub85c \ud734\uad50\ub839\uc774 \ub0b4\ub824\uc9c0\uc790 \uace0\ud5a5 \uccad\uc591\uc5d0 \ub0b4\ub824\uc654\ub294\ub370 \uc544\ubc84\uc9c0\uac00 \"\ub098\ub77c\uac00 \uc774\ubaa8\uc591\uc778\ub370 \ud559\uc0dd\ub4e4\uc774 \ub370\ubaa8\ub3c4 \ud558\uc9c0 \uc54a\ub290\ub0d0\"\uba70 \uc9c8\ucc45\uc744 \ubc1b\uace0 \ubc14\ub85c \uc0c1\uacbd\ud558\uc5ec \ud559\uc0dd\uc6b4\ub3d9 \uc368\ud074\uc5d0 \uac00\uc785 \ud588\ub2e4\uace0 \ud55c\ub2e4.", "paragraph_sentence": "1972\ub144 10\uc6d4 17\uc77c \uc720\uc2e0 \uc120\ud3ec\ub97c \uacc4\uae30\ub85c \ud559\uc0dd\uc6b4\ub3d9\uc5d0 \ub6f0\uc5b4\ub4e4\uc5c8\ub2e4. \uadf8\ub294 \uc720\uc2e0 \uc120\ud3ec\ub85c \ud734\uad50\ub839\uc774 \ub0b4\ub824\uc9c0\uc790 \uace0\ud5a5 \uccad\uc591\uc5d0 \ub0b4\ub824\uc654\ub294\ub370 \uc544\ubc84\uc9c0\uac00 \"\ub098\ub77c\uac00 \uc774\ubaa8\uc591\uc778\ub370 \ud559\uc0dd\ub4e4\uc774 \ub370\ubaa8\ub3c4 \ud558\uc9c0 \uc54a\ub290\ub0d0\"\uba70 \uc9c8\ucc45\uc744 \ubc1b\uace0 \ubc14\ub85c \uc0c1\uacbd\ud558\uc5ec \ud559\uc0dd\uc6b4\ub3d9 \uc368\ud074\uc5d0 \uac00\uc785 \ud588\ub2e4\uace0 \ud55c\ub2e4. \uc5b4\ub824\uc6b4 \ud658\uacbd\uc5d0\uc11c \uadf8\ub294 \uc9c1\uc811 \ub9c9\ub178\ub3d9\uc744 \ud558\uba70 \uc0dd\uacc4\uc640 \ud559\ube44\ub97c \ub9c8\ub828\ud574 \ub098\uac14\uc73c\ub098 1974\ub144 \ubbfc\uccad\ud559\ub828 \uc0ac\uac74\uc73c\ub85c \ud22c\uc625\ub418\uc5b4 1\ub144\uc744 \ubcf5\uc5ed\ud558\uace0 \ucd9c\uc18c\ud588\ub2e4. \uc774\ud6c4 \uc774\ud574\ucc2c\uc740 \uc0dd\uacc4\ub97c \uc704\ud574 \ubb34\uc5ed\ud68c\uc0ac\uc5d0 \ub2e4\ub2c8\uae30\ub3c4 \ud558\uace0 \ub3d9\uc544\uc77c\ubcf4\uc5d0\uc11c \ud574\uc9c1\ub41c \uae30\uc790\ub4e4\uc774 \ucc28\ub9b0 \ubc88\uc5ed\uc2e4\uc5d0\uc11c \ubc88\uc5ed\uc744 \ud558\uae30\ub3c4 \ud588\uc73c\uba70 \uc5e0\ub124\uc2a4\ud2f0 \ud55c\uad6d\uc9c0\ubd80 \uc0c1\uadfc\uc790\ub85c \uc77c\ud558\ub2e4 \ud3c9\uc18c \uad00\uc2ec\uc774 \ub9ce\ub358 \ucd9c\ud310\uc77c\uc744 \uc775\ud788\ub824\uace0 \ubc94\uc6b0\uc0ac\uc5d0\uc11c \uc77c\ud558\uae30\ub3c4 \ud588\ub2e4. 1978\ub144 \uc0ac\ud68c\ud559\uacfc \ud559\uc220\ubaa8\uc784\uc5d0\uc11c \ub9cc\ub098 \uc0ac\uadc0\uc5b4 \uc624\ub358 \uae40\uc815\uc625\uacfc \uacb0\ud63c\ud558\uace0 \uad11\uc7a5\uc11c\uc801\uc744 \uc124\ub9bd\ud558\uc600\ub2e4. \ucd9c\ud310\uc0ac '\ud55c\ub9c8\ub2f9'\uacfc '\ud3c9\ubbfc\uc11c\ub2f9'\uc744 \uc124\ub9bd\ud588\uc73c\ub098 \ubd88\uc628\uc11c\uc801\uc744 \ucd9c\ud310\ud588\ub2e4\ub294 \uc774\uc720\ub85c \ub4f1\ub85d\uc744 \ucde8\uc18c\ub2f9\ud558\uc5ec \ub3cc\ubca0\uac1c \ucd9c\ud310\uc0ac\ub97c \uc124\ub9bd\ud558\uace0 \uc0ac\ud68c\uacfc\ud559 \uc11c\uc801\uc744 \uc8fc\ub85c \ucd9c\ud310\ud588\ub2e4.", "paragraph_answer": "1972\ub144 10\uc6d4 17\uc77c \uc720\uc2e0 \uc120\ud3ec\ub97c \uacc4\uae30\ub85c \ud559\uc0dd\uc6b4\ub3d9\uc5d0 \ub6f0\uc5b4\ub4e4\uc5c8\ub2e4. \uadf8\ub294 \uc720\uc2e0 \uc120\ud3ec\ub85c \ud734\uad50\ub839\uc774 \ub0b4\ub824\uc9c0\uc790 \uace0\ud5a5 \uccad\uc591\uc5d0 \ub0b4\ub824\uc654\ub294\ub370 \uc544\ubc84\uc9c0\uac00 \"\ub098\ub77c\uac00 \uc774\ubaa8\uc591\uc778\ub370 \ud559\uc0dd\ub4e4\uc774 \ub370\ubaa8\ub3c4 \ud558\uc9c0 \uc54a\ub290\ub0d0\"\uba70 \uc9c8\ucc45\uc744 \ubc1b\uace0 \ubc14\ub85c \uc0c1\uacbd\ud558\uc5ec \ud559\uc0dd\uc6b4\ub3d9 \uc368\ud074\uc5d0 \uac00\uc785 \ud588\ub2e4\uace0 \ud55c\ub2e4. \uc5b4\ub824\uc6b4 \ud658\uacbd\uc5d0\uc11c \uadf8\ub294 \uc9c1\uc811 \ub9c9\ub178\ub3d9\uc744 \ud558\uba70 \uc0dd\uacc4\uc640 \ud559\ube44\ub97c \ub9c8\ub828\ud574 \ub098\uac14\uc73c\ub098 1974\ub144 \ubbfc\uccad\ud559\ub828 \uc0ac\uac74\uc73c\ub85c \ud22c\uc625\ub418\uc5b4 1\ub144\uc744 \ubcf5\uc5ed\ud558\uace0 \ucd9c\uc18c\ud588\ub2e4. \uc774\ud6c4 \uc774\ud574\ucc2c\uc740 \uc0dd\uacc4\ub97c \uc704\ud574 \ubb34\uc5ed\ud68c\uc0ac\uc5d0 \ub2e4\ub2c8\uae30\ub3c4 \ud558\uace0 \ub3d9\uc544\uc77c\ubcf4\uc5d0\uc11c \ud574\uc9c1\ub41c \uae30\uc790\ub4e4\uc774 \ucc28\ub9b0 \ubc88\uc5ed\uc2e4\uc5d0\uc11c \ubc88\uc5ed\uc744 \ud558\uae30\ub3c4 \ud588\uc73c\uba70 \uc5e0\ub124\uc2a4\ud2f0 \ud55c\uad6d\uc9c0\ubd80 \uc0c1\uadfc\uc790\ub85c \uc77c\ud558\ub2e4 \ud3c9\uc18c \uad00\uc2ec\uc774 \ub9ce\ub358 \ucd9c\ud310\uc77c\uc744 \uc775\ud788\ub824\uace0 \ubc94\uc6b0\uc0ac\uc5d0\uc11c \uc77c\ud558\uae30\ub3c4 \ud588\ub2e4. 1978\ub144 \uc0ac\ud68c\ud559\uacfc \ud559\uc220\ubaa8\uc784\uc5d0\uc11c \ub9cc\ub098 \uc0ac\uadc0\uc5b4 \uc624\ub358 \uae40\uc815\uc625\uacfc \uacb0\ud63c\ud558\uace0 \uad11\uc7a5\uc11c\uc801\uc744 \uc124\ub9bd\ud558\uc600\ub2e4. \ucd9c\ud310\uc0ac '\ud55c\ub9c8\ub2f9'\uacfc '\ud3c9\ubbfc\uc11c\ub2f9'\uc744 \uc124\ub9bd\ud588\uc73c\ub098 \ubd88\uc628\uc11c\uc801\uc744 \ucd9c\ud310\ud588\ub2e4\ub294 \uc774\uc720\ub85c \ub4f1\ub85d\uc744 \ucde8\uc18c\ub2f9\ud558\uc5ec \ub3cc\ubca0\uac1c \ucd9c\ud310\uc0ac\ub97c \uc124\ub9bd\ud558\uace0 \uc0ac\ud68c\uacfc\ud559 \uc11c\uc801\uc744 \uc8fc\ub85c \ucd9c\ud310\ud588\ub2e4.", "sentence_answer": "\uadf8\ub294 \uc720\uc2e0 \uc120\ud3ec\ub85c \ud734\uad50\ub839\uc774 \ub0b4\ub824\uc9c0\uc790 \uace0\ud5a5 \uccad\uc591\uc5d0 \ub0b4\ub824\uc654\ub294\ub370 \uc544\ubc84\uc9c0\uac00 \"\ub098\ub77c\uac00 \uc774\ubaa8\uc591\uc778\ub370 \ud559\uc0dd\ub4e4\uc774 \ub370\ubaa8\ub3c4 \ud558\uc9c0 \uc54a\ub290\ub0d0\"\uba70 \uc9c8\ucc45\uc744 \ubc1b\uace0 \ubc14\ub85c \uc0c1\uacbd\ud558\uc5ec \ud559\uc0dd\uc6b4\ub3d9 \uc368\ud074\uc5d0 \uac00\uc785 \ud588\ub2e4\uace0 \ud55c\ub2e4.", "paragraph_id": "6547933-0-1", "paragraph_question": "question: \uc774\ud574\ucc2c\uc774 \uace0\ud5a5\uc5d0 \ub0b4\ub824\uac14\ub2e4\uac00 \ub2e4\uc2dc \uc0c1\uacbd\ud558\uc5ec \ubc14\ub85c \ud55c \uc77c\uc740 \ubb34\uc5c7\uc778\uac00?, context: 1972\ub144 10\uc6d4 17\uc77c \uc720\uc2e0 \uc120\ud3ec\ub97c \uacc4\uae30\ub85c \ud559\uc0dd\uc6b4\ub3d9\uc5d0 \ub6f0\uc5b4\ub4e4\uc5c8\ub2e4. \uadf8\ub294 \uc720\uc2e0 \uc120\ud3ec\ub85c \ud734\uad50\ub839\uc774 \ub0b4\ub824\uc9c0\uc790 \uace0\ud5a5 \uccad\uc591\uc5d0 \ub0b4\ub824\uc654\ub294\ub370 \uc544\ubc84\uc9c0\uac00 \"\ub098\ub77c\uac00 \uc774\ubaa8\uc591\uc778\ub370 \ud559\uc0dd\ub4e4\uc774 \ub370\ubaa8\ub3c4 \ud558\uc9c0 \uc54a\ub290\ub0d0\"\uba70 \uc9c8\ucc45\uc744 \ubc1b\uace0 \ubc14\ub85c \uc0c1\uacbd\ud558\uc5ec \ud559\uc0dd\uc6b4\ub3d9 \uc368\ud074\uc5d0 \uac00\uc785\ud588\ub2e4\uace0 \ud55c\ub2e4. \uc5b4\ub824\uc6b4 \ud658\uacbd\uc5d0\uc11c \uadf8\ub294 \uc9c1\uc811 \ub9c9\ub178\ub3d9\uc744 \ud558\uba70 \uc0dd\uacc4\uc640 \ud559\ube44\ub97c \ub9c8\ub828\ud574 \ub098\uac14\uc73c\ub098 1974\ub144 \ubbfc\uccad\ud559\ub828 \uc0ac\uac74\uc73c\ub85c \ud22c\uc625\ub418\uc5b4 1\ub144\uc744 \ubcf5\uc5ed\ud558\uace0 \ucd9c\uc18c\ud588\ub2e4. \uc774\ud6c4 \uc774\ud574\ucc2c\uc740 \uc0dd\uacc4\ub97c \uc704\ud574 \ubb34\uc5ed\ud68c\uc0ac\uc5d0 \ub2e4\ub2c8\uae30\ub3c4 \ud558\uace0 \ub3d9\uc544\uc77c\ubcf4\uc5d0\uc11c \ud574\uc9c1\ub41c \uae30\uc790\ub4e4\uc774 \ucc28\ub9b0 \ubc88\uc5ed\uc2e4\uc5d0\uc11c \ubc88\uc5ed\uc744 \ud558\uae30\ub3c4 \ud588\uc73c\uba70 \uc5e0\ub124\uc2a4\ud2f0 \ud55c\uad6d\uc9c0\ubd80 \uc0c1\uadfc\uc790\ub85c \uc77c\ud558\ub2e4 \ud3c9\uc18c \uad00\uc2ec\uc774 \ub9ce\ub358 \ucd9c\ud310\uc77c\uc744 \uc775\ud788\ub824\uace0 \ubc94\uc6b0\uc0ac\uc5d0\uc11c \uc77c\ud558\uae30\ub3c4 \ud588\ub2e4. 1978\ub144 \uc0ac\ud68c\ud559\uacfc \ud559\uc220\ubaa8\uc784\uc5d0\uc11c \ub9cc\ub098 \uc0ac\uadc0\uc5b4 \uc624\ub358 \uae40\uc815\uc625\uacfc \uacb0\ud63c\ud558\uace0 \uad11\uc7a5\uc11c\uc801\uc744 \uc124\ub9bd\ud558\uc600\ub2e4. \ucd9c\ud310\uc0ac '\ud55c\ub9c8\ub2f9'\uacfc '\ud3c9\ubbfc\uc11c\ub2f9'\uc744 \uc124\ub9bd\ud588\uc73c\ub098 \ubd88\uc628\uc11c\uc801\uc744 \ucd9c\ud310\ud588\ub2e4\ub294 \uc774\uc720\ub85c \ub4f1\ub85d\uc744 \ucde8\uc18c\ub2f9\ud558\uc5ec \ub3cc\ubca0\uac1c \ucd9c\ud310\uc0ac\ub97c \uc124\ub9bd\ud558\uace0 \uc0ac\ud68c\uacfc\ud559 \uc11c\uc801\uc744 \uc8fc\ub85c \ucd9c\ud310\ud588\ub2e4."} {"question": "\uc724\uc131\ube48\uc774 \uccab \ucd9c\uc804\ud55c \uc62c\ub9bc\ud53d\uc5d0\uc11c\uc758 \uc21c\uc704\ub294?", "paragraph": "\uac15\uad11\ubc30 \uad50\uc218\uc5d0\uac8c \uc9d1\uc911\uc801\uc73c\ub85c \ud6c8\ub828\uc744 \ubc1b\uc73c\uba70 \ube60\ub974\uac8c \uc2a4\ucf08\ub808\ud1a4\uc744 \ubab8\uc5d0 \uc775\ud614\uace0, \uc785\ubb38 3\uac1c\uc6d4 \ub9cc\uc778 2012\ub144 9\uc6d4\uc5d0 \ub300\ud55c\ubbfc\uad6d \uad6d\uac00\ub300\ud45c \uc120\ubc1c\uc804\uc744 \ud1b5\uacfc\ud588\ub2e4. \uad6d\uac00\ub300\ud45c\uac00 \ub41c \uc724\uc131\ube48\uc740 \uc8fc\ub2c8\uc5b4 \uc2e0\ubd84\uc774\ub358 2014-15\uc2dc\uc98c \uc6d4\ub4dc\ucef5 2\ucc28 \ub300\ud68c\ubd80\ud130 3\uc704\ub97c \ucc28\uc9c0\ud558\uba70 \uc138\uacc4 \uc2a4\ucf08\ub808\ud1a4\uacc4\uc5d0 \ub4f1\uc7a5\ud588\ub2e4. \uc6d4\ub4dc\ucef5 5\ucc28 \ub300\ud68c\uc5d0\uc11c \uc740\uba54\ub2ec\uc744 \ubaa9\uc5d0 \uac78\uc5c8\ub2e4. 2014\ub144 \ucc98\uc74c\uc73c\ub85c \ucd9c\uc804\ud55c \uc62c\ub9bc\ud53d\uc778 2014\ub144 \ub3d9\uacc4 \uc62c\ub9bc\ud53d\uc5d0\uc11c\ub294 16\uc704\uc5d0 \uadf8\ucce4\uc9c0\ub9cc, \uc131\uc778\uc774 \ub41c 2015-16 \uc2dc\uc98c\uc5d0 '\uc2a4\ucf08\ub808\ud1a4\uacc4\uc758 \uc6b0\uc0ac\uc778 \ubcfc\ud2b8'\ub85c \ubd88\ub9ac\ub358 \ub9c8\ub974\ud2f4\uc2a4 \ub450\ucfe0\ub974\uc2a4\ub3c4 \ud55c \ucc28\ub840 \ub118\uc5c8\ub2e4. 2016\ub144 \uc138\uacc4 \uc120\uc218\uad8c \ub300\ud68c\uc5d0 \ucc38\uac00\ud574 \uc740\uba54\ub2ec\uc744 \ud68d\ub4dd\ud558\uc5ec \ub300\ud55c\ubbfc\uad6d \uc120\uc218\ub85c\ub294 \ucd5c\ucd08\ub85c \uc138\uacc4 \uc120\uc218\uad8c \ub300\ud68c \uc2a4\ucf08\ub808\ud1a4 \uba54\ub2ec\uc744 \ud68d\ub4dd\ud588\ub2e4. 2016\ub144 2\uc6d4 6\uc77c \uc370\ub9e4 \uc885\ubaa9\uc774 \ud0c4\uc0dd\ud55c \uc2a4\uc704\uc2a4 \uc0dd\ubaa8\ub9ac\uce20\uc5d0\uc11c \ud3bc\uccd0\uc9c4 \uc6d4\ub4dc\ucef5 7\ucc28 \ub300\ud68c\uc5d0\uc11c \uccab \uae08\uba54\ub2ec\uc744 \ubaa9\uc5d0 \uac74\ub2e4.", "answer": "16\uc704", "sentence": "2014\ub144 \ucc98\uc74c\uc73c\ub85c \ucd9c\uc804\ud55c \uc62c\ub9bc\ud53d\uc778 2014\ub144 \ub3d9\uacc4 \uc62c\ub9bc\ud53d\uc5d0\uc11c\ub294 16\uc704 \uc5d0", "paragraph_sentence": "\uac15\uad11\ubc30 \uad50\uc218\uc5d0\uac8c \uc9d1\uc911\uc801\uc73c\ub85c \ud6c8\ub828\uc744 \ubc1b\uc73c\uba70 \ube60\ub974\uac8c \uc2a4\ucf08\ub808\ud1a4\uc744 \ubab8\uc5d0 \uc775\ud614\uace0, \uc785\ubb38 3\uac1c\uc6d4 \ub9cc\uc778 2012\ub144 9\uc6d4\uc5d0 \ub300\ud55c\ubbfc\uad6d \uad6d\uac00\ub300\ud45c \uc120\ubc1c\uc804\uc744 \ud1b5\uacfc\ud588\ub2e4. \uad6d\uac00\ub300\ud45c\uac00 \ub41c \uc724\uc131\ube48\uc740 \uc8fc\ub2c8\uc5b4 \uc2e0\ubd84\uc774\ub358 2014-15\uc2dc\uc98c \uc6d4\ub4dc\ucef5 2\ucc28 \ub300\ud68c\ubd80\ud130 3\uc704\ub97c \ucc28\uc9c0\ud558\uba70 \uc138\uacc4 \uc2a4\ucf08\ub808\ud1a4\uacc4\uc5d0 \ub4f1\uc7a5\ud588\ub2e4. \uc6d4\ub4dc\ucef5 5\ucc28 \ub300\ud68c\uc5d0\uc11c \uc740\uba54\ub2ec\uc744 \ubaa9\uc5d0 \uac78\uc5c8\ub2e4. 2014\ub144 \ucc98\uc74c\uc73c\ub85c \ucd9c\uc804\ud55c \uc62c\ub9bc\ud53d\uc778 2014\ub144 \ub3d9\uacc4 \uc62c\ub9bc\ud53d\uc5d0\uc11c\ub294 16\uc704 \uc5d0 \uadf8\ucce4\uc9c0\ub9cc, \uc131\uc778\uc774 \ub41c 2015-16 \uc2dc\uc98c\uc5d0 '\uc2a4\ucf08\ub808\ud1a4\uacc4\uc758 \uc6b0\uc0ac\uc778 \ubcfc\ud2b8'\ub85c \ubd88\ub9ac\ub358 \ub9c8\ub974\ud2f4\uc2a4 \ub450\ucfe0\ub974\uc2a4\ub3c4 \ud55c \ucc28\ub840 \ub118\uc5c8\ub2e4. 2016\ub144 \uc138\uacc4 \uc120\uc218\uad8c \ub300\ud68c\uc5d0 \ucc38\uac00\ud574 \uc740\uba54\ub2ec\uc744 \ud68d\ub4dd\ud558\uc5ec \ub300\ud55c\ubbfc\uad6d \uc120\uc218\ub85c\ub294 \ucd5c\ucd08\ub85c \uc138\uacc4 \uc120\uc218\uad8c \ub300\ud68c \uc2a4\ucf08\ub808\ud1a4 \uba54\ub2ec\uc744 \ud68d\ub4dd\ud588\ub2e4. 2016\ub144 2\uc6d4 6\uc77c \uc370\ub9e4 \uc885\ubaa9\uc774 \ud0c4\uc0dd\ud55c \uc2a4\uc704\uc2a4 \uc0dd\ubaa8\ub9ac\uce20\uc5d0\uc11c \ud3bc\uccd0\uc9c4 \uc6d4\ub4dc\ucef5 7\ucc28 \ub300\ud68c\uc5d0\uc11c \uccab \uae08\uba54\ub2ec\uc744 \ubaa9\uc5d0 \uac74\ub2e4.", "paragraph_answer": "\uac15\uad11\ubc30 \uad50\uc218\uc5d0\uac8c \uc9d1\uc911\uc801\uc73c\ub85c \ud6c8\ub828\uc744 \ubc1b\uc73c\uba70 \ube60\ub974\uac8c \uc2a4\ucf08\ub808\ud1a4\uc744 \ubab8\uc5d0 \uc775\ud614\uace0, \uc785\ubb38 3\uac1c\uc6d4 \ub9cc\uc778 2012\ub144 9\uc6d4\uc5d0 \ub300\ud55c\ubbfc\uad6d \uad6d\uac00\ub300\ud45c \uc120\ubc1c\uc804\uc744 \ud1b5\uacfc\ud588\ub2e4. \uad6d\uac00\ub300\ud45c\uac00 \ub41c \uc724\uc131\ube48\uc740 \uc8fc\ub2c8\uc5b4 \uc2e0\ubd84\uc774\ub358 2014-15\uc2dc\uc98c \uc6d4\ub4dc\ucef5 2\ucc28 \ub300\ud68c\ubd80\ud130 3\uc704\ub97c \ucc28\uc9c0\ud558\uba70 \uc138\uacc4 \uc2a4\ucf08\ub808\ud1a4\uacc4\uc5d0 \ub4f1\uc7a5\ud588\ub2e4. \uc6d4\ub4dc\ucef5 5\ucc28 \ub300\ud68c\uc5d0\uc11c \uc740\uba54\ub2ec\uc744 \ubaa9\uc5d0 \uac78\uc5c8\ub2e4. 2014\ub144 \ucc98\uc74c\uc73c\ub85c \ucd9c\uc804\ud55c \uc62c\ub9bc\ud53d\uc778 2014\ub144 \ub3d9\uacc4 \uc62c\ub9bc\ud53d\uc5d0\uc11c\ub294 16\uc704 \uc5d0 \uadf8\ucce4\uc9c0\ub9cc, \uc131\uc778\uc774 \ub41c 2015-16 \uc2dc\uc98c\uc5d0 '\uc2a4\ucf08\ub808\ud1a4\uacc4\uc758 \uc6b0\uc0ac\uc778 \ubcfc\ud2b8'\ub85c \ubd88\ub9ac\ub358 \ub9c8\ub974\ud2f4\uc2a4 \ub450\ucfe0\ub974\uc2a4\ub3c4 \ud55c \ucc28\ub840 \ub118\uc5c8\ub2e4. 2016\ub144 \uc138\uacc4 \uc120\uc218\uad8c \ub300\ud68c\uc5d0 \ucc38\uac00\ud574 \uc740\uba54\ub2ec\uc744 \ud68d\ub4dd\ud558\uc5ec \ub300\ud55c\ubbfc\uad6d \uc120\uc218\ub85c\ub294 \ucd5c\ucd08\ub85c \uc138\uacc4 \uc120\uc218\uad8c \ub300\ud68c \uc2a4\ucf08\ub808\ud1a4 \uba54\ub2ec\uc744 \ud68d\ub4dd\ud588\ub2e4. 2016\ub144 2\uc6d4 6\uc77c \uc370\ub9e4 \uc885\ubaa9\uc774 \ud0c4\uc0dd\ud55c \uc2a4\uc704\uc2a4 \uc0dd\ubaa8\ub9ac\uce20\uc5d0\uc11c \ud3bc\uccd0\uc9c4 \uc6d4\ub4dc\ucef5 7\ucc28 \ub300\ud68c\uc5d0\uc11c \uccab \uae08\uba54\ub2ec\uc744 \ubaa9\uc5d0 \uac74\ub2e4.", "sentence_answer": "2014\ub144 \ucc98\uc74c\uc73c\ub85c \ucd9c\uc804\ud55c \uc62c\ub9bc\ud53d\uc778 2014\ub144 \ub3d9\uacc4 \uc62c\ub9bc\ud53d\uc5d0\uc11c\ub294 16\uc704 \uc5d0", "paragraph_id": "6543926-2-1", "paragraph_question": "question: \uc724\uc131\ube48\uc774 \uccab \ucd9c\uc804\ud55c \uc62c\ub9bc\ud53d\uc5d0\uc11c\uc758 \uc21c\uc704\ub294?, context: \uac15\uad11\ubc30 \uad50\uc218\uc5d0\uac8c \uc9d1\uc911\uc801\uc73c\ub85c \ud6c8\ub828\uc744 \ubc1b\uc73c\uba70 \ube60\ub974\uac8c \uc2a4\ucf08\ub808\ud1a4\uc744 \ubab8\uc5d0 \uc775\ud614\uace0, \uc785\ubb38 3\uac1c\uc6d4 \ub9cc\uc778 2012\ub144 9\uc6d4\uc5d0 \ub300\ud55c\ubbfc\uad6d \uad6d\uac00\ub300\ud45c \uc120\ubc1c\uc804\uc744 \ud1b5\uacfc\ud588\ub2e4. \uad6d\uac00\ub300\ud45c\uac00 \ub41c \uc724\uc131\ube48\uc740 \uc8fc\ub2c8\uc5b4 \uc2e0\ubd84\uc774\ub358 2014-15\uc2dc\uc98c \uc6d4\ub4dc\ucef5 2\ucc28 \ub300\ud68c\ubd80\ud130 3\uc704\ub97c \ucc28\uc9c0\ud558\uba70 \uc138\uacc4 \uc2a4\ucf08\ub808\ud1a4\uacc4\uc5d0 \ub4f1\uc7a5\ud588\ub2e4. \uc6d4\ub4dc\ucef5 5\ucc28 \ub300\ud68c\uc5d0\uc11c \uc740\uba54\ub2ec\uc744 \ubaa9\uc5d0 \uac78\uc5c8\ub2e4. 2014\ub144 \ucc98\uc74c\uc73c\ub85c \ucd9c\uc804\ud55c \uc62c\ub9bc\ud53d\uc778 2014\ub144 \ub3d9\uacc4 \uc62c\ub9bc\ud53d\uc5d0\uc11c\ub294 16\uc704\uc5d0 \uadf8\ucce4\uc9c0\ub9cc, \uc131\uc778\uc774 \ub41c 2015-16 \uc2dc\uc98c\uc5d0 '\uc2a4\ucf08\ub808\ud1a4\uacc4\uc758 \uc6b0\uc0ac\uc778 \ubcfc\ud2b8'\ub85c \ubd88\ub9ac\ub358 \ub9c8\ub974\ud2f4\uc2a4 \ub450\ucfe0\ub974\uc2a4\ub3c4 \ud55c \ucc28\ub840 \ub118\uc5c8\ub2e4. 2016\ub144 \uc138\uacc4 \uc120\uc218\uad8c \ub300\ud68c\uc5d0 \ucc38\uac00\ud574 \uc740\uba54\ub2ec\uc744 \ud68d\ub4dd\ud558\uc5ec \ub300\ud55c\ubbfc\uad6d \uc120\uc218\ub85c\ub294 \ucd5c\ucd08\ub85c \uc138\uacc4 \uc120\uc218\uad8c \ub300\ud68c \uc2a4\ucf08\ub808\ud1a4 \uba54\ub2ec\uc744 \ud68d\ub4dd\ud588\ub2e4. 2016\ub144 2\uc6d4 6\uc77c \uc370\ub9e4 \uc885\ubaa9\uc774 \ud0c4\uc0dd\ud55c \uc2a4\uc704\uc2a4 \uc0dd\ubaa8\ub9ac\uce20\uc5d0\uc11c \ud3bc\uccd0\uc9c4 \uc6d4\ub4dc\ucef5 7\ucc28 \ub300\ud68c\uc5d0\uc11c \uccab \uae08\uba54\ub2ec\uc744 \ubaa9\uc5d0 \uac74\ub2e4."} {"question": "\ubc14\uadf8\ub108\uc758 \uad50\ud5a5\uace1 \ub2e4\uc7a5\uc870 \uc81c4\uc545\uc7a5\uc740 \uba87 \ub9c8\ub514\uc778\uac00?", "paragraph": "\uc54c\ub808\uadf8\ub85c \ubab0\ud1a0 \uc5d0 \ube44\ubc14\uccb4 C\uc7a5\uc870 4/4\ubc15\uc790. \uc18c\ub098\ud0c0 \ud615\uc2dd\uc73c\ub85c\uc11c, 397\ub9c8\ub514\ub85c \uad6c\uc131\ub418\uc5b4 \uc788\ub2e4. C\uc7a5\uc870\uc758 \uc73c\ub738\ud654\uc74c\uc744 f\ub85c \uc5f0\uc8fc\ud55c \ud6c4 \ud55c \ub9c8\ub514\uc758 \ud734\uc9c0\uac00 \uc788\uace0, \uc6b0\ud6c4 \ubc14\uc774\uc62c\ub9b0\uc73c\ub85c \uc81c1\uc8fc\uc81c\uac00 \uc81c\uc2dc\ub41c\ub2e4. \uac19\uc740 \ud615\ud0dc\ub85c \uac70\uc758 \uc774 \uc8fc\uc81c\ub3d9\uae30\ub97c \uc774\uc6a9\ud55c D\ub2e8\uc870\uc758 \uc751\ub2f5\uc545\uad6c\uac00 \uc774\uc5b4\uc9c4\ub2e4. \uc27c\ud45c\ub97c \ub07c\uace0 \uc800\uc74c\ud604\uc5d0 E\uc7a5\uc870\uc801\uc778 \uc774\ud589\uc545\uad6c\uac00 \ub098\ud0c0\ub098\uc11c \uacbd\uacfc\uad6c\uac00 \ub41c\ub2e4. 23\uc9f8\ub9c8\ub514\ubd80\ud130 \uc8fc\uc81c\uac00 f\ub85c \ud655\ubcf4\ub41c \ud6c4\uc758 \uad6c\uc131\uc740 \ub9c8\uce58 \ubc1c\uc804\ubd80\uac00 \uc2dc\uc791\ub41c \ub4ef\uc774 \uc8fc\uc81c\ub97c \uc0ac\uc6a9\ud55c \ubd80\ubd84\uc774 \uacc4\uc18d\ub41c\ub2e4. \uadf8 \uacfc\uc815\uc5d0\uc11c G\uc7a5\uc870\ub97c \ud5a5\ud558\uc5ec \uc81c2\uc8fc\uc81c\ubd80\ub97c \uc778\ub3c4\ud558\uc5ec A\ub2e8\uc870\ub85c \ub418\uac70\ub098, C\ub2e8\uc870\ub85c \uc870\ubc14\uafc8\ud558\uba74\uc11c \uc804\uac1c\uac00 \uc774\ub8e8\uc5b4\uc9c4\ub2e4. ff\uc758 \ud074\ub77c\uc774\ub9e5\uc2a4\uc5d0 \uc774\ub974\ub7ec \ud604\uc545\uae30\uad70\uc774 8\ubd84\uc74c\ud45c\ub85c \ud558\ud589\uc74c\uacc4\ub97c \uc5f0\uc8fc\ud558\uba74\uc11c \ub514\ubbf8\ub204\uc5d4\ub3c4\ud558\uba70, p\uc5d0 \uc774\ub974\uba74 \ubaa9\uad00\uc545\uae30\uad70\uc774 \ud3f4\ub9ac\ud3ec\ub2c9\uc801\uc778 \uc81c2\uc8fc\uc81c\ubd80\ub97c \uc81c\uc2dc\ud55c\ub2e4. \uc774 3\uc131 \uccb4\uc81c\uc758 \uc8fc\uc81c\ubd80\ub294 \uac01 \uc131\ubd80\uc5d0 \ub3d9\uc77c\ud55c \uc8fc\uc81c\uc801 \uc911\uc694\uc131\uc774 \ubd80\uc5ec\ub418\uc5b4 \uc788\ub2e4. \uc774 \uc8fc\uc81c\ubd80\ub294 \uad00\uc545\uae30\ub97c \uc911\uc2ec\uc73c\ub85c, \uc81c1\uc8fc\uc81c\uc758 \uc18c\uc7ac\ub97c \uc0ac\uc6a9\ud55c \ud604\uc545\uae30\uc758 \ub300\uc120\uc728\uacfc\ub3c4 \ub54c\ub54c\ub85c \ud63c\ud569\ub418\uba74\uc11c \ubc1c\uc804\ud55c\ub2e4. \uc810\ucc28 \uc81c1\uc8fc\uc81c\ub97c \uc774\uc6a9\ud55c \ud604\uc545\uae30\uad70\uc774 \uc8fc\ub3c4\uad8c\uc744 \uc7a1\uc544\uac00\uace0 \uc81c\uc2dc\ubd80\ub97c \ub9c8\ubb34\ub9ac\ud558\ub294 \uacbd\uacfc\uc801 \uc545\uad6c\ub85c \uc774\uc5b4\uc9c4\ub2e4.", "answer": "397\ub9c8\ub514", "sentence": "\uc18c\ub098\ud0c0 \ud615\uc2dd\uc73c\ub85c\uc11c, 397\ub9c8\ub514 \ub85c \uad6c\uc131\ub418\uc5b4 \uc788\ub2e4.", "paragraph_sentence": "\uc54c\ub808\uadf8\ub85c \ubab0\ud1a0 \uc5d0 \ube44\ubc14\uccb4 C\uc7a5\uc870 4/4\ubc15\uc790. \uc18c\ub098\ud0c0 \ud615\uc2dd\uc73c\ub85c\uc11c, 397\ub9c8\ub514 \ub85c \uad6c\uc131\ub418\uc5b4 \uc788\ub2e4. C\uc7a5\uc870\uc758 \uc73c\ub738\ud654\uc74c\uc744 f\ub85c \uc5f0\uc8fc\ud55c \ud6c4 \ud55c \ub9c8\ub514\uc758 \ud734\uc9c0\uac00 \uc788\uace0, \uc6b0\ud6c4 \ubc14\uc774\uc62c\ub9b0\uc73c\ub85c \uc81c1\uc8fc\uc81c\uac00 \uc81c\uc2dc\ub41c\ub2e4. \uac19\uc740 \ud615\ud0dc\ub85c \uac70\uc758 \uc774 \uc8fc\uc81c\ub3d9\uae30\ub97c \uc774\uc6a9\ud55c D\ub2e8\uc870\uc758 \uc751\ub2f5\uc545\uad6c\uac00 \uc774\uc5b4\uc9c4\ub2e4. \uc27c\ud45c\ub97c \ub07c\uace0 \uc800\uc74c\ud604\uc5d0 E\uc7a5\uc870\uc801\uc778 \uc774\ud589\uc545\uad6c\uac00 \ub098\ud0c0\ub098\uc11c \uacbd\uacfc\uad6c\uac00 \ub41c\ub2e4. 23\uc9f8\ub9c8\ub514\ubd80\ud130 \uc8fc\uc81c\uac00 f\ub85c \ud655\ubcf4\ub41c \ud6c4\uc758 \uad6c\uc131\uc740 \ub9c8\uce58 \ubc1c\uc804\ubd80\uac00 \uc2dc\uc791\ub41c \ub4ef\uc774 \uc8fc\uc81c\ub97c \uc0ac\uc6a9\ud55c \ubd80\ubd84\uc774 \uacc4\uc18d\ub41c\ub2e4. \uadf8 \uacfc\uc815\uc5d0\uc11c G\uc7a5\uc870\ub97c \ud5a5\ud558\uc5ec \uc81c2\uc8fc\uc81c\ubd80\ub97c \uc778\ub3c4\ud558\uc5ec A\ub2e8\uc870\ub85c \ub418\uac70\ub098, C\ub2e8\uc870\ub85c \uc870\ubc14\uafc8\ud558\uba74\uc11c \uc804\uac1c\uac00 \uc774\ub8e8\uc5b4\uc9c4\ub2e4. ff\uc758 \ud074\ub77c\uc774\ub9e5\uc2a4\uc5d0 \uc774\ub974\ub7ec \ud604\uc545\uae30\uad70\uc774 8\ubd84\uc74c\ud45c\ub85c \ud558\ud589\uc74c\uacc4\ub97c \uc5f0\uc8fc\ud558\uba74\uc11c \ub514\ubbf8\ub204\uc5d4\ub3c4\ud558\uba70, p\uc5d0 \uc774\ub974\uba74 \ubaa9\uad00\uc545\uae30\uad70\uc774 \ud3f4\ub9ac\ud3ec\ub2c9\uc801\uc778 \uc81c2\uc8fc\uc81c\ubd80\ub97c \uc81c\uc2dc\ud55c\ub2e4. \uc774 3\uc131 \uccb4\uc81c\uc758 \uc8fc\uc81c\ubd80\ub294 \uac01 \uc131\ubd80\uc5d0 \ub3d9\uc77c\ud55c \uc8fc\uc81c\uc801 \uc911\uc694\uc131\uc774 \ubd80\uc5ec\ub418\uc5b4 \uc788\ub2e4. \uc774 \uc8fc\uc81c\ubd80\ub294 \uad00\uc545\uae30\ub97c \uc911\uc2ec\uc73c\ub85c, \uc81c1\uc8fc\uc81c\uc758 \uc18c\uc7ac\ub97c \uc0ac\uc6a9\ud55c \ud604\uc545\uae30\uc758 \ub300\uc120\uc728\uacfc\ub3c4 \ub54c\ub54c\ub85c \ud63c\ud569\ub418\uba74\uc11c \ubc1c\uc804\ud55c\ub2e4. \uc810\ucc28 \uc81c1\uc8fc\uc81c\ub97c \uc774\uc6a9\ud55c \ud604\uc545\uae30\uad70\uc774 \uc8fc\ub3c4\uad8c\uc744 \uc7a1\uc544\uac00\uace0 \uc81c\uc2dc\ubd80\ub97c \ub9c8\ubb34\ub9ac\ud558\ub294 \uacbd\uacfc\uc801 \uc545\uad6c\ub85c \uc774\uc5b4\uc9c4\ub2e4.", "paragraph_answer": "\uc54c\ub808\uadf8\ub85c \ubab0\ud1a0 \uc5d0 \ube44\ubc14\uccb4 C\uc7a5\uc870 4/4\ubc15\uc790. \uc18c\ub098\ud0c0 \ud615\uc2dd\uc73c\ub85c\uc11c, 397\ub9c8\ub514 \ub85c \uad6c\uc131\ub418\uc5b4 \uc788\ub2e4. C\uc7a5\uc870\uc758 \uc73c\ub738\ud654\uc74c\uc744 f\ub85c \uc5f0\uc8fc\ud55c \ud6c4 \ud55c \ub9c8\ub514\uc758 \ud734\uc9c0\uac00 \uc788\uace0, \uc6b0\ud6c4 \ubc14\uc774\uc62c\ub9b0\uc73c\ub85c \uc81c1\uc8fc\uc81c\uac00 \uc81c\uc2dc\ub41c\ub2e4. \uac19\uc740 \ud615\ud0dc\ub85c \uac70\uc758 \uc774 \uc8fc\uc81c\ub3d9\uae30\ub97c \uc774\uc6a9\ud55c D\ub2e8\uc870\uc758 \uc751\ub2f5\uc545\uad6c\uac00 \uc774\uc5b4\uc9c4\ub2e4. \uc27c\ud45c\ub97c \ub07c\uace0 \uc800\uc74c\ud604\uc5d0 E\uc7a5\uc870\uc801\uc778 \uc774\ud589\uc545\uad6c\uac00 \ub098\ud0c0\ub098\uc11c \uacbd\uacfc\uad6c\uac00 \ub41c\ub2e4. 23\uc9f8\ub9c8\ub514\ubd80\ud130 \uc8fc\uc81c\uac00 f\ub85c \ud655\ubcf4\ub41c \ud6c4\uc758 \uad6c\uc131\uc740 \ub9c8\uce58 \ubc1c\uc804\ubd80\uac00 \uc2dc\uc791\ub41c \ub4ef\uc774 \uc8fc\uc81c\ub97c \uc0ac\uc6a9\ud55c \ubd80\ubd84\uc774 \uacc4\uc18d\ub41c\ub2e4. \uadf8 \uacfc\uc815\uc5d0\uc11c G\uc7a5\uc870\ub97c \ud5a5\ud558\uc5ec \uc81c2\uc8fc\uc81c\ubd80\ub97c \uc778\ub3c4\ud558\uc5ec A\ub2e8\uc870\ub85c \ub418\uac70\ub098, C\ub2e8\uc870\ub85c \uc870\ubc14\uafc8\ud558\uba74\uc11c \uc804\uac1c\uac00 \uc774\ub8e8\uc5b4\uc9c4\ub2e4. ff\uc758 \ud074\ub77c\uc774\ub9e5\uc2a4\uc5d0 \uc774\ub974\ub7ec \ud604\uc545\uae30\uad70\uc774 8\ubd84\uc74c\ud45c\ub85c \ud558\ud589\uc74c\uacc4\ub97c \uc5f0\uc8fc\ud558\uba74\uc11c \ub514\ubbf8\ub204\uc5d4\ub3c4\ud558\uba70, p\uc5d0 \uc774\ub974\uba74 \ubaa9\uad00\uc545\uae30\uad70\uc774 \ud3f4\ub9ac\ud3ec\ub2c9\uc801\uc778 \uc81c2\uc8fc\uc81c\ubd80\ub97c \uc81c\uc2dc\ud55c\ub2e4. \uc774 3\uc131 \uccb4\uc81c\uc758 \uc8fc\uc81c\ubd80\ub294 \uac01 \uc131\ubd80\uc5d0 \ub3d9\uc77c\ud55c \uc8fc\uc81c\uc801 \uc911\uc694\uc131\uc774 \ubd80\uc5ec\ub418\uc5b4 \uc788\ub2e4. \uc774 \uc8fc\uc81c\ubd80\ub294 \uad00\uc545\uae30\ub97c \uc911\uc2ec\uc73c\ub85c, \uc81c1\uc8fc\uc81c\uc758 \uc18c\uc7ac\ub97c \uc0ac\uc6a9\ud55c \ud604\uc545\uae30\uc758 \ub300\uc120\uc728\uacfc\ub3c4 \ub54c\ub54c\ub85c \ud63c\ud569\ub418\uba74\uc11c \ubc1c\uc804\ud55c\ub2e4. \uc810\ucc28 \uc81c1\uc8fc\uc81c\ub97c \uc774\uc6a9\ud55c \ud604\uc545\uae30\uad70\uc774 \uc8fc\ub3c4\uad8c\uc744 \uc7a1\uc544\uac00\uace0 \uc81c\uc2dc\ubd80\ub97c \ub9c8\ubb34\ub9ac\ud558\ub294 \uacbd\uacfc\uc801 \uc545\uad6c\ub85c \uc774\uc5b4\uc9c4\ub2e4.", "sentence_answer": "\uc18c\ub098\ud0c0 \ud615\uc2dd\uc73c\ub85c\uc11c, 397\ub9c8\ub514 \ub85c \uad6c\uc131\ub418\uc5b4 \uc788\ub2e4.", "paragraph_id": "6487516-12-0", "paragraph_question": "question: \ubc14\uadf8\ub108\uc758 \uad50\ud5a5\uace1 \ub2e4\uc7a5\uc870 \uc81c4\uc545\uc7a5\uc740 \uba87 \ub9c8\ub514\uc778\uac00?, context: \uc54c\ub808\uadf8\ub85c \ubab0\ud1a0 \uc5d0 \ube44\ubc14\uccb4 C\uc7a5\uc870 4/4\ubc15\uc790. \uc18c\ub098\ud0c0 \ud615\uc2dd\uc73c\ub85c\uc11c, 397\ub9c8\ub514\ub85c \uad6c\uc131\ub418\uc5b4 \uc788\ub2e4. C\uc7a5\uc870\uc758 \uc73c\ub738\ud654\uc74c\uc744 f\ub85c \uc5f0\uc8fc\ud55c \ud6c4 \ud55c \ub9c8\ub514\uc758 \ud734\uc9c0\uac00 \uc788\uace0, \uc6b0\ud6c4 \ubc14\uc774\uc62c\ub9b0\uc73c\ub85c \uc81c1\uc8fc\uc81c\uac00 \uc81c\uc2dc\ub41c\ub2e4. \uac19\uc740 \ud615\ud0dc\ub85c \uac70\uc758 \uc774 \uc8fc\uc81c\ub3d9\uae30\ub97c \uc774\uc6a9\ud55c D\ub2e8\uc870\uc758 \uc751\ub2f5\uc545\uad6c\uac00 \uc774\uc5b4\uc9c4\ub2e4. \uc27c\ud45c\ub97c \ub07c\uace0 \uc800\uc74c\ud604\uc5d0 E\uc7a5\uc870\uc801\uc778 \uc774\ud589\uc545\uad6c\uac00 \ub098\ud0c0\ub098\uc11c \uacbd\uacfc\uad6c\uac00 \ub41c\ub2e4. 23\uc9f8\ub9c8\ub514\ubd80\ud130 \uc8fc\uc81c\uac00 f\ub85c \ud655\ubcf4\ub41c \ud6c4\uc758 \uad6c\uc131\uc740 \ub9c8\uce58 \ubc1c\uc804\ubd80\uac00 \uc2dc\uc791\ub41c \ub4ef\uc774 \uc8fc\uc81c\ub97c \uc0ac\uc6a9\ud55c \ubd80\ubd84\uc774 \uacc4\uc18d\ub41c\ub2e4. \uadf8 \uacfc\uc815\uc5d0\uc11c G\uc7a5\uc870\ub97c \ud5a5\ud558\uc5ec \uc81c2\uc8fc\uc81c\ubd80\ub97c \uc778\ub3c4\ud558\uc5ec A\ub2e8\uc870\ub85c \ub418\uac70\ub098, C\ub2e8\uc870\ub85c \uc870\ubc14\uafc8\ud558\uba74\uc11c \uc804\uac1c\uac00 \uc774\ub8e8\uc5b4\uc9c4\ub2e4. ff\uc758 \ud074\ub77c\uc774\ub9e5\uc2a4\uc5d0 \uc774\ub974\ub7ec \ud604\uc545\uae30\uad70\uc774 8\ubd84\uc74c\ud45c\ub85c \ud558\ud589\uc74c\uacc4\ub97c \uc5f0\uc8fc\ud558\uba74\uc11c \ub514\ubbf8\ub204\uc5d4\ub3c4\ud558\uba70, p\uc5d0 \uc774\ub974\uba74 \ubaa9\uad00\uc545\uae30\uad70\uc774 \ud3f4\ub9ac\ud3ec\ub2c9\uc801\uc778 \uc81c2\uc8fc\uc81c\ubd80\ub97c \uc81c\uc2dc\ud55c\ub2e4. \uc774 3\uc131 \uccb4\uc81c\uc758 \uc8fc\uc81c\ubd80\ub294 \uac01 \uc131\ubd80\uc5d0 \ub3d9\uc77c\ud55c \uc8fc\uc81c\uc801 \uc911\uc694\uc131\uc774 \ubd80\uc5ec\ub418\uc5b4 \uc788\ub2e4. \uc774 \uc8fc\uc81c\ubd80\ub294 \uad00\uc545\uae30\ub97c \uc911\uc2ec\uc73c\ub85c, \uc81c1\uc8fc\uc81c\uc758 \uc18c\uc7ac\ub97c \uc0ac\uc6a9\ud55c \ud604\uc545\uae30\uc758 \ub300\uc120\uc728\uacfc\ub3c4 \ub54c\ub54c\ub85c \ud63c\ud569\ub418\uba74\uc11c \ubc1c\uc804\ud55c\ub2e4. \uc810\ucc28 \uc81c1\uc8fc\uc81c\ub97c \uc774\uc6a9\ud55c \ud604\uc545\uae30\uad70\uc774 \uc8fc\ub3c4\uad8c\uc744 \uc7a1\uc544\uac00\uace0 \uc81c\uc2dc\ubd80\ub97c \ub9c8\ubb34\ub9ac\ud558\ub294 \uacbd\uacfc\uc801 \uc545\uad6c\ub85c \uc774\uc5b4\uc9c4\ub2e4."} {"question": "\uc774\uc2ac\ub78c \uad6d\uac00 \uc218\ub9bd\uc744 \uc704\ud574 \ub7ec\uc2dc\uc544\uc5d0 \ub300\ud55c \ud14c\ub7ec\ub97c \ubc8c\uc774\ub294 \uce85\uce74\uc2a4 \uc5d0\ubbf8\ub808\uc774\ud2b8 \ub2e8\uccb4\ub294,\uc5b4\ub290\uc9c0\uc5ed\uc5d0 \uad6d\uac00 \uc218\ub9bd\uc744 \uc6d0\ud558\ub294\uac00?", "paragraph": "2010\ub144 \ubaa8\uc2a4\ud06c\ubc14 \uc9c0\ud558\ucca0 \ud14c\ub7ec\ub294 2007\ub144 \uc774\uce58\ucf00\ub9ac\uc57c \uccb4\uccb8 \uacf5\ud654\uad6d\uc744 \uacc4\uc2b9\ud55c \uccb4\uccb8\uc758 \ud14c\ub7ec \ub2e8\uccb4\uc778 \uce85\uce74\uc2a4 \uc5d0\ubbf8\ub808\uc774\ud2b8\uac00 \uc8fc\ub3c4\ud588\ub2e4. \uce85\uce74\uc2a4 \uc5d0\ubbf8\ub808\uc774\ud2b8\uc758 \ucc3d\uc124 \ubc30\uacbd\uc744 \uac70\uc2ac\ub7ec \uc62c\ub77c\uac00\uba74 \uccb4\uccb8\uc758 \ub3c5\ub9bd\uc744 \ub458\ub7ec\uc2fc \uc815\uce58 \uc138\ub825\uc758 \ubd84\uc5f4\uc774 \uc788\ub2e4. 1999\ub144 2\ucc28 \uccb4\uccb8\uc804\uc7c1\uc5d0\uc11c \ub7ec\uc2dc\uc544 \uc5f0\ubc29\uc774 \uc2b9\ub9ac\ud55c \uc9c1\ud6c4, \uccb4\uccb8\uc758 \uc815\uce58\uc138\ub825\uc740 \ub450 \ud30c\ubc8c\ub85c \ub098\ub258\uc5c8\ub2e4. \ub7ec\uc2dc\uc544 \uc5f0\ubc29\uc5d0 \uc794\uc874\ud558\uae38 \ud76c\ub9dd\ud558\ub294 \uc138\ub825\uc740 \ub7ec\uc2dc\uc544\uc758 \uc790\uce58\uacf5\ud654\uad6d\uc73c\ub85c\uc368 \uccb4\uccb8 \uacf5\ud654\uad6d\uc744 \uac74\uad6d, \uc790\uce58\uad8c\uc744 \uc778\uc815\ubc1b\uc558\uc73c\ub098, \ubd84\ub9ac\ub3c5\ub9bd\ud30c\ub294 \uccb4\uccb8\uc758 \uc644\uc804\ud55c \ub3c5\ub9bd\uc744 \ubaa9\ud45c\ub85c \ub7ec\uc2dc\uc544\ub97c \uc0c1\ub300\ub85c \uacc4\uc18d\uc801\uc73c\ub85c \ubb34\uc7a5\ud22c\uc7c1\uc744 \ubc8c\uc600\ub2e4. \uc804\uc7c1\uc774 \uae38\uc5b4\uc9d0\uc5d0 \ub530\ub77c \uc628\uac74\ud30c \uc815\uce58\uc778\uc774\uc5c8\ub358 \ub9c8\uc2a4\ud558\ub3c4\ud504 \ub4f1, \uc8fc\uc694 \uc9c0\ub3c4\uce35\uc744 \uc783\uc73c\uba70 \uc138\ub825 \uc720\uc9c0\uc5d0 \ud070 \ud0c0\uaca9\uc744 \uc785\uc5c8\ub2e4. \uc5ec\uae30\uc5d0 \uccb4\uccb8 \uac15\uacbd\ud30c\uac00 \ubc8c\uc778 \ubbfc\uac04\uc778 \ud14c\ub7ec \uc0ac\uac74\ub4e4\ub85c \uc774\uce58\ucf00\ub9ac\uc57c \uccb4\uccb8 \uc815\ubd80\ub294 \uc624\ud788\ub824 \uad6d\uc81c\uc801\uc73c\ub85c \uc774\ubbf8\uc9c0\uac00 \uc545\ud654\ub418\uba74\uc11c \ubd84\ub9ac\ub3c5\ub9bd\ud30c\ub294 \uc790\uc5f0\uc2a4\ub808 \uc57d\ud654\ub418\uc5c8\ub2e4. \uc774\ub7f0 \uc0c1\ud669\uc5d0\uc11c 2007\ub144 \uc774\uce58\ucf00\ub9ac\uc57c \uccb4\uccb8 \uacf5\ud654\uad6d\uc758 \ub300\ud1b5\ub839\uc774\uc5c8\ub358 \uc774\uce58\ucf00\ub9ac\uc57c \uccb4\uccb8 \uc815\ubd80\ub97c \uc5c6\uc560\uace0 \ub3c4\ucfe0 \uc6b0\ub9c8\ub85c\ud504\ub294 \uc790\uc2e0\uc744 \uc218\uc7a5\uc73c\ub85c \uc120\ucd9c, \uce85\uce74\uc2a4 \uc5d0\ubbf8\ub808\uc774\ud2b8\uc774\ub77c\ub294 \ub2e8\uccb4\ub97c \uc0c8\ub86d\uac8c \ucc3d\uc124\ud588\ub2e4. \ud604\uc7ac \uc774\ub4e4\uc740 \ubd81\uce85\uce74\uc2a4 \uc9c0\uc5ed\uc5d0 \uc774\uc2ac\ub78c \uad6d\uac00\ub97c \uc218\ub9bd\ud558\ub294 \uac83\uc744 \ubaa9\ud45c\ub85c \ub7ec\uc2dc\uc544\uc5d0 \ub300\ud55c \ud14c\ub7ec\ub97c \ubc8c\uc774\uace0 \uc788\uc73c\uba70, \ub7ec\uc2dc\uc544 \ub4f1 \uc5ec\ub7ec \uad6d\uac00\ub294 \uce85\uce74\uc2a4 \uc218\uc7a5\uad6d\uc744 \ud14c\ub7ec \ub2e8\uccb4\ub85c \ubd84\ub958\ud558\uace0 \uc788\ub2e4. 2002\ub144 \uc774\ud6c4 \ud604\uc7ac\uae4c\uc9c0 \uc774\ub4e4\uc774 \ubc8c\uc774\ub294 \ud14c\ub7ec\uc5d0 \uc758\ud574 5000\uba85 \uc774\uc0c1\uc758 \uc0ac\uc0c1\uc790\uac00 \uc18d\ucd9c\ud558\uace0 \uc788\ub2e4.", "answer": "\ubd81\uce85\uce74\uc2a4", "sentence": "\ud604\uc7ac \uc774\ub4e4\uc740 \ubd81\uce85\uce74\uc2a4 \uc9c0\uc5ed\uc5d0 \uc774\uc2ac\ub78c \uad6d\uac00\ub97c \uc218\ub9bd\ud558\ub294 \uac83\uc744 \ubaa9\ud45c\ub85c \ub7ec\uc2dc\uc544\uc5d0 \ub300\ud55c \ud14c\ub7ec\ub97c \ubc8c\uc774\uace0 \uc788\uc73c\uba70, \ub7ec\uc2dc\uc544 \ub4f1 \uc5ec\ub7ec \uad6d\uac00\ub294 \uce85\uce74\uc2a4 \uc218\uc7a5\uad6d\uc744 \ud14c\ub7ec \ub2e8\uccb4\ub85c \ubd84\ub958\ud558\uace0 \uc788\ub2e4.", "paragraph_sentence": "2010\ub144 \ubaa8\uc2a4\ud06c\ubc14 \uc9c0\ud558\ucca0 \ud14c\ub7ec\ub294 2007\ub144 \uc774\uce58\ucf00\ub9ac\uc57c \uccb4\uccb8 \uacf5\ud654\uad6d\uc744 \uacc4\uc2b9\ud55c \uccb4\uccb8\uc758 \ud14c\ub7ec \ub2e8\uccb4\uc778 \uce85\uce74\uc2a4 \uc5d0\ubbf8\ub808\uc774\ud2b8\uac00 \uc8fc\ub3c4\ud588\ub2e4. \uce85\uce74\uc2a4 \uc5d0\ubbf8\ub808\uc774\ud2b8\uc758 \ucc3d\uc124 \ubc30\uacbd\uc744 \uac70\uc2ac\ub7ec \uc62c\ub77c\uac00\uba74 \uccb4\uccb8\uc758 \ub3c5\ub9bd\uc744 \ub458\ub7ec\uc2fc \uc815\uce58 \uc138\ub825\uc758 \ubd84\uc5f4\uc774 \uc788\ub2e4. 1999\ub144 2\ucc28 \uccb4\uccb8\uc804\uc7c1\uc5d0\uc11c \ub7ec\uc2dc\uc544 \uc5f0\ubc29\uc774 \uc2b9\ub9ac\ud55c \uc9c1\ud6c4, \uccb4\uccb8\uc758 \uc815\uce58\uc138\ub825\uc740 \ub450 \ud30c\ubc8c\ub85c \ub098\ub258\uc5c8\ub2e4. \ub7ec\uc2dc\uc544 \uc5f0\ubc29\uc5d0 \uc794\uc874\ud558\uae38 \ud76c\ub9dd\ud558\ub294 \uc138\ub825\uc740 \ub7ec\uc2dc\uc544\uc758 \uc790\uce58\uacf5\ud654\uad6d\uc73c\ub85c\uc368 \uccb4\uccb8 \uacf5\ud654\uad6d\uc744 \uac74\uad6d, \uc790\uce58\uad8c\uc744 \uc778\uc815\ubc1b\uc558\uc73c\ub098, \ubd84\ub9ac\ub3c5\ub9bd\ud30c\ub294 \uccb4\uccb8\uc758 \uc644\uc804\ud55c \ub3c5\ub9bd\uc744 \ubaa9\ud45c\ub85c \ub7ec\uc2dc\uc544\ub97c \uc0c1\ub300\ub85c \uacc4\uc18d\uc801\uc73c\ub85c \ubb34\uc7a5\ud22c\uc7c1\uc744 \ubc8c\uc600\ub2e4. \uc804\uc7c1\uc774 \uae38\uc5b4\uc9d0\uc5d0 \ub530\ub77c \uc628\uac74\ud30c \uc815\uce58\uc778\uc774\uc5c8\ub358 \ub9c8\uc2a4\ud558\ub3c4\ud504 \ub4f1, \uc8fc\uc694 \uc9c0\ub3c4\uce35\uc744 \uc783\uc73c\uba70 \uc138\ub825 \uc720\uc9c0\uc5d0 \ud070 \ud0c0\uaca9\uc744 \uc785\uc5c8\ub2e4. \uc5ec\uae30\uc5d0 \uccb4\uccb8 \uac15\uacbd\ud30c\uac00 \ubc8c\uc778 \ubbfc\uac04\uc778 \ud14c\ub7ec \uc0ac\uac74\ub4e4\ub85c \uc774\uce58\ucf00\ub9ac\uc57c \uccb4\uccb8 \uc815\ubd80\ub294 \uc624\ud788\ub824 \uad6d\uc81c\uc801\uc73c\ub85c \uc774\ubbf8\uc9c0\uac00 \uc545\ud654\ub418\uba74\uc11c \ubd84\ub9ac\ub3c5\ub9bd\ud30c\ub294 \uc790\uc5f0\uc2a4\ub808 \uc57d\ud654\ub418\uc5c8\ub2e4. \uc774\ub7f0 \uc0c1\ud669\uc5d0\uc11c 2007\ub144 \uc774\uce58\ucf00\ub9ac\uc57c \uccb4\uccb8 \uacf5\ud654\uad6d\uc758 \ub300\ud1b5\ub839\uc774\uc5c8\ub358 \uc774\uce58\ucf00\ub9ac\uc57c \uccb4\uccb8 \uc815\ubd80\ub97c \uc5c6\uc560\uace0 \ub3c4\ucfe0 \uc6b0\ub9c8\ub85c\ud504\ub294 \uc790\uc2e0\uc744 \uc218\uc7a5\uc73c\ub85c \uc120\ucd9c, \uce85\uce74\uc2a4 \uc5d0\ubbf8\ub808\uc774\ud2b8\uc774\ub77c\ub294 \ub2e8\uccb4\ub97c \uc0c8\ub86d\uac8c \ucc3d\uc124\ud588\ub2e4. \ud604\uc7ac \uc774\ub4e4\uc740 \ubd81\uce85\uce74\uc2a4 \uc9c0\uc5ed\uc5d0 \uc774\uc2ac\ub78c \uad6d\uac00\ub97c \uc218\ub9bd\ud558\ub294 \uac83\uc744 \ubaa9\ud45c\ub85c \ub7ec\uc2dc\uc544\uc5d0 \ub300\ud55c \ud14c\ub7ec\ub97c \ubc8c\uc774\uace0 \uc788\uc73c\uba70, \ub7ec\uc2dc\uc544 \ub4f1 \uc5ec\ub7ec \uad6d\uac00\ub294 \uce85\uce74\uc2a4 \uc218\uc7a5\uad6d\uc744 \ud14c\ub7ec \ub2e8\uccb4\ub85c \ubd84\ub958\ud558\uace0 \uc788\ub2e4. 2002\ub144 \uc774\ud6c4 \ud604\uc7ac\uae4c\uc9c0 \uc774\ub4e4\uc774 \ubc8c\uc774\ub294 \ud14c\ub7ec\uc5d0 \uc758\ud574 5000\uba85 \uc774\uc0c1\uc758 \uc0ac\uc0c1\uc790\uac00 \uc18d\ucd9c\ud558\uace0 \uc788\ub2e4.", "paragraph_answer": "2010\ub144 \ubaa8\uc2a4\ud06c\ubc14 \uc9c0\ud558\ucca0 \ud14c\ub7ec\ub294 2007\ub144 \uc774\uce58\ucf00\ub9ac\uc57c \uccb4\uccb8 \uacf5\ud654\uad6d\uc744 \uacc4\uc2b9\ud55c \uccb4\uccb8\uc758 \ud14c\ub7ec \ub2e8\uccb4\uc778 \uce85\uce74\uc2a4 \uc5d0\ubbf8\ub808\uc774\ud2b8\uac00 \uc8fc\ub3c4\ud588\ub2e4. \uce85\uce74\uc2a4 \uc5d0\ubbf8\ub808\uc774\ud2b8\uc758 \ucc3d\uc124 \ubc30\uacbd\uc744 \uac70\uc2ac\ub7ec \uc62c\ub77c\uac00\uba74 \uccb4\uccb8\uc758 \ub3c5\ub9bd\uc744 \ub458\ub7ec\uc2fc \uc815\uce58 \uc138\ub825\uc758 \ubd84\uc5f4\uc774 \uc788\ub2e4. 1999\ub144 2\ucc28 \uccb4\uccb8\uc804\uc7c1\uc5d0\uc11c \ub7ec\uc2dc\uc544 \uc5f0\ubc29\uc774 \uc2b9\ub9ac\ud55c \uc9c1\ud6c4, \uccb4\uccb8\uc758 \uc815\uce58\uc138\ub825\uc740 \ub450 \ud30c\ubc8c\ub85c \ub098\ub258\uc5c8\ub2e4. \ub7ec\uc2dc\uc544 \uc5f0\ubc29\uc5d0 \uc794\uc874\ud558\uae38 \ud76c\ub9dd\ud558\ub294 \uc138\ub825\uc740 \ub7ec\uc2dc\uc544\uc758 \uc790\uce58\uacf5\ud654\uad6d\uc73c\ub85c\uc368 \uccb4\uccb8 \uacf5\ud654\uad6d\uc744 \uac74\uad6d, \uc790\uce58\uad8c\uc744 \uc778\uc815\ubc1b\uc558\uc73c\ub098, \ubd84\ub9ac\ub3c5\ub9bd\ud30c\ub294 \uccb4\uccb8\uc758 \uc644\uc804\ud55c \ub3c5\ub9bd\uc744 \ubaa9\ud45c\ub85c \ub7ec\uc2dc\uc544\ub97c \uc0c1\ub300\ub85c \uacc4\uc18d\uc801\uc73c\ub85c \ubb34\uc7a5\ud22c\uc7c1\uc744 \ubc8c\uc600\ub2e4. \uc804\uc7c1\uc774 \uae38\uc5b4\uc9d0\uc5d0 \ub530\ub77c \uc628\uac74\ud30c \uc815\uce58\uc778\uc774\uc5c8\ub358 \ub9c8\uc2a4\ud558\ub3c4\ud504 \ub4f1, \uc8fc\uc694 \uc9c0\ub3c4\uce35\uc744 \uc783\uc73c\uba70 \uc138\ub825 \uc720\uc9c0\uc5d0 \ud070 \ud0c0\uaca9\uc744 \uc785\uc5c8\ub2e4. \uc5ec\uae30\uc5d0 \uccb4\uccb8 \uac15\uacbd\ud30c\uac00 \ubc8c\uc778 \ubbfc\uac04\uc778 \ud14c\ub7ec \uc0ac\uac74\ub4e4\ub85c \uc774\uce58\ucf00\ub9ac\uc57c \uccb4\uccb8 \uc815\ubd80\ub294 \uc624\ud788\ub824 \uad6d\uc81c\uc801\uc73c\ub85c \uc774\ubbf8\uc9c0\uac00 \uc545\ud654\ub418\uba74\uc11c \ubd84\ub9ac\ub3c5\ub9bd\ud30c\ub294 \uc790\uc5f0\uc2a4\ub808 \uc57d\ud654\ub418\uc5c8\ub2e4. \uc774\ub7f0 \uc0c1\ud669\uc5d0\uc11c 2007\ub144 \uc774\uce58\ucf00\ub9ac\uc57c \uccb4\uccb8 \uacf5\ud654\uad6d\uc758 \ub300\ud1b5\ub839\uc774\uc5c8\ub358 \uc774\uce58\ucf00\ub9ac\uc57c \uccb4\uccb8 \uc815\ubd80\ub97c \uc5c6\uc560\uace0 \ub3c4\ucfe0 \uc6b0\ub9c8\ub85c\ud504\ub294 \uc790\uc2e0\uc744 \uc218\uc7a5\uc73c\ub85c \uc120\ucd9c, \uce85\uce74\uc2a4 \uc5d0\ubbf8\ub808\uc774\ud2b8\uc774\ub77c\ub294 \ub2e8\uccb4\ub97c \uc0c8\ub86d\uac8c \ucc3d\uc124\ud588\ub2e4. \ud604\uc7ac \uc774\ub4e4\uc740 \ubd81\uce85\uce74\uc2a4 \uc9c0\uc5ed\uc5d0 \uc774\uc2ac\ub78c \uad6d\uac00\ub97c \uc218\ub9bd\ud558\ub294 \uac83\uc744 \ubaa9\ud45c\ub85c \ub7ec\uc2dc\uc544\uc5d0 \ub300\ud55c \ud14c\ub7ec\ub97c \ubc8c\uc774\uace0 \uc788\uc73c\uba70, \ub7ec\uc2dc\uc544 \ub4f1 \uc5ec\ub7ec \uad6d\uac00\ub294 \uce85\uce74\uc2a4 \uc218\uc7a5\uad6d\uc744 \ud14c\ub7ec \ub2e8\uccb4\ub85c \ubd84\ub958\ud558\uace0 \uc788\ub2e4. 2002\ub144 \uc774\ud6c4 \ud604\uc7ac\uae4c\uc9c0 \uc774\ub4e4\uc774 \ubc8c\uc774\ub294 \ud14c\ub7ec\uc5d0 \uc758\ud574 5000\uba85 \uc774\uc0c1\uc758 \uc0ac\uc0c1\uc790\uac00 \uc18d\ucd9c\ud558\uace0 \uc788\ub2e4.", "sentence_answer": "\ud604\uc7ac \uc774\ub4e4\uc740 \ubd81\uce85\uce74\uc2a4 \uc9c0\uc5ed\uc5d0 \uc774\uc2ac\ub78c \uad6d\uac00\ub97c \uc218\ub9bd\ud558\ub294 \uac83\uc744 \ubaa9\ud45c\ub85c \ub7ec\uc2dc\uc544\uc5d0 \ub300\ud55c \ud14c\ub7ec\ub97c \ubc8c\uc774\uace0 \uc788\uc73c\uba70, \ub7ec\uc2dc\uc544 \ub4f1 \uc5ec\ub7ec \uad6d\uac00\ub294 \uce85\uce74\uc2a4 \uc218\uc7a5\uad6d\uc744 \ud14c\ub7ec \ub2e8\uccb4\ub85c \ubd84\ub958\ud558\uace0 \uc788\ub2e4.", "paragraph_id": "6470774-1-0", "paragraph_question": "question: \uc774\uc2ac\ub78c \uad6d\uac00 \uc218\ub9bd\uc744 \uc704\ud574 \ub7ec\uc2dc\uc544\uc5d0 \ub300\ud55c \ud14c\ub7ec\ub97c \ubc8c\uc774\ub294 \uce85\uce74\uc2a4 \uc5d0\ubbf8\ub808\uc774\ud2b8 \ub2e8\uccb4\ub294,\uc5b4\ub290\uc9c0\uc5ed\uc5d0 \uad6d\uac00 \uc218\ub9bd\uc744 \uc6d0\ud558\ub294\uac00?, context: 2010\ub144 \ubaa8\uc2a4\ud06c\ubc14 \uc9c0\ud558\ucca0 \ud14c\ub7ec\ub294 2007\ub144 \uc774\uce58\ucf00\ub9ac\uc57c \uccb4\uccb8 \uacf5\ud654\uad6d\uc744 \uacc4\uc2b9\ud55c \uccb4\uccb8\uc758 \ud14c\ub7ec \ub2e8\uccb4\uc778 \uce85\uce74\uc2a4 \uc5d0\ubbf8\ub808\uc774\ud2b8\uac00 \uc8fc\ub3c4\ud588\ub2e4. \uce85\uce74\uc2a4 \uc5d0\ubbf8\ub808\uc774\ud2b8\uc758 \ucc3d\uc124 \ubc30\uacbd\uc744 \uac70\uc2ac\ub7ec \uc62c\ub77c\uac00\uba74 \uccb4\uccb8\uc758 \ub3c5\ub9bd\uc744 \ub458\ub7ec\uc2fc \uc815\uce58 \uc138\ub825\uc758 \ubd84\uc5f4\uc774 \uc788\ub2e4. 1999\ub144 2\ucc28 \uccb4\uccb8\uc804\uc7c1\uc5d0\uc11c \ub7ec\uc2dc\uc544 \uc5f0\ubc29\uc774 \uc2b9\ub9ac\ud55c \uc9c1\ud6c4, \uccb4\uccb8\uc758 \uc815\uce58\uc138\ub825\uc740 \ub450 \ud30c\ubc8c\ub85c \ub098\ub258\uc5c8\ub2e4. \ub7ec\uc2dc\uc544 \uc5f0\ubc29\uc5d0 \uc794\uc874\ud558\uae38 \ud76c\ub9dd\ud558\ub294 \uc138\ub825\uc740 \ub7ec\uc2dc\uc544\uc758 \uc790\uce58\uacf5\ud654\uad6d\uc73c\ub85c\uc368 \uccb4\uccb8 \uacf5\ud654\uad6d\uc744 \uac74\uad6d, \uc790\uce58\uad8c\uc744 \uc778\uc815\ubc1b\uc558\uc73c\ub098, \ubd84\ub9ac\ub3c5\ub9bd\ud30c\ub294 \uccb4\uccb8\uc758 \uc644\uc804\ud55c \ub3c5\ub9bd\uc744 \ubaa9\ud45c\ub85c \ub7ec\uc2dc\uc544\ub97c \uc0c1\ub300\ub85c \uacc4\uc18d\uc801\uc73c\ub85c \ubb34\uc7a5\ud22c\uc7c1\uc744 \ubc8c\uc600\ub2e4. \uc804\uc7c1\uc774 \uae38\uc5b4\uc9d0\uc5d0 \ub530\ub77c \uc628\uac74\ud30c \uc815\uce58\uc778\uc774\uc5c8\ub358 \ub9c8\uc2a4\ud558\ub3c4\ud504 \ub4f1, \uc8fc\uc694 \uc9c0\ub3c4\uce35\uc744 \uc783\uc73c\uba70 \uc138\ub825 \uc720\uc9c0\uc5d0 \ud070 \ud0c0\uaca9\uc744 \uc785\uc5c8\ub2e4. \uc5ec\uae30\uc5d0 \uccb4\uccb8 \uac15\uacbd\ud30c\uac00 \ubc8c\uc778 \ubbfc\uac04\uc778 \ud14c\ub7ec \uc0ac\uac74\ub4e4\ub85c \uc774\uce58\ucf00\ub9ac\uc57c \uccb4\uccb8 \uc815\ubd80\ub294 \uc624\ud788\ub824 \uad6d\uc81c\uc801\uc73c\ub85c \uc774\ubbf8\uc9c0\uac00 \uc545\ud654\ub418\uba74\uc11c \ubd84\ub9ac\ub3c5\ub9bd\ud30c\ub294 \uc790\uc5f0\uc2a4\ub808 \uc57d\ud654\ub418\uc5c8\ub2e4. \uc774\ub7f0 \uc0c1\ud669\uc5d0\uc11c 2007\ub144 \uc774\uce58\ucf00\ub9ac\uc57c \uccb4\uccb8 \uacf5\ud654\uad6d\uc758 \ub300\ud1b5\ub839\uc774\uc5c8\ub358 \uc774\uce58\ucf00\ub9ac\uc57c \uccb4\uccb8 \uc815\ubd80\ub97c \uc5c6\uc560\uace0 \ub3c4\ucfe0 \uc6b0\ub9c8\ub85c\ud504\ub294 \uc790\uc2e0\uc744 \uc218\uc7a5\uc73c\ub85c \uc120\ucd9c, \uce85\uce74\uc2a4 \uc5d0\ubbf8\ub808\uc774\ud2b8\uc774\ub77c\ub294 \ub2e8\uccb4\ub97c \uc0c8\ub86d\uac8c \ucc3d\uc124\ud588\ub2e4. \ud604\uc7ac \uc774\ub4e4\uc740 \ubd81\uce85\uce74\uc2a4 \uc9c0\uc5ed\uc5d0 \uc774\uc2ac\ub78c \uad6d\uac00\ub97c \uc218\ub9bd\ud558\ub294 \uac83\uc744 \ubaa9\ud45c\ub85c \ub7ec\uc2dc\uc544\uc5d0 \ub300\ud55c \ud14c\ub7ec\ub97c \ubc8c\uc774\uace0 \uc788\uc73c\uba70, \ub7ec\uc2dc\uc544 \ub4f1 \uc5ec\ub7ec \uad6d\uac00\ub294 \uce85\uce74\uc2a4 \uc218\uc7a5\uad6d\uc744 \ud14c\ub7ec \ub2e8\uccb4\ub85c \ubd84\ub958\ud558\uace0 \uc788\ub2e4. 2002\ub144 \uc774\ud6c4 \ud604\uc7ac\uae4c\uc9c0 \uc774\ub4e4\uc774 \ubc8c\uc774\ub294 \ud14c\ub7ec\uc5d0 \uc758\ud574 5000\uba85 \uc774\uc0c1\uc758 \uc0ac\uc0c1\uc790\uac00 \uc18d\ucd9c\ud558\uace0 \uc788\ub2e4."} {"question": "\uc138\uc778\ud2b8\ub9c8\ud2f4\ube48\ud2b8\ub9ac \uad50\ud68c\uc5d0\uc11c \uc0b4\ud574\ub41c \ud50c\ub791\ub4dc\ub974\uc778\uc758 \uc22b\uc790\ub294?", "paragraph": "6\uc6d4 14\uc77c \uc544\uce68, \uad70\uc911\uc740 \ud15c\uc2a4 \uac15\uc744 \ub530\ub77c \uc6c0\uc9c1\uc774\uba74\uc11c \uc6e8\uc2a4\ud2b8\ubbfc\uc2a4\ud130 \uc8fc\uc704\uc758 \uad00\ub9ac\ub4e4\uc758 \uc9d1\uc744 \ubd88\ud0dc\uc6b0\uace0 \uc6e8\uc2a4\ud2b8\ubbfc\uc2a4\ud130 \uac10\uc625\uc744 \uc5f4\uc5c8\ub2e4. \uadf8\ub7ec\uace0\ub294 \ub7f0\ub358 \uc911\uc2ec\ubd80\ub85c \ub3cc\uc544\uac00\uc11c \ub354 \ub9ce\uc740 \uac74\ubb3c\uc5d0 \ubd88\uc744 \uc9c0\ub974\uace0 \ub274\uac8c\uc774\ud2b8 \uac10\uc625\uc744 \uae09\uc2b5\ud588\ub2e4. \ud50c\ub791\ub4dc\ub974\uc778 \uc0ac\ub0e5\ub3c4 \uacc4\uc18d\ub418\uc5c8\uace0, \uc655\uc2e4\uace0\ubb38\uad00 \ub9ac\ucc98\ub4dc \ub77c\uc774\uc5b8\uc2a4\ub97c \ube44\ub86f\ud574 \ud50c\ub791\ub4dc\ub974 \uc5b5\uc591\uc73c\ub85c \ub9d0\ud558\ub294 \uc790\ub4e4\uc774 \uc0b4\ud574\ub2f9\ud588\ub2e4. \uc2dc\ud2f0\uc758 \ud55c \uad6c(\u5340, ward)\uc5d0\uc11c\ub294 \uc0b4\ud574\ub2f9\ud55c \ud50c\ub791\ub4dc\ub974\uc778 40\uba85\uc758 \uc2dc\uccb4\uac00 \uae38\uac70\ub9ac\uc5d0 \ub354\ubbf8\ub85c \uc313\uc600\uc73c\uba70, \ud50c\ub791\ub4dc\ub974\uc778\ub4e4\uc774 \uc8fc\ub85c \ucc38\uc11d\ud558\ub358 \uc138\uc778\ud2b8\ub9c8\ud2f4\ube48\ud2b8\ub9ac \uad50\ud68c\uc5d0\uc11c\ub294 35\uba85\uc774 \uc0b4\ud574\ub2f9\ud588\ub2e4. \uc0ac\ud559\uc790 \ub85c\ub4dc\ub2c8 \ud790\ud134(Rodney Hilton)\uc740 \uc774 \uacf5\uaca9\uc774 \ud50c\ub791\ub4dc\ub974\uc778 \ubc29\uc9c1\uacf5\ub4e4\uacfc \uc0c1\uc5c5\uc801 \uacbd\uc7c1\uad00\uacc4\uc5d0 \uc788\uc5c8\ub358 \ub7f0\ub358 \ubc29\uc9c1\uacf5 \uc870\ud569\uc758 \ud611\uc870 \ud558\uc5d0 \uc774\ub8e8\uc5b4\uc9c4 \uac83\uc774\ub77c\uace0 \uc8fc\uc7a5\ud55c\ub2e4.", "answer": "35\uba85", "sentence": "\ud50c\ub791\ub4dc\ub974\uc778\ub4e4\uc774 \uc8fc\ub85c \ucc38\uc11d\ud558\ub358 \uc138\uc778\ud2b8\ub9c8\ud2f4\ube48\ud2b8\ub9ac \uad50\ud68c\uc5d0\uc11c\ub294 35\uba85 \uc774 \uc0b4\ud574\ub2f9\ud588\ub2e4.", "paragraph_sentence": "6\uc6d4 14\uc77c \uc544\uce68, \uad70\uc911\uc740 \ud15c\uc2a4 \uac15\uc744 \ub530\ub77c \uc6c0\uc9c1\uc774\uba74\uc11c \uc6e8\uc2a4\ud2b8\ubbfc\uc2a4\ud130 \uc8fc\uc704\uc758 \uad00\ub9ac\ub4e4\uc758 \uc9d1\uc744 \ubd88\ud0dc\uc6b0\uace0 \uc6e8\uc2a4\ud2b8\ubbfc\uc2a4\ud130 \uac10\uc625\uc744 \uc5f4\uc5c8\ub2e4. \uadf8\ub7ec\uace0\ub294 \ub7f0\ub358 \uc911\uc2ec\ubd80\ub85c \ub3cc\uc544\uac00\uc11c \ub354 \ub9ce\uc740 \uac74\ubb3c\uc5d0 \ubd88\uc744 \uc9c0\ub974\uace0 \ub274\uac8c\uc774\ud2b8 \uac10\uc625\uc744 \uae09\uc2b5\ud588\ub2e4. \ud50c\ub791\ub4dc\ub974\uc778 \uc0ac\ub0e5\ub3c4 \uacc4\uc18d\ub418\uc5c8\uace0, \uc655\uc2e4\uace0\ubb38\uad00 \ub9ac\ucc98\ub4dc \ub77c\uc774\uc5b8\uc2a4\ub97c \ube44\ub86f\ud574 \ud50c\ub791\ub4dc\ub974 \uc5b5\uc591\uc73c\ub85c \ub9d0\ud558\ub294 \uc790\ub4e4\uc774 \uc0b4\ud574\ub2f9\ud588\ub2e4. \uc2dc\ud2f0\uc758 \ud55c \uad6c(\u5340, ward)\uc5d0\uc11c\ub294 \uc0b4\ud574\ub2f9\ud55c \ud50c\ub791\ub4dc\ub974\uc778 40\uba85\uc758 \uc2dc\uccb4\uac00 \uae38\uac70\ub9ac\uc5d0 \ub354\ubbf8\ub85c \uc313\uc600\uc73c\uba70, \ud50c\ub791\ub4dc\ub974\uc778\ub4e4\uc774 \uc8fc\ub85c \ucc38\uc11d\ud558\ub358 \uc138\uc778\ud2b8\ub9c8\ud2f4\ube48\ud2b8\ub9ac \uad50\ud68c\uc5d0\uc11c\ub294 35\uba85 \uc774 \uc0b4\ud574\ub2f9\ud588\ub2e4. \uc0ac\ud559\uc790 \ub85c\ub4dc\ub2c8 \ud790\ud134(Rodney Hilton)\uc740 \uc774 \uacf5\uaca9\uc774 \ud50c\ub791\ub4dc\ub974\uc778 \ubc29\uc9c1\uacf5\ub4e4\uacfc \uc0c1\uc5c5\uc801 \uacbd\uc7c1\uad00\uacc4\uc5d0 \uc788\uc5c8\ub358 \ub7f0\ub358 \ubc29\uc9c1\uacf5 \uc870\ud569\uc758 \ud611\uc870 \ud558\uc5d0 \uc774\ub8e8\uc5b4\uc9c4 \uac83\uc774\ub77c\uace0 \uc8fc\uc7a5\ud55c\ub2e4.", "paragraph_answer": "6\uc6d4 14\uc77c \uc544\uce68, \uad70\uc911\uc740 \ud15c\uc2a4 \uac15\uc744 \ub530\ub77c \uc6c0\uc9c1\uc774\uba74\uc11c \uc6e8\uc2a4\ud2b8\ubbfc\uc2a4\ud130 \uc8fc\uc704\uc758 \uad00\ub9ac\ub4e4\uc758 \uc9d1\uc744 \ubd88\ud0dc\uc6b0\uace0 \uc6e8\uc2a4\ud2b8\ubbfc\uc2a4\ud130 \uac10\uc625\uc744 \uc5f4\uc5c8\ub2e4. \uadf8\ub7ec\uace0\ub294 \ub7f0\ub358 \uc911\uc2ec\ubd80\ub85c \ub3cc\uc544\uac00\uc11c \ub354 \ub9ce\uc740 \uac74\ubb3c\uc5d0 \ubd88\uc744 \uc9c0\ub974\uace0 \ub274\uac8c\uc774\ud2b8 \uac10\uc625\uc744 \uae09\uc2b5\ud588\ub2e4. \ud50c\ub791\ub4dc\ub974\uc778 \uc0ac\ub0e5\ub3c4 \uacc4\uc18d\ub418\uc5c8\uace0, \uc655\uc2e4\uace0\ubb38\uad00 \ub9ac\ucc98\ub4dc \ub77c\uc774\uc5b8\uc2a4\ub97c \ube44\ub86f\ud574 \ud50c\ub791\ub4dc\ub974 \uc5b5\uc591\uc73c\ub85c \ub9d0\ud558\ub294 \uc790\ub4e4\uc774 \uc0b4\ud574\ub2f9\ud588\ub2e4. \uc2dc\ud2f0\uc758 \ud55c \uad6c(\u5340, ward)\uc5d0\uc11c\ub294 \uc0b4\ud574\ub2f9\ud55c \ud50c\ub791\ub4dc\ub974\uc778 40\uba85\uc758 \uc2dc\uccb4\uac00 \uae38\uac70\ub9ac\uc5d0 \ub354\ubbf8\ub85c \uc313\uc600\uc73c\uba70, \ud50c\ub791\ub4dc\ub974\uc778\ub4e4\uc774 \uc8fc\ub85c \ucc38\uc11d\ud558\ub358 \uc138\uc778\ud2b8\ub9c8\ud2f4\ube48\ud2b8\ub9ac \uad50\ud68c\uc5d0\uc11c\ub294 35\uba85 \uc774 \uc0b4\ud574\ub2f9\ud588\ub2e4. \uc0ac\ud559\uc790 \ub85c\ub4dc\ub2c8 \ud790\ud134(Rodney Hilton)\uc740 \uc774 \uacf5\uaca9\uc774 \ud50c\ub791\ub4dc\ub974\uc778 \ubc29\uc9c1\uacf5\ub4e4\uacfc \uc0c1\uc5c5\uc801 \uacbd\uc7c1\uad00\uacc4\uc5d0 \uc788\uc5c8\ub358 \ub7f0\ub358 \ubc29\uc9c1\uacf5 \uc870\ud569\uc758 \ud611\uc870 \ud558\uc5d0 \uc774\ub8e8\uc5b4\uc9c4 \uac83\uc774\ub77c\uace0 \uc8fc\uc7a5\ud55c\ub2e4.", "sentence_answer": "\ud50c\ub791\ub4dc\ub974\uc778\ub4e4\uc774 \uc8fc\ub85c \ucc38\uc11d\ud558\ub358 \uc138\uc778\ud2b8\ub9c8\ud2f4\ube48\ud2b8\ub9ac \uad50\ud68c\uc5d0\uc11c\ub294 35\uba85 \uc774 \uc0b4\ud574\ub2f9\ud588\ub2e4.", "paragraph_id": "5844060-20-1", "paragraph_question": "question: \uc138\uc778\ud2b8\ub9c8\ud2f4\ube48\ud2b8\ub9ac \uad50\ud68c\uc5d0\uc11c \uc0b4\ud574\ub41c \ud50c\ub791\ub4dc\ub974\uc778\uc758 \uc22b\uc790\ub294?, context: 6\uc6d4 14\uc77c \uc544\uce68, \uad70\uc911\uc740 \ud15c\uc2a4 \uac15\uc744 \ub530\ub77c \uc6c0\uc9c1\uc774\uba74\uc11c \uc6e8\uc2a4\ud2b8\ubbfc\uc2a4\ud130 \uc8fc\uc704\uc758 \uad00\ub9ac\ub4e4\uc758 \uc9d1\uc744 \ubd88\ud0dc\uc6b0\uace0 \uc6e8\uc2a4\ud2b8\ubbfc\uc2a4\ud130 \uac10\uc625\uc744 \uc5f4\uc5c8\ub2e4. \uadf8\ub7ec\uace0\ub294 \ub7f0\ub358 \uc911\uc2ec\ubd80\ub85c \ub3cc\uc544\uac00\uc11c \ub354 \ub9ce\uc740 \uac74\ubb3c\uc5d0 \ubd88\uc744 \uc9c0\ub974\uace0 \ub274\uac8c\uc774\ud2b8 \uac10\uc625\uc744 \uae09\uc2b5\ud588\ub2e4. \ud50c\ub791\ub4dc\ub974\uc778 \uc0ac\ub0e5\ub3c4 \uacc4\uc18d\ub418\uc5c8\uace0, \uc655\uc2e4\uace0\ubb38\uad00 \ub9ac\ucc98\ub4dc \ub77c\uc774\uc5b8\uc2a4\ub97c \ube44\ub86f\ud574 \ud50c\ub791\ub4dc\ub974 \uc5b5\uc591\uc73c\ub85c \ub9d0\ud558\ub294 \uc790\ub4e4\uc774 \uc0b4\ud574\ub2f9\ud588\ub2e4. \uc2dc\ud2f0\uc758 \ud55c \uad6c(\u5340, ward)\uc5d0\uc11c\ub294 \uc0b4\ud574\ub2f9\ud55c \ud50c\ub791\ub4dc\ub974\uc778 40\uba85\uc758 \uc2dc\uccb4\uac00 \uae38\uac70\ub9ac\uc5d0 \ub354\ubbf8\ub85c \uc313\uc600\uc73c\uba70, \ud50c\ub791\ub4dc\ub974\uc778\ub4e4\uc774 \uc8fc\ub85c \ucc38\uc11d\ud558\ub358 \uc138\uc778\ud2b8\ub9c8\ud2f4\ube48\ud2b8\ub9ac \uad50\ud68c\uc5d0\uc11c\ub294 35\uba85\uc774 \uc0b4\ud574\ub2f9\ud588\ub2e4. \uc0ac\ud559\uc790 \ub85c\ub4dc\ub2c8 \ud790\ud134(Rodney Hilton)\uc740 \uc774 \uacf5\uaca9\uc774 \ud50c\ub791\ub4dc\ub974\uc778 \ubc29\uc9c1\uacf5\ub4e4\uacfc \uc0c1\uc5c5\uc801 \uacbd\uc7c1\uad00\uacc4\uc5d0 \uc788\uc5c8\ub358 \ub7f0\ub358 \ubc29\uc9c1\uacf5 \uc870\ud569\uc758 \ud611\uc870 \ud558\uc5d0 \uc774\ub8e8\uc5b4\uc9c4 \uac83\uc774\ub77c\uace0 \uc8fc\uc7a5\ud55c\ub2e4."} {"question": "\uc601\ud654 \uac10\ub3c5 \ub098\ud64d\uc9c4\uc758 \uc7a5\ud3b8\uc601\ud654 \ub370\ubdd4\uc791\uc740?", "paragraph": "2003\ub144\uc5d0 \ub2e8\ud3b8 \uc601\ud654 \u300a5 Minutes\u300b\ub97c \ub9cc\ub4e4\uc5c8\uc73c\uba70, 2005\ub144\uc5d0 \ub2e8\ud3b8 \u300a\uc644\ubcbd\ud55c \ub3c4\ubbf8\uc694\ub9ac\u300b\ub85c \uc81c4\ud68c \ubbf8\uc7dd\uc13c \ub2e8\ud3b8 \uc601\ud654\uc81c \ucd5c\uc6b0\uc218 \uc791\ud488\uc0c1\uacfc \uc81c6\ud68c \ub300\ud55c\ubbfc\uad6d \uc601\uc0c1\ub300\uc804 \uc7a5\ub824\uc0c1\uc744 \uc218\uc0c1\ud558\uace0, \uc81c25\ud68c \ud558\uc640\uc774 \uad6d\uc81c\uc601\ud654\uc81c \ucd08\uccad\uc791\uacfc \ub808\uc2a4\ud398\uc2a4\ud2b8 2005\ub144 \ud398\uc2a4\ud2f0\ubc8c \ucd08\uc774\uc2a4\ub85c \uc120\uc815\ub418\uc5c8\ub2e4. 2007\ub144\uc5d0 \ub2e8\ud3b8 \u300a\ud55c(\u6c57)\u300b\uc73c\ub85c \uc81c44\ud68c \ub300\uc885\uc0c1 \ub2e8\ud3b8 \uc601\ud654 \uac10\ub3c5\uc0c1, \uc81c11\ud68c \ubd80\ucc9c\uad6d\uc81c\ud310\ud0c0\uc2a4\ud2f1\uc601\ud654\uc81c \ub2e8\ud3b8 \ubd80\ubb38 \uc2ec\uc0ac\uc704\uc6d0\uc0c1, \uc81c8\ud68c \ub300\ud55c\ubbfc\uad6d \uc601\uc0c1\ub300\uc804 \uc6b0\uc218\uc0c1\uc744 \uc218\uc0c1\ud55c \uacbd\ub825\uc774 \uc788\ub2e4. 2008\ub144\uc5d0 \uc7a5\ud3b8 \uc601\ud654 \ub370\ubdd4\uc791\uc778 \u300a\ucd94\uaca9\uc790\u300b\uac00 \uac1c\ubd09\ud558\uc600\uc73c\uba70,\u300a\ucd94\uaca9\uc790\u300b\ub294 2008\ub144 \ubc31\uc0c1\uc608\uc220\ub300\uc0c1\uc5d0\uc11c \uc601\ud654\ub300\uc0c1 \ubc0f \uc601\ud654\uc2e0\uc778\uac10\ub3c5\uc0c1\uc744 \uc218\uc0c1, 2008\ub144 \ub300\ud55c\ubbfc\uad6d\uc601\ud654\ub300\uc0c1\uc5d0\uc11c \ucd5c\uc6b0\uc218\uc791\ud488\uc0c1, \uac10\ub3c5\uc0c1, \uac01\ubcf8/\uac01\uc0c9\uc0c1\uc744 \ubc1b\uc558\ub2e4. \uc774\ud6c4 2010\ub144 \uc601\ud654 \u300a\ucd94\uaca9\uc790\u300b\uc758 \ub450 \uc8fc\uc5ed\uc778 \uae40\uc724\uc11d, \ud558\uc815\uc6b0\uc640 \ub2e4\uc2dc \uc758\uae30\ud22c\ud569\ud558\uc5ec \uc601\ud654 \u300a\ud669\ud574\u300b\ub97c \uc120\ubcf4\uc5ec \ub610\ud55c \ub9ce\uc740 \ud654\uc81c\ub97c \ub0b3\uc558\ub2e4. \uadf8\ub7ec\ub098 2010\ub144 \uc774\ud6c4 \uc624\ub79c \uc7a5\uace0\uc5d0 \ub4e4\uc5b4\uac00\uba70 \uc791\ud488\ud65c\ub3d9\uc744 \ud558\uc9c0 \uc54a\uc558\uace0 6\ub144 \ub9cc\uc778 2016\ub144 \uad6d\ub0b4\uc5d0\uc120 \uac70\uc758 \ucd5c\ucd08\ub85c \uc2dc\ub3c4\ub41c \uc624\uceec\ud2b8\ud638\ub7ec \uc7a5\ub974\uc758 \uc601\ud654 \u300a\uace1\uc131\u300b\uc73c\ub85c \ub610 \ud55c \ubc88 \uadf8 \uc9c4\uac00\ub97c \ubc1c\ud718\ud558\uba70 \uba85\uac10\ub3c5\uc73c\ub85c\uc11c\uc758 \uc785\uc9c0\ub97c \ub2e4\uc84c\ub2e4. \uc774 \uc601\ud654\ub85c \uc81c 16\ud68c \ub514\ub809\ud130\uc2a4 \ucef7 \uc2dc\uc0c1\uc2dd\uc5d0\uc11c \uc62c\ud574\uc758 \uac10\ub3c5\uc0c1\uc744 \uc218\uc0c1\ud588\uace0 \uad6d\ub0b4 \uac00\uc7a5 \uad8c\uc704\uc788\ub294 \uc601\ud654\uc81c\ub85c \uaf3d\ud788\ub294 \uc81c 37\ud68c \uccad\ub8e1\uc601\ud654\uc0c1 \uc2dc\uc0c1\uc2dd\uc5d0\uc11c \uac10\ub3c5\uc0c1\uc744 \uc218\uc0c1\ud588\ub2e4.", "answer": "\ucd94\uaca9\uc790", "sentence": "2008\ub144\uc5d0 \uc7a5\ud3b8 \uc601\ud654 \ub370\ubdd4\uc791\uc778 \u300a \ucd94\uaca9\uc790 \u300b\uac00 \uac1c\ubd09\ud558\uc600\uc73c\uba70,\u300a\ucd94\uaca9\uc790\u300b\ub294 2008\ub144 \ubc31\uc0c1\uc608\uc220\ub300\uc0c1\uc5d0\uc11c \uc601\ud654\ub300\uc0c1 \ubc0f \uc601\ud654\uc2e0\uc778\uac10\ub3c5\uc0c1\uc744 \uc218\uc0c1, 2008\ub144 \ub300\ud55c\ubbfc\uad6d\uc601\ud654\ub300\uc0c1\uc5d0\uc11c \ucd5c\uc6b0\uc218\uc791\ud488\uc0c1, \uac10\ub3c5\uc0c1, \uac01\ubcf8/\uac01\uc0c9\uc0c1\uc744 \ubc1b\uc558\ub2e4.", "paragraph_sentence": "2003\ub144\uc5d0 \ub2e8\ud3b8 \uc601\ud654 \u300a5 Minutes\u300b\ub97c \ub9cc\ub4e4\uc5c8\uc73c\uba70, 2005\ub144\uc5d0 \ub2e8\ud3b8 \u300a\uc644\ubcbd\ud55c \ub3c4\ubbf8\uc694\ub9ac\u300b\ub85c \uc81c4\ud68c \ubbf8\uc7dd\uc13c \ub2e8\ud3b8 \uc601\ud654\uc81c \ucd5c\uc6b0\uc218 \uc791\ud488\uc0c1\uacfc \uc81c6\ud68c \ub300\ud55c\ubbfc\uad6d \uc601\uc0c1\ub300\uc804 \uc7a5\ub824\uc0c1\uc744 \uc218\uc0c1\ud558\uace0, \uc81c25\ud68c \ud558\uc640\uc774 \uad6d\uc81c\uc601\ud654\uc81c \ucd08\uccad\uc791\uacfc \ub808\uc2a4\ud398\uc2a4\ud2b8 2005\ub144 \ud398\uc2a4\ud2f0\ubc8c \ucd08\uc774\uc2a4\ub85c \uc120\uc815\ub418\uc5c8\ub2e4. 2007\ub144\uc5d0 \ub2e8\ud3b8 \u300a\ud55c(\u6c57)\u300b\uc73c\ub85c \uc81c44\ud68c \ub300\uc885\uc0c1 \ub2e8\ud3b8 \uc601\ud654 \uac10\ub3c5\uc0c1, \uc81c11\ud68c \ubd80\ucc9c\uad6d\uc81c\ud310\ud0c0\uc2a4\ud2f1\uc601\ud654\uc81c \ub2e8\ud3b8 \ubd80\ubb38 \uc2ec\uc0ac\uc704\uc6d0\uc0c1, \uc81c8\ud68c \ub300\ud55c\ubbfc\uad6d \uc601\uc0c1\ub300\uc804 \uc6b0\uc218\uc0c1\uc744 \uc218\uc0c1\ud55c \uacbd\ub825\uc774 \uc788\ub2e4. 2008\ub144\uc5d0 \uc7a5\ud3b8 \uc601\ud654 \ub370\ubdd4\uc791\uc778 \u300a \ucd94\uaca9\uc790 \u300b\uac00 \uac1c\ubd09\ud558\uc600\uc73c\uba70,\u300a\ucd94\uaca9\uc790\u300b\ub294 2008\ub144 \ubc31\uc0c1\uc608\uc220\ub300\uc0c1\uc5d0\uc11c \uc601\ud654\ub300\uc0c1 \ubc0f \uc601\ud654\uc2e0\uc778\uac10\ub3c5\uc0c1\uc744 \uc218\uc0c1, 2008\ub144 \ub300\ud55c\ubbfc\uad6d\uc601\ud654\ub300\uc0c1\uc5d0\uc11c \ucd5c\uc6b0\uc218\uc791\ud488\uc0c1, \uac10\ub3c5\uc0c1, \uac01\ubcf8/\uac01\uc0c9\uc0c1\uc744 \ubc1b\uc558\ub2e4. \uc774\ud6c4 2010\ub144 \uc601\ud654 \u300a\ucd94\uaca9\uc790\u300b\uc758 \ub450 \uc8fc\uc5ed\uc778 \uae40\uc724\uc11d, \ud558\uc815\uc6b0\uc640 \ub2e4\uc2dc \uc758\uae30\ud22c\ud569\ud558\uc5ec \uc601\ud654 \u300a\ud669\ud574\u300b\ub97c \uc120\ubcf4\uc5ec \ub610\ud55c \ub9ce\uc740 \ud654\uc81c\ub97c \ub0b3\uc558\ub2e4. \uadf8\ub7ec\ub098 2010\ub144 \uc774\ud6c4 \uc624\ub79c \uc7a5\uace0\uc5d0 \ub4e4\uc5b4\uac00\uba70 \uc791\ud488\ud65c\ub3d9\uc744 \ud558\uc9c0 \uc54a\uc558\uace0 6\ub144 \ub9cc\uc778 2016\ub144 \uad6d\ub0b4\uc5d0\uc120 \uac70\uc758 \ucd5c\ucd08\ub85c \uc2dc\ub3c4\ub41c \uc624\uceec\ud2b8\ud638\ub7ec \uc7a5\ub974\uc758 \uc601\ud654 \u300a\uace1\uc131\u300b\uc73c\ub85c \ub610 \ud55c \ubc88 \uadf8 \uc9c4\uac00\ub97c \ubc1c\ud718\ud558\uba70 \uba85\uac10\ub3c5\uc73c\ub85c\uc11c\uc758 \uc785\uc9c0\ub97c \ub2e4\uc84c\ub2e4. \uc774 \uc601\ud654\ub85c \uc81c 16\ud68c \ub514\ub809\ud130\uc2a4 \ucef7 \uc2dc\uc0c1\uc2dd\uc5d0\uc11c \uc62c\ud574\uc758 \uac10\ub3c5\uc0c1\uc744 \uc218\uc0c1\ud588\uace0 \uad6d\ub0b4 \uac00\uc7a5 \uad8c\uc704\uc788\ub294 \uc601\ud654\uc81c\ub85c \uaf3d\ud788\ub294 \uc81c 37\ud68c \uccad\ub8e1\uc601\ud654\uc0c1 \uc2dc\uc0c1\uc2dd\uc5d0\uc11c \uac10\ub3c5\uc0c1\uc744 \uc218\uc0c1\ud588\ub2e4.", "paragraph_answer": "2003\ub144\uc5d0 \ub2e8\ud3b8 \uc601\ud654 \u300a5 Minutes\u300b\ub97c \ub9cc\ub4e4\uc5c8\uc73c\uba70, 2005\ub144\uc5d0 \ub2e8\ud3b8 \u300a\uc644\ubcbd\ud55c \ub3c4\ubbf8\uc694\ub9ac\u300b\ub85c \uc81c4\ud68c \ubbf8\uc7dd\uc13c \ub2e8\ud3b8 \uc601\ud654\uc81c \ucd5c\uc6b0\uc218 \uc791\ud488\uc0c1\uacfc \uc81c6\ud68c \ub300\ud55c\ubbfc\uad6d \uc601\uc0c1\ub300\uc804 \uc7a5\ub824\uc0c1\uc744 \uc218\uc0c1\ud558\uace0, \uc81c25\ud68c \ud558\uc640\uc774 \uad6d\uc81c\uc601\ud654\uc81c \ucd08\uccad\uc791\uacfc \ub808\uc2a4\ud398\uc2a4\ud2b8 2005\ub144 \ud398\uc2a4\ud2f0\ubc8c \ucd08\uc774\uc2a4\ub85c \uc120\uc815\ub418\uc5c8\ub2e4. 2007\ub144\uc5d0 \ub2e8\ud3b8 \u300a\ud55c(\u6c57)\u300b\uc73c\ub85c \uc81c44\ud68c \ub300\uc885\uc0c1 \ub2e8\ud3b8 \uc601\ud654 \uac10\ub3c5\uc0c1, \uc81c11\ud68c \ubd80\ucc9c\uad6d\uc81c\ud310\ud0c0\uc2a4\ud2f1\uc601\ud654\uc81c \ub2e8\ud3b8 \ubd80\ubb38 \uc2ec\uc0ac\uc704\uc6d0\uc0c1, \uc81c8\ud68c \ub300\ud55c\ubbfc\uad6d \uc601\uc0c1\ub300\uc804 \uc6b0\uc218\uc0c1\uc744 \uc218\uc0c1\ud55c \uacbd\ub825\uc774 \uc788\ub2e4. 2008\ub144\uc5d0 \uc7a5\ud3b8 \uc601\ud654 \ub370\ubdd4\uc791\uc778 \u300a \ucd94\uaca9\uc790 \u300b\uac00 \uac1c\ubd09\ud558\uc600\uc73c\uba70,\u300a\ucd94\uaca9\uc790\u300b\ub294 2008\ub144 \ubc31\uc0c1\uc608\uc220\ub300\uc0c1\uc5d0\uc11c \uc601\ud654\ub300\uc0c1 \ubc0f \uc601\ud654\uc2e0\uc778\uac10\ub3c5\uc0c1\uc744 \uc218\uc0c1, 2008\ub144 \ub300\ud55c\ubbfc\uad6d\uc601\ud654\ub300\uc0c1\uc5d0\uc11c \ucd5c\uc6b0\uc218\uc791\ud488\uc0c1, \uac10\ub3c5\uc0c1, \uac01\ubcf8/\uac01\uc0c9\uc0c1\uc744 \ubc1b\uc558\ub2e4. \uc774\ud6c4 2010\ub144 \uc601\ud654 \u300a\ucd94\uaca9\uc790\u300b\uc758 \ub450 \uc8fc\uc5ed\uc778 \uae40\uc724\uc11d, \ud558\uc815\uc6b0\uc640 \ub2e4\uc2dc \uc758\uae30\ud22c\ud569\ud558\uc5ec \uc601\ud654 \u300a\ud669\ud574\u300b\ub97c \uc120\ubcf4\uc5ec \ub610\ud55c \ub9ce\uc740 \ud654\uc81c\ub97c \ub0b3\uc558\ub2e4. \uadf8\ub7ec\ub098 2010\ub144 \uc774\ud6c4 \uc624\ub79c \uc7a5\uace0\uc5d0 \ub4e4\uc5b4\uac00\uba70 \uc791\ud488\ud65c\ub3d9\uc744 \ud558\uc9c0 \uc54a\uc558\uace0 6\ub144 \ub9cc\uc778 2016\ub144 \uad6d\ub0b4\uc5d0\uc120 \uac70\uc758 \ucd5c\ucd08\ub85c \uc2dc\ub3c4\ub41c \uc624\uceec\ud2b8\ud638\ub7ec \uc7a5\ub974\uc758 \uc601\ud654 \u300a\uace1\uc131\u300b\uc73c\ub85c \ub610 \ud55c \ubc88 \uadf8 \uc9c4\uac00\ub97c \ubc1c\ud718\ud558\uba70 \uba85\uac10\ub3c5\uc73c\ub85c\uc11c\uc758 \uc785\uc9c0\ub97c \ub2e4\uc84c\ub2e4. \uc774 \uc601\ud654\ub85c \uc81c 16\ud68c \ub514\ub809\ud130\uc2a4 \ucef7 \uc2dc\uc0c1\uc2dd\uc5d0\uc11c \uc62c\ud574\uc758 \uac10\ub3c5\uc0c1\uc744 \uc218\uc0c1\ud588\uace0 \uad6d\ub0b4 \uac00\uc7a5 \uad8c\uc704\uc788\ub294 \uc601\ud654\uc81c\ub85c \uaf3d\ud788\ub294 \uc81c 37\ud68c \uccad\ub8e1\uc601\ud654\uc0c1 \uc2dc\uc0c1\uc2dd\uc5d0\uc11c \uac10\ub3c5\uc0c1\uc744 \uc218\uc0c1\ud588\ub2e4.", "sentence_answer": "2008\ub144\uc5d0 \uc7a5\ud3b8 \uc601\ud654 \ub370\ubdd4\uc791\uc778 \u300a \ucd94\uaca9\uc790 \u300b\uac00 \uac1c\ubd09\ud558\uc600\uc73c\uba70,\u300a\ucd94\uaca9\uc790\u300b\ub294 2008\ub144 \ubc31\uc0c1\uc608\uc220\ub300\uc0c1\uc5d0\uc11c \uc601\ud654\ub300\uc0c1 \ubc0f \uc601\ud654\uc2e0\uc778\uac10\ub3c5\uc0c1\uc744 \uc218\uc0c1, 2008\ub144 \ub300\ud55c\ubbfc\uad6d\uc601\ud654\ub300\uc0c1\uc5d0\uc11c \ucd5c\uc6b0\uc218\uc791\ud488\uc0c1, \uac10\ub3c5\uc0c1, \uac01\ubcf8/\uac01\uc0c9\uc0c1\uc744 \ubc1b\uc558\ub2e4.", "paragraph_id": "6019825-0-2", "paragraph_question": "question: \uc601\ud654 \uac10\ub3c5 \ub098\ud64d\uc9c4\uc758 \uc7a5\ud3b8\uc601\ud654 \ub370\ubdd4\uc791\uc740?, context: 2003\ub144\uc5d0 \ub2e8\ud3b8 \uc601\ud654 \u300a5 Minutes\u300b\ub97c \ub9cc\ub4e4\uc5c8\uc73c\uba70, 2005\ub144\uc5d0 \ub2e8\ud3b8 \u300a\uc644\ubcbd\ud55c \ub3c4\ubbf8\uc694\ub9ac\u300b\ub85c \uc81c4\ud68c \ubbf8\uc7dd\uc13c \ub2e8\ud3b8 \uc601\ud654\uc81c \ucd5c\uc6b0\uc218 \uc791\ud488\uc0c1\uacfc \uc81c6\ud68c \ub300\ud55c\ubbfc\uad6d \uc601\uc0c1\ub300\uc804 \uc7a5\ub824\uc0c1\uc744 \uc218\uc0c1\ud558\uace0, \uc81c25\ud68c \ud558\uc640\uc774 \uad6d\uc81c\uc601\ud654\uc81c \ucd08\uccad\uc791\uacfc \ub808\uc2a4\ud398\uc2a4\ud2b8 2005\ub144 \ud398\uc2a4\ud2f0\ubc8c \ucd08\uc774\uc2a4\ub85c \uc120\uc815\ub418\uc5c8\ub2e4. 2007\ub144\uc5d0 \ub2e8\ud3b8 \u300a\ud55c(\u6c57)\u300b\uc73c\ub85c \uc81c44\ud68c \ub300\uc885\uc0c1 \ub2e8\ud3b8 \uc601\ud654 \uac10\ub3c5\uc0c1, \uc81c11\ud68c \ubd80\ucc9c\uad6d\uc81c\ud310\ud0c0\uc2a4\ud2f1\uc601\ud654\uc81c \ub2e8\ud3b8 \ubd80\ubb38 \uc2ec\uc0ac\uc704\uc6d0\uc0c1, \uc81c8\ud68c \ub300\ud55c\ubbfc\uad6d \uc601\uc0c1\ub300\uc804 \uc6b0\uc218\uc0c1\uc744 \uc218\uc0c1\ud55c \uacbd\ub825\uc774 \uc788\ub2e4. 2008\ub144\uc5d0 \uc7a5\ud3b8 \uc601\ud654 \ub370\ubdd4\uc791\uc778 \u300a\ucd94\uaca9\uc790\u300b\uac00 \uac1c\ubd09\ud558\uc600\uc73c\uba70,\u300a\ucd94\uaca9\uc790\u300b\ub294 2008\ub144 \ubc31\uc0c1\uc608\uc220\ub300\uc0c1\uc5d0\uc11c \uc601\ud654\ub300\uc0c1 \ubc0f \uc601\ud654\uc2e0\uc778\uac10\ub3c5\uc0c1\uc744 \uc218\uc0c1, 2008\ub144 \ub300\ud55c\ubbfc\uad6d\uc601\ud654\ub300\uc0c1\uc5d0\uc11c \ucd5c\uc6b0\uc218\uc791\ud488\uc0c1, \uac10\ub3c5\uc0c1, \uac01\ubcf8/\uac01\uc0c9\uc0c1\uc744 \ubc1b\uc558\ub2e4. \uc774\ud6c4 2010\ub144 \uc601\ud654 \u300a\ucd94\uaca9\uc790\u300b\uc758 \ub450 \uc8fc\uc5ed\uc778 \uae40\uc724\uc11d, \ud558\uc815\uc6b0\uc640 \ub2e4\uc2dc \uc758\uae30\ud22c\ud569\ud558\uc5ec \uc601\ud654 \u300a\ud669\ud574\u300b\ub97c \uc120\ubcf4\uc5ec \ub610\ud55c \ub9ce\uc740 \ud654\uc81c\ub97c \ub0b3\uc558\ub2e4. \uadf8\ub7ec\ub098 2010\ub144 \uc774\ud6c4 \uc624\ub79c \uc7a5\uace0\uc5d0 \ub4e4\uc5b4\uac00\uba70 \uc791\ud488\ud65c\ub3d9\uc744 \ud558\uc9c0 \uc54a\uc558\uace0 6\ub144 \ub9cc\uc778 2016\ub144 \uad6d\ub0b4\uc5d0\uc120 \uac70\uc758 \ucd5c\ucd08\ub85c \uc2dc\ub3c4\ub41c \uc624\uceec\ud2b8\ud638\ub7ec \uc7a5\ub974\uc758 \uc601\ud654 \u300a\uace1\uc131\u300b\uc73c\ub85c \ub610 \ud55c \ubc88 \uadf8 \uc9c4\uac00\ub97c \ubc1c\ud718\ud558\uba70 \uba85\uac10\ub3c5\uc73c\ub85c\uc11c\uc758 \uc785\uc9c0\ub97c \ub2e4\uc84c\ub2e4. \uc774 \uc601\ud654\ub85c \uc81c 16\ud68c \ub514\ub809\ud130\uc2a4 \ucef7 \uc2dc\uc0c1\uc2dd\uc5d0\uc11c \uc62c\ud574\uc758 \uac10\ub3c5\uc0c1\uc744 \uc218\uc0c1\ud588\uace0 \uad6d\ub0b4 \uac00\uc7a5 \uad8c\uc704\uc788\ub294 \uc601\ud654\uc81c\ub85c \uaf3d\ud788\ub294 \uc81c 37\ud68c \uccad\ub8e1\uc601\ud654\uc0c1 \uc2dc\uc0c1\uc2dd\uc5d0\uc11c \uac10\ub3c5\uc0c1\uc744 \uc218\uc0c1\ud588\ub2e4."} {"question": "\ud55c\uad6d\uce5c\uc6b0\ud68c\uac00 \uc870\uc9c1\ub41c \uc720\ub7fd\uc758 \ub300\ub959\uad6d\uac00\ub294?", "paragraph": "\ud55c\uad6d\uce5c\uc6b0\ud68c\ub294 \ubbf8\uad6d\ubfd0\ub9cc \uc544\ub2c8\ub77c \uc601\uad6d\uacfc \ud504\ub791\uc2a4\uc5d0\ub3c4 \uc870\uc9c1\ub418\uc5b4 \uad11\ubc94\uc704\ud55c \uad6d\uc81c\uc801\uc778 \uc870\uc9c1\uc744 \uac16\ucda4\uc5d0 \ub530\ub77c 3\u00b71\uc6b4\ub3d9\uc73c\ub85c \ub098\ud0c0\ub09c \ud55c\uad6d\uc758 \uc2e4\uc0c1\uc744 \uc804\ud30c\ud558\uace0 \uce5c\ud55c\uc5ec\ub860\uc744 \ud615\uc131\ud558\ub294\ub370 \uc0c1\ub2f9\ud55c \uc601\ud5a5\ub825\uc744 \ubbf8\ucce4\ub2e4. \uc77c\ubcf8\uce21\uc740 \ubbf8\uad6d\uc5d0 \uc788\ub294 \uc5ec\ub7ec \ud55c\uc778\ub2e8\uccb4 \uac00\uc6b4\ub370 \u201c\uc870\uc120\uc6b0\uc560\ub2e8\uc758 \ud589\ub3d9\uc5d0 \uad00\ud574\uc11c\ub294 \uae08\ud6c4 \ub2e4\uc18c \uc8fc\uc758\ub97c \uc694\ud560 \ud544\uc694\uac00 \uc788\uc744 \uac83\uc774\ub2e4\u201d\ud558\uc5ec \ud55c\uad6d\uce5c\uc6b0\ud68c\uc758 \uacb0\uc131\uacfc \ud65c\ub3d9\uc5d0 \ud2b9\ud788 \uc8fc\ubaa9\ud558\uc600\ub2e4. \uc6cc\uc2f1\ud134\uacfc \uac01 \uc8fc \uc8fc\uc7ac \uc77c\ubcf8\uc601\uc0ac\uad00\uc740 \uc774\ub4e4 \ub2e8\uccb4\uac00 \ubc18\uc77c\ubcf8 \uc120\ub3d9\uc744 \ud558\ub294 \ub2e8\uccb4\uc778\uc9c0 \uc5ec\ubd80\ub97c \uc0ac\ub78c\ub4e4\uc744 \uace0\uc6a9, \ud30c\uacac\ud558\uc5ec \uac10\uc2dc\ud558\uc600\ub2e4. \uadf8\ub7ec\ub098 \ucc3d\ub9bd \ucd08\uae30, \ube44\uc815\uce58\uc801 \ubaa9\uc801\uc73c\ub85c \uc124\ub9bd\ud558\uc600\uc73c\uba70 \uad6c\ubbf8 \uae30\ub3c5\uad50\uacc4 \uc778\uc0ac\ub4e4\uc5d0\uac8c \uc870\uc120\uc5d0 \ub300\ud55c \uc6b0\ud638\uc801 \uc5ec\ub860\uacfc \uce5c\ubaa9\uc744 \ub3c4\ubaa8\ud55c\ub2e4\ub294 \ub0b4\uc6a9\ub9cc\uc774 \uc788\uc5c8\uc73c\ubbc0\ub85c \ubcc4\ub2e4\ub978 \uad6c\uc2e4\uc744 \ucc3e\uc9c0 \ubabb\ud558\uc600\ub2e4.", "answer": "\ud504\ub791\uc2a4", "sentence": "\ud55c\uad6d\uce5c\uc6b0\ud68c\ub294 \ubbf8\uad6d\ubfd0\ub9cc \uc544\ub2c8\ub77c \uc601\uad6d\uacfc \ud504\ub791\uc2a4 \uc5d0\ub3c4 \uc870\uc9c1\ub418\uc5b4 \uad11\ubc94\uc704\ud55c \uad6d\uc81c\uc801\uc778 \uc870\uc9c1\uc744 \uac16\ucda4\uc5d0 \ub530\ub77c 3\u00b71\uc6b4\ub3d9\uc73c\ub85c \ub098\ud0c0\ub09c \ud55c\uad6d\uc758 \uc2e4\uc0c1\uc744 \uc804\ud30c\ud558\uace0 \uce5c\ud55c\uc5ec\ub860\uc744 \ud615\uc131\ud558\ub294\ub370 \uc0c1\ub2f9\ud55c \uc601\ud5a5\ub825\uc744 \ubbf8\ucce4\ub2e4.", "paragraph_sentence": " \ud55c\uad6d\uce5c\uc6b0\ud68c\ub294 \ubbf8\uad6d\ubfd0\ub9cc \uc544\ub2c8\ub77c \uc601\uad6d\uacfc \ud504\ub791\uc2a4 \uc5d0\ub3c4 \uc870\uc9c1\ub418\uc5b4 \uad11\ubc94\uc704\ud55c \uad6d\uc81c\uc801\uc778 \uc870\uc9c1\uc744 \uac16\ucda4\uc5d0 \ub530\ub77c 3\u00b71\uc6b4\ub3d9\uc73c\ub85c \ub098\ud0c0\ub09c \ud55c\uad6d\uc758 \uc2e4\uc0c1\uc744 \uc804\ud30c\ud558\uace0 \uce5c\ud55c\uc5ec\ub860\uc744 \ud615\uc131\ud558\ub294\ub370 \uc0c1\ub2f9\ud55c \uc601\ud5a5\ub825\uc744 \ubbf8\ucce4\ub2e4. \uc77c\ubcf8\uce21\uc740 \ubbf8\uad6d\uc5d0 \uc788\ub294 \uc5ec\ub7ec \ud55c\uc778\ub2e8\uccb4 \uac00\uc6b4\ub370 \u201c\uc870\uc120\uc6b0\uc560\ub2e8\uc758 \ud589\ub3d9\uc5d0 \uad00\ud574\uc11c\ub294 \uae08\ud6c4 \ub2e4\uc18c \uc8fc\uc758\ub97c \uc694\ud560 \ud544\uc694\uac00 \uc788\uc744 \uac83\uc774\ub2e4\u201d\ud558\uc5ec \ud55c\uad6d\uce5c\uc6b0\ud68c\uc758 \uacb0\uc131\uacfc \ud65c\ub3d9\uc5d0 \ud2b9\ud788 \uc8fc\ubaa9\ud558\uc600\ub2e4. \uc6cc\uc2f1\ud134\uacfc \uac01 \uc8fc \uc8fc\uc7ac \uc77c\ubcf8\uc601\uc0ac\uad00\uc740 \uc774\ub4e4 \ub2e8\uccb4\uac00 \ubc18\uc77c\ubcf8 \uc120\ub3d9\uc744 \ud558\ub294 \ub2e8\uccb4\uc778\uc9c0 \uc5ec\ubd80\ub97c \uc0ac\ub78c\ub4e4\uc744 \uace0\uc6a9, \ud30c\uacac\ud558\uc5ec \uac10\uc2dc\ud558\uc600\ub2e4. \uadf8\ub7ec\ub098 \ucc3d\ub9bd \ucd08\uae30, \ube44\uc815\uce58\uc801 \ubaa9\uc801\uc73c\ub85c \uc124\ub9bd\ud558\uc600\uc73c\uba70 \uad6c\ubbf8 \uae30\ub3c5\uad50\uacc4 \uc778\uc0ac\ub4e4\uc5d0\uac8c \uc870\uc120\uc5d0 \ub300\ud55c \uc6b0\ud638\uc801 \uc5ec\ub860\uacfc \uce5c\ubaa9\uc744 \ub3c4\ubaa8\ud55c\ub2e4\ub294 \ub0b4\uc6a9\ub9cc\uc774 \uc788\uc5c8\uc73c\ubbc0\ub85c \ubcc4\ub2e4\ub978 \uad6c\uc2e4\uc744 \ucc3e\uc9c0 \ubabb\ud558\uc600\ub2e4.", "paragraph_answer": "\ud55c\uad6d\uce5c\uc6b0\ud68c\ub294 \ubbf8\uad6d\ubfd0\ub9cc \uc544\ub2c8\ub77c \uc601\uad6d\uacfc \ud504\ub791\uc2a4 \uc5d0\ub3c4 \uc870\uc9c1\ub418\uc5b4 \uad11\ubc94\uc704\ud55c \uad6d\uc81c\uc801\uc778 \uc870\uc9c1\uc744 \uac16\ucda4\uc5d0 \ub530\ub77c 3\u00b71\uc6b4\ub3d9\uc73c\ub85c \ub098\ud0c0\ub09c \ud55c\uad6d\uc758 \uc2e4\uc0c1\uc744 \uc804\ud30c\ud558\uace0 \uce5c\ud55c\uc5ec\ub860\uc744 \ud615\uc131\ud558\ub294\ub370 \uc0c1\ub2f9\ud55c \uc601\ud5a5\ub825\uc744 \ubbf8\ucce4\ub2e4. \uc77c\ubcf8\uce21\uc740 \ubbf8\uad6d\uc5d0 \uc788\ub294 \uc5ec\ub7ec \ud55c\uc778\ub2e8\uccb4 \uac00\uc6b4\ub370 \u201c\uc870\uc120\uc6b0\uc560\ub2e8\uc758 \ud589\ub3d9\uc5d0 \uad00\ud574\uc11c\ub294 \uae08\ud6c4 \ub2e4\uc18c \uc8fc\uc758\ub97c \uc694\ud560 \ud544\uc694\uac00 \uc788\uc744 \uac83\uc774\ub2e4\u201d\ud558\uc5ec \ud55c\uad6d\uce5c\uc6b0\ud68c\uc758 \uacb0\uc131\uacfc \ud65c\ub3d9\uc5d0 \ud2b9\ud788 \uc8fc\ubaa9\ud558\uc600\ub2e4. \uc6cc\uc2f1\ud134\uacfc \uac01 \uc8fc \uc8fc\uc7ac \uc77c\ubcf8\uc601\uc0ac\uad00\uc740 \uc774\ub4e4 \ub2e8\uccb4\uac00 \ubc18\uc77c\ubcf8 \uc120\ub3d9\uc744 \ud558\ub294 \ub2e8\uccb4\uc778\uc9c0 \uc5ec\ubd80\ub97c \uc0ac\ub78c\ub4e4\uc744 \uace0\uc6a9, \ud30c\uacac\ud558\uc5ec \uac10\uc2dc\ud558\uc600\ub2e4. \uadf8\ub7ec\ub098 \ucc3d\ub9bd \ucd08\uae30, \ube44\uc815\uce58\uc801 \ubaa9\uc801\uc73c\ub85c \uc124\ub9bd\ud558\uc600\uc73c\uba70 \uad6c\ubbf8 \uae30\ub3c5\uad50\uacc4 \uc778\uc0ac\ub4e4\uc5d0\uac8c \uc870\uc120\uc5d0 \ub300\ud55c \uc6b0\ud638\uc801 \uc5ec\ub860\uacfc \uce5c\ubaa9\uc744 \ub3c4\ubaa8\ud55c\ub2e4\ub294 \ub0b4\uc6a9\ub9cc\uc774 \uc788\uc5c8\uc73c\ubbc0\ub85c \ubcc4\ub2e4\ub978 \uad6c\uc2e4\uc744 \ucc3e\uc9c0 \ubabb\ud558\uc600\ub2e4.", "sentence_answer": "\ud55c\uad6d\uce5c\uc6b0\ud68c\ub294 \ubbf8\uad6d\ubfd0\ub9cc \uc544\ub2c8\ub77c \uc601\uad6d\uacfc \ud504\ub791\uc2a4 \uc5d0\ub3c4 \uc870\uc9c1\ub418\uc5b4 \uad11\ubc94\uc704\ud55c \uad6d\uc81c\uc801\uc778 \uc870\uc9c1\uc744 \uac16\ucda4\uc5d0 \ub530\ub77c 3\u00b71\uc6b4\ub3d9\uc73c\ub85c \ub098\ud0c0\ub09c \ud55c\uad6d\uc758 \uc2e4\uc0c1\uc744 \uc804\ud30c\ud558\uace0 \uce5c\ud55c\uc5ec\ub860\uc744 \ud615\uc131\ud558\ub294\ub370 \uc0c1\ub2f9\ud55c \uc601\ud5a5\ub825\uc744 \ubbf8\ucce4\ub2e4.", "paragraph_id": "6569969-3-0", "paragraph_question": "question: \ud55c\uad6d\uce5c\uc6b0\ud68c\uac00 \uc870\uc9c1\ub41c \uc720\ub7fd\uc758 \ub300\ub959\uad6d\uac00\ub294?, context: \ud55c\uad6d\uce5c\uc6b0\ud68c\ub294 \ubbf8\uad6d\ubfd0\ub9cc \uc544\ub2c8\ub77c \uc601\uad6d\uacfc \ud504\ub791\uc2a4\uc5d0\ub3c4 \uc870\uc9c1\ub418\uc5b4 \uad11\ubc94\uc704\ud55c \uad6d\uc81c\uc801\uc778 \uc870\uc9c1\uc744 \uac16\ucda4\uc5d0 \ub530\ub77c 3\u00b71\uc6b4\ub3d9\uc73c\ub85c \ub098\ud0c0\ub09c \ud55c\uad6d\uc758 \uc2e4\uc0c1\uc744 \uc804\ud30c\ud558\uace0 \uce5c\ud55c\uc5ec\ub860\uc744 \ud615\uc131\ud558\ub294\ub370 \uc0c1\ub2f9\ud55c \uc601\ud5a5\ub825\uc744 \ubbf8\ucce4\ub2e4. \uc77c\ubcf8\uce21\uc740 \ubbf8\uad6d\uc5d0 \uc788\ub294 \uc5ec\ub7ec \ud55c\uc778\ub2e8\uccb4 \uac00\uc6b4\ub370 \u201c\uc870\uc120\uc6b0\uc560\ub2e8\uc758 \ud589\ub3d9\uc5d0 \uad00\ud574\uc11c\ub294 \uae08\ud6c4 \ub2e4\uc18c \uc8fc\uc758\ub97c \uc694\ud560 \ud544\uc694\uac00 \uc788\uc744 \uac83\uc774\ub2e4\u201d\ud558\uc5ec \ud55c\uad6d\uce5c\uc6b0\ud68c\uc758 \uacb0\uc131\uacfc \ud65c\ub3d9\uc5d0 \ud2b9\ud788 \uc8fc\ubaa9\ud558\uc600\ub2e4. \uc6cc\uc2f1\ud134\uacfc \uac01 \uc8fc \uc8fc\uc7ac \uc77c\ubcf8\uc601\uc0ac\uad00\uc740 \uc774\ub4e4 \ub2e8\uccb4\uac00 \ubc18\uc77c\ubcf8 \uc120\ub3d9\uc744 \ud558\ub294 \ub2e8\uccb4\uc778\uc9c0 \uc5ec\ubd80\ub97c \uc0ac\ub78c\ub4e4\uc744 \uace0\uc6a9, \ud30c\uacac\ud558\uc5ec \uac10\uc2dc\ud558\uc600\ub2e4. \uadf8\ub7ec\ub098 \ucc3d\ub9bd \ucd08\uae30, \ube44\uc815\uce58\uc801 \ubaa9\uc801\uc73c\ub85c \uc124\ub9bd\ud558\uc600\uc73c\uba70 \uad6c\ubbf8 \uae30\ub3c5\uad50\uacc4 \uc778\uc0ac\ub4e4\uc5d0\uac8c \uc870\uc120\uc5d0 \ub300\ud55c \uc6b0\ud638\uc801 \uc5ec\ub860\uacfc \uce5c\ubaa9\uc744 \ub3c4\ubaa8\ud55c\ub2e4\ub294 \ub0b4\uc6a9\ub9cc\uc774 \uc788\uc5c8\uc73c\ubbc0\ub85c \ubcc4\ub2e4\ub978 \uad6c\uc2e4\uc744 \ucc3e\uc9c0 \ubabb\ud558\uc600\ub2e4."} {"question": "\uc2dc\uc98c \ucd08\ubc18 \ubd80\uc9c4\ud55c \uc0ac\uce74\uad6c\uce58 \ub3c4\ubaa8\ud0c0\uce74 \ub300\uc2e0 1\ubc88 \ud0c0\uc790\ub85c \ub6f0\uc5c8\ub358 \uc120\uc218\ub294?", "paragraph": "\uc2dc\uc98c \ub3c4\uc911\ubd80\ud130 \uae34\ud14c\uc4f0 \uc120\uc218 \uc2dc\uc808\uc758 \uc751\uc6d0\uac00\ub85c \ubc14\ub00c\uba74\uc11c \uc804\uc6a9 \uc751\uc6d0\uac00\ub85c \uc0c8\ub86d\uac8c \ub9cc\ub4e4\uc5b4\uc84c\ub2e4. \uc2dc\ubc94 \uacbd\uae30\uc5d0\uc11c\ub294 \ud0c0\uc728 3\ud560 4\ud47c 9\ub9ac\uc758 \uc131\uc801\uc73c\ub85c \ud638\uc870\ub97c \ubcf4\uc5ec\uc8fc\uc5c8\uc9c0\ub9cc \uac1c\ub9c9 \uc774\ud6c4\ubd80\ud130 5\uc6d4 \ucd08\uc21c\uae4c\uc9c0 \ud0c0\uc728 1\ud560 \ub300\uc758 \uc131\uc801\uc73c\ub85c \ubd80\uc9c4\uc5d0 \ube60\uc9c0\ub294 \ub4f1 \uac19\uc740 \ud574\uc5d0 \uc785\ub2e8\ud55c \uae34\ud14c\uc4f0 \uc2dc\uc808\uc758 \ud300\uba54\uc774\ud2b8\uc774\uae30\ub3c4 \ud55c \uc624\ubb34\ub77c \ub098\uc624\uc720\ud0a4\uac00 1\ubc88 \ud0c0\uc790\ub85c \ub6f0\uac8c \ub418\uc5c8\ub2e4. \uadf8 \uc774\ud6c4\uc5d0\ub294 \uc11c\uc11c\ud788 \ud68c\ubcf5\ub418\uc5b4 6\uc6d4\ubd80\ud130\ub294 \uc624\ubb34\ub77c\uc640 \ub2e4\uc2dc \ubc14\ub00c\uba74\uc11c 1\ubc88 \ud0c0\uc790\ub85c \ubcf5\uadc0\ud588\ub2e4. 5\uc6d4 \uc774\ud6c4\uc5d0\ub294 7\uc6d4 \uc774\uc678\uc5d0 \ubaa8\ub450 \uc6d4\uac04 \ud0c0\uc728\uc774 3\ud560 \uc774\uc0c1\uc744 \uae30\ub85d\ud574 8\uc6d4\uc5d0\ub294 \uc6d4\uac04 \ud0c0\uc728 3\ud560 8\ud47c 6\ub9ac, \uc774\uce58\ub85c \uc774\ud6c4 \uad6c\ub2e8 \uc0ac\uc0c1 \ub450 \ubc88\uc9f8\uac00 \ub418\ub294 \uc6d4\uac04 40\uac1c\uc758 \uc548\ud0c0\ub97c \uae30\ub85d\ud588\ub2e4. \ucd5c\uc885\uc801\uc73c\ub85c \ud300\ub0b4 1\uc704\uc778 137\uacbd\uae30\uc5d0 \ucd9c\uc804\ud574 \ud300\ub0b4 \uaddc\uc815 \ud0c0\uc11d\uc744 \ucc44\uc6b4 \uc120\uc218\ub85c\uc11c \uc720\uc77c\ud55c \ud0c0\uc728 3\ud560\uc774 \ub418\ub294 3\ud560 1\ud47c 7\ub9ac\uc758 \ud0c0\uc728(\ub9ac\uadf8 2\uc704)\uacfc \uc548\ud0c0 \ubd80\ubb38 2\uc704\uc758 167\uc548\ud0c0, \uc804\ub144\ub3c4\uc758 \uac1c\uc778 \ucd5c\ub2e4 \uc131\uc801\uc744 \uacbd\uc2e0\ud558\ub294 \ud300\ub0b4 1\uc704\uc774\uc790 \ub9ac\uadf8 10\uc704\uc758 16\uac1c \ub3c4\ub8e8\ub97c \uae30\ub85d\ud588\ub2e4. \ub9ac\uadf8 6\uc704\uc758 \ucd9c\ub8e8\uc728\uc740 3\ud560 8\ud47c 1\ub9ac\ub85c \uc88c\uc644 \ud22c\uc218\uc640 \uc0c1\ub300\ud55c \ud0c0\uc728\uc774 2\ud560 9\ud47c 7\ub9ac\ub85c \uc804\ub144\ub3c4 \uc2dc\uc98c\uc5d0\uc11c\uc758 \uacfc\uc81c\ub97c \uc548\uae34 \ubd80\ubd84\uc5d0 \ub300\ud574\uc11c\ub3c4 \uac1c\uc120\ub418\uc5c8\ub2e4.", "answer": "\uc624\ubb34\ub77c \ub098\uc624\uc720\ud0a4", "sentence": "\uc2dc\ubc94 \uacbd\uae30\uc5d0\uc11c\ub294 \ud0c0\uc728 3\ud560 4\ud47c 9\ub9ac\uc758 \uc131\uc801\uc73c\ub85c \ud638\uc870\ub97c \ubcf4\uc5ec\uc8fc\uc5c8\uc9c0\ub9cc \uac1c\ub9c9 \uc774\ud6c4\ubd80\ud130 5\uc6d4 \ucd08\uc21c\uae4c\uc9c0 \ud0c0\uc728 1\ud560 \ub300\uc758 \uc131\uc801\uc73c\ub85c \ubd80\uc9c4\uc5d0 \ube60\uc9c0\ub294 \ub4f1 \uac19\uc740 \ud574\uc5d0 \uc785\ub2e8\ud55c \uae34\ud14c\uc4f0 \uc2dc\uc808\uc758 \ud300\uba54\uc774\ud2b8\uc774\uae30\ub3c4 \ud55c \uc624\ubb34\ub77c \ub098\uc624\uc720\ud0a4 \uac00 1\ubc88 \ud0c0\uc790\ub85c \ub6f0\uac8c \ub418\uc5c8\ub2e4.", "paragraph_sentence": "\uc2dc\uc98c \ub3c4\uc911\ubd80\ud130 \uae34\ud14c\uc4f0 \uc120\uc218 \uc2dc\uc808\uc758 \uc751\uc6d0\uac00\ub85c \ubc14\ub00c\uba74\uc11c \uc804\uc6a9 \uc751\uc6d0\uac00\ub85c \uc0c8\ub86d\uac8c \ub9cc\ub4e4\uc5b4\uc84c\ub2e4. \uc2dc\ubc94 \uacbd\uae30\uc5d0\uc11c\ub294 \ud0c0\uc728 3\ud560 4\ud47c 9\ub9ac\uc758 \uc131\uc801\uc73c\ub85c \ud638\uc870\ub97c \ubcf4\uc5ec\uc8fc\uc5c8\uc9c0\ub9cc \uac1c\ub9c9 \uc774\ud6c4\ubd80\ud130 5\uc6d4 \ucd08\uc21c\uae4c\uc9c0 \ud0c0\uc728 1\ud560 \ub300\uc758 \uc131\uc801\uc73c\ub85c \ubd80\uc9c4\uc5d0 \ube60\uc9c0\ub294 \ub4f1 \uac19\uc740 \ud574\uc5d0 \uc785\ub2e8\ud55c \uae34\ud14c\uc4f0 \uc2dc\uc808\uc758 \ud300\uba54\uc774\ud2b8\uc774\uae30\ub3c4 \ud55c \uc624\ubb34\ub77c \ub098\uc624\uc720\ud0a4 \uac00 1\ubc88 \ud0c0\uc790\ub85c \ub6f0\uac8c \ub418\uc5c8\ub2e4. \uadf8 \uc774\ud6c4\uc5d0\ub294 \uc11c\uc11c\ud788 \ud68c\ubcf5\ub418\uc5b4 6\uc6d4\ubd80\ud130\ub294 \uc624\ubb34\ub77c\uc640 \ub2e4\uc2dc \ubc14\ub00c\uba74\uc11c 1\ubc88 \ud0c0\uc790\ub85c \ubcf5\uadc0\ud588\ub2e4. 5\uc6d4 \uc774\ud6c4\uc5d0\ub294 7\uc6d4 \uc774\uc678\uc5d0 \ubaa8\ub450 \uc6d4\uac04 \ud0c0\uc728\uc774 3\ud560 \uc774\uc0c1\uc744 \uae30\ub85d\ud574 8\uc6d4\uc5d0\ub294 \uc6d4\uac04 \ud0c0\uc728 3\ud560 8\ud47c 6\ub9ac, \uc774\uce58\ub85c \uc774\ud6c4 \uad6c\ub2e8 \uc0ac\uc0c1 \ub450 \ubc88\uc9f8\uac00 \ub418\ub294 \uc6d4\uac04 40\uac1c\uc758 \uc548\ud0c0\ub97c \uae30\ub85d\ud588\ub2e4. \ucd5c\uc885\uc801\uc73c\ub85c \ud300\ub0b4 1\uc704\uc778 137\uacbd\uae30\uc5d0 \ucd9c\uc804\ud574 \ud300\ub0b4 \uaddc\uc815 \ud0c0\uc11d\uc744 \ucc44\uc6b4 \uc120\uc218\ub85c\uc11c \uc720\uc77c\ud55c \ud0c0\uc728 3\ud560\uc774 \ub418\ub294 3\ud560 1\ud47c 7\ub9ac\uc758 \ud0c0\uc728(\ub9ac\uadf8 2\uc704)\uacfc \uc548\ud0c0 \ubd80\ubb38 2\uc704\uc758 167\uc548\ud0c0, \uc804\ub144\ub3c4\uc758 \uac1c\uc778 \ucd5c\ub2e4 \uc131\uc801\uc744 \uacbd\uc2e0\ud558\ub294 \ud300\ub0b4 1\uc704\uc774\uc790 \ub9ac\uadf8 10\uc704\uc758 16\uac1c \ub3c4\ub8e8\ub97c \uae30\ub85d\ud588\ub2e4. \ub9ac\uadf8 6\uc704\uc758 \ucd9c\ub8e8\uc728\uc740 3\ud560 8\ud47c 1\ub9ac\ub85c \uc88c\uc644 \ud22c\uc218\uc640 \uc0c1\ub300\ud55c \ud0c0\uc728\uc774 2\ud560 9\ud47c 7\ub9ac\ub85c \uc804\ub144\ub3c4 \uc2dc\uc98c\uc5d0\uc11c\uc758 \uacfc\uc81c\ub97c \uc548\uae34 \ubd80\ubd84\uc5d0 \ub300\ud574\uc11c\ub3c4 \uac1c\uc120\ub418\uc5c8\ub2e4.", "paragraph_answer": "\uc2dc\uc98c \ub3c4\uc911\ubd80\ud130 \uae34\ud14c\uc4f0 \uc120\uc218 \uc2dc\uc808\uc758 \uc751\uc6d0\uac00\ub85c \ubc14\ub00c\uba74\uc11c \uc804\uc6a9 \uc751\uc6d0\uac00\ub85c \uc0c8\ub86d\uac8c \ub9cc\ub4e4\uc5b4\uc84c\ub2e4. \uc2dc\ubc94 \uacbd\uae30\uc5d0\uc11c\ub294 \ud0c0\uc728 3\ud560 4\ud47c 9\ub9ac\uc758 \uc131\uc801\uc73c\ub85c \ud638\uc870\ub97c \ubcf4\uc5ec\uc8fc\uc5c8\uc9c0\ub9cc \uac1c\ub9c9 \uc774\ud6c4\ubd80\ud130 5\uc6d4 \ucd08\uc21c\uae4c\uc9c0 \ud0c0\uc728 1\ud560 \ub300\uc758 \uc131\uc801\uc73c\ub85c \ubd80\uc9c4\uc5d0 \ube60\uc9c0\ub294 \ub4f1 \uac19\uc740 \ud574\uc5d0 \uc785\ub2e8\ud55c \uae34\ud14c\uc4f0 \uc2dc\uc808\uc758 \ud300\uba54\uc774\ud2b8\uc774\uae30\ub3c4 \ud55c \uc624\ubb34\ub77c \ub098\uc624\uc720\ud0a4 \uac00 1\ubc88 \ud0c0\uc790\ub85c \ub6f0\uac8c \ub418\uc5c8\ub2e4. \uadf8 \uc774\ud6c4\uc5d0\ub294 \uc11c\uc11c\ud788 \ud68c\ubcf5\ub418\uc5b4 6\uc6d4\ubd80\ud130\ub294 \uc624\ubb34\ub77c\uc640 \ub2e4\uc2dc \ubc14\ub00c\uba74\uc11c 1\ubc88 \ud0c0\uc790\ub85c \ubcf5\uadc0\ud588\ub2e4. 5\uc6d4 \uc774\ud6c4\uc5d0\ub294 7\uc6d4 \uc774\uc678\uc5d0 \ubaa8\ub450 \uc6d4\uac04 \ud0c0\uc728\uc774 3\ud560 \uc774\uc0c1\uc744 \uae30\ub85d\ud574 8\uc6d4\uc5d0\ub294 \uc6d4\uac04 \ud0c0\uc728 3\ud560 8\ud47c 6\ub9ac, \uc774\uce58\ub85c \uc774\ud6c4 \uad6c\ub2e8 \uc0ac\uc0c1 \ub450 \ubc88\uc9f8\uac00 \ub418\ub294 \uc6d4\uac04 40\uac1c\uc758 \uc548\ud0c0\ub97c \uae30\ub85d\ud588\ub2e4. \ucd5c\uc885\uc801\uc73c\ub85c \ud300\ub0b4 1\uc704\uc778 137\uacbd\uae30\uc5d0 \ucd9c\uc804\ud574 \ud300\ub0b4 \uaddc\uc815 \ud0c0\uc11d\uc744 \ucc44\uc6b4 \uc120\uc218\ub85c\uc11c \uc720\uc77c\ud55c \ud0c0\uc728 3\ud560\uc774 \ub418\ub294 3\ud560 1\ud47c 7\ub9ac\uc758 \ud0c0\uc728(\ub9ac\uadf8 2\uc704)\uacfc \uc548\ud0c0 \ubd80\ubb38 2\uc704\uc758 167\uc548\ud0c0, \uc804\ub144\ub3c4\uc758 \uac1c\uc778 \ucd5c\ub2e4 \uc131\uc801\uc744 \uacbd\uc2e0\ud558\ub294 \ud300\ub0b4 1\uc704\uc774\uc790 \ub9ac\uadf8 10\uc704\uc758 16\uac1c \ub3c4\ub8e8\ub97c \uae30\ub85d\ud588\ub2e4. \ub9ac\uadf8 6\uc704\uc758 \ucd9c\ub8e8\uc728\uc740 3\ud560 8\ud47c 1\ub9ac\ub85c \uc88c\uc644 \ud22c\uc218\uc640 \uc0c1\ub300\ud55c \ud0c0\uc728\uc774 2\ud560 9\ud47c 7\ub9ac\ub85c \uc804\ub144\ub3c4 \uc2dc\uc98c\uc5d0\uc11c\uc758 \uacfc\uc81c\ub97c \uc548\uae34 \ubd80\ubd84\uc5d0 \ub300\ud574\uc11c\ub3c4 \uac1c\uc120\ub418\uc5c8\ub2e4.", "sentence_answer": "\uc2dc\ubc94 \uacbd\uae30\uc5d0\uc11c\ub294 \ud0c0\uc728 3\ud560 4\ud47c 9\ub9ac\uc758 \uc131\uc801\uc73c\ub85c \ud638\uc870\ub97c \ubcf4\uc5ec\uc8fc\uc5c8\uc9c0\ub9cc \uac1c\ub9c9 \uc774\ud6c4\ubd80\ud130 5\uc6d4 \ucd08\uc21c\uae4c\uc9c0 \ud0c0\uc728 1\ud560 \ub300\uc758 \uc131\uc801\uc73c\ub85c \ubd80\uc9c4\uc5d0 \ube60\uc9c0\ub294 \ub4f1 \uac19\uc740 \ud574\uc5d0 \uc785\ub2e8\ud55c \uae34\ud14c\uc4f0 \uc2dc\uc808\uc758 \ud300\uba54\uc774\ud2b8\uc774\uae30\ub3c4 \ud55c \uc624\ubb34\ub77c \ub098\uc624\uc720\ud0a4 \uac00 1\ubc88 \ud0c0\uc790\ub85c \ub6f0\uac8c \ub418\uc5c8\ub2e4.", "paragraph_id": "6517457-2-0", "paragraph_question": "question: \uc2dc\uc98c \ucd08\ubc18 \ubd80\uc9c4\ud55c \uc0ac\uce74\uad6c\uce58 \ub3c4\ubaa8\ud0c0\uce74 \ub300\uc2e0 1\ubc88 \ud0c0\uc790\ub85c \ub6f0\uc5c8\ub358 \uc120\uc218\ub294?, context: \uc2dc\uc98c \ub3c4\uc911\ubd80\ud130 \uae34\ud14c\uc4f0 \uc120\uc218 \uc2dc\uc808\uc758 \uc751\uc6d0\uac00\ub85c \ubc14\ub00c\uba74\uc11c \uc804\uc6a9 \uc751\uc6d0\uac00\ub85c \uc0c8\ub86d\uac8c \ub9cc\ub4e4\uc5b4\uc84c\ub2e4. \uc2dc\ubc94 \uacbd\uae30\uc5d0\uc11c\ub294 \ud0c0\uc728 3\ud560 4\ud47c 9\ub9ac\uc758 \uc131\uc801\uc73c\ub85c \ud638\uc870\ub97c \ubcf4\uc5ec\uc8fc\uc5c8\uc9c0\ub9cc \uac1c\ub9c9 \uc774\ud6c4\ubd80\ud130 5\uc6d4 \ucd08\uc21c\uae4c\uc9c0 \ud0c0\uc728 1\ud560 \ub300\uc758 \uc131\uc801\uc73c\ub85c \ubd80\uc9c4\uc5d0 \ube60\uc9c0\ub294 \ub4f1 \uac19\uc740 \ud574\uc5d0 \uc785\ub2e8\ud55c \uae34\ud14c\uc4f0 \uc2dc\uc808\uc758 \ud300\uba54\uc774\ud2b8\uc774\uae30\ub3c4 \ud55c \uc624\ubb34\ub77c \ub098\uc624\uc720\ud0a4\uac00 1\ubc88 \ud0c0\uc790\ub85c \ub6f0\uac8c \ub418\uc5c8\ub2e4. \uadf8 \uc774\ud6c4\uc5d0\ub294 \uc11c\uc11c\ud788 \ud68c\ubcf5\ub418\uc5b4 6\uc6d4\ubd80\ud130\ub294 \uc624\ubb34\ub77c\uc640 \ub2e4\uc2dc \ubc14\ub00c\uba74\uc11c 1\ubc88 \ud0c0\uc790\ub85c \ubcf5\uadc0\ud588\ub2e4. 5\uc6d4 \uc774\ud6c4\uc5d0\ub294 7\uc6d4 \uc774\uc678\uc5d0 \ubaa8\ub450 \uc6d4\uac04 \ud0c0\uc728\uc774 3\ud560 \uc774\uc0c1\uc744 \uae30\ub85d\ud574 8\uc6d4\uc5d0\ub294 \uc6d4\uac04 \ud0c0\uc728 3\ud560 8\ud47c 6\ub9ac, \uc774\uce58\ub85c \uc774\ud6c4 \uad6c\ub2e8 \uc0ac\uc0c1 \ub450 \ubc88\uc9f8\uac00 \ub418\ub294 \uc6d4\uac04 40\uac1c\uc758 \uc548\ud0c0\ub97c \uae30\ub85d\ud588\ub2e4. \ucd5c\uc885\uc801\uc73c\ub85c \ud300\ub0b4 1\uc704\uc778 137\uacbd\uae30\uc5d0 \ucd9c\uc804\ud574 \ud300\ub0b4 \uaddc\uc815 \ud0c0\uc11d\uc744 \ucc44\uc6b4 \uc120\uc218\ub85c\uc11c \uc720\uc77c\ud55c \ud0c0\uc728 3\ud560\uc774 \ub418\ub294 3\ud560 1\ud47c 7\ub9ac\uc758 \ud0c0\uc728(\ub9ac\uadf8 2\uc704)\uacfc \uc548\ud0c0 \ubd80\ubb38 2\uc704\uc758 167\uc548\ud0c0, \uc804\ub144\ub3c4\uc758 \uac1c\uc778 \ucd5c\ub2e4 \uc131\uc801\uc744 \uacbd\uc2e0\ud558\ub294 \ud300\ub0b4 1\uc704\uc774\uc790 \ub9ac\uadf8 10\uc704\uc758 16\uac1c \ub3c4\ub8e8\ub97c \uae30\ub85d\ud588\ub2e4. \ub9ac\uadf8 6\uc704\uc758 \ucd9c\ub8e8\uc728\uc740 3\ud560 8\ud47c 1\ub9ac\ub85c \uc88c\uc644 \ud22c\uc218\uc640 \uc0c1\ub300\ud55c \ud0c0\uc728\uc774 2\ud560 9\ud47c 7\ub9ac\ub85c \uc804\ub144\ub3c4 \uc2dc\uc98c\uc5d0\uc11c\uc758 \uacfc\uc81c\ub97c \uc548\uae34 \ubd80\ubd84\uc5d0 \ub300\ud574\uc11c\ub3c4 \uac1c\uc120\ub418\uc5c8\ub2e4."} {"question": "\ub85c\ubca0\ub974\ud2b8\uc640 \uc218\ud559 \uadc0\uc2e0\uc758 \ub9cc\ub0a8\uc774 \ub9c8\ubb34\ub9ac \ub41c \ubc24\uc740 \uba87 \ubc88\uc9f8 \ubc24\uc778\uac00?", "paragraph": "\uc544\ud649 \ubc88\uc9f8 \ubc24\uc5d0, \ub85c\ubca0\ub974\ud2b8\ub294 \uafc8\uc5d0\uc11c \uac10\uae30\uc5d0 \uac78\ub824 \uce68\ub300\uc5d0 \ub204\uc6cc\uc788\uac8c \ub418\ub294\ub370 \uadf8\ub54c \uc218\ud559 \uadc0\uc2e0\uc774 \uc606\uc73c\ub85c \ub2e4\uac00\uc628\ub2e4. \uc218\ud559 \uadc0\uc2e0\uc740 \uadf8 \uc790\ub9ac\uc5d0\uc11c \ub85c\ubca0\ub974\ud2b8\uc5d0\uac8c \uc790\uc2e0\uc774 \ud3c9\ubc94\ud55c \uc22b\uc790\ub77c\uace0 \ubd80\ub974\ub294 \uc790\uc5f0\uc218\uc640 \ubb34\ud55c\uc758 \ud2b9\uc9d5, \uadf8\ub9ac\uace0 \uae09\uc218\ub97c \uac00\ub974\uccd0\uc900\ub2e4. \uc5f4 \ubc88\uc9f8 \ubc24\uc5d0 \ub85c\ubca0\ub974\ud2b8\ub294 \ubd81\uadf9\uc810\uc5d0 \uc788\ub294 \uc790\uae30\ub97c \ubc1c\uacac\ud55c\ub2e4. \uadf8\uacf3\uc5d0\uc11c \uc218\ud559 \uadc0\uc2e0\uc5d0\uac8c \uc774\uce58\uc5d0 \uc5b4\uae0b\ub098\ub294 \uc218\uc778 \ubb34\ub9ac\uc218\uc640, \ub9e4\ub4ed(\uaf2d\uc9d3\uc810)\uacfc \uc120(\ubaa8\uc11c\ub9ac) \ub4f1\uc758 \uc720\ud074\ub9ac\ub4dc \uae30\ud558\ud559\uc758 \ud2b9\uc9d5\uc744 \ubc30\uc6b4\ub2e4. \uc5f4\ud55c \ubc88\uc9f8 \ubc24\uc5d0 \uc811\uc5b4 \ub4e4\uc5c8\uc744 \ub54c, \ub85c\ubca0\ub974\ud2b8\ub294 \uc218\ud559\uc5d0 \ub300\ud55c \ud765\ubbf8\uac00 \ub9ce\uc774 \uae4a\uc5b4\uc838 \uc788\uc5c8\ub2e4. \uc774 \ubc24\uc5d0\ub294 \uc218\ud559 \uadc0\uc2e0\uc5d0\uac8c \uc99d\uba85\uc5d0 \ub300\ud574\uc11c \ubc30\uc6b0\uba70, 1 \ub354\ud558\uae30 1\uacfc \uac19\uc740 \uac04\ub2e8\ud55c \uacc4\uc0b0 \ubb38\uc81c\uc758 \ubcf5\uc7a1\ud55c \uc99d\uba85 \uacfc\uc815\uc744 \ubcf4\uac8c \ub41c\ub2e4. \uc5f4\ub450 \ubc88\uc9f8 \ubc24\uc5d0, \ub85c\ubca0\ub974\ud2b8\uc640 \uc218\ud559 \uadc0\uc2e0\uc740 \uc218\ud559 \ucc9c\uad6d\uc73c\ub85c \ucd08\ub300\ubc1b\uc73c\uba70, \uc774\uacf3\uc5d0\uc11c \ub458\uc758 \ub9cc\ub0a8\uc740 \ub9c8\ubb34\ub9ac\ub41c\ub2e4. \uc218\ud559 \ucc9c\uad6d\uc5d0\uc11c \ub85c\ubca0\ub974\ud2b8\ub294 \uc0c1\uc0c1 \uc18d\uc5d0\uc11c\ub9cc \uc874\uc7ac\ud558\ub294 \uc218\uc778 \ud5c8\uc218\uc640 \ud074\ub77c\uc778 \ubcd1\uc744 \ubc30\uc6b4\ub2e4. \uc218\ud559 \ucc9c\uad6d\uc744 \uac70\ub2d0\uba74\uc11c \uc218\ud559\uadc0\uc2e0 \ud14c\ud50c\ub85c\ub2e5\uc2ac\uc740 \ud53c\ubcf4\ub098\uce58\ub098 \uce78\ud1a0\ub974\uc640 \uac19\uc740 \uc720\uba85\ud55c \uc218\ud559\uc790\ub4e4\uc744 \uc18c\uac1c\uc2dc\ucf1c \uc900\ub2e4. \uc774\ud6c4 \ub85c\ubca0\ub974\ud2b8\uac00 \ud559\uad50\uc5d0 \ub3cc\uc544\uac00\uc11c \uadf8\ub3d9\uc548 \ubc30\uc6e0\ub358 \ub0b4\uc6a9\uc73c\ub85c \uc218\ud559 \ubb38\uc81c\ub97c \uc27d\uac8c \ud478\ub294 \uac83\uc73c\ub85c \uc774 \ucc45\uc740 \ub05d\uc774 \ub09c\ub2e4.", "answer": "\uc5f4\ub450 \ubc88\uc9f8 \ubc24", "sentence": "\uc5f4\ub450 \ubc88\uc9f8 \ubc24 \uc5d0, \ub85c\ubca0\ub974\ud2b8\uc640 \uc218\ud559 \uadc0\uc2e0\uc740 \uc218\ud559 \ucc9c\uad6d\uc73c\ub85c \ucd08\ub300\ubc1b\uc73c\uba70, \uc774\uacf3\uc5d0\uc11c \ub458\uc758 \ub9cc\ub0a8\uc740 \ub9c8\ubb34\ub9ac\ub41c\ub2e4.", "paragraph_sentence": "\uc544\ud649 \ubc88\uc9f8 \ubc24\uc5d0, \ub85c\ubca0\ub974\ud2b8\ub294 \uafc8\uc5d0\uc11c \uac10\uae30\uc5d0 \uac78\ub824 \uce68\ub300\uc5d0 \ub204\uc6cc\uc788\uac8c \ub418\ub294\ub370 \uadf8\ub54c \uc218\ud559 \uadc0\uc2e0\uc774 \uc606\uc73c\ub85c \ub2e4\uac00\uc628\ub2e4. \uc218\ud559 \uadc0\uc2e0\uc740 \uadf8 \uc790\ub9ac\uc5d0\uc11c \ub85c\ubca0\ub974\ud2b8\uc5d0\uac8c \uc790\uc2e0\uc774 \ud3c9\ubc94\ud55c \uc22b\uc790\ub77c\uace0 \ubd80\ub974\ub294 \uc790\uc5f0\uc218\uc640 \ubb34\ud55c\uc758 \ud2b9\uc9d5, \uadf8\ub9ac\uace0 \uae09\uc218\ub97c \uac00\ub974\uccd0\uc900\ub2e4. \uc5f4 \ubc88\uc9f8 \ubc24\uc5d0 \ub85c\ubca0\ub974\ud2b8\ub294 \ubd81\uadf9\uc810\uc5d0 \uc788\ub294 \uc790\uae30\ub97c \ubc1c\uacac\ud55c\ub2e4. \uadf8\uacf3\uc5d0\uc11c \uc218\ud559 \uadc0\uc2e0\uc5d0\uac8c \uc774\uce58\uc5d0 \uc5b4\uae0b\ub098\ub294 \uc218\uc778 \ubb34\ub9ac\uc218\uc640, \ub9e4\ub4ed(\uaf2d\uc9d3\uc810)\uacfc \uc120(\ubaa8\uc11c\ub9ac) \ub4f1\uc758 \uc720\ud074\ub9ac\ub4dc \uae30\ud558\ud559\uc758 \ud2b9\uc9d5\uc744 \ubc30\uc6b4\ub2e4. \uc5f4\ud55c \ubc88\uc9f8 \ubc24\uc5d0 \uc811\uc5b4 \ub4e4\uc5c8\uc744 \ub54c, \ub85c\ubca0\ub974\ud2b8\ub294 \uc218\ud559\uc5d0 \ub300\ud55c \ud765\ubbf8\uac00 \ub9ce\uc774 \uae4a\uc5b4\uc838 \uc788\uc5c8\ub2e4. \uc774 \ubc24\uc5d0\ub294 \uc218\ud559 \uadc0\uc2e0\uc5d0\uac8c \uc99d\uba85\uc5d0 \ub300\ud574\uc11c \ubc30\uc6b0\uba70, 1 \ub354\ud558\uae30 1\uacfc \uac19\uc740 \uac04\ub2e8\ud55c \uacc4\uc0b0 \ubb38\uc81c\uc758 \ubcf5\uc7a1\ud55c \uc99d\uba85 \uacfc\uc815\uc744 \ubcf4\uac8c \ub41c\ub2e4. \uc5f4\ub450 \ubc88\uc9f8 \ubc24 \uc5d0, \ub85c\ubca0\ub974\ud2b8\uc640 \uc218\ud559 \uadc0\uc2e0\uc740 \uc218\ud559 \ucc9c\uad6d\uc73c\ub85c \ucd08\ub300\ubc1b\uc73c\uba70, \uc774\uacf3\uc5d0\uc11c \ub458\uc758 \ub9cc\ub0a8\uc740 \ub9c8\ubb34\ub9ac\ub41c\ub2e4. \uc218\ud559 \ucc9c\uad6d\uc5d0\uc11c \ub85c\ubca0\ub974\ud2b8\ub294 \uc0c1\uc0c1 \uc18d\uc5d0\uc11c\ub9cc \uc874\uc7ac\ud558\ub294 \uc218\uc778 \ud5c8\uc218\uc640 \ud074\ub77c\uc778 \ubcd1\uc744 \ubc30\uc6b4\ub2e4. \uc218\ud559 \ucc9c\uad6d\uc744 \uac70\ub2d0\uba74\uc11c \uc218\ud559\uadc0\uc2e0 \ud14c\ud50c\ub85c\ub2e5\uc2ac\uc740 \ud53c\ubcf4\ub098\uce58\ub098 \uce78\ud1a0\ub974\uc640 \uac19\uc740 \uc720\uba85\ud55c \uc218\ud559\uc790\ub4e4\uc744 \uc18c\uac1c\uc2dc\ucf1c \uc900\ub2e4. \uc774\ud6c4 \ub85c\ubca0\ub974\ud2b8\uac00 \ud559\uad50\uc5d0 \ub3cc\uc544\uac00\uc11c \uadf8\ub3d9\uc548 \ubc30\uc6e0\ub358 \ub0b4\uc6a9\uc73c\ub85c \uc218\ud559 \ubb38\uc81c\ub97c \uc27d\uac8c \ud478\ub294 \uac83\uc73c\ub85c \uc774 \ucc45\uc740 \ub05d\uc774 \ub09c\ub2e4.", "paragraph_answer": "\uc544\ud649 \ubc88\uc9f8 \ubc24\uc5d0, \ub85c\ubca0\ub974\ud2b8\ub294 \uafc8\uc5d0\uc11c \uac10\uae30\uc5d0 \uac78\ub824 \uce68\ub300\uc5d0 \ub204\uc6cc\uc788\uac8c \ub418\ub294\ub370 \uadf8\ub54c \uc218\ud559 \uadc0\uc2e0\uc774 \uc606\uc73c\ub85c \ub2e4\uac00\uc628\ub2e4. \uc218\ud559 \uadc0\uc2e0\uc740 \uadf8 \uc790\ub9ac\uc5d0\uc11c \ub85c\ubca0\ub974\ud2b8\uc5d0\uac8c \uc790\uc2e0\uc774 \ud3c9\ubc94\ud55c \uc22b\uc790\ub77c\uace0 \ubd80\ub974\ub294 \uc790\uc5f0\uc218\uc640 \ubb34\ud55c\uc758 \ud2b9\uc9d5, \uadf8\ub9ac\uace0 \uae09\uc218\ub97c \uac00\ub974\uccd0\uc900\ub2e4. \uc5f4 \ubc88\uc9f8 \ubc24\uc5d0 \ub85c\ubca0\ub974\ud2b8\ub294 \ubd81\uadf9\uc810\uc5d0 \uc788\ub294 \uc790\uae30\ub97c \ubc1c\uacac\ud55c\ub2e4. \uadf8\uacf3\uc5d0\uc11c \uc218\ud559 \uadc0\uc2e0\uc5d0\uac8c \uc774\uce58\uc5d0 \uc5b4\uae0b\ub098\ub294 \uc218\uc778 \ubb34\ub9ac\uc218\uc640, \ub9e4\ub4ed(\uaf2d\uc9d3\uc810)\uacfc \uc120(\ubaa8\uc11c\ub9ac) \ub4f1\uc758 \uc720\ud074\ub9ac\ub4dc \uae30\ud558\ud559\uc758 \ud2b9\uc9d5\uc744 \ubc30\uc6b4\ub2e4. \uc5f4\ud55c \ubc88\uc9f8 \ubc24\uc5d0 \uc811\uc5b4 \ub4e4\uc5c8\uc744 \ub54c, \ub85c\ubca0\ub974\ud2b8\ub294 \uc218\ud559\uc5d0 \ub300\ud55c \ud765\ubbf8\uac00 \ub9ce\uc774 \uae4a\uc5b4\uc838 \uc788\uc5c8\ub2e4. \uc774 \ubc24\uc5d0\ub294 \uc218\ud559 \uadc0\uc2e0\uc5d0\uac8c \uc99d\uba85\uc5d0 \ub300\ud574\uc11c \ubc30\uc6b0\uba70, 1 \ub354\ud558\uae30 1\uacfc \uac19\uc740 \uac04\ub2e8\ud55c \uacc4\uc0b0 \ubb38\uc81c\uc758 \ubcf5\uc7a1\ud55c \uc99d\uba85 \uacfc\uc815\uc744 \ubcf4\uac8c \ub41c\ub2e4. \uc5f4\ub450 \ubc88\uc9f8 \ubc24 \uc5d0, \ub85c\ubca0\ub974\ud2b8\uc640 \uc218\ud559 \uadc0\uc2e0\uc740 \uc218\ud559 \ucc9c\uad6d\uc73c\ub85c \ucd08\ub300\ubc1b\uc73c\uba70, \uc774\uacf3\uc5d0\uc11c \ub458\uc758 \ub9cc\ub0a8\uc740 \ub9c8\ubb34\ub9ac\ub41c\ub2e4. \uc218\ud559 \ucc9c\uad6d\uc5d0\uc11c \ub85c\ubca0\ub974\ud2b8\ub294 \uc0c1\uc0c1 \uc18d\uc5d0\uc11c\ub9cc \uc874\uc7ac\ud558\ub294 \uc218\uc778 \ud5c8\uc218\uc640 \ud074\ub77c\uc778 \ubcd1\uc744 \ubc30\uc6b4\ub2e4. \uc218\ud559 \ucc9c\uad6d\uc744 \uac70\ub2d0\uba74\uc11c \uc218\ud559\uadc0\uc2e0 \ud14c\ud50c\ub85c\ub2e5\uc2ac\uc740 \ud53c\ubcf4\ub098\uce58\ub098 \uce78\ud1a0\ub974\uc640 \uac19\uc740 \uc720\uba85\ud55c \uc218\ud559\uc790\ub4e4\uc744 \uc18c\uac1c\uc2dc\ucf1c \uc900\ub2e4. \uc774\ud6c4 \ub85c\ubca0\ub974\ud2b8\uac00 \ud559\uad50\uc5d0 \ub3cc\uc544\uac00\uc11c \uadf8\ub3d9\uc548 \ubc30\uc6e0\ub358 \ub0b4\uc6a9\uc73c\ub85c \uc218\ud559 \ubb38\uc81c\ub97c \uc27d\uac8c \ud478\ub294 \uac83\uc73c\ub85c \uc774 \ucc45\uc740 \ub05d\uc774 \ub09c\ub2e4.", "sentence_answer": " \uc5f4\ub450 \ubc88\uc9f8 \ubc24 \uc5d0, \ub85c\ubca0\ub974\ud2b8\uc640 \uc218\ud559 \uadc0\uc2e0\uc740 \uc218\ud559 \ucc9c\uad6d\uc73c\ub85c \ucd08\ub300\ubc1b\uc73c\uba70, \uc774\uacf3\uc5d0\uc11c \ub458\uc758 \ub9cc\ub0a8\uc740 \ub9c8\ubb34\ub9ac\ub41c\ub2e4.", "paragraph_id": "6518864-1-2", "paragraph_question": "question: \ub85c\ubca0\ub974\ud2b8\uc640 \uc218\ud559 \uadc0\uc2e0\uc758 \ub9cc\ub0a8\uc774 \ub9c8\ubb34\ub9ac \ub41c \ubc24\uc740 \uba87 \ubc88\uc9f8 \ubc24\uc778\uac00?, context: \uc544\ud649 \ubc88\uc9f8 \ubc24\uc5d0, \ub85c\ubca0\ub974\ud2b8\ub294 \uafc8\uc5d0\uc11c \uac10\uae30\uc5d0 \uac78\ub824 \uce68\ub300\uc5d0 \ub204\uc6cc\uc788\uac8c \ub418\ub294\ub370 \uadf8\ub54c \uc218\ud559 \uadc0\uc2e0\uc774 \uc606\uc73c\ub85c \ub2e4\uac00\uc628\ub2e4. \uc218\ud559 \uadc0\uc2e0\uc740 \uadf8 \uc790\ub9ac\uc5d0\uc11c \ub85c\ubca0\ub974\ud2b8\uc5d0\uac8c \uc790\uc2e0\uc774 \ud3c9\ubc94\ud55c \uc22b\uc790\ub77c\uace0 \ubd80\ub974\ub294 \uc790\uc5f0\uc218\uc640 \ubb34\ud55c\uc758 \ud2b9\uc9d5, \uadf8\ub9ac\uace0 \uae09\uc218\ub97c \uac00\ub974\uccd0\uc900\ub2e4. \uc5f4 \ubc88\uc9f8 \ubc24\uc5d0 \ub85c\ubca0\ub974\ud2b8\ub294 \ubd81\uadf9\uc810\uc5d0 \uc788\ub294 \uc790\uae30\ub97c \ubc1c\uacac\ud55c\ub2e4. \uadf8\uacf3\uc5d0\uc11c \uc218\ud559 \uadc0\uc2e0\uc5d0\uac8c \uc774\uce58\uc5d0 \uc5b4\uae0b\ub098\ub294 \uc218\uc778 \ubb34\ub9ac\uc218\uc640, \ub9e4\ub4ed(\uaf2d\uc9d3\uc810)\uacfc \uc120(\ubaa8\uc11c\ub9ac) \ub4f1\uc758 \uc720\ud074\ub9ac\ub4dc \uae30\ud558\ud559\uc758 \ud2b9\uc9d5\uc744 \ubc30\uc6b4\ub2e4. \uc5f4\ud55c \ubc88\uc9f8 \ubc24\uc5d0 \uc811\uc5b4 \ub4e4\uc5c8\uc744 \ub54c, \ub85c\ubca0\ub974\ud2b8\ub294 \uc218\ud559\uc5d0 \ub300\ud55c \ud765\ubbf8\uac00 \ub9ce\uc774 \uae4a\uc5b4\uc838 \uc788\uc5c8\ub2e4. \uc774 \ubc24\uc5d0\ub294 \uc218\ud559 \uadc0\uc2e0\uc5d0\uac8c \uc99d\uba85\uc5d0 \ub300\ud574\uc11c \ubc30\uc6b0\uba70, 1 \ub354\ud558\uae30 1\uacfc \uac19\uc740 \uac04\ub2e8\ud55c \uacc4\uc0b0 \ubb38\uc81c\uc758 \ubcf5\uc7a1\ud55c \uc99d\uba85 \uacfc\uc815\uc744 \ubcf4\uac8c \ub41c\ub2e4. \uc5f4\ub450 \ubc88\uc9f8 \ubc24\uc5d0, \ub85c\ubca0\ub974\ud2b8\uc640 \uc218\ud559 \uadc0\uc2e0\uc740 \uc218\ud559 \ucc9c\uad6d\uc73c\ub85c \ucd08\ub300\ubc1b\uc73c\uba70, \uc774\uacf3\uc5d0\uc11c \ub458\uc758 \ub9cc\ub0a8\uc740 \ub9c8\ubb34\ub9ac\ub41c\ub2e4. \uc218\ud559 \ucc9c\uad6d\uc5d0\uc11c \ub85c\ubca0\ub974\ud2b8\ub294 \uc0c1\uc0c1 \uc18d\uc5d0\uc11c\ub9cc \uc874\uc7ac\ud558\ub294 \uc218\uc778 \ud5c8\uc218\uc640 \ud074\ub77c\uc778 \ubcd1\uc744 \ubc30\uc6b4\ub2e4. \uc218\ud559 \ucc9c\uad6d\uc744 \uac70\ub2d0\uba74\uc11c \uc218\ud559\uadc0\uc2e0 \ud14c\ud50c\ub85c\ub2e5\uc2ac\uc740 \ud53c\ubcf4\ub098\uce58\ub098 \uce78\ud1a0\ub974\uc640 \uac19\uc740 \uc720\uba85\ud55c \uc218\ud559\uc790\ub4e4\uc744 \uc18c\uac1c\uc2dc\ucf1c \uc900\ub2e4. \uc774\ud6c4 \ub85c\ubca0\ub974\ud2b8\uac00 \ud559\uad50\uc5d0 \ub3cc\uc544\uac00\uc11c \uadf8\ub3d9\uc548 \ubc30\uc6e0\ub358 \ub0b4\uc6a9\uc73c\ub85c \uc218\ud559 \ubb38\uc81c\ub97c \uc27d\uac8c \ud478\ub294 \uac83\uc73c\ub85c \uc774 \ucc45\uc740 \ub05d\uc774 \ub09c\ub2e4."} {"question": "\ubba4\uc9c0\uceec \ube4c\ub9ac \uc5d8\ub9ac\uc5b4\ud2b8\uac00 \uc2dc\uce74\uace0\uc758 \uacf5\uc5f0\uc744 \ub05d\ub098\uace0 \ub2e4\uc74c\uc73c\ub85c \uacf5\uc5f0\uc744 \ud55c \uacf3\uc740?", "paragraph": "\ubba4\uc9c0\uceec \ube4c\ub9ac \uc5d8\ub9ac\uc5b4\ud2b8\ub294 \uc2dc\uce74\uace0\uc5d0\uc11c 2010\ub144 3\uc6d4 18\uc77c \ud504\ub9ac\ubdf0\uc640 \ud568\uaed8 4\uc6d4 11\uc77c \ud3ec\ub4dc \uacf5\uc5f0 \uc608\uc220 \uc13c\ud130\uc5d0\uc11c \uacf5\uc2dd\uc801\uc73c\ub85c \ub9c9\uc744 \uc62c\ub838\ub2e4. \ud3ec\ub4dc \uacf5\uc5f0 \uc608\uc220 \uc13c\ud130\ub294 \ubd81\ubbf8 \ud22c\uc5b4 \uacf5\uc5f0\uc758 \uc2dc\uc791\uc774\uc5c8\ub2e4. \uadf8\ub7ec\ub098 \ud504\ub85c\ub4c0\uc11c \uc5d0\ub9ad \ud398\ub108\uc5d0 \uc758\ud558\uba74, \uadf8\ub294 \ubba4\uc9c0\uceec \ube4c\ub9ac \uc5d8\ub9ac\uc5b4\ud2b8\ub294 \"\uc2dc\uce74\uace0\uac00 '\uac00\ub77c'\ub77c\uace0 \ud558\uae30 \uc804\uae4c\uc9c0 \uc788\uc744 \uac83\uc774\ub2e4. \uc6b0\ub9ac\ub294 \ud55c \ubc88\uc5d0 \ud55c \uac00\uc9c0\ub9cc \ub9e1\uc744 \uc218 \uc788\ub2e4.\"\uace0 \ub9d0\ud588\ub2e4. \uadf8\ub7ec\ud558\uc5ec \ub2e4\ub978 \ubd81\ubbf8 \ub3c4\uc2dc\ub4e4\uc740 \uc2dc\uce74\uace0 \ud504\ub85c\ub355\uc158\uc774 \uacbd\uacfc\ub97c \uc9c0\ucf1c \ubcfc \ub3d9\uc548 \uae30\ub2e4\ub9b4 \uc218 \ubc16\uc5d0 \uc5c6\uc5c8\ub2e4. \uc5d0\ubc00\ub9ac \uc2a4\ud0a4\ub108\ub294 \uc70c\ud0a8\uc2a8 \ubd80\uc778\uc5ed\uc744 \ub9e1\uc558\uace0, \ud1a0\ubbf8 \ubca0\ucc4c\ub7ec, \uc9c0\uc138\ubeec \ubc14\uc2e4\ub9ac\uc624, \uc2dc\uc800 \ucf54\ub784\ub808\uc988, \uadf8\ub9ac\uace0 \uc874 \ud53c\ud130 \ube44\uc5d0\ub974\ub124\uac00 \ube4c\ub9ac\ub97c \uc5f0\uae30\ud588\ub2e4. \uc774\ud6c4 \ub9c8\ub974\ucfe0\uc2a4 \ud398\uc774, \ub9c8\uc77c\uc988 \uc5bc\ub9ad\uc774 \ube4c\ub9ac \uc5d8\ub9ac\uc5b4\ud2b8 \uc5ed\uc5d0 \ud569\uc138\ud588\ub2e4. \uc2dc\uce74\uace0 \ud504\ub85c\ub355\uc158\uc740 2010\ub144 11\uc6d4 28\uc77c\ub85c \uacf5\uc5f0\uc744 \ub05d\ub0c8\uace0, \ud1a0\ub860\ud1a0\ub85c \uc62e\uaca8\uac00\uac8c \ub41c\ub2e4. 37\uc8fc\ub3d9\uc548 288\ud68c\uc758 \uacf5\uc5f0\uc744 \ud588\ub2e4.", "answer": "\ud1a0\ub860\ud1a0", "sentence": "\uc2dc\uce74\uace0 \ud504\ub85c\ub355\uc158\uc740 2010\ub144 11\uc6d4 28\uc77c\ub85c \uacf5\uc5f0\uc744 \ub05d\ub0c8\uace0, \ud1a0\ub860\ud1a0 \ub85c \uc62e\uaca8\uac00\uac8c \ub41c\ub2e4.", "paragraph_sentence": "\ubba4\uc9c0\uceec \ube4c\ub9ac \uc5d8\ub9ac\uc5b4\ud2b8\ub294 \uc2dc\uce74\uace0\uc5d0\uc11c 2010\ub144 3\uc6d4 18\uc77c \ud504\ub9ac\ubdf0\uc640 \ud568\uaed8 4\uc6d4 11\uc77c \ud3ec\ub4dc \uacf5\uc5f0 \uc608\uc220 \uc13c\ud130\uc5d0\uc11c \uacf5\uc2dd\uc801\uc73c\ub85c \ub9c9\uc744 \uc62c\ub838\ub2e4. \ud3ec\ub4dc \uacf5\uc5f0 \uc608\uc220 \uc13c\ud130\ub294 \ubd81\ubbf8 \ud22c\uc5b4 \uacf5\uc5f0\uc758 \uc2dc\uc791\uc774\uc5c8\ub2e4. \uadf8\ub7ec\ub098 \ud504\ub85c\ub4c0\uc11c \uc5d0\ub9ad \ud398\ub108\uc5d0 \uc758\ud558\uba74, \uadf8\ub294 \ubba4\uc9c0\uceec \ube4c\ub9ac \uc5d8\ub9ac\uc5b4\ud2b8\ub294 \"\uc2dc\uce74\uace0\uac00 '\uac00\ub77c'\ub77c\uace0 \ud558\uae30 \uc804\uae4c\uc9c0 \uc788\uc744 \uac83\uc774\ub2e4. \uc6b0\ub9ac\ub294 \ud55c \ubc88\uc5d0 \ud55c \uac00\uc9c0\ub9cc \ub9e1\uc744 \uc218 \uc788\ub2e4. \"\uace0 \ub9d0\ud588\ub2e4. \uadf8\ub7ec\ud558\uc5ec \ub2e4\ub978 \ubd81\ubbf8 \ub3c4\uc2dc\ub4e4\uc740 \uc2dc\uce74\uace0 \ud504\ub85c\ub355\uc158\uc774 \uacbd\uacfc\ub97c \uc9c0\ucf1c \ubcfc \ub3d9\uc548 \uae30\ub2e4\ub9b4 \uc218 \ubc16\uc5d0 \uc5c6\uc5c8\ub2e4. \uc5d0\ubc00\ub9ac \uc2a4\ud0a4\ub108\ub294 \uc70c\ud0a8\uc2a8 \ubd80\uc778\uc5ed\uc744 \ub9e1\uc558\uace0, \ud1a0\ubbf8 \ubca0\ucc4c\ub7ec, \uc9c0\uc138\ubeec \ubc14\uc2e4\ub9ac\uc624, \uc2dc\uc800 \ucf54\ub784\ub808\uc988, \uadf8\ub9ac\uace0 \uc874 \ud53c\ud130 \ube44\uc5d0\ub974\ub124\uac00 \ube4c\ub9ac\ub97c \uc5f0\uae30\ud588\ub2e4. \uc774\ud6c4 \ub9c8\ub974\ucfe0\uc2a4 \ud398\uc774, \ub9c8\uc77c\uc988 \uc5bc\ub9ad\uc774 \ube4c\ub9ac \uc5d8\ub9ac\uc5b4\ud2b8 \uc5ed\uc5d0 \ud569\uc138\ud588\ub2e4. \uc2dc\uce74\uace0 \ud504\ub85c\ub355\uc158\uc740 2010\ub144 11\uc6d4 28\uc77c\ub85c \uacf5\uc5f0\uc744 \ub05d\ub0c8\uace0, \ud1a0\ub860\ud1a0 \ub85c \uc62e\uaca8\uac00\uac8c \ub41c\ub2e4. 37\uc8fc\ub3d9\uc548 288\ud68c\uc758 \uacf5\uc5f0\uc744 \ud588\ub2e4.", "paragraph_answer": "\ubba4\uc9c0\uceec \ube4c\ub9ac \uc5d8\ub9ac\uc5b4\ud2b8\ub294 \uc2dc\uce74\uace0\uc5d0\uc11c 2010\ub144 3\uc6d4 18\uc77c \ud504\ub9ac\ubdf0\uc640 \ud568\uaed8 4\uc6d4 11\uc77c \ud3ec\ub4dc \uacf5\uc5f0 \uc608\uc220 \uc13c\ud130\uc5d0\uc11c \uacf5\uc2dd\uc801\uc73c\ub85c \ub9c9\uc744 \uc62c\ub838\ub2e4. \ud3ec\ub4dc \uacf5\uc5f0 \uc608\uc220 \uc13c\ud130\ub294 \ubd81\ubbf8 \ud22c\uc5b4 \uacf5\uc5f0\uc758 \uc2dc\uc791\uc774\uc5c8\ub2e4. \uadf8\ub7ec\ub098 \ud504\ub85c\ub4c0\uc11c \uc5d0\ub9ad \ud398\ub108\uc5d0 \uc758\ud558\uba74, \uadf8\ub294 \ubba4\uc9c0\uceec \ube4c\ub9ac \uc5d8\ub9ac\uc5b4\ud2b8\ub294 \"\uc2dc\uce74\uace0\uac00 '\uac00\ub77c'\ub77c\uace0 \ud558\uae30 \uc804\uae4c\uc9c0 \uc788\uc744 \uac83\uc774\ub2e4. \uc6b0\ub9ac\ub294 \ud55c \ubc88\uc5d0 \ud55c \uac00\uc9c0\ub9cc \ub9e1\uc744 \uc218 \uc788\ub2e4.\"\uace0 \ub9d0\ud588\ub2e4. \uadf8\ub7ec\ud558\uc5ec \ub2e4\ub978 \ubd81\ubbf8 \ub3c4\uc2dc\ub4e4\uc740 \uc2dc\uce74\uace0 \ud504\ub85c\ub355\uc158\uc774 \uacbd\uacfc\ub97c \uc9c0\ucf1c \ubcfc \ub3d9\uc548 \uae30\ub2e4\ub9b4 \uc218 \ubc16\uc5d0 \uc5c6\uc5c8\ub2e4. \uc5d0\ubc00\ub9ac \uc2a4\ud0a4\ub108\ub294 \uc70c\ud0a8\uc2a8 \ubd80\uc778\uc5ed\uc744 \ub9e1\uc558\uace0, \ud1a0\ubbf8 \ubca0\ucc4c\ub7ec, \uc9c0\uc138\ubeec \ubc14\uc2e4\ub9ac\uc624, \uc2dc\uc800 \ucf54\ub784\ub808\uc988, \uadf8\ub9ac\uace0 \uc874 \ud53c\ud130 \ube44\uc5d0\ub974\ub124\uac00 \ube4c\ub9ac\ub97c \uc5f0\uae30\ud588\ub2e4. \uc774\ud6c4 \ub9c8\ub974\ucfe0\uc2a4 \ud398\uc774, \ub9c8\uc77c\uc988 \uc5bc\ub9ad\uc774 \ube4c\ub9ac \uc5d8\ub9ac\uc5b4\ud2b8 \uc5ed\uc5d0 \ud569\uc138\ud588\ub2e4. \uc2dc\uce74\uace0 \ud504\ub85c\ub355\uc158\uc740 2010\ub144 11\uc6d4 28\uc77c\ub85c \uacf5\uc5f0\uc744 \ub05d\ub0c8\uace0, \ud1a0\ub860\ud1a0 \ub85c \uc62e\uaca8\uac00\uac8c \ub41c\ub2e4. 37\uc8fc\ub3d9\uc548 288\ud68c\uc758 \uacf5\uc5f0\uc744 \ud588\ub2e4.", "sentence_answer": "\uc2dc\uce74\uace0 \ud504\ub85c\ub355\uc158\uc740 2010\ub144 11\uc6d4 28\uc77c\ub85c \uacf5\uc5f0\uc744 \ub05d\ub0c8\uace0, \ud1a0\ub860\ud1a0 \ub85c \uc62e\uaca8\uac00\uac8c \ub41c\ub2e4.", "paragraph_id": "6129078-3-1", "paragraph_question": "question: \ubba4\uc9c0\uceec \ube4c\ub9ac \uc5d8\ub9ac\uc5b4\ud2b8\uac00 \uc2dc\uce74\uace0\uc758 \uacf5\uc5f0\uc744 \ub05d\ub098\uace0 \ub2e4\uc74c\uc73c\ub85c \uacf5\uc5f0\uc744 \ud55c \uacf3\uc740?, context: \ubba4\uc9c0\uceec \ube4c\ub9ac \uc5d8\ub9ac\uc5b4\ud2b8\ub294 \uc2dc\uce74\uace0\uc5d0\uc11c 2010\ub144 3\uc6d4 18\uc77c \ud504\ub9ac\ubdf0\uc640 \ud568\uaed8 4\uc6d4 11\uc77c \ud3ec\ub4dc \uacf5\uc5f0 \uc608\uc220 \uc13c\ud130\uc5d0\uc11c \uacf5\uc2dd\uc801\uc73c\ub85c \ub9c9\uc744 \uc62c\ub838\ub2e4. \ud3ec\ub4dc \uacf5\uc5f0 \uc608\uc220 \uc13c\ud130\ub294 \ubd81\ubbf8 \ud22c\uc5b4 \uacf5\uc5f0\uc758 \uc2dc\uc791\uc774\uc5c8\ub2e4. \uadf8\ub7ec\ub098 \ud504\ub85c\ub4c0\uc11c \uc5d0\ub9ad \ud398\ub108\uc5d0 \uc758\ud558\uba74, \uadf8\ub294 \ubba4\uc9c0\uceec \ube4c\ub9ac \uc5d8\ub9ac\uc5b4\ud2b8\ub294 \"\uc2dc\uce74\uace0\uac00 '\uac00\ub77c'\ub77c\uace0 \ud558\uae30 \uc804\uae4c\uc9c0 \uc788\uc744 \uac83\uc774\ub2e4. \uc6b0\ub9ac\ub294 \ud55c \ubc88\uc5d0 \ud55c \uac00\uc9c0\ub9cc \ub9e1\uc744 \uc218 \uc788\ub2e4.\"\uace0 \ub9d0\ud588\ub2e4. \uadf8\ub7ec\ud558\uc5ec \ub2e4\ub978 \ubd81\ubbf8 \ub3c4\uc2dc\ub4e4\uc740 \uc2dc\uce74\uace0 \ud504\ub85c\ub355\uc158\uc774 \uacbd\uacfc\ub97c \uc9c0\ucf1c \ubcfc \ub3d9\uc548 \uae30\ub2e4\ub9b4 \uc218 \ubc16\uc5d0 \uc5c6\uc5c8\ub2e4. \uc5d0\ubc00\ub9ac \uc2a4\ud0a4\ub108\ub294 \uc70c\ud0a8\uc2a8 \ubd80\uc778\uc5ed\uc744 \ub9e1\uc558\uace0, \ud1a0\ubbf8 \ubca0\ucc4c\ub7ec, \uc9c0\uc138\ubeec \ubc14\uc2e4\ub9ac\uc624, \uc2dc\uc800 \ucf54\ub784\ub808\uc988, \uadf8\ub9ac\uace0 \uc874 \ud53c\ud130 \ube44\uc5d0\ub974\ub124\uac00 \ube4c\ub9ac\ub97c \uc5f0\uae30\ud588\ub2e4. \uc774\ud6c4 \ub9c8\ub974\ucfe0\uc2a4 \ud398\uc774, \ub9c8\uc77c\uc988 \uc5bc\ub9ad\uc774 \ube4c\ub9ac \uc5d8\ub9ac\uc5b4\ud2b8 \uc5ed\uc5d0 \ud569\uc138\ud588\ub2e4. \uc2dc\uce74\uace0 \ud504\ub85c\ub355\uc158\uc740 2010\ub144 11\uc6d4 28\uc77c\ub85c \uacf5\uc5f0\uc744 \ub05d\ub0c8\uace0, \ud1a0\ub860\ud1a0\ub85c \uc62e\uaca8\uac00\uac8c \ub41c\ub2e4. 37\uc8fc\ub3d9\uc548 288\ud68c\uc758 \uacf5\uc5f0\uc744 \ud588\ub2e4."} {"question": "FC \ubc14\ub974\uc140\ub85c\ub098 B\ud300\uc73c\ub85c \ubd88\ub9ac\uae30 \uc2dc\uc791\ub41c \uc2dc\uae30\ub294?", "paragraph": "FC \ubc14\ub974\uc140\ub85c\ub098 B\ub294 \uadf8\ub4e4\uc758 \uc2dc\ub2c8\uc5b4 \ud300\uc778 FC \ubc14\ub974\uc140\ub85c\ub098\uc640\ub294 \uad6c\ubcc4\ub418\ub294 \uc5ed\uc0ac\ub97c \uac00\uc9c0\uace0 \uc788\uc73c\uba70, 1990\ub144\ub300\uac00 \ub418\uc5b4\uc11c\uc57c FC \ubc14\ub974\uc140\ub85c\ub098 B \ud300\uc774 \ub418\uc5c8\ub2e4. \uadf8 \uc774\uc804\uc5d0\ub294 'CD \uc5d0\uc2a4\ud30c\ub0d0 \uc778\ub450\uc2a4\ud2b8\ub9ac\uc54c', 'CD \ucf58\ub2ec', '\ubc14\ub974\uc140\ub85c\ub098 \uc544\ud2c0\ub808\ud2f1' \ub4f1\uc73c\ub85c \ubd88\ub9ac\ub294 \ubcc4\uac1c\uc758 \ud074\ub7fd\uc774\uc5c8\ub2e4. 1993\ub144\ubd80\ud130 2007\ub144\uae4c\uc9c0\ub294 FC \ubc14\ub974\uc140\ub85c\ub098 C \ud300\ub3c4 \uc874\uc7ac\ud588\ub294\ub370, \uc774 \ud300\uc740 1967\ub144\uc5d0 \ucc3d\ub2e8\ud55c '\ubc14\ub974\uc140\ub85c\ub098 \uc544\ub9c8\ucd94\uc5b4'\ud300\uc774 \uadf8 \uc804\uc2e0\uc774\uc5c8\ub2e4. FC \ubc14\ub974\uc140\ub85c\ub098\ub294 \uc804\ud1b5\uc801\uc73c\ub85c \uc804 \uc138\uacc4\uc801\uc778 \uad6d\uc801\uc758 \uc120\uc218\ub4e4\uc744 \ubcf4\uc720\ud55c \ud300\uc73c\ub85c\ub3c4 \ub110\ub9ac \uc54c\ub824\uc838 \uc788\uc9c0\ub9cc, \uce78\ud14c\ub77c(\uc720\uc2a4 \uc2dc\uc2a4\ud15c)\ubd80\ud130 \ub6f0\uc5b4\ub09c \uc120\uc218\ub4e4\uc744 \uae38\ub7ec\ub0b4\ub294 \uc5ed\uc0ac\ub85c\ub3c4 \uc720\uba85\ud558\ub2e4. \uc218\ub9ce\uc740 \uc131\uc778 \uc120\uc218\ub4e4\uc774 \uc774\ub7ec\ud55c \uc720\uc18c\ub144 \ud300\uc744 \uac70\uccd0\uc11c \uc131\uc7a5\ud558\uc600\ub2e4.", "answer": "1990\ub144\ub300", "sentence": "FC \ubc14\ub974\uc140\ub85c\ub098 B\ub294 \uadf8\ub4e4\uc758 \uc2dc\ub2c8\uc5b4 \ud300\uc778 FC \ubc14\ub974\uc140\ub85c\ub098\uc640\ub294 \uad6c\ubcc4\ub418\ub294 \uc5ed\uc0ac\ub97c \uac00\uc9c0\uace0 \uc788\uc73c\uba70, 1990\ub144\ub300 \uac00 \ub418\uc5b4\uc11c\uc57c FC \ubc14\ub974\uc140\ub85c\ub098 B \ud300\uc774 \ub418\uc5c8\ub2e4.", "paragraph_sentence": " FC \ubc14\ub974\uc140\ub85c\ub098 B\ub294 \uadf8\ub4e4\uc758 \uc2dc\ub2c8\uc5b4 \ud300\uc778 FC \ubc14\ub974\uc140\ub85c\ub098\uc640\ub294 \uad6c\ubcc4\ub418\ub294 \uc5ed\uc0ac\ub97c \uac00\uc9c0\uace0 \uc788\uc73c\uba70, 1990\ub144\ub300 \uac00 \ub418\uc5b4\uc11c\uc57c FC \ubc14\ub974\uc140\ub85c\ub098 B \ud300\uc774 \ub418\uc5c8\ub2e4. \uadf8 \uc774\uc804\uc5d0\ub294 'CD \uc5d0\uc2a4\ud30c\ub0d0 \uc778\ub450\uc2a4\ud2b8\ub9ac\uc54c', 'CD \ucf58\ub2ec', '\ubc14\ub974\uc140\ub85c\ub098 \uc544\ud2c0\ub808\ud2f1' \ub4f1\uc73c\ub85c \ubd88\ub9ac\ub294 \ubcc4\uac1c\uc758 \ud074\ub7fd\uc774\uc5c8\ub2e4. 1993\ub144\ubd80\ud130 2007\ub144\uae4c\uc9c0\ub294 FC \ubc14\ub974\uc140\ub85c\ub098 C \ud300\ub3c4 \uc874\uc7ac\ud588\ub294\ub370, \uc774 \ud300\uc740 1967\ub144\uc5d0 \ucc3d\ub2e8\ud55c '\ubc14\ub974\uc140\ub85c\ub098 \uc544\ub9c8\ucd94\uc5b4'\ud300\uc774 \uadf8 \uc804\uc2e0\uc774\uc5c8\ub2e4. FC \ubc14\ub974\uc140\ub85c\ub098\ub294 \uc804\ud1b5\uc801\uc73c\ub85c \uc804 \uc138\uacc4\uc801\uc778 \uad6d\uc801\uc758 \uc120\uc218\ub4e4\uc744 \ubcf4\uc720\ud55c \ud300\uc73c\ub85c\ub3c4 \ub110\ub9ac \uc54c\ub824\uc838 \uc788\uc9c0\ub9cc, \uce78\ud14c\ub77c(\uc720\uc2a4 \uc2dc\uc2a4\ud15c)\ubd80\ud130 \ub6f0\uc5b4\ub09c \uc120\uc218\ub4e4\uc744 \uae38\ub7ec\ub0b4\ub294 \uc5ed\uc0ac\ub85c\ub3c4 \uc720\uba85\ud558\ub2e4. \uc218\ub9ce\uc740 \uc131\uc778 \uc120\uc218\ub4e4\uc774 \uc774\ub7ec\ud55c \uc720\uc18c\ub144 \ud300\uc744 \uac70\uccd0\uc11c \uc131\uc7a5\ud558\uc600\ub2e4.", "paragraph_answer": "FC \ubc14\ub974\uc140\ub85c\ub098 B\ub294 \uadf8\ub4e4\uc758 \uc2dc\ub2c8\uc5b4 \ud300\uc778 FC \ubc14\ub974\uc140\ub85c\ub098\uc640\ub294 \uad6c\ubcc4\ub418\ub294 \uc5ed\uc0ac\ub97c \uac00\uc9c0\uace0 \uc788\uc73c\uba70, 1990\ub144\ub300 \uac00 \ub418\uc5b4\uc11c\uc57c FC \ubc14\ub974\uc140\ub85c\ub098 B \ud300\uc774 \ub418\uc5c8\ub2e4. \uadf8 \uc774\uc804\uc5d0\ub294 'CD \uc5d0\uc2a4\ud30c\ub0d0 \uc778\ub450\uc2a4\ud2b8\ub9ac\uc54c', 'CD \ucf58\ub2ec', '\ubc14\ub974\uc140\ub85c\ub098 \uc544\ud2c0\ub808\ud2f1' \ub4f1\uc73c\ub85c \ubd88\ub9ac\ub294 \ubcc4\uac1c\uc758 \ud074\ub7fd\uc774\uc5c8\ub2e4. 1993\ub144\ubd80\ud130 2007\ub144\uae4c\uc9c0\ub294 FC \ubc14\ub974\uc140\ub85c\ub098 C \ud300\ub3c4 \uc874\uc7ac\ud588\ub294\ub370, \uc774 \ud300\uc740 1967\ub144\uc5d0 \ucc3d\ub2e8\ud55c '\ubc14\ub974\uc140\ub85c\ub098 \uc544\ub9c8\ucd94\uc5b4'\ud300\uc774 \uadf8 \uc804\uc2e0\uc774\uc5c8\ub2e4. FC \ubc14\ub974\uc140\ub85c\ub098\ub294 \uc804\ud1b5\uc801\uc73c\ub85c \uc804 \uc138\uacc4\uc801\uc778 \uad6d\uc801\uc758 \uc120\uc218\ub4e4\uc744 \ubcf4\uc720\ud55c \ud300\uc73c\ub85c\ub3c4 \ub110\ub9ac \uc54c\ub824\uc838 \uc788\uc9c0\ub9cc, \uce78\ud14c\ub77c(\uc720\uc2a4 \uc2dc\uc2a4\ud15c)\ubd80\ud130 \ub6f0\uc5b4\ub09c \uc120\uc218\ub4e4\uc744 \uae38\ub7ec\ub0b4\ub294 \uc5ed\uc0ac\ub85c\ub3c4 \uc720\uba85\ud558\ub2e4. \uc218\ub9ce\uc740 \uc131\uc778 \uc120\uc218\ub4e4\uc774 \uc774\ub7ec\ud55c \uc720\uc18c\ub144 \ud300\uc744 \uac70\uccd0\uc11c \uc131\uc7a5\ud558\uc600\ub2e4.", "sentence_answer": "FC \ubc14\ub974\uc140\ub85c\ub098 B\ub294 \uadf8\ub4e4\uc758 \uc2dc\ub2c8\uc5b4 \ud300\uc778 FC \ubc14\ub974\uc140\ub85c\ub098\uc640\ub294 \uad6c\ubcc4\ub418\ub294 \uc5ed\uc0ac\ub97c \uac00\uc9c0\uace0 \uc788\uc73c\uba70, 1990\ub144\ub300 \uac00 \ub418\uc5b4\uc11c\uc57c FC \ubc14\ub974\uc140\ub85c\ub098 B \ud300\uc774 \ub418\uc5c8\ub2e4.", "paragraph_id": "6472204-0-0", "paragraph_question": "question: FC \ubc14\ub974\uc140\ub85c\ub098 B\ud300\uc73c\ub85c \ubd88\ub9ac\uae30 \uc2dc\uc791\ub41c \uc2dc\uae30\ub294?, context: FC \ubc14\ub974\uc140\ub85c\ub098 B\ub294 \uadf8\ub4e4\uc758 \uc2dc\ub2c8\uc5b4 \ud300\uc778 FC \ubc14\ub974\uc140\ub85c\ub098\uc640\ub294 \uad6c\ubcc4\ub418\ub294 \uc5ed\uc0ac\ub97c \uac00\uc9c0\uace0 \uc788\uc73c\uba70, 1990\ub144\ub300\uac00 \ub418\uc5b4\uc11c\uc57c FC \ubc14\ub974\uc140\ub85c\ub098 B \ud300\uc774 \ub418\uc5c8\ub2e4. \uadf8 \uc774\uc804\uc5d0\ub294 'CD \uc5d0\uc2a4\ud30c\ub0d0 \uc778\ub450\uc2a4\ud2b8\ub9ac\uc54c', 'CD \ucf58\ub2ec', '\ubc14\ub974\uc140\ub85c\ub098 \uc544\ud2c0\ub808\ud2f1' \ub4f1\uc73c\ub85c \ubd88\ub9ac\ub294 \ubcc4\uac1c\uc758 \ud074\ub7fd\uc774\uc5c8\ub2e4. 1993\ub144\ubd80\ud130 2007\ub144\uae4c\uc9c0\ub294 FC \ubc14\ub974\uc140\ub85c\ub098 C \ud300\ub3c4 \uc874\uc7ac\ud588\ub294\ub370, \uc774 \ud300\uc740 1967\ub144\uc5d0 \ucc3d\ub2e8\ud55c '\ubc14\ub974\uc140\ub85c\ub098 \uc544\ub9c8\ucd94\uc5b4'\ud300\uc774 \uadf8 \uc804\uc2e0\uc774\uc5c8\ub2e4. FC \ubc14\ub974\uc140\ub85c\ub098\ub294 \uc804\ud1b5\uc801\uc73c\ub85c \uc804 \uc138\uacc4\uc801\uc778 \uad6d\uc801\uc758 \uc120\uc218\ub4e4\uc744 \ubcf4\uc720\ud55c \ud300\uc73c\ub85c\ub3c4 \ub110\ub9ac \uc54c\ub824\uc838 \uc788\uc9c0\ub9cc, \uce78\ud14c\ub77c(\uc720\uc2a4 \uc2dc\uc2a4\ud15c)\ubd80\ud130 \ub6f0\uc5b4\ub09c \uc120\uc218\ub4e4\uc744 \uae38\ub7ec\ub0b4\ub294 \uc5ed\uc0ac\ub85c\ub3c4 \uc720\uba85\ud558\ub2e4. \uc218\ub9ce\uc740 \uc131\uc778 \uc120\uc218\ub4e4\uc774 \uc774\ub7ec\ud55c \uc720\uc18c\ub144 \ud300\uc744 \uac70\uccd0\uc11c \uc131\uc7a5\ud558\uc600\ub2e4."} {"question": "\uae30\uc6d0\uc804 47\ub144\ubd80\ud130 \ud1a0\uac00\ub97c \uc785\uac8c\ub41c \uc625\ud2f0\ube44\uc544\ub204\uc2a4\uac00 \uadf8\ud574 \uc120\ucd9c\ub41c \uacf3\uc740 \uc5b4\ub514\uc778\uac00?", "paragraph": "\uae30\uc6d0\uc804 51\ub144, \uc728\ub9ac\uc544 \uce74\uc774\uc0ac\ub9ac\uc2a4\uac00 \uc8fd\uc790, \uc625\ud0c0\ube44\uc544\ub204\uc2a4\ub294 \uc678\ud560\uba38\ub2c8\uc758 \uc7a5\ub840\uc2dd\uc5d0\uc11c \ucd94\ubaa8\uc0ac\ub97c \ub0ad\ub3c5\ud558\uc600\ub2e4. \uc774\ub54c\ubd80\ud130 \uc5b4\uba38\ub2c8 \uc544\ud2f0\uc544\uc640 \uacc4\ubd80 \ud544\ub9ac\ud478\uc2a4\ub294 \uc625\ud0c0\ube44\uc544\ub204\uc2a4\ub97c \ub2e8\ub828\uc2dc\ud0a4\uae30 \uc704\ud574 \uc801\uadf9\uc801\uc73c\ub85c \ub098\uc130\ub2e4. \uc625\ud0c0\ube44\uc544\ub204\uc2a4\ub294 4\ub144 \ub4a4\uc778 \uae30\uc6d0\uc804 47\ub144\ubd80\ud130 \ud1a0\uac00\ub97c \uc785\uac8c \ub418\uc5c8\uace0 \uad6d\uac00 \uc0ac\uc81c\ub2e8(Collegium Pontificum)\uc5d0 \uc120\ucd9c\ub418\uc5c8\ub2e4. \ub2e4\uc74c \ud574\uc5d0 \uc625\ud0c0\ube44\uc544\ub204\uc2a4\ub294 \uce74\uc774\uc0ac\ub974\uac00 \uc138\uc6b4 \ubca0\ub204\uc2a4 \uac8c\ub124\ud2b8\ub9ad\uc2a4 \uc2e0\uc804\uc5d0\uc11c \ud589\ud574\uc9c0\ub294 \uadf8\ub9ac\uc2a4 \uacbd\uae30\ub97c \uad00\uc7a5\ud558\uc600\ub2e4. \ub2e4\ub9c8\uc2a4\ucfe0\uc2a4\uc758 \ub2c8\ucf5c\ub77c\uc624\uc2a4\uc5d0 \ub530\ub974\uba74, \uc625\ud0c0\ube44\uc544\ub204\uc2a4\ub294 \uce74\uc774\uc0ac\ub974\uc758 \uc544\ud504\ub9ac\uce74 \uc6d0\uc815\uc5d0 \ucc38\uc5ec\ud558\uace0 \uc2f6\uc5c8\uc73c\ub098 \uc5b4\uba38\ub2c8\uc758 \ubc18\ub300\ub85c \ubb34\uc0b0\ub418\uc5c8\ub2e4\uace0 \ud55c\ub2e4. \uae30\uc6d0\uc804 46\ub144, \uc544\ud2f0\uc544\ub294 \uc625\ud0c0\ube44\uc544\ub204\uc2a4\uac00 \uce74\uc774\uc0ac\ub974\uc758 \ud788\uc2a4\ud30c\ub2c8\uc544 \uc6d0\uc815\uc5d0 \ucc38\uc5ec\ud558\ub294 \uac83\uc5d0 \ub3d9\uc758\ud558\uc600\ub294\ub370, \uc774 \uc6d0\uc815\uc5d0\uc11c \uce74\uc774\uc0ac\ub974\ub294 \uc624\ub79c \uc219\uc801\uc774\uc5c8\ub358 \ud3fc\ud398\uc774\uc6b0\uc2a4 \ub9c8\uadf8\ub204\uc2a4\uc758 \uc544\ub4e4 \uadf8\ub098\uc774\uc6b0\uc2a4 \ud3fc\ud398\uc774\uc6b0\uc2a4\uc640 \uc2f8\uc6b0\ub824 \ud558\uc600\ub2e4. \ud558\uc9c0\ub9cc \uc625\ud0c0\ube44\uc544\ub204\uc2a4\ub294 \ubcd1\uc774 \ub4e4\uc5b4 \uc6d0\uc815\uc5d0 \ucc38\uc5ec\ud560 \uc218 \uc5c6\uac8c \ub418\uc5c8\ub2e4.", "answer": "\uad6d\uac00 \uc0ac\uc81c\ub2e8", "sentence": "\uc625\ud0c0\ube44\uc544\ub204\uc2a4\ub294 4\ub144 \ub4a4\uc778 \uae30\uc6d0\uc804 47\ub144\ubd80\ud130 \ud1a0\uac00\ub97c \uc785\uac8c \ub418\uc5c8\uace0 \uad6d\uac00 \uc0ac\uc81c\ub2e8 (Collegium Pontificum)", "paragraph_sentence": "\uae30\uc6d0\uc804 51\ub144, \uc728\ub9ac\uc544 \uce74\uc774\uc0ac\ub9ac\uc2a4\uac00 \uc8fd\uc790, \uc625\ud0c0\ube44\uc544\ub204\uc2a4\ub294 \uc678\ud560\uba38\ub2c8\uc758 \uc7a5\ub840\uc2dd\uc5d0\uc11c \ucd94\ubaa8\uc0ac\ub97c \ub0ad\ub3c5\ud558\uc600\ub2e4. \uc774\ub54c\ubd80\ud130 \uc5b4\uba38\ub2c8 \uc544\ud2f0\uc544\uc640 \uacc4\ubd80 \ud544\ub9ac\ud478\uc2a4\ub294 \uc625\ud0c0\ube44\uc544\ub204\uc2a4\ub97c \ub2e8\ub828\uc2dc\ud0a4\uae30 \uc704\ud574 \uc801\uadf9\uc801\uc73c\ub85c \ub098\uc130\ub2e4. \uc625\ud0c0\ube44\uc544\ub204\uc2a4\ub294 4\ub144 \ub4a4\uc778 \uae30\uc6d0\uc804 47\ub144\ubd80\ud130 \ud1a0\uac00\ub97c \uc785\uac8c \ub418\uc5c8\uace0 \uad6d\uac00 \uc0ac\uc81c\ub2e8 (Collegium Pontificum) \uc5d0 \uc120\ucd9c\ub418\uc5c8\ub2e4. \ub2e4\uc74c \ud574\uc5d0 \uc625\ud0c0\ube44\uc544\ub204\uc2a4\ub294 \uce74\uc774\uc0ac\ub974\uac00 \uc138\uc6b4 \ubca0\ub204\uc2a4 \uac8c\ub124\ud2b8\ub9ad\uc2a4 \uc2e0\uc804\uc5d0\uc11c \ud589\ud574\uc9c0\ub294 \uadf8\ub9ac\uc2a4 \uacbd\uae30\ub97c \uad00\uc7a5\ud558\uc600\ub2e4. \ub2e4\ub9c8\uc2a4\ucfe0\uc2a4\uc758 \ub2c8\ucf5c\ub77c\uc624\uc2a4\uc5d0 \ub530\ub974\uba74, \uc625\ud0c0\ube44\uc544\ub204\uc2a4\ub294 \uce74\uc774\uc0ac\ub974\uc758 \uc544\ud504\ub9ac\uce74 \uc6d0\uc815\uc5d0 \ucc38\uc5ec\ud558\uace0 \uc2f6\uc5c8\uc73c\ub098 \uc5b4\uba38\ub2c8\uc758 \ubc18\ub300\ub85c \ubb34\uc0b0\ub418\uc5c8\ub2e4\uace0 \ud55c\ub2e4. \uae30\uc6d0\uc804 46\ub144, \uc544\ud2f0\uc544\ub294 \uc625\ud0c0\ube44\uc544\ub204\uc2a4\uac00 \uce74\uc774\uc0ac\ub974\uc758 \ud788\uc2a4\ud30c\ub2c8\uc544 \uc6d0\uc815\uc5d0 \ucc38\uc5ec\ud558\ub294 \uac83\uc5d0 \ub3d9\uc758\ud558\uc600\ub294\ub370, \uc774 \uc6d0\uc815\uc5d0\uc11c \uce74\uc774\uc0ac\ub974\ub294 \uc624\ub79c \uc219\uc801\uc774\uc5c8\ub358 \ud3fc\ud398\uc774\uc6b0\uc2a4 \ub9c8\uadf8\ub204\uc2a4\uc758 \uc544\ub4e4 \uadf8\ub098\uc774\uc6b0\uc2a4 \ud3fc\ud398\uc774\uc6b0\uc2a4\uc640 \uc2f8\uc6b0\ub824 \ud558\uc600\ub2e4. \ud558\uc9c0\ub9cc \uc625\ud0c0\ube44\uc544\ub204\uc2a4\ub294 \ubcd1\uc774 \ub4e4\uc5b4 \uc6d0\uc815\uc5d0 \ucc38\uc5ec\ud560 \uc218 \uc5c6\uac8c \ub418\uc5c8\ub2e4.", "paragraph_answer": "\uae30\uc6d0\uc804 51\ub144, \uc728\ub9ac\uc544 \uce74\uc774\uc0ac\ub9ac\uc2a4\uac00 \uc8fd\uc790, \uc625\ud0c0\ube44\uc544\ub204\uc2a4\ub294 \uc678\ud560\uba38\ub2c8\uc758 \uc7a5\ub840\uc2dd\uc5d0\uc11c \ucd94\ubaa8\uc0ac\ub97c \ub0ad\ub3c5\ud558\uc600\ub2e4. \uc774\ub54c\ubd80\ud130 \uc5b4\uba38\ub2c8 \uc544\ud2f0\uc544\uc640 \uacc4\ubd80 \ud544\ub9ac\ud478\uc2a4\ub294 \uc625\ud0c0\ube44\uc544\ub204\uc2a4\ub97c \ub2e8\ub828\uc2dc\ud0a4\uae30 \uc704\ud574 \uc801\uadf9\uc801\uc73c\ub85c \ub098\uc130\ub2e4. \uc625\ud0c0\ube44\uc544\ub204\uc2a4\ub294 4\ub144 \ub4a4\uc778 \uae30\uc6d0\uc804 47\ub144\ubd80\ud130 \ud1a0\uac00\ub97c \uc785\uac8c \ub418\uc5c8\uace0 \uad6d\uac00 \uc0ac\uc81c\ub2e8 (Collegium Pontificum)\uc5d0 \uc120\ucd9c\ub418\uc5c8\ub2e4. \ub2e4\uc74c \ud574\uc5d0 \uc625\ud0c0\ube44\uc544\ub204\uc2a4\ub294 \uce74\uc774\uc0ac\ub974\uac00 \uc138\uc6b4 \ubca0\ub204\uc2a4 \uac8c\ub124\ud2b8\ub9ad\uc2a4 \uc2e0\uc804\uc5d0\uc11c \ud589\ud574\uc9c0\ub294 \uadf8\ub9ac\uc2a4 \uacbd\uae30\ub97c \uad00\uc7a5\ud558\uc600\ub2e4. \ub2e4\ub9c8\uc2a4\ucfe0\uc2a4\uc758 \ub2c8\ucf5c\ub77c\uc624\uc2a4\uc5d0 \ub530\ub974\uba74, \uc625\ud0c0\ube44\uc544\ub204\uc2a4\ub294 \uce74\uc774\uc0ac\ub974\uc758 \uc544\ud504\ub9ac\uce74 \uc6d0\uc815\uc5d0 \ucc38\uc5ec\ud558\uace0 \uc2f6\uc5c8\uc73c\ub098 \uc5b4\uba38\ub2c8\uc758 \ubc18\ub300\ub85c \ubb34\uc0b0\ub418\uc5c8\ub2e4\uace0 \ud55c\ub2e4. \uae30\uc6d0\uc804 46\ub144, \uc544\ud2f0\uc544\ub294 \uc625\ud0c0\ube44\uc544\ub204\uc2a4\uac00 \uce74\uc774\uc0ac\ub974\uc758 \ud788\uc2a4\ud30c\ub2c8\uc544 \uc6d0\uc815\uc5d0 \ucc38\uc5ec\ud558\ub294 \uac83\uc5d0 \ub3d9\uc758\ud558\uc600\ub294\ub370, \uc774 \uc6d0\uc815\uc5d0\uc11c \uce74\uc774\uc0ac\ub974\ub294 \uc624\ub79c \uc219\uc801\uc774\uc5c8\ub358 \ud3fc\ud398\uc774\uc6b0\uc2a4 \ub9c8\uadf8\ub204\uc2a4\uc758 \uc544\ub4e4 \uadf8\ub098\uc774\uc6b0\uc2a4 \ud3fc\ud398\uc774\uc6b0\uc2a4\uc640 \uc2f8\uc6b0\ub824 \ud558\uc600\ub2e4. \ud558\uc9c0\ub9cc \uc625\ud0c0\ube44\uc544\ub204\uc2a4\ub294 \ubcd1\uc774 \ub4e4\uc5b4 \uc6d0\uc815\uc5d0 \ucc38\uc5ec\ud560 \uc218 \uc5c6\uac8c \ub418\uc5c8\ub2e4.", "sentence_answer": "\uc625\ud0c0\ube44\uc544\ub204\uc2a4\ub294 4\ub144 \ub4a4\uc778 \uae30\uc6d0\uc804 47\ub144\ubd80\ud130 \ud1a0\uac00\ub97c \uc785\uac8c \ub418\uc5c8\uace0 \uad6d\uac00 \uc0ac\uc81c\ub2e8 (Collegium Pontificum)", "paragraph_id": "6657258-3-0", "paragraph_question": "question: \uae30\uc6d0\uc804 47\ub144\ubd80\ud130 \ud1a0\uac00\ub97c \uc785\uac8c\ub41c \uc625\ud2f0\ube44\uc544\ub204\uc2a4\uac00 \uadf8\ud574 \uc120\ucd9c\ub41c \uacf3\uc740 \uc5b4\ub514\uc778\uac00?, context: \uae30\uc6d0\uc804 51\ub144, \uc728\ub9ac\uc544 \uce74\uc774\uc0ac\ub9ac\uc2a4\uac00 \uc8fd\uc790, \uc625\ud0c0\ube44\uc544\ub204\uc2a4\ub294 \uc678\ud560\uba38\ub2c8\uc758 \uc7a5\ub840\uc2dd\uc5d0\uc11c \ucd94\ubaa8\uc0ac\ub97c \ub0ad\ub3c5\ud558\uc600\ub2e4. \uc774\ub54c\ubd80\ud130 \uc5b4\uba38\ub2c8 \uc544\ud2f0\uc544\uc640 \uacc4\ubd80 \ud544\ub9ac\ud478\uc2a4\ub294 \uc625\ud0c0\ube44\uc544\ub204\uc2a4\ub97c \ub2e8\ub828\uc2dc\ud0a4\uae30 \uc704\ud574 \uc801\uadf9\uc801\uc73c\ub85c \ub098\uc130\ub2e4. \uc625\ud0c0\ube44\uc544\ub204\uc2a4\ub294 4\ub144 \ub4a4\uc778 \uae30\uc6d0\uc804 47\ub144\ubd80\ud130 \ud1a0\uac00\ub97c \uc785\uac8c \ub418\uc5c8\uace0 \uad6d\uac00 \uc0ac\uc81c\ub2e8(Collegium Pontificum)\uc5d0 \uc120\ucd9c\ub418\uc5c8\ub2e4. \ub2e4\uc74c \ud574\uc5d0 \uc625\ud0c0\ube44\uc544\ub204\uc2a4\ub294 \uce74\uc774\uc0ac\ub974\uac00 \uc138\uc6b4 \ubca0\ub204\uc2a4 \uac8c\ub124\ud2b8\ub9ad\uc2a4 \uc2e0\uc804\uc5d0\uc11c \ud589\ud574\uc9c0\ub294 \uadf8\ub9ac\uc2a4 \uacbd\uae30\ub97c \uad00\uc7a5\ud558\uc600\ub2e4. \ub2e4\ub9c8\uc2a4\ucfe0\uc2a4\uc758 \ub2c8\ucf5c\ub77c\uc624\uc2a4\uc5d0 \ub530\ub974\uba74, \uc625\ud0c0\ube44\uc544\ub204\uc2a4\ub294 \uce74\uc774\uc0ac\ub974\uc758 \uc544\ud504\ub9ac\uce74 \uc6d0\uc815\uc5d0 \ucc38\uc5ec\ud558\uace0 \uc2f6\uc5c8\uc73c\ub098 \uc5b4\uba38\ub2c8\uc758 \ubc18\ub300\ub85c \ubb34\uc0b0\ub418\uc5c8\ub2e4\uace0 \ud55c\ub2e4. \uae30\uc6d0\uc804 46\ub144, \uc544\ud2f0\uc544\ub294 \uc625\ud0c0\ube44\uc544\ub204\uc2a4\uac00 \uce74\uc774\uc0ac\ub974\uc758 \ud788\uc2a4\ud30c\ub2c8\uc544 \uc6d0\uc815\uc5d0 \ucc38\uc5ec\ud558\ub294 \uac83\uc5d0 \ub3d9\uc758\ud558\uc600\ub294\ub370, \uc774 \uc6d0\uc815\uc5d0\uc11c \uce74\uc774\uc0ac\ub974\ub294 \uc624\ub79c \uc219\uc801\uc774\uc5c8\ub358 \ud3fc\ud398\uc774\uc6b0\uc2a4 \ub9c8\uadf8\ub204\uc2a4\uc758 \uc544\ub4e4 \uadf8\ub098\uc774\uc6b0\uc2a4 \ud3fc\ud398\uc774\uc6b0\uc2a4\uc640 \uc2f8\uc6b0\ub824 \ud558\uc600\ub2e4. \ud558\uc9c0\ub9cc \uc625\ud0c0\ube44\uc544\ub204\uc2a4\ub294 \ubcd1\uc774 \ub4e4\uc5b4 \uc6d0\uc815\uc5d0 \ucc38\uc5ec\ud560 \uc218 \uc5c6\uac8c \ub418\uc5c8\ub2e4."} {"question": "\uc0ac\uace0\uc5f4\ucc28\uc758 \uc218\uc2b5\uc5d0 \ub3c4\uc6c0\uc774 \ub418\uc5c8\ub358 \uc9c0\uc9c4\uc758 \ub3c4\uc2dc \uc774\ub984\uc740?", "paragraph": "\ud55c\uc2e0\u00b7\uc544\uc640\uc9c0 \ub300\uc9c0\uc9c4(\uace0\ubca0 \uc9c0\uc9c4)\uc758 \uacbd\ud5d8\uc774 \ud65c\uc6a9\ub418\uc5b4 \uc2e0\uc18d\ud55c \ub300\uc751\uc774 \uc774\ub8e8\uc5b4\uc84c\ub2e4. \uc0ac\uace0 \ubc1c\uc0dd \ub2f9\uc2dc \uac00\uc7a5 \uba3c\uc800 \uad6c\uc870\uc5d0 \ub098\uc120 \uac83\uc740 \uc778\uadfc \uc8fc\ubbfc\uc774\ub098 \uae30\uc5c5\uc774\uba70, \uae30\uc5c5 \uc911\uc5d0\ub294 \uacf5\uc7a5 \uac00\ub3d9\uc744 \uc77c\uc2dc\uc815\uc9c0\ud558\uace0 \ucc38\uac00\ud55c \uacf3\ub3c4 \uc788\uc5c8\ub2e4. \ubd80\uc0c1\uc790\uc758 \ubc18\uc218 \uac00\uae4c\uc774\ub97c \uc8fc\ubbfc\uc774\ub098 \uae30\uc5c5\uc774 \uc758\ub8cc \uae30\uad00\uc5d0 \ubc18\uc1a1\ud558\uc600\uc73c\uba70, \ud6c4\uc5d0 \uad6c\uc870\u00b7\uad6c\uc6d0 \ud65c\ub3d9 \uacf5\uc801\uc744 \uae30\ub9ac\uae30 \uc704\ud574 2005\ub144 7\uc6d4 76\uac1c \uae30\uc5c5\u00b7\ub2e8\uccb4\uc640 \uac1c\uc778 1\uba85\uc5d0 \ub300\ud574 \uc815\ubd80\ub85c\ubd80\ud130 \uac10\uc0ac\uc7a5\uc774, 8\uc6d4\uc5d0\ub294 48\uac1c \uae30\uc5c5\u00b7\ub2e8\uccb4\uc640 \uac1c\uc778 34\uba85\uc5d0 \ud6a8\uace0\ud604\uacbd\uc73c\ub85c\ubd80\ud130 \uac10\uc0ac\uc7a5\uc774, 9\uc6d4\uc5d0\ub294 32\uac1c \uae30\uc5c5\u00b7\ub2e8\uccb4\uc640 \uac1c\uc778 30\uba85\uc5d0 \ub300\ud574 \uc544\ub9c8\uac00\uc0ac\ud0a4 \uc2dc\ub85c\ubd80\ud130 \uac10\uc0ac\uc7a5\uc774 \uac01\uac01 \uc218\uc5ec\ub418\uc5c8\ub2e4. \ub610, 11\uc6d4\uc5d0\ub294 \uc790\uc0ac\uc758 \uc5c5\ubb34\ub97c \ubaa8\ub450 \uc815\uc9c0\uc2dc\ud0a4\uace0 \uad6c\uc870\ud65c\ub3d9\u00b7\ubd80\uc9c0 \ub0b4\ub97c \ud65c\ub3d9\uac70\uc810\uc73c\ub85c\uc11c \uad6c\uc870\uc5d0 \uc784\ud55c \uae30\uc5c5\uacfc 2\ucc28 \uc7ac\ud574 \ud68c\ud53c\ub97c \uc704\ud574 \uc120\ub85c \ub0b4\uc5d0 \ub4e4\uc5b4\uac00 \ubc18\ub300\ud3b8 \uc5f4\ucc28\uc5d0 \uc774\uc0c1\uc744 \uc54c\ub9b0 \uac1c\uc778 1\uba85\uc5d0 \ub300\ud574\uc11c \ud6c8\uc7a5\uc774 \uc218\uc5ec\ub418\uc5c8\ub2e4.", "answer": "\uace0\ubca0", "sentence": "\ud55c\uc2e0\u00b7\uc544\uc640\uc9c0 \ub300\uc9c0\uc9c4( \uace0\ubca0 \uc9c0\uc9c4)\uc758 \uacbd\ud5d8\uc774 \ud65c\uc6a9\ub418\uc5b4 \uc2e0\uc18d\ud55c \ub300\uc751\uc774 \uc774\ub8e8\uc5b4\uc84c\ub2e4.", "paragraph_sentence": " \ud55c\uc2e0\u00b7\uc544\uc640\uc9c0 \ub300\uc9c0\uc9c4( \uace0\ubca0 \uc9c0\uc9c4)\uc758 \uacbd\ud5d8\uc774 \ud65c\uc6a9\ub418\uc5b4 \uc2e0\uc18d\ud55c \ub300\uc751\uc774 \uc774\ub8e8\uc5b4\uc84c\ub2e4. \uc0ac\uace0 \ubc1c\uc0dd \ub2f9\uc2dc \uac00\uc7a5 \uba3c\uc800 \uad6c\uc870\uc5d0 \ub098\uc120 \uac83\uc740 \uc778\uadfc \uc8fc\ubbfc\uc774\ub098 \uae30\uc5c5\uc774\uba70, \uae30\uc5c5 \uc911\uc5d0\ub294 \uacf5\uc7a5 \uac00\ub3d9\uc744 \uc77c\uc2dc\uc815\uc9c0\ud558\uace0 \ucc38\uac00\ud55c \uacf3\ub3c4 \uc788\uc5c8\ub2e4. \ubd80\uc0c1\uc790\uc758 \ubc18\uc218 \uac00\uae4c\uc774\ub97c \uc8fc\ubbfc\uc774\ub098 \uae30\uc5c5\uc774 \uc758\ub8cc \uae30\uad00\uc5d0 \ubc18\uc1a1\ud558\uc600\uc73c\uba70, \ud6c4\uc5d0 \uad6c\uc870\u00b7\uad6c\uc6d0 \ud65c\ub3d9 \uacf5\uc801\uc744 \uae30\ub9ac\uae30 \uc704\ud574 2005\ub144 7\uc6d4 76\uac1c \uae30\uc5c5\u00b7\ub2e8\uccb4\uc640 \uac1c\uc778 1\uba85\uc5d0 \ub300\ud574 \uc815\ubd80\ub85c\ubd80\ud130 \uac10\uc0ac\uc7a5\uc774, 8\uc6d4\uc5d0\ub294 48\uac1c \uae30\uc5c5\u00b7\ub2e8\uccb4\uc640 \uac1c\uc778 34\uba85\uc5d0 \ud6a8\uace0\ud604\uacbd\uc73c\ub85c\ubd80\ud130 \uac10\uc0ac\uc7a5\uc774, 9\uc6d4\uc5d0\ub294 32\uac1c \uae30\uc5c5\u00b7\ub2e8\uccb4\uc640 \uac1c\uc778 30\uba85\uc5d0 \ub300\ud574 \uc544\ub9c8\uac00\uc0ac\ud0a4 \uc2dc\ub85c\ubd80\ud130 \uac10\uc0ac\uc7a5\uc774 \uac01\uac01 \uc218\uc5ec\ub418\uc5c8\ub2e4. \ub610, 11\uc6d4\uc5d0\ub294 \uc790\uc0ac\uc758 \uc5c5\ubb34\ub97c \ubaa8\ub450 \uc815\uc9c0\uc2dc\ud0a4\uace0 \uad6c\uc870\ud65c\ub3d9\u00b7\ubd80\uc9c0 \ub0b4\ub97c \ud65c\ub3d9\uac70\uc810\uc73c\ub85c\uc11c \uad6c\uc870\uc5d0 \uc784\ud55c \uae30\uc5c5\uacfc 2\ucc28 \uc7ac\ud574 \ud68c\ud53c\ub97c \uc704\ud574 \uc120\ub85c \ub0b4\uc5d0 \ub4e4\uc5b4\uac00 \ubc18\ub300\ud3b8 \uc5f4\ucc28\uc5d0 \uc774\uc0c1\uc744 \uc54c\ub9b0 \uac1c\uc778 1\uba85\uc5d0 \ub300\ud574\uc11c \ud6c8\uc7a5\uc774 \uc218\uc5ec\ub418\uc5c8\ub2e4.", "paragraph_answer": "\ud55c\uc2e0\u00b7\uc544\uc640\uc9c0 \ub300\uc9c0\uc9c4( \uace0\ubca0 \uc9c0\uc9c4)\uc758 \uacbd\ud5d8\uc774 \ud65c\uc6a9\ub418\uc5b4 \uc2e0\uc18d\ud55c \ub300\uc751\uc774 \uc774\ub8e8\uc5b4\uc84c\ub2e4. \uc0ac\uace0 \ubc1c\uc0dd \ub2f9\uc2dc \uac00\uc7a5 \uba3c\uc800 \uad6c\uc870\uc5d0 \ub098\uc120 \uac83\uc740 \uc778\uadfc \uc8fc\ubbfc\uc774\ub098 \uae30\uc5c5\uc774\uba70, \uae30\uc5c5 \uc911\uc5d0\ub294 \uacf5\uc7a5 \uac00\ub3d9\uc744 \uc77c\uc2dc\uc815\uc9c0\ud558\uace0 \ucc38\uac00\ud55c \uacf3\ub3c4 \uc788\uc5c8\ub2e4. \ubd80\uc0c1\uc790\uc758 \ubc18\uc218 \uac00\uae4c\uc774\ub97c \uc8fc\ubbfc\uc774\ub098 \uae30\uc5c5\uc774 \uc758\ub8cc \uae30\uad00\uc5d0 \ubc18\uc1a1\ud558\uc600\uc73c\uba70, \ud6c4\uc5d0 \uad6c\uc870\u00b7\uad6c\uc6d0 \ud65c\ub3d9 \uacf5\uc801\uc744 \uae30\ub9ac\uae30 \uc704\ud574 2005\ub144 7\uc6d4 76\uac1c \uae30\uc5c5\u00b7\ub2e8\uccb4\uc640 \uac1c\uc778 1\uba85\uc5d0 \ub300\ud574 \uc815\ubd80\ub85c\ubd80\ud130 \uac10\uc0ac\uc7a5\uc774, 8\uc6d4\uc5d0\ub294 48\uac1c \uae30\uc5c5\u00b7\ub2e8\uccb4\uc640 \uac1c\uc778 34\uba85\uc5d0 \ud6a8\uace0\ud604\uacbd\uc73c\ub85c\ubd80\ud130 \uac10\uc0ac\uc7a5\uc774, 9\uc6d4\uc5d0\ub294 32\uac1c \uae30\uc5c5\u00b7\ub2e8\uccb4\uc640 \uac1c\uc778 30\uba85\uc5d0 \ub300\ud574 \uc544\ub9c8\uac00\uc0ac\ud0a4 \uc2dc\ub85c\ubd80\ud130 \uac10\uc0ac\uc7a5\uc774 \uac01\uac01 \uc218\uc5ec\ub418\uc5c8\ub2e4. \ub610, 11\uc6d4\uc5d0\ub294 \uc790\uc0ac\uc758 \uc5c5\ubb34\ub97c \ubaa8\ub450 \uc815\uc9c0\uc2dc\ud0a4\uace0 \uad6c\uc870\ud65c\ub3d9\u00b7\ubd80\uc9c0 \ub0b4\ub97c \ud65c\ub3d9\uac70\uc810\uc73c\ub85c\uc11c \uad6c\uc870\uc5d0 \uc784\ud55c \uae30\uc5c5\uacfc 2\ucc28 \uc7ac\ud574 \ud68c\ud53c\ub97c \uc704\ud574 \uc120\ub85c \ub0b4\uc5d0 \ub4e4\uc5b4\uac00 \ubc18\ub300\ud3b8 \uc5f4\ucc28\uc5d0 \uc774\uc0c1\uc744 \uc54c\ub9b0 \uac1c\uc778 1\uba85\uc5d0 \ub300\ud574\uc11c \ud6c8\uc7a5\uc774 \uc218\uc5ec\ub418\uc5c8\ub2e4.", "sentence_answer": "\ud55c\uc2e0\u00b7\uc544\uc640\uc9c0 \ub300\uc9c0\uc9c4( \uace0\ubca0 \uc9c0\uc9c4)\uc758 \uacbd\ud5d8\uc774 \ud65c\uc6a9\ub418\uc5b4 \uc2e0\uc18d\ud55c \ub300\uc751\uc774 \uc774\ub8e8\uc5b4\uc84c\ub2e4.", "paragraph_id": "6470376-3-0", "paragraph_question": "question: \uc0ac\uace0\uc5f4\ucc28\uc758 \uc218\uc2b5\uc5d0 \ub3c4\uc6c0\uc774 \ub418\uc5c8\ub358 \uc9c0\uc9c4\uc758 \ub3c4\uc2dc \uc774\ub984\uc740?, context: \ud55c\uc2e0\u00b7\uc544\uc640\uc9c0 \ub300\uc9c0\uc9c4(\uace0\ubca0 \uc9c0\uc9c4)\uc758 \uacbd\ud5d8\uc774 \ud65c\uc6a9\ub418\uc5b4 \uc2e0\uc18d\ud55c \ub300\uc751\uc774 \uc774\ub8e8\uc5b4\uc84c\ub2e4. \uc0ac\uace0 \ubc1c\uc0dd \ub2f9\uc2dc \uac00\uc7a5 \uba3c\uc800 \uad6c\uc870\uc5d0 \ub098\uc120 \uac83\uc740 \uc778\uadfc \uc8fc\ubbfc\uc774\ub098 \uae30\uc5c5\uc774\uba70, \uae30\uc5c5 \uc911\uc5d0\ub294 \uacf5\uc7a5 \uac00\ub3d9\uc744 \uc77c\uc2dc\uc815\uc9c0\ud558\uace0 \ucc38\uac00\ud55c \uacf3\ub3c4 \uc788\uc5c8\ub2e4. \ubd80\uc0c1\uc790\uc758 \ubc18\uc218 \uac00\uae4c\uc774\ub97c \uc8fc\ubbfc\uc774\ub098 \uae30\uc5c5\uc774 \uc758\ub8cc \uae30\uad00\uc5d0 \ubc18\uc1a1\ud558\uc600\uc73c\uba70, \ud6c4\uc5d0 \uad6c\uc870\u00b7\uad6c\uc6d0 \ud65c\ub3d9 \uacf5\uc801\uc744 \uae30\ub9ac\uae30 \uc704\ud574 2005\ub144 7\uc6d4 76\uac1c \uae30\uc5c5\u00b7\ub2e8\uccb4\uc640 \uac1c\uc778 1\uba85\uc5d0 \ub300\ud574 \uc815\ubd80\ub85c\ubd80\ud130 \uac10\uc0ac\uc7a5\uc774, 8\uc6d4\uc5d0\ub294 48\uac1c \uae30\uc5c5\u00b7\ub2e8\uccb4\uc640 \uac1c\uc778 34\uba85\uc5d0 \ud6a8\uace0\ud604\uacbd\uc73c\ub85c\ubd80\ud130 \uac10\uc0ac\uc7a5\uc774, 9\uc6d4\uc5d0\ub294 32\uac1c \uae30\uc5c5\u00b7\ub2e8\uccb4\uc640 \uac1c\uc778 30\uba85\uc5d0 \ub300\ud574 \uc544\ub9c8\uac00\uc0ac\ud0a4 \uc2dc\ub85c\ubd80\ud130 \uac10\uc0ac\uc7a5\uc774 \uac01\uac01 \uc218\uc5ec\ub418\uc5c8\ub2e4. \ub610, 11\uc6d4\uc5d0\ub294 \uc790\uc0ac\uc758 \uc5c5\ubb34\ub97c \ubaa8\ub450 \uc815\uc9c0\uc2dc\ud0a4\uace0 \uad6c\uc870\ud65c\ub3d9\u00b7\ubd80\uc9c0 \ub0b4\ub97c \ud65c\ub3d9\uac70\uc810\uc73c\ub85c\uc11c \uad6c\uc870\uc5d0 \uc784\ud55c \uae30\uc5c5\uacfc 2\ucc28 \uc7ac\ud574 \ud68c\ud53c\ub97c \uc704\ud574 \uc120\ub85c \ub0b4\uc5d0 \ub4e4\uc5b4\uac00 \ubc18\ub300\ud3b8 \uc5f4\ucc28\uc5d0 \uc774\uc0c1\uc744 \uc54c\ub9b0 \uac1c\uc778 1\uba85\uc5d0 \ub300\ud574\uc11c \ud6c8\uc7a5\uc774 \uc218\uc5ec\ub418\uc5c8\ub2e4."} {"question": "\ub974\ub124 \ud1b5\uc774 \ud504\ub791\uc2a4 \uacfc\ud559 \uc544\uce74\ub370\ubbf8 \ud68c\uc6d0\uc774 \ub41c \ub144\ub3c4\ub294?", "paragraph": "\ud1b5\uc740 1923\ub144 \ud504\ub791\uc2a4\uc758 \uc791\uc740 \ub3c4\uc2dc \ubabd\ubca8\ub9ac\uc544\ub974\uc5d0\uc11c \ud0dc\uc5b4\ub0ac\ub2e4. \uc0dd-\ub8e8\uc774 \uace0\uad50(Lyc\u00e9e Saint-Louis)\uc640 \ud30c\ub9ac\uc758 \uace0\ub4f1\uc0ac\ubc94\ud559\uad50\uc5d0\uc11c \uc218\ud559\ud588\ub2e4. \uadf8\ub294 \ub2c8\ucf5c\ub77c \ubd80\ub974\ubc14\ud0a4\uc758 \ucc3d\uc2dc\uc790 \uc911 \ud558\ub098\uc778 \uc559\ub9ac \uce74\ub974\ud0d5\uc744 \ud1b5\ud574 \uc704\uc0c1\uc218\ud559\uc5d0 \uc785\ubb38\ud588\ub294\ub370, \uadf8\ub97c \uc9c0\ub3c4\uad50\uc218\ub85c \ud558\uc5ec 1951\ub144 \ud30c\ub9ac \ub300\ud559\uad50\uc5d0\uc11c \ubc15\uc0ac\ud559\uc704\ub97c \ubc1b\uc558\ub2e4. \uadf8\ub54c\uc758 \ubc15\uc0ac\ud559\uc704 \ub17c\ubb38\uc5d0\ub294 \ud6d7\ub0a0 \uadf8\uc5d0\uac8c \ud544\uc988\uc0c1\uc744 \uc120\uc0ac\ud558\ub294 \ubcf4\ucda9 \uacbd\uacc4 \uc774\ub860\uc758 \uae30\ubcf8 \uc544\uc774\ub514\uc5b4\uac00 \uc774\ubbf8 \uc81c\uc2dc\ub418\uc5b4 \uc788\uc5c8\ub2e4. \ub974\ub124\ub294 \uc9c0\ub3c4\uad50 \uc218 \uc559\ub9ac \uce74\ub974\ud0d5\uc744 \ube44\ub86f\ud55c \uc5c4\uaca9\ud558\uace0 \ud615\uc2dd\uc8fc\uc758\uc801\uc778 \ub2c8\ucf5c\ub77c \ubd80\ub974\ubc14\ud0a4\ub85c\ubd80\ud130 \ub9ce\uc740 \uac83\uc744 \ubc30\uc6e0\ub2e4. \uadf8\ub7ec\ub098 \uc790\uc720\ubd84\ubc29\ud558\uba70 \uc9c1\uad00\uc801\uc778 \uc131\ud5a5\uc758 \uadf8\ub294 \uc774\ub7ec\ud55c \uacbd\ud5a5\uc5d0 \uc644\uc804\ud788 \ub3d9\uc870\ud558\uc9c0 \uc54a\uc558\ub294\ub370, \uc774\ub294 \ub098\uc911\uc5d0 \uadf8\uac00 \uc218\ud559\uc744 \ub5a0\ub098\uac8c \ub41c \uc6d0\uc778\uc774 \ub418\uc5c8\uc744 \uac83\uc774\ub2e4. \ud55c\ud3b8 \uadf8\ub294 \uadf8\ub974\ub178\ube14(1953\ub144~1954\ub144)\uacfc \uc2a4\ud2b8\ub77c\uc2a4\ubd80\ub974 \ub300\ud559\uad50(1954\ub144~1963\ub144)\uc5d0\uc11c \uad50\uc218\uc9c1\uc5d0 \uc784\uc6a9\ub418\uc5b4 \uc218\ud559\uc744 \uac00\ub974\ucce4\ub2e4. 1964\ub144 \ud30c\ub9ac \uadfc\uad50 \ubdd4\ub974\uc26c\ub974\uc774\ubca0\ud2b8\uc5d0 \uc704\uce58\ud55c \uc5f0\uad6c\uc18c IH\u00c9S\ub85c \uc62e\uaca8 \ubc88\uac70\ub85c\uc6b4 \uc758\ubb34\uc5d0 \uc2dc\ub2ec\ub9ac\uc9c0 \uc54a\uace0 \uc8fd\uc744 \ub54c\uae4c\uc9c0 \uc790\uc720\ub86d\uac8c \uc5f0\uad6c\ud560 \uc218 \uc788\uac8c \ub418\uc5c8\ub2e4. 1976\ub144\uc5d0 \uadf8\ub294 \ud504\ub791\uc2a4 \uacfc\ud559 \uc544\uce74\ub370\ubbf8 \ud68c\uc6d0\uc774 \ub418\uc5c8\ub2e4.", "answer": "1976\ub144", "sentence": "1976\ub144 \uc5d0 \uadf8\ub294 \ud504\ub791\uc2a4 \uacfc\ud559 \uc544\uce74\ub370\ubbf8 \ud68c\uc6d0\uc774 \ub418\uc5c8\ub2e4.", "paragraph_sentence": "\ud1b5\uc740 1923\ub144 \ud504\ub791\uc2a4\uc758 \uc791\uc740 \ub3c4\uc2dc \ubabd\ubca8\ub9ac\uc544\ub974\uc5d0\uc11c \ud0dc\uc5b4\ub0ac\ub2e4. \uc0dd-\ub8e8\uc774 \uace0\uad50(Lyc\u00e9e Saint-Louis)\uc640 \ud30c\ub9ac\uc758 \uace0\ub4f1\uc0ac\ubc94\ud559\uad50\uc5d0\uc11c \uc218\ud559\ud588\ub2e4. \uadf8\ub294 \ub2c8\ucf5c\ub77c \ubd80\ub974\ubc14\ud0a4\uc758 \ucc3d\uc2dc\uc790 \uc911 \ud558\ub098\uc778 \uc559\ub9ac \uce74\ub974\ud0d5\uc744 \ud1b5\ud574 \uc704\uc0c1\uc218\ud559\uc5d0 \uc785\ubb38\ud588\ub294\ub370, \uadf8\ub97c \uc9c0\ub3c4\uad50\uc218\ub85c \ud558\uc5ec 1951\ub144 \ud30c\ub9ac \ub300\ud559\uad50\uc5d0\uc11c \ubc15\uc0ac\ud559\uc704\ub97c \ubc1b\uc558\ub2e4. \uadf8\ub54c\uc758 \ubc15\uc0ac\ud559\uc704 \ub17c\ubb38\uc5d0\ub294 \ud6d7\ub0a0 \uadf8\uc5d0\uac8c \ud544\uc988\uc0c1\uc744 \uc120\uc0ac\ud558\ub294 \ubcf4\ucda9 \uacbd\uacc4 \uc774\ub860\uc758 \uae30\ubcf8 \uc544\uc774\ub514\uc5b4\uac00 \uc774\ubbf8 \uc81c\uc2dc\ub418\uc5b4 \uc788\uc5c8\ub2e4. \ub974\ub124\ub294 \uc9c0\ub3c4\uad50 \uc218 \uc559\ub9ac \uce74\ub974\ud0d5\uc744 \ube44\ub86f\ud55c \uc5c4\uaca9\ud558\uace0 \ud615\uc2dd\uc8fc\uc758\uc801\uc778 \ub2c8\ucf5c\ub77c \ubd80\ub974\ubc14\ud0a4\ub85c\ubd80\ud130 \ub9ce\uc740 \uac83\uc744 \ubc30\uc6e0\ub2e4. \uadf8\ub7ec\ub098 \uc790\uc720\ubd84\ubc29\ud558\uba70 \uc9c1\uad00\uc801\uc778 \uc131\ud5a5\uc758 \uadf8\ub294 \uc774\ub7ec\ud55c \uacbd\ud5a5\uc5d0 \uc644\uc804\ud788 \ub3d9\uc870\ud558\uc9c0 \uc54a\uc558\ub294\ub370, \uc774\ub294 \ub098\uc911\uc5d0 \uadf8\uac00 \uc218\ud559\uc744 \ub5a0\ub098\uac8c \ub41c \uc6d0\uc778\uc774 \ub418\uc5c8\uc744 \uac83\uc774\ub2e4. \ud55c\ud3b8 \uadf8\ub294 \uadf8\ub974\ub178\ube14(1953\ub144~1954\ub144)\uacfc \uc2a4\ud2b8\ub77c\uc2a4\ubd80\ub974 \ub300\ud559\uad50(1954\ub144~1963\ub144)\uc5d0\uc11c \uad50\uc218\uc9c1\uc5d0 \uc784\uc6a9\ub418\uc5b4 \uc218\ud559\uc744 \uac00\ub974\ucce4\ub2e4. 1964\ub144 \ud30c\ub9ac \uadfc\uad50 \ubdd4\ub974\uc26c\ub974\uc774\ubca0\ud2b8\uc5d0 \uc704\uce58\ud55c \uc5f0\uad6c\uc18c IH\u00c9S\ub85c \uc62e\uaca8 \ubc88\uac70\ub85c\uc6b4 \uc758\ubb34\uc5d0 \uc2dc\ub2ec\ub9ac\uc9c0 \uc54a\uace0 \uc8fd\uc744 \ub54c\uae4c\uc9c0 \uc790\uc720\ub86d\uac8c \uc5f0\uad6c\ud560 \uc218 \uc788\uac8c \ub418\uc5c8\ub2e4. 1976\ub144 \uc5d0 \uadf8\ub294 \ud504\ub791\uc2a4 \uacfc\ud559 \uc544\uce74\ub370\ubbf8 \ud68c\uc6d0\uc774 \ub418\uc5c8\ub2e4. ", "paragraph_answer": "\ud1b5\uc740 1923\ub144 \ud504\ub791\uc2a4\uc758 \uc791\uc740 \ub3c4\uc2dc \ubabd\ubca8\ub9ac\uc544\ub974\uc5d0\uc11c \ud0dc\uc5b4\ub0ac\ub2e4. \uc0dd-\ub8e8\uc774 \uace0\uad50(Lyc\u00e9e Saint-Louis)\uc640 \ud30c\ub9ac\uc758 \uace0\ub4f1\uc0ac\ubc94\ud559\uad50\uc5d0\uc11c \uc218\ud559\ud588\ub2e4. \uadf8\ub294 \ub2c8\ucf5c\ub77c \ubd80\ub974\ubc14\ud0a4\uc758 \ucc3d\uc2dc\uc790 \uc911 \ud558\ub098\uc778 \uc559\ub9ac \uce74\ub974\ud0d5\uc744 \ud1b5\ud574 \uc704\uc0c1\uc218\ud559\uc5d0 \uc785\ubb38\ud588\ub294\ub370, \uadf8\ub97c \uc9c0\ub3c4\uad50\uc218\ub85c \ud558\uc5ec 1951\ub144 \ud30c\ub9ac \ub300\ud559\uad50\uc5d0\uc11c \ubc15\uc0ac\ud559\uc704\ub97c \ubc1b\uc558\ub2e4. \uadf8\ub54c\uc758 \ubc15\uc0ac\ud559\uc704 \ub17c\ubb38\uc5d0\ub294 \ud6d7\ub0a0 \uadf8\uc5d0\uac8c \ud544\uc988\uc0c1\uc744 \uc120\uc0ac\ud558\ub294 \ubcf4\ucda9 \uacbd\uacc4 \uc774\ub860\uc758 \uae30\ubcf8 \uc544\uc774\ub514\uc5b4\uac00 \uc774\ubbf8 \uc81c\uc2dc\ub418\uc5b4 \uc788\uc5c8\ub2e4. \ub974\ub124\ub294 \uc9c0\ub3c4\uad50 \uc218 \uc559\ub9ac \uce74\ub974\ud0d5\uc744 \ube44\ub86f\ud55c \uc5c4\uaca9\ud558\uace0 \ud615\uc2dd\uc8fc\uc758\uc801\uc778 \ub2c8\ucf5c\ub77c \ubd80\ub974\ubc14\ud0a4\ub85c\ubd80\ud130 \ub9ce\uc740 \uac83\uc744 \ubc30\uc6e0\ub2e4. \uadf8\ub7ec\ub098 \uc790\uc720\ubd84\ubc29\ud558\uba70 \uc9c1\uad00\uc801\uc778 \uc131\ud5a5\uc758 \uadf8\ub294 \uc774\ub7ec\ud55c \uacbd\ud5a5\uc5d0 \uc644\uc804\ud788 \ub3d9\uc870\ud558\uc9c0 \uc54a\uc558\ub294\ub370, \uc774\ub294 \ub098\uc911\uc5d0 \uadf8\uac00 \uc218\ud559\uc744 \ub5a0\ub098\uac8c \ub41c \uc6d0\uc778\uc774 \ub418\uc5c8\uc744 \uac83\uc774\ub2e4. \ud55c\ud3b8 \uadf8\ub294 \uadf8\ub974\ub178\ube14(1953\ub144~1954\ub144)\uacfc \uc2a4\ud2b8\ub77c\uc2a4\ubd80\ub974 \ub300\ud559\uad50(1954\ub144~1963\ub144)\uc5d0\uc11c \uad50\uc218\uc9c1\uc5d0 \uc784\uc6a9\ub418\uc5b4 \uc218\ud559\uc744 \uac00\ub974\ucce4\ub2e4. 1964\ub144 \ud30c\ub9ac \uadfc\uad50 \ubdd4\ub974\uc26c\ub974\uc774\ubca0\ud2b8\uc5d0 \uc704\uce58\ud55c \uc5f0\uad6c\uc18c IH\u00c9S\ub85c \uc62e\uaca8 \ubc88\uac70\ub85c\uc6b4 \uc758\ubb34\uc5d0 \uc2dc\ub2ec\ub9ac\uc9c0 \uc54a\uace0 \uc8fd\uc744 \ub54c\uae4c\uc9c0 \uc790\uc720\ub86d\uac8c \uc5f0\uad6c\ud560 \uc218 \uc788\uac8c \ub418\uc5c8\ub2e4. 1976\ub144 \uc5d0 \uadf8\ub294 \ud504\ub791\uc2a4 \uacfc\ud559 \uc544\uce74\ub370\ubbf8 \ud68c\uc6d0\uc774 \ub418\uc5c8\ub2e4.", "sentence_answer": " 1976\ub144 \uc5d0 \uadf8\ub294 \ud504\ub791\uc2a4 \uacfc\ud559 \uc544\uce74\ub370\ubbf8 \ud68c\uc6d0\uc774 \ub418\uc5c8\ub2e4.", "paragraph_id": "6498150-0-2", "paragraph_question": "question: \ub974\ub124 \ud1b5\uc774 \ud504\ub791\uc2a4 \uacfc\ud559 \uc544\uce74\ub370\ubbf8 \ud68c\uc6d0\uc774 \ub41c \ub144\ub3c4\ub294?, context: \ud1b5\uc740 1923\ub144 \ud504\ub791\uc2a4\uc758 \uc791\uc740 \ub3c4\uc2dc \ubabd\ubca8\ub9ac\uc544\ub974\uc5d0\uc11c \ud0dc\uc5b4\ub0ac\ub2e4. \uc0dd-\ub8e8\uc774 \uace0\uad50(Lyc\u00e9e Saint-Louis)\uc640 \ud30c\ub9ac\uc758 \uace0\ub4f1\uc0ac\ubc94\ud559\uad50\uc5d0\uc11c \uc218\ud559\ud588\ub2e4. \uadf8\ub294 \ub2c8\ucf5c\ub77c \ubd80\ub974\ubc14\ud0a4\uc758 \ucc3d\uc2dc\uc790 \uc911 \ud558\ub098\uc778 \uc559\ub9ac \uce74\ub974\ud0d5\uc744 \ud1b5\ud574 \uc704\uc0c1\uc218\ud559\uc5d0 \uc785\ubb38\ud588\ub294\ub370, \uadf8\ub97c \uc9c0\ub3c4\uad50\uc218\ub85c \ud558\uc5ec 1951\ub144 \ud30c\ub9ac \ub300\ud559\uad50\uc5d0\uc11c \ubc15\uc0ac\ud559\uc704\ub97c \ubc1b\uc558\ub2e4. \uadf8\ub54c\uc758 \ubc15\uc0ac\ud559\uc704 \ub17c\ubb38\uc5d0\ub294 \ud6d7\ub0a0 \uadf8\uc5d0\uac8c \ud544\uc988\uc0c1\uc744 \uc120\uc0ac\ud558\ub294 \ubcf4\ucda9 \uacbd\uacc4 \uc774\ub860\uc758 \uae30\ubcf8 \uc544\uc774\ub514\uc5b4\uac00 \uc774\ubbf8 \uc81c\uc2dc\ub418\uc5b4 \uc788\uc5c8\ub2e4. \ub974\ub124\ub294 \uc9c0\ub3c4\uad50 \uc218 \uc559\ub9ac \uce74\ub974\ud0d5\uc744 \ube44\ub86f\ud55c \uc5c4\uaca9\ud558\uace0 \ud615\uc2dd\uc8fc\uc758\uc801\uc778 \ub2c8\ucf5c\ub77c \ubd80\ub974\ubc14\ud0a4\ub85c\ubd80\ud130 \ub9ce\uc740 \uac83\uc744 \ubc30\uc6e0\ub2e4. \uadf8\ub7ec\ub098 \uc790\uc720\ubd84\ubc29\ud558\uba70 \uc9c1\uad00\uc801\uc778 \uc131\ud5a5\uc758 \uadf8\ub294 \uc774\ub7ec\ud55c \uacbd\ud5a5\uc5d0 \uc644\uc804\ud788 \ub3d9\uc870\ud558\uc9c0 \uc54a\uc558\ub294\ub370, \uc774\ub294 \ub098\uc911\uc5d0 \uadf8\uac00 \uc218\ud559\uc744 \ub5a0\ub098\uac8c \ub41c \uc6d0\uc778\uc774 \ub418\uc5c8\uc744 \uac83\uc774\ub2e4. \ud55c\ud3b8 \uadf8\ub294 \uadf8\ub974\ub178\ube14(1953\ub144~1954\ub144)\uacfc \uc2a4\ud2b8\ub77c\uc2a4\ubd80\ub974 \ub300\ud559\uad50(1954\ub144~1963\ub144)\uc5d0\uc11c \uad50\uc218\uc9c1\uc5d0 \uc784\uc6a9\ub418\uc5b4 \uc218\ud559\uc744 \uac00\ub974\ucce4\ub2e4. 1964\ub144 \ud30c\ub9ac \uadfc\uad50 \ubdd4\ub974\uc26c\ub974\uc774\ubca0\ud2b8\uc5d0 \uc704\uce58\ud55c \uc5f0\uad6c\uc18c IH\u00c9S\ub85c \uc62e\uaca8 \ubc88\uac70\ub85c\uc6b4 \uc758\ubb34\uc5d0 \uc2dc\ub2ec\ub9ac\uc9c0 \uc54a\uace0 \uc8fd\uc744 \ub54c\uae4c\uc9c0 \uc790\uc720\ub86d\uac8c \uc5f0\uad6c\ud560 \uc218 \uc788\uac8c \ub418\uc5c8\ub2e4. 1976\ub144\uc5d0 \uadf8\ub294 \ud504\ub791\uc2a4 \uacfc\ud559 \uc544\uce74\ub370\ubbf8 \ud68c\uc6d0\uc774 \ub418\uc5c8\ub2e4."} {"question": "\ube45\ud1a0\ub974 \uc548\uc740 \ucd1d \uba87\ubc88\uc758 \uc6b0\uc2b9\uc744 \ud558\uc600\uc744\uae4c\uc694?", "paragraph": "\ube45\ud1a0\ub974 \uc548(\ub7ec\uc2dc\uc544\uc5b4: \u0412\u0438\u043a\u0442\u043e\u0440 \u0410\u043d, 1985\ub144 11\uc6d4 23\uc77c ~ )\uc740 \ub300\ud55c\ubbfc\uad6d \ucd9c\uc2e0\uc758 \ub7ec\uc2dc\uc544 \uc1fc\ud2b8\ud2b8\ub799 \uc120\uc218\uc774\ub2e4. \uadc0\ud654 \uc804 \uc774\ub984\uc740 \uc548\ud604\uc218(\u5b89\u8ce2\u6d19)\uc600\uc73c\uba70, \ub300\ud55c\ubbfc\uad6d \uad6d\uc801\uc77c \ub2f9\uc2dc 2006\ub144 \ub3d9\uacc4 \uc62c\ub9bc\ud53d\uc5d0\uc11c \ub300\ud55c\ubbfc\uad6d \uc1fc\ud2b8\ud2b8\ub799 \uad6d\uac00\ub300\ud45c \uc120\uc218\ub85c \ucd9c\uc804\ud558\uc5ec \uae08\uba54\ub2ec 3\uac1c, \ub3d9\uba54\ub2ec 1\uac1c\ub97c \ud68d\ub4dd\ud558\uace0, 2003\ub144\ubd80\ud130 2007\ub144\uae4c\uc9c0 \uc138\uacc4 \uc1fc\ud2b8\ud2b8\ub799 \uc120\uc218\uad8c \ub300\ud68c\ub97c 5\uc5f0\uc18d \uc81c\ud328\ud558\uc600\uc73c\uba70, 2003\ub144 \uc1fc\ud2b8\ud2b8\ub799 \ub0a8\uc790 1500m\uc640 3000m\uc5d0\uc11c \uc138\uacc4\uc2e0\uae30\ub85d\uc744 \uc218\ub9bd\ud558\uc600\ub2e4.(\uc774 \uae30\ub85d\ub4e4\uc740 \ubaa8\ub450 2011\ub144 \ub300\ud55c\ubbfc\uad6d\uc758 \uc1fc\ud2b8\ud2b8\ub799 \uc120\uc218 \ub178\uc9c4\uaddc\ub85c\ubd80\ud130 \uae68\uc9c0\uac8c \ub41c\ub2e4.) 2011\ub144 \ub300\ud55c\ubbfc\uad6d \uad6d\uc801\uc744 \ud3ec\uae30\ud558\uace0 \ub7ec\uc2dc\uc544\ub85c \uadc0\ud654\ud558\uc5ec \ub7ec\uc2dc\uc544 \uad6d\uac00\ub300\ud45c\ub85c \ud65c\ub3d9 \uc911\uc774\ub2e4. 2014\ub144 \ub3d9\uacc4 \uc62c\ub9bc\ud53d\uc5d0\uc11c \ub7ec\uc2dc\uc544 \uad6d\uac00\ub300\ud45c \uc120\uc218\ub85c \ucd9c\uc804\ud558\uc5ec \uae08\uba54\ub2ec 3\uac1c, \ub3d9\uba54\ub2ec 1\uac1c\ub97c \ud68d\ub4dd\ud558\uc5ec \ud1a0\ub9ac\ub178 \ub300\ud68c \uc774\ud6c4 \ub2e4\uc2dc \ud55c \ubc88 3\uad00\uc655\uc5d0 \uc624\ub978\ub370 \uc774\uc5b4 2014\ub144 \uc138\uacc4 \uc1fc\ud2b8\ud2b8\ub799 \uc120\uc218\uad8c \ub300\ud68c\uc5d0\uc11c \uc6b0\uc2b9\uc744 \ud558\uac8c \ub418\uba70 \ucd1d \uc5ec\uc12f \ubc88\uc758 \uc6b0\uc2b9\uc744 \ucc28\uc9c0\ud588\ub2e4. \ub530\ub77c\uc11c \ube45\ud1a0\ub974 \uc548\uc740 \ud604\uc7ac \uc62c\ub9bc\ud53d\uc5d0\uc11c\ub9cc 8\uac1c\uc758 \uba54\ub2ec\uc744 \ub530 \uc1fc\ud2b8\ud2b8\ub799\uc5d0\uc11c\ub294 \ubbf8\uad6d\uc758 \uc544\ud3f4\ub85c \uc548\ud1a4 \uc624\ub178\uc640 \uba54\ub2ec \uc218\uc5d0\uc11c \ud0c0\uc774 \uae30\ub85d\uc744 \uc138\uc6e0\ub2e4.", "answer": "\uc5ec\uc12f \ubc88", "sentence": "\ucd1d \uc5ec\uc12f \ubc88 \uc758 \uc6b0\uc2b9\uc744 \ucc28\uc9c0\ud588\ub2e4.", "paragraph_sentence": "\ube45\ud1a0\ub974 \uc548(\ub7ec\uc2dc\uc544\uc5b4: \u0412\u0438\u043a\u0442\u043e\u0440 \u0410\u043d, 1985\ub144 11\uc6d4 23\uc77c ~ )\uc740 \ub300\ud55c\ubbfc\uad6d \ucd9c\uc2e0\uc758 \ub7ec\uc2dc\uc544 \uc1fc\ud2b8\ud2b8\ub799 \uc120\uc218\uc774\ub2e4. \uadc0\ud654 \uc804 \uc774\ub984\uc740 \uc548\ud604\uc218(\u5b89\u8ce2\u6d19)\uc600\uc73c\uba70, \ub300\ud55c\ubbfc\uad6d \uad6d\uc801\uc77c \ub2f9\uc2dc 2006\ub144 \ub3d9\uacc4 \uc62c\ub9bc\ud53d\uc5d0\uc11c \ub300\ud55c\ubbfc\uad6d \uc1fc\ud2b8\ud2b8\ub799 \uad6d\uac00\ub300\ud45c \uc120\uc218\ub85c \ucd9c\uc804\ud558\uc5ec \uae08\uba54\ub2ec 3\uac1c, \ub3d9\uba54\ub2ec 1\uac1c\ub97c \ud68d\ub4dd\ud558\uace0, 2003\ub144\ubd80\ud130 2007\ub144\uae4c\uc9c0 \uc138\uacc4 \uc1fc\ud2b8\ud2b8\ub799 \uc120\uc218\uad8c \ub300\ud68c\ub97c 5\uc5f0\uc18d \uc81c\ud328\ud558\uc600\uc73c\uba70, 2003\ub144 \uc1fc\ud2b8\ud2b8\ub799 \ub0a8\uc790 1500m\uc640 3000m\uc5d0\uc11c \uc138\uacc4\uc2e0\uae30\ub85d\uc744 \uc218\ub9bd\ud558\uc600\ub2e4.(\uc774 \uae30\ub85d\ub4e4\uc740 \ubaa8\ub450 2011\ub144 \ub300\ud55c\ubbfc\uad6d\uc758 \uc1fc\ud2b8\ud2b8\ub799 \uc120\uc218 \ub178\uc9c4\uaddc\ub85c\ubd80\ud130 \uae68\uc9c0\uac8c \ub41c\ub2e4. ) 2011\ub144 \ub300\ud55c\ubbfc\uad6d \uad6d\uc801\uc744 \ud3ec\uae30\ud558\uace0 \ub7ec\uc2dc\uc544\ub85c \uadc0\ud654\ud558\uc5ec \ub7ec\uc2dc\uc544 \uad6d\uac00\ub300\ud45c\ub85c \ud65c\ub3d9 \uc911\uc774\ub2e4. 2014\ub144 \ub3d9\uacc4 \uc62c\ub9bc\ud53d\uc5d0\uc11c \ub7ec\uc2dc\uc544 \uad6d\uac00\ub300\ud45c \uc120\uc218\ub85c \ucd9c\uc804\ud558\uc5ec \uae08\uba54\ub2ec 3\uac1c, \ub3d9\uba54\ub2ec 1\uac1c\ub97c \ud68d\ub4dd\ud558\uc5ec \ud1a0\ub9ac\ub178 \ub300\ud68c \uc774\ud6c4 \ub2e4\uc2dc \ud55c \ubc88 3\uad00\uc655\uc5d0 \uc624\ub978\ub370 \uc774\uc5b4 2014\ub144 \uc138\uacc4 \uc1fc\ud2b8\ud2b8\ub799 \uc120\uc218\uad8c \ub300\ud68c\uc5d0\uc11c \uc6b0\uc2b9\uc744 \ud558\uac8c \ub418\uba70 \ucd1d \uc5ec\uc12f \ubc88 \uc758 \uc6b0\uc2b9\uc744 \ucc28\uc9c0\ud588\ub2e4. \ub530\ub77c\uc11c \ube45\ud1a0\ub974 \uc548\uc740 \ud604\uc7ac \uc62c\ub9bc\ud53d\uc5d0\uc11c\ub9cc 8\uac1c\uc758 \uba54\ub2ec\uc744 \ub530 \uc1fc\ud2b8\ud2b8\ub799\uc5d0\uc11c\ub294 \ubbf8\uad6d\uc758 \uc544\ud3f4\ub85c \uc548\ud1a4 \uc624\ub178\uc640 \uba54\ub2ec \uc218\uc5d0\uc11c \ud0c0\uc774 \uae30\ub85d\uc744 \uc138\uc6e0\ub2e4.", "paragraph_answer": "\ube45\ud1a0\ub974 \uc548(\ub7ec\uc2dc\uc544\uc5b4: \u0412\u0438\u043a\u0442\u043e\u0440 \u0410\u043d, 1985\ub144 11\uc6d4 23\uc77c ~ )\uc740 \ub300\ud55c\ubbfc\uad6d \ucd9c\uc2e0\uc758 \ub7ec\uc2dc\uc544 \uc1fc\ud2b8\ud2b8\ub799 \uc120\uc218\uc774\ub2e4. \uadc0\ud654 \uc804 \uc774\ub984\uc740 \uc548\ud604\uc218(\u5b89\u8ce2\u6d19)\uc600\uc73c\uba70, \ub300\ud55c\ubbfc\uad6d \uad6d\uc801\uc77c \ub2f9\uc2dc 2006\ub144 \ub3d9\uacc4 \uc62c\ub9bc\ud53d\uc5d0\uc11c \ub300\ud55c\ubbfc\uad6d \uc1fc\ud2b8\ud2b8\ub799 \uad6d\uac00\ub300\ud45c \uc120\uc218\ub85c \ucd9c\uc804\ud558\uc5ec \uae08\uba54\ub2ec 3\uac1c, \ub3d9\uba54\ub2ec 1\uac1c\ub97c \ud68d\ub4dd\ud558\uace0, 2003\ub144\ubd80\ud130 2007\ub144\uae4c\uc9c0 \uc138\uacc4 \uc1fc\ud2b8\ud2b8\ub799 \uc120\uc218\uad8c \ub300\ud68c\ub97c 5\uc5f0\uc18d \uc81c\ud328\ud558\uc600\uc73c\uba70, 2003\ub144 \uc1fc\ud2b8\ud2b8\ub799 \ub0a8\uc790 1500m\uc640 3000m\uc5d0\uc11c \uc138\uacc4\uc2e0\uae30\ub85d\uc744 \uc218\ub9bd\ud558\uc600\ub2e4.(\uc774 \uae30\ub85d\ub4e4\uc740 \ubaa8\ub450 2011\ub144 \ub300\ud55c\ubbfc\uad6d\uc758 \uc1fc\ud2b8\ud2b8\ub799 \uc120\uc218 \ub178\uc9c4\uaddc\ub85c\ubd80\ud130 \uae68\uc9c0\uac8c \ub41c\ub2e4.) 2011\ub144 \ub300\ud55c\ubbfc\uad6d \uad6d\uc801\uc744 \ud3ec\uae30\ud558\uace0 \ub7ec\uc2dc\uc544\ub85c \uadc0\ud654\ud558\uc5ec \ub7ec\uc2dc\uc544 \uad6d\uac00\ub300\ud45c\ub85c \ud65c\ub3d9 \uc911\uc774\ub2e4. 2014\ub144 \ub3d9\uacc4 \uc62c\ub9bc\ud53d\uc5d0\uc11c \ub7ec\uc2dc\uc544 \uad6d\uac00\ub300\ud45c \uc120\uc218\ub85c \ucd9c\uc804\ud558\uc5ec \uae08\uba54\ub2ec 3\uac1c, \ub3d9\uba54\ub2ec 1\uac1c\ub97c \ud68d\ub4dd\ud558\uc5ec \ud1a0\ub9ac\ub178 \ub300\ud68c \uc774\ud6c4 \ub2e4\uc2dc \ud55c \ubc88 3\uad00\uc655\uc5d0 \uc624\ub978\ub370 \uc774\uc5b4 2014\ub144 \uc138\uacc4 \uc1fc\ud2b8\ud2b8\ub799 \uc120\uc218\uad8c \ub300\ud68c\uc5d0\uc11c \uc6b0\uc2b9\uc744 \ud558\uac8c \ub418\uba70 \ucd1d \uc5ec\uc12f \ubc88 \uc758 \uc6b0\uc2b9\uc744 \ucc28\uc9c0\ud588\ub2e4. \ub530\ub77c\uc11c \ube45\ud1a0\ub974 \uc548\uc740 \ud604\uc7ac \uc62c\ub9bc\ud53d\uc5d0\uc11c\ub9cc 8\uac1c\uc758 \uba54\ub2ec\uc744 \ub530 \uc1fc\ud2b8\ud2b8\ub799\uc5d0\uc11c\ub294 \ubbf8\uad6d\uc758 \uc544\ud3f4\ub85c \uc548\ud1a4 \uc624\ub178\uc640 \uba54\ub2ec \uc218\uc5d0\uc11c \ud0c0\uc774 \uae30\ub85d\uc744 \uc138\uc6e0\ub2e4.", "sentence_answer": "\ucd1d \uc5ec\uc12f \ubc88 \uc758 \uc6b0\uc2b9\uc744 \ucc28\uc9c0\ud588\ub2e4.", "paragraph_id": "6128328-0-0", "paragraph_question": "question: \ube45\ud1a0\ub974 \uc548\uc740 \ucd1d \uba87\ubc88\uc758 \uc6b0\uc2b9\uc744 \ud558\uc600\uc744\uae4c\uc694?, context: \ube45\ud1a0\ub974 \uc548(\ub7ec\uc2dc\uc544\uc5b4: \u0412\u0438\u043a\u0442\u043e\u0440 \u0410\u043d, 1985\ub144 11\uc6d4 23\uc77c ~ )\uc740 \ub300\ud55c\ubbfc\uad6d \ucd9c\uc2e0\uc758 \ub7ec\uc2dc\uc544 \uc1fc\ud2b8\ud2b8\ub799 \uc120\uc218\uc774\ub2e4. \uadc0\ud654 \uc804 \uc774\ub984\uc740 \uc548\ud604\uc218(\u5b89\u8ce2\u6d19)\uc600\uc73c\uba70, \ub300\ud55c\ubbfc\uad6d \uad6d\uc801\uc77c \ub2f9\uc2dc 2006\ub144 \ub3d9\uacc4 \uc62c\ub9bc\ud53d\uc5d0\uc11c \ub300\ud55c\ubbfc\uad6d \uc1fc\ud2b8\ud2b8\ub799 \uad6d\uac00\ub300\ud45c \uc120\uc218\ub85c \ucd9c\uc804\ud558\uc5ec \uae08\uba54\ub2ec 3\uac1c, \ub3d9\uba54\ub2ec 1\uac1c\ub97c \ud68d\ub4dd\ud558\uace0, 2003\ub144\ubd80\ud130 2007\ub144\uae4c\uc9c0 \uc138\uacc4 \uc1fc\ud2b8\ud2b8\ub799 \uc120\uc218\uad8c \ub300\ud68c\ub97c 5\uc5f0\uc18d \uc81c\ud328\ud558\uc600\uc73c\uba70, 2003\ub144 \uc1fc\ud2b8\ud2b8\ub799 \ub0a8\uc790 1500m\uc640 3000m\uc5d0\uc11c \uc138\uacc4\uc2e0\uae30\ub85d\uc744 \uc218\ub9bd\ud558\uc600\ub2e4.(\uc774 \uae30\ub85d\ub4e4\uc740 \ubaa8\ub450 2011\ub144 \ub300\ud55c\ubbfc\uad6d\uc758 \uc1fc\ud2b8\ud2b8\ub799 \uc120\uc218 \ub178\uc9c4\uaddc\ub85c\ubd80\ud130 \uae68\uc9c0\uac8c \ub41c\ub2e4.) 2011\ub144 \ub300\ud55c\ubbfc\uad6d \uad6d\uc801\uc744 \ud3ec\uae30\ud558\uace0 \ub7ec\uc2dc\uc544\ub85c \uadc0\ud654\ud558\uc5ec \ub7ec\uc2dc\uc544 \uad6d\uac00\ub300\ud45c\ub85c \ud65c\ub3d9 \uc911\uc774\ub2e4. 2014\ub144 \ub3d9\uacc4 \uc62c\ub9bc\ud53d\uc5d0\uc11c \ub7ec\uc2dc\uc544 \uad6d\uac00\ub300\ud45c \uc120\uc218\ub85c \ucd9c\uc804\ud558\uc5ec \uae08\uba54\ub2ec 3\uac1c, \ub3d9\uba54\ub2ec 1\uac1c\ub97c \ud68d\ub4dd\ud558\uc5ec \ud1a0\ub9ac\ub178 \ub300\ud68c \uc774\ud6c4 \ub2e4\uc2dc \ud55c \ubc88 3\uad00\uc655\uc5d0 \uc624\ub978\ub370 \uc774\uc5b4 2014\ub144 \uc138\uacc4 \uc1fc\ud2b8\ud2b8\ub799 \uc120\uc218\uad8c \ub300\ud68c\uc5d0\uc11c \uc6b0\uc2b9\uc744 \ud558\uac8c \ub418\uba70 \ucd1d \uc5ec\uc12f \ubc88\uc758 \uc6b0\uc2b9\uc744 \ucc28\uc9c0\ud588\ub2e4. \ub530\ub77c\uc11c \ube45\ud1a0\ub974 \uc548\uc740 \ud604\uc7ac \uc62c\ub9bc\ud53d\uc5d0\uc11c\ub9cc 8\uac1c\uc758 \uba54\ub2ec\uc744 \ub530 \uc1fc\ud2b8\ud2b8\ub799\uc5d0\uc11c\ub294 \ubbf8\uad6d\uc758 \uc544\ud3f4\ub85c \uc548\ud1a4 \uc624\ub178\uc640 \uba54\ub2ec \uc218\uc5d0\uc11c \ud0c0\uc774 \uae30\ub85d\uc744 \uc138\uc6e0\ub2e4."} {"question": "\ud55c\uad6d \ud504\ub85c \ubc30\uad6c\uc5d0\uc11c \uc2b9\ubd80\uc870\uc791\uc774 \uc0ac\uc2e4\ub85c \ud655\uc778\ub41c \ub0a0\uc9dc\ub294?", "paragraph": "2012\ub144 2\uc6d4 16\uc77c, \ud504\ub85c \ubc30\uad6c\uacc4\uc5d0\uc11c \uc2b9\ubd80\uc870\uc791 \uba87 \uac74\uc774 \uc0ac\uc2e4\ub85c \ud655\uc778\ub418\uba74\uc11c \uc57c\uad6c\uacc4\uc5d0 \ud5a5\ud55c \uc758\ud639\uc758 \uc2dc\uc120\ub3c4 \ud6e8\uc52c \ub9ce\uc544\uc84c\ub2e4. \ub610\ud55c \ub300\uad6c\uc9c0\uac80 \uac80\ucc30\uc740 \ubc15\ud604\uc900\uacfc \uae40\uc131\ud604\uc5d0\uac8c \uccab \ud68c \ubcfc\ub137\uc758 \ub300\uac00\ub85c \uc218\ubc31\ub9cc\uc6d0\uc744 \uc9c0\uae09\ud588\ub2e4\ub294 \ud55c \ube0c\ub85c\ucee4\uc758 \uc9c4\uc220\uc744 \ud655\ubcf4\ud558\uba74\uc11c, \uad6c\uccb4\uc801\uc778 \uc9c4\uc220\uc774 \ud655\ubcf4\ub418\uc5b4 \uc758\ud639\uc744 \uc0ac\uace0 \uc788\ub294 \ubd80\ubd84\uc5d0 \ub300\ud574\uc11c \uc218\uc0ac\uc5d0 \ucc29\uc218\ud558\uba74\uc11c \ubcf8\uaca9\uc801\uc778 \uc758\ud639 \ud30c\ud5e4\uce58\uae30\uc5d0 \ub098\uc130\ub2e4. \ubc15\ud604\uc900\uacfc \uae40\uc131\ud604\ub9cc\uc774 \uc9c4\uc220\uc744 \ud1a0\ub300\ub85c \ubcf8\uaca9 \uc218\uc0ac \ub300\uc0c1\uc5d0 \ud3ec\ud568\ub418\uc5b4 \uac80\ucc30\uc5d0 \uc18c\ud658\ub420 \ubc29\uce68\uc774\uc5c8\uc73c\ub098, \uc2b9\ubd80\uc870\uc791\uc744 \uc81c\uc758\ud55c \uc0ac\ub840\uc5d0 \ub300\ud574 \uc18d\uc18d\ud788 \uc81c\ubcf4\uac00 \ub4e4\uc5b4\uc624\uba74\uc11c \ud504\ub85c \uc57c\uad6c\uacc4 \uc804\uccb4\ub85c \uc218\uc0ac\uac00 \ud655\ub300\ub420 \uac00\ub2a5\uc131\uc774 \ucee4\uc84c\ub2e4. \ub610\ud55c \uc2b9\ubd80\uc870\uc791 \uc81c\uc758\ub97c \ubc1b\uc558\ub2e4\uace0 \uace0\ubc31\ud55c \ubb38\uc131\ud604\ub3c4 \ucc38\uace0\uc778 \uc790\uaca9\uc73c\ub85c \uc18c\ud658\ub420 \ubc29\uce68\uc784\uc774 \uc54c\ub824\uc84c\ub2e4.", "answer": "2012\ub144 2\uc6d4 16\uc77c", "sentence": "2012\ub144 2\uc6d4 16\uc77c ,", "paragraph_sentence": " 2012\ub144 2\uc6d4 16\uc77c , \ud504\ub85c \ubc30\uad6c\uacc4\uc5d0\uc11c \uc2b9\ubd80\uc870\uc791 \uba87 \uac74\uc774 \uc0ac\uc2e4\ub85c \ud655\uc778\ub418\uba74\uc11c \uc57c\uad6c\uacc4\uc5d0 \ud5a5\ud55c \uc758\ud639\uc758 \uc2dc\uc120\ub3c4 \ud6e8\uc52c \ub9ce\uc544\uc84c\ub2e4. \ub610\ud55c \ub300\uad6c\uc9c0\uac80 \uac80\ucc30\uc740 \ubc15\ud604\uc900\uacfc \uae40\uc131\ud604\uc5d0\uac8c \uccab \ud68c \ubcfc\ub137\uc758 \ub300\uac00\ub85c \uc218\ubc31\ub9cc\uc6d0\uc744 \uc9c0\uae09\ud588\ub2e4\ub294 \ud55c \ube0c\ub85c\ucee4\uc758 \uc9c4\uc220\uc744 \ud655\ubcf4\ud558\uba74\uc11c, \uad6c\uccb4\uc801\uc778 \uc9c4\uc220\uc774 \ud655\ubcf4\ub418\uc5b4 \uc758\ud639\uc744 \uc0ac\uace0 \uc788\ub294 \ubd80\ubd84\uc5d0 \ub300\ud574\uc11c \uc218\uc0ac\uc5d0 \ucc29\uc218\ud558\uba74\uc11c \ubcf8\uaca9\uc801\uc778 \uc758\ud639 \ud30c\ud5e4\uce58\uae30\uc5d0 \ub098\uc130\ub2e4. \ubc15\ud604\uc900\uacfc \uae40\uc131\ud604\ub9cc\uc774 \uc9c4\uc220\uc744 \ud1a0\ub300\ub85c \ubcf8\uaca9 \uc218\uc0ac \ub300\uc0c1\uc5d0 \ud3ec\ud568\ub418\uc5b4 \uac80\ucc30\uc5d0 \uc18c\ud658\ub420 \ubc29\uce68\uc774\uc5c8\uc73c\ub098, \uc2b9\ubd80\uc870\uc791\uc744 \uc81c\uc758\ud55c \uc0ac\ub840\uc5d0 \ub300\ud574 \uc18d\uc18d\ud788 \uc81c\ubcf4\uac00 \ub4e4\uc5b4\uc624\uba74\uc11c \ud504\ub85c \uc57c\uad6c\uacc4 \uc804\uccb4\ub85c \uc218\uc0ac\uac00 \ud655\ub300\ub420 \uac00\ub2a5\uc131\uc774 \ucee4\uc84c\ub2e4. \ub610\ud55c \uc2b9\ubd80\uc870\uc791 \uc81c\uc758\ub97c \ubc1b\uc558\ub2e4\uace0 \uace0\ubc31\ud55c \ubb38\uc131\ud604\ub3c4 \ucc38\uace0\uc778 \uc790\uaca9\uc73c\ub85c \uc18c\ud658\ub420 \ubc29\uce68\uc784\uc774 \uc54c\ub824\uc84c\ub2e4.", "paragraph_answer": " 2012\ub144 2\uc6d4 16\uc77c , \ud504\ub85c \ubc30\uad6c\uacc4\uc5d0\uc11c \uc2b9\ubd80\uc870\uc791 \uba87 \uac74\uc774 \uc0ac\uc2e4\ub85c \ud655\uc778\ub418\uba74\uc11c \uc57c\uad6c\uacc4\uc5d0 \ud5a5\ud55c \uc758\ud639\uc758 \uc2dc\uc120\ub3c4 \ud6e8\uc52c \ub9ce\uc544\uc84c\ub2e4. \ub610\ud55c \ub300\uad6c\uc9c0\uac80 \uac80\ucc30\uc740 \ubc15\ud604\uc900\uacfc \uae40\uc131\ud604\uc5d0\uac8c \uccab \ud68c \ubcfc\ub137\uc758 \ub300\uac00\ub85c \uc218\ubc31\ub9cc\uc6d0\uc744 \uc9c0\uae09\ud588\ub2e4\ub294 \ud55c \ube0c\ub85c\ucee4\uc758 \uc9c4\uc220\uc744 \ud655\ubcf4\ud558\uba74\uc11c, \uad6c\uccb4\uc801\uc778 \uc9c4\uc220\uc774 \ud655\ubcf4\ub418\uc5b4 \uc758\ud639\uc744 \uc0ac\uace0 \uc788\ub294 \ubd80\ubd84\uc5d0 \ub300\ud574\uc11c \uc218\uc0ac\uc5d0 \ucc29\uc218\ud558\uba74\uc11c \ubcf8\uaca9\uc801\uc778 \uc758\ud639 \ud30c\ud5e4\uce58\uae30\uc5d0 \ub098\uc130\ub2e4. \ubc15\ud604\uc900\uacfc \uae40\uc131\ud604\ub9cc\uc774 \uc9c4\uc220\uc744 \ud1a0\ub300\ub85c \ubcf8\uaca9 \uc218\uc0ac \ub300\uc0c1\uc5d0 \ud3ec\ud568\ub418\uc5b4 \uac80\ucc30\uc5d0 \uc18c\ud658\ub420 \ubc29\uce68\uc774\uc5c8\uc73c\ub098, \uc2b9\ubd80\uc870\uc791\uc744 \uc81c\uc758\ud55c \uc0ac\ub840\uc5d0 \ub300\ud574 \uc18d\uc18d\ud788 \uc81c\ubcf4\uac00 \ub4e4\uc5b4\uc624\uba74\uc11c \ud504\ub85c \uc57c\uad6c\uacc4 \uc804\uccb4\ub85c \uc218\uc0ac\uac00 \ud655\ub300\ub420 \uac00\ub2a5\uc131\uc774 \ucee4\uc84c\ub2e4. \ub610\ud55c \uc2b9\ubd80\uc870\uc791 \uc81c\uc758\ub97c \ubc1b\uc558\ub2e4\uace0 \uace0\ubc31\ud55c \ubb38\uc131\ud604\ub3c4 \ucc38\uace0\uc778 \uc790\uaca9\uc73c\ub85c \uc18c\ud658\ub420 \ubc29\uce68\uc784\uc774 \uc54c\ub824\uc84c\ub2e4.", "sentence_answer": " 2012\ub144 2\uc6d4 16\uc77c ,", "paragraph_id": "5780952-6-0", "paragraph_question": "question: \ud55c\uad6d \ud504\ub85c \ubc30\uad6c\uc5d0\uc11c \uc2b9\ubd80\uc870\uc791\uc774 \uc0ac\uc2e4\ub85c \ud655\uc778\ub41c \ub0a0\uc9dc\ub294?, context: 2012\ub144 2\uc6d4 16\uc77c, \ud504\ub85c \ubc30\uad6c\uacc4\uc5d0\uc11c \uc2b9\ubd80\uc870\uc791 \uba87 \uac74\uc774 \uc0ac\uc2e4\ub85c \ud655\uc778\ub418\uba74\uc11c \uc57c\uad6c\uacc4\uc5d0 \ud5a5\ud55c \uc758\ud639\uc758 \uc2dc\uc120\ub3c4 \ud6e8\uc52c \ub9ce\uc544\uc84c\ub2e4. \ub610\ud55c \ub300\uad6c\uc9c0\uac80 \uac80\ucc30\uc740 \ubc15\ud604\uc900\uacfc \uae40\uc131\ud604\uc5d0\uac8c \uccab \ud68c \ubcfc\ub137\uc758 \ub300\uac00\ub85c \uc218\ubc31\ub9cc\uc6d0\uc744 \uc9c0\uae09\ud588\ub2e4\ub294 \ud55c \ube0c\ub85c\ucee4\uc758 \uc9c4\uc220\uc744 \ud655\ubcf4\ud558\uba74\uc11c, \uad6c\uccb4\uc801\uc778 \uc9c4\uc220\uc774 \ud655\ubcf4\ub418\uc5b4 \uc758\ud639\uc744 \uc0ac\uace0 \uc788\ub294 \ubd80\ubd84\uc5d0 \ub300\ud574\uc11c \uc218\uc0ac\uc5d0 \ucc29\uc218\ud558\uba74\uc11c \ubcf8\uaca9\uc801\uc778 \uc758\ud639 \ud30c\ud5e4\uce58\uae30\uc5d0 \ub098\uc130\ub2e4. \ubc15\ud604\uc900\uacfc \uae40\uc131\ud604\ub9cc\uc774 \uc9c4\uc220\uc744 \ud1a0\ub300\ub85c \ubcf8\uaca9 \uc218\uc0ac \ub300\uc0c1\uc5d0 \ud3ec\ud568\ub418\uc5b4 \uac80\ucc30\uc5d0 \uc18c\ud658\ub420 \ubc29\uce68\uc774\uc5c8\uc73c\ub098, \uc2b9\ubd80\uc870\uc791\uc744 \uc81c\uc758\ud55c \uc0ac\ub840\uc5d0 \ub300\ud574 \uc18d\uc18d\ud788 \uc81c\ubcf4\uac00 \ub4e4\uc5b4\uc624\uba74\uc11c \ud504\ub85c \uc57c\uad6c\uacc4 \uc804\uccb4\ub85c \uc218\uc0ac\uac00 \ud655\ub300\ub420 \uac00\ub2a5\uc131\uc774 \ucee4\uc84c\ub2e4. \ub610\ud55c \uc2b9\ubd80\uc870\uc791 \uc81c\uc758\ub97c \ubc1b\uc558\ub2e4\uace0 \uace0\ubc31\ud55c \ubb38\uc131\ud604\ub3c4 \ucc38\uace0\uc778 \uc790\uaca9\uc73c\ub85c \uc18c\ud658\ub420 \ubc29\uce68\uc784\uc774 \uc54c\ub824\uc84c\ub2e4."} {"question": "\ub178\ubca8\uc0c1\uc744 \uc218\uc0c1\ud55c \ud649\ud0a8\uacfc \ud5c9\uc2ac\ub9ac\uc758 \ub17c\ubb38\uc5d0\uc11c \uc5b4\ub5a0\ud55c \uacf5\uc2dd\uc744 \uc0ac\uc6a9\ud558\ub294\ub370 \uc788\uc5b4\uc11c \uc6d0\uc800\ub97c \ubc1d\ud788\uc9c0 \uc54a\uc558\ub098?", "paragraph": "\uc548\ucca0\uc218\uc758 \ubc15\uc0ac\ud559\uc704 \ub17c\ubb38\uc774 \uc548\ucca0\uc218 \ub17c\ubb38\uc5d0 \ube44\ud574 2\ub144 \uc55e\uc11c \ubc15\uc0ac\ud559\uc704\ub97c \ubc1b\uc740 \uc11c\uc6b8\ub300\ud559\uad50 \uc11c \ubaa8 \uad50\uc218\uc758 \ubc15\uc0ac \ub17c\ubb38\uc744 \ud45c\uc808\ud588\ub2e4\ub294 \uc758\ud639\uc774 \uc788\ub2e4. \ubcfc\uce20\ub9cc \uace1\uc120\uc744 \uc720\ub3c4\ud558\ub294 \uc124\uba85\uc5d0\uc11c \uc720\ub3c4\uc2dd\uc744 \uc11c \uad50\uc218 \ub17c\ubb38\uc5d0\uc11c \uac70\uc758 \ubcf5\uc0ac \uc218\uc900\uc73c\ub85c 3\ud398\uc774\uc9c0\uc5d0 \uac78\uccd0 \ubca0\uaf08\ub2e4\ub294 \uac83\uc774\ub2e4. \ub354\uad70\ub2e4\ub098 \uc11c \uad50\uc218\uc758 \ub17c\ubb38\uc5d0\uc11c \uae30\uc220\ud55c \ubcfc\uce20\ub9cc \uacf5\uc2dd\uc740 \ub300\uad04\ud638\uac00 \ud558\ub098 \ube60\uc838\uc11c \uc624\ub958\uac00 \uc788\ub294 \uacf5\uc2dd\uc778\ub370, \uacf5\uad50\ub86d\uac8c\ub3c4 \uc548\ucca0\uc218\uc758 \ub17c\ubb38\uc5d0\uc11c\ub3c4 \uac19\uc740 \uacf5\uc2dd\uc5d0 \ub300\uad04\ud638\uac00 \ube60\uc838 \uc788\uc5c8\ub2e4. \ub530\ub77c\uc11c \ub17c\ubb38\uc744 \ubca0\ub07c\ub2e4\uac00 \uc624\ub958\uae4c\uc9c0 \ubca0\uaf08\ub2e4\ub294 \uc758\ud639\uc774 \uc788\ub2e4. \uadf8\ub7ec\ub098 \uc548\ucca0\uc218 \uce21\uc740 \ubcfc\uce20\ub9cc \uacf5\uc2dd\uc740 \ubcf4\ud3b8\uc801\uc73c\ub85c \uc54c\ub824\uc9c4 \uacf5\uc2dd\uc774\ub77c\uba70 \ub178\ubca8\uc0c1\uc744 \uc218\uc0c1\ud55c \ud649\ud0a8\uacfc \ud5c9\uc2ac\ub9ac\uc758 \ub17c\ubb38\uc5d0\uc11c\ub3c4 \ubcfc\uce20\ub9cc \uacf5\uc2dd\uc744 \uc0ac\uc6a9\ud558\ub294\ub370 \uc788\uc5b4\uc11c \uc6d0\uc800\ub97c \ubc1d\ud788\uc9c0 \uc54a\uc558\uace0, \uadf8\uac83\uc774 \uad00\ud589\uc774\ub77c\uace0 \ubc18\ubc15\ud558\uc600\ub2e4.", "answer": "\ubcfc\uce20\ub9cc \uacf5\uc2dd", "sentence": "\ub354\uad70\ub2e4\ub098 \uc11c \uad50\uc218\uc758 \ub17c\ubb38\uc5d0\uc11c \uae30\uc220\ud55c \ubcfc\uce20\ub9cc \uacf5\uc2dd \uc740 \ub300\uad04\ud638\uac00 \ud558\ub098 \ube60\uc838\uc11c \uc624\ub958\uac00 \uc788\ub294 \uacf5\uc2dd\uc778\ub370, \uacf5\uad50\ub86d\uac8c\ub3c4 \uc548\ucca0\uc218\uc758 \ub17c\ubb38\uc5d0\uc11c\ub3c4 \uac19\uc740 \uacf5\uc2dd\uc5d0 \ub300\uad04\ud638\uac00 \ube60\uc838 \uc788\uc5c8\ub2e4.", "paragraph_sentence": "\uc548\ucca0\uc218\uc758 \ubc15\uc0ac\ud559\uc704 \ub17c\ubb38\uc774 \uc548\ucca0\uc218 \ub17c\ubb38\uc5d0 \ube44\ud574 2\ub144 \uc55e\uc11c \ubc15\uc0ac\ud559\uc704\ub97c \ubc1b\uc740 \uc11c\uc6b8\ub300\ud559\uad50 \uc11c \ubaa8 \uad50\uc218\uc758 \ubc15\uc0ac \ub17c\ubb38\uc744 \ud45c\uc808\ud588\ub2e4\ub294 \uc758\ud639\uc774 \uc788\ub2e4. \ubcfc\uce20\ub9cc \uace1\uc120\uc744 \uc720\ub3c4\ud558\ub294 \uc124\uba85\uc5d0\uc11c \uc720\ub3c4\uc2dd\uc744 \uc11c \uad50\uc218 \ub17c\ubb38\uc5d0\uc11c \uac70\uc758 \ubcf5\uc0ac \uc218\uc900\uc73c\ub85c 3\ud398\uc774\uc9c0\uc5d0 \uac78\uccd0 \ubca0\uaf08\ub2e4\ub294 \uac83\uc774\ub2e4. \ub354\uad70\ub2e4\ub098 \uc11c \uad50\uc218\uc758 \ub17c\ubb38\uc5d0\uc11c \uae30\uc220\ud55c \ubcfc\uce20\ub9cc \uacf5\uc2dd \uc740 \ub300\uad04\ud638\uac00 \ud558\ub098 \ube60\uc838\uc11c \uc624\ub958\uac00 \uc788\ub294 \uacf5\uc2dd\uc778\ub370, \uacf5\uad50\ub86d\uac8c\ub3c4 \uc548\ucca0\uc218\uc758 \ub17c\ubb38\uc5d0\uc11c\ub3c4 \uac19\uc740 \uacf5\uc2dd\uc5d0 \ub300\uad04\ud638\uac00 \ube60\uc838 \uc788\uc5c8\ub2e4. \ub530\ub77c\uc11c \ub17c\ubb38\uc744 \ubca0\ub07c\ub2e4\uac00 \uc624\ub958\uae4c\uc9c0 \ubca0\uaf08\ub2e4\ub294 \uc758\ud639\uc774 \uc788\ub2e4. \uadf8\ub7ec\ub098 \uc548\ucca0\uc218 \uce21\uc740 \ubcfc\uce20\ub9cc \uacf5\uc2dd\uc740 \ubcf4\ud3b8\uc801\uc73c\ub85c \uc54c\ub824\uc9c4 \uacf5\uc2dd\uc774\ub77c\uba70 \ub178\ubca8\uc0c1\uc744 \uc218\uc0c1\ud55c \ud649\ud0a8\uacfc \ud5c9\uc2ac\ub9ac\uc758 \ub17c\ubb38\uc5d0\uc11c\ub3c4 \ubcfc\uce20\ub9cc \uacf5\uc2dd\uc744 \uc0ac\uc6a9\ud558\ub294\ub370 \uc788\uc5b4\uc11c \uc6d0\uc800\ub97c \ubc1d\ud788\uc9c0 \uc54a\uc558\uace0, \uadf8\uac83\uc774 \uad00\ud589\uc774\ub77c\uace0 \ubc18\ubc15\ud558\uc600\ub2e4.", "paragraph_answer": "\uc548\ucca0\uc218\uc758 \ubc15\uc0ac\ud559\uc704 \ub17c\ubb38\uc774 \uc548\ucca0\uc218 \ub17c\ubb38\uc5d0 \ube44\ud574 2\ub144 \uc55e\uc11c \ubc15\uc0ac\ud559\uc704\ub97c \ubc1b\uc740 \uc11c\uc6b8\ub300\ud559\uad50 \uc11c \ubaa8 \uad50\uc218\uc758 \ubc15\uc0ac \ub17c\ubb38\uc744 \ud45c\uc808\ud588\ub2e4\ub294 \uc758\ud639\uc774 \uc788\ub2e4. \ubcfc\uce20\ub9cc \uace1\uc120\uc744 \uc720\ub3c4\ud558\ub294 \uc124\uba85\uc5d0\uc11c \uc720\ub3c4\uc2dd\uc744 \uc11c \uad50\uc218 \ub17c\ubb38\uc5d0\uc11c \uac70\uc758 \ubcf5\uc0ac \uc218\uc900\uc73c\ub85c 3\ud398\uc774\uc9c0\uc5d0 \uac78\uccd0 \ubca0\uaf08\ub2e4\ub294 \uac83\uc774\ub2e4. \ub354\uad70\ub2e4\ub098 \uc11c \uad50\uc218\uc758 \ub17c\ubb38\uc5d0\uc11c \uae30\uc220\ud55c \ubcfc\uce20\ub9cc \uacf5\uc2dd \uc740 \ub300\uad04\ud638\uac00 \ud558\ub098 \ube60\uc838\uc11c \uc624\ub958\uac00 \uc788\ub294 \uacf5\uc2dd\uc778\ub370, \uacf5\uad50\ub86d\uac8c\ub3c4 \uc548\ucca0\uc218\uc758 \ub17c\ubb38\uc5d0\uc11c\ub3c4 \uac19\uc740 \uacf5\uc2dd\uc5d0 \ub300\uad04\ud638\uac00 \ube60\uc838 \uc788\uc5c8\ub2e4. \ub530\ub77c\uc11c \ub17c\ubb38\uc744 \ubca0\ub07c\ub2e4\uac00 \uc624\ub958\uae4c\uc9c0 \ubca0\uaf08\ub2e4\ub294 \uc758\ud639\uc774 \uc788\ub2e4. \uadf8\ub7ec\ub098 \uc548\ucca0\uc218 \uce21\uc740 \ubcfc\uce20\ub9cc \uacf5\uc2dd\uc740 \ubcf4\ud3b8\uc801\uc73c\ub85c \uc54c\ub824\uc9c4 \uacf5\uc2dd\uc774\ub77c\uba70 \ub178\ubca8\uc0c1\uc744 \uc218\uc0c1\ud55c \ud649\ud0a8\uacfc \ud5c9\uc2ac\ub9ac\uc758 \ub17c\ubb38\uc5d0\uc11c\ub3c4 \ubcfc\uce20\ub9cc \uacf5\uc2dd\uc744 \uc0ac\uc6a9\ud558\ub294\ub370 \uc788\uc5b4\uc11c \uc6d0\uc800\ub97c \ubc1d\ud788\uc9c0 \uc54a\uc558\uace0, \uadf8\uac83\uc774 \uad00\ud589\uc774\ub77c\uace0 \ubc18\ubc15\ud558\uc600\ub2e4.", "sentence_answer": "\ub354\uad70\ub2e4\ub098 \uc11c \uad50\uc218\uc758 \ub17c\ubb38\uc5d0\uc11c \uae30\uc220\ud55c \ubcfc\uce20\ub9cc \uacf5\uc2dd \uc740 \ub300\uad04\ud638\uac00 \ud558\ub098 \ube60\uc838\uc11c \uc624\ub958\uac00 \uc788\ub294 \uacf5\uc2dd\uc778\ub370, \uacf5\uad50\ub86d\uac8c\ub3c4 \uc548\ucca0\uc218\uc758 \ub17c\ubb38\uc5d0\uc11c\ub3c4 \uac19\uc740 \uacf5\uc2dd\uc5d0 \ub300\uad04\ud638\uac00 \ube60\uc838 \uc788\uc5c8\ub2e4.", "paragraph_id": "6539049-25-1", "paragraph_question": "question: \ub178\ubca8\uc0c1\uc744 \uc218\uc0c1\ud55c \ud649\ud0a8\uacfc \ud5c9\uc2ac\ub9ac\uc758 \ub17c\ubb38\uc5d0\uc11c \uc5b4\ub5a0\ud55c \uacf5\uc2dd\uc744 \uc0ac\uc6a9\ud558\ub294\ub370 \uc788\uc5b4\uc11c \uc6d0\uc800\ub97c \ubc1d\ud788\uc9c0 \uc54a\uc558\ub098?, context: \uc548\ucca0\uc218\uc758 \ubc15\uc0ac\ud559\uc704 \ub17c\ubb38\uc774 \uc548\ucca0\uc218 \ub17c\ubb38\uc5d0 \ube44\ud574 2\ub144 \uc55e\uc11c \ubc15\uc0ac\ud559\uc704\ub97c \ubc1b\uc740 \uc11c\uc6b8\ub300\ud559\uad50 \uc11c \ubaa8 \uad50\uc218\uc758 \ubc15\uc0ac \ub17c\ubb38\uc744 \ud45c\uc808\ud588\ub2e4\ub294 \uc758\ud639\uc774 \uc788\ub2e4. \ubcfc\uce20\ub9cc \uace1\uc120\uc744 \uc720\ub3c4\ud558\ub294 \uc124\uba85\uc5d0\uc11c \uc720\ub3c4\uc2dd\uc744 \uc11c \uad50\uc218 \ub17c\ubb38\uc5d0\uc11c \uac70\uc758 \ubcf5\uc0ac \uc218\uc900\uc73c\ub85c 3\ud398\uc774\uc9c0\uc5d0 \uac78\uccd0 \ubca0\uaf08\ub2e4\ub294 \uac83\uc774\ub2e4. \ub354\uad70\ub2e4\ub098 \uc11c \uad50\uc218\uc758 \ub17c\ubb38\uc5d0\uc11c \uae30\uc220\ud55c \ubcfc\uce20\ub9cc \uacf5\uc2dd\uc740 \ub300\uad04\ud638\uac00 \ud558\ub098 \ube60\uc838\uc11c \uc624\ub958\uac00 \uc788\ub294 \uacf5\uc2dd\uc778\ub370, \uacf5\uad50\ub86d\uac8c\ub3c4 \uc548\ucca0\uc218\uc758 \ub17c\ubb38\uc5d0\uc11c\ub3c4 \uac19\uc740 \uacf5\uc2dd\uc5d0 \ub300\uad04\ud638\uac00 \ube60\uc838 \uc788\uc5c8\ub2e4. \ub530\ub77c\uc11c \ub17c\ubb38\uc744 \ubca0\ub07c\ub2e4\uac00 \uc624\ub958\uae4c\uc9c0 \ubca0\uaf08\ub2e4\ub294 \uc758\ud639\uc774 \uc788\ub2e4. \uadf8\ub7ec\ub098 \uc548\ucca0\uc218 \uce21\uc740 \ubcfc\uce20\ub9cc \uacf5\uc2dd\uc740 \ubcf4\ud3b8\uc801\uc73c\ub85c \uc54c\ub824\uc9c4 \uacf5\uc2dd\uc774\ub77c\uba70 \ub178\ubca8\uc0c1\uc744 \uc218\uc0c1\ud55c \ud649\ud0a8\uacfc \ud5c9\uc2ac\ub9ac\uc758 \ub17c\ubb38\uc5d0\uc11c\ub3c4 \ubcfc\uce20\ub9cc \uacf5\uc2dd\uc744 \uc0ac\uc6a9\ud558\ub294\ub370 \uc788\uc5b4\uc11c \uc6d0\uc800\ub97c \ubc1d\ud788\uc9c0 \uc54a\uc558\uace0, \uadf8\uac83\uc774 \uad00\ud589\uc774\ub77c\uace0 \ubc18\ubc15\ud558\uc600\ub2e4."} {"question": "\ubcfc\ube68\uac04\uc0ac\ucd98\uae30 \uba64\ubc84\uc758 \uc774\ub984\uc740?", "paragraph": "\ubcfc\ube68\uac04\uc0ac\ucd98\uae30\ub294 \uacbd\uc0c1\ubd81\ub3c4 \uc601\uc8fc\uc5ec\uc790\uace0\ub4f1\ud559\uad50\uc5d0 1\ud559\ub144\uc758 \uc7ac\ud559 \uc911\uc774\ub358 2011\ub144 \uba64\ubc84\ub4e4\uc758 \ud558\uad63\uae38\uc5d0 \uacb0\uc131\ub418\uc5c8\ub2e4. 2014\ub144 \uc288\ud37c\uc2a4\ud0c0k 6\uc5d0 \uacbd\ubd81 \uc601\uc8fc \uc2dc\uace8\ubc34\ub4dc \ubcfc\ube68\uac04\uc0ac\ucd98\uae30\ub77c\ub294 \uc774\ub984\uc73c\ub85c \ucd9c\uc5f0\ud588\uc73c\ub098 \ud0c8\ub77d\ud558\uc600\ub2e4. \uc774\ub54c\uae4c\uc9c0\ub9cc \ud574\ub3c4 \uad8c\uc9c0\uc724\uc774 \ud300\uc758 \uae30\ud0c0\ub97c \ub2f4\ub2f9\ud558\uace0 \uc788\uc5c8\uc73c\ub098, \ud559\uc5c5\uc73c\ub85c \uc778\ud574 \uc548\uc9c0\uc601 \uc6b0\uc9c0\uc724\ub9cc\uc774 \ub370\ubdd4\ub97c \ud558\uac8c\ub418\uc5c8\ub2e4. \uc288\ud37c\uc2a4\ud0c0k6 \ucd9c\uc5f0 \uc774\ud6c4 1\ub144 6\uac1c\uc6d4\uc774 \uc9c0\ub098\uba74\uc11c \ub370\ubdd4\uc5d0 \ub300\ud55c \ud300 \ub0b4\uc5d0 \ubd88\uc548\uac10\uc774 \uc870\uc131\ub418\uc5c8\uace0 \ub54c\ubb38\uc5d0 \uace1 \uc791\uc5c5\uc5d0 \ub9ce\uc740 \uc2dc\uac04\uc744 \ud560\uc560\ud574\uc57c\ub9cc \ud588\ub2e4. \uc18c\uc18d\uc0ac\ub294 \ud504\ub85c\ub4c0\uc11c\ub97c \uc81c\uacf5\ud558\uace0 \ub370\ubdd4 \uc568\ubc94\uc744 \uc900\ube44\ud558\ub3c4\ub85d \ud558\uc600\uc73c\ub098 \uc74c\uc545\uc801 \uc0c9\uae54\uc774 \ub9de\uc9c0 \uc54a\uc544 \uace1\uc744 \uc0c8\ub85c \uc4f0\uac8c \ub418\uc5c8\ub2e4. \uace1\uc744 \ub9ce\uc774, \uae09\ud558\uac8c \uace1\uc744 \uc4f0\uac8c \ub418\uc5c8\ub294\ub370 \uc548\uc9c0\uc601\uc740 \uc774\ub7ec\ud55c \uc791\uc5c5\uc774 \ud300\uc758 \uc0c9\uae54\uc744 \uc7a1\ub294 \ub370 \uc7ac\uc0b0\uc774 \ub418\uc5c8\ub2e4\uace0 \ub9d0\ud588\ub2e4.", "answer": "\uc548\uc9c0\uc601 \uc6b0\uc9c0\uc724", "sentence": "\uc774\ub54c\uae4c\uc9c0\ub9cc \ud574\ub3c4 \uad8c\uc9c0\uc724\uc774 \ud300\uc758 \uae30\ud0c0\ub97c \ub2f4\ub2f9\ud558\uace0 \uc788\uc5c8\uc73c\ub098, \ud559\uc5c5\uc73c\ub85c \uc778\ud574 \uc548\uc9c0\uc601 \uc6b0\uc9c0\uc724 \ub9cc\uc774 \ub370\ubdd4\ub97c \ud558\uac8c\ub418\uc5c8\ub2e4.", "paragraph_sentence": "\ubcfc\ube68\uac04\uc0ac\ucd98\uae30\ub294 \uacbd\uc0c1\ubd81\ub3c4 \uc601\uc8fc\uc5ec\uc790\uace0\ub4f1\ud559\uad50\uc5d0 1\ud559\ub144\uc758 \uc7ac\ud559 \uc911\uc774\ub358 2011\ub144 \uba64\ubc84\ub4e4\uc758 \ud558\uad63\uae38\uc5d0 \uacb0\uc131\ub418\uc5c8\ub2e4. 2014\ub144 \uc288\ud37c\uc2a4\ud0c0k 6\uc5d0 \uacbd\ubd81 \uc601\uc8fc \uc2dc\uace8\ubc34\ub4dc \ubcfc\ube68\uac04\uc0ac\ucd98\uae30\ub77c\ub294 \uc774\ub984\uc73c\ub85c \ucd9c\uc5f0\ud588\uc73c\ub098 \ud0c8\ub77d\ud558\uc600\ub2e4. \uc774\ub54c\uae4c\uc9c0\ub9cc \ud574\ub3c4 \uad8c\uc9c0\uc724\uc774 \ud300\uc758 \uae30\ud0c0\ub97c \ub2f4\ub2f9\ud558\uace0 \uc788\uc5c8\uc73c\ub098, \ud559\uc5c5\uc73c\ub85c \uc778\ud574 \uc548\uc9c0\uc601 \uc6b0\uc9c0\uc724 \ub9cc\uc774 \ub370\ubdd4\ub97c \ud558\uac8c\ub418\uc5c8\ub2e4. \uc288\ud37c\uc2a4\ud0c0k6 \ucd9c\uc5f0 \uc774\ud6c4 1\ub144 6\uac1c\uc6d4\uc774 \uc9c0\ub098\uba74\uc11c \ub370\ubdd4\uc5d0 \ub300\ud55c \ud300 \ub0b4\uc5d0 \ubd88\uc548\uac10\uc774 \uc870\uc131\ub418\uc5c8\uace0 \ub54c\ubb38\uc5d0 \uace1 \uc791\uc5c5\uc5d0 \ub9ce\uc740 \uc2dc\uac04\uc744 \ud560\uc560\ud574\uc57c\ub9cc \ud588\ub2e4. \uc18c\uc18d\uc0ac\ub294 \ud504\ub85c\ub4c0\uc11c\ub97c \uc81c\uacf5\ud558\uace0 \ub370\ubdd4 \uc568\ubc94\uc744 \uc900\ube44\ud558\ub3c4\ub85d \ud558\uc600\uc73c\ub098 \uc74c\uc545\uc801 \uc0c9\uae54\uc774 \ub9de\uc9c0 \uc54a\uc544 \uace1\uc744 \uc0c8\ub85c \uc4f0\uac8c \ub418\uc5c8\ub2e4. \uace1\uc744 \ub9ce\uc774, \uae09\ud558\uac8c \uace1\uc744 \uc4f0\uac8c \ub418\uc5c8\ub294\ub370 \uc548\uc9c0\uc601\uc740 \uc774\ub7ec\ud55c \uc791\uc5c5\uc774 \ud300\uc758 \uc0c9\uae54\uc744 \uc7a1\ub294 \ub370 \uc7ac\uc0b0\uc774 \ub418\uc5c8\ub2e4\uace0 \ub9d0\ud588\ub2e4.", "paragraph_answer": "\ubcfc\ube68\uac04\uc0ac\ucd98\uae30\ub294 \uacbd\uc0c1\ubd81\ub3c4 \uc601\uc8fc\uc5ec\uc790\uace0\ub4f1\ud559\uad50\uc5d0 1\ud559\ub144\uc758 \uc7ac\ud559 \uc911\uc774\ub358 2011\ub144 \uba64\ubc84\ub4e4\uc758 \ud558\uad63\uae38\uc5d0 \uacb0\uc131\ub418\uc5c8\ub2e4. 2014\ub144 \uc288\ud37c\uc2a4\ud0c0k 6\uc5d0 \uacbd\ubd81 \uc601\uc8fc \uc2dc\uace8\ubc34\ub4dc \ubcfc\ube68\uac04\uc0ac\ucd98\uae30\ub77c\ub294 \uc774\ub984\uc73c\ub85c \ucd9c\uc5f0\ud588\uc73c\ub098 \ud0c8\ub77d\ud558\uc600\ub2e4. \uc774\ub54c\uae4c\uc9c0\ub9cc \ud574\ub3c4 \uad8c\uc9c0\uc724\uc774 \ud300\uc758 \uae30\ud0c0\ub97c \ub2f4\ub2f9\ud558\uace0 \uc788\uc5c8\uc73c\ub098, \ud559\uc5c5\uc73c\ub85c \uc778\ud574 \uc548\uc9c0\uc601 \uc6b0\uc9c0\uc724 \ub9cc\uc774 \ub370\ubdd4\ub97c \ud558\uac8c\ub418\uc5c8\ub2e4. \uc288\ud37c\uc2a4\ud0c0k6 \ucd9c\uc5f0 \uc774\ud6c4 1\ub144 6\uac1c\uc6d4\uc774 \uc9c0\ub098\uba74\uc11c \ub370\ubdd4\uc5d0 \ub300\ud55c \ud300 \ub0b4\uc5d0 \ubd88\uc548\uac10\uc774 \uc870\uc131\ub418\uc5c8\uace0 \ub54c\ubb38\uc5d0 \uace1 \uc791\uc5c5\uc5d0 \ub9ce\uc740 \uc2dc\uac04\uc744 \ud560\uc560\ud574\uc57c\ub9cc \ud588\ub2e4. \uc18c\uc18d\uc0ac\ub294 \ud504\ub85c\ub4c0\uc11c\ub97c \uc81c\uacf5\ud558\uace0 \ub370\ubdd4 \uc568\ubc94\uc744 \uc900\ube44\ud558\ub3c4\ub85d \ud558\uc600\uc73c\ub098 \uc74c\uc545\uc801 \uc0c9\uae54\uc774 \ub9de\uc9c0 \uc54a\uc544 \uace1\uc744 \uc0c8\ub85c \uc4f0\uac8c \ub418\uc5c8\ub2e4. \uace1\uc744 \ub9ce\uc774, \uae09\ud558\uac8c \uace1\uc744 \uc4f0\uac8c \ub418\uc5c8\ub294\ub370 \uc548\uc9c0\uc601\uc740 \uc774\ub7ec\ud55c \uc791\uc5c5\uc774 \ud300\uc758 \uc0c9\uae54\uc744 \uc7a1\ub294 \ub370 \uc7ac\uc0b0\uc774 \ub418\uc5c8\ub2e4\uace0 \ub9d0\ud588\ub2e4.", "sentence_answer": "\uc774\ub54c\uae4c\uc9c0\ub9cc \ud574\ub3c4 \uad8c\uc9c0\uc724\uc774 \ud300\uc758 \uae30\ud0c0\ub97c \ub2f4\ub2f9\ud558\uace0 \uc788\uc5c8\uc73c\ub098, \ud559\uc5c5\uc73c\ub85c \uc778\ud574 \uc548\uc9c0\uc601 \uc6b0\uc9c0\uc724 \ub9cc\uc774 \ub370\ubdd4\ub97c \ud558\uac8c\ub418\uc5c8\ub2e4.", "paragraph_id": "6115948-0-1", "paragraph_question": "question: \ubcfc\ube68\uac04\uc0ac\ucd98\uae30 \uba64\ubc84\uc758 \uc774\ub984\uc740?, context: \ubcfc\ube68\uac04\uc0ac\ucd98\uae30\ub294 \uacbd\uc0c1\ubd81\ub3c4 \uc601\uc8fc\uc5ec\uc790\uace0\ub4f1\ud559\uad50\uc5d0 1\ud559\ub144\uc758 \uc7ac\ud559 \uc911\uc774\ub358 2011\ub144 \uba64\ubc84\ub4e4\uc758 \ud558\uad63\uae38\uc5d0 \uacb0\uc131\ub418\uc5c8\ub2e4. 2014\ub144 \uc288\ud37c\uc2a4\ud0c0k 6\uc5d0 \uacbd\ubd81 \uc601\uc8fc \uc2dc\uace8\ubc34\ub4dc \ubcfc\ube68\uac04\uc0ac\ucd98\uae30\ub77c\ub294 \uc774\ub984\uc73c\ub85c \ucd9c\uc5f0\ud588\uc73c\ub098 \ud0c8\ub77d\ud558\uc600\ub2e4. \uc774\ub54c\uae4c\uc9c0\ub9cc \ud574\ub3c4 \uad8c\uc9c0\uc724\uc774 \ud300\uc758 \uae30\ud0c0\ub97c \ub2f4\ub2f9\ud558\uace0 \uc788\uc5c8\uc73c\ub098, \ud559\uc5c5\uc73c\ub85c \uc778\ud574 \uc548\uc9c0\uc601 \uc6b0\uc9c0\uc724\ub9cc\uc774 \ub370\ubdd4\ub97c \ud558\uac8c\ub418\uc5c8\ub2e4. \uc288\ud37c\uc2a4\ud0c0k6 \ucd9c\uc5f0 \uc774\ud6c4 1\ub144 6\uac1c\uc6d4\uc774 \uc9c0\ub098\uba74\uc11c \ub370\ubdd4\uc5d0 \ub300\ud55c \ud300 \ub0b4\uc5d0 \ubd88\uc548\uac10\uc774 \uc870\uc131\ub418\uc5c8\uace0 \ub54c\ubb38\uc5d0 \uace1 \uc791\uc5c5\uc5d0 \ub9ce\uc740 \uc2dc\uac04\uc744 \ud560\uc560\ud574\uc57c\ub9cc \ud588\ub2e4. \uc18c\uc18d\uc0ac\ub294 \ud504\ub85c\ub4c0\uc11c\ub97c \uc81c\uacf5\ud558\uace0 \ub370\ubdd4 \uc568\ubc94\uc744 \uc900\ube44\ud558\ub3c4\ub85d \ud558\uc600\uc73c\ub098 \uc74c\uc545\uc801 \uc0c9\uae54\uc774 \ub9de\uc9c0 \uc54a\uc544 \uace1\uc744 \uc0c8\ub85c \uc4f0\uac8c \ub418\uc5c8\ub2e4. \uace1\uc744 \ub9ce\uc774, \uae09\ud558\uac8c \uace1\uc744 \uc4f0\uac8c \ub418\uc5c8\ub294\ub370 \uc548\uc9c0\uc601\uc740 \uc774\ub7ec\ud55c \uc791\uc5c5\uc774 \ud300\uc758 \uc0c9\uae54\uc744 \uc7a1\ub294 \ub370 \uc7ac\uc0b0\uc774 \ub418\uc5c8\ub2e4\uace0 \ub9d0\ud588\ub2e4."} {"question": "\ud658\uad00\uc758 \uc9d1\uc548\uc5d0\uc11c \uc81c\uc791\ud55c \uc871\ubcf4\ub97c \ubb34\uc5c7\uc774\ub77c \ud588\ub294\uac00?", "paragraph": "\uc774\uc0c9\uc801\uc73c\ub85c \ud658\uad00\uc758 \uc9d1\uc548\uc5d0\uc11c \uc871\ubcf4\ub97c \uc81c\uc791\ud55c \uacbd\uc6b0\ub3c4 \uc788\uc5c8\ub294\ub370, \uc774\ub97c \u300a\ub0b4\uc2dc\ubcf4\u300b\ub77c \ud558\uc600\ub2e4. \uc77c\uc81c \uc2dc\ub300\uc5d0 \uc791\uc131\ub41c \u300a\uc591\uc138\uacc4\ubcf4(\u990a\u4e16\u7cfb\u8b5c)\u300b(1920\ub144)\uc640 \u300a\uc5f0\uc591\uad70\uc138\uacc4\u300b\uac00 \ub0a8\uc544 \uc788\uc73c\uba70, \ud544\uc0ac\ubcf8\uc774\ub2e4. \uc774 \uacbd\uc6b0 \ud658\uad00\uc774\ub77c\ub294 \uc9c1\uc5c5\uc801 \ud2b9\uc131\uc0c1 \uc591\uc790\ub97c \ud1b5\ud574 \ub0b4\uc2dc\uc9c1\uc744 \uc138\uc2b5\ud558\uc5ec \uc138\uacc4\ub97c \ub0b4\ub824\uc624\uba74\uc11c \uc591\uc790\uc758 \uc131\ub3c4 \uadf8\ub300\ub85c \uc720\uc9c0\ud558\uace0 \ubcf8\uad00\ub3c4 \uc801\uc5b4\uc11c \uacc4\ucd9c\uc744 \ubc1d\ud78c \uc810\uc774 \ud2b9\uc9d5\uc774\ub2e4. \uc591\ubc18\uc774\ub098 \uc911\uc778\uc774 \uc544\ub2cc \uc2e0\ubd84\uc73c\ub85c \uacc4\ubcf4\ub97c \ub0a8\uae34 \uc0ac\ud68c\uce35\uc740 \ub178\ube44\uc600\ub2e4. \ub178\ube44\uac00 \ub9cc\ub4e0 \uac83\uc774 \uc544\ub2c8\ub77c \ub178\ube44\ub97c \uad00\ub9ac\ud558\ub294 \uc790\ub4e4\uc774 \ub9cc\ub4e0 \uac83\uc73c\ub85c, \uc804\ub77c\ub0a8\ub3c4 \uc7a5\uc131\uad70\uc758 \ud544\uc554\uc11c\uc6d0 \uc18c\uc7a5 \ubb38\uc11c \uac00\uc6b4\ub370 \u300a\ub178\ube44\ubcf4\u300b\uac00 \ub0a8\uc544\uc788\ub2e4. 18\uc138\uae30\uacbd \ud544\uc554\uc11c\uc6d0\uc5d0 \uc18c\uc18d\ub41c \ub178\ube44\ub4e4\uc758 \uc778\uc801 \uc0ac\ud56d\uc744 \uc801\uc740 \uacc4\ubcf4\ub294 \ubaa8\uacc4\ub85c \uc774\uc5b4\uc9c0\ub294 \uc131\uaca9\uc744 \uac00\uc9c0\uace0 \uc788\ub294\ub370, \uc870\uc120\uc758 \uc2e0\ubd84 \uad00\ub828 \ubc95\ub960\uc5d0\uc11c \uc885\ubaa8\ubc95(\u5f9e\u6bcd\u6cd5)\uc744 \ud0dd\ud588\ub358 \uc0ac\uc815\uc5d0 \uae30\uc778\ud55c \uac83\uc774\ub2e4.", "answer": "\ub0b4\uc2dc\ubcf4", "sentence": "\uc774\ub97c \u300a \ub0b4\uc2dc\ubcf4 \u300b\ub77c \ud558\uc600\ub2e4.", "paragraph_sentence": "\uc774\uc0c9\uc801\uc73c\ub85c \ud658\uad00\uc758 \uc9d1\uc548\uc5d0\uc11c \uc871\ubcf4\ub97c \uc81c\uc791\ud55c \uacbd\uc6b0\ub3c4 \uc788\uc5c8\ub294\ub370, \uc774\ub97c \u300a \ub0b4\uc2dc\ubcf4 \u300b\ub77c \ud558\uc600\ub2e4. \uc77c\uc81c \uc2dc\ub300\uc5d0 \uc791\uc131\ub41c \u300a\uc591\uc138\uacc4\ubcf4(\u990a\u4e16\u7cfb\u8b5c)\u300b(1920\ub144)\uc640 \u300a\uc5f0\uc591\uad70\uc138\uacc4\u300b\uac00 \ub0a8\uc544 \uc788\uc73c\uba70, \ud544\uc0ac\ubcf8\uc774\ub2e4. \uc774 \uacbd\uc6b0 \ud658\uad00\uc774\ub77c\ub294 \uc9c1\uc5c5\uc801 \ud2b9\uc131\uc0c1 \uc591\uc790\ub97c \ud1b5\ud574 \ub0b4\uc2dc\uc9c1\uc744 \uc138\uc2b5\ud558\uc5ec \uc138\uacc4\ub97c \ub0b4\ub824\uc624\uba74\uc11c \uc591\uc790\uc758 \uc131\ub3c4 \uadf8\ub300\ub85c \uc720\uc9c0\ud558\uace0 \ubcf8\uad00\ub3c4 \uc801\uc5b4\uc11c \uacc4\ucd9c\uc744 \ubc1d\ud78c \uc810\uc774 \ud2b9\uc9d5\uc774\ub2e4. \uc591\ubc18\uc774\ub098 \uc911\uc778\uc774 \uc544\ub2cc \uc2e0\ubd84\uc73c\ub85c \uacc4\ubcf4\ub97c \ub0a8\uae34 \uc0ac\ud68c\uce35\uc740 \ub178\ube44\uc600\ub2e4. \ub178\ube44\uac00 \ub9cc\ub4e0 \uac83\uc774 \uc544\ub2c8\ub77c \ub178\ube44\ub97c \uad00\ub9ac\ud558\ub294 \uc790\ub4e4\uc774 \ub9cc\ub4e0 \uac83\uc73c\ub85c, \uc804\ub77c\ub0a8\ub3c4 \uc7a5\uc131\uad70\uc758 \ud544\uc554\uc11c\uc6d0 \uc18c\uc7a5 \ubb38\uc11c \uac00\uc6b4\ub370 \u300a\ub178\ube44\ubcf4\u300b\uac00 \ub0a8\uc544\uc788\ub2e4. 18\uc138\uae30\uacbd \ud544\uc554\uc11c\uc6d0\uc5d0 \uc18c\uc18d\ub41c \ub178\ube44\ub4e4\uc758 \uc778\uc801 \uc0ac\ud56d\uc744 \uc801\uc740 \uacc4\ubcf4\ub294 \ubaa8\uacc4\ub85c \uc774\uc5b4\uc9c0\ub294 \uc131\uaca9\uc744 \uac00\uc9c0\uace0 \uc788\ub294\ub370, \uc870\uc120\uc758 \uc2e0\ubd84 \uad00\ub828 \ubc95\ub960\uc5d0\uc11c \uc885\ubaa8\ubc95(\u5f9e\u6bcd\u6cd5)\uc744 \ud0dd\ud588\ub358 \uc0ac\uc815\uc5d0 \uae30\uc778\ud55c \uac83\uc774\ub2e4.", "paragraph_answer": "\uc774\uc0c9\uc801\uc73c\ub85c \ud658\uad00\uc758 \uc9d1\uc548\uc5d0\uc11c \uc871\ubcf4\ub97c \uc81c\uc791\ud55c \uacbd\uc6b0\ub3c4 \uc788\uc5c8\ub294\ub370, \uc774\ub97c \u300a \ub0b4\uc2dc\ubcf4 \u300b\ub77c \ud558\uc600\ub2e4. \uc77c\uc81c \uc2dc\ub300\uc5d0 \uc791\uc131\ub41c \u300a\uc591\uc138\uacc4\ubcf4(\u990a\u4e16\u7cfb\u8b5c)\u300b(1920\ub144)\uc640 \u300a\uc5f0\uc591\uad70\uc138\uacc4\u300b\uac00 \ub0a8\uc544 \uc788\uc73c\uba70, \ud544\uc0ac\ubcf8\uc774\ub2e4. \uc774 \uacbd\uc6b0 \ud658\uad00\uc774\ub77c\ub294 \uc9c1\uc5c5\uc801 \ud2b9\uc131\uc0c1 \uc591\uc790\ub97c \ud1b5\ud574 \ub0b4\uc2dc\uc9c1\uc744 \uc138\uc2b5\ud558\uc5ec \uc138\uacc4\ub97c \ub0b4\ub824\uc624\uba74\uc11c \uc591\uc790\uc758 \uc131\ub3c4 \uadf8\ub300\ub85c \uc720\uc9c0\ud558\uace0 \ubcf8\uad00\ub3c4 \uc801\uc5b4\uc11c \uacc4\ucd9c\uc744 \ubc1d\ud78c \uc810\uc774 \ud2b9\uc9d5\uc774\ub2e4. \uc591\ubc18\uc774\ub098 \uc911\uc778\uc774 \uc544\ub2cc \uc2e0\ubd84\uc73c\ub85c \uacc4\ubcf4\ub97c \ub0a8\uae34 \uc0ac\ud68c\uce35\uc740 \ub178\ube44\uc600\ub2e4. \ub178\ube44\uac00 \ub9cc\ub4e0 \uac83\uc774 \uc544\ub2c8\ub77c \ub178\ube44\ub97c \uad00\ub9ac\ud558\ub294 \uc790\ub4e4\uc774 \ub9cc\ub4e0 \uac83\uc73c\ub85c, \uc804\ub77c\ub0a8\ub3c4 \uc7a5\uc131\uad70\uc758 \ud544\uc554\uc11c\uc6d0 \uc18c\uc7a5 \ubb38\uc11c \uac00\uc6b4\ub370 \u300a\ub178\ube44\ubcf4\u300b\uac00 \ub0a8\uc544\uc788\ub2e4. 18\uc138\uae30\uacbd \ud544\uc554\uc11c\uc6d0\uc5d0 \uc18c\uc18d\ub41c \ub178\ube44\ub4e4\uc758 \uc778\uc801 \uc0ac\ud56d\uc744 \uc801\uc740 \uacc4\ubcf4\ub294 \ubaa8\uacc4\ub85c \uc774\uc5b4\uc9c0\ub294 \uc131\uaca9\uc744 \uac00\uc9c0\uace0 \uc788\ub294\ub370, \uc870\uc120\uc758 \uc2e0\ubd84 \uad00\ub828 \ubc95\ub960\uc5d0\uc11c \uc885\ubaa8\ubc95(\u5f9e\u6bcd\u6cd5)\uc744 \ud0dd\ud588\ub358 \uc0ac\uc815\uc5d0 \uae30\uc778\ud55c \uac83\uc774\ub2e4.", "sentence_answer": "\uc774\ub97c \u300a \ub0b4\uc2dc\ubcf4 \u300b\ub77c \ud558\uc600\ub2e4.", "paragraph_id": "6555296-4-0", "paragraph_question": "question: \ud658\uad00\uc758 \uc9d1\uc548\uc5d0\uc11c \uc81c\uc791\ud55c \uc871\ubcf4\ub97c \ubb34\uc5c7\uc774\ub77c \ud588\ub294\uac00?, context: \uc774\uc0c9\uc801\uc73c\ub85c \ud658\uad00\uc758 \uc9d1\uc548\uc5d0\uc11c \uc871\ubcf4\ub97c \uc81c\uc791\ud55c \uacbd\uc6b0\ub3c4 \uc788\uc5c8\ub294\ub370, \uc774\ub97c \u300a\ub0b4\uc2dc\ubcf4\u300b\ub77c \ud558\uc600\ub2e4. \uc77c\uc81c \uc2dc\ub300\uc5d0 \uc791\uc131\ub41c \u300a\uc591\uc138\uacc4\ubcf4(\u990a\u4e16\u7cfb\u8b5c)\u300b(1920\ub144)\uc640 \u300a\uc5f0\uc591\uad70\uc138\uacc4\u300b\uac00 \ub0a8\uc544 \uc788\uc73c\uba70, \ud544\uc0ac\ubcf8\uc774\ub2e4. \uc774 \uacbd\uc6b0 \ud658\uad00\uc774\ub77c\ub294 \uc9c1\uc5c5\uc801 \ud2b9\uc131\uc0c1 \uc591\uc790\ub97c \ud1b5\ud574 \ub0b4\uc2dc\uc9c1\uc744 \uc138\uc2b5\ud558\uc5ec \uc138\uacc4\ub97c \ub0b4\ub824\uc624\uba74\uc11c \uc591\uc790\uc758 \uc131\ub3c4 \uadf8\ub300\ub85c \uc720\uc9c0\ud558\uace0 \ubcf8\uad00\ub3c4 \uc801\uc5b4\uc11c \uacc4\ucd9c\uc744 \ubc1d\ud78c \uc810\uc774 \ud2b9\uc9d5\uc774\ub2e4. \uc591\ubc18\uc774\ub098 \uc911\uc778\uc774 \uc544\ub2cc \uc2e0\ubd84\uc73c\ub85c \uacc4\ubcf4\ub97c \ub0a8\uae34 \uc0ac\ud68c\uce35\uc740 \ub178\ube44\uc600\ub2e4. \ub178\ube44\uac00 \ub9cc\ub4e0 \uac83\uc774 \uc544\ub2c8\ub77c \ub178\ube44\ub97c \uad00\ub9ac\ud558\ub294 \uc790\ub4e4\uc774 \ub9cc\ub4e0 \uac83\uc73c\ub85c, \uc804\ub77c\ub0a8\ub3c4 \uc7a5\uc131\uad70\uc758 \ud544\uc554\uc11c\uc6d0 \uc18c\uc7a5 \ubb38\uc11c \uac00\uc6b4\ub370 \u300a\ub178\ube44\ubcf4\u300b\uac00 \ub0a8\uc544\uc788\ub2e4. 18\uc138\uae30\uacbd \ud544\uc554\uc11c\uc6d0\uc5d0 \uc18c\uc18d\ub41c \ub178\ube44\ub4e4\uc758 \uc778\uc801 \uc0ac\ud56d\uc744 \uc801\uc740 \uacc4\ubcf4\ub294 \ubaa8\uacc4\ub85c \uc774\uc5b4\uc9c0\ub294 \uc131\uaca9\uc744 \uac00\uc9c0\uace0 \uc788\ub294\ub370, \uc870\uc120\uc758 \uc2e0\ubd84 \uad00\ub828 \ubc95\ub960\uc5d0\uc11c \uc885\ubaa8\ubc95(\u5f9e\u6bcd\u6cd5)\uc744 \ud0dd\ud588\ub358 \uc0ac\uc815\uc5d0 \uae30\uc778\ud55c \uac83\uc774\ub2e4."} {"question": "\ubbfc\ubb3c\ubcf4\uae30\ubcf4\ub2e4 \uc218\uc740\uc774 \ub9ce\uc740 \ubb3c\uace0\uae30\ub294?", "paragraph": "\uac00\ubb3c\uce58\ub294 \ubbfc\ubb3c\uace0\uae30\ub85c\uc11c, \uc5fc\ub3c4\uac00 \ub192\uc740 \ubb3c\uc5d0\uc11c\ub294 \uc0b4 \uc218 \uc5c6\ub2e4. \uc801\uc751\ub825\uc774 \ub6f0\uc5b4\ub098 \ud55c\ubc18\ub3c4\uc5d0\uc11c\ub294 \ub2aa\uc774\ub098 \ubed8, \uc5f0\ubabb, \ud638\uc218, \ud558\ucc9c \ub4f1 \ucda9\ubd84\ud55c \uacf5\uac04\uacfc \uc218\uad8c, \uba39\uc774\uac00 \ud655\ubcf4\ub41c \uacf3\uc5d0\uc11c\ub77c\uba74 \uc5b4\ub514\uc5d0\uc11c\ub77c\ub3c4 \uc790\ub784 \uc218 \uc788\uc73c\uba70, \uc218\uc628\uacfc \uc624\uc5fc\ub3c4\uc5d0 \uac70\uc758 \uad6c\uc560\ubc1b\uc9c0 \uc54a\uc73c\ub098 \uadf8 \ub54c\ubb38\uc5d0 \ubab8 \uc18d\uc5d0 \uac01\uc885 \uc0b0\uc5c5 \ud3d0\uae30\ubb3c, \ub610\ub294 \uce58\uba85\uc801\uc778 \uc77c\ubd80 \uc911\uae08\uc18d\uc774 \ucd95\uc801\ub418\uae30\ub3c4 \ud55c\ub2e4. \ud2b9\ud788 \uc218\uc740\uc740 \ub300\ubd80\ubd84\uc758 \ubbfc\ubb3c\uace0\uae30\ubcf4\ub2e4 \uac00\ubb3c\uce58\uc5d0\uac8c\uc11c \ubcf4\ub2e4 \ub9ce\uc740 \uc591\uc774 \uac80\ucd9c\ub418\uc5c8\ub2e4. \ub300\uac1c\ub294 \uc595\uc740 \ubb3c\uc744 \uc120\ud638\ud55c\ub2e4. \uacf5\uaca9\uc131\uc774 \uc544\uc8fc \uac15\ud55c \uc721\uc2dd\uc131 \uc5b4\uc885\uc73c\ub85c \uc131\uc9c8\uc774 \uc0ac\ub0a9\uace0 \ud798\uacfc \uc545\ub825\uc774 \uac15\ud558\uc5ec \ub2aa\uc758 \ubb34\ubc95\uc790\ub77c\uace0 \ubd88\ub9ac\uba70, \uce58\uc5b4 \uc2dc\uae30\uc5d0\ub294 \uc8fc\ub85c \ubb3c\ubcbc\ub8e9\uc744 \uba39\uc9c0\ub9cc \uc131\uccb4\uac00 \ub418\uba74 \uc8fc\ub85c \uac1c\uad6c\ub9ac\u00b7\ub3c4\ub871\ub1fd \ub4f1\uc758 \uc591\uc11c\ub958\uc640 \uac00\uc7ac\u00b7\ubbfc\ubb3c\uc0c8\uc6b0 \ub4f1\uc758 \uac11\uac01\ub958, \ubd95\uc5b4\u00b7\ub0a9\uc790\ub8e8\u00b7\ubbf8\uafb8\ub77c\uc9c0 \ub4f1\uc758 \uc18c\ud615 \uc5b4\ub958\ub97c \uba39\uc9c0\ub9cc, \uba54\uae30 \ub4f1\uc758 \ub300\ud615 \uc5b4\ub958\ub97c \uc7a1\uc544\uba39\uac70\ub098 \ub3d9\uc871\uc744 \uacf5\uaca9\ud558\ub294 \uc77c\ub3c4 \uc788\ub2e4. \ube44\uac00 \uc62c \ub54c\ub294 \ub2aa \ubc11\uc744 \uae30\uba74\uc11c \ub2e4\ub2c8\uae30\ub3c4 \ud55c\ub2e4.", "answer": "\uac00\ubb3c\uce58", "sentence": "\uac00\ubb3c\uce58 \ub294 \ubbfc\ubb3c\uace0\uae30\ub85c\uc11c, \uc5fc\ub3c4\uac00 \ub192\uc740 \ubb3c\uc5d0\uc11c\ub294 \uc0b4 \uc218 \uc5c6\ub2e4.", "paragraph_sentence": " \uac00\ubb3c\uce58 \ub294 \ubbfc\ubb3c\uace0\uae30\ub85c\uc11c, \uc5fc\ub3c4\uac00 \ub192\uc740 \ubb3c\uc5d0\uc11c\ub294 \uc0b4 \uc218 \uc5c6\ub2e4. \uc801\uc751\ub825\uc774 \ub6f0\uc5b4\ub098 \ud55c\ubc18\ub3c4\uc5d0\uc11c\ub294 \ub2aa\uc774\ub098 \ubed8, \uc5f0\ubabb, \ud638\uc218, \ud558\ucc9c \ub4f1 \ucda9\ubd84\ud55c \uacf5\uac04\uacfc \uc218\uad8c, \uba39\uc774\uac00 \ud655\ubcf4\ub41c \uacf3\uc5d0\uc11c\ub77c\uba74 \uc5b4\ub514\uc5d0\uc11c\ub77c\ub3c4 \uc790\ub784 \uc218 \uc788\uc73c\uba70, \uc218\uc628\uacfc \uc624\uc5fc\ub3c4\uc5d0 \uac70\uc758 \uad6c\uc560\ubc1b\uc9c0 \uc54a\uc73c\ub098 \uadf8 \ub54c\ubb38\uc5d0 \ubab8 \uc18d\uc5d0 \uac01\uc885 \uc0b0\uc5c5 \ud3d0\uae30\ubb3c, \ub610\ub294 \uce58\uba85\uc801\uc778 \uc77c\ubd80 \uc911\uae08\uc18d\uc774 \ucd95\uc801\ub418\uae30\ub3c4 \ud55c\ub2e4. \ud2b9\ud788 \uc218\uc740\uc740 \ub300\ubd80\ubd84\uc758 \ubbfc\ubb3c\uace0\uae30\ubcf4\ub2e4 \uac00\ubb3c\uce58\uc5d0\uac8c\uc11c \ubcf4\ub2e4 \ub9ce\uc740 \uc591\uc774 \uac80\ucd9c\ub418\uc5c8\ub2e4. \ub300\uac1c\ub294 \uc595\uc740 \ubb3c\uc744 \uc120\ud638\ud55c\ub2e4. \uacf5\uaca9\uc131\uc774 \uc544\uc8fc \uac15\ud55c \uc721\uc2dd\uc131 \uc5b4\uc885\uc73c\ub85c \uc131\uc9c8\uc774 \uc0ac\ub0a9\uace0 \ud798\uacfc \uc545\ub825\uc774 \uac15\ud558\uc5ec \ub2aa\uc758 \ubb34\ubc95\uc790\ub77c\uace0 \ubd88\ub9ac\uba70, \uce58\uc5b4 \uc2dc\uae30\uc5d0\ub294 \uc8fc\ub85c \ubb3c\ubcbc\ub8e9\uc744 \uba39\uc9c0\ub9cc \uc131\uccb4\uac00 \ub418\uba74 \uc8fc\ub85c \uac1c\uad6c\ub9ac\u00b7\ub3c4\ub871\ub1fd \ub4f1\uc758 \uc591\uc11c\ub958\uc640 \uac00\uc7ac\u00b7\ubbfc\ubb3c\uc0c8\uc6b0 \ub4f1\uc758 \uac11\uac01\ub958, \ubd95\uc5b4\u00b7\ub0a9\uc790\ub8e8\u00b7\ubbf8\uafb8\ub77c\uc9c0 \ub4f1\uc758 \uc18c\ud615 \uc5b4\ub958\ub97c \uba39\uc9c0\ub9cc, \uba54\uae30 \ub4f1\uc758 \ub300\ud615 \uc5b4\ub958\ub97c \uc7a1\uc544\uba39\uac70\ub098 \ub3d9\uc871\uc744 \uacf5\uaca9\ud558\ub294 \uc77c\ub3c4 \uc788\ub2e4. \ube44\uac00 \uc62c \ub54c\ub294 \ub2aa \ubc11\uc744 \uae30\uba74\uc11c \ub2e4\ub2c8\uae30\ub3c4 \ud55c\ub2e4.", "paragraph_answer": " \uac00\ubb3c\uce58 \ub294 \ubbfc\ubb3c\uace0\uae30\ub85c\uc11c, \uc5fc\ub3c4\uac00 \ub192\uc740 \ubb3c\uc5d0\uc11c\ub294 \uc0b4 \uc218 \uc5c6\ub2e4. \uc801\uc751\ub825\uc774 \ub6f0\uc5b4\ub098 \ud55c\ubc18\ub3c4\uc5d0\uc11c\ub294 \ub2aa\uc774\ub098 \ubed8, \uc5f0\ubabb, \ud638\uc218, \ud558\ucc9c \ub4f1 \ucda9\ubd84\ud55c \uacf5\uac04\uacfc \uc218\uad8c, \uba39\uc774\uac00 \ud655\ubcf4\ub41c \uacf3\uc5d0\uc11c\ub77c\uba74 \uc5b4\ub514\uc5d0\uc11c\ub77c\ub3c4 \uc790\ub784 \uc218 \uc788\uc73c\uba70, \uc218\uc628\uacfc \uc624\uc5fc\ub3c4\uc5d0 \uac70\uc758 \uad6c\uc560\ubc1b\uc9c0 \uc54a\uc73c\ub098 \uadf8 \ub54c\ubb38\uc5d0 \ubab8 \uc18d\uc5d0 \uac01\uc885 \uc0b0\uc5c5 \ud3d0\uae30\ubb3c, \ub610\ub294 \uce58\uba85\uc801\uc778 \uc77c\ubd80 \uc911\uae08\uc18d\uc774 \ucd95\uc801\ub418\uae30\ub3c4 \ud55c\ub2e4. \ud2b9\ud788 \uc218\uc740\uc740 \ub300\ubd80\ubd84\uc758 \ubbfc\ubb3c\uace0\uae30\ubcf4\ub2e4 \uac00\ubb3c\uce58\uc5d0\uac8c\uc11c \ubcf4\ub2e4 \ub9ce\uc740 \uc591\uc774 \uac80\ucd9c\ub418\uc5c8\ub2e4. \ub300\uac1c\ub294 \uc595\uc740 \ubb3c\uc744 \uc120\ud638\ud55c\ub2e4. \uacf5\uaca9\uc131\uc774 \uc544\uc8fc \uac15\ud55c \uc721\uc2dd\uc131 \uc5b4\uc885\uc73c\ub85c \uc131\uc9c8\uc774 \uc0ac\ub0a9\uace0 \ud798\uacfc \uc545\ub825\uc774 \uac15\ud558\uc5ec \ub2aa\uc758 \ubb34\ubc95\uc790\ub77c\uace0 \ubd88\ub9ac\uba70, \uce58\uc5b4 \uc2dc\uae30\uc5d0\ub294 \uc8fc\ub85c \ubb3c\ubcbc\ub8e9\uc744 \uba39\uc9c0\ub9cc \uc131\uccb4\uac00 \ub418\uba74 \uc8fc\ub85c \uac1c\uad6c\ub9ac\u00b7\ub3c4\ub871\ub1fd \ub4f1\uc758 \uc591\uc11c\ub958\uc640 \uac00\uc7ac\u00b7\ubbfc\ubb3c\uc0c8\uc6b0 \ub4f1\uc758 \uac11\uac01\ub958, \ubd95\uc5b4\u00b7\ub0a9\uc790\ub8e8\u00b7\ubbf8\uafb8\ub77c\uc9c0 \ub4f1\uc758 \uc18c\ud615 \uc5b4\ub958\ub97c \uba39\uc9c0\ub9cc, \uba54\uae30 \ub4f1\uc758 \ub300\ud615 \uc5b4\ub958\ub97c \uc7a1\uc544\uba39\uac70\ub098 \ub3d9\uc871\uc744 \uacf5\uaca9\ud558\ub294 \uc77c\ub3c4 \uc788\ub2e4. \ube44\uac00 \uc62c \ub54c\ub294 \ub2aa \ubc11\uc744 \uae30\uba74\uc11c \ub2e4\ub2c8\uae30\ub3c4 \ud55c\ub2e4.", "sentence_answer": " \uac00\ubb3c\uce58 \ub294 \ubbfc\ubb3c\uace0\uae30\ub85c\uc11c, \uc5fc\ub3c4\uac00 \ub192\uc740 \ubb3c\uc5d0\uc11c\ub294 \uc0b4 \uc218 \uc5c6\ub2e4.", "paragraph_id": "6603220-1-1", "paragraph_question": "question: \ubbfc\ubb3c\ubcf4\uae30\ubcf4\ub2e4 \uc218\uc740\uc774 \ub9ce\uc740 \ubb3c\uace0\uae30\ub294?, context: \uac00\ubb3c\uce58\ub294 \ubbfc\ubb3c\uace0\uae30\ub85c\uc11c, \uc5fc\ub3c4\uac00 \ub192\uc740 \ubb3c\uc5d0\uc11c\ub294 \uc0b4 \uc218 \uc5c6\ub2e4. \uc801\uc751\ub825\uc774 \ub6f0\uc5b4\ub098 \ud55c\ubc18\ub3c4\uc5d0\uc11c\ub294 \ub2aa\uc774\ub098 \ubed8, \uc5f0\ubabb, \ud638\uc218, \ud558\ucc9c \ub4f1 \ucda9\ubd84\ud55c \uacf5\uac04\uacfc \uc218\uad8c, \uba39\uc774\uac00 \ud655\ubcf4\ub41c \uacf3\uc5d0\uc11c\ub77c\uba74 \uc5b4\ub514\uc5d0\uc11c\ub77c\ub3c4 \uc790\ub784 \uc218 \uc788\uc73c\uba70, \uc218\uc628\uacfc \uc624\uc5fc\ub3c4\uc5d0 \uac70\uc758 \uad6c\uc560\ubc1b\uc9c0 \uc54a\uc73c\ub098 \uadf8 \ub54c\ubb38\uc5d0 \ubab8 \uc18d\uc5d0 \uac01\uc885 \uc0b0\uc5c5 \ud3d0\uae30\ubb3c, \ub610\ub294 \uce58\uba85\uc801\uc778 \uc77c\ubd80 \uc911\uae08\uc18d\uc774 \ucd95\uc801\ub418\uae30\ub3c4 \ud55c\ub2e4. \ud2b9\ud788 \uc218\uc740\uc740 \ub300\ubd80\ubd84\uc758 \ubbfc\ubb3c\uace0\uae30\ubcf4\ub2e4 \uac00\ubb3c\uce58\uc5d0\uac8c\uc11c \ubcf4\ub2e4 \ub9ce\uc740 \uc591\uc774 \uac80\ucd9c\ub418\uc5c8\ub2e4. \ub300\uac1c\ub294 \uc595\uc740 \ubb3c\uc744 \uc120\ud638\ud55c\ub2e4. \uacf5\uaca9\uc131\uc774 \uc544\uc8fc \uac15\ud55c \uc721\uc2dd\uc131 \uc5b4\uc885\uc73c\ub85c \uc131\uc9c8\uc774 \uc0ac\ub0a9\uace0 \ud798\uacfc \uc545\ub825\uc774 \uac15\ud558\uc5ec \ub2aa\uc758 \ubb34\ubc95\uc790\ub77c\uace0 \ubd88\ub9ac\uba70, \uce58\uc5b4 \uc2dc\uae30\uc5d0\ub294 \uc8fc\ub85c \ubb3c\ubcbc\ub8e9\uc744 \uba39\uc9c0\ub9cc \uc131\uccb4\uac00 \ub418\uba74 \uc8fc\ub85c \uac1c\uad6c\ub9ac\u00b7\ub3c4\ub871\ub1fd \ub4f1\uc758 \uc591\uc11c\ub958\uc640 \uac00\uc7ac\u00b7\ubbfc\ubb3c\uc0c8\uc6b0 \ub4f1\uc758 \uac11\uac01\ub958, \ubd95\uc5b4\u00b7\ub0a9\uc790\ub8e8\u00b7\ubbf8\uafb8\ub77c\uc9c0 \ub4f1\uc758 \uc18c\ud615 \uc5b4\ub958\ub97c \uba39\uc9c0\ub9cc, \uba54\uae30 \ub4f1\uc758 \ub300\ud615 \uc5b4\ub958\ub97c \uc7a1\uc544\uba39\uac70\ub098 \ub3d9\uc871\uc744 \uacf5\uaca9\ud558\ub294 \uc77c\ub3c4 \uc788\ub2e4. \ube44\uac00 \uc62c \ub54c\ub294 \ub2aa \ubc11\uc744 \uae30\uba74\uc11c \ub2e4\ub2c8\uae30\ub3c4 \ud55c\ub2e4."} {"question": "\uc724\uc11d\uc5f4\uc740 18\ub300 \ub300\uc120 \uc5ec\ub860\uc870\uc791\uc0ac\uac74 \uc218\uc0ac\uc911 \uad6d\uac00\uc815\ubcf4\uc6d0 \uc9c1\uc6d0\ub4e4\uc758 \uc5b4\ub5a4 \ud65c\ub3d9\uacfc \uad00\ub828\ud55c \uacb0\uc815\uc801 \uc99d\uac70\ub97c \ud3ec\ucc29\ud558\uc600\ub098\uc694?", "paragraph": "\ub300\ud55c\ubbfc\uad6d \uad6d\uac00\uc815\ubcf4\uc6d0 \uc5ec\ub860 \uc870\uc791 \uc0ac\uac74 \uadf8 \ud6c4 \uad6d\uac00\uc815\ubcf4\uc6d0 \ub300\uc120 \uc5ec\ub860\uc870\uc791 \uc0ac\uac74 \ud2b9\ubcc4\uc218\uc0ac\ud300\uc7a5\uc744 \ub9e1\uc544 \uc57d 8\uac1c\uc6d4\uac04\uc758 \uc218\uc0ac\ub97c \ud1b5\ud574 2012\ub144 \uc81c18\ub300 \ub300\uc120\uc5d0\uc11c \uad6d\uac00\uc815\ubcf4\uc6d0\uc774 \uc870\uc9c1\uc801\uc73c\ub85c SNS \uc0c1\uc5d0\uc11c \uc57c\ub2f9 \ub300\uc120 \ud6c4\ubcf4\ub97c \ube44\ubc29\ud558\uace0 \uc5ec\ub2f9 \ud6c4\ubcf4\ub97c \uc9c0\uc9c0\ud558\ub294 \uc5ec\ub860\uc870\uc791\uc744 \uc2e4\ud589\ud55c \uc0ac\uc2e4\uc744 \ubc1d\ud600\ub0b4\uc5c8\ub2e4. \uc218\uc0ac \ub3c4\uc911 \uad6d\uac00\uc815\ubcf4\uc6d0 \uc9c1\uc6d0\ub4e4\uc758 \ub300\ub300\uc801\uc778 \ud2b8\uc704\ud130 \ud65c\ub3d9\uacfc \uad00\ub828\ud55c \uacb0\uc815\uc801\uc778 \uc99d\uac70\ub97c \ud3ec\ucc29\ud558\uace0 \uad6d\uac00\uc815\ubcf4\uc6d0 \uc9c1\uc6d0\ub4e4\uc5d0 \ub300\ud55c \uccb4\ud3ec\uc640 \uc555\uc218\uc218\uc0c9\uc744 \uc9d1\ud589\ud558\uc600\ub294\ub370 \uc11c\uc6b8\uc911\uc559\uc9c0\ubc29\uac80\ucc30\uccad \uc870\uc601\uace4 \uac80\uc0ac\uc7a5\uc5d0\uac8c \ubcf4\uace0\ud588\uc9c0\ub9cc \ub3d9\uc758\ubc1b\uc9c0 \ubabb\ud558\uc790 \uc790\uc2e0\uc758 \ucc45\uc784\uc73c\ub85c \uccb4\ud3ec\ub97c \uac15\ud589\ud558\uc600\uae30 \ub54c\ubb38\uc774\ub77c \ud558\uc5ec \ud2b9\ubcc4\uc218\uc0ac\ud300\uc7a5\uc5d0\uc11c \uacbd\uc9c8\ub418\uc5c8\ub2e4\ub294 \ubcf4\ub3c4\uac00 \uc788\ub2e4. \ub2f9\uc2dc \uc57c\ub2f9\uacfc \uc2dc\ubbfc\ub2e8\uccb4, \uc885\uad50\uacc4, \uc5b8\ub860 \ub4f1\uc5d0\uc11c\ub294 \ud2b9\ubcc4\uc218\uc0ac\ud300\uc7a5 \uacbd\uc9c8\uc774 \uc218\uc0ac\ubc29\ud574\ud589\uc704\ub77c\uace0 \ud558\uba74\uc11c \ud2b9\ubcc4\uc218\uc0ac\ud300\uc7a5\uc758 \uc989\uac01\uc801\uc778 \ubcf5\uadc0\uc640 \ud2b9\uac80 \uc2e4\uc2dc \ub4f1\uc744 \uc8fc\uc7a5\ud558\uc600\ub2e4.", "answer": "\ud2b8\uc704\ud130 \ud65c\ub3d9", "sentence": "\uc218\uc0ac \ub3c4\uc911 \uad6d\uac00\uc815\ubcf4\uc6d0 \uc9c1\uc6d0\ub4e4\uc758 \ub300\ub300\uc801\uc778 \ud2b8\uc704\ud130 \ud65c\ub3d9 \uacfc \uad00\ub828\ud55c \uacb0\uc815\uc801\uc778 \uc99d\uac70\ub97c \ud3ec\ucc29\ud558\uace0 \uad6d\uac00\uc815\ubcf4\uc6d0 \uc9c1\uc6d0\ub4e4\uc5d0 \ub300\ud55c \uccb4\ud3ec\uc640 \uc555\uc218\uc218\uc0c9\uc744 \uc9d1\ud589\ud558\uc600\ub294\ub370 \uc11c\uc6b8\uc911\uc559\uc9c0\ubc29\uac80\ucc30\uccad \uc870\uc601\uace4 \uac80\uc0ac\uc7a5\uc5d0\uac8c \ubcf4\uace0\ud588\uc9c0\ub9cc \ub3d9\uc758\ubc1b\uc9c0 \ubabb\ud558\uc790 \uc790\uc2e0\uc758 \ucc45\uc784\uc73c\ub85c \uccb4\ud3ec\ub97c \uac15\ud589\ud558\uc600\uae30 \ub54c\ubb38\uc774\ub77c \ud558\uc5ec \ud2b9\ubcc4\uc218\uc0ac\ud300\uc7a5\uc5d0\uc11c \uacbd\uc9c8\ub418\uc5c8\ub2e4\ub294 \ubcf4\ub3c4\uac00 \uc788\ub2e4.", "paragraph_sentence": "\ub300\ud55c\ubbfc\uad6d \uad6d\uac00\uc815\ubcf4\uc6d0 \uc5ec\ub860 \uc870\uc791 \uc0ac\uac74 \uadf8 \ud6c4 \uad6d\uac00\uc815\ubcf4\uc6d0 \ub300\uc120 \uc5ec\ub860\uc870\uc791 \uc0ac\uac74 \ud2b9\ubcc4\uc218\uc0ac\ud300\uc7a5\uc744 \ub9e1\uc544 \uc57d 8\uac1c\uc6d4\uac04\uc758 \uc218\uc0ac\ub97c \ud1b5\ud574 2012\ub144 \uc81c18\ub300 \ub300\uc120\uc5d0\uc11c \uad6d\uac00\uc815\ubcf4\uc6d0\uc774 \uc870\uc9c1\uc801\uc73c\ub85c SNS \uc0c1\uc5d0\uc11c \uc57c\ub2f9 \ub300\uc120 \ud6c4\ubcf4\ub97c \ube44\ubc29\ud558\uace0 \uc5ec\ub2f9 \ud6c4\ubcf4\ub97c \uc9c0\uc9c0\ud558\ub294 \uc5ec\ub860\uc870\uc791\uc744 \uc2e4\ud589\ud55c \uc0ac\uc2e4\uc744 \ubc1d\ud600\ub0b4\uc5c8\ub2e4. \uc218\uc0ac \ub3c4\uc911 \uad6d\uac00\uc815\ubcf4\uc6d0 \uc9c1\uc6d0\ub4e4\uc758 \ub300\ub300\uc801\uc778 \ud2b8\uc704\ud130 \ud65c\ub3d9 \uacfc \uad00\ub828\ud55c \uacb0\uc815\uc801\uc778 \uc99d\uac70\ub97c \ud3ec\ucc29\ud558\uace0 \uad6d\uac00\uc815\ubcf4\uc6d0 \uc9c1\uc6d0\ub4e4\uc5d0 \ub300\ud55c \uccb4\ud3ec\uc640 \uc555\uc218\uc218\uc0c9\uc744 \uc9d1\ud589\ud558\uc600\ub294\ub370 \uc11c\uc6b8\uc911\uc559\uc9c0\ubc29\uac80\ucc30\uccad \uc870\uc601\uace4 \uac80\uc0ac\uc7a5\uc5d0\uac8c \ubcf4\uace0\ud588\uc9c0\ub9cc \ub3d9\uc758\ubc1b\uc9c0 \ubabb\ud558\uc790 \uc790\uc2e0\uc758 \ucc45\uc784\uc73c\ub85c \uccb4\ud3ec\ub97c \uac15\ud589\ud558\uc600\uae30 \ub54c\ubb38\uc774\ub77c \ud558\uc5ec \ud2b9\ubcc4\uc218\uc0ac\ud300\uc7a5\uc5d0\uc11c \uacbd\uc9c8\ub418\uc5c8\ub2e4\ub294 \ubcf4\ub3c4\uac00 \uc788\ub2e4. \ub2f9\uc2dc \uc57c\ub2f9\uacfc \uc2dc\ubbfc\ub2e8\uccb4, \uc885\uad50\uacc4, \uc5b8\ub860 \ub4f1\uc5d0\uc11c\ub294 \ud2b9\ubcc4\uc218\uc0ac\ud300\uc7a5 \uacbd\uc9c8\uc774 \uc218\uc0ac\ubc29\ud574\ud589\uc704\ub77c\uace0 \ud558\uba74\uc11c \ud2b9\ubcc4\uc218\uc0ac\ud300\uc7a5\uc758 \uc989\uac01\uc801\uc778 \ubcf5\uadc0\uc640 \ud2b9\uac80 \uc2e4\uc2dc \ub4f1\uc744 \uc8fc\uc7a5\ud558\uc600\ub2e4.", "paragraph_answer": "\ub300\ud55c\ubbfc\uad6d \uad6d\uac00\uc815\ubcf4\uc6d0 \uc5ec\ub860 \uc870\uc791 \uc0ac\uac74 \uadf8 \ud6c4 \uad6d\uac00\uc815\ubcf4\uc6d0 \ub300\uc120 \uc5ec\ub860\uc870\uc791 \uc0ac\uac74 \ud2b9\ubcc4\uc218\uc0ac\ud300\uc7a5\uc744 \ub9e1\uc544 \uc57d 8\uac1c\uc6d4\uac04\uc758 \uc218\uc0ac\ub97c \ud1b5\ud574 2012\ub144 \uc81c18\ub300 \ub300\uc120\uc5d0\uc11c \uad6d\uac00\uc815\ubcf4\uc6d0\uc774 \uc870\uc9c1\uc801\uc73c\ub85c SNS \uc0c1\uc5d0\uc11c \uc57c\ub2f9 \ub300\uc120 \ud6c4\ubcf4\ub97c \ube44\ubc29\ud558\uace0 \uc5ec\ub2f9 \ud6c4\ubcf4\ub97c \uc9c0\uc9c0\ud558\ub294 \uc5ec\ub860\uc870\uc791\uc744 \uc2e4\ud589\ud55c \uc0ac\uc2e4\uc744 \ubc1d\ud600\ub0b4\uc5c8\ub2e4. \uc218\uc0ac \ub3c4\uc911 \uad6d\uac00\uc815\ubcf4\uc6d0 \uc9c1\uc6d0\ub4e4\uc758 \ub300\ub300\uc801\uc778 \ud2b8\uc704\ud130 \ud65c\ub3d9 \uacfc \uad00\ub828\ud55c \uacb0\uc815\uc801\uc778 \uc99d\uac70\ub97c \ud3ec\ucc29\ud558\uace0 \uad6d\uac00\uc815\ubcf4\uc6d0 \uc9c1\uc6d0\ub4e4\uc5d0 \ub300\ud55c \uccb4\ud3ec\uc640 \uc555\uc218\uc218\uc0c9\uc744 \uc9d1\ud589\ud558\uc600\ub294\ub370 \uc11c\uc6b8\uc911\uc559\uc9c0\ubc29\uac80\ucc30\uccad \uc870\uc601\uace4 \uac80\uc0ac\uc7a5\uc5d0\uac8c \ubcf4\uace0\ud588\uc9c0\ub9cc \ub3d9\uc758\ubc1b\uc9c0 \ubabb\ud558\uc790 \uc790\uc2e0\uc758 \ucc45\uc784\uc73c\ub85c \uccb4\ud3ec\ub97c \uac15\ud589\ud558\uc600\uae30 \ub54c\ubb38\uc774\ub77c \ud558\uc5ec \ud2b9\ubcc4\uc218\uc0ac\ud300\uc7a5\uc5d0\uc11c \uacbd\uc9c8\ub418\uc5c8\ub2e4\ub294 \ubcf4\ub3c4\uac00 \uc788\ub2e4. \ub2f9\uc2dc \uc57c\ub2f9\uacfc \uc2dc\ubbfc\ub2e8\uccb4, \uc885\uad50\uacc4, \uc5b8\ub860 \ub4f1\uc5d0\uc11c\ub294 \ud2b9\ubcc4\uc218\uc0ac\ud300\uc7a5 \uacbd\uc9c8\uc774 \uc218\uc0ac\ubc29\ud574\ud589\uc704\ub77c\uace0 \ud558\uba74\uc11c \ud2b9\ubcc4\uc218\uc0ac\ud300\uc7a5\uc758 \uc989\uac01\uc801\uc778 \ubcf5\uadc0\uc640 \ud2b9\uac80 \uc2e4\uc2dc \ub4f1\uc744 \uc8fc\uc7a5\ud558\uc600\ub2e4.", "sentence_answer": "\uc218\uc0ac \ub3c4\uc911 \uad6d\uac00\uc815\ubcf4\uc6d0 \uc9c1\uc6d0\ub4e4\uc758 \ub300\ub300\uc801\uc778 \ud2b8\uc704\ud130 \ud65c\ub3d9 \uacfc \uad00\ub828\ud55c \uacb0\uc815\uc801\uc778 \uc99d\uac70\ub97c \ud3ec\ucc29\ud558\uace0 \uad6d\uac00\uc815\ubcf4\uc6d0 \uc9c1\uc6d0\ub4e4\uc5d0 \ub300\ud55c \uccb4\ud3ec\uc640 \uc555\uc218\uc218\uc0c9\uc744 \uc9d1\ud589\ud558\uc600\ub294\ub370 \uc11c\uc6b8\uc911\uc559\uc9c0\ubc29\uac80\ucc30\uccad \uc870\uc601\uace4 \uac80\uc0ac\uc7a5\uc5d0\uac8c \ubcf4\uace0\ud588\uc9c0\ub9cc \ub3d9\uc758\ubc1b\uc9c0 \ubabb\ud558\uc790 \uc790\uc2e0\uc758 \ucc45\uc784\uc73c\ub85c \uccb4\ud3ec\ub97c \uac15\ud589\ud558\uc600\uae30 \ub54c\ubb38\uc774\ub77c \ud558\uc5ec \ud2b9\ubcc4\uc218\uc0ac\ud300\uc7a5\uc5d0\uc11c \uacbd\uc9c8\ub418\uc5c8\ub2e4\ub294 \ubcf4\ub3c4\uac00 \uc788\ub2e4.", "paragraph_id": "5835542-0-2", "paragraph_question": "question: \uc724\uc11d\uc5f4\uc740 18\ub300 \ub300\uc120 \uc5ec\ub860\uc870\uc791\uc0ac\uac74 \uc218\uc0ac\uc911 \uad6d\uac00\uc815\ubcf4\uc6d0 \uc9c1\uc6d0\ub4e4\uc758 \uc5b4\ub5a4 \ud65c\ub3d9\uacfc \uad00\ub828\ud55c \uacb0\uc815\uc801 \uc99d\uac70\ub97c \ud3ec\ucc29\ud558\uc600\ub098\uc694?, context: \ub300\ud55c\ubbfc\uad6d \uad6d\uac00\uc815\ubcf4\uc6d0 \uc5ec\ub860 \uc870\uc791 \uc0ac\uac74 \uadf8 \ud6c4 \uad6d\uac00\uc815\ubcf4\uc6d0 \ub300\uc120 \uc5ec\ub860\uc870\uc791 \uc0ac\uac74 \ud2b9\ubcc4\uc218\uc0ac\ud300\uc7a5\uc744 \ub9e1\uc544 \uc57d 8\uac1c\uc6d4\uac04\uc758 \uc218\uc0ac\ub97c \ud1b5\ud574 2012\ub144 \uc81c18\ub300 \ub300\uc120\uc5d0\uc11c \uad6d\uac00\uc815\ubcf4\uc6d0\uc774 \uc870\uc9c1\uc801\uc73c\ub85c SNS \uc0c1\uc5d0\uc11c \uc57c\ub2f9 \ub300\uc120 \ud6c4\ubcf4\ub97c \ube44\ubc29\ud558\uace0 \uc5ec\ub2f9 \ud6c4\ubcf4\ub97c \uc9c0\uc9c0\ud558\ub294 \uc5ec\ub860\uc870\uc791\uc744 \uc2e4\ud589\ud55c \uc0ac\uc2e4\uc744 \ubc1d\ud600\ub0b4\uc5c8\ub2e4. \uc218\uc0ac \ub3c4\uc911 \uad6d\uac00\uc815\ubcf4\uc6d0 \uc9c1\uc6d0\ub4e4\uc758 \ub300\ub300\uc801\uc778 \ud2b8\uc704\ud130 \ud65c\ub3d9\uacfc \uad00\ub828\ud55c \uacb0\uc815\uc801\uc778 \uc99d\uac70\ub97c \ud3ec\ucc29\ud558\uace0 \uad6d\uac00\uc815\ubcf4\uc6d0 \uc9c1\uc6d0\ub4e4\uc5d0 \ub300\ud55c \uccb4\ud3ec\uc640 \uc555\uc218\uc218\uc0c9\uc744 \uc9d1\ud589\ud558\uc600\ub294\ub370 \uc11c\uc6b8\uc911\uc559\uc9c0\ubc29\uac80\ucc30\uccad \uc870\uc601\uace4 \uac80\uc0ac\uc7a5\uc5d0\uac8c \ubcf4\uace0\ud588\uc9c0\ub9cc \ub3d9\uc758\ubc1b\uc9c0 \ubabb\ud558\uc790 \uc790\uc2e0\uc758 \ucc45\uc784\uc73c\ub85c \uccb4\ud3ec\ub97c \uac15\ud589\ud558\uc600\uae30 \ub54c\ubb38\uc774\ub77c \ud558\uc5ec \ud2b9\ubcc4\uc218\uc0ac\ud300\uc7a5\uc5d0\uc11c \uacbd\uc9c8\ub418\uc5c8\ub2e4\ub294 \ubcf4\ub3c4\uac00 \uc788\ub2e4. \ub2f9\uc2dc \uc57c\ub2f9\uacfc \uc2dc\ubbfc\ub2e8\uccb4, \uc885\uad50\uacc4, \uc5b8\ub860 \ub4f1\uc5d0\uc11c\ub294 \ud2b9\ubcc4\uc218\uc0ac\ud300\uc7a5 \uacbd\uc9c8\uc774 \uc218\uc0ac\ubc29\ud574\ud589\uc704\ub77c\uace0 \ud558\uba74\uc11c \ud2b9\ubcc4\uc218\uc0ac\ud300\uc7a5\uc758 \uc989\uac01\uc801\uc778 \ubcf5\uadc0\uc640 \ud2b9\uac80 \uc2e4\uc2dc \ub4f1\uc744 \uc8fc\uc7a5\ud558\uc600\ub2e4."} {"question": "194\ub144 \ubca0\ub97c\ub9b0\uc5d0\uc11c \ube44\uc804\ubb38\uac00 \ubc34\ub4dc\ub97c \uc704\ud574 \uc5f4\ub9b0 \uacbd\uc5f0\ub300\ud68c\uc5d0\uc11c \uc6b0\uc2b9\ud55c \ubc34\ub4dc\ub294 \uba87 \uba85\uc73c\ub85c \uc774\ub8e8\uc5b4\uc84c\ub294\uac00?", "paragraph": "1994\ub144 \ubca0\ub97c\ub9b0\uc5d0\uc11c \ube44\uc804\ubb38\uac00 \ubc34\ub4dc\ub97c \uc704\ud55c \uacbd\uc5f0\ub300\ud68c\uac00 \uc5f4\ub838\ub2e4. \ucf58\ud14c\uc2a4\ud2b8\uc5d0\uc11c \uc6b0\uc2b9\ud558\ub294 \ubc34\ub4dc\uc5d0\uac8c\ub294 \ub124 \uace1\uc9dc\ub9ac \ub370\ubaa8\uc568\ubc94\uc744 \uc804\ubb38 \ub179\uc74c\uc2e4\uc5d0\uc11c \uc81c\uc791\ud560 \uc218 \uc788\ub294 \uae30\ud68c\uac00 \uc8fc\uc5b4\uc9c0\ub294 \ub300\ud68c\uc600\ub2e4. \ud06c\ub8e8\uc2a4\ud398, \uc288\ub098\uc774\ub354, \ub9ac\ub378, \uadf8\ub9ac\uace0 \ub9b0\ub370\ub9cc \ub124\uba85\uc73c\ub85c \uc774\ub8e8\uc5b4\uc9c4 \ubc34\ub4dc\uac00 \uacbd\uc5f0\uc5d0 \ucc38\uac00\ud558\uc5ec \uc6b0\uc2b9\ud558\uc600\uace0, \uc774\ub294 \ud30c\uc6b8 \ub780\ub354\uc2a4\uc758 \uad00\uc2ec\uc744 \ub04c\uac8c \ub41c\ub2e4. \ud604 \ub78c\uc288\ud0c0\uc778\uc758 \uae30\ud0c0\ub9ac\uc2a4\ud2b8 \ud30c\uc6b8 \ub780\ub354\uc2a4\ub294 \ucd08\uae30 \ub78c\uc288\ud0c0\uc778\uc758 \uc2dc\ubc94 \ud2b8\ub799\uc744 \ub4e3\uace0 \ud070 \uad00\uc2ec\uc744 \ubcf4\uc774\uba70 4\uc778 \ubc34\ub4dc \ub78c\uc288\ud0c0\uc778\uc5d0 \ucc38\uc5ec\ud558\uac8c \ub41c\ub2e4. \uc18c\ub9ac\uc758 \uc644\uc131\ub3c4\ub97c \ub192\uc774\uae30 \uc704\ud574\uc11c \ub78c\uc288\ud0c0\uc778\uc740 \ud30c\uc6b8 \ub780\ub354\uc2a4\uc640 Feeling B\ub77c\ub294 \ud391\ud06c\ubc34\ub4dc\uc5d0\uc11c \ud65c\uc57d\ud558\ub358 \uac74\ubc18 \uc8fc\uc790 \ud06c\ub9ac\uc2a4\ud2f0\uc548 \ub85c\ub80c\uce20\uc5d0\uac8c \ud569\ub958\ub97c \uad8c\uc720\ud558\uac8c \ub41c\ub2e4. \ucc98\uc74c \ub85c\ub80c\uce20\ub294 \ud569\ub958\uc81c\uc548\uc744 \ud0d0\ud0c1\uce58 \uc54a\uc544\ud588\uc9c0\ub9cc, \uacb0\uad6d \uc218\ub77d\ud558\uac8c \ub41c\ub2e4.", "answer": "\ub124\uba85", "sentence": "\ud06c\ub8e8\uc2a4\ud398, \uc288\ub098\uc774\ub354, \ub9ac\ub378, \uadf8\ub9ac\uace0 \ub9b0\ub370\ub9cc \ub124\uba85 \uc73c\ub85c \uc774\ub8e8\uc5b4\uc9c4 \ubc34\ub4dc\uac00 \uacbd\uc5f0\uc5d0 \ucc38\uac00\ud558\uc5ec \uc6b0\uc2b9\ud558\uc600\uace0, \uc774\ub294 \ud30c\uc6b8 \ub780\ub354\uc2a4\uc758 \uad00\uc2ec\uc744 \ub04c\uac8c \ub41c\ub2e4.", "paragraph_sentence": "1994\ub144 \ubca0\ub97c\ub9b0\uc5d0\uc11c \ube44\uc804\ubb38\uac00 \ubc34\ub4dc\ub97c \uc704\ud55c \uacbd\uc5f0\ub300\ud68c\uac00 \uc5f4\ub838\ub2e4. \ucf58\ud14c\uc2a4\ud2b8\uc5d0\uc11c \uc6b0\uc2b9\ud558\ub294 \ubc34\ub4dc\uc5d0\uac8c\ub294 \ub124 \uace1\uc9dc\ub9ac \ub370\ubaa8\uc568\ubc94\uc744 \uc804\ubb38 \ub179\uc74c\uc2e4\uc5d0\uc11c \uc81c\uc791\ud560 \uc218 \uc788\ub294 \uae30\ud68c\uac00 \uc8fc\uc5b4\uc9c0\ub294 \ub300\ud68c\uc600\ub2e4. \ud06c\ub8e8\uc2a4\ud398, \uc288\ub098\uc774\ub354, \ub9ac\ub378, \uadf8\ub9ac\uace0 \ub9b0\ub370\ub9cc \ub124\uba85 \uc73c\ub85c \uc774\ub8e8\uc5b4\uc9c4 \ubc34\ub4dc\uac00 \uacbd\uc5f0\uc5d0 \ucc38\uac00\ud558\uc5ec \uc6b0\uc2b9\ud558\uc600\uace0, \uc774\ub294 \ud30c\uc6b8 \ub780\ub354\uc2a4\uc758 \uad00\uc2ec\uc744 \ub04c\uac8c \ub41c\ub2e4. \ud604 \ub78c\uc288\ud0c0\uc778\uc758 \uae30\ud0c0\ub9ac\uc2a4\ud2b8 \ud30c\uc6b8 \ub780\ub354\uc2a4\ub294 \ucd08\uae30 \ub78c\uc288\ud0c0\uc778\uc758 \uc2dc\ubc94 \ud2b8\ub799\uc744 \ub4e3\uace0 \ud070 \uad00\uc2ec\uc744 \ubcf4\uc774\uba70 4\uc778 \ubc34\ub4dc \ub78c\uc288\ud0c0\uc778\uc5d0 \ucc38\uc5ec\ud558\uac8c \ub41c\ub2e4. \uc18c\ub9ac\uc758 \uc644\uc131\ub3c4\ub97c \ub192\uc774\uae30 \uc704\ud574\uc11c \ub78c\uc288\ud0c0\uc778\uc740 \ud30c\uc6b8 \ub780\ub354\uc2a4\uc640 Feeling B\ub77c\ub294 \ud391\ud06c\ubc34\ub4dc\uc5d0\uc11c \ud65c\uc57d\ud558\ub358 \uac74\ubc18 \uc8fc\uc790 \ud06c\ub9ac\uc2a4\ud2f0\uc548 \ub85c\ub80c\uce20\uc5d0\uac8c \ud569\ub958\ub97c \uad8c\uc720\ud558\uac8c \ub41c\ub2e4. \ucc98\uc74c \ub85c\ub80c\uce20\ub294 \ud569\ub958\uc81c\uc548\uc744 \ud0d0\ud0c1\uce58 \uc54a\uc544\ud588\uc9c0\ub9cc, \uacb0\uad6d \uc218\ub77d\ud558\uac8c \ub41c\ub2e4.", "paragraph_answer": "1994\ub144 \ubca0\ub97c\ub9b0\uc5d0\uc11c \ube44\uc804\ubb38\uac00 \ubc34\ub4dc\ub97c \uc704\ud55c \uacbd\uc5f0\ub300\ud68c\uac00 \uc5f4\ub838\ub2e4. \ucf58\ud14c\uc2a4\ud2b8\uc5d0\uc11c \uc6b0\uc2b9\ud558\ub294 \ubc34\ub4dc\uc5d0\uac8c\ub294 \ub124 \uace1\uc9dc\ub9ac \ub370\ubaa8\uc568\ubc94\uc744 \uc804\ubb38 \ub179\uc74c\uc2e4\uc5d0\uc11c \uc81c\uc791\ud560 \uc218 \uc788\ub294 \uae30\ud68c\uac00 \uc8fc\uc5b4\uc9c0\ub294 \ub300\ud68c\uc600\ub2e4. \ud06c\ub8e8\uc2a4\ud398, \uc288\ub098\uc774\ub354, \ub9ac\ub378, \uadf8\ub9ac\uace0 \ub9b0\ub370\ub9cc \ub124\uba85 \uc73c\ub85c \uc774\ub8e8\uc5b4\uc9c4 \ubc34\ub4dc\uac00 \uacbd\uc5f0\uc5d0 \ucc38\uac00\ud558\uc5ec \uc6b0\uc2b9\ud558\uc600\uace0, \uc774\ub294 \ud30c\uc6b8 \ub780\ub354\uc2a4\uc758 \uad00\uc2ec\uc744 \ub04c\uac8c \ub41c\ub2e4. \ud604 \ub78c\uc288\ud0c0\uc778\uc758 \uae30\ud0c0\ub9ac\uc2a4\ud2b8 \ud30c\uc6b8 \ub780\ub354\uc2a4\ub294 \ucd08\uae30 \ub78c\uc288\ud0c0\uc778\uc758 \uc2dc\ubc94 \ud2b8\ub799\uc744 \ub4e3\uace0 \ud070 \uad00\uc2ec\uc744 \ubcf4\uc774\uba70 4\uc778 \ubc34\ub4dc \ub78c\uc288\ud0c0\uc778\uc5d0 \ucc38\uc5ec\ud558\uac8c \ub41c\ub2e4. \uc18c\ub9ac\uc758 \uc644\uc131\ub3c4\ub97c \ub192\uc774\uae30 \uc704\ud574\uc11c \ub78c\uc288\ud0c0\uc778\uc740 \ud30c\uc6b8 \ub780\ub354\uc2a4\uc640 Feeling B\ub77c\ub294 \ud391\ud06c\ubc34\ub4dc\uc5d0\uc11c \ud65c\uc57d\ud558\ub358 \uac74\ubc18 \uc8fc\uc790 \ud06c\ub9ac\uc2a4\ud2f0\uc548 \ub85c\ub80c\uce20\uc5d0\uac8c \ud569\ub958\ub97c \uad8c\uc720\ud558\uac8c \ub41c\ub2e4. \ucc98\uc74c \ub85c\ub80c\uce20\ub294 \ud569\ub958\uc81c\uc548\uc744 \ud0d0\ud0c1\uce58 \uc54a\uc544\ud588\uc9c0\ub9cc, \uacb0\uad6d \uc218\ub77d\ud558\uac8c \ub41c\ub2e4.", "sentence_answer": "\ud06c\ub8e8\uc2a4\ud398, \uc288\ub098\uc774\ub354, \ub9ac\ub378, \uadf8\ub9ac\uace0 \ub9b0\ub370\ub9cc \ub124\uba85 \uc73c\ub85c \uc774\ub8e8\uc5b4\uc9c4 \ubc34\ub4dc\uac00 \uacbd\uc5f0\uc5d0 \ucc38\uac00\ud558\uc5ec \uc6b0\uc2b9\ud558\uc600\uace0, \uc774\ub294 \ud30c\uc6b8 \ub780\ub354\uc2a4\uc758 \uad00\uc2ec\uc744 \ub04c\uac8c \ub41c\ub2e4.", "paragraph_id": "6534087-1-0", "paragraph_question": "question: 194\ub144 \ubca0\ub97c\ub9b0\uc5d0\uc11c \ube44\uc804\ubb38\uac00 \ubc34\ub4dc\ub97c \uc704\ud574 \uc5f4\ub9b0 \uacbd\uc5f0\ub300\ud68c\uc5d0\uc11c \uc6b0\uc2b9\ud55c \ubc34\ub4dc\ub294 \uba87 \uba85\uc73c\ub85c \uc774\ub8e8\uc5b4\uc84c\ub294\uac00?, context: 1994\ub144 \ubca0\ub97c\ub9b0\uc5d0\uc11c \ube44\uc804\ubb38\uac00 \ubc34\ub4dc\ub97c \uc704\ud55c \uacbd\uc5f0\ub300\ud68c\uac00 \uc5f4\ub838\ub2e4. \ucf58\ud14c\uc2a4\ud2b8\uc5d0\uc11c \uc6b0\uc2b9\ud558\ub294 \ubc34\ub4dc\uc5d0\uac8c\ub294 \ub124 \uace1\uc9dc\ub9ac \ub370\ubaa8\uc568\ubc94\uc744 \uc804\ubb38 \ub179\uc74c\uc2e4\uc5d0\uc11c \uc81c\uc791\ud560 \uc218 \uc788\ub294 \uae30\ud68c\uac00 \uc8fc\uc5b4\uc9c0\ub294 \ub300\ud68c\uc600\ub2e4. \ud06c\ub8e8\uc2a4\ud398, \uc288\ub098\uc774\ub354, \ub9ac\ub378, \uadf8\ub9ac\uace0 \ub9b0\ub370\ub9cc \ub124\uba85\uc73c\ub85c \uc774\ub8e8\uc5b4\uc9c4 \ubc34\ub4dc\uac00 \uacbd\uc5f0\uc5d0 \ucc38\uac00\ud558\uc5ec \uc6b0\uc2b9\ud558\uc600\uace0, \uc774\ub294 \ud30c\uc6b8 \ub780\ub354\uc2a4\uc758 \uad00\uc2ec\uc744 \ub04c\uac8c \ub41c\ub2e4. \ud604 \ub78c\uc288\ud0c0\uc778\uc758 \uae30\ud0c0\ub9ac\uc2a4\ud2b8 \ud30c\uc6b8 \ub780\ub354\uc2a4\ub294 \ucd08\uae30 \ub78c\uc288\ud0c0\uc778\uc758 \uc2dc\ubc94 \ud2b8\ub799\uc744 \ub4e3\uace0 \ud070 \uad00\uc2ec\uc744 \ubcf4\uc774\uba70 4\uc778 \ubc34\ub4dc \ub78c\uc288\ud0c0\uc778\uc5d0 \ucc38\uc5ec\ud558\uac8c \ub41c\ub2e4. \uc18c\ub9ac\uc758 \uc644\uc131\ub3c4\ub97c \ub192\uc774\uae30 \uc704\ud574\uc11c \ub78c\uc288\ud0c0\uc778\uc740 \ud30c\uc6b8 \ub780\ub354\uc2a4\uc640 Feeling B\ub77c\ub294 \ud391\ud06c\ubc34\ub4dc\uc5d0\uc11c \ud65c\uc57d\ud558\ub358 \uac74\ubc18 \uc8fc\uc790 \ud06c\ub9ac\uc2a4\ud2f0\uc548 \ub85c\ub80c\uce20\uc5d0\uac8c \ud569\ub958\ub97c \uad8c\uc720\ud558\uac8c \ub41c\ub2e4. \ucc98\uc74c \ub85c\ub80c\uce20\ub294 \ud569\ub958\uc81c\uc548\uc744 \ud0d0\ud0c1\uce58 \uc54a\uc544\ud588\uc9c0\ub9cc, \uacb0\uad6d \uc218\ub77d\ud558\uac8c \ub41c\ub2e4."} {"question": "\ud0dc\uc870\ub294 \uba87\ub144\ub3c4\uc5d0 \ud0dc\uc5b4\ub0ac\ub098?", "paragraph": "\uc1a1\uc545(\u677e\u5dbd)\uc758 \ud638\uc871 \uc655\ub96d(\u738b\u9686)\uacfc \uadf8 \ubd80\uc778 \ud55c\uc528(\u97d3\u6c0f) \uc0ac\uc774\uc5d0\uc11c 877\ub144\uc5d0 \ud0dc\uc5b4\ub0ac\uc73c\uba70, 896\ub144\uc5d0 \uad81\uc608(\u5f13\u88d4)\uc758 \ud718\ud558\uc5d0 \ub4e4\uc5b4\uac00 \uadf8\uc758 \uc7a5\uc218(\u5c07\u5e25)\uac00 \ub418\uc5c8\ub2e4. \ud6c4\ubc31\uc81c(\u5f8c\u767e\u6fdf)\uc640\uc758 \uad50\uc804\uc5d0\uc11c \uac70\ub4ed \uc2b9\ub9ac\ud558\uc5ec \uc804\ub77c\ub3c4(\u5168\u7f85\u9053)\uc640 \uacbd\uc0c1\ub3c4(\u6176\u5c19\u9053) \uc11c\ubd80 \uc9c0\uc5ed\uc5d0\uc11c \uacac\ud6e4(\u7504\u8431)\uc758 \uad70\uc0ac(\u8ecd\u4e8b)\ub97c \uc5ec\ub7ec \ubc88 \uaca9\ud30c\ud558\uace0 906\ub144 \uc0c1\uc8fc(\u5c19\u5dde) \uc758 \uc0ac\ud654\uc9c4, 909\ub144 \uc9c4\ub3c4 \ubd80\uadfc\uc758 \ub3c4\uc11c\uc640 \ub098\uc8fc(\u7f85\u5dde)\ub97c \uacf5\ub7b5\ud558\uc600\ub2e4. \uc774\uc5b4 \ucda9\uc8fc(\u5fe0\u5dde)\uc640 \uccad\uc8fc(\u6df8\u5dde) \ub4f1\uc758 \ucda9\uccad\ub3c4(\u5fe0\u6df8\u9053) \uc9c0\uc5ed\uacfc \uacbd\uc0c1\ubd81\ub3c4(\u6176\u5c19\u5317\u9053)\uc758 \uc0c1\uc8fc(\u5c19\u5dde) \ub4f1\uc744 \uc810\ub839\ud558\uc5ec \ud0dc\ubd09\uad6d(\u6cf0\u5c01\u570b)\uc758 \uc138\ub825\uad8c\uc744 \ub113\ud614\uc73c\uba70, \ub098\uc8fc(\u7f85\u5dde)\ub97c \uacf5\ub7b5\ud558\uc5ec \ud6c4\ubc31\uc81c(\u5f8c\u767e\u6fdf)\uc758 \ubc30\ud6c4\ub97c \uc704\ud611\ud558\uace0, \uc911\uad6d\uacfc\uc758 \ubc43\uae38\uc744 \ucc28\ub2e8\ud558\uc5ec \uad6d\ub825\uc744 \ud655\uc7a5\uc2dc\ucf30\ub2e4. \ub6f0\uc5b4\ub09c \uc804\uacfc\ub97c \ubc14\ud0d5\uc73c\ub85c \uad81\uc608(\u5f13\u88d4)\uc758 \ucd1d\uc560\ub97c \ubc1b\uc73c\uba70 \ub9c8\ud754 \uc0b4\ub3c4 \ub418\uc9c0 \uc54a\uc740 \uc80a\uc740 \ub098\uc774\uc5d0 \ubc31\uad00\uc758 \uc6b0\ub450\uba38\ub9ac\uc778 \uc2dc\uc911(\u4f8d\u4e2d)\uc5d0\uae4c\uc9c0 \uc62c\ub790\uace0, 918\ub144\uc5d0 \uad81\uc608(\u5f13\u88d4)\uc758 \ub3c5\ub2e8\uacfc \uc804\ud6a1\uc744 \ubb38\uc81c\uc0bc\uc740 \uc5ec\ub7ec \ud638\uc871\ub4e4\uacfc \ubc30\ud604\uacbd(\u88f5\u7384\u6176), \ud64d\uc720(\u6d2a\u5112) \ub4f1 \ubb34\uc7a5\ub4e4\uc758 \uc9c0\uc9c0\ub85c \uac70\ubcd1\ud558\uc5ec \ub9c8\uce68\ub0b4 \uad81\uc608(\u5f13\u88d4)\ub97c \ucd95\ucd9c\ud558\uace0 \uace0\ub824(\u9ad8\u9e97)\ub97c \uc138\uc6e0\ub2e4.", "answer": "877\ub144", "sentence": "\uc1a1\uc545(\u677e\u5dbd)\uc758 \ud638\uc871 \uc655\ub96d(\u738b\u9686)\uacfc \uadf8 \ubd80\uc778 \ud55c\uc528(\u97d3\u6c0f) \uc0ac\uc774\uc5d0\uc11c 877\ub144 \uc5d0 \ud0dc\uc5b4\ub0ac\uc73c\uba70, 896\ub144\uc5d0 \uad81\uc608(\u5f13\u88d4)\uc758 \ud718\ud558\uc5d0 \ub4e4\uc5b4\uac00 \uadf8\uc758 \uc7a5\uc218(\u5c07\u5e25)\uac00 \ub418\uc5c8\ub2e4.", "paragraph_sentence": " \uc1a1\uc545(\u677e\u5dbd)\uc758 \ud638\uc871 \uc655\ub96d(\u738b\u9686)\uacfc \uadf8 \ubd80\uc778 \ud55c\uc528(\u97d3\u6c0f) \uc0ac\uc774\uc5d0\uc11c 877\ub144 \uc5d0 \ud0dc\uc5b4\ub0ac\uc73c\uba70, 896\ub144\uc5d0 \uad81\uc608(\u5f13\u88d4)\uc758 \ud718\ud558\uc5d0 \ub4e4\uc5b4\uac00 \uadf8\uc758 \uc7a5\uc218(\u5c07\u5e25)\uac00 \ub418\uc5c8\ub2e4. \ud6c4\ubc31\uc81c(\u5f8c\u767e\u6fdf)\uc640\uc758 \uad50\uc804\uc5d0\uc11c \uac70\ub4ed \uc2b9\ub9ac\ud558\uc5ec \uc804\ub77c\ub3c4(\u5168\u7f85\u9053)\uc640 \uacbd\uc0c1\ub3c4(\u6176\u5c19\u9053) \uc11c\ubd80 \uc9c0\uc5ed\uc5d0\uc11c \uacac\ud6e4(\u7504\u8431)\uc758 \uad70\uc0ac(\u8ecd\u4e8b)\ub97c \uc5ec\ub7ec \ubc88 \uaca9\ud30c\ud558\uace0 906\ub144 \uc0c1\uc8fc(\u5c19\u5dde) \uc758 \uc0ac\ud654\uc9c4, 909\ub144 \uc9c4\ub3c4 \ubd80\uadfc\uc758 \ub3c4\uc11c\uc640 \ub098\uc8fc(\u7f85\u5dde)\ub97c \uacf5\ub7b5\ud558\uc600\ub2e4. \uc774\uc5b4 \ucda9\uc8fc(\u5fe0\u5dde)\uc640 \uccad\uc8fc(\u6df8\u5dde) \ub4f1\uc758 \ucda9\uccad\ub3c4(\u5fe0\u6df8\u9053) \uc9c0\uc5ed\uacfc \uacbd\uc0c1\ubd81\ub3c4(\u6176\u5c19\u5317\u9053)\uc758 \uc0c1\uc8fc(\u5c19\u5dde) \ub4f1\uc744 \uc810\ub839\ud558\uc5ec \ud0dc\ubd09\uad6d(\u6cf0\u5c01\u570b)\uc758 \uc138\ub825\uad8c\uc744 \ub113\ud614\uc73c\uba70, \ub098\uc8fc(\u7f85\u5dde)\ub97c \uacf5\ub7b5\ud558\uc5ec \ud6c4\ubc31\uc81c(\u5f8c\u767e\u6fdf)\uc758 \ubc30\ud6c4\ub97c \uc704\ud611\ud558\uace0, \uc911\uad6d\uacfc\uc758 \ubc43\uae38\uc744 \ucc28\ub2e8\ud558\uc5ec \uad6d\ub825\uc744 \ud655\uc7a5\uc2dc\ucf30\ub2e4. \ub6f0\uc5b4\ub09c \uc804\uacfc\ub97c \ubc14\ud0d5\uc73c\ub85c \uad81\uc608(\u5f13\u88d4)\uc758 \ucd1d\uc560\ub97c \ubc1b\uc73c\uba70 \ub9c8\ud754 \uc0b4\ub3c4 \ub418\uc9c0 \uc54a\uc740 \uc80a\uc740 \ub098\uc774\uc5d0 \ubc31\uad00\uc758 \uc6b0\ub450\uba38\ub9ac\uc778 \uc2dc\uc911(\u4f8d\u4e2d)\uc5d0\uae4c\uc9c0 \uc62c\ub790\uace0, 918\ub144\uc5d0 \uad81\uc608(\u5f13\u88d4)\uc758 \ub3c5\ub2e8\uacfc \uc804\ud6a1\uc744 \ubb38\uc81c\uc0bc\uc740 \uc5ec\ub7ec \ud638\uc871\ub4e4\uacfc \ubc30\ud604\uacbd(\u88f5\u7384\u6176), \ud64d\uc720(\u6d2a\u5112) \ub4f1 \ubb34\uc7a5\ub4e4\uc758 \uc9c0\uc9c0\ub85c \uac70\ubcd1\ud558\uc5ec \ub9c8\uce68\ub0b4 \uad81\uc608(\u5f13\u88d4)\ub97c \ucd95\ucd9c\ud558\uace0 \uace0\ub824(\u9ad8\u9e97)\ub97c \uc138\uc6e0\ub2e4.", "paragraph_answer": "\uc1a1\uc545(\u677e\u5dbd)\uc758 \ud638\uc871 \uc655\ub96d(\u738b\u9686)\uacfc \uadf8 \ubd80\uc778 \ud55c\uc528(\u97d3\u6c0f) \uc0ac\uc774\uc5d0\uc11c 877\ub144 \uc5d0 \ud0dc\uc5b4\ub0ac\uc73c\uba70, 896\ub144\uc5d0 \uad81\uc608(\u5f13\u88d4)\uc758 \ud718\ud558\uc5d0 \ub4e4\uc5b4\uac00 \uadf8\uc758 \uc7a5\uc218(\u5c07\u5e25)\uac00 \ub418\uc5c8\ub2e4. \ud6c4\ubc31\uc81c(\u5f8c\u767e\u6fdf)\uc640\uc758 \uad50\uc804\uc5d0\uc11c \uac70\ub4ed \uc2b9\ub9ac\ud558\uc5ec \uc804\ub77c\ub3c4(\u5168\u7f85\u9053)\uc640 \uacbd\uc0c1\ub3c4(\u6176\u5c19\u9053) \uc11c\ubd80 \uc9c0\uc5ed\uc5d0\uc11c \uacac\ud6e4(\u7504\u8431)\uc758 \uad70\uc0ac(\u8ecd\u4e8b)\ub97c \uc5ec\ub7ec \ubc88 \uaca9\ud30c\ud558\uace0 906\ub144 \uc0c1\uc8fc(\u5c19\u5dde) \uc758 \uc0ac\ud654\uc9c4, 909\ub144 \uc9c4\ub3c4 \ubd80\uadfc\uc758 \ub3c4\uc11c\uc640 \ub098\uc8fc(\u7f85\u5dde)\ub97c \uacf5\ub7b5\ud558\uc600\ub2e4. \uc774\uc5b4 \ucda9\uc8fc(\u5fe0\u5dde)\uc640 \uccad\uc8fc(\u6df8\u5dde) \ub4f1\uc758 \ucda9\uccad\ub3c4(\u5fe0\u6df8\u9053) \uc9c0\uc5ed\uacfc \uacbd\uc0c1\ubd81\ub3c4(\u6176\u5c19\u5317\u9053)\uc758 \uc0c1\uc8fc(\u5c19\u5dde) \ub4f1\uc744 \uc810\ub839\ud558\uc5ec \ud0dc\ubd09\uad6d(\u6cf0\u5c01\u570b)\uc758 \uc138\ub825\uad8c\uc744 \ub113\ud614\uc73c\uba70, \ub098\uc8fc(\u7f85\u5dde)\ub97c \uacf5\ub7b5\ud558\uc5ec \ud6c4\ubc31\uc81c(\u5f8c\u767e\u6fdf)\uc758 \ubc30\ud6c4\ub97c \uc704\ud611\ud558\uace0, \uc911\uad6d\uacfc\uc758 \ubc43\uae38\uc744 \ucc28\ub2e8\ud558\uc5ec \uad6d\ub825\uc744 \ud655\uc7a5\uc2dc\ucf30\ub2e4. \ub6f0\uc5b4\ub09c \uc804\uacfc\ub97c \ubc14\ud0d5\uc73c\ub85c \uad81\uc608(\u5f13\u88d4)\uc758 \ucd1d\uc560\ub97c \ubc1b\uc73c\uba70 \ub9c8\ud754 \uc0b4\ub3c4 \ub418\uc9c0 \uc54a\uc740 \uc80a\uc740 \ub098\uc774\uc5d0 \ubc31\uad00\uc758 \uc6b0\ub450\uba38\ub9ac\uc778 \uc2dc\uc911(\u4f8d\u4e2d)\uc5d0\uae4c\uc9c0 \uc62c\ub790\uace0, 918\ub144\uc5d0 \uad81\uc608(\u5f13\u88d4)\uc758 \ub3c5\ub2e8\uacfc \uc804\ud6a1\uc744 \ubb38\uc81c\uc0bc\uc740 \uc5ec\ub7ec \ud638\uc871\ub4e4\uacfc \ubc30\ud604\uacbd(\u88f5\u7384\u6176), \ud64d\uc720(\u6d2a\u5112) \ub4f1 \ubb34\uc7a5\ub4e4\uc758 \uc9c0\uc9c0\ub85c \uac70\ubcd1\ud558\uc5ec \ub9c8\uce68\ub0b4 \uad81\uc608(\u5f13\u88d4)\ub97c \ucd95\ucd9c\ud558\uace0 \uace0\ub824(\u9ad8\u9e97)\ub97c \uc138\uc6e0\ub2e4.", "sentence_answer": "\uc1a1\uc545(\u677e\u5dbd)\uc758 \ud638\uc871 \uc655\ub96d(\u738b\u9686)\uacfc \uadf8 \ubd80\uc778 \ud55c\uc528(\u97d3\u6c0f) \uc0ac\uc774\uc5d0\uc11c 877\ub144 \uc5d0 \ud0dc\uc5b4\ub0ac\uc73c\uba70, 896\ub144\uc5d0 \uad81\uc608(\u5f13\u88d4)\uc758 \ud718\ud558\uc5d0 \ub4e4\uc5b4\uac00 \uadf8\uc758 \uc7a5\uc218(\u5c07\u5e25)\uac00 \ub418\uc5c8\ub2e4.", "paragraph_id": "6528126-0-0", "paragraph_question": "question: \ud0dc\uc870\ub294 \uba87\ub144\ub3c4\uc5d0 \ud0dc\uc5b4\ub0ac\ub098?, context: \uc1a1\uc545(\u677e\u5dbd)\uc758 \ud638\uc871 \uc655\ub96d(\u738b\u9686)\uacfc \uadf8 \ubd80\uc778 \ud55c\uc528(\u97d3\u6c0f) \uc0ac\uc774\uc5d0\uc11c 877\ub144\uc5d0 \ud0dc\uc5b4\ub0ac\uc73c\uba70, 896\ub144\uc5d0 \uad81\uc608(\u5f13\u88d4)\uc758 \ud718\ud558\uc5d0 \ub4e4\uc5b4\uac00 \uadf8\uc758 \uc7a5\uc218(\u5c07\u5e25)\uac00 \ub418\uc5c8\ub2e4. \ud6c4\ubc31\uc81c(\u5f8c\u767e\u6fdf)\uc640\uc758 \uad50\uc804\uc5d0\uc11c \uac70\ub4ed \uc2b9\ub9ac\ud558\uc5ec \uc804\ub77c\ub3c4(\u5168\u7f85\u9053)\uc640 \uacbd\uc0c1\ub3c4(\u6176\u5c19\u9053) \uc11c\ubd80 \uc9c0\uc5ed\uc5d0\uc11c \uacac\ud6e4(\u7504\u8431)\uc758 \uad70\uc0ac(\u8ecd\u4e8b)\ub97c \uc5ec\ub7ec \ubc88 \uaca9\ud30c\ud558\uace0 906\ub144 \uc0c1\uc8fc(\u5c19\u5dde) \uc758 \uc0ac\ud654\uc9c4, 909\ub144 \uc9c4\ub3c4 \ubd80\uadfc\uc758 \ub3c4\uc11c\uc640 \ub098\uc8fc(\u7f85\u5dde)\ub97c \uacf5\ub7b5\ud558\uc600\ub2e4. \uc774\uc5b4 \ucda9\uc8fc(\u5fe0\u5dde)\uc640 \uccad\uc8fc(\u6df8\u5dde) \ub4f1\uc758 \ucda9\uccad\ub3c4(\u5fe0\u6df8\u9053) \uc9c0\uc5ed\uacfc \uacbd\uc0c1\ubd81\ub3c4(\u6176\u5c19\u5317\u9053)\uc758 \uc0c1\uc8fc(\u5c19\u5dde) \ub4f1\uc744 \uc810\ub839\ud558\uc5ec \ud0dc\ubd09\uad6d(\u6cf0\u5c01\u570b)\uc758 \uc138\ub825\uad8c\uc744 \ub113\ud614\uc73c\uba70, \ub098\uc8fc(\u7f85\u5dde)\ub97c \uacf5\ub7b5\ud558\uc5ec \ud6c4\ubc31\uc81c(\u5f8c\u767e\u6fdf)\uc758 \ubc30\ud6c4\ub97c \uc704\ud611\ud558\uace0, \uc911\uad6d\uacfc\uc758 \ubc43\uae38\uc744 \ucc28\ub2e8\ud558\uc5ec \uad6d\ub825\uc744 \ud655\uc7a5\uc2dc\ucf30\ub2e4. \ub6f0\uc5b4\ub09c \uc804\uacfc\ub97c \ubc14\ud0d5\uc73c\ub85c \uad81\uc608(\u5f13\u88d4)\uc758 \ucd1d\uc560\ub97c \ubc1b\uc73c\uba70 \ub9c8\ud754 \uc0b4\ub3c4 \ub418\uc9c0 \uc54a\uc740 \uc80a\uc740 \ub098\uc774\uc5d0 \ubc31\uad00\uc758 \uc6b0\ub450\uba38\ub9ac\uc778 \uc2dc\uc911(\u4f8d\u4e2d)\uc5d0\uae4c\uc9c0 \uc62c\ub790\uace0, 918\ub144\uc5d0 \uad81\uc608(\u5f13\u88d4)\uc758 \ub3c5\ub2e8\uacfc \uc804\ud6a1\uc744 \ubb38\uc81c\uc0bc\uc740 \uc5ec\ub7ec \ud638\uc871\ub4e4\uacfc \ubc30\ud604\uacbd(\u88f5\u7384\u6176), \ud64d\uc720(\u6d2a\u5112) \ub4f1 \ubb34\uc7a5\ub4e4\uc758 \uc9c0\uc9c0\ub85c \uac70\ubcd1\ud558\uc5ec \ub9c8\uce68\ub0b4 \uad81\uc608(\u5f13\u88d4)\ub97c \ucd95\ucd9c\ud558\uace0 \uace0\ub824(\u9ad8\u9e97)\ub97c \uc138\uc6e0\ub2e4."} {"question": "\ud669\uad50\uc548\uc740 \uba87 \ub300 \ubc95\ubb34\ubd80 \uc7a5\uad00\uc774 \ub418\uc5c8\ub294\uac00?", "paragraph": "1981\ub144 \uc0ac\ubc95\uc2dc\ud5d8\uc5d0 \ud569\uaca9\ud558\uc5ec 1982\ub144 12\uc6d4 \ucd98\ucc9c\uc9c0\ubc29\uac80\ucc30\uccad \uac80\uc0ac\uc2dc\ubcf4\ub85c \uc2dc\uc791, \uacf5\uc548\ud1b5, \ubbf8\uc2a4\ud130\uad6d\ubcf4\ubc95\uc73c\ub85c \ubd88\ub9ac\ub358 \uac80\uc0ac\ub85c \ud65c\ub3d9\ud588\ub2e4. 2011\ub144 \ubd80\uc0b0\uace0\ub4f1\uac80\ucc30\uccad \uac80\uc0ac\uc7a5\uc744 \uc9c0\ub0b8 \ub4a4, 2011\ub144 9\uc6d4 19\uc77c\ubd80\ud130 2013\ub144 1\uc6d4\uae4c\uc9c0 \ub300\ud55c\ubbfc\uad6d\uc758 \ubc95\ubb34\ubc95\uc778 \ud0dc\ud3c9\uc591 \uace0\ubb38 \ubcc0\ud638\uc0ac\ub85c \ud65c\ub3d9\ud588\ub2e4. 2013\ub144 \uc81c63\ub300 \ubc95\ubb34\ubd80 \uc7a5\uad00\uc774 \ub418\uc5c8\ub2e4. \ubc95\ubb34\ubd80 \uc7a5\uad00\uc73c\ub85c \uc7ac\uc9c1 \uc911 \uc774\uc11d\uae30 \uc0ac\uac74 \uc218\uc0ac\uc640 \ud1b5\ud569\uc9c4\ubcf4\ub2f9 \ud574\uc0b0 \uacfc\uc815\uc5d0\uc11c \uc8fc\ub3c4\uc801\uc778 \uc5ed\ud560\uc744 \ud558\uc600\ub2e4. \ubc95\uc758 \ubb38\ud131\uc744 \ub0ae\ucd94\uae30 \uc704\ud55c \uc77c\ud658\uc73c\ub85c \ubb34\ubcc0\ucd0c \uc9c0\uc5ed \uc8fc\ubbfc\ub4e4\uc744 \uc704\ud574 \ub9c8\uc744\ubcc0\ud638\uc0ac \uc81c\ub3c4\ub97c \uc2e0\uc124\ud558\uc600\ub294\ub370, \uc2dc\ud589 \ucd08\uae30\uc5d0\ub294 \uc2e4\ud6a8\uc131\uc774 \uc801\ub2e4\ub294 \ube44\ud310\uc744 \ubc1b\uc558\uc73c\ub098 \uc2dc\ud589 \uc790\uccb4\uc5d0 \ub300\ud558\uc5ec\ub294 \uc9c0\uc5ed \uc8fc\ubbfc\ub4e4\ub85c\ubd80\ud130 \uae0d\uc815\uc801\uc778 \ud3c9\uac00\ub97c \ubc1b\uc558\ub2e4. \uc774\ud6c4 \uacc4\uc18d \uc81c\ub3c4\ub97c \ubcf4\uc644\ud558\uc5ec \ub300\ud55c\ubbfc\uad6d\uc758 \uc804\uccb4 \ubb34\ubcc0\ucd0c 1,412\uacf3\uc5d0 \ub9c8\uc744\ubcc0\ud638\uc0ac 1,455\uba85\uc744 \ubc30\uce58\ud558\uc600\ub2e4.", "answer": "\uc81c63\ub300", "sentence": "2013\ub144 \uc81c63\ub300 \ubc95\ubb34\ubd80 \uc7a5\uad00\uc774 \ub418\uc5c8\ub2e4.", "paragraph_sentence": "1981\ub144 \uc0ac\ubc95\uc2dc\ud5d8\uc5d0 \ud569\uaca9\ud558\uc5ec 1982\ub144 12\uc6d4 \ucd98\ucc9c\uc9c0\ubc29\uac80\ucc30\uccad \uac80\uc0ac\uc2dc\ubcf4\ub85c \uc2dc\uc791, \uacf5\uc548\ud1b5, \ubbf8\uc2a4\ud130\uad6d\ubcf4\ubc95\uc73c\ub85c \ubd88\ub9ac\ub358 \uac80\uc0ac\ub85c \ud65c\ub3d9\ud588\ub2e4. 2011\ub144 \ubd80\uc0b0\uace0\ub4f1\uac80\ucc30\uccad \uac80\uc0ac\uc7a5\uc744 \uc9c0\ub0b8 \ub4a4, 2011\ub144 9\uc6d4 19\uc77c\ubd80\ud130 2013\ub144 1\uc6d4\uae4c\uc9c0 \ub300\ud55c\ubbfc\uad6d\uc758 \ubc95\ubb34\ubc95\uc778 \ud0dc\ud3c9\uc591 \uace0\ubb38 \ubcc0\ud638\uc0ac\ub85c \ud65c\ub3d9\ud588\ub2e4. 2013\ub144 \uc81c63\ub300 \ubc95\ubb34\ubd80 \uc7a5\uad00\uc774 \ub418\uc5c8\ub2e4. \ubc95\ubb34\ubd80 \uc7a5\uad00\uc73c\ub85c \uc7ac\uc9c1 \uc911 \uc774\uc11d\uae30 \uc0ac\uac74 \uc218\uc0ac\uc640 \ud1b5\ud569\uc9c4\ubcf4\ub2f9 \ud574\uc0b0 \uacfc\uc815\uc5d0\uc11c \uc8fc\ub3c4\uc801\uc778 \uc5ed\ud560\uc744 \ud558\uc600\ub2e4. \ubc95\uc758 \ubb38\ud131\uc744 \ub0ae\ucd94\uae30 \uc704\ud55c \uc77c\ud658\uc73c\ub85c \ubb34\ubcc0\ucd0c \uc9c0\uc5ed \uc8fc\ubbfc\ub4e4\uc744 \uc704\ud574 \ub9c8\uc744\ubcc0\ud638\uc0ac \uc81c\ub3c4\ub97c \uc2e0\uc124\ud558\uc600\ub294\ub370, \uc2dc\ud589 \ucd08\uae30\uc5d0\ub294 \uc2e4\ud6a8\uc131\uc774 \uc801\ub2e4\ub294 \ube44\ud310\uc744 \ubc1b\uc558\uc73c\ub098 \uc2dc\ud589 \uc790\uccb4\uc5d0 \ub300\ud558\uc5ec\ub294 \uc9c0\uc5ed \uc8fc\ubbfc\ub4e4\ub85c\ubd80\ud130 \uae0d\uc815\uc801\uc778 \ud3c9\uac00\ub97c \ubc1b\uc558\ub2e4. \uc774\ud6c4 \uacc4\uc18d \uc81c\ub3c4\ub97c \ubcf4\uc644\ud558\uc5ec \ub300\ud55c\ubbfc\uad6d\uc758 \uc804\uccb4 \ubb34\ubcc0\ucd0c 1,412\uacf3\uc5d0 \ub9c8\uc744\ubcc0\ud638\uc0ac 1,455\uba85\uc744 \ubc30\uce58\ud558\uc600\ub2e4.", "paragraph_answer": "1981\ub144 \uc0ac\ubc95\uc2dc\ud5d8\uc5d0 \ud569\uaca9\ud558\uc5ec 1982\ub144 12\uc6d4 \ucd98\ucc9c\uc9c0\ubc29\uac80\ucc30\uccad \uac80\uc0ac\uc2dc\ubcf4\ub85c \uc2dc\uc791, \uacf5\uc548\ud1b5, \ubbf8\uc2a4\ud130\uad6d\ubcf4\ubc95\uc73c\ub85c \ubd88\ub9ac\ub358 \uac80\uc0ac\ub85c \ud65c\ub3d9\ud588\ub2e4. 2011\ub144 \ubd80\uc0b0\uace0\ub4f1\uac80\ucc30\uccad \uac80\uc0ac\uc7a5\uc744 \uc9c0\ub0b8 \ub4a4, 2011\ub144 9\uc6d4 19\uc77c\ubd80\ud130 2013\ub144 1\uc6d4\uae4c\uc9c0 \ub300\ud55c\ubbfc\uad6d\uc758 \ubc95\ubb34\ubc95\uc778 \ud0dc\ud3c9\uc591 \uace0\ubb38 \ubcc0\ud638\uc0ac\ub85c \ud65c\ub3d9\ud588\ub2e4. 2013\ub144 \uc81c63\ub300 \ubc95\ubb34\ubd80 \uc7a5\uad00\uc774 \ub418\uc5c8\ub2e4. \ubc95\ubb34\ubd80 \uc7a5\uad00\uc73c\ub85c \uc7ac\uc9c1 \uc911 \uc774\uc11d\uae30 \uc0ac\uac74 \uc218\uc0ac\uc640 \ud1b5\ud569\uc9c4\ubcf4\ub2f9 \ud574\uc0b0 \uacfc\uc815\uc5d0\uc11c \uc8fc\ub3c4\uc801\uc778 \uc5ed\ud560\uc744 \ud558\uc600\ub2e4. \ubc95\uc758 \ubb38\ud131\uc744 \ub0ae\ucd94\uae30 \uc704\ud55c \uc77c\ud658\uc73c\ub85c \ubb34\ubcc0\ucd0c \uc9c0\uc5ed \uc8fc\ubbfc\ub4e4\uc744 \uc704\ud574 \ub9c8\uc744\ubcc0\ud638\uc0ac \uc81c\ub3c4\ub97c \uc2e0\uc124\ud558\uc600\ub294\ub370, \uc2dc\ud589 \ucd08\uae30\uc5d0\ub294 \uc2e4\ud6a8\uc131\uc774 \uc801\ub2e4\ub294 \ube44\ud310\uc744 \ubc1b\uc558\uc73c\ub098 \uc2dc\ud589 \uc790\uccb4\uc5d0 \ub300\ud558\uc5ec\ub294 \uc9c0\uc5ed \uc8fc\ubbfc\ub4e4\ub85c\ubd80\ud130 \uae0d\uc815\uc801\uc778 \ud3c9\uac00\ub97c \ubc1b\uc558\ub2e4. \uc774\ud6c4 \uacc4\uc18d \uc81c\ub3c4\ub97c \ubcf4\uc644\ud558\uc5ec \ub300\ud55c\ubbfc\uad6d\uc758 \uc804\uccb4 \ubb34\ubcc0\ucd0c 1,412\uacf3\uc5d0 \ub9c8\uc744\ubcc0\ud638\uc0ac 1,455\uba85\uc744 \ubc30\uce58\ud558\uc600\ub2e4.", "sentence_answer": "2013\ub144 \uc81c63\ub300 \ubc95\ubb34\ubd80 \uc7a5\uad00\uc774 \ub418\uc5c8\ub2e4.", "paragraph_id": "6366843-0-1", "paragraph_question": "question: \ud669\uad50\uc548\uc740 \uba87 \ub300 \ubc95\ubb34\ubd80 \uc7a5\uad00\uc774 \ub418\uc5c8\ub294\uac00?, context: 1981\ub144 \uc0ac\ubc95\uc2dc\ud5d8\uc5d0 \ud569\uaca9\ud558\uc5ec 1982\ub144 12\uc6d4 \ucd98\ucc9c\uc9c0\ubc29\uac80\ucc30\uccad \uac80\uc0ac\uc2dc\ubcf4\ub85c \uc2dc\uc791, \uacf5\uc548\ud1b5, \ubbf8\uc2a4\ud130\uad6d\ubcf4\ubc95\uc73c\ub85c \ubd88\ub9ac\ub358 \uac80\uc0ac\ub85c \ud65c\ub3d9\ud588\ub2e4. 2011\ub144 \ubd80\uc0b0\uace0\ub4f1\uac80\ucc30\uccad \uac80\uc0ac\uc7a5\uc744 \uc9c0\ub0b8 \ub4a4, 2011\ub144 9\uc6d4 19\uc77c\ubd80\ud130 2013\ub144 1\uc6d4\uae4c\uc9c0 \ub300\ud55c\ubbfc\uad6d\uc758 \ubc95\ubb34\ubc95\uc778 \ud0dc\ud3c9\uc591 \uace0\ubb38 \ubcc0\ud638\uc0ac\ub85c \ud65c\ub3d9\ud588\ub2e4. 2013\ub144 \uc81c63\ub300 \ubc95\ubb34\ubd80 \uc7a5\uad00\uc774 \ub418\uc5c8\ub2e4. \ubc95\ubb34\ubd80 \uc7a5\uad00\uc73c\ub85c \uc7ac\uc9c1 \uc911 \uc774\uc11d\uae30 \uc0ac\uac74 \uc218\uc0ac\uc640 \ud1b5\ud569\uc9c4\ubcf4\ub2f9 \ud574\uc0b0 \uacfc\uc815\uc5d0\uc11c \uc8fc\ub3c4\uc801\uc778 \uc5ed\ud560\uc744 \ud558\uc600\ub2e4. \ubc95\uc758 \ubb38\ud131\uc744 \ub0ae\ucd94\uae30 \uc704\ud55c \uc77c\ud658\uc73c\ub85c \ubb34\ubcc0\ucd0c \uc9c0\uc5ed \uc8fc\ubbfc\ub4e4\uc744 \uc704\ud574 \ub9c8\uc744\ubcc0\ud638\uc0ac \uc81c\ub3c4\ub97c \uc2e0\uc124\ud558\uc600\ub294\ub370, \uc2dc\ud589 \ucd08\uae30\uc5d0\ub294 \uc2e4\ud6a8\uc131\uc774 \uc801\ub2e4\ub294 \ube44\ud310\uc744 \ubc1b\uc558\uc73c\ub098 \uc2dc\ud589 \uc790\uccb4\uc5d0 \ub300\ud558\uc5ec\ub294 \uc9c0\uc5ed \uc8fc\ubbfc\ub4e4\ub85c\ubd80\ud130 \uae0d\uc815\uc801\uc778 \ud3c9\uac00\ub97c \ubc1b\uc558\ub2e4. \uc774\ud6c4 \uacc4\uc18d \uc81c\ub3c4\ub97c \ubcf4\uc644\ud558\uc5ec \ub300\ud55c\ubbfc\uad6d\uc758 \uc804\uccb4 \ubb34\ubcc0\ucd0c 1,412\uacf3\uc5d0 \ub9c8\uc744\ubcc0\ud638\uc0ac 1,455\uba85\uc744 \ubc30\uce58\ud558\uc600\ub2e4."} {"question": "\uc544\uc2a4\ud2b8\ub77c\ud55c\uc758 \ub291\ub300\uac00 \uce74\uc2a4\ud53c\ubb3c\ubc94\uc744 \uc0ac\ub0e5\ud55c \uacf3\uc740 \uc5b4\ub514\uc778\uac00?", "paragraph": "\ub291\ub300\ub294 \uc8fc\ub85c \ub300\ud615 \uc720\uc81c\ub958\ub97c \uba39\uc9c0\ub9cc \ub2e4\ub978 \uba39\uc774\ub3c4 \uac00\ub9ac\uc9c0 \uc54a\ub294\ub2e4. \ub291\ub300\uac00 \ubcf4\uc644\ud558\uae30 \uc704\ud574 \uba39\ub294 \uc18c\ud615 \uba39\uc774\ub85c\ub294 \ub9c8\uba4b, \ud1a0\ub07c, \uc624\uc18c\ub9ac, \uc5ec\uc6b0, \uc871\uc81c\ube44, \uc5bc\ub8e9\ub2e4\ub78c\uc950, \uc0dd\uc950, \ud584\uc2a4\ud130, \ub4e4\uc950 \ub4f1 \uc124\uce58\ub958 \ubc0f \uae30\ud0c0 \uc2dd\ucda9\ubaa9\uc774 \uc788\ub2e4. \ub610\ud55c \ubb3c\uc0c8\uc758 \uc54c\ub3c4 \uc790\uc8fc \uba39\ub294\ub2e4. \uc774\ub7ec\ud55c \uba39\uc774\uac00 \ubd80\uc871\ud55c \uacbd\uc6b0, \ub291\ub300\ub294 \ub3c4\ub9c8\ubc40, \ubc40, \uac1c\uad6c\ub9ac, \ub4dc\ubb3c\uac8c\ub294 \ub450\uaebc\ube44 \uc640 \ub300\ud615 \uace4\ucda9\ub3c4 \ub610\ud55c \uba39\uc774\ub85c \uc0bc\ub294\ub2e4. \uba39\uc774\uac00 \ubd80\uc871\ud55c \uc2dc\uae30\uc5d0\ub294 \uc18c \ub3c4\ucd95\uc7a5 \ub4f1\uc5d0\uc11c \ub098\uc628 \ubd80\uc721\uc744 \uba39\uae30\ub3c4 \ud55c\ub2e4. \uac70\uce5c \uaca8\uc6b8 \ub3d9\uc548 \uc885\uc885 \ubb34\ub9ac\uc5d0\uc11c \uc57d\ud558\uac70\ub098 \ubd80\uc0c1\ub2f9\ud55c \ub291\ub300\ub97c \uacf5\uaca9\ud558\uac70\ub098 \uc8fd\uc740 \ubb34\ub9ac\uc758 \ub291\ub300\ub97c \uba39\uae30\ub3c4 \ud558\ub294 \uac83 \ub4f1 \ub3d9\uc885\ud3ec\uc2dd\ub3c4 \uc885\uc885 \uc788\ub294 \uc77c\uc774\ub2e4. \uc544\uc2a4\ud2b8\ub77c\ud55c\uc758 \ub291\ub300 \ubb34\ub9ac\ub294 \uce74\uc2a4\ud53c \ud574\uc5d0\uc11c \uce74\uc2a4\ud53c\ubb3c\ubc94\uc744 \uc0ac\ub0e5\ud55c \uc801\ub3c4 \uc788\uc73c\uba70 \uc54c\ub798\uc2a4\uce74\uc640 \uc11c\ubd80 \uce90\ub098\ub2e4\uc758 \ub291\ub300 \ubb34\ub9ac\ub4e4\uc740 \uc5f0\uc5b4\ub97c \uc0ac\ub0e5\ud55c \uc801\ub3c4 \uc788\ub2e4. \uc778\uac04\uc740 \uac70\uc758 \uba39\uc774\ub85c \ub178\ub9ac\uc9c0 \uc54a\uc9c0\ub9cc \uac00\ub054 \uacf5\uaca9\ud558\ub294 \uc0ac\ub840\uac00 \uc788\ub2e4.", "answer": "\uce74\uc2a4\ud53c \ud574", "sentence": "\uc544\uc2a4\ud2b8\ub77c\ud55c\uc758 \ub291\ub300 \ubb34\ub9ac\ub294 \uce74\uc2a4\ud53c \ud574 \uc5d0\uc11c \uce74\uc2a4\ud53c\ubb3c\ubc94\uc744 \uc0ac\ub0e5\ud55c \uc801\ub3c4 \uc788\uc73c\uba70 \uc54c\ub798\uc2a4\uce74\uc640 \uc11c\ubd80 \uce90\ub098\ub2e4\uc758 \ub291\ub300 \ubb34\ub9ac\ub4e4\uc740 \uc5f0\uc5b4\ub97c \uc0ac\ub0e5\ud55c \uc801\ub3c4 \uc788\ub2e4.", "paragraph_sentence": "\ub291\ub300\ub294 \uc8fc\ub85c \ub300\ud615 \uc720\uc81c\ub958\ub97c \uba39\uc9c0\ub9cc \ub2e4\ub978 \uba39\uc774\ub3c4 \uac00\ub9ac\uc9c0 \uc54a\ub294\ub2e4. \ub291\ub300\uac00 \ubcf4\uc644\ud558\uae30 \uc704\ud574 \uba39\ub294 \uc18c\ud615 \uba39\uc774\ub85c\ub294 \ub9c8\uba4b, \ud1a0\ub07c, \uc624\uc18c\ub9ac, \uc5ec\uc6b0, \uc871\uc81c\ube44, \uc5bc\ub8e9\ub2e4\ub78c\uc950, \uc0dd\uc950, \ud584\uc2a4\ud130, \ub4e4\uc950 \ub4f1 \uc124\uce58\ub958 \ubc0f \uae30\ud0c0 \uc2dd\ucda9\ubaa9\uc774 \uc788\ub2e4. \ub610\ud55c \ubb3c\uc0c8\uc758 \uc54c\ub3c4 \uc790\uc8fc \uba39\ub294\ub2e4. \uc774\ub7ec\ud55c \uba39\uc774\uac00 \ubd80\uc871\ud55c \uacbd\uc6b0, \ub291\ub300\ub294 \ub3c4\ub9c8\ubc40, \ubc40, \uac1c\uad6c\ub9ac, \ub4dc\ubb3c\uac8c\ub294 \ub450\uaebc\ube44 \uc640 \ub300\ud615 \uace4\ucda9\ub3c4 \ub610\ud55c \uba39\uc774\ub85c \uc0bc\ub294\ub2e4. \uba39\uc774\uac00 \ubd80\uc871\ud55c \uc2dc\uae30\uc5d0\ub294 \uc18c \ub3c4\ucd95\uc7a5 \ub4f1\uc5d0\uc11c \ub098\uc628 \ubd80\uc721\uc744 \uba39\uae30\ub3c4 \ud55c\ub2e4. \uac70\uce5c \uaca8\uc6b8 \ub3d9\uc548 \uc885\uc885 \ubb34\ub9ac\uc5d0\uc11c \uc57d\ud558\uac70\ub098 \ubd80\uc0c1\ub2f9\ud55c \ub291\ub300\ub97c \uacf5\uaca9\ud558\uac70\ub098 \uc8fd\uc740 \ubb34\ub9ac\uc758 \ub291\ub300\ub97c \uba39\uae30\ub3c4 \ud558\ub294 \uac83 \ub4f1 \ub3d9\uc885\ud3ec\uc2dd\ub3c4 \uc885\uc885 \uc788\ub294 \uc77c\uc774\ub2e4. \uc544\uc2a4\ud2b8\ub77c\ud55c\uc758 \ub291\ub300 \ubb34\ub9ac\ub294 \uce74\uc2a4\ud53c \ud574 \uc5d0\uc11c \uce74\uc2a4\ud53c\ubb3c\ubc94\uc744 \uc0ac\ub0e5\ud55c \uc801\ub3c4 \uc788\uc73c\uba70 \uc54c\ub798\uc2a4\uce74\uc640 \uc11c\ubd80 \uce90\ub098\ub2e4\uc758 \ub291\ub300 \ubb34\ub9ac\ub4e4\uc740 \uc5f0\uc5b4\ub97c \uc0ac\ub0e5\ud55c \uc801\ub3c4 \uc788\ub2e4. \uc778\uac04\uc740 \uac70\uc758 \uba39\uc774\ub85c \ub178\ub9ac\uc9c0 \uc54a\uc9c0\ub9cc \uac00\ub054 \uacf5\uaca9\ud558\ub294 \uc0ac\ub840\uac00 \uc788\ub2e4.", "paragraph_answer": "\ub291\ub300\ub294 \uc8fc\ub85c \ub300\ud615 \uc720\uc81c\ub958\ub97c \uba39\uc9c0\ub9cc \ub2e4\ub978 \uba39\uc774\ub3c4 \uac00\ub9ac\uc9c0 \uc54a\ub294\ub2e4. \ub291\ub300\uac00 \ubcf4\uc644\ud558\uae30 \uc704\ud574 \uba39\ub294 \uc18c\ud615 \uba39\uc774\ub85c\ub294 \ub9c8\uba4b, \ud1a0\ub07c, \uc624\uc18c\ub9ac, \uc5ec\uc6b0, \uc871\uc81c\ube44, \uc5bc\ub8e9\ub2e4\ub78c\uc950, \uc0dd\uc950, \ud584\uc2a4\ud130, \ub4e4\uc950 \ub4f1 \uc124\uce58\ub958 \ubc0f \uae30\ud0c0 \uc2dd\ucda9\ubaa9\uc774 \uc788\ub2e4. \ub610\ud55c \ubb3c\uc0c8\uc758 \uc54c\ub3c4 \uc790\uc8fc \uba39\ub294\ub2e4. \uc774\ub7ec\ud55c \uba39\uc774\uac00 \ubd80\uc871\ud55c \uacbd\uc6b0, \ub291\ub300\ub294 \ub3c4\ub9c8\ubc40, \ubc40, \uac1c\uad6c\ub9ac, \ub4dc\ubb3c\uac8c\ub294 \ub450\uaebc\ube44 \uc640 \ub300\ud615 \uace4\ucda9\ub3c4 \ub610\ud55c \uba39\uc774\ub85c \uc0bc\ub294\ub2e4. \uba39\uc774\uac00 \ubd80\uc871\ud55c \uc2dc\uae30\uc5d0\ub294 \uc18c \ub3c4\ucd95\uc7a5 \ub4f1\uc5d0\uc11c \ub098\uc628 \ubd80\uc721\uc744 \uba39\uae30\ub3c4 \ud55c\ub2e4. \uac70\uce5c \uaca8\uc6b8 \ub3d9\uc548 \uc885\uc885 \ubb34\ub9ac\uc5d0\uc11c \uc57d\ud558\uac70\ub098 \ubd80\uc0c1\ub2f9\ud55c \ub291\ub300\ub97c \uacf5\uaca9\ud558\uac70\ub098 \uc8fd\uc740 \ubb34\ub9ac\uc758 \ub291\ub300\ub97c \uba39\uae30\ub3c4 \ud558\ub294 \uac83 \ub4f1 \ub3d9\uc885\ud3ec\uc2dd\ub3c4 \uc885\uc885 \uc788\ub294 \uc77c\uc774\ub2e4. \uc544\uc2a4\ud2b8\ub77c\ud55c\uc758 \ub291\ub300 \ubb34\ub9ac\ub294 \uce74\uc2a4\ud53c \ud574 \uc5d0\uc11c \uce74\uc2a4\ud53c\ubb3c\ubc94\uc744 \uc0ac\ub0e5\ud55c \uc801\ub3c4 \uc788\uc73c\uba70 \uc54c\ub798\uc2a4\uce74\uc640 \uc11c\ubd80 \uce90\ub098\ub2e4\uc758 \ub291\ub300 \ubb34\ub9ac\ub4e4\uc740 \uc5f0\uc5b4\ub97c \uc0ac\ub0e5\ud55c \uc801\ub3c4 \uc788\ub2e4. \uc778\uac04\uc740 \uac70\uc758 \uba39\uc774\ub85c \ub178\ub9ac\uc9c0 \uc54a\uc9c0\ub9cc \uac00\ub054 \uacf5\uaca9\ud558\ub294 \uc0ac\ub840\uac00 \uc788\ub2e4.", "sentence_answer": "\uc544\uc2a4\ud2b8\ub77c\ud55c\uc758 \ub291\ub300 \ubb34\ub9ac\ub294 \uce74\uc2a4\ud53c \ud574 \uc5d0\uc11c \uce74\uc2a4\ud53c\ubb3c\ubc94\uc744 \uc0ac\ub0e5\ud55c \uc801\ub3c4 \uc788\uc73c\uba70 \uc54c\ub798\uc2a4\uce74\uc640 \uc11c\ubd80 \uce90\ub098\ub2e4\uc758 \ub291\ub300 \ubb34\ub9ac\ub4e4\uc740 \uc5f0\uc5b4\ub97c \uc0ac\ub0e5\ud55c \uc801\ub3c4 \uc788\ub2e4.", "paragraph_id": "6572454-28-0", "paragraph_question": "question: \uc544\uc2a4\ud2b8\ub77c\ud55c\uc758 \ub291\ub300\uac00 \uce74\uc2a4\ud53c\ubb3c\ubc94\uc744 \uc0ac\ub0e5\ud55c \uacf3\uc740 \uc5b4\ub514\uc778\uac00?, context: \ub291\ub300\ub294 \uc8fc\ub85c \ub300\ud615 \uc720\uc81c\ub958\ub97c \uba39\uc9c0\ub9cc \ub2e4\ub978 \uba39\uc774\ub3c4 \uac00\ub9ac\uc9c0 \uc54a\ub294\ub2e4. \ub291\ub300\uac00 \ubcf4\uc644\ud558\uae30 \uc704\ud574 \uba39\ub294 \uc18c\ud615 \uba39\uc774\ub85c\ub294 \ub9c8\uba4b, \ud1a0\ub07c, \uc624\uc18c\ub9ac, \uc5ec\uc6b0, \uc871\uc81c\ube44, \uc5bc\ub8e9\ub2e4\ub78c\uc950, \uc0dd\uc950, \ud584\uc2a4\ud130, \ub4e4\uc950 \ub4f1 \uc124\uce58\ub958 \ubc0f \uae30\ud0c0 \uc2dd\ucda9\ubaa9\uc774 \uc788\ub2e4. \ub610\ud55c \ubb3c\uc0c8\uc758 \uc54c\ub3c4 \uc790\uc8fc \uba39\ub294\ub2e4. \uc774\ub7ec\ud55c \uba39\uc774\uac00 \ubd80\uc871\ud55c \uacbd\uc6b0, \ub291\ub300\ub294 \ub3c4\ub9c8\ubc40, \ubc40, \uac1c\uad6c\ub9ac, \ub4dc\ubb3c\uac8c\ub294 \ub450\uaebc\ube44 \uc640 \ub300\ud615 \uace4\ucda9\ub3c4 \ub610\ud55c \uba39\uc774\ub85c \uc0bc\ub294\ub2e4. \uba39\uc774\uac00 \ubd80\uc871\ud55c \uc2dc\uae30\uc5d0\ub294 \uc18c \ub3c4\ucd95\uc7a5 \ub4f1\uc5d0\uc11c \ub098\uc628 \ubd80\uc721\uc744 \uba39\uae30\ub3c4 \ud55c\ub2e4. \uac70\uce5c \uaca8\uc6b8 \ub3d9\uc548 \uc885\uc885 \ubb34\ub9ac\uc5d0\uc11c \uc57d\ud558\uac70\ub098 \ubd80\uc0c1\ub2f9\ud55c \ub291\ub300\ub97c \uacf5\uaca9\ud558\uac70\ub098 \uc8fd\uc740 \ubb34\ub9ac\uc758 \ub291\ub300\ub97c \uba39\uae30\ub3c4 \ud558\ub294 \uac83 \ub4f1 \ub3d9\uc885\ud3ec\uc2dd\ub3c4 \uc885\uc885 \uc788\ub294 \uc77c\uc774\ub2e4. \uc544\uc2a4\ud2b8\ub77c\ud55c\uc758 \ub291\ub300 \ubb34\ub9ac\ub294 \uce74\uc2a4\ud53c \ud574\uc5d0\uc11c \uce74\uc2a4\ud53c\ubb3c\ubc94\uc744 \uc0ac\ub0e5\ud55c \uc801\ub3c4 \uc788\uc73c\uba70 \uc54c\ub798\uc2a4\uce74\uc640 \uc11c\ubd80 \uce90\ub098\ub2e4\uc758 \ub291\ub300 \ubb34\ub9ac\ub4e4\uc740 \uc5f0\uc5b4\ub97c \uc0ac\ub0e5\ud55c \uc801\ub3c4 \uc788\ub2e4. \uc778\uac04\uc740 \uac70\uc758 \uba39\uc774\ub85c \ub178\ub9ac\uc9c0 \uc54a\uc9c0\ub9cc \uac00\ub054 \uacf5\uaca9\ud558\ub294 \uc0ac\ub840\uac00 \uc788\ub2e4."} {"question": "\ub0a8\ubd81 \uacf5\ub3d9 \uc131\uba85\uc744 \ubc1c\ud45c\ud55c \uac83\uc740 \uc5b8\uc81c\uc778\uac00?", "paragraph": "1971\ub144 \ubc15\uc815\ud76c\ub294 \ub300\ud55c\ubbfc\uad6d\uc758 \ub18d\uc5c5\uc744 \ubcf4\ub2e4 \ud070 \uaddc\ubaa8\ub85c \ud655\ub300\ud558\uae30 \uc704\ud574 \uc544\ub974\ud5e8\ud2f0\ub098 \uc815\ubd80\ub85c\ubd80\ud130 \uc5ec\uc758\ub3c4\uc758 70\ubc30\ub098 \ub418\ub294 \uaddc\ubaa8\uc758 \ub545\uc744 \uad6c\ub9e4\ud558\uc600\ub294\ub370 \uc774 \ub545\uc5d0 \uc2e0\uc6d0 \uc870\uc0ac \ub4f1 \uac16\uac00\uc9c0 \uc2ec\uc0ac\ub97c \uac70\uccd0 \uc5c4\uc120\ud55c \ub18d\ubbfc\ub4e4\uc744 \ud30c\uacac\ud558\uc600\ub2e4. \uadf8\ub7ec\ub098 \ub300\ud55c\ubbfc\uad6d\uc5d0\uc11c\uc758 \uc5ec\ub984\uc774 \uc544\ub974\ud5e8\ud2f0\ub098\uc5d0\uc11c\ub294 \uaca8\uc6b8\uc778 \uac83\ubd80\ud130 \uc2dc\uc791\ud574\uc11c \ub300\ud55c\ubbfc\uad6d\uacfc \uc544\ub974\ud5e8\ud2f0\ub098\uc758 \uae30\ud6c4\ub294 \uc804\ud600 \ub9de\uc9c0 \uc54a\ub294 \ub370\ub2e4\uac00 \ubc15\uc815\ud76c\uac00 \uad6c\ub9e4\ud55c \ub545 \uc911\uc5d0\ub294 \uc18c\uae08\uae30\uac00 \ub9ce\uc740 \ub545\uc774 \uc788\ub294\uac00 \ud558\uba74 \uc5ec\ub7ec \uc885\ub958\uc758 \ud669\ubb34\uc9c0\uac00 \ub9ce\uc558\ub2e4. \uacb0\uad6d \ubc15\uc815\ud76c\uc758 \uc774 \ud504\ub85c\uc81d\ud2b8\ub294 \uc0ac\uc2e4\uc0c1 \uc2e4\ud328\ub85c \ub3cc\uc544\uac14\uc73c\uba70 \ubc15\uc815\ud76c\uac00 \uad6c\uc785\ud55c \ub545\uc740 \uc544\uc9c1\ub3c4 \ub300\ud55c\ubbfc\uad6d \uc815\ubd80\uac00 \uc18c\uc720\ud558\uc9c0\ub9cc \ud574\ub9c8\ub2e4 \uad00\ub9ac\ube44 \uba85\ubaa9\uc73c\ub85c \ub300\ud55c\ubbfc\uad6d \uc815\ubd80\uac00 \uc544\ub974\ud5e8\ud2f0\ub098 \uc815\ubd80\uc5d0 \uc138\uae08\uc744 \ub0b4\ub294 \ud615\uad6d\uc774 \ub418\uac70\ub098 \uc544\ub974\ud5e8\ud2f0\ub098 \uc815\ubd80\uc5d0 \ubc18\ud658\ud558\uac70\ub098 \ud604\uc9c0 \ub18d\ubbfc\ub4e4\uc5d0\uac8c \uc18c\uc720\uad8c\uc744 \uc774\uc804\ud588\ub2e4. 1972\ub144 \uc81c3\ucc28 \uacbd\uc81c\uac1c\ubc1c 5\uac1c\ub144\uacc4\ud68d\uc744 \uc2e4\uc2dc\ud558\uc600\uace0 1\uc6d4 27\uc77c \uc81c3\ucc28 \uc778\ub825\uac1c\ubc1c 5\uac1c\ub144 \uacc4\ud68d\uc744 \ud655\uc815\ud558\uc600\ub2e4. 2\uc6d4 9\uc77c\uc5d0\ub294 \ub179\uc0c9\ud601\uba85\uc744 \ucd94\uc9c4, \ud1b5\uc77c\ubcbc\ub97c \uac1c\ubc1c\ud558\uc600\uc73c\uba70 \uc300\uc758 \ud55c\uad6d \uc790\uccb4\uc0dd\uc0b0 \ubc0f \uc644\uc804 \uc790\uae09\uc790\uc871\uc740 1976\ub144\uc5d0 \ub2ec\uc131\ud55c\ub2e4. 1972\ub144 7\uc6d4 4\uc77c \ubd84\ub2e8 \uc774\ud6c4\ub85c \ucd5c\ucd08\ub85c 7\u00b74 \ub0a8\ubd81 \uacf5\ub3d9 \uc131\uba85\uc744 \ubc1c\ud45c\ud558\uc600\ub2e4. 8\uc6d4 3\uc77c \uae30\uc5c5\uc0ac\ucc44 \ub3d9\uacb0 \ub4f1 \uae34\uae09 \uba85\ub839\uc744 \ubc1c\ud45c\ud558\uc600\ub2e4.", "answer": "1972\ub144 7\uc6d4 4\uc77c", "sentence": "1972\ub144 7\uc6d4 4\uc77c \ubd84\ub2e8 \uc774\ud6c4\ub85c \ucd5c\ucd08\ub85c 7\u00b74 \ub0a8\ubd81 \uacf5\ub3d9 \uc131\uba85\uc744 \ubc1c\ud45c\ud558\uc600\ub2e4.", "paragraph_sentence": "1971\ub144 \ubc15\uc815\ud76c\ub294 \ub300\ud55c\ubbfc\uad6d\uc758 \ub18d\uc5c5\uc744 \ubcf4\ub2e4 \ud070 \uaddc\ubaa8\ub85c \ud655\ub300\ud558\uae30 \uc704\ud574 \uc544\ub974\ud5e8\ud2f0\ub098 \uc815\ubd80\ub85c\ubd80\ud130 \uc5ec\uc758\ub3c4\uc758 70\ubc30\ub098 \ub418\ub294 \uaddc\ubaa8\uc758 \ub545\uc744 \uad6c\ub9e4\ud558\uc600\ub294\ub370 \uc774 \ub545\uc5d0 \uc2e0\uc6d0 \uc870\uc0ac \ub4f1 \uac16\uac00\uc9c0 \uc2ec\uc0ac\ub97c \uac70\uccd0 \uc5c4\uc120\ud55c \ub18d\ubbfc\ub4e4\uc744 \ud30c\uacac\ud558\uc600\ub2e4. \uadf8\ub7ec\ub098 \ub300\ud55c\ubbfc\uad6d\uc5d0\uc11c\uc758 \uc5ec\ub984\uc774 \uc544\ub974\ud5e8\ud2f0\ub098\uc5d0\uc11c\ub294 \uaca8\uc6b8\uc778 \uac83\ubd80\ud130 \uc2dc\uc791\ud574\uc11c \ub300\ud55c\ubbfc\uad6d\uacfc \uc544\ub974\ud5e8\ud2f0\ub098\uc758 \uae30\ud6c4\ub294 \uc804\ud600 \ub9de\uc9c0 \uc54a\ub294 \ub370\ub2e4\uac00 \ubc15\uc815\ud76c\uac00 \uad6c\ub9e4\ud55c \ub545 \uc911\uc5d0\ub294 \uc18c\uae08\uae30\uac00 \ub9ce\uc740 \ub545\uc774 \uc788\ub294\uac00 \ud558\uba74 \uc5ec\ub7ec \uc885\ub958\uc758 \ud669\ubb34\uc9c0\uac00 \ub9ce\uc558\ub2e4. \uacb0\uad6d \ubc15\uc815\ud76c\uc758 \uc774 \ud504\ub85c\uc81d\ud2b8\ub294 \uc0ac\uc2e4\uc0c1 \uc2e4\ud328\ub85c \ub3cc\uc544\uac14\uc73c\uba70 \ubc15\uc815\ud76c\uac00 \uad6c\uc785\ud55c \ub545\uc740 \uc544\uc9c1\ub3c4 \ub300\ud55c\ubbfc\uad6d \uc815\ubd80\uac00 \uc18c\uc720\ud558\uc9c0\ub9cc \ud574\ub9c8\ub2e4 \uad00\ub9ac\ube44 \uba85\ubaa9\uc73c\ub85c \ub300\ud55c\ubbfc\uad6d \uc815\ubd80\uac00 \uc544\ub974\ud5e8\ud2f0\ub098 \uc815\ubd80\uc5d0 \uc138\uae08\uc744 \ub0b4\ub294 \ud615\uad6d\uc774 \ub418\uac70\ub098 \uc544\ub974\ud5e8\ud2f0\ub098 \uc815\ubd80\uc5d0 \ubc18\ud658\ud558\uac70\ub098 \ud604\uc9c0 \ub18d\ubbfc\ub4e4\uc5d0\uac8c \uc18c\uc720\uad8c\uc744 \uc774\uc804\ud588\ub2e4. 1972\ub144 \uc81c3\ucc28 \uacbd\uc81c\uac1c\ubc1c 5\uac1c\ub144\uacc4\ud68d\uc744 \uc2e4\uc2dc\ud558\uc600\uace0 1\uc6d4 27\uc77c \uc81c3\ucc28 \uc778\ub825\uac1c\ubc1c 5\uac1c\ub144 \uacc4\ud68d\uc744 \ud655\uc815\ud558\uc600\ub2e4. 2\uc6d4 9\uc77c\uc5d0\ub294 \ub179\uc0c9\ud601\uba85\uc744 \ucd94\uc9c4, \ud1b5\uc77c\ubcbc\ub97c \uac1c\ubc1c\ud558\uc600\uc73c\uba70 \uc300\uc758 \ud55c\uad6d \uc790\uccb4\uc0dd\uc0b0 \ubc0f \uc644\uc804 \uc790\uae09\uc790\uc871\uc740 1976\ub144\uc5d0 \ub2ec\uc131\ud55c\ub2e4. 1972\ub144 7\uc6d4 4\uc77c \ubd84\ub2e8 \uc774\ud6c4\ub85c \ucd5c\ucd08\ub85c 7\u00b74 \ub0a8\ubd81 \uacf5\ub3d9 \uc131\uba85\uc744 \ubc1c\ud45c\ud558\uc600\ub2e4. 8\uc6d4 3\uc77c \uae30\uc5c5\uc0ac\ucc44 \ub3d9\uacb0 \ub4f1 \uae34\uae09 \uba85\ub839\uc744 \ubc1c\ud45c\ud558\uc600\ub2e4.", "paragraph_answer": "1971\ub144 \ubc15\uc815\ud76c\ub294 \ub300\ud55c\ubbfc\uad6d\uc758 \ub18d\uc5c5\uc744 \ubcf4\ub2e4 \ud070 \uaddc\ubaa8\ub85c \ud655\ub300\ud558\uae30 \uc704\ud574 \uc544\ub974\ud5e8\ud2f0\ub098 \uc815\ubd80\ub85c\ubd80\ud130 \uc5ec\uc758\ub3c4\uc758 70\ubc30\ub098 \ub418\ub294 \uaddc\ubaa8\uc758 \ub545\uc744 \uad6c\ub9e4\ud558\uc600\ub294\ub370 \uc774 \ub545\uc5d0 \uc2e0\uc6d0 \uc870\uc0ac \ub4f1 \uac16\uac00\uc9c0 \uc2ec\uc0ac\ub97c \uac70\uccd0 \uc5c4\uc120\ud55c \ub18d\ubbfc\ub4e4\uc744 \ud30c\uacac\ud558\uc600\ub2e4. \uadf8\ub7ec\ub098 \ub300\ud55c\ubbfc\uad6d\uc5d0\uc11c\uc758 \uc5ec\ub984\uc774 \uc544\ub974\ud5e8\ud2f0\ub098\uc5d0\uc11c\ub294 \uaca8\uc6b8\uc778 \uac83\ubd80\ud130 \uc2dc\uc791\ud574\uc11c \ub300\ud55c\ubbfc\uad6d\uacfc \uc544\ub974\ud5e8\ud2f0\ub098\uc758 \uae30\ud6c4\ub294 \uc804\ud600 \ub9de\uc9c0 \uc54a\ub294 \ub370\ub2e4\uac00 \ubc15\uc815\ud76c\uac00 \uad6c\ub9e4\ud55c \ub545 \uc911\uc5d0\ub294 \uc18c\uae08\uae30\uac00 \ub9ce\uc740 \ub545\uc774 \uc788\ub294\uac00 \ud558\uba74 \uc5ec\ub7ec \uc885\ub958\uc758 \ud669\ubb34\uc9c0\uac00 \ub9ce\uc558\ub2e4. \uacb0\uad6d \ubc15\uc815\ud76c\uc758 \uc774 \ud504\ub85c\uc81d\ud2b8\ub294 \uc0ac\uc2e4\uc0c1 \uc2e4\ud328\ub85c \ub3cc\uc544\uac14\uc73c\uba70 \ubc15\uc815\ud76c\uac00 \uad6c\uc785\ud55c \ub545\uc740 \uc544\uc9c1\ub3c4 \ub300\ud55c\ubbfc\uad6d \uc815\ubd80\uac00 \uc18c\uc720\ud558\uc9c0\ub9cc \ud574\ub9c8\ub2e4 \uad00\ub9ac\ube44 \uba85\ubaa9\uc73c\ub85c \ub300\ud55c\ubbfc\uad6d \uc815\ubd80\uac00 \uc544\ub974\ud5e8\ud2f0\ub098 \uc815\ubd80\uc5d0 \uc138\uae08\uc744 \ub0b4\ub294 \ud615\uad6d\uc774 \ub418\uac70\ub098 \uc544\ub974\ud5e8\ud2f0\ub098 \uc815\ubd80\uc5d0 \ubc18\ud658\ud558\uac70\ub098 \ud604\uc9c0 \ub18d\ubbfc\ub4e4\uc5d0\uac8c \uc18c\uc720\uad8c\uc744 \uc774\uc804\ud588\ub2e4. 1972\ub144 \uc81c3\ucc28 \uacbd\uc81c\uac1c\ubc1c 5\uac1c\ub144\uacc4\ud68d\uc744 \uc2e4\uc2dc\ud558\uc600\uace0 1\uc6d4 27\uc77c \uc81c3\ucc28 \uc778\ub825\uac1c\ubc1c 5\uac1c\ub144 \uacc4\ud68d\uc744 \ud655\uc815\ud558\uc600\ub2e4. 2\uc6d4 9\uc77c\uc5d0\ub294 \ub179\uc0c9\ud601\uba85\uc744 \ucd94\uc9c4, \ud1b5\uc77c\ubcbc\ub97c \uac1c\ubc1c\ud558\uc600\uc73c\uba70 \uc300\uc758 \ud55c\uad6d \uc790\uccb4\uc0dd\uc0b0 \ubc0f \uc644\uc804 \uc790\uae09\uc790\uc871\uc740 1976\ub144\uc5d0 \ub2ec\uc131\ud55c\ub2e4. 1972\ub144 7\uc6d4 4\uc77c \ubd84\ub2e8 \uc774\ud6c4\ub85c \ucd5c\ucd08\ub85c 7\u00b74 \ub0a8\ubd81 \uacf5\ub3d9 \uc131\uba85\uc744 \ubc1c\ud45c\ud558\uc600\ub2e4. 8\uc6d4 3\uc77c \uae30\uc5c5\uc0ac\ucc44 \ub3d9\uacb0 \ub4f1 \uae34\uae09 \uba85\ub839\uc744 \ubc1c\ud45c\ud558\uc600\ub2e4.", "sentence_answer": " 1972\ub144 7\uc6d4 4\uc77c \ubd84\ub2e8 \uc774\ud6c4\ub85c \ucd5c\ucd08\ub85c 7\u00b74 \ub0a8\ubd81 \uacf5\ub3d9 \uc131\uba85\uc744 \ubc1c\ud45c\ud558\uc600\ub2e4.", "paragraph_id": "6530241-34-2", "paragraph_question": "question: \ub0a8\ubd81 \uacf5\ub3d9 \uc131\uba85\uc744 \ubc1c\ud45c\ud55c \uac83\uc740 \uc5b8\uc81c\uc778\uac00?, context: 1971\ub144 \ubc15\uc815\ud76c\ub294 \ub300\ud55c\ubbfc\uad6d\uc758 \ub18d\uc5c5\uc744 \ubcf4\ub2e4 \ud070 \uaddc\ubaa8\ub85c \ud655\ub300\ud558\uae30 \uc704\ud574 \uc544\ub974\ud5e8\ud2f0\ub098 \uc815\ubd80\ub85c\ubd80\ud130 \uc5ec\uc758\ub3c4\uc758 70\ubc30\ub098 \ub418\ub294 \uaddc\ubaa8\uc758 \ub545\uc744 \uad6c\ub9e4\ud558\uc600\ub294\ub370 \uc774 \ub545\uc5d0 \uc2e0\uc6d0 \uc870\uc0ac \ub4f1 \uac16\uac00\uc9c0 \uc2ec\uc0ac\ub97c \uac70\uccd0 \uc5c4\uc120\ud55c \ub18d\ubbfc\ub4e4\uc744 \ud30c\uacac\ud558\uc600\ub2e4. \uadf8\ub7ec\ub098 \ub300\ud55c\ubbfc\uad6d\uc5d0\uc11c\uc758 \uc5ec\ub984\uc774 \uc544\ub974\ud5e8\ud2f0\ub098\uc5d0\uc11c\ub294 \uaca8\uc6b8\uc778 \uac83\ubd80\ud130 \uc2dc\uc791\ud574\uc11c \ub300\ud55c\ubbfc\uad6d\uacfc \uc544\ub974\ud5e8\ud2f0\ub098\uc758 \uae30\ud6c4\ub294 \uc804\ud600 \ub9de\uc9c0 \uc54a\ub294 \ub370\ub2e4\uac00 \ubc15\uc815\ud76c\uac00 \uad6c\ub9e4\ud55c \ub545 \uc911\uc5d0\ub294 \uc18c\uae08\uae30\uac00 \ub9ce\uc740 \ub545\uc774 \uc788\ub294\uac00 \ud558\uba74 \uc5ec\ub7ec \uc885\ub958\uc758 \ud669\ubb34\uc9c0\uac00 \ub9ce\uc558\ub2e4. \uacb0\uad6d \ubc15\uc815\ud76c\uc758 \uc774 \ud504\ub85c\uc81d\ud2b8\ub294 \uc0ac\uc2e4\uc0c1 \uc2e4\ud328\ub85c \ub3cc\uc544\uac14\uc73c\uba70 \ubc15\uc815\ud76c\uac00 \uad6c\uc785\ud55c \ub545\uc740 \uc544\uc9c1\ub3c4 \ub300\ud55c\ubbfc\uad6d \uc815\ubd80\uac00 \uc18c\uc720\ud558\uc9c0\ub9cc \ud574\ub9c8\ub2e4 \uad00\ub9ac\ube44 \uba85\ubaa9\uc73c\ub85c \ub300\ud55c\ubbfc\uad6d \uc815\ubd80\uac00 \uc544\ub974\ud5e8\ud2f0\ub098 \uc815\ubd80\uc5d0 \uc138\uae08\uc744 \ub0b4\ub294 \ud615\uad6d\uc774 \ub418\uac70\ub098 \uc544\ub974\ud5e8\ud2f0\ub098 \uc815\ubd80\uc5d0 \ubc18\ud658\ud558\uac70\ub098 \ud604\uc9c0 \ub18d\ubbfc\ub4e4\uc5d0\uac8c \uc18c\uc720\uad8c\uc744 \uc774\uc804\ud588\ub2e4. 1972\ub144 \uc81c3\ucc28 \uacbd\uc81c\uac1c\ubc1c 5\uac1c\ub144\uacc4\ud68d\uc744 \uc2e4\uc2dc\ud558\uc600\uace0 1\uc6d4 27\uc77c \uc81c3\ucc28 \uc778\ub825\uac1c\ubc1c 5\uac1c\ub144 \uacc4\ud68d\uc744 \ud655\uc815\ud558\uc600\ub2e4. 2\uc6d4 9\uc77c\uc5d0\ub294 \ub179\uc0c9\ud601\uba85\uc744 \ucd94\uc9c4, \ud1b5\uc77c\ubcbc\ub97c \uac1c\ubc1c\ud558\uc600\uc73c\uba70 \uc300\uc758 \ud55c\uad6d \uc790\uccb4\uc0dd\uc0b0 \ubc0f \uc644\uc804 \uc790\uae09\uc790\uc871\uc740 1976\ub144\uc5d0 \ub2ec\uc131\ud55c\ub2e4. 1972\ub144 7\uc6d4 4\uc77c \ubd84\ub2e8 \uc774\ud6c4\ub85c \ucd5c\ucd08\ub85c 7\u00b74 \ub0a8\ubd81 \uacf5\ub3d9 \uc131\uba85\uc744 \ubc1c\ud45c\ud558\uc600\ub2e4. 8\uc6d4 3\uc77c \uae30\uc5c5\uc0ac\ucc44 \ub3d9\uacb0 \ub4f1 \uae34\uae09 \uba85\ub839\uc744 \ubc1c\ud45c\ud558\uc600\ub2e4."} {"question": "\uae30\uc6d0\uc804 1100\ub144\uacbd \uc791\uc131\ub41c \ubb38\uc11c\ub4e4\uc758 \ud544\uae30 \uc7ac\ub8cc\ub098 \uc2dd\ubb3c\uc744 \ubb34\uc5c7\uc774\ub77c \ud558\ub294\uac00?", "paragraph": "\uc544\uba54\ub178\ud398 \ub610\ub294 \uc544\uba54\ub124\ub9c8\ud504\ud2b8\uc758 \uc624\ub178\ub9c8\uc2a4\ud2f0\ucf58 \ubb38\uc11c\ub294 \ub78c\uc138\uc2a4 \uc655\uc870\uc758 \ud30c\ub77c\uc624\ub4e4\uc774 \ubc14\ub2e4 \ubbfc\uc871\uc744 \uac00\ub098\uc548\uc5d0 \uc815\ucc29\uc2dc\ucf30\ub2e4\ub294 \uc8fc\uc7a5\uc5d0 \uc5b4\ub290 \uc815\ub3c4 \uadfc\uac70\ub97c \uc81c\uacf5\ud55c\ub2e4. \uae30\uc6d0\uc804 1100\ub144\uacbd, \uc81c21\uc655\uc870 \ub9d0\uc5fd\uc5d0 \uc791\uc131\ub41c \uc774 \ubb38\uc11c\ub294 \ub2e8\uc21c\ud788 \uc774\ub984\ub9cc\uc744 \ub098\uc5f4\ud55c\ub2e4. \ud314\ub808\uc2a4\ud0c0\uc778 \uc18c\uc7ac\ud55c 4\uac1c \uc9c0\uba85\uc744 \ud3ec\ud568\ud558\uc5ec 6\uac1c\uc758 \uc9c0\uba85\uc744 \uc5f4\uac70\ud55c \ud6c4, \ubb38\uc11c\ub294 268\ud589\uc5d0\uc11c \uc170\ub374, 269\ud589\uc5d0\uc11c \ud2f0\uc608\ucee4, 270\ud589\uc5d0\uc11c \ud3a0\ub808\uc138\ud2b8\ub97c \uba85\uae30\ud558\uba70, \uc774\ub4e4\uc774 \uc55e\uc11c \uae30\uc7ac\ub41c \uc9c0\uc5ed\uc744 \uc810\uac70\ud588\ub2e4\uace0 \uc0dd\uac01\ud560 \uc218\ub3c4 \uc788\ub2e4. \uac19\uc740 \ube44\ubc00\ucc3d\uace0\uc5d0\uc11c \ubc1c\uacac\ub41c \ud30c\ud53c\ub8e8\uc2a4\uc5d0 \uc4f0\uc5ec\uc9c4 \uc6e8\ub098\ubb38 \uc774\uc57c\uae30\ub294 \ub2f9\uc2dc \ud2f0\uc608\ucee4\ub97c \ud56d\uad6c\ub3c4\uc2dc \ub3c4\ub974\uc5d0 \uc704\uce58\uc2dc\ucf30\ub2e4. \uc131\uc11c\uc5d0 \ub098\uc624\ub294 \ud574\uc591\ubbfc\uc871\uc778 \ub2e8\uc774 \ucd5c\ucd08\uc5d0 \ud544\ub9ac\uc2a4\ud2f4\uacfc \ud2f0\uc608\ucee4 \uc0ac\uc774\uc5d0 \uc704\uce58\ud588\ub2e4\ub294 \uc0ac\uc2e4\uc740, \uc774\ub4e4\uc774 \uc6d0\ub798 \ub374\uc60c\uc774\uc5c8\uc744 \uc218\ub3c4 \uc788\ub2e4\ub294 \uc8fc\uc7a5\uc774 \ub098\uc62c \uc218 \uc788\uac8c \ud55c\ub2e4. \uc170\ub374\uc740 \uba54\uae30\ub3c4\uc640 \uc694\ub974\ub2e8 \uacc4\uace1 \uadfc\ucc98\uc5d0 \uc815\ucc29\ud55c \uac83\uc73c\ub85c \ubcf4\uc774\uba70, \uc6e8\uc2dc\uc6e8\uc2dc(\uc131\uc11c\uc5d0\uc11c\ub294 \uc544\uc140)\ub294 \ub354 \ubd81\ucabd\uc5d0 \uc704\uce58\ud588\uc744 \uac83\uc774\ub2e4.", "answer": "\ud30c\ud53c\ub8e8\uc2a4", "sentence": "\uac19\uc740 \ube44\ubc00\ucc3d\uace0\uc5d0\uc11c \ubc1c\uacac\ub41c \ud30c\ud53c\ub8e8\uc2a4 \uc5d0 \uc4f0\uc5ec\uc9c4 \uc6e8\ub098\ubb38 \uc774\uc57c\uae30\ub294 \ub2f9\uc2dc \ud2f0\uc608\ucee4\ub97c \ud56d\uad6c\ub3c4\uc2dc \ub3c4\ub974\uc5d0 \uc704\uce58\uc2dc\ucf30\ub2e4.", "paragraph_sentence": "\uc544\uba54\ub178\ud398 \ub610\ub294 \uc544\uba54\ub124\ub9c8\ud504\ud2b8\uc758 \uc624\ub178\ub9c8\uc2a4\ud2f0\ucf58 \ubb38\uc11c\ub294 \ub78c\uc138\uc2a4 \uc655\uc870\uc758 \ud30c\ub77c\uc624\ub4e4\uc774 \ubc14\ub2e4 \ubbfc\uc871\uc744 \uac00\ub098\uc548\uc5d0 \uc815\ucc29\uc2dc\ucf30\ub2e4\ub294 \uc8fc\uc7a5\uc5d0 \uc5b4\ub290 \uc815\ub3c4 \uadfc\uac70\ub97c \uc81c\uacf5\ud55c\ub2e4. \uae30\uc6d0\uc804 1100\ub144\uacbd, \uc81c21\uc655\uc870 \ub9d0\uc5fd\uc5d0 \uc791\uc131\ub41c \uc774 \ubb38\uc11c\ub294 \ub2e8\uc21c\ud788 \uc774\ub984\ub9cc\uc744 \ub098\uc5f4\ud55c\ub2e4. \ud314\ub808\uc2a4\ud0c0\uc778 \uc18c\uc7ac\ud55c 4\uac1c \uc9c0\uba85\uc744 \ud3ec\ud568\ud558\uc5ec 6\uac1c\uc758 \uc9c0\uba85\uc744 \uc5f4\uac70\ud55c \ud6c4, \ubb38\uc11c\ub294 268\ud589\uc5d0\uc11c \uc170\ub374, 269\ud589\uc5d0\uc11c \ud2f0\uc608\ucee4, 270\ud589\uc5d0\uc11c \ud3a0\ub808\uc138\ud2b8\ub97c \uba85\uae30\ud558\uba70, \uc774\ub4e4\uc774 \uc55e\uc11c \uae30\uc7ac\ub41c \uc9c0\uc5ed\uc744 \uc810\uac70\ud588\ub2e4\uace0 \uc0dd\uac01\ud560 \uc218\ub3c4 \uc788\ub2e4. \uac19\uc740 \ube44\ubc00\ucc3d\uace0\uc5d0\uc11c \ubc1c\uacac\ub41c \ud30c\ud53c\ub8e8\uc2a4 \uc5d0 \uc4f0\uc5ec\uc9c4 \uc6e8\ub098\ubb38 \uc774\uc57c\uae30\ub294 \ub2f9\uc2dc \ud2f0\uc608\ucee4\ub97c \ud56d\uad6c\ub3c4\uc2dc \ub3c4\ub974\uc5d0 \uc704\uce58\uc2dc\ucf30\ub2e4. \uc131\uc11c\uc5d0 \ub098\uc624\ub294 \ud574\uc591\ubbfc\uc871\uc778 \ub2e8\uc774 \ucd5c\ucd08\uc5d0 \ud544\ub9ac\uc2a4\ud2f4\uacfc \ud2f0\uc608\ucee4 \uc0ac\uc774\uc5d0 \uc704\uce58\ud588\ub2e4\ub294 \uc0ac\uc2e4\uc740, \uc774\ub4e4\uc774 \uc6d0\ub798 \ub374\uc60c\uc774\uc5c8\uc744 \uc218\ub3c4 \uc788\ub2e4\ub294 \uc8fc\uc7a5\uc774 \ub098\uc62c \uc218 \uc788\uac8c \ud55c\ub2e4. \uc170\ub374\uc740 \uba54\uae30\ub3c4\uc640 \uc694\ub974\ub2e8 \uacc4\uace1 \uadfc\ucc98\uc5d0 \uc815\ucc29\ud55c \uac83\uc73c\ub85c \ubcf4\uc774\uba70, \uc6e8\uc2dc\uc6e8\uc2dc(\uc131\uc11c\uc5d0\uc11c\ub294 \uc544\uc140)\ub294 \ub354 \ubd81\ucabd\uc5d0 \uc704\uce58\ud588\uc744 \uac83\uc774\ub2e4.", "paragraph_answer": "\uc544\uba54\ub178\ud398 \ub610\ub294 \uc544\uba54\ub124\ub9c8\ud504\ud2b8\uc758 \uc624\ub178\ub9c8\uc2a4\ud2f0\ucf58 \ubb38\uc11c\ub294 \ub78c\uc138\uc2a4 \uc655\uc870\uc758 \ud30c\ub77c\uc624\ub4e4\uc774 \ubc14\ub2e4 \ubbfc\uc871\uc744 \uac00\ub098\uc548\uc5d0 \uc815\ucc29\uc2dc\ucf30\ub2e4\ub294 \uc8fc\uc7a5\uc5d0 \uc5b4\ub290 \uc815\ub3c4 \uadfc\uac70\ub97c \uc81c\uacf5\ud55c\ub2e4. \uae30\uc6d0\uc804 1100\ub144\uacbd, \uc81c21\uc655\uc870 \ub9d0\uc5fd\uc5d0 \uc791\uc131\ub41c \uc774 \ubb38\uc11c\ub294 \ub2e8\uc21c\ud788 \uc774\ub984\ub9cc\uc744 \ub098\uc5f4\ud55c\ub2e4. \ud314\ub808\uc2a4\ud0c0\uc778 \uc18c\uc7ac\ud55c 4\uac1c \uc9c0\uba85\uc744 \ud3ec\ud568\ud558\uc5ec 6\uac1c\uc758 \uc9c0\uba85\uc744 \uc5f4\uac70\ud55c \ud6c4, \ubb38\uc11c\ub294 268\ud589\uc5d0\uc11c \uc170\ub374, 269\ud589\uc5d0\uc11c \ud2f0\uc608\ucee4, 270\ud589\uc5d0\uc11c \ud3a0\ub808\uc138\ud2b8\ub97c \uba85\uae30\ud558\uba70, \uc774\ub4e4\uc774 \uc55e\uc11c \uae30\uc7ac\ub41c \uc9c0\uc5ed\uc744 \uc810\uac70\ud588\ub2e4\uace0 \uc0dd\uac01\ud560 \uc218\ub3c4 \uc788\ub2e4. \uac19\uc740 \ube44\ubc00\ucc3d\uace0\uc5d0\uc11c \ubc1c\uacac\ub41c \ud30c\ud53c\ub8e8\uc2a4 \uc5d0 \uc4f0\uc5ec\uc9c4 \uc6e8\ub098\ubb38 \uc774\uc57c\uae30\ub294 \ub2f9\uc2dc \ud2f0\uc608\ucee4\ub97c \ud56d\uad6c\ub3c4\uc2dc \ub3c4\ub974\uc5d0 \uc704\uce58\uc2dc\ucf30\ub2e4. \uc131\uc11c\uc5d0 \ub098\uc624\ub294 \ud574\uc591\ubbfc\uc871\uc778 \ub2e8\uc774 \ucd5c\ucd08\uc5d0 \ud544\ub9ac\uc2a4\ud2f4\uacfc \ud2f0\uc608\ucee4 \uc0ac\uc774\uc5d0 \uc704\uce58\ud588\ub2e4\ub294 \uc0ac\uc2e4\uc740, \uc774\ub4e4\uc774 \uc6d0\ub798 \ub374\uc60c\uc774\uc5c8\uc744 \uc218\ub3c4 \uc788\ub2e4\ub294 \uc8fc\uc7a5\uc774 \ub098\uc62c \uc218 \uc788\uac8c \ud55c\ub2e4. \uc170\ub374\uc740 \uba54\uae30\ub3c4\uc640 \uc694\ub974\ub2e8 \uacc4\uace1 \uadfc\ucc98\uc5d0 \uc815\ucc29\ud55c \uac83\uc73c\ub85c \ubcf4\uc774\uba70, \uc6e8\uc2dc\uc6e8\uc2dc(\uc131\uc11c\uc5d0\uc11c\ub294 \uc544\uc140)\ub294 \ub354 \ubd81\ucabd\uc5d0 \uc704\uce58\ud588\uc744 \uac83\uc774\ub2e4.", "sentence_answer": "\uac19\uc740 \ube44\ubc00\ucc3d\uace0\uc5d0\uc11c \ubc1c\uacac\ub41c \ud30c\ud53c\ub8e8\uc2a4 \uc5d0 \uc4f0\uc5ec\uc9c4 \uc6e8\ub098\ubb38 \uc774\uc57c\uae30\ub294 \ub2f9\uc2dc \ud2f0\uc608\ucee4\ub97c \ud56d\uad6c\ub3c4\uc2dc \ub3c4\ub974\uc5d0 \uc704\uce58\uc2dc\ucf30\ub2e4.", "paragraph_id": "6529902-1-1", "paragraph_question": "question: \uae30\uc6d0\uc804 1100\ub144\uacbd \uc791\uc131\ub41c \ubb38\uc11c\ub4e4\uc758 \ud544\uae30 \uc7ac\ub8cc\ub098 \uc2dd\ubb3c\uc744 \ubb34\uc5c7\uc774\ub77c \ud558\ub294\uac00?, context: \uc544\uba54\ub178\ud398 \ub610\ub294 \uc544\uba54\ub124\ub9c8\ud504\ud2b8\uc758 \uc624\ub178\ub9c8\uc2a4\ud2f0\ucf58 \ubb38\uc11c\ub294 \ub78c\uc138\uc2a4 \uc655\uc870\uc758 \ud30c\ub77c\uc624\ub4e4\uc774 \ubc14\ub2e4 \ubbfc\uc871\uc744 \uac00\ub098\uc548\uc5d0 \uc815\ucc29\uc2dc\ucf30\ub2e4\ub294 \uc8fc\uc7a5\uc5d0 \uc5b4\ub290 \uc815\ub3c4 \uadfc\uac70\ub97c \uc81c\uacf5\ud55c\ub2e4. \uae30\uc6d0\uc804 1100\ub144\uacbd, \uc81c21\uc655\uc870 \ub9d0\uc5fd\uc5d0 \uc791\uc131\ub41c \uc774 \ubb38\uc11c\ub294 \ub2e8\uc21c\ud788 \uc774\ub984\ub9cc\uc744 \ub098\uc5f4\ud55c\ub2e4. \ud314\ub808\uc2a4\ud0c0\uc778 \uc18c\uc7ac\ud55c 4\uac1c \uc9c0\uba85\uc744 \ud3ec\ud568\ud558\uc5ec 6\uac1c\uc758 \uc9c0\uba85\uc744 \uc5f4\uac70\ud55c \ud6c4, \ubb38\uc11c\ub294 268\ud589\uc5d0\uc11c \uc170\ub374, 269\ud589\uc5d0\uc11c \ud2f0\uc608\ucee4, 270\ud589\uc5d0\uc11c \ud3a0\ub808\uc138\ud2b8\ub97c \uba85\uae30\ud558\uba70, \uc774\ub4e4\uc774 \uc55e\uc11c \uae30\uc7ac\ub41c \uc9c0\uc5ed\uc744 \uc810\uac70\ud588\ub2e4\uace0 \uc0dd\uac01\ud560 \uc218\ub3c4 \uc788\ub2e4. \uac19\uc740 \ube44\ubc00\ucc3d\uace0\uc5d0\uc11c \ubc1c\uacac\ub41c \ud30c\ud53c\ub8e8\uc2a4\uc5d0 \uc4f0\uc5ec\uc9c4 \uc6e8\ub098\ubb38 \uc774\uc57c\uae30\ub294 \ub2f9\uc2dc \ud2f0\uc608\ucee4\ub97c \ud56d\uad6c\ub3c4\uc2dc \ub3c4\ub974\uc5d0 \uc704\uce58\uc2dc\ucf30\ub2e4. \uc131\uc11c\uc5d0 \ub098\uc624\ub294 \ud574\uc591\ubbfc\uc871\uc778 \ub2e8\uc774 \ucd5c\ucd08\uc5d0 \ud544\ub9ac\uc2a4\ud2f4\uacfc \ud2f0\uc608\ucee4 \uc0ac\uc774\uc5d0 \uc704\uce58\ud588\ub2e4\ub294 \uc0ac\uc2e4\uc740, \uc774\ub4e4\uc774 \uc6d0\ub798 \ub374\uc60c\uc774\uc5c8\uc744 \uc218\ub3c4 \uc788\ub2e4\ub294 \uc8fc\uc7a5\uc774 \ub098\uc62c \uc218 \uc788\uac8c \ud55c\ub2e4. \uc170\ub374\uc740 \uba54\uae30\ub3c4\uc640 \uc694\ub974\ub2e8 \uacc4\uace1 \uadfc\ucc98\uc5d0 \uc815\ucc29\ud55c \uac83\uc73c\ub85c \ubcf4\uc774\uba70, \uc6e8\uc2dc\uc6e8\uc2dc(\uc131\uc11c\uc5d0\uc11c\ub294 \uc544\uc140)\ub294 \ub354 \ubd81\ucabd\uc5d0 \uc704\uce58\ud588\uc744 \uac83\uc774\ub2e4."} {"question": "\ud0dc\uc885\uc774 \ucf54\ub07c\ub9ac\ub97c \uae30\ub974\ub77c\uace0 \ud55c \uae30\uad00\uc740?", "paragraph": "\uc7ac\uc704 11\ub144\uc778 1411\ub144 2\uc6d4 22\uc77c, \uc77c\ubcf8\uc655 \uc6d0\uc758\uc9c0(\u6e90\u7fa9\u6301, \uc2e4\uc81c\ub85c\ub294 \ubb34\ub85c\ub9c8\uce58 \ub9c9\ubd80\uc758 \uc1fc\uad70 \uc544\uc2dc\uce74\uac00 \uc694\uc2dc\ubaa8\uce58(\u8db3\u5229\u7fa9\u6301))\uac00 \ucf54\ub07c\ub9ac\ub97c \ubc14\ucce4\ub2e4. \uc774\ub294 \uae30\ub85d \uc0c1\uc73c\ub85c \ud55c\ubc18\ub3c4\uc5d0 \ucc98\uc74c\uc73c\ub85c \ub4e4\uc5b4\uc628 \ucf54\ub07c\ub9ac\uc600\ub2e4. \ud0dc\uc885\uc740 \uc774\ub97c \uc0ac\ubcf5\uc2dc(\u53f8\u50d5\u5bfa)\uc5d0\uc11c \uae30\ub974\uac8c \ud588\ub2e4. \uc774\ub4ec\ud574 12\uc6d4 10\uc77c, \uc804 \uacf5\uc870\uc804\uc11c \uc774\uc6b0(\u674e\u7440)\uac00 \ucf54\ub07c\ub9ac\ub97c \ubcf4\uace0 \uce68\uc744 \ubc49\uc5c8\ub294\ub370, \ucf54\ub07c\ub9ac\uac00 \ub178\ud558\uc5ec \uc774\uc6b0\ub97c \ubc1f\uc544 \uc8fd\uc778 \uc0ac\uac74\uc774 \ubc8c\uc5b4\uc84c\ub2e4. 1413\ub144 \ubcd1\uc870\ud310\uc11c \uc720\uc815\ud604(\u67f3\u5ef7\u986f)\uc774 \uc9c4\uc5b8\uc744 \uc62c\ub824 \uc77c\ubcf8\uc5d0\uc11c \ubc14\uce5c \ucf54\ub07c\ub9ac\ub97c \uc804\ub77c\ub3c4 \ud574\ub3c4(\u6d77\u5cf6)\uc5d0 \ub450\uae38 \uccad\ud558\ub2c8, \ud0dc\uc885\uc774 \uc774\ub97c \ubc1b\uc544\ub4e4\uc600\ub2e4. \uadc0\uc591 \uac04 \ucf54\ub07c\ub9ac\uac00 \ud480\uc744 \uba39\uc9c0 \uc54a\uace0 \ub0a0\ub85c \uc218\ucc99\ud574\uc9c4\ub2e4\ub294 \ubcf4\uace0\uac00 \uc62c\ub77c\uc624\uc790 1414\ub144 5\uc6d4 3\uc77c \ud0dc\uc885\uc740 \ucf54\ub07c\ub9ac\ub97c \ub2e4\uc2dc \uc721\uc9c0\ub85c \ubd88\ub7ec\uc624\uac8c \ud588\ub2e4.", "answer": "\uc0ac\ubcf5\uc2dc", "sentence": "\ud0dc\uc885\uc740 \uc774\ub97c \uc0ac\ubcf5\uc2dc (\u53f8\u50d5\u5bfa)", "paragraph_sentence": "\uc7ac\uc704 11\ub144\uc778 1411\ub144 2\uc6d4 22\uc77c, \uc77c\ubcf8\uc655 \uc6d0\uc758\uc9c0(\u6e90\u7fa9\u6301, \uc2e4\uc81c\ub85c\ub294 \ubb34\ub85c\ub9c8\uce58 \ub9c9\ubd80\uc758 \uc1fc\uad70 \uc544\uc2dc\uce74\uac00 \uc694\uc2dc\ubaa8\uce58(\u8db3\u5229\u7fa9\u6301))\uac00 \ucf54\ub07c\ub9ac\ub97c \ubc14\ucce4\ub2e4. \uc774\ub294 \uae30\ub85d \uc0c1\uc73c\ub85c \ud55c\ubc18\ub3c4\uc5d0 \ucc98\uc74c\uc73c\ub85c \ub4e4\uc5b4\uc628 \ucf54\ub07c\ub9ac\uc600\ub2e4. \ud0dc\uc885\uc740 \uc774\ub97c \uc0ac\ubcf5\uc2dc (\u53f8\u50d5\u5bfa) \uc5d0\uc11c \uae30\ub974\uac8c \ud588\ub2e4. \uc774\ub4ec\ud574 12\uc6d4 10\uc77c, \uc804 \uacf5\uc870\uc804\uc11c \uc774\uc6b0(\u674e\u7440)\uac00 \ucf54\ub07c\ub9ac\ub97c \ubcf4\uace0 \uce68\uc744 \ubc49\uc5c8\ub294\ub370, \ucf54\ub07c\ub9ac\uac00 \ub178\ud558\uc5ec \uc774\uc6b0\ub97c \ubc1f\uc544 \uc8fd\uc778 \uc0ac\uac74\uc774 \ubc8c\uc5b4\uc84c\ub2e4. 1413\ub144 \ubcd1\uc870\ud310\uc11c \uc720\uc815\ud604(\u67f3\u5ef7\u986f)\uc774 \uc9c4\uc5b8\uc744 \uc62c\ub824 \uc77c\ubcf8\uc5d0\uc11c \ubc14\uce5c \ucf54\ub07c\ub9ac\ub97c \uc804\ub77c\ub3c4 \ud574\ub3c4(\u6d77\u5cf6)\uc5d0 \ub450\uae38 \uccad\ud558\ub2c8, \ud0dc\uc885\uc774 \uc774\ub97c \ubc1b\uc544\ub4e4\uc600\ub2e4. \uadc0\uc591 \uac04 \ucf54\ub07c\ub9ac\uac00 \ud480\uc744 \uba39\uc9c0 \uc54a\uace0 \ub0a0\ub85c \uc218\ucc99\ud574\uc9c4\ub2e4\ub294 \ubcf4\uace0\uac00 \uc62c\ub77c\uc624\uc790 1414\ub144 5\uc6d4 3\uc77c \ud0dc\uc885\uc740 \ucf54\ub07c\ub9ac\ub97c \ub2e4\uc2dc \uc721\uc9c0\ub85c \ubd88\ub7ec\uc624\uac8c \ud588\ub2e4.", "paragraph_answer": "\uc7ac\uc704 11\ub144\uc778 1411\ub144 2\uc6d4 22\uc77c, \uc77c\ubcf8\uc655 \uc6d0\uc758\uc9c0(\u6e90\u7fa9\u6301, \uc2e4\uc81c\ub85c\ub294 \ubb34\ub85c\ub9c8\uce58 \ub9c9\ubd80\uc758 \uc1fc\uad70 \uc544\uc2dc\uce74\uac00 \uc694\uc2dc\ubaa8\uce58(\u8db3\u5229\u7fa9\u6301))\uac00 \ucf54\ub07c\ub9ac\ub97c \ubc14\ucce4\ub2e4. \uc774\ub294 \uae30\ub85d \uc0c1\uc73c\ub85c \ud55c\ubc18\ub3c4\uc5d0 \ucc98\uc74c\uc73c\ub85c \ub4e4\uc5b4\uc628 \ucf54\ub07c\ub9ac\uc600\ub2e4. \ud0dc\uc885\uc740 \uc774\ub97c \uc0ac\ubcf5\uc2dc (\u53f8\u50d5\u5bfa)\uc5d0\uc11c \uae30\ub974\uac8c \ud588\ub2e4. \uc774\ub4ec\ud574 12\uc6d4 10\uc77c, \uc804 \uacf5\uc870\uc804\uc11c \uc774\uc6b0(\u674e\u7440)\uac00 \ucf54\ub07c\ub9ac\ub97c \ubcf4\uace0 \uce68\uc744 \ubc49\uc5c8\ub294\ub370, \ucf54\ub07c\ub9ac\uac00 \ub178\ud558\uc5ec \uc774\uc6b0\ub97c \ubc1f\uc544 \uc8fd\uc778 \uc0ac\uac74\uc774 \ubc8c\uc5b4\uc84c\ub2e4. 1413\ub144 \ubcd1\uc870\ud310\uc11c \uc720\uc815\ud604(\u67f3\u5ef7\u986f)\uc774 \uc9c4\uc5b8\uc744 \uc62c\ub824 \uc77c\ubcf8\uc5d0\uc11c \ubc14\uce5c \ucf54\ub07c\ub9ac\ub97c \uc804\ub77c\ub3c4 \ud574\ub3c4(\u6d77\u5cf6)\uc5d0 \ub450\uae38 \uccad\ud558\ub2c8, \ud0dc\uc885\uc774 \uc774\ub97c \ubc1b\uc544\ub4e4\uc600\ub2e4. \uadc0\uc591 \uac04 \ucf54\ub07c\ub9ac\uac00 \ud480\uc744 \uba39\uc9c0 \uc54a\uace0 \ub0a0\ub85c \uc218\ucc99\ud574\uc9c4\ub2e4\ub294 \ubcf4\uace0\uac00 \uc62c\ub77c\uc624\uc790 1414\ub144 5\uc6d4 3\uc77c \ud0dc\uc885\uc740 \ucf54\ub07c\ub9ac\ub97c \ub2e4\uc2dc \uc721\uc9c0\ub85c \ubd88\ub7ec\uc624\uac8c \ud588\ub2e4.", "sentence_answer": "\ud0dc\uc885\uc740 \uc774\ub97c \uc0ac\ubcf5\uc2dc (\u53f8\u50d5\u5bfa)", "paragraph_id": "6551325-3-1", "paragraph_question": "question: \ud0dc\uc885\uc774 \ucf54\ub07c\ub9ac\ub97c \uae30\ub974\ub77c\uace0 \ud55c \uae30\uad00\uc740?, context: \uc7ac\uc704 11\ub144\uc778 1411\ub144 2\uc6d4 22\uc77c, \uc77c\ubcf8\uc655 \uc6d0\uc758\uc9c0(\u6e90\u7fa9\u6301, \uc2e4\uc81c\ub85c\ub294 \ubb34\ub85c\ub9c8\uce58 \ub9c9\ubd80\uc758 \uc1fc\uad70 \uc544\uc2dc\uce74\uac00 \uc694\uc2dc\ubaa8\uce58(\u8db3\u5229\u7fa9\u6301))\uac00 \ucf54\ub07c\ub9ac\ub97c \ubc14\ucce4\ub2e4. \uc774\ub294 \uae30\ub85d \uc0c1\uc73c\ub85c \ud55c\ubc18\ub3c4\uc5d0 \ucc98\uc74c\uc73c\ub85c \ub4e4\uc5b4\uc628 \ucf54\ub07c\ub9ac\uc600\ub2e4. \ud0dc\uc885\uc740 \uc774\ub97c \uc0ac\ubcf5\uc2dc(\u53f8\u50d5\u5bfa)\uc5d0\uc11c \uae30\ub974\uac8c \ud588\ub2e4. \uc774\ub4ec\ud574 12\uc6d4 10\uc77c, \uc804 \uacf5\uc870\uc804\uc11c \uc774\uc6b0(\u674e\u7440)\uac00 \ucf54\ub07c\ub9ac\ub97c \ubcf4\uace0 \uce68\uc744 \ubc49\uc5c8\ub294\ub370, \ucf54\ub07c\ub9ac\uac00 \ub178\ud558\uc5ec \uc774\uc6b0\ub97c \ubc1f\uc544 \uc8fd\uc778 \uc0ac\uac74\uc774 \ubc8c\uc5b4\uc84c\ub2e4. 1413\ub144 \ubcd1\uc870\ud310\uc11c \uc720\uc815\ud604(\u67f3\u5ef7\u986f)\uc774 \uc9c4\uc5b8\uc744 \uc62c\ub824 \uc77c\ubcf8\uc5d0\uc11c \ubc14\uce5c \ucf54\ub07c\ub9ac\ub97c \uc804\ub77c\ub3c4 \ud574\ub3c4(\u6d77\u5cf6)\uc5d0 \ub450\uae38 \uccad\ud558\ub2c8, \ud0dc\uc885\uc774 \uc774\ub97c \ubc1b\uc544\ub4e4\uc600\ub2e4. \uadc0\uc591 \uac04 \ucf54\ub07c\ub9ac\uac00 \ud480\uc744 \uba39\uc9c0 \uc54a\uace0 \ub0a0\ub85c \uc218\ucc99\ud574\uc9c4\ub2e4\ub294 \ubcf4\uace0\uac00 \uc62c\ub77c\uc624\uc790 1414\ub144 5\uc6d4 3\uc77c \ud0dc\uc885\uc740 \ucf54\ub07c\ub9ac\ub97c \ub2e4\uc2dc \uc721\uc9c0\ub85c \ubd88\ub7ec\uc624\uac8c \ud588\ub2e4."} {"question": "\uce74\uc774\uc0ac\ub974\uac00 \uc554\uc0b4\ub2f9\ud55c \ub0a0\uc9dc\ub294?", "paragraph": "\uae30\uc6d0\uc804 44\ub144 3\uc6d4 15\uc77c \uce74\uc774\uc0ac\ub974\uac00 \uc554\uc0b4\ub2f9\ud560 \ub54c, \uc625\ud0c0\ube44\uc544\ub204\uc2a4\ub294 \uc77c\ub9ac\ub9ac\uc544\uc758 \uc544\ud3f4\ub85c\ub2c8\uc544\uc5d0\uc11c \uacf5\ubd80\uc640 \uad70\uc0ac \ud6c8\ub828\uc744 \ubcd1\ud589\ud558\uace0 \uc788\uc5c8\ub2e4. \uadf8\ub294 \uce74\uc774\uc0ac\ub974\uac00 \uc554\uc0b4\ub418\uc790 \uad70\uc0ac\ub4e4\uacfc \ud568\uaed8 \ub9c8\ucf00\ub3c4\ub2c8\uc544\ub85c \ud53c\uc2e0\ud558\ub77c\ub294 \uc7a5\uad50\ub4e4\uc758 \uc870\uc5b8\uc744 \uac70\uc808\ud558\uace0 \uc774\ud0c8\ub9ac\uc544\ub85c \uac00 \uc790\uc2e0\uc758 \uc7a0\uc7ac\uc801\uc778 \uc815\uce58\uc801 \uc138\ub825\uc744 \uaddc\ud569\ud558\ub824 \ud558\uc600\ub2e4. \ube0c\ub8ec\ub514\uc2dc\uc6c0 \uadfc\ucc98\uc5d0 \uc788\ub294 \ub8e8\ud53c\uc544\uc774\ub77c\ub294 \uacf3\uc5d0 \uc0c1\ub959\ud558\uace0 \ub09c \ub4a4, \uce74\uc774\uc0ac\ub974\uc758 \uc720\uc11c\uc5d0 \uc801\ud78c \ub0b4\uc6a9\uc744 \ub4e4\uc740 \uc625\ud0c0\ube44\uc544\ub204\uc2a4\ub294 \uc7ac\uc0b0\uc758 2/3\ub97c \uc0c1\uc18d\ubc1b\ub294 \uac83\ubfd0\ub9cc \uc544\ub2c8\ub77c \uc815\uce58\uc801\uc73c\ub85c\ub3c4 \uc790\uc2e0\uc774 \uce74\uc774\uc0ac\ub974\uc758 \uc815\uce58\uc801\uc778 \uc9c4\uc815\ud55c \ud6c4\uacc4\uc790\uac00 \ub418\uae30\ub85c \ud558\uc600\ub2e4. \ub2f9\uc2dc \uce74\uc774\uc0ac\ub974\uc5d0\uac8c\ub294 \uc0b4\uc544 \uc788\ub294 \uc801\uc790\ub140\uac00 \uc5c6\uc5c8\uae30\uc5d0 \uadf8\uc758 \uc774\uc190(\u96e2\u5b6b, \ub204\uc774\uc758 \uce5c\uc190 \ubc0f \uc678\uc190)\uc778 \uc625\ud0c0\ube44\uc544\ub204\uc2a4\uac00 \uc81c1 \uc0c1\uc18d\uc790\uc774\uc790 \uc544\ub4e4\ub85c \uc785\uc591\ub418\uc5c8\ub2e4. \uce74\uc774\uc0ac\ub974\uac00 \uc8fd\uace0 \ub09c \ub4a4 \uc815\uc2dd \uc785\uc591\ub41c \uc625\ud0c0\ube44\uc544\ub204\uc2a4\ub294 \uc591\ubd80\uc758 \uc774\ub984\uc778 \uac00\uc774\uc6b0\uc2a4 \uc728\ub9ac\uc6b0\uc2a4 \uce74\uc774\uc0ac\ub974(Gaius Julius Caesar)\ub97c \uc4f0\uac8c \ub418\uc5c8\ub2e4. \ub85c\ub9c8 \uc804\ud1b5\uc5d0 \ub530\ub77c \uadf8\ub294 \u2018\uac00\uc774\uc6b0\uc2a4 \uc728\ub9ac\uc6b0\uc2a4 \uce74\uc774\uc0ac\ub974\u2019\ub77c\ub294 \uc774\ub984\uc5d0 \u2018\ud22c\ub9ac\ub204\uc2a4\u2019 \ub300\uc2e0 \u2018\uc625\ud0c0\ube44\uc544\ub204\uc2a4\u2019(Octavianus)\ub77c\ub294 \uac00\uba85\uc744 \ubd99\uc5ec \uc790\uc2e0\uc758 \ucd9c\uc2e0 \uac00\ubb38\uc744 \ub098\ud0c0\ub0b4\uc5c8\ub2e4. \uc544\uc9c1 \uc625\ud0c0\ube44\uc544\ub204\uc2a4\uac00 \ub2f9\uc2dc \uc774 \uc774\ub984\uc744 \uc37c\ub2e4\ub294 \uacb0\uc815\uc801\uc778 \uc99d\uac70\uac00 \uc874\uc7ac\ud558\uc9c0\ub294 \uc54a\uc9c0\ub9cc, \uc5b4\uca0c\ub4e0 \uc774 \uc774\ub984\uc740 \uadf8\uc758 \ucd9c\uc2e0\uc744 \uba85\ubc31\ud558\uac8c \ub4dc\ub7ec\ub0b4\uac8c \ud558\uc5ec \ub9ce\uc740 \uc0ac\ub78c\uc5d0\uac8c \uc54c\ub824\uc9c0\uac8c \ub418\uc5c8\ub2e4. \ud6d7\ub0a0 \ub9c8\ub974\ucfe0\uc2a4 \uc548\ud1a0\ub2c8\uc6b0\uc2a4\uac00 \uce74\uc774\uc0ac\ub974\uac00 \uc625\ud0c0\ube44\uc544\ub204\uc2a4\ub97c \uc131\uc801\uc73c\ub85c \ucd1d\uc560\ud588\uae30 \ub54c\ubb38\uc5d0 \uc785\uc591\ud55c \uac83\uc774\ub77c\uba70 \uac15\ub3c4 \ub192\uac8c \ube44\ub09c\ud558\uc600\uc9c0\ub9cc, \uc774\ub294 \uc911\uc0c1\ubaa8\ub7b5\uc774\uc5c8\ub2e4\ub294 \uc758\uacac\uc774 \uc788\ub2e4.", "answer": "\uae30\uc6d0\uc804 44\ub144 3\uc6d4 15\uc77c", "sentence": "\uae30\uc6d0\uc804 44\ub144 3\uc6d4 15\uc77c \uce74\uc774\uc0ac\ub974\uac00 \uc554\uc0b4\ub2f9\ud560 \ub54c, \uc625\ud0c0\ube44\uc544\ub204\uc2a4\ub294 \uc77c\ub9ac\ub9ac\uc544\uc758 \uc544\ud3f4\ub85c\ub2c8\uc544\uc5d0\uc11c \uacf5\ubd80\uc640 \uad70\uc0ac \ud6c8\ub828\uc744 \ubcd1\ud589\ud558\uace0 \uc788\uc5c8\ub2e4.", "paragraph_sentence": " \uae30\uc6d0\uc804 44\ub144 3\uc6d4 15\uc77c \uce74\uc774\uc0ac\ub974\uac00 \uc554\uc0b4\ub2f9\ud560 \ub54c, \uc625\ud0c0\ube44\uc544\ub204\uc2a4\ub294 \uc77c\ub9ac\ub9ac\uc544\uc758 \uc544\ud3f4\ub85c\ub2c8\uc544\uc5d0\uc11c \uacf5\ubd80\uc640 \uad70\uc0ac \ud6c8\ub828\uc744 \ubcd1\ud589\ud558\uace0 \uc788\uc5c8\ub2e4. \uadf8\ub294 \uce74\uc774\uc0ac\ub974\uac00 \uc554\uc0b4\ub418\uc790 \uad70\uc0ac\ub4e4\uacfc \ud568\uaed8 \ub9c8\ucf00\ub3c4\ub2c8\uc544\ub85c \ud53c\uc2e0\ud558\ub77c\ub294 \uc7a5\uad50\ub4e4\uc758 \uc870\uc5b8\uc744 \uac70\uc808\ud558\uace0 \uc774\ud0c8\ub9ac\uc544\ub85c \uac00 \uc790\uc2e0\uc758 \uc7a0\uc7ac\uc801\uc778 \uc815\uce58\uc801 \uc138\ub825\uc744 \uaddc\ud569\ud558\ub824 \ud558\uc600\ub2e4. \ube0c\ub8ec\ub514\uc2dc\uc6c0 \uadfc\ucc98\uc5d0 \uc788\ub294 \ub8e8\ud53c\uc544\uc774\ub77c\ub294 \uacf3\uc5d0 \uc0c1\ub959\ud558\uace0 \ub09c \ub4a4, \uce74\uc774\uc0ac\ub974\uc758 \uc720\uc11c\uc5d0 \uc801\ud78c \ub0b4\uc6a9\uc744 \ub4e4\uc740 \uc625\ud0c0\ube44\uc544\ub204\uc2a4\ub294 \uc7ac\uc0b0\uc758 2/3\ub97c \uc0c1\uc18d\ubc1b\ub294 \uac83\ubfd0\ub9cc \uc544\ub2c8\ub77c \uc815\uce58\uc801\uc73c\ub85c\ub3c4 \uc790\uc2e0\uc774 \uce74\uc774\uc0ac\ub974\uc758 \uc815\uce58\uc801\uc778 \uc9c4\uc815\ud55c \ud6c4\uacc4\uc790\uac00 \ub418\uae30\ub85c \ud558\uc600\ub2e4. \ub2f9\uc2dc \uce74\uc774\uc0ac\ub974\uc5d0\uac8c\ub294 \uc0b4\uc544 \uc788\ub294 \uc801\uc790\ub140\uac00 \uc5c6\uc5c8\uae30\uc5d0 \uadf8\uc758 \uc774\uc190(\u96e2\u5b6b, \ub204\uc774\uc758 \uce5c\uc190 \ubc0f \uc678\uc190)\uc778 \uc625\ud0c0\ube44\uc544\ub204\uc2a4\uac00 \uc81c1 \uc0c1\uc18d\uc790\uc774\uc790 \uc544\ub4e4\ub85c \uc785\uc591\ub418\uc5c8\ub2e4. \uce74\uc774\uc0ac\ub974\uac00 \uc8fd\uace0 \ub09c \ub4a4 \uc815\uc2dd \uc785\uc591\ub41c \uc625\ud0c0\ube44\uc544\ub204\uc2a4\ub294 \uc591\ubd80\uc758 \uc774\ub984\uc778 \uac00\uc774\uc6b0\uc2a4 \uc728\ub9ac\uc6b0\uc2a4 \uce74\uc774\uc0ac\ub974(Gaius Julius Caesar)\ub97c \uc4f0\uac8c \ub418\uc5c8\ub2e4. \ub85c\ub9c8 \uc804\ud1b5\uc5d0 \ub530\ub77c \uadf8\ub294 \u2018\uac00\uc774\uc6b0\uc2a4 \uc728\ub9ac\uc6b0\uc2a4 \uce74\uc774\uc0ac\ub974\u2019\ub77c\ub294 \uc774\ub984\uc5d0 \u2018\ud22c\ub9ac\ub204\uc2a4\u2019 \ub300\uc2e0 \u2018\uc625\ud0c0\ube44\uc544\ub204\uc2a4\u2019(Octavianus)\ub77c\ub294 \uac00\uba85\uc744 \ubd99\uc5ec \uc790\uc2e0\uc758 \ucd9c\uc2e0 \uac00\ubb38\uc744 \ub098\ud0c0\ub0b4\uc5c8\ub2e4. \uc544\uc9c1 \uc625\ud0c0\ube44\uc544\ub204\uc2a4\uac00 \ub2f9\uc2dc \uc774 \uc774\ub984\uc744 \uc37c\ub2e4\ub294 \uacb0\uc815\uc801\uc778 \uc99d\uac70\uac00 \uc874\uc7ac\ud558\uc9c0\ub294 \uc54a\uc9c0\ub9cc, \uc5b4\uca0c\ub4e0 \uc774 \uc774\ub984\uc740 \uadf8\uc758 \ucd9c\uc2e0\uc744 \uba85\ubc31\ud558\uac8c \ub4dc\ub7ec\ub0b4\uac8c \ud558\uc5ec \ub9ce\uc740 \uc0ac\ub78c\uc5d0\uac8c \uc54c\ub824\uc9c0\uac8c \ub418\uc5c8\ub2e4. \ud6d7\ub0a0 \ub9c8\ub974\ucfe0\uc2a4 \uc548\ud1a0\ub2c8\uc6b0\uc2a4\uac00 \uce74\uc774\uc0ac\ub974\uac00 \uc625\ud0c0\ube44\uc544\ub204\uc2a4\ub97c \uc131\uc801\uc73c\ub85c \ucd1d\uc560\ud588\uae30 \ub54c\ubb38\uc5d0 \uc785\uc591\ud55c \uac83\uc774\ub77c\uba70 \uac15\ub3c4 \ub192\uac8c \ube44\ub09c\ud558\uc600\uc9c0\ub9cc, \uc774\ub294 \uc911\uc0c1\ubaa8\ub7b5\uc774\uc5c8\ub2e4\ub294 \uc758\uacac\uc774 \uc788\ub2e4.", "paragraph_answer": " \uae30\uc6d0\uc804 44\ub144 3\uc6d4 15\uc77c \uce74\uc774\uc0ac\ub974\uac00 \uc554\uc0b4\ub2f9\ud560 \ub54c, \uc625\ud0c0\ube44\uc544\ub204\uc2a4\ub294 \uc77c\ub9ac\ub9ac\uc544\uc758 \uc544\ud3f4\ub85c\ub2c8\uc544\uc5d0\uc11c \uacf5\ubd80\uc640 \uad70\uc0ac \ud6c8\ub828\uc744 \ubcd1\ud589\ud558\uace0 \uc788\uc5c8\ub2e4. \uadf8\ub294 \uce74\uc774\uc0ac\ub974\uac00 \uc554\uc0b4\ub418\uc790 \uad70\uc0ac\ub4e4\uacfc \ud568\uaed8 \ub9c8\ucf00\ub3c4\ub2c8\uc544\ub85c \ud53c\uc2e0\ud558\ub77c\ub294 \uc7a5\uad50\ub4e4\uc758 \uc870\uc5b8\uc744 \uac70\uc808\ud558\uace0 \uc774\ud0c8\ub9ac\uc544\ub85c \uac00 \uc790\uc2e0\uc758 \uc7a0\uc7ac\uc801\uc778 \uc815\uce58\uc801 \uc138\ub825\uc744 \uaddc\ud569\ud558\ub824 \ud558\uc600\ub2e4. \ube0c\ub8ec\ub514\uc2dc\uc6c0 \uadfc\ucc98\uc5d0 \uc788\ub294 \ub8e8\ud53c\uc544\uc774\ub77c\ub294 \uacf3\uc5d0 \uc0c1\ub959\ud558\uace0 \ub09c \ub4a4, \uce74\uc774\uc0ac\ub974\uc758 \uc720\uc11c\uc5d0 \uc801\ud78c \ub0b4\uc6a9\uc744 \ub4e4\uc740 \uc625\ud0c0\ube44\uc544\ub204\uc2a4\ub294 \uc7ac\uc0b0\uc758 2/3\ub97c \uc0c1\uc18d\ubc1b\ub294 \uac83\ubfd0\ub9cc \uc544\ub2c8\ub77c \uc815\uce58\uc801\uc73c\ub85c\ub3c4 \uc790\uc2e0\uc774 \uce74\uc774\uc0ac\ub974\uc758 \uc815\uce58\uc801\uc778 \uc9c4\uc815\ud55c \ud6c4\uacc4\uc790\uac00 \ub418\uae30\ub85c \ud558\uc600\ub2e4. \ub2f9\uc2dc \uce74\uc774\uc0ac\ub974\uc5d0\uac8c\ub294 \uc0b4\uc544 \uc788\ub294 \uc801\uc790\ub140\uac00 \uc5c6\uc5c8\uae30\uc5d0 \uadf8\uc758 \uc774\uc190(\u96e2\u5b6b, \ub204\uc774\uc758 \uce5c\uc190 \ubc0f \uc678\uc190)\uc778 \uc625\ud0c0\ube44\uc544\ub204\uc2a4\uac00 \uc81c1 \uc0c1\uc18d\uc790\uc774\uc790 \uc544\ub4e4\ub85c \uc785\uc591\ub418\uc5c8\ub2e4. \uce74\uc774\uc0ac\ub974\uac00 \uc8fd\uace0 \ub09c \ub4a4 \uc815\uc2dd \uc785\uc591\ub41c \uc625\ud0c0\ube44\uc544\ub204\uc2a4\ub294 \uc591\ubd80\uc758 \uc774\ub984\uc778 \uac00\uc774\uc6b0\uc2a4 \uc728\ub9ac\uc6b0\uc2a4 \uce74\uc774\uc0ac\ub974(Gaius Julius Caesar)\ub97c \uc4f0\uac8c \ub418\uc5c8\ub2e4. \ub85c\ub9c8 \uc804\ud1b5\uc5d0 \ub530\ub77c \uadf8\ub294 \u2018\uac00\uc774\uc6b0\uc2a4 \uc728\ub9ac\uc6b0\uc2a4 \uce74\uc774\uc0ac\ub974\u2019\ub77c\ub294 \uc774\ub984\uc5d0 \u2018\ud22c\ub9ac\ub204\uc2a4\u2019 \ub300\uc2e0 \u2018\uc625\ud0c0\ube44\uc544\ub204\uc2a4\u2019(Octavianus)\ub77c\ub294 \uac00\uba85\uc744 \ubd99\uc5ec \uc790\uc2e0\uc758 \ucd9c\uc2e0 \uac00\ubb38\uc744 \ub098\ud0c0\ub0b4\uc5c8\ub2e4. \uc544\uc9c1 \uc625\ud0c0\ube44\uc544\ub204\uc2a4\uac00 \ub2f9\uc2dc \uc774 \uc774\ub984\uc744 \uc37c\ub2e4\ub294 \uacb0\uc815\uc801\uc778 \uc99d\uac70\uac00 \uc874\uc7ac\ud558\uc9c0\ub294 \uc54a\uc9c0\ub9cc, \uc5b4\uca0c\ub4e0 \uc774 \uc774\ub984\uc740 \uadf8\uc758 \ucd9c\uc2e0\uc744 \uba85\ubc31\ud558\uac8c \ub4dc\ub7ec\ub0b4\uac8c \ud558\uc5ec \ub9ce\uc740 \uc0ac\ub78c\uc5d0\uac8c \uc54c\ub824\uc9c0\uac8c \ub418\uc5c8\ub2e4. \ud6d7\ub0a0 \ub9c8\ub974\ucfe0\uc2a4 \uc548\ud1a0\ub2c8\uc6b0\uc2a4\uac00 \uce74\uc774\uc0ac\ub974\uac00 \uc625\ud0c0\ube44\uc544\ub204\uc2a4\ub97c \uc131\uc801\uc73c\ub85c \ucd1d\uc560\ud588\uae30 \ub54c\ubb38\uc5d0 \uc785\uc591\ud55c \uac83\uc774\ub77c\uba70 \uac15\ub3c4 \ub192\uac8c \ube44\ub09c\ud558\uc600\uc9c0\ub9cc, \uc774\ub294 \uc911\uc0c1\ubaa8\ub7b5\uc774\uc5c8\ub2e4\ub294 \uc758\uacac\uc774 \uc788\ub2e4.", "sentence_answer": " \uae30\uc6d0\uc804 44\ub144 3\uc6d4 15\uc77c \uce74\uc774\uc0ac\ub974\uac00 \uc554\uc0b4\ub2f9\ud560 \ub54c, \uc625\ud0c0\ube44\uc544\ub204\uc2a4\ub294 \uc77c\ub9ac\ub9ac\uc544\uc758 \uc544\ud3f4\ub85c\ub2c8\uc544\uc5d0\uc11c \uacf5\ubd80\uc640 \uad70\uc0ac \ud6c8\ub828\uc744 \ubcd1\ud589\ud558\uace0 \uc788\uc5c8\ub2e4.", "paragraph_id": "6585254-4-0", "paragraph_question": "question: \uce74\uc774\uc0ac\ub974\uac00 \uc554\uc0b4\ub2f9\ud55c \ub0a0\uc9dc\ub294?, context: \uae30\uc6d0\uc804 44\ub144 3\uc6d4 15\uc77c \uce74\uc774\uc0ac\ub974\uac00 \uc554\uc0b4\ub2f9\ud560 \ub54c, \uc625\ud0c0\ube44\uc544\ub204\uc2a4\ub294 \uc77c\ub9ac\ub9ac\uc544\uc758 \uc544\ud3f4\ub85c\ub2c8\uc544\uc5d0\uc11c \uacf5\ubd80\uc640 \uad70\uc0ac \ud6c8\ub828\uc744 \ubcd1\ud589\ud558\uace0 \uc788\uc5c8\ub2e4. \uadf8\ub294 \uce74\uc774\uc0ac\ub974\uac00 \uc554\uc0b4\ub418\uc790 \uad70\uc0ac\ub4e4\uacfc \ud568\uaed8 \ub9c8\ucf00\ub3c4\ub2c8\uc544\ub85c \ud53c\uc2e0\ud558\ub77c\ub294 \uc7a5\uad50\ub4e4\uc758 \uc870\uc5b8\uc744 \uac70\uc808\ud558\uace0 \uc774\ud0c8\ub9ac\uc544\ub85c \uac00 \uc790\uc2e0\uc758 \uc7a0\uc7ac\uc801\uc778 \uc815\uce58\uc801 \uc138\ub825\uc744 \uaddc\ud569\ud558\ub824 \ud558\uc600\ub2e4. \ube0c\ub8ec\ub514\uc2dc\uc6c0 \uadfc\ucc98\uc5d0 \uc788\ub294 \ub8e8\ud53c\uc544\uc774\ub77c\ub294 \uacf3\uc5d0 \uc0c1\ub959\ud558\uace0 \ub09c \ub4a4, \uce74\uc774\uc0ac\ub974\uc758 \uc720\uc11c\uc5d0 \uc801\ud78c \ub0b4\uc6a9\uc744 \ub4e4\uc740 \uc625\ud0c0\ube44\uc544\ub204\uc2a4\ub294 \uc7ac\uc0b0\uc758 2/3\ub97c \uc0c1\uc18d\ubc1b\ub294 \uac83\ubfd0\ub9cc \uc544\ub2c8\ub77c \uc815\uce58\uc801\uc73c\ub85c\ub3c4 \uc790\uc2e0\uc774 \uce74\uc774\uc0ac\ub974\uc758 \uc815\uce58\uc801\uc778 \uc9c4\uc815\ud55c \ud6c4\uacc4\uc790\uac00 \ub418\uae30\ub85c \ud558\uc600\ub2e4. \ub2f9\uc2dc \uce74\uc774\uc0ac\ub974\uc5d0\uac8c\ub294 \uc0b4\uc544 \uc788\ub294 \uc801\uc790\ub140\uac00 \uc5c6\uc5c8\uae30\uc5d0 \uadf8\uc758 \uc774\uc190(\u96e2\u5b6b, \ub204\uc774\uc758 \uce5c\uc190 \ubc0f \uc678\uc190)\uc778 \uc625\ud0c0\ube44\uc544\ub204\uc2a4\uac00 \uc81c1 \uc0c1\uc18d\uc790\uc774\uc790 \uc544\ub4e4\ub85c \uc785\uc591\ub418\uc5c8\ub2e4. \uce74\uc774\uc0ac\ub974\uac00 \uc8fd\uace0 \ub09c \ub4a4 \uc815\uc2dd \uc785\uc591\ub41c \uc625\ud0c0\ube44\uc544\ub204\uc2a4\ub294 \uc591\ubd80\uc758 \uc774\ub984\uc778 \uac00\uc774\uc6b0\uc2a4 \uc728\ub9ac\uc6b0\uc2a4 \uce74\uc774\uc0ac\ub974(Gaius Julius Caesar)\ub97c \uc4f0\uac8c \ub418\uc5c8\ub2e4. \ub85c\ub9c8 \uc804\ud1b5\uc5d0 \ub530\ub77c \uadf8\ub294 \u2018\uac00\uc774\uc6b0\uc2a4 \uc728\ub9ac\uc6b0\uc2a4 \uce74\uc774\uc0ac\ub974\u2019\ub77c\ub294 \uc774\ub984\uc5d0 \u2018\ud22c\ub9ac\ub204\uc2a4\u2019 \ub300\uc2e0 \u2018\uc625\ud0c0\ube44\uc544\ub204\uc2a4\u2019(Octavianus)\ub77c\ub294 \uac00\uba85\uc744 \ubd99\uc5ec \uc790\uc2e0\uc758 \ucd9c\uc2e0 \uac00\ubb38\uc744 \ub098\ud0c0\ub0b4\uc5c8\ub2e4. \uc544\uc9c1 \uc625\ud0c0\ube44\uc544\ub204\uc2a4\uac00 \ub2f9\uc2dc \uc774 \uc774\ub984\uc744 \uc37c\ub2e4\ub294 \uacb0\uc815\uc801\uc778 \uc99d\uac70\uac00 \uc874\uc7ac\ud558\uc9c0\ub294 \uc54a\uc9c0\ub9cc, \uc5b4\uca0c\ub4e0 \uc774 \uc774\ub984\uc740 \uadf8\uc758 \ucd9c\uc2e0\uc744 \uba85\ubc31\ud558\uac8c \ub4dc\ub7ec\ub0b4\uac8c \ud558\uc5ec \ub9ce\uc740 \uc0ac\ub78c\uc5d0\uac8c \uc54c\ub824\uc9c0\uac8c \ub418\uc5c8\ub2e4. \ud6d7\ub0a0 \ub9c8\ub974\ucfe0\uc2a4 \uc548\ud1a0\ub2c8\uc6b0\uc2a4\uac00 \uce74\uc774\uc0ac\ub974\uac00 \uc625\ud0c0\ube44\uc544\ub204\uc2a4\ub97c \uc131\uc801\uc73c\ub85c \ucd1d\uc560\ud588\uae30 \ub54c\ubb38\uc5d0 \uc785\uc591\ud55c \uac83\uc774\ub77c\uba70 \uac15\ub3c4 \ub192\uac8c \ube44\ub09c\ud558\uc600\uc9c0\ub9cc, \uc774\ub294 \uc911\uc0c1\ubaa8\ub7b5\uc774\uc5c8\ub2e4\ub294 \uc758\uacac\uc774 \uc788\ub2e4."} {"question": "\ube0c\ub8e8\uc11c\uc2a4\ub294 1881\ub144 \uc5b4\ub5a4 \ud300\uacfc \uacc4\uc57d\ud588\ub294\uac00?", "paragraph": "1881\ub144 \uc2dc\uc98c\uc5d0 \uc791\ub144\uc5d0 \uc798 \ucce4\uc5c8\ub358 \uc0c1\ub300\uc778 \ubc14\uc774\uc2a8\uc2a4\uc640 \uacc4\uc57d\ud558\uba74\uc11c, \ucc98\uc74c\uc73c\ub85c \uc8fc\uc804\uc73c\ub85c \ubc1c\ub3cb\uc6c0\ud560 \uae30\ud68c\ub97c \uc5bb\uc5c8\ub2e4. \uc774\ud6c4 1885\ub144 \uc2dc\uc98c \ud6c4 \uc18c\uc18d\ud300\uc774 \uc0ac\ub77c\uc9c0\uae30 \uc804\uae4c\uc9c0 \uacc4\uc18d \uc774\uacf3\uc5d0\uc11c \ub6f0\uc5c8\ub2e4. \uc774\uacf3\uc5d0\uc11c\uc758 \uccab \uc2dc\uc98c\uc5d0\uc11c .319\ub97c \uccd0\ub0c8\uace0, \ub0b4\uc154\ub110 \ub9ac\uadf8 (NL) \ud648\ub7f0, \uc7a5\ud0c0\uc728 \ubd80\ubb38\uc5d0\uc11c \uac01\uac01 1\uc704\ub97c \ucc28\uc9c0\ud588\ub2e4. \ub2f9\uc2dc \ube0c\ub8e8\uc11c\uc2a4\uc640 \ud300 \ub3d9\ub8cc \uc7ad \ub85c, \ud558\ub514 \ub9ac\ucc98\ub4dc\uc2a8, \ub514\ucee8 \ud654\uc774\ud2b8, \uc774 \ub124 \uba85\uc740 \ud568\uaed8 \"\ube45 \ud3ec\"(Big Four)\ub85c \ubd88\ub838\ub2e4. 1882\ub144\uacfc 1883\ub144\uc5d0\ub294 \uac01\uac01 .368, .374\uc758 \ud0c0\uc728\uc744 \uae30\ub85d\ud558\uba70 \ucc98\uc74c\uc73c\ub85c 2\ub144 \uc5f0\uc18d \ud0c0\uaca9\uc655\uc5d0 \uc62c\ub790\ub2e4. \ubc84\ud384\ub85c\uc5d0 \uc788\uc73c\uba74\uc11c \ub0b4\uc154\ub110 \ub9ac\uadf8 \uc7a5\ud0c0\uc728 1\uc704 5\ud68c, \uc548\ud0c0, \ub204\ud0c0\uc218 1\uc704 \uac01\uac01 2\ud68c, 3\ub8e8\ud0c0\uc640 \ud0c0\uc810 1\uc704\ub97c \uac01\uac01 1\ud68c\ub97c \uae30\ub85d\ud588\ub2e4. 1883\ub144\uc5d0\ub294 97\ud0c0\uc810\uc744 \uae30\ub85d\ud574 \uc0c8 \uba54\uc774\uc800 \ub9ac\uadf8 \uae30\ub85d\uc744 \uc138\uc6e0\ub294\ub370, \uc804 \ub144\ub3c4 83\ud0c0\uc810\uc73c\ub85c \uba54\uc774\uc800 \ub9ac\uadf8 \uae30\ub85d\uc744 \uac16\uace0 \uc788\ub358 \ucea1 \uc564\uc2a8\uc774 \ubc14\ub85c \ub2e4\uc74c \ud574\uc5d0 102\ud0c0\uc810\uc744 \uae30\ub85d\ud558\uba70 1\uc704\ub97c \ud0c8\ud658\ud588\ub2e4. 1883\ub144 7\uc6d4 19\uc77c \uacbd\uae30\uc5d0\uc11c 2\ub8e8\ud0c0 2\uac1c\ub97c \ud3ec\ud568\ud574 6\ud0c0\uc218 6\uc548\ud0c0\uc758 \ub9f9\ud0c0\ub97c \ud718\ub458\ub800\uace0, \ud300\uc740 \uc0c1\ub300 \ud544\ub77c\ub378\ud53c\uc544 \ucffc\ucee4\uc2a4\ub97c \uc0c1\ub300\ub85c 25-5\uc758 \ud070 \uc810\uc218\ucc28 \ub300\uc2b9\uc744 \uac70\ub450\uc5c8\ub2e4.", "answer": "\ubc14\uc774\uc2a8\uc2a4", "sentence": "\uc0c1\ub300\uc778 \ubc14\uc774\uc2a8\uc2a4 \uc640 \uacc4\uc57d\ud558\uba74\uc11c, \ucc98\uc74c\uc73c\ub85c \uc8fc\uc804\uc73c\ub85c \ubc1c\ub3cb\uc6c0\ud560 \uae30\ud68c\ub97c \uc5bb\uc5c8\ub2e4.", "paragraph_sentence": "1881\ub144 \uc2dc\uc98c\uc5d0 \uc791\ub144\uc5d0 \uc798 \ucce4\uc5c8\ub358 \uc0c1\ub300\uc778 \ubc14\uc774\uc2a8\uc2a4 \uc640 \uacc4\uc57d\ud558\uba74\uc11c, \ucc98\uc74c\uc73c\ub85c \uc8fc\uc804\uc73c\ub85c \ubc1c\ub3cb\uc6c0\ud560 \uae30\ud68c\ub97c \uc5bb\uc5c8\ub2e4. \uc774\ud6c4 1885\ub144 \uc2dc\uc98c \ud6c4 \uc18c\uc18d\ud300\uc774 \uc0ac\ub77c\uc9c0\uae30 \uc804\uae4c\uc9c0 \uacc4\uc18d \uc774\uacf3\uc5d0\uc11c \ub6f0\uc5c8\ub2e4. \uc774\uacf3\uc5d0\uc11c\uc758 \uccab \uc2dc\uc98c\uc5d0\uc11c .319\ub97c \uccd0\ub0c8\uace0, \ub0b4\uc154\ub110 \ub9ac\uadf8 (NL) \ud648\ub7f0, \uc7a5\ud0c0\uc728 \ubd80\ubb38\uc5d0\uc11c \uac01\uac01 1\uc704\ub97c \ucc28\uc9c0\ud588\ub2e4. \ub2f9\uc2dc \ube0c\ub8e8\uc11c\uc2a4\uc640 \ud300 \ub3d9\ub8cc \uc7ad \ub85c, \ud558\ub514 \ub9ac\ucc98\ub4dc\uc2a8, \ub514\ucee8 \ud654\uc774\ud2b8, \uc774 \ub124 \uba85\uc740 \ud568\uaed8 \"\ube45 \ud3ec\"(Big Four)\ub85c \ubd88\ub838\ub2e4. 1882\ub144\uacfc 1883\ub144\uc5d0\ub294 \uac01\uac01 .368, .374\uc758 \ud0c0\uc728\uc744 \uae30\ub85d\ud558\uba70 \ucc98\uc74c\uc73c\ub85c 2\ub144 \uc5f0\uc18d \ud0c0\uaca9\uc655\uc5d0 \uc62c\ub790\ub2e4. \ubc84\ud384\ub85c\uc5d0 \uc788\uc73c\uba74\uc11c \ub0b4\uc154\ub110 \ub9ac\uadf8 \uc7a5\ud0c0\uc728 1\uc704 5\ud68c, \uc548\ud0c0, \ub204\ud0c0\uc218 1\uc704 \uac01\uac01 2\ud68c, 3\ub8e8\ud0c0\uc640 \ud0c0\uc810 1\uc704\ub97c \uac01\uac01 1\ud68c\ub97c \uae30\ub85d\ud588\ub2e4. 1883\ub144\uc5d0\ub294 97\ud0c0\uc810\uc744 \uae30\ub85d\ud574 \uc0c8 \uba54\uc774\uc800 \ub9ac\uadf8 \uae30\ub85d\uc744 \uc138\uc6e0\ub294\ub370, \uc804 \ub144\ub3c4 83\ud0c0\uc810\uc73c\ub85c \uba54\uc774\uc800 \ub9ac\uadf8 \uae30\ub85d\uc744 \uac16\uace0 \uc788\ub358 \ucea1 \uc564\uc2a8\uc774 \ubc14\ub85c \ub2e4\uc74c \ud574\uc5d0 102\ud0c0\uc810\uc744 \uae30\ub85d\ud558\uba70 1\uc704\ub97c \ud0c8\ud658\ud588\ub2e4. 1883\ub144 7\uc6d4 19\uc77c \uacbd\uae30\uc5d0\uc11c 2\ub8e8\ud0c0 2\uac1c\ub97c \ud3ec\ud568\ud574 6\ud0c0\uc218 6\uc548\ud0c0\uc758 \ub9f9\ud0c0\ub97c \ud718\ub458\ub800\uace0, \ud300\uc740 \uc0c1\ub300 \ud544\ub77c\ub378\ud53c\uc544 \ucffc\ucee4\uc2a4\ub97c \uc0c1\ub300\ub85c 25-5\uc758 \ud070 \uc810\uc218\ucc28 \ub300\uc2b9\uc744 \uac70\ub450\uc5c8\ub2e4.", "paragraph_answer": "1881\ub144 \uc2dc\uc98c\uc5d0 \uc791\ub144\uc5d0 \uc798 \ucce4\uc5c8\ub358 \uc0c1\ub300\uc778 \ubc14\uc774\uc2a8\uc2a4 \uc640 \uacc4\uc57d\ud558\uba74\uc11c, \ucc98\uc74c\uc73c\ub85c \uc8fc\uc804\uc73c\ub85c \ubc1c\ub3cb\uc6c0\ud560 \uae30\ud68c\ub97c \uc5bb\uc5c8\ub2e4. \uc774\ud6c4 1885\ub144 \uc2dc\uc98c \ud6c4 \uc18c\uc18d\ud300\uc774 \uc0ac\ub77c\uc9c0\uae30 \uc804\uae4c\uc9c0 \uacc4\uc18d \uc774\uacf3\uc5d0\uc11c \ub6f0\uc5c8\ub2e4. \uc774\uacf3\uc5d0\uc11c\uc758 \uccab \uc2dc\uc98c\uc5d0\uc11c .319\ub97c \uccd0\ub0c8\uace0, \ub0b4\uc154\ub110 \ub9ac\uadf8 (NL) \ud648\ub7f0, \uc7a5\ud0c0\uc728 \ubd80\ubb38\uc5d0\uc11c \uac01\uac01 1\uc704\ub97c \ucc28\uc9c0\ud588\ub2e4. \ub2f9\uc2dc \ube0c\ub8e8\uc11c\uc2a4\uc640 \ud300 \ub3d9\ub8cc \uc7ad \ub85c, \ud558\ub514 \ub9ac\ucc98\ub4dc\uc2a8, \ub514\ucee8 \ud654\uc774\ud2b8, \uc774 \ub124 \uba85\uc740 \ud568\uaed8 \"\ube45 \ud3ec\"(Big Four)\ub85c \ubd88\ub838\ub2e4. 1882\ub144\uacfc 1883\ub144\uc5d0\ub294 \uac01\uac01 .368, .374\uc758 \ud0c0\uc728\uc744 \uae30\ub85d\ud558\uba70 \ucc98\uc74c\uc73c\ub85c 2\ub144 \uc5f0\uc18d \ud0c0\uaca9\uc655\uc5d0 \uc62c\ub790\ub2e4. \ubc84\ud384\ub85c\uc5d0 \uc788\uc73c\uba74\uc11c \ub0b4\uc154\ub110 \ub9ac\uadf8 \uc7a5\ud0c0\uc728 1\uc704 5\ud68c, \uc548\ud0c0, \ub204\ud0c0\uc218 1\uc704 \uac01\uac01 2\ud68c, 3\ub8e8\ud0c0\uc640 \ud0c0\uc810 1\uc704\ub97c \uac01\uac01 1\ud68c\ub97c \uae30\ub85d\ud588\ub2e4. 1883\ub144\uc5d0\ub294 97\ud0c0\uc810\uc744 \uae30\ub85d\ud574 \uc0c8 \uba54\uc774\uc800 \ub9ac\uadf8 \uae30\ub85d\uc744 \uc138\uc6e0\ub294\ub370, \uc804 \ub144\ub3c4 83\ud0c0\uc810\uc73c\ub85c \uba54\uc774\uc800 \ub9ac\uadf8 \uae30\ub85d\uc744 \uac16\uace0 \uc788\ub358 \ucea1 \uc564\uc2a8\uc774 \ubc14\ub85c \ub2e4\uc74c \ud574\uc5d0 102\ud0c0\uc810\uc744 \uae30\ub85d\ud558\uba70 1\uc704\ub97c \ud0c8\ud658\ud588\ub2e4. 1883\ub144 7\uc6d4 19\uc77c \uacbd\uae30\uc5d0\uc11c 2\ub8e8\ud0c0 2\uac1c\ub97c \ud3ec\ud568\ud574 6\ud0c0\uc218 6\uc548\ud0c0\uc758 \ub9f9\ud0c0\ub97c \ud718\ub458\ub800\uace0, \ud300\uc740 \uc0c1\ub300 \ud544\ub77c\ub378\ud53c\uc544 \ucffc\ucee4\uc2a4\ub97c \uc0c1\ub300\ub85c 25-5\uc758 \ud070 \uc810\uc218\ucc28 \ub300\uc2b9\uc744 \uac70\ub450\uc5c8\ub2e4.", "sentence_answer": "\uc0c1\ub300\uc778 \ubc14\uc774\uc2a8\uc2a4 \uc640 \uacc4\uc57d\ud558\uba74\uc11c, \ucc98\uc74c\uc73c\ub85c \uc8fc\uc804\uc73c\ub85c \ubc1c\ub3cb\uc6c0\ud560 \uae30\ud68c\ub97c \uc5bb\uc5c8\ub2e4.", "paragraph_id": "6486767-0-0", "paragraph_question": "question: \ube0c\ub8e8\uc11c\uc2a4\ub294 1881\ub144 \uc5b4\ub5a4 \ud300\uacfc \uacc4\uc57d\ud588\ub294\uac00?, context: 1881\ub144 \uc2dc\uc98c\uc5d0 \uc791\ub144\uc5d0 \uc798 \ucce4\uc5c8\ub358 \uc0c1\ub300\uc778 \ubc14\uc774\uc2a8\uc2a4\uc640 \uacc4\uc57d\ud558\uba74\uc11c, \ucc98\uc74c\uc73c\ub85c \uc8fc\uc804\uc73c\ub85c \ubc1c\ub3cb\uc6c0\ud560 \uae30\ud68c\ub97c \uc5bb\uc5c8\ub2e4. \uc774\ud6c4 1885\ub144 \uc2dc\uc98c \ud6c4 \uc18c\uc18d\ud300\uc774 \uc0ac\ub77c\uc9c0\uae30 \uc804\uae4c\uc9c0 \uacc4\uc18d \uc774\uacf3\uc5d0\uc11c \ub6f0\uc5c8\ub2e4. \uc774\uacf3\uc5d0\uc11c\uc758 \uccab \uc2dc\uc98c\uc5d0\uc11c .319\ub97c \uccd0\ub0c8\uace0, \ub0b4\uc154\ub110 \ub9ac\uadf8 (NL) \ud648\ub7f0, \uc7a5\ud0c0\uc728 \ubd80\ubb38\uc5d0\uc11c \uac01\uac01 1\uc704\ub97c \ucc28\uc9c0\ud588\ub2e4. \ub2f9\uc2dc \ube0c\ub8e8\uc11c\uc2a4\uc640 \ud300 \ub3d9\ub8cc \uc7ad \ub85c, \ud558\ub514 \ub9ac\ucc98\ub4dc\uc2a8, \ub514\ucee8 \ud654\uc774\ud2b8, \uc774 \ub124 \uba85\uc740 \ud568\uaed8 \"\ube45 \ud3ec\"(Big Four)\ub85c \ubd88\ub838\ub2e4. 1882\ub144\uacfc 1883\ub144\uc5d0\ub294 \uac01\uac01 .368, .374\uc758 \ud0c0\uc728\uc744 \uae30\ub85d\ud558\uba70 \ucc98\uc74c\uc73c\ub85c 2\ub144 \uc5f0\uc18d \ud0c0\uaca9\uc655\uc5d0 \uc62c\ub790\ub2e4. \ubc84\ud384\ub85c\uc5d0 \uc788\uc73c\uba74\uc11c \ub0b4\uc154\ub110 \ub9ac\uadf8 \uc7a5\ud0c0\uc728 1\uc704 5\ud68c, \uc548\ud0c0, \ub204\ud0c0\uc218 1\uc704 \uac01\uac01 2\ud68c, 3\ub8e8\ud0c0\uc640 \ud0c0\uc810 1\uc704\ub97c \uac01\uac01 1\ud68c\ub97c \uae30\ub85d\ud588\ub2e4. 1883\ub144\uc5d0\ub294 97\ud0c0\uc810\uc744 \uae30\ub85d\ud574 \uc0c8 \uba54\uc774\uc800 \ub9ac\uadf8 \uae30\ub85d\uc744 \uc138\uc6e0\ub294\ub370, \uc804 \ub144\ub3c4 83\ud0c0\uc810\uc73c\ub85c \uba54\uc774\uc800 \ub9ac\uadf8 \uae30\ub85d\uc744 \uac16\uace0 \uc788\ub358 \ucea1 \uc564\uc2a8\uc774 \ubc14\ub85c \ub2e4\uc74c \ud574\uc5d0 102\ud0c0\uc810\uc744 \uae30\ub85d\ud558\uba70 1\uc704\ub97c \ud0c8\ud658\ud588\ub2e4. 1883\ub144 7\uc6d4 19\uc77c \uacbd\uae30\uc5d0\uc11c 2\ub8e8\ud0c0 2\uac1c\ub97c \ud3ec\ud568\ud574 6\ud0c0\uc218 6\uc548\ud0c0\uc758 \ub9f9\ud0c0\ub97c \ud718\ub458\ub800\uace0, \ud300\uc740 \uc0c1\ub300 \ud544\ub77c\ub378\ud53c\uc544 \ucffc\ucee4\uc2a4\ub97c \uc0c1\ub300\ub85c 25-5\uc758 \ud070 \uc810\uc218\ucc28 \ub300\uc2b9\uc744 \uac70\ub450\uc5c8\ub2e4."} {"question": "5\uce35 \uac1d\uc2e4\uc5d0 \uba38\ubb3c\ub358 \ud544\ub9ac\ud540 \uac00\uc218 \ubd80\ubd80\ub294 \uc5b4\ub514\ub85c \uc774\ub3d9\ud558\uc600\ub098?", "paragraph": "\uc774\ub0a0 \uc624\ud6c4\uc5d4 \ub2e4\uce58\uace0 \uc0ac\uace0 \ud604\uc7a5\uc5d0\uc11c \uac00\uae4c\uc2a4\ub85c \uad6c\uc870\ub41c 4.5t \ud654\ubb3c\ucc28 \uae30\uc0ac \ucd5c\ubaa8\uc528\ub3c4 \ud720\uccb4\uc5b4\ub97c \ud0c0\uace0 \uc99d\uc778\uc73c\ub85c \ucd9c\uc11d\ud588\ub2e4. \ucd5c\uc528\ub294 \uc0ac\uace0 \ub2f9\uc2dc \ub9e4\uc810\uc5d0\uc11c \ub5a1\uad6d\uc744 \uba39\uae30 \uc704\ud574 \ub728\uac70\uc6b4 \ubb3c\uc744 \ubd93\ub358 \ub3c4\uc911 \ubc30\uac00 \uae30\uc6b8\uc5b4\uc838 \ubb3c\ud1b5\uc774 \uc4f0\ub7ec\uc9c0\ub294 \ubc14\ub78c\uc5d0 \ub2e4\ub9ac\uc5d0 \ud654\uc0c1\uc744 \uc785\uc5c8\ub2e4. \ucd5c\uc528\ub294 \"\uac11\uc790\uae30 \uc624\ub978\ucabd\uc73c\ub85c \ubc30\uac00 \uae30\uc6b8\uba74\uc11c \uc544\ub798\ucabd\uc5d0\uc11c '\uaf5d\uaf5d\uaf5d'\ud558\ub294 \uc18c\ub9ac\uac00 \ub4e4\ub838\ub2e4\"\uba70 \"\ubc30 \ubc11 \ucabd\uc778 \uac83 \uac19\uc558\ub294\ub370 \ud654\ubb3c\uc774 \uc3e0\ub9ac\ub294 \uc18c\ub9ac \uac19\uc558\ub2e4\"\uace0 \ub9d0\ud588\ub2e4. \uadf8\ub294 \ud0c8\ucd9c \ub2f9\uc2dc \uc0c1\ud669\uc5d0 \ub300\ud574 \"\uc606\uc5d0 \uc788\ub358 \uc5ec\ud559\uc0dd\uc5d0\uac8c \uad6c\uba85\uc870\ub07c\ub97c \ubc97\uc5b4\uc918 \uad6c\uba85\uc870\ub07c\ub3c4 \uc5c6\ub294 \uc0c1\ud669\uc774\uc5c8\uc9c0\ub9cc, \ubc30\uac00 \uae08\ubc29 \uce68\ubab0\ud560 \uac83 \uac19\uc544 \ubc14\ub2e4\ub85c \ub6f0\uc5b4\ub4e4\uc5c8\ub2e4\"\uba70 \"\uadf8\ub7f0\ub370 \ub0b4\uac00 \uad6c\uba85\uc870\ub07c\ub97c \ubc97\uc5b4\uc900 \uc5ec\ud559\uc0dd\uc740 \ubb34\uc11c\uc6e0\ub294\uc9c0 \ub450 \uc190\uc744 \ub5a8\uba74\uc11c \ub6f0\uc5b4\ub0b4\ub9ac\uc9c0 \ubabb\ud588\ub2e4. \uadf8\ub54c \uadf8 \ub208\ube5b\uc744 \uc78a\uc744 \uc218\uac00 \uc5c6\ub2e4\"\uace0 \ud68c\uc0c1\ud588\ub2e4. 5\uce35 \uac1d\uc2e4\uc5d0 \uba38\ubb3c\ub2e4 \uad6c\uc870\ub41c \ud544\ub9ac\ud540 \uac00\uc218 \ubd80\ubd80\ub294 \"\uc870\ud0c0\uc2e4\ub85c \uac00\uba74 \uc548\uc804\ud55c \ucd9c\uad6c\uac00 \uc788\ub2e4\ub294 \uc0dd\uac01\uc5d0 \ubc30\uac00 \uae30\uc6b8\uc790 \uc870\ud0c0\uc2e4\ub85c \uac14\ub2e4\"\uba70 \"\uc120\uc6d0 1\uba85\uacfc \ud568\uaed8 \ub77c\uc774\ud504\ubcf4\ud2b8\ub97c \ud3bc\uce58\ub824\uace0 \ud588\uc9c0\ub9cc, \uc791\ub3d9\ud558\uc9c0 \uc54a\uc558\ub2e4\"\uace0 \uc9c4\uc220\ud588\ub2e4.", "answer": "\uc870\ud0c0\uc2e4", "sentence": "\" \uc870\ud0c0\uc2e4 \ub85c \uac00\uba74 \uc548\uc804\ud55c \ucd9c\uad6c\uac00 \uc788\ub2e4\ub294 \uc0dd\uac01\uc5d0 \ubc30\uac00 \uae30\uc6b8\uc790 \uc870\ud0c0\uc2e4\ub85c \uac14\ub2e4\"\uba70 \"\uc120\uc6d0 1\uba85\uacfc \ud568\uaed8 \ub77c\uc774\ud504\ubcf4\ud2b8\ub97c \ud3bc\uce58\ub824\uace0 \ud588\uc9c0\ub9cc, \uc791\ub3d9\ud558\uc9c0 \uc54a\uc558\ub2e4\"\uace0 \uc9c4\uc220\ud588\ub2e4.", "paragraph_sentence": "\uc774\ub0a0 \uc624\ud6c4\uc5d4 \ub2e4\uce58\uace0 \uc0ac\uace0 \ud604\uc7a5\uc5d0\uc11c \uac00\uae4c\uc2a4\ub85c \uad6c\uc870\ub41c 4.5t \ud654\ubb3c\ucc28 \uae30\uc0ac \ucd5c\ubaa8\uc528\ub3c4 \ud720\uccb4\uc5b4\ub97c \ud0c0\uace0 \uc99d\uc778\uc73c\ub85c \ucd9c\uc11d\ud588\ub2e4. \ucd5c\uc528\ub294 \uc0ac\uace0 \ub2f9\uc2dc \ub9e4\uc810\uc5d0\uc11c \ub5a1\uad6d\uc744 \uba39\uae30 \uc704\ud574 \ub728\uac70\uc6b4 \ubb3c\uc744 \ubd93\ub358 \ub3c4\uc911 \ubc30\uac00 \uae30\uc6b8\uc5b4\uc838 \ubb3c\ud1b5\uc774 \uc4f0\ub7ec\uc9c0\ub294 \ubc14\ub78c\uc5d0 \ub2e4\ub9ac\uc5d0 \ud654\uc0c1\uc744 \uc785\uc5c8\ub2e4. \ucd5c\uc528\ub294 \"\uac11\uc790\uae30 \uc624\ub978\ucabd\uc73c\ub85c \ubc30\uac00 \uae30\uc6b8\uba74\uc11c \uc544\ub798\ucabd\uc5d0\uc11c '\uaf5d\uaf5d\uaf5d'\ud558\ub294 \uc18c\ub9ac\uac00 \ub4e4\ub838\ub2e4\"\uba70 \"\ubc30 \ubc11 \ucabd\uc778 \uac83 \uac19\uc558\ub294\ub370 \ud654\ubb3c\uc774 \uc3e0\ub9ac\ub294 \uc18c\ub9ac \uac19\uc558\ub2e4\"\uace0 \ub9d0\ud588\ub2e4. \uadf8\ub294 \ud0c8\ucd9c \ub2f9\uc2dc \uc0c1\ud669\uc5d0 \ub300\ud574 \"\uc606\uc5d0 \uc788\ub358 \uc5ec\ud559\uc0dd\uc5d0\uac8c \uad6c\uba85\uc870\ub07c\ub97c \ubc97\uc5b4\uc918 \uad6c\uba85\uc870\ub07c\ub3c4 \uc5c6\ub294 \uc0c1\ud669\uc774\uc5c8\uc9c0\ub9cc, \ubc30\uac00 \uae08\ubc29 \uce68\ubab0\ud560 \uac83 \uac19\uc544 \ubc14\ub2e4\ub85c \ub6f0\uc5b4\ub4e4\uc5c8\ub2e4\"\uba70 \" \uadf8\ub7f0\ub370 \ub0b4\uac00 \uad6c\uba85\uc870\ub07c\ub97c \ubc97\uc5b4\uc900 \uc5ec\ud559\uc0dd\uc740 \ubb34\uc11c\uc6e0\ub294\uc9c0 \ub450 \uc190\uc744 \ub5a8\uba74\uc11c \ub6f0\uc5b4\ub0b4\ub9ac\uc9c0 \ubabb\ud588\ub2e4. \uadf8\ub54c \uadf8 \ub208\ube5b\uc744 \uc78a\uc744 \uc218\uac00 \uc5c6\ub2e4\"\uace0 \ud68c\uc0c1\ud588\ub2e4. 5\uce35 \uac1d\uc2e4\uc5d0 \uba38\ubb3c\ub2e4 \uad6c\uc870\ub41c \ud544\ub9ac\ud540 \uac00\uc218 \ubd80\ubd80\ub294 \" \uc870\ud0c0\uc2e4 \ub85c \uac00\uba74 \uc548\uc804\ud55c \ucd9c\uad6c\uac00 \uc788\ub2e4\ub294 \uc0dd\uac01\uc5d0 \ubc30\uac00 \uae30\uc6b8\uc790 \uc870\ud0c0\uc2e4\ub85c \uac14\ub2e4\"\uba70 \"\uc120\uc6d0 1\uba85\uacfc \ud568\uaed8 \ub77c\uc774\ud504\ubcf4\ud2b8\ub97c \ud3bc\uce58\ub824\uace0 \ud588\uc9c0\ub9cc, \uc791\ub3d9\ud558\uc9c0 \uc54a\uc558\ub2e4\"\uace0 \uc9c4\uc220\ud588\ub2e4. ", "paragraph_answer": "\uc774\ub0a0 \uc624\ud6c4\uc5d4 \ub2e4\uce58\uace0 \uc0ac\uace0 \ud604\uc7a5\uc5d0\uc11c \uac00\uae4c\uc2a4\ub85c \uad6c\uc870\ub41c 4.5t \ud654\ubb3c\ucc28 \uae30\uc0ac \ucd5c\ubaa8\uc528\ub3c4 \ud720\uccb4\uc5b4\ub97c \ud0c0\uace0 \uc99d\uc778\uc73c\ub85c \ucd9c\uc11d\ud588\ub2e4. \ucd5c\uc528\ub294 \uc0ac\uace0 \ub2f9\uc2dc \ub9e4\uc810\uc5d0\uc11c \ub5a1\uad6d\uc744 \uba39\uae30 \uc704\ud574 \ub728\uac70\uc6b4 \ubb3c\uc744 \ubd93\ub358 \ub3c4\uc911 \ubc30\uac00 \uae30\uc6b8\uc5b4\uc838 \ubb3c\ud1b5\uc774 \uc4f0\ub7ec\uc9c0\ub294 \ubc14\ub78c\uc5d0 \ub2e4\ub9ac\uc5d0 \ud654\uc0c1\uc744 \uc785\uc5c8\ub2e4. \ucd5c\uc528\ub294 \"\uac11\uc790\uae30 \uc624\ub978\ucabd\uc73c\ub85c \ubc30\uac00 \uae30\uc6b8\uba74\uc11c \uc544\ub798\ucabd\uc5d0\uc11c '\uaf5d\uaf5d\uaf5d'\ud558\ub294 \uc18c\ub9ac\uac00 \ub4e4\ub838\ub2e4\"\uba70 \"\ubc30 \ubc11 \ucabd\uc778 \uac83 \uac19\uc558\ub294\ub370 \ud654\ubb3c\uc774 \uc3e0\ub9ac\ub294 \uc18c\ub9ac \uac19\uc558\ub2e4\"\uace0 \ub9d0\ud588\ub2e4. \uadf8\ub294 \ud0c8\ucd9c \ub2f9\uc2dc \uc0c1\ud669\uc5d0 \ub300\ud574 \"\uc606\uc5d0 \uc788\ub358 \uc5ec\ud559\uc0dd\uc5d0\uac8c \uad6c\uba85\uc870\ub07c\ub97c \ubc97\uc5b4\uc918 \uad6c\uba85\uc870\ub07c\ub3c4 \uc5c6\ub294 \uc0c1\ud669\uc774\uc5c8\uc9c0\ub9cc, \ubc30\uac00 \uae08\ubc29 \uce68\ubab0\ud560 \uac83 \uac19\uc544 \ubc14\ub2e4\ub85c \ub6f0\uc5b4\ub4e4\uc5c8\ub2e4\"\uba70 \"\uadf8\ub7f0\ub370 \ub0b4\uac00 \uad6c\uba85\uc870\ub07c\ub97c \ubc97\uc5b4\uc900 \uc5ec\ud559\uc0dd\uc740 \ubb34\uc11c\uc6e0\ub294\uc9c0 \ub450 \uc190\uc744 \ub5a8\uba74\uc11c \ub6f0\uc5b4\ub0b4\ub9ac\uc9c0 \ubabb\ud588\ub2e4. \uadf8\ub54c \uadf8 \ub208\ube5b\uc744 \uc78a\uc744 \uc218\uac00 \uc5c6\ub2e4\"\uace0 \ud68c\uc0c1\ud588\ub2e4. 5\uce35 \uac1d\uc2e4\uc5d0 \uba38\ubb3c\ub2e4 \uad6c\uc870\ub41c \ud544\ub9ac\ud540 \uac00\uc218 \ubd80\ubd80\ub294 \" \uc870\ud0c0\uc2e4 \ub85c \uac00\uba74 \uc548\uc804\ud55c \ucd9c\uad6c\uac00 \uc788\ub2e4\ub294 \uc0dd\uac01\uc5d0 \ubc30\uac00 \uae30\uc6b8\uc790 \uc870\ud0c0\uc2e4\ub85c \uac14\ub2e4\"\uba70 \"\uc120\uc6d0 1\uba85\uacfc \ud568\uaed8 \ub77c\uc774\ud504\ubcf4\ud2b8\ub97c \ud3bc\uce58\ub824\uace0 \ud588\uc9c0\ub9cc, \uc791\ub3d9\ud558\uc9c0 \uc54a\uc558\ub2e4\"\uace0 \uc9c4\uc220\ud588\ub2e4.", "sentence_answer": "\" \uc870\ud0c0\uc2e4 \ub85c \uac00\uba74 \uc548\uc804\ud55c \ucd9c\uad6c\uac00 \uc788\ub2e4\ub294 \uc0dd\uac01\uc5d0 \ubc30\uac00 \uae30\uc6b8\uc790 \uc870\ud0c0\uc2e4\ub85c \uac14\ub2e4\"\uba70 \"\uc120\uc6d0 1\uba85\uacfc \ud568\uaed8 \ub77c\uc774\ud504\ubcf4\ud2b8\ub97c \ud3bc\uce58\ub824\uace0 \ud588\uc9c0\ub9cc, \uc791\ub3d9\ud558\uc9c0 \uc54a\uc558\ub2e4\"\uace0 \uc9c4\uc220\ud588\ub2e4.", "paragraph_id": "6518129-10-1", "paragraph_question": "question: 5\uce35 \uac1d\uc2e4\uc5d0 \uba38\ubb3c\ub358 \ud544\ub9ac\ud540 \uac00\uc218 \ubd80\ubd80\ub294 \uc5b4\ub514\ub85c \uc774\ub3d9\ud558\uc600\ub098?, context: \uc774\ub0a0 \uc624\ud6c4\uc5d4 \ub2e4\uce58\uace0 \uc0ac\uace0 \ud604\uc7a5\uc5d0\uc11c \uac00\uae4c\uc2a4\ub85c \uad6c\uc870\ub41c 4.5t \ud654\ubb3c\ucc28 \uae30\uc0ac \ucd5c\ubaa8\uc528\ub3c4 \ud720\uccb4\uc5b4\ub97c \ud0c0\uace0 \uc99d\uc778\uc73c\ub85c \ucd9c\uc11d\ud588\ub2e4. \ucd5c\uc528\ub294 \uc0ac\uace0 \ub2f9\uc2dc \ub9e4\uc810\uc5d0\uc11c \ub5a1\uad6d\uc744 \uba39\uae30 \uc704\ud574 \ub728\uac70\uc6b4 \ubb3c\uc744 \ubd93\ub358 \ub3c4\uc911 \ubc30\uac00 \uae30\uc6b8\uc5b4\uc838 \ubb3c\ud1b5\uc774 \uc4f0\ub7ec\uc9c0\ub294 \ubc14\ub78c\uc5d0 \ub2e4\ub9ac\uc5d0 \ud654\uc0c1\uc744 \uc785\uc5c8\ub2e4. \ucd5c\uc528\ub294 \"\uac11\uc790\uae30 \uc624\ub978\ucabd\uc73c\ub85c \ubc30\uac00 \uae30\uc6b8\uba74\uc11c \uc544\ub798\ucabd\uc5d0\uc11c '\uaf5d\uaf5d\uaf5d'\ud558\ub294 \uc18c\ub9ac\uac00 \ub4e4\ub838\ub2e4\"\uba70 \"\ubc30 \ubc11 \ucabd\uc778 \uac83 \uac19\uc558\ub294\ub370 \ud654\ubb3c\uc774 \uc3e0\ub9ac\ub294 \uc18c\ub9ac \uac19\uc558\ub2e4\"\uace0 \ub9d0\ud588\ub2e4. \uadf8\ub294 \ud0c8\ucd9c \ub2f9\uc2dc \uc0c1\ud669\uc5d0 \ub300\ud574 \"\uc606\uc5d0 \uc788\ub358 \uc5ec\ud559\uc0dd\uc5d0\uac8c \uad6c\uba85\uc870\ub07c\ub97c \ubc97\uc5b4\uc918 \uad6c\uba85\uc870\ub07c\ub3c4 \uc5c6\ub294 \uc0c1\ud669\uc774\uc5c8\uc9c0\ub9cc, \ubc30\uac00 \uae08\ubc29 \uce68\ubab0\ud560 \uac83 \uac19\uc544 \ubc14\ub2e4\ub85c \ub6f0\uc5b4\ub4e4\uc5c8\ub2e4\"\uba70 \"\uadf8\ub7f0\ub370 \ub0b4\uac00 \uad6c\uba85\uc870\ub07c\ub97c \ubc97\uc5b4\uc900 \uc5ec\ud559\uc0dd\uc740 \ubb34\uc11c\uc6e0\ub294\uc9c0 \ub450 \uc190\uc744 \ub5a8\uba74\uc11c \ub6f0\uc5b4\ub0b4\ub9ac\uc9c0 \ubabb\ud588\ub2e4. \uadf8\ub54c \uadf8 \ub208\ube5b\uc744 \uc78a\uc744 \uc218\uac00 \uc5c6\ub2e4\"\uace0 \ud68c\uc0c1\ud588\ub2e4. 5\uce35 \uac1d\uc2e4\uc5d0 \uba38\ubb3c\ub2e4 \uad6c\uc870\ub41c \ud544\ub9ac\ud540 \uac00\uc218 \ubd80\ubd80\ub294 \"\uc870\ud0c0\uc2e4\ub85c \uac00\uba74 \uc548\uc804\ud55c \ucd9c\uad6c\uac00 \uc788\ub2e4\ub294 \uc0dd\uac01\uc5d0 \ubc30\uac00 \uae30\uc6b8\uc790 \uc870\ud0c0\uc2e4\ub85c \uac14\ub2e4\"\uba70 \"\uc120\uc6d0 1\uba85\uacfc \ud568\uaed8 \ub77c\uc774\ud504\ubcf4\ud2b8\ub97c \ud3bc\uce58\ub824\uace0 \ud588\uc9c0\ub9cc, \uc791\ub3d9\ud558\uc9c0 \uc54a\uc558\ub2e4\"\uace0 \uc9c4\uc220\ud588\ub2e4."} {"question": "\ubcc4\uc77c \uc5c6\uc774 \uc0b0\ub2e4 \uc774\uc804\uc5d0 \uc7a5\uae30\ud558\uc758 \ubc34\ub4dc\ub294 \uc5b4\ub5a4 \ube44\ud310\uc744 \ubc1b\uc544\uc654\ub294\uac00?", "paragraph": "\u300a\uc5f0\ud569\ub274\uc2a4\u300b \uc774\uc740\uc815 \uae30\uc790\uc5d0 \ub530\ub974\uba74, \uc804\uccb4\uc801\uc73c\ub85c \uc218\ub85d\uace1\ub4e4\uc740 \ub370\ubdd4 \uc2f1\uae00\uc758 \uc0c9\uae54\uc774 \ud655\uc7a5\ub41c \uac83\ub4e4\uc774\ub2e4. \ubd95\uac00\ubd95\uac00\ub808\ucf54\ub4dc\ub294 \"\uc7a5\uae30\ud558 \ud2b9\uc720\uc758 \ub9d0\ud558\ub4ef \ub178\ub798\ud558\ub294 \ubc29\uc2dd, \uac10\uc131\uc801\uc778 \uc61b\ub0a0 \uac00\uc694 \ub290\ub08c\uc5d0 \ubc15\uc790\uac10 \ub118\uce58\ub294 \ud391\ud06c, \uc9c1\uc120\uc801\uc778 \ub85c\ud070\ub864 \ub4f1\uc774 \ub2e4\uc591\ud558\uac8c \uc2dc\ub3c4\ub3fc '\ud37c\ud3ec\uba3c\uc2a4 \uc704\uc8fc\uc758 \ucf54\ubbf9 \ubc34\ub4dc'\ub77c\ub294 \ube44\ud310\uc744 \uae70 \uac83\"\uc774\ub77c\uace0 \ub9d0\ud588\ub2e4. \uc7a5\uae30\ud558\ub294 \"\ub178\ub798\uac00 \uc911\uc2ec\uc774 \ub418\ub294 \ud3ec\ud06c\ub85d \uc74c\ubc18\"\uc774\ub77c\uace0 \uc124\uba85\ud588\ub2e4. \ud558\uc9c0\ub9cc, \uadf8\ub294 \uc778\ud130\ubdf0\uc5d0\uc11c \"\ub531\ud788 \ud1b5\uae30\ud0c0\ub780 \uc545\uae30\uc5d0 \uad49\uc7a5\ud55c \uc560\ucc29\uc774 \uc788\ub2e4\uac70\ub098 \ud3ec\ud06c\ub77c\ub294 \uc7a5\ub974 \uc790\uccb4\uc5d0 \uc560\ucc29\uc774 \uc788\ub294 \uac74 \uc544\ub2c8\uc5c8\uc5b4\uc694. \ucc98\uc74c\uc5d0 \uc800 \ud63c\uc790 \uc74c\ubc18\uc744 \ub2e4 \ub9cc\ub4e4 \uc0dd\uac01\uc73c\ub85c \ubc29\uc5d0\uc11c \uc5f0\uc8fc\ud558\uae30 \uc218\uc6b4 \uac8c \ud1b5\uae30\ud0c0\uc600\ub358 \uac70\uc600\uc8e0. \uc6b0\ub9ac\uc758 \uc7a5\ub974\uac00 \ud3ec\ud06c \ub85d\uc774\ub77c\uace0 \ub9d0\ud558\uace0 \ub2e4\ub2cc \uac83\ub3c4, \ub9cc\ub4e4\ub2e4 \ubcf4\ub2c8\uae4c \uadf8\ub9ac\uace0 \uc885\ud569\uc744 \ud558\ub2e4 \ubcf4\ub2c8\uae4c \uadf8\ub807\uac8c [\ud3ec\ud06c \ub85d\uc774] \ub418\uc5c8\ub358 \uac70\uc8e0.\"\ub77c\uace0 \ub9d0\ud55c \ubc14 \uc788\ub2e4.", "answer": "\ud37c\ud3ec\uba3c\uc2a4 \uc704\uc8fc\uc758 \ucf54\ubbf9 \ubc34\ub4dc", "sentence": "\ubd95\uac00\ubd95\uac00\ub808\ucf54\ub4dc\ub294 \"\uc7a5\uae30\ud558 \ud2b9\uc720\uc758 \ub9d0\ud558\ub4ef \ub178\ub798\ud558\ub294 \ubc29\uc2dd, \uac10\uc131\uc801\uc778 \uc61b\ub0a0 \uac00\uc694 \ub290\ub08c\uc5d0 \ubc15\uc790\uac10 \ub118\uce58\ub294 \ud391\ud06c, \uc9c1\uc120\uc801\uc778 \ub85c\ud070\ub864 \ub4f1\uc774 \ub2e4\uc591\ud558\uac8c \uc2dc\ub3c4\ub3fc ' \ud37c\ud3ec\uba3c\uc2a4 \uc704\uc8fc\uc758 \ucf54\ubbf9 \ubc34\ub4dc '\ub77c\ub294 \ube44\ud310\uc744 \uae70 \uac83\"\uc774\ub77c\uace0 \ub9d0\ud588\ub2e4.", "paragraph_sentence": "\u300a\uc5f0\ud569\ub274\uc2a4\u300b \uc774\uc740\uc815 \uae30\uc790\uc5d0 \ub530\ub974\uba74, \uc804\uccb4\uc801\uc73c\ub85c \uc218\ub85d\uace1\ub4e4\uc740 \ub370\ubdd4 \uc2f1\uae00\uc758 \uc0c9\uae54\uc774 \ud655\uc7a5\ub41c \uac83\ub4e4\uc774\ub2e4. \ubd95\uac00\ubd95\uac00\ub808\ucf54\ub4dc\ub294 \"\uc7a5\uae30\ud558 \ud2b9\uc720\uc758 \ub9d0\ud558\ub4ef \ub178\ub798\ud558\ub294 \ubc29\uc2dd, \uac10\uc131\uc801\uc778 \uc61b\ub0a0 \uac00\uc694 \ub290\ub08c\uc5d0 \ubc15\uc790\uac10 \ub118\uce58\ub294 \ud391\ud06c, \uc9c1\uc120\uc801\uc778 \ub85c\ud070\ub864 \ub4f1\uc774 \ub2e4\uc591\ud558\uac8c \uc2dc\ub3c4\ub3fc ' \ud37c\ud3ec\uba3c\uc2a4 \uc704\uc8fc\uc758 \ucf54\ubbf9 \ubc34\ub4dc '\ub77c\ub294 \ube44\ud310\uc744 \uae70 \uac83\"\uc774\ub77c\uace0 \ub9d0\ud588\ub2e4. \uc7a5\uae30\ud558\ub294 \"\ub178\ub798\uac00 \uc911\uc2ec\uc774 \ub418\ub294 \ud3ec\ud06c\ub85d \uc74c\ubc18\"\uc774\ub77c\uace0 \uc124\uba85\ud588\ub2e4. \ud558\uc9c0\ub9cc, \uadf8\ub294 \uc778\ud130\ubdf0\uc5d0\uc11c \"\ub531\ud788 \ud1b5\uae30\ud0c0\ub780 \uc545\uae30\uc5d0 \uad49\uc7a5\ud55c \uc560\ucc29\uc774 \uc788\ub2e4\uac70\ub098 \ud3ec\ud06c\ub77c\ub294 \uc7a5\ub974 \uc790\uccb4\uc5d0 \uc560\ucc29\uc774 \uc788\ub294 \uac74 \uc544\ub2c8\uc5c8\uc5b4\uc694. \ucc98\uc74c\uc5d0 \uc800 \ud63c\uc790 \uc74c\ubc18\uc744 \ub2e4 \ub9cc\ub4e4 \uc0dd\uac01\uc73c\ub85c \ubc29\uc5d0\uc11c \uc5f0\uc8fc\ud558\uae30 \uc218\uc6b4 \uac8c \ud1b5\uae30\ud0c0\uc600\ub358 \uac70\uc600\uc8e0. \uc6b0\ub9ac\uc758 \uc7a5\ub974\uac00 \ud3ec\ud06c \ub85d\uc774\ub77c\uace0 \ub9d0\ud558\uace0 \ub2e4\ub2cc \uac83\ub3c4, \ub9cc\ub4e4\ub2e4 \ubcf4\ub2c8\uae4c \uadf8\ub9ac\uace0 \uc885\ud569\uc744 \ud558\ub2e4 \ubcf4\ub2c8\uae4c \uadf8\ub807\uac8c [\ud3ec\ud06c \ub85d\uc774] \ub418\uc5c8\ub358 \uac70\uc8e0. \"\ub77c\uace0 \ub9d0\ud55c \ubc14 \uc788\ub2e4.", "paragraph_answer": "\u300a\uc5f0\ud569\ub274\uc2a4\u300b \uc774\uc740\uc815 \uae30\uc790\uc5d0 \ub530\ub974\uba74, \uc804\uccb4\uc801\uc73c\ub85c \uc218\ub85d\uace1\ub4e4\uc740 \ub370\ubdd4 \uc2f1\uae00\uc758 \uc0c9\uae54\uc774 \ud655\uc7a5\ub41c \uac83\ub4e4\uc774\ub2e4. \ubd95\uac00\ubd95\uac00\ub808\ucf54\ub4dc\ub294 \"\uc7a5\uae30\ud558 \ud2b9\uc720\uc758 \ub9d0\ud558\ub4ef \ub178\ub798\ud558\ub294 \ubc29\uc2dd, \uac10\uc131\uc801\uc778 \uc61b\ub0a0 \uac00\uc694 \ub290\ub08c\uc5d0 \ubc15\uc790\uac10 \ub118\uce58\ub294 \ud391\ud06c, \uc9c1\uc120\uc801\uc778 \ub85c\ud070\ub864 \ub4f1\uc774 \ub2e4\uc591\ud558\uac8c \uc2dc\ub3c4\ub3fc ' \ud37c\ud3ec\uba3c\uc2a4 \uc704\uc8fc\uc758 \ucf54\ubbf9 \ubc34\ub4dc '\ub77c\ub294 \ube44\ud310\uc744 \uae70 \uac83\"\uc774\ub77c\uace0 \ub9d0\ud588\ub2e4. \uc7a5\uae30\ud558\ub294 \"\ub178\ub798\uac00 \uc911\uc2ec\uc774 \ub418\ub294 \ud3ec\ud06c\ub85d \uc74c\ubc18\"\uc774\ub77c\uace0 \uc124\uba85\ud588\ub2e4. \ud558\uc9c0\ub9cc, \uadf8\ub294 \uc778\ud130\ubdf0\uc5d0\uc11c \"\ub531\ud788 \ud1b5\uae30\ud0c0\ub780 \uc545\uae30\uc5d0 \uad49\uc7a5\ud55c \uc560\ucc29\uc774 \uc788\ub2e4\uac70\ub098 \ud3ec\ud06c\ub77c\ub294 \uc7a5\ub974 \uc790\uccb4\uc5d0 \uc560\ucc29\uc774 \uc788\ub294 \uac74 \uc544\ub2c8\uc5c8\uc5b4\uc694. \ucc98\uc74c\uc5d0 \uc800 \ud63c\uc790 \uc74c\ubc18\uc744 \ub2e4 \ub9cc\ub4e4 \uc0dd\uac01\uc73c\ub85c \ubc29\uc5d0\uc11c \uc5f0\uc8fc\ud558\uae30 \uc218\uc6b4 \uac8c \ud1b5\uae30\ud0c0\uc600\ub358 \uac70\uc600\uc8e0. \uc6b0\ub9ac\uc758 \uc7a5\ub974\uac00 \ud3ec\ud06c \ub85d\uc774\ub77c\uace0 \ub9d0\ud558\uace0 \ub2e4\ub2cc \uac83\ub3c4, \ub9cc\ub4e4\ub2e4 \ubcf4\ub2c8\uae4c \uadf8\ub9ac\uace0 \uc885\ud569\uc744 \ud558\ub2e4 \ubcf4\ub2c8\uae4c \uadf8\ub807\uac8c [\ud3ec\ud06c \ub85d\uc774] \ub418\uc5c8\ub358 \uac70\uc8e0.\"\ub77c\uace0 \ub9d0\ud55c \ubc14 \uc788\ub2e4.", "sentence_answer": "\ubd95\uac00\ubd95\uac00\ub808\ucf54\ub4dc\ub294 \"\uc7a5\uae30\ud558 \ud2b9\uc720\uc758 \ub9d0\ud558\ub4ef \ub178\ub798\ud558\ub294 \ubc29\uc2dd, \uac10\uc131\uc801\uc778 \uc61b\ub0a0 \uac00\uc694 \ub290\ub08c\uc5d0 \ubc15\uc790\uac10 \ub118\uce58\ub294 \ud391\ud06c, \uc9c1\uc120\uc801\uc778 \ub85c\ud070\ub864 \ub4f1\uc774 \ub2e4\uc591\ud558\uac8c \uc2dc\ub3c4\ub3fc ' \ud37c\ud3ec\uba3c\uc2a4 \uc704\uc8fc\uc758 \ucf54\ubbf9 \ubc34\ub4dc '\ub77c\ub294 \ube44\ud310\uc744 \uae70 \uac83\"\uc774\ub77c\uace0 \ub9d0\ud588\ub2e4.", "paragraph_id": "6657153-7-0", "paragraph_question": "question: \ubcc4\uc77c \uc5c6\uc774 \uc0b0\ub2e4 \uc774\uc804\uc5d0 \uc7a5\uae30\ud558\uc758 \ubc34\ub4dc\ub294 \uc5b4\ub5a4 \ube44\ud310\uc744 \ubc1b\uc544\uc654\ub294\uac00?, context: \u300a\uc5f0\ud569\ub274\uc2a4\u300b \uc774\uc740\uc815 \uae30\uc790\uc5d0 \ub530\ub974\uba74, \uc804\uccb4\uc801\uc73c\ub85c \uc218\ub85d\uace1\ub4e4\uc740 \ub370\ubdd4 \uc2f1\uae00\uc758 \uc0c9\uae54\uc774 \ud655\uc7a5\ub41c \uac83\ub4e4\uc774\ub2e4. \ubd95\uac00\ubd95\uac00\ub808\ucf54\ub4dc\ub294 \"\uc7a5\uae30\ud558 \ud2b9\uc720\uc758 \ub9d0\ud558\ub4ef \ub178\ub798\ud558\ub294 \ubc29\uc2dd, \uac10\uc131\uc801\uc778 \uc61b\ub0a0 \uac00\uc694 \ub290\ub08c\uc5d0 \ubc15\uc790\uac10 \ub118\uce58\ub294 \ud391\ud06c, \uc9c1\uc120\uc801\uc778 \ub85c\ud070\ub864 \ub4f1\uc774 \ub2e4\uc591\ud558\uac8c \uc2dc\ub3c4\ub3fc '\ud37c\ud3ec\uba3c\uc2a4 \uc704\uc8fc\uc758 \ucf54\ubbf9 \ubc34\ub4dc'\ub77c\ub294 \ube44\ud310\uc744 \uae70 \uac83\"\uc774\ub77c\uace0 \ub9d0\ud588\ub2e4. \uc7a5\uae30\ud558\ub294 \"\ub178\ub798\uac00 \uc911\uc2ec\uc774 \ub418\ub294 \ud3ec\ud06c\ub85d \uc74c\ubc18\"\uc774\ub77c\uace0 \uc124\uba85\ud588\ub2e4. \ud558\uc9c0\ub9cc, \uadf8\ub294 \uc778\ud130\ubdf0\uc5d0\uc11c \"\ub531\ud788 \ud1b5\uae30\ud0c0\ub780 \uc545\uae30\uc5d0 \uad49\uc7a5\ud55c \uc560\ucc29\uc774 \uc788\ub2e4\uac70\ub098 \ud3ec\ud06c\ub77c\ub294 \uc7a5\ub974 \uc790\uccb4\uc5d0 \uc560\ucc29\uc774 \uc788\ub294 \uac74 \uc544\ub2c8\uc5c8\uc5b4\uc694. \ucc98\uc74c\uc5d0 \uc800 \ud63c\uc790 \uc74c\ubc18\uc744 \ub2e4 \ub9cc\ub4e4 \uc0dd\uac01\uc73c\ub85c \ubc29\uc5d0\uc11c \uc5f0\uc8fc\ud558\uae30 \uc218\uc6b4 \uac8c \ud1b5\uae30\ud0c0\uc600\ub358 \uac70\uc600\uc8e0. \uc6b0\ub9ac\uc758 \uc7a5\ub974\uac00 \ud3ec\ud06c \ub85d\uc774\ub77c\uace0 \ub9d0\ud558\uace0 \ub2e4\ub2cc \uac83\ub3c4, \ub9cc\ub4e4\ub2e4 \ubcf4\ub2c8\uae4c \uadf8\ub9ac\uace0 \uc885\ud569\uc744 \ud558\ub2e4 \ubcf4\ub2c8\uae4c \uadf8\ub807\uac8c [\ud3ec\ud06c \ub85d\uc774] \ub418\uc5c8\ub358 \uac70\uc8e0.\"\ub77c\uace0 \ub9d0\ud55c \ubc14 \uc788\ub2e4."} {"question": "\ucc30\uc2a4 \ud37c\uc2a4, \uc904\ub9ac\uc548 \ubaa8\ub85c, \uc564\ub4dc\ub8e8 \ud55c\uc13c \ub4f1\uc774 1999\ub144\ubd80\ud130 \ubc1c\ud589\ud55c \uc2dc\uc0ac\ube44\ud3c9\uc2e0\ubb38\uc758 \uc774\ub984\uc740?", "paragraph": "1990\ub144\ub300 \ub9d0\uc5d0 \uc2dc\ub4dc\ub2c8 \ub300\ud559\uad50\uc5d0 \uc7ac\ud559 \uc911\uc774\uc5c8\ub358 \ucc30\uc2a4 \ud37c\uc2a4, \ub3c4\ubbf8\ub2c9 \ub098\uc787, \uc904\ub9ac\uc548 \ubaa8\ub85c, \ud06c\ub808\uc775 \ub93c\uce74\uc140\uc774 \ud559\ub0b4\uc2e0\ubb38\uc5d0 \uc2dc\uc0ac\ud48d\uc790\uae30\uc0ac\ub97c \uc62c\ub824 \ud070 \ubc18\ud5a5\uc744 \uc5bb\uc5c8\uc73c\uba70, \uc774\ud6c4\uc5d0 \ucc44\uc2a4 \ub9ac\uce58\uc544\ub378\ub85c, \ud06c\ub9ac\uc2a4 \ud14c\uc77c\ub7ec, \uc564\ub4dc\ub8e8 \ud55c\uc13c\uc744 \uc601\uc785\ud558\uc5ec 1999\ub144\ubd80\ud130 \uc2dc\uc0ac\ube44\ud3c9\uc2e0\ubb38 'The Chaser'\ub97c \ubc1c\ud589\ud558\uae30 \uc2dc\uc791\ud588\ub2e4. \ucd08\ubc18\uc5d0\ub294 \ube44\uc8fc\ub958 \ud76c\uadf9\ud48d\uc790\uc9d1\ub2e8\uc73c\ub85c \uc2dc\uc791\ud588\uc73c\ub098, \ubc18\uc804\ud3c9\ud654\uc758 \uae30\uce58\ub97c \ub0b4\uac78\uba70 \uac00\uac10\uc5c6\ub294 \uae30\uc131 \uc815\uce58\uad8c \ud48d\uc790\ub85c \uc720\uba85\uc138\ub97c \ud0c0\uae30 \uc2dc\uc791\ud588\uc73c\uba70, 2002\ub144\ubd80\ud130 2\ub144\uac04 'CNNNN'(Chaser NoN-stop News Network), 2004\ub144 \uc624\uc2a4\ud2b8\ub808\uc77c\ub9ac\uc544 \ucd1d\uc120\ub2f9\uc2dc '\uccb4\uc774\uc11c\uc758 \uc120\ud0dd'(The Chaser Decides), 2005\ub144\uc758 '\uccb4\uc774\uc11c \ub274\uc2a4\ud2b9\ubcf4'(Chaser News Alert)\ub97c \ud638\uc8fc\ubc29\uc1a1\uacf5\uc0ac \ud154\ub808\ube44\uc804(ABC TV)\ub97c \ud1b5\ud574 \ubc29\uc1a1\ud558\uba74\uc11c \uc790\uad6d\ub0b4 \uc785\uc9c0\ub97c \ub2e4\uc84c\ub2e4. \ud604\uc7ac\ub3c4 '\uccb4\uc774\uc11c \uc5f0\ubcf4'(The Chaser Annual)\ub97c \ud574\ub9c8\ub2e4 \ubc1c\ud589\ud558\uace0, \uadf8\ub4e4\uc758 \uacf5\uc2dd \ud648\ud398\uc774\uc9c0(\uc678\ubd80 \ub9c1\ud06c \ucc38\uc870)\uc5d0\uc11c\ub294 \uad6c\uc131\uc6d0 \uac01\uc790\uac00 \uce7c\ub7fc\ub9ac\uc2a4\ud2b8\ub85c\uc11c \ub0a0\uce74\ub85c\uc6b4 \uc2dc\uc0ac\ube44\ud3c9\uc744 \uc774\uc5b4\ub098\uac00\uace0 \uc788\ub2e4.", "answer": "The Chaser", "sentence": "1990\ub144\ub300 \ub9d0\uc5d0 \uc2dc\ub4dc\ub2c8 \ub300\ud559\uad50\uc5d0 \uc7ac\ud559 \uc911\uc774\uc5c8\ub358 \ucc30\uc2a4 \ud37c\uc2a4, \ub3c4\ubbf8\ub2c9 \ub098\uc787, \uc904\ub9ac\uc548 \ubaa8\ub85c, \ud06c\ub808\uc775 \ub93c\uce74\uc140\uc774 \ud559\ub0b4\uc2e0\ubb38\uc5d0 \uc2dc\uc0ac\ud48d\uc790\uae30\uc0ac\ub97c \uc62c\ub824 \ud070 \ubc18\ud5a5\uc744 \uc5bb\uc5c8\uc73c\uba70, \uc774\ud6c4\uc5d0 \ucc44\uc2a4 \ub9ac\uce58\uc544\ub378\ub85c, \ud06c\ub9ac\uc2a4 \ud14c\uc77c\ub7ec, \uc564\ub4dc\ub8e8 \ud55c\uc13c\uc744 \uc601\uc785\ud558\uc5ec 1999\ub144\ubd80\ud130 \uc2dc\uc0ac\ube44\ud3c9\uc2e0\ubb38 ' The Chaser '\ub97c \ubc1c\ud589\ud558\uae30 \uc2dc\uc791\ud588\ub2e4.", "paragraph_sentence": " 1990\ub144\ub300 \ub9d0\uc5d0 \uc2dc\ub4dc\ub2c8 \ub300\ud559\uad50\uc5d0 \uc7ac\ud559 \uc911\uc774\uc5c8\ub358 \ucc30\uc2a4 \ud37c\uc2a4, \ub3c4\ubbf8\ub2c9 \ub098\uc787, \uc904\ub9ac\uc548 \ubaa8\ub85c, \ud06c\ub808\uc775 \ub93c\uce74\uc140\uc774 \ud559\ub0b4\uc2e0\ubb38\uc5d0 \uc2dc\uc0ac\ud48d\uc790\uae30\uc0ac\ub97c \uc62c\ub824 \ud070 \ubc18\ud5a5\uc744 \uc5bb\uc5c8\uc73c\uba70, \uc774\ud6c4\uc5d0 \ucc44\uc2a4 \ub9ac\uce58\uc544\ub378\ub85c, \ud06c\ub9ac\uc2a4 \ud14c\uc77c\ub7ec, \uc564\ub4dc\ub8e8 \ud55c\uc13c\uc744 \uc601\uc785\ud558\uc5ec 1999\ub144\ubd80\ud130 \uc2dc\uc0ac\ube44\ud3c9\uc2e0\ubb38 ' The Chaser '\ub97c \ubc1c\ud589\ud558\uae30 \uc2dc\uc791\ud588\ub2e4. \ucd08\ubc18\uc5d0\ub294 \ube44\uc8fc\ub958 \ud76c\uadf9\ud48d\uc790\uc9d1\ub2e8\uc73c\ub85c \uc2dc\uc791\ud588\uc73c\ub098, \ubc18\uc804\ud3c9\ud654\uc758 \uae30\uce58\ub97c \ub0b4\uac78\uba70 \uac00\uac10\uc5c6\ub294 \uae30\uc131 \uc815\uce58\uad8c \ud48d\uc790\ub85c \uc720\uba85\uc138\ub97c \ud0c0\uae30 \uc2dc\uc791\ud588\uc73c\uba70, 2002\ub144\ubd80\ud130 2\ub144\uac04 'CNNNN'(Chaser NoN-stop News Network), 2004\ub144 \uc624\uc2a4\ud2b8\ub808\uc77c\ub9ac\uc544 \ucd1d\uc120\ub2f9\uc2dc '\uccb4\uc774\uc11c\uc758 \uc120\ud0dd'(The Chaser Decides), 2005\ub144\uc758 '\uccb4\uc774\uc11c \ub274\uc2a4\ud2b9\ubcf4'(Chaser News Alert)\ub97c \ud638\uc8fc\ubc29\uc1a1\uacf5\uc0ac \ud154\ub808\ube44\uc804(ABC TV)\ub97c \ud1b5\ud574 \ubc29\uc1a1\ud558\uba74\uc11c \uc790\uad6d\ub0b4 \uc785\uc9c0\ub97c \ub2e4\uc84c\ub2e4. \ud604\uc7ac\ub3c4 '\uccb4\uc774\uc11c \uc5f0\ubcf4'(The Chaser Annual)\ub97c \ud574\ub9c8\ub2e4 \ubc1c\ud589\ud558\uace0, \uadf8\ub4e4\uc758 \uacf5\uc2dd \ud648\ud398\uc774\uc9c0(\uc678\ubd80 \ub9c1\ud06c \ucc38\uc870)\uc5d0\uc11c\ub294 \uad6c\uc131\uc6d0 \uac01\uc790\uac00 \uce7c\ub7fc\ub9ac\uc2a4\ud2b8\ub85c\uc11c \ub0a0\uce74\ub85c\uc6b4 \uc2dc\uc0ac\ube44\ud3c9\uc744 \uc774\uc5b4\ub098\uac00\uace0 \uc788\ub2e4.", "paragraph_answer": "1990\ub144\ub300 \ub9d0\uc5d0 \uc2dc\ub4dc\ub2c8 \ub300\ud559\uad50\uc5d0 \uc7ac\ud559 \uc911\uc774\uc5c8\ub358 \ucc30\uc2a4 \ud37c\uc2a4, \ub3c4\ubbf8\ub2c9 \ub098\uc787, \uc904\ub9ac\uc548 \ubaa8\ub85c, \ud06c\ub808\uc775 \ub93c\uce74\uc140\uc774 \ud559\ub0b4\uc2e0\ubb38\uc5d0 \uc2dc\uc0ac\ud48d\uc790\uae30\uc0ac\ub97c \uc62c\ub824 \ud070 \ubc18\ud5a5\uc744 \uc5bb\uc5c8\uc73c\uba70, \uc774\ud6c4\uc5d0 \ucc44\uc2a4 \ub9ac\uce58\uc544\ub378\ub85c, \ud06c\ub9ac\uc2a4 \ud14c\uc77c\ub7ec, \uc564\ub4dc\ub8e8 \ud55c\uc13c\uc744 \uc601\uc785\ud558\uc5ec 1999\ub144\ubd80\ud130 \uc2dc\uc0ac\ube44\ud3c9\uc2e0\ubb38 ' The Chaser '\ub97c \ubc1c\ud589\ud558\uae30 \uc2dc\uc791\ud588\ub2e4. \ucd08\ubc18\uc5d0\ub294 \ube44\uc8fc\ub958 \ud76c\uadf9\ud48d\uc790\uc9d1\ub2e8\uc73c\ub85c \uc2dc\uc791\ud588\uc73c\ub098, \ubc18\uc804\ud3c9\ud654\uc758 \uae30\uce58\ub97c \ub0b4\uac78\uba70 \uac00\uac10\uc5c6\ub294 \uae30\uc131 \uc815\uce58\uad8c \ud48d\uc790\ub85c \uc720\uba85\uc138\ub97c \ud0c0\uae30 \uc2dc\uc791\ud588\uc73c\uba70, 2002\ub144\ubd80\ud130 2\ub144\uac04 'CNNNN'(Chaser NoN-stop News Network), 2004\ub144 \uc624\uc2a4\ud2b8\ub808\uc77c\ub9ac\uc544 \ucd1d\uc120\ub2f9\uc2dc '\uccb4\uc774\uc11c\uc758 \uc120\ud0dd'(The Chaser Decides), 2005\ub144\uc758 '\uccb4\uc774\uc11c \ub274\uc2a4\ud2b9\ubcf4'(Chaser News Alert)\ub97c \ud638\uc8fc\ubc29\uc1a1\uacf5\uc0ac \ud154\ub808\ube44\uc804(ABC TV)\ub97c \ud1b5\ud574 \ubc29\uc1a1\ud558\uba74\uc11c \uc790\uad6d\ub0b4 \uc785\uc9c0\ub97c \ub2e4\uc84c\ub2e4. \ud604\uc7ac\ub3c4 '\uccb4\uc774\uc11c \uc5f0\ubcf4'(The Chaser Annual)\ub97c \ud574\ub9c8\ub2e4 \ubc1c\ud589\ud558\uace0, \uadf8\ub4e4\uc758 \uacf5\uc2dd \ud648\ud398\uc774\uc9c0(\uc678\ubd80 \ub9c1\ud06c \ucc38\uc870)\uc5d0\uc11c\ub294 \uad6c\uc131\uc6d0 \uac01\uc790\uac00 \uce7c\ub7fc\ub9ac\uc2a4\ud2b8\ub85c\uc11c \ub0a0\uce74\ub85c\uc6b4 \uc2dc\uc0ac\ube44\ud3c9\uc744 \uc774\uc5b4\ub098\uac00\uace0 \uc788\ub2e4.", "sentence_answer": "1990\ub144\ub300 \ub9d0\uc5d0 \uc2dc\ub4dc\ub2c8 \ub300\ud559\uad50\uc5d0 \uc7ac\ud559 \uc911\uc774\uc5c8\ub358 \ucc30\uc2a4 \ud37c\uc2a4, \ub3c4\ubbf8\ub2c9 \ub098\uc787, \uc904\ub9ac\uc548 \ubaa8\ub85c, \ud06c\ub808\uc775 \ub93c\uce74\uc140\uc774 \ud559\ub0b4\uc2e0\ubb38\uc5d0 \uc2dc\uc0ac\ud48d\uc790\uae30\uc0ac\ub97c \uc62c\ub824 \ud070 \ubc18\ud5a5\uc744 \uc5bb\uc5c8\uc73c\uba70, \uc774\ud6c4\uc5d0 \ucc44\uc2a4 \ub9ac\uce58\uc544\ub378\ub85c, \ud06c\ub9ac\uc2a4 \ud14c\uc77c\ub7ec, \uc564\ub4dc\ub8e8 \ud55c\uc13c\uc744 \uc601\uc785\ud558\uc5ec 1999\ub144\ubd80\ud130 \uc2dc\uc0ac\ube44\ud3c9\uc2e0\ubb38 ' The Chaser '\ub97c \ubc1c\ud589\ud558\uae30 \uc2dc\uc791\ud588\ub2e4.", "paragraph_id": "6559656-0-1", "paragraph_question": "question: \ucc30\uc2a4 \ud37c\uc2a4, \uc904\ub9ac\uc548 \ubaa8\ub85c, \uc564\ub4dc\ub8e8 \ud55c\uc13c \ub4f1\uc774 1999\ub144\ubd80\ud130 \ubc1c\ud589\ud55c \uc2dc\uc0ac\ube44\ud3c9\uc2e0\ubb38\uc758 \uc774\ub984\uc740?, context: 1990\ub144\ub300 \ub9d0\uc5d0 \uc2dc\ub4dc\ub2c8 \ub300\ud559\uad50\uc5d0 \uc7ac\ud559 \uc911\uc774\uc5c8\ub358 \ucc30\uc2a4 \ud37c\uc2a4, \ub3c4\ubbf8\ub2c9 \ub098\uc787, \uc904\ub9ac\uc548 \ubaa8\ub85c, \ud06c\ub808\uc775 \ub93c\uce74\uc140\uc774 \ud559\ub0b4\uc2e0\ubb38\uc5d0 \uc2dc\uc0ac\ud48d\uc790\uae30\uc0ac\ub97c \uc62c\ub824 \ud070 \ubc18\ud5a5\uc744 \uc5bb\uc5c8\uc73c\uba70, \uc774\ud6c4\uc5d0 \ucc44\uc2a4 \ub9ac\uce58\uc544\ub378\ub85c, \ud06c\ub9ac\uc2a4 \ud14c\uc77c\ub7ec, \uc564\ub4dc\ub8e8 \ud55c\uc13c\uc744 \uc601\uc785\ud558\uc5ec 1999\ub144\ubd80\ud130 \uc2dc\uc0ac\ube44\ud3c9\uc2e0\ubb38 'The Chaser'\ub97c \ubc1c\ud589\ud558\uae30 \uc2dc\uc791\ud588\ub2e4. \ucd08\ubc18\uc5d0\ub294 \ube44\uc8fc\ub958 \ud76c\uadf9\ud48d\uc790\uc9d1\ub2e8\uc73c\ub85c \uc2dc\uc791\ud588\uc73c\ub098, \ubc18\uc804\ud3c9\ud654\uc758 \uae30\uce58\ub97c \ub0b4\uac78\uba70 \uac00\uac10\uc5c6\ub294 \uae30\uc131 \uc815\uce58\uad8c \ud48d\uc790\ub85c \uc720\uba85\uc138\ub97c \ud0c0\uae30 \uc2dc\uc791\ud588\uc73c\uba70, 2002\ub144\ubd80\ud130 2\ub144\uac04 'CNNNN'(Chaser NoN-stop News Network), 2004\ub144 \uc624\uc2a4\ud2b8\ub808\uc77c\ub9ac\uc544 \ucd1d\uc120\ub2f9\uc2dc '\uccb4\uc774\uc11c\uc758 \uc120\ud0dd'(The Chaser Decides), 2005\ub144\uc758 '\uccb4\uc774\uc11c \ub274\uc2a4\ud2b9\ubcf4'(Chaser News Alert)\ub97c \ud638\uc8fc\ubc29\uc1a1\uacf5\uc0ac \ud154\ub808\ube44\uc804(ABC TV)\ub97c \ud1b5\ud574 \ubc29\uc1a1\ud558\uba74\uc11c \uc790\uad6d\ub0b4 \uc785\uc9c0\ub97c \ub2e4\uc84c\ub2e4. \ud604\uc7ac\ub3c4 '\uccb4\uc774\uc11c \uc5f0\ubcf4'(The Chaser Annual)\ub97c \ud574\ub9c8\ub2e4 \ubc1c\ud589\ud558\uace0, \uadf8\ub4e4\uc758 \uacf5\uc2dd \ud648\ud398\uc774\uc9c0(\uc678\ubd80 \ub9c1\ud06c \ucc38\uc870)\uc5d0\uc11c\ub294 \uad6c\uc131\uc6d0 \uac01\uc790\uac00 \uce7c\ub7fc\ub9ac\uc2a4\ud2b8\ub85c\uc11c \ub0a0\uce74\ub85c\uc6b4 \uc2dc\uc0ac\ube44\ud3c9\uc744 \uc774\uc5b4\ub098\uac00\uace0 \uc788\ub2e4."} {"question": "\uc815\uc6d0 \uc124\uacc4\uc5d0 \uc801\uc6a9\ub41c \ub300\uaddc\ubaa8 \uae30\ud559\ud559\uc694\uc18c\uc758 \uac1c\ub150\uc744 \ub9cc\ub4e0 \uc774\ub294?", "paragraph": "\ub8e8\uc774 14\uc138\uac00 \uad6d\uac00\uc758 \ud2c0\uc744 \uc644\ube44\ud558\uba70 \uc655\uad8c\uc744 \uac15\ud654\ud558\uba70 \uc655\uc774 \ubb38\ud654\ub97c \uc8fc\ub3c4\ud558\uace0 \ud6c4\uc6d0\ud558\uba70 \ubc1c\ub2ec\ud558\uac8c \ub41c\ub2e4. \uc774\ud0c8\ub9ac\uc544\uc5d0\uc11c \ud655\ub9bd\ub41c \ucd95 \uc911\uc2ec\uc758 \uc815\uc6d0 \uc124\uacc4 \uae30\ubc95\uc774 \ud504\ub791\uc2a4\uc5d0\uc11c\ub294 \uc0c8\ub85c\uc6b4 \uc2a4\ucf00\uc77c\ub85c \uc801\uc6a9\ub418\uc5c8\uace0 \uc6b0\uc8fc\uc801 \uc2a4\ucf00\uc77c\ub85c \ud655\uc7a5\ud558\uac8c \ub418\uc5c8\ub2e4. 17\uc138\uae30 \ud504\ub791\uc2a4\ub294 \uc559\ub4dc\ub808 \ub974 \ub178\ud2b8\ub85c\ub77c\ub294 \uc655\ub9bd\uc815\uc6d0\uc758 \uc124\uac8c\uac00\uac00 \uc8fc\ub3c4 \ud588\uc73c\uba70 \uc815\uc6d0\uc744 \ud1b5\ud574\uc11c \ud0dc\uc591\uc655\uc758 \uad8c\uc704\uac00 \ub4e4\uc5b4\ub098\ub3c4\ub85d \uac70\ub300\ud55c \uc2a4\ucf00\uc77c\uac10\uc744 \ud1b5\ud574\uc11c \ubcf4\ub294 \uc774\ub85c \ud558\uc5ec\uae08 \uacbd\uc678\uac10\uc774 \uc77c\uac8c \ud558\ub294 \ubc29\ubc95\uc744 \uc0ac\uc6a9\ud588\ub2e4. \uadf8\ub294 \uc7a5\uc5c4\ud568\uc744 \ud45c\uc2dc\ud558\ub294\ub370 \uba74\uc801\uc744 \uc0ac\uc6a9\ud558\uace0 \ubc14\ub85c\ud06c \uc591\uc2dd\uc5d0 \uc6b0\uc544\ud568\uc744 \uac00\ubbf8, \uc18c\ub85c, \ud3c9\uba74\uae30\ud558\ud559, \uc815\uc6d0\uc758 \ub2f4\uc744 \uc0b0\ub9bc\uc73c\ub85c \ucc98\ub9ac\ud558\ub294 \ubc29\ubc95\uc744 \uc0ac\uc6a9\ud588\uace0 \uc774\ub294 18~19\uc138\uae30 \uc804 \uc720\ub7fd\uc5d0 \uc601\ud5a5\uc744 \uc8fc\uc5c8\ub2e4. \uadf8\ub294 \uae30\uc874 \ub974\ub124\uc0c1\uc2a4 \uc815\uc6d0\uc5d0\uc11c \ud0c8\ud53c, \uc7a5\uc5c4\ud558\uba74\uc11c \uc138\ub828\ub41c \uc2a4\ud0c0\uc77c\uc744 \ubcf4\uc5ec\uc8fc\uace0 \uacf5\uac04\uc744 \uae30\ud558\ud559\uc801\uc73c\ub85c \ud30c\uc545, \uc815\uc6d0 \ub514\uc790\uc778\uc758 \uc2a4\ucf00\uc77c\uc744 \ud655\uc7a5\uc2dc\ucf30\ub2e4, \ub610\ud55c \ub370\uce74\ub974\ud2b8\uc758 \uc218\ud559\uc801 \uac1c\ub150\uc744 \uc815\uc6d0 \uc124\uacc4\uc5d0 \uc801\uc6a9\ud558\uc5ec \ub300\uaddc\ubaa8 \uae30\ud558\ud559\uc694\uc18c\ub97c \uc815\uc6d0 \uad6c\uc131\uc758 \ud1a0\ub300\uac00 \ub418\uac8c\ud558\uc600\ub2e4.", "answer": "\ub370\uce74\ub974\ud2b8", "sentence": "\uadf8\ub294 \uae30\uc874 \ub974\ub124\uc0c1\uc2a4 \uc815\uc6d0\uc5d0\uc11c \ud0c8\ud53c, \uc7a5\uc5c4\ud558\uba74\uc11c \uc138\ub828\ub41c \uc2a4\ud0c0\uc77c\uc744 \ubcf4\uc5ec\uc8fc\uace0 \uacf5\uac04\uc744 \uae30\ud558\ud559\uc801\uc73c\ub85c \ud30c\uc545, \uc815\uc6d0 \ub514\uc790\uc778\uc758 \uc2a4\ucf00\uc77c\uc744 \ud655\uc7a5\uc2dc\ucf30\ub2e4, \ub610\ud55c \ub370\uce74\ub974\ud2b8 \uc758 \uc218\ud559\uc801 \uac1c\ub150\uc744 \uc815\uc6d0 \uc124\uacc4\uc5d0 \uc801\uc6a9\ud558\uc5ec \ub300\uaddc\ubaa8 \uae30\ud558\ud559\uc694\uc18c\ub97c \uc815\uc6d0 \uad6c\uc131\uc758 \ud1a0\ub300\uac00 \ub418\uac8c\ud558\uc600\ub2e4.", "paragraph_sentence": "\ub8e8\uc774 14\uc138\uac00 \uad6d\uac00\uc758 \ud2c0\uc744 \uc644\ube44\ud558\uba70 \uc655\uad8c\uc744 \uac15\ud654\ud558\uba70 \uc655\uc774 \ubb38\ud654\ub97c \uc8fc\ub3c4\ud558\uace0 \ud6c4\uc6d0\ud558\uba70 \ubc1c\ub2ec\ud558\uac8c \ub41c\ub2e4. \uc774\ud0c8\ub9ac\uc544\uc5d0\uc11c \ud655\ub9bd\ub41c \ucd95 \uc911\uc2ec\uc758 \uc815\uc6d0 \uc124\uacc4 \uae30\ubc95\uc774 \ud504\ub791\uc2a4\uc5d0\uc11c\ub294 \uc0c8\ub85c\uc6b4 \uc2a4\ucf00\uc77c\ub85c \uc801\uc6a9\ub418\uc5c8\uace0 \uc6b0\uc8fc\uc801 \uc2a4\ucf00\uc77c\ub85c \ud655\uc7a5\ud558\uac8c \ub418\uc5c8\ub2e4. 17\uc138\uae30 \ud504\ub791\uc2a4\ub294 \uc559\ub4dc\ub808 \ub974 \ub178\ud2b8\ub85c\ub77c\ub294 \uc655\ub9bd\uc815\uc6d0\uc758 \uc124\uac8c\uac00\uac00 \uc8fc\ub3c4 \ud588\uc73c\uba70 \uc815\uc6d0\uc744 \ud1b5\ud574\uc11c \ud0dc\uc591\uc655\uc758 \uad8c\uc704\uac00 \ub4e4\uc5b4\ub098\ub3c4\ub85d \uac70\ub300\ud55c \uc2a4\ucf00\uc77c\uac10\uc744 \ud1b5\ud574\uc11c \ubcf4\ub294 \uc774\ub85c \ud558\uc5ec\uae08 \uacbd\uc678\uac10\uc774 \uc77c\uac8c \ud558\ub294 \ubc29\ubc95\uc744 \uc0ac\uc6a9\ud588\ub2e4. \uadf8\ub294 \uc7a5\uc5c4\ud568\uc744 \ud45c\uc2dc\ud558\ub294\ub370 \uba74\uc801\uc744 \uc0ac\uc6a9\ud558\uace0 \ubc14\ub85c\ud06c \uc591\uc2dd\uc5d0 \uc6b0\uc544\ud568\uc744 \uac00\ubbf8, \uc18c\ub85c, \ud3c9\uba74\uae30\ud558\ud559, \uc815\uc6d0\uc758 \ub2f4\uc744 \uc0b0\ub9bc\uc73c\ub85c \ucc98\ub9ac\ud558\ub294 \ubc29\ubc95\uc744 \uc0ac\uc6a9\ud588\uace0 \uc774\ub294 18~19\uc138\uae30 \uc804 \uc720\ub7fd\uc5d0 \uc601\ud5a5\uc744 \uc8fc\uc5c8\ub2e4. \uadf8\ub294 \uae30\uc874 \ub974\ub124\uc0c1\uc2a4 \uc815\uc6d0\uc5d0\uc11c \ud0c8\ud53c, \uc7a5\uc5c4\ud558\uba74\uc11c \uc138\ub828\ub41c \uc2a4\ud0c0\uc77c\uc744 \ubcf4\uc5ec\uc8fc\uace0 \uacf5\uac04\uc744 \uae30\ud558\ud559\uc801\uc73c\ub85c \ud30c\uc545, \uc815\uc6d0 \ub514\uc790\uc778\uc758 \uc2a4\ucf00\uc77c\uc744 \ud655\uc7a5\uc2dc\ucf30\ub2e4, \ub610\ud55c \ub370\uce74\ub974\ud2b8 \uc758 \uc218\ud559\uc801 \uac1c\ub150\uc744 \uc815\uc6d0 \uc124\uacc4\uc5d0 \uc801\uc6a9\ud558\uc5ec \ub300\uaddc\ubaa8 \uae30\ud558\ud559\uc694\uc18c\ub97c \uc815\uc6d0 \uad6c\uc131\uc758 \ud1a0\ub300\uac00 \ub418\uac8c\ud558\uc600\ub2e4. ", "paragraph_answer": "\ub8e8\uc774 14\uc138\uac00 \uad6d\uac00\uc758 \ud2c0\uc744 \uc644\ube44\ud558\uba70 \uc655\uad8c\uc744 \uac15\ud654\ud558\uba70 \uc655\uc774 \ubb38\ud654\ub97c \uc8fc\ub3c4\ud558\uace0 \ud6c4\uc6d0\ud558\uba70 \ubc1c\ub2ec\ud558\uac8c \ub41c\ub2e4. \uc774\ud0c8\ub9ac\uc544\uc5d0\uc11c \ud655\ub9bd\ub41c \ucd95 \uc911\uc2ec\uc758 \uc815\uc6d0 \uc124\uacc4 \uae30\ubc95\uc774 \ud504\ub791\uc2a4\uc5d0\uc11c\ub294 \uc0c8\ub85c\uc6b4 \uc2a4\ucf00\uc77c\ub85c \uc801\uc6a9\ub418\uc5c8\uace0 \uc6b0\uc8fc\uc801 \uc2a4\ucf00\uc77c\ub85c \ud655\uc7a5\ud558\uac8c \ub418\uc5c8\ub2e4. 17\uc138\uae30 \ud504\ub791\uc2a4\ub294 \uc559\ub4dc\ub808 \ub974 \ub178\ud2b8\ub85c\ub77c\ub294 \uc655\ub9bd\uc815\uc6d0\uc758 \uc124\uac8c\uac00\uac00 \uc8fc\ub3c4 \ud588\uc73c\uba70 \uc815\uc6d0\uc744 \ud1b5\ud574\uc11c \ud0dc\uc591\uc655\uc758 \uad8c\uc704\uac00 \ub4e4\uc5b4\ub098\ub3c4\ub85d \uac70\ub300\ud55c \uc2a4\ucf00\uc77c\uac10\uc744 \ud1b5\ud574\uc11c \ubcf4\ub294 \uc774\ub85c \ud558\uc5ec\uae08 \uacbd\uc678\uac10\uc774 \uc77c\uac8c \ud558\ub294 \ubc29\ubc95\uc744 \uc0ac\uc6a9\ud588\ub2e4. \uadf8\ub294 \uc7a5\uc5c4\ud568\uc744 \ud45c\uc2dc\ud558\ub294\ub370 \uba74\uc801\uc744 \uc0ac\uc6a9\ud558\uace0 \ubc14\ub85c\ud06c \uc591\uc2dd\uc5d0 \uc6b0\uc544\ud568\uc744 \uac00\ubbf8, \uc18c\ub85c, \ud3c9\uba74\uae30\ud558\ud559, \uc815\uc6d0\uc758 \ub2f4\uc744 \uc0b0\ub9bc\uc73c\ub85c \ucc98\ub9ac\ud558\ub294 \ubc29\ubc95\uc744 \uc0ac\uc6a9\ud588\uace0 \uc774\ub294 18~19\uc138\uae30 \uc804 \uc720\ub7fd\uc5d0 \uc601\ud5a5\uc744 \uc8fc\uc5c8\ub2e4. \uadf8\ub294 \uae30\uc874 \ub974\ub124\uc0c1\uc2a4 \uc815\uc6d0\uc5d0\uc11c \ud0c8\ud53c, \uc7a5\uc5c4\ud558\uba74\uc11c \uc138\ub828\ub41c \uc2a4\ud0c0\uc77c\uc744 \ubcf4\uc5ec\uc8fc\uace0 \uacf5\uac04\uc744 \uae30\ud558\ud559\uc801\uc73c\ub85c \ud30c\uc545, \uc815\uc6d0 \ub514\uc790\uc778\uc758 \uc2a4\ucf00\uc77c\uc744 \ud655\uc7a5\uc2dc\ucf30\ub2e4, \ub610\ud55c \ub370\uce74\ub974\ud2b8 \uc758 \uc218\ud559\uc801 \uac1c\ub150\uc744 \uc815\uc6d0 \uc124\uacc4\uc5d0 \uc801\uc6a9\ud558\uc5ec \ub300\uaddc\ubaa8 \uae30\ud558\ud559\uc694\uc18c\ub97c \uc815\uc6d0 \uad6c\uc131\uc758 \ud1a0\ub300\uac00 \ub418\uac8c\ud558\uc600\ub2e4.", "sentence_answer": "\uadf8\ub294 \uae30\uc874 \ub974\ub124\uc0c1\uc2a4 \uc815\uc6d0\uc5d0\uc11c \ud0c8\ud53c, \uc7a5\uc5c4\ud558\uba74\uc11c \uc138\ub828\ub41c \uc2a4\ud0c0\uc77c\uc744 \ubcf4\uc5ec\uc8fc\uace0 \uacf5\uac04\uc744 \uae30\ud558\ud559\uc801\uc73c\ub85c \ud30c\uc545, \uc815\uc6d0 \ub514\uc790\uc778\uc758 \uc2a4\ucf00\uc77c\uc744 \ud655\uc7a5\uc2dc\ucf30\ub2e4, \ub610\ud55c \ub370\uce74\ub974\ud2b8 \uc758 \uc218\ud559\uc801 \uac1c\ub150\uc744 \uc815\uc6d0 \uc124\uacc4\uc5d0 \uc801\uc6a9\ud558\uc5ec \ub300\uaddc\ubaa8 \uae30\ud558\ud559\uc694\uc18c\ub97c \uc815\uc6d0 \uad6c\uc131\uc758 \ud1a0\ub300\uac00 \ub418\uac8c\ud558\uc600\ub2e4.", "paragraph_id": "5874187-0-1", "paragraph_question": "question: \uc815\uc6d0 \uc124\uacc4\uc5d0 \uc801\uc6a9\ub41c \ub300\uaddc\ubaa8 \uae30\ud559\ud559\uc694\uc18c\uc758 \uac1c\ub150\uc744 \ub9cc\ub4e0 \uc774\ub294?, context: \ub8e8\uc774 14\uc138\uac00 \uad6d\uac00\uc758 \ud2c0\uc744 \uc644\ube44\ud558\uba70 \uc655\uad8c\uc744 \uac15\ud654\ud558\uba70 \uc655\uc774 \ubb38\ud654\ub97c \uc8fc\ub3c4\ud558\uace0 \ud6c4\uc6d0\ud558\uba70 \ubc1c\ub2ec\ud558\uac8c \ub41c\ub2e4. \uc774\ud0c8\ub9ac\uc544\uc5d0\uc11c \ud655\ub9bd\ub41c \ucd95 \uc911\uc2ec\uc758 \uc815\uc6d0 \uc124\uacc4 \uae30\ubc95\uc774 \ud504\ub791\uc2a4\uc5d0\uc11c\ub294 \uc0c8\ub85c\uc6b4 \uc2a4\ucf00\uc77c\ub85c \uc801\uc6a9\ub418\uc5c8\uace0 \uc6b0\uc8fc\uc801 \uc2a4\ucf00\uc77c\ub85c \ud655\uc7a5\ud558\uac8c \ub418\uc5c8\ub2e4. 17\uc138\uae30 \ud504\ub791\uc2a4\ub294 \uc559\ub4dc\ub808 \ub974 \ub178\ud2b8\ub85c\ub77c\ub294 \uc655\ub9bd\uc815\uc6d0\uc758 \uc124\uac8c\uac00\uac00 \uc8fc\ub3c4 \ud588\uc73c\uba70 \uc815\uc6d0\uc744 \ud1b5\ud574\uc11c \ud0dc\uc591\uc655\uc758 \uad8c\uc704\uac00 \ub4e4\uc5b4\ub098\ub3c4\ub85d \uac70\ub300\ud55c \uc2a4\ucf00\uc77c\uac10\uc744 \ud1b5\ud574\uc11c \ubcf4\ub294 \uc774\ub85c \ud558\uc5ec\uae08 \uacbd\uc678\uac10\uc774 \uc77c\uac8c \ud558\ub294 \ubc29\ubc95\uc744 \uc0ac\uc6a9\ud588\ub2e4. \uadf8\ub294 \uc7a5\uc5c4\ud568\uc744 \ud45c\uc2dc\ud558\ub294\ub370 \uba74\uc801\uc744 \uc0ac\uc6a9\ud558\uace0 \ubc14\ub85c\ud06c \uc591\uc2dd\uc5d0 \uc6b0\uc544\ud568\uc744 \uac00\ubbf8, \uc18c\ub85c, \ud3c9\uba74\uae30\ud558\ud559, \uc815\uc6d0\uc758 \ub2f4\uc744 \uc0b0\ub9bc\uc73c\ub85c \ucc98\ub9ac\ud558\ub294 \ubc29\ubc95\uc744 \uc0ac\uc6a9\ud588\uace0 \uc774\ub294 18~19\uc138\uae30 \uc804 \uc720\ub7fd\uc5d0 \uc601\ud5a5\uc744 \uc8fc\uc5c8\ub2e4. \uadf8\ub294 \uae30\uc874 \ub974\ub124\uc0c1\uc2a4 \uc815\uc6d0\uc5d0\uc11c \ud0c8\ud53c, \uc7a5\uc5c4\ud558\uba74\uc11c \uc138\ub828\ub41c \uc2a4\ud0c0\uc77c\uc744 \ubcf4\uc5ec\uc8fc\uace0 \uacf5\uac04\uc744 \uae30\ud558\ud559\uc801\uc73c\ub85c \ud30c\uc545, \uc815\uc6d0 \ub514\uc790\uc778\uc758 \uc2a4\ucf00\uc77c\uc744 \ud655\uc7a5\uc2dc\ucf30\ub2e4, \ub610\ud55c \ub370\uce74\ub974\ud2b8\uc758 \uc218\ud559\uc801 \uac1c\ub150\uc744 \uc815\uc6d0 \uc124\uacc4\uc5d0 \uc801\uc6a9\ud558\uc5ec \ub300\uaddc\ubaa8 \uae30\ud558\ud559\uc694\uc18c\ub97c \uc815\uc6d0 \uad6c\uc131\uc758 \ud1a0\ub300\uac00 \ub418\uac8c\ud558\uc600\ub2e4."} {"question": "\uce7c\ubc45\uc758 \uc2e0\ud559\uc774 \ubcc0\ud654 \uc5c6\uc774 \uacc4\uc18d\ub418\uc5c8\uc74c\uc744 \uc758\ubbf8\ud558\ub294 \uc99d\uac70\ub294?", "paragraph": "\uce7c\ube48\uc740 \uc790\uc2e0\uc758 \uc0ac\uc0c1\uc744 \uc8fc\uc11d, \uc124\uad50, \ub17c\ubb38\uc5d0\uc11c \uc9c0\uc18d\uc801\uc73c\ub85c \ubc1c\uc804\uc2dc\ucf30\ub2e4. \uadf8\uc758 \uc2e0\ud559\uc758 \uac00\uc7a5 \uac04\uacb0\ud55c \ud45c\ud604\uc740 \ubc14\ub85c \uae30\ub3c5\uad50 \uac15\uc694 \uc774\ub2e4. \uae30\ub3c5\uad50\uac15\uc694\uc758 \ub2e4\uc591\ud55c \ud310\uc740 \uac1c\ud601\uc790\ub85c\uc11c \uac70\uc758 \ubaa8\ub4e0 \uacbd\ub825\uc744 \uc313\uc558\uc73c\uba70, \uc774 \ucc45\uc758 \uc5f0\uc18d\ub41c \uac1c\uc815\uc740 \uadf8\uc758 \uc2e0\ud559\uc774 \uadf8\uc758 \uccad\ub144\uc2dc\uc808\uc5d0\uc11c \ub9d0\ub144\uae4c\uc9c0 \uac70\uc758 \ubcc0\ud654\ud558\uc9c0 \uc54a\uc558\uc74c\uc744 \ubcf4\uc5ec\uc900\ub2e4. 1536 \ub144 \ucd08\ud310\uc740 6 \uc7a5\uc73c\ub85c \uad6c\uc131\ub418\uc5c8\uc9c0\ub9cc, 1539 \ub144\uc5d0 \ucd9c\ud310 \ub41c \ub450 \ubc88\uc9f8 \ud310\uc740 \uba5c\ub780\ud788\ud1a4\uc758 \uc2e0\ud559\ud1b5\ub860 (Loci Communes)\uc5d0 \ub098\uc624\ub294 \uc8fc\uc81c\uc5d0 \uc5ec\ub7ec \uc7a5\ub4e4\uc744 \ucd94\uac00\ud588\uae30 \ub54c\ubb38\uc5d0 3 \ubc30\ub85c \ub298\uc5b4\ub0ac\ub2e4. 1543 \ub144\uc5d0 \uadf8\ub294 \ub2e4\uc2dc \uc0c8\ub85c\uc6b4 \uc790\ub8cc\ub97c \ucd94\uac00\ud558\uace0 \uc0ac\ub3c4 \uc2e0\uacbd\uc5d0 \uad00\ud55c \uc7a5\uc744 \ud655\uc7a5\ud558\uc600\ub2e4. 1559\ub144\uc5d0\ub9c8\uc9c0\ub9c9 \ud310\uc774 \ub098\uc654\ub2e4. 80\uac1c \uc7a5\uc744 \uac16\ub294 4\uad8c\uc73c\ub85c \uad6c\uc131\ub418\uc5c8\uace0, \uac01 \ucc45\uc740 \uc2e0\uc870\uc758 \uc9c4\uc220\uc5d0 \ub530\ub77c \uc9c4\uc220\ub418\uc5c8\ub2e4. \ucc3d\uc870\uc8fc \ud558\ub098\ub2d8\uc5d0 \ub300\ud55c \uc81c 1 \uad8c, \uadf8\ub9ac\uc2a4\ub3c4\uc758 \uad6c\uc18d\uc8fc\uc5d0 \ub300\ud55c \uc81c 2 \uad8c, \uc81c 3\uad8c\uc740 \uc131\ub839\uc744 \ud1b5\ud574 \uadf8\ub9ac\uc2a4\ub3c4\uc758 \uc740\ud61c\ub97c \ubc1b\uc74c, \uadf8\ub9ac\uace0 4 \uad8c\uc740 \uadf8\ub9ac\uc2a4\ub3c4\uc758 \uc0ac\ud68c \ub610\ub294 \uad50\ud68c\uc5d0 \uad00\ud574\uc11c \ub2e4\ub8ec\ub2e4.", "answer": "\uc5f0\uc18d\ub41c \uac1c\uc815", "sentence": "\uc774 \ucc45\uc758 \uc5f0\uc18d\ub41c \uac1c\uc815 \uc740 \uadf8\uc758 \uc2e0\ud559\uc774 \uadf8\uc758 \uccad\ub144\uc2dc\uc808\uc5d0\uc11c \ub9d0\ub144\uae4c\uc9c0 \uac70\uc758 \ubcc0\ud654\ud558\uc9c0 \uc54a\uc558\uc74c\uc744 \ubcf4\uc5ec\uc900\ub2e4.", "paragraph_sentence": "\uce7c\ube48\uc740 \uc790\uc2e0\uc758 \uc0ac\uc0c1\uc744 \uc8fc\uc11d, \uc124\uad50, \ub17c\ubb38\uc5d0\uc11c \uc9c0\uc18d\uc801\uc73c\ub85c \ubc1c\uc804\uc2dc\ucf30\ub2e4. \uadf8\uc758 \uc2e0\ud559\uc758 \uac00\uc7a5 \uac04\uacb0\ud55c \ud45c\ud604\uc740 \ubc14\ub85c \uae30\ub3c5\uad50 \uac15\uc694 \uc774\ub2e4. \uae30\ub3c5\uad50\uac15\uc694\uc758 \ub2e4\uc591\ud55c \ud310\uc740 \uac1c\ud601\uc790\ub85c\uc11c \uac70\uc758 \ubaa8\ub4e0 \uacbd\ub825\uc744 \uc313\uc558\uc73c\uba70, \uc774 \ucc45\uc758 \uc5f0\uc18d\ub41c \uac1c\uc815 \uc740 \uadf8\uc758 \uc2e0\ud559\uc774 \uadf8\uc758 \uccad\ub144\uc2dc\uc808\uc5d0\uc11c \ub9d0\ub144\uae4c\uc9c0 \uac70\uc758 \ubcc0\ud654\ud558\uc9c0 \uc54a\uc558\uc74c\uc744 \ubcf4\uc5ec\uc900\ub2e4. 1536 \ub144 \ucd08\ud310\uc740 6 \uc7a5\uc73c\ub85c \uad6c\uc131\ub418\uc5c8\uc9c0\ub9cc, 1539 \ub144\uc5d0 \ucd9c\ud310 \ub41c \ub450 \ubc88\uc9f8 \ud310\uc740 \uba5c\ub780\ud788\ud1a4\uc758 \uc2e0\ud559\ud1b5\ub860 (Loci Communes)\uc5d0 \ub098\uc624\ub294 \uc8fc\uc81c\uc5d0 \uc5ec\ub7ec \uc7a5\ub4e4\uc744 \ucd94\uac00\ud588\uae30 \ub54c\ubb38\uc5d0 3 \ubc30\ub85c \ub298\uc5b4\ub0ac\ub2e4. 1543 \ub144\uc5d0 \uadf8\ub294 \ub2e4\uc2dc \uc0c8\ub85c\uc6b4 \uc790\ub8cc\ub97c \ucd94\uac00\ud558\uace0 \uc0ac\ub3c4 \uc2e0\uacbd\uc5d0 \uad00\ud55c \uc7a5\uc744 \ud655\uc7a5\ud558\uc600\ub2e4. 1559\ub144\uc5d0\ub9c8\uc9c0\ub9c9 \ud310\uc774 \ub098\uc654\ub2e4. 80\uac1c \uc7a5\uc744 \uac16\ub294 4\uad8c\uc73c\ub85c \uad6c\uc131\ub418\uc5c8\uace0, \uac01 \ucc45\uc740 \uc2e0\uc870\uc758 \uc9c4\uc220\uc5d0 \ub530\ub77c \uc9c4\uc220\ub418\uc5c8\ub2e4. \ucc3d\uc870\uc8fc \ud558\ub098\ub2d8\uc5d0 \ub300\ud55c \uc81c 1 \uad8c, \uadf8\ub9ac\uc2a4\ub3c4\uc758 \uad6c\uc18d\uc8fc\uc5d0 \ub300\ud55c \uc81c 2 \uad8c, \uc81c 3\uad8c\uc740 \uc131\ub839\uc744 \ud1b5\ud574 \uadf8\ub9ac\uc2a4\ub3c4\uc758 \uc740\ud61c\ub97c \ubc1b\uc74c, \uadf8\ub9ac\uace0 4 \uad8c\uc740 \uadf8\ub9ac\uc2a4\ub3c4\uc758 \uc0ac\ud68c \ub610\ub294 \uad50\ud68c\uc5d0 \uad00\ud574\uc11c \ub2e4\ub8ec\ub2e4.", "paragraph_answer": "\uce7c\ube48\uc740 \uc790\uc2e0\uc758 \uc0ac\uc0c1\uc744 \uc8fc\uc11d, \uc124\uad50, \ub17c\ubb38\uc5d0\uc11c \uc9c0\uc18d\uc801\uc73c\ub85c \ubc1c\uc804\uc2dc\ucf30\ub2e4. \uadf8\uc758 \uc2e0\ud559\uc758 \uac00\uc7a5 \uac04\uacb0\ud55c \ud45c\ud604\uc740 \ubc14\ub85c \uae30\ub3c5\uad50 \uac15\uc694 \uc774\ub2e4. \uae30\ub3c5\uad50\uac15\uc694\uc758 \ub2e4\uc591\ud55c \ud310\uc740 \uac1c\ud601\uc790\ub85c\uc11c \uac70\uc758 \ubaa8\ub4e0 \uacbd\ub825\uc744 \uc313\uc558\uc73c\uba70, \uc774 \ucc45\uc758 \uc5f0\uc18d\ub41c \uac1c\uc815 \uc740 \uadf8\uc758 \uc2e0\ud559\uc774 \uadf8\uc758 \uccad\ub144\uc2dc\uc808\uc5d0\uc11c \ub9d0\ub144\uae4c\uc9c0 \uac70\uc758 \ubcc0\ud654\ud558\uc9c0 \uc54a\uc558\uc74c\uc744 \ubcf4\uc5ec\uc900\ub2e4. 1536 \ub144 \ucd08\ud310\uc740 6 \uc7a5\uc73c\ub85c \uad6c\uc131\ub418\uc5c8\uc9c0\ub9cc, 1539 \ub144\uc5d0 \ucd9c\ud310 \ub41c \ub450 \ubc88\uc9f8 \ud310\uc740 \uba5c\ub780\ud788\ud1a4\uc758 \uc2e0\ud559\ud1b5\ub860 (Loci Communes)\uc5d0 \ub098\uc624\ub294 \uc8fc\uc81c\uc5d0 \uc5ec\ub7ec \uc7a5\ub4e4\uc744 \ucd94\uac00\ud588\uae30 \ub54c\ubb38\uc5d0 3 \ubc30\ub85c \ub298\uc5b4\ub0ac\ub2e4. 1543 \ub144\uc5d0 \uadf8\ub294 \ub2e4\uc2dc \uc0c8\ub85c\uc6b4 \uc790\ub8cc\ub97c \ucd94\uac00\ud558\uace0 \uc0ac\ub3c4 \uc2e0\uacbd\uc5d0 \uad00\ud55c \uc7a5\uc744 \ud655\uc7a5\ud558\uc600\ub2e4. 1559\ub144\uc5d0\ub9c8\uc9c0\ub9c9 \ud310\uc774 \ub098\uc654\ub2e4. 80\uac1c \uc7a5\uc744 \uac16\ub294 4\uad8c\uc73c\ub85c \uad6c\uc131\ub418\uc5c8\uace0, \uac01 \ucc45\uc740 \uc2e0\uc870\uc758 \uc9c4\uc220\uc5d0 \ub530\ub77c \uc9c4\uc220\ub418\uc5c8\ub2e4. \ucc3d\uc870\uc8fc \ud558\ub098\ub2d8\uc5d0 \ub300\ud55c \uc81c 1 \uad8c, \uadf8\ub9ac\uc2a4\ub3c4\uc758 \uad6c\uc18d\uc8fc\uc5d0 \ub300\ud55c \uc81c 2 \uad8c, \uc81c 3\uad8c\uc740 \uc131\ub839\uc744 \ud1b5\ud574 \uadf8\ub9ac\uc2a4\ub3c4\uc758 \uc740\ud61c\ub97c \ubc1b\uc74c, \uadf8\ub9ac\uace0 4 \uad8c\uc740 \uadf8\ub9ac\uc2a4\ub3c4\uc758 \uc0ac\ud68c \ub610\ub294 \uad50\ud68c\uc5d0 \uad00\ud574\uc11c \ub2e4\ub8ec\ub2e4.", "sentence_answer": "\uc774 \ucc45\uc758 \uc5f0\uc18d\ub41c \uac1c\uc815 \uc740 \uadf8\uc758 \uc2e0\ud559\uc774 \uadf8\uc758 \uccad\ub144\uc2dc\uc808\uc5d0\uc11c \ub9d0\ub144\uae4c\uc9c0 \uac70\uc758 \ubcc0\ud654\ud558\uc9c0 \uc54a\uc558\uc74c\uc744 \ubcf4\uc5ec\uc900\ub2e4.", "paragraph_id": "5857167-1-1", "paragraph_question": "question: \uce7c\ubc45\uc758 \uc2e0\ud559\uc774 \ubcc0\ud654 \uc5c6\uc774 \uacc4\uc18d\ub418\uc5c8\uc74c\uc744 \uc758\ubbf8\ud558\ub294 \uc99d\uac70\ub294?, context: \uce7c\ube48\uc740 \uc790\uc2e0\uc758 \uc0ac\uc0c1\uc744 \uc8fc\uc11d, \uc124\uad50, \ub17c\ubb38\uc5d0\uc11c \uc9c0\uc18d\uc801\uc73c\ub85c \ubc1c\uc804\uc2dc\ucf30\ub2e4. \uadf8\uc758 \uc2e0\ud559\uc758 \uac00\uc7a5 \uac04\uacb0\ud55c \ud45c\ud604\uc740 \ubc14\ub85c \uae30\ub3c5\uad50 \uac15\uc694 \uc774\ub2e4. \uae30\ub3c5\uad50\uac15\uc694\uc758 \ub2e4\uc591\ud55c \ud310\uc740 \uac1c\ud601\uc790\ub85c\uc11c \uac70\uc758 \ubaa8\ub4e0 \uacbd\ub825\uc744 \uc313\uc558\uc73c\uba70, \uc774 \ucc45\uc758 \uc5f0\uc18d\ub41c \uac1c\uc815\uc740 \uadf8\uc758 \uc2e0\ud559\uc774 \uadf8\uc758 \uccad\ub144\uc2dc\uc808\uc5d0\uc11c \ub9d0\ub144\uae4c\uc9c0 \uac70\uc758 \ubcc0\ud654\ud558\uc9c0 \uc54a\uc558\uc74c\uc744 \ubcf4\uc5ec\uc900\ub2e4. 1536 \ub144 \ucd08\ud310\uc740 6 \uc7a5\uc73c\ub85c \uad6c\uc131\ub418\uc5c8\uc9c0\ub9cc, 1539 \ub144\uc5d0 \ucd9c\ud310 \ub41c \ub450 \ubc88\uc9f8 \ud310\uc740 \uba5c\ub780\ud788\ud1a4\uc758 \uc2e0\ud559\ud1b5\ub860 (Loci Communes)\uc5d0 \ub098\uc624\ub294 \uc8fc\uc81c\uc5d0 \uc5ec\ub7ec \uc7a5\ub4e4\uc744 \ucd94\uac00\ud588\uae30 \ub54c\ubb38\uc5d0 3 \ubc30\ub85c \ub298\uc5b4\ub0ac\ub2e4. 1543 \ub144\uc5d0 \uadf8\ub294 \ub2e4\uc2dc \uc0c8\ub85c\uc6b4 \uc790\ub8cc\ub97c \ucd94\uac00\ud558\uace0 \uc0ac\ub3c4 \uc2e0\uacbd\uc5d0 \uad00\ud55c \uc7a5\uc744 \ud655\uc7a5\ud558\uc600\ub2e4. 1559\ub144\uc5d0\ub9c8\uc9c0\ub9c9 \ud310\uc774 \ub098\uc654\ub2e4. 80\uac1c \uc7a5\uc744 \uac16\ub294 4\uad8c\uc73c\ub85c \uad6c\uc131\ub418\uc5c8\uace0, \uac01 \ucc45\uc740 \uc2e0\uc870\uc758 \uc9c4\uc220\uc5d0 \ub530\ub77c \uc9c4\uc220\ub418\uc5c8\ub2e4. \ucc3d\uc870\uc8fc \ud558\ub098\ub2d8\uc5d0 \ub300\ud55c \uc81c 1 \uad8c, \uadf8\ub9ac\uc2a4\ub3c4\uc758 \uad6c\uc18d\uc8fc\uc5d0 \ub300\ud55c \uc81c 2 \uad8c, \uc81c 3\uad8c\uc740 \uc131\ub839\uc744 \ud1b5\ud574 \uadf8\ub9ac\uc2a4\ub3c4\uc758 \uc740\ud61c\ub97c \ubc1b\uc74c, \uadf8\ub9ac\uace0 4 \uad8c\uc740 \uadf8\ub9ac\uc2a4\ub3c4\uc758 \uc0ac\ud68c \ub610\ub294 \uad50\ud68c\uc5d0 \uad00\ud574\uc11c \ub2e4\ub8ec\ub2e4."} {"question": "\uc774\ucc44\ud604\uc758 \ucd94\ucc9c\uc73c\ub85c \uae40\ub450\ubd09\uc774 1496\ub144 2\uc6d4\uc5d0 \ucc3d\ub2f9\ud55c \uac83\uc740?", "paragraph": "\ub300\ud559 \uc9c4\ud559\uc744 \ud3ec\uae30\ud55c \uae40\ub300\uc911\uc740 \ubaa9\ud3ec\uc0c1\uc5c5\ud559\uad50(\ubaa9\uc0c1\uace0\ub4f1\ud559\uad50)\ub97c 1944\ub144 \ubd04\uc5d0 \uc878\uc5c5\ud558\uc600\ub2e4. \uc878\uc5c5 \ud6c4\uc5d4 \uc77c\ubcf8\uc758 \uc9d5\uc9d1\uc744 \ud53c\ud558\uae30 \uc704\ud574 \ubaa9\ud3ec\uc0c1\uc120\ud68c\uc0ac\uc5d0 \uacbd\ub9ac\uc0ac\uc6d0\uc73c\ub85c \uc785\uc0ac\ud558\uc600\ub2e4.1945\ub144 \ud574\ubc29\uc774 \ub418\uc790 \uae40\ub300\uc911\uc740 \uc885\uc5c5\uc6d0 \ub300\ud45c\ub85c \ucd94\ub300\ub418\uc5b4 \uc885\uc5c5\uc6d0\ub2e8\uccb4\uac00 \uc870\uc9c1\ud55c \ud68c\uc0ac \uacbd\uc601\uc704\uc6d0\ud68c \uc704\uc6d0\uc7a5\uc5d0 \uc120\ucd9c\ub418\uc5c8\ub2e4. \uc815\uce58\uc5d0 \uad00\uc2ec\uc774 \ub9ce\uc558\ub358 \uadf8\ub294 \uc774 \ud574 \uc5ec\uc6b4\ud615\uc774 \uc774\ub044\ub294 \uac74\uad6d\uc900\ube44\uc704\uc6d0\ud68c \ubaa9\ud3ec\uc9c0\ubd80\uc5d0 \ucc38\uc5ec\ud558\uc5ec \uc120\uc804\ubd80\uc6d0\uc73c\ub85c \ud65c\ub3d9\ud558\uac8c \ub41c\ub2e4. \uac74\uad6d\uc900\ube44\uc704\uc6d0\ud68c\uc758 \uc120\uc804\ucc45\uc778 \uc784\uc601\ucd98\uc758 \ucd94\ucc9c\uc73c\ub85c \uac74\uc900 \ubaa9\ud3ec\uc2dc \uc9c0\ubd80\uc5d0 \uac00\uc785\ud574 \uc120\uc804\ucc45\uc5d0 \uc885\uc0ac\ud558\ub294 \ud55c\ud3b8 \ubaa9\ud3ec\uccad\ub144\ub3d9\ub9f9\uc5d0 \uac00\ub2f4\ud558\uc5ec \ud65c\ub3d9\ud558\uc600\ub2e4. \uadf8\ud574 9\uc6d4 \uac74\uad6d\uc900\ube44\uc704\uc6d0\ud68c\uac00 \uc778\ubbfc\uacf5\ud654\uad6d \uc218\ub9bd\uc73c\ub85c \ubc1c\uc804\uc801 \ud574\uccb4\ub77c\ub294 \uc774\ub984\ud558\uc5d0 \ud574\uccb4\ub418\uc790 \uc774\ucc44\ud604\uc758 \ucd94\ucc9c\uc73c\ub85c \uae40\ub450\ubd09\uc774 1946\ub144 2\uc6d4\uc5d0 \uc5f0\uc548\ud30c \uacf5\uc0b0\uc8fc\uc758\uc790\ub4e4\uacfc \ud568\uaed8 \ucc3d\ub2f9\ud55c \uacf5\uc0b0\uc8fc\uc758 \uc815\ub2f9\uc778 \uc870\uc120\uc2e0\ubbfc\ub2f9\uc5d0 \uc785\ub2f9\ud558\uc5ec \uc870\uc9c1\ubd80\uc7a5\uc73c\ub85c \ud65c\ub3d9\ud558\ub294 \ud55c\ud3b8, \ubbfc\uc8fc\uccad\ub144\ub3d9\ub9f9 \ubaa9\ud3ec\uc2dc \uc9c0\ubd80\uc5d0 \uac00\uc785\ud558\uc5ec \ubd80\uc704\uc6d0\uc7a5\uc73c\ub85c \ud65c\ub3d9\uc744 \ubcd1\ud589\ud558\uc600\ub2e4.", "answer": "\uc870\uc120\uc2e0\ubbfc\ub2f9", "sentence": "\uadf8\ud574 9\uc6d4 \uac74\uad6d\uc900\ube44\uc704\uc6d0\ud68c\uac00 \uc778\ubbfc\uacf5\ud654\uad6d \uc218\ub9bd\uc73c\ub85c \ubc1c\uc804\uc801 \ud574\uccb4\ub77c\ub294 \uc774\ub984\ud558\uc5d0 \ud574\uccb4\ub418\uc790 \uc774\ucc44\ud604\uc758 \ucd94\ucc9c\uc73c\ub85c \uae40\ub450\ubd09\uc774 1946\ub144 2\uc6d4\uc5d0 \uc5f0\uc548\ud30c \uacf5\uc0b0\uc8fc\uc758\uc790\ub4e4\uacfc \ud568\uaed8 \ucc3d\ub2f9\ud55c \uacf5\uc0b0\uc8fc\uc758 \uc815\ub2f9\uc778 \uc870\uc120\uc2e0\ubbfc\ub2f9 \uc5d0 \uc785\ub2f9\ud558\uc5ec \uc870\uc9c1\ubd80\uc7a5\uc73c\ub85c \ud65c\ub3d9\ud558\ub294 \ud55c\ud3b8, \ubbfc\uc8fc\uccad\ub144\ub3d9\ub9f9 \ubaa9\ud3ec\uc2dc \uc9c0\ubd80\uc5d0 \uac00\uc785\ud558\uc5ec \ubd80\uc704\uc6d0\uc7a5\uc73c\ub85c \ud65c\ub3d9\uc744 \ubcd1\ud589\ud558\uc600\ub2e4.", "paragraph_sentence": "\ub300\ud559 \uc9c4\ud559\uc744 \ud3ec\uae30\ud55c \uae40\ub300\uc911\uc740 \ubaa9\ud3ec\uc0c1\uc5c5\ud559\uad50(\ubaa9\uc0c1\uace0\ub4f1\ud559\uad50)\ub97c 1944\ub144 \ubd04\uc5d0 \uc878\uc5c5\ud558\uc600\ub2e4. \uc878\uc5c5 \ud6c4\uc5d4 \uc77c\ubcf8\uc758 \uc9d5\uc9d1\uc744 \ud53c\ud558\uae30 \uc704\ud574 \ubaa9\ud3ec\uc0c1\uc120\ud68c\uc0ac\uc5d0 \uacbd\ub9ac\uc0ac\uc6d0\uc73c\ub85c \uc785\uc0ac\ud558\uc600\ub2e4.1945\ub144 \ud574\ubc29\uc774 \ub418\uc790 \uae40\ub300\uc911\uc740 \uc885\uc5c5\uc6d0 \ub300\ud45c\ub85c \ucd94\ub300\ub418\uc5b4 \uc885\uc5c5\uc6d0\ub2e8\uccb4\uac00 \uc870\uc9c1\ud55c \ud68c\uc0ac \uacbd\uc601\uc704\uc6d0\ud68c \uc704\uc6d0\uc7a5\uc5d0 \uc120\ucd9c\ub418\uc5c8\ub2e4. \uc815\uce58\uc5d0 \uad00\uc2ec\uc774 \ub9ce\uc558\ub358 \uadf8\ub294 \uc774 \ud574 \uc5ec\uc6b4\ud615\uc774 \uc774\ub044\ub294 \uac74\uad6d\uc900\ube44\uc704\uc6d0\ud68c \ubaa9\ud3ec\uc9c0\ubd80\uc5d0 \ucc38\uc5ec\ud558\uc5ec \uc120\uc804\ubd80\uc6d0\uc73c\ub85c \ud65c\ub3d9\ud558\uac8c \ub41c\ub2e4. \uac74\uad6d\uc900\ube44\uc704\uc6d0\ud68c\uc758 \uc120\uc804\ucc45\uc778 \uc784\uc601\ucd98\uc758 \ucd94\ucc9c\uc73c\ub85c \uac74\uc900 \ubaa9\ud3ec\uc2dc \uc9c0\ubd80\uc5d0 \uac00\uc785\ud574 \uc120\uc804\ucc45\uc5d0 \uc885\uc0ac\ud558\ub294 \ud55c\ud3b8 \ubaa9\ud3ec\uccad\ub144\ub3d9\ub9f9\uc5d0 \uac00\ub2f4\ud558\uc5ec \ud65c\ub3d9\ud558\uc600\ub2e4. \uadf8\ud574 9\uc6d4 \uac74\uad6d\uc900\ube44\uc704\uc6d0\ud68c\uac00 \uc778\ubbfc\uacf5\ud654\uad6d \uc218\ub9bd\uc73c\ub85c \ubc1c\uc804\uc801 \ud574\uccb4\ub77c\ub294 \uc774\ub984\ud558\uc5d0 \ud574\uccb4\ub418\uc790 \uc774\ucc44\ud604\uc758 \ucd94\ucc9c\uc73c\ub85c \uae40\ub450\ubd09\uc774 1946\ub144 2\uc6d4\uc5d0 \uc5f0\uc548\ud30c \uacf5\uc0b0\uc8fc\uc758\uc790\ub4e4\uacfc \ud568\uaed8 \ucc3d\ub2f9\ud55c \uacf5\uc0b0\uc8fc\uc758 \uc815\ub2f9\uc778 \uc870\uc120\uc2e0\ubbfc\ub2f9 \uc5d0 \uc785\ub2f9\ud558\uc5ec \uc870\uc9c1\ubd80\uc7a5\uc73c\ub85c \ud65c\ub3d9\ud558\ub294 \ud55c\ud3b8, \ubbfc\uc8fc\uccad\ub144\ub3d9\ub9f9 \ubaa9\ud3ec\uc2dc \uc9c0\ubd80\uc5d0 \uac00\uc785\ud558\uc5ec \ubd80\uc704\uc6d0\uc7a5\uc73c\ub85c \ud65c\ub3d9\uc744 \ubcd1\ud589\ud558\uc600\ub2e4. ", "paragraph_answer": "\ub300\ud559 \uc9c4\ud559\uc744 \ud3ec\uae30\ud55c \uae40\ub300\uc911\uc740 \ubaa9\ud3ec\uc0c1\uc5c5\ud559\uad50(\ubaa9\uc0c1\uace0\ub4f1\ud559\uad50)\ub97c 1944\ub144 \ubd04\uc5d0 \uc878\uc5c5\ud558\uc600\ub2e4. \uc878\uc5c5 \ud6c4\uc5d4 \uc77c\ubcf8\uc758 \uc9d5\uc9d1\uc744 \ud53c\ud558\uae30 \uc704\ud574 \ubaa9\ud3ec\uc0c1\uc120\ud68c\uc0ac\uc5d0 \uacbd\ub9ac\uc0ac\uc6d0\uc73c\ub85c \uc785\uc0ac\ud558\uc600\ub2e4.1945\ub144 \ud574\ubc29\uc774 \ub418\uc790 \uae40\ub300\uc911\uc740 \uc885\uc5c5\uc6d0 \ub300\ud45c\ub85c \ucd94\ub300\ub418\uc5b4 \uc885\uc5c5\uc6d0\ub2e8\uccb4\uac00 \uc870\uc9c1\ud55c \ud68c\uc0ac \uacbd\uc601\uc704\uc6d0\ud68c \uc704\uc6d0\uc7a5\uc5d0 \uc120\ucd9c\ub418\uc5c8\ub2e4. \uc815\uce58\uc5d0 \uad00\uc2ec\uc774 \ub9ce\uc558\ub358 \uadf8\ub294 \uc774 \ud574 \uc5ec\uc6b4\ud615\uc774 \uc774\ub044\ub294 \uac74\uad6d\uc900\ube44\uc704\uc6d0\ud68c \ubaa9\ud3ec\uc9c0\ubd80\uc5d0 \ucc38\uc5ec\ud558\uc5ec \uc120\uc804\ubd80\uc6d0\uc73c\ub85c \ud65c\ub3d9\ud558\uac8c \ub41c\ub2e4. \uac74\uad6d\uc900\ube44\uc704\uc6d0\ud68c\uc758 \uc120\uc804\ucc45\uc778 \uc784\uc601\ucd98\uc758 \ucd94\ucc9c\uc73c\ub85c \uac74\uc900 \ubaa9\ud3ec\uc2dc \uc9c0\ubd80\uc5d0 \uac00\uc785\ud574 \uc120\uc804\ucc45\uc5d0 \uc885\uc0ac\ud558\ub294 \ud55c\ud3b8 \ubaa9\ud3ec\uccad\ub144\ub3d9\ub9f9\uc5d0 \uac00\ub2f4\ud558\uc5ec \ud65c\ub3d9\ud558\uc600\ub2e4. \uadf8\ud574 9\uc6d4 \uac74\uad6d\uc900\ube44\uc704\uc6d0\ud68c\uac00 \uc778\ubbfc\uacf5\ud654\uad6d \uc218\ub9bd\uc73c\ub85c \ubc1c\uc804\uc801 \ud574\uccb4\ub77c\ub294 \uc774\ub984\ud558\uc5d0 \ud574\uccb4\ub418\uc790 \uc774\ucc44\ud604\uc758 \ucd94\ucc9c\uc73c\ub85c \uae40\ub450\ubd09\uc774 1946\ub144 2\uc6d4\uc5d0 \uc5f0\uc548\ud30c \uacf5\uc0b0\uc8fc\uc758\uc790\ub4e4\uacfc \ud568\uaed8 \ucc3d\ub2f9\ud55c \uacf5\uc0b0\uc8fc\uc758 \uc815\ub2f9\uc778 \uc870\uc120\uc2e0\ubbfc\ub2f9 \uc5d0 \uc785\ub2f9\ud558\uc5ec \uc870\uc9c1\ubd80\uc7a5\uc73c\ub85c \ud65c\ub3d9\ud558\ub294 \ud55c\ud3b8, \ubbfc\uc8fc\uccad\ub144\ub3d9\ub9f9 \ubaa9\ud3ec\uc2dc \uc9c0\ubd80\uc5d0 \uac00\uc785\ud558\uc5ec \ubd80\uc704\uc6d0\uc7a5\uc73c\ub85c \ud65c\ub3d9\uc744 \ubcd1\ud589\ud558\uc600\ub2e4.", "sentence_answer": "\uadf8\ud574 9\uc6d4 \uac74\uad6d\uc900\ube44\uc704\uc6d0\ud68c\uac00 \uc778\ubbfc\uacf5\ud654\uad6d \uc218\ub9bd\uc73c\ub85c \ubc1c\uc804\uc801 \ud574\uccb4\ub77c\ub294 \uc774\ub984\ud558\uc5d0 \ud574\uccb4\ub418\uc790 \uc774\ucc44\ud604\uc758 \ucd94\ucc9c\uc73c\ub85c \uae40\ub450\ubd09\uc774 1946\ub144 2\uc6d4\uc5d0 \uc5f0\uc548\ud30c \uacf5\uc0b0\uc8fc\uc758\uc790\ub4e4\uacfc \ud568\uaed8 \ucc3d\ub2f9\ud55c \uacf5\uc0b0\uc8fc\uc758 \uc815\ub2f9\uc778 \uc870\uc120\uc2e0\ubbfc\ub2f9 \uc5d0 \uc785\ub2f9\ud558\uc5ec \uc870\uc9c1\ubd80\uc7a5\uc73c\ub85c \ud65c\ub3d9\ud558\ub294 \ud55c\ud3b8, \ubbfc\uc8fc\uccad\ub144\ub3d9\ub9f9 \ubaa9\ud3ec\uc2dc \uc9c0\ubd80\uc5d0 \uac00\uc785\ud558\uc5ec \ubd80\uc704\uc6d0\uc7a5\uc73c\ub85c \ud65c\ub3d9\uc744 \ubcd1\ud589\ud558\uc600\ub2e4.", "paragraph_id": "6656766-4-1", "paragraph_question": "question: \uc774\ucc44\ud604\uc758 \ucd94\ucc9c\uc73c\ub85c \uae40\ub450\ubd09\uc774 1496\ub144 2\uc6d4\uc5d0 \ucc3d\ub2f9\ud55c \uac83\uc740?, context: \ub300\ud559 \uc9c4\ud559\uc744 \ud3ec\uae30\ud55c \uae40\ub300\uc911\uc740 \ubaa9\ud3ec\uc0c1\uc5c5\ud559\uad50(\ubaa9\uc0c1\uace0\ub4f1\ud559\uad50)\ub97c 1944\ub144 \ubd04\uc5d0 \uc878\uc5c5\ud558\uc600\ub2e4. \uc878\uc5c5 \ud6c4\uc5d4 \uc77c\ubcf8\uc758 \uc9d5\uc9d1\uc744 \ud53c\ud558\uae30 \uc704\ud574 \ubaa9\ud3ec\uc0c1\uc120\ud68c\uc0ac\uc5d0 \uacbd\ub9ac\uc0ac\uc6d0\uc73c\ub85c \uc785\uc0ac\ud558\uc600\ub2e4.1945\ub144 \ud574\ubc29\uc774 \ub418\uc790 \uae40\ub300\uc911\uc740 \uc885\uc5c5\uc6d0 \ub300\ud45c\ub85c \ucd94\ub300\ub418\uc5b4 \uc885\uc5c5\uc6d0\ub2e8\uccb4\uac00 \uc870\uc9c1\ud55c \ud68c\uc0ac \uacbd\uc601\uc704\uc6d0\ud68c \uc704\uc6d0\uc7a5\uc5d0 \uc120\ucd9c\ub418\uc5c8\ub2e4. \uc815\uce58\uc5d0 \uad00\uc2ec\uc774 \ub9ce\uc558\ub358 \uadf8\ub294 \uc774 \ud574 \uc5ec\uc6b4\ud615\uc774 \uc774\ub044\ub294 \uac74\uad6d\uc900\ube44\uc704\uc6d0\ud68c \ubaa9\ud3ec\uc9c0\ubd80\uc5d0 \ucc38\uc5ec\ud558\uc5ec \uc120\uc804\ubd80\uc6d0\uc73c\ub85c \ud65c\ub3d9\ud558\uac8c \ub41c\ub2e4. \uac74\uad6d\uc900\ube44\uc704\uc6d0\ud68c\uc758 \uc120\uc804\ucc45\uc778 \uc784\uc601\ucd98\uc758 \ucd94\ucc9c\uc73c\ub85c \uac74\uc900 \ubaa9\ud3ec\uc2dc \uc9c0\ubd80\uc5d0 \uac00\uc785\ud574 \uc120\uc804\ucc45\uc5d0 \uc885\uc0ac\ud558\ub294 \ud55c\ud3b8 \ubaa9\ud3ec\uccad\ub144\ub3d9\ub9f9\uc5d0 \uac00\ub2f4\ud558\uc5ec \ud65c\ub3d9\ud558\uc600\ub2e4. \uadf8\ud574 9\uc6d4 \uac74\uad6d\uc900\ube44\uc704\uc6d0\ud68c\uac00 \uc778\ubbfc\uacf5\ud654\uad6d \uc218\ub9bd\uc73c\ub85c \ubc1c\uc804\uc801 \ud574\uccb4\ub77c\ub294 \uc774\ub984\ud558\uc5d0 \ud574\uccb4\ub418\uc790 \uc774\ucc44\ud604\uc758 \ucd94\ucc9c\uc73c\ub85c \uae40\ub450\ubd09\uc774 1946\ub144 2\uc6d4\uc5d0 \uc5f0\uc548\ud30c \uacf5\uc0b0\uc8fc\uc758\uc790\ub4e4\uacfc \ud568\uaed8 \ucc3d\ub2f9\ud55c \uacf5\uc0b0\uc8fc\uc758 \uc815\ub2f9\uc778 \uc870\uc120\uc2e0\ubbfc\ub2f9\uc5d0 \uc785\ub2f9\ud558\uc5ec \uc870\uc9c1\ubd80\uc7a5\uc73c\ub85c \ud65c\ub3d9\ud558\ub294 \ud55c\ud3b8, \ubbfc\uc8fc\uccad\ub144\ub3d9\ub9f9 \ubaa9\ud3ec\uc2dc \uc9c0\ubd80\uc5d0 \uac00\uc785\ud558\uc5ec \ubd80\uc704\uc6d0\uc7a5\uc73c\ub85c \ud65c\ub3d9\uc744 \ubcd1\ud589\ud558\uc600\ub2e4."} {"question": "\ucd5c\uadfc \ubd81\ud55c\uc740 \uc5b4\ub5a0\ud55c \ubc1c\uc0ac\ub9cc \uacc4\uc18d \uc5f0\uc2b5\ud558\uace0 \uc788\ub098\uc694?", "paragraph": "\ub9cc\uc720\uc778\ub825\uc758 \ubc95\uce59\uc73c\ub85c \uc778\ud558\uc5ec \ubbf8\uc0ac\uc77c\uc740 \uc704\uc5d0\uc11c \uc544\ub798\ub85c \ub5a8\uc5b4\uc9c0\uba74\uc11c \uac00\uc18d\ub3c4\uac00 \uc810\uc810 \ub354 \ubd99\uae30 \ub54c\ubb38\uc5d0 \uace0\uace0\ub3c4\uc5d0\uc11c\ub294 \ub099\ud558 \uc18d\ub3c4\uac00 \ub290\ub9ac\ub354\ub77c\ub3c4 \uc911\uace0\ub3c4\uc5d0\uc11c\ub294 \ub099\ud558 \uc18d\ub3c4\uac00 \ub354 \ube68\ub77c\uc9c0\uace0 \uc800\uace0\ub3c4\uc5d0\uc11c\ub294 \ub099\ud558 \uc18d\ub3c4\uac00 \ub354\uc6b1 \ub354 \ube68\ub77c\uc9c4\ub2e4. \ubd81\ud55c\uc774 \ubc1c\uc0ac\ud55c \ubb34\uc218\ub2e8 \ubbf8\uc0ac\uc77c\uc740 \uace0\ub3c4 1,400 km \uc774\uc0c1\uc73c\ub85c \uc0c1\uc2b9\ud588\uace0 \ubbf8\uc0ac\uc77c \ub099\ud558\uc18d\ub3c4\ub294 \uc0ac\ub4dc\ub85c \uc694\uaca9\uc774 \uac00\ub2a5\ud55c \uace0\ub3c4 150 km \uc9c0\uc810\uc5d0\uc11c\ub294 \ub9c8\ud558 11 \uc815\ub3c4\uc758 \uc18d\ub3c4\uc600\uc9c0\ub9cc, \ub099\ud558\ud558\uba74\uc11c \uac00\uc18d\ub3c4\uac00 \ub354 \ubd99\uc5b4\uc11c \ud328\ud2b8\ub9ac\uc5b4\ud2b8\ub85c \uc694\uaca9\uc774 \uac00\ub2a5\ud55c \uace0\ub3c4 30 km \uc9c0\uc810\uc5d0\uc11c\ub294 \ub099\ud558 \uc18d\ub3c4\uac00 \ub9c8\ud558 11 \ubcf4\ub2e4 \ud6e8\uc52c \ub354 \ube68\ub77c\uc84c\ub2e4. \ub9cc\uc720\uc778\ub825\uc758 \ubc95\uce59\uc73c\ub85c \uc778\ud55c \uac00\uc18d\ub3c4 \ub54c\ubb38\uc5d0 \ud328\ud2b8\ub9ac\uc5b4\ud2b8\ub85c \uc694\uaca9\uc774 \uac00\ub2a5\ud55c 30 km \uc800\uace0\ub3c4\uc5d0\uc11c\ub294 \ubbf8\uc0ac\uc77c\uc5d0 \uac00\uc18d\ub3c4\uac00 \uc774\ubbf8 \ub108\ubb34 \ub9ce\uc774 \ubd99\uc5b4\uc11c \uc694\uaca9\uc744 \uc131\uacf5\ud558\uae30\uac00 \ub9e4\uc6b0 \ud798\ub4e4\ub2e4. \uadf8\ub807\uae30 \ub54c\ubb38\uc5d0 \ubbf8\uc0ac\uc77c\uc5d0 \uac00\uc18d\ub3c4\uac00 \ubd99\uae30 \uc804\uc5d0 150 km \uace0\uace0\ub3c4\uc5d0 \ub192\uc774 \uc788\uc744 \ub54c \uc694\uaca9\uc744 \ud574\uc57c\uc9c0\ub9cc \uc694\uaca9\uc5d0 \uc131\uacf5\ud560 \uac00\ub2a5\uc131\uc774 \ub354 \ub192\ub2e4. \uc0ac\ub4dc\ub85c \uc694\uaca9\uc774 \uac00\ub2a5\ud55c 150 km \uace0\ub3c4\uc5d0 \ub192\uc774 \uc788\uc744 \ub54c\uc5d0\ub294 \ubbf8\uc0ac\uc77c \ub099\ud558 \uc18d\ub3c4\uac00 \ub290\ub9ac\ub354\ub77c\ub3c4 \ubbf8\uc0ac\uc77c\uc740 \ub099\ud558\ud558\uba74\uc11c \uac00\uc18d\ub3c4\uac00 \ub354 \ubd99\uae30 \ub54c\ubb38\uc5d0 \ud328\ud2b8\ub9ac\uc5b4\ud2b8\ub85c \uc694\uaca9\uc774 \uac00\ub2a5\ud55c 30 km \uc800\uace0\ub3c4\uc5d0\uc11c\ub294 \ubbf8\uc0ac\uc77c \ub099\ud558 \uc18d\ub3c4\uac00 \ud6e8\uc52c \ub354 \ube68\ub77c\uc9c0\uae30 \ub54c\ubb38\uc774\ub2e4. \ucd5c\uadfc \ubd81\ud55c\uc740 \ub178\uace8\uc801\uc73c\ub85c \uace0\uac01\ub3c4 \ubc1c\uc0ac\ub9cc \ubc18\ubcf5\uc801\uc73c\ub85c \uc5f0\uc2b5\ud6c8\ub828\ud558\uace0 \uc788\uc73c\uba70 \ubd81\ud55c\uc774 \uc774\ub7ec\ud55c \uace0\uac01\ub3c4 \ubc1c\uc0ac\ub97c \ubc18\ubcf5\uc801\uc73c\ub85c \uc5f0\uc2b5\ud6c8\ub828\ud558\ub294 \uc774\uc720\ub294 \uc2e4\uc804\uc5d0\uc11c \uace0\uac01\ub3c4 \ubc1c\uc0ac\ub97c \uc815\ud655\ud558\uac8c \uba85\uc911\ud558\uaca0\ub2e4\ub294 \uc758\uc9c0\uac00 \uc788\ub294 \uac83\uc774\ub2e4.", "answer": "\uace0\uac01\ub3c4 \ubc1c\uc0ac", "sentence": "\ucd5c\uadfc \ubd81\ud55c\uc740 \ub178\uace8\uc801\uc73c\ub85c \uace0\uac01\ub3c4 \ubc1c\uc0ac \ub9cc \ubc18\ubcf5\uc801\uc73c\ub85c \uc5f0\uc2b5\ud6c8\ub828\ud558\uace0 \uc788\uc73c\uba70 \ubd81\ud55c\uc774 \uc774\ub7ec\ud55c \uace0\uac01\ub3c4 \ubc1c\uc0ac\ub97c \ubc18\ubcf5\uc801\uc73c\ub85c \uc5f0\uc2b5\ud6c8\ub828\ud558\ub294 \uc774\uc720\ub294 \uc2e4\uc804\uc5d0\uc11c \uace0\uac01\ub3c4 \ubc1c\uc0ac\ub97c \uc815\ud655\ud558\uac8c \uba85\uc911\ud558\uaca0\ub2e4\ub294 \uc758\uc9c0\uac00 \uc788\ub294 \uac83\uc774\ub2e4.", "paragraph_sentence": "\ub9cc\uc720\uc778\ub825\uc758 \ubc95\uce59\uc73c\ub85c \uc778\ud558\uc5ec \ubbf8\uc0ac\uc77c\uc740 \uc704\uc5d0\uc11c \uc544\ub798\ub85c \ub5a8\uc5b4\uc9c0\uba74\uc11c \uac00\uc18d\ub3c4\uac00 \uc810\uc810 \ub354 \ubd99\uae30 \ub54c\ubb38\uc5d0 \uace0\uace0\ub3c4\uc5d0\uc11c\ub294 \ub099\ud558 \uc18d\ub3c4\uac00 \ub290\ub9ac\ub354\ub77c\ub3c4 \uc911\uace0\ub3c4\uc5d0\uc11c\ub294 \ub099\ud558 \uc18d\ub3c4\uac00 \ub354 \ube68\ub77c\uc9c0\uace0 \uc800\uace0\ub3c4\uc5d0\uc11c\ub294 \ub099\ud558 \uc18d\ub3c4\uac00 \ub354\uc6b1 \ub354 \ube68\ub77c\uc9c4\ub2e4. \ubd81\ud55c\uc774 \ubc1c\uc0ac\ud55c \ubb34\uc218\ub2e8 \ubbf8\uc0ac\uc77c\uc740 \uace0\ub3c4 1,400 km \uc774\uc0c1\uc73c\ub85c \uc0c1\uc2b9\ud588\uace0 \ubbf8\uc0ac\uc77c \ub099\ud558\uc18d\ub3c4\ub294 \uc0ac\ub4dc\ub85c \uc694\uaca9\uc774 \uac00\ub2a5\ud55c \uace0\ub3c4 150 km \uc9c0\uc810\uc5d0\uc11c\ub294 \ub9c8\ud558 11 \uc815\ub3c4\uc758 \uc18d\ub3c4\uc600\uc9c0\ub9cc, \ub099\ud558\ud558\uba74\uc11c \uac00\uc18d\ub3c4\uac00 \ub354 \ubd99\uc5b4\uc11c \ud328\ud2b8\ub9ac\uc5b4\ud2b8\ub85c \uc694\uaca9\uc774 \uac00\ub2a5\ud55c \uace0\ub3c4 30 km \uc9c0\uc810\uc5d0\uc11c\ub294 \ub099\ud558 \uc18d\ub3c4\uac00 \ub9c8\ud558 11 \ubcf4\ub2e4 \ud6e8\uc52c \ub354 \ube68\ub77c\uc84c\ub2e4. \ub9cc\uc720\uc778\ub825\uc758 \ubc95\uce59\uc73c\ub85c \uc778\ud55c \uac00\uc18d\ub3c4 \ub54c\ubb38\uc5d0 \ud328\ud2b8\ub9ac\uc5b4\ud2b8\ub85c \uc694\uaca9\uc774 \uac00\ub2a5\ud55c 30 km \uc800\uace0\ub3c4\uc5d0\uc11c\ub294 \ubbf8\uc0ac\uc77c\uc5d0 \uac00\uc18d\ub3c4\uac00 \uc774\ubbf8 \ub108\ubb34 \ub9ce\uc774 \ubd99\uc5b4\uc11c \uc694\uaca9\uc744 \uc131\uacf5\ud558\uae30\uac00 \ub9e4\uc6b0 \ud798\ub4e4\ub2e4. \uadf8\ub807\uae30 \ub54c\ubb38\uc5d0 \ubbf8\uc0ac\uc77c\uc5d0 \uac00\uc18d\ub3c4\uac00 \ubd99\uae30 \uc804\uc5d0 150 km \uace0\uace0\ub3c4\uc5d0 \ub192\uc774 \uc788\uc744 \ub54c \uc694\uaca9\uc744 \ud574\uc57c\uc9c0\ub9cc \uc694\uaca9\uc5d0 \uc131\uacf5\ud560 \uac00\ub2a5\uc131\uc774 \ub354 \ub192\ub2e4. \uc0ac\ub4dc\ub85c \uc694\uaca9\uc774 \uac00\ub2a5\ud55c 150 km \uace0\ub3c4\uc5d0 \ub192\uc774 \uc788\uc744 \ub54c\uc5d0\ub294 \ubbf8\uc0ac\uc77c \ub099\ud558 \uc18d\ub3c4\uac00 \ub290\ub9ac\ub354\ub77c\ub3c4 \ubbf8\uc0ac\uc77c\uc740 \ub099\ud558\ud558\uba74\uc11c \uac00\uc18d\ub3c4\uac00 \ub354 \ubd99\uae30 \ub54c\ubb38\uc5d0 \ud328\ud2b8\ub9ac\uc5b4\ud2b8\ub85c \uc694\uaca9\uc774 \uac00\ub2a5\ud55c 30 km \uc800\uace0\ub3c4\uc5d0\uc11c\ub294 \ubbf8\uc0ac\uc77c \ub099\ud558 \uc18d\ub3c4\uac00 \ud6e8\uc52c \ub354 \ube68\ub77c\uc9c0\uae30 \ub54c\ubb38\uc774\ub2e4. \ucd5c\uadfc \ubd81\ud55c\uc740 \ub178\uace8\uc801\uc73c\ub85c \uace0\uac01\ub3c4 \ubc1c\uc0ac \ub9cc \ubc18\ubcf5\uc801\uc73c\ub85c \uc5f0\uc2b5\ud6c8\ub828\ud558\uace0 \uc788\uc73c\uba70 \ubd81\ud55c\uc774 \uc774\ub7ec\ud55c \uace0\uac01\ub3c4 \ubc1c\uc0ac\ub97c \ubc18\ubcf5\uc801\uc73c\ub85c \uc5f0\uc2b5\ud6c8\ub828\ud558\ub294 \uc774\uc720\ub294 \uc2e4\uc804\uc5d0\uc11c \uace0\uac01\ub3c4 \ubc1c\uc0ac\ub97c \uc815\ud655\ud558\uac8c \uba85\uc911\ud558\uaca0\ub2e4\ub294 \uc758\uc9c0\uac00 \uc788\ub294 \uac83\uc774\ub2e4. ", "paragraph_answer": "\ub9cc\uc720\uc778\ub825\uc758 \ubc95\uce59\uc73c\ub85c \uc778\ud558\uc5ec \ubbf8\uc0ac\uc77c\uc740 \uc704\uc5d0\uc11c \uc544\ub798\ub85c \ub5a8\uc5b4\uc9c0\uba74\uc11c \uac00\uc18d\ub3c4\uac00 \uc810\uc810 \ub354 \ubd99\uae30 \ub54c\ubb38\uc5d0 \uace0\uace0\ub3c4\uc5d0\uc11c\ub294 \ub099\ud558 \uc18d\ub3c4\uac00 \ub290\ub9ac\ub354\ub77c\ub3c4 \uc911\uace0\ub3c4\uc5d0\uc11c\ub294 \ub099\ud558 \uc18d\ub3c4\uac00 \ub354 \ube68\ub77c\uc9c0\uace0 \uc800\uace0\ub3c4\uc5d0\uc11c\ub294 \ub099\ud558 \uc18d\ub3c4\uac00 \ub354\uc6b1 \ub354 \ube68\ub77c\uc9c4\ub2e4. \ubd81\ud55c\uc774 \ubc1c\uc0ac\ud55c \ubb34\uc218\ub2e8 \ubbf8\uc0ac\uc77c\uc740 \uace0\ub3c4 1,400 km \uc774\uc0c1\uc73c\ub85c \uc0c1\uc2b9\ud588\uace0 \ubbf8\uc0ac\uc77c \ub099\ud558\uc18d\ub3c4\ub294 \uc0ac\ub4dc\ub85c \uc694\uaca9\uc774 \uac00\ub2a5\ud55c \uace0\ub3c4 150 km \uc9c0\uc810\uc5d0\uc11c\ub294 \ub9c8\ud558 11 \uc815\ub3c4\uc758 \uc18d\ub3c4\uc600\uc9c0\ub9cc, \ub099\ud558\ud558\uba74\uc11c \uac00\uc18d\ub3c4\uac00 \ub354 \ubd99\uc5b4\uc11c \ud328\ud2b8\ub9ac\uc5b4\ud2b8\ub85c \uc694\uaca9\uc774 \uac00\ub2a5\ud55c \uace0\ub3c4 30 km \uc9c0\uc810\uc5d0\uc11c\ub294 \ub099\ud558 \uc18d\ub3c4\uac00 \ub9c8\ud558 11 \ubcf4\ub2e4 \ud6e8\uc52c \ub354 \ube68\ub77c\uc84c\ub2e4. \ub9cc\uc720\uc778\ub825\uc758 \ubc95\uce59\uc73c\ub85c \uc778\ud55c \uac00\uc18d\ub3c4 \ub54c\ubb38\uc5d0 \ud328\ud2b8\ub9ac\uc5b4\ud2b8\ub85c \uc694\uaca9\uc774 \uac00\ub2a5\ud55c 30 km \uc800\uace0\ub3c4\uc5d0\uc11c\ub294 \ubbf8\uc0ac\uc77c\uc5d0 \uac00\uc18d\ub3c4\uac00 \uc774\ubbf8 \ub108\ubb34 \ub9ce\uc774 \ubd99\uc5b4\uc11c \uc694\uaca9\uc744 \uc131\uacf5\ud558\uae30\uac00 \ub9e4\uc6b0 \ud798\ub4e4\ub2e4. \uadf8\ub807\uae30 \ub54c\ubb38\uc5d0 \ubbf8\uc0ac\uc77c\uc5d0 \uac00\uc18d\ub3c4\uac00 \ubd99\uae30 \uc804\uc5d0 150 km \uace0\uace0\ub3c4\uc5d0 \ub192\uc774 \uc788\uc744 \ub54c \uc694\uaca9\uc744 \ud574\uc57c\uc9c0\ub9cc \uc694\uaca9\uc5d0 \uc131\uacf5\ud560 \uac00\ub2a5\uc131\uc774 \ub354 \ub192\ub2e4. \uc0ac\ub4dc\ub85c \uc694\uaca9\uc774 \uac00\ub2a5\ud55c 150 km \uace0\ub3c4\uc5d0 \ub192\uc774 \uc788\uc744 \ub54c\uc5d0\ub294 \ubbf8\uc0ac\uc77c \ub099\ud558 \uc18d\ub3c4\uac00 \ub290\ub9ac\ub354\ub77c\ub3c4 \ubbf8\uc0ac\uc77c\uc740 \ub099\ud558\ud558\uba74\uc11c \uac00\uc18d\ub3c4\uac00 \ub354 \ubd99\uae30 \ub54c\ubb38\uc5d0 \ud328\ud2b8\ub9ac\uc5b4\ud2b8\ub85c \uc694\uaca9\uc774 \uac00\ub2a5\ud55c 30 km \uc800\uace0\ub3c4\uc5d0\uc11c\ub294 \ubbf8\uc0ac\uc77c \ub099\ud558 \uc18d\ub3c4\uac00 \ud6e8\uc52c \ub354 \ube68\ub77c\uc9c0\uae30 \ub54c\ubb38\uc774\ub2e4. \ucd5c\uadfc \ubd81\ud55c\uc740 \ub178\uace8\uc801\uc73c\ub85c \uace0\uac01\ub3c4 \ubc1c\uc0ac \ub9cc \ubc18\ubcf5\uc801\uc73c\ub85c \uc5f0\uc2b5\ud6c8\ub828\ud558\uace0 \uc788\uc73c\uba70 \ubd81\ud55c\uc774 \uc774\ub7ec\ud55c \uace0\uac01\ub3c4 \ubc1c\uc0ac\ub97c \ubc18\ubcf5\uc801\uc73c\ub85c \uc5f0\uc2b5\ud6c8\ub828\ud558\ub294 \uc774\uc720\ub294 \uc2e4\uc804\uc5d0\uc11c \uace0\uac01\ub3c4 \ubc1c\uc0ac\ub97c \uc815\ud655\ud558\uac8c \uba85\uc911\ud558\uaca0\ub2e4\ub294 \uc758\uc9c0\uac00 \uc788\ub294 \uac83\uc774\ub2e4.", "sentence_answer": "\ucd5c\uadfc \ubd81\ud55c\uc740 \ub178\uace8\uc801\uc73c\ub85c \uace0\uac01\ub3c4 \ubc1c\uc0ac \ub9cc \ubc18\ubcf5\uc801\uc73c\ub85c \uc5f0\uc2b5\ud6c8\ub828\ud558\uace0 \uc788\uc73c\uba70 \ubd81\ud55c\uc774 \uc774\ub7ec\ud55c \uace0\uac01\ub3c4 \ubc1c\uc0ac\ub97c \ubc18\ubcf5\uc801\uc73c\ub85c \uc5f0\uc2b5\ud6c8\ub828\ud558\ub294 \uc774\uc720\ub294 \uc2e4\uc804\uc5d0\uc11c \uace0\uac01\ub3c4 \ubc1c\uc0ac\ub97c \uc815\ud655\ud558\uac8c \uba85\uc911\ud558\uaca0\ub2e4\ub294 \uc758\uc9c0\uac00 \uc788\ub294 \uac83\uc774\ub2e4.", "paragraph_id": "6546295-9-1", "paragraph_question": "question: \ucd5c\uadfc \ubd81\ud55c\uc740 \uc5b4\ub5a0\ud55c \ubc1c\uc0ac\ub9cc \uacc4\uc18d \uc5f0\uc2b5\ud558\uace0 \uc788\ub098\uc694?, context: \ub9cc\uc720\uc778\ub825\uc758 \ubc95\uce59\uc73c\ub85c \uc778\ud558\uc5ec \ubbf8\uc0ac\uc77c\uc740 \uc704\uc5d0\uc11c \uc544\ub798\ub85c \ub5a8\uc5b4\uc9c0\uba74\uc11c \uac00\uc18d\ub3c4\uac00 \uc810\uc810 \ub354 \ubd99\uae30 \ub54c\ubb38\uc5d0 \uace0\uace0\ub3c4\uc5d0\uc11c\ub294 \ub099\ud558 \uc18d\ub3c4\uac00 \ub290\ub9ac\ub354\ub77c\ub3c4 \uc911\uace0\ub3c4\uc5d0\uc11c\ub294 \ub099\ud558 \uc18d\ub3c4\uac00 \ub354 \ube68\ub77c\uc9c0\uace0 \uc800\uace0\ub3c4\uc5d0\uc11c\ub294 \ub099\ud558 \uc18d\ub3c4\uac00 \ub354\uc6b1 \ub354 \ube68\ub77c\uc9c4\ub2e4. \ubd81\ud55c\uc774 \ubc1c\uc0ac\ud55c \ubb34\uc218\ub2e8 \ubbf8\uc0ac\uc77c\uc740 \uace0\ub3c4 1,400 km \uc774\uc0c1\uc73c\ub85c \uc0c1\uc2b9\ud588\uace0 \ubbf8\uc0ac\uc77c \ub099\ud558\uc18d\ub3c4\ub294 \uc0ac\ub4dc\ub85c \uc694\uaca9\uc774 \uac00\ub2a5\ud55c \uace0\ub3c4 150 km \uc9c0\uc810\uc5d0\uc11c\ub294 \ub9c8\ud558 11 \uc815\ub3c4\uc758 \uc18d\ub3c4\uc600\uc9c0\ub9cc, \ub099\ud558\ud558\uba74\uc11c \uac00\uc18d\ub3c4\uac00 \ub354 \ubd99\uc5b4\uc11c \ud328\ud2b8\ub9ac\uc5b4\ud2b8\ub85c \uc694\uaca9\uc774 \uac00\ub2a5\ud55c \uace0\ub3c4 30 km \uc9c0\uc810\uc5d0\uc11c\ub294 \ub099\ud558 \uc18d\ub3c4\uac00 \ub9c8\ud558 11 \ubcf4\ub2e4 \ud6e8\uc52c \ub354 \ube68\ub77c\uc84c\ub2e4. \ub9cc\uc720\uc778\ub825\uc758 \ubc95\uce59\uc73c\ub85c \uc778\ud55c \uac00\uc18d\ub3c4 \ub54c\ubb38\uc5d0 \ud328\ud2b8\ub9ac\uc5b4\ud2b8\ub85c \uc694\uaca9\uc774 \uac00\ub2a5\ud55c 30 km \uc800\uace0\ub3c4\uc5d0\uc11c\ub294 \ubbf8\uc0ac\uc77c\uc5d0 \uac00\uc18d\ub3c4\uac00 \uc774\ubbf8 \ub108\ubb34 \ub9ce\uc774 \ubd99\uc5b4\uc11c \uc694\uaca9\uc744 \uc131\uacf5\ud558\uae30\uac00 \ub9e4\uc6b0 \ud798\ub4e4\ub2e4. \uadf8\ub807\uae30 \ub54c\ubb38\uc5d0 \ubbf8\uc0ac\uc77c\uc5d0 \uac00\uc18d\ub3c4\uac00 \ubd99\uae30 \uc804\uc5d0 150 km \uace0\uace0\ub3c4\uc5d0 \ub192\uc774 \uc788\uc744 \ub54c \uc694\uaca9\uc744 \ud574\uc57c\uc9c0\ub9cc \uc694\uaca9\uc5d0 \uc131\uacf5\ud560 \uac00\ub2a5\uc131\uc774 \ub354 \ub192\ub2e4. \uc0ac\ub4dc\ub85c \uc694\uaca9\uc774 \uac00\ub2a5\ud55c 150 km \uace0\ub3c4\uc5d0 \ub192\uc774 \uc788\uc744 \ub54c\uc5d0\ub294 \ubbf8\uc0ac\uc77c \ub099\ud558 \uc18d\ub3c4\uac00 \ub290\ub9ac\ub354\ub77c\ub3c4 \ubbf8\uc0ac\uc77c\uc740 \ub099\ud558\ud558\uba74\uc11c \uac00\uc18d\ub3c4\uac00 \ub354 \ubd99\uae30 \ub54c\ubb38\uc5d0 \ud328\ud2b8\ub9ac\uc5b4\ud2b8\ub85c \uc694\uaca9\uc774 \uac00\ub2a5\ud55c 30 km \uc800\uace0\ub3c4\uc5d0\uc11c\ub294 \ubbf8\uc0ac\uc77c \ub099\ud558 \uc18d\ub3c4\uac00 \ud6e8\uc52c \ub354 \ube68\ub77c\uc9c0\uae30 \ub54c\ubb38\uc774\ub2e4. \ucd5c\uadfc \ubd81\ud55c\uc740 \ub178\uace8\uc801\uc73c\ub85c \uace0\uac01\ub3c4 \ubc1c\uc0ac\ub9cc \ubc18\ubcf5\uc801\uc73c\ub85c \uc5f0\uc2b5\ud6c8\ub828\ud558\uace0 \uc788\uc73c\uba70 \ubd81\ud55c\uc774 \uc774\ub7ec\ud55c \uace0\uac01\ub3c4 \ubc1c\uc0ac\ub97c \ubc18\ubcf5\uc801\uc73c\ub85c \uc5f0\uc2b5\ud6c8\ub828\ud558\ub294 \uc774\uc720\ub294 \uc2e4\uc804\uc5d0\uc11c \uace0\uac01\ub3c4 \ubc1c\uc0ac\ub97c \uc815\ud655\ud558\uac8c \uba85\uc911\ud558\uaca0\ub2e4\ub294 \uc758\uc9c0\uac00 \uc788\ub294 \uac83\uc774\ub2e4."} {"question": "\ub85c\uc800 \ud398\ub354\ub7ec\uac00 2009\ub144 \uacb0\ud63c\ud55c \uc0ac\ub78c\uc758 \uc774\ub984\uc740?", "paragraph": "\ud398\ub354\ub7ec\ub294 2009\ub144 4\uc6d4 \uc804 \uc5ec\uc790 \ud504\ub85c \ud14c\ub2c8\uc2a4 \uc120\uc218\uc774\uc790 \uadf8\uc758 \uc624\ub79c \uc5f0\uc778\uc774\uc5c8\ub358 \ubbf8\ub974\uce74 \ubc14\ube0c\ub9ac\ub125\uacfc \uacb0\ud63c\ud558\uc600\ub2e4. \ub450 \uc0ac\ub78c\uc740 2000\ub144 \uc2dc\ub4dc\ub2c8 \uc62c\ub9bc\ud53d\uc5d0\uc11c \ud14c\ub2c8\uc2a4 \uc885\ubaa9 \uc2a4\uc704\uc2a4 \uad6d\uac00\ub300\ud45c\ub85c \ud568\uaed8 \ucd9c\uc804\ud558\uba74\uc11c \ub9cc\ub098\uac8c \ub418\uc5c8\ub2e4. \ubc14\ube0c\ub9ac\ub125\uc740 \uace0\uc9c8\uc801\uc778 \ubc1c \ubd80\uc0c1 \ub54c\ubb38\uc5d0 2002\ub144 \ud22c\uc5b4\uc5d0\uc11c \uc740\ud1f4\ud558\uc600\uace0 \uc774\ud6c4 \ud398\ub354\ub7ec\uc758 \ud64d\ubcf4 \ub9e4\ub2c8\uc800 \uc5ed\ud560\uc744 \ub2f4\ub2f9\ud558\uc600\ub2e4. \uc624\ub79c \uc5f0\uc778 \uad00\uacc4\ub97c \uc9c0\uc18d\ud558\ub358 \uc774\ub4e4\uc740 2009\ub144 3\uc6d4\uc5d0 \ud398\ub354\ub7ec\uc758 \ud648\ud398\uc774\uc9c0\ub97c \ud1b5\ud574 \ubc14\ube0c\ub9ac\ub125\uc758 \uc784\uc2e0\uc744 \ubc1c\ud45c\ud558\uc600\uc73c\uba70, \uc774\ud6c4 \ub450 \uc0ac\ub78c\uc740 2009\ub144 4\uc6d4 11\uc77c \ubc14\uc824\uc5d0\uc11c \uac00\uae4c\uc6b4 \uce5c\ucc99\ub4e4\uacfc \uce5c\uad6c\ub4e4\ub9cc\uc774 \ucc38\uc11d\ud55c \uac00\uc6b4\ub370 \uc870\ucd10\ud55c \uacb0\ud63c\uc2dd\uc744 \uc62c\ub838\ub2e4. 2009\ub144 7\uc6d4 24\uc77c, \ud398\ub354\ub7ec\ub294 \uc790\uc2e0\uc758 \ud398\uc774\uc2a4\ubd81 \ud648\ud398\uc774\uc9c0\ub97c \ud1b5\ud574 \ub450 \uc30d\ub465\uc774 \ub538 \ubc00\ub77c \ub85c\uc988(Myla Rose)\uc640 \uc0e4\ub97c\ub80c \ub9ac\ubc14(Charlene Riva)\uac00 7\uc6d4 23\uc77c\uc5d0 \ud0c4\uc0dd\ud588\uc74c\uc744 \uc54c\ub838\ub2e4.", "answer": "\ubbf8\ub974\uce74 \ubc14\ube0c\ub9ac\ub125", "sentence": "\ud398\ub354\ub7ec\ub294 2009\ub144 4\uc6d4 \uc804 \uc5ec\uc790 \ud504\ub85c \ud14c\ub2c8\uc2a4 \uc120\uc218\uc774\uc790 \uadf8\uc758 \uc624\ub79c \uc5f0\uc778\uc774\uc5c8\ub358 \ubbf8\ub974\uce74 \ubc14\ube0c\ub9ac\ub125 \uacfc \uacb0\ud63c\ud558\uc600\ub2e4.", "paragraph_sentence": " \ud398\ub354\ub7ec\ub294 2009\ub144 4\uc6d4 \uc804 \uc5ec\uc790 \ud504\ub85c \ud14c\ub2c8\uc2a4 \uc120\uc218\uc774\uc790 \uadf8\uc758 \uc624\ub79c \uc5f0\uc778\uc774\uc5c8\ub358 \ubbf8\ub974\uce74 \ubc14\ube0c\ub9ac\ub125 \uacfc \uacb0\ud63c\ud558\uc600\ub2e4. \ub450 \uc0ac\ub78c\uc740 2000\ub144 \uc2dc\ub4dc\ub2c8 \uc62c\ub9bc\ud53d\uc5d0\uc11c \ud14c\ub2c8\uc2a4 \uc885\ubaa9 \uc2a4\uc704\uc2a4 \uad6d\uac00\ub300\ud45c\ub85c \ud568\uaed8 \ucd9c\uc804\ud558\uba74\uc11c \ub9cc\ub098\uac8c \ub418\uc5c8\ub2e4. \ubc14\ube0c\ub9ac\ub125\uc740 \uace0\uc9c8\uc801\uc778 \ubc1c \ubd80\uc0c1 \ub54c\ubb38\uc5d0 2002\ub144 \ud22c\uc5b4\uc5d0\uc11c \uc740\ud1f4\ud558\uc600\uace0 \uc774\ud6c4 \ud398\ub354\ub7ec\uc758 \ud64d\ubcf4 \ub9e4\ub2c8\uc800 \uc5ed\ud560\uc744 \ub2f4\ub2f9\ud558\uc600\ub2e4. \uc624\ub79c \uc5f0\uc778 \uad00\uacc4\ub97c \uc9c0\uc18d\ud558\ub358 \uc774\ub4e4\uc740 2009\ub144 3\uc6d4\uc5d0 \ud398\ub354\ub7ec\uc758 \ud648\ud398\uc774\uc9c0\ub97c \ud1b5\ud574 \ubc14\ube0c\ub9ac\ub125\uc758 \uc784\uc2e0\uc744 \ubc1c\ud45c\ud558\uc600\uc73c\uba70, \uc774\ud6c4 \ub450 \uc0ac\ub78c\uc740 2009\ub144 4\uc6d4 11\uc77c \ubc14\uc824\uc5d0\uc11c \uac00\uae4c\uc6b4 \uce5c\ucc99\ub4e4\uacfc \uce5c\uad6c\ub4e4\ub9cc\uc774 \ucc38\uc11d\ud55c \uac00\uc6b4\ub370 \uc870\ucd10\ud55c \uacb0\ud63c\uc2dd\uc744 \uc62c\ub838\ub2e4. 2009\ub144 7\uc6d4 24\uc77c, \ud398\ub354\ub7ec\ub294 \uc790\uc2e0\uc758 \ud398\uc774\uc2a4\ubd81 \ud648\ud398\uc774\uc9c0\ub97c \ud1b5\ud574 \ub450 \uc30d\ub465\uc774 \ub538 \ubc00\ub77c \ub85c\uc988(Myla Rose)\uc640 \uc0e4\ub97c\ub80c \ub9ac\ubc14(Charlene Riva)\uac00 7\uc6d4 23\uc77c\uc5d0 \ud0c4\uc0dd\ud588\uc74c\uc744 \uc54c\ub838\ub2e4.", "paragraph_answer": "\ud398\ub354\ub7ec\ub294 2009\ub144 4\uc6d4 \uc804 \uc5ec\uc790 \ud504\ub85c \ud14c\ub2c8\uc2a4 \uc120\uc218\uc774\uc790 \uadf8\uc758 \uc624\ub79c \uc5f0\uc778\uc774\uc5c8\ub358 \ubbf8\ub974\uce74 \ubc14\ube0c\ub9ac\ub125 \uacfc \uacb0\ud63c\ud558\uc600\ub2e4. \ub450 \uc0ac\ub78c\uc740 2000\ub144 \uc2dc\ub4dc\ub2c8 \uc62c\ub9bc\ud53d\uc5d0\uc11c \ud14c\ub2c8\uc2a4 \uc885\ubaa9 \uc2a4\uc704\uc2a4 \uad6d\uac00\ub300\ud45c\ub85c \ud568\uaed8 \ucd9c\uc804\ud558\uba74\uc11c \ub9cc\ub098\uac8c \ub418\uc5c8\ub2e4. \ubc14\ube0c\ub9ac\ub125\uc740 \uace0\uc9c8\uc801\uc778 \ubc1c \ubd80\uc0c1 \ub54c\ubb38\uc5d0 2002\ub144 \ud22c\uc5b4\uc5d0\uc11c \uc740\ud1f4\ud558\uc600\uace0 \uc774\ud6c4 \ud398\ub354\ub7ec\uc758 \ud64d\ubcf4 \ub9e4\ub2c8\uc800 \uc5ed\ud560\uc744 \ub2f4\ub2f9\ud558\uc600\ub2e4. \uc624\ub79c \uc5f0\uc778 \uad00\uacc4\ub97c \uc9c0\uc18d\ud558\ub358 \uc774\ub4e4\uc740 2009\ub144 3\uc6d4\uc5d0 \ud398\ub354\ub7ec\uc758 \ud648\ud398\uc774\uc9c0\ub97c \ud1b5\ud574 \ubc14\ube0c\ub9ac\ub125\uc758 \uc784\uc2e0\uc744 \ubc1c\ud45c\ud558\uc600\uc73c\uba70, \uc774\ud6c4 \ub450 \uc0ac\ub78c\uc740 2009\ub144 4\uc6d4 11\uc77c \ubc14\uc824\uc5d0\uc11c \uac00\uae4c\uc6b4 \uce5c\ucc99\ub4e4\uacfc \uce5c\uad6c\ub4e4\ub9cc\uc774 \ucc38\uc11d\ud55c \uac00\uc6b4\ub370 \uc870\ucd10\ud55c \uacb0\ud63c\uc2dd\uc744 \uc62c\ub838\ub2e4. 2009\ub144 7\uc6d4 24\uc77c, \ud398\ub354\ub7ec\ub294 \uc790\uc2e0\uc758 \ud398\uc774\uc2a4\ubd81 \ud648\ud398\uc774\uc9c0\ub97c \ud1b5\ud574 \ub450 \uc30d\ub465\uc774 \ub538 \ubc00\ub77c \ub85c\uc988(Myla Rose)\uc640 \uc0e4\ub97c\ub80c \ub9ac\ubc14(Charlene Riva)\uac00 7\uc6d4 23\uc77c\uc5d0 \ud0c4\uc0dd\ud588\uc74c\uc744 \uc54c\ub838\ub2e4.", "sentence_answer": "\ud398\ub354\ub7ec\ub294 2009\ub144 4\uc6d4 \uc804 \uc5ec\uc790 \ud504\ub85c \ud14c\ub2c8\uc2a4 \uc120\uc218\uc774\uc790 \uadf8\uc758 \uc624\ub79c \uc5f0\uc778\uc774\uc5c8\ub358 \ubbf8\ub974\uce74 \ubc14\ube0c\ub9ac\ub125 \uacfc \uacb0\ud63c\ud558\uc600\ub2e4.", "paragraph_id": "6496547-1-0", "paragraph_question": "question: \ub85c\uc800 \ud398\ub354\ub7ec\uac00 2009\ub144 \uacb0\ud63c\ud55c \uc0ac\ub78c\uc758 \uc774\ub984\uc740?, context: \ud398\ub354\ub7ec\ub294 2009\ub144 4\uc6d4 \uc804 \uc5ec\uc790 \ud504\ub85c \ud14c\ub2c8\uc2a4 \uc120\uc218\uc774\uc790 \uadf8\uc758 \uc624\ub79c \uc5f0\uc778\uc774\uc5c8\ub358 \ubbf8\ub974\uce74 \ubc14\ube0c\ub9ac\ub125\uacfc \uacb0\ud63c\ud558\uc600\ub2e4. \ub450 \uc0ac\ub78c\uc740 2000\ub144 \uc2dc\ub4dc\ub2c8 \uc62c\ub9bc\ud53d\uc5d0\uc11c \ud14c\ub2c8\uc2a4 \uc885\ubaa9 \uc2a4\uc704\uc2a4 \uad6d\uac00\ub300\ud45c\ub85c \ud568\uaed8 \ucd9c\uc804\ud558\uba74\uc11c \ub9cc\ub098\uac8c \ub418\uc5c8\ub2e4. \ubc14\ube0c\ub9ac\ub125\uc740 \uace0\uc9c8\uc801\uc778 \ubc1c \ubd80\uc0c1 \ub54c\ubb38\uc5d0 2002\ub144 \ud22c\uc5b4\uc5d0\uc11c \uc740\ud1f4\ud558\uc600\uace0 \uc774\ud6c4 \ud398\ub354\ub7ec\uc758 \ud64d\ubcf4 \ub9e4\ub2c8\uc800 \uc5ed\ud560\uc744 \ub2f4\ub2f9\ud558\uc600\ub2e4. \uc624\ub79c \uc5f0\uc778 \uad00\uacc4\ub97c \uc9c0\uc18d\ud558\ub358 \uc774\ub4e4\uc740 2009\ub144 3\uc6d4\uc5d0 \ud398\ub354\ub7ec\uc758 \ud648\ud398\uc774\uc9c0\ub97c \ud1b5\ud574 \ubc14\ube0c\ub9ac\ub125\uc758 \uc784\uc2e0\uc744 \ubc1c\ud45c\ud558\uc600\uc73c\uba70, \uc774\ud6c4 \ub450 \uc0ac\ub78c\uc740 2009\ub144 4\uc6d4 11\uc77c \ubc14\uc824\uc5d0\uc11c \uac00\uae4c\uc6b4 \uce5c\ucc99\ub4e4\uacfc \uce5c\uad6c\ub4e4\ub9cc\uc774 \ucc38\uc11d\ud55c \uac00\uc6b4\ub370 \uc870\ucd10\ud55c \uacb0\ud63c\uc2dd\uc744 \uc62c\ub838\ub2e4. 2009\ub144 7\uc6d4 24\uc77c, \ud398\ub354\ub7ec\ub294 \uc790\uc2e0\uc758 \ud398\uc774\uc2a4\ubd81 \ud648\ud398\uc774\uc9c0\ub97c \ud1b5\ud574 \ub450 \uc30d\ub465\uc774 \ub538 \ubc00\ub77c \ub85c\uc988(Myla Rose)\uc640 \uc0e4\ub97c\ub80c \ub9ac\ubc14(Charlene Riva)\uac00 7\uc6d4 23\uc77c\uc5d0 \ud0c4\uc0dd\ud588\uc74c\uc744 \uc54c\ub838\ub2e4."} {"question": "\uc12c\uba38\uc2ac\ub7a8 2017\uc758 3vs3 \ud0dc\uadf8\ud300 \ub9e4\uce58\uc5d0\uc11c \ub354 \ubbf8\uc988\ub294 \ubcf4\uc774\uc988\uc640 \ud300\uc744 \ub9fa\uc740 \ub204\uad6c\ub97c \uc0c1\ub300\ub85c \uc2b9\ub9ac\ud558\uc600\ub294\uac00?", "paragraph": "2017\ub144 6\uc6d4 19\uc77c WWE Raw\uc5d0\uc11c \ubcf4 \ub308\ub7ec\uc2a4, \ucee4\ud2f0\uc2a4 \uc561\uc2ac\uacfc \ud568\uaed8 \ubbf8\uc988\ud22c\ub77c\uc9c0\ub77c\ub294 \ud300\uc744 \uacb0\uc815\uc744 \ud558\uac8c \ub41c\ub2e4. \uadf8\ub9ac\uace0 \uadf8\ub808\uc774\ud2b8 \ubcfc\uc2a4 \uc624\ube0c \ud30c\uc774\uc5b4 2017\uc5d0\uc11c \ud0c0\uc774\ud2c0\uc744 \ub05d\ub0b4 \ubc29\uc5b4\ub97c \ud558\uace0 \uc0ac\uc218\ud55c\ub2e4. \uadf8\ub9ac\uace0 \uc774\ud6c4 \uc12c\uba38\uc2ac\ub7a8 2017\uc5d0\uc11c \ud558\ub514 \ubcf4\uc774\uc988(\uc81c\ud504 \ud558\ub514, \ub9e4\ud2b8 \ud558\ub514)\uc640 \ud300\uc744 \ub9fa\uc740 \uc81c\uc774\uc2a8 \uc870\ub358\uc744 \uc0c1\ub300\ub85c 3vs3 \ud0dc\uadf8\ud300 \ub9e4\uce58\uc5d0\uc11c \uc2b9\ub9ac\ub97c \ud558\uace0 2017\ub144 9\uc6d4 4\uc77c WWE RAW\uc5d0\uc11c\ub294 \uc55e\uc804 \uc8fc\uc5d0 15\uc778 \ubc30\ud2c0\ub85c\uc584\uc744 \uc6b0\uc2b9\ud55c \uc81c\ud504 \ud558\ub514\uc640 \ubc29\uc5b4\uc804\uc744 \uce58\ub904 \uc2b9\ub9ac\ud55c\ub2e4. \uadf8\ub9ac\uace0 \ub178 \uba38\uc2dc 2017\uc5d0\uc11c \uc0c1\ub300 \uc81c\uc774\uc2a8 \uc870\ub358\uc744 \uc2b9\ub9ac\ub97c \ud55c\ub2e4. \ub2e4\uc74c\ub0a0 \ubbf8\uc988 TV\ub97c \uc5f4\uc5b4 \ub85c\ub9cc \ub808\uc778\uc988\ub97c \uac8c\uc2a4\ud2b8\ub85c \ubd80\ub978 \ub4a4 \uc124\uc804\uc744 \ubc8c\uc774\ub2e4\uac00 \uc575\uae00\uc758 \uc911\uc7ac\ub85c \uacbd\uae30\ub97c \uce58\ub974\uc9c0\ub9cc \ubbf8\uc988\ud22c\ub77c\uc9c0\uc758 \ub09c\uc785\uc73c\ub85c \ub178 \ucee8\ud14c\uc2a4\ud2b8 \uc774\ud6c4 \ub85c\ub9cc\uc5d0\uac8c \uc2a4\uceec \ud06c\ub7ec\uc2f1 \ud53c\ub0a0\ub808\ub97c \uba39\uc778 \ub4a4 \ub354 \uc2e4\ub4dc\uc758 \uc0c1\uc9d5\uc778 \ud53c\uc2a4\ud2b8 \ubc94\ud504\ub97c \ucde8\ud588\ub2e4. 2017\ub144 10\uc6d4 2\uc77c\uc5d0\ub294 \uc790\uc2e0\uc758 \uc778\ud130\ucee8\ud2f0\ub128\ud0c8 \ud0c0\uc774\ud2c0\uc744 \uac78\uace0 \ub85c\ub9cc\uacfc \ub300\uacb0\ud588\ub294\ub370 \uc774\ubc88\uc5d4 \ub354 \ubc14\uc640 \ud568\uaed8 \ub85c\ub9cc\uc744 \uacf5\uaca9 \uc2e4\ub4dc\uc758 \ud53c\ub2c8\uc26c\ub294 '\ud2b8\ub9ac\ud50c \ud30c\uc6cc\ubc24'\uc744 \uc2dc\uc804\ud588\ub2e4. WWE TLC 2017 \ub2f9\uc77c\uc5d0\ub294 \ube0c\ub77c\uc6b4 \uc2a4\ud2b8\ub85c\uc6b0\uba3c, \ucf00\uc778\uc774 \ucd94\uac00\ub3fc 5vs3 TLC\ub9e4\uce58\ub85c \uce58\ub904\uc84c\ub294\ub370 \ub354 \ubc14, \ucf00\uc778\uacfc \uc9dc\uace0 \uc2a4\ud2b8\ub85c\uc6b0\uba3c\uc744 \ubc30\uc81c\ud55c \ucc44 \uacbd\uae30\ub97c \uce58\ub974\ub2e4\uac00 \ub85c\ub9cc \ub300\ud0c0\ub85c \ub098\uc628 \uc575\uae00\uc5d0\uac8c \ub2f9\ud574 \ud328\ubc30\ud55c\ub2e4. \uc11c\ubc14\uc774\ubc84 \uc2dc\ub9ac\uc988 2017\uc5d0\uc11c\ub294 US\ucc54\ud53c\uc5b8 \ubc30\ub7f0 \ucf54\ube48\uacfc\uc758 \ub300\uacb0\uc774 \uc608\uc815\ub41c \uac00\uc6b4\ub370 \uacb0\uad6d \ud328\ud558\uace0 \uc774\ubc88\uc5d0\ub294 2017\ub144 11\uc6d4 20\uc77c WWE Raw\uc5d0\uc11c \ub05d\ub0b4 \ud0c0\uc774\ud2c0\uc744 \uc783\uc73c\uba70 \ub05d\ub0b4 \uc601\ud654\ub97c \ucc0d\uc5b4 \uac10\ucd94\uace0 \ub9d0\uc558\uc73c\ub098 \uadf8\ub9ac\uace0... 2018\ub144 1\uc6d4 8\uc77c WWE Raw\uc5d0\uc11c \ub4f1\uc7a5\uc744 \ud55c\ub2e4. \uadf8 \ud6c4... \uc774\ub4e4 \ubcf4, \ucee4\ud2f0\uc2a4\uc640 \uac19\uc774 \uc81c\uc774\uc2a8, \uc138\uc2a4, \ub85c\ub9cc\uae4c\uc9c0 \uacf5\uaca9\uc744 \ud55c\ub2e4. 2018\ub144 1\uc6d4 22\uc77c WWE Raw\uc5d0\uc11c \ub85c\ub9cc\uc744 \uc774\uae30\uba74\uc11c 2\ubc88\uc9f8 WWE \uc778\ud130\ucf58\ud2f0\ub128\ud0c8 \ucc54\ud53c\uc5b8\uc744 \uac00\uc838\uac00\uac8c \ub418\uc5c8\ub2e4!! 2018\ub144 1\uc6d4 28\uc77c \ub85c\uc584\ub7fc\ube14 \ub9e4\uce58\uc5d0 \ucc38\uac00\ub97c \ud588\uc73c\ub098 \uc2e4\ud328\ud588\ub2e4. 2018\ub144 1\uc6d4 29\uc77c WWE Raw\uc5d0\uc11c \ud0c0\uc774\ud2c0\uc744 \uac78\uace0 \ub85c\ub9cc\uc744 \uc0c1\ub300\ub97c \ud588\uc9c0\ub9cc \ub864\uc5c5\uc73c\ub85c \ub9c8\ubb34\ub9ac\ub85c \uc774\uae34\ub2e4!! \uadf8\ub7ec\ub2e4\uac00 WWE \ub808\uc2ac\ub9e4\ub2c8\uc544 34\uc5d0\uc11c \ud2b8\ub9ac\ud50c \uc4f0\ub81b\ub9e4\uce58\uc5d0\uc11c \ud0c0\uc774\ud2c0 \ub3c4\uc804 \ud588\uc9c0\ub9cc \ubc29\uc5b4\ub97c \ud558\uc9c0 \ubabb\ud558\uace0 \ud540 \ubca0\uc77c\ub7ec\uc758 \uc808\uce5c \uc138\uc2a4 \ub864\ub9b0\uc2a4\uac00 \uac00\uc838\uac00\uac8c \ub418\uba74\uc11c \uacb0\uad6d \ub610 \ud55c\ubc88 \ucc54\ud53c\uc5b8 \ubca8\ud2b8\ub9c8\uc800 \uc783\uc5b4\ubc84\ub9ac\uac8c \ub41c\ub2e4.", "answer": "\uc81c\uc774\uc2a8 \uc870\ub358", "sentence": "\uadf8\ub9ac\uace0 \uc774\ud6c4 \uc12c\uba38\uc2ac\ub7a8 2017\uc5d0\uc11c \ud558\ub514 \ubcf4\uc774\uc988(\uc81c\ud504 \ud558\ub514, \ub9e4\ud2b8 \ud558\ub514)\uc640 \ud300\uc744 \ub9fa\uc740 \uc81c\uc774\uc2a8 \uc870\ub358 \uc744 \uc0c1\ub300\ub85c 3vs3 \ud0dc\uadf8\ud300 \ub9e4\uce58\uc5d0\uc11c \uc2b9\ub9ac\ub97c \ud558\uace0 2017\ub144 9\uc6d4 4\uc77c WWE RAW\uc5d0\uc11c\ub294 \uc55e\uc804 \uc8fc\uc5d0 15\uc778 \ubc30\ud2c0\ub85c\uc584\uc744 \uc6b0\uc2b9\ud55c \uc81c\ud504 \ud558\ub514\uc640 \ubc29\uc5b4\uc804\uc744 \uce58\ub904 \uc2b9\ub9ac\ud55c\ub2e4.", "paragraph_sentence": "2017\ub144 6\uc6d4 19\uc77c WWE Raw\uc5d0\uc11c \ubcf4 \ub308\ub7ec\uc2a4, \ucee4\ud2f0\uc2a4 \uc561\uc2ac\uacfc \ud568\uaed8 \ubbf8\uc988\ud22c\ub77c\uc9c0\ub77c\ub294 \ud300\uc744 \uacb0\uc815\uc744 \ud558\uac8c \ub41c\ub2e4. \uadf8\ub9ac\uace0 \uadf8\ub808\uc774\ud2b8 \ubcfc\uc2a4 \uc624\ube0c \ud30c\uc774\uc5b4 2017\uc5d0\uc11c \ud0c0\uc774\ud2c0\uc744 \ub05d\ub0b4 \ubc29\uc5b4\ub97c \ud558\uace0 \uc0ac\uc218\ud55c\ub2e4. \uadf8\ub9ac\uace0 \uc774\ud6c4 \uc12c\uba38\uc2ac\ub7a8 2017\uc5d0\uc11c \ud558\ub514 \ubcf4\uc774\uc988(\uc81c\ud504 \ud558\ub514, \ub9e4\ud2b8 \ud558\ub514)\uc640 \ud300\uc744 \ub9fa\uc740 \uc81c\uc774\uc2a8 \uc870\ub358 \uc744 \uc0c1\ub300\ub85c 3vs3 \ud0dc\uadf8\ud300 \ub9e4\uce58\uc5d0\uc11c \uc2b9\ub9ac\ub97c \ud558\uace0 2017\ub144 9\uc6d4 4\uc77c WWE RAW\uc5d0\uc11c\ub294 \uc55e\uc804 \uc8fc\uc5d0 15\uc778 \ubc30\ud2c0\ub85c\uc584\uc744 \uc6b0\uc2b9\ud55c \uc81c\ud504 \ud558\ub514\uc640 \ubc29\uc5b4\uc804\uc744 \uce58\ub904 \uc2b9\ub9ac\ud55c\ub2e4. \uadf8\ub9ac\uace0 \ub178 \uba38\uc2dc 2017\uc5d0\uc11c \uc0c1\ub300 \uc81c\uc774\uc2a8 \uc870\ub358\uc744 \uc2b9\ub9ac\ub97c \ud55c\ub2e4. \ub2e4\uc74c\ub0a0 \ubbf8\uc988 TV\ub97c \uc5f4\uc5b4 \ub85c\ub9cc \ub808\uc778\uc988\ub97c \uac8c\uc2a4\ud2b8\ub85c \ubd80\ub978 \ub4a4 \uc124\uc804\uc744 \ubc8c\uc774\ub2e4\uac00 \uc575\uae00\uc758 \uc911\uc7ac\ub85c \uacbd\uae30\ub97c \uce58\ub974\uc9c0\ub9cc \ubbf8\uc988\ud22c\ub77c\uc9c0\uc758 \ub09c\uc785\uc73c\ub85c \ub178 \ucee8\ud14c\uc2a4\ud2b8 \uc774\ud6c4 \ub85c\ub9cc\uc5d0\uac8c \uc2a4\uceec \ud06c\ub7ec\uc2f1 \ud53c\ub0a0\ub808\ub97c \uba39\uc778 \ub4a4 \ub354 \uc2e4\ub4dc\uc758 \uc0c1\uc9d5\uc778 \ud53c\uc2a4\ud2b8 \ubc94\ud504\ub97c \ucde8\ud588\ub2e4. 2017\ub144 10\uc6d4 2\uc77c\uc5d0\ub294 \uc790\uc2e0\uc758 \uc778\ud130\ucee8\ud2f0\ub128\ud0c8 \ud0c0\uc774\ud2c0\uc744 \uac78\uace0 \ub85c\ub9cc\uacfc \ub300\uacb0\ud588\ub294\ub370 \uc774\ubc88\uc5d4 \ub354 \ubc14\uc640 \ud568\uaed8 \ub85c\ub9cc\uc744 \uacf5\uaca9 \uc2e4\ub4dc\uc758 \ud53c\ub2c8\uc26c\ub294 '\ud2b8\ub9ac\ud50c \ud30c\uc6cc\ubc24'\uc744 \uc2dc\uc804\ud588\ub2e4. WWE TLC 2017 \ub2f9\uc77c\uc5d0\ub294 \ube0c\ub77c\uc6b4 \uc2a4\ud2b8\ub85c\uc6b0\uba3c, \ucf00\uc778\uc774 \ucd94\uac00\ub3fc 5vs3 TLC\ub9e4\uce58\ub85c \uce58\ub904\uc84c\ub294\ub370 \ub354 \ubc14, \ucf00\uc778\uacfc \uc9dc\uace0 \uc2a4\ud2b8\ub85c\uc6b0\uba3c\uc744 \ubc30\uc81c\ud55c \ucc44 \uacbd\uae30\ub97c \uce58\ub974\ub2e4\uac00 \ub85c\ub9cc \ub300\ud0c0\ub85c \ub098\uc628 \uc575\uae00\uc5d0\uac8c \ub2f9\ud574 \ud328\ubc30\ud55c\ub2e4. \uc11c\ubc14\uc774\ubc84 \uc2dc\ub9ac\uc988 2017\uc5d0\uc11c\ub294 US\ucc54\ud53c\uc5b8 \ubc30\ub7f0 \ucf54\ube48\uacfc\uc758 \ub300\uacb0\uc774 \uc608\uc815\ub41c \uac00\uc6b4\ub370 \uacb0\uad6d \ud328\ud558\uace0 \uc774\ubc88\uc5d0\ub294 2017\ub144 11\uc6d4 20\uc77c WWE Raw\uc5d0\uc11c \ub05d\ub0b4 \ud0c0\uc774\ud2c0\uc744 \uc783\uc73c\uba70 \ub05d\ub0b4 \uc601\ud654\ub97c \ucc0d\uc5b4 \uac10\ucd94\uace0 \ub9d0\uc558\uc73c\ub098 \uadf8\ub9ac\uace0... 2018\ub144 1\uc6d4 8\uc77c WWE Raw\uc5d0\uc11c \ub4f1\uc7a5\uc744 \ud55c\ub2e4. \uadf8 \ud6c4... \uc774\ub4e4 \ubcf4, \ucee4\ud2f0\uc2a4\uc640 \uac19\uc774 \uc81c\uc774\uc2a8, \uc138\uc2a4, \ub85c\ub9cc\uae4c\uc9c0 \uacf5\uaca9\uc744 \ud55c\ub2e4. 2018\ub144 1\uc6d4 22\uc77c WWE Raw\uc5d0\uc11c \ub85c\ub9cc\uc744 \uc774\uae30\uba74\uc11c 2\ubc88\uc9f8 WWE \uc778\ud130\ucf58\ud2f0\ub128\ud0c8 \ucc54\ud53c\uc5b8\uc744 \uac00\uc838\uac00\uac8c \ub418\uc5c8\ub2e4!! 2018\ub144 1\uc6d4 28\uc77c \ub85c\uc584\ub7fc\ube14 \ub9e4\uce58\uc5d0 \ucc38\uac00\ub97c \ud588\uc73c\ub098 \uc2e4\ud328\ud588\ub2e4. 2018\ub144 1\uc6d4 29\uc77c WWE Raw\uc5d0\uc11c \ud0c0\uc774\ud2c0\uc744 \uac78\uace0 \ub85c\ub9cc\uc744 \uc0c1\ub300\ub97c \ud588\uc9c0\ub9cc \ub864\uc5c5\uc73c\ub85c \ub9c8\ubb34\ub9ac\ub85c \uc774\uae34\ub2e4! ! \uadf8\ub7ec\ub2e4\uac00 WWE \ub808\uc2ac\ub9e4\ub2c8\uc544 34\uc5d0\uc11c \ud2b8\ub9ac\ud50c \uc4f0\ub81b\ub9e4\uce58\uc5d0\uc11c \ud0c0\uc774\ud2c0 \ub3c4\uc804 \ud588\uc9c0\ub9cc \ubc29\uc5b4\ub97c \ud558\uc9c0 \ubabb\ud558\uace0 \ud540 \ubca0\uc77c\ub7ec\uc758 \uc808\uce5c \uc138\uc2a4 \ub864\ub9b0\uc2a4\uac00 \uac00\uc838\uac00\uac8c \ub418\uba74\uc11c \uacb0\uad6d \ub610 \ud55c\ubc88 \ucc54\ud53c\uc5b8 \ubca8\ud2b8\ub9c8\uc800 \uc783\uc5b4\ubc84\ub9ac\uac8c \ub41c\ub2e4.", "paragraph_answer": "2017\ub144 6\uc6d4 19\uc77c WWE Raw\uc5d0\uc11c \ubcf4 \ub308\ub7ec\uc2a4, \ucee4\ud2f0\uc2a4 \uc561\uc2ac\uacfc \ud568\uaed8 \ubbf8\uc988\ud22c\ub77c\uc9c0\ub77c\ub294 \ud300\uc744 \uacb0\uc815\uc744 \ud558\uac8c \ub41c\ub2e4. \uadf8\ub9ac\uace0 \uadf8\ub808\uc774\ud2b8 \ubcfc\uc2a4 \uc624\ube0c \ud30c\uc774\uc5b4 2017\uc5d0\uc11c \ud0c0\uc774\ud2c0\uc744 \ub05d\ub0b4 \ubc29\uc5b4\ub97c \ud558\uace0 \uc0ac\uc218\ud55c\ub2e4. \uadf8\ub9ac\uace0 \uc774\ud6c4 \uc12c\uba38\uc2ac\ub7a8 2017\uc5d0\uc11c \ud558\ub514 \ubcf4\uc774\uc988(\uc81c\ud504 \ud558\ub514, \ub9e4\ud2b8 \ud558\ub514)\uc640 \ud300\uc744 \ub9fa\uc740 \uc81c\uc774\uc2a8 \uc870\ub358 \uc744 \uc0c1\ub300\ub85c 3vs3 \ud0dc\uadf8\ud300 \ub9e4\uce58\uc5d0\uc11c \uc2b9\ub9ac\ub97c \ud558\uace0 2017\ub144 9\uc6d4 4\uc77c WWE RAW\uc5d0\uc11c\ub294 \uc55e\uc804 \uc8fc\uc5d0 15\uc778 \ubc30\ud2c0\ub85c\uc584\uc744 \uc6b0\uc2b9\ud55c \uc81c\ud504 \ud558\ub514\uc640 \ubc29\uc5b4\uc804\uc744 \uce58\ub904 \uc2b9\ub9ac\ud55c\ub2e4. \uadf8\ub9ac\uace0 \ub178 \uba38\uc2dc 2017\uc5d0\uc11c \uc0c1\ub300 \uc81c\uc774\uc2a8 \uc870\ub358\uc744 \uc2b9\ub9ac\ub97c \ud55c\ub2e4. \ub2e4\uc74c\ub0a0 \ubbf8\uc988 TV\ub97c \uc5f4\uc5b4 \ub85c\ub9cc \ub808\uc778\uc988\ub97c \uac8c\uc2a4\ud2b8\ub85c \ubd80\ub978 \ub4a4 \uc124\uc804\uc744 \ubc8c\uc774\ub2e4\uac00 \uc575\uae00\uc758 \uc911\uc7ac\ub85c \uacbd\uae30\ub97c \uce58\ub974\uc9c0\ub9cc \ubbf8\uc988\ud22c\ub77c\uc9c0\uc758 \ub09c\uc785\uc73c\ub85c \ub178 \ucee8\ud14c\uc2a4\ud2b8 \uc774\ud6c4 \ub85c\ub9cc\uc5d0\uac8c \uc2a4\uceec \ud06c\ub7ec\uc2f1 \ud53c\ub0a0\ub808\ub97c \uba39\uc778 \ub4a4 \ub354 \uc2e4\ub4dc\uc758 \uc0c1\uc9d5\uc778 \ud53c\uc2a4\ud2b8 \ubc94\ud504\ub97c \ucde8\ud588\ub2e4. 2017\ub144 10\uc6d4 2\uc77c\uc5d0\ub294 \uc790\uc2e0\uc758 \uc778\ud130\ucee8\ud2f0\ub128\ud0c8 \ud0c0\uc774\ud2c0\uc744 \uac78\uace0 \ub85c\ub9cc\uacfc \ub300\uacb0\ud588\ub294\ub370 \uc774\ubc88\uc5d4 \ub354 \ubc14\uc640 \ud568\uaed8 \ub85c\ub9cc\uc744 \uacf5\uaca9 \uc2e4\ub4dc\uc758 \ud53c\ub2c8\uc26c\ub294 '\ud2b8\ub9ac\ud50c \ud30c\uc6cc\ubc24'\uc744 \uc2dc\uc804\ud588\ub2e4. WWE TLC 2017 \ub2f9\uc77c\uc5d0\ub294 \ube0c\ub77c\uc6b4 \uc2a4\ud2b8\ub85c\uc6b0\uba3c, \ucf00\uc778\uc774 \ucd94\uac00\ub3fc 5vs3 TLC\ub9e4\uce58\ub85c \uce58\ub904\uc84c\ub294\ub370 \ub354 \ubc14, \ucf00\uc778\uacfc \uc9dc\uace0 \uc2a4\ud2b8\ub85c\uc6b0\uba3c\uc744 \ubc30\uc81c\ud55c \ucc44 \uacbd\uae30\ub97c \uce58\ub974\ub2e4\uac00 \ub85c\ub9cc \ub300\ud0c0\ub85c \ub098\uc628 \uc575\uae00\uc5d0\uac8c \ub2f9\ud574 \ud328\ubc30\ud55c\ub2e4. \uc11c\ubc14\uc774\ubc84 \uc2dc\ub9ac\uc988 2017\uc5d0\uc11c\ub294 US\ucc54\ud53c\uc5b8 \ubc30\ub7f0 \ucf54\ube48\uacfc\uc758 \ub300\uacb0\uc774 \uc608\uc815\ub41c \uac00\uc6b4\ub370 \uacb0\uad6d \ud328\ud558\uace0 \uc774\ubc88\uc5d0\ub294 2017\ub144 11\uc6d4 20\uc77c WWE Raw\uc5d0\uc11c \ub05d\ub0b4 \ud0c0\uc774\ud2c0\uc744 \uc783\uc73c\uba70 \ub05d\ub0b4 \uc601\ud654\ub97c \ucc0d\uc5b4 \uac10\ucd94\uace0 \ub9d0\uc558\uc73c\ub098 \uadf8\ub9ac\uace0... 2018\ub144 1\uc6d4 8\uc77c WWE Raw\uc5d0\uc11c \ub4f1\uc7a5\uc744 \ud55c\ub2e4. \uadf8 \ud6c4... \uc774\ub4e4 \ubcf4, \ucee4\ud2f0\uc2a4\uc640 \uac19\uc774 \uc81c\uc774\uc2a8, \uc138\uc2a4, \ub85c\ub9cc\uae4c\uc9c0 \uacf5\uaca9\uc744 \ud55c\ub2e4. 2018\ub144 1\uc6d4 22\uc77c WWE Raw\uc5d0\uc11c \ub85c\ub9cc\uc744 \uc774\uae30\uba74\uc11c 2\ubc88\uc9f8 WWE \uc778\ud130\ucf58\ud2f0\ub128\ud0c8 \ucc54\ud53c\uc5b8\uc744 \uac00\uc838\uac00\uac8c \ub418\uc5c8\ub2e4!! 2018\ub144 1\uc6d4 28\uc77c \ub85c\uc584\ub7fc\ube14 \ub9e4\uce58\uc5d0 \ucc38\uac00\ub97c \ud588\uc73c\ub098 \uc2e4\ud328\ud588\ub2e4. 2018\ub144 1\uc6d4 29\uc77c WWE Raw\uc5d0\uc11c \ud0c0\uc774\ud2c0\uc744 \uac78\uace0 \ub85c\ub9cc\uc744 \uc0c1\ub300\ub97c \ud588\uc9c0\ub9cc \ub864\uc5c5\uc73c\ub85c \ub9c8\ubb34\ub9ac\ub85c \uc774\uae34\ub2e4!! \uadf8\ub7ec\ub2e4\uac00 WWE \ub808\uc2ac\ub9e4\ub2c8\uc544 34\uc5d0\uc11c \ud2b8\ub9ac\ud50c \uc4f0\ub81b\ub9e4\uce58\uc5d0\uc11c \ud0c0\uc774\ud2c0 \ub3c4\uc804 \ud588\uc9c0\ub9cc \ubc29\uc5b4\ub97c \ud558\uc9c0 \ubabb\ud558\uace0 \ud540 \ubca0\uc77c\ub7ec\uc758 \uc808\uce5c \uc138\uc2a4 \ub864\ub9b0\uc2a4\uac00 \uac00\uc838\uac00\uac8c \ub418\uba74\uc11c \uacb0\uad6d \ub610 \ud55c\ubc88 \ucc54\ud53c\uc5b8 \ubca8\ud2b8\ub9c8\uc800 \uc783\uc5b4\ubc84\ub9ac\uac8c \ub41c\ub2e4.", "sentence_answer": "\uadf8\ub9ac\uace0 \uc774\ud6c4 \uc12c\uba38\uc2ac\ub7a8 2017\uc5d0\uc11c \ud558\ub514 \ubcf4\uc774\uc988(\uc81c\ud504 \ud558\ub514, \ub9e4\ud2b8 \ud558\ub514)\uc640 \ud300\uc744 \ub9fa\uc740 \uc81c\uc774\uc2a8 \uc870\ub358 \uc744 \uc0c1\ub300\ub85c 3vs3 \ud0dc\uadf8\ud300 \ub9e4\uce58\uc5d0\uc11c \uc2b9\ub9ac\ub97c \ud558\uace0 2017\ub144 9\uc6d4 4\uc77c WWE RAW\uc5d0\uc11c\ub294 \uc55e\uc804 \uc8fc\uc5d0 15\uc778 \ubc30\ud2c0\ub85c\uc584\uc744 \uc6b0\uc2b9\ud55c \uc81c\ud504 \ud558\ub514\uc640 \ubc29\uc5b4\uc804\uc744 \uce58\ub904 \uc2b9\ub9ac\ud55c\ub2e4.", "paragraph_id": "6491175-10-0", "paragraph_question": "question: \uc12c\uba38\uc2ac\ub7a8 2017\uc758 3vs3 \ud0dc\uadf8\ud300 \ub9e4\uce58\uc5d0\uc11c \ub354 \ubbf8\uc988\ub294 \ubcf4\uc774\uc988\uc640 \ud300\uc744 \ub9fa\uc740 \ub204\uad6c\ub97c \uc0c1\ub300\ub85c \uc2b9\ub9ac\ud558\uc600\ub294\uac00?, context: 2017\ub144 6\uc6d4 19\uc77c WWE Raw\uc5d0\uc11c \ubcf4 \ub308\ub7ec\uc2a4, \ucee4\ud2f0\uc2a4 \uc561\uc2ac\uacfc \ud568\uaed8 \ubbf8\uc988\ud22c\ub77c\uc9c0\ub77c\ub294 \ud300\uc744 \uacb0\uc815\uc744 \ud558\uac8c \ub41c\ub2e4. \uadf8\ub9ac\uace0 \uadf8\ub808\uc774\ud2b8 \ubcfc\uc2a4 \uc624\ube0c \ud30c\uc774\uc5b4 2017\uc5d0\uc11c \ud0c0\uc774\ud2c0\uc744 \ub05d\ub0b4 \ubc29\uc5b4\ub97c \ud558\uace0 \uc0ac\uc218\ud55c\ub2e4. \uadf8\ub9ac\uace0 \uc774\ud6c4 \uc12c\uba38\uc2ac\ub7a8 2017\uc5d0\uc11c \ud558\ub514 \ubcf4\uc774\uc988(\uc81c\ud504 \ud558\ub514, \ub9e4\ud2b8 \ud558\ub514)\uc640 \ud300\uc744 \ub9fa\uc740 \uc81c\uc774\uc2a8 \uc870\ub358\uc744 \uc0c1\ub300\ub85c 3vs3 \ud0dc\uadf8\ud300 \ub9e4\uce58\uc5d0\uc11c \uc2b9\ub9ac\ub97c \ud558\uace0 2017\ub144 9\uc6d4 4\uc77c WWE RAW\uc5d0\uc11c\ub294 \uc55e\uc804 \uc8fc\uc5d0 15\uc778 \ubc30\ud2c0\ub85c\uc584\uc744 \uc6b0\uc2b9\ud55c \uc81c\ud504 \ud558\ub514\uc640 \ubc29\uc5b4\uc804\uc744 \uce58\ub904 \uc2b9\ub9ac\ud55c\ub2e4. \uadf8\ub9ac\uace0 \ub178 \uba38\uc2dc 2017\uc5d0\uc11c \uc0c1\ub300 \uc81c\uc774\uc2a8 \uc870\ub358\uc744 \uc2b9\ub9ac\ub97c \ud55c\ub2e4. \ub2e4\uc74c\ub0a0 \ubbf8\uc988 TV\ub97c \uc5f4\uc5b4 \ub85c\ub9cc \ub808\uc778\uc988\ub97c \uac8c\uc2a4\ud2b8\ub85c \ubd80\ub978 \ub4a4 \uc124\uc804\uc744 \ubc8c\uc774\ub2e4\uac00 \uc575\uae00\uc758 \uc911\uc7ac\ub85c \uacbd\uae30\ub97c \uce58\ub974\uc9c0\ub9cc \ubbf8\uc988\ud22c\ub77c\uc9c0\uc758 \ub09c\uc785\uc73c\ub85c \ub178 \ucee8\ud14c\uc2a4\ud2b8 \uc774\ud6c4 \ub85c\ub9cc\uc5d0\uac8c \uc2a4\uceec \ud06c\ub7ec\uc2f1 \ud53c\ub0a0\ub808\ub97c \uba39\uc778 \ub4a4 \ub354 \uc2e4\ub4dc\uc758 \uc0c1\uc9d5\uc778 \ud53c\uc2a4\ud2b8 \ubc94\ud504\ub97c \ucde8\ud588\ub2e4. 2017\ub144 10\uc6d4 2\uc77c\uc5d0\ub294 \uc790\uc2e0\uc758 \uc778\ud130\ucee8\ud2f0\ub128\ud0c8 \ud0c0\uc774\ud2c0\uc744 \uac78\uace0 \ub85c\ub9cc\uacfc \ub300\uacb0\ud588\ub294\ub370 \uc774\ubc88\uc5d4 \ub354 \ubc14\uc640 \ud568\uaed8 \ub85c\ub9cc\uc744 \uacf5\uaca9 \uc2e4\ub4dc\uc758 \ud53c\ub2c8\uc26c\ub294 '\ud2b8\ub9ac\ud50c \ud30c\uc6cc\ubc24'\uc744 \uc2dc\uc804\ud588\ub2e4. WWE TLC 2017 \ub2f9\uc77c\uc5d0\ub294 \ube0c\ub77c\uc6b4 \uc2a4\ud2b8\ub85c\uc6b0\uba3c, \ucf00\uc778\uc774 \ucd94\uac00\ub3fc 5vs3 TLC\ub9e4\uce58\ub85c \uce58\ub904\uc84c\ub294\ub370 \ub354 \ubc14, \ucf00\uc778\uacfc \uc9dc\uace0 \uc2a4\ud2b8\ub85c\uc6b0\uba3c\uc744 \ubc30\uc81c\ud55c \ucc44 \uacbd\uae30\ub97c \uce58\ub974\ub2e4\uac00 \ub85c\ub9cc \ub300\ud0c0\ub85c \ub098\uc628 \uc575\uae00\uc5d0\uac8c \ub2f9\ud574 \ud328\ubc30\ud55c\ub2e4. \uc11c\ubc14\uc774\ubc84 \uc2dc\ub9ac\uc988 2017\uc5d0\uc11c\ub294 US\ucc54\ud53c\uc5b8 \ubc30\ub7f0 \ucf54\ube48\uacfc\uc758 \ub300\uacb0\uc774 \uc608\uc815\ub41c \uac00\uc6b4\ub370 \uacb0\uad6d \ud328\ud558\uace0 \uc774\ubc88\uc5d0\ub294 2017\ub144 11\uc6d4 20\uc77c WWE Raw\uc5d0\uc11c \ub05d\ub0b4 \ud0c0\uc774\ud2c0\uc744 \uc783\uc73c\uba70 \ub05d\ub0b4 \uc601\ud654\ub97c \ucc0d\uc5b4 \uac10\ucd94\uace0 \ub9d0\uc558\uc73c\ub098 \uadf8\ub9ac\uace0... 2018\ub144 1\uc6d4 8\uc77c WWE Raw\uc5d0\uc11c \ub4f1\uc7a5\uc744 \ud55c\ub2e4. \uadf8 \ud6c4... \uc774\ub4e4 \ubcf4, \ucee4\ud2f0\uc2a4\uc640 \uac19\uc774 \uc81c\uc774\uc2a8, \uc138\uc2a4, \ub85c\ub9cc\uae4c\uc9c0 \uacf5\uaca9\uc744 \ud55c\ub2e4. 2018\ub144 1\uc6d4 22\uc77c WWE Raw\uc5d0\uc11c \ub85c\ub9cc\uc744 \uc774\uae30\uba74\uc11c 2\ubc88\uc9f8 WWE \uc778\ud130\ucf58\ud2f0\ub128\ud0c8 \ucc54\ud53c\uc5b8\uc744 \uac00\uc838\uac00\uac8c \ub418\uc5c8\ub2e4!! 2018\ub144 1\uc6d4 28\uc77c \ub85c\uc584\ub7fc\ube14 \ub9e4\uce58\uc5d0 \ucc38\uac00\ub97c \ud588\uc73c\ub098 \uc2e4\ud328\ud588\ub2e4. 2018\ub144 1\uc6d4 29\uc77c WWE Raw\uc5d0\uc11c \ud0c0\uc774\ud2c0\uc744 \uac78\uace0 \ub85c\ub9cc\uc744 \uc0c1\ub300\ub97c \ud588\uc9c0\ub9cc \ub864\uc5c5\uc73c\ub85c \ub9c8\ubb34\ub9ac\ub85c \uc774\uae34\ub2e4!! \uadf8\ub7ec\ub2e4\uac00 WWE \ub808\uc2ac\ub9e4\ub2c8\uc544 34\uc5d0\uc11c \ud2b8\ub9ac\ud50c \uc4f0\ub81b\ub9e4\uce58\uc5d0\uc11c \ud0c0\uc774\ud2c0 \ub3c4\uc804 \ud588\uc9c0\ub9cc \ubc29\uc5b4\ub97c \ud558\uc9c0 \ubabb\ud558\uace0 \ud540 \ubca0\uc77c\ub7ec\uc758 \uc808\uce5c \uc138\uc2a4 \ub864\ub9b0\uc2a4\uac00 \uac00\uc838\uac00\uac8c \ub418\uba74\uc11c \uacb0\uad6d \ub610 \ud55c\ubc88 \ucc54\ud53c\uc5b8 \ubca8\ud2b8\ub9c8\uc800 \uc783\uc5b4\ubc84\ub9ac\uac8c \ub41c\ub2e4."} {"question": "\ubb34\ub974\uc2dc\uc5d8\ub77c\uace0\uc758 \ucfe0\ud398 \ubc84\uc804\uc740 \ucd1d \uba87 \ub300\uac00 \uc81c\uc791\ub418\uc5c8\uc2b5\ub2c8\uae4c?", "paragraph": "\ubb34\ub974\uc2dc\uc5d8\ub77c\uace0 LP640 \ubca0\ub974\uc0ac\uccb4(Versace)\ub294 LP640\uc758 \ud2b9\ubcc4 \ud55c\uc815\ud310 \ubaa8\ub378\uc774\ub2e4. 2006 \ud30c\ub9ac \ubaa8\ud1a0 \uc1fc\uc5d0\uc11c \ucc98\uc74c\uc73c\ub85c \uacf5\uac1c\ub410\ub2e4. \uc548\ud30e\uc73c\ub85c \ud751\ubc31 \uc870\ud654\ub97c \uc774\ub8ec \uc774\uc0c9\uc870\uac00 \ud2b9\uc9d5\uc778 \ucc28\ub7c9 \uc790\ub3d9\ucc28\uc774\ub2e4. \ucfe0\ud398 \ubc84\uc804\uc740 \ucd1d 20\ub300\uac00 \uc81c\uc791\ub410\ub294\ub370, 10\ub300\ub294 \ud770\uc0c9(\uc2e4\ub0b4\ub294 \uac80\uc740\uc0c9)\uc774\uace0, 10\ub300\ub294 \uac80\uc740\uc0c9(\uc2e4\ub0b4\ub294 \ud770\uc0c9)\uc774\ub2e4. \uc778\ud14c\ub9ac\uc5b4\ub294 \ubca0\ub974\uc0ac\uccb4 \uac00\uc8fd\uc73c\ub85c \uce58\uc7a5\ub410\ub294\ub370, \uc911\uc694\ud55c \uac83\uc740 \uc774 \uc790\ub3d9\ucc28\uac00 \uc774\ud0c8\ub9ac\uc544 \ucd9c\uc2e0\uc758 \uc720\uba85 \ud328\uc158\ub514\uc790\uc774\ub108 \uc9c0\uc544\ub2c8 \ubca0\ub974\uc0ac\uccb4(Gianni Versace)\ub97c \uae30\ub9ac\ub294 \uc790\ub3d9\ucc28\ub77c\ub294 \uac83\uc774\ub2e4.(\uc9c0\uc544\ub2c8 \ubca0\ub974\uc0ac\uccb4\ub294 \uc808\ub300 \uc790\ub3d9\ucc28 \ub514\uc790\uc774\ub108\uac00 \uc544\ub2c8\ub2e4.) \ub54c\ubb38\uc5d0 \uc2e4\ub0b4\uc5d0\ub294 \uadf8\uc758 \ud328\uc158 \ub85c\uace0\uac00 \uc0c8\uaca8\uc9c4 \uba85\ud310\uc774 \uc788\ub2e4. 20\ub300\uc758 \ucfe0\ud398 \uc678\uc5d0 20\ub300\uc758 LP640 \ub85c\ub4dc\uc2a4\ud130 \ubca0\ub974\uc0ac\uccb4\ub3c4 \ub9cc\ub4e4\uc5b4\uc84c\ub2e4. \uc55e\uc11c \ub9d0\ud55c \uac83\ucc98\ub7fc 10\ub300\ub294 \uc678\uad00\uc774 \ud770\uc0c9\uc774\uace0 \uc2e4\ub0b4\uac00 \uac80\uc740\uc0c9\uc774\uba70, \ub2e4\ub978 10\ub300\ub294 \uc678\uad00\uc774 \uac80\uc740\uc0c9\uc774\uace0 \uc2e4\ub0b4\uac00 \ud770\uc0c9\uc774\ub2e4.(\ucd1d \uc81c\uc791 \ub300\uc218\ub294 40\ub300\uc774\ub2e4.)", "answer": "20\ub300", "sentence": "\ucfe0\ud398 \ubc84\uc804\uc740 \ucd1d 20\ub300 \uac00 \uc81c\uc791\ub410\ub294\ub370, 10\ub300\ub294 \ud770\uc0c9(\uc2e4\ub0b4\ub294 \uac80\uc740\uc0c9)\uc774\uace0, 10\ub300\ub294 \uac80\uc740\uc0c9(\uc2e4\ub0b4\ub294 \ud770\uc0c9)\uc774\ub2e4.", "paragraph_sentence": "\ubb34\ub974\uc2dc\uc5d8\ub77c\uace0 LP640 \ubca0\ub974\uc0ac\uccb4(Versace)\ub294 LP640\uc758 \ud2b9\ubcc4 \ud55c\uc815\ud310 \ubaa8\ub378\uc774\ub2e4. 2006 \ud30c\ub9ac \ubaa8\ud1a0 \uc1fc\uc5d0\uc11c \ucc98\uc74c\uc73c\ub85c \uacf5\uac1c\ub410\ub2e4. \uc548\ud30e\uc73c\ub85c \ud751\ubc31 \uc870\ud654\ub97c \uc774\ub8ec \uc774\uc0c9\uc870\uac00 \ud2b9\uc9d5\uc778 \ucc28\ub7c9 \uc790\ub3d9\ucc28\uc774\ub2e4. \ucfe0\ud398 \ubc84\uc804\uc740 \ucd1d 20\ub300 \uac00 \uc81c\uc791\ub410\ub294\ub370, 10\ub300\ub294 \ud770\uc0c9(\uc2e4\ub0b4\ub294 \uac80\uc740\uc0c9)\uc774\uace0, 10\ub300\ub294 \uac80\uc740\uc0c9(\uc2e4\ub0b4\ub294 \ud770\uc0c9)\uc774\ub2e4. \uc778\ud14c\ub9ac\uc5b4\ub294 \ubca0\ub974\uc0ac\uccb4 \uac00\uc8fd\uc73c\ub85c \uce58\uc7a5\ub410\ub294\ub370, \uc911\uc694\ud55c \uac83\uc740 \uc774 \uc790\ub3d9\ucc28\uac00 \uc774\ud0c8\ub9ac\uc544 \ucd9c\uc2e0\uc758 \uc720\uba85 \ud328\uc158\ub514\uc790\uc774\ub108 \uc9c0\uc544\ub2c8 \ubca0\ub974\uc0ac\uccb4(Gianni Versace)\ub97c \uae30\ub9ac\ub294 \uc790\ub3d9\ucc28\ub77c\ub294 \uac83\uc774\ub2e4.(\uc9c0\uc544\ub2c8 \ubca0\ub974\uc0ac\uccb4\ub294 \uc808\ub300 \uc790\ub3d9\ucc28 \ub514\uc790\uc774\ub108\uac00 \uc544\ub2c8\ub2e4.) \ub54c\ubb38\uc5d0 \uc2e4\ub0b4\uc5d0\ub294 \uadf8\uc758 \ud328\uc158 \ub85c\uace0\uac00 \uc0c8\uaca8\uc9c4 \uba85\ud310\uc774 \uc788\ub2e4. 20\ub300\uc758 \ucfe0\ud398 \uc678\uc5d0 20\ub300\uc758 LP640 \ub85c\ub4dc\uc2a4\ud130 \ubca0\ub974\uc0ac\uccb4\ub3c4 \ub9cc\ub4e4\uc5b4\uc84c\ub2e4. \uc55e\uc11c \ub9d0\ud55c \uac83\ucc98\ub7fc 10\ub300\ub294 \uc678\uad00\uc774 \ud770\uc0c9\uc774\uace0 \uc2e4\ub0b4\uac00 \uac80\uc740\uc0c9\uc774\uba70, \ub2e4\ub978 10\ub300\ub294 \uc678\uad00\uc774 \uac80\uc740\uc0c9\uc774\uace0 \uc2e4\ub0b4\uac00 \ud770\uc0c9\uc774\ub2e4.(\ucd1d \uc81c\uc791 \ub300\uc218\ub294 40\ub300\uc774\ub2e4.)", "paragraph_answer": "\ubb34\ub974\uc2dc\uc5d8\ub77c\uace0 LP640 \ubca0\ub974\uc0ac\uccb4(Versace)\ub294 LP640\uc758 \ud2b9\ubcc4 \ud55c\uc815\ud310 \ubaa8\ub378\uc774\ub2e4. 2006 \ud30c\ub9ac \ubaa8\ud1a0 \uc1fc\uc5d0\uc11c \ucc98\uc74c\uc73c\ub85c \uacf5\uac1c\ub410\ub2e4. \uc548\ud30e\uc73c\ub85c \ud751\ubc31 \uc870\ud654\ub97c \uc774\ub8ec \uc774\uc0c9\uc870\uac00 \ud2b9\uc9d5\uc778 \ucc28\ub7c9 \uc790\ub3d9\ucc28\uc774\ub2e4. \ucfe0\ud398 \ubc84\uc804\uc740 \ucd1d 20\ub300 \uac00 \uc81c\uc791\ub410\ub294\ub370, 10\ub300\ub294 \ud770\uc0c9(\uc2e4\ub0b4\ub294 \uac80\uc740\uc0c9)\uc774\uace0, 10\ub300\ub294 \uac80\uc740\uc0c9(\uc2e4\ub0b4\ub294 \ud770\uc0c9)\uc774\ub2e4. \uc778\ud14c\ub9ac\uc5b4\ub294 \ubca0\ub974\uc0ac\uccb4 \uac00\uc8fd\uc73c\ub85c \uce58\uc7a5\ub410\ub294\ub370, \uc911\uc694\ud55c \uac83\uc740 \uc774 \uc790\ub3d9\ucc28\uac00 \uc774\ud0c8\ub9ac\uc544 \ucd9c\uc2e0\uc758 \uc720\uba85 \ud328\uc158\ub514\uc790\uc774\ub108 \uc9c0\uc544\ub2c8 \ubca0\ub974\uc0ac\uccb4(Gianni Versace)\ub97c \uae30\ub9ac\ub294 \uc790\ub3d9\ucc28\ub77c\ub294 \uac83\uc774\ub2e4.(\uc9c0\uc544\ub2c8 \ubca0\ub974\uc0ac\uccb4\ub294 \uc808\ub300 \uc790\ub3d9\ucc28 \ub514\uc790\uc774\ub108\uac00 \uc544\ub2c8\ub2e4.) \ub54c\ubb38\uc5d0 \uc2e4\ub0b4\uc5d0\ub294 \uadf8\uc758 \ud328\uc158 \ub85c\uace0\uac00 \uc0c8\uaca8\uc9c4 \uba85\ud310\uc774 \uc788\ub2e4. 20\ub300\uc758 \ucfe0\ud398 \uc678\uc5d0 20\ub300\uc758 LP640 \ub85c\ub4dc\uc2a4\ud130 \ubca0\ub974\uc0ac\uccb4\ub3c4 \ub9cc\ub4e4\uc5b4\uc84c\ub2e4. \uc55e\uc11c \ub9d0\ud55c \uac83\ucc98\ub7fc 10\ub300\ub294 \uc678\uad00\uc774 \ud770\uc0c9\uc774\uace0 \uc2e4\ub0b4\uac00 \uac80\uc740\uc0c9\uc774\uba70, \ub2e4\ub978 10\ub300\ub294 \uc678\uad00\uc774 \uac80\uc740\uc0c9\uc774\uace0 \uc2e4\ub0b4\uac00 \ud770\uc0c9\uc774\ub2e4.(\ucd1d \uc81c\uc791 \ub300\uc218\ub294 40\ub300\uc774\ub2e4.)", "sentence_answer": "\ucfe0\ud398 \ubc84\uc804\uc740 \ucd1d 20\ub300 \uac00 \uc81c\uc791\ub410\ub294\ub370, 10\ub300\ub294 \ud770\uc0c9(\uc2e4\ub0b4\ub294 \uac80\uc740\uc0c9)\uc774\uace0, 10\ub300\ub294 \uac80\uc740\uc0c9(\uc2e4\ub0b4\ub294 \ud770\uc0c9)\uc774\ub2e4.", "paragraph_id": "6513600-5-0", "paragraph_question": "question: \ubb34\ub974\uc2dc\uc5d8\ub77c\uace0\uc758 \ucfe0\ud398 \ubc84\uc804\uc740 \ucd1d \uba87 \ub300\uac00 \uc81c\uc791\ub418\uc5c8\uc2b5\ub2c8\uae4c?, context: \ubb34\ub974\uc2dc\uc5d8\ub77c\uace0 LP640 \ubca0\ub974\uc0ac\uccb4(Versace)\ub294 LP640\uc758 \ud2b9\ubcc4 \ud55c\uc815\ud310 \ubaa8\ub378\uc774\ub2e4. 2006 \ud30c\ub9ac \ubaa8\ud1a0 \uc1fc\uc5d0\uc11c \ucc98\uc74c\uc73c\ub85c \uacf5\uac1c\ub410\ub2e4. \uc548\ud30e\uc73c\ub85c \ud751\ubc31 \uc870\ud654\ub97c \uc774\ub8ec \uc774\uc0c9\uc870\uac00 \ud2b9\uc9d5\uc778 \ucc28\ub7c9 \uc790\ub3d9\ucc28\uc774\ub2e4. \ucfe0\ud398 \ubc84\uc804\uc740 \ucd1d 20\ub300\uac00 \uc81c\uc791\ub410\ub294\ub370, 10\ub300\ub294 \ud770\uc0c9(\uc2e4\ub0b4\ub294 \uac80\uc740\uc0c9)\uc774\uace0, 10\ub300\ub294 \uac80\uc740\uc0c9(\uc2e4\ub0b4\ub294 \ud770\uc0c9)\uc774\ub2e4. \uc778\ud14c\ub9ac\uc5b4\ub294 \ubca0\ub974\uc0ac\uccb4 \uac00\uc8fd\uc73c\ub85c \uce58\uc7a5\ub410\ub294\ub370, \uc911\uc694\ud55c \uac83\uc740 \uc774 \uc790\ub3d9\ucc28\uac00 \uc774\ud0c8\ub9ac\uc544 \ucd9c\uc2e0\uc758 \uc720\uba85 \ud328\uc158\ub514\uc790\uc774\ub108 \uc9c0\uc544\ub2c8 \ubca0\ub974\uc0ac\uccb4(Gianni Versace)\ub97c \uae30\ub9ac\ub294 \uc790\ub3d9\ucc28\ub77c\ub294 \uac83\uc774\ub2e4.(\uc9c0\uc544\ub2c8 \ubca0\ub974\uc0ac\uccb4\ub294 \uc808\ub300 \uc790\ub3d9\ucc28 \ub514\uc790\uc774\ub108\uac00 \uc544\ub2c8\ub2e4.) \ub54c\ubb38\uc5d0 \uc2e4\ub0b4\uc5d0\ub294 \uadf8\uc758 \ud328\uc158 \ub85c\uace0\uac00 \uc0c8\uaca8\uc9c4 \uba85\ud310\uc774 \uc788\ub2e4. 20\ub300\uc758 \ucfe0\ud398 \uc678\uc5d0 20\ub300\uc758 LP640 \ub85c\ub4dc\uc2a4\ud130 \ubca0\ub974\uc0ac\uccb4\ub3c4 \ub9cc\ub4e4\uc5b4\uc84c\ub2e4. \uc55e\uc11c \ub9d0\ud55c \uac83\ucc98\ub7fc 10\ub300\ub294 \uc678\uad00\uc774 \ud770\uc0c9\uc774\uace0 \uc2e4\ub0b4\uac00 \uac80\uc740\uc0c9\uc774\uba70, \ub2e4\ub978 10\ub300\ub294 \uc678\uad00\uc774 \uac80\uc740\uc0c9\uc774\uace0 \uc2e4\ub0b4\uac00 \ud770\uc0c9\uc774\ub2e4.(\ucd1d \uc81c\uc791 \ub300\uc218\ub294 40\ub300\uc774\ub2e4.)"} {"question": "\uc559\ub9ac \uc911\ub839\uc774 \uc790\uc0b4\ud558\uace0 \ucc38\ubaa8\ubcf8\ubd80\uc758 \uc74c\ubaa8\uac00 \ub4dc\ub7ec\ub098\uc790 \uacbd\uc545\ud55c \uc0ac\ub78c\ub4e4\uc740 \ub204\uad6c\uc778\uac00?", "paragraph": "\uc5d0\uc2a4\ud14c\ub77c\uc9c0\uc640 \ud568\uaed8 \ubb38\uc11c\ub97c \uc870\uc791\ud55c \uc559\ub9ac \uc911\ub839\uc774 \uccb4\ud3ec\ub418\uace0 \uac10\ubc29\uc5d0\uc11c \uc790\uc0b4\ud558\uba70 \ucc38\ubaa8\ubcf8\ubd80\uc758 \uc74c\ubaa8\uac00 \ub4dc\ub7ec\ub098\uc790 \uc88c\ud30c \uacf5\ud654\uc8fc\uc758\uc790\ub4e4\uc740 \uacbd\uc545\ud588\ub2e4. \uadf8\ub4e4\uc740 \uac00\ud1a8\ub9ad \uad50\ud68c\uc640 \uad70\ubd80\uac00 \uc548\ubcf4\ub97c \uad6c\uc2e4\ub85c \uc815\uce58\uc640 \uacf5\uacf5\uc0dd\ud65c\uc744 \uc5bc\ub9c8\ub098 \uce68\uc2dd\ud574 \ub4e4\uc5b4\uac14\ub294\uac00\ub97c \uc778\uc2dd\ud558\uae30 \uc2dc\uc791\ud588\ub2e4. \uc7a5 \uc870\ub808\uc2a4\ub294 \uc774 \ub54c \ub4dc\ub808\ud4cc\uc2a4 \uc0ac\uac74\uc744 \ud30c\ud5e4\uce58\ub294 \uc548\ub0b4\uc11c \u300a\uc99d\uac70\ub4e4\u300b\uc774\ub77c\ub294 \ucc45\uc744 \ud3b4\ub0c8\ub2e4. \uc774 \ub54c\ubd80\ud130 \ub4dc\ub808\ud4cc\uc2a4\uc758 \uc7ac\uc2ec\uc744 \uc694\uad6c\ud558\ub294 \ubaa9\uc18c\ub9ac\uac00 \ub192\uc544\uc84c\ub2e4. \ub85c\ub85c\ub974\uc9c0\ub294 \uac10\uc625\uc5d0\uc11c \uc790\uc0b4\ud55c \uc559\ub9ac \uc911\ub839\uc774 \uc790\uc0b4\ud558\uc9c0 \uc54a\uc558\ub2e4\ub294 \uae30\uc0ac\ub97c \uc2e4\uc5c8\ub2e4. \ub85c\ub85c\ub974\ub294 \"\uc608\uc218\ud68c\uc6d0\ub4e4\uc774 \uc559\ub9ac \uc911\ub839\uc758 \uc190\uc5d0 \uba74\ub3c4\ub0a0\uc744 \uc950\uc5b4\uc8fc\uba74\uc11c \ubd88\uba85\uc608 \ud1f4\uc5ed\uc73c\ub85c \ucd94\ubc29\ub418\uaca0\ub290\ub0d0 \uc544\ub2c8\uba74 \uadf8\uc758 \ubbf8\ub9dd\uc778\uc5d0\uac8c \uc5f0\uae08\uc744 \uc8fc\ub294 \uc870\uac74\uc73c\ub85c \uc790\uc0b4\ud558\uaca0\ub290\ub0d0 \ub458 \uc911 \ud558\ub098\ub97c \ud0dd\ud558\ub77c\uace0 \uac15\uc694\ud588\ub2e4\"\ub294 \uac83\uc774\uc5c8\ub2e4.", "answer": "\uc88c\ud30c \uacf5\ud654\uc8fc\uc758\uc790", "sentence": "\uc5d0\uc2a4\ud14c\ub77c\uc9c0\uc640 \ud568\uaed8 \ubb38\uc11c\ub97c \uc870\uc791\ud55c \uc559\ub9ac \uc911\ub839\uc774 \uccb4\ud3ec\ub418\uace0 \uac10\ubc29\uc5d0\uc11c \uc790\uc0b4\ud558\uba70 \ucc38\ubaa8\ubcf8\ubd80\uc758 \uc74c\ubaa8\uac00 \ub4dc\ub7ec\ub098\uc790 \uc88c\ud30c \uacf5\ud654\uc8fc\uc758\uc790 \ub4e4\uc740 \uacbd\uc545\ud588\ub2e4.", "paragraph_sentence": " \uc5d0\uc2a4\ud14c\ub77c\uc9c0\uc640 \ud568\uaed8 \ubb38\uc11c\ub97c \uc870\uc791\ud55c \uc559\ub9ac \uc911\ub839\uc774 \uccb4\ud3ec\ub418\uace0 \uac10\ubc29\uc5d0\uc11c \uc790\uc0b4\ud558\uba70 \ucc38\ubaa8\ubcf8\ubd80\uc758 \uc74c\ubaa8\uac00 \ub4dc\ub7ec\ub098\uc790 \uc88c\ud30c \uacf5\ud654\uc8fc\uc758\uc790 \ub4e4\uc740 \uacbd\uc545\ud588\ub2e4. \uadf8\ub4e4\uc740 \uac00\ud1a8\ub9ad \uad50\ud68c\uc640 \uad70\ubd80\uac00 \uc548\ubcf4\ub97c \uad6c\uc2e4\ub85c \uc815\uce58\uc640 \uacf5\uacf5\uc0dd\ud65c\uc744 \uc5bc\ub9c8\ub098 \uce68\uc2dd\ud574 \ub4e4\uc5b4\uac14\ub294\uac00\ub97c \uc778\uc2dd\ud558\uae30 \uc2dc\uc791\ud588\ub2e4. \uc7a5 \uc870\ub808\uc2a4\ub294 \uc774 \ub54c \ub4dc\ub808\ud4cc\uc2a4 \uc0ac\uac74\uc744 \ud30c\ud5e4\uce58\ub294 \uc548\ub0b4\uc11c \u300a\uc99d\uac70\ub4e4\u300b\uc774\ub77c\ub294 \ucc45\uc744 \ud3b4\ub0c8\ub2e4. \uc774 \ub54c\ubd80\ud130 \ub4dc\ub808\ud4cc\uc2a4\uc758 \uc7ac\uc2ec\uc744 \uc694\uad6c\ud558\ub294 \ubaa9\uc18c\ub9ac\uac00 \ub192\uc544\uc84c\ub2e4. \ub85c\ub85c\ub974\uc9c0\ub294 \uac10\uc625\uc5d0\uc11c \uc790\uc0b4\ud55c \uc559\ub9ac \uc911\ub839\uc774 \uc790\uc0b4\ud558\uc9c0 \uc54a\uc558\ub2e4\ub294 \uae30\uc0ac\ub97c \uc2e4\uc5c8\ub2e4. \ub85c\ub85c\ub974\ub294 \"\uc608\uc218\ud68c\uc6d0\ub4e4\uc774 \uc559\ub9ac \uc911\ub839\uc758 \uc190\uc5d0 \uba74\ub3c4\ub0a0\uc744 \uc950\uc5b4\uc8fc\uba74\uc11c \ubd88\uba85\uc608 \ud1f4\uc5ed\uc73c\ub85c \ucd94\ubc29\ub418\uaca0\ub290\ub0d0 \uc544\ub2c8\uba74 \uadf8\uc758 \ubbf8\ub9dd\uc778\uc5d0\uac8c \uc5f0\uae08\uc744 \uc8fc\ub294 \uc870\uac74\uc73c\ub85c \uc790\uc0b4\ud558\uaca0\ub290\ub0d0 \ub458 \uc911 \ud558\ub098\ub97c \ud0dd\ud558\ub77c\uace0 \uac15\uc694\ud588\ub2e4\"\ub294 \uac83\uc774\uc5c8\ub2e4.", "paragraph_answer": "\uc5d0\uc2a4\ud14c\ub77c\uc9c0\uc640 \ud568\uaed8 \ubb38\uc11c\ub97c \uc870\uc791\ud55c \uc559\ub9ac \uc911\ub839\uc774 \uccb4\ud3ec\ub418\uace0 \uac10\ubc29\uc5d0\uc11c \uc790\uc0b4\ud558\uba70 \ucc38\ubaa8\ubcf8\ubd80\uc758 \uc74c\ubaa8\uac00 \ub4dc\ub7ec\ub098\uc790 \uc88c\ud30c \uacf5\ud654\uc8fc\uc758\uc790 \ub4e4\uc740 \uacbd\uc545\ud588\ub2e4. \uadf8\ub4e4\uc740 \uac00\ud1a8\ub9ad \uad50\ud68c\uc640 \uad70\ubd80\uac00 \uc548\ubcf4\ub97c \uad6c\uc2e4\ub85c \uc815\uce58\uc640 \uacf5\uacf5\uc0dd\ud65c\uc744 \uc5bc\ub9c8\ub098 \uce68\uc2dd\ud574 \ub4e4\uc5b4\uac14\ub294\uac00\ub97c \uc778\uc2dd\ud558\uae30 \uc2dc\uc791\ud588\ub2e4. \uc7a5 \uc870\ub808\uc2a4\ub294 \uc774 \ub54c \ub4dc\ub808\ud4cc\uc2a4 \uc0ac\uac74\uc744 \ud30c\ud5e4\uce58\ub294 \uc548\ub0b4\uc11c \u300a\uc99d\uac70\ub4e4\u300b\uc774\ub77c\ub294 \ucc45\uc744 \ud3b4\ub0c8\ub2e4. \uc774 \ub54c\ubd80\ud130 \ub4dc\ub808\ud4cc\uc2a4\uc758 \uc7ac\uc2ec\uc744 \uc694\uad6c\ud558\ub294 \ubaa9\uc18c\ub9ac\uac00 \ub192\uc544\uc84c\ub2e4. \ub85c\ub85c\ub974\uc9c0\ub294 \uac10\uc625\uc5d0\uc11c \uc790\uc0b4\ud55c \uc559\ub9ac \uc911\ub839\uc774 \uc790\uc0b4\ud558\uc9c0 \uc54a\uc558\ub2e4\ub294 \uae30\uc0ac\ub97c \uc2e4\uc5c8\ub2e4. \ub85c\ub85c\ub974\ub294 \"\uc608\uc218\ud68c\uc6d0\ub4e4\uc774 \uc559\ub9ac \uc911\ub839\uc758 \uc190\uc5d0 \uba74\ub3c4\ub0a0\uc744 \uc950\uc5b4\uc8fc\uba74\uc11c \ubd88\uba85\uc608 \ud1f4\uc5ed\uc73c\ub85c \ucd94\ubc29\ub418\uaca0\ub290\ub0d0 \uc544\ub2c8\uba74 \uadf8\uc758 \ubbf8\ub9dd\uc778\uc5d0\uac8c \uc5f0\uae08\uc744 \uc8fc\ub294 \uc870\uac74\uc73c\ub85c \uc790\uc0b4\ud558\uaca0\ub290\ub0d0 \ub458 \uc911 \ud558\ub098\ub97c \ud0dd\ud558\ub77c\uace0 \uac15\uc694\ud588\ub2e4\"\ub294 \uac83\uc774\uc5c8\ub2e4.", "sentence_answer": "\uc5d0\uc2a4\ud14c\ub77c\uc9c0\uc640 \ud568\uaed8 \ubb38\uc11c\ub97c \uc870\uc791\ud55c \uc559\ub9ac \uc911\ub839\uc774 \uccb4\ud3ec\ub418\uace0 \uac10\ubc29\uc5d0\uc11c \uc790\uc0b4\ud558\uba70 \ucc38\ubaa8\ubcf8\ubd80\uc758 \uc74c\ubaa8\uac00 \ub4dc\ub7ec\ub098\uc790 \uc88c\ud30c \uacf5\ud654\uc8fc\uc758\uc790 \ub4e4\uc740 \uacbd\uc545\ud588\ub2e4.", "paragraph_id": "6513104-8-0", "paragraph_question": "question: \uc559\ub9ac \uc911\ub839\uc774 \uc790\uc0b4\ud558\uace0 \ucc38\ubaa8\ubcf8\ubd80\uc758 \uc74c\ubaa8\uac00 \ub4dc\ub7ec\ub098\uc790 \uacbd\uc545\ud55c \uc0ac\ub78c\ub4e4\uc740 \ub204\uad6c\uc778\uac00?, context: \uc5d0\uc2a4\ud14c\ub77c\uc9c0\uc640 \ud568\uaed8 \ubb38\uc11c\ub97c \uc870\uc791\ud55c \uc559\ub9ac \uc911\ub839\uc774 \uccb4\ud3ec\ub418\uace0 \uac10\ubc29\uc5d0\uc11c \uc790\uc0b4\ud558\uba70 \ucc38\ubaa8\ubcf8\ubd80\uc758 \uc74c\ubaa8\uac00 \ub4dc\ub7ec\ub098\uc790 \uc88c\ud30c \uacf5\ud654\uc8fc\uc758\uc790\ub4e4\uc740 \uacbd\uc545\ud588\ub2e4. \uadf8\ub4e4\uc740 \uac00\ud1a8\ub9ad \uad50\ud68c\uc640 \uad70\ubd80\uac00 \uc548\ubcf4\ub97c \uad6c\uc2e4\ub85c \uc815\uce58\uc640 \uacf5\uacf5\uc0dd\ud65c\uc744 \uc5bc\ub9c8\ub098 \uce68\uc2dd\ud574 \ub4e4\uc5b4\uac14\ub294\uac00\ub97c \uc778\uc2dd\ud558\uae30 \uc2dc\uc791\ud588\ub2e4. \uc7a5 \uc870\ub808\uc2a4\ub294 \uc774 \ub54c \ub4dc\ub808\ud4cc\uc2a4 \uc0ac\uac74\uc744 \ud30c\ud5e4\uce58\ub294 \uc548\ub0b4\uc11c \u300a\uc99d\uac70\ub4e4\u300b\uc774\ub77c\ub294 \ucc45\uc744 \ud3b4\ub0c8\ub2e4. \uc774 \ub54c\ubd80\ud130 \ub4dc\ub808\ud4cc\uc2a4\uc758 \uc7ac\uc2ec\uc744 \uc694\uad6c\ud558\ub294 \ubaa9\uc18c\ub9ac\uac00 \ub192\uc544\uc84c\ub2e4. \ub85c\ub85c\ub974\uc9c0\ub294 \uac10\uc625\uc5d0\uc11c \uc790\uc0b4\ud55c \uc559\ub9ac \uc911\ub839\uc774 \uc790\uc0b4\ud558\uc9c0 \uc54a\uc558\ub2e4\ub294 \uae30\uc0ac\ub97c \uc2e4\uc5c8\ub2e4. \ub85c\ub85c\ub974\ub294 \"\uc608\uc218\ud68c\uc6d0\ub4e4\uc774 \uc559\ub9ac \uc911\ub839\uc758 \uc190\uc5d0 \uba74\ub3c4\ub0a0\uc744 \uc950\uc5b4\uc8fc\uba74\uc11c \ubd88\uba85\uc608 \ud1f4\uc5ed\uc73c\ub85c \ucd94\ubc29\ub418\uaca0\ub290\ub0d0 \uc544\ub2c8\uba74 \uadf8\uc758 \ubbf8\ub9dd\uc778\uc5d0\uac8c \uc5f0\uae08\uc744 \uc8fc\ub294 \uc870\uac74\uc73c\ub85c \uc790\uc0b4\ud558\uaca0\ub290\ub0d0 \ub458 \uc911 \ud558\ub098\ub97c \ud0dd\ud558\ub77c\uace0 \uac15\uc694\ud588\ub2e4\"\ub294 \uac83\uc774\uc5c8\ub2e4."} {"question": "\uc790\ub3d9\ucc28\ub4e4\uc740 \uc5b4\ub5a0\ud55c \ud615\ud0dc\ub85c \ub0a0\uc544\ub2e4\ub2c8\ub294\uac00?", "paragraph": "7\uc6d4 2\uc77c, \uc774\uc0c1\ud55c \ud604\uc0c1\uc774 \uc9c0\uad6c\ub97c \uac10\uc2fc\ub2e4. \ud558\ub298\uc740 \uc774\uae00\uc774\uae00 \ubd88\ud0c0\uc624\ub974\uace0 \ub545\uc740 \uc9c0\uc9c4\uc774 \ub09c \ub4ef \uaca9\ub82c\ud788 \uc694\ub3d9\uce5c\ub2e4. \uc9c1\uacbd 550km, \ubb34\uac8c\uac00 \ub2ec\uc758 4\ubd84\uc758 1\uc774\ub098 \ub418\ub294 \uac70\ub300\ud55c \uad34 \ube44\ud589\ubb3c\uccb4\uac00 \ud0dc\uc591\uc744 \uac00\ub824 \uc9c0\uad6c\ub294 \uadf8 \ube5b\uc744 \uc783\uc5b4\uac04\ub2e4. \uc228\ub9c9\ud788\ub294 \uacf5\ud3ec\uac00 \uc138\uacc4\uc758 \uc8fc\uc694\ub3c4\uc2dc\ub97c \uc5c4\uc2b5\ud558\uae30 \uc2dc\uc791\ud55c\ub2e4. 7\uc6d4 3\uc77c, \uac70\ub300\ud558\uace0 \uae30\uad34\ud55c \ube44\ud589\ubb3c\uccb4\uc5d0\uc11c \ub0b4\ubfdc\ub294 \uac00\uacf5\ud560 \uc704\ub825\uc758 \ubd88\uae30\ub465\uc740 \ub274\uc695\uc758 \ub9c8\ucc9c\ub8e8\uc640 \uc6cc\uc2f1\ud134\uc758 \ubc31\uc545\uad00, \uc774\uc9d1\ud2b8\uc758 \ud53c\ub77c\ubbf8\ub4dc \ub4f1\uc744 \uc21c\uc2dd\uac04\uc5d0 \uc7bf\ub354\ubbf8\ub85c \ub9cc\ub4e4\uace0, \uac70\ub9ac\uc758 \uc790\ub3d9\ucc28\ub4e4\uc740 \ud734\uc9c0\uc870\uac01\ucc98\ub7fc \uacf5\uc911\uc744 \ub0a0\uc544\ub2e4\ub2cc\ub2e4. \uc0ac\ub78c\ub4e4\uc740 \ub2f9\ud669\ud558\uc5ec \uc228\uc744 \uacf3\uc744 \ucc3e\uc9c0\ub9cc \ub3c4\ub9dd\uce60 \uacf3\uc740 \uc544\ubb34\ub370\ub3c4 \uc5c6\ub2e4. \uc778\uac04\uc758 \ud798\uc73c\ub85c \uac10\ub2f9\ud560 \uc218 \uc5c6\ub294 \uc5c4\uccad\ub09c \ud798\uc774 \uc9c0\uad6c\uc5d0 \ub3c4\ub2ec\ud588\uc74c\uc774 \ubc1d\ud600\uc9c4\ub2e4. \uadf8\ub4e4\uc758 \ubaa9\uc801\uc740 \uc644\uc804\ud55c \uc9c0\uad6c\uc758 \ud30c\uba78. 7\uc6d4 4\uc77c, \uc5c4\uccad\ub09c \ud30c\uad34 \uc18d\uc5d0\uc11c \uc0b4\uc544\ub0a8\uc740 \uc9c0\uad6c\uc758 \uc0dd\uc874\uc790\ub4e4\uc740 \uadf8 \ud798\uc744 \ubaa8\uc544 \uac70\ub300\ud55c \uad34\ube44\ud589\ubb3c\uccb4\uc5d0 \ub300\ud56d\ud558\ub824 \ud55c\ub2e4. \uadf8\ub7ec\ub098 \uc678\uacc4\uc778\ub4e4\uc758 \uc9c0\uad6c\ud30c\uad34\ub294 \ub354\uc6b1 \ub9f9\ub82c\ud574\uc9c0\uace0 \uc778\ub958 \uc804\uccb4\uc758 \uc0dd\uc874\uacfc \uc678\uacc4\uc778\uc73c\ub85c\ubd80\ud130 \uc9c0\uad6c\uc758 \ub3c5\ub9bd\uc740 \uc810\uc810 \uc5b4\ub824\uc6cc\uc9c0\ub294\ub370.", "answer": "\ud734\uc9c0\uc870\uac01", "sentence": "\uac70\ub9ac\uc758 \uc790\ub3d9\ucc28\ub4e4\uc740 \ud734\uc9c0\uc870\uac01 \ucc98\ub7fc \uacf5\uc911\uc744 \ub0a0\uc544\ub2e4\ub2cc\ub2e4.", "paragraph_sentence": "7\uc6d4 2\uc77c, \uc774\uc0c1\ud55c \ud604\uc0c1\uc774 \uc9c0\uad6c\ub97c \uac10\uc2fc\ub2e4. \ud558\ub298\uc740 \uc774\uae00\uc774\uae00 \ubd88\ud0c0\uc624\ub974\uace0 \ub545\uc740 \uc9c0\uc9c4\uc774 \ub09c \ub4ef \uaca9\ub82c\ud788 \uc694\ub3d9\uce5c\ub2e4. \uc9c1\uacbd 550km, \ubb34\uac8c\uac00 \ub2ec\uc758 4\ubd84\uc758 1\uc774\ub098 \ub418\ub294 \uac70\ub300\ud55c \uad34 \ube44\ud589\ubb3c\uccb4\uac00 \ud0dc\uc591\uc744 \uac00\ub824 \uc9c0\uad6c\ub294 \uadf8 \ube5b\uc744 \uc783\uc5b4\uac04\ub2e4. \uc228\ub9c9\ud788\ub294 \uacf5\ud3ec\uac00 \uc138\uacc4\uc758 \uc8fc\uc694\ub3c4\uc2dc\ub97c \uc5c4\uc2b5\ud558\uae30 \uc2dc\uc791\ud55c\ub2e4. 7\uc6d4 3\uc77c, \uac70\ub300\ud558\uace0 \uae30\uad34\ud55c \ube44\ud589\ubb3c\uccb4\uc5d0\uc11c \ub0b4\ubfdc\ub294 \uac00\uacf5\ud560 \uc704\ub825\uc758 \ubd88\uae30\ub465\uc740 \ub274\uc695\uc758 \ub9c8\ucc9c\ub8e8\uc640 \uc6cc\uc2f1\ud134\uc758 \ubc31\uc545\uad00, \uc774\uc9d1\ud2b8\uc758 \ud53c\ub77c\ubbf8\ub4dc \ub4f1\uc744 \uc21c\uc2dd\uac04\uc5d0 \uc7bf\ub354\ubbf8\ub85c \ub9cc\ub4e4\uace0, \uac70\ub9ac\uc758 \uc790\ub3d9\ucc28\ub4e4\uc740 \ud734\uc9c0\uc870\uac01 \ucc98\ub7fc \uacf5\uc911\uc744 \ub0a0\uc544\ub2e4\ub2cc\ub2e4. \uc0ac\ub78c\ub4e4\uc740 \ub2f9\ud669\ud558\uc5ec \uc228\uc744 \uacf3\uc744 \ucc3e\uc9c0\ub9cc \ub3c4\ub9dd\uce60 \uacf3\uc740 \uc544\ubb34\ub370\ub3c4 \uc5c6\ub2e4. \uc778\uac04\uc758 \ud798\uc73c\ub85c \uac10\ub2f9\ud560 \uc218 \uc5c6\ub294 \uc5c4\uccad\ub09c \ud798\uc774 \uc9c0\uad6c\uc5d0 \ub3c4\ub2ec\ud588\uc74c\uc774 \ubc1d\ud600\uc9c4\ub2e4. \uadf8\ub4e4\uc758 \ubaa9\uc801\uc740 \uc644\uc804\ud55c \uc9c0\uad6c\uc758 \ud30c\uba78. 7\uc6d4 4\uc77c, \uc5c4\uccad\ub09c \ud30c\uad34 \uc18d\uc5d0\uc11c \uc0b4\uc544\ub0a8\uc740 \uc9c0\uad6c\uc758 \uc0dd\uc874\uc790\ub4e4\uc740 \uadf8 \ud798\uc744 \ubaa8\uc544 \uac70\ub300\ud55c \uad34\ube44\ud589\ubb3c\uccb4\uc5d0 \ub300\ud56d\ud558\ub824 \ud55c\ub2e4. \uadf8\ub7ec\ub098 \uc678\uacc4\uc778\ub4e4\uc758 \uc9c0\uad6c\ud30c\uad34\ub294 \ub354\uc6b1 \ub9f9\ub82c\ud574\uc9c0\uace0 \uc778\ub958 \uc804\uccb4\uc758 \uc0dd\uc874\uacfc \uc678\uacc4\uc778\uc73c\ub85c\ubd80\ud130 \uc9c0\uad6c\uc758 \ub3c5\ub9bd\uc740 \uc810\uc810 \uc5b4\ub824\uc6cc\uc9c0\ub294\ub370.", "paragraph_answer": "7\uc6d4 2\uc77c, \uc774\uc0c1\ud55c \ud604\uc0c1\uc774 \uc9c0\uad6c\ub97c \uac10\uc2fc\ub2e4. \ud558\ub298\uc740 \uc774\uae00\uc774\uae00 \ubd88\ud0c0\uc624\ub974\uace0 \ub545\uc740 \uc9c0\uc9c4\uc774 \ub09c \ub4ef \uaca9\ub82c\ud788 \uc694\ub3d9\uce5c\ub2e4. \uc9c1\uacbd 550km, \ubb34\uac8c\uac00 \ub2ec\uc758 4\ubd84\uc758 1\uc774\ub098 \ub418\ub294 \uac70\ub300\ud55c \uad34 \ube44\ud589\ubb3c\uccb4\uac00 \ud0dc\uc591\uc744 \uac00\ub824 \uc9c0\uad6c\ub294 \uadf8 \ube5b\uc744 \uc783\uc5b4\uac04\ub2e4. \uc228\ub9c9\ud788\ub294 \uacf5\ud3ec\uac00 \uc138\uacc4\uc758 \uc8fc\uc694\ub3c4\uc2dc\ub97c \uc5c4\uc2b5\ud558\uae30 \uc2dc\uc791\ud55c\ub2e4. 7\uc6d4 3\uc77c, \uac70\ub300\ud558\uace0 \uae30\uad34\ud55c \ube44\ud589\ubb3c\uccb4\uc5d0\uc11c \ub0b4\ubfdc\ub294 \uac00\uacf5\ud560 \uc704\ub825\uc758 \ubd88\uae30\ub465\uc740 \ub274\uc695\uc758 \ub9c8\ucc9c\ub8e8\uc640 \uc6cc\uc2f1\ud134\uc758 \ubc31\uc545\uad00, \uc774\uc9d1\ud2b8\uc758 \ud53c\ub77c\ubbf8\ub4dc \ub4f1\uc744 \uc21c\uc2dd\uac04\uc5d0 \uc7bf\ub354\ubbf8\ub85c \ub9cc\ub4e4\uace0, \uac70\ub9ac\uc758 \uc790\ub3d9\ucc28\ub4e4\uc740 \ud734\uc9c0\uc870\uac01 \ucc98\ub7fc \uacf5\uc911\uc744 \ub0a0\uc544\ub2e4\ub2cc\ub2e4. \uc0ac\ub78c\ub4e4\uc740 \ub2f9\ud669\ud558\uc5ec \uc228\uc744 \uacf3\uc744 \ucc3e\uc9c0\ub9cc \ub3c4\ub9dd\uce60 \uacf3\uc740 \uc544\ubb34\ub370\ub3c4 \uc5c6\ub2e4. \uc778\uac04\uc758 \ud798\uc73c\ub85c \uac10\ub2f9\ud560 \uc218 \uc5c6\ub294 \uc5c4\uccad\ub09c \ud798\uc774 \uc9c0\uad6c\uc5d0 \ub3c4\ub2ec\ud588\uc74c\uc774 \ubc1d\ud600\uc9c4\ub2e4. \uadf8\ub4e4\uc758 \ubaa9\uc801\uc740 \uc644\uc804\ud55c \uc9c0\uad6c\uc758 \ud30c\uba78. 7\uc6d4 4\uc77c, \uc5c4\uccad\ub09c \ud30c\uad34 \uc18d\uc5d0\uc11c \uc0b4\uc544\ub0a8\uc740 \uc9c0\uad6c\uc758 \uc0dd\uc874\uc790\ub4e4\uc740 \uadf8 \ud798\uc744 \ubaa8\uc544 \uac70\ub300\ud55c \uad34\ube44\ud589\ubb3c\uccb4\uc5d0 \ub300\ud56d\ud558\ub824 \ud55c\ub2e4. \uadf8\ub7ec\ub098 \uc678\uacc4\uc778\ub4e4\uc758 \uc9c0\uad6c\ud30c\uad34\ub294 \ub354\uc6b1 \ub9f9\ub82c\ud574\uc9c0\uace0 \uc778\ub958 \uc804\uccb4\uc758 \uc0dd\uc874\uacfc \uc678\uacc4\uc778\uc73c\ub85c\ubd80\ud130 \uc9c0\uad6c\uc758 \ub3c5\ub9bd\uc740 \uc810\uc810 \uc5b4\ub824\uc6cc\uc9c0\ub294\ub370.", "sentence_answer": "\uac70\ub9ac\uc758 \uc790\ub3d9\ucc28\ub4e4\uc740 \ud734\uc9c0\uc870\uac01 \ucc98\ub7fc \uacf5\uc911\uc744 \ub0a0\uc544\ub2e4\ub2cc\ub2e4.", "paragraph_id": "6590942-0-1", "paragraph_question": "question: \uc790\ub3d9\ucc28\ub4e4\uc740 \uc5b4\ub5a0\ud55c \ud615\ud0dc\ub85c \ub0a0\uc544\ub2e4\ub2c8\ub294\uac00?, context: 7\uc6d4 2\uc77c, \uc774\uc0c1\ud55c \ud604\uc0c1\uc774 \uc9c0\uad6c\ub97c \uac10\uc2fc\ub2e4. \ud558\ub298\uc740 \uc774\uae00\uc774\uae00 \ubd88\ud0c0\uc624\ub974\uace0 \ub545\uc740 \uc9c0\uc9c4\uc774 \ub09c \ub4ef \uaca9\ub82c\ud788 \uc694\ub3d9\uce5c\ub2e4. \uc9c1\uacbd 550km, \ubb34\uac8c\uac00 \ub2ec\uc758 4\ubd84\uc758 1\uc774\ub098 \ub418\ub294 \uac70\ub300\ud55c \uad34 \ube44\ud589\ubb3c\uccb4\uac00 \ud0dc\uc591\uc744 \uac00\ub824 \uc9c0\uad6c\ub294 \uadf8 \ube5b\uc744 \uc783\uc5b4\uac04\ub2e4. \uc228\ub9c9\ud788\ub294 \uacf5\ud3ec\uac00 \uc138\uacc4\uc758 \uc8fc\uc694\ub3c4\uc2dc\ub97c \uc5c4\uc2b5\ud558\uae30 \uc2dc\uc791\ud55c\ub2e4. 7\uc6d4 3\uc77c, \uac70\ub300\ud558\uace0 \uae30\uad34\ud55c \ube44\ud589\ubb3c\uccb4\uc5d0\uc11c \ub0b4\ubfdc\ub294 \uac00\uacf5\ud560 \uc704\ub825\uc758 \ubd88\uae30\ub465\uc740 \ub274\uc695\uc758 \ub9c8\ucc9c\ub8e8\uc640 \uc6cc\uc2f1\ud134\uc758 \ubc31\uc545\uad00, \uc774\uc9d1\ud2b8\uc758 \ud53c\ub77c\ubbf8\ub4dc \ub4f1\uc744 \uc21c\uc2dd\uac04\uc5d0 \uc7bf\ub354\ubbf8\ub85c \ub9cc\ub4e4\uace0, \uac70\ub9ac\uc758 \uc790\ub3d9\ucc28\ub4e4\uc740 \ud734\uc9c0\uc870\uac01\ucc98\ub7fc \uacf5\uc911\uc744 \ub0a0\uc544\ub2e4\ub2cc\ub2e4. \uc0ac\ub78c\ub4e4\uc740 \ub2f9\ud669\ud558\uc5ec \uc228\uc744 \uacf3\uc744 \ucc3e\uc9c0\ub9cc \ub3c4\ub9dd\uce60 \uacf3\uc740 \uc544\ubb34\ub370\ub3c4 \uc5c6\ub2e4. \uc778\uac04\uc758 \ud798\uc73c\ub85c \uac10\ub2f9\ud560 \uc218 \uc5c6\ub294 \uc5c4\uccad\ub09c \ud798\uc774 \uc9c0\uad6c\uc5d0 \ub3c4\ub2ec\ud588\uc74c\uc774 \ubc1d\ud600\uc9c4\ub2e4. \uadf8\ub4e4\uc758 \ubaa9\uc801\uc740 \uc644\uc804\ud55c \uc9c0\uad6c\uc758 \ud30c\uba78. 7\uc6d4 4\uc77c, \uc5c4\uccad\ub09c \ud30c\uad34 \uc18d\uc5d0\uc11c \uc0b4\uc544\ub0a8\uc740 \uc9c0\uad6c\uc758 \uc0dd\uc874\uc790\ub4e4\uc740 \uadf8 \ud798\uc744 \ubaa8\uc544 \uac70\ub300\ud55c \uad34\ube44\ud589\ubb3c\uccb4\uc5d0 \ub300\ud56d\ud558\ub824 \ud55c\ub2e4. \uadf8\ub7ec\ub098 \uc678\uacc4\uc778\ub4e4\uc758 \uc9c0\uad6c\ud30c\uad34\ub294 \ub354\uc6b1 \ub9f9\ub82c\ud574\uc9c0\uace0 \uc778\ub958 \uc804\uccb4\uc758 \uc0dd\uc874\uacfc \uc678\uacc4\uc778\uc73c\ub85c\ubd80\ud130 \uc9c0\uad6c\uc758 \ub3c5\ub9bd\uc740 \uc810\uc810 \uc5b4\ub824\uc6cc\uc9c0\ub294\ub370."} {"question": "\ud504\ub85c\uc774\uc13c\uc744 \uacac\uc81c\ud560 \ud544\uc694\uc131\uc744 \ub290\ub080 \ud504\ub791\uc2a4\uac00 \ub3d9\ub9f9\uc744 \ub9fa\uc740 \ub098\ub77c\ub294?", "paragraph": "1740\ub144\ubd80\ud130 \uc591\uad6d\uc740 \uc624\uc2a4\ud2b8\ub9ac\uc544 \uc655\uc704 \uacc4\uc2b9 \uc804\uc7c1(1740~48\ub144)\uc73c\ub85c \ub300\ub9bd\ud588\ub2e4. \uc804\uc7c1\uc5d0\uc11c \uc624\uc2a4\ud2b8\ub9ac\uc544\uac00 \uc2b9\ub9ac\ud558\uace0, \ud504\ub791\uc2a4\ub294 \uc544\ubb34\uac83\ub3c4 \uc5bb\uc9c0 \ubabb\ud588\ub2e4. \uadf8\ub7ec\ub098, \ud504\ub85c\uc774\uc13c\uc744 \uacac\uc81c\ud560 \ud544\uc694\uc131\uc744 \ub290\ub080 \ud504\ub791\uc2a4\ub294 \uc624\uc2a4\ud2b8\ub9ac\uc544\uc640 \ub3d9\ub9f9\uc744 \ub9fa\uc5c8\ub2e4(1756\ub144). \uc774 \ub3d9\ub9f9\uc73c\ub85c \uc624\uc2a4\ud2b8\ub9ac\uc544 \ucd9c\uc2e0\uc758 \ub9c8\ub9ac \uc559\ud22c\uc544\ub124\ud2b8\uac00 \ud504\ub791\uc2a4\ub85c \uc640\uc11c \ub8e8\uc774 16\uc138\uc640 \ud63c\uc778\ud558\uc600\ub2e4. \ud504\ub791\uc2a4 \ud601\uba85 \uc9c1\ud6c4, \ub808\uc624\ud3f4\ud2b8\uac00 \uc624\uc2a4\ud2b8\ub9ac\uc544 \ub300\uacf5\uc774 \ub418\uc790(1790\ub144), \uc624\uc2a4\ud2b8\ub9ac\uc544\ub294 \ud504\ub791\uc2a4\uc640\uc758 1756\ub144\uc758 \ub3d9\ub9f9\uc744 \ud30c\uae30\ud558\uc600\uace0, \uc808\ub300\uc655\uad8c\uc744 \uc704\ud611\ud558\ub294 \ud504\ub791\uc2a4\uc758 \uacf5\ud654\uc8fc\uc758\uc5d0 \ub9de\uc11c\uae30\ub85c \ud558\uc600\ub2e4. \uc624\uc2a4\ud2b8\ub9ac\uc544\ub294 \ud504\ub791\uc2a4\ub85c \uc9c4\uaca9\ud558\uc5ec \ub871\uc704\uc640 \ubca0\ub974\ub42d\uc5d0\uc11c \uc2b9\ub9ac\ub97c \uac70\ub450\uc5c8\uc73c\ub098, \ud504\ub791\uc2a4\uc758 \ubc18\uaca9(\ubc1c\ubbf8 \uc804\ud22c)\uc73c\ub85c \uae30\uc138\uac00 \uaebe\uc600\ub2e4. \uc624\uc2a4\ud2b8\ub9ac\uc544\uac00 \uc9c0\ubc30\ud558\uace0 \uc788\ub358 \ub0a8\ubd80 \ub124\ub35c\ub780\ub4dc(\ud604\uc7ac\uc758 \ubca8\uae30\uc5d0)\uc5d0\uc11c\ub3c4 \ud504\ub791\uc2a4\uac00 \uc624\uc2a4\ud2b8\ub9ac\uc544\ub97c \ubb3c\ub9ac\ucce4\ub2e4.", "answer": "\uc624\uc2a4\ud2b8\ub9ac\uc544", "sentence": "1740\ub144\ubd80\ud130 \uc591\uad6d\uc740 \uc624\uc2a4\ud2b8\ub9ac\uc544 \uc655\uc704 \uacc4\uc2b9 \uc804\uc7c1(1740~48\ub144)\uc73c\ub85c \ub300\ub9bd\ud588\ub2e4.", "paragraph_sentence": " 1740\ub144\ubd80\ud130 \uc591\uad6d\uc740 \uc624\uc2a4\ud2b8\ub9ac\uc544 \uc655\uc704 \uacc4\uc2b9 \uc804\uc7c1(1740~48\ub144)\uc73c\ub85c \ub300\ub9bd\ud588\ub2e4. \uc804\uc7c1\uc5d0\uc11c \uc624\uc2a4\ud2b8\ub9ac\uc544\uac00 \uc2b9\ub9ac\ud558\uace0, \ud504\ub791\uc2a4\ub294 \uc544\ubb34\uac83\ub3c4 \uc5bb\uc9c0 \ubabb\ud588\ub2e4. \uadf8\ub7ec\ub098, \ud504\ub85c\uc774\uc13c\uc744 \uacac\uc81c\ud560 \ud544\uc694\uc131\uc744 \ub290\ub080 \ud504\ub791\uc2a4\ub294 \uc624\uc2a4\ud2b8\ub9ac\uc544\uc640 \ub3d9\ub9f9\uc744 \ub9fa\uc5c8\ub2e4(1756\ub144). \uc774 \ub3d9\ub9f9\uc73c\ub85c \uc624\uc2a4\ud2b8\ub9ac\uc544 \ucd9c\uc2e0\uc758 \ub9c8\ub9ac \uc559\ud22c\uc544\ub124\ud2b8\uac00 \ud504\ub791\uc2a4\ub85c \uc640\uc11c \ub8e8\uc774 16\uc138\uc640 \ud63c\uc778\ud558\uc600\ub2e4. \ud504\ub791\uc2a4 \ud601\uba85 \uc9c1\ud6c4, \ub808\uc624\ud3f4\ud2b8\uac00 \uc624\uc2a4\ud2b8\ub9ac\uc544 \ub300\uacf5\uc774 \ub418\uc790(1790\ub144), \uc624\uc2a4\ud2b8\ub9ac\uc544\ub294 \ud504\ub791\uc2a4\uc640\uc758 1756\ub144\uc758 \ub3d9\ub9f9\uc744 \ud30c\uae30\ud558\uc600\uace0, \uc808\ub300\uc655\uad8c\uc744 \uc704\ud611\ud558\ub294 \ud504\ub791\uc2a4\uc758 \uacf5\ud654\uc8fc\uc758\uc5d0 \ub9de\uc11c\uae30\ub85c \ud558\uc600\ub2e4. \uc624\uc2a4\ud2b8\ub9ac\uc544\ub294 \ud504\ub791\uc2a4\ub85c \uc9c4\uaca9\ud558\uc5ec \ub871\uc704\uc640 \ubca0\ub974\ub42d\uc5d0\uc11c \uc2b9\ub9ac\ub97c \uac70\ub450\uc5c8\uc73c\ub098, \ud504\ub791\uc2a4\uc758 \ubc18\uaca9(\ubc1c\ubbf8 \uc804\ud22c)\uc73c\ub85c \uae30\uc138\uac00 \uaebe\uc600\ub2e4. \uc624\uc2a4\ud2b8\ub9ac\uc544\uac00 \uc9c0\ubc30\ud558\uace0 \uc788\ub358 \ub0a8\ubd80 \ub124\ub35c\ub780\ub4dc(\ud604\uc7ac\uc758 \ubca8\uae30\uc5d0)\uc5d0\uc11c\ub3c4 \ud504\ub791\uc2a4\uac00 \uc624\uc2a4\ud2b8\ub9ac\uc544\ub97c \ubb3c\ub9ac\ucce4\ub2e4.", "paragraph_answer": "1740\ub144\ubd80\ud130 \uc591\uad6d\uc740 \uc624\uc2a4\ud2b8\ub9ac\uc544 \uc655\uc704 \uacc4\uc2b9 \uc804\uc7c1(1740~48\ub144)\uc73c\ub85c \ub300\ub9bd\ud588\ub2e4. \uc804\uc7c1\uc5d0\uc11c \uc624\uc2a4\ud2b8\ub9ac\uc544\uac00 \uc2b9\ub9ac\ud558\uace0, \ud504\ub791\uc2a4\ub294 \uc544\ubb34\uac83\ub3c4 \uc5bb\uc9c0 \ubabb\ud588\ub2e4. \uadf8\ub7ec\ub098, \ud504\ub85c\uc774\uc13c\uc744 \uacac\uc81c\ud560 \ud544\uc694\uc131\uc744 \ub290\ub080 \ud504\ub791\uc2a4\ub294 \uc624\uc2a4\ud2b8\ub9ac\uc544\uc640 \ub3d9\ub9f9\uc744 \ub9fa\uc5c8\ub2e4(1756\ub144). \uc774 \ub3d9\ub9f9\uc73c\ub85c \uc624\uc2a4\ud2b8\ub9ac\uc544 \ucd9c\uc2e0\uc758 \ub9c8\ub9ac \uc559\ud22c\uc544\ub124\ud2b8\uac00 \ud504\ub791\uc2a4\ub85c \uc640\uc11c \ub8e8\uc774 16\uc138\uc640 \ud63c\uc778\ud558\uc600\ub2e4. \ud504\ub791\uc2a4 \ud601\uba85 \uc9c1\ud6c4, \ub808\uc624\ud3f4\ud2b8\uac00 \uc624\uc2a4\ud2b8\ub9ac\uc544 \ub300\uacf5\uc774 \ub418\uc790(1790\ub144), \uc624\uc2a4\ud2b8\ub9ac\uc544\ub294 \ud504\ub791\uc2a4\uc640\uc758 1756\ub144\uc758 \ub3d9\ub9f9\uc744 \ud30c\uae30\ud558\uc600\uace0, \uc808\ub300\uc655\uad8c\uc744 \uc704\ud611\ud558\ub294 \ud504\ub791\uc2a4\uc758 \uacf5\ud654\uc8fc\uc758\uc5d0 \ub9de\uc11c\uae30\ub85c \ud558\uc600\ub2e4. \uc624\uc2a4\ud2b8\ub9ac\uc544\ub294 \ud504\ub791\uc2a4\ub85c \uc9c4\uaca9\ud558\uc5ec \ub871\uc704\uc640 \ubca0\ub974\ub42d\uc5d0\uc11c \uc2b9\ub9ac\ub97c \uac70\ub450\uc5c8\uc73c\ub098, \ud504\ub791\uc2a4\uc758 \ubc18\uaca9(\ubc1c\ubbf8 \uc804\ud22c)\uc73c\ub85c \uae30\uc138\uac00 \uaebe\uc600\ub2e4. \uc624\uc2a4\ud2b8\ub9ac\uc544\uac00 \uc9c0\ubc30\ud558\uace0 \uc788\ub358 \ub0a8\ubd80 \ub124\ub35c\ub780\ub4dc(\ud604\uc7ac\uc758 \ubca8\uae30\uc5d0)\uc5d0\uc11c\ub3c4 \ud504\ub791\uc2a4\uac00 \uc624\uc2a4\ud2b8\ub9ac\uc544\ub97c \ubb3c\ub9ac\ucce4\ub2e4.", "sentence_answer": "1740\ub144\ubd80\ud130 \uc591\uad6d\uc740 \uc624\uc2a4\ud2b8\ub9ac\uc544 \uc655\uc704 \uacc4\uc2b9 \uc804\uc7c1(1740~48\ub144)\uc73c\ub85c \ub300\ub9bd\ud588\ub2e4.", "paragraph_id": "6568624-5-0", "paragraph_question": "question: \ud504\ub85c\uc774\uc13c\uc744 \uacac\uc81c\ud560 \ud544\uc694\uc131\uc744 \ub290\ub080 \ud504\ub791\uc2a4\uac00 \ub3d9\ub9f9\uc744 \ub9fa\uc740 \ub098\ub77c\ub294?, context: 1740\ub144\ubd80\ud130 \uc591\uad6d\uc740 \uc624\uc2a4\ud2b8\ub9ac\uc544 \uc655\uc704 \uacc4\uc2b9 \uc804\uc7c1(1740~48\ub144)\uc73c\ub85c \ub300\ub9bd\ud588\ub2e4. \uc804\uc7c1\uc5d0\uc11c \uc624\uc2a4\ud2b8\ub9ac\uc544\uac00 \uc2b9\ub9ac\ud558\uace0, \ud504\ub791\uc2a4\ub294 \uc544\ubb34\uac83\ub3c4 \uc5bb\uc9c0 \ubabb\ud588\ub2e4. \uadf8\ub7ec\ub098, \ud504\ub85c\uc774\uc13c\uc744 \uacac\uc81c\ud560 \ud544\uc694\uc131\uc744 \ub290\ub080 \ud504\ub791\uc2a4\ub294 \uc624\uc2a4\ud2b8\ub9ac\uc544\uc640 \ub3d9\ub9f9\uc744 \ub9fa\uc5c8\ub2e4(1756\ub144). \uc774 \ub3d9\ub9f9\uc73c\ub85c \uc624\uc2a4\ud2b8\ub9ac\uc544 \ucd9c\uc2e0\uc758 \ub9c8\ub9ac \uc559\ud22c\uc544\ub124\ud2b8\uac00 \ud504\ub791\uc2a4\ub85c \uc640\uc11c \ub8e8\uc774 16\uc138\uc640 \ud63c\uc778\ud558\uc600\ub2e4. \ud504\ub791\uc2a4 \ud601\uba85 \uc9c1\ud6c4, \ub808\uc624\ud3f4\ud2b8\uac00 \uc624\uc2a4\ud2b8\ub9ac\uc544 \ub300\uacf5\uc774 \ub418\uc790(1790\ub144), \uc624\uc2a4\ud2b8\ub9ac\uc544\ub294 \ud504\ub791\uc2a4\uc640\uc758 1756\ub144\uc758 \ub3d9\ub9f9\uc744 \ud30c\uae30\ud558\uc600\uace0, \uc808\ub300\uc655\uad8c\uc744 \uc704\ud611\ud558\ub294 \ud504\ub791\uc2a4\uc758 \uacf5\ud654\uc8fc\uc758\uc5d0 \ub9de\uc11c\uae30\ub85c \ud558\uc600\ub2e4. \uc624\uc2a4\ud2b8\ub9ac\uc544\ub294 \ud504\ub791\uc2a4\ub85c \uc9c4\uaca9\ud558\uc5ec \ub871\uc704\uc640 \ubca0\ub974\ub42d\uc5d0\uc11c \uc2b9\ub9ac\ub97c \uac70\ub450\uc5c8\uc73c\ub098, \ud504\ub791\uc2a4\uc758 \ubc18\uaca9(\ubc1c\ubbf8 \uc804\ud22c)\uc73c\ub85c \uae30\uc138\uac00 \uaebe\uc600\ub2e4. \uc624\uc2a4\ud2b8\ub9ac\uc544\uac00 \uc9c0\ubc30\ud558\uace0 \uc788\ub358 \ub0a8\ubd80 \ub124\ub35c\ub780\ub4dc(\ud604\uc7ac\uc758 \ubca8\uae30\uc5d0)\uc5d0\uc11c\ub3c4 \ud504\ub791\uc2a4\uac00 \uc624\uc2a4\ud2b8\ub9ac\uc544\ub97c \ubb3c\ub9ac\ucce4\ub2e4."} {"question": "1864\ub144 \uc2dc\ubaa8\ub178\uc138\ud0a4 \uc804\uc7c1\uc73c\ub85c \ube7c\uc557\uacbc\ub358 \uc870\uc288 \ubc88\uc758 \ub300\ud3ec 1\ubb38\uc744 \ud658\uc218\ud55c \ub098\ub77c\ub294?", "paragraph": "\uc77c\ubcf8\uc740 1864\ub144\uc5d0 \uc2dc\ubaa8\ub178\uc138\ud0a4 \uc804\uc7c1\uc73c\ub85c \ube7c\uc557\uacbc\ub358 \uc870\uc288 \ubc88\uc758 \ub300\ud3ec 1\ubb38\uc744 \ud504\ub791\uc2a4\uc5d0\uac8c\uc11c 1984\ub144\uc5d0 \ud658\uc218\ud558\uc600\ub2e4. \uc870\uc288 \ubc88\uc740 1863\ub144 5\uc6d4\ubd80\ud130 \uc77c\ubcf8 \uac04\ubaac \ud574\ud611\uc744 \ud56d\ud574\ud558\ub358 \uc678\uad6d \uc120\ubc15\uc744 \ud5a5\ud558\uc5ec \ud3ec\uaca9\uc744 \uc2dc\uc791\ud558\uc600\uace0 \uc774\uc5d0 \ubbf8\uad6d, \uc601\uad6d, \ud504\ub791\uc2a4, \ub124\ub35c\ub780\ub4dc\uc758 4\uac1c\uad6d \uc5f0\ud569\ud568\ub300\uac00 \uc870\uc288 \ubc88\uc744 \ubcf4\ubcf5\ud558\uc600\ub2e4. \ub2f9\uc2dc \ud504\ub791\uc2a4\ub294 \uc870\uc288 \ubc88\uc758 \ud3ec\ub300\uc5d0 \uc788\ub358 100\uc5ec \ubb38\uc758 \ub300\ud3ec \uc911 \uc77c\ubd80\ub97c \uc804\ub9ac\ud488\uc73c\ub85c \uac00\uc838\uac14\ub2e4. \ud504\ub791\uc2a4\ub294 2\ub144\ub9c8\ub2e4 \uc790\ub3d9 \uc5f0\uc7a5\ub418\ub294 \uc0c1\ud638 \uc784\ub300 \ubc29\uc2dd\uc73c\ub85c \ub300\ud3ec\ub97c \ubc18\ud658\ud558\uc600\uace0 \ub300\uc2e0\uc5d0 \uc77c\ubcf8\uc740 \uc61b \uc601\uc8fc\uc758 \uac11\uc637 \ud55c \ubc8c\uc744 \ubcf4\ub0c8\ub2e4. 1867\ub144 \ud30c\ub9ac \ub9cc\uad6d\ubc15\ub78c\ud68c\uc5d0 \ub3c4\ucfc4 \uc2dc\ub098\uac00\uc640\uc758 \ud63c\uc13c\uc9c0 \ubc94\uc885\uc774 \ucd9c\ud488\ub418\uc5c8\ub2e4\uac00 \ubc15\ub78c\ud68c\uac00 \ub05d\ub098\uace0 \uc2a4\uc704\uc2a4 \uc81c\ub124\ubc14 \uc544\ub9ac\uc544\ub098 \ubbf8\uc220\uad00(Mus\u00e9e Ariana)\uc73c\ub85c \uc62e\uaca8\uc84c\ub2e4. 1921\ub144\uc5d0 \uc77c\ubcf8 \uc815\ubd80\ub294 \uc81c\ub124\ubc14 \uc2dc \ub2f9\uad6d\uc5d0 \uc885\uc758 \ubc18\ud658\uc744 \uc694\uccad\ud558\uc600\uace0, 1929\ub144\uc5d0 \uc81c\ub124\ubc14 \uc2dc\uc758\ud68c\ub294 \uae30\uc99d \ubc18\ud658\uc744 \uc758\uacb0\ud558\uc5ec 1930\ub144\uc5d0 \uc77c\ubcf8\uc5d0 \ubc18\ud658\ub418\uc5c8\ub2e4. \uc2dc\ub098\uac00\uc640\uc758 \uc808\uc740 1990\ub144\uc5d0 \ubcf5\uc81c \uc885\uc744 \ub9cc\ub4e4\uc5b4 \uc2a4\uc704\uc2a4 \uc815\ubd80\uc5d0 \uae30\uc99d\ud558\uc600\ub2e4.", "answer": "\uc77c\ubcf8", "sentence": "\uc77c\ubcf8 \uc740 1864\ub144\uc5d0 \uc2dc\ubaa8\ub178\uc138\ud0a4 \uc804\uc7c1\uc73c\ub85c \ube7c\uc557\uacbc\ub358 \uc870\uc288 \ubc88\uc758 \ub300\ud3ec 1\ubb38\uc744 \ud504\ub791\uc2a4\uc5d0\uac8c\uc11c 1984\ub144\uc5d0 \ud658\uc218\ud558\uc600\ub2e4.", "paragraph_sentence": " \uc77c\ubcf8 \uc740 1864\ub144\uc5d0 \uc2dc\ubaa8\ub178\uc138\ud0a4 \uc804\uc7c1\uc73c\ub85c \ube7c\uc557\uacbc\ub358 \uc870\uc288 \ubc88\uc758 \ub300\ud3ec 1\ubb38\uc744 \ud504\ub791\uc2a4\uc5d0\uac8c\uc11c 1984\ub144\uc5d0 \ud658\uc218\ud558\uc600\ub2e4. \uc870\uc288 \ubc88\uc740 1863\ub144 5\uc6d4\ubd80\ud130 \uc77c\ubcf8 \uac04\ubaac \ud574\ud611\uc744 \ud56d\ud574\ud558\ub358 \uc678\uad6d \uc120\ubc15\uc744 \ud5a5\ud558\uc5ec \ud3ec\uaca9\uc744 \uc2dc\uc791\ud558\uc600\uace0 \uc774\uc5d0 \ubbf8\uad6d, \uc601\uad6d, \ud504\ub791\uc2a4, \ub124\ub35c\ub780\ub4dc\uc758 4\uac1c\uad6d \uc5f0\ud569\ud568\ub300\uac00 \uc870\uc288 \ubc88\uc744 \ubcf4\ubcf5\ud558\uc600\ub2e4. \ub2f9\uc2dc \ud504\ub791\uc2a4\ub294 \uc870\uc288 \ubc88\uc758 \ud3ec\ub300\uc5d0 \uc788\ub358 100\uc5ec \ubb38\uc758 \ub300\ud3ec \uc911 \uc77c\ubd80\ub97c \uc804\ub9ac\ud488\uc73c\ub85c \uac00\uc838\uac14\ub2e4. \ud504\ub791\uc2a4\ub294 2\ub144\ub9c8\ub2e4 \uc790\ub3d9 \uc5f0\uc7a5\ub418\ub294 \uc0c1\ud638 \uc784\ub300 \ubc29\uc2dd\uc73c\ub85c \ub300\ud3ec\ub97c \ubc18\ud658\ud558\uc600\uace0 \ub300\uc2e0\uc5d0 \uc77c\ubcf8\uc740 \uc61b \uc601\uc8fc\uc758 \uac11\uc637 \ud55c \ubc8c\uc744 \ubcf4\ub0c8\ub2e4. 1867\ub144 \ud30c\ub9ac \ub9cc\uad6d\ubc15\ub78c\ud68c\uc5d0 \ub3c4\ucfc4 \uc2dc\ub098\uac00\uc640\uc758 \ud63c\uc13c\uc9c0 \ubc94\uc885\uc774 \ucd9c\ud488\ub418\uc5c8\ub2e4\uac00 \ubc15\ub78c\ud68c\uac00 \ub05d\ub098\uace0 \uc2a4\uc704\uc2a4 \uc81c\ub124\ubc14 \uc544\ub9ac\uc544\ub098 \ubbf8\uc220\uad00(Mus\u00e9e Ariana)\uc73c\ub85c \uc62e\uaca8\uc84c\ub2e4. 1921\ub144\uc5d0 \uc77c\ubcf8 \uc815\ubd80\ub294 \uc81c\ub124\ubc14 \uc2dc \ub2f9\uad6d\uc5d0 \uc885\uc758 \ubc18\ud658\uc744 \uc694\uccad\ud558\uc600\uace0, 1929\ub144\uc5d0 \uc81c\ub124\ubc14 \uc2dc\uc758\ud68c\ub294 \uae30\uc99d \ubc18\ud658\uc744 \uc758\uacb0\ud558\uc5ec 1930\ub144\uc5d0 \uc77c\ubcf8\uc5d0 \ubc18\ud658\ub418\uc5c8\ub2e4. \uc2dc\ub098\uac00\uc640\uc758 \uc808\uc740 1990\ub144\uc5d0 \ubcf5\uc81c \uc885\uc744 \ub9cc\ub4e4\uc5b4 \uc2a4\uc704\uc2a4 \uc815\ubd80\uc5d0 \uae30\uc99d\ud558\uc600\ub2e4.", "paragraph_answer": " \uc77c\ubcf8 \uc740 1864\ub144\uc5d0 \uc2dc\ubaa8\ub178\uc138\ud0a4 \uc804\uc7c1\uc73c\ub85c \ube7c\uc557\uacbc\ub358 \uc870\uc288 \ubc88\uc758 \ub300\ud3ec 1\ubb38\uc744 \ud504\ub791\uc2a4\uc5d0\uac8c\uc11c 1984\ub144\uc5d0 \ud658\uc218\ud558\uc600\ub2e4. \uc870\uc288 \ubc88\uc740 1863\ub144 5\uc6d4\ubd80\ud130 \uc77c\ubcf8 \uac04\ubaac \ud574\ud611\uc744 \ud56d\ud574\ud558\ub358 \uc678\uad6d \uc120\ubc15\uc744 \ud5a5\ud558\uc5ec \ud3ec\uaca9\uc744 \uc2dc\uc791\ud558\uc600\uace0 \uc774\uc5d0 \ubbf8\uad6d, \uc601\uad6d, \ud504\ub791\uc2a4, \ub124\ub35c\ub780\ub4dc\uc758 4\uac1c\uad6d \uc5f0\ud569\ud568\ub300\uac00 \uc870\uc288 \ubc88\uc744 \ubcf4\ubcf5\ud558\uc600\ub2e4. \ub2f9\uc2dc \ud504\ub791\uc2a4\ub294 \uc870\uc288 \ubc88\uc758 \ud3ec\ub300\uc5d0 \uc788\ub358 100\uc5ec \ubb38\uc758 \ub300\ud3ec \uc911 \uc77c\ubd80\ub97c \uc804\ub9ac\ud488\uc73c\ub85c \uac00\uc838\uac14\ub2e4. \ud504\ub791\uc2a4\ub294 2\ub144\ub9c8\ub2e4 \uc790\ub3d9 \uc5f0\uc7a5\ub418\ub294 \uc0c1\ud638 \uc784\ub300 \ubc29\uc2dd\uc73c\ub85c \ub300\ud3ec\ub97c \ubc18\ud658\ud558\uc600\uace0 \ub300\uc2e0\uc5d0 \uc77c\ubcf8\uc740 \uc61b \uc601\uc8fc\uc758 \uac11\uc637 \ud55c \ubc8c\uc744 \ubcf4\ub0c8\ub2e4. 1867\ub144 \ud30c\ub9ac \ub9cc\uad6d\ubc15\ub78c\ud68c\uc5d0 \ub3c4\ucfc4 \uc2dc\ub098\uac00\uc640\uc758 \ud63c\uc13c\uc9c0 \ubc94\uc885\uc774 \ucd9c\ud488\ub418\uc5c8\ub2e4\uac00 \ubc15\ub78c\ud68c\uac00 \ub05d\ub098\uace0 \uc2a4\uc704\uc2a4 \uc81c\ub124\ubc14 \uc544\ub9ac\uc544\ub098 \ubbf8\uc220\uad00(Mus\u00e9e Ariana)\uc73c\ub85c \uc62e\uaca8\uc84c\ub2e4. 1921\ub144\uc5d0 \uc77c\ubcf8 \uc815\ubd80\ub294 \uc81c\ub124\ubc14 \uc2dc \ub2f9\uad6d\uc5d0 \uc885\uc758 \ubc18\ud658\uc744 \uc694\uccad\ud558\uc600\uace0, 1929\ub144\uc5d0 \uc81c\ub124\ubc14 \uc2dc\uc758\ud68c\ub294 \uae30\uc99d \ubc18\ud658\uc744 \uc758\uacb0\ud558\uc5ec 1930\ub144\uc5d0 \uc77c\ubcf8\uc5d0 \ubc18\ud658\ub418\uc5c8\ub2e4. \uc2dc\ub098\uac00\uc640\uc758 \uc808\uc740 1990\ub144\uc5d0 \ubcf5\uc81c \uc885\uc744 \ub9cc\ub4e4\uc5b4 \uc2a4\uc704\uc2a4 \uc815\ubd80\uc5d0 \uae30\uc99d\ud558\uc600\ub2e4.", "sentence_answer": " \uc77c\ubcf8 \uc740 1864\ub144\uc5d0 \uc2dc\ubaa8\ub178\uc138\ud0a4 \uc804\uc7c1\uc73c\ub85c \ube7c\uc557\uacbc\ub358 \uc870\uc288 \ubc88\uc758 \ub300\ud3ec 1\ubb38\uc744 \ud504\ub791\uc2a4\uc5d0\uac8c\uc11c 1984\ub144\uc5d0 \ud658\uc218\ud558\uc600\ub2e4.", "paragraph_id": "6090994-12-0", "paragraph_question": "question: 1864\ub144 \uc2dc\ubaa8\ub178\uc138\ud0a4 \uc804\uc7c1\uc73c\ub85c \ube7c\uc557\uacbc\ub358 \uc870\uc288 \ubc88\uc758 \ub300\ud3ec 1\ubb38\uc744 \ud658\uc218\ud55c \ub098\ub77c\ub294?, context: \uc77c\ubcf8\uc740 1864\ub144\uc5d0 \uc2dc\ubaa8\ub178\uc138\ud0a4 \uc804\uc7c1\uc73c\ub85c \ube7c\uc557\uacbc\ub358 \uc870\uc288 \ubc88\uc758 \ub300\ud3ec 1\ubb38\uc744 \ud504\ub791\uc2a4\uc5d0\uac8c\uc11c 1984\ub144\uc5d0 \ud658\uc218\ud558\uc600\ub2e4. \uc870\uc288 \ubc88\uc740 1863\ub144 5\uc6d4\ubd80\ud130 \uc77c\ubcf8 \uac04\ubaac \ud574\ud611\uc744 \ud56d\ud574\ud558\ub358 \uc678\uad6d \uc120\ubc15\uc744 \ud5a5\ud558\uc5ec \ud3ec\uaca9\uc744 \uc2dc\uc791\ud558\uc600\uace0 \uc774\uc5d0 \ubbf8\uad6d, \uc601\uad6d, \ud504\ub791\uc2a4, \ub124\ub35c\ub780\ub4dc\uc758 4\uac1c\uad6d \uc5f0\ud569\ud568\ub300\uac00 \uc870\uc288 \ubc88\uc744 \ubcf4\ubcf5\ud558\uc600\ub2e4. \ub2f9\uc2dc \ud504\ub791\uc2a4\ub294 \uc870\uc288 \ubc88\uc758 \ud3ec\ub300\uc5d0 \uc788\ub358 100\uc5ec \ubb38\uc758 \ub300\ud3ec \uc911 \uc77c\ubd80\ub97c \uc804\ub9ac\ud488\uc73c\ub85c \uac00\uc838\uac14\ub2e4. \ud504\ub791\uc2a4\ub294 2\ub144\ub9c8\ub2e4 \uc790\ub3d9 \uc5f0\uc7a5\ub418\ub294 \uc0c1\ud638 \uc784\ub300 \ubc29\uc2dd\uc73c\ub85c \ub300\ud3ec\ub97c \ubc18\ud658\ud558\uc600\uace0 \ub300\uc2e0\uc5d0 \uc77c\ubcf8\uc740 \uc61b \uc601\uc8fc\uc758 \uac11\uc637 \ud55c \ubc8c\uc744 \ubcf4\ub0c8\ub2e4. 1867\ub144 \ud30c\ub9ac \ub9cc\uad6d\ubc15\ub78c\ud68c\uc5d0 \ub3c4\ucfc4 \uc2dc\ub098\uac00\uc640\uc758 \ud63c\uc13c\uc9c0 \ubc94\uc885\uc774 \ucd9c\ud488\ub418\uc5c8\ub2e4\uac00 \ubc15\ub78c\ud68c\uac00 \ub05d\ub098\uace0 \uc2a4\uc704\uc2a4 \uc81c\ub124\ubc14 \uc544\ub9ac\uc544\ub098 \ubbf8\uc220\uad00(Mus\u00e9e Ariana)\uc73c\ub85c \uc62e\uaca8\uc84c\ub2e4. 1921\ub144\uc5d0 \uc77c\ubcf8 \uc815\ubd80\ub294 \uc81c\ub124\ubc14 \uc2dc \ub2f9\uad6d\uc5d0 \uc885\uc758 \ubc18\ud658\uc744 \uc694\uccad\ud558\uc600\uace0, 1929\ub144\uc5d0 \uc81c\ub124\ubc14 \uc2dc\uc758\ud68c\ub294 \uae30\uc99d \ubc18\ud658\uc744 \uc758\uacb0\ud558\uc5ec 1930\ub144\uc5d0 \uc77c\ubcf8\uc5d0 \ubc18\ud658\ub418\uc5c8\ub2e4. \uc2dc\ub098\uac00\uc640\uc758 \uc808\uc740 1990\ub144\uc5d0 \ubcf5\uc81c \uc885\uc744 \ub9cc\ub4e4\uc5b4 \uc2a4\uc704\uc2a4 \uc815\ubd80\uc5d0 \uae30\uc99d\ud558\uc600\ub2e4."} {"question": "\ubc95\ud559\uc804\ubb38\ub300\ud559\uc6d0 \uc785\ud559\uc804\ud615\uc744 \ub450 \uac1c\uc758 \uad70\uc73c\ub85c \ub098\ub204\uc5b4 \uba70\uce60 \uc529 \uc2e4\uc2dc\ub420 \uc608\uc815\uc778\uac00?", "paragraph": "\uc544\uc9c1 \ud655\uc815\ub41c \uac00\uc774\ub4dc\ub77c\uc778\uc740 \uc5c6\uc73c\ub098 \ubc95\ud559\uc801\uc131\uc2dc\ud5d8(Legal Education Eligibility Test: LEET)\uacfc \ud559\ubd80 \ud3c9\uc810\ud3c9\uade0 \uadf8\ub9ac\uace0 \uc678\uad6d\uc5b4 \ub2a5\ub825\ub4f1 \uc138\uac00\uc9c0\ub97c \uc8fc\uc694\ud558\uac8c \ubcfc \uac83\uc73c\ub85c \ubcf4\uc778\ub2e4. \uacfc\uc678\ud65c\ub3d9\uacfc \ubd09\uc0ac\ud65c\ub3d9 \ub4f1\ub3c4 \ubc18\uc601\uc774 \ub420 \uac83\uc73c\ub85c \ubcf4\uc778\ub2e4. 3\uc6d4 20\uc77c\uacbd\uc5d0 \ub300\ud559\uc5d0\uc11c \ubc1c\ud45c\ud560 \uac83\uc73c\ub85c \ubcf4\uc774\uba70 \ubc95\ud559\uc801\uc131\uc2dc\ud5d8\uc740 5\uc6d4\uc5d0 \uacf5\uace0\ud560 \uc608\uc815\uc774\ub2e4. \ud604\uc7ac \uc900\ube44 \ud559\uc0dd\uc758 \uc555\ub3c4\uc801\uc778 \ube44\uc728\uc740 \uc11c\uc6b8\ub300 \uc785\ud559\uc744 \ubaa9\ud45c\ub85c \ud558\uace0 \uc788\ub2e4.\ud559\uad50\ubcc4\ub85c \ub17c\uc220\uc744 \ub530\ub85c \ubcf4\uc9c0 \uc54a\uae30\ub85c \ud558\uc600\ub2e4. \ub450\uac1c\uc758 \uad70\uc73c\ub85c \ub098\ub204\uc5b4 \uccab\uc9f8\uad70\uc740 11\uc6d4 10~15\uc77c, \ub458\uc9f8\uad70\uc740 11\uc6d4 17~22\uc77c 6\uc77c\uc529 \uc804\ud615\uc774 \uc2e4\uc2dc\ub420 \uc608\uc815\uc774\ub2e4. \uac01 \ub300\ud559\uc774 \ub450 \ubc88\uc5d0 \ub098\ub220 \ud559\uc0dd\uc744 \ubf51\uc744\uc9c0, \ud55c \ubc88\uc5d0 \ubaa8\ub450 \ubf51\uc744\uc9c0\ub294 \uc790\uccb4\uc801\uc73c\ub85c \uc815\ud558\uae30\ub85c \ud588\ub2e4.", "answer": "6\uc77c", "sentence": "\uccab\uc9f8\uad70\uc740 11\uc6d4 10~15\uc77c, \ub458\uc9f8\uad70\uc740 11\uc6d4 17~22\uc77c 6\uc77c \uc529 \uc804\ud615\uc774 \uc2e4\uc2dc\ub420 \uc608\uc815\uc774\ub2e4.", "paragraph_sentence": "\uc544\uc9c1 \ud655\uc815\ub41c \uac00\uc774\ub4dc\ub77c\uc778\uc740 \uc5c6\uc73c\ub098 \ubc95\ud559\uc801\uc131\uc2dc\ud5d8(Legal Education Eligibility Test: LEET)\uacfc \ud559\ubd80 \ud3c9\uc810\ud3c9\uade0 \uadf8\ub9ac\uace0 \uc678\uad6d\uc5b4 \ub2a5\ub825\ub4f1 \uc138\uac00\uc9c0\ub97c \uc8fc\uc694\ud558\uac8c \ubcfc \uac83\uc73c\ub85c \ubcf4\uc778\ub2e4. \uacfc\uc678\ud65c\ub3d9\uacfc \ubd09\uc0ac\ud65c\ub3d9 \ub4f1\ub3c4 \ubc18\uc601\uc774 \ub420 \uac83\uc73c\ub85c \ubcf4\uc778\ub2e4. 3\uc6d4 20\uc77c\uacbd\uc5d0 \ub300\ud559\uc5d0\uc11c \ubc1c\ud45c\ud560 \uac83\uc73c\ub85c \ubcf4\uc774\uba70 \ubc95\ud559\uc801\uc131\uc2dc\ud5d8\uc740 5\uc6d4\uc5d0 \uacf5\uace0\ud560 \uc608\uc815\uc774\ub2e4. \ud604\uc7ac \uc900\ube44 \ud559\uc0dd\uc758 \uc555\ub3c4\uc801\uc778 \ube44\uc728\uc740 \uc11c\uc6b8\ub300 \uc785\ud559\uc744 \ubaa9\ud45c\ub85c \ud558\uace0 \uc788\ub2e4.\ud559\uad50\ubcc4\ub85c \ub17c\uc220\uc744 \ub530\ub85c \ubcf4\uc9c0 \uc54a\uae30\ub85c \ud558\uc600\ub2e4. \ub450\uac1c\uc758 \uad70\uc73c\ub85c \ub098\ub204\uc5b4 \uccab\uc9f8\uad70\uc740 11\uc6d4 10~15\uc77c, \ub458\uc9f8\uad70\uc740 11\uc6d4 17~22\uc77c 6\uc77c \uc529 \uc804\ud615\uc774 \uc2e4\uc2dc\ub420 \uc608\uc815\uc774\ub2e4. \uac01 \ub300\ud559\uc774 \ub450 \ubc88\uc5d0 \ub098\ub220 \ud559\uc0dd\uc744 \ubf51\uc744\uc9c0, \ud55c \ubc88\uc5d0 \ubaa8\ub450 \ubf51\uc744\uc9c0\ub294 \uc790\uccb4\uc801\uc73c\ub85c \uc815\ud558\uae30\ub85c \ud588\ub2e4.", "paragraph_answer": "\uc544\uc9c1 \ud655\uc815\ub41c \uac00\uc774\ub4dc\ub77c\uc778\uc740 \uc5c6\uc73c\ub098 \ubc95\ud559\uc801\uc131\uc2dc\ud5d8(Legal Education Eligibility Test: LEET)\uacfc \ud559\ubd80 \ud3c9\uc810\ud3c9\uade0 \uadf8\ub9ac\uace0 \uc678\uad6d\uc5b4 \ub2a5\ub825\ub4f1 \uc138\uac00\uc9c0\ub97c \uc8fc\uc694\ud558\uac8c \ubcfc \uac83\uc73c\ub85c \ubcf4\uc778\ub2e4. \uacfc\uc678\ud65c\ub3d9\uacfc \ubd09\uc0ac\ud65c\ub3d9 \ub4f1\ub3c4 \ubc18\uc601\uc774 \ub420 \uac83\uc73c\ub85c \ubcf4\uc778\ub2e4. 3\uc6d4 20\uc77c\uacbd\uc5d0 \ub300\ud559\uc5d0\uc11c \ubc1c\ud45c\ud560 \uac83\uc73c\ub85c \ubcf4\uc774\uba70 \ubc95\ud559\uc801\uc131\uc2dc\ud5d8\uc740 5\uc6d4\uc5d0 \uacf5\uace0\ud560 \uc608\uc815\uc774\ub2e4. \ud604\uc7ac \uc900\ube44 \ud559\uc0dd\uc758 \uc555\ub3c4\uc801\uc778 \ube44\uc728\uc740 \uc11c\uc6b8\ub300 \uc785\ud559\uc744 \ubaa9\ud45c\ub85c \ud558\uace0 \uc788\ub2e4.\ud559\uad50\ubcc4\ub85c \ub17c\uc220\uc744 \ub530\ub85c \ubcf4\uc9c0 \uc54a\uae30\ub85c \ud558\uc600\ub2e4. \ub450\uac1c\uc758 \uad70\uc73c\ub85c \ub098\ub204\uc5b4 \uccab\uc9f8\uad70\uc740 11\uc6d4 10~15\uc77c, \ub458\uc9f8\uad70\uc740 11\uc6d4 17~22\uc77c 6\uc77c \uc529 \uc804\ud615\uc774 \uc2e4\uc2dc\ub420 \uc608\uc815\uc774\ub2e4. \uac01 \ub300\ud559\uc774 \ub450 \ubc88\uc5d0 \ub098\ub220 \ud559\uc0dd\uc744 \ubf51\uc744\uc9c0, \ud55c \ubc88\uc5d0 \ubaa8\ub450 \ubf51\uc744\uc9c0\ub294 \uc790\uccb4\uc801\uc73c\ub85c \uc815\ud558\uae30\ub85c \ud588\ub2e4.", "sentence_answer": "\uccab\uc9f8\uad70\uc740 11\uc6d4 10~15\uc77c, \ub458\uc9f8\uad70\uc740 11\uc6d4 17~22\uc77c 6\uc77c \uc529 \uc804\ud615\uc774 \uc2e4\uc2dc\ub420 \uc608\uc815\uc774\ub2e4.", "paragraph_id": "6512325-4-1", "paragraph_question": "question: \ubc95\ud559\uc804\ubb38\ub300\ud559\uc6d0 \uc785\ud559\uc804\ud615\uc744 \ub450 \uac1c\uc758 \uad70\uc73c\ub85c \ub098\ub204\uc5b4 \uba70\uce60 \uc529 \uc2e4\uc2dc\ub420 \uc608\uc815\uc778\uac00?, context: \uc544\uc9c1 \ud655\uc815\ub41c \uac00\uc774\ub4dc\ub77c\uc778\uc740 \uc5c6\uc73c\ub098 \ubc95\ud559\uc801\uc131\uc2dc\ud5d8(Legal Education Eligibility Test: LEET)\uacfc \ud559\ubd80 \ud3c9\uc810\ud3c9\uade0 \uadf8\ub9ac\uace0 \uc678\uad6d\uc5b4 \ub2a5\ub825\ub4f1 \uc138\uac00\uc9c0\ub97c \uc8fc\uc694\ud558\uac8c \ubcfc \uac83\uc73c\ub85c \ubcf4\uc778\ub2e4. \uacfc\uc678\ud65c\ub3d9\uacfc \ubd09\uc0ac\ud65c\ub3d9 \ub4f1\ub3c4 \ubc18\uc601\uc774 \ub420 \uac83\uc73c\ub85c \ubcf4\uc778\ub2e4. 3\uc6d4 20\uc77c\uacbd\uc5d0 \ub300\ud559\uc5d0\uc11c \ubc1c\ud45c\ud560 \uac83\uc73c\ub85c \ubcf4\uc774\uba70 \ubc95\ud559\uc801\uc131\uc2dc\ud5d8\uc740 5\uc6d4\uc5d0 \uacf5\uace0\ud560 \uc608\uc815\uc774\ub2e4. \ud604\uc7ac \uc900\ube44 \ud559\uc0dd\uc758 \uc555\ub3c4\uc801\uc778 \ube44\uc728\uc740 \uc11c\uc6b8\ub300 \uc785\ud559\uc744 \ubaa9\ud45c\ub85c \ud558\uace0 \uc788\ub2e4.\ud559\uad50\ubcc4\ub85c \ub17c\uc220\uc744 \ub530\ub85c \ubcf4\uc9c0 \uc54a\uae30\ub85c \ud558\uc600\ub2e4. \ub450\uac1c\uc758 \uad70\uc73c\ub85c \ub098\ub204\uc5b4 \uccab\uc9f8\uad70\uc740 11\uc6d4 10~15\uc77c, \ub458\uc9f8\uad70\uc740 11\uc6d4 17~22\uc77c 6\uc77c\uc529 \uc804\ud615\uc774 \uc2e4\uc2dc\ub420 \uc608\uc815\uc774\ub2e4. \uac01 \ub300\ud559\uc774 \ub450 \ubc88\uc5d0 \ub098\ub220 \ud559\uc0dd\uc744 \ubf51\uc744\uc9c0, \ud55c \ubc88\uc5d0 \ubaa8\ub450 \ubf51\uc744\uc9c0\ub294 \uc790\uccb4\uc801\uc73c\ub85c \uc815\ud558\uae30\ub85c \ud588\ub2e4."} {"question": "\uacb0\uacfc\uc801\uc73c\ub85c \uc774\uc2a4\uce78\ub378\uacfc \uac19\uc740 \ud589\uc131\uc758 \uc774\ub984\uc740?", "paragraph": "\uadf8 \uc88c\ud45c\uc5d0 \ub3c4\ub2ec\ud558\uace0\uc11c, \ud55c \uc678\uacc4\uc778\uc774 \uc720\ud0a4\uc758 \ubab8 \uc548\uc5d0 \ub4e4\uc5b4\uac00\uace0\ub294 \uac00\ubbf8\ub77c\uc2a4\uc640 \uc774\uc2a4\uce78\ub378\uc740 \uac19\uc740 \uc758\uc2dd\uccb4\uc758 \ub450 \uc591\uba74\uc774\ub77c\uace0 \uc15c\uba85\ud55c\ub2e4. \uadf8 \uc678\uacc4\uc778\uc740 \uadf8\ub4e4\uc758 \ud589\uc131\uc740 \uba38\uc9c0 \uc54a\uc544 \uba78\ub9dd\ud558\ub824 \ud558\uba70 \uac00\ubbf8\ub77c\uc2a4\ub294 \uc9c0\uad6c\uc778\ub4e4\uc744 \ubaa8\ub450 \ub9d0\uc0b4\ud55c \ub4a4\uc5d0 \uc9c0\uad6c\ub97c \uc0b4\uae30 \uc88b\uc740 \ud658\uacbd\uc73c\ub85c \uac1c\ub7c9\ud558\uace0 \uc788\ub294 \uac83\uc774\ub77c \uc124\uba85\ud55c\ub2e4. \ud558\uc9c0\ub9cc, \uc774\uc2a4\uce78\ub378\uc740 \uc790\uc2e0\ub4e4\uc758 \ud589\uc131\uacfc \uc6b4\uba85\uc744 \uac19\uc774 \ud558\ub824\uace0 \ud588\uae30 \ub54c\ubb38\uc5d0 \uc774\uc2a4\uce78\ub378\uc744 \uac00\ub454 \uac83\uc774\ub77c\uace0 \ub9d0\ud55c\ub2e4. \uc774\uc2a4\uce78\ub378\uc740 \uc720\ud0a4\uc5d0 \uc9c0\uad6c\uc5d0 \ubc29\uc0ac\ub2a5\uc744 \uc81c\uac70\ud560 \uc218 \uc788\ub294 \ub2a5\ub825\uc744 \uc8fc\uc785\uc2dc\ud0a4\uace0, \uadf8\ub4e4\uc740 \ub2e4\uc2dc \uc9c0\uad6c\ub97c \ud5a5\ud574 \ub3cc\uc544\uac00\ub824\uace0 \ud55c\ub2e4. \ud558\uc9c0\ub9cc \ud589\uc131\uc744 \ud0c8\ucd9c\ud558\ub294 \ub3d9\uc548 \uc0ac\uc774\ud1a0\uc640 \uc0ac\ub098\ub2e4\ub294 \uac00\ubbf8\ub77c\uc2a4\uc758 \ud798\uc758 \uadfc\uc6d0\uc744 \uc5c6\uc560\uae30 \uc704\ud574 \uace0\uc804\ubd84\ud22c\ud558\ub2e4 \ubaa9\uc228\uc744 \uc783\ub294\ub2e4. \uadf8\ub9ac\uace0 \ub4dc\ub514\uc5b4 \uc57c\ub9c8\ud1a0\ub294 \uc9c0\uad6c\ub85c \ub3cc\uc544\uac00\uac8c \ub418\uace0 \ub3cc\uc544\uac00\ub294 \ub3d9\uc548 \uc624\ud0a4\ub2e4 \uc120\uc7a5\uc740 \uc8fd\uc74c\uc744 \ub9de\uc774\ud55c\ub2e4. \uc120\uc6d0\ub4e4\uc740 \ub2e4\uc2dc \uc9c0\uad6c\ub85c \ub3cc\uc544\uc624\ub294 \uac83\uc744 \ub300\ub2e8\ud788 \uae30\ubed0\ud558\uc9c0\ub9cc \ub9c8\uc9c0\ub9c9\uc73c\ub85c \uc0b4\uc544\ub0a8\uc740 \uac00\ubbf8\ub77c\uc2a4 \ud568\ub300\uac00 \ub9e4\ubcf5\ud588\ub2e4\uac00 \uc2b5\uaca9\ud558\uace0 \uc57c\ub9c8\ud1a0\uc758 \uac70\uc758 \ubaa8\ub4e0 \ubd80\ubd84\uc774 \ud30c\uad34\ub420 \uc9c0\uacbd\uc5d0 \uc774\ub978\ub2e4. \uadf8\ub9ac\uace0 \ub370\uc2ac\ub7ec\uac00 \ub098\ud0c0\ub098\uc11c\ub294 \uc9c0\uad6c\ub97c \uacf5\uaca9\ud560 \uc0dd\uac01\uc740 \ub354 \uc774\uc0c1 \uc5c6\uc9c0\ub9cc, \ub300\ubd80\ubd84\uc758 \uac00\ubbf8\ub77c\uc2a4\uac00 \uc8fd\uc74c\uc744 \ub9c8\uc9c0\ud588\uace0, \ub370\uc2ac\ub7ec \uc790\uc2e0\ub4e4\uc740 \ubcf5\uc218\ub97c \uc704\ud574 \uc9c0\uad6c\ub97c \uc644\uc804\ud788 \ud30c\uad34\ud558\uaca0\ub2e4\uace0 \ub9d0\ud55c\ub2e4. \ucf54\ub2e4\uc774\ub294 \uc790\uc2e0\uc744 \uc81c\uc678\ud55c \ubaa8\ub4e0 \uc0ac\ub78c\uc744 \uc57c\ub9c8\ud1a0 \ubc16\uc73c\ub85c \ub0b4\ubcf4\ub0b4\uace0\ub294 \uac00\ubbf8\ub77c\uc2a4\uc5d0\uac8c \ud3ec\uad6c\uac00 \ub9c9\ud78c \ud30c\ub3d9\ud3ec\ub97c \ubc1c\uc0ac\ud574\uc11c, \uc790\uc2e0\uacfc \uc57c\ub9c8\ud1a0, \uadf8\ub9ac\uace0 \uac00\ubbf8\ub77c\uc2a4\uc640 \ud568\uaed8 \uc8fd\uc74c\uc744 \ub9de\uc774\ud55c\ub2e4.", "answer": "\uac00\ubbf8\ub77c\uc2a4", "sentence": "\uadf8 \uc88c\ud45c\uc5d0 \ub3c4\ub2ec\ud558\uace0\uc11c, \ud55c \uc678\uacc4\uc778\uc774 \uc720\ud0a4\uc758 \ubab8 \uc548\uc5d0 \ub4e4\uc5b4\uac00\uace0\ub294 \uac00\ubbf8\ub77c\uc2a4 \uc640 \uc774\uc2a4\uce78\ub378\uc740 \uac19\uc740 \uc758\uc2dd\uccb4\uc758 \ub450 \uc591\uba74\uc774\ub77c\uace0 \uc15c\uba85\ud55c\ub2e4.", "paragraph_sentence": " \uadf8 \uc88c\ud45c\uc5d0 \ub3c4\ub2ec\ud558\uace0\uc11c, \ud55c \uc678\uacc4\uc778\uc774 \uc720\ud0a4\uc758 \ubab8 \uc548\uc5d0 \ub4e4\uc5b4\uac00\uace0\ub294 \uac00\ubbf8\ub77c\uc2a4 \uc640 \uc774\uc2a4\uce78\ub378\uc740 \uac19\uc740 \uc758\uc2dd\uccb4\uc758 \ub450 \uc591\uba74\uc774\ub77c\uace0 \uc15c\uba85\ud55c\ub2e4. \uadf8 \uc678\uacc4\uc778\uc740 \uadf8\ub4e4\uc758 \ud589\uc131\uc740 \uba38\uc9c0 \uc54a\uc544 \uba78\ub9dd\ud558\ub824 \ud558\uba70 \uac00\ubbf8\ub77c\uc2a4\ub294 \uc9c0\uad6c\uc778\ub4e4\uc744 \ubaa8\ub450 \ub9d0\uc0b4\ud55c \ub4a4\uc5d0 \uc9c0\uad6c\ub97c \uc0b4\uae30 \uc88b\uc740 \ud658\uacbd\uc73c\ub85c \uac1c\ub7c9\ud558\uace0 \uc788\ub294 \uac83\uc774\ub77c \uc124\uba85\ud55c\ub2e4. \ud558\uc9c0\ub9cc, \uc774\uc2a4\uce78\ub378\uc740 \uc790\uc2e0\ub4e4\uc758 \ud589\uc131\uacfc \uc6b4\uba85\uc744 \uac19\uc774 \ud558\ub824\uace0 \ud588\uae30 \ub54c\ubb38\uc5d0 \uc774\uc2a4\uce78\ub378\uc744 \uac00\ub454 \uac83\uc774\ub77c\uace0 \ub9d0\ud55c\ub2e4. \uc774\uc2a4\uce78\ub378\uc740 \uc720\ud0a4\uc5d0 \uc9c0\uad6c\uc5d0 \ubc29\uc0ac\ub2a5\uc744 \uc81c\uac70\ud560 \uc218 \uc788\ub294 \ub2a5\ub825\uc744 \uc8fc\uc785\uc2dc\ud0a4\uace0, \uadf8\ub4e4\uc740 \ub2e4\uc2dc \uc9c0\uad6c\ub97c \ud5a5\ud574 \ub3cc\uc544\uac00\ub824\uace0 \ud55c\ub2e4. \ud558\uc9c0\ub9cc \ud589\uc131\uc744 \ud0c8\ucd9c\ud558\ub294 \ub3d9\uc548 \uc0ac\uc774\ud1a0\uc640 \uc0ac\ub098\ub2e4\ub294 \uac00\ubbf8\ub77c\uc2a4\uc758 \ud798\uc758 \uadfc\uc6d0\uc744 \uc5c6\uc560\uae30 \uc704\ud574 \uace0\uc804\ubd84\ud22c\ud558\ub2e4 \ubaa9\uc228\uc744 \uc783\ub294\ub2e4. \uadf8\ub9ac\uace0 \ub4dc\ub514\uc5b4 \uc57c\ub9c8\ud1a0\ub294 \uc9c0\uad6c\ub85c \ub3cc\uc544\uac00\uac8c \ub418\uace0 \ub3cc\uc544\uac00\ub294 \ub3d9\uc548 \uc624\ud0a4\ub2e4 \uc120\uc7a5\uc740 \uc8fd\uc74c\uc744 \ub9de\uc774\ud55c\ub2e4. \uc120\uc6d0\ub4e4\uc740 \ub2e4\uc2dc \uc9c0\uad6c\ub85c \ub3cc\uc544\uc624\ub294 \uac83\uc744 \ub300\ub2e8\ud788 \uae30\ubed0\ud558\uc9c0\ub9cc \ub9c8\uc9c0\ub9c9\uc73c\ub85c \uc0b4\uc544\ub0a8\uc740 \uac00\ubbf8\ub77c\uc2a4 \ud568\ub300\uac00 \ub9e4\ubcf5\ud588\ub2e4\uac00 \uc2b5\uaca9\ud558\uace0 \uc57c\ub9c8\ud1a0\uc758 \uac70\uc758 \ubaa8\ub4e0 \ubd80\ubd84\uc774 \ud30c\uad34\ub420 \uc9c0\uacbd\uc5d0 \uc774\ub978\ub2e4. \uadf8\ub9ac\uace0 \ub370\uc2ac\ub7ec\uac00 \ub098\ud0c0\ub098\uc11c\ub294 \uc9c0\uad6c\ub97c \uacf5\uaca9\ud560 \uc0dd\uac01\uc740 \ub354 \uc774\uc0c1 \uc5c6\uc9c0\ub9cc, \ub300\ubd80\ubd84\uc758 \uac00\ubbf8\ub77c\uc2a4\uac00 \uc8fd\uc74c\uc744 \ub9c8\uc9c0\ud588\uace0, \ub370\uc2ac\ub7ec \uc790\uc2e0\ub4e4\uc740 \ubcf5\uc218\ub97c \uc704\ud574 \uc9c0\uad6c\ub97c \uc644\uc804\ud788 \ud30c\uad34\ud558\uaca0\ub2e4\uace0 \ub9d0\ud55c\ub2e4. \ucf54\ub2e4\uc774\ub294 \uc790\uc2e0\uc744 \uc81c\uc678\ud55c \ubaa8\ub4e0 \uc0ac\ub78c\uc744 \uc57c\ub9c8\ud1a0 \ubc16\uc73c\ub85c \ub0b4\ubcf4\ub0b4\uace0\ub294 \uac00\ubbf8\ub77c\uc2a4\uc5d0\uac8c \ud3ec\uad6c\uac00 \ub9c9\ud78c \ud30c\ub3d9\ud3ec\ub97c \ubc1c\uc0ac\ud574\uc11c, \uc790\uc2e0\uacfc \uc57c\ub9c8\ud1a0, \uadf8\ub9ac\uace0 \uac00\ubbf8\ub77c\uc2a4\uc640 \ud568\uaed8 \uc8fd\uc74c\uc744 \ub9de\uc774\ud55c\ub2e4.", "paragraph_answer": "\uadf8 \uc88c\ud45c\uc5d0 \ub3c4\ub2ec\ud558\uace0\uc11c, \ud55c \uc678\uacc4\uc778\uc774 \uc720\ud0a4\uc758 \ubab8 \uc548\uc5d0 \ub4e4\uc5b4\uac00\uace0\ub294 \uac00\ubbf8\ub77c\uc2a4 \uc640 \uc774\uc2a4\uce78\ub378\uc740 \uac19\uc740 \uc758\uc2dd\uccb4\uc758 \ub450 \uc591\uba74\uc774\ub77c\uace0 \uc15c\uba85\ud55c\ub2e4. \uadf8 \uc678\uacc4\uc778\uc740 \uadf8\ub4e4\uc758 \ud589\uc131\uc740 \uba38\uc9c0 \uc54a\uc544 \uba78\ub9dd\ud558\ub824 \ud558\uba70 \uac00\ubbf8\ub77c\uc2a4\ub294 \uc9c0\uad6c\uc778\ub4e4\uc744 \ubaa8\ub450 \ub9d0\uc0b4\ud55c \ub4a4\uc5d0 \uc9c0\uad6c\ub97c \uc0b4\uae30 \uc88b\uc740 \ud658\uacbd\uc73c\ub85c \uac1c\ub7c9\ud558\uace0 \uc788\ub294 \uac83\uc774\ub77c \uc124\uba85\ud55c\ub2e4. \ud558\uc9c0\ub9cc, \uc774\uc2a4\uce78\ub378\uc740 \uc790\uc2e0\ub4e4\uc758 \ud589\uc131\uacfc \uc6b4\uba85\uc744 \uac19\uc774 \ud558\ub824\uace0 \ud588\uae30 \ub54c\ubb38\uc5d0 \uc774\uc2a4\uce78\ub378\uc744 \uac00\ub454 \uac83\uc774\ub77c\uace0 \ub9d0\ud55c\ub2e4. \uc774\uc2a4\uce78\ub378\uc740 \uc720\ud0a4\uc5d0 \uc9c0\uad6c\uc5d0 \ubc29\uc0ac\ub2a5\uc744 \uc81c\uac70\ud560 \uc218 \uc788\ub294 \ub2a5\ub825\uc744 \uc8fc\uc785\uc2dc\ud0a4\uace0, \uadf8\ub4e4\uc740 \ub2e4\uc2dc \uc9c0\uad6c\ub97c \ud5a5\ud574 \ub3cc\uc544\uac00\ub824\uace0 \ud55c\ub2e4. \ud558\uc9c0\ub9cc \ud589\uc131\uc744 \ud0c8\ucd9c\ud558\ub294 \ub3d9\uc548 \uc0ac\uc774\ud1a0\uc640 \uc0ac\ub098\ub2e4\ub294 \uac00\ubbf8\ub77c\uc2a4\uc758 \ud798\uc758 \uadfc\uc6d0\uc744 \uc5c6\uc560\uae30 \uc704\ud574 \uace0\uc804\ubd84\ud22c\ud558\ub2e4 \ubaa9\uc228\uc744 \uc783\ub294\ub2e4. \uadf8\ub9ac\uace0 \ub4dc\ub514\uc5b4 \uc57c\ub9c8\ud1a0\ub294 \uc9c0\uad6c\ub85c \ub3cc\uc544\uac00\uac8c \ub418\uace0 \ub3cc\uc544\uac00\ub294 \ub3d9\uc548 \uc624\ud0a4\ub2e4 \uc120\uc7a5\uc740 \uc8fd\uc74c\uc744 \ub9de\uc774\ud55c\ub2e4. \uc120\uc6d0\ub4e4\uc740 \ub2e4\uc2dc \uc9c0\uad6c\ub85c \ub3cc\uc544\uc624\ub294 \uac83\uc744 \ub300\ub2e8\ud788 \uae30\ubed0\ud558\uc9c0\ub9cc \ub9c8\uc9c0\ub9c9\uc73c\ub85c \uc0b4\uc544\ub0a8\uc740 \uac00\ubbf8\ub77c\uc2a4 \ud568\ub300\uac00 \ub9e4\ubcf5\ud588\ub2e4\uac00 \uc2b5\uaca9\ud558\uace0 \uc57c\ub9c8\ud1a0\uc758 \uac70\uc758 \ubaa8\ub4e0 \ubd80\ubd84\uc774 \ud30c\uad34\ub420 \uc9c0\uacbd\uc5d0 \uc774\ub978\ub2e4. \uadf8\ub9ac\uace0 \ub370\uc2ac\ub7ec\uac00 \ub098\ud0c0\ub098\uc11c\ub294 \uc9c0\uad6c\ub97c \uacf5\uaca9\ud560 \uc0dd\uac01\uc740 \ub354 \uc774\uc0c1 \uc5c6\uc9c0\ub9cc, \ub300\ubd80\ubd84\uc758 \uac00\ubbf8\ub77c\uc2a4\uac00 \uc8fd\uc74c\uc744 \ub9c8\uc9c0\ud588\uace0, \ub370\uc2ac\ub7ec \uc790\uc2e0\ub4e4\uc740 \ubcf5\uc218\ub97c \uc704\ud574 \uc9c0\uad6c\ub97c \uc644\uc804\ud788 \ud30c\uad34\ud558\uaca0\ub2e4\uace0 \ub9d0\ud55c\ub2e4. \ucf54\ub2e4\uc774\ub294 \uc790\uc2e0\uc744 \uc81c\uc678\ud55c \ubaa8\ub4e0 \uc0ac\ub78c\uc744 \uc57c\ub9c8\ud1a0 \ubc16\uc73c\ub85c \ub0b4\ubcf4\ub0b4\uace0\ub294 \uac00\ubbf8\ub77c\uc2a4\uc5d0\uac8c \ud3ec\uad6c\uac00 \ub9c9\ud78c \ud30c\ub3d9\ud3ec\ub97c \ubc1c\uc0ac\ud574\uc11c, \uc790\uc2e0\uacfc \uc57c\ub9c8\ud1a0, \uadf8\ub9ac\uace0 \uac00\ubbf8\ub77c\uc2a4\uc640 \ud568\uaed8 \uc8fd\uc74c\uc744 \ub9de\uc774\ud55c\ub2e4.", "sentence_answer": "\uadf8 \uc88c\ud45c\uc5d0 \ub3c4\ub2ec\ud558\uace0\uc11c, \ud55c \uc678\uacc4\uc778\uc774 \uc720\ud0a4\uc758 \ubab8 \uc548\uc5d0 \ub4e4\uc5b4\uac00\uace0\ub294 \uac00\ubbf8\ub77c\uc2a4 \uc640 \uc774\uc2a4\uce78\ub378\uc740 \uac19\uc740 \uc758\uc2dd\uccb4\uc758 \ub450 \uc591\uba74\uc774\ub77c\uace0 \uc15c\uba85\ud55c\ub2e4.", "paragraph_id": "5823634-6-1", "paragraph_question": "question: \uacb0\uacfc\uc801\uc73c\ub85c \uc774\uc2a4\uce78\ub378\uacfc \uac19\uc740 \ud589\uc131\uc758 \uc774\ub984\uc740?, context: \uadf8 \uc88c\ud45c\uc5d0 \ub3c4\ub2ec\ud558\uace0\uc11c, \ud55c \uc678\uacc4\uc778\uc774 \uc720\ud0a4\uc758 \ubab8 \uc548\uc5d0 \ub4e4\uc5b4\uac00\uace0\ub294 \uac00\ubbf8\ub77c\uc2a4\uc640 \uc774\uc2a4\uce78\ub378\uc740 \uac19\uc740 \uc758\uc2dd\uccb4\uc758 \ub450 \uc591\uba74\uc774\ub77c\uace0 \uc15c\uba85\ud55c\ub2e4. \uadf8 \uc678\uacc4\uc778\uc740 \uadf8\ub4e4\uc758 \ud589\uc131\uc740 \uba38\uc9c0 \uc54a\uc544 \uba78\ub9dd\ud558\ub824 \ud558\uba70 \uac00\ubbf8\ub77c\uc2a4\ub294 \uc9c0\uad6c\uc778\ub4e4\uc744 \ubaa8\ub450 \ub9d0\uc0b4\ud55c \ub4a4\uc5d0 \uc9c0\uad6c\ub97c \uc0b4\uae30 \uc88b\uc740 \ud658\uacbd\uc73c\ub85c \uac1c\ub7c9\ud558\uace0 \uc788\ub294 \uac83\uc774\ub77c \uc124\uba85\ud55c\ub2e4. \ud558\uc9c0\ub9cc, \uc774\uc2a4\uce78\ub378\uc740 \uc790\uc2e0\ub4e4\uc758 \ud589\uc131\uacfc \uc6b4\uba85\uc744 \uac19\uc774 \ud558\ub824\uace0 \ud588\uae30 \ub54c\ubb38\uc5d0 \uc774\uc2a4\uce78\ub378\uc744 \uac00\ub454 \uac83\uc774\ub77c\uace0 \ub9d0\ud55c\ub2e4. \uc774\uc2a4\uce78\ub378\uc740 \uc720\ud0a4\uc5d0 \uc9c0\uad6c\uc5d0 \ubc29\uc0ac\ub2a5\uc744 \uc81c\uac70\ud560 \uc218 \uc788\ub294 \ub2a5\ub825\uc744 \uc8fc\uc785\uc2dc\ud0a4\uace0, \uadf8\ub4e4\uc740 \ub2e4\uc2dc \uc9c0\uad6c\ub97c \ud5a5\ud574 \ub3cc\uc544\uac00\ub824\uace0 \ud55c\ub2e4. \ud558\uc9c0\ub9cc \ud589\uc131\uc744 \ud0c8\ucd9c\ud558\ub294 \ub3d9\uc548 \uc0ac\uc774\ud1a0\uc640 \uc0ac\ub098\ub2e4\ub294 \uac00\ubbf8\ub77c\uc2a4\uc758 \ud798\uc758 \uadfc\uc6d0\uc744 \uc5c6\uc560\uae30 \uc704\ud574 \uace0\uc804\ubd84\ud22c\ud558\ub2e4 \ubaa9\uc228\uc744 \uc783\ub294\ub2e4. \uadf8\ub9ac\uace0 \ub4dc\ub514\uc5b4 \uc57c\ub9c8\ud1a0\ub294 \uc9c0\uad6c\ub85c \ub3cc\uc544\uac00\uac8c \ub418\uace0 \ub3cc\uc544\uac00\ub294 \ub3d9\uc548 \uc624\ud0a4\ub2e4 \uc120\uc7a5\uc740 \uc8fd\uc74c\uc744 \ub9de\uc774\ud55c\ub2e4. \uc120\uc6d0\ub4e4\uc740 \ub2e4\uc2dc \uc9c0\uad6c\ub85c \ub3cc\uc544\uc624\ub294 \uac83\uc744 \ub300\ub2e8\ud788 \uae30\ubed0\ud558\uc9c0\ub9cc \ub9c8\uc9c0\ub9c9\uc73c\ub85c \uc0b4\uc544\ub0a8\uc740 \uac00\ubbf8\ub77c\uc2a4 \ud568\ub300\uac00 \ub9e4\ubcf5\ud588\ub2e4\uac00 \uc2b5\uaca9\ud558\uace0 \uc57c\ub9c8\ud1a0\uc758 \uac70\uc758 \ubaa8\ub4e0 \ubd80\ubd84\uc774 \ud30c\uad34\ub420 \uc9c0\uacbd\uc5d0 \uc774\ub978\ub2e4. \uadf8\ub9ac\uace0 \ub370\uc2ac\ub7ec\uac00 \ub098\ud0c0\ub098\uc11c\ub294 \uc9c0\uad6c\ub97c \uacf5\uaca9\ud560 \uc0dd\uac01\uc740 \ub354 \uc774\uc0c1 \uc5c6\uc9c0\ub9cc, \ub300\ubd80\ubd84\uc758 \uac00\ubbf8\ub77c\uc2a4\uac00 \uc8fd\uc74c\uc744 \ub9c8\uc9c0\ud588\uace0, \ub370\uc2ac\ub7ec \uc790\uc2e0\ub4e4\uc740 \ubcf5\uc218\ub97c \uc704\ud574 \uc9c0\uad6c\ub97c \uc644\uc804\ud788 \ud30c\uad34\ud558\uaca0\ub2e4\uace0 \ub9d0\ud55c\ub2e4. \ucf54\ub2e4\uc774\ub294 \uc790\uc2e0\uc744 \uc81c\uc678\ud55c \ubaa8\ub4e0 \uc0ac\ub78c\uc744 \uc57c\ub9c8\ud1a0 \ubc16\uc73c\ub85c \ub0b4\ubcf4\ub0b4\uace0\ub294 \uac00\ubbf8\ub77c\uc2a4\uc5d0\uac8c \ud3ec\uad6c\uac00 \ub9c9\ud78c \ud30c\ub3d9\ud3ec\ub97c \ubc1c\uc0ac\ud574\uc11c, \uc790\uc2e0\uacfc \uc57c\ub9c8\ud1a0, \uadf8\ub9ac\uace0 \uac00\ubbf8\ub77c\uc2a4\uc640 \ud568\uaed8 \uc8fd\uc74c\uc744 \ub9de\uc774\ud55c\ub2e4."} {"question": "\ud0c0\uc774\uc644\uc744 \uac15\ud0c0\ud574 3000mm\uc774\uc0c1\uc758 \ud3ed\uc6b0\ub97c \ub0b4\ub9ac\uac8c \ud55c \ud0dc\ud48d\uc740?", "paragraph": "\ud0c0\uc774\uc644\uc5d0\uc11c\ub294 \uc57d 2\ub2ec\uc804\uc5d0 \ud0c0\uc774\uc644\uc744 \uac15\ud0c0\ud574 3000 mm\uac00 \ub118\ub294 \uae30\ub85d\uc801\uc778 \ud3ed\uc6b0\ub97c \ub0b4\ub9ac\uac8c \ud588\ub358 \ud0dc\ud48d \ubaa8\ub77c\uaf3f\uc73c\ub85c \uc778\ud55c \ud53c\ud574\ub85c \ud0dc\ud48d \ud30c\ub9c8\uac00 \uc628\ub2e4\ub294 \uc18c\uc2dd\uc5d0 \ub0a8\ubd80 \ud0c0\uc774\uc644 \uc8fc\ubbfc\ub4e4\uc774 \uc9c0\uccd0\uc788\uc5c8\ub2e4. 10\uc6d4 3\uc77c\uc5d0 \ubaa8\ub77c\uaf3f\uc73c\ub85c \uc778\ud55c \ud53c\ud574\uac00 \uac00\uc7a5 \uadf9\uc2ec\ud588\ub358 \uac00\uc624\uc29d \uc8fc\ubcc0 \uc9c0\uc5ed 6\uac1c\uc758 \ub9c8\uc744\uc5d0 \uac70\uc8fc\ud558\ub358 \uc8fc\ubbfc\ub4e4\uc758 \ub300\ud53c\uac00 \uc2dc\uc791\ub418\uc5c8\ub2e4. \uc911\uc559 \uae30\uc0c1 \uc0ac\ubb34\uad6d\uc5d0\uc11c \ud30c\ub9c8\uac00 \ud544\ub9ac\ud540 \ubd81\ucabd\uc73c\ub85c \uc774\ub3d9\ud560 \uac83\uc744 \uc608\uce21\ud558\uace0 \ud0c0\uc774\uc644 \ub0a8\ubd80\uc5d0 \uc704\uce58\ud55c \ud5dd\ucd98 \ubc18\ub3c4\uc5d0 \ud0dc\ud48d \uc8fc\uc758\ubcf4\ub97c \ub0b4\ub838\ub2e4. \ud0c0\uc774\uc644 \uacf5\ud56d\uc5d0\uc11c \uc774\ub959\ud558\ub294 55\uac1c\uc758 \ube44\ud589\ud3b8\uc774 \ucde8\uc18c\ub418\uac70\ub098 \uc9c0\uc5f0\ub418\uc5c8\ub2e4. 10\uc6d4 5\uc77c\uc5d0 \ud0c0\uc774\uc644 \ub0a8\ubd80\uc5d0\uc11c \uac70\uc8fc\ud558\ub294 \uc0ac\ub78c\ub4e4\uc5d0\uac8c \uac15\uc81c\ub300\ud53c\ub839\uc774 \ub0b4\ub824\uc84c\uace0 \uc774\uc5d0 \ub530\ub77c \uc57d 6\ucc9c\uba85\uc774 \ub300\ud53c\uc18c\ub97c \ucc3e\uc544\uc11c \ub300\ud53c\ud588\ub2e4. \ub610\ud55c \uc815\ubd80\uc5d0\uc11c \uae34\uae09\ud558\uac8c 200\uba85\uc758 \uc218\uc11d \uad70\uc0ac\ub4e4\uc5d0\uac8c \ub300\ud53c\uc640 \uad6c\ud638\uc791\uc804\uc744 \uc9c0\uc2dc\ud588\ub2e4.", "answer": "\ud0dc\ud48d \ubaa8\ub77c\uaf3f", "sentence": "\ud0c0\uc774\uc644\uc5d0\uc11c\ub294 \uc57d 2\ub2ec\uc804\uc5d0 \ud0c0\uc774\uc644\uc744 \uac15\ud0c0\ud574 3000 mm\uac00 \ub118\ub294 \uae30\ub85d\uc801\uc778 \ud3ed\uc6b0\ub97c \ub0b4\ub9ac\uac8c \ud588\ub358 \ud0dc\ud48d \ubaa8\ub77c\uaf3f \uc73c\ub85c \uc778\ud55c \ud53c\ud574\ub85c \ud0dc\ud48d \ud30c\ub9c8\uac00 \uc628\ub2e4\ub294 \uc18c\uc2dd\uc5d0 \ub0a8\ubd80 \ud0c0\uc774\uc644 \uc8fc\ubbfc\ub4e4\uc774 \uc9c0\uccd0\uc788\uc5c8\ub2e4.", "paragraph_sentence": " \ud0c0\uc774\uc644\uc5d0\uc11c\ub294 \uc57d 2\ub2ec\uc804\uc5d0 \ud0c0\uc774\uc644\uc744 \uac15\ud0c0\ud574 3000 mm\uac00 \ub118\ub294 \uae30\ub85d\uc801\uc778 \ud3ed\uc6b0\ub97c \ub0b4\ub9ac\uac8c \ud588\ub358 \ud0dc\ud48d \ubaa8\ub77c\uaf3f \uc73c\ub85c \uc778\ud55c \ud53c\ud574\ub85c \ud0dc\ud48d \ud30c\ub9c8\uac00 \uc628\ub2e4\ub294 \uc18c\uc2dd\uc5d0 \ub0a8\ubd80 \ud0c0\uc774\uc644 \uc8fc\ubbfc\ub4e4\uc774 \uc9c0\uccd0\uc788\uc5c8\ub2e4. 10\uc6d4 3\uc77c\uc5d0 \ubaa8\ub77c\uaf3f\uc73c\ub85c \uc778\ud55c \ud53c\ud574\uac00 \uac00\uc7a5 \uadf9\uc2ec\ud588\ub358 \uac00\uc624\uc29d \uc8fc\ubcc0 \uc9c0\uc5ed 6\uac1c\uc758 \ub9c8\uc744\uc5d0 \uac70\uc8fc\ud558\ub358 \uc8fc\ubbfc\ub4e4\uc758 \ub300\ud53c\uac00 \uc2dc\uc791\ub418\uc5c8\ub2e4. \uc911\uc559 \uae30\uc0c1 \uc0ac\ubb34\uad6d\uc5d0\uc11c \ud30c\ub9c8\uac00 \ud544\ub9ac\ud540 \ubd81\ucabd\uc73c\ub85c \uc774\ub3d9\ud560 \uac83\uc744 \uc608\uce21\ud558\uace0 \ud0c0\uc774\uc644 \ub0a8\ubd80\uc5d0 \uc704\uce58\ud55c \ud5dd\ucd98 \ubc18\ub3c4\uc5d0 \ud0dc\ud48d \uc8fc\uc758\ubcf4\ub97c \ub0b4\ub838\ub2e4. \ud0c0\uc774\uc644 \uacf5\ud56d\uc5d0\uc11c \uc774\ub959\ud558\ub294 55\uac1c\uc758 \ube44\ud589\ud3b8\uc774 \ucde8\uc18c\ub418\uac70\ub098 \uc9c0\uc5f0\ub418\uc5c8\ub2e4. 10\uc6d4 5\uc77c\uc5d0 \ud0c0\uc774\uc644 \ub0a8\ubd80\uc5d0\uc11c \uac70\uc8fc\ud558\ub294 \uc0ac\ub78c\ub4e4\uc5d0\uac8c \uac15\uc81c\ub300\ud53c\ub839\uc774 \ub0b4\ub824\uc84c\uace0 \uc774\uc5d0 \ub530\ub77c \uc57d 6\ucc9c\uba85\uc774 \ub300\ud53c\uc18c\ub97c \ucc3e\uc544\uc11c \ub300\ud53c\ud588\ub2e4. \ub610\ud55c \uc815\ubd80\uc5d0\uc11c \uae34\uae09\ud558\uac8c 200\uba85\uc758 \uc218\uc11d \uad70\uc0ac\ub4e4\uc5d0\uac8c \ub300\ud53c\uc640 \uad6c\ud638\uc791\uc804\uc744 \uc9c0\uc2dc\ud588\ub2e4.", "paragraph_answer": "\ud0c0\uc774\uc644\uc5d0\uc11c\ub294 \uc57d 2\ub2ec\uc804\uc5d0 \ud0c0\uc774\uc644\uc744 \uac15\ud0c0\ud574 3000 mm\uac00 \ub118\ub294 \uae30\ub85d\uc801\uc778 \ud3ed\uc6b0\ub97c \ub0b4\ub9ac\uac8c \ud588\ub358 \ud0dc\ud48d \ubaa8\ub77c\uaf3f \uc73c\ub85c \uc778\ud55c \ud53c\ud574\ub85c \ud0dc\ud48d \ud30c\ub9c8\uac00 \uc628\ub2e4\ub294 \uc18c\uc2dd\uc5d0 \ub0a8\ubd80 \ud0c0\uc774\uc644 \uc8fc\ubbfc\ub4e4\uc774 \uc9c0\uccd0\uc788\uc5c8\ub2e4. 10\uc6d4 3\uc77c\uc5d0 \ubaa8\ub77c\uaf3f\uc73c\ub85c \uc778\ud55c \ud53c\ud574\uac00 \uac00\uc7a5 \uadf9\uc2ec\ud588\ub358 \uac00\uc624\uc29d \uc8fc\ubcc0 \uc9c0\uc5ed 6\uac1c\uc758 \ub9c8\uc744\uc5d0 \uac70\uc8fc\ud558\ub358 \uc8fc\ubbfc\ub4e4\uc758 \ub300\ud53c\uac00 \uc2dc\uc791\ub418\uc5c8\ub2e4. \uc911\uc559 \uae30\uc0c1 \uc0ac\ubb34\uad6d\uc5d0\uc11c \ud30c\ub9c8\uac00 \ud544\ub9ac\ud540 \ubd81\ucabd\uc73c\ub85c \uc774\ub3d9\ud560 \uac83\uc744 \uc608\uce21\ud558\uace0 \ud0c0\uc774\uc644 \ub0a8\ubd80\uc5d0 \uc704\uce58\ud55c \ud5dd\ucd98 \ubc18\ub3c4\uc5d0 \ud0dc\ud48d \uc8fc\uc758\ubcf4\ub97c \ub0b4\ub838\ub2e4. \ud0c0\uc774\uc644 \uacf5\ud56d\uc5d0\uc11c \uc774\ub959\ud558\ub294 55\uac1c\uc758 \ube44\ud589\ud3b8\uc774 \ucde8\uc18c\ub418\uac70\ub098 \uc9c0\uc5f0\ub418\uc5c8\ub2e4. 10\uc6d4 5\uc77c\uc5d0 \ud0c0\uc774\uc644 \ub0a8\ubd80\uc5d0\uc11c \uac70\uc8fc\ud558\ub294 \uc0ac\ub78c\ub4e4\uc5d0\uac8c \uac15\uc81c\ub300\ud53c\ub839\uc774 \ub0b4\ub824\uc84c\uace0 \uc774\uc5d0 \ub530\ub77c \uc57d 6\ucc9c\uba85\uc774 \ub300\ud53c\uc18c\ub97c \ucc3e\uc544\uc11c \ub300\ud53c\ud588\ub2e4. \ub610\ud55c \uc815\ubd80\uc5d0\uc11c \uae34\uae09\ud558\uac8c 200\uba85\uc758 \uc218\uc11d \uad70\uc0ac\ub4e4\uc5d0\uac8c \ub300\ud53c\uc640 \uad6c\ud638\uc791\uc804\uc744 \uc9c0\uc2dc\ud588\ub2e4.", "sentence_answer": "\ud0c0\uc774\uc644\uc5d0\uc11c\ub294 \uc57d 2\ub2ec\uc804\uc5d0 \ud0c0\uc774\uc644\uc744 \uac15\ud0c0\ud574 3000 mm\uac00 \ub118\ub294 \uae30\ub85d\uc801\uc778 \ud3ed\uc6b0\ub97c \ub0b4\ub9ac\uac8c \ud588\ub358 \ud0dc\ud48d \ubaa8\ub77c\uaf3f \uc73c\ub85c \uc778\ud55c \ud53c\ud574\ub85c \ud0dc\ud48d \ud30c\ub9c8\uac00 \uc628\ub2e4\ub294 \uc18c\uc2dd\uc5d0 \ub0a8\ubd80 \ud0c0\uc774\uc644 \uc8fc\ubbfc\ub4e4\uc774 \uc9c0\uccd0\uc788\uc5c8\ub2e4.", "paragraph_id": "6565500-1-0", "paragraph_question": "question: \ud0c0\uc774\uc644\uc744 \uac15\ud0c0\ud574 3000mm\uc774\uc0c1\uc758 \ud3ed\uc6b0\ub97c \ub0b4\ub9ac\uac8c \ud55c \ud0dc\ud48d\uc740?, context: \ud0c0\uc774\uc644\uc5d0\uc11c\ub294 \uc57d 2\ub2ec\uc804\uc5d0 \ud0c0\uc774\uc644\uc744 \uac15\ud0c0\ud574 3000 mm\uac00 \ub118\ub294 \uae30\ub85d\uc801\uc778 \ud3ed\uc6b0\ub97c \ub0b4\ub9ac\uac8c \ud588\ub358 \ud0dc\ud48d \ubaa8\ub77c\uaf3f\uc73c\ub85c \uc778\ud55c \ud53c\ud574\ub85c \ud0dc\ud48d \ud30c\ub9c8\uac00 \uc628\ub2e4\ub294 \uc18c\uc2dd\uc5d0 \ub0a8\ubd80 \ud0c0\uc774\uc644 \uc8fc\ubbfc\ub4e4\uc774 \uc9c0\uccd0\uc788\uc5c8\ub2e4. 10\uc6d4 3\uc77c\uc5d0 \ubaa8\ub77c\uaf3f\uc73c\ub85c \uc778\ud55c \ud53c\ud574\uac00 \uac00\uc7a5 \uadf9\uc2ec\ud588\ub358 \uac00\uc624\uc29d \uc8fc\ubcc0 \uc9c0\uc5ed 6\uac1c\uc758 \ub9c8\uc744\uc5d0 \uac70\uc8fc\ud558\ub358 \uc8fc\ubbfc\ub4e4\uc758 \ub300\ud53c\uac00 \uc2dc\uc791\ub418\uc5c8\ub2e4. \uc911\uc559 \uae30\uc0c1 \uc0ac\ubb34\uad6d\uc5d0\uc11c \ud30c\ub9c8\uac00 \ud544\ub9ac\ud540 \ubd81\ucabd\uc73c\ub85c \uc774\ub3d9\ud560 \uac83\uc744 \uc608\uce21\ud558\uace0 \ud0c0\uc774\uc644 \ub0a8\ubd80\uc5d0 \uc704\uce58\ud55c \ud5dd\ucd98 \ubc18\ub3c4\uc5d0 \ud0dc\ud48d \uc8fc\uc758\ubcf4\ub97c \ub0b4\ub838\ub2e4. \ud0c0\uc774\uc644 \uacf5\ud56d\uc5d0\uc11c \uc774\ub959\ud558\ub294 55\uac1c\uc758 \ube44\ud589\ud3b8\uc774 \ucde8\uc18c\ub418\uac70\ub098 \uc9c0\uc5f0\ub418\uc5c8\ub2e4. 10\uc6d4 5\uc77c\uc5d0 \ud0c0\uc774\uc644 \ub0a8\ubd80\uc5d0\uc11c \uac70\uc8fc\ud558\ub294 \uc0ac\ub78c\ub4e4\uc5d0\uac8c \uac15\uc81c\ub300\ud53c\ub839\uc774 \ub0b4\ub824\uc84c\uace0 \uc774\uc5d0 \ub530\ub77c \uc57d 6\ucc9c\uba85\uc774 \ub300\ud53c\uc18c\ub97c \ucc3e\uc544\uc11c \ub300\ud53c\ud588\ub2e4. \ub610\ud55c \uc815\ubd80\uc5d0\uc11c \uae34\uae09\ud558\uac8c 200\uba85\uc758 \uc218\uc11d \uad70\uc0ac\ub4e4\uc5d0\uac8c \ub300\ud53c\uc640 \uad6c\ud638\uc791\uc804\uc744 \uc9c0\uc2dc\ud588\ub2e4."} {"question": "\uad8c\uc778\uc219\uc774 \ub178\ub3d9\uc6b4\ub3d9\uc744 \uc2dc\uc791\ud55c \ub144\ub3c4\ub294?", "paragraph": "\uad8c\uc778\uc219\uc740 1964\ub144\uc5d0 \ud0dc\uc5b4\ub0ac\ub2e4. 1982\ub144 \uc11c\uc6b8\ub300\ud559\uad50 \uc758\ub958\ud559\uacfc\uc5d0 \uc785\ud559\ud55c \ud6c4 \ud559\uc0dd\uc6b4\ub3d9\uc744 \ud558\uc600\uace0, 1985\ub144\ubd80\ud130 \ub178\ub3d9\uc6b4\ub3d9\uc744 \ud558\uc600\ub2e4. 1989\ub144\uc5d0 \uc11c\uc6b8 \uad6c\ub85c\ub3d9\uc5d0 '\ub178\ub3d9\uc778\uad8c\ud68c\uad00'\uc744 \uc138\uc6cc \uc0c1\ub2f4 \uad50\uc721 \ud65c\ub3d9\uc744 \ud558\uc600\ub2e4. 1994\ub144\uc5d0 \ubbf8\uad6d\uc73c\ub85c \uac74\ub108\uac00 \ub7ff\uac70\uc2a4 \ub300\ud559\uad50\uc5d0\uc11c \uc5ec\uc131\ud559 \uc11d\uc0ac\ud559\uc704\ub97c, 2000\ub144\uc5d0 \ud074\ub77d(Clark)\ub300\ud559\uad50\uc5d0\uc11c \uc5ec\uc131\ud559 \ubc15\uc0ac\ud559\uc704\ub97c \ubc1b\uc558\ub2e4. \ud558\ubc84\ub4dc \ub300\ud559\uad50 \ud55c\uad6d\uc5f0\uad6c\uc18c\uc5d0\uc11c \ubc15\uc0ac\ud6c4 \uacfc\uc815 \uc5f0\uad6c\uc6d0\uc73c\ub85c\uc11c 1\ub144 \ub3d9\uc548 \uadfc\ubb34\ud558\uc600\uace0, \ucf5c\ub86c\ube44\uc544 \ub300\ud559\uad50 \ub3d9\uc544\uc2dc\uc544\uc5f0\uad6c\uc18c\uc5d0\uc11c \ucd08\uccad\uc5f0\uad6c\uc6d0\uc744 \uc9c0\ub0c8\ub2e4. \ub0a8\ud50c\ub85c\ub9ac\ub2e4 \uc8fc\ub9bd\ub300\ud559\uad50(University of South Florida) \uc5ec\uc131\ud559\uacfc \uad50\uc218\ub97c \uc5ed\uc784\ud588\uc73c\uba70, 2003\ub144\ubd80\ud130 \uba85\uc9c0\ub300\ud559\uad50 \uae30\ucd08\uad50\uc721\ub300\ud559\uc5d0\uc11c \uc5ec\uc131\ud559\uc744 \uac00\ub974\ucce4\ub2e4. 2014\ub144 2\uc6d4, \ud55c\uad6d\uc131\ud3ed\ub825\uc0c1\ub2f4\uc18c \ubd80\uc124 \uc5f0\uad6c\uc18c <\uc6b8\ub9bc> \ucd08\ub300 \uc18c\uc7a5\uc744 \ub9e1\uc558\ub2e4.", "answer": "1985\ub144", "sentence": "1982\ub144 \uc11c\uc6b8\ub300\ud559\uad50 \uc758\ub958\ud559\uacfc\uc5d0 \uc785\ud559\ud55c \ud6c4 \ud559\uc0dd\uc6b4\ub3d9\uc744 \ud558\uc600\uace0, 1985\ub144 \ubd80\ud130 \ub178\ub3d9\uc6b4\ub3d9\uc744 \ud558\uc600\ub2e4.", "paragraph_sentence": "\uad8c\uc778\uc219\uc740 1964\ub144\uc5d0 \ud0dc\uc5b4\ub0ac\ub2e4. 1982\ub144 \uc11c\uc6b8\ub300\ud559\uad50 \uc758\ub958\ud559\uacfc\uc5d0 \uc785\ud559\ud55c \ud6c4 \ud559\uc0dd\uc6b4\ub3d9\uc744 \ud558\uc600\uace0, 1985\ub144 \ubd80\ud130 \ub178\ub3d9\uc6b4\ub3d9\uc744 \ud558\uc600\ub2e4. 1989\ub144\uc5d0 \uc11c\uc6b8 \uad6c\ub85c\ub3d9\uc5d0 '\ub178\ub3d9\uc778\uad8c\ud68c\uad00'\uc744 \uc138\uc6cc \uc0c1\ub2f4 \uad50\uc721 \ud65c\ub3d9\uc744 \ud558\uc600\ub2e4. 1994\ub144\uc5d0 \ubbf8\uad6d\uc73c\ub85c \uac74\ub108\uac00 \ub7ff\uac70\uc2a4 \ub300\ud559\uad50\uc5d0\uc11c \uc5ec\uc131\ud559 \uc11d\uc0ac\ud559\uc704\ub97c, 2000\ub144\uc5d0 \ud074\ub77d(Clark)\ub300\ud559\uad50\uc5d0\uc11c \uc5ec\uc131\ud559 \ubc15\uc0ac\ud559\uc704\ub97c \ubc1b\uc558\ub2e4. \ud558\ubc84\ub4dc \ub300\ud559\uad50 \ud55c\uad6d\uc5f0\uad6c\uc18c\uc5d0\uc11c \ubc15\uc0ac\ud6c4 \uacfc\uc815 \uc5f0\uad6c\uc6d0\uc73c\ub85c\uc11c 1\ub144 \ub3d9\uc548 \uadfc\ubb34\ud558\uc600\uace0, \ucf5c\ub86c\ube44\uc544 \ub300\ud559\uad50 \ub3d9\uc544\uc2dc\uc544\uc5f0\uad6c\uc18c\uc5d0\uc11c \ucd08\uccad\uc5f0\uad6c\uc6d0\uc744 \uc9c0\ub0c8\ub2e4. \ub0a8\ud50c\ub85c\ub9ac\ub2e4 \uc8fc\ub9bd\ub300\ud559\uad50(University of South Florida) \uc5ec\uc131\ud559\uacfc \uad50\uc218\ub97c \uc5ed\uc784\ud588\uc73c\uba70, 2003\ub144\ubd80\ud130 \uba85\uc9c0\ub300\ud559\uad50 \uae30\ucd08\uad50\uc721\ub300\ud559\uc5d0\uc11c \uc5ec\uc131\ud559\uc744 \uac00\ub974\ucce4\ub2e4. 2014\ub144 2\uc6d4, \ud55c\uad6d\uc131\ud3ed\ub825\uc0c1\ub2f4\uc18c \ubd80\uc124 \uc5f0\uad6c\uc18c <\uc6b8\ub9bc> \ucd08\ub300 \uc18c\uc7a5\uc744 \ub9e1\uc558\ub2e4.", "paragraph_answer": "\uad8c\uc778\uc219\uc740 1964\ub144\uc5d0 \ud0dc\uc5b4\ub0ac\ub2e4. 1982\ub144 \uc11c\uc6b8\ub300\ud559\uad50 \uc758\ub958\ud559\uacfc\uc5d0 \uc785\ud559\ud55c \ud6c4 \ud559\uc0dd\uc6b4\ub3d9\uc744 \ud558\uc600\uace0, 1985\ub144 \ubd80\ud130 \ub178\ub3d9\uc6b4\ub3d9\uc744 \ud558\uc600\ub2e4. 1989\ub144\uc5d0 \uc11c\uc6b8 \uad6c\ub85c\ub3d9\uc5d0 '\ub178\ub3d9\uc778\uad8c\ud68c\uad00'\uc744 \uc138\uc6cc \uc0c1\ub2f4 \uad50\uc721 \ud65c\ub3d9\uc744 \ud558\uc600\ub2e4. 1994\ub144\uc5d0 \ubbf8\uad6d\uc73c\ub85c \uac74\ub108\uac00 \ub7ff\uac70\uc2a4 \ub300\ud559\uad50\uc5d0\uc11c \uc5ec\uc131\ud559 \uc11d\uc0ac\ud559\uc704\ub97c, 2000\ub144\uc5d0 \ud074\ub77d(Clark)\ub300\ud559\uad50\uc5d0\uc11c \uc5ec\uc131\ud559 \ubc15\uc0ac\ud559\uc704\ub97c \ubc1b\uc558\ub2e4. \ud558\ubc84\ub4dc \ub300\ud559\uad50 \ud55c\uad6d\uc5f0\uad6c\uc18c\uc5d0\uc11c \ubc15\uc0ac\ud6c4 \uacfc\uc815 \uc5f0\uad6c\uc6d0\uc73c\ub85c\uc11c 1\ub144 \ub3d9\uc548 \uadfc\ubb34\ud558\uc600\uace0, \ucf5c\ub86c\ube44\uc544 \ub300\ud559\uad50 \ub3d9\uc544\uc2dc\uc544\uc5f0\uad6c\uc18c\uc5d0\uc11c \ucd08\uccad\uc5f0\uad6c\uc6d0\uc744 \uc9c0\ub0c8\ub2e4. \ub0a8\ud50c\ub85c\ub9ac\ub2e4 \uc8fc\ub9bd\ub300\ud559\uad50(University of South Florida) \uc5ec\uc131\ud559\uacfc \uad50\uc218\ub97c \uc5ed\uc784\ud588\uc73c\uba70, 2003\ub144\ubd80\ud130 \uba85\uc9c0\ub300\ud559\uad50 \uae30\ucd08\uad50\uc721\ub300\ud559\uc5d0\uc11c \uc5ec\uc131\ud559\uc744 \uac00\ub974\ucce4\ub2e4. 2014\ub144 2\uc6d4, \ud55c\uad6d\uc131\ud3ed\ub825\uc0c1\ub2f4\uc18c \ubd80\uc124 \uc5f0\uad6c\uc18c <\uc6b8\ub9bc> \ucd08\ub300 \uc18c\uc7a5\uc744 \ub9e1\uc558\ub2e4.", "sentence_answer": "1982\ub144 \uc11c\uc6b8\ub300\ud559\uad50 \uc758\ub958\ud559\uacfc\uc5d0 \uc785\ud559\ud55c \ud6c4 \ud559\uc0dd\uc6b4\ub3d9\uc744 \ud558\uc600\uace0, 1985\ub144 \ubd80\ud130 \ub178\ub3d9\uc6b4\ub3d9\uc744 \ud558\uc600\ub2e4.", "paragraph_id": "6477667-0-1", "paragraph_question": "question: \uad8c\uc778\uc219\uc774 \ub178\ub3d9\uc6b4\ub3d9\uc744 \uc2dc\uc791\ud55c \ub144\ub3c4\ub294?, context: \uad8c\uc778\uc219\uc740 1964\ub144\uc5d0 \ud0dc\uc5b4\ub0ac\ub2e4. 1982\ub144 \uc11c\uc6b8\ub300\ud559\uad50 \uc758\ub958\ud559\uacfc\uc5d0 \uc785\ud559\ud55c \ud6c4 \ud559\uc0dd\uc6b4\ub3d9\uc744 \ud558\uc600\uace0, 1985\ub144\ubd80\ud130 \ub178\ub3d9\uc6b4\ub3d9\uc744 \ud558\uc600\ub2e4. 1989\ub144\uc5d0 \uc11c\uc6b8 \uad6c\ub85c\ub3d9\uc5d0 '\ub178\ub3d9\uc778\uad8c\ud68c\uad00'\uc744 \uc138\uc6cc \uc0c1\ub2f4 \uad50\uc721 \ud65c\ub3d9\uc744 \ud558\uc600\ub2e4. 1994\ub144\uc5d0 \ubbf8\uad6d\uc73c\ub85c \uac74\ub108\uac00 \ub7ff\uac70\uc2a4 \ub300\ud559\uad50\uc5d0\uc11c \uc5ec\uc131\ud559 \uc11d\uc0ac\ud559\uc704\ub97c, 2000\ub144\uc5d0 \ud074\ub77d(Clark)\ub300\ud559\uad50\uc5d0\uc11c \uc5ec\uc131\ud559 \ubc15\uc0ac\ud559\uc704\ub97c \ubc1b\uc558\ub2e4. \ud558\ubc84\ub4dc \ub300\ud559\uad50 \ud55c\uad6d\uc5f0\uad6c\uc18c\uc5d0\uc11c \ubc15\uc0ac\ud6c4 \uacfc\uc815 \uc5f0\uad6c\uc6d0\uc73c\ub85c\uc11c 1\ub144 \ub3d9\uc548 \uadfc\ubb34\ud558\uc600\uace0, \ucf5c\ub86c\ube44\uc544 \ub300\ud559\uad50 \ub3d9\uc544\uc2dc\uc544\uc5f0\uad6c\uc18c\uc5d0\uc11c \ucd08\uccad\uc5f0\uad6c\uc6d0\uc744 \uc9c0\ub0c8\ub2e4. \ub0a8\ud50c\ub85c\ub9ac\ub2e4 \uc8fc\ub9bd\ub300\ud559\uad50(University of South Florida) \uc5ec\uc131\ud559\uacfc \uad50\uc218\ub97c \uc5ed\uc784\ud588\uc73c\uba70, 2003\ub144\ubd80\ud130 \uba85\uc9c0\ub300\ud559\uad50 \uae30\ucd08\uad50\uc721\ub300\ud559\uc5d0\uc11c \uc5ec\uc131\ud559\uc744 \uac00\ub974\ucce4\ub2e4. 2014\ub144 2\uc6d4, \ud55c\uad6d\uc131\ud3ed\ub825\uc0c1\ub2f4\uc18c \ubd80\uc124 \uc5f0\uad6c\uc18c <\uc6b8\ub9bc> \ucd08\ub300 \uc18c\uc7a5\uc744 \ub9e1\uc558\ub2e4."} {"question": "2000\ub144 6\uc6d4\uc5d0 \uc870\uc131\ubbfc\uc744 \ud3c9\uac00\ud558\ub294 \uae00\uc744 \uc5f0\uc7ac\ud55c \uc0ac\ub78c\uc740?", "paragraph": "\uc870\uc131\ubbfc\uc740 2002\ub144\ubd80\ud130 2004\ub144\uae4c\uc9c0 3\ub144\uc5d0 \uac78\uccd0 \ucd5c\uc9c4\uc2e4(\uac00\uc871)\uc744 \uc0c1\ub300\ub85c \uc815\uc870\uc758 \uc758\ubb34 \uc704\ubc18 \ud589\uc704, \ud3ed\ub825 \ud589\uc704, \uc0ac\uae30 \ud589\uc704, \uba85\uc608\ud6fc\uc190 \ud589\uc704, \ud5c8\uc704\uc0ac\uc2e4\uc720\ud3ec \ud589\uc704\ub97c \uc800\uc9c8\ub800\ub294\ub370, 2002\ub144 12\uc6d4 18\uc77c \uc624\ud6c4 2\uc2dc \ud604\uc7ac \uc815\uc870\uc758 \uc758\ubb34 \uc704\ubc18 \ud589\uc704, \ud3ed\ub825 \ud589\uc704, \uc0ac\uae30 \ud589\uc704\ub97c \uc774\ubbf8 \uc800\uc9c0\ub978 \uc0c1\ud0dc\uc600\uace0 \uadf8 \uac01\uac01\uc5d0 \ub300\ud574 \uc5ec\ub860 \uc870\uc791\uc744 \uac00\ub3d9\uc2dc\ucf1c \ud53c\ud574\uc790 \ucd5c\uc9c4\uc2e4\uc744 \uac00\ud574\uc790\ub85c \ubaa8\ub294 \uc5b8\ub860\ud50c\ub808\uc774\ub97c \ud588\ub2e4. \uc774 \uc5b8\ub860\ud50c\ub808\uc774\ub97c \ud1b5\ud574 \uba85\uc608\ud6fc\uc190 \ud589\uc704, \ud5c8\uc704\uc0ac\uc2e4\uc720\ud3ec \ud589\uc704\ub3c4 \ucd94\uac00\ub418\uc5c8\ub2e4. \uc878\uc9c0\uc5d0 \uac00\ud574\uc790\ub85c \ubab0\ub9b0 \ucd5c\uc9c4\uc2e4\uc740 \uc0ac\uc2e4\uad00\uacc4\ub97c \ubc14\ub85c\uc7a1\ub294 \uc5b8\ub860 \ub300\uc751\uc744 \ubc14\ub85c \ud588\ub2e4. \uae30\uc790\ub098 \ub300\uc911 \uc785\uc7a5\uc5d0\uc11c\ub294 \ub204\uad6c\uc758 \ub9d0\uc774 \ub9de\ub294\uc9c0 \uc54c \uc218 \uc5c6\uc744 \uac83\uc774\uae30\uc5d0 \uac1d\uad00\uc801 \ud310\ub2e8\uc744 \ub0b4\ub9b4 \uc218 \uc788\ub3c4\ub85d \uc774\ud63c\uc720\ucc45\uc8fc\uc758\ub77c\ub294 \uc900\uac70\ud2c0\uc744 \ud65c\uc6a9\ud558\uc600\ub294\ub370, \ucd5c\uc9c4\uc2e4\uc740 '\uc774\ud63c\ud560 \uc758\uc0ac\uac00 \uc804\ud600 \uc5c6\uc74c'\uc744 \ucc9c\uba85\ud568\uc73c\ub85c\uc368 \uc790\uc2e0\uc758 \ub9d0\uc774 \ud5c8\uc704\uc774\uba74 \uc870\uc131\ubbfc\uc5d0 \uc758\ud574 \uc790\uc2e0\uc774 \uc18c\uc1a1\uc774\ud63c\uc758 \ub300\uc0c1\uc774 \ub418\ub3c4\ub85d \ud310\uc744 \ub2e8\uc21c\ud654\uc2dc\ucf30\ub2e4. 2000\ub144 6\uc6d4 \ucd5c\uc9c4\uc2e4\uc740 \uc870\uc131\ubbfc\uc744 \ud3c9\uac00\ud558\ub294 \uae00\uc744 \uc2a4\ud3ec\uce20\uc11c\uc6b8\uc5d0 \uc5f0\uc7ac\ud55c \uc801\uc774 \uc788\uae30 \ub54c\ubb38\uc5d0 \uc870\uc131\ubbfc\uc758 \uc2e4\uccb4 \uaddc\uba85\uacfc \uad00\ub828\ud558\uc5ec \uacb0\uc790\ud574\uc9c0\ud560 \ud544\uc694\ub3c4 \uc788\ub294 \uc0c1\ud669\uc774\uc5c8\ub2e4.", "answer": "\ucd5c\uc9c4\uc2e4", "sentence": "\uc870\uc131\ubbfc\uc740 2002\ub144\ubd80\ud130 2004\ub144\uae4c\uc9c0 3\ub144\uc5d0 \uac78\uccd0 \ucd5c\uc9c4\uc2e4 (\uac00\uc871)", "paragraph_sentence": " \uc870\uc131\ubbfc\uc740 2002\ub144\ubd80\ud130 2004\ub144\uae4c\uc9c0 3\ub144\uc5d0 \uac78\uccd0 \ucd5c\uc9c4\uc2e4 (\uac00\uc871) \uc744 \uc0c1\ub300\ub85c \uc815\uc870\uc758 \uc758\ubb34 \uc704\ubc18 \ud589\uc704, \ud3ed\ub825 \ud589\uc704, \uc0ac\uae30 \ud589\uc704, \uba85\uc608\ud6fc\uc190 \ud589\uc704, \ud5c8\uc704\uc0ac\uc2e4\uc720\ud3ec \ud589\uc704\ub97c \uc800\uc9c8\ub800\ub294\ub370, 2002\ub144 12\uc6d4 18\uc77c \uc624\ud6c4 2\uc2dc \ud604\uc7ac \uc815\uc870\uc758 \uc758\ubb34 \uc704\ubc18 \ud589\uc704, \ud3ed\ub825 \ud589\uc704, \uc0ac\uae30 \ud589\uc704\ub97c \uc774\ubbf8 \uc800\uc9c0\ub978 \uc0c1\ud0dc\uc600\uace0 \uadf8 \uac01\uac01\uc5d0 \ub300\ud574 \uc5ec\ub860 \uc870\uc791\uc744 \uac00\ub3d9\uc2dc\ucf1c \ud53c\ud574\uc790 \ucd5c\uc9c4\uc2e4\uc744 \uac00\ud574\uc790\ub85c \ubaa8\ub294 \uc5b8\ub860\ud50c\ub808\uc774\ub97c \ud588\ub2e4. \uc774 \uc5b8\ub860\ud50c\ub808\uc774\ub97c \ud1b5\ud574 \uba85\uc608\ud6fc\uc190 \ud589\uc704, \ud5c8\uc704\uc0ac\uc2e4\uc720\ud3ec \ud589\uc704\ub3c4 \ucd94\uac00\ub418\uc5c8\ub2e4. \uc878\uc9c0\uc5d0 \uac00\ud574\uc790\ub85c \ubab0\ub9b0 \ucd5c\uc9c4\uc2e4\uc740 \uc0ac\uc2e4\uad00\uacc4\ub97c \ubc14\ub85c\uc7a1\ub294 \uc5b8\ub860 \ub300\uc751\uc744 \ubc14\ub85c \ud588\ub2e4. \uae30\uc790\ub098 \ub300\uc911 \uc785\uc7a5\uc5d0\uc11c\ub294 \ub204\uad6c\uc758 \ub9d0\uc774 \ub9de\ub294\uc9c0 \uc54c \uc218 \uc5c6\uc744 \uac83\uc774\uae30\uc5d0 \uac1d\uad00\uc801 \ud310\ub2e8\uc744 \ub0b4\ub9b4 \uc218 \uc788\ub3c4\ub85d \uc774\ud63c\uc720\ucc45\uc8fc\uc758\ub77c\ub294 \uc900\uac70\ud2c0\uc744 \ud65c\uc6a9\ud558\uc600\ub294\ub370, \ucd5c\uc9c4\uc2e4\uc740 '\uc774\ud63c\ud560 \uc758\uc0ac\uac00 \uc804\ud600 \uc5c6\uc74c'\uc744 \ucc9c\uba85\ud568\uc73c\ub85c\uc368 \uc790\uc2e0\uc758 \ub9d0\uc774 \ud5c8\uc704\uc774\uba74 \uc870\uc131\ubbfc\uc5d0 \uc758\ud574 \uc790\uc2e0\uc774 \uc18c\uc1a1\uc774\ud63c\uc758 \ub300\uc0c1\uc774 \ub418\ub3c4\ub85d \ud310\uc744 \ub2e8\uc21c\ud654\uc2dc\ucf30\ub2e4. 2000\ub144 6\uc6d4 \ucd5c\uc9c4\uc2e4\uc740 \uc870\uc131\ubbfc\uc744 \ud3c9\uac00\ud558\ub294 \uae00\uc744 \uc2a4\ud3ec\uce20\uc11c\uc6b8\uc5d0 \uc5f0\uc7ac\ud55c \uc801\uc774 \uc788\uae30 \ub54c\ubb38\uc5d0 \uc870\uc131\ubbfc\uc758 \uc2e4\uccb4 \uaddc\uba85\uacfc \uad00\ub828\ud558\uc5ec \uacb0\uc790\ud574\uc9c0\ud560 \ud544\uc694\ub3c4 \uc788\ub294 \uc0c1\ud669\uc774\uc5c8\ub2e4.", "paragraph_answer": "\uc870\uc131\ubbfc\uc740 2002\ub144\ubd80\ud130 2004\ub144\uae4c\uc9c0 3\ub144\uc5d0 \uac78\uccd0 \ucd5c\uc9c4\uc2e4 (\uac00\uc871)\uc744 \uc0c1\ub300\ub85c \uc815\uc870\uc758 \uc758\ubb34 \uc704\ubc18 \ud589\uc704, \ud3ed\ub825 \ud589\uc704, \uc0ac\uae30 \ud589\uc704, \uba85\uc608\ud6fc\uc190 \ud589\uc704, \ud5c8\uc704\uc0ac\uc2e4\uc720\ud3ec \ud589\uc704\ub97c \uc800\uc9c8\ub800\ub294\ub370, 2002\ub144 12\uc6d4 18\uc77c \uc624\ud6c4 2\uc2dc \ud604\uc7ac \uc815\uc870\uc758 \uc758\ubb34 \uc704\ubc18 \ud589\uc704, \ud3ed\ub825 \ud589\uc704, \uc0ac\uae30 \ud589\uc704\ub97c \uc774\ubbf8 \uc800\uc9c0\ub978 \uc0c1\ud0dc\uc600\uace0 \uadf8 \uac01\uac01\uc5d0 \ub300\ud574 \uc5ec\ub860 \uc870\uc791\uc744 \uac00\ub3d9\uc2dc\ucf1c \ud53c\ud574\uc790 \ucd5c\uc9c4\uc2e4\uc744 \uac00\ud574\uc790\ub85c \ubaa8\ub294 \uc5b8\ub860\ud50c\ub808\uc774\ub97c \ud588\ub2e4. \uc774 \uc5b8\ub860\ud50c\ub808\uc774\ub97c \ud1b5\ud574 \uba85\uc608\ud6fc\uc190 \ud589\uc704, \ud5c8\uc704\uc0ac\uc2e4\uc720\ud3ec \ud589\uc704\ub3c4 \ucd94\uac00\ub418\uc5c8\ub2e4. \uc878\uc9c0\uc5d0 \uac00\ud574\uc790\ub85c \ubab0\ub9b0 \ucd5c\uc9c4\uc2e4\uc740 \uc0ac\uc2e4\uad00\uacc4\ub97c \ubc14\ub85c\uc7a1\ub294 \uc5b8\ub860 \ub300\uc751\uc744 \ubc14\ub85c \ud588\ub2e4. \uae30\uc790\ub098 \ub300\uc911 \uc785\uc7a5\uc5d0\uc11c\ub294 \ub204\uad6c\uc758 \ub9d0\uc774 \ub9de\ub294\uc9c0 \uc54c \uc218 \uc5c6\uc744 \uac83\uc774\uae30\uc5d0 \uac1d\uad00\uc801 \ud310\ub2e8\uc744 \ub0b4\ub9b4 \uc218 \uc788\ub3c4\ub85d \uc774\ud63c\uc720\ucc45\uc8fc\uc758\ub77c\ub294 \uc900\uac70\ud2c0\uc744 \ud65c\uc6a9\ud558\uc600\ub294\ub370, \ucd5c\uc9c4\uc2e4\uc740 '\uc774\ud63c\ud560 \uc758\uc0ac\uac00 \uc804\ud600 \uc5c6\uc74c'\uc744 \ucc9c\uba85\ud568\uc73c\ub85c\uc368 \uc790\uc2e0\uc758 \ub9d0\uc774 \ud5c8\uc704\uc774\uba74 \uc870\uc131\ubbfc\uc5d0 \uc758\ud574 \uc790\uc2e0\uc774 \uc18c\uc1a1\uc774\ud63c\uc758 \ub300\uc0c1\uc774 \ub418\ub3c4\ub85d \ud310\uc744 \ub2e8\uc21c\ud654\uc2dc\ucf30\ub2e4. 2000\ub144 6\uc6d4 \ucd5c\uc9c4\uc2e4\uc740 \uc870\uc131\ubbfc\uc744 \ud3c9\uac00\ud558\ub294 \uae00\uc744 \uc2a4\ud3ec\uce20\uc11c\uc6b8\uc5d0 \uc5f0\uc7ac\ud55c \uc801\uc774 \uc788\uae30 \ub54c\ubb38\uc5d0 \uc870\uc131\ubbfc\uc758 \uc2e4\uccb4 \uaddc\uba85\uacfc \uad00\ub828\ud558\uc5ec \uacb0\uc790\ud574\uc9c0\ud560 \ud544\uc694\ub3c4 \uc788\ub294 \uc0c1\ud669\uc774\uc5c8\ub2e4.", "sentence_answer": "\uc870\uc131\ubbfc\uc740 2002\ub144\ubd80\ud130 2004\ub144\uae4c\uc9c0 3\ub144\uc5d0 \uac78\uccd0 \ucd5c\uc9c4\uc2e4 (\uac00\uc871)", "paragraph_id": "6560235-8-1", "paragraph_question": "question: 2000\ub144 6\uc6d4\uc5d0 \uc870\uc131\ubbfc\uc744 \ud3c9\uac00\ud558\ub294 \uae00\uc744 \uc5f0\uc7ac\ud55c \uc0ac\ub78c\uc740?, context: \uc870\uc131\ubbfc\uc740 2002\ub144\ubd80\ud130 2004\ub144\uae4c\uc9c0 3\ub144\uc5d0 \uac78\uccd0 \ucd5c\uc9c4\uc2e4(\uac00\uc871)\uc744 \uc0c1\ub300\ub85c \uc815\uc870\uc758 \uc758\ubb34 \uc704\ubc18 \ud589\uc704, \ud3ed\ub825 \ud589\uc704, \uc0ac\uae30 \ud589\uc704, \uba85\uc608\ud6fc\uc190 \ud589\uc704, \ud5c8\uc704\uc0ac\uc2e4\uc720\ud3ec \ud589\uc704\ub97c \uc800\uc9c8\ub800\ub294\ub370, 2002\ub144 12\uc6d4 18\uc77c \uc624\ud6c4 2\uc2dc \ud604\uc7ac \uc815\uc870\uc758 \uc758\ubb34 \uc704\ubc18 \ud589\uc704, \ud3ed\ub825 \ud589\uc704, \uc0ac\uae30 \ud589\uc704\ub97c \uc774\ubbf8 \uc800\uc9c0\ub978 \uc0c1\ud0dc\uc600\uace0 \uadf8 \uac01\uac01\uc5d0 \ub300\ud574 \uc5ec\ub860 \uc870\uc791\uc744 \uac00\ub3d9\uc2dc\ucf1c \ud53c\ud574\uc790 \ucd5c\uc9c4\uc2e4\uc744 \uac00\ud574\uc790\ub85c \ubaa8\ub294 \uc5b8\ub860\ud50c\ub808\uc774\ub97c \ud588\ub2e4. \uc774 \uc5b8\ub860\ud50c\ub808\uc774\ub97c \ud1b5\ud574 \uba85\uc608\ud6fc\uc190 \ud589\uc704, \ud5c8\uc704\uc0ac\uc2e4\uc720\ud3ec \ud589\uc704\ub3c4 \ucd94\uac00\ub418\uc5c8\ub2e4. \uc878\uc9c0\uc5d0 \uac00\ud574\uc790\ub85c \ubab0\ub9b0 \ucd5c\uc9c4\uc2e4\uc740 \uc0ac\uc2e4\uad00\uacc4\ub97c \ubc14\ub85c\uc7a1\ub294 \uc5b8\ub860 \ub300\uc751\uc744 \ubc14\ub85c \ud588\ub2e4. \uae30\uc790\ub098 \ub300\uc911 \uc785\uc7a5\uc5d0\uc11c\ub294 \ub204\uad6c\uc758 \ub9d0\uc774 \ub9de\ub294\uc9c0 \uc54c \uc218 \uc5c6\uc744 \uac83\uc774\uae30\uc5d0 \uac1d\uad00\uc801 \ud310\ub2e8\uc744 \ub0b4\ub9b4 \uc218 \uc788\ub3c4\ub85d \uc774\ud63c\uc720\ucc45\uc8fc\uc758\ub77c\ub294 \uc900\uac70\ud2c0\uc744 \ud65c\uc6a9\ud558\uc600\ub294\ub370, \ucd5c\uc9c4\uc2e4\uc740 '\uc774\ud63c\ud560 \uc758\uc0ac\uac00 \uc804\ud600 \uc5c6\uc74c'\uc744 \ucc9c\uba85\ud568\uc73c\ub85c\uc368 \uc790\uc2e0\uc758 \ub9d0\uc774 \ud5c8\uc704\uc774\uba74 \uc870\uc131\ubbfc\uc5d0 \uc758\ud574 \uc790\uc2e0\uc774 \uc18c\uc1a1\uc774\ud63c\uc758 \ub300\uc0c1\uc774 \ub418\ub3c4\ub85d \ud310\uc744 \ub2e8\uc21c\ud654\uc2dc\ucf30\ub2e4. 2000\ub144 6\uc6d4 \ucd5c\uc9c4\uc2e4\uc740 \uc870\uc131\ubbfc\uc744 \ud3c9\uac00\ud558\ub294 \uae00\uc744 \uc2a4\ud3ec\uce20\uc11c\uc6b8\uc5d0 \uc5f0\uc7ac\ud55c \uc801\uc774 \uc788\uae30 \ub54c\ubb38\uc5d0 \uc870\uc131\ubbfc\uc758 \uc2e4\uccb4 \uaddc\uba85\uacfc \uad00\ub828\ud558\uc5ec \uacb0\uc790\ud574\uc9c0\ud560 \ud544\uc694\ub3c4 \uc788\ub294 \uc0c1\ud669\uc774\uc5c8\ub2e4."} {"question": "\uae40\ub300\uc911\uc744 \uac15\ud558\uac8c \ube44\ub09c\ud588\uc73c\ub098 \uc0ac\ud6c4\uc5d0\ub294 \uc628\uac74\ud55c \ucd94\ubaa8\uc758 \ubcc0\uc744 \ub0b4\ub193\uc740 \uc0ac\ub78c\uc740?", "paragraph": "\ud55c\ud3b8 \uc0dd\uc804\uc5d0 \uae40\ub300\uc911\uc744 \uac15\ud558\uac8c \ube44\ub09c\ud588\ub358 \uae40\ub3d9\uae38 \uc804 \uc5f0\uc138\ub300 \uad50\uc218\uac00 DJ \uc0ac\ud6c4 \uc787\ub2ec\uc544 \uc628\uac74\ud55c \ucd94\ubaa8\uc758 \ubcc0\uc744 \ub0b4\ub193\uc558\ub2e4. \uae40 \uc804 \uad50\uc218\ub294 21\uc77c \uc790\uc2e0\uc758 \ud648\ud398\uc774\uc9c0\uc5d0 '\uae40\ub300\uc911 \uc120\ubc30\ub2d8 \uc804\uc0c1\uc11c'\ub77c\ub294 \uc81c\ubaa9\uc758 \uae00\uc5d0\uc11c \"\uc800\uc2b9\uae38\uc744 \ub5a0\ub0a0 \ub54c \ubcc4\ub85c \uace0\uc0dd\ud558\uc9c0 \uc54a\uace0 \ud3b8\uc548\ud558\uac8c \uac00\ub294 \uac83\uc774 \uc778\uac04\uc774 \ub204\ub9b4 \uc218 \uc788\ub294 \ub2e4\uc12f \uac00\uc9c0 \ubcf5\uc911\uc5d0 \ud558\ub098\ub77c\uace0 \ub4e4\uc5c8\uc2b5\ub2c8\ub2e4\"\ub77c\ub294 \ub9d0\uc744 \uc2dc\uc791\uc73c\ub85c DJ \uc0ac\ub9dd\uc5d0 \ub300\ud55c \ub290\ub08c\uc744 \uc801\uc5b4\ub098\uac14\ub2e4. \uae40 \uc804 \uad50\uc218\ub294 \"\ub355\uc2a4\ub7ec\uc6b4 \uc0b6\uc774\uc5c8\uae30\uc5d0 \ud55c\ub098\ub77c\uc758 \ub300\ud1b5\ub839\uc790\ub9ac\uc5d0\ub3c4 \uc62c\ub790\uace0, \ub9cc\uc778\uc774 \ubd80\ub7ec\uc6cc\ud558\ub294 \ub178\ubca8\ud3c9\ud654\uc0c1 \uc218\uc0c1\uc790\ub3c4 \ub418\uc168\uc744 \uac83 \uc544\ub2d9\ub2c8\uae4c. \uc6b0\ub9ac\ub098\ub77c\uc758 \uc804\uc9c1 \ud604\uc9c1 \ub300\ud1b5\ub839\ub4e4\uc774 \ubaa8\ub450 \ubcd1\ubb38\uc548 \uac00\uc11c \ucf8c\uc720\ub97c \ube4c\uc5c8\ub2e4\ub294 \uac83\ub3c4 \uc0ac\ub78c \uc0ac\ub294 \uc138\uc0c1\uc5d0 \ud754\ud55c \uc77c\uc740 \uc544\ub2c8\ub77c\uace0 \uc0dd\uac01\ud569\ub2c8\ub2e4\"\ub77c\uba70 \uae40\ub300\uc911\uc758 \uc0b6\uc744 \uae30\ub838\ub2e4. \"\uc11c\uac70\uc758 \uc18c\uc2dd\uc774 \uc804\ud574\uc9c0\uc790\ub9c8\uc790 \ubbf8\uad6d\uc758 \ub300\ud1b5\ub839\uc744 \ube44\ub86f\ud558\uc5ec \uc601\uad6d \uc218\uc0c1, \ub3c5\uc77c\uc218\uc0c1\uc774 \uc55e\uc744 \ub2e4\ud22c\ub294 \ub4ef \uba3c\uc800 \uc560\ub3c4\uc758 \ub73b\uc744 \ud45c\ud558\uc600\uace0, \uc774\uc6c3\ub098\ub77c \uc77c\ubcf8\uc758 \uad6d\uc601 \ubc29\uc1a1 NHK\ub294 \ub2e4\ub978 \ud504\ub85c\uadf8\ub7a8\uc744 \uc911\ub2e8\ud558\uace0 \uc120\ubc30\ub2d8\uc758 \uc11c\uac70 \uc18c\uc2dd\uc744 \uad6d\ubbfc\uc5d0\uac8c \uc54c\ub838\ub2e4\uace0 \ub4e4\uc5c8\uc2b5\ub2c8\ub2e4\"\uba74\uc11c \uc138\uacc4\uc801 \uba85\uc131\uc744 \uc5bb\uc740 DJ\uc5d0 \ub300\ud55c \uc874\uacbd\uc758 \ub9c8\uc74c\ub3c4 \ud45c\ud604\ud588\ub2e4. \uac1c\uc778\uc801\uc778 \uc778\uc5f0\uc744 \uac15\ub3c4\ud558\uba74\uc11c \uc560\ub3c4\ub97c \ub098\ud0c0\ub0b4\uae30\ub3c4 \ud588\ub2e4. \uae40 \uc804 \uad50\uc218\ub294 \"\uad70\uc0ac\uc815\uad8c\ud558\uc5d0\uc11c \uc5ec\ub7ec \ub3d9\uc9c0\ub4e4\uacfc \ud568\uaed8 \ubbfc\uc8fc\ud654\uc758 \ud22c\uc7c1\uc744 \ud558\ub358 \ub54c\uc5d0\ub294 (DJ\uac00)\ubd88\ub7ec\uc11c \uc810\uc2ec \ub610\ub294 \uc800\ub141\uc744 \uc0ac\uc8fc\uc2e0 \uc801\ub3c4 \uc788\uace0 \uc800\ub3c4 \ud6c4\ubc30\ub85c\uc368\uc758 \ub3c4\ub9ac\ub97c \ub2e4\ud558\ub178\ub77c\uace0 \ucd5c\uc120\uc744 \ub2e4 \ud55c \uac83\ub3c4 \uc0ac\uc2e4\"\uc774\ub77c\uba70 \"\uadf8 \ub54c(\uc8fd\uc740 \uc774\ud6c4)\uc5d0\ub294 \ub2e4\uc2dc \ub9cc\ub098\uac8c \ub420 \uac83\uc73c\ub85c \ubbff\uace0 \uc2f6\uc2b5\ub2c8\ub2e4. \uc544\ubb34\ub834 \uae4a\uc740 \uc778\uc5f0\uc744 \uac00\uc84c\uac70\ub298, \uc601\uc601 \ub2e4\uc2dc\ub294 \ub9cc\ub0a0 \uc218 \uc5c6\ub2e4\uace0 \uc0dd\uac01\ud558\uc9c0 \uc54a\uc2b5\ub2c8\ub2e4\"\ub77c\uace0 \ubc1d\ud614\ub2e4. \uadf8\ub294 \ub9c8\uc9c0\ub9c9\uc73c\ub85c \"\uc5b8\uc820\uac00\ub294 \ub2e4\uc2dc \ub9cc\ub098\uc11c \ud749\uae08\uc744 \ud130\ub193\uace0 \ub530\uc838\uc57c \ud560 \uc77c\ub3c4 \uc788\uc2b5\ub2c8\ub2e4\ub9c8\ub294 \uc624\ub298\uc740 \uadf8\ub7f0 \ub9c8\uc74c\uc774 \ub418\uc9c0\uac00 \uc54a\uc2b5\ub2c8\ub2e4. \uc120\ubc30\ub2d8\uc740 \ud1b5\uc77c\ub41c \uc870\uad6d\uc758 \ucd08\ub300 \ub300\ud1b5\ub839\uc774 \ub418\uace0 \uc2f6\uc73c\uc168\uc744 \ud150\ub370, \uadf8 \ub73b\uc744 \uc774\ub8e8\uc9c0 \ubabb\ud574 \uc720\uac10\uc774\uc2dc\uaca0\uc2b5\ub2c8\ub2e4\"\ub77c\uba70 \uae00\uc744 \ub05d\ub9fa\uc5c8\ub2e4. \uadf8\ub294 \uc774\uc5b4 22\uc77c\uc5d0\ub294 '\uae40\ub300\uc911 \ud615\ub2d8\uaed8'\ub77c\ub294 \ub354\uc6b1 \uc0b4\uac00\uc6b4 \uc81c\ubaa9\uc758 \uae00\uc744 \uc62c\ub838\ub2e4. \uae40 \uc804 \uad50\uc218\ub294 \"\ud55c \uc2dc\ub300\uc758 \ud48d\uc6b4\uc544 \uae40\ub300\uc911 \ud615\uc744 '\ud615\ub2d8'\uc774\ub77c\uace0 \ubd80\ub974\uace0 \uc2f6\uc740 \uc0ac\ub78c\uc740 \ubd80\uc9c0\uae30\uc218\uc774\uc5c8\uc744 \uac83\uc785\ub2c8\ub2e4. \uc800\ub3c4 \uadf8\ub7f0 \uc790\ub4e4 \uc911\uc758 \ud55c \uc0ac\ub78c\uc774 \ub418\uc5b4 \uadf8\ub807\uac8c \ubd88\ub7ec\ubcf4\uace0 \uc2f6\uc740 \uac83\ubfd0\uc774\ub2c8 \ub108\uadf8\ub7fd\uac8c \ubc1b\uc544 \uc8fc\uc2dc\uae30\ub97c \ubc14\ub78d\ub2c8\ub2e4\"\ub77c\uace0 \ub9d0\ud588\ub2e4. \uc55e\uc11c \uadf8\ub294 \uae40 \uc804 \ub300\ud1b5\ub839\uc758 \uc0ac\ub9dd \uc9c1\ud6c4\uc778 19\uc77c \"\uae40\ub300\uc911 \uc804 \ub300\ud1b5\ub839\uc758 \uc11c\uac70\ub294 \uc6b0\ub9ac \ubaa8\ub450\uc5d0\uac8c \ucc29\uc7a1\ud55c \uc2ec\uc815\uacfc \uc778\uc0dd\uc758 \ubb34\uc0c1\ud568\uc744 \uc808\uc2e4\ud788 \ub290\ub07c\uac8c \ud55c\ub2e4\"\uba70 \"\ud30c\ub780\ub9cc\uc7a5\ud55c \uc0b6\uc744 \uc0b4\uace0 \uc774\uc81c \ud3c9\ud654\ub86d\uac8c \uadf8 \uc0dd\uc774 \ub9c9\uc744 \ub0b4\ub838\uc73c\ub2c8 \ub2f9\uc7a5\uc5d0 \ud560 \ub9d0\uc744 \ucc3e\uae30 \uc5b4\ub835\ub2e4\"\uace0 \ub9d0\ud588\ub2e4.", "answer": "\uae40\ub3d9\uae38", "sentence": "\ud55c\ud3b8 \uc0dd\uc804\uc5d0 \uae40\ub300\uc911\uc744 \uac15\ud558\uac8c \ube44\ub09c\ud588\ub358 \uae40\ub3d9\uae38 \uc804 \uc5f0\uc138\ub300 \uad50\uc218\uac00 DJ \uc0ac\ud6c4 \uc787\ub2ec\uc544 \uc628\uac74\ud55c \ucd94\ubaa8\uc758 \ubcc0\uc744 \ub0b4\ub193\uc558\ub2e4.", "paragraph_sentence": " \ud55c\ud3b8 \uc0dd\uc804\uc5d0 \uae40\ub300\uc911\uc744 \uac15\ud558\uac8c \ube44\ub09c\ud588\ub358 \uae40\ub3d9\uae38 \uc804 \uc5f0\uc138\ub300 \uad50\uc218\uac00 DJ \uc0ac\ud6c4 \uc787\ub2ec\uc544 \uc628\uac74\ud55c \ucd94\ubaa8\uc758 \ubcc0\uc744 \ub0b4\ub193\uc558\ub2e4. \uae40 \uc804 \uad50\uc218\ub294 21\uc77c \uc790\uc2e0\uc758 \ud648\ud398\uc774\uc9c0\uc5d0 '\uae40\ub300\uc911 \uc120\ubc30\ub2d8 \uc804\uc0c1\uc11c'\ub77c\ub294 \uc81c\ubaa9\uc758 \uae00\uc5d0\uc11c \"\uc800\uc2b9\uae38\uc744 \ub5a0\ub0a0 \ub54c \ubcc4\ub85c \uace0\uc0dd\ud558\uc9c0 \uc54a\uace0 \ud3b8\uc548\ud558\uac8c \uac00\ub294 \uac83\uc774 \uc778\uac04\uc774 \ub204\ub9b4 \uc218 \uc788\ub294 \ub2e4\uc12f \uac00\uc9c0 \ubcf5\uc911\uc5d0 \ud558\ub098\ub77c\uace0 \ub4e4\uc5c8\uc2b5\ub2c8\ub2e4\"\ub77c\ub294 \ub9d0\uc744 \uc2dc\uc791\uc73c\ub85c DJ \uc0ac\ub9dd\uc5d0 \ub300\ud55c \ub290\ub08c\uc744 \uc801\uc5b4\ub098\uac14\ub2e4. \uae40 \uc804 \uad50\uc218\ub294 \"\ub355\uc2a4\ub7ec\uc6b4 \uc0b6\uc774\uc5c8\uae30\uc5d0 \ud55c\ub098\ub77c\uc758 \ub300\ud1b5\ub839\uc790\ub9ac\uc5d0\ub3c4 \uc62c\ub790\uace0, \ub9cc\uc778\uc774 \ubd80\ub7ec\uc6cc\ud558\ub294 \ub178\ubca8\ud3c9\ud654\uc0c1 \uc218\uc0c1\uc790\ub3c4 \ub418\uc168\uc744 \uac83 \uc544\ub2d9\ub2c8\uae4c. \uc6b0\ub9ac\ub098\ub77c\uc758 \uc804\uc9c1 \ud604\uc9c1 \ub300\ud1b5\ub839\ub4e4\uc774 \ubaa8\ub450 \ubcd1\ubb38\uc548 \uac00\uc11c \ucf8c\uc720\ub97c \ube4c\uc5c8\ub2e4\ub294 \uac83\ub3c4 \uc0ac\ub78c \uc0ac\ub294 \uc138\uc0c1\uc5d0 \ud754\ud55c \uc77c\uc740 \uc544\ub2c8\ub77c\uace0 \uc0dd\uac01\ud569\ub2c8\ub2e4\"\ub77c\uba70 \uae40\ub300\uc911\uc758 \uc0b6\uc744 \uae30\ub838\ub2e4. \"\uc11c\uac70\uc758 \uc18c\uc2dd\uc774 \uc804\ud574\uc9c0\uc790\ub9c8\uc790 \ubbf8\uad6d\uc758 \ub300\ud1b5\ub839\uc744 \ube44\ub86f\ud558\uc5ec \uc601\uad6d \uc218\uc0c1, \ub3c5\uc77c\uc218\uc0c1\uc774 \uc55e\uc744 \ub2e4\ud22c\ub294 \ub4ef \uba3c\uc800 \uc560\ub3c4\uc758 \ub73b\uc744 \ud45c\ud558\uc600\uace0, \uc774\uc6c3\ub098\ub77c \uc77c\ubcf8\uc758 \uad6d\uc601 \ubc29\uc1a1 NHK\ub294 \ub2e4\ub978 \ud504\ub85c\uadf8\ub7a8\uc744 \uc911\ub2e8\ud558\uace0 \uc120\ubc30\ub2d8\uc758 \uc11c\uac70 \uc18c\uc2dd\uc744 \uad6d\ubbfc\uc5d0\uac8c \uc54c\ub838\ub2e4\uace0 \ub4e4\uc5c8\uc2b5\ub2c8\ub2e4\"\uba74\uc11c \uc138\uacc4\uc801 \uba85\uc131\uc744 \uc5bb\uc740 DJ\uc5d0 \ub300\ud55c \uc874\uacbd\uc758 \ub9c8\uc74c\ub3c4 \ud45c\ud604\ud588\ub2e4. \uac1c\uc778\uc801\uc778 \uc778\uc5f0\uc744 \uac15\ub3c4\ud558\uba74\uc11c \uc560\ub3c4\ub97c \ub098\ud0c0\ub0b4\uae30\ub3c4 \ud588\ub2e4. \uae40 \uc804 \uad50\uc218\ub294 \"\uad70\uc0ac\uc815\uad8c\ud558\uc5d0\uc11c \uc5ec\ub7ec \ub3d9\uc9c0\ub4e4\uacfc \ud568\uaed8 \ubbfc\uc8fc\ud654\uc758 \ud22c\uc7c1\uc744 \ud558\ub358 \ub54c\uc5d0\ub294 (DJ\uac00)\ubd88\ub7ec\uc11c \uc810\uc2ec \ub610\ub294 \uc800\ub141\uc744 \uc0ac\uc8fc\uc2e0 \uc801\ub3c4 \uc788\uace0 \uc800\ub3c4 \ud6c4\ubc30\ub85c\uc368\uc758 \ub3c4\ub9ac\ub97c \ub2e4\ud558\ub178\ub77c\uace0 \ucd5c\uc120\uc744 \ub2e4 \ud55c \uac83\ub3c4 \uc0ac\uc2e4\"\uc774\ub77c\uba70 \"\uadf8 \ub54c(\uc8fd\uc740 \uc774\ud6c4)\uc5d0\ub294 \ub2e4\uc2dc \ub9cc\ub098\uac8c \ub420 \uac83\uc73c\ub85c \ubbff\uace0 \uc2f6\uc2b5\ub2c8\ub2e4. \uc544\ubb34\ub834 \uae4a\uc740 \uc778\uc5f0\uc744 \uac00\uc84c\uac70\ub298, \uc601\uc601 \ub2e4\uc2dc\ub294 \ub9cc\ub0a0 \uc218 \uc5c6\ub2e4\uace0 \uc0dd\uac01\ud558\uc9c0 \uc54a\uc2b5\ub2c8\ub2e4\"\ub77c\uace0 \ubc1d\ud614\ub2e4. \uadf8\ub294 \ub9c8\uc9c0\ub9c9\uc73c\ub85c \"\uc5b8\uc820\uac00\ub294 \ub2e4\uc2dc \ub9cc\ub098\uc11c \ud749\uae08\uc744 \ud130\ub193\uace0 \ub530\uc838\uc57c \ud560 \uc77c\ub3c4 \uc788\uc2b5\ub2c8\ub2e4\ub9c8\ub294 \uc624\ub298\uc740 \uadf8\ub7f0 \ub9c8\uc74c\uc774 \ub418\uc9c0\uac00 \uc54a\uc2b5\ub2c8\ub2e4. \uc120\ubc30\ub2d8\uc740 \ud1b5\uc77c\ub41c \uc870\uad6d\uc758 \ucd08\ub300 \ub300\ud1b5\ub839\uc774 \ub418\uace0 \uc2f6\uc73c\uc168\uc744 \ud150\ub370, \uadf8 \ub73b\uc744 \uc774\ub8e8\uc9c0 \ubabb\ud574 \uc720\uac10\uc774\uc2dc\uaca0\uc2b5\ub2c8\ub2e4\"\ub77c\uba70 \uae00\uc744 \ub05d\ub9fa\uc5c8\ub2e4. \uadf8\ub294 \uc774\uc5b4 22\uc77c\uc5d0\ub294 '\uae40\ub300\uc911 \ud615\ub2d8\uaed8'\ub77c\ub294 \ub354\uc6b1 \uc0b4\uac00\uc6b4 \uc81c\ubaa9\uc758 \uae00\uc744 \uc62c\ub838\ub2e4. \uae40 \uc804 \uad50\uc218\ub294 \"\ud55c \uc2dc\ub300\uc758 \ud48d\uc6b4\uc544 \uae40\ub300\uc911 \ud615\uc744 '\ud615\ub2d8'\uc774\ub77c\uace0 \ubd80\ub974\uace0 \uc2f6\uc740 \uc0ac\ub78c\uc740 \ubd80\uc9c0\uae30\uc218\uc774\uc5c8\uc744 \uac83\uc785\ub2c8\ub2e4. \uc800\ub3c4 \uadf8\ub7f0 \uc790\ub4e4 \uc911\uc758 \ud55c \uc0ac\ub78c\uc774 \ub418\uc5b4 \uadf8\ub807\uac8c \ubd88\ub7ec\ubcf4\uace0 \uc2f6\uc740 \uac83\ubfd0\uc774\ub2c8 \ub108\uadf8\ub7fd\uac8c \ubc1b\uc544 \uc8fc\uc2dc\uae30\ub97c \ubc14\ub78d\ub2c8\ub2e4\"\ub77c\uace0 \ub9d0\ud588\ub2e4. \uc55e\uc11c \uadf8\ub294 \uae40 \uc804 \ub300\ud1b5\ub839\uc758 \uc0ac\ub9dd \uc9c1\ud6c4\uc778 19\uc77c \"\uae40\ub300\uc911 \uc804 \ub300\ud1b5\ub839\uc758 \uc11c\uac70\ub294 \uc6b0\ub9ac \ubaa8\ub450\uc5d0\uac8c \ucc29\uc7a1\ud55c \uc2ec\uc815\uacfc \uc778\uc0dd\uc758 \ubb34\uc0c1\ud568\uc744 \uc808\uc2e4\ud788 \ub290\ub07c\uac8c \ud55c\ub2e4\"\uba70 \"\ud30c\ub780\ub9cc\uc7a5\ud55c \uc0b6\uc744 \uc0b4\uace0 \uc774\uc81c \ud3c9\ud654\ub86d\uac8c \uadf8 \uc0dd\uc774 \ub9c9\uc744 \ub0b4\ub838\uc73c\ub2c8 \ub2f9\uc7a5\uc5d0 \ud560 \ub9d0\uc744 \ucc3e\uae30 \uc5b4\ub835\ub2e4\"\uace0 \ub9d0\ud588\ub2e4.", "paragraph_answer": "\ud55c\ud3b8 \uc0dd\uc804\uc5d0 \uae40\ub300\uc911\uc744 \uac15\ud558\uac8c \ube44\ub09c\ud588\ub358 \uae40\ub3d9\uae38 \uc804 \uc5f0\uc138\ub300 \uad50\uc218\uac00 DJ \uc0ac\ud6c4 \uc787\ub2ec\uc544 \uc628\uac74\ud55c \ucd94\ubaa8\uc758 \ubcc0\uc744 \ub0b4\ub193\uc558\ub2e4. \uae40 \uc804 \uad50\uc218\ub294 21\uc77c \uc790\uc2e0\uc758 \ud648\ud398\uc774\uc9c0\uc5d0 '\uae40\ub300\uc911 \uc120\ubc30\ub2d8 \uc804\uc0c1\uc11c'\ub77c\ub294 \uc81c\ubaa9\uc758 \uae00\uc5d0\uc11c \"\uc800\uc2b9\uae38\uc744 \ub5a0\ub0a0 \ub54c \ubcc4\ub85c \uace0\uc0dd\ud558\uc9c0 \uc54a\uace0 \ud3b8\uc548\ud558\uac8c \uac00\ub294 \uac83\uc774 \uc778\uac04\uc774 \ub204\ub9b4 \uc218 \uc788\ub294 \ub2e4\uc12f \uac00\uc9c0 \ubcf5\uc911\uc5d0 \ud558\ub098\ub77c\uace0 \ub4e4\uc5c8\uc2b5\ub2c8\ub2e4\"\ub77c\ub294 \ub9d0\uc744 \uc2dc\uc791\uc73c\ub85c DJ \uc0ac\ub9dd\uc5d0 \ub300\ud55c \ub290\ub08c\uc744 \uc801\uc5b4\ub098\uac14\ub2e4. \uae40 \uc804 \uad50\uc218\ub294 \"\ub355\uc2a4\ub7ec\uc6b4 \uc0b6\uc774\uc5c8\uae30\uc5d0 \ud55c\ub098\ub77c\uc758 \ub300\ud1b5\ub839\uc790\ub9ac\uc5d0\ub3c4 \uc62c\ub790\uace0, \ub9cc\uc778\uc774 \ubd80\ub7ec\uc6cc\ud558\ub294 \ub178\ubca8\ud3c9\ud654\uc0c1 \uc218\uc0c1\uc790\ub3c4 \ub418\uc168\uc744 \uac83 \uc544\ub2d9\ub2c8\uae4c. \uc6b0\ub9ac\ub098\ub77c\uc758 \uc804\uc9c1 \ud604\uc9c1 \ub300\ud1b5\ub839\ub4e4\uc774 \ubaa8\ub450 \ubcd1\ubb38\uc548 \uac00\uc11c \ucf8c\uc720\ub97c \ube4c\uc5c8\ub2e4\ub294 \uac83\ub3c4 \uc0ac\ub78c \uc0ac\ub294 \uc138\uc0c1\uc5d0 \ud754\ud55c \uc77c\uc740 \uc544\ub2c8\ub77c\uace0 \uc0dd\uac01\ud569\ub2c8\ub2e4\"\ub77c\uba70 \uae40\ub300\uc911\uc758 \uc0b6\uc744 \uae30\ub838\ub2e4. \"\uc11c\uac70\uc758 \uc18c\uc2dd\uc774 \uc804\ud574\uc9c0\uc790\ub9c8\uc790 \ubbf8\uad6d\uc758 \ub300\ud1b5\ub839\uc744 \ube44\ub86f\ud558\uc5ec \uc601\uad6d \uc218\uc0c1, \ub3c5\uc77c\uc218\uc0c1\uc774 \uc55e\uc744 \ub2e4\ud22c\ub294 \ub4ef \uba3c\uc800 \uc560\ub3c4\uc758 \ub73b\uc744 \ud45c\ud558\uc600\uace0, \uc774\uc6c3\ub098\ub77c \uc77c\ubcf8\uc758 \uad6d\uc601 \ubc29\uc1a1 NHK\ub294 \ub2e4\ub978 \ud504\ub85c\uadf8\ub7a8\uc744 \uc911\ub2e8\ud558\uace0 \uc120\ubc30\ub2d8\uc758 \uc11c\uac70 \uc18c\uc2dd\uc744 \uad6d\ubbfc\uc5d0\uac8c \uc54c\ub838\ub2e4\uace0 \ub4e4\uc5c8\uc2b5\ub2c8\ub2e4\"\uba74\uc11c \uc138\uacc4\uc801 \uba85\uc131\uc744 \uc5bb\uc740 DJ\uc5d0 \ub300\ud55c \uc874\uacbd\uc758 \ub9c8\uc74c\ub3c4 \ud45c\ud604\ud588\ub2e4. \uac1c\uc778\uc801\uc778 \uc778\uc5f0\uc744 \uac15\ub3c4\ud558\uba74\uc11c \uc560\ub3c4\ub97c \ub098\ud0c0\ub0b4\uae30\ub3c4 \ud588\ub2e4. \uae40 \uc804 \uad50\uc218\ub294 \"\uad70\uc0ac\uc815\uad8c\ud558\uc5d0\uc11c \uc5ec\ub7ec \ub3d9\uc9c0\ub4e4\uacfc \ud568\uaed8 \ubbfc\uc8fc\ud654\uc758 \ud22c\uc7c1\uc744 \ud558\ub358 \ub54c\uc5d0\ub294 (DJ\uac00)\ubd88\ub7ec\uc11c \uc810\uc2ec \ub610\ub294 \uc800\ub141\uc744 \uc0ac\uc8fc\uc2e0 \uc801\ub3c4 \uc788\uace0 \uc800\ub3c4 \ud6c4\ubc30\ub85c\uc368\uc758 \ub3c4\ub9ac\ub97c \ub2e4\ud558\ub178\ub77c\uace0 \ucd5c\uc120\uc744 \ub2e4 \ud55c \uac83\ub3c4 \uc0ac\uc2e4\"\uc774\ub77c\uba70 \"\uadf8 \ub54c(\uc8fd\uc740 \uc774\ud6c4)\uc5d0\ub294 \ub2e4\uc2dc \ub9cc\ub098\uac8c \ub420 \uac83\uc73c\ub85c \ubbff\uace0 \uc2f6\uc2b5\ub2c8\ub2e4. \uc544\ubb34\ub834 \uae4a\uc740 \uc778\uc5f0\uc744 \uac00\uc84c\uac70\ub298, \uc601\uc601 \ub2e4\uc2dc\ub294 \ub9cc\ub0a0 \uc218 \uc5c6\ub2e4\uace0 \uc0dd\uac01\ud558\uc9c0 \uc54a\uc2b5\ub2c8\ub2e4\"\ub77c\uace0 \ubc1d\ud614\ub2e4. \uadf8\ub294 \ub9c8\uc9c0\ub9c9\uc73c\ub85c \"\uc5b8\uc820\uac00\ub294 \ub2e4\uc2dc \ub9cc\ub098\uc11c \ud749\uae08\uc744 \ud130\ub193\uace0 \ub530\uc838\uc57c \ud560 \uc77c\ub3c4 \uc788\uc2b5\ub2c8\ub2e4\ub9c8\ub294 \uc624\ub298\uc740 \uadf8\ub7f0 \ub9c8\uc74c\uc774 \ub418\uc9c0\uac00 \uc54a\uc2b5\ub2c8\ub2e4. \uc120\ubc30\ub2d8\uc740 \ud1b5\uc77c\ub41c \uc870\uad6d\uc758 \ucd08\ub300 \ub300\ud1b5\ub839\uc774 \ub418\uace0 \uc2f6\uc73c\uc168\uc744 \ud150\ub370, \uadf8 \ub73b\uc744 \uc774\ub8e8\uc9c0 \ubabb\ud574 \uc720\uac10\uc774\uc2dc\uaca0\uc2b5\ub2c8\ub2e4\"\ub77c\uba70 \uae00\uc744 \ub05d\ub9fa\uc5c8\ub2e4. \uadf8\ub294 \uc774\uc5b4 22\uc77c\uc5d0\ub294 '\uae40\ub300\uc911 \ud615\ub2d8\uaed8'\ub77c\ub294 \ub354\uc6b1 \uc0b4\uac00\uc6b4 \uc81c\ubaa9\uc758 \uae00\uc744 \uc62c\ub838\ub2e4. \uae40 \uc804 \uad50\uc218\ub294 \"\ud55c \uc2dc\ub300\uc758 \ud48d\uc6b4\uc544 \uae40\ub300\uc911 \ud615\uc744 '\ud615\ub2d8'\uc774\ub77c\uace0 \ubd80\ub974\uace0 \uc2f6\uc740 \uc0ac\ub78c\uc740 \ubd80\uc9c0\uae30\uc218\uc774\uc5c8\uc744 \uac83\uc785\ub2c8\ub2e4. \uc800\ub3c4 \uadf8\ub7f0 \uc790\ub4e4 \uc911\uc758 \ud55c \uc0ac\ub78c\uc774 \ub418\uc5b4 \uadf8\ub807\uac8c \ubd88\ub7ec\ubcf4\uace0 \uc2f6\uc740 \uac83\ubfd0\uc774\ub2c8 \ub108\uadf8\ub7fd\uac8c \ubc1b\uc544 \uc8fc\uc2dc\uae30\ub97c \ubc14\ub78d\ub2c8\ub2e4\"\ub77c\uace0 \ub9d0\ud588\ub2e4. \uc55e\uc11c \uadf8\ub294 \uae40 \uc804 \ub300\ud1b5\ub839\uc758 \uc0ac\ub9dd \uc9c1\ud6c4\uc778 19\uc77c \"\uae40\ub300\uc911 \uc804 \ub300\ud1b5\ub839\uc758 \uc11c\uac70\ub294 \uc6b0\ub9ac \ubaa8\ub450\uc5d0\uac8c \ucc29\uc7a1\ud55c \uc2ec\uc815\uacfc \uc778\uc0dd\uc758 \ubb34\uc0c1\ud568\uc744 \uc808\uc2e4\ud788 \ub290\ub07c\uac8c \ud55c\ub2e4\"\uba70 \"\ud30c\ub780\ub9cc\uc7a5\ud55c \uc0b6\uc744 \uc0b4\uace0 \uc774\uc81c \ud3c9\ud654\ub86d\uac8c \uadf8 \uc0dd\uc774 \ub9c9\uc744 \ub0b4\ub838\uc73c\ub2c8 \ub2f9\uc7a5\uc5d0 \ud560 \ub9d0\uc744 \ucc3e\uae30 \uc5b4\ub835\ub2e4\"\uace0 \ub9d0\ud588\ub2e4.", "sentence_answer": "\ud55c\ud3b8 \uc0dd\uc804\uc5d0 \uae40\ub300\uc911\uc744 \uac15\ud558\uac8c \ube44\ub09c\ud588\ub358 \uae40\ub3d9\uae38 \uc804 \uc5f0\uc138\ub300 \uad50\uc218\uac00 DJ \uc0ac\ud6c4 \uc787\ub2ec\uc544 \uc628\uac74\ud55c \ucd94\ubaa8\uc758 \ubcc0\uc744 \ub0b4\ub193\uc558\ub2e4.", "paragraph_id": "6576830-50-0", "paragraph_question": "question: \uae40\ub300\uc911\uc744 \uac15\ud558\uac8c \ube44\ub09c\ud588\uc73c\ub098 \uc0ac\ud6c4\uc5d0\ub294 \uc628\uac74\ud55c \ucd94\ubaa8\uc758 \ubcc0\uc744 \ub0b4\ub193\uc740 \uc0ac\ub78c\uc740?, context: \ud55c\ud3b8 \uc0dd\uc804\uc5d0 \uae40\ub300\uc911\uc744 \uac15\ud558\uac8c \ube44\ub09c\ud588\ub358 \uae40\ub3d9\uae38 \uc804 \uc5f0\uc138\ub300 \uad50\uc218\uac00 DJ \uc0ac\ud6c4 \uc787\ub2ec\uc544 \uc628\uac74\ud55c \ucd94\ubaa8\uc758 \ubcc0\uc744 \ub0b4\ub193\uc558\ub2e4. \uae40 \uc804 \uad50\uc218\ub294 21\uc77c \uc790\uc2e0\uc758 \ud648\ud398\uc774\uc9c0\uc5d0 '\uae40\ub300\uc911 \uc120\ubc30\ub2d8 \uc804\uc0c1\uc11c'\ub77c\ub294 \uc81c\ubaa9\uc758 \uae00\uc5d0\uc11c \"\uc800\uc2b9\uae38\uc744 \ub5a0\ub0a0 \ub54c \ubcc4\ub85c \uace0\uc0dd\ud558\uc9c0 \uc54a\uace0 \ud3b8\uc548\ud558\uac8c \uac00\ub294 \uac83\uc774 \uc778\uac04\uc774 \ub204\ub9b4 \uc218 \uc788\ub294 \ub2e4\uc12f \uac00\uc9c0 \ubcf5\uc911\uc5d0 \ud558\ub098\ub77c\uace0 \ub4e4\uc5c8\uc2b5\ub2c8\ub2e4\"\ub77c\ub294 \ub9d0\uc744 \uc2dc\uc791\uc73c\ub85c DJ \uc0ac\ub9dd\uc5d0 \ub300\ud55c \ub290\ub08c\uc744 \uc801\uc5b4\ub098\uac14\ub2e4. \uae40 \uc804 \uad50\uc218\ub294 \"\ub355\uc2a4\ub7ec\uc6b4 \uc0b6\uc774\uc5c8\uae30\uc5d0 \ud55c\ub098\ub77c\uc758 \ub300\ud1b5\ub839\uc790\ub9ac\uc5d0\ub3c4 \uc62c\ub790\uace0, \ub9cc\uc778\uc774 \ubd80\ub7ec\uc6cc\ud558\ub294 \ub178\ubca8\ud3c9\ud654\uc0c1 \uc218\uc0c1\uc790\ub3c4 \ub418\uc168\uc744 \uac83 \uc544\ub2d9\ub2c8\uae4c. \uc6b0\ub9ac\ub098\ub77c\uc758 \uc804\uc9c1 \ud604\uc9c1 \ub300\ud1b5\ub839\ub4e4\uc774 \ubaa8\ub450 \ubcd1\ubb38\uc548 \uac00\uc11c \ucf8c\uc720\ub97c \ube4c\uc5c8\ub2e4\ub294 \uac83\ub3c4 \uc0ac\ub78c \uc0ac\ub294 \uc138\uc0c1\uc5d0 \ud754\ud55c \uc77c\uc740 \uc544\ub2c8\ub77c\uace0 \uc0dd\uac01\ud569\ub2c8\ub2e4\"\ub77c\uba70 \uae40\ub300\uc911\uc758 \uc0b6\uc744 \uae30\ub838\ub2e4. \"\uc11c\uac70\uc758 \uc18c\uc2dd\uc774 \uc804\ud574\uc9c0\uc790\ub9c8\uc790 \ubbf8\uad6d\uc758 \ub300\ud1b5\ub839\uc744 \ube44\ub86f\ud558\uc5ec \uc601\uad6d \uc218\uc0c1, \ub3c5\uc77c\uc218\uc0c1\uc774 \uc55e\uc744 \ub2e4\ud22c\ub294 \ub4ef \uba3c\uc800 \uc560\ub3c4\uc758 \ub73b\uc744 \ud45c\ud558\uc600\uace0, \uc774\uc6c3\ub098\ub77c \uc77c\ubcf8\uc758 \uad6d\uc601 \ubc29\uc1a1 NHK\ub294 \ub2e4\ub978 \ud504\ub85c\uadf8\ub7a8\uc744 \uc911\ub2e8\ud558\uace0 \uc120\ubc30\ub2d8\uc758 \uc11c\uac70 \uc18c\uc2dd\uc744 \uad6d\ubbfc\uc5d0\uac8c \uc54c\ub838\ub2e4\uace0 \ub4e4\uc5c8\uc2b5\ub2c8\ub2e4\"\uba74\uc11c \uc138\uacc4\uc801 \uba85\uc131\uc744 \uc5bb\uc740 DJ\uc5d0 \ub300\ud55c \uc874\uacbd\uc758 \ub9c8\uc74c\ub3c4 \ud45c\ud604\ud588\ub2e4. \uac1c\uc778\uc801\uc778 \uc778\uc5f0\uc744 \uac15\ub3c4\ud558\uba74\uc11c \uc560\ub3c4\ub97c \ub098\ud0c0\ub0b4\uae30\ub3c4 \ud588\ub2e4. \uae40 \uc804 \uad50\uc218\ub294 \"\uad70\uc0ac\uc815\uad8c\ud558\uc5d0\uc11c \uc5ec\ub7ec \ub3d9\uc9c0\ub4e4\uacfc \ud568\uaed8 \ubbfc\uc8fc\ud654\uc758 \ud22c\uc7c1\uc744 \ud558\ub358 \ub54c\uc5d0\ub294 (DJ\uac00)\ubd88\ub7ec\uc11c \uc810\uc2ec \ub610\ub294 \uc800\ub141\uc744 \uc0ac\uc8fc\uc2e0 \uc801\ub3c4 \uc788\uace0 \uc800\ub3c4 \ud6c4\ubc30\ub85c\uc368\uc758 \ub3c4\ub9ac\ub97c \ub2e4\ud558\ub178\ub77c\uace0 \ucd5c\uc120\uc744 \ub2e4 \ud55c \uac83\ub3c4 \uc0ac\uc2e4\"\uc774\ub77c\uba70 \"\uadf8 \ub54c(\uc8fd\uc740 \uc774\ud6c4)\uc5d0\ub294 \ub2e4\uc2dc \ub9cc\ub098\uac8c \ub420 \uac83\uc73c\ub85c \ubbff\uace0 \uc2f6\uc2b5\ub2c8\ub2e4. \uc544\ubb34\ub834 \uae4a\uc740 \uc778\uc5f0\uc744 \uac00\uc84c\uac70\ub298, \uc601\uc601 \ub2e4\uc2dc\ub294 \ub9cc\ub0a0 \uc218 \uc5c6\ub2e4\uace0 \uc0dd\uac01\ud558\uc9c0 \uc54a\uc2b5\ub2c8\ub2e4\"\ub77c\uace0 \ubc1d\ud614\ub2e4. \uadf8\ub294 \ub9c8\uc9c0\ub9c9\uc73c\ub85c \"\uc5b8\uc820\uac00\ub294 \ub2e4\uc2dc \ub9cc\ub098\uc11c \ud749\uae08\uc744 \ud130\ub193\uace0 \ub530\uc838\uc57c \ud560 \uc77c\ub3c4 \uc788\uc2b5\ub2c8\ub2e4\ub9c8\ub294 \uc624\ub298\uc740 \uadf8\ub7f0 \ub9c8\uc74c\uc774 \ub418\uc9c0\uac00 \uc54a\uc2b5\ub2c8\ub2e4. \uc120\ubc30\ub2d8\uc740 \ud1b5\uc77c\ub41c \uc870\uad6d\uc758 \ucd08\ub300 \ub300\ud1b5\ub839\uc774 \ub418\uace0 \uc2f6\uc73c\uc168\uc744 \ud150\ub370, \uadf8 \ub73b\uc744 \uc774\ub8e8\uc9c0 \ubabb\ud574 \uc720\uac10\uc774\uc2dc\uaca0\uc2b5\ub2c8\ub2e4\"\ub77c\uba70 \uae00\uc744 \ub05d\ub9fa\uc5c8\ub2e4. \uadf8\ub294 \uc774\uc5b4 22\uc77c\uc5d0\ub294 '\uae40\ub300\uc911 \ud615\ub2d8\uaed8'\ub77c\ub294 \ub354\uc6b1 \uc0b4\uac00\uc6b4 \uc81c\ubaa9\uc758 \uae00\uc744 \uc62c\ub838\ub2e4. \uae40 \uc804 \uad50\uc218\ub294 \"\ud55c \uc2dc\ub300\uc758 \ud48d\uc6b4\uc544 \uae40\ub300\uc911 \ud615\uc744 '\ud615\ub2d8'\uc774\ub77c\uace0 \ubd80\ub974\uace0 \uc2f6\uc740 \uc0ac\ub78c\uc740 \ubd80\uc9c0\uae30\uc218\uc774\uc5c8\uc744 \uac83\uc785\ub2c8\ub2e4. \uc800\ub3c4 \uadf8\ub7f0 \uc790\ub4e4 \uc911\uc758 \ud55c \uc0ac\ub78c\uc774 \ub418\uc5b4 \uadf8\ub807\uac8c \ubd88\ub7ec\ubcf4\uace0 \uc2f6\uc740 \uac83\ubfd0\uc774\ub2c8 \ub108\uadf8\ub7fd\uac8c \ubc1b\uc544 \uc8fc\uc2dc\uae30\ub97c \ubc14\ub78d\ub2c8\ub2e4\"\ub77c\uace0 \ub9d0\ud588\ub2e4. \uc55e\uc11c \uadf8\ub294 \uae40 \uc804 \ub300\ud1b5\ub839\uc758 \uc0ac\ub9dd \uc9c1\ud6c4\uc778 19\uc77c \"\uae40\ub300\uc911 \uc804 \ub300\ud1b5\ub839\uc758 \uc11c\uac70\ub294 \uc6b0\ub9ac \ubaa8\ub450\uc5d0\uac8c \ucc29\uc7a1\ud55c \uc2ec\uc815\uacfc \uc778\uc0dd\uc758 \ubb34\uc0c1\ud568\uc744 \uc808\uc2e4\ud788 \ub290\ub07c\uac8c \ud55c\ub2e4\"\uba70 \"\ud30c\ub780\ub9cc\uc7a5\ud55c \uc0b6\uc744 \uc0b4\uace0 \uc774\uc81c \ud3c9\ud654\ub86d\uac8c \uadf8 \uc0dd\uc774 \ub9c9\uc744 \ub0b4\ub838\uc73c\ub2c8 \ub2f9\uc7a5\uc5d0 \ud560 \ub9d0\uc744 \ucc3e\uae30 \uc5b4\ub835\ub2e4\"\uace0 \ub9d0\ud588\ub2e4."} {"question": "\ucc30\uc2a4 \uc6e8\ube0c\ub294 \ub450\ubc88\uc9f8 \uc2dc\uc98c\uc5d0\uc11c \uba87 \ubc88\uc9f8 \uc5d0\ud53c\uc18c\ub4dc\uac00 \ucd5c\uace0\uc758 \ud558\uc774\ub77c\uc774\ud2b8\ub77c\uace0 \ud3c9\uac00\ud588\ub294\uac00?", "paragraph": "\uccab \uc5d0\ud53c\uc18c\ub4dc\uc778 \u3008It Came from the Nightosphere\u3009\ub294 \ub300\uccb4\ub85c \ud638\ud3c9\uc744 \ubc1b\uc558\ub2e4. \u300aDVD \ud1a0\ud06c\u300b(DVD Talk)\uc758 \ud0c0\uc77c\ub7ec \ud3ec\uc2a4\ud130(Tyler Foster)\ub294 \ub9c8\ub974\uc140\ub9b0\uacfc \uadf8\uc758 \uc544\ube60 \uc0ac\uc774\uc758 \uac08\ub4f1\uc774 \ub2e4\ub8e8\uc5b4\uc9c0\ub294 \ubc29\uc2dd\uacfc \uadf8 \ub3c4\uc911\uc5d0 \ub098\uc628 \ud3ad\uadc4\uc5d0 \uad00\ud55c \uac1c\uadf8\uac00 \uc88b\uc558\ub2e4\uace0 \uc758\uacac\uc744 \ubc1d\ud614\ub2e4. \ub610\ud55c \u201c\ub0b4 \uc758\uacac\uc73c\ub85c\ub294 \uc5b4\ub5a4 \uc5d0\ud53c\uc18c\ub4dc\uc5d0 \ub178\ub798\uac00 \ub4e4\uc5b4\uac00\ub358 \ud50c\ub7ec\uc2a4 \uc694\uc18c\uac00 \ub420 \uac83\uc774\ub2e4.\u201d\ub77c\uace0 \ub9d0\ud588\ub2e4. \u300aMTV Geek\u300b\uc758 \ucc30\uc2a4 \uc6e8\ube0c(Charles Webb)\ub294 \ub3d9\uba85\uc758 DVD\uc5d0 \uc218\ub85d\ub41c \uc5d0\ud53c\uc18c\ub4dc \uc911\uc5d0\uc11c \uc774 \ud3b8\uc774 \u2018\ucd5c\uace0\uc758 \ud558\uc774\ub77c\uc774\ud2b8\u2019\ub77c\uace0 \uce6d\ud588\ub2e4. IGN\uc758 \uc791\uac00 \ub9e4\ud2b8 \ud30c\uc6b8\ub7ec(Matt Fowler)\ub294 \uc774 \uc5d0\ud53c\uc18c\ub4dc\ub97c \uac00\ub9ac\ucf1c \u2018\uba85\uc791\u2019(classic)\uc774\ub77c\ub294 \ud45c\ud604\uc744 \uc0ac\uc6a9\ud588\ub2e4. \ub610\ud55c 2011\ub144 \ud504\ub77c\uc784\ud0c0\uc784 \uc5d0\ubbf8\uc0c1 \u2018\ub6f0\uc5b4\ub09c \ub2e8\ud3b8 \uc560\ub2c8\uba54\uc774\uc158 \ud504\ub85c\uadf8\ub7a8\u2019\uc758 \ud6c4\ubcf4\uc5d0 \uc624\ub974\uae30\ub3c4 \ud558\uc600\ub2e4.", "answer": "\uccab \uc5d0\ud53c\uc18c\ub4dc", "sentence": "\uccab \uc5d0\ud53c\uc18c\ub4dc \uc778 \u3008It Came from the Nightosphere\u3009\ub294 \ub300\uccb4\ub85c \ud638\ud3c9\uc744 \ubc1b\uc558\ub2e4.", "paragraph_sentence": " \uccab \uc5d0\ud53c\uc18c\ub4dc \uc778 \u3008It Came from the Nightosphere\u3009\ub294 \ub300\uccb4\ub85c \ud638\ud3c9\uc744 \ubc1b\uc558\ub2e4. \u300aDVD \ud1a0\ud06c\u300b(DVD Talk)\uc758 \ud0c0\uc77c\ub7ec \ud3ec\uc2a4\ud130(Tyler Foster)\ub294 \ub9c8\ub974\uc140\ub9b0\uacfc \uadf8\uc758 \uc544\ube60 \uc0ac\uc774\uc758 \uac08\ub4f1\uc774 \ub2e4\ub8e8\uc5b4\uc9c0\ub294 \ubc29\uc2dd\uacfc \uadf8 \ub3c4\uc911\uc5d0 \ub098\uc628 \ud3ad\uadc4\uc5d0 \uad00\ud55c \uac1c\uadf8\uac00 \uc88b\uc558\ub2e4\uace0 \uc758\uacac\uc744 \ubc1d\ud614\ub2e4. \ub610\ud55c \u201c\ub0b4 \uc758\uacac\uc73c\ub85c\ub294 \uc5b4\ub5a4 \uc5d0\ud53c\uc18c\ub4dc\uc5d0 \ub178\ub798\uac00 \ub4e4\uc5b4\uac00\ub358 \ud50c\ub7ec\uc2a4 \uc694\uc18c\uac00 \ub420 \uac83\uc774\ub2e4. \u201d\ub77c\uace0 \ub9d0\ud588\ub2e4. \u300aMTV Geek\u300b\uc758 \ucc30\uc2a4 \uc6e8\ube0c(Charles Webb)\ub294 \ub3d9\uba85\uc758 DVD\uc5d0 \uc218\ub85d\ub41c \uc5d0\ud53c\uc18c\ub4dc \uc911\uc5d0\uc11c \uc774 \ud3b8\uc774 \u2018\ucd5c\uace0\uc758 \ud558\uc774\ub77c\uc774\ud2b8\u2019\ub77c\uace0 \uce6d\ud588\ub2e4. IGN\uc758 \uc791\uac00 \ub9e4\ud2b8 \ud30c\uc6b8\ub7ec(Matt Fowler)\ub294 \uc774 \uc5d0\ud53c\uc18c\ub4dc\ub97c \uac00\ub9ac\ucf1c \u2018\uba85\uc791\u2019(classic)\uc774\ub77c\ub294 \ud45c\ud604\uc744 \uc0ac\uc6a9\ud588\ub2e4. \ub610\ud55c 2011\ub144 \ud504\ub77c\uc784\ud0c0\uc784 \uc5d0\ubbf8\uc0c1 \u2018\ub6f0\uc5b4\ub09c \ub2e8\ud3b8 \uc560\ub2c8\uba54\uc774\uc158 \ud504\ub85c\uadf8\ub7a8\u2019\uc758 \ud6c4\ubcf4\uc5d0 \uc624\ub974\uae30\ub3c4 \ud558\uc600\ub2e4.", "paragraph_answer": " \uccab \uc5d0\ud53c\uc18c\ub4dc \uc778 \u3008It Came from the Nightosphere\u3009\ub294 \ub300\uccb4\ub85c \ud638\ud3c9\uc744 \ubc1b\uc558\ub2e4. \u300aDVD \ud1a0\ud06c\u300b(DVD Talk)\uc758 \ud0c0\uc77c\ub7ec \ud3ec\uc2a4\ud130(Tyler Foster)\ub294 \ub9c8\ub974\uc140\ub9b0\uacfc \uadf8\uc758 \uc544\ube60 \uc0ac\uc774\uc758 \uac08\ub4f1\uc774 \ub2e4\ub8e8\uc5b4\uc9c0\ub294 \ubc29\uc2dd\uacfc \uadf8 \ub3c4\uc911\uc5d0 \ub098\uc628 \ud3ad\uadc4\uc5d0 \uad00\ud55c \uac1c\uadf8\uac00 \uc88b\uc558\ub2e4\uace0 \uc758\uacac\uc744 \ubc1d\ud614\ub2e4. \ub610\ud55c \u201c\ub0b4 \uc758\uacac\uc73c\ub85c\ub294 \uc5b4\ub5a4 \uc5d0\ud53c\uc18c\ub4dc\uc5d0 \ub178\ub798\uac00 \ub4e4\uc5b4\uac00\ub358 \ud50c\ub7ec\uc2a4 \uc694\uc18c\uac00 \ub420 \uac83\uc774\ub2e4.\u201d\ub77c\uace0 \ub9d0\ud588\ub2e4. \u300aMTV Geek\u300b\uc758 \ucc30\uc2a4 \uc6e8\ube0c(Charles Webb)\ub294 \ub3d9\uba85\uc758 DVD\uc5d0 \uc218\ub85d\ub41c \uc5d0\ud53c\uc18c\ub4dc \uc911\uc5d0\uc11c \uc774 \ud3b8\uc774 \u2018\ucd5c\uace0\uc758 \ud558\uc774\ub77c\uc774\ud2b8\u2019\ub77c\uace0 \uce6d\ud588\ub2e4. IGN\uc758 \uc791\uac00 \ub9e4\ud2b8 \ud30c\uc6b8\ub7ec(Matt Fowler)\ub294 \uc774 \uc5d0\ud53c\uc18c\ub4dc\ub97c \uac00\ub9ac\ucf1c \u2018\uba85\uc791\u2019(classic)\uc774\ub77c\ub294 \ud45c\ud604\uc744 \uc0ac\uc6a9\ud588\ub2e4. \ub610\ud55c 2011\ub144 \ud504\ub77c\uc784\ud0c0\uc784 \uc5d0\ubbf8\uc0c1 \u2018\ub6f0\uc5b4\ub09c \ub2e8\ud3b8 \uc560\ub2c8\uba54\uc774\uc158 \ud504\ub85c\uadf8\ub7a8\u2019\uc758 \ud6c4\ubcf4\uc5d0 \uc624\ub974\uae30\ub3c4 \ud558\uc600\ub2e4.", "sentence_answer": " \uccab \uc5d0\ud53c\uc18c\ub4dc \uc778 \u3008It Came from the Nightosphere\u3009\ub294 \ub300\uccb4\ub85c \ud638\ud3c9\uc744 \ubc1b\uc558\ub2e4.", "paragraph_id": "6358308-7-0", "paragraph_question": "question: \ucc30\uc2a4 \uc6e8\ube0c\ub294 \ub450\ubc88\uc9f8 \uc2dc\uc98c\uc5d0\uc11c \uba87 \ubc88\uc9f8 \uc5d0\ud53c\uc18c\ub4dc\uac00 \ucd5c\uace0\uc758 \ud558\uc774\ub77c\uc774\ud2b8\ub77c\uace0 \ud3c9\uac00\ud588\ub294\uac00?, context: \uccab \uc5d0\ud53c\uc18c\ub4dc\uc778 \u3008It Came from the Nightosphere\u3009\ub294 \ub300\uccb4\ub85c \ud638\ud3c9\uc744 \ubc1b\uc558\ub2e4. \u300aDVD \ud1a0\ud06c\u300b(DVD Talk)\uc758 \ud0c0\uc77c\ub7ec \ud3ec\uc2a4\ud130(Tyler Foster)\ub294 \ub9c8\ub974\uc140\ub9b0\uacfc \uadf8\uc758 \uc544\ube60 \uc0ac\uc774\uc758 \uac08\ub4f1\uc774 \ub2e4\ub8e8\uc5b4\uc9c0\ub294 \ubc29\uc2dd\uacfc \uadf8 \ub3c4\uc911\uc5d0 \ub098\uc628 \ud3ad\uadc4\uc5d0 \uad00\ud55c \uac1c\uadf8\uac00 \uc88b\uc558\ub2e4\uace0 \uc758\uacac\uc744 \ubc1d\ud614\ub2e4. \ub610\ud55c \u201c\ub0b4 \uc758\uacac\uc73c\ub85c\ub294 \uc5b4\ub5a4 \uc5d0\ud53c\uc18c\ub4dc\uc5d0 \ub178\ub798\uac00 \ub4e4\uc5b4\uac00\ub358 \ud50c\ub7ec\uc2a4 \uc694\uc18c\uac00 \ub420 \uac83\uc774\ub2e4.\u201d\ub77c\uace0 \ub9d0\ud588\ub2e4. \u300aMTV Geek\u300b\uc758 \ucc30\uc2a4 \uc6e8\ube0c(Charles Webb)\ub294 \ub3d9\uba85\uc758 DVD\uc5d0 \uc218\ub85d\ub41c \uc5d0\ud53c\uc18c\ub4dc \uc911\uc5d0\uc11c \uc774 \ud3b8\uc774 \u2018\ucd5c\uace0\uc758 \ud558\uc774\ub77c\uc774\ud2b8\u2019\ub77c\uace0 \uce6d\ud588\ub2e4. IGN\uc758 \uc791\uac00 \ub9e4\ud2b8 \ud30c\uc6b8\ub7ec(Matt Fowler)\ub294 \uc774 \uc5d0\ud53c\uc18c\ub4dc\ub97c \uac00\ub9ac\ucf1c \u2018\uba85\uc791\u2019(classic)\uc774\ub77c\ub294 \ud45c\ud604\uc744 \uc0ac\uc6a9\ud588\ub2e4. \ub610\ud55c 2011\ub144 \ud504\ub77c\uc784\ud0c0\uc784 \uc5d0\ubbf8\uc0c1 \u2018\ub6f0\uc5b4\ub09c \ub2e8\ud3b8 \uc560\ub2c8\uba54\uc774\uc158 \ud504\ub85c\uadf8\ub7a8\u2019\uc758 \ud6c4\ubcf4\uc5d0 \uc624\ub974\uae30\ub3c4 \ud558\uc600\ub2e4."} {"question": "\uc67c\ucabd \uc190\uac00\ub77d\ub4e4\uc5d0 \ud560\ub2f9\ub41c \uc601\uc5ed\uc740 \uc5b4\ub824\uc11c \ubb34\uc5c7\uc744 \ubc30\uc6b0\uae30 \uc2dc\uc791\ud55c \uc0ac\ub78c\ub4e4\uc5d0\uac8c\uc11c \ub354 \ucef8\ub294\uac00?", "paragraph": "\ud55c \uc5f0\uad6c\ub294 \uc21c\uc218\uc74c\uc5d0 \ub300\ud55c \uccad\uac01\ud53c\uc9c8\uc758 \ubc18\uc751\uc744 \uae30\ub85d\ud558\uc600\ub2e4. \uadf8 \ubc18\uc751\uc740 \ube44\uc74c\uc545\uac00\ub4e4\uc5d0\uac8c\uc11c\ubcf4\ub2e4 \uc9c1\uc5c5\uc801\uc778 \uc74c\uc545\uac00\ub4e4\uc5d0\uac8c\uc11c \uc57d \ub450 \ubc30 \uc815\ub3c4\ub098 \ub192\uc558\ub2e4. \uc774 \uc74c\uc545\uac00\ub4e4\uc758 \ub1cc\uc5d0 \ub300\ud55c MRI \uac80\uc0ac\uc5d0\uc11c\ub294 \uc6b0\ubc18\uad6c\uc758 \uce21\ub450 \ud53c\uc9c8\uc758 \ud55c \uc601\uc5ed\uc774 \uc57d 30% \ub354 \ud06c\ub2e4\ub294 \uac83\uc774 \ubc1c\uacac\ud558\uc600\ub2e4(Schneider et al., 2002). \ubb3c\ub860 \uc774 \uc790\ub8cc\ub294 \uc774\ub4e4\uc774 \uc74c\uc545\uc5d0 \uc804\ubb38\ud654\ub41c \ub1cc\ub97c \ud0c0\uace0\ub0ac\uae30 \ub54c\ubb38\uc5d0 \uc9c1\uc5c5 \uc74c\uc545\uac00\uac00 \ub41c \uac83\uc778\uc9c0 \uc544\ub2c8\uba74 \uc5b4\ub9b0 \uc2dc\uc808\uc758 \ub9ce\uc740 \uc5f0\uc2b5\uc774 \ud2b9\uc815 \ub1cc \uc601\uc5ed\uc774 \uc131\uc7a5\ud558\ub294\ub370 \ub3c4\uc6c0\uc774 \ub418\uc5c8\ub294\uc9c0\ub97c \ub9d0\ud574\uc8fc\uc9c0\ub294 \uc54a\ub294\ub2e4. \uc774\uc640 \uad00\ub828\ub41c \ud55c \uc5f0\uad6c\ub294 \ube44\uc74c\uc545\uac00\ub4e4\uacfc \ud604\uc545\uae30(\ub300\ubd80\ubd84 \ubc14\uc774\uc62c\ub9b0)\ub97c \uc5f0\uc8fc\ud55c \uacbd\ud5d8\uc774 \ub9ce\uc740 \uc0ac\ub78c\ub4e4\uc758 \uc911\uc2ec\ud6c4\ud68c\ub97c \ube44\uad50\ud558\uc600\ub2e4. \uc911\uc2ec\ud6c4\ud68c\ub294 \uc77c\ucc28 \uccb4\uac10\uac01\ud53c\uc9c8\ub85c\uc11c, \uc774 \ud68c \uc0c1\uc758 \uac01 \uc601\uc5ed\uc774 \uc2e0\uccb4\uc758 \ud2b9\uc815 \ubd80\uc704\uc5d0 \ubc18\uc751\ud55c\ub2e4. \ud604\uc545\uae30 \uc5f0\uc8fc\uc790\ub4e4\uc740 \ud604\uc744 \uc9da\ub294\ub370 \uc67c\uc190\uc744 \uc368\uc57c \ud558\ub294\ub370, \uc774\ub4e4\uc758 \uc790\uae30\ub1cc\ub3c4\uae30\ub85d\ubc95(MEG) \uacb0\uacfc\ub294 \uc911\uc2ec\ud6c4\ud68c\uc5d0 \uc788\ub294 \uc67c\uc190\uc758 \uc190\uac00\ub77d\ub4e4\uc5d0 \ub300\ud55c \ud45c\uc0c1\uc774 \ud655\uc7a5\ub418\uc5b4 \uc788\uc74c\uc744 \ubcf4\uc5ec\uc8fc\uc5c8\ub2e4(Elbert, Pantev, Wienbruch, Rockstroh, & Taub 1995). \uc67c\ucabd \uc190\uac00\ub77d\ub4e4\uc5d0 \ud560\ub2f9\ub41c \uc601\uc5ed\uc740 \ud604\uc545\uae30\ub97c \uc5b4\ub824\uc11c\ubd80\ud130 \ubc30\uc6b0\uae30 \uc2dc\uc791\ud55c \uc0ac\ub78c\ub4e4\uc5d0\uac8c\uc11c \ub354 \ucef8\ub2e4. \uc774\ub4e4\uc740 \ubb3c\ub860 \uc774 \uc5f0\uad6c\uac00 \ud589\ud574\uc84c\ub358 \uc2dc\uc810\uc5d0 \ub354 \uc624\ub7ab\ub3d9\uc548 \ud6c8\ub828\uc744 \ud574 \uc628 \uc0ac\ub78c\ub4e4\uc774\uae30\ub3c4 \ud558\ub2e4.", "answer": "\ud604\uc545\uae30", "sentence": "\uc774\uc640 \uad00\ub828\ub41c \ud55c \uc5f0\uad6c\ub294 \ube44\uc74c\uc545\uac00\ub4e4\uacfc \ud604\uc545\uae30 (\ub300\ubd80\ubd84 \ubc14\uc774\uc62c\ub9b0)\ub97c \uc5f0\uc8fc\ud55c \uacbd\ud5d8\uc774 \ub9ce\uc740 \uc0ac\ub78c\ub4e4\uc758 \uc911\uc2ec\ud6c4\ud68c\ub97c \ube44\uad50\ud558\uc600\ub2e4.", "paragraph_sentence": "\ud55c \uc5f0\uad6c\ub294 \uc21c\uc218\uc74c\uc5d0 \ub300\ud55c \uccad\uac01\ud53c\uc9c8\uc758 \ubc18\uc751\uc744 \uae30\ub85d\ud558\uc600\ub2e4. \uadf8 \ubc18\uc751\uc740 \ube44\uc74c\uc545\uac00\ub4e4\uc5d0\uac8c\uc11c\ubcf4\ub2e4 \uc9c1\uc5c5\uc801\uc778 \uc74c\uc545\uac00\ub4e4\uc5d0\uac8c\uc11c \uc57d \ub450 \ubc30 \uc815\ub3c4\ub098 \ub192\uc558\ub2e4. \uc774 \uc74c\uc545\uac00\ub4e4\uc758 \ub1cc\uc5d0 \ub300\ud55c MRI \uac80\uc0ac\uc5d0\uc11c\ub294 \uc6b0\ubc18\uad6c\uc758 \uce21\ub450 \ud53c\uc9c8\uc758 \ud55c \uc601\uc5ed\uc774 \uc57d 30% \ub354 \ud06c\ub2e4\ub294 \uac83\uc774 \ubc1c\uacac\ud558\uc600\ub2e4(Schneider et al. , 2002) . \ubb3c\ub860 \uc774 \uc790\ub8cc\ub294 \uc774\ub4e4\uc774 \uc74c\uc545\uc5d0 \uc804\ubb38\ud654\ub41c \ub1cc\ub97c \ud0c0\uace0\ub0ac\uae30 \ub54c\ubb38\uc5d0 \uc9c1\uc5c5 \uc74c\uc545\uac00\uac00 \ub41c \uac83\uc778\uc9c0 \uc544\ub2c8\uba74 \uc5b4\ub9b0 \uc2dc\uc808\uc758 \ub9ce\uc740 \uc5f0\uc2b5\uc774 \ud2b9\uc815 \ub1cc \uc601\uc5ed\uc774 \uc131\uc7a5\ud558\ub294\ub370 \ub3c4\uc6c0\uc774 \ub418\uc5c8\ub294\uc9c0\ub97c \ub9d0\ud574\uc8fc\uc9c0\ub294 \uc54a\ub294\ub2e4. \uc774\uc640 \uad00\ub828\ub41c \ud55c \uc5f0\uad6c\ub294 \ube44\uc74c\uc545\uac00\ub4e4\uacfc \ud604\uc545\uae30 (\ub300\ubd80\ubd84 \ubc14\uc774\uc62c\ub9b0)\ub97c \uc5f0\uc8fc\ud55c \uacbd\ud5d8\uc774 \ub9ce\uc740 \uc0ac\ub78c\ub4e4\uc758 \uc911\uc2ec\ud6c4\ud68c\ub97c \ube44\uad50\ud558\uc600\ub2e4. \uc911\uc2ec\ud6c4\ud68c\ub294 \uc77c\ucc28 \uccb4\uac10\uac01\ud53c\uc9c8\ub85c\uc11c, \uc774 \ud68c \uc0c1\uc758 \uac01 \uc601\uc5ed\uc774 \uc2e0\uccb4\uc758 \ud2b9\uc815 \ubd80\uc704\uc5d0 \ubc18\uc751\ud55c\ub2e4. \ud604\uc545\uae30 \uc5f0\uc8fc\uc790\ub4e4\uc740 \ud604\uc744 \uc9da\ub294\ub370 \uc67c\uc190\uc744 \uc368\uc57c \ud558\ub294\ub370, \uc774\ub4e4\uc758 \uc790\uae30\ub1cc\ub3c4\uae30\ub85d\ubc95(MEG) \uacb0\uacfc\ub294 \uc911\uc2ec\ud6c4\ud68c\uc5d0 \uc788\ub294 \uc67c\uc190\uc758 \uc190\uac00\ub77d\ub4e4\uc5d0 \ub300\ud55c \ud45c\uc0c1\uc774 \ud655\uc7a5\ub418\uc5b4 \uc788\uc74c\uc744 \ubcf4\uc5ec\uc8fc\uc5c8\ub2e4(Elbert, Pantev, Wienbruch, Rockstroh, & Taub 1995). \uc67c\ucabd \uc190\uac00\ub77d\ub4e4\uc5d0 \ud560\ub2f9\ub41c \uc601\uc5ed\uc740 \ud604\uc545\uae30\ub97c \uc5b4\ub824\uc11c\ubd80\ud130 \ubc30\uc6b0\uae30 \uc2dc\uc791\ud55c \uc0ac\ub78c\ub4e4\uc5d0\uac8c\uc11c \ub354 \ucef8\ub2e4. \uc774\ub4e4\uc740 \ubb3c\ub860 \uc774 \uc5f0\uad6c\uac00 \ud589\ud574\uc84c\ub358 \uc2dc\uc810\uc5d0 \ub354 \uc624\ub7ab\ub3d9\uc548 \ud6c8\ub828\uc744 \ud574 \uc628 \uc0ac\ub78c\ub4e4\uc774\uae30\ub3c4 \ud558\ub2e4.", "paragraph_answer": "\ud55c \uc5f0\uad6c\ub294 \uc21c\uc218\uc74c\uc5d0 \ub300\ud55c \uccad\uac01\ud53c\uc9c8\uc758 \ubc18\uc751\uc744 \uae30\ub85d\ud558\uc600\ub2e4. \uadf8 \ubc18\uc751\uc740 \ube44\uc74c\uc545\uac00\ub4e4\uc5d0\uac8c\uc11c\ubcf4\ub2e4 \uc9c1\uc5c5\uc801\uc778 \uc74c\uc545\uac00\ub4e4\uc5d0\uac8c\uc11c \uc57d \ub450 \ubc30 \uc815\ub3c4\ub098 \ub192\uc558\ub2e4. \uc774 \uc74c\uc545\uac00\ub4e4\uc758 \ub1cc\uc5d0 \ub300\ud55c MRI \uac80\uc0ac\uc5d0\uc11c\ub294 \uc6b0\ubc18\uad6c\uc758 \uce21\ub450 \ud53c\uc9c8\uc758 \ud55c \uc601\uc5ed\uc774 \uc57d 30% \ub354 \ud06c\ub2e4\ub294 \uac83\uc774 \ubc1c\uacac\ud558\uc600\ub2e4(Schneider et al., 2002). \ubb3c\ub860 \uc774 \uc790\ub8cc\ub294 \uc774\ub4e4\uc774 \uc74c\uc545\uc5d0 \uc804\ubb38\ud654\ub41c \ub1cc\ub97c \ud0c0\uace0\ub0ac\uae30 \ub54c\ubb38\uc5d0 \uc9c1\uc5c5 \uc74c\uc545\uac00\uac00 \ub41c \uac83\uc778\uc9c0 \uc544\ub2c8\uba74 \uc5b4\ub9b0 \uc2dc\uc808\uc758 \ub9ce\uc740 \uc5f0\uc2b5\uc774 \ud2b9\uc815 \ub1cc \uc601\uc5ed\uc774 \uc131\uc7a5\ud558\ub294\ub370 \ub3c4\uc6c0\uc774 \ub418\uc5c8\ub294\uc9c0\ub97c \ub9d0\ud574\uc8fc\uc9c0\ub294 \uc54a\ub294\ub2e4. \uc774\uc640 \uad00\ub828\ub41c \ud55c \uc5f0\uad6c\ub294 \ube44\uc74c\uc545\uac00\ub4e4\uacfc \ud604\uc545\uae30 (\ub300\ubd80\ubd84 \ubc14\uc774\uc62c\ub9b0)\ub97c \uc5f0\uc8fc\ud55c \uacbd\ud5d8\uc774 \ub9ce\uc740 \uc0ac\ub78c\ub4e4\uc758 \uc911\uc2ec\ud6c4\ud68c\ub97c \ube44\uad50\ud558\uc600\ub2e4. \uc911\uc2ec\ud6c4\ud68c\ub294 \uc77c\ucc28 \uccb4\uac10\uac01\ud53c\uc9c8\ub85c\uc11c, \uc774 \ud68c \uc0c1\uc758 \uac01 \uc601\uc5ed\uc774 \uc2e0\uccb4\uc758 \ud2b9\uc815 \ubd80\uc704\uc5d0 \ubc18\uc751\ud55c\ub2e4. \ud604\uc545\uae30 \uc5f0\uc8fc\uc790\ub4e4\uc740 \ud604\uc744 \uc9da\ub294\ub370 \uc67c\uc190\uc744 \uc368\uc57c \ud558\ub294\ub370, \uc774\ub4e4\uc758 \uc790\uae30\ub1cc\ub3c4\uae30\ub85d\ubc95(MEG) \uacb0\uacfc\ub294 \uc911\uc2ec\ud6c4\ud68c\uc5d0 \uc788\ub294 \uc67c\uc190\uc758 \uc190\uac00\ub77d\ub4e4\uc5d0 \ub300\ud55c \ud45c\uc0c1\uc774 \ud655\uc7a5\ub418\uc5b4 \uc788\uc74c\uc744 \ubcf4\uc5ec\uc8fc\uc5c8\ub2e4(Elbert, Pantev, Wienbruch, Rockstroh, & Taub 1995). \uc67c\ucabd \uc190\uac00\ub77d\ub4e4\uc5d0 \ud560\ub2f9\ub41c \uc601\uc5ed\uc740 \ud604\uc545\uae30\ub97c \uc5b4\ub824\uc11c\ubd80\ud130 \ubc30\uc6b0\uae30 \uc2dc\uc791\ud55c \uc0ac\ub78c\ub4e4\uc5d0\uac8c\uc11c \ub354 \ucef8\ub2e4. \uc774\ub4e4\uc740 \ubb3c\ub860 \uc774 \uc5f0\uad6c\uac00 \ud589\ud574\uc84c\ub358 \uc2dc\uc810\uc5d0 \ub354 \uc624\ub7ab\ub3d9\uc548 \ud6c8\ub828\uc744 \ud574 \uc628 \uc0ac\ub78c\ub4e4\uc774\uae30\ub3c4 \ud558\ub2e4.", "sentence_answer": "\uc774\uc640 \uad00\ub828\ub41c \ud55c \uc5f0\uad6c\ub294 \ube44\uc74c\uc545\uac00\ub4e4\uacfc \ud604\uc545\uae30 (\ub300\ubd80\ubd84 \ubc14\uc774\uc62c\ub9b0)\ub97c \uc5f0\uc8fc\ud55c \uacbd\ud5d8\uc774 \ub9ce\uc740 \uc0ac\ub78c\ub4e4\uc758 \uc911\uc2ec\ud6c4\ud68c\ub97c \ube44\uad50\ud558\uc600\ub2e4.", "paragraph_id": "6584757-4-2", "paragraph_question": "question: \uc67c\ucabd \uc190\uac00\ub77d\ub4e4\uc5d0 \ud560\ub2f9\ub41c \uc601\uc5ed\uc740 \uc5b4\ub824\uc11c \ubb34\uc5c7\uc744 \ubc30\uc6b0\uae30 \uc2dc\uc791\ud55c \uc0ac\ub78c\ub4e4\uc5d0\uac8c\uc11c \ub354 \ucef8\ub294\uac00?, context: \ud55c \uc5f0\uad6c\ub294 \uc21c\uc218\uc74c\uc5d0 \ub300\ud55c \uccad\uac01\ud53c\uc9c8\uc758 \ubc18\uc751\uc744 \uae30\ub85d\ud558\uc600\ub2e4. \uadf8 \ubc18\uc751\uc740 \ube44\uc74c\uc545\uac00\ub4e4\uc5d0\uac8c\uc11c\ubcf4\ub2e4 \uc9c1\uc5c5\uc801\uc778 \uc74c\uc545\uac00\ub4e4\uc5d0\uac8c\uc11c \uc57d \ub450 \ubc30 \uc815\ub3c4\ub098 \ub192\uc558\ub2e4. \uc774 \uc74c\uc545\uac00\ub4e4\uc758 \ub1cc\uc5d0 \ub300\ud55c MRI \uac80\uc0ac\uc5d0\uc11c\ub294 \uc6b0\ubc18\uad6c\uc758 \uce21\ub450 \ud53c\uc9c8\uc758 \ud55c \uc601\uc5ed\uc774 \uc57d 30% \ub354 \ud06c\ub2e4\ub294 \uac83\uc774 \ubc1c\uacac\ud558\uc600\ub2e4(Schneider et al., 2002). \ubb3c\ub860 \uc774 \uc790\ub8cc\ub294 \uc774\ub4e4\uc774 \uc74c\uc545\uc5d0 \uc804\ubb38\ud654\ub41c \ub1cc\ub97c \ud0c0\uace0\ub0ac\uae30 \ub54c\ubb38\uc5d0 \uc9c1\uc5c5 \uc74c\uc545\uac00\uac00 \ub41c \uac83\uc778\uc9c0 \uc544\ub2c8\uba74 \uc5b4\ub9b0 \uc2dc\uc808\uc758 \ub9ce\uc740 \uc5f0\uc2b5\uc774 \ud2b9\uc815 \ub1cc \uc601\uc5ed\uc774 \uc131\uc7a5\ud558\ub294\ub370 \ub3c4\uc6c0\uc774 \ub418\uc5c8\ub294\uc9c0\ub97c \ub9d0\ud574\uc8fc\uc9c0\ub294 \uc54a\ub294\ub2e4. \uc774\uc640 \uad00\ub828\ub41c \ud55c \uc5f0\uad6c\ub294 \ube44\uc74c\uc545\uac00\ub4e4\uacfc \ud604\uc545\uae30(\ub300\ubd80\ubd84 \ubc14\uc774\uc62c\ub9b0)\ub97c \uc5f0\uc8fc\ud55c \uacbd\ud5d8\uc774 \ub9ce\uc740 \uc0ac\ub78c\ub4e4\uc758 \uc911\uc2ec\ud6c4\ud68c\ub97c \ube44\uad50\ud558\uc600\ub2e4. \uc911\uc2ec\ud6c4\ud68c\ub294 \uc77c\ucc28 \uccb4\uac10\uac01\ud53c\uc9c8\ub85c\uc11c, \uc774 \ud68c \uc0c1\uc758 \uac01 \uc601\uc5ed\uc774 \uc2e0\uccb4\uc758 \ud2b9\uc815 \ubd80\uc704\uc5d0 \ubc18\uc751\ud55c\ub2e4. \ud604\uc545\uae30 \uc5f0\uc8fc\uc790\ub4e4\uc740 \ud604\uc744 \uc9da\ub294\ub370 \uc67c\uc190\uc744 \uc368\uc57c \ud558\ub294\ub370, \uc774\ub4e4\uc758 \uc790\uae30\ub1cc\ub3c4\uae30\ub85d\ubc95(MEG) \uacb0\uacfc\ub294 \uc911\uc2ec\ud6c4\ud68c\uc5d0 \uc788\ub294 \uc67c\uc190\uc758 \uc190\uac00\ub77d\ub4e4\uc5d0 \ub300\ud55c \ud45c\uc0c1\uc774 \ud655\uc7a5\ub418\uc5b4 \uc788\uc74c\uc744 \ubcf4\uc5ec\uc8fc\uc5c8\ub2e4(Elbert, Pantev, Wienbruch, Rockstroh, & Taub 1995). \uc67c\ucabd \uc190\uac00\ub77d\ub4e4\uc5d0 \ud560\ub2f9\ub41c \uc601\uc5ed\uc740 \ud604\uc545\uae30\ub97c \uc5b4\ub824\uc11c\ubd80\ud130 \ubc30\uc6b0\uae30 \uc2dc\uc791\ud55c \uc0ac\ub78c\ub4e4\uc5d0\uac8c\uc11c \ub354 \ucef8\ub2e4. \uc774\ub4e4\uc740 \ubb3c\ub860 \uc774 \uc5f0\uad6c\uac00 \ud589\ud574\uc84c\ub358 \uc2dc\uc810\uc5d0 \ub354 \uc624\ub7ab\ub3d9\uc548 \ud6c8\ub828\uc744 \ud574 \uc628 \uc0ac\ub78c\ub4e4\uc774\uae30\ub3c4 \ud558\ub2e4."} {"question": "\uce74\ub274\ub3c4\uc758 \uc601\ud5a5\uc744 \ubc1b\uace0 \ud6c4\uacc4\uc790\ub85c\uc11c \ud504\ub791\uc2a4 \uc601\ud654\uc608\uc220\uc758 \uc9c0\ub3c4\uc801\uc778 \uc704\uce58\ub97c \ucc28\uc9c0\ud55c \uc80a\uc740 \uc800\ub110\ub9ac\uc2a4\ud2b8 \ubb38\ud559\uc790\uc758 \uc774\ub984\uc740?", "paragraph": "\ud504\ub791\uc2a4\uc758 \ub8e8\uc774 \ub378\ub93c\ud06c(1890-1924)\ub294 \uc80a\uc740 \uc800\ub110\ub9ac\uc2a4\ud2b8 \ubb38\ud559\uc790\uc600\uc73c\uba70, \uce74\ub274\ub3c4\uc758 \uc601\ud5a5\uc744 \ubc1b\uace0 \uadf8\uc758 \ud6c4\uacc4\uc790\ub85c\uc11c \ud504\ub791\uc2a4\uc758 \uc601\ud654\uc608\uc220\uc758 \uc9c0\ub3c4\uc801\uc778 \uc704\uce58\ub97c \ucc28\uc9c0\ud558\uac8c \ub418\uc5c8\ub2e4. \uadf8\ub294 \uc601\ud654\uc758 \uc601\uc0c1\uc774 \ub2e8\uc21c\ud55c \uc0ac\uc9c4\uc801\uc778 \uc758\ubbf8\ub97c \uac16\ub294 \uac83\uc774 \uc544\ub2c8\ub77c, \uadf8 \uc720\uae30\uc801\uc778 \ub0b4\uc7ac\uc131\uc744 \uc8fc\uc7a5\ud558\uace0 \uc774\uac83\uc744 \ud3ec\ud1a0\uc81c\ub2c8(photegenie)\ub77c \uc774\ub984\uc9c0\uc5c8\ub2e4. \ub378\ub93c\ud06c\ub294 \ud3ec\ud1a0\uc81c\ub2c8\uc758 4\uac00\uc9c0 \uae30\ubcf8\uc801 \uc694\uc18c\ub85c\uc11c '\uc7a5\uce58' '\uc870\uba85(\ube5b)' '\uc5b5\uc591(\u6291\u63da)' '\ub9c8\uc2a4\ud06c(\u9762)'\ub97c \ub4e4\uace0, \uc774 \uc694\uc18c\ub4e4\uc744 \uac10\ub3c5\uc774 \uc790\uc720\ub85c\uc774 \uc120\ud0dd\ud558\uace0 \uc9dc\ub9de\ucd98 \ucc3d\uc870 \uc704\uc5d0 \uc601\ud654\uc758 \uc608\uc220\uc131\uc774 \uc131\ub9bd\ud55c\ub2e4\uace0 \uc8fc\uc7a5\ud55c\ub2e4. \uadf8\ub294 \ud2b9\ud788 '\uc5b5\uc591'\uc744 \uc911\uc694\uc2dc\ud588\ub2e4. \uc774\ub7ec\ud55c \uc0ac\uace0\ubc29\uc2dd\uc740 \ub9c8\uce68\ub0b4 \uc601\ud654\uc758 #\ubabd\ud0c0\uc8fc\ub860\uc758 \ubc1c\uc804\uc744 \uac00\uc838\uc654\ub2e4(\ubabd\ud0c0\uc8fc\ub77c\ub294 \ub9d0\uc744 \uc601\ud654\uc758 \ud14c\ud06c\ub2c9\uc758 \uc6a9\uc5b4\ub85c\uc11c \uba3c\uc800 \ubc1c\uacac\ud55c \uc0ac\ub78c\uc740 \ub378\ub93c\ud06c\uc600\ub2e4). \ub378\ub93c\ud06c\ub294 1920\ub144\ub300\uc5d0 \ubc1c\ub2ec\ud55c \ud504\ub791\uc2a4 \uc804\uc704 \uc601\ud654(\uc544\ubc29\uac00\ub974\ub4dc)\uc758 \uc774\ub860\uc801\uc778 \ub9ac\ub354\uac00 \ub418\uace0, \uc790\uae30 \uc790\uc2e0\uc774 <\uad11\uc5f4(\u72c2\u71b1)>\uacfc \uadf8 \ubc16\uc758 \uc791\ud488\uc744 \ub9cc\ub4e4\uc5c8\uc73c\ub098 \uc560\uc11d\ud558\uac8c\ub3c4 \uc694\uc808(\u592d\u6298)\ud588\ub2e4. \uadf8\uc758 \uc774\ub860\uc740 \ud504\ub791\uc2a4\uc5d0\uc11c\ub294 \ub808\uc628 \ubb34\uc2dc\ub098\ud06c, \uc7a5 \uc5e1\uc2a4\ud0f1, \uc81c\ub974\uba54\ub290 \ub4ac\ub77c\ud06c, \ub974\ub124 \ud074\ub808\ub974 \ub4f1\uc758 \uc804\uc704\uc8fc\uc758 \uc804\uc131\uae30\uc758 \uc774\ub860\uac00 \uc791\uac00\ub4e4\uc5d0\uac8c \uacc4\uc2b9\ub418\uace0 \uc2e4\ucc9c\ub418\uc5b4 \uac14\uc73c\ub098, \ubbf8\uc220\u00b7\uc74c\uc545 \ub4f1\uc758 \ub2e4\ub978 \uc608\uc220\uc758 \uc804\uc704\uc640 \ud2b9\ud788 \ubc00\uc811\ud55c \uacb0\ud569\uc744 \ubcf4\uc5ec \uc0ac\uc77c\ub7f0\ud2b8 \uc601\ud654\uc2dc\ub300\uc758 \uc601\uc0c1\uc8fc\uc758(\u6620\u50cf\u4e3b\u7fa9)\uc758 \uc815\uc810\uc744 \ud615\uc131\ud558\uac8c \ub418\uc5c8\ub2e4.", "answer": "\ub8e8\uc774 \ub378\ub93c\ud06c", "sentence": "\ud504\ub791\uc2a4\uc758 \ub8e8\uc774 \ub378\ub93c\ud06c (1890-1924)\ub294 \uc80a\uc740 \uc800\ub110\ub9ac\uc2a4\ud2b8 \ubb38\ud559\uc790\uc600\uc73c\uba70, \uce74\ub274\ub3c4\uc758 \uc601\ud5a5\uc744 \ubc1b\uace0 \uadf8\uc758 \ud6c4\uacc4\uc790\ub85c\uc11c \ud504\ub791\uc2a4\uc758 \uc601\ud654\uc608\uc220\uc758 \uc9c0\ub3c4\uc801\uc778 \uc704\uce58\ub97c \ucc28\uc9c0\ud558\uac8c \ub418\uc5c8\ub2e4.", "paragraph_sentence": " \ud504\ub791\uc2a4\uc758 \ub8e8\uc774 \ub378\ub93c\ud06c (1890-1924)\ub294 \uc80a\uc740 \uc800\ub110\ub9ac\uc2a4\ud2b8 \ubb38\ud559\uc790\uc600\uc73c\uba70, \uce74\ub274\ub3c4\uc758 \uc601\ud5a5\uc744 \ubc1b\uace0 \uadf8\uc758 \ud6c4\uacc4\uc790\ub85c\uc11c \ud504\ub791\uc2a4\uc758 \uc601\ud654\uc608\uc220\uc758 \uc9c0\ub3c4\uc801\uc778 \uc704\uce58\ub97c \ucc28\uc9c0\ud558\uac8c \ub418\uc5c8\ub2e4. \uadf8\ub294 \uc601\ud654\uc758 \uc601\uc0c1\uc774 \ub2e8\uc21c\ud55c \uc0ac\uc9c4\uc801\uc778 \uc758\ubbf8\ub97c \uac16\ub294 \uac83\uc774 \uc544\ub2c8\ub77c, \uadf8 \uc720\uae30\uc801\uc778 \ub0b4\uc7ac\uc131\uc744 \uc8fc\uc7a5\ud558\uace0 \uc774\uac83\uc744 \ud3ec\ud1a0\uc81c\ub2c8(photegenie)\ub77c \uc774\ub984\uc9c0\uc5c8\ub2e4. \ub378\ub93c\ud06c\ub294 \ud3ec\ud1a0\uc81c\ub2c8\uc758 4\uac00\uc9c0 \uae30\ubcf8\uc801 \uc694\uc18c\ub85c\uc11c '\uc7a5\uce58' '\uc870\uba85(\ube5b)' '\uc5b5\uc591(\u6291\u63da)' '\ub9c8\uc2a4\ud06c(\u9762)'\ub97c \ub4e4\uace0, \uc774 \uc694\uc18c\ub4e4\uc744 \uac10\ub3c5\uc774 \uc790\uc720\ub85c\uc774 \uc120\ud0dd\ud558\uace0 \uc9dc\ub9de\ucd98 \ucc3d\uc870 \uc704\uc5d0 \uc601\ud654\uc758 \uc608\uc220\uc131\uc774 \uc131\ub9bd\ud55c\ub2e4\uace0 \uc8fc\uc7a5\ud55c\ub2e4. \uadf8\ub294 \ud2b9\ud788 '\uc5b5\uc591'\uc744 \uc911\uc694\uc2dc\ud588\ub2e4. \uc774\ub7ec\ud55c \uc0ac\uace0\ubc29\uc2dd\uc740 \ub9c8\uce68\ub0b4 \uc601\ud654\uc758 #\ubabd\ud0c0\uc8fc\ub860\uc758 \ubc1c\uc804\uc744 \uac00\uc838\uc654\ub2e4(\ubabd\ud0c0\uc8fc\ub77c\ub294 \ub9d0\uc744 \uc601\ud654\uc758 \ud14c\ud06c\ub2c9\uc758 \uc6a9\uc5b4\ub85c\uc11c \uba3c\uc800 \ubc1c\uacac\ud55c \uc0ac\ub78c\uc740 \ub378\ub93c\ud06c\uc600\ub2e4). \ub378\ub93c\ud06c\ub294 1920\ub144\ub300\uc5d0 \ubc1c\ub2ec\ud55c \ud504\ub791\uc2a4 \uc804\uc704 \uc601\ud654(\uc544\ubc29\uac00\ub974\ub4dc)\uc758 \uc774\ub860\uc801\uc778 \ub9ac\ub354\uac00 \ub418\uace0, \uc790\uae30 \uc790\uc2e0\uc774 <\uad11\uc5f4(\u72c2\u71b1)>\uacfc \uadf8 \ubc16\uc758 \uc791\ud488\uc744 \ub9cc\ub4e4\uc5c8\uc73c\ub098 \uc560\uc11d\ud558\uac8c\ub3c4 \uc694\uc808(\u592d\u6298)\ud588\ub2e4. \uadf8\uc758 \uc774\ub860\uc740 \ud504\ub791\uc2a4\uc5d0\uc11c\ub294 \ub808\uc628 \ubb34\uc2dc\ub098\ud06c, \uc7a5 \uc5e1\uc2a4\ud0f1, \uc81c\ub974\uba54\ub290 \ub4ac\ub77c\ud06c, \ub974\ub124 \ud074\ub808\ub974 \ub4f1\uc758 \uc804\uc704\uc8fc\uc758 \uc804\uc131\uae30\uc758 \uc774\ub860\uac00 \uc791\uac00\ub4e4\uc5d0\uac8c \uacc4\uc2b9\ub418\uace0 \uc2e4\ucc9c\ub418\uc5b4 \uac14\uc73c\ub098, \ubbf8\uc220\u00b7\uc74c\uc545 \ub4f1\uc758 \ub2e4\ub978 \uc608\uc220\uc758 \uc804\uc704\uc640 \ud2b9\ud788 \ubc00\uc811\ud55c \uacb0\ud569\uc744 \ubcf4\uc5ec \uc0ac\uc77c\ub7f0\ud2b8 \uc601\ud654\uc2dc\ub300\uc758 \uc601\uc0c1\uc8fc\uc758(\u6620\u50cf\u4e3b\u7fa9)\uc758 \uc815\uc810\uc744 \ud615\uc131\ud558\uac8c \ub418\uc5c8\ub2e4.", "paragraph_answer": "\ud504\ub791\uc2a4\uc758 \ub8e8\uc774 \ub378\ub93c\ud06c (1890-1924)\ub294 \uc80a\uc740 \uc800\ub110\ub9ac\uc2a4\ud2b8 \ubb38\ud559\uc790\uc600\uc73c\uba70, \uce74\ub274\ub3c4\uc758 \uc601\ud5a5\uc744 \ubc1b\uace0 \uadf8\uc758 \ud6c4\uacc4\uc790\ub85c\uc11c \ud504\ub791\uc2a4\uc758 \uc601\ud654\uc608\uc220\uc758 \uc9c0\ub3c4\uc801\uc778 \uc704\uce58\ub97c \ucc28\uc9c0\ud558\uac8c \ub418\uc5c8\ub2e4. \uadf8\ub294 \uc601\ud654\uc758 \uc601\uc0c1\uc774 \ub2e8\uc21c\ud55c \uc0ac\uc9c4\uc801\uc778 \uc758\ubbf8\ub97c \uac16\ub294 \uac83\uc774 \uc544\ub2c8\ub77c, \uadf8 \uc720\uae30\uc801\uc778 \ub0b4\uc7ac\uc131\uc744 \uc8fc\uc7a5\ud558\uace0 \uc774\uac83\uc744 \ud3ec\ud1a0\uc81c\ub2c8(photegenie)\ub77c \uc774\ub984\uc9c0\uc5c8\ub2e4. \ub378\ub93c\ud06c\ub294 \ud3ec\ud1a0\uc81c\ub2c8\uc758 4\uac00\uc9c0 \uae30\ubcf8\uc801 \uc694\uc18c\ub85c\uc11c '\uc7a5\uce58' '\uc870\uba85(\ube5b)' '\uc5b5\uc591(\u6291\u63da)' '\ub9c8\uc2a4\ud06c(\u9762)'\ub97c \ub4e4\uace0, \uc774 \uc694\uc18c\ub4e4\uc744 \uac10\ub3c5\uc774 \uc790\uc720\ub85c\uc774 \uc120\ud0dd\ud558\uace0 \uc9dc\ub9de\ucd98 \ucc3d\uc870 \uc704\uc5d0 \uc601\ud654\uc758 \uc608\uc220\uc131\uc774 \uc131\ub9bd\ud55c\ub2e4\uace0 \uc8fc\uc7a5\ud55c\ub2e4. \uadf8\ub294 \ud2b9\ud788 '\uc5b5\uc591'\uc744 \uc911\uc694\uc2dc\ud588\ub2e4. \uc774\ub7ec\ud55c \uc0ac\uace0\ubc29\uc2dd\uc740 \ub9c8\uce68\ub0b4 \uc601\ud654\uc758 #\ubabd\ud0c0\uc8fc\ub860\uc758 \ubc1c\uc804\uc744 \uac00\uc838\uc654\ub2e4(\ubabd\ud0c0\uc8fc\ub77c\ub294 \ub9d0\uc744 \uc601\ud654\uc758 \ud14c\ud06c\ub2c9\uc758 \uc6a9\uc5b4\ub85c\uc11c \uba3c\uc800 \ubc1c\uacac\ud55c \uc0ac\ub78c\uc740 \ub378\ub93c\ud06c\uc600\ub2e4). \ub378\ub93c\ud06c\ub294 1920\ub144\ub300\uc5d0 \ubc1c\ub2ec\ud55c \ud504\ub791\uc2a4 \uc804\uc704 \uc601\ud654(\uc544\ubc29\uac00\ub974\ub4dc)\uc758 \uc774\ub860\uc801\uc778 \ub9ac\ub354\uac00 \ub418\uace0, \uc790\uae30 \uc790\uc2e0\uc774 <\uad11\uc5f4(\u72c2\u71b1)>\uacfc \uadf8 \ubc16\uc758 \uc791\ud488\uc744 \ub9cc\ub4e4\uc5c8\uc73c\ub098 \uc560\uc11d\ud558\uac8c\ub3c4 \uc694\uc808(\u592d\u6298)\ud588\ub2e4. \uadf8\uc758 \uc774\ub860\uc740 \ud504\ub791\uc2a4\uc5d0\uc11c\ub294 \ub808\uc628 \ubb34\uc2dc\ub098\ud06c, \uc7a5 \uc5e1\uc2a4\ud0f1, \uc81c\ub974\uba54\ub290 \ub4ac\ub77c\ud06c, \ub974\ub124 \ud074\ub808\ub974 \ub4f1\uc758 \uc804\uc704\uc8fc\uc758 \uc804\uc131\uae30\uc758 \uc774\ub860\uac00 \uc791\uac00\ub4e4\uc5d0\uac8c \uacc4\uc2b9\ub418\uace0 \uc2e4\ucc9c\ub418\uc5b4 \uac14\uc73c\ub098, \ubbf8\uc220\u00b7\uc74c\uc545 \ub4f1\uc758 \ub2e4\ub978 \uc608\uc220\uc758 \uc804\uc704\uc640 \ud2b9\ud788 \ubc00\uc811\ud55c \uacb0\ud569\uc744 \ubcf4\uc5ec \uc0ac\uc77c\ub7f0\ud2b8 \uc601\ud654\uc2dc\ub300\uc758 \uc601\uc0c1\uc8fc\uc758(\u6620\u50cf\u4e3b\u7fa9)\uc758 \uc815\uc810\uc744 \ud615\uc131\ud558\uac8c \ub418\uc5c8\ub2e4.", "sentence_answer": "\ud504\ub791\uc2a4\uc758 \ub8e8\uc774 \ub378\ub93c\ud06c (1890-1924)\ub294 \uc80a\uc740 \uc800\ub110\ub9ac\uc2a4\ud2b8 \ubb38\ud559\uc790\uc600\uc73c\uba70, \uce74\ub274\ub3c4\uc758 \uc601\ud5a5\uc744 \ubc1b\uace0 \uadf8\uc758 \ud6c4\uacc4\uc790\ub85c\uc11c \ud504\ub791\uc2a4\uc758 \uc601\ud654\uc608\uc220\uc758 \uc9c0\ub3c4\uc801\uc778 \uc704\uce58\ub97c \ucc28\uc9c0\ud558\uac8c \ub418\uc5c8\ub2e4.", "paragraph_id": "6541198-1-1", "paragraph_question": "question: \uce74\ub274\ub3c4\uc758 \uc601\ud5a5\uc744 \ubc1b\uace0 \ud6c4\uacc4\uc790\ub85c\uc11c \ud504\ub791\uc2a4 \uc601\ud654\uc608\uc220\uc758 \uc9c0\ub3c4\uc801\uc778 \uc704\uce58\ub97c \ucc28\uc9c0\ud55c \uc80a\uc740 \uc800\ub110\ub9ac\uc2a4\ud2b8 \ubb38\ud559\uc790\uc758 \uc774\ub984\uc740?, context: \ud504\ub791\uc2a4\uc758 \ub8e8\uc774 \ub378\ub93c\ud06c(1890-1924)\ub294 \uc80a\uc740 \uc800\ub110\ub9ac\uc2a4\ud2b8 \ubb38\ud559\uc790\uc600\uc73c\uba70, \uce74\ub274\ub3c4\uc758 \uc601\ud5a5\uc744 \ubc1b\uace0 \uadf8\uc758 \ud6c4\uacc4\uc790\ub85c\uc11c \ud504\ub791\uc2a4\uc758 \uc601\ud654\uc608\uc220\uc758 \uc9c0\ub3c4\uc801\uc778 \uc704\uce58\ub97c \ucc28\uc9c0\ud558\uac8c \ub418\uc5c8\ub2e4. \uadf8\ub294 \uc601\ud654\uc758 \uc601\uc0c1\uc774 \ub2e8\uc21c\ud55c \uc0ac\uc9c4\uc801\uc778 \uc758\ubbf8\ub97c \uac16\ub294 \uac83\uc774 \uc544\ub2c8\ub77c, \uadf8 \uc720\uae30\uc801\uc778 \ub0b4\uc7ac\uc131\uc744 \uc8fc\uc7a5\ud558\uace0 \uc774\uac83\uc744 \ud3ec\ud1a0\uc81c\ub2c8(photegenie)\ub77c \uc774\ub984\uc9c0\uc5c8\ub2e4. \ub378\ub93c\ud06c\ub294 \ud3ec\ud1a0\uc81c\ub2c8\uc758 4\uac00\uc9c0 \uae30\ubcf8\uc801 \uc694\uc18c\ub85c\uc11c '\uc7a5\uce58' '\uc870\uba85(\ube5b)' '\uc5b5\uc591(\u6291\u63da)' '\ub9c8\uc2a4\ud06c(\u9762)'\ub97c \ub4e4\uace0, \uc774 \uc694\uc18c\ub4e4\uc744 \uac10\ub3c5\uc774 \uc790\uc720\ub85c\uc774 \uc120\ud0dd\ud558\uace0 \uc9dc\ub9de\ucd98 \ucc3d\uc870 \uc704\uc5d0 \uc601\ud654\uc758 \uc608\uc220\uc131\uc774 \uc131\ub9bd\ud55c\ub2e4\uace0 \uc8fc\uc7a5\ud55c\ub2e4. \uadf8\ub294 \ud2b9\ud788 '\uc5b5\uc591'\uc744 \uc911\uc694\uc2dc\ud588\ub2e4. \uc774\ub7ec\ud55c \uc0ac\uace0\ubc29\uc2dd\uc740 \ub9c8\uce68\ub0b4 \uc601\ud654\uc758 #\ubabd\ud0c0\uc8fc\ub860\uc758 \ubc1c\uc804\uc744 \uac00\uc838\uc654\ub2e4(\ubabd\ud0c0\uc8fc\ub77c\ub294 \ub9d0\uc744 \uc601\ud654\uc758 \ud14c\ud06c\ub2c9\uc758 \uc6a9\uc5b4\ub85c\uc11c \uba3c\uc800 \ubc1c\uacac\ud55c \uc0ac\ub78c\uc740 \ub378\ub93c\ud06c\uc600\ub2e4). \ub378\ub93c\ud06c\ub294 1920\ub144\ub300\uc5d0 \ubc1c\ub2ec\ud55c \ud504\ub791\uc2a4 \uc804\uc704 \uc601\ud654(\uc544\ubc29\uac00\ub974\ub4dc)\uc758 \uc774\ub860\uc801\uc778 \ub9ac\ub354\uac00 \ub418\uace0, \uc790\uae30 \uc790\uc2e0\uc774 <\uad11\uc5f4(\u72c2\u71b1)>\uacfc \uadf8 \ubc16\uc758 \uc791\ud488\uc744 \ub9cc\ub4e4\uc5c8\uc73c\ub098 \uc560\uc11d\ud558\uac8c\ub3c4 \uc694\uc808(\u592d\u6298)\ud588\ub2e4. \uadf8\uc758 \uc774\ub860\uc740 \ud504\ub791\uc2a4\uc5d0\uc11c\ub294 \ub808\uc628 \ubb34\uc2dc\ub098\ud06c, \uc7a5 \uc5e1\uc2a4\ud0f1, \uc81c\ub974\uba54\ub290 \ub4ac\ub77c\ud06c, \ub974\ub124 \ud074\ub808\ub974 \ub4f1\uc758 \uc804\uc704\uc8fc\uc758 \uc804\uc131\uae30\uc758 \uc774\ub860\uac00 \uc791\uac00\ub4e4\uc5d0\uac8c \uacc4\uc2b9\ub418\uace0 \uc2e4\ucc9c\ub418\uc5b4 \uac14\uc73c\ub098, \ubbf8\uc220\u00b7\uc74c\uc545 \ub4f1\uc758 \ub2e4\ub978 \uc608\uc220\uc758 \uc804\uc704\uc640 \ud2b9\ud788 \ubc00\uc811\ud55c \uacb0\ud569\uc744 \ubcf4\uc5ec \uc0ac\uc77c\ub7f0\ud2b8 \uc601\ud654\uc2dc\ub300\uc758 \uc601\uc0c1\uc8fc\uc758(\u6620\u50cf\u4e3b\u7fa9)\uc758 \uc815\uc810\uc744 \ud615\uc131\ud558\uac8c \ub418\uc5c8\ub2e4."} {"question": "\ub7ec\uc2dc\uc544 \uc804\ud568 \ud3f4\ud0c0\ubc14\uc758 \ubd80\ud3ec\ub294 12\uac1c\uc758 \uce74\ub124 \uba87\uc778\uce58 \uc18d\uc0ac\ud3ec\ub85c \uc774\ub904\uc838 \uc788\uc2b5\ub2c8\uae4c?", "paragraph": "\ud398\ud2b8\ub85c\ud30c\ube14\ub86d\uc2a4\ud06c\uae09 \uc804\ud568\uc758 \uc8fc\ud3ec\ub294 2\uac1c\uc758 \ud2b8\uc708 \ud3ec\ud0d1\uc5d0 \uc7a5\ucc29\ub41c 4\ubb38\uc758 12\uc778\uce58\ud3ec\ub85c \uad6c\uc131\ub3c4\uc5b4 \uc788\uace0, \uc120\ub8e8\uc758 \uc55e\ub4a4\ub85c \ubc30\uce58\ud588\ub2e4. 90\ucd08\ub2f9 1\ubc1c\uc744 \ubc1c\uc0ac\ud558\ub3c4\ub85d \uc124\uacc4\ub418\uc5c8\uc9c0\ub9cc, \uc2e4\uc81c \ubc1c\uc0ac \uc18d\ub3c4\ub294 \uc808\ubc18\uc774 \uc18c\uc694\ub418\uc5c8\ub2e4. \ubd80\ud3ec\ub294 12\uac1c\uc758 \uce74\ub124 6\uc778\uce58 \uc18d\uc0ac\ud3ec\ub85c \uc774\ub8e8\uc5b4\uc838 \uc788\uc5c8\ub2e4. \uc774\ub4e4 \uc911 8\ubb38\uc740 4\uac1c\uc758 \ud2b8\uc708 \uac74 \ud604\uce21 \ud3ec\ud0d1\uc5d0 \uc7a5\ucc29\ub418\uc5c8\uace0, \ub098\uba38\uc9c0 \ucd1d\uc740 \uc120\uccb4 \uc911\uc559\ubd80\uc758 \uc591\uce21 \ubcf4\ud638\ub418\uc9c0 \uc54a\uc740 \ucd1d\uc548\uc5d0 \ubc30\uce58\ub418\uc5c8\ub2e4. QF 1.9\uc778\uce58 \ud638\uce58\ud0a4\uc2a4\ud3ec 12\uac1c, \ub9e5\uc2ec 1.5\uc778\uce58 \uc18d\uc0ac\ud3ec 28\uac1c)\ub97c \ud3ec\ud568\ud55c \ub354 \uc791\uc740 \ucd1d\ub4e4\uc774 \uc5b4\ub8b0\uc815 \ubc29\uc5b4\ub97c \uc704\ud574 \ubc30\uce58\ub418\uc5c8\ub2e4. \ub610\ud55c 6\uac1c\uc758 \uc5b4\ub8b0\ubc1c\uc0ac\uad00\uc73c\ub85c \uc218\uc0c1\uc5d0 15\uc778\uce58 \ubc1c\uc0ac\uad00 4\uac1c\uc640 \ud604\uce21\ud3ec\uc5d0 \uc7a5\ucc29\ub41c 18\uc778\uce58 \uc218\uc911 \ubc1c\uc0ac\uad00 2\uac1c\ub85c \ubb34\uc7a5\ud588\ub2e4. \uc575\ucee4\ub9ac\uc9c0\ub97c \ubcf4\ud638\ud558\uae30 \uc704\ud574 50\uac1c\uc758 \uae30\ub8b0\ub3c4 \uc0ac\uc6a9\ub418\uc5c8\ub2e4.", "answer": "6\uc778\uce58", "sentence": "\ubd80\ud3ec\ub294 12\uac1c\uc758 \uce74\ub124 6\uc778\uce58 \uc18d\uc0ac\ud3ec\ub85c \uc774\ub8e8\uc5b4\uc838 \uc788\uc5c8\ub2e4.", "paragraph_sentence": "\ud398\ud2b8\ub85c\ud30c\ube14\ub86d\uc2a4\ud06c\uae09 \uc804\ud568\uc758 \uc8fc\ud3ec\ub294 2\uac1c\uc758 \ud2b8\uc708 \ud3ec\ud0d1\uc5d0 \uc7a5\ucc29\ub41c 4\ubb38\uc758 12\uc778\uce58\ud3ec\ub85c \uad6c\uc131\ub3c4\uc5b4 \uc788\uace0, \uc120\ub8e8\uc758 \uc55e\ub4a4\ub85c \ubc30\uce58\ud588\ub2e4. 90\ucd08\ub2f9 1\ubc1c\uc744 \ubc1c\uc0ac\ud558\ub3c4\ub85d \uc124\uacc4\ub418\uc5c8\uc9c0\ub9cc, \uc2e4\uc81c \ubc1c\uc0ac \uc18d\ub3c4\ub294 \uc808\ubc18\uc774 \uc18c\uc694\ub418\uc5c8\ub2e4. \ubd80\ud3ec\ub294 12\uac1c\uc758 \uce74\ub124 6\uc778\uce58 \uc18d\uc0ac\ud3ec\ub85c \uc774\ub8e8\uc5b4\uc838 \uc788\uc5c8\ub2e4. \uc774\ub4e4 \uc911 8\ubb38\uc740 4\uac1c\uc758 \ud2b8\uc708 \uac74 \ud604\uce21 \ud3ec\ud0d1\uc5d0 \uc7a5\ucc29\ub418\uc5c8\uace0, \ub098\uba38\uc9c0 \ucd1d\uc740 \uc120\uccb4 \uc911\uc559\ubd80\uc758 \uc591\uce21 \ubcf4\ud638\ub418\uc9c0 \uc54a\uc740 \ucd1d\uc548\uc5d0 \ubc30\uce58\ub418\uc5c8\ub2e4. QF 1.9\uc778\uce58 \ud638\uce58\ud0a4\uc2a4\ud3ec 12\uac1c, \ub9e5\uc2ec 1.5\uc778\uce58 \uc18d\uc0ac\ud3ec 28\uac1c)\ub97c \ud3ec\ud568\ud55c \ub354 \uc791\uc740 \ucd1d\ub4e4\uc774 \uc5b4\ub8b0\uc815 \ubc29\uc5b4\ub97c \uc704\ud574 \ubc30\uce58\ub418\uc5c8\ub2e4. \ub610\ud55c 6\uac1c\uc758 \uc5b4\ub8b0\ubc1c\uc0ac\uad00\uc73c\ub85c \uc218\uc0c1\uc5d0 15\uc778\uce58 \ubc1c\uc0ac\uad00 4\uac1c\uc640 \ud604\uce21\ud3ec\uc5d0 \uc7a5\ucc29\ub41c 18\uc778\uce58 \uc218\uc911 \ubc1c\uc0ac\uad00 2\uac1c\ub85c \ubb34\uc7a5\ud588\ub2e4. \uc575\ucee4\ub9ac\uc9c0\ub97c \ubcf4\ud638\ud558\uae30 \uc704\ud574 50\uac1c\uc758 \uae30\ub8b0\ub3c4 \uc0ac\uc6a9\ub418\uc5c8\ub2e4.", "paragraph_answer": "\ud398\ud2b8\ub85c\ud30c\ube14\ub86d\uc2a4\ud06c\uae09 \uc804\ud568\uc758 \uc8fc\ud3ec\ub294 2\uac1c\uc758 \ud2b8\uc708 \ud3ec\ud0d1\uc5d0 \uc7a5\ucc29\ub41c 4\ubb38\uc758 12\uc778\uce58\ud3ec\ub85c \uad6c\uc131\ub3c4\uc5b4 \uc788\uace0, \uc120\ub8e8\uc758 \uc55e\ub4a4\ub85c \ubc30\uce58\ud588\ub2e4. 90\ucd08\ub2f9 1\ubc1c\uc744 \ubc1c\uc0ac\ud558\ub3c4\ub85d \uc124\uacc4\ub418\uc5c8\uc9c0\ub9cc, \uc2e4\uc81c \ubc1c\uc0ac \uc18d\ub3c4\ub294 \uc808\ubc18\uc774 \uc18c\uc694\ub418\uc5c8\ub2e4. \ubd80\ud3ec\ub294 12\uac1c\uc758 \uce74\ub124 6\uc778\uce58 \uc18d\uc0ac\ud3ec\ub85c \uc774\ub8e8\uc5b4\uc838 \uc788\uc5c8\ub2e4. \uc774\ub4e4 \uc911 8\ubb38\uc740 4\uac1c\uc758 \ud2b8\uc708 \uac74 \ud604\uce21 \ud3ec\ud0d1\uc5d0 \uc7a5\ucc29\ub418\uc5c8\uace0, \ub098\uba38\uc9c0 \ucd1d\uc740 \uc120\uccb4 \uc911\uc559\ubd80\uc758 \uc591\uce21 \ubcf4\ud638\ub418\uc9c0 \uc54a\uc740 \ucd1d\uc548\uc5d0 \ubc30\uce58\ub418\uc5c8\ub2e4. QF 1.9\uc778\uce58 \ud638\uce58\ud0a4\uc2a4\ud3ec 12\uac1c, \ub9e5\uc2ec 1.5\uc778\uce58 \uc18d\uc0ac\ud3ec 28\uac1c)\ub97c \ud3ec\ud568\ud55c \ub354 \uc791\uc740 \ucd1d\ub4e4\uc774 \uc5b4\ub8b0\uc815 \ubc29\uc5b4\ub97c \uc704\ud574 \ubc30\uce58\ub418\uc5c8\ub2e4. \ub610\ud55c 6\uac1c\uc758 \uc5b4\ub8b0\ubc1c\uc0ac\uad00\uc73c\ub85c \uc218\uc0c1\uc5d0 15\uc778\uce58 \ubc1c\uc0ac\uad00 4\uac1c\uc640 \ud604\uce21\ud3ec\uc5d0 \uc7a5\ucc29\ub41c 18\uc778\uce58 \uc218\uc911 \ubc1c\uc0ac\uad00 2\uac1c\ub85c \ubb34\uc7a5\ud588\ub2e4. \uc575\ucee4\ub9ac\uc9c0\ub97c \ubcf4\ud638\ud558\uae30 \uc704\ud574 50\uac1c\uc758 \uae30\ub8b0\ub3c4 \uc0ac\uc6a9\ub418\uc5c8\ub2e4.", "sentence_answer": "\ubd80\ud3ec\ub294 12\uac1c\uc758 \uce74\ub124 6\uc778\uce58 \uc18d\uc0ac\ud3ec\ub85c \uc774\ub8e8\uc5b4\uc838 \uc788\uc5c8\ub2e4.", "paragraph_id": "6534100-0-0", "paragraph_question": "question: \ub7ec\uc2dc\uc544 \uc804\ud568 \ud3f4\ud0c0\ubc14\uc758 \ubd80\ud3ec\ub294 12\uac1c\uc758 \uce74\ub124 \uba87\uc778\uce58 \uc18d\uc0ac\ud3ec\ub85c \uc774\ub904\uc838 \uc788\uc2b5\ub2c8\uae4c?, context: \ud398\ud2b8\ub85c\ud30c\ube14\ub86d\uc2a4\ud06c\uae09 \uc804\ud568\uc758 \uc8fc\ud3ec\ub294 2\uac1c\uc758 \ud2b8\uc708 \ud3ec\ud0d1\uc5d0 \uc7a5\ucc29\ub41c 4\ubb38\uc758 12\uc778\uce58\ud3ec\ub85c \uad6c\uc131\ub3c4\uc5b4 \uc788\uace0, \uc120\ub8e8\uc758 \uc55e\ub4a4\ub85c \ubc30\uce58\ud588\ub2e4. 90\ucd08\ub2f9 1\ubc1c\uc744 \ubc1c\uc0ac\ud558\ub3c4\ub85d \uc124\uacc4\ub418\uc5c8\uc9c0\ub9cc, \uc2e4\uc81c \ubc1c\uc0ac \uc18d\ub3c4\ub294 \uc808\ubc18\uc774 \uc18c\uc694\ub418\uc5c8\ub2e4. \ubd80\ud3ec\ub294 12\uac1c\uc758 \uce74\ub124 6\uc778\uce58 \uc18d\uc0ac\ud3ec\ub85c \uc774\ub8e8\uc5b4\uc838 \uc788\uc5c8\ub2e4. \uc774\ub4e4 \uc911 8\ubb38\uc740 4\uac1c\uc758 \ud2b8\uc708 \uac74 \ud604\uce21 \ud3ec\ud0d1\uc5d0 \uc7a5\ucc29\ub418\uc5c8\uace0, \ub098\uba38\uc9c0 \ucd1d\uc740 \uc120\uccb4 \uc911\uc559\ubd80\uc758 \uc591\uce21 \ubcf4\ud638\ub418\uc9c0 \uc54a\uc740 \ucd1d\uc548\uc5d0 \ubc30\uce58\ub418\uc5c8\ub2e4. QF 1.9\uc778\uce58 \ud638\uce58\ud0a4\uc2a4\ud3ec 12\uac1c, \ub9e5\uc2ec 1.5\uc778\uce58 \uc18d\uc0ac\ud3ec 28\uac1c)\ub97c \ud3ec\ud568\ud55c \ub354 \uc791\uc740 \ucd1d\ub4e4\uc774 \uc5b4\ub8b0\uc815 \ubc29\uc5b4\ub97c \uc704\ud574 \ubc30\uce58\ub418\uc5c8\ub2e4. \ub610\ud55c 6\uac1c\uc758 \uc5b4\ub8b0\ubc1c\uc0ac\uad00\uc73c\ub85c \uc218\uc0c1\uc5d0 15\uc778\uce58 \ubc1c\uc0ac\uad00 4\uac1c\uc640 \ud604\uce21\ud3ec\uc5d0 \uc7a5\ucc29\ub41c 18\uc778\uce58 \uc218\uc911 \ubc1c\uc0ac\uad00 2\uac1c\ub85c \ubb34\uc7a5\ud588\ub2e4. \uc575\ucee4\ub9ac\uc9c0\ub97c \ubcf4\ud638\ud558\uae30 \uc704\ud574 50\uac1c\uc758 \uae30\ub8b0\ub3c4 \uc0ac\uc6a9\ub418\uc5c8\ub2e4."} {"question": "\uc57c\ub9c8\ubaa8\ub9ac \ub9c8\uc0ac\ud6c4\ubbf8\uc758 8\uc6d4 4\uc77c\uc758 \uc218\ube44 \ud50c\ub808\uc774\ub294 \uc720\ud29c\ube0c\uc5d0\uc11c \uc870\ud68c\uc218\uac00 \uc5bc\ub9c8\uc778\uac00?", "paragraph": "2\uc6d4\uc5d0 \uc5f4\ub9b0 \uc2a4\ud504\ub9c1 \ucea0\ud504\uc5d0\uc11c \uc67c\ucabd \ub113\uc801\ub2e4\ub9ac \ub4b7\uadfc\uc721 \ud1b5\uc99d\uc5d0 \uc758\ud55c \uc804\ub825 \uc774\ud0c8\ub85c 3\uad70\uc5d0\uc11c \uc7ac\ud65c\ud6c8\ub828\ud558\ub294 \ub4f1\uc758 \uc21c\ud0c4\ucc2e\uc740 \uc138\uc6d4\uc744 \ubcf4\ub0b4\uac8c \ub418\uc5c8\ub2e4. \uadf8\ub7ec\ub2e4\uac00 3\uc6d4 \uc911\uc5d0 \uc5f4\ub9b0 2\uad70 \uad50\uc721 \ub9ac\uadf8\uc5d0\uc11c \uc2e4\uc804\uc5d0 \ubcf5\uadc0\ud588\uace0 4\uc6d4 4\uc77c\uc5d0 1\uad70\uc5d0 \ub4f1\ub85d, 3\ud560\uc744 \ub118\uc744 \uc815\ub3c4\uc758 \uc2b9\ubd80\uac00 \uac15\ud55c \ud0c0\uaca9\uc73c\ub85c 3\ubc88 \ud0c0\uc790\ub85c\ub3c4 \uae30\uc6a9\ub418\ub294 \ub4f1 \ubd80\uc0c1\uc774\ub098 \uc7ac\ud65c \ud6c8\ub828\uc5d0 \uc758\ud574 \ucd9c\ubc1c\uc774 \ub2a6\uc5b4\uc9c0\uae34 \ud588\uc73c\ub098 \uc678\uc57c\uc218 \uacbd\uc7c1\uc5d0 \ucc38\uc5ec\ud588\ub2e4. \uadf8 \ud574\uc758 \ud558\uc774\ub77c\uc774\ud2b8\ub294 8\uc6d4 4\uc77c\uc5d0 \uc5f4\ub9b0 \uc694\ucf54\ud558\ub9c8 \ubca0\uc774\uc2a4\ud0c0\uc2a4\uc804\uc5d0\uc11c \ubb34\ub77c\ud0c0 \uc288\uc774\uce58\uac00 \ub54c\ub9b0 \uc88c\uc911\uac04 \ub2f4\uc7a5\uc73c\ub85c \uac70\uc758 \ub118\uc5b4\uac08 \ubed4\ud55c \ud648\ub7f0\uc131 \ud0c0\uad6c\ub97c \ud39c\uc2a4\uc5d0 \ud55c \uac78\uc74c \uae30\uc5b4\uc62c\ub77c\uac00 \ubcf4\uae30\uc88b\uac8c \uc7a1\uc544\ub0c8\ub2e4. \uc774 \uc218\ube44 \ud50c\ub808\uc774\ub294 \ubbf8\uad6d \uc57c\ud6c4!\uc758 \ud1b1 \ub274\uc2a4\ub85c \uc624\ub974\ub294 \ub4f1 \ud574\uc678 \uc5b8\ub860\ub4e4\ub85c\ubd80\ud130 \ud070 \uc8fc\ubaa9\uc744 \ubc1b\uc558\ub2e4. \ubbf8\uad6d CBS\ub294 \uc57c\ub9c8\ubaa8\ub9ac \ub9c8\uc0ac\ud6c4\ubbf8\ub97c \uc18c\uac1c\ud558\uba74\uc11c \u201c\uc77c\ubcf8 \uc57c\uad6c \uc5ed\uc0ac\uc0c1 \uac00\uc7a5 \ucda9\uaca9\uc801\uc778 \uce90\uce58\uc600\ub2e4\u201d\ub77c\uace0 \ud3c9\uac00\ud588\ub2e4. \ubbf8\uad6d \uc2a4\ud3ec\uce20 \uc804\ubb38 \ubc29\uc1a1 ESPN \uc5ed\uc2dc \uc804 \uc138\uacc4\uc758 \ubaa8\ub4e0 \uc2a4\ud3ec\uce20 \uacbd\uae30\uc5d0\uc11c \uc120\uc815\ub418\ub294 \u2018\uae08\uc8fc\uc758 \ud1b1 10\u2019\uc758 1\uc704\ub85c \uc120\uc815\ud574 \uc774\ub97c \ub300\ub300\uc801\uc73c\ub85c \ub2e4\ub8e8\uc5c8\ub2e4. \ub610 \uc778\ud130\ub137 \ub3d9\uc601\uc0c1 \uc0ac\uc774\ud2b8\uc778 \uc720\ud29c\ube0c\uc5d0\uc11c\ub3c4 \uad00\ub828 \ub3d9\uc601\uc0c1 \uc870\ud68c\uc218\uac00 100\ub9cc \uac74\uc744 \ub118\ub294 \ub4f1 \ud558\ub8fb\ubc24 \ub9cc\uc5d0 \uc77c\ubcf8 \uad6d\ub0b4\uc678\ub85c\ubd80\ud130 \uc5c4\uccad\ub09c \ubc18\ud5a5\uc744 \ubd88\ub7ec \uc77c\uc73c\ucf30\ub2e4.", "answer": "100\ub9cc \uac74", "sentence": "\uad00\ub828 \ub3d9\uc601\uc0c1 \uc870\ud68c\uc218\uac00 100\ub9cc \uac74 \uc744 \ub118\ub294 \ub4f1 \ud558\ub8fb\ubc24 \ub9cc\uc5d0 \uc77c\ubcf8 \uad6d\ub0b4\uc678\ub85c\ubd80\ud130 \uc5c4\uccad\ub09c \ubc18\ud5a5\uc744 \ubd88\ub7ec \uc77c\uc73c\ucf30\ub2e4.", "paragraph_sentence": "2\uc6d4\uc5d0 \uc5f4\ub9b0 \uc2a4\ud504\ub9c1 \ucea0\ud504\uc5d0\uc11c \uc67c\ucabd \ub113\uc801\ub2e4\ub9ac \ub4b7\uadfc\uc721 \ud1b5\uc99d\uc5d0 \uc758\ud55c \uc804\ub825 \uc774\ud0c8\ub85c 3\uad70\uc5d0\uc11c \uc7ac\ud65c\ud6c8\ub828\ud558\ub294 \ub4f1\uc758 \uc21c\ud0c4\ucc2e\uc740 \uc138\uc6d4\uc744 \ubcf4\ub0b4\uac8c \ub418\uc5c8\ub2e4. \uadf8\ub7ec\ub2e4\uac00 3\uc6d4 \uc911\uc5d0 \uc5f4\ub9b0 2\uad70 \uad50\uc721 \ub9ac\uadf8\uc5d0\uc11c \uc2e4\uc804\uc5d0 \ubcf5\uadc0\ud588\uace0 4\uc6d4 4\uc77c\uc5d0 1\uad70\uc5d0 \ub4f1\ub85d, 3\ud560\uc744 \ub118\uc744 \uc815\ub3c4\uc758 \uc2b9\ubd80\uac00 \uac15\ud55c \ud0c0\uaca9\uc73c\ub85c 3\ubc88 \ud0c0\uc790\ub85c\ub3c4 \uae30\uc6a9\ub418\ub294 \ub4f1 \ubd80\uc0c1\uc774\ub098 \uc7ac\ud65c \ud6c8\ub828\uc5d0 \uc758\ud574 \ucd9c\ubc1c\uc774 \ub2a6\uc5b4\uc9c0\uae34 \ud588\uc73c\ub098 \uc678\uc57c\uc218 \uacbd\uc7c1\uc5d0 \ucc38\uc5ec\ud588\ub2e4. \uadf8 \ud574\uc758 \ud558\uc774\ub77c\uc774\ud2b8\ub294 8\uc6d4 4\uc77c\uc5d0 \uc5f4\ub9b0 \uc694\ucf54\ud558\ub9c8 \ubca0\uc774\uc2a4\ud0c0\uc2a4\uc804\uc5d0\uc11c \ubb34\ub77c\ud0c0 \uc288\uc774\uce58\uac00 \ub54c\ub9b0 \uc88c\uc911\uac04 \ub2f4\uc7a5\uc73c\ub85c \uac70\uc758 \ub118\uc5b4\uac08 \ubed4\ud55c \ud648\ub7f0\uc131 \ud0c0\uad6c\ub97c \ud39c\uc2a4\uc5d0 \ud55c \uac78\uc74c \uae30\uc5b4\uc62c\ub77c\uac00 \ubcf4\uae30\uc88b\uac8c \uc7a1\uc544\ub0c8\ub2e4. \uc774 \uc218\ube44 \ud50c\ub808\uc774\ub294 \ubbf8\uad6d \uc57c\ud6c4!\uc758 \ud1b1 \ub274\uc2a4\ub85c \uc624\ub974\ub294 \ub4f1 \ud574\uc678 \uc5b8\ub860\ub4e4\ub85c\ubd80\ud130 \ud070 \uc8fc\ubaa9\uc744 \ubc1b\uc558\ub2e4. \ubbf8\uad6d CBS\ub294 \uc57c\ub9c8\ubaa8\ub9ac \ub9c8\uc0ac\ud6c4\ubbf8\ub97c \uc18c\uac1c\ud558\uba74\uc11c \u201c\uc77c\ubcf8 \uc57c\uad6c \uc5ed\uc0ac\uc0c1 \uac00\uc7a5 \ucda9\uaca9\uc801\uc778 \uce90\uce58\uc600\ub2e4\u201d\ub77c\uace0 \ud3c9\uac00\ud588\ub2e4. \ubbf8\uad6d \uc2a4\ud3ec\uce20 \uc804\ubb38 \ubc29\uc1a1 ESPN \uc5ed\uc2dc \uc804 \uc138\uacc4\uc758 \ubaa8\ub4e0 \uc2a4\ud3ec\uce20 \uacbd\uae30\uc5d0\uc11c \uc120\uc815\ub418\ub294 \u2018\uae08\uc8fc\uc758 \ud1b1 10\u2019\uc758 1\uc704\ub85c \uc120\uc815\ud574 \uc774\ub97c \ub300\ub300\uc801\uc73c\ub85c \ub2e4\ub8e8\uc5c8\ub2e4. \ub610 \uc778\ud130\ub137 \ub3d9\uc601\uc0c1 \uc0ac\uc774\ud2b8\uc778 \uc720\ud29c\ube0c\uc5d0\uc11c\ub3c4 \uad00\ub828 \ub3d9\uc601\uc0c1 \uc870\ud68c\uc218\uac00 100\ub9cc \uac74 \uc744 \ub118\ub294 \ub4f1 \ud558\ub8fb\ubc24 \ub9cc\uc5d0 \uc77c\ubcf8 \uad6d\ub0b4\uc678\ub85c\ubd80\ud130 \uc5c4\uccad\ub09c \ubc18\ud5a5\uc744 \ubd88\ub7ec \uc77c\uc73c\ucf30\ub2e4. ", "paragraph_answer": "2\uc6d4\uc5d0 \uc5f4\ub9b0 \uc2a4\ud504\ub9c1 \ucea0\ud504\uc5d0\uc11c \uc67c\ucabd \ub113\uc801\ub2e4\ub9ac \ub4b7\uadfc\uc721 \ud1b5\uc99d\uc5d0 \uc758\ud55c \uc804\ub825 \uc774\ud0c8\ub85c 3\uad70\uc5d0\uc11c \uc7ac\ud65c\ud6c8\ub828\ud558\ub294 \ub4f1\uc758 \uc21c\ud0c4\ucc2e\uc740 \uc138\uc6d4\uc744 \ubcf4\ub0b4\uac8c \ub418\uc5c8\ub2e4. \uadf8\ub7ec\ub2e4\uac00 3\uc6d4 \uc911\uc5d0 \uc5f4\ub9b0 2\uad70 \uad50\uc721 \ub9ac\uadf8\uc5d0\uc11c \uc2e4\uc804\uc5d0 \ubcf5\uadc0\ud588\uace0 4\uc6d4 4\uc77c\uc5d0 1\uad70\uc5d0 \ub4f1\ub85d, 3\ud560\uc744 \ub118\uc744 \uc815\ub3c4\uc758 \uc2b9\ubd80\uac00 \uac15\ud55c \ud0c0\uaca9\uc73c\ub85c 3\ubc88 \ud0c0\uc790\ub85c\ub3c4 \uae30\uc6a9\ub418\ub294 \ub4f1 \ubd80\uc0c1\uc774\ub098 \uc7ac\ud65c \ud6c8\ub828\uc5d0 \uc758\ud574 \ucd9c\ubc1c\uc774 \ub2a6\uc5b4\uc9c0\uae34 \ud588\uc73c\ub098 \uc678\uc57c\uc218 \uacbd\uc7c1\uc5d0 \ucc38\uc5ec\ud588\ub2e4. \uadf8 \ud574\uc758 \ud558\uc774\ub77c\uc774\ud2b8\ub294 8\uc6d4 4\uc77c\uc5d0 \uc5f4\ub9b0 \uc694\ucf54\ud558\ub9c8 \ubca0\uc774\uc2a4\ud0c0\uc2a4\uc804\uc5d0\uc11c \ubb34\ub77c\ud0c0 \uc288\uc774\uce58\uac00 \ub54c\ub9b0 \uc88c\uc911\uac04 \ub2f4\uc7a5\uc73c\ub85c \uac70\uc758 \ub118\uc5b4\uac08 \ubed4\ud55c \ud648\ub7f0\uc131 \ud0c0\uad6c\ub97c \ud39c\uc2a4\uc5d0 \ud55c \uac78\uc74c \uae30\uc5b4\uc62c\ub77c\uac00 \ubcf4\uae30\uc88b\uac8c \uc7a1\uc544\ub0c8\ub2e4. \uc774 \uc218\ube44 \ud50c\ub808\uc774\ub294 \ubbf8\uad6d \uc57c\ud6c4!\uc758 \ud1b1 \ub274\uc2a4\ub85c \uc624\ub974\ub294 \ub4f1 \ud574\uc678 \uc5b8\ub860\ub4e4\ub85c\ubd80\ud130 \ud070 \uc8fc\ubaa9\uc744 \ubc1b\uc558\ub2e4. \ubbf8\uad6d CBS\ub294 \uc57c\ub9c8\ubaa8\ub9ac \ub9c8\uc0ac\ud6c4\ubbf8\ub97c \uc18c\uac1c\ud558\uba74\uc11c \u201c\uc77c\ubcf8 \uc57c\uad6c \uc5ed\uc0ac\uc0c1 \uac00\uc7a5 \ucda9\uaca9\uc801\uc778 \uce90\uce58\uc600\ub2e4\u201d\ub77c\uace0 \ud3c9\uac00\ud588\ub2e4. \ubbf8\uad6d \uc2a4\ud3ec\uce20 \uc804\ubb38 \ubc29\uc1a1 ESPN \uc5ed\uc2dc \uc804 \uc138\uacc4\uc758 \ubaa8\ub4e0 \uc2a4\ud3ec\uce20 \uacbd\uae30\uc5d0\uc11c \uc120\uc815\ub418\ub294 \u2018\uae08\uc8fc\uc758 \ud1b1 10\u2019\uc758 1\uc704\ub85c \uc120\uc815\ud574 \uc774\ub97c \ub300\ub300\uc801\uc73c\ub85c \ub2e4\ub8e8\uc5c8\ub2e4. \ub610 \uc778\ud130\ub137 \ub3d9\uc601\uc0c1 \uc0ac\uc774\ud2b8\uc778 \uc720\ud29c\ube0c\uc5d0\uc11c\ub3c4 \uad00\ub828 \ub3d9\uc601\uc0c1 \uc870\ud68c\uc218\uac00 100\ub9cc \uac74 \uc744 \ub118\ub294 \ub4f1 \ud558\ub8fb\ubc24 \ub9cc\uc5d0 \uc77c\ubcf8 \uad6d\ub0b4\uc678\ub85c\ubd80\ud130 \uc5c4\uccad\ub09c \ubc18\ud5a5\uc744 \ubd88\ub7ec \uc77c\uc73c\ucf30\ub2e4.", "sentence_answer": "\uad00\ub828 \ub3d9\uc601\uc0c1 \uc870\ud68c\uc218\uac00 100\ub9cc \uac74 \uc744 \ub118\ub294 \ub4f1 \ud558\ub8fb\ubc24 \ub9cc\uc5d0 \uc77c\ubcf8 \uad6d\ub0b4\uc678\ub85c\ubd80\ud130 \uc5c4\uccad\ub09c \ubc18\ud5a5\uc744 \ubd88\ub7ec \uc77c\uc73c\ucf30\ub2e4.", "paragraph_id": "6538696-2-0", "paragraph_question": "question: \uc57c\ub9c8\ubaa8\ub9ac \ub9c8\uc0ac\ud6c4\ubbf8\uc758 8\uc6d4 4\uc77c\uc758 \uc218\ube44 \ud50c\ub808\uc774\ub294 \uc720\ud29c\ube0c\uc5d0\uc11c \uc870\ud68c\uc218\uac00 \uc5bc\ub9c8\uc778\uac00?, context: 2\uc6d4\uc5d0 \uc5f4\ub9b0 \uc2a4\ud504\ub9c1 \ucea0\ud504\uc5d0\uc11c \uc67c\ucabd \ub113\uc801\ub2e4\ub9ac \ub4b7\uadfc\uc721 \ud1b5\uc99d\uc5d0 \uc758\ud55c \uc804\ub825 \uc774\ud0c8\ub85c 3\uad70\uc5d0\uc11c \uc7ac\ud65c\ud6c8\ub828\ud558\ub294 \ub4f1\uc758 \uc21c\ud0c4\ucc2e\uc740 \uc138\uc6d4\uc744 \ubcf4\ub0b4\uac8c \ub418\uc5c8\ub2e4. \uadf8\ub7ec\ub2e4\uac00 3\uc6d4 \uc911\uc5d0 \uc5f4\ub9b0 2\uad70 \uad50\uc721 \ub9ac\uadf8\uc5d0\uc11c \uc2e4\uc804\uc5d0 \ubcf5\uadc0\ud588\uace0 4\uc6d4 4\uc77c\uc5d0 1\uad70\uc5d0 \ub4f1\ub85d, 3\ud560\uc744 \ub118\uc744 \uc815\ub3c4\uc758 \uc2b9\ubd80\uac00 \uac15\ud55c \ud0c0\uaca9\uc73c\ub85c 3\ubc88 \ud0c0\uc790\ub85c\ub3c4 \uae30\uc6a9\ub418\ub294 \ub4f1 \ubd80\uc0c1\uc774\ub098 \uc7ac\ud65c \ud6c8\ub828\uc5d0 \uc758\ud574 \ucd9c\ubc1c\uc774 \ub2a6\uc5b4\uc9c0\uae34 \ud588\uc73c\ub098 \uc678\uc57c\uc218 \uacbd\uc7c1\uc5d0 \ucc38\uc5ec\ud588\ub2e4. \uadf8 \ud574\uc758 \ud558\uc774\ub77c\uc774\ud2b8\ub294 8\uc6d4 4\uc77c\uc5d0 \uc5f4\ub9b0 \uc694\ucf54\ud558\ub9c8 \ubca0\uc774\uc2a4\ud0c0\uc2a4\uc804\uc5d0\uc11c \ubb34\ub77c\ud0c0 \uc288\uc774\uce58\uac00 \ub54c\ub9b0 \uc88c\uc911\uac04 \ub2f4\uc7a5\uc73c\ub85c \uac70\uc758 \ub118\uc5b4\uac08 \ubed4\ud55c \ud648\ub7f0\uc131 \ud0c0\uad6c\ub97c \ud39c\uc2a4\uc5d0 \ud55c \uac78\uc74c \uae30\uc5b4\uc62c\ub77c\uac00 \ubcf4\uae30\uc88b\uac8c \uc7a1\uc544\ub0c8\ub2e4. \uc774 \uc218\ube44 \ud50c\ub808\uc774\ub294 \ubbf8\uad6d \uc57c\ud6c4!\uc758 \ud1b1 \ub274\uc2a4\ub85c \uc624\ub974\ub294 \ub4f1 \ud574\uc678 \uc5b8\ub860\ub4e4\ub85c\ubd80\ud130 \ud070 \uc8fc\ubaa9\uc744 \ubc1b\uc558\ub2e4. \ubbf8\uad6d CBS\ub294 \uc57c\ub9c8\ubaa8\ub9ac \ub9c8\uc0ac\ud6c4\ubbf8\ub97c \uc18c\uac1c\ud558\uba74\uc11c \u201c\uc77c\ubcf8 \uc57c\uad6c \uc5ed\uc0ac\uc0c1 \uac00\uc7a5 \ucda9\uaca9\uc801\uc778 \uce90\uce58\uc600\ub2e4\u201d\ub77c\uace0 \ud3c9\uac00\ud588\ub2e4. \ubbf8\uad6d \uc2a4\ud3ec\uce20 \uc804\ubb38 \ubc29\uc1a1 ESPN \uc5ed\uc2dc \uc804 \uc138\uacc4\uc758 \ubaa8\ub4e0 \uc2a4\ud3ec\uce20 \uacbd\uae30\uc5d0\uc11c \uc120\uc815\ub418\ub294 \u2018\uae08\uc8fc\uc758 \ud1b1 10\u2019\uc758 1\uc704\ub85c \uc120\uc815\ud574 \uc774\ub97c \ub300\ub300\uc801\uc73c\ub85c \ub2e4\ub8e8\uc5c8\ub2e4. \ub610 \uc778\ud130\ub137 \ub3d9\uc601\uc0c1 \uc0ac\uc774\ud2b8\uc778 \uc720\ud29c\ube0c\uc5d0\uc11c\ub3c4 \uad00\ub828 \ub3d9\uc601\uc0c1 \uc870\ud68c\uc218\uac00 100\ub9cc \uac74\uc744 \ub118\ub294 \ub4f1 \ud558\ub8fb\ubc24 \ub9cc\uc5d0 \uc77c\ubcf8 \uad6d\ub0b4\uc678\ub85c\ubd80\ud130 \uc5c4\uccad\ub09c \ubc18\ud5a5\uc744 \ubd88\ub7ec \uc77c\uc73c\ucf30\ub2e4."} {"question": "\uac8c\uc784 \ud558\ud504\ub77c\uc774\ud504\uc5d0 \ub4f1\uc7a5\ud558\ub294 \uc791\uc911 \ud68c\uc0ac\uc778 \uc560\ud37c\ucc98 \uc0ac\uc774\uc5b8\uc2a4\uc758 \uacbd\uc7c1 \ud68c\uc0ac\ub294?", "paragraph": "\ubcf4\ub9ac\uc54c\ub9ac\uc2a4 \ud638\ub294 \uc560\ud37c\ucc98 \uc0ac\uc774\uc5b8\uc2a4\uc758 \uc5f0\uad6c \uc120\ubc15\uc73c\ub85c, \ud558\ud504\ub77c\uc774\ud504 2: \uc5d0\ud53c\uc18c\ub4dc 2\uc640 \ud3ec\ud0c8 2\uc5d0\uc11c \uc5b8\uae09\ub418\ub294 \uc874\uc7ac\ub2e4. \uc560\ud37c\ucc98 \uc0ac\uc774\uc5b8\uc2a4\ub294 \uc815\ubd80 \uc9c0\uc6d0\uae08\uc744 \uac00\uc9c0\uace0 \ube14\ub799 \uba54\uc0ac\uc640 \uacbd\uc7c1\uc744 \ud558\ub2e4\uac00 \ubcf4\ub9ac\uc54c\ub9ac\uc2a4 \ud638\ub97c \ud154\ub808\ud3ec\ud2b8\uc2dc\ucf1c\ubc84\ub9ac\ub294 \uc0ac\uace0\ub97c \uc77c\uc73c\ucf30\uace0, \uadf8\ub807\uac8c \ubcf4\ub9ac\uc54c\ub9ac\uc2a4 \ud638\ub294 \uac11\uc791\uc2a4\ub808 \uc0ac\ub77c\uc838 \uacfc\ud559\uacc4\uc5d0\uc11c \uc804\uc124\ub85c \ub0a8\uac8c \ub418\uc5c8\ub2e4. \ud558\ud504 \ub77c\uc774\ud504 2: \uc5d0\ud53c\uc18c\ub4dc 2\uc758 \ud6c4\ubc18\ubd80\uc5d0\uc11c\ub294 \uc0ac\ub77c\uc9c4 \ubcf4\ub9ac\uc54c\ub9ac\uc2a4 \ud638\uac00 \ubd81\uadf9\uc5d0\uc11c \ubc1c\uacac\ub418\uc5c8\ub2e4\ub294 \uac83\uc774 \ubc1d\ud600\uc9c0\uba70, \ud558\ud504\ub77c\uc774\ud504 \uc2dc\ub9ac\uc988\uc5d0 \ub4f1\uc7a5\ud558\ub294 \uac00\uc0c1\uc758 \uce90\ub9ad\ud130\uc778 \uc77c\ub77c\uc774 \ubc34\uc2a4\uc640 \uc544\uc774\uc791 \ud074\ub77c\uc774\ub108\ub294 \uc774 \uc120\ubc15 \uc548\uc5d0 \uc788\ub294 \uc54c \uc218 \uc5c6\ub294 \uae30\uc220\uc5d0 \ub300\ud574 \uc5c6\uc560\ubc84\ub824\uc57c \ud558\ub294\uc9c0 \uc774\uc6a9\ud574\uc57c \ud558\ub294\uc9c0\uc5d0 \ub300\ud574 \ub17c\uc7c1\uc744 \ubc8c\uc778 \uc801\uc774 \uc788\ub2e4. \ub610\ud55c \ud3ec\ud0c8 2\uc5d0\uc11c\ub294 \ubcf4\ub9ac\uc54c\ub9ac\uc2a4 \ud638\ub77c\uace0 \uc801\ud600 \uc788\ub294 \ud29c\ube0c\ub97c \ubc1c\uacac\ud560 \uc218 \uc788\uc73c\uba70, \uc774\uc5d0 \ub300\ud55c \ub3c4\uc804\uacfc\uc81c\uac00 \uc874\uc7ac\ud55c\ub2e4.", "answer": "\ube14\ub799 \uba54\uc0ac\uc640", "sentence": "\uc560\ud37c\ucc98 \uc0ac\uc774\uc5b8\uc2a4\ub294 \uc815\ubd80 \uc9c0\uc6d0\uae08\uc744 \uac00\uc9c0\uace0 \ube14\ub799 \uba54\uc0ac\uc640 \uacbd\uc7c1\uc744 \ud558\ub2e4\uac00 \ubcf4\ub9ac\uc54c\ub9ac\uc2a4 \ud638\ub97c \ud154\ub808\ud3ec\ud2b8\uc2dc\ucf1c\ubc84\ub9ac\ub294 \uc0ac\uace0\ub97c \uc77c\uc73c\ucf30\uace0, \uadf8\ub807\uac8c \ubcf4\ub9ac\uc54c\ub9ac\uc2a4 \ud638\ub294 \uac11\uc791\uc2a4\ub808 \uc0ac\ub77c\uc838 \uacfc\ud559\uacc4\uc5d0\uc11c \uc804\uc124\ub85c \ub0a8\uac8c \ub418\uc5c8\ub2e4.", "paragraph_sentence": "\ubcf4\ub9ac\uc54c\ub9ac\uc2a4 \ud638\ub294 \uc560\ud37c\ucc98 \uc0ac\uc774\uc5b8\uc2a4\uc758 \uc5f0\uad6c \uc120\ubc15\uc73c\ub85c, \ud558\ud504\ub77c\uc774\ud504 2: \uc5d0\ud53c\uc18c\ub4dc 2\uc640 \ud3ec\ud0c8 2\uc5d0\uc11c \uc5b8\uae09\ub418\ub294 \uc874\uc7ac\ub2e4. \uc560\ud37c\ucc98 \uc0ac\uc774\uc5b8\uc2a4\ub294 \uc815\ubd80 \uc9c0\uc6d0\uae08\uc744 \uac00\uc9c0\uace0 \ube14\ub799 \uba54\uc0ac\uc640 \uacbd\uc7c1\uc744 \ud558\ub2e4\uac00 \ubcf4\ub9ac\uc54c\ub9ac\uc2a4 \ud638\ub97c \ud154\ub808\ud3ec\ud2b8\uc2dc\ucf1c\ubc84\ub9ac\ub294 \uc0ac\uace0\ub97c \uc77c\uc73c\ucf30\uace0, \uadf8\ub807\uac8c \ubcf4\ub9ac\uc54c\ub9ac\uc2a4 \ud638\ub294 \uac11\uc791\uc2a4\ub808 \uc0ac\ub77c\uc838 \uacfc\ud559\uacc4\uc5d0\uc11c \uc804\uc124\ub85c \ub0a8\uac8c \ub418\uc5c8\ub2e4. \ud558\ud504 \ub77c\uc774\ud504 2: \uc5d0\ud53c\uc18c\ub4dc 2\uc758 \ud6c4\ubc18\ubd80\uc5d0\uc11c\ub294 \uc0ac\ub77c\uc9c4 \ubcf4\ub9ac\uc54c\ub9ac\uc2a4 \ud638\uac00 \ubd81\uadf9\uc5d0\uc11c \ubc1c\uacac\ub418\uc5c8\ub2e4\ub294 \uac83\uc774 \ubc1d\ud600\uc9c0\uba70, \ud558\ud504\ub77c\uc774\ud504 \uc2dc\ub9ac\uc988\uc5d0 \ub4f1\uc7a5\ud558\ub294 \uac00\uc0c1\uc758 \uce90\ub9ad\ud130\uc778 \uc77c\ub77c\uc774 \ubc34\uc2a4\uc640 \uc544\uc774\uc791 \ud074\ub77c\uc774\ub108\ub294 \uc774 \uc120\ubc15 \uc548\uc5d0 \uc788\ub294 \uc54c \uc218 \uc5c6\ub294 \uae30\uc220\uc5d0 \ub300\ud574 \uc5c6\uc560\ubc84\ub824\uc57c \ud558\ub294\uc9c0 \uc774\uc6a9\ud574\uc57c \ud558\ub294\uc9c0\uc5d0 \ub300\ud574 \ub17c\uc7c1\uc744 \ubc8c\uc778 \uc801\uc774 \uc788\ub2e4. \ub610\ud55c \ud3ec\ud0c8 2\uc5d0\uc11c\ub294 \ubcf4\ub9ac\uc54c\ub9ac\uc2a4 \ud638\ub77c\uace0 \uc801\ud600 \uc788\ub294 \ud29c\ube0c\ub97c \ubc1c\uacac\ud560 \uc218 \uc788\uc73c\uba70, \uc774\uc5d0 \ub300\ud55c \ub3c4\uc804\uacfc\uc81c\uac00 \uc874\uc7ac\ud55c\ub2e4.", "paragraph_answer": "\ubcf4\ub9ac\uc54c\ub9ac\uc2a4 \ud638\ub294 \uc560\ud37c\ucc98 \uc0ac\uc774\uc5b8\uc2a4\uc758 \uc5f0\uad6c \uc120\ubc15\uc73c\ub85c, \ud558\ud504\ub77c\uc774\ud504 2: \uc5d0\ud53c\uc18c\ub4dc 2\uc640 \ud3ec\ud0c8 2\uc5d0\uc11c \uc5b8\uae09\ub418\ub294 \uc874\uc7ac\ub2e4. \uc560\ud37c\ucc98 \uc0ac\uc774\uc5b8\uc2a4\ub294 \uc815\ubd80 \uc9c0\uc6d0\uae08\uc744 \uac00\uc9c0\uace0 \ube14\ub799 \uba54\uc0ac\uc640 \uacbd\uc7c1\uc744 \ud558\ub2e4\uac00 \ubcf4\ub9ac\uc54c\ub9ac\uc2a4 \ud638\ub97c \ud154\ub808\ud3ec\ud2b8\uc2dc\ucf1c\ubc84\ub9ac\ub294 \uc0ac\uace0\ub97c \uc77c\uc73c\ucf30\uace0, \uadf8\ub807\uac8c \ubcf4\ub9ac\uc54c\ub9ac\uc2a4 \ud638\ub294 \uac11\uc791\uc2a4\ub808 \uc0ac\ub77c\uc838 \uacfc\ud559\uacc4\uc5d0\uc11c \uc804\uc124\ub85c \ub0a8\uac8c \ub418\uc5c8\ub2e4. \ud558\ud504 \ub77c\uc774\ud504 2: \uc5d0\ud53c\uc18c\ub4dc 2\uc758 \ud6c4\ubc18\ubd80\uc5d0\uc11c\ub294 \uc0ac\ub77c\uc9c4 \ubcf4\ub9ac\uc54c\ub9ac\uc2a4 \ud638\uac00 \ubd81\uadf9\uc5d0\uc11c \ubc1c\uacac\ub418\uc5c8\ub2e4\ub294 \uac83\uc774 \ubc1d\ud600\uc9c0\uba70, \ud558\ud504\ub77c\uc774\ud504 \uc2dc\ub9ac\uc988\uc5d0 \ub4f1\uc7a5\ud558\ub294 \uac00\uc0c1\uc758 \uce90\ub9ad\ud130\uc778 \uc77c\ub77c\uc774 \ubc34\uc2a4\uc640 \uc544\uc774\uc791 \ud074\ub77c\uc774\ub108\ub294 \uc774 \uc120\ubc15 \uc548\uc5d0 \uc788\ub294 \uc54c \uc218 \uc5c6\ub294 \uae30\uc220\uc5d0 \ub300\ud574 \uc5c6\uc560\ubc84\ub824\uc57c \ud558\ub294\uc9c0 \uc774\uc6a9\ud574\uc57c \ud558\ub294\uc9c0\uc5d0 \ub300\ud574 \ub17c\uc7c1\uc744 \ubc8c\uc778 \uc801\uc774 \uc788\ub2e4. \ub610\ud55c \ud3ec\ud0c8 2\uc5d0\uc11c\ub294 \ubcf4\ub9ac\uc54c\ub9ac\uc2a4 \ud638\ub77c\uace0 \uc801\ud600 \uc788\ub294 \ud29c\ube0c\ub97c \ubc1c\uacac\ud560 \uc218 \uc788\uc73c\uba70, \uc774\uc5d0 \ub300\ud55c \ub3c4\uc804\uacfc\uc81c\uac00 \uc874\uc7ac\ud55c\ub2e4.", "sentence_answer": "\uc560\ud37c\ucc98 \uc0ac\uc774\uc5b8\uc2a4\ub294 \uc815\ubd80 \uc9c0\uc6d0\uae08\uc744 \uac00\uc9c0\uace0 \ube14\ub799 \uba54\uc0ac\uc640 \uacbd\uc7c1\uc744 \ud558\ub2e4\uac00 \ubcf4\ub9ac\uc54c\ub9ac\uc2a4 \ud638\ub97c \ud154\ub808\ud3ec\ud2b8\uc2dc\ucf1c\ubc84\ub9ac\ub294 \uc0ac\uace0\ub97c \uc77c\uc73c\ucf30\uace0, \uadf8\ub807\uac8c \ubcf4\ub9ac\uc54c\ub9ac\uc2a4 \ud638\ub294 \uac11\uc791\uc2a4\ub808 \uc0ac\ub77c\uc838 \uacfc\ud559\uacc4\uc5d0\uc11c \uc804\uc124\ub85c \ub0a8\uac8c \ub418\uc5c8\ub2e4.", "paragraph_id": "6540211-1-1", "paragraph_question": "question: \uac8c\uc784 \ud558\ud504\ub77c\uc774\ud504\uc5d0 \ub4f1\uc7a5\ud558\ub294 \uc791\uc911 \ud68c\uc0ac\uc778 \uc560\ud37c\ucc98 \uc0ac\uc774\uc5b8\uc2a4\uc758 \uacbd\uc7c1 \ud68c\uc0ac\ub294?, context: \ubcf4\ub9ac\uc54c\ub9ac\uc2a4 \ud638\ub294 \uc560\ud37c\ucc98 \uc0ac\uc774\uc5b8\uc2a4\uc758 \uc5f0\uad6c \uc120\ubc15\uc73c\ub85c, \ud558\ud504\ub77c\uc774\ud504 2: \uc5d0\ud53c\uc18c\ub4dc 2\uc640 \ud3ec\ud0c8 2\uc5d0\uc11c \uc5b8\uae09\ub418\ub294 \uc874\uc7ac\ub2e4. \uc560\ud37c\ucc98 \uc0ac\uc774\uc5b8\uc2a4\ub294 \uc815\ubd80 \uc9c0\uc6d0\uae08\uc744 \uac00\uc9c0\uace0 \ube14\ub799 \uba54\uc0ac\uc640 \uacbd\uc7c1\uc744 \ud558\ub2e4\uac00 \ubcf4\ub9ac\uc54c\ub9ac\uc2a4 \ud638\ub97c \ud154\ub808\ud3ec\ud2b8\uc2dc\ucf1c\ubc84\ub9ac\ub294 \uc0ac\uace0\ub97c \uc77c\uc73c\ucf30\uace0, \uadf8\ub807\uac8c \ubcf4\ub9ac\uc54c\ub9ac\uc2a4 \ud638\ub294 \uac11\uc791\uc2a4\ub808 \uc0ac\ub77c\uc838 \uacfc\ud559\uacc4\uc5d0\uc11c \uc804\uc124\ub85c \ub0a8\uac8c \ub418\uc5c8\ub2e4. \ud558\ud504 \ub77c\uc774\ud504 2: \uc5d0\ud53c\uc18c\ub4dc 2\uc758 \ud6c4\ubc18\ubd80\uc5d0\uc11c\ub294 \uc0ac\ub77c\uc9c4 \ubcf4\ub9ac\uc54c\ub9ac\uc2a4 \ud638\uac00 \ubd81\uadf9\uc5d0\uc11c \ubc1c\uacac\ub418\uc5c8\ub2e4\ub294 \uac83\uc774 \ubc1d\ud600\uc9c0\uba70, \ud558\ud504\ub77c\uc774\ud504 \uc2dc\ub9ac\uc988\uc5d0 \ub4f1\uc7a5\ud558\ub294 \uac00\uc0c1\uc758 \uce90\ub9ad\ud130\uc778 \uc77c\ub77c\uc774 \ubc34\uc2a4\uc640 \uc544\uc774\uc791 \ud074\ub77c\uc774\ub108\ub294 \uc774 \uc120\ubc15 \uc548\uc5d0 \uc788\ub294 \uc54c \uc218 \uc5c6\ub294 \uae30\uc220\uc5d0 \ub300\ud574 \uc5c6\uc560\ubc84\ub824\uc57c \ud558\ub294\uc9c0 \uc774\uc6a9\ud574\uc57c \ud558\ub294\uc9c0\uc5d0 \ub300\ud574 \ub17c\uc7c1\uc744 \ubc8c\uc778 \uc801\uc774 \uc788\ub2e4. \ub610\ud55c \ud3ec\ud0c8 2\uc5d0\uc11c\ub294 \ubcf4\ub9ac\uc54c\ub9ac\uc2a4 \ud638\ub77c\uace0 \uc801\ud600 \uc788\ub294 \ud29c\ube0c\ub97c \ubc1c\uacac\ud560 \uc218 \uc788\uc73c\uba70, \uc774\uc5d0 \ub300\ud55c \ub3c4\uc804\uacfc\uc81c\uac00 \uc874\uc7ac\ud55c\ub2e4."} {"question": "11\uc6d4\uc5d0 \ubca0\uc774\ube44\ubcf5\uc2a4\uac00 \uc911\uad6d\uc5d0\uc11c \ud3d0\ub9c9\uc2dd \uacf5\uc5f0\uc5d0 \ucc38\uc5ec\ud55c \ubc15\ub78c\ud68c \uc774\ub984\uc740?", "paragraph": "8\uc6d4\uc5d0\ub294 \uc77c\ubcf8 \ub098\uce74\uc624\uce74 \ud6c4\uc0dd\ud68c\uad00\uc5d0\uc11c \uc5f4\ub9b0 \u20182004\ub2c8\uac00\ud0c0-\uc11c\uc6b8 \uc0b0\uc5c5 \uad00\uad11 \ubb38\ud654\uad50\ub958\uc0ac\uc5c5\uc804\u2019 \uc804\uc57c\uc81c \ubb34\ub300\uc5d0 \uc11c\uae30 \uc704\ud574 \uc77c\ubcf8\uc744 \ubc29\ubb38\ud588\uace0 \ub3c4\ucfc4\uc758 \ud640\ub9ac\ub370\uc774\ud638\ud154\uc5d0\uc11c \uae30\uc790\ud68c\uacac\uacfc \ubbf8\ub2c8 \ub77c\uc774\ube0c \ubb34\ub300\ub97c \uc120\ubcf4\uc600\uc73c\uba70 \uc77c\ubcf8 \uc5b8\ub860\ub4e4\uc740 \ubca0\uc774\ube44\ubcf5\uc2a4\uc758 \uc77c\ubcf8 \ubc29\ubb38\uc744 \uc77c\uc81c\ud788 \ubcf4\ub3c4\ud588\ub2e4.11\uc6d4\uc5d0\ub294 \uc6d0\uc790\ubc14\uc624\ucd1d\ub9ac\uc55e\uc5d0\uc11c \uc911\uad6d \ub0a8\ub155\uc5d0\uc11c \uc5f4\ub9ac\ub294 `\uc81c1\ud68c CAFAIR(\ub3d9\uc544\uc2dc\uc544\uc5f0\ub9f9) \ubc15\ub78c\ud68c` \ud3d0\ub9c9\uc2dd \uacf5\uc5f0\uc5d0 \ucc38\uc11d\ud574 \uc5f4\ucc3d\uc758 \ubb34\ub300\ub97c \uafb8\uba84\uace0 \uc774 \ud589\uc0ac\uc758 \ucc38\uc5ec\ub294 \uacf5\uc5f0\uc744 \uc8fc\uad00\ud558\ub294 CCTV \uad00\uacc4\uc790\uc758 \uc694\uccad\uc5d0 \ub530\ub978 \uac83\uc73c\ub85c \ud604\uc9c0\uc5d0\uc11c\uc758 \ubca0\uc774\ube44\ubcf5\uc2a4 \uc778\uae30\ub97c \uadf8\ub300\ub85c \ubc18\uc99d\uc2dc\ucf30\ub2e4.\uac00\uc218 \ube44\uc640 12\uc6d4\ubd80\ud130 \uc911\uad6d \ubca0\uc774\uc9d5\uc5d0\uc11c \ub300\uaddc\ubaa8 \ud569\ub3d9 \ucf58\uc11c\ud2b8\ub97c \uac16\uae30\ub3c4 \ud588\ub294\ub370 \uc774 \uacf5\uc5f0\uc740 \uc774\ubca4\ud2b8\uc131 \ud589\uc0ac\uac00 \uc544\ub2cc \uc720\ub8cc \uacf5\uc5f0\uc73c\ub85c\ub294 2002\ub144 \ubca0\uc774\ube44\ubcf5\uc2a4 \ub2e8\ub3c5 \ucf58\uc11c\ud2b8 \uc774\ud6c4 2\ub144 \ub9cc\uc5d0 \ucc98\uc74c\uc73c\ub85c \uad6d\ub0b4 \uac00\uc218\ub85c\uc11c \uc774\ub840\uc801\uc778 \uacf5\uc5f0 \ud5c8\uac00\ub97c \ubc1b\uc558\uc73c\uba70, \uc8fc\ucd5c\uce21\uc778 Vnet midas\uc640 DR china\ub294 \uc774 \uacf5\uc5f0\uc744 \uc8fc\ucd5c\ud558\uae30 \uc704\ud574 \uc911\uad6d \uc815\ubd80\uc640 6\uac1c\uc6d4\uac04\uc758 \uad50\uc12d \ub05d\uc5d0 \uc5b4\ub835\uac8c \uacf5\uc5f0 \ube44\uc900\uc744 \ubc1b\uc544 \uc808\ucc28\uac00 \uae4c\ub2e4\ub85c\uc6b4 \uc911\uad6d\uacf5\uc5f0\uc744 \uc131\uc0ac\uc2dc\ucf30\ub2e4.", "answer": "CAFAIR", "sentence": "8\uc6d4\uc5d0\ub294 \uc77c\ubcf8 \ub098\uce74\uc624\uce74 \ud6c4\uc0dd\ud68c\uad00\uc5d0\uc11c \uc5f4\ub9b0 \u20182004\ub2c8\uac00\ud0c0-\uc11c\uc6b8 \uc0b0\uc5c5 \uad00\uad11 \ubb38\ud654\uad50\ub958\uc0ac\uc5c5\uc804\u2019 \uc804\uc57c\uc81c \ubb34\ub300\uc5d0 \uc11c\uae30 \uc704\ud574 \uc77c\ubcf8\uc744 \ubc29\ubb38\ud588\uace0 \ub3c4\ucfc4\uc758 \ud640\ub9ac\ub370\uc774\ud638\ud154\uc5d0\uc11c \uae30\uc790\ud68c\uacac\uacfc \ubbf8\ub2c8 \ub77c\uc774\ube0c \ubb34\ub300\ub97c \uc120\ubcf4\uc600\uc73c\uba70 \uc77c\ubcf8 \uc5b8\ub860\ub4e4\uc740 \ubca0\uc774\ube44\ubcf5\uc2a4\uc758 \uc77c\ubcf8 \ubc29\ubb38\uc744 \uc77c\uc81c\ud788 \ubcf4\ub3c4\ud588\ub2e4.11\uc6d4\uc5d0\ub294 \uc6d0\uc790\ubc14\uc624\ucd1d\ub9ac\uc55e\uc5d0\uc11c \uc911\uad6d \ub0a8\ub155\uc5d0\uc11c \uc5f4\ub9ac\ub294 `\uc81c1\ud68c CAFAIR (\ub3d9\uc544\uc2dc\uc544\uc5f0\ub9f9)", "paragraph_sentence": " 8\uc6d4\uc5d0\ub294 \uc77c\ubcf8 \ub098\uce74\uc624\uce74 \ud6c4\uc0dd\ud68c\uad00\uc5d0\uc11c \uc5f4\ub9b0 \u20182004\ub2c8\uac00\ud0c0-\uc11c\uc6b8 \uc0b0\uc5c5 \uad00\uad11 \ubb38\ud654\uad50\ub958\uc0ac\uc5c5\uc804\u2019 \uc804\uc57c\uc81c \ubb34\ub300\uc5d0 \uc11c\uae30 \uc704\ud574 \uc77c\ubcf8\uc744 \ubc29\ubb38\ud588\uace0 \ub3c4\ucfc4\uc758 \ud640\ub9ac\ub370\uc774\ud638\ud154\uc5d0\uc11c \uae30\uc790\ud68c\uacac\uacfc \ubbf8\ub2c8 \ub77c\uc774\ube0c \ubb34\ub300\ub97c \uc120\ubcf4\uc600\uc73c\uba70 \uc77c\ubcf8 \uc5b8\ub860\ub4e4\uc740 \ubca0\uc774\ube44\ubcf5\uc2a4\uc758 \uc77c\ubcf8 \ubc29\ubb38\uc744 \uc77c\uc81c\ud788 \ubcf4\ub3c4\ud588\ub2e4.11\uc6d4\uc5d0\ub294 \uc6d0\uc790\ubc14\uc624\ucd1d\ub9ac\uc55e\uc5d0\uc11c \uc911\uad6d \ub0a8\ub155\uc5d0\uc11c \uc5f4\ub9ac\ub294 `\uc81c1\ud68c CAFAIR (\ub3d9\uc544\uc2dc\uc544\uc5f0\ub9f9) \ubc15\ub78c\ud68c` \ud3d0\ub9c9\uc2dd \uacf5\uc5f0\uc5d0 \ucc38\uc11d\ud574 \uc5f4\ucc3d\uc758 \ubb34\ub300\ub97c \uafb8\uba84\uace0 \uc774 \ud589\uc0ac\uc758 \ucc38\uc5ec\ub294 \uacf5\uc5f0\uc744 \uc8fc\uad00\ud558\ub294 CCTV \uad00\uacc4\uc790\uc758 \uc694\uccad\uc5d0 \ub530\ub978 \uac83\uc73c\ub85c \ud604\uc9c0\uc5d0\uc11c\uc758 \ubca0\uc774\ube44\ubcf5\uc2a4 \uc778\uae30\ub97c \uadf8\ub300\ub85c \ubc18\uc99d\uc2dc\ucf30\ub2e4.\uac00\uc218 \ube44\uc640 12\uc6d4\ubd80\ud130 \uc911\uad6d \ubca0\uc774\uc9d5\uc5d0\uc11c \ub300\uaddc\ubaa8 \ud569\ub3d9 \ucf58\uc11c\ud2b8\ub97c \uac16\uae30\ub3c4 \ud588\ub294\ub370 \uc774 \uacf5\uc5f0\uc740 \uc774\ubca4\ud2b8\uc131 \ud589\uc0ac\uac00 \uc544\ub2cc \uc720\ub8cc \uacf5\uc5f0\uc73c\ub85c\ub294 2002\ub144 \ubca0\uc774\ube44\ubcf5\uc2a4 \ub2e8\ub3c5 \ucf58\uc11c\ud2b8 \uc774\ud6c4 2\ub144 \ub9cc\uc5d0 \ucc98\uc74c\uc73c\ub85c \uad6d\ub0b4 \uac00\uc218\ub85c\uc11c \uc774\ub840\uc801\uc778 \uacf5\uc5f0 \ud5c8\uac00\ub97c \ubc1b\uc558\uc73c\uba70, \uc8fc\ucd5c\uce21\uc778 Vnet midas\uc640 DR china\ub294 \uc774 \uacf5\uc5f0\uc744 \uc8fc\ucd5c\ud558\uae30 \uc704\ud574 \uc911\uad6d \uc815\ubd80\uc640 6\uac1c\uc6d4\uac04\uc758 \uad50\uc12d \ub05d\uc5d0 \uc5b4\ub835\uac8c \uacf5\uc5f0 \ube44\uc900\uc744 \ubc1b\uc544 \uc808\ucc28\uac00 \uae4c\ub2e4\ub85c\uc6b4 \uc911\uad6d\uacf5\uc5f0\uc744 \uc131\uc0ac\uc2dc\ucf30\ub2e4.", "paragraph_answer": "8\uc6d4\uc5d0\ub294 \uc77c\ubcf8 \ub098\uce74\uc624\uce74 \ud6c4\uc0dd\ud68c\uad00\uc5d0\uc11c \uc5f4\ub9b0 \u20182004\ub2c8\uac00\ud0c0-\uc11c\uc6b8 \uc0b0\uc5c5 \uad00\uad11 \ubb38\ud654\uad50\ub958\uc0ac\uc5c5\uc804\u2019 \uc804\uc57c\uc81c \ubb34\ub300\uc5d0 \uc11c\uae30 \uc704\ud574 \uc77c\ubcf8\uc744 \ubc29\ubb38\ud588\uace0 \ub3c4\ucfc4\uc758 \ud640\ub9ac\ub370\uc774\ud638\ud154\uc5d0\uc11c \uae30\uc790\ud68c\uacac\uacfc \ubbf8\ub2c8 \ub77c\uc774\ube0c \ubb34\ub300\ub97c \uc120\ubcf4\uc600\uc73c\uba70 \uc77c\ubcf8 \uc5b8\ub860\ub4e4\uc740 \ubca0\uc774\ube44\ubcf5\uc2a4\uc758 \uc77c\ubcf8 \ubc29\ubb38\uc744 \uc77c\uc81c\ud788 \ubcf4\ub3c4\ud588\ub2e4.11\uc6d4\uc5d0\ub294 \uc6d0\uc790\ubc14\uc624\ucd1d\ub9ac\uc55e\uc5d0\uc11c \uc911\uad6d \ub0a8\ub155\uc5d0\uc11c \uc5f4\ub9ac\ub294 `\uc81c1\ud68c CAFAIR (\ub3d9\uc544\uc2dc\uc544\uc5f0\ub9f9) \ubc15\ub78c\ud68c` \ud3d0\ub9c9\uc2dd \uacf5\uc5f0\uc5d0 \ucc38\uc11d\ud574 \uc5f4\ucc3d\uc758 \ubb34\ub300\ub97c \uafb8\uba84\uace0 \uc774 \ud589\uc0ac\uc758 \ucc38\uc5ec\ub294 \uacf5\uc5f0\uc744 \uc8fc\uad00\ud558\ub294 CCTV \uad00\uacc4\uc790\uc758 \uc694\uccad\uc5d0 \ub530\ub978 \uac83\uc73c\ub85c \ud604\uc9c0\uc5d0\uc11c\uc758 \ubca0\uc774\ube44\ubcf5\uc2a4 \uc778\uae30\ub97c \uadf8\ub300\ub85c \ubc18\uc99d\uc2dc\ucf30\ub2e4.\uac00\uc218 \ube44\uc640 12\uc6d4\ubd80\ud130 \uc911\uad6d \ubca0\uc774\uc9d5\uc5d0\uc11c \ub300\uaddc\ubaa8 \ud569\ub3d9 \ucf58\uc11c\ud2b8\ub97c \uac16\uae30\ub3c4 \ud588\ub294\ub370 \uc774 \uacf5\uc5f0\uc740 \uc774\ubca4\ud2b8\uc131 \ud589\uc0ac\uac00 \uc544\ub2cc \uc720\ub8cc \uacf5\uc5f0\uc73c\ub85c\ub294 2002\ub144 \ubca0\uc774\ube44\ubcf5\uc2a4 \ub2e8\ub3c5 \ucf58\uc11c\ud2b8 \uc774\ud6c4 2\ub144 \ub9cc\uc5d0 \ucc98\uc74c\uc73c\ub85c \uad6d\ub0b4 \uac00\uc218\ub85c\uc11c \uc774\ub840\uc801\uc778 \uacf5\uc5f0 \ud5c8\uac00\ub97c \ubc1b\uc558\uc73c\uba70, \uc8fc\ucd5c\uce21\uc778 Vnet midas\uc640 DR china\ub294 \uc774 \uacf5\uc5f0\uc744 \uc8fc\ucd5c\ud558\uae30 \uc704\ud574 \uc911\uad6d \uc815\ubd80\uc640 6\uac1c\uc6d4\uac04\uc758 \uad50\uc12d \ub05d\uc5d0 \uc5b4\ub835\uac8c \uacf5\uc5f0 \ube44\uc900\uc744 \ubc1b\uc544 \uc808\ucc28\uac00 \uae4c\ub2e4\ub85c\uc6b4 \uc911\uad6d\uacf5\uc5f0\uc744 \uc131\uc0ac\uc2dc\ucf30\ub2e4.", "sentence_answer": "8\uc6d4\uc5d0\ub294 \uc77c\ubcf8 \ub098\uce74\uc624\uce74 \ud6c4\uc0dd\ud68c\uad00\uc5d0\uc11c \uc5f4\ub9b0 \u20182004\ub2c8\uac00\ud0c0-\uc11c\uc6b8 \uc0b0\uc5c5 \uad00\uad11 \ubb38\ud654\uad50\ub958\uc0ac\uc5c5\uc804\u2019 \uc804\uc57c\uc81c \ubb34\ub300\uc5d0 \uc11c\uae30 \uc704\ud574 \uc77c\ubcf8\uc744 \ubc29\ubb38\ud588\uace0 \ub3c4\ucfc4\uc758 \ud640\ub9ac\ub370\uc774\ud638\ud154\uc5d0\uc11c \uae30\uc790\ud68c\uacac\uacfc \ubbf8\ub2c8 \ub77c\uc774\ube0c \ubb34\ub300\ub97c \uc120\ubcf4\uc600\uc73c\uba70 \uc77c\ubcf8 \uc5b8\ub860\ub4e4\uc740 \ubca0\uc774\ube44\ubcf5\uc2a4\uc758 \uc77c\ubcf8 \ubc29\ubb38\uc744 \uc77c\uc81c\ud788 \ubcf4\ub3c4\ud588\ub2e4.11\uc6d4\uc5d0\ub294 \uc6d0\uc790\ubc14\uc624\ucd1d\ub9ac\uc55e\uc5d0\uc11c \uc911\uad6d \ub0a8\ub155\uc5d0\uc11c \uc5f4\ub9ac\ub294 `\uc81c1\ud68c CAFAIR (\ub3d9\uc544\uc2dc\uc544\uc5f0\ub9f9)", "paragraph_id": "6508291-26-1", "paragraph_question": "question: 11\uc6d4\uc5d0 \ubca0\uc774\ube44\ubcf5\uc2a4\uac00 \uc911\uad6d\uc5d0\uc11c \ud3d0\ub9c9\uc2dd \uacf5\uc5f0\uc5d0 \ucc38\uc5ec\ud55c \ubc15\ub78c\ud68c \uc774\ub984\uc740?, context: 8\uc6d4\uc5d0\ub294 \uc77c\ubcf8 \ub098\uce74\uc624\uce74 \ud6c4\uc0dd\ud68c\uad00\uc5d0\uc11c \uc5f4\ub9b0 \u20182004\ub2c8\uac00\ud0c0-\uc11c\uc6b8 \uc0b0\uc5c5 \uad00\uad11 \ubb38\ud654\uad50\ub958\uc0ac\uc5c5\uc804\u2019 \uc804\uc57c\uc81c \ubb34\ub300\uc5d0 \uc11c\uae30 \uc704\ud574 \uc77c\ubcf8\uc744 \ubc29\ubb38\ud588\uace0 \ub3c4\ucfc4\uc758 \ud640\ub9ac\ub370\uc774\ud638\ud154\uc5d0\uc11c \uae30\uc790\ud68c\uacac\uacfc \ubbf8\ub2c8 \ub77c\uc774\ube0c \ubb34\ub300\ub97c \uc120\ubcf4\uc600\uc73c\uba70 \uc77c\ubcf8 \uc5b8\ub860\ub4e4\uc740 \ubca0\uc774\ube44\ubcf5\uc2a4\uc758 \uc77c\ubcf8 \ubc29\ubb38\uc744 \uc77c\uc81c\ud788 \ubcf4\ub3c4\ud588\ub2e4.11\uc6d4\uc5d0\ub294 \uc6d0\uc790\ubc14\uc624\ucd1d\ub9ac\uc55e\uc5d0\uc11c \uc911\uad6d \ub0a8\ub155\uc5d0\uc11c \uc5f4\ub9ac\ub294 `\uc81c1\ud68c CAFAIR(\ub3d9\uc544\uc2dc\uc544\uc5f0\ub9f9) \ubc15\ub78c\ud68c` \ud3d0\ub9c9\uc2dd \uacf5\uc5f0\uc5d0 \ucc38\uc11d\ud574 \uc5f4\ucc3d\uc758 \ubb34\ub300\ub97c \uafb8\uba84\uace0 \uc774 \ud589\uc0ac\uc758 \ucc38\uc5ec\ub294 \uacf5\uc5f0\uc744 \uc8fc\uad00\ud558\ub294 CCTV \uad00\uacc4\uc790\uc758 \uc694\uccad\uc5d0 \ub530\ub978 \uac83\uc73c\ub85c \ud604\uc9c0\uc5d0\uc11c\uc758 \ubca0\uc774\ube44\ubcf5\uc2a4 \uc778\uae30\ub97c \uadf8\ub300\ub85c \ubc18\uc99d\uc2dc\ucf30\ub2e4.\uac00\uc218 \ube44\uc640 12\uc6d4\ubd80\ud130 \uc911\uad6d \ubca0\uc774\uc9d5\uc5d0\uc11c \ub300\uaddc\ubaa8 \ud569\ub3d9 \ucf58\uc11c\ud2b8\ub97c \uac16\uae30\ub3c4 \ud588\ub294\ub370 \uc774 \uacf5\uc5f0\uc740 \uc774\ubca4\ud2b8\uc131 \ud589\uc0ac\uac00 \uc544\ub2cc \uc720\ub8cc \uacf5\uc5f0\uc73c\ub85c\ub294 2002\ub144 \ubca0\uc774\ube44\ubcf5\uc2a4 \ub2e8\ub3c5 \ucf58\uc11c\ud2b8 \uc774\ud6c4 2\ub144 \ub9cc\uc5d0 \ucc98\uc74c\uc73c\ub85c \uad6d\ub0b4 \uac00\uc218\ub85c\uc11c \uc774\ub840\uc801\uc778 \uacf5\uc5f0 \ud5c8\uac00\ub97c \ubc1b\uc558\uc73c\uba70, \uc8fc\ucd5c\uce21\uc778 Vnet midas\uc640 DR china\ub294 \uc774 \uacf5\uc5f0\uc744 \uc8fc\ucd5c\ud558\uae30 \uc704\ud574 \uc911\uad6d \uc815\ubd80\uc640 6\uac1c\uc6d4\uac04\uc758 \uad50\uc12d \ub05d\uc5d0 \uc5b4\ub835\uac8c \uacf5\uc5f0 \ube44\uc900\uc744 \ubc1b\uc544 \uc808\ucc28\uac00 \uae4c\ub2e4\ub85c\uc6b4 \uc911\uad6d\uacf5\uc5f0\uc744 \uc131\uc0ac\uc2dc\ucf30\ub2e4."} {"question": "2003\ub144 \ucf00\uc774\ub85c\uc2a4 \uac10\ub3c5\uc774 \uc774\ub044\ub294 \ub808\uc54c \ub9c8\ub4dc\ub9ac\ub4dc\uac00 \uc120\ud638\ud55c \uc120\uc218 \ubc30\uce58 \ud615\ud0dc\ub294 \ubb34\uc5c7\uc778\uac00?", "paragraph": "2003\ub144 8\uc6d4 \ub9d0, \ub808\uc54c \ub9c8\ub4dc\ub9ac\ub4dc\ub294 \ub9c8\uc694\ub974\uce74\uc640\uc758 2\uacbd\uae30\ub97c \ud1b5\ud574 \uc218\ud398\ub974\ucf54\ud30c \ub370 \uc5d0\uc2a4\ud30c\ub0d0 \uc6b0\uc2b9\uc744 \ucc28\uc9c0\ud588\uace0, \ubca0\ucef4\uc740 \uc548\ubc29\uc5d0\uc11c \ubc8c\uc5b4\uc9c4 2\ucc28\uc804\uc5d0\uc11c \uc410\uae30\uace8\uc744 \uae30\ub85d\ud574 3-0 \uc2b9\ub9ac\ub97c \ub3c4\uc640 \uc815\uaddc \uc2dc\uc98c\uc744 \uc55e\ub450\uace0 \uc548\uc815\uc801\uc73c\ub85c \uc2dc\uc791\ud560 \ubb34\ub300\ub97c \ub9c8\ub828\ud588\ub2e4. \uac70\ubb3c\ub4e4\uc774 \uc990\ube44\ud55c \uc120\uc218\ub2e8\uc758 \uc77c\uc6d0\uc73c\ub85c, 3\uba85\uc758 FIFA \uc62c\ud574\uc758 \uc120\uc218\ub85c \uc9c0\ubaa9\ub41c \uc9c0\ub124\ub518 \uc9c0\ub2e8, \ud638\ub098\uc6b0\ub450, \uadf8\ub9ac\uace0 \ub8e8\uc774\uc2a4 \ud53c\uad6c\uac00 \uc788\uc5c8\uace0, \uadf8\ub4e4 \ub9d0\uace0\ub3c4 \ud638\ubca0\ub974\ud22c \uce74\ub97c\ub8e8\uc2a4, \ub77c\uc6b8, \uadf8\ub9ac\uace0 \uc774\ucf00\ub974 \uce74\uc2dc\uc57c\uc2a4\uac00 \ud3ec\uc9c4\ud574 \uc788\uc5c8\uc9c0\ub9cc, \ubca0\ucef4\uc740 \uc801\uc751\uc5d0 \uae34 \uc2dc\uac04\uc744 \ud544\uc694\ub85c \ud558\uc9c0 \uc54a\uc558\uace0, \uadf8\uac00 \ucd9c\uc804\ud55c \ucc98\uc74c\uc758 16\uacbd\uae30\uc5d0\uc11c 5\uace8\uc744 \uae30\ub85d\ud588\ub2e4. (\ub77c \ub9ac\uac00 \uc2e0\uace0\uc2dd\uc744 \uce58\ub978 \uacbd\uae30\uc5d0\uc11c\ub294 3\ubd84\ub3c4 \uc548 \ub418\uc5b4\uc11c \ub4dd\uc810\uc744 \uc62c\ub9ac\uae30\ub3c4 \ud588\ub2e4.) \ucf00\uc774\ub85c\uc2a4 \uac10\ub3c5\uc740 5-3-2 \ubc30\uce58 \ud615\ud0dc\ub97c \uc120\ud638\ud588\ub294\ub370, \ub450 \uba85\uc758 \uce21\uba74 \uc218\ube44\uc218 \ubbf8\uccbc \uc0b4\uac00\ub3c4\uc640 \ud638\ubca0\ub974\ud22c \uce74\ub97c\ub8e8\uc2a4\uac00 \uce21\uba74\uc744 \ud1b5\ud574 \uacf5\uaca9\uc744 \uac00\ub2f4\ud588\uace0, \ubca0\ucef4\uc740 3\uc778\uc73c\ub85c \uad6c\uc131\ub41c \uc911\uc6d0\uc5d0 \uc9c0\ub2e8\uacfc \ud53c\uad6c\uc640 \ud568\uaed8 \ubc30\uce58\ub418\uc5c8\ub2e4.", "answer": "5-3-2 \ubc30\uce58", "sentence": "\ucf00\uc774\ub85c\uc2a4 \uac10\ub3c5\uc740 5-3-2 \ubc30\uce58 \ud615\ud0dc\ub97c \uc120\ud638\ud588\ub294\ub370, \ub450 \uba85\uc758 \uce21\uba74 \uc218\ube44\uc218 \ubbf8\uccbc \uc0b4\uac00\ub3c4\uc640 \ud638\ubca0\ub974\ud22c \uce74\ub97c\ub8e8\uc2a4\uac00 \uce21\uba74\uc744 \ud1b5\ud574 \uacf5\uaca9\uc744 \uac00\ub2f4\ud588\uace0, \ubca0\ucef4\uc740 3\uc778\uc73c\ub85c \uad6c\uc131\ub41c \uc911\uc6d0\uc5d0 \uc9c0\ub2e8\uacfc \ud53c\uad6c\uc640 \ud568\uaed8 \ubc30\uce58\ub418\uc5c8\ub2e4.", "paragraph_sentence": "2003\ub144 8\uc6d4 \ub9d0, \ub808\uc54c \ub9c8\ub4dc\ub9ac\ub4dc\ub294 \ub9c8\uc694\ub974\uce74\uc640\uc758 2\uacbd\uae30\ub97c \ud1b5\ud574 \uc218\ud398\ub974\ucf54\ud30c \ub370 \uc5d0\uc2a4\ud30c\ub0d0 \uc6b0\uc2b9\uc744 \ucc28\uc9c0\ud588\uace0, \ubca0\ucef4\uc740 \uc548\ubc29\uc5d0\uc11c \ubc8c\uc5b4\uc9c4 2\ucc28\uc804\uc5d0\uc11c \uc410\uae30\uace8\uc744 \uae30\ub85d\ud574 3-0 \uc2b9\ub9ac\ub97c \ub3c4\uc640 \uc815\uaddc \uc2dc\uc98c\uc744 \uc55e\ub450\uace0 \uc548\uc815\uc801\uc73c\ub85c \uc2dc\uc791\ud560 \ubb34\ub300\ub97c \ub9c8\ub828\ud588\ub2e4. \uac70\ubb3c\ub4e4\uc774 \uc990\ube44\ud55c \uc120\uc218\ub2e8\uc758 \uc77c\uc6d0\uc73c\ub85c, 3\uba85\uc758 FIFA \uc62c\ud574\uc758 \uc120\uc218\ub85c \uc9c0\ubaa9\ub41c \uc9c0\ub124\ub518 \uc9c0\ub2e8, \ud638\ub098\uc6b0\ub450, \uadf8\ub9ac\uace0 \ub8e8\uc774\uc2a4 \ud53c\uad6c\uac00 \uc788\uc5c8\uace0, \uadf8\ub4e4 \ub9d0\uace0\ub3c4 \ud638\ubca0\ub974\ud22c \uce74\ub97c\ub8e8\uc2a4, \ub77c\uc6b8, \uadf8\ub9ac\uace0 \uc774\ucf00\ub974 \uce74\uc2dc\uc57c\uc2a4\uac00 \ud3ec\uc9c4\ud574 \uc788\uc5c8\uc9c0\ub9cc, \ubca0\ucef4\uc740 \uc801\uc751\uc5d0 \uae34 \uc2dc\uac04\uc744 \ud544\uc694\ub85c \ud558\uc9c0 \uc54a\uc558\uace0, \uadf8\uac00 \ucd9c\uc804\ud55c \ucc98\uc74c\uc758 16\uacbd\uae30\uc5d0\uc11c 5\uace8\uc744 \uae30\ub85d\ud588\ub2e4. (\ub77c \ub9ac\uac00 \uc2e0\uace0\uc2dd\uc744 \uce58\ub978 \uacbd\uae30\uc5d0\uc11c\ub294 3\ubd84\ub3c4 \uc548 \ub418\uc5b4\uc11c \ub4dd\uc810\uc744 \uc62c\ub9ac\uae30\ub3c4 \ud588\ub2e4. ) \ucf00\uc774\ub85c\uc2a4 \uac10\ub3c5\uc740 5-3-2 \ubc30\uce58 \ud615\ud0dc\ub97c \uc120\ud638\ud588\ub294\ub370, \ub450 \uba85\uc758 \uce21\uba74 \uc218\ube44\uc218 \ubbf8\uccbc \uc0b4\uac00\ub3c4\uc640 \ud638\ubca0\ub974\ud22c \uce74\ub97c\ub8e8\uc2a4\uac00 \uce21\uba74\uc744 \ud1b5\ud574 \uacf5\uaca9\uc744 \uac00\ub2f4\ud588\uace0, \ubca0\ucef4\uc740 3\uc778\uc73c\ub85c \uad6c\uc131\ub41c \uc911\uc6d0\uc5d0 \uc9c0\ub2e8\uacfc \ud53c\uad6c\uc640 \ud568\uaed8 \ubc30\uce58\ub418\uc5c8\ub2e4. ", "paragraph_answer": "2003\ub144 8\uc6d4 \ub9d0, \ub808\uc54c \ub9c8\ub4dc\ub9ac\ub4dc\ub294 \ub9c8\uc694\ub974\uce74\uc640\uc758 2\uacbd\uae30\ub97c \ud1b5\ud574 \uc218\ud398\ub974\ucf54\ud30c \ub370 \uc5d0\uc2a4\ud30c\ub0d0 \uc6b0\uc2b9\uc744 \ucc28\uc9c0\ud588\uace0, \ubca0\ucef4\uc740 \uc548\ubc29\uc5d0\uc11c \ubc8c\uc5b4\uc9c4 2\ucc28\uc804\uc5d0\uc11c \uc410\uae30\uace8\uc744 \uae30\ub85d\ud574 3-0 \uc2b9\ub9ac\ub97c \ub3c4\uc640 \uc815\uaddc \uc2dc\uc98c\uc744 \uc55e\ub450\uace0 \uc548\uc815\uc801\uc73c\ub85c \uc2dc\uc791\ud560 \ubb34\ub300\ub97c \ub9c8\ub828\ud588\ub2e4. \uac70\ubb3c\ub4e4\uc774 \uc990\ube44\ud55c \uc120\uc218\ub2e8\uc758 \uc77c\uc6d0\uc73c\ub85c, 3\uba85\uc758 FIFA \uc62c\ud574\uc758 \uc120\uc218\ub85c \uc9c0\ubaa9\ub41c \uc9c0\ub124\ub518 \uc9c0\ub2e8, \ud638\ub098\uc6b0\ub450, \uadf8\ub9ac\uace0 \ub8e8\uc774\uc2a4 \ud53c\uad6c\uac00 \uc788\uc5c8\uace0, \uadf8\ub4e4 \ub9d0\uace0\ub3c4 \ud638\ubca0\ub974\ud22c \uce74\ub97c\ub8e8\uc2a4, \ub77c\uc6b8, \uadf8\ub9ac\uace0 \uc774\ucf00\ub974 \uce74\uc2dc\uc57c\uc2a4\uac00 \ud3ec\uc9c4\ud574 \uc788\uc5c8\uc9c0\ub9cc, \ubca0\ucef4\uc740 \uc801\uc751\uc5d0 \uae34 \uc2dc\uac04\uc744 \ud544\uc694\ub85c \ud558\uc9c0 \uc54a\uc558\uace0, \uadf8\uac00 \ucd9c\uc804\ud55c \ucc98\uc74c\uc758 16\uacbd\uae30\uc5d0\uc11c 5\uace8\uc744 \uae30\ub85d\ud588\ub2e4. (\ub77c \ub9ac\uac00 \uc2e0\uace0\uc2dd\uc744 \uce58\ub978 \uacbd\uae30\uc5d0\uc11c\ub294 3\ubd84\ub3c4 \uc548 \ub418\uc5b4\uc11c \ub4dd\uc810\uc744 \uc62c\ub9ac\uae30\ub3c4 \ud588\ub2e4.) \ucf00\uc774\ub85c\uc2a4 \uac10\ub3c5\uc740 5-3-2 \ubc30\uce58 \ud615\ud0dc\ub97c \uc120\ud638\ud588\ub294\ub370, \ub450 \uba85\uc758 \uce21\uba74 \uc218\ube44\uc218 \ubbf8\uccbc \uc0b4\uac00\ub3c4\uc640 \ud638\ubca0\ub974\ud22c \uce74\ub97c\ub8e8\uc2a4\uac00 \uce21\uba74\uc744 \ud1b5\ud574 \uacf5\uaca9\uc744 \uac00\ub2f4\ud588\uace0, \ubca0\ucef4\uc740 3\uc778\uc73c\ub85c \uad6c\uc131\ub41c \uc911\uc6d0\uc5d0 \uc9c0\ub2e8\uacfc \ud53c\uad6c\uc640 \ud568\uaed8 \ubc30\uce58\ub418\uc5c8\ub2e4.", "sentence_answer": "\ucf00\uc774\ub85c\uc2a4 \uac10\ub3c5\uc740 5-3-2 \ubc30\uce58 \ud615\ud0dc\ub97c \uc120\ud638\ud588\ub294\ub370, \ub450 \uba85\uc758 \uce21\uba74 \uc218\ube44\uc218 \ubbf8\uccbc \uc0b4\uac00\ub3c4\uc640 \ud638\ubca0\ub974\ud22c \uce74\ub97c\ub8e8\uc2a4\uac00 \uce21\uba74\uc744 \ud1b5\ud574 \uacf5\uaca9\uc744 \uac00\ub2f4\ud588\uace0, \ubca0\ucef4\uc740 3\uc778\uc73c\ub85c \uad6c\uc131\ub41c \uc911\uc6d0\uc5d0 \uc9c0\ub2e8\uacfc \ud53c\uad6c\uc640 \ud568\uaed8 \ubc30\uce58\ub418\uc5c8\ub2e4.", "paragraph_id": "6602120-14-1", "paragraph_question": "question: 2003\ub144 \ucf00\uc774\ub85c\uc2a4 \uac10\ub3c5\uc774 \uc774\ub044\ub294 \ub808\uc54c \ub9c8\ub4dc\ub9ac\ub4dc\uac00 \uc120\ud638\ud55c \uc120\uc218 \ubc30\uce58 \ud615\ud0dc\ub294 \ubb34\uc5c7\uc778\uac00?, context: 2003\ub144 8\uc6d4 \ub9d0, \ub808\uc54c \ub9c8\ub4dc\ub9ac\ub4dc\ub294 \ub9c8\uc694\ub974\uce74\uc640\uc758 2\uacbd\uae30\ub97c \ud1b5\ud574 \uc218\ud398\ub974\ucf54\ud30c \ub370 \uc5d0\uc2a4\ud30c\ub0d0 \uc6b0\uc2b9\uc744 \ucc28\uc9c0\ud588\uace0, \ubca0\ucef4\uc740 \uc548\ubc29\uc5d0\uc11c \ubc8c\uc5b4\uc9c4 2\ucc28\uc804\uc5d0\uc11c \uc410\uae30\uace8\uc744 \uae30\ub85d\ud574 3-0 \uc2b9\ub9ac\ub97c \ub3c4\uc640 \uc815\uaddc \uc2dc\uc98c\uc744 \uc55e\ub450\uace0 \uc548\uc815\uc801\uc73c\ub85c \uc2dc\uc791\ud560 \ubb34\ub300\ub97c \ub9c8\ub828\ud588\ub2e4. \uac70\ubb3c\ub4e4\uc774 \uc990\ube44\ud55c \uc120\uc218\ub2e8\uc758 \uc77c\uc6d0\uc73c\ub85c, 3\uba85\uc758 FIFA \uc62c\ud574\uc758 \uc120\uc218\ub85c \uc9c0\ubaa9\ub41c \uc9c0\ub124\ub518 \uc9c0\ub2e8, \ud638\ub098\uc6b0\ub450, \uadf8\ub9ac\uace0 \ub8e8\uc774\uc2a4 \ud53c\uad6c\uac00 \uc788\uc5c8\uace0, \uadf8\ub4e4 \ub9d0\uace0\ub3c4 \ud638\ubca0\ub974\ud22c \uce74\ub97c\ub8e8\uc2a4, \ub77c\uc6b8, \uadf8\ub9ac\uace0 \uc774\ucf00\ub974 \uce74\uc2dc\uc57c\uc2a4\uac00 \ud3ec\uc9c4\ud574 \uc788\uc5c8\uc9c0\ub9cc, \ubca0\ucef4\uc740 \uc801\uc751\uc5d0 \uae34 \uc2dc\uac04\uc744 \ud544\uc694\ub85c \ud558\uc9c0 \uc54a\uc558\uace0, \uadf8\uac00 \ucd9c\uc804\ud55c \ucc98\uc74c\uc758 16\uacbd\uae30\uc5d0\uc11c 5\uace8\uc744 \uae30\ub85d\ud588\ub2e4. (\ub77c \ub9ac\uac00 \uc2e0\uace0\uc2dd\uc744 \uce58\ub978 \uacbd\uae30\uc5d0\uc11c\ub294 3\ubd84\ub3c4 \uc548 \ub418\uc5b4\uc11c \ub4dd\uc810\uc744 \uc62c\ub9ac\uae30\ub3c4 \ud588\ub2e4.) \ucf00\uc774\ub85c\uc2a4 \uac10\ub3c5\uc740 5-3-2 \ubc30\uce58 \ud615\ud0dc\ub97c \uc120\ud638\ud588\ub294\ub370, \ub450 \uba85\uc758 \uce21\uba74 \uc218\ube44\uc218 \ubbf8\uccbc \uc0b4\uac00\ub3c4\uc640 \ud638\ubca0\ub974\ud22c \uce74\ub97c\ub8e8\uc2a4\uac00 \uce21\uba74\uc744 \ud1b5\ud574 \uacf5\uaca9\uc744 \uac00\ub2f4\ud588\uace0, \ubca0\ucef4\uc740 3\uc778\uc73c\ub85c \uad6c\uc131\ub41c \uc911\uc6d0\uc5d0 \uc9c0\ub2e8\uacfc \ud53c\uad6c\uc640 \ud568\uaed8 \ubc30\uce58\ub418\uc5c8\ub2e4."} {"question": "\ub450\ud23d\uc0c1\uc5b4 \uc54c\uc9d1\uc758 \uac00\ub85c \ud06c\uae30\ub294?", "paragraph": "\uc554\ucef7\uc740 \ud55c \ubc88\uc5d0 \ub450 \uac1c\uc758 \ub09c\uad00\uc73c\ub85c \uac01\uac01 \ud558\ub098\uc529 \uc54c\uc744 \ub450 \uac1c \ub0b3\ub294\ub2e4. \uc54c\uc740 \ubd80\ub4dc\ub7ec\uc6b4 \ud22c\uba85\ud55c \ud669\uc0c9\uc758 \uaf43\ubcd1 \ubaa8\uc591 \uc54c\uc9d1\uc5d0 \uc2f8\uc5ec \uc788\ub2e4. \uc54c\uc9d1\uc758 \ud06c\uae30\ub294 \uac00\ub85c 1.9 \uc13c\ud2f0\ubbf8\ud130, \uc138\ub85c 5.5 \uc13c\ud2f0\ubbf8\ud130\uc774\ub2e4. \uc54c\uc9d1\uc758 \ub124 \ubaa8\uc11c\ub9ac\uc5d0\ub294 \ub369\uad74 \ubaa8\uc591\uc758 \ub3cc\uae30\uac00 \uae38\ucb49\ud558\ub2c8 \ub3cb\uc544 \uc788\ub2e4. \uc54c\uc740 \uc815\ud574\uc9c4 \uc0b0\ub780\uc9c0\uc5d0 \ub0b3\ub294\ub370, \ud558\ucf54\ub2e4\ud14c\uc758 \uc218\uc2ec 100 \ubbf8\ud130 \uc9c0\uc810\uc5d0\uc11c \uc0b0\ub780\uc9c0\uac00 \ud558\ub098 \ubc1c\uacac\ub41c \ubc14 \uc788\ub2e4. \ubc30\uc544\uc758 \uc2e0\uc7a5\uc774 3.6 \uc13c\ud2f0\ubbf8\ud130\uc5d0 \ub2ec\ud558\uba74 \uc678\ubd80 \uc544\uac00\ubbf8\uc640 \uc544\uc9c1 \ubc1c\ub2ec\uc774 \ub35c \ub05d\ub09c \uc9c0\ub290\ub7ec\ubbf8\ub97c \ud655\uc778\ud560 \uc218 \uc788\uc73c\uba70, \uc544\uc9c1 \ubab8\uc5d0 \uc0c9\uc18c\ub294 \uc5c6\ub2e4. \ubc30\uc544\uc758 \uc2e0\uc7a5\uc774 5.8 \uc13c\ud2f0\ubbf8\ud130\uc5d0 \ub2ec\ud558\uba74 \uc678\ubd80 \uc544\uac00\ubbf8\uac00 \uac70\uc758 \uc0ac\ub77c\uc9c0\uace0, \uc791\uc740 \ube44\ub298\ub4e4\uc774 \ubab8\uc744 \ub36e\uae30 \uc2dc\uc791\ud55c \uac83\uc774 \ubcf4\uc778\ub2e4. \uc2e0\uc7a5\uc774 7.9 \uc13c\ud2f0\ubbf8\ud130\uc5d0 \ub2ec\ud558\uba74 \uc9c0\ub290\ub7ec\ubbf8\uc640 \uc0c9\uc18c\uac00 \uc81c\ub300\ub85c \ubc1c\ub2ec\ud588\uc73c\uba70, \ud06c\uae30\ub97c \uc81c\uc678\ud558\uba74 \uac89\ubcf4\uae30\uc5d0 \uc131\uccb4\uc640 \uac70\uc758 \ube44\uc2b7\ud574\uc9c4\ub2e4. \uc54c\uc774 \ubd80\ud654\ud558\ub294 \ub370\ub294 \uc218\uc628 11.3 \u00b0C (52.3 \u00b0F)\uc5d0\uc11c 15\uac1c\uc6d4, \uc218\uc628 14.5 \u00b0C (58.1 \u00b0F)\uc5d0\uc11c 7 ~ 9\uac1c\uc6d4\uc774 \uac78\ub9b0\ub2e4. \ub9c9 \ubd80\ud654\ud55c \uc0c8\ub07c \uc0c1\uc5b4\uc758 \uc2e0\uc7a5\uc740 8 \uc13c\ud2f0\ubbf8\ud130 \uc774\uc0c1\uc774\ub2e4. \uc0dd\uc2dd\uc774 \uac00\ub2a5\ud55c \uc131\uccb4\uc758 \uc2e0\uc7a5\uc740 \uc218\uc628\uc774 \ub0b4\ub824\uac10\uc5d0 \ub530\ub77c \uc99d\uac00\ud558\ub294 \uacbd\ud5a5\uc774 \uc788\ub2e4. \ud558\ucf54\ub2e4\ud14c \uac19\uc740 \ubd81\ucabd\uc5d0\uc11c\ub294 \uc554\uc218\uac00 \ubaa8\ub450 \uc2e0\uc7a5 38 \uc13c\ud2f0\ubbf8\ud130\ucbe4\uc5d0\uc11c \uc131\uc219\ud558\uba70, \uc77c\ubd80 \uc554\ucef7\uc740 \uc2e0\uc7a5 47 \uc13c\ud2f0\ubbf8\ud130\uac00 \ub118\uc5b4\ub3c4 \uc131\uc219\ud558\uc9c0 \uc54a\uae30\ub3c4 \ud55c\ub2e4. \ubc18\uba74 \ub300\ub9c8\ub3c4 \ubd80\uadfc\uc758 \ub0a8\ucabd\uc5d0\uc11c\ub294 \uc554\uc218\uac00 \ubaa8\ub450 \uc2e0\uc7a5 33 \uc13c\ud2f0\ubbf8\ud130\ucbe4\uc5d0\uc11c \uc131\uc219\ud55c\ub2e4. \ucd5c\ub300\uc218\uba85\uc740 12\ub144 \uc774\uc0c1\uc774\ub2e4.", "answer": "1.9 \uc13c\ud2f0\ubbf8\ud130", "sentence": "\uc54c\uc9d1\uc758 \ud06c\uae30\ub294 \uac00\ub85c 1.9 \uc13c\ud2f0\ubbf8\ud130 ,", "paragraph_sentence": "\uc554\ucef7\uc740 \ud55c \ubc88\uc5d0 \ub450 \uac1c\uc758 \ub09c\uad00\uc73c\ub85c \uac01\uac01 \ud558\ub098\uc529 \uc54c\uc744 \ub450 \uac1c \ub0b3\ub294\ub2e4. \uc54c\uc740 \ubd80\ub4dc\ub7ec\uc6b4 \ud22c\uba85\ud55c \ud669\uc0c9\uc758 \uaf43\ubcd1 \ubaa8\uc591 \uc54c\uc9d1\uc5d0 \uc2f8\uc5ec \uc788\ub2e4. \uc54c\uc9d1\uc758 \ud06c\uae30\ub294 \uac00\ub85c 1.9 \uc13c\ud2f0\ubbf8\ud130 , \uc138\ub85c 5.5 \uc13c\ud2f0\ubbf8\ud130\uc774\ub2e4. \uc54c\uc9d1\uc758 \ub124 \ubaa8\uc11c\ub9ac\uc5d0\ub294 \ub369\uad74 \ubaa8\uc591\uc758 \ub3cc\uae30\uac00 \uae38\ucb49\ud558\ub2c8 \ub3cb\uc544 \uc788\ub2e4. \uc54c\uc740 \uc815\ud574\uc9c4 \uc0b0\ub780\uc9c0\uc5d0 \ub0b3\ub294\ub370, \ud558\ucf54\ub2e4\ud14c\uc758 \uc218\uc2ec 100 \ubbf8\ud130 \uc9c0\uc810\uc5d0\uc11c \uc0b0\ub780\uc9c0\uac00 \ud558\ub098 \ubc1c\uacac\ub41c \ubc14 \uc788\ub2e4. \ubc30\uc544\uc758 \uc2e0\uc7a5\uc774 3.6 \uc13c\ud2f0\ubbf8\ud130\uc5d0 \ub2ec\ud558\uba74 \uc678\ubd80 \uc544\uac00\ubbf8\uc640 \uc544\uc9c1 \ubc1c\ub2ec\uc774 \ub35c \ub05d\ub09c \uc9c0\ub290\ub7ec\ubbf8\ub97c \ud655\uc778\ud560 \uc218 \uc788\uc73c\uba70, \uc544\uc9c1 \ubab8\uc5d0 \uc0c9\uc18c\ub294 \uc5c6\ub2e4. \ubc30\uc544\uc758 \uc2e0\uc7a5\uc774 5.8 \uc13c\ud2f0\ubbf8\ud130\uc5d0 \ub2ec\ud558\uba74 \uc678\ubd80 \uc544\uac00\ubbf8\uac00 \uac70\uc758 \uc0ac\ub77c\uc9c0\uace0, \uc791\uc740 \ube44\ub298\ub4e4\uc774 \ubab8\uc744 \ub36e\uae30 \uc2dc\uc791\ud55c \uac83\uc774 \ubcf4\uc778\ub2e4. \uc2e0\uc7a5\uc774 7.9 \uc13c\ud2f0\ubbf8\ud130\uc5d0 \ub2ec\ud558\uba74 \uc9c0\ub290\ub7ec\ubbf8\uc640 \uc0c9\uc18c\uac00 \uc81c\ub300\ub85c \ubc1c\ub2ec\ud588\uc73c\uba70, \ud06c\uae30\ub97c \uc81c\uc678\ud558\uba74 \uac89\ubcf4\uae30\uc5d0 \uc131\uccb4\uc640 \uac70\uc758 \ube44\uc2b7\ud574\uc9c4\ub2e4. \uc54c\uc774 \ubd80\ud654\ud558\ub294 \ub370\ub294 \uc218\uc628 11.3 \u00b0C (52.3 \u00b0F)\uc5d0\uc11c 15\uac1c\uc6d4, \uc218\uc628 14.5 \u00b0C (58.1 \u00b0F)\uc5d0\uc11c 7 ~ 9\uac1c\uc6d4\uc774 \uac78\ub9b0\ub2e4. \ub9c9 \ubd80\ud654\ud55c \uc0c8\ub07c \uc0c1\uc5b4\uc758 \uc2e0\uc7a5\uc740 8 \uc13c\ud2f0\ubbf8\ud130 \uc774\uc0c1\uc774\ub2e4. \uc0dd\uc2dd\uc774 \uac00\ub2a5\ud55c \uc131\uccb4\uc758 \uc2e0\uc7a5\uc740 \uc218\uc628\uc774 \ub0b4\ub824\uac10\uc5d0 \ub530\ub77c \uc99d\uac00\ud558\ub294 \uacbd\ud5a5\uc774 \uc788\ub2e4. \ud558\ucf54\ub2e4\ud14c \uac19\uc740 \ubd81\ucabd\uc5d0\uc11c\ub294 \uc554\uc218\uac00 \ubaa8\ub450 \uc2e0\uc7a5 38 \uc13c\ud2f0\ubbf8\ud130\ucbe4\uc5d0\uc11c \uc131\uc219\ud558\uba70, \uc77c\ubd80 \uc554\ucef7\uc740 \uc2e0\uc7a5 47 \uc13c\ud2f0\ubbf8\ud130\uac00 \ub118\uc5b4\ub3c4 \uc131\uc219\ud558\uc9c0 \uc54a\uae30\ub3c4 \ud55c\ub2e4. \ubc18\uba74 \ub300\ub9c8\ub3c4 \ubd80\uadfc\uc758 \ub0a8\ucabd\uc5d0\uc11c\ub294 \uc554\uc218\uac00 \ubaa8\ub450 \uc2e0\uc7a5 33 \uc13c\ud2f0\ubbf8\ud130\ucbe4\uc5d0\uc11c \uc131\uc219\ud55c\ub2e4. \ucd5c\ub300\uc218\uba85\uc740 12\ub144 \uc774\uc0c1\uc774\ub2e4.", "paragraph_answer": "\uc554\ucef7\uc740 \ud55c \ubc88\uc5d0 \ub450 \uac1c\uc758 \ub09c\uad00\uc73c\ub85c \uac01\uac01 \ud558\ub098\uc529 \uc54c\uc744 \ub450 \uac1c \ub0b3\ub294\ub2e4. \uc54c\uc740 \ubd80\ub4dc\ub7ec\uc6b4 \ud22c\uba85\ud55c \ud669\uc0c9\uc758 \uaf43\ubcd1 \ubaa8\uc591 \uc54c\uc9d1\uc5d0 \uc2f8\uc5ec \uc788\ub2e4. \uc54c\uc9d1\uc758 \ud06c\uae30\ub294 \uac00\ub85c 1.9 \uc13c\ud2f0\ubbf8\ud130 , \uc138\ub85c 5.5 \uc13c\ud2f0\ubbf8\ud130\uc774\ub2e4. \uc54c\uc9d1\uc758 \ub124 \ubaa8\uc11c\ub9ac\uc5d0\ub294 \ub369\uad74 \ubaa8\uc591\uc758 \ub3cc\uae30\uac00 \uae38\ucb49\ud558\ub2c8 \ub3cb\uc544 \uc788\ub2e4. \uc54c\uc740 \uc815\ud574\uc9c4 \uc0b0\ub780\uc9c0\uc5d0 \ub0b3\ub294\ub370, \ud558\ucf54\ub2e4\ud14c\uc758 \uc218\uc2ec 100 \ubbf8\ud130 \uc9c0\uc810\uc5d0\uc11c \uc0b0\ub780\uc9c0\uac00 \ud558\ub098 \ubc1c\uacac\ub41c \ubc14 \uc788\ub2e4. \ubc30\uc544\uc758 \uc2e0\uc7a5\uc774 3.6 \uc13c\ud2f0\ubbf8\ud130\uc5d0 \ub2ec\ud558\uba74 \uc678\ubd80 \uc544\uac00\ubbf8\uc640 \uc544\uc9c1 \ubc1c\ub2ec\uc774 \ub35c \ub05d\ub09c \uc9c0\ub290\ub7ec\ubbf8\ub97c \ud655\uc778\ud560 \uc218 \uc788\uc73c\uba70, \uc544\uc9c1 \ubab8\uc5d0 \uc0c9\uc18c\ub294 \uc5c6\ub2e4. \ubc30\uc544\uc758 \uc2e0\uc7a5\uc774 5.8 \uc13c\ud2f0\ubbf8\ud130\uc5d0 \ub2ec\ud558\uba74 \uc678\ubd80 \uc544\uac00\ubbf8\uac00 \uac70\uc758 \uc0ac\ub77c\uc9c0\uace0, \uc791\uc740 \ube44\ub298\ub4e4\uc774 \ubab8\uc744 \ub36e\uae30 \uc2dc\uc791\ud55c \uac83\uc774 \ubcf4\uc778\ub2e4. \uc2e0\uc7a5\uc774 7.9 \uc13c\ud2f0\ubbf8\ud130\uc5d0 \ub2ec\ud558\uba74 \uc9c0\ub290\ub7ec\ubbf8\uc640 \uc0c9\uc18c\uac00 \uc81c\ub300\ub85c \ubc1c\ub2ec\ud588\uc73c\uba70, \ud06c\uae30\ub97c \uc81c\uc678\ud558\uba74 \uac89\ubcf4\uae30\uc5d0 \uc131\uccb4\uc640 \uac70\uc758 \ube44\uc2b7\ud574\uc9c4\ub2e4. \uc54c\uc774 \ubd80\ud654\ud558\ub294 \ub370\ub294 \uc218\uc628 11.3 \u00b0C (52.3 \u00b0F)\uc5d0\uc11c 15\uac1c\uc6d4, \uc218\uc628 14.5 \u00b0C (58.1 \u00b0F)\uc5d0\uc11c 7 ~ 9\uac1c\uc6d4\uc774 \uac78\ub9b0\ub2e4. \ub9c9 \ubd80\ud654\ud55c \uc0c8\ub07c \uc0c1\uc5b4\uc758 \uc2e0\uc7a5\uc740 8 \uc13c\ud2f0\ubbf8\ud130 \uc774\uc0c1\uc774\ub2e4. \uc0dd\uc2dd\uc774 \uac00\ub2a5\ud55c \uc131\uccb4\uc758 \uc2e0\uc7a5\uc740 \uc218\uc628\uc774 \ub0b4\ub824\uac10\uc5d0 \ub530\ub77c \uc99d\uac00\ud558\ub294 \uacbd\ud5a5\uc774 \uc788\ub2e4. \ud558\ucf54\ub2e4\ud14c \uac19\uc740 \ubd81\ucabd\uc5d0\uc11c\ub294 \uc554\uc218\uac00 \ubaa8\ub450 \uc2e0\uc7a5 38 \uc13c\ud2f0\ubbf8\ud130\ucbe4\uc5d0\uc11c \uc131\uc219\ud558\uba70, \uc77c\ubd80 \uc554\ucef7\uc740 \uc2e0\uc7a5 47 \uc13c\ud2f0\ubbf8\ud130\uac00 \ub118\uc5b4\ub3c4 \uc131\uc219\ud558\uc9c0 \uc54a\uae30\ub3c4 \ud55c\ub2e4. \ubc18\uba74 \ub300\ub9c8\ub3c4 \ubd80\uadfc\uc758 \ub0a8\ucabd\uc5d0\uc11c\ub294 \uc554\uc218\uac00 \ubaa8\ub450 \uc2e0\uc7a5 33 \uc13c\ud2f0\ubbf8\ud130\ucbe4\uc5d0\uc11c \uc131\uc219\ud55c\ub2e4. \ucd5c\ub300\uc218\uba85\uc740 12\ub144 \uc774\uc0c1\uc774\ub2e4.", "sentence_answer": "\uc54c\uc9d1\uc758 \ud06c\uae30\ub294 \uac00\ub85c 1.9 \uc13c\ud2f0\ubbf8\ud130 ,", "paragraph_id": "6492527-3-1", "paragraph_question": "question: \ub450\ud23d\uc0c1\uc5b4 \uc54c\uc9d1\uc758 \uac00\ub85c \ud06c\uae30\ub294?, context: \uc554\ucef7\uc740 \ud55c \ubc88\uc5d0 \ub450 \uac1c\uc758 \ub09c\uad00\uc73c\ub85c \uac01\uac01 \ud558\ub098\uc529 \uc54c\uc744 \ub450 \uac1c \ub0b3\ub294\ub2e4. \uc54c\uc740 \ubd80\ub4dc\ub7ec\uc6b4 \ud22c\uba85\ud55c \ud669\uc0c9\uc758 \uaf43\ubcd1 \ubaa8\uc591 \uc54c\uc9d1\uc5d0 \uc2f8\uc5ec \uc788\ub2e4. \uc54c\uc9d1\uc758 \ud06c\uae30\ub294 \uac00\ub85c 1.9 \uc13c\ud2f0\ubbf8\ud130, \uc138\ub85c 5.5 \uc13c\ud2f0\ubbf8\ud130\uc774\ub2e4. \uc54c\uc9d1\uc758 \ub124 \ubaa8\uc11c\ub9ac\uc5d0\ub294 \ub369\uad74 \ubaa8\uc591\uc758 \ub3cc\uae30\uac00 \uae38\ucb49\ud558\ub2c8 \ub3cb\uc544 \uc788\ub2e4. \uc54c\uc740 \uc815\ud574\uc9c4 \uc0b0\ub780\uc9c0\uc5d0 \ub0b3\ub294\ub370, \ud558\ucf54\ub2e4\ud14c\uc758 \uc218\uc2ec 100 \ubbf8\ud130 \uc9c0\uc810\uc5d0\uc11c \uc0b0\ub780\uc9c0\uac00 \ud558\ub098 \ubc1c\uacac\ub41c \ubc14 \uc788\ub2e4. \ubc30\uc544\uc758 \uc2e0\uc7a5\uc774 3.6 \uc13c\ud2f0\ubbf8\ud130\uc5d0 \ub2ec\ud558\uba74 \uc678\ubd80 \uc544\uac00\ubbf8\uc640 \uc544\uc9c1 \ubc1c\ub2ec\uc774 \ub35c \ub05d\ub09c \uc9c0\ub290\ub7ec\ubbf8\ub97c \ud655\uc778\ud560 \uc218 \uc788\uc73c\uba70, \uc544\uc9c1 \ubab8\uc5d0 \uc0c9\uc18c\ub294 \uc5c6\ub2e4. \ubc30\uc544\uc758 \uc2e0\uc7a5\uc774 5.8 \uc13c\ud2f0\ubbf8\ud130\uc5d0 \ub2ec\ud558\uba74 \uc678\ubd80 \uc544\uac00\ubbf8\uac00 \uac70\uc758 \uc0ac\ub77c\uc9c0\uace0, \uc791\uc740 \ube44\ub298\ub4e4\uc774 \ubab8\uc744 \ub36e\uae30 \uc2dc\uc791\ud55c \uac83\uc774 \ubcf4\uc778\ub2e4. \uc2e0\uc7a5\uc774 7.9 \uc13c\ud2f0\ubbf8\ud130\uc5d0 \ub2ec\ud558\uba74 \uc9c0\ub290\ub7ec\ubbf8\uc640 \uc0c9\uc18c\uac00 \uc81c\ub300\ub85c \ubc1c\ub2ec\ud588\uc73c\uba70, \ud06c\uae30\ub97c \uc81c\uc678\ud558\uba74 \uac89\ubcf4\uae30\uc5d0 \uc131\uccb4\uc640 \uac70\uc758 \ube44\uc2b7\ud574\uc9c4\ub2e4. \uc54c\uc774 \ubd80\ud654\ud558\ub294 \ub370\ub294 \uc218\uc628 11.3 \u00b0C (52.3 \u00b0F)\uc5d0\uc11c 15\uac1c\uc6d4, \uc218\uc628 14.5 \u00b0C (58.1 \u00b0F)\uc5d0\uc11c 7 ~ 9\uac1c\uc6d4\uc774 \uac78\ub9b0\ub2e4. \ub9c9 \ubd80\ud654\ud55c \uc0c8\ub07c \uc0c1\uc5b4\uc758 \uc2e0\uc7a5\uc740 8 \uc13c\ud2f0\ubbf8\ud130 \uc774\uc0c1\uc774\ub2e4. \uc0dd\uc2dd\uc774 \uac00\ub2a5\ud55c \uc131\uccb4\uc758 \uc2e0\uc7a5\uc740 \uc218\uc628\uc774 \ub0b4\ub824\uac10\uc5d0 \ub530\ub77c \uc99d\uac00\ud558\ub294 \uacbd\ud5a5\uc774 \uc788\ub2e4. \ud558\ucf54\ub2e4\ud14c \uac19\uc740 \ubd81\ucabd\uc5d0\uc11c\ub294 \uc554\uc218\uac00 \ubaa8\ub450 \uc2e0\uc7a5 38 \uc13c\ud2f0\ubbf8\ud130\ucbe4\uc5d0\uc11c \uc131\uc219\ud558\uba70, \uc77c\ubd80 \uc554\ucef7\uc740 \uc2e0\uc7a5 47 \uc13c\ud2f0\ubbf8\ud130\uac00 \ub118\uc5b4\ub3c4 \uc131\uc219\ud558\uc9c0 \uc54a\uae30\ub3c4 \ud55c\ub2e4. \ubc18\uba74 \ub300\ub9c8\ub3c4 \ubd80\uadfc\uc758 \ub0a8\ucabd\uc5d0\uc11c\ub294 \uc554\uc218\uac00 \ubaa8\ub450 \uc2e0\uc7a5 33 \uc13c\ud2f0\ubbf8\ud130\ucbe4\uc5d0\uc11c \uc131\uc219\ud55c\ub2e4. \ucd5c\ub300\uc218\uba85\uc740 12\ub144 \uc774\uc0c1\uc774\ub2e4."} {"question": "\uc911\uad6d \uc815\ubd80\ub294 \uc815\uc0c1\uc801\uc778 \ubb34\uc5ed \uad00\uacc4\ub97c \ud1b5\ud55c \uc218\ucd9c\uc774\ub77c \ud558\uc9c0\ub9cc, \ubd81\ud55c\uc774 \uc5f4\ubcd1\uc2dd\uc5d0\uc11c \uc120\ubcf4\uc778 \uc911\uad6d\uc81c \ubbf8\uc0ac\uc77c \uc774\ub984\uc740?", "paragraph": "\ud575 \ubb3c\uc790 \ubfd0\ub9cc \uc544\ub2c8\ub77c \ubd81\ud55c\uc774 \uc5f4\ubcd1\uc2dd\uc5d0 \uc120\ubcf4\uc778 \uc911\uad6d\uc81c \uc7a0\uc218\ud568\ubc1c\uc0ac\ud0c4\ub3c4\ubbf8\uc0ac\uc77c(SLBM)\uc5d0 \ub300\ud574 \uc911\uad6d \uc815\ubd80\ub294 \uc815\uc0c1\uc801\uc778 \ubb34\uc5ed \uad00\uacc4\uc5d0 \ub530\ub978 \uac83\uc774\ub77c\ub294 \uc8fc\uc7a5\uc744 \ud588\ub2e4. \uadf8\ub7ec\ub098 \uc720\uc5d4 \uc548\ubcf4\ub9ac \uacb0\uc758\uc5d0 \ub530\ub974\uba74 \ubd81\ud55c\uc5d0 \uad70\uc0ac \uc7a5\ube44\ub97c \uc218\ucd9c\ud558\uba74 \uc81c\uc7ac \uc704\ubc18\uc774 \ub41c\ub2e4. \ubd81\ud55c\uc774 \uc3dc \ud0c4\ub3c4\ubbf8\uc0ac\uc77c \uc794\ud574\ub97c \ubbf8\uad6d\uc774 \uc785\uc218\ud574 \ubd84\uc11d\ud55c \uacb0\uacfc, \ubd80\ud488 \uc81c\uc791\uc5d0 \uc911\uad6d\uc774 \uad00\uc5ec\ud55c \uc815\ud669\uc774 \ub4dc\ub7ec\ub0ac\ub2e4. \ubd81\ud55c\uc740 2012\ub144 \uc804\ud6c4\ub85c \ub300\ub959\uac04\ud0c4\ub3c4\ubbf8\uc0ac\uc77c(ICBM) \uc81c\uc870\uc5d0 \ud544\uc694\ud55c \ud575\uc2ec \uae30\uc220\uc744 \uc911\uad6d\uacfc\uc758 \uacf5\ub3d9 \uc5f0\uad6c \ud615\ud0dc\ub97c \ud1b5\ud574\uc11c \ud655\ubcf4\ud588\ub2e4. \uc911\uad6d\uc740 \ubd81\ud55c\uc5d0\uac8c \uac01\uc885 \ud0c4\ub3c4\ubbf8\uc0ac\uc77c\uc744 \uc81c\uacf5\ud574\uc92c\uc73c\uba74\uc11c \ud55c\uad6d\uc5d0\uac8c\ub294 \ubc29\uc5b4\uc801\uc778 \ubb34\uae30\uc870\ucc28\ub3c4 \ubc30\uce58\ud558\uc9c0 \ub9d0\ub77c\uba70 \ubc18\ub300\ud558\uace0 \uc788\ub2e4. \ubd81\ud55c\uc758 \ud575\ud0c4\ub450\uc640 \ud0c4\ub3c4\ubbf8\uc0ac\uc77c\uc740 '\ub85c\ub9e8\uc2a4'\uc774\uace0 \ud55c\uad6d\uc758 \uc0ac\ub4dc\ub294 '\ubd88\ub95c'\uc774\ub77c\ub294 \ub17c\ub9ac\uc778 \uac83\uc774\ub2e4. \ub610\ud55c \uc911\uad6d\uc740 \ubd81\ud55c\uc758 \uc0ac\uac70\ub9ac 4,000 km \ud0c4\ub3c4\ubbf8\uc0ac\uc77c\uc740 \uc804\ud600 \ubc18\ub300\ud558\uc9c0 \uc54a\uc73c\uba74\uc11c \ud55c\uad6d\uc758 \uc0ac\uac70\ub9ac 800 km \ud0c4\ub3c4\ubbf8\uc0ac\uc77c\uc740 \uac15\ub825\ud558\uac8c \ubc18\ub300\ud558\uace0 \uc788\ub2e4. \uc911\uad6d\uc740 \ud55c\ubc18\ub3c4 \ub0a8\ubd81\ud1b5\uc77c\uc744 \uc911\uad6d\uc5d0 \ub300\ud55c \uc704\ud611\uc73c\ub85c \ubcf4\uace0 \uc788\uace0 \ud55c\ubc18\ub3c4 \ub0a8\ubd81\ud1b5\uc77c\uc5d0 \ub300\ud574 \ubd80\uc815\uc801\uc774\ub2e4.", "answer": "\uc7a0\uc218\ud568\ubc1c\uc0ac\ud0c4\ub3c4\ubbf8\uc0ac\uc77c", "sentence": "\uc911\uad6d\uc81c \uc7a0\uc218\ud568\ubc1c\uc0ac\ud0c4\ub3c4\ubbf8\uc0ac\uc77c (SLBM)", "paragraph_sentence": "\ud575 \ubb3c\uc790 \ubfd0\ub9cc \uc544\ub2c8\ub77c \ubd81\ud55c\uc774 \uc5f4\ubcd1\uc2dd\uc5d0 \uc120\ubcf4\uc778 \uc911\uad6d\uc81c \uc7a0\uc218\ud568\ubc1c\uc0ac\ud0c4\ub3c4\ubbf8\uc0ac\uc77c (SLBM) \uc5d0 \ub300\ud574 \uc911\uad6d \uc815\ubd80\ub294 \uc815\uc0c1\uc801\uc778 \ubb34\uc5ed \uad00\uacc4\uc5d0 \ub530\ub978 \uac83\uc774\ub77c\ub294 \uc8fc\uc7a5\uc744 \ud588\ub2e4. \uadf8\ub7ec\ub098 \uc720\uc5d4 \uc548\ubcf4\ub9ac \uacb0\uc758\uc5d0 \ub530\ub974\uba74 \ubd81\ud55c\uc5d0 \uad70\uc0ac \uc7a5\ube44\ub97c \uc218\ucd9c\ud558\uba74 \uc81c\uc7ac \uc704\ubc18\uc774 \ub41c\ub2e4. \ubd81\ud55c\uc774 \uc3dc \ud0c4\ub3c4\ubbf8\uc0ac\uc77c \uc794\ud574\ub97c \ubbf8\uad6d\uc774 \uc785\uc218\ud574 \ubd84\uc11d\ud55c \uacb0\uacfc, \ubd80\ud488 \uc81c\uc791\uc5d0 \uc911\uad6d\uc774 \uad00\uc5ec\ud55c \uc815\ud669\uc774 \ub4dc\ub7ec\ub0ac\ub2e4. \ubd81\ud55c\uc740 2012\ub144 \uc804\ud6c4\ub85c \ub300\ub959\uac04\ud0c4\ub3c4\ubbf8\uc0ac\uc77c(ICBM) \uc81c\uc870\uc5d0 \ud544\uc694\ud55c \ud575\uc2ec \uae30\uc220\uc744 \uc911\uad6d\uacfc\uc758 \uacf5\ub3d9 \uc5f0\uad6c \ud615\ud0dc\ub97c \ud1b5\ud574\uc11c \ud655\ubcf4\ud588\ub2e4. \uc911\uad6d\uc740 \ubd81\ud55c\uc5d0\uac8c \uac01\uc885 \ud0c4\ub3c4\ubbf8\uc0ac\uc77c\uc744 \uc81c\uacf5\ud574\uc92c\uc73c\uba74\uc11c \ud55c\uad6d\uc5d0\uac8c\ub294 \ubc29\uc5b4\uc801\uc778 \ubb34\uae30\uc870\ucc28\ub3c4 \ubc30\uce58\ud558\uc9c0 \ub9d0\ub77c\uba70 \ubc18\ub300\ud558\uace0 \uc788\ub2e4. \ubd81\ud55c\uc758 \ud575\ud0c4\ub450\uc640 \ud0c4\ub3c4\ubbf8\uc0ac\uc77c\uc740 '\ub85c\ub9e8\uc2a4'\uc774\uace0 \ud55c\uad6d\uc758 \uc0ac\ub4dc\ub294 '\ubd88\ub95c'\uc774\ub77c\ub294 \ub17c\ub9ac\uc778 \uac83\uc774\ub2e4. \ub610\ud55c \uc911\uad6d\uc740 \ubd81\ud55c\uc758 \uc0ac\uac70\ub9ac 4,000 km \ud0c4\ub3c4\ubbf8\uc0ac\uc77c\uc740 \uc804\ud600 \ubc18\ub300\ud558\uc9c0 \uc54a\uc73c\uba74\uc11c \ud55c\uad6d\uc758 \uc0ac\uac70\ub9ac 800 km \ud0c4\ub3c4\ubbf8\uc0ac\uc77c\uc740 \uac15\ub825\ud558\uac8c \ubc18\ub300\ud558\uace0 \uc788\ub2e4. \uc911\uad6d\uc740 \ud55c\ubc18\ub3c4 \ub0a8\ubd81\ud1b5\uc77c\uc744 \uc911\uad6d\uc5d0 \ub300\ud55c \uc704\ud611\uc73c\ub85c \ubcf4\uace0 \uc788\uace0 \ud55c\ubc18\ub3c4 \ub0a8\ubd81\ud1b5\uc77c\uc5d0 \ub300\ud574 \ubd80\uc815\uc801\uc774\ub2e4.", "paragraph_answer": "\ud575 \ubb3c\uc790 \ubfd0\ub9cc \uc544\ub2c8\ub77c \ubd81\ud55c\uc774 \uc5f4\ubcd1\uc2dd\uc5d0 \uc120\ubcf4\uc778 \uc911\uad6d\uc81c \uc7a0\uc218\ud568\ubc1c\uc0ac\ud0c4\ub3c4\ubbf8\uc0ac\uc77c (SLBM)\uc5d0 \ub300\ud574 \uc911\uad6d \uc815\ubd80\ub294 \uc815\uc0c1\uc801\uc778 \ubb34\uc5ed \uad00\uacc4\uc5d0 \ub530\ub978 \uac83\uc774\ub77c\ub294 \uc8fc\uc7a5\uc744 \ud588\ub2e4. \uadf8\ub7ec\ub098 \uc720\uc5d4 \uc548\ubcf4\ub9ac \uacb0\uc758\uc5d0 \ub530\ub974\uba74 \ubd81\ud55c\uc5d0 \uad70\uc0ac \uc7a5\ube44\ub97c \uc218\ucd9c\ud558\uba74 \uc81c\uc7ac \uc704\ubc18\uc774 \ub41c\ub2e4. \ubd81\ud55c\uc774 \uc3dc \ud0c4\ub3c4\ubbf8\uc0ac\uc77c \uc794\ud574\ub97c \ubbf8\uad6d\uc774 \uc785\uc218\ud574 \ubd84\uc11d\ud55c \uacb0\uacfc, \ubd80\ud488 \uc81c\uc791\uc5d0 \uc911\uad6d\uc774 \uad00\uc5ec\ud55c \uc815\ud669\uc774 \ub4dc\ub7ec\ub0ac\ub2e4. \ubd81\ud55c\uc740 2012\ub144 \uc804\ud6c4\ub85c \ub300\ub959\uac04\ud0c4\ub3c4\ubbf8\uc0ac\uc77c(ICBM) \uc81c\uc870\uc5d0 \ud544\uc694\ud55c \ud575\uc2ec \uae30\uc220\uc744 \uc911\uad6d\uacfc\uc758 \uacf5\ub3d9 \uc5f0\uad6c \ud615\ud0dc\ub97c \ud1b5\ud574\uc11c \ud655\ubcf4\ud588\ub2e4. \uc911\uad6d\uc740 \ubd81\ud55c\uc5d0\uac8c \uac01\uc885 \ud0c4\ub3c4\ubbf8\uc0ac\uc77c\uc744 \uc81c\uacf5\ud574\uc92c\uc73c\uba74\uc11c \ud55c\uad6d\uc5d0\uac8c\ub294 \ubc29\uc5b4\uc801\uc778 \ubb34\uae30\uc870\ucc28\ub3c4 \ubc30\uce58\ud558\uc9c0 \ub9d0\ub77c\uba70 \ubc18\ub300\ud558\uace0 \uc788\ub2e4. \ubd81\ud55c\uc758 \ud575\ud0c4\ub450\uc640 \ud0c4\ub3c4\ubbf8\uc0ac\uc77c\uc740 '\ub85c\ub9e8\uc2a4'\uc774\uace0 \ud55c\uad6d\uc758 \uc0ac\ub4dc\ub294 '\ubd88\ub95c'\uc774\ub77c\ub294 \ub17c\ub9ac\uc778 \uac83\uc774\ub2e4. \ub610\ud55c \uc911\uad6d\uc740 \ubd81\ud55c\uc758 \uc0ac\uac70\ub9ac 4,000 km \ud0c4\ub3c4\ubbf8\uc0ac\uc77c\uc740 \uc804\ud600 \ubc18\ub300\ud558\uc9c0 \uc54a\uc73c\uba74\uc11c \ud55c\uad6d\uc758 \uc0ac\uac70\ub9ac 800 km \ud0c4\ub3c4\ubbf8\uc0ac\uc77c\uc740 \uac15\ub825\ud558\uac8c \ubc18\ub300\ud558\uace0 \uc788\ub2e4. \uc911\uad6d\uc740 \ud55c\ubc18\ub3c4 \ub0a8\ubd81\ud1b5\uc77c\uc744 \uc911\uad6d\uc5d0 \ub300\ud55c \uc704\ud611\uc73c\ub85c \ubcf4\uace0 \uc788\uace0 \ud55c\ubc18\ub3c4 \ub0a8\ubd81\ud1b5\uc77c\uc5d0 \ub300\ud574 \ubd80\uc815\uc801\uc774\ub2e4.", "sentence_answer": "\uc911\uad6d\uc81c \uc7a0\uc218\ud568\ubc1c\uc0ac\ud0c4\ub3c4\ubbf8\uc0ac\uc77c (SLBM)", "paragraph_id": "6546295-15-0", "paragraph_question": "question: \uc911\uad6d \uc815\ubd80\ub294 \uc815\uc0c1\uc801\uc778 \ubb34\uc5ed \uad00\uacc4\ub97c \ud1b5\ud55c \uc218\ucd9c\uc774\ub77c \ud558\uc9c0\ub9cc, \ubd81\ud55c\uc774 \uc5f4\ubcd1\uc2dd\uc5d0\uc11c \uc120\ubcf4\uc778 \uc911\uad6d\uc81c \ubbf8\uc0ac\uc77c \uc774\ub984\uc740?, context: \ud575 \ubb3c\uc790 \ubfd0\ub9cc \uc544\ub2c8\ub77c \ubd81\ud55c\uc774 \uc5f4\ubcd1\uc2dd\uc5d0 \uc120\ubcf4\uc778 \uc911\uad6d\uc81c \uc7a0\uc218\ud568\ubc1c\uc0ac\ud0c4\ub3c4\ubbf8\uc0ac\uc77c(SLBM)\uc5d0 \ub300\ud574 \uc911\uad6d \uc815\ubd80\ub294 \uc815\uc0c1\uc801\uc778 \ubb34\uc5ed \uad00\uacc4\uc5d0 \ub530\ub978 \uac83\uc774\ub77c\ub294 \uc8fc\uc7a5\uc744 \ud588\ub2e4. \uadf8\ub7ec\ub098 \uc720\uc5d4 \uc548\ubcf4\ub9ac \uacb0\uc758\uc5d0 \ub530\ub974\uba74 \ubd81\ud55c\uc5d0 \uad70\uc0ac \uc7a5\ube44\ub97c \uc218\ucd9c\ud558\uba74 \uc81c\uc7ac \uc704\ubc18\uc774 \ub41c\ub2e4. \ubd81\ud55c\uc774 \uc3dc \ud0c4\ub3c4\ubbf8\uc0ac\uc77c \uc794\ud574\ub97c \ubbf8\uad6d\uc774 \uc785\uc218\ud574 \ubd84\uc11d\ud55c \uacb0\uacfc, \ubd80\ud488 \uc81c\uc791\uc5d0 \uc911\uad6d\uc774 \uad00\uc5ec\ud55c \uc815\ud669\uc774 \ub4dc\ub7ec\ub0ac\ub2e4. \ubd81\ud55c\uc740 2012\ub144 \uc804\ud6c4\ub85c \ub300\ub959\uac04\ud0c4\ub3c4\ubbf8\uc0ac\uc77c(ICBM) \uc81c\uc870\uc5d0 \ud544\uc694\ud55c \ud575\uc2ec \uae30\uc220\uc744 \uc911\uad6d\uacfc\uc758 \uacf5\ub3d9 \uc5f0\uad6c \ud615\ud0dc\ub97c \ud1b5\ud574\uc11c \ud655\ubcf4\ud588\ub2e4. \uc911\uad6d\uc740 \ubd81\ud55c\uc5d0\uac8c \uac01\uc885 \ud0c4\ub3c4\ubbf8\uc0ac\uc77c\uc744 \uc81c\uacf5\ud574\uc92c\uc73c\uba74\uc11c \ud55c\uad6d\uc5d0\uac8c\ub294 \ubc29\uc5b4\uc801\uc778 \ubb34\uae30\uc870\ucc28\ub3c4 \ubc30\uce58\ud558\uc9c0 \ub9d0\ub77c\uba70 \ubc18\ub300\ud558\uace0 \uc788\ub2e4. \ubd81\ud55c\uc758 \ud575\ud0c4\ub450\uc640 \ud0c4\ub3c4\ubbf8\uc0ac\uc77c\uc740 '\ub85c\ub9e8\uc2a4'\uc774\uace0 \ud55c\uad6d\uc758 \uc0ac\ub4dc\ub294 '\ubd88\ub95c'\uc774\ub77c\ub294 \ub17c\ub9ac\uc778 \uac83\uc774\ub2e4. \ub610\ud55c \uc911\uad6d\uc740 \ubd81\ud55c\uc758 \uc0ac\uac70\ub9ac 4,000 km \ud0c4\ub3c4\ubbf8\uc0ac\uc77c\uc740 \uc804\ud600 \ubc18\ub300\ud558\uc9c0 \uc54a\uc73c\uba74\uc11c \ud55c\uad6d\uc758 \uc0ac\uac70\ub9ac 800 km \ud0c4\ub3c4\ubbf8\uc0ac\uc77c\uc740 \uac15\ub825\ud558\uac8c \ubc18\ub300\ud558\uace0 \uc788\ub2e4. \uc911\uad6d\uc740 \ud55c\ubc18\ub3c4 \ub0a8\ubd81\ud1b5\uc77c\uc744 \uc911\uad6d\uc5d0 \ub300\ud55c \uc704\ud611\uc73c\ub85c \ubcf4\uace0 \uc788\uace0 \ud55c\ubc18\ub3c4 \ub0a8\ubd81\ud1b5\uc77c\uc5d0 \ub300\ud574 \ubd80\uc815\uc801\uc774\ub2e4."} {"question": "2008\ub144 2\uc6d4 8\uc77c \uc13c\ud2b8\ub7f4\uc2dc\ud2f0 9\uce35 \ub300\ud68c\uc758\uc2e4\uc5d0\uc11c \uc9c4\ud589\ub41c \ub300\ud68c\ub294 \ubb34\uc5c7\uc778\uac00?", "paragraph": "\uacb0\uad6d 2008\ub144 2\uc6d4 8\uc77c, \uc13c\ud2b8\ub7f4\uc2dc\ud2f0 9\uce35 \ub300\ud68c\uc758\uc2e4\uc5d0\uc11c \uc9c4\ud589\ub41c \ubbfc\uc8fc\ub178\ub3d9\ub2f9 \uc784\uc2dc\ub2f9\ub300\ud68c\uc5d0\uc11c \uc790\uc8fc\ud30c \ub300\uc758\uc6d0\ub4e4\uc758 \ubc18\ub300\ub85c \ud601\uc2e0\uc548\uc758 \ud575\uc2ec \ub0b4\uc6a9\uc740 \ubd80\uacb0\ub418\uc5c8\uc73c\uba70 \uc790\uc8fc\ud30c\uc758 \uc218\uc815\uc548\ub300\ub85c \ud1b5\uacfc\ub418\uc5c8\ub2e4. \ud601\uc2e0\uc548\uc774 \ubd80\uacb0\ub418\uc790 \uc2ec\uc0c1\uc815 \uc704\uc6d0\uc7a5 \ub4f1 \ube44\ub300\uc704\uc6d0 \uc804\uc6d0\uc774 \uc0ac\ud1f4\ud558\uc600\uace0, \ubbfc\uc8fc\ub178\ub3d9\ub2f9 \uc81c1\uae30\uc640 \uc81c2\uae30 \uc9c0\ub3c4\ubd80\ub97c \uad6c\uc131\ud558\uc600\ub358 \uc778\ubb3c\ub4e4 \uac00\uc6b4\ub370 \uae40\ud61c\uacbd, \uc8fc\ub300\ud658, \uae40\uc885\ucca0, \uc2ec\uc7ac\uc625, \ud64d\uc2b9\ud558, \uae40\uae30\uc218 \ub4f1\uc774 \ud0c8\ub2f9\ud558\uc600\uc73c\uba70, \ubbfc\uc8fc\ub178\ub3d9\ub2f9 \uc18c\uc18d \uad6d\ud68c\uc758\uc6d0 \uc2ec\uc0c1\uc815, \ub178\ud68c\ucc2c, \uc870\uc2b9\uc218, \ub2e8\ubcd1\ud638\ub3c4 \ud0c8\ub2f9\ud558\uc600\ub2e4. \uc784\uc2dc\ub2f9\ub300\ud68c \uc774\ud6c4 2008\ub144 4\uc6d4\uae4c\uc9c0 \ubbfc\uc8fc\ub178\ub3d9\ub2f9\uc744 \ud0c8\ub2f9\ud55c \uc0ac\ub78c\uc740 16,904\uba85\uc5d0 \uc774\ub974\ub800\uace0, 2008\ub144 1\uc6d4 \ub2f9\uc2dc 11\ub9cc\uc5ec \uba85\uc5d0 \uc774\ub974\ub358 \ubbfc\uc8fc\ub178\ub3d9\ub2f9\uc758 \ub2f9\uc6d0 \uc218\ub294 9\ub9cc 4\ucc9c\uc5ec \uba85\uc73c\ub85c \uc904\uc5c8\ub2e4. \ubbfc\uc8fc\ub178\ub3d9\ub2f9\uc744 \ud0c8\ub2f9\ud55c \uc0ac\ub78c\ub4e4 \uc911 \uc77c\ubd80\ub294 2008\ub144 2\uc6d4 21\uc77c '\uc9c4\ubcf4\uc2e0\ub2f9\uc5f0\ub300\ud68c\uc758' \uc900\ube44\uc704\uc6d0\ud68c\ub97c \uacb0\uc131\ud558\uc600\uc73c\uba70, 2008\ub144 3\uc6d4 \uc9c4\ubcf4\uc2e0\ub2f9 \ucc3d\ub2f9\uc774 \uacf5\uc2dd \uc120\uc5b8\ub418\uc5c8\uace0, \ubbfc\uc8fc\ub178\ub3d9\ub2f9\uc740 \ubd84\ub2f9\ub418\uc5c8\ub2e4.", "answer": "\ubbfc\uc8fc\ub178\ub3d9\ub2f9 \uc784\uc2dc\ub2f9\ub300\ud68c", "sentence": "\uacb0\uad6d 2008\ub144 2\uc6d4 8\uc77c, \uc13c\ud2b8\ub7f4\uc2dc\ud2f0 9\uce35 \ub300\ud68c\uc758\uc2e4\uc5d0\uc11c \uc9c4\ud589\ub41c \ubbfc\uc8fc\ub178\ub3d9\ub2f9 \uc784\uc2dc\ub2f9\ub300\ud68c \uc5d0\uc11c \uc790\uc8fc\ud30c \ub300\uc758\uc6d0\ub4e4\uc758 \ubc18\ub300\ub85c \ud601\uc2e0\uc548\uc758 \ud575\uc2ec \ub0b4\uc6a9\uc740 \ubd80\uacb0\ub418\uc5c8\uc73c\uba70 \uc790\uc8fc\ud30c\uc758 \uc218\uc815\uc548\ub300\ub85c \ud1b5\uacfc\ub418\uc5c8\ub2e4.", "paragraph_sentence": " \uacb0\uad6d 2008\ub144 2\uc6d4 8\uc77c, \uc13c\ud2b8\ub7f4\uc2dc\ud2f0 9\uce35 \ub300\ud68c\uc758\uc2e4\uc5d0\uc11c \uc9c4\ud589\ub41c \ubbfc\uc8fc\ub178\ub3d9\ub2f9 \uc784\uc2dc\ub2f9\ub300\ud68c \uc5d0\uc11c \uc790\uc8fc\ud30c \ub300\uc758\uc6d0\ub4e4\uc758 \ubc18\ub300\ub85c \ud601\uc2e0\uc548\uc758 \ud575\uc2ec \ub0b4\uc6a9\uc740 \ubd80\uacb0\ub418\uc5c8\uc73c\uba70 \uc790\uc8fc\ud30c\uc758 \uc218\uc815\uc548\ub300\ub85c \ud1b5\uacfc\ub418\uc5c8\ub2e4. \ud601\uc2e0\uc548\uc774 \ubd80\uacb0\ub418\uc790 \uc2ec\uc0c1\uc815 \uc704\uc6d0\uc7a5 \ub4f1 \ube44\ub300\uc704\uc6d0 \uc804\uc6d0\uc774 \uc0ac\ud1f4\ud558\uc600\uace0, \ubbfc\uc8fc\ub178\ub3d9\ub2f9 \uc81c1\uae30\uc640 \uc81c2\uae30 \uc9c0\ub3c4\ubd80\ub97c \uad6c\uc131\ud558\uc600\ub358 \uc778\ubb3c\ub4e4 \uac00\uc6b4\ub370 \uae40\ud61c\uacbd, \uc8fc\ub300\ud658, \uae40\uc885\ucca0, \uc2ec\uc7ac\uc625, \ud64d\uc2b9\ud558, \uae40\uae30\uc218 \ub4f1\uc774 \ud0c8\ub2f9\ud558\uc600\uc73c\uba70, \ubbfc\uc8fc\ub178\ub3d9\ub2f9 \uc18c\uc18d \uad6d\ud68c\uc758\uc6d0 \uc2ec\uc0c1\uc815, \ub178\ud68c\ucc2c, \uc870\uc2b9\uc218, \ub2e8\ubcd1\ud638\ub3c4 \ud0c8\ub2f9\ud558\uc600\ub2e4. \uc784\uc2dc\ub2f9\ub300\ud68c \uc774\ud6c4 2008\ub144 4\uc6d4\uae4c\uc9c0 \ubbfc\uc8fc\ub178\ub3d9\ub2f9\uc744 \ud0c8\ub2f9\ud55c \uc0ac\ub78c\uc740 16,904\uba85\uc5d0 \uc774\ub974\ub800\uace0, 2008\ub144 1\uc6d4 \ub2f9\uc2dc 11\ub9cc\uc5ec \uba85\uc5d0 \uc774\ub974\ub358 \ubbfc\uc8fc\ub178\ub3d9\ub2f9\uc758 \ub2f9\uc6d0 \uc218\ub294 9\ub9cc 4\ucc9c\uc5ec \uba85\uc73c\ub85c \uc904\uc5c8\ub2e4. \ubbfc\uc8fc\ub178\ub3d9\ub2f9\uc744 \ud0c8\ub2f9\ud55c \uc0ac\ub78c\ub4e4 \uc911 \uc77c\ubd80\ub294 2008\ub144 2\uc6d4 21\uc77c '\uc9c4\ubcf4\uc2e0\ub2f9\uc5f0\ub300\ud68c\uc758' \uc900\ube44\uc704\uc6d0\ud68c\ub97c \uacb0\uc131\ud558\uc600\uc73c\uba70, 2008\ub144 3\uc6d4 \uc9c4\ubcf4\uc2e0\ub2f9 \ucc3d\ub2f9\uc774 \uacf5\uc2dd \uc120\uc5b8\ub418\uc5c8\uace0, \ubbfc\uc8fc\ub178\ub3d9\ub2f9\uc740 \ubd84\ub2f9\ub418\uc5c8\ub2e4.", "paragraph_answer": "\uacb0\uad6d 2008\ub144 2\uc6d4 8\uc77c, \uc13c\ud2b8\ub7f4\uc2dc\ud2f0 9\uce35 \ub300\ud68c\uc758\uc2e4\uc5d0\uc11c \uc9c4\ud589\ub41c \ubbfc\uc8fc\ub178\ub3d9\ub2f9 \uc784\uc2dc\ub2f9\ub300\ud68c \uc5d0\uc11c \uc790\uc8fc\ud30c \ub300\uc758\uc6d0\ub4e4\uc758 \ubc18\ub300\ub85c \ud601\uc2e0\uc548\uc758 \ud575\uc2ec \ub0b4\uc6a9\uc740 \ubd80\uacb0\ub418\uc5c8\uc73c\uba70 \uc790\uc8fc\ud30c\uc758 \uc218\uc815\uc548\ub300\ub85c \ud1b5\uacfc\ub418\uc5c8\ub2e4. \ud601\uc2e0\uc548\uc774 \ubd80\uacb0\ub418\uc790 \uc2ec\uc0c1\uc815 \uc704\uc6d0\uc7a5 \ub4f1 \ube44\ub300\uc704\uc6d0 \uc804\uc6d0\uc774 \uc0ac\ud1f4\ud558\uc600\uace0, \ubbfc\uc8fc\ub178\ub3d9\ub2f9 \uc81c1\uae30\uc640 \uc81c2\uae30 \uc9c0\ub3c4\ubd80\ub97c \uad6c\uc131\ud558\uc600\ub358 \uc778\ubb3c\ub4e4 \uac00\uc6b4\ub370 \uae40\ud61c\uacbd, \uc8fc\ub300\ud658, \uae40\uc885\ucca0, \uc2ec\uc7ac\uc625, \ud64d\uc2b9\ud558, \uae40\uae30\uc218 \ub4f1\uc774 \ud0c8\ub2f9\ud558\uc600\uc73c\uba70, \ubbfc\uc8fc\ub178\ub3d9\ub2f9 \uc18c\uc18d \uad6d\ud68c\uc758\uc6d0 \uc2ec\uc0c1\uc815, \ub178\ud68c\ucc2c, \uc870\uc2b9\uc218, \ub2e8\ubcd1\ud638\ub3c4 \ud0c8\ub2f9\ud558\uc600\ub2e4. \uc784\uc2dc\ub2f9\ub300\ud68c \uc774\ud6c4 2008\ub144 4\uc6d4\uae4c\uc9c0 \ubbfc\uc8fc\ub178\ub3d9\ub2f9\uc744 \ud0c8\ub2f9\ud55c \uc0ac\ub78c\uc740 16,904\uba85\uc5d0 \uc774\ub974\ub800\uace0, 2008\ub144 1\uc6d4 \ub2f9\uc2dc 11\ub9cc\uc5ec \uba85\uc5d0 \uc774\ub974\ub358 \ubbfc\uc8fc\ub178\ub3d9\ub2f9\uc758 \ub2f9\uc6d0 \uc218\ub294 9\ub9cc 4\ucc9c\uc5ec \uba85\uc73c\ub85c \uc904\uc5c8\ub2e4. \ubbfc\uc8fc\ub178\ub3d9\ub2f9\uc744 \ud0c8\ub2f9\ud55c \uc0ac\ub78c\ub4e4 \uc911 \uc77c\ubd80\ub294 2008\ub144 2\uc6d4 21\uc77c '\uc9c4\ubcf4\uc2e0\ub2f9\uc5f0\ub300\ud68c\uc758' \uc900\ube44\uc704\uc6d0\ud68c\ub97c \uacb0\uc131\ud558\uc600\uc73c\uba70, 2008\ub144 3\uc6d4 \uc9c4\ubcf4\uc2e0\ub2f9 \ucc3d\ub2f9\uc774 \uacf5\uc2dd \uc120\uc5b8\ub418\uc5c8\uace0, \ubbfc\uc8fc\ub178\ub3d9\ub2f9\uc740 \ubd84\ub2f9\ub418\uc5c8\ub2e4.", "sentence_answer": "\uacb0\uad6d 2008\ub144 2\uc6d4 8\uc77c, \uc13c\ud2b8\ub7f4\uc2dc\ud2f0 9\uce35 \ub300\ud68c\uc758\uc2e4\uc5d0\uc11c \uc9c4\ud589\ub41c \ubbfc\uc8fc\ub178\ub3d9\ub2f9 \uc784\uc2dc\ub2f9\ub300\ud68c \uc5d0\uc11c \uc790\uc8fc\ud30c \ub300\uc758\uc6d0\ub4e4\uc758 \ubc18\ub300\ub85c \ud601\uc2e0\uc548\uc758 \ud575\uc2ec \ub0b4\uc6a9\uc740 \ubd80\uacb0\ub418\uc5c8\uc73c\uba70 \uc790\uc8fc\ud30c\uc758 \uc218\uc815\uc548\ub300\ub85c \ud1b5\uacfc\ub418\uc5c8\ub2e4.", "paragraph_id": "6515322-1-0", "paragraph_question": "question: 2008\ub144 2\uc6d4 8\uc77c \uc13c\ud2b8\ub7f4\uc2dc\ud2f0 9\uce35 \ub300\ud68c\uc758\uc2e4\uc5d0\uc11c \uc9c4\ud589\ub41c \ub300\ud68c\ub294 \ubb34\uc5c7\uc778\uac00?, context: \uacb0\uad6d 2008\ub144 2\uc6d4 8\uc77c, \uc13c\ud2b8\ub7f4\uc2dc\ud2f0 9\uce35 \ub300\ud68c\uc758\uc2e4\uc5d0\uc11c \uc9c4\ud589\ub41c \ubbfc\uc8fc\ub178\ub3d9\ub2f9 \uc784\uc2dc\ub2f9\ub300\ud68c\uc5d0\uc11c \uc790\uc8fc\ud30c \ub300\uc758\uc6d0\ub4e4\uc758 \ubc18\ub300\ub85c \ud601\uc2e0\uc548\uc758 \ud575\uc2ec \ub0b4\uc6a9\uc740 \ubd80\uacb0\ub418\uc5c8\uc73c\uba70 \uc790\uc8fc\ud30c\uc758 \uc218\uc815\uc548\ub300\ub85c \ud1b5\uacfc\ub418\uc5c8\ub2e4. \ud601\uc2e0\uc548\uc774 \ubd80\uacb0\ub418\uc790 \uc2ec\uc0c1\uc815 \uc704\uc6d0\uc7a5 \ub4f1 \ube44\ub300\uc704\uc6d0 \uc804\uc6d0\uc774 \uc0ac\ud1f4\ud558\uc600\uace0, \ubbfc\uc8fc\ub178\ub3d9\ub2f9 \uc81c1\uae30\uc640 \uc81c2\uae30 \uc9c0\ub3c4\ubd80\ub97c \uad6c\uc131\ud558\uc600\ub358 \uc778\ubb3c\ub4e4 \uac00\uc6b4\ub370 \uae40\ud61c\uacbd, \uc8fc\ub300\ud658, \uae40\uc885\ucca0, \uc2ec\uc7ac\uc625, \ud64d\uc2b9\ud558, \uae40\uae30\uc218 \ub4f1\uc774 \ud0c8\ub2f9\ud558\uc600\uc73c\uba70, \ubbfc\uc8fc\ub178\ub3d9\ub2f9 \uc18c\uc18d \uad6d\ud68c\uc758\uc6d0 \uc2ec\uc0c1\uc815, \ub178\ud68c\ucc2c, \uc870\uc2b9\uc218, \ub2e8\ubcd1\ud638\ub3c4 \ud0c8\ub2f9\ud558\uc600\ub2e4. \uc784\uc2dc\ub2f9\ub300\ud68c \uc774\ud6c4 2008\ub144 4\uc6d4\uae4c\uc9c0 \ubbfc\uc8fc\ub178\ub3d9\ub2f9\uc744 \ud0c8\ub2f9\ud55c \uc0ac\ub78c\uc740 16,904\uba85\uc5d0 \uc774\ub974\ub800\uace0, 2008\ub144 1\uc6d4 \ub2f9\uc2dc 11\ub9cc\uc5ec \uba85\uc5d0 \uc774\ub974\ub358 \ubbfc\uc8fc\ub178\ub3d9\ub2f9\uc758 \ub2f9\uc6d0 \uc218\ub294 9\ub9cc 4\ucc9c\uc5ec \uba85\uc73c\ub85c \uc904\uc5c8\ub2e4. \ubbfc\uc8fc\ub178\ub3d9\ub2f9\uc744 \ud0c8\ub2f9\ud55c \uc0ac\ub78c\ub4e4 \uc911 \uc77c\ubd80\ub294 2008\ub144 2\uc6d4 21\uc77c '\uc9c4\ubcf4\uc2e0\ub2f9\uc5f0\ub300\ud68c\uc758' \uc900\ube44\uc704\uc6d0\ud68c\ub97c \uacb0\uc131\ud558\uc600\uc73c\uba70, 2008\ub144 3\uc6d4 \uc9c4\ubcf4\uc2e0\ub2f9 \ucc3d\ub2f9\uc774 \uacf5\uc2dd \uc120\uc5b8\ub418\uc5c8\uace0, \ubbfc\uc8fc\ub178\ub3d9\ub2f9\uc740 \ubd84\ub2f9\ub418\uc5c8\ub2e4."} {"question": "\uc7ac\ud6c8, \uc601\ub144, \uc870\uc11c\uad50, \uc721\ud604\uc5d0\uac8c \uc2a4\uc2a4\ub85c \ubaa9\uc228\uc744 \ub04a\uac8c \ud558\uace0 \uba87 \uc0ac\ub78c\uc744 \uc0ac\ud615\uc2dc\ud0a8 \ub0a0\uc9dc\ub294?", "paragraph": "\uadf8\ub7ec\ub098 \uac15\ud654\uc758 \uae30\ubcf8\uc801\uc778 \uc6d0\uce59\uc774 \uc815\ud574\uc84c\ub2e4 \ud558\ub354\ub77c\ub3c4 \uc138\ubd80\uc0ac\ud56d\uc758 \ud1a0\uc758\ub294 \uac04\ub2e8\ud558\uc9c0\uac00 \uc54a\uc558\ub2e4. \uc6b0\uc120 \ub3c5\uc77c\uc81c\uad6d\uc774 \ub4e4\uace0\ub098\uc628 \uc758\ud654\ub2e8 \ucc98\ubc8c\uacfc \ucc45\uc784\uc790\uc758 \uc9d5\uacc4\ubb38\uc81c, \uc989 \uc758\ud654\ub2e8\uc744 \uc9c0\uc9c0\ud55c \uc655\uacfc \ub300\uc2e0\ub4e4\uc758 \ucc98\ubc8c\ubb38\uc81c\uac00 \uc81c\uae30\ub418\uc5c8\ub2e4. \uccad\ub098\ub77c \uc870\uc815\uc5d0\uc11c\ub294 1900\ub144 9\uc6d4 25\uc77c\uc5d0 \uc7a5\uce5c\uc655 \uc7ac\ud6c8, \uc774\uce5c\uc655 \ubd80\uc815, \ud328\ub975 \uc7ac\ub834, \ub2e8\uad70\uc655 \uc7ac\uc758, \ubcf4\uad6d\uacf5 \uc7ac\ub780, \ub3c4\ucc30\uc6d0\uc88c\ub3c4\uc5b4\uc0ac \uc601\ub144(\u82f1\u5e74), \ud611\ud310\ub300\ud559\uc0ac \uc774\ubd80\uc0c1\uc11c \uac15\uc758(\u525b\u7fa9), \ud615\ubd80\uc0c1\uc11c \uc870\uc11c\uad50 \ub4f1\uc5d0 \ub300\ud558\uc5ec \uc791\uc704\ub97c \ubc15\ud0c8\ud558\uace0 \ud30c\uc9c1\ud558\ub294 \ub4f1 \ucc98\ubc8c\uc744 \ub0b4\ub838\ub2e4. \uadf8\ub7ec\ub098 \uc5f4\uac15\ub4e4\uc740 \uadf8 \ucc98\ubc8c\uc774 \uc57d\ud558\uace0 \ucc98\ubc8c\ubc1b\uc740 \uc0ac\ub78c\uc774 \uc801\ub2e4\uace0 \ud56d\uc758\ud558\uc600\ub2e4. \uc774\uc5d0 11\uc6d4 13\uc77c\uc5d0 \ub450 \ubc88\uc9f8 \ucc98\ubc8c\uc774 \uc788\uc5c8\ub294\ub370\ub3c4 \uc5f4\uac15\uc740 \uc5ec\uc804\ud788 \ubd88\ub9cc\uc744 \ub098\ud0c0\ub0c8\ub2e4. \uacb0\uad6d, 1901\ub144 2\uc6d4 13\uc77c\uacfc 2\uc6d4 21\uc77c\uc5d0 \uac78\uccd0 \uc138 \ubc88\uc9f8\uc640 \ub124 \ubc88\uc9f8\uc758 \ucc98\ubc8c\uc744 \ub2e8\ud589\ud558\uc5ec \uc7ac\ud6c8,\uc601\ub144,\uc870\uc11c\uad50,\uc721\ud604\uc5d0\uac8c \uc2a4\uc2a4\ub85c \ubaa9\uc228\uc744 \ub04a\uac8c \ud558\uace0 \uba87 \uc0ac\ub78c\uc744 \uc0ac\ud615\uc2dc\ucf30\ub2e4. \uadf8\ub9ac\uace0 \uc758\ud654\ub2e8\uc758 \uc18c\ud0d5\uc744 \uc694\uad6c\ud558\ub2e4\uac00 \ud30c\uc9c1\ub418\uc5c8\ub358 \uc0ac\ub78c\uc744 \ubcf5\uc9c1\uc2dc\ucf1c \uba85\uc608\ub97c \ud68c\ubcf5\uc2dc\ucf30\ub2e4.", "answer": "1901\ub144 2\uc6d4 13\uc77c", "sentence": "\uacb0\uad6d, 1901\ub144 2\uc6d4 13\uc77c \uacfc 2\uc6d4 21\uc77c\uc5d0 \uac78\uccd0 \uc138 \ubc88\uc9f8\uc640 \ub124 \ubc88\uc9f8\uc758 \ucc98\ubc8c\uc744 \ub2e8\ud589\ud558\uc5ec \uc7ac\ud6c8,\uc601\ub144,\uc870\uc11c\uad50,\uc721\ud604\uc5d0\uac8c \uc2a4\uc2a4\ub85c \ubaa9\uc228\uc744 \ub04a\uac8c \ud558\uace0 \uba87 \uc0ac\ub78c\uc744 \uc0ac\ud615\uc2dc\ucf30\ub2e4.", "paragraph_sentence": "\uadf8\ub7ec\ub098 \uac15\ud654\uc758 \uae30\ubcf8\uc801\uc778 \uc6d0\uce59\uc774 \uc815\ud574\uc84c\ub2e4 \ud558\ub354\ub77c\ub3c4 \uc138\ubd80\uc0ac\ud56d\uc758 \ud1a0\uc758\ub294 \uac04\ub2e8\ud558\uc9c0\uac00 \uc54a\uc558\ub2e4. \uc6b0\uc120 \ub3c5\uc77c\uc81c\uad6d\uc774 \ub4e4\uace0\ub098\uc628 \uc758\ud654\ub2e8 \ucc98\ubc8c\uacfc \ucc45\uc784\uc790\uc758 \uc9d5\uacc4\ubb38\uc81c, \uc989 \uc758\ud654\ub2e8\uc744 \uc9c0\uc9c0\ud55c \uc655\uacfc \ub300\uc2e0\ub4e4\uc758 \ucc98\ubc8c\ubb38\uc81c\uac00 \uc81c\uae30\ub418\uc5c8\ub2e4. \uccad\ub098\ub77c \uc870\uc815\uc5d0\uc11c\ub294 1900\ub144 9\uc6d4 25\uc77c\uc5d0 \uc7a5\uce5c\uc655 \uc7ac\ud6c8, \uc774\uce5c\uc655 \ubd80\uc815, \ud328\ub975 \uc7ac\ub834, \ub2e8\uad70\uc655 \uc7ac\uc758, \ubcf4\uad6d\uacf5 \uc7ac\ub780, \ub3c4\ucc30\uc6d0\uc88c\ub3c4\uc5b4\uc0ac \uc601\ub144(\u82f1\u5e74), \ud611\ud310\ub300\ud559\uc0ac \uc774\ubd80\uc0c1\uc11c \uac15\uc758(\u525b\u7fa9), \ud615\ubd80\uc0c1\uc11c \uc870\uc11c\uad50 \ub4f1\uc5d0 \ub300\ud558\uc5ec \uc791\uc704\ub97c \ubc15\ud0c8\ud558\uace0 \ud30c\uc9c1\ud558\ub294 \ub4f1 \ucc98\ubc8c\uc744 \ub0b4\ub838\ub2e4. \uadf8\ub7ec\ub098 \uc5f4\uac15\ub4e4\uc740 \uadf8 \ucc98\ubc8c\uc774 \uc57d\ud558\uace0 \ucc98\ubc8c\ubc1b\uc740 \uc0ac\ub78c\uc774 \uc801\ub2e4\uace0 \ud56d\uc758\ud558\uc600\ub2e4. \uc774\uc5d0 11\uc6d4 13\uc77c\uc5d0 \ub450 \ubc88\uc9f8 \ucc98\ubc8c\uc774 \uc788\uc5c8\ub294\ub370\ub3c4 \uc5f4\uac15\uc740 \uc5ec\uc804\ud788 \ubd88\ub9cc\uc744 \ub098\ud0c0\ub0c8\ub2e4. \uacb0\uad6d, 1901\ub144 2\uc6d4 13\uc77c \uacfc 2\uc6d4 21\uc77c\uc5d0 \uac78\uccd0 \uc138 \ubc88\uc9f8\uc640 \ub124 \ubc88\uc9f8\uc758 \ucc98\ubc8c\uc744 \ub2e8\ud589\ud558\uc5ec \uc7ac\ud6c8,\uc601\ub144,\uc870\uc11c\uad50,\uc721\ud604\uc5d0\uac8c \uc2a4\uc2a4\ub85c \ubaa9\uc228\uc744 \ub04a\uac8c \ud558\uace0 \uba87 \uc0ac\ub78c\uc744 \uc0ac\ud615\uc2dc\ucf30\ub2e4. \uadf8\ub9ac\uace0 \uc758\ud654\ub2e8\uc758 \uc18c\ud0d5\uc744 \uc694\uad6c\ud558\ub2e4\uac00 \ud30c\uc9c1\ub418\uc5c8\ub358 \uc0ac\ub78c\uc744 \ubcf5\uc9c1\uc2dc\ucf1c \uba85\uc608\ub97c \ud68c\ubcf5\uc2dc\ucf30\ub2e4.", "paragraph_answer": "\uadf8\ub7ec\ub098 \uac15\ud654\uc758 \uae30\ubcf8\uc801\uc778 \uc6d0\uce59\uc774 \uc815\ud574\uc84c\ub2e4 \ud558\ub354\ub77c\ub3c4 \uc138\ubd80\uc0ac\ud56d\uc758 \ud1a0\uc758\ub294 \uac04\ub2e8\ud558\uc9c0\uac00 \uc54a\uc558\ub2e4. \uc6b0\uc120 \ub3c5\uc77c\uc81c\uad6d\uc774 \ub4e4\uace0\ub098\uc628 \uc758\ud654\ub2e8 \ucc98\ubc8c\uacfc \ucc45\uc784\uc790\uc758 \uc9d5\uacc4\ubb38\uc81c, \uc989 \uc758\ud654\ub2e8\uc744 \uc9c0\uc9c0\ud55c \uc655\uacfc \ub300\uc2e0\ub4e4\uc758 \ucc98\ubc8c\ubb38\uc81c\uac00 \uc81c\uae30\ub418\uc5c8\ub2e4. \uccad\ub098\ub77c \uc870\uc815\uc5d0\uc11c\ub294 1900\ub144 9\uc6d4 25\uc77c\uc5d0 \uc7a5\uce5c\uc655 \uc7ac\ud6c8, \uc774\uce5c\uc655 \ubd80\uc815, \ud328\ub975 \uc7ac\ub834, \ub2e8\uad70\uc655 \uc7ac\uc758, \ubcf4\uad6d\uacf5 \uc7ac\ub780, \ub3c4\ucc30\uc6d0\uc88c\ub3c4\uc5b4\uc0ac \uc601\ub144(\u82f1\u5e74), \ud611\ud310\ub300\ud559\uc0ac \uc774\ubd80\uc0c1\uc11c \uac15\uc758(\u525b\u7fa9), \ud615\ubd80\uc0c1\uc11c \uc870\uc11c\uad50 \ub4f1\uc5d0 \ub300\ud558\uc5ec \uc791\uc704\ub97c \ubc15\ud0c8\ud558\uace0 \ud30c\uc9c1\ud558\ub294 \ub4f1 \ucc98\ubc8c\uc744 \ub0b4\ub838\ub2e4. \uadf8\ub7ec\ub098 \uc5f4\uac15\ub4e4\uc740 \uadf8 \ucc98\ubc8c\uc774 \uc57d\ud558\uace0 \ucc98\ubc8c\ubc1b\uc740 \uc0ac\ub78c\uc774 \uc801\ub2e4\uace0 \ud56d\uc758\ud558\uc600\ub2e4. \uc774\uc5d0 11\uc6d4 13\uc77c\uc5d0 \ub450 \ubc88\uc9f8 \ucc98\ubc8c\uc774 \uc788\uc5c8\ub294\ub370\ub3c4 \uc5f4\uac15\uc740 \uc5ec\uc804\ud788 \ubd88\ub9cc\uc744 \ub098\ud0c0\ub0c8\ub2e4. \uacb0\uad6d, 1901\ub144 2\uc6d4 13\uc77c \uacfc 2\uc6d4 21\uc77c\uc5d0 \uac78\uccd0 \uc138 \ubc88\uc9f8\uc640 \ub124 \ubc88\uc9f8\uc758 \ucc98\ubc8c\uc744 \ub2e8\ud589\ud558\uc5ec \uc7ac\ud6c8,\uc601\ub144,\uc870\uc11c\uad50,\uc721\ud604\uc5d0\uac8c \uc2a4\uc2a4\ub85c \ubaa9\uc228\uc744 \ub04a\uac8c \ud558\uace0 \uba87 \uc0ac\ub78c\uc744 \uc0ac\ud615\uc2dc\ucf30\ub2e4. \uadf8\ub9ac\uace0 \uc758\ud654\ub2e8\uc758 \uc18c\ud0d5\uc744 \uc694\uad6c\ud558\ub2e4\uac00 \ud30c\uc9c1\ub418\uc5c8\ub358 \uc0ac\ub78c\uc744 \ubcf5\uc9c1\uc2dc\ucf1c \uba85\uc608\ub97c \ud68c\ubcf5\uc2dc\ucf30\ub2e4.", "sentence_answer": "\uacb0\uad6d, 1901\ub144 2\uc6d4 13\uc77c \uacfc 2\uc6d4 21\uc77c\uc5d0 \uac78\uccd0 \uc138 \ubc88\uc9f8\uc640 \ub124 \ubc88\uc9f8\uc758 \ucc98\ubc8c\uc744 \ub2e8\ud589\ud558\uc5ec \uc7ac\ud6c8,\uc601\ub144,\uc870\uc11c\uad50,\uc721\ud604\uc5d0\uac8c \uc2a4\uc2a4\ub85c \ubaa9\uc228\uc744 \ub04a\uac8c \ud558\uace0 \uba87 \uc0ac\ub78c\uc744 \uc0ac\ud615\uc2dc\ucf30\ub2e4.", "paragraph_id": "6544419-22-2", "paragraph_question": "question: \uc7ac\ud6c8, \uc601\ub144, \uc870\uc11c\uad50, \uc721\ud604\uc5d0\uac8c \uc2a4\uc2a4\ub85c \ubaa9\uc228\uc744 \ub04a\uac8c \ud558\uace0 \uba87 \uc0ac\ub78c\uc744 \uc0ac\ud615\uc2dc\ud0a8 \ub0a0\uc9dc\ub294?, context: \uadf8\ub7ec\ub098 \uac15\ud654\uc758 \uae30\ubcf8\uc801\uc778 \uc6d0\uce59\uc774 \uc815\ud574\uc84c\ub2e4 \ud558\ub354\ub77c\ub3c4 \uc138\ubd80\uc0ac\ud56d\uc758 \ud1a0\uc758\ub294 \uac04\ub2e8\ud558\uc9c0\uac00 \uc54a\uc558\ub2e4. \uc6b0\uc120 \ub3c5\uc77c\uc81c\uad6d\uc774 \ub4e4\uace0\ub098\uc628 \uc758\ud654\ub2e8 \ucc98\ubc8c\uacfc \ucc45\uc784\uc790\uc758 \uc9d5\uacc4\ubb38\uc81c, \uc989 \uc758\ud654\ub2e8\uc744 \uc9c0\uc9c0\ud55c \uc655\uacfc \ub300\uc2e0\ub4e4\uc758 \ucc98\ubc8c\ubb38\uc81c\uac00 \uc81c\uae30\ub418\uc5c8\ub2e4. \uccad\ub098\ub77c \uc870\uc815\uc5d0\uc11c\ub294 1900\ub144 9\uc6d4 25\uc77c\uc5d0 \uc7a5\uce5c\uc655 \uc7ac\ud6c8, \uc774\uce5c\uc655 \ubd80\uc815, \ud328\ub975 \uc7ac\ub834, \ub2e8\uad70\uc655 \uc7ac\uc758, \ubcf4\uad6d\uacf5 \uc7ac\ub780, \ub3c4\ucc30\uc6d0\uc88c\ub3c4\uc5b4\uc0ac \uc601\ub144(\u82f1\u5e74), \ud611\ud310\ub300\ud559\uc0ac \uc774\ubd80\uc0c1\uc11c \uac15\uc758(\u525b\u7fa9), \ud615\ubd80\uc0c1\uc11c \uc870\uc11c\uad50 \ub4f1\uc5d0 \ub300\ud558\uc5ec \uc791\uc704\ub97c \ubc15\ud0c8\ud558\uace0 \ud30c\uc9c1\ud558\ub294 \ub4f1 \ucc98\ubc8c\uc744 \ub0b4\ub838\ub2e4. \uadf8\ub7ec\ub098 \uc5f4\uac15\ub4e4\uc740 \uadf8 \ucc98\ubc8c\uc774 \uc57d\ud558\uace0 \ucc98\ubc8c\ubc1b\uc740 \uc0ac\ub78c\uc774 \uc801\ub2e4\uace0 \ud56d\uc758\ud558\uc600\ub2e4. \uc774\uc5d0 11\uc6d4 13\uc77c\uc5d0 \ub450 \ubc88\uc9f8 \ucc98\ubc8c\uc774 \uc788\uc5c8\ub294\ub370\ub3c4 \uc5f4\uac15\uc740 \uc5ec\uc804\ud788 \ubd88\ub9cc\uc744 \ub098\ud0c0\ub0c8\ub2e4. \uacb0\uad6d, 1901\ub144 2\uc6d4 13\uc77c\uacfc 2\uc6d4 21\uc77c\uc5d0 \uac78\uccd0 \uc138 \ubc88\uc9f8\uc640 \ub124 \ubc88\uc9f8\uc758 \ucc98\ubc8c\uc744 \ub2e8\ud589\ud558\uc5ec \uc7ac\ud6c8,\uc601\ub144,\uc870\uc11c\uad50,\uc721\ud604\uc5d0\uac8c \uc2a4\uc2a4\ub85c \ubaa9\uc228\uc744 \ub04a\uac8c \ud558\uace0 \uba87 \uc0ac\ub78c\uc744 \uc0ac\ud615\uc2dc\ucf30\ub2e4. \uadf8\ub9ac\uace0 \uc758\ud654\ub2e8\uc758 \uc18c\ud0d5\uc744 \uc694\uad6c\ud558\ub2e4\uac00 \ud30c\uc9c1\ub418\uc5c8\ub358 \uc0ac\ub78c\uc744 \ubcf5\uc9c1\uc2dc\ucf1c \uba85\uc608\ub97c \ud68c\ubcf5\uc2dc\ucf30\ub2e4."} {"question": "\uc870\uc9c0\uc544\ub294 2008\ub144 \ub7ec\uc2dc\uc544\uc640\uc758 \ubb34\uc5c7\uc744 \ub2e8\uc808\ud558\uae30\ub85c \uc120\uc5b8\ud558\uc600\ub098?", "paragraph": "2008\ub144 8\uc6d4\uc5d0 \uc870\uc9c0\uc544\uc640 \ub7ec\uc2dc\uc544\ub294 \uc9e7\uc9c0\ub9cc \ud53c\ud574\uac00 \ud070 \uc804\uc7c1\uc744 \ubc8c\uc600\uc73c\uba70 \uadf8 \uacb0\uacfc \ub3c5\ub9bd \uad6d\uac00 \uc5f0\ud569\uc5d0\uc11c \ud0c8\ud1f4\ud55c\ub2e4\uace0 \uc120\uc5b8\ud588\ub2e4. \uadf8\ub9ac\uace0 2008\ub144 8\uc6d4 30\uc77c\uc5d0 \uc870\uc9c0\uc544\ub294 \ub7ec\uc2dc\uc544\uc640\uc758 \uc678\uad50\uad00\uacc4\ub97c \ub2e8\uc808\ud558\uae30\ub85c \uc120\uc5b8\ud588\ub2e4. \ub7ec\uc2dc\uc544\uac00 \uc555\ud558\uc9c0\uc57c\uc640 \ub0a8\uc624\uc138\ud2f0\uc57c\uc758 \ub3c5\ub9bd\uc744 \uc2b9\uc778\ud558\uc790, \uc870\uc9c0\uc544\ub294 \uc790\uce58 \uacf5\ud654\uad6d\uacfc 1991\ub144\uc5d0 \ub9fa\uc5c8\ub358 \ud734\uc804 \ud611\uc815\uc744 \ud3d0\uae30\ud588\ub2e4. \uacb0\uad6d, 9\uc6d4 3\uc77c\uc5d0 \uc870\uc9c0\uc544\uc640 \ub7ec\uc2dc\uc544\uc758 \uc678\uad50\uad00\uacc4\ub294 \uacf5\uc2dd\uc801\uc73c\ub85c \ub2e8\uc808\ub418\uc5c8\ub2e4. \uc804\uc7c1\uc774 \ub05d\ub09c \ub4a4 9\uc6d4 23\uc77c\uc5d0\ub294 \uc870\uc9c0\uc544\ub294 \ub0a8\uc624\uc138\ud2f0\uc57c \ub0a8\ucabd\uc0c1\uacf5\uc744 \uc9c0\ub098\uace0 \uc788\ub358 \ub7ec\uc2dc\uc544 \uc815\ucc30\uae30\ub97c \uaca9\ucd94\ud588\ub2e4\uace0 \uc8fc\uc7a5\ud588\uc9c0\ub9cc, \ub7ec\uc2dc\uc544\uc5d0\uc120 \uc774\ub97c \ubd80\uc778\ud588\ub2e4. \ucd5c\uadfc BBC\ub294 \uc870\uc9c0\uc544\uad70\uc774 \ub0a8\uc624\uc138\ud2f0\uc57c\uc5d0\uc11c \ubbfc\uac04\uc778 \ud559\uc0b4\uc744 \uc790\ud589\ud588\ub2e4\uace0 \ubcf4\ub3c4\ud558\uc600\uc73c\ub098, \ubbf8\ud5e4\uc77c \uc0ac\uce74\uc2dc\ube4c\ub9ac \ub300\ud1b5\ub839\uc740 \uc774\ub97c \ubd80\uc778\ud588\ub2e4. \uc870\uc9c0\uc544\ub294 1994\ub144\uc5d0 \ub2c8\uce74\ub77c\uacfc\uc640 \uc678\uad50\uad00\uacc4\ub97c \ub9fa\uc5c8\uc9c0\ub9cc, \ub0a8\uc624\uc138\ud2f0\uc57c\uc640 \uc555\ud558\uc2a4 \uacf5\ud654\uad6d\uc758 \ub3c5\ub9bd\uc744 \uc778\uc815\ud588\ub2e4\ub294 \uc774\uc720\ub85c 2008\ub144 11\uc6d4 29\uc77c\uc5d0 \uc678\uad50\uad00\uacc4\ub97c \ub2e8\uc808\ud588\ub2e4. \uc870\uc9c0\uc544\uc5d0\uc11c\ub294 \ubc18\uc815\ubd80 \uc2dc\uc704\uac00 \uc790\uc8fc \uc77c\uc5b4\ub098\uace0 \uc788\uc73c\uba70 \uc57c\ub2f9\uacfc \uc815\ubd80 \uac04\uc5d0 \ucda9\ub3cc\uc774 \uc77c\uc5b4\ub0a8\uc5d0 \ub530\ub77c \uc815\uad6d \ubd88\uc548\uc774 \uace0\uc870\ub418\uace0 \uc788\ub2e4. \uc870\uc9c0\uc544\uc640 \ub7ec\uc2dc\uc544\ub294 \ub0a8\uc624\uc138\ud2f0\uc57c \uc804\uc7c1\uc774 \ub05d\ub09c \uc9c0 2\ub144\uc774 \ud758\ub800\uc9c0\ub9cc \uae34\uc7a5\uc740 \uc544\uc9c1 \ud480\ub9ac\uc9c0\uc54a\uace0 \uc788\ub2e4. \uc870\uc9c0\uc544\ub294 \ucd1d\uc120\uc5d0\uc11c \uce5c\ub7ec\uc131\ud5a5\uc758 \uc57c\ub2f9\uc774 \uc2b9\ub9ac\ud588\ub2e4. \uadf8\ub9ac\uace0 \ucd1d\uc120\uc5d0\uc11c \uc2b9\ub9ac\ud558\uba74\uc11c 2013\ub144\ubd80\ud130\ub294 \uc758\uc6d0\ub0b4\uac01\uc81c\ud615\ud0dc\uc758 \uc81c\ub3c4\ub97c \ub3c4\uc785\ud560 \uac83\uc774\ub77c\uace0 \ubc1d\ud614\ub2e4. \uc774\ubc14\ub2c8\uc2dc\ube4c\ub9ac\ub294 \ucd1d\uc120\uc5d0\uc11c \uc2b9\ub9ac\ud588\uc9c0\ub9cc, \uccab \ubc29\ubb38\uad6d\uc740 \ubbf8\uad6d\uc774\ub77c\uace0 \ubc1d\ud614\ub2e4. 2012\ub144 10\uc6d4 4\uc77c\uc5d0 \ubca8\ub77c\ub8e8\uc2a4\uc758 \uc54c\ub809\uc0b0\ub4dc\ub974 \ub8e8\uce74\uc174\ucf54\uac00 \uc628\uac16 \ubc29\ubc95\uc73c\ub85c \uc870\uc9c0\uc544\ub97c CIS \ubcf5\uadc0\ub97c \uc8fc\ub3c4\ud558\uaca0\ub2e4\uace0 \ubc1d\ud614\uc9c0\ub9cc, \uc870\uc9c0\uc544\uce21\uc5d0\uc11c\ub294 \ubca8\ub77c\ub8e8\uc2a4 \ub300\ud1b5\ub839\uc758 \ubc1c\uc5b8\uc740 \uc2e0\uc911\ud558\uc9c0 \ubabb\ud588\ub2e4\uace0 \ubc1d\ud614\ub2e4.", "answer": "\uc678\uad50\uad00\uacc4", "sentence": "\uadf8\ub9ac\uace0 2008\ub144 8\uc6d4 30\uc77c\uc5d0 \uc870\uc9c0\uc544\ub294 \ub7ec\uc2dc\uc544\uc640\uc758 \uc678\uad50\uad00\uacc4 \ub97c \ub2e8\uc808\ud558\uae30\ub85c \uc120\uc5b8\ud588\ub2e4.", "paragraph_sentence": "2008\ub144 8\uc6d4\uc5d0 \uc870\uc9c0\uc544\uc640 \ub7ec\uc2dc\uc544\ub294 \uc9e7\uc9c0\ub9cc \ud53c\ud574\uac00 \ud070 \uc804\uc7c1\uc744 \ubc8c\uc600\uc73c\uba70 \uadf8 \uacb0\uacfc \ub3c5\ub9bd \uad6d\uac00 \uc5f0\ud569\uc5d0\uc11c \ud0c8\ud1f4\ud55c\ub2e4\uace0 \uc120\uc5b8\ud588\ub2e4. \uadf8\ub9ac\uace0 2008\ub144 8\uc6d4 30\uc77c\uc5d0 \uc870\uc9c0\uc544\ub294 \ub7ec\uc2dc\uc544\uc640\uc758 \uc678\uad50\uad00\uacc4 \ub97c \ub2e8\uc808\ud558\uae30\ub85c \uc120\uc5b8\ud588\ub2e4. \ub7ec\uc2dc\uc544\uac00 \uc555\ud558\uc9c0\uc57c\uc640 \ub0a8\uc624\uc138\ud2f0\uc57c\uc758 \ub3c5\ub9bd\uc744 \uc2b9\uc778\ud558\uc790, \uc870\uc9c0\uc544\ub294 \uc790\uce58 \uacf5\ud654\uad6d\uacfc 1991\ub144\uc5d0 \ub9fa\uc5c8\ub358 \ud734\uc804 \ud611\uc815\uc744 \ud3d0\uae30\ud588\ub2e4. \uacb0\uad6d, 9\uc6d4 3\uc77c\uc5d0 \uc870\uc9c0\uc544\uc640 \ub7ec\uc2dc\uc544\uc758 \uc678\uad50\uad00\uacc4\ub294 \uacf5\uc2dd\uc801\uc73c\ub85c \ub2e8\uc808\ub418\uc5c8\ub2e4. \uc804\uc7c1\uc774 \ub05d\ub09c \ub4a4 9\uc6d4 23\uc77c\uc5d0\ub294 \uc870\uc9c0\uc544\ub294 \ub0a8\uc624\uc138\ud2f0\uc57c \ub0a8\ucabd\uc0c1\uacf5\uc744 \uc9c0\ub098\uace0 \uc788\ub358 \ub7ec\uc2dc\uc544 \uc815\ucc30\uae30\ub97c \uaca9\ucd94\ud588\ub2e4\uace0 \uc8fc\uc7a5\ud588\uc9c0\ub9cc, \ub7ec\uc2dc\uc544\uc5d0\uc120 \uc774\ub97c \ubd80\uc778\ud588\ub2e4. \ucd5c\uadfc BBC\ub294 \uc870\uc9c0\uc544\uad70\uc774 \ub0a8\uc624\uc138\ud2f0\uc57c\uc5d0\uc11c \ubbfc\uac04\uc778 \ud559\uc0b4\uc744 \uc790\ud589\ud588\ub2e4\uace0 \ubcf4\ub3c4\ud558\uc600\uc73c\ub098, \ubbf8\ud5e4\uc77c \uc0ac\uce74\uc2dc\ube4c\ub9ac \ub300\ud1b5\ub839\uc740 \uc774\ub97c \ubd80\uc778\ud588\ub2e4. \uc870\uc9c0\uc544\ub294 1994\ub144\uc5d0 \ub2c8\uce74\ub77c\uacfc\uc640 \uc678\uad50\uad00\uacc4\ub97c \ub9fa\uc5c8\uc9c0\ub9cc, \ub0a8\uc624\uc138\ud2f0\uc57c\uc640 \uc555\ud558\uc2a4 \uacf5\ud654\uad6d\uc758 \ub3c5\ub9bd\uc744 \uc778\uc815\ud588\ub2e4\ub294 \uc774\uc720\ub85c 2008\ub144 11\uc6d4 29\uc77c\uc5d0 \uc678\uad50\uad00\uacc4\ub97c \ub2e8\uc808\ud588\ub2e4. \uc870\uc9c0\uc544\uc5d0\uc11c\ub294 \ubc18\uc815\ubd80 \uc2dc\uc704\uac00 \uc790\uc8fc \uc77c\uc5b4\ub098\uace0 \uc788\uc73c\uba70 \uc57c\ub2f9\uacfc \uc815\ubd80 \uac04\uc5d0 \ucda9\ub3cc\uc774 \uc77c\uc5b4\ub0a8\uc5d0 \ub530\ub77c \uc815\uad6d \ubd88\uc548\uc774 \uace0\uc870\ub418\uace0 \uc788\ub2e4. \uc870\uc9c0\uc544\uc640 \ub7ec\uc2dc\uc544\ub294 \ub0a8\uc624\uc138\ud2f0\uc57c \uc804\uc7c1\uc774 \ub05d\ub09c \uc9c0 2\ub144\uc774 \ud758\ub800\uc9c0\ub9cc \uae34\uc7a5\uc740 \uc544\uc9c1 \ud480\ub9ac\uc9c0\uc54a\uace0 \uc788\ub2e4. \uc870\uc9c0\uc544\ub294 \ucd1d\uc120\uc5d0\uc11c \uce5c\ub7ec\uc131\ud5a5\uc758 \uc57c\ub2f9\uc774 \uc2b9\ub9ac\ud588\ub2e4. \uadf8\ub9ac\uace0 \ucd1d\uc120\uc5d0\uc11c \uc2b9\ub9ac\ud558\uba74\uc11c 2013\ub144\ubd80\ud130\ub294 \uc758\uc6d0\ub0b4\uac01\uc81c\ud615\ud0dc\uc758 \uc81c\ub3c4\ub97c \ub3c4\uc785\ud560 \uac83\uc774\ub77c\uace0 \ubc1d\ud614\ub2e4. \uc774\ubc14\ub2c8\uc2dc\ube4c\ub9ac\ub294 \ucd1d\uc120\uc5d0\uc11c \uc2b9\ub9ac\ud588\uc9c0\ub9cc, \uccab \ubc29\ubb38\uad6d\uc740 \ubbf8\uad6d\uc774\ub77c\uace0 \ubc1d\ud614\ub2e4. 2012\ub144 10\uc6d4 4\uc77c\uc5d0 \ubca8\ub77c\ub8e8\uc2a4\uc758 \uc54c\ub809\uc0b0\ub4dc\ub974 \ub8e8\uce74\uc174\ucf54\uac00 \uc628\uac16 \ubc29\ubc95\uc73c\ub85c \uc870\uc9c0\uc544\ub97c CIS \ubcf5\uadc0\ub97c \uc8fc\ub3c4\ud558\uaca0\ub2e4\uace0 \ubc1d\ud614\uc9c0\ub9cc, \uc870\uc9c0\uc544\uce21\uc5d0\uc11c\ub294 \ubca8\ub77c\ub8e8\uc2a4 \ub300\ud1b5\ub839\uc758 \ubc1c\uc5b8\uc740 \uc2e0\uc911\ud558\uc9c0 \ubabb\ud588\ub2e4\uace0 \ubc1d\ud614\ub2e4.", "paragraph_answer": "2008\ub144 8\uc6d4\uc5d0 \uc870\uc9c0\uc544\uc640 \ub7ec\uc2dc\uc544\ub294 \uc9e7\uc9c0\ub9cc \ud53c\ud574\uac00 \ud070 \uc804\uc7c1\uc744 \ubc8c\uc600\uc73c\uba70 \uadf8 \uacb0\uacfc \ub3c5\ub9bd \uad6d\uac00 \uc5f0\ud569\uc5d0\uc11c \ud0c8\ud1f4\ud55c\ub2e4\uace0 \uc120\uc5b8\ud588\ub2e4. \uadf8\ub9ac\uace0 2008\ub144 8\uc6d4 30\uc77c\uc5d0 \uc870\uc9c0\uc544\ub294 \ub7ec\uc2dc\uc544\uc640\uc758 \uc678\uad50\uad00\uacc4 \ub97c \ub2e8\uc808\ud558\uae30\ub85c \uc120\uc5b8\ud588\ub2e4. \ub7ec\uc2dc\uc544\uac00 \uc555\ud558\uc9c0\uc57c\uc640 \ub0a8\uc624\uc138\ud2f0\uc57c\uc758 \ub3c5\ub9bd\uc744 \uc2b9\uc778\ud558\uc790, \uc870\uc9c0\uc544\ub294 \uc790\uce58 \uacf5\ud654\uad6d\uacfc 1991\ub144\uc5d0 \ub9fa\uc5c8\ub358 \ud734\uc804 \ud611\uc815\uc744 \ud3d0\uae30\ud588\ub2e4. \uacb0\uad6d, 9\uc6d4 3\uc77c\uc5d0 \uc870\uc9c0\uc544\uc640 \ub7ec\uc2dc\uc544\uc758 \uc678\uad50\uad00\uacc4\ub294 \uacf5\uc2dd\uc801\uc73c\ub85c \ub2e8\uc808\ub418\uc5c8\ub2e4. \uc804\uc7c1\uc774 \ub05d\ub09c \ub4a4 9\uc6d4 23\uc77c\uc5d0\ub294 \uc870\uc9c0\uc544\ub294 \ub0a8\uc624\uc138\ud2f0\uc57c \ub0a8\ucabd\uc0c1\uacf5\uc744 \uc9c0\ub098\uace0 \uc788\ub358 \ub7ec\uc2dc\uc544 \uc815\ucc30\uae30\ub97c \uaca9\ucd94\ud588\ub2e4\uace0 \uc8fc\uc7a5\ud588\uc9c0\ub9cc, \ub7ec\uc2dc\uc544\uc5d0\uc120 \uc774\ub97c \ubd80\uc778\ud588\ub2e4. \ucd5c\uadfc BBC\ub294 \uc870\uc9c0\uc544\uad70\uc774 \ub0a8\uc624\uc138\ud2f0\uc57c\uc5d0\uc11c \ubbfc\uac04\uc778 \ud559\uc0b4\uc744 \uc790\ud589\ud588\ub2e4\uace0 \ubcf4\ub3c4\ud558\uc600\uc73c\ub098, \ubbf8\ud5e4\uc77c \uc0ac\uce74\uc2dc\ube4c\ub9ac \ub300\ud1b5\ub839\uc740 \uc774\ub97c \ubd80\uc778\ud588\ub2e4. \uc870\uc9c0\uc544\ub294 1994\ub144\uc5d0 \ub2c8\uce74\ub77c\uacfc\uc640 \uc678\uad50\uad00\uacc4\ub97c \ub9fa\uc5c8\uc9c0\ub9cc, \ub0a8\uc624\uc138\ud2f0\uc57c\uc640 \uc555\ud558\uc2a4 \uacf5\ud654\uad6d\uc758 \ub3c5\ub9bd\uc744 \uc778\uc815\ud588\ub2e4\ub294 \uc774\uc720\ub85c 2008\ub144 11\uc6d4 29\uc77c\uc5d0 \uc678\uad50\uad00\uacc4\ub97c \ub2e8\uc808\ud588\ub2e4. \uc870\uc9c0\uc544\uc5d0\uc11c\ub294 \ubc18\uc815\ubd80 \uc2dc\uc704\uac00 \uc790\uc8fc \uc77c\uc5b4\ub098\uace0 \uc788\uc73c\uba70 \uc57c\ub2f9\uacfc \uc815\ubd80 \uac04\uc5d0 \ucda9\ub3cc\uc774 \uc77c\uc5b4\ub0a8\uc5d0 \ub530\ub77c \uc815\uad6d \ubd88\uc548\uc774 \uace0\uc870\ub418\uace0 \uc788\ub2e4. \uc870\uc9c0\uc544\uc640 \ub7ec\uc2dc\uc544\ub294 \ub0a8\uc624\uc138\ud2f0\uc57c \uc804\uc7c1\uc774 \ub05d\ub09c \uc9c0 2\ub144\uc774 \ud758\ub800\uc9c0\ub9cc \uae34\uc7a5\uc740 \uc544\uc9c1 \ud480\ub9ac\uc9c0\uc54a\uace0 \uc788\ub2e4. \uc870\uc9c0\uc544\ub294 \ucd1d\uc120\uc5d0\uc11c \uce5c\ub7ec\uc131\ud5a5\uc758 \uc57c\ub2f9\uc774 \uc2b9\ub9ac\ud588\ub2e4. \uadf8\ub9ac\uace0 \ucd1d\uc120\uc5d0\uc11c \uc2b9\ub9ac\ud558\uba74\uc11c 2013\ub144\ubd80\ud130\ub294 \uc758\uc6d0\ub0b4\uac01\uc81c\ud615\ud0dc\uc758 \uc81c\ub3c4\ub97c \ub3c4\uc785\ud560 \uac83\uc774\ub77c\uace0 \ubc1d\ud614\ub2e4. \uc774\ubc14\ub2c8\uc2dc\ube4c\ub9ac\ub294 \ucd1d\uc120\uc5d0\uc11c \uc2b9\ub9ac\ud588\uc9c0\ub9cc, \uccab \ubc29\ubb38\uad6d\uc740 \ubbf8\uad6d\uc774\ub77c\uace0 \ubc1d\ud614\ub2e4. 2012\ub144 10\uc6d4 4\uc77c\uc5d0 \ubca8\ub77c\ub8e8\uc2a4\uc758 \uc54c\ub809\uc0b0\ub4dc\ub974 \ub8e8\uce74\uc174\ucf54\uac00 \uc628\uac16 \ubc29\ubc95\uc73c\ub85c \uc870\uc9c0\uc544\ub97c CIS \ubcf5\uadc0\ub97c \uc8fc\ub3c4\ud558\uaca0\ub2e4\uace0 \ubc1d\ud614\uc9c0\ub9cc, \uc870\uc9c0\uc544\uce21\uc5d0\uc11c\ub294 \ubca8\ub77c\ub8e8\uc2a4 \ub300\ud1b5\ub839\uc758 \ubc1c\uc5b8\uc740 \uc2e0\uc911\ud558\uc9c0 \ubabb\ud588\ub2e4\uace0 \ubc1d\ud614\ub2e4.", "sentence_answer": "\uadf8\ub9ac\uace0 2008\ub144 8\uc6d4 30\uc77c\uc5d0 \uc870\uc9c0\uc544\ub294 \ub7ec\uc2dc\uc544\uc640\uc758 \uc678\uad50\uad00\uacc4 \ub97c \ub2e8\uc808\ud558\uae30\ub85c \uc120\uc5b8\ud588\ub2e4.", "paragraph_id": "6595159-4-0", "paragraph_question": "question: \uc870\uc9c0\uc544\ub294 2008\ub144 \ub7ec\uc2dc\uc544\uc640\uc758 \ubb34\uc5c7\uc744 \ub2e8\uc808\ud558\uae30\ub85c \uc120\uc5b8\ud558\uc600\ub098?, context: 2008\ub144 8\uc6d4\uc5d0 \uc870\uc9c0\uc544\uc640 \ub7ec\uc2dc\uc544\ub294 \uc9e7\uc9c0\ub9cc \ud53c\ud574\uac00 \ud070 \uc804\uc7c1\uc744 \ubc8c\uc600\uc73c\uba70 \uadf8 \uacb0\uacfc \ub3c5\ub9bd \uad6d\uac00 \uc5f0\ud569\uc5d0\uc11c \ud0c8\ud1f4\ud55c\ub2e4\uace0 \uc120\uc5b8\ud588\ub2e4. \uadf8\ub9ac\uace0 2008\ub144 8\uc6d4 30\uc77c\uc5d0 \uc870\uc9c0\uc544\ub294 \ub7ec\uc2dc\uc544\uc640\uc758 \uc678\uad50\uad00\uacc4\ub97c \ub2e8\uc808\ud558\uae30\ub85c \uc120\uc5b8\ud588\ub2e4. \ub7ec\uc2dc\uc544\uac00 \uc555\ud558\uc9c0\uc57c\uc640 \ub0a8\uc624\uc138\ud2f0\uc57c\uc758 \ub3c5\ub9bd\uc744 \uc2b9\uc778\ud558\uc790, \uc870\uc9c0\uc544\ub294 \uc790\uce58 \uacf5\ud654\uad6d\uacfc 1991\ub144\uc5d0 \ub9fa\uc5c8\ub358 \ud734\uc804 \ud611\uc815\uc744 \ud3d0\uae30\ud588\ub2e4. \uacb0\uad6d, 9\uc6d4 3\uc77c\uc5d0 \uc870\uc9c0\uc544\uc640 \ub7ec\uc2dc\uc544\uc758 \uc678\uad50\uad00\uacc4\ub294 \uacf5\uc2dd\uc801\uc73c\ub85c \ub2e8\uc808\ub418\uc5c8\ub2e4. \uc804\uc7c1\uc774 \ub05d\ub09c \ub4a4 9\uc6d4 23\uc77c\uc5d0\ub294 \uc870\uc9c0\uc544\ub294 \ub0a8\uc624\uc138\ud2f0\uc57c \ub0a8\ucabd\uc0c1\uacf5\uc744 \uc9c0\ub098\uace0 \uc788\ub358 \ub7ec\uc2dc\uc544 \uc815\ucc30\uae30\ub97c \uaca9\ucd94\ud588\ub2e4\uace0 \uc8fc\uc7a5\ud588\uc9c0\ub9cc, \ub7ec\uc2dc\uc544\uc5d0\uc120 \uc774\ub97c \ubd80\uc778\ud588\ub2e4. \ucd5c\uadfc BBC\ub294 \uc870\uc9c0\uc544\uad70\uc774 \ub0a8\uc624\uc138\ud2f0\uc57c\uc5d0\uc11c \ubbfc\uac04\uc778 \ud559\uc0b4\uc744 \uc790\ud589\ud588\ub2e4\uace0 \ubcf4\ub3c4\ud558\uc600\uc73c\ub098, \ubbf8\ud5e4\uc77c \uc0ac\uce74\uc2dc\ube4c\ub9ac \ub300\ud1b5\ub839\uc740 \uc774\ub97c \ubd80\uc778\ud588\ub2e4. \uc870\uc9c0\uc544\ub294 1994\ub144\uc5d0 \ub2c8\uce74\ub77c\uacfc\uc640 \uc678\uad50\uad00\uacc4\ub97c \ub9fa\uc5c8\uc9c0\ub9cc, \ub0a8\uc624\uc138\ud2f0\uc57c\uc640 \uc555\ud558\uc2a4 \uacf5\ud654\uad6d\uc758 \ub3c5\ub9bd\uc744 \uc778\uc815\ud588\ub2e4\ub294 \uc774\uc720\ub85c 2008\ub144 11\uc6d4 29\uc77c\uc5d0 \uc678\uad50\uad00\uacc4\ub97c \ub2e8\uc808\ud588\ub2e4. \uc870\uc9c0\uc544\uc5d0\uc11c\ub294 \ubc18\uc815\ubd80 \uc2dc\uc704\uac00 \uc790\uc8fc \uc77c\uc5b4\ub098\uace0 \uc788\uc73c\uba70 \uc57c\ub2f9\uacfc \uc815\ubd80 \uac04\uc5d0 \ucda9\ub3cc\uc774 \uc77c\uc5b4\ub0a8\uc5d0 \ub530\ub77c \uc815\uad6d \ubd88\uc548\uc774 \uace0\uc870\ub418\uace0 \uc788\ub2e4. \uc870\uc9c0\uc544\uc640 \ub7ec\uc2dc\uc544\ub294 \ub0a8\uc624\uc138\ud2f0\uc57c \uc804\uc7c1\uc774 \ub05d\ub09c \uc9c0 2\ub144\uc774 \ud758\ub800\uc9c0\ub9cc \uae34\uc7a5\uc740 \uc544\uc9c1 \ud480\ub9ac\uc9c0\uc54a\uace0 \uc788\ub2e4. \uc870\uc9c0\uc544\ub294 \ucd1d\uc120\uc5d0\uc11c \uce5c\ub7ec\uc131\ud5a5\uc758 \uc57c\ub2f9\uc774 \uc2b9\ub9ac\ud588\ub2e4. \uadf8\ub9ac\uace0 \ucd1d\uc120\uc5d0\uc11c \uc2b9\ub9ac\ud558\uba74\uc11c 2013\ub144\ubd80\ud130\ub294 \uc758\uc6d0\ub0b4\uac01\uc81c\ud615\ud0dc\uc758 \uc81c\ub3c4\ub97c \ub3c4\uc785\ud560 \uac83\uc774\ub77c\uace0 \ubc1d\ud614\ub2e4. \uc774\ubc14\ub2c8\uc2dc\ube4c\ub9ac\ub294 \ucd1d\uc120\uc5d0\uc11c \uc2b9\ub9ac\ud588\uc9c0\ub9cc, \uccab \ubc29\ubb38\uad6d\uc740 \ubbf8\uad6d\uc774\ub77c\uace0 \ubc1d\ud614\ub2e4. 2012\ub144 10\uc6d4 4\uc77c\uc5d0 \ubca8\ub77c\ub8e8\uc2a4\uc758 \uc54c\ub809\uc0b0\ub4dc\ub974 \ub8e8\uce74\uc174\ucf54\uac00 \uc628\uac16 \ubc29\ubc95\uc73c\ub85c \uc870\uc9c0\uc544\ub97c CIS \ubcf5\uadc0\ub97c \uc8fc\ub3c4\ud558\uaca0\ub2e4\uace0 \ubc1d\ud614\uc9c0\ub9cc, \uc870\uc9c0\uc544\uce21\uc5d0\uc11c\ub294 \ubca8\ub77c\ub8e8\uc2a4 \ub300\ud1b5\ub839\uc758 \ubc1c\uc5b8\uc740 \uc2e0\uc911\ud558\uc9c0 \ubabb\ud588\ub2e4\uace0 \ubc1d\ud614\ub2e4."} {"question": "\uc6b0\uc8fc\uc120 \uc220\ub77c\ucf54\uc758 \ud654\uc7ac\ub85c \ub9ac\ud50c\ub9ac, \ub274\ud2b8, \ube44\uc20d, \ud789\uc2a4\uc0c1\ubcd1\uc774 \ubd88\uc2dc\ucc29\ud558\uac8c \ub41c \uacf3\uc740 \uc5b4\ub514\uc778\uac00?", "paragraph": "\uc9c0\uad6c\ub85c \ud5a5\ud558\ub358 \uc2dd\ubbfc\uc9c0 \ud574\ubcd1\ub300\uc18c\uc18d \uc6b0\uc8fc\uc120 \uc220\ub77c\ucf54(SULACO)\uc5d0\uc11c \ub3d9\uba74\uc744 \ud558\uace0 \uc788\ub358 \ub9ac\ud50c\ub9ac, \ub274\ud2b8, \ube44\uc20d, \ud789\uc2a4\uc0c1\ubcd1\uc740 \uc6b0\uc8fc\uc120\ub0b4\uc758 \ud654\uc7ac\ub85c \uc778\ud574 \uae34\uae09\ud0c8\ucd9c \uad6c\uba85\uc815\uc73c\ub85c \uc62e\uaca8\uc800 \uadfc\ucc98 \ud53c\uc624\ub9ac\ub098 161\ud589\uc131\uc5d0 \ubd88\uc2dc\ucc29 \ud558\uac8c\ub41c\ub2e4. \uc774 \ud589\uc131\uc740 \uc6e8\uc774\ub79c\ub4dc \uc720\ud0c0\ub2c8\uc758 \ub178\ub3d9\uad50\ub3c4\uc18c \ud589\uc131\uc73c\ub85c, \uc0b4\uc778\uc790\uc640 \uac15\uac04\ubc94\ub4f1 \uc774\uc911\uc5fc\uc0c9\uccb4\ub97c \ubcf4\uc720\ud55c \uc218\uc2ed\uba85\uc758 \uc8c4\uc218\ub4e4\uacfc \uba87\uba85\uc758 \uad50\ub3c4\uad00\ub4e4\uc774 \uac70\uc8fc\ud558\uace0, \uc790\ub825 \ud0c8\ucd9c\uc740 \ubd88\uac00\ub2a5\ud55c \ud589\uc131\uc774\uc600\ub2e4. \uc758\ubb34\uc2e4\uc5d0\uc11c \uae68\uc5b4\ub09c \ub9ac\ud50c\ub9ac\ub294 \uc790\uc2e0\uc744 \uc81c\uc678\ud55c \ub274\ud2b8, \ud789\uc2a4\ub294 \uad6c\uba85\uc815\uc5d0\uc11c \uc0b4\uc544\ub0a8\uc9c0 \ubabb\ud588\ub2e4\ub294 \uac83\uc744 \uc54c\uace0 \uad34\ub85c\uc6cc\ud55c\ub2e4. \uadf8\ub7ec\ub098 \uad6c\uba85\uc815 \ub0b4\uc5d0\uc11c \uc1e0\uac00 \ub179\uc740 \uc790\uad6d\uc744 \ubcf4\uace0 \uc5d0\uc774\ub9ac\uc5b8\uc774 \uac19\uc774 \ub530\ub77c\uc654\uc74c\uc744 \uc9c1\uac10\ud558\uace0, \ub274\ud2b8\uc758 \uc0ac\uccb4\ub97c \ud074\ub808\uba58\uc2a4\uc758 \ub3c4\uc6c0\uc744 \uc5bb\uc5b4 \ubd80\uac80\ud588\ub2e4. \ub2e4\ud589\ud788 \ub274\ud2b8\uc758 \uc0ac\uccb4\uc5d0\uc11c\ub294 \uc5d0\uc774\ub9ac\uc5b8\uc758 \ud754\uc801\uc774 \ubc1c\uacac\uc774 \ub418\uc9c0 \uc54a\uc558\uc73c\ub098, \ub9ac\ud50c\ub9ac\uc758 \uac04\uace1\ud55c \ubd80\ud0c1\uc73c\ub85c \uc564\ub4dc\ub958\uc2a4 \uad50\ub3c4\uc18c\uc7a5 \uc785\ud68c\ud558\uc5d0 \uc6a9\uad11\ub85c\uc5d0 \ud789\uc2a4\uc640 \ub274\ud2b8\uc758 \uc0ac\uccb4\ub97c \ub358\uc838 \uc7a5\ub840\ub97c \uce58\ub800\ub2e4. \ubc14\ub85c \uadf8 \uc2dc\uac04, \uc5d0\uc774\ub9ac\uc5b8\uc774 \uac1c\uc758 \ubc43\uac00\uc8fd\uc744 \ucc22\uace0 \ub098\uc654\uace0(\uc624\ub9ac\uc9c0\ub110 \ubc84\uc804), \uc8fd\uc740 \uc18c\uc758 \uac08\ube44\ub97c \ub6ab\uace0 \ub098\uc654\uc73c\uba70(\uc7ac\ud3b8\uc9d1\ud310), \uadf8\ub300\ub85c \uad50\ub3c4\uc18c \uc5b4\ub518\uac00\ub85c \uc0ac\ub77c\uc9c4\ub2e4.", "answer": "\ud53c\uc624\ub9ac\ub098 161\ud589\uc131", "sentence": "\uc9c0\uad6c\ub85c \ud5a5\ud558\ub358 \uc2dd\ubbfc\uc9c0 \ud574\ubcd1\ub300\uc18c\uc18d \uc6b0\uc8fc\uc120 \uc220\ub77c\ucf54(SULACO)\uc5d0\uc11c \ub3d9\uba74\uc744 \ud558\uace0 \uc788\ub358 \ub9ac\ud50c\ub9ac, \ub274\ud2b8, \ube44\uc20d, \ud789\uc2a4\uc0c1\ubcd1\uc740 \uc6b0\uc8fc\uc120\ub0b4\uc758 \ud654\uc7ac\ub85c \uc778\ud574 \uae34\uae09\ud0c8\ucd9c \uad6c\uba85\uc815\uc73c\ub85c \uc62e\uaca8\uc800 \uadfc\ucc98 \ud53c\uc624\ub9ac\ub098 161\ud589\uc131 \uc5d0 \ubd88\uc2dc\ucc29 \ud558\uac8c\ub41c\ub2e4.", "paragraph_sentence": " \uc9c0\uad6c\ub85c \ud5a5\ud558\ub358 \uc2dd\ubbfc\uc9c0 \ud574\ubcd1\ub300\uc18c\uc18d \uc6b0\uc8fc\uc120 \uc220\ub77c\ucf54(SULACO)\uc5d0\uc11c \ub3d9\uba74\uc744 \ud558\uace0 \uc788\ub358 \ub9ac\ud50c\ub9ac, \ub274\ud2b8, \ube44\uc20d, \ud789\uc2a4\uc0c1\ubcd1\uc740 \uc6b0\uc8fc\uc120\ub0b4\uc758 \ud654\uc7ac\ub85c \uc778\ud574 \uae34\uae09\ud0c8\ucd9c \uad6c\uba85\uc815\uc73c\ub85c \uc62e\uaca8\uc800 \uadfc\ucc98 \ud53c\uc624\ub9ac\ub098 161\ud589\uc131 \uc5d0 \ubd88\uc2dc\ucc29 \ud558\uac8c\ub41c\ub2e4. \uc774 \ud589\uc131\uc740 \uc6e8\uc774\ub79c\ub4dc \uc720\ud0c0\ub2c8\uc758 \ub178\ub3d9\uad50\ub3c4\uc18c \ud589\uc131\uc73c\ub85c, \uc0b4\uc778\uc790\uc640 \uac15\uac04\ubc94\ub4f1 \uc774\uc911\uc5fc\uc0c9\uccb4\ub97c \ubcf4\uc720\ud55c \uc218\uc2ed\uba85\uc758 \uc8c4\uc218\ub4e4\uacfc \uba87\uba85\uc758 \uad50\ub3c4\uad00\ub4e4\uc774 \uac70\uc8fc\ud558\uace0, \uc790\ub825 \ud0c8\ucd9c\uc740 \ubd88\uac00\ub2a5\ud55c \ud589\uc131\uc774\uc600\ub2e4. \uc758\ubb34\uc2e4\uc5d0\uc11c \uae68\uc5b4\ub09c \ub9ac\ud50c\ub9ac\ub294 \uc790\uc2e0\uc744 \uc81c\uc678\ud55c \ub274\ud2b8, \ud789\uc2a4\ub294 \uad6c\uba85\uc815\uc5d0\uc11c \uc0b4\uc544\ub0a8\uc9c0 \ubabb\ud588\ub2e4\ub294 \uac83\uc744 \uc54c\uace0 \uad34\ub85c\uc6cc\ud55c\ub2e4. \uadf8\ub7ec\ub098 \uad6c\uba85\uc815 \ub0b4\uc5d0\uc11c \uc1e0\uac00 \ub179\uc740 \uc790\uad6d\uc744 \ubcf4\uace0 \uc5d0\uc774\ub9ac\uc5b8\uc774 \uac19\uc774 \ub530\ub77c\uc654\uc74c\uc744 \uc9c1\uac10\ud558\uace0, \ub274\ud2b8\uc758 \uc0ac\uccb4\ub97c \ud074\ub808\uba58\uc2a4\uc758 \ub3c4\uc6c0\uc744 \uc5bb\uc5b4 \ubd80\uac80\ud588\ub2e4. \ub2e4\ud589\ud788 \ub274\ud2b8\uc758 \uc0ac\uccb4\uc5d0\uc11c\ub294 \uc5d0\uc774\ub9ac\uc5b8\uc758 \ud754\uc801\uc774 \ubc1c\uacac\uc774 \ub418\uc9c0 \uc54a\uc558\uc73c\ub098, \ub9ac\ud50c\ub9ac\uc758 \uac04\uace1\ud55c \ubd80\ud0c1\uc73c\ub85c \uc564\ub4dc\ub958\uc2a4 \uad50\ub3c4\uc18c\uc7a5 \uc785\ud68c\ud558\uc5d0 \uc6a9\uad11\ub85c\uc5d0 \ud789\uc2a4\uc640 \ub274\ud2b8\uc758 \uc0ac\uccb4\ub97c \ub358\uc838 \uc7a5\ub840\ub97c \uce58\ub800\ub2e4. \ubc14\ub85c \uadf8 \uc2dc\uac04, \uc5d0\uc774\ub9ac\uc5b8\uc774 \uac1c\uc758 \ubc43\uac00\uc8fd\uc744 \ucc22\uace0 \ub098\uc654\uace0(\uc624\ub9ac\uc9c0\ub110 \ubc84\uc804), \uc8fd\uc740 \uc18c\uc758 \uac08\ube44\ub97c \ub6ab\uace0 \ub098\uc654\uc73c\uba70(\uc7ac\ud3b8\uc9d1\ud310), \uadf8\ub300\ub85c \uad50\ub3c4\uc18c \uc5b4\ub518\uac00\ub85c \uc0ac\ub77c\uc9c4\ub2e4.", "paragraph_answer": "\uc9c0\uad6c\ub85c \ud5a5\ud558\ub358 \uc2dd\ubbfc\uc9c0 \ud574\ubcd1\ub300\uc18c\uc18d \uc6b0\uc8fc\uc120 \uc220\ub77c\ucf54(SULACO)\uc5d0\uc11c \ub3d9\uba74\uc744 \ud558\uace0 \uc788\ub358 \ub9ac\ud50c\ub9ac, \ub274\ud2b8, \ube44\uc20d, \ud789\uc2a4\uc0c1\ubcd1\uc740 \uc6b0\uc8fc\uc120\ub0b4\uc758 \ud654\uc7ac\ub85c \uc778\ud574 \uae34\uae09\ud0c8\ucd9c \uad6c\uba85\uc815\uc73c\ub85c \uc62e\uaca8\uc800 \uadfc\ucc98 \ud53c\uc624\ub9ac\ub098 161\ud589\uc131 \uc5d0 \ubd88\uc2dc\ucc29 \ud558\uac8c\ub41c\ub2e4. \uc774 \ud589\uc131\uc740 \uc6e8\uc774\ub79c\ub4dc \uc720\ud0c0\ub2c8\uc758 \ub178\ub3d9\uad50\ub3c4\uc18c \ud589\uc131\uc73c\ub85c, \uc0b4\uc778\uc790\uc640 \uac15\uac04\ubc94\ub4f1 \uc774\uc911\uc5fc\uc0c9\uccb4\ub97c \ubcf4\uc720\ud55c \uc218\uc2ed\uba85\uc758 \uc8c4\uc218\ub4e4\uacfc \uba87\uba85\uc758 \uad50\ub3c4\uad00\ub4e4\uc774 \uac70\uc8fc\ud558\uace0, \uc790\ub825 \ud0c8\ucd9c\uc740 \ubd88\uac00\ub2a5\ud55c \ud589\uc131\uc774\uc600\ub2e4. \uc758\ubb34\uc2e4\uc5d0\uc11c \uae68\uc5b4\ub09c \ub9ac\ud50c\ub9ac\ub294 \uc790\uc2e0\uc744 \uc81c\uc678\ud55c \ub274\ud2b8, \ud789\uc2a4\ub294 \uad6c\uba85\uc815\uc5d0\uc11c \uc0b4\uc544\ub0a8\uc9c0 \ubabb\ud588\ub2e4\ub294 \uac83\uc744 \uc54c\uace0 \uad34\ub85c\uc6cc\ud55c\ub2e4. \uadf8\ub7ec\ub098 \uad6c\uba85\uc815 \ub0b4\uc5d0\uc11c \uc1e0\uac00 \ub179\uc740 \uc790\uad6d\uc744 \ubcf4\uace0 \uc5d0\uc774\ub9ac\uc5b8\uc774 \uac19\uc774 \ub530\ub77c\uc654\uc74c\uc744 \uc9c1\uac10\ud558\uace0, \ub274\ud2b8\uc758 \uc0ac\uccb4\ub97c \ud074\ub808\uba58\uc2a4\uc758 \ub3c4\uc6c0\uc744 \uc5bb\uc5b4 \ubd80\uac80\ud588\ub2e4. \ub2e4\ud589\ud788 \ub274\ud2b8\uc758 \uc0ac\uccb4\uc5d0\uc11c\ub294 \uc5d0\uc774\ub9ac\uc5b8\uc758 \ud754\uc801\uc774 \ubc1c\uacac\uc774 \ub418\uc9c0 \uc54a\uc558\uc73c\ub098, \ub9ac\ud50c\ub9ac\uc758 \uac04\uace1\ud55c \ubd80\ud0c1\uc73c\ub85c \uc564\ub4dc\ub958\uc2a4 \uad50\ub3c4\uc18c\uc7a5 \uc785\ud68c\ud558\uc5d0 \uc6a9\uad11\ub85c\uc5d0 \ud789\uc2a4\uc640 \ub274\ud2b8\uc758 \uc0ac\uccb4\ub97c \ub358\uc838 \uc7a5\ub840\ub97c \uce58\ub800\ub2e4. \ubc14\ub85c \uadf8 \uc2dc\uac04, \uc5d0\uc774\ub9ac\uc5b8\uc774 \uac1c\uc758 \ubc43\uac00\uc8fd\uc744 \ucc22\uace0 \ub098\uc654\uace0(\uc624\ub9ac\uc9c0\ub110 \ubc84\uc804), \uc8fd\uc740 \uc18c\uc758 \uac08\ube44\ub97c \ub6ab\uace0 \ub098\uc654\uc73c\uba70(\uc7ac\ud3b8\uc9d1\ud310), \uadf8\ub300\ub85c \uad50\ub3c4\uc18c \uc5b4\ub518\uac00\ub85c \uc0ac\ub77c\uc9c4\ub2e4.", "sentence_answer": "\uc9c0\uad6c\ub85c \ud5a5\ud558\ub358 \uc2dd\ubbfc\uc9c0 \ud574\ubcd1\ub300\uc18c\uc18d \uc6b0\uc8fc\uc120 \uc220\ub77c\ucf54(SULACO)\uc5d0\uc11c \ub3d9\uba74\uc744 \ud558\uace0 \uc788\ub358 \ub9ac\ud50c\ub9ac, \ub274\ud2b8, \ube44\uc20d, \ud789\uc2a4\uc0c1\ubcd1\uc740 \uc6b0\uc8fc\uc120\ub0b4\uc758 \ud654\uc7ac\ub85c \uc778\ud574 \uae34\uae09\ud0c8\ucd9c \uad6c\uba85\uc815\uc73c\ub85c \uc62e\uaca8\uc800 \uadfc\ucc98 \ud53c\uc624\ub9ac\ub098 161\ud589\uc131 \uc5d0 \ubd88\uc2dc\ucc29 \ud558\uac8c\ub41c\ub2e4.", "paragraph_id": "6540728-0-0", "paragraph_question": "question: \uc6b0\uc8fc\uc120 \uc220\ub77c\ucf54\uc758 \ud654\uc7ac\ub85c \ub9ac\ud50c\ub9ac, \ub274\ud2b8, \ube44\uc20d, \ud789\uc2a4\uc0c1\ubcd1\uc774 \ubd88\uc2dc\ucc29\ud558\uac8c \ub41c \uacf3\uc740 \uc5b4\ub514\uc778\uac00?, context: \uc9c0\uad6c\ub85c \ud5a5\ud558\ub358 \uc2dd\ubbfc\uc9c0 \ud574\ubcd1\ub300\uc18c\uc18d \uc6b0\uc8fc\uc120 \uc220\ub77c\ucf54(SULACO)\uc5d0\uc11c \ub3d9\uba74\uc744 \ud558\uace0 \uc788\ub358 \ub9ac\ud50c\ub9ac, \ub274\ud2b8, \ube44\uc20d, \ud789\uc2a4\uc0c1\ubcd1\uc740 \uc6b0\uc8fc\uc120\ub0b4\uc758 \ud654\uc7ac\ub85c \uc778\ud574 \uae34\uae09\ud0c8\ucd9c \uad6c\uba85\uc815\uc73c\ub85c \uc62e\uaca8\uc800 \uadfc\ucc98 \ud53c\uc624\ub9ac\ub098 161\ud589\uc131\uc5d0 \ubd88\uc2dc\ucc29 \ud558\uac8c\ub41c\ub2e4. \uc774 \ud589\uc131\uc740 \uc6e8\uc774\ub79c\ub4dc \uc720\ud0c0\ub2c8\uc758 \ub178\ub3d9\uad50\ub3c4\uc18c \ud589\uc131\uc73c\ub85c, \uc0b4\uc778\uc790\uc640 \uac15\uac04\ubc94\ub4f1 \uc774\uc911\uc5fc\uc0c9\uccb4\ub97c \ubcf4\uc720\ud55c \uc218\uc2ed\uba85\uc758 \uc8c4\uc218\ub4e4\uacfc \uba87\uba85\uc758 \uad50\ub3c4\uad00\ub4e4\uc774 \uac70\uc8fc\ud558\uace0, \uc790\ub825 \ud0c8\ucd9c\uc740 \ubd88\uac00\ub2a5\ud55c \ud589\uc131\uc774\uc600\ub2e4. \uc758\ubb34\uc2e4\uc5d0\uc11c \uae68\uc5b4\ub09c \ub9ac\ud50c\ub9ac\ub294 \uc790\uc2e0\uc744 \uc81c\uc678\ud55c \ub274\ud2b8, \ud789\uc2a4\ub294 \uad6c\uba85\uc815\uc5d0\uc11c \uc0b4\uc544\ub0a8\uc9c0 \ubabb\ud588\ub2e4\ub294 \uac83\uc744 \uc54c\uace0 \uad34\ub85c\uc6cc\ud55c\ub2e4. \uadf8\ub7ec\ub098 \uad6c\uba85\uc815 \ub0b4\uc5d0\uc11c \uc1e0\uac00 \ub179\uc740 \uc790\uad6d\uc744 \ubcf4\uace0 \uc5d0\uc774\ub9ac\uc5b8\uc774 \uac19\uc774 \ub530\ub77c\uc654\uc74c\uc744 \uc9c1\uac10\ud558\uace0, \ub274\ud2b8\uc758 \uc0ac\uccb4\ub97c \ud074\ub808\uba58\uc2a4\uc758 \ub3c4\uc6c0\uc744 \uc5bb\uc5b4 \ubd80\uac80\ud588\ub2e4. \ub2e4\ud589\ud788 \ub274\ud2b8\uc758 \uc0ac\uccb4\uc5d0\uc11c\ub294 \uc5d0\uc774\ub9ac\uc5b8\uc758 \ud754\uc801\uc774 \ubc1c\uacac\uc774 \ub418\uc9c0 \uc54a\uc558\uc73c\ub098, \ub9ac\ud50c\ub9ac\uc758 \uac04\uace1\ud55c \ubd80\ud0c1\uc73c\ub85c \uc564\ub4dc\ub958\uc2a4 \uad50\ub3c4\uc18c\uc7a5 \uc785\ud68c\ud558\uc5d0 \uc6a9\uad11\ub85c\uc5d0 \ud789\uc2a4\uc640 \ub274\ud2b8\uc758 \uc0ac\uccb4\ub97c \ub358\uc838 \uc7a5\ub840\ub97c \uce58\ub800\ub2e4. \ubc14\ub85c \uadf8 \uc2dc\uac04, \uc5d0\uc774\ub9ac\uc5b8\uc774 \uac1c\uc758 \ubc43\uac00\uc8fd\uc744 \ucc22\uace0 \ub098\uc654\uace0(\uc624\ub9ac\uc9c0\ub110 \ubc84\uc804), \uc8fd\uc740 \uc18c\uc758 \uac08\ube44\ub97c \ub6ab\uace0 \ub098\uc654\uc73c\uba70(\uc7ac\ud3b8\uc9d1\ud310), \uadf8\ub300\ub85c \uad50\ub3c4\uc18c \uc5b4\ub518\uac00\ub85c \uc0ac\ub77c\uc9c4\ub2e4."} {"question": "\uc81c 2\ucc28 \uc138\uacc4 \ub300\uc804\uc774 \uc77c\uc5b4\ub0a0 \ub54c \uc720\ud589\ud588\ub358 \ud328\uc158 \uc911 \uad70\ubcf5 \ub514\uc790\uc778 \ud78c\ud2b8\ub97c \uc5bb\uc740 \uc2a4\ud0c0\uc77c\uc744 \ubb34\uc5c7\uc774\ub77c \ud558\ub294\uac00?", "paragraph": "\ub514\uc624\ub974\ub294 \uc804\ud6c4\uc758 \ucc38\ub2f4\ud568\uacfc \uc6b0\uc6b8\ud568\uc744 \uc544\ub984\ub2e4\uc6e0\ub358 \uacfc\uac70\uc640 \uc548\ub77d\ud568, \ubd80\ub4dc\ub7ec\uc6b4 \uacfc\uac70\ub85c \ubcf5\uc6d0\ud558\uace0\uc790 \ud588\uc73c\uba70, \uc77c\uc0c1\uc5d0\uc11c \ubc97\uc5b4\ub09c \ud658\uc0c1\uc801\uc778 \uc2e0\ud654\uc5d0 \uc785\uac01\ud574 \ud654\ub824\ud568\uacfc \uadc0\uc871\uc801 \uc544\ub984\ub2e4\uc6c0\uc744 \ud45c\ud604\ud588\ub2e4. \ub2f9\uc2dc \uc804\uc7c1\uc5d0 \uacc4\uc18d\ub418\uba74\uc11c \uad81\ud54d\ud55c \uc0dd\ud65c\uc5d0 \uc2dc\ub2ec\ub9b0 \uc720\ub7fd\uc778\ub4e4\uc740 \uc5ec\uc131\uc131\uc744 \uac15\uc870\ud558\uace0 \ube44\uc2fc \ucc9c\uc744 \ub9ce\uc774 \uc0ac\uc6a9\ud558\uac8c \ub418\ub294 \ub274 \ub8e9\uc744 \uc9c0\ub098\uce5c \uc0ac\uce58\ub77c\uace0 \ube44\ub09c\ud558\uae30\ub3c4 \ud588\ub2e4. \uc81c2\ucc28 \uc138\uacc4 \ub300\uc804\uc774 \uc77c\uc5b4\ub0a0 \ub2f9\uc2dc\ub294 \ub0a8\ub140\uc758 \uad6c\ubd84\uc774 \ud655\uc2e4\uce58 \uc54a\uc558\uace0 \uacbd\uc81c\uc801\uc73c\ub85c \uc5b4\ub824\uc6e0\uae30 \ub54c\ubb38\uc5d0 \uc637\uac10\uc744 \uc808\uc57d\ud558\ub294 \ud615\ud0dc\uc640 \ubc00\ub9ac\ud130\ub9ac \ub8e9\uc774 \uc720\ud589\ud588\ub2e4. \ud558\uc9c0\ub9cc \uc624\ub79c \uc804\uc7c1\uc73c\ub85c \uc5b5\ub20c\ub824\uc788\ub358 \uc5ec\uc131\ubbf8\uc5d0 \ub300\ud55c \uc695\ub9dd\uc774 \ud55c\ubc88\uc5d0 \ubd84\ucd9c\ub418\uba74\uc11c \ub274 \ub8e9\uc740 \ud328\uc158\uacc4\uc758 \uc0c8 \ubc14\ub78c\uc744 \ubd88\ub7ec\uc654\ub2e4. \uc804\uc7c1\uc5d0 \uc601\ud5a5\uc744 \ubc1b\uc740 1940\ub144\ub300 \ud328\uc158\uc5d0 \ub300\ud55c \ubc18\uc791\uc6a9\uc758 \uacb0\uacfc\uac00 \ub300\uc131\uacf5\uc744 \uc774\ub8ec \uac83\uc774\uc5c8\ub2e4. \ub610\ud55c \ub514\uc624\ub974\uc758 \ub274 \ub8e9\uc740 \uc804\uc7c1\uc774 \ub05d\ub098\uace0 \ub0a8\uc790\ub4e4\uc5d0\uac8c \uc77c\uc790\ub9ac\ub97c \ub3cc\ub824\uc8fc\uace0\uc790 \uc804\uc7c1\ub3d9\uc548 \uacf5\uc7a5\uc5d0\uc11c \uc77c\ud588\ub358 \uc5ec\uc131\ub4e4\uc5d0\uac8c \uc120\uc804\ud558\ub358 \uc11c\uad6c\uc815\ubd80\uc758 \uc815\ucc45\uacfc \uc798 \ub4e4\uc5b4\ub9de\ub294 \uc5ec\uc131\uc0c1\uc744 \ub9cc\ub4e4\uc5c8\ub2e4.", "answer": "\ubc00\ub9ac\ud130\ub9ac \ub8e9", "sentence": "\uc81c2\ucc28 \uc138\uacc4 \ub300\uc804\uc774 \uc77c\uc5b4\ub0a0 \ub2f9\uc2dc\ub294 \ub0a8\ub140\uc758 \uad6c\ubd84\uc774 \ud655\uc2e4\uce58 \uc54a\uc558\uace0 \uacbd\uc81c\uc801\uc73c\ub85c \uc5b4\ub824\uc6e0\uae30 \ub54c\ubb38\uc5d0 \uc637\uac10\uc744 \uc808\uc57d\ud558\ub294 \ud615\ud0dc\uc640 \ubc00\ub9ac\ud130\ub9ac \ub8e9 \uc774 \uc720\ud589\ud588\ub2e4.", "paragraph_sentence": "\ub514\uc624\ub974\ub294 \uc804\ud6c4\uc758 \ucc38\ub2f4\ud568\uacfc \uc6b0\uc6b8\ud568\uc744 \uc544\ub984\ub2e4\uc6e0\ub358 \uacfc\uac70\uc640 \uc548\ub77d\ud568, \ubd80\ub4dc\ub7ec\uc6b4 \uacfc\uac70\ub85c \ubcf5\uc6d0\ud558\uace0\uc790 \ud588\uc73c\uba70, \uc77c\uc0c1\uc5d0\uc11c \ubc97\uc5b4\ub09c \ud658\uc0c1\uc801\uc778 \uc2e0\ud654\uc5d0 \uc785\uac01\ud574 \ud654\ub824\ud568\uacfc \uadc0\uc871\uc801 \uc544\ub984\ub2e4\uc6c0\uc744 \ud45c\ud604\ud588\ub2e4. \ub2f9\uc2dc \uc804\uc7c1\uc5d0 \uacc4\uc18d\ub418\uba74\uc11c \uad81\ud54d\ud55c \uc0dd\ud65c\uc5d0 \uc2dc\ub2ec\ub9b0 \uc720\ub7fd\uc778\ub4e4\uc740 \uc5ec\uc131\uc131\uc744 \uac15\uc870\ud558\uace0 \ube44\uc2fc \ucc9c\uc744 \ub9ce\uc774 \uc0ac\uc6a9\ud558\uac8c \ub418\ub294 \ub274 \ub8e9\uc744 \uc9c0\ub098\uce5c \uc0ac\uce58\ub77c\uace0 \ube44\ub09c\ud558\uae30\ub3c4 \ud588\ub2e4. \uc81c2\ucc28 \uc138\uacc4 \ub300\uc804\uc774 \uc77c\uc5b4\ub0a0 \ub2f9\uc2dc\ub294 \ub0a8\ub140\uc758 \uad6c\ubd84\uc774 \ud655\uc2e4\uce58 \uc54a\uc558\uace0 \uacbd\uc81c\uc801\uc73c\ub85c \uc5b4\ub824\uc6e0\uae30 \ub54c\ubb38\uc5d0 \uc637\uac10\uc744 \uc808\uc57d\ud558\ub294 \ud615\ud0dc\uc640 \ubc00\ub9ac\ud130\ub9ac \ub8e9 \uc774 \uc720\ud589\ud588\ub2e4. \ud558\uc9c0\ub9cc \uc624\ub79c \uc804\uc7c1\uc73c\ub85c \uc5b5\ub20c\ub824\uc788\ub358 \uc5ec\uc131\ubbf8\uc5d0 \ub300\ud55c \uc695\ub9dd\uc774 \ud55c\ubc88\uc5d0 \ubd84\ucd9c\ub418\uba74\uc11c \ub274 \ub8e9\uc740 \ud328\uc158\uacc4\uc758 \uc0c8 \ubc14\ub78c\uc744 \ubd88\ub7ec\uc654\ub2e4. \uc804\uc7c1\uc5d0 \uc601\ud5a5\uc744 \ubc1b\uc740 1940\ub144\ub300 \ud328\uc158\uc5d0 \ub300\ud55c \ubc18\uc791\uc6a9\uc758 \uacb0\uacfc\uac00 \ub300\uc131\uacf5\uc744 \uc774\ub8ec \uac83\uc774\uc5c8\ub2e4. \ub610\ud55c \ub514\uc624\ub974\uc758 \ub274 \ub8e9\uc740 \uc804\uc7c1\uc774 \ub05d\ub098\uace0 \ub0a8\uc790\ub4e4\uc5d0\uac8c \uc77c\uc790\ub9ac\ub97c \ub3cc\ub824\uc8fc\uace0\uc790 \uc804\uc7c1\ub3d9\uc548 \uacf5\uc7a5\uc5d0\uc11c \uc77c\ud588\ub358 \uc5ec\uc131\ub4e4\uc5d0\uac8c \uc120\uc804\ud558\ub358 \uc11c\uad6c\uc815\ubd80\uc758 \uc815\ucc45\uacfc \uc798 \ub4e4\uc5b4\ub9de\ub294 \uc5ec\uc131\uc0c1\uc744 \ub9cc\ub4e4\uc5c8\ub2e4.", "paragraph_answer": "\ub514\uc624\ub974\ub294 \uc804\ud6c4\uc758 \ucc38\ub2f4\ud568\uacfc \uc6b0\uc6b8\ud568\uc744 \uc544\ub984\ub2e4\uc6e0\ub358 \uacfc\uac70\uc640 \uc548\ub77d\ud568, \ubd80\ub4dc\ub7ec\uc6b4 \uacfc\uac70\ub85c \ubcf5\uc6d0\ud558\uace0\uc790 \ud588\uc73c\uba70, \uc77c\uc0c1\uc5d0\uc11c \ubc97\uc5b4\ub09c \ud658\uc0c1\uc801\uc778 \uc2e0\ud654\uc5d0 \uc785\uac01\ud574 \ud654\ub824\ud568\uacfc \uadc0\uc871\uc801 \uc544\ub984\ub2e4\uc6c0\uc744 \ud45c\ud604\ud588\ub2e4. \ub2f9\uc2dc \uc804\uc7c1\uc5d0 \uacc4\uc18d\ub418\uba74\uc11c \uad81\ud54d\ud55c \uc0dd\ud65c\uc5d0 \uc2dc\ub2ec\ub9b0 \uc720\ub7fd\uc778\ub4e4\uc740 \uc5ec\uc131\uc131\uc744 \uac15\uc870\ud558\uace0 \ube44\uc2fc \ucc9c\uc744 \ub9ce\uc774 \uc0ac\uc6a9\ud558\uac8c \ub418\ub294 \ub274 \ub8e9\uc744 \uc9c0\ub098\uce5c \uc0ac\uce58\ub77c\uace0 \ube44\ub09c\ud558\uae30\ub3c4 \ud588\ub2e4. \uc81c2\ucc28 \uc138\uacc4 \ub300\uc804\uc774 \uc77c\uc5b4\ub0a0 \ub2f9\uc2dc\ub294 \ub0a8\ub140\uc758 \uad6c\ubd84\uc774 \ud655\uc2e4\uce58 \uc54a\uc558\uace0 \uacbd\uc81c\uc801\uc73c\ub85c \uc5b4\ub824\uc6e0\uae30 \ub54c\ubb38\uc5d0 \uc637\uac10\uc744 \uc808\uc57d\ud558\ub294 \ud615\ud0dc\uc640 \ubc00\ub9ac\ud130\ub9ac \ub8e9 \uc774 \uc720\ud589\ud588\ub2e4. \ud558\uc9c0\ub9cc \uc624\ub79c \uc804\uc7c1\uc73c\ub85c \uc5b5\ub20c\ub824\uc788\ub358 \uc5ec\uc131\ubbf8\uc5d0 \ub300\ud55c \uc695\ub9dd\uc774 \ud55c\ubc88\uc5d0 \ubd84\ucd9c\ub418\uba74\uc11c \ub274 \ub8e9\uc740 \ud328\uc158\uacc4\uc758 \uc0c8 \ubc14\ub78c\uc744 \ubd88\ub7ec\uc654\ub2e4. \uc804\uc7c1\uc5d0 \uc601\ud5a5\uc744 \ubc1b\uc740 1940\ub144\ub300 \ud328\uc158\uc5d0 \ub300\ud55c \ubc18\uc791\uc6a9\uc758 \uacb0\uacfc\uac00 \ub300\uc131\uacf5\uc744 \uc774\ub8ec \uac83\uc774\uc5c8\ub2e4. \ub610\ud55c \ub514\uc624\ub974\uc758 \ub274 \ub8e9\uc740 \uc804\uc7c1\uc774 \ub05d\ub098\uace0 \ub0a8\uc790\ub4e4\uc5d0\uac8c \uc77c\uc790\ub9ac\ub97c \ub3cc\ub824\uc8fc\uace0\uc790 \uc804\uc7c1\ub3d9\uc548 \uacf5\uc7a5\uc5d0\uc11c \uc77c\ud588\ub358 \uc5ec\uc131\ub4e4\uc5d0\uac8c \uc120\uc804\ud558\ub358 \uc11c\uad6c\uc815\ubd80\uc758 \uc815\ucc45\uacfc \uc798 \ub4e4\uc5b4\ub9de\ub294 \uc5ec\uc131\uc0c1\uc744 \ub9cc\ub4e4\uc5c8\ub2e4.", "sentence_answer": "\uc81c2\ucc28 \uc138\uacc4 \ub300\uc804\uc774 \uc77c\uc5b4\ub0a0 \ub2f9\uc2dc\ub294 \ub0a8\ub140\uc758 \uad6c\ubd84\uc774 \ud655\uc2e4\uce58 \uc54a\uc558\uace0 \uacbd\uc81c\uc801\uc73c\ub85c \uc5b4\ub824\uc6e0\uae30 \ub54c\ubb38\uc5d0 \uc637\uac10\uc744 \uc808\uc57d\ud558\ub294 \ud615\ud0dc\uc640 \ubc00\ub9ac\ud130\ub9ac \ub8e9 \uc774 \uc720\ud589\ud588\ub2e4.", "paragraph_id": "6564029-2-1", "paragraph_question": "question: \uc81c 2\ucc28 \uc138\uacc4 \ub300\uc804\uc774 \uc77c\uc5b4\ub0a0 \ub54c \uc720\ud589\ud588\ub358 \ud328\uc158 \uc911 \uad70\ubcf5 \ub514\uc790\uc778 \ud78c\ud2b8\ub97c \uc5bb\uc740 \uc2a4\ud0c0\uc77c\uc744 \ubb34\uc5c7\uc774\ub77c \ud558\ub294\uac00?, context: \ub514\uc624\ub974\ub294 \uc804\ud6c4\uc758 \ucc38\ub2f4\ud568\uacfc \uc6b0\uc6b8\ud568\uc744 \uc544\ub984\ub2e4\uc6e0\ub358 \uacfc\uac70\uc640 \uc548\ub77d\ud568, \ubd80\ub4dc\ub7ec\uc6b4 \uacfc\uac70\ub85c \ubcf5\uc6d0\ud558\uace0\uc790 \ud588\uc73c\uba70, \uc77c\uc0c1\uc5d0\uc11c \ubc97\uc5b4\ub09c \ud658\uc0c1\uc801\uc778 \uc2e0\ud654\uc5d0 \uc785\uac01\ud574 \ud654\ub824\ud568\uacfc \uadc0\uc871\uc801 \uc544\ub984\ub2e4\uc6c0\uc744 \ud45c\ud604\ud588\ub2e4. \ub2f9\uc2dc \uc804\uc7c1\uc5d0 \uacc4\uc18d\ub418\uba74\uc11c \uad81\ud54d\ud55c \uc0dd\ud65c\uc5d0 \uc2dc\ub2ec\ub9b0 \uc720\ub7fd\uc778\ub4e4\uc740 \uc5ec\uc131\uc131\uc744 \uac15\uc870\ud558\uace0 \ube44\uc2fc \ucc9c\uc744 \ub9ce\uc774 \uc0ac\uc6a9\ud558\uac8c \ub418\ub294 \ub274 \ub8e9\uc744 \uc9c0\ub098\uce5c \uc0ac\uce58\ub77c\uace0 \ube44\ub09c\ud558\uae30\ub3c4 \ud588\ub2e4. \uc81c2\ucc28 \uc138\uacc4 \ub300\uc804\uc774 \uc77c\uc5b4\ub0a0 \ub2f9\uc2dc\ub294 \ub0a8\ub140\uc758 \uad6c\ubd84\uc774 \ud655\uc2e4\uce58 \uc54a\uc558\uace0 \uacbd\uc81c\uc801\uc73c\ub85c \uc5b4\ub824\uc6e0\uae30 \ub54c\ubb38\uc5d0 \uc637\uac10\uc744 \uc808\uc57d\ud558\ub294 \ud615\ud0dc\uc640 \ubc00\ub9ac\ud130\ub9ac \ub8e9\uc774 \uc720\ud589\ud588\ub2e4. \ud558\uc9c0\ub9cc \uc624\ub79c \uc804\uc7c1\uc73c\ub85c \uc5b5\ub20c\ub824\uc788\ub358 \uc5ec\uc131\ubbf8\uc5d0 \ub300\ud55c \uc695\ub9dd\uc774 \ud55c\ubc88\uc5d0 \ubd84\ucd9c\ub418\uba74\uc11c \ub274 \ub8e9\uc740 \ud328\uc158\uacc4\uc758 \uc0c8 \ubc14\ub78c\uc744 \ubd88\ub7ec\uc654\ub2e4. \uc804\uc7c1\uc5d0 \uc601\ud5a5\uc744 \ubc1b\uc740 1940\ub144\ub300 \ud328\uc158\uc5d0 \ub300\ud55c \ubc18\uc791\uc6a9\uc758 \uacb0\uacfc\uac00 \ub300\uc131\uacf5\uc744 \uc774\ub8ec \uac83\uc774\uc5c8\ub2e4. \ub610\ud55c \ub514\uc624\ub974\uc758 \ub274 \ub8e9\uc740 \uc804\uc7c1\uc774 \ub05d\ub098\uace0 \ub0a8\uc790\ub4e4\uc5d0\uac8c \uc77c\uc790\ub9ac\ub97c \ub3cc\ub824\uc8fc\uace0\uc790 \uc804\uc7c1\ub3d9\uc548 \uacf5\uc7a5\uc5d0\uc11c \uc77c\ud588\ub358 \uc5ec\uc131\ub4e4\uc5d0\uac8c \uc120\uc804\ud558\ub358 \uc11c\uad6c\uc815\ubd80\uc758 \uc815\ucc45\uacfc \uc798 \ub4e4\uc5b4\ub9de\ub294 \uc5ec\uc131\uc0c1\uc744 \ub9cc\ub4e4\uc5c8\ub2e4."} {"question": "\ud0dc\uc591\uc758 \ud6c4\uc608\uac00 \ud604\uc2e4\uc744 \ud0c8\ud53c\ud558\uace0 \ub300\uc911\uc774 \uc6d0\ud558\ub294 \uc695\ub9dd\uc744 \ucda9\uc871\uc2dc\ud0a4\ub294\ub370 \uc778\uae30\uc758 \uc694\uc778\uc774 \uc788\ub2e4\uace0 \ud55c \uc0ac\ub78c\uc740?", "paragraph": "\uc601\uad6d BBC\ub294\u300a\ud0dc\uc591\uc758 \ud6c4\uc608\u300b\uac00 \ud55c\ub958\uc758 \uc815\uc810\uc744 \ucc0d\uc744 \uac83\uc774\ub77c \uc608\uc0c1\ud588\ub2e4. \uc720\uac74\uc2dd\uc740 \"\ud0c4\ud0c4\ud558\uba74\uc11c\ub3c4 \ube60\ub978 \uc2a4\ud1a0\ub9ac \uc804\uac1c\uc640 \uc601\ud654 \uac19\uc740 \uc9c8\uac10, \uc778\ub958\uc560\uc5d0\uc11c \ud53c\uc5b4\ub098\ub294 \ucee4\ud50c\ub4e4\uc758 \uc0ac\ub791\uc774 \uc801\uc808\ud788 \ubc30\ubd84\ub41c \uac83\uc774 \uc778\uae30\uc694\uc778\uc73c\ub85c \uaf3d\ud788\uace0 \uc788\ub2e4.\u201d\uace0 \uc804\ud588\ub2e4. \ub0a8\uad81\uc778\uc740 \ud3c9\ub860\uc5d0\uc11c \u300a\ud0dc\uc591\uc758 \ud6c4\uc608\u300b\uc758 \uc778\uae30\uc694\uc778\uc5d0 \ub300\ud574 \ub2e4\uc74c\uacfc \uac19\uc774 \ubd84\uc11d\ud588\ub2e4. \u300a\ud0dc\uc591\uc758 \ud6c4\uc608\u300b\ub294 \"\uc8fc\uc5b4\uc9c4 \uc2dc\uac04\uc5d0 \uadf9\uc801\uc778 \uc774\uc57c\uae30\ub97c \ubc30\uce58\ud558\ub294 \ud6a8\uc728\uc131\uc73c\ub85c \ud1b1\ub2c8\ubc14\ud034\ucc98\ub7fc \ub9de\ubb3c\ub824 \uc788\ub2e4. \uc774\uc57c\uae30\ud558\uace0\uc790 \ud558\ub294 \ubc14\uc640, \uadf8 \uc774\uc57c\uae30\ub97c \uc2dc\uac01\ud654\ud558\uc5ec \ubcf4\uc5ec\uc8fc\uace0\uc790 \ud558\ub294 \uc695\ub9dd\uc744 \uc5b4\ub514\uae4c\uc9c0 \uc2e4\ud604\ud560 \uc218 \uc788\ub290\ub0d0\ub294 \uc9c8\ubb38\uc5d0 \ub2f5\ud558\ub294, \uadf9\ub2e8\uc801\uc778 \ubc1c\uc804\uc758 \ud55c \uc608\ub2e4. \u300a\ud0dc\uc591\uc758 \ud6c4\uc608\u300b\uc5d0\uc11c \ub300\uc911\uc740 \"\ub2e4\uc774\ub0b4\ubbf9\ud55c \ud074\ub77c\uc774\ub9e5\uc2a4, \uadf8\ub9ac\uace0 \uae30\uc6d0\ud558\ub358 \uac83\uc774 \uc774\ub8e8\uc5b4\uc84c\uc744 \ub54c\uc758 \uce74\ud0c0\ub974\uc2dc\uc2a4\"\ub97c \ub290\ub07c\uac8c \ub41c\ub2e4. \u300a\ud0dc\uc591\uc758 \ud6c4\uc608\u300b\ub294 \"\uc774 \uc695\ub9dd\uc744 \uc2ed\ubd84 \ucda9\uc871\uc2dc\ud0a4\uae30 \uc704\ud574 \ud604\uc2e4\uc774\ub098 \uace0\uc99d\uc740 \uacfc\uac10\ud788 \ubb34\uc2dc\ud558\uace0, \uc77c\ubc18 \ub300\uc911\uc758 \ub208\ub192\uc774\uc5d0 \uc11c\uc11c, \uadf8\ub4e4\uc774 \ubcf4\uace0 \uc2f6\uc740 \uc7a5\uba74\uc774\ub77c\uba74 \ubb34\uc5c7\uc774\ub4e0 \ubcf4\uc5ec\uc900\ub2e4. (\uc911\ub7b5) \ub300\uc911\uc774 \ubcf4\uace0 \uc2f6\uc5b4 \ud558\ub294 \uac83\uc740 \ucd1d\ud0c4\uc774 \ubc30\uc5d0 \ubc15\ud614\ub2e4\uace0 \ubc14\uc774\ud0c8\uc744 \uc7ac\uba74\uc11c \ubcd1\uc6d0\uc73c\ub85c \uc2e0\uc18d\ud788 \ud6c4\uc1a1\ud558\ub294 \uc7a5\uba74\ub3c4 \uc544\ub2c8\uace0, \ub2e4\uc774\uc544\ubaac\ub4dc\ub294 \uc548 \ucc0d\ud614\uc9c0\ub9cc \ud544\ub984\uc744 \ub4e4\uace0 \uc640\uc11c \uc815\ud669\uc0c1 \uc758\uc2ec \uac00\ub2a5\ud560 \uc218 \uc788\ub2e4\uace0 \ub9dd\uc124\uc774\ub294 \uc7a5\uba74\ub3c4, \uccad\uc9c4\uae30\ub97c \uc548 \ub300\uace0 \ubc30\ub97c \uc5f4\uc5b4 \uc870\uc2ec\uc870\uc2ec \ucd09\uc9c4\ud574 \ud658\uc790\uc758 \ubc18\uc751\uc744 \uc0b4\ud53c\ub294 \uac83\ub3c4 \uc544\ub2c8\ub2e4. \ub300\uc911\uacfc TV \uc548\uc5d0\ub294 \uc9c1\uad00\uc801\uc778 \uc138\uc0c1\uc774 \uc788\uc744 \ubfd0, \uc544\ubb34\ub3c4 \uadf8\uac83\uc774 \ud604\uc2e4\uc801\uc778 \uacbd\uacc4\uc5d0 \uc788\uc5b4\uc57c \ud55c\ub2e4\uace0 \uac15\uc694\ud558\uc9c0 \uc54a\ub294\ub2e4. \uadf8\ub798\uc11c, \uc774 \ub4dc\ub77c\ub9c8\ub294 \uacfc\uac10\ud788 \ud604\uc2e4\uc744 \ud0c8\ud53c\ud558\uace0 \ubd95\uad34\uc2dc\ucf1c \uc790\uae30\ub4e4\uc774 \uac00\uc7a5 \uc798 \ud558\ub294 \uc774\uc57c\uae30\ub97c \ud568\uc73c\ub85c\uc368 \ubaa8\ub450\uc758 \uc695\ub9dd\uc744 \ubc1c\ud604\uc2dc\ud0a8\ub2e4. (\uc911\ub7b5) \ucd1d\ud0c4\uc740 \uc989\uc2dc \ud5e4\uc9d1\uc5b4 \ubf51\uc544\uc57c \ud558\uba70, \uc5d1\uc2a4\ub808\uc774\uc5d0\ub294 \ubd84\uba85\ud788 \ud558\uc580 \ub2e4\uc774\uc544\ubaac\ub4dc\uac00 \ucc0d\ud600\uc57c \ud55c\ub2e4. \ud1b5\ub150\uc0c1 \ubc18\uad70 \uc9c0\ub3c4\uc790\ub294 \uc544\ud504\ub9ac\uce74 \uc778\uc774\uace0, \ud30c\ubcd1\ub41c \ub098\ub77c \uc0ac\ub78c\ub4e4\uc740 \uc544\ub78d\uc5b4\ub97c \uc4f0\uace0, \uac31\uc740 \uc601\uc5b4\ub97c \uc4f0\ub294 \ubc31\uc778\uc774\uc5b4\uc57c \ud560 \uac83 \uac19\"\ub2e4\uace0 \ub300\uc911\uc774 \uc0dd\uac01\ud558\uae30 \ub54c\ubb38\uc5d0 \u300a\ud0dc\uc591\uc758 \ud6c4\uc608\u300b\ub294 \"\uadf8\ub7f0 \ub098\ub77c\uc640 \uc2dc\uacf5\uc744 \uad6c\ud604\ud574\ubc84\"\ub838\ub2e4. \"\uc81c\uc791\uc9c4\uc774 \uc758\ub3c4\uc801\uc73c\ub85c \ud3bc\uccd0\ub0b8, \ubb3c\ub9ac\uc801\uc73c\ub85c \ub4a4\uc8fd\ubc15\uc8fd\uc778 \uc774 \uacf5\uac04\uc5d0\uc11c, \uc774\uc57c\uae30\ub294 \ub0a0\uac1c\uac00 \ub3cb\uc544 \uad00\uac1d\uc758 \ub208\ub3d9\uc790\ub97c \ud33d\ud33d \ub3cc\ub9ac\uba70 \ubc1c\ud604\ub41c\ub2e4. \uc5ec\uc8fc\uc778\uacf5\uc758 \ucc28\ub294 \ub9e4\ud68c \uace4\ub450\ubc15\uc9c8\uce58\uace0, \ucd94\ub77d\ud558\uace0, \uc9c0\ub8b0\ubc2d\uc5d0 \ub4e4\uc5b4\uac04\ub2e4. \ub354\ubd88\uc5b4 \uc9c0\uc9c4\ub3c4 \ub098\uace0, \uc801\uad70\ub3c4 \uccd0\ub4e4\uc5b4 \uc624\uace0, \uc804\uc5fc\ubcd1\ub3c4 \ub3cc\uace0, \uac31\uc740 \ub9e4\ubc88 \ub3c4\ub07c\ub208\uc73c\ub85c \ucd1d\uc744 \uc3dc\ub2e4. \uadf8 \uc7a5\uce58\ub85c \uacf5\uace0\ud574\uc9c0\ub294 \uac83\uc740 \uc8fc\uc5b4\uc9c4 \uc721\uc2ed \ubd84\uc5d0\ub3c4 \uba87 \ubc88\uc529 \ud074\ub77c\uc774\ub9e5\uc2a4\ub97c \ub9cc\ub4e4 \uc218 \uc788\ub294 \uc2dc\uac01\uc801 \uad6c\ud604\uacfc, \uba87 \ubc88\uc529 \uba5c\ub85c\uc801 \uce74\ud0c0\ub974\uc2dc\uc2a4\ub97c \ub290\ub084 \uc218 \uc788\ub294 \ub2ec\ub2ec\ud55c \uc7a5\uba74\uc774\ub2e4.\" \ub610\ud55c \ub0a8\uad81\uc778\uc740 \"\uadf8 \uc640\uc911\uc5d0 \uc8fc\uc778\uacf5\ub4e4\uc774 \uc8fc\uace0\ubc1b\ub294 \ub300\uc0ac\ub4e4\uc740 \uc704\ud2b8\ub098 \uae34\ubc15\ud568\uc5d0\uc11c \ud765\ubbf8\ub86d\uace0 \uae30\ubc1c\"\ud558\uace0 \uc7a5\uba74\uc740 \"\ucc38\uc2e0\"\ud558\ub2e4 \ud3c9\ud588\ub2e4. \uadf8\ub798\uc11c \u300a\ud0dc\uc591\uc758 \ud6c4\uc608\u300b\uc5d0\ub294 \"\uc27d\uac8c \ub118\ubcf4\uae30 \ud798\ub4e0 \uba85\ub791\ud568\uc774 \uac00\ub4dd\ud558\ub2e4.\" \ub0a8\uad81\uc778\uc740 \ud0dc\uc591\uc758 \ud6c4\uc608\uac00 \ub300\uc911\uc758 \"\uc695\ub9dd\uc758 \ud604\uc7ac \uc8fc\uc18c\"\ub97c \ud3ec\ucc29\ud55c \ub4a4 \uc2dc\ub3c4\ud55c \"\ub098\ub984\ub300\ub85c\uc758 \uc791\ud488\uc138\uacc4\uc640 \uc791\ud654 \ubc29\ubc95\"\uc774 \uc628 \uc138\uacc4\ub97c \"\uc9c0\ubc30\ud560 \uc815\ub3c4\uc758 \uc7a5\ub974\ud654\ub41c \ud558\ub098\uc758 \ud604\uc0c1\"\uc774 \ub418\uc5c8\ub2e4\uace0 \uacb0\ub860\uc9c0\uc5c8\ub2e4.", "answer": "\ub0a8\uad81\uc778", "sentence": "\ub0a8\uad81\uc778 \uc740 \ud3c9\ub860\uc5d0\uc11c \u300a\ud0dc\uc591\uc758 \ud6c4\uc608\u300b\uc758 \uc778\uae30\uc694\uc778\uc5d0 \ub300\ud574 \ub2e4\uc74c\uacfc \uac19\uc774 \ubd84\uc11d\ud588\ub2e4.", "paragraph_sentence": "\uc601\uad6d BBC\ub294\u300a\ud0dc\uc591\uc758 \ud6c4\uc608\u300b\uac00 \ud55c\ub958\uc758 \uc815\uc810\uc744 \ucc0d\uc744 \uac83\uc774\ub77c \uc608\uc0c1\ud588\ub2e4. \uc720\uac74\uc2dd\uc740 \"\ud0c4\ud0c4\ud558\uba74\uc11c\ub3c4 \ube60\ub978 \uc2a4\ud1a0\ub9ac \uc804\uac1c\uc640 \uc601\ud654 \uac19\uc740 \uc9c8\uac10, \uc778\ub958\uc560\uc5d0\uc11c \ud53c\uc5b4\ub098\ub294 \ucee4\ud50c\ub4e4\uc758 \uc0ac\ub791\uc774 \uc801\uc808\ud788 \ubc30\ubd84\ub41c \uac83\uc774 \uc778\uae30\uc694\uc778\uc73c\ub85c \uaf3d\ud788\uace0 \uc788\ub2e4. \u201d\uace0 \uc804\ud588\ub2e4. \ub0a8\uad81\uc778 \uc740 \ud3c9\ub860\uc5d0\uc11c \u300a\ud0dc\uc591\uc758 \ud6c4\uc608\u300b\uc758 \uc778\uae30\uc694\uc778\uc5d0 \ub300\ud574 \ub2e4\uc74c\uacfc \uac19\uc774 \ubd84\uc11d\ud588\ub2e4. \u300a\ud0dc\uc591\uc758 \ud6c4\uc608\u300b\ub294 \"\uc8fc\uc5b4\uc9c4 \uc2dc\uac04\uc5d0 \uadf9\uc801\uc778 \uc774\uc57c\uae30\ub97c \ubc30\uce58\ud558\ub294 \ud6a8\uc728\uc131\uc73c\ub85c \ud1b1\ub2c8\ubc14\ud034\ucc98\ub7fc \ub9de\ubb3c\ub824 \uc788\ub2e4. \uc774\uc57c\uae30\ud558\uace0\uc790 \ud558\ub294 \ubc14\uc640, \uadf8 \uc774\uc57c\uae30\ub97c \uc2dc\uac01\ud654\ud558\uc5ec \ubcf4\uc5ec\uc8fc\uace0\uc790 \ud558\ub294 \uc695\ub9dd\uc744 \uc5b4\ub514\uae4c\uc9c0 \uc2e4\ud604\ud560 \uc218 \uc788\ub290\ub0d0\ub294 \uc9c8\ubb38\uc5d0 \ub2f5\ud558\ub294, \uadf9\ub2e8\uc801\uc778 \ubc1c\uc804\uc758 \ud55c \uc608\ub2e4. \u300a\ud0dc\uc591\uc758 \ud6c4\uc608\u300b\uc5d0\uc11c \ub300\uc911\uc740 \"\ub2e4\uc774\ub0b4\ubbf9\ud55c \ud074\ub77c\uc774\ub9e5\uc2a4, \uadf8\ub9ac\uace0 \uae30\uc6d0\ud558\ub358 \uac83\uc774 \uc774\ub8e8\uc5b4\uc84c\uc744 \ub54c\uc758 \uce74\ud0c0\ub974\uc2dc\uc2a4\"\ub97c \ub290\ub07c\uac8c \ub41c\ub2e4. \u300a\ud0dc\uc591\uc758 \ud6c4\uc608\u300b\ub294 \"\uc774 \uc695\ub9dd\uc744 \uc2ed\ubd84 \ucda9\uc871\uc2dc\ud0a4\uae30 \uc704\ud574 \ud604\uc2e4\uc774\ub098 \uace0\uc99d\uc740 \uacfc\uac10\ud788 \ubb34\uc2dc\ud558\uace0, \uc77c\ubc18 \ub300\uc911\uc758 \ub208\ub192\uc774\uc5d0 \uc11c\uc11c, \uadf8\ub4e4\uc774 \ubcf4\uace0 \uc2f6\uc740 \uc7a5\uba74\uc774\ub77c\uba74 \ubb34\uc5c7\uc774\ub4e0 \ubcf4\uc5ec\uc900\ub2e4. (\uc911\ub7b5) \ub300\uc911\uc774 \ubcf4\uace0 \uc2f6\uc5b4 \ud558\ub294 \uac83\uc740 \ucd1d\ud0c4\uc774 \ubc30\uc5d0 \ubc15\ud614\ub2e4\uace0 \ubc14\uc774\ud0c8\uc744 \uc7ac\uba74\uc11c \ubcd1\uc6d0\uc73c\ub85c \uc2e0\uc18d\ud788 \ud6c4\uc1a1\ud558\ub294 \uc7a5\uba74\ub3c4 \uc544\ub2c8\uace0, \ub2e4\uc774\uc544\ubaac\ub4dc\ub294 \uc548 \ucc0d\ud614\uc9c0\ub9cc \ud544\ub984\uc744 \ub4e4\uace0 \uc640\uc11c \uc815\ud669\uc0c1 \uc758\uc2ec \uac00\ub2a5\ud560 \uc218 \uc788\ub2e4\uace0 \ub9dd\uc124\uc774\ub294 \uc7a5\uba74\ub3c4, \uccad\uc9c4\uae30\ub97c \uc548 \ub300\uace0 \ubc30\ub97c \uc5f4\uc5b4 \uc870\uc2ec\uc870\uc2ec \ucd09\uc9c4\ud574 \ud658\uc790\uc758 \ubc18\uc751\uc744 \uc0b4\ud53c\ub294 \uac83\ub3c4 \uc544\ub2c8\ub2e4. \ub300\uc911\uacfc TV \uc548\uc5d0\ub294 \uc9c1\uad00\uc801\uc778 \uc138\uc0c1\uc774 \uc788\uc744 \ubfd0, \uc544\ubb34\ub3c4 \uadf8\uac83\uc774 \ud604\uc2e4\uc801\uc778 \uacbd\uacc4\uc5d0 \uc788\uc5b4\uc57c \ud55c\ub2e4\uace0 \uac15\uc694\ud558\uc9c0 \uc54a\ub294\ub2e4. \uadf8\ub798\uc11c, \uc774 \ub4dc\ub77c\ub9c8\ub294 \uacfc\uac10\ud788 \ud604\uc2e4\uc744 \ud0c8\ud53c\ud558\uace0 \ubd95\uad34\uc2dc\ucf1c \uc790\uae30\ub4e4\uc774 \uac00\uc7a5 \uc798 \ud558\ub294 \uc774\uc57c\uae30\ub97c \ud568\uc73c\ub85c\uc368 \ubaa8\ub450\uc758 \uc695\ub9dd\uc744 \ubc1c\ud604\uc2dc\ud0a8\ub2e4. (\uc911\ub7b5) \ucd1d\ud0c4\uc740 \uc989\uc2dc \ud5e4\uc9d1\uc5b4 \ubf51\uc544\uc57c \ud558\uba70, \uc5d1\uc2a4\ub808\uc774\uc5d0\ub294 \ubd84\uba85\ud788 \ud558\uc580 \ub2e4\uc774\uc544\ubaac\ub4dc\uac00 \ucc0d\ud600\uc57c \ud55c\ub2e4. \ud1b5\ub150\uc0c1 \ubc18\uad70 \uc9c0\ub3c4\uc790\ub294 \uc544\ud504\ub9ac\uce74 \uc778\uc774\uace0, \ud30c\ubcd1\ub41c \ub098\ub77c \uc0ac\ub78c\ub4e4\uc740 \uc544\ub78d\uc5b4\ub97c \uc4f0\uace0, \uac31\uc740 \uc601\uc5b4\ub97c \uc4f0\ub294 \ubc31\uc778\uc774\uc5b4\uc57c \ud560 \uac83 \uac19\"\ub2e4\uace0 \ub300\uc911\uc774 \uc0dd\uac01\ud558\uae30 \ub54c\ubb38\uc5d0 \u300a\ud0dc\uc591\uc758 \ud6c4\uc608\u300b\ub294 \"\uadf8\ub7f0 \ub098\ub77c\uc640 \uc2dc\uacf5\uc744 \uad6c\ud604\ud574\ubc84\"\ub838\ub2e4. \"\uc81c\uc791\uc9c4\uc774 \uc758\ub3c4\uc801\uc73c\ub85c \ud3bc\uccd0\ub0b8, \ubb3c\ub9ac\uc801\uc73c\ub85c \ub4a4\uc8fd\ubc15\uc8fd\uc778 \uc774 \uacf5\uac04\uc5d0\uc11c, \uc774\uc57c\uae30\ub294 \ub0a0\uac1c\uac00 \ub3cb\uc544 \uad00\uac1d\uc758 \ub208\ub3d9\uc790\ub97c \ud33d\ud33d \ub3cc\ub9ac\uba70 \ubc1c\ud604\ub41c\ub2e4. \uc5ec\uc8fc\uc778\uacf5\uc758 \ucc28\ub294 \ub9e4\ud68c \uace4\ub450\ubc15\uc9c8\uce58\uace0, \ucd94\ub77d\ud558\uace0, \uc9c0\ub8b0\ubc2d\uc5d0 \ub4e4\uc5b4\uac04\ub2e4. \ub354\ubd88\uc5b4 \uc9c0\uc9c4\ub3c4 \ub098\uace0, \uc801\uad70\ub3c4 \uccd0\ub4e4\uc5b4 \uc624\uace0, \uc804\uc5fc\ubcd1\ub3c4 \ub3cc\uace0, \uac31\uc740 \ub9e4\ubc88 \ub3c4\ub07c\ub208\uc73c\ub85c \ucd1d\uc744 \uc3dc\ub2e4. \uadf8 \uc7a5\uce58\ub85c \uacf5\uace0\ud574\uc9c0\ub294 \uac83\uc740 \uc8fc\uc5b4\uc9c4 \uc721\uc2ed \ubd84\uc5d0\ub3c4 \uba87 \ubc88\uc529 \ud074\ub77c\uc774\ub9e5\uc2a4\ub97c \ub9cc\ub4e4 \uc218 \uc788\ub294 \uc2dc\uac01\uc801 \uad6c\ud604\uacfc, \uba87 \ubc88\uc529 \uba5c\ub85c\uc801 \uce74\ud0c0\ub974\uc2dc\uc2a4\ub97c \ub290\ub084 \uc218 \uc788\ub294 \ub2ec\ub2ec\ud55c \uc7a5\uba74\uc774\ub2e4. \" \ub610\ud55c \ub0a8\uad81\uc778\uc740 \"\uadf8 \uc640\uc911\uc5d0 \uc8fc\uc778\uacf5\ub4e4\uc774 \uc8fc\uace0\ubc1b\ub294 \ub300\uc0ac\ub4e4\uc740 \uc704\ud2b8\ub098 \uae34\ubc15\ud568\uc5d0\uc11c \ud765\ubbf8\ub86d\uace0 \uae30\ubc1c\"\ud558\uace0 \uc7a5\uba74\uc740 \"\ucc38\uc2e0\"\ud558\ub2e4 \ud3c9\ud588\ub2e4. \uadf8\ub798\uc11c \u300a\ud0dc\uc591\uc758 \ud6c4\uc608\u300b\uc5d0\ub294 \"\uc27d\uac8c \ub118\ubcf4\uae30 \ud798\ub4e0 \uba85\ub791\ud568\uc774 \uac00\ub4dd\ud558\ub2e4. \" \ub0a8\uad81\uc778\uc740 \ud0dc\uc591\uc758 \ud6c4\uc608\uac00 \ub300\uc911\uc758 \"\uc695\ub9dd\uc758 \ud604\uc7ac \uc8fc\uc18c\"\ub97c \ud3ec\ucc29\ud55c \ub4a4 \uc2dc\ub3c4\ud55c \"\ub098\ub984\ub300\ub85c\uc758 \uc791\ud488\uc138\uacc4\uc640 \uc791\ud654 \ubc29\ubc95\"\uc774 \uc628 \uc138\uacc4\ub97c \"\uc9c0\ubc30\ud560 \uc815\ub3c4\uc758 \uc7a5\ub974\ud654\ub41c \ud558\ub098\uc758 \ud604\uc0c1\"\uc774 \ub418\uc5c8\ub2e4\uace0 \uacb0\ub860\uc9c0\uc5c8\ub2e4.", "paragraph_answer": "\uc601\uad6d BBC\ub294\u300a\ud0dc\uc591\uc758 \ud6c4\uc608\u300b\uac00 \ud55c\ub958\uc758 \uc815\uc810\uc744 \ucc0d\uc744 \uac83\uc774\ub77c \uc608\uc0c1\ud588\ub2e4. \uc720\uac74\uc2dd\uc740 \"\ud0c4\ud0c4\ud558\uba74\uc11c\ub3c4 \ube60\ub978 \uc2a4\ud1a0\ub9ac \uc804\uac1c\uc640 \uc601\ud654 \uac19\uc740 \uc9c8\uac10, \uc778\ub958\uc560\uc5d0\uc11c \ud53c\uc5b4\ub098\ub294 \ucee4\ud50c\ub4e4\uc758 \uc0ac\ub791\uc774 \uc801\uc808\ud788 \ubc30\ubd84\ub41c \uac83\uc774 \uc778\uae30\uc694\uc778\uc73c\ub85c \uaf3d\ud788\uace0 \uc788\ub2e4.\u201d\uace0 \uc804\ud588\ub2e4. \ub0a8\uad81\uc778 \uc740 \ud3c9\ub860\uc5d0\uc11c \u300a\ud0dc\uc591\uc758 \ud6c4\uc608\u300b\uc758 \uc778\uae30\uc694\uc778\uc5d0 \ub300\ud574 \ub2e4\uc74c\uacfc \uac19\uc774 \ubd84\uc11d\ud588\ub2e4. \u300a\ud0dc\uc591\uc758 \ud6c4\uc608\u300b\ub294 \"\uc8fc\uc5b4\uc9c4 \uc2dc\uac04\uc5d0 \uadf9\uc801\uc778 \uc774\uc57c\uae30\ub97c \ubc30\uce58\ud558\ub294 \ud6a8\uc728\uc131\uc73c\ub85c \ud1b1\ub2c8\ubc14\ud034\ucc98\ub7fc \ub9de\ubb3c\ub824 \uc788\ub2e4. \uc774\uc57c\uae30\ud558\uace0\uc790 \ud558\ub294 \ubc14\uc640, \uadf8 \uc774\uc57c\uae30\ub97c \uc2dc\uac01\ud654\ud558\uc5ec \ubcf4\uc5ec\uc8fc\uace0\uc790 \ud558\ub294 \uc695\ub9dd\uc744 \uc5b4\ub514\uae4c\uc9c0 \uc2e4\ud604\ud560 \uc218 \uc788\ub290\ub0d0\ub294 \uc9c8\ubb38\uc5d0 \ub2f5\ud558\ub294, \uadf9\ub2e8\uc801\uc778 \ubc1c\uc804\uc758 \ud55c \uc608\ub2e4. \u300a\ud0dc\uc591\uc758 \ud6c4\uc608\u300b\uc5d0\uc11c \ub300\uc911\uc740 \"\ub2e4\uc774\ub0b4\ubbf9\ud55c \ud074\ub77c\uc774\ub9e5\uc2a4, \uadf8\ub9ac\uace0 \uae30\uc6d0\ud558\ub358 \uac83\uc774 \uc774\ub8e8\uc5b4\uc84c\uc744 \ub54c\uc758 \uce74\ud0c0\ub974\uc2dc\uc2a4\"\ub97c \ub290\ub07c\uac8c \ub41c\ub2e4. \u300a\ud0dc\uc591\uc758 \ud6c4\uc608\u300b\ub294 \"\uc774 \uc695\ub9dd\uc744 \uc2ed\ubd84 \ucda9\uc871\uc2dc\ud0a4\uae30 \uc704\ud574 \ud604\uc2e4\uc774\ub098 \uace0\uc99d\uc740 \uacfc\uac10\ud788 \ubb34\uc2dc\ud558\uace0, \uc77c\ubc18 \ub300\uc911\uc758 \ub208\ub192\uc774\uc5d0 \uc11c\uc11c, \uadf8\ub4e4\uc774 \ubcf4\uace0 \uc2f6\uc740 \uc7a5\uba74\uc774\ub77c\uba74 \ubb34\uc5c7\uc774\ub4e0 \ubcf4\uc5ec\uc900\ub2e4. (\uc911\ub7b5) \ub300\uc911\uc774 \ubcf4\uace0 \uc2f6\uc5b4 \ud558\ub294 \uac83\uc740 \ucd1d\ud0c4\uc774 \ubc30\uc5d0 \ubc15\ud614\ub2e4\uace0 \ubc14\uc774\ud0c8\uc744 \uc7ac\uba74\uc11c \ubcd1\uc6d0\uc73c\ub85c \uc2e0\uc18d\ud788 \ud6c4\uc1a1\ud558\ub294 \uc7a5\uba74\ub3c4 \uc544\ub2c8\uace0, \ub2e4\uc774\uc544\ubaac\ub4dc\ub294 \uc548 \ucc0d\ud614\uc9c0\ub9cc \ud544\ub984\uc744 \ub4e4\uace0 \uc640\uc11c \uc815\ud669\uc0c1 \uc758\uc2ec \uac00\ub2a5\ud560 \uc218 \uc788\ub2e4\uace0 \ub9dd\uc124\uc774\ub294 \uc7a5\uba74\ub3c4, \uccad\uc9c4\uae30\ub97c \uc548 \ub300\uace0 \ubc30\ub97c \uc5f4\uc5b4 \uc870\uc2ec\uc870\uc2ec \ucd09\uc9c4\ud574 \ud658\uc790\uc758 \ubc18\uc751\uc744 \uc0b4\ud53c\ub294 \uac83\ub3c4 \uc544\ub2c8\ub2e4. \ub300\uc911\uacfc TV \uc548\uc5d0\ub294 \uc9c1\uad00\uc801\uc778 \uc138\uc0c1\uc774 \uc788\uc744 \ubfd0, \uc544\ubb34\ub3c4 \uadf8\uac83\uc774 \ud604\uc2e4\uc801\uc778 \uacbd\uacc4\uc5d0 \uc788\uc5b4\uc57c \ud55c\ub2e4\uace0 \uac15\uc694\ud558\uc9c0 \uc54a\ub294\ub2e4. \uadf8\ub798\uc11c, \uc774 \ub4dc\ub77c\ub9c8\ub294 \uacfc\uac10\ud788 \ud604\uc2e4\uc744 \ud0c8\ud53c\ud558\uace0 \ubd95\uad34\uc2dc\ucf1c \uc790\uae30\ub4e4\uc774 \uac00\uc7a5 \uc798 \ud558\ub294 \uc774\uc57c\uae30\ub97c \ud568\uc73c\ub85c\uc368 \ubaa8\ub450\uc758 \uc695\ub9dd\uc744 \ubc1c\ud604\uc2dc\ud0a8\ub2e4. (\uc911\ub7b5) \ucd1d\ud0c4\uc740 \uc989\uc2dc \ud5e4\uc9d1\uc5b4 \ubf51\uc544\uc57c \ud558\uba70, \uc5d1\uc2a4\ub808\uc774\uc5d0\ub294 \ubd84\uba85\ud788 \ud558\uc580 \ub2e4\uc774\uc544\ubaac\ub4dc\uac00 \ucc0d\ud600\uc57c \ud55c\ub2e4. \ud1b5\ub150\uc0c1 \ubc18\uad70 \uc9c0\ub3c4\uc790\ub294 \uc544\ud504\ub9ac\uce74 \uc778\uc774\uace0, \ud30c\ubcd1\ub41c \ub098\ub77c \uc0ac\ub78c\ub4e4\uc740 \uc544\ub78d\uc5b4\ub97c \uc4f0\uace0, \uac31\uc740 \uc601\uc5b4\ub97c \uc4f0\ub294 \ubc31\uc778\uc774\uc5b4\uc57c \ud560 \uac83 \uac19\"\ub2e4\uace0 \ub300\uc911\uc774 \uc0dd\uac01\ud558\uae30 \ub54c\ubb38\uc5d0 \u300a\ud0dc\uc591\uc758 \ud6c4\uc608\u300b\ub294 \"\uadf8\ub7f0 \ub098\ub77c\uc640 \uc2dc\uacf5\uc744 \uad6c\ud604\ud574\ubc84\"\ub838\ub2e4. \"\uc81c\uc791\uc9c4\uc774 \uc758\ub3c4\uc801\uc73c\ub85c \ud3bc\uccd0\ub0b8, \ubb3c\ub9ac\uc801\uc73c\ub85c \ub4a4\uc8fd\ubc15\uc8fd\uc778 \uc774 \uacf5\uac04\uc5d0\uc11c, \uc774\uc57c\uae30\ub294 \ub0a0\uac1c\uac00 \ub3cb\uc544 \uad00\uac1d\uc758 \ub208\ub3d9\uc790\ub97c \ud33d\ud33d \ub3cc\ub9ac\uba70 \ubc1c\ud604\ub41c\ub2e4. \uc5ec\uc8fc\uc778\uacf5\uc758 \ucc28\ub294 \ub9e4\ud68c \uace4\ub450\ubc15\uc9c8\uce58\uace0, \ucd94\ub77d\ud558\uace0, \uc9c0\ub8b0\ubc2d\uc5d0 \ub4e4\uc5b4\uac04\ub2e4. \ub354\ubd88\uc5b4 \uc9c0\uc9c4\ub3c4 \ub098\uace0, \uc801\uad70\ub3c4 \uccd0\ub4e4\uc5b4 \uc624\uace0, \uc804\uc5fc\ubcd1\ub3c4 \ub3cc\uace0, \uac31\uc740 \ub9e4\ubc88 \ub3c4\ub07c\ub208\uc73c\ub85c \ucd1d\uc744 \uc3dc\ub2e4. \uadf8 \uc7a5\uce58\ub85c \uacf5\uace0\ud574\uc9c0\ub294 \uac83\uc740 \uc8fc\uc5b4\uc9c4 \uc721\uc2ed \ubd84\uc5d0\ub3c4 \uba87 \ubc88\uc529 \ud074\ub77c\uc774\ub9e5\uc2a4\ub97c \ub9cc\ub4e4 \uc218 \uc788\ub294 \uc2dc\uac01\uc801 \uad6c\ud604\uacfc, \uba87 \ubc88\uc529 \uba5c\ub85c\uc801 \uce74\ud0c0\ub974\uc2dc\uc2a4\ub97c \ub290\ub084 \uc218 \uc788\ub294 \ub2ec\ub2ec\ud55c \uc7a5\uba74\uc774\ub2e4.\" \ub610\ud55c \ub0a8\uad81\uc778\uc740 \"\uadf8 \uc640\uc911\uc5d0 \uc8fc\uc778\uacf5\ub4e4\uc774 \uc8fc\uace0\ubc1b\ub294 \ub300\uc0ac\ub4e4\uc740 \uc704\ud2b8\ub098 \uae34\ubc15\ud568\uc5d0\uc11c \ud765\ubbf8\ub86d\uace0 \uae30\ubc1c\"\ud558\uace0 \uc7a5\uba74\uc740 \"\ucc38\uc2e0\"\ud558\ub2e4 \ud3c9\ud588\ub2e4. \uadf8\ub798\uc11c \u300a\ud0dc\uc591\uc758 \ud6c4\uc608\u300b\uc5d0\ub294 \"\uc27d\uac8c \ub118\ubcf4\uae30 \ud798\ub4e0 \uba85\ub791\ud568\uc774 \uac00\ub4dd\ud558\ub2e4.\" \ub0a8\uad81\uc778\uc740 \ud0dc\uc591\uc758 \ud6c4\uc608\uac00 \ub300\uc911\uc758 \"\uc695\ub9dd\uc758 \ud604\uc7ac \uc8fc\uc18c\"\ub97c \ud3ec\ucc29\ud55c \ub4a4 \uc2dc\ub3c4\ud55c \"\ub098\ub984\ub300\ub85c\uc758 \uc791\ud488\uc138\uacc4\uc640 \uc791\ud654 \ubc29\ubc95\"\uc774 \uc628 \uc138\uacc4\ub97c \"\uc9c0\ubc30\ud560 \uc815\ub3c4\uc758 \uc7a5\ub974\ud654\ub41c \ud558\ub098\uc758 \ud604\uc0c1\"\uc774 \ub418\uc5c8\ub2e4\uace0 \uacb0\ub860\uc9c0\uc5c8\ub2e4.", "sentence_answer": " \ub0a8\uad81\uc778 \uc740 \ud3c9\ub860\uc5d0\uc11c \u300a\ud0dc\uc591\uc758 \ud6c4\uc608\u300b\uc758 \uc778\uae30\uc694\uc778\uc5d0 \ub300\ud574 \ub2e4\uc74c\uacfc \uac19\uc774 \ubd84\uc11d\ud588\ub2e4.", "paragraph_id": "6598975-10-1", "paragraph_question": "question: \ud0dc\uc591\uc758 \ud6c4\uc608\uac00 \ud604\uc2e4\uc744 \ud0c8\ud53c\ud558\uace0 \ub300\uc911\uc774 \uc6d0\ud558\ub294 \uc695\ub9dd\uc744 \ucda9\uc871\uc2dc\ud0a4\ub294\ub370 \uc778\uae30\uc758 \uc694\uc778\uc774 \uc788\ub2e4\uace0 \ud55c \uc0ac\ub78c\uc740?, context: \uc601\uad6d BBC\ub294\u300a\ud0dc\uc591\uc758 \ud6c4\uc608\u300b\uac00 \ud55c\ub958\uc758 \uc815\uc810\uc744 \ucc0d\uc744 \uac83\uc774\ub77c \uc608\uc0c1\ud588\ub2e4. \uc720\uac74\uc2dd\uc740 \"\ud0c4\ud0c4\ud558\uba74\uc11c\ub3c4 \ube60\ub978 \uc2a4\ud1a0\ub9ac \uc804\uac1c\uc640 \uc601\ud654 \uac19\uc740 \uc9c8\uac10, \uc778\ub958\uc560\uc5d0\uc11c \ud53c\uc5b4\ub098\ub294 \ucee4\ud50c\ub4e4\uc758 \uc0ac\ub791\uc774 \uc801\uc808\ud788 \ubc30\ubd84\ub41c \uac83\uc774 \uc778\uae30\uc694\uc778\uc73c\ub85c \uaf3d\ud788\uace0 \uc788\ub2e4.\u201d\uace0 \uc804\ud588\ub2e4. \ub0a8\uad81\uc778\uc740 \ud3c9\ub860\uc5d0\uc11c \u300a\ud0dc\uc591\uc758 \ud6c4\uc608\u300b\uc758 \uc778\uae30\uc694\uc778\uc5d0 \ub300\ud574 \ub2e4\uc74c\uacfc \uac19\uc774 \ubd84\uc11d\ud588\ub2e4. \u300a\ud0dc\uc591\uc758 \ud6c4\uc608\u300b\ub294 \"\uc8fc\uc5b4\uc9c4 \uc2dc\uac04\uc5d0 \uadf9\uc801\uc778 \uc774\uc57c\uae30\ub97c \ubc30\uce58\ud558\ub294 \ud6a8\uc728\uc131\uc73c\ub85c \ud1b1\ub2c8\ubc14\ud034\ucc98\ub7fc \ub9de\ubb3c\ub824 \uc788\ub2e4. \uc774\uc57c\uae30\ud558\uace0\uc790 \ud558\ub294 \ubc14\uc640, \uadf8 \uc774\uc57c\uae30\ub97c \uc2dc\uac01\ud654\ud558\uc5ec \ubcf4\uc5ec\uc8fc\uace0\uc790 \ud558\ub294 \uc695\ub9dd\uc744 \uc5b4\ub514\uae4c\uc9c0 \uc2e4\ud604\ud560 \uc218 \uc788\ub290\ub0d0\ub294 \uc9c8\ubb38\uc5d0 \ub2f5\ud558\ub294, \uadf9\ub2e8\uc801\uc778 \ubc1c\uc804\uc758 \ud55c \uc608\ub2e4. \u300a\ud0dc\uc591\uc758 \ud6c4\uc608\u300b\uc5d0\uc11c \ub300\uc911\uc740 \"\ub2e4\uc774\ub0b4\ubbf9\ud55c \ud074\ub77c\uc774\ub9e5\uc2a4, \uadf8\ub9ac\uace0 \uae30\uc6d0\ud558\ub358 \uac83\uc774 \uc774\ub8e8\uc5b4\uc84c\uc744 \ub54c\uc758 \uce74\ud0c0\ub974\uc2dc\uc2a4\"\ub97c \ub290\ub07c\uac8c \ub41c\ub2e4. \u300a\ud0dc\uc591\uc758 \ud6c4\uc608\u300b\ub294 \"\uc774 \uc695\ub9dd\uc744 \uc2ed\ubd84 \ucda9\uc871\uc2dc\ud0a4\uae30 \uc704\ud574 \ud604\uc2e4\uc774\ub098 \uace0\uc99d\uc740 \uacfc\uac10\ud788 \ubb34\uc2dc\ud558\uace0, \uc77c\ubc18 \ub300\uc911\uc758 \ub208\ub192\uc774\uc5d0 \uc11c\uc11c, \uadf8\ub4e4\uc774 \ubcf4\uace0 \uc2f6\uc740 \uc7a5\uba74\uc774\ub77c\uba74 \ubb34\uc5c7\uc774\ub4e0 \ubcf4\uc5ec\uc900\ub2e4. (\uc911\ub7b5) \ub300\uc911\uc774 \ubcf4\uace0 \uc2f6\uc5b4 \ud558\ub294 \uac83\uc740 \ucd1d\ud0c4\uc774 \ubc30\uc5d0 \ubc15\ud614\ub2e4\uace0 \ubc14\uc774\ud0c8\uc744 \uc7ac\uba74\uc11c \ubcd1\uc6d0\uc73c\ub85c \uc2e0\uc18d\ud788 \ud6c4\uc1a1\ud558\ub294 \uc7a5\uba74\ub3c4 \uc544\ub2c8\uace0, \ub2e4\uc774\uc544\ubaac\ub4dc\ub294 \uc548 \ucc0d\ud614\uc9c0\ub9cc \ud544\ub984\uc744 \ub4e4\uace0 \uc640\uc11c \uc815\ud669\uc0c1 \uc758\uc2ec \uac00\ub2a5\ud560 \uc218 \uc788\ub2e4\uace0 \ub9dd\uc124\uc774\ub294 \uc7a5\uba74\ub3c4, \uccad\uc9c4\uae30\ub97c \uc548 \ub300\uace0 \ubc30\ub97c \uc5f4\uc5b4 \uc870\uc2ec\uc870\uc2ec \ucd09\uc9c4\ud574 \ud658\uc790\uc758 \ubc18\uc751\uc744 \uc0b4\ud53c\ub294 \uac83\ub3c4 \uc544\ub2c8\ub2e4. \ub300\uc911\uacfc TV \uc548\uc5d0\ub294 \uc9c1\uad00\uc801\uc778 \uc138\uc0c1\uc774 \uc788\uc744 \ubfd0, \uc544\ubb34\ub3c4 \uadf8\uac83\uc774 \ud604\uc2e4\uc801\uc778 \uacbd\uacc4\uc5d0 \uc788\uc5b4\uc57c \ud55c\ub2e4\uace0 \uac15\uc694\ud558\uc9c0 \uc54a\ub294\ub2e4. \uadf8\ub798\uc11c, \uc774 \ub4dc\ub77c\ub9c8\ub294 \uacfc\uac10\ud788 \ud604\uc2e4\uc744 \ud0c8\ud53c\ud558\uace0 \ubd95\uad34\uc2dc\ucf1c \uc790\uae30\ub4e4\uc774 \uac00\uc7a5 \uc798 \ud558\ub294 \uc774\uc57c\uae30\ub97c \ud568\uc73c\ub85c\uc368 \ubaa8\ub450\uc758 \uc695\ub9dd\uc744 \ubc1c\ud604\uc2dc\ud0a8\ub2e4. (\uc911\ub7b5) \ucd1d\ud0c4\uc740 \uc989\uc2dc \ud5e4\uc9d1\uc5b4 \ubf51\uc544\uc57c \ud558\uba70, \uc5d1\uc2a4\ub808\uc774\uc5d0\ub294 \ubd84\uba85\ud788 \ud558\uc580 \ub2e4\uc774\uc544\ubaac\ub4dc\uac00 \ucc0d\ud600\uc57c \ud55c\ub2e4. \ud1b5\ub150\uc0c1 \ubc18\uad70 \uc9c0\ub3c4\uc790\ub294 \uc544\ud504\ub9ac\uce74 \uc778\uc774\uace0, \ud30c\ubcd1\ub41c \ub098\ub77c \uc0ac\ub78c\ub4e4\uc740 \uc544\ub78d\uc5b4\ub97c \uc4f0\uace0, \uac31\uc740 \uc601\uc5b4\ub97c \uc4f0\ub294 \ubc31\uc778\uc774\uc5b4\uc57c \ud560 \uac83 \uac19\"\ub2e4\uace0 \ub300\uc911\uc774 \uc0dd\uac01\ud558\uae30 \ub54c\ubb38\uc5d0 \u300a\ud0dc\uc591\uc758 \ud6c4\uc608\u300b\ub294 \"\uadf8\ub7f0 \ub098\ub77c\uc640 \uc2dc\uacf5\uc744 \uad6c\ud604\ud574\ubc84\"\ub838\ub2e4. \"\uc81c\uc791\uc9c4\uc774 \uc758\ub3c4\uc801\uc73c\ub85c \ud3bc\uccd0\ub0b8, \ubb3c\ub9ac\uc801\uc73c\ub85c \ub4a4\uc8fd\ubc15\uc8fd\uc778 \uc774 \uacf5\uac04\uc5d0\uc11c, \uc774\uc57c\uae30\ub294 \ub0a0\uac1c\uac00 \ub3cb\uc544 \uad00\uac1d\uc758 \ub208\ub3d9\uc790\ub97c \ud33d\ud33d \ub3cc\ub9ac\uba70 \ubc1c\ud604\ub41c\ub2e4. \uc5ec\uc8fc\uc778\uacf5\uc758 \ucc28\ub294 \ub9e4\ud68c \uace4\ub450\ubc15\uc9c8\uce58\uace0, \ucd94\ub77d\ud558\uace0, \uc9c0\ub8b0\ubc2d\uc5d0 \ub4e4\uc5b4\uac04\ub2e4. \ub354\ubd88\uc5b4 \uc9c0\uc9c4\ub3c4 \ub098\uace0, \uc801\uad70\ub3c4 \uccd0\ub4e4\uc5b4 \uc624\uace0, \uc804\uc5fc\ubcd1\ub3c4 \ub3cc\uace0, \uac31\uc740 \ub9e4\ubc88 \ub3c4\ub07c\ub208\uc73c\ub85c \ucd1d\uc744 \uc3dc\ub2e4. \uadf8 \uc7a5\uce58\ub85c \uacf5\uace0\ud574\uc9c0\ub294 \uac83\uc740 \uc8fc\uc5b4\uc9c4 \uc721\uc2ed \ubd84\uc5d0\ub3c4 \uba87 \ubc88\uc529 \ud074\ub77c\uc774\ub9e5\uc2a4\ub97c \ub9cc\ub4e4 \uc218 \uc788\ub294 \uc2dc\uac01\uc801 \uad6c\ud604\uacfc, \uba87 \ubc88\uc529 \uba5c\ub85c\uc801 \uce74\ud0c0\ub974\uc2dc\uc2a4\ub97c \ub290\ub084 \uc218 \uc788\ub294 \ub2ec\ub2ec\ud55c \uc7a5\uba74\uc774\ub2e4.\" \ub610\ud55c \ub0a8\uad81\uc778\uc740 \"\uadf8 \uc640\uc911\uc5d0 \uc8fc\uc778\uacf5\ub4e4\uc774 \uc8fc\uace0\ubc1b\ub294 \ub300\uc0ac\ub4e4\uc740 \uc704\ud2b8\ub098 \uae34\ubc15\ud568\uc5d0\uc11c \ud765\ubbf8\ub86d\uace0 \uae30\ubc1c\"\ud558\uace0 \uc7a5\uba74\uc740 \"\ucc38\uc2e0\"\ud558\ub2e4 \ud3c9\ud588\ub2e4. \uadf8\ub798\uc11c \u300a\ud0dc\uc591\uc758 \ud6c4\uc608\u300b\uc5d0\ub294 \"\uc27d\uac8c \ub118\ubcf4\uae30 \ud798\ub4e0 \uba85\ub791\ud568\uc774 \uac00\ub4dd\ud558\ub2e4.\" \ub0a8\uad81\uc778\uc740 \ud0dc\uc591\uc758 \ud6c4\uc608\uac00 \ub300\uc911\uc758 \"\uc695\ub9dd\uc758 \ud604\uc7ac \uc8fc\uc18c\"\ub97c \ud3ec\ucc29\ud55c \ub4a4 \uc2dc\ub3c4\ud55c \"\ub098\ub984\ub300\ub85c\uc758 \uc791\ud488\uc138\uacc4\uc640 \uc791\ud654 \ubc29\ubc95\"\uc774 \uc628 \uc138\uacc4\ub97c \"\uc9c0\ubc30\ud560 \uc815\ub3c4\uc758 \uc7a5\ub974\ud654\ub41c \ud558\ub098\uc758 \ud604\uc0c1\"\uc774 \ub418\uc5c8\ub2e4\uace0 \uacb0\ub860\uc9c0\uc5c8\ub2e4."} {"question": "\uc2dc\ud654\ud638\uc5d0 \uc0b4\uace0 \uc788\ub294 \uc5b4\ub5a4 \uc0dd\ubb3c\uc5d0\uc11c \uae30\uc900\uce58\uc758 10\ubc30 \uc774\uc0c1 \ub418\ub294 \uc911\uae08\uc18d\uc774 \uac80\ucd9c\ub418\uc5c8\ub098?", "paragraph": "\uc2dc\ud654\ud638\uc758 \uc624\uc5fc \ubb3c\uc9c8 \ubc30\ucd9c \ub17c\ub780\uc740 1996\ub144 \uc81c1 \ubc30\uc218\uac11\ubb38 \uc2dc\ubc94 \ud574\uc218\uc720\ud1b5 \ub54c \ubd80\ud130 \uc2dc\uc791\ub418\uc5c8\ub2e4. \uc2dc\ubc94\uc720\ud1b5\uc774\ub77c\ub294 \uba85\ubaa9\uc73c\ub85c \uc624\uc5fc\ub41c \ud638\uc22b\ubb3c\uc744 \ubb34\ub2e8 \ubc29\ub958\ud558\uc5ec \uc778\uadfc \ud574\uc5ed\uc758 \uc624\uc5fc \ubb38\uc81c\uac00 \ub300\ub450\ub41c \uac83\uc774\ub2e4. \uc774\uc5d0 \uc778\uadfc \uc8fc\ubbfc\uacfc \ud658\uacbd\ub2e8\uccb4\uc758 \ubc18\ubc1c\uc744 \uc0c0\uace0 \ubc29\ub958\uae08\uc9c0 \uac00\ucc98\ubd84 \uc18c\uc1a1\uc73c\ub85c \uac80\ucc30 \uc218\uc0ac\uac00 \uc9c4\ud589\ub418\uc5c8\ub2e4. \uadf8 \ud6c4 \uacc4\uc18d\ub41c \ud574\uc218\uc720\ud1b5\uc73c\ub85c 2000\ub144\uc5d0 \ub4e4\uc5b4 \uc218\uc9c8\uc740 \ube60\ub974\uac8c \uac1c\uc120\ub418\uc5c8\uc9c0\ub9cc, 2001\ub144 \ud55c\uad6d\ud574\uc591\uc218\uc0b0\uac1c\ubc1c\uc6d0 \ubcf4\uace0\uc11c\uc5d0\uc11c \ub530\ub974\uba74 \ubc18\uc6d4\uacf5\ub2e8 \ud558\ub958\uc9c0\uc5ed\uc758 \ud1f4\uc801\ud1a0\ub294 \uc5ec\uc804\ud788 \uc911\uae08\uc18d \uc624\uc5fc\uc774 \uc2ec\uac01\ud55c \uac83\uc73c\ub85c \ub098\ud0c0\ub0ac\ub2e4. \uc774\uc5d0 \ud574\uc591\uc218\uc0b0\ubd80\ub294 \uc870\ub825\ubc1c\uc804\uc18c \uac74\uc124\uacfc \ud568\uaed8 \uc624\uc5fc \ud1f4\uc801\ud1a0 \uc900\uc124\uc0ac\uc5c5\ub3c4 \ubcd1\ud589\ud55c\ub2e4\ub294 \uc785\uc7a5\uc774\uc5c8\uc9c0\ub9cc, 2006\ub144 \uc218\uc790\uc6d0\uacf5\uc0ac \ub0b4\ubd80 \ubcf4\uace0\uc11c\uc5d0\uc11c\ub294 \uc5ec\uc804\ud788 \ud1f4\uc801\ud1a0\uc5d0\uc11c \uace0\ub18d\ub3c4 \uc911\uae08\uc18d\uc774 \uac80\ucd9c\ub41c \uac83\uc73c\ub85c \ud655\uc778\ub418\uc5c8\ub2e4. \ud1a0\uc591 13\uacf3 \uc911 12\uacf3\uc5d0\uc11c \ubbf8\uad6d \ud574\uc591\ub300\uae30\uccad \uae30\uc900\uc744 \ub118\ub294 \uad6c\ub9ac\uac00 \uac80\ucd9c\ub418\uc5c8\uace0, \ub0a9\uc774\ub098 \uc544\uc5f0\ub3c4 \ubbf8\uad6d \uae30\uc900\uc73c\ub85c \uc911\uac04\uc624\uc5fc\uc774\ub098 \uc2ec\ud55c\uc624\uc5fc \ub2e8\uacc4\uc600\ub2e4. \uc2dc\ud654\ud638 \uac2f\uc9c0\ub801\uc774\uc5d0\uc11c\ub294 10\ubc30 \uc774\uc0c1 \ub9ce\uc740 \uc911\uae08\uc18d\uc774 \uac80\ucd9c\ub418\uc5c8\ub2e4. \uc774\ub7f0 \uc624\uc5fc\uc740 \uba39\uc774 \uc0ac\uc2ac\uc5d0 \ub530\ub77c \ub3d9\ubb3c\uc758 \ubab8\uc73c\ub85c \ucd95\uc801\ub418\uc5b4 \uba39\uac70\ub9ac\ub97c \uc704\ud611\ud558\ub294 \uacb0\uacfc\ub97c \ub0b3\ub294\ub2e4. \uadf8\ub7f0\ub370 \uc870\ub825\ubc1c\uc804\uc18c\uac00 \uac74\uc124\ub418\uba74 \uc774\ub4e4\uc774 \ubc14\ub2e4\ub85c \ud758\ub7ec\ub4e4\uc5b4\uac08 \uc218 \uc788\ub2e4\ub294 \uac83\uc774\ub2e4. \ub354\ubd88\uc5b4 \ub300\ud55c\ubbfc\uad6d\uc5d0\uc11c\ub294 \ud1f4\uc801\ud1a0\uc5d0 \ub300\ud55c \uc815\ud655\ud55c \uae30\uc900\uc774 \uc5c6\ub294 \uc0c1\ud669\uc774\uc5c8\uace0, \uc624\uc5fc\ub41c \ud1f4\uc801\ud1a0 \uc900\uc124\uc5d0 \ucd5c\uc18c 100\uc5b5\uc6d0, \ucd5c\ub300 400\uc5b5\uc6d0\uc774 \ub4e4 \uac83\uc73c\ub85c \ucd94\uc0b0\ub418\ub294 \uc0c1\ud669\uc774\uc5c8\ub2e4. \uc774\uc5d0 \uc218\uc790\uc6d0\uacf5\uc0ac \uad00\uacc4\uc790\ub294 \u201c\uc2dc\ud654\ud638 \uc870\ub825\ubc1c\uc804\uc18c\uac00 \uc720\uc5d4\uae30\ud6c4\ubcc0\ud654\ud611\uc57d\uc73c\ub85c\ubd80\ud130 \uccad\uc815\uac1c\ubc1c\uccb4\uc81c \uc2b9\uc778\uc744 \ubc1b\uc544 \uce5c\ud658\uacbd\uc801\u201d\uc774\ub77c\uba74\uc11c, \u201c\uc870\ub825\ubc1c\uc804\uc18c \ud130\ube48 \uac00\ub3d9\uc601\ud5a5\uad8c\uc5d0 \ub4dc\ub294 \uba74\uc801\uc774 \ubcc4\ub85c \ub113\uc9c0 \uc54a\uc544 \ubc14\ub2e4\ub85c \ube60\uc838\ub098\uac00\ub294 \ud1f4\uc801\ubb3c\uc774 \ud06c\uac8c \ub9ce\uc9c0 \uc54a\uc744 \uac83\uc774\ub2e4. \ud1f4\uc801\ubb3c\uc601\ud5a5 \uc6a9\uc5ed\uc870\uc0ac \uacb0\uacfc\ub97c \ubcf4\uace0 \ubb38\uc81c\uac00 \uc788\ub2e4\uba74 \uc870\uce58\ud560 \uac83\u201d\uc774\ub77c \ub2f5\ud558\uc600\ub2e4.", "answer": "\uac2f\uc9c0\ub801\uc774", "sentence": "\uc2dc\ud654\ud638 \uac2f\uc9c0\ub801\uc774 \uc5d0\uc11c\ub294 10\ubc30 \uc774\uc0c1 \ub9ce\uc740 \uc911\uae08\uc18d\uc774 \uac80\ucd9c\ub418\uc5c8\ub2e4.", "paragraph_sentence": "\uc2dc\ud654\ud638\uc758 \uc624\uc5fc \ubb3c\uc9c8 \ubc30\ucd9c \ub17c\ub780\uc740 1996\ub144 \uc81c1 \ubc30\uc218\uac11\ubb38 \uc2dc\ubc94 \ud574\uc218\uc720\ud1b5 \ub54c \ubd80\ud130 \uc2dc\uc791\ub418\uc5c8\ub2e4. \uc2dc\ubc94\uc720\ud1b5\uc774\ub77c\ub294 \uba85\ubaa9\uc73c\ub85c \uc624\uc5fc\ub41c \ud638\uc22b\ubb3c\uc744 \ubb34\ub2e8 \ubc29\ub958\ud558\uc5ec \uc778\uadfc \ud574\uc5ed\uc758 \uc624\uc5fc \ubb38\uc81c\uac00 \ub300\ub450\ub41c \uac83\uc774\ub2e4. \uc774\uc5d0 \uc778\uadfc \uc8fc\ubbfc\uacfc \ud658\uacbd\ub2e8\uccb4\uc758 \ubc18\ubc1c\uc744 \uc0c0\uace0 \ubc29\ub958\uae08\uc9c0 \uac00\ucc98\ubd84 \uc18c\uc1a1\uc73c\ub85c \uac80\ucc30 \uc218\uc0ac\uac00 \uc9c4\ud589\ub418\uc5c8\ub2e4. \uadf8 \ud6c4 \uacc4\uc18d\ub41c \ud574\uc218\uc720\ud1b5\uc73c\ub85c 2000\ub144\uc5d0 \ub4e4\uc5b4 \uc218\uc9c8\uc740 \ube60\ub974\uac8c \uac1c\uc120\ub418\uc5c8\uc9c0\ub9cc, 2001\ub144 \ud55c\uad6d\ud574\uc591\uc218\uc0b0\uac1c\ubc1c\uc6d0 \ubcf4\uace0\uc11c\uc5d0\uc11c \ub530\ub974\uba74 \ubc18\uc6d4\uacf5\ub2e8 \ud558\ub958\uc9c0\uc5ed\uc758 \ud1f4\uc801\ud1a0\ub294 \uc5ec\uc804\ud788 \uc911\uae08\uc18d \uc624\uc5fc\uc774 \uc2ec\uac01\ud55c \uac83\uc73c\ub85c \ub098\ud0c0\ub0ac\ub2e4. \uc774\uc5d0 \ud574\uc591\uc218\uc0b0\ubd80\ub294 \uc870\ub825\ubc1c\uc804\uc18c \uac74\uc124\uacfc \ud568\uaed8 \uc624\uc5fc \ud1f4\uc801\ud1a0 \uc900\uc124\uc0ac\uc5c5\ub3c4 \ubcd1\ud589\ud55c\ub2e4\ub294 \uc785\uc7a5\uc774\uc5c8\uc9c0\ub9cc, 2006\ub144 \uc218\uc790\uc6d0\uacf5\uc0ac \ub0b4\ubd80 \ubcf4\uace0\uc11c\uc5d0\uc11c\ub294 \uc5ec\uc804\ud788 \ud1f4\uc801\ud1a0\uc5d0\uc11c \uace0\ub18d\ub3c4 \uc911\uae08\uc18d\uc774 \uac80\ucd9c\ub41c \uac83\uc73c\ub85c \ud655\uc778\ub418\uc5c8\ub2e4. \ud1a0\uc591 13\uacf3 \uc911 12\uacf3\uc5d0\uc11c \ubbf8\uad6d \ud574\uc591\ub300\uae30\uccad \uae30\uc900\uc744 \ub118\ub294 \uad6c\ub9ac\uac00 \uac80\ucd9c\ub418\uc5c8\uace0, \ub0a9\uc774\ub098 \uc544\uc5f0\ub3c4 \ubbf8\uad6d \uae30\uc900\uc73c\ub85c \uc911\uac04\uc624\uc5fc\uc774\ub098 \uc2ec\ud55c\uc624\uc5fc \ub2e8\uacc4\uc600\ub2e4. \uc2dc\ud654\ud638 \uac2f\uc9c0\ub801\uc774 \uc5d0\uc11c\ub294 10\ubc30 \uc774\uc0c1 \ub9ce\uc740 \uc911\uae08\uc18d\uc774 \uac80\ucd9c\ub418\uc5c8\ub2e4. \uc774\ub7f0 \uc624\uc5fc\uc740 \uba39\uc774 \uc0ac\uc2ac\uc5d0 \ub530\ub77c \ub3d9\ubb3c\uc758 \ubab8\uc73c\ub85c \ucd95\uc801\ub418\uc5b4 \uba39\uac70\ub9ac\ub97c \uc704\ud611\ud558\ub294 \uacb0\uacfc\ub97c \ub0b3\ub294\ub2e4. \uadf8\ub7f0\ub370 \uc870\ub825\ubc1c\uc804\uc18c\uac00 \uac74\uc124\ub418\uba74 \uc774\ub4e4\uc774 \ubc14\ub2e4\ub85c \ud758\ub7ec\ub4e4\uc5b4\uac08 \uc218 \uc788\ub2e4\ub294 \uac83\uc774\ub2e4. \ub354\ubd88\uc5b4 \ub300\ud55c\ubbfc\uad6d\uc5d0\uc11c\ub294 \ud1f4\uc801\ud1a0\uc5d0 \ub300\ud55c \uc815\ud655\ud55c \uae30\uc900\uc774 \uc5c6\ub294 \uc0c1\ud669\uc774\uc5c8\uace0, \uc624\uc5fc\ub41c \ud1f4\uc801\ud1a0 \uc900\uc124\uc5d0 \ucd5c\uc18c 100\uc5b5\uc6d0, \ucd5c\ub300 400\uc5b5\uc6d0\uc774 \ub4e4 \uac83\uc73c\ub85c \ucd94\uc0b0\ub418\ub294 \uc0c1\ud669\uc774\uc5c8\ub2e4. \uc774\uc5d0 \uc218\uc790\uc6d0\uacf5\uc0ac \uad00\uacc4\uc790\ub294 \u201c\uc2dc\ud654\ud638 \uc870\ub825\ubc1c\uc804\uc18c\uac00 \uc720\uc5d4\uae30\ud6c4\ubcc0\ud654\ud611\uc57d\uc73c\ub85c\ubd80\ud130 \uccad\uc815\uac1c\ubc1c\uccb4\uc81c \uc2b9\uc778\uc744 \ubc1b\uc544 \uce5c\ud658\uacbd\uc801\u201d\uc774\ub77c\uba74\uc11c, \u201c\uc870\ub825\ubc1c\uc804\uc18c \ud130\ube48 \uac00\ub3d9\uc601\ud5a5\uad8c\uc5d0 \ub4dc\ub294 \uba74\uc801\uc774 \ubcc4\ub85c \ub113\uc9c0 \uc54a\uc544 \ubc14\ub2e4\ub85c \ube60\uc838\ub098\uac00\ub294 \ud1f4\uc801\ubb3c\uc774 \ud06c\uac8c \ub9ce\uc9c0 \uc54a\uc744 \uac83\uc774\ub2e4. \ud1f4\uc801\ubb3c\uc601\ud5a5 \uc6a9\uc5ed\uc870\uc0ac \uacb0\uacfc\ub97c \ubcf4\uace0 \ubb38\uc81c\uac00 \uc788\ub2e4\uba74 \uc870\uce58\ud560 \uac83\u201d\uc774\ub77c \ub2f5\ud558\uc600\ub2e4.", "paragraph_answer": "\uc2dc\ud654\ud638\uc758 \uc624\uc5fc \ubb3c\uc9c8 \ubc30\ucd9c \ub17c\ub780\uc740 1996\ub144 \uc81c1 \ubc30\uc218\uac11\ubb38 \uc2dc\ubc94 \ud574\uc218\uc720\ud1b5 \ub54c \ubd80\ud130 \uc2dc\uc791\ub418\uc5c8\ub2e4. \uc2dc\ubc94\uc720\ud1b5\uc774\ub77c\ub294 \uba85\ubaa9\uc73c\ub85c \uc624\uc5fc\ub41c \ud638\uc22b\ubb3c\uc744 \ubb34\ub2e8 \ubc29\ub958\ud558\uc5ec \uc778\uadfc \ud574\uc5ed\uc758 \uc624\uc5fc \ubb38\uc81c\uac00 \ub300\ub450\ub41c \uac83\uc774\ub2e4. \uc774\uc5d0 \uc778\uadfc \uc8fc\ubbfc\uacfc \ud658\uacbd\ub2e8\uccb4\uc758 \ubc18\ubc1c\uc744 \uc0c0\uace0 \ubc29\ub958\uae08\uc9c0 \uac00\ucc98\ubd84 \uc18c\uc1a1\uc73c\ub85c \uac80\ucc30 \uc218\uc0ac\uac00 \uc9c4\ud589\ub418\uc5c8\ub2e4. \uadf8 \ud6c4 \uacc4\uc18d\ub41c \ud574\uc218\uc720\ud1b5\uc73c\ub85c 2000\ub144\uc5d0 \ub4e4\uc5b4 \uc218\uc9c8\uc740 \ube60\ub974\uac8c \uac1c\uc120\ub418\uc5c8\uc9c0\ub9cc, 2001\ub144 \ud55c\uad6d\ud574\uc591\uc218\uc0b0\uac1c\ubc1c\uc6d0 \ubcf4\uace0\uc11c\uc5d0\uc11c \ub530\ub974\uba74 \ubc18\uc6d4\uacf5\ub2e8 \ud558\ub958\uc9c0\uc5ed\uc758 \ud1f4\uc801\ud1a0\ub294 \uc5ec\uc804\ud788 \uc911\uae08\uc18d \uc624\uc5fc\uc774 \uc2ec\uac01\ud55c \uac83\uc73c\ub85c \ub098\ud0c0\ub0ac\ub2e4. \uc774\uc5d0 \ud574\uc591\uc218\uc0b0\ubd80\ub294 \uc870\ub825\ubc1c\uc804\uc18c \uac74\uc124\uacfc \ud568\uaed8 \uc624\uc5fc \ud1f4\uc801\ud1a0 \uc900\uc124\uc0ac\uc5c5\ub3c4 \ubcd1\ud589\ud55c\ub2e4\ub294 \uc785\uc7a5\uc774\uc5c8\uc9c0\ub9cc, 2006\ub144 \uc218\uc790\uc6d0\uacf5\uc0ac \ub0b4\ubd80 \ubcf4\uace0\uc11c\uc5d0\uc11c\ub294 \uc5ec\uc804\ud788 \ud1f4\uc801\ud1a0\uc5d0\uc11c \uace0\ub18d\ub3c4 \uc911\uae08\uc18d\uc774 \uac80\ucd9c\ub41c \uac83\uc73c\ub85c \ud655\uc778\ub418\uc5c8\ub2e4. \ud1a0\uc591 13\uacf3 \uc911 12\uacf3\uc5d0\uc11c \ubbf8\uad6d \ud574\uc591\ub300\uae30\uccad \uae30\uc900\uc744 \ub118\ub294 \uad6c\ub9ac\uac00 \uac80\ucd9c\ub418\uc5c8\uace0, \ub0a9\uc774\ub098 \uc544\uc5f0\ub3c4 \ubbf8\uad6d \uae30\uc900\uc73c\ub85c \uc911\uac04\uc624\uc5fc\uc774\ub098 \uc2ec\ud55c\uc624\uc5fc \ub2e8\uacc4\uc600\ub2e4. \uc2dc\ud654\ud638 \uac2f\uc9c0\ub801\uc774 \uc5d0\uc11c\ub294 10\ubc30 \uc774\uc0c1 \ub9ce\uc740 \uc911\uae08\uc18d\uc774 \uac80\ucd9c\ub418\uc5c8\ub2e4. \uc774\ub7f0 \uc624\uc5fc\uc740 \uba39\uc774 \uc0ac\uc2ac\uc5d0 \ub530\ub77c \ub3d9\ubb3c\uc758 \ubab8\uc73c\ub85c \ucd95\uc801\ub418\uc5b4 \uba39\uac70\ub9ac\ub97c \uc704\ud611\ud558\ub294 \uacb0\uacfc\ub97c \ub0b3\ub294\ub2e4. \uadf8\ub7f0\ub370 \uc870\ub825\ubc1c\uc804\uc18c\uac00 \uac74\uc124\ub418\uba74 \uc774\ub4e4\uc774 \ubc14\ub2e4\ub85c \ud758\ub7ec\ub4e4\uc5b4\uac08 \uc218 \uc788\ub2e4\ub294 \uac83\uc774\ub2e4. \ub354\ubd88\uc5b4 \ub300\ud55c\ubbfc\uad6d\uc5d0\uc11c\ub294 \ud1f4\uc801\ud1a0\uc5d0 \ub300\ud55c \uc815\ud655\ud55c \uae30\uc900\uc774 \uc5c6\ub294 \uc0c1\ud669\uc774\uc5c8\uace0, \uc624\uc5fc\ub41c \ud1f4\uc801\ud1a0 \uc900\uc124\uc5d0 \ucd5c\uc18c 100\uc5b5\uc6d0, \ucd5c\ub300 400\uc5b5\uc6d0\uc774 \ub4e4 \uac83\uc73c\ub85c \ucd94\uc0b0\ub418\ub294 \uc0c1\ud669\uc774\uc5c8\ub2e4. \uc774\uc5d0 \uc218\uc790\uc6d0\uacf5\uc0ac \uad00\uacc4\uc790\ub294 \u201c\uc2dc\ud654\ud638 \uc870\ub825\ubc1c\uc804\uc18c\uac00 \uc720\uc5d4\uae30\ud6c4\ubcc0\ud654\ud611\uc57d\uc73c\ub85c\ubd80\ud130 \uccad\uc815\uac1c\ubc1c\uccb4\uc81c \uc2b9\uc778\uc744 \ubc1b\uc544 \uce5c\ud658\uacbd\uc801\u201d\uc774\ub77c\uba74\uc11c, \u201c\uc870\ub825\ubc1c\uc804\uc18c \ud130\ube48 \uac00\ub3d9\uc601\ud5a5\uad8c\uc5d0 \ub4dc\ub294 \uba74\uc801\uc774 \ubcc4\ub85c \ub113\uc9c0 \uc54a\uc544 \ubc14\ub2e4\ub85c \ube60\uc838\ub098\uac00\ub294 \ud1f4\uc801\ubb3c\uc774 \ud06c\uac8c \ub9ce\uc9c0 \uc54a\uc744 \uac83\uc774\ub2e4. \ud1f4\uc801\ubb3c\uc601\ud5a5 \uc6a9\uc5ed\uc870\uc0ac \uacb0\uacfc\ub97c \ubcf4\uace0 \ubb38\uc81c\uac00 \uc788\ub2e4\uba74 \uc870\uce58\ud560 \uac83\u201d\uc774\ub77c \ub2f5\ud558\uc600\ub2e4.", "sentence_answer": "\uc2dc\ud654\ud638 \uac2f\uc9c0\ub801\uc774 \uc5d0\uc11c\ub294 10\ubc30 \uc774\uc0c1 \ub9ce\uc740 \uc911\uae08\uc18d\uc774 \uac80\ucd9c\ub418\uc5c8\ub2e4.", "paragraph_id": "6547169-12-1", "paragraph_question": "question: \uc2dc\ud654\ud638\uc5d0 \uc0b4\uace0 \uc788\ub294 \uc5b4\ub5a4 \uc0dd\ubb3c\uc5d0\uc11c \uae30\uc900\uce58\uc758 10\ubc30 \uc774\uc0c1 \ub418\ub294 \uc911\uae08\uc18d\uc774 \uac80\ucd9c\ub418\uc5c8\ub098?, context: \uc2dc\ud654\ud638\uc758 \uc624\uc5fc \ubb3c\uc9c8 \ubc30\ucd9c \ub17c\ub780\uc740 1996\ub144 \uc81c1 \ubc30\uc218\uac11\ubb38 \uc2dc\ubc94 \ud574\uc218\uc720\ud1b5 \ub54c \ubd80\ud130 \uc2dc\uc791\ub418\uc5c8\ub2e4. \uc2dc\ubc94\uc720\ud1b5\uc774\ub77c\ub294 \uba85\ubaa9\uc73c\ub85c \uc624\uc5fc\ub41c \ud638\uc22b\ubb3c\uc744 \ubb34\ub2e8 \ubc29\ub958\ud558\uc5ec \uc778\uadfc \ud574\uc5ed\uc758 \uc624\uc5fc \ubb38\uc81c\uac00 \ub300\ub450\ub41c \uac83\uc774\ub2e4. \uc774\uc5d0 \uc778\uadfc \uc8fc\ubbfc\uacfc \ud658\uacbd\ub2e8\uccb4\uc758 \ubc18\ubc1c\uc744 \uc0c0\uace0 \ubc29\ub958\uae08\uc9c0 \uac00\ucc98\ubd84 \uc18c\uc1a1\uc73c\ub85c \uac80\ucc30 \uc218\uc0ac\uac00 \uc9c4\ud589\ub418\uc5c8\ub2e4. \uadf8 \ud6c4 \uacc4\uc18d\ub41c \ud574\uc218\uc720\ud1b5\uc73c\ub85c 2000\ub144\uc5d0 \ub4e4\uc5b4 \uc218\uc9c8\uc740 \ube60\ub974\uac8c \uac1c\uc120\ub418\uc5c8\uc9c0\ub9cc, 2001\ub144 \ud55c\uad6d\ud574\uc591\uc218\uc0b0\uac1c\ubc1c\uc6d0 \ubcf4\uace0\uc11c\uc5d0\uc11c \ub530\ub974\uba74 \ubc18\uc6d4\uacf5\ub2e8 \ud558\ub958\uc9c0\uc5ed\uc758 \ud1f4\uc801\ud1a0\ub294 \uc5ec\uc804\ud788 \uc911\uae08\uc18d \uc624\uc5fc\uc774 \uc2ec\uac01\ud55c \uac83\uc73c\ub85c \ub098\ud0c0\ub0ac\ub2e4. \uc774\uc5d0 \ud574\uc591\uc218\uc0b0\ubd80\ub294 \uc870\ub825\ubc1c\uc804\uc18c \uac74\uc124\uacfc \ud568\uaed8 \uc624\uc5fc \ud1f4\uc801\ud1a0 \uc900\uc124\uc0ac\uc5c5\ub3c4 \ubcd1\ud589\ud55c\ub2e4\ub294 \uc785\uc7a5\uc774\uc5c8\uc9c0\ub9cc, 2006\ub144 \uc218\uc790\uc6d0\uacf5\uc0ac \ub0b4\ubd80 \ubcf4\uace0\uc11c\uc5d0\uc11c\ub294 \uc5ec\uc804\ud788 \ud1f4\uc801\ud1a0\uc5d0\uc11c \uace0\ub18d\ub3c4 \uc911\uae08\uc18d\uc774 \uac80\ucd9c\ub41c \uac83\uc73c\ub85c \ud655\uc778\ub418\uc5c8\ub2e4. \ud1a0\uc591 13\uacf3 \uc911 12\uacf3\uc5d0\uc11c \ubbf8\uad6d \ud574\uc591\ub300\uae30\uccad \uae30\uc900\uc744 \ub118\ub294 \uad6c\ub9ac\uac00 \uac80\ucd9c\ub418\uc5c8\uace0, \ub0a9\uc774\ub098 \uc544\uc5f0\ub3c4 \ubbf8\uad6d \uae30\uc900\uc73c\ub85c \uc911\uac04\uc624\uc5fc\uc774\ub098 \uc2ec\ud55c\uc624\uc5fc \ub2e8\uacc4\uc600\ub2e4. \uc2dc\ud654\ud638 \uac2f\uc9c0\ub801\uc774\uc5d0\uc11c\ub294 10\ubc30 \uc774\uc0c1 \ub9ce\uc740 \uc911\uae08\uc18d\uc774 \uac80\ucd9c\ub418\uc5c8\ub2e4. \uc774\ub7f0 \uc624\uc5fc\uc740 \uba39\uc774 \uc0ac\uc2ac\uc5d0 \ub530\ub77c \ub3d9\ubb3c\uc758 \ubab8\uc73c\ub85c \ucd95\uc801\ub418\uc5b4 \uba39\uac70\ub9ac\ub97c \uc704\ud611\ud558\ub294 \uacb0\uacfc\ub97c \ub0b3\ub294\ub2e4. \uadf8\ub7f0\ub370 \uc870\ub825\ubc1c\uc804\uc18c\uac00 \uac74\uc124\ub418\uba74 \uc774\ub4e4\uc774 \ubc14\ub2e4\ub85c \ud758\ub7ec\ub4e4\uc5b4\uac08 \uc218 \uc788\ub2e4\ub294 \uac83\uc774\ub2e4. \ub354\ubd88\uc5b4 \ub300\ud55c\ubbfc\uad6d\uc5d0\uc11c\ub294 \ud1f4\uc801\ud1a0\uc5d0 \ub300\ud55c \uc815\ud655\ud55c \uae30\uc900\uc774 \uc5c6\ub294 \uc0c1\ud669\uc774\uc5c8\uace0, \uc624\uc5fc\ub41c \ud1f4\uc801\ud1a0 \uc900\uc124\uc5d0 \ucd5c\uc18c 100\uc5b5\uc6d0, \ucd5c\ub300 400\uc5b5\uc6d0\uc774 \ub4e4 \uac83\uc73c\ub85c \ucd94\uc0b0\ub418\ub294 \uc0c1\ud669\uc774\uc5c8\ub2e4. \uc774\uc5d0 \uc218\uc790\uc6d0\uacf5\uc0ac \uad00\uacc4\uc790\ub294 \u201c\uc2dc\ud654\ud638 \uc870\ub825\ubc1c\uc804\uc18c\uac00 \uc720\uc5d4\uae30\ud6c4\ubcc0\ud654\ud611\uc57d\uc73c\ub85c\ubd80\ud130 \uccad\uc815\uac1c\ubc1c\uccb4\uc81c \uc2b9\uc778\uc744 \ubc1b\uc544 \uce5c\ud658\uacbd\uc801\u201d\uc774\ub77c\uba74\uc11c, \u201c\uc870\ub825\ubc1c\uc804\uc18c \ud130\ube48 \uac00\ub3d9\uc601\ud5a5\uad8c\uc5d0 \ub4dc\ub294 \uba74\uc801\uc774 \ubcc4\ub85c \ub113\uc9c0 \uc54a\uc544 \ubc14\ub2e4\ub85c \ube60\uc838\ub098\uac00\ub294 \ud1f4\uc801\ubb3c\uc774 \ud06c\uac8c \ub9ce\uc9c0 \uc54a\uc744 \uac83\uc774\ub2e4. \ud1f4\uc801\ubb3c\uc601\ud5a5 \uc6a9\uc5ed\uc870\uc0ac \uacb0\uacfc\ub97c \ubcf4\uace0 \ubb38\uc81c\uac00 \uc788\ub2e4\uba74 \uc870\uce58\ud560 \uac83\u201d\uc774\ub77c \ub2f5\ud558\uc600\ub2e4."} {"question": "\uc18c\ub140\uc2dc\ub300\uac00 \ub370\ubdd4 \uc2f1\uae00\uc744 \ubc1c\ub9e4\ud55c \uac83\uc740 \uba87 \ub144\uc778\uac00?", "paragraph": "\uc18c\ub140\uc2dc\ub300(\u5c11\u5973\u6642\u4ee3, \uc601\uc5b4: Girls' Generation \ub610\ub294 SNSD)\ub294 \ub300\ud55c\ubbfc\uad6d\uc758 8\uc778\uc870 \uac78 \uadf8\ub8f9\uc774\ub2e4. \uc18c\ub140\ub4e4\uc774 \ud3c9\uc815\ud560 \uc2dc\ub300\uac00 \uc654\ub2e4\ub294 \uc758\ubbf8\ub97c \uac00\uc9c0\uace0 \uc788\ub2e4. \uc18c\ub140\uc2dc\ub300\ub294 2007\ub144 8\uc6d4 2\uc77c \ub370\ubdd4 \uc2f1\uae00 \u3008\ub2e4\uc2dc \ub9cc\ub09c \uc138\uacc4\u3009\ub97c \ubc1c\ub9e4\ud558\uace0 8\uc6d4 5\uc77c SBS \u300a\uc778\uae30\uac00\uc694\u300b\ub97c \ud1b5\ud574 \uc815\uc2dd \ub370\ubdd4\ud558\uc600\ub2e4. \uac19\uc740 \ud574 11\uc6d4 1\uc77c \ud55c\uad6d \uccab \uc815\uaddc \uc568\ubc94 \u300a\uc18c\ub140\uc2dc\ub300\u300b\ub97c \ubc1c\ub9e4\ud558\uc600\uc73c\uba70, \u3008Kissing You\u3009\uc640 \u3008Baby Baby\u3009 \ub4f1\uc73c\ub85c \ud65c\ub3d9\ud558\uba70 1\uc704\uc5d0 \uc62c\ub790\ub2e4. \uc774\ud6c4 2009\ub144 1\uc6d4 7\uc77c \uccab EP \uc568\ubc94 \u300aGee\u300b\ub97c \ubc1c\ub9e4\ud558\uc600\ub2e4. \uadf8\ud574 6\uc6d4 29\uc77c, \ub450 \ubc88\uc9f8 EP \uc568\ubc94 \u300a\uc18c\uc6d0\uc744 \ub9d0\ud574\ubd10\u300b\ub97c \ubc1c\ub9e4\ud574 10\ub9cc \uc7a5\uc744 \ub118\uac8c \ud314\uc558\ub2e4. 2009\ub144 12\uc6d4 29\uc77c\ubd80\ud130\ub294 Into The New World\ub77c\ub294 \uccab \uc544\uc2dc\uc544 \ud22c\uc5b4\ub3c4 \uac1c\ucd5c\ud558\uc600\ub2e4. 2010\ub144 1\uc6d4 28\uc77c\uc5d0\ub294 \ub450 \ubc88\uc9f8 \uc815\uaddc \uc568\ubc94 \u300aOh!\u300b\ub97c \ubc1c\ub9e4\ud574 20\ub9cc \uc7a5\uc744 \ud310\ub9e4\ud558\uba70 4\uc5f0\uc18d \uc568\ubc94 \ud310\ub9e4\ub7c9 10\ub9cc \uc7a5\uc744 \ub3cc\ud30c\ud558\uc600\ub2e4.", "answer": "2007\ub144", "sentence": "\uc18c\ub140\uc2dc\ub300\ub294 2007\ub144 8\uc6d4 2\uc77c \ub370\ubdd4 \uc2f1\uae00 \u3008\ub2e4\uc2dc \ub9cc\ub09c \uc138\uacc4\u3009\ub97c \ubc1c\ub9e4\ud558\uace0 8\uc6d4 5\uc77c SBS \u300a\uc778\uae30\uac00\uc694\u300b\ub97c \ud1b5\ud574 \uc815\uc2dd \ub370\ubdd4\ud558\uc600\ub2e4.", "paragraph_sentence": "\uc18c\ub140\uc2dc\ub300(\u5c11\u5973\u6642\u4ee3, \uc601\uc5b4: Girls' Generation \ub610\ub294 SNSD)\ub294 \ub300\ud55c\ubbfc\uad6d\uc758 8\uc778\uc870 \uac78 \uadf8\ub8f9\uc774\ub2e4. \uc18c\ub140\ub4e4\uc774 \ud3c9\uc815\ud560 \uc2dc\ub300\uac00 \uc654\ub2e4\ub294 \uc758\ubbf8\ub97c \uac00\uc9c0\uace0 \uc788\ub2e4. \uc18c\ub140\uc2dc\ub300\ub294 2007\ub144 8\uc6d4 2\uc77c \ub370\ubdd4 \uc2f1\uae00 \u3008\ub2e4\uc2dc \ub9cc\ub09c \uc138\uacc4\u3009\ub97c \ubc1c\ub9e4\ud558\uace0 8\uc6d4 5\uc77c SBS \u300a\uc778\uae30\uac00\uc694\u300b\ub97c \ud1b5\ud574 \uc815\uc2dd \ub370\ubdd4\ud558\uc600\ub2e4. \uac19\uc740 \ud574 11\uc6d4 1\uc77c \ud55c\uad6d \uccab \uc815\uaddc \uc568\ubc94 \u300a\uc18c\ub140\uc2dc\ub300\u300b\ub97c \ubc1c\ub9e4\ud558\uc600\uc73c\uba70, \u3008Kissing You\u3009\uc640 \u3008Baby Baby\u3009 \ub4f1\uc73c\ub85c \ud65c\ub3d9\ud558\uba70 1\uc704\uc5d0 \uc62c\ub790\ub2e4. \uc774\ud6c4 2009\ub144 1\uc6d4 7\uc77c \uccab EP \uc568\ubc94 \u300aGee\u300b\ub97c \ubc1c\ub9e4\ud558\uc600\ub2e4. \uadf8\ud574 6\uc6d4 29\uc77c, \ub450 \ubc88\uc9f8 EP \uc568\ubc94 \u300a\uc18c\uc6d0\uc744 \ub9d0\ud574\ubd10\u300b\ub97c \ubc1c\ub9e4\ud574 10\ub9cc \uc7a5\uc744 \ub118\uac8c \ud314\uc558\ub2e4. 2009\ub144 12\uc6d4 29\uc77c\ubd80\ud130\ub294 Into The New World\ub77c\ub294 \uccab \uc544\uc2dc\uc544 \ud22c\uc5b4\ub3c4 \uac1c\ucd5c\ud558\uc600\ub2e4. 2010\ub144 1\uc6d4 28\uc77c\uc5d0\ub294 \ub450 \ubc88\uc9f8 \uc815\uaddc \uc568\ubc94 \u300aOh!\u300b\ub97c \ubc1c\ub9e4\ud574 20\ub9cc \uc7a5\uc744 \ud310\ub9e4\ud558\uba70 4\uc5f0\uc18d \uc568\ubc94 \ud310\ub9e4\ub7c9 10\ub9cc \uc7a5\uc744 \ub3cc\ud30c\ud558\uc600\ub2e4.", "paragraph_answer": "\uc18c\ub140\uc2dc\ub300(\u5c11\u5973\u6642\u4ee3, \uc601\uc5b4: Girls' Generation \ub610\ub294 SNSD)\ub294 \ub300\ud55c\ubbfc\uad6d\uc758 8\uc778\uc870 \uac78 \uadf8\ub8f9\uc774\ub2e4. \uc18c\ub140\ub4e4\uc774 \ud3c9\uc815\ud560 \uc2dc\ub300\uac00 \uc654\ub2e4\ub294 \uc758\ubbf8\ub97c \uac00\uc9c0\uace0 \uc788\ub2e4. \uc18c\ub140\uc2dc\ub300\ub294 2007\ub144 8\uc6d4 2\uc77c \ub370\ubdd4 \uc2f1\uae00 \u3008\ub2e4\uc2dc \ub9cc\ub09c \uc138\uacc4\u3009\ub97c \ubc1c\ub9e4\ud558\uace0 8\uc6d4 5\uc77c SBS \u300a\uc778\uae30\uac00\uc694\u300b\ub97c \ud1b5\ud574 \uc815\uc2dd \ub370\ubdd4\ud558\uc600\ub2e4. \uac19\uc740 \ud574 11\uc6d4 1\uc77c \ud55c\uad6d \uccab \uc815\uaddc \uc568\ubc94 \u300a\uc18c\ub140\uc2dc\ub300\u300b\ub97c \ubc1c\ub9e4\ud558\uc600\uc73c\uba70, \u3008Kissing You\u3009\uc640 \u3008Baby Baby\u3009 \ub4f1\uc73c\ub85c \ud65c\ub3d9\ud558\uba70 1\uc704\uc5d0 \uc62c\ub790\ub2e4. \uc774\ud6c4 2009\ub144 1\uc6d4 7\uc77c \uccab EP \uc568\ubc94 \u300aGee\u300b\ub97c \ubc1c\ub9e4\ud558\uc600\ub2e4. \uadf8\ud574 6\uc6d4 29\uc77c, \ub450 \ubc88\uc9f8 EP \uc568\ubc94 \u300a\uc18c\uc6d0\uc744 \ub9d0\ud574\ubd10\u300b\ub97c \ubc1c\ub9e4\ud574 10\ub9cc \uc7a5\uc744 \ub118\uac8c \ud314\uc558\ub2e4. 2009\ub144 12\uc6d4 29\uc77c\ubd80\ud130\ub294 Into The New World\ub77c\ub294 \uccab \uc544\uc2dc\uc544 \ud22c\uc5b4\ub3c4 \uac1c\ucd5c\ud558\uc600\ub2e4. 2010\ub144 1\uc6d4 28\uc77c\uc5d0\ub294 \ub450 \ubc88\uc9f8 \uc815\uaddc \uc568\ubc94 \u300aOh!\u300b\ub97c \ubc1c\ub9e4\ud574 20\ub9cc \uc7a5\uc744 \ud310\ub9e4\ud558\uba70 4\uc5f0\uc18d \uc568\ubc94 \ud310\ub9e4\ub7c9 10\ub9cc \uc7a5\uc744 \ub3cc\ud30c\ud558\uc600\ub2e4.", "sentence_answer": "\uc18c\ub140\uc2dc\ub300\ub294 2007\ub144 8\uc6d4 2\uc77c \ub370\ubdd4 \uc2f1\uae00 \u3008\ub2e4\uc2dc \ub9cc\ub09c \uc138\uacc4\u3009\ub97c \ubc1c\ub9e4\ud558\uace0 8\uc6d4 5\uc77c SBS \u300a\uc778\uae30\uac00\uc694\u300b\ub97c \ud1b5\ud574 \uc815\uc2dd \ub370\ubdd4\ud558\uc600\ub2e4.", "paragraph_id": "6445662-0-1", "paragraph_question": "question: \uc18c\ub140\uc2dc\ub300\uac00 \ub370\ubdd4 \uc2f1\uae00\uc744 \ubc1c\ub9e4\ud55c \uac83\uc740 \uba87 \ub144\uc778\uac00?, context: \uc18c\ub140\uc2dc\ub300(\u5c11\u5973\u6642\u4ee3, \uc601\uc5b4: Girls' Generation \ub610\ub294 SNSD)\ub294 \ub300\ud55c\ubbfc\uad6d\uc758 8\uc778\uc870 \uac78 \uadf8\ub8f9\uc774\ub2e4. \uc18c\ub140\ub4e4\uc774 \ud3c9\uc815\ud560 \uc2dc\ub300\uac00 \uc654\ub2e4\ub294 \uc758\ubbf8\ub97c \uac00\uc9c0\uace0 \uc788\ub2e4. \uc18c\ub140\uc2dc\ub300\ub294 2007\ub144 8\uc6d4 2\uc77c \ub370\ubdd4 \uc2f1\uae00 \u3008\ub2e4\uc2dc \ub9cc\ub09c \uc138\uacc4\u3009\ub97c \ubc1c\ub9e4\ud558\uace0 8\uc6d4 5\uc77c SBS \u300a\uc778\uae30\uac00\uc694\u300b\ub97c \ud1b5\ud574 \uc815\uc2dd \ub370\ubdd4\ud558\uc600\ub2e4. \uac19\uc740 \ud574 11\uc6d4 1\uc77c \ud55c\uad6d \uccab \uc815\uaddc \uc568\ubc94 \u300a\uc18c\ub140\uc2dc\ub300\u300b\ub97c \ubc1c\ub9e4\ud558\uc600\uc73c\uba70, \u3008Kissing You\u3009\uc640 \u3008Baby Baby\u3009 \ub4f1\uc73c\ub85c \ud65c\ub3d9\ud558\uba70 1\uc704\uc5d0 \uc62c\ub790\ub2e4. \uc774\ud6c4 2009\ub144 1\uc6d4 7\uc77c \uccab EP \uc568\ubc94 \u300aGee\u300b\ub97c \ubc1c\ub9e4\ud558\uc600\ub2e4. \uadf8\ud574 6\uc6d4 29\uc77c, \ub450 \ubc88\uc9f8 EP \uc568\ubc94 \u300a\uc18c\uc6d0\uc744 \ub9d0\ud574\ubd10\u300b\ub97c \ubc1c\ub9e4\ud574 10\ub9cc \uc7a5\uc744 \ub118\uac8c \ud314\uc558\ub2e4. 2009\ub144 12\uc6d4 29\uc77c\ubd80\ud130\ub294 Into The New World\ub77c\ub294 \uccab \uc544\uc2dc\uc544 \ud22c\uc5b4\ub3c4 \uac1c\ucd5c\ud558\uc600\ub2e4. 2010\ub144 1\uc6d4 28\uc77c\uc5d0\ub294 \ub450 \ubc88\uc9f8 \uc815\uaddc \uc568\ubc94 \u300aOh!\u300b\ub97c \ubc1c\ub9e4\ud574 20\ub9cc \uc7a5\uc744 \ud310\ub9e4\ud558\uba70 4\uc5f0\uc18d \uc568\ubc94 \ud310\ub9e4\ub7c9 10\ub9cc \uc7a5\uc744 \ub3cc\ud30c\ud558\uc600\ub2e4."} {"question": "1909\ub144 \uc548\ucc3d\ud638\uac00 \uae40\uc88c\uc9c4, \uc774\uac11\uacfc \ud568\uaed8 \ucc3d\ub9bd\ud55c \ud559\ud68c\ub294?", "paragraph": "1905\ub144(\uad11\ubb34 8\ub144) 11\uc6d4 17\uc77c \ub300\ud55c\uc81c\uad6d\uc758 \ubc15\uc81c\uc21c\uacfc \uc77c\ubcf8 \uc815\ubd80\uc758 \ud558\uc57c\uc2dc \uace4\uc2a4\ucf00\uc5d0 \uc758\ud574 \uccb4\uacb0\ub41c \uc744\uc0ac\uc870\uc57d\uc73c\ub85c \ub300\ud55c\uc81c\uad6d\uc758 \uc678\uad50\uad8c\uc744 \uc77c\ubcf8 \uc81c\uad6d\uc774 \ube7c\uc557\uc790, \uadf8\ub294 \uc870\uc57d \ubc18\ub300 \ud56d\uc758 \uc131\uba85\uc11c\ub97c \ubc1c\ud45c\ud558\uace0, \uc724\uce58\ud638, \uc11c\uc7ac\ud544 \ub4f1\uacfc \ud568\uaed8 \uc744\uc0ac \ubcf4\ud638 \uc870\uc57d\uc5d0 \ubc18\ub300\ud558\ub294 \uc6b4\ub3d9\uc744 \uc8fc\ub3c4\ud558\uc600\ub2e4. 1907\ub144 \uc548\ucc3d\ud638\ub294 \uadc0\uad6d\ud558\uc5ec \ub300\ud55c \uc0ac\ub78c \uc740 \uc2e4\ub825\uc744 \uae38\ub7ec\uc57c \ud55c\ub2e4\uace0 \uc5ed\uc124\ud558\uc600\ub2e4. \uc560\uad6d \uacc4\ubabd \uc6b4\ub3d9 \uc678\uc5d0\ub3c4 1904\ub144 \uc758\uce5c\uc655, \uc815\uc7ac\uad00(\u912d\u5728\u5bec) \ub4f1\uacfc\uc758 \uacf5\ub9bd\ud611\ud68c(\u5171\u7acb\u5354\u6703) \ucc3d\ub9bd, 1909\ub144 \uae40\uc88c\uc9c4\u00b7\uc774\uac11(\u674e\u7532) \ub4f1\uacfc\uc758 \uc11c\ubd81\ud559\ud68c \ucc3d\ub9bd\uc744 \ud558\uc600\uace0 \ub610\ud55c 1907\ub144 \ubbf8\uad6d\uc5d0\uc11c \uadc0\uad6d\ud55c \uc774\ud6c4 \uc2e0\ubbfc\ud68c, \ub300\ud55c\ud611\ud68c \ub300\uc131\ud559\uad50 \uc124\ub9bd \ub4f1\uc744 \uc704\ud574 \uc11c\uc6b8\uacfc \ud3c9\uc591 \ub4f1\uc5d0\uc11c \uc5ec\ub7ec \ucc28\ub840 \uc5f0\uc124\uc744 \ud588\ub2e4. \ub3c4\uc0b0\uc740 \ud0c1\uc6d4\ud55c \uc6c5\ubcc0\ub825\uc73c\ub85c \uc218\ucc9c \uc778\ud30c\uc758 \ub9c8\uc74c\uc744 \uc0ac\ub85c\uc7a1\uace4 \ud558\uc600\ub2e4. \uc744\uc0ac \ubcf4\ud638 \uc870\uc57d \uc774\ud6c4 \uadf8\ub294 \uc2e4\ub825 \uc591\uc131\uc744 \ud1b5\ud574 \ub3c5\ub9bd\uc744 \ub2ec\uc131\ud574\uc57c \ud55c\ub2e4\ub294 \uc720\uae38\uc900\uc758 \uacc4\ubabd \uac15\uc5f0 \ud65c\ub3d9\uc5d0 \uac10\uba85\ubc1b\uc544 \uae4a\uc774 \uacf5\uac10\ud558\uac8c \ub41c\ub2e4. \uc774\ud6c4 \uadf8\ub294 \uc2e4\ub825\uc591\uc131\uc744 \uc704\ud574\uc11c\ub294 \uad50\uc721\uc774 \ud544\uc694\ud558\uba70, \ubbfc\uc911\ub4e4\uc758 \uc758\uc2dd\uac1c\uc120\uc774 \uc120\ud589\ub418\uc5b4\uc57c \ud55c\ub2e4\uace0 \ubcf4\uc558\ub2e4.", "answer": "\uc11c\ubd81\ud559\ud68c", "sentence": "\uc560\uad6d \uacc4\ubabd \uc6b4\ub3d9 \uc678\uc5d0\ub3c4 1904\ub144 \uc758\uce5c\uc655, \uc815\uc7ac\uad00(\u912d\u5728\u5bec) \ub4f1\uacfc\uc758 \uacf5\ub9bd\ud611\ud68c(\u5171\u7acb\u5354\u6703) \ucc3d\ub9bd, 1909\ub144 \uae40\uc88c\uc9c4\u00b7\uc774\uac11(\u674e\u7532) \ub4f1\uacfc\uc758 \uc11c\ubd81\ud559\ud68c \ucc3d\ub9bd\uc744 \ud558\uc600\uace0 \ub610\ud55c 1907\ub144 \ubbf8\uad6d\uc5d0\uc11c \uadc0\uad6d\ud55c \uc774\ud6c4 \uc2e0\ubbfc\ud68c, \ub300\ud55c\ud611\ud68c \ub300\uc131\ud559\uad50 \uc124\ub9bd \ub4f1\uc744 \uc704\ud574 \uc11c\uc6b8\uacfc \ud3c9\uc591 \ub4f1\uc5d0\uc11c \uc5ec\ub7ec \ucc28\ub840 \uc5f0\uc124\uc744 \ud588\ub2e4.", "paragraph_sentence": "1905\ub144(\uad11\ubb34 8\ub144) 11\uc6d4 17\uc77c \ub300\ud55c\uc81c\uad6d\uc758 \ubc15\uc81c\uc21c\uacfc \uc77c\ubcf8 \uc815\ubd80\uc758 \ud558\uc57c\uc2dc \uace4\uc2a4\ucf00\uc5d0 \uc758\ud574 \uccb4\uacb0\ub41c \uc744\uc0ac\uc870\uc57d\uc73c\ub85c \ub300\ud55c\uc81c\uad6d\uc758 \uc678\uad50\uad8c\uc744 \uc77c\ubcf8 \uc81c\uad6d\uc774 \ube7c\uc557\uc790, \uadf8\ub294 \uc870\uc57d \ubc18\ub300 \ud56d\uc758 \uc131\uba85\uc11c\ub97c \ubc1c\ud45c\ud558\uace0, \uc724\uce58\ud638, \uc11c\uc7ac\ud544 \ub4f1\uacfc \ud568\uaed8 \uc744\uc0ac \ubcf4\ud638 \uc870\uc57d\uc5d0 \ubc18\ub300\ud558\ub294 \uc6b4\ub3d9\uc744 \uc8fc\ub3c4\ud558\uc600\ub2e4. 1907\ub144 \uc548\ucc3d\ud638\ub294 \uadc0\uad6d\ud558\uc5ec \ub300\ud55c \uc0ac\ub78c \uc740 \uc2e4\ub825\uc744 \uae38\ub7ec\uc57c \ud55c\ub2e4\uace0 \uc5ed\uc124\ud558\uc600\ub2e4. \uc560\uad6d \uacc4\ubabd \uc6b4\ub3d9 \uc678\uc5d0\ub3c4 1904\ub144 \uc758\uce5c\uc655, \uc815\uc7ac\uad00(\u912d\u5728\u5bec) \ub4f1\uacfc\uc758 \uacf5\ub9bd\ud611\ud68c(\u5171\u7acb\u5354\u6703) \ucc3d\ub9bd, 1909\ub144 \uae40\uc88c\uc9c4\u00b7\uc774\uac11(\u674e\u7532) \ub4f1\uacfc\uc758 \uc11c\ubd81\ud559\ud68c \ucc3d\ub9bd\uc744 \ud558\uc600\uace0 \ub610\ud55c 1907\ub144 \ubbf8\uad6d\uc5d0\uc11c \uadc0\uad6d\ud55c \uc774\ud6c4 \uc2e0\ubbfc\ud68c, \ub300\ud55c\ud611\ud68c \ub300\uc131\ud559\uad50 \uc124\ub9bd \ub4f1\uc744 \uc704\ud574 \uc11c\uc6b8\uacfc \ud3c9\uc591 \ub4f1\uc5d0\uc11c \uc5ec\ub7ec \ucc28\ub840 \uc5f0\uc124\uc744 \ud588\ub2e4. \ub3c4\uc0b0\uc740 \ud0c1\uc6d4\ud55c \uc6c5\ubcc0\ub825\uc73c\ub85c \uc218\ucc9c \uc778\ud30c\uc758 \ub9c8\uc74c\uc744 \uc0ac\ub85c\uc7a1\uace4 \ud558\uc600\ub2e4. \uc744\uc0ac \ubcf4\ud638 \uc870\uc57d \uc774\ud6c4 \uadf8\ub294 \uc2e4\ub825 \uc591\uc131\uc744 \ud1b5\ud574 \ub3c5\ub9bd\uc744 \ub2ec\uc131\ud574\uc57c \ud55c\ub2e4\ub294 \uc720\uae38\uc900\uc758 \uacc4\ubabd \uac15\uc5f0 \ud65c\ub3d9\uc5d0 \uac10\uba85\ubc1b\uc544 \uae4a\uc774 \uacf5\uac10\ud558\uac8c \ub41c\ub2e4. \uc774\ud6c4 \uadf8\ub294 \uc2e4\ub825\uc591\uc131\uc744 \uc704\ud574\uc11c\ub294 \uad50\uc721\uc774 \ud544\uc694\ud558\uba70, \ubbfc\uc911\ub4e4\uc758 \uc758\uc2dd\uac1c\uc120\uc774 \uc120\ud589\ub418\uc5b4\uc57c \ud55c\ub2e4\uace0 \ubcf4\uc558\ub2e4.", "paragraph_answer": "1905\ub144(\uad11\ubb34 8\ub144) 11\uc6d4 17\uc77c \ub300\ud55c\uc81c\uad6d\uc758 \ubc15\uc81c\uc21c\uacfc \uc77c\ubcf8 \uc815\ubd80\uc758 \ud558\uc57c\uc2dc \uace4\uc2a4\ucf00\uc5d0 \uc758\ud574 \uccb4\uacb0\ub41c \uc744\uc0ac\uc870\uc57d\uc73c\ub85c \ub300\ud55c\uc81c\uad6d\uc758 \uc678\uad50\uad8c\uc744 \uc77c\ubcf8 \uc81c\uad6d\uc774 \ube7c\uc557\uc790, \uadf8\ub294 \uc870\uc57d \ubc18\ub300 \ud56d\uc758 \uc131\uba85\uc11c\ub97c \ubc1c\ud45c\ud558\uace0, \uc724\uce58\ud638, \uc11c\uc7ac\ud544 \ub4f1\uacfc \ud568\uaed8 \uc744\uc0ac \ubcf4\ud638 \uc870\uc57d\uc5d0 \ubc18\ub300\ud558\ub294 \uc6b4\ub3d9\uc744 \uc8fc\ub3c4\ud558\uc600\ub2e4. 1907\ub144 \uc548\ucc3d\ud638\ub294 \uadc0\uad6d\ud558\uc5ec \ub300\ud55c \uc0ac\ub78c \uc740 \uc2e4\ub825\uc744 \uae38\ub7ec\uc57c \ud55c\ub2e4\uace0 \uc5ed\uc124\ud558\uc600\ub2e4. \uc560\uad6d \uacc4\ubabd \uc6b4\ub3d9 \uc678\uc5d0\ub3c4 1904\ub144 \uc758\uce5c\uc655, \uc815\uc7ac\uad00(\u912d\u5728\u5bec) \ub4f1\uacfc\uc758 \uacf5\ub9bd\ud611\ud68c(\u5171\u7acb\u5354\u6703) \ucc3d\ub9bd, 1909\ub144 \uae40\uc88c\uc9c4\u00b7\uc774\uac11(\u674e\u7532) \ub4f1\uacfc\uc758 \uc11c\ubd81\ud559\ud68c \ucc3d\ub9bd\uc744 \ud558\uc600\uace0 \ub610\ud55c 1907\ub144 \ubbf8\uad6d\uc5d0\uc11c \uadc0\uad6d\ud55c \uc774\ud6c4 \uc2e0\ubbfc\ud68c, \ub300\ud55c\ud611\ud68c \ub300\uc131\ud559\uad50 \uc124\ub9bd \ub4f1\uc744 \uc704\ud574 \uc11c\uc6b8\uacfc \ud3c9\uc591 \ub4f1\uc5d0\uc11c \uc5ec\ub7ec \ucc28\ub840 \uc5f0\uc124\uc744 \ud588\ub2e4. \ub3c4\uc0b0\uc740 \ud0c1\uc6d4\ud55c \uc6c5\ubcc0\ub825\uc73c\ub85c \uc218\ucc9c \uc778\ud30c\uc758 \ub9c8\uc74c\uc744 \uc0ac\ub85c\uc7a1\uace4 \ud558\uc600\ub2e4. \uc744\uc0ac \ubcf4\ud638 \uc870\uc57d \uc774\ud6c4 \uadf8\ub294 \uc2e4\ub825 \uc591\uc131\uc744 \ud1b5\ud574 \ub3c5\ub9bd\uc744 \ub2ec\uc131\ud574\uc57c \ud55c\ub2e4\ub294 \uc720\uae38\uc900\uc758 \uacc4\ubabd \uac15\uc5f0 \ud65c\ub3d9\uc5d0 \uac10\uba85\ubc1b\uc544 \uae4a\uc774 \uacf5\uac10\ud558\uac8c \ub41c\ub2e4. \uc774\ud6c4 \uadf8\ub294 \uc2e4\ub825\uc591\uc131\uc744 \uc704\ud574\uc11c\ub294 \uad50\uc721\uc774 \ud544\uc694\ud558\uba70, \ubbfc\uc911\ub4e4\uc758 \uc758\uc2dd\uac1c\uc120\uc774 \uc120\ud589\ub418\uc5b4\uc57c \ud55c\ub2e4\uace0 \ubcf4\uc558\ub2e4.", "sentence_answer": "\uc560\uad6d \uacc4\ubabd \uc6b4\ub3d9 \uc678\uc5d0\ub3c4 1904\ub144 \uc758\uce5c\uc655, \uc815\uc7ac\uad00(\u912d\u5728\u5bec) \ub4f1\uacfc\uc758 \uacf5\ub9bd\ud611\ud68c(\u5171\u7acb\u5354\u6703) \ucc3d\ub9bd, 1909\ub144 \uae40\uc88c\uc9c4\u00b7\uc774\uac11(\u674e\u7532) \ub4f1\uacfc\uc758 \uc11c\ubd81\ud559\ud68c \ucc3d\ub9bd\uc744 \ud558\uc600\uace0 \ub610\ud55c 1907\ub144 \ubbf8\uad6d\uc5d0\uc11c \uadc0\uad6d\ud55c \uc774\ud6c4 \uc2e0\ubbfc\ud68c, \ub300\ud55c\ud611\ud68c \ub300\uc131\ud559\uad50 \uc124\ub9bd \ub4f1\uc744 \uc704\ud574 \uc11c\uc6b8\uacfc \ud3c9\uc591 \ub4f1\uc5d0\uc11c \uc5ec\ub7ec \ucc28\ub840 \uc5f0\uc124\uc744 \ud588\ub2e4.", "paragraph_id": "6538990-2-1", "paragraph_question": "question: 1909\ub144 \uc548\ucc3d\ud638\uac00 \uae40\uc88c\uc9c4, \uc774\uac11\uacfc \ud568\uaed8 \ucc3d\ub9bd\ud55c \ud559\ud68c\ub294?, context: 1905\ub144(\uad11\ubb34 8\ub144) 11\uc6d4 17\uc77c \ub300\ud55c\uc81c\uad6d\uc758 \ubc15\uc81c\uc21c\uacfc \uc77c\ubcf8 \uc815\ubd80\uc758 \ud558\uc57c\uc2dc \uace4\uc2a4\ucf00\uc5d0 \uc758\ud574 \uccb4\uacb0\ub41c \uc744\uc0ac\uc870\uc57d\uc73c\ub85c \ub300\ud55c\uc81c\uad6d\uc758 \uc678\uad50\uad8c\uc744 \uc77c\ubcf8 \uc81c\uad6d\uc774 \ube7c\uc557\uc790, \uadf8\ub294 \uc870\uc57d \ubc18\ub300 \ud56d\uc758 \uc131\uba85\uc11c\ub97c \ubc1c\ud45c\ud558\uace0, \uc724\uce58\ud638, \uc11c\uc7ac\ud544 \ub4f1\uacfc \ud568\uaed8 \uc744\uc0ac \ubcf4\ud638 \uc870\uc57d\uc5d0 \ubc18\ub300\ud558\ub294 \uc6b4\ub3d9\uc744 \uc8fc\ub3c4\ud558\uc600\ub2e4. 1907\ub144 \uc548\ucc3d\ud638\ub294 \uadc0\uad6d\ud558\uc5ec \ub300\ud55c \uc0ac\ub78c \uc740 \uc2e4\ub825\uc744 \uae38\ub7ec\uc57c \ud55c\ub2e4\uace0 \uc5ed\uc124\ud558\uc600\ub2e4. \uc560\uad6d \uacc4\ubabd \uc6b4\ub3d9 \uc678\uc5d0\ub3c4 1904\ub144 \uc758\uce5c\uc655, \uc815\uc7ac\uad00(\u912d\u5728\u5bec) \ub4f1\uacfc\uc758 \uacf5\ub9bd\ud611\ud68c(\u5171\u7acb\u5354\u6703) \ucc3d\ub9bd, 1909\ub144 \uae40\uc88c\uc9c4\u00b7\uc774\uac11(\u674e\u7532) \ub4f1\uacfc\uc758 \uc11c\ubd81\ud559\ud68c \ucc3d\ub9bd\uc744 \ud558\uc600\uace0 \ub610\ud55c 1907\ub144 \ubbf8\uad6d\uc5d0\uc11c \uadc0\uad6d\ud55c \uc774\ud6c4 \uc2e0\ubbfc\ud68c, \ub300\ud55c\ud611\ud68c \ub300\uc131\ud559\uad50 \uc124\ub9bd \ub4f1\uc744 \uc704\ud574 \uc11c\uc6b8\uacfc \ud3c9\uc591 \ub4f1\uc5d0\uc11c \uc5ec\ub7ec \ucc28\ub840 \uc5f0\uc124\uc744 \ud588\ub2e4. \ub3c4\uc0b0\uc740 \ud0c1\uc6d4\ud55c \uc6c5\ubcc0\ub825\uc73c\ub85c \uc218\ucc9c \uc778\ud30c\uc758 \ub9c8\uc74c\uc744 \uc0ac\ub85c\uc7a1\uace4 \ud558\uc600\ub2e4. \uc744\uc0ac \ubcf4\ud638 \uc870\uc57d \uc774\ud6c4 \uadf8\ub294 \uc2e4\ub825 \uc591\uc131\uc744 \ud1b5\ud574 \ub3c5\ub9bd\uc744 \ub2ec\uc131\ud574\uc57c \ud55c\ub2e4\ub294 \uc720\uae38\uc900\uc758 \uacc4\ubabd \uac15\uc5f0 \ud65c\ub3d9\uc5d0 \uac10\uba85\ubc1b\uc544 \uae4a\uc774 \uacf5\uac10\ud558\uac8c \ub41c\ub2e4. \uc774\ud6c4 \uadf8\ub294 \uc2e4\ub825\uc591\uc131\uc744 \uc704\ud574\uc11c\ub294 \uad50\uc721\uc774 \ud544\uc694\ud558\uba70, \ubbfc\uc911\ub4e4\uc758 \uc758\uc2dd\uac1c\uc120\uc774 \uc120\ud589\ub418\uc5b4\uc57c \ud55c\ub2e4\uace0 \ubcf4\uc558\ub2e4."} {"question": "1988\ub144 \uc11c\uc6b8\uc62c\ub9bc\ud53d \ud3d0\ub9c9 \ub54c \uc804\ud30c\ub41c \uc870\uc6a9\ud544\uc758 \ub178\ub798\ub294?", "paragraph": "\uc870\uc6a9\ud544\uc740 \ub77d \uc74c\uc545\uacfc \ubc1c\ub77c\ub4dc \uc74c\uc545, \ud2b8\ub85c\ud2b8 \uc74c\uc545 \ub610\ub294 \u3008\uac15\uc6d0\ub3c4 \uc544\ub9ac\ub791\u3009\ucc98\ub7fc \ud55c\uad6d \ubbfc\uc694\ub97c \ub9ac\uba54\uc774\ud06c\ud558\ub294 \ub4f1 \ub300\ud55c\ubbfc\uad6d\uc758 \ub300\uc911\uc74c\uc545 \uac70\uc758 \ubaa8\ub4e0 \uc7a5\ub974\ub97c \uc18c\ud654\ud574 \ub0b4\uba70 \ub2e4\uc591\ud55c \uc5f0\ub839\uce35\uc744 \ud655\ubcf4\ud558\uc600\ub2e4. \ub610\ud55c \uc870\uc6a9\ud544\uc740 \ub300\ud55c\ubbfc\uad6d \uac00\uc694\uacc4\uc5d0\uc11c \uc624\ube60\ubd80\ub300\ub85c \ubd88\ub9ac\ub294 \uc80a\uc740 \uc5ec\uc131 \ud32c\uce35\uc744 \uc774\ub04c\uace0 \ub2e4\ub2c8\uae30\ub3c4 \ud588\uc73c\uba70 \uc870\uc6a9\ud544 \ub4f1\uc7a5 \uc774\uc804\uc5d0 \ub300\ud55c\ubbfc\uad6d\uc758 \ucd5c\uace0 \uc778\uae30 \uac00\uc218\ub85c \ud638\ud3c9\uc744 \ubc1b\ub358 \ub0a8\uc9c4\uacfc \ub098\ud6c8\uc544\uc758 \ub4a4\ub97c \uc774\uc5b4 \ub300\ud55c\ubbfc\uad6d \uac00\uc694\uacc4\ub97c \uc8fc\ub984\uc7a1\uc558\ub2e4. \ud558\uc9c0\ub9cc 1985\ub144 \uc870\ud3ed \ucd9c\uc2e0 \uae40 \ubaa8\uc528\ub85c\ubd80\ud130 \ud3ed\ud589\uc5d0 \ud718\ub9d0\ub9ac\uae30\uae4c\uc9c0 \ud588\ub2e4. \uacbd\ucc30\uc740 \uc870\uc528\uc5d0 \ub300\ud55c \ud3ed\ud589\uc0ac\uac74\uc5d0\ub300\ud574 \uc218\uc0ac\ub97c \ud3bc\uce58\uae30\ub85c \ud588\uc73c\uba70 \ubcf4\ub514\uac00\ub4dc\ub97c 5\uba85\uc73c\ub85c \ub298\ub9ac\uae30\ub85c \ud558\uc600\ub2e4. \ud640\ub85c \ub300\ud55c\ubbfc\uad6d \uac00\uc694\uc0ac\uc758 \ud55c \ud68d\uc744 \uae0b\uae30 \uc2dc\uc791\ud558\ub358 \uc870\uc6a9\ud544\uc740 1986\ub144\uc5d0 \uc77c\ubcf8\uc5d0 \uc9c4\ucd9c\ud558\uc5ec \ubc1c\ub9e4\ud55c \uc568\ubc94 \u3008\ucd94\uc5b5\uc758 \ubbf8\uc544 1\u3009\uc774 100\ub9cc\uc7a5\uc774\uc0c1 \ud310\ub9e4\ud558\ub294 \ub300\uc131\uacfc\ub97c \uac70\ub450\uc5b4 \uadf8 \ud574 \uace8\ub4e0\ub514\uc2a4\ud06c\uc0c1\uc744 \uc218\uc0c1\ud558\uc600\ub2e4. \ub300\ud55c\ubbfc\uad6d\uc5d0\uc11c\ub294 \ubaa8\ub4e0 \uc7a5\ub974\ub97c \uc12d\ub835\ud55c \uac00\uc218\ub85c \ud1b5\ud558\uc9c0\ub9cc \uc77c\ubcf8\uc5d0\uc11c\ub294 \uc870\uc6a9\ud544\uc744 \uc5d4\uce74\uc758 \ud669\uc81c\ub85c \ubd80\ub978\ub2e4. 1988\ub144\uc5d0\ub294 10\uc9d1 \uc74c\ubc18\uc744 \ubc1c\ud45c\ud588\ub294\ub370 \uadf8 \uc74c\ubc18\uc758 \uc218\ub85d\uace1 \u3008\uc11c\uc6b8 \uc11c\uc6b8 \uc11c\uc6b8\u3009,\u3008\ubaa8\ub098\ub9ac\uc790\u3009\uac00 \ub300\ubc15\uc744 \ud130\ub728\ub9ac\uba74\uc11c \uc5ec\ub7ec \uac00\uc218\uc640 \uacbd\uc7c1\uc744 \ud558\uc600\ub2e4. \uc774 \uc911 \u3008\uc11c\uc6b8 \uc11c\uc6b8 \uc11c\uc6b8\u3009\uc740 1988\ub144 \uc11c\uc6b8\uc62c\ub9bc\ud53d \ud3d0\ub9c9 \ub54c \ubc29\uc1a1\uc5d0\uc11c \uc804\ud30c\ub418\uba74\uc11c \ubd80\ud130 \uc778\uae30\ub97c \uc5bb\uc5c8\ub294\ub370 \uc870\uc6a9\ud544\uc740 \ub2f9\uc2dc \uc778\ud130\ubdf0 \ub54c \uc62c\ub9bc\ud53d\uc774 \ub05d\ub098\uba74 \uc6b0\ub9ac \uc0ac\ud68c\uac00 \uc6b0\uc6b8\ud574 \uc9c8 \uac83\uc774\ub77c\uace0 \uc0dd\uac01\ud574\uc11c \ub9cc\ub4e4\uac8c \ub418\uc5c8\ub2e4\uace0 \ubc1d\ud614\ub2e4.", "answer": "\uc11c\uc6b8 \uc11c\uc6b8 \uc11c\uc6b8", "sentence": "1988\ub144\uc5d0\ub294 10\uc9d1 \uc74c\ubc18\uc744 \ubc1c\ud45c\ud588\ub294\ub370 \uadf8 \uc74c\ubc18\uc758 \uc218\ub85d\uace1 \u3008 \uc11c\uc6b8 \uc11c\uc6b8 \uc11c\uc6b8 \u3009,\u3008\ubaa8\ub098\ub9ac\uc790\u3009\uac00 \ub300\ubc15\uc744 \ud130\ub728\ub9ac\uba74\uc11c \uc5ec\ub7ec \uac00\uc218\uc640 \uacbd\uc7c1\uc744 \ud558\uc600\ub2e4.", "paragraph_sentence": "\uc870\uc6a9\ud544\uc740 \ub77d \uc74c\uc545\uacfc \ubc1c\ub77c\ub4dc \uc74c\uc545, \ud2b8\ub85c\ud2b8 \uc74c\uc545 \ub610\ub294 \u3008\uac15\uc6d0\ub3c4 \uc544\ub9ac\ub791\u3009\ucc98\ub7fc \ud55c\uad6d \ubbfc\uc694\ub97c \ub9ac\uba54\uc774\ud06c\ud558\ub294 \ub4f1 \ub300\ud55c\ubbfc\uad6d\uc758 \ub300\uc911\uc74c\uc545 \uac70\uc758 \ubaa8\ub4e0 \uc7a5\ub974\ub97c \uc18c\ud654\ud574 \ub0b4\uba70 \ub2e4\uc591\ud55c \uc5f0\ub839\uce35\uc744 \ud655\ubcf4\ud558\uc600\ub2e4. \ub610\ud55c \uc870\uc6a9\ud544\uc740 \ub300\ud55c\ubbfc\uad6d \uac00\uc694\uacc4\uc5d0\uc11c \uc624\ube60\ubd80\ub300\ub85c \ubd88\ub9ac\ub294 \uc80a\uc740 \uc5ec\uc131 \ud32c\uce35\uc744 \uc774\ub04c\uace0 \ub2e4\ub2c8\uae30\ub3c4 \ud588\uc73c\uba70 \uc870\uc6a9\ud544 \ub4f1\uc7a5 \uc774\uc804\uc5d0 \ub300\ud55c\ubbfc\uad6d\uc758 \ucd5c\uace0 \uc778\uae30 \uac00\uc218\ub85c \ud638\ud3c9\uc744 \ubc1b\ub358 \ub0a8\uc9c4\uacfc \ub098\ud6c8\uc544\uc758 \ub4a4\ub97c \uc774\uc5b4 \ub300\ud55c\ubbfc\uad6d \uac00\uc694\uacc4\ub97c \uc8fc\ub984\uc7a1\uc558\ub2e4. \ud558\uc9c0\ub9cc 1985\ub144 \uc870\ud3ed \ucd9c\uc2e0 \uae40 \ubaa8\uc528\ub85c\ubd80\ud130 \ud3ed\ud589\uc5d0 \ud718\ub9d0\ub9ac\uae30\uae4c\uc9c0 \ud588\ub2e4. \uacbd\ucc30\uc740 \uc870\uc528\uc5d0 \ub300\ud55c \ud3ed\ud589\uc0ac\uac74\uc5d0\ub300\ud574 \uc218\uc0ac\ub97c \ud3bc\uce58\uae30\ub85c \ud588\uc73c\uba70 \ubcf4\ub514\uac00\ub4dc\ub97c 5\uba85\uc73c\ub85c \ub298\ub9ac\uae30\ub85c \ud558\uc600\ub2e4. \ud640\ub85c \ub300\ud55c\ubbfc\uad6d \uac00\uc694\uc0ac\uc758 \ud55c \ud68d\uc744 \uae0b\uae30 \uc2dc\uc791\ud558\ub358 \uc870\uc6a9\ud544\uc740 1986\ub144\uc5d0 \uc77c\ubcf8\uc5d0 \uc9c4\ucd9c\ud558\uc5ec \ubc1c\ub9e4\ud55c \uc568\ubc94 \u3008\ucd94\uc5b5\uc758 \ubbf8\uc544 1\u3009\uc774 100\ub9cc\uc7a5\uc774\uc0c1 \ud310\ub9e4\ud558\ub294 \ub300\uc131\uacfc\ub97c \uac70\ub450\uc5b4 \uadf8 \ud574 \uace8\ub4e0\ub514\uc2a4\ud06c\uc0c1\uc744 \uc218\uc0c1\ud558\uc600\ub2e4. \ub300\ud55c\ubbfc\uad6d\uc5d0\uc11c\ub294 \ubaa8\ub4e0 \uc7a5\ub974\ub97c \uc12d\ub835\ud55c \uac00\uc218\ub85c \ud1b5\ud558\uc9c0\ub9cc \uc77c\ubcf8\uc5d0\uc11c\ub294 \uc870\uc6a9\ud544\uc744 \uc5d4\uce74\uc758 \ud669\uc81c\ub85c \ubd80\ub978\ub2e4. 1988\ub144\uc5d0\ub294 10\uc9d1 \uc74c\ubc18\uc744 \ubc1c\ud45c\ud588\ub294\ub370 \uadf8 \uc74c\ubc18\uc758 \uc218\ub85d\uace1 \u3008 \uc11c\uc6b8 \uc11c\uc6b8 \uc11c\uc6b8 \u3009,\u3008\ubaa8\ub098\ub9ac\uc790\u3009\uac00 \ub300\ubc15\uc744 \ud130\ub728\ub9ac\uba74\uc11c \uc5ec\ub7ec \uac00\uc218\uc640 \uacbd\uc7c1\uc744 \ud558\uc600\ub2e4. \uc774 \uc911 \u3008\uc11c\uc6b8 \uc11c\uc6b8 \uc11c\uc6b8\u3009\uc740 1988\ub144 \uc11c\uc6b8\uc62c\ub9bc\ud53d \ud3d0\ub9c9 \ub54c \ubc29\uc1a1\uc5d0\uc11c \uc804\ud30c\ub418\uba74\uc11c \ubd80\ud130 \uc778\uae30\ub97c \uc5bb\uc5c8\ub294\ub370 \uc870\uc6a9\ud544\uc740 \ub2f9\uc2dc \uc778\ud130\ubdf0 \ub54c \uc62c\ub9bc\ud53d\uc774 \ub05d\ub098\uba74 \uc6b0\ub9ac \uc0ac\ud68c\uac00 \uc6b0\uc6b8\ud574 \uc9c8 \uac83\uc774\ub77c\uace0 \uc0dd\uac01\ud574\uc11c \ub9cc\ub4e4\uac8c \ub418\uc5c8\ub2e4\uace0 \ubc1d\ud614\ub2e4.", "paragraph_answer": "\uc870\uc6a9\ud544\uc740 \ub77d \uc74c\uc545\uacfc \ubc1c\ub77c\ub4dc \uc74c\uc545, \ud2b8\ub85c\ud2b8 \uc74c\uc545 \ub610\ub294 \u3008\uac15\uc6d0\ub3c4 \uc544\ub9ac\ub791\u3009\ucc98\ub7fc \ud55c\uad6d \ubbfc\uc694\ub97c \ub9ac\uba54\uc774\ud06c\ud558\ub294 \ub4f1 \ub300\ud55c\ubbfc\uad6d\uc758 \ub300\uc911\uc74c\uc545 \uac70\uc758 \ubaa8\ub4e0 \uc7a5\ub974\ub97c \uc18c\ud654\ud574 \ub0b4\uba70 \ub2e4\uc591\ud55c \uc5f0\ub839\uce35\uc744 \ud655\ubcf4\ud558\uc600\ub2e4. \ub610\ud55c \uc870\uc6a9\ud544\uc740 \ub300\ud55c\ubbfc\uad6d \uac00\uc694\uacc4\uc5d0\uc11c \uc624\ube60\ubd80\ub300\ub85c \ubd88\ub9ac\ub294 \uc80a\uc740 \uc5ec\uc131 \ud32c\uce35\uc744 \uc774\ub04c\uace0 \ub2e4\ub2c8\uae30\ub3c4 \ud588\uc73c\uba70 \uc870\uc6a9\ud544 \ub4f1\uc7a5 \uc774\uc804\uc5d0 \ub300\ud55c\ubbfc\uad6d\uc758 \ucd5c\uace0 \uc778\uae30 \uac00\uc218\ub85c \ud638\ud3c9\uc744 \ubc1b\ub358 \ub0a8\uc9c4\uacfc \ub098\ud6c8\uc544\uc758 \ub4a4\ub97c \uc774\uc5b4 \ub300\ud55c\ubbfc\uad6d \uac00\uc694\uacc4\ub97c \uc8fc\ub984\uc7a1\uc558\ub2e4. \ud558\uc9c0\ub9cc 1985\ub144 \uc870\ud3ed \ucd9c\uc2e0 \uae40 \ubaa8\uc528\ub85c\ubd80\ud130 \ud3ed\ud589\uc5d0 \ud718\ub9d0\ub9ac\uae30\uae4c\uc9c0 \ud588\ub2e4. \uacbd\ucc30\uc740 \uc870\uc528\uc5d0 \ub300\ud55c \ud3ed\ud589\uc0ac\uac74\uc5d0\ub300\ud574 \uc218\uc0ac\ub97c \ud3bc\uce58\uae30\ub85c \ud588\uc73c\uba70 \ubcf4\ub514\uac00\ub4dc\ub97c 5\uba85\uc73c\ub85c \ub298\ub9ac\uae30\ub85c \ud558\uc600\ub2e4. \ud640\ub85c \ub300\ud55c\ubbfc\uad6d \uac00\uc694\uc0ac\uc758 \ud55c \ud68d\uc744 \uae0b\uae30 \uc2dc\uc791\ud558\ub358 \uc870\uc6a9\ud544\uc740 1986\ub144\uc5d0 \uc77c\ubcf8\uc5d0 \uc9c4\ucd9c\ud558\uc5ec \ubc1c\ub9e4\ud55c \uc568\ubc94 \u3008\ucd94\uc5b5\uc758 \ubbf8\uc544 1\u3009\uc774 100\ub9cc\uc7a5\uc774\uc0c1 \ud310\ub9e4\ud558\ub294 \ub300\uc131\uacfc\ub97c \uac70\ub450\uc5b4 \uadf8 \ud574 \uace8\ub4e0\ub514\uc2a4\ud06c\uc0c1\uc744 \uc218\uc0c1\ud558\uc600\ub2e4. \ub300\ud55c\ubbfc\uad6d\uc5d0\uc11c\ub294 \ubaa8\ub4e0 \uc7a5\ub974\ub97c \uc12d\ub835\ud55c \uac00\uc218\ub85c \ud1b5\ud558\uc9c0\ub9cc \uc77c\ubcf8\uc5d0\uc11c\ub294 \uc870\uc6a9\ud544\uc744 \uc5d4\uce74\uc758 \ud669\uc81c\ub85c \ubd80\ub978\ub2e4. 1988\ub144\uc5d0\ub294 10\uc9d1 \uc74c\ubc18\uc744 \ubc1c\ud45c\ud588\ub294\ub370 \uadf8 \uc74c\ubc18\uc758 \uc218\ub85d\uace1 \u3008 \uc11c\uc6b8 \uc11c\uc6b8 \uc11c\uc6b8 \u3009,\u3008\ubaa8\ub098\ub9ac\uc790\u3009\uac00 \ub300\ubc15\uc744 \ud130\ub728\ub9ac\uba74\uc11c \uc5ec\ub7ec \uac00\uc218\uc640 \uacbd\uc7c1\uc744 \ud558\uc600\ub2e4. \uc774 \uc911 \u3008\uc11c\uc6b8 \uc11c\uc6b8 \uc11c\uc6b8\u3009\uc740 1988\ub144 \uc11c\uc6b8\uc62c\ub9bc\ud53d \ud3d0\ub9c9 \ub54c \ubc29\uc1a1\uc5d0\uc11c \uc804\ud30c\ub418\uba74\uc11c \ubd80\ud130 \uc778\uae30\ub97c \uc5bb\uc5c8\ub294\ub370 \uc870\uc6a9\ud544\uc740 \ub2f9\uc2dc \uc778\ud130\ubdf0 \ub54c \uc62c\ub9bc\ud53d\uc774 \ub05d\ub098\uba74 \uc6b0\ub9ac \uc0ac\ud68c\uac00 \uc6b0\uc6b8\ud574 \uc9c8 \uac83\uc774\ub77c\uace0 \uc0dd\uac01\ud574\uc11c \ub9cc\ub4e4\uac8c \ub418\uc5c8\ub2e4\uace0 \ubc1d\ud614\ub2e4.", "sentence_answer": "1988\ub144\uc5d0\ub294 10\uc9d1 \uc74c\ubc18\uc744 \ubc1c\ud45c\ud588\ub294\ub370 \uadf8 \uc74c\ubc18\uc758 \uc218\ub85d\uace1 \u3008 \uc11c\uc6b8 \uc11c\uc6b8 \uc11c\uc6b8 \u3009,\u3008\ubaa8\ub098\ub9ac\uc790\u3009\uac00 \ub300\ubc15\uc744 \ud130\ub728\ub9ac\uba74\uc11c \uc5ec\ub7ec \uac00\uc218\uc640 \uacbd\uc7c1\uc744 \ud558\uc600\ub2e4.", "paragraph_id": "6553457-1-2", "paragraph_question": "question: 1988\ub144 \uc11c\uc6b8\uc62c\ub9bc\ud53d \ud3d0\ub9c9 \ub54c \uc804\ud30c\ub41c \uc870\uc6a9\ud544\uc758 \ub178\ub798\ub294?, context: \uc870\uc6a9\ud544\uc740 \ub77d \uc74c\uc545\uacfc \ubc1c\ub77c\ub4dc \uc74c\uc545, \ud2b8\ub85c\ud2b8 \uc74c\uc545 \ub610\ub294 \u3008\uac15\uc6d0\ub3c4 \uc544\ub9ac\ub791\u3009\ucc98\ub7fc \ud55c\uad6d \ubbfc\uc694\ub97c \ub9ac\uba54\uc774\ud06c\ud558\ub294 \ub4f1 \ub300\ud55c\ubbfc\uad6d\uc758 \ub300\uc911\uc74c\uc545 \uac70\uc758 \ubaa8\ub4e0 \uc7a5\ub974\ub97c \uc18c\ud654\ud574 \ub0b4\uba70 \ub2e4\uc591\ud55c \uc5f0\ub839\uce35\uc744 \ud655\ubcf4\ud558\uc600\ub2e4. \ub610\ud55c \uc870\uc6a9\ud544\uc740 \ub300\ud55c\ubbfc\uad6d \uac00\uc694\uacc4\uc5d0\uc11c \uc624\ube60\ubd80\ub300\ub85c \ubd88\ub9ac\ub294 \uc80a\uc740 \uc5ec\uc131 \ud32c\uce35\uc744 \uc774\ub04c\uace0 \ub2e4\ub2c8\uae30\ub3c4 \ud588\uc73c\uba70 \uc870\uc6a9\ud544 \ub4f1\uc7a5 \uc774\uc804\uc5d0 \ub300\ud55c\ubbfc\uad6d\uc758 \ucd5c\uace0 \uc778\uae30 \uac00\uc218\ub85c \ud638\ud3c9\uc744 \ubc1b\ub358 \ub0a8\uc9c4\uacfc \ub098\ud6c8\uc544\uc758 \ub4a4\ub97c \uc774\uc5b4 \ub300\ud55c\ubbfc\uad6d \uac00\uc694\uacc4\ub97c \uc8fc\ub984\uc7a1\uc558\ub2e4. \ud558\uc9c0\ub9cc 1985\ub144 \uc870\ud3ed \ucd9c\uc2e0 \uae40 \ubaa8\uc528\ub85c\ubd80\ud130 \ud3ed\ud589\uc5d0 \ud718\ub9d0\ub9ac\uae30\uae4c\uc9c0 \ud588\ub2e4. \uacbd\ucc30\uc740 \uc870\uc528\uc5d0 \ub300\ud55c \ud3ed\ud589\uc0ac\uac74\uc5d0\ub300\ud574 \uc218\uc0ac\ub97c \ud3bc\uce58\uae30\ub85c \ud588\uc73c\uba70 \ubcf4\ub514\uac00\ub4dc\ub97c 5\uba85\uc73c\ub85c \ub298\ub9ac\uae30\ub85c \ud558\uc600\ub2e4. \ud640\ub85c \ub300\ud55c\ubbfc\uad6d \uac00\uc694\uc0ac\uc758 \ud55c \ud68d\uc744 \uae0b\uae30 \uc2dc\uc791\ud558\ub358 \uc870\uc6a9\ud544\uc740 1986\ub144\uc5d0 \uc77c\ubcf8\uc5d0 \uc9c4\ucd9c\ud558\uc5ec \ubc1c\ub9e4\ud55c \uc568\ubc94 \u3008\ucd94\uc5b5\uc758 \ubbf8\uc544 1\u3009\uc774 100\ub9cc\uc7a5\uc774\uc0c1 \ud310\ub9e4\ud558\ub294 \ub300\uc131\uacfc\ub97c \uac70\ub450\uc5b4 \uadf8 \ud574 \uace8\ub4e0\ub514\uc2a4\ud06c\uc0c1\uc744 \uc218\uc0c1\ud558\uc600\ub2e4. \ub300\ud55c\ubbfc\uad6d\uc5d0\uc11c\ub294 \ubaa8\ub4e0 \uc7a5\ub974\ub97c \uc12d\ub835\ud55c \uac00\uc218\ub85c \ud1b5\ud558\uc9c0\ub9cc \uc77c\ubcf8\uc5d0\uc11c\ub294 \uc870\uc6a9\ud544\uc744 \uc5d4\uce74\uc758 \ud669\uc81c\ub85c \ubd80\ub978\ub2e4. 1988\ub144\uc5d0\ub294 10\uc9d1 \uc74c\ubc18\uc744 \ubc1c\ud45c\ud588\ub294\ub370 \uadf8 \uc74c\ubc18\uc758 \uc218\ub85d\uace1 \u3008\uc11c\uc6b8 \uc11c\uc6b8 \uc11c\uc6b8\u3009,\u3008\ubaa8\ub098\ub9ac\uc790\u3009\uac00 \ub300\ubc15\uc744 \ud130\ub728\ub9ac\uba74\uc11c \uc5ec\ub7ec \uac00\uc218\uc640 \uacbd\uc7c1\uc744 \ud558\uc600\ub2e4. \uc774 \uc911 \u3008\uc11c\uc6b8 \uc11c\uc6b8 \uc11c\uc6b8\u3009\uc740 1988\ub144 \uc11c\uc6b8\uc62c\ub9bc\ud53d \ud3d0\ub9c9 \ub54c \ubc29\uc1a1\uc5d0\uc11c \uc804\ud30c\ub418\uba74\uc11c \ubd80\ud130 \uc778\uae30\ub97c \uc5bb\uc5c8\ub294\ub370 \uc870\uc6a9\ud544\uc740 \ub2f9\uc2dc \uc778\ud130\ubdf0 \ub54c \uc62c\ub9bc\ud53d\uc774 \ub05d\ub098\uba74 \uc6b0\ub9ac \uc0ac\ud68c\uac00 \uc6b0\uc6b8\ud574 \uc9c8 \uac83\uc774\ub77c\uace0 \uc0dd\uac01\ud574\uc11c \ub9cc\ub4e4\uac8c \ub418\uc5c8\ub2e4\uace0 \ubc1d\ud614\ub2e4."} {"question": "\ud55c\uad6d\uc758 \uc0ac\ub4dc \ubc30\uce58\uc5d0 \ucd5c\uc21c\uc2e4\uc774 \uac1c\uc785\ud588\uc74c\uc744 \ucd94\uce21\ud55c \uc778\ubb3c\uc740?", "paragraph": "\ud55c\uaca8\ub808 \uc0ac\uc124\uc740 \"\ud30c\uba74\ub41c \ubc15\uadfc\ud61c \uc804 \ub300\ud1b5\ub839\uc744 \ubcf4\uc88c \ud558\ub358 \ucc38\ubaa8\ub85c\uc368, \uae40\uad00\uc9c4 \uc548\ubcf4 \uc2e4\uc7a5\uacfc \ud669\uad50\uc548 \ucd1d\ub9ac\uac00, \uc548\ubcf4\uc0c1\ud669\uc744 \uc720\uc9c0 \ubc0f \uad00\ub9ac\ub97c \ud558\ub294 \uc218\uc900\uc744 \ub118\uc5b4\uc11c\uc11c, \uacb0\uc815\uc790 \ud589\uc138\ub97c \ud55c\uac83\uc740 \uad8c\ub825 \ub0a8\uc6a9\uc774\uba70, \uc6d4\uad8c \ud589\uc704\ub2e4\"\ub77c\uba70 \ube44\ud310\uc744 \ud558\uc600\ub2e4. \ub1cc\ubb3c \uc218\ub8b0 \ud610\uc758\ub4f1\uc73c\ub85c \ud0c4\ud575\ub41c \ubc15\uadfc\ud61c \uc815\ubd80\uc5d0\uc11c\ub294, \uad6d\ubc29\ubd80\uc5d0\uc11c\ub294 2016\ub144 7\uc6d4 5\uc77c \uc0ac\ub4dc\uc2dc\uc2a4\ud15c\uc740 \uacb0\uc815\ub41c \ubc14\uac00 \uc5c6\uace0 \uc544\uc9c1 \uc2e4\ubb34 \uac80\ud1a0\uc911\uc774\ub77c\ub294 \ubcf4\ub3c4\ub97c \uacf5\uc2dd\uc801\uc73c\ub85c \ubc1c\ud45c\ub41c\uc9c0, \uc774\ud2c0\ud6c4 \uc778 7\uc6d4 7\uc77c \uac11\uc790\uae30 \uccad\uc640\ub300 \uc694\uccad\uc73c\ub85c \uad6d\uac00 \uc548\uc804 \ubcf4\uc7a5 \ud68c\uc758\ub97c \uc18c\uc9d1\ub418\uace0 \uc774\uacf3\uc5d0\uc11c \uacb0\uc815\ub418\uc5c8\ub294\ub370, \uc774\ub54c \uc8fc\ubb34 \ubd80\ucc98\uc778 \uad6d\ubc29 \uc7a5\uad00\uc774 \ubd88\ucc38\ud558\uace0, \ucc28\uad00\uc774 \ub300\uc2e0 \ucc38\uac00\ud55c \uc0c1\ud0dc\uc5d0\uc11c \uc9c4\ud589\ub418\uc5c8\uace0, \uc548\uac74\uc5ed\uc2dc \uad00\ub828 \ubd80\ucc98\uc778 \uad6d\ubc29\ubd80\uc640 \uc678\uad50\ubd80\uac00 \uc544\ub2cc \uae40\uad00\uc9c4 \uccad\uc640\ub300 \uc548\ubcf4 \uc2e4\uc7a5\uc774 \uccad\uc640\ub300\uc758 \uc758\uc9c0\uc778, \"\uc815\ubb34\uc801 \ud310\ub2e8\"\uc774\ub77c\uace0\ud558\uba74\uc11c, \uc989\uc11d \uc548\uac74\uc73c\ub85c \uc0ac\ub4dc \ubc30\uce58\ub97c \uc0c1\uc815\ud558\uc5ec \ud1b5\uacfc \ub418\uc5c8\ub2e4\ub294 \uc8fc\uc7a5\uc774 \uc788\ub2e4. \uc989 \uc0ac\ub4dc \uc2dc\uc2a4\ud15c\uc774 \ud544\uc694\uc131 \ubc0f \uc801\ud569\uc131\uc5d0 \ub300\ud574\uc11c\ub294 \uc0ac\uc2e4\uc0c1 \uccad\uc640\ub300 \uc548\ubcf4\uc2e4\uacfc \uc8fc\ud55c\ubbf8\uad70\uc774 \uad6d\ubc29\ubd80\ub97c \uac70\uce58\uc9c0 \uc54a\uace0 \uc9c1\uc811 \uc811\ucd09\ud55c \uac83\uc774\uba70, \uc774\ud6c4\uc5d0 \uad6d\ubc29\ubd80\ub294 \uc774\uac83\uc744 \uc704\ud55c \uc218\uc2b5\uc73c\ub85c \uad70\uc0ac\uc801 \ud569\ub9ac\uc131\uc774 \uace0\ub824\ub418\uc9c0 \uc54a\uc740 \uacb0\uc815\uc774\uc5c8\ub2e4\ub294 \uc8fc\uc7a5\uc774 \uc788\ub2e4. \uc548\ubbfc\uc11d \ub354\ubd88\uc5b4\ubbfc\uc8fc\ub2f9 \uc758\uc6d0\uc740 2016\ub144 11\uc6d4 24\uc77c \u2018\uae40\uc5b4\uc900\uc758 \ub274\uc2a4\uacf5\uc7a5\u2019\uc5d0 \ucd9c\uc5f0\ud574 \u201c\uc9c0\ub09c 6\uc6d4 \ucd5c \uc528\uac00 \ub85d\ud788\ub4dc\ub9c8\ud2f4 \ud68c\uc7a5\uacfc \ub9cc\ub09c \uac83\uc73c\ub85c \uc54c\uace0 \uc788\ub2e4\u201d\uba70 \uc0ac\ub4dc(THAAD) \ub3c4\uc785\uc5d0 \ucd5c \uc528\uac00 \uac1c\uc785\ud588\ub2e4\uace0 \ucd94\uce21\ud558\uc600\ub2e4. \ub610\ud55c \uc548 \uc758\uc6d0\uc740 \u201c\ucd5c \uc528\uc640 \ub85d\ud788\ub4dc\ub9c8\ud2f4\uc744 \uc5f0\uacb0\ud574\uc900 \uc778\ubb3c\uc740 \ud604 \uc815\uad8c\uc758 \uc678\uad50\uc548\ubcf4\ubd84\uc57c \uc2e4\uc138\u201d\ub77c\uba70 \uc774 \uc778\ubb3c\uc774 6\ub144 \uc804 \ubbf8\uad6d \uccb4\ub958 \ub2f9\uc2dc \ub85d\ud788\ub4dc\ub9c8\ud2f4\uc5d0\uc11c 2\ub144\uac04 \uccb4\ub958\ube44\ub97c \uc9c0\uc6d0\ud574\uc92c\ub2e4\uace0 \uc8fc\uc7a5\ud588\ub2e4. \ub610 \uc548 \uc758\uc6d0\uc740\u201c\ucd5c\uc21c\uc2e4\uc774 \uc9c0\uae08\uae4c\uc9c0 \ud574\uba39\uc740 \uac83\uc740 \uc0ac\ub4dc\uc640 \ube44\uad50\ud558\uba74 \uaecc\uac12\u201d\uc774\ub77c\uace0 \ucd94\ub860\ud558\uba70 \uc0ac\ub4dc\ub85c \uc778\ud574 \uc774\uc775\uc744 \ubcf8 \uc218\ub9ce\uc740 \uad00\uacc4\uc790\ub4e4\uc744 \uc0c1\uc0c1\ud558\ub3c4\ub85d \uc120\ub3d9\ud588\ub2e4. \uc55e\uc11c 11\uc6d4 15\uc77c \uc548 \uc758\uc6d0\uc740 \u201c\uc544\uc8fc \uc2e0\ub8b0\ud560 \ub9cc\ud55c \uad70 \ub0b4\ubd80\uc5d0\uc11c \uc81c\ubcf4\ub97c \ubc1b\uc558\ub2e4\u201d\uba70 \u201c2010~2015\ub144\uc5d0 \ube44\ud574 2015~2021\ub144 \ub85d\ud788\ub4dc\ub9c8\ud2f4 \uacc4\uc57d \uccb4\uacb0\uc774 \ub208\uc5d0 \ub744\uac8c \ub298\uc5c8\ub2e4\u201d\uace0 \uc9c4\uc0c1\uaddc\uba85\uc744 \uc8fc\uc7a5\ud588\ub2e4. \ud574\ub2f9 \uc758\ud639\uc740, \u201c2016\ub144 6\uc6d4\uc5d0 \ucd5c\uc21c\uc2e4\uc774 \ub85d\ud788\ub4dc\ub9c8\ud2f4 \ub9cc\ub098\uace0, 7\uc6d4\uc5d0 \uad00\ub828 \ubd80\ucc98\uc778 \uad6d\ubc29 \uc7a5\uad00\ub3c4 \ubb34\uc2dc\ub97c \ud558\uace0, \uc0ac\ub4dc\uac00 \uacb0\uc815\ub418\uc5c8\uace0...\uc774\uc81c \ud55c\uc77c\uc815\ubcf4\ubcf4\ud638\ud611\uc815\uc73c\ub85c \uc0ac\ub4dc \uc815\ubcf4\uac00 \uc77c\ubcf8\uc5d0\uac8c \uc81c\uacf5\ub418\uba74, \uc774\uac74 \uc644\uc804 \ub098\ub77c\ub97c \ud314\uc544\uba39\uc740 \uac83\u201d\uc774\ub77c\uace0 \ucd94\uce21\ud588\ub2e4. \ub610\ud55c \u201c\uc9c0\ub09c\ub2ec \ub3cc\ub358 \uc815\ubcf4\uc9c0\uac00 \uc0ac\uc2e4\uc774 \ub9de\ub294\uac74\uac00? \uc6d0\ub798 \ubb34\uae30\uac70\ub798 \uc815\uc724\ud68c\uac00 \ud558\ub358 \uc77c\uc774\uc5c8\ub294\ub370 \ucd5c\uc21c\uc2e4\uc774 \ub9b0\ub2e4\uae40\uc5d0\uac8c \uc218\uc870\uc6d0\uc758 \ub9ac\ubca0\uc774\ud2b8 \ucc59\uacbc\ub2e4\ub294 \ub9d0\uacfc \ub85d\ud788\ub4dc\ub9c8\ud2f4\uc774 \ud5a5\ud6c4 10\ub144\uac04 \uad6d\ub0b4\uc5d0\uc11c \ubc1b\ub294 \ubb34\uae30\ub300\uae08\uc740 66\uc870 \uc778\ub370 \uc774\uac74\uc5d0 \ub300\ud55c \ucd5c\uc21c\uc2e4\uc758 \uac1c\uc785\uc740 \ub0b4\ub780\uc8c4\u201d\ub77c\uace0 \ucd94\uce21\ud588\ub2e4.", "answer": "\uc548\ubbfc\uc11d", "sentence": "\uc548\ubbfc\uc11d \ub354\ubd88\uc5b4\ubbfc\uc8fc\ub2f9 \uc758\uc6d0\uc740 2016\ub144 11\uc6d4 24\uc77c \u2018\uae40\uc5b4\uc900\uc758 \ub274\uc2a4\uacf5\uc7a5\u2019\uc5d0 \ucd9c\uc5f0\ud574 \u201c\uc9c0\ub09c 6\uc6d4 \ucd5c \uc528\uac00 \ub85d\ud788\ub4dc\ub9c8\ud2f4 \ud68c\uc7a5\uacfc \ub9cc\ub09c \uac83\uc73c\ub85c \uc54c\uace0 \uc788\ub2e4\u201d\uba70 \uc0ac\ub4dc(THAAD) \ub3c4\uc785\uc5d0 \ucd5c \uc528\uac00 \uac1c\uc785\ud588\ub2e4\uace0 \ucd94\uce21\ud558\uc600\ub2e4.", "paragraph_sentence": "\ud55c\uaca8\ub808 \uc0ac\uc124\uc740 \"\ud30c\uba74\ub41c \ubc15\uadfc\ud61c \uc804 \ub300\ud1b5\ub839\uc744 \ubcf4\uc88c \ud558\ub358 \ucc38\ubaa8\ub85c\uc368, \uae40\uad00\uc9c4 \uc548\ubcf4 \uc2e4\uc7a5\uacfc \ud669\uad50\uc548 \ucd1d\ub9ac\uac00, \uc548\ubcf4\uc0c1\ud669\uc744 \uc720\uc9c0 \ubc0f \uad00\ub9ac\ub97c \ud558\ub294 \uc218\uc900\uc744 \ub118\uc5b4\uc11c\uc11c, \uacb0\uc815\uc790 \ud589\uc138\ub97c \ud55c\uac83\uc740 \uad8c\ub825 \ub0a8\uc6a9\uc774\uba70, \uc6d4\uad8c \ud589\uc704\ub2e4\"\ub77c\uba70 \ube44\ud310\uc744 \ud558\uc600\ub2e4. \ub1cc\ubb3c \uc218\ub8b0 \ud610\uc758\ub4f1\uc73c\ub85c \ud0c4\ud575\ub41c \ubc15\uadfc\ud61c \uc815\ubd80\uc5d0\uc11c\ub294, \uad6d\ubc29\ubd80\uc5d0\uc11c\ub294 2016\ub144 7\uc6d4 5\uc77c \uc0ac\ub4dc\uc2dc\uc2a4\ud15c\uc740 \uacb0\uc815\ub41c \ubc14\uac00 \uc5c6\uace0 \uc544\uc9c1 \uc2e4\ubb34 \uac80\ud1a0\uc911\uc774\ub77c\ub294 \ubcf4\ub3c4\ub97c \uacf5\uc2dd\uc801\uc73c\ub85c \ubc1c\ud45c\ub41c\uc9c0, \uc774\ud2c0\ud6c4 \uc778 7\uc6d4 7\uc77c \uac11\uc790\uae30 \uccad\uc640\ub300 \uc694\uccad\uc73c\ub85c \uad6d\uac00 \uc548\uc804 \ubcf4\uc7a5 \ud68c\uc758\ub97c \uc18c\uc9d1\ub418\uace0 \uc774\uacf3\uc5d0\uc11c \uacb0\uc815\ub418\uc5c8\ub294\ub370, \uc774\ub54c \uc8fc\ubb34 \ubd80\ucc98\uc778 \uad6d\ubc29 \uc7a5\uad00\uc774 \ubd88\ucc38\ud558\uace0, \ucc28\uad00\uc774 \ub300\uc2e0 \ucc38\uac00\ud55c \uc0c1\ud0dc\uc5d0\uc11c \uc9c4\ud589\ub418\uc5c8\uace0, \uc548\uac74\uc5ed\uc2dc \uad00\ub828 \ubd80\ucc98\uc778 \uad6d\ubc29\ubd80\uc640 \uc678\uad50\ubd80\uac00 \uc544\ub2cc \uae40\uad00\uc9c4 \uccad\uc640\ub300 \uc548\ubcf4 \uc2e4\uc7a5\uc774 \uccad\uc640\ub300\uc758 \uc758\uc9c0\uc778, \"\uc815\ubb34\uc801 \ud310\ub2e8\"\uc774\ub77c\uace0\ud558\uba74\uc11c, \uc989\uc11d \uc548\uac74\uc73c\ub85c \uc0ac\ub4dc \ubc30\uce58\ub97c \uc0c1\uc815\ud558\uc5ec \ud1b5\uacfc \ub418\uc5c8\ub2e4\ub294 \uc8fc\uc7a5\uc774 \uc788\ub2e4. \uc989 \uc0ac\ub4dc \uc2dc\uc2a4\ud15c\uc774 \ud544\uc694\uc131 \ubc0f \uc801\ud569\uc131\uc5d0 \ub300\ud574\uc11c\ub294 \uc0ac\uc2e4\uc0c1 \uccad\uc640\ub300 \uc548\ubcf4\uc2e4\uacfc \uc8fc\ud55c\ubbf8\uad70\uc774 \uad6d\ubc29\ubd80\ub97c \uac70\uce58\uc9c0 \uc54a\uace0 \uc9c1\uc811 \uc811\ucd09\ud55c \uac83\uc774\uba70, \uc774\ud6c4\uc5d0 \uad6d\ubc29\ubd80\ub294 \uc774\uac83\uc744 \uc704\ud55c \uc218\uc2b5\uc73c\ub85c \uad70\uc0ac\uc801 \ud569\ub9ac\uc131\uc774 \uace0\ub824\ub418\uc9c0 \uc54a\uc740 \uacb0\uc815\uc774\uc5c8\ub2e4\ub294 \uc8fc\uc7a5\uc774 \uc788\ub2e4. \uc548\ubbfc\uc11d \ub354\ubd88\uc5b4\ubbfc\uc8fc\ub2f9 \uc758\uc6d0\uc740 2016\ub144 11\uc6d4 24\uc77c \u2018\uae40\uc5b4\uc900\uc758 \ub274\uc2a4\uacf5\uc7a5\u2019\uc5d0 \ucd9c\uc5f0\ud574 \u201c\uc9c0\ub09c 6\uc6d4 \ucd5c \uc528\uac00 \ub85d\ud788\ub4dc\ub9c8\ud2f4 \ud68c\uc7a5\uacfc \ub9cc\ub09c \uac83\uc73c\ub85c \uc54c\uace0 \uc788\ub2e4\u201d\uba70 \uc0ac\ub4dc(THAAD) \ub3c4\uc785\uc5d0 \ucd5c \uc528\uac00 \uac1c\uc785\ud588\ub2e4\uace0 \ucd94\uce21\ud558\uc600\ub2e4. \ub610\ud55c \uc548 \uc758\uc6d0\uc740 \u201c\ucd5c \uc528\uc640 \ub85d\ud788\ub4dc\ub9c8\ud2f4\uc744 \uc5f0\uacb0\ud574\uc900 \uc778\ubb3c\uc740 \ud604 \uc815\uad8c\uc758 \uc678\uad50\uc548\ubcf4\ubd84\uc57c \uc2e4\uc138\u201d\ub77c\uba70 \uc774 \uc778\ubb3c\uc774 6\ub144 \uc804 \ubbf8\uad6d \uccb4\ub958 \ub2f9\uc2dc \ub85d\ud788\ub4dc\ub9c8\ud2f4\uc5d0\uc11c 2\ub144\uac04 \uccb4\ub958\ube44\ub97c \uc9c0\uc6d0\ud574\uc92c\ub2e4\uace0 \uc8fc\uc7a5\ud588\ub2e4. \ub610 \uc548 \uc758\uc6d0\uc740\u201c\ucd5c\uc21c\uc2e4\uc774 \uc9c0\uae08\uae4c\uc9c0 \ud574\uba39\uc740 \uac83\uc740 \uc0ac\ub4dc\uc640 \ube44\uad50\ud558\uba74 \uaecc\uac12\u201d\uc774\ub77c\uace0 \ucd94\ub860\ud558\uba70 \uc0ac\ub4dc\ub85c \uc778\ud574 \uc774\uc775\uc744 \ubcf8 \uc218\ub9ce\uc740 \uad00\uacc4\uc790\ub4e4\uc744 \uc0c1\uc0c1\ud558\ub3c4\ub85d \uc120\ub3d9\ud588\ub2e4. \uc55e\uc11c 11\uc6d4 15\uc77c \uc548 \uc758\uc6d0\uc740 \u201c\uc544\uc8fc \uc2e0\ub8b0\ud560 \ub9cc\ud55c \uad70 \ub0b4\ubd80\uc5d0\uc11c \uc81c\ubcf4\ub97c \ubc1b\uc558\ub2e4\u201d\uba70 \u201c2010~2015\ub144\uc5d0 \ube44\ud574 2015~2021\ub144 \ub85d\ud788\ub4dc\ub9c8\ud2f4 \uacc4\uc57d \uccb4\uacb0\uc774 \ub208\uc5d0 \ub744\uac8c \ub298\uc5c8\ub2e4\u201d\uace0 \uc9c4\uc0c1\uaddc\uba85\uc744 \uc8fc\uc7a5\ud588\ub2e4. \ud574\ub2f9 \uc758\ud639\uc740, \u201c2016\ub144 6\uc6d4\uc5d0 \ucd5c\uc21c\uc2e4\uc774 \ub85d\ud788\ub4dc\ub9c8\ud2f4 \ub9cc\ub098\uace0, 7\uc6d4\uc5d0 \uad00\ub828 \ubd80\ucc98\uc778 \uad6d\ubc29 \uc7a5\uad00\ub3c4 \ubb34\uc2dc\ub97c \ud558\uace0, \uc0ac\ub4dc\uac00 \uacb0\uc815\ub418\uc5c8\uace0...\uc774\uc81c \ud55c\uc77c\uc815\ubcf4\ubcf4\ud638\ud611\uc815\uc73c\ub85c \uc0ac\ub4dc \uc815\ubcf4\uac00 \uc77c\ubcf8\uc5d0\uac8c \uc81c\uacf5\ub418\uba74, \uc774\uac74 \uc644\uc804 \ub098\ub77c\ub97c \ud314\uc544\uba39\uc740 \uac83\u201d\uc774\ub77c\uace0 \ucd94\uce21\ud588\ub2e4. \ub610\ud55c \u201c\uc9c0\ub09c\ub2ec \ub3cc\ub358 \uc815\ubcf4\uc9c0\uac00 \uc0ac\uc2e4\uc774 \ub9de\ub294\uac74\uac00? \uc6d0\ub798 \ubb34\uae30\uac70\ub798 \uc815\uc724\ud68c\uac00 \ud558\ub358 \uc77c\uc774\uc5c8\ub294\ub370 \ucd5c\uc21c\uc2e4\uc774 \ub9b0\ub2e4\uae40\uc5d0\uac8c \uc218\uc870\uc6d0\uc758 \ub9ac\ubca0\uc774\ud2b8 \ucc59\uacbc\ub2e4\ub294 \ub9d0\uacfc \ub85d\ud788\ub4dc\ub9c8\ud2f4\uc774 \ud5a5\ud6c4 10\ub144\uac04 \uad6d\ub0b4\uc5d0\uc11c \ubc1b\ub294 \ubb34\uae30\ub300\uae08\uc740 66\uc870 \uc778\ub370 \uc774\uac74\uc5d0 \ub300\ud55c \ucd5c\uc21c\uc2e4\uc758 \uac1c\uc785\uc740 \ub0b4\ub780\uc8c4\u201d\ub77c\uace0 \ucd94\uce21\ud588\ub2e4.", "paragraph_answer": "\ud55c\uaca8\ub808 \uc0ac\uc124\uc740 \"\ud30c\uba74\ub41c \ubc15\uadfc\ud61c \uc804 \ub300\ud1b5\ub839\uc744 \ubcf4\uc88c \ud558\ub358 \ucc38\ubaa8\ub85c\uc368, \uae40\uad00\uc9c4 \uc548\ubcf4 \uc2e4\uc7a5\uacfc \ud669\uad50\uc548 \ucd1d\ub9ac\uac00, \uc548\ubcf4\uc0c1\ud669\uc744 \uc720\uc9c0 \ubc0f \uad00\ub9ac\ub97c \ud558\ub294 \uc218\uc900\uc744 \ub118\uc5b4\uc11c\uc11c, \uacb0\uc815\uc790 \ud589\uc138\ub97c \ud55c\uac83\uc740 \uad8c\ub825 \ub0a8\uc6a9\uc774\uba70, \uc6d4\uad8c \ud589\uc704\ub2e4\"\ub77c\uba70 \ube44\ud310\uc744 \ud558\uc600\ub2e4. \ub1cc\ubb3c \uc218\ub8b0 \ud610\uc758\ub4f1\uc73c\ub85c \ud0c4\ud575\ub41c \ubc15\uadfc\ud61c \uc815\ubd80\uc5d0\uc11c\ub294, \uad6d\ubc29\ubd80\uc5d0\uc11c\ub294 2016\ub144 7\uc6d4 5\uc77c \uc0ac\ub4dc\uc2dc\uc2a4\ud15c\uc740 \uacb0\uc815\ub41c \ubc14\uac00 \uc5c6\uace0 \uc544\uc9c1 \uc2e4\ubb34 \uac80\ud1a0\uc911\uc774\ub77c\ub294 \ubcf4\ub3c4\ub97c \uacf5\uc2dd\uc801\uc73c\ub85c \ubc1c\ud45c\ub41c\uc9c0, \uc774\ud2c0\ud6c4 \uc778 7\uc6d4 7\uc77c \uac11\uc790\uae30 \uccad\uc640\ub300 \uc694\uccad\uc73c\ub85c \uad6d\uac00 \uc548\uc804 \ubcf4\uc7a5 \ud68c\uc758\ub97c \uc18c\uc9d1\ub418\uace0 \uc774\uacf3\uc5d0\uc11c \uacb0\uc815\ub418\uc5c8\ub294\ub370, \uc774\ub54c \uc8fc\ubb34 \ubd80\ucc98\uc778 \uad6d\ubc29 \uc7a5\uad00\uc774 \ubd88\ucc38\ud558\uace0, \ucc28\uad00\uc774 \ub300\uc2e0 \ucc38\uac00\ud55c \uc0c1\ud0dc\uc5d0\uc11c \uc9c4\ud589\ub418\uc5c8\uace0, \uc548\uac74\uc5ed\uc2dc \uad00\ub828 \ubd80\ucc98\uc778 \uad6d\ubc29\ubd80\uc640 \uc678\uad50\ubd80\uac00 \uc544\ub2cc \uae40\uad00\uc9c4 \uccad\uc640\ub300 \uc548\ubcf4 \uc2e4\uc7a5\uc774 \uccad\uc640\ub300\uc758 \uc758\uc9c0\uc778, \"\uc815\ubb34\uc801 \ud310\ub2e8\"\uc774\ub77c\uace0\ud558\uba74\uc11c, \uc989\uc11d \uc548\uac74\uc73c\ub85c \uc0ac\ub4dc \ubc30\uce58\ub97c \uc0c1\uc815\ud558\uc5ec \ud1b5\uacfc \ub418\uc5c8\ub2e4\ub294 \uc8fc\uc7a5\uc774 \uc788\ub2e4. \uc989 \uc0ac\ub4dc \uc2dc\uc2a4\ud15c\uc774 \ud544\uc694\uc131 \ubc0f \uc801\ud569\uc131\uc5d0 \ub300\ud574\uc11c\ub294 \uc0ac\uc2e4\uc0c1 \uccad\uc640\ub300 \uc548\ubcf4\uc2e4\uacfc \uc8fc\ud55c\ubbf8\uad70\uc774 \uad6d\ubc29\ubd80\ub97c \uac70\uce58\uc9c0 \uc54a\uace0 \uc9c1\uc811 \uc811\ucd09\ud55c \uac83\uc774\uba70, \uc774\ud6c4\uc5d0 \uad6d\ubc29\ubd80\ub294 \uc774\uac83\uc744 \uc704\ud55c \uc218\uc2b5\uc73c\ub85c \uad70\uc0ac\uc801 \ud569\ub9ac\uc131\uc774 \uace0\ub824\ub418\uc9c0 \uc54a\uc740 \uacb0\uc815\uc774\uc5c8\ub2e4\ub294 \uc8fc\uc7a5\uc774 \uc788\ub2e4. \uc548\ubbfc\uc11d \ub354\ubd88\uc5b4\ubbfc\uc8fc\ub2f9 \uc758\uc6d0\uc740 2016\ub144 11\uc6d4 24\uc77c \u2018\uae40\uc5b4\uc900\uc758 \ub274\uc2a4\uacf5\uc7a5\u2019\uc5d0 \ucd9c\uc5f0\ud574 \u201c\uc9c0\ub09c 6\uc6d4 \ucd5c \uc528\uac00 \ub85d\ud788\ub4dc\ub9c8\ud2f4 \ud68c\uc7a5\uacfc \ub9cc\ub09c \uac83\uc73c\ub85c \uc54c\uace0 \uc788\ub2e4\u201d\uba70 \uc0ac\ub4dc(THAAD) \ub3c4\uc785\uc5d0 \ucd5c \uc528\uac00 \uac1c\uc785\ud588\ub2e4\uace0 \ucd94\uce21\ud558\uc600\ub2e4. \ub610\ud55c \uc548 \uc758\uc6d0\uc740 \u201c\ucd5c \uc528\uc640 \ub85d\ud788\ub4dc\ub9c8\ud2f4\uc744 \uc5f0\uacb0\ud574\uc900 \uc778\ubb3c\uc740 \ud604 \uc815\uad8c\uc758 \uc678\uad50\uc548\ubcf4\ubd84\uc57c \uc2e4\uc138\u201d\ub77c\uba70 \uc774 \uc778\ubb3c\uc774 6\ub144 \uc804 \ubbf8\uad6d \uccb4\ub958 \ub2f9\uc2dc \ub85d\ud788\ub4dc\ub9c8\ud2f4\uc5d0\uc11c 2\ub144\uac04 \uccb4\ub958\ube44\ub97c \uc9c0\uc6d0\ud574\uc92c\ub2e4\uace0 \uc8fc\uc7a5\ud588\ub2e4. \ub610 \uc548 \uc758\uc6d0\uc740\u201c\ucd5c\uc21c\uc2e4\uc774 \uc9c0\uae08\uae4c\uc9c0 \ud574\uba39\uc740 \uac83\uc740 \uc0ac\ub4dc\uc640 \ube44\uad50\ud558\uba74 \uaecc\uac12\u201d\uc774\ub77c\uace0 \ucd94\ub860\ud558\uba70 \uc0ac\ub4dc\ub85c \uc778\ud574 \uc774\uc775\uc744 \ubcf8 \uc218\ub9ce\uc740 \uad00\uacc4\uc790\ub4e4\uc744 \uc0c1\uc0c1\ud558\ub3c4\ub85d \uc120\ub3d9\ud588\ub2e4. \uc55e\uc11c 11\uc6d4 15\uc77c \uc548 \uc758\uc6d0\uc740 \u201c\uc544\uc8fc \uc2e0\ub8b0\ud560 \ub9cc\ud55c \uad70 \ub0b4\ubd80\uc5d0\uc11c \uc81c\ubcf4\ub97c \ubc1b\uc558\ub2e4\u201d\uba70 \u201c2010~2015\ub144\uc5d0 \ube44\ud574 2015~2021\ub144 \ub85d\ud788\ub4dc\ub9c8\ud2f4 \uacc4\uc57d \uccb4\uacb0\uc774 \ub208\uc5d0 \ub744\uac8c \ub298\uc5c8\ub2e4\u201d\uace0 \uc9c4\uc0c1\uaddc\uba85\uc744 \uc8fc\uc7a5\ud588\ub2e4. \ud574\ub2f9 \uc758\ud639\uc740, \u201c2016\ub144 6\uc6d4\uc5d0 \ucd5c\uc21c\uc2e4\uc774 \ub85d\ud788\ub4dc\ub9c8\ud2f4 \ub9cc\ub098\uace0, 7\uc6d4\uc5d0 \uad00\ub828 \ubd80\ucc98\uc778 \uad6d\ubc29 \uc7a5\uad00\ub3c4 \ubb34\uc2dc\ub97c \ud558\uace0, \uc0ac\ub4dc\uac00 \uacb0\uc815\ub418\uc5c8\uace0...\uc774\uc81c \ud55c\uc77c\uc815\ubcf4\ubcf4\ud638\ud611\uc815\uc73c\ub85c \uc0ac\ub4dc \uc815\ubcf4\uac00 \uc77c\ubcf8\uc5d0\uac8c \uc81c\uacf5\ub418\uba74, \uc774\uac74 \uc644\uc804 \ub098\ub77c\ub97c \ud314\uc544\uba39\uc740 \uac83\u201d\uc774\ub77c\uace0 \ucd94\uce21\ud588\ub2e4. \ub610\ud55c \u201c\uc9c0\ub09c\ub2ec \ub3cc\ub358 \uc815\ubcf4\uc9c0\uac00 \uc0ac\uc2e4\uc774 \ub9de\ub294\uac74\uac00? \uc6d0\ub798 \ubb34\uae30\uac70\ub798 \uc815\uc724\ud68c\uac00 \ud558\ub358 \uc77c\uc774\uc5c8\ub294\ub370 \ucd5c\uc21c\uc2e4\uc774 \ub9b0\ub2e4\uae40\uc5d0\uac8c \uc218\uc870\uc6d0\uc758 \ub9ac\ubca0\uc774\ud2b8 \ucc59\uacbc\ub2e4\ub294 \ub9d0\uacfc \ub85d\ud788\ub4dc\ub9c8\ud2f4\uc774 \ud5a5\ud6c4 10\ub144\uac04 \uad6d\ub0b4\uc5d0\uc11c \ubc1b\ub294 \ubb34\uae30\ub300\uae08\uc740 66\uc870 \uc778\ub370 \uc774\uac74\uc5d0 \ub300\ud55c \ucd5c\uc21c\uc2e4\uc758 \uac1c\uc785\uc740 \ub0b4\ub780\uc8c4\u201d\ub77c\uace0 \ucd94\uce21\ud588\ub2e4.", "sentence_answer": " \uc548\ubbfc\uc11d \ub354\ubd88\uc5b4\ubbfc\uc8fc\ub2f9 \uc758\uc6d0\uc740 2016\ub144 11\uc6d4 24\uc77c \u2018\uae40\uc5b4\uc900\uc758 \ub274\uc2a4\uacf5\uc7a5\u2019\uc5d0 \ucd9c\uc5f0\ud574 \u201c\uc9c0\ub09c 6\uc6d4 \ucd5c \uc528\uac00 \ub85d\ud788\ub4dc\ub9c8\ud2f4 \ud68c\uc7a5\uacfc \ub9cc\ub09c \uac83\uc73c\ub85c \uc54c\uace0 \uc788\ub2e4\u201d\uba70 \uc0ac\ub4dc(THAAD) \ub3c4\uc785\uc5d0 \ucd5c \uc528\uac00 \uac1c\uc785\ud588\ub2e4\uace0 \ucd94\uce21\ud558\uc600\ub2e4.", "paragraph_id": "6486360-43-0", "paragraph_question": "question: \ud55c\uad6d\uc758 \uc0ac\ub4dc \ubc30\uce58\uc5d0 \ucd5c\uc21c\uc2e4\uc774 \uac1c\uc785\ud588\uc74c\uc744 \ucd94\uce21\ud55c \uc778\ubb3c\uc740?, context: \ud55c\uaca8\ub808 \uc0ac\uc124\uc740 \"\ud30c\uba74\ub41c \ubc15\uadfc\ud61c \uc804 \ub300\ud1b5\ub839\uc744 \ubcf4\uc88c \ud558\ub358 \ucc38\ubaa8\ub85c\uc368, \uae40\uad00\uc9c4 \uc548\ubcf4 \uc2e4\uc7a5\uacfc \ud669\uad50\uc548 \ucd1d\ub9ac\uac00, \uc548\ubcf4\uc0c1\ud669\uc744 \uc720\uc9c0 \ubc0f \uad00\ub9ac\ub97c \ud558\ub294 \uc218\uc900\uc744 \ub118\uc5b4\uc11c\uc11c, \uacb0\uc815\uc790 \ud589\uc138\ub97c \ud55c\uac83\uc740 \uad8c\ub825 \ub0a8\uc6a9\uc774\uba70, \uc6d4\uad8c \ud589\uc704\ub2e4\"\ub77c\uba70 \ube44\ud310\uc744 \ud558\uc600\ub2e4. \ub1cc\ubb3c \uc218\ub8b0 \ud610\uc758\ub4f1\uc73c\ub85c \ud0c4\ud575\ub41c \ubc15\uadfc\ud61c \uc815\ubd80\uc5d0\uc11c\ub294, \uad6d\ubc29\ubd80\uc5d0\uc11c\ub294 2016\ub144 7\uc6d4 5\uc77c \uc0ac\ub4dc\uc2dc\uc2a4\ud15c\uc740 \uacb0\uc815\ub41c \ubc14\uac00 \uc5c6\uace0 \uc544\uc9c1 \uc2e4\ubb34 \uac80\ud1a0\uc911\uc774\ub77c\ub294 \ubcf4\ub3c4\ub97c \uacf5\uc2dd\uc801\uc73c\ub85c \ubc1c\ud45c\ub41c\uc9c0, \uc774\ud2c0\ud6c4 \uc778 7\uc6d4 7\uc77c \uac11\uc790\uae30 \uccad\uc640\ub300 \uc694\uccad\uc73c\ub85c \uad6d\uac00 \uc548\uc804 \ubcf4\uc7a5 \ud68c\uc758\ub97c \uc18c\uc9d1\ub418\uace0 \uc774\uacf3\uc5d0\uc11c \uacb0\uc815\ub418\uc5c8\ub294\ub370, \uc774\ub54c \uc8fc\ubb34 \ubd80\ucc98\uc778 \uad6d\ubc29 \uc7a5\uad00\uc774 \ubd88\ucc38\ud558\uace0, \ucc28\uad00\uc774 \ub300\uc2e0 \ucc38\uac00\ud55c \uc0c1\ud0dc\uc5d0\uc11c \uc9c4\ud589\ub418\uc5c8\uace0, \uc548\uac74\uc5ed\uc2dc \uad00\ub828 \ubd80\ucc98\uc778 \uad6d\ubc29\ubd80\uc640 \uc678\uad50\ubd80\uac00 \uc544\ub2cc \uae40\uad00\uc9c4 \uccad\uc640\ub300 \uc548\ubcf4 \uc2e4\uc7a5\uc774 \uccad\uc640\ub300\uc758 \uc758\uc9c0\uc778, \"\uc815\ubb34\uc801 \ud310\ub2e8\"\uc774\ub77c\uace0\ud558\uba74\uc11c, \uc989\uc11d \uc548\uac74\uc73c\ub85c \uc0ac\ub4dc \ubc30\uce58\ub97c \uc0c1\uc815\ud558\uc5ec \ud1b5\uacfc \ub418\uc5c8\ub2e4\ub294 \uc8fc\uc7a5\uc774 \uc788\ub2e4. \uc989 \uc0ac\ub4dc \uc2dc\uc2a4\ud15c\uc774 \ud544\uc694\uc131 \ubc0f \uc801\ud569\uc131\uc5d0 \ub300\ud574\uc11c\ub294 \uc0ac\uc2e4\uc0c1 \uccad\uc640\ub300 \uc548\ubcf4\uc2e4\uacfc \uc8fc\ud55c\ubbf8\uad70\uc774 \uad6d\ubc29\ubd80\ub97c \uac70\uce58\uc9c0 \uc54a\uace0 \uc9c1\uc811 \uc811\ucd09\ud55c \uac83\uc774\uba70, \uc774\ud6c4\uc5d0 \uad6d\ubc29\ubd80\ub294 \uc774\uac83\uc744 \uc704\ud55c \uc218\uc2b5\uc73c\ub85c \uad70\uc0ac\uc801 \ud569\ub9ac\uc131\uc774 \uace0\ub824\ub418\uc9c0 \uc54a\uc740 \uacb0\uc815\uc774\uc5c8\ub2e4\ub294 \uc8fc\uc7a5\uc774 \uc788\ub2e4. \uc548\ubbfc\uc11d \ub354\ubd88\uc5b4\ubbfc\uc8fc\ub2f9 \uc758\uc6d0\uc740 2016\ub144 11\uc6d4 24\uc77c \u2018\uae40\uc5b4\uc900\uc758 \ub274\uc2a4\uacf5\uc7a5\u2019\uc5d0 \ucd9c\uc5f0\ud574 \u201c\uc9c0\ub09c 6\uc6d4 \ucd5c \uc528\uac00 \ub85d\ud788\ub4dc\ub9c8\ud2f4 \ud68c\uc7a5\uacfc \ub9cc\ub09c \uac83\uc73c\ub85c \uc54c\uace0 \uc788\ub2e4\u201d\uba70 \uc0ac\ub4dc(THAAD) \ub3c4\uc785\uc5d0 \ucd5c \uc528\uac00 \uac1c\uc785\ud588\ub2e4\uace0 \ucd94\uce21\ud558\uc600\ub2e4. \ub610\ud55c \uc548 \uc758\uc6d0\uc740 \u201c\ucd5c \uc528\uc640 \ub85d\ud788\ub4dc\ub9c8\ud2f4\uc744 \uc5f0\uacb0\ud574\uc900 \uc778\ubb3c\uc740 \ud604 \uc815\uad8c\uc758 \uc678\uad50\uc548\ubcf4\ubd84\uc57c \uc2e4\uc138\u201d\ub77c\uba70 \uc774 \uc778\ubb3c\uc774 6\ub144 \uc804 \ubbf8\uad6d \uccb4\ub958 \ub2f9\uc2dc \ub85d\ud788\ub4dc\ub9c8\ud2f4\uc5d0\uc11c 2\ub144\uac04 \uccb4\ub958\ube44\ub97c \uc9c0\uc6d0\ud574\uc92c\ub2e4\uace0 \uc8fc\uc7a5\ud588\ub2e4. \ub610 \uc548 \uc758\uc6d0\uc740\u201c\ucd5c\uc21c\uc2e4\uc774 \uc9c0\uae08\uae4c\uc9c0 \ud574\uba39\uc740 \uac83\uc740 \uc0ac\ub4dc\uc640 \ube44\uad50\ud558\uba74 \uaecc\uac12\u201d\uc774\ub77c\uace0 \ucd94\ub860\ud558\uba70 \uc0ac\ub4dc\ub85c \uc778\ud574 \uc774\uc775\uc744 \ubcf8 \uc218\ub9ce\uc740 \uad00\uacc4\uc790\ub4e4\uc744 \uc0c1\uc0c1\ud558\ub3c4\ub85d \uc120\ub3d9\ud588\ub2e4. \uc55e\uc11c 11\uc6d4 15\uc77c \uc548 \uc758\uc6d0\uc740 \u201c\uc544\uc8fc \uc2e0\ub8b0\ud560 \ub9cc\ud55c \uad70 \ub0b4\ubd80\uc5d0\uc11c \uc81c\ubcf4\ub97c \ubc1b\uc558\ub2e4\u201d\uba70 \u201c2010~2015\ub144\uc5d0 \ube44\ud574 2015~2021\ub144 \ub85d\ud788\ub4dc\ub9c8\ud2f4 \uacc4\uc57d \uccb4\uacb0\uc774 \ub208\uc5d0 \ub744\uac8c \ub298\uc5c8\ub2e4\u201d\uace0 \uc9c4\uc0c1\uaddc\uba85\uc744 \uc8fc\uc7a5\ud588\ub2e4. \ud574\ub2f9 \uc758\ud639\uc740, \u201c2016\ub144 6\uc6d4\uc5d0 \ucd5c\uc21c\uc2e4\uc774 \ub85d\ud788\ub4dc\ub9c8\ud2f4 \ub9cc\ub098\uace0, 7\uc6d4\uc5d0 \uad00\ub828 \ubd80\ucc98\uc778 \uad6d\ubc29 \uc7a5\uad00\ub3c4 \ubb34\uc2dc\ub97c \ud558\uace0, \uc0ac\ub4dc\uac00 \uacb0\uc815\ub418\uc5c8\uace0...\uc774\uc81c \ud55c\uc77c\uc815\ubcf4\ubcf4\ud638\ud611\uc815\uc73c\ub85c \uc0ac\ub4dc \uc815\ubcf4\uac00 \uc77c\ubcf8\uc5d0\uac8c \uc81c\uacf5\ub418\uba74, \uc774\uac74 \uc644\uc804 \ub098\ub77c\ub97c \ud314\uc544\uba39\uc740 \uac83\u201d\uc774\ub77c\uace0 \ucd94\uce21\ud588\ub2e4. \ub610\ud55c \u201c\uc9c0\ub09c\ub2ec \ub3cc\ub358 \uc815\ubcf4\uc9c0\uac00 \uc0ac\uc2e4\uc774 \ub9de\ub294\uac74\uac00? \uc6d0\ub798 \ubb34\uae30\uac70\ub798 \uc815\uc724\ud68c\uac00 \ud558\ub358 \uc77c\uc774\uc5c8\ub294\ub370 \ucd5c\uc21c\uc2e4\uc774 \ub9b0\ub2e4\uae40\uc5d0\uac8c \uc218\uc870\uc6d0\uc758 \ub9ac\ubca0\uc774\ud2b8 \ucc59\uacbc\ub2e4\ub294 \ub9d0\uacfc \ub85d\ud788\ub4dc\ub9c8\ud2f4\uc774 \ud5a5\ud6c4 10\ub144\uac04 \uad6d\ub0b4\uc5d0\uc11c \ubc1b\ub294 \ubb34\uae30\ub300\uae08\uc740 66\uc870 \uc778\ub370 \uc774\uac74\uc5d0 \ub300\ud55c \ucd5c\uc21c\uc2e4\uc758 \uac1c\uc785\uc740 \ub0b4\ub780\uc8c4\u201d\ub77c\uace0 \ucd94\uce21\ud588\ub2e4."} {"question": "19\ub300 \ub300\uc120\uc5d0\uc11c \ubb38\uc7ac\uc778 \ud6c4\ubcf4\ub294 \ud328\uad8c\uc8fc\uc758\uc774\ub77c\uba70 \ube44\ud310\ud55c \uc0ac\ub78c\uc740 \ub204\uad6c\uc778\uac00?", "paragraph": "1\uc6d4 3\uc77c \ub354\ubd88\uc5b4\ubbfc\uc8fc\ub2f9\uc758 \uc0b0\ud558\uc5f0\uad6c\uae30\uad00\uc778 \ubbfc\uc8fc\uc5f0\uad6c\uc6d0\uc758 \uac1c\ud5cc\uc800\uc9c0\ubcf4\uace0\uc11c \uc0ac\uac74\uc73c\ub85c \ubbfc\uc8fc\ub2f9 \ub0b4 \uacc4\ud30c \uac08\ub4f1\uc774 \uc2ec\ud654\ub418\uc790 \ubc15\uc6d0\uc21c \uc11c\uc6b8\uc2dc\uc7a5\uc740 \ubb38\uc7ac\uc778 \ud6c4\ubcf4\ub97c \ud328\uad8c\uc8fc\uc758\uc758 \uc0c1\uc9d5\uc774\uba70 \uccad\uc0b0\ud574\uc57c \ud560 \uae30\ub4dd\uad8c \uc138\ub825\uc774\ub77c\uba70 \ube44\ud310\ud558\uc600\uace0, \uc548\ud76c\uc815 \ucda9\ub0a8\uc9c0\uc0ac\ub294 \"\ub2f9\uc5d0\uc11c \uac00\uc7a5 \ud070 \uc601\ud5a5\ub825\uc744 \uac00\uc9c4 \ubb38 \uc804 \ub300\ud45c\uc5d0\uac8c \ubb38\uc81c\uc758\uc2dd\uc744 \ubcf4\uc774\uace0 \ubb38\uc81c\uc81c\uae30\ub97c \ud558\ub294 \uac83\ub9c8\uc800 \ub108\ubb34 \ube44\ud310\ud560 \uc218\ub294 \uc5c6\ub2e4\"\ub294 \uc758\uacac\uc744 \ubc1d\ud614\ub2e4. \uac1c\ud5cc\uc5d0 \ucc2c\uc131\ud588\ub358 \uae40\ubd80\uacb8 \uc758\uc6d0\uc774 \uac1c\ud5cc\uc800\uc9c0\ubcf4\uace0\uc11c\uc5d0 \ub300\ud574 \ud3b8\ud5a5\uc801\uc774\uace0 \ub2f9\uc758 \ud1b5\ud569\uacfc \ub2e8\uacb0\uc744 \uc640\ud574\uc2dc\ud0a4\ub294 \ud589\uc704\ub77c \ube44\ud310\ud558\uc790 \ubb38\uc7ac\uc778 \uc9c0\uc9c0\uc790\ub4e4\ub85c\ubd80\ud130\uc758 \ud56d\uc758\uc131 \ubb38\uc790\uac00 \uc804\uc1a1\ub418\uc5c8\ub2e4. \uc774\uc7ac\uba85 \uc131\ub0a8\uc2dc\uc7a5\uc740 \uc774\uc5d0 \ub300\ud574 \ub2f9\uc744 \ub9dd\uce58\uace0 \ubbfc\uc8fc\uc8fc\uc758\ub97c \ud30c\uad34\ud558\ub294 \ud589\uc704\ub77c \ube44\ud310\ud558\uc600\ub2e4. \ucd5c\uc131 \uace0\uc591\uc2dc\uc7a5\uc740 \uc815\uacc4\uac1c\ud3b8\uacfc \uac1c\ud5cc\uc774 \uc815\uce58\uad8c\uc758 \uc774\ud574\uad00\uacc4\uc5d0 \ub530\ub77c\uc11c\ub9cc \uc774\uc57c\uae30 \ub418\ub294 \uc0c1\ud669\uc744 \ube44\ud310\ud558\uba70 \uc815\uce58\uac1c\ud601\uacfc \uc790\uce58\ubd84\uad8c \uad6d\uac00, \uacf5\uc815\ud55c \uad6d\uac00, \uc815\uc758\ub85c\uc6b4 \uad6d\uac00, \uccad\ub834\ud55c \uad6d\uac00\ub97c \ub9cc\ub4e4\uaca0\ub2e4\ub294 \uc758\uacac\uc744 \ubc1d\ud614\ub2e4.", "answer": "\ubc15\uc6d0\uc21c", "sentence": "1\uc6d4 3\uc77c \ub354\ubd88\uc5b4\ubbfc\uc8fc\ub2f9\uc758 \uc0b0\ud558\uc5f0\uad6c\uae30\uad00\uc778 \ubbfc\uc8fc\uc5f0\uad6c\uc6d0\uc758 \uac1c\ud5cc\uc800\uc9c0\ubcf4\uace0\uc11c \uc0ac\uac74\uc73c\ub85c \ubbfc\uc8fc\ub2f9 \ub0b4 \uacc4\ud30c \uac08\ub4f1\uc774 \uc2ec\ud654\ub418\uc790 \ubc15\uc6d0\uc21c \uc11c\uc6b8\uc2dc\uc7a5\uc740 \ubb38\uc7ac\uc778 \ud6c4\ubcf4\ub97c \ud328\uad8c\uc8fc\uc758\uc758 \uc0c1\uc9d5\uc774\uba70 \uccad\uc0b0\ud574\uc57c \ud560 \uae30\ub4dd\uad8c \uc138\ub825\uc774\ub77c\uba70 \ube44\ud310\ud558\uc600\uace0, \uc548\ud76c\uc815 \ucda9\ub0a8\uc9c0\uc0ac\ub294 \"\ub2f9\uc5d0\uc11c \uac00\uc7a5 \ud070 \uc601\ud5a5\ub825\uc744 \uac00\uc9c4 \ubb38 \uc804 \ub300\ud45c\uc5d0\uac8c \ubb38\uc81c\uc758\uc2dd\uc744 \ubcf4\uc774\uace0 \ubb38\uc81c\uc81c\uae30\ub97c \ud558\ub294 \uac83\ub9c8\uc800 \ub108\ubb34 \ube44\ud310\ud560 \uc218\ub294 \uc5c6\ub2e4\"\ub294 \uc758\uacac\uc744 \ubc1d\ud614\ub2e4.", "paragraph_sentence": " 1\uc6d4 3\uc77c \ub354\ubd88\uc5b4\ubbfc\uc8fc\ub2f9\uc758 \uc0b0\ud558\uc5f0\uad6c\uae30\uad00\uc778 \ubbfc\uc8fc\uc5f0\uad6c\uc6d0\uc758 \uac1c\ud5cc\uc800\uc9c0\ubcf4\uace0\uc11c \uc0ac\uac74\uc73c\ub85c \ubbfc\uc8fc\ub2f9 \ub0b4 \uacc4\ud30c \uac08\ub4f1\uc774 \uc2ec\ud654\ub418\uc790 \ubc15\uc6d0\uc21c \uc11c\uc6b8\uc2dc\uc7a5\uc740 \ubb38\uc7ac\uc778 \ud6c4\ubcf4\ub97c \ud328\uad8c\uc8fc\uc758\uc758 \uc0c1\uc9d5\uc774\uba70 \uccad\uc0b0\ud574\uc57c \ud560 \uae30\ub4dd\uad8c \uc138\ub825\uc774\ub77c\uba70 \ube44\ud310\ud558\uc600\uace0, \uc548\ud76c\uc815 \ucda9\ub0a8\uc9c0\uc0ac\ub294 \"\ub2f9\uc5d0\uc11c \uac00\uc7a5 \ud070 \uc601\ud5a5\ub825\uc744 \uac00\uc9c4 \ubb38 \uc804 \ub300\ud45c\uc5d0\uac8c \ubb38\uc81c\uc758\uc2dd\uc744 \ubcf4\uc774\uace0 \ubb38\uc81c\uc81c\uae30\ub97c \ud558\ub294 \uac83\ub9c8\uc800 \ub108\ubb34 \ube44\ud310\ud560 \uc218\ub294 \uc5c6\ub2e4\"\ub294 \uc758\uacac\uc744 \ubc1d\ud614\ub2e4. \uac1c\ud5cc\uc5d0 \ucc2c\uc131\ud588\ub358 \uae40\ubd80\uacb8 \uc758\uc6d0\uc774 \uac1c\ud5cc\uc800\uc9c0\ubcf4\uace0\uc11c\uc5d0 \ub300\ud574 \ud3b8\ud5a5\uc801\uc774\uace0 \ub2f9\uc758 \ud1b5\ud569\uacfc \ub2e8\uacb0\uc744 \uc640\ud574\uc2dc\ud0a4\ub294 \ud589\uc704\ub77c \ube44\ud310\ud558\uc790 \ubb38\uc7ac\uc778 \uc9c0\uc9c0\uc790\ub4e4\ub85c\ubd80\ud130\uc758 \ud56d\uc758\uc131 \ubb38\uc790\uac00 \uc804\uc1a1\ub418\uc5c8\ub2e4. \uc774\uc7ac\uba85 \uc131\ub0a8\uc2dc\uc7a5\uc740 \uc774\uc5d0 \ub300\ud574 \ub2f9\uc744 \ub9dd\uce58\uace0 \ubbfc\uc8fc\uc8fc\uc758\ub97c \ud30c\uad34\ud558\ub294 \ud589\uc704\ub77c \ube44\ud310\ud558\uc600\ub2e4. \ucd5c\uc131 \uace0\uc591\uc2dc\uc7a5\uc740 \uc815\uacc4\uac1c\ud3b8\uacfc \uac1c\ud5cc\uc774 \uc815\uce58\uad8c\uc758 \uc774\ud574\uad00\uacc4\uc5d0 \ub530\ub77c\uc11c\ub9cc \uc774\uc57c\uae30 \ub418\ub294 \uc0c1\ud669\uc744 \ube44\ud310\ud558\uba70 \uc815\uce58\uac1c\ud601\uacfc \uc790\uce58\ubd84\uad8c \uad6d\uac00, \uacf5\uc815\ud55c \uad6d\uac00, \uc815\uc758\ub85c\uc6b4 \uad6d\uac00, \uccad\ub834\ud55c \uad6d\uac00\ub97c \ub9cc\ub4e4\uaca0\ub2e4\ub294 \uc758\uacac\uc744 \ubc1d\ud614\ub2e4.", "paragraph_answer": "1\uc6d4 3\uc77c \ub354\ubd88\uc5b4\ubbfc\uc8fc\ub2f9\uc758 \uc0b0\ud558\uc5f0\uad6c\uae30\uad00\uc778 \ubbfc\uc8fc\uc5f0\uad6c\uc6d0\uc758 \uac1c\ud5cc\uc800\uc9c0\ubcf4\uace0\uc11c \uc0ac\uac74\uc73c\ub85c \ubbfc\uc8fc\ub2f9 \ub0b4 \uacc4\ud30c \uac08\ub4f1\uc774 \uc2ec\ud654\ub418\uc790 \ubc15\uc6d0\uc21c \uc11c\uc6b8\uc2dc\uc7a5\uc740 \ubb38\uc7ac\uc778 \ud6c4\ubcf4\ub97c \ud328\uad8c\uc8fc\uc758\uc758 \uc0c1\uc9d5\uc774\uba70 \uccad\uc0b0\ud574\uc57c \ud560 \uae30\ub4dd\uad8c \uc138\ub825\uc774\ub77c\uba70 \ube44\ud310\ud558\uc600\uace0, \uc548\ud76c\uc815 \ucda9\ub0a8\uc9c0\uc0ac\ub294 \"\ub2f9\uc5d0\uc11c \uac00\uc7a5 \ud070 \uc601\ud5a5\ub825\uc744 \uac00\uc9c4 \ubb38 \uc804 \ub300\ud45c\uc5d0\uac8c \ubb38\uc81c\uc758\uc2dd\uc744 \ubcf4\uc774\uace0 \ubb38\uc81c\uc81c\uae30\ub97c \ud558\ub294 \uac83\ub9c8\uc800 \ub108\ubb34 \ube44\ud310\ud560 \uc218\ub294 \uc5c6\ub2e4\"\ub294 \uc758\uacac\uc744 \ubc1d\ud614\ub2e4. \uac1c\ud5cc\uc5d0 \ucc2c\uc131\ud588\ub358 \uae40\ubd80\uacb8 \uc758\uc6d0\uc774 \uac1c\ud5cc\uc800\uc9c0\ubcf4\uace0\uc11c\uc5d0 \ub300\ud574 \ud3b8\ud5a5\uc801\uc774\uace0 \ub2f9\uc758 \ud1b5\ud569\uacfc \ub2e8\uacb0\uc744 \uc640\ud574\uc2dc\ud0a4\ub294 \ud589\uc704\ub77c \ube44\ud310\ud558\uc790 \ubb38\uc7ac\uc778 \uc9c0\uc9c0\uc790\ub4e4\ub85c\ubd80\ud130\uc758 \ud56d\uc758\uc131 \ubb38\uc790\uac00 \uc804\uc1a1\ub418\uc5c8\ub2e4. \uc774\uc7ac\uba85 \uc131\ub0a8\uc2dc\uc7a5\uc740 \uc774\uc5d0 \ub300\ud574 \ub2f9\uc744 \ub9dd\uce58\uace0 \ubbfc\uc8fc\uc8fc\uc758\ub97c \ud30c\uad34\ud558\ub294 \ud589\uc704\ub77c \ube44\ud310\ud558\uc600\ub2e4. \ucd5c\uc131 \uace0\uc591\uc2dc\uc7a5\uc740 \uc815\uacc4\uac1c\ud3b8\uacfc \uac1c\ud5cc\uc774 \uc815\uce58\uad8c\uc758 \uc774\ud574\uad00\uacc4\uc5d0 \ub530\ub77c\uc11c\ub9cc \uc774\uc57c\uae30 \ub418\ub294 \uc0c1\ud669\uc744 \ube44\ud310\ud558\uba70 \uc815\uce58\uac1c\ud601\uacfc \uc790\uce58\ubd84\uad8c \uad6d\uac00, \uacf5\uc815\ud55c \uad6d\uac00, \uc815\uc758\ub85c\uc6b4 \uad6d\uac00, \uccad\ub834\ud55c \uad6d\uac00\ub97c \ub9cc\ub4e4\uaca0\ub2e4\ub294 \uc758\uacac\uc744 \ubc1d\ud614\ub2e4.", "sentence_answer": "1\uc6d4 3\uc77c \ub354\ubd88\uc5b4\ubbfc\uc8fc\ub2f9\uc758 \uc0b0\ud558\uc5f0\uad6c\uae30\uad00\uc778 \ubbfc\uc8fc\uc5f0\uad6c\uc6d0\uc758 \uac1c\ud5cc\uc800\uc9c0\ubcf4\uace0\uc11c \uc0ac\uac74\uc73c\ub85c \ubbfc\uc8fc\ub2f9 \ub0b4 \uacc4\ud30c \uac08\ub4f1\uc774 \uc2ec\ud654\ub418\uc790 \ubc15\uc6d0\uc21c \uc11c\uc6b8\uc2dc\uc7a5\uc740 \ubb38\uc7ac\uc778 \ud6c4\ubcf4\ub97c \ud328\uad8c\uc8fc\uc758\uc758 \uc0c1\uc9d5\uc774\uba70 \uccad\uc0b0\ud574\uc57c \ud560 \uae30\ub4dd\uad8c \uc138\ub825\uc774\ub77c\uba70 \ube44\ud310\ud558\uc600\uace0, \uc548\ud76c\uc815 \ucda9\ub0a8\uc9c0\uc0ac\ub294 \"\ub2f9\uc5d0\uc11c \uac00\uc7a5 \ud070 \uc601\ud5a5\ub825\uc744 \uac00\uc9c4 \ubb38 \uc804 \ub300\ud45c\uc5d0\uac8c \ubb38\uc81c\uc758\uc2dd\uc744 \ubcf4\uc774\uace0 \ubb38\uc81c\uc81c\uae30\ub97c \ud558\ub294 \uac83\ub9c8\uc800 \ub108\ubb34 \ube44\ud310\ud560 \uc218\ub294 \uc5c6\ub2e4\"\ub294 \uc758\uacac\uc744 \ubc1d\ud614\ub2e4.", "paragraph_id": "6486153-1-0", "paragraph_question": "question: 19\ub300 \ub300\uc120\uc5d0\uc11c \ubb38\uc7ac\uc778 \ud6c4\ubcf4\ub294 \ud328\uad8c\uc8fc\uc758\uc774\ub77c\uba70 \ube44\ud310\ud55c \uc0ac\ub78c\uc740 \ub204\uad6c\uc778\uac00?, context: 1\uc6d4 3\uc77c \ub354\ubd88\uc5b4\ubbfc\uc8fc\ub2f9\uc758 \uc0b0\ud558\uc5f0\uad6c\uae30\uad00\uc778 \ubbfc\uc8fc\uc5f0\uad6c\uc6d0\uc758 \uac1c\ud5cc\uc800\uc9c0\ubcf4\uace0\uc11c \uc0ac\uac74\uc73c\ub85c \ubbfc\uc8fc\ub2f9 \ub0b4 \uacc4\ud30c \uac08\ub4f1\uc774 \uc2ec\ud654\ub418\uc790 \ubc15\uc6d0\uc21c \uc11c\uc6b8\uc2dc\uc7a5\uc740 \ubb38\uc7ac\uc778 \ud6c4\ubcf4\ub97c \ud328\uad8c\uc8fc\uc758\uc758 \uc0c1\uc9d5\uc774\uba70 \uccad\uc0b0\ud574\uc57c \ud560 \uae30\ub4dd\uad8c \uc138\ub825\uc774\ub77c\uba70 \ube44\ud310\ud558\uc600\uace0, \uc548\ud76c\uc815 \ucda9\ub0a8\uc9c0\uc0ac\ub294 \"\ub2f9\uc5d0\uc11c \uac00\uc7a5 \ud070 \uc601\ud5a5\ub825\uc744 \uac00\uc9c4 \ubb38 \uc804 \ub300\ud45c\uc5d0\uac8c \ubb38\uc81c\uc758\uc2dd\uc744 \ubcf4\uc774\uace0 \ubb38\uc81c\uc81c\uae30\ub97c \ud558\ub294 \uac83\ub9c8\uc800 \ub108\ubb34 \ube44\ud310\ud560 \uc218\ub294 \uc5c6\ub2e4\"\ub294 \uc758\uacac\uc744 \ubc1d\ud614\ub2e4. \uac1c\ud5cc\uc5d0 \ucc2c\uc131\ud588\ub358 \uae40\ubd80\uacb8 \uc758\uc6d0\uc774 \uac1c\ud5cc\uc800\uc9c0\ubcf4\uace0\uc11c\uc5d0 \ub300\ud574 \ud3b8\ud5a5\uc801\uc774\uace0 \ub2f9\uc758 \ud1b5\ud569\uacfc \ub2e8\uacb0\uc744 \uc640\ud574\uc2dc\ud0a4\ub294 \ud589\uc704\ub77c \ube44\ud310\ud558\uc790 \ubb38\uc7ac\uc778 \uc9c0\uc9c0\uc790\ub4e4\ub85c\ubd80\ud130\uc758 \ud56d\uc758\uc131 \ubb38\uc790\uac00 \uc804\uc1a1\ub418\uc5c8\ub2e4. \uc774\uc7ac\uba85 \uc131\ub0a8\uc2dc\uc7a5\uc740 \uc774\uc5d0 \ub300\ud574 \ub2f9\uc744 \ub9dd\uce58\uace0 \ubbfc\uc8fc\uc8fc\uc758\ub97c \ud30c\uad34\ud558\ub294 \ud589\uc704\ub77c \ube44\ud310\ud558\uc600\ub2e4. \ucd5c\uc131 \uace0\uc591\uc2dc\uc7a5\uc740 \uc815\uacc4\uac1c\ud3b8\uacfc \uac1c\ud5cc\uc774 \uc815\uce58\uad8c\uc758 \uc774\ud574\uad00\uacc4\uc5d0 \ub530\ub77c\uc11c\ub9cc \uc774\uc57c\uae30 \ub418\ub294 \uc0c1\ud669\uc744 \ube44\ud310\ud558\uba70 \uc815\uce58\uac1c\ud601\uacfc \uc790\uce58\ubd84\uad8c \uad6d\uac00, \uacf5\uc815\ud55c \uad6d\uac00, \uc815\uc758\ub85c\uc6b4 \uad6d\uac00, \uccad\ub834\ud55c \uad6d\uac00\ub97c \ub9cc\ub4e4\uaca0\ub2e4\ub294 \uc758\uacac\uc744 \ubc1d\ud614\ub2e4."} {"question": "\uc18c\ube44\uc5d0\ud2b8 \ub0b4\ubb34\uc778\ubbfc\uc704\uc6d0\ud68c\uac00 \uc18c\ub828 \uc804\uc2dc \uc7ac\ud32c\uc5d0 \ub118\uae34 \ud3f4\ub780\ub4dc \ubc18\uccb4\uc81c \uc778\uc0ac \uc218\ub294?", "paragraph": "\ud3f4\ub780\ub4dc \uc778\ubbfc\ud574\ubc29\uc704\uc6d0\ud68c\ub294 \uc790\uc2e0\ub4e4\uc774 \ud3f4\ub780\ub4dc \uc0ac\ud68c\ub97c \ub300\ud45c\ud558\ub294 \uc9c0\uc5ed \uc790\uc0dd\uc758 \ub2e8\uccb4\ub85c \uac00\uc7a5\ud558\uba74\uc11c, \uc815\ubd80 \ub2f9\uad6d\uc758 \uc5ed\ud560\uc744 \uac00\uc9c0\uace0 \uc81c2\ucc28 \uc138\uacc4 \ub300\uc804 \uc774\uc804 \ub7f0\ub358\uc5d0 \uc788\ub358 \ud3f4\ub780\ub4dc \ub9dd\uba85 \uc815\ubd80\uc5d0 \ub3c4\uc804\ud558\uc600\ub2e4. \uc584\ud0c0 \ud68c\ub2f4\uc5d0\uc11c \uc57d\uc18d\ud55c \"\uc790\uc720\ub86d\uace0 \uacf5\uba85\uc815\ub300\ud55c \uc120\uac70\"\uac00 \uce58\ub7ec\uc9c8\uc9c0\uc5d0 \ub300\ud55c \uc758\uc2ec\uc774 \uc810\ucc28 \ucee4\uc9c0\uac8c \ub418\uc5c8\ub2e4. \ub098\uce58\uc5d0 \uc800\ud56d\ud55c \uc138\ub825\uc744 \ube44\ub86f\ud558\uc5ec \ube44\uacf5\uc0b0\uc8fc\uc758\uc790\uc640 \ube68\uce58\uc0b0\uc740 \uc870\uc9c1\uc801\uc73c\ub85c \ucc98\ud615\ub418\uc5c8\ub2e4. \uc778\ubbfc \ud574\ubc29 \uc704\uc6d0\ud68c\uac00 \uc790\uc2e0\ub4e4\uc774 \uc815\ubd80\uc5d0 \ucc38\uc5ec\uc2dc\ud0a4\uae38 \uc6d0\ud558\ub294 \uc0ac\ub78c\uc744 \uace0\ub97c \uc218 \uc788\ub294 \uad8c\ub9ac\uac00 \uc788\ub2e4\uace0 \uc8fc\uc7a5\ud558\uc790, \uc0c8\ub85c\uc774 \uc790\uc720\ub86d\uac8c \uc815\ubd80\uac00 \ub4e4\uc5b4\uc11c\ub9ac\ub77c\ub294 \uae30\ub300\ub294 \ubb3c\uac70\ud488\uc774 \ub418\uc5c8\uace0, \uc18c\ube44\uc5d0\ud2b8\uc758 \ub0b4\ubb34\uc778\ubbfc\uc704\uc6d0\ud68c\ub294 1945\ub144 3\uc6d4 \uc815\ubd80 \uc7ac\uad6c\uc131 \ud611\uc0c1\uc5d0 \ucc38\uc5ec\ud558\ub824\ub294 \ud3f4\ub780\ub4dc \ubc18\uccb4\uc81c \uc778\uc0ac 16\uba85\uc744 \uc18c\ub828\uc73c\ub85c \uc7a1\uc544\uac00 \uadf8\ud574 6\uc6d4\uc5d0 \uc804\uc2dc \uc7ac\ud310\uc5d0 \ub118\uacbc\ub2e4.", "answer": "16\uba85", "sentence": "\uc778\ubbfc \ud574\ubc29 \uc704\uc6d0\ud68c\uac00 \uc790\uc2e0\ub4e4\uc774 \uc815\ubd80\uc5d0 \ucc38\uc5ec\uc2dc\ud0a4\uae38 \uc6d0\ud558\ub294 \uc0ac\ub78c\uc744 \uace0\ub97c \uc218 \uc788\ub294 \uad8c\ub9ac\uac00 \uc788\ub2e4\uace0 \uc8fc\uc7a5\ud558\uc790, \uc0c8\ub85c\uc774 \uc790\uc720\ub86d\uac8c \uc815\ubd80\uac00 \ub4e4\uc5b4\uc11c\ub9ac\ub77c\ub294 \uae30\ub300\ub294 \ubb3c\uac70\ud488\uc774 \ub418\uc5c8\uace0, \uc18c\ube44\uc5d0\ud2b8\uc758 \ub0b4\ubb34\uc778\ubbfc\uc704\uc6d0\ud68c\ub294 1945\ub144 3\uc6d4 \uc815\ubd80 \uc7ac\uad6c\uc131 \ud611\uc0c1\uc5d0 \ucc38\uc5ec\ud558\ub824\ub294 \ud3f4\ub780\ub4dc \ubc18\uccb4\uc81c \uc778\uc0ac 16\uba85 \uc744 \uc18c\ub828\uc73c\ub85c \uc7a1\uc544\uac00 \uadf8\ud574 6\uc6d4\uc5d0 \uc804\uc2dc \uc7ac\ud310\uc5d0 \ub118\uacbc\ub2e4.", "paragraph_sentence": "\ud3f4\ub780\ub4dc \uc778\ubbfc\ud574\ubc29\uc704\uc6d0\ud68c\ub294 \uc790\uc2e0\ub4e4\uc774 \ud3f4\ub780\ub4dc \uc0ac\ud68c\ub97c \ub300\ud45c\ud558\ub294 \uc9c0\uc5ed \uc790\uc0dd\uc758 \ub2e8\uccb4\ub85c \uac00\uc7a5\ud558\uba74\uc11c, \uc815\ubd80 \ub2f9\uad6d\uc758 \uc5ed\ud560\uc744 \uac00\uc9c0\uace0 \uc81c2\ucc28 \uc138\uacc4 \ub300\uc804 \uc774\uc804 \ub7f0\ub358\uc5d0 \uc788\ub358 \ud3f4\ub780\ub4dc \ub9dd\uba85 \uc815\ubd80\uc5d0 \ub3c4\uc804\ud558\uc600\ub2e4. \uc584\ud0c0 \ud68c\ub2f4\uc5d0\uc11c \uc57d\uc18d\ud55c \"\uc790\uc720\ub86d\uace0 \uacf5\uba85\uc815\ub300\ud55c \uc120\uac70\"\uac00 \uce58\ub7ec\uc9c8\uc9c0\uc5d0 \ub300\ud55c \uc758\uc2ec\uc774 \uc810\ucc28 \ucee4\uc9c0\uac8c \ub418\uc5c8\ub2e4. \ub098\uce58\uc5d0 \uc800\ud56d\ud55c \uc138\ub825\uc744 \ube44\ub86f\ud558\uc5ec \ube44\uacf5\uc0b0\uc8fc\uc758\uc790\uc640 \ube68\uce58\uc0b0\uc740 \uc870\uc9c1\uc801\uc73c\ub85c \ucc98\ud615\ub418\uc5c8\ub2e4. \uc778\ubbfc \ud574\ubc29 \uc704\uc6d0\ud68c\uac00 \uc790\uc2e0\ub4e4\uc774 \uc815\ubd80\uc5d0 \ucc38\uc5ec\uc2dc\ud0a4\uae38 \uc6d0\ud558\ub294 \uc0ac\ub78c\uc744 \uace0\ub97c \uc218 \uc788\ub294 \uad8c\ub9ac\uac00 \uc788\ub2e4\uace0 \uc8fc\uc7a5\ud558\uc790, \uc0c8\ub85c\uc774 \uc790\uc720\ub86d\uac8c \uc815\ubd80\uac00 \ub4e4\uc5b4\uc11c\ub9ac\ub77c\ub294 \uae30\ub300\ub294 \ubb3c\uac70\ud488\uc774 \ub418\uc5c8\uace0, \uc18c\ube44\uc5d0\ud2b8\uc758 \ub0b4\ubb34\uc778\ubbfc\uc704\uc6d0\ud68c\ub294 1945\ub144 3\uc6d4 \uc815\ubd80 \uc7ac\uad6c\uc131 \ud611\uc0c1\uc5d0 \ucc38\uc5ec\ud558\ub824\ub294 \ud3f4\ub780\ub4dc \ubc18\uccb4\uc81c \uc778\uc0ac 16\uba85 \uc744 \uc18c\ub828\uc73c\ub85c \uc7a1\uc544\uac00 \uadf8\ud574 6\uc6d4\uc5d0 \uc804\uc2dc \uc7ac\ud310\uc5d0 \ub118\uacbc\ub2e4. ", "paragraph_answer": "\ud3f4\ub780\ub4dc \uc778\ubbfc\ud574\ubc29\uc704\uc6d0\ud68c\ub294 \uc790\uc2e0\ub4e4\uc774 \ud3f4\ub780\ub4dc \uc0ac\ud68c\ub97c \ub300\ud45c\ud558\ub294 \uc9c0\uc5ed \uc790\uc0dd\uc758 \ub2e8\uccb4\ub85c \uac00\uc7a5\ud558\uba74\uc11c, \uc815\ubd80 \ub2f9\uad6d\uc758 \uc5ed\ud560\uc744 \uac00\uc9c0\uace0 \uc81c2\ucc28 \uc138\uacc4 \ub300\uc804 \uc774\uc804 \ub7f0\ub358\uc5d0 \uc788\ub358 \ud3f4\ub780\ub4dc \ub9dd\uba85 \uc815\ubd80\uc5d0 \ub3c4\uc804\ud558\uc600\ub2e4. \uc584\ud0c0 \ud68c\ub2f4\uc5d0\uc11c \uc57d\uc18d\ud55c \"\uc790\uc720\ub86d\uace0 \uacf5\uba85\uc815\ub300\ud55c \uc120\uac70\"\uac00 \uce58\ub7ec\uc9c8\uc9c0\uc5d0 \ub300\ud55c \uc758\uc2ec\uc774 \uc810\ucc28 \ucee4\uc9c0\uac8c \ub418\uc5c8\ub2e4. \ub098\uce58\uc5d0 \uc800\ud56d\ud55c \uc138\ub825\uc744 \ube44\ub86f\ud558\uc5ec \ube44\uacf5\uc0b0\uc8fc\uc758\uc790\uc640 \ube68\uce58\uc0b0\uc740 \uc870\uc9c1\uc801\uc73c\ub85c \ucc98\ud615\ub418\uc5c8\ub2e4. \uc778\ubbfc \ud574\ubc29 \uc704\uc6d0\ud68c\uac00 \uc790\uc2e0\ub4e4\uc774 \uc815\ubd80\uc5d0 \ucc38\uc5ec\uc2dc\ud0a4\uae38 \uc6d0\ud558\ub294 \uc0ac\ub78c\uc744 \uace0\ub97c \uc218 \uc788\ub294 \uad8c\ub9ac\uac00 \uc788\ub2e4\uace0 \uc8fc\uc7a5\ud558\uc790, \uc0c8\ub85c\uc774 \uc790\uc720\ub86d\uac8c \uc815\ubd80\uac00 \ub4e4\uc5b4\uc11c\ub9ac\ub77c\ub294 \uae30\ub300\ub294 \ubb3c\uac70\ud488\uc774 \ub418\uc5c8\uace0, \uc18c\ube44\uc5d0\ud2b8\uc758 \ub0b4\ubb34\uc778\ubbfc\uc704\uc6d0\ud68c\ub294 1945\ub144 3\uc6d4 \uc815\ubd80 \uc7ac\uad6c\uc131 \ud611\uc0c1\uc5d0 \ucc38\uc5ec\ud558\ub824\ub294 \ud3f4\ub780\ub4dc \ubc18\uccb4\uc81c \uc778\uc0ac 16\uba85 \uc744 \uc18c\ub828\uc73c\ub85c \uc7a1\uc544\uac00 \uadf8\ud574 6\uc6d4\uc5d0 \uc804\uc2dc \uc7ac\ud310\uc5d0 \ub118\uacbc\ub2e4.", "sentence_answer": "\uc778\ubbfc \ud574\ubc29 \uc704\uc6d0\ud68c\uac00 \uc790\uc2e0\ub4e4\uc774 \uc815\ubd80\uc5d0 \ucc38\uc5ec\uc2dc\ud0a4\uae38 \uc6d0\ud558\ub294 \uc0ac\ub78c\uc744 \uace0\ub97c \uc218 \uc788\ub294 \uad8c\ub9ac\uac00 \uc788\ub2e4\uace0 \uc8fc\uc7a5\ud558\uc790, \uc0c8\ub85c\uc774 \uc790\uc720\ub86d\uac8c \uc815\ubd80\uac00 \ub4e4\uc5b4\uc11c\ub9ac\ub77c\ub294 \uae30\ub300\ub294 \ubb3c\uac70\ud488\uc774 \ub418\uc5c8\uace0, \uc18c\ube44\uc5d0\ud2b8\uc758 \ub0b4\ubb34\uc778\ubbfc\uc704\uc6d0\ud68c\ub294 1945\ub144 3\uc6d4 \uc815\ubd80 \uc7ac\uad6c\uc131 \ud611\uc0c1\uc5d0 \ucc38\uc5ec\ud558\ub824\ub294 \ud3f4\ub780\ub4dc \ubc18\uccb4\uc81c \uc778\uc0ac 16\uba85 \uc744 \uc18c\ub828\uc73c\ub85c \uc7a1\uc544\uac00 \uadf8\ud574 6\uc6d4\uc5d0 \uc804\uc2dc \uc7ac\ud310\uc5d0 \ub118\uacbc\ub2e4.", "paragraph_id": "5924186-2-1", "paragraph_question": "question: \uc18c\ube44\uc5d0\ud2b8 \ub0b4\ubb34\uc778\ubbfc\uc704\uc6d0\ud68c\uac00 \uc18c\ub828 \uc804\uc2dc \uc7ac\ud32c\uc5d0 \ub118\uae34 \ud3f4\ub780\ub4dc \ubc18\uccb4\uc81c \uc778\uc0ac \uc218\ub294?, context: \ud3f4\ub780\ub4dc \uc778\ubbfc\ud574\ubc29\uc704\uc6d0\ud68c\ub294 \uc790\uc2e0\ub4e4\uc774 \ud3f4\ub780\ub4dc \uc0ac\ud68c\ub97c \ub300\ud45c\ud558\ub294 \uc9c0\uc5ed \uc790\uc0dd\uc758 \ub2e8\uccb4\ub85c \uac00\uc7a5\ud558\uba74\uc11c, \uc815\ubd80 \ub2f9\uad6d\uc758 \uc5ed\ud560\uc744 \uac00\uc9c0\uace0 \uc81c2\ucc28 \uc138\uacc4 \ub300\uc804 \uc774\uc804 \ub7f0\ub358\uc5d0 \uc788\ub358 \ud3f4\ub780\ub4dc \ub9dd\uba85 \uc815\ubd80\uc5d0 \ub3c4\uc804\ud558\uc600\ub2e4. \uc584\ud0c0 \ud68c\ub2f4\uc5d0\uc11c \uc57d\uc18d\ud55c \"\uc790\uc720\ub86d\uace0 \uacf5\uba85\uc815\ub300\ud55c \uc120\uac70\"\uac00 \uce58\ub7ec\uc9c8\uc9c0\uc5d0 \ub300\ud55c \uc758\uc2ec\uc774 \uc810\ucc28 \ucee4\uc9c0\uac8c \ub418\uc5c8\ub2e4. \ub098\uce58\uc5d0 \uc800\ud56d\ud55c \uc138\ub825\uc744 \ube44\ub86f\ud558\uc5ec \ube44\uacf5\uc0b0\uc8fc\uc758\uc790\uc640 \ube68\uce58\uc0b0\uc740 \uc870\uc9c1\uc801\uc73c\ub85c \ucc98\ud615\ub418\uc5c8\ub2e4. \uc778\ubbfc \ud574\ubc29 \uc704\uc6d0\ud68c\uac00 \uc790\uc2e0\ub4e4\uc774 \uc815\ubd80\uc5d0 \ucc38\uc5ec\uc2dc\ud0a4\uae38 \uc6d0\ud558\ub294 \uc0ac\ub78c\uc744 \uace0\ub97c \uc218 \uc788\ub294 \uad8c\ub9ac\uac00 \uc788\ub2e4\uace0 \uc8fc\uc7a5\ud558\uc790, \uc0c8\ub85c\uc774 \uc790\uc720\ub86d\uac8c \uc815\ubd80\uac00 \ub4e4\uc5b4\uc11c\ub9ac\ub77c\ub294 \uae30\ub300\ub294 \ubb3c\uac70\ud488\uc774 \ub418\uc5c8\uace0, \uc18c\ube44\uc5d0\ud2b8\uc758 \ub0b4\ubb34\uc778\ubbfc\uc704\uc6d0\ud68c\ub294 1945\ub144 3\uc6d4 \uc815\ubd80 \uc7ac\uad6c\uc131 \ud611\uc0c1\uc5d0 \ucc38\uc5ec\ud558\ub824\ub294 \ud3f4\ub780\ub4dc \ubc18\uccb4\uc81c \uc778\uc0ac 16\uba85\uc744 \uc18c\ub828\uc73c\ub85c \uc7a1\uc544\uac00 \uadf8\ud574 6\uc6d4\uc5d0 \uc804\uc2dc \uc7ac\ud310\uc5d0 \ub118\uacbc\ub2e4."} {"question": "\ub18d\uc2dd\ud488\ubd80\uc5d0\uc11c 1\ub9cc\ud1a4\ub2f9 \uc5f0\uac04 \uc18c\ubaa8\ub418\ub294 \uc300 \uc7ac\uace0\uad00\ub9ac\ube44\uc6a9\uc740 \uc5bc\ub9c8\uc778\uac00?", "paragraph": "\uadf8\ub7ec\ub098 \ub18d\uc2dd\ud488\ubd80\ub294 \ub2f9\uc2dc \ucf69 \uc7ac\ubc30 \uba74\uc801\uc774 \ub298\uc5b4 \uac00\uaca9\uc774 \ud558\ub77d\ud55c \uac83\uc774 \uc544\ub2c8\ub77c \ucf69 \uc800\uc728\uad00\uc138\ud560\ub2f9\ub7c9(TRQ)\uc744 \ub298\ub9b0 \uac83\uc774 \ubb38\uc81c\uc600\ub2e4\uace0 \ubc18\ubc15\ud588\ub2e4. \ub610 \uc300 \uc7ac\uace0\ub7c9 \uad00\ub9ac\ube44\ubcf4\ub2e4 \uc300 \uc0dd\uc0b0\uc870\uc815\uc81c\uc5d0 \ub4dc\ub294 \ube44\uc6a9\uc774 \ub354 \uc801\ub2e4\ub294 \uc785\uc7a5\uc774\ub2e4. \ub18d\uc2dd\ud488\ubd80\uc5d0 \ub530\ub974\uba74 1\u33ca\ub2f9 300\ub9cc\uc6d0\uc744 \uc9c0\uae09\ud574 2017\ub144\uc5d0 3\ub9cc\u33ca, 2018\ub144\uc5d0 6\ub9cc\u33ca\uc758 \ubcbc \uc7ac\ubc30\uba74\uc801\uc744 \uac10\ucd95\ud558\uba74 \ucd1d 45\ub9cc7200\ud1a4\uc758 \uc300 \uc0dd\uc0b0\uc744 \uc904\uc77c \uc218 \uc788\ub2e4. \uc774\ub807\uac8c \ub418\uba74 45\ub9cc7200\ud1a4\uc758 \uc300\uc744 \uc2dc\uc7a5\uaca9\ub9ac\ud560 \ub54c \uc18c\uc694\ub418\ub294 \ube44\uc6a9 3759\uc5b5\uc6d0\uc774 \uc808\uac10\ub41c\ub2e4. \ub610 \uc300\uac12 \uc548\uc815\uc73c\ub85c \uc300\ubcc0\ub3d9\uc9c1\ubd88\uae08 \uc9c0\uae09\uc561\uc774 2700\uc5b5\uc6d0 \uc904\uc5b4\ub4e4 \uac83\uc73c\ub85c \ubd84\uc11d\ub41c\ub2e4. \ub610 1\ub9cc\ud1a4\ub2f9 \uc5f0\uac04 \uc57d 36\uc5b5\uc6d0\uc774 \uc18c\uc694\ub418\ub294 \uc300 \uc7ac\uace0\uad00\ub9ac\ube44\uc6a9\ub3c4 \uc904\uc77c \uc218 \uc788\ub2e4. \uc989 \uc300 \uc0dd\uc0b0\uc870\uc815\uc81c \ub3c4\uc785\uc73c\ub85c 2\ub144\uac04 \ucd1d 2700\uc5b5\uc6d0\uc758 \uc608\uc0b0\uc774 \uc18c\uc694\ub418\uc9c0\ub9cc \uc774\ub85c \uc778\ud55c \ube44\uc6a9\uc808\uac10 \ud6a8\uacfc\ub294 8100\uc5b5\uc6d0\uc5d0 \uc774\ub978\ub2e4. \uc57d 5400\uc5b5\uc6d0 \uac00\ub7c9\uc744 \uc904\uc77c \uc218 \uc788\ub294 \uc148\uc774\ub2e4.", "answer": "\uc57d 36\uc5b5\uc6d0", "sentence": "\ub610 1\ub9cc\ud1a4\ub2f9 \uc5f0\uac04 \uc57d 36\uc5b5\uc6d0 \uc774 \uc18c\uc694\ub418\ub294 \uc300 \uc7ac\uace0\uad00\ub9ac\ube44\uc6a9\ub3c4 \uc904\uc77c \uc218 \uc788\ub2e4.", "paragraph_sentence": "\uadf8\ub7ec\ub098 \ub18d\uc2dd\ud488\ubd80\ub294 \ub2f9\uc2dc \ucf69 \uc7ac\ubc30 \uba74\uc801\uc774 \ub298\uc5b4 \uac00\uaca9\uc774 \ud558\ub77d\ud55c \uac83\uc774 \uc544\ub2c8\ub77c \ucf69 \uc800\uc728\uad00\uc138\ud560\ub2f9\ub7c9(TRQ)\uc744 \ub298\ub9b0 \uac83\uc774 \ubb38\uc81c\uc600\ub2e4\uace0 \ubc18\ubc15\ud588\ub2e4. \ub610 \uc300 \uc7ac\uace0\ub7c9 \uad00\ub9ac\ube44\ubcf4\ub2e4 \uc300 \uc0dd\uc0b0\uc870\uc815\uc81c\uc5d0 \ub4dc\ub294 \ube44\uc6a9\uc774 \ub354 \uc801\ub2e4\ub294 \uc785\uc7a5\uc774\ub2e4. \ub18d\uc2dd\ud488\ubd80\uc5d0 \ub530\ub974\uba74 1\u33ca\ub2f9 300\ub9cc\uc6d0\uc744 \uc9c0\uae09\ud574 2017\ub144\uc5d0 3\ub9cc\u33ca, 2018\ub144\uc5d0 6\ub9cc\u33ca\uc758 \ubcbc \uc7ac\ubc30\uba74\uc801\uc744 \uac10\ucd95\ud558\uba74 \ucd1d 45\ub9cc7200\ud1a4\uc758 \uc300 \uc0dd\uc0b0\uc744 \uc904\uc77c \uc218 \uc788\ub2e4. \uc774\ub807\uac8c \ub418\uba74 45\ub9cc7200\ud1a4\uc758 \uc300\uc744 \uc2dc\uc7a5\uaca9\ub9ac\ud560 \ub54c \uc18c\uc694\ub418\ub294 \ube44\uc6a9 3759\uc5b5\uc6d0\uc774 \uc808\uac10\ub41c\ub2e4. \ub610 \uc300\uac12 \uc548\uc815\uc73c\ub85c \uc300\ubcc0\ub3d9\uc9c1\ubd88\uae08 \uc9c0\uae09\uc561\uc774 2700\uc5b5\uc6d0 \uc904\uc5b4\ub4e4 \uac83\uc73c\ub85c \ubd84\uc11d\ub41c\ub2e4. \ub610 1\ub9cc\ud1a4\ub2f9 \uc5f0\uac04 \uc57d 36\uc5b5\uc6d0 \uc774 \uc18c\uc694\ub418\ub294 \uc300 \uc7ac\uace0\uad00\ub9ac\ube44\uc6a9\ub3c4 \uc904\uc77c \uc218 \uc788\ub2e4. \uc989 \uc300 \uc0dd\uc0b0\uc870\uc815\uc81c \ub3c4\uc785\uc73c\ub85c 2\ub144\uac04 \ucd1d 2700\uc5b5\uc6d0\uc758 \uc608\uc0b0\uc774 \uc18c\uc694\ub418\uc9c0\ub9cc \uc774\ub85c \uc778\ud55c \ube44\uc6a9\uc808\uac10 \ud6a8\uacfc\ub294 8100\uc5b5\uc6d0\uc5d0 \uc774\ub978\ub2e4. \uc57d 5400\uc5b5\uc6d0 \uac00\ub7c9\uc744 \uc904\uc77c \uc218 \uc788\ub294 \uc148\uc774\ub2e4.", "paragraph_answer": "\uadf8\ub7ec\ub098 \ub18d\uc2dd\ud488\ubd80\ub294 \ub2f9\uc2dc \ucf69 \uc7ac\ubc30 \uba74\uc801\uc774 \ub298\uc5b4 \uac00\uaca9\uc774 \ud558\ub77d\ud55c \uac83\uc774 \uc544\ub2c8\ub77c \ucf69 \uc800\uc728\uad00\uc138\ud560\ub2f9\ub7c9(TRQ)\uc744 \ub298\ub9b0 \uac83\uc774 \ubb38\uc81c\uc600\ub2e4\uace0 \ubc18\ubc15\ud588\ub2e4. \ub610 \uc300 \uc7ac\uace0\ub7c9 \uad00\ub9ac\ube44\ubcf4\ub2e4 \uc300 \uc0dd\uc0b0\uc870\uc815\uc81c\uc5d0 \ub4dc\ub294 \ube44\uc6a9\uc774 \ub354 \uc801\ub2e4\ub294 \uc785\uc7a5\uc774\ub2e4. \ub18d\uc2dd\ud488\ubd80\uc5d0 \ub530\ub974\uba74 1\u33ca\ub2f9 300\ub9cc\uc6d0\uc744 \uc9c0\uae09\ud574 2017\ub144\uc5d0 3\ub9cc\u33ca, 2018\ub144\uc5d0 6\ub9cc\u33ca\uc758 \ubcbc \uc7ac\ubc30\uba74\uc801\uc744 \uac10\ucd95\ud558\uba74 \ucd1d 45\ub9cc7200\ud1a4\uc758 \uc300 \uc0dd\uc0b0\uc744 \uc904\uc77c \uc218 \uc788\ub2e4. \uc774\ub807\uac8c \ub418\uba74 45\ub9cc7200\ud1a4\uc758 \uc300\uc744 \uc2dc\uc7a5\uaca9\ub9ac\ud560 \ub54c \uc18c\uc694\ub418\ub294 \ube44\uc6a9 3759\uc5b5\uc6d0\uc774 \uc808\uac10\ub41c\ub2e4. \ub610 \uc300\uac12 \uc548\uc815\uc73c\ub85c \uc300\ubcc0\ub3d9\uc9c1\ubd88\uae08 \uc9c0\uae09\uc561\uc774 2700\uc5b5\uc6d0 \uc904\uc5b4\ub4e4 \uac83\uc73c\ub85c \ubd84\uc11d\ub41c\ub2e4. \ub610 1\ub9cc\ud1a4\ub2f9 \uc5f0\uac04 \uc57d 36\uc5b5\uc6d0 \uc774 \uc18c\uc694\ub418\ub294 \uc300 \uc7ac\uace0\uad00\ub9ac\ube44\uc6a9\ub3c4 \uc904\uc77c \uc218 \uc788\ub2e4. \uc989 \uc300 \uc0dd\uc0b0\uc870\uc815\uc81c \ub3c4\uc785\uc73c\ub85c 2\ub144\uac04 \ucd1d 2700\uc5b5\uc6d0\uc758 \uc608\uc0b0\uc774 \uc18c\uc694\ub418\uc9c0\ub9cc \uc774\ub85c \uc778\ud55c \ube44\uc6a9\uc808\uac10 \ud6a8\uacfc\ub294 8100\uc5b5\uc6d0\uc5d0 \uc774\ub978\ub2e4. \uc57d 5400\uc5b5\uc6d0 \uac00\ub7c9\uc744 \uc904\uc77c \uc218 \uc788\ub294 \uc148\uc774\ub2e4.", "sentence_answer": "\ub610 1\ub9cc\ud1a4\ub2f9 \uc5f0\uac04 \uc57d 36\uc5b5\uc6d0 \uc774 \uc18c\uc694\ub418\ub294 \uc300 \uc7ac\uace0\uad00\ub9ac\ube44\uc6a9\ub3c4 \uc904\uc77c \uc218 \uc788\ub2e4.", "paragraph_id": "6538576-2-0", "paragraph_question": "question: \ub18d\uc2dd\ud488\ubd80\uc5d0\uc11c 1\ub9cc\ud1a4\ub2f9 \uc5f0\uac04 \uc18c\ubaa8\ub418\ub294 \uc300 \uc7ac\uace0\uad00\ub9ac\ube44\uc6a9\uc740 \uc5bc\ub9c8\uc778\uac00?, context: \uadf8\ub7ec\ub098 \ub18d\uc2dd\ud488\ubd80\ub294 \ub2f9\uc2dc \ucf69 \uc7ac\ubc30 \uba74\uc801\uc774 \ub298\uc5b4 \uac00\uaca9\uc774 \ud558\ub77d\ud55c \uac83\uc774 \uc544\ub2c8\ub77c \ucf69 \uc800\uc728\uad00\uc138\ud560\ub2f9\ub7c9(TRQ)\uc744 \ub298\ub9b0 \uac83\uc774 \ubb38\uc81c\uc600\ub2e4\uace0 \ubc18\ubc15\ud588\ub2e4. \ub610 \uc300 \uc7ac\uace0\ub7c9 \uad00\ub9ac\ube44\ubcf4\ub2e4 \uc300 \uc0dd\uc0b0\uc870\uc815\uc81c\uc5d0 \ub4dc\ub294 \ube44\uc6a9\uc774 \ub354 \uc801\ub2e4\ub294 \uc785\uc7a5\uc774\ub2e4. \ub18d\uc2dd\ud488\ubd80\uc5d0 \ub530\ub974\uba74 1\u33ca\ub2f9 300\ub9cc\uc6d0\uc744 \uc9c0\uae09\ud574 2017\ub144\uc5d0 3\ub9cc\u33ca, 2018\ub144\uc5d0 6\ub9cc\u33ca\uc758 \ubcbc \uc7ac\ubc30\uba74\uc801\uc744 \uac10\ucd95\ud558\uba74 \ucd1d 45\ub9cc7200\ud1a4\uc758 \uc300 \uc0dd\uc0b0\uc744 \uc904\uc77c \uc218 \uc788\ub2e4. \uc774\ub807\uac8c \ub418\uba74 45\ub9cc7200\ud1a4\uc758 \uc300\uc744 \uc2dc\uc7a5\uaca9\ub9ac\ud560 \ub54c \uc18c\uc694\ub418\ub294 \ube44\uc6a9 3759\uc5b5\uc6d0\uc774 \uc808\uac10\ub41c\ub2e4. \ub610 \uc300\uac12 \uc548\uc815\uc73c\ub85c \uc300\ubcc0\ub3d9\uc9c1\ubd88\uae08 \uc9c0\uae09\uc561\uc774 2700\uc5b5\uc6d0 \uc904\uc5b4\ub4e4 \uac83\uc73c\ub85c \ubd84\uc11d\ub41c\ub2e4. \ub610 1\ub9cc\ud1a4\ub2f9 \uc5f0\uac04 \uc57d 36\uc5b5\uc6d0\uc774 \uc18c\uc694\ub418\ub294 \uc300 \uc7ac\uace0\uad00\ub9ac\ube44\uc6a9\ub3c4 \uc904\uc77c \uc218 \uc788\ub2e4. \uc989 \uc300 \uc0dd\uc0b0\uc870\uc815\uc81c \ub3c4\uc785\uc73c\ub85c 2\ub144\uac04 \ucd1d 2700\uc5b5\uc6d0\uc758 \uc608\uc0b0\uc774 \uc18c\uc694\ub418\uc9c0\ub9cc \uc774\ub85c \uc778\ud55c \ube44\uc6a9\uc808\uac10 \ud6a8\uacfc\ub294 8100\uc5b5\uc6d0\uc5d0 \uc774\ub978\ub2e4. \uc57d 5400\uc5b5\uc6d0 \uac00\ub7c9\uc744 \uc904\uc77c \uc218 \uc788\ub294 \uc148\uc774\ub2e4."} {"question": "\uc911\uc559\uc758 \uc0b0\uc9c0\ub098 \uace0\uc6d0\uc5d0\uc11c \uc6b0\uae30\uc640 \uac74\uae30\uc911 \uc9c0\ub0b4\uae30 \uc88b\uc740 \uae30\uac04\uc740 \uc5b4\ub290 \ucabd\uc778\uac00?", "paragraph": "\ub3d9\uc11c\uac00 \uc0b0\ub9e5\uc73c\ub85c \uc774\ub8e8\uc5b4\uc9c0\uba74\uc11c \uad6d\ud1a0\ub97c \uc138 \uac1c\uc758 \uc9c0\ubc29\uc73c\ub85c \ub098\ub208\ub2e4. \ud0dc\ud3c9\uc591\ucabd\uc758 \uc881\uc740 \ud574\uc548\ud3c9\uc57c\uc640 \ubd81\ubd80\uc758 \ub113\uc740 \uc800\uc2b5\uc9c0\ub294 \ub365\uace0 \uac74\uac15\uc0c1 \uc88b\uc9c0 \uc54a\uc740 \uc9c0\ubc29\uc774\ub2e4. \uc774\ub97c \ube7c\uace0\uc11c\ub294 \uc0b0\uc545\uc9c0\ub300\uac00 \ub9ce\uc740\ub370, \uc911\uc559\uc758 \uc0b0\uc9c0\ub098 \uace0\uc6d0\uc740 \uc0c1\ub2f9\ud788 \uc2dc\uc6d0\ud558\uace0 \ucf8c\uc801\ud558\uba70, \uac74\uae30\uc640 \uc6b0\uae30 \uc911\uc5d0\uc11c \ud2b9\ud788 \uac74\uae30\uc5d0 \uc9c0\ub0b4\uae30 \uc88b\ub2e4. \ub530\ub77c\uc11c \uc778\uad6c\uc758 \ud0dc\ubc18\uc774 \uc911\uc559\uace0\uc9c0\uc5d0 \uc9d1\uc911\ud574 \uc788\uc73c\uba70, \uc218\ub3c4\uc778 \uacfc\ud14c\ub9d0\ub77c \uc2dc\ub3c4 \ud574\ubc1c \uc57d 1,500m\uc758 \uace0\uc9c0\uc5d0 \uc788\ub2e4. \uc911\uc559\uc0b0\uc9c0\ub294 \uba55\uc2dc\ucf54\uc5d0\uc11c \uacc4\uc18d\ub418\ub294 \ud5d8\uc900\ud55c \uc2b5\uace1 \uc0b0\ub9e5\uc774\uba70, \ud0dc\ud3c9\uc591 \uc5f0\uc548\uc744 \ub530\ub77c \ud654\uc0b0\uc774 \uc990\ube44\ud558\ub2e4. \ud654\uc0b0\uc131 \ud1a0\uc591\uc740 \ube44\uc625\ud558\ubbc0\ub85c \uc774 \uc9c0\uc5ed\uc5d0\ub294 \ub18d\uc5c5\uc774 \ubc1c\ub2ec\ud574 \uc788\ub2e4. 2018\ub144 \ud478\uc5d0\uace0\uc0b0 \ubd84\ud654\uac00 \uc77c\uc5b4\ub0a0 \ub9cc\ud07c \ud65c\ud654\uc0b0\uc774\ub098 \uc9c0\uc9c4\uc758 \ud65c\ub3d9\ub3c4 \uaca9\uc2ec\ud558\ub2e4. \uc57d 50\ub144 \uc804\uc758 \ub300\uc9c0\uc9c4 \ub54c\uc5d0\ub294 \uc218\ub3c4\uac00 \uac70\uc758 \ud30c\uad34\ub418\uc5c8\ub2e4.", "answer": "\uac74\uae30", "sentence": "\uc774\ub97c \ube7c\uace0\uc11c\ub294 \uc0b0\uc545\uc9c0\ub300\uac00 \ub9ce\uc740\ub370, \uc911\uc559\uc758 \uc0b0\uc9c0\ub098 \uace0\uc6d0\uc740 \uc0c1\ub2f9\ud788 \uc2dc\uc6d0\ud558\uace0 \ucf8c\uc801\ud558\uba70, \uac74\uae30 \uc640 \uc6b0\uae30 \uc911\uc5d0\uc11c \ud2b9\ud788 \uac74\uae30\uc5d0 \uc9c0\ub0b4\uae30 \uc88b\ub2e4.", "paragraph_sentence": "\ub3d9\uc11c\uac00 \uc0b0\ub9e5\uc73c\ub85c \uc774\ub8e8\uc5b4\uc9c0\uba74\uc11c \uad6d\ud1a0\ub97c \uc138 \uac1c\uc758 \uc9c0\ubc29\uc73c\ub85c \ub098\ub208\ub2e4. \ud0dc\ud3c9\uc591\ucabd\uc758 \uc881\uc740 \ud574\uc548\ud3c9\uc57c\uc640 \ubd81\ubd80\uc758 \ub113\uc740 \uc800\uc2b5\uc9c0\ub294 \ub365\uace0 \uac74\uac15\uc0c1 \uc88b\uc9c0 \uc54a\uc740 \uc9c0\ubc29\uc774\ub2e4. \uc774\ub97c \ube7c\uace0\uc11c\ub294 \uc0b0\uc545\uc9c0\ub300\uac00 \ub9ce\uc740\ub370, \uc911\uc559\uc758 \uc0b0\uc9c0\ub098 \uace0\uc6d0\uc740 \uc0c1\ub2f9\ud788 \uc2dc\uc6d0\ud558\uace0 \ucf8c\uc801\ud558\uba70, \uac74\uae30 \uc640 \uc6b0\uae30 \uc911\uc5d0\uc11c \ud2b9\ud788 \uac74\uae30\uc5d0 \uc9c0\ub0b4\uae30 \uc88b\ub2e4. \ub530\ub77c\uc11c \uc778\uad6c\uc758 \ud0dc\ubc18\uc774 \uc911\uc559\uace0\uc9c0\uc5d0 \uc9d1\uc911\ud574 \uc788\uc73c\uba70, \uc218\ub3c4\uc778 \uacfc\ud14c\ub9d0\ub77c \uc2dc\ub3c4 \ud574\ubc1c \uc57d 1,500m\uc758 \uace0\uc9c0\uc5d0 \uc788\ub2e4. \uc911\uc559\uc0b0\uc9c0\ub294 \uba55\uc2dc\ucf54\uc5d0\uc11c \uacc4\uc18d\ub418\ub294 \ud5d8\uc900\ud55c \uc2b5\uace1 \uc0b0\ub9e5\uc774\uba70, \ud0dc\ud3c9\uc591 \uc5f0\uc548\uc744 \ub530\ub77c \ud654\uc0b0\uc774 \uc990\ube44\ud558\ub2e4. \ud654\uc0b0\uc131 \ud1a0\uc591\uc740 \ube44\uc625\ud558\ubbc0\ub85c \uc774 \uc9c0\uc5ed\uc5d0\ub294 \ub18d\uc5c5\uc774 \ubc1c\ub2ec\ud574 \uc788\ub2e4. 2018\ub144 \ud478\uc5d0\uace0\uc0b0 \ubd84\ud654\uac00 \uc77c\uc5b4\ub0a0 \ub9cc\ud07c \ud65c\ud654\uc0b0\uc774\ub098 \uc9c0\uc9c4\uc758 \ud65c\ub3d9\ub3c4 \uaca9\uc2ec\ud558\ub2e4. \uc57d 50\ub144 \uc804\uc758 \ub300\uc9c0\uc9c4 \ub54c\uc5d0\ub294 \uc218\ub3c4\uac00 \uac70\uc758 \ud30c\uad34\ub418\uc5c8\ub2e4.", "paragraph_answer": "\ub3d9\uc11c\uac00 \uc0b0\ub9e5\uc73c\ub85c \uc774\ub8e8\uc5b4\uc9c0\uba74\uc11c \uad6d\ud1a0\ub97c \uc138 \uac1c\uc758 \uc9c0\ubc29\uc73c\ub85c \ub098\ub208\ub2e4. \ud0dc\ud3c9\uc591\ucabd\uc758 \uc881\uc740 \ud574\uc548\ud3c9\uc57c\uc640 \ubd81\ubd80\uc758 \ub113\uc740 \uc800\uc2b5\uc9c0\ub294 \ub365\uace0 \uac74\uac15\uc0c1 \uc88b\uc9c0 \uc54a\uc740 \uc9c0\ubc29\uc774\ub2e4. \uc774\ub97c \ube7c\uace0\uc11c\ub294 \uc0b0\uc545\uc9c0\ub300\uac00 \ub9ce\uc740\ub370, \uc911\uc559\uc758 \uc0b0\uc9c0\ub098 \uace0\uc6d0\uc740 \uc0c1\ub2f9\ud788 \uc2dc\uc6d0\ud558\uace0 \ucf8c\uc801\ud558\uba70, \uac74\uae30 \uc640 \uc6b0\uae30 \uc911\uc5d0\uc11c \ud2b9\ud788 \uac74\uae30\uc5d0 \uc9c0\ub0b4\uae30 \uc88b\ub2e4. \ub530\ub77c\uc11c \uc778\uad6c\uc758 \ud0dc\ubc18\uc774 \uc911\uc559\uace0\uc9c0\uc5d0 \uc9d1\uc911\ud574 \uc788\uc73c\uba70, \uc218\ub3c4\uc778 \uacfc\ud14c\ub9d0\ub77c \uc2dc\ub3c4 \ud574\ubc1c \uc57d 1,500m\uc758 \uace0\uc9c0\uc5d0 \uc788\ub2e4. \uc911\uc559\uc0b0\uc9c0\ub294 \uba55\uc2dc\ucf54\uc5d0\uc11c \uacc4\uc18d\ub418\ub294 \ud5d8\uc900\ud55c \uc2b5\uace1 \uc0b0\ub9e5\uc774\uba70, \ud0dc\ud3c9\uc591 \uc5f0\uc548\uc744 \ub530\ub77c \ud654\uc0b0\uc774 \uc990\ube44\ud558\ub2e4. \ud654\uc0b0\uc131 \ud1a0\uc591\uc740 \ube44\uc625\ud558\ubbc0\ub85c \uc774 \uc9c0\uc5ed\uc5d0\ub294 \ub18d\uc5c5\uc774 \ubc1c\ub2ec\ud574 \uc788\ub2e4. 2018\ub144 \ud478\uc5d0\uace0\uc0b0 \ubd84\ud654\uac00 \uc77c\uc5b4\ub0a0 \ub9cc\ud07c \ud65c\ud654\uc0b0\uc774\ub098 \uc9c0\uc9c4\uc758 \ud65c\ub3d9\ub3c4 \uaca9\uc2ec\ud558\ub2e4. \uc57d 50\ub144 \uc804\uc758 \ub300\uc9c0\uc9c4 \ub54c\uc5d0\ub294 \uc218\ub3c4\uac00 \uac70\uc758 \ud30c\uad34\ub418\uc5c8\ub2e4.", "sentence_answer": "\uc774\ub97c \ube7c\uace0\uc11c\ub294 \uc0b0\uc545\uc9c0\ub300\uac00 \ub9ce\uc740\ub370, \uc911\uc559\uc758 \uc0b0\uc9c0\ub098 \uace0\uc6d0\uc740 \uc0c1\ub2f9\ud788 \uc2dc\uc6d0\ud558\uace0 \ucf8c\uc801\ud558\uba70, \uac74\uae30 \uc640 \uc6b0\uae30 \uc911\uc5d0\uc11c \ud2b9\ud788 \uac74\uae30\uc5d0 \uc9c0\ub0b4\uae30 \uc88b\ub2e4.", "paragraph_id": "5956261-2-0", "paragraph_question": "question: \uc911\uc559\uc758 \uc0b0\uc9c0\ub098 \uace0\uc6d0\uc5d0\uc11c \uc6b0\uae30\uc640 \uac74\uae30\uc911 \uc9c0\ub0b4\uae30 \uc88b\uc740 \uae30\uac04\uc740 \uc5b4\ub290 \ucabd\uc778\uac00?, context: \ub3d9\uc11c\uac00 \uc0b0\ub9e5\uc73c\ub85c \uc774\ub8e8\uc5b4\uc9c0\uba74\uc11c \uad6d\ud1a0\ub97c \uc138 \uac1c\uc758 \uc9c0\ubc29\uc73c\ub85c \ub098\ub208\ub2e4. \ud0dc\ud3c9\uc591\ucabd\uc758 \uc881\uc740 \ud574\uc548\ud3c9\uc57c\uc640 \ubd81\ubd80\uc758 \ub113\uc740 \uc800\uc2b5\uc9c0\ub294 \ub365\uace0 \uac74\uac15\uc0c1 \uc88b\uc9c0 \uc54a\uc740 \uc9c0\ubc29\uc774\ub2e4. \uc774\ub97c \ube7c\uace0\uc11c\ub294 \uc0b0\uc545\uc9c0\ub300\uac00 \ub9ce\uc740\ub370, \uc911\uc559\uc758 \uc0b0\uc9c0\ub098 \uace0\uc6d0\uc740 \uc0c1\ub2f9\ud788 \uc2dc\uc6d0\ud558\uace0 \ucf8c\uc801\ud558\uba70, \uac74\uae30\uc640 \uc6b0\uae30 \uc911\uc5d0\uc11c \ud2b9\ud788 \uac74\uae30\uc5d0 \uc9c0\ub0b4\uae30 \uc88b\ub2e4. \ub530\ub77c\uc11c \uc778\uad6c\uc758 \ud0dc\ubc18\uc774 \uc911\uc559\uace0\uc9c0\uc5d0 \uc9d1\uc911\ud574 \uc788\uc73c\uba70, \uc218\ub3c4\uc778 \uacfc\ud14c\ub9d0\ub77c \uc2dc\ub3c4 \ud574\ubc1c \uc57d 1,500m\uc758 \uace0\uc9c0\uc5d0 \uc788\ub2e4. \uc911\uc559\uc0b0\uc9c0\ub294 \uba55\uc2dc\ucf54\uc5d0\uc11c \uacc4\uc18d\ub418\ub294 \ud5d8\uc900\ud55c \uc2b5\uace1 \uc0b0\ub9e5\uc774\uba70, \ud0dc\ud3c9\uc591 \uc5f0\uc548\uc744 \ub530\ub77c \ud654\uc0b0\uc774 \uc990\ube44\ud558\ub2e4. \ud654\uc0b0\uc131 \ud1a0\uc591\uc740 \ube44\uc625\ud558\ubbc0\ub85c \uc774 \uc9c0\uc5ed\uc5d0\ub294 \ub18d\uc5c5\uc774 \ubc1c\ub2ec\ud574 \uc788\ub2e4. 2018\ub144 \ud478\uc5d0\uace0\uc0b0 \ubd84\ud654\uac00 \uc77c\uc5b4\ub0a0 \ub9cc\ud07c \ud65c\ud654\uc0b0\uc774\ub098 \uc9c0\uc9c4\uc758 \ud65c\ub3d9\ub3c4 \uaca9\uc2ec\ud558\ub2e4. \uc57d 50\ub144 \uc804\uc758 \ub300\uc9c0\uc9c4 \ub54c\uc5d0\ub294 \uc218\ub3c4\uac00 \uac70\uc758 \ud30c\uad34\ub418\uc5c8\ub2e4."} {"question": "\uccb4\ucf54 \ubd81\ucabd\uc758 \ub098\uc05c \ud658\uacbd\uc758 \uc6d0\uc778\uc740?", "paragraph": "\uccb4\ucf54\ub294 \uc0b0, \uc644\ub9cc\ud55c \uace0\uc6d0\uacfc, \uc800\uc9c0\ub300, \ub3d9\uad74, \ud611\uace1, \ub113\uc740 \ud3c9\uc6d0, \uc2b5\uc9c0, \ud638\uc218, \uc5f0\ubabb, \ub310 \ub4f1 \uc815\ub9d0 \uc544\ub984\ub2f5\uace0 \ub2e4\ucc44\ub85c\uc6b4 \uc790\uc5f0\uc744 \uc790\ub791\ud55c\ub2e4. \uadf8\ub7ec\ub098 \ubd88\ud589\ud788\ub3c4, \ubd81\ucabd\uc73c\ub85c \uac00\uba74 \uac08\uc218\ub85d, \uc9c0\ub3c5\ud55c \ub300\uae30 \uc624\uc5fc\uacfc \uadf9\uce58\uc5d0 \uc774\ub978 \uc0b0\uc131\ube44 \uc624\uc5fc\uc744 \uacbd\ud5d8\ud558\uac8c \ub418\ub294\ub370, \uc774\ub294 19\uc138\uae30 \ubb34\ubd84\ubcc4\ud55c \uc0b0\uc5c5\ud654\uc758 \uc601\ud5a5\uc774\ub2e4. \uacbd\uc791\uc744 \uc704\ud574 \uc218\uc138\uae30\uc5d0 \uac78\uccd0 \ub098\ubb34\ub97c \ubca0\uc5b4\ub0c8\uc74c\uc5d0\ub3c4 \ubd88\uad6c\ud558\uace0, \uccb4\ucf54\uc758 1/3\uc740 \uc5ec\uc804\ud788 \uc232\uc73c\ub85c \ub458\ub7ec \uc2f8\uc5ec \uc788\ub2e4. \uc0ac\ub78c\uc758 \uc190\uc774 \ub2ff\uc9c0 \uc54a\uc740 \ucc9c\uc5f0\uc758 \uc232\uc740 \ub300\ubd80\ubd84 \uacbd\uc791\uc774 \ubd88\uac00\ub2a5\ud55c \uc0b0\uc545 \uc9c0\uc5ed\uc5d0 \uc788\ub2e4. \uc218\ubaa9 \ud55c\uacc4\uc120(1,400m)\uc774\uc0c1\uc73c\ub85c\ub294 \ucd08\uc6d0, \uad00\ubaa9\ub9bc, \uc9c0\uc758\ub958 \ub4f1\uc774 \uc788\ub2e4 \uacf0, \ub291\ub300, \uc2a4\ub77c\uc18c\ub2c8\uc640 \uadf8 \uc678 \uc57c\uc0dd\uace0\uc591\uc774, \ub9c8\uba4b, \uc218\ub2ec, \ub2f4\ube44, \ubc0d\ud06c \ub4f1\uc758 \uc57c\uc0dd\ub3d9\ubb3c\uc774 \uc11c\uc2dd\ud558\uace0 \uc788\ub2e4.", "answer": "19\uc138\uae30 \ubb34\ubd84\ubcc4\ud55c \uc0b0\uc5c5\ud654", "sentence": "\uc774\ub294 19\uc138\uae30 \ubb34\ubd84\ubcc4\ud55c \uc0b0\uc5c5\ud654 \uc758 \uc601\ud5a5\uc774\ub2e4.", "paragraph_sentence": "\uccb4\ucf54\ub294 \uc0b0, \uc644\ub9cc\ud55c \uace0\uc6d0\uacfc, \uc800\uc9c0\ub300, \ub3d9\uad74, \ud611\uace1, \ub113\uc740 \ud3c9\uc6d0, \uc2b5\uc9c0, \ud638\uc218, \uc5f0\ubabb, \ub310 \ub4f1 \uc815\ub9d0 \uc544\ub984\ub2f5\uace0 \ub2e4\ucc44\ub85c\uc6b4 \uc790\uc5f0\uc744 \uc790\ub791\ud55c\ub2e4. \uadf8\ub7ec\ub098 \ubd88\ud589\ud788\ub3c4, \ubd81\ucabd\uc73c\ub85c \uac00\uba74 \uac08\uc218\ub85d, \uc9c0\ub3c5\ud55c \ub300\uae30 \uc624\uc5fc\uacfc \uadf9\uce58\uc5d0 \uc774\ub978 \uc0b0\uc131\ube44 \uc624\uc5fc\uc744 \uacbd\ud5d8\ud558\uac8c \ub418\ub294\ub370, \uc774\ub294 19\uc138\uae30 \ubb34\ubd84\ubcc4\ud55c \uc0b0\uc5c5\ud654 \uc758 \uc601\ud5a5\uc774\ub2e4. \uacbd\uc791\uc744 \uc704\ud574 \uc218\uc138\uae30\uc5d0 \uac78\uccd0 \ub098\ubb34\ub97c \ubca0\uc5b4\ub0c8\uc74c\uc5d0\ub3c4 \ubd88\uad6c\ud558\uace0, \uccb4\ucf54\uc758 1/3\uc740 \uc5ec\uc804\ud788 \uc232\uc73c\ub85c \ub458\ub7ec \uc2f8\uc5ec \uc788\ub2e4. \uc0ac\ub78c\uc758 \uc190\uc774 \ub2ff\uc9c0 \uc54a\uc740 \ucc9c\uc5f0\uc758 \uc232\uc740 \ub300\ubd80\ubd84 \uacbd\uc791\uc774 \ubd88\uac00\ub2a5\ud55c \uc0b0\uc545 \uc9c0\uc5ed\uc5d0 \uc788\ub2e4. \uc218\ubaa9 \ud55c\uacc4\uc120(1,400m)\uc774\uc0c1\uc73c\ub85c\ub294 \ucd08\uc6d0, \uad00\ubaa9\ub9bc, \uc9c0\uc758\ub958 \ub4f1\uc774 \uc788\ub2e4 \uacf0, \ub291\ub300, \uc2a4\ub77c\uc18c\ub2c8\uc640 \uadf8 \uc678 \uc57c\uc0dd\uace0\uc591\uc774, \ub9c8\uba4b, \uc218\ub2ec, \ub2f4\ube44, \ubc0d\ud06c \ub4f1\uc758 \uc57c\uc0dd\ub3d9\ubb3c\uc774 \uc11c\uc2dd\ud558\uace0 \uc788\ub2e4.", "paragraph_answer": "\uccb4\ucf54\ub294 \uc0b0, \uc644\ub9cc\ud55c \uace0\uc6d0\uacfc, \uc800\uc9c0\ub300, \ub3d9\uad74, \ud611\uace1, \ub113\uc740 \ud3c9\uc6d0, \uc2b5\uc9c0, \ud638\uc218, \uc5f0\ubabb, \ub310 \ub4f1 \uc815\ub9d0 \uc544\ub984\ub2f5\uace0 \ub2e4\ucc44\ub85c\uc6b4 \uc790\uc5f0\uc744 \uc790\ub791\ud55c\ub2e4. \uadf8\ub7ec\ub098 \ubd88\ud589\ud788\ub3c4, \ubd81\ucabd\uc73c\ub85c \uac00\uba74 \uac08\uc218\ub85d, \uc9c0\ub3c5\ud55c \ub300\uae30 \uc624\uc5fc\uacfc \uadf9\uce58\uc5d0 \uc774\ub978 \uc0b0\uc131\ube44 \uc624\uc5fc\uc744 \uacbd\ud5d8\ud558\uac8c \ub418\ub294\ub370, \uc774\ub294 19\uc138\uae30 \ubb34\ubd84\ubcc4\ud55c \uc0b0\uc5c5\ud654 \uc758 \uc601\ud5a5\uc774\ub2e4. \uacbd\uc791\uc744 \uc704\ud574 \uc218\uc138\uae30\uc5d0 \uac78\uccd0 \ub098\ubb34\ub97c \ubca0\uc5b4\ub0c8\uc74c\uc5d0\ub3c4 \ubd88\uad6c\ud558\uace0, \uccb4\ucf54\uc758 1/3\uc740 \uc5ec\uc804\ud788 \uc232\uc73c\ub85c \ub458\ub7ec \uc2f8\uc5ec \uc788\ub2e4. \uc0ac\ub78c\uc758 \uc190\uc774 \ub2ff\uc9c0 \uc54a\uc740 \ucc9c\uc5f0\uc758 \uc232\uc740 \ub300\ubd80\ubd84 \uacbd\uc791\uc774 \ubd88\uac00\ub2a5\ud55c \uc0b0\uc545 \uc9c0\uc5ed\uc5d0 \uc788\ub2e4. \uc218\ubaa9 \ud55c\uacc4\uc120(1,400m)\uc774\uc0c1\uc73c\ub85c\ub294 \ucd08\uc6d0, \uad00\ubaa9\ub9bc, \uc9c0\uc758\ub958 \ub4f1\uc774 \uc788\ub2e4 \uacf0, \ub291\ub300, \uc2a4\ub77c\uc18c\ub2c8\uc640 \uadf8 \uc678 \uc57c\uc0dd\uace0\uc591\uc774, \ub9c8\uba4b, \uc218\ub2ec, \ub2f4\ube44, \ubc0d\ud06c \ub4f1\uc758 \uc57c\uc0dd\ub3d9\ubb3c\uc774 \uc11c\uc2dd\ud558\uace0 \uc788\ub2e4.", "sentence_answer": "\uc774\ub294 19\uc138\uae30 \ubb34\ubd84\ubcc4\ud55c \uc0b0\uc5c5\ud654 \uc758 \uc601\ud5a5\uc774\ub2e4.", "paragraph_id": "5899453-2-0", "paragraph_question": "question: \uccb4\ucf54 \ubd81\ucabd\uc758 \ub098\uc05c \ud658\uacbd\uc758 \uc6d0\uc778\uc740?, context: \uccb4\ucf54\ub294 \uc0b0, \uc644\ub9cc\ud55c \uace0\uc6d0\uacfc, \uc800\uc9c0\ub300, \ub3d9\uad74, \ud611\uace1, \ub113\uc740 \ud3c9\uc6d0, \uc2b5\uc9c0, \ud638\uc218, \uc5f0\ubabb, \ub310 \ub4f1 \uc815\ub9d0 \uc544\ub984\ub2f5\uace0 \ub2e4\ucc44\ub85c\uc6b4 \uc790\uc5f0\uc744 \uc790\ub791\ud55c\ub2e4. \uadf8\ub7ec\ub098 \ubd88\ud589\ud788\ub3c4, \ubd81\ucabd\uc73c\ub85c \uac00\uba74 \uac08\uc218\ub85d, \uc9c0\ub3c5\ud55c \ub300\uae30 \uc624\uc5fc\uacfc \uadf9\uce58\uc5d0 \uc774\ub978 \uc0b0\uc131\ube44 \uc624\uc5fc\uc744 \uacbd\ud5d8\ud558\uac8c \ub418\ub294\ub370, \uc774\ub294 19\uc138\uae30 \ubb34\ubd84\ubcc4\ud55c \uc0b0\uc5c5\ud654\uc758 \uc601\ud5a5\uc774\ub2e4. \uacbd\uc791\uc744 \uc704\ud574 \uc218\uc138\uae30\uc5d0 \uac78\uccd0 \ub098\ubb34\ub97c \ubca0\uc5b4\ub0c8\uc74c\uc5d0\ub3c4 \ubd88\uad6c\ud558\uace0, \uccb4\ucf54\uc758 1/3\uc740 \uc5ec\uc804\ud788 \uc232\uc73c\ub85c \ub458\ub7ec \uc2f8\uc5ec \uc788\ub2e4. \uc0ac\ub78c\uc758 \uc190\uc774 \ub2ff\uc9c0 \uc54a\uc740 \ucc9c\uc5f0\uc758 \uc232\uc740 \ub300\ubd80\ubd84 \uacbd\uc791\uc774 \ubd88\uac00\ub2a5\ud55c \uc0b0\uc545 \uc9c0\uc5ed\uc5d0 \uc788\ub2e4. \uc218\ubaa9 \ud55c\uacc4\uc120(1,400m)\uc774\uc0c1\uc73c\ub85c\ub294 \ucd08\uc6d0, \uad00\ubaa9\ub9bc, \uc9c0\uc758\ub958 \ub4f1\uc774 \uc788\ub2e4 \uacf0, \ub291\ub300, \uc2a4\ub77c\uc18c\ub2c8\uc640 \uadf8 \uc678 \uc57c\uc0dd\uace0\uc591\uc774, \ub9c8\uba4b, \uc218\ub2ec, \ub2f4\ube44, \ubc0d\ud06c \ub4f1\uc758 \uc57c\uc0dd\ub3d9\ubb3c\uc774 \uc11c\uc2dd\ud558\uace0 \uc788\ub2e4."} {"question": "Born This Way \ubc1c\ub9e4 \uc774\ud6c4 \uac00\uac00\uac00 \uc2dc\uc791\ud55c \uc6d4\ub4dc \ud22c\uc5b4\uc758 \uc774\ub984\uc740?", "paragraph": "Born This Way \ubc1c\ub9e4 \uc774\ud6c4 \uac00\uac00\ub294 The Born This Way Ball\uc774\ub77c\ub294 \uc6d4\ub4dc \ud22c\uc5b4\ub97c \uc2dc\uc791\ud558\uaca0\ub2e4\uace0 \ubc1c\ud45c\ud588\uace0, 2012\ub144 4\uc6d4 \uc11c\uc6b8 \uc62c\ub9bc\ud53d \uc8fc\uacbd\uae30\uc7a5\uc5d0\uc11c \uccab \uacf5\uc5f0\uc744 \ud588\ub2e4. \uc138\uacc4\uc801\uc73c\ub85c \ucd1d 110\ud68c\ub85c \uc608\uc815\ub418\uc5b4\uc788\ub294 \ud22c\uc5b4\ub294 \uc0c1\uc5c5\uc801\uc73c\ub85c\ub3c4 \uc131\uacf5\ud558\uace0 \ud3c9\ub860\uac00\ub85c\ubd80\ud130\ub3c4 \uc88b\uc740 \ud3c9\uc744 \ubc1b\uc558\uc9c0\ub9cc, \uc77c\ubd80 \ubcf4\uc218\ub2e8\uccb4\uc5d0\uc11c\ub294 \ube44\ub09c\ud588\ub2e4. \uc774\ub7ec\ud55c \ube44\ub09c\uc758 \ub300\ubd80\ubd84\uc740 \uc544\uc2dc\uc544 \uc9c0\uc5ed\uc5d0\uc11c \uc77c\uc5b4\ub0ac\ub294\ub370, \uc778\ub3c4\ub124\uc2dc\uc544 \uc790\uce74\ub974\ud0c0 \ud22c\uc5b4\uc5d0\uc11c \uc774\uc2ac\ub78c \ub2e8\uccb4\ub4e4\uc774 \"\uc545\ub9c8\uc758 \ubb38\ud654\ub97c \ud655\uc0b0\uc2dc\ud0a4\uace0 \uc788\ub2e4\"\uba70 \ud22c\uc5b4 \ubc18\ub300 \uc2dc\uc704\ub97c \ubc8c\uc600\ub2e4. \uc774\ub7ec\ud55c \ubc18\ub300\uc5d0\ub3c4 52,000\uc7a5\uc758 \ud2f0\ucf13\uc774 \uba87 \uc77c\ub9cc\uc5d0 \ub9e4\uc9c4\ub418\ub294 \ub4f1 \uc608\uc815\ub300\ub85c \uc9c4\ud589 \ud560 \uc608\uc815\uc774\uc600\uc9c0\ub9cc, \ubb34\ub825 \uc2dc\uc704 \uc704\ud611 \ub4f1\uc73c\ub85c \uc778\ud574 \uc804\uba74 \ucde8\uc18c\ub418\uc5c8\ub2e4. 2013\ub144 2\uc6d4 12\uc77c \uac00\uac00\uc758 \uc624\ub978\ucabd \uc5c9\ub369\uc774 \uad00\uc808\uc640\uc21c\uc774 \ud30c\uc5f4\ub418\uba74\uc11c Born This Way Ball \ub098\uba38\uc9c0 \uacf5\uc5f0\uc774 \ubaa8\ub450 \ubb34\uc0b0\ub418\uc5c8\ub2e4\uace0 \ubc1c\ud45c\ud588\ub2e4. 2\uc6d4 20\uc77c \ube14\ub85c\uadf8\uc5d0 \uc5c9\ub369\uc774 \uc218\uc220\uc744 \ubc1b\uace0 \ud68c\ubcf5 \uc911\uc5d0 \uc788\ub2e4\uace0 \uae00\uc744 \uc62c\ub838\ub2e4.", "answer": "The Born This Way Ball", "sentence": "Born This Way \ubc1c\ub9e4 \uc774\ud6c4 \uac00\uac00\ub294 The Born This Way Ball \uc774\ub77c\ub294 \uc6d4\ub4dc \ud22c\uc5b4\ub97c \uc2dc\uc791\ud558\uaca0\ub2e4\uace0 \ubc1c\ud45c\ud588\uace0, 2012\ub144 4\uc6d4 \uc11c\uc6b8 \uc62c\ub9bc\ud53d \uc8fc\uacbd\uae30\uc7a5\uc5d0\uc11c \uccab \uacf5\uc5f0\uc744 \ud588\ub2e4.", "paragraph_sentence": " Born This Way \ubc1c\ub9e4 \uc774\ud6c4 \uac00\uac00\ub294 The Born This Way Ball \uc774\ub77c\ub294 \uc6d4\ub4dc \ud22c\uc5b4\ub97c \uc2dc\uc791\ud558\uaca0\ub2e4\uace0 \ubc1c\ud45c\ud588\uace0, 2012\ub144 4\uc6d4 \uc11c\uc6b8 \uc62c\ub9bc\ud53d \uc8fc\uacbd\uae30\uc7a5\uc5d0\uc11c \uccab \uacf5\uc5f0\uc744 \ud588\ub2e4. \uc138\uacc4\uc801\uc73c\ub85c \ucd1d 110\ud68c\ub85c \uc608\uc815\ub418\uc5b4\uc788\ub294 \ud22c\uc5b4\ub294 \uc0c1\uc5c5\uc801\uc73c\ub85c\ub3c4 \uc131\uacf5\ud558\uace0 \ud3c9\ub860\uac00\ub85c\ubd80\ud130\ub3c4 \uc88b\uc740 \ud3c9\uc744 \ubc1b\uc558\uc9c0\ub9cc, \uc77c\ubd80 \ubcf4\uc218\ub2e8\uccb4\uc5d0\uc11c\ub294 \ube44\ub09c\ud588\ub2e4. \uc774\ub7ec\ud55c \ube44\ub09c\uc758 \ub300\ubd80\ubd84\uc740 \uc544\uc2dc\uc544 \uc9c0\uc5ed\uc5d0\uc11c \uc77c\uc5b4\ub0ac\ub294\ub370, \uc778\ub3c4\ub124\uc2dc\uc544 \uc790\uce74\ub974\ud0c0 \ud22c\uc5b4\uc5d0\uc11c \uc774\uc2ac\ub78c \ub2e8\uccb4\ub4e4\uc774 \"\uc545\ub9c8\uc758 \ubb38\ud654\ub97c \ud655\uc0b0\uc2dc\ud0a4\uace0 \uc788\ub2e4\"\uba70 \ud22c\uc5b4 \ubc18\ub300 \uc2dc\uc704\ub97c \ubc8c\uc600\ub2e4. \uc774\ub7ec\ud55c \ubc18\ub300\uc5d0\ub3c4 52,000\uc7a5\uc758 \ud2f0\ucf13\uc774 \uba87 \uc77c\ub9cc\uc5d0 \ub9e4\uc9c4\ub418\ub294 \ub4f1 \uc608\uc815\ub300\ub85c \uc9c4\ud589 \ud560 \uc608\uc815\uc774\uc600\uc9c0\ub9cc, \ubb34\ub825 \uc2dc\uc704 \uc704\ud611 \ub4f1\uc73c\ub85c \uc778\ud574 \uc804\uba74 \ucde8\uc18c\ub418\uc5c8\ub2e4. 2013\ub144 2\uc6d4 12\uc77c \uac00\uac00\uc758 \uc624\ub978\ucabd \uc5c9\ub369\uc774 \uad00\uc808\uc640\uc21c\uc774 \ud30c\uc5f4\ub418\uba74\uc11c Born This Way Ball \ub098\uba38\uc9c0 \uacf5\uc5f0\uc774 \ubaa8\ub450 \ubb34\uc0b0\ub418\uc5c8\ub2e4\uace0 \ubc1c\ud45c\ud588\ub2e4. 2\uc6d4 20\uc77c \ube14\ub85c\uadf8\uc5d0 \uc5c9\ub369\uc774 \uc218\uc220\uc744 \ubc1b\uace0 \ud68c\ubcf5 \uc911\uc5d0 \uc788\ub2e4\uace0 \uae00\uc744 \uc62c\ub838\ub2e4.", "paragraph_answer": "Born This Way \ubc1c\ub9e4 \uc774\ud6c4 \uac00\uac00\ub294 The Born This Way Ball \uc774\ub77c\ub294 \uc6d4\ub4dc \ud22c\uc5b4\ub97c \uc2dc\uc791\ud558\uaca0\ub2e4\uace0 \ubc1c\ud45c\ud588\uace0, 2012\ub144 4\uc6d4 \uc11c\uc6b8 \uc62c\ub9bc\ud53d \uc8fc\uacbd\uae30\uc7a5\uc5d0\uc11c \uccab \uacf5\uc5f0\uc744 \ud588\ub2e4. \uc138\uacc4\uc801\uc73c\ub85c \ucd1d 110\ud68c\ub85c \uc608\uc815\ub418\uc5b4\uc788\ub294 \ud22c\uc5b4\ub294 \uc0c1\uc5c5\uc801\uc73c\ub85c\ub3c4 \uc131\uacf5\ud558\uace0 \ud3c9\ub860\uac00\ub85c\ubd80\ud130\ub3c4 \uc88b\uc740 \ud3c9\uc744 \ubc1b\uc558\uc9c0\ub9cc, \uc77c\ubd80 \ubcf4\uc218\ub2e8\uccb4\uc5d0\uc11c\ub294 \ube44\ub09c\ud588\ub2e4. \uc774\ub7ec\ud55c \ube44\ub09c\uc758 \ub300\ubd80\ubd84\uc740 \uc544\uc2dc\uc544 \uc9c0\uc5ed\uc5d0\uc11c \uc77c\uc5b4\ub0ac\ub294\ub370, \uc778\ub3c4\ub124\uc2dc\uc544 \uc790\uce74\ub974\ud0c0 \ud22c\uc5b4\uc5d0\uc11c \uc774\uc2ac\ub78c \ub2e8\uccb4\ub4e4\uc774 \"\uc545\ub9c8\uc758 \ubb38\ud654\ub97c \ud655\uc0b0\uc2dc\ud0a4\uace0 \uc788\ub2e4\"\uba70 \ud22c\uc5b4 \ubc18\ub300 \uc2dc\uc704\ub97c \ubc8c\uc600\ub2e4. \uc774\ub7ec\ud55c \ubc18\ub300\uc5d0\ub3c4 52,000\uc7a5\uc758 \ud2f0\ucf13\uc774 \uba87 \uc77c\ub9cc\uc5d0 \ub9e4\uc9c4\ub418\ub294 \ub4f1 \uc608\uc815\ub300\ub85c \uc9c4\ud589 \ud560 \uc608\uc815\uc774\uc600\uc9c0\ub9cc, \ubb34\ub825 \uc2dc\uc704 \uc704\ud611 \ub4f1\uc73c\ub85c \uc778\ud574 \uc804\uba74 \ucde8\uc18c\ub418\uc5c8\ub2e4. 2013\ub144 2\uc6d4 12\uc77c \uac00\uac00\uc758 \uc624\ub978\ucabd \uc5c9\ub369\uc774 \uad00\uc808\uc640\uc21c\uc774 \ud30c\uc5f4\ub418\uba74\uc11c Born This Way Ball \ub098\uba38\uc9c0 \uacf5\uc5f0\uc774 \ubaa8\ub450 \ubb34\uc0b0\ub418\uc5c8\ub2e4\uace0 \ubc1c\ud45c\ud588\ub2e4. 2\uc6d4 20\uc77c \ube14\ub85c\uadf8\uc5d0 \uc5c9\ub369\uc774 \uc218\uc220\uc744 \ubc1b\uace0 \ud68c\ubcf5 \uc911\uc5d0 \uc788\ub2e4\uace0 \uae00\uc744 \uc62c\ub838\ub2e4.", "sentence_answer": "Born This Way \ubc1c\ub9e4 \uc774\ud6c4 \uac00\uac00\ub294 The Born This Way Ball \uc774\ub77c\ub294 \uc6d4\ub4dc \ud22c\uc5b4\ub97c \uc2dc\uc791\ud558\uaca0\ub2e4\uace0 \ubc1c\ud45c\ud588\uace0, 2012\ub144 4\uc6d4 \uc11c\uc6b8 \uc62c\ub9bc\ud53d \uc8fc\uacbd\uae30\uc7a5\uc5d0\uc11c \uccab \uacf5\uc5f0\uc744 \ud588\ub2e4.", "paragraph_id": "6579539-12-0", "paragraph_question": "question: Born This Way \ubc1c\ub9e4 \uc774\ud6c4 \uac00\uac00\uac00 \uc2dc\uc791\ud55c \uc6d4\ub4dc \ud22c\uc5b4\uc758 \uc774\ub984\uc740?, context: Born This Way \ubc1c\ub9e4 \uc774\ud6c4 \uac00\uac00\ub294 The Born This Way Ball\uc774\ub77c\ub294 \uc6d4\ub4dc \ud22c\uc5b4\ub97c \uc2dc\uc791\ud558\uaca0\ub2e4\uace0 \ubc1c\ud45c\ud588\uace0, 2012\ub144 4\uc6d4 \uc11c\uc6b8 \uc62c\ub9bc\ud53d \uc8fc\uacbd\uae30\uc7a5\uc5d0\uc11c \uccab \uacf5\uc5f0\uc744 \ud588\ub2e4. \uc138\uacc4\uc801\uc73c\ub85c \ucd1d 110\ud68c\ub85c \uc608\uc815\ub418\uc5b4\uc788\ub294 \ud22c\uc5b4\ub294 \uc0c1\uc5c5\uc801\uc73c\ub85c\ub3c4 \uc131\uacf5\ud558\uace0 \ud3c9\ub860\uac00\ub85c\ubd80\ud130\ub3c4 \uc88b\uc740 \ud3c9\uc744 \ubc1b\uc558\uc9c0\ub9cc, \uc77c\ubd80 \ubcf4\uc218\ub2e8\uccb4\uc5d0\uc11c\ub294 \ube44\ub09c\ud588\ub2e4. \uc774\ub7ec\ud55c \ube44\ub09c\uc758 \ub300\ubd80\ubd84\uc740 \uc544\uc2dc\uc544 \uc9c0\uc5ed\uc5d0\uc11c \uc77c\uc5b4\ub0ac\ub294\ub370, \uc778\ub3c4\ub124\uc2dc\uc544 \uc790\uce74\ub974\ud0c0 \ud22c\uc5b4\uc5d0\uc11c \uc774\uc2ac\ub78c \ub2e8\uccb4\ub4e4\uc774 \"\uc545\ub9c8\uc758 \ubb38\ud654\ub97c \ud655\uc0b0\uc2dc\ud0a4\uace0 \uc788\ub2e4\"\uba70 \ud22c\uc5b4 \ubc18\ub300 \uc2dc\uc704\ub97c \ubc8c\uc600\ub2e4. \uc774\ub7ec\ud55c \ubc18\ub300\uc5d0\ub3c4 52,000\uc7a5\uc758 \ud2f0\ucf13\uc774 \uba87 \uc77c\ub9cc\uc5d0 \ub9e4\uc9c4\ub418\ub294 \ub4f1 \uc608\uc815\ub300\ub85c \uc9c4\ud589 \ud560 \uc608\uc815\uc774\uc600\uc9c0\ub9cc, \ubb34\ub825 \uc2dc\uc704 \uc704\ud611 \ub4f1\uc73c\ub85c \uc778\ud574 \uc804\uba74 \ucde8\uc18c\ub418\uc5c8\ub2e4. 2013\ub144 2\uc6d4 12\uc77c \uac00\uac00\uc758 \uc624\ub978\ucabd \uc5c9\ub369\uc774 \uad00\uc808\uc640\uc21c\uc774 \ud30c\uc5f4\ub418\uba74\uc11c Born This Way Ball \ub098\uba38\uc9c0 \uacf5\uc5f0\uc774 \ubaa8\ub450 \ubb34\uc0b0\ub418\uc5c8\ub2e4\uace0 \ubc1c\ud45c\ud588\ub2e4. 2\uc6d4 20\uc77c \ube14\ub85c\uadf8\uc5d0 \uc5c9\ub369\uc774 \uc218\uc220\uc744 \ubc1b\uace0 \ud68c\ubcf5 \uc911\uc5d0 \uc788\ub2e4\uace0 \uae00\uc744 \uc62c\ub838\ub2e4."} {"question": "\uc624\uce74\uc790\uce74 \uc774\uc0ac\uc624\uac00 \ub4e4\uc5b4\uac14\ub358 \uad6d\uac00\uc8fc\uc758 \ub2e8\uccb4 \uae34\ub178\ub9c8\ucf54\ud1a0\ubb34\uc2a4\ube44\uc5d0 \ud568\uaed8 \uc788\ub358 \uc911\ud559\uad50 \uc2dc\uc808 \uce5c\uad6c\ub294 \ub204\uad6c\uc785\ub2c8\uae4c?", "paragraph": "\uc8fc\ub3d9\uc790 \uc624\uce74\uc790\ud0a4 \uc774\uc0ac\uc624(\uc77c\ubcf8\uc5b4: \u5ca1\u5d0e\u529f, \ubcf8\ub798 \ud55c\uc790 \u5ca1\u5d0e\u5141\u4f50\u592b, 1920\ub144 7\uc6d4 17\uc77c ~ 2006\ub144)\ub294 \uc2dc\ub9c8\ub124\ud604\uc5d0\uc11c \ud0dc\uc5b4\ub098, 1939\ub144 3\uc6d4 \ub9c8\uc4f0\uc5d0 \uc911\ud559\uad50\ub97c \uc878\uc5c5\ud55c \ud6c4, 2\ub144 \uac04 \ub9cc\uc8fc\uc758 \ubbf8\uc4f0\uc774 \ubb3c\uc0b0 \ud391\ud188(\u5949\u5929) \uc9c0\uc810\uc5d0\uc11c \uadfc\ubb34\ud558\ub358 \uc911, \uad6d\uc218\uc8fc\uc758 \uc0ac\uc0c1\uc744 \ubc1b\uc544\ub4e4\uc600\ub2e4. 1942\ub144 11\uc6d4 \uc77c\ubcf8\uc73c\ub85c \ub3cc\uc544\uc640 \uc2b9\ub824\uac00 \ub418\uace0\uc790 \uc77c\ub828\uc885 \uacc4\uc5f4\uc758 \ub9bf\uc1fc \ub300\ud559 \uc804\ubb38\ubd80\uc5d0 \uc785\ud559\ud558\uba74\uc11c \uc911\ud559\uad50 \uc2dc\uc808\uc758 \uce5c\uad6c\uc600\ub358 \ud788\ub85c\uc5d0 \uace0\ubd84(\u5e83\u6c5f\u5b64\u6587, \ub2e4\ub978 \ud55c\uc790 \ud45c\uae30 \u5e83\u6c5f\u5f18\u6587)\uac00 \uac00\uc785\ud574 \uc788\uc5c8\ub358 \uad6d\uac00\uc8fc\uc758 \ub2e8\uccb4 \uae34\ub178\ub9c8\ucf54\ud1a0\ubb34\uc2a4\ube44\uc5d0 \ub4e4\uc5b4\uac00, \uad6d\uac00 \ud601\uc2e0 \uc6b4\ub3d9\uc5d0 \ucc38\uac00\ud558\uac8c \ub418\uc5c8\ub2e4. \ub2f9\uc2dc\uc758 \uc624\uce74\uc790\ud0a4\uc5d0\uac8c \uc601\ud5a5\uc744 \uc900 \uc11c\uc801\uc73c\ub85c \ubbf8\uc4f0\ub2e4 \uc774\uc640\uc624\uc758 \u300a\uc1fc\uc640 \ud48d\uc6b4\ub85d\u300b, \ub9c8\uc4f0\ub098\uac00 \ubaa8\ud1a0\ud0a4\uc758 \u300a\ud669\uad6d\uccb4\uc81c\u300b, \uc544\ub9c8\ub178 \ub2e4\uc4f0\uc624\uc758 \u300a\uad6d\uccb4\ud669\ub3c4\u300b \ub4f1\uc774 \uc788\ub2e4.", "answer": "\ud788\ub85c\uc5d0 \uace0\ubd84", "sentence": "1942\ub144 11\uc6d4 \uc77c\ubcf8\uc73c\ub85c \ub3cc\uc544\uc640 \uc2b9\ub824\uac00 \ub418\uace0\uc790 \uc77c\ub828\uc885 \uacc4\uc5f4\uc758 \ub9bf\uc1fc \ub300\ud559 \uc804\ubb38\ubd80\uc5d0 \uc785\ud559\ud558\uba74\uc11c \uc911\ud559\uad50 \uc2dc\uc808\uc758 \uce5c\uad6c\uc600\ub358 \ud788\ub85c\uc5d0 \uace0\ubd84 (\u5e83\u6c5f\u5b64\u6587, \ub2e4\ub978 \ud55c\uc790 \ud45c\uae30 \u5e83\u6c5f\u5f18\u6587)\uac00 \uac00\uc785\ud574 \uc788\uc5c8\ub358 \uad6d\uac00\uc8fc\uc758 \ub2e8\uccb4 \uae34\ub178\ub9c8\ucf54\ud1a0\ubb34\uc2a4\ube44\uc5d0 \ub4e4\uc5b4\uac00, \uad6d\uac00 \ud601\uc2e0 \uc6b4\ub3d9\uc5d0 \ucc38\uac00\ud558\uac8c \ub418\uc5c8\ub2e4.", "paragraph_sentence": "\uc8fc\ub3d9\uc790 \uc624\uce74\uc790\ud0a4 \uc774\uc0ac\uc624(\uc77c\ubcf8\uc5b4: \u5ca1\u5d0e\u529f, \ubcf8\ub798 \ud55c\uc790 \u5ca1\u5d0e\u5141\u4f50\u592b, 1920\ub144 7\uc6d4 17\uc77c ~ 2006\ub144)\ub294 \uc2dc\ub9c8\ub124\ud604\uc5d0\uc11c \ud0dc\uc5b4\ub098, 1939\ub144 3\uc6d4 \ub9c8\uc4f0\uc5d0 \uc911\ud559\uad50\ub97c \uc878\uc5c5\ud55c \ud6c4, 2\ub144 \uac04 \ub9cc\uc8fc\uc758 \ubbf8\uc4f0\uc774 \ubb3c\uc0b0 \ud391\ud188(\u5949\u5929) \uc9c0\uc810\uc5d0\uc11c \uadfc\ubb34\ud558\ub358 \uc911, \uad6d\uc218\uc8fc\uc758 \uc0ac\uc0c1\uc744 \ubc1b\uc544\ub4e4\uc600\ub2e4. 1942\ub144 11\uc6d4 \uc77c\ubcf8\uc73c\ub85c \ub3cc\uc544\uc640 \uc2b9\ub824\uac00 \ub418\uace0\uc790 \uc77c\ub828\uc885 \uacc4\uc5f4\uc758 \ub9bf\uc1fc \ub300\ud559 \uc804\ubb38\ubd80\uc5d0 \uc785\ud559\ud558\uba74\uc11c \uc911\ud559\uad50 \uc2dc\uc808\uc758 \uce5c\uad6c\uc600\ub358 \ud788\ub85c\uc5d0 \uace0\ubd84 (\u5e83\u6c5f\u5b64\u6587, \ub2e4\ub978 \ud55c\uc790 \ud45c\uae30 \u5e83\u6c5f\u5f18\u6587)\uac00 \uac00\uc785\ud574 \uc788\uc5c8\ub358 \uad6d\uac00\uc8fc\uc758 \ub2e8\uccb4 \uae34\ub178\ub9c8\ucf54\ud1a0\ubb34\uc2a4\ube44\uc5d0 \ub4e4\uc5b4\uac00, \uad6d\uac00 \ud601\uc2e0 \uc6b4\ub3d9\uc5d0 \ucc38\uac00\ud558\uac8c \ub418\uc5c8\ub2e4. \ub2f9\uc2dc\uc758 \uc624\uce74\uc790\ud0a4\uc5d0\uac8c \uc601\ud5a5\uc744 \uc900 \uc11c\uc801\uc73c\ub85c \ubbf8\uc4f0\ub2e4 \uc774\uc640\uc624\uc758 \u300a\uc1fc\uc640 \ud48d\uc6b4\ub85d\u300b, \ub9c8\uc4f0\ub098\uac00 \ubaa8\ud1a0\ud0a4\uc758 \u300a\ud669\uad6d\uccb4\uc81c\u300b, \uc544\ub9c8\ub178 \ub2e4\uc4f0\uc624\uc758 \u300a\uad6d\uccb4\ud669\ub3c4\u300b \ub4f1\uc774 \uc788\ub2e4.", "paragraph_answer": "\uc8fc\ub3d9\uc790 \uc624\uce74\uc790\ud0a4 \uc774\uc0ac\uc624(\uc77c\ubcf8\uc5b4: \u5ca1\u5d0e\u529f, \ubcf8\ub798 \ud55c\uc790 \u5ca1\u5d0e\u5141\u4f50\u592b, 1920\ub144 7\uc6d4 17\uc77c ~ 2006\ub144)\ub294 \uc2dc\ub9c8\ub124\ud604\uc5d0\uc11c \ud0dc\uc5b4\ub098, 1939\ub144 3\uc6d4 \ub9c8\uc4f0\uc5d0 \uc911\ud559\uad50\ub97c \uc878\uc5c5\ud55c \ud6c4, 2\ub144 \uac04 \ub9cc\uc8fc\uc758 \ubbf8\uc4f0\uc774 \ubb3c\uc0b0 \ud391\ud188(\u5949\u5929) \uc9c0\uc810\uc5d0\uc11c \uadfc\ubb34\ud558\ub358 \uc911, \uad6d\uc218\uc8fc\uc758 \uc0ac\uc0c1\uc744 \ubc1b\uc544\ub4e4\uc600\ub2e4. 1942\ub144 11\uc6d4 \uc77c\ubcf8\uc73c\ub85c \ub3cc\uc544\uc640 \uc2b9\ub824\uac00 \ub418\uace0\uc790 \uc77c\ub828\uc885 \uacc4\uc5f4\uc758 \ub9bf\uc1fc \ub300\ud559 \uc804\ubb38\ubd80\uc5d0 \uc785\ud559\ud558\uba74\uc11c \uc911\ud559\uad50 \uc2dc\uc808\uc758 \uce5c\uad6c\uc600\ub358 \ud788\ub85c\uc5d0 \uace0\ubd84 (\u5e83\u6c5f\u5b64\u6587, \ub2e4\ub978 \ud55c\uc790 \ud45c\uae30 \u5e83\u6c5f\u5f18\u6587)\uac00 \uac00\uc785\ud574 \uc788\uc5c8\ub358 \uad6d\uac00\uc8fc\uc758 \ub2e8\uccb4 \uae34\ub178\ub9c8\ucf54\ud1a0\ubb34\uc2a4\ube44\uc5d0 \ub4e4\uc5b4\uac00, \uad6d\uac00 \ud601\uc2e0 \uc6b4\ub3d9\uc5d0 \ucc38\uac00\ud558\uac8c \ub418\uc5c8\ub2e4. \ub2f9\uc2dc\uc758 \uc624\uce74\uc790\ud0a4\uc5d0\uac8c \uc601\ud5a5\uc744 \uc900 \uc11c\uc801\uc73c\ub85c \ubbf8\uc4f0\ub2e4 \uc774\uc640\uc624\uc758 \u300a\uc1fc\uc640 \ud48d\uc6b4\ub85d\u300b, \ub9c8\uc4f0\ub098\uac00 \ubaa8\ud1a0\ud0a4\uc758 \u300a\ud669\uad6d\uccb4\uc81c\u300b, \uc544\ub9c8\ub178 \ub2e4\uc4f0\uc624\uc758 \u300a\uad6d\uccb4\ud669\ub3c4\u300b \ub4f1\uc774 \uc788\ub2e4.", "sentence_answer": "1942\ub144 11\uc6d4 \uc77c\ubcf8\uc73c\ub85c \ub3cc\uc544\uc640 \uc2b9\ub824\uac00 \ub418\uace0\uc790 \uc77c\ub828\uc885 \uacc4\uc5f4\uc758 \ub9bf\uc1fc \ub300\ud559 \uc804\ubb38\ubd80\uc5d0 \uc785\ud559\ud558\uba74\uc11c \uc911\ud559\uad50 \uc2dc\uc808\uc758 \uce5c\uad6c\uc600\ub358 \ud788\ub85c\uc5d0 \uace0\ubd84 (\u5e83\u6c5f\u5b64\u6587, \ub2e4\ub978 \ud55c\uc790 \ud45c\uae30 \u5e83\u6c5f\u5f18\u6587)\uac00 \uac00\uc785\ud574 \uc788\uc5c8\ub358 \uad6d\uac00\uc8fc\uc758 \ub2e8\uccb4 \uae34\ub178\ub9c8\ucf54\ud1a0\ubb34\uc2a4\ube44\uc5d0 \ub4e4\uc5b4\uac00, \uad6d\uac00 \ud601\uc2e0 \uc6b4\ub3d9\uc5d0 \ucc38\uac00\ud558\uac8c \ub418\uc5c8\ub2e4.", "paragraph_id": "6534177-3-1", "paragraph_question": "question: \uc624\uce74\uc790\uce74 \uc774\uc0ac\uc624\uac00 \ub4e4\uc5b4\uac14\ub358 \uad6d\uac00\uc8fc\uc758 \ub2e8\uccb4 \uae34\ub178\ub9c8\ucf54\ud1a0\ubb34\uc2a4\ube44\uc5d0 \ud568\uaed8 \uc788\ub358 \uc911\ud559\uad50 \uc2dc\uc808 \uce5c\uad6c\ub294 \ub204\uad6c\uc785\ub2c8\uae4c?, context: \uc8fc\ub3d9\uc790 \uc624\uce74\uc790\ud0a4 \uc774\uc0ac\uc624(\uc77c\ubcf8\uc5b4: \u5ca1\u5d0e\u529f, \ubcf8\ub798 \ud55c\uc790 \u5ca1\u5d0e\u5141\u4f50\u592b, 1920\ub144 7\uc6d4 17\uc77c ~ 2006\ub144)\ub294 \uc2dc\ub9c8\ub124\ud604\uc5d0\uc11c \ud0dc\uc5b4\ub098, 1939\ub144 3\uc6d4 \ub9c8\uc4f0\uc5d0 \uc911\ud559\uad50\ub97c \uc878\uc5c5\ud55c \ud6c4, 2\ub144 \uac04 \ub9cc\uc8fc\uc758 \ubbf8\uc4f0\uc774 \ubb3c\uc0b0 \ud391\ud188(\u5949\u5929) \uc9c0\uc810\uc5d0\uc11c \uadfc\ubb34\ud558\ub358 \uc911, \uad6d\uc218\uc8fc\uc758 \uc0ac\uc0c1\uc744 \ubc1b\uc544\ub4e4\uc600\ub2e4. 1942\ub144 11\uc6d4 \uc77c\ubcf8\uc73c\ub85c \ub3cc\uc544\uc640 \uc2b9\ub824\uac00 \ub418\uace0\uc790 \uc77c\ub828\uc885 \uacc4\uc5f4\uc758 \ub9bf\uc1fc \ub300\ud559 \uc804\ubb38\ubd80\uc5d0 \uc785\ud559\ud558\uba74\uc11c \uc911\ud559\uad50 \uc2dc\uc808\uc758 \uce5c\uad6c\uc600\ub358 \ud788\ub85c\uc5d0 \uace0\ubd84(\u5e83\u6c5f\u5b64\u6587, \ub2e4\ub978 \ud55c\uc790 \ud45c\uae30 \u5e83\u6c5f\u5f18\u6587)\uac00 \uac00\uc785\ud574 \uc788\uc5c8\ub358 \uad6d\uac00\uc8fc\uc758 \ub2e8\uccb4 \uae34\ub178\ub9c8\ucf54\ud1a0\ubb34\uc2a4\ube44\uc5d0 \ub4e4\uc5b4\uac00, \uad6d\uac00 \ud601\uc2e0 \uc6b4\ub3d9\uc5d0 \ucc38\uac00\ud558\uac8c \ub418\uc5c8\ub2e4. \ub2f9\uc2dc\uc758 \uc624\uce74\uc790\ud0a4\uc5d0\uac8c \uc601\ud5a5\uc744 \uc900 \uc11c\uc801\uc73c\ub85c \ubbf8\uc4f0\ub2e4 \uc774\uc640\uc624\uc758 \u300a\uc1fc\uc640 \ud48d\uc6b4\ub85d\u300b, \ub9c8\uc4f0\ub098\uac00 \ubaa8\ud1a0\ud0a4\uc758 \u300a\ud669\uad6d\uccb4\uc81c\u300b, \uc544\ub9c8\ub178 \ub2e4\uc4f0\uc624\uc758 \u300a\uad6d\uccb4\ud669\ub3c4\u300b \ub4f1\uc774 \uc788\ub2e4."} {"question": "\ube5b\uc758 \uc9c4\ub3d9\uc218\uac00 \uc911\ub825\ub54c\ubb38\uc5d0 \ud3b8\uc774\ud558\uac8c \ub418\ub294\uac83\uc744 \ubb34\uc5c7\uc774\ub77c\uace0 \ud558\ub294\uac00?", "paragraph": "\uccab \ubc88\uc9f8 \uc0c8\ub85c\uc6b4 \ud6a8\uacfc\ub294 \ube5b\uc758 \uc9c4\ub3d9\uc218\uac00 \uc911\ub825\uc73c\ub85c \uc778\ud574 \ud3b8\uc774\ud55c\ub2e4(\uc911\ub825\uc801\uc0c9\ud3b8\uc774)\ub294 \uac83\uc774\ub2e4. \uac00\uc18d\ub418\ub294 \ub85c\ucf13\uc5d0 \ud0d1\uc2b9 \uc911\uc778 \uad00\ucc30\uc790 \ub450 \uba85\uc774 \uc788\ub2e4\uace0 \ud558\uc790. \uc6b0\uc8fc\uc120 \uc704\uc5d0\uc11c\ub294 \"\uc704\"\uc640 \"\uc544\ub798\"\uc758 \uc790\uc5f0\uc2a4\ub7ec\uc6b4 \uac1c\ub150\uc774 \uc874\uc7ac\ud55c\ub2e4. \uc6b0\uc8fc\uc120\uc774 \uac00\uc18d\ub418\ub294 \ubc29\ud5a5\uc774 \"\uc704\"\uc774\uace0, \uc774\uc5d0 \ub300\ud55c \uad00\uc131\uc73c\ub85c \uace0\uc815\ub418\uc9c0 \uc54a\uc740 \ubb3c\uccb4\uac00 \ubc18\ub300 \ubc29\ud5a5\uc73c\ub85c \uc6c0\uc9c1\uc774\uba74\uc11c \ub099\ud558\ud560 \ud14c\ub2c8 \uadf8\ucabd\uc774 \"\uc544\ub798\"\uc774\ub2e4. \uad00\ucc30\uc790 \uc911 \ud55c \uba85\uc774 \ub2e4\ub978 \ud55c \uba85 \ubcf4\ub2e4 \"\ub354 \uc704\"\uc5d0 \uc788\ub2e4\uace0 \uac00\uc815\ud574 \ubcf4\uc790. \uc544\ub798\ucabd\uc5d0 \uc788\ub294 \uad00\ucc30\uc790\uac00 \uc704\ucabd\uc5d0 \uc788\ub294 \uad00\ucc30\uc790\uc5d0\uac8c \ube5b\uc73c\ub85c \uc2e0\ud638\ub97c \ubcf4\ub0b4\uba74, \ub85c\ucf13\uc758 \uac00\uc18d \ub54c\ubb38\uc5d0 \ube5b\uc774 \uc801\uc0c9\ud3b8\uc774\ub97c \ud558\uac8c \ub41c\ub2e4(\uc774\ub294 \ud2b9\uc218\uc0c1\ub300\ub860\uc5d0\uc11c \uacc4\uc0b0\ub418\ub294 \ubc14\uc774\ub2e4). \uc704\ucabd\uc758 \uad00\ucc30\uc790\ub294 \uc544\ub798\ucabd\uc758 \uad00\ucc30\uc790\uac00 \ube5b\uc744 \ucc98\uc74c \uc3d8\uc558\uc744 \ub54c\uc640 \ube44\uad50\ud558\uc5ec \ub354 \ub0ae\uc740 \uc9c4\ub3d9\uc218\uc758 \ube5b\uc744 \uad00\ucc30\ud558\uac8c \ub420 \uac83\uc774\ub2e4. \uc5ed\uc73c\ub85c \uc704\ucabd\uc758 \uad00\ucc30\uc790\uac00 \uc544\ub798\ucabd\uc758 \uad00\ucc30\uc790\uc5d0\uac8c \ubcf4\ub0b4\ub294 \ube5b \uc2e0\ud638\ub294 \uccad\uc0c9\ud3b8\uc774, \uc989 \uc9c4\ub3d9\uc218\uac00 \ub354 \ub192\uc544\uc9c8 \uac83\uc774\ub2e4. \uc544\uc778\uc288\ud0c0\uc778\uc740 \uc774\ub7ec\ud55c \uc9c4\ub3d9\uc218 \ud3b8\uc774\uac00 \uc911\ub825\uc7a5 \uc548\uc5d0\uc11c \ubc18\ub4dc\uc2dc \uad00\ucc30\ub418\uc5b4\uc57c \ud55c\ub2e4\uace0 \ud588\ub2e4. \uc88c\uce21 \uadf8\ub9bc\uc774 \uc774\uac83\uc744 \ub098\ud0c0\ub0b8 \uac83\uc73c\ub85c\uc11c, \ube5b\uc774 \uc911\ub825\uc801 \uac00\uc18d\uc5d0 \ubc18\ub300\ub418\ub294 \ubc29\ud5a5\uc73c\ub85c \uc6c0\uc9c1\uc774\uba74\uc11c \uc11c\uc11c\ud788 \uc801\uc0c9\ud3b8\uc774\ud568\uc744 \ubcf4\uc5ec\uc900\ub2e4. \uc774 \ud6a8\uacfc\ub294 \uc2e4\ud5d8\uc801\uc73c\ub85c \uac80\uc99d\ub418\uc5c8\uc73c\uba70, \uadf8\uc5d0 \ub300\ud574\uc11c\ub294 \ud6c4\uc220\ud55c\ub2e4.", "answer": "\uc911\ub825\uc801\uc0c9\ud3b8\uc774", "sentence": "\uccab \ubc88\uc9f8 \uc0c8\ub85c\uc6b4 \ud6a8\uacfc\ub294 \ube5b\uc758 \uc9c4\ub3d9\uc218\uac00 \uc911\ub825\uc73c\ub85c \uc778\ud574 \ud3b8\uc774\ud55c\ub2e4( \uc911\ub825\uc801\uc0c9\ud3b8\uc774 )", "paragraph_sentence": " \uccab \ubc88\uc9f8 \uc0c8\ub85c\uc6b4 \ud6a8\uacfc\ub294 \ube5b\uc758 \uc9c4\ub3d9\uc218\uac00 \uc911\ub825\uc73c\ub85c \uc778\ud574 \ud3b8\uc774\ud55c\ub2e4( \uc911\ub825\uc801\uc0c9\ud3b8\uc774 ) \ub294 \uac83\uc774\ub2e4. \uac00\uc18d\ub418\ub294 \ub85c\ucf13\uc5d0 \ud0d1\uc2b9 \uc911\uc778 \uad00\ucc30\uc790 \ub450 \uba85\uc774 \uc788\ub2e4\uace0 \ud558\uc790. \uc6b0\uc8fc\uc120 \uc704\uc5d0\uc11c\ub294 \"\uc704\"\uc640 \"\uc544\ub798\"\uc758 \uc790\uc5f0\uc2a4\ub7ec\uc6b4 \uac1c\ub150\uc774 \uc874\uc7ac\ud55c\ub2e4. \uc6b0\uc8fc\uc120\uc774 \uac00\uc18d\ub418\ub294 \ubc29\ud5a5\uc774 \"\uc704\"\uc774\uace0, \uc774\uc5d0 \ub300\ud55c \uad00\uc131\uc73c\ub85c \uace0\uc815\ub418\uc9c0 \uc54a\uc740 \ubb3c\uccb4\uac00 \ubc18\ub300 \ubc29\ud5a5\uc73c\ub85c \uc6c0\uc9c1\uc774\uba74\uc11c \ub099\ud558\ud560 \ud14c\ub2c8 \uadf8\ucabd\uc774 \"\uc544\ub798\"\uc774\ub2e4. \uad00\ucc30\uc790 \uc911 \ud55c \uba85\uc774 \ub2e4\ub978 \ud55c \uba85 \ubcf4\ub2e4 \"\ub354 \uc704\"\uc5d0 \uc788\ub2e4\uace0 \uac00\uc815\ud574 \ubcf4\uc790. \uc544\ub798\ucabd\uc5d0 \uc788\ub294 \uad00\ucc30\uc790\uac00 \uc704\ucabd\uc5d0 \uc788\ub294 \uad00\ucc30\uc790\uc5d0\uac8c \ube5b\uc73c\ub85c \uc2e0\ud638\ub97c \ubcf4\ub0b4\uba74, \ub85c\ucf13\uc758 \uac00\uc18d \ub54c\ubb38\uc5d0 \ube5b\uc774 \uc801\uc0c9\ud3b8\uc774\ub97c \ud558\uac8c \ub41c\ub2e4(\uc774\ub294 \ud2b9\uc218\uc0c1\ub300\ub860\uc5d0\uc11c \uacc4\uc0b0\ub418\ub294 \ubc14\uc774\ub2e4). \uc704\ucabd\uc758 \uad00\ucc30\uc790\ub294 \uc544\ub798\ucabd\uc758 \uad00\ucc30\uc790\uac00 \ube5b\uc744 \ucc98\uc74c \uc3d8\uc558\uc744 \ub54c\uc640 \ube44\uad50\ud558\uc5ec \ub354 \ub0ae\uc740 \uc9c4\ub3d9\uc218\uc758 \ube5b\uc744 \uad00\ucc30\ud558\uac8c \ub420 \uac83\uc774\ub2e4. \uc5ed\uc73c\ub85c \uc704\ucabd\uc758 \uad00\ucc30\uc790\uac00 \uc544\ub798\ucabd\uc758 \uad00\ucc30\uc790\uc5d0\uac8c \ubcf4\ub0b4\ub294 \ube5b \uc2e0\ud638\ub294 \uccad\uc0c9\ud3b8\uc774, \uc989 \uc9c4\ub3d9\uc218\uac00 \ub354 \ub192\uc544\uc9c8 \uac83\uc774\ub2e4. \uc544\uc778\uc288\ud0c0\uc778\uc740 \uc774\ub7ec\ud55c \uc9c4\ub3d9\uc218 \ud3b8\uc774\uac00 \uc911\ub825\uc7a5 \uc548\uc5d0\uc11c \ubc18\ub4dc\uc2dc \uad00\ucc30\ub418\uc5b4\uc57c \ud55c\ub2e4\uace0 \ud588\ub2e4. \uc88c\uce21 \uadf8\ub9bc\uc774 \uc774\uac83\uc744 \ub098\ud0c0\ub0b8 \uac83\uc73c\ub85c\uc11c, \ube5b\uc774 \uc911\ub825\uc801 \uac00\uc18d\uc5d0 \ubc18\ub300\ub418\ub294 \ubc29\ud5a5\uc73c\ub85c \uc6c0\uc9c1\uc774\uba74\uc11c \uc11c\uc11c\ud788 \uc801\uc0c9\ud3b8\uc774\ud568\uc744 \ubcf4\uc5ec\uc900\ub2e4. \uc774 \ud6a8\uacfc\ub294 \uc2e4\ud5d8\uc801\uc73c\ub85c \uac80\uc99d\ub418\uc5c8\uc73c\uba70, \uadf8\uc5d0 \ub300\ud574\uc11c\ub294 \ud6c4\uc220\ud55c\ub2e4.", "paragraph_answer": "\uccab \ubc88\uc9f8 \uc0c8\ub85c\uc6b4 \ud6a8\uacfc\ub294 \ube5b\uc758 \uc9c4\ub3d9\uc218\uac00 \uc911\ub825\uc73c\ub85c \uc778\ud574 \ud3b8\uc774\ud55c\ub2e4( \uc911\ub825\uc801\uc0c9\ud3b8\uc774 )\ub294 \uac83\uc774\ub2e4. \uac00\uc18d\ub418\ub294 \ub85c\ucf13\uc5d0 \ud0d1\uc2b9 \uc911\uc778 \uad00\ucc30\uc790 \ub450 \uba85\uc774 \uc788\ub2e4\uace0 \ud558\uc790. \uc6b0\uc8fc\uc120 \uc704\uc5d0\uc11c\ub294 \"\uc704\"\uc640 \"\uc544\ub798\"\uc758 \uc790\uc5f0\uc2a4\ub7ec\uc6b4 \uac1c\ub150\uc774 \uc874\uc7ac\ud55c\ub2e4. \uc6b0\uc8fc\uc120\uc774 \uac00\uc18d\ub418\ub294 \ubc29\ud5a5\uc774 \"\uc704\"\uc774\uace0, \uc774\uc5d0 \ub300\ud55c \uad00\uc131\uc73c\ub85c \uace0\uc815\ub418\uc9c0 \uc54a\uc740 \ubb3c\uccb4\uac00 \ubc18\ub300 \ubc29\ud5a5\uc73c\ub85c \uc6c0\uc9c1\uc774\uba74\uc11c \ub099\ud558\ud560 \ud14c\ub2c8 \uadf8\ucabd\uc774 \"\uc544\ub798\"\uc774\ub2e4. \uad00\ucc30\uc790 \uc911 \ud55c \uba85\uc774 \ub2e4\ub978 \ud55c \uba85 \ubcf4\ub2e4 \"\ub354 \uc704\"\uc5d0 \uc788\ub2e4\uace0 \uac00\uc815\ud574 \ubcf4\uc790. \uc544\ub798\ucabd\uc5d0 \uc788\ub294 \uad00\ucc30\uc790\uac00 \uc704\ucabd\uc5d0 \uc788\ub294 \uad00\ucc30\uc790\uc5d0\uac8c \ube5b\uc73c\ub85c \uc2e0\ud638\ub97c \ubcf4\ub0b4\uba74, \ub85c\ucf13\uc758 \uac00\uc18d \ub54c\ubb38\uc5d0 \ube5b\uc774 \uc801\uc0c9\ud3b8\uc774\ub97c \ud558\uac8c \ub41c\ub2e4(\uc774\ub294 \ud2b9\uc218\uc0c1\ub300\ub860\uc5d0\uc11c \uacc4\uc0b0\ub418\ub294 \ubc14\uc774\ub2e4). \uc704\ucabd\uc758 \uad00\ucc30\uc790\ub294 \uc544\ub798\ucabd\uc758 \uad00\ucc30\uc790\uac00 \ube5b\uc744 \ucc98\uc74c \uc3d8\uc558\uc744 \ub54c\uc640 \ube44\uad50\ud558\uc5ec \ub354 \ub0ae\uc740 \uc9c4\ub3d9\uc218\uc758 \ube5b\uc744 \uad00\ucc30\ud558\uac8c \ub420 \uac83\uc774\ub2e4. \uc5ed\uc73c\ub85c \uc704\ucabd\uc758 \uad00\ucc30\uc790\uac00 \uc544\ub798\ucabd\uc758 \uad00\ucc30\uc790\uc5d0\uac8c \ubcf4\ub0b4\ub294 \ube5b \uc2e0\ud638\ub294 \uccad\uc0c9\ud3b8\uc774, \uc989 \uc9c4\ub3d9\uc218\uac00 \ub354 \ub192\uc544\uc9c8 \uac83\uc774\ub2e4. \uc544\uc778\uc288\ud0c0\uc778\uc740 \uc774\ub7ec\ud55c \uc9c4\ub3d9\uc218 \ud3b8\uc774\uac00 \uc911\ub825\uc7a5 \uc548\uc5d0\uc11c \ubc18\ub4dc\uc2dc \uad00\ucc30\ub418\uc5b4\uc57c \ud55c\ub2e4\uace0 \ud588\ub2e4. \uc88c\uce21 \uadf8\ub9bc\uc774 \uc774\uac83\uc744 \ub098\ud0c0\ub0b8 \uac83\uc73c\ub85c\uc11c, \ube5b\uc774 \uc911\ub825\uc801 \uac00\uc18d\uc5d0 \ubc18\ub300\ub418\ub294 \ubc29\ud5a5\uc73c\ub85c \uc6c0\uc9c1\uc774\uba74\uc11c \uc11c\uc11c\ud788 \uc801\uc0c9\ud3b8\uc774\ud568\uc744 \ubcf4\uc5ec\uc900\ub2e4. \uc774 \ud6a8\uacfc\ub294 \uc2e4\ud5d8\uc801\uc73c\ub85c \uac80\uc99d\ub418\uc5c8\uc73c\uba70, \uadf8\uc5d0 \ub300\ud574\uc11c\ub294 \ud6c4\uc220\ud55c\ub2e4.", "sentence_answer": "\uccab \ubc88\uc9f8 \uc0c8\ub85c\uc6b4 \ud6a8\uacfc\ub294 \ube5b\uc758 \uc9c4\ub3d9\uc218\uac00 \uc911\ub825\uc73c\ub85c \uc778\ud574 \ud3b8\uc774\ud55c\ub2e4( \uc911\ub825\uc801\uc0c9\ud3b8\uc774 )", "paragraph_id": "6548364-1-1", "paragraph_question": "question: \ube5b\uc758 \uc9c4\ub3d9\uc218\uac00 \uc911\ub825\ub54c\ubb38\uc5d0 \ud3b8\uc774\ud558\uac8c \ub418\ub294\uac83\uc744 \ubb34\uc5c7\uc774\ub77c\uace0 \ud558\ub294\uac00?, context: \uccab \ubc88\uc9f8 \uc0c8\ub85c\uc6b4 \ud6a8\uacfc\ub294 \ube5b\uc758 \uc9c4\ub3d9\uc218\uac00 \uc911\ub825\uc73c\ub85c \uc778\ud574 \ud3b8\uc774\ud55c\ub2e4(\uc911\ub825\uc801\uc0c9\ud3b8\uc774)\ub294 \uac83\uc774\ub2e4. \uac00\uc18d\ub418\ub294 \ub85c\ucf13\uc5d0 \ud0d1\uc2b9 \uc911\uc778 \uad00\ucc30\uc790 \ub450 \uba85\uc774 \uc788\ub2e4\uace0 \ud558\uc790. \uc6b0\uc8fc\uc120 \uc704\uc5d0\uc11c\ub294 \"\uc704\"\uc640 \"\uc544\ub798\"\uc758 \uc790\uc5f0\uc2a4\ub7ec\uc6b4 \uac1c\ub150\uc774 \uc874\uc7ac\ud55c\ub2e4. \uc6b0\uc8fc\uc120\uc774 \uac00\uc18d\ub418\ub294 \ubc29\ud5a5\uc774 \"\uc704\"\uc774\uace0, \uc774\uc5d0 \ub300\ud55c \uad00\uc131\uc73c\ub85c \uace0\uc815\ub418\uc9c0 \uc54a\uc740 \ubb3c\uccb4\uac00 \ubc18\ub300 \ubc29\ud5a5\uc73c\ub85c \uc6c0\uc9c1\uc774\uba74\uc11c \ub099\ud558\ud560 \ud14c\ub2c8 \uadf8\ucabd\uc774 \"\uc544\ub798\"\uc774\ub2e4. \uad00\ucc30\uc790 \uc911 \ud55c \uba85\uc774 \ub2e4\ub978 \ud55c \uba85 \ubcf4\ub2e4 \"\ub354 \uc704\"\uc5d0 \uc788\ub2e4\uace0 \uac00\uc815\ud574 \ubcf4\uc790. \uc544\ub798\ucabd\uc5d0 \uc788\ub294 \uad00\ucc30\uc790\uac00 \uc704\ucabd\uc5d0 \uc788\ub294 \uad00\ucc30\uc790\uc5d0\uac8c \ube5b\uc73c\ub85c \uc2e0\ud638\ub97c \ubcf4\ub0b4\uba74, \ub85c\ucf13\uc758 \uac00\uc18d \ub54c\ubb38\uc5d0 \ube5b\uc774 \uc801\uc0c9\ud3b8\uc774\ub97c \ud558\uac8c \ub41c\ub2e4(\uc774\ub294 \ud2b9\uc218\uc0c1\ub300\ub860\uc5d0\uc11c \uacc4\uc0b0\ub418\ub294 \ubc14\uc774\ub2e4). \uc704\ucabd\uc758 \uad00\ucc30\uc790\ub294 \uc544\ub798\ucabd\uc758 \uad00\ucc30\uc790\uac00 \ube5b\uc744 \ucc98\uc74c \uc3d8\uc558\uc744 \ub54c\uc640 \ube44\uad50\ud558\uc5ec \ub354 \ub0ae\uc740 \uc9c4\ub3d9\uc218\uc758 \ube5b\uc744 \uad00\ucc30\ud558\uac8c \ub420 \uac83\uc774\ub2e4. \uc5ed\uc73c\ub85c \uc704\ucabd\uc758 \uad00\ucc30\uc790\uac00 \uc544\ub798\ucabd\uc758 \uad00\ucc30\uc790\uc5d0\uac8c \ubcf4\ub0b4\ub294 \ube5b \uc2e0\ud638\ub294 \uccad\uc0c9\ud3b8\uc774, \uc989 \uc9c4\ub3d9\uc218\uac00 \ub354 \ub192\uc544\uc9c8 \uac83\uc774\ub2e4. \uc544\uc778\uc288\ud0c0\uc778\uc740 \uc774\ub7ec\ud55c \uc9c4\ub3d9\uc218 \ud3b8\uc774\uac00 \uc911\ub825\uc7a5 \uc548\uc5d0\uc11c \ubc18\ub4dc\uc2dc \uad00\ucc30\ub418\uc5b4\uc57c \ud55c\ub2e4\uace0 \ud588\ub2e4. \uc88c\uce21 \uadf8\ub9bc\uc774 \uc774\uac83\uc744 \ub098\ud0c0\ub0b8 \uac83\uc73c\ub85c\uc11c, \ube5b\uc774 \uc911\ub825\uc801 \uac00\uc18d\uc5d0 \ubc18\ub300\ub418\ub294 \ubc29\ud5a5\uc73c\ub85c \uc6c0\uc9c1\uc774\uba74\uc11c \uc11c\uc11c\ud788 \uc801\uc0c9\ud3b8\uc774\ud568\uc744 \ubcf4\uc5ec\uc900\ub2e4. \uc774 \ud6a8\uacfc\ub294 \uc2e4\ud5d8\uc801\uc73c\ub85c \uac80\uc99d\ub418\uc5c8\uc73c\uba70, \uadf8\uc5d0 \ub300\ud574\uc11c\ub294 \ud6c4\uc220\ud55c\ub2e4."} {"question": "\ub300\ud55c\ub3c5\ub9bd\uc120\uc5b8\uc11c\uc5d0 \uc11c\uba85\uc744 \ud55c \ud574\ub294 \uc5b8\uc81c\uc778\uac00?", "paragraph": "\uadf8\ub7ec\ub098 \uc77c\uc81c\uc758 \uac10\uc2dc\uac00 \uc2ec\ud558\uc5ec\uc9c0\uc790 1913\ub144 5\uc6d4\uc5d0 \ub9cc\uc8fc\ub85c \ub9dd\uba85\ud558\uc5ec \ud1b5\ud654\ud604(\u901a\u5316\u7e23) \ud569\ub2c8\ud558(\u54c8\u6ce5\u6cb3)\uc5d0 \ub3c5\ub9bd\uad70 \uc591\uc131\uacfc \ud6c4\uc9c4\uad50\uc721\uc744 \uc704\ud574 \uc124\ub9bd\ub41c \uc2e0\ud765\uac15\uc2b5\uc18c(\u65b0\u8208\u8b1b\u7fd2\u6240)\uc758 \uc18c\uc7a5(\u6240\u9577) \uc11c\ub9ac\uc5d0 \ucde8\uc784\ud558\uc600\ub2e4. 1917\ub144 6\uc6d4\uc5d0\ub294 \uc2e0\ud765\uac15\uc2b5\uc18c\uc758 \uc18c\uc7a5\uc774 \ub418\uc5c8\uc73c\uba70, \uac19\uc740 \ud574 11\uc6d4\uc5d0 \uc2e0\ud765\uac15\uc2b5\uc18c\uac00 \uc2e0\ud765\ubb34\uad00\ud559\uad50(\u65b0\u8208\u6b66\u5b98\u5b78\u6821)\ub85c \uac1c\uce6d\ub428\uc5d0 \ub530\ub77c \uad50\uc7a5\uc73c\ub85c \ucde8\uc784\ud558\uc5ec \uc870\uad6d \uad11\ubcf5\uc758 \ud575\uc2ec \uc778\ubb3c\uc778 \uccad\ub144\uad70\uad00\uc758 \uc591\uc131\uc5d0 \ub354\uc6b1 \uc8fc\ub825\ud558\uc600\ub2e4. 1918\ub144 11\uc6d4\uc5d0\ub294 \uae38\ub9bc(\u5409\u6797)\uc5d0\uc11c \uc5ec\uc900(\u5442\u6e96) \uc815\uc548\ub9bd(\u912d\u5b89\u7acb) \uae40\ub3d9\uc0bc(\u91d1\u6771\u4e09) \uae40\uc88c\uc9c4(\u91d1\u4f50\u93ad) \ub4f1 39\uc778\uc758 1\uc778\uc73c\ub85c \ub300\ud55c\ub3c5\ub9bd\uc120\uc5b8\uc11c\uc5d0 \uc11c\uba85\ud558\uc600\ub2e4. 1919\ub144 3\u00b71\ub3c5\ub9bd\uc6b4\ub3d9\uc774 \uc77c\uc5b4\ub098\uc790, \ud658\uc778\ud604(\u6853\u4ec1\u7e23)\uc5d0 \uac70\uc8fc\ud558\ub358 \uadf8\ub294 \uc190\uadf9\uc7a5(\u5b6b\u514b\u7ae0) \ub3c5\uace0\uc6b1(\u7368\u5b64\u65ed) \ub3c5\uace0\ud658(\u7368\u5b64\u7165) \ub4f1\uacfc \ud568\uaed8 \uadfc\ucc98\uc758 \uccad\ub144\ub4e4\uc744 \ubaa8\uc9d1\ud558\uc5ec \ub3c5\ub9bd\uc6b4\ub3d9 \ub2e8\uccb4\uc778 \ud55c\uad50\uacf5\ud68c(\u97d3\u50d1\u516c\u6703)\ub97c \ubc1c\uc871\uc2dc\ucf30\ub2e4.", "answer": "1918\ub144", "sentence": "1918\ub144 11\uc6d4\uc5d0\ub294 \uae38\ub9bc(\u5409\u6797)\uc5d0\uc11c \uc5ec\uc900(\u5442\u6e96) \uc815\uc548\ub9bd(\u912d\u5b89\u7acb) \uae40\ub3d9\uc0bc(\u91d1\u6771\u4e09) \uae40\uc88c\uc9c4(\u91d1\u4f50\u93ad) \ub4f1 39\uc778\uc758 1\uc778\uc73c\ub85c \ub300\ud55c\ub3c5\ub9bd\uc120\uc5b8\uc11c\uc5d0 \uc11c\uba85\ud558\uc600\ub2e4.", "paragraph_sentence": "\uadf8\ub7ec\ub098 \uc77c\uc81c\uc758 \uac10\uc2dc\uac00 \uc2ec\ud558\uc5ec\uc9c0\uc790 1913\ub144 5\uc6d4\uc5d0 \ub9cc\uc8fc\ub85c \ub9dd\uba85\ud558\uc5ec \ud1b5\ud654\ud604(\u901a\u5316\u7e23) \ud569\ub2c8\ud558(\u54c8\u6ce5\u6cb3)\uc5d0 \ub3c5\ub9bd\uad70 \uc591\uc131\uacfc \ud6c4\uc9c4\uad50\uc721\uc744 \uc704\ud574 \uc124\ub9bd\ub41c \uc2e0\ud765\uac15\uc2b5\uc18c(\u65b0\u8208\u8b1b\u7fd2\u6240)\uc758 \uc18c\uc7a5(\u6240\u9577) \uc11c\ub9ac\uc5d0 \ucde8\uc784\ud558\uc600\ub2e4. 1917\ub144 6\uc6d4\uc5d0\ub294 \uc2e0\ud765\uac15\uc2b5\uc18c\uc758 \uc18c\uc7a5\uc774 \ub418\uc5c8\uc73c\uba70, \uac19\uc740 \ud574 11\uc6d4\uc5d0 \uc2e0\ud765\uac15\uc2b5\uc18c\uac00 \uc2e0\ud765\ubb34\uad00\ud559\uad50(\u65b0\u8208\u6b66\u5b98\u5b78\u6821)\ub85c \uac1c\uce6d\ub428\uc5d0 \ub530\ub77c \uad50\uc7a5\uc73c\ub85c \ucde8\uc784\ud558\uc5ec \uc870\uad6d \uad11\ubcf5\uc758 \ud575\uc2ec \uc778\ubb3c\uc778 \uccad\ub144\uad70\uad00\uc758 \uc591\uc131\uc5d0 \ub354\uc6b1 \uc8fc\ub825\ud558\uc600\ub2e4. 1918\ub144 11\uc6d4\uc5d0\ub294 \uae38\ub9bc(\u5409\u6797)\uc5d0\uc11c \uc5ec\uc900(\u5442\u6e96) \uc815\uc548\ub9bd(\u912d\u5b89\u7acb) \uae40\ub3d9\uc0bc(\u91d1\u6771\u4e09) \uae40\uc88c\uc9c4(\u91d1\u4f50\u93ad) \ub4f1 39\uc778\uc758 1\uc778\uc73c\ub85c \ub300\ud55c\ub3c5\ub9bd\uc120\uc5b8\uc11c\uc5d0 \uc11c\uba85\ud558\uc600\ub2e4. 1919\ub144 3\u00b71\ub3c5\ub9bd\uc6b4\ub3d9\uc774 \uc77c\uc5b4\ub098\uc790, \ud658\uc778\ud604(\u6853\u4ec1\u7e23)\uc5d0 \uac70\uc8fc\ud558\ub358 \uadf8\ub294 \uc190\uadf9\uc7a5(\u5b6b\u514b\u7ae0) \ub3c5\uace0\uc6b1(\u7368\u5b64\u65ed) \ub3c5\uace0\ud658(\u7368\u5b64\u7165) \ub4f1\uacfc \ud568\uaed8 \uadfc\ucc98\uc758 \uccad\ub144\ub4e4\uc744 \ubaa8\uc9d1\ud558\uc5ec \ub3c5\ub9bd\uc6b4\ub3d9 \ub2e8\uccb4\uc778 \ud55c\uad50\uacf5\ud68c(\u97d3\u50d1\u516c\u6703)\ub97c \ubc1c\uc871\uc2dc\ucf30\ub2e4.", "paragraph_answer": "\uadf8\ub7ec\ub098 \uc77c\uc81c\uc758 \uac10\uc2dc\uac00 \uc2ec\ud558\uc5ec\uc9c0\uc790 1913\ub144 5\uc6d4\uc5d0 \ub9cc\uc8fc\ub85c \ub9dd\uba85\ud558\uc5ec \ud1b5\ud654\ud604(\u901a\u5316\u7e23) \ud569\ub2c8\ud558(\u54c8\u6ce5\u6cb3)\uc5d0 \ub3c5\ub9bd\uad70 \uc591\uc131\uacfc \ud6c4\uc9c4\uad50\uc721\uc744 \uc704\ud574 \uc124\ub9bd\ub41c \uc2e0\ud765\uac15\uc2b5\uc18c(\u65b0\u8208\u8b1b\u7fd2\u6240)\uc758 \uc18c\uc7a5(\u6240\u9577) \uc11c\ub9ac\uc5d0 \ucde8\uc784\ud558\uc600\ub2e4. 1917\ub144 6\uc6d4\uc5d0\ub294 \uc2e0\ud765\uac15\uc2b5\uc18c\uc758 \uc18c\uc7a5\uc774 \ub418\uc5c8\uc73c\uba70, \uac19\uc740 \ud574 11\uc6d4\uc5d0 \uc2e0\ud765\uac15\uc2b5\uc18c\uac00 \uc2e0\ud765\ubb34\uad00\ud559\uad50(\u65b0\u8208\u6b66\u5b98\u5b78\u6821)\ub85c \uac1c\uce6d\ub428\uc5d0 \ub530\ub77c \uad50\uc7a5\uc73c\ub85c \ucde8\uc784\ud558\uc5ec \uc870\uad6d \uad11\ubcf5\uc758 \ud575\uc2ec \uc778\ubb3c\uc778 \uccad\ub144\uad70\uad00\uc758 \uc591\uc131\uc5d0 \ub354\uc6b1 \uc8fc\ub825\ud558\uc600\ub2e4. 1918\ub144 11\uc6d4\uc5d0\ub294 \uae38\ub9bc(\u5409\u6797)\uc5d0\uc11c \uc5ec\uc900(\u5442\u6e96) \uc815\uc548\ub9bd(\u912d\u5b89\u7acb) \uae40\ub3d9\uc0bc(\u91d1\u6771\u4e09) \uae40\uc88c\uc9c4(\u91d1\u4f50\u93ad) \ub4f1 39\uc778\uc758 1\uc778\uc73c\ub85c \ub300\ud55c\ub3c5\ub9bd\uc120\uc5b8\uc11c\uc5d0 \uc11c\uba85\ud558\uc600\ub2e4. 1919\ub144 3\u00b71\ub3c5\ub9bd\uc6b4\ub3d9\uc774 \uc77c\uc5b4\ub098\uc790, \ud658\uc778\ud604(\u6853\u4ec1\u7e23)\uc5d0 \uac70\uc8fc\ud558\ub358 \uadf8\ub294 \uc190\uadf9\uc7a5(\u5b6b\u514b\u7ae0) \ub3c5\uace0\uc6b1(\u7368\u5b64\u65ed) \ub3c5\uace0\ud658(\u7368\u5b64\u7165) \ub4f1\uacfc \ud568\uaed8 \uadfc\ucc98\uc758 \uccad\ub144\ub4e4\uc744 \ubaa8\uc9d1\ud558\uc5ec \ub3c5\ub9bd\uc6b4\ub3d9 \ub2e8\uccb4\uc778 \ud55c\uad50\uacf5\ud68c(\u97d3\u50d1\u516c\u6703)\ub97c \ubc1c\uc871\uc2dc\ucf30\ub2e4.", "sentence_answer": " 1918\ub144 11\uc6d4\uc5d0\ub294 \uae38\ub9bc(\u5409\u6797)\uc5d0\uc11c \uc5ec\uc900(\u5442\u6e96) \uc815\uc548\ub9bd(\u912d\u5b89\u7acb) \uae40\ub3d9\uc0bc(\u91d1\u6771\u4e09) \uae40\uc88c\uc9c4(\u91d1\u4f50\u93ad) \ub4f1 39\uc778\uc758 1\uc778\uc73c\ub85c \ub300\ud55c\ub3c5\ub9bd\uc120\uc5b8\uc11c\uc5d0 \uc11c\uba85\ud558\uc600\ub2e4.", "paragraph_id": "6161564-1-1", "paragraph_question": "question: \ub300\ud55c\ub3c5\ub9bd\uc120\uc5b8\uc11c\uc5d0 \uc11c\uba85\uc744 \ud55c \ud574\ub294 \uc5b8\uc81c\uc778\uac00?, context: \uadf8\ub7ec\ub098 \uc77c\uc81c\uc758 \uac10\uc2dc\uac00 \uc2ec\ud558\uc5ec\uc9c0\uc790 1913\ub144 5\uc6d4\uc5d0 \ub9cc\uc8fc\ub85c \ub9dd\uba85\ud558\uc5ec \ud1b5\ud654\ud604(\u901a\u5316\u7e23) \ud569\ub2c8\ud558(\u54c8\u6ce5\u6cb3)\uc5d0 \ub3c5\ub9bd\uad70 \uc591\uc131\uacfc \ud6c4\uc9c4\uad50\uc721\uc744 \uc704\ud574 \uc124\ub9bd\ub41c \uc2e0\ud765\uac15\uc2b5\uc18c(\u65b0\u8208\u8b1b\u7fd2\u6240)\uc758 \uc18c\uc7a5(\u6240\u9577) \uc11c\ub9ac\uc5d0 \ucde8\uc784\ud558\uc600\ub2e4. 1917\ub144 6\uc6d4\uc5d0\ub294 \uc2e0\ud765\uac15\uc2b5\uc18c\uc758 \uc18c\uc7a5\uc774 \ub418\uc5c8\uc73c\uba70, \uac19\uc740 \ud574 11\uc6d4\uc5d0 \uc2e0\ud765\uac15\uc2b5\uc18c\uac00 \uc2e0\ud765\ubb34\uad00\ud559\uad50(\u65b0\u8208\u6b66\u5b98\u5b78\u6821)\ub85c \uac1c\uce6d\ub428\uc5d0 \ub530\ub77c \uad50\uc7a5\uc73c\ub85c \ucde8\uc784\ud558\uc5ec \uc870\uad6d \uad11\ubcf5\uc758 \ud575\uc2ec \uc778\ubb3c\uc778 \uccad\ub144\uad70\uad00\uc758 \uc591\uc131\uc5d0 \ub354\uc6b1 \uc8fc\ub825\ud558\uc600\ub2e4. 1918\ub144 11\uc6d4\uc5d0\ub294 \uae38\ub9bc(\u5409\u6797)\uc5d0\uc11c \uc5ec\uc900(\u5442\u6e96) \uc815\uc548\ub9bd(\u912d\u5b89\u7acb) \uae40\ub3d9\uc0bc(\u91d1\u6771\u4e09) \uae40\uc88c\uc9c4(\u91d1\u4f50\u93ad) \ub4f1 39\uc778\uc758 1\uc778\uc73c\ub85c \ub300\ud55c\ub3c5\ub9bd\uc120\uc5b8\uc11c\uc5d0 \uc11c\uba85\ud558\uc600\ub2e4. 1919\ub144 3\u00b71\ub3c5\ub9bd\uc6b4\ub3d9\uc774 \uc77c\uc5b4\ub098\uc790, \ud658\uc778\ud604(\u6853\u4ec1\u7e23)\uc5d0 \uac70\uc8fc\ud558\ub358 \uadf8\ub294 \uc190\uadf9\uc7a5(\u5b6b\u514b\u7ae0) \ub3c5\uace0\uc6b1(\u7368\u5b64\u65ed) \ub3c5\uace0\ud658(\u7368\u5b64\u7165) \ub4f1\uacfc \ud568\uaed8 \uadfc\ucc98\uc758 \uccad\ub144\ub4e4\uc744 \ubaa8\uc9d1\ud558\uc5ec \ub3c5\ub9bd\uc6b4\ub3d9 \ub2e8\uccb4\uc778 \ud55c\uad50\uacf5\ud68c(\u97d3\u50d1\u516c\u6703)\ub97c \ubc1c\uc871\uc2dc\ucf30\ub2e4."} {"question": "Airbag \ubcf4\uceec\ud53c\uccd0\ub9c1\uc5d0 \ub300\ud574 \uae0d\uc815\uc801\uc778 \ud3c9\uc744 \ub0b4\ub193\uc740 \uc778\ubb3c\uc740?", "paragraph": "\uc74c\uc545 \ud3c9\ub860 \uc0ac\uc774\ud2b8 \uc774\uc998\uc758 \ud669\uc120\uc5c5\uc740 \"Airbag\"\uc740 \ud0c0\ube14\ub85c \uc790\uc2e0\uc758 \uc790\uc804\uc801 \uc774\uc57c\uae30\ub85c\uc11c, \uace1\uc758 \uc815\uc11c\uac00 \"\uc608\uc0c1\ub300\ub85c \uc6b0\uc6b8\"\ud558\ub2e4\uace0 \ud3c9\uac00\ud588\ub2e4. \uadf8\ub294 \ud0c0\ube14\ub85c\uc758 \ub7a9\uc774 \ub178\ub798\uc758 \uc911\uc2ec\uc5d0 \uc11c\uc788\ub2e4\uba70, \ub098\uc5bc\uc758 \ubcf4\uceec\uacfc \uc5b4\ucfe0\uc2a4\ud2f1 \uc0ac\uc6b4\ub4dc \ubcf4\ub2e4\ub3c4 \"\ud0c0\ube14\ub85c\uc758 \ub798\ud551\uc774 \uc774 \uace1\uc758 \uc8fc\uc778\uacf5\"\uc774\ub77c \ud310\ub2e8\ud588\ub2e4. \uadf8\ub294 \ud0c0\ube14\ub85c\uc758 \"\uc74c\uc545\uc801 \uac10\uac01\uc740 \ucef4\ubc31\uc791\uc5d0\uc11c\ub3c4 \uc5ec\uacfc \uc5c6\uc774 \ubc1c\ud718\"\ub41c\ub2e4\uba70 \ud0c0\ube14\ub85c\uc758 \ub6f0\uc5b4\ub09c \uac10\uac01\uc744 \ud638\ud3c9\ud588\uc73c\uba70, \"\uc544\ud508 \ub9cc\ud07c \uc88b\uc740 \uc791\ud488\uc774 \ub098\uc628\ub2e4\ub294 \uc544\ud2f0\uc2a4\ud2b8\ub4e4\uc758 \uc544\uc774\ub7ec\ub2c8\ud55c \ubc95\uce59\uc744 \ub300\ubcc0\ud558\ub294 \uc99d\uac70 \uc791\"\uc774\ub77c\uba70 \ud3c9\uc810 5\uc810 \ub9cc\uc810\uc5d0 3.5\uc810\uc744 \uc92c\ub2e4. \uc74c\uc545 \ub9e4\uac70\uc9c4 \ub9ac\ub4dc\uba38\uc758 \uc774\ubcd1\uc8fc\ub294 \u300a\uc5f4\uaf43\u300b \uc74c\ubc18\uc758 \ub9ac\ubdf0\uc5d0\uc11c \"\ub098\uc5bc\uc744 \ube44\ub86f\ud55c \uc801\uc808\ud55c \ubcf4\uceec\uc758 \ucc38\uc5ec\ub3c4 \uace1\uc758 \uc644\uc131\ub3c4\uc5d0 \ud070 \uc5ed\ud560\uc744 \ud558\uace0 \uc788\ub2e4\"\uba70 \uc774 \uace1\uc758 \ubcf4\uceec \ud53c\uccd0\ub9c1\uc5d0 \ub300\ud574 \uae0d\uc815\uc801\uc778 \ud3c9\uc744 \ub0b4\ub193\uc558\ub2e4.", "answer": "\uc774\ubcd1\uc8fc", "sentence": "\uc74c\uc545 \ub9e4\uac70\uc9c4 \ub9ac\ub4dc\uba38\uc758 \uc774\ubcd1\uc8fc \ub294 \u300a\uc5f4\uaf43\u300b \uc74c\ubc18\uc758 \ub9ac\ubdf0\uc5d0\uc11c \"\ub098\uc5bc\uc744 \ube44\ub86f\ud55c \uc801\uc808\ud55c \ubcf4\uceec\uc758 \ucc38\uc5ec\ub3c4 \uace1\uc758 \uc644\uc131\ub3c4\uc5d0 \ud070 \uc5ed\ud560\uc744 \ud558\uace0 \uc788\ub2e4\"\uba70 \uc774 \uace1\uc758 \ubcf4\uceec \ud53c\uccd0\ub9c1\uc5d0 \ub300\ud574 \uae0d\uc815\uc801\uc778 \ud3c9\uc744 \ub0b4\ub193\uc558\ub2e4.", "paragraph_sentence": "\uc74c\uc545 \ud3c9\ub860 \uc0ac\uc774\ud2b8 \uc774\uc998\uc758 \ud669\uc120\uc5c5\uc740 \"Airbag\"\uc740 \ud0c0\ube14\ub85c \uc790\uc2e0\uc758 \uc790\uc804\uc801 \uc774\uc57c\uae30\ub85c\uc11c, \uace1\uc758 \uc815\uc11c\uac00 \"\uc608\uc0c1\ub300\ub85c \uc6b0\uc6b8\"\ud558\ub2e4\uace0 \ud3c9\uac00\ud588\ub2e4. \uadf8\ub294 \ud0c0\ube14\ub85c\uc758 \ub7a9\uc774 \ub178\ub798\uc758 \uc911\uc2ec\uc5d0 \uc11c\uc788\ub2e4\uba70, \ub098\uc5bc\uc758 \ubcf4\uceec\uacfc \uc5b4\ucfe0\uc2a4\ud2f1 \uc0ac\uc6b4\ub4dc \ubcf4\ub2e4\ub3c4 \"\ud0c0\ube14\ub85c\uc758 \ub798\ud551\uc774 \uc774 \uace1\uc758 \uc8fc\uc778\uacf5\"\uc774\ub77c \ud310\ub2e8\ud588\ub2e4. \uadf8\ub294 \ud0c0\ube14\ub85c\uc758 \"\uc74c\uc545\uc801 \uac10\uac01\uc740 \ucef4\ubc31\uc791\uc5d0\uc11c\ub3c4 \uc5ec\uacfc \uc5c6\uc774 \ubc1c\ud718\"\ub41c\ub2e4\uba70 \ud0c0\ube14\ub85c\uc758 \ub6f0\uc5b4\ub09c \uac10\uac01\uc744 \ud638\ud3c9\ud588\uc73c\uba70, \"\uc544\ud508 \ub9cc\ud07c \uc88b\uc740 \uc791\ud488\uc774 \ub098\uc628\ub2e4\ub294 \uc544\ud2f0\uc2a4\ud2b8\ub4e4\uc758 \uc544\uc774\ub7ec\ub2c8\ud55c \ubc95\uce59\uc744 \ub300\ubcc0\ud558\ub294 \uc99d\uac70 \uc791\"\uc774\ub77c\uba70 \ud3c9\uc810 5\uc810 \ub9cc\uc810\uc5d0 3.5\uc810\uc744 \uc92c\ub2e4. \uc74c\uc545 \ub9e4\uac70\uc9c4 \ub9ac\ub4dc\uba38\uc758 \uc774\ubcd1\uc8fc \ub294 \u300a\uc5f4\uaf43\u300b \uc74c\ubc18\uc758 \ub9ac\ubdf0\uc5d0\uc11c \"\ub098\uc5bc\uc744 \ube44\ub86f\ud55c \uc801\uc808\ud55c \ubcf4\uceec\uc758 \ucc38\uc5ec\ub3c4 \uace1\uc758 \uc644\uc131\ub3c4\uc5d0 \ud070 \uc5ed\ud560\uc744 \ud558\uace0 \uc788\ub2e4\"\uba70 \uc774 \uace1\uc758 \ubcf4\uceec \ud53c\uccd0\ub9c1\uc5d0 \ub300\ud574 \uae0d\uc815\uc801\uc778 \ud3c9\uc744 \ub0b4\ub193\uc558\ub2e4. ", "paragraph_answer": "\uc74c\uc545 \ud3c9\ub860 \uc0ac\uc774\ud2b8 \uc774\uc998\uc758 \ud669\uc120\uc5c5\uc740 \"Airbag\"\uc740 \ud0c0\ube14\ub85c \uc790\uc2e0\uc758 \uc790\uc804\uc801 \uc774\uc57c\uae30\ub85c\uc11c, \uace1\uc758 \uc815\uc11c\uac00 \"\uc608\uc0c1\ub300\ub85c \uc6b0\uc6b8\"\ud558\ub2e4\uace0 \ud3c9\uac00\ud588\ub2e4. \uadf8\ub294 \ud0c0\ube14\ub85c\uc758 \ub7a9\uc774 \ub178\ub798\uc758 \uc911\uc2ec\uc5d0 \uc11c\uc788\ub2e4\uba70, \ub098\uc5bc\uc758 \ubcf4\uceec\uacfc \uc5b4\ucfe0\uc2a4\ud2f1 \uc0ac\uc6b4\ub4dc \ubcf4\ub2e4\ub3c4 \"\ud0c0\ube14\ub85c\uc758 \ub798\ud551\uc774 \uc774 \uace1\uc758 \uc8fc\uc778\uacf5\"\uc774\ub77c \ud310\ub2e8\ud588\ub2e4. \uadf8\ub294 \ud0c0\ube14\ub85c\uc758 \"\uc74c\uc545\uc801 \uac10\uac01\uc740 \ucef4\ubc31\uc791\uc5d0\uc11c\ub3c4 \uc5ec\uacfc \uc5c6\uc774 \ubc1c\ud718\"\ub41c\ub2e4\uba70 \ud0c0\ube14\ub85c\uc758 \ub6f0\uc5b4\ub09c \uac10\uac01\uc744 \ud638\ud3c9\ud588\uc73c\uba70, \"\uc544\ud508 \ub9cc\ud07c \uc88b\uc740 \uc791\ud488\uc774 \ub098\uc628\ub2e4\ub294 \uc544\ud2f0\uc2a4\ud2b8\ub4e4\uc758 \uc544\uc774\ub7ec\ub2c8\ud55c \ubc95\uce59\uc744 \ub300\ubcc0\ud558\ub294 \uc99d\uac70 \uc791\"\uc774\ub77c\uba70 \ud3c9\uc810 5\uc810 \ub9cc\uc810\uc5d0 3.5\uc810\uc744 \uc92c\ub2e4. \uc74c\uc545 \ub9e4\uac70\uc9c4 \ub9ac\ub4dc\uba38\uc758 \uc774\ubcd1\uc8fc \ub294 \u300a\uc5f4\uaf43\u300b \uc74c\ubc18\uc758 \ub9ac\ubdf0\uc5d0\uc11c \"\ub098\uc5bc\uc744 \ube44\ub86f\ud55c \uc801\uc808\ud55c \ubcf4\uceec\uc758 \ucc38\uc5ec\ub3c4 \uace1\uc758 \uc644\uc131\ub3c4\uc5d0 \ud070 \uc5ed\ud560\uc744 \ud558\uace0 \uc788\ub2e4\"\uba70 \uc774 \uace1\uc758 \ubcf4\uceec \ud53c\uccd0\ub9c1\uc5d0 \ub300\ud574 \uae0d\uc815\uc801\uc778 \ud3c9\uc744 \ub0b4\ub193\uc558\ub2e4.", "sentence_answer": "\uc74c\uc545 \ub9e4\uac70\uc9c4 \ub9ac\ub4dc\uba38\uc758 \uc774\ubcd1\uc8fc \ub294 \u300a\uc5f4\uaf43\u300b \uc74c\ubc18\uc758 \ub9ac\ubdf0\uc5d0\uc11c \"\ub098\uc5bc\uc744 \ube44\ub86f\ud55c \uc801\uc808\ud55c \ubcf4\uceec\uc758 \ucc38\uc5ec\ub3c4 \uace1\uc758 \uc644\uc131\ub3c4\uc5d0 \ud070 \uc5ed\ud560\uc744 \ud558\uace0 \uc788\ub2e4\"\uba70 \uc774 \uace1\uc758 \ubcf4\uceec \ud53c\uccd0\ub9c1\uc5d0 \ub300\ud574 \uae0d\uc815\uc801\uc778 \ud3c9\uc744 \ub0b4\ub193\uc558\ub2e4.", "paragraph_id": "6656680-1-1", "paragraph_question": "question: Airbag \ubcf4\uceec\ud53c\uccd0\ub9c1\uc5d0 \ub300\ud574 \uae0d\uc815\uc801\uc778 \ud3c9\uc744 \ub0b4\ub193\uc740 \uc778\ubb3c\uc740?, context: \uc74c\uc545 \ud3c9\ub860 \uc0ac\uc774\ud2b8 \uc774\uc998\uc758 \ud669\uc120\uc5c5\uc740 \"Airbag\"\uc740 \ud0c0\ube14\ub85c \uc790\uc2e0\uc758 \uc790\uc804\uc801 \uc774\uc57c\uae30\ub85c\uc11c, \uace1\uc758 \uc815\uc11c\uac00 \"\uc608\uc0c1\ub300\ub85c \uc6b0\uc6b8\"\ud558\ub2e4\uace0 \ud3c9\uac00\ud588\ub2e4. \uadf8\ub294 \ud0c0\ube14\ub85c\uc758 \ub7a9\uc774 \ub178\ub798\uc758 \uc911\uc2ec\uc5d0 \uc11c\uc788\ub2e4\uba70, \ub098\uc5bc\uc758 \ubcf4\uceec\uacfc \uc5b4\ucfe0\uc2a4\ud2f1 \uc0ac\uc6b4\ub4dc \ubcf4\ub2e4\ub3c4 \"\ud0c0\ube14\ub85c\uc758 \ub798\ud551\uc774 \uc774 \uace1\uc758 \uc8fc\uc778\uacf5\"\uc774\ub77c \ud310\ub2e8\ud588\ub2e4. \uadf8\ub294 \ud0c0\ube14\ub85c\uc758 \"\uc74c\uc545\uc801 \uac10\uac01\uc740 \ucef4\ubc31\uc791\uc5d0\uc11c\ub3c4 \uc5ec\uacfc \uc5c6\uc774 \ubc1c\ud718\"\ub41c\ub2e4\uba70 \ud0c0\ube14\ub85c\uc758 \ub6f0\uc5b4\ub09c \uac10\uac01\uc744 \ud638\ud3c9\ud588\uc73c\uba70, \"\uc544\ud508 \ub9cc\ud07c \uc88b\uc740 \uc791\ud488\uc774 \ub098\uc628\ub2e4\ub294 \uc544\ud2f0\uc2a4\ud2b8\ub4e4\uc758 \uc544\uc774\ub7ec\ub2c8\ud55c \ubc95\uce59\uc744 \ub300\ubcc0\ud558\ub294 \uc99d\uac70 \uc791\"\uc774\ub77c\uba70 \ud3c9\uc810 5\uc810 \ub9cc\uc810\uc5d0 3.5\uc810\uc744 \uc92c\ub2e4. \uc74c\uc545 \ub9e4\uac70\uc9c4 \ub9ac\ub4dc\uba38\uc758 \uc774\ubcd1\uc8fc\ub294 \u300a\uc5f4\uaf43\u300b \uc74c\ubc18\uc758 \ub9ac\ubdf0\uc5d0\uc11c \"\ub098\uc5bc\uc744 \ube44\ub86f\ud55c \uc801\uc808\ud55c \ubcf4\uceec\uc758 \ucc38\uc5ec\ub3c4 \uace1\uc758 \uc644\uc131\ub3c4\uc5d0 \ud070 \uc5ed\ud560\uc744 \ud558\uace0 \uc788\ub2e4\"\uba70 \uc774 \uace1\uc758 \ubcf4\uceec \ud53c\uccd0\ub9c1\uc5d0 \ub300\ud574 \uae0d\uc815\uc801\uc778 \ud3c9\uc744 \ub0b4\ub193\uc558\ub2e4."} {"question": "\ub258\ub978\ubca0\ub974\ud06c \ud5cc\uc7a5 \uaddc\uc815 \uc81c 6\uc870 c\ud638\ub97c \uadfc\uac70\ub85c \uc720\uc8c4 \ud310\uacb0\uc744 \ubc1b\uc740 4\uba85\uc740 \ub204\uad6c\uc758 \uba85\ub839\uc744 \ub530\ub978 \uac83\uc778\uac00?", "paragraph": "\ubb38\ud654\uc7ac\uac00 \uc6d0\uc18c\uc720\uad6d\uc744 \ub5a0\ub098 \ud574\uc678\ub85c \ubc18\ucd9c\ub418\ub294 \ubd88\ubc95\uc801\uc778 \uc608\ub85c\ub294 \uc804\uc7c1\uc73c\ub85c \uc778\ud55c \uc720\ucd9c, \ub3c4\uad74 \ub610\ub294 \ub3c4\ub09c\uc744 \ud1b5\ud55c \ubd88\ubc95 \ubc00\ubc18\ucd9c, \uc2dd\ubbfc\uc9c0\ubc30 \ub610\ub294 \uc678\uad6d\uad70 \uc810\ub839\uc73c\ub85c \uc778\ud55c \uc720\ucd9c, \ud55c \uad6d\uac00\uac00 \uc5ec\ub7ec \ub098\ub77c\ub85c \ubd84\ub9ac\ub418\uba74\uc11c \uc18c\uc720\uad8c\uc774 \uc774\uc804\ub41c \uacbd\uc6b0 \ub4f1\uc774\ub2e4. \ub258\ub978\ubca0\ub974\ud06c \uc804\ubc94\uc7ac\ud310\uc18c \ud5cc\uc7a5 \uaddc\uc815 \uc81c6\uc870 c\ud638\ub294 \u201c\uad70\uc0ac\uc801 \ud544\uc694\uc5d0 \uc758\ud558\uc5ec \uc815\ub2f9\ud654\ub418\uc9c0 \uc544\ub2c8\ud558\ub294 \uacf5\uc720 \ub610\ub294 \uc0ac\uc720\uc7ac\uc0b0\uc758 \uc57d\ud0c8\ud589\uc704\ub294 \uba85\ubc31\ud55c \uc804\uc7c1\ubc94\uc8c4\ud589\uc704\u201d\ub77c\uace0 \uaddc\uc815\ud558\uc600\uace0, \uc774 \uaddc\uc815\uc744 \uadfc\uac70\ub85c \uc81c2\ucc28 \uc138\uacc4 \ub300\uc804 \uc911\uc5d0 \ud788\ud2c0\ub7ec\uc758 \uba85\ub839\uc73c\ub85c \ubb38\ud654\uc7ac\ub97c \uc57d\ud0c8\ud558\uc600\ub358 \ud53c\uace0\uc778 4\uc778\uc5d0\uac8c \uc720\uc8c4 \ud310\uacb0\uc744 \ub0b4\ub838\ub2e4. \u2018\ubb38\ud654\uc7ac\uc758 \ubd88\ubc95 \ubc18\ucd9c\uc785 \ubc0f \uc18c\uc720\uad8c \uc591\ub3c4 \uae08\uc9c0\uc640 \uc608\ubc29 \uc218\ub2e8\uc5d0 \uad00\ud55c \ud611\uc57d\u2019 \uc81c11\uc870\uc5d0 \ub530\ub974\uba74 \u2018\uc678\uad6d \uad70\ub300\uc5d0 \uc758\ud55c \uc77c\uad6d\uc758 \uc810\ub839\uc73c\ub85c\ubd80\ud130 \uc9c1\uc811\uc801\uc73c\ub85c \ub610\ub294 \uac04\uc811\uc801\uc73c\ub85c \ubc1c\uc0dd\ud558\ub294 \uac15\uc81c\uc801\uc778 \ubb38\ud654\uc7ac\uc758 \ubc18\ucd9c\uacfc \uc18c\uc720\uad8c\uc758 \uc591\ub3c4\ub294 \ubd88\ubc95\uc73c\ub85c \uac04\uc8fc\ub41c\ub2e4.\u2019\ub77c\uace0 \uaddc\uc815\ud558\uace0 \uc788\ub2e4.", "answer": "\ud788\ud2c0\ub7ec", "sentence": "\ub258\ub978\ubca0\ub974\ud06c \uc804\ubc94\uc7ac\ud310\uc18c \ud5cc\uc7a5 \uaddc\uc815 \uc81c6\uc870 c\ud638\ub294 \u201c\uad70\uc0ac\uc801 \ud544\uc694\uc5d0 \uc758\ud558\uc5ec \uc815\ub2f9\ud654\ub418\uc9c0 \uc544\ub2c8\ud558\ub294 \uacf5\uc720 \ub610\ub294 \uc0ac\uc720\uc7ac\uc0b0\uc758 \uc57d\ud0c8\ud589\uc704\ub294 \uba85\ubc31\ud55c \uc804\uc7c1\ubc94\uc8c4\ud589\uc704\u201d\ub77c\uace0 \uaddc\uc815\ud558\uc600\uace0, \uc774 \uaddc\uc815\uc744 \uadfc\uac70\ub85c \uc81c2\ucc28 \uc138\uacc4 \ub300\uc804 \uc911\uc5d0 \ud788\ud2c0\ub7ec \uc758 \uba85\ub839\uc73c\ub85c \ubb38\ud654\uc7ac\ub97c \uc57d\ud0c8\ud558\uc600\ub358 \ud53c\uace0\uc778 4\uc778\uc5d0\uac8c \uc720\uc8c4 \ud310\uacb0\uc744 \ub0b4\ub838\ub2e4.", "paragraph_sentence": "\ubb38\ud654\uc7ac\uac00 \uc6d0\uc18c\uc720\uad6d\uc744 \ub5a0\ub098 \ud574\uc678\ub85c \ubc18\ucd9c\ub418\ub294 \ubd88\ubc95\uc801\uc778 \uc608\ub85c\ub294 \uc804\uc7c1\uc73c\ub85c \uc778\ud55c \uc720\ucd9c, \ub3c4\uad74 \ub610\ub294 \ub3c4\ub09c\uc744 \ud1b5\ud55c \ubd88\ubc95 \ubc00\ubc18\ucd9c, \uc2dd\ubbfc\uc9c0\ubc30 \ub610\ub294 \uc678\uad6d\uad70 \uc810\ub839\uc73c\ub85c \uc778\ud55c \uc720\ucd9c, \ud55c \uad6d\uac00\uac00 \uc5ec\ub7ec \ub098\ub77c\ub85c \ubd84\ub9ac\ub418\uba74\uc11c \uc18c\uc720\uad8c\uc774 \uc774\uc804\ub41c \uacbd\uc6b0 \ub4f1\uc774\ub2e4. \ub258\ub978\ubca0\ub974\ud06c \uc804\ubc94\uc7ac\ud310\uc18c \ud5cc\uc7a5 \uaddc\uc815 \uc81c6\uc870 c\ud638\ub294 \u201c\uad70\uc0ac\uc801 \ud544\uc694\uc5d0 \uc758\ud558\uc5ec \uc815\ub2f9\ud654\ub418\uc9c0 \uc544\ub2c8\ud558\ub294 \uacf5\uc720 \ub610\ub294 \uc0ac\uc720\uc7ac\uc0b0\uc758 \uc57d\ud0c8\ud589\uc704\ub294 \uba85\ubc31\ud55c \uc804\uc7c1\ubc94\uc8c4\ud589\uc704\u201d\ub77c\uace0 \uaddc\uc815\ud558\uc600\uace0, \uc774 \uaddc\uc815\uc744 \uadfc\uac70\ub85c \uc81c2\ucc28 \uc138\uacc4 \ub300\uc804 \uc911\uc5d0 \ud788\ud2c0\ub7ec \uc758 \uba85\ub839\uc73c\ub85c \ubb38\ud654\uc7ac\ub97c \uc57d\ud0c8\ud558\uc600\ub358 \ud53c\uace0\uc778 4\uc778\uc5d0\uac8c \uc720\uc8c4 \ud310\uacb0\uc744 \ub0b4\ub838\ub2e4. \u2018\ubb38\ud654\uc7ac\uc758 \ubd88\ubc95 \ubc18\ucd9c\uc785 \ubc0f \uc18c\uc720\uad8c \uc591\ub3c4 \uae08\uc9c0\uc640 \uc608\ubc29 \uc218\ub2e8\uc5d0 \uad00\ud55c \ud611\uc57d\u2019 \uc81c11\uc870\uc5d0 \ub530\ub974\uba74 \u2018\uc678\uad6d \uad70\ub300\uc5d0 \uc758\ud55c \uc77c\uad6d\uc758 \uc810\ub839\uc73c\ub85c\ubd80\ud130 \uc9c1\uc811\uc801\uc73c\ub85c \ub610\ub294 \uac04\uc811\uc801\uc73c\ub85c \ubc1c\uc0dd\ud558\ub294 \uac15\uc81c\uc801\uc778 \ubb38\ud654\uc7ac\uc758 \ubc18\ucd9c\uacfc \uc18c\uc720\uad8c\uc758 \uc591\ub3c4\ub294 \ubd88\ubc95\uc73c\ub85c \uac04\uc8fc\ub41c\ub2e4. \u2019\ub77c\uace0 \uaddc\uc815\ud558\uace0 \uc788\ub2e4.", "paragraph_answer": "\ubb38\ud654\uc7ac\uac00 \uc6d0\uc18c\uc720\uad6d\uc744 \ub5a0\ub098 \ud574\uc678\ub85c \ubc18\ucd9c\ub418\ub294 \ubd88\ubc95\uc801\uc778 \uc608\ub85c\ub294 \uc804\uc7c1\uc73c\ub85c \uc778\ud55c \uc720\ucd9c, \ub3c4\uad74 \ub610\ub294 \ub3c4\ub09c\uc744 \ud1b5\ud55c \ubd88\ubc95 \ubc00\ubc18\ucd9c, \uc2dd\ubbfc\uc9c0\ubc30 \ub610\ub294 \uc678\uad6d\uad70 \uc810\ub839\uc73c\ub85c \uc778\ud55c \uc720\ucd9c, \ud55c \uad6d\uac00\uac00 \uc5ec\ub7ec \ub098\ub77c\ub85c \ubd84\ub9ac\ub418\uba74\uc11c \uc18c\uc720\uad8c\uc774 \uc774\uc804\ub41c \uacbd\uc6b0 \ub4f1\uc774\ub2e4. \ub258\ub978\ubca0\ub974\ud06c \uc804\ubc94\uc7ac\ud310\uc18c \ud5cc\uc7a5 \uaddc\uc815 \uc81c6\uc870 c\ud638\ub294 \u201c\uad70\uc0ac\uc801 \ud544\uc694\uc5d0 \uc758\ud558\uc5ec \uc815\ub2f9\ud654\ub418\uc9c0 \uc544\ub2c8\ud558\ub294 \uacf5\uc720 \ub610\ub294 \uc0ac\uc720\uc7ac\uc0b0\uc758 \uc57d\ud0c8\ud589\uc704\ub294 \uba85\ubc31\ud55c \uc804\uc7c1\ubc94\uc8c4\ud589\uc704\u201d\ub77c\uace0 \uaddc\uc815\ud558\uc600\uace0, \uc774 \uaddc\uc815\uc744 \uadfc\uac70\ub85c \uc81c2\ucc28 \uc138\uacc4 \ub300\uc804 \uc911\uc5d0 \ud788\ud2c0\ub7ec \uc758 \uba85\ub839\uc73c\ub85c \ubb38\ud654\uc7ac\ub97c \uc57d\ud0c8\ud558\uc600\ub358 \ud53c\uace0\uc778 4\uc778\uc5d0\uac8c \uc720\uc8c4 \ud310\uacb0\uc744 \ub0b4\ub838\ub2e4. \u2018\ubb38\ud654\uc7ac\uc758 \ubd88\ubc95 \ubc18\ucd9c\uc785 \ubc0f \uc18c\uc720\uad8c \uc591\ub3c4 \uae08\uc9c0\uc640 \uc608\ubc29 \uc218\ub2e8\uc5d0 \uad00\ud55c \ud611\uc57d\u2019 \uc81c11\uc870\uc5d0 \ub530\ub974\uba74 \u2018\uc678\uad6d \uad70\ub300\uc5d0 \uc758\ud55c \uc77c\uad6d\uc758 \uc810\ub839\uc73c\ub85c\ubd80\ud130 \uc9c1\uc811\uc801\uc73c\ub85c \ub610\ub294 \uac04\uc811\uc801\uc73c\ub85c \ubc1c\uc0dd\ud558\ub294 \uac15\uc81c\uc801\uc778 \ubb38\ud654\uc7ac\uc758 \ubc18\ucd9c\uacfc \uc18c\uc720\uad8c\uc758 \uc591\ub3c4\ub294 \ubd88\ubc95\uc73c\ub85c \uac04\uc8fc\ub41c\ub2e4.\u2019\ub77c\uace0 \uaddc\uc815\ud558\uace0 \uc788\ub2e4.", "sentence_answer": "\ub258\ub978\ubca0\ub974\ud06c \uc804\ubc94\uc7ac\ud310\uc18c \ud5cc\uc7a5 \uaddc\uc815 \uc81c6\uc870 c\ud638\ub294 \u201c\uad70\uc0ac\uc801 \ud544\uc694\uc5d0 \uc758\ud558\uc5ec \uc815\ub2f9\ud654\ub418\uc9c0 \uc544\ub2c8\ud558\ub294 \uacf5\uc720 \ub610\ub294 \uc0ac\uc720\uc7ac\uc0b0\uc758 \uc57d\ud0c8\ud589\uc704\ub294 \uba85\ubc31\ud55c \uc804\uc7c1\ubc94\uc8c4\ud589\uc704\u201d\ub77c\uace0 \uaddc\uc815\ud558\uc600\uace0, \uc774 \uaddc\uc815\uc744 \uadfc\uac70\ub85c \uc81c2\ucc28 \uc138\uacc4 \ub300\uc804 \uc911\uc5d0 \ud788\ud2c0\ub7ec \uc758 \uba85\ub839\uc73c\ub85c \ubb38\ud654\uc7ac\ub97c \uc57d\ud0c8\ud558\uc600\ub358 \ud53c\uace0\uc778 4\uc778\uc5d0\uac8c \uc720\uc8c4 \ud310\uacb0\uc744 \ub0b4\ub838\ub2e4.", "paragraph_id": "6657070-8-0", "paragraph_question": "question: \ub258\ub978\ubca0\ub974\ud06c \ud5cc\uc7a5 \uaddc\uc815 \uc81c 6\uc870 c\ud638\ub97c \uadfc\uac70\ub85c \uc720\uc8c4 \ud310\uacb0\uc744 \ubc1b\uc740 4\uba85\uc740 \ub204\uad6c\uc758 \uba85\ub839\uc744 \ub530\ub978 \uac83\uc778\uac00?, context: \ubb38\ud654\uc7ac\uac00 \uc6d0\uc18c\uc720\uad6d\uc744 \ub5a0\ub098 \ud574\uc678\ub85c \ubc18\ucd9c\ub418\ub294 \ubd88\ubc95\uc801\uc778 \uc608\ub85c\ub294 \uc804\uc7c1\uc73c\ub85c \uc778\ud55c \uc720\ucd9c, \ub3c4\uad74 \ub610\ub294 \ub3c4\ub09c\uc744 \ud1b5\ud55c \ubd88\ubc95 \ubc00\ubc18\ucd9c, \uc2dd\ubbfc\uc9c0\ubc30 \ub610\ub294 \uc678\uad6d\uad70 \uc810\ub839\uc73c\ub85c \uc778\ud55c \uc720\ucd9c, \ud55c \uad6d\uac00\uac00 \uc5ec\ub7ec \ub098\ub77c\ub85c \ubd84\ub9ac\ub418\uba74\uc11c \uc18c\uc720\uad8c\uc774 \uc774\uc804\ub41c \uacbd\uc6b0 \ub4f1\uc774\ub2e4. \ub258\ub978\ubca0\ub974\ud06c \uc804\ubc94\uc7ac\ud310\uc18c \ud5cc\uc7a5 \uaddc\uc815 \uc81c6\uc870 c\ud638\ub294 \u201c\uad70\uc0ac\uc801 \ud544\uc694\uc5d0 \uc758\ud558\uc5ec \uc815\ub2f9\ud654\ub418\uc9c0 \uc544\ub2c8\ud558\ub294 \uacf5\uc720 \ub610\ub294 \uc0ac\uc720\uc7ac\uc0b0\uc758 \uc57d\ud0c8\ud589\uc704\ub294 \uba85\ubc31\ud55c \uc804\uc7c1\ubc94\uc8c4\ud589\uc704\u201d\ub77c\uace0 \uaddc\uc815\ud558\uc600\uace0, \uc774 \uaddc\uc815\uc744 \uadfc\uac70\ub85c \uc81c2\ucc28 \uc138\uacc4 \ub300\uc804 \uc911\uc5d0 \ud788\ud2c0\ub7ec\uc758 \uba85\ub839\uc73c\ub85c \ubb38\ud654\uc7ac\ub97c \uc57d\ud0c8\ud558\uc600\ub358 \ud53c\uace0\uc778 4\uc778\uc5d0\uac8c \uc720\uc8c4 \ud310\uacb0\uc744 \ub0b4\ub838\ub2e4. \u2018\ubb38\ud654\uc7ac\uc758 \ubd88\ubc95 \ubc18\ucd9c\uc785 \ubc0f \uc18c\uc720\uad8c \uc591\ub3c4 \uae08\uc9c0\uc640 \uc608\ubc29 \uc218\ub2e8\uc5d0 \uad00\ud55c \ud611\uc57d\u2019 \uc81c11\uc870\uc5d0 \ub530\ub974\uba74 \u2018\uc678\uad6d \uad70\ub300\uc5d0 \uc758\ud55c \uc77c\uad6d\uc758 \uc810\ub839\uc73c\ub85c\ubd80\ud130 \uc9c1\uc811\uc801\uc73c\ub85c \ub610\ub294 \uac04\uc811\uc801\uc73c\ub85c \ubc1c\uc0dd\ud558\ub294 \uac15\uc81c\uc801\uc778 \ubb38\ud654\uc7ac\uc758 \ubc18\ucd9c\uacfc \uc18c\uc720\uad8c\uc758 \uc591\ub3c4\ub294 \ubd88\ubc95\uc73c\ub85c \uac04\uc8fc\ub41c\ub2e4.\u2019\ub77c\uace0 \uaddc\uc815\ud558\uace0 \uc788\ub2e4."} {"question": "\uc815\ubd80\uac00 \uae30\ub3c5\uad50 \ubcf5\uc74c\uc758 \ubcf4\uae09\uc744 \uc5b5\uc81c\ud558\uae30 \uc704\ud574 \ub2e4\uc2dc \uc870\uc9c1\ud55c \uac83\uc740?", "paragraph": "TSPM\uc740 \ubb38\ud654\ud601\uba85\uc774 \uc2dc\uc791\ub418\ub358 \ud574\uc778 1966\ub144\uc5d0 \ud574\uccb4\ub418\uc5c8\ub2e4. TSPM\uc774 \uc7ac\uc870\uc9c1\ub418\ub358 \ud574\uc778 1979\ub144 \ucd08\uc5d0\ub294 \uadf8 \uc601\ud5a5\ub825\uc774 \ubca0\uc774\uc9d5\uc774\ub098 \uc0c1\ud558\uc774 \ub4f1\uc758 \ub300\ub3c4\uc2dc \uc9c0\uc5ed\uc73c\ub85c \uc81c\ud55c\ub418\uc5b4 \uc788\uc5c8\ub358 \ubc18\uba74\uc5d0 \uae30\ub3c5\uad50 \uc2e0\uc559\uc740 \ud5c8\ub09c, \uc800\uc7a5, \ud478\uc820, \uc548\ud6c4\uc774\uc640 \uac19\uc740 \uc9c0\uc5ed\uc5d0\uc11c \uae09\uc18d\ud788 \ud655\uc0b0\ub418\uc5c8\ub2e4. \uc911\uad6d \ub0b4\uc758 \ub9ce\uc740 \uadf8\ub9ac\uc2a4\ub3c4\uc778\ub4e4\uc774 TSPM\uc744 \uc758\uc2ec\ud588\ub294\ub370, \uadf8 \uc774\uc720\ub294 TSPM\uc774 \ud615\uc131\ub420 \ucd08\uae30\uc778 1950\ub144\ub300\uc5d0 \ud604\ub300\uc8fc\uc758 \uc2e0\ud559\uc744 \uac15\uc870\ud588\uc744 \ubfd0 \uc544\ub2c8\ub77c \uc911\uad6d \uc815\ubd80\uac00 \ub9ce\uc740 \uc131\uacbd \uad50\uc0ac\ub4e4\uc744 \uc5b5\uc555\ud558\uace0 \ubc15\ud574\ud560 \ub54c \uadf8\ub4e4\uc774 \ud611\ub825\ud588\uae30 \ub54c\ubb38\uc774\ub2e4. \uc774\ub7ec\ud55c \uadf8\ub9ac\uc2a4\ub3c4\uc778\ub4e4\uc740 1979\ub144\uc5d0 TSPM\uc774 \uc7ac\uc124\ub9bd\ub41c \uac83\uc740 \uc815\ubd80\uac00 \uc790\uc2e0\ub4e4\uc5d0 \ub300\ud55c \ud1b5\uc81c\ub97c \uac15\ud654\ud558\uace0 \uae30\ub3c5\uad50 \ubcf5\uc74c\uc774 \ud655\uc0b0\ub418\ub294 \uac83\uc744 \ub2a6\ucd94\ub824\ub294 \uc2dc\ub3c4\ub85c \ubcf4\uc558\ub2e4.", "answer": "TSPM", "sentence": "TSPM \uc740 \ubb38\ud654\ud601\uba85\uc774 \uc2dc\uc791\ub418\ub358 \ud574\uc778 1966\ub144\uc5d0 \ud574\uccb4\ub418\uc5c8\ub2e4.", "paragraph_sentence": " TSPM \uc740 \ubb38\ud654\ud601\uba85\uc774 \uc2dc\uc791\ub418\ub358 \ud574\uc778 1966\ub144\uc5d0 \ud574\uccb4\ub418\uc5c8\ub2e4. TSPM\uc774 \uc7ac\uc870\uc9c1\ub418\ub358 \ud574\uc778 1979\ub144 \ucd08\uc5d0\ub294 \uadf8 \uc601\ud5a5\ub825\uc774 \ubca0\uc774\uc9d5\uc774\ub098 \uc0c1\ud558\uc774 \ub4f1\uc758 \ub300\ub3c4\uc2dc \uc9c0\uc5ed\uc73c\ub85c \uc81c\ud55c\ub418\uc5b4 \uc788\uc5c8\ub358 \ubc18\uba74\uc5d0 \uae30\ub3c5\uad50 \uc2e0\uc559\uc740 \ud5c8\ub09c, \uc800\uc7a5, \ud478\uc820, \uc548\ud6c4\uc774\uc640 \uac19\uc740 \uc9c0\uc5ed\uc5d0\uc11c \uae09\uc18d\ud788 \ud655\uc0b0\ub418\uc5c8\ub2e4. \uc911\uad6d \ub0b4\uc758 \ub9ce\uc740 \uadf8\ub9ac\uc2a4\ub3c4\uc778\ub4e4\uc774 TSPM\uc744 \uc758\uc2ec\ud588\ub294\ub370, \uadf8 \uc774\uc720\ub294 TSPM\uc774 \ud615\uc131\ub420 \ucd08\uae30\uc778 1950\ub144\ub300\uc5d0 \ud604\ub300\uc8fc\uc758 \uc2e0\ud559\uc744 \uac15\uc870\ud588\uc744 \ubfd0 \uc544\ub2c8\ub77c \uc911\uad6d \uc815\ubd80\uac00 \ub9ce\uc740 \uc131\uacbd \uad50\uc0ac\ub4e4\uc744 \uc5b5\uc555\ud558\uace0 \ubc15\ud574\ud560 \ub54c \uadf8\ub4e4\uc774 \ud611\ub825\ud588\uae30 \ub54c\ubb38\uc774\ub2e4. \uc774\ub7ec\ud55c \uadf8\ub9ac\uc2a4\ub3c4\uc778\ub4e4\uc740 1979\ub144\uc5d0 TSPM\uc774 \uc7ac\uc124\ub9bd\ub41c \uac83\uc740 \uc815\ubd80\uac00 \uc790\uc2e0\ub4e4\uc5d0 \ub300\ud55c \ud1b5\uc81c\ub97c \uac15\ud654\ud558\uace0 \uae30\ub3c5\uad50 \ubcf5\uc74c\uc774 \ud655\uc0b0\ub418\ub294 \uac83\uc744 \ub2a6\ucd94\ub824\ub294 \uc2dc\ub3c4\ub85c \ubcf4\uc558\ub2e4.", "paragraph_answer": " TSPM \uc740 \ubb38\ud654\ud601\uba85\uc774 \uc2dc\uc791\ub418\ub358 \ud574\uc778 1966\ub144\uc5d0 \ud574\uccb4\ub418\uc5c8\ub2e4. TSPM\uc774 \uc7ac\uc870\uc9c1\ub418\ub358 \ud574\uc778 1979\ub144 \ucd08\uc5d0\ub294 \uadf8 \uc601\ud5a5\ub825\uc774 \ubca0\uc774\uc9d5\uc774\ub098 \uc0c1\ud558\uc774 \ub4f1\uc758 \ub300\ub3c4\uc2dc \uc9c0\uc5ed\uc73c\ub85c \uc81c\ud55c\ub418\uc5b4 \uc788\uc5c8\ub358 \ubc18\uba74\uc5d0 \uae30\ub3c5\uad50 \uc2e0\uc559\uc740 \ud5c8\ub09c, \uc800\uc7a5, \ud478\uc820, \uc548\ud6c4\uc774\uc640 \uac19\uc740 \uc9c0\uc5ed\uc5d0\uc11c \uae09\uc18d\ud788 \ud655\uc0b0\ub418\uc5c8\ub2e4. \uc911\uad6d \ub0b4\uc758 \ub9ce\uc740 \uadf8\ub9ac\uc2a4\ub3c4\uc778\ub4e4\uc774 TSPM\uc744 \uc758\uc2ec\ud588\ub294\ub370, \uadf8 \uc774\uc720\ub294 TSPM\uc774 \ud615\uc131\ub420 \ucd08\uae30\uc778 1950\ub144\ub300\uc5d0 \ud604\ub300\uc8fc\uc758 \uc2e0\ud559\uc744 \uac15\uc870\ud588\uc744 \ubfd0 \uc544\ub2c8\ub77c \uc911\uad6d \uc815\ubd80\uac00 \ub9ce\uc740 \uc131\uacbd \uad50\uc0ac\ub4e4\uc744 \uc5b5\uc555\ud558\uace0 \ubc15\ud574\ud560 \ub54c \uadf8\ub4e4\uc774 \ud611\ub825\ud588\uae30 \ub54c\ubb38\uc774\ub2e4. \uc774\ub7ec\ud55c \uadf8\ub9ac\uc2a4\ub3c4\uc778\ub4e4\uc740 1979\ub144\uc5d0 TSPM\uc774 \uc7ac\uc124\ub9bd\ub41c \uac83\uc740 \uc815\ubd80\uac00 \uc790\uc2e0\ub4e4\uc5d0 \ub300\ud55c \ud1b5\uc81c\ub97c \uac15\ud654\ud558\uace0 \uae30\ub3c5\uad50 \ubcf5\uc74c\uc774 \ud655\uc0b0\ub418\ub294 \uac83\uc744 \ub2a6\ucd94\ub824\ub294 \uc2dc\ub3c4\ub85c \ubcf4\uc558\ub2e4.", "sentence_answer": " TSPM \uc740 \ubb38\ud654\ud601\uba85\uc774 \uc2dc\uc791\ub418\ub358 \ud574\uc778 1966\ub144\uc5d0 \ud574\uccb4\ub418\uc5c8\ub2e4.", "paragraph_id": "6460508-4-1", "paragraph_question": "question: \uc815\ubd80\uac00 \uae30\ub3c5\uad50 \ubcf5\uc74c\uc758 \ubcf4\uae09\uc744 \uc5b5\uc81c\ud558\uae30 \uc704\ud574 \ub2e4\uc2dc \uc870\uc9c1\ud55c \uac83\uc740?, context: TSPM\uc740 \ubb38\ud654\ud601\uba85\uc774 \uc2dc\uc791\ub418\ub358 \ud574\uc778 1966\ub144\uc5d0 \ud574\uccb4\ub418\uc5c8\ub2e4. TSPM\uc774 \uc7ac\uc870\uc9c1\ub418\ub358 \ud574\uc778 1979\ub144 \ucd08\uc5d0\ub294 \uadf8 \uc601\ud5a5\ub825\uc774 \ubca0\uc774\uc9d5\uc774\ub098 \uc0c1\ud558\uc774 \ub4f1\uc758 \ub300\ub3c4\uc2dc \uc9c0\uc5ed\uc73c\ub85c \uc81c\ud55c\ub418\uc5b4 \uc788\uc5c8\ub358 \ubc18\uba74\uc5d0 \uae30\ub3c5\uad50 \uc2e0\uc559\uc740 \ud5c8\ub09c, \uc800\uc7a5, \ud478\uc820, \uc548\ud6c4\uc774\uc640 \uac19\uc740 \uc9c0\uc5ed\uc5d0\uc11c \uae09\uc18d\ud788 \ud655\uc0b0\ub418\uc5c8\ub2e4. \uc911\uad6d \ub0b4\uc758 \ub9ce\uc740 \uadf8\ub9ac\uc2a4\ub3c4\uc778\ub4e4\uc774 TSPM\uc744 \uc758\uc2ec\ud588\ub294\ub370, \uadf8 \uc774\uc720\ub294 TSPM\uc774 \ud615\uc131\ub420 \ucd08\uae30\uc778 1950\ub144\ub300\uc5d0 \ud604\ub300\uc8fc\uc758 \uc2e0\ud559\uc744 \uac15\uc870\ud588\uc744 \ubfd0 \uc544\ub2c8\ub77c \uc911\uad6d \uc815\ubd80\uac00 \ub9ce\uc740 \uc131\uacbd \uad50\uc0ac\ub4e4\uc744 \uc5b5\uc555\ud558\uace0 \ubc15\ud574\ud560 \ub54c \uadf8\ub4e4\uc774 \ud611\ub825\ud588\uae30 \ub54c\ubb38\uc774\ub2e4. \uc774\ub7ec\ud55c \uadf8\ub9ac\uc2a4\ub3c4\uc778\ub4e4\uc740 1979\ub144\uc5d0 TSPM\uc774 \uc7ac\uc124\ub9bd\ub41c \uac83\uc740 \uc815\ubd80\uac00 \uc790\uc2e0\ub4e4\uc5d0 \ub300\ud55c \ud1b5\uc81c\ub97c \uac15\ud654\ud558\uace0 \uae30\ub3c5\uad50 \ubcf5\uc74c\uc774 \ud655\uc0b0\ub418\ub294 \uac83\uc744 \ub2a6\ucd94\ub824\ub294 \uc2dc\ub3c4\ub85c \ubcf4\uc558\ub2e4."} {"question": "\ubd88\ub7c9\ud55c \uc0ac\ub78c\ub4e4\uc774 \ubd88\ub7c9\ub2e8\uccb4\uc5d0 \ucc98\uc74c \uc8fc\ub85c \ub4e4\uc5b4\uac00\uac8c \ub418\ub294 \uc2dc\uae30\ub294?", "paragraph": "\ubd88\ub7c9\ud589\uc704\uc18c\ub144\uc758 \ub300\ud45c \uaca9\uc774\ub77c\uace0 \ubcf4\uc5ec\uc9c0\ub294 \uac83\uc774 \ub9ce\uc544, 1970 ~ 1980\ub144\ub300\uc5d0\ub294 \ubb54\uac00 \ub3c5\ud2b9\ud55c \ubcf5\uc7a5\uc774\ub098 \uba38\ub9ac \ubaa8\uc591 \ub4f1\uc774 \ud3ed\uc8fc\uc871\uc5d0\uc758 \uc18c\uc18d \uc720\ubb34\uc5d0 \uad00\ub828\ub418\uc9c0 \uc54a\uace0 \ubd88\ub7c9\uc18c\ub144 \uc804\ubc18\uc5d0 \ub110\ub9ac \ud37c\uc838 \uc788\uc5c8\ub2e4. \ubd90\uc774 \uc885\uc5b8 \ubc11\ud6c4\ub294 \ud1b5\uc0c1\uc758 \ubab8\ucc28\ub9bc\uc778\ub3c4 \uc801\uc9c0 \uc54a\ub2e4. \uadf8\ub8f9\ub9c8\ub2e4\uc9c0\uc5ed\uc131\uc774 \uac15\ud558\uac8c, \uc8fc\ub85c \uc911\ud559\uad50 \uc2dc\ub300\uc5d0 \uc0ac\uc774\uac00 \uc88b\uc558\ub358 \ubd88\ub7c9 \uadf8\ub8f9\uc744 \ubc14\ud0d5\uc73c\ub85c \ud574\uc11c \uacb0\uc131\ub418\uc5b4, \ucd9c\uc2e0\uc911\ud559\uad50\ub098 \ud604\uc9c0 \uadfc\uad8c\uc758 \uce5c\uad6c\u00b7\ud6c4\ubc30\uc758 \ubd88\ub7c9 \uadf8\ub8f9\uc744 \uad8c\uc720, \ubcf5\uc218\uc758 \ud3ed\uc8fc\uc871 \uadf8\ub8f9\uc758 \uc5f0\ud569\ud654 \ub4f1\uc744 \ub418\ud480\uc774\ud558\uace0, \uc870\uc9c1\uc758 \uc720\uc9c0\uc640 \uc138\ub825\ud655\ub300\ub97c \ub3c4\ubaa8\ud574 \uac04\ub2e4. \uadf8 \ub54c\ubb38\uc5d0, \uadf8\uac83\ub4e4 \uc911\ud559\uad50\u00b7\uace0\uad50\uc758 \ubd88\ub7c9 \uadf8\ub8f9\uc774 \uacb0\uc18d\ud55c \uc0ac\uad50\ub2e8\uccb4\uc640 \uac19\uc740 \uc874\uc7ac\uac00 \ub418\uace0, \uac00\uc785 \uc608\ube44\uad70\uc774 \ub418\ub294 \uacbd\ub3c4\ud55c \ubd88\ub7c9\uc18c\ub144\uc5d0 \ub300\ud558\uc5ec\ub3c4 \ubc30\ud6c4\uc5d0\uc11c \uc77c\uc815\ud55c \uc601\ud5a5\uc744 \ub07c\uce58\uace0 \uc788\ub2e4. \uadf8\uc704\uc5d0 \ud3ed\ub825\ub2e8\uc758 \ud558\ubd80\uc870\uc9c1\uc73c\ub85c\uc11c \uae30\ub2a5, \ub610\ub294 \ub3d9 \ub2e8\uccb4\uc758 \uac00\uc785 \uc54c\uc120\uc758 \uc7a5\uc18c\uac00 \ub418\ub294 \ucf00\uc774\uc2a4\ub3c4 \ub9ce\ub2e4.", "answer": "\uc911\ud559\uad50", "sentence": "\uadf8\ub8f9\ub9c8\ub2e4\uc9c0\uc5ed\uc131\uc774 \uac15\ud558\uac8c, \uc8fc\ub85c \uc911\ud559\uad50 \uc2dc\ub300\uc5d0 \uc0ac\uc774\uac00 \uc88b\uc558\ub358 \ubd88\ub7c9 \uadf8\ub8f9\uc744 \ubc14\ud0d5\uc73c\ub85c \ud574\uc11c \uacb0\uc131\ub418\uc5b4, \ucd9c\uc2e0\uc911\ud559\uad50\ub098 \ud604\uc9c0 \uadfc\uad8c\uc758 \uce5c\uad6c\u00b7\ud6c4\ubc30\uc758 \ubd88\ub7c9 \uadf8\ub8f9\uc744 \uad8c\uc720, \ubcf5\uc218\uc758 \ud3ed\uc8fc\uc871 \uadf8\ub8f9\uc758 \uc5f0\ud569\ud654 \ub4f1\uc744 \ub418\ud480\uc774\ud558\uace0, \uc870\uc9c1\uc758 \uc720\uc9c0\uc640 \uc138\ub825\ud655\ub300\ub97c \ub3c4\ubaa8\ud574 \uac04\ub2e4.", "paragraph_sentence": "\ubd88\ub7c9\ud589\uc704\uc18c\ub144\uc758 \ub300\ud45c \uaca9\uc774\ub77c\uace0 \ubcf4\uc5ec\uc9c0\ub294 \uac83\uc774 \ub9ce\uc544, 1970 ~ 1980\ub144\ub300\uc5d0\ub294 \ubb54\uac00 \ub3c5\ud2b9\ud55c \ubcf5\uc7a5\uc774\ub098 \uba38\ub9ac \ubaa8\uc591 \ub4f1\uc774 \ud3ed\uc8fc\uc871\uc5d0\uc758 \uc18c\uc18d \uc720\ubb34\uc5d0 \uad00\ub828\ub418\uc9c0 \uc54a\uace0 \ubd88\ub7c9\uc18c\ub144 \uc804\ubc18\uc5d0 \ub110\ub9ac \ud37c\uc838 \uc788\uc5c8\ub2e4. \ubd90\uc774 \uc885\uc5b8 \ubc11\ud6c4\ub294 \ud1b5\uc0c1\uc758 \ubab8\ucc28\ub9bc\uc778\ub3c4 \uc801\uc9c0 \uc54a\ub2e4. \uadf8\ub8f9\ub9c8\ub2e4\uc9c0\uc5ed\uc131\uc774 \uac15\ud558\uac8c, \uc8fc\ub85c \uc911\ud559\uad50 \uc2dc\ub300\uc5d0 \uc0ac\uc774\uac00 \uc88b\uc558\ub358 \ubd88\ub7c9 \uadf8\ub8f9\uc744 \ubc14\ud0d5\uc73c\ub85c \ud574\uc11c \uacb0\uc131\ub418\uc5b4, \ucd9c\uc2e0\uc911\ud559\uad50\ub098 \ud604\uc9c0 \uadfc\uad8c\uc758 \uce5c\uad6c\u00b7\ud6c4\ubc30\uc758 \ubd88\ub7c9 \uadf8\ub8f9\uc744 \uad8c\uc720, \ubcf5\uc218\uc758 \ud3ed\uc8fc\uc871 \uadf8\ub8f9\uc758 \uc5f0\ud569\ud654 \ub4f1\uc744 \ub418\ud480\uc774\ud558\uace0, \uc870\uc9c1\uc758 \uc720\uc9c0\uc640 \uc138\ub825\ud655\ub300\ub97c \ub3c4\ubaa8\ud574 \uac04\ub2e4. \uadf8 \ub54c\ubb38\uc5d0, \uadf8\uac83\ub4e4 \uc911\ud559\uad50\u00b7\uace0\uad50\uc758 \ubd88\ub7c9 \uadf8\ub8f9\uc774 \uacb0\uc18d\ud55c \uc0ac\uad50\ub2e8\uccb4\uc640 \uac19\uc740 \uc874\uc7ac\uac00 \ub418\uace0, \uac00\uc785 \uc608\ube44\uad70\uc774 \ub418\ub294 \uacbd\ub3c4\ud55c \ubd88\ub7c9\uc18c\ub144\uc5d0 \ub300\ud558\uc5ec\ub3c4 \ubc30\ud6c4\uc5d0\uc11c \uc77c\uc815\ud55c \uc601\ud5a5\uc744 \ub07c\uce58\uace0 \uc788\ub2e4. \uadf8\uc704\uc5d0 \ud3ed\ub825\ub2e8\uc758 \ud558\ubd80\uc870\uc9c1\uc73c\ub85c\uc11c \uae30\ub2a5, \ub610\ub294 \ub3d9 \ub2e8\uccb4\uc758 \uac00\uc785 \uc54c\uc120\uc758 \uc7a5\uc18c\uac00 \ub418\ub294 \ucf00\uc774\uc2a4\ub3c4 \ub9ce\ub2e4.", "paragraph_answer": "\ubd88\ub7c9\ud589\uc704\uc18c\ub144\uc758 \ub300\ud45c \uaca9\uc774\ub77c\uace0 \ubcf4\uc5ec\uc9c0\ub294 \uac83\uc774 \ub9ce\uc544, 1970 ~ 1980\ub144\ub300\uc5d0\ub294 \ubb54\uac00 \ub3c5\ud2b9\ud55c \ubcf5\uc7a5\uc774\ub098 \uba38\ub9ac \ubaa8\uc591 \ub4f1\uc774 \ud3ed\uc8fc\uc871\uc5d0\uc758 \uc18c\uc18d \uc720\ubb34\uc5d0 \uad00\ub828\ub418\uc9c0 \uc54a\uace0 \ubd88\ub7c9\uc18c\ub144 \uc804\ubc18\uc5d0 \ub110\ub9ac \ud37c\uc838 \uc788\uc5c8\ub2e4. \ubd90\uc774 \uc885\uc5b8 \ubc11\ud6c4\ub294 \ud1b5\uc0c1\uc758 \ubab8\ucc28\ub9bc\uc778\ub3c4 \uc801\uc9c0 \uc54a\ub2e4. \uadf8\ub8f9\ub9c8\ub2e4\uc9c0\uc5ed\uc131\uc774 \uac15\ud558\uac8c, \uc8fc\ub85c \uc911\ud559\uad50 \uc2dc\ub300\uc5d0 \uc0ac\uc774\uac00 \uc88b\uc558\ub358 \ubd88\ub7c9 \uadf8\ub8f9\uc744 \ubc14\ud0d5\uc73c\ub85c \ud574\uc11c \uacb0\uc131\ub418\uc5b4, \ucd9c\uc2e0\uc911\ud559\uad50\ub098 \ud604\uc9c0 \uadfc\uad8c\uc758 \uce5c\uad6c\u00b7\ud6c4\ubc30\uc758 \ubd88\ub7c9 \uadf8\ub8f9\uc744 \uad8c\uc720, \ubcf5\uc218\uc758 \ud3ed\uc8fc\uc871 \uadf8\ub8f9\uc758 \uc5f0\ud569\ud654 \ub4f1\uc744 \ub418\ud480\uc774\ud558\uace0, \uc870\uc9c1\uc758 \uc720\uc9c0\uc640 \uc138\ub825\ud655\ub300\ub97c \ub3c4\ubaa8\ud574 \uac04\ub2e4. \uadf8 \ub54c\ubb38\uc5d0, \uadf8\uac83\ub4e4 \uc911\ud559\uad50\u00b7\uace0\uad50\uc758 \ubd88\ub7c9 \uadf8\ub8f9\uc774 \uacb0\uc18d\ud55c \uc0ac\uad50\ub2e8\uccb4\uc640 \uac19\uc740 \uc874\uc7ac\uac00 \ub418\uace0, \uac00\uc785 \uc608\ube44\uad70\uc774 \ub418\ub294 \uacbd\ub3c4\ud55c \ubd88\ub7c9\uc18c\ub144\uc5d0 \ub300\ud558\uc5ec\ub3c4 \ubc30\ud6c4\uc5d0\uc11c \uc77c\uc815\ud55c \uc601\ud5a5\uc744 \ub07c\uce58\uace0 \uc788\ub2e4. \uadf8\uc704\uc5d0 \ud3ed\ub825\ub2e8\uc758 \ud558\ubd80\uc870\uc9c1\uc73c\ub85c\uc11c \uae30\ub2a5, \ub610\ub294 \ub3d9 \ub2e8\uccb4\uc758 \uac00\uc785 \uc54c\uc120\uc758 \uc7a5\uc18c\uac00 \ub418\ub294 \ucf00\uc774\uc2a4\ub3c4 \ub9ce\ub2e4.", "sentence_answer": "\uadf8\ub8f9\ub9c8\ub2e4\uc9c0\uc5ed\uc131\uc774 \uac15\ud558\uac8c, \uc8fc\ub85c \uc911\ud559\uad50 \uc2dc\ub300\uc5d0 \uc0ac\uc774\uac00 \uc88b\uc558\ub358 \ubd88\ub7c9 \uadf8\ub8f9\uc744 \ubc14\ud0d5\uc73c\ub85c \ud574\uc11c \uacb0\uc131\ub418\uc5b4, \ucd9c\uc2e0\uc911\ud559\uad50\ub098 \ud604\uc9c0 \uadfc\uad8c\uc758 \uce5c\uad6c\u00b7\ud6c4\ubc30\uc758 \ubd88\ub7c9 \uadf8\ub8f9\uc744 \uad8c\uc720, \ubcf5\uc218\uc758 \ud3ed\uc8fc\uc871 \uadf8\ub8f9\uc758 \uc5f0\ud569\ud654 \ub4f1\uc744 \ub418\ud480\uc774\ud558\uace0, \uc870\uc9c1\uc758 \uc720\uc9c0\uc640 \uc138\ub825\ud655\ub300\ub97c \ub3c4\ubaa8\ud574 \uac04\ub2e4.", "paragraph_id": "5922316-5-0", "paragraph_question": "question: \ubd88\ub7c9\ud55c \uc0ac\ub78c\ub4e4\uc774 \ubd88\ub7c9\ub2e8\uccb4\uc5d0 \ucc98\uc74c \uc8fc\ub85c \ub4e4\uc5b4\uac00\uac8c \ub418\ub294 \uc2dc\uae30\ub294?, context: \ubd88\ub7c9\ud589\uc704\uc18c\ub144\uc758 \ub300\ud45c \uaca9\uc774\ub77c\uace0 \ubcf4\uc5ec\uc9c0\ub294 \uac83\uc774 \ub9ce\uc544, 1970 ~ 1980\ub144\ub300\uc5d0\ub294 \ubb54\uac00 \ub3c5\ud2b9\ud55c \ubcf5\uc7a5\uc774\ub098 \uba38\ub9ac \ubaa8\uc591 \ub4f1\uc774 \ud3ed\uc8fc\uc871\uc5d0\uc758 \uc18c\uc18d \uc720\ubb34\uc5d0 \uad00\ub828\ub418\uc9c0 \uc54a\uace0 \ubd88\ub7c9\uc18c\ub144 \uc804\ubc18\uc5d0 \ub110\ub9ac \ud37c\uc838 \uc788\uc5c8\ub2e4. \ubd90\uc774 \uc885\uc5b8 \ubc11\ud6c4\ub294 \ud1b5\uc0c1\uc758 \ubab8\ucc28\ub9bc\uc778\ub3c4 \uc801\uc9c0 \uc54a\ub2e4. \uadf8\ub8f9\ub9c8\ub2e4\uc9c0\uc5ed\uc131\uc774 \uac15\ud558\uac8c, \uc8fc\ub85c \uc911\ud559\uad50 \uc2dc\ub300\uc5d0 \uc0ac\uc774\uac00 \uc88b\uc558\ub358 \ubd88\ub7c9 \uadf8\ub8f9\uc744 \ubc14\ud0d5\uc73c\ub85c \ud574\uc11c \uacb0\uc131\ub418\uc5b4, \ucd9c\uc2e0\uc911\ud559\uad50\ub098 \ud604\uc9c0 \uadfc\uad8c\uc758 \uce5c\uad6c\u00b7\ud6c4\ubc30\uc758 \ubd88\ub7c9 \uadf8\ub8f9\uc744 \uad8c\uc720, \ubcf5\uc218\uc758 \ud3ed\uc8fc\uc871 \uadf8\ub8f9\uc758 \uc5f0\ud569\ud654 \ub4f1\uc744 \ub418\ud480\uc774\ud558\uace0, \uc870\uc9c1\uc758 \uc720\uc9c0\uc640 \uc138\ub825\ud655\ub300\ub97c \ub3c4\ubaa8\ud574 \uac04\ub2e4. \uadf8 \ub54c\ubb38\uc5d0, \uadf8\uac83\ub4e4 \uc911\ud559\uad50\u00b7\uace0\uad50\uc758 \ubd88\ub7c9 \uadf8\ub8f9\uc774 \uacb0\uc18d\ud55c \uc0ac\uad50\ub2e8\uccb4\uc640 \uac19\uc740 \uc874\uc7ac\uac00 \ub418\uace0, \uac00\uc785 \uc608\ube44\uad70\uc774 \ub418\ub294 \uacbd\ub3c4\ud55c \ubd88\ub7c9\uc18c\ub144\uc5d0 \ub300\ud558\uc5ec\ub3c4 \ubc30\ud6c4\uc5d0\uc11c \uc77c\uc815\ud55c \uc601\ud5a5\uc744 \ub07c\uce58\uace0 \uc788\ub2e4. \uadf8\uc704\uc5d0 \ud3ed\ub825\ub2e8\uc758 \ud558\ubd80\uc870\uc9c1\uc73c\ub85c\uc11c \uae30\ub2a5, \ub610\ub294 \ub3d9 \ub2e8\uccb4\uc758 \uac00\uc785 \uc54c\uc120\uc758 \uc7a5\uc18c\uac00 \ub418\ub294 \ucf00\uc774\uc2a4\ub3c4 \ub9ce\ub2e4."} {"question": "\uc791\uc13c\ud558\uc6b0\uc820 \uc218\uc6a9\uc18c\uc5d0\uc11c \uc5d4\uc9c4\uc758 \ubc30\uae30\uac00\uc2a4\ub97c \uc774\uc6a9\ud55c \uac00\uc2a4\ucc28\ub97c \uc0ac\uc6a9\ud55c \uac83\uc740 \uc5b8\uc81c\ubd80\ud130\uc778\uac00?", "paragraph": "1939\ub144 12\uc6d4\uc758 \uc2dc\uc791\uacfc \ud568\uaed8 \ub098\uce58\ub294 \uac00\uc2a4\ub97c \uc774\uc6a9\ud55c \ub2e4\uc911 \uc0b4\uc778\uc758 \uc0c8\ub85c\uc6b4 \ubc29\ubc95\uc744 \uc18c\uac1c\ud558\uc600\ub2e4. \uccab\uc9f8\ub85c, \uac00\uc2a4 \uc2e4\ub9b0\ub354\ub97c \uc7a5\ucc29\ud55c \uc2e4\ud5d8\uc6a9 \uac00\uc2a4\ucc28(gas van)\uc640 \ubc00\ud3d0\ub41c \ud2b8\ub801\ud06c \uce78\ub9c9\uc774\ub294 T-4 \uc791\uc804(\ub098\uce58\uc758 \uc778\uc885\uc815\ucc45 \uc77c\ud658\uc73c\ub85c \uc548\ub77d\uc0ac\ub97c \uc774\uc6a9\ud55c \uc7a5\uc560\uc778 \ud559\uc0b4 \uacc4\ud68d)\uc758 \uc77c\ud658\uc73c\ub85c\uc368 \ud3ec\uba54\ub77c\ub2c8\uc544, \ub3d9\ud504\ub85c\uc774\uc13c \uadf8\ub9ac\uace0 \uc810\ub839\ub41c \ud3f4\ub780\ub4dc\uc758 \uc694\uc591\uc6d0\uc5d0 \uc788\ub294 \uc815\uc2e0 \uc9c8\ud658\uc790\ub4e4\uc744 \uc8fd\uc774\ub294\ub370 \uc0ac\uc6a9\ub418\uc5c8\ub2e4. \uc791\uc13c\ud558\uc6b0\uc820 \uc218\uc6a9\uc18c\uc5d0\uc120 100\uba85\uae4c\uc9c0 \uc218\uc6a9\ud560 \uc218 \uc788\ub294 \ub354 \ud070 \uac00\uc2a4\ucc28\uac00 \uc2e4\ub9b0\ub354 \ub300\uc2e0 \uc5d4\uc9c4\uc758 \ubc30\uae30\uac00\uc2a4\ub97c \uc774\uc6a9\ud558\uba74\uc11c 1941\ub144 11\uc6d4\ubd80\ud130 \uc0ac\uc6a9\ub418\uc5c8\ub2e4. \uc774 \uac00\uc2a4\ucc28\ub4e4\uc740 1941\ub144 12\uc6d4 \ud5e4\uc6c0\ub178 \uc9d1\ub2e8 \uc218\uc6a9\uc18c(Che\u0142mno extermination camp)\uc5d0 \uc18c\uac1c\ub418\uc5c8\uace0 \uadf8\uac83 \uc911 15\ub300\ub294 \uc810\ub839\ub41c \uc18c\ub828\uc5d0\uc11c \ub300\ub7c9\uc0b4\uc0c1\uc870\uc9c1\uc778 \uc544\uc778\uc790\uce20\uadf8\ub8e8\ud39c\uc5d0 \uc758\ud574 \uc0ac\uc6a9\ub418\uc5c8\ub2e4. \uc774 \uac00\uc2a4\ucc28\ub294 \uad6d\uac00\ubcf4\uc548\ubcf8\ubd80(SS-Reichssicherheitshauptamt)\uc758 \uac10\ub3c5 \uc544\ub798\uc5d0\uc11c \uac1c\ubc1c\ub418\uc5c8\uace0 \uc6b4\uc601\ub418\uc5c8\ub2e4. \ub610\ud55c \ub300\ub7b5 50\ub9cc\uba85\uc758 \uc0ac\ub78c\ub4e4\uc744 \uc8fd\uc774\ub294\ub370 \uc0ac\uc6a9\ub418\uc5c8\ub294\ub370 \uadf8 \uc0ac\ub78c\ub4e4\uc740 \uc8fc\ub85c \uc720\ub300\uc778\uc774\uc5c8\uc9c0\ub9cc \uc9d1\uc2dc\uc640 \ub2e4\ub978 \ub098\ub77c \uc0ac\ub78c\ub4e4\ub3c4 \ud3ec\ud568\ub418\uc5c8\ub2e4. \uac00\uc2a4\ucc28\ub294 \uc2e0\uc911\ud558\uac8c \uac10\ub3c5\ubc1b\uc558\uace0 \ud55c\ub2ec \ub3d9\uc548\uc758 \uad00\ucc30 \ud6c4\uc758 \ud55c \uae30\ub85d\uc5d0\uc11c\ub294 \u201c\uae30\uacc4\uc5d0 \uc5b4\ub5a0\ud55c \uacb0\ud568\ub3c4 \ub098\ud0c0\ub098\uc9c0 \uc54a\uace0 \uc138 \ub300\uc758 \uac00\uc2a4\ucc28\ub97c \uc774\uc6a9\ud558\uc5ec 9\ub9cc7\ucc9c\ubc88\uc774 \uc9c4\ud589\ub418\uc5c8\ub2e4\u201d \ub77c\uace0 \uc4f0\uc5ec\uc838 \uc788\ub2e4.", "answer": "1941\ub144 11\uc6d4", "sentence": "\uc791\uc13c\ud558\uc6b0\uc820 \uc218\uc6a9\uc18c\uc5d0\uc120 100\uba85\uae4c\uc9c0 \uc218\uc6a9\ud560 \uc218 \uc788\ub294 \ub354 \ud070 \uac00\uc2a4\ucc28\uac00 \uc2e4\ub9b0\ub354 \ub300\uc2e0 \uc5d4\uc9c4\uc758 \ubc30\uae30\uac00\uc2a4\ub97c \uc774\uc6a9\ud558\uba74\uc11c 1941\ub144 11\uc6d4 \ubd80\ud130 \uc0ac\uc6a9\ub418\uc5c8\ub2e4.", "paragraph_sentence": "1939\ub144 12\uc6d4\uc758 \uc2dc\uc791\uacfc \ud568\uaed8 \ub098\uce58\ub294 \uac00\uc2a4\ub97c \uc774\uc6a9\ud55c \ub2e4\uc911 \uc0b4\uc778\uc758 \uc0c8\ub85c\uc6b4 \ubc29\ubc95\uc744 \uc18c\uac1c\ud558\uc600\ub2e4. \uccab\uc9f8\ub85c, \uac00\uc2a4 \uc2e4\ub9b0\ub354\ub97c \uc7a5\ucc29\ud55c \uc2e4\ud5d8\uc6a9 \uac00\uc2a4\ucc28(gas van)\uc640 \ubc00\ud3d0\ub41c \ud2b8\ub801\ud06c \uce78\ub9c9\uc774\ub294 T-4 \uc791\uc804(\ub098\uce58\uc758 \uc778\uc885\uc815\ucc45 \uc77c\ud658\uc73c\ub85c \uc548\ub77d\uc0ac\ub97c \uc774\uc6a9\ud55c \uc7a5\uc560\uc778 \ud559\uc0b4 \uacc4\ud68d)\uc758 \uc77c\ud658\uc73c\ub85c\uc368 \ud3ec\uba54\ub77c\ub2c8\uc544, \ub3d9\ud504\ub85c\uc774\uc13c \uadf8\ub9ac\uace0 \uc810\ub839\ub41c \ud3f4\ub780\ub4dc\uc758 \uc694\uc591\uc6d0\uc5d0 \uc788\ub294 \uc815\uc2e0 \uc9c8\ud658\uc790\ub4e4\uc744 \uc8fd\uc774\ub294\ub370 \uc0ac\uc6a9\ub418\uc5c8\ub2e4. \uc791\uc13c\ud558\uc6b0\uc820 \uc218\uc6a9\uc18c\uc5d0\uc120 100\uba85\uae4c\uc9c0 \uc218\uc6a9\ud560 \uc218 \uc788\ub294 \ub354 \ud070 \uac00\uc2a4\ucc28\uac00 \uc2e4\ub9b0\ub354 \ub300\uc2e0 \uc5d4\uc9c4\uc758 \ubc30\uae30\uac00\uc2a4\ub97c \uc774\uc6a9\ud558\uba74\uc11c 1941\ub144 11\uc6d4 \ubd80\ud130 \uc0ac\uc6a9\ub418\uc5c8\ub2e4. \uc774 \uac00\uc2a4\ucc28\ub4e4\uc740 1941\ub144 12\uc6d4 \ud5e4\uc6c0\ub178 \uc9d1\ub2e8 \uc218\uc6a9\uc18c(Che\u0142mno extermination camp)\uc5d0 \uc18c\uac1c\ub418\uc5c8\uace0 \uadf8\uac83 \uc911 15\ub300\ub294 \uc810\ub839\ub41c \uc18c\ub828\uc5d0\uc11c \ub300\ub7c9\uc0b4\uc0c1\uc870\uc9c1\uc778 \uc544\uc778\uc790\uce20\uadf8\ub8e8\ud39c\uc5d0 \uc758\ud574 \uc0ac\uc6a9\ub418\uc5c8\ub2e4. \uc774 \uac00\uc2a4\ucc28\ub294 \uad6d\uac00\ubcf4\uc548\ubcf8\ubd80(SS-Reichssicherheitshauptamt)\uc758 \uac10\ub3c5 \uc544\ub798\uc5d0\uc11c \uac1c\ubc1c\ub418\uc5c8\uace0 \uc6b4\uc601\ub418\uc5c8\ub2e4. \ub610\ud55c \ub300\ub7b5 50\ub9cc\uba85\uc758 \uc0ac\ub78c\ub4e4\uc744 \uc8fd\uc774\ub294\ub370 \uc0ac\uc6a9\ub418\uc5c8\ub294\ub370 \uadf8 \uc0ac\ub78c\ub4e4\uc740 \uc8fc\ub85c \uc720\ub300\uc778\uc774\uc5c8\uc9c0\ub9cc \uc9d1\uc2dc\uc640 \ub2e4\ub978 \ub098\ub77c \uc0ac\ub78c\ub4e4\ub3c4 \ud3ec\ud568\ub418\uc5c8\ub2e4. \uac00\uc2a4\ucc28\ub294 \uc2e0\uc911\ud558\uac8c \uac10\ub3c5\ubc1b\uc558\uace0 \ud55c\ub2ec \ub3d9\uc548\uc758 \uad00\ucc30 \ud6c4\uc758 \ud55c \uae30\ub85d\uc5d0\uc11c\ub294 \u201c\uae30\uacc4\uc5d0 \uc5b4\ub5a0\ud55c \uacb0\ud568\ub3c4 \ub098\ud0c0\ub098\uc9c0 \uc54a\uace0 \uc138 \ub300\uc758 \uac00\uc2a4\ucc28\ub97c \uc774\uc6a9\ud558\uc5ec 9\ub9cc7\ucc9c\ubc88\uc774 \uc9c4\ud589\ub418\uc5c8\ub2e4\u201d \ub77c\uace0 \uc4f0\uc5ec\uc838 \uc788\ub2e4.", "paragraph_answer": "1939\ub144 12\uc6d4\uc758 \uc2dc\uc791\uacfc \ud568\uaed8 \ub098\uce58\ub294 \uac00\uc2a4\ub97c \uc774\uc6a9\ud55c \ub2e4\uc911 \uc0b4\uc778\uc758 \uc0c8\ub85c\uc6b4 \ubc29\ubc95\uc744 \uc18c\uac1c\ud558\uc600\ub2e4. \uccab\uc9f8\ub85c, \uac00\uc2a4 \uc2e4\ub9b0\ub354\ub97c \uc7a5\ucc29\ud55c \uc2e4\ud5d8\uc6a9 \uac00\uc2a4\ucc28(gas van)\uc640 \ubc00\ud3d0\ub41c \ud2b8\ub801\ud06c \uce78\ub9c9\uc774\ub294 T-4 \uc791\uc804(\ub098\uce58\uc758 \uc778\uc885\uc815\ucc45 \uc77c\ud658\uc73c\ub85c \uc548\ub77d\uc0ac\ub97c \uc774\uc6a9\ud55c \uc7a5\uc560\uc778 \ud559\uc0b4 \uacc4\ud68d)\uc758 \uc77c\ud658\uc73c\ub85c\uc368 \ud3ec\uba54\ub77c\ub2c8\uc544, \ub3d9\ud504\ub85c\uc774\uc13c \uadf8\ub9ac\uace0 \uc810\ub839\ub41c \ud3f4\ub780\ub4dc\uc758 \uc694\uc591\uc6d0\uc5d0 \uc788\ub294 \uc815\uc2e0 \uc9c8\ud658\uc790\ub4e4\uc744 \uc8fd\uc774\ub294\ub370 \uc0ac\uc6a9\ub418\uc5c8\ub2e4. \uc791\uc13c\ud558\uc6b0\uc820 \uc218\uc6a9\uc18c\uc5d0\uc120 100\uba85\uae4c\uc9c0 \uc218\uc6a9\ud560 \uc218 \uc788\ub294 \ub354 \ud070 \uac00\uc2a4\ucc28\uac00 \uc2e4\ub9b0\ub354 \ub300\uc2e0 \uc5d4\uc9c4\uc758 \ubc30\uae30\uac00\uc2a4\ub97c \uc774\uc6a9\ud558\uba74\uc11c 1941\ub144 11\uc6d4 \ubd80\ud130 \uc0ac\uc6a9\ub418\uc5c8\ub2e4. \uc774 \uac00\uc2a4\ucc28\ub4e4\uc740 1941\ub144 12\uc6d4 \ud5e4\uc6c0\ub178 \uc9d1\ub2e8 \uc218\uc6a9\uc18c(Che\u0142mno extermination camp)\uc5d0 \uc18c\uac1c\ub418\uc5c8\uace0 \uadf8\uac83 \uc911 15\ub300\ub294 \uc810\ub839\ub41c \uc18c\ub828\uc5d0\uc11c \ub300\ub7c9\uc0b4\uc0c1\uc870\uc9c1\uc778 \uc544\uc778\uc790\uce20\uadf8\ub8e8\ud39c\uc5d0 \uc758\ud574 \uc0ac\uc6a9\ub418\uc5c8\ub2e4. \uc774 \uac00\uc2a4\ucc28\ub294 \uad6d\uac00\ubcf4\uc548\ubcf8\ubd80(SS-Reichssicherheitshauptamt)\uc758 \uac10\ub3c5 \uc544\ub798\uc5d0\uc11c \uac1c\ubc1c\ub418\uc5c8\uace0 \uc6b4\uc601\ub418\uc5c8\ub2e4. \ub610\ud55c \ub300\ub7b5 50\ub9cc\uba85\uc758 \uc0ac\ub78c\ub4e4\uc744 \uc8fd\uc774\ub294\ub370 \uc0ac\uc6a9\ub418\uc5c8\ub294\ub370 \uadf8 \uc0ac\ub78c\ub4e4\uc740 \uc8fc\ub85c \uc720\ub300\uc778\uc774\uc5c8\uc9c0\ub9cc \uc9d1\uc2dc\uc640 \ub2e4\ub978 \ub098\ub77c \uc0ac\ub78c\ub4e4\ub3c4 \ud3ec\ud568\ub418\uc5c8\ub2e4. \uac00\uc2a4\ucc28\ub294 \uc2e0\uc911\ud558\uac8c \uac10\ub3c5\ubc1b\uc558\uace0 \ud55c\ub2ec \ub3d9\uc548\uc758 \uad00\ucc30 \ud6c4\uc758 \ud55c \uae30\ub85d\uc5d0\uc11c\ub294 \u201c\uae30\uacc4\uc5d0 \uc5b4\ub5a0\ud55c \uacb0\ud568\ub3c4 \ub098\ud0c0\ub098\uc9c0 \uc54a\uace0 \uc138 \ub300\uc758 \uac00\uc2a4\ucc28\ub97c \uc774\uc6a9\ud558\uc5ec 9\ub9cc7\ucc9c\ubc88\uc774 \uc9c4\ud589\ub418\uc5c8\ub2e4\u201d \ub77c\uace0 \uc4f0\uc5ec\uc838 \uc788\ub2e4.", "sentence_answer": "\uc791\uc13c\ud558\uc6b0\uc820 \uc218\uc6a9\uc18c\uc5d0\uc120 100\uba85\uae4c\uc9c0 \uc218\uc6a9\ud560 \uc218 \uc788\ub294 \ub354 \ud070 \uac00\uc2a4\ucc28\uac00 \uc2e4\ub9b0\ub354 \ub300\uc2e0 \uc5d4\uc9c4\uc758 \ubc30\uae30\uac00\uc2a4\ub97c \uc774\uc6a9\ud558\uba74\uc11c 1941\ub144 11\uc6d4 \ubd80\ud130 \uc0ac\uc6a9\ub418\uc5c8\ub2e4.", "paragraph_id": "6571961-13-1", "paragraph_question": "question: \uc791\uc13c\ud558\uc6b0\uc820 \uc218\uc6a9\uc18c\uc5d0\uc11c \uc5d4\uc9c4\uc758 \ubc30\uae30\uac00\uc2a4\ub97c \uc774\uc6a9\ud55c \uac00\uc2a4\ucc28\ub97c \uc0ac\uc6a9\ud55c \uac83\uc740 \uc5b8\uc81c\ubd80\ud130\uc778\uac00?, context: 1939\ub144 12\uc6d4\uc758 \uc2dc\uc791\uacfc \ud568\uaed8 \ub098\uce58\ub294 \uac00\uc2a4\ub97c \uc774\uc6a9\ud55c \ub2e4\uc911 \uc0b4\uc778\uc758 \uc0c8\ub85c\uc6b4 \ubc29\ubc95\uc744 \uc18c\uac1c\ud558\uc600\ub2e4. \uccab\uc9f8\ub85c, \uac00\uc2a4 \uc2e4\ub9b0\ub354\ub97c \uc7a5\ucc29\ud55c \uc2e4\ud5d8\uc6a9 \uac00\uc2a4\ucc28(gas van)\uc640 \ubc00\ud3d0\ub41c \ud2b8\ub801\ud06c \uce78\ub9c9\uc774\ub294 T-4 \uc791\uc804(\ub098\uce58\uc758 \uc778\uc885\uc815\ucc45 \uc77c\ud658\uc73c\ub85c \uc548\ub77d\uc0ac\ub97c \uc774\uc6a9\ud55c \uc7a5\uc560\uc778 \ud559\uc0b4 \uacc4\ud68d)\uc758 \uc77c\ud658\uc73c\ub85c\uc368 \ud3ec\uba54\ub77c\ub2c8\uc544, \ub3d9\ud504\ub85c\uc774\uc13c \uadf8\ub9ac\uace0 \uc810\ub839\ub41c \ud3f4\ub780\ub4dc\uc758 \uc694\uc591\uc6d0\uc5d0 \uc788\ub294 \uc815\uc2e0 \uc9c8\ud658\uc790\ub4e4\uc744 \uc8fd\uc774\ub294\ub370 \uc0ac\uc6a9\ub418\uc5c8\ub2e4. \uc791\uc13c\ud558\uc6b0\uc820 \uc218\uc6a9\uc18c\uc5d0\uc120 100\uba85\uae4c\uc9c0 \uc218\uc6a9\ud560 \uc218 \uc788\ub294 \ub354 \ud070 \uac00\uc2a4\ucc28\uac00 \uc2e4\ub9b0\ub354 \ub300\uc2e0 \uc5d4\uc9c4\uc758 \ubc30\uae30\uac00\uc2a4\ub97c \uc774\uc6a9\ud558\uba74\uc11c 1941\ub144 11\uc6d4\ubd80\ud130 \uc0ac\uc6a9\ub418\uc5c8\ub2e4. \uc774 \uac00\uc2a4\ucc28\ub4e4\uc740 1941\ub144 12\uc6d4 \ud5e4\uc6c0\ub178 \uc9d1\ub2e8 \uc218\uc6a9\uc18c(Che\u0142mno extermination camp)\uc5d0 \uc18c\uac1c\ub418\uc5c8\uace0 \uadf8\uac83 \uc911 15\ub300\ub294 \uc810\ub839\ub41c \uc18c\ub828\uc5d0\uc11c \ub300\ub7c9\uc0b4\uc0c1\uc870\uc9c1\uc778 \uc544\uc778\uc790\uce20\uadf8\ub8e8\ud39c\uc5d0 \uc758\ud574 \uc0ac\uc6a9\ub418\uc5c8\ub2e4. \uc774 \uac00\uc2a4\ucc28\ub294 \uad6d\uac00\ubcf4\uc548\ubcf8\ubd80(SS-Reichssicherheitshauptamt)\uc758 \uac10\ub3c5 \uc544\ub798\uc5d0\uc11c \uac1c\ubc1c\ub418\uc5c8\uace0 \uc6b4\uc601\ub418\uc5c8\ub2e4. \ub610\ud55c \ub300\ub7b5 50\ub9cc\uba85\uc758 \uc0ac\ub78c\ub4e4\uc744 \uc8fd\uc774\ub294\ub370 \uc0ac\uc6a9\ub418\uc5c8\ub294\ub370 \uadf8 \uc0ac\ub78c\ub4e4\uc740 \uc8fc\ub85c \uc720\ub300\uc778\uc774\uc5c8\uc9c0\ub9cc \uc9d1\uc2dc\uc640 \ub2e4\ub978 \ub098\ub77c \uc0ac\ub78c\ub4e4\ub3c4 \ud3ec\ud568\ub418\uc5c8\ub2e4. \uac00\uc2a4\ucc28\ub294 \uc2e0\uc911\ud558\uac8c \uac10\ub3c5\ubc1b\uc558\uace0 \ud55c\ub2ec \ub3d9\uc548\uc758 \uad00\ucc30 \ud6c4\uc758 \ud55c \uae30\ub85d\uc5d0\uc11c\ub294 \u201c\uae30\uacc4\uc5d0 \uc5b4\ub5a0\ud55c \uacb0\ud568\ub3c4 \ub098\ud0c0\ub098\uc9c0 \uc54a\uace0 \uc138 \ub300\uc758 \uac00\uc2a4\ucc28\ub97c \uc774\uc6a9\ud558\uc5ec 9\ub9cc7\ucc9c\ubc88\uc774 \uc9c4\ud589\ub418\uc5c8\ub2e4\u201d \ub77c\uace0 \uc4f0\uc5ec\uc838 \uc788\ub2e4."} {"question": "\uad6d\ubbfc\uc758\ud68c\uac00 \uc815\ubd80 \uc5c6\uc774 \ud5cc\ubc95\uc744 \ub9cc\ub4e4\uc5b4 \uadf8\ub4e4\uc758 \uc9c0\uc704\uac00 \ubd88\uc548\ud588\ub2e4\uace0 \ube44\ub09c\ud55c \uc778\ubb3c\uc740?", "paragraph": "\uc81c\ud5cc, \uc785\ubc95\uacfc\uc815\uc5d0\uc11c \uad6d\ubbfc\uc758\ud68c\uc758 \uc8fc\ubaa9\ud560 \ub9cc\ud55c \uc5c5\uc801\uc5d0\ub3c4 \ubd88\uad6c\ud558\uace0 \ud504\ub791\ud06c \ub85c\ub80c\uce20 \ubb90\ub7ec(Frank Lorenz M\u00fcller)\ub294 \uad6d\ubbfc\uc758\ud68c\uac00 \"\uc801\uadf9\uc801\uc774\uace0 \uc2e4\uc6a9\uc801\uc73c\ub85c \ud589\ub3d9\ud558\ub294 \ub2e8\uccb4\"\ub85c\uae4c\uc9c0 \uc131\uc7a5\ud558\uc9c0\ub294 \ubabb\ud588\uc73c\uba70, \uadf8\uc800 \ub108\ubb34 \uc804\ubb38\uc801\uc774\uace0 \ucca0\ud559\uc801\uc73c\ub85c \ub0a8\uc558\ub2e4\uace0 \uc0dd\uac01\ud55c\ub2e4. \ubca0\ub978\ud558\ub974\ud2b8 \ub9cc(Bernhard Mann)\uc740 \uad6d\ubbfc\uc758\ud68c\uac00 1848\ub144 5\uc6d4\uacfc 6\uc6d4\uc5d0 \uadf8\ub4e4\uc758 \uc720\ub9ac\ud55c \uc0c1\ud669\uc744 \uc9c0\ub098\uce58\uac8c \uc790\uadf9\ud588\uc73c\uba70 \uc815\ubd80 \uc5c6\uc774 \ud5cc\ubc95\uc744 \ub9cc\ub4e4\uc5b4 \uadf8\ub4e4\uc758 \uc9c0\uc704\uac00 \ubd88\uc548\ud588\ub2e4\uace0 \ube44\ub09c\ud55c\ub2e4. \ubfd0\ub9cc \uc544\ub2c8\ub77c \uad6d\ubbfc\uc758\ud68c\ub294 \uc758\ud68c\uc8fc\uc758\uc640 \uc815\ub2f9\ud615\uc131\uc774 \ub3c5\uc77c \uc804\uccb4\uc5d0\uc11c \uc774\ub8e8\uc5b4\uc9c0\uac8c \ud558\uae30 \uc704\ud574\uc11c \uadf8\ub4e4\uc5d0\uac8c \uc644\uc804\ud788 \uc885\uc18d\ub41c \uc911\uc559 \uad8c\ub825\uc744 \ub9cc\ub4e4\uae30 \uc704\ud574 \ub178\ub825\ud588\uc5b4\uc57c \ud588\ub2e4. \ub2e4\ub9cc \uadfc\ub300 \uc815\ub2f9 \uc81c\ub3c4\uac00 \ubb34\uc5c7\ubcf4\ub2e4\ub3c4 \uc5b8\ub860\u00b7\uc9d1\ud68c\u00b7\uacb0\uc0ac\uc758 \uc790\uc720\ub97c \ud1b5\ud574\uc11c\uc57c \ube44\ub85c\uc18c \uac00\ub2a5\ud574\uc84c\ub2e4\ub294 \uc810\uacfc \uc758\ud68c\uc8fc\uc758\uc5d0 \ub300\ud55c \uacbd\ud5d8\uc774 \uc624\uc2a4\ud2b8\ub9ac\uc544 \uc815\uce58\uac00\ub4e4\uc740 \ub9e4\uc6b0 \uc801\uc5c8\uace0 \ud504\ub85c\uc774\uc13c \uc815\uce58\uac00\ub4e4\uc740 \uc5b4\uca0c\ub4e0 \uc804\ud600 \uc5c6\uc5c8\ub2e4\ub294 \uc810\uc740 \ub367\ubd99\uc5ec\uc57c \uc815\ub2f9\ud55c \ud3c9\uac00\uac00 \ub420 \uac83\uc774\ub2e4.", "answer": "\ubca0\ub978\ud558\ub974\ud2b8 \ub9cc", "sentence": "\ubca0\ub978\ud558\ub974\ud2b8 \ub9cc (Bernhard Mann)", "paragraph_sentence": "\uc81c\ud5cc, \uc785\ubc95\uacfc\uc815\uc5d0\uc11c \uad6d\ubbfc\uc758\ud68c\uc758 \uc8fc\ubaa9\ud560 \ub9cc\ud55c \uc5c5\uc801\uc5d0\ub3c4 \ubd88\uad6c\ud558\uace0 \ud504\ub791\ud06c \ub85c\ub80c\uce20 \ubb90\ub7ec(Frank Lorenz M\u00fcller)\ub294 \uad6d\ubbfc\uc758\ud68c\uac00 \"\uc801\uadf9\uc801\uc774\uace0 \uc2e4\uc6a9\uc801\uc73c\ub85c \ud589\ub3d9\ud558\ub294 \ub2e8\uccb4\"\ub85c\uae4c\uc9c0 \uc131\uc7a5\ud558\uc9c0\ub294 \ubabb\ud588\uc73c\uba70, \uadf8\uc800 \ub108\ubb34 \uc804\ubb38\uc801\uc774\uace0 \ucca0\ud559\uc801\uc73c\ub85c \ub0a8\uc558\ub2e4\uace0 \uc0dd\uac01\ud55c\ub2e4. \ubca0\ub978\ud558\ub974\ud2b8 \ub9cc (Bernhard Mann) \uc740 \uad6d\ubbfc\uc758\ud68c\uac00 1848\ub144 5\uc6d4\uacfc 6\uc6d4\uc5d0 \uadf8\ub4e4\uc758 \uc720\ub9ac\ud55c \uc0c1\ud669\uc744 \uc9c0\ub098\uce58\uac8c \uc790\uadf9\ud588\uc73c\uba70 \uc815\ubd80 \uc5c6\uc774 \ud5cc\ubc95\uc744 \ub9cc\ub4e4\uc5b4 \uadf8\ub4e4\uc758 \uc9c0\uc704\uac00 \ubd88\uc548\ud588\ub2e4\uace0 \ube44\ub09c\ud55c\ub2e4. \ubfd0\ub9cc \uc544\ub2c8\ub77c \uad6d\ubbfc\uc758\ud68c\ub294 \uc758\ud68c\uc8fc\uc758\uc640 \uc815\ub2f9\ud615\uc131\uc774 \ub3c5\uc77c \uc804\uccb4\uc5d0\uc11c \uc774\ub8e8\uc5b4\uc9c0\uac8c \ud558\uae30 \uc704\ud574\uc11c \uadf8\ub4e4\uc5d0\uac8c \uc644\uc804\ud788 \uc885\uc18d\ub41c \uc911\uc559 \uad8c\ub825\uc744 \ub9cc\ub4e4\uae30 \uc704\ud574 \ub178\ub825\ud588\uc5b4\uc57c \ud588\ub2e4. \ub2e4\ub9cc \uadfc\ub300 \uc815\ub2f9 \uc81c\ub3c4\uac00 \ubb34\uc5c7\ubcf4\ub2e4\ub3c4 \uc5b8\ub860\u00b7\uc9d1\ud68c\u00b7\uacb0\uc0ac\uc758 \uc790\uc720\ub97c \ud1b5\ud574\uc11c\uc57c \ube44\ub85c\uc18c \uac00\ub2a5\ud574\uc84c\ub2e4\ub294 \uc810\uacfc \uc758\ud68c\uc8fc\uc758\uc5d0 \ub300\ud55c \uacbd\ud5d8\uc774 \uc624\uc2a4\ud2b8\ub9ac\uc544 \uc815\uce58\uac00\ub4e4\uc740 \ub9e4\uc6b0 \uc801\uc5c8\uace0 \ud504\ub85c\uc774\uc13c \uc815\uce58\uac00\ub4e4\uc740 \uc5b4\uca0c\ub4e0 \uc804\ud600 \uc5c6\uc5c8\ub2e4\ub294 \uc810\uc740 \ub367\ubd99\uc5ec\uc57c \uc815\ub2f9\ud55c \ud3c9\uac00\uac00 \ub420 \uac83\uc774\ub2e4.", "paragraph_answer": "\uc81c\ud5cc, \uc785\ubc95\uacfc\uc815\uc5d0\uc11c \uad6d\ubbfc\uc758\ud68c\uc758 \uc8fc\ubaa9\ud560 \ub9cc\ud55c \uc5c5\uc801\uc5d0\ub3c4 \ubd88\uad6c\ud558\uace0 \ud504\ub791\ud06c \ub85c\ub80c\uce20 \ubb90\ub7ec(Frank Lorenz M\u00fcller)\ub294 \uad6d\ubbfc\uc758\ud68c\uac00 \"\uc801\uadf9\uc801\uc774\uace0 \uc2e4\uc6a9\uc801\uc73c\ub85c \ud589\ub3d9\ud558\ub294 \ub2e8\uccb4\"\ub85c\uae4c\uc9c0 \uc131\uc7a5\ud558\uc9c0\ub294 \ubabb\ud588\uc73c\uba70, \uadf8\uc800 \ub108\ubb34 \uc804\ubb38\uc801\uc774\uace0 \ucca0\ud559\uc801\uc73c\ub85c \ub0a8\uc558\ub2e4\uace0 \uc0dd\uac01\ud55c\ub2e4. \ubca0\ub978\ud558\ub974\ud2b8 \ub9cc (Bernhard Mann)\uc740 \uad6d\ubbfc\uc758\ud68c\uac00 1848\ub144 5\uc6d4\uacfc 6\uc6d4\uc5d0 \uadf8\ub4e4\uc758 \uc720\ub9ac\ud55c \uc0c1\ud669\uc744 \uc9c0\ub098\uce58\uac8c \uc790\uadf9\ud588\uc73c\uba70 \uc815\ubd80 \uc5c6\uc774 \ud5cc\ubc95\uc744 \ub9cc\ub4e4\uc5b4 \uadf8\ub4e4\uc758 \uc9c0\uc704\uac00 \ubd88\uc548\ud588\ub2e4\uace0 \ube44\ub09c\ud55c\ub2e4. \ubfd0\ub9cc \uc544\ub2c8\ub77c \uad6d\ubbfc\uc758\ud68c\ub294 \uc758\ud68c\uc8fc\uc758\uc640 \uc815\ub2f9\ud615\uc131\uc774 \ub3c5\uc77c \uc804\uccb4\uc5d0\uc11c \uc774\ub8e8\uc5b4\uc9c0\uac8c \ud558\uae30 \uc704\ud574\uc11c \uadf8\ub4e4\uc5d0\uac8c \uc644\uc804\ud788 \uc885\uc18d\ub41c \uc911\uc559 \uad8c\ub825\uc744 \ub9cc\ub4e4\uae30 \uc704\ud574 \ub178\ub825\ud588\uc5b4\uc57c \ud588\ub2e4. \ub2e4\ub9cc \uadfc\ub300 \uc815\ub2f9 \uc81c\ub3c4\uac00 \ubb34\uc5c7\ubcf4\ub2e4\ub3c4 \uc5b8\ub860\u00b7\uc9d1\ud68c\u00b7\uacb0\uc0ac\uc758 \uc790\uc720\ub97c \ud1b5\ud574\uc11c\uc57c \ube44\ub85c\uc18c \uac00\ub2a5\ud574\uc84c\ub2e4\ub294 \uc810\uacfc \uc758\ud68c\uc8fc\uc758\uc5d0 \ub300\ud55c \uacbd\ud5d8\uc774 \uc624\uc2a4\ud2b8\ub9ac\uc544 \uc815\uce58\uac00\ub4e4\uc740 \ub9e4\uc6b0 \uc801\uc5c8\uace0 \ud504\ub85c\uc774\uc13c \uc815\uce58\uac00\ub4e4\uc740 \uc5b4\uca0c\ub4e0 \uc804\ud600 \uc5c6\uc5c8\ub2e4\ub294 \uc810\uc740 \ub367\ubd99\uc5ec\uc57c \uc815\ub2f9\ud55c \ud3c9\uac00\uac00 \ub420 \uac83\uc774\ub2e4.", "sentence_answer": " \ubca0\ub978\ud558\ub974\ud2b8 \ub9cc (Bernhard Mann)", "paragraph_id": "6568666-33-1", "paragraph_question": "question: \uad6d\ubbfc\uc758\ud68c\uac00 \uc815\ubd80 \uc5c6\uc774 \ud5cc\ubc95\uc744 \ub9cc\ub4e4\uc5b4 \uadf8\ub4e4\uc758 \uc9c0\uc704\uac00 \ubd88\uc548\ud588\ub2e4\uace0 \ube44\ub09c\ud55c \uc778\ubb3c\uc740?, context: \uc81c\ud5cc, \uc785\ubc95\uacfc\uc815\uc5d0\uc11c \uad6d\ubbfc\uc758\ud68c\uc758 \uc8fc\ubaa9\ud560 \ub9cc\ud55c \uc5c5\uc801\uc5d0\ub3c4 \ubd88\uad6c\ud558\uace0 \ud504\ub791\ud06c \ub85c\ub80c\uce20 \ubb90\ub7ec(Frank Lorenz M\u00fcller)\ub294 \uad6d\ubbfc\uc758\ud68c\uac00 \"\uc801\uadf9\uc801\uc774\uace0 \uc2e4\uc6a9\uc801\uc73c\ub85c \ud589\ub3d9\ud558\ub294 \ub2e8\uccb4\"\ub85c\uae4c\uc9c0 \uc131\uc7a5\ud558\uc9c0\ub294 \ubabb\ud588\uc73c\uba70, \uadf8\uc800 \ub108\ubb34 \uc804\ubb38\uc801\uc774\uace0 \ucca0\ud559\uc801\uc73c\ub85c \ub0a8\uc558\ub2e4\uace0 \uc0dd\uac01\ud55c\ub2e4. \ubca0\ub978\ud558\ub974\ud2b8 \ub9cc(Bernhard Mann)\uc740 \uad6d\ubbfc\uc758\ud68c\uac00 1848\ub144 5\uc6d4\uacfc 6\uc6d4\uc5d0 \uadf8\ub4e4\uc758 \uc720\ub9ac\ud55c \uc0c1\ud669\uc744 \uc9c0\ub098\uce58\uac8c \uc790\uadf9\ud588\uc73c\uba70 \uc815\ubd80 \uc5c6\uc774 \ud5cc\ubc95\uc744 \ub9cc\ub4e4\uc5b4 \uadf8\ub4e4\uc758 \uc9c0\uc704\uac00 \ubd88\uc548\ud588\ub2e4\uace0 \ube44\ub09c\ud55c\ub2e4. \ubfd0\ub9cc \uc544\ub2c8\ub77c \uad6d\ubbfc\uc758\ud68c\ub294 \uc758\ud68c\uc8fc\uc758\uc640 \uc815\ub2f9\ud615\uc131\uc774 \ub3c5\uc77c \uc804\uccb4\uc5d0\uc11c \uc774\ub8e8\uc5b4\uc9c0\uac8c \ud558\uae30 \uc704\ud574\uc11c \uadf8\ub4e4\uc5d0\uac8c \uc644\uc804\ud788 \uc885\uc18d\ub41c \uc911\uc559 \uad8c\ub825\uc744 \ub9cc\ub4e4\uae30 \uc704\ud574 \ub178\ub825\ud588\uc5b4\uc57c \ud588\ub2e4. \ub2e4\ub9cc \uadfc\ub300 \uc815\ub2f9 \uc81c\ub3c4\uac00 \ubb34\uc5c7\ubcf4\ub2e4\ub3c4 \uc5b8\ub860\u00b7\uc9d1\ud68c\u00b7\uacb0\uc0ac\uc758 \uc790\uc720\ub97c \ud1b5\ud574\uc11c\uc57c \ube44\ub85c\uc18c \uac00\ub2a5\ud574\uc84c\ub2e4\ub294 \uc810\uacfc \uc758\ud68c\uc8fc\uc758\uc5d0 \ub300\ud55c \uacbd\ud5d8\uc774 \uc624\uc2a4\ud2b8\ub9ac\uc544 \uc815\uce58\uac00\ub4e4\uc740 \ub9e4\uc6b0 \uc801\uc5c8\uace0 \ud504\ub85c\uc774\uc13c \uc815\uce58\uac00\ub4e4\uc740 \uc5b4\uca0c\ub4e0 \uc804\ud600 \uc5c6\uc5c8\ub2e4\ub294 \uc810\uc740 \ub367\ubd99\uc5ec\uc57c \uc815\ub2f9\ud55c \ud3c9\uac00\uac00 \ub420 \uac83\uc774\ub2e4."} {"question": "\ud55c\uad6d\uc774 \uc0c1\ub2f9\ud55c \uc561\uc218\uc758 \ubc29\uc704\ube44\ub97c \ubd84\ub2f4\ud558\uace0 \uc788\uc74c\uc744 \ubc1d\ud78c \ud2b8\ub7fc\ud504 \uc815\ubd80 \uce21 \uc778\uc0ac\ub294?", "paragraph": "\uc0ac\ub4dc \ubc30\uce58 \ube44\uc6a9\uacfc \uc0ac\ub4dc \uc6b4\uc601 \ube44\uc6a9\uc740 \ubaa8\ub450 \ubbf8\uad6d\uc774 \uc804\uc561 \ubd80\ub2f4\ud558\ub294 \uac83\uc73c\ub85c \uc774\ubbf8 \ud569\uc758\ub418\uc5c8\ub2e4. \uc0c8\ub85c\uc6b4 \uc0ac\ub4dc\ub97c \ucd94\uac00\ub85c \uc0dd\uc0b0\ud558\ub294 \uac83\uc774 \uc544\ub2c8\ub77c \ubbf8\uad6d \ubcf8\ud1a0\uc5d0\uc11c \uc774\ubbf8 \uc624\ub798\uc804\ubd80\ud130 \uc6b4\uc6a9 \uc911\uc774\uc5c8\ub358 \uc0ac\ub4dc \ubd80\ub300\ub97c \ud55c\ubc18\ub3c4\ub85c \uc774\ub3d9 \ud558\uae30\ub85c \ud569\uc758\ub418\uc5c8\ub2e4. 2017\ub144 2\uc6d4, \ud2b8\ub7fc\ud504 \ud589\uc815\ubd80\uc758 \ub809\uc2a4 \ud2f8\ub7ec\uc2a8 \uad6d\ubb34\uc7a5\uad00\uc740 \ud55c\uad6d\uc740 \uc774\ubbf8 \uc0c1\ub2f9\ud55c \ubc29\uc704\ube44\ub97c \ubd84\ub2f4\ud558\uace0 \uc788\ub2e4\ub294 \uc785\uc7a5\uc744 \ubc1d\ud614\ub2e4. \ud2f8\ub7ec\uc2a8 \uc7a5\uad00\uc740 \u201c\ud55c\uad6d\uacfc \uc77c\ubcf8\uc740 \uc774\ubbf8 \ubbf8\uad70\uc744 \uc9c0\uc6d0\ud558\ub294\ub370 \uc774\ubbf8 \ub9ce\uc740 \uc591\uc744 \uae30\uc5ec\ud558\uace0 \uc788\ub2e4\u201d\uba74\uc11c \u201c\ud5a5\ud6c4 \uad00\ub828 \ub300\ud654\uac00 \uc0dd\uc0b0\uc801\uc73c\ub85c \uc9c4\ud589\ub418\uace0, \uacf5\ud3c9\ud55c \ubd84\ub2f4\uae08 \ud569\uc758\uac00 \uc774\ub904\uc9c8 \uac83\uc73c\ub85c \ub099\uad00\ud55c\ub2e4\u201d\uace0 \ub9d0\ud588\ub2e4. \ub3c4\ub110\ub4dc \ud2b8\ub7fc\ud504 \ub300\ud1b5\ub839\uc774 \uc0ac\ub4dc \ubc30\uce58 \ube44\uc6a9 10\uc5b5 \ub2ec\ub7ec(\uc57d 1\uc8701\ucc9c\uc5b5\uc6d0)\ub97c \ud55c\uad6d\uc5d0\uac8c \ub0b4\ub77c\uace0 \uc694\uad6c\ud55c \uac83\uc5d0 \ub300\ud558\uc5ec \uae40\uad00\uc9c4 \uad6d\uac00\uc548\ubcf4\uc2e4\uc7a5\uc740 2017\ub144 4\uc6d4 30\uc77c \ud5c8\ubc84\ud2b8 \ub9e5\ub9c8\uc2a4\ud130 \ubbf8\uad6d \ubc31\uc545\uad00 \uad6d\uac00\uc548\ubcf4\ubcf4\uc88c\uad00\uacfc \uc804\ud654 \ud611\uc758\ub97c \uac16\uace0 \uc0ac\ub4dc \ubc30\uce58 \ube44\uc6a9\uc740 \ubbf8\uad6d\uc774 \uc804\uc561 \ubd80\ub2f4\ud55c\ub2e4\ub294 \uae30\uc874 \ud569\uc758 \ub0b4\uc6a9\uc744 \uc7ac\ud655\uc778\ud588\ub2e4.", "answer": "\ub809\uc2a4 \ud2f8\ub7ec\uc2a8", "sentence": "2017\ub144 2\uc6d4, \ud2b8\ub7fc\ud504 \ud589\uc815\ubd80\uc758 \ub809\uc2a4 \ud2f8\ub7ec\uc2a8 \uad6d\ubb34\uc7a5\uad00\uc740 \ud55c\uad6d\uc740 \uc774\ubbf8 \uc0c1\ub2f9\ud55c \ubc29\uc704\ube44\ub97c \ubd84\ub2f4\ud558\uace0 \uc788\ub2e4\ub294 \uc785\uc7a5\uc744 \ubc1d\ud614\ub2e4.", "paragraph_sentence": "\uc0ac\ub4dc \ubc30\uce58 \ube44\uc6a9\uacfc \uc0ac\ub4dc \uc6b4\uc601 \ube44\uc6a9\uc740 \ubaa8\ub450 \ubbf8\uad6d\uc774 \uc804\uc561 \ubd80\ub2f4\ud558\ub294 \uac83\uc73c\ub85c \uc774\ubbf8 \ud569\uc758\ub418\uc5c8\ub2e4. \uc0c8\ub85c\uc6b4 \uc0ac\ub4dc\ub97c \ucd94\uac00\ub85c \uc0dd\uc0b0\ud558\ub294 \uac83\uc774 \uc544\ub2c8\ub77c \ubbf8\uad6d \ubcf8\ud1a0\uc5d0\uc11c \uc774\ubbf8 \uc624\ub798\uc804\ubd80\ud130 \uc6b4\uc6a9 \uc911\uc774\uc5c8\ub358 \uc0ac\ub4dc \ubd80\ub300\ub97c \ud55c\ubc18\ub3c4\ub85c \uc774\ub3d9 \ud558\uae30\ub85c \ud569\uc758\ub418\uc5c8\ub2e4. 2017\ub144 2\uc6d4, \ud2b8\ub7fc\ud504 \ud589\uc815\ubd80\uc758 \ub809\uc2a4 \ud2f8\ub7ec\uc2a8 \uad6d\ubb34\uc7a5\uad00\uc740 \ud55c\uad6d\uc740 \uc774\ubbf8 \uc0c1\ub2f9\ud55c \ubc29\uc704\ube44\ub97c \ubd84\ub2f4\ud558\uace0 \uc788\ub2e4\ub294 \uc785\uc7a5\uc744 \ubc1d\ud614\ub2e4. \ud2f8\ub7ec\uc2a8 \uc7a5\uad00\uc740 \u201c\ud55c\uad6d\uacfc \uc77c\ubcf8\uc740 \uc774\ubbf8 \ubbf8\uad70\uc744 \uc9c0\uc6d0\ud558\ub294\ub370 \uc774\ubbf8 \ub9ce\uc740 \uc591\uc744 \uae30\uc5ec\ud558\uace0 \uc788\ub2e4\u201d\uba74\uc11c \u201c\ud5a5\ud6c4 \uad00\ub828 \ub300\ud654\uac00 \uc0dd\uc0b0\uc801\uc73c\ub85c \uc9c4\ud589\ub418\uace0, \uacf5\ud3c9\ud55c \ubd84\ub2f4\uae08 \ud569\uc758\uac00 \uc774\ub904\uc9c8 \uac83\uc73c\ub85c \ub099\uad00\ud55c\ub2e4\u201d\uace0 \ub9d0\ud588\ub2e4. \ub3c4\ub110\ub4dc \ud2b8\ub7fc\ud504 \ub300\ud1b5\ub839\uc774 \uc0ac\ub4dc \ubc30\uce58 \ube44\uc6a9 10\uc5b5 \ub2ec\ub7ec(\uc57d 1\uc8701\ucc9c\uc5b5\uc6d0)\ub97c \ud55c\uad6d\uc5d0\uac8c \ub0b4\ub77c\uace0 \uc694\uad6c\ud55c \uac83\uc5d0 \ub300\ud558\uc5ec \uae40\uad00\uc9c4 \uad6d\uac00\uc548\ubcf4\uc2e4\uc7a5\uc740 2017\ub144 4\uc6d4 30\uc77c \ud5c8\ubc84\ud2b8 \ub9e5\ub9c8\uc2a4\ud130 \ubbf8\uad6d \ubc31\uc545\uad00 \uad6d\uac00\uc548\ubcf4\ubcf4\uc88c\uad00\uacfc \uc804\ud654 \ud611\uc758\ub97c \uac16\uace0 \uc0ac\ub4dc \ubc30\uce58 \ube44\uc6a9\uc740 \ubbf8\uad6d\uc774 \uc804\uc561 \ubd80\ub2f4\ud55c\ub2e4\ub294 \uae30\uc874 \ud569\uc758 \ub0b4\uc6a9\uc744 \uc7ac\ud655\uc778\ud588\ub2e4.", "paragraph_answer": "\uc0ac\ub4dc \ubc30\uce58 \ube44\uc6a9\uacfc \uc0ac\ub4dc \uc6b4\uc601 \ube44\uc6a9\uc740 \ubaa8\ub450 \ubbf8\uad6d\uc774 \uc804\uc561 \ubd80\ub2f4\ud558\ub294 \uac83\uc73c\ub85c \uc774\ubbf8 \ud569\uc758\ub418\uc5c8\ub2e4. \uc0c8\ub85c\uc6b4 \uc0ac\ub4dc\ub97c \ucd94\uac00\ub85c \uc0dd\uc0b0\ud558\ub294 \uac83\uc774 \uc544\ub2c8\ub77c \ubbf8\uad6d \ubcf8\ud1a0\uc5d0\uc11c \uc774\ubbf8 \uc624\ub798\uc804\ubd80\ud130 \uc6b4\uc6a9 \uc911\uc774\uc5c8\ub358 \uc0ac\ub4dc \ubd80\ub300\ub97c \ud55c\ubc18\ub3c4\ub85c \uc774\ub3d9 \ud558\uae30\ub85c \ud569\uc758\ub418\uc5c8\ub2e4. 2017\ub144 2\uc6d4, \ud2b8\ub7fc\ud504 \ud589\uc815\ubd80\uc758 \ub809\uc2a4 \ud2f8\ub7ec\uc2a8 \uad6d\ubb34\uc7a5\uad00\uc740 \ud55c\uad6d\uc740 \uc774\ubbf8 \uc0c1\ub2f9\ud55c \ubc29\uc704\ube44\ub97c \ubd84\ub2f4\ud558\uace0 \uc788\ub2e4\ub294 \uc785\uc7a5\uc744 \ubc1d\ud614\ub2e4. \ud2f8\ub7ec\uc2a8 \uc7a5\uad00\uc740 \u201c\ud55c\uad6d\uacfc \uc77c\ubcf8\uc740 \uc774\ubbf8 \ubbf8\uad70\uc744 \uc9c0\uc6d0\ud558\ub294\ub370 \uc774\ubbf8 \ub9ce\uc740 \uc591\uc744 \uae30\uc5ec\ud558\uace0 \uc788\ub2e4\u201d\uba74\uc11c \u201c\ud5a5\ud6c4 \uad00\ub828 \ub300\ud654\uac00 \uc0dd\uc0b0\uc801\uc73c\ub85c \uc9c4\ud589\ub418\uace0, \uacf5\ud3c9\ud55c \ubd84\ub2f4\uae08 \ud569\uc758\uac00 \uc774\ub904\uc9c8 \uac83\uc73c\ub85c \ub099\uad00\ud55c\ub2e4\u201d\uace0 \ub9d0\ud588\ub2e4. \ub3c4\ub110\ub4dc \ud2b8\ub7fc\ud504 \ub300\ud1b5\ub839\uc774 \uc0ac\ub4dc \ubc30\uce58 \ube44\uc6a9 10\uc5b5 \ub2ec\ub7ec(\uc57d 1\uc8701\ucc9c\uc5b5\uc6d0)\ub97c \ud55c\uad6d\uc5d0\uac8c \ub0b4\ub77c\uace0 \uc694\uad6c\ud55c \uac83\uc5d0 \ub300\ud558\uc5ec \uae40\uad00\uc9c4 \uad6d\uac00\uc548\ubcf4\uc2e4\uc7a5\uc740 2017\ub144 4\uc6d4 30\uc77c \ud5c8\ubc84\ud2b8 \ub9e5\ub9c8\uc2a4\ud130 \ubbf8\uad6d \ubc31\uc545\uad00 \uad6d\uac00\uc548\ubcf4\ubcf4\uc88c\uad00\uacfc \uc804\ud654 \ud611\uc758\ub97c \uac16\uace0 \uc0ac\ub4dc \ubc30\uce58 \ube44\uc6a9\uc740 \ubbf8\uad6d\uc774 \uc804\uc561 \ubd80\ub2f4\ud55c\ub2e4\ub294 \uae30\uc874 \ud569\uc758 \ub0b4\uc6a9\uc744 \uc7ac\ud655\uc778\ud588\ub2e4.", "sentence_answer": "2017\ub144 2\uc6d4, \ud2b8\ub7fc\ud504 \ud589\uc815\ubd80\uc758 \ub809\uc2a4 \ud2f8\ub7ec\uc2a8 \uad6d\ubb34\uc7a5\uad00\uc740 \ud55c\uad6d\uc740 \uc774\ubbf8 \uc0c1\ub2f9\ud55c \ubc29\uc704\ube44\ub97c \ubd84\ub2f4\ud558\uace0 \uc788\ub2e4\ub294 \uc785\uc7a5\uc744 \ubc1d\ud614\ub2e4.", "paragraph_id": "6486360-17-1", "paragraph_question": "question: \ud55c\uad6d\uc774 \uc0c1\ub2f9\ud55c \uc561\uc218\uc758 \ubc29\uc704\ube44\ub97c \ubd84\ub2f4\ud558\uace0 \uc788\uc74c\uc744 \ubc1d\ud78c \ud2b8\ub7fc\ud504 \uc815\ubd80 \uce21 \uc778\uc0ac\ub294?, context: \uc0ac\ub4dc \ubc30\uce58 \ube44\uc6a9\uacfc \uc0ac\ub4dc \uc6b4\uc601 \ube44\uc6a9\uc740 \ubaa8\ub450 \ubbf8\uad6d\uc774 \uc804\uc561 \ubd80\ub2f4\ud558\ub294 \uac83\uc73c\ub85c \uc774\ubbf8 \ud569\uc758\ub418\uc5c8\ub2e4. \uc0c8\ub85c\uc6b4 \uc0ac\ub4dc\ub97c \ucd94\uac00\ub85c \uc0dd\uc0b0\ud558\ub294 \uac83\uc774 \uc544\ub2c8\ub77c \ubbf8\uad6d \ubcf8\ud1a0\uc5d0\uc11c \uc774\ubbf8 \uc624\ub798\uc804\ubd80\ud130 \uc6b4\uc6a9 \uc911\uc774\uc5c8\ub358 \uc0ac\ub4dc \ubd80\ub300\ub97c \ud55c\ubc18\ub3c4\ub85c \uc774\ub3d9 \ud558\uae30\ub85c \ud569\uc758\ub418\uc5c8\ub2e4. 2017\ub144 2\uc6d4, \ud2b8\ub7fc\ud504 \ud589\uc815\ubd80\uc758 \ub809\uc2a4 \ud2f8\ub7ec\uc2a8 \uad6d\ubb34\uc7a5\uad00\uc740 \ud55c\uad6d\uc740 \uc774\ubbf8 \uc0c1\ub2f9\ud55c \ubc29\uc704\ube44\ub97c \ubd84\ub2f4\ud558\uace0 \uc788\ub2e4\ub294 \uc785\uc7a5\uc744 \ubc1d\ud614\ub2e4. \ud2f8\ub7ec\uc2a8 \uc7a5\uad00\uc740 \u201c\ud55c\uad6d\uacfc \uc77c\ubcf8\uc740 \uc774\ubbf8 \ubbf8\uad70\uc744 \uc9c0\uc6d0\ud558\ub294\ub370 \uc774\ubbf8 \ub9ce\uc740 \uc591\uc744 \uae30\uc5ec\ud558\uace0 \uc788\ub2e4\u201d\uba74\uc11c \u201c\ud5a5\ud6c4 \uad00\ub828 \ub300\ud654\uac00 \uc0dd\uc0b0\uc801\uc73c\ub85c \uc9c4\ud589\ub418\uace0, \uacf5\ud3c9\ud55c \ubd84\ub2f4\uae08 \ud569\uc758\uac00 \uc774\ub904\uc9c8 \uac83\uc73c\ub85c \ub099\uad00\ud55c\ub2e4\u201d\uace0 \ub9d0\ud588\ub2e4. \ub3c4\ub110\ub4dc \ud2b8\ub7fc\ud504 \ub300\ud1b5\ub839\uc774 \uc0ac\ub4dc \ubc30\uce58 \ube44\uc6a9 10\uc5b5 \ub2ec\ub7ec(\uc57d 1\uc8701\ucc9c\uc5b5\uc6d0)\ub97c \ud55c\uad6d\uc5d0\uac8c \ub0b4\ub77c\uace0 \uc694\uad6c\ud55c \uac83\uc5d0 \ub300\ud558\uc5ec \uae40\uad00\uc9c4 \uad6d\uac00\uc548\ubcf4\uc2e4\uc7a5\uc740 2017\ub144 4\uc6d4 30\uc77c \ud5c8\ubc84\ud2b8 \ub9e5\ub9c8\uc2a4\ud130 \ubbf8\uad6d \ubc31\uc545\uad00 \uad6d\uac00\uc548\ubcf4\ubcf4\uc88c\uad00\uacfc \uc804\ud654 \ud611\uc758\ub97c \uac16\uace0 \uc0ac\ub4dc \ubc30\uce58 \ube44\uc6a9\uc740 \ubbf8\uad6d\uc774 \uc804\uc561 \ubd80\ub2f4\ud55c\ub2e4\ub294 \uae30\uc874 \ud569\uc758 \ub0b4\uc6a9\uc744 \uc7ac\ud655\uc778\ud588\ub2e4."} {"question": "18\uc138\uae30\uc5d0 \ucd08\uc5d0 \uc8fc\ub85c \uac15\uc870\ub41c \ucca0\ud559\uc740?", "paragraph": "18\uc138\uae30\uc5d0\ub294 \uacc4\ubabd\uc8fc\uc758\uc758 \ucca0\ud559 \ubcc0\ud601\uc774 \uc720\ub7fd\uc5d0 \ud37c\uc9c0\uba74\uc11c \uadf8\ub9ac\uc2a4 \uc2e0\ud654\uc5d0 \ub300\ud55c \ubc18\ubc1c \uc791\uc6a9\uc774 \uc77c\uc5b4\ub098\uac8c \ub418\uc5c8\uace0, \uadf8\ub9ac\uc2a4\uc640 \ub85c\ub9c8\uc758 \uacfc\ud559\uacfc \ucca0\ud559\uc5d0 \ub300\ud55c \uc5c5\uc801\uc744 \uac15\uc870\ud558\ub294 \ud48d\uc870\uac00 \uac15\ud574\uc84c\ub2e4. \uadf8\ub7ec\ub098 \uc2e0\ud654\ub294 \uac01\ubcf8 \uc791\uac00\uc5d0\uac8c\ub294 \uac00\uacf5\ub418\uc9c0 \uc54a\uc740 \uc18c\uc7ac\uc758 \uc911\uc694\ud55c \ucd9c\ucc98\ub85c \uacc4\uc18d\ud574\uc11c \uc4f0\uc600\uc73c\uba70, \uc774\ub97c \ud1b5\ud574 \ud5e8\ub378\uc758 \uc624\ud398\ub77c \u300a\uc544\ub4dc\uba54\ud1a0\u300b, \u300a\uc138\uba5c\ub808\u300b, \ubaa8\ucc28\ub974\ud2b8\uc758 \u300a\uc774\ub3c4\uba54\ub124\uc624\u300b, \uae00\ub8e8\ud06c\uc758 \u300a\uc544\uc6b8\ub9ac\ub370\uc758 \uc774\ud53c\uac8c\ub124\uc774\uc544\u300b\uc758 \ub9ac\ube0c\ub808\ud1a0\uac00 \ub9cc\ub4e4\uc5b4\uc84c\ub2e4. \uc138\uae30 \ub9d0\uc5d0\ub294 \ub0ad\ub9cc\uc8fc\uc758\uac00 \ub300\ub450\ub418\uba74\uc11c \uc2e0\ud654\ub97c \ud3ec\ud568\ud55c \uadf8\ub9ac\uc2a4\uc758 \ubaa8\ub4e0 \uac83\uc5d0 \uc5f4\uc911\ud558\uae30 \uc2dc\uc791\ud588\ub2e4. \uc601\uad6d\uc5d0\uc11c\ub294 \uc774 \ub54c \uadf8\ub9ac\uc2a4 \ube44\uadf9\uacfc \ud638\uba54\ub85c\uc2a4\uc5d0 \ub300\ud55c \uc0c8\ub85c\uc6b4 \ubc88\uc5ed\uc774 \ub300\ub300\uc801\uc73c\ub85c \uc774\ub8e8\uc5b4\uc9c0\uba70 \ud0a4\uce20, \ubc14\uc774\ub7f0, \uc178\ub9ac\uc640 \uac19\uc740 \ub2f9\ub300\uc758 \uc2dc\uc778\ub4e4\uc5d0\uac8c \uc601\uac10\uc744 \uc8fc\uc5c8\ub2e4. \ube45\ud1a0\ub9ac\uc544 \uc5ec\uc655\uc758 \uacc4\uad00 \uc2dc\uc778 \uc54c\ud504\ub808\ub4dc \ud14c\ub2c8\uc2a8 \uacbd\uc740 \ud5ec\ub808\ub2c8\uc998 \uacbd\ud5a5\uc73c\ub85c \uc778\ud574 \uc544\uc11c \uc655\uc758 \uc601\uad6d \uc655\uc815\uc744 \ud638\uba54\ub85c\uc2a4 \uc11c\uc0ac\uc2dc \ud48d\uc73c\ub85c \ubb18\uc0ac\ud558\uae30\ub3c4 \ud588\ub2e4. 1816\ub144 \ud30c\ub974\ud14c\ub17c \ub300\ub9ac\uc11d \uc870\uac01\uad70 \uad6c\uc785\uc774 \uc790\uadf9\uc774 \ub418\uc5b4, \uc2dc\uac01 \uc608\uc220\uc5d0\uc11c\uc758 \uc601\ud5a5\ub3c4 \ubc1c\uc744 \ub9de\ucd94\uc5b4 \ub098\ud0c0\ub0ac\ub2e4. \ub808\uc774\ud134 \uacbd\uacfc \ub85c\ub80c\uc2a4 \uc568\uba38 \ud0dc\ub514\ub9c8\uc758 \ub9ce\uc740 \"\uadf8\ub9ac\uc2a4\" \ud68c\ud654\ub294 \uadf8\ub9ac\uc2a4 \uc774\uc0c1\uc758 \ub9e4\uac1c\ub85c\uc368 \uc9c4\uc815\uc73c\ub85c \uc778\uc815 \ubc1b\uc558\ub2e4. 18\uc138\uae30 \ub3c5\uc77c\uc758 \uc791\uace1\uac00 \ud06c\ub9ac\uc2a4\ud1a0\ud504 \uae00\ub8e8\ud06c \ub610\ud55c \uadf8\ub9ac\uc2a4 \uc2e0\ud654\uc758 \uc601\ud5a5\uc744 \ubc1b\uc558\ub2e4.", "answer": "\uacc4\ubabd\uc8fc\uc758\uc758 \ucca0\ud559", "sentence": "18\uc138\uae30\uc5d0\ub294 \uacc4\ubabd\uc8fc\uc758\uc758 \ucca0\ud559 \ubcc0\ud601\uc774 \uc720\ub7fd\uc5d0 \ud37c\uc9c0\uba74\uc11c \uadf8\ub9ac\uc2a4 \uc2e0\ud654\uc5d0 \ub300\ud55c \ubc18\ubc1c \uc791\uc6a9\uc774 \uc77c\uc5b4\ub098\uac8c \ub418\uc5c8\uace0, \uadf8\ub9ac\uc2a4\uc640 \ub85c\ub9c8\uc758 \uacfc\ud559\uacfc \ucca0\ud559\uc5d0 \ub300\ud55c \uc5c5\uc801\uc744 \uac15\uc870\ud558\ub294 \ud48d\uc870\uac00 \uac15\ud574\uc84c\ub2e4.", "paragraph_sentence": " 18\uc138\uae30\uc5d0\ub294 \uacc4\ubabd\uc8fc\uc758\uc758 \ucca0\ud559 \ubcc0\ud601\uc774 \uc720\ub7fd\uc5d0 \ud37c\uc9c0\uba74\uc11c \uadf8\ub9ac\uc2a4 \uc2e0\ud654\uc5d0 \ub300\ud55c \ubc18\ubc1c \uc791\uc6a9\uc774 \uc77c\uc5b4\ub098\uac8c \ub418\uc5c8\uace0, \uadf8\ub9ac\uc2a4\uc640 \ub85c\ub9c8\uc758 \uacfc\ud559\uacfc \ucca0\ud559\uc5d0 \ub300\ud55c \uc5c5\uc801\uc744 \uac15\uc870\ud558\ub294 \ud48d\uc870\uac00 \uac15\ud574\uc84c\ub2e4. \uadf8\ub7ec\ub098 \uc2e0\ud654\ub294 \uac01\ubcf8 \uc791\uac00\uc5d0\uac8c\ub294 \uac00\uacf5\ub418\uc9c0 \uc54a\uc740 \uc18c\uc7ac\uc758 \uc911\uc694\ud55c \ucd9c\ucc98\ub85c \uacc4\uc18d\ud574\uc11c \uc4f0\uc600\uc73c\uba70, \uc774\ub97c \ud1b5\ud574 \ud5e8\ub378\uc758 \uc624\ud398\ub77c \u300a\uc544\ub4dc\uba54\ud1a0\u300b, \u300a\uc138\uba5c\ub808\u300b, \ubaa8\ucc28\ub974\ud2b8\uc758 \u300a\uc774\ub3c4\uba54\ub124\uc624\u300b, \uae00\ub8e8\ud06c\uc758 \u300a\uc544\uc6b8\ub9ac\ub370\uc758 \uc774\ud53c\uac8c\ub124\uc774\uc544\u300b\uc758 \ub9ac\ube0c\ub808\ud1a0\uac00 \ub9cc\ub4e4\uc5b4\uc84c\ub2e4. \uc138\uae30 \ub9d0\uc5d0\ub294 \ub0ad\ub9cc\uc8fc\uc758\uac00 \ub300\ub450\ub418\uba74\uc11c \uc2e0\ud654\ub97c \ud3ec\ud568\ud55c \uadf8\ub9ac\uc2a4\uc758 \ubaa8\ub4e0 \uac83\uc5d0 \uc5f4\uc911\ud558\uae30 \uc2dc\uc791\ud588\ub2e4. \uc601\uad6d\uc5d0\uc11c\ub294 \uc774 \ub54c \uadf8\ub9ac\uc2a4 \ube44\uadf9\uacfc \ud638\uba54\ub85c\uc2a4\uc5d0 \ub300\ud55c \uc0c8\ub85c\uc6b4 \ubc88\uc5ed\uc774 \ub300\ub300\uc801\uc73c\ub85c \uc774\ub8e8\uc5b4\uc9c0\uba70 \ud0a4\uce20, \ubc14\uc774\ub7f0, \uc178\ub9ac\uc640 \uac19\uc740 \ub2f9\ub300\uc758 \uc2dc\uc778\ub4e4\uc5d0\uac8c \uc601\uac10\uc744 \uc8fc\uc5c8\ub2e4. \ube45\ud1a0\ub9ac\uc544 \uc5ec\uc655\uc758 \uacc4\uad00 \uc2dc\uc778 \uc54c\ud504\ub808\ub4dc \ud14c\ub2c8\uc2a8 \uacbd\uc740 \ud5ec\ub808\ub2c8\uc998 \uacbd\ud5a5\uc73c\ub85c \uc778\ud574 \uc544\uc11c \uc655\uc758 \uc601\uad6d \uc655\uc815\uc744 \ud638\uba54\ub85c\uc2a4 \uc11c\uc0ac\uc2dc \ud48d\uc73c\ub85c \ubb18\uc0ac\ud558\uae30\ub3c4 \ud588\ub2e4. 1816\ub144 \ud30c\ub974\ud14c\ub17c \ub300\ub9ac\uc11d \uc870\uac01\uad70 \uad6c\uc785\uc774 \uc790\uadf9\uc774 \ub418\uc5b4, \uc2dc\uac01 \uc608\uc220\uc5d0\uc11c\uc758 \uc601\ud5a5\ub3c4 \ubc1c\uc744 \ub9de\ucd94\uc5b4 \ub098\ud0c0\ub0ac\ub2e4. \ub808\uc774\ud134 \uacbd\uacfc \ub85c\ub80c\uc2a4 \uc568\uba38 \ud0dc\ub514\ub9c8\uc758 \ub9ce\uc740 \"\uadf8\ub9ac\uc2a4\" \ud68c\ud654\ub294 \uadf8\ub9ac\uc2a4 \uc774\uc0c1\uc758 \ub9e4\uac1c\ub85c\uc368 \uc9c4\uc815\uc73c\ub85c \uc778\uc815 \ubc1b\uc558\ub2e4. 18\uc138\uae30 \ub3c5\uc77c\uc758 \uc791\uace1\uac00 \ud06c\ub9ac\uc2a4\ud1a0\ud504 \uae00\ub8e8\ud06c \ub610\ud55c \uadf8\ub9ac\uc2a4 \uc2e0\ud654\uc758 \uc601\ud5a5\uc744 \ubc1b\uc558\ub2e4.", "paragraph_answer": "18\uc138\uae30\uc5d0\ub294 \uacc4\ubabd\uc8fc\uc758\uc758 \ucca0\ud559 \ubcc0\ud601\uc774 \uc720\ub7fd\uc5d0 \ud37c\uc9c0\uba74\uc11c \uadf8\ub9ac\uc2a4 \uc2e0\ud654\uc5d0 \ub300\ud55c \ubc18\ubc1c \uc791\uc6a9\uc774 \uc77c\uc5b4\ub098\uac8c \ub418\uc5c8\uace0, \uadf8\ub9ac\uc2a4\uc640 \ub85c\ub9c8\uc758 \uacfc\ud559\uacfc \ucca0\ud559\uc5d0 \ub300\ud55c \uc5c5\uc801\uc744 \uac15\uc870\ud558\ub294 \ud48d\uc870\uac00 \uac15\ud574\uc84c\ub2e4. \uadf8\ub7ec\ub098 \uc2e0\ud654\ub294 \uac01\ubcf8 \uc791\uac00\uc5d0\uac8c\ub294 \uac00\uacf5\ub418\uc9c0 \uc54a\uc740 \uc18c\uc7ac\uc758 \uc911\uc694\ud55c \ucd9c\ucc98\ub85c \uacc4\uc18d\ud574\uc11c \uc4f0\uc600\uc73c\uba70, \uc774\ub97c \ud1b5\ud574 \ud5e8\ub378\uc758 \uc624\ud398\ub77c \u300a\uc544\ub4dc\uba54\ud1a0\u300b, \u300a\uc138\uba5c\ub808\u300b, \ubaa8\ucc28\ub974\ud2b8\uc758 \u300a\uc774\ub3c4\uba54\ub124\uc624\u300b, \uae00\ub8e8\ud06c\uc758 \u300a\uc544\uc6b8\ub9ac\ub370\uc758 \uc774\ud53c\uac8c\ub124\uc774\uc544\u300b\uc758 \ub9ac\ube0c\ub808\ud1a0\uac00 \ub9cc\ub4e4\uc5b4\uc84c\ub2e4. \uc138\uae30 \ub9d0\uc5d0\ub294 \ub0ad\ub9cc\uc8fc\uc758\uac00 \ub300\ub450\ub418\uba74\uc11c \uc2e0\ud654\ub97c \ud3ec\ud568\ud55c \uadf8\ub9ac\uc2a4\uc758 \ubaa8\ub4e0 \uac83\uc5d0 \uc5f4\uc911\ud558\uae30 \uc2dc\uc791\ud588\ub2e4. \uc601\uad6d\uc5d0\uc11c\ub294 \uc774 \ub54c \uadf8\ub9ac\uc2a4 \ube44\uadf9\uacfc \ud638\uba54\ub85c\uc2a4\uc5d0 \ub300\ud55c \uc0c8\ub85c\uc6b4 \ubc88\uc5ed\uc774 \ub300\ub300\uc801\uc73c\ub85c \uc774\ub8e8\uc5b4\uc9c0\uba70 \ud0a4\uce20, \ubc14\uc774\ub7f0, \uc178\ub9ac\uc640 \uac19\uc740 \ub2f9\ub300\uc758 \uc2dc\uc778\ub4e4\uc5d0\uac8c \uc601\uac10\uc744 \uc8fc\uc5c8\ub2e4. \ube45\ud1a0\ub9ac\uc544 \uc5ec\uc655\uc758 \uacc4\uad00 \uc2dc\uc778 \uc54c\ud504\ub808\ub4dc \ud14c\ub2c8\uc2a8 \uacbd\uc740 \ud5ec\ub808\ub2c8\uc998 \uacbd\ud5a5\uc73c\ub85c \uc778\ud574 \uc544\uc11c \uc655\uc758 \uc601\uad6d \uc655\uc815\uc744 \ud638\uba54\ub85c\uc2a4 \uc11c\uc0ac\uc2dc \ud48d\uc73c\ub85c \ubb18\uc0ac\ud558\uae30\ub3c4 \ud588\ub2e4. 1816\ub144 \ud30c\ub974\ud14c\ub17c \ub300\ub9ac\uc11d \uc870\uac01\uad70 \uad6c\uc785\uc774 \uc790\uadf9\uc774 \ub418\uc5b4, \uc2dc\uac01 \uc608\uc220\uc5d0\uc11c\uc758 \uc601\ud5a5\ub3c4 \ubc1c\uc744 \ub9de\ucd94\uc5b4 \ub098\ud0c0\ub0ac\ub2e4. \ub808\uc774\ud134 \uacbd\uacfc \ub85c\ub80c\uc2a4 \uc568\uba38 \ud0dc\ub514\ub9c8\uc758 \ub9ce\uc740 \"\uadf8\ub9ac\uc2a4\" \ud68c\ud654\ub294 \uadf8\ub9ac\uc2a4 \uc774\uc0c1\uc758 \ub9e4\uac1c\ub85c\uc368 \uc9c4\uc815\uc73c\ub85c \uc778\uc815 \ubc1b\uc558\ub2e4. 18\uc138\uae30 \ub3c5\uc77c\uc758 \uc791\uace1\uac00 \ud06c\ub9ac\uc2a4\ud1a0\ud504 \uae00\ub8e8\ud06c \ub610\ud55c \uadf8\ub9ac\uc2a4 \uc2e0\ud654\uc758 \uc601\ud5a5\uc744 \ubc1b\uc558\ub2e4.", "sentence_answer": "18\uc138\uae30\uc5d0\ub294 \uacc4\ubabd\uc8fc\uc758\uc758 \ucca0\ud559 \ubcc0\ud601\uc774 \uc720\ub7fd\uc5d0 \ud37c\uc9c0\uba74\uc11c \uadf8\ub9ac\uc2a4 \uc2e0\ud654\uc5d0 \ub300\ud55c \ubc18\ubc1c \uc791\uc6a9\uc774 \uc77c\uc5b4\ub098\uac8c \ub418\uc5c8\uace0, \uadf8\ub9ac\uc2a4\uc640 \ub85c\ub9c8\uc758 \uacfc\ud559\uacfc \ucca0\ud559\uc5d0 \ub300\ud55c \uc5c5\uc801\uc744 \uac15\uc870\ud558\ub294 \ud48d\uc870\uac00 \uac15\ud574\uc84c\ub2e4.", "paragraph_id": "5975097-1-0", "paragraph_question": "question: 18\uc138\uae30\uc5d0 \ucd08\uc5d0 \uc8fc\ub85c \uac15\uc870\ub41c \ucca0\ud559\uc740?, context: 18\uc138\uae30\uc5d0\ub294 \uacc4\ubabd\uc8fc\uc758\uc758 \ucca0\ud559 \ubcc0\ud601\uc774 \uc720\ub7fd\uc5d0 \ud37c\uc9c0\uba74\uc11c \uadf8\ub9ac\uc2a4 \uc2e0\ud654\uc5d0 \ub300\ud55c \ubc18\ubc1c \uc791\uc6a9\uc774 \uc77c\uc5b4\ub098\uac8c \ub418\uc5c8\uace0, \uadf8\ub9ac\uc2a4\uc640 \ub85c\ub9c8\uc758 \uacfc\ud559\uacfc \ucca0\ud559\uc5d0 \ub300\ud55c \uc5c5\uc801\uc744 \uac15\uc870\ud558\ub294 \ud48d\uc870\uac00 \uac15\ud574\uc84c\ub2e4. \uadf8\ub7ec\ub098 \uc2e0\ud654\ub294 \uac01\ubcf8 \uc791\uac00\uc5d0\uac8c\ub294 \uac00\uacf5\ub418\uc9c0 \uc54a\uc740 \uc18c\uc7ac\uc758 \uc911\uc694\ud55c \ucd9c\ucc98\ub85c \uacc4\uc18d\ud574\uc11c \uc4f0\uc600\uc73c\uba70, \uc774\ub97c \ud1b5\ud574 \ud5e8\ub378\uc758 \uc624\ud398\ub77c \u300a\uc544\ub4dc\uba54\ud1a0\u300b, \u300a\uc138\uba5c\ub808\u300b, \ubaa8\ucc28\ub974\ud2b8\uc758 \u300a\uc774\ub3c4\uba54\ub124\uc624\u300b, \uae00\ub8e8\ud06c\uc758 \u300a\uc544\uc6b8\ub9ac\ub370\uc758 \uc774\ud53c\uac8c\ub124\uc774\uc544\u300b\uc758 \ub9ac\ube0c\ub808\ud1a0\uac00 \ub9cc\ub4e4\uc5b4\uc84c\ub2e4. \uc138\uae30 \ub9d0\uc5d0\ub294 \ub0ad\ub9cc\uc8fc\uc758\uac00 \ub300\ub450\ub418\uba74\uc11c \uc2e0\ud654\ub97c \ud3ec\ud568\ud55c \uadf8\ub9ac\uc2a4\uc758 \ubaa8\ub4e0 \uac83\uc5d0 \uc5f4\uc911\ud558\uae30 \uc2dc\uc791\ud588\ub2e4. \uc601\uad6d\uc5d0\uc11c\ub294 \uc774 \ub54c \uadf8\ub9ac\uc2a4 \ube44\uadf9\uacfc \ud638\uba54\ub85c\uc2a4\uc5d0 \ub300\ud55c \uc0c8\ub85c\uc6b4 \ubc88\uc5ed\uc774 \ub300\ub300\uc801\uc73c\ub85c \uc774\ub8e8\uc5b4\uc9c0\uba70 \ud0a4\uce20, \ubc14\uc774\ub7f0, \uc178\ub9ac\uc640 \uac19\uc740 \ub2f9\ub300\uc758 \uc2dc\uc778\ub4e4\uc5d0\uac8c \uc601\uac10\uc744 \uc8fc\uc5c8\ub2e4. \ube45\ud1a0\ub9ac\uc544 \uc5ec\uc655\uc758 \uacc4\uad00 \uc2dc\uc778 \uc54c\ud504\ub808\ub4dc \ud14c\ub2c8\uc2a8 \uacbd\uc740 \ud5ec\ub808\ub2c8\uc998 \uacbd\ud5a5\uc73c\ub85c \uc778\ud574 \uc544\uc11c \uc655\uc758 \uc601\uad6d \uc655\uc815\uc744 \ud638\uba54\ub85c\uc2a4 \uc11c\uc0ac\uc2dc \ud48d\uc73c\ub85c \ubb18\uc0ac\ud558\uae30\ub3c4 \ud588\ub2e4. 1816\ub144 \ud30c\ub974\ud14c\ub17c \ub300\ub9ac\uc11d \uc870\uac01\uad70 \uad6c\uc785\uc774 \uc790\uadf9\uc774 \ub418\uc5b4, \uc2dc\uac01 \uc608\uc220\uc5d0\uc11c\uc758 \uc601\ud5a5\ub3c4 \ubc1c\uc744 \ub9de\ucd94\uc5b4 \ub098\ud0c0\ub0ac\ub2e4. \ub808\uc774\ud134 \uacbd\uacfc \ub85c\ub80c\uc2a4 \uc568\uba38 \ud0dc\ub514\ub9c8\uc758 \ub9ce\uc740 \"\uadf8\ub9ac\uc2a4\" \ud68c\ud654\ub294 \uadf8\ub9ac\uc2a4 \uc774\uc0c1\uc758 \ub9e4\uac1c\ub85c\uc368 \uc9c4\uc815\uc73c\ub85c \uc778\uc815 \ubc1b\uc558\ub2e4. 18\uc138\uae30 \ub3c5\uc77c\uc758 \uc791\uace1\uac00 \ud06c\ub9ac\uc2a4\ud1a0\ud504 \uae00\ub8e8\ud06c \ub610\ud55c \uadf8\ub9ac\uc2a4 \uc2e0\ud654\uc758 \uc601\ud5a5\uc744 \ubc1b\uc558\ub2e4."} {"question": "\ubc15\uc720\ucc9c\uc758 2\ucc28 \ud5c8\uc704 \uace0\uc18c \uc5ec\uc131\uc774 \uac70\uc9d3\uc73c\ub85c \ub9d0\ud588\uc74c\uc774 \ub4dc\ub7ec\ub09c \uc9c1\uc5c5\uc740?", "paragraph": "\uadf8\ub798\uc11c \uac80\uc0ac\uac00 \"\ud53c\ud574\uc790\ub77c \uc8fc\uc7a5\ud558\ub294 \ucc9c\uba85 \uc774\uc0c1\uc758 \uc5ec\uc798 \uc218\uc0ac\ud588\uc9c0\ub9cc 2\ucc28\ud5c8\uc704\uace0\uc18c\ub140\ucc98\ub7fc \uc99d\uac70\ub294 \ud558\ub098\ub3c4 \ubaa8\uc73c\uc9c0 \ubabb\ud558\uace0 \ube44\uc0c1\uc2dd\uc801\uc778 \uc8fc\uc7a5\ub9cc \ud558\ub294 \uc5ec\uc794 \ucc98\uc74c\" \uc774\ub77c\uba70 \ub9e4\uc6b0 \ud669\ub2f9\ud574\ud588\ub2e4.2\ucc28\ub140\ub294 \uc5b8\ub860\ud50c\ub808\uc774\ub85c \ubc15\uc720\ucc9c \uac00\ud574\uc790 \ubab0\uc774 \uacf5\uaca9+\uae30\uc790\ud68c\uacac(\ubcd1\ud48d\ub4a4\uc5d0 \uc228\uc5b4 \ud3c9\ubc94\ud55c \uc5ec\ub300\uc0dd\uc774\ub77c\uba70 \uac10\uc131\ud314\uc774 \uc5b8\ud50c\ub4f1\uc744\ud588\uc73c\ub098 \uc2e4\uc81c\ub85c \ub2f9\uc2dc 2\uc0b4\ubc30\uae30 \uc544\uae30\ub454 \uc544\uc90c\ub9c8\ub85c \ub4dc\ub7ec\ub0ac\ub2e4)+\uc7ac\uc815\uc2e0\uccad \uac81\ubc15(\uc7ac\uc815\uc2e0\uccad\ud560\ud14c\ub2c8 \ubb34\uc8c4\ub2ec\ub77c \uac81\ubc15\ud588\uc73c\ub098 \uacb0\uad6d \ud53c\ud574\uc790\ub780 \uc99d\uac70\uc5c6\ub2e4\uace0 \uc7ac\uc815\uc2e0\uccad \uae30\uac01\ub2f9\ud588\ub2e4)+\uc5ec\uc131\ub2e8\uccb4 \uc774\uc6a9\ud55c \ud611\ubc15(\ubb34\uc8c4 \uc548\uc8fc\uae30\ub9cc\ud574\ubd10 \uc5ec\uc131\ub2e8\uccb4 \ub2e4 \ub4e4\uace0\uc77c\uc5b4\ub0a0\uac70\uc57c!)+\uc7ac\ud310\uc7a5\uc5d0 \uc5ec\uc131\ub2e8\uccb4 \ubab0\uace0\uac00 \uac80\uc0ac \uacf5\uaca9\ud558\uac8c \ub9cc\ub4dc\ub294\ub4f1 \uc628\uac16 \uc218\ub2e8\uacfc \ubc29\ubc95\uc744 \uac00\ub9ac\uc9c0\uc54a\uc558\ub2e4. \uc2e4\uc81c\ub85c 2\ucc28\uace0\uc18c\ub140\ub294 \uc5ec\uc131\ub2e8\uccb4\ub85c\ubd80\ud130 \uac01\uc885 \uc9c0\uc6d0\uc744 \uc544\ub08c\uc5c6\uc774 \ubc1b\uc740 \uac83\uc73c\ub85c \ub4dc\ub7ec\ub0ac\ub2e4, 1\ucc28\ub140\uac00 \uac10\uc625\uc5d0 \uac00\uc790 \uc5ec\uc131\ub2e8\uccb4\ub294 \ub208\uc774 \ub4a4\uc9d1\ud600 \ud53c\ucf13\ub4e4\uace0 \ubc95\uc6d0 \uad11\ud654\ubb38 \ud55c\ubcf5\ud310\ub4f1\uc11c \uc2dc\uc704 2\ucc28\ub140 \uc7ac\ud310\uc7a5\uc5d0 \ubab0\ub824\uac00 \uc18c\ub780\ud53c\uc6b0\uba70 \ud6fc\ubc29\ub193\uae30\ub4f1\uc73c\ub85c 2\ucc28\ub140 \uc7ac\ud310\uc5d0 \uc544\uc8fc \uae4a\uc219\ud788 \uac1c\uc785\ud55c \uac83\uc73c\ub85c \ub4dc\ub7ec\ub0ac\ub2e4, \uc774\ubbf8 \ubc15\uc720\ucc9c \uace0\uc18c4\uac74\uc740 \ubaa8\ub450 \uac70\uc9d3\uc73c\ub85c \ub4dc\ub7ec\ub0ac\uace0 1\ucc282\ucc28\ub140\uac00 \ud310\uc0ac \uac80\uc0ac \uc9d5\uc5ed\ud615\uae4c\uc9c0 \ub098\uc654\uc74c\uc5d0\ub3c4 \uc774\uac19\uc740 \ud53c\ud574\uc790 2\ucc28\uac00\ud574\ub85c \uc2e4\uc81c\ub85c \ubc15\uc720\ucc9c\uc740 \ub450\ubc88 \uc8fd\uc73c\ub824\ud588\uc5c8\ub358 \uac83\uc73c\ub85c \ub4dc\ub7ec\ub0ac\ub2e4.", "answer": "\uc5ec\ub300\uc0dd", "sentence": "\uadf8\ub798\uc11c \uac80\uc0ac\uac00 \"\ud53c\ud574\uc790\ub77c \uc8fc\uc7a5\ud558\ub294 \ucc9c\uba85 \uc774\uc0c1\uc758 \uc5ec\uc798 \uc218\uc0ac\ud588\uc9c0\ub9cc 2\ucc28\ud5c8\uc704\uace0\uc18c\ub140\ucc98\ub7fc \uc99d\uac70\ub294 \ud558\ub098\ub3c4 \ubaa8\uc73c\uc9c0 \ubabb\ud558\uace0 \ube44\uc0c1\uc2dd\uc801\uc778 \uc8fc\uc7a5\ub9cc \ud558\ub294 \uc5ec\uc794 \ucc98\uc74c\" \uc774\ub77c\uba70 \ub9e4\uc6b0 \ud669\ub2f9\ud574\ud588\ub2e4.2\ucc28\ub140\ub294 \uc5b8\ub860\ud50c\ub808\uc774\ub85c \ubc15\uc720\ucc9c \uac00\ud574\uc790 \ubab0\uc774 \uacf5\uaca9+\uae30\uc790\ud68c\uacac(\ubcd1\ud48d\ub4a4\uc5d0 \uc228\uc5b4 \ud3c9\ubc94\ud55c \uc5ec\ub300\uc0dd \uc774\ub77c\uba70 \uac10\uc131\ud314\uc774 \uc5b8\ud50c\ub4f1\uc744\ud588\uc73c\ub098 \uc2e4\uc81c\ub85c \ub2f9\uc2dc 2\uc0b4\ubc30\uae30 \uc544\uae30\ub454 \uc544\uc90c\ub9c8\ub85c \ub4dc\ub7ec\ub0ac\ub2e4)+\uc7ac\uc815\uc2e0\uccad \uac81\ubc15(\uc7ac\uc815\uc2e0\uccad\ud560\ud14c\ub2c8 \ubb34\uc8c4\ub2ec\ub77c \uac81\ubc15\ud588\uc73c\ub098 \uacb0\uad6d \ud53c\ud574\uc790\ub780 \uc99d\uac70\uc5c6\ub2e4\uace0 \uc7ac\uc815\uc2e0\uccad \uae30\uac01\ub2f9\ud588\ub2e4)+\uc5ec\uc131\ub2e8\uccb4 \uc774\uc6a9\ud55c \ud611\ubc15(\ubb34\uc8c4 \uc548\uc8fc\uae30\ub9cc\ud574\ubd10 \uc5ec\uc131\ub2e8\uccb4 \ub2e4 \ub4e4\uace0\uc77c\uc5b4\ub0a0\uac70\uc57c!)+\uc7ac\ud310\uc7a5\uc5d0 \uc5ec\uc131\ub2e8\uccb4 \ubab0\uace0\uac00 \uac80\uc0ac \uacf5\uaca9\ud558\uac8c \ub9cc\ub4dc\ub294\ub4f1 \uc628\uac16 \uc218\ub2e8\uacfc \ubc29\ubc95\uc744 \uac00\ub9ac\uc9c0\uc54a\uc558\ub2e4.", "paragraph_sentence": " \uadf8\ub798\uc11c \uac80\uc0ac\uac00 \"\ud53c\ud574\uc790\ub77c \uc8fc\uc7a5\ud558\ub294 \ucc9c\uba85 \uc774\uc0c1\uc758 \uc5ec\uc798 \uc218\uc0ac\ud588\uc9c0\ub9cc 2\ucc28\ud5c8\uc704\uace0\uc18c\ub140\ucc98\ub7fc \uc99d\uac70\ub294 \ud558\ub098\ub3c4 \ubaa8\uc73c\uc9c0 \ubabb\ud558\uace0 \ube44\uc0c1\uc2dd\uc801\uc778 \uc8fc\uc7a5\ub9cc \ud558\ub294 \uc5ec\uc794 \ucc98\uc74c\" \uc774\ub77c\uba70 \ub9e4\uc6b0 \ud669\ub2f9\ud574\ud588\ub2e4.2\ucc28\ub140\ub294 \uc5b8\ub860\ud50c\ub808\uc774\ub85c \ubc15\uc720\ucc9c \uac00\ud574\uc790 \ubab0\uc774 \uacf5\uaca9+\uae30\uc790\ud68c\uacac(\ubcd1\ud48d\ub4a4\uc5d0 \uc228\uc5b4 \ud3c9\ubc94\ud55c \uc5ec\ub300\uc0dd \uc774\ub77c\uba70 \uac10\uc131\ud314\uc774 \uc5b8\ud50c\ub4f1\uc744\ud588\uc73c\ub098 \uc2e4\uc81c\ub85c \ub2f9\uc2dc 2\uc0b4\ubc30\uae30 \uc544\uae30\ub454 \uc544\uc90c\ub9c8\ub85c \ub4dc\ub7ec\ub0ac\ub2e4)+\uc7ac\uc815\uc2e0\uccad \uac81\ubc15(\uc7ac\uc815\uc2e0\uccad\ud560\ud14c\ub2c8 \ubb34\uc8c4\ub2ec\ub77c \uac81\ubc15\ud588\uc73c\ub098 \uacb0\uad6d \ud53c\ud574\uc790\ub780 \uc99d\uac70\uc5c6\ub2e4\uace0 \uc7ac\uc815\uc2e0\uccad \uae30\uac01\ub2f9\ud588\ub2e4)+\uc5ec\uc131\ub2e8\uccb4 \uc774\uc6a9\ud55c \ud611\ubc15(\ubb34\uc8c4 \uc548\uc8fc\uae30\ub9cc\ud574\ubd10 \uc5ec\uc131\ub2e8\uccb4 \ub2e4 \ub4e4\uace0\uc77c\uc5b4\ub0a0\uac70\uc57c!)+\uc7ac\ud310\uc7a5\uc5d0 \uc5ec\uc131\ub2e8\uccb4 \ubab0\uace0\uac00 \uac80\uc0ac \uacf5\uaca9\ud558\uac8c \ub9cc\ub4dc\ub294\ub4f1 \uc628\uac16 \uc218\ub2e8\uacfc \ubc29\ubc95\uc744 \uac00\ub9ac\uc9c0\uc54a\uc558\ub2e4. \uc2e4\uc81c\ub85c 2\ucc28\uace0\uc18c\ub140\ub294 \uc5ec\uc131\ub2e8\uccb4\ub85c\ubd80\ud130 \uac01\uc885 \uc9c0\uc6d0\uc744 \uc544\ub08c\uc5c6\uc774 \ubc1b\uc740 \uac83\uc73c\ub85c \ub4dc\ub7ec\ub0ac\ub2e4, 1\ucc28\ub140\uac00 \uac10\uc625\uc5d0 \uac00\uc790 \uc5ec\uc131\ub2e8\uccb4\ub294 \ub208\uc774 \ub4a4\uc9d1\ud600 \ud53c\ucf13\ub4e4\uace0 \ubc95\uc6d0 \uad11\ud654\ubb38 \ud55c\ubcf5\ud310\ub4f1\uc11c \uc2dc\uc704 2\ucc28\ub140 \uc7ac\ud310\uc7a5\uc5d0 \ubab0\ub824\uac00 \uc18c\ub780\ud53c\uc6b0\uba70 \ud6fc\ubc29\ub193\uae30\ub4f1\uc73c\ub85c 2\ucc28\ub140 \uc7ac\ud310\uc5d0 \uc544\uc8fc \uae4a\uc219\ud788 \uac1c\uc785\ud55c \uac83\uc73c\ub85c \ub4dc\ub7ec\ub0ac\ub2e4, \uc774\ubbf8 \ubc15\uc720\ucc9c \uace0\uc18c4\uac74\uc740 \ubaa8\ub450 \uac70\uc9d3\uc73c\ub85c \ub4dc\ub7ec\ub0ac\uace0 1\ucc282\ucc28\ub140\uac00 \ud310\uc0ac \uac80\uc0ac \uc9d5\uc5ed\ud615\uae4c\uc9c0 \ub098\uc654\uc74c\uc5d0\ub3c4 \uc774\uac19\uc740 \ud53c\ud574\uc790 2\ucc28\uac00\ud574\ub85c \uc2e4\uc81c\ub85c \ubc15\uc720\ucc9c\uc740 \ub450\ubc88 \uc8fd\uc73c\ub824\ud588\uc5c8\ub358 \uac83\uc73c\ub85c \ub4dc\ub7ec\ub0ac\ub2e4.", "paragraph_answer": "\uadf8\ub798\uc11c \uac80\uc0ac\uac00 \"\ud53c\ud574\uc790\ub77c \uc8fc\uc7a5\ud558\ub294 \ucc9c\uba85 \uc774\uc0c1\uc758 \uc5ec\uc798 \uc218\uc0ac\ud588\uc9c0\ub9cc 2\ucc28\ud5c8\uc704\uace0\uc18c\ub140\ucc98\ub7fc \uc99d\uac70\ub294 \ud558\ub098\ub3c4 \ubaa8\uc73c\uc9c0 \ubabb\ud558\uace0 \ube44\uc0c1\uc2dd\uc801\uc778 \uc8fc\uc7a5\ub9cc \ud558\ub294 \uc5ec\uc794 \ucc98\uc74c\" \uc774\ub77c\uba70 \ub9e4\uc6b0 \ud669\ub2f9\ud574\ud588\ub2e4.2\ucc28\ub140\ub294 \uc5b8\ub860\ud50c\ub808\uc774\ub85c \ubc15\uc720\ucc9c \uac00\ud574\uc790 \ubab0\uc774 \uacf5\uaca9+\uae30\uc790\ud68c\uacac(\ubcd1\ud48d\ub4a4\uc5d0 \uc228\uc5b4 \ud3c9\ubc94\ud55c \uc5ec\ub300\uc0dd \uc774\ub77c\uba70 \uac10\uc131\ud314\uc774 \uc5b8\ud50c\ub4f1\uc744\ud588\uc73c\ub098 \uc2e4\uc81c\ub85c \ub2f9\uc2dc 2\uc0b4\ubc30\uae30 \uc544\uae30\ub454 \uc544\uc90c\ub9c8\ub85c \ub4dc\ub7ec\ub0ac\ub2e4)+\uc7ac\uc815\uc2e0\uccad \uac81\ubc15(\uc7ac\uc815\uc2e0\uccad\ud560\ud14c\ub2c8 \ubb34\uc8c4\ub2ec\ub77c \uac81\ubc15\ud588\uc73c\ub098 \uacb0\uad6d \ud53c\ud574\uc790\ub780 \uc99d\uac70\uc5c6\ub2e4\uace0 \uc7ac\uc815\uc2e0\uccad \uae30\uac01\ub2f9\ud588\ub2e4)+\uc5ec\uc131\ub2e8\uccb4 \uc774\uc6a9\ud55c \ud611\ubc15(\ubb34\uc8c4 \uc548\uc8fc\uae30\ub9cc\ud574\ubd10 \uc5ec\uc131\ub2e8\uccb4 \ub2e4 \ub4e4\uace0\uc77c\uc5b4\ub0a0\uac70\uc57c!)+\uc7ac\ud310\uc7a5\uc5d0 \uc5ec\uc131\ub2e8\uccb4 \ubab0\uace0\uac00 \uac80\uc0ac \uacf5\uaca9\ud558\uac8c \ub9cc\ub4dc\ub294\ub4f1 \uc628\uac16 \uc218\ub2e8\uacfc \ubc29\ubc95\uc744 \uac00\ub9ac\uc9c0\uc54a\uc558\ub2e4. \uc2e4\uc81c\ub85c 2\ucc28\uace0\uc18c\ub140\ub294 \uc5ec\uc131\ub2e8\uccb4\ub85c\ubd80\ud130 \uac01\uc885 \uc9c0\uc6d0\uc744 \uc544\ub08c\uc5c6\uc774 \ubc1b\uc740 \uac83\uc73c\ub85c \ub4dc\ub7ec\ub0ac\ub2e4, 1\ucc28\ub140\uac00 \uac10\uc625\uc5d0 \uac00\uc790 \uc5ec\uc131\ub2e8\uccb4\ub294 \ub208\uc774 \ub4a4\uc9d1\ud600 \ud53c\ucf13\ub4e4\uace0 \ubc95\uc6d0 \uad11\ud654\ubb38 \ud55c\ubcf5\ud310\ub4f1\uc11c \uc2dc\uc704 2\ucc28\ub140 \uc7ac\ud310\uc7a5\uc5d0 \ubab0\ub824\uac00 \uc18c\ub780\ud53c\uc6b0\uba70 \ud6fc\ubc29\ub193\uae30\ub4f1\uc73c\ub85c 2\ucc28\ub140 \uc7ac\ud310\uc5d0 \uc544\uc8fc \uae4a\uc219\ud788 \uac1c\uc785\ud55c \uac83\uc73c\ub85c \ub4dc\ub7ec\ub0ac\ub2e4, \uc774\ubbf8 \ubc15\uc720\ucc9c \uace0\uc18c4\uac74\uc740 \ubaa8\ub450 \uac70\uc9d3\uc73c\ub85c \ub4dc\ub7ec\ub0ac\uace0 1\ucc282\ucc28\ub140\uac00 \ud310\uc0ac \uac80\uc0ac \uc9d5\uc5ed\ud615\uae4c\uc9c0 \ub098\uc654\uc74c\uc5d0\ub3c4 \uc774\uac19\uc740 \ud53c\ud574\uc790 2\ucc28\uac00\ud574\ub85c \uc2e4\uc81c\ub85c \ubc15\uc720\ucc9c\uc740 \ub450\ubc88 \uc8fd\uc73c\ub824\ud588\uc5c8\ub358 \uac83\uc73c\ub85c \ub4dc\ub7ec\ub0ac\ub2e4.", "sentence_answer": "\uadf8\ub798\uc11c \uac80\uc0ac\uac00 \"\ud53c\ud574\uc790\ub77c \uc8fc\uc7a5\ud558\ub294 \ucc9c\uba85 \uc774\uc0c1\uc758 \uc5ec\uc798 \uc218\uc0ac\ud588\uc9c0\ub9cc 2\ucc28\ud5c8\uc704\uace0\uc18c\ub140\ucc98\ub7fc \uc99d\uac70\ub294 \ud558\ub098\ub3c4 \ubaa8\uc73c\uc9c0 \ubabb\ud558\uace0 \ube44\uc0c1\uc2dd\uc801\uc778 \uc8fc\uc7a5\ub9cc \ud558\ub294 \uc5ec\uc794 \ucc98\uc74c\" \uc774\ub77c\uba70 \ub9e4\uc6b0 \ud669\ub2f9\ud574\ud588\ub2e4.2\ucc28\ub140\ub294 \uc5b8\ub860\ud50c\ub808\uc774\ub85c \ubc15\uc720\ucc9c \uac00\ud574\uc790 \ubab0\uc774 \uacf5\uaca9+\uae30\uc790\ud68c\uacac(\ubcd1\ud48d\ub4a4\uc5d0 \uc228\uc5b4 \ud3c9\ubc94\ud55c \uc5ec\ub300\uc0dd \uc774\ub77c\uba70 \uac10\uc131\ud314\uc774 \uc5b8\ud50c\ub4f1\uc744\ud588\uc73c\ub098 \uc2e4\uc81c\ub85c \ub2f9\uc2dc 2\uc0b4\ubc30\uae30 \uc544\uae30\ub454 \uc544\uc90c\ub9c8\ub85c \ub4dc\ub7ec\ub0ac\ub2e4)+\uc7ac\uc815\uc2e0\uccad \uac81\ubc15(\uc7ac\uc815\uc2e0\uccad\ud560\ud14c\ub2c8 \ubb34\uc8c4\ub2ec\ub77c \uac81\ubc15\ud588\uc73c\ub098 \uacb0\uad6d \ud53c\ud574\uc790\ub780 \uc99d\uac70\uc5c6\ub2e4\uace0 \uc7ac\uc815\uc2e0\uccad \uae30\uac01\ub2f9\ud588\ub2e4)+\uc5ec\uc131\ub2e8\uccb4 \uc774\uc6a9\ud55c \ud611\ubc15(\ubb34\uc8c4 \uc548\uc8fc\uae30\ub9cc\ud574\ubd10 \uc5ec\uc131\ub2e8\uccb4 \ub2e4 \ub4e4\uace0\uc77c\uc5b4\ub0a0\uac70\uc57c!)+\uc7ac\ud310\uc7a5\uc5d0 \uc5ec\uc131\ub2e8\uccb4 \ubab0\uace0\uac00 \uac80\uc0ac \uacf5\uaca9\ud558\uac8c \ub9cc\ub4dc\ub294\ub4f1 \uc628\uac16 \uc218\ub2e8\uacfc \ubc29\ubc95\uc744 \uac00\ub9ac\uc9c0\uc54a\uc558\ub2e4.", "paragraph_id": "6516355-2-0", "paragraph_question": "question: \ubc15\uc720\ucc9c\uc758 2\ucc28 \ud5c8\uc704 \uace0\uc18c \uc5ec\uc131\uc774 \uac70\uc9d3\uc73c\ub85c \ub9d0\ud588\uc74c\uc774 \ub4dc\ub7ec\ub09c \uc9c1\uc5c5\uc740?, context: \uadf8\ub798\uc11c \uac80\uc0ac\uac00 \"\ud53c\ud574\uc790\ub77c \uc8fc\uc7a5\ud558\ub294 \ucc9c\uba85 \uc774\uc0c1\uc758 \uc5ec\uc798 \uc218\uc0ac\ud588\uc9c0\ub9cc 2\ucc28\ud5c8\uc704\uace0\uc18c\ub140\ucc98\ub7fc \uc99d\uac70\ub294 \ud558\ub098\ub3c4 \ubaa8\uc73c\uc9c0 \ubabb\ud558\uace0 \ube44\uc0c1\uc2dd\uc801\uc778 \uc8fc\uc7a5\ub9cc \ud558\ub294 \uc5ec\uc794 \ucc98\uc74c\" \uc774\ub77c\uba70 \ub9e4\uc6b0 \ud669\ub2f9\ud574\ud588\ub2e4.2\ucc28\ub140\ub294 \uc5b8\ub860\ud50c\ub808\uc774\ub85c \ubc15\uc720\ucc9c \uac00\ud574\uc790 \ubab0\uc774 \uacf5\uaca9+\uae30\uc790\ud68c\uacac(\ubcd1\ud48d\ub4a4\uc5d0 \uc228\uc5b4 \ud3c9\ubc94\ud55c \uc5ec\ub300\uc0dd\uc774\ub77c\uba70 \uac10\uc131\ud314\uc774 \uc5b8\ud50c\ub4f1\uc744\ud588\uc73c\ub098 \uc2e4\uc81c\ub85c \ub2f9\uc2dc 2\uc0b4\ubc30\uae30 \uc544\uae30\ub454 \uc544\uc90c\ub9c8\ub85c \ub4dc\ub7ec\ub0ac\ub2e4)+\uc7ac\uc815\uc2e0\uccad \uac81\ubc15(\uc7ac\uc815\uc2e0\uccad\ud560\ud14c\ub2c8 \ubb34\uc8c4\ub2ec\ub77c \uac81\ubc15\ud588\uc73c\ub098 \uacb0\uad6d \ud53c\ud574\uc790\ub780 \uc99d\uac70\uc5c6\ub2e4\uace0 \uc7ac\uc815\uc2e0\uccad \uae30\uac01\ub2f9\ud588\ub2e4)+\uc5ec\uc131\ub2e8\uccb4 \uc774\uc6a9\ud55c \ud611\ubc15(\ubb34\uc8c4 \uc548\uc8fc\uae30\ub9cc\ud574\ubd10 \uc5ec\uc131\ub2e8\uccb4 \ub2e4 \ub4e4\uace0\uc77c\uc5b4\ub0a0\uac70\uc57c!)+\uc7ac\ud310\uc7a5\uc5d0 \uc5ec\uc131\ub2e8\uccb4 \ubab0\uace0\uac00 \uac80\uc0ac \uacf5\uaca9\ud558\uac8c \ub9cc\ub4dc\ub294\ub4f1 \uc628\uac16 \uc218\ub2e8\uacfc \ubc29\ubc95\uc744 \uac00\ub9ac\uc9c0\uc54a\uc558\ub2e4. \uc2e4\uc81c\ub85c 2\ucc28\uace0\uc18c\ub140\ub294 \uc5ec\uc131\ub2e8\uccb4\ub85c\ubd80\ud130 \uac01\uc885 \uc9c0\uc6d0\uc744 \uc544\ub08c\uc5c6\uc774 \ubc1b\uc740 \uac83\uc73c\ub85c \ub4dc\ub7ec\ub0ac\ub2e4, 1\ucc28\ub140\uac00 \uac10\uc625\uc5d0 \uac00\uc790 \uc5ec\uc131\ub2e8\uccb4\ub294 \ub208\uc774 \ub4a4\uc9d1\ud600 \ud53c\ucf13\ub4e4\uace0 \ubc95\uc6d0 \uad11\ud654\ubb38 \ud55c\ubcf5\ud310\ub4f1\uc11c \uc2dc\uc704 2\ucc28\ub140 \uc7ac\ud310\uc7a5\uc5d0 \ubab0\ub824\uac00 \uc18c\ub780\ud53c\uc6b0\uba70 \ud6fc\ubc29\ub193\uae30\ub4f1\uc73c\ub85c 2\ucc28\ub140 \uc7ac\ud310\uc5d0 \uc544\uc8fc \uae4a\uc219\ud788 \uac1c\uc785\ud55c \uac83\uc73c\ub85c \ub4dc\ub7ec\ub0ac\ub2e4, \uc774\ubbf8 \ubc15\uc720\ucc9c \uace0\uc18c4\uac74\uc740 \ubaa8\ub450 \uac70\uc9d3\uc73c\ub85c \ub4dc\ub7ec\ub0ac\uace0 1\ucc282\ucc28\ub140\uac00 \ud310\uc0ac \uac80\uc0ac \uc9d5\uc5ed\ud615\uae4c\uc9c0 \ub098\uc654\uc74c\uc5d0\ub3c4 \uc774\uac19\uc740 \ud53c\ud574\uc790 2\ucc28\uac00\ud574\ub85c \uc2e4\uc81c\ub85c \ubc15\uc720\ucc9c\uc740 \ub450\ubc88 \uc8fd\uc73c\ub824\ud588\uc5c8\ub358 \uac83\uc73c\ub85c \ub4dc\ub7ec\ub0ac\ub2e4."} {"question": "\uc624\uc2a4\ud2b8\ub808\uc77c\ub9ac\uc544\uae4c\uce58\uac00 \uc1e0\ub625\uad6c\ub9ac\ub97c \ucc3e\uc744\ub54c \uc774\uc6a9\ud558\ub294 \uac83\uc740?", "paragraph": "\uc624\uc2a4\ud2b8\ub808\uc77c\ub9ac\uc544\uae4c\uce58\ub294 \uc7a1\uc2dd\uc131\uc73c\ub85c, \uc8fc\uc704\uc758 \ub545\uc5d0 \uc788\ub294 \uba39\uc744 \uc218 \uc788\ub294 \uac74 \uac70\uc758 \ub2e4 \uba39\ub294\ub2e4. \uc9c0\ub801\uc774, \ub178\ub798\uae30, \ub2ec\ud33d\uc774, \uac70\ubbf8, \uc804\uac08 \uac19\uc740 \ubb34\ucc99\ucd94\ub3d9\ubb3c\ub4e4\ubd80\ud130 \ubc14\ud034\ubc8c\ub808, \uac1c\ubbf8, \ub531\uc815\ubc8c\ub808, \ub098\ubc29, \uc560\ubc8c\ub808\ub97c \ube44\ub86f\ud55c \uace4\ucda9\ub4e4, \ub3c4\ub9c8\ubc40(\uc2a4\ud0b9\ud06c), \uac1c\uad6c\ub9ac, \uc0dd\uc950 \uac19\uc740 \uc791\uc740 \ucc99\ucd94\ub3d9\ubb3c\ub4e4, \uace1\uc2dd, \ub369\uc774\uc904\uae30, \ubb34\ud654\uacfc, \uacac\uacfc\ub958 \ub4f1\uc774 \uba39\uc774\ub85c \ud655\uc778\ub41c \ubc14 \uc788\ub2e4. \uc2ec\uc9c0\uc5b4\ub294 \ub4f1\uaecd\uc9c8\uc5d0\uc11c \ub3c5\uc774 \ubd84\ube44\ub418\ub294 \uc0ac\ud0d5\uc218\uc218\ub450\uaebc\ube44\ub97c \uacf5\uc911\uc73c\ub85c \ub4a4\uc9d1\uc5b4\uc11c \ubc30\ub97c \ucc22\uc5b4\uba39\uc744 \uc904\ub3c4 \uc548\ub2e4. \uc624\uc2a4\ud2b8\ub808\uc77c\ub9ac\uc544\uae4c\uce58\ub294 \ub300\uac1c \ub545 \uc704\uc5d0\uc11c \ub07c\ub2c8\ub97c \ud574\uacb0\ud558\ub294\ub370, \uac1c\ud65c\uc9c0\ub97c \uc870\uc9c1\uc801\uc73c\ub85c \uc11c\uc131\ub300\uba74\uc11c \uace4\ucda9\uacfc \uc560\ubc8c\ub808\ub97c \ucc3e\ub294\ub2e4. \ud55c \uc5f0\uad6c\uc5d0\uc11c\ub294 \uc624\uc2a4\ud2b8\ub808\uc77c\ub9ac\uc544\uae4c\uce58\uac00 \uc18c\ub9ac \ub610\ub294 \uc9c4\ub3d9\uc744 \uc774\uc6a9\ud574 \uc1e0\ub625\uad6c\ub9ac\ub97c \ucc3e\uc544\ub0bc \uc218 \uc788\uc74c\uc774 \ubc1d\ud600\uc84c\ub2e4. \uc624\uc2a4\ud2b8\ub808\uc77c\ub9ac\uc544\uae4c\uce58\ub294 \ubd80\ub9ac\ub97c \uc774\uc6a9\ud574 \ub545\uc744 \ud5e4\uc9d1\uac70\ub098 \ud30c\ud3b8\uc870\uac01\uc744 \ub4a4\uc9d1\uc5b4 \uba39\uc774\uac00 \uc788\ub294\uc9c0 \uc870\uc0ac\ud560 \uc218 \uc788\ub2e4. \uc791\uc740 \uba39\uc774\ub294 \ud1b5\uc9f8\ub85c \uc0bc\ud0a4\ub294\ub370, \uafc0\ubc8c\uc774\ub098 \ub9d0\ubc8c\ub958\ub294 \uc0bc\ud0a4\uae30 \uc804\uc5d0 \ub3c5\uce68\uc744 \ubb38\uc9c8\ub7ec\uc11c \ubf51\uc544\ubc84\ub9b0\ub2e4.", "answer": "\uc18c\ub9ac \ub610\ub294 \uc9c4\ub3d9", "sentence": "\ud55c \uc5f0\uad6c\uc5d0\uc11c\ub294 \uc624\uc2a4\ud2b8\ub808\uc77c\ub9ac\uc544\uae4c\uce58\uac00 \uc18c\ub9ac \ub610\ub294 \uc9c4\ub3d9 \uc744 \uc774\uc6a9\ud574 \uc1e0\ub625\uad6c\ub9ac\ub97c \ucc3e\uc544\ub0bc \uc218 \uc788\uc74c\uc774 \ubc1d\ud600\uc84c\ub2e4.", "paragraph_sentence": "\uc624\uc2a4\ud2b8\ub808\uc77c\ub9ac\uc544\uae4c\uce58\ub294 \uc7a1\uc2dd\uc131\uc73c\ub85c, \uc8fc\uc704\uc758 \ub545\uc5d0 \uc788\ub294 \uba39\uc744 \uc218 \uc788\ub294 \uac74 \uac70\uc758 \ub2e4 \uba39\ub294\ub2e4. \uc9c0\ub801\uc774, \ub178\ub798\uae30, \ub2ec\ud33d\uc774, \uac70\ubbf8, \uc804\uac08 \uac19\uc740 \ubb34\ucc99\ucd94\ub3d9\ubb3c\ub4e4\ubd80\ud130 \ubc14\ud034\ubc8c\ub808, \uac1c\ubbf8, \ub531\uc815\ubc8c\ub808, \ub098\ubc29, \uc560\ubc8c\ub808\ub97c \ube44\ub86f\ud55c \uace4\ucda9\ub4e4, \ub3c4\ub9c8\ubc40(\uc2a4\ud0b9\ud06c), \uac1c\uad6c\ub9ac, \uc0dd\uc950 \uac19\uc740 \uc791\uc740 \ucc99\ucd94\ub3d9\ubb3c\ub4e4, \uace1\uc2dd, \ub369\uc774\uc904\uae30, \ubb34\ud654\uacfc, \uacac\uacfc\ub958 \ub4f1\uc774 \uba39\uc774\ub85c \ud655\uc778\ub41c \ubc14 \uc788\ub2e4. \uc2ec\uc9c0\uc5b4\ub294 \ub4f1\uaecd\uc9c8\uc5d0\uc11c \ub3c5\uc774 \ubd84\ube44\ub418\ub294 \uc0ac\ud0d5\uc218\uc218\ub450\uaebc\ube44\ub97c \uacf5\uc911\uc73c\ub85c \ub4a4\uc9d1\uc5b4\uc11c \ubc30\ub97c \ucc22\uc5b4\uba39\uc744 \uc904\ub3c4 \uc548\ub2e4. \uc624\uc2a4\ud2b8\ub808\uc77c\ub9ac\uc544\uae4c\uce58\ub294 \ub300\uac1c \ub545 \uc704\uc5d0\uc11c \ub07c\ub2c8\ub97c \ud574\uacb0\ud558\ub294\ub370, \uac1c\ud65c\uc9c0\ub97c \uc870\uc9c1\uc801\uc73c\ub85c \uc11c\uc131\ub300\uba74\uc11c \uace4\ucda9\uacfc \uc560\ubc8c\ub808\ub97c \ucc3e\ub294\ub2e4. \ud55c \uc5f0\uad6c\uc5d0\uc11c\ub294 \uc624\uc2a4\ud2b8\ub808\uc77c\ub9ac\uc544\uae4c\uce58\uac00 \uc18c\ub9ac \ub610\ub294 \uc9c4\ub3d9 \uc744 \uc774\uc6a9\ud574 \uc1e0\ub625\uad6c\ub9ac\ub97c \ucc3e\uc544\ub0bc \uc218 \uc788\uc74c\uc774 \ubc1d\ud600\uc84c\ub2e4. \uc624\uc2a4\ud2b8\ub808\uc77c\ub9ac\uc544\uae4c\uce58\ub294 \ubd80\ub9ac\ub97c \uc774\uc6a9\ud574 \ub545\uc744 \ud5e4\uc9d1\uac70\ub098 \ud30c\ud3b8\uc870\uac01\uc744 \ub4a4\uc9d1\uc5b4 \uba39\uc774\uac00 \uc788\ub294\uc9c0 \uc870\uc0ac\ud560 \uc218 \uc788\ub2e4. \uc791\uc740 \uba39\uc774\ub294 \ud1b5\uc9f8\ub85c \uc0bc\ud0a4\ub294\ub370, \uafc0\ubc8c\uc774\ub098 \ub9d0\ubc8c\ub958\ub294 \uc0bc\ud0a4\uae30 \uc804\uc5d0 \ub3c5\uce68\uc744 \ubb38\uc9c8\ub7ec\uc11c \ubf51\uc544\ubc84\ub9b0\ub2e4.", "paragraph_answer": "\uc624\uc2a4\ud2b8\ub808\uc77c\ub9ac\uc544\uae4c\uce58\ub294 \uc7a1\uc2dd\uc131\uc73c\ub85c, \uc8fc\uc704\uc758 \ub545\uc5d0 \uc788\ub294 \uba39\uc744 \uc218 \uc788\ub294 \uac74 \uac70\uc758 \ub2e4 \uba39\ub294\ub2e4. \uc9c0\ub801\uc774, \ub178\ub798\uae30, \ub2ec\ud33d\uc774, \uac70\ubbf8, \uc804\uac08 \uac19\uc740 \ubb34\ucc99\ucd94\ub3d9\ubb3c\ub4e4\ubd80\ud130 \ubc14\ud034\ubc8c\ub808, \uac1c\ubbf8, \ub531\uc815\ubc8c\ub808, \ub098\ubc29, \uc560\ubc8c\ub808\ub97c \ube44\ub86f\ud55c \uace4\ucda9\ub4e4, \ub3c4\ub9c8\ubc40(\uc2a4\ud0b9\ud06c), \uac1c\uad6c\ub9ac, \uc0dd\uc950 \uac19\uc740 \uc791\uc740 \ucc99\ucd94\ub3d9\ubb3c\ub4e4, \uace1\uc2dd, \ub369\uc774\uc904\uae30, \ubb34\ud654\uacfc, \uacac\uacfc\ub958 \ub4f1\uc774 \uba39\uc774\ub85c \ud655\uc778\ub41c \ubc14 \uc788\ub2e4. \uc2ec\uc9c0\uc5b4\ub294 \ub4f1\uaecd\uc9c8\uc5d0\uc11c \ub3c5\uc774 \ubd84\ube44\ub418\ub294 \uc0ac\ud0d5\uc218\uc218\ub450\uaebc\ube44\ub97c \uacf5\uc911\uc73c\ub85c \ub4a4\uc9d1\uc5b4\uc11c \ubc30\ub97c \ucc22\uc5b4\uba39\uc744 \uc904\ub3c4 \uc548\ub2e4. \uc624\uc2a4\ud2b8\ub808\uc77c\ub9ac\uc544\uae4c\uce58\ub294 \ub300\uac1c \ub545 \uc704\uc5d0\uc11c \ub07c\ub2c8\ub97c \ud574\uacb0\ud558\ub294\ub370, \uac1c\ud65c\uc9c0\ub97c \uc870\uc9c1\uc801\uc73c\ub85c \uc11c\uc131\ub300\uba74\uc11c \uace4\ucda9\uacfc \uc560\ubc8c\ub808\ub97c \ucc3e\ub294\ub2e4. \ud55c \uc5f0\uad6c\uc5d0\uc11c\ub294 \uc624\uc2a4\ud2b8\ub808\uc77c\ub9ac\uc544\uae4c\uce58\uac00 \uc18c\ub9ac \ub610\ub294 \uc9c4\ub3d9 \uc744 \uc774\uc6a9\ud574 \uc1e0\ub625\uad6c\ub9ac\ub97c \ucc3e\uc544\ub0bc \uc218 \uc788\uc74c\uc774 \ubc1d\ud600\uc84c\ub2e4. \uc624\uc2a4\ud2b8\ub808\uc77c\ub9ac\uc544\uae4c\uce58\ub294 \ubd80\ub9ac\ub97c \uc774\uc6a9\ud574 \ub545\uc744 \ud5e4\uc9d1\uac70\ub098 \ud30c\ud3b8\uc870\uac01\uc744 \ub4a4\uc9d1\uc5b4 \uba39\uc774\uac00 \uc788\ub294\uc9c0 \uc870\uc0ac\ud560 \uc218 \uc788\ub2e4. \uc791\uc740 \uba39\uc774\ub294 \ud1b5\uc9f8\ub85c \uc0bc\ud0a4\ub294\ub370, \uafc0\ubc8c\uc774\ub098 \ub9d0\ubc8c\ub958\ub294 \uc0bc\ud0a4\uae30 \uc804\uc5d0 \ub3c5\uce68\uc744 \ubb38\uc9c8\ub7ec\uc11c \ubf51\uc544\ubc84\ub9b0\ub2e4.", "sentence_answer": "\ud55c \uc5f0\uad6c\uc5d0\uc11c\ub294 \uc624\uc2a4\ud2b8\ub808\uc77c\ub9ac\uc544\uae4c\uce58\uac00 \uc18c\ub9ac \ub610\ub294 \uc9c4\ub3d9 \uc744 \uc774\uc6a9\ud574 \uc1e0\ub625\uad6c\ub9ac\ub97c \ucc3e\uc544\ub0bc \uc218 \uc788\uc74c\uc774 \ubc1d\ud600\uc84c\ub2e4.", "paragraph_id": "6524880-10-1", "paragraph_question": "question: \uc624\uc2a4\ud2b8\ub808\uc77c\ub9ac\uc544\uae4c\uce58\uac00 \uc1e0\ub625\uad6c\ub9ac\ub97c \ucc3e\uc744\ub54c \uc774\uc6a9\ud558\ub294 \uac83\uc740?, context: \uc624\uc2a4\ud2b8\ub808\uc77c\ub9ac\uc544\uae4c\uce58\ub294 \uc7a1\uc2dd\uc131\uc73c\ub85c, \uc8fc\uc704\uc758 \ub545\uc5d0 \uc788\ub294 \uba39\uc744 \uc218 \uc788\ub294 \uac74 \uac70\uc758 \ub2e4 \uba39\ub294\ub2e4. \uc9c0\ub801\uc774, \ub178\ub798\uae30, \ub2ec\ud33d\uc774, \uac70\ubbf8, \uc804\uac08 \uac19\uc740 \ubb34\ucc99\ucd94\ub3d9\ubb3c\ub4e4\ubd80\ud130 \ubc14\ud034\ubc8c\ub808, \uac1c\ubbf8, \ub531\uc815\ubc8c\ub808, \ub098\ubc29, \uc560\ubc8c\ub808\ub97c \ube44\ub86f\ud55c \uace4\ucda9\ub4e4, \ub3c4\ub9c8\ubc40(\uc2a4\ud0b9\ud06c), \uac1c\uad6c\ub9ac, \uc0dd\uc950 \uac19\uc740 \uc791\uc740 \ucc99\ucd94\ub3d9\ubb3c\ub4e4, \uace1\uc2dd, \ub369\uc774\uc904\uae30, \ubb34\ud654\uacfc, \uacac\uacfc\ub958 \ub4f1\uc774 \uba39\uc774\ub85c \ud655\uc778\ub41c \ubc14 \uc788\ub2e4. \uc2ec\uc9c0\uc5b4\ub294 \ub4f1\uaecd\uc9c8\uc5d0\uc11c \ub3c5\uc774 \ubd84\ube44\ub418\ub294 \uc0ac\ud0d5\uc218\uc218\ub450\uaebc\ube44\ub97c \uacf5\uc911\uc73c\ub85c \ub4a4\uc9d1\uc5b4\uc11c \ubc30\ub97c \ucc22\uc5b4\uba39\uc744 \uc904\ub3c4 \uc548\ub2e4. \uc624\uc2a4\ud2b8\ub808\uc77c\ub9ac\uc544\uae4c\uce58\ub294 \ub300\uac1c \ub545 \uc704\uc5d0\uc11c \ub07c\ub2c8\ub97c \ud574\uacb0\ud558\ub294\ub370, \uac1c\ud65c\uc9c0\ub97c \uc870\uc9c1\uc801\uc73c\ub85c \uc11c\uc131\ub300\uba74\uc11c \uace4\ucda9\uacfc \uc560\ubc8c\ub808\ub97c \ucc3e\ub294\ub2e4. \ud55c \uc5f0\uad6c\uc5d0\uc11c\ub294 \uc624\uc2a4\ud2b8\ub808\uc77c\ub9ac\uc544\uae4c\uce58\uac00 \uc18c\ub9ac \ub610\ub294 \uc9c4\ub3d9\uc744 \uc774\uc6a9\ud574 \uc1e0\ub625\uad6c\ub9ac\ub97c \ucc3e\uc544\ub0bc \uc218 \uc788\uc74c\uc774 \ubc1d\ud600\uc84c\ub2e4. \uc624\uc2a4\ud2b8\ub808\uc77c\ub9ac\uc544\uae4c\uce58\ub294 \ubd80\ub9ac\ub97c \uc774\uc6a9\ud574 \ub545\uc744 \ud5e4\uc9d1\uac70\ub098 \ud30c\ud3b8\uc870\uac01\uc744 \ub4a4\uc9d1\uc5b4 \uba39\uc774\uac00 \uc788\ub294\uc9c0 \uc870\uc0ac\ud560 \uc218 \uc788\ub2e4. \uc791\uc740 \uba39\uc774\ub294 \ud1b5\uc9f8\ub85c \uc0bc\ud0a4\ub294\ub370, \uafc0\ubc8c\uc774\ub098 \ub9d0\ubc8c\ub958\ub294 \uc0bc\ud0a4\uae30 \uc804\uc5d0 \ub3c5\uce68\uc744 \ubb38\uc9c8\ub7ec\uc11c \ubf51\uc544\ubc84\ub9b0\ub2e4."} {"question": "\uc544\ud06c\ub808 \uacf5\ubc29\uc804 \ub2f9\uc2dc \uc544\ud06c\ub808\uc758 \uc778\uad6c\ub294 \ub300\ub7b5 \uba87 \uba85\uc778\uac00?", "paragraph": "\uc774 \ub2f9\uc2dc \uc544\ud06c\ub808\uc758 \uc778\uad6c\ub294 4\ub9cc 5\ucc9c\uba85\uc73c\ub85c \ucd94\uc0b0\ub418\uba70 \uad70\ub300\ub294 \uc57d 1\ub9cc 4\ucc9c\uba85\uc5d0 5\uc6d4 6\uc77c \ud0a4\ud504\ub85c\uc2a4\uc758 \uc655\uc774\uc790 \uc608\ub8e8\uc0b4\ub818 \uc655\uc774\uc5c8\ub358 \uc559\ub9ac2\uc138\uc5d0 \uc758\ud574 \ubcf4\ub0b4\uc9c4 2\ucc9c\uba85\uc774 \ub354\ud588\uc84c\ub2e4. \uadf8\ub9ac\uace0 \ub9d8\ub8e8\ud06c \uad70\ub300\uc758 \uaddc\ubaa8\uc5d0 \ub300\ud574\uc11c\ub294 \uc2e0\ub8b0\ud560 \ub9cc\ud55c \uc790\ub8cc\uac00 \uc5c6\uc9c0\ub9cc \uc77c\ubd80 \uae30\ub85d\uc5d0 \ub530\ub974\uba74 \ub3c4\ud569 22\ub9cc\uba85\uc73c\ub85c \uad6c\uc131\ub418\uc5b4 \uc788\ub2e4\uace0 \ud55c\ub2e4. \ud558\uc9c0\ub9cc \uc2e4\uc81c \uad70\uc138\ub294 \uc774\ubcf4\ub2e4 \ud6e8\uc52c \uc801\uc740 3\ub9cc~7\ub9cc\uba85 \uc815\ub3c4\ub85c \ubcf4\uc778\ub2e4. \uc5ec\uae30\uc5d0 \uacf5\uc131\uc804\uc5d0 \uc790\uc6d0\ud55c \ubcd1\ub825, \uc989 \ud22c\uc11d\uae30 100\uc5ec\ub300(\uc2e4\uc81c\ub85c\ub294 67\ub300 \uc0ac\uc6a9)\uc774 \ud22c\uc785\ub418\uc5b4 \uc788\uc5c8\ub2e4. \uc8fc\ub825\ubcd1\ub825\uc740 \uc131\uc804\uae30\uc0ac\ub2e8\uacfc \uad6c\ud638\uae30\uc0ac\ub2e8, \ud15c\ud50c\uae30\uc0ac\ub2e8, \uc131 \ud1a0\uba38\uc2a4 \uae30\uc0ac\ub2e8, \ud29c\ud2bc\uae30\uc0ac\ub2e8 \ub4f1 \uae30\uc0ac\ub2e8 \ubcd1\ub825\uc73c\ub85c, \ub3c4\ud569 1\ucc9c\uba85 \uc815\ub3c4\ub85c \ucd94\uc0b0\ub41c\ub2e4. \ubc18\uba74 \uc774\uc2ac\ub78c\uce21 \uc8fc\ub825\uc740 \ub9d8\ub8e8\ud06c \uad70\ub300\ub85c, \uc2e4\uc81c \uad70\uc138\ub97c \uce21\uc815\ud558\uae30 \uc5b4\ub835\ub2e4.", "answer": "4\ub9cc 5\ucc9c\uba85", "sentence": "\uc774 \ub2f9\uc2dc \uc544\ud06c\ub808\uc758 \uc778\uad6c\ub294 4\ub9cc 5\ucc9c\uba85 \uc73c\ub85c \ucd94\uc0b0\ub418\uba70 \uad70\ub300\ub294 \uc57d 1\ub9cc 4\ucc9c\uba85\uc5d0 5\uc6d4 6\uc77c \ud0a4\ud504\ub85c\uc2a4\uc758 \uc655\uc774\uc790 \uc608\ub8e8\uc0b4\ub818 \uc655\uc774\uc5c8\ub358 \uc559\ub9ac2\uc138\uc5d0 \uc758\ud574 \ubcf4\ub0b4\uc9c4 2\ucc9c\uba85\uc774 \ub354\ud588\uc84c\ub2e4.", "paragraph_sentence": " \uc774 \ub2f9\uc2dc \uc544\ud06c\ub808\uc758 \uc778\uad6c\ub294 4\ub9cc 5\ucc9c\uba85 \uc73c\ub85c \ucd94\uc0b0\ub418\uba70 \uad70\ub300\ub294 \uc57d 1\ub9cc 4\ucc9c\uba85\uc5d0 5\uc6d4 6\uc77c \ud0a4\ud504\ub85c\uc2a4\uc758 \uc655\uc774\uc790 \uc608\ub8e8\uc0b4\ub818 \uc655\uc774\uc5c8\ub358 \uc559\ub9ac2\uc138\uc5d0 \uc758\ud574 \ubcf4\ub0b4\uc9c4 2\ucc9c\uba85\uc774 \ub354\ud588\uc84c\ub2e4. \uadf8\ub9ac\uace0 \ub9d8\ub8e8\ud06c \uad70\ub300\uc758 \uaddc\ubaa8\uc5d0 \ub300\ud574\uc11c\ub294 \uc2e0\ub8b0\ud560 \ub9cc\ud55c \uc790\ub8cc\uac00 \uc5c6\uc9c0\ub9cc \uc77c\ubd80 \uae30\ub85d\uc5d0 \ub530\ub974\uba74 \ub3c4\ud569 22\ub9cc\uba85\uc73c\ub85c \uad6c\uc131\ub418\uc5b4 \uc788\ub2e4\uace0 \ud55c\ub2e4. \ud558\uc9c0\ub9cc \uc2e4\uc81c \uad70\uc138\ub294 \uc774\ubcf4\ub2e4 \ud6e8\uc52c \uc801\uc740 3\ub9cc~7\ub9cc\uba85 \uc815\ub3c4\ub85c \ubcf4\uc778\ub2e4. \uc5ec\uae30\uc5d0 \uacf5\uc131\uc804\uc5d0 \uc790\uc6d0\ud55c \ubcd1\ub825, \uc989 \ud22c\uc11d\uae30 100\uc5ec\ub300(\uc2e4\uc81c\ub85c\ub294 67\ub300 \uc0ac\uc6a9)\uc774 \ud22c\uc785\ub418\uc5b4 \uc788\uc5c8\ub2e4. \uc8fc\ub825\ubcd1\ub825\uc740 \uc131\uc804\uae30\uc0ac\ub2e8\uacfc \uad6c\ud638\uae30\uc0ac\ub2e8, \ud15c\ud50c\uae30\uc0ac\ub2e8, \uc131 \ud1a0\uba38\uc2a4 \uae30\uc0ac\ub2e8, \ud29c\ud2bc\uae30\uc0ac\ub2e8 \ub4f1 \uae30\uc0ac\ub2e8 \ubcd1\ub825\uc73c\ub85c, \ub3c4\ud569 1\ucc9c\uba85 \uc815\ub3c4\ub85c \ucd94\uc0b0\ub41c\ub2e4. \ubc18\uba74 \uc774\uc2ac\ub78c\uce21 \uc8fc\ub825\uc740 \ub9d8\ub8e8\ud06c \uad70\ub300\ub85c, \uc2e4\uc81c \uad70\uc138\ub97c \uce21\uc815\ud558\uae30 \uc5b4\ub835\ub2e4.", "paragraph_answer": "\uc774 \ub2f9\uc2dc \uc544\ud06c\ub808\uc758 \uc778\uad6c\ub294 4\ub9cc 5\ucc9c\uba85 \uc73c\ub85c \ucd94\uc0b0\ub418\uba70 \uad70\ub300\ub294 \uc57d 1\ub9cc 4\ucc9c\uba85\uc5d0 5\uc6d4 6\uc77c \ud0a4\ud504\ub85c\uc2a4\uc758 \uc655\uc774\uc790 \uc608\ub8e8\uc0b4\ub818 \uc655\uc774\uc5c8\ub358 \uc559\ub9ac2\uc138\uc5d0 \uc758\ud574 \ubcf4\ub0b4\uc9c4 2\ucc9c\uba85\uc774 \ub354\ud588\uc84c\ub2e4. \uadf8\ub9ac\uace0 \ub9d8\ub8e8\ud06c \uad70\ub300\uc758 \uaddc\ubaa8\uc5d0 \ub300\ud574\uc11c\ub294 \uc2e0\ub8b0\ud560 \ub9cc\ud55c \uc790\ub8cc\uac00 \uc5c6\uc9c0\ub9cc \uc77c\ubd80 \uae30\ub85d\uc5d0 \ub530\ub974\uba74 \ub3c4\ud569 22\ub9cc\uba85\uc73c\ub85c \uad6c\uc131\ub418\uc5b4 \uc788\ub2e4\uace0 \ud55c\ub2e4. \ud558\uc9c0\ub9cc \uc2e4\uc81c \uad70\uc138\ub294 \uc774\ubcf4\ub2e4 \ud6e8\uc52c \uc801\uc740 3\ub9cc~7\ub9cc\uba85 \uc815\ub3c4\ub85c \ubcf4\uc778\ub2e4. \uc5ec\uae30\uc5d0 \uacf5\uc131\uc804\uc5d0 \uc790\uc6d0\ud55c \ubcd1\ub825, \uc989 \ud22c\uc11d\uae30 100\uc5ec\ub300(\uc2e4\uc81c\ub85c\ub294 67\ub300 \uc0ac\uc6a9)\uc774 \ud22c\uc785\ub418\uc5b4 \uc788\uc5c8\ub2e4. \uc8fc\ub825\ubcd1\ub825\uc740 \uc131\uc804\uae30\uc0ac\ub2e8\uacfc \uad6c\ud638\uae30\uc0ac\ub2e8, \ud15c\ud50c\uae30\uc0ac\ub2e8, \uc131 \ud1a0\uba38\uc2a4 \uae30\uc0ac\ub2e8, \ud29c\ud2bc\uae30\uc0ac\ub2e8 \ub4f1 \uae30\uc0ac\ub2e8 \ubcd1\ub825\uc73c\ub85c, \ub3c4\ud569 1\ucc9c\uba85 \uc815\ub3c4\ub85c \ucd94\uc0b0\ub41c\ub2e4. \ubc18\uba74 \uc774\uc2ac\ub78c\uce21 \uc8fc\ub825\uc740 \ub9d8\ub8e8\ud06c \uad70\ub300\ub85c, \uc2e4\uc81c \uad70\uc138\ub97c \uce21\uc815\ud558\uae30 \uc5b4\ub835\ub2e4.", "sentence_answer": "\uc774 \ub2f9\uc2dc \uc544\ud06c\ub808\uc758 \uc778\uad6c\ub294 4\ub9cc 5\ucc9c\uba85 \uc73c\ub85c \ucd94\uc0b0\ub418\uba70 \uad70\ub300\ub294 \uc57d 1\ub9cc 4\ucc9c\uba85\uc5d0 5\uc6d4 6\uc77c \ud0a4\ud504\ub85c\uc2a4\uc758 \uc655\uc774\uc790 \uc608\ub8e8\uc0b4\ub818 \uc655\uc774\uc5c8\ub358 \uc559\ub9ac2\uc138\uc5d0 \uc758\ud574 \ubcf4\ub0b4\uc9c4 2\ucc9c\uba85\uc774 \ub354\ud588\uc84c\ub2e4.", "paragraph_id": "6521183-3-0", "paragraph_question": "question: \uc544\ud06c\ub808 \uacf5\ubc29\uc804 \ub2f9\uc2dc \uc544\ud06c\ub808\uc758 \uc778\uad6c\ub294 \ub300\ub7b5 \uba87 \uba85\uc778\uac00?, context: \uc774 \ub2f9\uc2dc \uc544\ud06c\ub808\uc758 \uc778\uad6c\ub294 4\ub9cc 5\ucc9c\uba85\uc73c\ub85c \ucd94\uc0b0\ub418\uba70 \uad70\ub300\ub294 \uc57d 1\ub9cc 4\ucc9c\uba85\uc5d0 5\uc6d4 6\uc77c \ud0a4\ud504\ub85c\uc2a4\uc758 \uc655\uc774\uc790 \uc608\ub8e8\uc0b4\ub818 \uc655\uc774\uc5c8\ub358 \uc559\ub9ac2\uc138\uc5d0 \uc758\ud574 \ubcf4\ub0b4\uc9c4 2\ucc9c\uba85\uc774 \ub354\ud588\uc84c\ub2e4. \uadf8\ub9ac\uace0 \ub9d8\ub8e8\ud06c \uad70\ub300\uc758 \uaddc\ubaa8\uc5d0 \ub300\ud574\uc11c\ub294 \uc2e0\ub8b0\ud560 \ub9cc\ud55c \uc790\ub8cc\uac00 \uc5c6\uc9c0\ub9cc \uc77c\ubd80 \uae30\ub85d\uc5d0 \ub530\ub974\uba74 \ub3c4\ud569 22\ub9cc\uba85\uc73c\ub85c \uad6c\uc131\ub418\uc5b4 \uc788\ub2e4\uace0 \ud55c\ub2e4. \ud558\uc9c0\ub9cc \uc2e4\uc81c \uad70\uc138\ub294 \uc774\ubcf4\ub2e4 \ud6e8\uc52c \uc801\uc740 3\ub9cc~7\ub9cc\uba85 \uc815\ub3c4\ub85c \ubcf4\uc778\ub2e4. \uc5ec\uae30\uc5d0 \uacf5\uc131\uc804\uc5d0 \uc790\uc6d0\ud55c \ubcd1\ub825, \uc989 \ud22c\uc11d\uae30 100\uc5ec\ub300(\uc2e4\uc81c\ub85c\ub294 67\ub300 \uc0ac\uc6a9)\uc774 \ud22c\uc785\ub418\uc5b4 \uc788\uc5c8\ub2e4. \uc8fc\ub825\ubcd1\ub825\uc740 \uc131\uc804\uae30\uc0ac\ub2e8\uacfc \uad6c\ud638\uae30\uc0ac\ub2e8, \ud15c\ud50c\uae30\uc0ac\ub2e8, \uc131 \ud1a0\uba38\uc2a4 \uae30\uc0ac\ub2e8, \ud29c\ud2bc\uae30\uc0ac\ub2e8 \ub4f1 \uae30\uc0ac\ub2e8 \ubcd1\ub825\uc73c\ub85c, \ub3c4\ud569 1\ucc9c\uba85 \uc815\ub3c4\ub85c \ucd94\uc0b0\ub41c\ub2e4. \ubc18\uba74 \uc774\uc2ac\ub78c\uce21 \uc8fc\ub825\uc740 \ub9d8\ub8e8\ud06c \uad70\ub300\ub85c, \uc2e4\uc81c \uad70\uc138\ub97c \uce21\uc815\ud558\uae30 \uc5b4\ub835\ub2e4."} {"question": "\uc1a1\ud61c\uad50\uc758 \ubcf8\uad00\uc740?", "paragraph": "\uc1a1\ud61c\uad50(\u5b8b\u6167\u55ac, 1981\ub144 11\uc6d4 22\uc77c ~ )\ub294 \ub300\ud55c\ubbfc\uad6d\uc758 \ubc30\uc6b0\uc774\ub2e4. \ubcf8\uad00\uc740 \uc5ec\uc0b0. \uadf8\ub140\ub294 1996\ub144 SK \uc2a4\ub9c8\ud2b8 \ubaa8\ub378 \uc120\ubc1c\ub300\ud68c \ub300\uc0c1 \uc218\uc0c1\uc73c\ub85c \ub370\ubdd4\ud55c \ud6c4 1996\ub144 \ub4dc\ub77c\ub9c8 \u300a\uccab\uc0ac\ub791\u300b\uc744 \ud1b5\ud574 \uc5f0\uae30\uc790\ub85c \ub370\ubdd4\ud558\uc600\ub2e4. \uc1a1\ud61c\uad50\ub294 1998\ub144 \uc2dc\ud2b8\ucf64 \u300a\uc21c\ud48d\uc0b0\ubd80\uc778\uacfc\u300b\uc640 2000\ub144 \ub4dc\ub77c\ub9c8 \u300a\uac00\uc744\ub3d9\ud654\u300b\ub97c \ud1b5\ud574 \ub300\uc911\uc801 \uc778\uc9c0\ub3c4\ub97c \ub192\uc774\uba70 \ud1b1\uc2a4\ud0c0\uc758 \ubc18\uc5f4\uc5d0 \uc62c\ub790\ub2e4. \uc774\ud6c4 \uadf8\ub140\ub294 \ub4dc\ub77c\ub9c8 \u300a\uc62c\uc778\u300b, \u300a\ud480\ud558\uc6b0\uc2a4\u300b, \u300a\uadf8\ub4e4\uc774 \uc0ac\ub294 \uc138\uc0c1\u300b, \u300a\uadf8 \uaca8\uc6b8, \ubc14\ub78c\uc774 \ubd84\ub2e4\u300b, \u300a\ud0dc\uc591\uc758 \ud6c4\uc608\u300b\uacfc \uc601\ud654 \u300a\ud30c\ub791\uc8fc\uc758\ubcf4\u300b, \u300a\ud669\uc9c4\uc774\u300b, \u300a\uc624\ub298\u300b, \u300a\ud0dc\ud3c9\ub95c\u300b, \u300a\ub450\uadfc\ub450\uadfc \ub0b4 \uc778\uc0dd\u300b \ub4f1 \uc5ec\ub7ec \uc791\ud488\uc5d0 \ucd9c\uc5f0\ud558\uc600\ub2e4. \uc1a1\ud61c\uad50\ub294 \ub610\ud55c \ud574\uc678 \ud32c\ub4e4\uc758 \ub728\uac70\uc6b4 \uad00\uc2ec\uc744 \ubc1b\ub294 \ud55c\ub958\ud038\uc73c\ub85c \uc0ac\ub791 \ubc1b\ub294 \ub300\ud45c\uc801\uc778 \ud55c\ub958\uc2a4\ud0c0 \uc911 \ud55c\uba85\uc73c\ub85c \uc655\uac00\uc704, \uc624\uc6b0\uc0bc \uac10\ub3c5 \ub4f1\uc758 \uc791\ud488\uc5d0 \ucd9c\uc5f0\ud558\uba70 \uc911\uad6d\uc5d0\uc11c\uc758 \uc785\uc9c0\ub97c \ud06c\uac8c \ub2e4\uc84c\ub2e4.", "answer": "\uc5ec\uc0b0", "sentence": "\ubcf8\uad00\uc740 \uc5ec\uc0b0 .", "paragraph_sentence": "\uc1a1\ud61c\uad50(\u5b8b\u6167\u55ac, 1981\ub144 11\uc6d4 22\uc77c ~ )\ub294 \ub300\ud55c\ubbfc\uad6d\uc758 \ubc30\uc6b0\uc774\ub2e4. \ubcf8\uad00\uc740 \uc5ec\uc0b0 . \uadf8\ub140\ub294 1996\ub144 SK \uc2a4\ub9c8\ud2b8 \ubaa8\ub378 \uc120\ubc1c\ub300\ud68c \ub300\uc0c1 \uc218\uc0c1\uc73c\ub85c \ub370\ubdd4\ud55c \ud6c4 1996\ub144 \ub4dc\ub77c\ub9c8 \u300a\uccab\uc0ac\ub791\u300b\uc744 \ud1b5\ud574 \uc5f0\uae30\uc790\ub85c \ub370\ubdd4\ud558\uc600\ub2e4. \uc1a1\ud61c\uad50\ub294 1998\ub144 \uc2dc\ud2b8\ucf64 \u300a\uc21c\ud48d\uc0b0\ubd80\uc778\uacfc\u300b\uc640 2000\ub144 \ub4dc\ub77c\ub9c8 \u300a\uac00\uc744\ub3d9\ud654\u300b\ub97c \ud1b5\ud574 \ub300\uc911\uc801 \uc778\uc9c0\ub3c4\ub97c \ub192\uc774\uba70 \ud1b1\uc2a4\ud0c0\uc758 \ubc18\uc5f4\uc5d0 \uc62c\ub790\ub2e4. \uc774\ud6c4 \uadf8\ub140\ub294 \ub4dc\ub77c\ub9c8 \u300a\uc62c\uc778\u300b, \u300a\ud480\ud558\uc6b0\uc2a4\u300b, \u300a\uadf8\ub4e4\uc774 \uc0ac\ub294 \uc138\uc0c1\u300b, \u300a\uadf8 \uaca8\uc6b8, \ubc14\ub78c\uc774 \ubd84\ub2e4\u300b, \u300a\ud0dc\uc591\uc758 \ud6c4\uc608\u300b\uacfc \uc601\ud654 \u300a\ud30c\ub791\uc8fc\uc758\ubcf4\u300b, \u300a\ud669\uc9c4\uc774\u300b, \u300a\uc624\ub298\u300b, \u300a\ud0dc\ud3c9\ub95c\u300b, \u300a\ub450\uadfc\ub450\uadfc \ub0b4 \uc778\uc0dd\u300b \ub4f1 \uc5ec\ub7ec \uc791\ud488\uc5d0 \ucd9c\uc5f0\ud558\uc600\ub2e4. \uc1a1\ud61c\uad50\ub294 \ub610\ud55c \ud574\uc678 \ud32c\ub4e4\uc758 \ub728\uac70\uc6b4 \uad00\uc2ec\uc744 \ubc1b\ub294 \ud55c\ub958\ud038\uc73c\ub85c \uc0ac\ub791 \ubc1b\ub294 \ub300\ud45c\uc801\uc778 \ud55c\ub958\uc2a4\ud0c0 \uc911 \ud55c\uba85\uc73c\ub85c \uc655\uac00\uc704, \uc624\uc6b0\uc0bc \uac10\ub3c5 \ub4f1\uc758 \uc791\ud488\uc5d0 \ucd9c\uc5f0\ud558\uba70 \uc911\uad6d\uc5d0\uc11c\uc758 \uc785\uc9c0\ub97c \ud06c\uac8c \ub2e4\uc84c\ub2e4.", "paragraph_answer": "\uc1a1\ud61c\uad50(\u5b8b\u6167\u55ac, 1981\ub144 11\uc6d4 22\uc77c ~ )\ub294 \ub300\ud55c\ubbfc\uad6d\uc758 \ubc30\uc6b0\uc774\ub2e4. \ubcf8\uad00\uc740 \uc5ec\uc0b0 . \uadf8\ub140\ub294 1996\ub144 SK \uc2a4\ub9c8\ud2b8 \ubaa8\ub378 \uc120\ubc1c\ub300\ud68c \ub300\uc0c1 \uc218\uc0c1\uc73c\ub85c \ub370\ubdd4\ud55c \ud6c4 1996\ub144 \ub4dc\ub77c\ub9c8 \u300a\uccab\uc0ac\ub791\u300b\uc744 \ud1b5\ud574 \uc5f0\uae30\uc790\ub85c \ub370\ubdd4\ud558\uc600\ub2e4. \uc1a1\ud61c\uad50\ub294 1998\ub144 \uc2dc\ud2b8\ucf64 \u300a\uc21c\ud48d\uc0b0\ubd80\uc778\uacfc\u300b\uc640 2000\ub144 \ub4dc\ub77c\ub9c8 \u300a\uac00\uc744\ub3d9\ud654\u300b\ub97c \ud1b5\ud574 \ub300\uc911\uc801 \uc778\uc9c0\ub3c4\ub97c \ub192\uc774\uba70 \ud1b1\uc2a4\ud0c0\uc758 \ubc18\uc5f4\uc5d0 \uc62c\ub790\ub2e4. \uc774\ud6c4 \uadf8\ub140\ub294 \ub4dc\ub77c\ub9c8 \u300a\uc62c\uc778\u300b, \u300a\ud480\ud558\uc6b0\uc2a4\u300b, \u300a\uadf8\ub4e4\uc774 \uc0ac\ub294 \uc138\uc0c1\u300b, \u300a\uadf8 \uaca8\uc6b8, \ubc14\ub78c\uc774 \ubd84\ub2e4\u300b, \u300a\ud0dc\uc591\uc758 \ud6c4\uc608\u300b\uacfc \uc601\ud654 \u300a\ud30c\ub791\uc8fc\uc758\ubcf4\u300b, \u300a\ud669\uc9c4\uc774\u300b, \u300a\uc624\ub298\u300b, \u300a\ud0dc\ud3c9\ub95c\u300b, \u300a\ub450\uadfc\ub450\uadfc \ub0b4 \uc778\uc0dd\u300b \ub4f1 \uc5ec\ub7ec \uc791\ud488\uc5d0 \ucd9c\uc5f0\ud558\uc600\ub2e4. \uc1a1\ud61c\uad50\ub294 \ub610\ud55c \ud574\uc678 \ud32c\ub4e4\uc758 \ub728\uac70\uc6b4 \uad00\uc2ec\uc744 \ubc1b\ub294 \ud55c\ub958\ud038\uc73c\ub85c \uc0ac\ub791 \ubc1b\ub294 \ub300\ud45c\uc801\uc778 \ud55c\ub958\uc2a4\ud0c0 \uc911 \ud55c\uba85\uc73c\ub85c \uc655\uac00\uc704, \uc624\uc6b0\uc0bc \uac10\ub3c5 \ub4f1\uc758 \uc791\ud488\uc5d0 \ucd9c\uc5f0\ud558\uba70 \uc911\uad6d\uc5d0\uc11c\uc758 \uc785\uc9c0\ub97c \ud06c\uac8c \ub2e4\uc84c\ub2e4.", "sentence_answer": "\ubcf8\uad00\uc740 \uc5ec\uc0b0 .", "paragraph_id": "6450729-0-1", "paragraph_question": "question: \uc1a1\ud61c\uad50\uc758 \ubcf8\uad00\uc740?, context: \uc1a1\ud61c\uad50(\u5b8b\u6167\u55ac, 1981\ub144 11\uc6d4 22\uc77c ~ )\ub294 \ub300\ud55c\ubbfc\uad6d\uc758 \ubc30\uc6b0\uc774\ub2e4. \ubcf8\uad00\uc740 \uc5ec\uc0b0. \uadf8\ub140\ub294 1996\ub144 SK \uc2a4\ub9c8\ud2b8 \ubaa8\ub378 \uc120\ubc1c\ub300\ud68c \ub300\uc0c1 \uc218\uc0c1\uc73c\ub85c \ub370\ubdd4\ud55c \ud6c4 1996\ub144 \ub4dc\ub77c\ub9c8 \u300a\uccab\uc0ac\ub791\u300b\uc744 \ud1b5\ud574 \uc5f0\uae30\uc790\ub85c \ub370\ubdd4\ud558\uc600\ub2e4. \uc1a1\ud61c\uad50\ub294 1998\ub144 \uc2dc\ud2b8\ucf64 \u300a\uc21c\ud48d\uc0b0\ubd80\uc778\uacfc\u300b\uc640 2000\ub144 \ub4dc\ub77c\ub9c8 \u300a\uac00\uc744\ub3d9\ud654\u300b\ub97c \ud1b5\ud574 \ub300\uc911\uc801 \uc778\uc9c0\ub3c4\ub97c \ub192\uc774\uba70 \ud1b1\uc2a4\ud0c0\uc758 \ubc18\uc5f4\uc5d0 \uc62c\ub790\ub2e4. \uc774\ud6c4 \uadf8\ub140\ub294 \ub4dc\ub77c\ub9c8 \u300a\uc62c\uc778\u300b, \u300a\ud480\ud558\uc6b0\uc2a4\u300b, \u300a\uadf8\ub4e4\uc774 \uc0ac\ub294 \uc138\uc0c1\u300b, \u300a\uadf8 \uaca8\uc6b8, \ubc14\ub78c\uc774 \ubd84\ub2e4\u300b, \u300a\ud0dc\uc591\uc758 \ud6c4\uc608\u300b\uacfc \uc601\ud654 \u300a\ud30c\ub791\uc8fc\uc758\ubcf4\u300b, \u300a\ud669\uc9c4\uc774\u300b, \u300a\uc624\ub298\u300b, \u300a\ud0dc\ud3c9\ub95c\u300b, \u300a\ub450\uadfc\ub450\uadfc \ub0b4 \uc778\uc0dd\u300b \ub4f1 \uc5ec\ub7ec \uc791\ud488\uc5d0 \ucd9c\uc5f0\ud558\uc600\ub2e4. \uc1a1\ud61c\uad50\ub294 \ub610\ud55c \ud574\uc678 \ud32c\ub4e4\uc758 \ub728\uac70\uc6b4 \uad00\uc2ec\uc744 \ubc1b\ub294 \ud55c\ub958\ud038\uc73c\ub85c \uc0ac\ub791 \ubc1b\ub294 \ub300\ud45c\uc801\uc778 \ud55c\ub958\uc2a4\ud0c0 \uc911 \ud55c\uba85\uc73c\ub85c \uc655\uac00\uc704, \uc624\uc6b0\uc0bc \uac10\ub3c5 \ub4f1\uc758 \uc791\ud488\uc5d0 \ucd9c\uc5f0\ud558\uba70 \uc911\uad6d\uc5d0\uc11c\uc758 \uc785\uc9c0\ub97c \ud06c\uac8c \ub2e4\uc84c\ub2e4."} {"question": "\ubc00\uc591\uc2dc \ud558\ub0a8\uc74d\uc5d0\uc11c \ub18d\uc0ac\ub97c \uc9c0\uc73c\uba70 \uc9c0\ub0b4\ub294 \uc778\uad6c\uc218\ub294?", "paragraph": "\ubc00\uc591\uc2dc \ud558\ub0a8\uc74d\uc5d0 8,000\uc5ec\uba85\uc774 \ub18d\uc0ac\ub97c \uc9c0\uc73c\uba70 \uc9c0\ub0b4\uace0 \uc788\ub2e4. \ub545\uc774 \uae30\ub984\uc838 \uac10\uc790\uc640 \ub538\uae30\uac00 \ub9ce\uc774 \ub09c\ub2e4. \ud558\uc9c0\ub9cc \uc2e0\uacf5\ud56d\uc774 \ub4e4\uc5b4\uc11c\uba74 \ub354\uc774\uc0c1 \ub18d\uc0ac\ub97c \uc9c0\uc744 \uc218 \uc5c6\uac8c \ub41c\ub2e4. \uc774 \ub54c\ubb38\uc5d0 \uc77c\ubd80 \uc8fc\ubbfc\ub4e4\uc740 \uc2e0\uacf5\ud56d \uc720\uce58\ub97c \ubc18\ub300\ud558\uace0 \uc788\uc5c8\ub2e4. \ud55c \uc8fc\ubbfc\uc740 \"\ub18d\uc0ac\uc9d3\uace0 \uc788\ub294\ub370 \ub098\uac00\ub77c\uace0 \ud558\uba74 \ubcf4\uc0c1\ub3c4 \uc5c6\uace0 \ubc14\ub85c \uac70\uc9c0\ub2e4\"\ub77c\uba70 \"\uc6b0\ub9ac\ub294 \ud3c9\uc0c1 \ube44\ud589\uae30 \ud0c8\uc77c \ub450\ubc88\ub3c4 \uc5c6\ub294\ub370\"\ub77c\uba70 \uaca9\ubd84\ud588\ub2e4. \uac10\uc790\ubc2d\uc744 \uc6b4\uc601\ud558\ub294 \ub2e4\ub978 \uc8fc\ubbfc\ub3c4 \"\uac10\uc790\ubc2d\uc5d0\uc11c \ud55c\ub2e4\ub808 200~205\ub9cc\uc6d0\uc529 \uc218\uc775\uc774 \ub098\uc624\ub294\ub370 \uc774\uac83\uc744 \ucca0\uac70\ud558\uba74 \uc815\ubd80\uc5d0\uc11c 200\ub9cc\uc6d0 \uc774\uc0c1 \uc8fc\uaca0\ub098\"\ub77c\uba70 \"\ub2f9\uc7a5 \ub098\uac00\uba74 1~2\ub144\uc740 \uba39\uace0\uc0b4\uaca0\uc9c0\ub9cc \uadf8 \uc774\ud6c4\ub294 \ub300\ucc45\uc774 \uc5c6\ub2e4\"\ub77c\uba70 \ubc18\ub300\uc785\uc7a5\uc744 \ubd84\uba85\ud788 \ud588\ub2e4. \uc2e0\uacf5\ud56d \ud6c4\ubcf4 \uc9c0\uc5ed \uc8fc\ubbfc\ub4e4\uc740 \uc2e0\uacf5\ud56d \uc720\uce58\uc640 \uad00\ub828\ud574\uc11c \uc790\uc2e0\ub4e4\uc758 \uc758\uacac\uc774 \ubc30\uc81c\ub418\uc5c8\ub2e4\ub294 \uc0ac\uc2e4\uc5d0 \ubd84\uac1c\ud558\ub294 \uc0ac\ub78c\ub4e4\ub3c4 \ub9ce\uc558\ub2e4. \uc2e0\uacf5\ud56d \uc720\uce58\uc640 \uad00\ub828\ud574\uc11c \uc8fc\ubbfc\ub4e4\uacfc\uc758 \ub300\ud654\uac00 \ub354 \ud544\uc694\ud558\ub2e4\ub294 \uc9c0\uc801\uc774\ub2e4. \ud558\uc9c0\ub9cc \ubc00\uc591\uce21\uc740 \ud6c4\ubcf4\uc9c0 \uc8fc\ubbfc\ub4e4\uacfc \ub300\ud654\ub97c \ud560 \ub2e8\uacc4\ub294 \uc544\ub2c8\ub77c\uace0 \uc798\ub77c \ub9d0\ud55c\ub2e4. \uc2e0\uacf5\ud56d \uc704\uce58\uac00 \ud655\uc815\ub418\uba74 \uadf8 \ub54c\ubd80\ud130 \uc774\uc57c\uae30\ub97c \ud55c\ub2e4\ub294 \uac83\uc774\ub2e4. \ubd80\uc0b0\uad11\uc5ed\uc2dc\ub294 \"\ud604\uc7ac \uc5b4\uc5c5\uc744 \ud558\uace0 \uc788\ub294 \uc5b4\ubbfc\ub4e4\uc5d0\uac8c\ub294 \uc801\uc808\ud55c \ubcf4\uc0c1\uc774 \ub420 \uc218 \uc788\ub3c4\ub85d \uc2dc\uac00 \uc815\ubd80\uc5d0 \uc694\uad6c\ub97c \ud558\uace0 \uc5b4\ubbfc\ud3b8\uc5d0\uc11c \uc77c\uc744 \ud558\uaca0\ub2e4\"\uace0 \ub9d0\ud588\ub2e4.", "answer": "8,000\uc5ec\uba85", "sentence": "\ubc00\uc591\uc2dc \ud558\ub0a8\uc74d\uc5d0 8,000\uc5ec\uba85 \uc774 \ub18d\uc0ac\ub97c \uc9c0\uc73c\uba70 \uc9c0\ub0b4\uace0 \uc788\ub2e4.", "paragraph_sentence": " \ubc00\uc591\uc2dc \ud558\ub0a8\uc74d\uc5d0 8,000\uc5ec\uba85 \uc774 \ub18d\uc0ac\ub97c \uc9c0\uc73c\uba70 \uc9c0\ub0b4\uace0 \uc788\ub2e4. \ub545\uc774 \uae30\ub984\uc838 \uac10\uc790\uc640 \ub538\uae30\uac00 \ub9ce\uc774 \ub09c\ub2e4. \ud558\uc9c0\ub9cc \uc2e0\uacf5\ud56d\uc774 \ub4e4\uc5b4\uc11c\uba74 \ub354\uc774\uc0c1 \ub18d\uc0ac\ub97c \uc9c0\uc744 \uc218 \uc5c6\uac8c \ub41c\ub2e4. \uc774 \ub54c\ubb38\uc5d0 \uc77c\ubd80 \uc8fc\ubbfc\ub4e4\uc740 \uc2e0\uacf5\ud56d \uc720\uce58\ub97c \ubc18\ub300\ud558\uace0 \uc788\uc5c8\ub2e4. \ud55c \uc8fc\ubbfc\uc740 \"\ub18d\uc0ac\uc9d3\uace0 \uc788\ub294\ub370 \ub098\uac00\ub77c\uace0 \ud558\uba74 \ubcf4\uc0c1\ub3c4 \uc5c6\uace0 \ubc14\ub85c \uac70\uc9c0\ub2e4\"\ub77c\uba70 \"\uc6b0\ub9ac\ub294 \ud3c9\uc0c1 \ube44\ud589\uae30 \ud0c8\uc77c \ub450\ubc88\ub3c4 \uc5c6\ub294\ub370\"\ub77c\uba70 \uaca9\ubd84\ud588\ub2e4. \uac10\uc790\ubc2d\uc744 \uc6b4\uc601\ud558\ub294 \ub2e4\ub978 \uc8fc\ubbfc\ub3c4 \"\uac10\uc790\ubc2d\uc5d0\uc11c \ud55c\ub2e4\ub808 200~205\ub9cc\uc6d0\uc529 \uc218\uc775\uc774 \ub098\uc624\ub294\ub370 \uc774\uac83\uc744 \ucca0\uac70\ud558\uba74 \uc815\ubd80\uc5d0\uc11c 200\ub9cc\uc6d0 \uc774\uc0c1 \uc8fc\uaca0\ub098\"\ub77c\uba70 \"\ub2f9\uc7a5 \ub098\uac00\uba74 1~2\ub144\uc740 \uba39\uace0\uc0b4\uaca0\uc9c0\ub9cc \uadf8 \uc774\ud6c4\ub294 \ub300\ucc45\uc774 \uc5c6\ub2e4\"\ub77c\uba70 \ubc18\ub300\uc785\uc7a5\uc744 \ubd84\uba85\ud788 \ud588\ub2e4. \uc2e0\uacf5\ud56d \ud6c4\ubcf4 \uc9c0\uc5ed \uc8fc\ubbfc\ub4e4\uc740 \uc2e0\uacf5\ud56d \uc720\uce58\uc640 \uad00\ub828\ud574\uc11c \uc790\uc2e0\ub4e4\uc758 \uc758\uacac\uc774 \ubc30\uc81c\ub418\uc5c8\ub2e4\ub294 \uc0ac\uc2e4\uc5d0 \ubd84\uac1c\ud558\ub294 \uc0ac\ub78c\ub4e4\ub3c4 \ub9ce\uc558\ub2e4. \uc2e0\uacf5\ud56d \uc720\uce58\uc640 \uad00\ub828\ud574\uc11c \uc8fc\ubbfc\ub4e4\uacfc\uc758 \ub300\ud654\uac00 \ub354 \ud544\uc694\ud558\ub2e4\ub294 \uc9c0\uc801\uc774\ub2e4. \ud558\uc9c0\ub9cc \ubc00\uc591\uce21\uc740 \ud6c4\ubcf4\uc9c0 \uc8fc\ubbfc\ub4e4\uacfc \ub300\ud654\ub97c \ud560 \ub2e8\uacc4\ub294 \uc544\ub2c8\ub77c\uace0 \uc798\ub77c \ub9d0\ud55c\ub2e4. \uc2e0\uacf5\ud56d \uc704\uce58\uac00 \ud655\uc815\ub418\uba74 \uadf8 \ub54c\ubd80\ud130 \uc774\uc57c\uae30\ub97c \ud55c\ub2e4\ub294 \uac83\uc774\ub2e4. \ubd80\uc0b0\uad11\uc5ed\uc2dc\ub294 \"\ud604\uc7ac \uc5b4\uc5c5\uc744 \ud558\uace0 \uc788\ub294 \uc5b4\ubbfc\ub4e4\uc5d0\uac8c\ub294 \uc801\uc808\ud55c \ubcf4\uc0c1\uc774 \ub420 \uc218 \uc788\ub3c4\ub85d \uc2dc\uac00 \uc815\ubd80\uc5d0 \uc694\uad6c\ub97c \ud558\uace0 \uc5b4\ubbfc\ud3b8\uc5d0\uc11c \uc77c\uc744 \ud558\uaca0\ub2e4\"\uace0 \ub9d0\ud588\ub2e4.", "paragraph_answer": "\ubc00\uc591\uc2dc \ud558\ub0a8\uc74d\uc5d0 8,000\uc5ec\uba85 \uc774 \ub18d\uc0ac\ub97c \uc9c0\uc73c\uba70 \uc9c0\ub0b4\uace0 \uc788\ub2e4. \ub545\uc774 \uae30\ub984\uc838 \uac10\uc790\uc640 \ub538\uae30\uac00 \ub9ce\uc774 \ub09c\ub2e4. \ud558\uc9c0\ub9cc \uc2e0\uacf5\ud56d\uc774 \ub4e4\uc5b4\uc11c\uba74 \ub354\uc774\uc0c1 \ub18d\uc0ac\ub97c \uc9c0\uc744 \uc218 \uc5c6\uac8c \ub41c\ub2e4. \uc774 \ub54c\ubb38\uc5d0 \uc77c\ubd80 \uc8fc\ubbfc\ub4e4\uc740 \uc2e0\uacf5\ud56d \uc720\uce58\ub97c \ubc18\ub300\ud558\uace0 \uc788\uc5c8\ub2e4. \ud55c \uc8fc\ubbfc\uc740 \"\ub18d\uc0ac\uc9d3\uace0 \uc788\ub294\ub370 \ub098\uac00\ub77c\uace0 \ud558\uba74 \ubcf4\uc0c1\ub3c4 \uc5c6\uace0 \ubc14\ub85c \uac70\uc9c0\ub2e4\"\ub77c\uba70 \"\uc6b0\ub9ac\ub294 \ud3c9\uc0c1 \ube44\ud589\uae30 \ud0c8\uc77c \ub450\ubc88\ub3c4 \uc5c6\ub294\ub370\"\ub77c\uba70 \uaca9\ubd84\ud588\ub2e4. \uac10\uc790\ubc2d\uc744 \uc6b4\uc601\ud558\ub294 \ub2e4\ub978 \uc8fc\ubbfc\ub3c4 \"\uac10\uc790\ubc2d\uc5d0\uc11c \ud55c\ub2e4\ub808 200~205\ub9cc\uc6d0\uc529 \uc218\uc775\uc774 \ub098\uc624\ub294\ub370 \uc774\uac83\uc744 \ucca0\uac70\ud558\uba74 \uc815\ubd80\uc5d0\uc11c 200\ub9cc\uc6d0 \uc774\uc0c1 \uc8fc\uaca0\ub098\"\ub77c\uba70 \"\ub2f9\uc7a5 \ub098\uac00\uba74 1~2\ub144\uc740 \uba39\uace0\uc0b4\uaca0\uc9c0\ub9cc \uadf8 \uc774\ud6c4\ub294 \ub300\ucc45\uc774 \uc5c6\ub2e4\"\ub77c\uba70 \ubc18\ub300\uc785\uc7a5\uc744 \ubd84\uba85\ud788 \ud588\ub2e4. \uc2e0\uacf5\ud56d \ud6c4\ubcf4 \uc9c0\uc5ed \uc8fc\ubbfc\ub4e4\uc740 \uc2e0\uacf5\ud56d \uc720\uce58\uc640 \uad00\ub828\ud574\uc11c \uc790\uc2e0\ub4e4\uc758 \uc758\uacac\uc774 \ubc30\uc81c\ub418\uc5c8\ub2e4\ub294 \uc0ac\uc2e4\uc5d0 \ubd84\uac1c\ud558\ub294 \uc0ac\ub78c\ub4e4\ub3c4 \ub9ce\uc558\ub2e4. \uc2e0\uacf5\ud56d \uc720\uce58\uc640 \uad00\ub828\ud574\uc11c \uc8fc\ubbfc\ub4e4\uacfc\uc758 \ub300\ud654\uac00 \ub354 \ud544\uc694\ud558\ub2e4\ub294 \uc9c0\uc801\uc774\ub2e4. \ud558\uc9c0\ub9cc \ubc00\uc591\uce21\uc740 \ud6c4\ubcf4\uc9c0 \uc8fc\ubbfc\ub4e4\uacfc \ub300\ud654\ub97c \ud560 \ub2e8\uacc4\ub294 \uc544\ub2c8\ub77c\uace0 \uc798\ub77c \ub9d0\ud55c\ub2e4. \uc2e0\uacf5\ud56d \uc704\uce58\uac00 \ud655\uc815\ub418\uba74 \uadf8 \ub54c\ubd80\ud130 \uc774\uc57c\uae30\ub97c \ud55c\ub2e4\ub294 \uac83\uc774\ub2e4. \ubd80\uc0b0\uad11\uc5ed\uc2dc\ub294 \"\ud604\uc7ac \uc5b4\uc5c5\uc744 \ud558\uace0 \uc788\ub294 \uc5b4\ubbfc\ub4e4\uc5d0\uac8c\ub294 \uc801\uc808\ud55c \ubcf4\uc0c1\uc774 \ub420 \uc218 \uc788\ub3c4\ub85d \uc2dc\uac00 \uc815\ubd80\uc5d0 \uc694\uad6c\ub97c \ud558\uace0 \uc5b4\ubbfc\ud3b8\uc5d0\uc11c \uc77c\uc744 \ud558\uaca0\ub2e4\"\uace0 \ub9d0\ud588\ub2e4.", "sentence_answer": "\ubc00\uc591\uc2dc \ud558\ub0a8\uc74d\uc5d0 8,000\uc5ec\uba85 \uc774 \ub18d\uc0ac\ub97c \uc9c0\uc73c\uba70 \uc9c0\ub0b4\uace0 \uc788\ub2e4.", "paragraph_id": "6524243-6-0", "paragraph_question": "question: \ubc00\uc591\uc2dc \ud558\ub0a8\uc74d\uc5d0\uc11c \ub18d\uc0ac\ub97c \uc9c0\uc73c\uba70 \uc9c0\ub0b4\ub294 \uc778\uad6c\uc218\ub294?, context: \ubc00\uc591\uc2dc \ud558\ub0a8\uc74d\uc5d0 8,000\uc5ec\uba85\uc774 \ub18d\uc0ac\ub97c \uc9c0\uc73c\uba70 \uc9c0\ub0b4\uace0 \uc788\ub2e4. \ub545\uc774 \uae30\ub984\uc838 \uac10\uc790\uc640 \ub538\uae30\uac00 \ub9ce\uc774 \ub09c\ub2e4. \ud558\uc9c0\ub9cc \uc2e0\uacf5\ud56d\uc774 \ub4e4\uc5b4\uc11c\uba74 \ub354\uc774\uc0c1 \ub18d\uc0ac\ub97c \uc9c0\uc744 \uc218 \uc5c6\uac8c \ub41c\ub2e4. \uc774 \ub54c\ubb38\uc5d0 \uc77c\ubd80 \uc8fc\ubbfc\ub4e4\uc740 \uc2e0\uacf5\ud56d \uc720\uce58\ub97c \ubc18\ub300\ud558\uace0 \uc788\uc5c8\ub2e4. \ud55c \uc8fc\ubbfc\uc740 \"\ub18d\uc0ac\uc9d3\uace0 \uc788\ub294\ub370 \ub098\uac00\ub77c\uace0 \ud558\uba74 \ubcf4\uc0c1\ub3c4 \uc5c6\uace0 \ubc14\ub85c \uac70\uc9c0\ub2e4\"\ub77c\uba70 \"\uc6b0\ub9ac\ub294 \ud3c9\uc0c1 \ube44\ud589\uae30 \ud0c8\uc77c \ub450\ubc88\ub3c4 \uc5c6\ub294\ub370\"\ub77c\uba70 \uaca9\ubd84\ud588\ub2e4. \uac10\uc790\ubc2d\uc744 \uc6b4\uc601\ud558\ub294 \ub2e4\ub978 \uc8fc\ubbfc\ub3c4 \"\uac10\uc790\ubc2d\uc5d0\uc11c \ud55c\ub2e4\ub808 200~205\ub9cc\uc6d0\uc529 \uc218\uc775\uc774 \ub098\uc624\ub294\ub370 \uc774\uac83\uc744 \ucca0\uac70\ud558\uba74 \uc815\ubd80\uc5d0\uc11c 200\ub9cc\uc6d0 \uc774\uc0c1 \uc8fc\uaca0\ub098\"\ub77c\uba70 \"\ub2f9\uc7a5 \ub098\uac00\uba74 1~2\ub144\uc740 \uba39\uace0\uc0b4\uaca0\uc9c0\ub9cc \uadf8 \uc774\ud6c4\ub294 \ub300\ucc45\uc774 \uc5c6\ub2e4\"\ub77c\uba70 \ubc18\ub300\uc785\uc7a5\uc744 \ubd84\uba85\ud788 \ud588\ub2e4. \uc2e0\uacf5\ud56d \ud6c4\ubcf4 \uc9c0\uc5ed \uc8fc\ubbfc\ub4e4\uc740 \uc2e0\uacf5\ud56d \uc720\uce58\uc640 \uad00\ub828\ud574\uc11c \uc790\uc2e0\ub4e4\uc758 \uc758\uacac\uc774 \ubc30\uc81c\ub418\uc5c8\ub2e4\ub294 \uc0ac\uc2e4\uc5d0 \ubd84\uac1c\ud558\ub294 \uc0ac\ub78c\ub4e4\ub3c4 \ub9ce\uc558\ub2e4. \uc2e0\uacf5\ud56d \uc720\uce58\uc640 \uad00\ub828\ud574\uc11c \uc8fc\ubbfc\ub4e4\uacfc\uc758 \ub300\ud654\uac00 \ub354 \ud544\uc694\ud558\ub2e4\ub294 \uc9c0\uc801\uc774\ub2e4. \ud558\uc9c0\ub9cc \ubc00\uc591\uce21\uc740 \ud6c4\ubcf4\uc9c0 \uc8fc\ubbfc\ub4e4\uacfc \ub300\ud654\ub97c \ud560 \ub2e8\uacc4\ub294 \uc544\ub2c8\ub77c\uace0 \uc798\ub77c \ub9d0\ud55c\ub2e4. \uc2e0\uacf5\ud56d \uc704\uce58\uac00 \ud655\uc815\ub418\uba74 \uadf8 \ub54c\ubd80\ud130 \uc774\uc57c\uae30\ub97c \ud55c\ub2e4\ub294 \uac83\uc774\ub2e4. \ubd80\uc0b0\uad11\uc5ed\uc2dc\ub294 \"\ud604\uc7ac \uc5b4\uc5c5\uc744 \ud558\uace0 \uc788\ub294 \uc5b4\ubbfc\ub4e4\uc5d0\uac8c\ub294 \uc801\uc808\ud55c \ubcf4\uc0c1\uc774 \ub420 \uc218 \uc788\ub3c4\ub85d \uc2dc\uac00 \uc815\ubd80\uc5d0 \uc694\uad6c\ub97c \ud558\uace0 \uc5b4\ubbfc\ud3b8\uc5d0\uc11c \uc77c\uc744 \ud558\uaca0\ub2e4\"\uace0 \ub9d0\ud588\ub2e4."} {"question": "\uc601\uc870 49\ub144 \uc2e0\ubb38\uace0\ub97c \uc6b8\ub824 \uc18c\ub839\uc6d0\uc744 \ub2a5\uc73c\ub85c \ubd09\ud558\uc790\uace0 \uccad\ud55c \uc0ac\ub78c\uc758 \uc774\ub984\uc740?", "paragraph": "\uadf8\ub140\uc758 \uc544\ub4e4 \uc5f0\uc789\uad70\uc740 \uc655\uc138\uc81c\ub97c \uac70\uccd0 \uc655\uc73c\ub85c \ub4f1\uadf9\ud558\ub2c8, \ubc14\ub85c \uc870\uc120\uc758 \uc81c21\ub300 \uc655 \uc601\uc870(\u82f1\u7956)\uc774\ub2e4. \uc601\uc870\ub294 \uc989\uc704 \uc6d0\ub144, \uc5b4\uba38\ub2c8 \ucd5c\uc528\uc758 \uc0ac\ub2f9\uc744 \uc9c0\uc5b4 \uc219\ube48\ubb18(\u6dd1\u5b2a\u5edf)\ub77c \ud558\uc600\uace0, \uc601\uc87020\ub144(1744\ub144) \uc721\uc0c1\ubb18(\u6bd3\u7965\u5edf)\ub77c\uace0 \uc62c\ub838\ub2e4\uac00 \ub2e4\uc2dc \uc601\uc870 29\ub144(1753\ub144) \uc721\uc0c1\uad81(\u6bd3\u7965\u5bae)\uc73c\ub85c \uc2b9\uaca9\uc2dc\ucf30\ub2e4. \uc721\uc0c1\uad81\uc740 \ud604\uc7ac \uce60\uad81\uc5d0 \ud569\uc0ac\ub418\uc5b4 \uc788\ub2e4. \ubb18\uc18c \ub610\ud55c \uc601\uc870 20\ub144\uc5d0 \uc18c\ub839\ubb18(\u662d\u5be7\u5893)\ub77c\uace0 \uc62c\ub838\ub2e4\uac00 29\ub144\uc5d0 \uc18c\ub839\uc6d0(\u662d\u5be7\u5712)\uc73c\ub85c \ub2e4\uc2dc \uc2b9\uaca9\uc2dc\ucf30\uc73c\uba70, \uc0ac\ub2f9\uacfc \ubb34\ub364\uc5d0 \uad81\ud638\uc640 \uc6d0\ud638\ub97c \uc62c\ub9b4 \ub54c \ud568\uaed8 \ud654\uacbd(\u548c\u656c)\uc758 \uc2dc\ud638\ub97c \uc62c\ub838\ub2e4. \ud6c4\uc77c \uc5ec\ub7ec \ucc28\ub840\uc5d0 \uac78\uccd0 \ud718\ub355\uc548\uc21c\uc218\ubcf5(\u5fbd\u5fb7\u5b89\u7d14\u7d8f\u798f)\uc758 \uc874\ud638\uac00 \ub354 \uc62c\ub824\uc84c\ub2e4. \uc601\uc870 29\ub144 \uc721\uc0c1\uad81\uc5d0 \uad00\uc81c(\u5b98\u796d)\ub97c \uc9c0\ub0b8 \ud6c4, \uba85\ub098\ub77c \ud6a8\uc885(\u5b5d\u5b97)\uc774 \uc0dd\ubaa8\ub97c \ucd94\uc874\ud558\uc5ec \ubaa8\ube44(\u67d0\u59a3)\ub77c \ud558\uc600\ub2e4\uac00 \ub2e4\uc2dc \ud669\ubaa8(\u7687\u6bcd)\ub77c\uace0 \uce6d\ud55c \uc608\uc5d0 \ub530\ub77c \uc219\ube48\uc740 \uc655\uc758 \uc0ac\uce5c(\u79c1\u89aa)\uc774 \uc544\ub2cc \uc120\ube44(\u5148\u59a3)\ub85c \uc815\ud638\ub418\uc5c8\ub2e4. \uc601\uc870\ub294 \uc0b4\uc544\uc0dd\uc804 '\uc0ac\uce5c\uc740 \ud56d\uc0c1 \uc18c\uc2ec(\u5c0f\u5fc3)\ud558\uace0 \uc2e0\uc911\ud558\uc600\ub2e4.'\uace0 \uc790\uc8fc \ud68c\uace0\ud588\ub2e4. \uc601\uc870 49\ub144, \uc6a9\uac15\uc758 \ubc15\ud765\uc870\uac00 \uc2e0\ubb38\uace0\ub97c \uc6b8\ub824 \uc18c\ub839\uc6d0\uc744 \ub2a5\uc73c\ub85c \ubd09\ud558\ub294 \uc77c\uc744 \uccad\ud588\uc73c\ub098 \uc2dd\ub7c9\uc744 \uc8fc\uc5b4 \ucad3\uc544\ubcf4\ub0c8\ub2e4.", "answer": "\ubc15\ud765\uc870", "sentence": "\uc601\uc870 49\ub144, \uc6a9\uac15\uc758 \ubc15\ud765\uc870 \uac00 \uc2e0\ubb38\uace0\ub97c \uc6b8\ub824 \uc18c\ub839\uc6d0\uc744 \ub2a5\uc73c\ub85c \ubd09\ud558\ub294 \uc77c\uc744 \uccad\ud588\uc73c\ub098 \uc2dd\ub7c9\uc744 \uc8fc\uc5b4 \ucad3\uc544\ubcf4\ub0c8\ub2e4.", "paragraph_sentence": "\uadf8\ub140\uc758 \uc544\ub4e4 \uc5f0\uc789\uad70\uc740 \uc655\uc138\uc81c\ub97c \uac70\uccd0 \uc655\uc73c\ub85c \ub4f1\uadf9\ud558\ub2c8, \ubc14\ub85c \uc870\uc120\uc758 \uc81c21\ub300 \uc655 \uc601\uc870(\u82f1\u7956)\uc774\ub2e4. \uc601\uc870\ub294 \uc989\uc704 \uc6d0\ub144, \uc5b4\uba38\ub2c8 \ucd5c\uc528\uc758 \uc0ac\ub2f9\uc744 \uc9c0\uc5b4 \uc219\ube48\ubb18(\u6dd1\u5b2a\u5edf)\ub77c \ud558\uc600\uace0, \uc601\uc87020\ub144(1744\ub144) \uc721\uc0c1\ubb18(\u6bd3\u7965\u5edf)\ub77c\uace0 \uc62c\ub838\ub2e4\uac00 \ub2e4\uc2dc \uc601\uc870 29\ub144(1753\ub144) \uc721\uc0c1\uad81(\u6bd3\u7965\u5bae)\uc73c\ub85c \uc2b9\uaca9\uc2dc\ucf30\ub2e4. \uc721\uc0c1\uad81\uc740 \ud604\uc7ac \uce60\uad81\uc5d0 \ud569\uc0ac\ub418\uc5b4 \uc788\ub2e4. \ubb18\uc18c \ub610\ud55c \uc601\uc870 20\ub144\uc5d0 \uc18c\ub839\ubb18(\u662d\u5be7\u5893)\ub77c\uace0 \uc62c\ub838\ub2e4\uac00 29\ub144\uc5d0 \uc18c\ub839\uc6d0(\u662d\u5be7\u5712)\uc73c\ub85c \ub2e4\uc2dc \uc2b9\uaca9\uc2dc\ucf30\uc73c\uba70, \uc0ac\ub2f9\uacfc \ubb34\ub364\uc5d0 \uad81\ud638\uc640 \uc6d0\ud638\ub97c \uc62c\ub9b4 \ub54c \ud568\uaed8 \ud654\uacbd(\u548c\u656c)\uc758 \uc2dc\ud638\ub97c \uc62c\ub838\ub2e4. \ud6c4\uc77c \uc5ec\ub7ec \ucc28\ub840\uc5d0 \uac78\uccd0 \ud718\ub355\uc548\uc21c\uc218\ubcf5(\u5fbd\u5fb7\u5b89\u7d14\u7d8f\u798f)\uc758 \uc874\ud638\uac00 \ub354 \uc62c\ub824\uc84c\ub2e4. \uc601\uc870 29\ub144 \uc721\uc0c1\uad81\uc5d0 \uad00\uc81c(\u5b98\u796d)\ub97c \uc9c0\ub0b8 \ud6c4, \uba85\ub098\ub77c \ud6a8\uc885(\u5b5d\u5b97)\uc774 \uc0dd\ubaa8\ub97c \ucd94\uc874\ud558\uc5ec \ubaa8\ube44(\u67d0\u59a3)\ub77c \ud558\uc600\ub2e4\uac00 \ub2e4\uc2dc \ud669\ubaa8(\u7687\u6bcd)\ub77c\uace0 \uce6d\ud55c \uc608\uc5d0 \ub530\ub77c \uc219\ube48\uc740 \uc655\uc758 \uc0ac\uce5c(\u79c1\u89aa)\uc774 \uc544\ub2cc \uc120\ube44(\u5148\u59a3)\ub85c \uc815\ud638\ub418\uc5c8\ub2e4. \uc601\uc870\ub294 \uc0b4\uc544\uc0dd\uc804 '\uc0ac\uce5c\uc740 \ud56d\uc0c1 \uc18c\uc2ec(\u5c0f\u5fc3)\ud558\uace0 \uc2e0\uc911\ud558\uc600\ub2e4. '\uace0 \uc790\uc8fc \ud68c\uace0\ud588\ub2e4. \uc601\uc870 49\ub144, \uc6a9\uac15\uc758 \ubc15\ud765\uc870 \uac00 \uc2e0\ubb38\uace0\ub97c \uc6b8\ub824 \uc18c\ub839\uc6d0\uc744 \ub2a5\uc73c\ub85c \ubd09\ud558\ub294 \uc77c\uc744 \uccad\ud588\uc73c\ub098 \uc2dd\ub7c9\uc744 \uc8fc\uc5b4 \ucad3\uc544\ubcf4\ub0c8\ub2e4. ", "paragraph_answer": "\uadf8\ub140\uc758 \uc544\ub4e4 \uc5f0\uc789\uad70\uc740 \uc655\uc138\uc81c\ub97c \uac70\uccd0 \uc655\uc73c\ub85c \ub4f1\uadf9\ud558\ub2c8, \ubc14\ub85c \uc870\uc120\uc758 \uc81c21\ub300 \uc655 \uc601\uc870(\u82f1\u7956)\uc774\ub2e4. \uc601\uc870\ub294 \uc989\uc704 \uc6d0\ub144, \uc5b4\uba38\ub2c8 \ucd5c\uc528\uc758 \uc0ac\ub2f9\uc744 \uc9c0\uc5b4 \uc219\ube48\ubb18(\u6dd1\u5b2a\u5edf)\ub77c \ud558\uc600\uace0, \uc601\uc87020\ub144(1744\ub144) \uc721\uc0c1\ubb18(\u6bd3\u7965\u5edf)\ub77c\uace0 \uc62c\ub838\ub2e4\uac00 \ub2e4\uc2dc \uc601\uc870 29\ub144(1753\ub144) \uc721\uc0c1\uad81(\u6bd3\u7965\u5bae)\uc73c\ub85c \uc2b9\uaca9\uc2dc\ucf30\ub2e4. \uc721\uc0c1\uad81\uc740 \ud604\uc7ac \uce60\uad81\uc5d0 \ud569\uc0ac\ub418\uc5b4 \uc788\ub2e4. \ubb18\uc18c \ub610\ud55c \uc601\uc870 20\ub144\uc5d0 \uc18c\ub839\ubb18(\u662d\u5be7\u5893)\ub77c\uace0 \uc62c\ub838\ub2e4\uac00 29\ub144\uc5d0 \uc18c\ub839\uc6d0(\u662d\u5be7\u5712)\uc73c\ub85c \ub2e4\uc2dc \uc2b9\uaca9\uc2dc\ucf30\uc73c\uba70, \uc0ac\ub2f9\uacfc \ubb34\ub364\uc5d0 \uad81\ud638\uc640 \uc6d0\ud638\ub97c \uc62c\ub9b4 \ub54c \ud568\uaed8 \ud654\uacbd(\u548c\u656c)\uc758 \uc2dc\ud638\ub97c \uc62c\ub838\ub2e4. \ud6c4\uc77c \uc5ec\ub7ec \ucc28\ub840\uc5d0 \uac78\uccd0 \ud718\ub355\uc548\uc21c\uc218\ubcf5(\u5fbd\u5fb7\u5b89\u7d14\u7d8f\u798f)\uc758 \uc874\ud638\uac00 \ub354 \uc62c\ub824\uc84c\ub2e4. \uc601\uc870 29\ub144 \uc721\uc0c1\uad81\uc5d0 \uad00\uc81c(\u5b98\u796d)\ub97c \uc9c0\ub0b8 \ud6c4, \uba85\ub098\ub77c \ud6a8\uc885(\u5b5d\u5b97)\uc774 \uc0dd\ubaa8\ub97c \ucd94\uc874\ud558\uc5ec \ubaa8\ube44(\u67d0\u59a3)\ub77c \ud558\uc600\ub2e4\uac00 \ub2e4\uc2dc \ud669\ubaa8(\u7687\u6bcd)\ub77c\uace0 \uce6d\ud55c \uc608\uc5d0 \ub530\ub77c \uc219\ube48\uc740 \uc655\uc758 \uc0ac\uce5c(\u79c1\u89aa)\uc774 \uc544\ub2cc \uc120\ube44(\u5148\u59a3)\ub85c \uc815\ud638\ub418\uc5c8\ub2e4. \uc601\uc870\ub294 \uc0b4\uc544\uc0dd\uc804 '\uc0ac\uce5c\uc740 \ud56d\uc0c1 \uc18c\uc2ec(\u5c0f\u5fc3)\ud558\uace0 \uc2e0\uc911\ud558\uc600\ub2e4.'\uace0 \uc790\uc8fc \ud68c\uace0\ud588\ub2e4. \uc601\uc870 49\ub144, \uc6a9\uac15\uc758 \ubc15\ud765\uc870 \uac00 \uc2e0\ubb38\uace0\ub97c \uc6b8\ub824 \uc18c\ub839\uc6d0\uc744 \ub2a5\uc73c\ub85c \ubd09\ud558\ub294 \uc77c\uc744 \uccad\ud588\uc73c\ub098 \uc2dd\ub7c9\uc744 \uc8fc\uc5b4 \ucad3\uc544\ubcf4\ub0c8\ub2e4.", "sentence_answer": "\uc601\uc870 49\ub144, \uc6a9\uac15\uc758 \ubc15\ud765\uc870 \uac00 \uc2e0\ubb38\uace0\ub97c \uc6b8\ub824 \uc18c\ub839\uc6d0\uc744 \ub2a5\uc73c\ub85c \ubd09\ud558\ub294 \uc77c\uc744 \uccad\ud588\uc73c\ub098 \uc2dd\ub7c9\uc744 \uc8fc\uc5b4 \ucad3\uc544\ubcf4\ub0c8\ub2e4.", "paragraph_id": "6518926-3-2", "paragraph_question": "question: \uc601\uc870 49\ub144 \uc2e0\ubb38\uace0\ub97c \uc6b8\ub824 \uc18c\ub839\uc6d0\uc744 \ub2a5\uc73c\ub85c \ubd09\ud558\uc790\uace0 \uccad\ud55c \uc0ac\ub78c\uc758 \uc774\ub984\uc740?, context: \uadf8\ub140\uc758 \uc544\ub4e4 \uc5f0\uc789\uad70\uc740 \uc655\uc138\uc81c\ub97c \uac70\uccd0 \uc655\uc73c\ub85c \ub4f1\uadf9\ud558\ub2c8, \ubc14\ub85c \uc870\uc120\uc758 \uc81c21\ub300 \uc655 \uc601\uc870(\u82f1\u7956)\uc774\ub2e4. \uc601\uc870\ub294 \uc989\uc704 \uc6d0\ub144, \uc5b4\uba38\ub2c8 \ucd5c\uc528\uc758 \uc0ac\ub2f9\uc744 \uc9c0\uc5b4 \uc219\ube48\ubb18(\u6dd1\u5b2a\u5edf)\ub77c \ud558\uc600\uace0, \uc601\uc87020\ub144(1744\ub144) \uc721\uc0c1\ubb18(\u6bd3\u7965\u5edf)\ub77c\uace0 \uc62c\ub838\ub2e4\uac00 \ub2e4\uc2dc \uc601\uc870 29\ub144(1753\ub144) \uc721\uc0c1\uad81(\u6bd3\u7965\u5bae)\uc73c\ub85c \uc2b9\uaca9\uc2dc\ucf30\ub2e4. \uc721\uc0c1\uad81\uc740 \ud604\uc7ac \uce60\uad81\uc5d0 \ud569\uc0ac\ub418\uc5b4 \uc788\ub2e4. \ubb18\uc18c \ub610\ud55c \uc601\uc870 20\ub144\uc5d0 \uc18c\ub839\ubb18(\u662d\u5be7\u5893)\ub77c\uace0 \uc62c\ub838\ub2e4\uac00 29\ub144\uc5d0 \uc18c\ub839\uc6d0(\u662d\u5be7\u5712)\uc73c\ub85c \ub2e4\uc2dc \uc2b9\uaca9\uc2dc\ucf30\uc73c\uba70, \uc0ac\ub2f9\uacfc \ubb34\ub364\uc5d0 \uad81\ud638\uc640 \uc6d0\ud638\ub97c \uc62c\ub9b4 \ub54c \ud568\uaed8 \ud654\uacbd(\u548c\u656c)\uc758 \uc2dc\ud638\ub97c \uc62c\ub838\ub2e4. \ud6c4\uc77c \uc5ec\ub7ec \ucc28\ub840\uc5d0 \uac78\uccd0 \ud718\ub355\uc548\uc21c\uc218\ubcf5(\u5fbd\u5fb7\u5b89\u7d14\u7d8f\u798f)\uc758 \uc874\ud638\uac00 \ub354 \uc62c\ub824\uc84c\ub2e4. \uc601\uc870 29\ub144 \uc721\uc0c1\uad81\uc5d0 \uad00\uc81c(\u5b98\u796d)\ub97c \uc9c0\ub0b8 \ud6c4, \uba85\ub098\ub77c \ud6a8\uc885(\u5b5d\u5b97)\uc774 \uc0dd\ubaa8\ub97c \ucd94\uc874\ud558\uc5ec \ubaa8\ube44(\u67d0\u59a3)\ub77c \ud558\uc600\ub2e4\uac00 \ub2e4\uc2dc \ud669\ubaa8(\u7687\u6bcd)\ub77c\uace0 \uce6d\ud55c \uc608\uc5d0 \ub530\ub77c \uc219\ube48\uc740 \uc655\uc758 \uc0ac\uce5c(\u79c1\u89aa)\uc774 \uc544\ub2cc \uc120\ube44(\u5148\u59a3)\ub85c \uc815\ud638\ub418\uc5c8\ub2e4. \uc601\uc870\ub294 \uc0b4\uc544\uc0dd\uc804 '\uc0ac\uce5c\uc740 \ud56d\uc0c1 \uc18c\uc2ec(\u5c0f\u5fc3)\ud558\uace0 \uc2e0\uc911\ud558\uc600\ub2e4.'\uace0 \uc790\uc8fc \ud68c\uace0\ud588\ub2e4. \uc601\uc870 49\ub144, \uc6a9\uac15\uc758 \ubc15\ud765\uc870\uac00 \uc2e0\ubb38\uace0\ub97c \uc6b8\ub824 \uc18c\ub839\uc6d0\uc744 \ub2a5\uc73c\ub85c \ubd09\ud558\ub294 \uc77c\uc744 \uccad\ud588\uc73c\ub098 \uc2dd\ub7c9\uc744 \uc8fc\uc5b4 \ucad3\uc544\ubcf4\ub0c8\ub2e4."} {"question": "\ud55c\uad6d\uce5c\uc6b0\ud68c \ucc3d\ub9bd \ubc1c\uae30\ud68c\uac00 \uc5f4\ub9b0 \ub0a0\uc740 \uc5b8\uc81c\uc778\uac00?", "paragraph": "\uc9c0\uc9c0\uc790\ub97c \ubaa8\uc740 \uc11c\uc7ac\ud544\uc740 \ud1b0\ud0a8\uc2a4, \ubca0\ub124\ub515\ud2b8 \ub4f1\uacfc \ud568\uaed8 1919\ub144 5\uc6d4 2\uc77c \ud544\ub77c\ub378\ud53c\uc544 \uc2dc \uc2dc\ub0b4 \uc2dc\ud2f0\ud074\ub7fd \ube4c\ub529\uc5d0\uc11c \ubbf8\uad6d\ub0b4 \uce5c\ud55c\ud30c \uc885\uad50\uacc4, \uad50\uc721\uacc4, \uc2e4\uc5c5\uacc4\uc758 \ubd84\uc57c\ubcc4 \uc800\uba85\uc778\uc0ac 22\uba85\uc744 \ucd08\uccad\ud558\uc5ec \ud55c\uad6d\uce5c\uc6b0\ud68c \ucc3d\ub9bd \ubc1c\uae30\ud68c\ub97c \uc5f4\uc5c8\ub2e4. \uc5ec\uae30\uc11c \uc11c\uc7ac\ud544\uc740 \uc6b0\uc120 \ubbf8\uad6d \uc120\uad50\uc0ac\ub4e4\uc774 \uc870\uc120\uc5d0 \uc758\ud559\uacfc \uae30\ub3c5\uad50, \uc2e0 \ubb38\uba85\uc758 \uc804\ud30c\uc640 \uc704\uc0dd\uc801\uc778 \uc591\uc625\uc9d1 \uac74\uc124 \ub4f1\uc758 \ud61c\ud0dd\uc744 \uc900 \uac83\uc5d0 \ub300\ud574 \uae4a\uc740 \uac10\uc0ac\ub97c \ud45c\ud55c\ub2e4\uace0 \ud558\uace0, \uc774\uc5b4 '\ubbf8\uad6d\uad6d\ubbfc\uc774 \ud55c\uad6d\uc5d0 \uae30\ub3c5\uad50\ub97c \uc804\ud30c\ud558\uc5ec \uc790\uc720\uc640 \ub3c5\ub9bd\uc744 \ud55c\uc778\ub4e4\uc5d0\uac8c \uac00\ub974\ucce4\ub294\ub370, \uc9c0\uae08 \uc870\uc120\uc778\ub4e4\uc774 \uc774\ub97c \uc704\ud574 \uc2f8\uc6b0\ub294 \uac83\uc744 \ubbf8\uad6d\uad6d\ubbfc\uc774 \ubaa8\ub978 \uccb4 \ud55c\ub2e4\uba74 \uc774\uac83\uc740 \ud06c\uac8c \ubaa8\uc21c\ub418\ub294 \uc77c\uc774\uc694, \uc9c0\uae08\uaecf \ubbf8\uad6d \uc120\uad50\uc0ac\ub4e4\uacfc \uc758\uc0ac\ub4e4\uc758 \uc778\ub3c4\uc801 \uc9c0\uc6d0\uc740 \uc218\ud3ec\ub85c \ub3cc\uc544\uac00\ub294 \uac83\uc774\uba70, \ub3c4\ub9ac\uac00 \uc544\ub2c8\ub77c\ub294 \ub0b4\uc6a9\uc73c\ub85c \ud55c\uad6d\uce5c\uc6b0\ud68c\uc758 \uc124\ub9bd \ud544\uc694\uc131'\uc744 \uc5ed\uc124\ud588\ub2e4. \ub610\ud55c \uc11c\uc7ac\ud544\uc740 \uc870\uc120\uc778\ub4e4\uc740 \uc740\ud61c\ub97c \uc544\ub294 \uc0ac\ub78c\ub4e4\uc774\ub2c8 \ubbf8\uad6d\uc778\ub4e4\uc774 \ud55c\uad6d\uc758 \ub3c5\ub9bd\uc5d0 \uc801\uadf9 \uc9c0\uc9c0\ub97c \ubcf4\ub0b8\ub2e4\uba74 \ud55c\uad6d \uc5ed\uc2dc \ubbf8\uad6d\uc744 \ubc30\uc2e0\ud558\ub294 \uc77c\uc740 \uc5c6\uc744 \uac83\uc784\uc744 \ud638\uc18c\ud558\uc600\ub2e4.", "answer": "1919\ub144 5\uc6d4 2\uc77c", "sentence": "\uc9c0\uc9c0\uc790\ub97c \ubaa8\uc740 \uc11c\uc7ac\ud544\uc740 \ud1b0\ud0a8\uc2a4, \ubca0\ub124\ub515\ud2b8 \ub4f1\uacfc \ud568\uaed8 1919\ub144 5\uc6d4 2\uc77c \ud544\ub77c\ub378\ud53c\uc544 \uc2dc \uc2dc\ub0b4 \uc2dc\ud2f0\ud074\ub7fd \ube4c\ub529\uc5d0\uc11c \ubbf8\uad6d\ub0b4 \uce5c\ud55c\ud30c \uc885\uad50\uacc4, \uad50\uc721\uacc4, \uc2e4\uc5c5\uacc4\uc758 \ubd84\uc57c\ubcc4 \uc800\uba85\uc778\uc0ac 22\uba85\uc744 \ucd08\uccad\ud558\uc5ec \ud55c\uad6d\uce5c\uc6b0\ud68c \ucc3d\ub9bd \ubc1c\uae30\ud68c\ub97c \uc5f4\uc5c8\ub2e4.", "paragraph_sentence": " \uc9c0\uc9c0\uc790\ub97c \ubaa8\uc740 \uc11c\uc7ac\ud544\uc740 \ud1b0\ud0a8\uc2a4, \ubca0\ub124\ub515\ud2b8 \ub4f1\uacfc \ud568\uaed8 1919\ub144 5\uc6d4 2\uc77c \ud544\ub77c\ub378\ud53c\uc544 \uc2dc \uc2dc\ub0b4 \uc2dc\ud2f0\ud074\ub7fd \ube4c\ub529\uc5d0\uc11c \ubbf8\uad6d\ub0b4 \uce5c\ud55c\ud30c \uc885\uad50\uacc4, \uad50\uc721\uacc4, \uc2e4\uc5c5\uacc4\uc758 \ubd84\uc57c\ubcc4 \uc800\uba85\uc778\uc0ac 22\uba85\uc744 \ucd08\uccad\ud558\uc5ec \ud55c\uad6d\uce5c\uc6b0\ud68c \ucc3d\ub9bd \ubc1c\uae30\ud68c\ub97c \uc5f4\uc5c8\ub2e4. \uc5ec\uae30\uc11c \uc11c\uc7ac\ud544\uc740 \uc6b0\uc120 \ubbf8\uad6d \uc120\uad50\uc0ac\ub4e4\uc774 \uc870\uc120\uc5d0 \uc758\ud559\uacfc \uae30\ub3c5\uad50, \uc2e0 \ubb38\uba85\uc758 \uc804\ud30c\uc640 \uc704\uc0dd\uc801\uc778 \uc591\uc625\uc9d1 \uac74\uc124 \ub4f1\uc758 \ud61c\ud0dd\uc744 \uc900 \uac83\uc5d0 \ub300\ud574 \uae4a\uc740 \uac10\uc0ac\ub97c \ud45c\ud55c\ub2e4\uace0 \ud558\uace0, \uc774\uc5b4 '\ubbf8\uad6d\uad6d\ubbfc\uc774 \ud55c\uad6d\uc5d0 \uae30\ub3c5\uad50\ub97c \uc804\ud30c\ud558\uc5ec \uc790\uc720\uc640 \ub3c5\ub9bd\uc744 \ud55c\uc778\ub4e4\uc5d0\uac8c \uac00\ub974\ucce4\ub294\ub370, \uc9c0\uae08 \uc870\uc120\uc778\ub4e4\uc774 \uc774\ub97c \uc704\ud574 \uc2f8\uc6b0\ub294 \uac83\uc744 \ubbf8\uad6d\uad6d\ubbfc\uc774 \ubaa8\ub978 \uccb4 \ud55c\ub2e4\uba74 \uc774\uac83\uc740 \ud06c\uac8c \ubaa8\uc21c\ub418\ub294 \uc77c\uc774\uc694, \uc9c0\uae08\uaecf \ubbf8\uad6d \uc120\uad50\uc0ac\ub4e4\uacfc \uc758\uc0ac\ub4e4\uc758 \uc778\ub3c4\uc801 \uc9c0\uc6d0\uc740 \uc218\ud3ec\ub85c \ub3cc\uc544\uac00\ub294 \uac83\uc774\uba70, \ub3c4\ub9ac\uac00 \uc544\ub2c8\ub77c\ub294 \ub0b4\uc6a9\uc73c\ub85c \ud55c\uad6d\uce5c\uc6b0\ud68c\uc758 \uc124\ub9bd \ud544\uc694\uc131'\uc744 \uc5ed\uc124\ud588\ub2e4. \ub610\ud55c \uc11c\uc7ac\ud544\uc740 \uc870\uc120\uc778\ub4e4\uc740 \uc740\ud61c\ub97c \uc544\ub294 \uc0ac\ub78c\ub4e4\uc774\ub2c8 \ubbf8\uad6d\uc778\ub4e4\uc774 \ud55c\uad6d\uc758 \ub3c5\ub9bd\uc5d0 \uc801\uadf9 \uc9c0\uc9c0\ub97c \ubcf4\ub0b8\ub2e4\uba74 \ud55c\uad6d \uc5ed\uc2dc \ubbf8\uad6d\uc744 \ubc30\uc2e0\ud558\ub294 \uc77c\uc740 \uc5c6\uc744 \uac83\uc784\uc744 \ud638\uc18c\ud558\uc600\ub2e4.", "paragraph_answer": "\uc9c0\uc9c0\uc790\ub97c \ubaa8\uc740 \uc11c\uc7ac\ud544\uc740 \ud1b0\ud0a8\uc2a4, \ubca0\ub124\ub515\ud2b8 \ub4f1\uacfc \ud568\uaed8 1919\ub144 5\uc6d4 2\uc77c \ud544\ub77c\ub378\ud53c\uc544 \uc2dc \uc2dc\ub0b4 \uc2dc\ud2f0\ud074\ub7fd \ube4c\ub529\uc5d0\uc11c \ubbf8\uad6d\ub0b4 \uce5c\ud55c\ud30c \uc885\uad50\uacc4, \uad50\uc721\uacc4, \uc2e4\uc5c5\uacc4\uc758 \ubd84\uc57c\ubcc4 \uc800\uba85\uc778\uc0ac 22\uba85\uc744 \ucd08\uccad\ud558\uc5ec \ud55c\uad6d\uce5c\uc6b0\ud68c \ucc3d\ub9bd \ubc1c\uae30\ud68c\ub97c \uc5f4\uc5c8\ub2e4. \uc5ec\uae30\uc11c \uc11c\uc7ac\ud544\uc740 \uc6b0\uc120 \ubbf8\uad6d \uc120\uad50\uc0ac\ub4e4\uc774 \uc870\uc120\uc5d0 \uc758\ud559\uacfc \uae30\ub3c5\uad50, \uc2e0 \ubb38\uba85\uc758 \uc804\ud30c\uc640 \uc704\uc0dd\uc801\uc778 \uc591\uc625\uc9d1 \uac74\uc124 \ub4f1\uc758 \ud61c\ud0dd\uc744 \uc900 \uac83\uc5d0 \ub300\ud574 \uae4a\uc740 \uac10\uc0ac\ub97c \ud45c\ud55c\ub2e4\uace0 \ud558\uace0, \uc774\uc5b4 '\ubbf8\uad6d\uad6d\ubbfc\uc774 \ud55c\uad6d\uc5d0 \uae30\ub3c5\uad50\ub97c \uc804\ud30c\ud558\uc5ec \uc790\uc720\uc640 \ub3c5\ub9bd\uc744 \ud55c\uc778\ub4e4\uc5d0\uac8c \uac00\ub974\ucce4\ub294\ub370, \uc9c0\uae08 \uc870\uc120\uc778\ub4e4\uc774 \uc774\ub97c \uc704\ud574 \uc2f8\uc6b0\ub294 \uac83\uc744 \ubbf8\uad6d\uad6d\ubbfc\uc774 \ubaa8\ub978 \uccb4 \ud55c\ub2e4\uba74 \uc774\uac83\uc740 \ud06c\uac8c \ubaa8\uc21c\ub418\ub294 \uc77c\uc774\uc694, \uc9c0\uae08\uaecf \ubbf8\uad6d \uc120\uad50\uc0ac\ub4e4\uacfc \uc758\uc0ac\ub4e4\uc758 \uc778\ub3c4\uc801 \uc9c0\uc6d0\uc740 \uc218\ud3ec\ub85c \ub3cc\uc544\uac00\ub294 \uac83\uc774\uba70, \ub3c4\ub9ac\uac00 \uc544\ub2c8\ub77c\ub294 \ub0b4\uc6a9\uc73c\ub85c \ud55c\uad6d\uce5c\uc6b0\ud68c\uc758 \uc124\ub9bd \ud544\uc694\uc131'\uc744 \uc5ed\uc124\ud588\ub2e4. \ub610\ud55c \uc11c\uc7ac\ud544\uc740 \uc870\uc120\uc778\ub4e4\uc740 \uc740\ud61c\ub97c \uc544\ub294 \uc0ac\ub78c\ub4e4\uc774\ub2c8 \ubbf8\uad6d\uc778\ub4e4\uc774 \ud55c\uad6d\uc758 \ub3c5\ub9bd\uc5d0 \uc801\uadf9 \uc9c0\uc9c0\ub97c \ubcf4\ub0b8\ub2e4\uba74 \ud55c\uad6d \uc5ed\uc2dc \ubbf8\uad6d\uc744 \ubc30\uc2e0\ud558\ub294 \uc77c\uc740 \uc5c6\uc744 \uac83\uc784\uc744 \ud638\uc18c\ud558\uc600\ub2e4.", "sentence_answer": "\uc9c0\uc9c0\uc790\ub97c \ubaa8\uc740 \uc11c\uc7ac\ud544\uc740 \ud1b0\ud0a8\uc2a4, \ubca0\ub124\ub515\ud2b8 \ub4f1\uacfc \ud568\uaed8 1919\ub144 5\uc6d4 2\uc77c \ud544\ub77c\ub378\ud53c\uc544 \uc2dc \uc2dc\ub0b4 \uc2dc\ud2f0\ud074\ub7fd \ube4c\ub529\uc5d0\uc11c \ubbf8\uad6d\ub0b4 \uce5c\ud55c\ud30c \uc885\uad50\uacc4, \uad50\uc721\uacc4, \uc2e4\uc5c5\uacc4\uc758 \ubd84\uc57c\ubcc4 \uc800\uba85\uc778\uc0ac 22\uba85\uc744 \ucd08\uccad\ud558\uc5ec \ud55c\uad6d\uce5c\uc6b0\ud68c \ucc3d\ub9bd \ubc1c\uae30\ud68c\ub97c \uc5f4\uc5c8\ub2e4.", "paragraph_id": "6569986-1-1", "paragraph_question": "question: \ud55c\uad6d\uce5c\uc6b0\ud68c \ucc3d\ub9bd \ubc1c\uae30\ud68c\uac00 \uc5f4\ub9b0 \ub0a0\uc740 \uc5b8\uc81c\uc778\uac00?, context: \uc9c0\uc9c0\uc790\ub97c \ubaa8\uc740 \uc11c\uc7ac\ud544\uc740 \ud1b0\ud0a8\uc2a4, \ubca0\ub124\ub515\ud2b8 \ub4f1\uacfc \ud568\uaed8 1919\ub144 5\uc6d4 2\uc77c \ud544\ub77c\ub378\ud53c\uc544 \uc2dc \uc2dc\ub0b4 \uc2dc\ud2f0\ud074\ub7fd \ube4c\ub529\uc5d0\uc11c \ubbf8\uad6d\ub0b4 \uce5c\ud55c\ud30c \uc885\uad50\uacc4, \uad50\uc721\uacc4, \uc2e4\uc5c5\uacc4\uc758 \ubd84\uc57c\ubcc4 \uc800\uba85\uc778\uc0ac 22\uba85\uc744 \ucd08\uccad\ud558\uc5ec \ud55c\uad6d\uce5c\uc6b0\ud68c \ucc3d\ub9bd \ubc1c\uae30\ud68c\ub97c \uc5f4\uc5c8\ub2e4. \uc5ec\uae30\uc11c \uc11c\uc7ac\ud544\uc740 \uc6b0\uc120 \ubbf8\uad6d \uc120\uad50\uc0ac\ub4e4\uc774 \uc870\uc120\uc5d0 \uc758\ud559\uacfc \uae30\ub3c5\uad50, \uc2e0 \ubb38\uba85\uc758 \uc804\ud30c\uc640 \uc704\uc0dd\uc801\uc778 \uc591\uc625\uc9d1 \uac74\uc124 \ub4f1\uc758 \ud61c\ud0dd\uc744 \uc900 \uac83\uc5d0 \ub300\ud574 \uae4a\uc740 \uac10\uc0ac\ub97c \ud45c\ud55c\ub2e4\uace0 \ud558\uace0, \uc774\uc5b4 '\ubbf8\uad6d\uad6d\ubbfc\uc774 \ud55c\uad6d\uc5d0 \uae30\ub3c5\uad50\ub97c \uc804\ud30c\ud558\uc5ec \uc790\uc720\uc640 \ub3c5\ub9bd\uc744 \ud55c\uc778\ub4e4\uc5d0\uac8c \uac00\ub974\ucce4\ub294\ub370, \uc9c0\uae08 \uc870\uc120\uc778\ub4e4\uc774 \uc774\ub97c \uc704\ud574 \uc2f8\uc6b0\ub294 \uac83\uc744 \ubbf8\uad6d\uad6d\ubbfc\uc774 \ubaa8\ub978 \uccb4 \ud55c\ub2e4\uba74 \uc774\uac83\uc740 \ud06c\uac8c \ubaa8\uc21c\ub418\ub294 \uc77c\uc774\uc694, \uc9c0\uae08\uaecf \ubbf8\uad6d \uc120\uad50\uc0ac\ub4e4\uacfc \uc758\uc0ac\ub4e4\uc758 \uc778\ub3c4\uc801 \uc9c0\uc6d0\uc740 \uc218\ud3ec\ub85c \ub3cc\uc544\uac00\ub294 \uac83\uc774\uba70, \ub3c4\ub9ac\uac00 \uc544\ub2c8\ub77c\ub294 \ub0b4\uc6a9\uc73c\ub85c \ud55c\uad6d\uce5c\uc6b0\ud68c\uc758 \uc124\ub9bd \ud544\uc694\uc131'\uc744 \uc5ed\uc124\ud588\ub2e4. \ub610\ud55c \uc11c\uc7ac\ud544\uc740 \uc870\uc120\uc778\ub4e4\uc740 \uc740\ud61c\ub97c \uc544\ub294 \uc0ac\ub78c\ub4e4\uc774\ub2c8 \ubbf8\uad6d\uc778\ub4e4\uc774 \ud55c\uad6d\uc758 \ub3c5\ub9bd\uc5d0 \uc801\uadf9 \uc9c0\uc9c0\ub97c \ubcf4\ub0b8\ub2e4\uba74 \ud55c\uad6d \uc5ed\uc2dc \ubbf8\uad6d\uc744 \ubc30\uc2e0\ud558\ub294 \uc77c\uc740 \uc5c6\uc744 \uac83\uc784\uc744 \ud638\uc18c\ud558\uc600\ub2e4."} {"question": "\ud55c\ud6a8\uc8fc\uac00 \ucc98\uc74c \ucd9c\uc5f0\ud55c \uc0ac\uadf9\uc740?", "paragraph": "\ud55c\ud6a8\uc8fc\ub294 2007\ub144 \ubc29\uc1a1\ub41c \ub4dc\ub77c\ub9c8 \u300a\ud558\ub298\ub9cc\ud07c \ub545\ub9cc\ud07c\u300b\uc5d0\uc11c \uc5ec\uc8fc\uc778\uacf5 \uc11d\uc9c0\uc218 \uc5ed\uc744 \ub9e1\uc544 40%\ub300\uc758 \ub192\uc740 \uc2dc\uccad\ub960\uc744 \uae30\ub85d\ud558\ub294 \uc131\uacfc\ub97c \uc774\ub918\uc73c\uba70, \ubc1d\uc740 \uce90\ub9ad\ud130\ub97c \uad6c\ucd95\ud558\uba70 \uc5f0\uae30\uc758 \ud3ed\ub3c4 \ub113\ud614\ub2e4. \uc778\ud130\ubdf0\ub97c \ud1b5\ud574 \u201c\u2018\ubd04\uc758 \uc648\uce20\u2019\uc5d0 \ucd5c\uc120\uc744 \ub2e4\ud588\uc9c0\ub9cc \uc2dc\uccad\ub960\uc774 \uc88b\uc9c0 \uc54a\uc544 \uc758\uae30\uc18c\uce68\ud588\ub294\ub370 \u2018\ud558\ub298\ub9cc\ud07c \ub545\ub9cc\ud07c\u2019\uc774 \uc2dc\uccad\ub960\uc774 \ub192\uc544 \uc5f0\uae30\uc790\ub85c\uc11c \uc790\uc2e0\uac10\ub3c4 \uc0dd\uacbc\ub2e4\u201d\uace0 \ub9d0\ud588\ub2e4. KBS \uc5f0\uae30\ub300\uc0c1 \uc5ec\uc790\uc778\uae30\uc0c1\uacfc \ubca0\uc2a4\ud2b8\ucee4\ud50c\uc0c1\uc744 \uc218\uc0c1\ud588\ub2e4. \ud55c\ud6a8\uc8fc\ub294 \uc774\ub4ec\ud574, 2008\ub144 \ud4e8\uc804 \uc0ac\uadf9 \u300a\uc77c\uc9c0\ub9e4\u300b\uc5d0\uc11c \uc870\uc120 \ucd5c\ucd08\ub85c \uc5ec\uad00\uc0ac\uc5c5\uc744 \ud558\ub294 \uc9c0\uc801\uc774\uace0 \ucd1d\uba85\ud55c \uc5ec\uc131 \ubcc0\uc740\ucc44 \uc5ed\uc744 \ub9e1\uc544 \uccab \uc0ac\uadf9\uc5d0 \ucd9c\uc5f0\ud588\ub2e4. \uc774 \uc791\ud488\uc740 \ucd5c\uace0 31%\uc758 \ub192\uc740 \uc2dc\uccad\ub960\uc744 \uae30\ub85d\ud558\uba70 \ud070 \uc778\uae30\ub97c \ub04c\uc5c8\uace0, \ud55c\ud6a8\uc8fc\ub294 \uc2dc\ud2b8\ucf64, \ubbf8\ub2c8\uc2dc\ub9ac\uc988, \uc77c\uc77c\uadf9\uc5d0 \uc774\uc5b4 \uc0ac\uadf9\uae4c\uc9c0 \ud65c\ub3d9\uc601\uc5ed\uc744 \ub113\ud600 \uc131\uacf5\uc744 \ud588\uc73c\uba70 \ub2e4\uc591\ud55c \uc7a5\ub974\ub97c \uc18c\ud654\ud560 \uc218 \uc788\ub294 \uae30\ubc18\uc744 \ub2e4\uc84c\ub2e4. SBS \uc5f0\uae30\ub300\uc0c1 \ub274\uc2a4\ud0c0\uc0c1\uc744 \uc218\uc0c1\ud588\ub2e4. \uac19\uc740 \ud574 8\uc6d4\uc5d0\ub294, \uc784\uc131\uc6b4 \uac10\ub3c5\uc758 \ub3c5\ub9bd\uc601\ud654 \u300a\ub2ec\ub824\ub77c \uc790\uc804\uac70\u300b\uc5d0 \ucd9c\uc5f0\ud558\uc600\ub2e4. \uc774\uc724\uae30 \uac10\ub3c5\uc758 \uc601\ud654 \u300a\uba4b\uc9c4 \ud558\ub8e8\u300b\uc5d0 \ubaa9\uc18c\ub9ac\ub85c \ud2b9\ubcc4 \ucd9c\uc5f0\ud588\ub2e4.", "answer": "\uc77c\uc9c0\ub9e4", "sentence": "\ud55c\ud6a8\uc8fc\ub294 \uc774\ub4ec\ud574, 2008\ub144 \ud4e8\uc804 \uc0ac\uadf9 \u300a \uc77c\uc9c0\ub9e4 \u300b\uc5d0\uc11c \uc870\uc120 \ucd5c\ucd08\ub85c \uc5ec\uad00\uc0ac\uc5c5\uc744 \ud558\ub294 \uc9c0\uc801\uc774\uace0 \ucd1d\uba85\ud55c \uc5ec\uc131 \ubcc0\uc740\ucc44 \uc5ed\uc744 \ub9e1\uc544 \uccab \uc0ac\uadf9\uc5d0 \ucd9c\uc5f0\ud588\ub2e4.", "paragraph_sentence": "\ud55c\ud6a8\uc8fc\ub294 2007\ub144 \ubc29\uc1a1\ub41c \ub4dc\ub77c\ub9c8 \u300a\ud558\ub298\ub9cc\ud07c \ub545\ub9cc\ud07c\u300b\uc5d0\uc11c \uc5ec\uc8fc\uc778\uacf5 \uc11d\uc9c0\uc218 \uc5ed\uc744 \ub9e1\uc544 40%\ub300\uc758 \ub192\uc740 \uc2dc\uccad\ub960\uc744 \uae30\ub85d\ud558\ub294 \uc131\uacfc\ub97c \uc774\ub918\uc73c\uba70, \ubc1d\uc740 \uce90\ub9ad\ud130\ub97c \uad6c\ucd95\ud558\uba70 \uc5f0\uae30\uc758 \ud3ed\ub3c4 \ub113\ud614\ub2e4. \uc778\ud130\ubdf0\ub97c \ud1b5\ud574 \u201c\u2018\ubd04\uc758 \uc648\uce20\u2019\uc5d0 \ucd5c\uc120\uc744 \ub2e4\ud588\uc9c0\ub9cc \uc2dc\uccad\ub960\uc774 \uc88b\uc9c0 \uc54a\uc544 \uc758\uae30\uc18c\uce68\ud588\ub294\ub370 \u2018\ud558\ub298\ub9cc\ud07c \ub545\ub9cc\ud07c\u2019\uc774 \uc2dc\uccad\ub960\uc774 \ub192\uc544 \uc5f0\uae30\uc790\ub85c\uc11c \uc790\uc2e0\uac10\ub3c4 \uc0dd\uacbc\ub2e4\u201d\uace0 \ub9d0\ud588\ub2e4. KBS \uc5f0\uae30\ub300\uc0c1 \uc5ec\uc790\uc778\uae30\uc0c1\uacfc \ubca0\uc2a4\ud2b8\ucee4\ud50c\uc0c1\uc744 \uc218\uc0c1\ud588\ub2e4. \ud55c\ud6a8\uc8fc\ub294 \uc774\ub4ec\ud574, 2008\ub144 \ud4e8\uc804 \uc0ac\uadf9 \u300a \uc77c\uc9c0\ub9e4 \u300b\uc5d0\uc11c \uc870\uc120 \ucd5c\ucd08\ub85c \uc5ec\uad00\uc0ac\uc5c5\uc744 \ud558\ub294 \uc9c0\uc801\uc774\uace0 \ucd1d\uba85\ud55c \uc5ec\uc131 \ubcc0\uc740\ucc44 \uc5ed\uc744 \ub9e1\uc544 \uccab \uc0ac\uadf9\uc5d0 \ucd9c\uc5f0\ud588\ub2e4. \uc774 \uc791\ud488\uc740 \ucd5c\uace0 31%\uc758 \ub192\uc740 \uc2dc\uccad\ub960\uc744 \uae30\ub85d\ud558\uba70 \ud070 \uc778\uae30\ub97c \ub04c\uc5c8\uace0, \ud55c\ud6a8\uc8fc\ub294 \uc2dc\ud2b8\ucf64, \ubbf8\ub2c8\uc2dc\ub9ac\uc988, \uc77c\uc77c\uadf9\uc5d0 \uc774\uc5b4 \uc0ac\uadf9\uae4c\uc9c0 \ud65c\ub3d9\uc601\uc5ed\uc744 \ub113\ud600 \uc131\uacf5\uc744 \ud588\uc73c\uba70 \ub2e4\uc591\ud55c \uc7a5\ub974\ub97c \uc18c\ud654\ud560 \uc218 \uc788\ub294 \uae30\ubc18\uc744 \ub2e4\uc84c\ub2e4. SBS \uc5f0\uae30\ub300\uc0c1 \ub274\uc2a4\ud0c0\uc0c1\uc744 \uc218\uc0c1\ud588\ub2e4. \uac19\uc740 \ud574 8\uc6d4\uc5d0\ub294, \uc784\uc131\uc6b4 \uac10\ub3c5\uc758 \ub3c5\ub9bd\uc601\ud654 \u300a\ub2ec\ub824\ub77c \uc790\uc804\uac70\u300b\uc5d0 \ucd9c\uc5f0\ud558\uc600\ub2e4. \uc774\uc724\uae30 \uac10\ub3c5\uc758 \uc601\ud654 \u300a\uba4b\uc9c4 \ud558\ub8e8\u300b\uc5d0 \ubaa9\uc18c\ub9ac\ub85c \ud2b9\ubcc4 \ucd9c\uc5f0\ud588\ub2e4.", "paragraph_answer": "\ud55c\ud6a8\uc8fc\ub294 2007\ub144 \ubc29\uc1a1\ub41c \ub4dc\ub77c\ub9c8 \u300a\ud558\ub298\ub9cc\ud07c \ub545\ub9cc\ud07c\u300b\uc5d0\uc11c \uc5ec\uc8fc\uc778\uacf5 \uc11d\uc9c0\uc218 \uc5ed\uc744 \ub9e1\uc544 40%\ub300\uc758 \ub192\uc740 \uc2dc\uccad\ub960\uc744 \uae30\ub85d\ud558\ub294 \uc131\uacfc\ub97c \uc774\ub918\uc73c\uba70, \ubc1d\uc740 \uce90\ub9ad\ud130\ub97c \uad6c\ucd95\ud558\uba70 \uc5f0\uae30\uc758 \ud3ed\ub3c4 \ub113\ud614\ub2e4. \uc778\ud130\ubdf0\ub97c \ud1b5\ud574 \u201c\u2018\ubd04\uc758 \uc648\uce20\u2019\uc5d0 \ucd5c\uc120\uc744 \ub2e4\ud588\uc9c0\ub9cc \uc2dc\uccad\ub960\uc774 \uc88b\uc9c0 \uc54a\uc544 \uc758\uae30\uc18c\uce68\ud588\ub294\ub370 \u2018\ud558\ub298\ub9cc\ud07c \ub545\ub9cc\ud07c\u2019\uc774 \uc2dc\uccad\ub960\uc774 \ub192\uc544 \uc5f0\uae30\uc790\ub85c\uc11c \uc790\uc2e0\uac10\ub3c4 \uc0dd\uacbc\ub2e4\u201d\uace0 \ub9d0\ud588\ub2e4. KBS \uc5f0\uae30\ub300\uc0c1 \uc5ec\uc790\uc778\uae30\uc0c1\uacfc \ubca0\uc2a4\ud2b8\ucee4\ud50c\uc0c1\uc744 \uc218\uc0c1\ud588\ub2e4. \ud55c\ud6a8\uc8fc\ub294 \uc774\ub4ec\ud574, 2008\ub144 \ud4e8\uc804 \uc0ac\uadf9 \u300a \uc77c\uc9c0\ub9e4 \u300b\uc5d0\uc11c \uc870\uc120 \ucd5c\ucd08\ub85c \uc5ec\uad00\uc0ac\uc5c5\uc744 \ud558\ub294 \uc9c0\uc801\uc774\uace0 \ucd1d\uba85\ud55c \uc5ec\uc131 \ubcc0\uc740\ucc44 \uc5ed\uc744 \ub9e1\uc544 \uccab \uc0ac\uadf9\uc5d0 \ucd9c\uc5f0\ud588\ub2e4. \uc774 \uc791\ud488\uc740 \ucd5c\uace0 31%\uc758 \ub192\uc740 \uc2dc\uccad\ub960\uc744 \uae30\ub85d\ud558\uba70 \ud070 \uc778\uae30\ub97c \ub04c\uc5c8\uace0, \ud55c\ud6a8\uc8fc\ub294 \uc2dc\ud2b8\ucf64, \ubbf8\ub2c8\uc2dc\ub9ac\uc988, \uc77c\uc77c\uadf9\uc5d0 \uc774\uc5b4 \uc0ac\uadf9\uae4c\uc9c0 \ud65c\ub3d9\uc601\uc5ed\uc744 \ub113\ud600 \uc131\uacf5\uc744 \ud588\uc73c\uba70 \ub2e4\uc591\ud55c \uc7a5\ub974\ub97c \uc18c\ud654\ud560 \uc218 \uc788\ub294 \uae30\ubc18\uc744 \ub2e4\uc84c\ub2e4. SBS \uc5f0\uae30\ub300\uc0c1 \ub274\uc2a4\ud0c0\uc0c1\uc744 \uc218\uc0c1\ud588\ub2e4. \uac19\uc740 \ud574 8\uc6d4\uc5d0\ub294, \uc784\uc131\uc6b4 \uac10\ub3c5\uc758 \ub3c5\ub9bd\uc601\ud654 \u300a\ub2ec\ub824\ub77c \uc790\uc804\uac70\u300b\uc5d0 \ucd9c\uc5f0\ud558\uc600\ub2e4. \uc774\uc724\uae30 \uac10\ub3c5\uc758 \uc601\ud654 \u300a\uba4b\uc9c4 \ud558\ub8e8\u300b\uc5d0 \ubaa9\uc18c\ub9ac\ub85c \ud2b9\ubcc4 \ucd9c\uc5f0\ud588\ub2e4.", "sentence_answer": "\ud55c\ud6a8\uc8fc\ub294 \uc774\ub4ec\ud574, 2008\ub144 \ud4e8\uc804 \uc0ac\uadf9 \u300a \uc77c\uc9c0\ub9e4 \u300b\uc5d0\uc11c \uc870\uc120 \ucd5c\ucd08\ub85c \uc5ec\uad00\uc0ac\uc5c5\uc744 \ud558\ub294 \uc9c0\uc801\uc774\uace0 \ucd1d\uba85\ud55c \uc5ec\uc131 \ubcc0\uc740\ucc44 \uc5ed\uc744 \ub9e1\uc544 \uccab \uc0ac\uadf9\uc5d0 \ucd9c\uc5f0\ud588\ub2e4.", "paragraph_id": "6570194-3-0", "paragraph_question": "question: \ud55c\ud6a8\uc8fc\uac00 \ucc98\uc74c \ucd9c\uc5f0\ud55c \uc0ac\uadf9\uc740?, context: \ud55c\ud6a8\uc8fc\ub294 2007\ub144 \ubc29\uc1a1\ub41c \ub4dc\ub77c\ub9c8 \u300a\ud558\ub298\ub9cc\ud07c \ub545\ub9cc\ud07c\u300b\uc5d0\uc11c \uc5ec\uc8fc\uc778\uacf5 \uc11d\uc9c0\uc218 \uc5ed\uc744 \ub9e1\uc544 40%\ub300\uc758 \ub192\uc740 \uc2dc\uccad\ub960\uc744 \uae30\ub85d\ud558\ub294 \uc131\uacfc\ub97c \uc774\ub918\uc73c\uba70, \ubc1d\uc740 \uce90\ub9ad\ud130\ub97c \uad6c\ucd95\ud558\uba70 \uc5f0\uae30\uc758 \ud3ed\ub3c4 \ub113\ud614\ub2e4. \uc778\ud130\ubdf0\ub97c \ud1b5\ud574 \u201c\u2018\ubd04\uc758 \uc648\uce20\u2019\uc5d0 \ucd5c\uc120\uc744 \ub2e4\ud588\uc9c0\ub9cc \uc2dc\uccad\ub960\uc774 \uc88b\uc9c0 \uc54a\uc544 \uc758\uae30\uc18c\uce68\ud588\ub294\ub370 \u2018\ud558\ub298\ub9cc\ud07c \ub545\ub9cc\ud07c\u2019\uc774 \uc2dc\uccad\ub960\uc774 \ub192\uc544 \uc5f0\uae30\uc790\ub85c\uc11c \uc790\uc2e0\uac10\ub3c4 \uc0dd\uacbc\ub2e4\u201d\uace0 \ub9d0\ud588\ub2e4. KBS \uc5f0\uae30\ub300\uc0c1 \uc5ec\uc790\uc778\uae30\uc0c1\uacfc \ubca0\uc2a4\ud2b8\ucee4\ud50c\uc0c1\uc744 \uc218\uc0c1\ud588\ub2e4. \ud55c\ud6a8\uc8fc\ub294 \uc774\ub4ec\ud574, 2008\ub144 \ud4e8\uc804 \uc0ac\uadf9 \u300a\uc77c\uc9c0\ub9e4\u300b\uc5d0\uc11c \uc870\uc120 \ucd5c\ucd08\ub85c \uc5ec\uad00\uc0ac\uc5c5\uc744 \ud558\ub294 \uc9c0\uc801\uc774\uace0 \ucd1d\uba85\ud55c \uc5ec\uc131 \ubcc0\uc740\ucc44 \uc5ed\uc744 \ub9e1\uc544 \uccab \uc0ac\uadf9\uc5d0 \ucd9c\uc5f0\ud588\ub2e4. \uc774 \uc791\ud488\uc740 \ucd5c\uace0 31%\uc758 \ub192\uc740 \uc2dc\uccad\ub960\uc744 \uae30\ub85d\ud558\uba70 \ud070 \uc778\uae30\ub97c \ub04c\uc5c8\uace0, \ud55c\ud6a8\uc8fc\ub294 \uc2dc\ud2b8\ucf64, \ubbf8\ub2c8\uc2dc\ub9ac\uc988, \uc77c\uc77c\uadf9\uc5d0 \uc774\uc5b4 \uc0ac\uadf9\uae4c\uc9c0 \ud65c\ub3d9\uc601\uc5ed\uc744 \ub113\ud600 \uc131\uacf5\uc744 \ud588\uc73c\uba70 \ub2e4\uc591\ud55c \uc7a5\ub974\ub97c \uc18c\ud654\ud560 \uc218 \uc788\ub294 \uae30\ubc18\uc744 \ub2e4\uc84c\ub2e4. SBS \uc5f0\uae30\ub300\uc0c1 \ub274\uc2a4\ud0c0\uc0c1\uc744 \uc218\uc0c1\ud588\ub2e4. \uac19\uc740 \ud574 8\uc6d4\uc5d0\ub294, \uc784\uc131\uc6b4 \uac10\ub3c5\uc758 \ub3c5\ub9bd\uc601\ud654 \u300a\ub2ec\ub824\ub77c \uc790\uc804\uac70\u300b\uc5d0 \ucd9c\uc5f0\ud558\uc600\ub2e4. \uc774\uc724\uae30 \uac10\ub3c5\uc758 \uc601\ud654 \u300a\uba4b\uc9c4 \ud558\ub8e8\u300b\uc5d0 \ubaa9\uc18c\ub9ac\ub85c \ud2b9\ubcc4 \ucd9c\uc5f0\ud588\ub2e4."} {"question": "\ucd5c\uc9c0\uc6b0\uac00 \ub370\ubdd4\ud55c \ub4dc\ub77c\ub9c8\ub294?", "paragraph": "\ucd5c\uc9c0\uc6b0\ub294 \uc5b4\ub9b0 \uc2dc\uc808\ubd80\ud130 \uce74\uba54\ub77c\ub97c \uc88b\uc544\ud558\ub358 \uc131\ud5a5\uc744 \uc0b4\ub824 \uc5f0\uae30\uc778\uc73c\ub85c \uc131\uc7a5\ud560 \uc0dd\uac01\uc744 \uc790\uc8fc \ud574\uc654\ub2e4\uace0 \ud55c\ub2e4. \uc774\ud6c4 \ubd80\uc0b0\uc5d0\uc11c \uace0\ub4f1\ud559\uad50\ub97c \uc878\uc5c5\ud558\uace0, \uce5c\ucc99\uc758 \uad8c\uc720\ub85c \uc5f0\uae30\uc778\uc774 \ub418\uace0\uc790 \uc11c\uc6b8\ub85c \ud63c\uc790 \uc0c1\uacbd\ud588\uace0, 1994\ub144 MBC 23\uae30 \uacf5\ucc44 \ud0e4\ub7f0\ud2b8\ub85c \uc120\ubc1c \ub418\uc5c8\ub2e4. 1995\ub144 \ub4dc\ub77c\ub9c8 \u300a\uc804\uc7c1\uacfc \uc0ac\ub791\u300b\ub85c \ub370\ubdd4\ud55c \ucd5c\uc9c0\uc6b0\ub294 \uc774 \uc791\ud488\uc5d0\uc11c\ubd80\ud130 \ubcf8\uba85\uc778 \ucd5c\ubbf8\ud5a5 \ub300\uc2e0\uc5d0 \ucd5c\uc9c0\uc6b0\ub77c\ub294 \uc608\uba85\uc744 \uc0ac\uc6a9\ud558\uac8c \ub418\uc5c8\ub2e4. \uacf5\ucc44 \ud0e4\ub7f0\ud2b8\ub85c\uc11c \ud154\ub808\ube44\uc804 \ub4dc\ub77c\ub9c8\uc5d0 \ub2e8\uc5ed\uc73c\ub85c \ucd9c\uc5f0\ud558\ub358 \uc2dc\uae30\uc5d0 1996\ub144 5\uc6d4, \uc885\ub85c 3\uac00 \ud53c\uce74\ub514\ub9ac \uadf9\uc7a5\uc5d0\uc11c \ud504\ub791\uc2a4\uc758 \uc5ec\ubc30\uc6b0 \uc774\uc790\ubca8 \uc544\uc790\ub2c8\uac00 \uc8fc\uc5f0\ud55c \uc601\ud654 \u300a\ub514\uc544\ubcfc\ub9ad\u300b\uc758 \ud504\ub85c\ubaa8\uc154\ub110 \uc774\ubca4\ud2b8\uc778 \"\uc774\uc790\ubca8 \uc544\uc790\ub2c8 \ub2ee\uc740 \uaf34 \uc120\ubc1c\ub300\ud68c\"\uc5d0\uc11c \ub300\uc0c1\uc744 \ud0c4 \ub3c5\ud2b9\ud55c \uc774\ub825\uc744 \uac00\uc9c0\uace0 \uc788\ub2e4.", "answer": "\uc804\uc7c1\uacfc \uc0ac\ub791", "sentence": "1995\ub144 \ub4dc\ub77c\ub9c8 \u300a \uc804\uc7c1\uacfc \uc0ac\ub791 \u300b\ub85c \ub370\ubdd4\ud55c \ucd5c\uc9c0\uc6b0\ub294 \uc774 \uc791\ud488\uc5d0\uc11c\ubd80\ud130 \ubcf8\uba85\uc778 \ucd5c\ubbf8\ud5a5 \ub300\uc2e0\uc5d0 \ucd5c\uc9c0\uc6b0\ub77c\ub294 \uc608\uba85\uc744 \uc0ac\uc6a9\ud558\uac8c \ub418\uc5c8\ub2e4.", "paragraph_sentence": "\ucd5c\uc9c0\uc6b0\ub294 \uc5b4\ub9b0 \uc2dc\uc808\ubd80\ud130 \uce74\uba54\ub77c\ub97c \uc88b\uc544\ud558\ub358 \uc131\ud5a5\uc744 \uc0b4\ub824 \uc5f0\uae30\uc778\uc73c\ub85c \uc131\uc7a5\ud560 \uc0dd\uac01\uc744 \uc790\uc8fc \ud574\uc654\ub2e4\uace0 \ud55c\ub2e4. \uc774\ud6c4 \ubd80\uc0b0\uc5d0\uc11c \uace0\ub4f1\ud559\uad50\ub97c \uc878\uc5c5\ud558\uace0, \uce5c\ucc99\uc758 \uad8c\uc720\ub85c \uc5f0\uae30\uc778\uc774 \ub418\uace0\uc790 \uc11c\uc6b8\ub85c \ud63c\uc790 \uc0c1\uacbd\ud588\uace0, 1994\ub144 MBC 23\uae30 \uacf5\ucc44 \ud0e4\ub7f0\ud2b8\ub85c \uc120\ubc1c \ub418\uc5c8\ub2e4. 1995\ub144 \ub4dc\ub77c\ub9c8 \u300a \uc804\uc7c1\uacfc \uc0ac\ub791 \u300b\ub85c \ub370\ubdd4\ud55c \ucd5c\uc9c0\uc6b0\ub294 \uc774 \uc791\ud488\uc5d0\uc11c\ubd80\ud130 \ubcf8\uba85\uc778 \ucd5c\ubbf8\ud5a5 \ub300\uc2e0\uc5d0 \ucd5c\uc9c0\uc6b0\ub77c\ub294 \uc608\uba85\uc744 \uc0ac\uc6a9\ud558\uac8c \ub418\uc5c8\ub2e4. \uacf5\ucc44 \ud0e4\ub7f0\ud2b8\ub85c\uc11c \ud154\ub808\ube44\uc804 \ub4dc\ub77c\ub9c8\uc5d0 \ub2e8\uc5ed\uc73c\ub85c \ucd9c\uc5f0\ud558\ub358 \uc2dc\uae30\uc5d0 1996\ub144 5\uc6d4, \uc885\ub85c 3\uac00 \ud53c\uce74\ub514\ub9ac \uadf9\uc7a5\uc5d0\uc11c \ud504\ub791\uc2a4\uc758 \uc5ec\ubc30\uc6b0 \uc774\uc790\ubca8 \uc544\uc790\ub2c8\uac00 \uc8fc\uc5f0\ud55c \uc601\ud654 \u300a\ub514\uc544\ubcfc\ub9ad\u300b\uc758 \ud504\ub85c\ubaa8\uc154\ub110 \uc774\ubca4\ud2b8\uc778 \"\uc774\uc790\ubca8 \uc544\uc790\ub2c8 \ub2ee\uc740 \uaf34 \uc120\ubc1c\ub300\ud68c\"\uc5d0\uc11c \ub300\uc0c1\uc744 \ud0c4 \ub3c5\ud2b9\ud55c \uc774\ub825\uc744 \uac00\uc9c0\uace0 \uc788\ub2e4.", "paragraph_answer": "\ucd5c\uc9c0\uc6b0\ub294 \uc5b4\ub9b0 \uc2dc\uc808\ubd80\ud130 \uce74\uba54\ub77c\ub97c \uc88b\uc544\ud558\ub358 \uc131\ud5a5\uc744 \uc0b4\ub824 \uc5f0\uae30\uc778\uc73c\ub85c \uc131\uc7a5\ud560 \uc0dd\uac01\uc744 \uc790\uc8fc \ud574\uc654\ub2e4\uace0 \ud55c\ub2e4. \uc774\ud6c4 \ubd80\uc0b0\uc5d0\uc11c \uace0\ub4f1\ud559\uad50\ub97c \uc878\uc5c5\ud558\uace0, \uce5c\ucc99\uc758 \uad8c\uc720\ub85c \uc5f0\uae30\uc778\uc774 \ub418\uace0\uc790 \uc11c\uc6b8\ub85c \ud63c\uc790 \uc0c1\uacbd\ud588\uace0, 1994\ub144 MBC 23\uae30 \uacf5\ucc44 \ud0e4\ub7f0\ud2b8\ub85c \uc120\ubc1c \ub418\uc5c8\ub2e4. 1995\ub144 \ub4dc\ub77c\ub9c8 \u300a \uc804\uc7c1\uacfc \uc0ac\ub791 \u300b\ub85c \ub370\ubdd4\ud55c \ucd5c\uc9c0\uc6b0\ub294 \uc774 \uc791\ud488\uc5d0\uc11c\ubd80\ud130 \ubcf8\uba85\uc778 \ucd5c\ubbf8\ud5a5 \ub300\uc2e0\uc5d0 \ucd5c\uc9c0\uc6b0\ub77c\ub294 \uc608\uba85\uc744 \uc0ac\uc6a9\ud558\uac8c \ub418\uc5c8\ub2e4. \uacf5\ucc44 \ud0e4\ub7f0\ud2b8\ub85c\uc11c \ud154\ub808\ube44\uc804 \ub4dc\ub77c\ub9c8\uc5d0 \ub2e8\uc5ed\uc73c\ub85c \ucd9c\uc5f0\ud558\ub358 \uc2dc\uae30\uc5d0 1996\ub144 5\uc6d4, \uc885\ub85c 3\uac00 \ud53c\uce74\ub514\ub9ac \uadf9\uc7a5\uc5d0\uc11c \ud504\ub791\uc2a4\uc758 \uc5ec\ubc30\uc6b0 \uc774\uc790\ubca8 \uc544\uc790\ub2c8\uac00 \uc8fc\uc5f0\ud55c \uc601\ud654 \u300a\ub514\uc544\ubcfc\ub9ad\u300b\uc758 \ud504\ub85c\ubaa8\uc154\ub110 \uc774\ubca4\ud2b8\uc778 \"\uc774\uc790\ubca8 \uc544\uc790\ub2c8 \ub2ee\uc740 \uaf34 \uc120\ubc1c\ub300\ud68c\"\uc5d0\uc11c \ub300\uc0c1\uc744 \ud0c4 \ub3c5\ud2b9\ud55c \uc774\ub825\uc744 \uac00\uc9c0\uace0 \uc788\ub2e4.", "sentence_answer": "1995\ub144 \ub4dc\ub77c\ub9c8 \u300a \uc804\uc7c1\uacfc \uc0ac\ub791 \u300b\ub85c \ub370\ubdd4\ud55c \ucd5c\uc9c0\uc6b0\ub294 \uc774 \uc791\ud488\uc5d0\uc11c\ubd80\ud130 \ubcf8\uba85\uc778 \ucd5c\ubbf8\ud5a5 \ub300\uc2e0\uc5d0 \ucd5c\uc9c0\uc6b0\ub77c\ub294 \uc608\uba85\uc744 \uc0ac\uc6a9\ud558\uac8c \ub418\uc5c8\ub2e4.", "paragraph_id": "6560154-0-2", "paragraph_question": "question: \ucd5c\uc9c0\uc6b0\uac00 \ub370\ubdd4\ud55c \ub4dc\ub77c\ub9c8\ub294?, context: \ucd5c\uc9c0\uc6b0\ub294 \uc5b4\ub9b0 \uc2dc\uc808\ubd80\ud130 \uce74\uba54\ub77c\ub97c \uc88b\uc544\ud558\ub358 \uc131\ud5a5\uc744 \uc0b4\ub824 \uc5f0\uae30\uc778\uc73c\ub85c \uc131\uc7a5\ud560 \uc0dd\uac01\uc744 \uc790\uc8fc \ud574\uc654\ub2e4\uace0 \ud55c\ub2e4. \uc774\ud6c4 \ubd80\uc0b0\uc5d0\uc11c \uace0\ub4f1\ud559\uad50\ub97c \uc878\uc5c5\ud558\uace0, \uce5c\ucc99\uc758 \uad8c\uc720\ub85c \uc5f0\uae30\uc778\uc774 \ub418\uace0\uc790 \uc11c\uc6b8\ub85c \ud63c\uc790 \uc0c1\uacbd\ud588\uace0, 1994\ub144 MBC 23\uae30 \uacf5\ucc44 \ud0e4\ub7f0\ud2b8\ub85c \uc120\ubc1c \ub418\uc5c8\ub2e4. 1995\ub144 \ub4dc\ub77c\ub9c8 \u300a\uc804\uc7c1\uacfc \uc0ac\ub791\u300b\ub85c \ub370\ubdd4\ud55c \ucd5c\uc9c0\uc6b0\ub294 \uc774 \uc791\ud488\uc5d0\uc11c\ubd80\ud130 \ubcf8\uba85\uc778 \ucd5c\ubbf8\ud5a5 \ub300\uc2e0\uc5d0 \ucd5c\uc9c0\uc6b0\ub77c\ub294 \uc608\uba85\uc744 \uc0ac\uc6a9\ud558\uac8c \ub418\uc5c8\ub2e4. \uacf5\ucc44 \ud0e4\ub7f0\ud2b8\ub85c\uc11c \ud154\ub808\ube44\uc804 \ub4dc\ub77c\ub9c8\uc5d0 \ub2e8\uc5ed\uc73c\ub85c \ucd9c\uc5f0\ud558\ub358 \uc2dc\uae30\uc5d0 1996\ub144 5\uc6d4, \uc885\ub85c 3\uac00 \ud53c\uce74\ub514\ub9ac \uadf9\uc7a5\uc5d0\uc11c \ud504\ub791\uc2a4\uc758 \uc5ec\ubc30\uc6b0 \uc774\uc790\ubca8 \uc544\uc790\ub2c8\uac00 \uc8fc\uc5f0\ud55c \uc601\ud654 \u300a\ub514\uc544\ubcfc\ub9ad\u300b\uc758 \ud504\ub85c\ubaa8\uc154\ub110 \uc774\ubca4\ud2b8\uc778 \"\uc774\uc790\ubca8 \uc544\uc790\ub2c8 \ub2ee\uc740 \uaf34 \uc120\ubc1c\ub300\ud68c\"\uc5d0\uc11c \ub300\uc0c1\uc744 \ud0c4 \ub3c5\ud2b9\ud55c \uc774\ub825\uc744 \uac00\uc9c0\uace0 \uc788\ub2e4."} {"question": "\ub8e8\uc774\uc2a4\uac00 \uc120\uaca8\ud65c\ub3d9\uc744 \ubc8c\uc774\ub2e4 \uc0ac\ub9dd\ud55c \uad6d\uac00\ub294 \uc5b4\ub514\uc778\uac00?", "paragraph": "\ud53c\uc5b4\uc120 \ubc15\uc0ac \ubd80\ubd80\ub294 \uc2ac\ud558\uc5d0 2\ub0a8 5\ub140\ub97c \ub450\uc5c8\ub2e4. \uadf8\uc758 \uc77c\uacf1 \uc790\ub140\ub4e4\uc774 \ubaa8\ub450 \uc120\uad50\uc640 \uad50\uc721 \uadf8\ub9ac\uace0 \uc0ac\ud68c \ubd09\uc0ac\uc790\uac00 \ub418\uc5c8\ub2e4. \uadf8\uc758 \uc7a5\ub140 \ud5ec\ub80c\uc740 \ud504\ub808\ub4dc\ub9ad \ucee4\ud2f0\uc2a4\uc640 \uacb0\ud63c\ud574\uc11c \uc77c\ubcf8\uc5d0\uc11c 40\ub144\uac04 \uc120\uad50\ud65c\ub3d9\uc744 \ud558\uc600\uace0 \ucc28\ub140 \ub85c\ub77c\ub294 \uc544\uba54\ub9ac\uce74 \uc6d0\uc8fc\ubbfc \ubd80\uc871\ub4e4\uc744 \uc0c1\ub300\ub85c \uc120\uad50\ud65c\ub3d9\uc744 \ud558\uc600\ub2e4. \uc548\ub098\uc640 \uc5d0\ub514\uc2a4\ub294 \ub274\uc695\uc2dc\uc640 \ud544\ub77c\ub378\ud53c\uc544\uc5d0\uc11c \uc5ec\uc131\uc815\ucc29 \uc0ac\uc5c5\uc5d0 \uc885\uc0ac\ud558\uc600\uace0 \ud6d7\ub0a0 \uc548\ub098\ub294 \ud30c\ud478\uc544 \ub274\uae30\ub2c8\uc5d0\uc11c 1\ub144\uac04 \uc120\uad50\ud65c\ub3d9\uc5d0 \uc885\uc0ac\ud558\uc600\ub2e4. \uc7a5\ub0a8 \ub378\ub77c\ubc18 \ud53c\uc5b4\uc120\uc740 \ud504\ub9b0\uc2a4\ud134 \uc2e0\ud559\uad50\ub97c \uc878\uc5c5\ud55c \ud6c4 \ubd80\uce5c\uacfc \ud568\uaed8 \u300c\uc138\uacc4\uc120\uad50\ub17c\ud3c9\u300d\uc9c0\ub97c \uc704\ud574 \uc77c\ud558\uc600\uace0, \ucc28\ub0a8 \ud328\ub7f0\ub4dc\ub294 \uc758\uc0ac\ub85c\uc11c \uac74\uac15\uc758 \uc190\uc0c1\uc73c\ub85c \uc77c\uc744 \uadf8\ub9cc\ub458 \ub54c\uae4c\uc9c0 \uc911\ub0a8\ubbf8\uc5d0\uc11c \uc120\uad50\ud65c\ub3d9\uc5d0 \ud5cc\uc2e0\ud558\uc600\ub2e4. \uadf8\ub9ac\uace0 \ub9c8\uc9c0\ub9c9 \ub538\uc778 \ub8e8\uc774\uc2a4\ub294 \uc778\ub3c4\uc5d0\uc11c \uc120\uad50\ud65c\ub3d9\uc744 \ubc8c\uc774\ub2e4\uac00 1903\ub144\uc5d0 \uc0ac\ub9dd\ud588\ub2e4. \ub610 \ub8e8\uc774\uc2a4\ub294 \ub274\uc695 \uc8fc \ud3ec \uc2dc\uc5d0 \uc18c\uc7ac\ud55c YMCA\uc640 \ub9e4\uc0ac\ucd94\uc138\uce20 \uc8fc \ub85c\uc6f0\uc5d0 \uc788\ub294 YMCA\uc5d0\uc11c \ube44\uc11c \uc77c\uc744 \ud558\ub2e4\uac00 \ubbf8\uad6d\uc5d0\uc11c \uac00\uc7a5 \uc624\ub798\ub41c \uc5ec\uc131\uc120\uad50\ud68c\uc778 \uc5ec\uc131\uc5f0\ud569\uc120\uad50\ud68c \uc0b0\ud558\uc5d0\uc11c \uc790\uc6d0\ubd09\uc0ac\ub97c \ud558\uc600\uace0 \uc778\ub3c4\uc5d0\uc11c \ube48\ubbfc\uce35 \uc5ec\uc131\ub4e4\uacfc \uc18c\ub140\ub4e4\uc744 \uc704\ud574 \uc77c\ud558\ub2e4\uac00 \uc7a5\ud2f0\ud478\uc2a4\ub85c \uc0ac\ub9dd\ud558\uc600\ub2e4. \ud6c4\uc5d0 \uadf8\uc758 \uac00\uc871\ub4e4\uc740 \uadf8\ub140\ub97c \uae30\ub150\ud574\uc11c \uc778\ub3c4 \uc911\ubd80\uc5d0 \uc788\ub294 \ub3c4\uc2dc, \ub204\uacf5\uc5d0 \uae30\ub150 \uc608\ubc30\ub2f9\uc744 \uc124\ub9bd\ud558\uc600\ub2e4.", "answer": "\uc778\ub3c4", "sentence": "\uadf8\ub9ac\uace0 \ub9c8\uc9c0\ub9c9 \ub538\uc778 \ub8e8\uc774\uc2a4\ub294 \uc778\ub3c4 \uc5d0\uc11c \uc120\uad50\ud65c\ub3d9\uc744 \ubc8c\uc774\ub2e4\uac00 1903\ub144\uc5d0 \uc0ac\ub9dd\ud588\ub2e4.", "paragraph_sentence": "\ud53c\uc5b4\uc120 \ubc15\uc0ac \ubd80\ubd80\ub294 \uc2ac\ud558\uc5d0 2\ub0a8 5\ub140\ub97c \ub450\uc5c8\ub2e4. \uadf8\uc758 \uc77c\uacf1 \uc790\ub140\ub4e4\uc774 \ubaa8\ub450 \uc120\uad50\uc640 \uad50\uc721 \uadf8\ub9ac\uace0 \uc0ac\ud68c \ubd09\uc0ac\uc790\uac00 \ub418\uc5c8\ub2e4. \uadf8\uc758 \uc7a5\ub140 \ud5ec\ub80c\uc740 \ud504\ub808\ub4dc\ub9ad \ucee4\ud2f0\uc2a4\uc640 \uacb0\ud63c\ud574\uc11c \uc77c\ubcf8\uc5d0\uc11c 40\ub144\uac04 \uc120\uad50\ud65c\ub3d9\uc744 \ud558\uc600\uace0 \ucc28\ub140 \ub85c\ub77c\ub294 \uc544\uba54\ub9ac\uce74 \uc6d0\uc8fc\ubbfc \ubd80\uc871\ub4e4\uc744 \uc0c1\ub300\ub85c \uc120\uad50\ud65c\ub3d9\uc744 \ud558\uc600\ub2e4. \uc548\ub098\uc640 \uc5d0\ub514\uc2a4\ub294 \ub274\uc695\uc2dc\uc640 \ud544\ub77c\ub378\ud53c\uc544\uc5d0\uc11c \uc5ec\uc131\uc815\ucc29 \uc0ac\uc5c5\uc5d0 \uc885\uc0ac\ud558\uc600\uace0 \ud6d7\ub0a0 \uc548\ub098\ub294 \ud30c\ud478\uc544 \ub274\uae30\ub2c8\uc5d0\uc11c 1\ub144\uac04 \uc120\uad50\ud65c\ub3d9\uc5d0 \uc885\uc0ac\ud558\uc600\ub2e4. \uc7a5\ub0a8 \ub378\ub77c\ubc18 \ud53c\uc5b4\uc120\uc740 \ud504\ub9b0\uc2a4\ud134 \uc2e0\ud559\uad50\ub97c \uc878\uc5c5\ud55c \ud6c4 \ubd80\uce5c\uacfc \ud568\uaed8 \u300c\uc138\uacc4\uc120\uad50\ub17c\ud3c9\u300d\uc9c0\ub97c \uc704\ud574 \uc77c\ud558\uc600\uace0, \ucc28\ub0a8 \ud328\ub7f0\ub4dc\ub294 \uc758\uc0ac\ub85c\uc11c \uac74\uac15\uc758 \uc190\uc0c1\uc73c\ub85c \uc77c\uc744 \uadf8\ub9cc\ub458 \ub54c\uae4c\uc9c0 \uc911\ub0a8\ubbf8\uc5d0\uc11c \uc120\uad50\ud65c\ub3d9\uc5d0 \ud5cc\uc2e0\ud558\uc600\ub2e4. \uadf8\ub9ac\uace0 \ub9c8\uc9c0\ub9c9 \ub538\uc778 \ub8e8\uc774\uc2a4\ub294 \uc778\ub3c4 \uc5d0\uc11c \uc120\uad50\ud65c\ub3d9\uc744 \ubc8c\uc774\ub2e4\uac00 1903\ub144\uc5d0 \uc0ac\ub9dd\ud588\ub2e4. \ub610 \ub8e8\uc774\uc2a4\ub294 \ub274\uc695 \uc8fc \ud3ec \uc2dc\uc5d0 \uc18c\uc7ac\ud55c YMCA\uc640 \ub9e4\uc0ac\ucd94\uc138\uce20 \uc8fc \ub85c\uc6f0\uc5d0 \uc788\ub294 YMCA\uc5d0\uc11c \ube44\uc11c \uc77c\uc744 \ud558\ub2e4\uac00 \ubbf8\uad6d\uc5d0\uc11c \uac00\uc7a5 \uc624\ub798\ub41c \uc5ec\uc131\uc120\uad50\ud68c\uc778 \uc5ec\uc131\uc5f0\ud569\uc120\uad50\ud68c \uc0b0\ud558\uc5d0\uc11c \uc790\uc6d0\ubd09\uc0ac\ub97c \ud558\uc600\uace0 \uc778\ub3c4\uc5d0\uc11c \ube48\ubbfc\uce35 \uc5ec\uc131\ub4e4\uacfc \uc18c\ub140\ub4e4\uc744 \uc704\ud574 \uc77c\ud558\ub2e4\uac00 \uc7a5\ud2f0\ud478\uc2a4\ub85c \uc0ac\ub9dd\ud558\uc600\ub2e4. \ud6c4\uc5d0 \uadf8\uc758 \uac00\uc871\ub4e4\uc740 \uadf8\ub140\ub97c \uae30\ub150\ud574\uc11c \uc778\ub3c4 \uc911\ubd80\uc5d0 \uc788\ub294 \ub3c4\uc2dc, \ub204\uacf5\uc5d0 \uae30\ub150 \uc608\ubc30\ub2f9\uc744 \uc124\ub9bd\ud558\uc600\ub2e4.", "paragraph_answer": "\ud53c\uc5b4\uc120 \ubc15\uc0ac \ubd80\ubd80\ub294 \uc2ac\ud558\uc5d0 2\ub0a8 5\ub140\ub97c \ub450\uc5c8\ub2e4. \uadf8\uc758 \uc77c\uacf1 \uc790\ub140\ub4e4\uc774 \ubaa8\ub450 \uc120\uad50\uc640 \uad50\uc721 \uadf8\ub9ac\uace0 \uc0ac\ud68c \ubd09\uc0ac\uc790\uac00 \ub418\uc5c8\ub2e4. \uadf8\uc758 \uc7a5\ub140 \ud5ec\ub80c\uc740 \ud504\ub808\ub4dc\ub9ad \ucee4\ud2f0\uc2a4\uc640 \uacb0\ud63c\ud574\uc11c \uc77c\ubcf8\uc5d0\uc11c 40\ub144\uac04 \uc120\uad50\ud65c\ub3d9\uc744 \ud558\uc600\uace0 \ucc28\ub140 \ub85c\ub77c\ub294 \uc544\uba54\ub9ac\uce74 \uc6d0\uc8fc\ubbfc \ubd80\uc871\ub4e4\uc744 \uc0c1\ub300\ub85c \uc120\uad50\ud65c\ub3d9\uc744 \ud558\uc600\ub2e4. \uc548\ub098\uc640 \uc5d0\ub514\uc2a4\ub294 \ub274\uc695\uc2dc\uc640 \ud544\ub77c\ub378\ud53c\uc544\uc5d0\uc11c \uc5ec\uc131\uc815\ucc29 \uc0ac\uc5c5\uc5d0 \uc885\uc0ac\ud558\uc600\uace0 \ud6d7\ub0a0 \uc548\ub098\ub294 \ud30c\ud478\uc544 \ub274\uae30\ub2c8\uc5d0\uc11c 1\ub144\uac04 \uc120\uad50\ud65c\ub3d9\uc5d0 \uc885\uc0ac\ud558\uc600\ub2e4. \uc7a5\ub0a8 \ub378\ub77c\ubc18 \ud53c\uc5b4\uc120\uc740 \ud504\ub9b0\uc2a4\ud134 \uc2e0\ud559\uad50\ub97c \uc878\uc5c5\ud55c \ud6c4 \ubd80\uce5c\uacfc \ud568\uaed8 \u300c\uc138\uacc4\uc120\uad50\ub17c\ud3c9\u300d\uc9c0\ub97c \uc704\ud574 \uc77c\ud558\uc600\uace0, \ucc28\ub0a8 \ud328\ub7f0\ub4dc\ub294 \uc758\uc0ac\ub85c\uc11c \uac74\uac15\uc758 \uc190\uc0c1\uc73c\ub85c \uc77c\uc744 \uadf8\ub9cc\ub458 \ub54c\uae4c\uc9c0 \uc911\ub0a8\ubbf8\uc5d0\uc11c \uc120\uad50\ud65c\ub3d9\uc5d0 \ud5cc\uc2e0\ud558\uc600\ub2e4. \uadf8\ub9ac\uace0 \ub9c8\uc9c0\ub9c9 \ub538\uc778 \ub8e8\uc774\uc2a4\ub294 \uc778\ub3c4 \uc5d0\uc11c \uc120\uad50\ud65c\ub3d9\uc744 \ubc8c\uc774\ub2e4\uac00 1903\ub144\uc5d0 \uc0ac\ub9dd\ud588\ub2e4. \ub610 \ub8e8\uc774\uc2a4\ub294 \ub274\uc695 \uc8fc \ud3ec \uc2dc\uc5d0 \uc18c\uc7ac\ud55c YMCA\uc640 \ub9e4\uc0ac\ucd94\uc138\uce20 \uc8fc \ub85c\uc6f0\uc5d0 \uc788\ub294 YMCA\uc5d0\uc11c \ube44\uc11c \uc77c\uc744 \ud558\ub2e4\uac00 \ubbf8\uad6d\uc5d0\uc11c \uac00\uc7a5 \uc624\ub798\ub41c \uc5ec\uc131\uc120\uad50\ud68c\uc778 \uc5ec\uc131\uc5f0\ud569\uc120\uad50\ud68c \uc0b0\ud558\uc5d0\uc11c \uc790\uc6d0\ubd09\uc0ac\ub97c \ud558\uc600\uace0 \uc778\ub3c4\uc5d0\uc11c \ube48\ubbfc\uce35 \uc5ec\uc131\ub4e4\uacfc \uc18c\ub140\ub4e4\uc744 \uc704\ud574 \uc77c\ud558\ub2e4\uac00 \uc7a5\ud2f0\ud478\uc2a4\ub85c \uc0ac\ub9dd\ud558\uc600\ub2e4. \ud6c4\uc5d0 \uadf8\uc758 \uac00\uc871\ub4e4\uc740 \uadf8\ub140\ub97c \uae30\ub150\ud574\uc11c \uc778\ub3c4 \uc911\ubd80\uc5d0 \uc788\ub294 \ub3c4\uc2dc, \ub204\uacf5\uc5d0 \uae30\ub150 \uc608\ubc30\ub2f9\uc744 \uc124\ub9bd\ud558\uc600\ub2e4.", "sentence_answer": "\uadf8\ub9ac\uace0 \ub9c8\uc9c0\ub9c9 \ub538\uc778 \ub8e8\uc774\uc2a4\ub294 \uc778\ub3c4 \uc5d0\uc11c \uc120\uad50\ud65c\ub3d9\uc744 \ubc8c\uc774\ub2e4\uac00 1903\ub144\uc5d0 \uc0ac\ub9dd\ud588\ub2e4.", "paragraph_id": "6533532-4-2", "paragraph_question": "question: \ub8e8\uc774\uc2a4\uac00 \uc120\uaca8\ud65c\ub3d9\uc744 \ubc8c\uc774\ub2e4 \uc0ac\ub9dd\ud55c \uad6d\uac00\ub294 \uc5b4\ub514\uc778\uac00?, context: \ud53c\uc5b4\uc120 \ubc15\uc0ac \ubd80\ubd80\ub294 \uc2ac\ud558\uc5d0 2\ub0a8 5\ub140\ub97c \ub450\uc5c8\ub2e4. \uadf8\uc758 \uc77c\uacf1 \uc790\ub140\ub4e4\uc774 \ubaa8\ub450 \uc120\uad50\uc640 \uad50\uc721 \uadf8\ub9ac\uace0 \uc0ac\ud68c \ubd09\uc0ac\uc790\uac00 \ub418\uc5c8\ub2e4. \uadf8\uc758 \uc7a5\ub140 \ud5ec\ub80c\uc740 \ud504\ub808\ub4dc\ub9ad \ucee4\ud2f0\uc2a4\uc640 \uacb0\ud63c\ud574\uc11c \uc77c\ubcf8\uc5d0\uc11c 40\ub144\uac04 \uc120\uad50\ud65c\ub3d9\uc744 \ud558\uc600\uace0 \ucc28\ub140 \ub85c\ub77c\ub294 \uc544\uba54\ub9ac\uce74 \uc6d0\uc8fc\ubbfc \ubd80\uc871\ub4e4\uc744 \uc0c1\ub300\ub85c \uc120\uad50\ud65c\ub3d9\uc744 \ud558\uc600\ub2e4. \uc548\ub098\uc640 \uc5d0\ub514\uc2a4\ub294 \ub274\uc695\uc2dc\uc640 \ud544\ub77c\ub378\ud53c\uc544\uc5d0\uc11c \uc5ec\uc131\uc815\ucc29 \uc0ac\uc5c5\uc5d0 \uc885\uc0ac\ud558\uc600\uace0 \ud6d7\ub0a0 \uc548\ub098\ub294 \ud30c\ud478\uc544 \ub274\uae30\ub2c8\uc5d0\uc11c 1\ub144\uac04 \uc120\uad50\ud65c\ub3d9\uc5d0 \uc885\uc0ac\ud558\uc600\ub2e4. \uc7a5\ub0a8 \ub378\ub77c\ubc18 \ud53c\uc5b4\uc120\uc740 \ud504\ub9b0\uc2a4\ud134 \uc2e0\ud559\uad50\ub97c \uc878\uc5c5\ud55c \ud6c4 \ubd80\uce5c\uacfc \ud568\uaed8 \u300c\uc138\uacc4\uc120\uad50\ub17c\ud3c9\u300d\uc9c0\ub97c \uc704\ud574 \uc77c\ud558\uc600\uace0, \ucc28\ub0a8 \ud328\ub7f0\ub4dc\ub294 \uc758\uc0ac\ub85c\uc11c \uac74\uac15\uc758 \uc190\uc0c1\uc73c\ub85c \uc77c\uc744 \uadf8\ub9cc\ub458 \ub54c\uae4c\uc9c0 \uc911\ub0a8\ubbf8\uc5d0\uc11c \uc120\uad50\ud65c\ub3d9\uc5d0 \ud5cc\uc2e0\ud558\uc600\ub2e4. \uadf8\ub9ac\uace0 \ub9c8\uc9c0\ub9c9 \ub538\uc778 \ub8e8\uc774\uc2a4\ub294 \uc778\ub3c4\uc5d0\uc11c \uc120\uad50\ud65c\ub3d9\uc744 \ubc8c\uc774\ub2e4\uac00 1903\ub144\uc5d0 \uc0ac\ub9dd\ud588\ub2e4. \ub610 \ub8e8\uc774\uc2a4\ub294 \ub274\uc695 \uc8fc \ud3ec \uc2dc\uc5d0 \uc18c\uc7ac\ud55c YMCA\uc640 \ub9e4\uc0ac\ucd94\uc138\uce20 \uc8fc \ub85c\uc6f0\uc5d0 \uc788\ub294 YMCA\uc5d0\uc11c \ube44\uc11c \uc77c\uc744 \ud558\ub2e4\uac00 \ubbf8\uad6d\uc5d0\uc11c \uac00\uc7a5 \uc624\ub798\ub41c \uc5ec\uc131\uc120\uad50\ud68c\uc778 \uc5ec\uc131\uc5f0\ud569\uc120\uad50\ud68c \uc0b0\ud558\uc5d0\uc11c \uc790\uc6d0\ubd09\uc0ac\ub97c \ud558\uc600\uace0 \uc778\ub3c4\uc5d0\uc11c \ube48\ubbfc\uce35 \uc5ec\uc131\ub4e4\uacfc \uc18c\ub140\ub4e4\uc744 \uc704\ud574 \uc77c\ud558\ub2e4\uac00 \uc7a5\ud2f0\ud478\uc2a4\ub85c \uc0ac\ub9dd\ud558\uc600\ub2e4. \ud6c4\uc5d0 \uadf8\uc758 \uac00\uc871\ub4e4\uc740 \uadf8\ub140\ub97c \uae30\ub150\ud574\uc11c \uc778\ub3c4 \uc911\ubd80\uc5d0 \uc788\ub294 \ub3c4\uc2dc, \ub204\uacf5\uc5d0 \uae30\ub150 \uc608\ubc30\ub2f9\uc744 \uc124\ub9bd\ud558\uc600\ub2e4."} {"question": "\uc5b8\uc81c \uc2e0\uc81c\ud488 \uac1c\ubc1c\uc744 \uc704\ud574 \uae30\uc220\uc5f0\uad6c\uc18c\ub97c \uc124\ub9bd\ud558\uc600\ub098?", "paragraph": "1980\ub144\ub300 \uc2b9\uac15\uae30 \ubc1c\uc8fc\uac00 \ud3ed\uc99d\ud558\uba74\uc11c \uc2b9\uac15\uae30 \uc5c5\uacc4\ub294 \uae09\uc131\uc7a5\ud558\uc600\uc73c\ub098 \ub2f9\uc2dc \uae08\uc131\uc0ac, \uc2e0\uc601\uc804\uae30, \ub3d9\uc591\uc5d0\ub808\ubca0\uc774\ud130 \ub4f1\uc758 \ub300\ud615\uc0ac\uc640 \ud55c\uad6d\ud6c4\uc9c0\ud14c\ud06c, \ud55c\uad6d\uc2b9\uac15\uae30\uc81c\uc791\uc18c, \uc911\uc559\uc5d8\ub9ac\ubca0\uc774\ud130, \uc720\uc77c\uc5d8\ub9ac\ubca0\uc774\ud130, \ud55c\uad6d\uc5d4\uc9c0\ub2c8\uc5b4\ub9c1 \ub4f1\uc758 \uc911\uc18c\uc0ac\ub4e4\uc740 \uc2dc\uc124\uacfc \uc778\ub825\uc774 \ubd80\uc871\ud574 \uc218\uc694\uc5d0 \uc81c\ub300\ub85c \ubd80\uc751\ud558\uc9c0 \ubabb\ud558\ub294 \uc77c\uc774 \ubc1c\uc0dd\ud588\ub2e4. \ub2f9\uc2dc \ub300\ud55c\ubbfc\uad6d\uc758 \ucd5c\ub300 \uadf8\ub8f9 \uc911 \ud558\ub098\uc600\ub358 \ud604\ub300\ub294 \ud604\ub300\uac74\uc124\uc758 \uc790\uccb4 \uc2b9\uac15\uae30 \uc5f0\uac04 \uc18c\uc694\ub7c9\uc774 \uc218\ubc31 \ub300\uc5d0 \uc774\ub974\uace0 \uadf8\ub8f9 \ub0b4 \uc870\uc120\uc18c\uc758 \uc120\ubc15 \uac74\uc870\uc5d0 \ud544\uc694\ud55c \uc120\ubc15\uc6a9 \uc2b9\uac15\uae30\uc758 \uc218\uc694, \uadf8\ub9ac\uace0 \uadf8\ub8f9 \ud655\uc7a5\uc5d0 \ub530\ub978 \ud574\uc678 \uac74\uc124\uc2dc\uc7a5\uacfc \uc804\uad6d \uac01\uc9c0\uc5d0 \uac74\uc124\ub418\ub294 \uadf8\ub8f9 \uc0b0\ud558 \uac74\ubb3c\uc5d0 \ud544\uc694\ud55c \uc2b9\uac15\uae30 \uc218\uc694 \ub4f1\uc744 \uac10\uc548\ud560 \ub54c \uc2b9\uac15\uae30 \uc0b0\uc5c5\uc5d0 \ub300\ud55c \ucc38\uc5ec\uac00 \uc808\uc2e4\ud558\ub2e4\ub294 \uac83\uc744 \uae68\ub2ec\uc558\ub2e4. \uc774\uc5d0 1982\ub144 \ud604\ub300\ub294 \ud604\ub300\uc911\uc804\uae30 \ub0b4\uc5d0 \uc6b4\ubc18\uae30\uacc4 \uc0ac\uc5c5\ubcf8\ubd80\ub97c \ubc1c\uc871\uc2dc\ucf30\uace0 2\ub144 \ud6c4\uc778 1984\ub144 5\uc6d4 23\uc77c \ubbf8\uad6d\uc758 \ub2e4\uad6d\uc801 \ud68c\uc0ac\uc778 \uc6e8\uc2a4\ud305\ud558\uc6b0\uc2a4\uc640 \ud569\uc791 \uacc4\uc57d\uc744 \ub9fa\uc5b4 \ud604\ub300\uc5d8\ub9ac\ubca0\uc774\ud130(\ucd08\ub300\uc0ac\uc7a5 \uc9c0\uc8fc\ud604)\ub97c \ucd9c\ubc94\ud558\uc600\ub2e4. \uc774\uc5b4 1986\ub144\uc5d0\ub294 \uc2e0\uc81c\ud488 \uac1c\ubc1c\uc758 \uc0b0\uc2e4\uc778 \uae30\uc220\uc5f0\uad6c\uc18c\ub97c \uc124\ub9bd\ud558\uc5ec, 100\uc5ec \uba85\uc758 \uc5f0\uad6c\uc6d0\uc774 \uc0c1\uc8fc\ud558\uba70 \uae30\uc220\uac1c\ubc1c\uc5d0 \ud798\uc4f0\ub3c4\ub85d \ud558\uc600\ub2e4.", "answer": "1986\ub144", "sentence": "\uc774\uc5b4 1986\ub144 \uc5d0\ub294 \uc2e0\uc81c\ud488 \uac1c\ubc1c\uc758 \uc0b0\uc2e4\uc778 \uae30\uc220\uc5f0\uad6c\uc18c\ub97c \uc124\ub9bd\ud558\uc5ec, 100\uc5ec \uba85\uc758 \uc5f0\uad6c\uc6d0\uc774 \uc0c1\uc8fc\ud558\uba70 \uae30\uc220\uac1c\ubc1c\uc5d0 \ud798\uc4f0\ub3c4\ub85d \ud558\uc600\ub2e4.", "paragraph_sentence": "1980\ub144\ub300 \uc2b9\uac15\uae30 \ubc1c\uc8fc\uac00 \ud3ed\uc99d\ud558\uba74\uc11c \uc2b9\uac15\uae30 \uc5c5\uacc4\ub294 \uae09\uc131\uc7a5\ud558\uc600\uc73c\ub098 \ub2f9\uc2dc \uae08\uc131\uc0ac, \uc2e0\uc601\uc804\uae30, \ub3d9\uc591\uc5d0\ub808\ubca0\uc774\ud130 \ub4f1\uc758 \ub300\ud615\uc0ac\uc640 \ud55c\uad6d\ud6c4\uc9c0\ud14c\ud06c, \ud55c\uad6d\uc2b9\uac15\uae30\uc81c\uc791\uc18c, \uc911\uc559\uc5d8\ub9ac\ubca0\uc774\ud130, \uc720\uc77c\uc5d8\ub9ac\ubca0\uc774\ud130, \ud55c\uad6d\uc5d4\uc9c0\ub2c8\uc5b4\ub9c1 \ub4f1\uc758 \uc911\uc18c\uc0ac\ub4e4\uc740 \uc2dc\uc124\uacfc \uc778\ub825\uc774 \ubd80\uc871\ud574 \uc218\uc694\uc5d0 \uc81c\ub300\ub85c \ubd80\uc751\ud558\uc9c0 \ubabb\ud558\ub294 \uc77c\uc774 \ubc1c\uc0dd\ud588\ub2e4. \ub2f9\uc2dc \ub300\ud55c\ubbfc\uad6d\uc758 \ucd5c\ub300 \uadf8\ub8f9 \uc911 \ud558\ub098\uc600\ub358 \ud604\ub300\ub294 \ud604\ub300\uac74\uc124\uc758 \uc790\uccb4 \uc2b9\uac15\uae30 \uc5f0\uac04 \uc18c\uc694\ub7c9\uc774 \uc218\ubc31 \ub300\uc5d0 \uc774\ub974\uace0 \uadf8\ub8f9 \ub0b4 \uc870\uc120\uc18c\uc758 \uc120\ubc15 \uac74\uc870\uc5d0 \ud544\uc694\ud55c \uc120\ubc15\uc6a9 \uc2b9\uac15\uae30\uc758 \uc218\uc694, \uadf8\ub9ac\uace0 \uadf8\ub8f9 \ud655\uc7a5\uc5d0 \ub530\ub978 \ud574\uc678 \uac74\uc124\uc2dc\uc7a5\uacfc \uc804\uad6d \uac01\uc9c0\uc5d0 \uac74\uc124\ub418\ub294 \uadf8\ub8f9 \uc0b0\ud558 \uac74\ubb3c\uc5d0 \ud544\uc694\ud55c \uc2b9\uac15\uae30 \uc218\uc694 \ub4f1\uc744 \uac10\uc548\ud560 \ub54c \uc2b9\uac15\uae30 \uc0b0\uc5c5\uc5d0 \ub300\ud55c \ucc38\uc5ec\uac00 \uc808\uc2e4\ud558\ub2e4\ub294 \uac83\uc744 \uae68\ub2ec\uc558\ub2e4. \uc774\uc5d0 1982\ub144 \ud604\ub300\ub294 \ud604\ub300\uc911\uc804\uae30 \ub0b4\uc5d0 \uc6b4\ubc18\uae30\uacc4 \uc0ac\uc5c5\ubcf8\ubd80\ub97c \ubc1c\uc871\uc2dc\ucf30\uace0 2\ub144 \ud6c4\uc778 1984\ub144 5\uc6d4 23\uc77c \ubbf8\uad6d\uc758 \ub2e4\uad6d\uc801 \ud68c\uc0ac\uc778 \uc6e8\uc2a4\ud305\ud558\uc6b0\uc2a4\uc640 \ud569\uc791 \uacc4\uc57d\uc744 \ub9fa\uc5b4 \ud604\ub300\uc5d8\ub9ac\ubca0\uc774\ud130(\ucd08\ub300\uc0ac\uc7a5 \uc9c0\uc8fc\ud604)\ub97c \ucd9c\ubc94\ud558\uc600\ub2e4. \uc774\uc5b4 1986\ub144 \uc5d0\ub294 \uc2e0\uc81c\ud488 \uac1c\ubc1c\uc758 \uc0b0\uc2e4\uc778 \uae30\uc220\uc5f0\uad6c\uc18c\ub97c \uc124\ub9bd\ud558\uc5ec, 100\uc5ec \uba85\uc758 \uc5f0\uad6c\uc6d0\uc774 \uc0c1\uc8fc\ud558\uba70 \uae30\uc220\uac1c\ubc1c\uc5d0 \ud798\uc4f0\ub3c4\ub85d \ud558\uc600\ub2e4. ", "paragraph_answer": "1980\ub144\ub300 \uc2b9\uac15\uae30 \ubc1c\uc8fc\uac00 \ud3ed\uc99d\ud558\uba74\uc11c \uc2b9\uac15\uae30 \uc5c5\uacc4\ub294 \uae09\uc131\uc7a5\ud558\uc600\uc73c\ub098 \ub2f9\uc2dc \uae08\uc131\uc0ac, \uc2e0\uc601\uc804\uae30, \ub3d9\uc591\uc5d0\ub808\ubca0\uc774\ud130 \ub4f1\uc758 \ub300\ud615\uc0ac\uc640 \ud55c\uad6d\ud6c4\uc9c0\ud14c\ud06c, \ud55c\uad6d\uc2b9\uac15\uae30\uc81c\uc791\uc18c, \uc911\uc559\uc5d8\ub9ac\ubca0\uc774\ud130, \uc720\uc77c\uc5d8\ub9ac\ubca0\uc774\ud130, \ud55c\uad6d\uc5d4\uc9c0\ub2c8\uc5b4\ub9c1 \ub4f1\uc758 \uc911\uc18c\uc0ac\ub4e4\uc740 \uc2dc\uc124\uacfc \uc778\ub825\uc774 \ubd80\uc871\ud574 \uc218\uc694\uc5d0 \uc81c\ub300\ub85c \ubd80\uc751\ud558\uc9c0 \ubabb\ud558\ub294 \uc77c\uc774 \ubc1c\uc0dd\ud588\ub2e4. \ub2f9\uc2dc \ub300\ud55c\ubbfc\uad6d\uc758 \ucd5c\ub300 \uadf8\ub8f9 \uc911 \ud558\ub098\uc600\ub358 \ud604\ub300\ub294 \ud604\ub300\uac74\uc124\uc758 \uc790\uccb4 \uc2b9\uac15\uae30 \uc5f0\uac04 \uc18c\uc694\ub7c9\uc774 \uc218\ubc31 \ub300\uc5d0 \uc774\ub974\uace0 \uadf8\ub8f9 \ub0b4 \uc870\uc120\uc18c\uc758 \uc120\ubc15 \uac74\uc870\uc5d0 \ud544\uc694\ud55c \uc120\ubc15\uc6a9 \uc2b9\uac15\uae30\uc758 \uc218\uc694, \uadf8\ub9ac\uace0 \uadf8\ub8f9 \ud655\uc7a5\uc5d0 \ub530\ub978 \ud574\uc678 \uac74\uc124\uc2dc\uc7a5\uacfc \uc804\uad6d \uac01\uc9c0\uc5d0 \uac74\uc124\ub418\ub294 \uadf8\ub8f9 \uc0b0\ud558 \uac74\ubb3c\uc5d0 \ud544\uc694\ud55c \uc2b9\uac15\uae30 \uc218\uc694 \ub4f1\uc744 \uac10\uc548\ud560 \ub54c \uc2b9\uac15\uae30 \uc0b0\uc5c5\uc5d0 \ub300\ud55c \ucc38\uc5ec\uac00 \uc808\uc2e4\ud558\ub2e4\ub294 \uac83\uc744 \uae68\ub2ec\uc558\ub2e4. \uc774\uc5d0 1982\ub144 \ud604\ub300\ub294 \ud604\ub300\uc911\uc804\uae30 \ub0b4\uc5d0 \uc6b4\ubc18\uae30\uacc4 \uc0ac\uc5c5\ubcf8\ubd80\ub97c \ubc1c\uc871\uc2dc\ucf30\uace0 2\ub144 \ud6c4\uc778 1984\ub144 5\uc6d4 23\uc77c \ubbf8\uad6d\uc758 \ub2e4\uad6d\uc801 \ud68c\uc0ac\uc778 \uc6e8\uc2a4\ud305\ud558\uc6b0\uc2a4\uc640 \ud569\uc791 \uacc4\uc57d\uc744 \ub9fa\uc5b4 \ud604\ub300\uc5d8\ub9ac\ubca0\uc774\ud130(\ucd08\ub300\uc0ac\uc7a5 \uc9c0\uc8fc\ud604)\ub97c \ucd9c\ubc94\ud558\uc600\ub2e4. \uc774\uc5b4 1986\ub144 \uc5d0\ub294 \uc2e0\uc81c\ud488 \uac1c\ubc1c\uc758 \uc0b0\uc2e4\uc778 \uae30\uc220\uc5f0\uad6c\uc18c\ub97c \uc124\ub9bd\ud558\uc5ec, 100\uc5ec \uba85\uc758 \uc5f0\uad6c\uc6d0\uc774 \uc0c1\uc8fc\ud558\uba70 \uae30\uc220\uac1c\ubc1c\uc5d0 \ud798\uc4f0\ub3c4\ub85d \ud558\uc600\ub2e4.", "sentence_answer": "\uc774\uc5b4 1986\ub144 \uc5d0\ub294 \uc2e0\uc81c\ud488 \uac1c\ubc1c\uc758 \uc0b0\uc2e4\uc778 \uae30\uc220\uc5f0\uad6c\uc18c\ub97c \uc124\ub9bd\ud558\uc5ec, 100\uc5ec \uba85\uc758 \uc5f0\uad6c\uc6d0\uc774 \uc0c1\uc8fc\ud558\uba70 \uae30\uc220\uac1c\ubc1c\uc5d0 \ud798\uc4f0\ub3c4\ub85d \ud558\uc600\ub2e4.", "paragraph_id": "6600939-0-1", "paragraph_question": "question: \uc5b8\uc81c \uc2e0\uc81c\ud488 \uac1c\ubc1c\uc744 \uc704\ud574 \uae30\uc220\uc5f0\uad6c\uc18c\ub97c \uc124\ub9bd\ud558\uc600\ub098?, context: 1980\ub144\ub300 \uc2b9\uac15\uae30 \ubc1c\uc8fc\uac00 \ud3ed\uc99d\ud558\uba74\uc11c \uc2b9\uac15\uae30 \uc5c5\uacc4\ub294 \uae09\uc131\uc7a5\ud558\uc600\uc73c\ub098 \ub2f9\uc2dc \uae08\uc131\uc0ac, \uc2e0\uc601\uc804\uae30, \ub3d9\uc591\uc5d0\ub808\ubca0\uc774\ud130 \ub4f1\uc758 \ub300\ud615\uc0ac\uc640 \ud55c\uad6d\ud6c4\uc9c0\ud14c\ud06c, \ud55c\uad6d\uc2b9\uac15\uae30\uc81c\uc791\uc18c, \uc911\uc559\uc5d8\ub9ac\ubca0\uc774\ud130, \uc720\uc77c\uc5d8\ub9ac\ubca0\uc774\ud130, \ud55c\uad6d\uc5d4\uc9c0\ub2c8\uc5b4\ub9c1 \ub4f1\uc758 \uc911\uc18c\uc0ac\ub4e4\uc740 \uc2dc\uc124\uacfc \uc778\ub825\uc774 \ubd80\uc871\ud574 \uc218\uc694\uc5d0 \uc81c\ub300\ub85c \ubd80\uc751\ud558\uc9c0 \ubabb\ud558\ub294 \uc77c\uc774 \ubc1c\uc0dd\ud588\ub2e4. \ub2f9\uc2dc \ub300\ud55c\ubbfc\uad6d\uc758 \ucd5c\ub300 \uadf8\ub8f9 \uc911 \ud558\ub098\uc600\ub358 \ud604\ub300\ub294 \ud604\ub300\uac74\uc124\uc758 \uc790\uccb4 \uc2b9\uac15\uae30 \uc5f0\uac04 \uc18c\uc694\ub7c9\uc774 \uc218\ubc31 \ub300\uc5d0 \uc774\ub974\uace0 \uadf8\ub8f9 \ub0b4 \uc870\uc120\uc18c\uc758 \uc120\ubc15 \uac74\uc870\uc5d0 \ud544\uc694\ud55c \uc120\ubc15\uc6a9 \uc2b9\uac15\uae30\uc758 \uc218\uc694, \uadf8\ub9ac\uace0 \uadf8\ub8f9 \ud655\uc7a5\uc5d0 \ub530\ub978 \ud574\uc678 \uac74\uc124\uc2dc\uc7a5\uacfc \uc804\uad6d \uac01\uc9c0\uc5d0 \uac74\uc124\ub418\ub294 \uadf8\ub8f9 \uc0b0\ud558 \uac74\ubb3c\uc5d0 \ud544\uc694\ud55c \uc2b9\uac15\uae30 \uc218\uc694 \ub4f1\uc744 \uac10\uc548\ud560 \ub54c \uc2b9\uac15\uae30 \uc0b0\uc5c5\uc5d0 \ub300\ud55c \ucc38\uc5ec\uac00 \uc808\uc2e4\ud558\ub2e4\ub294 \uac83\uc744 \uae68\ub2ec\uc558\ub2e4. \uc774\uc5d0 1982\ub144 \ud604\ub300\ub294 \ud604\ub300\uc911\uc804\uae30 \ub0b4\uc5d0 \uc6b4\ubc18\uae30\uacc4 \uc0ac\uc5c5\ubcf8\ubd80\ub97c \ubc1c\uc871\uc2dc\ucf30\uace0 2\ub144 \ud6c4\uc778 1984\ub144 5\uc6d4 23\uc77c \ubbf8\uad6d\uc758 \ub2e4\uad6d\uc801 \ud68c\uc0ac\uc778 \uc6e8\uc2a4\ud305\ud558\uc6b0\uc2a4\uc640 \ud569\uc791 \uacc4\uc57d\uc744 \ub9fa\uc5b4 \ud604\ub300\uc5d8\ub9ac\ubca0\uc774\ud130(\ucd08\ub300\uc0ac\uc7a5 \uc9c0\uc8fc\ud604)\ub97c \ucd9c\ubc94\ud558\uc600\ub2e4. \uc774\uc5b4 1986\ub144\uc5d0\ub294 \uc2e0\uc81c\ud488 \uac1c\ubc1c\uc758 \uc0b0\uc2e4\uc778 \uae30\uc220\uc5f0\uad6c\uc18c\ub97c \uc124\ub9bd\ud558\uc5ec, 100\uc5ec \uba85\uc758 \uc5f0\uad6c\uc6d0\uc774 \uc0c1\uc8fc\ud558\uba70 \uae30\uc220\uac1c\ubc1c\uc5d0 \ud798\uc4f0\ub3c4\ub85d \ud558\uc600\ub2e4."} {"question": "\uc5e0\ud30c\uc774\uc5b4\uc5d0\uc11c\ub294 \ub85c\uc2a4\ud2b8\ub97c \uc5ed\ub300 \ucd5c\uace0 \ub4dc\ub77c\ub9c8 \uba87\uac1c \uc911 \ud558\ub098\ub85c \uc120\uc815\ud558\uc600\ub294\uac00?", "paragraph": "\u300a\ub85c\uc2a4\ud2b8\u300b\ub294 \ub354 \ubcf4\uc2a4\ud134 \uae00\ub85c\ube0c\uc758 \ub9e4\ud29c \uae38\ubc84\ud2b8, \ud53c\ud50c \uc704\ud074\ub9ac\uc758 \ud1b0 \uae38\ub9ac\ub9c8\ud1a0, \uc0b0 \uc870\uc2a4 \uba38\ud050\ub9ac \ub274\uc2a4, USA \ud22c\ub370\uc774\uc758 \ub85c\ubc84\ud2b8 \ube44\uc548\ucf54\uac00 \uc8fc\ucd5c\ud55c \"Best of 2005 TV Coverage: Critic Top Ten Lists\"\uc5d0\uc11c 1\uc704\uc5d0 \uc120\uc815\ub418\uc5c8\ub2e4. \ud0c0\uc784\uc9c0\uc758 \uc81c\uc784\uc2a4 \ud3ec\ub2c8\uc5d0\uc6cc\uc9c0\ud06c\ub294 2007\ub144 \ubc29\uc601 \ub41c \ucd5c\uace0\uc758 \ub4dc\ub77c\ub9c8 \uc911 2\uc704\uc5d0 \uc120\uc815\ub418\uc5c8\ub2e4 \uadf8 \ud574, \ub610 \ud0c0\uc784\uc9c0\ub294 \uc5ed\ub300 \ucd5c\uace0\uc758 \ub4dc\ub77c\ub9c8 100\uac1c \uc911 \ud558\ub098\ub85c \uc120\uc815\ub418\uc5c8\ub2e4. \uc5e0\ud30c\uc774\uc5b4\uc5d0\uc11c\ub294 \u300a\ub85c\uc2a4\ud2b8\u300b\ub97c \uc5ed\ub300 \ucd5c\uace0\uc758 \ub4dc\ub77c\ub9c8 50\uac1c \uc911 \ud558\ub098\ub85c \uc120\uc815\ud588\ub2e4. \ub274\uc695 \ud0c0\uc784\uc988\uc758 \uae30\uc790 \ube4c \uce74\ud130\ub294 \u300a\ub85c\uc2a4\ud2b8\u300b\ub97c \"\uc774 \ub4dc\ub77c\ub9c8\ub294 \uc544\ub9c8 \ud154\ub808\ube44\uc804 \uc5f0\uc18d\uadf9 \uc5ed\uc0ac\uc5d0\uc11c \ub4dc\ub77c\ub9c8\ub97c \ub17c \ud560\ub54c \uce6d\ucc2c\ud558\uc9c0 \uc54a\uc744 \uc218 \uc5c6\ub294 \ub4dc\ub77c\ub9c8\ub2e4.\"\ub77c\uace0 \ud3c9\ud588\ub2e4. \uc778\uc0c1\uc801\uc778 \uc624\ud504\ub2dd\uc744 \ubcf4\uace0 \ub85c\uc774\ud130\ub294 \"\ud788\ud2b8 \ub4dc\ub77c\ub9c8\"\ub77c\uace0 \uc608\uc0c1\ud558\uba70 \"\ub77c\ub514\uc624\ub098 \uc2a4\ud398\uc15c \uc2a4\ud06c\ub9b0\uc744 \ud3ec\ud568, \u300a\ub85c\uc2a4\ud2b8\u300b\ub85c \uc778\ud574\uc11c ABC\uac00 \uadfc 5\ub144\uac04 \ucd5c\uace0\uc758 \uad11\uace0 \uc218\uc785\uc744 \uc62c\ub9ac\uace0 \uc788\ub2e4\"\uace0 \ubcf4\ub3c4\ud588\ub2e4.", "answer": "50", "sentence": "\uc5e0\ud30c\uc774\uc5b4\uc5d0\uc11c\ub294 \u300a\ub85c\uc2a4\ud2b8\u300b\ub97c \uc5ed\ub300 \ucd5c\uace0\uc758 \ub4dc\ub77c\ub9c8 50 \uac1c \uc911 \ud558\ub098\ub85c \uc120\uc815\ud588\ub2e4.", "paragraph_sentence": "\u300a\ub85c\uc2a4\ud2b8\u300b\ub294 \ub354 \ubcf4\uc2a4\ud134 \uae00\ub85c\ube0c\uc758 \ub9e4\ud29c \uae38\ubc84\ud2b8, \ud53c\ud50c \uc704\ud074\ub9ac\uc758 \ud1b0 \uae38\ub9ac\ub9c8\ud1a0, \uc0b0 \uc870\uc2a4 \uba38\ud050\ub9ac \ub274\uc2a4, USA \ud22c\ub370\uc774\uc758 \ub85c\ubc84\ud2b8 \ube44\uc548\ucf54\uac00 \uc8fc\ucd5c\ud55c \"Best of 2005 TV Coverage: Critic Top Ten Lists\"\uc5d0\uc11c 1\uc704\uc5d0 \uc120\uc815\ub418\uc5c8\ub2e4. \ud0c0\uc784\uc9c0\uc758 \uc81c\uc784\uc2a4 \ud3ec\ub2c8\uc5d0\uc6cc\uc9c0\ud06c\ub294 2007\ub144 \ubc29\uc601 \ub41c \ucd5c\uace0\uc758 \ub4dc\ub77c\ub9c8 \uc911 2\uc704\uc5d0 \uc120\uc815\ub418\uc5c8\ub2e4 \uadf8 \ud574, \ub610 \ud0c0\uc784\uc9c0\ub294 \uc5ed\ub300 \ucd5c\uace0\uc758 \ub4dc\ub77c\ub9c8 100\uac1c \uc911 \ud558\ub098\ub85c \uc120\uc815\ub418\uc5c8\ub2e4. \uc5e0\ud30c\uc774\uc5b4\uc5d0\uc11c\ub294 \u300a\ub85c\uc2a4\ud2b8\u300b\ub97c \uc5ed\ub300 \ucd5c\uace0\uc758 \ub4dc\ub77c\ub9c8 50 \uac1c \uc911 \ud558\ub098\ub85c \uc120\uc815\ud588\ub2e4. \ub274\uc695 \ud0c0\uc784\uc988\uc758 \uae30\uc790 \ube4c \uce74\ud130\ub294 \u300a\ub85c\uc2a4\ud2b8\u300b\ub97c \"\uc774 \ub4dc\ub77c\ub9c8\ub294 \uc544\ub9c8 \ud154\ub808\ube44\uc804 \uc5f0\uc18d\uadf9 \uc5ed\uc0ac\uc5d0\uc11c \ub4dc\ub77c\ub9c8\ub97c \ub17c \ud560\ub54c \uce6d\ucc2c\ud558\uc9c0 \uc54a\uc744 \uc218 \uc5c6\ub294 \ub4dc\ub77c\ub9c8\ub2e4. \"\ub77c\uace0 \ud3c9\ud588\ub2e4. \uc778\uc0c1\uc801\uc778 \uc624\ud504\ub2dd\uc744 \ubcf4\uace0 \ub85c\uc774\ud130\ub294 \"\ud788\ud2b8 \ub4dc\ub77c\ub9c8\"\ub77c\uace0 \uc608\uc0c1\ud558\uba70 \"\ub77c\ub514\uc624\ub098 \uc2a4\ud398\uc15c \uc2a4\ud06c\ub9b0\uc744 \ud3ec\ud568, \u300a\ub85c\uc2a4\ud2b8\u300b\ub85c \uc778\ud574\uc11c ABC\uac00 \uadfc 5\ub144\uac04 \ucd5c\uace0\uc758 \uad11\uace0 \uc218\uc785\uc744 \uc62c\ub9ac\uace0 \uc788\ub2e4\"\uace0 \ubcf4\ub3c4\ud588\ub2e4.", "paragraph_answer": "\u300a\ub85c\uc2a4\ud2b8\u300b\ub294 \ub354 \ubcf4\uc2a4\ud134 \uae00\ub85c\ube0c\uc758 \ub9e4\ud29c \uae38\ubc84\ud2b8, \ud53c\ud50c \uc704\ud074\ub9ac\uc758 \ud1b0 \uae38\ub9ac\ub9c8\ud1a0, \uc0b0 \uc870\uc2a4 \uba38\ud050\ub9ac \ub274\uc2a4, USA \ud22c\ub370\uc774\uc758 \ub85c\ubc84\ud2b8 \ube44\uc548\ucf54\uac00 \uc8fc\ucd5c\ud55c \"Best of 2005 TV Coverage: Critic Top Ten Lists\"\uc5d0\uc11c 1\uc704\uc5d0 \uc120\uc815\ub418\uc5c8\ub2e4. \ud0c0\uc784\uc9c0\uc758 \uc81c\uc784\uc2a4 \ud3ec\ub2c8\uc5d0\uc6cc\uc9c0\ud06c\ub294 2007\ub144 \ubc29\uc601 \ub41c \ucd5c\uace0\uc758 \ub4dc\ub77c\ub9c8 \uc911 2\uc704\uc5d0 \uc120\uc815\ub418\uc5c8\ub2e4 \uadf8 \ud574, \ub610 \ud0c0\uc784\uc9c0\ub294 \uc5ed\ub300 \ucd5c\uace0\uc758 \ub4dc\ub77c\ub9c8 100\uac1c \uc911 \ud558\ub098\ub85c \uc120\uc815\ub418\uc5c8\ub2e4. \uc5e0\ud30c\uc774\uc5b4\uc5d0\uc11c\ub294 \u300a\ub85c\uc2a4\ud2b8\u300b\ub97c \uc5ed\ub300 \ucd5c\uace0\uc758 \ub4dc\ub77c\ub9c8 50 \uac1c \uc911 \ud558\ub098\ub85c \uc120\uc815\ud588\ub2e4. \ub274\uc695 \ud0c0\uc784\uc988\uc758 \uae30\uc790 \ube4c \uce74\ud130\ub294 \u300a\ub85c\uc2a4\ud2b8\u300b\ub97c \"\uc774 \ub4dc\ub77c\ub9c8\ub294 \uc544\ub9c8 \ud154\ub808\ube44\uc804 \uc5f0\uc18d\uadf9 \uc5ed\uc0ac\uc5d0\uc11c \ub4dc\ub77c\ub9c8\ub97c \ub17c \ud560\ub54c \uce6d\ucc2c\ud558\uc9c0 \uc54a\uc744 \uc218 \uc5c6\ub294 \ub4dc\ub77c\ub9c8\ub2e4.\"\ub77c\uace0 \ud3c9\ud588\ub2e4. \uc778\uc0c1\uc801\uc778 \uc624\ud504\ub2dd\uc744 \ubcf4\uace0 \ub85c\uc774\ud130\ub294 \"\ud788\ud2b8 \ub4dc\ub77c\ub9c8\"\ub77c\uace0 \uc608\uc0c1\ud558\uba70 \"\ub77c\ub514\uc624\ub098 \uc2a4\ud398\uc15c \uc2a4\ud06c\ub9b0\uc744 \ud3ec\ud568, \u300a\ub85c\uc2a4\ud2b8\u300b\ub85c \uc778\ud574\uc11c ABC\uac00 \uadfc 5\ub144\uac04 \ucd5c\uace0\uc758 \uad11\uace0 \uc218\uc785\uc744 \uc62c\ub9ac\uace0 \uc788\ub2e4\"\uace0 \ubcf4\ub3c4\ud588\ub2e4.", "sentence_answer": "\uc5e0\ud30c\uc774\uc5b4\uc5d0\uc11c\ub294 \u300a\ub85c\uc2a4\ud2b8\u300b\ub97c \uc5ed\ub300 \ucd5c\uace0\uc758 \ub4dc\ub77c\ub9c8 50 \uac1c \uc911 \ud558\ub098\ub85c \uc120\uc815\ud588\ub2e4.", "paragraph_id": "6465056-8-1", "paragraph_question": "question: \uc5e0\ud30c\uc774\uc5b4\uc5d0\uc11c\ub294 \ub85c\uc2a4\ud2b8\ub97c \uc5ed\ub300 \ucd5c\uace0 \ub4dc\ub77c\ub9c8 \uba87\uac1c \uc911 \ud558\ub098\ub85c \uc120\uc815\ud558\uc600\ub294\uac00?, context: \u300a\ub85c\uc2a4\ud2b8\u300b\ub294 \ub354 \ubcf4\uc2a4\ud134 \uae00\ub85c\ube0c\uc758 \ub9e4\ud29c \uae38\ubc84\ud2b8, \ud53c\ud50c \uc704\ud074\ub9ac\uc758 \ud1b0 \uae38\ub9ac\ub9c8\ud1a0, \uc0b0 \uc870\uc2a4 \uba38\ud050\ub9ac \ub274\uc2a4, USA \ud22c\ub370\uc774\uc758 \ub85c\ubc84\ud2b8 \ube44\uc548\ucf54\uac00 \uc8fc\ucd5c\ud55c \"Best of 2005 TV Coverage: Critic Top Ten Lists\"\uc5d0\uc11c 1\uc704\uc5d0 \uc120\uc815\ub418\uc5c8\ub2e4. \ud0c0\uc784\uc9c0\uc758 \uc81c\uc784\uc2a4 \ud3ec\ub2c8\uc5d0\uc6cc\uc9c0\ud06c\ub294 2007\ub144 \ubc29\uc601 \ub41c \ucd5c\uace0\uc758 \ub4dc\ub77c\ub9c8 \uc911 2\uc704\uc5d0 \uc120\uc815\ub418\uc5c8\ub2e4 \uadf8 \ud574, \ub610 \ud0c0\uc784\uc9c0\ub294 \uc5ed\ub300 \ucd5c\uace0\uc758 \ub4dc\ub77c\ub9c8 100\uac1c \uc911 \ud558\ub098\ub85c \uc120\uc815\ub418\uc5c8\ub2e4. \uc5e0\ud30c\uc774\uc5b4\uc5d0\uc11c\ub294 \u300a\ub85c\uc2a4\ud2b8\u300b\ub97c \uc5ed\ub300 \ucd5c\uace0\uc758 \ub4dc\ub77c\ub9c8 50\uac1c \uc911 \ud558\ub098\ub85c \uc120\uc815\ud588\ub2e4. \ub274\uc695 \ud0c0\uc784\uc988\uc758 \uae30\uc790 \ube4c \uce74\ud130\ub294 \u300a\ub85c\uc2a4\ud2b8\u300b\ub97c \"\uc774 \ub4dc\ub77c\ub9c8\ub294 \uc544\ub9c8 \ud154\ub808\ube44\uc804 \uc5f0\uc18d\uadf9 \uc5ed\uc0ac\uc5d0\uc11c \ub4dc\ub77c\ub9c8\ub97c \ub17c \ud560\ub54c \uce6d\ucc2c\ud558\uc9c0 \uc54a\uc744 \uc218 \uc5c6\ub294 \ub4dc\ub77c\ub9c8\ub2e4.\"\ub77c\uace0 \ud3c9\ud588\ub2e4. \uc778\uc0c1\uc801\uc778 \uc624\ud504\ub2dd\uc744 \ubcf4\uace0 \ub85c\uc774\ud130\ub294 \"\ud788\ud2b8 \ub4dc\ub77c\ub9c8\"\ub77c\uace0 \uc608\uc0c1\ud558\uba70 \"\ub77c\ub514\uc624\ub098 \uc2a4\ud398\uc15c \uc2a4\ud06c\ub9b0\uc744 \ud3ec\ud568, \u300a\ub85c\uc2a4\ud2b8\u300b\ub85c \uc778\ud574\uc11c ABC\uac00 \uadfc 5\ub144\uac04 \ucd5c\uace0\uc758 \uad11\uace0 \uc218\uc785\uc744 \uc62c\ub9ac\uace0 \uc788\ub2e4\"\uace0 \ubcf4\ub3c4\ud588\ub2e4."} {"question": "\ubaa8\uc544\uc2dc\ub974 \ud398\ub808\uc774\ub77c \uac10\ub3c5\uc740 \uc5b4\ub290 \ub098\ub77c\uc5d0\uc11c U-20, U-23 \uad6d\uac00\ub300\ud45c\ud300 \uc218\uc11d\ucf54\uce58\uc9c1\uc744 \uc218\ud589\ud55c \uacbd\ud5d8\uc774 \uc788\ub098?", "paragraph": "\ube0c\ub77c\uc9c8 \uc720\uc218\uc758 \uba85\ubb38 \ud074\ub7fd\uacfc \ube0c\ub77c\uc9c8 U-20, U-23 \uad6d\uac00\ub300\ud45c\ud300\uc758 \uc218\uc11d\ucf54\uce58\uc9c1\uc744 \uc218\ud589\ud55c \uacbd\ud5d8\uc774 \uc788\ub294 \ubaa8\uc544\uc2dc\ub974 \ud398\ub808\uc774\ub77c \uac10\ub3c5\uc740 \uc790\uad6d \uc120\uc218\ub4e4\uc5d0 \ub300\ud55c \uc815\ubcf4\ub825\uc744 \ubc14\ud0d5\uc73c\ub85c \ube0c\ub77c\uc9c8 \uccad\uc18c\ub144 \ub300\ud45c\ud300 \ucd9c\uc2e0\uc758 \ub808\uc548\ub4dc\ub8e8 \ub9ac\ub9c8\uc640 \ucf54\ub9b0\uce58\uc559\uc2a4 \uc2dc\uc808 \uc720\ub9dd\uc8fc\ub85c \uc774\ub984\uc744 \ub0a0\ub838\ub358 \uc9c0\ub124\uc6b0\uc1a1\uc744 \uc601\uc785\ud558\uba70 \ud0c4\ud0c4\ud55c \uc678\uad6d\uc778 \uc120\uc218 \ub77c\uc778\uc744 \uad6c\ucd95\ud558\uc600\uace0 \uc774\uc9c4\ud638, \uc720\uacbd\ub82c \ub4f1\uc744 \uc601\uc785\ud558\uba70 \uacf5\uc218\uc5d0 \uac78\uce5c \ubcf4\uac15\uc744 \uc2e4\uc2dc\ud558\uc600\ub2e4. \uac1c\ub9c9 \uc774\ud6c4 \uafb8\uc900\ud788 \uc911\uc704\uad8c \uc21c\uc704\ub97c \uc720\uc9c0\ud558\uba70 \uae30\uc5c5 \uad6c\ub2e8\uc744 \uc81c\uc678\ud558\uace0\ub294 \uc720\uc77c\ud558\uac8c \uc0c1\uc704 \uc2a4\ud50c\ub9bf\uc5d0 \uc18d\ud558\ub294 8\uc704 \uc774\ub0b4\uc758 \uc21c\uc704\ub97c \uc720\uc9c0\ud574 \uc654\uc73c\ub098 28\ub77c\uc6b4\ub4dc\uc5d0 9\uc704\ub85c \ud558\ub77d\ud55c \ud6c4 \uc2a4\ud50c\ub9bf\uc744 \ub098\ub204\ub294 \ub9c8\uc9c0\ub9c9 \ub77c\uc6b4\ub4dc\uc778 30\ub77c\uc6b4\ub4dc\uc5d0\uc11c \uc11c\uc6b8\uc5d0 \ud328\ud558\uba70 \ud558\uc704 \uc2a4\ud50c\ub9bf\uc5d0 \uc18d\ud558\uac8c \ub418\uc5c8\ub2e4. \ud558\uc9c0\ub9cc \uc2e4\uad00\uc911 \uc9d1\uacc4\ub85c \ub300\ub2e4\uc218 K\ub9ac\uadf8 \ud300\ub4e4\uc758 \ud3c9\uade0 \uad00\uc911 \uc218\uac00 \ud558\ub77d\ud55c \uc0c1\ud669\uc5d0\uc11c \ud3c9\uade0 \uad00\uc911 \uc218 \uc0c1\uc2b9\uc744 \uae30\ub85d\ud558\uc5ec \uc9c0\uc5ed \ud32c\ub4e4\uc758 \uad00\uc2ec\uc744 \ub04c\uc5b4\ubaa8\uc73c\ub294 \ub370 \uc131\uacf5\ud558\uc600\ub2e4.", "answer": "\ube0c\ub77c\uc9c8", "sentence": "\ube0c\ub77c\uc9c8 \uc720\uc218\uc758 \uba85\ubb38 \ud074\ub7fd\uacfc \ube0c\ub77c\uc9c8 U-20, U-23 \uad6d\uac00\ub300\ud45c\ud300\uc758 \uc218\uc11d\ucf54\uce58\uc9c1\uc744 \uc218\ud589\ud55c \uacbd\ud5d8\uc774 \uc788\ub294 \ubaa8\uc544\uc2dc\ub974 \ud398\ub808\uc774\ub77c \uac10\ub3c5\uc740 \uc790\uad6d \uc120\uc218\ub4e4\uc5d0 \ub300\ud55c \uc815\ubcf4\ub825\uc744 \ubc14\ud0d5\uc73c\ub85c \ube0c\ub77c\uc9c8 \uccad\uc18c\ub144 \ub300\ud45c\ud300 \ucd9c\uc2e0\uc758 \ub808\uc548\ub4dc\ub8e8 \ub9ac\ub9c8\uc640 \ucf54\ub9b0\uce58\uc559\uc2a4 \uc2dc\uc808 \uc720\ub9dd\uc8fc\ub85c \uc774\ub984\uc744 \ub0a0\ub838\ub358 \uc9c0\ub124\uc6b0\uc1a1\uc744 \uc601\uc785\ud558\uba70 \ud0c4\ud0c4\ud55c \uc678\uad6d\uc778 \uc120\uc218 \ub77c\uc778\uc744 \uad6c\ucd95\ud558\uc600\uace0 \uc774\uc9c4\ud638, \uc720\uacbd\ub82c \ub4f1\uc744 \uc601\uc785\ud558\uba70 \uacf5\uc218\uc5d0 \uac78\uce5c \ubcf4\uac15\uc744 \uc2e4\uc2dc\ud558\uc600\ub2e4.", "paragraph_sentence": " \ube0c\ub77c\uc9c8 \uc720\uc218\uc758 \uba85\ubb38 \ud074\ub7fd\uacfc \ube0c\ub77c\uc9c8 U-20, U-23 \uad6d\uac00\ub300\ud45c\ud300\uc758 \uc218\uc11d\ucf54\uce58\uc9c1\uc744 \uc218\ud589\ud55c \uacbd\ud5d8\uc774 \uc788\ub294 \ubaa8\uc544\uc2dc\ub974 \ud398\ub808\uc774\ub77c \uac10\ub3c5\uc740 \uc790\uad6d \uc120\uc218\ub4e4\uc5d0 \ub300\ud55c \uc815\ubcf4\ub825\uc744 \ubc14\ud0d5\uc73c\ub85c \ube0c\ub77c\uc9c8 \uccad\uc18c\ub144 \ub300\ud45c\ud300 \ucd9c\uc2e0\uc758 \ub808\uc548\ub4dc\ub8e8 \ub9ac\ub9c8\uc640 \ucf54\ub9b0\uce58\uc559\uc2a4 \uc2dc\uc808 \uc720\ub9dd\uc8fc\ub85c \uc774\ub984\uc744 \ub0a0\ub838\ub358 \uc9c0\ub124\uc6b0\uc1a1\uc744 \uc601\uc785\ud558\uba70 \ud0c4\ud0c4\ud55c \uc678\uad6d\uc778 \uc120\uc218 \ub77c\uc778\uc744 \uad6c\ucd95\ud558\uc600\uace0 \uc774\uc9c4\ud638, \uc720\uacbd\ub82c \ub4f1\uc744 \uc601\uc785\ud558\uba70 \uacf5\uc218\uc5d0 \uac78\uce5c \ubcf4\uac15\uc744 \uc2e4\uc2dc\ud558\uc600\ub2e4. \uac1c\ub9c9 \uc774\ud6c4 \uafb8\uc900\ud788 \uc911\uc704\uad8c \uc21c\uc704\ub97c \uc720\uc9c0\ud558\uba70 \uae30\uc5c5 \uad6c\ub2e8\uc744 \uc81c\uc678\ud558\uace0\ub294 \uc720\uc77c\ud558\uac8c \uc0c1\uc704 \uc2a4\ud50c\ub9bf\uc5d0 \uc18d\ud558\ub294 8\uc704 \uc774\ub0b4\uc758 \uc21c\uc704\ub97c \uc720\uc9c0\ud574 \uc654\uc73c\ub098 28\ub77c\uc6b4\ub4dc\uc5d0 9\uc704\ub85c \ud558\ub77d\ud55c \ud6c4 \uc2a4\ud50c\ub9bf\uc744 \ub098\ub204\ub294 \ub9c8\uc9c0\ub9c9 \ub77c\uc6b4\ub4dc\uc778 30\ub77c\uc6b4\ub4dc\uc5d0\uc11c \uc11c\uc6b8\uc5d0 \ud328\ud558\uba70 \ud558\uc704 \uc2a4\ud50c\ub9bf\uc5d0 \uc18d\ud558\uac8c \ub418\uc5c8\ub2e4. \ud558\uc9c0\ub9cc \uc2e4\uad00\uc911 \uc9d1\uacc4\ub85c \ub300\ub2e4\uc218 K\ub9ac\uadf8 \ud300\ub4e4\uc758 \ud3c9\uade0 \uad00\uc911 \uc218\uac00 \ud558\ub77d\ud55c \uc0c1\ud669\uc5d0\uc11c \ud3c9\uade0 \uad00\uc911 \uc218 \uc0c1\uc2b9\uc744 \uae30\ub85d\ud558\uc5ec \uc9c0\uc5ed \ud32c\ub4e4\uc758 \uad00\uc2ec\uc744 \ub04c\uc5b4\ubaa8\uc73c\ub294 \ub370 \uc131\uacf5\ud558\uc600\ub2e4.", "paragraph_answer": " \ube0c\ub77c\uc9c8 \uc720\uc218\uc758 \uba85\ubb38 \ud074\ub7fd\uacfc \ube0c\ub77c\uc9c8 U-20, U-23 \uad6d\uac00\ub300\ud45c\ud300\uc758 \uc218\uc11d\ucf54\uce58\uc9c1\uc744 \uc218\ud589\ud55c \uacbd\ud5d8\uc774 \uc788\ub294 \ubaa8\uc544\uc2dc\ub974 \ud398\ub808\uc774\ub77c \uac10\ub3c5\uc740 \uc790\uad6d \uc120\uc218\ub4e4\uc5d0 \ub300\ud55c \uc815\ubcf4\ub825\uc744 \ubc14\ud0d5\uc73c\ub85c \ube0c\ub77c\uc9c8 \uccad\uc18c\ub144 \ub300\ud45c\ud300 \ucd9c\uc2e0\uc758 \ub808\uc548\ub4dc\ub8e8 \ub9ac\ub9c8\uc640 \ucf54\ub9b0\uce58\uc559\uc2a4 \uc2dc\uc808 \uc720\ub9dd\uc8fc\ub85c \uc774\ub984\uc744 \ub0a0\ub838\ub358 \uc9c0\ub124\uc6b0\uc1a1\uc744 \uc601\uc785\ud558\uba70 \ud0c4\ud0c4\ud55c \uc678\uad6d\uc778 \uc120\uc218 \ub77c\uc778\uc744 \uad6c\ucd95\ud558\uc600\uace0 \uc774\uc9c4\ud638, \uc720\uacbd\ub82c \ub4f1\uc744 \uc601\uc785\ud558\uba70 \uacf5\uc218\uc5d0 \uac78\uce5c \ubcf4\uac15\uc744 \uc2e4\uc2dc\ud558\uc600\ub2e4. \uac1c\ub9c9 \uc774\ud6c4 \uafb8\uc900\ud788 \uc911\uc704\uad8c \uc21c\uc704\ub97c \uc720\uc9c0\ud558\uba70 \uae30\uc5c5 \uad6c\ub2e8\uc744 \uc81c\uc678\ud558\uace0\ub294 \uc720\uc77c\ud558\uac8c \uc0c1\uc704 \uc2a4\ud50c\ub9bf\uc5d0 \uc18d\ud558\ub294 8\uc704 \uc774\ub0b4\uc758 \uc21c\uc704\ub97c \uc720\uc9c0\ud574 \uc654\uc73c\ub098 28\ub77c\uc6b4\ub4dc\uc5d0 9\uc704\ub85c \ud558\ub77d\ud55c \ud6c4 \uc2a4\ud50c\ub9bf\uc744 \ub098\ub204\ub294 \ub9c8\uc9c0\ub9c9 \ub77c\uc6b4\ub4dc\uc778 30\ub77c\uc6b4\ub4dc\uc5d0\uc11c \uc11c\uc6b8\uc5d0 \ud328\ud558\uba70 \ud558\uc704 \uc2a4\ud50c\ub9bf\uc5d0 \uc18d\ud558\uac8c \ub418\uc5c8\ub2e4. \ud558\uc9c0\ub9cc \uc2e4\uad00\uc911 \uc9d1\uacc4\ub85c \ub300\ub2e4\uc218 K\ub9ac\uadf8 \ud300\ub4e4\uc758 \ud3c9\uade0 \uad00\uc911 \uc218\uac00 \ud558\ub77d\ud55c \uc0c1\ud669\uc5d0\uc11c \ud3c9\uade0 \uad00\uc911 \uc218 \uc0c1\uc2b9\uc744 \uae30\ub85d\ud558\uc5ec \uc9c0\uc5ed \ud32c\ub4e4\uc758 \uad00\uc2ec\uc744 \ub04c\uc5b4\ubaa8\uc73c\ub294 \ub370 \uc131\uacf5\ud558\uc600\ub2e4.", "sentence_answer": " \ube0c\ub77c\uc9c8 \uc720\uc218\uc758 \uba85\ubb38 \ud074\ub7fd\uacfc \ube0c\ub77c\uc9c8 U-20, U-23 \uad6d\uac00\ub300\ud45c\ud300\uc758 \uc218\uc11d\ucf54\uce58\uc9c1\uc744 \uc218\ud589\ud55c \uacbd\ud5d8\uc774 \uc788\ub294 \ubaa8\uc544\uc2dc\ub974 \ud398\ub808\uc774\ub77c \uac10\ub3c5\uc740 \uc790\uad6d \uc120\uc218\ub4e4\uc5d0 \ub300\ud55c \uc815\ubcf4\ub825\uc744 \ubc14\ud0d5\uc73c\ub85c \ube0c\ub77c\uc9c8 \uccad\uc18c\ub144 \ub300\ud45c\ud300 \ucd9c\uc2e0\uc758 \ub808\uc548\ub4dc\ub8e8 \ub9ac\ub9c8\uc640 \ucf54\ub9b0\uce58\uc559\uc2a4 \uc2dc\uc808 \uc720\ub9dd\uc8fc\ub85c \uc774\ub984\uc744 \ub0a0\ub838\ub358 \uc9c0\ub124\uc6b0\uc1a1\uc744 \uc601\uc785\ud558\uba70 \ud0c4\ud0c4\ud55c \uc678\uad6d\uc778 \uc120\uc218 \ub77c\uc778\uc744 \uad6c\ucd95\ud558\uc600\uace0 \uc774\uc9c4\ud638, \uc720\uacbd\ub82c \ub4f1\uc744 \uc601\uc785\ud558\uba70 \uacf5\uc218\uc5d0 \uac78\uce5c \ubcf4\uac15\uc744 \uc2e4\uc2dc\ud558\uc600\ub2e4.", "paragraph_id": "6546195-4-0", "paragraph_question": "question: \ubaa8\uc544\uc2dc\ub974 \ud398\ub808\uc774\ub77c \uac10\ub3c5\uc740 \uc5b4\ub290 \ub098\ub77c\uc5d0\uc11c U-20, U-23 \uad6d\uac00\ub300\ud45c\ud300 \uc218\uc11d\ucf54\uce58\uc9c1\uc744 \uc218\ud589\ud55c \uacbd\ud5d8\uc774 \uc788\ub098?, context: \ube0c\ub77c\uc9c8 \uc720\uc218\uc758 \uba85\ubb38 \ud074\ub7fd\uacfc \ube0c\ub77c\uc9c8 U-20, U-23 \uad6d\uac00\ub300\ud45c\ud300\uc758 \uc218\uc11d\ucf54\uce58\uc9c1\uc744 \uc218\ud589\ud55c \uacbd\ud5d8\uc774 \uc788\ub294 \ubaa8\uc544\uc2dc\ub974 \ud398\ub808\uc774\ub77c \uac10\ub3c5\uc740 \uc790\uad6d \uc120\uc218\ub4e4\uc5d0 \ub300\ud55c \uc815\ubcf4\ub825\uc744 \ubc14\ud0d5\uc73c\ub85c \ube0c\ub77c\uc9c8 \uccad\uc18c\ub144 \ub300\ud45c\ud300 \ucd9c\uc2e0\uc758 \ub808\uc548\ub4dc\ub8e8 \ub9ac\ub9c8\uc640 \ucf54\ub9b0\uce58\uc559\uc2a4 \uc2dc\uc808 \uc720\ub9dd\uc8fc\ub85c \uc774\ub984\uc744 \ub0a0\ub838\ub358 \uc9c0\ub124\uc6b0\uc1a1\uc744 \uc601\uc785\ud558\uba70 \ud0c4\ud0c4\ud55c \uc678\uad6d\uc778 \uc120\uc218 \ub77c\uc778\uc744 \uad6c\ucd95\ud558\uc600\uace0 \uc774\uc9c4\ud638, \uc720\uacbd\ub82c \ub4f1\uc744 \uc601\uc785\ud558\uba70 \uacf5\uc218\uc5d0 \uac78\uce5c \ubcf4\uac15\uc744 \uc2e4\uc2dc\ud558\uc600\ub2e4. \uac1c\ub9c9 \uc774\ud6c4 \uafb8\uc900\ud788 \uc911\uc704\uad8c \uc21c\uc704\ub97c \uc720\uc9c0\ud558\uba70 \uae30\uc5c5 \uad6c\ub2e8\uc744 \uc81c\uc678\ud558\uace0\ub294 \uc720\uc77c\ud558\uac8c \uc0c1\uc704 \uc2a4\ud50c\ub9bf\uc5d0 \uc18d\ud558\ub294 8\uc704 \uc774\ub0b4\uc758 \uc21c\uc704\ub97c \uc720\uc9c0\ud574 \uc654\uc73c\ub098 28\ub77c\uc6b4\ub4dc\uc5d0 9\uc704\ub85c \ud558\ub77d\ud55c \ud6c4 \uc2a4\ud50c\ub9bf\uc744 \ub098\ub204\ub294 \ub9c8\uc9c0\ub9c9 \ub77c\uc6b4\ub4dc\uc778 30\ub77c\uc6b4\ub4dc\uc5d0\uc11c \uc11c\uc6b8\uc5d0 \ud328\ud558\uba70 \ud558\uc704 \uc2a4\ud50c\ub9bf\uc5d0 \uc18d\ud558\uac8c \ub418\uc5c8\ub2e4. \ud558\uc9c0\ub9cc \uc2e4\uad00\uc911 \uc9d1\uacc4\ub85c \ub300\ub2e4\uc218 K\ub9ac\uadf8 \ud300\ub4e4\uc758 \ud3c9\uade0 \uad00\uc911 \uc218\uac00 \ud558\ub77d\ud55c \uc0c1\ud669\uc5d0\uc11c \ud3c9\uade0 \uad00\uc911 \uc218 \uc0c1\uc2b9\uc744 \uae30\ub85d\ud558\uc5ec \uc9c0\uc5ed \ud32c\ub4e4\uc758 \uad00\uc2ec\uc744 \ub04c\uc5b4\ubaa8\uc73c\ub294 \ub370 \uc131\uacf5\ud558\uc600\ub2e4."} {"question": "\uc870\uc601\ub0a8\uc774 \uc790\ub2c8\uc724 \uc1fc\uc5d0 \ubcf4\uc870 \uc9c4\ud589\uc790\ub85c \ud65c\ub3d9\ud588\ub358 \uac83\uc740 \uc5b8\uc81c\uc778\uac00?", "paragraph": "1989\ub144\uc5d0 \uc790\ub2c8 \uc724\uc774 \uc9c4\ud589\ud558\ub294 KBS2 \u300a\uc790\ub2c8\uc724 \uc1fc\u300b\uc5d0 \ubcf4\uc870 \uc9c4\ud589\uc790\ub85c \ud65c\ub3d9\ud55c \ud6c4 1990\ub144\ub300\uc5d0 \ub4e4\uc5b4\uc11c\uba74\uc11c \uac00\uc218\ub85c\uc11c\uc758 \uc870\uc601\ub0a8\ubcf4\ub2e4\ub294 \ubc29\uc1a1\uc778\uc73c\ub85c\uc11c\uc758 \uc870\uc601\ub0a8\uc774 \ub300\uc911\ub4e4\uc5d0\uac8c \ub9ce\uc740 \uad00\uc2ec\uc744 \ubc1b\uc558\ub2e4. \ub300\ud45c\uc801\uc73c\ub85c 1993\ub144\uc5d0 \uc774\uacbd\uc2e4\uacfc \ud568\uaed8 1\ub300 \uc9c4\ud589\uc790\ub97c \ub9e1\uc740 KBS2 \u300a\uccb4\ud5d8 \uc0b6\uc758 \ud604\uc7a5\u300b\uc774 \uc8fc\ubaa9\uc744 \ubc1b\uc558\ub2e4. \uc724\uc5ec\uc815\uacfc \uc774\ud63c \ud6c4 1995\ub144\uc5d0 \ubc31\uc740\uc2e4\uacfc \uacb0\ud63c\ud558\uace0 \ub538 \uc870\uc740\uc9c0\ub97c \uc785\uc591\ud558\uba70 \uc0c8\ub85c\uc6b4 \uac00\uc815\uc744 \uafb8\ub838\uc9c0\ub9cc \ud604\uc7ac\ub294 \ub538\uacfc \uac19\uc774 \uc0dd\ud65c\ud558\uace0 \uc788\ub2e4. \uadf8\ub9ac\uace0 1990\ub144\ub300\uc5d0 \uc77c\uc5b4\ub09c \uc640\uc6b0 \uc544\ud30c\ud2b8 \ubd95\uad34\uc0ac\uac74\uc774 \uc77c\uc5b4\ub0ac\uc744\ub54c \ud568\uacbd\ub3c4 \uc2e0\ubbfc\uc694 \uc2e0\uace0\uc0b0 \ud0c0\ub839\uc744 \uac00\uc0ac\ub9cc \ubc14\uafb8\uc5b4 \"\uc640\uc6b0 \uc544\ud30c\ud2b8\uac00 \uc6b0\ub8e8\ub8e8\ub8e8\ub8e8\ub8e8 \ubb34\ub108\uc9c0\ub294 \uc18c\ub9ac\uc5d0...\"\ub85c \uc2dc\uc791\ud558\ub294 \ub178\ub798\ub85c \ubd88\ub7ec \uadf8 \uccab\uad6c\uc808\uc774 \ubb38\uc81c\uac00 \ub418\uc5b4 \uc2e0\ubb38\uacfc \ub274\uc218\uc5d0 \ub098\uae30\ub3c4 \ud558\uc5ec \ud070 \ud654\uc81c\uac00 \ub418\uc5c8\ub2e4.", "answer": "1989\ub144", "sentence": "1989\ub144 \uc5d0 \uc790\ub2c8 \uc724\uc774 \uc9c4\ud589\ud558\ub294 KBS2 \u300a\uc790\ub2c8\uc724 \uc1fc\u300b\uc5d0 \ubcf4\uc870 \uc9c4\ud589\uc790\ub85c \ud65c\ub3d9\ud55c \ud6c4 1990\ub144\ub300\uc5d0 \ub4e4\uc5b4\uc11c\uba74\uc11c \uac00\uc218\ub85c\uc11c\uc758 \uc870\uc601\ub0a8\ubcf4\ub2e4\ub294 \ubc29\uc1a1\uc778\uc73c\ub85c\uc11c\uc758 \uc870\uc601\ub0a8\uc774 \ub300\uc911\ub4e4\uc5d0\uac8c \ub9ce\uc740 \uad00\uc2ec\uc744 \ubc1b\uc558\ub2e4.", "paragraph_sentence": " 1989\ub144 \uc5d0 \uc790\ub2c8 \uc724\uc774 \uc9c4\ud589\ud558\ub294 KBS2 \u300a\uc790\ub2c8\uc724 \uc1fc\u300b\uc5d0 \ubcf4\uc870 \uc9c4\ud589\uc790\ub85c \ud65c\ub3d9\ud55c \ud6c4 1990\ub144\ub300\uc5d0 \ub4e4\uc5b4\uc11c\uba74\uc11c \uac00\uc218\ub85c\uc11c\uc758 \uc870\uc601\ub0a8\ubcf4\ub2e4\ub294 \ubc29\uc1a1\uc778\uc73c\ub85c\uc11c\uc758 \uc870\uc601\ub0a8\uc774 \ub300\uc911\ub4e4\uc5d0\uac8c \ub9ce\uc740 \uad00\uc2ec\uc744 \ubc1b\uc558\ub2e4. \ub300\ud45c\uc801\uc73c\ub85c 1993\ub144\uc5d0 \uc774\uacbd\uc2e4\uacfc \ud568\uaed8 1\ub300 \uc9c4\ud589\uc790\ub97c \ub9e1\uc740 KBS2 \u300a\uccb4\ud5d8 \uc0b6\uc758 \ud604\uc7a5\u300b\uc774 \uc8fc\ubaa9\uc744 \ubc1b\uc558\ub2e4. \uc724\uc5ec\uc815\uacfc \uc774\ud63c \ud6c4 1995\ub144\uc5d0 \ubc31\uc740\uc2e4\uacfc \uacb0\ud63c\ud558\uace0 \ub538 \uc870\uc740\uc9c0\ub97c \uc785\uc591\ud558\uba70 \uc0c8\ub85c\uc6b4 \uac00\uc815\uc744 \uafb8\ub838\uc9c0\ub9cc \ud604\uc7ac\ub294 \ub538\uacfc \uac19\uc774 \uc0dd\ud65c\ud558\uace0 \uc788\ub2e4. \uadf8\ub9ac\uace0 1990\ub144\ub300\uc5d0 \uc77c\uc5b4\ub09c \uc640\uc6b0 \uc544\ud30c\ud2b8 \ubd95\uad34\uc0ac\uac74\uc774 \uc77c\uc5b4\ub0ac\uc744\ub54c \ud568\uacbd\ub3c4 \uc2e0\ubbfc\uc694 \uc2e0\uace0\uc0b0 \ud0c0\ub839\uc744 \uac00\uc0ac\ub9cc \ubc14\uafb8\uc5b4 \"\uc640\uc6b0 \uc544\ud30c\ud2b8\uac00 \uc6b0\ub8e8\ub8e8\ub8e8\ub8e8\ub8e8 \ubb34\ub108\uc9c0\ub294 \uc18c\ub9ac\uc5d0...\"\ub85c \uc2dc\uc791\ud558\ub294 \ub178\ub798\ub85c \ubd88\ub7ec \uadf8 \uccab\uad6c\uc808\uc774 \ubb38\uc81c\uac00 \ub418\uc5b4 \uc2e0\ubb38\uacfc \ub274\uc218\uc5d0 \ub098\uae30\ub3c4 \ud558\uc5ec \ud070 \ud654\uc81c\uac00 \ub418\uc5c8\ub2e4.", "paragraph_answer": " 1989\ub144 \uc5d0 \uc790\ub2c8 \uc724\uc774 \uc9c4\ud589\ud558\ub294 KBS2 \u300a\uc790\ub2c8\uc724 \uc1fc\u300b\uc5d0 \ubcf4\uc870 \uc9c4\ud589\uc790\ub85c \ud65c\ub3d9\ud55c \ud6c4 1990\ub144\ub300\uc5d0 \ub4e4\uc5b4\uc11c\uba74\uc11c \uac00\uc218\ub85c\uc11c\uc758 \uc870\uc601\ub0a8\ubcf4\ub2e4\ub294 \ubc29\uc1a1\uc778\uc73c\ub85c\uc11c\uc758 \uc870\uc601\ub0a8\uc774 \ub300\uc911\ub4e4\uc5d0\uac8c \ub9ce\uc740 \uad00\uc2ec\uc744 \ubc1b\uc558\ub2e4. \ub300\ud45c\uc801\uc73c\ub85c 1993\ub144\uc5d0 \uc774\uacbd\uc2e4\uacfc \ud568\uaed8 1\ub300 \uc9c4\ud589\uc790\ub97c \ub9e1\uc740 KBS2 \u300a\uccb4\ud5d8 \uc0b6\uc758 \ud604\uc7a5\u300b\uc774 \uc8fc\ubaa9\uc744 \ubc1b\uc558\ub2e4. \uc724\uc5ec\uc815\uacfc \uc774\ud63c \ud6c4 1995\ub144\uc5d0 \ubc31\uc740\uc2e4\uacfc \uacb0\ud63c\ud558\uace0 \ub538 \uc870\uc740\uc9c0\ub97c \uc785\uc591\ud558\uba70 \uc0c8\ub85c\uc6b4 \uac00\uc815\uc744 \uafb8\ub838\uc9c0\ub9cc \ud604\uc7ac\ub294 \ub538\uacfc \uac19\uc774 \uc0dd\ud65c\ud558\uace0 \uc788\ub2e4. \uadf8\ub9ac\uace0 1990\ub144\ub300\uc5d0 \uc77c\uc5b4\ub09c \uc640\uc6b0 \uc544\ud30c\ud2b8 \ubd95\uad34\uc0ac\uac74\uc774 \uc77c\uc5b4\ub0ac\uc744\ub54c \ud568\uacbd\ub3c4 \uc2e0\ubbfc\uc694 \uc2e0\uace0\uc0b0 \ud0c0\ub839\uc744 \uac00\uc0ac\ub9cc \ubc14\uafb8\uc5b4 \"\uc640\uc6b0 \uc544\ud30c\ud2b8\uac00 \uc6b0\ub8e8\ub8e8\ub8e8\ub8e8\ub8e8 \ubb34\ub108\uc9c0\ub294 \uc18c\ub9ac\uc5d0...\"\ub85c \uc2dc\uc791\ud558\ub294 \ub178\ub798\ub85c \ubd88\ub7ec \uadf8 \uccab\uad6c\uc808\uc774 \ubb38\uc81c\uac00 \ub418\uc5b4 \uc2e0\ubb38\uacfc \ub274\uc218\uc5d0 \ub098\uae30\ub3c4 \ud558\uc5ec \ud070 \ud654\uc81c\uac00 \ub418\uc5c8\ub2e4.", "sentence_answer": " 1989\ub144 \uc5d0 \uc790\ub2c8 \uc724\uc774 \uc9c4\ud589\ud558\ub294 KBS2 \u300a\uc790\ub2c8\uc724 \uc1fc\u300b\uc5d0 \ubcf4\uc870 \uc9c4\ud589\uc790\ub85c \ud65c\ub3d9\ud55c \ud6c4 1990\ub144\ub300\uc5d0 \ub4e4\uc5b4\uc11c\uba74\uc11c \uac00\uc218\ub85c\uc11c\uc758 \uc870\uc601\ub0a8\ubcf4\ub2e4\ub294 \ubc29\uc1a1\uc778\uc73c\ub85c\uc11c\uc758 \uc870\uc601\ub0a8\uc774 \ub300\uc911\ub4e4\uc5d0\uac8c \ub9ce\uc740 \uad00\uc2ec\uc744 \ubc1b\uc558\ub2e4.", "paragraph_id": "6553421-1-0", "paragraph_question": "question: \uc870\uc601\ub0a8\uc774 \uc790\ub2c8\uc724 \uc1fc\uc5d0 \ubcf4\uc870 \uc9c4\ud589\uc790\ub85c \ud65c\ub3d9\ud588\ub358 \uac83\uc740 \uc5b8\uc81c\uc778\uac00?, context: 1989\ub144\uc5d0 \uc790\ub2c8 \uc724\uc774 \uc9c4\ud589\ud558\ub294 KBS2 \u300a\uc790\ub2c8\uc724 \uc1fc\u300b\uc5d0 \ubcf4\uc870 \uc9c4\ud589\uc790\ub85c \ud65c\ub3d9\ud55c \ud6c4 1990\ub144\ub300\uc5d0 \ub4e4\uc5b4\uc11c\uba74\uc11c \uac00\uc218\ub85c\uc11c\uc758 \uc870\uc601\ub0a8\ubcf4\ub2e4\ub294 \ubc29\uc1a1\uc778\uc73c\ub85c\uc11c\uc758 \uc870\uc601\ub0a8\uc774 \ub300\uc911\ub4e4\uc5d0\uac8c \ub9ce\uc740 \uad00\uc2ec\uc744 \ubc1b\uc558\ub2e4. \ub300\ud45c\uc801\uc73c\ub85c 1993\ub144\uc5d0 \uc774\uacbd\uc2e4\uacfc \ud568\uaed8 1\ub300 \uc9c4\ud589\uc790\ub97c \ub9e1\uc740 KBS2 \u300a\uccb4\ud5d8 \uc0b6\uc758 \ud604\uc7a5\u300b\uc774 \uc8fc\ubaa9\uc744 \ubc1b\uc558\ub2e4. \uc724\uc5ec\uc815\uacfc \uc774\ud63c \ud6c4 1995\ub144\uc5d0 \ubc31\uc740\uc2e4\uacfc \uacb0\ud63c\ud558\uace0 \ub538 \uc870\uc740\uc9c0\ub97c \uc785\uc591\ud558\uba70 \uc0c8\ub85c\uc6b4 \uac00\uc815\uc744 \uafb8\ub838\uc9c0\ub9cc \ud604\uc7ac\ub294 \ub538\uacfc \uac19\uc774 \uc0dd\ud65c\ud558\uace0 \uc788\ub2e4. \uadf8\ub9ac\uace0 1990\ub144\ub300\uc5d0 \uc77c\uc5b4\ub09c \uc640\uc6b0 \uc544\ud30c\ud2b8 \ubd95\uad34\uc0ac\uac74\uc774 \uc77c\uc5b4\ub0ac\uc744\ub54c \ud568\uacbd\ub3c4 \uc2e0\ubbfc\uc694 \uc2e0\uace0\uc0b0 \ud0c0\ub839\uc744 \uac00\uc0ac\ub9cc \ubc14\uafb8\uc5b4 \"\uc640\uc6b0 \uc544\ud30c\ud2b8\uac00 \uc6b0\ub8e8\ub8e8\ub8e8\ub8e8\ub8e8 \ubb34\ub108\uc9c0\ub294 \uc18c\ub9ac\uc5d0...\"\ub85c \uc2dc\uc791\ud558\ub294 \ub178\ub798\ub85c \ubd88\ub7ec \uadf8 \uccab\uad6c\uc808\uc774 \ubb38\uc81c\uac00 \ub418\uc5b4 \uc2e0\ubb38\uacfc \ub274\uc218\uc5d0 \ub098\uae30\ub3c4 \ud558\uc5ec \ud070 \ud654\uc81c\uac00 \ub418\uc5c8\ub2e4."} {"question": "\uae40\uaddc\uc2dd\uc740 \uc5b4\ub5a4 \ubd80\ubd84\uc744 \uc808\uac1c\ud558\ub294 \uc218\uc220\uc744 \ubc1b\uc558\ub098?", "paragraph": "\uacc4\uc18d\ub41c \ub450\ud1b5\uc73c\ub85c 1919\ub144 \ub9d0 \uae40\uaddc\uc2dd\uc740 \ubbf8\uad6d\uccb4\ub958 \uc911 \ub450\uace8 \uc804\uba74 \uc88c\uce21\ubd80\ubd84\uc744 \uc808\uac1c\ud558\ub294 \uc218\uc220\uc744 \ud588\ub2e4. 1919\ub144 \ub9d0 \ub1cc\uc885\uc591\uc744 \uc81c\uac70\ud558\ub294 \uc218\uc220\uc744 \ubc1b\uc558\ub294\ub370, \uc218\uc220 \ubd80\uc791\uc6a9\uc73c\ub85c \uadf8 \ub4a4\ub85c \uac04\uc9c8\ubcd1\uc774 \ub098\ud0c0\ub098 \uace0\ud1b5\uc744 \ubc1b\uae30\ub3c4 \ud558\uc600\ub2e4. \uadf8\uc758 \ubcd1\uc6d0\ube44\ub294 \uc774\uc2b9\ub9cc\uc774 \ubd80\ub2f4\ud558\uc600\ub2e4. \uadf8\ub7ec\ub098 \uad6c\ubbf8\uc704\uc6d0\ubd80 \ud65c\ub3d9 \ub0b4\ub0b4 \uadf8\ub294 \uc774\uc2b9\ub9cc\uacfc \uac08\ub4f1\ud558\uac8c \ub418\uc5c8\uace0, \uc0c1\ud558\uc774\uc5d0\uc11c\ub294 \uc774\uc2b9\ub9cc\uc774 \uadf8\uc758 \uc18d\uc744 \uc369\uc5ec\uc11c \ubcd1\uc138\uac00 \uc545\ud654\ub418\uc5c8\ub2e4\ub294 \ub8e8\uba38\uac00 \ub3cc\uae30\ub3c4 \ud588\ub2e4. \uc774\ub294 \uadf8\uc758 \ubd80\uc778 \uae40\uc21c\uc560\uac00 \uc0ac\uc2e4\uc774 \uc544\ub2c8\ub77c\uace0 \ud574\uba85\ud558\uba74\uc11c \uac00\ub77c\uc549\uac8c \ub418\uc5c8\ub2e4. \uc774\ud6c4 \uc911\uad6d \uc0c1\ud558\uc774\uc5d0 \ub3c5\ub9bd\uc6b4\ub3d9 \uc790\uae08\uc744 \ubcf4\ub0b4\ub294 \uc77c\uc744 \ud558\ub2e4\uac00 1920\ub144 \uc0c1\ud558\uc774\uc5d0 \uadc0\ud658\ud558\uc5ec \ub300\ud55c\ubbfc\uad6d \uc784\uc2dc\uc815\ubd80 \ud559\ubb34\ucd1d\uc7a5 \ub4f1\uc5d0 \uc120\uc784\ub418\uc5c8\ub2e4. \uc911\uad6d\uc5d0 \ub3c4\ucc29\ud55c \ud6c4 \uae40\uaddc\uc2dd\uc740 \uc784\uc2dc\uc815\ubd80\uc758 \uc815\ubb34\ud65c\ub3d9\uacfc \uad50\uc721\uc0ac\uc5c5\uc5d0\ub3c4 \ud798\uc744 \uc3df\uc558\ub2e4. 4\uc6d4 25\uc77c \uad6c\ubbf8\uc704\uc6d0\ubd80 \ubd80\uc704\uc6d0\uc7a5\uc9c1\uc744 \uc0ac\ud1f4\ud558\uc600\ub2e4.", "answer": "\ub450\uace8 \uc804\uba74 \uc88c\uce21\ubd80\ubd84", "sentence": "\uacc4\uc18d\ub41c \ub450\ud1b5\uc73c\ub85c 1919\ub144 \ub9d0 \uae40\uaddc\uc2dd\uc740 \ubbf8\uad6d\uccb4\ub958 \uc911 \ub450\uace8 \uc804\uba74 \uc88c\uce21\ubd80\ubd84 \uc744 \uc808\uac1c\ud558\ub294 \uc218\uc220\uc744 \ud588\ub2e4.", "paragraph_sentence": " \uacc4\uc18d\ub41c \ub450\ud1b5\uc73c\ub85c 1919\ub144 \ub9d0 \uae40\uaddc\uc2dd\uc740 \ubbf8\uad6d\uccb4\ub958 \uc911 \ub450\uace8 \uc804\uba74 \uc88c\uce21\ubd80\ubd84 \uc744 \uc808\uac1c\ud558\ub294 \uc218\uc220\uc744 \ud588\ub2e4. 1919\ub144 \ub9d0 \ub1cc\uc885\uc591\uc744 \uc81c\uac70\ud558\ub294 \uc218\uc220\uc744 \ubc1b\uc558\ub294\ub370, \uc218\uc220 \ubd80\uc791\uc6a9\uc73c\ub85c \uadf8 \ub4a4\ub85c \uac04\uc9c8\ubcd1\uc774 \ub098\ud0c0\ub098 \uace0\ud1b5\uc744 \ubc1b\uae30\ub3c4 \ud558\uc600\ub2e4. \uadf8\uc758 \ubcd1\uc6d0\ube44\ub294 \uc774\uc2b9\ub9cc\uc774 \ubd80\ub2f4\ud558\uc600\ub2e4. \uadf8\ub7ec\ub098 \uad6c\ubbf8\uc704\uc6d0\ubd80 \ud65c\ub3d9 \ub0b4\ub0b4 \uadf8\ub294 \uc774\uc2b9\ub9cc\uacfc \uac08\ub4f1\ud558\uac8c \ub418\uc5c8\uace0, \uc0c1\ud558\uc774\uc5d0\uc11c\ub294 \uc774\uc2b9\ub9cc\uc774 \uadf8\uc758 \uc18d\uc744 \uc369\uc5ec\uc11c \ubcd1\uc138\uac00 \uc545\ud654\ub418\uc5c8\ub2e4\ub294 \ub8e8\uba38\uac00 \ub3cc\uae30\ub3c4 \ud588\ub2e4. \uc774\ub294 \uadf8\uc758 \ubd80\uc778 \uae40\uc21c\uc560\uac00 \uc0ac\uc2e4\uc774 \uc544\ub2c8\ub77c\uace0 \ud574\uba85\ud558\uba74\uc11c \uac00\ub77c\uc549\uac8c \ub418\uc5c8\ub2e4. \uc774\ud6c4 \uc911\uad6d \uc0c1\ud558\uc774\uc5d0 \ub3c5\ub9bd\uc6b4\ub3d9 \uc790\uae08\uc744 \ubcf4\ub0b4\ub294 \uc77c\uc744 \ud558\ub2e4\uac00 1920\ub144 \uc0c1\ud558\uc774\uc5d0 \uadc0\ud658\ud558\uc5ec \ub300\ud55c\ubbfc\uad6d \uc784\uc2dc\uc815\ubd80 \ud559\ubb34\ucd1d\uc7a5 \ub4f1\uc5d0 \uc120\uc784\ub418\uc5c8\ub2e4. \uc911\uad6d\uc5d0 \ub3c4\ucc29\ud55c \ud6c4 \uae40\uaddc\uc2dd\uc740 \uc784\uc2dc\uc815\ubd80\uc758 \uc815\ubb34\ud65c\ub3d9\uacfc \uad50\uc721\uc0ac\uc5c5\uc5d0\ub3c4 \ud798\uc744 \uc3df\uc558\ub2e4. 4\uc6d4 25\uc77c \uad6c\ubbf8\uc704\uc6d0\ubd80 \ubd80\uc704\uc6d0\uc7a5\uc9c1\uc744 \uc0ac\ud1f4\ud558\uc600\ub2e4.", "paragraph_answer": "\uacc4\uc18d\ub41c \ub450\ud1b5\uc73c\ub85c 1919\ub144 \ub9d0 \uae40\uaddc\uc2dd\uc740 \ubbf8\uad6d\uccb4\ub958 \uc911 \ub450\uace8 \uc804\uba74 \uc88c\uce21\ubd80\ubd84 \uc744 \uc808\uac1c\ud558\ub294 \uc218\uc220\uc744 \ud588\ub2e4. 1919\ub144 \ub9d0 \ub1cc\uc885\uc591\uc744 \uc81c\uac70\ud558\ub294 \uc218\uc220\uc744 \ubc1b\uc558\ub294\ub370, \uc218\uc220 \ubd80\uc791\uc6a9\uc73c\ub85c \uadf8 \ub4a4\ub85c \uac04\uc9c8\ubcd1\uc774 \ub098\ud0c0\ub098 \uace0\ud1b5\uc744 \ubc1b\uae30\ub3c4 \ud558\uc600\ub2e4. \uadf8\uc758 \ubcd1\uc6d0\ube44\ub294 \uc774\uc2b9\ub9cc\uc774 \ubd80\ub2f4\ud558\uc600\ub2e4. \uadf8\ub7ec\ub098 \uad6c\ubbf8\uc704\uc6d0\ubd80 \ud65c\ub3d9 \ub0b4\ub0b4 \uadf8\ub294 \uc774\uc2b9\ub9cc\uacfc \uac08\ub4f1\ud558\uac8c \ub418\uc5c8\uace0, \uc0c1\ud558\uc774\uc5d0\uc11c\ub294 \uc774\uc2b9\ub9cc\uc774 \uadf8\uc758 \uc18d\uc744 \uc369\uc5ec\uc11c \ubcd1\uc138\uac00 \uc545\ud654\ub418\uc5c8\ub2e4\ub294 \ub8e8\uba38\uac00 \ub3cc\uae30\ub3c4 \ud588\ub2e4. \uc774\ub294 \uadf8\uc758 \ubd80\uc778 \uae40\uc21c\uc560\uac00 \uc0ac\uc2e4\uc774 \uc544\ub2c8\ub77c\uace0 \ud574\uba85\ud558\uba74\uc11c \uac00\ub77c\uc549\uac8c \ub418\uc5c8\ub2e4. \uc774\ud6c4 \uc911\uad6d \uc0c1\ud558\uc774\uc5d0 \ub3c5\ub9bd\uc6b4\ub3d9 \uc790\uae08\uc744 \ubcf4\ub0b4\ub294 \uc77c\uc744 \ud558\ub2e4\uac00 1920\ub144 \uc0c1\ud558\uc774\uc5d0 \uadc0\ud658\ud558\uc5ec \ub300\ud55c\ubbfc\uad6d \uc784\uc2dc\uc815\ubd80 \ud559\ubb34\ucd1d\uc7a5 \ub4f1\uc5d0 \uc120\uc784\ub418\uc5c8\ub2e4. \uc911\uad6d\uc5d0 \ub3c4\ucc29\ud55c \ud6c4 \uae40\uaddc\uc2dd\uc740 \uc784\uc2dc\uc815\ubd80\uc758 \uc815\ubb34\ud65c\ub3d9\uacfc \uad50\uc721\uc0ac\uc5c5\uc5d0\ub3c4 \ud798\uc744 \uc3df\uc558\ub2e4. 4\uc6d4 25\uc77c \uad6c\ubbf8\uc704\uc6d0\ubd80 \ubd80\uc704\uc6d0\uc7a5\uc9c1\uc744 \uc0ac\ud1f4\ud558\uc600\ub2e4.", "sentence_answer": "\uacc4\uc18d\ub41c \ub450\ud1b5\uc73c\ub85c 1919\ub144 \ub9d0 \uae40\uaddc\uc2dd\uc740 \ubbf8\uad6d\uccb4\ub958 \uc911 \ub450\uace8 \uc804\uba74 \uc88c\uce21\ubd80\ubd84 \uc744 \uc808\uac1c\ud558\ub294 \uc218\uc220\uc744 \ud588\ub2e4.", "paragraph_id": "5991262-10-0", "paragraph_question": "question: \uae40\uaddc\uc2dd\uc740 \uc5b4\ub5a4 \ubd80\ubd84\uc744 \uc808\uac1c\ud558\ub294 \uc218\uc220\uc744 \ubc1b\uc558\ub098?, context: \uacc4\uc18d\ub41c \ub450\ud1b5\uc73c\ub85c 1919\ub144 \ub9d0 \uae40\uaddc\uc2dd\uc740 \ubbf8\uad6d\uccb4\ub958 \uc911 \ub450\uace8 \uc804\uba74 \uc88c\uce21\ubd80\ubd84\uc744 \uc808\uac1c\ud558\ub294 \uc218\uc220\uc744 \ud588\ub2e4. 1919\ub144 \ub9d0 \ub1cc\uc885\uc591\uc744 \uc81c\uac70\ud558\ub294 \uc218\uc220\uc744 \ubc1b\uc558\ub294\ub370, \uc218\uc220 \ubd80\uc791\uc6a9\uc73c\ub85c \uadf8 \ub4a4\ub85c \uac04\uc9c8\ubcd1\uc774 \ub098\ud0c0\ub098 \uace0\ud1b5\uc744 \ubc1b\uae30\ub3c4 \ud558\uc600\ub2e4. \uadf8\uc758 \ubcd1\uc6d0\ube44\ub294 \uc774\uc2b9\ub9cc\uc774 \ubd80\ub2f4\ud558\uc600\ub2e4. \uadf8\ub7ec\ub098 \uad6c\ubbf8\uc704\uc6d0\ubd80 \ud65c\ub3d9 \ub0b4\ub0b4 \uadf8\ub294 \uc774\uc2b9\ub9cc\uacfc \uac08\ub4f1\ud558\uac8c \ub418\uc5c8\uace0, \uc0c1\ud558\uc774\uc5d0\uc11c\ub294 \uc774\uc2b9\ub9cc\uc774 \uadf8\uc758 \uc18d\uc744 \uc369\uc5ec\uc11c \ubcd1\uc138\uac00 \uc545\ud654\ub418\uc5c8\ub2e4\ub294 \ub8e8\uba38\uac00 \ub3cc\uae30\ub3c4 \ud588\ub2e4. \uc774\ub294 \uadf8\uc758 \ubd80\uc778 \uae40\uc21c\uc560\uac00 \uc0ac\uc2e4\uc774 \uc544\ub2c8\ub77c\uace0 \ud574\uba85\ud558\uba74\uc11c \uac00\ub77c\uc549\uac8c \ub418\uc5c8\ub2e4. \uc774\ud6c4 \uc911\uad6d \uc0c1\ud558\uc774\uc5d0 \ub3c5\ub9bd\uc6b4\ub3d9 \uc790\uae08\uc744 \ubcf4\ub0b4\ub294 \uc77c\uc744 \ud558\ub2e4\uac00 1920\ub144 \uc0c1\ud558\uc774\uc5d0 \uadc0\ud658\ud558\uc5ec \ub300\ud55c\ubbfc\uad6d \uc784\uc2dc\uc815\ubd80 \ud559\ubb34\ucd1d\uc7a5 \ub4f1\uc5d0 \uc120\uc784\ub418\uc5c8\ub2e4. \uc911\uad6d\uc5d0 \ub3c4\ucc29\ud55c \ud6c4 \uae40\uaddc\uc2dd\uc740 \uc784\uc2dc\uc815\ubd80\uc758 \uc815\ubb34\ud65c\ub3d9\uacfc \uad50\uc721\uc0ac\uc5c5\uc5d0\ub3c4 \ud798\uc744 \uc3df\uc558\ub2e4. 4\uc6d4 25\uc77c \uad6c\ubbf8\uc704\uc6d0\ubd80 \ubd80\uc704\uc6d0\uc7a5\uc9c1\uc744 \uc0ac\ud1f4\ud558\uc600\ub2e4."} {"question": "\uc5ec\ub7ec \uc804\ucc28 \uc0dd\uc0b0\uc5d0 \uc601\ud5a5\uc744 \ub07c\uce58\uae30\ub3c4 \ud558\uba70, \ub3c5\uc77c\uad70\uc774 \uc0ac\uc6a9\ud55c \uc804\ud22c\ucc28\ub7c9 \uc911\uc5d0 \uac00\uc7a5 \ub9ce\uc774 \uc0dd\uc0b0\ub41c \uac83\uc740?", "paragraph": "4\ud638 \uc804\ucc28(\ub3c5\uc77c\uc5b4: Panzerkampfwagen IV; Pz.Kpfw. IV \ud310\ucc98\uce84\ud504\ubc14\uac90 \ud53c\uc5b4)\ub294 \uc81c2\ucc28 \uc138\uacc4\ub300\uc804 \ub2f9\uc2dc \ub3c5\uc77c\uad70\uc774 \uc0ac\uc6a9\ud588\ub358 \uc911(\u4e2d)\uc804\ucc28\uc774\ub2e4. \ubcf8\ub798 \ubcf4\ubcd1 \uc9c0\uc6d0\uc744 \ubaa9\uc801\uc73c\ub85c \uacc4\ud68d\ub418\uc5c8\uc73c\uba70 \ub300\uc804\ucc28\uc804\uc740 \ubcf8 \uc6a9\ub3c4\uac00 \uc544\ub2c8\uc5c8\uc73c\ub098, \ub300\uc804 \ubc1c\ubc1c\ud6c4 \ub300\uc804\ucc28\uc804\uc744 \ub9e1\uc558\ub358 3\ud638 \uc804\ucc28\uc758 \ubb38\uc81c\uac00 \ub3c4\ucd9c\ub418\uace0 \ub3d9\ubd80\uc804\uc120\uc5d0\uc11c T-34\uc640 \ub9c8\uc8fc\ud558\uac8c \ub418\uba74\uc11c \uad6c\ud615\ud654\ub41c 3\ud638 \uc804\ucc28\ub97c \ub300\uc2e0\ud574 \uc8fc\ub825\uc744 \ub3c4\ub9e1\uac8c \ub418\uc5c8\ub2e4. 1936\ub144 \uac1c\ubc1c\uc774 \uc2dc\uc791\ub41c \uc774\ud6c4\ub85c \uc138\uacc4\ub300\uc804\uc774 \ub05d\ub0a0 \ub54c\uae4c\uc9c0 \ub04a\uc784\uc5c6\uc774 \uc0dd\uc0b0\ub41c \uc720\uc77c\ud55c \uc804\ucc28\ub85c\uc11c \ub3c5\uc77c\uad70\uc774 \ud65c\ub3d9\ud55c \ubaa8\ub4e0 \uc804\uc120\uc5d0 \ud22c\uc785\ub418\uc5c8\uc73c\uba70 \ub3c5\uc77c\uad70 \uc804\ucc28 \uc911\uc5d0\uc11c \uac00\uc7a5 \ub9ce\uc774 \uc0dd\uc0b0\ub41c \uc804\ucc28\ub85c\uc11c \uad6c\ucd95\uc804\ucc28\uc640 \uc790\uc8fc\ub300\uacf5\ud3ec \ub4f1\uacfc \uac19\uc740 \ub2e4\uc218\uc758 \uc804\ud22c \ucc28\ub7c9\uc758 \ubaa8\ud0dc\uac00 \ub418\uae30\ub3c4 \ud558\uc600\ub2e4. \uc57d 300\ub300\uac00 \ub3c5\uc77c \ubc16\uc73c\ub85c \uc218\ucd9c\ub418\uc5c8\ub294\ub370 \uc774\ub4e4 \uc911 \uc77c\ubd80\ub294 \ub2e4\uc2dc \uc2dc\ub9ac\uc544\ub85c \uc218\ucd9c\ub418\uc5b4 6\uc77c \uc804\uc7c1\uc5d0 \ud22c\uc785\ub418\uc5c8\ub2e4.", "answer": "4\ud638 \uc804\ucc28", "sentence": "4\ud638 \uc804\ucc28 (\ub3c5\uc77c\uc5b4: Panzerkampfwagen IV; Pz.", "paragraph_sentence": " 4\ud638 \uc804\ucc28 (\ub3c5\uc77c\uc5b4: Panzerkampfwagen IV; Pz. Kpfw. IV \ud310\ucc98\uce84\ud504\ubc14\uac90 \ud53c\uc5b4)\ub294 \uc81c2\ucc28 \uc138\uacc4\ub300\uc804 \ub2f9\uc2dc \ub3c5\uc77c\uad70\uc774 \uc0ac\uc6a9\ud588\ub358 \uc911(\u4e2d)\uc804\ucc28\uc774\ub2e4. \ubcf8\ub798 \ubcf4\ubcd1 \uc9c0\uc6d0\uc744 \ubaa9\uc801\uc73c\ub85c \uacc4\ud68d\ub418\uc5c8\uc73c\uba70 \ub300\uc804\ucc28\uc804\uc740 \ubcf8 \uc6a9\ub3c4\uac00 \uc544\ub2c8\uc5c8\uc73c\ub098, \ub300\uc804 \ubc1c\ubc1c\ud6c4 \ub300\uc804\ucc28\uc804\uc744 \ub9e1\uc558\ub358 3\ud638 \uc804\ucc28\uc758 \ubb38\uc81c\uac00 \ub3c4\ucd9c\ub418\uace0 \ub3d9\ubd80\uc804\uc120\uc5d0\uc11c T-34\uc640 \ub9c8\uc8fc\ud558\uac8c \ub418\uba74\uc11c \uad6c\ud615\ud654\ub41c 3\ud638 \uc804\ucc28\ub97c \ub300\uc2e0\ud574 \uc8fc\ub825\uc744 \ub3c4\ub9e1\uac8c \ub418\uc5c8\ub2e4. 1936\ub144 \uac1c\ubc1c\uc774 \uc2dc\uc791\ub41c \uc774\ud6c4\ub85c \uc138\uacc4\ub300\uc804\uc774 \ub05d\ub0a0 \ub54c\uae4c\uc9c0 \ub04a\uc784\uc5c6\uc774 \uc0dd\uc0b0\ub41c \uc720\uc77c\ud55c \uc804\ucc28\ub85c\uc11c \ub3c5\uc77c\uad70\uc774 \ud65c\ub3d9\ud55c \ubaa8\ub4e0 \uc804\uc120\uc5d0 \ud22c\uc785\ub418\uc5c8\uc73c\uba70 \ub3c5\uc77c\uad70 \uc804\ucc28 \uc911\uc5d0\uc11c \uac00\uc7a5 \ub9ce\uc774 \uc0dd\uc0b0\ub41c \uc804\ucc28\ub85c\uc11c \uad6c\ucd95\uc804\ucc28\uc640 \uc790\uc8fc\ub300\uacf5\ud3ec \ub4f1\uacfc \uac19\uc740 \ub2e4\uc218\uc758 \uc804\ud22c \ucc28\ub7c9\uc758 \ubaa8\ud0dc\uac00 \ub418\uae30\ub3c4 \ud558\uc600\ub2e4. \uc57d 300\ub300\uac00 \ub3c5\uc77c \ubc16\uc73c\ub85c \uc218\ucd9c\ub418\uc5c8\ub294\ub370 \uc774\ub4e4 \uc911 \uc77c\ubd80\ub294 \ub2e4\uc2dc \uc2dc\ub9ac\uc544\ub85c \uc218\ucd9c\ub418\uc5b4 6\uc77c \uc804\uc7c1\uc5d0 \ud22c\uc785\ub418\uc5c8\ub2e4.", "paragraph_answer": " 4\ud638 \uc804\ucc28 (\ub3c5\uc77c\uc5b4: Panzerkampfwagen IV; Pz.Kpfw. IV \ud310\ucc98\uce84\ud504\ubc14\uac90 \ud53c\uc5b4)\ub294 \uc81c2\ucc28 \uc138\uacc4\ub300\uc804 \ub2f9\uc2dc \ub3c5\uc77c\uad70\uc774 \uc0ac\uc6a9\ud588\ub358 \uc911(\u4e2d)\uc804\ucc28\uc774\ub2e4. \ubcf8\ub798 \ubcf4\ubcd1 \uc9c0\uc6d0\uc744 \ubaa9\uc801\uc73c\ub85c \uacc4\ud68d\ub418\uc5c8\uc73c\uba70 \ub300\uc804\ucc28\uc804\uc740 \ubcf8 \uc6a9\ub3c4\uac00 \uc544\ub2c8\uc5c8\uc73c\ub098, \ub300\uc804 \ubc1c\ubc1c\ud6c4 \ub300\uc804\ucc28\uc804\uc744 \ub9e1\uc558\ub358 3\ud638 \uc804\ucc28\uc758 \ubb38\uc81c\uac00 \ub3c4\ucd9c\ub418\uace0 \ub3d9\ubd80\uc804\uc120\uc5d0\uc11c T-34\uc640 \ub9c8\uc8fc\ud558\uac8c \ub418\uba74\uc11c \uad6c\ud615\ud654\ub41c 3\ud638 \uc804\ucc28\ub97c \ub300\uc2e0\ud574 \uc8fc\ub825\uc744 \ub3c4\ub9e1\uac8c \ub418\uc5c8\ub2e4. 1936\ub144 \uac1c\ubc1c\uc774 \uc2dc\uc791\ub41c \uc774\ud6c4\ub85c \uc138\uacc4\ub300\uc804\uc774 \ub05d\ub0a0 \ub54c\uae4c\uc9c0 \ub04a\uc784\uc5c6\uc774 \uc0dd\uc0b0\ub41c \uc720\uc77c\ud55c \uc804\ucc28\ub85c\uc11c \ub3c5\uc77c\uad70\uc774 \ud65c\ub3d9\ud55c \ubaa8\ub4e0 \uc804\uc120\uc5d0 \ud22c\uc785\ub418\uc5c8\uc73c\uba70 \ub3c5\uc77c\uad70 \uc804\ucc28 \uc911\uc5d0\uc11c \uac00\uc7a5 \ub9ce\uc774 \uc0dd\uc0b0\ub41c \uc804\ucc28\ub85c\uc11c \uad6c\ucd95\uc804\ucc28\uc640 \uc790\uc8fc\ub300\uacf5\ud3ec \ub4f1\uacfc \uac19\uc740 \ub2e4\uc218\uc758 \uc804\ud22c \ucc28\ub7c9\uc758 \ubaa8\ud0dc\uac00 \ub418\uae30\ub3c4 \ud558\uc600\ub2e4. \uc57d 300\ub300\uac00 \ub3c5\uc77c \ubc16\uc73c\ub85c \uc218\ucd9c\ub418\uc5c8\ub294\ub370 \uc774\ub4e4 \uc911 \uc77c\ubd80\ub294 \ub2e4\uc2dc \uc2dc\ub9ac\uc544\ub85c \uc218\ucd9c\ub418\uc5b4 6\uc77c \uc804\uc7c1\uc5d0 \ud22c\uc785\ub418\uc5c8\ub2e4.", "sentence_answer": " 4\ud638 \uc804\ucc28 (\ub3c5\uc77c\uc5b4: Panzerkampfwagen IV; Pz.", "paragraph_id": "5781884-0-0", "paragraph_question": "question: \uc5ec\ub7ec \uc804\ucc28 \uc0dd\uc0b0\uc5d0 \uc601\ud5a5\uc744 \ub07c\uce58\uae30\ub3c4 \ud558\uba70, \ub3c5\uc77c\uad70\uc774 \uc0ac\uc6a9\ud55c \uc804\ud22c\ucc28\ub7c9 \uc911\uc5d0 \uac00\uc7a5 \ub9ce\uc774 \uc0dd\uc0b0\ub41c \uac83\uc740?, context: 4\ud638 \uc804\ucc28(\ub3c5\uc77c\uc5b4: Panzerkampfwagen IV; Pz.Kpfw. IV \ud310\ucc98\uce84\ud504\ubc14\uac90 \ud53c\uc5b4)\ub294 \uc81c2\ucc28 \uc138\uacc4\ub300\uc804 \ub2f9\uc2dc \ub3c5\uc77c\uad70\uc774 \uc0ac\uc6a9\ud588\ub358 \uc911(\u4e2d)\uc804\ucc28\uc774\ub2e4. \ubcf8\ub798 \ubcf4\ubcd1 \uc9c0\uc6d0\uc744 \ubaa9\uc801\uc73c\ub85c \uacc4\ud68d\ub418\uc5c8\uc73c\uba70 \ub300\uc804\ucc28\uc804\uc740 \ubcf8 \uc6a9\ub3c4\uac00 \uc544\ub2c8\uc5c8\uc73c\ub098, \ub300\uc804 \ubc1c\ubc1c\ud6c4 \ub300\uc804\ucc28\uc804\uc744 \ub9e1\uc558\ub358 3\ud638 \uc804\ucc28\uc758 \ubb38\uc81c\uac00 \ub3c4\ucd9c\ub418\uace0 \ub3d9\ubd80\uc804\uc120\uc5d0\uc11c T-34\uc640 \ub9c8\uc8fc\ud558\uac8c \ub418\uba74\uc11c \uad6c\ud615\ud654\ub41c 3\ud638 \uc804\ucc28\ub97c \ub300\uc2e0\ud574 \uc8fc\ub825\uc744 \ub3c4\ub9e1\uac8c \ub418\uc5c8\ub2e4. 1936\ub144 \uac1c\ubc1c\uc774 \uc2dc\uc791\ub41c \uc774\ud6c4\ub85c \uc138\uacc4\ub300\uc804\uc774 \ub05d\ub0a0 \ub54c\uae4c\uc9c0 \ub04a\uc784\uc5c6\uc774 \uc0dd\uc0b0\ub41c \uc720\uc77c\ud55c \uc804\ucc28\ub85c\uc11c \ub3c5\uc77c\uad70\uc774 \ud65c\ub3d9\ud55c \ubaa8\ub4e0 \uc804\uc120\uc5d0 \ud22c\uc785\ub418\uc5c8\uc73c\uba70 \ub3c5\uc77c\uad70 \uc804\ucc28 \uc911\uc5d0\uc11c \uac00\uc7a5 \ub9ce\uc774 \uc0dd\uc0b0\ub41c \uc804\ucc28\ub85c\uc11c \uad6c\ucd95\uc804\ucc28\uc640 \uc790\uc8fc\ub300\uacf5\ud3ec \ub4f1\uacfc \uac19\uc740 \ub2e4\uc218\uc758 \uc804\ud22c \ucc28\ub7c9\uc758 \ubaa8\ud0dc\uac00 \ub418\uae30\ub3c4 \ud558\uc600\ub2e4. \uc57d 300\ub300\uac00 \ub3c5\uc77c \ubc16\uc73c\ub85c \uc218\ucd9c\ub418\uc5c8\ub294\ub370 \uc774\ub4e4 \uc911 \uc77c\ubd80\ub294 \ub2e4\uc2dc \uc2dc\ub9ac\uc544\ub85c \uc218\ucd9c\ub418\uc5b4 6\uc77c \uc804\uc7c1\uc5d0 \ud22c\uc785\ub418\uc5c8\ub2e4."} {"question": "\ubb3c\ub9ac\ud559, \ud654\ud559, \uc0dd\ubb3c\ud559, \uc9c0\uad6c\uacfc\ud559 \ub4f1\uc758 \uae30\ucd08 \ud559\ubb38 \ubd84\uc57c\ub294 \ubb34\uc2a8 \uacfc\ud559\uc5d0 \uc18d\ud558\ub098\uc694?", "paragraph": "\uc778\ubb38\uacfc\ud559\uc5d0 \uc18d\ud558\ub294 \uae30\ucd08 \ud559\ubb38 \ubd84\uc57c\ub85c\ub294 \uc5b8\uc5b4\ud559, \ubb38\ud559, \uc5ed\uc0ac\ud559, \ucca0\ud559, \uc885\uad50\ud559, \uc2ec\ub9ac\ud559 \ub4f1\uc774 \uc788\uace0, \uc790\uc5f0\uacfc\ud559\uc5d0 \uc18d\ud558\ub294 \uae30\ucd08 \ud559\ubb38 \ubd84\uc57c\ub85c\ub294 \uacfc\ud559\uacfc \uae30\uc220 \ubc1c\uc804\uc758 \uae30\ucd08\uac00 \ub418\ub294 \ubb3c\ub9ac\ud559, \ud654\ud559, \uc0dd\ubb3c\ud559,\uc9c0\uad6c\uacfc\ud559 \ub4f1\uc774 \uc788\ub2e4. \uace0\ub300\uc5d0\ub294 \uc774\ub7ec\ud55c \uc778\ubb38\u00b7\uc0ac\ud68c\uacfc\ud559\uacfc \uc790\uc5f0\uacfc\ud559\uc758 \uad6c\ubd84\uc774 \uba85\ud655\ud558\uc9c0 \uc54a\uc544\uc11c \ub9ce\uc740 \uace0\ub300 \ud559\uc790\ub4e4\uc740 \uc774\ub7f0 \uc5ec\ub7ec \ubd84\uc57c\uc5d0 \ub300\ud574 \uae30\ucd08 \uc9c0\uc2dd\ub4e4\uc744 \ub450\ub8e8\ub450\ub8e8 \uac00\uc9c0\uace0 \uc788\ub294 \uacbd\uc6b0\uac00 \ub9ce\uc558\uc73c\ub098, \ubb38\uba85 \ubc1c\ub2ec\uacfc \ub354\ubd88\uc5b4 \uc810\ucc28 \uac01\uac01\uc758 \ud559\ubb38 \ubd84\uc57c\uc758 \uc9c0\uc2dd \ucd95\uc801\uc774 \ub9ce\uc544\uc9c0\uba74\uc11c \uc790\uc5f0\uc2a4\ub7fd\uac8c \uac1c\ubcc4 \ubd84\uc57c\ub4e4\uc774 \ubd84\ud654\ub418\uc5b4 \uac01\uac01\uc758 \uc774\ub984\uc744 \uac00\uc9c0\uac8c \ub418\uc5c8\ub2e4. \uadf8\ub7ec\ub098, \uc5ec\uc804\ud788 \ub9ce\uc740 \uae30\ucd08 \ud559\ubb38 \ubd84\uc57c\ub294 \uc18c\uc704 \ub9d0\ud558\ub294 \uc751\uc6a9\uacfc\ud559(\ub610\ub294 \uc2e4\uc6a9\uacfc\ud559) \ubd84\uc57c\uc5d0 \ube44\ud574\uc11c \uc5ed\uc0ac\uac00 \uc624\ub798\ub41c \uac83\uc774 \ub9ce\uace0, \ub610\ud55c \uc774\ub7ec\ud55c \uc751\uc6a9\uacfc\ud559\uc758 \uae30\ucd08\uac00 \ub418\ub294 \ud559\ubb38\uc778 \ub9cc\ud07c \ub9ce\uc740 \uad50\uc721\uacfc\uc815\uc5d0\uc11c\ub294 \uc774\ub7ec\ud55c \uae30\ucd08 \ud559\ubb38\uc758 \uad50\uc721\uc744 \uac15\uc870\ud558\ub294 \uacbd\ud5a5\uc774 \uac15\ud558\ub2e4.", "answer": "\uc790\uc5f0\uacfc\ud559", "sentence": "\uc778\ubb38\uacfc\ud559\uc5d0 \uc18d\ud558\ub294 \uae30\ucd08 \ud559\ubb38 \ubd84\uc57c\ub85c\ub294 \uc5b8\uc5b4\ud559, \ubb38\ud559, \uc5ed\uc0ac\ud559, \ucca0\ud559, \uc885\uad50\ud559, \uc2ec\ub9ac\ud559 \ub4f1\uc774 \uc788\uace0, \uc790\uc5f0\uacfc\ud559 \uc5d0 \uc18d\ud558\ub294 \uae30\ucd08 \ud559\ubb38 \ubd84\uc57c\ub85c\ub294 \uacfc\ud559\uacfc \uae30\uc220 \ubc1c\uc804\uc758 \uae30\ucd08\uac00 \ub418\ub294 \ubb3c\ub9ac\ud559, \ud654\ud559, \uc0dd\ubb3c\ud559,\uc9c0\uad6c\uacfc\ud559 \ub4f1\uc774 \uc788\ub2e4.", "paragraph_sentence": " \uc778\ubb38\uacfc\ud559\uc5d0 \uc18d\ud558\ub294 \uae30\ucd08 \ud559\ubb38 \ubd84\uc57c\ub85c\ub294 \uc5b8\uc5b4\ud559, \ubb38\ud559, \uc5ed\uc0ac\ud559, \ucca0\ud559, \uc885\uad50\ud559, \uc2ec\ub9ac\ud559 \ub4f1\uc774 \uc788\uace0, \uc790\uc5f0\uacfc\ud559 \uc5d0 \uc18d\ud558\ub294 \uae30\ucd08 \ud559\ubb38 \ubd84\uc57c\ub85c\ub294 \uacfc\ud559\uacfc \uae30\uc220 \ubc1c\uc804\uc758 \uae30\ucd08\uac00 \ub418\ub294 \ubb3c\ub9ac\ud559, \ud654\ud559, \uc0dd\ubb3c\ud559,\uc9c0\uad6c\uacfc\ud559 \ub4f1\uc774 \uc788\ub2e4. \uace0\ub300\uc5d0\ub294 \uc774\ub7ec\ud55c \uc778\ubb38\u00b7\uc0ac\ud68c\uacfc\ud559\uacfc \uc790\uc5f0\uacfc\ud559\uc758 \uad6c\ubd84\uc774 \uba85\ud655\ud558\uc9c0 \uc54a\uc544\uc11c \ub9ce\uc740 \uace0\ub300 \ud559\uc790\ub4e4\uc740 \uc774\ub7f0 \uc5ec\ub7ec \ubd84\uc57c\uc5d0 \ub300\ud574 \uae30\ucd08 \uc9c0\uc2dd\ub4e4\uc744 \ub450\ub8e8\ub450\ub8e8 \uac00\uc9c0\uace0 \uc788\ub294 \uacbd\uc6b0\uac00 \ub9ce\uc558\uc73c\ub098, \ubb38\uba85 \ubc1c\ub2ec\uacfc \ub354\ubd88\uc5b4 \uc810\ucc28 \uac01\uac01\uc758 \ud559\ubb38 \ubd84\uc57c\uc758 \uc9c0\uc2dd \ucd95\uc801\uc774 \ub9ce\uc544\uc9c0\uba74\uc11c \uc790\uc5f0\uc2a4\ub7fd\uac8c \uac1c\ubcc4 \ubd84\uc57c\ub4e4\uc774 \ubd84\ud654\ub418\uc5b4 \uac01\uac01\uc758 \uc774\ub984\uc744 \uac00\uc9c0\uac8c \ub418\uc5c8\ub2e4. \uadf8\ub7ec\ub098, \uc5ec\uc804\ud788 \ub9ce\uc740 \uae30\ucd08 \ud559\ubb38 \ubd84\uc57c\ub294 \uc18c\uc704 \ub9d0\ud558\ub294 \uc751\uc6a9\uacfc\ud559(\ub610\ub294 \uc2e4\uc6a9\uacfc\ud559) \ubd84\uc57c\uc5d0 \ube44\ud574\uc11c \uc5ed\uc0ac\uac00 \uc624\ub798\ub41c \uac83\uc774 \ub9ce\uace0, \ub610\ud55c \uc774\ub7ec\ud55c \uc751\uc6a9\uacfc\ud559\uc758 \uae30\ucd08\uac00 \ub418\ub294 \ud559\ubb38\uc778 \ub9cc\ud07c \ub9ce\uc740 \uad50\uc721\uacfc\uc815\uc5d0\uc11c\ub294 \uc774\ub7ec\ud55c \uae30\ucd08 \ud559\ubb38\uc758 \uad50\uc721\uc744 \uac15\uc870\ud558\ub294 \uacbd\ud5a5\uc774 \uac15\ud558\ub2e4.", "paragraph_answer": "\uc778\ubb38\uacfc\ud559\uc5d0 \uc18d\ud558\ub294 \uae30\ucd08 \ud559\ubb38 \ubd84\uc57c\ub85c\ub294 \uc5b8\uc5b4\ud559, \ubb38\ud559, \uc5ed\uc0ac\ud559, \ucca0\ud559, \uc885\uad50\ud559, \uc2ec\ub9ac\ud559 \ub4f1\uc774 \uc788\uace0, \uc790\uc5f0\uacfc\ud559 \uc5d0 \uc18d\ud558\ub294 \uae30\ucd08 \ud559\ubb38 \ubd84\uc57c\ub85c\ub294 \uacfc\ud559\uacfc \uae30\uc220 \ubc1c\uc804\uc758 \uae30\ucd08\uac00 \ub418\ub294 \ubb3c\ub9ac\ud559, \ud654\ud559, \uc0dd\ubb3c\ud559,\uc9c0\uad6c\uacfc\ud559 \ub4f1\uc774 \uc788\ub2e4. \uace0\ub300\uc5d0\ub294 \uc774\ub7ec\ud55c \uc778\ubb38\u00b7\uc0ac\ud68c\uacfc\ud559\uacfc \uc790\uc5f0\uacfc\ud559\uc758 \uad6c\ubd84\uc774 \uba85\ud655\ud558\uc9c0 \uc54a\uc544\uc11c \ub9ce\uc740 \uace0\ub300 \ud559\uc790\ub4e4\uc740 \uc774\ub7f0 \uc5ec\ub7ec \ubd84\uc57c\uc5d0 \ub300\ud574 \uae30\ucd08 \uc9c0\uc2dd\ub4e4\uc744 \ub450\ub8e8\ub450\ub8e8 \uac00\uc9c0\uace0 \uc788\ub294 \uacbd\uc6b0\uac00 \ub9ce\uc558\uc73c\ub098, \ubb38\uba85 \ubc1c\ub2ec\uacfc \ub354\ubd88\uc5b4 \uc810\ucc28 \uac01\uac01\uc758 \ud559\ubb38 \ubd84\uc57c\uc758 \uc9c0\uc2dd \ucd95\uc801\uc774 \ub9ce\uc544\uc9c0\uba74\uc11c \uc790\uc5f0\uc2a4\ub7fd\uac8c \uac1c\ubcc4 \ubd84\uc57c\ub4e4\uc774 \ubd84\ud654\ub418\uc5b4 \uac01\uac01\uc758 \uc774\ub984\uc744 \uac00\uc9c0\uac8c \ub418\uc5c8\ub2e4. \uadf8\ub7ec\ub098, \uc5ec\uc804\ud788 \ub9ce\uc740 \uae30\ucd08 \ud559\ubb38 \ubd84\uc57c\ub294 \uc18c\uc704 \ub9d0\ud558\ub294 \uc751\uc6a9\uacfc\ud559(\ub610\ub294 \uc2e4\uc6a9\uacfc\ud559) \ubd84\uc57c\uc5d0 \ube44\ud574\uc11c \uc5ed\uc0ac\uac00 \uc624\ub798\ub41c \uac83\uc774 \ub9ce\uace0, \ub610\ud55c \uc774\ub7ec\ud55c \uc751\uc6a9\uacfc\ud559\uc758 \uae30\ucd08\uac00 \ub418\ub294 \ud559\ubb38\uc778 \ub9cc\ud07c \ub9ce\uc740 \uad50\uc721\uacfc\uc815\uc5d0\uc11c\ub294 \uc774\ub7ec\ud55c \uae30\ucd08 \ud559\ubb38\uc758 \uad50\uc721\uc744 \uac15\uc870\ud558\ub294 \uacbd\ud5a5\uc774 \uac15\ud558\ub2e4.", "sentence_answer": "\uc778\ubb38\uacfc\ud559\uc5d0 \uc18d\ud558\ub294 \uae30\ucd08 \ud559\ubb38 \ubd84\uc57c\ub85c\ub294 \uc5b8\uc5b4\ud559, \ubb38\ud559, \uc5ed\uc0ac\ud559, \ucca0\ud559, \uc885\uad50\ud559, \uc2ec\ub9ac\ud559 \ub4f1\uc774 \uc788\uace0, \uc790\uc5f0\uacfc\ud559 \uc5d0 \uc18d\ud558\ub294 \uae30\ucd08 \ud559\ubb38 \ubd84\uc57c\ub85c\ub294 \uacfc\ud559\uacfc \uae30\uc220 \ubc1c\uc804\uc758 \uae30\ucd08\uac00 \ub418\ub294 \ubb3c\ub9ac\ud559, \ud654\ud559, \uc0dd\ubb3c\ud559,\uc9c0\uad6c\uacfc\ud559 \ub4f1\uc774 \uc788\ub2e4.", "paragraph_id": "5986335-0-0", "paragraph_question": "question: \ubb3c\ub9ac\ud559, \ud654\ud559, \uc0dd\ubb3c\ud559, \uc9c0\uad6c\uacfc\ud559 \ub4f1\uc758 \uae30\ucd08 \ud559\ubb38 \ubd84\uc57c\ub294 \ubb34\uc2a8 \uacfc\ud559\uc5d0 \uc18d\ud558\ub098\uc694?, context: \uc778\ubb38\uacfc\ud559\uc5d0 \uc18d\ud558\ub294 \uae30\ucd08 \ud559\ubb38 \ubd84\uc57c\ub85c\ub294 \uc5b8\uc5b4\ud559, \ubb38\ud559, \uc5ed\uc0ac\ud559, \ucca0\ud559, \uc885\uad50\ud559, \uc2ec\ub9ac\ud559 \ub4f1\uc774 \uc788\uace0, \uc790\uc5f0\uacfc\ud559\uc5d0 \uc18d\ud558\ub294 \uae30\ucd08 \ud559\ubb38 \ubd84\uc57c\ub85c\ub294 \uacfc\ud559\uacfc \uae30\uc220 \ubc1c\uc804\uc758 \uae30\ucd08\uac00 \ub418\ub294 \ubb3c\ub9ac\ud559, \ud654\ud559, \uc0dd\ubb3c\ud559,\uc9c0\uad6c\uacfc\ud559 \ub4f1\uc774 \uc788\ub2e4. \uace0\ub300\uc5d0\ub294 \uc774\ub7ec\ud55c \uc778\ubb38\u00b7\uc0ac\ud68c\uacfc\ud559\uacfc \uc790\uc5f0\uacfc\ud559\uc758 \uad6c\ubd84\uc774 \uba85\ud655\ud558\uc9c0 \uc54a\uc544\uc11c \ub9ce\uc740 \uace0\ub300 \ud559\uc790\ub4e4\uc740 \uc774\ub7f0 \uc5ec\ub7ec \ubd84\uc57c\uc5d0 \ub300\ud574 \uae30\ucd08 \uc9c0\uc2dd\ub4e4\uc744 \ub450\ub8e8\ub450\ub8e8 \uac00\uc9c0\uace0 \uc788\ub294 \uacbd\uc6b0\uac00 \ub9ce\uc558\uc73c\ub098, \ubb38\uba85 \ubc1c\ub2ec\uacfc \ub354\ubd88\uc5b4 \uc810\ucc28 \uac01\uac01\uc758 \ud559\ubb38 \ubd84\uc57c\uc758 \uc9c0\uc2dd \ucd95\uc801\uc774 \ub9ce\uc544\uc9c0\uba74\uc11c \uc790\uc5f0\uc2a4\ub7fd\uac8c \uac1c\ubcc4 \ubd84\uc57c\ub4e4\uc774 \ubd84\ud654\ub418\uc5b4 \uac01\uac01\uc758 \uc774\ub984\uc744 \uac00\uc9c0\uac8c \ub418\uc5c8\ub2e4. \uadf8\ub7ec\ub098, \uc5ec\uc804\ud788 \ub9ce\uc740 \uae30\ucd08 \ud559\ubb38 \ubd84\uc57c\ub294 \uc18c\uc704 \ub9d0\ud558\ub294 \uc751\uc6a9\uacfc\ud559(\ub610\ub294 \uc2e4\uc6a9\uacfc\ud559) \ubd84\uc57c\uc5d0 \ube44\ud574\uc11c \uc5ed\uc0ac\uac00 \uc624\ub798\ub41c \uac83\uc774 \ub9ce\uace0, \ub610\ud55c \uc774\ub7ec\ud55c \uc751\uc6a9\uacfc\ud559\uc758 \uae30\ucd08\uac00 \ub418\ub294 \ud559\ubb38\uc778 \ub9cc\ud07c \ub9ce\uc740 \uad50\uc721\uacfc\uc815\uc5d0\uc11c\ub294 \uc774\ub7ec\ud55c \uae30\ucd08 \ud559\ubb38\uc758 \uad50\uc721\uc744 \uac15\uc870\ud558\ub294 \uacbd\ud5a5\uc774 \uac15\ud558\ub2e4."} {"question": "\uc2a4\ubbf8\uc2a4 \ubd80\ub300\uc758 \ud328\uc804 \uc774\ud6c4 \ub518 \uc18c\uc7a5\uc774 \uc774\ub044\ub294 \uc721\uad70 24\uc0ac\ub2e8\uc774 \ud22c\uc785\ub41c \uacf3\uc740 \uc5b4\ub514\uc778\uac00?", "paragraph": "\uc2a4\ubbf8\uc2a4 \ubd80\ub300\uc758 \ud328\uc804 \uc774\ud6c4, \ubbf8\uad70\uc740 \uc70c\ub9ac\uc5c4 F. \ub518 \uc18c\uc7a5\uc774 \uc774\ub044\ub294 \uc721\uad70 24\uc0ac\ub2e8\uc744 \ub300\uc804\uc5d0 \ud22c\uc785\ud588\ub2e4. \ub518 \uc18c\uc7a5\uc740 \ub300\uc804\uc5d0\uc11c \uc801\uc758 \uc120\ubd09\uc744 \uaebe\uace0, \ud55c\uac15\uae4c\uc9c0 \ubd81\uc0c1\ud558\uc5ec \ubc29\uc5b4\uc120\uc744 \uad6c\ucd95\ud558\uaca0\ub2e4\uace0 \uc7a5\ub2f4\ud588\uc73c\ub098, \ubbf8 \uc721\uad70 24\uc0ac\ub2e8\ub3c4 T-34\uc5d0 \ub300\uc801\ud560 \ud654\uae30\uac00 \uc5c6\uc5c8\ub2e4. \ub300\uc804\uc744 \uacf5\uaca9\ud558\ub294 \uc870\uc120\uc778\ubbfc\uad70\uc740 \uad70\uc778\ub4e4\uc758 \uc22b\uc790\ub3c4 \ub9ce\uc558\uace0, \uc0ac\uae30\uac00 \ub192\uc544 T-34\ub97c \uc55e\uc138\uc6cc \ubbf8 \uc721\uad70 24\uc0ac\ub2e8\uc744 \uacf5\uaca9\ud588\ub2e4. \uacc4\uc18d\ub41c \uc778\ubbfc\uad70\uc758 \uacf5\uaca9\uc5d0 \uc0c1\ub2f9\ud55c \uc190\uc2e4\uc744 \ub0b4\ub358 \ub300\uc804\uc758 \ubbf8 \uc721\uad70 24\uc0ac\ub2e8\uc740 \ub9c8\uce68\ub0b4 7\uc6d4 20\uc77c \ud328\uc8fc\ud558\uace0 \ub9d0\uc558\ub2e4. \uc0ac\ubc29\uc5d0\uc11c\ub294 \uc555\ub3c4\uc801\uc778 \uc22b\uc790\uc758 \uc870\uc120\uc778\ubbfc\uad70\uc774 \ubc00\ub824\uc654\uace0, \uc9c0\ud718\uacc4\ud1b5\uc774 \ubb34\ub108\uc9c4 \uac00\uc6b4\ub370 \ubbf8 \uc721\uad70 24\uc0ac\ub2e8\uc740 \ubb34\uc9c8\uc11c\ud558\uac8c \ud6c4\ud1f4\ud588\ub2e4. \uc81c24\uc0ac\ub2e8\uc7a5\uc778 \uc70c\ub9ac\uc5c4 F. \ub518 \uc18c\uc7a5\uc740 \ud568\uaed8 \ud6c4\ud1f4\ud558\ub358 \ubd80\uc0c1\ubcd1\uc5d0\uac8c \uba39\uc77c \ubb3c\uc744 \uad6c\ud558\ub358 \uc911 \ub0ad\ub5a0\ub7ec\uc9c0\uc5d0\uc11c \uc2e4\uc871\ud558\uc5ec \uae38\uc744 \uc783\uc5c8\ub2e4\uac00 \uc778\ubbfc\uad70\uc5d0 \uc758\ud574 \ud3ec\ub85c\ub85c \ubd99\uc7a1\ud614\ub2e4. \ud55c\ud3b8, \ub300\ud55c\ubbfc\uad6d\uc740 \ub300\uc804\uc744 \ube7c\uc557\uae30\uace0 \ub09c \ub2e4\uc74c\uc5d0 \ub300\uad6c\ub97c \uac70\uccd0 \ubd80\uc0b0\uc744 \uc784\uc2dc\uc218\ub3c4\ub85c \uc815\ud588\ub2e4.", "answer": "\ub300\uc804", "sentence": "\uc2a4\ubbf8\uc2a4 \ubd80\ub300\uc758 \ud328\uc804 \uc774\ud6c4, \ubbf8\uad70\uc740 \uc70c\ub9ac\uc5c4 F. \ub518 \uc18c\uc7a5\uc774 \uc774\ub044\ub294 \uc721\uad70 24\uc0ac\ub2e8\uc744 \ub300\uc804 \uc5d0 \ud22c\uc785\ud588\ub2e4.", "paragraph_sentence": " \uc2a4\ubbf8\uc2a4 \ubd80\ub300\uc758 \ud328\uc804 \uc774\ud6c4, \ubbf8\uad70\uc740 \uc70c\ub9ac\uc5c4 F. \ub518 \uc18c\uc7a5\uc774 \uc774\ub044\ub294 \uc721\uad70 24\uc0ac\ub2e8\uc744 \ub300\uc804 \uc5d0 \ud22c\uc785\ud588\ub2e4. \ub518 \uc18c\uc7a5\uc740 \ub300\uc804\uc5d0\uc11c \uc801\uc758 \uc120\ubd09\uc744 \uaebe\uace0, \ud55c\uac15\uae4c\uc9c0 \ubd81\uc0c1\ud558\uc5ec \ubc29\uc5b4\uc120\uc744 \uad6c\ucd95\ud558\uaca0\ub2e4\uace0 \uc7a5\ub2f4\ud588\uc73c\ub098, \ubbf8 \uc721\uad70 24\uc0ac\ub2e8\ub3c4 T-34\uc5d0 \ub300\uc801\ud560 \ud654\uae30\uac00 \uc5c6\uc5c8\ub2e4. \ub300\uc804\uc744 \uacf5\uaca9\ud558\ub294 \uc870\uc120\uc778\ubbfc\uad70\uc740 \uad70\uc778\ub4e4\uc758 \uc22b\uc790\ub3c4 \ub9ce\uc558\uace0, \uc0ac\uae30\uac00 \ub192\uc544 T-34\ub97c \uc55e\uc138\uc6cc \ubbf8 \uc721\uad70 24\uc0ac\ub2e8\uc744 \uacf5\uaca9\ud588\ub2e4. \uacc4\uc18d\ub41c \uc778\ubbfc\uad70\uc758 \uacf5\uaca9\uc5d0 \uc0c1\ub2f9\ud55c \uc190\uc2e4\uc744 \ub0b4\ub358 \ub300\uc804\uc758 \ubbf8 \uc721\uad70 24\uc0ac\ub2e8\uc740 \ub9c8\uce68\ub0b4 7\uc6d4 20\uc77c \ud328\uc8fc\ud558\uace0 \ub9d0\uc558\ub2e4. \uc0ac\ubc29\uc5d0\uc11c\ub294 \uc555\ub3c4\uc801\uc778 \uc22b\uc790\uc758 \uc870\uc120\uc778\ubbfc\uad70\uc774 \ubc00\ub824\uc654\uace0, \uc9c0\ud718\uacc4\ud1b5\uc774 \ubb34\ub108\uc9c4 \uac00\uc6b4\ub370 \ubbf8 \uc721\uad70 24\uc0ac\ub2e8\uc740 \ubb34\uc9c8\uc11c\ud558\uac8c \ud6c4\ud1f4\ud588\ub2e4. \uc81c24\uc0ac\ub2e8\uc7a5\uc778 \uc70c\ub9ac\uc5c4 F. \ub518 \uc18c\uc7a5\uc740 \ud568\uaed8 \ud6c4\ud1f4\ud558\ub358 \ubd80\uc0c1\ubcd1\uc5d0\uac8c \uba39\uc77c \ubb3c\uc744 \uad6c\ud558\ub358 \uc911 \ub0ad\ub5a0\ub7ec\uc9c0\uc5d0\uc11c \uc2e4\uc871\ud558\uc5ec \uae38\uc744 \uc783\uc5c8\ub2e4\uac00 \uc778\ubbfc\uad70\uc5d0 \uc758\ud574 \ud3ec\ub85c\ub85c \ubd99\uc7a1\ud614\ub2e4. \ud55c\ud3b8, \ub300\ud55c\ubbfc\uad6d\uc740 \ub300\uc804\uc744 \ube7c\uc557\uae30\uace0 \ub09c \ub2e4\uc74c\uc5d0 \ub300\uad6c\ub97c \uac70\uccd0 \ubd80\uc0b0\uc744 \uc784\uc2dc\uc218\ub3c4\ub85c \uc815\ud588\ub2e4.", "paragraph_answer": "\uc2a4\ubbf8\uc2a4 \ubd80\ub300\uc758 \ud328\uc804 \uc774\ud6c4, \ubbf8\uad70\uc740 \uc70c\ub9ac\uc5c4 F. \ub518 \uc18c\uc7a5\uc774 \uc774\ub044\ub294 \uc721\uad70 24\uc0ac\ub2e8\uc744 \ub300\uc804 \uc5d0 \ud22c\uc785\ud588\ub2e4. \ub518 \uc18c\uc7a5\uc740 \ub300\uc804\uc5d0\uc11c \uc801\uc758 \uc120\ubd09\uc744 \uaebe\uace0, \ud55c\uac15\uae4c\uc9c0 \ubd81\uc0c1\ud558\uc5ec \ubc29\uc5b4\uc120\uc744 \uad6c\ucd95\ud558\uaca0\ub2e4\uace0 \uc7a5\ub2f4\ud588\uc73c\ub098, \ubbf8 \uc721\uad70 24\uc0ac\ub2e8\ub3c4 T-34\uc5d0 \ub300\uc801\ud560 \ud654\uae30\uac00 \uc5c6\uc5c8\ub2e4. \ub300\uc804\uc744 \uacf5\uaca9\ud558\ub294 \uc870\uc120\uc778\ubbfc\uad70\uc740 \uad70\uc778\ub4e4\uc758 \uc22b\uc790\ub3c4 \ub9ce\uc558\uace0, \uc0ac\uae30\uac00 \ub192\uc544 T-34\ub97c \uc55e\uc138\uc6cc \ubbf8 \uc721\uad70 24\uc0ac\ub2e8\uc744 \uacf5\uaca9\ud588\ub2e4. \uacc4\uc18d\ub41c \uc778\ubbfc\uad70\uc758 \uacf5\uaca9\uc5d0 \uc0c1\ub2f9\ud55c \uc190\uc2e4\uc744 \ub0b4\ub358 \ub300\uc804\uc758 \ubbf8 \uc721\uad70 24\uc0ac\ub2e8\uc740 \ub9c8\uce68\ub0b4 7\uc6d4 20\uc77c \ud328\uc8fc\ud558\uace0 \ub9d0\uc558\ub2e4. \uc0ac\ubc29\uc5d0\uc11c\ub294 \uc555\ub3c4\uc801\uc778 \uc22b\uc790\uc758 \uc870\uc120\uc778\ubbfc\uad70\uc774 \ubc00\ub824\uc654\uace0, \uc9c0\ud718\uacc4\ud1b5\uc774 \ubb34\ub108\uc9c4 \uac00\uc6b4\ub370 \ubbf8 \uc721\uad70 24\uc0ac\ub2e8\uc740 \ubb34\uc9c8\uc11c\ud558\uac8c \ud6c4\ud1f4\ud588\ub2e4. \uc81c24\uc0ac\ub2e8\uc7a5\uc778 \uc70c\ub9ac\uc5c4 F. \ub518 \uc18c\uc7a5\uc740 \ud568\uaed8 \ud6c4\ud1f4\ud558\ub358 \ubd80\uc0c1\ubcd1\uc5d0\uac8c \uba39\uc77c \ubb3c\uc744 \uad6c\ud558\ub358 \uc911 \ub0ad\ub5a0\ub7ec\uc9c0\uc5d0\uc11c \uc2e4\uc871\ud558\uc5ec \uae38\uc744 \uc783\uc5c8\ub2e4\uac00 \uc778\ubbfc\uad70\uc5d0 \uc758\ud574 \ud3ec\ub85c\ub85c \ubd99\uc7a1\ud614\ub2e4. \ud55c\ud3b8, \ub300\ud55c\ubbfc\uad6d\uc740 \ub300\uc804\uc744 \ube7c\uc557\uae30\uace0 \ub09c \ub2e4\uc74c\uc5d0 \ub300\uad6c\ub97c \uac70\uccd0 \ubd80\uc0b0\uc744 \uc784\uc2dc\uc218\ub3c4\ub85c \uc815\ud588\ub2e4.", "sentence_answer": "\uc2a4\ubbf8\uc2a4 \ubd80\ub300\uc758 \ud328\uc804 \uc774\ud6c4, \ubbf8\uad70\uc740 \uc70c\ub9ac\uc5c4 F. \ub518 \uc18c\uc7a5\uc774 \uc774\ub044\ub294 \uc721\uad70 24\uc0ac\ub2e8\uc744 \ub300\uc804 \uc5d0 \ud22c\uc785\ud588\ub2e4.", "paragraph_id": "6569867-37-0", "paragraph_question": "question: \uc2a4\ubbf8\uc2a4 \ubd80\ub300\uc758 \ud328\uc804 \uc774\ud6c4 \ub518 \uc18c\uc7a5\uc774 \uc774\ub044\ub294 \uc721\uad70 24\uc0ac\ub2e8\uc774 \ud22c\uc785\ub41c \uacf3\uc740 \uc5b4\ub514\uc778\uac00?, context: \uc2a4\ubbf8\uc2a4 \ubd80\ub300\uc758 \ud328\uc804 \uc774\ud6c4, \ubbf8\uad70\uc740 \uc70c\ub9ac\uc5c4 F. \ub518 \uc18c\uc7a5\uc774 \uc774\ub044\ub294 \uc721\uad70 24\uc0ac\ub2e8\uc744 \ub300\uc804\uc5d0 \ud22c\uc785\ud588\ub2e4. \ub518 \uc18c\uc7a5\uc740 \ub300\uc804\uc5d0\uc11c \uc801\uc758 \uc120\ubd09\uc744 \uaebe\uace0, \ud55c\uac15\uae4c\uc9c0 \ubd81\uc0c1\ud558\uc5ec \ubc29\uc5b4\uc120\uc744 \uad6c\ucd95\ud558\uaca0\ub2e4\uace0 \uc7a5\ub2f4\ud588\uc73c\ub098, \ubbf8 \uc721\uad70 24\uc0ac\ub2e8\ub3c4 T-34\uc5d0 \ub300\uc801\ud560 \ud654\uae30\uac00 \uc5c6\uc5c8\ub2e4. \ub300\uc804\uc744 \uacf5\uaca9\ud558\ub294 \uc870\uc120\uc778\ubbfc\uad70\uc740 \uad70\uc778\ub4e4\uc758 \uc22b\uc790\ub3c4 \ub9ce\uc558\uace0, \uc0ac\uae30\uac00 \ub192\uc544 T-34\ub97c \uc55e\uc138\uc6cc \ubbf8 \uc721\uad70 24\uc0ac\ub2e8\uc744 \uacf5\uaca9\ud588\ub2e4. \uacc4\uc18d\ub41c \uc778\ubbfc\uad70\uc758 \uacf5\uaca9\uc5d0 \uc0c1\ub2f9\ud55c \uc190\uc2e4\uc744 \ub0b4\ub358 \ub300\uc804\uc758 \ubbf8 \uc721\uad70 24\uc0ac\ub2e8\uc740 \ub9c8\uce68\ub0b4 7\uc6d4 20\uc77c \ud328\uc8fc\ud558\uace0 \ub9d0\uc558\ub2e4. \uc0ac\ubc29\uc5d0\uc11c\ub294 \uc555\ub3c4\uc801\uc778 \uc22b\uc790\uc758 \uc870\uc120\uc778\ubbfc\uad70\uc774 \ubc00\ub824\uc654\uace0, \uc9c0\ud718\uacc4\ud1b5\uc774 \ubb34\ub108\uc9c4 \uac00\uc6b4\ub370 \ubbf8 \uc721\uad70 24\uc0ac\ub2e8\uc740 \ubb34\uc9c8\uc11c\ud558\uac8c \ud6c4\ud1f4\ud588\ub2e4. \uc81c24\uc0ac\ub2e8\uc7a5\uc778 \uc70c\ub9ac\uc5c4 F. \ub518 \uc18c\uc7a5\uc740 \ud568\uaed8 \ud6c4\ud1f4\ud558\ub358 \ubd80\uc0c1\ubcd1\uc5d0\uac8c \uba39\uc77c \ubb3c\uc744 \uad6c\ud558\ub358 \uc911 \ub0ad\ub5a0\ub7ec\uc9c0\uc5d0\uc11c \uc2e4\uc871\ud558\uc5ec \uae38\uc744 \uc783\uc5c8\ub2e4\uac00 \uc778\ubbfc\uad70\uc5d0 \uc758\ud574 \ud3ec\ub85c\ub85c \ubd99\uc7a1\ud614\ub2e4. \ud55c\ud3b8, \ub300\ud55c\ubbfc\uad6d\uc740 \ub300\uc804\uc744 \ube7c\uc557\uae30\uace0 \ub09c \ub2e4\uc74c\uc5d0 \ub300\uad6c\ub97c \uac70\uccd0 \ubd80\uc0b0\uc744 \uc784\uc2dc\uc218\ub3c4\ub85c \uc815\ud588\ub2e4."} {"question": "\uac70\ub300\ud55c \uc77c\uc2dd \uc774\ud6c4 \uc138\uacc4 \ub3c4\ucc98\uc5d0 \ub098\ud0c0\ub09c \ud2b9\ubcc4\ud55c \ub2a5\ub825\uc744 \uac00\uc9c4 \uc0ac\ub78c\ub4e4\uc744 \ubb34\uc5c7\uc774\ub77c \ud558\ub294\uac00?", "paragraph": "1860\ub144 \uac70\ub300\ud55c \uc77c\uc2dd \uc774\ud6c4 \uc138\uacc4 \ub3c4\ucc98\uc5d0\uc11c\ub294 \ud669\ubb34\uc9c0 \uc704\uc5d0 \ub3c4\uc2dc\uac00 \uc0dd\uaca8\ub098\uac70\ub098 \ud2b9\ubcc4\ud55c \ub2a5\ub825\uc744 \uac00\uc9c4 \uc0ac\ub78c\ub4e4\uc774 \ub098\ud0c0\ub098\uac8c \ub41c\ub2e4. \uc774\ub4e4\uc740 \uc0ac\uc774\ud37c(Cypher)\ub77c\uace0 \ubd88\ub9ac\uba70 \ud0c4\uc555\uc758 \ub300\uc0c1\uc774 \ub418\ub294\ub370, \uc5ec\uae30 \ubc18\ud558\uc5ec \ud5ec\ub9ac\uc624\uc2a4\ub77c\ub294 \ud68c\uc0ac\uac00 \uc124\ub9bd\ub41c\ub2e4. \ubfd0\ub9cc \uc544\ub2c8\ub77c \uc81c\ub3c4\uc758 \uc190\uc774 \ubbf8\uce58\uce58 \uc54a\ub294 \uacf3\uc5d0\uc11c\ub3c4 \uc2a4\uc2a4\ub85c\uc758 \uad8c\ub9ac\ub97c \ubcf4\ud638\ud558\uae30 \uc704\ud574 \uc9c0\ud558\uc5f0\ud569\uc774 \uacb0\uc131\ub41c\ub2e4. \ub450 \uc138\ub825\uc740 \uac01\uc790\uc758 \uc601\uc5ed\uc5d0\uc11c \uacf5\uc874\ud574\uc654\uc9c0\ub9cc, \uba87 \ucc28\ub840 \ub9c8\ucc30\ub3c4 \uc874\uc7ac\ud558\uc600\ub2e4. \uadf8\ub7ec\ub358 \ub3c4\uc911 \ub178\uc778\uc774 \ub9cc\ub4e0 \uc548\ud0c0\ub9ac\uc6b0\uc2a4\ub77c\ub294 \uc9d1\ub2e8\uc774 \uc0dd\uaca8\ub098 \uad11\uc2e0\uc801\uc778 \uc9c0\uc9c0\ub97c \ubc14\ud0d5\uc73c\ub85c \uae09\uaca9\ud558\uac8c \uc131\uc7a5\ud588\uace0, \uc778\uac04\uc744 \uc0c1\ub300\ub85c \ub054\ucc0d\ud55c \uc2e4\ud5d8\uc744 \uc790\ud589\ud558\uae30\uc5d0 \uc774\ub978\ub2e4. \uc138\ub825\uc744 \ud0a4\uc6b4 \uc548\ud0c0\ub9ac\uc6b0\uc2a4\ub294 \uc138\uacc4\ub97c \ud5a5\ud574 \uc804\uba74\uc804\uc744 \uc120\ud3ec\ud558\uc9c0\ub9cc, \ud68c\uc0ac\uc640 \uc5f0\ud569\uc740 \ub3d9\ub9f9\uc744 \ub9fa\uc5b4 \uc778\ud615\uc2e4 \ub04a\uae30 \uc791\uc804\uc744 \ud1b5\ud574 \ub178\uc778\uc744 \uc81c\uac70\ud558\uace0 \uc548\ud0c0\ub9ac\uc6b0\uc2a4\ub97c \uada4\uba78\uc2dc\ud0a8\ub2e4. \uada4\uba78 \uc774\ud6c4 \uc7a0\uae50\ub3d9\uc548 \ud3c9\ud654\uc758 \uc2dc\ub300\uac00 \ucc3e\uc544\uc654\uc73c\ub098, \ud3c9\ud654\ub294 \ub2e4\uc2dc\uae08 \uae68\uc9c0\ub294 \uae30\ubbf8\uac00 \ubcf4\uc774\uace0, \ud769\uc5b4\uc9c4 \uc548\ud0c0\ub9ac\uc6b0\uc2a4 \uc138\ub825\ub9c8\uc800 \uacf3\uacf3\uc5d0\uc11c \ubaa8\uc2b5\uc744 \ub4dc\ub7ec\ub0b8\ub2e4.", "answer": "\uc0ac\uc774\ud37c", "sentence": "\uc774\ub4e4\uc740 \uc0ac\uc774\ud37c (Cypher)", "paragraph_sentence": "1860\ub144 \uac70\ub300\ud55c \uc77c\uc2dd \uc774\ud6c4 \uc138\uacc4 \ub3c4\ucc98\uc5d0\uc11c\ub294 \ud669\ubb34\uc9c0 \uc704\uc5d0 \ub3c4\uc2dc\uac00 \uc0dd\uaca8\ub098\uac70\ub098 \ud2b9\ubcc4\ud55c \ub2a5\ub825\uc744 \uac00\uc9c4 \uc0ac\ub78c\ub4e4\uc774 \ub098\ud0c0\ub098\uac8c \ub41c\ub2e4. \uc774\ub4e4\uc740 \uc0ac\uc774\ud37c (Cypher) \ub77c\uace0 \ubd88\ub9ac\uba70 \ud0c4\uc555\uc758 \ub300\uc0c1\uc774 \ub418\ub294\ub370, \uc5ec\uae30 \ubc18\ud558\uc5ec \ud5ec\ub9ac\uc624\uc2a4\ub77c\ub294 \ud68c\uc0ac\uac00 \uc124\ub9bd\ub41c\ub2e4. \ubfd0\ub9cc \uc544\ub2c8\ub77c \uc81c\ub3c4\uc758 \uc190\uc774 \ubbf8\uce58\uce58 \uc54a\ub294 \uacf3\uc5d0\uc11c\ub3c4 \uc2a4\uc2a4\ub85c\uc758 \uad8c\ub9ac\ub97c \ubcf4\ud638\ud558\uae30 \uc704\ud574 \uc9c0\ud558\uc5f0\ud569\uc774 \uacb0\uc131\ub41c\ub2e4. \ub450 \uc138\ub825\uc740 \uac01\uc790\uc758 \uc601\uc5ed\uc5d0\uc11c \uacf5\uc874\ud574\uc654\uc9c0\ub9cc, \uba87 \ucc28\ub840 \ub9c8\ucc30\ub3c4 \uc874\uc7ac\ud558\uc600\ub2e4. \uadf8\ub7ec\ub358 \ub3c4\uc911 \ub178\uc778\uc774 \ub9cc\ub4e0 \uc548\ud0c0\ub9ac\uc6b0\uc2a4\ub77c\ub294 \uc9d1\ub2e8\uc774 \uc0dd\uaca8\ub098 \uad11\uc2e0\uc801\uc778 \uc9c0\uc9c0\ub97c \ubc14\ud0d5\uc73c\ub85c \uae09\uaca9\ud558\uac8c \uc131\uc7a5\ud588\uace0, \uc778\uac04\uc744 \uc0c1\ub300\ub85c \ub054\ucc0d\ud55c \uc2e4\ud5d8\uc744 \uc790\ud589\ud558\uae30\uc5d0 \uc774\ub978\ub2e4. \uc138\ub825\uc744 \ud0a4\uc6b4 \uc548\ud0c0\ub9ac\uc6b0\uc2a4\ub294 \uc138\uacc4\ub97c \ud5a5\ud574 \uc804\uba74\uc804\uc744 \uc120\ud3ec\ud558\uc9c0\ub9cc, \ud68c\uc0ac\uc640 \uc5f0\ud569\uc740 \ub3d9\ub9f9\uc744 \ub9fa\uc5b4 \uc778\ud615\uc2e4 \ub04a\uae30 \uc791\uc804\uc744 \ud1b5\ud574 \ub178\uc778\uc744 \uc81c\uac70\ud558\uace0 \uc548\ud0c0\ub9ac\uc6b0\uc2a4\ub97c \uada4\uba78\uc2dc\ud0a8\ub2e4. \uada4\uba78 \uc774\ud6c4 \uc7a0\uae50\ub3d9\uc548 \ud3c9\ud654\uc758 \uc2dc\ub300\uac00 \ucc3e\uc544\uc654\uc73c\ub098, \ud3c9\ud654\ub294 \ub2e4\uc2dc\uae08 \uae68\uc9c0\ub294 \uae30\ubbf8\uac00 \ubcf4\uc774\uace0, \ud769\uc5b4\uc9c4 \uc548\ud0c0\ub9ac\uc6b0\uc2a4 \uc138\ub825\ub9c8\uc800 \uacf3\uacf3\uc5d0\uc11c \ubaa8\uc2b5\uc744 \ub4dc\ub7ec\ub0b8\ub2e4.", "paragraph_answer": "1860\ub144 \uac70\ub300\ud55c \uc77c\uc2dd \uc774\ud6c4 \uc138\uacc4 \ub3c4\ucc98\uc5d0\uc11c\ub294 \ud669\ubb34\uc9c0 \uc704\uc5d0 \ub3c4\uc2dc\uac00 \uc0dd\uaca8\ub098\uac70\ub098 \ud2b9\ubcc4\ud55c \ub2a5\ub825\uc744 \uac00\uc9c4 \uc0ac\ub78c\ub4e4\uc774 \ub098\ud0c0\ub098\uac8c \ub41c\ub2e4. \uc774\ub4e4\uc740 \uc0ac\uc774\ud37c (Cypher)\ub77c\uace0 \ubd88\ub9ac\uba70 \ud0c4\uc555\uc758 \ub300\uc0c1\uc774 \ub418\ub294\ub370, \uc5ec\uae30 \ubc18\ud558\uc5ec \ud5ec\ub9ac\uc624\uc2a4\ub77c\ub294 \ud68c\uc0ac\uac00 \uc124\ub9bd\ub41c\ub2e4. \ubfd0\ub9cc \uc544\ub2c8\ub77c \uc81c\ub3c4\uc758 \uc190\uc774 \ubbf8\uce58\uce58 \uc54a\ub294 \uacf3\uc5d0\uc11c\ub3c4 \uc2a4\uc2a4\ub85c\uc758 \uad8c\ub9ac\ub97c \ubcf4\ud638\ud558\uae30 \uc704\ud574 \uc9c0\ud558\uc5f0\ud569\uc774 \uacb0\uc131\ub41c\ub2e4. \ub450 \uc138\ub825\uc740 \uac01\uc790\uc758 \uc601\uc5ed\uc5d0\uc11c \uacf5\uc874\ud574\uc654\uc9c0\ub9cc, \uba87 \ucc28\ub840 \ub9c8\ucc30\ub3c4 \uc874\uc7ac\ud558\uc600\ub2e4. \uadf8\ub7ec\ub358 \ub3c4\uc911 \ub178\uc778\uc774 \ub9cc\ub4e0 \uc548\ud0c0\ub9ac\uc6b0\uc2a4\ub77c\ub294 \uc9d1\ub2e8\uc774 \uc0dd\uaca8\ub098 \uad11\uc2e0\uc801\uc778 \uc9c0\uc9c0\ub97c \ubc14\ud0d5\uc73c\ub85c \uae09\uaca9\ud558\uac8c \uc131\uc7a5\ud588\uace0, \uc778\uac04\uc744 \uc0c1\ub300\ub85c \ub054\ucc0d\ud55c \uc2e4\ud5d8\uc744 \uc790\ud589\ud558\uae30\uc5d0 \uc774\ub978\ub2e4. \uc138\ub825\uc744 \ud0a4\uc6b4 \uc548\ud0c0\ub9ac\uc6b0\uc2a4\ub294 \uc138\uacc4\ub97c \ud5a5\ud574 \uc804\uba74\uc804\uc744 \uc120\ud3ec\ud558\uc9c0\ub9cc, \ud68c\uc0ac\uc640 \uc5f0\ud569\uc740 \ub3d9\ub9f9\uc744 \ub9fa\uc5b4 \uc778\ud615\uc2e4 \ub04a\uae30 \uc791\uc804\uc744 \ud1b5\ud574 \ub178\uc778\uc744 \uc81c\uac70\ud558\uace0 \uc548\ud0c0\ub9ac\uc6b0\uc2a4\ub97c \uada4\uba78\uc2dc\ud0a8\ub2e4. \uada4\uba78 \uc774\ud6c4 \uc7a0\uae50\ub3d9\uc548 \ud3c9\ud654\uc758 \uc2dc\ub300\uac00 \ucc3e\uc544\uc654\uc73c\ub098, \ud3c9\ud654\ub294 \ub2e4\uc2dc\uae08 \uae68\uc9c0\ub294 \uae30\ubbf8\uac00 \ubcf4\uc774\uace0, \ud769\uc5b4\uc9c4 \uc548\ud0c0\ub9ac\uc6b0\uc2a4 \uc138\ub825\ub9c8\uc800 \uacf3\uacf3\uc5d0\uc11c \ubaa8\uc2b5\uc744 \ub4dc\ub7ec\ub0b8\ub2e4.", "sentence_answer": "\uc774\ub4e4\uc740 \uc0ac\uc774\ud37c (Cypher)", "paragraph_id": "6583642-0-0", "paragraph_question": "question: \uac70\ub300\ud55c \uc77c\uc2dd \uc774\ud6c4 \uc138\uacc4 \ub3c4\ucc98\uc5d0 \ub098\ud0c0\ub09c \ud2b9\ubcc4\ud55c \ub2a5\ub825\uc744 \uac00\uc9c4 \uc0ac\ub78c\ub4e4\uc744 \ubb34\uc5c7\uc774\ub77c \ud558\ub294\uac00?, context: 1860\ub144 \uac70\ub300\ud55c \uc77c\uc2dd \uc774\ud6c4 \uc138\uacc4 \ub3c4\ucc98\uc5d0\uc11c\ub294 \ud669\ubb34\uc9c0 \uc704\uc5d0 \ub3c4\uc2dc\uac00 \uc0dd\uaca8\ub098\uac70\ub098 \ud2b9\ubcc4\ud55c \ub2a5\ub825\uc744 \uac00\uc9c4 \uc0ac\ub78c\ub4e4\uc774 \ub098\ud0c0\ub098\uac8c \ub41c\ub2e4. \uc774\ub4e4\uc740 \uc0ac\uc774\ud37c(Cypher)\ub77c\uace0 \ubd88\ub9ac\uba70 \ud0c4\uc555\uc758 \ub300\uc0c1\uc774 \ub418\ub294\ub370, \uc5ec\uae30 \ubc18\ud558\uc5ec \ud5ec\ub9ac\uc624\uc2a4\ub77c\ub294 \ud68c\uc0ac\uac00 \uc124\ub9bd\ub41c\ub2e4. \ubfd0\ub9cc \uc544\ub2c8\ub77c \uc81c\ub3c4\uc758 \uc190\uc774 \ubbf8\uce58\uce58 \uc54a\ub294 \uacf3\uc5d0\uc11c\ub3c4 \uc2a4\uc2a4\ub85c\uc758 \uad8c\ub9ac\ub97c \ubcf4\ud638\ud558\uae30 \uc704\ud574 \uc9c0\ud558\uc5f0\ud569\uc774 \uacb0\uc131\ub41c\ub2e4. \ub450 \uc138\ub825\uc740 \uac01\uc790\uc758 \uc601\uc5ed\uc5d0\uc11c \uacf5\uc874\ud574\uc654\uc9c0\ub9cc, \uba87 \ucc28\ub840 \ub9c8\ucc30\ub3c4 \uc874\uc7ac\ud558\uc600\ub2e4. \uadf8\ub7ec\ub358 \ub3c4\uc911 \ub178\uc778\uc774 \ub9cc\ub4e0 \uc548\ud0c0\ub9ac\uc6b0\uc2a4\ub77c\ub294 \uc9d1\ub2e8\uc774 \uc0dd\uaca8\ub098 \uad11\uc2e0\uc801\uc778 \uc9c0\uc9c0\ub97c \ubc14\ud0d5\uc73c\ub85c \uae09\uaca9\ud558\uac8c \uc131\uc7a5\ud588\uace0, \uc778\uac04\uc744 \uc0c1\ub300\ub85c \ub054\ucc0d\ud55c \uc2e4\ud5d8\uc744 \uc790\ud589\ud558\uae30\uc5d0 \uc774\ub978\ub2e4. \uc138\ub825\uc744 \ud0a4\uc6b4 \uc548\ud0c0\ub9ac\uc6b0\uc2a4\ub294 \uc138\uacc4\ub97c \ud5a5\ud574 \uc804\uba74\uc804\uc744 \uc120\ud3ec\ud558\uc9c0\ub9cc, \ud68c\uc0ac\uc640 \uc5f0\ud569\uc740 \ub3d9\ub9f9\uc744 \ub9fa\uc5b4 \uc778\ud615\uc2e4 \ub04a\uae30 \uc791\uc804\uc744 \ud1b5\ud574 \ub178\uc778\uc744 \uc81c\uac70\ud558\uace0 \uc548\ud0c0\ub9ac\uc6b0\uc2a4\ub97c \uada4\uba78\uc2dc\ud0a8\ub2e4. \uada4\uba78 \uc774\ud6c4 \uc7a0\uae50\ub3d9\uc548 \ud3c9\ud654\uc758 \uc2dc\ub300\uac00 \ucc3e\uc544\uc654\uc73c\ub098, \ud3c9\ud654\ub294 \ub2e4\uc2dc\uae08 \uae68\uc9c0\ub294 \uae30\ubbf8\uac00 \ubcf4\uc774\uace0, \ud769\uc5b4\uc9c4 \uc548\ud0c0\ub9ac\uc6b0\uc2a4 \uc138\ub825\ub9c8\uc800 \uacf3\uacf3\uc5d0\uc11c \ubaa8\uc2b5\uc744 \ub4dc\ub7ec\ub0b8\ub2e4."} {"question": "\uc870\uc131\ubbfc\uc758 \uc77c\ubcf8 \ud65c\ub3d9\uc2dc\uc808 \ubc29\uc5b4\uc728\uc740?", "paragraph": "1998\ub144 7\uc6d4 \uc624\ub978\ucabd \ud314\uafc8\uce58 \ubd80\uc0c1\uc73c\ub85c 2\ubc88\uc758 \uc218\uc220\uacfc \uc7ac\ud65c\uc744 \ubc18\ubcf5\ud558\uc600\ub2e4. 1999\ub144 2\uc6d4 \ub2e4\uc2dc \uc624\ub978\ucabd \ud314\uafc8\uce58\ub97c \ubd80\uc0c1\ub2f9\ud588\ub2e4. \uadf8\ub294 \uc27d\uac8c \ubd80\uc0c1\uc5d0\uc11c \ud68c\ubcf5\ub418\uc5c8\uc73c\ub098 \uc7ac\uae30\uc5d0 \uc2e4\ud328\ud558\uace0, \ubd80\uc0c1 \uc774\ud6c4 \uc2e4\uc801\uc774 \ubd80\uc9c4\ud558\uc600\ub2e4. 2000\ub144 12\uc6d4 \ucd5c\uc9c4\uc2e4\uacfc \uacb0\ud63c\ud588\ub2e4. \uadf8\ub294 \uc694\ubbf8\uc6b0\ub9ac \uc790\uc774\uc5b8\uce20 \uc18c\uc18d \uc2dc\uc808 \ube60\ub978 \uc9c1\uad6c\uc640 \ud3ec\ud06c\ubcfc \uc704\uc8fc\uc758 \ud22c\uad6c\ub97c \ud558\uc600\ub2e4. \uc77c\ubcf8 \ud504\ub85c \uc57c\uad6c \ud1b5\uc0b0 11\uc2b9 10\ud328 11\uc138\uc774\ube0c, \ubc29\uc5b4\uc728 2.84\ub97c \ub0a8\uae30\uace0 2002\ub144 10\uc6d4\uc5d0 \uc694\ubbf8\uc6b0\ub9ac\ub97c \uc790\uc9c4 \ud1f4\ub2e8\ud558\uc600\ub2e4. \ud1f4\ub2e8 \uc774\ud6c4 \uadf8\ub294 \ub300\ud55c\ubbfc\uad6d\uc758 \uc57c\uad6c\uacc4\ub85c \uc9c4\ucd9c\ud558\ub824 \ud558\uc600\uc73c\ub098 \uad50\uc12d \uacfc\uc815\uc5d0\uc11c \uc2e4\ud328\ud558\uba74\uc11c \uc7a5\uae30\uac04\uc758 \uacf5\ubc31\uae30\ub97c \uac16\uac8c \ub418\uc5c8\ub2e4. 2002\ub144 12\uc6d4 \ucd5c\uc9c4\uc2e4\uacfc \ud30c\uacbd\uc5d0 \uc774\ub974\uc790 \uadf8\ub294 \ub610 \"\ub2e4\uc2dc \uc57c\uad6c\ub97c \ud558\uace0 \uc2f6\ub2e4\"\uba70 \uc57c\uad6c \ubcf5\uadc0\ub97c \uafc8\uafe8\uc73c\ub098 \uadf8\ub9c8\uc800 \uc88c\uc808\ub418\uc5c8\ub2e4.", "answer": "2.84", "sentence": "\uc77c\ubcf8 \ud504\ub85c \uc57c\uad6c \ud1b5\uc0b0 11\uc2b9 10\ud328 11\uc138\uc774\ube0c, \ubc29\uc5b4\uc728 2.84 \ub97c \ub0a8\uae30\uace0 2002\ub144 10\uc6d4\uc5d0 \uc694\ubbf8\uc6b0\ub9ac\ub97c \uc790\uc9c4 \ud1f4\ub2e8\ud558\uc600\ub2e4.", "paragraph_sentence": "1998\ub144 7\uc6d4 \uc624\ub978\ucabd \ud314\uafc8\uce58 \ubd80\uc0c1\uc73c\ub85c 2\ubc88\uc758 \uc218\uc220\uacfc \uc7ac\ud65c\uc744 \ubc18\ubcf5\ud558\uc600\ub2e4. 1999\ub144 2\uc6d4 \ub2e4\uc2dc \uc624\ub978\ucabd \ud314\uafc8\uce58\ub97c \ubd80\uc0c1\ub2f9\ud588\ub2e4. \uadf8\ub294 \uc27d\uac8c \ubd80\uc0c1\uc5d0\uc11c \ud68c\ubcf5\ub418\uc5c8\uc73c\ub098 \uc7ac\uae30\uc5d0 \uc2e4\ud328\ud558\uace0, \ubd80\uc0c1 \uc774\ud6c4 \uc2e4\uc801\uc774 \ubd80\uc9c4\ud558\uc600\ub2e4. 2000\ub144 12\uc6d4 \ucd5c\uc9c4\uc2e4\uacfc \uacb0\ud63c\ud588\ub2e4. \uadf8\ub294 \uc694\ubbf8\uc6b0\ub9ac \uc790\uc774\uc5b8\uce20 \uc18c\uc18d \uc2dc\uc808 \ube60\ub978 \uc9c1\uad6c\uc640 \ud3ec\ud06c\ubcfc \uc704\uc8fc\uc758 \ud22c\uad6c\ub97c \ud558\uc600\ub2e4. \uc77c\ubcf8 \ud504\ub85c \uc57c\uad6c \ud1b5\uc0b0 11\uc2b9 10\ud328 11\uc138\uc774\ube0c, \ubc29\uc5b4\uc728 2.84 \ub97c \ub0a8\uae30\uace0 2002\ub144 10\uc6d4\uc5d0 \uc694\ubbf8\uc6b0\ub9ac\ub97c \uc790\uc9c4 \ud1f4\ub2e8\ud558\uc600\ub2e4. \ud1f4\ub2e8 \uc774\ud6c4 \uadf8\ub294 \ub300\ud55c\ubbfc\uad6d\uc758 \uc57c\uad6c\uacc4\ub85c \uc9c4\ucd9c\ud558\ub824 \ud558\uc600\uc73c\ub098 \uad50\uc12d \uacfc\uc815\uc5d0\uc11c \uc2e4\ud328\ud558\uba74\uc11c \uc7a5\uae30\uac04\uc758 \uacf5\ubc31\uae30\ub97c \uac16\uac8c \ub418\uc5c8\ub2e4. 2002\ub144 12\uc6d4 \ucd5c\uc9c4\uc2e4\uacfc \ud30c\uacbd\uc5d0 \uc774\ub974\uc790 \uadf8\ub294 \ub610 \"\ub2e4\uc2dc \uc57c\uad6c\ub97c \ud558\uace0 \uc2f6\ub2e4\"\uba70 \uc57c\uad6c \ubcf5\uadc0\ub97c \uafc8\uafe8\uc73c\ub098 \uadf8\ub9c8\uc800 \uc88c\uc808\ub418\uc5c8\ub2e4.", "paragraph_answer": "1998\ub144 7\uc6d4 \uc624\ub978\ucabd \ud314\uafc8\uce58 \ubd80\uc0c1\uc73c\ub85c 2\ubc88\uc758 \uc218\uc220\uacfc \uc7ac\ud65c\uc744 \ubc18\ubcf5\ud558\uc600\ub2e4. 1999\ub144 2\uc6d4 \ub2e4\uc2dc \uc624\ub978\ucabd \ud314\uafc8\uce58\ub97c \ubd80\uc0c1\ub2f9\ud588\ub2e4. \uadf8\ub294 \uc27d\uac8c \ubd80\uc0c1\uc5d0\uc11c \ud68c\ubcf5\ub418\uc5c8\uc73c\ub098 \uc7ac\uae30\uc5d0 \uc2e4\ud328\ud558\uace0, \ubd80\uc0c1 \uc774\ud6c4 \uc2e4\uc801\uc774 \ubd80\uc9c4\ud558\uc600\ub2e4. 2000\ub144 12\uc6d4 \ucd5c\uc9c4\uc2e4\uacfc \uacb0\ud63c\ud588\ub2e4. \uadf8\ub294 \uc694\ubbf8\uc6b0\ub9ac \uc790\uc774\uc5b8\uce20 \uc18c\uc18d \uc2dc\uc808 \ube60\ub978 \uc9c1\uad6c\uc640 \ud3ec\ud06c\ubcfc \uc704\uc8fc\uc758 \ud22c\uad6c\ub97c \ud558\uc600\ub2e4. \uc77c\ubcf8 \ud504\ub85c \uc57c\uad6c \ud1b5\uc0b0 11\uc2b9 10\ud328 11\uc138\uc774\ube0c, \ubc29\uc5b4\uc728 2.84 \ub97c \ub0a8\uae30\uace0 2002\ub144 10\uc6d4\uc5d0 \uc694\ubbf8\uc6b0\ub9ac\ub97c \uc790\uc9c4 \ud1f4\ub2e8\ud558\uc600\ub2e4. \ud1f4\ub2e8 \uc774\ud6c4 \uadf8\ub294 \ub300\ud55c\ubbfc\uad6d\uc758 \uc57c\uad6c\uacc4\ub85c \uc9c4\ucd9c\ud558\ub824 \ud558\uc600\uc73c\ub098 \uad50\uc12d \uacfc\uc815\uc5d0\uc11c \uc2e4\ud328\ud558\uba74\uc11c \uc7a5\uae30\uac04\uc758 \uacf5\ubc31\uae30\ub97c \uac16\uac8c \ub418\uc5c8\ub2e4. 2002\ub144 12\uc6d4 \ucd5c\uc9c4\uc2e4\uacfc \ud30c\uacbd\uc5d0 \uc774\ub974\uc790 \uadf8\ub294 \ub610 \"\ub2e4\uc2dc \uc57c\uad6c\ub97c \ud558\uace0 \uc2f6\ub2e4\"\uba70 \uc57c\uad6c \ubcf5\uadc0\ub97c \uafc8\uafe8\uc73c\ub098 \uadf8\ub9c8\uc800 \uc88c\uc808\ub418\uc5c8\ub2e4.", "sentence_answer": "\uc77c\ubcf8 \ud504\ub85c \uc57c\uad6c \ud1b5\uc0b0 11\uc2b9 10\ud328 11\uc138\uc774\ube0c, \ubc29\uc5b4\uc728 2.84 \ub97c \ub0a8\uae30\uace0 2002\ub144 10\uc6d4\uc5d0 \uc694\ubbf8\uc6b0\ub9ac\ub97c \uc790\uc9c4 \ud1f4\ub2e8\ud558\uc600\ub2e4.", "paragraph_id": "5879548-2-1", "paragraph_question": "question: \uc870\uc131\ubbfc\uc758 \uc77c\ubcf8 \ud65c\ub3d9\uc2dc\uc808 \ubc29\uc5b4\uc728\uc740?, context: 1998\ub144 7\uc6d4 \uc624\ub978\ucabd \ud314\uafc8\uce58 \ubd80\uc0c1\uc73c\ub85c 2\ubc88\uc758 \uc218\uc220\uacfc \uc7ac\ud65c\uc744 \ubc18\ubcf5\ud558\uc600\ub2e4. 1999\ub144 2\uc6d4 \ub2e4\uc2dc \uc624\ub978\ucabd \ud314\uafc8\uce58\ub97c \ubd80\uc0c1\ub2f9\ud588\ub2e4. \uadf8\ub294 \uc27d\uac8c \ubd80\uc0c1\uc5d0\uc11c \ud68c\ubcf5\ub418\uc5c8\uc73c\ub098 \uc7ac\uae30\uc5d0 \uc2e4\ud328\ud558\uace0, \ubd80\uc0c1 \uc774\ud6c4 \uc2e4\uc801\uc774 \ubd80\uc9c4\ud558\uc600\ub2e4. 2000\ub144 12\uc6d4 \ucd5c\uc9c4\uc2e4\uacfc \uacb0\ud63c\ud588\ub2e4. \uadf8\ub294 \uc694\ubbf8\uc6b0\ub9ac \uc790\uc774\uc5b8\uce20 \uc18c\uc18d \uc2dc\uc808 \ube60\ub978 \uc9c1\uad6c\uc640 \ud3ec\ud06c\ubcfc \uc704\uc8fc\uc758 \ud22c\uad6c\ub97c \ud558\uc600\ub2e4. \uc77c\ubcf8 \ud504\ub85c \uc57c\uad6c \ud1b5\uc0b0 11\uc2b9 10\ud328 11\uc138\uc774\ube0c, \ubc29\uc5b4\uc728 2.84\ub97c \ub0a8\uae30\uace0 2002\ub144 10\uc6d4\uc5d0 \uc694\ubbf8\uc6b0\ub9ac\ub97c \uc790\uc9c4 \ud1f4\ub2e8\ud558\uc600\ub2e4. \ud1f4\ub2e8 \uc774\ud6c4 \uadf8\ub294 \ub300\ud55c\ubbfc\uad6d\uc758 \uc57c\uad6c\uacc4\ub85c \uc9c4\ucd9c\ud558\ub824 \ud558\uc600\uc73c\ub098 \uad50\uc12d \uacfc\uc815\uc5d0\uc11c \uc2e4\ud328\ud558\uba74\uc11c \uc7a5\uae30\uac04\uc758 \uacf5\ubc31\uae30\ub97c \uac16\uac8c \ub418\uc5c8\ub2e4. 2002\ub144 12\uc6d4 \ucd5c\uc9c4\uc2e4\uacfc \ud30c\uacbd\uc5d0 \uc774\ub974\uc790 \uadf8\ub294 \ub610 \"\ub2e4\uc2dc \uc57c\uad6c\ub97c \ud558\uace0 \uc2f6\ub2e4\"\uba70 \uc57c\uad6c \ubcf5\uadc0\ub97c \uafc8\uafe8\uc73c\ub098 \uadf8\ub9c8\uc800 \uc88c\uc808\ub418\uc5c8\ub2e4."} {"question": "\uc2a4\uc6e8\ub374\uc5d0\uc11c \uae30\ucd08\ub178\ub839\uc5f0\uae08\uc744 \uba87 \ub144\ub3c4\uc5d0 \uc81c\uc815\ud558\uc600\ub098\uc694?", "paragraph": "\uc2a4\uc6e8\ub374\uc740 \uc0ac\ud68c\uc790\uc720\uc8fc\uc758\uc790\ub4e4\uc774 \ubcf5\uc9c0\uad6d\uac00\ub97c \uc8fc\uc7a5\ud558\uae30 \uc2dc\uc791\ud558\uc600\uace0, 1913\ub144 \ubcf4\uc218\uc815\ub2f9\uc778 \uc2a4\uc6e8\ub374 \uc790\uc720\ub2f9\uc5d0\uc11c \uae30\ucd08\ub178\ub839\uc5f0\uae08\uc744 \uc81c\uc815\ud558\uc600\ub2e4. \uc77c\ubd80\uc5d0\uc120 \ubcf5\uc9c0\uad6d\uac00 \uc774\uc57c\uae30\ub97c \ud558\uba74 \uc88c\ud30c\ub77c\uace0 \uc798\ubabb \ub9d0\ud558\ub294 \uc774\ub4e4\uc774 \uc788\ub294\ub370, \uc2a4\uc6e8\ub374\uc744 \ubcf5\uc9c0\uad6d\uac00\ub85c \ub9cc\ub4e4\uc5b4\uac04 \uc774\ub4e4\uc740 \ubcf4\uc218\uc8fc\uc758\uc790\ub4e4\uc774\uc5c8\ub2e4. \uc624\ud788\ub824 \uc88c\ud30c\uc815\ub2f9\uc778 \uc2a4\uc6e8\ub374 \uc0ac\ud68c\ubbfc\uc8fc\ub178\ub3d9\ub2f9\uc740 \ubcf5\uc9c0\ub97c \ubbfc\uc911\uc758 \uc544\ud3b8\uc774\ub77c\uace0 \uc0dd\uac01\ud574\uc11c \ube44\ud310\uc801\uc774\uc5c8\ub2e4, \ud558\uc9c0\ub9cc 1929\ub144 \ubbf8\uad6d \uacbd\uc81c\ub300\uacf5\ud669\uc758 \uc601\ud5a5\uc73c\ub85c \ub178\ub3d9\uc790\ub4e4\uc758 \uc0b6\uc774 \ub098\ube60\uc9c0\uc790, \uc2a4\uc6e8\ub374 \uc0ac\ud68c\ubbfc\uc8fc\ub178\ub3d9\ub2f9\uc740 \uc778\ubbfc\uc758 \uc9d1 \uc774\ub860 \uc989 \uad6d\uac00\uac00 \uc0ac\ud68c/\uacbd\uc81c\uc801 \uc57d\uc790\ub4e4\uc744 \ubc30\ub824\ud558\ub294 \ub530\ub73b\ud55c \uacf5\ub3d9\uccb4\uac00 \ub418\ub3c4\ub85d \ud55c\ub2e4\ub294 \uc774\ub860\uc744 \uc801\uadf9\uc801 \ub178\ub3d9\uc815\ucc45\uacfc \ubcf5\uc9c0\ub85c\uc368 \uc2e4\ucc9c\ud588\ub2e4. \uc801\uadf9\uc801 \ub178\ub3d9\uc815\ucc45\uc740 \uc815\ubd80\uac00 \uc2e4\uc5c5\ubb38\uc81c\ub97c \ud574\uacb0\ud558\uae30 \uc704\ud574 \uc0ac\uc6a9\uc790\uc640 \ub178\ub3d9\uc790\ub97c \uc5f0\uacb0\ud558\ub294 \uc815\ucc45\uc744 \ub73b\ud558\uba70, \ud55c\uad6d\uc5d0 \ube57\ub300\uba74 \ubbfc\uc8fc\ub178\ucd1d\uc778 \uc2a4\uc6e8\ub374 \ub178\ub3d9\uc870\ud569\ucd1d\uc5f0\ub9f9(LO)\uc5d0\uc11c \uc8fc\uc7a5\ud55c \ub3d9\uc77c\ub178\ub3d9 \ub3d9\uc77c\uc784\uae08\uc744 \ubc1b\uc544\ub4e4\uc600\ub2e4.(\ucc38\uace0\ub85c \ud55c\uad6d\uc758 \ubbfc\uc8fc\ub178\ucd1d\ub3c4 \ub3d9\uc77c\ub178\ub3d9 \ub3d9\uc77c\uc784\uae08\uc774 \uac15\ub839\uc5d0 \uc788\ub2e4.) \ubcf5\uc9c0\uc815\ucc45\uc740 \uae30\ucd08\ub178\ub839\uc5f0\uae08\uc73c\ub85c\uc368 \ub178\ub3d9\ub2a5\ub825\uc774 \uc5c6\ub294 \ub178\ub144\uce35\ub4e4\uc774 \uae30\ucd08\uc0dd\ud65c\uc744 \ud560 \uc218 \uc788\ub3c4\ub85d \ud588\uc73c\uba70, \uc5ec\uc131\ub4e4\uc774 \ucde8\uc9c1\ud560 \uc218 \uc788\ub3c4\ub85d \uac00\uc871\ubcf5\uc9c0\uc815\ucc45\uc744 \uc2e4\uc2dc\ud588\ub2e4.(\ucd9c\ucc98:\u300a\ub300\ud55c\ubbfc\uad6d \ubcf5\uc9c0_7\uac00\uc9c0 \uc9c4\uc2e4\uacfc \uac70\uc9d3\u300b/\uc774\uc815\uc6b0 \uc678 \uc9c0\uc74c/\ub450\ub9ac\ubbf8\ub514\uc5b4 43-45\ucabd)", "answer": "1913\ub144", "sentence": "\uc2a4\uc6e8\ub374\uc740 \uc0ac\ud68c\uc790\uc720\uc8fc\uc758\uc790\ub4e4\uc774 \ubcf5\uc9c0\uad6d\uac00\ub97c \uc8fc\uc7a5\ud558\uae30 \uc2dc\uc791\ud558\uc600\uace0, 1913\ub144 \ubcf4\uc218\uc815\ub2f9\uc778 \uc2a4\uc6e8\ub374 \uc790\uc720\ub2f9\uc5d0\uc11c \uae30\ucd08\ub178\ub839\uc5f0\uae08\uc744 \uc81c\uc815\ud558\uc600\ub2e4.", "paragraph_sentence": " \uc2a4\uc6e8\ub374\uc740 \uc0ac\ud68c\uc790\uc720\uc8fc\uc758\uc790\ub4e4\uc774 \ubcf5\uc9c0\uad6d\uac00\ub97c \uc8fc\uc7a5\ud558\uae30 \uc2dc\uc791\ud558\uc600\uace0, 1913\ub144 \ubcf4\uc218\uc815\ub2f9\uc778 \uc2a4\uc6e8\ub374 \uc790\uc720\ub2f9\uc5d0\uc11c \uae30\ucd08\ub178\ub839\uc5f0\uae08\uc744 \uc81c\uc815\ud558\uc600\ub2e4. \uc77c\ubd80\uc5d0\uc120 \ubcf5\uc9c0\uad6d\uac00 \uc774\uc57c\uae30\ub97c \ud558\uba74 \uc88c\ud30c\ub77c\uace0 \uc798\ubabb \ub9d0\ud558\ub294 \uc774\ub4e4\uc774 \uc788\ub294\ub370, \uc2a4\uc6e8\ub374\uc744 \ubcf5\uc9c0\uad6d\uac00\ub85c \ub9cc\ub4e4\uc5b4\uac04 \uc774\ub4e4\uc740 \ubcf4\uc218\uc8fc\uc758\uc790\ub4e4\uc774\uc5c8\ub2e4. \uc624\ud788\ub824 \uc88c\ud30c\uc815\ub2f9\uc778 \uc2a4\uc6e8\ub374 \uc0ac\ud68c\ubbfc\uc8fc\ub178\ub3d9\ub2f9\uc740 \ubcf5\uc9c0\ub97c \ubbfc\uc911\uc758 \uc544\ud3b8\uc774\ub77c\uace0 \uc0dd\uac01\ud574\uc11c \ube44\ud310\uc801\uc774\uc5c8\ub2e4, \ud558\uc9c0\ub9cc 1929\ub144 \ubbf8\uad6d \uacbd\uc81c\ub300\uacf5\ud669\uc758 \uc601\ud5a5\uc73c\ub85c \ub178\ub3d9\uc790\ub4e4\uc758 \uc0b6\uc774 \ub098\ube60\uc9c0\uc790, \uc2a4\uc6e8\ub374 \uc0ac\ud68c\ubbfc\uc8fc\ub178\ub3d9\ub2f9\uc740 \uc778\ubbfc\uc758 \uc9d1 \uc774\ub860 \uc989 \uad6d\uac00\uac00 \uc0ac\ud68c/\uacbd\uc81c\uc801 \uc57d\uc790\ub4e4\uc744 \ubc30\ub824\ud558\ub294 \ub530\ub73b\ud55c \uacf5\ub3d9\uccb4\uac00 \ub418\ub3c4\ub85d \ud55c\ub2e4\ub294 \uc774\ub860\uc744 \uc801\uadf9\uc801 \ub178\ub3d9\uc815\ucc45\uacfc \ubcf5\uc9c0\ub85c\uc368 \uc2e4\ucc9c\ud588\ub2e4. \uc801\uadf9\uc801 \ub178\ub3d9\uc815\ucc45\uc740 \uc815\ubd80\uac00 \uc2e4\uc5c5\ubb38\uc81c\ub97c \ud574\uacb0\ud558\uae30 \uc704\ud574 \uc0ac\uc6a9\uc790\uc640 \ub178\ub3d9\uc790\ub97c \uc5f0\uacb0\ud558\ub294 \uc815\ucc45\uc744 \ub73b\ud558\uba70, \ud55c\uad6d\uc5d0 \ube57\ub300\uba74 \ubbfc\uc8fc\ub178\ucd1d\uc778 \uc2a4\uc6e8\ub374 \ub178\ub3d9\uc870\ud569\ucd1d\uc5f0\ub9f9(LO)\uc5d0\uc11c \uc8fc\uc7a5\ud55c \ub3d9\uc77c\ub178\ub3d9 \ub3d9\uc77c\uc784\uae08\uc744 \ubc1b\uc544\ub4e4\uc600\ub2e4.(\ucc38\uace0\ub85c \ud55c\uad6d\uc758 \ubbfc\uc8fc\ub178\ucd1d\ub3c4 \ub3d9\uc77c\ub178\ub3d9 \ub3d9\uc77c\uc784\uae08\uc774 \uac15\ub839\uc5d0 \uc788\ub2e4. ) \ubcf5\uc9c0\uc815\ucc45\uc740 \uae30\ucd08\ub178\ub839\uc5f0\uae08\uc73c\ub85c\uc368 \ub178\ub3d9\ub2a5\ub825\uc774 \uc5c6\ub294 \ub178\ub144\uce35\ub4e4\uc774 \uae30\ucd08\uc0dd\ud65c\uc744 \ud560 \uc218 \uc788\ub3c4\ub85d \ud588\uc73c\uba70, \uc5ec\uc131\ub4e4\uc774 \ucde8\uc9c1\ud560 \uc218 \uc788\ub3c4\ub85d \uac00\uc871\ubcf5\uc9c0\uc815\ucc45\uc744 \uc2e4\uc2dc\ud588\ub2e4.(\ucd9c\ucc98:\u300a\ub300\ud55c\ubbfc\uad6d \ubcf5\uc9c0_7\uac00\uc9c0 \uc9c4\uc2e4\uacfc \uac70\uc9d3\u300b/\uc774\uc815\uc6b0 \uc678 \uc9c0\uc74c/\ub450\ub9ac\ubbf8\ub514\uc5b4 43-45\ucabd)", "paragraph_answer": "\uc2a4\uc6e8\ub374\uc740 \uc0ac\ud68c\uc790\uc720\uc8fc\uc758\uc790\ub4e4\uc774 \ubcf5\uc9c0\uad6d\uac00\ub97c \uc8fc\uc7a5\ud558\uae30 \uc2dc\uc791\ud558\uc600\uace0, 1913\ub144 \ubcf4\uc218\uc815\ub2f9\uc778 \uc2a4\uc6e8\ub374 \uc790\uc720\ub2f9\uc5d0\uc11c \uae30\ucd08\ub178\ub839\uc5f0\uae08\uc744 \uc81c\uc815\ud558\uc600\ub2e4. \uc77c\ubd80\uc5d0\uc120 \ubcf5\uc9c0\uad6d\uac00 \uc774\uc57c\uae30\ub97c \ud558\uba74 \uc88c\ud30c\ub77c\uace0 \uc798\ubabb \ub9d0\ud558\ub294 \uc774\ub4e4\uc774 \uc788\ub294\ub370, \uc2a4\uc6e8\ub374\uc744 \ubcf5\uc9c0\uad6d\uac00\ub85c \ub9cc\ub4e4\uc5b4\uac04 \uc774\ub4e4\uc740 \ubcf4\uc218\uc8fc\uc758\uc790\ub4e4\uc774\uc5c8\ub2e4. \uc624\ud788\ub824 \uc88c\ud30c\uc815\ub2f9\uc778 \uc2a4\uc6e8\ub374 \uc0ac\ud68c\ubbfc\uc8fc\ub178\ub3d9\ub2f9\uc740 \ubcf5\uc9c0\ub97c \ubbfc\uc911\uc758 \uc544\ud3b8\uc774\ub77c\uace0 \uc0dd\uac01\ud574\uc11c \ube44\ud310\uc801\uc774\uc5c8\ub2e4, \ud558\uc9c0\ub9cc 1929\ub144 \ubbf8\uad6d \uacbd\uc81c\ub300\uacf5\ud669\uc758 \uc601\ud5a5\uc73c\ub85c \ub178\ub3d9\uc790\ub4e4\uc758 \uc0b6\uc774 \ub098\ube60\uc9c0\uc790, \uc2a4\uc6e8\ub374 \uc0ac\ud68c\ubbfc\uc8fc\ub178\ub3d9\ub2f9\uc740 \uc778\ubbfc\uc758 \uc9d1 \uc774\ub860 \uc989 \uad6d\uac00\uac00 \uc0ac\ud68c/\uacbd\uc81c\uc801 \uc57d\uc790\ub4e4\uc744 \ubc30\ub824\ud558\ub294 \ub530\ub73b\ud55c \uacf5\ub3d9\uccb4\uac00 \ub418\ub3c4\ub85d \ud55c\ub2e4\ub294 \uc774\ub860\uc744 \uc801\uadf9\uc801 \ub178\ub3d9\uc815\ucc45\uacfc \ubcf5\uc9c0\ub85c\uc368 \uc2e4\ucc9c\ud588\ub2e4. \uc801\uadf9\uc801 \ub178\ub3d9\uc815\ucc45\uc740 \uc815\ubd80\uac00 \uc2e4\uc5c5\ubb38\uc81c\ub97c \ud574\uacb0\ud558\uae30 \uc704\ud574 \uc0ac\uc6a9\uc790\uc640 \ub178\ub3d9\uc790\ub97c \uc5f0\uacb0\ud558\ub294 \uc815\ucc45\uc744 \ub73b\ud558\uba70, \ud55c\uad6d\uc5d0 \ube57\ub300\uba74 \ubbfc\uc8fc\ub178\ucd1d\uc778 \uc2a4\uc6e8\ub374 \ub178\ub3d9\uc870\ud569\ucd1d\uc5f0\ub9f9(LO)\uc5d0\uc11c \uc8fc\uc7a5\ud55c \ub3d9\uc77c\ub178\ub3d9 \ub3d9\uc77c\uc784\uae08\uc744 \ubc1b\uc544\ub4e4\uc600\ub2e4.(\ucc38\uace0\ub85c \ud55c\uad6d\uc758 \ubbfc\uc8fc\ub178\ucd1d\ub3c4 \ub3d9\uc77c\ub178\ub3d9 \ub3d9\uc77c\uc784\uae08\uc774 \uac15\ub839\uc5d0 \uc788\ub2e4.) \ubcf5\uc9c0\uc815\ucc45\uc740 \uae30\ucd08\ub178\ub839\uc5f0\uae08\uc73c\ub85c\uc368 \ub178\ub3d9\ub2a5\ub825\uc774 \uc5c6\ub294 \ub178\ub144\uce35\ub4e4\uc774 \uae30\ucd08\uc0dd\ud65c\uc744 \ud560 \uc218 \uc788\ub3c4\ub85d \ud588\uc73c\uba70, \uc5ec\uc131\ub4e4\uc774 \ucde8\uc9c1\ud560 \uc218 \uc788\ub3c4\ub85d \uac00\uc871\ubcf5\uc9c0\uc815\ucc45\uc744 \uc2e4\uc2dc\ud588\ub2e4.(\ucd9c\ucc98:\u300a\ub300\ud55c\ubbfc\uad6d \ubcf5\uc9c0_7\uac00\uc9c0 \uc9c4\uc2e4\uacfc \uac70\uc9d3\u300b/\uc774\uc815\uc6b0 \uc678 \uc9c0\uc74c/\ub450\ub9ac\ubbf8\ub514\uc5b4 43-45\ucabd)", "sentence_answer": "\uc2a4\uc6e8\ub374\uc740 \uc0ac\ud68c\uc790\uc720\uc8fc\uc758\uc790\ub4e4\uc774 \ubcf5\uc9c0\uad6d\uac00\ub97c \uc8fc\uc7a5\ud558\uae30 \uc2dc\uc791\ud558\uc600\uace0, 1913\ub144 \ubcf4\uc218\uc815\ub2f9\uc778 \uc2a4\uc6e8\ub374 \uc790\uc720\ub2f9\uc5d0\uc11c \uae30\ucd08\ub178\ub839\uc5f0\uae08\uc744 \uc81c\uc815\ud558\uc600\ub2e4.", "paragraph_id": "6508937-3-0", "paragraph_question": "question: \uc2a4\uc6e8\ub374\uc5d0\uc11c \uae30\ucd08\ub178\ub839\uc5f0\uae08\uc744 \uba87 \ub144\ub3c4\uc5d0 \uc81c\uc815\ud558\uc600\ub098\uc694?, context: \uc2a4\uc6e8\ub374\uc740 \uc0ac\ud68c\uc790\uc720\uc8fc\uc758\uc790\ub4e4\uc774 \ubcf5\uc9c0\uad6d\uac00\ub97c \uc8fc\uc7a5\ud558\uae30 \uc2dc\uc791\ud558\uc600\uace0, 1913\ub144 \ubcf4\uc218\uc815\ub2f9\uc778 \uc2a4\uc6e8\ub374 \uc790\uc720\ub2f9\uc5d0\uc11c \uae30\ucd08\ub178\ub839\uc5f0\uae08\uc744 \uc81c\uc815\ud558\uc600\ub2e4. \uc77c\ubd80\uc5d0\uc120 \ubcf5\uc9c0\uad6d\uac00 \uc774\uc57c\uae30\ub97c \ud558\uba74 \uc88c\ud30c\ub77c\uace0 \uc798\ubabb \ub9d0\ud558\ub294 \uc774\ub4e4\uc774 \uc788\ub294\ub370, \uc2a4\uc6e8\ub374\uc744 \ubcf5\uc9c0\uad6d\uac00\ub85c \ub9cc\ub4e4\uc5b4\uac04 \uc774\ub4e4\uc740 \ubcf4\uc218\uc8fc\uc758\uc790\ub4e4\uc774\uc5c8\ub2e4. \uc624\ud788\ub824 \uc88c\ud30c\uc815\ub2f9\uc778 \uc2a4\uc6e8\ub374 \uc0ac\ud68c\ubbfc\uc8fc\ub178\ub3d9\ub2f9\uc740 \ubcf5\uc9c0\ub97c \ubbfc\uc911\uc758 \uc544\ud3b8\uc774\ub77c\uace0 \uc0dd\uac01\ud574\uc11c \ube44\ud310\uc801\uc774\uc5c8\ub2e4, \ud558\uc9c0\ub9cc 1929\ub144 \ubbf8\uad6d \uacbd\uc81c\ub300\uacf5\ud669\uc758 \uc601\ud5a5\uc73c\ub85c \ub178\ub3d9\uc790\ub4e4\uc758 \uc0b6\uc774 \ub098\ube60\uc9c0\uc790, \uc2a4\uc6e8\ub374 \uc0ac\ud68c\ubbfc\uc8fc\ub178\ub3d9\ub2f9\uc740 \uc778\ubbfc\uc758 \uc9d1 \uc774\ub860 \uc989 \uad6d\uac00\uac00 \uc0ac\ud68c/\uacbd\uc81c\uc801 \uc57d\uc790\ub4e4\uc744 \ubc30\ub824\ud558\ub294 \ub530\ub73b\ud55c \uacf5\ub3d9\uccb4\uac00 \ub418\ub3c4\ub85d \ud55c\ub2e4\ub294 \uc774\ub860\uc744 \uc801\uadf9\uc801 \ub178\ub3d9\uc815\ucc45\uacfc \ubcf5\uc9c0\ub85c\uc368 \uc2e4\ucc9c\ud588\ub2e4. \uc801\uadf9\uc801 \ub178\ub3d9\uc815\ucc45\uc740 \uc815\ubd80\uac00 \uc2e4\uc5c5\ubb38\uc81c\ub97c \ud574\uacb0\ud558\uae30 \uc704\ud574 \uc0ac\uc6a9\uc790\uc640 \ub178\ub3d9\uc790\ub97c \uc5f0\uacb0\ud558\ub294 \uc815\ucc45\uc744 \ub73b\ud558\uba70, \ud55c\uad6d\uc5d0 \ube57\ub300\uba74 \ubbfc\uc8fc\ub178\ucd1d\uc778 \uc2a4\uc6e8\ub374 \ub178\ub3d9\uc870\ud569\ucd1d\uc5f0\ub9f9(LO)\uc5d0\uc11c \uc8fc\uc7a5\ud55c \ub3d9\uc77c\ub178\ub3d9 \ub3d9\uc77c\uc784\uae08\uc744 \ubc1b\uc544\ub4e4\uc600\ub2e4.(\ucc38\uace0\ub85c \ud55c\uad6d\uc758 \ubbfc\uc8fc\ub178\ucd1d\ub3c4 \ub3d9\uc77c\ub178\ub3d9 \ub3d9\uc77c\uc784\uae08\uc774 \uac15\ub839\uc5d0 \uc788\ub2e4.) \ubcf5\uc9c0\uc815\ucc45\uc740 \uae30\ucd08\ub178\ub839\uc5f0\uae08\uc73c\ub85c\uc368 \ub178\ub3d9\ub2a5\ub825\uc774 \uc5c6\ub294 \ub178\ub144\uce35\ub4e4\uc774 \uae30\ucd08\uc0dd\ud65c\uc744 \ud560 \uc218 \uc788\ub3c4\ub85d \ud588\uc73c\uba70, \uc5ec\uc131\ub4e4\uc774 \ucde8\uc9c1\ud560 \uc218 \uc788\ub3c4\ub85d \uac00\uc871\ubcf5\uc9c0\uc815\ucc45\uc744 \uc2e4\uc2dc\ud588\ub2e4.(\ucd9c\ucc98:\u300a\ub300\ud55c\ubbfc\uad6d \ubcf5\uc9c0_7\uac00\uc9c0 \uc9c4\uc2e4\uacfc \uac70\uc9d3\u300b/\uc774\uc815\uc6b0 \uc678 \uc9c0\uc74c/\ub450\ub9ac\ubbf8\ub514\uc5b4 43-45\ucabd)"} {"question": "\uc120\uc655\uc744 \uc774\uc5b4 \ub098\ub77c\ub97c \uc798 \ud1b5\uce58\ud55c \uc655\uc5d0\uac8c \uc8fc\uc5b4\uc9c0\ub294 \uc885\ud638\ub294?", "paragraph": "\ubb18\ud638 \uc911 \uc885\ud638\ub97c \ubd99\uc77c \ub54c\uc5d0\ub294 \uc138 \uac00\uc9c0\uc758 \uc6d0\uce59\uc774 \uc788\ub294\ub370, \uac00\uc7a5 \uc911\uc694\ud55c \uc6d0\uce59\uc740 \ub098\ub77c\ub97c \uc138\uc6b4 \uc655(\u5275\u696d\u4e4b\u4e3b)\uc5d0\uac90 \u2018\uc870\u2019\ub97c, \uc120\uc655\uc758 \ub73b\uc744 \uacc4\uc2b9\ud558\uc5ec \ub098\ub77c\ub97c \uc798 \ub2e4\uc2a4\ub9b0 \uc655(\u7e7c\u9ad4\u4e4b\u4e3b)\uc5d0\uac90 \u2018\uc885\u2019\uc744 \ubd99\uc778\ub2e4. \uc774\uac83\uc740 \ubc14\ub85c \u201c\ubcc4\uc790\ub294 \u2018\uc870\u2019\uac00 \ub418\uace0 \ud6c4\uc608\ub294 \u2018\uc885\u2019\uc774 \ub41c\ub2e4\u201d\ub294 \uc6d0\uce59\uacfc \uc77c\ub9e5\uc0c1\ud1b5\ud55c\ub2e4. \ub4a4\uc774\uc5b4 \ud6c4\ub300\uc5d0 \uc774\ub974\ub7ec\uc11c \ubaa8\ub4e0 \uad70\uc8fc\ub4e4\uc5d0\uac8c \ubb18\ud638\ub97c \ubd99\uc774\uac8c \ub420 \ub54c\uc5d4 \u2018\uacf5\uc774 \uc788\uc73c\uba74 \uc870, \ub355\uc774 \uc788\uc73c\uba74 \uc885\u2019(\u6709\u529f\u66f0\u7956 \u6709\u5fb7\u66f0\u5b97), \uc989 \ub098\ub77c\ub97c \uad6c\ud55c \uacf5\uc774 \uc788\ub2e4\uba74 \u2018\uc870\u2019\ub97c, \ub098\ub77c\ub97c \ub355\uc73c\ub85c \ub2e4\uc2a4\ub9b0 \uc774\uc5d0\uac90 \u2018\uc885\u2019\uc744 \uc0ac\uc6a9\ud55c\ub2e4. \ub9c8\uc9c0\ub9c9\uc73c\ub85c \u2018\uc0c8\ub86d\uac8c \uc77c\uc5b4\ub098\uba74 \uc870, \uacc4\uc2b9\ud558\uba74 \uc885\u2019(\u5165\u627f\u66f0\u7956 \u7e7c\u627f\u66f0\u5b97)\uc774\ub77c\ub294 \uc6d0\uce59\uc744 \ub354\ud558\uc5ec \uc801\uc6a9\ud558\uc600\ub2e4. \uc774 \ub73b\uc740 \uc774\uc804\uc758 \uc655\ud1b5\uc774 \ub04a\uc5b4\uc838 \uc0c8\ub85c\uc6b4 \ubc29\uacc4 \ucd9c\uc2e0 \uc655\uc774 \uc989\uc704\ud558\uc600\ub2e4\uba74 \u2018\uc870\u2019\ub97c, \uc120\uc655\uc758 \uc790\ub9ac\ub97c \uacc4\uc2b9\ud574 \ub300\ub300\ub85c \uadf8 \uc655\uc704\ub97c \uc804\ud588\ub2e4\uba74 \u2018\uc885\u2019\uc73c\ub85c \ubaa8\uc168\ub2e4.", "answer": "\uc885", "sentence": "\ubb18\ud638 \uc911 \uc885 \ud638\ub97c \ubd99\uc77c \ub54c\uc5d0\ub294 \uc138 \uac00\uc9c0\uc758 \uc6d0\uce59\uc774 \uc788\ub294\ub370, \uac00\uc7a5 \uc911\uc694\ud55c \uc6d0\uce59\uc740 \ub098\ub77c\ub97c \uc138\uc6b4 \uc655(\u5275\u696d\u4e4b\u4e3b)\uc5d0\uac90 \u2018\uc870\u2019\ub97c, \uc120\uc655\uc758 \ub73b\uc744 \uacc4\uc2b9\ud558\uc5ec \ub098\ub77c\ub97c \uc798 \ub2e4\uc2a4\ub9b0 \uc655(\u7e7c\u9ad4\u4e4b\u4e3b)\uc5d0\uac90 \u2018\uc885\u2019\uc744 \ubd99\uc778\ub2e4.", "paragraph_sentence": " \ubb18\ud638 \uc911 \uc885 \ud638\ub97c \ubd99\uc77c \ub54c\uc5d0\ub294 \uc138 \uac00\uc9c0\uc758 \uc6d0\uce59\uc774 \uc788\ub294\ub370, \uac00\uc7a5 \uc911\uc694\ud55c \uc6d0\uce59\uc740 \ub098\ub77c\ub97c \uc138\uc6b4 \uc655(\u5275\u696d\u4e4b\u4e3b)\uc5d0\uac90 \u2018\uc870\u2019\ub97c, \uc120\uc655\uc758 \ub73b\uc744 \uacc4\uc2b9\ud558\uc5ec \ub098\ub77c\ub97c \uc798 \ub2e4\uc2a4\ub9b0 \uc655(\u7e7c\u9ad4\u4e4b\u4e3b)\uc5d0\uac90 \u2018\uc885\u2019\uc744 \ubd99\uc778\ub2e4. \uc774\uac83\uc740 \ubc14\ub85c \u201c\ubcc4\uc790\ub294 \u2018\uc870\u2019\uac00 \ub418\uace0 \ud6c4\uc608\ub294 \u2018\uc885\u2019\uc774 \ub41c\ub2e4\u201d\ub294 \uc6d0\uce59\uacfc \uc77c\ub9e5\uc0c1\ud1b5\ud55c\ub2e4. \ub4a4\uc774\uc5b4 \ud6c4\ub300\uc5d0 \uc774\ub974\ub7ec\uc11c \ubaa8\ub4e0 \uad70\uc8fc\ub4e4\uc5d0\uac8c \ubb18\ud638\ub97c \ubd99\uc774\uac8c \ub420 \ub54c\uc5d4 \u2018\uacf5\uc774 \uc788\uc73c\uba74 \uc870, \ub355\uc774 \uc788\uc73c\uba74 \uc885\u2019(\u6709\u529f\u66f0\u7956 \u6709\u5fb7\u66f0\u5b97), \uc989 \ub098\ub77c\ub97c \uad6c\ud55c \uacf5\uc774 \uc788\ub2e4\uba74 \u2018\uc870\u2019\ub97c, \ub098\ub77c\ub97c \ub355\uc73c\ub85c \ub2e4\uc2a4\ub9b0 \uc774\uc5d0\uac90 \u2018\uc885\u2019\uc744 \uc0ac\uc6a9\ud55c\ub2e4. \ub9c8\uc9c0\ub9c9\uc73c\ub85c \u2018\uc0c8\ub86d\uac8c \uc77c\uc5b4\ub098\uba74 \uc870, \uacc4\uc2b9\ud558\uba74 \uc885\u2019(\u5165\u627f\u66f0\u7956 \u7e7c\u627f\u66f0\u5b97)\uc774\ub77c\ub294 \uc6d0\uce59\uc744 \ub354\ud558\uc5ec \uc801\uc6a9\ud558\uc600\ub2e4. \uc774 \ub73b\uc740 \uc774\uc804\uc758 \uc655\ud1b5\uc774 \ub04a\uc5b4\uc838 \uc0c8\ub85c\uc6b4 \ubc29\uacc4 \ucd9c\uc2e0 \uc655\uc774 \uc989\uc704\ud558\uc600\ub2e4\uba74 \u2018\uc870\u2019\ub97c, \uc120\uc655\uc758 \uc790\ub9ac\ub97c \uacc4\uc2b9\ud574 \ub300\ub300\ub85c \uadf8 \uc655\uc704\ub97c \uc804\ud588\ub2e4\uba74 \u2018\uc885\u2019\uc73c\ub85c \ubaa8\uc168\ub2e4.", "paragraph_answer": "\ubb18\ud638 \uc911 \uc885 \ud638\ub97c \ubd99\uc77c \ub54c\uc5d0\ub294 \uc138 \uac00\uc9c0\uc758 \uc6d0\uce59\uc774 \uc788\ub294\ub370, \uac00\uc7a5 \uc911\uc694\ud55c \uc6d0\uce59\uc740 \ub098\ub77c\ub97c \uc138\uc6b4 \uc655(\u5275\u696d\u4e4b\u4e3b)\uc5d0\uac90 \u2018\uc870\u2019\ub97c, \uc120\uc655\uc758 \ub73b\uc744 \uacc4\uc2b9\ud558\uc5ec \ub098\ub77c\ub97c \uc798 \ub2e4\uc2a4\ub9b0 \uc655(\u7e7c\u9ad4\u4e4b\u4e3b)\uc5d0\uac90 \u2018\uc885\u2019\uc744 \ubd99\uc778\ub2e4. \uc774\uac83\uc740 \ubc14\ub85c \u201c\ubcc4\uc790\ub294 \u2018\uc870\u2019\uac00 \ub418\uace0 \ud6c4\uc608\ub294 \u2018\uc885\u2019\uc774 \ub41c\ub2e4\u201d\ub294 \uc6d0\uce59\uacfc \uc77c\ub9e5\uc0c1\ud1b5\ud55c\ub2e4. \ub4a4\uc774\uc5b4 \ud6c4\ub300\uc5d0 \uc774\ub974\ub7ec\uc11c \ubaa8\ub4e0 \uad70\uc8fc\ub4e4\uc5d0\uac8c \ubb18\ud638\ub97c \ubd99\uc774\uac8c \ub420 \ub54c\uc5d4 \u2018\uacf5\uc774 \uc788\uc73c\uba74 \uc870, \ub355\uc774 \uc788\uc73c\uba74 \uc885\u2019(\u6709\u529f\u66f0\u7956 \u6709\u5fb7\u66f0\u5b97), \uc989 \ub098\ub77c\ub97c \uad6c\ud55c \uacf5\uc774 \uc788\ub2e4\uba74 \u2018\uc870\u2019\ub97c, \ub098\ub77c\ub97c \ub355\uc73c\ub85c \ub2e4\uc2a4\ub9b0 \uc774\uc5d0\uac90 \u2018\uc885\u2019\uc744 \uc0ac\uc6a9\ud55c\ub2e4. \ub9c8\uc9c0\ub9c9\uc73c\ub85c \u2018\uc0c8\ub86d\uac8c \uc77c\uc5b4\ub098\uba74 \uc870, \uacc4\uc2b9\ud558\uba74 \uc885\u2019(\u5165\u627f\u66f0\u7956 \u7e7c\u627f\u66f0\u5b97)\uc774\ub77c\ub294 \uc6d0\uce59\uc744 \ub354\ud558\uc5ec \uc801\uc6a9\ud558\uc600\ub2e4. \uc774 \ub73b\uc740 \uc774\uc804\uc758 \uc655\ud1b5\uc774 \ub04a\uc5b4\uc838 \uc0c8\ub85c\uc6b4 \ubc29\uacc4 \ucd9c\uc2e0 \uc655\uc774 \uc989\uc704\ud558\uc600\ub2e4\uba74 \u2018\uc870\u2019\ub97c, \uc120\uc655\uc758 \uc790\ub9ac\ub97c \uacc4\uc2b9\ud574 \ub300\ub300\ub85c \uadf8 \uc655\uc704\ub97c \uc804\ud588\ub2e4\uba74 \u2018\uc885\u2019\uc73c\ub85c \ubaa8\uc168\ub2e4.", "sentence_answer": "\ubb18\ud638 \uc911 \uc885 \ud638\ub97c \ubd99\uc77c \ub54c\uc5d0\ub294 \uc138 \uac00\uc9c0\uc758 \uc6d0\uce59\uc774 \uc788\ub294\ub370, \uac00\uc7a5 \uc911\uc694\ud55c \uc6d0\uce59\uc740 \ub098\ub77c\ub97c \uc138\uc6b4 \uc655(\u5275\u696d\u4e4b\u4e3b)\uc5d0\uac90 \u2018\uc870\u2019\ub97c, \uc120\uc655\uc758 \ub73b\uc744 \uacc4\uc2b9\ud558\uc5ec \ub098\ub77c\ub97c \uc798 \ub2e4\uc2a4\ub9b0 \uc655(\u7e7c\u9ad4\u4e4b\u4e3b)\uc5d0\uac90 \u2018\uc885\u2019\uc744 \ubd99\uc778\ub2e4.", "paragraph_id": "6504691-2-0", "paragraph_question": "question: \uc120\uc655\uc744 \uc774\uc5b4 \ub098\ub77c\ub97c \uc798 \ud1b5\uce58\ud55c \uc655\uc5d0\uac8c \uc8fc\uc5b4\uc9c0\ub294 \uc885\ud638\ub294?, context: \ubb18\ud638 \uc911 \uc885\ud638\ub97c \ubd99\uc77c \ub54c\uc5d0\ub294 \uc138 \uac00\uc9c0\uc758 \uc6d0\uce59\uc774 \uc788\ub294\ub370, \uac00\uc7a5 \uc911\uc694\ud55c \uc6d0\uce59\uc740 \ub098\ub77c\ub97c \uc138\uc6b4 \uc655(\u5275\u696d\u4e4b\u4e3b)\uc5d0\uac90 \u2018\uc870\u2019\ub97c, \uc120\uc655\uc758 \ub73b\uc744 \uacc4\uc2b9\ud558\uc5ec \ub098\ub77c\ub97c \uc798 \ub2e4\uc2a4\ub9b0 \uc655(\u7e7c\u9ad4\u4e4b\u4e3b)\uc5d0\uac90 \u2018\uc885\u2019\uc744 \ubd99\uc778\ub2e4. \uc774\uac83\uc740 \ubc14\ub85c \u201c\ubcc4\uc790\ub294 \u2018\uc870\u2019\uac00 \ub418\uace0 \ud6c4\uc608\ub294 \u2018\uc885\u2019\uc774 \ub41c\ub2e4\u201d\ub294 \uc6d0\uce59\uacfc \uc77c\ub9e5\uc0c1\ud1b5\ud55c\ub2e4. \ub4a4\uc774\uc5b4 \ud6c4\ub300\uc5d0 \uc774\ub974\ub7ec\uc11c \ubaa8\ub4e0 \uad70\uc8fc\ub4e4\uc5d0\uac8c \ubb18\ud638\ub97c \ubd99\uc774\uac8c \ub420 \ub54c\uc5d4 \u2018\uacf5\uc774 \uc788\uc73c\uba74 \uc870, \ub355\uc774 \uc788\uc73c\uba74 \uc885\u2019(\u6709\u529f\u66f0\u7956 \u6709\u5fb7\u66f0\u5b97), \uc989 \ub098\ub77c\ub97c \uad6c\ud55c \uacf5\uc774 \uc788\ub2e4\uba74 \u2018\uc870\u2019\ub97c, \ub098\ub77c\ub97c \ub355\uc73c\ub85c \ub2e4\uc2a4\ub9b0 \uc774\uc5d0\uac90 \u2018\uc885\u2019\uc744 \uc0ac\uc6a9\ud55c\ub2e4. \ub9c8\uc9c0\ub9c9\uc73c\ub85c \u2018\uc0c8\ub86d\uac8c \uc77c\uc5b4\ub098\uba74 \uc870, \uacc4\uc2b9\ud558\uba74 \uc885\u2019(\u5165\u627f\u66f0\u7956 \u7e7c\u627f\u66f0\u5b97)\uc774\ub77c\ub294 \uc6d0\uce59\uc744 \ub354\ud558\uc5ec \uc801\uc6a9\ud558\uc600\ub2e4. \uc774 \ub73b\uc740 \uc774\uc804\uc758 \uc655\ud1b5\uc774 \ub04a\uc5b4\uc838 \uc0c8\ub85c\uc6b4 \ubc29\uacc4 \ucd9c\uc2e0 \uc655\uc774 \uc989\uc704\ud558\uc600\ub2e4\uba74 \u2018\uc870\u2019\ub97c, \uc120\uc655\uc758 \uc790\ub9ac\ub97c \uacc4\uc2b9\ud574 \ub300\ub300\ub85c \uadf8 \uc655\uc704\ub97c \uc804\ud588\ub2e4\uba74 \u2018\uc885\u2019\uc73c\ub85c \ubaa8\uc168\ub2e4."} {"question": "\uc77c\ubcf8 \uc774\uc2dc\uac00\ud0a4 \uc12c\uc758 \uc5f0\ud3c9\uade0 \uae30\uc628\uc740?", "paragraph": "\ub09c\uc138\uc774 \uc81c\ub3c4\ub97c \ud3ec\ud568\ud55c \uc77c\ubcf8 \uc5f4\ub3c4\ub294 \ub0a8\ubd81\uc73c\ub85c 5,000km\uc5d0 \uc774\ub974\uba70 \ub0c9\ub300 \uae30\ud6c4\uc5d0\uc11c \uc5f4\ub300 \uae30\ud6c4\uae4c\uc9c0 \uac78\uccd0\uc788\ub294 \ub370\ub2e4\uac00 \ubcf5\uc7a1\ud55c \uc9c0\ud615\uacfc \ud574\ub958\uc758 \uc601\ud5a5\uc774 \ud06c\uae30 \ub54c\ubb38\uc5d0 \uc9c0\uc5ed\ubcc4 \uae30\ud6c4\uc758 \ucc28\uc774\uac00 \ud604\uc800\ud558\ub2e4. \uc5f0\ud3c9\uade0 \uae30\uc628\uc740 \ub0a8\ucabd \uc624\ud0a4\ub098\uc640\ud604\uc758 \ub098\ud558 \uc2dc\uac00 \uc12d\uc528 22.7\ub3c4\ub97c \uae30\ub85d\ud558\uace0 \ub610 \uc57c\uc5d0\uc57c\ub9c8 \uc81c\ub3c4\uc758 \uc774\uc2dc\uac00\ud0a4 \uc12c\uc740 \uc12d\uc528 24\ub3c4\ub85c\uc11c \uc5f4\ub300 \uc6b0\ub9bc \uae30\ud6c4\uc5d0 \uc18d\ud55c\ub2e4, \uaddc\uc288 \uc9c0\ubc29\uc758 \uac00\uace0\uc2dc\ub9c8 \uc2dc\ub294 \uc12d\uc528 18.3\ub3c4, \uc624\uc0ac\uce74 \uc2dc\ub294 \uc12d\uc528 16.5\ub3c4, \ub3c4\ucfc4\ub3c4\uc758 \uad6c \uc9c0\uc5ed\uc740 \uc12d\uc528 15.9\ub3c4\uc774\ub2e4 \ubd81\ucabd \ud64b\uce74\uc774\ub3c4\uc758 \ub124\ubb34\ub85c \uc2dc\uac00 \uc12d\uc528 6.1\ub3c4\ub85c \ucd5c\uc800\ub97c \uae30\ub85d\ud55c\ub2e4. \uadf8\ub7ec\ub098 \ud64b\uce74\uc774\ub3c4\uc640 \uc624\ud0a4\ub098\uc640 \ud604\uc758 \uc57c\uc5d0\uc57c\ub9c8 \uc81c\ub3c4, \ub3c4\ucfc4\ub3c4\uc758 \uc624\ud0a4\ub178\ud1a0\ub9ac \uc12c, \ubbf8\ub098\ubbf8\ud1a0\ub9ac \uc12c\uc744 \uc81c\uc678\ud558\uba74 \ub300\ubd80\ubd84\uc758 \uc9c0\uc5ed\uc740 \uc628\ub300 \uae30\ud6c4\uc5d0 \uc18d\ud574 \uc628\ub09c\ud55c \uae30\ud6c4\uc774\uace0 \uc0ac\uacc4\uc808 \uad6c\ubd84\uc774 \ub69c\ub837\ud558\ub2e4. \ud0dc\ud3c9\uc591 \uc5f0\uc548 \uc9c0\ub300\ub294 \uc5ec\ub984\uc774\uba74 \ub0a8\ub3d9 \uacc4\uc808\ud48d\uc774 \ubd88\uc5b4 \ub9e4\uc6b0 \ubb34\ub365\uace0, \uaca8\uc6b8\uc5d0\ub294 \uac74\uc870\ud558\uace0 \ub9d1\uc740 \ub0a0\uc774 \ub9ce\uc544\uc9c4\ub2e4. \ud55c\ud3b8 \ub3d9\ud574 \ucabd\uc5d0\uc11c\ub294 \uaca8\uc6b8\uc5d0 \uc2dc\ubca0\ub9ac\uc544 \uacc4\uc808\ud48d\uc73c\ub85c \uc778\ud55c \uac15\uc124\uc774 \ub9ce\uc544 \ub2c8\uac00\ud0c0 \ud604\uc744 \ud3ec\ud568\ud55c \ud638\ucfe0\ub9ac\ucfe0 \uc9c0\ubc29\uc740 \uc138\uacc4\uc801\uc778 \ub2e4\uc124 \uc9c0\ub300\ub85c \uc190\uaf3d\ud600 4~5m\uc5d0 \uc774\ub974\ub294 \ub208\uc774 \uc313\uc774\ub294 \uacf3\ub3c4 \uc788\ub2e4. \ud64b\uce74\uc774\ub3c4\ub97c \uc81c\uc678\ud55c \uc9c0\uc5ed\uc5d0\uc11c\ub294 6\uc6d4 \ucd08\uc21c\ubd80\ud130 7\uc6d4 \uc911\uc21c\uc5d0 \uac78\uccd0 \uace0\uc628\ub2e4\uc2b5\ud55c \uc7a5\ub9c8\uac00 \ub0b4\ub9ac\uba70 8\uc6d4\ubd80\ud130 10\uc6d4\uc5d0 \uac78\uccd0 \uc77c\ubcf8 \uc5f4\ub3c4 \ub0a8\uc11c\ubd80 \uc9c0\uc5ed\uc740 \ud0dc\ud48d\uc758 \uc601\ud5a5\uc744 \uc2ec\ud558\uac8c \ubc1b\uc544 \ud48d\uc218\ud574\uc758 \ub300\ubd80\ubd84\uc744 \ucd08\ub798\ud55c\ub2e4.", "answer": "\uc12d\uc528 24\ub3c4", "sentence": "\uc5f0\ud3c9\uade0 \uae30\uc628\uc740 \ub0a8\ucabd \uc624\ud0a4\ub098\uc640\ud604\uc758 \ub098\ud558 \uc2dc\uac00 \uc12d\uc528 22.7\ub3c4\ub97c \uae30\ub85d\ud558\uace0 \ub610 \uc57c\uc5d0\uc57c\ub9c8 \uc81c\ub3c4\uc758 \uc774\uc2dc\uac00\ud0a4 \uc12c\uc740 \uc12d\uc528 24\ub3c4 \ub85c\uc11c \uc5f4\ub300 \uc6b0\ub9bc \uae30\ud6c4\uc5d0 \uc18d\ud55c\ub2e4, \uaddc\uc288 \uc9c0\ubc29\uc758 \uac00\uace0\uc2dc\ub9c8 \uc2dc\ub294 \uc12d\uc528 18.3\ub3c4, \uc624\uc0ac\uce74 \uc2dc\ub294 \uc12d\uc528 16.5\ub3c4, \ub3c4\ucfc4\ub3c4\uc758 \uad6c \uc9c0\uc5ed\uc740 \uc12d\uc528 15.9\ub3c4\uc774\ub2e4 \ubd81\ucabd \ud64b\uce74\uc774\ub3c4\uc758 \ub124\ubb34\ub85c \uc2dc\uac00 \uc12d\uc528 6.1\ub3c4\ub85c \ucd5c\uc800\ub97c \uae30\ub85d\ud55c\ub2e4.", "paragraph_sentence": "\ub09c\uc138\uc774 \uc81c\ub3c4\ub97c \ud3ec\ud568\ud55c \uc77c\ubcf8 \uc5f4\ub3c4\ub294 \ub0a8\ubd81\uc73c\ub85c 5,000km\uc5d0 \uc774\ub974\uba70 \ub0c9\ub300 \uae30\ud6c4\uc5d0\uc11c \uc5f4\ub300 \uae30\ud6c4\uae4c\uc9c0 \uac78\uccd0\uc788\ub294 \ub370\ub2e4\uac00 \ubcf5\uc7a1\ud55c \uc9c0\ud615\uacfc \ud574\ub958\uc758 \uc601\ud5a5\uc774 \ud06c\uae30 \ub54c\ubb38\uc5d0 \uc9c0\uc5ed\ubcc4 \uae30\ud6c4\uc758 \ucc28\uc774\uac00 \ud604\uc800\ud558\ub2e4. \uc5f0\ud3c9\uade0 \uae30\uc628\uc740 \ub0a8\ucabd \uc624\ud0a4\ub098\uc640\ud604\uc758 \ub098\ud558 \uc2dc\uac00 \uc12d\uc528 22.7\ub3c4\ub97c \uae30\ub85d\ud558\uace0 \ub610 \uc57c\uc5d0\uc57c\ub9c8 \uc81c\ub3c4\uc758 \uc774\uc2dc\uac00\ud0a4 \uc12c\uc740 \uc12d\uc528 24\ub3c4 \ub85c\uc11c \uc5f4\ub300 \uc6b0\ub9bc \uae30\ud6c4\uc5d0 \uc18d\ud55c\ub2e4, \uaddc\uc288 \uc9c0\ubc29\uc758 \uac00\uace0\uc2dc\ub9c8 \uc2dc\ub294 \uc12d\uc528 18.3\ub3c4, \uc624\uc0ac\uce74 \uc2dc\ub294 \uc12d\uc528 16.5\ub3c4, \ub3c4\ucfc4\ub3c4\uc758 \uad6c \uc9c0\uc5ed\uc740 \uc12d\uc528 15.9\ub3c4\uc774\ub2e4 \ubd81\ucabd \ud64b\uce74\uc774\ub3c4\uc758 \ub124\ubb34\ub85c \uc2dc\uac00 \uc12d\uc528 6.1\ub3c4\ub85c \ucd5c\uc800\ub97c \uae30\ub85d\ud55c\ub2e4. \uadf8\ub7ec\ub098 \ud64b\uce74\uc774\ub3c4\uc640 \uc624\ud0a4\ub098\uc640 \ud604\uc758 \uc57c\uc5d0\uc57c\ub9c8 \uc81c\ub3c4, \ub3c4\ucfc4\ub3c4\uc758 \uc624\ud0a4\ub178\ud1a0\ub9ac \uc12c, \ubbf8\ub098\ubbf8\ud1a0\ub9ac \uc12c\uc744 \uc81c\uc678\ud558\uba74 \ub300\ubd80\ubd84\uc758 \uc9c0\uc5ed\uc740 \uc628\ub300 \uae30\ud6c4\uc5d0 \uc18d\ud574 \uc628\ub09c\ud55c \uae30\ud6c4\uc774\uace0 \uc0ac\uacc4\uc808 \uad6c\ubd84\uc774 \ub69c\ub837\ud558\ub2e4. \ud0dc\ud3c9\uc591 \uc5f0\uc548 \uc9c0\ub300\ub294 \uc5ec\ub984\uc774\uba74 \ub0a8\ub3d9 \uacc4\uc808\ud48d\uc774 \ubd88\uc5b4 \ub9e4\uc6b0 \ubb34\ub365\uace0, \uaca8\uc6b8\uc5d0\ub294 \uac74\uc870\ud558\uace0 \ub9d1\uc740 \ub0a0\uc774 \ub9ce\uc544\uc9c4\ub2e4. \ud55c\ud3b8 \ub3d9\ud574 \ucabd\uc5d0\uc11c\ub294 \uaca8\uc6b8\uc5d0 \uc2dc\ubca0\ub9ac\uc544 \uacc4\uc808\ud48d\uc73c\ub85c \uc778\ud55c \uac15\uc124\uc774 \ub9ce\uc544 \ub2c8\uac00\ud0c0 \ud604\uc744 \ud3ec\ud568\ud55c \ud638\ucfe0\ub9ac\ucfe0 \uc9c0\ubc29\uc740 \uc138\uacc4\uc801\uc778 \ub2e4\uc124 \uc9c0\ub300\ub85c \uc190\uaf3d\ud600 4~5m\uc5d0 \uc774\ub974\ub294 \ub208\uc774 \uc313\uc774\ub294 \uacf3\ub3c4 \uc788\ub2e4. \ud64b\uce74\uc774\ub3c4\ub97c \uc81c\uc678\ud55c \uc9c0\uc5ed\uc5d0\uc11c\ub294 6\uc6d4 \ucd08\uc21c\ubd80\ud130 7\uc6d4 \uc911\uc21c\uc5d0 \uac78\uccd0 \uace0\uc628\ub2e4\uc2b5\ud55c \uc7a5\ub9c8\uac00 \ub0b4\ub9ac\uba70 8\uc6d4\ubd80\ud130 10\uc6d4\uc5d0 \uac78\uccd0 \uc77c\ubcf8 \uc5f4\ub3c4 \ub0a8\uc11c\ubd80 \uc9c0\uc5ed\uc740 \ud0dc\ud48d\uc758 \uc601\ud5a5\uc744 \uc2ec\ud558\uac8c \ubc1b\uc544 \ud48d\uc218\ud574\uc758 \ub300\ubd80\ubd84\uc744 \ucd08\ub798\ud55c\ub2e4.", "paragraph_answer": "\ub09c\uc138\uc774 \uc81c\ub3c4\ub97c \ud3ec\ud568\ud55c \uc77c\ubcf8 \uc5f4\ub3c4\ub294 \ub0a8\ubd81\uc73c\ub85c 5,000km\uc5d0 \uc774\ub974\uba70 \ub0c9\ub300 \uae30\ud6c4\uc5d0\uc11c \uc5f4\ub300 \uae30\ud6c4\uae4c\uc9c0 \uac78\uccd0\uc788\ub294 \ub370\ub2e4\uac00 \ubcf5\uc7a1\ud55c \uc9c0\ud615\uacfc \ud574\ub958\uc758 \uc601\ud5a5\uc774 \ud06c\uae30 \ub54c\ubb38\uc5d0 \uc9c0\uc5ed\ubcc4 \uae30\ud6c4\uc758 \ucc28\uc774\uac00 \ud604\uc800\ud558\ub2e4. \uc5f0\ud3c9\uade0 \uae30\uc628\uc740 \ub0a8\ucabd \uc624\ud0a4\ub098\uc640\ud604\uc758 \ub098\ud558 \uc2dc\uac00 \uc12d\uc528 22.7\ub3c4\ub97c \uae30\ub85d\ud558\uace0 \ub610 \uc57c\uc5d0\uc57c\ub9c8 \uc81c\ub3c4\uc758 \uc774\uc2dc\uac00\ud0a4 \uc12c\uc740 \uc12d\uc528 24\ub3c4 \ub85c\uc11c \uc5f4\ub300 \uc6b0\ub9bc \uae30\ud6c4\uc5d0 \uc18d\ud55c\ub2e4, \uaddc\uc288 \uc9c0\ubc29\uc758 \uac00\uace0\uc2dc\ub9c8 \uc2dc\ub294 \uc12d\uc528 18.3\ub3c4, \uc624\uc0ac\uce74 \uc2dc\ub294 \uc12d\uc528 16.5\ub3c4, \ub3c4\ucfc4\ub3c4\uc758 \uad6c \uc9c0\uc5ed\uc740 \uc12d\uc528 15.9\ub3c4\uc774\ub2e4 \ubd81\ucabd \ud64b\uce74\uc774\ub3c4\uc758 \ub124\ubb34\ub85c \uc2dc\uac00 \uc12d\uc528 6.1\ub3c4\ub85c \ucd5c\uc800\ub97c \uae30\ub85d\ud55c\ub2e4. \uadf8\ub7ec\ub098 \ud64b\uce74\uc774\ub3c4\uc640 \uc624\ud0a4\ub098\uc640 \ud604\uc758 \uc57c\uc5d0\uc57c\ub9c8 \uc81c\ub3c4, \ub3c4\ucfc4\ub3c4\uc758 \uc624\ud0a4\ub178\ud1a0\ub9ac \uc12c, \ubbf8\ub098\ubbf8\ud1a0\ub9ac \uc12c\uc744 \uc81c\uc678\ud558\uba74 \ub300\ubd80\ubd84\uc758 \uc9c0\uc5ed\uc740 \uc628\ub300 \uae30\ud6c4\uc5d0 \uc18d\ud574 \uc628\ub09c\ud55c \uae30\ud6c4\uc774\uace0 \uc0ac\uacc4\uc808 \uad6c\ubd84\uc774 \ub69c\ub837\ud558\ub2e4. \ud0dc\ud3c9\uc591 \uc5f0\uc548 \uc9c0\ub300\ub294 \uc5ec\ub984\uc774\uba74 \ub0a8\ub3d9 \uacc4\uc808\ud48d\uc774 \ubd88\uc5b4 \ub9e4\uc6b0 \ubb34\ub365\uace0, \uaca8\uc6b8\uc5d0\ub294 \uac74\uc870\ud558\uace0 \ub9d1\uc740 \ub0a0\uc774 \ub9ce\uc544\uc9c4\ub2e4. \ud55c\ud3b8 \ub3d9\ud574 \ucabd\uc5d0\uc11c\ub294 \uaca8\uc6b8\uc5d0 \uc2dc\ubca0\ub9ac\uc544 \uacc4\uc808\ud48d\uc73c\ub85c \uc778\ud55c \uac15\uc124\uc774 \ub9ce\uc544 \ub2c8\uac00\ud0c0 \ud604\uc744 \ud3ec\ud568\ud55c \ud638\ucfe0\ub9ac\ucfe0 \uc9c0\ubc29\uc740 \uc138\uacc4\uc801\uc778 \ub2e4\uc124 \uc9c0\ub300\ub85c \uc190\uaf3d\ud600 4~5m\uc5d0 \uc774\ub974\ub294 \ub208\uc774 \uc313\uc774\ub294 \uacf3\ub3c4 \uc788\ub2e4. \ud64b\uce74\uc774\ub3c4\ub97c \uc81c\uc678\ud55c \uc9c0\uc5ed\uc5d0\uc11c\ub294 6\uc6d4 \ucd08\uc21c\ubd80\ud130 7\uc6d4 \uc911\uc21c\uc5d0 \uac78\uccd0 \uace0\uc628\ub2e4\uc2b5\ud55c \uc7a5\ub9c8\uac00 \ub0b4\ub9ac\uba70 8\uc6d4\ubd80\ud130 10\uc6d4\uc5d0 \uac78\uccd0 \uc77c\ubcf8 \uc5f4\ub3c4 \ub0a8\uc11c\ubd80 \uc9c0\uc5ed\uc740 \ud0dc\ud48d\uc758 \uc601\ud5a5\uc744 \uc2ec\ud558\uac8c \ubc1b\uc544 \ud48d\uc218\ud574\uc758 \ub300\ubd80\ubd84\uc744 \ucd08\ub798\ud55c\ub2e4.", "sentence_answer": "\uc5f0\ud3c9\uade0 \uae30\uc628\uc740 \ub0a8\ucabd \uc624\ud0a4\ub098\uc640\ud604\uc758 \ub098\ud558 \uc2dc\uac00 \uc12d\uc528 22.7\ub3c4\ub97c \uae30\ub85d\ud558\uace0 \ub610 \uc57c\uc5d0\uc57c\ub9c8 \uc81c\ub3c4\uc758 \uc774\uc2dc\uac00\ud0a4 \uc12c\uc740 \uc12d\uc528 24\ub3c4 \ub85c\uc11c \uc5f4\ub300 \uc6b0\ub9bc \uae30\ud6c4\uc5d0 \uc18d\ud55c\ub2e4, \uaddc\uc288 \uc9c0\ubc29\uc758 \uac00\uace0\uc2dc\ub9c8 \uc2dc\ub294 \uc12d\uc528 18.3\ub3c4, \uc624\uc0ac\uce74 \uc2dc\ub294 \uc12d\uc528 16.5\ub3c4, \ub3c4\ucfc4\ub3c4\uc758 \uad6c \uc9c0\uc5ed\uc740 \uc12d\uc528 15.9\ub3c4\uc774\ub2e4 \ubd81\ucabd \ud64b\uce74\uc774\ub3c4\uc758 \ub124\ubb34\ub85c \uc2dc\uac00 \uc12d\uc528 6.1\ub3c4\ub85c \ucd5c\uc800\ub97c \uae30\ub85d\ud55c\ub2e4.", "paragraph_id": "6657580-12-0", "paragraph_question": "question: \uc77c\ubcf8 \uc774\uc2dc\uac00\ud0a4 \uc12c\uc758 \uc5f0\ud3c9\uade0 \uae30\uc628\uc740?, context: \ub09c\uc138\uc774 \uc81c\ub3c4\ub97c \ud3ec\ud568\ud55c \uc77c\ubcf8 \uc5f4\ub3c4\ub294 \ub0a8\ubd81\uc73c\ub85c 5,000km\uc5d0 \uc774\ub974\uba70 \ub0c9\ub300 \uae30\ud6c4\uc5d0\uc11c \uc5f4\ub300 \uae30\ud6c4\uae4c\uc9c0 \uac78\uccd0\uc788\ub294 \ub370\ub2e4\uac00 \ubcf5\uc7a1\ud55c \uc9c0\ud615\uacfc \ud574\ub958\uc758 \uc601\ud5a5\uc774 \ud06c\uae30 \ub54c\ubb38\uc5d0 \uc9c0\uc5ed\ubcc4 \uae30\ud6c4\uc758 \ucc28\uc774\uac00 \ud604\uc800\ud558\ub2e4. \uc5f0\ud3c9\uade0 \uae30\uc628\uc740 \ub0a8\ucabd \uc624\ud0a4\ub098\uc640\ud604\uc758 \ub098\ud558 \uc2dc\uac00 \uc12d\uc528 22.7\ub3c4\ub97c \uae30\ub85d\ud558\uace0 \ub610 \uc57c\uc5d0\uc57c\ub9c8 \uc81c\ub3c4\uc758 \uc774\uc2dc\uac00\ud0a4 \uc12c\uc740 \uc12d\uc528 24\ub3c4\ub85c\uc11c \uc5f4\ub300 \uc6b0\ub9bc \uae30\ud6c4\uc5d0 \uc18d\ud55c\ub2e4, \uaddc\uc288 \uc9c0\ubc29\uc758 \uac00\uace0\uc2dc\ub9c8 \uc2dc\ub294 \uc12d\uc528 18.3\ub3c4, \uc624\uc0ac\uce74 \uc2dc\ub294 \uc12d\uc528 16.5\ub3c4, \ub3c4\ucfc4\ub3c4\uc758 \uad6c \uc9c0\uc5ed\uc740 \uc12d\uc528 15.9\ub3c4\uc774\ub2e4 \ubd81\ucabd \ud64b\uce74\uc774\ub3c4\uc758 \ub124\ubb34\ub85c \uc2dc\uac00 \uc12d\uc528 6.1\ub3c4\ub85c \ucd5c\uc800\ub97c \uae30\ub85d\ud55c\ub2e4. \uadf8\ub7ec\ub098 \ud64b\uce74\uc774\ub3c4\uc640 \uc624\ud0a4\ub098\uc640 \ud604\uc758 \uc57c\uc5d0\uc57c\ub9c8 \uc81c\ub3c4, \ub3c4\ucfc4\ub3c4\uc758 \uc624\ud0a4\ub178\ud1a0\ub9ac \uc12c, \ubbf8\ub098\ubbf8\ud1a0\ub9ac \uc12c\uc744 \uc81c\uc678\ud558\uba74 \ub300\ubd80\ubd84\uc758 \uc9c0\uc5ed\uc740 \uc628\ub300 \uae30\ud6c4\uc5d0 \uc18d\ud574 \uc628\ub09c\ud55c \uae30\ud6c4\uc774\uace0 \uc0ac\uacc4\uc808 \uad6c\ubd84\uc774 \ub69c\ub837\ud558\ub2e4. \ud0dc\ud3c9\uc591 \uc5f0\uc548 \uc9c0\ub300\ub294 \uc5ec\ub984\uc774\uba74 \ub0a8\ub3d9 \uacc4\uc808\ud48d\uc774 \ubd88\uc5b4 \ub9e4\uc6b0 \ubb34\ub365\uace0, \uaca8\uc6b8\uc5d0\ub294 \uac74\uc870\ud558\uace0 \ub9d1\uc740 \ub0a0\uc774 \ub9ce\uc544\uc9c4\ub2e4. \ud55c\ud3b8 \ub3d9\ud574 \ucabd\uc5d0\uc11c\ub294 \uaca8\uc6b8\uc5d0 \uc2dc\ubca0\ub9ac\uc544 \uacc4\uc808\ud48d\uc73c\ub85c \uc778\ud55c \uac15\uc124\uc774 \ub9ce\uc544 \ub2c8\uac00\ud0c0 \ud604\uc744 \ud3ec\ud568\ud55c \ud638\ucfe0\ub9ac\ucfe0 \uc9c0\ubc29\uc740 \uc138\uacc4\uc801\uc778 \ub2e4\uc124 \uc9c0\ub300\ub85c \uc190\uaf3d\ud600 4~5m\uc5d0 \uc774\ub974\ub294 \ub208\uc774 \uc313\uc774\ub294 \uacf3\ub3c4 \uc788\ub2e4. \ud64b\uce74\uc774\ub3c4\ub97c \uc81c\uc678\ud55c \uc9c0\uc5ed\uc5d0\uc11c\ub294 6\uc6d4 \ucd08\uc21c\ubd80\ud130 7\uc6d4 \uc911\uc21c\uc5d0 \uac78\uccd0 \uace0\uc628\ub2e4\uc2b5\ud55c \uc7a5\ub9c8\uac00 \ub0b4\ub9ac\uba70 8\uc6d4\ubd80\ud130 10\uc6d4\uc5d0 \uac78\uccd0 \uc77c\ubcf8 \uc5f4\ub3c4 \ub0a8\uc11c\ubd80 \uc9c0\uc5ed\uc740 \ud0dc\ud48d\uc758 \uc601\ud5a5\uc744 \uc2ec\ud558\uac8c \ubc1b\uc544 \ud48d\uc218\ud574\uc758 \ub300\ubd80\ubd84\uc744 \ucd08\ub798\ud55c\ub2e4."} {"question": "\uc544\ud504\ub9ac\uce74\uc758 \uc774\uc2ac\ub78c\uad50\ub3c4\uc758 \uc808\ubc18\uc774 \uac70\uc8fc\ud558\uace0 \uc788\ub294 \uacf3\uc740?", "paragraph": "\uc774\uc2ac\ub78c\uad50\ub294 7\uc138\uae30\uc5d0 \uc544\ub77c\ube44\uc544\ubc18\ub3c4\uc5d0\uc11c \uc544\ud504\ub9ac\uce74\ub85c \uc804\ud574\uc838 \uc9c0\uc911\ud574 \uc5f0\uc548\uc9c0\uc5ed\uc73c\ub85c \ud37c\uc84c\ub2e4. \uc774\uc2ac\ub78c\uad50\uac00 \uc0ac\ud558\ub77c\uc0ac\ub9c9\uc744 \ub118\uc5b4 \ube14\ub799\uc544\ud504\ub9ac\uce74\uae4c\uc9c0 \ud37c\uc9c0\uae30 \uc2dc\uc791\ud55c \uac83\uc740 11\uc138\uae30 \ubb34\ub835\uc73c\ub85c, \ud3c9\ud654\uc801\uc778 \ud3ec\uad50\ud65c\ub3d9\uacfc \uc9c0\ud558\ub4dc(\u8056\u6230), \uadf8\ub9ac\uace0 \ubd81\uc544\ud504\ub9ac\uce74\uc640 \ube14\ub799\uc544\ud504\ub9ac\uce74\uc640\uc758 \uad50\uc5ed\ud65c\ub3d9\uc5d0 \ud798\uc785\uc5b4 \ub110\ub9ac \uc804\ud30c\ub418\uc5c8\ub2e4. \uc774\uc2ac\ub78c\uad50\uc758 \ube14\ub799\uc544\ud504\ub9ac\uce74\ub85c\uc758 \ud655\ub300\uacfc\uc815\uc740 19\uc138\uae30 \ub9d0\uc5d0 \uc2dc\uc791\ub41c \uc5f4\uac15\uc758 \uc544\ud504\ub9ac\uce74 \uc2dd\ubbfc\uc9c0\ubd84\ud560\uae30\uc5d0\ub3c4 \uacc4\uc18d\ub418\uc5c8\ub2e4. \uc624\ub298\ub0a0 \uc544\ud504\ub9ac\uce74\uc758 \uc774\uc2ac\ub78c\uad8c\uc740 \ubd81\ubd80\uc640 \uc11c\ubd80 \uc9c0\uc5ed\uc744 \uc911\uc2ec\uc73c\ub85c \ud558\uba70, \ub3d9\ubd80\uc5d0\uc11c\ub294 \uc218\ub2e8\u00b7\uc18c\ub9d0\ub9ac\uc544\u00b7\uc5d0\ud2f0\uc624\ud53c\uc544, \uadf8\ub9ac\uace0 \ucf00\ub0d0\uc5d0\uc11c \ud0c4\uc790\ub2c8\uc544\ub97c \uac70\uccd0 \ubaa8\uc7a0\ube44\ud06c \ubd81\ubd80\uc5d0 \uc774\ub974\ub294 \uc5f0\uc548\ubd80\uc5d0\ub3c4 \ub760\ubaa8\uc591\uc73c\ub85c \uc774\uc2ac\ub78c\uad8c\uc774 \ud615\uc131\ub418\uc5b4 \uc788\ub2e4. \uc911\uc559\uc544\ud504\ub9ac\uce74\uc5d0\ub3c4 \ucc28\ub4dc(\uc778\uad6c\uc758 50%)\uc640 \uce74\uba54\ub8ec(20%) \ub4f1\uc758 \uad6d\uac00\uc5d0 \ub9ce\uc740 \uc774\uc2ac\ub78c\uad50\ub3c4\uac00 \uc788\ub2e4. \uc544\ud504\ub9ac\uce74\uc758 \uc774\uc2ac\ub78c\uad50\ub3c4\uc758 \ucd1d\uc218\ub294 1981\ub144 1\uc5b5 4,574\ub9cc \uba85\uc73c\ub85c, \uadf8 \uac00\uc6b4\ub370 \ubc18\uc218\uac00 \ubd81\uc544\ud504\ub9ac\uce74, 1/4\uc774 \uc11c\uc544\ud504\ub9ac\uce74, \ub098\uba38\uc9c0 1/4\uc774 \ub3d9\ubd80 \ubc0f \uc911\uc559\uc544\ud504\ub9ac\uce74\uc5d0 \uac01\uac01 \uac70\uc8fc\ud558\uace0 \uc788\ub2e4.", "answer": "\ubd81\uc544\ud504\ub9ac\uce74", "sentence": "\uc774\uc2ac\ub78c\uad50\uac00 \uc0ac\ud558\ub77c\uc0ac\ub9c9\uc744 \ub118\uc5b4 \ube14\ub799\uc544\ud504\ub9ac\uce74\uae4c\uc9c0 \ud37c\uc9c0\uae30 \uc2dc\uc791\ud55c \uac83\uc740 11\uc138\uae30 \ubb34\ub835\uc73c\ub85c, \ud3c9\ud654\uc801\uc778 \ud3ec\uad50\ud65c\ub3d9\uacfc \uc9c0\ud558\ub4dc(\u8056\u6230), \uadf8\ub9ac\uace0 \ubd81\uc544\ud504\ub9ac\uce74 \uc640 \ube14\ub799\uc544\ud504\ub9ac\uce74\uc640\uc758 \uad50\uc5ed\ud65c\ub3d9\uc5d0 \ud798\uc785\uc5b4 \ub110\ub9ac \uc804\ud30c\ub418\uc5c8\ub2e4.", "paragraph_sentence": "\uc774\uc2ac\ub78c\uad50\ub294 7\uc138\uae30\uc5d0 \uc544\ub77c\ube44\uc544\ubc18\ub3c4\uc5d0\uc11c \uc544\ud504\ub9ac\uce74\ub85c \uc804\ud574\uc838 \uc9c0\uc911\ud574 \uc5f0\uc548\uc9c0\uc5ed\uc73c\ub85c \ud37c\uc84c\ub2e4. \uc774\uc2ac\ub78c\uad50\uac00 \uc0ac\ud558\ub77c\uc0ac\ub9c9\uc744 \ub118\uc5b4 \ube14\ub799\uc544\ud504\ub9ac\uce74\uae4c\uc9c0 \ud37c\uc9c0\uae30 \uc2dc\uc791\ud55c \uac83\uc740 11\uc138\uae30 \ubb34\ub835\uc73c\ub85c, \ud3c9\ud654\uc801\uc778 \ud3ec\uad50\ud65c\ub3d9\uacfc \uc9c0\ud558\ub4dc(\u8056\u6230), \uadf8\ub9ac\uace0 \ubd81\uc544\ud504\ub9ac\uce74 \uc640 \ube14\ub799\uc544\ud504\ub9ac\uce74\uc640\uc758 \uad50\uc5ed\ud65c\ub3d9\uc5d0 \ud798\uc785\uc5b4 \ub110\ub9ac \uc804\ud30c\ub418\uc5c8\ub2e4. \uc774\uc2ac\ub78c\uad50\uc758 \ube14\ub799\uc544\ud504\ub9ac\uce74\ub85c\uc758 \ud655\ub300\uacfc\uc815\uc740 19\uc138\uae30 \ub9d0\uc5d0 \uc2dc\uc791\ub41c \uc5f4\uac15\uc758 \uc544\ud504\ub9ac\uce74 \uc2dd\ubbfc\uc9c0\ubd84\ud560\uae30\uc5d0\ub3c4 \uacc4\uc18d\ub418\uc5c8\ub2e4. \uc624\ub298\ub0a0 \uc544\ud504\ub9ac\uce74\uc758 \uc774\uc2ac\ub78c\uad8c\uc740 \ubd81\ubd80\uc640 \uc11c\ubd80 \uc9c0\uc5ed\uc744 \uc911\uc2ec\uc73c\ub85c \ud558\uba70, \ub3d9\ubd80\uc5d0\uc11c\ub294 \uc218\ub2e8\u00b7\uc18c\ub9d0\ub9ac\uc544\u00b7\uc5d0\ud2f0\uc624\ud53c\uc544, \uadf8\ub9ac\uace0 \ucf00\ub0d0\uc5d0\uc11c \ud0c4\uc790\ub2c8\uc544\ub97c \uac70\uccd0 \ubaa8\uc7a0\ube44\ud06c \ubd81\ubd80\uc5d0 \uc774\ub974\ub294 \uc5f0\uc548\ubd80\uc5d0\ub3c4 \ub760\ubaa8\uc591\uc73c\ub85c \uc774\uc2ac\ub78c\uad8c\uc774 \ud615\uc131\ub418\uc5b4 \uc788\ub2e4. \uc911\uc559\uc544\ud504\ub9ac\uce74\uc5d0\ub3c4 \ucc28\ub4dc(\uc778\uad6c\uc758 50%)\uc640 \uce74\uba54\ub8ec(20%) \ub4f1\uc758 \uad6d\uac00\uc5d0 \ub9ce\uc740 \uc774\uc2ac\ub78c\uad50\ub3c4\uac00 \uc788\ub2e4. \uc544\ud504\ub9ac\uce74\uc758 \uc774\uc2ac\ub78c\uad50\ub3c4\uc758 \ucd1d\uc218\ub294 1981\ub144 1\uc5b5 4,574\ub9cc \uba85\uc73c\ub85c, \uadf8 \uac00\uc6b4\ub370 \ubc18\uc218\uac00 \ubd81\uc544\ud504\ub9ac\uce74, 1/4\uc774 \uc11c\uc544\ud504\ub9ac\uce74, \ub098\uba38\uc9c0 1/4\uc774 \ub3d9\ubd80 \ubc0f \uc911\uc559\uc544\ud504\ub9ac\uce74\uc5d0 \uac01\uac01 \uac70\uc8fc\ud558\uace0 \uc788\ub2e4.", "paragraph_answer": "\uc774\uc2ac\ub78c\uad50\ub294 7\uc138\uae30\uc5d0 \uc544\ub77c\ube44\uc544\ubc18\ub3c4\uc5d0\uc11c \uc544\ud504\ub9ac\uce74\ub85c \uc804\ud574\uc838 \uc9c0\uc911\ud574 \uc5f0\uc548\uc9c0\uc5ed\uc73c\ub85c \ud37c\uc84c\ub2e4. \uc774\uc2ac\ub78c\uad50\uac00 \uc0ac\ud558\ub77c\uc0ac\ub9c9\uc744 \ub118\uc5b4 \ube14\ub799\uc544\ud504\ub9ac\uce74\uae4c\uc9c0 \ud37c\uc9c0\uae30 \uc2dc\uc791\ud55c \uac83\uc740 11\uc138\uae30 \ubb34\ub835\uc73c\ub85c, \ud3c9\ud654\uc801\uc778 \ud3ec\uad50\ud65c\ub3d9\uacfc \uc9c0\ud558\ub4dc(\u8056\u6230), \uadf8\ub9ac\uace0 \ubd81\uc544\ud504\ub9ac\uce74 \uc640 \ube14\ub799\uc544\ud504\ub9ac\uce74\uc640\uc758 \uad50\uc5ed\ud65c\ub3d9\uc5d0 \ud798\uc785\uc5b4 \ub110\ub9ac \uc804\ud30c\ub418\uc5c8\ub2e4. \uc774\uc2ac\ub78c\uad50\uc758 \ube14\ub799\uc544\ud504\ub9ac\uce74\ub85c\uc758 \ud655\ub300\uacfc\uc815\uc740 19\uc138\uae30 \ub9d0\uc5d0 \uc2dc\uc791\ub41c \uc5f4\uac15\uc758 \uc544\ud504\ub9ac\uce74 \uc2dd\ubbfc\uc9c0\ubd84\ud560\uae30\uc5d0\ub3c4 \uacc4\uc18d\ub418\uc5c8\ub2e4. \uc624\ub298\ub0a0 \uc544\ud504\ub9ac\uce74\uc758 \uc774\uc2ac\ub78c\uad8c\uc740 \ubd81\ubd80\uc640 \uc11c\ubd80 \uc9c0\uc5ed\uc744 \uc911\uc2ec\uc73c\ub85c \ud558\uba70, \ub3d9\ubd80\uc5d0\uc11c\ub294 \uc218\ub2e8\u00b7\uc18c\ub9d0\ub9ac\uc544\u00b7\uc5d0\ud2f0\uc624\ud53c\uc544, \uadf8\ub9ac\uace0 \ucf00\ub0d0\uc5d0\uc11c \ud0c4\uc790\ub2c8\uc544\ub97c \uac70\uccd0 \ubaa8\uc7a0\ube44\ud06c \ubd81\ubd80\uc5d0 \uc774\ub974\ub294 \uc5f0\uc548\ubd80\uc5d0\ub3c4 \ub760\ubaa8\uc591\uc73c\ub85c \uc774\uc2ac\ub78c\uad8c\uc774 \ud615\uc131\ub418\uc5b4 \uc788\ub2e4. \uc911\uc559\uc544\ud504\ub9ac\uce74\uc5d0\ub3c4 \ucc28\ub4dc(\uc778\uad6c\uc758 50%)\uc640 \uce74\uba54\ub8ec(20%) \ub4f1\uc758 \uad6d\uac00\uc5d0 \ub9ce\uc740 \uc774\uc2ac\ub78c\uad50\ub3c4\uac00 \uc788\ub2e4. \uc544\ud504\ub9ac\uce74\uc758 \uc774\uc2ac\ub78c\uad50\ub3c4\uc758 \ucd1d\uc218\ub294 1981\ub144 1\uc5b5 4,574\ub9cc \uba85\uc73c\ub85c, \uadf8 \uac00\uc6b4\ub370 \ubc18\uc218\uac00 \ubd81\uc544\ud504\ub9ac\uce74, 1/4\uc774 \uc11c\uc544\ud504\ub9ac\uce74, \ub098\uba38\uc9c0 1/4\uc774 \ub3d9\ubd80 \ubc0f \uc911\uc559\uc544\ud504\ub9ac\uce74\uc5d0 \uac01\uac01 \uac70\uc8fc\ud558\uace0 \uc788\ub2e4.", "sentence_answer": "\uc774\uc2ac\ub78c\uad50\uac00 \uc0ac\ud558\ub77c\uc0ac\ub9c9\uc744 \ub118\uc5b4 \ube14\ub799\uc544\ud504\ub9ac\uce74\uae4c\uc9c0 \ud37c\uc9c0\uae30 \uc2dc\uc791\ud55c \uac83\uc740 11\uc138\uae30 \ubb34\ub835\uc73c\ub85c, \ud3c9\ud654\uc801\uc778 \ud3ec\uad50\ud65c\ub3d9\uacfc \uc9c0\ud558\ub4dc(\u8056\u6230), \uadf8\ub9ac\uace0 \ubd81\uc544\ud504\ub9ac\uce74 \uc640 \ube14\ub799\uc544\ud504\ub9ac\uce74\uc640\uc758 \uad50\uc5ed\ud65c\ub3d9\uc5d0 \ud798\uc785\uc5b4 \ub110\ub9ac \uc804\ud30c\ub418\uc5c8\ub2e4.", "paragraph_id": "6538805-6-2", "paragraph_question": "question: \uc544\ud504\ub9ac\uce74\uc758 \uc774\uc2ac\ub78c\uad50\ub3c4\uc758 \uc808\ubc18\uc774 \uac70\uc8fc\ud558\uace0 \uc788\ub294 \uacf3\uc740?, context: \uc774\uc2ac\ub78c\uad50\ub294 7\uc138\uae30\uc5d0 \uc544\ub77c\ube44\uc544\ubc18\ub3c4\uc5d0\uc11c \uc544\ud504\ub9ac\uce74\ub85c \uc804\ud574\uc838 \uc9c0\uc911\ud574 \uc5f0\uc548\uc9c0\uc5ed\uc73c\ub85c \ud37c\uc84c\ub2e4. \uc774\uc2ac\ub78c\uad50\uac00 \uc0ac\ud558\ub77c\uc0ac\ub9c9\uc744 \ub118\uc5b4 \ube14\ub799\uc544\ud504\ub9ac\uce74\uae4c\uc9c0 \ud37c\uc9c0\uae30 \uc2dc\uc791\ud55c \uac83\uc740 11\uc138\uae30 \ubb34\ub835\uc73c\ub85c, \ud3c9\ud654\uc801\uc778 \ud3ec\uad50\ud65c\ub3d9\uacfc \uc9c0\ud558\ub4dc(\u8056\u6230), \uadf8\ub9ac\uace0 \ubd81\uc544\ud504\ub9ac\uce74\uc640 \ube14\ub799\uc544\ud504\ub9ac\uce74\uc640\uc758 \uad50\uc5ed\ud65c\ub3d9\uc5d0 \ud798\uc785\uc5b4 \ub110\ub9ac \uc804\ud30c\ub418\uc5c8\ub2e4. \uc774\uc2ac\ub78c\uad50\uc758 \ube14\ub799\uc544\ud504\ub9ac\uce74\ub85c\uc758 \ud655\ub300\uacfc\uc815\uc740 19\uc138\uae30 \ub9d0\uc5d0 \uc2dc\uc791\ub41c \uc5f4\uac15\uc758 \uc544\ud504\ub9ac\uce74 \uc2dd\ubbfc\uc9c0\ubd84\ud560\uae30\uc5d0\ub3c4 \uacc4\uc18d\ub418\uc5c8\ub2e4. \uc624\ub298\ub0a0 \uc544\ud504\ub9ac\uce74\uc758 \uc774\uc2ac\ub78c\uad8c\uc740 \ubd81\ubd80\uc640 \uc11c\ubd80 \uc9c0\uc5ed\uc744 \uc911\uc2ec\uc73c\ub85c \ud558\uba70, \ub3d9\ubd80\uc5d0\uc11c\ub294 \uc218\ub2e8\u00b7\uc18c\ub9d0\ub9ac\uc544\u00b7\uc5d0\ud2f0\uc624\ud53c\uc544, \uadf8\ub9ac\uace0 \ucf00\ub0d0\uc5d0\uc11c \ud0c4\uc790\ub2c8\uc544\ub97c \uac70\uccd0 \ubaa8\uc7a0\ube44\ud06c \ubd81\ubd80\uc5d0 \uc774\ub974\ub294 \uc5f0\uc548\ubd80\uc5d0\ub3c4 \ub760\ubaa8\uc591\uc73c\ub85c \uc774\uc2ac\ub78c\uad8c\uc774 \ud615\uc131\ub418\uc5b4 \uc788\ub2e4. \uc911\uc559\uc544\ud504\ub9ac\uce74\uc5d0\ub3c4 \ucc28\ub4dc(\uc778\uad6c\uc758 50%)\uc640 \uce74\uba54\ub8ec(20%) \ub4f1\uc758 \uad6d\uac00\uc5d0 \ub9ce\uc740 \uc774\uc2ac\ub78c\uad50\ub3c4\uac00 \uc788\ub2e4. \uc544\ud504\ub9ac\uce74\uc758 \uc774\uc2ac\ub78c\uad50\ub3c4\uc758 \ucd1d\uc218\ub294 1981\ub144 1\uc5b5 4,574\ub9cc \uba85\uc73c\ub85c, \uadf8 \uac00\uc6b4\ub370 \ubc18\uc218\uac00 \ubd81\uc544\ud504\ub9ac\uce74, 1/4\uc774 \uc11c\uc544\ud504\ub9ac\uce74, \ub098\uba38\uc9c0 1/4\uc774 \ub3d9\ubd80 \ubc0f \uc911\uc559\uc544\ud504\ub9ac\uce74\uc5d0 \uac01\uac01 \uac70\uc8fc\ud558\uace0 \uc788\ub2e4."} {"question": "\uc138\uc778\ud2b8 \uc870\uc9c0 \uc5ed\uacfc \ubca0\uc774 \uc5ed\uc5d0\uc11c\uc758 \uc120\ub85c \uad6c\uc870 \ubb38\uc81c\ub294 \uc2b9\uac15\uc7a5\uc758 \ubb34\uc5c7 \ub54c\ubb38\uc778\uac00?", "paragraph": "\ubca0\uc774 \uc5ed\uc5d0\uc11c\ub294 \uc5d0\uae00\ub9b0\ud134 \uc5ed\uc5d0\uc11c \uc6b0\ub4dc\ubc14\uc778 \uc5ed\uc73c\ub85c \uac00\ub294 \uc5f4\ucc28\uac00 \uc544\ub7ab\uce35\uc5d0 \ub3c4\ucc29\ud588\ub294\ub370, \ud0ac \uc5ed\uc5d0\uc11c \uc6b0\ub4dc\ubc14\uc778\uc73c\ub85c \uac00\ub294 \uc5f4\ucc28\ub294 \uc717\uce35\uc5d0 \ub3c4\ucc29\ud588\ub2e4. \uc5f4\ucc28\uac00 \uc774\ub807\uac8c \ubc88\uac08\uc544 \ub3c4\ucc29\ud558\uc790, \uc774 \uc5ed\uc744 \uc774\uc6a9\ud558\ub294 \uc2b9\uac1d\ub4e4\uc740 \uc5b4\ub514\uc11c \uc5f4\ucc28\ub97c \ud0c0\uc57c\ud560 \uc9c0 \uc54c\uc9c0 \ubabb\ud588\ub2e4. \uac19\uc740 \ubb38\uc81c\ub294 \uc138\uc778\ud2b8 \uc870\uc9c0 \uc5ed\uc5d0\uc11c\ub3c4 \ubc1c\uc0dd\ud588\ub294\ub370, \uc5d0\uae00\ub9b0\ud134 \uc5ed\uc5d0\uc11c \ud0ac \uc5ed\uc73c\ub85c \uac00\ub294 \uc5f4\ucc28\ub294 \uc138\uc778\ud2b8 \uc870\uc9c0 \uc5ed \uc717\uce35 \uc2b9\uac15\uc7a5\uc5d0 \ub3c4\ucc29\ud558\uace0 \uc6b0\ub4dc\ubc14\uc778 \uc5ed\uc5d0\uc11c \ud0ac \uc5ed\uc73c\ub85c \uac00\ub294 \uc5f4\ucc28\ub294 \uc138\uc778\ud2b8 \uc870\uc9c0 \uc5ed \uc544\ub7ab \uce35\uc5d0 \ub3c4\ucc29\ud574(\ubca0\uc774 \uc5ed\uacfc \uc5f4\ucc28\uac00 \ub3c4\ucc29\ud558\ub294 \uce35\uc774 \ub2e4\ub974\ub2e4) \uc2b9\uac1d\ub4e4\uc758 \ud63c\ub780\ub9cc \uac00\uc911\ub418\uc5c8\ub2e4. \ud55c\ud3b8 \uc5d0\uae00\ub9b0\ud134 \uc5ed\uc73c\ub85c \uac00\ub294 \uc5f4\ucc28\ub294 \uc120\ub85c \uad6c\uc870\uc0c1 \ub450 \uc5ed \ub2e4 \uc544\ub7ab\uce35\uc5d0 \ub3c4\ucc29\ud558\uace0, \uc804 \uc5ed\uc778 \ubba4\uc9c0\uc5c4 \uc5ed\uc740 \ud55c \uce35\uc5d0 \ub450 \ubc29\uba74\uc758 \uc5f4\ucc28\uac00 \ub4e4\uc5b4\uc624\uae30 \ub54c\ubb38\uc5d0 \uc774\ub7f0 \ubb38\uc81c\uac00 \uc5c6\uc5c8\ub2e4. \uc138\uc778\ud2b8 \uc870\uc9c0 \uc5ed\uacfc \ubca0\uc774 \uc5ed\uc5d0\uc11c\uc758 \uc120\ub85c \uad6c\uc870 \ubb38\uc81c\ub294 \ub458 \ub2e4 \uc2b9\uac15\uc7a5\uc774 \ubcf5\uce35\uc774\uae30 \ub54c\ubb38\uc774\ub2e4. (\uc138\uc778\ud2b8 \uc870\uc9c0 \uc5ed\uc5d0\uc11c\ub294 \ud0ac \uc5ed\uc73c\ub85c \uac00\ub294 \uc5f4\ucc28\uac00, \ubca0\uc774 \uc5ed\uc5d0\uc11c\ub294 \uc6b0\ub4dc\ubc14\uc778 \uc5ed\uc73c\ub85c \uac00\ub294 \uc5f4\ucc28\uac00 \uc717\uce35\uacfc \uc544\ub7ab\uce35\uc5d0 \ub2e4 \uc654\ub2e4.) \uadf8 \uc0c1\ud669\uc5d0\uc11c \uc591 \uc5f4\ucc28\uac00 \ud55c \uce35, \ud55c \uc2b9\uac15\uc7a5\uc5d0 \ub3c4\ucc29\ud558\ub294 \uac83\uc740 \ubd88\uac00\ub2a5\ud588\ub294\ub370, \uc774\ub294 \uc720\ub2c8\ubc84\uc2dc\ud2f0 \uc120\uc5d0\uc11c \uac19\uc740 \uce35\uc5d0 \uc788\ub294 \uc591 \uc120\ub85c\uac00 \uac19\uc740 \ubc29\ud5a5\uc73c\ub85c \uac14\uae30 \ub54c\ubb38\uc774\ub2e4.", "answer": "\ubcf5\uce35", "sentence": "\uc138\uc778\ud2b8 \uc870\uc9c0 \uc5ed\uacfc \ubca0\uc774 \uc5ed\uc5d0\uc11c\uc758 \uc120\ub85c \uad6c\uc870 \ubb38\uc81c\ub294 \ub458 \ub2e4 \uc2b9\uac15\uc7a5\uc774 \ubcf5\uce35 \uc774\uae30 \ub54c\ubb38\uc774\ub2e4.", "paragraph_sentence": "\ubca0\uc774 \uc5ed\uc5d0\uc11c\ub294 \uc5d0\uae00\ub9b0\ud134 \uc5ed\uc5d0\uc11c \uc6b0\ub4dc\ubc14\uc778 \uc5ed\uc73c\ub85c \uac00\ub294 \uc5f4\ucc28\uac00 \uc544\ub7ab\uce35\uc5d0 \ub3c4\ucc29\ud588\ub294\ub370, \ud0ac \uc5ed\uc5d0\uc11c \uc6b0\ub4dc\ubc14\uc778\uc73c\ub85c \uac00\ub294 \uc5f4\ucc28\ub294 \uc717\uce35\uc5d0 \ub3c4\ucc29\ud588\ub2e4. \uc5f4\ucc28\uac00 \uc774\ub807\uac8c \ubc88\uac08\uc544 \ub3c4\ucc29\ud558\uc790, \uc774 \uc5ed\uc744 \uc774\uc6a9\ud558\ub294 \uc2b9\uac1d\ub4e4\uc740 \uc5b4\ub514\uc11c \uc5f4\ucc28\ub97c \ud0c0\uc57c\ud560 \uc9c0 \uc54c\uc9c0 \ubabb\ud588\ub2e4. \uac19\uc740 \ubb38\uc81c\ub294 \uc138\uc778\ud2b8 \uc870\uc9c0 \uc5ed\uc5d0\uc11c\ub3c4 \ubc1c\uc0dd\ud588\ub294\ub370, \uc5d0\uae00\ub9b0\ud134 \uc5ed\uc5d0\uc11c \ud0ac \uc5ed\uc73c\ub85c \uac00\ub294 \uc5f4\ucc28\ub294 \uc138\uc778\ud2b8 \uc870\uc9c0 \uc5ed \uc717\uce35 \uc2b9\uac15\uc7a5\uc5d0 \ub3c4\ucc29\ud558\uace0 \uc6b0\ub4dc\ubc14\uc778 \uc5ed\uc5d0\uc11c \ud0ac \uc5ed\uc73c\ub85c \uac00\ub294 \uc5f4\ucc28\ub294 \uc138\uc778\ud2b8 \uc870\uc9c0 \uc5ed \uc544\ub7ab \uce35\uc5d0 \ub3c4\ucc29\ud574(\ubca0\uc774 \uc5ed\uacfc \uc5f4\ucc28\uac00 \ub3c4\ucc29\ud558\ub294 \uce35\uc774 \ub2e4\ub974\ub2e4) \uc2b9\uac1d\ub4e4\uc758 \ud63c\ub780\ub9cc \uac00\uc911\ub418\uc5c8\ub2e4. \ud55c\ud3b8 \uc5d0\uae00\ub9b0\ud134 \uc5ed\uc73c\ub85c \uac00\ub294 \uc5f4\ucc28\ub294 \uc120\ub85c \uad6c\uc870\uc0c1 \ub450 \uc5ed \ub2e4 \uc544\ub7ab\uce35\uc5d0 \ub3c4\ucc29\ud558\uace0, \uc804 \uc5ed\uc778 \ubba4\uc9c0\uc5c4 \uc5ed\uc740 \ud55c \uce35\uc5d0 \ub450 \ubc29\uba74\uc758 \uc5f4\ucc28\uac00 \ub4e4\uc5b4\uc624\uae30 \ub54c\ubb38\uc5d0 \uc774\ub7f0 \ubb38\uc81c\uac00 \uc5c6\uc5c8\ub2e4. \uc138\uc778\ud2b8 \uc870\uc9c0 \uc5ed\uacfc \ubca0\uc774 \uc5ed\uc5d0\uc11c\uc758 \uc120\ub85c \uad6c\uc870 \ubb38\uc81c\ub294 \ub458 \ub2e4 \uc2b9\uac15\uc7a5\uc774 \ubcf5\uce35 \uc774\uae30 \ub54c\ubb38\uc774\ub2e4. (\uc138\uc778\ud2b8 \uc870\uc9c0 \uc5ed\uc5d0\uc11c\ub294 \ud0ac \uc5ed\uc73c\ub85c \uac00\ub294 \uc5f4\ucc28\uac00, \ubca0\uc774 \uc5ed\uc5d0\uc11c\ub294 \uc6b0\ub4dc\ubc14\uc778 \uc5ed\uc73c\ub85c \uac00\ub294 \uc5f4\ucc28\uac00 \uc717\uce35\uacfc \uc544\ub7ab\uce35\uc5d0 \ub2e4 \uc654\ub2e4.) \uadf8 \uc0c1\ud669\uc5d0\uc11c \uc591 \uc5f4\ucc28\uac00 \ud55c \uce35, \ud55c \uc2b9\uac15\uc7a5\uc5d0 \ub3c4\ucc29\ud558\ub294 \uac83\uc740 \ubd88\uac00\ub2a5\ud588\ub294\ub370, \uc774\ub294 \uc720\ub2c8\ubc84\uc2dc\ud2f0 \uc120\uc5d0\uc11c \uac19\uc740 \uce35\uc5d0 \uc788\ub294 \uc591 \uc120\ub85c\uac00 \uac19\uc740 \ubc29\ud5a5\uc73c\ub85c \uac14\uae30 \ub54c\ubb38\uc774\ub2e4.", "paragraph_answer": "\ubca0\uc774 \uc5ed\uc5d0\uc11c\ub294 \uc5d0\uae00\ub9b0\ud134 \uc5ed\uc5d0\uc11c \uc6b0\ub4dc\ubc14\uc778 \uc5ed\uc73c\ub85c \uac00\ub294 \uc5f4\ucc28\uac00 \uc544\ub7ab\uce35\uc5d0 \ub3c4\ucc29\ud588\ub294\ub370, \ud0ac \uc5ed\uc5d0\uc11c \uc6b0\ub4dc\ubc14\uc778\uc73c\ub85c \uac00\ub294 \uc5f4\ucc28\ub294 \uc717\uce35\uc5d0 \ub3c4\ucc29\ud588\ub2e4. \uc5f4\ucc28\uac00 \uc774\ub807\uac8c \ubc88\uac08\uc544 \ub3c4\ucc29\ud558\uc790, \uc774 \uc5ed\uc744 \uc774\uc6a9\ud558\ub294 \uc2b9\uac1d\ub4e4\uc740 \uc5b4\ub514\uc11c \uc5f4\ucc28\ub97c \ud0c0\uc57c\ud560 \uc9c0 \uc54c\uc9c0 \ubabb\ud588\ub2e4. \uac19\uc740 \ubb38\uc81c\ub294 \uc138\uc778\ud2b8 \uc870\uc9c0 \uc5ed\uc5d0\uc11c\ub3c4 \ubc1c\uc0dd\ud588\ub294\ub370, \uc5d0\uae00\ub9b0\ud134 \uc5ed\uc5d0\uc11c \ud0ac \uc5ed\uc73c\ub85c \uac00\ub294 \uc5f4\ucc28\ub294 \uc138\uc778\ud2b8 \uc870\uc9c0 \uc5ed \uc717\uce35 \uc2b9\uac15\uc7a5\uc5d0 \ub3c4\ucc29\ud558\uace0 \uc6b0\ub4dc\ubc14\uc778 \uc5ed\uc5d0\uc11c \ud0ac \uc5ed\uc73c\ub85c \uac00\ub294 \uc5f4\ucc28\ub294 \uc138\uc778\ud2b8 \uc870\uc9c0 \uc5ed \uc544\ub7ab \uce35\uc5d0 \ub3c4\ucc29\ud574(\ubca0\uc774 \uc5ed\uacfc \uc5f4\ucc28\uac00 \ub3c4\ucc29\ud558\ub294 \uce35\uc774 \ub2e4\ub974\ub2e4) \uc2b9\uac1d\ub4e4\uc758 \ud63c\ub780\ub9cc \uac00\uc911\ub418\uc5c8\ub2e4. \ud55c\ud3b8 \uc5d0\uae00\ub9b0\ud134 \uc5ed\uc73c\ub85c \uac00\ub294 \uc5f4\ucc28\ub294 \uc120\ub85c \uad6c\uc870\uc0c1 \ub450 \uc5ed \ub2e4 \uc544\ub7ab\uce35\uc5d0 \ub3c4\ucc29\ud558\uace0, \uc804 \uc5ed\uc778 \ubba4\uc9c0\uc5c4 \uc5ed\uc740 \ud55c \uce35\uc5d0 \ub450 \ubc29\uba74\uc758 \uc5f4\ucc28\uac00 \ub4e4\uc5b4\uc624\uae30 \ub54c\ubb38\uc5d0 \uc774\ub7f0 \ubb38\uc81c\uac00 \uc5c6\uc5c8\ub2e4. \uc138\uc778\ud2b8 \uc870\uc9c0 \uc5ed\uacfc \ubca0\uc774 \uc5ed\uc5d0\uc11c\uc758 \uc120\ub85c \uad6c\uc870 \ubb38\uc81c\ub294 \ub458 \ub2e4 \uc2b9\uac15\uc7a5\uc774 \ubcf5\uce35 \uc774\uae30 \ub54c\ubb38\uc774\ub2e4. (\uc138\uc778\ud2b8 \uc870\uc9c0 \uc5ed\uc5d0\uc11c\ub294 \ud0ac \uc5ed\uc73c\ub85c \uac00\ub294 \uc5f4\ucc28\uac00, \ubca0\uc774 \uc5ed\uc5d0\uc11c\ub294 \uc6b0\ub4dc\ubc14\uc778 \uc5ed\uc73c\ub85c \uac00\ub294 \uc5f4\ucc28\uac00 \uc717\uce35\uacfc \uc544\ub7ab\uce35\uc5d0 \ub2e4 \uc654\ub2e4.) \uadf8 \uc0c1\ud669\uc5d0\uc11c \uc591 \uc5f4\ucc28\uac00 \ud55c \uce35, \ud55c \uc2b9\uac15\uc7a5\uc5d0 \ub3c4\ucc29\ud558\ub294 \uac83\uc740 \ubd88\uac00\ub2a5\ud588\ub294\ub370, \uc774\ub294 \uc720\ub2c8\ubc84\uc2dc\ud2f0 \uc120\uc5d0\uc11c \uac19\uc740 \uce35\uc5d0 \uc788\ub294 \uc591 \uc120\ub85c\uac00 \uac19\uc740 \ubc29\ud5a5\uc73c\ub85c \uac14\uae30 \ub54c\ubb38\uc774\ub2e4.", "sentence_answer": "\uc138\uc778\ud2b8 \uc870\uc9c0 \uc5ed\uacfc \ubca0\uc774 \uc5ed\uc5d0\uc11c\uc758 \uc120\ub85c \uad6c\uc870 \ubb38\uc81c\ub294 \ub458 \ub2e4 \uc2b9\uac15\uc7a5\uc774 \ubcf5\uce35 \uc774\uae30 \ub54c\ubb38\uc774\ub2e4.", "paragraph_id": "6566081-7-1", "paragraph_question": "question: \uc138\uc778\ud2b8 \uc870\uc9c0 \uc5ed\uacfc \ubca0\uc774 \uc5ed\uc5d0\uc11c\uc758 \uc120\ub85c \uad6c\uc870 \ubb38\uc81c\ub294 \uc2b9\uac15\uc7a5\uc758 \ubb34\uc5c7 \ub54c\ubb38\uc778\uac00?, context: \ubca0\uc774 \uc5ed\uc5d0\uc11c\ub294 \uc5d0\uae00\ub9b0\ud134 \uc5ed\uc5d0\uc11c \uc6b0\ub4dc\ubc14\uc778 \uc5ed\uc73c\ub85c \uac00\ub294 \uc5f4\ucc28\uac00 \uc544\ub7ab\uce35\uc5d0 \ub3c4\ucc29\ud588\ub294\ub370, \ud0ac \uc5ed\uc5d0\uc11c \uc6b0\ub4dc\ubc14\uc778\uc73c\ub85c \uac00\ub294 \uc5f4\ucc28\ub294 \uc717\uce35\uc5d0 \ub3c4\ucc29\ud588\ub2e4. \uc5f4\ucc28\uac00 \uc774\ub807\uac8c \ubc88\uac08\uc544 \ub3c4\ucc29\ud558\uc790, \uc774 \uc5ed\uc744 \uc774\uc6a9\ud558\ub294 \uc2b9\uac1d\ub4e4\uc740 \uc5b4\ub514\uc11c \uc5f4\ucc28\ub97c \ud0c0\uc57c\ud560 \uc9c0 \uc54c\uc9c0 \ubabb\ud588\ub2e4. \uac19\uc740 \ubb38\uc81c\ub294 \uc138\uc778\ud2b8 \uc870\uc9c0 \uc5ed\uc5d0\uc11c\ub3c4 \ubc1c\uc0dd\ud588\ub294\ub370, \uc5d0\uae00\ub9b0\ud134 \uc5ed\uc5d0\uc11c \ud0ac \uc5ed\uc73c\ub85c \uac00\ub294 \uc5f4\ucc28\ub294 \uc138\uc778\ud2b8 \uc870\uc9c0 \uc5ed \uc717\uce35 \uc2b9\uac15\uc7a5\uc5d0 \ub3c4\ucc29\ud558\uace0 \uc6b0\ub4dc\ubc14\uc778 \uc5ed\uc5d0\uc11c \ud0ac \uc5ed\uc73c\ub85c \uac00\ub294 \uc5f4\ucc28\ub294 \uc138\uc778\ud2b8 \uc870\uc9c0 \uc5ed \uc544\ub7ab \uce35\uc5d0 \ub3c4\ucc29\ud574(\ubca0\uc774 \uc5ed\uacfc \uc5f4\ucc28\uac00 \ub3c4\ucc29\ud558\ub294 \uce35\uc774 \ub2e4\ub974\ub2e4) \uc2b9\uac1d\ub4e4\uc758 \ud63c\ub780\ub9cc \uac00\uc911\ub418\uc5c8\ub2e4. \ud55c\ud3b8 \uc5d0\uae00\ub9b0\ud134 \uc5ed\uc73c\ub85c \uac00\ub294 \uc5f4\ucc28\ub294 \uc120\ub85c \uad6c\uc870\uc0c1 \ub450 \uc5ed \ub2e4 \uc544\ub7ab\uce35\uc5d0 \ub3c4\ucc29\ud558\uace0, \uc804 \uc5ed\uc778 \ubba4\uc9c0\uc5c4 \uc5ed\uc740 \ud55c \uce35\uc5d0 \ub450 \ubc29\uba74\uc758 \uc5f4\ucc28\uac00 \ub4e4\uc5b4\uc624\uae30 \ub54c\ubb38\uc5d0 \uc774\ub7f0 \ubb38\uc81c\uac00 \uc5c6\uc5c8\ub2e4. \uc138\uc778\ud2b8 \uc870\uc9c0 \uc5ed\uacfc \ubca0\uc774 \uc5ed\uc5d0\uc11c\uc758 \uc120\ub85c \uad6c\uc870 \ubb38\uc81c\ub294 \ub458 \ub2e4 \uc2b9\uac15\uc7a5\uc774 \ubcf5\uce35\uc774\uae30 \ub54c\ubb38\uc774\ub2e4. (\uc138\uc778\ud2b8 \uc870\uc9c0 \uc5ed\uc5d0\uc11c\ub294 \ud0ac \uc5ed\uc73c\ub85c \uac00\ub294 \uc5f4\ucc28\uac00, \ubca0\uc774 \uc5ed\uc5d0\uc11c\ub294 \uc6b0\ub4dc\ubc14\uc778 \uc5ed\uc73c\ub85c \uac00\ub294 \uc5f4\ucc28\uac00 \uc717\uce35\uacfc \uc544\ub7ab\uce35\uc5d0 \ub2e4 \uc654\ub2e4.) \uadf8 \uc0c1\ud669\uc5d0\uc11c \uc591 \uc5f4\ucc28\uac00 \ud55c \uce35, \ud55c \uc2b9\uac15\uc7a5\uc5d0 \ub3c4\ucc29\ud558\ub294 \uac83\uc740 \ubd88\uac00\ub2a5\ud588\ub294\ub370, \uc774\ub294 \uc720\ub2c8\ubc84\uc2dc\ud2f0 \uc120\uc5d0\uc11c \uac19\uc740 \uce35\uc5d0 \uc788\ub294 \uc591 \uc120\ub85c\uac00 \uac19\uc740 \ubc29\ud5a5\uc73c\ub85c \uac14\uae30 \ub54c\ubb38\uc774\ub2e4."} {"question": "2015\ub144 \uae40\uc544\ub791\uc774 \uac1c\uc778\uc804 \uae08\uba54\ub2ec 2\uac1c\ub97c \ud68d\ub4dd\ud55c \ub3d9\uacc4 \uc720\ub2c8\ubc84\uc2dc\uc544\ub4dc \uac1c\ucd5c \ub098\ub77c\ub294?", "paragraph": "\uae40\uc544\ub791\uc740 2014-2015 \uc2dc\uc98c \uad6d\uac00\ub300\ud45c \uc120\ubc1c\uc804\uc5d0\uc11c \uc5b4\uae68 \ubd80\uc0c1\uc744 \uc785\uc5c8\ub294\ub370 \uc774\uac83\uc740 \uc218\uc220\uc744 \ud544\uc694\ub85c \ud558\ub294 \ud68c\uc804\uadfc\uac1c(Rotator cuff) \uc190\uc0c1\uc774\uc5c8\uc73c\uba70 2015\ub144 \uad6d\uac00\ub300\ud45c \uc120\ubc1c\uc804\uc774 \ub05d\ub09c \ud6c4 \uc218\uc220\uc744 \ud558\uc600\ub2e4. 2014-2015 \uc2dc\uc98c\uc5d0\uc11c \uae40\uc544\ub791\uc740 5\ucc28\ub840 ISU \uc1fc\ud2b8\ud2b8\ub799 \uc6d4\ub4dc\ucef5\uc5d0 \ucd9c\uc804\ud558\uc5ec \uae08\uba54\ub2ec 3\uac1c \uc740\uba54\ub2ec 4\uac1c \ub3d9\uba54\ub2ec 1\uac1c\ub97c \ud68d\ub4dd\ud558\uc600\uc73c\uba70, 2015\ub144 2\uc6d4 \uc2a4\ud398\uc778 \uadf8\ub77c\ub098\ub2e4\uc5d0\uc11c \uc5f4\ub9b0 \ub3d9\uacc4 \uc720\ub2c8\ubc84\uc2dc\uc544\ub4dc \uc1fc\ud2b8\ud2b8\ub799 1000\ubbf8\ud130\uc640 1500\ubbf8\ud130\uc5d0\uc11c \uc6b0\uc2b9\ud558\uc5ec \uac1c\uc778\uc804 \uae08\uba54\ub2ec 2\uac1c\ub97c \ubaa9\uc5d0 \uac78\uc5c8\ub2e4. \ub2f9\uc2dc \uae40\uc544\ub791\uc740 \"\uae08\uba54\ub2ec 2\uac1c\ub97c \ucc28\uc9c0\ud55c \uac83\uc740 \uae30\uc058\uc9c0\ub9cc 3000\ubbf8\ud130 \uacc4\uc8fc\uc5d0\uc11c \uc911\uad6d\uc5d0\uac8c \uc11d\ud328\ud55c \uac83\uc774 \uc544\uc27d\ub2e4. \ub0b4\uac00 3\uad00\uc655\uc744 \ubabb\ud574\uc11c \uc544\uc26c\uc6b4 \uac83\uc774 \uc544\ub2c8\ub77c, \ud300 \uc804\uccb4\uac00 \ubaa8\ub450 \uae08\uba54\ub2ec\uc744 \ud568\uaed8 \ud560 \uc218 \uc788\ub294 \uae30\ud68c\ub97c \ub193\ucce4\uae30 \ub54c\ubb38\"\uc774\ub77c\uace0 \uae30\uc068\ubcf4\ub2e4\ub294 \uc544\uc26c\uc6c0\uc744 \ud45c\ud604\ud558\uc600\ub2e4.", "answer": "\uc2a4\ud398\uc778", "sentence": "2015\ub144 2\uc6d4 \uc2a4\ud398\uc778 \uadf8\ub77c\ub098\ub2e4\uc5d0\uc11c \uc5f4\ub9b0 \ub3d9\uacc4 \uc720\ub2c8\ubc84\uc2dc\uc544\ub4dc \uc1fc\ud2b8\ud2b8\ub799 1000\ubbf8\ud130\uc640 1500\ubbf8\ud130\uc5d0\uc11c \uc6b0\uc2b9\ud558\uc5ec \uac1c\uc778\uc804 \uae08\uba54\ub2ec 2\uac1c\ub97c \ubaa9\uc5d0 \uac78\uc5c8\ub2e4.", "paragraph_sentence": "\uae40\uc544\ub791\uc740 2014-2015 \uc2dc\uc98c \uad6d\uac00\ub300\ud45c \uc120\ubc1c\uc804\uc5d0\uc11c \uc5b4\uae68 \ubd80\uc0c1\uc744 \uc785\uc5c8\ub294\ub370 \uc774\uac83\uc740 \uc218\uc220\uc744 \ud544\uc694\ub85c \ud558\ub294 \ud68c\uc804\uadfc\uac1c(Rotator cuff) \uc190\uc0c1\uc774\uc5c8\uc73c\uba70 2015\ub144 \uad6d\uac00\ub300\ud45c \uc120\ubc1c\uc804\uc774 \ub05d\ub09c \ud6c4 \uc218\uc220\uc744 \ud558\uc600\ub2e4. 2014-2015 \uc2dc\uc98c\uc5d0\uc11c \uae40\uc544\ub791\uc740 5\ucc28\ub840 ISU \uc1fc\ud2b8\ud2b8\ub799 \uc6d4\ub4dc\ucef5\uc5d0 \ucd9c\uc804\ud558\uc5ec \uae08\uba54\ub2ec 3\uac1c \uc740\uba54\ub2ec 4\uac1c \ub3d9\uba54\ub2ec 1\uac1c\ub97c \ud68d\ub4dd\ud558\uc600\uc73c\uba70, 2015\ub144 2\uc6d4 \uc2a4\ud398\uc778 \uadf8\ub77c\ub098\ub2e4\uc5d0\uc11c \uc5f4\ub9b0 \ub3d9\uacc4 \uc720\ub2c8\ubc84\uc2dc\uc544\ub4dc \uc1fc\ud2b8\ud2b8\ub799 1000\ubbf8\ud130\uc640 1500\ubbf8\ud130\uc5d0\uc11c \uc6b0\uc2b9\ud558\uc5ec \uac1c\uc778\uc804 \uae08\uba54\ub2ec 2\uac1c\ub97c \ubaa9\uc5d0 \uac78\uc5c8\ub2e4. \ub2f9\uc2dc \uae40\uc544\ub791\uc740 \"\uae08\uba54\ub2ec 2\uac1c\ub97c \ucc28\uc9c0\ud55c \uac83\uc740 \uae30\uc058\uc9c0\ub9cc 3000\ubbf8\ud130 \uacc4\uc8fc\uc5d0\uc11c \uc911\uad6d\uc5d0\uac8c \uc11d\ud328\ud55c \uac83\uc774 \uc544\uc27d\ub2e4. \ub0b4\uac00 3\uad00\uc655\uc744 \ubabb\ud574\uc11c \uc544\uc26c\uc6b4 \uac83\uc774 \uc544\ub2c8\ub77c, \ud300 \uc804\uccb4\uac00 \ubaa8\ub450 \uae08\uba54\ub2ec\uc744 \ud568\uaed8 \ud560 \uc218 \uc788\ub294 \uae30\ud68c\ub97c \ub193\ucce4\uae30 \ub54c\ubb38\"\uc774\ub77c\uace0 \uae30\uc068\ubcf4\ub2e4\ub294 \uc544\uc26c\uc6c0\uc744 \ud45c\ud604\ud558\uc600\ub2e4.", "paragraph_answer": "\uae40\uc544\ub791\uc740 2014-2015 \uc2dc\uc98c \uad6d\uac00\ub300\ud45c \uc120\ubc1c\uc804\uc5d0\uc11c \uc5b4\uae68 \ubd80\uc0c1\uc744 \uc785\uc5c8\ub294\ub370 \uc774\uac83\uc740 \uc218\uc220\uc744 \ud544\uc694\ub85c \ud558\ub294 \ud68c\uc804\uadfc\uac1c(Rotator cuff) \uc190\uc0c1\uc774\uc5c8\uc73c\uba70 2015\ub144 \uad6d\uac00\ub300\ud45c \uc120\ubc1c\uc804\uc774 \ub05d\ub09c \ud6c4 \uc218\uc220\uc744 \ud558\uc600\ub2e4. 2014-2015 \uc2dc\uc98c\uc5d0\uc11c \uae40\uc544\ub791\uc740 5\ucc28\ub840 ISU \uc1fc\ud2b8\ud2b8\ub799 \uc6d4\ub4dc\ucef5\uc5d0 \ucd9c\uc804\ud558\uc5ec \uae08\uba54\ub2ec 3\uac1c \uc740\uba54\ub2ec 4\uac1c \ub3d9\uba54\ub2ec 1\uac1c\ub97c \ud68d\ub4dd\ud558\uc600\uc73c\uba70, 2015\ub144 2\uc6d4 \uc2a4\ud398\uc778 \uadf8\ub77c\ub098\ub2e4\uc5d0\uc11c \uc5f4\ub9b0 \ub3d9\uacc4 \uc720\ub2c8\ubc84\uc2dc\uc544\ub4dc \uc1fc\ud2b8\ud2b8\ub799 1000\ubbf8\ud130\uc640 1500\ubbf8\ud130\uc5d0\uc11c \uc6b0\uc2b9\ud558\uc5ec \uac1c\uc778\uc804 \uae08\uba54\ub2ec 2\uac1c\ub97c \ubaa9\uc5d0 \uac78\uc5c8\ub2e4. \ub2f9\uc2dc \uae40\uc544\ub791\uc740 \"\uae08\uba54\ub2ec 2\uac1c\ub97c \ucc28\uc9c0\ud55c \uac83\uc740 \uae30\uc058\uc9c0\ub9cc 3000\ubbf8\ud130 \uacc4\uc8fc\uc5d0\uc11c \uc911\uad6d\uc5d0\uac8c \uc11d\ud328\ud55c \uac83\uc774 \uc544\uc27d\ub2e4. \ub0b4\uac00 3\uad00\uc655\uc744 \ubabb\ud574\uc11c \uc544\uc26c\uc6b4 \uac83\uc774 \uc544\ub2c8\ub77c, \ud300 \uc804\uccb4\uac00 \ubaa8\ub450 \uae08\uba54\ub2ec\uc744 \ud568\uaed8 \ud560 \uc218 \uc788\ub294 \uae30\ud68c\ub97c \ub193\ucce4\uae30 \ub54c\ubb38\"\uc774\ub77c\uace0 \uae30\uc068\ubcf4\ub2e4\ub294 \uc544\uc26c\uc6c0\uc744 \ud45c\ud604\ud558\uc600\ub2e4.", "sentence_answer": "2015\ub144 2\uc6d4 \uc2a4\ud398\uc778 \uadf8\ub77c\ub098\ub2e4\uc5d0\uc11c \uc5f4\ub9b0 \ub3d9\uacc4 \uc720\ub2c8\ubc84\uc2dc\uc544\ub4dc \uc1fc\ud2b8\ud2b8\ub799 1000\ubbf8\ud130\uc640 1500\ubbf8\ud130\uc5d0\uc11c \uc6b0\uc2b9\ud558\uc5ec \uac1c\uc778\uc804 \uae08\uba54\ub2ec 2\uac1c\ub97c \ubaa9\uc5d0 \uac78\uc5c8\ub2e4.", "paragraph_id": "5998192-2-0", "paragraph_question": "question: 2015\ub144 \uae40\uc544\ub791\uc774 \uac1c\uc778\uc804 \uae08\uba54\ub2ec 2\uac1c\ub97c \ud68d\ub4dd\ud55c \ub3d9\uacc4 \uc720\ub2c8\ubc84\uc2dc\uc544\ub4dc \uac1c\ucd5c \ub098\ub77c\ub294?, context: \uae40\uc544\ub791\uc740 2014-2015 \uc2dc\uc98c \uad6d\uac00\ub300\ud45c \uc120\ubc1c\uc804\uc5d0\uc11c \uc5b4\uae68 \ubd80\uc0c1\uc744 \uc785\uc5c8\ub294\ub370 \uc774\uac83\uc740 \uc218\uc220\uc744 \ud544\uc694\ub85c \ud558\ub294 \ud68c\uc804\uadfc\uac1c(Rotator cuff) \uc190\uc0c1\uc774\uc5c8\uc73c\uba70 2015\ub144 \uad6d\uac00\ub300\ud45c \uc120\ubc1c\uc804\uc774 \ub05d\ub09c \ud6c4 \uc218\uc220\uc744 \ud558\uc600\ub2e4. 2014-2015 \uc2dc\uc98c\uc5d0\uc11c \uae40\uc544\ub791\uc740 5\ucc28\ub840 ISU \uc1fc\ud2b8\ud2b8\ub799 \uc6d4\ub4dc\ucef5\uc5d0 \ucd9c\uc804\ud558\uc5ec \uae08\uba54\ub2ec 3\uac1c \uc740\uba54\ub2ec 4\uac1c \ub3d9\uba54\ub2ec 1\uac1c\ub97c \ud68d\ub4dd\ud558\uc600\uc73c\uba70, 2015\ub144 2\uc6d4 \uc2a4\ud398\uc778 \uadf8\ub77c\ub098\ub2e4\uc5d0\uc11c \uc5f4\ub9b0 \ub3d9\uacc4 \uc720\ub2c8\ubc84\uc2dc\uc544\ub4dc \uc1fc\ud2b8\ud2b8\ub799 1000\ubbf8\ud130\uc640 1500\ubbf8\ud130\uc5d0\uc11c \uc6b0\uc2b9\ud558\uc5ec \uac1c\uc778\uc804 \uae08\uba54\ub2ec 2\uac1c\ub97c \ubaa9\uc5d0 \uac78\uc5c8\ub2e4. \ub2f9\uc2dc \uae40\uc544\ub791\uc740 \"\uae08\uba54\ub2ec 2\uac1c\ub97c \ucc28\uc9c0\ud55c \uac83\uc740 \uae30\uc058\uc9c0\ub9cc 3000\ubbf8\ud130 \uacc4\uc8fc\uc5d0\uc11c \uc911\uad6d\uc5d0\uac8c \uc11d\ud328\ud55c \uac83\uc774 \uc544\uc27d\ub2e4. \ub0b4\uac00 3\uad00\uc655\uc744 \ubabb\ud574\uc11c \uc544\uc26c\uc6b4 \uac83\uc774 \uc544\ub2c8\ub77c, \ud300 \uc804\uccb4\uac00 \ubaa8\ub450 \uae08\uba54\ub2ec\uc744 \ud568\uaed8 \ud560 \uc218 \uc788\ub294 \uae30\ud68c\ub97c \ub193\ucce4\uae30 \ub54c\ubb38\"\uc774\ub77c\uace0 \uae30\uc068\ubcf4\ub2e4\ub294 \uc544\uc26c\uc6c0\uc744 \ud45c\ud604\ud558\uc600\ub2e4."} {"question": "\uc804\ud558\ub97c \ub760\uc9c0 \uc54a\ub294 \ubb3c\uc9c8\uc740 \uc911\uc131\ubbf8\uc790 \uc678\uc5d0 \ub610 \ubb34\uc5c7\uc774 \uc788\ub294\uac00?", "paragraph": "\ub819\ud1a4\uc5d0\ub294 \uc804\uc790, \ubba4\uc628, \ud0c0\uc6b0\uc628\uc758 \uc138 \ub9db\uae54\uc774 \uc874\uc7ac\ud55c\ub2e4. \uac01 \ub9db\uae54\uc740 \uc57d\ud55c \uc544\uc774\uc18c\uc2a4\ud540\uc774\ub77c\ub294 \uc785\uc790\uc30d\uc73c\ub85c \ud45c\ud604\ub41c\ub2e4. \uc785\uc790\uc30d \uc911 \ud558\ub098\ub294 \uac01 \ub9db\uae54\uc758 \uc774\ub984\uacfc \ub3d9\uc77c\ud55c \uc774\ub984\uc758 \ub192\uc740 \uc804\ud558\ub97c \uc9c0\ub2cc \uc785\uc790\uc774\uba70, \ub2e4\ub978 \ud558\ub098\ub294 \uac70\uc758 \uc9c8\ub7c9\uc774 \uc5c6\ub294 \uc911\uc131\uc758 \uc785\uc790\uc778 \uc911\uc131\ubbf8\uc790\uc774\ub2e4. \uc608\ub97c \ub4e4\uba74, \uc804\uc790\ub294 \uc804\ud558\ub97c \uc9c0\ub2cc \uc804\uc790\uc640 \uc804\ud558\ub97c \uc9c0\ub2c8\uc9c0 \uc54a\uace0 \uac70\uc758 \uc9c8\ub7c9\uc774 \uc5c6\ub294 \uc804\uc790 \ubc18\uc911\uc131\ubbf8\uc790\ub85c \uad6c\uc131\ub41c\ub2e4\ub294 \uac83\uc774\ub2e4. \uc774 \ubaa8\ub4e0 6\uac1c\uc758 \uc785\uc790\ub294 \uac01\uac01 \ubc18\uc785\uc790\ub97c \uac00\uc9c0\uace0 \uc788\ub2e4. \uc608\ub97c \ub4e4\uba74, \uc804\uc790\uc640 \uc591\uc804\uc790\ub294 \uc11c\ub85c \ubc18\uc785\uc790\uc774\ub2e4. \ubaa8\ub4e0 \uc54c\ub824\uc9c4 \uc804\ud558\ub97c \uc9c0\ub2cc \ub819\ud1a4\uc740 \uae30\ubcf8 \uc804\ud558\ub9cc\ud07c\uc758 \uc804\ud558\ub97c \uc9c0\ub2c8\uace0 \uc788\uc73c\uba70, \uc74c\uc778\uc9c0 \uc591\uc778\uc9c0\ub294 \uc785\uc790\uc778\uc9c0 \ubc18\uc785\uc790\uc778\uc9c0\uc5d0 \ub530\ub77c \uc88c\uc6b0\ub41c\ub2e4. \ubc18\uba74 \uc911\uc131\ubbf8\uc790\uc640 \ubc18\uc911\uc131\ubbf8\uc790\ub294 \uc804\ud558\ub97c \ub760\uc9c0 \uc54a\ub294\ub2e4.", "answer": "\ubc18\uc911\uc131\ubbf8\uc790", "sentence": "\uc804\uc790 \ubc18\uc911\uc131\ubbf8\uc790 \ub85c \uad6c\uc131\ub41c\ub2e4\ub294 \uac83\uc774\ub2e4.", "paragraph_sentence": "\ub819\ud1a4\uc5d0\ub294 \uc804\uc790, \ubba4\uc628, \ud0c0\uc6b0\uc628\uc758 \uc138 \ub9db\uae54\uc774 \uc874\uc7ac\ud55c\ub2e4. \uac01 \ub9db\uae54\uc740 \uc57d\ud55c \uc544\uc774\uc18c\uc2a4\ud540\uc774\ub77c\ub294 \uc785\uc790\uc30d\uc73c\ub85c \ud45c\ud604\ub41c\ub2e4. \uc785\uc790\uc30d \uc911 \ud558\ub098\ub294 \uac01 \ub9db\uae54\uc758 \uc774\ub984\uacfc \ub3d9\uc77c\ud55c \uc774\ub984\uc758 \ub192\uc740 \uc804\ud558\ub97c \uc9c0\ub2cc \uc785\uc790\uc774\uba70, \ub2e4\ub978 \ud558\ub098\ub294 \uac70\uc758 \uc9c8\ub7c9\uc774 \uc5c6\ub294 \uc911\uc131\uc758 \uc785\uc790\uc778 \uc911\uc131\ubbf8\uc790\uc774\ub2e4. \uc608\ub97c \ub4e4\uba74, \uc804\uc790\ub294 \uc804\ud558\ub97c \uc9c0\ub2cc \uc804\uc790\uc640 \uc804\ud558\ub97c \uc9c0\ub2c8\uc9c0 \uc54a\uace0 \uac70\uc758 \uc9c8\ub7c9\uc774 \uc5c6\ub294 \uc804\uc790 \ubc18\uc911\uc131\ubbf8\uc790 \ub85c \uad6c\uc131\ub41c\ub2e4\ub294 \uac83\uc774\ub2e4. \uc774 \ubaa8\ub4e0 6\uac1c\uc758 \uc785\uc790\ub294 \uac01\uac01 \ubc18\uc785\uc790\ub97c \uac00\uc9c0\uace0 \uc788\ub2e4. \uc608\ub97c \ub4e4\uba74, \uc804\uc790\uc640 \uc591\uc804\uc790\ub294 \uc11c\ub85c \ubc18\uc785\uc790\uc774\ub2e4. \ubaa8\ub4e0 \uc54c\ub824\uc9c4 \uc804\ud558\ub97c \uc9c0\ub2cc \ub819\ud1a4\uc740 \uae30\ubcf8 \uc804\ud558\ub9cc\ud07c\uc758 \uc804\ud558\ub97c \uc9c0\ub2c8\uace0 \uc788\uc73c\uba70, \uc74c\uc778\uc9c0 \uc591\uc778\uc9c0\ub294 \uc785\uc790\uc778\uc9c0 \ubc18\uc785\uc790\uc778\uc9c0\uc5d0 \ub530\ub77c \uc88c\uc6b0\ub41c\ub2e4. \ubc18\uba74 \uc911\uc131\ubbf8\uc790\uc640 \ubc18\uc911\uc131\ubbf8\uc790\ub294 \uc804\ud558\ub97c \ub760\uc9c0 \uc54a\ub294\ub2e4.", "paragraph_answer": "\ub819\ud1a4\uc5d0\ub294 \uc804\uc790, \ubba4\uc628, \ud0c0\uc6b0\uc628\uc758 \uc138 \ub9db\uae54\uc774 \uc874\uc7ac\ud55c\ub2e4. \uac01 \ub9db\uae54\uc740 \uc57d\ud55c \uc544\uc774\uc18c\uc2a4\ud540\uc774\ub77c\ub294 \uc785\uc790\uc30d\uc73c\ub85c \ud45c\ud604\ub41c\ub2e4. \uc785\uc790\uc30d \uc911 \ud558\ub098\ub294 \uac01 \ub9db\uae54\uc758 \uc774\ub984\uacfc \ub3d9\uc77c\ud55c \uc774\ub984\uc758 \ub192\uc740 \uc804\ud558\ub97c \uc9c0\ub2cc \uc785\uc790\uc774\uba70, \ub2e4\ub978 \ud558\ub098\ub294 \uac70\uc758 \uc9c8\ub7c9\uc774 \uc5c6\ub294 \uc911\uc131\uc758 \uc785\uc790\uc778 \uc911\uc131\ubbf8\uc790\uc774\ub2e4. \uc608\ub97c \ub4e4\uba74, \uc804\uc790\ub294 \uc804\ud558\ub97c \uc9c0\ub2cc \uc804\uc790\uc640 \uc804\ud558\ub97c \uc9c0\ub2c8\uc9c0 \uc54a\uace0 \uac70\uc758 \uc9c8\ub7c9\uc774 \uc5c6\ub294 \uc804\uc790 \ubc18\uc911\uc131\ubbf8\uc790 \ub85c \uad6c\uc131\ub41c\ub2e4\ub294 \uac83\uc774\ub2e4. \uc774 \ubaa8\ub4e0 6\uac1c\uc758 \uc785\uc790\ub294 \uac01\uac01 \ubc18\uc785\uc790\ub97c \uac00\uc9c0\uace0 \uc788\ub2e4. \uc608\ub97c \ub4e4\uba74, \uc804\uc790\uc640 \uc591\uc804\uc790\ub294 \uc11c\ub85c \ubc18\uc785\uc790\uc774\ub2e4. \ubaa8\ub4e0 \uc54c\ub824\uc9c4 \uc804\ud558\ub97c \uc9c0\ub2cc \ub819\ud1a4\uc740 \uae30\ubcf8 \uc804\ud558\ub9cc\ud07c\uc758 \uc804\ud558\ub97c \uc9c0\ub2c8\uace0 \uc788\uc73c\uba70, \uc74c\uc778\uc9c0 \uc591\uc778\uc9c0\ub294 \uc785\uc790\uc778\uc9c0 \ubc18\uc785\uc790\uc778\uc9c0\uc5d0 \ub530\ub77c \uc88c\uc6b0\ub41c\ub2e4. \ubc18\uba74 \uc911\uc131\ubbf8\uc790\uc640 \ubc18\uc911\uc131\ubbf8\uc790\ub294 \uc804\ud558\ub97c \ub760\uc9c0 \uc54a\ub294\ub2e4.", "sentence_answer": "\uc804\uc790 \ubc18\uc911\uc131\ubbf8\uc790 \ub85c \uad6c\uc131\ub41c\ub2e4\ub294 \uac83\uc774\ub2e4.", "paragraph_id": "6489762-1-1", "paragraph_question": "question: \uc804\ud558\ub97c \ub760\uc9c0 \uc54a\ub294 \ubb3c\uc9c8\uc740 \uc911\uc131\ubbf8\uc790 \uc678\uc5d0 \ub610 \ubb34\uc5c7\uc774 \uc788\ub294\uac00?, context: \ub819\ud1a4\uc5d0\ub294 \uc804\uc790, \ubba4\uc628, \ud0c0\uc6b0\uc628\uc758 \uc138 \ub9db\uae54\uc774 \uc874\uc7ac\ud55c\ub2e4. \uac01 \ub9db\uae54\uc740 \uc57d\ud55c \uc544\uc774\uc18c\uc2a4\ud540\uc774\ub77c\ub294 \uc785\uc790\uc30d\uc73c\ub85c \ud45c\ud604\ub41c\ub2e4. \uc785\uc790\uc30d \uc911 \ud558\ub098\ub294 \uac01 \ub9db\uae54\uc758 \uc774\ub984\uacfc \ub3d9\uc77c\ud55c \uc774\ub984\uc758 \ub192\uc740 \uc804\ud558\ub97c \uc9c0\ub2cc \uc785\uc790\uc774\uba70, \ub2e4\ub978 \ud558\ub098\ub294 \uac70\uc758 \uc9c8\ub7c9\uc774 \uc5c6\ub294 \uc911\uc131\uc758 \uc785\uc790\uc778 \uc911\uc131\ubbf8\uc790\uc774\ub2e4. \uc608\ub97c \ub4e4\uba74, \uc804\uc790\ub294 \uc804\ud558\ub97c \uc9c0\ub2cc \uc804\uc790\uc640 \uc804\ud558\ub97c \uc9c0\ub2c8\uc9c0 \uc54a\uace0 \uac70\uc758 \uc9c8\ub7c9\uc774 \uc5c6\ub294 \uc804\uc790 \ubc18\uc911\uc131\ubbf8\uc790\ub85c \uad6c\uc131\ub41c\ub2e4\ub294 \uac83\uc774\ub2e4. \uc774 \ubaa8\ub4e0 6\uac1c\uc758 \uc785\uc790\ub294 \uac01\uac01 \ubc18\uc785\uc790\ub97c \uac00\uc9c0\uace0 \uc788\ub2e4. \uc608\ub97c \ub4e4\uba74, \uc804\uc790\uc640 \uc591\uc804\uc790\ub294 \uc11c\ub85c \ubc18\uc785\uc790\uc774\ub2e4. \ubaa8\ub4e0 \uc54c\ub824\uc9c4 \uc804\ud558\ub97c \uc9c0\ub2cc \ub819\ud1a4\uc740 \uae30\ubcf8 \uc804\ud558\ub9cc\ud07c\uc758 \uc804\ud558\ub97c \uc9c0\ub2c8\uace0 \uc788\uc73c\uba70, \uc74c\uc778\uc9c0 \uc591\uc778\uc9c0\ub294 \uc785\uc790\uc778\uc9c0 \ubc18\uc785\uc790\uc778\uc9c0\uc5d0 \ub530\ub77c \uc88c\uc6b0\ub41c\ub2e4. \ubc18\uba74 \uc911\uc131\ubbf8\uc790\uc640 \ubc18\uc911\uc131\ubbf8\uc790\ub294 \uc804\ud558\ub97c \ub760\uc9c0 \uc54a\ub294\ub2e4."} {"question": "\uc624\uc2a4\ud2b8\ub808\uc77c\ub9ac\uc544\uae4c\uce58\uc758 \ubc88\uc2dd\uae30\ub294 8\uc6d4 \ub9d0\ubd80\ud130 \uc5b8\uc81c\uae4c\uc9c0\uc778\uac00?", "paragraph": "\uc624\uc2a4\ud2b8\ub808\uc77c\ub9ac\uc544\uae4c\uce58\ub294 \uc624\uc2a4\ud2b8\ub808\uc77c\ub9ac\uc544\uc758 \uac70\uc758 \ubaa8\ub4e0 \ub3c4\uc2dc \uc9c0\uc5ed\uc5d0 \ud3b8\uc7ac\ud558\uace0 \uc788\uc73c\uba70, \uc778\uac04\uc5d0\uac8c\ub3c4 \uc774\ubbf8 \uc775\uc219\ud574\uc838 \uc788\ub2e4. 8\uc6d4 \ub9d0\uc5d0\uc11c 10\uc6d4 \ucd08 \ubc88\uc2dd\uae30\uc5d0 \uc18c\uc218\uc758 \uac1c\uccb4\ub4e4\uc774 \uc778\uac04\uc5d0\uac8c \ub192\uc740 \uacf5\uaca9\uc131\uc744 \ub4dc\ub7ec\ub0b4\ub294 \uacbd\uc6b0\uac00 \uc788\uc73c\uba70, \uadf8\ub7ec\ud55c \uac1c\uccb4\ub294 \ub465\uc9c0 \ubc11\uc744 \uc9c0\ub098\uac00\ub294 \uc778\uac04\uc5d0\uac8c \uae09\uac15\ud558\ub97c \ud574\uc11c \ub36e\uce5c\ub2e4. \uc774\ub7f0 \uc77c\uc774 \uc5bc\ub9c8\ub098 \uc77c\uc5b4\ub098\ub294\uc9c0 \ucd94\uc0b0\ud558\ub294 \uac83\uc740 \uc5b4\ub824\uc6b4 \uc77c\uc774\uc9c0\ub9cc, 9% \uc774\ud558\ub85c \uc720\uc758\ubbf8\ud558\ub2e4. \uc0ac\ub78c\uc744 \ub36e\uce5c \uac1c\uccb4\ub4e4\uc740 \uac70\uc758 \ubaa8\ub450(99%) \uc218\ucef7\uc774\uc5c8\ub2e4. \ub188\ub4e4\uc740 \ub465\uc9c0\uc5d0\uc11c \ubc18\uacbd 50 \ubbf8\ud130 \uc548\uc5d0 \uc788\ub294 \ubcf4\ud589\uc790, \ubc18\uacbd 100 \ubbf8\ud130 \uc548\uc5d0 \uc788\ub294 \uc790\uc804\uac70 \ud0d1\uc2b9\uc790\ub97c \uacf5\uaca9\ud55c\ub2e4\uace0 \uc775\ud788 \uc54c\ub824\uc838 \uc788\ub2e4. \uacf5\uaca9\uc740 \uc54c\uc774 \ubd80\ud654\ud558\uba74\uc11c \uc2dc\uc791\ub418\uace0, \uc0c8\ub07c\uac00 \uc790\ub784\uc218\ub85d \uacf5\uaca9 \ube48\ub3c4\ub3c4 \ub192\uc544\uc9c0\uace0 \uacf5\uaca9\uc131\ub3c4 \uc2ec\ud574\uc9c4\ub2e4. \uadf8\ub7ec\ub2e4\uac00 \uc0c8\ub07c\uac00 \uc911\ubcd1\uc544\ub9ac\uac00 \ub418\uc5b4 \ub465\uc9c0\ub97c \ub5a0\ub098\uba74 \ucc28\uce30 \uc7a6\uc544\ub4e0\ub2e4.", "answer": "10\uc6d4 \ucd08", "sentence": "8\uc6d4 \ub9d0\uc5d0\uc11c 10\uc6d4 \ucd08 \ubc88\uc2dd\uae30\uc5d0 \uc18c\uc218\uc758 \uac1c\uccb4\ub4e4\uc774 \uc778\uac04\uc5d0\uac8c \ub192\uc740 \uacf5\uaca9\uc131\uc744 \ub4dc\ub7ec\ub0b4\ub294 \uacbd\uc6b0\uac00 \uc788\uc73c\uba70, \uadf8\ub7ec\ud55c \uac1c\uccb4\ub294 \ub465\uc9c0 \ubc11\uc744 \uc9c0\ub098\uac00\ub294 \uc778\uac04\uc5d0\uac8c \uae09\uac15\ud558\ub97c \ud574\uc11c \ub36e\uce5c\ub2e4.", "paragraph_sentence": "\uc624\uc2a4\ud2b8\ub808\uc77c\ub9ac\uc544\uae4c\uce58\ub294 \uc624\uc2a4\ud2b8\ub808\uc77c\ub9ac\uc544\uc758 \uac70\uc758 \ubaa8\ub4e0 \ub3c4\uc2dc \uc9c0\uc5ed\uc5d0 \ud3b8\uc7ac\ud558\uace0 \uc788\uc73c\uba70, \uc778\uac04\uc5d0\uac8c\ub3c4 \uc774\ubbf8 \uc775\uc219\ud574\uc838 \uc788\ub2e4. 8\uc6d4 \ub9d0\uc5d0\uc11c 10\uc6d4 \ucd08 \ubc88\uc2dd\uae30\uc5d0 \uc18c\uc218\uc758 \uac1c\uccb4\ub4e4\uc774 \uc778\uac04\uc5d0\uac8c \ub192\uc740 \uacf5\uaca9\uc131\uc744 \ub4dc\ub7ec\ub0b4\ub294 \uacbd\uc6b0\uac00 \uc788\uc73c\uba70, \uadf8\ub7ec\ud55c \uac1c\uccb4\ub294 \ub465\uc9c0 \ubc11\uc744 \uc9c0\ub098\uac00\ub294 \uc778\uac04\uc5d0\uac8c \uae09\uac15\ud558\ub97c \ud574\uc11c \ub36e\uce5c\ub2e4. \uc774\ub7f0 \uc77c\uc774 \uc5bc\ub9c8\ub098 \uc77c\uc5b4\ub098\ub294\uc9c0 \ucd94\uc0b0\ud558\ub294 \uac83\uc740 \uc5b4\ub824\uc6b4 \uc77c\uc774\uc9c0\ub9cc, 9% \uc774\ud558\ub85c \uc720\uc758\ubbf8\ud558\ub2e4. \uc0ac\ub78c\uc744 \ub36e\uce5c \uac1c\uccb4\ub4e4\uc740 \uac70\uc758 \ubaa8\ub450(99%) \uc218\ucef7\uc774\uc5c8\ub2e4. \ub188\ub4e4\uc740 \ub465\uc9c0\uc5d0\uc11c \ubc18\uacbd 50 \ubbf8\ud130 \uc548\uc5d0 \uc788\ub294 \ubcf4\ud589\uc790, \ubc18\uacbd 100 \ubbf8\ud130 \uc548\uc5d0 \uc788\ub294 \uc790\uc804\uac70 \ud0d1\uc2b9\uc790\ub97c \uacf5\uaca9\ud55c\ub2e4\uace0 \uc775\ud788 \uc54c\ub824\uc838 \uc788\ub2e4. \uacf5\uaca9\uc740 \uc54c\uc774 \ubd80\ud654\ud558\uba74\uc11c \uc2dc\uc791\ub418\uace0, \uc0c8\ub07c\uac00 \uc790\ub784\uc218\ub85d \uacf5\uaca9 \ube48\ub3c4\ub3c4 \ub192\uc544\uc9c0\uace0 \uacf5\uaca9\uc131\ub3c4 \uc2ec\ud574\uc9c4\ub2e4. \uadf8\ub7ec\ub2e4\uac00 \uc0c8\ub07c\uac00 \uc911\ubcd1\uc544\ub9ac\uac00 \ub418\uc5b4 \ub465\uc9c0\ub97c \ub5a0\ub098\uba74 \ucc28\uce30 \uc7a6\uc544\ub4e0\ub2e4.", "paragraph_answer": "\uc624\uc2a4\ud2b8\ub808\uc77c\ub9ac\uc544\uae4c\uce58\ub294 \uc624\uc2a4\ud2b8\ub808\uc77c\ub9ac\uc544\uc758 \uac70\uc758 \ubaa8\ub4e0 \ub3c4\uc2dc \uc9c0\uc5ed\uc5d0 \ud3b8\uc7ac\ud558\uace0 \uc788\uc73c\uba70, \uc778\uac04\uc5d0\uac8c\ub3c4 \uc774\ubbf8 \uc775\uc219\ud574\uc838 \uc788\ub2e4. 8\uc6d4 \ub9d0\uc5d0\uc11c 10\uc6d4 \ucd08 \ubc88\uc2dd\uae30\uc5d0 \uc18c\uc218\uc758 \uac1c\uccb4\ub4e4\uc774 \uc778\uac04\uc5d0\uac8c \ub192\uc740 \uacf5\uaca9\uc131\uc744 \ub4dc\ub7ec\ub0b4\ub294 \uacbd\uc6b0\uac00 \uc788\uc73c\uba70, \uadf8\ub7ec\ud55c \uac1c\uccb4\ub294 \ub465\uc9c0 \ubc11\uc744 \uc9c0\ub098\uac00\ub294 \uc778\uac04\uc5d0\uac8c \uae09\uac15\ud558\ub97c \ud574\uc11c \ub36e\uce5c\ub2e4. \uc774\ub7f0 \uc77c\uc774 \uc5bc\ub9c8\ub098 \uc77c\uc5b4\ub098\ub294\uc9c0 \ucd94\uc0b0\ud558\ub294 \uac83\uc740 \uc5b4\ub824\uc6b4 \uc77c\uc774\uc9c0\ub9cc, 9% \uc774\ud558\ub85c \uc720\uc758\ubbf8\ud558\ub2e4. \uc0ac\ub78c\uc744 \ub36e\uce5c \uac1c\uccb4\ub4e4\uc740 \uac70\uc758 \ubaa8\ub450(99%) \uc218\ucef7\uc774\uc5c8\ub2e4. \ub188\ub4e4\uc740 \ub465\uc9c0\uc5d0\uc11c \ubc18\uacbd 50 \ubbf8\ud130 \uc548\uc5d0 \uc788\ub294 \ubcf4\ud589\uc790, \ubc18\uacbd 100 \ubbf8\ud130 \uc548\uc5d0 \uc788\ub294 \uc790\uc804\uac70 \ud0d1\uc2b9\uc790\ub97c \uacf5\uaca9\ud55c\ub2e4\uace0 \uc775\ud788 \uc54c\ub824\uc838 \uc788\ub2e4. \uacf5\uaca9\uc740 \uc54c\uc774 \ubd80\ud654\ud558\uba74\uc11c \uc2dc\uc791\ub418\uace0, \uc0c8\ub07c\uac00 \uc790\ub784\uc218\ub85d \uacf5\uaca9 \ube48\ub3c4\ub3c4 \ub192\uc544\uc9c0\uace0 \uacf5\uaca9\uc131\ub3c4 \uc2ec\ud574\uc9c4\ub2e4. \uadf8\ub7ec\ub2e4\uac00 \uc0c8\ub07c\uac00 \uc911\ubcd1\uc544\ub9ac\uac00 \ub418\uc5b4 \ub465\uc9c0\ub97c \ub5a0\ub098\uba74 \ucc28\uce30 \uc7a6\uc544\ub4e0\ub2e4.", "sentence_answer": "8\uc6d4 \ub9d0\uc5d0\uc11c 10\uc6d4 \ucd08 \ubc88\uc2dd\uae30\uc5d0 \uc18c\uc218\uc758 \uac1c\uccb4\ub4e4\uc774 \uc778\uac04\uc5d0\uac8c \ub192\uc740 \uacf5\uaca9\uc131\uc744 \ub4dc\ub7ec\ub0b4\ub294 \uacbd\uc6b0\uac00 \uc788\uc73c\uba70, \uadf8\ub7ec\ud55c \uac1c\uccb4\ub294 \ub465\uc9c0 \ubc11\uc744 \uc9c0\ub098\uac00\ub294 \uc778\uac04\uc5d0\uac8c \uae09\uac15\ud558\ub97c \ud574\uc11c \ub36e\uce5c\ub2e4.", "paragraph_id": "6541265-13-0", "paragraph_question": "question: \uc624\uc2a4\ud2b8\ub808\uc77c\ub9ac\uc544\uae4c\uce58\uc758 \ubc88\uc2dd\uae30\ub294 8\uc6d4 \ub9d0\ubd80\ud130 \uc5b8\uc81c\uae4c\uc9c0\uc778\uac00?, context: \uc624\uc2a4\ud2b8\ub808\uc77c\ub9ac\uc544\uae4c\uce58\ub294 \uc624\uc2a4\ud2b8\ub808\uc77c\ub9ac\uc544\uc758 \uac70\uc758 \ubaa8\ub4e0 \ub3c4\uc2dc \uc9c0\uc5ed\uc5d0 \ud3b8\uc7ac\ud558\uace0 \uc788\uc73c\uba70, \uc778\uac04\uc5d0\uac8c\ub3c4 \uc774\ubbf8 \uc775\uc219\ud574\uc838 \uc788\ub2e4. 8\uc6d4 \ub9d0\uc5d0\uc11c 10\uc6d4 \ucd08 \ubc88\uc2dd\uae30\uc5d0 \uc18c\uc218\uc758 \uac1c\uccb4\ub4e4\uc774 \uc778\uac04\uc5d0\uac8c \ub192\uc740 \uacf5\uaca9\uc131\uc744 \ub4dc\ub7ec\ub0b4\ub294 \uacbd\uc6b0\uac00 \uc788\uc73c\uba70, \uadf8\ub7ec\ud55c \uac1c\uccb4\ub294 \ub465\uc9c0 \ubc11\uc744 \uc9c0\ub098\uac00\ub294 \uc778\uac04\uc5d0\uac8c \uae09\uac15\ud558\ub97c \ud574\uc11c \ub36e\uce5c\ub2e4. \uc774\ub7f0 \uc77c\uc774 \uc5bc\ub9c8\ub098 \uc77c\uc5b4\ub098\ub294\uc9c0 \ucd94\uc0b0\ud558\ub294 \uac83\uc740 \uc5b4\ub824\uc6b4 \uc77c\uc774\uc9c0\ub9cc, 9% \uc774\ud558\ub85c \uc720\uc758\ubbf8\ud558\ub2e4. \uc0ac\ub78c\uc744 \ub36e\uce5c \uac1c\uccb4\ub4e4\uc740 \uac70\uc758 \ubaa8\ub450(99%) \uc218\ucef7\uc774\uc5c8\ub2e4. \ub188\ub4e4\uc740 \ub465\uc9c0\uc5d0\uc11c \ubc18\uacbd 50 \ubbf8\ud130 \uc548\uc5d0 \uc788\ub294 \ubcf4\ud589\uc790, \ubc18\uacbd 100 \ubbf8\ud130 \uc548\uc5d0 \uc788\ub294 \uc790\uc804\uac70 \ud0d1\uc2b9\uc790\ub97c \uacf5\uaca9\ud55c\ub2e4\uace0 \uc775\ud788 \uc54c\ub824\uc838 \uc788\ub2e4. \uacf5\uaca9\uc740 \uc54c\uc774 \ubd80\ud654\ud558\uba74\uc11c \uc2dc\uc791\ub418\uace0, \uc0c8\ub07c\uac00 \uc790\ub784\uc218\ub85d \uacf5\uaca9 \ube48\ub3c4\ub3c4 \ub192\uc544\uc9c0\uace0 \uacf5\uaca9\uc131\ub3c4 \uc2ec\ud574\uc9c4\ub2e4. \uadf8\ub7ec\ub2e4\uac00 \uc0c8\ub07c\uac00 \uc911\ubcd1\uc544\ub9ac\uac00 \ub418\uc5b4 \ub465\uc9c0\ub97c \ub5a0\ub098\uba74 \ucc28\uce30 \uc7a6\uc544\ub4e0\ub2e4."} {"question": "\uc591\ubb34\uc640 \ubcc0\ubc95\uc0ac\uc0c1\uc744 \uc804\ubc18\uc801\uc73c\ub85c \ubd80\uc815\ud558\uae30 \uc704\ud574 \uc77c\uc73c\ucf1c\ub7b4 \ud55c \uc6b4\ub3d9\uc740?", "paragraph": "\uc758\ud654\uad8c\uc758 \ud65c\ub3d9\uc774 \ud65c\ubc1c\ud788 \uc77c\uc5b4\ub098\uace0 \uc788\uc744 \ub54c \uccad\uc870\uc758 \ud0dc\ub3c4\ub294 \ud655\uace0\ud558\uc9c0 \uc54a\uc544\uc11c '\ucd08\ubb34(\u527f\u64ab) \uc591\uba74' \uc815\ucc45\uc744 \ud3c8\ub2e4. \uc774\ub294 \uc870\uc815\uc548\uc5d0\ub3c4 \ub450 \ud30c\ub85c \ub098\ub258\uc5b4 \uc788\uc74c\uc744 \uc758\ubbf8\ud55c\ub2e4. \uc989 \ud55c \ud30c\ub294 \uc758\ud654\ub2e8\uc744 \ubc31\ub828\uad50\uc640 \uac19\uc740 \uc0ac\uad50\uc5d0\uc11c \ub098\uc628 \uac83\uc774\ubbc0\ub85c \ud558\ub8e8\ube68\ub9ac \uc5c4\uaca9\ud558\uac8c \ud1a0\ubc8c\ud558\uc5ec \uc678\uad6d\uc778\ub4e4\uc774 \ubb34\ub825\uc744 \uc0ac\uc6a9\ud558\uc5ec \uac04\uc12d\ud558\ub824\ub294 \uad6c\uc2e4\uc744 \uc8fc\uc9c0\ub9d0\uc544\uc57c \ud55c\ub2e4\uace0 \uc8fc\uc7a5\ud558\uc600\ub2e4. \ub610 \ud55c \ud30c\ub294 \uc758\ud654\ub2e8\uc758 \uc870\uc9c1\uacfc \uadf8\ub4e4\uc774 \uc9c4\ud589\ud558\uace0 \uc788\ub294 \ubc18\uae30\ub3c5\uad50 \ud22c\uc7c1\uacfc\ub294 \uad6c\ubcc4\ud558\uc5ec \ubc18\uae30\ub3c5\uad50 \uc6b4\ub3d9\uc5d0 \ub300\ud558\uc5ec \uc5c4\uaca9\ud558\uac8c \ub2e4\uc2a4\ub824 \ub9cc\uc5f0\ub418\uc9c0 \ubabb\ud558\ub3c4\ub85d \ud558\uace0 \uadf8 \uc870\uc9c1\uc744 \ud5a5\ub2e8(\u9115\u5718)\uc73c\ub85c \ud3b8\uc131\ud558\uc5ec \uad00\uc758 \ud1b5\uc81c\ub97c \ubc1b\ub3c4\ub85d \ud558\uba74\uc11c \uc758\ud654\ub2e8\uc744 \uc774\uc6a9\ud558\uc790\ub294 \uc8fc\uc7a5\uc774\uc5c8\ub2e4. \ud2b9\ud788 \ud6c4\uc790\ub4e4\uc740 \uad11\uc11c\uc81c\ub97c \ube44\ud638\ud558\uace0 \uc788\ub294 \uc5f4\uac15\uc5d0 \ub300\ud56d\ud558\uace0, \uc11c\ud0dc\ud6c4\uc758 \ud6c8\uc815(\u8a13\u653f)\uc744 \uacc4\uc18d\ud560 \uc218 \uc788\uc744 \ubfd0\ub9cc \uc544\ub2c8\ub77c \ub2e4\ub978 \uc0ac\ub78c\uc744 \ud669\uc81c\ub85c \uc549\ud788\ub824\ub294 \uc11c\ud0dc\ud6c4\uc758 \ub73b\uc5d0 \uc601\ud569\ud558\ub824 \ud55c \uac83\uc774\ub2e4. \ubfd0\ub9cc \uc544\ub2c8\ub77c \uc758\ud654\ub2e8 \uc6b4\ub3d9\uc744 \uac16\uace0 \ubc18\uae30\ub3c5\uad50 \uc6b4\ub3d9\uc744 \uc77c\uc73c\ucf1c \uc591\ubb34\uc640 \ubcc0\ubc95\uc0ac\uc0c1\uc744 \uc804\ubc18\uc801\uc73c\ub85c \ubd80\uc815\ud558\ub824\ub294 \uc218\uad6c\uc801\uc778 \uc758\uc2dd\uc744 \uac16\uace0 \uc788\uc5c8\ub2e4.", "answer": "\ubc18\uae30\ub3c5\uad50 \uc6b4\ub3d9", "sentence": "\ub610 \ud55c \ud30c\ub294 \uc758\ud654\ub2e8\uc758 \uc870\uc9c1\uacfc \uadf8\ub4e4\uc774 \uc9c4\ud589\ud558\uace0 \uc788\ub294 \ubc18\uae30\ub3c5\uad50 \ud22c\uc7c1\uacfc\ub294 \uad6c\ubcc4\ud558\uc5ec \ubc18\uae30\ub3c5\uad50 \uc6b4\ub3d9 \uc5d0 \ub300\ud558\uc5ec \uc5c4\uaca9\ud558\uac8c \ub2e4\uc2a4\ub824 \ub9cc\uc5f0\ub418\uc9c0 \ubabb\ud558\ub3c4\ub85d \ud558\uace0 \uadf8 \uc870\uc9c1\uc744 \ud5a5\ub2e8(\u9115\u5718)\uc73c\ub85c \ud3b8\uc131\ud558\uc5ec \uad00\uc758 \ud1b5\uc81c\ub97c \ubc1b\ub3c4\ub85d \ud558\uba74\uc11c \uc758\ud654\ub2e8\uc744 \uc774\uc6a9\ud558\uc790\ub294 \uc8fc\uc7a5\uc774\uc5c8\ub2e4.", "paragraph_sentence": "\uc758\ud654\uad8c\uc758 \ud65c\ub3d9\uc774 \ud65c\ubc1c\ud788 \uc77c\uc5b4\ub098\uace0 \uc788\uc744 \ub54c \uccad\uc870\uc758 \ud0dc\ub3c4\ub294 \ud655\uace0\ud558\uc9c0 \uc54a\uc544\uc11c '\ucd08\ubb34(\u527f\u64ab) \uc591\uba74' \uc815\ucc45\uc744 \ud3c8\ub2e4. \uc774\ub294 \uc870\uc815\uc548\uc5d0\ub3c4 \ub450 \ud30c\ub85c \ub098\ub258\uc5b4 \uc788\uc74c\uc744 \uc758\ubbf8\ud55c\ub2e4. \uc989 \ud55c \ud30c\ub294 \uc758\ud654\ub2e8\uc744 \ubc31\ub828\uad50\uc640 \uac19\uc740 \uc0ac\uad50\uc5d0\uc11c \ub098\uc628 \uac83\uc774\ubbc0\ub85c \ud558\ub8e8\ube68\ub9ac \uc5c4\uaca9\ud558\uac8c \ud1a0\ubc8c\ud558\uc5ec \uc678\uad6d\uc778\ub4e4\uc774 \ubb34\ub825\uc744 \uc0ac\uc6a9\ud558\uc5ec \uac04\uc12d\ud558\ub824\ub294 \uad6c\uc2e4\uc744 \uc8fc\uc9c0\ub9d0\uc544\uc57c \ud55c\ub2e4\uace0 \uc8fc\uc7a5\ud558\uc600\ub2e4. \ub610 \ud55c \ud30c\ub294 \uc758\ud654\ub2e8\uc758 \uc870\uc9c1\uacfc \uadf8\ub4e4\uc774 \uc9c4\ud589\ud558\uace0 \uc788\ub294 \ubc18\uae30\ub3c5\uad50 \ud22c\uc7c1\uacfc\ub294 \uad6c\ubcc4\ud558\uc5ec \ubc18\uae30\ub3c5\uad50 \uc6b4\ub3d9 \uc5d0 \ub300\ud558\uc5ec \uc5c4\uaca9\ud558\uac8c \ub2e4\uc2a4\ub824 \ub9cc\uc5f0\ub418\uc9c0 \ubabb\ud558\ub3c4\ub85d \ud558\uace0 \uadf8 \uc870\uc9c1\uc744 \ud5a5\ub2e8(\u9115\u5718)\uc73c\ub85c \ud3b8\uc131\ud558\uc5ec \uad00\uc758 \ud1b5\uc81c\ub97c \ubc1b\ub3c4\ub85d \ud558\uba74\uc11c \uc758\ud654\ub2e8\uc744 \uc774\uc6a9\ud558\uc790\ub294 \uc8fc\uc7a5\uc774\uc5c8\ub2e4. \ud2b9\ud788 \ud6c4\uc790\ub4e4\uc740 \uad11\uc11c\uc81c\ub97c \ube44\ud638\ud558\uace0 \uc788\ub294 \uc5f4\uac15\uc5d0 \ub300\ud56d\ud558\uace0, \uc11c\ud0dc\ud6c4\uc758 \ud6c8\uc815(\u8a13\u653f)\uc744 \uacc4\uc18d\ud560 \uc218 \uc788\uc744 \ubfd0\ub9cc \uc544\ub2c8\ub77c \ub2e4\ub978 \uc0ac\ub78c\uc744 \ud669\uc81c\ub85c \uc549\ud788\ub824\ub294 \uc11c\ud0dc\ud6c4\uc758 \ub73b\uc5d0 \uc601\ud569\ud558\ub824 \ud55c \uac83\uc774\ub2e4. \ubfd0\ub9cc \uc544\ub2c8\ub77c \uc758\ud654\ub2e8 \uc6b4\ub3d9\uc744 \uac16\uace0 \ubc18\uae30\ub3c5\uad50 \uc6b4\ub3d9\uc744 \uc77c\uc73c\ucf1c \uc591\ubb34\uc640 \ubcc0\ubc95\uc0ac\uc0c1\uc744 \uc804\ubc18\uc801\uc73c\ub85c \ubd80\uc815\ud558\ub824\ub294 \uc218\uad6c\uc801\uc778 \uc758\uc2dd\uc744 \uac16\uace0 \uc788\uc5c8\ub2e4.", "paragraph_answer": "\uc758\ud654\uad8c\uc758 \ud65c\ub3d9\uc774 \ud65c\ubc1c\ud788 \uc77c\uc5b4\ub098\uace0 \uc788\uc744 \ub54c \uccad\uc870\uc758 \ud0dc\ub3c4\ub294 \ud655\uace0\ud558\uc9c0 \uc54a\uc544\uc11c '\ucd08\ubb34(\u527f\u64ab) \uc591\uba74' \uc815\ucc45\uc744 \ud3c8\ub2e4. \uc774\ub294 \uc870\uc815\uc548\uc5d0\ub3c4 \ub450 \ud30c\ub85c \ub098\ub258\uc5b4 \uc788\uc74c\uc744 \uc758\ubbf8\ud55c\ub2e4. \uc989 \ud55c \ud30c\ub294 \uc758\ud654\ub2e8\uc744 \ubc31\ub828\uad50\uc640 \uac19\uc740 \uc0ac\uad50\uc5d0\uc11c \ub098\uc628 \uac83\uc774\ubbc0\ub85c \ud558\ub8e8\ube68\ub9ac \uc5c4\uaca9\ud558\uac8c \ud1a0\ubc8c\ud558\uc5ec \uc678\uad6d\uc778\ub4e4\uc774 \ubb34\ub825\uc744 \uc0ac\uc6a9\ud558\uc5ec \uac04\uc12d\ud558\ub824\ub294 \uad6c\uc2e4\uc744 \uc8fc\uc9c0\ub9d0\uc544\uc57c \ud55c\ub2e4\uace0 \uc8fc\uc7a5\ud558\uc600\ub2e4. \ub610 \ud55c \ud30c\ub294 \uc758\ud654\ub2e8\uc758 \uc870\uc9c1\uacfc \uadf8\ub4e4\uc774 \uc9c4\ud589\ud558\uace0 \uc788\ub294 \ubc18\uae30\ub3c5\uad50 \ud22c\uc7c1\uacfc\ub294 \uad6c\ubcc4\ud558\uc5ec \ubc18\uae30\ub3c5\uad50 \uc6b4\ub3d9 \uc5d0 \ub300\ud558\uc5ec \uc5c4\uaca9\ud558\uac8c \ub2e4\uc2a4\ub824 \ub9cc\uc5f0\ub418\uc9c0 \ubabb\ud558\ub3c4\ub85d \ud558\uace0 \uadf8 \uc870\uc9c1\uc744 \ud5a5\ub2e8(\u9115\u5718)\uc73c\ub85c \ud3b8\uc131\ud558\uc5ec \uad00\uc758 \ud1b5\uc81c\ub97c \ubc1b\ub3c4\ub85d \ud558\uba74\uc11c \uc758\ud654\ub2e8\uc744 \uc774\uc6a9\ud558\uc790\ub294 \uc8fc\uc7a5\uc774\uc5c8\ub2e4. \ud2b9\ud788 \ud6c4\uc790\ub4e4\uc740 \uad11\uc11c\uc81c\ub97c \ube44\ud638\ud558\uace0 \uc788\ub294 \uc5f4\uac15\uc5d0 \ub300\ud56d\ud558\uace0, \uc11c\ud0dc\ud6c4\uc758 \ud6c8\uc815(\u8a13\u653f)\uc744 \uacc4\uc18d\ud560 \uc218 \uc788\uc744 \ubfd0\ub9cc \uc544\ub2c8\ub77c \ub2e4\ub978 \uc0ac\ub78c\uc744 \ud669\uc81c\ub85c \uc549\ud788\ub824\ub294 \uc11c\ud0dc\ud6c4\uc758 \ub73b\uc5d0 \uc601\ud569\ud558\ub824 \ud55c \uac83\uc774\ub2e4. \ubfd0\ub9cc \uc544\ub2c8\ub77c \uc758\ud654\ub2e8 \uc6b4\ub3d9\uc744 \uac16\uace0 \ubc18\uae30\ub3c5\uad50 \uc6b4\ub3d9\uc744 \uc77c\uc73c\ucf1c \uc591\ubb34\uc640 \ubcc0\ubc95\uc0ac\uc0c1\uc744 \uc804\ubc18\uc801\uc73c\ub85c \ubd80\uc815\ud558\ub824\ub294 \uc218\uad6c\uc801\uc778 \uc758\uc2dd\uc744 \uac16\uace0 \uc788\uc5c8\ub2e4.", "sentence_answer": "\ub610 \ud55c \ud30c\ub294 \uc758\ud654\ub2e8\uc758 \uc870\uc9c1\uacfc \uadf8\ub4e4\uc774 \uc9c4\ud589\ud558\uace0 \uc788\ub294 \ubc18\uae30\ub3c5\uad50 \ud22c\uc7c1\uacfc\ub294 \uad6c\ubcc4\ud558\uc5ec \ubc18\uae30\ub3c5\uad50 \uc6b4\ub3d9 \uc5d0 \ub300\ud558\uc5ec \uc5c4\uaca9\ud558\uac8c \ub2e4\uc2a4\ub824 \ub9cc\uc5f0\ub418\uc9c0 \ubabb\ud558\ub3c4\ub85d \ud558\uace0 \uadf8 \uc870\uc9c1\uc744 \ud5a5\ub2e8(\u9115\u5718)\uc73c\ub85c \ud3b8\uc131\ud558\uc5ec \uad00\uc758 \ud1b5\uc81c\ub97c \ubc1b\ub3c4\ub85d \ud558\uba74\uc11c \uc758\ud654\ub2e8\uc744 \uc774\uc6a9\ud558\uc790\ub294 \uc8fc\uc7a5\uc774\uc5c8\ub2e4.", "paragraph_id": "6544419-6-2", "paragraph_question": "question: \uc591\ubb34\uc640 \ubcc0\ubc95\uc0ac\uc0c1\uc744 \uc804\ubc18\uc801\uc73c\ub85c \ubd80\uc815\ud558\uae30 \uc704\ud574 \uc77c\uc73c\ucf1c\ub7b4 \ud55c \uc6b4\ub3d9\uc740?, context: \uc758\ud654\uad8c\uc758 \ud65c\ub3d9\uc774 \ud65c\ubc1c\ud788 \uc77c\uc5b4\ub098\uace0 \uc788\uc744 \ub54c \uccad\uc870\uc758 \ud0dc\ub3c4\ub294 \ud655\uace0\ud558\uc9c0 \uc54a\uc544\uc11c '\ucd08\ubb34(\u527f\u64ab) \uc591\uba74' \uc815\ucc45\uc744 \ud3c8\ub2e4. \uc774\ub294 \uc870\uc815\uc548\uc5d0\ub3c4 \ub450 \ud30c\ub85c \ub098\ub258\uc5b4 \uc788\uc74c\uc744 \uc758\ubbf8\ud55c\ub2e4. \uc989 \ud55c \ud30c\ub294 \uc758\ud654\ub2e8\uc744 \ubc31\ub828\uad50\uc640 \uac19\uc740 \uc0ac\uad50\uc5d0\uc11c \ub098\uc628 \uac83\uc774\ubbc0\ub85c \ud558\ub8e8\ube68\ub9ac \uc5c4\uaca9\ud558\uac8c \ud1a0\ubc8c\ud558\uc5ec \uc678\uad6d\uc778\ub4e4\uc774 \ubb34\ub825\uc744 \uc0ac\uc6a9\ud558\uc5ec \uac04\uc12d\ud558\ub824\ub294 \uad6c\uc2e4\uc744 \uc8fc\uc9c0\ub9d0\uc544\uc57c \ud55c\ub2e4\uace0 \uc8fc\uc7a5\ud558\uc600\ub2e4. \ub610 \ud55c \ud30c\ub294 \uc758\ud654\ub2e8\uc758 \uc870\uc9c1\uacfc \uadf8\ub4e4\uc774 \uc9c4\ud589\ud558\uace0 \uc788\ub294 \ubc18\uae30\ub3c5\uad50 \ud22c\uc7c1\uacfc\ub294 \uad6c\ubcc4\ud558\uc5ec \ubc18\uae30\ub3c5\uad50 \uc6b4\ub3d9\uc5d0 \ub300\ud558\uc5ec \uc5c4\uaca9\ud558\uac8c \ub2e4\uc2a4\ub824 \ub9cc\uc5f0\ub418\uc9c0 \ubabb\ud558\ub3c4\ub85d \ud558\uace0 \uadf8 \uc870\uc9c1\uc744 \ud5a5\ub2e8(\u9115\u5718)\uc73c\ub85c \ud3b8\uc131\ud558\uc5ec \uad00\uc758 \ud1b5\uc81c\ub97c \ubc1b\ub3c4\ub85d \ud558\uba74\uc11c \uc758\ud654\ub2e8\uc744 \uc774\uc6a9\ud558\uc790\ub294 \uc8fc\uc7a5\uc774\uc5c8\ub2e4. \ud2b9\ud788 \ud6c4\uc790\ub4e4\uc740 \uad11\uc11c\uc81c\ub97c \ube44\ud638\ud558\uace0 \uc788\ub294 \uc5f4\uac15\uc5d0 \ub300\ud56d\ud558\uace0, \uc11c\ud0dc\ud6c4\uc758 \ud6c8\uc815(\u8a13\u653f)\uc744 \uacc4\uc18d\ud560 \uc218 \uc788\uc744 \ubfd0\ub9cc \uc544\ub2c8\ub77c \ub2e4\ub978 \uc0ac\ub78c\uc744 \ud669\uc81c\ub85c \uc549\ud788\ub824\ub294 \uc11c\ud0dc\ud6c4\uc758 \ub73b\uc5d0 \uc601\ud569\ud558\ub824 \ud55c \uac83\uc774\ub2e4. \ubfd0\ub9cc \uc544\ub2c8\ub77c \uc758\ud654\ub2e8 \uc6b4\ub3d9\uc744 \uac16\uace0 \ubc18\uae30\ub3c5\uad50 \uc6b4\ub3d9\uc744 \uc77c\uc73c\ucf1c \uc591\ubb34\uc640 \ubcc0\ubc95\uc0ac\uc0c1\uc744 \uc804\ubc18\uc801\uc73c\ub85c \ubd80\uc815\ud558\ub824\ub294 \uc218\uad6c\uc801\uc778 \uc758\uc2dd\uc744 \uac16\uace0 \uc788\uc5c8\ub2e4."} {"question": "\ud45c\uc900\uc740 \uc815\ud655\ub3c4\ub97c \uc704\ud574 \uba87 \ub144\ub3c4 \ubc84\uc804\uc5d0\uc11c \uc6d4 \ud45c\uae30 \uc2dc \uc5f0\ub3c4 \uc0dd\ub7b5\uc744 \ud5c8\uc6a9\ud558\uc9c0 \uc54a\uc558\ub294\uac00?", "paragraph": "\ud45c\uc900\uc740 \uc815\ud655\ub3c4\ub97c \ub0ae\ucdb0 \uc791\uc131\ub41c \ub2ec\ub825 \ub0a0\uc9dc\ub3c4 \ud5c8\uc6a9\ud55c\ub2e4. \ud55c \uac00\uc9c0 \uc608\ub85c \"1981\ub144 4\uc6d4\"\uc740 \"1981-04\"\uac00 \ub41c\ub2e4. \"--04-05\" \uc791\uc131\uc744 \ud5c8\uc6a9\ud55c 2000\ub144 \ubc84\uc804\uc5d0\uc11c \uc774\ub294 \"4\uc6d4 5\uc77c\"\uc744 \ub73b\ud558\uc9c0\ub9cc 2004\ub144 \ubc84\uc804\uc5d0\uc11c\ub294 \uc6d4 \ud45c\uae30 \uc2dc \uc5f0\ub3c4\ub97c \uc0dd\ub7b5\uc774 \ud5c8\uc6a9\ub418\uc9c0 \uc54a\ub294\ub2e4. \ub610 \ud558\ub098\ub294 \uc5f0\ub3c4 \ucc38\uc870\ub97c \uc704\ud574 \uac04\ub2e8\ud788 \"1981\"\ub77c\uace0 \uc4f0\uba70 1900\ubd80\ud130 1999 \uc0ac\uc774\uc5d0\uc11c \uc138\uae30\ub97c \ucc38\uc870\ub97c \uc704\ud574 \"19\"\ub77c\uace0 \uc4f0\uae30\ub3c4 \ud55c\ub2e4. \ud45c\uc900\uc5d0\uc11c \uc644\ubcbd\ud55c \ub2ec\ub825 \ub0a0\uc9dc \ud45c\ud604\uc744 \uc704\ud574 YYYY-MM-DD\uc640 YYYYMMDD \ud615\uc2dd \ubaa8\ub450\ub97c \ud5c8\uc6a9\ud560\uc9c0\ub77c\ub3c4, \uc77c\uc790\uc778 DD\uac00 \uc0dd\ub7b5\ub418\uba74 YYYY-MM \ud615\uc2dd\ub9cc \ud5c8\uc6a9\ub41c\ub2e4. YYYYMM \ud615\uc2dd \ub0a0\uc9dc\ub97c \ud5c8\uc6a9\ud558\uc9c0 \uc54a\uae30\uc5d0, \ud45c\uc900\uc740 \uc77c\ubd80\ub97c \uc904\uc778 \ud45c\ud604(truncated representation)\uc778 YYMMDD\uc640\uc758 \ud63c\uc120\uc744 \ub9c9\ub294\ub2e4(\uc5ec\uc804\ud788 \uc790\uc8fc \uc0ac\uc6a9\ub41c\ub2e4).", "answer": "2004\ub144 \ubc84\uc804", "sentence": "\"--04-05\" \uc791\uc131\uc744 \ud5c8\uc6a9\ud55c 2000\ub144 \ubc84\uc804\uc5d0\uc11c \uc774\ub294 \"4\uc6d4 5\uc77c\"\uc744 \ub73b\ud558\uc9c0\ub9cc 2004\ub144 \ubc84\uc804 \uc5d0\uc11c\ub294 \uc6d4 \ud45c\uae30 \uc2dc \uc5f0\ub3c4\ub97c \uc0dd\ub7b5\uc774 \ud5c8\uc6a9\ub418\uc9c0 \uc54a\ub294\ub2e4.", "paragraph_sentence": "\ud45c\uc900\uc740 \uc815\ud655\ub3c4\ub97c \ub0ae\ucdb0 \uc791\uc131\ub41c \ub2ec\ub825 \ub0a0\uc9dc\ub3c4 \ud5c8\uc6a9\ud55c\ub2e4. \ud55c \uac00\uc9c0 \uc608\ub85c \"1981\ub144 4\uc6d4\"\uc740 \"1981-04\"\uac00 \ub41c\ub2e4. \"--04-05\" \uc791\uc131\uc744 \ud5c8\uc6a9\ud55c 2000\ub144 \ubc84\uc804\uc5d0\uc11c \uc774\ub294 \"4\uc6d4 5\uc77c\"\uc744 \ub73b\ud558\uc9c0\ub9cc 2004\ub144 \ubc84\uc804 \uc5d0\uc11c\ub294 \uc6d4 \ud45c\uae30 \uc2dc \uc5f0\ub3c4\ub97c \uc0dd\ub7b5\uc774 \ud5c8\uc6a9\ub418\uc9c0 \uc54a\ub294\ub2e4. \ub610 \ud558\ub098\ub294 \uc5f0\ub3c4 \ucc38\uc870\ub97c \uc704\ud574 \uac04\ub2e8\ud788 \"1981\"\ub77c\uace0 \uc4f0\uba70 1900\ubd80\ud130 1999 \uc0ac\uc774\uc5d0\uc11c \uc138\uae30\ub97c \ucc38\uc870\ub97c \uc704\ud574 \"19\"\ub77c\uace0 \uc4f0\uae30\ub3c4 \ud55c\ub2e4. \ud45c\uc900\uc5d0\uc11c \uc644\ubcbd\ud55c \ub2ec\ub825 \ub0a0\uc9dc \ud45c\ud604\uc744 \uc704\ud574 YYYY-MM-DD\uc640 YYYYMMDD \ud615\uc2dd \ubaa8\ub450\ub97c \ud5c8\uc6a9\ud560\uc9c0\ub77c\ub3c4, \uc77c\uc790\uc778 DD\uac00 \uc0dd\ub7b5\ub418\uba74 YYYY-MM \ud615\uc2dd\ub9cc \ud5c8\uc6a9\ub41c\ub2e4. YYYYMM \ud615\uc2dd \ub0a0\uc9dc\ub97c \ud5c8\uc6a9\ud558\uc9c0 \uc54a\uae30\uc5d0, \ud45c\uc900\uc740 \uc77c\ubd80\ub97c \uc904\uc778 \ud45c\ud604(truncated representation)\uc778 YYMMDD\uc640\uc758 \ud63c\uc120\uc744 \ub9c9\ub294\ub2e4(\uc5ec\uc804\ud788 \uc790\uc8fc \uc0ac\uc6a9\ub41c\ub2e4).", "paragraph_answer": "\ud45c\uc900\uc740 \uc815\ud655\ub3c4\ub97c \ub0ae\ucdb0 \uc791\uc131\ub41c \ub2ec\ub825 \ub0a0\uc9dc\ub3c4 \ud5c8\uc6a9\ud55c\ub2e4. \ud55c \uac00\uc9c0 \uc608\ub85c \"1981\ub144 4\uc6d4\"\uc740 \"1981-04\"\uac00 \ub41c\ub2e4. \"--04-05\" \uc791\uc131\uc744 \ud5c8\uc6a9\ud55c 2000\ub144 \ubc84\uc804\uc5d0\uc11c \uc774\ub294 \"4\uc6d4 5\uc77c\"\uc744 \ub73b\ud558\uc9c0\ub9cc 2004\ub144 \ubc84\uc804 \uc5d0\uc11c\ub294 \uc6d4 \ud45c\uae30 \uc2dc \uc5f0\ub3c4\ub97c \uc0dd\ub7b5\uc774 \ud5c8\uc6a9\ub418\uc9c0 \uc54a\ub294\ub2e4. \ub610 \ud558\ub098\ub294 \uc5f0\ub3c4 \ucc38\uc870\ub97c \uc704\ud574 \uac04\ub2e8\ud788 \"1981\"\ub77c\uace0 \uc4f0\uba70 1900\ubd80\ud130 1999 \uc0ac\uc774\uc5d0\uc11c \uc138\uae30\ub97c \ucc38\uc870\ub97c \uc704\ud574 \"19\"\ub77c\uace0 \uc4f0\uae30\ub3c4 \ud55c\ub2e4. \ud45c\uc900\uc5d0\uc11c \uc644\ubcbd\ud55c \ub2ec\ub825 \ub0a0\uc9dc \ud45c\ud604\uc744 \uc704\ud574 YYYY-MM-DD\uc640 YYYYMMDD \ud615\uc2dd \ubaa8\ub450\ub97c \ud5c8\uc6a9\ud560\uc9c0\ub77c\ub3c4, \uc77c\uc790\uc778 DD\uac00 \uc0dd\ub7b5\ub418\uba74 YYYY-MM \ud615\uc2dd\ub9cc \ud5c8\uc6a9\ub41c\ub2e4. YYYYMM \ud615\uc2dd \ub0a0\uc9dc\ub97c \ud5c8\uc6a9\ud558\uc9c0 \uc54a\uae30\uc5d0, \ud45c\uc900\uc740 \uc77c\ubd80\ub97c \uc904\uc778 \ud45c\ud604(truncated representation)\uc778 YYMMDD\uc640\uc758 \ud63c\uc120\uc744 \ub9c9\ub294\ub2e4(\uc5ec\uc804\ud788 \uc790\uc8fc \uc0ac\uc6a9\ub41c\ub2e4).", "sentence_answer": "\"--04-05\" \uc791\uc131\uc744 \ud5c8\uc6a9\ud55c 2000\ub144 \ubc84\uc804\uc5d0\uc11c \uc774\ub294 \"4\uc6d4 5\uc77c\"\uc744 \ub73b\ud558\uc9c0\ub9cc 2004\ub144 \ubc84\uc804 \uc5d0\uc11c\ub294 \uc6d4 \ud45c\uae30 \uc2dc \uc5f0\ub3c4\ub97c \uc0dd\ub7b5\uc774 \ud5c8\uc6a9\ub418\uc9c0 \uc54a\ub294\ub2e4.", "paragraph_id": "6494055-0-1", "paragraph_question": "question: \ud45c\uc900\uc740 \uc815\ud655\ub3c4\ub97c \uc704\ud574 \uba87 \ub144\ub3c4 \ubc84\uc804\uc5d0\uc11c \uc6d4 \ud45c\uae30 \uc2dc \uc5f0\ub3c4 \uc0dd\ub7b5\uc744 \ud5c8\uc6a9\ud558\uc9c0 \uc54a\uc558\ub294\uac00?, context: \ud45c\uc900\uc740 \uc815\ud655\ub3c4\ub97c \ub0ae\ucdb0 \uc791\uc131\ub41c \ub2ec\ub825 \ub0a0\uc9dc\ub3c4 \ud5c8\uc6a9\ud55c\ub2e4. \ud55c \uac00\uc9c0 \uc608\ub85c \"1981\ub144 4\uc6d4\"\uc740 \"1981-04\"\uac00 \ub41c\ub2e4. \"--04-05\" \uc791\uc131\uc744 \ud5c8\uc6a9\ud55c 2000\ub144 \ubc84\uc804\uc5d0\uc11c \uc774\ub294 \"4\uc6d4 5\uc77c\"\uc744 \ub73b\ud558\uc9c0\ub9cc 2004\ub144 \ubc84\uc804\uc5d0\uc11c\ub294 \uc6d4 \ud45c\uae30 \uc2dc \uc5f0\ub3c4\ub97c \uc0dd\ub7b5\uc774 \ud5c8\uc6a9\ub418\uc9c0 \uc54a\ub294\ub2e4. \ub610 \ud558\ub098\ub294 \uc5f0\ub3c4 \ucc38\uc870\ub97c \uc704\ud574 \uac04\ub2e8\ud788 \"1981\"\ub77c\uace0 \uc4f0\uba70 1900\ubd80\ud130 1999 \uc0ac\uc774\uc5d0\uc11c \uc138\uae30\ub97c \ucc38\uc870\ub97c \uc704\ud574 \"19\"\ub77c\uace0 \uc4f0\uae30\ub3c4 \ud55c\ub2e4. \ud45c\uc900\uc5d0\uc11c \uc644\ubcbd\ud55c \ub2ec\ub825 \ub0a0\uc9dc \ud45c\ud604\uc744 \uc704\ud574 YYYY-MM-DD\uc640 YYYYMMDD \ud615\uc2dd \ubaa8\ub450\ub97c \ud5c8\uc6a9\ud560\uc9c0\ub77c\ub3c4, \uc77c\uc790\uc778 DD\uac00 \uc0dd\ub7b5\ub418\uba74 YYYY-MM \ud615\uc2dd\ub9cc \ud5c8\uc6a9\ub41c\ub2e4. YYYYMM \ud615\uc2dd \ub0a0\uc9dc\ub97c \ud5c8\uc6a9\ud558\uc9c0 \uc54a\uae30\uc5d0, \ud45c\uc900\uc740 \uc77c\ubd80\ub97c \uc904\uc778 \ud45c\ud604(truncated representation)\uc778 YYMMDD\uc640\uc758 \ud63c\uc120\uc744 \ub9c9\ub294\ub2e4(\uc5ec\uc804\ud788 \uc790\uc8fc \uc0ac\uc6a9\ub41c\ub2e4)."} {"question": "\uae40\uc9c0\ud6c8\uc740 \ud22c\ud22c\uc758 \uba87\uc9d1\ubd80\ud130 \uba64\ubc84\ub85c \ud65c\ub3d9\ud558\uc600\ub294\uac00?", "paragraph": "\uc624\uc9c0\ud6c8, \uc720\ud604\uc7ac\uac00 \uc785\ub300\ub97c \ud558\uba74\uc11c \uae40\uc9c0\ud6c8\uc740 1995\ub144 6\uc6d4\uc5d0 \uac1d\uc6d0 \uba64\ubc84 \uae40\uc900, \uc784\uc131\uc740, \ud669\ud61c\uc601\uacfc \ud568\uaed8 \ud22c\ud22c 2\uc9d1\uc758 \uba64\ubc84\ub85c \ud65c\ub3d9\ud558\uc600\ub2e4. \ud22c\ud22c 2\uc9d1 \uc2dc\uae30\uc778 1995\ub144\uc5d0\ub294 \uadf8\ub8f9\uc758 \ub9ac\ub354\uc640 \ubcf4\uceec\ub85c\uc11c \ud65c\ub3d9\ud558\uc600\uace0, \uc5e0\ub137\uc758 '\uace0 \uc5e0\ub137 \uace0'\ub85c \uccab MC \ub370\ubdd4\ub97c \ud558\uac8c \ub418\uc5c8\ub2e4. (2\uc9d1 \uc218\ub85d\uace1 \uc911 \uc8fc\ub41c \ubc29\uc1a1 \ud65c\ub3d9\uace1\uc740 '\ubc14\ub78c\ub09c \uc5ec\uc790'\uc640 '\ub2c8\uac00 \ub0b4\uac83\uc774 \ub418\uac08\uc218\ub85d'\uc774\uba70 2\uc9d1 \ub178\ub798\uc758 \ub300\ubd80\ubd84\uc744 \uae40\uc9c0\ud6c8\uc774 \ubd88\ub800\ub2e4. \ub098\uba38\uc9c0 \uba64\ubc84\ub4e4\uc740 \uac1d\uc6d0 \uba64\ubc84 \ud615\uc2dd\uc73c\ub85c \uc568\ubc94 \ub2f9 1-2\uace1\uc5d0\ub9cc \ucc38\uc5ec\ud558\uc600\ub2e4. \ubc14\ub78c\ub09c \uc5ec\uc790\ub294 1\uc704 \ud6c4\ubcf4\uc5d0\ub3c4 \uc624\ub974\ub294 \ub4f1 \ucd5c\uc0c1\uc704\uad8c\uc744 \uc720\uc9c0\ud558\uc600\ub2e4. \uadf8\ub294 1996\ub144 1\uc6d4 5\uc77c\uc790 MBC \uc778\uae30\uac00\uc694 BEST 50\uc5d0\uc11c '\ub2c8\uac00 \ub0b4\uac83\uc774 \ub418\uac08\uc218\ub85d'\uc758 \ubb34\ub300 \ub97c \ub05d\uc73c\ub85c \ud22c\ud22c \ud65c\ub3d9\uc744 \ub9c8\uce58\uace0 \uad70\ub300\uc5d0 \uc785\ub300\ud558\uac8c \ub418\uc5c8\ub2e4. \ud65c\ub3d9 \uc911\uc5d0 \uc785\ub300\ud55c\ud130\ub77c \uadf8\uc758 \uc785\ub300 \ud6c4\uc5d0\ub3c4 \ub2c8\uac00 \ub0b4\uac83\uc774 \ub418\uac08\uc218\ub85d\uc740 \ud55c\ub3d9\uc548 \uac00\uc694\ud1b110\uc5d0 \uc21c\uc704\uad8c\uc5d0 \uba38\ubb3c\ub800\ub2e4.", "answer": "2\uc9d1", "sentence": "1995\ub144 6\uc6d4\uc5d0 \uac1d\uc6d0 \uba64\ubc84 \uae40\uc900, \uc784\uc131\uc740, \ud669\ud61c\uc601\uacfc \ud568\uaed8 \ud22c\ud22c 2\uc9d1 \uc758 \uba64\ubc84\ub85c \ud65c\ub3d9\ud558\uc600\ub2e4.", "paragraph_sentence": "\uc624\uc9c0\ud6c8, \uc720\ud604\uc7ac\uac00 \uc785\ub300\ub97c \ud558\uba74\uc11c \uae40\uc9c0\ud6c8\uc740 1995\ub144 6\uc6d4\uc5d0 \uac1d\uc6d0 \uba64\ubc84 \uae40\uc900, \uc784\uc131\uc740, \ud669\ud61c\uc601\uacfc \ud568\uaed8 \ud22c\ud22c 2\uc9d1 \uc758 \uba64\ubc84\ub85c \ud65c\ub3d9\ud558\uc600\ub2e4. \ud22c\ud22c 2\uc9d1 \uc2dc\uae30\uc778 1995\ub144\uc5d0\ub294 \uadf8\ub8f9\uc758 \ub9ac\ub354\uc640 \ubcf4\uceec\ub85c\uc11c \ud65c\ub3d9\ud558\uc600\uace0, \uc5e0\ub137\uc758 '\uace0 \uc5e0\ub137 \uace0'\ub85c \uccab MC \ub370\ubdd4\ub97c \ud558\uac8c \ub418\uc5c8\ub2e4. (2\uc9d1 \uc218\ub85d\uace1 \uc911 \uc8fc\ub41c \ubc29\uc1a1 \ud65c\ub3d9\uace1\uc740 '\ubc14\ub78c\ub09c \uc5ec\uc790'\uc640 '\ub2c8\uac00 \ub0b4\uac83\uc774 \ub418\uac08\uc218\ub85d'\uc774\uba70 2\uc9d1 \ub178\ub798\uc758 \ub300\ubd80\ubd84\uc744 \uae40\uc9c0\ud6c8\uc774 \ubd88\ub800\ub2e4. \ub098\uba38\uc9c0 \uba64\ubc84\ub4e4\uc740 \uac1d\uc6d0 \uba64\ubc84 \ud615\uc2dd\uc73c\ub85c \uc568\ubc94 \ub2f9 1-2\uace1\uc5d0\ub9cc \ucc38\uc5ec\ud558\uc600\ub2e4. \ubc14\ub78c\ub09c \uc5ec\uc790\ub294 1\uc704 \ud6c4\ubcf4\uc5d0\ub3c4 \uc624\ub974\ub294 \ub4f1 \ucd5c\uc0c1\uc704\uad8c\uc744 \uc720\uc9c0\ud558\uc600\ub2e4. \uadf8\ub294 1996\ub144 1\uc6d4 5\uc77c\uc790 MBC \uc778\uae30\uac00\uc694 BEST 50\uc5d0\uc11c '\ub2c8\uac00 \ub0b4\uac83\uc774 \ub418\uac08\uc218\ub85d'\uc758 \ubb34\ub300 \ub97c \ub05d\uc73c\ub85c \ud22c\ud22c \ud65c\ub3d9\uc744 \ub9c8\uce58\uace0 \uad70\ub300\uc5d0 \uc785\ub300\ud558\uac8c \ub418\uc5c8\ub2e4. \ud65c\ub3d9 \uc911\uc5d0 \uc785\ub300\ud55c\ud130\ub77c \uadf8\uc758 \uc785\ub300 \ud6c4\uc5d0\ub3c4 \ub2c8\uac00 \ub0b4\uac83\uc774 \ub418\uac08\uc218\ub85d\uc740 \ud55c\ub3d9\uc548 \uac00\uc694\ud1b110\uc5d0 \uc21c\uc704\uad8c\uc5d0 \uba38\ubb3c\ub800\ub2e4.", "paragraph_answer": "\uc624\uc9c0\ud6c8, \uc720\ud604\uc7ac\uac00 \uc785\ub300\ub97c \ud558\uba74\uc11c \uae40\uc9c0\ud6c8\uc740 1995\ub144 6\uc6d4\uc5d0 \uac1d\uc6d0 \uba64\ubc84 \uae40\uc900, \uc784\uc131\uc740, \ud669\ud61c\uc601\uacfc \ud568\uaed8 \ud22c\ud22c 2\uc9d1 \uc758 \uba64\ubc84\ub85c \ud65c\ub3d9\ud558\uc600\ub2e4. \ud22c\ud22c 2\uc9d1 \uc2dc\uae30\uc778 1995\ub144\uc5d0\ub294 \uadf8\ub8f9\uc758 \ub9ac\ub354\uc640 \ubcf4\uceec\ub85c\uc11c \ud65c\ub3d9\ud558\uc600\uace0, \uc5e0\ub137\uc758 '\uace0 \uc5e0\ub137 \uace0'\ub85c \uccab MC \ub370\ubdd4\ub97c \ud558\uac8c \ub418\uc5c8\ub2e4. (2\uc9d1 \uc218\ub85d\uace1 \uc911 \uc8fc\ub41c \ubc29\uc1a1 \ud65c\ub3d9\uace1\uc740 '\ubc14\ub78c\ub09c \uc5ec\uc790'\uc640 '\ub2c8\uac00 \ub0b4\uac83\uc774 \ub418\uac08\uc218\ub85d'\uc774\uba70 2\uc9d1 \ub178\ub798\uc758 \ub300\ubd80\ubd84\uc744 \uae40\uc9c0\ud6c8\uc774 \ubd88\ub800\ub2e4. \ub098\uba38\uc9c0 \uba64\ubc84\ub4e4\uc740 \uac1d\uc6d0 \uba64\ubc84 \ud615\uc2dd\uc73c\ub85c \uc568\ubc94 \ub2f9 1-2\uace1\uc5d0\ub9cc \ucc38\uc5ec\ud558\uc600\ub2e4. \ubc14\ub78c\ub09c \uc5ec\uc790\ub294 1\uc704 \ud6c4\ubcf4\uc5d0\ub3c4 \uc624\ub974\ub294 \ub4f1 \ucd5c\uc0c1\uc704\uad8c\uc744 \uc720\uc9c0\ud558\uc600\ub2e4. \uadf8\ub294 1996\ub144 1\uc6d4 5\uc77c\uc790 MBC \uc778\uae30\uac00\uc694 BEST 50\uc5d0\uc11c '\ub2c8\uac00 \ub0b4\uac83\uc774 \ub418\uac08\uc218\ub85d'\uc758 \ubb34\ub300 \ub97c \ub05d\uc73c\ub85c \ud22c\ud22c \ud65c\ub3d9\uc744 \ub9c8\uce58\uace0 \uad70\ub300\uc5d0 \uc785\ub300\ud558\uac8c \ub418\uc5c8\ub2e4. \ud65c\ub3d9 \uc911\uc5d0 \uc785\ub300\ud55c\ud130\ub77c \uadf8\uc758 \uc785\ub300 \ud6c4\uc5d0\ub3c4 \ub2c8\uac00 \ub0b4\uac83\uc774 \ub418\uac08\uc218\ub85d\uc740 \ud55c\ub3d9\uc548 \uac00\uc694\ud1b110\uc5d0 \uc21c\uc704\uad8c\uc5d0 \uba38\ubb3c\ub800\ub2e4.", "sentence_answer": "1995\ub144 6\uc6d4\uc5d0 \uac1d\uc6d0 \uba64\ubc84 \uae40\uc900, \uc784\uc131\uc740, \ud669\ud61c\uc601\uacfc \ud568\uaed8 \ud22c\ud22c 2\uc9d1 \uc758 \uba64\ubc84\ub85c \ud65c\ub3d9\ud558\uc600\ub2e4.", "paragraph_id": "6488559-1-0", "paragraph_question": "question: \uae40\uc9c0\ud6c8\uc740 \ud22c\ud22c\uc758 \uba87\uc9d1\ubd80\ud130 \uba64\ubc84\ub85c \ud65c\ub3d9\ud558\uc600\ub294\uac00?, context: \uc624\uc9c0\ud6c8, \uc720\ud604\uc7ac\uac00 \uc785\ub300\ub97c \ud558\uba74\uc11c \uae40\uc9c0\ud6c8\uc740 1995\ub144 6\uc6d4\uc5d0 \uac1d\uc6d0 \uba64\ubc84 \uae40\uc900, \uc784\uc131\uc740, \ud669\ud61c\uc601\uacfc \ud568\uaed8 \ud22c\ud22c 2\uc9d1\uc758 \uba64\ubc84\ub85c \ud65c\ub3d9\ud558\uc600\ub2e4. \ud22c\ud22c 2\uc9d1 \uc2dc\uae30\uc778 1995\ub144\uc5d0\ub294 \uadf8\ub8f9\uc758 \ub9ac\ub354\uc640 \ubcf4\uceec\ub85c\uc11c \ud65c\ub3d9\ud558\uc600\uace0, \uc5e0\ub137\uc758 '\uace0 \uc5e0\ub137 \uace0'\ub85c \uccab MC \ub370\ubdd4\ub97c \ud558\uac8c \ub418\uc5c8\ub2e4. (2\uc9d1 \uc218\ub85d\uace1 \uc911 \uc8fc\ub41c \ubc29\uc1a1 \ud65c\ub3d9\uace1\uc740 '\ubc14\ub78c\ub09c \uc5ec\uc790'\uc640 '\ub2c8\uac00 \ub0b4\uac83\uc774 \ub418\uac08\uc218\ub85d'\uc774\uba70 2\uc9d1 \ub178\ub798\uc758 \ub300\ubd80\ubd84\uc744 \uae40\uc9c0\ud6c8\uc774 \ubd88\ub800\ub2e4. \ub098\uba38\uc9c0 \uba64\ubc84\ub4e4\uc740 \uac1d\uc6d0 \uba64\ubc84 \ud615\uc2dd\uc73c\ub85c \uc568\ubc94 \ub2f9 1-2\uace1\uc5d0\ub9cc \ucc38\uc5ec\ud558\uc600\ub2e4. \ubc14\ub78c\ub09c \uc5ec\uc790\ub294 1\uc704 \ud6c4\ubcf4\uc5d0\ub3c4 \uc624\ub974\ub294 \ub4f1 \ucd5c\uc0c1\uc704\uad8c\uc744 \uc720\uc9c0\ud558\uc600\ub2e4. \uadf8\ub294 1996\ub144 1\uc6d4 5\uc77c\uc790 MBC \uc778\uae30\uac00\uc694 BEST 50\uc5d0\uc11c '\ub2c8\uac00 \ub0b4\uac83\uc774 \ub418\uac08\uc218\ub85d'\uc758 \ubb34\ub300 \ub97c \ub05d\uc73c\ub85c \ud22c\ud22c \ud65c\ub3d9\uc744 \ub9c8\uce58\uace0 \uad70\ub300\uc5d0 \uc785\ub300\ud558\uac8c \ub418\uc5c8\ub2e4. \ud65c\ub3d9 \uc911\uc5d0 \uc785\ub300\ud55c\ud130\ub77c \uadf8\uc758 \uc785\ub300 \ud6c4\uc5d0\ub3c4 \ub2c8\uac00 \ub0b4\uac83\uc774 \ub418\uac08\uc218\ub85d\uc740 \ud55c\ub3d9\uc548 \uac00\uc694\ud1b110\uc5d0 \uc21c\uc704\uad8c\uc5d0 \uba38\ubb3c\ub800\ub2e4."} {"question": "\ub291\ub300\uac00 \uc720\uae30\uacac\uc758 \uac1c\uccb4\uc218\ub97c \uc81c\ud55c\ud558\ub294 \uc5ed\ud560\uc744 \ud558\ub294 \uacf3\uc740 \uc5b4\ub514\uc778\uac00?", "paragraph": "\ub291\ub300\ub294 \uacbd\uc6b0\uc5d0\ub294 \uac1c\ub97c \uc8fd\uc774\uba74\uc11c \uc911\uc694\ud55c \uc74c\uc2dd \uacf5\uae09\uc73c\ub85c \uac1c\uc5d0 \uc758\uc874\ud558\ub294 \uc77c\ubd80 \uac1c\uccb4\uac00 \uc788\ub2e4. \uc77c\ubc18\uc801\uc73c\ub85c \ub291\ub300\ub294 \ud06c\uae30\uc5d0\uc11c \uac1c\ub97c \ub2a5\uac00\ud558\uae30 \ub54c\ubb38\uc5d0 \ubb3c\ub9ac\uc801 \uacf5\uaca9\uc5d0\uc11c \uac1c\ub294 \uc77c\ub2e8 \ub291\ub300\uc758 \ud070 \uba38\ub9ac\uc758 \uc774\ube68\uc5d0 \ubb3c\ub9ac\uac8c \ub41c\ub2e4. \ub610\ud55c, \ub291\ub300\uc640 \uac1c\uac00 \uc2f8\uc6b0\ub294 \uc2a4\ud0c0\uc77c\uc740 \ub2e4\ub974\ub2e4. \uac1c\ub294 \uc77c\ubc18\uc801\uc73c\ub85c \uacf5\uaca9\ud560 \uc2dc \uba38\ub9ac, \ubaa9 \ubc0f \uc5b4\uae68 \ubd80\uadfc\uc73c\ub85c \uc81c\ud55c\ub418\uc9c0\ub9cc \ub291\ub300\ub294 \uc0c1\ub300\uc758 \ubab8\uacfc \uc0ac\uc9c0\ub97c \ubb38\ub2e4. \ud06c\ub85c\uc544\ud2f0\uc544\uc5d0\uc11c\ub294 \ub291\ub300\uac00 \uc591\ubcf4\ub2e4 \uac1c\ub97c \ub354\uc6b1 \ub9ce\uc774 \uc8fd\uc774\uace0 \ub7ec\uc2dc\uc544\uc5d0\uc11c\ub294 \ub291\ub300\uac00 \uc720\uae30\uacac\uc758 \uac1c\uccb4\uc218\ub97c \uc81c\ud55c\ud558\ub294 \uc5ed\ud560\uc744 \ud55c\ub2e4. \uac1c\uac00 \uc0ac\ub78c\ub4e4\uacfc \ud568\uaed8 \uacf5\uaca9\ud560 \ub54c, \ub291\ub300\ub294 \ube44\uc815\uc0c1\uc801\uc778 \ub300\ub2f4\ud55c \ud589\ub3d9\uc744 \ud558\uba70 \ub54c\ub85c\ub294 \uac00\uae4c\uc6b4 \uc0ac\ub78c\uc744 \ubb34\uc2dc\ud558\uae30\ub3c4 \ud55c\ub2e4. \ub291\ub300\uc758 \uac1c \uacf5\uaca9\uc740 \uc9d1\uc758 \ub4b7\ub9c8\ub2f9\uc774\ub098 \uc232 \ubaa8\ub450\uc5d0\uc11c \uc77c\uc5b4\ub0a0 \uc218 \uc788\ub2e4. \ub9c8\uc744 \uc678\uacfd\uc5d0\uc11c\ub294 \ub291\ub300\uac00 \uac1c\ub97c \uae30\ub2e4\ub9ac\uae30 \uc704\ud574 \ub9e4\ubcf5\ud558\uba70 \uadf8 \ub4a4\uc5d0 \ub2e4\ub978 \ub291\ub300\ub4e4\ub85c \ud558\uc5ec\uae08 \uac1c\ub97c \uc720\ud639\ud558\uac8c \ud558\uae30\ub3c4 \ud55c\ub2e4. \ub291\ub300\uc758 \uc0ac\ub0e5\uac1c \uacf5\uaca9\uc740 \uc2a4\uce78\ub514\ub098\ube44\uc544\uc640 \uc704\uc2a4\ucf58\uc2e0 \uc8fc\uc5d0\uc11c \ud070 \ubb38\uc81c\ub85c \ub300\ub450\ub41c\ub2e4. \uc2a4\uce78\ub514\ub098\ube44\uc544\uc5d0\uc11c \uac00\uc7a5 \ube48\ubc88\ud558\uac8c \uc8fd\uc740 \uc0ac\ub0e5\uac1c\ub294 \ud574\ub9ac\uc5b4 \ud488\uc885\uc73c\ub85c \ub299\uc740 \ub3d9\ubb3c\uc774 \uc5b4\ub9b0 \ub3d9\ubb3c\ubcf4\ub2e4 \uc18c\uc2ec\ud558\uc5ec \ub291\ub300\uc5d0 \ub300\ud55c \ubc18\uc751\uc744 \ub2e4\ub974\uac8c \ud560 \uac00\ub2a5\uc131\uc774 \uc788\uae30 \ub54c\ubb38\uc5d0 \uc704\ud5d8\ud558\ub2e4. \uac1c\uc758 \ub291\ub300\ub85c \uc778\ud55c \ubd80\uc0c1\uc740 \uc885\uc885 \ud5c8\ubc85\uc9c0\uc640 \ub4b7\ub2e4\ub9ac\uc5d0 \ub098\ud0c0\ub09c\ub2e4. \uce58\uba85\uc801\uc778 \uc0c1\ucc98\ub294 \ubcf4\ud1b5 \ubaa9 \ub4a4\ub97c \ubb3c\ub9b0 \uc0c1\ucc98\uc774\ub2e4. \uc2a4\uc6e8\ub374 \uc7d8\ud230\ub4dc(J\u00e4mthund) \uac19\uc740 \ub300\ud615 \uac1c\uc758 \uc0ac\ub0e5\uc740 \uc790\uc2e0\uc758 \ubc29\uc5b4\ud558\uae30 \ub354\uc6b1 \uc218\uc6d4\ud558\uae30 \ub54c\ubb38\uc5d0 \uc0b4\uc544\ub0a8\uc744 \uac00\ub2a5\uc131\uc774 \ub354\uc6b1 \ub192\ub2e4.", "answer": "\ub7ec\uc2dc\uc544", "sentence": "\ud06c\ub85c\uc544\ud2f0\uc544\uc5d0\uc11c\ub294 \ub291\ub300\uac00 \uc591\ubcf4\ub2e4 \uac1c\ub97c \ub354\uc6b1 \ub9ce\uc774 \uc8fd\uc774\uace0 \ub7ec\uc2dc\uc544 \uc5d0\uc11c\ub294 \ub291\ub300\uac00 \uc720\uae30\uacac\uc758 \uac1c\uccb4\uc218\ub97c \uc81c\ud55c\ud558\ub294 \uc5ed\ud560\uc744 \ud55c\ub2e4.", "paragraph_sentence": "\ub291\ub300\ub294 \uacbd\uc6b0\uc5d0\ub294 \uac1c\ub97c \uc8fd\uc774\uba74\uc11c \uc911\uc694\ud55c \uc74c\uc2dd \uacf5\uae09\uc73c\ub85c \uac1c\uc5d0 \uc758\uc874\ud558\ub294 \uc77c\ubd80 \uac1c\uccb4\uac00 \uc788\ub2e4. \uc77c\ubc18\uc801\uc73c\ub85c \ub291\ub300\ub294 \ud06c\uae30\uc5d0\uc11c \uac1c\ub97c \ub2a5\uac00\ud558\uae30 \ub54c\ubb38\uc5d0 \ubb3c\ub9ac\uc801 \uacf5\uaca9\uc5d0\uc11c \uac1c\ub294 \uc77c\ub2e8 \ub291\ub300\uc758 \ud070 \uba38\ub9ac\uc758 \uc774\ube68\uc5d0 \ubb3c\ub9ac\uac8c \ub41c\ub2e4. \ub610\ud55c, \ub291\ub300\uc640 \uac1c\uac00 \uc2f8\uc6b0\ub294 \uc2a4\ud0c0\uc77c\uc740 \ub2e4\ub974\ub2e4. \uac1c\ub294 \uc77c\ubc18\uc801\uc73c\ub85c \uacf5\uaca9\ud560 \uc2dc \uba38\ub9ac, \ubaa9 \ubc0f \uc5b4\uae68 \ubd80\uadfc\uc73c\ub85c \uc81c\ud55c\ub418\uc9c0\ub9cc \ub291\ub300\ub294 \uc0c1\ub300\uc758 \ubab8\uacfc \uc0ac\uc9c0\ub97c \ubb38\ub2e4. \ud06c\ub85c\uc544\ud2f0\uc544\uc5d0\uc11c\ub294 \ub291\ub300\uac00 \uc591\ubcf4\ub2e4 \uac1c\ub97c \ub354\uc6b1 \ub9ce\uc774 \uc8fd\uc774\uace0 \ub7ec\uc2dc\uc544 \uc5d0\uc11c\ub294 \ub291\ub300\uac00 \uc720\uae30\uacac\uc758 \uac1c\uccb4\uc218\ub97c \uc81c\ud55c\ud558\ub294 \uc5ed\ud560\uc744 \ud55c\ub2e4. \uac1c\uac00 \uc0ac\ub78c\ub4e4\uacfc \ud568\uaed8 \uacf5\uaca9\ud560 \ub54c, \ub291\ub300\ub294 \ube44\uc815\uc0c1\uc801\uc778 \ub300\ub2f4\ud55c \ud589\ub3d9\uc744 \ud558\uba70 \ub54c\ub85c\ub294 \uac00\uae4c\uc6b4 \uc0ac\ub78c\uc744 \ubb34\uc2dc\ud558\uae30\ub3c4 \ud55c\ub2e4. \ub291\ub300\uc758 \uac1c \uacf5\uaca9\uc740 \uc9d1\uc758 \ub4b7\ub9c8\ub2f9\uc774\ub098 \uc232 \ubaa8\ub450\uc5d0\uc11c \uc77c\uc5b4\ub0a0 \uc218 \uc788\ub2e4. \ub9c8\uc744 \uc678\uacfd\uc5d0\uc11c\ub294 \ub291\ub300\uac00 \uac1c\ub97c \uae30\ub2e4\ub9ac\uae30 \uc704\ud574 \ub9e4\ubcf5\ud558\uba70 \uadf8 \ub4a4\uc5d0 \ub2e4\ub978 \ub291\ub300\ub4e4\ub85c \ud558\uc5ec\uae08 \uac1c\ub97c \uc720\ud639\ud558\uac8c \ud558\uae30\ub3c4 \ud55c\ub2e4. \ub291\ub300\uc758 \uc0ac\ub0e5\uac1c \uacf5\uaca9\uc740 \uc2a4\uce78\ub514\ub098\ube44\uc544\uc640 \uc704\uc2a4\ucf58\uc2e0 \uc8fc\uc5d0\uc11c \ud070 \ubb38\uc81c\ub85c \ub300\ub450\ub41c\ub2e4. \uc2a4\uce78\ub514\ub098\ube44\uc544\uc5d0\uc11c \uac00\uc7a5 \ube48\ubc88\ud558\uac8c \uc8fd\uc740 \uc0ac\ub0e5\uac1c\ub294 \ud574\ub9ac\uc5b4 \ud488\uc885\uc73c\ub85c \ub299\uc740 \ub3d9\ubb3c\uc774 \uc5b4\ub9b0 \ub3d9\ubb3c\ubcf4\ub2e4 \uc18c\uc2ec\ud558\uc5ec \ub291\ub300\uc5d0 \ub300\ud55c \ubc18\uc751\uc744 \ub2e4\ub974\uac8c \ud560 \uac00\ub2a5\uc131\uc774 \uc788\uae30 \ub54c\ubb38\uc5d0 \uc704\ud5d8\ud558\ub2e4. \uac1c\uc758 \ub291\ub300\ub85c \uc778\ud55c \ubd80\uc0c1\uc740 \uc885\uc885 \ud5c8\ubc85\uc9c0\uc640 \ub4b7\ub2e4\ub9ac\uc5d0 \ub098\ud0c0\ub09c\ub2e4. \uce58\uba85\uc801\uc778 \uc0c1\ucc98\ub294 \ubcf4\ud1b5 \ubaa9 \ub4a4\ub97c \ubb3c\ub9b0 \uc0c1\ucc98\uc774\ub2e4. \uc2a4\uc6e8\ub374 \uc7d8\ud230\ub4dc(J\u00e4mthund) \uac19\uc740 \ub300\ud615 \uac1c\uc758 \uc0ac\ub0e5\uc740 \uc790\uc2e0\uc758 \ubc29\uc5b4\ud558\uae30 \ub354\uc6b1 \uc218\uc6d4\ud558\uae30 \ub54c\ubb38\uc5d0 \uc0b4\uc544\ub0a8\uc744 \uac00\ub2a5\uc131\uc774 \ub354\uc6b1 \ub192\ub2e4.", "paragraph_answer": "\ub291\ub300\ub294 \uacbd\uc6b0\uc5d0\ub294 \uac1c\ub97c \uc8fd\uc774\uba74\uc11c \uc911\uc694\ud55c \uc74c\uc2dd \uacf5\uae09\uc73c\ub85c \uac1c\uc5d0 \uc758\uc874\ud558\ub294 \uc77c\ubd80 \uac1c\uccb4\uac00 \uc788\ub2e4. \uc77c\ubc18\uc801\uc73c\ub85c \ub291\ub300\ub294 \ud06c\uae30\uc5d0\uc11c \uac1c\ub97c \ub2a5\uac00\ud558\uae30 \ub54c\ubb38\uc5d0 \ubb3c\ub9ac\uc801 \uacf5\uaca9\uc5d0\uc11c \uac1c\ub294 \uc77c\ub2e8 \ub291\ub300\uc758 \ud070 \uba38\ub9ac\uc758 \uc774\ube68\uc5d0 \ubb3c\ub9ac\uac8c \ub41c\ub2e4. \ub610\ud55c, \ub291\ub300\uc640 \uac1c\uac00 \uc2f8\uc6b0\ub294 \uc2a4\ud0c0\uc77c\uc740 \ub2e4\ub974\ub2e4. \uac1c\ub294 \uc77c\ubc18\uc801\uc73c\ub85c \uacf5\uaca9\ud560 \uc2dc \uba38\ub9ac, \ubaa9 \ubc0f \uc5b4\uae68 \ubd80\uadfc\uc73c\ub85c \uc81c\ud55c\ub418\uc9c0\ub9cc \ub291\ub300\ub294 \uc0c1\ub300\uc758 \ubab8\uacfc \uc0ac\uc9c0\ub97c \ubb38\ub2e4. \ud06c\ub85c\uc544\ud2f0\uc544\uc5d0\uc11c\ub294 \ub291\ub300\uac00 \uc591\ubcf4\ub2e4 \uac1c\ub97c \ub354\uc6b1 \ub9ce\uc774 \uc8fd\uc774\uace0 \ub7ec\uc2dc\uc544 \uc5d0\uc11c\ub294 \ub291\ub300\uac00 \uc720\uae30\uacac\uc758 \uac1c\uccb4\uc218\ub97c \uc81c\ud55c\ud558\ub294 \uc5ed\ud560\uc744 \ud55c\ub2e4. \uac1c\uac00 \uc0ac\ub78c\ub4e4\uacfc \ud568\uaed8 \uacf5\uaca9\ud560 \ub54c, \ub291\ub300\ub294 \ube44\uc815\uc0c1\uc801\uc778 \ub300\ub2f4\ud55c \ud589\ub3d9\uc744 \ud558\uba70 \ub54c\ub85c\ub294 \uac00\uae4c\uc6b4 \uc0ac\ub78c\uc744 \ubb34\uc2dc\ud558\uae30\ub3c4 \ud55c\ub2e4. \ub291\ub300\uc758 \uac1c \uacf5\uaca9\uc740 \uc9d1\uc758 \ub4b7\ub9c8\ub2f9\uc774\ub098 \uc232 \ubaa8\ub450\uc5d0\uc11c \uc77c\uc5b4\ub0a0 \uc218 \uc788\ub2e4. \ub9c8\uc744 \uc678\uacfd\uc5d0\uc11c\ub294 \ub291\ub300\uac00 \uac1c\ub97c \uae30\ub2e4\ub9ac\uae30 \uc704\ud574 \ub9e4\ubcf5\ud558\uba70 \uadf8 \ub4a4\uc5d0 \ub2e4\ub978 \ub291\ub300\ub4e4\ub85c \ud558\uc5ec\uae08 \uac1c\ub97c \uc720\ud639\ud558\uac8c \ud558\uae30\ub3c4 \ud55c\ub2e4. \ub291\ub300\uc758 \uc0ac\ub0e5\uac1c \uacf5\uaca9\uc740 \uc2a4\uce78\ub514\ub098\ube44\uc544\uc640 \uc704\uc2a4\ucf58\uc2e0 \uc8fc\uc5d0\uc11c \ud070 \ubb38\uc81c\ub85c \ub300\ub450\ub41c\ub2e4. \uc2a4\uce78\ub514\ub098\ube44\uc544\uc5d0\uc11c \uac00\uc7a5 \ube48\ubc88\ud558\uac8c \uc8fd\uc740 \uc0ac\ub0e5\uac1c\ub294 \ud574\ub9ac\uc5b4 \ud488\uc885\uc73c\ub85c \ub299\uc740 \ub3d9\ubb3c\uc774 \uc5b4\ub9b0 \ub3d9\ubb3c\ubcf4\ub2e4 \uc18c\uc2ec\ud558\uc5ec \ub291\ub300\uc5d0 \ub300\ud55c \ubc18\uc751\uc744 \ub2e4\ub974\uac8c \ud560 \uac00\ub2a5\uc131\uc774 \uc788\uae30 \ub54c\ubb38\uc5d0 \uc704\ud5d8\ud558\ub2e4. \uac1c\uc758 \ub291\ub300\ub85c \uc778\ud55c \ubd80\uc0c1\uc740 \uc885\uc885 \ud5c8\ubc85\uc9c0\uc640 \ub4b7\ub2e4\ub9ac\uc5d0 \ub098\ud0c0\ub09c\ub2e4. \uce58\uba85\uc801\uc778 \uc0c1\ucc98\ub294 \ubcf4\ud1b5 \ubaa9 \ub4a4\ub97c \ubb3c\ub9b0 \uc0c1\ucc98\uc774\ub2e4. \uc2a4\uc6e8\ub374 \uc7d8\ud230\ub4dc(J\u00e4mthund) \uac19\uc740 \ub300\ud615 \uac1c\uc758 \uc0ac\ub0e5\uc740 \uc790\uc2e0\uc758 \ubc29\uc5b4\ud558\uae30 \ub354\uc6b1 \uc218\uc6d4\ud558\uae30 \ub54c\ubb38\uc5d0 \uc0b4\uc544\ub0a8\uc744 \uac00\ub2a5\uc131\uc774 \ub354\uc6b1 \ub192\ub2e4.", "sentence_answer": "\ud06c\ub85c\uc544\ud2f0\uc544\uc5d0\uc11c\ub294 \ub291\ub300\uac00 \uc591\ubcf4\ub2e4 \uac1c\ub97c \ub354\uc6b1 \ub9ce\uc774 \uc8fd\uc774\uace0 \ub7ec\uc2dc\uc544 \uc5d0\uc11c\ub294 \ub291\ub300\uac00 \uc720\uae30\uacac\uc758 \uac1c\uccb4\uc218\ub97c \uc81c\ud55c\ud558\ub294 \uc5ed\ud560\uc744 \ud55c\ub2e4.", "paragraph_id": "6572454-49-1", "paragraph_question": "question: \ub291\ub300\uac00 \uc720\uae30\uacac\uc758 \uac1c\uccb4\uc218\ub97c \uc81c\ud55c\ud558\ub294 \uc5ed\ud560\uc744 \ud558\ub294 \uacf3\uc740 \uc5b4\ub514\uc778\uac00?, context: \ub291\ub300\ub294 \uacbd\uc6b0\uc5d0\ub294 \uac1c\ub97c \uc8fd\uc774\uba74\uc11c \uc911\uc694\ud55c \uc74c\uc2dd \uacf5\uae09\uc73c\ub85c \uac1c\uc5d0 \uc758\uc874\ud558\ub294 \uc77c\ubd80 \uac1c\uccb4\uac00 \uc788\ub2e4. \uc77c\ubc18\uc801\uc73c\ub85c \ub291\ub300\ub294 \ud06c\uae30\uc5d0\uc11c \uac1c\ub97c \ub2a5\uac00\ud558\uae30 \ub54c\ubb38\uc5d0 \ubb3c\ub9ac\uc801 \uacf5\uaca9\uc5d0\uc11c \uac1c\ub294 \uc77c\ub2e8 \ub291\ub300\uc758 \ud070 \uba38\ub9ac\uc758 \uc774\ube68\uc5d0 \ubb3c\ub9ac\uac8c \ub41c\ub2e4. \ub610\ud55c, \ub291\ub300\uc640 \uac1c\uac00 \uc2f8\uc6b0\ub294 \uc2a4\ud0c0\uc77c\uc740 \ub2e4\ub974\ub2e4. \uac1c\ub294 \uc77c\ubc18\uc801\uc73c\ub85c \uacf5\uaca9\ud560 \uc2dc \uba38\ub9ac, \ubaa9 \ubc0f \uc5b4\uae68 \ubd80\uadfc\uc73c\ub85c \uc81c\ud55c\ub418\uc9c0\ub9cc \ub291\ub300\ub294 \uc0c1\ub300\uc758 \ubab8\uacfc \uc0ac\uc9c0\ub97c \ubb38\ub2e4. \ud06c\ub85c\uc544\ud2f0\uc544\uc5d0\uc11c\ub294 \ub291\ub300\uac00 \uc591\ubcf4\ub2e4 \uac1c\ub97c \ub354\uc6b1 \ub9ce\uc774 \uc8fd\uc774\uace0 \ub7ec\uc2dc\uc544\uc5d0\uc11c\ub294 \ub291\ub300\uac00 \uc720\uae30\uacac\uc758 \uac1c\uccb4\uc218\ub97c \uc81c\ud55c\ud558\ub294 \uc5ed\ud560\uc744 \ud55c\ub2e4. \uac1c\uac00 \uc0ac\ub78c\ub4e4\uacfc \ud568\uaed8 \uacf5\uaca9\ud560 \ub54c, \ub291\ub300\ub294 \ube44\uc815\uc0c1\uc801\uc778 \ub300\ub2f4\ud55c \ud589\ub3d9\uc744 \ud558\uba70 \ub54c\ub85c\ub294 \uac00\uae4c\uc6b4 \uc0ac\ub78c\uc744 \ubb34\uc2dc\ud558\uae30\ub3c4 \ud55c\ub2e4. \ub291\ub300\uc758 \uac1c \uacf5\uaca9\uc740 \uc9d1\uc758 \ub4b7\ub9c8\ub2f9\uc774\ub098 \uc232 \ubaa8\ub450\uc5d0\uc11c \uc77c\uc5b4\ub0a0 \uc218 \uc788\ub2e4. \ub9c8\uc744 \uc678\uacfd\uc5d0\uc11c\ub294 \ub291\ub300\uac00 \uac1c\ub97c \uae30\ub2e4\ub9ac\uae30 \uc704\ud574 \ub9e4\ubcf5\ud558\uba70 \uadf8 \ub4a4\uc5d0 \ub2e4\ub978 \ub291\ub300\ub4e4\ub85c \ud558\uc5ec\uae08 \uac1c\ub97c \uc720\ud639\ud558\uac8c \ud558\uae30\ub3c4 \ud55c\ub2e4. \ub291\ub300\uc758 \uc0ac\ub0e5\uac1c \uacf5\uaca9\uc740 \uc2a4\uce78\ub514\ub098\ube44\uc544\uc640 \uc704\uc2a4\ucf58\uc2e0 \uc8fc\uc5d0\uc11c \ud070 \ubb38\uc81c\ub85c \ub300\ub450\ub41c\ub2e4. \uc2a4\uce78\ub514\ub098\ube44\uc544\uc5d0\uc11c \uac00\uc7a5 \ube48\ubc88\ud558\uac8c \uc8fd\uc740 \uc0ac\ub0e5\uac1c\ub294 \ud574\ub9ac\uc5b4 \ud488\uc885\uc73c\ub85c \ub299\uc740 \ub3d9\ubb3c\uc774 \uc5b4\ub9b0 \ub3d9\ubb3c\ubcf4\ub2e4 \uc18c\uc2ec\ud558\uc5ec \ub291\ub300\uc5d0 \ub300\ud55c \ubc18\uc751\uc744 \ub2e4\ub974\uac8c \ud560 \uac00\ub2a5\uc131\uc774 \uc788\uae30 \ub54c\ubb38\uc5d0 \uc704\ud5d8\ud558\ub2e4. \uac1c\uc758 \ub291\ub300\ub85c \uc778\ud55c \ubd80\uc0c1\uc740 \uc885\uc885 \ud5c8\ubc85\uc9c0\uc640 \ub4b7\ub2e4\ub9ac\uc5d0 \ub098\ud0c0\ub09c\ub2e4. \uce58\uba85\uc801\uc778 \uc0c1\ucc98\ub294 \ubcf4\ud1b5 \ubaa9 \ub4a4\ub97c \ubb3c\ub9b0 \uc0c1\ucc98\uc774\ub2e4. \uc2a4\uc6e8\ub374 \uc7d8\ud230\ub4dc(J\u00e4mthund) \uac19\uc740 \ub300\ud615 \uac1c\uc758 \uc0ac\ub0e5\uc740 \uc790\uc2e0\uc758 \ubc29\uc5b4\ud558\uae30 \ub354\uc6b1 \uc218\uc6d4\ud558\uae30 \ub54c\ubb38\uc5d0 \uc0b4\uc544\ub0a8\uc744 \uac00\ub2a5\uc131\uc774 \ub354\uc6b1 \ub192\ub2e4."} {"question": "\uc640\ud2b8 \ud0c0\uc77c\ub7ec\uc758 \ub09c\uc5d0 \uac00\ub2f4\ud55c \uc131\uc9c1\uc790\ub4e4 \uc911 \uc885\uad50\uc801 \uc2e0\ub150\uc5d0 \uc758\ud574 \ucc38\uc5ec\ud55c \uc774\ub4e4\uc774 \uc9c0\ub154\ub358 \uc131\ud5a5\uc740?", "paragraph": "1381\ub144 \ubc18\ub780\uc5d0 \uac00\ub2f4\ud55c \uc774\ub4e4\uc758 \uc808\ub300 \ub2e4\uc218\ub294, \uc758\ud68c\uc5d0 \uc758\ud574 \ub300\ubcc0\ub418\uc9c0 \ubabb\ud558\ub294 \uc774\ub4e4\uc774\uc5c8\uc73c\uba70, \uad6d\uac00\uc758 \uc758\uc0ac\uacb0\uc815 \uacfc\uc815\uc5d0\uc11c \uc18c\uc678\ub41c \uc774\ub4e4\uc774\uc5c8\ub2e4. \ud558\uc9c0\ub9cc \uc77c\ubd80 \uacbd\uc6b0\uc5d0\ub294 \uc0c1\ub300\uc801\uc73c\ub85c \ubd80\uc720\ud55c \uc2e0\uc0ac\uce35\uc774 \ubc18\ub780\uc5d0 \uac00\ub2f4\ud558\uac70\ub098 \ubc18\ub780\uc744 \uc774\ub04c\uae30\ub3c4 \ud588\ub294\ub370, \ub178\ud3ec\ud06c\uc758 \ub85c\uc800 \ubca0\uc774\ucee8 \uacbd\uc774 \ub300\ud45c\uc801\uc778 \uc0ac\ub840\uc774\ub2e4. \uc774\ub4e4 \uc911 \uc77c\ubd80\ub294 \ub09c\ubbfc\ub4e4\uc758 \uac15\uc555\uc5d0 \uc758\ud574 \ubc18\ub780\uc5d0 \ub3d9\ucc38\ud558\uac8c \ub41c \uac83\uc774\ub77c\uace0 \uc8fc\uc7a5\ud588\ub2e4. \ub2e4\uc218\uc758 \uc131\uc9c1\uc790\ub4e4 \uc5ed\uc2dc \ubc18\ub780\uc5d0\uc11c \uc5ed\ud560\uc744 \ub9e1\uc558\ub2e4. \uc874 \ubcfc\uc774\ub098 \uc874 \ub77c\uc704 \uac19\uc740 \uc720\uba85\ud55c \uc9c0\ub3c4\uc790\ubfd0 \uc544\ub2c8\ub77c, \ub3d9\ub0a8\ubd80\uc758 \ubc18\ub780\uc5d0 \uac00\ub2f4\ud55c \uc131\uc9c1\uc790\uac00 \uac70\uc758 20\uba85 \uac00\uae4c\uc774 \uae30\ub85d\uc0c1 \ub0a8\uc544 \uc788\ub2e4. \uc774\ub4e4 \uc131\uc9c1\uc790\ub4e4 \uc911 \uc77c\ubd80\ub294 \uc0ac\ud68c \ubd88\ub9cc\uc758 \uc5f0\uc7a5\uc73c\ub85c, \uc77c\ubd80\ub294 \uc0c1\ub300\uc801 \ube48\uace4\uc73c\ub85c \uc778\ud55c \ubd88\ub9ac\ud55c \uc870\uac74 \ub54c\ubb38\uc5d0, \ub610\ub2e4\ub978 \uc77c\ubd80\ub294 \uac15\ub82c\ud55c \uae09\uc9c4\uc801 \uc885\uad50 \uc2e0\ub150\uc73c\ub85c \uc778\ud558\uc5ec \ubc18\ub780\uc5d0 \ucc38\uac00\ud588\ub2e4.", "answer": "\uae09\uc9c4\uc801", "sentence": "\uc774\ub4e4 \uc131\uc9c1\uc790\ub4e4 \uc911 \uc77c\ubd80\ub294 \uc0ac\ud68c \ubd88\ub9cc\uc758 \uc5f0\uc7a5\uc73c\ub85c, \uc77c\ubd80\ub294 \uc0c1\ub300\uc801 \ube48\uace4\uc73c\ub85c \uc778\ud55c \ubd88\ub9ac\ud55c \uc870\uac74 \ub54c\ubb38\uc5d0, \ub610\ub2e4\ub978 \uc77c\ubd80\ub294 \uac15\ub82c\ud55c \uae09\uc9c4\uc801 \uc885\uad50 \uc2e0\ub150\uc73c\ub85c \uc778\ud558\uc5ec \ubc18\ub780\uc5d0 \ucc38\uac00\ud588\ub2e4.", "paragraph_sentence": "1381\ub144 \ubc18\ub780\uc5d0 \uac00\ub2f4\ud55c \uc774\ub4e4\uc758 \uc808\ub300 \ub2e4\uc218\ub294, \uc758\ud68c\uc5d0 \uc758\ud574 \ub300\ubcc0\ub418\uc9c0 \ubabb\ud558\ub294 \uc774\ub4e4\uc774\uc5c8\uc73c\uba70, \uad6d\uac00\uc758 \uc758\uc0ac\uacb0\uc815 \uacfc\uc815\uc5d0\uc11c \uc18c\uc678\ub41c \uc774\ub4e4\uc774\uc5c8\ub2e4. \ud558\uc9c0\ub9cc \uc77c\ubd80 \uacbd\uc6b0\uc5d0\ub294 \uc0c1\ub300\uc801\uc73c\ub85c \ubd80\uc720\ud55c \uc2e0\uc0ac\uce35\uc774 \ubc18\ub780\uc5d0 \uac00\ub2f4\ud558\uac70\ub098 \ubc18\ub780\uc744 \uc774\ub04c\uae30\ub3c4 \ud588\ub294\ub370, \ub178\ud3ec\ud06c\uc758 \ub85c\uc800 \ubca0\uc774\ucee8 \uacbd\uc774 \ub300\ud45c\uc801\uc778 \uc0ac\ub840\uc774\ub2e4. \uc774\ub4e4 \uc911 \uc77c\ubd80\ub294 \ub09c\ubbfc\ub4e4\uc758 \uac15\uc555\uc5d0 \uc758\ud574 \ubc18\ub780\uc5d0 \ub3d9\ucc38\ud558\uac8c \ub41c \uac83\uc774\ub77c\uace0 \uc8fc\uc7a5\ud588\ub2e4. \ub2e4\uc218\uc758 \uc131\uc9c1\uc790\ub4e4 \uc5ed\uc2dc \ubc18\ub780\uc5d0\uc11c \uc5ed\ud560\uc744 \ub9e1\uc558\ub2e4. \uc874 \ubcfc\uc774\ub098 \uc874 \ub77c\uc704 \uac19\uc740 \uc720\uba85\ud55c \uc9c0\ub3c4\uc790\ubfd0 \uc544\ub2c8\ub77c, \ub3d9\ub0a8\ubd80\uc758 \ubc18\ub780\uc5d0 \uac00\ub2f4\ud55c \uc131\uc9c1\uc790\uac00 \uac70\uc758 20\uba85 \uac00\uae4c\uc774 \uae30\ub85d\uc0c1 \ub0a8\uc544 \uc788\ub2e4. \uc774\ub4e4 \uc131\uc9c1\uc790\ub4e4 \uc911 \uc77c\ubd80\ub294 \uc0ac\ud68c \ubd88\ub9cc\uc758 \uc5f0\uc7a5\uc73c\ub85c, \uc77c\ubd80\ub294 \uc0c1\ub300\uc801 \ube48\uace4\uc73c\ub85c \uc778\ud55c \ubd88\ub9ac\ud55c \uc870\uac74 \ub54c\ubb38\uc5d0, \ub610\ub2e4\ub978 \uc77c\ubd80\ub294 \uac15\ub82c\ud55c \uae09\uc9c4\uc801 \uc885\uad50 \uc2e0\ub150\uc73c\ub85c \uc778\ud558\uc5ec \ubc18\ub780\uc5d0 \ucc38\uac00\ud588\ub2e4. ", "paragraph_answer": "1381\ub144 \ubc18\ub780\uc5d0 \uac00\ub2f4\ud55c \uc774\ub4e4\uc758 \uc808\ub300 \ub2e4\uc218\ub294, \uc758\ud68c\uc5d0 \uc758\ud574 \ub300\ubcc0\ub418\uc9c0 \ubabb\ud558\ub294 \uc774\ub4e4\uc774\uc5c8\uc73c\uba70, \uad6d\uac00\uc758 \uc758\uc0ac\uacb0\uc815 \uacfc\uc815\uc5d0\uc11c \uc18c\uc678\ub41c \uc774\ub4e4\uc774\uc5c8\ub2e4. \ud558\uc9c0\ub9cc \uc77c\ubd80 \uacbd\uc6b0\uc5d0\ub294 \uc0c1\ub300\uc801\uc73c\ub85c \ubd80\uc720\ud55c \uc2e0\uc0ac\uce35\uc774 \ubc18\ub780\uc5d0 \uac00\ub2f4\ud558\uac70\ub098 \ubc18\ub780\uc744 \uc774\ub04c\uae30\ub3c4 \ud588\ub294\ub370, \ub178\ud3ec\ud06c\uc758 \ub85c\uc800 \ubca0\uc774\ucee8 \uacbd\uc774 \ub300\ud45c\uc801\uc778 \uc0ac\ub840\uc774\ub2e4. \uc774\ub4e4 \uc911 \uc77c\ubd80\ub294 \ub09c\ubbfc\ub4e4\uc758 \uac15\uc555\uc5d0 \uc758\ud574 \ubc18\ub780\uc5d0 \ub3d9\ucc38\ud558\uac8c \ub41c \uac83\uc774\ub77c\uace0 \uc8fc\uc7a5\ud588\ub2e4. \ub2e4\uc218\uc758 \uc131\uc9c1\uc790\ub4e4 \uc5ed\uc2dc \ubc18\ub780\uc5d0\uc11c \uc5ed\ud560\uc744 \ub9e1\uc558\ub2e4. \uc874 \ubcfc\uc774\ub098 \uc874 \ub77c\uc704 \uac19\uc740 \uc720\uba85\ud55c \uc9c0\ub3c4\uc790\ubfd0 \uc544\ub2c8\ub77c, \ub3d9\ub0a8\ubd80\uc758 \ubc18\ub780\uc5d0 \uac00\ub2f4\ud55c \uc131\uc9c1\uc790\uac00 \uac70\uc758 20\uba85 \uac00\uae4c\uc774 \uae30\ub85d\uc0c1 \ub0a8\uc544 \uc788\ub2e4. \uc774\ub4e4 \uc131\uc9c1\uc790\ub4e4 \uc911 \uc77c\ubd80\ub294 \uc0ac\ud68c \ubd88\ub9cc\uc758 \uc5f0\uc7a5\uc73c\ub85c, \uc77c\ubd80\ub294 \uc0c1\ub300\uc801 \ube48\uace4\uc73c\ub85c \uc778\ud55c \ubd88\ub9ac\ud55c \uc870\uac74 \ub54c\ubb38\uc5d0, \ub610\ub2e4\ub978 \uc77c\ubd80\ub294 \uac15\ub82c\ud55c \uae09\uc9c4\uc801 \uc885\uad50 \uc2e0\ub150\uc73c\ub85c \uc778\ud558\uc5ec \ubc18\ub780\uc5d0 \ucc38\uac00\ud588\ub2e4.", "sentence_answer": "\uc774\ub4e4 \uc131\uc9c1\uc790\ub4e4 \uc911 \uc77c\ubd80\ub294 \uc0ac\ud68c \ubd88\ub9cc\uc758 \uc5f0\uc7a5\uc73c\ub85c, \uc77c\ubd80\ub294 \uc0c1\ub300\uc801 \ube48\uace4\uc73c\ub85c \uc778\ud55c \ubd88\ub9ac\ud55c \uc870\uac74 \ub54c\ubb38\uc5d0, \ub610\ub2e4\ub978 \uc77c\ubd80\ub294 \uac15\ub82c\ud55c \uae09\uc9c4\uc801 \uc885\uad50 \uc2e0\ub150\uc73c\ub85c \uc778\ud558\uc5ec \ubc18\ub780\uc5d0 \ucc38\uac00\ud588\ub2e4.", "paragraph_id": "6541489-35-2", "paragraph_question": "question: \uc640\ud2b8 \ud0c0\uc77c\ub7ec\uc758 \ub09c\uc5d0 \uac00\ub2f4\ud55c \uc131\uc9c1\uc790\ub4e4 \uc911 \uc885\uad50\uc801 \uc2e0\ub150\uc5d0 \uc758\ud574 \ucc38\uc5ec\ud55c \uc774\ub4e4\uc774 \uc9c0\ub154\ub358 \uc131\ud5a5\uc740?, context: 1381\ub144 \ubc18\ub780\uc5d0 \uac00\ub2f4\ud55c \uc774\ub4e4\uc758 \uc808\ub300 \ub2e4\uc218\ub294, \uc758\ud68c\uc5d0 \uc758\ud574 \ub300\ubcc0\ub418\uc9c0 \ubabb\ud558\ub294 \uc774\ub4e4\uc774\uc5c8\uc73c\uba70, \uad6d\uac00\uc758 \uc758\uc0ac\uacb0\uc815 \uacfc\uc815\uc5d0\uc11c \uc18c\uc678\ub41c \uc774\ub4e4\uc774\uc5c8\ub2e4. \ud558\uc9c0\ub9cc \uc77c\ubd80 \uacbd\uc6b0\uc5d0\ub294 \uc0c1\ub300\uc801\uc73c\ub85c \ubd80\uc720\ud55c \uc2e0\uc0ac\uce35\uc774 \ubc18\ub780\uc5d0 \uac00\ub2f4\ud558\uac70\ub098 \ubc18\ub780\uc744 \uc774\ub04c\uae30\ub3c4 \ud588\ub294\ub370, \ub178\ud3ec\ud06c\uc758 \ub85c\uc800 \ubca0\uc774\ucee8 \uacbd\uc774 \ub300\ud45c\uc801\uc778 \uc0ac\ub840\uc774\ub2e4. \uc774\ub4e4 \uc911 \uc77c\ubd80\ub294 \ub09c\ubbfc\ub4e4\uc758 \uac15\uc555\uc5d0 \uc758\ud574 \ubc18\ub780\uc5d0 \ub3d9\ucc38\ud558\uac8c \ub41c \uac83\uc774\ub77c\uace0 \uc8fc\uc7a5\ud588\ub2e4. \ub2e4\uc218\uc758 \uc131\uc9c1\uc790\ub4e4 \uc5ed\uc2dc \ubc18\ub780\uc5d0\uc11c \uc5ed\ud560\uc744 \ub9e1\uc558\ub2e4. \uc874 \ubcfc\uc774\ub098 \uc874 \ub77c\uc704 \uac19\uc740 \uc720\uba85\ud55c \uc9c0\ub3c4\uc790\ubfd0 \uc544\ub2c8\ub77c, \ub3d9\ub0a8\ubd80\uc758 \ubc18\ub780\uc5d0 \uac00\ub2f4\ud55c \uc131\uc9c1\uc790\uac00 \uac70\uc758 20\uba85 \uac00\uae4c\uc774 \uae30\ub85d\uc0c1 \ub0a8\uc544 \uc788\ub2e4. \uc774\ub4e4 \uc131\uc9c1\uc790\ub4e4 \uc911 \uc77c\ubd80\ub294 \uc0ac\ud68c \ubd88\ub9cc\uc758 \uc5f0\uc7a5\uc73c\ub85c, \uc77c\ubd80\ub294 \uc0c1\ub300\uc801 \ube48\uace4\uc73c\ub85c \uc778\ud55c \ubd88\ub9ac\ud55c \uc870\uac74 \ub54c\ubb38\uc5d0, \ub610\ub2e4\ub978 \uc77c\ubd80\ub294 \uac15\ub82c\ud55c \uae09\uc9c4\uc801 \uc885\uad50 \uc2e0\ub150\uc73c\ub85c \uc778\ud558\uc5ec \ubc18\ub780\uc5d0 \ucc38\uac00\ud588\ub2e4."} {"question": "\ubd80\uc0b0\ub300\ud559\uad50\ub97c \ud2b9\uc131\ud654 \uacf5\uacfc\ub300\ud559\uc73c\ub85c \uc9c0\uc815\ud55c \ub54c\ub294?", "paragraph": "1970\ub144\ub300 \ub4e4\uc5b4\uc11c \uc2e4\ud5d8 \ub300\ud559\uacfc \ub300\ud559 \ud2b9\uc131\ud654\ub85c \uace0\ub4f1\uad50\uc721\uc758 \uac1c\ud601\uc744 \uaf80\ud558\ub358 \ubc15\uc815\ud76c \uc815\ubd80\ub294 1973\ub144 10\uc6d4\uc5d0 \ubb38\uad50\ubd80\uc5d0\uc11c \uc804\uad6d 18\uac1c \ub300\ud559 52\uac1c \ud559\uacfc\ub97c 1\ucc28 \uc9c0\uc5ed \ud2b9\uc131\ud654 \ud559\uacfc\ub85c \uc120\uc815\ud558\uc600\ub2e4. \ubd80\uc0b0\ub300\ud559\uad50\ub294 \uae30\uacc4\uacf5\ud559\uacfc, \uae30\uacc4\uc124\uacc4\ud559\uacfc, \uc870\uc120\uacf5\ud559\uacfc, \uc804\uae30\uacf5\ud559\uacfc\uac00 \ud2b9\uc131\ud654\ub85c \uc120\uc815\ub418\uc5c8\uc73c\uba70, 1976\ub144 9\uc6d4 \ubb38\uad50\ubd80\ub294 \ubd80\uc0b0\ub300\ud559\uad50\ub97c \uae30\uacc4\uacf5\uacfc\ub300\ud559\uc73c\ub85c, \uacbd\ubd81\ub300\ud559\uad50\ub97c \uc804\uc790\uacf5\uacfc\ub300\ud559\uc73c\ub85c, \uc804\ub0a8\ub300\ud559\uad50\ub97c \ud654\ud559\uacf5\uacfc\ub300\ud559\uc73c\ub85c \ud2b9\uc131\ud654 \ud558\uc5ec 3\uac1c \ub300\ud559\uc5d0 11\uc5b5 5\ucc9c\ub9cc\uc6d0\uc744 \ub4e4\uc5ec \uc2e4\ud5d8 \uc2dc\uc124\uc744 \ud655\ucda9\ud558\uae30\ub85c \ud588\ub2e4. \uc774\ub4ec\ud574\uc778 1977\ub144 4\uc6d4\uc5d0\ub294 \ubd80\uc0b0\ub300\ud559\uad50 \uacbd\ubd81\ub300\ud559\uad50, \uc804\ub0a8\ub300\ud559\uad50, \ucda9\ub0a8\ub300\ud559\uad50\ub97c \ud2b9\uc131\ud654 \uacf5\uacfc\ub300\ud559\uc73c\ub85c \uc9c0\uc815\ud558\uc5ec \uc9d1\uc911 \uc721\uc131\ud558\uae30\ub85c \ud558\uace0 \ubd80\uc0b0\ub300\ud559\uad50\uc5d0\uc11c\ub294 \uae30\uacc4\uacf5\ud559\uacfc, \uc804\uae30\uae30\uacc4\uacf5\ud559\uacfc(\u73fe \uc804\uc790\uacf5\ud559\uacfc, \uc804\uae30\ucef4\ud4e8\ud130\uacf5\ud559\ubd80), \uc870\uc120\uacf5\ud559\uacfc, \uc7ac\ub8cc\uacf5\ud559\uacfc\uac00 \ud2b9\uc131\ud654 \ud559\uacfc\ub85c \uc120\uc815\ub418\uc5c8\ub2e4.", "answer": "1977\ub144 4\uc6d4", "sentence": "\uc774\ub4ec\ud574\uc778 1977\ub144 4\uc6d4 \uc5d0\ub294 \ubd80\uc0b0\ub300\ud559\uad50 \uacbd\ubd81\ub300\ud559\uad50, \uc804\ub0a8\ub300\ud559\uad50, \ucda9\ub0a8\ub300\ud559\uad50\ub97c \ud2b9\uc131\ud654 \uacf5\uacfc\ub300\ud559\uc73c\ub85c \uc9c0\uc815\ud558\uc5ec \uc9d1\uc911 \uc721\uc131\ud558\uae30\ub85c \ud558\uace0 \ubd80\uc0b0\ub300\ud559\uad50\uc5d0\uc11c\ub294 \uae30\uacc4\uacf5\ud559\uacfc, \uc804\uae30\uae30\uacc4\uacf5\ud559\uacfc(\u73fe \uc804\uc790\uacf5\ud559\uacfc, \uc804\uae30\ucef4\ud4e8\ud130\uacf5\ud559\ubd80), \uc870\uc120\uacf5\ud559\uacfc, \uc7ac\ub8cc\uacf5\ud559\uacfc\uac00 \ud2b9\uc131\ud654 \ud559\uacfc\ub85c \uc120\uc815\ub418\uc5c8\ub2e4.", "paragraph_sentence": "1970\ub144\ub300 \ub4e4\uc5b4\uc11c \uc2e4\ud5d8 \ub300\ud559\uacfc \ub300\ud559 \ud2b9\uc131\ud654\ub85c \uace0\ub4f1\uad50\uc721\uc758 \uac1c\ud601\uc744 \uaf80\ud558\ub358 \ubc15\uc815\ud76c \uc815\ubd80\ub294 1973\ub144 10\uc6d4\uc5d0 \ubb38\uad50\ubd80\uc5d0\uc11c \uc804\uad6d 18\uac1c \ub300\ud559 52\uac1c \ud559\uacfc\ub97c 1\ucc28 \uc9c0\uc5ed \ud2b9\uc131\ud654 \ud559\uacfc\ub85c \uc120\uc815\ud558\uc600\ub2e4. \ubd80\uc0b0\ub300\ud559\uad50\ub294 \uae30\uacc4\uacf5\ud559\uacfc, \uae30\uacc4\uc124\uacc4\ud559\uacfc, \uc870\uc120\uacf5\ud559\uacfc, \uc804\uae30\uacf5\ud559\uacfc\uac00 \ud2b9\uc131\ud654\ub85c \uc120\uc815\ub418\uc5c8\uc73c\uba70, 1976\ub144 9\uc6d4 \ubb38\uad50\ubd80\ub294 \ubd80\uc0b0\ub300\ud559\uad50\ub97c \uae30\uacc4\uacf5\uacfc\ub300\ud559\uc73c\ub85c, \uacbd\ubd81\ub300\ud559\uad50\ub97c \uc804\uc790\uacf5\uacfc\ub300\ud559\uc73c\ub85c, \uc804\ub0a8\ub300\ud559\uad50\ub97c \ud654\ud559\uacf5\uacfc\ub300\ud559\uc73c\ub85c \ud2b9\uc131\ud654 \ud558\uc5ec 3\uac1c \ub300\ud559\uc5d0 11\uc5b5 5\ucc9c\ub9cc\uc6d0\uc744 \ub4e4\uc5ec \uc2e4\ud5d8 \uc2dc\uc124\uc744 \ud655\ucda9\ud558\uae30\ub85c \ud588\ub2e4. \uc774\ub4ec\ud574\uc778 1977\ub144 4\uc6d4 \uc5d0\ub294 \ubd80\uc0b0\ub300\ud559\uad50 \uacbd\ubd81\ub300\ud559\uad50, \uc804\ub0a8\ub300\ud559\uad50, \ucda9\ub0a8\ub300\ud559\uad50\ub97c \ud2b9\uc131\ud654 \uacf5\uacfc\ub300\ud559\uc73c\ub85c \uc9c0\uc815\ud558\uc5ec \uc9d1\uc911 \uc721\uc131\ud558\uae30\ub85c \ud558\uace0 \ubd80\uc0b0\ub300\ud559\uad50\uc5d0\uc11c\ub294 \uae30\uacc4\uacf5\ud559\uacfc, \uc804\uae30\uae30\uacc4\uacf5\ud559\uacfc(\u73fe \uc804\uc790\uacf5\ud559\uacfc, \uc804\uae30\ucef4\ud4e8\ud130\uacf5\ud559\ubd80), \uc870\uc120\uacf5\ud559\uacfc, \uc7ac\ub8cc\uacf5\ud559\uacfc\uac00 \ud2b9\uc131\ud654 \ud559\uacfc\ub85c \uc120\uc815\ub418\uc5c8\ub2e4. ", "paragraph_answer": "1970\ub144\ub300 \ub4e4\uc5b4\uc11c \uc2e4\ud5d8 \ub300\ud559\uacfc \ub300\ud559 \ud2b9\uc131\ud654\ub85c \uace0\ub4f1\uad50\uc721\uc758 \uac1c\ud601\uc744 \uaf80\ud558\ub358 \ubc15\uc815\ud76c \uc815\ubd80\ub294 1973\ub144 10\uc6d4\uc5d0 \ubb38\uad50\ubd80\uc5d0\uc11c \uc804\uad6d 18\uac1c \ub300\ud559 52\uac1c \ud559\uacfc\ub97c 1\ucc28 \uc9c0\uc5ed \ud2b9\uc131\ud654 \ud559\uacfc\ub85c \uc120\uc815\ud558\uc600\ub2e4. \ubd80\uc0b0\ub300\ud559\uad50\ub294 \uae30\uacc4\uacf5\ud559\uacfc, \uae30\uacc4\uc124\uacc4\ud559\uacfc, \uc870\uc120\uacf5\ud559\uacfc, \uc804\uae30\uacf5\ud559\uacfc\uac00 \ud2b9\uc131\ud654\ub85c \uc120\uc815\ub418\uc5c8\uc73c\uba70, 1976\ub144 9\uc6d4 \ubb38\uad50\ubd80\ub294 \ubd80\uc0b0\ub300\ud559\uad50\ub97c \uae30\uacc4\uacf5\uacfc\ub300\ud559\uc73c\ub85c, \uacbd\ubd81\ub300\ud559\uad50\ub97c \uc804\uc790\uacf5\uacfc\ub300\ud559\uc73c\ub85c, \uc804\ub0a8\ub300\ud559\uad50\ub97c \ud654\ud559\uacf5\uacfc\ub300\ud559\uc73c\ub85c \ud2b9\uc131\ud654 \ud558\uc5ec 3\uac1c \ub300\ud559\uc5d0 11\uc5b5 5\ucc9c\ub9cc\uc6d0\uc744 \ub4e4\uc5ec \uc2e4\ud5d8 \uc2dc\uc124\uc744 \ud655\ucda9\ud558\uae30\ub85c \ud588\ub2e4. \uc774\ub4ec\ud574\uc778 1977\ub144 4\uc6d4 \uc5d0\ub294 \ubd80\uc0b0\ub300\ud559\uad50 \uacbd\ubd81\ub300\ud559\uad50, \uc804\ub0a8\ub300\ud559\uad50, \ucda9\ub0a8\ub300\ud559\uad50\ub97c \ud2b9\uc131\ud654 \uacf5\uacfc\ub300\ud559\uc73c\ub85c \uc9c0\uc815\ud558\uc5ec \uc9d1\uc911 \uc721\uc131\ud558\uae30\ub85c \ud558\uace0 \ubd80\uc0b0\ub300\ud559\uad50\uc5d0\uc11c\ub294 \uae30\uacc4\uacf5\ud559\uacfc, \uc804\uae30\uae30\uacc4\uacf5\ud559\uacfc(\u73fe \uc804\uc790\uacf5\ud559\uacfc, \uc804\uae30\ucef4\ud4e8\ud130\uacf5\ud559\ubd80), \uc870\uc120\uacf5\ud559\uacfc, \uc7ac\ub8cc\uacf5\ud559\uacfc\uac00 \ud2b9\uc131\ud654 \ud559\uacfc\ub85c \uc120\uc815\ub418\uc5c8\ub2e4.", "sentence_answer": "\uc774\ub4ec\ud574\uc778 1977\ub144 4\uc6d4 \uc5d0\ub294 \ubd80\uc0b0\ub300\ud559\uad50 \uacbd\ubd81\ub300\ud559\uad50, \uc804\ub0a8\ub300\ud559\uad50, \ucda9\ub0a8\ub300\ud559\uad50\ub97c \ud2b9\uc131\ud654 \uacf5\uacfc\ub300\ud559\uc73c\ub85c \uc9c0\uc815\ud558\uc5ec \uc9d1\uc911 \uc721\uc131\ud558\uae30\ub85c \ud558\uace0 \ubd80\uc0b0\ub300\ud559\uad50\uc5d0\uc11c\ub294 \uae30\uacc4\uacf5\ud559\uacfc, \uc804\uae30\uae30\uacc4\uacf5\ud559\uacfc(\u73fe \uc804\uc790\uacf5\ud559\uacfc, \uc804\uae30\ucef4\ud4e8\ud130\uacf5\ud559\ubd80), \uc870\uc120\uacf5\ud559\uacfc, \uc7ac\ub8cc\uacf5\ud559\uacfc\uac00 \ud2b9\uc131\ud654 \ud559\uacfc\ub85c \uc120\uc815\ub418\uc5c8\ub2e4.", "paragraph_id": "6582250-3-1", "paragraph_question": "question: \ubd80\uc0b0\ub300\ud559\uad50\ub97c \ud2b9\uc131\ud654 \uacf5\uacfc\ub300\ud559\uc73c\ub85c \uc9c0\uc815\ud55c \ub54c\ub294?, context: 1970\ub144\ub300 \ub4e4\uc5b4\uc11c \uc2e4\ud5d8 \ub300\ud559\uacfc \ub300\ud559 \ud2b9\uc131\ud654\ub85c \uace0\ub4f1\uad50\uc721\uc758 \uac1c\ud601\uc744 \uaf80\ud558\ub358 \ubc15\uc815\ud76c \uc815\ubd80\ub294 1973\ub144 10\uc6d4\uc5d0 \ubb38\uad50\ubd80\uc5d0\uc11c \uc804\uad6d 18\uac1c \ub300\ud559 52\uac1c \ud559\uacfc\ub97c 1\ucc28 \uc9c0\uc5ed \ud2b9\uc131\ud654 \ud559\uacfc\ub85c \uc120\uc815\ud558\uc600\ub2e4. \ubd80\uc0b0\ub300\ud559\uad50\ub294 \uae30\uacc4\uacf5\ud559\uacfc, \uae30\uacc4\uc124\uacc4\ud559\uacfc, \uc870\uc120\uacf5\ud559\uacfc, \uc804\uae30\uacf5\ud559\uacfc\uac00 \ud2b9\uc131\ud654\ub85c \uc120\uc815\ub418\uc5c8\uc73c\uba70, 1976\ub144 9\uc6d4 \ubb38\uad50\ubd80\ub294 \ubd80\uc0b0\ub300\ud559\uad50\ub97c \uae30\uacc4\uacf5\uacfc\ub300\ud559\uc73c\ub85c, \uacbd\ubd81\ub300\ud559\uad50\ub97c \uc804\uc790\uacf5\uacfc\ub300\ud559\uc73c\ub85c, \uc804\ub0a8\ub300\ud559\uad50\ub97c \ud654\ud559\uacf5\uacfc\ub300\ud559\uc73c\ub85c \ud2b9\uc131\ud654 \ud558\uc5ec 3\uac1c \ub300\ud559\uc5d0 11\uc5b5 5\ucc9c\ub9cc\uc6d0\uc744 \ub4e4\uc5ec \uc2e4\ud5d8 \uc2dc\uc124\uc744 \ud655\ucda9\ud558\uae30\ub85c \ud588\ub2e4. \uc774\ub4ec\ud574\uc778 1977\ub144 4\uc6d4\uc5d0\ub294 \ubd80\uc0b0\ub300\ud559\uad50 \uacbd\ubd81\ub300\ud559\uad50, \uc804\ub0a8\ub300\ud559\uad50, \ucda9\ub0a8\ub300\ud559\uad50\ub97c \ud2b9\uc131\ud654 \uacf5\uacfc\ub300\ud559\uc73c\ub85c \uc9c0\uc815\ud558\uc5ec \uc9d1\uc911 \uc721\uc131\ud558\uae30\ub85c \ud558\uace0 \ubd80\uc0b0\ub300\ud559\uad50\uc5d0\uc11c\ub294 \uae30\uacc4\uacf5\ud559\uacfc, \uc804\uae30\uae30\uacc4\uacf5\ud559\uacfc(\u73fe \uc804\uc790\uacf5\ud559\uacfc, \uc804\uae30\ucef4\ud4e8\ud130\uacf5\ud559\ubd80), \uc870\uc120\uacf5\ud559\uacfc, \uc7ac\ub8cc\uacf5\ud559\uacfc\uac00 \ud2b9\uc131\ud654 \ud559\uacfc\ub85c \uc120\uc815\ub418\uc5c8\ub2e4."} {"question": "2017\ub144 \uc77c\ubcf8\uc774 \ubd80\uc0b0 \uc18c\ub140\uc0c1 \ubb38\uc81c\uac00 \ubc1c\uc0dd \ud558\uc600\uc744 \ub54c \uc18c\ud658\uc2dc\ud0a8 \uc8fc\ud55c \ub300\uc0ac\ub294 \ub204\uad6c\uc778\uac00?", "paragraph": "3\u00b71\uc808 \uae30\ub150\uc2dd\uc5d0\uc11c \ubb38\uc7ac\uc778\uc740 \uc77c\ubcf8\uc758 \uc704\uc548\ubd80 \ub9cc\ud589\uc744 \ubc18\uc778\ub95c\uc801 \uc778\uad8c\ubc94\uc8c4\ub85c, \uc77c\ubcf8\uc744 \uac00\ud574\uc790\ub85c \ud45c\ud604\ud558\uba74\uc11c \"\uac00\ud574\uc790\uc778 \uc77c\ubcf8 \uc815\ubd80\uac00 '\ub05d\ub0ac\ub2e4'\ub77c\uace0 \ub9d0\ud574\uc11c\ub294 \uc548 \ub41c\ub2e4\"\uace0 \uac15\ud558\uac8c \ube44\ud310\ud558\uace0 \"\uc77c\ubcf8\uc740 \uc778\ub958 \ubcf4\ud3b8\uc758 \uc591\uc2ec\uc73c\ub85c \uc5ed\uc0ac\uc758 \uc9c4\uc2e4\uacfc \uc815\uc758\ub97c \ub9c8\uc8fc\ud560 \uc218 \uc788\uc5b4\uc57c \ud55c\ub2e4\"\uace0 \ud588\ub2e4. \uc774\uc5d0 \uc77c\ubcf8\uc740 \ud55c\uc77c \uc591\uad6d\uc740 \uc774\ubbf8 \uc704\uc548\ubd80 \ubb38\uc81c\ub97c \ucd5c\uc885\uc801\uc774\uace0 \ubd88\uac00\uc5ed\uc801\uc73c\ub85c \ud574\uacb0\ud588\ub2e4\uba74\uc11c \"\ubb38 \ub300\ud1b5\ub839\uc758 \ubc1c\uc5b8\uc740 \ud55c\uc77c\ud569\uc758\uc5d0 \ubc18\ud558\ub294 \uac83\uc73c\ub85c \uc808\ub300 \ubc1b\uc544\ub4e4\uc77c \uc218 \uc5c6\ub2e4\"\uace0 \ubc18\ubc1c\ud588\ub2e4. \ud558\uc9c0\ub9cc \uc77c\ubcf8\uc740 2017\ub144 \ubd80\uc0b0 \uc18c\ub140\uc0c1 \ubb38\uc81c \ub2f9\uc2dc \ub098\uac00\ubbf8\ub124 \uc57c\uc2a4\ub9c8\uc0ac \uc8fc\ud55c \ub300\uc0ac\ub97c \uc18c\ud658\uc2dc\ud0a4\ub294 \uac15\uacbd \uc870\uce58\ub97c \ucde8\ud55c \ub4a4\uc5d0\ub294 \ub3c4\ubc1c\uc801\uc778 \uc561\uc158\uc744 \ucde8\ud558\uc9c0 \uc54a\uace0 \uc788\ub2e4. \uc774\ub294 \ud55c\uc77c \uac04\uc758 \uacf5\uc870\ub97c \ud6fc\uc190\uc2dc\ud0ac \uc6b0\ub824\uc640 \ud568\uaed8 \ub0a8\ubd81 \ub300\ud654\u00b7\ubd81\ubbf8 \ub300\ud654\uac00 \ubb34\ub974\uc775\uc740 \uc0c1\ud669\uc5d0\uc11c \ub300\ubd81 \uc678\uad50\uc5d0\uc11c '\uc77c\ubcf8 \ud328\uc2f1'\uc774 \uc77c\uc5b4\ub0a0 \uc218 \uc788\uae30 \ub54c\ubb38\uc778 \uac83\uc73c\ub85c \ubd84\uc11d\ub41c\ub2e4. \ud55c\ud3b8, \uc774\ubc88 \uae30\ub150\uc0ac\uc5d0\uc11c\ub294 \uac00\uc7a5 \uba3c\uc800 \uc77c\ubcf8\uc758 \uc2dd\ubbfc\uc9c0\uac00 \ub41c \uc6b0\ub9ac \ub545\uc774 \ub3c5\ub3c4\ub77c\uba70 \ub3c5\ub3c4 \ubb38\uc81c\uc5d0\uc11c\ub3c4 \ubaa9\uc18c\ub9ac\ub97c \ub192\uc600\ub294\ub370 \uc774\ub294 \uc77c\ubcf8 \uc815\ubd80\uac00 \uace0\uad50 \uad50\uacfc\uc11c \uc9c0\uce68\uc5d0 \ub3c5\ub3c4 \uc601\uc720\uad8c \uc8fc\uc7a5\uc744 \ub123\uace0 \ub3c4\ucfc4\uc758 \uc911\uc2ec\uc5d0 \ub3c5\ub3c4\uc804\uc2dc\uad00\uc744 \uc138\uc6b0\ub294 \ub4f1 \ub3c4\ubc1c\uc774 \uc788\uc790 \uc774\uc5d0 \ub300\ud55c \uc81c\ub3d9\uc744 \uac78 \ud544\uc694\uc131\uc744 \ub290\ub080 \uac83\uc774\ub77c\ub294 \ud3c9\uc774\ub2e4.", "answer": "\ub098\uac00\ubbf8\ub124 \uc57c\uc2a4\ub9c8\uc0ac", "sentence": "\uc77c\ubcf8\uc740 2017\ub144 \ubd80\uc0b0 \uc18c\ub140\uc0c1 \ubb38\uc81c \ub2f9\uc2dc \ub098\uac00\ubbf8\ub124 \uc57c\uc2a4\ub9c8\uc0ac \uc8fc\ud55c \ub300\uc0ac\ub97c \uc18c\ud658\uc2dc\ud0a4\ub294 \uac15\uacbd \uc870\uce58\ub97c \ucde8\ud55c \ub4a4\uc5d0\ub294 \ub3c4\ubc1c\uc801\uc778 \uc561\uc158\uc744 \ucde8\ud558\uc9c0 \uc54a\uace0 \uc788\ub2e4.", "paragraph_sentence": "3\u00b71\uc808 \uae30\ub150\uc2dd\uc5d0\uc11c \ubb38\uc7ac\uc778\uc740 \uc77c\ubcf8\uc758 \uc704\uc548\ubd80 \ub9cc\ud589\uc744 \ubc18\uc778\ub95c\uc801 \uc778\uad8c\ubc94\uc8c4\ub85c, \uc77c\ubcf8\uc744 \uac00\ud574\uc790\ub85c \ud45c\ud604\ud558\uba74\uc11c \"\uac00\ud574\uc790\uc778 \uc77c\ubcf8 \uc815\ubd80\uac00 '\ub05d\ub0ac\ub2e4'\ub77c\uace0 \ub9d0\ud574\uc11c\ub294 \uc548 \ub41c\ub2e4\"\uace0 \uac15\ud558\uac8c \ube44\ud310\ud558\uace0 \"\uc77c\ubcf8\uc740 \uc778\ub958 \ubcf4\ud3b8\uc758 \uc591\uc2ec\uc73c\ub85c \uc5ed\uc0ac\uc758 \uc9c4\uc2e4\uacfc \uc815\uc758\ub97c \ub9c8\uc8fc\ud560 \uc218 \uc788\uc5b4\uc57c \ud55c\ub2e4\"\uace0 \ud588\ub2e4. \uc774\uc5d0 \uc77c\ubcf8\uc740 \ud55c\uc77c \uc591\uad6d\uc740 \uc774\ubbf8 \uc704\uc548\ubd80 \ubb38\uc81c\ub97c \ucd5c\uc885\uc801\uc774\uace0 \ubd88\uac00\uc5ed\uc801\uc73c\ub85c \ud574\uacb0\ud588\ub2e4\uba74\uc11c \"\ubb38 \ub300\ud1b5\ub839\uc758 \ubc1c\uc5b8\uc740 \ud55c\uc77c\ud569\uc758\uc5d0 \ubc18\ud558\ub294 \uac83\uc73c\ub85c \uc808\ub300 \ubc1b\uc544\ub4e4\uc77c \uc218 \uc5c6\ub2e4\"\uace0 \ubc18\ubc1c\ud588\ub2e4. \ud558\uc9c0\ub9cc \uc77c\ubcf8\uc740 2017\ub144 \ubd80\uc0b0 \uc18c\ub140\uc0c1 \ubb38\uc81c \ub2f9\uc2dc \ub098\uac00\ubbf8\ub124 \uc57c\uc2a4\ub9c8\uc0ac \uc8fc\ud55c \ub300\uc0ac\ub97c \uc18c\ud658\uc2dc\ud0a4\ub294 \uac15\uacbd \uc870\uce58\ub97c \ucde8\ud55c \ub4a4\uc5d0\ub294 \ub3c4\ubc1c\uc801\uc778 \uc561\uc158\uc744 \ucde8\ud558\uc9c0 \uc54a\uace0 \uc788\ub2e4. \uc774\ub294 \ud55c\uc77c \uac04\uc758 \uacf5\uc870\ub97c \ud6fc\uc190\uc2dc\ud0ac \uc6b0\ub824\uc640 \ud568\uaed8 \ub0a8\ubd81 \ub300\ud654\u00b7\ubd81\ubbf8 \ub300\ud654\uac00 \ubb34\ub974\uc775\uc740 \uc0c1\ud669\uc5d0\uc11c \ub300\ubd81 \uc678\uad50\uc5d0\uc11c '\uc77c\ubcf8 \ud328\uc2f1'\uc774 \uc77c\uc5b4\ub0a0 \uc218 \uc788\uae30 \ub54c\ubb38\uc778 \uac83\uc73c\ub85c \ubd84\uc11d\ub41c\ub2e4. \ud55c\ud3b8, \uc774\ubc88 \uae30\ub150\uc0ac\uc5d0\uc11c\ub294 \uac00\uc7a5 \uba3c\uc800 \uc77c\ubcf8\uc758 \uc2dd\ubbfc\uc9c0\uac00 \ub41c \uc6b0\ub9ac \ub545\uc774 \ub3c5\ub3c4\ub77c\uba70 \ub3c5\ub3c4 \ubb38\uc81c\uc5d0\uc11c\ub3c4 \ubaa9\uc18c\ub9ac\ub97c \ub192\uc600\ub294\ub370 \uc774\ub294 \uc77c\ubcf8 \uc815\ubd80\uac00 \uace0\uad50 \uad50\uacfc\uc11c \uc9c0\uce68\uc5d0 \ub3c5\ub3c4 \uc601\uc720\uad8c \uc8fc\uc7a5\uc744 \ub123\uace0 \ub3c4\ucfc4\uc758 \uc911\uc2ec\uc5d0 \ub3c5\ub3c4\uc804\uc2dc\uad00\uc744 \uc138\uc6b0\ub294 \ub4f1 \ub3c4\ubc1c\uc774 \uc788\uc790 \uc774\uc5d0 \ub300\ud55c \uc81c\ub3d9\uc744 \uac78 \ud544\uc694\uc131\uc744 \ub290\ub080 \uac83\uc774\ub77c\ub294 \ud3c9\uc774\ub2e4.", "paragraph_answer": "3\u00b71\uc808 \uae30\ub150\uc2dd\uc5d0\uc11c \ubb38\uc7ac\uc778\uc740 \uc77c\ubcf8\uc758 \uc704\uc548\ubd80 \ub9cc\ud589\uc744 \ubc18\uc778\ub95c\uc801 \uc778\uad8c\ubc94\uc8c4\ub85c, \uc77c\ubcf8\uc744 \uac00\ud574\uc790\ub85c \ud45c\ud604\ud558\uba74\uc11c \"\uac00\ud574\uc790\uc778 \uc77c\ubcf8 \uc815\ubd80\uac00 '\ub05d\ub0ac\ub2e4'\ub77c\uace0 \ub9d0\ud574\uc11c\ub294 \uc548 \ub41c\ub2e4\"\uace0 \uac15\ud558\uac8c \ube44\ud310\ud558\uace0 \"\uc77c\ubcf8\uc740 \uc778\ub958 \ubcf4\ud3b8\uc758 \uc591\uc2ec\uc73c\ub85c \uc5ed\uc0ac\uc758 \uc9c4\uc2e4\uacfc \uc815\uc758\ub97c \ub9c8\uc8fc\ud560 \uc218 \uc788\uc5b4\uc57c \ud55c\ub2e4\"\uace0 \ud588\ub2e4. \uc774\uc5d0 \uc77c\ubcf8\uc740 \ud55c\uc77c \uc591\uad6d\uc740 \uc774\ubbf8 \uc704\uc548\ubd80 \ubb38\uc81c\ub97c \ucd5c\uc885\uc801\uc774\uace0 \ubd88\uac00\uc5ed\uc801\uc73c\ub85c \ud574\uacb0\ud588\ub2e4\uba74\uc11c \"\ubb38 \ub300\ud1b5\ub839\uc758 \ubc1c\uc5b8\uc740 \ud55c\uc77c\ud569\uc758\uc5d0 \ubc18\ud558\ub294 \uac83\uc73c\ub85c \uc808\ub300 \ubc1b\uc544\ub4e4\uc77c \uc218 \uc5c6\ub2e4\"\uace0 \ubc18\ubc1c\ud588\ub2e4. \ud558\uc9c0\ub9cc \uc77c\ubcf8\uc740 2017\ub144 \ubd80\uc0b0 \uc18c\ub140\uc0c1 \ubb38\uc81c \ub2f9\uc2dc \ub098\uac00\ubbf8\ub124 \uc57c\uc2a4\ub9c8\uc0ac \uc8fc\ud55c \ub300\uc0ac\ub97c \uc18c\ud658\uc2dc\ud0a4\ub294 \uac15\uacbd \uc870\uce58\ub97c \ucde8\ud55c \ub4a4\uc5d0\ub294 \ub3c4\ubc1c\uc801\uc778 \uc561\uc158\uc744 \ucde8\ud558\uc9c0 \uc54a\uace0 \uc788\ub2e4. \uc774\ub294 \ud55c\uc77c \uac04\uc758 \uacf5\uc870\ub97c \ud6fc\uc190\uc2dc\ud0ac \uc6b0\ub824\uc640 \ud568\uaed8 \ub0a8\ubd81 \ub300\ud654\u00b7\ubd81\ubbf8 \ub300\ud654\uac00 \ubb34\ub974\uc775\uc740 \uc0c1\ud669\uc5d0\uc11c \ub300\ubd81 \uc678\uad50\uc5d0\uc11c '\uc77c\ubcf8 \ud328\uc2f1'\uc774 \uc77c\uc5b4\ub0a0 \uc218 \uc788\uae30 \ub54c\ubb38\uc778 \uac83\uc73c\ub85c \ubd84\uc11d\ub41c\ub2e4. \ud55c\ud3b8, \uc774\ubc88 \uae30\ub150\uc0ac\uc5d0\uc11c\ub294 \uac00\uc7a5 \uba3c\uc800 \uc77c\ubcf8\uc758 \uc2dd\ubbfc\uc9c0\uac00 \ub41c \uc6b0\ub9ac \ub545\uc774 \ub3c5\ub3c4\ub77c\uba70 \ub3c5\ub3c4 \ubb38\uc81c\uc5d0\uc11c\ub3c4 \ubaa9\uc18c\ub9ac\ub97c \ub192\uc600\ub294\ub370 \uc774\ub294 \uc77c\ubcf8 \uc815\ubd80\uac00 \uace0\uad50 \uad50\uacfc\uc11c \uc9c0\uce68\uc5d0 \ub3c5\ub3c4 \uc601\uc720\uad8c \uc8fc\uc7a5\uc744 \ub123\uace0 \ub3c4\ucfc4\uc758 \uc911\uc2ec\uc5d0 \ub3c5\ub3c4\uc804\uc2dc\uad00\uc744 \uc138\uc6b0\ub294 \ub4f1 \ub3c4\ubc1c\uc774 \uc788\uc790 \uc774\uc5d0 \ub300\ud55c \uc81c\ub3d9\uc744 \uac78 \ud544\uc694\uc131\uc744 \ub290\ub080 \uac83\uc774\ub77c\ub294 \ud3c9\uc774\ub2e4.", "sentence_answer": "\uc77c\ubcf8\uc740 2017\ub144 \ubd80\uc0b0 \uc18c\ub140\uc0c1 \ubb38\uc81c \ub2f9\uc2dc \ub098\uac00\ubbf8\ub124 \uc57c\uc2a4\ub9c8\uc0ac \uc8fc\ud55c \ub300\uc0ac\ub97c \uc18c\ud658\uc2dc\ud0a4\ub294 \uac15\uacbd \uc870\uce58\ub97c \ucde8\ud55c \ub4a4\uc5d0\ub294 \ub3c4\ubc1c\uc801\uc778 \uc561\uc158\uc744 \ucde8\ud558\uc9c0 \uc54a\uace0 \uc788\ub2e4.", "paragraph_id": "6580768-16-1", "paragraph_question": "question: 2017\ub144 \uc77c\ubcf8\uc774 \ubd80\uc0b0 \uc18c\ub140\uc0c1 \ubb38\uc81c\uac00 \ubc1c\uc0dd \ud558\uc600\uc744 \ub54c \uc18c\ud658\uc2dc\ud0a8 \uc8fc\ud55c \ub300\uc0ac\ub294 \ub204\uad6c\uc778\uac00?, context: 3\u00b71\uc808 \uae30\ub150\uc2dd\uc5d0\uc11c \ubb38\uc7ac\uc778\uc740 \uc77c\ubcf8\uc758 \uc704\uc548\ubd80 \ub9cc\ud589\uc744 \ubc18\uc778\ub95c\uc801 \uc778\uad8c\ubc94\uc8c4\ub85c, \uc77c\ubcf8\uc744 \uac00\ud574\uc790\ub85c \ud45c\ud604\ud558\uba74\uc11c \"\uac00\ud574\uc790\uc778 \uc77c\ubcf8 \uc815\ubd80\uac00 '\ub05d\ub0ac\ub2e4'\ub77c\uace0 \ub9d0\ud574\uc11c\ub294 \uc548 \ub41c\ub2e4\"\uace0 \uac15\ud558\uac8c \ube44\ud310\ud558\uace0 \"\uc77c\ubcf8\uc740 \uc778\ub958 \ubcf4\ud3b8\uc758 \uc591\uc2ec\uc73c\ub85c \uc5ed\uc0ac\uc758 \uc9c4\uc2e4\uacfc \uc815\uc758\ub97c \ub9c8\uc8fc\ud560 \uc218 \uc788\uc5b4\uc57c \ud55c\ub2e4\"\uace0 \ud588\ub2e4. \uc774\uc5d0 \uc77c\ubcf8\uc740 \ud55c\uc77c \uc591\uad6d\uc740 \uc774\ubbf8 \uc704\uc548\ubd80 \ubb38\uc81c\ub97c \ucd5c\uc885\uc801\uc774\uace0 \ubd88\uac00\uc5ed\uc801\uc73c\ub85c \ud574\uacb0\ud588\ub2e4\uba74\uc11c \"\ubb38 \ub300\ud1b5\ub839\uc758 \ubc1c\uc5b8\uc740 \ud55c\uc77c\ud569\uc758\uc5d0 \ubc18\ud558\ub294 \uac83\uc73c\ub85c \uc808\ub300 \ubc1b\uc544\ub4e4\uc77c \uc218 \uc5c6\ub2e4\"\uace0 \ubc18\ubc1c\ud588\ub2e4. \ud558\uc9c0\ub9cc \uc77c\ubcf8\uc740 2017\ub144 \ubd80\uc0b0 \uc18c\ub140\uc0c1 \ubb38\uc81c \ub2f9\uc2dc \ub098\uac00\ubbf8\ub124 \uc57c\uc2a4\ub9c8\uc0ac \uc8fc\ud55c \ub300\uc0ac\ub97c \uc18c\ud658\uc2dc\ud0a4\ub294 \uac15\uacbd \uc870\uce58\ub97c \ucde8\ud55c \ub4a4\uc5d0\ub294 \ub3c4\ubc1c\uc801\uc778 \uc561\uc158\uc744 \ucde8\ud558\uc9c0 \uc54a\uace0 \uc788\ub2e4. \uc774\ub294 \ud55c\uc77c \uac04\uc758 \uacf5\uc870\ub97c \ud6fc\uc190\uc2dc\ud0ac \uc6b0\ub824\uc640 \ud568\uaed8 \ub0a8\ubd81 \ub300\ud654\u00b7\ubd81\ubbf8 \ub300\ud654\uac00 \ubb34\ub974\uc775\uc740 \uc0c1\ud669\uc5d0\uc11c \ub300\ubd81 \uc678\uad50\uc5d0\uc11c '\uc77c\ubcf8 \ud328\uc2f1'\uc774 \uc77c\uc5b4\ub0a0 \uc218 \uc788\uae30 \ub54c\ubb38\uc778 \uac83\uc73c\ub85c \ubd84\uc11d\ub41c\ub2e4. \ud55c\ud3b8, \uc774\ubc88 \uae30\ub150\uc0ac\uc5d0\uc11c\ub294 \uac00\uc7a5 \uba3c\uc800 \uc77c\ubcf8\uc758 \uc2dd\ubbfc\uc9c0\uac00 \ub41c \uc6b0\ub9ac \ub545\uc774 \ub3c5\ub3c4\ub77c\uba70 \ub3c5\ub3c4 \ubb38\uc81c\uc5d0\uc11c\ub3c4 \ubaa9\uc18c\ub9ac\ub97c \ub192\uc600\ub294\ub370 \uc774\ub294 \uc77c\ubcf8 \uc815\ubd80\uac00 \uace0\uad50 \uad50\uacfc\uc11c \uc9c0\uce68\uc5d0 \ub3c5\ub3c4 \uc601\uc720\uad8c \uc8fc\uc7a5\uc744 \ub123\uace0 \ub3c4\ucfc4\uc758 \uc911\uc2ec\uc5d0 \ub3c5\ub3c4\uc804\uc2dc\uad00\uc744 \uc138\uc6b0\ub294 \ub4f1 \ub3c4\ubc1c\uc774 \uc788\uc790 \uc774\uc5d0 \ub300\ud55c \uc81c\ub3d9\uc744 \uac78 \ud544\uc694\uc131\uc744 \ub290\ub080 \uac83\uc774\ub77c\ub294 \ud3c9\uc774\ub2e4."} {"question": "\uae40\uc9c0\ud6c8\uc774 \ud22c\ud22c 1\uae30\uc2dc\uc808\uc774\uc600\ub358 1994\ub144 \uac00\uc694\ud1b110\uc5d0\uc11c \ubc1b\uc740 \ucd5c\uace0\uc758 \uc0c1\uc740 \ubb34\uc5c7\uc778\uac00?", "paragraph": "\uae40\uc9c0\ud6c8\uc740 1990\ub144\ub300\ubd80\ud130 \uc5c4\uccad\ub09c \uc778\uae30\ub97c \uc5bb\uc5b4\uc11c \ud22c\ud22c \ud65c\ub3d9\uc2dc\uae30\uc5d0 \ub3c8\ub3c4 \ub9ce\uc774 \ubc8c\uc5c8\uc744 \uac83 \uac19\ub2e4\ub294 \uc624\ud574\ub97c \ub9ce\uc774 \ubc1b\uc558\ub294\ub370, \uadf8 \ub2f9\uc2dc\uc5d0 \ub370\ubdd4 \uc804 \uc5f0\uc2b5 \uae30\uac04\uae4c\uc9c0 \ud3ec\ud568\ud574\uc11c \uc57d 2\ub144 \uac04\uc758 \uc218\uc785\uc73c\ub85c \uc57d 1800\ub9cc\uc6d0 \uc815\ub3c4\uc758 \uc218\uc785\ub9cc \ubc1b\uc558\ub2e4\uace0 \ud55c\ub2e4. \ud22c\ud22c 1\uae30 \uc2dc\uc808\uc778 1994\ub144, \uc18c\uc18d\uc0ac\uc5d0\uc11c \ubc25\uc744 \uc81c\uacf5\ud558\uc9c0 \uc54a\uc744 \ub54c\uc5d0\ub294 \uc218\uc911\uc5d0 \ub3c8\uc774 \uc5c6\uc5b4 \uba64\ubc84\ub4e4\uc774 \ubc25\uc744 \uad76\uae30\ub3c4 \ud588\ub2e4\uace0 \ud55c\ub2e4. \uadf8\ub294 1990\ub144\ub300 \ud65c\ub3d9\ud558\ub358 \ub304\uc2a4\uac00\uc218\ub4e4\uc744 \ubcf4\uba74 \ud654\ub824\ud574 \ubcf4\uc774\uace0,\ubc1d\uace0 \uc5ed\ub3d9\uc801\uc778 \ubaa8\uc2b5\ub4e4\uc744 \ubcf4\uc5ec\uc8fc\uc9c0\ub9cc \uacb0\uad6d \ub4b7\uba74\uc5d0\ub294 \ud798\ub4e0 \ub54c\uac00 \uc788\uc5c8\ub2e4\uace0 \ub9d0\ud558\uba74\uc11c, 1994\ub144 \uac00\uc694\ud1b110\uc5d0\uc11c \uace8\ub4e0\ucef5\uc744 \ud0c0\ub358 \ub0a0 \ub0a8\uc790 \uba64\ubc84 \uc14b(\uae40\uc9c0\ud6c8, \uc624\uc9c0\ud6c8, \uc720\ud604\uc7ac)\uc774 \ubc29\uc1a1 \ub05d\ub098\uace0 \uc219\uc18c\uc5d0 \ub4e4\uc5b4\uac00\ubcf4\ub2c8 \ub2f4\ubc30\uac12\uc774 \uc5c6\uc5b4 \uae38\uc5d0 \ub5a8\uc5b4\uc9c4 \ub2f4\ubc30\uaf41\ucd08 \uc911 \uae34 \ub2f4\ubc30\uaf41\ucd08\ub4e4\ub9cc \uc8fc\uc6cc\uc11c \ud53c\uc5c8\ub2e4\ub294 \uc77c\ud654\ub97c \uadf8 \uc608\ub85c \uc81c\uc2dc\ud558\uc600\ub2e4.", "answer": "\uace8\ub4e0\ucef5", "sentence": "\uadf8\ub294 1990\ub144\ub300 \ud65c\ub3d9\ud558\ub358 \ub304\uc2a4\uac00\uc218\ub4e4\uc744 \ubcf4\uba74 \ud654\ub824\ud574 \ubcf4\uc774\uace0,\ubc1d\uace0 \uc5ed\ub3d9\uc801\uc778 \ubaa8\uc2b5\ub4e4\uc744 \ubcf4\uc5ec\uc8fc\uc9c0\ub9cc \uacb0\uad6d \ub4b7\uba74\uc5d0\ub294 \ud798\ub4e0 \ub54c\uac00 \uc788\uc5c8\ub2e4\uace0 \ub9d0\ud558\uba74\uc11c, 1994\ub144 \uac00\uc694\ud1b110\uc5d0\uc11c \uace8\ub4e0\ucef5 \uc744 \ud0c0\ub358 \ub0a0 \ub0a8\uc790 \uba64\ubc84 \uc14b(\uae40\uc9c0\ud6c8, \uc624\uc9c0\ud6c8, \uc720\ud604\uc7ac)\uc774 \ubc29\uc1a1 \ub05d\ub098\uace0 \uc219\uc18c\uc5d0 \ub4e4\uc5b4\uac00\ubcf4\ub2c8 \ub2f4\ubc30\uac12\uc774 \uc5c6\uc5b4 \uae38\uc5d0 \ub5a8\uc5b4\uc9c4 \ub2f4\ubc30\uaf41\ucd08 \uc911 \uae34 \ub2f4\ubc30\uaf41\ucd08\ub4e4\ub9cc \uc8fc\uc6cc\uc11c \ud53c\uc5c8\ub2e4\ub294 \uc77c\ud654\ub97c \uadf8 \uc608\ub85c \uc81c\uc2dc\ud558\uc600\ub2e4.", "paragraph_sentence": "\uae40\uc9c0\ud6c8\uc740 1990\ub144\ub300\ubd80\ud130 \uc5c4\uccad\ub09c \uc778\uae30\ub97c \uc5bb\uc5b4\uc11c \ud22c\ud22c \ud65c\ub3d9\uc2dc\uae30\uc5d0 \ub3c8\ub3c4 \ub9ce\uc774 \ubc8c\uc5c8\uc744 \uac83 \uac19\ub2e4\ub294 \uc624\ud574\ub97c \ub9ce\uc774 \ubc1b\uc558\ub294\ub370, \uadf8 \ub2f9\uc2dc\uc5d0 \ub370\ubdd4 \uc804 \uc5f0\uc2b5 \uae30\uac04\uae4c\uc9c0 \ud3ec\ud568\ud574\uc11c \uc57d 2\ub144 \uac04\uc758 \uc218\uc785\uc73c\ub85c \uc57d 1800\ub9cc\uc6d0 \uc815\ub3c4\uc758 \uc218\uc785\ub9cc \ubc1b\uc558\ub2e4\uace0 \ud55c\ub2e4. \ud22c\ud22c 1\uae30 \uc2dc\uc808\uc778 1994\ub144, \uc18c\uc18d\uc0ac\uc5d0\uc11c \ubc25\uc744 \uc81c\uacf5\ud558\uc9c0 \uc54a\uc744 \ub54c\uc5d0\ub294 \uc218\uc911\uc5d0 \ub3c8\uc774 \uc5c6\uc5b4 \uba64\ubc84\ub4e4\uc774 \ubc25\uc744 \uad76\uae30\ub3c4 \ud588\ub2e4\uace0 \ud55c\ub2e4. \uadf8\ub294 1990\ub144\ub300 \ud65c\ub3d9\ud558\ub358 \ub304\uc2a4\uac00\uc218\ub4e4\uc744 \ubcf4\uba74 \ud654\ub824\ud574 \ubcf4\uc774\uace0,\ubc1d\uace0 \uc5ed\ub3d9\uc801\uc778 \ubaa8\uc2b5\ub4e4\uc744 \ubcf4\uc5ec\uc8fc\uc9c0\ub9cc \uacb0\uad6d \ub4b7\uba74\uc5d0\ub294 \ud798\ub4e0 \ub54c\uac00 \uc788\uc5c8\ub2e4\uace0 \ub9d0\ud558\uba74\uc11c, 1994\ub144 \uac00\uc694\ud1b110\uc5d0\uc11c \uace8\ub4e0\ucef5 \uc744 \ud0c0\ub358 \ub0a0 \ub0a8\uc790 \uba64\ubc84 \uc14b(\uae40\uc9c0\ud6c8, \uc624\uc9c0\ud6c8, \uc720\ud604\uc7ac)\uc774 \ubc29\uc1a1 \ub05d\ub098\uace0 \uc219\uc18c\uc5d0 \ub4e4\uc5b4\uac00\ubcf4\ub2c8 \ub2f4\ubc30\uac12\uc774 \uc5c6\uc5b4 \uae38\uc5d0 \ub5a8\uc5b4\uc9c4 \ub2f4\ubc30\uaf41\ucd08 \uc911 \uae34 \ub2f4\ubc30\uaf41\ucd08\ub4e4\ub9cc \uc8fc\uc6cc\uc11c \ud53c\uc5c8\ub2e4\ub294 \uc77c\ud654\ub97c \uadf8 \uc608\ub85c \uc81c\uc2dc\ud558\uc600\ub2e4. ", "paragraph_answer": "\uae40\uc9c0\ud6c8\uc740 1990\ub144\ub300\ubd80\ud130 \uc5c4\uccad\ub09c \uc778\uae30\ub97c \uc5bb\uc5b4\uc11c \ud22c\ud22c \ud65c\ub3d9\uc2dc\uae30\uc5d0 \ub3c8\ub3c4 \ub9ce\uc774 \ubc8c\uc5c8\uc744 \uac83 \uac19\ub2e4\ub294 \uc624\ud574\ub97c \ub9ce\uc774 \ubc1b\uc558\ub294\ub370, \uadf8 \ub2f9\uc2dc\uc5d0 \ub370\ubdd4 \uc804 \uc5f0\uc2b5 \uae30\uac04\uae4c\uc9c0 \ud3ec\ud568\ud574\uc11c \uc57d 2\ub144 \uac04\uc758 \uc218\uc785\uc73c\ub85c \uc57d 1800\ub9cc\uc6d0 \uc815\ub3c4\uc758 \uc218\uc785\ub9cc \ubc1b\uc558\ub2e4\uace0 \ud55c\ub2e4. \ud22c\ud22c 1\uae30 \uc2dc\uc808\uc778 1994\ub144, \uc18c\uc18d\uc0ac\uc5d0\uc11c \ubc25\uc744 \uc81c\uacf5\ud558\uc9c0 \uc54a\uc744 \ub54c\uc5d0\ub294 \uc218\uc911\uc5d0 \ub3c8\uc774 \uc5c6\uc5b4 \uba64\ubc84\ub4e4\uc774 \ubc25\uc744 \uad76\uae30\ub3c4 \ud588\ub2e4\uace0 \ud55c\ub2e4. \uadf8\ub294 1990\ub144\ub300 \ud65c\ub3d9\ud558\ub358 \ub304\uc2a4\uac00\uc218\ub4e4\uc744 \ubcf4\uba74 \ud654\ub824\ud574 \ubcf4\uc774\uace0,\ubc1d\uace0 \uc5ed\ub3d9\uc801\uc778 \ubaa8\uc2b5\ub4e4\uc744 \ubcf4\uc5ec\uc8fc\uc9c0\ub9cc \uacb0\uad6d \ub4b7\uba74\uc5d0\ub294 \ud798\ub4e0 \ub54c\uac00 \uc788\uc5c8\ub2e4\uace0 \ub9d0\ud558\uba74\uc11c, 1994\ub144 \uac00\uc694\ud1b110\uc5d0\uc11c \uace8\ub4e0\ucef5 \uc744 \ud0c0\ub358 \ub0a0 \ub0a8\uc790 \uba64\ubc84 \uc14b(\uae40\uc9c0\ud6c8, \uc624\uc9c0\ud6c8, \uc720\ud604\uc7ac)\uc774 \ubc29\uc1a1 \ub05d\ub098\uace0 \uc219\uc18c\uc5d0 \ub4e4\uc5b4\uac00\ubcf4\ub2c8 \ub2f4\ubc30\uac12\uc774 \uc5c6\uc5b4 \uae38\uc5d0 \ub5a8\uc5b4\uc9c4 \ub2f4\ubc30\uaf41\ucd08 \uc911 \uae34 \ub2f4\ubc30\uaf41\ucd08\ub4e4\ub9cc \uc8fc\uc6cc\uc11c \ud53c\uc5c8\ub2e4\ub294 \uc77c\ud654\ub97c \uadf8 \uc608\ub85c \uc81c\uc2dc\ud558\uc600\ub2e4.", "sentence_answer": "\uadf8\ub294 1990\ub144\ub300 \ud65c\ub3d9\ud558\ub358 \ub304\uc2a4\uac00\uc218\ub4e4\uc744 \ubcf4\uba74 \ud654\ub824\ud574 \ubcf4\uc774\uace0,\ubc1d\uace0 \uc5ed\ub3d9\uc801\uc778 \ubaa8\uc2b5\ub4e4\uc744 \ubcf4\uc5ec\uc8fc\uc9c0\ub9cc \uacb0\uad6d \ub4b7\uba74\uc5d0\ub294 \ud798\ub4e0 \ub54c\uac00 \uc788\uc5c8\ub2e4\uace0 \ub9d0\ud558\uba74\uc11c, 1994\ub144 \uac00\uc694\ud1b110\uc5d0\uc11c \uace8\ub4e0\ucef5 \uc744 \ud0c0\ub358 \ub0a0 \ub0a8\uc790 \uba64\ubc84 \uc14b(\uae40\uc9c0\ud6c8, \uc624\uc9c0\ud6c8, \uc720\ud604\uc7ac)\uc774 \ubc29\uc1a1 \ub05d\ub098\uace0 \uc219\uc18c\uc5d0 \ub4e4\uc5b4\uac00\ubcf4\ub2c8 \ub2f4\ubc30\uac12\uc774 \uc5c6\uc5b4 \uae38\uc5d0 \ub5a8\uc5b4\uc9c4 \ub2f4\ubc30\uaf41\ucd08 \uc911 \uae34 \ub2f4\ubc30\uaf41\ucd08\ub4e4\ub9cc \uc8fc\uc6cc\uc11c \ud53c\uc5c8\ub2e4\ub294 \uc77c\ud654\ub97c \uadf8 \uc608\ub85c \uc81c\uc2dc\ud558\uc600\ub2e4.", "paragraph_id": "6488559-2-1", "paragraph_question": "question: \uae40\uc9c0\ud6c8\uc774 \ud22c\ud22c 1\uae30\uc2dc\uc808\uc774\uc600\ub358 1994\ub144 \uac00\uc694\ud1b110\uc5d0\uc11c \ubc1b\uc740 \ucd5c\uace0\uc758 \uc0c1\uc740 \ubb34\uc5c7\uc778\uac00?, context: \uae40\uc9c0\ud6c8\uc740 1990\ub144\ub300\ubd80\ud130 \uc5c4\uccad\ub09c \uc778\uae30\ub97c \uc5bb\uc5b4\uc11c \ud22c\ud22c \ud65c\ub3d9\uc2dc\uae30\uc5d0 \ub3c8\ub3c4 \ub9ce\uc774 \ubc8c\uc5c8\uc744 \uac83 \uac19\ub2e4\ub294 \uc624\ud574\ub97c \ub9ce\uc774 \ubc1b\uc558\ub294\ub370, \uadf8 \ub2f9\uc2dc\uc5d0 \ub370\ubdd4 \uc804 \uc5f0\uc2b5 \uae30\uac04\uae4c\uc9c0 \ud3ec\ud568\ud574\uc11c \uc57d 2\ub144 \uac04\uc758 \uc218\uc785\uc73c\ub85c \uc57d 1800\ub9cc\uc6d0 \uc815\ub3c4\uc758 \uc218\uc785\ub9cc \ubc1b\uc558\ub2e4\uace0 \ud55c\ub2e4. \ud22c\ud22c 1\uae30 \uc2dc\uc808\uc778 1994\ub144, \uc18c\uc18d\uc0ac\uc5d0\uc11c \ubc25\uc744 \uc81c\uacf5\ud558\uc9c0 \uc54a\uc744 \ub54c\uc5d0\ub294 \uc218\uc911\uc5d0 \ub3c8\uc774 \uc5c6\uc5b4 \uba64\ubc84\ub4e4\uc774 \ubc25\uc744 \uad76\uae30\ub3c4 \ud588\ub2e4\uace0 \ud55c\ub2e4. \uadf8\ub294 1990\ub144\ub300 \ud65c\ub3d9\ud558\ub358 \ub304\uc2a4\uac00\uc218\ub4e4\uc744 \ubcf4\uba74 \ud654\ub824\ud574 \ubcf4\uc774\uace0,\ubc1d\uace0 \uc5ed\ub3d9\uc801\uc778 \ubaa8\uc2b5\ub4e4\uc744 \ubcf4\uc5ec\uc8fc\uc9c0\ub9cc \uacb0\uad6d \ub4b7\uba74\uc5d0\ub294 \ud798\ub4e0 \ub54c\uac00 \uc788\uc5c8\ub2e4\uace0 \ub9d0\ud558\uba74\uc11c, 1994\ub144 \uac00\uc694\ud1b110\uc5d0\uc11c \uace8\ub4e0\ucef5\uc744 \ud0c0\ub358 \ub0a0 \ub0a8\uc790 \uba64\ubc84 \uc14b(\uae40\uc9c0\ud6c8, \uc624\uc9c0\ud6c8, \uc720\ud604\uc7ac)\uc774 \ubc29\uc1a1 \ub05d\ub098\uace0 \uc219\uc18c\uc5d0 \ub4e4\uc5b4\uac00\ubcf4\ub2c8 \ub2f4\ubc30\uac12\uc774 \uc5c6\uc5b4 \uae38\uc5d0 \ub5a8\uc5b4\uc9c4 \ub2f4\ubc30\uaf41\ucd08 \uc911 \uae34 \ub2f4\ubc30\uaf41\ucd08\ub4e4\ub9cc \uc8fc\uc6cc\uc11c \ud53c\uc5c8\ub2e4\ub294 \uc77c\ud654\ub97c \uadf8 \uc608\ub85c \uc81c\uc2dc\ud558\uc600\ub2e4."} {"question": "\ub0a8\ubd81 \uc815\uc0c1\ud68c\ub2f4\uc740 \ud310\ubb38\uc810\uc5d0\uc11c \uba87 \uc6d4 \uba87\uc77c\uc5d0 \uac1c\ucd5c\ub418\ub294\uac00?", "paragraph": "4\uc6d4 27\uc77c \uc624\uc804 9\uc2dc 30\ubd84\uacbd\uc5d0 \ub0a8\ubd81 \uc815\uc0c1\uc774 \ud310\ubb38\uc810\uc5d0\uc11c \ub9cc\ub098 \uac00\ubcbc\uc6b4 \uc778\uc0ac\ub97c \ub098\ub208 \ub4a4 \ub300\ud55c\ubbfc\uad6d \uad6d\uad70 \uc758\uc7a5\ub300\uc758 \uc57d\uc2dd \uc0ac\uc5f4\uc744 \ubc1b\uace0 \uace7\uc774\uc5b4 100\ubd84\uc5d0 \uac78\uccd0 \uc624\uc804 \ud68c\ub2f4\uc744 \uc9c4\ud589\ud588\ub2e4. \uc624\ud6c4\uc5d0\ub294 \uacf5\ub3d9 \uc2dd\uc218\uc640 30\ubd84 \uac04 \ub3c4\ubcf4\ub2e4\ub9ac\uc5d0\uc11c \uc0ac\uc2e4\uc0c1\uc758 \ub2e8\ub3c5\ud68c\ub2f4\uc774 \uc9c4\ud589\ub418\uc5c8\uace0 \uc624\ud6c4 6\uc2dc\uacbd\uc5d0 \uacf5\ub3d9\uc120\uc5b8\uc778 \ud310\ubb38\uc810 \uc120\uc5b8\uc744 \ubc1c\ud45c\ud558\uc600\ub2e4. \ub0a8\ubd81 \uc815\uc0c1\uc740 \uc774\ub0a0 10\uac1c\uc758 \ud589\uc0ac\ub97c \ud568\uaed8 \uc9c4\ud589\ud588\uc73c\uba70 18\uc2dc 40\ubd84\uacbd \uc2dc\uc791\ub41c \ub9cc\ucc2c\uc740 2\uc2dc\uac04 \ub118\uac8c \uc774\uc5b4\uc838 21\uc2dc 30\ubd84\uacbd\uc5d0\uc11c\uc57c \ub05d\ub0ac\ub2e4. \ub9cc\ucc2c \ud589\uc0ac\uc5d0\ub294 \ub9ac\uc124\uc8fc \uad6d\ubb34\uc704\uc6d0\uc7a5 \ubd80\uc778\ub3c4 \ucc38\uc11d\ud588\ub294\ub370 \uc774\ub294 \ubd81\ud55c\uc774 \uc815\uc0c1 \uad6d\uac00\ub77c\ub294 \uac83\uc744 \uad6d\uc81c\uc0ac\ud68c\uc5d0 \uc778\uc2dd\uc2dc\ud0a4\uae30 \uc704\ud55c \ub178\ub825\uc774\ub77c\ub294 \ud3c9\uc774\ub2e4. \ud310\ubb38\uc810 \uc120\uc5b8\uc740 \ub0a8\ubd81 \uc815\uc0c1\uc774 \uacf5\ub3d9\uc73c\ub85c \uc5b8\ub860 \ubc1c\ud45c\ub97c \ud588\ub294\ub370 \uc774\ub294 \ucd5c\ucd08\uc774\uba70 \ubd81\ud55c \ucd5c\uace0 \uc9c0\ub3c4\uc790\uc758 \uc5f0\uc124\uc774 \ubd81\ud55c \uc678\ubd80\uc5d0\uc11c \uc0dd\uc911\uacc4\ub41c \uac83\ub3c4 \uc774\ubc88\uc774 \ucd5c\ucd08\uc774\ub2e4. \ubb38\uc7ac\uc778\uc740 \ub354 \uc774\uc0c1 \ud55c\ubc18\ub3c4\uc5d0\uc11c \uc804\uc7c1\uc744 \uc5c6\uc744 \uac83\uc774\uace0 \ud3c9\ud654\uc758 \uc2dc\ub300\uac00 \uc5f4\ub9ac\uace0 \uc788\uc74c\uc744 \uc120\uc5b8\ud588\uc73c\uba70, \uae40\uc815\uc740\uc740 \uc804\uc7c1\uc5c6\ub294 \ub545\uc5d0\uc11c \uc628 \uaca8\ub808\uac00 \ubc88\uc601\uacfc \ud589\ubcf5\uc744 \ub204\ub9ac\ub294 \uc0c8 \uc2dc\ub300\ub97c \uc5f4\uc5b4\ub098\uac08 \ud655\uace0\ud55c \uc758\uc9c0\ub97c \uac19\uc774 \ud558\uace0 \uc788\ub2e4\uace0 \ud588\ub2e4.", "answer": "4\uc6d4 27\uc77c", "sentence": "4\uc6d4 27\uc77c \uc624\uc804 9\uc2dc 30\ubd84\uacbd\uc5d0 \ub0a8\ubd81 \uc815\uc0c1\uc774 \ud310\ubb38\uc810\uc5d0\uc11c \ub9cc\ub098 \uac00\ubcbc\uc6b4 \uc778\uc0ac\ub97c \ub098\ub208 \ub4a4 \ub300\ud55c\ubbfc\uad6d \uad6d\uad70 \uc758\uc7a5\ub300\uc758 \uc57d\uc2dd \uc0ac\uc5f4\uc744 \ubc1b\uace0 \uace7\uc774\uc5b4 100\ubd84\uc5d0 \uac78\uccd0 \uc624\uc804 \ud68c\ub2f4\uc744 \uc9c4\ud589\ud588\ub2e4.", "paragraph_sentence": " 4\uc6d4 27\uc77c \uc624\uc804 9\uc2dc 30\ubd84\uacbd\uc5d0 \ub0a8\ubd81 \uc815\uc0c1\uc774 \ud310\ubb38\uc810\uc5d0\uc11c \ub9cc\ub098 \uac00\ubcbc\uc6b4 \uc778\uc0ac\ub97c \ub098\ub208 \ub4a4 \ub300\ud55c\ubbfc\uad6d \uad6d\uad70 \uc758\uc7a5\ub300\uc758 \uc57d\uc2dd \uc0ac\uc5f4\uc744 \ubc1b\uace0 \uace7\uc774\uc5b4 100\ubd84\uc5d0 \uac78\uccd0 \uc624\uc804 \ud68c\ub2f4\uc744 \uc9c4\ud589\ud588\ub2e4. \uc624\ud6c4\uc5d0\ub294 \uacf5\ub3d9 \uc2dd\uc218\uc640 30\ubd84 \uac04 \ub3c4\ubcf4\ub2e4\ub9ac\uc5d0\uc11c \uc0ac\uc2e4\uc0c1\uc758 \ub2e8\ub3c5\ud68c\ub2f4\uc774 \uc9c4\ud589\ub418\uc5c8\uace0 \uc624\ud6c4 6\uc2dc\uacbd\uc5d0 \uacf5\ub3d9\uc120\uc5b8\uc778 \ud310\ubb38\uc810 \uc120\uc5b8\uc744 \ubc1c\ud45c\ud558\uc600\ub2e4. \ub0a8\ubd81 \uc815\uc0c1\uc740 \uc774\ub0a0 10\uac1c\uc758 \ud589\uc0ac\ub97c \ud568\uaed8 \uc9c4\ud589\ud588\uc73c\uba70 18\uc2dc 40\ubd84\uacbd \uc2dc\uc791\ub41c \ub9cc\ucc2c\uc740 2\uc2dc\uac04 \ub118\uac8c \uc774\uc5b4\uc838 21\uc2dc 30\ubd84\uacbd\uc5d0\uc11c\uc57c \ub05d\ub0ac\ub2e4. \ub9cc\ucc2c \ud589\uc0ac\uc5d0\ub294 \ub9ac\uc124\uc8fc \uad6d\ubb34\uc704\uc6d0\uc7a5 \ubd80\uc778\ub3c4 \ucc38\uc11d\ud588\ub294\ub370 \uc774\ub294 \ubd81\ud55c\uc774 \uc815\uc0c1 \uad6d\uac00\ub77c\ub294 \uac83\uc744 \uad6d\uc81c\uc0ac\ud68c\uc5d0 \uc778\uc2dd\uc2dc\ud0a4\uae30 \uc704\ud55c \ub178\ub825\uc774\ub77c\ub294 \ud3c9\uc774\ub2e4. \ud310\ubb38\uc810 \uc120\uc5b8\uc740 \ub0a8\ubd81 \uc815\uc0c1\uc774 \uacf5\ub3d9\uc73c\ub85c \uc5b8\ub860 \ubc1c\ud45c\ub97c \ud588\ub294\ub370 \uc774\ub294 \ucd5c\ucd08\uc774\uba70 \ubd81\ud55c \ucd5c\uace0 \uc9c0\ub3c4\uc790\uc758 \uc5f0\uc124\uc774 \ubd81\ud55c \uc678\ubd80\uc5d0\uc11c \uc0dd\uc911\uacc4\ub41c \uac83\ub3c4 \uc774\ubc88\uc774 \ucd5c\ucd08\uc774\ub2e4. \ubb38\uc7ac\uc778\uc740 \ub354 \uc774\uc0c1 \ud55c\ubc18\ub3c4\uc5d0\uc11c \uc804\uc7c1\uc744 \uc5c6\uc744 \uac83\uc774\uace0 \ud3c9\ud654\uc758 \uc2dc\ub300\uac00 \uc5f4\ub9ac\uace0 \uc788\uc74c\uc744 \uc120\uc5b8\ud588\uc73c\uba70, \uae40\uc815\uc740\uc740 \uc804\uc7c1\uc5c6\ub294 \ub545\uc5d0\uc11c \uc628 \uaca8\ub808\uac00 \ubc88\uc601\uacfc \ud589\ubcf5\uc744 \ub204\ub9ac\ub294 \uc0c8 \uc2dc\ub300\ub97c \uc5f4\uc5b4\ub098\uac08 \ud655\uace0\ud55c \uc758\uc9c0\ub97c \uac19\uc774 \ud558\uace0 \uc788\ub2e4\uace0 \ud588\ub2e4.", "paragraph_answer": " 4\uc6d4 27\uc77c \uc624\uc804 9\uc2dc 30\ubd84\uacbd\uc5d0 \ub0a8\ubd81 \uc815\uc0c1\uc774 \ud310\ubb38\uc810\uc5d0\uc11c \ub9cc\ub098 \uac00\ubcbc\uc6b4 \uc778\uc0ac\ub97c \ub098\ub208 \ub4a4 \ub300\ud55c\ubbfc\uad6d \uad6d\uad70 \uc758\uc7a5\ub300\uc758 \uc57d\uc2dd \uc0ac\uc5f4\uc744 \ubc1b\uace0 \uace7\uc774\uc5b4 100\ubd84\uc5d0 \uac78\uccd0 \uc624\uc804 \ud68c\ub2f4\uc744 \uc9c4\ud589\ud588\ub2e4. \uc624\ud6c4\uc5d0\ub294 \uacf5\ub3d9 \uc2dd\uc218\uc640 30\ubd84 \uac04 \ub3c4\ubcf4\ub2e4\ub9ac\uc5d0\uc11c \uc0ac\uc2e4\uc0c1\uc758 \ub2e8\ub3c5\ud68c\ub2f4\uc774 \uc9c4\ud589\ub418\uc5c8\uace0 \uc624\ud6c4 6\uc2dc\uacbd\uc5d0 \uacf5\ub3d9\uc120\uc5b8\uc778 \ud310\ubb38\uc810 \uc120\uc5b8\uc744 \ubc1c\ud45c\ud558\uc600\ub2e4. \ub0a8\ubd81 \uc815\uc0c1\uc740 \uc774\ub0a0 10\uac1c\uc758 \ud589\uc0ac\ub97c \ud568\uaed8 \uc9c4\ud589\ud588\uc73c\uba70 18\uc2dc 40\ubd84\uacbd \uc2dc\uc791\ub41c \ub9cc\ucc2c\uc740 2\uc2dc\uac04 \ub118\uac8c \uc774\uc5b4\uc838 21\uc2dc 30\ubd84\uacbd\uc5d0\uc11c\uc57c \ub05d\ub0ac\ub2e4. \ub9cc\ucc2c \ud589\uc0ac\uc5d0\ub294 \ub9ac\uc124\uc8fc \uad6d\ubb34\uc704\uc6d0\uc7a5 \ubd80\uc778\ub3c4 \ucc38\uc11d\ud588\ub294\ub370 \uc774\ub294 \ubd81\ud55c\uc774 \uc815\uc0c1 \uad6d\uac00\ub77c\ub294 \uac83\uc744 \uad6d\uc81c\uc0ac\ud68c\uc5d0 \uc778\uc2dd\uc2dc\ud0a4\uae30 \uc704\ud55c \ub178\ub825\uc774\ub77c\ub294 \ud3c9\uc774\ub2e4. \ud310\ubb38\uc810 \uc120\uc5b8\uc740 \ub0a8\ubd81 \uc815\uc0c1\uc774 \uacf5\ub3d9\uc73c\ub85c \uc5b8\ub860 \ubc1c\ud45c\ub97c \ud588\ub294\ub370 \uc774\ub294 \ucd5c\ucd08\uc774\uba70 \ubd81\ud55c \ucd5c\uace0 \uc9c0\ub3c4\uc790\uc758 \uc5f0\uc124\uc774 \ubd81\ud55c \uc678\ubd80\uc5d0\uc11c \uc0dd\uc911\uacc4\ub41c \uac83\ub3c4 \uc774\ubc88\uc774 \ucd5c\ucd08\uc774\ub2e4. \ubb38\uc7ac\uc778\uc740 \ub354 \uc774\uc0c1 \ud55c\ubc18\ub3c4\uc5d0\uc11c \uc804\uc7c1\uc744 \uc5c6\uc744 \uac83\uc774\uace0 \ud3c9\ud654\uc758 \uc2dc\ub300\uac00 \uc5f4\ub9ac\uace0 \uc788\uc74c\uc744 \uc120\uc5b8\ud588\uc73c\uba70, \uae40\uc815\uc740\uc740 \uc804\uc7c1\uc5c6\ub294 \ub545\uc5d0\uc11c \uc628 \uaca8\ub808\uac00 \ubc88\uc601\uacfc \ud589\ubcf5\uc744 \ub204\ub9ac\ub294 \uc0c8 \uc2dc\ub300\ub97c \uc5f4\uc5b4\ub098\uac08 \ud655\uace0\ud55c \uc758\uc9c0\ub97c \uac19\uc774 \ud558\uace0 \uc788\ub2e4\uace0 \ud588\ub2e4.", "sentence_answer": " 4\uc6d4 27\uc77c \uc624\uc804 9\uc2dc 30\ubd84\uacbd\uc5d0 \ub0a8\ubd81 \uc815\uc0c1\uc774 \ud310\ubb38\uc810\uc5d0\uc11c \ub9cc\ub098 \uac00\ubcbc\uc6b4 \uc778\uc0ac\ub97c \ub098\ub208 \ub4a4 \ub300\ud55c\ubbfc\uad6d \uad6d\uad70 \uc758\uc7a5\ub300\uc758 \uc57d\uc2dd \uc0ac\uc5f4\uc744 \ubc1b\uace0 \uace7\uc774\uc5b4 100\ubd84\uc5d0 \uac78\uccd0 \uc624\uc804 \ud68c\ub2f4\uc744 \uc9c4\ud589\ud588\ub2e4.", "paragraph_id": "6546926-49-0", "paragraph_question": "question: \ub0a8\ubd81 \uc815\uc0c1\ud68c\ub2f4\uc740 \ud310\ubb38\uc810\uc5d0\uc11c \uba87 \uc6d4 \uba87\uc77c\uc5d0 \uac1c\ucd5c\ub418\ub294\uac00?, context: 4\uc6d4 27\uc77c \uc624\uc804 9\uc2dc 30\ubd84\uacbd\uc5d0 \ub0a8\ubd81 \uc815\uc0c1\uc774 \ud310\ubb38\uc810\uc5d0\uc11c \ub9cc\ub098 \uac00\ubcbc\uc6b4 \uc778\uc0ac\ub97c \ub098\ub208 \ub4a4 \ub300\ud55c\ubbfc\uad6d \uad6d\uad70 \uc758\uc7a5\ub300\uc758 \uc57d\uc2dd \uc0ac\uc5f4\uc744 \ubc1b\uace0 \uace7\uc774\uc5b4 100\ubd84\uc5d0 \uac78\uccd0 \uc624\uc804 \ud68c\ub2f4\uc744 \uc9c4\ud589\ud588\ub2e4. \uc624\ud6c4\uc5d0\ub294 \uacf5\ub3d9 \uc2dd\uc218\uc640 30\ubd84 \uac04 \ub3c4\ubcf4\ub2e4\ub9ac\uc5d0\uc11c \uc0ac\uc2e4\uc0c1\uc758 \ub2e8\ub3c5\ud68c\ub2f4\uc774 \uc9c4\ud589\ub418\uc5c8\uace0 \uc624\ud6c4 6\uc2dc\uacbd\uc5d0 \uacf5\ub3d9\uc120\uc5b8\uc778 \ud310\ubb38\uc810 \uc120\uc5b8\uc744 \ubc1c\ud45c\ud558\uc600\ub2e4. \ub0a8\ubd81 \uc815\uc0c1\uc740 \uc774\ub0a0 10\uac1c\uc758 \ud589\uc0ac\ub97c \ud568\uaed8 \uc9c4\ud589\ud588\uc73c\uba70 18\uc2dc 40\ubd84\uacbd \uc2dc\uc791\ub41c \ub9cc\ucc2c\uc740 2\uc2dc\uac04 \ub118\uac8c \uc774\uc5b4\uc838 21\uc2dc 30\ubd84\uacbd\uc5d0\uc11c\uc57c \ub05d\ub0ac\ub2e4. \ub9cc\ucc2c \ud589\uc0ac\uc5d0\ub294 \ub9ac\uc124\uc8fc \uad6d\ubb34\uc704\uc6d0\uc7a5 \ubd80\uc778\ub3c4 \ucc38\uc11d\ud588\ub294\ub370 \uc774\ub294 \ubd81\ud55c\uc774 \uc815\uc0c1 \uad6d\uac00\ub77c\ub294 \uac83\uc744 \uad6d\uc81c\uc0ac\ud68c\uc5d0 \uc778\uc2dd\uc2dc\ud0a4\uae30 \uc704\ud55c \ub178\ub825\uc774\ub77c\ub294 \ud3c9\uc774\ub2e4. \ud310\ubb38\uc810 \uc120\uc5b8\uc740 \ub0a8\ubd81 \uc815\uc0c1\uc774 \uacf5\ub3d9\uc73c\ub85c \uc5b8\ub860 \ubc1c\ud45c\ub97c \ud588\ub294\ub370 \uc774\ub294 \ucd5c\ucd08\uc774\uba70 \ubd81\ud55c \ucd5c\uace0 \uc9c0\ub3c4\uc790\uc758 \uc5f0\uc124\uc774 \ubd81\ud55c \uc678\ubd80\uc5d0\uc11c \uc0dd\uc911\uacc4\ub41c \uac83\ub3c4 \uc774\ubc88\uc774 \ucd5c\ucd08\uc774\ub2e4. \ubb38\uc7ac\uc778\uc740 \ub354 \uc774\uc0c1 \ud55c\ubc18\ub3c4\uc5d0\uc11c \uc804\uc7c1\uc744 \uc5c6\uc744 \uac83\uc774\uace0 \ud3c9\ud654\uc758 \uc2dc\ub300\uac00 \uc5f4\ub9ac\uace0 \uc788\uc74c\uc744 \uc120\uc5b8\ud588\uc73c\uba70, \uae40\uc815\uc740\uc740 \uc804\uc7c1\uc5c6\ub294 \ub545\uc5d0\uc11c \uc628 \uaca8\ub808\uac00 \ubc88\uc601\uacfc \ud589\ubcf5\uc744 \ub204\ub9ac\ub294 \uc0c8 \uc2dc\ub300\ub97c \uc5f4\uc5b4\ub098\uac08 \ud655\uace0\ud55c \uc758\uc9c0\ub97c \uac19\uc774 \ud558\uace0 \uc788\ub2e4\uace0 \ud588\ub2e4."} {"question": "\uc2b9\ubd80\uc870\uc791\uc740 \uc5b4\ub5a4 \ubc95\uc5d0 \uc800\ucd09\ub418\ub294 \uc704\ubc18\ud589\uc704\uc778\uac00?", "paragraph": "2\uc6d4 28\uc77c, \ub300\uad6c\uc9c0\ubc29\uac80\ucc30\uccad\uc740 \uad6d\ubbfc\uccb4\uc721\uc9c4\ud765\ubc95 \uc704\ubc18 \ud610\uc758\ub85c LG \ud2b8\uc708\uc2a4 \uc120\uc218\uc778 \uae40\uc131\ud604\uc744 \uccb4\ud3ec\ud588\ub2e4. \uae40\uc131\ud604\uc740 8\uc2dc\uac04 \uac00\ub7c9 \uc870\uc0ac\ub97c \ubc1b\uc558\uace0, \ub125\uc13c \uc2dc\uc808\uc774\ub358 2011\uc2dc\uc98c 4~5\uc6d4 \ube0c\ub85c\ucee4\uc640 \uc9dc\uace0 \ub450 \ucc28\ub840\uc5d0 \uac78\uccd0 \uc77c\ubd80\ub7ec '1\ud68c \uccab \ubcfc\ub137'\uc744 \ub358\uc838 \uc2b9\ubd80\ub97c \uc870\uc791\ud558\uace0 \uc774\uc5d0 \ub530\ub978 \uc0ac\ub840\uae08 1000\ub9cc \uc6d0\uc744 \ucc59\uae34 \ud610\uc758\ub97c \ubc1b\uc558\ub2e4. \uac80\ucc30\uc740 \uae40\uc131\ud604\uc774 \ub300\ud559 \uc57c\uad6c \ucd9c\uc2e0 \uace0\uad50 \uc120\ubc30\uc778 \ube0c\ub85c\ucee4 \uae40 \ubaa8\uc528\uc640\uc758 \uc790\uae08 \ud750\ub984 \ub4f1 \uad6c\uccb4\uc801 \uc815\ud669\uc744 \ubb3b\uc790 \ub300\ubd80\ubd84 \ud610\uc758\ub97c \uc2dc\uc778\ud588\ub2e4\uace0 \uc804\ud588\ub2e4. 2\uc6d4 29\uc77c, \ub300\uad6c\uc9c0\uac80\uc740 \uc2b9\ubd80\uc870\uc791\uc5d0 \uac00\ub2f4\ud55c \ud610\uc758\ub85c \uae40\uc131\ud604\uc5d0 \ub300\ud574 \uad6c\uc18d \uc601\uc7a5\uc744 \uccad\uad6c\ud588\uace0, 3\uc6d4 1\uc77c, \ub300\uad6c\uc9c0\ubc29\ubc95\uc6d0\uc740 \"\uc99d\uac70\ub97c \uc5c6\uc560\uac70\ub098 \ub3c4\uc8fc\ud560 \uc6b0\ub824\uac00 \uc788\ub2e4\"\uba70 \uc601\uc7a5\uc744 \ubc1c\ubd80\ud588\uc73c\uba70, \uae40\uc131\ud604\uc740 \uad6c\uc18d \uc18c\uac10\ub418\uc5c8\ub2e4. \uae40\uc131\ud604\uc758 \ubcc0\ud638\uc0ac\ub294 \"\uc2b9\ubd80\uc870\uc791\uc744 \ud55c \uc0ac\uc2e4\uc740 \uc778\uc815\ud558\uc9c0\ub9cc, \uc2b9\ubd80\uc870\uc791\uc5d0 \uc2e4\ud328\ud55c \ub4a4\ubd80\ud130 \ube0c\ub85c\ucee4\ub4e4\uc758 \ud611\ubc15\uacfc \uacf5\uac08\uc5d0 \ubabb \uc774\uaca8 \uc790\uc2e0\uc774 \ubc1b\uc740 \uc0ac\ub840\ube44\uc5d0 \uc9d1 \ubcf4\uc99d\uae08\uae4c\uc9c0 \ubcf4\ud0dc 3000\ub9cc\uc6d0\uc744 \ub72f\uae30\uae30\ub3c4 \ud588\ub2e4\"\uace0 \uc8fc\uc7a5\ud588\ub2e4.", "answer": "\uad6d\ubbfc\uccb4\uc721\uc9c4\ud765\ubc95", "sentence": "2\uc6d4 28\uc77c, \ub300\uad6c\uc9c0\ubc29\uac80\ucc30\uccad\uc740 \uad6d\ubbfc\uccb4\uc721\uc9c4\ud765\ubc95 \uc704\ubc18 \ud610\uc758\ub85c LG \ud2b8\uc708\uc2a4 \uc120\uc218\uc778 \uae40\uc131\ud604\uc744 \uccb4\ud3ec\ud588\ub2e4.", "paragraph_sentence": " 2\uc6d4 28\uc77c, \ub300\uad6c\uc9c0\ubc29\uac80\ucc30\uccad\uc740 \uad6d\ubbfc\uccb4\uc721\uc9c4\ud765\ubc95 \uc704\ubc18 \ud610\uc758\ub85c LG \ud2b8\uc708\uc2a4 \uc120\uc218\uc778 \uae40\uc131\ud604\uc744 \uccb4\ud3ec\ud588\ub2e4. \uae40\uc131\ud604\uc740 8\uc2dc\uac04 \uac00\ub7c9 \uc870\uc0ac\ub97c \ubc1b\uc558\uace0, \ub125\uc13c \uc2dc\uc808\uc774\ub358 2011\uc2dc\uc98c 4~5\uc6d4 \ube0c\ub85c\ucee4\uc640 \uc9dc\uace0 \ub450 \ucc28\ub840\uc5d0 \uac78\uccd0 \uc77c\ubd80\ub7ec '1\ud68c \uccab \ubcfc\ub137'\uc744 \ub358\uc838 \uc2b9\ubd80\ub97c \uc870\uc791\ud558\uace0 \uc774\uc5d0 \ub530\ub978 \uc0ac\ub840\uae08 1000\ub9cc \uc6d0\uc744 \ucc59\uae34 \ud610\uc758\ub97c \ubc1b\uc558\ub2e4. \uac80\ucc30\uc740 \uae40\uc131\ud604\uc774 \ub300\ud559 \uc57c\uad6c \ucd9c\uc2e0 \uace0\uad50 \uc120\ubc30\uc778 \ube0c\ub85c\ucee4 \uae40 \ubaa8\uc528\uc640\uc758 \uc790\uae08 \ud750\ub984 \ub4f1 \uad6c\uccb4\uc801 \uc815\ud669\uc744 \ubb3b\uc790 \ub300\ubd80\ubd84 \ud610\uc758\ub97c \uc2dc\uc778\ud588\ub2e4\uace0 \uc804\ud588\ub2e4. 2\uc6d4 29\uc77c, \ub300\uad6c\uc9c0\uac80\uc740 \uc2b9\ubd80\uc870\uc791\uc5d0 \uac00\ub2f4\ud55c \ud610\uc758\ub85c \uae40\uc131\ud604\uc5d0 \ub300\ud574 \uad6c\uc18d \uc601\uc7a5\uc744 \uccad\uad6c\ud588\uace0, 3\uc6d4 1\uc77c, \ub300\uad6c\uc9c0\ubc29\ubc95\uc6d0\uc740 \"\uc99d\uac70\ub97c \uc5c6\uc560\uac70\ub098 \ub3c4\uc8fc\ud560 \uc6b0\ub824\uac00 \uc788\ub2e4\"\uba70 \uc601\uc7a5\uc744 \ubc1c\ubd80\ud588\uc73c\uba70, \uae40\uc131\ud604\uc740 \uad6c\uc18d \uc18c\uac10\ub418\uc5c8\ub2e4. \uae40\uc131\ud604\uc758 \ubcc0\ud638\uc0ac\ub294 \"\uc2b9\ubd80\uc870\uc791\uc744 \ud55c \uc0ac\uc2e4\uc740 \uc778\uc815\ud558\uc9c0\ub9cc, \uc2b9\ubd80\uc870\uc791\uc5d0 \uc2e4\ud328\ud55c \ub4a4\ubd80\ud130 \ube0c\ub85c\ucee4\ub4e4\uc758 \ud611\ubc15\uacfc \uacf5\uac08\uc5d0 \ubabb \uc774\uaca8 \uc790\uc2e0\uc774 \ubc1b\uc740 \uc0ac\ub840\ube44\uc5d0 \uc9d1 \ubcf4\uc99d\uae08\uae4c\uc9c0 \ubcf4\ud0dc 3000\ub9cc\uc6d0\uc744 \ub72f\uae30\uae30\ub3c4 \ud588\ub2e4\"\uace0 \uc8fc\uc7a5\ud588\ub2e4.", "paragraph_answer": "2\uc6d4 28\uc77c, \ub300\uad6c\uc9c0\ubc29\uac80\ucc30\uccad\uc740 \uad6d\ubbfc\uccb4\uc721\uc9c4\ud765\ubc95 \uc704\ubc18 \ud610\uc758\ub85c LG \ud2b8\uc708\uc2a4 \uc120\uc218\uc778 \uae40\uc131\ud604\uc744 \uccb4\ud3ec\ud588\ub2e4. \uae40\uc131\ud604\uc740 8\uc2dc\uac04 \uac00\ub7c9 \uc870\uc0ac\ub97c \ubc1b\uc558\uace0, \ub125\uc13c \uc2dc\uc808\uc774\ub358 2011\uc2dc\uc98c 4~5\uc6d4 \ube0c\ub85c\ucee4\uc640 \uc9dc\uace0 \ub450 \ucc28\ub840\uc5d0 \uac78\uccd0 \uc77c\ubd80\ub7ec '1\ud68c \uccab \ubcfc\ub137'\uc744 \ub358\uc838 \uc2b9\ubd80\ub97c \uc870\uc791\ud558\uace0 \uc774\uc5d0 \ub530\ub978 \uc0ac\ub840\uae08 1000\ub9cc \uc6d0\uc744 \ucc59\uae34 \ud610\uc758\ub97c \ubc1b\uc558\ub2e4. \uac80\ucc30\uc740 \uae40\uc131\ud604\uc774 \ub300\ud559 \uc57c\uad6c \ucd9c\uc2e0 \uace0\uad50 \uc120\ubc30\uc778 \ube0c\ub85c\ucee4 \uae40 \ubaa8\uc528\uc640\uc758 \uc790\uae08 \ud750\ub984 \ub4f1 \uad6c\uccb4\uc801 \uc815\ud669\uc744 \ubb3b\uc790 \ub300\ubd80\ubd84 \ud610\uc758\ub97c \uc2dc\uc778\ud588\ub2e4\uace0 \uc804\ud588\ub2e4. 2\uc6d4 29\uc77c, \ub300\uad6c\uc9c0\uac80\uc740 \uc2b9\ubd80\uc870\uc791\uc5d0 \uac00\ub2f4\ud55c \ud610\uc758\ub85c \uae40\uc131\ud604\uc5d0 \ub300\ud574 \uad6c\uc18d \uc601\uc7a5\uc744 \uccad\uad6c\ud588\uace0, 3\uc6d4 1\uc77c, \ub300\uad6c\uc9c0\ubc29\ubc95\uc6d0\uc740 \"\uc99d\uac70\ub97c \uc5c6\uc560\uac70\ub098 \ub3c4\uc8fc\ud560 \uc6b0\ub824\uac00 \uc788\ub2e4\"\uba70 \uc601\uc7a5\uc744 \ubc1c\ubd80\ud588\uc73c\uba70, \uae40\uc131\ud604\uc740 \uad6c\uc18d \uc18c\uac10\ub418\uc5c8\ub2e4. \uae40\uc131\ud604\uc758 \ubcc0\ud638\uc0ac\ub294 \"\uc2b9\ubd80\uc870\uc791\uc744 \ud55c \uc0ac\uc2e4\uc740 \uc778\uc815\ud558\uc9c0\ub9cc, \uc2b9\ubd80\uc870\uc791\uc5d0 \uc2e4\ud328\ud55c \ub4a4\ubd80\ud130 \ube0c\ub85c\ucee4\ub4e4\uc758 \ud611\ubc15\uacfc \uacf5\uac08\uc5d0 \ubabb \uc774\uaca8 \uc790\uc2e0\uc774 \ubc1b\uc740 \uc0ac\ub840\ube44\uc5d0 \uc9d1 \ubcf4\uc99d\uae08\uae4c\uc9c0 \ubcf4\ud0dc 3000\ub9cc\uc6d0\uc744 \ub72f\uae30\uae30\ub3c4 \ud588\ub2e4\"\uace0 \uc8fc\uc7a5\ud588\ub2e4.", "sentence_answer": "2\uc6d4 28\uc77c, \ub300\uad6c\uc9c0\ubc29\uac80\ucc30\uccad\uc740 \uad6d\ubbfc\uccb4\uc721\uc9c4\ud765\ubc95 \uc704\ubc18 \ud610\uc758\ub85c LG \ud2b8\uc708\uc2a4 \uc120\uc218\uc778 \uae40\uc131\ud604\uc744 \uccb4\ud3ec\ud588\ub2e4.", "paragraph_id": "5780952-8-0", "paragraph_question": "question: \uc2b9\ubd80\uc870\uc791\uc740 \uc5b4\ub5a4 \ubc95\uc5d0 \uc800\ucd09\ub418\ub294 \uc704\ubc18\ud589\uc704\uc778\uac00?, context: 2\uc6d4 28\uc77c, \ub300\uad6c\uc9c0\ubc29\uac80\ucc30\uccad\uc740 \uad6d\ubbfc\uccb4\uc721\uc9c4\ud765\ubc95 \uc704\ubc18 \ud610\uc758\ub85c LG \ud2b8\uc708\uc2a4 \uc120\uc218\uc778 \uae40\uc131\ud604\uc744 \uccb4\ud3ec\ud588\ub2e4. \uae40\uc131\ud604\uc740 8\uc2dc\uac04 \uac00\ub7c9 \uc870\uc0ac\ub97c \ubc1b\uc558\uace0, \ub125\uc13c \uc2dc\uc808\uc774\ub358 2011\uc2dc\uc98c 4~5\uc6d4 \ube0c\ub85c\ucee4\uc640 \uc9dc\uace0 \ub450 \ucc28\ub840\uc5d0 \uac78\uccd0 \uc77c\ubd80\ub7ec '1\ud68c \uccab \ubcfc\ub137'\uc744 \ub358\uc838 \uc2b9\ubd80\ub97c \uc870\uc791\ud558\uace0 \uc774\uc5d0 \ub530\ub978 \uc0ac\ub840\uae08 1000\ub9cc \uc6d0\uc744 \ucc59\uae34 \ud610\uc758\ub97c \ubc1b\uc558\ub2e4. \uac80\ucc30\uc740 \uae40\uc131\ud604\uc774 \ub300\ud559 \uc57c\uad6c \ucd9c\uc2e0 \uace0\uad50 \uc120\ubc30\uc778 \ube0c\ub85c\ucee4 \uae40 \ubaa8\uc528\uc640\uc758 \uc790\uae08 \ud750\ub984 \ub4f1 \uad6c\uccb4\uc801 \uc815\ud669\uc744 \ubb3b\uc790 \ub300\ubd80\ubd84 \ud610\uc758\ub97c \uc2dc\uc778\ud588\ub2e4\uace0 \uc804\ud588\ub2e4. 2\uc6d4 29\uc77c, \ub300\uad6c\uc9c0\uac80\uc740 \uc2b9\ubd80\uc870\uc791\uc5d0 \uac00\ub2f4\ud55c \ud610\uc758\ub85c \uae40\uc131\ud604\uc5d0 \ub300\ud574 \uad6c\uc18d \uc601\uc7a5\uc744 \uccad\uad6c\ud588\uace0, 3\uc6d4 1\uc77c, \ub300\uad6c\uc9c0\ubc29\ubc95\uc6d0\uc740 \"\uc99d\uac70\ub97c \uc5c6\uc560\uac70\ub098 \ub3c4\uc8fc\ud560 \uc6b0\ub824\uac00 \uc788\ub2e4\"\uba70 \uc601\uc7a5\uc744 \ubc1c\ubd80\ud588\uc73c\uba70, \uae40\uc131\ud604\uc740 \uad6c\uc18d \uc18c\uac10\ub418\uc5c8\ub2e4. \uae40\uc131\ud604\uc758 \ubcc0\ud638\uc0ac\ub294 \"\uc2b9\ubd80\uc870\uc791\uc744 \ud55c \uc0ac\uc2e4\uc740 \uc778\uc815\ud558\uc9c0\ub9cc, \uc2b9\ubd80\uc870\uc791\uc5d0 \uc2e4\ud328\ud55c \ub4a4\ubd80\ud130 \ube0c\ub85c\ucee4\ub4e4\uc758 \ud611\ubc15\uacfc \uacf5\uac08\uc5d0 \ubabb \uc774\uaca8 \uc790\uc2e0\uc774 \ubc1b\uc740 \uc0ac\ub840\ube44\uc5d0 \uc9d1 \ubcf4\uc99d\uae08\uae4c\uc9c0 \ubcf4\ud0dc 3000\ub9cc\uc6d0\uc744 \ub72f\uae30\uae30\ub3c4 \ud588\ub2e4\"\uace0 \uc8fc\uc7a5\ud588\ub2e4."} {"question": "\uc774\ud718\uc18c \ubc15\uc0ac\uc758 \uc804\uacf5\ubd84\uc57c\ub294 \ubb34\uc5c7\uc778\uac00?", "paragraph": "20\uc138\uae30 \ud6c4\ubc18 \uc785\uc790\ubb3c\ub9ac\ud559\uc5d0\uc11c \uc790\ubc1c\uc801\uc73c\ub85c \ub300\uce6d\uc131\uc774 \ubd80\uc11c\uc9c4 \uac8c\uc774\uc9c0 \uc774\ub860\uc758 \uc7ac\uaddc\uaca9\ud654 \ubb38\uc81c\uc758 \ud574\uacb0\uc5d0 \uacb0\uc815\uc801\uc778 \uc5ed\ud560\uc744 \ud558\uc600\uace0, \ub9f5\uc2dc \ucffc\ud06c\uc758 \uc9c8\ub7c9\uc744 \uc608\uce21\ud558\uc5ec \uadf8 \ud0d0\uc0c9\uc5d0 \uacf5\ud5cc\ud558\uc600\ub2e4. \ubb3c\ub9ac\ud559\uc790\ub85c\uc11c \ubcf8\uaca9\uc801\uc778 \ud65c\ub3d9\uc744 \uc2dc\uc791\ud55c \uc774\ub798 \uc57d 20\ub144\uac04 \ubaa8\ub450 110\ud3b8\uc758 \ub17c\ubb38\uc744 \ubc1c\ud45c\ud558\uc600\uc73c\uba70, \uc774 \uc911 77\ud3b8\uc758 \ub17c\ubb38\uc774 \ud559\uc220\uc9c0\uc5d0 \uac8c\uc7ac\ub410\ub2e4. 10\ud68c \uc774\uc0c1 \uc778\uc6a9\ub41c \ub17c\ubb38\uc740 \uc774 \uc911 69\ud3b8\uc5d0 \ub2ec\ud558\uba70, 500\ud68c \uc774\uc0c1 \uc778\uc6a9\ub41c \ub17c\ubb38\uc740 \ubaa8\ub450 8\ud3b8\uc774\ub2e4. 2013\ub144 10\uc6d4 \ud604\uc7ac \uadf8\uc758 \ubaa8\ub4e0 \ub17c\ubb38\ub4e4\uc740 13,400\ud68c \uc774\uc0c1 \uc778\uc6a9\ub418\uace0 \uc788\ub2e4. \ub300\ud45c\uc801\uc778 \uc81c\uc790\ub85c \uac15\uc8fc\uc0c1 \uace0\ub824\ub300\ud559\uad50 \ubb3c\ub9ac\ud559\uacfc \uba85\uc608\uad50\uc218\uac00 \uc788\uc73c\uba70 \ub610\ud55c \uae40\uc9c4\uba85\uc758 \uc18c\uc124 \u300a\ubb34\uad81\ud654 \uaf43\uc774 \ud53c\uc5c8\uc2b5\ub2c8\ub2e4\u300b\uc5d0 \ub4f1\uc7a5\ud558\ub294 \uac00\uc0c1 \uc778\ubb3c \u2018\uc774\uc6a9\ud6c4\u2019\uc758 \ubaa8\ud2f0\ube0c\uac00 \ub41c \uc778\ubb3c\uc774\uae30\ub3c4 \ud558\ub2e4.", "answer": "\ubb3c\ub9ac\ud559", "sentence": "20\uc138\uae30 \ud6c4\ubc18 \uc785\uc790 \ubb3c\ub9ac\ud559 \uc5d0\uc11c \uc790\ubc1c\uc801\uc73c\ub85c \ub300\uce6d\uc131\uc774 \ubd80\uc11c\uc9c4 \uac8c\uc774\uc9c0 \uc774\ub860\uc758 \uc7ac\uaddc\uaca9\ud654 \ubb38\uc81c\uc758 \ud574\uacb0\uc5d0 \uacb0\uc815\uc801\uc778 \uc5ed\ud560\uc744 \ud558\uc600\uace0, \ub9f5\uc2dc \ucffc\ud06c\uc758 \uc9c8\ub7c9\uc744 \uc608\uce21\ud558\uc5ec \uadf8 \ud0d0\uc0c9\uc5d0 \uacf5\ud5cc\ud558\uc600\ub2e4.", "paragraph_sentence": " 20\uc138\uae30 \ud6c4\ubc18 \uc785\uc790 \ubb3c\ub9ac\ud559 \uc5d0\uc11c \uc790\ubc1c\uc801\uc73c\ub85c \ub300\uce6d\uc131\uc774 \ubd80\uc11c\uc9c4 \uac8c\uc774\uc9c0 \uc774\ub860\uc758 \uc7ac\uaddc\uaca9\ud654 \ubb38\uc81c\uc758 \ud574\uacb0\uc5d0 \uacb0\uc815\uc801\uc778 \uc5ed\ud560\uc744 \ud558\uc600\uace0, \ub9f5\uc2dc \ucffc\ud06c\uc758 \uc9c8\ub7c9\uc744 \uc608\uce21\ud558\uc5ec \uadf8 \ud0d0\uc0c9\uc5d0 \uacf5\ud5cc\ud558\uc600\ub2e4. \ubb3c\ub9ac\ud559\uc790\ub85c\uc11c \ubcf8\uaca9\uc801\uc778 \ud65c\ub3d9\uc744 \uc2dc\uc791\ud55c \uc774\ub798 \uc57d 20\ub144\uac04 \ubaa8\ub450 110\ud3b8\uc758 \ub17c\ubb38\uc744 \ubc1c\ud45c\ud558\uc600\uc73c\uba70, \uc774 \uc911 77\ud3b8\uc758 \ub17c\ubb38\uc774 \ud559\uc220\uc9c0\uc5d0 \uac8c\uc7ac\ub410\ub2e4. 10\ud68c \uc774\uc0c1 \uc778\uc6a9\ub41c \ub17c\ubb38\uc740 \uc774 \uc911 69\ud3b8\uc5d0 \ub2ec\ud558\uba70, 500\ud68c \uc774\uc0c1 \uc778\uc6a9\ub41c \ub17c\ubb38\uc740 \ubaa8\ub450 8\ud3b8\uc774\ub2e4. 2013\ub144 10\uc6d4 \ud604\uc7ac \uadf8\uc758 \ubaa8\ub4e0 \ub17c\ubb38\ub4e4\uc740 13,400\ud68c \uc774\uc0c1 \uc778\uc6a9\ub418\uace0 \uc788\ub2e4. \ub300\ud45c\uc801\uc778 \uc81c\uc790\ub85c \uac15\uc8fc\uc0c1 \uace0\ub824\ub300\ud559\uad50 \ubb3c\ub9ac\ud559\uacfc \uba85\uc608\uad50\uc218\uac00 \uc788\uc73c\uba70 \ub610\ud55c \uae40\uc9c4\uba85\uc758 \uc18c\uc124 \u300a\ubb34\uad81\ud654 \uaf43\uc774 \ud53c\uc5c8\uc2b5\ub2c8\ub2e4\u300b\uc5d0 \ub4f1\uc7a5\ud558\ub294 \uac00\uc0c1 \uc778\ubb3c \u2018\uc774\uc6a9\ud6c4\u2019\uc758 \ubaa8\ud2f0\ube0c\uac00 \ub41c \uc778\ubb3c\uc774\uae30\ub3c4 \ud558\ub2e4.", "paragraph_answer": "20\uc138\uae30 \ud6c4\ubc18 \uc785\uc790 \ubb3c\ub9ac\ud559 \uc5d0\uc11c \uc790\ubc1c\uc801\uc73c\ub85c \ub300\uce6d\uc131\uc774 \ubd80\uc11c\uc9c4 \uac8c\uc774\uc9c0 \uc774\ub860\uc758 \uc7ac\uaddc\uaca9\ud654 \ubb38\uc81c\uc758 \ud574\uacb0\uc5d0 \uacb0\uc815\uc801\uc778 \uc5ed\ud560\uc744 \ud558\uc600\uace0, \ub9f5\uc2dc \ucffc\ud06c\uc758 \uc9c8\ub7c9\uc744 \uc608\uce21\ud558\uc5ec \uadf8 \ud0d0\uc0c9\uc5d0 \uacf5\ud5cc\ud558\uc600\ub2e4. \ubb3c\ub9ac\ud559\uc790\ub85c\uc11c \ubcf8\uaca9\uc801\uc778 \ud65c\ub3d9\uc744 \uc2dc\uc791\ud55c \uc774\ub798 \uc57d 20\ub144\uac04 \ubaa8\ub450 110\ud3b8\uc758 \ub17c\ubb38\uc744 \ubc1c\ud45c\ud558\uc600\uc73c\uba70, \uc774 \uc911 77\ud3b8\uc758 \ub17c\ubb38\uc774 \ud559\uc220\uc9c0\uc5d0 \uac8c\uc7ac\ub410\ub2e4. 10\ud68c \uc774\uc0c1 \uc778\uc6a9\ub41c \ub17c\ubb38\uc740 \uc774 \uc911 69\ud3b8\uc5d0 \ub2ec\ud558\uba70, 500\ud68c \uc774\uc0c1 \uc778\uc6a9\ub41c \ub17c\ubb38\uc740 \ubaa8\ub450 8\ud3b8\uc774\ub2e4. 2013\ub144 10\uc6d4 \ud604\uc7ac \uadf8\uc758 \ubaa8\ub4e0 \ub17c\ubb38\ub4e4\uc740 13,400\ud68c \uc774\uc0c1 \uc778\uc6a9\ub418\uace0 \uc788\ub2e4. \ub300\ud45c\uc801\uc778 \uc81c\uc790\ub85c \uac15\uc8fc\uc0c1 \uace0\ub824\ub300\ud559\uad50 \ubb3c\ub9ac\ud559\uacfc \uba85\uc608\uad50\uc218\uac00 \uc788\uc73c\uba70 \ub610\ud55c \uae40\uc9c4\uba85\uc758 \uc18c\uc124 \u300a\ubb34\uad81\ud654 \uaf43\uc774 \ud53c\uc5c8\uc2b5\ub2c8\ub2e4\u300b\uc5d0 \ub4f1\uc7a5\ud558\ub294 \uac00\uc0c1 \uc778\ubb3c \u2018\uc774\uc6a9\ud6c4\u2019\uc758 \ubaa8\ud2f0\ube0c\uac00 \ub41c \uc778\ubb3c\uc774\uae30\ub3c4 \ud558\ub2e4.", "sentence_answer": "20\uc138\uae30 \ud6c4\ubc18 \uc785\uc790 \ubb3c\ub9ac\ud559 \uc5d0\uc11c \uc790\ubc1c\uc801\uc73c\ub85c \ub300\uce6d\uc131\uc774 \ubd80\uc11c\uc9c4 \uac8c\uc774\uc9c0 \uc774\ub860\uc758 \uc7ac\uaddc\uaca9\ud654 \ubb38\uc81c\uc758 \ud574\uacb0\uc5d0 \uacb0\uc815\uc801\uc778 \uc5ed\ud560\uc744 \ud558\uc600\uace0, \ub9f5\uc2dc \ucffc\ud06c\uc758 \uc9c8\ub7c9\uc744 \uc608\uce21\ud558\uc5ec \uadf8 \ud0d0\uc0c9\uc5d0 \uacf5\ud5cc\ud558\uc600\ub2e4.", "paragraph_id": "6547981-0-1", "paragraph_question": "question: \uc774\ud718\uc18c \ubc15\uc0ac\uc758 \uc804\uacf5\ubd84\uc57c\ub294 \ubb34\uc5c7\uc778\uac00?, context: 20\uc138\uae30 \ud6c4\ubc18 \uc785\uc790\ubb3c\ub9ac\ud559\uc5d0\uc11c \uc790\ubc1c\uc801\uc73c\ub85c \ub300\uce6d\uc131\uc774 \ubd80\uc11c\uc9c4 \uac8c\uc774\uc9c0 \uc774\ub860\uc758 \uc7ac\uaddc\uaca9\ud654 \ubb38\uc81c\uc758 \ud574\uacb0\uc5d0 \uacb0\uc815\uc801\uc778 \uc5ed\ud560\uc744 \ud558\uc600\uace0, \ub9f5\uc2dc \ucffc\ud06c\uc758 \uc9c8\ub7c9\uc744 \uc608\uce21\ud558\uc5ec \uadf8 \ud0d0\uc0c9\uc5d0 \uacf5\ud5cc\ud558\uc600\ub2e4. \ubb3c\ub9ac\ud559\uc790\ub85c\uc11c \ubcf8\uaca9\uc801\uc778 \ud65c\ub3d9\uc744 \uc2dc\uc791\ud55c \uc774\ub798 \uc57d 20\ub144\uac04 \ubaa8\ub450 110\ud3b8\uc758 \ub17c\ubb38\uc744 \ubc1c\ud45c\ud558\uc600\uc73c\uba70, \uc774 \uc911 77\ud3b8\uc758 \ub17c\ubb38\uc774 \ud559\uc220\uc9c0\uc5d0 \uac8c\uc7ac\ub410\ub2e4. 10\ud68c \uc774\uc0c1 \uc778\uc6a9\ub41c \ub17c\ubb38\uc740 \uc774 \uc911 69\ud3b8\uc5d0 \ub2ec\ud558\uba70, 500\ud68c \uc774\uc0c1 \uc778\uc6a9\ub41c \ub17c\ubb38\uc740 \ubaa8\ub450 8\ud3b8\uc774\ub2e4. 2013\ub144 10\uc6d4 \ud604\uc7ac \uadf8\uc758 \ubaa8\ub4e0 \ub17c\ubb38\ub4e4\uc740 13,400\ud68c \uc774\uc0c1 \uc778\uc6a9\ub418\uace0 \uc788\ub2e4. \ub300\ud45c\uc801\uc778 \uc81c\uc790\ub85c \uac15\uc8fc\uc0c1 \uace0\ub824\ub300\ud559\uad50 \ubb3c\ub9ac\ud559\uacfc \uba85\uc608\uad50\uc218\uac00 \uc788\uc73c\uba70 \ub610\ud55c \uae40\uc9c4\uba85\uc758 \uc18c\uc124 \u300a\ubb34\uad81\ud654 \uaf43\uc774 \ud53c\uc5c8\uc2b5\ub2c8\ub2e4\u300b\uc5d0 \ub4f1\uc7a5\ud558\ub294 \uac00\uc0c1 \uc778\ubb3c \u2018\uc774\uc6a9\ud6c4\u2019\uc758 \ubaa8\ud2f0\ube0c\uac00 \ub41c \uc778\ubb3c\uc774\uae30\ub3c4 \ud558\ub2e4."} {"question": "1935\ub144 \ud788\ud2c0\ub7ec\uac00 \uc18c\uac1c\ud55c \uac83\uc73c\ub85c \uc544\ub9ac\uc544\uc778\uacfc \uc720\ub300\uc778\uc758 \uacb0\ud63c\uc744 \uae08\uc9c0\uc2dc\ud0a8 \ubc95\uc740 \ubb34\uc5c7\uc778\uac00?", "paragraph": "1935\ub144, \ud788\ud2c0\ub7ec\ub294 \ub258\ub978\ubca0\ub974\ud06c\ubc95\uc744 \uc18c\uac1c\ud588\ub294\ub370, \uc774 \ubc95\uc740 \uc544\ub9ac\uc548\uc774 \uc720\ub300\uc778\uacfc \uc131\uc801\uad00\uacc4\ub97c \ub9fa\uac70\ub098 \uacb0\ud63c\uc744 \ud558\ub294 \uac83\uc744 \uae08\uc9c0\uc2dc\ud0a4\ub294 \uac83\uc774\ub2e4. \ube44\ub85d \uc774 \ubc95\uc774 \ub098\uc911\uc5d0 \uc9d1\uc2dc\uc640 \ud751\uc778 \uadf8\ub9ac\uace0 \uadf8\ub4e4\uc758 \uc0ac\uc0dd\uc544 \ub610\ud55c \ud3ec\ud568\ud558\uac8c \ub418\uc5c8\uc9c0\ub9cc (\ub3c5\uc77c \ud608\ud1b5\uacfc \ub3c5\uc77c \uba85\uc608\ub97c \uc9c0\ud0a4\ub294 \ubc95) \uc774 \ubc95\uc73c\ub85c \uc778\ud558\uc5ec \uc720\ub300\uc778\ub4e4\uc740 \uadf8\ub4e4\uc758 \uc2dc\ubbfc\uad8c\uc744 \ubc15\ud0c8\ub2f9\ud588\ub2e4. \uc774\uc640 \ub3d9\uc2dc\uc5d0 \ub098\uce58\ub294 \uc774\ub7ec\ud55c \uc720\ub300\uc778 \uc81c\ud55c\ubc95\uc744 \uc815\ub2f9\ud654\ud558\uae30 \uc704\ud574 \uc778\uc885\uc624\uc5fc \uac1c\ub150\uc744 \ub110\ub9ac \uc54c\ub9ac\ub294 \uacfc\uc7a5\ub41c \uc120\uc804\uc744 \ud3bc\ucce4\ub2e4. \ud788\ud2c0\ub7ec\ub294 \u201c\ud608\ud1b5 \ubc95\u201d\uc744 \ubb38\uc81c\uc5d0 \ub300\ud55c \ud569\ubc95\uc801\uc778 \uaddc\uc81c\uac00 \ubbf8\ub798\uc5d0 \uc77c\uc5b4\ub098\uac8c \ub420 \uc2e4\ud328\ub97c \ud1b5\ud574 \ubc95\uc774 \uad6d\uac00 \uc0ac\ud68c\uc8fc\uc758 \uc815\ub2f9\uc758 \ub9c8\uc9c0\ub9c9 \ud574\uacb0\ucc45\uc774 \ub420 \uac83\uc784\uc744 \ub9d0\ud588\ub2e4. \ud788\ud2c0\ub7ec\ub294 \ub9cc\uc57d \uc720\ub300\uc778 \ubb38\uc81c\uac00 \uc774\uc640 \uac19\uc740 \ubc95\ub4e4\ub85c \ud574\uacb0\ub418\uc9c0 \uc54a\ub294\ub2e4\uba74, \ubc95\uc5d0 \uc758\ud55c \ucd5c\ud6c4\uc758 \ud574\uacb0\ucc45\uc744 \uc81c\uc2dc\ud558\uae30 \uc704\ud574 \uad6d\uac00\uc0ac\ud68c\uc8fc\uc758 \ub2f9\uc774 \uc774\uac83\uc744 \ub9e1\uc544\uc57c \ud55c\ub2e4\uace0 \ub9d0\ud588\ub2e4. \u201c\ucd5c\ud6c4\uc758 \ud574\uacb0\ucc45\u201d \ub610\ub294 \u201c\uad81\uadf9\uc801 \ud574\uacb0\u201d\uc740 \ub098\uce58\uc5d0\uac8c \uc720\ub300\uc778 \ubab0\uc0b4\uc815\ucc45\uc758 \uc644\uace1\ud55c \ud45c\ud604\uc774\uc600\ub2e4. 1939\ub144 1\uc6d4, \uadf8\ub294 \uacf5\uac1c\uc5f0\uc124\uc5d0\uc11c \uc774\ub807\uac8c \ub9d0\ud588\ub2e4.", "answer": "\ub258\ub978\ubca0\ub974\ud06c\ubc95", "sentence": "1935\ub144, \ud788\ud2c0\ub7ec\ub294 \ub258\ub978\ubca0\ub974\ud06c\ubc95 \uc744 \uc18c\uac1c\ud588\ub294\ub370, \uc774 \ubc95\uc740 \uc544\ub9ac\uc548\uc774 \uc720\ub300\uc778\uacfc \uc131\uc801\uad00\uacc4\ub97c \ub9fa\uac70\ub098 \uacb0\ud63c\uc744 \ud558\ub294 \uac83\uc744 \uae08\uc9c0\uc2dc\ud0a4\ub294 \uac83\uc774\ub2e4.", "paragraph_sentence": " 1935\ub144, \ud788\ud2c0\ub7ec\ub294 \ub258\ub978\ubca0\ub974\ud06c\ubc95 \uc744 \uc18c\uac1c\ud588\ub294\ub370, \uc774 \ubc95\uc740 \uc544\ub9ac\uc548\uc774 \uc720\ub300\uc778\uacfc \uc131\uc801\uad00\uacc4\ub97c \ub9fa\uac70\ub098 \uacb0\ud63c\uc744 \ud558\ub294 \uac83\uc744 \uae08\uc9c0\uc2dc\ud0a4\ub294 \uac83\uc774\ub2e4. \ube44\ub85d \uc774 \ubc95\uc774 \ub098\uc911\uc5d0 \uc9d1\uc2dc\uc640 \ud751\uc778 \uadf8\ub9ac\uace0 \uadf8\ub4e4\uc758 \uc0ac\uc0dd\uc544 \ub610\ud55c \ud3ec\ud568\ud558\uac8c \ub418\uc5c8\uc9c0\ub9cc (\ub3c5\uc77c \ud608\ud1b5\uacfc \ub3c5\uc77c \uba85\uc608\ub97c \uc9c0\ud0a4\ub294 \ubc95) \uc774 \ubc95\uc73c\ub85c \uc778\ud558\uc5ec \uc720\ub300\uc778\ub4e4\uc740 \uadf8\ub4e4\uc758 \uc2dc\ubbfc\uad8c\uc744 \ubc15\ud0c8\ub2f9\ud588\ub2e4. \uc774\uc640 \ub3d9\uc2dc\uc5d0 \ub098\uce58\ub294 \uc774\ub7ec\ud55c \uc720\ub300\uc778 \uc81c\ud55c\ubc95\uc744 \uc815\ub2f9\ud654\ud558\uae30 \uc704\ud574 \uc778\uc885\uc624\uc5fc \uac1c\ub150\uc744 \ub110\ub9ac \uc54c\ub9ac\ub294 \uacfc\uc7a5\ub41c \uc120\uc804\uc744 \ud3bc\ucce4\ub2e4. \ud788\ud2c0\ub7ec\ub294 \u201c\ud608\ud1b5 \ubc95\u201d\uc744 \ubb38\uc81c\uc5d0 \ub300\ud55c \ud569\ubc95\uc801\uc778 \uaddc\uc81c\uac00 \ubbf8\ub798\uc5d0 \uc77c\uc5b4\ub098\uac8c \ub420 \uc2e4\ud328\ub97c \ud1b5\ud574 \ubc95\uc774 \uad6d\uac00 \uc0ac\ud68c\uc8fc\uc758 \uc815\ub2f9\uc758 \ub9c8\uc9c0\ub9c9 \ud574\uacb0\ucc45\uc774 \ub420 \uac83\uc784\uc744 \ub9d0\ud588\ub2e4. \ud788\ud2c0\ub7ec\ub294 \ub9cc\uc57d \uc720\ub300\uc778 \ubb38\uc81c\uac00 \uc774\uc640 \uac19\uc740 \ubc95\ub4e4\ub85c \ud574\uacb0\ub418\uc9c0 \uc54a\ub294\ub2e4\uba74, \ubc95\uc5d0 \uc758\ud55c \ucd5c\ud6c4\uc758 \ud574\uacb0\ucc45\uc744 \uc81c\uc2dc\ud558\uae30 \uc704\ud574 \uad6d\uac00\uc0ac\ud68c\uc8fc\uc758 \ub2f9\uc774 \uc774\uac83\uc744 \ub9e1\uc544\uc57c \ud55c\ub2e4\uace0 \ub9d0\ud588\ub2e4. \u201c\ucd5c\ud6c4\uc758 \ud574\uacb0\ucc45\u201d \ub610\ub294 \u201c\uad81\uadf9\uc801 \ud574\uacb0\u201d\uc740 \ub098\uce58\uc5d0\uac8c \uc720\ub300\uc778 \ubab0\uc0b4\uc815\ucc45\uc758 \uc644\uace1\ud55c \ud45c\ud604\uc774\uc600\ub2e4. 1939\ub144 1\uc6d4, \uadf8\ub294 \uacf5\uac1c\uc5f0\uc124\uc5d0\uc11c \uc774\ub807\uac8c \ub9d0\ud588\ub2e4.", "paragraph_answer": "1935\ub144, \ud788\ud2c0\ub7ec\ub294 \ub258\ub978\ubca0\ub974\ud06c\ubc95 \uc744 \uc18c\uac1c\ud588\ub294\ub370, \uc774 \ubc95\uc740 \uc544\ub9ac\uc548\uc774 \uc720\ub300\uc778\uacfc \uc131\uc801\uad00\uacc4\ub97c \ub9fa\uac70\ub098 \uacb0\ud63c\uc744 \ud558\ub294 \uac83\uc744 \uae08\uc9c0\uc2dc\ud0a4\ub294 \uac83\uc774\ub2e4. \ube44\ub85d \uc774 \ubc95\uc774 \ub098\uc911\uc5d0 \uc9d1\uc2dc\uc640 \ud751\uc778 \uadf8\ub9ac\uace0 \uadf8\ub4e4\uc758 \uc0ac\uc0dd\uc544 \ub610\ud55c \ud3ec\ud568\ud558\uac8c \ub418\uc5c8\uc9c0\ub9cc (\ub3c5\uc77c \ud608\ud1b5\uacfc \ub3c5\uc77c \uba85\uc608\ub97c \uc9c0\ud0a4\ub294 \ubc95) \uc774 \ubc95\uc73c\ub85c \uc778\ud558\uc5ec \uc720\ub300\uc778\ub4e4\uc740 \uadf8\ub4e4\uc758 \uc2dc\ubbfc\uad8c\uc744 \ubc15\ud0c8\ub2f9\ud588\ub2e4. \uc774\uc640 \ub3d9\uc2dc\uc5d0 \ub098\uce58\ub294 \uc774\ub7ec\ud55c \uc720\ub300\uc778 \uc81c\ud55c\ubc95\uc744 \uc815\ub2f9\ud654\ud558\uae30 \uc704\ud574 \uc778\uc885\uc624\uc5fc \uac1c\ub150\uc744 \ub110\ub9ac \uc54c\ub9ac\ub294 \uacfc\uc7a5\ub41c \uc120\uc804\uc744 \ud3bc\ucce4\ub2e4. \ud788\ud2c0\ub7ec\ub294 \u201c\ud608\ud1b5 \ubc95\u201d\uc744 \ubb38\uc81c\uc5d0 \ub300\ud55c \ud569\ubc95\uc801\uc778 \uaddc\uc81c\uac00 \ubbf8\ub798\uc5d0 \uc77c\uc5b4\ub098\uac8c \ub420 \uc2e4\ud328\ub97c \ud1b5\ud574 \ubc95\uc774 \uad6d\uac00 \uc0ac\ud68c\uc8fc\uc758 \uc815\ub2f9\uc758 \ub9c8\uc9c0\ub9c9 \ud574\uacb0\ucc45\uc774 \ub420 \uac83\uc784\uc744 \ub9d0\ud588\ub2e4. \ud788\ud2c0\ub7ec\ub294 \ub9cc\uc57d \uc720\ub300\uc778 \ubb38\uc81c\uac00 \uc774\uc640 \uac19\uc740 \ubc95\ub4e4\ub85c \ud574\uacb0\ub418\uc9c0 \uc54a\ub294\ub2e4\uba74, \ubc95\uc5d0 \uc758\ud55c \ucd5c\ud6c4\uc758 \ud574\uacb0\ucc45\uc744 \uc81c\uc2dc\ud558\uae30 \uc704\ud574 \uad6d\uac00\uc0ac\ud68c\uc8fc\uc758 \ub2f9\uc774 \uc774\uac83\uc744 \ub9e1\uc544\uc57c \ud55c\ub2e4\uace0 \ub9d0\ud588\ub2e4. \u201c\ucd5c\ud6c4\uc758 \ud574\uacb0\ucc45\u201d \ub610\ub294 \u201c\uad81\uadf9\uc801 \ud574\uacb0\u201d\uc740 \ub098\uce58\uc5d0\uac8c \uc720\ub300\uc778 \ubab0\uc0b4\uc815\ucc45\uc758 \uc644\uace1\ud55c \ud45c\ud604\uc774\uc600\ub2e4. 1939\ub144 1\uc6d4, \uadf8\ub294 \uacf5\uac1c\uc5f0\uc124\uc5d0\uc11c \uc774\ub807\uac8c \ub9d0\ud588\ub2e4.", "sentence_answer": "1935\ub144, \ud788\ud2c0\ub7ec\ub294 \ub258\ub978\ubca0\ub974\ud06c\ubc95 \uc744 \uc18c\uac1c\ud588\ub294\ub370, \uc774 \ubc95\uc740 \uc544\ub9ac\uc548\uc774 \uc720\ub300\uc778\uacfc \uc131\uc801\uad00\uacc4\ub97c \ub9fa\uac70\ub098 \uacb0\ud63c\uc744 \ud558\ub294 \uac83\uc744 \uae08\uc9c0\uc2dc\ud0a4\ub294 \uac83\uc774\ub2e4.", "paragraph_id": "6571960-7-0", "paragraph_question": "question: 1935\ub144 \ud788\ud2c0\ub7ec\uac00 \uc18c\uac1c\ud55c \uac83\uc73c\ub85c \uc544\ub9ac\uc544\uc778\uacfc \uc720\ub300\uc778\uc758 \uacb0\ud63c\uc744 \uae08\uc9c0\uc2dc\ud0a8 \ubc95\uc740 \ubb34\uc5c7\uc778\uac00?, context: 1935\ub144, \ud788\ud2c0\ub7ec\ub294 \ub258\ub978\ubca0\ub974\ud06c\ubc95\uc744 \uc18c\uac1c\ud588\ub294\ub370, \uc774 \ubc95\uc740 \uc544\ub9ac\uc548\uc774 \uc720\ub300\uc778\uacfc \uc131\uc801\uad00\uacc4\ub97c \ub9fa\uac70\ub098 \uacb0\ud63c\uc744 \ud558\ub294 \uac83\uc744 \uae08\uc9c0\uc2dc\ud0a4\ub294 \uac83\uc774\ub2e4. \ube44\ub85d \uc774 \ubc95\uc774 \ub098\uc911\uc5d0 \uc9d1\uc2dc\uc640 \ud751\uc778 \uadf8\ub9ac\uace0 \uadf8\ub4e4\uc758 \uc0ac\uc0dd\uc544 \ub610\ud55c \ud3ec\ud568\ud558\uac8c \ub418\uc5c8\uc9c0\ub9cc (\ub3c5\uc77c \ud608\ud1b5\uacfc \ub3c5\uc77c \uba85\uc608\ub97c \uc9c0\ud0a4\ub294 \ubc95) \uc774 \ubc95\uc73c\ub85c \uc778\ud558\uc5ec \uc720\ub300\uc778\ub4e4\uc740 \uadf8\ub4e4\uc758 \uc2dc\ubbfc\uad8c\uc744 \ubc15\ud0c8\ub2f9\ud588\ub2e4. \uc774\uc640 \ub3d9\uc2dc\uc5d0 \ub098\uce58\ub294 \uc774\ub7ec\ud55c \uc720\ub300\uc778 \uc81c\ud55c\ubc95\uc744 \uc815\ub2f9\ud654\ud558\uae30 \uc704\ud574 \uc778\uc885\uc624\uc5fc \uac1c\ub150\uc744 \ub110\ub9ac \uc54c\ub9ac\ub294 \uacfc\uc7a5\ub41c \uc120\uc804\uc744 \ud3bc\ucce4\ub2e4. \ud788\ud2c0\ub7ec\ub294 \u201c\ud608\ud1b5 \ubc95\u201d\uc744 \ubb38\uc81c\uc5d0 \ub300\ud55c \ud569\ubc95\uc801\uc778 \uaddc\uc81c\uac00 \ubbf8\ub798\uc5d0 \uc77c\uc5b4\ub098\uac8c \ub420 \uc2e4\ud328\ub97c \ud1b5\ud574 \ubc95\uc774 \uad6d\uac00 \uc0ac\ud68c\uc8fc\uc758 \uc815\ub2f9\uc758 \ub9c8\uc9c0\ub9c9 \ud574\uacb0\ucc45\uc774 \ub420 \uac83\uc784\uc744 \ub9d0\ud588\ub2e4. \ud788\ud2c0\ub7ec\ub294 \ub9cc\uc57d \uc720\ub300\uc778 \ubb38\uc81c\uac00 \uc774\uc640 \uac19\uc740 \ubc95\ub4e4\ub85c \ud574\uacb0\ub418\uc9c0 \uc54a\ub294\ub2e4\uba74, \ubc95\uc5d0 \uc758\ud55c \ucd5c\ud6c4\uc758 \ud574\uacb0\ucc45\uc744 \uc81c\uc2dc\ud558\uae30 \uc704\ud574 \uad6d\uac00\uc0ac\ud68c\uc8fc\uc758 \ub2f9\uc774 \uc774\uac83\uc744 \ub9e1\uc544\uc57c \ud55c\ub2e4\uace0 \ub9d0\ud588\ub2e4. \u201c\ucd5c\ud6c4\uc758 \ud574\uacb0\ucc45\u201d \ub610\ub294 \u201c\uad81\uadf9\uc801 \ud574\uacb0\u201d\uc740 \ub098\uce58\uc5d0\uac8c \uc720\ub300\uc778 \ubab0\uc0b4\uc815\ucc45\uc758 \uc644\uace1\ud55c \ud45c\ud604\uc774\uc600\ub2e4. 1939\ub144 1\uc6d4, \uadf8\ub294 \uacf5\uac1c\uc5f0\uc124\uc5d0\uc11c \uc774\ub807\uac8c \ub9d0\ud588\ub2e4."} {"question": "\uc131\ub0a8\uc2dc\ubbfc\uc0ac\ud68c\ub2e8\uccb4\ud611\uc758\ud68c\uac00 \uc774\uc7ac\uba85 \uc2dc\uc7a5\uc758 \ub17c\ubb38 \ud45c\uc808 \uc758\ud639\uc744 \ud574\uba85\ud558\ub77c\uace0 \uc694\uad6c\ud55c \ub300\ud559\uad50\ub294?", "paragraph": "\uc774\uc7ac\uba85 \uc2dc\uc7a5\uc758 \ub17c\ubb38 \ud45c\uc808 \uc758\ud639\uc740 2013\ub144 9\uc6d4 \ubbf8\ub514\uc5b4\uc6cc\uce58 \uc0b0\ud558 \uc5f0\uad6c\uc9c4\uc2e4\uc131\uac80\uc99d\uc13c\ud130\uac00 \"\ub17c\ubb38\uc758 50\u223c98%\uac00 \ud45c\uc808\ub85c \uc758\uc2ec\ub41c\ub2e4\"\uace0 \uc81c\uae30\ud558\uba74\uc11c \ucd09\ubc1c\ub410\ub2e4. \uadf8\ud574 12\uc6d4 \uc131\ub0a8\uc2dc\ubbfc\uc0ac\ud68c\ub2e8\uccb4\ud611\uc758\ud68c\ub294 \uc774 \uc2dc\uc7a5\uacfc \uac00\ucc9c\ub300\uc5d0 \ud574\uba85\uc744 \uc694\uad6c\ud588\ub2e4. \uc774\ud6c4 \uac00\ucc9c\ub300\ub294 \uc5f0\uad6c\uc724\ub9ac\uc704 \uc870\uc0ac \uc808\ucc28\ub97c \uc9c4\ud589\ud588\ub2e4. \"5\ub144\uc774 \uc9c0\ub098 \ud559\uce59\uc0c1 \uc2ec\uc0ac\ud560 \uadfc\uac70\uac00 \uc5c6\uc9c0\ub9cc, \ub048\uc9c8\uae34 \uc678\ubd80\uc758 \uc758\ud639 \uc81c\uae30\uc5d0 \uacf5\uc801 \uae30\uad00\uc73c\ub85c\uc11c \ub300\uc678\uc2e0\ub8b0\ub3c4\ub97c \uac04\uacfc\ud560 \uc218 \uc5c6\ub2e4\ub294 \ud310\ub2e8\uc5d0\uc11c\uc600\ub2e4\"\ub294 \uac8c \uac00\ucc9c\ub300\uc758 \uc124\uba85\uc774\ub2e4. \uadf8 \uacfc\uc815\uc5d0\uc11c \"\uc608\ube44\uc870\uc0ac\uac00 \ud559\uce59\uc5d0 \ubc18\ud55c\ub2e4\"\ub294 \uc774\uc758\uc81c\uae30\uac00 \uc788\uc5c8\uc73c\ub098 \"\uc678\ubd80\uc758 \uc815\uce58\uc801 \uacf5\ubc29\uc73c\ub85c \uc5ec\ub860\uc774 \ub728\uac70\uc6b4 \ub9cc\ud07c, \ubcf8\uc870\uc0ac\uc5d0\uc11c '\uae30\uac04 \ub3c4\uacfc' \ud574\uc11d\ub3c4 \uac00\ub9ac\ub294 \uac8c \uc88b\ub2e4\"\ub294 \uc758\uacac\uc774 \ub300\uc138\uc5ec\uc11c (12\uc6d4 31\uc77c) \ubcf8\uc870\uc0ac\ub85c \ub118\uacbc\ub2e4\"\ub294 \uc124\uba85\uc744 \ub367\ubd99\uc600\ub2e4.", "answer": "\uac00\ucc9c\ub300", "sentence": "\uadf8\ud574 12\uc6d4 \uc131\ub0a8\uc2dc\ubbfc\uc0ac\ud68c\ub2e8\uccb4\ud611\uc758\ud68c\ub294 \uc774 \uc2dc\uc7a5\uacfc \uac00\ucc9c\ub300 \uc5d0 \ud574\uba85\uc744 \uc694\uad6c\ud588\ub2e4.", "paragraph_sentence": "\uc774\uc7ac\uba85 \uc2dc\uc7a5\uc758 \ub17c\ubb38 \ud45c\uc808 \uc758\ud639\uc740 2013\ub144 9\uc6d4 \ubbf8\ub514\uc5b4\uc6cc\uce58 \uc0b0\ud558 \uc5f0\uad6c\uc9c4\uc2e4\uc131\uac80\uc99d\uc13c\ud130\uac00 \"\ub17c\ubb38\uc758 50\u223c98%\uac00 \ud45c\uc808\ub85c \uc758\uc2ec\ub41c\ub2e4\"\uace0 \uc81c\uae30\ud558\uba74\uc11c \ucd09\ubc1c\ub410\ub2e4. \uadf8\ud574 12\uc6d4 \uc131\ub0a8\uc2dc\ubbfc\uc0ac\ud68c\ub2e8\uccb4\ud611\uc758\ud68c\ub294 \uc774 \uc2dc\uc7a5\uacfc \uac00\ucc9c\ub300 \uc5d0 \ud574\uba85\uc744 \uc694\uad6c\ud588\ub2e4. \uc774\ud6c4 \uac00\ucc9c\ub300\ub294 \uc5f0\uad6c\uc724\ub9ac\uc704 \uc870\uc0ac \uc808\ucc28\ub97c \uc9c4\ud589\ud588\ub2e4. \"5\ub144\uc774 \uc9c0\ub098 \ud559\uce59\uc0c1 \uc2ec\uc0ac\ud560 \uadfc\uac70\uac00 \uc5c6\uc9c0\ub9cc, \ub048\uc9c8\uae34 \uc678\ubd80\uc758 \uc758\ud639 \uc81c\uae30\uc5d0 \uacf5\uc801 \uae30\uad00\uc73c\ub85c\uc11c \ub300\uc678\uc2e0\ub8b0\ub3c4\ub97c \uac04\uacfc\ud560 \uc218 \uc5c6\ub2e4\ub294 \ud310\ub2e8\uc5d0\uc11c\uc600\ub2e4\"\ub294 \uac8c \uac00\ucc9c\ub300\uc758 \uc124\uba85\uc774\ub2e4. \uadf8 \uacfc\uc815\uc5d0\uc11c \"\uc608\ube44\uc870\uc0ac\uac00 \ud559\uce59\uc5d0 \ubc18\ud55c\ub2e4\"\ub294 \uc774\uc758\uc81c\uae30\uac00 \uc788\uc5c8\uc73c\ub098 \"\uc678\ubd80\uc758 \uc815\uce58\uc801 \uacf5\ubc29\uc73c\ub85c \uc5ec\ub860\uc774 \ub728\uac70\uc6b4 \ub9cc\ud07c, \ubcf8\uc870\uc0ac\uc5d0\uc11c '\uae30\uac04 \ub3c4\uacfc' \ud574\uc11d\ub3c4 \uac00\ub9ac\ub294 \uac8c \uc88b\ub2e4\"\ub294 \uc758\uacac\uc774 \ub300\uc138\uc5ec\uc11c (12\uc6d4 31\uc77c) \ubcf8\uc870\uc0ac\ub85c \ub118\uacbc\ub2e4\"\ub294 \uc124\uba85\uc744 \ub367\ubd99\uc600\ub2e4.", "paragraph_answer": "\uc774\uc7ac\uba85 \uc2dc\uc7a5\uc758 \ub17c\ubb38 \ud45c\uc808 \uc758\ud639\uc740 2013\ub144 9\uc6d4 \ubbf8\ub514\uc5b4\uc6cc\uce58 \uc0b0\ud558 \uc5f0\uad6c\uc9c4\uc2e4\uc131\uac80\uc99d\uc13c\ud130\uac00 \"\ub17c\ubb38\uc758 50\u223c98%\uac00 \ud45c\uc808\ub85c \uc758\uc2ec\ub41c\ub2e4\"\uace0 \uc81c\uae30\ud558\uba74\uc11c \ucd09\ubc1c\ub410\ub2e4. \uadf8\ud574 12\uc6d4 \uc131\ub0a8\uc2dc\ubbfc\uc0ac\ud68c\ub2e8\uccb4\ud611\uc758\ud68c\ub294 \uc774 \uc2dc\uc7a5\uacfc \uac00\ucc9c\ub300 \uc5d0 \ud574\uba85\uc744 \uc694\uad6c\ud588\ub2e4. \uc774\ud6c4 \uac00\ucc9c\ub300\ub294 \uc5f0\uad6c\uc724\ub9ac\uc704 \uc870\uc0ac \uc808\ucc28\ub97c \uc9c4\ud589\ud588\ub2e4. \"5\ub144\uc774 \uc9c0\ub098 \ud559\uce59\uc0c1 \uc2ec\uc0ac\ud560 \uadfc\uac70\uac00 \uc5c6\uc9c0\ub9cc, \ub048\uc9c8\uae34 \uc678\ubd80\uc758 \uc758\ud639 \uc81c\uae30\uc5d0 \uacf5\uc801 \uae30\uad00\uc73c\ub85c\uc11c \ub300\uc678\uc2e0\ub8b0\ub3c4\ub97c \uac04\uacfc\ud560 \uc218 \uc5c6\ub2e4\ub294 \ud310\ub2e8\uc5d0\uc11c\uc600\ub2e4\"\ub294 \uac8c \uac00\ucc9c\ub300\uc758 \uc124\uba85\uc774\ub2e4. \uadf8 \uacfc\uc815\uc5d0\uc11c \"\uc608\ube44\uc870\uc0ac\uac00 \ud559\uce59\uc5d0 \ubc18\ud55c\ub2e4\"\ub294 \uc774\uc758\uc81c\uae30\uac00 \uc788\uc5c8\uc73c\ub098 \"\uc678\ubd80\uc758 \uc815\uce58\uc801 \uacf5\ubc29\uc73c\ub85c \uc5ec\ub860\uc774 \ub728\uac70\uc6b4 \ub9cc\ud07c, \ubcf8\uc870\uc0ac\uc5d0\uc11c '\uae30\uac04 \ub3c4\uacfc' \ud574\uc11d\ub3c4 \uac00\ub9ac\ub294 \uac8c \uc88b\ub2e4\"\ub294 \uc758\uacac\uc774 \ub300\uc138\uc5ec\uc11c (12\uc6d4 31\uc77c) \ubcf8\uc870\uc0ac\ub85c \ub118\uacbc\ub2e4\"\ub294 \uc124\uba85\uc744 \ub367\ubd99\uc600\ub2e4.", "sentence_answer": "\uadf8\ud574 12\uc6d4 \uc131\ub0a8\uc2dc\ubbfc\uc0ac\ud68c\ub2e8\uccb4\ud611\uc758\ud68c\ub294 \uc774 \uc2dc\uc7a5\uacfc \uac00\ucc9c\ub300 \uc5d0 \ud574\uba85\uc744 \uc694\uad6c\ud588\ub2e4.", "paragraph_id": "6590184-1-0", "paragraph_question": "question: \uc131\ub0a8\uc2dc\ubbfc\uc0ac\ud68c\ub2e8\uccb4\ud611\uc758\ud68c\uac00 \uc774\uc7ac\uba85 \uc2dc\uc7a5\uc758 \ub17c\ubb38 \ud45c\uc808 \uc758\ud639\uc744 \ud574\uba85\ud558\ub77c\uace0 \uc694\uad6c\ud55c \ub300\ud559\uad50\ub294?, context: \uc774\uc7ac\uba85 \uc2dc\uc7a5\uc758 \ub17c\ubb38 \ud45c\uc808 \uc758\ud639\uc740 2013\ub144 9\uc6d4 \ubbf8\ub514\uc5b4\uc6cc\uce58 \uc0b0\ud558 \uc5f0\uad6c\uc9c4\uc2e4\uc131\uac80\uc99d\uc13c\ud130\uac00 \"\ub17c\ubb38\uc758 50\u223c98%\uac00 \ud45c\uc808\ub85c \uc758\uc2ec\ub41c\ub2e4\"\uace0 \uc81c\uae30\ud558\uba74\uc11c \ucd09\ubc1c\ub410\ub2e4. \uadf8\ud574 12\uc6d4 \uc131\ub0a8\uc2dc\ubbfc\uc0ac\ud68c\ub2e8\uccb4\ud611\uc758\ud68c\ub294 \uc774 \uc2dc\uc7a5\uacfc \uac00\ucc9c\ub300\uc5d0 \ud574\uba85\uc744 \uc694\uad6c\ud588\ub2e4. \uc774\ud6c4 \uac00\ucc9c\ub300\ub294 \uc5f0\uad6c\uc724\ub9ac\uc704 \uc870\uc0ac \uc808\ucc28\ub97c \uc9c4\ud589\ud588\ub2e4. \"5\ub144\uc774 \uc9c0\ub098 \ud559\uce59\uc0c1 \uc2ec\uc0ac\ud560 \uadfc\uac70\uac00 \uc5c6\uc9c0\ub9cc, \ub048\uc9c8\uae34 \uc678\ubd80\uc758 \uc758\ud639 \uc81c\uae30\uc5d0 \uacf5\uc801 \uae30\uad00\uc73c\ub85c\uc11c \ub300\uc678\uc2e0\ub8b0\ub3c4\ub97c \uac04\uacfc\ud560 \uc218 \uc5c6\ub2e4\ub294 \ud310\ub2e8\uc5d0\uc11c\uc600\ub2e4\"\ub294 \uac8c \uac00\ucc9c\ub300\uc758 \uc124\uba85\uc774\ub2e4. \uadf8 \uacfc\uc815\uc5d0\uc11c \"\uc608\ube44\uc870\uc0ac\uac00 \ud559\uce59\uc5d0 \ubc18\ud55c\ub2e4\"\ub294 \uc774\uc758\uc81c\uae30\uac00 \uc788\uc5c8\uc73c\ub098 \"\uc678\ubd80\uc758 \uc815\uce58\uc801 \uacf5\ubc29\uc73c\ub85c \uc5ec\ub860\uc774 \ub728\uac70\uc6b4 \ub9cc\ud07c, \ubcf8\uc870\uc0ac\uc5d0\uc11c '\uae30\uac04 \ub3c4\uacfc' \ud574\uc11d\ub3c4 \uac00\ub9ac\ub294 \uac8c \uc88b\ub2e4\"\ub294 \uc758\uacac\uc774 \ub300\uc138\uc5ec\uc11c (12\uc6d4 31\uc77c) \ubcf8\uc870\uc0ac\ub85c \ub118\uacbc\ub2e4\"\ub294 \uc124\uba85\uc744 \ub367\ubd99\uc600\ub2e4."} {"question": "2018\ub144 \ub125\uc13c \ud788\uc5b4\ub85c\uc758 \ub300\ud45c\uc774\uc0ac\ub294 \ubb34\uc2a8 \ud610\uc758\ub85c \ubc95\uc815\uad6c\uc18d \ub418\uc5c8\ub294\uac00?", "paragraph": "\uadf8\ub7ec\ub098 2018\ub144 2\uc6d4 \ub125\uc13c \ud788\uc5b4\ub85c\uc988\uc758 \uc774\uc7a5\uc11d \ub300\ud45c\uc774\uc0ac\uac00 \ud6a1\ub839 \ub4f1\uc758 \ud610\uc758\ub85c \ubc95\uc815 \uad6c\uc18d\ub418\uc5c8\uace0, \uadf8\uc640 \uad00\ub828\ub41c \uac80\ucc30\uc758 \uc218\uc0ac\uc640 \uc5b8\ub860\uc758 \ubcf4\ub3c4\uac00 \uc9c4\ud589\ub418\ub358 \ub3c4\uc911, 5\uc6d4 28\uc77c KBS\ub294 \ub125\uc13c \ud788\uc5b4\ub85c\uc988\uac00 \ub4b7\ub3c8\uc744 \ubc1b\uace0 \ud2b8\ub808\uc774\ub4dc\ub97c \uc9c4\ud589\ud558\uc600\ub2e4\ub294 \ub2e8\ub3c5 \ubcf4\ub3c4\ub97c \ub0b4\ubcf4\ub0b4\uc5c8\ub2e4. 2017\ub144 3\uc6d4 \uac15\uc724\uad6c \u2013 \uae40\ud55c\ubcc4 \ud2b8\ub808\uc774\ub4dc \ub2f9\uc2dc\uc5d0 \uc54c\ub824\uc9c4 \uac83\uacfc\ub294 \ub2ec\ub9ac \uc2e4\uc81c\ub85c\ub294 \ud604\uae08 1\uc5b5 \uc6d0\uc744 NC\ub85c\ubd80\ud130 \ubc1b\uc558\uace0, 2017\ub144 7\uc6d4 \uc724\uc11d\ubbfc \u2013 \uc815\ub300\ud604, \uc11c\uc758\ud0dc \ud2b8\ub808\uc774\ub4dc \ub2f9\uc2dc\uc5d0 \ud604\uae08 5\uc5b5 \uc6d0\uc744 kt\ub85c\ubd80\ud130 \ubc1b\uc558\ub2e4\ub294 \uac83\uc774\ub2e4. \uac70\uae30\uc5d0 \ud604\uae08 \ud2b8\ub808\uc774\ub4dc\ub85c\ubd80\ud130 \ubc1c\uc0dd\ud55c 6\uc5b5 \uc6d0\uc758 0.5%\uc778 3\ubc31\ub9cc\uc6d0\uc744 \uc774\uc7a5\uc11d \ub300\ud45c\uc774\uc0ac\uc640 \uace0\ud615\uc6b1 \ub2e8\uc7a5\uc774 \uac01\uac01 \uc778\uc13c\ud2f0\ube0c\ub85c \uc9c0\uae09 \ubc1b\uc558\ub2e4\ub294 \uac83\ub3c4 \ucd94\uac00\ub85c \ubc1d\ud600\uc84c\ub2e4. \uc774\uc5d0 \ub300\ud574\uc11c \uace0\ud615\uc6b1 \ub2e8\uc7a5\uc740 \ubc1c\ud45c\ud558\uc9c0 \uc54a\uc740 \uae08\uc561\uc778 6\uc5b5 \uc6d0\uc744 NC\uc640 kt\ub85c\ubd80\ud130 \uac01\uac01 \uc218\ub839\ud588\ub2e4\ub294 \uac83\uc740 \uc778\uc815\ud588\uc9c0\ub9cc, \uc778\uc13c\ud2f0\ube0c\ub97c \ubc1b\uc558\ub2e4\ub294 \uc0ac\uc2e4\uc740 \ubd80\uc778\ud558\uc600\ub2e4.", "answer": "\ud6a1\ub839", "sentence": "\uadf8\ub7ec\ub098 2018\ub144 2\uc6d4 \ub125\uc13c \ud788\uc5b4\ub85c\uc988\uc758 \uc774\uc7a5\uc11d \ub300\ud45c\uc774\uc0ac\uac00 \ud6a1\ub839 \ub4f1\uc758 \ud610\uc758\ub85c \ubc95\uc815 \uad6c\uc18d\ub418\uc5c8\uace0, \uadf8\uc640 \uad00\ub828\ub41c \uac80\ucc30\uc758 \uc218\uc0ac\uc640 \uc5b8\ub860\uc758 \ubcf4\ub3c4\uac00 \uc9c4\ud589\ub418\ub358 \ub3c4\uc911, 5\uc6d4 28\uc77c KBS\ub294 \ub125\uc13c \ud788\uc5b4\ub85c\uc988\uac00 \ub4b7\ub3c8\uc744 \ubc1b\uace0 \ud2b8\ub808\uc774\ub4dc\ub97c \uc9c4\ud589\ud558\uc600\ub2e4\ub294 \ub2e8\ub3c5 \ubcf4\ub3c4\ub97c \ub0b4\ubcf4\ub0b4\uc5c8\ub2e4.", "paragraph_sentence": " \uadf8\ub7ec\ub098 2018\ub144 2\uc6d4 \ub125\uc13c \ud788\uc5b4\ub85c\uc988\uc758 \uc774\uc7a5\uc11d \ub300\ud45c\uc774\uc0ac\uac00 \ud6a1\ub839 \ub4f1\uc758 \ud610\uc758\ub85c \ubc95\uc815 \uad6c\uc18d\ub418\uc5c8\uace0, \uadf8\uc640 \uad00\ub828\ub41c \uac80\ucc30\uc758 \uc218\uc0ac\uc640 \uc5b8\ub860\uc758 \ubcf4\ub3c4\uac00 \uc9c4\ud589\ub418\ub358 \ub3c4\uc911, 5\uc6d4 28\uc77c KBS\ub294 \ub125\uc13c \ud788\uc5b4\ub85c\uc988\uac00 \ub4b7\ub3c8\uc744 \ubc1b\uace0 \ud2b8\ub808\uc774\ub4dc\ub97c \uc9c4\ud589\ud558\uc600\ub2e4\ub294 \ub2e8\ub3c5 \ubcf4\ub3c4\ub97c \ub0b4\ubcf4\ub0b4\uc5c8\ub2e4. 2017\ub144 3\uc6d4 \uac15\uc724\uad6c \u2013 \uae40\ud55c\ubcc4 \ud2b8\ub808\uc774\ub4dc \ub2f9\uc2dc\uc5d0 \uc54c\ub824\uc9c4 \uac83\uacfc\ub294 \ub2ec\ub9ac \uc2e4\uc81c\ub85c\ub294 \ud604\uae08 1\uc5b5 \uc6d0\uc744 NC\ub85c\ubd80\ud130 \ubc1b\uc558\uace0, 2017\ub144 7\uc6d4 \uc724\uc11d\ubbfc \u2013 \uc815\ub300\ud604, \uc11c\uc758\ud0dc \ud2b8\ub808\uc774\ub4dc \ub2f9\uc2dc\uc5d0 \ud604\uae08 5\uc5b5 \uc6d0\uc744 kt\ub85c\ubd80\ud130 \ubc1b\uc558\ub2e4\ub294 \uac83\uc774\ub2e4. \uac70\uae30\uc5d0 \ud604\uae08 \ud2b8\ub808\uc774\ub4dc\ub85c\ubd80\ud130 \ubc1c\uc0dd\ud55c 6\uc5b5 \uc6d0\uc758 0.5%\uc778 3\ubc31\ub9cc\uc6d0\uc744 \uc774\uc7a5\uc11d \ub300\ud45c\uc774\uc0ac\uc640 \uace0\ud615\uc6b1 \ub2e8\uc7a5\uc774 \uac01\uac01 \uc778\uc13c\ud2f0\ube0c\ub85c \uc9c0\uae09 \ubc1b\uc558\ub2e4\ub294 \uac83\ub3c4 \ucd94\uac00\ub85c \ubc1d\ud600\uc84c\ub2e4. \uc774\uc5d0 \ub300\ud574\uc11c \uace0\ud615\uc6b1 \ub2e8\uc7a5\uc740 \ubc1c\ud45c\ud558\uc9c0 \uc54a\uc740 \uae08\uc561\uc778 6\uc5b5 \uc6d0\uc744 NC\uc640 kt\ub85c\ubd80\ud130 \uac01\uac01 \uc218\ub839\ud588\ub2e4\ub294 \uac83\uc740 \uc778\uc815\ud588\uc9c0\ub9cc, \uc778\uc13c\ud2f0\ube0c\ub97c \ubc1b\uc558\ub2e4\ub294 \uc0ac\uc2e4\uc740 \ubd80\uc778\ud558\uc600\ub2e4.", "paragraph_answer": "\uadf8\ub7ec\ub098 2018\ub144 2\uc6d4 \ub125\uc13c \ud788\uc5b4\ub85c\uc988\uc758 \uc774\uc7a5\uc11d \ub300\ud45c\uc774\uc0ac\uac00 \ud6a1\ub839 \ub4f1\uc758 \ud610\uc758\ub85c \ubc95\uc815 \uad6c\uc18d\ub418\uc5c8\uace0, \uadf8\uc640 \uad00\ub828\ub41c \uac80\ucc30\uc758 \uc218\uc0ac\uc640 \uc5b8\ub860\uc758 \ubcf4\ub3c4\uac00 \uc9c4\ud589\ub418\ub358 \ub3c4\uc911, 5\uc6d4 28\uc77c KBS\ub294 \ub125\uc13c \ud788\uc5b4\ub85c\uc988\uac00 \ub4b7\ub3c8\uc744 \ubc1b\uace0 \ud2b8\ub808\uc774\ub4dc\ub97c \uc9c4\ud589\ud558\uc600\ub2e4\ub294 \ub2e8\ub3c5 \ubcf4\ub3c4\ub97c \ub0b4\ubcf4\ub0b4\uc5c8\ub2e4. 2017\ub144 3\uc6d4 \uac15\uc724\uad6c \u2013 \uae40\ud55c\ubcc4 \ud2b8\ub808\uc774\ub4dc \ub2f9\uc2dc\uc5d0 \uc54c\ub824\uc9c4 \uac83\uacfc\ub294 \ub2ec\ub9ac \uc2e4\uc81c\ub85c\ub294 \ud604\uae08 1\uc5b5 \uc6d0\uc744 NC\ub85c\ubd80\ud130 \ubc1b\uc558\uace0, 2017\ub144 7\uc6d4 \uc724\uc11d\ubbfc \u2013 \uc815\ub300\ud604, \uc11c\uc758\ud0dc \ud2b8\ub808\uc774\ub4dc \ub2f9\uc2dc\uc5d0 \ud604\uae08 5\uc5b5 \uc6d0\uc744 kt\ub85c\ubd80\ud130 \ubc1b\uc558\ub2e4\ub294 \uac83\uc774\ub2e4. \uac70\uae30\uc5d0 \ud604\uae08 \ud2b8\ub808\uc774\ub4dc\ub85c\ubd80\ud130 \ubc1c\uc0dd\ud55c 6\uc5b5 \uc6d0\uc758 0.5%\uc778 3\ubc31\ub9cc\uc6d0\uc744 \uc774\uc7a5\uc11d \ub300\ud45c\uc774\uc0ac\uc640 \uace0\ud615\uc6b1 \ub2e8\uc7a5\uc774 \uac01\uac01 \uc778\uc13c\ud2f0\ube0c\ub85c \uc9c0\uae09 \ubc1b\uc558\ub2e4\ub294 \uac83\ub3c4 \ucd94\uac00\ub85c \ubc1d\ud600\uc84c\ub2e4. \uc774\uc5d0 \ub300\ud574\uc11c \uace0\ud615\uc6b1 \ub2e8\uc7a5\uc740 \ubc1c\ud45c\ud558\uc9c0 \uc54a\uc740 \uae08\uc561\uc778 6\uc5b5 \uc6d0\uc744 NC\uc640 kt\ub85c\ubd80\ud130 \uac01\uac01 \uc218\ub839\ud588\ub2e4\ub294 \uac83\uc740 \uc778\uc815\ud588\uc9c0\ub9cc, \uc778\uc13c\ud2f0\ube0c\ub97c \ubc1b\uc558\ub2e4\ub294 \uc0ac\uc2e4\uc740 \ubd80\uc778\ud558\uc600\ub2e4.", "sentence_answer": "\uadf8\ub7ec\ub098 2018\ub144 2\uc6d4 \ub125\uc13c \ud788\uc5b4\ub85c\uc988\uc758 \uc774\uc7a5\uc11d \ub300\ud45c\uc774\uc0ac\uac00 \ud6a1\ub839 \ub4f1\uc758 \ud610\uc758\ub85c \ubc95\uc815 \uad6c\uc18d\ub418\uc5c8\uace0, \uadf8\uc640 \uad00\ub828\ub41c \uac80\ucc30\uc758 \uc218\uc0ac\uc640 \uc5b8\ub860\uc758 \ubcf4\ub3c4\uac00 \uc9c4\ud589\ub418\ub358 \ub3c4\uc911, 5\uc6d4 28\uc77c KBS\ub294 \ub125\uc13c \ud788\uc5b4\ub85c\uc988\uac00 \ub4b7\ub3c8\uc744 \ubc1b\uace0 \ud2b8\ub808\uc774\ub4dc\ub97c \uc9c4\ud589\ud558\uc600\ub2e4\ub294 \ub2e8\ub3c5 \ubcf4\ub3c4\ub97c \ub0b4\ubcf4\ub0b4\uc5c8\ub2e4.", "paragraph_id": "6533929-20-0", "paragraph_question": "question: 2018\ub144 \ub125\uc13c \ud788\uc5b4\ub85c\uc758 \ub300\ud45c\uc774\uc0ac\ub294 \ubb34\uc2a8 \ud610\uc758\ub85c \ubc95\uc815\uad6c\uc18d \ub418\uc5c8\ub294\uac00?, context: \uadf8\ub7ec\ub098 2018\ub144 2\uc6d4 \ub125\uc13c \ud788\uc5b4\ub85c\uc988\uc758 \uc774\uc7a5\uc11d \ub300\ud45c\uc774\uc0ac\uac00 \ud6a1\ub839 \ub4f1\uc758 \ud610\uc758\ub85c \ubc95\uc815 \uad6c\uc18d\ub418\uc5c8\uace0, \uadf8\uc640 \uad00\ub828\ub41c \uac80\ucc30\uc758 \uc218\uc0ac\uc640 \uc5b8\ub860\uc758 \ubcf4\ub3c4\uac00 \uc9c4\ud589\ub418\ub358 \ub3c4\uc911, 5\uc6d4 28\uc77c KBS\ub294 \ub125\uc13c \ud788\uc5b4\ub85c\uc988\uac00 \ub4b7\ub3c8\uc744 \ubc1b\uace0 \ud2b8\ub808\uc774\ub4dc\ub97c \uc9c4\ud589\ud558\uc600\ub2e4\ub294 \ub2e8\ub3c5 \ubcf4\ub3c4\ub97c \ub0b4\ubcf4\ub0b4\uc5c8\ub2e4. 2017\ub144 3\uc6d4 \uac15\uc724\uad6c \u2013 \uae40\ud55c\ubcc4 \ud2b8\ub808\uc774\ub4dc \ub2f9\uc2dc\uc5d0 \uc54c\ub824\uc9c4 \uac83\uacfc\ub294 \ub2ec\ub9ac \uc2e4\uc81c\ub85c\ub294 \ud604\uae08 1\uc5b5 \uc6d0\uc744 NC\ub85c\ubd80\ud130 \ubc1b\uc558\uace0, 2017\ub144 7\uc6d4 \uc724\uc11d\ubbfc \u2013 \uc815\ub300\ud604, \uc11c\uc758\ud0dc \ud2b8\ub808\uc774\ub4dc \ub2f9\uc2dc\uc5d0 \ud604\uae08 5\uc5b5 \uc6d0\uc744 kt\ub85c\ubd80\ud130 \ubc1b\uc558\ub2e4\ub294 \uac83\uc774\ub2e4. \uac70\uae30\uc5d0 \ud604\uae08 \ud2b8\ub808\uc774\ub4dc\ub85c\ubd80\ud130 \ubc1c\uc0dd\ud55c 6\uc5b5 \uc6d0\uc758 0.5%\uc778 3\ubc31\ub9cc\uc6d0\uc744 \uc774\uc7a5\uc11d \ub300\ud45c\uc774\uc0ac\uc640 \uace0\ud615\uc6b1 \ub2e8\uc7a5\uc774 \uac01\uac01 \uc778\uc13c\ud2f0\ube0c\ub85c \uc9c0\uae09 \ubc1b\uc558\ub2e4\ub294 \uac83\ub3c4 \ucd94\uac00\ub85c \ubc1d\ud600\uc84c\ub2e4. \uc774\uc5d0 \ub300\ud574\uc11c \uace0\ud615\uc6b1 \ub2e8\uc7a5\uc740 \ubc1c\ud45c\ud558\uc9c0 \uc54a\uc740 \uae08\uc561\uc778 6\uc5b5 \uc6d0\uc744 NC\uc640 kt\ub85c\ubd80\ud130 \uac01\uac01 \uc218\ub839\ud588\ub2e4\ub294 \uac83\uc740 \uc778\uc815\ud588\uc9c0\ub9cc, \uc778\uc13c\ud2f0\ube0c\ub97c \ubc1b\uc558\ub2e4\ub294 \uc0ac\uc2e4\uc740 \ubd80\uc778\ud558\uc600\ub2e4."} {"question": "\ub124\ub974\uce5c\uc2a4\ud06c\uc5d0\uc11c \uccad\ub098\ub77c\uc640 \ub7ec\uc2dc\uc544\uc758 \uad6d\uacbd\uc73c\ub85c \ud655\uc815 \uc9c0\uc740 \uac15\uc740?", "paragraph": "\uadf8\ub9ac\ud558\uc5ec \uccad\uad70\uacfc \ub7ec\uc2dc\uc544\uad70\uc774 \ud751\ub8e1\uac15 \ubd80\uadfc\uc5d0\uc11c \uad6d\uc9c0\uc801\uc73c\ub85c \uc2f8\uc6b0\uace0 \ud734\uc804\ud558\uae30\ub97c \ubc18\ubcf5\ud55c \ub4a4\uc5d0, 1689\ub144(\uac15\ud76c 28\ub144)\uc5d0 \uac15\ud76c\uc81c\ub294 \uc601\uc2dc\uc704\ub0b4\ub300\uc2e0\uc774\uc790 \uc790\uc2e0\uc758 \ucc98\uc219(\u59bb\u53d4)\uc778 \uc0c9\uc561\ub3c4\ub97c \ud760\ucc28\ub300\uc2e0\uc73c\ub85c \uba85\ud558\uc5ec \ub7ec\uc2dc\uc544\uc640 \ud611\uc0c1\uc744 \ubcf4\uac8c \ud558\uc600\ub2e4. \ub7ec\uc2dc\uc544\uc640 \uccad\ub098\ub77c \ub300\ud45c\uac00 \ub124\ub974\uce5c\uc2a4\ud06c\uc5d0\uc11c \ub9cc\ub098 \ud5e4\uc774\ub8fd\uac15\uc758 \uc9c0\ub958\uc778 \uace0\ub974\ube44\ud2b8\uc0ac \uac15\uacfc \uc2a4\ud0c0\ub178\ubcf4\uc774 \uc0b0\ub9e5\uc744 \uccad\ub098\ub77c\uc640 \ub7ec\uc2dc\uc544\uc758 \uad6d\uacbd\uc73c\ub85c \ud655\uc815 \uc9c0\uc73c\ub2c8, \uc774\uac83\uc774 \ub124\ub974\uce5c\uc2a4\ud06c \uc870\uc57d\uc774\ub2e4. \uc774 \uc870\uc57d\uc740 \uc911\uad6d\uc0ac \uc774\ub798 \ucc98\uc74c\uc73c\ub85c \uad6d\uac00\uac04\uc758 \ud3c9\ub4f1\ud55c \uc704\uce58\uc5d0\uc11c \ub9fa\uc740 \uc870\uc57d\uc73c\ub85c\uc11c \ub2f9\uc2dc \ub3d9\uc591 \uad6d\uac00\ub4e4\uc774 \ub9fa\ub294 \ub300\uad6d-\uc18c\uad6d \uac04\uc758 \uc870\uc57d\uacfc\ub294 \ub2e4\ub978 \uc720\ub7fd\uc2dd \uc870\uc57d\uc774\uc5c8\ub2e4. \uc5ec\uae30\uc11c \uccad\ub098\ub77c\uc758 \uc81c\uc548\uc774 \ub300\ubd80\ubd84 \uc218\uc6a9\ub418\uc5b4 \uccad\ub098\ub77c\ub294 \ub3d9\ubd81\ucabd\uc758 \ub113\uc740 \uc601\ud1a0\ub97c \uc5bb\uac8c \ub418\uc5c8\ub2e4. \ub610\ud55c, \uccad\ub098\ub77c\uc640 \ub7ec\uc2dc\uc544 \uac04\uc758 \ubb34\uc5ed\uc5d0\ub3c4 \uc790\uc720\ub97c \ubcf4\uc7a5\ud558\uc600\uc73c\ub098, \ub7ec\uc2dc\uc544\uc5d0\ub294 \uadf8\ub2e4\uc9c0 \ub4dd\uc744 \ubcfc \ub9cc\ud55c \uc870\ud56d\uc740 \ub9ce\uc9c0 \uc54a\uc544 \ud6d7\ub0a0 \ud568\ud48d\uc81c\uc758 \uce58\uc138 \ub54c, \ub7ec\uc2dc\uc544\uac00 \uc560\ub85c\ud638 \uc804\uc7c1\uc5d0\uc11c \uccad\ub098\ub77c\uc758 \uc6d0\uc870\ub97c \ube4c\ubbf8\ub85c \ub9cc\uc8fc\uc640 \uc5f0\ud574\uc8fc\uc758 \ub9ce\uc740 \ub545\uc744 \ub3c4\ub85c \uac00\uc838\uac00\uac8c \ud558\ub294 \uc6d0\uc778\uc774 \ub418\uc5c8\ub2e4.", "answer": "\uace0\ub974\ube44\ud2b8\uc0ac \uac15", "sentence": "\ub7ec\uc2dc\uc544\uc640 \uccad\ub098\ub77c \ub300\ud45c\uac00 \ub124\ub974\uce5c\uc2a4\ud06c\uc5d0\uc11c \ub9cc\ub098 \ud5e4\uc774\ub8fd\uac15\uc758 \uc9c0\ub958\uc778 \uace0\ub974\ube44\ud2b8\uc0ac \uac15 \uacfc \uc2a4\ud0c0\ub178\ubcf4\uc774 \uc0b0\ub9e5\uc744 \uccad\ub098\ub77c\uc640 \ub7ec\uc2dc\uc544\uc758 \uad6d\uacbd\uc73c\ub85c \ud655\uc815 \uc9c0\uc73c\ub2c8, \uc774\uac83\uc774 \ub124\ub974\uce5c\uc2a4\ud06c \uc870\uc57d\uc774\ub2e4.", "paragraph_sentence": "\uadf8\ub9ac\ud558\uc5ec \uccad\uad70\uacfc \ub7ec\uc2dc\uc544\uad70\uc774 \ud751\ub8e1\uac15 \ubd80\uadfc\uc5d0\uc11c \uad6d\uc9c0\uc801\uc73c\ub85c \uc2f8\uc6b0\uace0 \ud734\uc804\ud558\uae30\ub97c \ubc18\ubcf5\ud55c \ub4a4\uc5d0, 1689\ub144(\uac15\ud76c 28\ub144)\uc5d0 \uac15\ud76c\uc81c\ub294 \uc601\uc2dc\uc704\ub0b4\ub300\uc2e0\uc774\uc790 \uc790\uc2e0\uc758 \ucc98\uc219(\u59bb\u53d4)\uc778 \uc0c9\uc561\ub3c4\ub97c \ud760\ucc28\ub300\uc2e0\uc73c\ub85c \uba85\ud558\uc5ec \ub7ec\uc2dc\uc544\uc640 \ud611\uc0c1\uc744 \ubcf4\uac8c \ud558\uc600\ub2e4. \ub7ec\uc2dc\uc544\uc640 \uccad\ub098\ub77c \ub300\ud45c\uac00 \ub124\ub974\uce5c\uc2a4\ud06c\uc5d0\uc11c \ub9cc\ub098 \ud5e4\uc774\ub8fd\uac15\uc758 \uc9c0\ub958\uc778 \uace0\ub974\ube44\ud2b8\uc0ac \uac15 \uacfc \uc2a4\ud0c0\ub178\ubcf4\uc774 \uc0b0\ub9e5\uc744 \uccad\ub098\ub77c\uc640 \ub7ec\uc2dc\uc544\uc758 \uad6d\uacbd\uc73c\ub85c \ud655\uc815 \uc9c0\uc73c\ub2c8, \uc774\uac83\uc774 \ub124\ub974\uce5c\uc2a4\ud06c \uc870\uc57d\uc774\ub2e4. \uc774 \uc870\uc57d\uc740 \uc911\uad6d\uc0ac \uc774\ub798 \ucc98\uc74c\uc73c\ub85c \uad6d\uac00\uac04\uc758 \ud3c9\ub4f1\ud55c \uc704\uce58\uc5d0\uc11c \ub9fa\uc740 \uc870\uc57d\uc73c\ub85c\uc11c \ub2f9\uc2dc \ub3d9\uc591 \uad6d\uac00\ub4e4\uc774 \ub9fa\ub294 \ub300\uad6d-\uc18c\uad6d \uac04\uc758 \uc870\uc57d\uacfc\ub294 \ub2e4\ub978 \uc720\ub7fd\uc2dd \uc870\uc57d\uc774\uc5c8\ub2e4. \uc5ec\uae30\uc11c \uccad\ub098\ub77c\uc758 \uc81c\uc548\uc774 \ub300\ubd80\ubd84 \uc218\uc6a9\ub418\uc5b4 \uccad\ub098\ub77c\ub294 \ub3d9\ubd81\ucabd\uc758 \ub113\uc740 \uc601\ud1a0\ub97c \uc5bb\uac8c \ub418\uc5c8\ub2e4. \ub610\ud55c, \uccad\ub098\ub77c\uc640 \ub7ec\uc2dc\uc544 \uac04\uc758 \ubb34\uc5ed\uc5d0\ub3c4 \uc790\uc720\ub97c \ubcf4\uc7a5\ud558\uc600\uc73c\ub098, \ub7ec\uc2dc\uc544\uc5d0\ub294 \uadf8\ub2e4\uc9c0 \ub4dd\uc744 \ubcfc \ub9cc\ud55c \uc870\ud56d\uc740 \ub9ce\uc9c0 \uc54a\uc544 \ud6d7\ub0a0 \ud568\ud48d\uc81c\uc758 \uce58\uc138 \ub54c, \ub7ec\uc2dc\uc544\uac00 \uc560\ub85c\ud638 \uc804\uc7c1\uc5d0\uc11c \uccad\ub098\ub77c\uc758 \uc6d0\uc870\ub97c \ube4c\ubbf8\ub85c \ub9cc\uc8fc\uc640 \uc5f0\ud574\uc8fc\uc758 \ub9ce\uc740 \ub545\uc744 \ub3c4\ub85c \uac00\uc838\uac00\uac8c \ud558\ub294 \uc6d0\uc778\uc774 \ub418\uc5c8\ub2e4.", "paragraph_answer": "\uadf8\ub9ac\ud558\uc5ec \uccad\uad70\uacfc \ub7ec\uc2dc\uc544\uad70\uc774 \ud751\ub8e1\uac15 \ubd80\uadfc\uc5d0\uc11c \uad6d\uc9c0\uc801\uc73c\ub85c \uc2f8\uc6b0\uace0 \ud734\uc804\ud558\uae30\ub97c \ubc18\ubcf5\ud55c \ub4a4\uc5d0, 1689\ub144(\uac15\ud76c 28\ub144)\uc5d0 \uac15\ud76c\uc81c\ub294 \uc601\uc2dc\uc704\ub0b4\ub300\uc2e0\uc774\uc790 \uc790\uc2e0\uc758 \ucc98\uc219(\u59bb\u53d4)\uc778 \uc0c9\uc561\ub3c4\ub97c \ud760\ucc28\ub300\uc2e0\uc73c\ub85c \uba85\ud558\uc5ec \ub7ec\uc2dc\uc544\uc640 \ud611\uc0c1\uc744 \ubcf4\uac8c \ud558\uc600\ub2e4. \ub7ec\uc2dc\uc544\uc640 \uccad\ub098\ub77c \ub300\ud45c\uac00 \ub124\ub974\uce5c\uc2a4\ud06c\uc5d0\uc11c \ub9cc\ub098 \ud5e4\uc774\ub8fd\uac15\uc758 \uc9c0\ub958\uc778 \uace0\ub974\ube44\ud2b8\uc0ac \uac15 \uacfc \uc2a4\ud0c0\ub178\ubcf4\uc774 \uc0b0\ub9e5\uc744 \uccad\ub098\ub77c\uc640 \ub7ec\uc2dc\uc544\uc758 \uad6d\uacbd\uc73c\ub85c \ud655\uc815 \uc9c0\uc73c\ub2c8, \uc774\uac83\uc774 \ub124\ub974\uce5c\uc2a4\ud06c \uc870\uc57d\uc774\ub2e4. \uc774 \uc870\uc57d\uc740 \uc911\uad6d\uc0ac \uc774\ub798 \ucc98\uc74c\uc73c\ub85c \uad6d\uac00\uac04\uc758 \ud3c9\ub4f1\ud55c \uc704\uce58\uc5d0\uc11c \ub9fa\uc740 \uc870\uc57d\uc73c\ub85c\uc11c \ub2f9\uc2dc \ub3d9\uc591 \uad6d\uac00\ub4e4\uc774 \ub9fa\ub294 \ub300\uad6d-\uc18c\uad6d \uac04\uc758 \uc870\uc57d\uacfc\ub294 \ub2e4\ub978 \uc720\ub7fd\uc2dd \uc870\uc57d\uc774\uc5c8\ub2e4. \uc5ec\uae30\uc11c \uccad\ub098\ub77c\uc758 \uc81c\uc548\uc774 \ub300\ubd80\ubd84 \uc218\uc6a9\ub418\uc5b4 \uccad\ub098\ub77c\ub294 \ub3d9\ubd81\ucabd\uc758 \ub113\uc740 \uc601\ud1a0\ub97c \uc5bb\uac8c \ub418\uc5c8\ub2e4. \ub610\ud55c, \uccad\ub098\ub77c\uc640 \ub7ec\uc2dc\uc544 \uac04\uc758 \ubb34\uc5ed\uc5d0\ub3c4 \uc790\uc720\ub97c \ubcf4\uc7a5\ud558\uc600\uc73c\ub098, \ub7ec\uc2dc\uc544\uc5d0\ub294 \uadf8\ub2e4\uc9c0 \ub4dd\uc744 \ubcfc \ub9cc\ud55c \uc870\ud56d\uc740 \ub9ce\uc9c0 \uc54a\uc544 \ud6d7\ub0a0 \ud568\ud48d\uc81c\uc758 \uce58\uc138 \ub54c, \ub7ec\uc2dc\uc544\uac00 \uc560\ub85c\ud638 \uc804\uc7c1\uc5d0\uc11c \uccad\ub098\ub77c\uc758 \uc6d0\uc870\ub97c \ube4c\ubbf8\ub85c \ub9cc\uc8fc\uc640 \uc5f0\ud574\uc8fc\uc758 \ub9ce\uc740 \ub545\uc744 \ub3c4\ub85c \uac00\uc838\uac00\uac8c \ud558\ub294 \uc6d0\uc778\uc774 \ub418\uc5c8\ub2e4.", "sentence_answer": "\ub7ec\uc2dc\uc544\uc640 \uccad\ub098\ub77c \ub300\ud45c\uac00 \ub124\ub974\uce5c\uc2a4\ud06c\uc5d0\uc11c \ub9cc\ub098 \ud5e4\uc774\ub8fd\uac15\uc758 \uc9c0\ub958\uc778 \uace0\ub974\ube44\ud2b8\uc0ac \uac15 \uacfc \uc2a4\ud0c0\ub178\ubcf4\uc774 \uc0b0\ub9e5\uc744 \uccad\ub098\ub77c\uc640 \ub7ec\uc2dc\uc544\uc758 \uad6d\uacbd\uc73c\ub85c \ud655\uc815 \uc9c0\uc73c\ub2c8, \uc774\uac83\uc774 \ub124\ub974\uce5c\uc2a4\ud06c \uc870\uc57d\uc774\ub2e4.", "paragraph_id": "6478740-20-0", "paragraph_question": "question: \ub124\ub974\uce5c\uc2a4\ud06c\uc5d0\uc11c \uccad\ub098\ub77c\uc640 \ub7ec\uc2dc\uc544\uc758 \uad6d\uacbd\uc73c\ub85c \ud655\uc815 \uc9c0\uc740 \uac15\uc740?, context: \uadf8\ub9ac\ud558\uc5ec \uccad\uad70\uacfc \ub7ec\uc2dc\uc544\uad70\uc774 \ud751\ub8e1\uac15 \ubd80\uadfc\uc5d0\uc11c \uad6d\uc9c0\uc801\uc73c\ub85c \uc2f8\uc6b0\uace0 \ud734\uc804\ud558\uae30\ub97c \ubc18\ubcf5\ud55c \ub4a4\uc5d0, 1689\ub144(\uac15\ud76c 28\ub144)\uc5d0 \uac15\ud76c\uc81c\ub294 \uc601\uc2dc\uc704\ub0b4\ub300\uc2e0\uc774\uc790 \uc790\uc2e0\uc758 \ucc98\uc219(\u59bb\u53d4)\uc778 \uc0c9\uc561\ub3c4\ub97c \ud760\ucc28\ub300\uc2e0\uc73c\ub85c \uba85\ud558\uc5ec \ub7ec\uc2dc\uc544\uc640 \ud611\uc0c1\uc744 \ubcf4\uac8c \ud558\uc600\ub2e4. \ub7ec\uc2dc\uc544\uc640 \uccad\ub098\ub77c \ub300\ud45c\uac00 \ub124\ub974\uce5c\uc2a4\ud06c\uc5d0\uc11c \ub9cc\ub098 \ud5e4\uc774\ub8fd\uac15\uc758 \uc9c0\ub958\uc778 \uace0\ub974\ube44\ud2b8\uc0ac \uac15\uacfc \uc2a4\ud0c0\ub178\ubcf4\uc774 \uc0b0\ub9e5\uc744 \uccad\ub098\ub77c\uc640 \ub7ec\uc2dc\uc544\uc758 \uad6d\uacbd\uc73c\ub85c \ud655\uc815 \uc9c0\uc73c\ub2c8, \uc774\uac83\uc774 \ub124\ub974\uce5c\uc2a4\ud06c \uc870\uc57d\uc774\ub2e4. \uc774 \uc870\uc57d\uc740 \uc911\uad6d\uc0ac \uc774\ub798 \ucc98\uc74c\uc73c\ub85c \uad6d\uac00\uac04\uc758 \ud3c9\ub4f1\ud55c \uc704\uce58\uc5d0\uc11c \ub9fa\uc740 \uc870\uc57d\uc73c\ub85c\uc11c \ub2f9\uc2dc \ub3d9\uc591 \uad6d\uac00\ub4e4\uc774 \ub9fa\ub294 \ub300\uad6d-\uc18c\uad6d \uac04\uc758 \uc870\uc57d\uacfc\ub294 \ub2e4\ub978 \uc720\ub7fd\uc2dd \uc870\uc57d\uc774\uc5c8\ub2e4. \uc5ec\uae30\uc11c \uccad\ub098\ub77c\uc758 \uc81c\uc548\uc774 \ub300\ubd80\ubd84 \uc218\uc6a9\ub418\uc5b4 \uccad\ub098\ub77c\ub294 \ub3d9\ubd81\ucabd\uc758 \ub113\uc740 \uc601\ud1a0\ub97c \uc5bb\uac8c \ub418\uc5c8\ub2e4. \ub610\ud55c, \uccad\ub098\ub77c\uc640 \ub7ec\uc2dc\uc544 \uac04\uc758 \ubb34\uc5ed\uc5d0\ub3c4 \uc790\uc720\ub97c \ubcf4\uc7a5\ud558\uc600\uc73c\ub098, \ub7ec\uc2dc\uc544\uc5d0\ub294 \uadf8\ub2e4\uc9c0 \ub4dd\uc744 \ubcfc \ub9cc\ud55c \uc870\ud56d\uc740 \ub9ce\uc9c0 \uc54a\uc544 \ud6d7\ub0a0 \ud568\ud48d\uc81c\uc758 \uce58\uc138 \ub54c, \ub7ec\uc2dc\uc544\uac00 \uc560\ub85c\ud638 \uc804\uc7c1\uc5d0\uc11c \uccad\ub098\ub77c\uc758 \uc6d0\uc870\ub97c \ube4c\ubbf8\ub85c \ub9cc\uc8fc\uc640 \uc5f0\ud574\uc8fc\uc758 \ub9ce\uc740 \ub545\uc744 \ub3c4\ub85c \uac00\uc838\uac00\uac8c \ud558\ub294 \uc6d0\uc778\uc774 \ub418\uc5c8\ub2e4."} {"question": "\uace0\ub798\uc0c1\uc5b4\uac00 \uac00\uc9c0\ub294 \uc791\uc740 \uc774\ube68\uc758 \uac2f\uc218\ub294?", "paragraph": "\uace0\ub798\uc0c1\uc5b4\ub294 \ud3ed\uc774 1.5m\uae4c\uc9c0 \ub298\uc5b4\ub0a0 \uc218 \uc788\ub294 \uc785\uc744 \uac00\uc84c\uace0, 300\uc5d0\uc11c 350\uac1c\uc758 \uc791\uc740 \uc774\ube68\uc744 \uac00\uc84c\uc73c\uba70, 10\uac1c\uc758 \uc5ec\uacfc \uae30\uad00\uc774 \uc788\ub2e4. \uace0\ub798\uc0c1\uc5b4\ub294 10\uac1c\uc758 \ud070 \uc544\uac00\ubbf8\ub97c \uac00\uc9c0\uace0 \uc788\ub2e4. \uba38\ub9ac\ub294 \ub113\uace0 \ub0a9\uc791\ud558\uba70 \uc55e\uc5d0 2\uac1c\uc758 \uc791\uc740 \ub208\uc774 \ub2ec\ub838\ub2e4.\uace0\ub798\uc0c1\uc5b4\ub294 \uc804\uccb4\uc801\uc73c\ub85c \ud68c\uc0c9\uc778 \ud3b8\uc774\uba70, \ud558\uc580\uc0c9 \ubcf5\ubd80\ub97c \uac00\uc9c0\uace0 \uc788\ub2e4. \ud53c\ubd80\uc5d0\ub294 \uac1c\uccb4\ub9c8\ub2e4 \ub2e4\ub978 \uc605\uc740 \ub178\ub780\uc0c9\uc758 \ubc18\uc810\uacfc \uc904\ubb34\ub2ac\ub97c \uac00\uc9c0\uace0 \uc788\ub2e4. \uace0\ub798\uc0c1\uc5b4\ub294 \uc606\uba74\uc5d0 3\uac1c\uc758 \ub3cc\ucd9c\ub41c \ubd80\ubd84\uc774 \uc788\ub2e4. \ud53c\ubd80\uc758 \ub450\uaed8\ub294 \uc57d 10cm \uc815\ub3c4 \ud55c\ub2e4. \ud55c \uc30d\uc758 \ub4f1\uc9c0\ub290\ub7ec\ubbf8\uc640 \uac00\uc2b4\uc9c0\ub290\ub7ec\ubbf8\ub97c \uac00\uc9c0\uace0 \uc788\uae30\ub3c4 \ud558\ub2e4. \uc131\uc778\uc758 \uaf2c\ub9ac\ub294 \uc57d\uac04 \ucd08\uc2b9\ub2ec \ubaa8\uc591\uc778 \ubc18\uba74, \uce58\uc5b4\uc758 \uaf2c\ub9ac\ub294 \ub0ae\uc740 \ucabd\uc5d0 \uc788\ub294 \uc9c0\ub290\ub7ec\ubbf8\uac00 \uc704\ucabd\uc5d0 \uc788\ub294 \uc9c0\ub290\ub7ec\ubbf8\ubcf4\ub2e4 \uc791\uc740 \ubaa8\uc591\uc774\ub2e4. \uace0\ub798\uc0c1\uc5b4\uc758 \ubd84\uc218\uacf5\uc740 \ub208 \ubc14\ub85c \ub4a4\uc5d0 \uc788\ub2e4.", "answer": "300\uc5d0\uc11c 350\uac1c", "sentence": "\uace0\ub798\uc0c1\uc5b4\ub294 \ud3ed\uc774 1.5m\uae4c\uc9c0 \ub298\uc5b4\ub0a0 \uc218 \uc788\ub294 \uc785\uc744 \uac00\uc84c\uace0, 300\uc5d0\uc11c 350\uac1c \uc758 \uc791\uc740 \uc774\ube68\uc744 \uac00\uc84c\uc73c\uba70, 10\uac1c\uc758 \uc5ec\uacfc \uae30\uad00\uc774 \uc788\ub2e4.", "paragraph_sentence": " \uace0\ub798\uc0c1\uc5b4\ub294 \ud3ed\uc774 1.5m\uae4c\uc9c0 \ub298\uc5b4\ub0a0 \uc218 \uc788\ub294 \uc785\uc744 \uac00\uc84c\uace0, 300\uc5d0\uc11c 350\uac1c \uc758 \uc791\uc740 \uc774\ube68\uc744 \uac00\uc84c\uc73c\uba70, 10\uac1c\uc758 \uc5ec\uacfc \uae30\uad00\uc774 \uc788\ub2e4. \uace0\ub798\uc0c1\uc5b4\ub294 10\uac1c\uc758 \ud070 \uc544\uac00\ubbf8\ub97c \uac00\uc9c0\uace0 \uc788\ub2e4. \uba38\ub9ac\ub294 \ub113\uace0 \ub0a9\uc791\ud558\uba70 \uc55e\uc5d0 2\uac1c\uc758 \uc791\uc740 \ub208\uc774 \ub2ec\ub838\ub2e4.\uace0\ub798\uc0c1\uc5b4\ub294 \uc804\uccb4\uc801\uc73c\ub85c \ud68c\uc0c9\uc778 \ud3b8\uc774\uba70, \ud558\uc580\uc0c9 \ubcf5\ubd80\ub97c \uac00\uc9c0\uace0 \uc788\ub2e4. \ud53c\ubd80\uc5d0\ub294 \uac1c\uccb4\ub9c8\ub2e4 \ub2e4\ub978 \uc605\uc740 \ub178\ub780\uc0c9\uc758 \ubc18\uc810\uacfc \uc904\ubb34\ub2ac\ub97c \uac00\uc9c0\uace0 \uc788\ub2e4. \uace0\ub798\uc0c1\uc5b4\ub294 \uc606\uba74\uc5d0 3\uac1c\uc758 \ub3cc\ucd9c\ub41c \ubd80\ubd84\uc774 \uc788\ub2e4. \ud53c\ubd80\uc758 \ub450\uaed8\ub294 \uc57d 10cm \uc815\ub3c4 \ud55c\ub2e4. \ud55c \uc30d\uc758 \ub4f1\uc9c0\ub290\ub7ec\ubbf8\uc640 \uac00\uc2b4\uc9c0\ub290\ub7ec\ubbf8\ub97c \uac00\uc9c0\uace0 \uc788\uae30\ub3c4 \ud558\ub2e4. \uc131\uc778\uc758 \uaf2c\ub9ac\ub294 \uc57d\uac04 \ucd08\uc2b9\ub2ec \ubaa8\uc591\uc778 \ubc18\uba74, \uce58\uc5b4\uc758 \uaf2c\ub9ac\ub294 \ub0ae\uc740 \ucabd\uc5d0 \uc788\ub294 \uc9c0\ub290\ub7ec\ubbf8\uac00 \uc704\ucabd\uc5d0 \uc788\ub294 \uc9c0\ub290\ub7ec\ubbf8\ubcf4\ub2e4 \uc791\uc740 \ubaa8\uc591\uc774\ub2e4. \uace0\ub798\uc0c1\uc5b4\uc758 \ubd84\uc218\uacf5\uc740 \ub208 \ubc14\ub85c \ub4a4\uc5d0 \uc788\ub2e4.", "paragraph_answer": "\uace0\ub798\uc0c1\uc5b4\ub294 \ud3ed\uc774 1.5m\uae4c\uc9c0 \ub298\uc5b4\ub0a0 \uc218 \uc788\ub294 \uc785\uc744 \uac00\uc84c\uace0, 300\uc5d0\uc11c 350\uac1c \uc758 \uc791\uc740 \uc774\ube68\uc744 \uac00\uc84c\uc73c\uba70, 10\uac1c\uc758 \uc5ec\uacfc \uae30\uad00\uc774 \uc788\ub2e4. \uace0\ub798\uc0c1\uc5b4\ub294 10\uac1c\uc758 \ud070 \uc544\uac00\ubbf8\ub97c \uac00\uc9c0\uace0 \uc788\ub2e4. \uba38\ub9ac\ub294 \ub113\uace0 \ub0a9\uc791\ud558\uba70 \uc55e\uc5d0 2\uac1c\uc758 \uc791\uc740 \ub208\uc774 \ub2ec\ub838\ub2e4.\uace0\ub798\uc0c1\uc5b4\ub294 \uc804\uccb4\uc801\uc73c\ub85c \ud68c\uc0c9\uc778 \ud3b8\uc774\uba70, \ud558\uc580\uc0c9 \ubcf5\ubd80\ub97c \uac00\uc9c0\uace0 \uc788\ub2e4. \ud53c\ubd80\uc5d0\ub294 \uac1c\uccb4\ub9c8\ub2e4 \ub2e4\ub978 \uc605\uc740 \ub178\ub780\uc0c9\uc758 \ubc18\uc810\uacfc \uc904\ubb34\ub2ac\ub97c \uac00\uc9c0\uace0 \uc788\ub2e4. \uace0\ub798\uc0c1\uc5b4\ub294 \uc606\uba74\uc5d0 3\uac1c\uc758 \ub3cc\ucd9c\ub41c \ubd80\ubd84\uc774 \uc788\ub2e4. \ud53c\ubd80\uc758 \ub450\uaed8\ub294 \uc57d 10cm \uc815\ub3c4 \ud55c\ub2e4. \ud55c \uc30d\uc758 \ub4f1\uc9c0\ub290\ub7ec\ubbf8\uc640 \uac00\uc2b4\uc9c0\ub290\ub7ec\ubbf8\ub97c \uac00\uc9c0\uace0 \uc788\uae30\ub3c4 \ud558\ub2e4. \uc131\uc778\uc758 \uaf2c\ub9ac\ub294 \uc57d\uac04 \ucd08\uc2b9\ub2ec \ubaa8\uc591\uc778 \ubc18\uba74, \uce58\uc5b4\uc758 \uaf2c\ub9ac\ub294 \ub0ae\uc740 \ucabd\uc5d0 \uc788\ub294 \uc9c0\ub290\ub7ec\ubbf8\uac00 \uc704\ucabd\uc5d0 \uc788\ub294 \uc9c0\ub290\ub7ec\ubbf8\ubcf4\ub2e4 \uc791\uc740 \ubaa8\uc591\uc774\ub2e4. \uace0\ub798\uc0c1\uc5b4\uc758 \ubd84\uc218\uacf5\uc740 \ub208 \ubc14\ub85c \ub4a4\uc5d0 \uc788\ub2e4.", "sentence_answer": "\uace0\ub798\uc0c1\uc5b4\ub294 \ud3ed\uc774 1.5m\uae4c\uc9c0 \ub298\uc5b4\ub0a0 \uc218 \uc788\ub294 \uc785\uc744 \uac00\uc84c\uace0, 300\uc5d0\uc11c 350\uac1c \uc758 \uc791\uc740 \uc774\ube68\uc744 \uac00\uc84c\uc73c\uba70, 10\uac1c\uc758 \uc5ec\uacfc \uae30\uad00\uc774 \uc788\ub2e4.", "paragraph_id": "6575642-1-0", "paragraph_question": "question: \uace0\ub798\uc0c1\uc5b4\uac00 \uac00\uc9c0\ub294 \uc791\uc740 \uc774\ube68\uc758 \uac2f\uc218\ub294?, context: \uace0\ub798\uc0c1\uc5b4\ub294 \ud3ed\uc774 1.5m\uae4c\uc9c0 \ub298\uc5b4\ub0a0 \uc218 \uc788\ub294 \uc785\uc744 \uac00\uc84c\uace0, 300\uc5d0\uc11c 350\uac1c\uc758 \uc791\uc740 \uc774\ube68\uc744 \uac00\uc84c\uc73c\uba70, 10\uac1c\uc758 \uc5ec\uacfc \uae30\uad00\uc774 \uc788\ub2e4. \uace0\ub798\uc0c1\uc5b4\ub294 10\uac1c\uc758 \ud070 \uc544\uac00\ubbf8\ub97c \uac00\uc9c0\uace0 \uc788\ub2e4. \uba38\ub9ac\ub294 \ub113\uace0 \ub0a9\uc791\ud558\uba70 \uc55e\uc5d0 2\uac1c\uc758 \uc791\uc740 \ub208\uc774 \ub2ec\ub838\ub2e4.\uace0\ub798\uc0c1\uc5b4\ub294 \uc804\uccb4\uc801\uc73c\ub85c \ud68c\uc0c9\uc778 \ud3b8\uc774\uba70, \ud558\uc580\uc0c9 \ubcf5\ubd80\ub97c \uac00\uc9c0\uace0 \uc788\ub2e4. \ud53c\ubd80\uc5d0\ub294 \uac1c\uccb4\ub9c8\ub2e4 \ub2e4\ub978 \uc605\uc740 \ub178\ub780\uc0c9\uc758 \ubc18\uc810\uacfc \uc904\ubb34\ub2ac\ub97c \uac00\uc9c0\uace0 \uc788\ub2e4. \uace0\ub798\uc0c1\uc5b4\ub294 \uc606\uba74\uc5d0 3\uac1c\uc758 \ub3cc\ucd9c\ub41c \ubd80\ubd84\uc774 \uc788\ub2e4. \ud53c\ubd80\uc758 \ub450\uaed8\ub294 \uc57d 10cm \uc815\ub3c4 \ud55c\ub2e4. \ud55c \uc30d\uc758 \ub4f1\uc9c0\ub290\ub7ec\ubbf8\uc640 \uac00\uc2b4\uc9c0\ub290\ub7ec\ubbf8\ub97c \uac00\uc9c0\uace0 \uc788\uae30\ub3c4 \ud558\ub2e4. \uc131\uc778\uc758 \uaf2c\ub9ac\ub294 \uc57d\uac04 \ucd08\uc2b9\ub2ec \ubaa8\uc591\uc778 \ubc18\uba74, \uce58\uc5b4\uc758 \uaf2c\ub9ac\ub294 \ub0ae\uc740 \ucabd\uc5d0 \uc788\ub294 \uc9c0\ub290\ub7ec\ubbf8\uac00 \uc704\ucabd\uc5d0 \uc788\ub294 \uc9c0\ub290\ub7ec\ubbf8\ubcf4\ub2e4 \uc791\uc740 \ubaa8\uc591\uc774\ub2e4. \uace0\ub798\uc0c1\uc5b4\uc758 \ubd84\uc218\uacf5\uc740 \ub208 \ubc14\ub85c \ub4a4\uc5d0 \uc788\ub2e4."} {"question": "\uc77c\ubc18\uacfc\uc815\uc744 \uc774\uc218\ud55c \uc878\uc5c5\uc0dd\uc740 \uc804\uacf5 \uba85\uce6d\uc5d0 \ubb50\ub77c\uace0 \ucd94\uac00 \uae30\uc7ac\ub418\ub098?", "paragraph": "\uacf5\ud559\uad50\uc721\uc778\uc99d\uc81c\ub3c4\ub85c \uc778\uc99d\ubc1b\uc740 \ud559\ubd80\uc640 \ud559\uacfc \ub610\ub294 \uc804\uacf5\uc740 \ud55c\uad6d\uacf5\ud559\uad50\uc721\uc778\uc99d\uc6d0\uc758 \uc778\uc99d \uae30\uc900\uc5d0 \ub9de\ucdb0 \uad50\uc721 \ud504\ub85c\uadf8\ub7a8\uc744 \uad6c\uc131\ud55c\ub2e4. \uacf5\ud559\uad50\uc721\uc778\uc99d\uc81c\ub3c4\ub97c \uc2dc\ud589\ud558\ub294 \uacf3\uc758 \ud559\uc0dd\uc740 \uba3c\uc800 \uc778\uc99d\ud504\ub85c\uadf8\ub7a8\uc5d0 \ub530\ub77c \uc804\uacf5\uc744 \uc774\uc218\ud558\ub294 \uc804\ubb38\uacfc\uc815\uacfc \uc778\uc99d\ud504\ub85c\uadf8\ub7a8\uc744 \ud3ec\uae30\ud558\ub294 \uc77c\ubc18\uacfc\uc815\uc744 \ub450\uace0 \uc120\ud0dd\ud560 \uc218 \uc788\ub2e4. \uc804\ubb38\uacfc\uc815\uc744 \uc774\uc218\ud558\ub294 \ud559\uc0dd\uc740 \ud55c\uad6d\uacf5\ud559\uad50\uc721\uc778\uc99d\uc6d0\uacfc \ud574\ub2f9 \uc778\uc99d\ud504\ub85c\uadf8\ub7a8\uc5d0\uc11c \uc81c\uc2dc\ud558\ub294 \uc778\uc99d\uae30\uc900\uc744 \ub3d9\uc2dc\uc5d0 \ucda9\uc871\ud558\uc5ec\uc57c \ud558\uba70, \uc815\ud574\uc9c4 \uc778\uc99d\ud504\ub85c\uadf8\ub7a8\uc758 \uad50\uacfc\ubaa9 \ucee4\ub9ac\ud058\ub7fc\uc5d0 \ub530\ub77c \uacfc\ubaa9\uc744 \uc774\uc218\ud574\uc57c \uacf5\ud559\uc778\uc99d\uc744 \ubc1b\uc744 \uc218 \uc788\ub2e4. \uc804\ubb38\uacfc\uc815\uc744 \uc774\uc218\ud55c \uc878\uc5c5\uc0dd\uc740 \ud559\uc801\ubd80\uc640 \uac01\uc885 \uc99d\uba85\uc11c \ub4f1\uc5d0 \uc778\uc99d \uc0ac\uc2e4\uc774 \uae30\uc7ac\ub418\uba70, \uc99d\uba85\uc11c\uc5d0\uc11c \uc804\uacf5\uc744 \ud45c\uae30\ud560 \ub54c, \uc778\uc99d\ud504\ub85c\uadf8\ub7a8 \uc774\uc218\uc790\ub294 \uc804\uacf5 \uba85\uce6d\uc5d0 \u2018\uc804\ubb38\u2019\uc744 \ucd94\uac00\ud558\uc5ec \ud45c\uae30\ud55c\ub2e4. \uadf8\ub7ec\ub098 \uc77c\ubc18\uacfc\uc815\uc744 \uc774\uc218\ud55c \uc878\uc5c5\uc0dd\uc740 \uc804\uacf5 \uba85\uce6d\uc5d0 \u2018\uc77c\ubc18\u2019\uc744 \ucd94\uac00 \uae30\uc7ac\ud574 \uc804\ubb38\uacfc\uc815 \uc774\uc218\uc790\uc640 \ucc28\uc774\ub97c \ub450\uac8c \ub41c\ub2e4.", "answer": "\uc77c\ubc18", "sentence": "\uacf5\ud559\uad50\uc721\uc778\uc99d\uc81c\ub3c4\ub97c \uc2dc\ud589\ud558\ub294 \uacf3\uc758 \ud559\uc0dd\uc740 \uba3c\uc800 \uc778\uc99d\ud504\ub85c\uadf8\ub7a8\uc5d0 \ub530\ub77c \uc804\uacf5\uc744 \uc774\uc218\ud558\ub294 \uc804\ubb38\uacfc\uc815\uacfc \uc778\uc99d\ud504\ub85c\uadf8\ub7a8\uc744 \ud3ec\uae30\ud558\ub294 \uc77c\ubc18 \uacfc\uc815\uc744 \ub450\uace0 \uc120\ud0dd\ud560 \uc218 \uc788\ub2e4.", "paragraph_sentence": "\uacf5\ud559\uad50\uc721\uc778\uc99d\uc81c\ub3c4\ub85c \uc778\uc99d\ubc1b\uc740 \ud559\ubd80\uc640 \ud559\uacfc \ub610\ub294 \uc804\uacf5\uc740 \ud55c\uad6d\uacf5\ud559\uad50\uc721\uc778\uc99d\uc6d0\uc758 \uc778\uc99d \uae30\uc900\uc5d0 \ub9de\ucdb0 \uad50\uc721 \ud504\ub85c\uadf8\ub7a8\uc744 \uad6c\uc131\ud55c\ub2e4. \uacf5\ud559\uad50\uc721\uc778\uc99d\uc81c\ub3c4\ub97c \uc2dc\ud589\ud558\ub294 \uacf3\uc758 \ud559\uc0dd\uc740 \uba3c\uc800 \uc778\uc99d\ud504\ub85c\uadf8\ub7a8\uc5d0 \ub530\ub77c \uc804\uacf5\uc744 \uc774\uc218\ud558\ub294 \uc804\ubb38\uacfc\uc815\uacfc \uc778\uc99d\ud504\ub85c\uadf8\ub7a8\uc744 \ud3ec\uae30\ud558\ub294 \uc77c\ubc18 \uacfc\uc815\uc744 \ub450\uace0 \uc120\ud0dd\ud560 \uc218 \uc788\ub2e4. \uc804\ubb38\uacfc\uc815\uc744 \uc774\uc218\ud558\ub294 \ud559\uc0dd\uc740 \ud55c\uad6d\uacf5\ud559\uad50\uc721\uc778\uc99d\uc6d0\uacfc \ud574\ub2f9 \uc778\uc99d\ud504\ub85c\uadf8\ub7a8\uc5d0\uc11c \uc81c\uc2dc\ud558\ub294 \uc778\uc99d\uae30\uc900\uc744 \ub3d9\uc2dc\uc5d0 \ucda9\uc871\ud558\uc5ec\uc57c \ud558\uba70, \uc815\ud574\uc9c4 \uc778\uc99d\ud504\ub85c\uadf8\ub7a8\uc758 \uad50\uacfc\ubaa9 \ucee4\ub9ac\ud058\ub7fc\uc5d0 \ub530\ub77c \uacfc\ubaa9\uc744 \uc774\uc218\ud574\uc57c \uacf5\ud559\uc778\uc99d\uc744 \ubc1b\uc744 \uc218 \uc788\ub2e4. \uc804\ubb38\uacfc\uc815\uc744 \uc774\uc218\ud55c \uc878\uc5c5\uc0dd\uc740 \ud559\uc801\ubd80\uc640 \uac01\uc885 \uc99d\uba85\uc11c \ub4f1\uc5d0 \uc778\uc99d \uc0ac\uc2e4\uc774 \uae30\uc7ac\ub418\uba70, \uc99d\uba85\uc11c\uc5d0\uc11c \uc804\uacf5\uc744 \ud45c\uae30\ud560 \ub54c, \uc778\uc99d\ud504\ub85c\uadf8\ub7a8 \uc774\uc218\uc790\ub294 \uc804\uacf5 \uba85\uce6d\uc5d0 \u2018\uc804\ubb38\u2019\uc744 \ucd94\uac00\ud558\uc5ec \ud45c\uae30\ud55c\ub2e4. \uadf8\ub7ec\ub098 \uc77c\ubc18\uacfc\uc815\uc744 \uc774\uc218\ud55c \uc878\uc5c5\uc0dd\uc740 \uc804\uacf5 \uba85\uce6d\uc5d0 \u2018\uc77c\ubc18\u2019\uc744 \ucd94\uac00 \uae30\uc7ac\ud574 \uc804\ubb38\uacfc\uc815 \uc774\uc218\uc790\uc640 \ucc28\uc774\ub97c \ub450\uac8c \ub41c\ub2e4.", "paragraph_answer": "\uacf5\ud559\uad50\uc721\uc778\uc99d\uc81c\ub3c4\ub85c \uc778\uc99d\ubc1b\uc740 \ud559\ubd80\uc640 \ud559\uacfc \ub610\ub294 \uc804\uacf5\uc740 \ud55c\uad6d\uacf5\ud559\uad50\uc721\uc778\uc99d\uc6d0\uc758 \uc778\uc99d \uae30\uc900\uc5d0 \ub9de\ucdb0 \uad50\uc721 \ud504\ub85c\uadf8\ub7a8\uc744 \uad6c\uc131\ud55c\ub2e4. \uacf5\ud559\uad50\uc721\uc778\uc99d\uc81c\ub3c4\ub97c \uc2dc\ud589\ud558\ub294 \uacf3\uc758 \ud559\uc0dd\uc740 \uba3c\uc800 \uc778\uc99d\ud504\ub85c\uadf8\ub7a8\uc5d0 \ub530\ub77c \uc804\uacf5\uc744 \uc774\uc218\ud558\ub294 \uc804\ubb38\uacfc\uc815\uacfc \uc778\uc99d\ud504\ub85c\uadf8\ub7a8\uc744 \ud3ec\uae30\ud558\ub294 \uc77c\ubc18 \uacfc\uc815\uc744 \ub450\uace0 \uc120\ud0dd\ud560 \uc218 \uc788\ub2e4. \uc804\ubb38\uacfc\uc815\uc744 \uc774\uc218\ud558\ub294 \ud559\uc0dd\uc740 \ud55c\uad6d\uacf5\ud559\uad50\uc721\uc778\uc99d\uc6d0\uacfc \ud574\ub2f9 \uc778\uc99d\ud504\ub85c\uadf8\ub7a8\uc5d0\uc11c \uc81c\uc2dc\ud558\ub294 \uc778\uc99d\uae30\uc900\uc744 \ub3d9\uc2dc\uc5d0 \ucda9\uc871\ud558\uc5ec\uc57c \ud558\uba70, \uc815\ud574\uc9c4 \uc778\uc99d\ud504\ub85c\uadf8\ub7a8\uc758 \uad50\uacfc\ubaa9 \ucee4\ub9ac\ud058\ub7fc\uc5d0 \ub530\ub77c \uacfc\ubaa9\uc744 \uc774\uc218\ud574\uc57c \uacf5\ud559\uc778\uc99d\uc744 \ubc1b\uc744 \uc218 \uc788\ub2e4. \uc804\ubb38\uacfc\uc815\uc744 \uc774\uc218\ud55c \uc878\uc5c5\uc0dd\uc740 \ud559\uc801\ubd80\uc640 \uac01\uc885 \uc99d\uba85\uc11c \ub4f1\uc5d0 \uc778\uc99d \uc0ac\uc2e4\uc774 \uae30\uc7ac\ub418\uba70, \uc99d\uba85\uc11c\uc5d0\uc11c \uc804\uacf5\uc744 \ud45c\uae30\ud560 \ub54c, \uc778\uc99d\ud504\ub85c\uadf8\ub7a8 \uc774\uc218\uc790\ub294 \uc804\uacf5 \uba85\uce6d\uc5d0 \u2018\uc804\ubb38\u2019\uc744 \ucd94\uac00\ud558\uc5ec \ud45c\uae30\ud55c\ub2e4. \uadf8\ub7ec\ub098 \uc77c\ubc18\uacfc\uc815\uc744 \uc774\uc218\ud55c \uc878\uc5c5\uc0dd\uc740 \uc804\uacf5 \uba85\uce6d\uc5d0 \u2018\uc77c\ubc18\u2019\uc744 \ucd94\uac00 \uae30\uc7ac\ud574 \uc804\ubb38\uacfc\uc815 \uc774\uc218\uc790\uc640 \ucc28\uc774\ub97c \ub450\uac8c \ub41c\ub2e4.", "sentence_answer": "\uacf5\ud559\uad50\uc721\uc778\uc99d\uc81c\ub3c4\ub97c \uc2dc\ud589\ud558\ub294 \uacf3\uc758 \ud559\uc0dd\uc740 \uba3c\uc800 \uc778\uc99d\ud504\ub85c\uadf8\ub7a8\uc5d0 \ub530\ub77c \uc804\uacf5\uc744 \uc774\uc218\ud558\ub294 \uc804\ubb38\uacfc\uc815\uacfc \uc778\uc99d\ud504\ub85c\uadf8\ub7a8\uc744 \ud3ec\uae30\ud558\ub294 \uc77c\ubc18 \uacfc\uc815\uc744 \ub450\uace0 \uc120\ud0dd\ud560 \uc218 \uc788\ub2e4.", "paragraph_id": "6582250-9-2", "paragraph_question": "question: \uc77c\ubc18\uacfc\uc815\uc744 \uc774\uc218\ud55c \uc878\uc5c5\uc0dd\uc740 \uc804\uacf5 \uba85\uce6d\uc5d0 \ubb50\ub77c\uace0 \ucd94\uac00 \uae30\uc7ac\ub418\ub098?, context: \uacf5\ud559\uad50\uc721\uc778\uc99d\uc81c\ub3c4\ub85c \uc778\uc99d\ubc1b\uc740 \ud559\ubd80\uc640 \ud559\uacfc \ub610\ub294 \uc804\uacf5\uc740 \ud55c\uad6d\uacf5\ud559\uad50\uc721\uc778\uc99d\uc6d0\uc758 \uc778\uc99d \uae30\uc900\uc5d0 \ub9de\ucdb0 \uad50\uc721 \ud504\ub85c\uadf8\ub7a8\uc744 \uad6c\uc131\ud55c\ub2e4. \uacf5\ud559\uad50\uc721\uc778\uc99d\uc81c\ub3c4\ub97c \uc2dc\ud589\ud558\ub294 \uacf3\uc758 \ud559\uc0dd\uc740 \uba3c\uc800 \uc778\uc99d\ud504\ub85c\uadf8\ub7a8\uc5d0 \ub530\ub77c \uc804\uacf5\uc744 \uc774\uc218\ud558\ub294 \uc804\ubb38\uacfc\uc815\uacfc \uc778\uc99d\ud504\ub85c\uadf8\ub7a8\uc744 \ud3ec\uae30\ud558\ub294 \uc77c\ubc18\uacfc\uc815\uc744 \ub450\uace0 \uc120\ud0dd\ud560 \uc218 \uc788\ub2e4. \uc804\ubb38\uacfc\uc815\uc744 \uc774\uc218\ud558\ub294 \ud559\uc0dd\uc740 \ud55c\uad6d\uacf5\ud559\uad50\uc721\uc778\uc99d\uc6d0\uacfc \ud574\ub2f9 \uc778\uc99d\ud504\ub85c\uadf8\ub7a8\uc5d0\uc11c \uc81c\uc2dc\ud558\ub294 \uc778\uc99d\uae30\uc900\uc744 \ub3d9\uc2dc\uc5d0 \ucda9\uc871\ud558\uc5ec\uc57c \ud558\uba70, \uc815\ud574\uc9c4 \uc778\uc99d\ud504\ub85c\uadf8\ub7a8\uc758 \uad50\uacfc\ubaa9 \ucee4\ub9ac\ud058\ub7fc\uc5d0 \ub530\ub77c \uacfc\ubaa9\uc744 \uc774\uc218\ud574\uc57c \uacf5\ud559\uc778\uc99d\uc744 \ubc1b\uc744 \uc218 \uc788\ub2e4. \uc804\ubb38\uacfc\uc815\uc744 \uc774\uc218\ud55c \uc878\uc5c5\uc0dd\uc740 \ud559\uc801\ubd80\uc640 \uac01\uc885 \uc99d\uba85\uc11c \ub4f1\uc5d0 \uc778\uc99d \uc0ac\uc2e4\uc774 \uae30\uc7ac\ub418\uba70, \uc99d\uba85\uc11c\uc5d0\uc11c \uc804\uacf5\uc744 \ud45c\uae30\ud560 \ub54c, \uc778\uc99d\ud504\ub85c\uadf8\ub7a8 \uc774\uc218\uc790\ub294 \uc804\uacf5 \uba85\uce6d\uc5d0 \u2018\uc804\ubb38\u2019\uc744 \ucd94\uac00\ud558\uc5ec \ud45c\uae30\ud55c\ub2e4. \uadf8\ub7ec\ub098 \uc77c\ubc18\uacfc\uc815\uc744 \uc774\uc218\ud55c \uc878\uc5c5\uc0dd\uc740 \uc804\uacf5 \uba85\uce6d\uc5d0 \u2018\uc77c\ubc18\u2019\uc744 \ucd94\uac00 \uae30\uc7ac\ud574 \uc804\ubb38\uacfc\uc815 \uc774\uc218\uc790\uc640 \ucc28\uc774\ub97c \ub450\uac8c \ub41c\ub2e4."} {"question": "1934\ub144\uc5d0 \ub9cc\uc288\ud0c0\uc778\uc740 \ubb34\uc5c7\uc758 \ub3c4\uc785\uc744 \ubc18\ub300\ud55c \uc720\uc77c\ud55c \uad6d\uac00\ubc29\uc704\uad70 \uc7a5\uad50\uc600\ub098?", "paragraph": "\ubc18\uc720\ub300\uc8fc\uc758\ub294 \uc774 \uc2dc\uae30 \ub3c5\uc77c\uc744 \ube44\ub86f\ud55c \uc720\ub7fd \uc804\uccb4\uc5d0 \ud37c\uc838 \uc788\uc5c8\uace0, \ub9cc\uc288\ud0c0\uc778\uc758 \ud0dc\ub3c4 \uc5ed\uc2dc \uadf8\uc640 \ub2e4\ub97c \ubc14 \uc5c6\uc5c8\ub2e4. \ub9cc\uc288\ud0c0\uc778\uc758 \ud589\ub3d9\uc740 \ud788\ud2c0\ub7ec\uc640 \ub098\uce58 \uc815\uad8c\uc5d0 \ub300\ud55c \ucda9\uc131\uc758 \ubc1c\ub85c\uc774\uae30\ub3c4 \ud588\uc73c\uba70, \uc77c\ubd80\ub294 \uc804\ud1b5\uc801\uc778 \ud504\ub85c\uc774\uc13c \uad70\uc0ac \uac00\uce58\uc5d0 \uae30\ubc18\ud55c \uc758\ubb34\uac10 \ub54c\ubb38\uc774\uae30\ub3c4 \ud588\ub2e4. \ub9cc\uc288\ud0c0\uc778\uc758 \ud788\ud2c0\ub7ec \ube44\ud310\uc740 \uc804\uc7c1 \uc218\ud589 \uc640\uc911 \uc758\uacac\uc774 \uc77c\uce58\ud558\uc9c0 \uc54a\uc558\ub358 \uc77c\ub4e4\uc5d0 \ub300\ud574\uc11c\ub9cc \ud55c\uc815\ub418\uc5b4 \uc788\ub2e4. \uc548\ud1a0\ub2c8 \ube44\ubcf4\ub974, \ube44\ub178\uc774\ud2b8 \ub974\uba54\uc774(Beno\u00eet Lemay)\ub97c \ube44\ub86f\ud55c \uc77c\ubd80 \uc5ed\uc0ac\ud559\uc790\ub4e4\uc740 \ub9cc\uc288\ud0c0\uc778\uc758 \uc870\uc0c1 \uc911\uc5d0 \uc720\ub300\uacc4\uc640 \uc2ac\ub77c\ube0c\uacc4\uac00 \uc788\ub2e4\ub294 \uc758\uacac\uc744 \ub0b4\ub193\uae30\ub3c4 \ud55c\ub2e4. \ub9cc\uc288\ud0c0\uc778\uc740 1934\ub144\uc758 \uc544\ub9ac\uc544\uc778 \uc870\ud56d \ub3c4\uc785\uc744 \ubc18\ub300\ud55c \uc720\uc77c\ud55c \uad6d\uac00\ubc29\uc704\uad70 \uc7a5\uad50\uc600\ub2e4. \ub9cc\uc288\ud0c0\uc778\uc740 \ubca0\ud06c \uc7a5\uad70\uc5d0\uac8c \ubcf4\ub0b4\ub294 \ud56d\uc758 \ud3b8\uc9c0\uc5d0\uc11c \ub204\uad6c\ub4e0\uc9c0 \uad70\uc5d0 \ubcf5\ubb34\ud558\ub824\uace0 \uc790\uc6d0\ud55c \uc0ac\ub78c\uc774\ub77c\uba74 \uc790\uc2e0\uc758 \uac00\uce58\ub97c \uc774\ubbf8 \uc99d\uba85\ud55c \uc0ac\ub78c\uc774\ub77c\uace0 \uc37c\ub2e4.", "answer": "\uc544\ub9ac\uc544\uc778 \uc870\ud56d", "sentence": "\ub9cc\uc288\ud0c0\uc778\uc740 1934\ub144\uc758 \uc544\ub9ac\uc544\uc778 \uc870\ud56d \ub3c4\uc785\uc744 \ubc18\ub300\ud55c \uc720\uc77c\ud55c \uad6d\uac00\ubc29\uc704\uad70 \uc7a5\uad50\uc600\ub2e4.", "paragraph_sentence": "\ubc18\uc720\ub300\uc8fc\uc758\ub294 \uc774 \uc2dc\uae30 \ub3c5\uc77c\uc744 \ube44\ub86f\ud55c \uc720\ub7fd \uc804\uccb4\uc5d0 \ud37c\uc838 \uc788\uc5c8\uace0, \ub9cc\uc288\ud0c0\uc778\uc758 \ud0dc\ub3c4 \uc5ed\uc2dc \uadf8\uc640 \ub2e4\ub97c \ubc14 \uc5c6\uc5c8\ub2e4. \ub9cc\uc288\ud0c0\uc778\uc758 \ud589\ub3d9\uc740 \ud788\ud2c0\ub7ec\uc640 \ub098\uce58 \uc815\uad8c\uc5d0 \ub300\ud55c \ucda9\uc131\uc758 \ubc1c\ub85c\uc774\uae30\ub3c4 \ud588\uc73c\uba70, \uc77c\ubd80\ub294 \uc804\ud1b5\uc801\uc778 \ud504\ub85c\uc774\uc13c \uad70\uc0ac \uac00\uce58\uc5d0 \uae30\ubc18\ud55c \uc758\ubb34\uac10 \ub54c\ubb38\uc774\uae30\ub3c4 \ud588\ub2e4. \ub9cc\uc288\ud0c0\uc778\uc758 \ud788\ud2c0\ub7ec \ube44\ud310\uc740 \uc804\uc7c1 \uc218\ud589 \uc640\uc911 \uc758\uacac\uc774 \uc77c\uce58\ud558\uc9c0 \uc54a\uc558\ub358 \uc77c\ub4e4\uc5d0 \ub300\ud574\uc11c\ub9cc \ud55c\uc815\ub418\uc5b4 \uc788\ub2e4. \uc548\ud1a0\ub2c8 \ube44\ubcf4\ub974, \ube44\ub178\uc774\ud2b8 \ub974\uba54\uc774(Beno\u00eet Lemay)\ub97c \ube44\ub86f\ud55c \uc77c\ubd80 \uc5ed\uc0ac\ud559\uc790\ub4e4\uc740 \ub9cc\uc288\ud0c0\uc778\uc758 \uc870\uc0c1 \uc911\uc5d0 \uc720\ub300\uacc4\uc640 \uc2ac\ub77c\ube0c\uacc4\uac00 \uc788\ub2e4\ub294 \uc758\uacac\uc744 \ub0b4\ub193\uae30\ub3c4 \ud55c\ub2e4. \ub9cc\uc288\ud0c0\uc778\uc740 1934\ub144\uc758 \uc544\ub9ac\uc544\uc778 \uc870\ud56d \ub3c4\uc785\uc744 \ubc18\ub300\ud55c \uc720\uc77c\ud55c \uad6d\uac00\ubc29\uc704\uad70 \uc7a5\uad50\uc600\ub2e4. \ub9cc\uc288\ud0c0\uc778\uc740 \ubca0\ud06c \uc7a5\uad70\uc5d0\uac8c \ubcf4\ub0b4\ub294 \ud56d\uc758 \ud3b8\uc9c0\uc5d0\uc11c \ub204\uad6c\ub4e0\uc9c0 \uad70\uc5d0 \ubcf5\ubb34\ud558\ub824\uace0 \uc790\uc6d0\ud55c \uc0ac\ub78c\uc774\ub77c\uba74 \uc790\uc2e0\uc758 \uac00\uce58\ub97c \uc774\ubbf8 \uc99d\uba85\ud55c \uc0ac\ub78c\uc774\ub77c\uace0 \uc37c\ub2e4.", "paragraph_answer": "\ubc18\uc720\ub300\uc8fc\uc758\ub294 \uc774 \uc2dc\uae30 \ub3c5\uc77c\uc744 \ube44\ub86f\ud55c \uc720\ub7fd \uc804\uccb4\uc5d0 \ud37c\uc838 \uc788\uc5c8\uace0, \ub9cc\uc288\ud0c0\uc778\uc758 \ud0dc\ub3c4 \uc5ed\uc2dc \uadf8\uc640 \ub2e4\ub97c \ubc14 \uc5c6\uc5c8\ub2e4. \ub9cc\uc288\ud0c0\uc778\uc758 \ud589\ub3d9\uc740 \ud788\ud2c0\ub7ec\uc640 \ub098\uce58 \uc815\uad8c\uc5d0 \ub300\ud55c \ucda9\uc131\uc758 \ubc1c\ub85c\uc774\uae30\ub3c4 \ud588\uc73c\uba70, \uc77c\ubd80\ub294 \uc804\ud1b5\uc801\uc778 \ud504\ub85c\uc774\uc13c \uad70\uc0ac \uac00\uce58\uc5d0 \uae30\ubc18\ud55c \uc758\ubb34\uac10 \ub54c\ubb38\uc774\uae30\ub3c4 \ud588\ub2e4. \ub9cc\uc288\ud0c0\uc778\uc758 \ud788\ud2c0\ub7ec \ube44\ud310\uc740 \uc804\uc7c1 \uc218\ud589 \uc640\uc911 \uc758\uacac\uc774 \uc77c\uce58\ud558\uc9c0 \uc54a\uc558\ub358 \uc77c\ub4e4\uc5d0 \ub300\ud574\uc11c\ub9cc \ud55c\uc815\ub418\uc5b4 \uc788\ub2e4. \uc548\ud1a0\ub2c8 \ube44\ubcf4\ub974, \ube44\ub178\uc774\ud2b8 \ub974\uba54\uc774(Beno\u00eet Lemay)\ub97c \ube44\ub86f\ud55c \uc77c\ubd80 \uc5ed\uc0ac\ud559\uc790\ub4e4\uc740 \ub9cc\uc288\ud0c0\uc778\uc758 \uc870\uc0c1 \uc911\uc5d0 \uc720\ub300\uacc4\uc640 \uc2ac\ub77c\ube0c\uacc4\uac00 \uc788\ub2e4\ub294 \uc758\uacac\uc744 \ub0b4\ub193\uae30\ub3c4 \ud55c\ub2e4. \ub9cc\uc288\ud0c0\uc778\uc740 1934\ub144\uc758 \uc544\ub9ac\uc544\uc778 \uc870\ud56d \ub3c4\uc785\uc744 \ubc18\ub300\ud55c \uc720\uc77c\ud55c \uad6d\uac00\ubc29\uc704\uad70 \uc7a5\uad50\uc600\ub2e4. \ub9cc\uc288\ud0c0\uc778\uc740 \ubca0\ud06c \uc7a5\uad70\uc5d0\uac8c \ubcf4\ub0b4\ub294 \ud56d\uc758 \ud3b8\uc9c0\uc5d0\uc11c \ub204\uad6c\ub4e0\uc9c0 \uad70\uc5d0 \ubcf5\ubb34\ud558\ub824\uace0 \uc790\uc6d0\ud55c \uc0ac\ub78c\uc774\ub77c\uba74 \uc790\uc2e0\uc758 \uac00\uce58\ub97c \uc774\ubbf8 \uc99d\uba85\ud55c \uc0ac\ub78c\uc774\ub77c\uace0 \uc37c\ub2e4.", "sentence_answer": "\ub9cc\uc288\ud0c0\uc778\uc740 1934\ub144\uc758 \uc544\ub9ac\uc544\uc778 \uc870\ud56d \ub3c4\uc785\uc744 \ubc18\ub300\ud55c \uc720\uc77c\ud55c \uad6d\uac00\ubc29\uc704\uad70 \uc7a5\uad50\uc600\ub2e4.", "paragraph_id": "6540644-37-1", "paragraph_question": "question: 1934\ub144\uc5d0 \ub9cc\uc288\ud0c0\uc778\uc740 \ubb34\uc5c7\uc758 \ub3c4\uc785\uc744 \ubc18\ub300\ud55c \uc720\uc77c\ud55c \uad6d\uac00\ubc29\uc704\uad70 \uc7a5\uad50\uc600\ub098?, context: \ubc18\uc720\ub300\uc8fc\uc758\ub294 \uc774 \uc2dc\uae30 \ub3c5\uc77c\uc744 \ube44\ub86f\ud55c \uc720\ub7fd \uc804\uccb4\uc5d0 \ud37c\uc838 \uc788\uc5c8\uace0, \ub9cc\uc288\ud0c0\uc778\uc758 \ud0dc\ub3c4 \uc5ed\uc2dc \uadf8\uc640 \ub2e4\ub97c \ubc14 \uc5c6\uc5c8\ub2e4. \ub9cc\uc288\ud0c0\uc778\uc758 \ud589\ub3d9\uc740 \ud788\ud2c0\ub7ec\uc640 \ub098\uce58 \uc815\uad8c\uc5d0 \ub300\ud55c \ucda9\uc131\uc758 \ubc1c\ub85c\uc774\uae30\ub3c4 \ud588\uc73c\uba70, \uc77c\ubd80\ub294 \uc804\ud1b5\uc801\uc778 \ud504\ub85c\uc774\uc13c \uad70\uc0ac \uac00\uce58\uc5d0 \uae30\ubc18\ud55c \uc758\ubb34\uac10 \ub54c\ubb38\uc774\uae30\ub3c4 \ud588\ub2e4. \ub9cc\uc288\ud0c0\uc778\uc758 \ud788\ud2c0\ub7ec \ube44\ud310\uc740 \uc804\uc7c1 \uc218\ud589 \uc640\uc911 \uc758\uacac\uc774 \uc77c\uce58\ud558\uc9c0 \uc54a\uc558\ub358 \uc77c\ub4e4\uc5d0 \ub300\ud574\uc11c\ub9cc \ud55c\uc815\ub418\uc5b4 \uc788\ub2e4. \uc548\ud1a0\ub2c8 \ube44\ubcf4\ub974, \ube44\ub178\uc774\ud2b8 \ub974\uba54\uc774(Beno\u00eet Lemay)\ub97c \ube44\ub86f\ud55c \uc77c\ubd80 \uc5ed\uc0ac\ud559\uc790\ub4e4\uc740 \ub9cc\uc288\ud0c0\uc778\uc758 \uc870\uc0c1 \uc911\uc5d0 \uc720\ub300\uacc4\uc640 \uc2ac\ub77c\ube0c\uacc4\uac00 \uc788\ub2e4\ub294 \uc758\uacac\uc744 \ub0b4\ub193\uae30\ub3c4 \ud55c\ub2e4. \ub9cc\uc288\ud0c0\uc778\uc740 1934\ub144\uc758 \uc544\ub9ac\uc544\uc778 \uc870\ud56d \ub3c4\uc785\uc744 \ubc18\ub300\ud55c \uc720\uc77c\ud55c \uad6d\uac00\ubc29\uc704\uad70 \uc7a5\uad50\uc600\ub2e4. \ub9cc\uc288\ud0c0\uc778\uc740 \ubca0\ud06c \uc7a5\uad70\uc5d0\uac8c \ubcf4\ub0b4\ub294 \ud56d\uc758 \ud3b8\uc9c0\uc5d0\uc11c \ub204\uad6c\ub4e0\uc9c0 \uad70\uc5d0 \ubcf5\ubb34\ud558\ub824\uace0 \uc790\uc6d0\ud55c \uc0ac\ub78c\uc774\ub77c\uba74 \uc790\uc2e0\uc758 \uac00\uce58\ub97c \uc774\ubbf8 \uc99d\uba85\ud55c \uc0ac\ub78c\uc774\ub77c\uace0 \uc37c\ub2e4."} {"question": "\ucd5c\ucd08\ub85c \ud55c\ubc18\ub3c4\uc5d0 \ub3c4\ucc29\ud55c \ubbf8\uad70\uc740 \uc5b4\ub514\uc5d0 \ucc29\ub959\ud558\uc600\ub294\uac00?", "paragraph": "\ubbf8\uad6d\uc740 \ubbf8\uad6d \uc721\uad70 \uc81c24\uad70\ub2e8 45,000\uba85\uc744 \uc810\ub839\uc791\uc804\uc5d0 \ud22c\uc785\ud588\ub2e4. \ud55c\ubc18\ub3c4\uc5d0 \uc0c1\ub959\ud55c \ucd5c\ucd08\uc758 \ubbf8\uad70\uc740 1945\ub144 9\uc6d4 4\uc77c \uc11c\uc6b8 \uadfc\ucc98 \uae40\ud3ec\ube44\ud589\uc7a5\uc5d0 \ucc29\ub959\ud55c \uc18c\uaddc\ubaa8 \ubd80\ub300\uc600\ub2e4. 9\uc6d4 8\uc77c\uc5d0\ub294 \uc778\ucc9c\uc5d0 \uc81c7\ubcf4\ubcd1\uc0ac\ub2e8 \uc18c\uc18d \ubcd1\uc0ac 14\uba85\uc774 \uc0c1\ub959\ud588\uc73c\uba70, \ub2e4\uc74c \ub0a0 \ub300\uaddc\ubaa8 \uc0c1\ub959\uc774 \uc2dc\uc791\ub418\uc5c8\ub2e4. \uc800\uc790 \ud3f4 M. \uc5d0\ub4dc\uc6cc\ub4dc\uc5d0 \ub530\ub974\uba74 \ubbf8\uad6d \uc815\ubd80\ub294 \ub2f9\uc2dc \ud55c\uad6d\uc5d0 \ub300\ud574\uc11c\ub294 \uac70\uc758 \uad00\uc2ec\uc774 \uc5c6\uc5c8\uc73c\uba70, \uc774\ub97c \uc9c4\ud589\ud55c \uc0ac\ub78c\uc740 \uac70\uc758 \ub300\ubd80\ubd84\uc758 \uc138\uacc4\ub300\uc804 \uc804\ud6c4 \uacb0\uc815\uc758 \uac70\uc758 \ub9e1\uace0 \uc5f0\ud569\uad70 \uc810\ub839\ud558\uc758 \uc77c\ubcf8\uc758 \uc9c0\ud718\uad00\uc778 \ub300\uc7a5 \ub354\uae00\ub7ec\uc2a4 \ub9e5\uc544\ub354\uc600\ub2e4. \uadf8\ub7ec\ub098, \ub9e5\uc544\ub354 \uc7a5\uad70\uc740 \uc774\ubbf8 \uc77c\ubcf8\uc5d0\uc11c \uc218\ud589\ud574\uc57c \ud560 \uc791\uc804\uc5d0 \ub5a0\ubc00\ub838\uace0, \uacb0\uad6d \uc774 \uc5f4\uc57d\ud55c \ud55c\ubc18\ub3c4 \uc810\ub839\uc791\uc804\uc758 \uc9c0\ud718\uad00\uc744 \uc911\uc7a5 \uc874 \ud558\uc9c0\uc5d0\uac8c \ub9e1\uacbc\ub2e4. \ud638\uc9c0\ub294 \uc11c\uc6b8 \ubc18\ub2e4 \ud638\ud154\uc5d0 \uc0ac\ub839\ubd80\ub97c \uc138\uc6b0\uace0 \uad70\uc0ac \uc815\ubd80\ub97c \uc218\ub9bd\ud588\uc73c\uba70, \ud55c\uad6d\uc758 \uacf5\uc2dd \uc5b8\uc5b4\ub97c \uc601\uc5b4\ub85c \uc815\ud588\uc73c\uba70 \ubbf8\uad6d\uc5d0\uac8c \ud638\uc758\uc801\uc778 \ub3c5\ub9bd \uc815\ubd80\ub97c \uc218\ub9bd\ud558\uae30 \uc2dc\uc791\ud588\ub2e4.", "answer": "\uae40\ud3ec\ube44\ud589\uc7a5", "sentence": "\ud55c\ubc18\ub3c4\uc5d0 \uc0c1\ub959\ud55c \ucd5c\ucd08\uc758 \ubbf8\uad70\uc740 1945\ub144 9\uc6d4 4\uc77c \uc11c\uc6b8 \uadfc\ucc98 \uae40\ud3ec\ube44\ud589\uc7a5 \uc5d0 \ucc29\ub959\ud55c \uc18c\uaddc\ubaa8 \ubd80\ub300\uc600\ub2e4.", "paragraph_sentence": "\ubbf8\uad6d\uc740 \ubbf8\uad6d \uc721\uad70 \uc81c24\uad70\ub2e8 45,000\uba85\uc744 \uc810\ub839\uc791\uc804\uc5d0 \ud22c\uc785\ud588\ub2e4. \ud55c\ubc18\ub3c4\uc5d0 \uc0c1\ub959\ud55c \ucd5c\ucd08\uc758 \ubbf8\uad70\uc740 1945\ub144 9\uc6d4 4\uc77c \uc11c\uc6b8 \uadfc\ucc98 \uae40\ud3ec\ube44\ud589\uc7a5 \uc5d0 \ucc29\ub959\ud55c \uc18c\uaddc\ubaa8 \ubd80\ub300\uc600\ub2e4. 9\uc6d4 8\uc77c\uc5d0\ub294 \uc778\ucc9c\uc5d0 \uc81c7\ubcf4\ubcd1\uc0ac\ub2e8 \uc18c\uc18d \ubcd1\uc0ac 14\uba85\uc774 \uc0c1\ub959\ud588\uc73c\uba70, \ub2e4\uc74c \ub0a0 \ub300\uaddc\ubaa8 \uc0c1\ub959\uc774 \uc2dc\uc791\ub418\uc5c8\ub2e4. \uc800\uc790 \ud3f4 M. \uc5d0\ub4dc\uc6cc\ub4dc\uc5d0 \ub530\ub974\uba74 \ubbf8\uad6d \uc815\ubd80\ub294 \ub2f9\uc2dc \ud55c\uad6d\uc5d0 \ub300\ud574\uc11c\ub294 \uac70\uc758 \uad00\uc2ec\uc774 \uc5c6\uc5c8\uc73c\uba70, \uc774\ub97c \uc9c4\ud589\ud55c \uc0ac\ub78c\uc740 \uac70\uc758 \ub300\ubd80\ubd84\uc758 \uc138\uacc4\ub300\uc804 \uc804\ud6c4 \uacb0\uc815\uc758 \uac70\uc758 \ub9e1\uace0 \uc5f0\ud569\uad70 \uc810\ub839\ud558\uc758 \uc77c\ubcf8\uc758 \uc9c0\ud718\uad00\uc778 \ub300\uc7a5 \ub354\uae00\ub7ec\uc2a4 \ub9e5\uc544\ub354\uc600\ub2e4. \uadf8\ub7ec\ub098, \ub9e5\uc544\ub354 \uc7a5\uad70\uc740 \uc774\ubbf8 \uc77c\ubcf8\uc5d0\uc11c \uc218\ud589\ud574\uc57c \ud560 \uc791\uc804\uc5d0 \ub5a0\ubc00\ub838\uace0, \uacb0\uad6d \uc774 \uc5f4\uc57d\ud55c \ud55c\ubc18\ub3c4 \uc810\ub839\uc791\uc804\uc758 \uc9c0\ud718\uad00\uc744 \uc911\uc7a5 \uc874 \ud558\uc9c0\uc5d0\uac8c \ub9e1\uacbc\ub2e4. \ud638\uc9c0\ub294 \uc11c\uc6b8 \ubc18\ub2e4 \ud638\ud154\uc5d0 \uc0ac\ub839\ubd80\ub97c \uc138\uc6b0\uace0 \uad70\uc0ac \uc815\ubd80\ub97c \uc218\ub9bd\ud588\uc73c\uba70, \ud55c\uad6d\uc758 \uacf5\uc2dd \uc5b8\uc5b4\ub97c \uc601\uc5b4\ub85c \uc815\ud588\uc73c\uba70 \ubbf8\uad6d\uc5d0\uac8c \ud638\uc758\uc801\uc778 \ub3c5\ub9bd \uc815\ubd80\ub97c \uc218\ub9bd\ud558\uae30 \uc2dc\uc791\ud588\ub2e4.", "paragraph_answer": "\ubbf8\uad6d\uc740 \ubbf8\uad6d \uc721\uad70 \uc81c24\uad70\ub2e8 45,000\uba85\uc744 \uc810\ub839\uc791\uc804\uc5d0 \ud22c\uc785\ud588\ub2e4. \ud55c\ubc18\ub3c4\uc5d0 \uc0c1\ub959\ud55c \ucd5c\ucd08\uc758 \ubbf8\uad70\uc740 1945\ub144 9\uc6d4 4\uc77c \uc11c\uc6b8 \uadfc\ucc98 \uae40\ud3ec\ube44\ud589\uc7a5 \uc5d0 \ucc29\ub959\ud55c \uc18c\uaddc\ubaa8 \ubd80\ub300\uc600\ub2e4. 9\uc6d4 8\uc77c\uc5d0\ub294 \uc778\ucc9c\uc5d0 \uc81c7\ubcf4\ubcd1\uc0ac\ub2e8 \uc18c\uc18d \ubcd1\uc0ac 14\uba85\uc774 \uc0c1\ub959\ud588\uc73c\uba70, \ub2e4\uc74c \ub0a0 \ub300\uaddc\ubaa8 \uc0c1\ub959\uc774 \uc2dc\uc791\ub418\uc5c8\ub2e4. \uc800\uc790 \ud3f4 M. \uc5d0\ub4dc\uc6cc\ub4dc\uc5d0 \ub530\ub974\uba74 \ubbf8\uad6d \uc815\ubd80\ub294 \ub2f9\uc2dc \ud55c\uad6d\uc5d0 \ub300\ud574\uc11c\ub294 \uac70\uc758 \uad00\uc2ec\uc774 \uc5c6\uc5c8\uc73c\uba70, \uc774\ub97c \uc9c4\ud589\ud55c \uc0ac\ub78c\uc740 \uac70\uc758 \ub300\ubd80\ubd84\uc758 \uc138\uacc4\ub300\uc804 \uc804\ud6c4 \uacb0\uc815\uc758 \uac70\uc758 \ub9e1\uace0 \uc5f0\ud569\uad70 \uc810\ub839\ud558\uc758 \uc77c\ubcf8\uc758 \uc9c0\ud718\uad00\uc778 \ub300\uc7a5 \ub354\uae00\ub7ec\uc2a4 \ub9e5\uc544\ub354\uc600\ub2e4. \uadf8\ub7ec\ub098, \ub9e5\uc544\ub354 \uc7a5\uad70\uc740 \uc774\ubbf8 \uc77c\ubcf8\uc5d0\uc11c \uc218\ud589\ud574\uc57c \ud560 \uc791\uc804\uc5d0 \ub5a0\ubc00\ub838\uace0, \uacb0\uad6d \uc774 \uc5f4\uc57d\ud55c \ud55c\ubc18\ub3c4 \uc810\ub839\uc791\uc804\uc758 \uc9c0\ud718\uad00\uc744 \uc911\uc7a5 \uc874 \ud558\uc9c0\uc5d0\uac8c \ub9e1\uacbc\ub2e4. \ud638\uc9c0\ub294 \uc11c\uc6b8 \ubc18\ub2e4 \ud638\ud154\uc5d0 \uc0ac\ub839\ubd80\ub97c \uc138\uc6b0\uace0 \uad70\uc0ac \uc815\ubd80\ub97c \uc218\ub9bd\ud588\uc73c\uba70, \ud55c\uad6d\uc758 \uacf5\uc2dd \uc5b8\uc5b4\ub97c \uc601\uc5b4\ub85c \uc815\ud588\uc73c\uba70 \ubbf8\uad6d\uc5d0\uac8c \ud638\uc758\uc801\uc778 \ub3c5\ub9bd \uc815\ubd80\ub97c \uc218\ub9bd\ud558\uae30 \uc2dc\uc791\ud588\ub2e4.", "sentence_answer": "\ud55c\ubc18\ub3c4\uc5d0 \uc0c1\ub959\ud55c \ucd5c\ucd08\uc758 \ubbf8\uad70\uc740 1945\ub144 9\uc6d4 4\uc77c \uc11c\uc6b8 \uadfc\ucc98 \uae40\ud3ec\ube44\ud589\uc7a5 \uc5d0 \ucc29\ub959\ud55c \uc18c\uaddc\ubaa8 \ubd80\ub300\uc600\ub2e4.", "paragraph_id": "6122322-0-0", "paragraph_question": "question: \ucd5c\ucd08\ub85c \ud55c\ubc18\ub3c4\uc5d0 \ub3c4\ucc29\ud55c \ubbf8\uad70\uc740 \uc5b4\ub514\uc5d0 \ucc29\ub959\ud558\uc600\ub294\uac00?, context: \ubbf8\uad6d\uc740 \ubbf8\uad6d \uc721\uad70 \uc81c24\uad70\ub2e8 45,000\uba85\uc744 \uc810\ub839\uc791\uc804\uc5d0 \ud22c\uc785\ud588\ub2e4. \ud55c\ubc18\ub3c4\uc5d0 \uc0c1\ub959\ud55c \ucd5c\ucd08\uc758 \ubbf8\uad70\uc740 1945\ub144 9\uc6d4 4\uc77c \uc11c\uc6b8 \uadfc\ucc98 \uae40\ud3ec\ube44\ud589\uc7a5\uc5d0 \ucc29\ub959\ud55c \uc18c\uaddc\ubaa8 \ubd80\ub300\uc600\ub2e4. 9\uc6d4 8\uc77c\uc5d0\ub294 \uc778\ucc9c\uc5d0 \uc81c7\ubcf4\ubcd1\uc0ac\ub2e8 \uc18c\uc18d \ubcd1\uc0ac 14\uba85\uc774 \uc0c1\ub959\ud588\uc73c\uba70, \ub2e4\uc74c \ub0a0 \ub300\uaddc\ubaa8 \uc0c1\ub959\uc774 \uc2dc\uc791\ub418\uc5c8\ub2e4. \uc800\uc790 \ud3f4 M. \uc5d0\ub4dc\uc6cc\ub4dc\uc5d0 \ub530\ub974\uba74 \ubbf8\uad6d \uc815\ubd80\ub294 \ub2f9\uc2dc \ud55c\uad6d\uc5d0 \ub300\ud574\uc11c\ub294 \uac70\uc758 \uad00\uc2ec\uc774 \uc5c6\uc5c8\uc73c\uba70, \uc774\ub97c \uc9c4\ud589\ud55c \uc0ac\ub78c\uc740 \uac70\uc758 \ub300\ubd80\ubd84\uc758 \uc138\uacc4\ub300\uc804 \uc804\ud6c4 \uacb0\uc815\uc758 \uac70\uc758 \ub9e1\uace0 \uc5f0\ud569\uad70 \uc810\ub839\ud558\uc758 \uc77c\ubcf8\uc758 \uc9c0\ud718\uad00\uc778 \ub300\uc7a5 \ub354\uae00\ub7ec\uc2a4 \ub9e5\uc544\ub354\uc600\ub2e4. \uadf8\ub7ec\ub098, \ub9e5\uc544\ub354 \uc7a5\uad70\uc740 \uc774\ubbf8 \uc77c\ubcf8\uc5d0\uc11c \uc218\ud589\ud574\uc57c \ud560 \uc791\uc804\uc5d0 \ub5a0\ubc00\ub838\uace0, \uacb0\uad6d \uc774 \uc5f4\uc57d\ud55c \ud55c\ubc18\ub3c4 \uc810\ub839\uc791\uc804\uc758 \uc9c0\ud718\uad00\uc744 \uc911\uc7a5 \uc874 \ud558\uc9c0\uc5d0\uac8c \ub9e1\uacbc\ub2e4. \ud638\uc9c0\ub294 \uc11c\uc6b8 \ubc18\ub2e4 \ud638\ud154\uc5d0 \uc0ac\ub839\ubd80\ub97c \uc138\uc6b0\uace0 \uad70\uc0ac \uc815\ubd80\ub97c \uc218\ub9bd\ud588\uc73c\uba70, \ud55c\uad6d\uc758 \uacf5\uc2dd \uc5b8\uc5b4\ub97c \uc601\uc5b4\ub85c \uc815\ud588\uc73c\uba70 \ubbf8\uad6d\uc5d0\uac8c \ud638\uc758\uc801\uc778 \ub3c5\ub9bd \uc815\ubd80\ub97c \uc218\ub9bd\ud558\uae30 \uc2dc\uc791\ud588\ub2e4."} {"question": "1909\ub144 \uc2dc\uc98c\uc5d0\uc11c \uc7ad \ucff0\uc2a4\uac00 \uc644\ud22c\ud55c \uacbd\uae30\uc758 \uc22b\uc790\ub294?", "paragraph": "1909\ub144, \uc7ad \ucff0\uc2a4\ub294 \uac1c\ub9c9 \uc774\ud6c4 \uccab 3\uc2b9\uc744 \ubaa8\ub450 \uc644\ubd09\uc2b9\uc73c\ub85c \uac70\ub450\uc5c8\ub294\ub370, \uadf8 \uc911 2\uc2b9\uc774 \uc6d4\ud130 \uc874\uc2a8\uacfc \uc5d0\ub4dc \uc6d4\uc2dc\uc640\uc758 \ub9de\ub300\uacb0\uc5d0\uc11c \ub530\ub0b8 \uac83\uc774\uc5c8\ub2e4. \uadf8\ub294 \uc544\uc9c1 \uc644\uc804\ud558\uc9c0 \ubabb\ud55c \ubab8 \uc0c1\ud0dc\uc784\uc5d0\ub3c4 \ubd88\uad6c\ud558\uace0 18\uacbd\uae30\uc5d0\uc11c \uc644\ud22c\ub97c \ud558\uba70 \ub370\ubdd4 \uc774\ud6c4 \ucc98\uc74c\uc73c\ub85c 200\uc774\ub2dd\uc744 \ub3cc\ud30c\ud588\ub2e4. \uc774\ubc88 \uc2dc\uc98c \ubaa8\ub450 30\uacbd\uae30(\uc120\ubc1c 24\uacbd\uae30)\uc5d0 \ub098\uc11c 205 \u20443\uc774\ub2dd\uc744 \ub358\uc838 12\uc2b9 11\ud328, 2.32\uc758 \ud3c9\uade0\uc790\ucc45\uc810\uc744 \uae30\ub85d\ud588\ub2e4. \ucff0\uc2a4\ub294 \uc774 \ub54c\uae4c\uc9c0 \ubaa8\ub450 35\uc2b9 35\ud328\uc758 \ub3d9\ub960\uc744 \uae30\ub85d\ud588\uace0, \uadf8\uc5d0\uac8c \uc788\uc5b4\uc11c 1910\ub144\uc740 \uc57c\uad6c \uacbd\ub825\uc744 \uc55e\uc73c\ub85c \uc5b4\ub5bb\uac8c \uc774\uc5b4\ub098\uac08 \uac83\uc778\uc9c0 \uc131\uc801\uc73c\ub85c \ubcf4\uc5ec\uc8fc\uc5b4\uc57c \ud558\ub294 \ubd84\uae30\uc810\uc774 \ub418\ub294 \ud574\uc600\ub2e4. \uadf8\ub294 1909\ub144 \uc2dc\uc98c \uc911\ubd80\ud130 \uc790\uc2e0\uc758 \uc8fc\ubb34\uae30 \ucee4\ube0c\uc640\ub294 \ub2e4\ub978 \ub5a8\uc5b4\uc9c0\ub294 \uc720\uc778\uad6c\ub97c \uac1c\ubc1c\ud558\uae30 \uc2dc\uc791\ud588\uace0, \ub2e4\uc74c\ud574 \uc2a4\ud504\ub9c1 \ud2b8\ub808\uc774\ub2dd \ub54c \uc774\ub97c \uc5f0\ub9c8\ud558\ub294 \ub3d9\uc2dc\uc5d0 \uad6c\uc18d\uc744 \uc904\uc774\uace0 \uc81c\uad6c\ub825\uc5d0 \ub354 \ub9ce\uc740 \uc2e0\uacbd\uc744 \uc4f0\ub824\uace0 \uc560\uc37c\ub2e4.", "answer": "18\uacbd\uae30", "sentence": "\uadf8\ub294 \uc544\uc9c1 \uc644\uc804\ud558\uc9c0 \ubabb\ud55c \ubab8 \uc0c1\ud0dc\uc784\uc5d0\ub3c4 \ubd88\uad6c\ud558\uace0 18\uacbd\uae30 \uc5d0\uc11c \uc644\ud22c\ub97c \ud558\uba70 \ub370\ubdd4 \uc774\ud6c4 \ucc98\uc74c\uc73c\ub85c 200\uc774\ub2dd\uc744 \ub3cc\ud30c\ud588\ub2e4.", "paragraph_sentence": "1909\ub144, \uc7ad \ucff0\uc2a4\ub294 \uac1c\ub9c9 \uc774\ud6c4 \uccab 3\uc2b9\uc744 \ubaa8\ub450 \uc644\ubd09\uc2b9\uc73c\ub85c \uac70\ub450\uc5c8\ub294\ub370, \uadf8 \uc911 2\uc2b9\uc774 \uc6d4\ud130 \uc874\uc2a8\uacfc \uc5d0\ub4dc \uc6d4\uc2dc\uc640\uc758 \ub9de\ub300\uacb0\uc5d0\uc11c \ub530\ub0b8 \uac83\uc774\uc5c8\ub2e4. \uadf8\ub294 \uc544\uc9c1 \uc644\uc804\ud558\uc9c0 \ubabb\ud55c \ubab8 \uc0c1\ud0dc\uc784\uc5d0\ub3c4 \ubd88\uad6c\ud558\uace0 18\uacbd\uae30 \uc5d0\uc11c \uc644\ud22c\ub97c \ud558\uba70 \ub370\ubdd4 \uc774\ud6c4 \ucc98\uc74c\uc73c\ub85c 200\uc774\ub2dd\uc744 \ub3cc\ud30c\ud588\ub2e4. \uc774\ubc88 \uc2dc\uc98c \ubaa8\ub450 30\uacbd\uae30(\uc120\ubc1c 24\uacbd\uae30)\uc5d0 \ub098\uc11c 205 \u20443\uc774\ub2dd\uc744 \ub358\uc838 12\uc2b9 11\ud328, 2.32\uc758 \ud3c9\uade0\uc790\ucc45\uc810\uc744 \uae30\ub85d\ud588\ub2e4. \ucff0\uc2a4\ub294 \uc774 \ub54c\uae4c\uc9c0 \ubaa8\ub450 35\uc2b9 35\ud328\uc758 \ub3d9\ub960\uc744 \uae30\ub85d\ud588\uace0, \uadf8\uc5d0\uac8c \uc788\uc5b4\uc11c 1910\ub144\uc740 \uc57c\uad6c \uacbd\ub825\uc744 \uc55e\uc73c\ub85c \uc5b4\ub5bb\uac8c \uc774\uc5b4\ub098\uac08 \uac83\uc778\uc9c0 \uc131\uc801\uc73c\ub85c \ubcf4\uc5ec\uc8fc\uc5b4\uc57c \ud558\ub294 \ubd84\uae30\uc810\uc774 \ub418\ub294 \ud574\uc600\ub2e4. \uadf8\ub294 1909\ub144 \uc2dc\uc98c \uc911\ubd80\ud130 \uc790\uc2e0\uc758 \uc8fc\ubb34\uae30 \ucee4\ube0c\uc640\ub294 \ub2e4\ub978 \ub5a8\uc5b4\uc9c0\ub294 \uc720\uc778\uad6c\ub97c \uac1c\ubc1c\ud558\uae30 \uc2dc\uc791\ud588\uace0, \ub2e4\uc74c\ud574 \uc2a4\ud504\ub9c1 \ud2b8\ub808\uc774\ub2dd \ub54c \uc774\ub97c \uc5f0\ub9c8\ud558\ub294 \ub3d9\uc2dc\uc5d0 \uad6c\uc18d\uc744 \uc904\uc774\uace0 \uc81c\uad6c\ub825\uc5d0 \ub354 \ub9ce\uc740 \uc2e0\uacbd\uc744 \uc4f0\ub824\uace0 \uc560\uc37c\ub2e4.", "paragraph_answer": "1909\ub144, \uc7ad \ucff0\uc2a4\ub294 \uac1c\ub9c9 \uc774\ud6c4 \uccab 3\uc2b9\uc744 \ubaa8\ub450 \uc644\ubd09\uc2b9\uc73c\ub85c \uac70\ub450\uc5c8\ub294\ub370, \uadf8 \uc911 2\uc2b9\uc774 \uc6d4\ud130 \uc874\uc2a8\uacfc \uc5d0\ub4dc \uc6d4\uc2dc\uc640\uc758 \ub9de\ub300\uacb0\uc5d0\uc11c \ub530\ub0b8 \uac83\uc774\uc5c8\ub2e4. \uadf8\ub294 \uc544\uc9c1 \uc644\uc804\ud558\uc9c0 \ubabb\ud55c \ubab8 \uc0c1\ud0dc\uc784\uc5d0\ub3c4 \ubd88\uad6c\ud558\uace0 18\uacbd\uae30 \uc5d0\uc11c \uc644\ud22c\ub97c \ud558\uba70 \ub370\ubdd4 \uc774\ud6c4 \ucc98\uc74c\uc73c\ub85c 200\uc774\ub2dd\uc744 \ub3cc\ud30c\ud588\ub2e4. \uc774\ubc88 \uc2dc\uc98c \ubaa8\ub450 30\uacbd\uae30(\uc120\ubc1c 24\uacbd\uae30)\uc5d0 \ub098\uc11c 205 \u20443\uc774\ub2dd\uc744 \ub358\uc838 12\uc2b9 11\ud328, 2.32\uc758 \ud3c9\uade0\uc790\ucc45\uc810\uc744 \uae30\ub85d\ud588\ub2e4. \ucff0\uc2a4\ub294 \uc774 \ub54c\uae4c\uc9c0 \ubaa8\ub450 35\uc2b9 35\ud328\uc758 \ub3d9\ub960\uc744 \uae30\ub85d\ud588\uace0, \uadf8\uc5d0\uac8c \uc788\uc5b4\uc11c 1910\ub144\uc740 \uc57c\uad6c \uacbd\ub825\uc744 \uc55e\uc73c\ub85c \uc5b4\ub5bb\uac8c \uc774\uc5b4\ub098\uac08 \uac83\uc778\uc9c0 \uc131\uc801\uc73c\ub85c \ubcf4\uc5ec\uc8fc\uc5b4\uc57c \ud558\ub294 \ubd84\uae30\uc810\uc774 \ub418\ub294 \ud574\uc600\ub2e4. \uadf8\ub294 1909\ub144 \uc2dc\uc98c \uc911\ubd80\ud130 \uc790\uc2e0\uc758 \uc8fc\ubb34\uae30 \ucee4\ube0c\uc640\ub294 \ub2e4\ub978 \ub5a8\uc5b4\uc9c0\ub294 \uc720\uc778\uad6c\ub97c \uac1c\ubc1c\ud558\uae30 \uc2dc\uc791\ud588\uace0, \ub2e4\uc74c\ud574 \uc2a4\ud504\ub9c1 \ud2b8\ub808\uc774\ub2dd \ub54c \uc774\ub97c \uc5f0\ub9c8\ud558\ub294 \ub3d9\uc2dc\uc5d0 \uad6c\uc18d\uc744 \uc904\uc774\uace0 \uc81c\uad6c\ub825\uc5d0 \ub354 \ub9ce\uc740 \uc2e0\uacbd\uc744 \uc4f0\ub824\uace0 \uc560\uc37c\ub2e4.", "sentence_answer": "\uadf8\ub294 \uc544\uc9c1 \uc644\uc804\ud558\uc9c0 \ubabb\ud55c \ubab8 \uc0c1\ud0dc\uc784\uc5d0\ub3c4 \ubd88\uad6c\ud558\uace0 18\uacbd\uae30 \uc5d0\uc11c \uc644\ud22c\ub97c \ud558\uba70 \ub370\ubdd4 \uc774\ud6c4 \ucc98\uc74c\uc73c\ub85c 200\uc774\ub2dd\uc744 \ub3cc\ud30c\ud588\ub2e4.", "paragraph_id": "6549325-3-1", "paragraph_question": "question: 1909\ub144 \uc2dc\uc98c\uc5d0\uc11c \uc7ad \ucff0\uc2a4\uac00 \uc644\ud22c\ud55c \uacbd\uae30\uc758 \uc22b\uc790\ub294?, context: 1909\ub144, \uc7ad \ucff0\uc2a4\ub294 \uac1c\ub9c9 \uc774\ud6c4 \uccab 3\uc2b9\uc744 \ubaa8\ub450 \uc644\ubd09\uc2b9\uc73c\ub85c \uac70\ub450\uc5c8\ub294\ub370, \uadf8 \uc911 2\uc2b9\uc774 \uc6d4\ud130 \uc874\uc2a8\uacfc \uc5d0\ub4dc \uc6d4\uc2dc\uc640\uc758 \ub9de\ub300\uacb0\uc5d0\uc11c \ub530\ub0b8 \uac83\uc774\uc5c8\ub2e4. \uadf8\ub294 \uc544\uc9c1 \uc644\uc804\ud558\uc9c0 \ubabb\ud55c \ubab8 \uc0c1\ud0dc\uc784\uc5d0\ub3c4 \ubd88\uad6c\ud558\uace0 18\uacbd\uae30\uc5d0\uc11c \uc644\ud22c\ub97c \ud558\uba70 \ub370\ubdd4 \uc774\ud6c4 \ucc98\uc74c\uc73c\ub85c 200\uc774\ub2dd\uc744 \ub3cc\ud30c\ud588\ub2e4. \uc774\ubc88 \uc2dc\uc98c \ubaa8\ub450 30\uacbd\uae30(\uc120\ubc1c 24\uacbd\uae30)\uc5d0 \ub098\uc11c 205 \u20443\uc774\ub2dd\uc744 \ub358\uc838 12\uc2b9 11\ud328, 2.32\uc758 \ud3c9\uade0\uc790\ucc45\uc810\uc744 \uae30\ub85d\ud588\ub2e4. \ucff0\uc2a4\ub294 \uc774 \ub54c\uae4c\uc9c0 \ubaa8\ub450 35\uc2b9 35\ud328\uc758 \ub3d9\ub960\uc744 \uae30\ub85d\ud588\uace0, \uadf8\uc5d0\uac8c \uc788\uc5b4\uc11c 1910\ub144\uc740 \uc57c\uad6c \uacbd\ub825\uc744 \uc55e\uc73c\ub85c \uc5b4\ub5bb\uac8c \uc774\uc5b4\ub098\uac08 \uac83\uc778\uc9c0 \uc131\uc801\uc73c\ub85c \ubcf4\uc5ec\uc8fc\uc5b4\uc57c \ud558\ub294 \ubd84\uae30\uc810\uc774 \ub418\ub294 \ud574\uc600\ub2e4. \uadf8\ub294 1909\ub144 \uc2dc\uc98c \uc911\ubd80\ud130 \uc790\uc2e0\uc758 \uc8fc\ubb34\uae30 \ucee4\ube0c\uc640\ub294 \ub2e4\ub978 \ub5a8\uc5b4\uc9c0\ub294 \uc720\uc778\uad6c\ub97c \uac1c\ubc1c\ud558\uae30 \uc2dc\uc791\ud588\uace0, \ub2e4\uc74c\ud574 \uc2a4\ud504\ub9c1 \ud2b8\ub808\uc774\ub2dd \ub54c \uc774\ub97c \uc5f0\ub9c8\ud558\ub294 \ub3d9\uc2dc\uc5d0 \uad6c\uc18d\uc744 \uc904\uc774\uace0 \uc81c\uad6c\ub825\uc5d0 \ub354 \ub9ce\uc740 \uc2e0\uacbd\uc744 \uc4f0\ub824\uace0 \uc560\uc37c\ub2e4."} {"question": "\ub204\uac00 \uc774\ud0c8\ub9ac\uc544\uc5d0\uc11c \ub3d9\ub85c\ub9c8 \ud669\uc81c\uc758 \ub300\ub9ac\uc5ed\ud560\uc744 \uc218\ud589\ud588\ub098?", "paragraph": "\uadf8\ub808\uace0\ub9ac\uc624 1\uc138\uc758 \ud6c4\uc784 \uad50\ud669\ub4e4\uc740 \uc774\ud0c8\ub9ac\uc544\uc5d0\uc11c \ub3d9\ub85c\ub9c8 \ud669\uc81c\ub97c \ub300\ub9ac\ud558\ub294 \ub77c\ubca4\ub098 \ucd1d\ub3c5\uc758 \uc601\ud5a5\uc744 \ub9ce\uc774 \ubc1b\uc558\ub2e4. \uadf8\ub7ec\ub098 \uc774\uc2ac\ub78c\uad50 \uc138\ub825\uc774 \uc815\ubcf5 \uc804\uc7c1\uc744 \ud1b5\ud574 \uc601\uc5ed\uc744 \uacc4\uc18d\ud574\uc11c \ud655\uc7a5\ud574\uac00\ub294 \ubc18\uba74\uc5d0 \ub3d9\ub85c\ub9c8 \uc81c\uad6d\uc740 \uac08\uc218\ub85d \uad6d\ub825\uc774 \uae30\uc6b8\uae30 \uc2dc\uc791\ud588\uc744 \ubfd0\ub9cc \uc544\ub2c8\ub77c \ub791\uace0\ubc14\ub4dc\ub974\uc871\uc758 \uce68\uc785\uc73c\ub85c\ubd80\ud130 \ub3d9\ub85c\ub9c8 \uc81c\uad6d\uc774 \uad50\ud669\uc758 \ud1a0\uc9c0\ub97c \ubcf4\ud638\ud574\uc8fc\uc9c0 \ubabb\ud558\ub294 \uc0c1\ud669\uc774 \uc774\uc5b4\uc9c0\uba74\uc11c \uad50\ud669\uc740 \ucc28\uce30 \ub3d9\ub85c\ub9c8 \ud669\uc81c\ub85c\ubd80\ud130 \uac70\ub9ac\ub97c \ub450\uae30 \uc2dc\uc791\ud588\ub2e4. \uacb0\uad6d \uad50\ud669 \uc2a4\ud14c\ud30c\ub178 2\uc138\ub294 \ucf58\uc2a4\ud0c4\ud2f0\ub204\uc2a4 5\uc138 \ud669\uc81c \ub300\uc2e0 \ud504\ub791\ud06c\uc871\uc5d0\uac8c \uc790\uc2e0\uc758 \ud1a0\uc9c0\ub97c \ubcf4\ud638\ud574 \uc904 \uac83\uc744 \uc694\uccad\ud558\uac8c \ub418\uc5c8\ub2e4. \ud504\ub791\ud06c \uad6d\uc655 \ud53c\ud540 3\uc138\ub294 \ub791\uace0\ubc14\ub974\ub4dc\uc871\uc744 \uc9c4\uc555\ud558\uace0 \uc774\ud0c8\ub9ac\uc544 \ub545\uc758 \uc77c\ubd80\ub97c \uad50\ud669\uc5d0\uac8c \uae30\uc99d\ud558\uc600\ub2e4. \uc774\ub85c\uc368 \uad50\ud669\uc740 \uc911\uc138\ubd80\ud130 \uadfc\ub300\uc5d0 \uac78\uccd0 \ub2e8\uc21c\ud55c \uad50\ud68c \uc9c0\ub3c4\uc790\uc77c \ubfd0\ub9cc \uc544\ub2c8\ub77c \uc774\ud0c8\ub9ac\uc544 \ubc18\ub3c4 \uc548\uc5d0 \uc788\ub294 \ub9c8\ub974\ucf00, \uc6c0\ube0c\ub9ac\uc544, \ub77c\uce58\uc624 \uc9c0\ubc29\uc73c\ub85c \uc774\ub8e8\uc5b4\uc9c4 \uad11\ud65c\ud55c \uc601\ud1a0\uc758 \ud1b5\uce58\uc790\uac00 \ub418\uc5c8\ub2e4. \uc774\ud0c8\ub9ac\uc544 \ubc18\ub3c4\uc758 5\ubd84\uc758 1\uc744 \ucc28\uc9c0\ud558\ub294 \uc774 \uc601\ud1a0\ub97c \uad50\ud669\ub839\uc774\ub77c\uace0 \ubd88\ub800\ub2e4. \uadf8\ub9ac\uace0 800\ub144\uc5d0\ub294 \uad50\ud669 \ub808\uc624 3\uc138\uac00 \uce74\ub864\ub8e8\uc2a4\uc5d0\uac8c \ud669\uc81c\uc758 \uad00\uc744 \uc50c\uc6cc \uc8fc\uc5b4 \uadf8\ub97c \uc11c\ubc29 \uc81c\uad6d\uc758 \ud669\uc81c\ub85c \uc784\uba85\ud558\uae30\uc5d0 \uc774\ub974\ub800\ub2e4. \uc774\ub294 962\ub144 \uc624\ud1a0 1\uc138\uac00 \uad50\ud669 \uc694\ud55c 12\uc138\uc5d0 \uc758\ud574 \uc2e0\uc131 \ub85c\ub9c8 \uc81c\uad6d\uc758 \ud669\uc81c\ub85c \ub4f1\uadf9\ud558\uba74\uc11c \ub418\ud480\uc774\ub418\uc5c8\ub2e4. \uc774\ub7ec\ud55c \uc5f0\uc720\ub85c \uc11c\ubc29\uc5d0\uc11c\ub294 \uad50\ud669\uc73c\ub85c\ubd80\ud130 \uc9c1\uc811 \ud669\uc81c\uc758 \uad00\uc744 \ubc1b\uc544 \uc368\uc57c\ub9cc \ube44\ub85c\uc18c \ud669\uc81c\ub85c \uc778\uc815\ub420 \uc218 \uc788\ub2e4\ub294 \uac83\uc774 \ud558\ub098\uc758 \uad00\ub840\ub85c \uc790\ub9ac\uc7a1\uc558\ub2e4.", "answer": "\ub77c\ubca4\ub098 \ucd1d\ub3c5", "sentence": "\uadf8\ub808\uace0\ub9ac\uc624 1\uc138\uc758 \ud6c4\uc784 \uad50\ud669\ub4e4\uc740 \uc774\ud0c8\ub9ac\uc544\uc5d0\uc11c \ub3d9\ub85c\ub9c8 \ud669\uc81c\ub97c \ub300\ub9ac\ud558\ub294 \ub77c\ubca4\ub098 \ucd1d\ub3c5 \uc758 \uc601\ud5a5\uc744 \ub9ce\uc774 \ubc1b\uc558\ub2e4.", "paragraph_sentence": " \uadf8\ub808\uace0\ub9ac\uc624 1\uc138\uc758 \ud6c4\uc784 \uad50\ud669\ub4e4\uc740 \uc774\ud0c8\ub9ac\uc544\uc5d0\uc11c \ub3d9\ub85c\ub9c8 \ud669\uc81c\ub97c \ub300\ub9ac\ud558\ub294 \ub77c\ubca4\ub098 \ucd1d\ub3c5 \uc758 \uc601\ud5a5\uc744 \ub9ce\uc774 \ubc1b\uc558\ub2e4. \uadf8\ub7ec\ub098 \uc774\uc2ac\ub78c\uad50 \uc138\ub825\uc774 \uc815\ubcf5 \uc804\uc7c1\uc744 \ud1b5\ud574 \uc601\uc5ed\uc744 \uacc4\uc18d\ud574\uc11c \ud655\uc7a5\ud574\uac00\ub294 \ubc18\uba74\uc5d0 \ub3d9\ub85c\ub9c8 \uc81c\uad6d\uc740 \uac08\uc218\ub85d \uad6d\ub825\uc774 \uae30\uc6b8\uae30 \uc2dc\uc791\ud588\uc744 \ubfd0\ub9cc \uc544\ub2c8\ub77c \ub791\uace0\ubc14\ub4dc\ub974\uc871\uc758 \uce68\uc785\uc73c\ub85c\ubd80\ud130 \ub3d9\ub85c\ub9c8 \uc81c\uad6d\uc774 \uad50\ud669\uc758 \ud1a0\uc9c0\ub97c \ubcf4\ud638\ud574\uc8fc\uc9c0 \ubabb\ud558\ub294 \uc0c1\ud669\uc774 \uc774\uc5b4\uc9c0\uba74\uc11c \uad50\ud669\uc740 \ucc28\uce30 \ub3d9\ub85c\ub9c8 \ud669\uc81c\ub85c\ubd80\ud130 \uac70\ub9ac\ub97c \ub450\uae30 \uc2dc\uc791\ud588\ub2e4. \uacb0\uad6d \uad50\ud669 \uc2a4\ud14c\ud30c\ub178 2\uc138\ub294 \ucf58\uc2a4\ud0c4\ud2f0\ub204\uc2a4 5\uc138 \ud669\uc81c \ub300\uc2e0 \ud504\ub791\ud06c\uc871\uc5d0\uac8c \uc790\uc2e0\uc758 \ud1a0\uc9c0\ub97c \ubcf4\ud638\ud574 \uc904 \uac83\uc744 \uc694\uccad\ud558\uac8c \ub418\uc5c8\ub2e4. \ud504\ub791\ud06c \uad6d\uc655 \ud53c\ud540 3\uc138\ub294 \ub791\uace0\ubc14\ub974\ub4dc\uc871\uc744 \uc9c4\uc555\ud558\uace0 \uc774\ud0c8\ub9ac\uc544 \ub545\uc758 \uc77c\ubd80\ub97c \uad50\ud669\uc5d0\uac8c \uae30\uc99d\ud558\uc600\ub2e4. \uc774\ub85c\uc368 \uad50\ud669\uc740 \uc911\uc138\ubd80\ud130 \uadfc\ub300\uc5d0 \uac78\uccd0 \ub2e8\uc21c\ud55c \uad50\ud68c \uc9c0\ub3c4\uc790\uc77c \ubfd0\ub9cc \uc544\ub2c8\ub77c \uc774\ud0c8\ub9ac\uc544 \ubc18\ub3c4 \uc548\uc5d0 \uc788\ub294 \ub9c8\ub974\ucf00, \uc6c0\ube0c\ub9ac\uc544, \ub77c\uce58\uc624 \uc9c0\ubc29\uc73c\ub85c \uc774\ub8e8\uc5b4\uc9c4 \uad11\ud65c\ud55c \uc601\ud1a0\uc758 \ud1b5\uce58\uc790\uac00 \ub418\uc5c8\ub2e4. \uc774\ud0c8\ub9ac\uc544 \ubc18\ub3c4\uc758 5\ubd84\uc758 1\uc744 \ucc28\uc9c0\ud558\ub294 \uc774 \uc601\ud1a0\ub97c \uad50\ud669\ub839\uc774\ub77c\uace0 \ubd88\ub800\ub2e4. \uadf8\ub9ac\uace0 800\ub144\uc5d0\ub294 \uad50\ud669 \ub808\uc624 3\uc138\uac00 \uce74\ub864\ub8e8\uc2a4\uc5d0\uac8c \ud669\uc81c\uc758 \uad00\uc744 \uc50c\uc6cc \uc8fc\uc5b4 \uadf8\ub97c \uc11c\ubc29 \uc81c\uad6d\uc758 \ud669\uc81c\ub85c \uc784\uba85\ud558\uae30\uc5d0 \uc774\ub974\ub800\ub2e4. \uc774\ub294 962\ub144 \uc624\ud1a0 1\uc138\uac00 \uad50\ud669 \uc694\ud55c 12\uc138\uc5d0 \uc758\ud574 \uc2e0\uc131 \ub85c\ub9c8 \uc81c\uad6d\uc758 \ud669\uc81c\ub85c \ub4f1\uadf9\ud558\uba74\uc11c \ub418\ud480\uc774\ub418\uc5c8\ub2e4. \uc774\ub7ec\ud55c \uc5f0\uc720\ub85c \uc11c\ubc29\uc5d0\uc11c\ub294 \uad50\ud669\uc73c\ub85c\ubd80\ud130 \uc9c1\uc811 \ud669\uc81c\uc758 \uad00\uc744 \ubc1b\uc544 \uc368\uc57c\ub9cc \ube44\ub85c\uc18c \ud669\uc81c\ub85c \uc778\uc815\ub420 \uc218 \uc788\ub2e4\ub294 \uac83\uc774 \ud558\ub098\uc758 \uad00\ub840\ub85c \uc790\ub9ac\uc7a1\uc558\ub2e4.", "paragraph_answer": "\uadf8\ub808\uace0\ub9ac\uc624 1\uc138\uc758 \ud6c4\uc784 \uad50\ud669\ub4e4\uc740 \uc774\ud0c8\ub9ac\uc544\uc5d0\uc11c \ub3d9\ub85c\ub9c8 \ud669\uc81c\ub97c \ub300\ub9ac\ud558\ub294 \ub77c\ubca4\ub098 \ucd1d\ub3c5 \uc758 \uc601\ud5a5\uc744 \ub9ce\uc774 \ubc1b\uc558\ub2e4. \uadf8\ub7ec\ub098 \uc774\uc2ac\ub78c\uad50 \uc138\ub825\uc774 \uc815\ubcf5 \uc804\uc7c1\uc744 \ud1b5\ud574 \uc601\uc5ed\uc744 \uacc4\uc18d\ud574\uc11c \ud655\uc7a5\ud574\uac00\ub294 \ubc18\uba74\uc5d0 \ub3d9\ub85c\ub9c8 \uc81c\uad6d\uc740 \uac08\uc218\ub85d \uad6d\ub825\uc774 \uae30\uc6b8\uae30 \uc2dc\uc791\ud588\uc744 \ubfd0\ub9cc \uc544\ub2c8\ub77c \ub791\uace0\ubc14\ub4dc\ub974\uc871\uc758 \uce68\uc785\uc73c\ub85c\ubd80\ud130 \ub3d9\ub85c\ub9c8 \uc81c\uad6d\uc774 \uad50\ud669\uc758 \ud1a0\uc9c0\ub97c \ubcf4\ud638\ud574\uc8fc\uc9c0 \ubabb\ud558\ub294 \uc0c1\ud669\uc774 \uc774\uc5b4\uc9c0\uba74\uc11c \uad50\ud669\uc740 \ucc28\uce30 \ub3d9\ub85c\ub9c8 \ud669\uc81c\ub85c\ubd80\ud130 \uac70\ub9ac\ub97c \ub450\uae30 \uc2dc\uc791\ud588\ub2e4. \uacb0\uad6d \uad50\ud669 \uc2a4\ud14c\ud30c\ub178 2\uc138\ub294 \ucf58\uc2a4\ud0c4\ud2f0\ub204\uc2a4 5\uc138 \ud669\uc81c \ub300\uc2e0 \ud504\ub791\ud06c\uc871\uc5d0\uac8c \uc790\uc2e0\uc758 \ud1a0\uc9c0\ub97c \ubcf4\ud638\ud574 \uc904 \uac83\uc744 \uc694\uccad\ud558\uac8c \ub418\uc5c8\ub2e4. \ud504\ub791\ud06c \uad6d\uc655 \ud53c\ud540 3\uc138\ub294 \ub791\uace0\ubc14\ub974\ub4dc\uc871\uc744 \uc9c4\uc555\ud558\uace0 \uc774\ud0c8\ub9ac\uc544 \ub545\uc758 \uc77c\ubd80\ub97c \uad50\ud669\uc5d0\uac8c \uae30\uc99d\ud558\uc600\ub2e4. \uc774\ub85c\uc368 \uad50\ud669\uc740 \uc911\uc138\ubd80\ud130 \uadfc\ub300\uc5d0 \uac78\uccd0 \ub2e8\uc21c\ud55c \uad50\ud68c \uc9c0\ub3c4\uc790\uc77c \ubfd0\ub9cc \uc544\ub2c8\ub77c \uc774\ud0c8\ub9ac\uc544 \ubc18\ub3c4 \uc548\uc5d0 \uc788\ub294 \ub9c8\ub974\ucf00, \uc6c0\ube0c\ub9ac\uc544, \ub77c\uce58\uc624 \uc9c0\ubc29\uc73c\ub85c \uc774\ub8e8\uc5b4\uc9c4 \uad11\ud65c\ud55c \uc601\ud1a0\uc758 \ud1b5\uce58\uc790\uac00 \ub418\uc5c8\ub2e4. \uc774\ud0c8\ub9ac\uc544 \ubc18\ub3c4\uc758 5\ubd84\uc758 1\uc744 \ucc28\uc9c0\ud558\ub294 \uc774 \uc601\ud1a0\ub97c \uad50\ud669\ub839\uc774\ub77c\uace0 \ubd88\ub800\ub2e4. \uadf8\ub9ac\uace0 800\ub144\uc5d0\ub294 \uad50\ud669 \ub808\uc624 3\uc138\uac00 \uce74\ub864\ub8e8\uc2a4\uc5d0\uac8c \ud669\uc81c\uc758 \uad00\uc744 \uc50c\uc6cc \uc8fc\uc5b4 \uadf8\ub97c \uc11c\ubc29 \uc81c\uad6d\uc758 \ud669\uc81c\ub85c \uc784\uba85\ud558\uae30\uc5d0 \uc774\ub974\ub800\ub2e4. \uc774\ub294 962\ub144 \uc624\ud1a0 1\uc138\uac00 \uad50\ud669 \uc694\ud55c 12\uc138\uc5d0 \uc758\ud574 \uc2e0\uc131 \ub85c\ub9c8 \uc81c\uad6d\uc758 \ud669\uc81c\ub85c \ub4f1\uadf9\ud558\uba74\uc11c \ub418\ud480\uc774\ub418\uc5c8\ub2e4. \uc774\ub7ec\ud55c \uc5f0\uc720\ub85c \uc11c\ubc29\uc5d0\uc11c\ub294 \uad50\ud669\uc73c\ub85c\ubd80\ud130 \uc9c1\uc811 \ud669\uc81c\uc758 \uad00\uc744 \ubc1b\uc544 \uc368\uc57c\ub9cc \ube44\ub85c\uc18c \ud669\uc81c\ub85c \uc778\uc815\ub420 \uc218 \uc788\ub2e4\ub294 \uac83\uc774 \ud558\ub098\uc758 \uad00\ub840\ub85c \uc790\ub9ac\uc7a1\uc558\ub2e4.", "sentence_answer": "\uadf8\ub808\uace0\ub9ac\uc624 1\uc138\uc758 \ud6c4\uc784 \uad50\ud669\ub4e4\uc740 \uc774\ud0c8\ub9ac\uc544\uc5d0\uc11c \ub3d9\ub85c\ub9c8 \ud669\uc81c\ub97c \ub300\ub9ac\ud558\ub294 \ub77c\ubca4\ub098 \ucd1d\ub3c5 \uc758 \uc601\ud5a5\uc744 \ub9ce\uc774 \ubc1b\uc558\ub2e4.", "paragraph_id": "6656709-5-0", "paragraph_question": "question: \ub204\uac00 \uc774\ud0c8\ub9ac\uc544\uc5d0\uc11c \ub3d9\ub85c\ub9c8 \ud669\uc81c\uc758 \ub300\ub9ac\uc5ed\ud560\uc744 \uc218\ud589\ud588\ub098?, context: \uadf8\ub808\uace0\ub9ac\uc624 1\uc138\uc758 \ud6c4\uc784 \uad50\ud669\ub4e4\uc740 \uc774\ud0c8\ub9ac\uc544\uc5d0\uc11c \ub3d9\ub85c\ub9c8 \ud669\uc81c\ub97c \ub300\ub9ac\ud558\ub294 \ub77c\ubca4\ub098 \ucd1d\ub3c5\uc758 \uc601\ud5a5\uc744 \ub9ce\uc774 \ubc1b\uc558\ub2e4. \uadf8\ub7ec\ub098 \uc774\uc2ac\ub78c\uad50 \uc138\ub825\uc774 \uc815\ubcf5 \uc804\uc7c1\uc744 \ud1b5\ud574 \uc601\uc5ed\uc744 \uacc4\uc18d\ud574\uc11c \ud655\uc7a5\ud574\uac00\ub294 \ubc18\uba74\uc5d0 \ub3d9\ub85c\ub9c8 \uc81c\uad6d\uc740 \uac08\uc218\ub85d \uad6d\ub825\uc774 \uae30\uc6b8\uae30 \uc2dc\uc791\ud588\uc744 \ubfd0\ub9cc \uc544\ub2c8\ub77c \ub791\uace0\ubc14\ub4dc\ub974\uc871\uc758 \uce68\uc785\uc73c\ub85c\ubd80\ud130 \ub3d9\ub85c\ub9c8 \uc81c\uad6d\uc774 \uad50\ud669\uc758 \ud1a0\uc9c0\ub97c \ubcf4\ud638\ud574\uc8fc\uc9c0 \ubabb\ud558\ub294 \uc0c1\ud669\uc774 \uc774\uc5b4\uc9c0\uba74\uc11c \uad50\ud669\uc740 \ucc28\uce30 \ub3d9\ub85c\ub9c8 \ud669\uc81c\ub85c\ubd80\ud130 \uac70\ub9ac\ub97c \ub450\uae30 \uc2dc\uc791\ud588\ub2e4. \uacb0\uad6d \uad50\ud669 \uc2a4\ud14c\ud30c\ub178 2\uc138\ub294 \ucf58\uc2a4\ud0c4\ud2f0\ub204\uc2a4 5\uc138 \ud669\uc81c \ub300\uc2e0 \ud504\ub791\ud06c\uc871\uc5d0\uac8c \uc790\uc2e0\uc758 \ud1a0\uc9c0\ub97c \ubcf4\ud638\ud574 \uc904 \uac83\uc744 \uc694\uccad\ud558\uac8c \ub418\uc5c8\ub2e4. \ud504\ub791\ud06c \uad6d\uc655 \ud53c\ud540 3\uc138\ub294 \ub791\uace0\ubc14\ub974\ub4dc\uc871\uc744 \uc9c4\uc555\ud558\uace0 \uc774\ud0c8\ub9ac\uc544 \ub545\uc758 \uc77c\ubd80\ub97c \uad50\ud669\uc5d0\uac8c \uae30\uc99d\ud558\uc600\ub2e4. \uc774\ub85c\uc368 \uad50\ud669\uc740 \uc911\uc138\ubd80\ud130 \uadfc\ub300\uc5d0 \uac78\uccd0 \ub2e8\uc21c\ud55c \uad50\ud68c \uc9c0\ub3c4\uc790\uc77c \ubfd0\ub9cc \uc544\ub2c8\ub77c \uc774\ud0c8\ub9ac\uc544 \ubc18\ub3c4 \uc548\uc5d0 \uc788\ub294 \ub9c8\ub974\ucf00, \uc6c0\ube0c\ub9ac\uc544, \ub77c\uce58\uc624 \uc9c0\ubc29\uc73c\ub85c \uc774\ub8e8\uc5b4\uc9c4 \uad11\ud65c\ud55c \uc601\ud1a0\uc758 \ud1b5\uce58\uc790\uac00 \ub418\uc5c8\ub2e4. \uc774\ud0c8\ub9ac\uc544 \ubc18\ub3c4\uc758 5\ubd84\uc758 1\uc744 \ucc28\uc9c0\ud558\ub294 \uc774 \uc601\ud1a0\ub97c \uad50\ud669\ub839\uc774\ub77c\uace0 \ubd88\ub800\ub2e4. \uadf8\ub9ac\uace0 800\ub144\uc5d0\ub294 \uad50\ud669 \ub808\uc624 3\uc138\uac00 \uce74\ub864\ub8e8\uc2a4\uc5d0\uac8c \ud669\uc81c\uc758 \uad00\uc744 \uc50c\uc6cc \uc8fc\uc5b4 \uadf8\ub97c \uc11c\ubc29 \uc81c\uad6d\uc758 \ud669\uc81c\ub85c \uc784\uba85\ud558\uae30\uc5d0 \uc774\ub974\ub800\ub2e4. \uc774\ub294 962\ub144 \uc624\ud1a0 1\uc138\uac00 \uad50\ud669 \uc694\ud55c 12\uc138\uc5d0 \uc758\ud574 \uc2e0\uc131 \ub85c\ub9c8 \uc81c\uad6d\uc758 \ud669\uc81c\ub85c \ub4f1\uadf9\ud558\uba74\uc11c \ub418\ud480\uc774\ub418\uc5c8\ub2e4. \uc774\ub7ec\ud55c \uc5f0\uc720\ub85c \uc11c\ubc29\uc5d0\uc11c\ub294 \uad50\ud669\uc73c\ub85c\ubd80\ud130 \uc9c1\uc811 \ud669\uc81c\uc758 \uad00\uc744 \ubc1b\uc544 \uc368\uc57c\ub9cc \ube44\ub85c\uc18c \ud669\uc81c\ub85c \uc778\uc815\ub420 \uc218 \uc788\ub2e4\ub294 \uac83\uc774 \ud558\ub098\uc758 \uad00\ub840\ub85c \uc790\ub9ac\uc7a1\uc558\ub2e4."} {"question": "1660\ub144 \uc655\ub9bd\ud559\ud68c\uac00 \uc124\ub9bd\ud558\ub294 \ub370 \ud070 \uae30\uc5ec\ub97c \ud55c \uc601\uad6d \uad6d\uc655\uc740?", "paragraph": "17\uc138\uae30 \uc804\ubc18\uc5d0 \uac78\uccd0 \uc601\uad6d\uacfc\ud559\uc740 \uae09\uc18d\ub3c4\ub85c \uc131\uc7a5\ud558\uc600\ub2e4. \uc790\uc5f0\ucca0\ud559\uc790 \ub85c\ubc84\ud2b8 \ubcf4\uc77c\uc744 \ube44\ub86f\ud558\uc5ec \uc2dd\ubb3c\ud559\uc790 \uc874 \ub808\uc774\u00b7 \ub3d9\ubb3c\ud559\uc790 \ud504\ub79c\uc2dc\uc2a4 \uc70c\ub7ec\ubc14\uc774\u00b7 \ubb3c\ub9ac\ud559\uc790 \uc544\uc774\uc791 \ub274\ud134 \uac19\uc740 \ub6f0\uc5b4\ub09c \ud559\uc790\ub4e4\uc774 \ucd9c\ud604\ud558\uc5ec \ub9ce\uc740 \uc5c5\uc801\uc744 \ub0a8\uacbc\uc73c\uba70, 1660\ub144\uc5d0\ub294 \uc601\uad6d \uad6d\uc655 \ucc30\uc2a4 2\uc138\uc758 \uc9c0\uc6d0 \ud558\uc5d0 \uacfc\ud559\uc790\ub4e4\uc758 \ud559\uc220\ub2e8\uccb4\uc778 \uc655\ub9bd\ud559\ud68c\uac00 \uc124\ub9bd\ub418\uc5c8\ub2e4. \ud2b9\ud788 \uc655\ub9bd\ud559\ud68c\ub294 1665\ub144\ubd80\ud130 \uc138\uacc4 \ucd5c\ucd08\uc758 \ud559\uc220\uc9c0 \uc911 \ud558\ub098\uc778 \u300a\ucca0\ud559\ud68c\ubcf4(Philosophical Transactions)\u300b\ub97c \ubc1c\ud589\ud568\uc73c\ub85c\uc368 \ud559\uc790\ub4e4\uc774 \uccb4\uacc4\uc801\uc73c\ub85c \uad50\ub958\ud560 \uc218 \uc788\ub3c4\ub85d \ub3c4\uc654\ub2e4. \uacfc\ud559\ubc1c\uc804\uc774 \ub2f9\uc2dc \uc601\uad6d\uc758 \uc0ac\ud68c\uc801 \ubc30\uacbd\uc758 \uc601\ud5a5\uc744 \ud06c\uac8c \ubc1b\uc558\uc73c\ub9ac\ub77c \uc0dd\uac01\ud55c \uba38\ud134\uc740 \uc774\ub97c \uc8fc\uc81c\ub85c \ud559\uc704\uc5f0\uad6c\ub85c \uc9c4\ud589\ud558\uc600\uc73c\uba70, 1938\ub144\uc5d0\ub294 \ub17c\ubb38 \u300a17\uc138\uae30 \uc601\uad6d\uc758 \uacfc\ud559, \uae30\uc220\uacfc \uc0ac\ud68c Science, Technology and Society in Seventeenth-Century England\u300b\uc744 \ubc1c\ud45c\ud55c\ub2e4.", "answer": "\ucc30\uc2a4 2\uc138", "sentence": "\uc790\uc5f0\ucca0\ud559\uc790 \ub85c\ubc84\ud2b8 \ubcf4\uc77c\uc744 \ube44\ub86f\ud558\uc5ec \uc2dd\ubb3c\ud559\uc790 \uc874 \ub808\uc774\u00b7 \ub3d9\ubb3c\ud559\uc790 \ud504\ub79c\uc2dc\uc2a4 \uc70c\ub7ec\ubc14\uc774\u00b7 \ubb3c\ub9ac\ud559\uc790 \uc544\uc774\uc791 \ub274\ud134 \uac19\uc740 \ub6f0\uc5b4\ub09c \ud559\uc790\ub4e4\uc774 \ucd9c\ud604\ud558\uc5ec \ub9ce\uc740 \uc5c5\uc801\uc744 \ub0a8\uacbc\uc73c\uba70, 1660\ub144\uc5d0\ub294 \uc601\uad6d \uad6d\uc655 \ucc30\uc2a4 2\uc138 \uc758 \uc9c0\uc6d0 \ud558\uc5d0 \uacfc\ud559\uc790\ub4e4\uc758 \ud559\uc220\ub2e8\uccb4\uc778 \uc655\ub9bd\ud559\ud68c\uac00 \uc124\ub9bd\ub418\uc5c8\ub2e4.", "paragraph_sentence": "17\uc138\uae30 \uc804\ubc18\uc5d0 \uac78\uccd0 \uc601\uad6d\uacfc\ud559\uc740 \uae09\uc18d\ub3c4\ub85c \uc131\uc7a5\ud558\uc600\ub2e4. \uc790\uc5f0\ucca0\ud559\uc790 \ub85c\ubc84\ud2b8 \ubcf4\uc77c\uc744 \ube44\ub86f\ud558\uc5ec \uc2dd\ubb3c\ud559\uc790 \uc874 \ub808\uc774\u00b7 \ub3d9\ubb3c\ud559\uc790 \ud504\ub79c\uc2dc\uc2a4 \uc70c\ub7ec\ubc14\uc774\u00b7 \ubb3c\ub9ac\ud559\uc790 \uc544\uc774\uc791 \ub274\ud134 \uac19\uc740 \ub6f0\uc5b4\ub09c \ud559\uc790\ub4e4\uc774 \ucd9c\ud604\ud558\uc5ec \ub9ce\uc740 \uc5c5\uc801\uc744 \ub0a8\uacbc\uc73c\uba70, 1660\ub144\uc5d0\ub294 \uc601\uad6d \uad6d\uc655 \ucc30\uc2a4 2\uc138 \uc758 \uc9c0\uc6d0 \ud558\uc5d0 \uacfc\ud559\uc790\ub4e4\uc758 \ud559\uc220\ub2e8\uccb4\uc778 \uc655\ub9bd\ud559\ud68c\uac00 \uc124\ub9bd\ub418\uc5c8\ub2e4. \ud2b9\ud788 \uc655\ub9bd\ud559\ud68c\ub294 1665\ub144\ubd80\ud130 \uc138\uacc4 \ucd5c\ucd08\uc758 \ud559\uc220\uc9c0 \uc911 \ud558\ub098\uc778 \u300a\ucca0\ud559\ud68c\ubcf4(Philosophical Transactions)\u300b\ub97c \ubc1c\ud589\ud568\uc73c\ub85c\uc368 \ud559\uc790\ub4e4\uc774 \uccb4\uacc4\uc801\uc73c\ub85c \uad50\ub958\ud560 \uc218 \uc788\ub3c4\ub85d \ub3c4\uc654\ub2e4. \uacfc\ud559\ubc1c\uc804\uc774 \ub2f9\uc2dc \uc601\uad6d\uc758 \uc0ac\ud68c\uc801 \ubc30\uacbd\uc758 \uc601\ud5a5\uc744 \ud06c\uac8c \ubc1b\uc558\uc73c\ub9ac\ub77c \uc0dd\uac01\ud55c \uba38\ud134\uc740 \uc774\ub97c \uc8fc\uc81c\ub85c \ud559\uc704\uc5f0\uad6c\ub85c \uc9c4\ud589\ud558\uc600\uc73c\uba70, 1938\ub144\uc5d0\ub294 \ub17c\ubb38 \u300a17\uc138\uae30 \uc601\uad6d\uc758 \uacfc\ud559, \uae30\uc220\uacfc \uc0ac\ud68c Science, Technology and Society in Seventeenth-Century England\u300b\uc744 \ubc1c\ud45c\ud55c\ub2e4.", "paragraph_answer": "17\uc138\uae30 \uc804\ubc18\uc5d0 \uac78\uccd0 \uc601\uad6d\uacfc\ud559\uc740 \uae09\uc18d\ub3c4\ub85c \uc131\uc7a5\ud558\uc600\ub2e4. \uc790\uc5f0\ucca0\ud559\uc790 \ub85c\ubc84\ud2b8 \ubcf4\uc77c\uc744 \ube44\ub86f\ud558\uc5ec \uc2dd\ubb3c\ud559\uc790 \uc874 \ub808\uc774\u00b7 \ub3d9\ubb3c\ud559\uc790 \ud504\ub79c\uc2dc\uc2a4 \uc70c\ub7ec\ubc14\uc774\u00b7 \ubb3c\ub9ac\ud559\uc790 \uc544\uc774\uc791 \ub274\ud134 \uac19\uc740 \ub6f0\uc5b4\ub09c \ud559\uc790\ub4e4\uc774 \ucd9c\ud604\ud558\uc5ec \ub9ce\uc740 \uc5c5\uc801\uc744 \ub0a8\uacbc\uc73c\uba70, 1660\ub144\uc5d0\ub294 \uc601\uad6d \uad6d\uc655 \ucc30\uc2a4 2\uc138 \uc758 \uc9c0\uc6d0 \ud558\uc5d0 \uacfc\ud559\uc790\ub4e4\uc758 \ud559\uc220\ub2e8\uccb4\uc778 \uc655\ub9bd\ud559\ud68c\uac00 \uc124\ub9bd\ub418\uc5c8\ub2e4. \ud2b9\ud788 \uc655\ub9bd\ud559\ud68c\ub294 1665\ub144\ubd80\ud130 \uc138\uacc4 \ucd5c\ucd08\uc758 \ud559\uc220\uc9c0 \uc911 \ud558\ub098\uc778 \u300a\ucca0\ud559\ud68c\ubcf4(Philosophical Transactions)\u300b\ub97c \ubc1c\ud589\ud568\uc73c\ub85c\uc368 \ud559\uc790\ub4e4\uc774 \uccb4\uacc4\uc801\uc73c\ub85c \uad50\ub958\ud560 \uc218 \uc788\ub3c4\ub85d \ub3c4\uc654\ub2e4. \uacfc\ud559\ubc1c\uc804\uc774 \ub2f9\uc2dc \uc601\uad6d\uc758 \uc0ac\ud68c\uc801 \ubc30\uacbd\uc758 \uc601\ud5a5\uc744 \ud06c\uac8c \ubc1b\uc558\uc73c\ub9ac\ub77c \uc0dd\uac01\ud55c \uba38\ud134\uc740 \uc774\ub97c \uc8fc\uc81c\ub85c \ud559\uc704\uc5f0\uad6c\ub85c \uc9c4\ud589\ud558\uc600\uc73c\uba70, 1938\ub144\uc5d0\ub294 \ub17c\ubb38 \u300a17\uc138\uae30 \uc601\uad6d\uc758 \uacfc\ud559, \uae30\uc220\uacfc \uc0ac\ud68c Science, Technology and Society in Seventeenth-Century England\u300b\uc744 \ubc1c\ud45c\ud55c\ub2e4.", "sentence_answer": "\uc790\uc5f0\ucca0\ud559\uc790 \ub85c\ubc84\ud2b8 \ubcf4\uc77c\uc744 \ube44\ub86f\ud558\uc5ec \uc2dd\ubb3c\ud559\uc790 \uc874 \ub808\uc774\u00b7 \ub3d9\ubb3c\ud559\uc790 \ud504\ub79c\uc2dc\uc2a4 \uc70c\ub7ec\ubc14\uc774\u00b7 \ubb3c\ub9ac\ud559\uc790 \uc544\uc774\uc791 \ub274\ud134 \uac19\uc740 \ub6f0\uc5b4\ub09c \ud559\uc790\ub4e4\uc774 \ucd9c\ud604\ud558\uc5ec \ub9ce\uc740 \uc5c5\uc801\uc744 \ub0a8\uacbc\uc73c\uba70, 1660\ub144\uc5d0\ub294 \uc601\uad6d \uad6d\uc655 \ucc30\uc2a4 2\uc138 \uc758 \uc9c0\uc6d0 \ud558\uc5d0 \uacfc\ud559\uc790\ub4e4\uc758 \ud559\uc220\ub2e8\uccb4\uc778 \uc655\ub9bd\ud559\ud68c\uac00 \uc124\ub9bd\ub418\uc5c8\ub2e4.", "paragraph_id": "6489931-4-0", "paragraph_question": "question: 1660\ub144 \uc655\ub9bd\ud559\ud68c\uac00 \uc124\ub9bd\ud558\ub294 \ub370 \ud070 \uae30\uc5ec\ub97c \ud55c \uc601\uad6d \uad6d\uc655\uc740?, context: 17\uc138\uae30 \uc804\ubc18\uc5d0 \uac78\uccd0 \uc601\uad6d\uacfc\ud559\uc740 \uae09\uc18d\ub3c4\ub85c \uc131\uc7a5\ud558\uc600\ub2e4. \uc790\uc5f0\ucca0\ud559\uc790 \ub85c\ubc84\ud2b8 \ubcf4\uc77c\uc744 \ube44\ub86f\ud558\uc5ec \uc2dd\ubb3c\ud559\uc790 \uc874 \ub808\uc774\u00b7 \ub3d9\ubb3c\ud559\uc790 \ud504\ub79c\uc2dc\uc2a4 \uc70c\ub7ec\ubc14\uc774\u00b7 \ubb3c\ub9ac\ud559\uc790 \uc544\uc774\uc791 \ub274\ud134 \uac19\uc740 \ub6f0\uc5b4\ub09c \ud559\uc790\ub4e4\uc774 \ucd9c\ud604\ud558\uc5ec \ub9ce\uc740 \uc5c5\uc801\uc744 \ub0a8\uacbc\uc73c\uba70, 1660\ub144\uc5d0\ub294 \uc601\uad6d \uad6d\uc655 \ucc30\uc2a4 2\uc138\uc758 \uc9c0\uc6d0 \ud558\uc5d0 \uacfc\ud559\uc790\ub4e4\uc758 \ud559\uc220\ub2e8\uccb4\uc778 \uc655\ub9bd\ud559\ud68c\uac00 \uc124\ub9bd\ub418\uc5c8\ub2e4. \ud2b9\ud788 \uc655\ub9bd\ud559\ud68c\ub294 1665\ub144\ubd80\ud130 \uc138\uacc4 \ucd5c\ucd08\uc758 \ud559\uc220\uc9c0 \uc911 \ud558\ub098\uc778 \u300a\ucca0\ud559\ud68c\ubcf4(Philosophical Transactions)\u300b\ub97c \ubc1c\ud589\ud568\uc73c\ub85c\uc368 \ud559\uc790\ub4e4\uc774 \uccb4\uacc4\uc801\uc73c\ub85c \uad50\ub958\ud560 \uc218 \uc788\ub3c4\ub85d \ub3c4\uc654\ub2e4. \uacfc\ud559\ubc1c\uc804\uc774 \ub2f9\uc2dc \uc601\uad6d\uc758 \uc0ac\ud68c\uc801 \ubc30\uacbd\uc758 \uc601\ud5a5\uc744 \ud06c\uac8c \ubc1b\uc558\uc73c\ub9ac\ub77c \uc0dd\uac01\ud55c \uba38\ud134\uc740 \uc774\ub97c \uc8fc\uc81c\ub85c \ud559\uc704\uc5f0\uad6c\ub85c \uc9c4\ud589\ud558\uc600\uc73c\uba70, 1938\ub144\uc5d0\ub294 \ub17c\ubb38 \u300a17\uc138\uae30 \uc601\uad6d\uc758 \uacfc\ud559, \uae30\uc220\uacfc \uc0ac\ud68c Science, Technology and Society in Seventeenth-Century England\u300b\uc744 \ubc1c\ud45c\ud55c\ub2e4."} {"question": "\uc624\ubc84\uc6cc\uce58 \uac1c\ubc1c\ub2e8\uacc4\uc5d0\uc11c \uc911\uc694\uc2dc \ud588\ub358 \ubaa9\ud45c\ub294?", "paragraph": "\u300a\uc624\ubc84\uc6cc\uce58\u300b\ub294 \uc0c9\ub9f9\uc744 \uc704\ud55c \uc811\uadfc\uc131 \uc124\uc815\uc744 \ud3ec\ud568, \ud3ed\ub113\uc740 \uc0ac\uc6a9\uc790\ub97c \ud655\ubcf4\ud560 \uc218 \uc788\ub294 \uc5ec\uc12f \uac00\uc9c0 \ud2b9\uc9d5\uc744 \ud568\uaed8 \uac1c\ubc1c\ud558\uc600\ub2e4. \uac1c\ubc1c \ub2e8\uacc4\uc5d0\uc11c \uc911\uc694\ud588\ub358 \ubaa9\ud45c \ud558\ub098\ub294 \ud50c\ub808\uc774\uc5b4\ub97c \uc704\ud55c '\uc804\ud22c\uc758 \uba85\ub8cc\uc131'\uc73c\ub85c, \ud50c\ub808\uc774\uc5b4\uac00 \uc0c8\ub85c\uc6b4 \uc9c0\uc5ed\uc73c\ub85c \uc774\ub3d9\ud558\uba74 \uc801 \uce90\ub9ad\ud130\uac00 \ub69c\ub837\ud558\uac8c \ubcf4\uc774\ub3c4\ub85d \ud588\ub2e4. \uc774\uac83\uc740 \ud50c\ub808\uc774\uc5b4\uac00 \uc804\uc7a5\uc5d0\uc11c \uc0ac\uc6a9\ud558\ub294 \uc0c9\uc870\uc640 \ucc44\ub3c4 \uc218\uc900\uc758 \ub300\uc870, \uba85\ubc31\ud558\uac8c \ub2e4\ub978 \uc724\uacfd\uc744 \uac16\uac8c \ub9cc\ub4e0 \uce90\ub9ad\ud130\ub4e4\ub85c \ud50c\ub808\uc774\uc5b4\uac00 \uba3c \uac70\ub9ac\uc5d0\uc11c \uc601\uc6c5\uc774 \uc544\uad70\uc778\uc9c0 \uc801\uad70\uc778\uc9c0 \uad6c\ubcc4\ud560 \uc218 \uc788\ub3c4\ub85d \ud568\uc73c\ub85c\uc368 \uac00\ub2a5\ud558\uc600\ub2e4. \ud300\uc740 \uac1c\ubc1c\ub3d9\uc548 \uacbd\uae30 \uc911\uac04\uc5d0 \ud50c\ub808\uc774\uc5b4\uac00 \uc601\uc6c5\uc744 \ubc14\uafc0 \uc218 \uc788\ub3c4\ub85d \ud558\ub294 \uac83\uc774 \uac8c\uc784\ud50c\ub808\uc774\uc5d0 \uc911\uc694\ud55c \uc5ed\ud560\uc744 \ud568\uc744 \uc54c\uc544\ub0c8\ub2e4. \u300a\uc624\ubc84\uc6cc\uce58\u300b\ub294 \ucc98\uc74c\uc5d0 \uc18c\uc561 \uacb0\uc81c\ub098 \ub2e4\uc6b4\ub85c\ub4dc \ucf58\ud150\uce20 \uad6c\uc785\uc774 \ud3ec\ud568\ub41c \ubd80\ubd84 \uc720\ub8cc\ud654 \uac8c\uc784 \ud615\ud0dc\ub85c \ucd9c\uc2dc\ub97c \uacc4\ud68d\ud588\uc73c\ub098, \uc774\ub7ec\ud55c \ubc1c\uacac\uc73c\ub85c \uc778\ud574 \ub2e8\ub3c5 \uad6c\ub9e4\ud615 \uac8c\uc784\uc73c\ub85c \ub178\uc120\uc744 \ubcc0\uacbd\ud558\uc600\ub2e4. \ucf08\ub7ec\ub294 \ud300\uc774 \ud50c\ub808\uc774\uc5b4\uac00 \uc0c1\ud669\uc5d0 \ub530\ub77c \uc5b4\ub290 \uc601\uc6c5\uc73c\ub85c\ub4e0 \uad50\uccb4\ud560 \uc218 \uc788\uae30\ub97c \uc6d0\ud588\uace0, \ud50c\ub808\uc774\uc5b4\ub4e4\uc774 \ud300\uc5d0\uc11c \uc544\ubb34\ub3c4 \uc601\uc6c5\uc744 \uad6c\uc785\ud558\uc9c0 \uc54a\ub294\ub2e4\uba74 \uc18c\uc561 \uacb0\uc81c\ub098 \ub2e4\uc6b4\ub85c\ub4dc \ucf58\ud150\uce20\uac00 \ub2a5\ub825\uc744 \uc81c\ud55c\ud560 \uc218 \uc788\ub294 \uac83\uc5d0 \uac00\uae4c\uc6cc\uc9c4\ub2e4\uace0 \ub9d0\ud558\uc600\ub2e4. \ucf08\ub7ec\ub294 \ub610\ud55c \uc18c\uc561 \uacb0\uc81c \ubc0f \ub2e4\uc6b4\ub85c\ub4dc \ucf58\ud150\uce20\ub85c \uc778\ud574 \uac8c\uc774\uba38\uac00 \ubaa8\ub4e0 \ud50c\ub808\uc774\uc5b4 \ub300\uc2e0\uc5d0 \ub611\uac19\uc740 \ucee8\ud150\uce20\ub97c \uac00\uc9c0\uace0 \uc788\ub294 \uce5c\uad6c\uc640\ub9cc \ud50c\ub808\uc774\ud558\uac8c \ub418\uc5b4 \ud50c\ub808\uc774\uc5b4 \uacf5\ub3d9\uccb4\ub97c \ud574\uccb4\uc2dc\ud0ac \uc218 \uc788\ub2e4\uace0 \uc5b8\uae09\ud558\uc600\ub2e4. \uac1c\ubc1c\uc758 \ub354 \ub192\uc740 \ubaa9\ud45c\ub294 \ub2e4\ub978 \uacbd\uc7c1 \uac8c\uc784 \ud658\uacbd\uc5d0\uc11c \uc885\uc885 \uc77c\uc5b4\ub098\ub294 \ubd80\uc815\uc801 \uc131\ud5a5\uc744 \ud53c\ud558\uace0, \uc774\uc57c\uae30\uac00 \uae0d\uc815\uc801\uc778 \uba54\uc2dc\uc9c0\ub97c \uc8fc\ub3c4\ub85d \ud558\ub294 \uc9c4\uc804\uc5d0 \ub367\ubd99\uc5ec \ubd80\uc815\uc801\uc778 \uc601\ud5a5\uc744 \uc900\ub2e4\uace0 \ub290\ub07c\ub294 \uc694\uc18c\ub97c \uc5c6\uc560\uae30 \uc704\ud574 \uac8c\uc784\ud50c\ub808\uc774 \ub514\uc790\uc778\uc5d0\uc11c \uba85\ud655\ud55c \uc120\ud0dd\uc744 \ud558\ub294 \uac83\uc774\uc5c8\ub2e4. \uc774\ub7ec\ud55c \uc120\ud0dd \uc911 \ud558\ub098\ub294 \ub2e4\uc591\ud55c \ud1b5\uacc4 \uc694\uc57d\uc5d0\uc11c \uc8fd\uc784/\uc8fd\uc74c(kill/death) \ud45c\uc2dc\ub97c \uc5c6\uc560\ub294 \uac83\uc774\uc5c8\uc73c\uba70, \uce90\ud50c\ub7f0\uc740 \"\uc5b4\ub5a4 \uc601\uc6c5\ub4e4\uc740 \ud65c\uc57d\uc744 \uc704\ud574\uc11c \uc8fd\uc784\uc744 \ud544\uc694\ub85c \ud558\uc9c0 \uc54a\ub294\ub2e4\"\ub77c\uace0 \uc5b8\uae09\ud558\uc600\ub2e4. \uce5c\uc120\uc801\uc778 \ud50c\ub808\uc774 \ud658\uacbd\uc744 \uc99d\uc9c4\uc2dc\ud0a4\uae30 \uc704\ud574, \ube14\ub9ac\uc790\ub4dc\ub294 '\ud0c8\uc8fc'(\uacbd\uae30\uac00 \ub05d\ub098\uae30 \uc804\uc5d0 \uc77c\ubd80\ub7ec \ub098\uac00\ub294 \ud589\uc704)\ub97c \ud558\ub294 \ud50c\ub808\uc774\uc5b4\uc5d0\uac8c \uacbd\uae30 \ud6c4 \uacbd\ud5d8\uce58\ub97c \uac10\uc18c\uc2dc\ud0a4\uace0, \ub354 \uc2ec\ud560 \uacbd\uc6b0 \ud50c\ub808\uc774\ub97c \uc601\uad6c\ud788 \uae08\uc9c0\ud558\ub294 \ubd88\uc774\uc775\uc744 \ubd80\uc5ec\ud55c\ub2e4. \ube14\ub9ac\uc790\ub4dc\ub294 2016\ub144 7\uc6d4 \uc870\uc900\uc120 \ubcf4\uc815 \ud504\ub85c\uadf8\ub7a8(\uc5d0\uc784\ud575)\uc744 \ube44\ub86f\ud55c \uc81c3\uc790 \uc678\ubd80 \ubcf4\uc815 \ud504\ub85c\uadf8\ub7a8\uc744 \uc0ac\uc6a9\ud558\ub294 \ud50c\ub808\uc774\uc5b4\ub4e4\uc5d0\uac8c \ub300\uaddc\ubaa8 \uc81c\uc7ac\ub97c \uac00\ud558\uc600\ub2e4. \u300a\uc624\ubc84\uc6cc\uce58\u300b \ucd9c\uc2dc \uc774\ud6c4, \ube14\ub9ac\uc790\ub4dc\ub294 \ud50c\ub808\uc774\uc5b4\uc5d0\uac8c \uac8c\uc784\uc5d0\uc11c \ud5c8\uc6a9\ub418\uc9c0 \uc54a\ub294 \uae30\ub2a5\uc744 \uc81c\uacf5\ud558\ub294 \uc6cc\uce58\uc624\ubc84 \ud0c0\uc77c\ub7f0\ud2b8\ub97c \uac1c\ubc1c\ud55c \ubcf4\uc2a4\ub79c\ub4dc \ud575\uc2a4\uc5d0 \ubc95\uc801 \uace0\uc18c\ub97c \ucde8\ud558\uc600\ub2e4.", "answer": "\uc804\ud22c\uc758 \uba85\ub8cc\uc131", "sentence": "\uac1c\ubc1c \ub2e8\uacc4\uc5d0\uc11c \uc911\uc694\ud588\ub358 \ubaa9\ud45c \ud558\ub098\ub294 \ud50c\ub808\uc774\uc5b4\ub97c \uc704\ud55c ' \uc804\ud22c\uc758 \uba85\ub8cc\uc131 '\uc73c\ub85c, \ud50c\ub808\uc774\uc5b4\uac00 \uc0c8\ub85c\uc6b4 \uc9c0\uc5ed\uc73c\ub85c \uc774\ub3d9\ud558\uba74 \uc801 \uce90\ub9ad\ud130\uac00 \ub69c\ub837\ud558\uac8c \ubcf4\uc774\ub3c4\ub85d \ud588\ub2e4.", "paragraph_sentence": "\u300a\uc624\ubc84\uc6cc\uce58\u300b\ub294 \uc0c9\ub9f9\uc744 \uc704\ud55c \uc811\uadfc\uc131 \uc124\uc815\uc744 \ud3ec\ud568, \ud3ed\ub113\uc740 \uc0ac\uc6a9\uc790\ub97c \ud655\ubcf4\ud560 \uc218 \uc788\ub294 \uc5ec\uc12f \uac00\uc9c0 \ud2b9\uc9d5\uc744 \ud568\uaed8 \uac1c\ubc1c\ud558\uc600\ub2e4. \uac1c\ubc1c \ub2e8\uacc4\uc5d0\uc11c \uc911\uc694\ud588\ub358 \ubaa9\ud45c \ud558\ub098\ub294 \ud50c\ub808\uc774\uc5b4\ub97c \uc704\ud55c ' \uc804\ud22c\uc758 \uba85\ub8cc\uc131 '\uc73c\ub85c, \ud50c\ub808\uc774\uc5b4\uac00 \uc0c8\ub85c\uc6b4 \uc9c0\uc5ed\uc73c\ub85c \uc774\ub3d9\ud558\uba74 \uc801 \uce90\ub9ad\ud130\uac00 \ub69c\ub837\ud558\uac8c \ubcf4\uc774\ub3c4\ub85d \ud588\ub2e4. \uc774\uac83\uc740 \ud50c\ub808\uc774\uc5b4\uac00 \uc804\uc7a5\uc5d0\uc11c \uc0ac\uc6a9\ud558\ub294 \uc0c9\uc870\uc640 \ucc44\ub3c4 \uc218\uc900\uc758 \ub300\uc870, \uba85\ubc31\ud558\uac8c \ub2e4\ub978 \uc724\uacfd\uc744 \uac16\uac8c \ub9cc\ub4e0 \uce90\ub9ad\ud130\ub4e4\ub85c \ud50c\ub808\uc774\uc5b4\uac00 \uba3c \uac70\ub9ac\uc5d0\uc11c \uc601\uc6c5\uc774 \uc544\uad70\uc778\uc9c0 \uc801\uad70\uc778\uc9c0 \uad6c\ubcc4\ud560 \uc218 \uc788\ub3c4\ub85d \ud568\uc73c\ub85c\uc368 \uac00\ub2a5\ud558\uc600\ub2e4. \ud300\uc740 \uac1c\ubc1c\ub3d9\uc548 \uacbd\uae30 \uc911\uac04\uc5d0 \ud50c\ub808\uc774\uc5b4\uac00 \uc601\uc6c5\uc744 \ubc14\uafc0 \uc218 \uc788\ub3c4\ub85d \ud558\ub294 \uac83\uc774 \uac8c\uc784\ud50c\ub808\uc774\uc5d0 \uc911\uc694\ud55c \uc5ed\ud560\uc744 \ud568\uc744 \uc54c\uc544\ub0c8\ub2e4. \u300a\uc624\ubc84\uc6cc\uce58\u300b\ub294 \ucc98\uc74c\uc5d0 \uc18c\uc561 \uacb0\uc81c\ub098 \ub2e4\uc6b4\ub85c\ub4dc \ucf58\ud150\uce20 \uad6c\uc785\uc774 \ud3ec\ud568\ub41c \ubd80\ubd84 \uc720\ub8cc\ud654 \uac8c\uc784 \ud615\ud0dc\ub85c \ucd9c\uc2dc\ub97c \uacc4\ud68d\ud588\uc73c\ub098, \uc774\ub7ec\ud55c \ubc1c\uacac\uc73c\ub85c \uc778\ud574 \ub2e8\ub3c5 \uad6c\ub9e4\ud615 \uac8c\uc784\uc73c\ub85c \ub178\uc120\uc744 \ubcc0\uacbd\ud558\uc600\ub2e4. \ucf08\ub7ec\ub294 \ud300\uc774 \ud50c\ub808\uc774\uc5b4\uac00 \uc0c1\ud669\uc5d0 \ub530\ub77c \uc5b4\ub290 \uc601\uc6c5\uc73c\ub85c\ub4e0 \uad50\uccb4\ud560 \uc218 \uc788\uae30\ub97c \uc6d0\ud588\uace0, \ud50c\ub808\uc774\uc5b4\ub4e4\uc774 \ud300\uc5d0\uc11c \uc544\ubb34\ub3c4 \uc601\uc6c5\uc744 \uad6c\uc785\ud558\uc9c0 \uc54a\ub294\ub2e4\uba74 \uc18c\uc561 \uacb0\uc81c\ub098 \ub2e4\uc6b4\ub85c\ub4dc \ucf58\ud150\uce20\uac00 \ub2a5\ub825\uc744 \uc81c\ud55c\ud560 \uc218 \uc788\ub294 \uac83\uc5d0 \uac00\uae4c\uc6cc\uc9c4\ub2e4\uace0 \ub9d0\ud558\uc600\ub2e4. \ucf08\ub7ec\ub294 \ub610\ud55c \uc18c\uc561 \uacb0\uc81c \ubc0f \ub2e4\uc6b4\ub85c\ub4dc \ucf58\ud150\uce20\ub85c \uc778\ud574 \uac8c\uc774\uba38\uac00 \ubaa8\ub4e0 \ud50c\ub808\uc774\uc5b4 \ub300\uc2e0\uc5d0 \ub611\uac19\uc740 \ucee8\ud150\uce20\ub97c \uac00\uc9c0\uace0 \uc788\ub294 \uce5c\uad6c\uc640\ub9cc \ud50c\ub808\uc774\ud558\uac8c \ub418\uc5b4 \ud50c\ub808\uc774\uc5b4 \uacf5\ub3d9\uccb4\ub97c \ud574\uccb4\uc2dc\ud0ac \uc218 \uc788\ub2e4\uace0 \uc5b8\uae09\ud558\uc600\ub2e4. \uac1c\ubc1c\uc758 \ub354 \ub192\uc740 \ubaa9\ud45c\ub294 \ub2e4\ub978 \uacbd\uc7c1 \uac8c\uc784 \ud658\uacbd\uc5d0\uc11c \uc885\uc885 \uc77c\uc5b4\ub098\ub294 \ubd80\uc815\uc801 \uc131\ud5a5\uc744 \ud53c\ud558\uace0, \uc774\uc57c\uae30\uac00 \uae0d\uc815\uc801\uc778 \uba54\uc2dc\uc9c0\ub97c \uc8fc\ub3c4\ub85d \ud558\ub294 \uc9c4\uc804\uc5d0 \ub367\ubd99\uc5ec \ubd80\uc815\uc801\uc778 \uc601\ud5a5\uc744 \uc900\ub2e4\uace0 \ub290\ub07c\ub294 \uc694\uc18c\ub97c \uc5c6\uc560\uae30 \uc704\ud574 \uac8c\uc784\ud50c\ub808\uc774 \ub514\uc790\uc778\uc5d0\uc11c \uba85\ud655\ud55c \uc120\ud0dd\uc744 \ud558\ub294 \uac83\uc774\uc5c8\ub2e4. \uc774\ub7ec\ud55c \uc120\ud0dd \uc911 \ud558\ub098\ub294 \ub2e4\uc591\ud55c \ud1b5\uacc4 \uc694\uc57d\uc5d0\uc11c \uc8fd\uc784/\uc8fd\uc74c(kill/death) \ud45c\uc2dc\ub97c \uc5c6\uc560\ub294 \uac83\uc774\uc5c8\uc73c\uba70, \uce90\ud50c\ub7f0\uc740 \"\uc5b4\ub5a4 \uc601\uc6c5\ub4e4\uc740 \ud65c\uc57d\uc744 \uc704\ud574\uc11c \uc8fd\uc784\uc744 \ud544\uc694\ub85c \ud558\uc9c0 \uc54a\ub294\ub2e4\"\ub77c\uace0 \uc5b8\uae09\ud558\uc600\ub2e4. \uce5c\uc120\uc801\uc778 \ud50c\ub808\uc774 \ud658\uacbd\uc744 \uc99d\uc9c4\uc2dc\ud0a4\uae30 \uc704\ud574, \ube14\ub9ac\uc790\ub4dc\ub294 '\ud0c8\uc8fc'(\uacbd\uae30\uac00 \ub05d\ub098\uae30 \uc804\uc5d0 \uc77c\ubd80\ub7ec \ub098\uac00\ub294 \ud589\uc704)\ub97c \ud558\ub294 \ud50c\ub808\uc774\uc5b4\uc5d0\uac8c \uacbd\uae30 \ud6c4 \uacbd\ud5d8\uce58\ub97c \uac10\uc18c\uc2dc\ud0a4\uace0, \ub354 \uc2ec\ud560 \uacbd\uc6b0 \ud50c\ub808\uc774\ub97c \uc601\uad6c\ud788 \uae08\uc9c0\ud558\ub294 \ubd88\uc774\uc775\uc744 \ubd80\uc5ec\ud55c\ub2e4. \ube14\ub9ac\uc790\ub4dc\ub294 2016\ub144 7\uc6d4 \uc870\uc900\uc120 \ubcf4\uc815 \ud504\ub85c\uadf8\ub7a8(\uc5d0\uc784\ud575)\uc744 \ube44\ub86f\ud55c \uc81c3\uc790 \uc678\ubd80 \ubcf4\uc815 \ud504\ub85c\uadf8\ub7a8\uc744 \uc0ac\uc6a9\ud558\ub294 \ud50c\ub808\uc774\uc5b4\ub4e4\uc5d0\uac8c \ub300\uaddc\ubaa8 \uc81c\uc7ac\ub97c \uac00\ud558\uc600\ub2e4. \u300a\uc624\ubc84\uc6cc\uce58\u300b \ucd9c\uc2dc \uc774\ud6c4, \ube14\ub9ac\uc790\ub4dc\ub294 \ud50c\ub808\uc774\uc5b4\uc5d0\uac8c \uac8c\uc784\uc5d0\uc11c \ud5c8\uc6a9\ub418\uc9c0 \uc54a\ub294 \uae30\ub2a5\uc744 \uc81c\uacf5\ud558\ub294 \uc6cc\uce58\uc624\ubc84 \ud0c0\uc77c\ub7f0\ud2b8\ub97c \uac1c\ubc1c\ud55c \ubcf4\uc2a4\ub79c\ub4dc \ud575\uc2a4\uc5d0 \ubc95\uc801 \uace0\uc18c\ub97c \ucde8\ud558\uc600\ub2e4.", "paragraph_answer": "\u300a\uc624\ubc84\uc6cc\uce58\u300b\ub294 \uc0c9\ub9f9\uc744 \uc704\ud55c \uc811\uadfc\uc131 \uc124\uc815\uc744 \ud3ec\ud568, \ud3ed\ub113\uc740 \uc0ac\uc6a9\uc790\ub97c \ud655\ubcf4\ud560 \uc218 \uc788\ub294 \uc5ec\uc12f \uac00\uc9c0 \ud2b9\uc9d5\uc744 \ud568\uaed8 \uac1c\ubc1c\ud558\uc600\ub2e4. \uac1c\ubc1c \ub2e8\uacc4\uc5d0\uc11c \uc911\uc694\ud588\ub358 \ubaa9\ud45c \ud558\ub098\ub294 \ud50c\ub808\uc774\uc5b4\ub97c \uc704\ud55c ' \uc804\ud22c\uc758 \uba85\ub8cc\uc131 '\uc73c\ub85c, \ud50c\ub808\uc774\uc5b4\uac00 \uc0c8\ub85c\uc6b4 \uc9c0\uc5ed\uc73c\ub85c \uc774\ub3d9\ud558\uba74 \uc801 \uce90\ub9ad\ud130\uac00 \ub69c\ub837\ud558\uac8c \ubcf4\uc774\ub3c4\ub85d \ud588\ub2e4. \uc774\uac83\uc740 \ud50c\ub808\uc774\uc5b4\uac00 \uc804\uc7a5\uc5d0\uc11c \uc0ac\uc6a9\ud558\ub294 \uc0c9\uc870\uc640 \ucc44\ub3c4 \uc218\uc900\uc758 \ub300\uc870, \uba85\ubc31\ud558\uac8c \ub2e4\ub978 \uc724\uacfd\uc744 \uac16\uac8c \ub9cc\ub4e0 \uce90\ub9ad\ud130\ub4e4\ub85c \ud50c\ub808\uc774\uc5b4\uac00 \uba3c \uac70\ub9ac\uc5d0\uc11c \uc601\uc6c5\uc774 \uc544\uad70\uc778\uc9c0 \uc801\uad70\uc778\uc9c0 \uad6c\ubcc4\ud560 \uc218 \uc788\ub3c4\ub85d \ud568\uc73c\ub85c\uc368 \uac00\ub2a5\ud558\uc600\ub2e4. \ud300\uc740 \uac1c\ubc1c\ub3d9\uc548 \uacbd\uae30 \uc911\uac04\uc5d0 \ud50c\ub808\uc774\uc5b4\uac00 \uc601\uc6c5\uc744 \ubc14\uafc0 \uc218 \uc788\ub3c4\ub85d \ud558\ub294 \uac83\uc774 \uac8c\uc784\ud50c\ub808\uc774\uc5d0 \uc911\uc694\ud55c \uc5ed\ud560\uc744 \ud568\uc744 \uc54c\uc544\ub0c8\ub2e4. \u300a\uc624\ubc84\uc6cc\uce58\u300b\ub294 \ucc98\uc74c\uc5d0 \uc18c\uc561 \uacb0\uc81c\ub098 \ub2e4\uc6b4\ub85c\ub4dc \ucf58\ud150\uce20 \uad6c\uc785\uc774 \ud3ec\ud568\ub41c \ubd80\ubd84 \uc720\ub8cc\ud654 \uac8c\uc784 \ud615\ud0dc\ub85c \ucd9c\uc2dc\ub97c \uacc4\ud68d\ud588\uc73c\ub098, \uc774\ub7ec\ud55c \ubc1c\uacac\uc73c\ub85c \uc778\ud574 \ub2e8\ub3c5 \uad6c\ub9e4\ud615 \uac8c\uc784\uc73c\ub85c \ub178\uc120\uc744 \ubcc0\uacbd\ud558\uc600\ub2e4. \ucf08\ub7ec\ub294 \ud300\uc774 \ud50c\ub808\uc774\uc5b4\uac00 \uc0c1\ud669\uc5d0 \ub530\ub77c \uc5b4\ub290 \uc601\uc6c5\uc73c\ub85c\ub4e0 \uad50\uccb4\ud560 \uc218 \uc788\uae30\ub97c \uc6d0\ud588\uace0, \ud50c\ub808\uc774\uc5b4\ub4e4\uc774 \ud300\uc5d0\uc11c \uc544\ubb34\ub3c4 \uc601\uc6c5\uc744 \uad6c\uc785\ud558\uc9c0 \uc54a\ub294\ub2e4\uba74 \uc18c\uc561 \uacb0\uc81c\ub098 \ub2e4\uc6b4\ub85c\ub4dc \ucf58\ud150\uce20\uac00 \ub2a5\ub825\uc744 \uc81c\ud55c\ud560 \uc218 \uc788\ub294 \uac83\uc5d0 \uac00\uae4c\uc6cc\uc9c4\ub2e4\uace0 \ub9d0\ud558\uc600\ub2e4. \ucf08\ub7ec\ub294 \ub610\ud55c \uc18c\uc561 \uacb0\uc81c \ubc0f \ub2e4\uc6b4\ub85c\ub4dc \ucf58\ud150\uce20\ub85c \uc778\ud574 \uac8c\uc774\uba38\uac00 \ubaa8\ub4e0 \ud50c\ub808\uc774\uc5b4 \ub300\uc2e0\uc5d0 \ub611\uac19\uc740 \ucee8\ud150\uce20\ub97c \uac00\uc9c0\uace0 \uc788\ub294 \uce5c\uad6c\uc640\ub9cc \ud50c\ub808\uc774\ud558\uac8c \ub418\uc5b4 \ud50c\ub808\uc774\uc5b4 \uacf5\ub3d9\uccb4\ub97c \ud574\uccb4\uc2dc\ud0ac \uc218 \uc788\ub2e4\uace0 \uc5b8\uae09\ud558\uc600\ub2e4. \uac1c\ubc1c\uc758 \ub354 \ub192\uc740 \ubaa9\ud45c\ub294 \ub2e4\ub978 \uacbd\uc7c1 \uac8c\uc784 \ud658\uacbd\uc5d0\uc11c \uc885\uc885 \uc77c\uc5b4\ub098\ub294 \ubd80\uc815\uc801 \uc131\ud5a5\uc744 \ud53c\ud558\uace0, \uc774\uc57c\uae30\uac00 \uae0d\uc815\uc801\uc778 \uba54\uc2dc\uc9c0\ub97c \uc8fc\ub3c4\ub85d \ud558\ub294 \uc9c4\uc804\uc5d0 \ub367\ubd99\uc5ec \ubd80\uc815\uc801\uc778 \uc601\ud5a5\uc744 \uc900\ub2e4\uace0 \ub290\ub07c\ub294 \uc694\uc18c\ub97c \uc5c6\uc560\uae30 \uc704\ud574 \uac8c\uc784\ud50c\ub808\uc774 \ub514\uc790\uc778\uc5d0\uc11c \uba85\ud655\ud55c \uc120\ud0dd\uc744 \ud558\ub294 \uac83\uc774\uc5c8\ub2e4. \uc774\ub7ec\ud55c \uc120\ud0dd \uc911 \ud558\ub098\ub294 \ub2e4\uc591\ud55c \ud1b5\uacc4 \uc694\uc57d\uc5d0\uc11c \uc8fd\uc784/\uc8fd\uc74c(kill/death) \ud45c\uc2dc\ub97c \uc5c6\uc560\ub294 \uac83\uc774\uc5c8\uc73c\uba70, \uce90\ud50c\ub7f0\uc740 \"\uc5b4\ub5a4 \uc601\uc6c5\ub4e4\uc740 \ud65c\uc57d\uc744 \uc704\ud574\uc11c \uc8fd\uc784\uc744 \ud544\uc694\ub85c \ud558\uc9c0 \uc54a\ub294\ub2e4\"\ub77c\uace0 \uc5b8\uae09\ud558\uc600\ub2e4. \uce5c\uc120\uc801\uc778 \ud50c\ub808\uc774 \ud658\uacbd\uc744 \uc99d\uc9c4\uc2dc\ud0a4\uae30 \uc704\ud574, \ube14\ub9ac\uc790\ub4dc\ub294 '\ud0c8\uc8fc'(\uacbd\uae30\uac00 \ub05d\ub098\uae30 \uc804\uc5d0 \uc77c\ubd80\ub7ec \ub098\uac00\ub294 \ud589\uc704)\ub97c \ud558\ub294 \ud50c\ub808\uc774\uc5b4\uc5d0\uac8c \uacbd\uae30 \ud6c4 \uacbd\ud5d8\uce58\ub97c \uac10\uc18c\uc2dc\ud0a4\uace0, \ub354 \uc2ec\ud560 \uacbd\uc6b0 \ud50c\ub808\uc774\ub97c \uc601\uad6c\ud788 \uae08\uc9c0\ud558\ub294 \ubd88\uc774\uc775\uc744 \ubd80\uc5ec\ud55c\ub2e4. \ube14\ub9ac\uc790\ub4dc\ub294 2016\ub144 7\uc6d4 \uc870\uc900\uc120 \ubcf4\uc815 \ud504\ub85c\uadf8\ub7a8(\uc5d0\uc784\ud575)\uc744 \ube44\ub86f\ud55c \uc81c3\uc790 \uc678\ubd80 \ubcf4\uc815 \ud504\ub85c\uadf8\ub7a8\uc744 \uc0ac\uc6a9\ud558\ub294 \ud50c\ub808\uc774\uc5b4\ub4e4\uc5d0\uac8c \ub300\uaddc\ubaa8 \uc81c\uc7ac\ub97c \uac00\ud558\uc600\ub2e4. \u300a\uc624\ubc84\uc6cc\uce58\u300b \ucd9c\uc2dc \uc774\ud6c4, \ube14\ub9ac\uc790\ub4dc\ub294 \ud50c\ub808\uc774\uc5b4\uc5d0\uac8c \uac8c\uc784\uc5d0\uc11c \ud5c8\uc6a9\ub418\uc9c0 \uc54a\ub294 \uae30\ub2a5\uc744 \uc81c\uacf5\ud558\ub294 \uc6cc\uce58\uc624\ubc84 \ud0c0\uc77c\ub7f0\ud2b8\ub97c \uac1c\ubc1c\ud55c \ubcf4\uc2a4\ub79c\ub4dc \ud575\uc2a4\uc5d0 \ubc95\uc801 \uace0\uc18c\ub97c \ucde8\ud558\uc600\ub2e4.", "sentence_answer": "\uac1c\ubc1c \ub2e8\uacc4\uc5d0\uc11c \uc911\uc694\ud588\ub358 \ubaa9\ud45c \ud558\ub098\ub294 \ud50c\ub808\uc774\uc5b4\ub97c \uc704\ud55c ' \uc804\ud22c\uc758 \uba85\ub8cc\uc131 '\uc73c\ub85c, \ud50c\ub808\uc774\uc5b4\uac00 \uc0c8\ub85c\uc6b4 \uc9c0\uc5ed\uc73c\ub85c \uc774\ub3d9\ud558\uba74 \uc801 \uce90\ub9ad\ud130\uac00 \ub69c\ub837\ud558\uac8c \ubcf4\uc774\ub3c4\ub85d \ud588\ub2e4.", "paragraph_id": "6459046-15-0", "paragraph_question": "question: \uc624\ubc84\uc6cc\uce58 \uac1c\ubc1c\ub2e8\uacc4\uc5d0\uc11c \uc911\uc694\uc2dc \ud588\ub358 \ubaa9\ud45c\ub294?, context: \u300a\uc624\ubc84\uc6cc\uce58\u300b\ub294 \uc0c9\ub9f9\uc744 \uc704\ud55c \uc811\uadfc\uc131 \uc124\uc815\uc744 \ud3ec\ud568, \ud3ed\ub113\uc740 \uc0ac\uc6a9\uc790\ub97c \ud655\ubcf4\ud560 \uc218 \uc788\ub294 \uc5ec\uc12f \uac00\uc9c0 \ud2b9\uc9d5\uc744 \ud568\uaed8 \uac1c\ubc1c\ud558\uc600\ub2e4. \uac1c\ubc1c \ub2e8\uacc4\uc5d0\uc11c \uc911\uc694\ud588\ub358 \ubaa9\ud45c \ud558\ub098\ub294 \ud50c\ub808\uc774\uc5b4\ub97c \uc704\ud55c '\uc804\ud22c\uc758 \uba85\ub8cc\uc131'\uc73c\ub85c, \ud50c\ub808\uc774\uc5b4\uac00 \uc0c8\ub85c\uc6b4 \uc9c0\uc5ed\uc73c\ub85c \uc774\ub3d9\ud558\uba74 \uc801 \uce90\ub9ad\ud130\uac00 \ub69c\ub837\ud558\uac8c \ubcf4\uc774\ub3c4\ub85d \ud588\ub2e4. \uc774\uac83\uc740 \ud50c\ub808\uc774\uc5b4\uac00 \uc804\uc7a5\uc5d0\uc11c \uc0ac\uc6a9\ud558\ub294 \uc0c9\uc870\uc640 \ucc44\ub3c4 \uc218\uc900\uc758 \ub300\uc870, \uba85\ubc31\ud558\uac8c \ub2e4\ub978 \uc724\uacfd\uc744 \uac16\uac8c \ub9cc\ub4e0 \uce90\ub9ad\ud130\ub4e4\ub85c \ud50c\ub808\uc774\uc5b4\uac00 \uba3c \uac70\ub9ac\uc5d0\uc11c \uc601\uc6c5\uc774 \uc544\uad70\uc778\uc9c0 \uc801\uad70\uc778\uc9c0 \uad6c\ubcc4\ud560 \uc218 \uc788\ub3c4\ub85d \ud568\uc73c\ub85c\uc368 \uac00\ub2a5\ud558\uc600\ub2e4. \ud300\uc740 \uac1c\ubc1c\ub3d9\uc548 \uacbd\uae30 \uc911\uac04\uc5d0 \ud50c\ub808\uc774\uc5b4\uac00 \uc601\uc6c5\uc744 \ubc14\uafc0 \uc218 \uc788\ub3c4\ub85d \ud558\ub294 \uac83\uc774 \uac8c\uc784\ud50c\ub808\uc774\uc5d0 \uc911\uc694\ud55c \uc5ed\ud560\uc744 \ud568\uc744 \uc54c\uc544\ub0c8\ub2e4. \u300a\uc624\ubc84\uc6cc\uce58\u300b\ub294 \ucc98\uc74c\uc5d0 \uc18c\uc561 \uacb0\uc81c\ub098 \ub2e4\uc6b4\ub85c\ub4dc \ucf58\ud150\uce20 \uad6c\uc785\uc774 \ud3ec\ud568\ub41c \ubd80\ubd84 \uc720\ub8cc\ud654 \uac8c\uc784 \ud615\ud0dc\ub85c \ucd9c\uc2dc\ub97c \uacc4\ud68d\ud588\uc73c\ub098, \uc774\ub7ec\ud55c \ubc1c\uacac\uc73c\ub85c \uc778\ud574 \ub2e8\ub3c5 \uad6c\ub9e4\ud615 \uac8c\uc784\uc73c\ub85c \ub178\uc120\uc744 \ubcc0\uacbd\ud558\uc600\ub2e4. \ucf08\ub7ec\ub294 \ud300\uc774 \ud50c\ub808\uc774\uc5b4\uac00 \uc0c1\ud669\uc5d0 \ub530\ub77c \uc5b4\ub290 \uc601\uc6c5\uc73c\ub85c\ub4e0 \uad50\uccb4\ud560 \uc218 \uc788\uae30\ub97c \uc6d0\ud588\uace0, \ud50c\ub808\uc774\uc5b4\ub4e4\uc774 \ud300\uc5d0\uc11c \uc544\ubb34\ub3c4 \uc601\uc6c5\uc744 \uad6c\uc785\ud558\uc9c0 \uc54a\ub294\ub2e4\uba74 \uc18c\uc561 \uacb0\uc81c\ub098 \ub2e4\uc6b4\ub85c\ub4dc \ucf58\ud150\uce20\uac00 \ub2a5\ub825\uc744 \uc81c\ud55c\ud560 \uc218 \uc788\ub294 \uac83\uc5d0 \uac00\uae4c\uc6cc\uc9c4\ub2e4\uace0 \ub9d0\ud558\uc600\ub2e4. \ucf08\ub7ec\ub294 \ub610\ud55c \uc18c\uc561 \uacb0\uc81c \ubc0f \ub2e4\uc6b4\ub85c\ub4dc \ucf58\ud150\uce20\ub85c \uc778\ud574 \uac8c\uc774\uba38\uac00 \ubaa8\ub4e0 \ud50c\ub808\uc774\uc5b4 \ub300\uc2e0\uc5d0 \ub611\uac19\uc740 \ucee8\ud150\uce20\ub97c \uac00\uc9c0\uace0 \uc788\ub294 \uce5c\uad6c\uc640\ub9cc \ud50c\ub808\uc774\ud558\uac8c \ub418\uc5b4 \ud50c\ub808\uc774\uc5b4 \uacf5\ub3d9\uccb4\ub97c \ud574\uccb4\uc2dc\ud0ac \uc218 \uc788\ub2e4\uace0 \uc5b8\uae09\ud558\uc600\ub2e4. \uac1c\ubc1c\uc758 \ub354 \ub192\uc740 \ubaa9\ud45c\ub294 \ub2e4\ub978 \uacbd\uc7c1 \uac8c\uc784 \ud658\uacbd\uc5d0\uc11c \uc885\uc885 \uc77c\uc5b4\ub098\ub294 \ubd80\uc815\uc801 \uc131\ud5a5\uc744 \ud53c\ud558\uace0, \uc774\uc57c\uae30\uac00 \uae0d\uc815\uc801\uc778 \uba54\uc2dc\uc9c0\ub97c \uc8fc\ub3c4\ub85d \ud558\ub294 \uc9c4\uc804\uc5d0 \ub367\ubd99\uc5ec \ubd80\uc815\uc801\uc778 \uc601\ud5a5\uc744 \uc900\ub2e4\uace0 \ub290\ub07c\ub294 \uc694\uc18c\ub97c \uc5c6\uc560\uae30 \uc704\ud574 \uac8c\uc784\ud50c\ub808\uc774 \ub514\uc790\uc778\uc5d0\uc11c \uba85\ud655\ud55c \uc120\ud0dd\uc744 \ud558\ub294 \uac83\uc774\uc5c8\ub2e4. \uc774\ub7ec\ud55c \uc120\ud0dd \uc911 \ud558\ub098\ub294 \ub2e4\uc591\ud55c \ud1b5\uacc4 \uc694\uc57d\uc5d0\uc11c \uc8fd\uc784/\uc8fd\uc74c(kill/death) \ud45c\uc2dc\ub97c \uc5c6\uc560\ub294 \uac83\uc774\uc5c8\uc73c\uba70, \uce90\ud50c\ub7f0\uc740 \"\uc5b4\ub5a4 \uc601\uc6c5\ub4e4\uc740 \ud65c\uc57d\uc744 \uc704\ud574\uc11c \uc8fd\uc784\uc744 \ud544\uc694\ub85c \ud558\uc9c0 \uc54a\ub294\ub2e4\"\ub77c\uace0 \uc5b8\uae09\ud558\uc600\ub2e4. \uce5c\uc120\uc801\uc778 \ud50c\ub808\uc774 \ud658\uacbd\uc744 \uc99d\uc9c4\uc2dc\ud0a4\uae30 \uc704\ud574, \ube14\ub9ac\uc790\ub4dc\ub294 '\ud0c8\uc8fc'(\uacbd\uae30\uac00 \ub05d\ub098\uae30 \uc804\uc5d0 \uc77c\ubd80\ub7ec \ub098\uac00\ub294 \ud589\uc704)\ub97c \ud558\ub294 \ud50c\ub808\uc774\uc5b4\uc5d0\uac8c \uacbd\uae30 \ud6c4 \uacbd\ud5d8\uce58\ub97c \uac10\uc18c\uc2dc\ud0a4\uace0, \ub354 \uc2ec\ud560 \uacbd\uc6b0 \ud50c\ub808\uc774\ub97c \uc601\uad6c\ud788 \uae08\uc9c0\ud558\ub294 \ubd88\uc774\uc775\uc744 \ubd80\uc5ec\ud55c\ub2e4. \ube14\ub9ac\uc790\ub4dc\ub294 2016\ub144 7\uc6d4 \uc870\uc900\uc120 \ubcf4\uc815 \ud504\ub85c\uadf8\ub7a8(\uc5d0\uc784\ud575)\uc744 \ube44\ub86f\ud55c \uc81c3\uc790 \uc678\ubd80 \ubcf4\uc815 \ud504\ub85c\uadf8\ub7a8\uc744 \uc0ac\uc6a9\ud558\ub294 \ud50c\ub808\uc774\uc5b4\ub4e4\uc5d0\uac8c \ub300\uaddc\ubaa8 \uc81c\uc7ac\ub97c \uac00\ud558\uc600\ub2e4. \u300a\uc624\ubc84\uc6cc\uce58\u300b \ucd9c\uc2dc \uc774\ud6c4, \ube14\ub9ac\uc790\ub4dc\ub294 \ud50c\ub808\uc774\uc5b4\uc5d0\uac8c \uac8c\uc784\uc5d0\uc11c \ud5c8\uc6a9\ub418\uc9c0 \uc54a\ub294 \uae30\ub2a5\uc744 \uc81c\uacf5\ud558\ub294 \uc6cc\uce58\uc624\ubc84 \ud0c0\uc77c\ub7f0\ud2b8\ub97c \uac1c\ubc1c\ud55c \ubcf4\uc2a4\ub79c\ub4dc \ud575\uc2a4\uc5d0 \ubc95\uc801 \uace0\uc18c\ub97c \ucde8\ud558\uc600\ub2e4."} {"question": "\uc628\ub3c4\ub97c 100\ub3c4\ub85c \uc62c\ub838\uc744 \ub54c 0.1% \ud33d\ucc3d\ud558\uace0 \ucef5\uc744 \ub9cc\ub4e4\ub54c \uc8fc\ub85c \uc4f0\uc774\ub294 \uc720\ub9ac\ub294 \ubb34\uc5c7\uc778\uac00\uc694?", "paragraph": "\ucc28\uac00\uc6b4 \uc720\ub9ac\ucef5\uc5d0 \uac11\uc790\uae30 \ub728\uac70\uc6b4 \ubb3c\uc744 \ubd93\uac70\ub098 \ubc18\ub300\ub85c \ub728\uac70\uc6b4 \uc720\ub9ac\ucef5\uc5d0 \ucc28\uac00\uc6b4 \ubb3c\uc744 \ubd80\uc73c\uba74 \uae68\uc9c4\ub2e4. \uc774\uac83\uc740 \ub728\uac70\uc6b4 \ubb3c\uc774\ub098 \ucc28\uac00\uc6b4 \ubb3c\uc774 \ub2ff\uc740 \ubd80\ubd84\uc740 \ud33d\ucc3d\ud558\uac70\ub098 \uc218\ucd95\ud558\uc9c0\ub9cc \ub728\uac70\uc6b4 \ubb3c\uc774\ub098 \ucc28\uac00\uc6b4 \ubb3c\uc774 \ub2ff\uc9c0 \uc54a\ub294 \ubc14\uae65\uba74\uc740 \uadf8\ub300\ub85c\uc774\ubbc0\ub85c \ucef5\uc5d0 \ubb34\ub9ac\ud55c \ud798\uc774 \uac00\ud574\uc9c0\uae30 \ub54c\ubb38\uc774\ub2e4. \ubcf4\ud1b5\uc758 \ucef5\uc744 \ub9cc\ub4dc\ub294 \ub370 \uc4f0\uc774\ub294 \uc18c\ub2e4\uc11d\ud68c\uc720\ub9ac\ub098 \uace0\uae09\uc2dd\uae30 \ub4f1\uc73c\ub85c \uc4f0\uc774\ub294 \ud06c\ub9ac\uc2a4\ud138\uae00\ub77c\uc2a4(\ub0a9\uc720\ub9ac)\ub294 \uc628\ub3c4\ub97c 100\u2103\ub85c \uc62c\ub9ac\uba74 0.1%\ud33d\ucc3d\ud55c\ub2e4. \uadf8\ub7ec\ub098 \uc57d\ud488\uc5d0 \uce68\uc2dd\ub418\uc9c0 \uc54a\ub294 \uc720\ub9ac\ub85c\uc11c \ud654\ud559\uc2e4\ud5d8\uae30\uad6c \ub4f1\uc73c\ub85c \uc4f0\uc774\ub294 \ubd95\uaddc\uc0b0\uc720\ub9ac\ub098 \uc800 \uc54c\uce7c\ub9ac\ubd95\uaddc\uc0b0\uc720\ub9ac\ub294 100\u2103 \uc628\ub3c4\ucc28\uc5d0\uc11c 0.03% \uc815\ub3c4\ubc16\uc5d0 \ud33d\ucc3d\ud558\uc9c0 \uc54a\ub294\ub2e4. \uadf8 \ub54c\ubb38\uc5d0 \uc18c\ub2e4\uc11d\ud68c\uc720\ub9ac\uc758 3\ubc30 \uc815\ub3c4 \uc628\ub3c4\ucc28\uc5d0 \uacac\ub51c \uc218 \uc788\uc73c\uba70, \uc99d\uae30\uc0b4\uade0\uc774\ub098 \ube44\ub4f1\uc5d0 \uacac\ub514\ubbc0\ub85c \uc774 \uc810\ub3c4 \ud654\ud559\uae30\uad6c\u00b7\uc758\ub8cc\uae30\uad6c\uc6a9\uc73c\ub85c\uc11c \uc774\uc6a9\ub418\ub294 \uc774\uc720 \uc911\uc758 \ud558\ub098\ub2e4.", "answer": "\ud06c\ub9ac\uc2a4\ud138\uae00\ub77c\uc2a4", "sentence": "\ubcf4\ud1b5\uc758 \ucef5\uc744 \ub9cc\ub4dc\ub294 \ub370 \uc4f0\uc774\ub294 \uc18c\ub2e4\uc11d\ud68c\uc720\ub9ac\ub098 \uace0\uae09\uc2dd\uae30 \ub4f1\uc73c\ub85c \uc4f0\uc774\ub294 \ud06c\ub9ac\uc2a4\ud138\uae00\ub77c\uc2a4 (\ub0a9\uc720\ub9ac)\ub294 \uc628\ub3c4\ub97c 100\u2103\ub85c \uc62c\ub9ac\uba74 0.1%\ud33d\ucc3d\ud55c\ub2e4.", "paragraph_sentence": "\ucc28\uac00\uc6b4 \uc720\ub9ac\ucef5\uc5d0 \uac11\uc790\uae30 \ub728\uac70\uc6b4 \ubb3c\uc744 \ubd93\uac70\ub098 \ubc18\ub300\ub85c \ub728\uac70\uc6b4 \uc720\ub9ac\ucef5\uc5d0 \ucc28\uac00\uc6b4 \ubb3c\uc744 \ubd80\uc73c\uba74 \uae68\uc9c4\ub2e4. \uc774\uac83\uc740 \ub728\uac70\uc6b4 \ubb3c\uc774\ub098 \ucc28\uac00\uc6b4 \ubb3c\uc774 \ub2ff\uc740 \ubd80\ubd84\uc740 \ud33d\ucc3d\ud558\uac70\ub098 \uc218\ucd95\ud558\uc9c0\ub9cc \ub728\uac70\uc6b4 \ubb3c\uc774\ub098 \ucc28\uac00\uc6b4 \ubb3c\uc774 \ub2ff\uc9c0 \uc54a\ub294 \ubc14\uae65\uba74\uc740 \uadf8\ub300\ub85c\uc774\ubbc0\ub85c \ucef5\uc5d0 \ubb34\ub9ac\ud55c \ud798\uc774 \uac00\ud574\uc9c0\uae30 \ub54c\ubb38\uc774\ub2e4. \ubcf4\ud1b5\uc758 \ucef5\uc744 \ub9cc\ub4dc\ub294 \ub370 \uc4f0\uc774\ub294 \uc18c\ub2e4\uc11d\ud68c\uc720\ub9ac\ub098 \uace0\uae09\uc2dd\uae30 \ub4f1\uc73c\ub85c \uc4f0\uc774\ub294 \ud06c\ub9ac\uc2a4\ud138\uae00\ub77c\uc2a4 (\ub0a9\uc720\ub9ac)\ub294 \uc628\ub3c4\ub97c 100\u2103\ub85c \uc62c\ub9ac\uba74 0.1%\ud33d\ucc3d\ud55c\ub2e4. \uadf8\ub7ec\ub098 \uc57d\ud488\uc5d0 \uce68\uc2dd\ub418\uc9c0 \uc54a\ub294 \uc720\ub9ac\ub85c\uc11c \ud654\ud559\uc2e4\ud5d8\uae30\uad6c \ub4f1\uc73c\ub85c \uc4f0\uc774\ub294 \ubd95\uaddc\uc0b0\uc720\ub9ac\ub098 \uc800 \uc54c\uce7c\ub9ac\ubd95\uaddc\uc0b0\uc720\ub9ac\ub294 100\u2103 \uc628\ub3c4\ucc28\uc5d0\uc11c 0.03% \uc815\ub3c4\ubc16\uc5d0 \ud33d\ucc3d\ud558\uc9c0 \uc54a\ub294\ub2e4. \uadf8 \ub54c\ubb38\uc5d0 \uc18c\ub2e4\uc11d\ud68c\uc720\ub9ac\uc758 3\ubc30 \uc815\ub3c4 \uc628\ub3c4\ucc28\uc5d0 \uacac\ub51c \uc218 \uc788\uc73c\uba70, \uc99d\uae30\uc0b4\uade0\uc774\ub098 \ube44\ub4f1\uc5d0 \uacac\ub514\ubbc0\ub85c \uc774 \uc810\ub3c4 \ud654\ud559\uae30\uad6c\u00b7\uc758\ub8cc\uae30\uad6c\uc6a9\uc73c\ub85c\uc11c \uc774\uc6a9\ub418\ub294 \uc774\uc720 \uc911\uc758 \ud558\ub098\ub2e4.", "paragraph_answer": "\ucc28\uac00\uc6b4 \uc720\ub9ac\ucef5\uc5d0 \uac11\uc790\uae30 \ub728\uac70\uc6b4 \ubb3c\uc744 \ubd93\uac70\ub098 \ubc18\ub300\ub85c \ub728\uac70\uc6b4 \uc720\ub9ac\ucef5\uc5d0 \ucc28\uac00\uc6b4 \ubb3c\uc744 \ubd80\uc73c\uba74 \uae68\uc9c4\ub2e4. \uc774\uac83\uc740 \ub728\uac70\uc6b4 \ubb3c\uc774\ub098 \ucc28\uac00\uc6b4 \ubb3c\uc774 \ub2ff\uc740 \ubd80\ubd84\uc740 \ud33d\ucc3d\ud558\uac70\ub098 \uc218\ucd95\ud558\uc9c0\ub9cc \ub728\uac70\uc6b4 \ubb3c\uc774\ub098 \ucc28\uac00\uc6b4 \ubb3c\uc774 \ub2ff\uc9c0 \uc54a\ub294 \ubc14\uae65\uba74\uc740 \uadf8\ub300\ub85c\uc774\ubbc0\ub85c \ucef5\uc5d0 \ubb34\ub9ac\ud55c \ud798\uc774 \uac00\ud574\uc9c0\uae30 \ub54c\ubb38\uc774\ub2e4. \ubcf4\ud1b5\uc758 \ucef5\uc744 \ub9cc\ub4dc\ub294 \ub370 \uc4f0\uc774\ub294 \uc18c\ub2e4\uc11d\ud68c\uc720\ub9ac\ub098 \uace0\uae09\uc2dd\uae30 \ub4f1\uc73c\ub85c \uc4f0\uc774\ub294 \ud06c\ub9ac\uc2a4\ud138\uae00\ub77c\uc2a4 (\ub0a9\uc720\ub9ac)\ub294 \uc628\ub3c4\ub97c 100\u2103\ub85c \uc62c\ub9ac\uba74 0.1%\ud33d\ucc3d\ud55c\ub2e4. \uadf8\ub7ec\ub098 \uc57d\ud488\uc5d0 \uce68\uc2dd\ub418\uc9c0 \uc54a\ub294 \uc720\ub9ac\ub85c\uc11c \ud654\ud559\uc2e4\ud5d8\uae30\uad6c \ub4f1\uc73c\ub85c \uc4f0\uc774\ub294 \ubd95\uaddc\uc0b0\uc720\ub9ac\ub098 \uc800 \uc54c\uce7c\ub9ac\ubd95\uaddc\uc0b0\uc720\ub9ac\ub294 100\u2103 \uc628\ub3c4\ucc28\uc5d0\uc11c 0.03% \uc815\ub3c4\ubc16\uc5d0 \ud33d\ucc3d\ud558\uc9c0 \uc54a\ub294\ub2e4. \uadf8 \ub54c\ubb38\uc5d0 \uc18c\ub2e4\uc11d\ud68c\uc720\ub9ac\uc758 3\ubc30 \uc815\ub3c4 \uc628\ub3c4\ucc28\uc5d0 \uacac\ub51c \uc218 \uc788\uc73c\uba70, \uc99d\uae30\uc0b4\uade0\uc774\ub098 \ube44\ub4f1\uc5d0 \uacac\ub514\ubbc0\ub85c \uc774 \uc810\ub3c4 \ud654\ud559\uae30\uad6c\u00b7\uc758\ub8cc\uae30\uad6c\uc6a9\uc73c\ub85c\uc11c \uc774\uc6a9\ub418\ub294 \uc774\uc720 \uc911\uc758 \ud558\ub098\ub2e4.", "sentence_answer": "\ubcf4\ud1b5\uc758 \ucef5\uc744 \ub9cc\ub4dc\ub294 \ub370 \uc4f0\uc774\ub294 \uc18c\ub2e4\uc11d\ud68c\uc720\ub9ac\ub098 \uace0\uae09\uc2dd\uae30 \ub4f1\uc73c\ub85c \uc4f0\uc774\ub294 \ud06c\ub9ac\uc2a4\ud138\uae00\ub77c\uc2a4 (\ub0a9\uc720\ub9ac)\ub294 \uc628\ub3c4\ub97c 100\u2103\ub85c \uc62c\ub9ac\uba74 0.1%\ud33d\ucc3d\ud55c\ub2e4.", "paragraph_id": "6542977-5-1", "paragraph_question": "question: \uc628\ub3c4\ub97c 100\ub3c4\ub85c \uc62c\ub838\uc744 \ub54c 0.1% \ud33d\ucc3d\ud558\uace0 \ucef5\uc744 \ub9cc\ub4e4\ub54c \uc8fc\ub85c \uc4f0\uc774\ub294 \uc720\ub9ac\ub294 \ubb34\uc5c7\uc778\uac00\uc694?, context: \ucc28\uac00\uc6b4 \uc720\ub9ac\ucef5\uc5d0 \uac11\uc790\uae30 \ub728\uac70\uc6b4 \ubb3c\uc744 \ubd93\uac70\ub098 \ubc18\ub300\ub85c \ub728\uac70\uc6b4 \uc720\ub9ac\ucef5\uc5d0 \ucc28\uac00\uc6b4 \ubb3c\uc744 \ubd80\uc73c\uba74 \uae68\uc9c4\ub2e4. \uc774\uac83\uc740 \ub728\uac70\uc6b4 \ubb3c\uc774\ub098 \ucc28\uac00\uc6b4 \ubb3c\uc774 \ub2ff\uc740 \ubd80\ubd84\uc740 \ud33d\ucc3d\ud558\uac70\ub098 \uc218\ucd95\ud558\uc9c0\ub9cc \ub728\uac70\uc6b4 \ubb3c\uc774\ub098 \ucc28\uac00\uc6b4 \ubb3c\uc774 \ub2ff\uc9c0 \uc54a\ub294 \ubc14\uae65\uba74\uc740 \uadf8\ub300\ub85c\uc774\ubbc0\ub85c \ucef5\uc5d0 \ubb34\ub9ac\ud55c \ud798\uc774 \uac00\ud574\uc9c0\uae30 \ub54c\ubb38\uc774\ub2e4. \ubcf4\ud1b5\uc758 \ucef5\uc744 \ub9cc\ub4dc\ub294 \ub370 \uc4f0\uc774\ub294 \uc18c\ub2e4\uc11d\ud68c\uc720\ub9ac\ub098 \uace0\uae09\uc2dd\uae30 \ub4f1\uc73c\ub85c \uc4f0\uc774\ub294 \ud06c\ub9ac\uc2a4\ud138\uae00\ub77c\uc2a4(\ub0a9\uc720\ub9ac)\ub294 \uc628\ub3c4\ub97c 100\u2103\ub85c \uc62c\ub9ac\uba74 0.1%\ud33d\ucc3d\ud55c\ub2e4. \uadf8\ub7ec\ub098 \uc57d\ud488\uc5d0 \uce68\uc2dd\ub418\uc9c0 \uc54a\ub294 \uc720\ub9ac\ub85c\uc11c \ud654\ud559\uc2e4\ud5d8\uae30\uad6c \ub4f1\uc73c\ub85c \uc4f0\uc774\ub294 \ubd95\uaddc\uc0b0\uc720\ub9ac\ub098 \uc800 \uc54c\uce7c\ub9ac\ubd95\uaddc\uc0b0\uc720\ub9ac\ub294 100\u2103 \uc628\ub3c4\ucc28\uc5d0\uc11c 0.03% \uc815\ub3c4\ubc16\uc5d0 \ud33d\ucc3d\ud558\uc9c0 \uc54a\ub294\ub2e4. \uadf8 \ub54c\ubb38\uc5d0 \uc18c\ub2e4\uc11d\ud68c\uc720\ub9ac\uc758 3\ubc30 \uc815\ub3c4 \uc628\ub3c4\ucc28\uc5d0 \uacac\ub51c \uc218 \uc788\uc73c\uba70, \uc99d\uae30\uc0b4\uade0\uc774\ub098 \ube44\ub4f1\uc5d0 \uacac\ub514\ubbc0\ub85c \uc774 \uc810\ub3c4 \ud654\ud559\uae30\uad6c\u00b7\uc758\ub8cc\uae30\uad6c\uc6a9\uc73c\ub85c\uc11c \uc774\uc6a9\ub418\ub294 \uc774\uc720 \uc911\uc758 \ud558\ub098\ub2e4."} {"question": "\"\uc790\uc5f0\uc740 \uc218\ud559\uc774\ub77c\ub294 \uc5b8\uc5b4\ub85c \uc50c\uc5ec\uc9c4 \ucc45\uc774\ub2e4\"\ub77c\uace0 \ub9d0\ud55c \uc0ac\ub78c\uc740 \ub204\uad6c\uc778\uac00?", "paragraph": "\uac08\ub9b4\ub808\uc774\ub294 \u201c\uc790\uc5f0\uc740 \uc218\ud559\uc774\ub77c\ub294 \uc5b8\uc5b4\ub85c \uc50c\uc5ec\uc9c4 \ucc45\uc774\ub2e4.\u201d\ub77c\uace0 \ub9d0\ud558\uba74\uc11c \uc790\uc720 \ub099\ud558 \ud604\uc0c1\uc744 \uc218\ud559\uc801\uc73c\ub85c \uae30\uc220\ud558\ub824 \ud558\uc600\ub2e4. \uac08\ub9b4\ub808\uc774\uc758 \uc774\ub7ec\ud55c \uacbd\ud5a5\uc131\uc740 15\uc138\uae30 \ub9d0\uacbd\ubd80\ud130 \uc218\ud559\uc801\uc778 \uc138\uacc4\uad00\uc744 \uac15\uc870\ud558\ub294 \ud50c\ub77c\ud1a4\uc801 \ubd84\uc704\uae30\uac00 \uc774\ud0c8\ub9ac\uc544\ub97c \uc911\uc2ec\uc73c\ub85c \ud37c\uc838 \uc788\ub358 \uc5ed\uc0ac\uc801 \ubc30\uacbd\uc5d0 \ud06c\uac8c \uc601\ud5a5\uc744 \ubc1b\uc558\uc744 \uac83\uc774\ub2e4. \uadf8\ub9ac\uace0 \uac08\ub9b4\ub808\uc774\ub294 \uc624\ub298\ub0a0\uc758 \ud559\uc790\ub4e4\uacfc \uac19\uc740 \ubaa9\uc801\uc73c\ub85c \uc5f0\uad6c\ud558\uace0 \uae00\uc744 \uc4f0\uc9c0 \uc54a\uc558\ub2e4. \uc5f0\uad6c \uacb0\uacfc\ub97c \ud559\uc220\uc9c0\uc5d0 \ucd9c\ud310\ud558\uc5ec \ub3d9\ub8cc\ub4e4\uc758 \uc778\uc815\uc744 \ubc1b\uae30 \uc704\ud574\uc11c\ub3c4 \uc544\ub2c8\uc5c8\ub2e4. \uc790\uc5f0 \ud604\uc0c1\uc5d0 \ub300\ud55c \ud638\uae30\uc2ec\uc5d0 \ub354\ud574\uc11c \uac08\ub9b4\ub808\uc774\ub294 \ub2f9\uc2dc \uc790\uc2e0\uc744 \uc7ac\uc815\uc801, \uc815\uce58\uc801\uc73c\ub85c \ud6c4\uc6d0\ud574 \uc904 \uc218 \uc788\ub294 \uad8c\ub825\uc790\ub4e4\uc744 \uc5fc\ub450\uc5d0 \ub450\uace0 \uae00\uc744 \uc37c\ub2e4. \ubaa9\uc131 \uc8fc\ubcc0\uc744 \ub3c4\ub294 4\uac1c\uc758 \uc704\uc131\uc744 \ub9dd\uc6d0\uacbd\uc73c\ub85c \uad00\ucc30\ud588\uc744 \ub54c, \uadf8\ub294 \uc774\ub4e4\uc744 \u2018\uba54\ub514\uce58\uc758 \ubcc4\ub4e4\u2019\uc774\ub77c\uace0 \ubd80\ub97c \uac83\uc744 \uc81c\uc548\ud558\uae30\ub3c4 \ud558\uc600\ub2e4. \uba54\ub514\uce58\uac00\ub294 \uc774\ub7ec\ud55c \uac08\ub9b4\ub808\uc774\ub97c \uad81\uc815 \uc218\ud559\uc790\ub85c \ucc44\uc6a9\ud558\uc5ec \uc88b\uc740 \uc870\uac74\uc5d0\uc11c \uc5f0\uad6c\uc5d0 \ubab0\ub450\ud558\ub3c4\ub85d \ubc30\ub824\ud558\uc600\ub2e4.", "answer": "\uac08\ub9b4\ub808\uc774", "sentence": "\uac08\ub9b4\ub808\uc774 \ub294 \u201c\uc790\uc5f0\uc740 \uc218\ud559\uc774\ub77c\ub294 \uc5b8\uc5b4\ub85c \uc50c\uc5ec\uc9c4 \ucc45\uc774\ub2e4.", "paragraph_sentence": " \uac08\ub9b4\ub808\uc774 \ub294 \u201c\uc790\uc5f0\uc740 \uc218\ud559\uc774\ub77c\ub294 \uc5b8\uc5b4\ub85c \uc50c\uc5ec\uc9c4 \ucc45\uc774\ub2e4. \u201d\ub77c\uace0 \ub9d0\ud558\uba74\uc11c \uc790\uc720 \ub099\ud558 \ud604\uc0c1\uc744 \uc218\ud559\uc801\uc73c\ub85c \uae30\uc220\ud558\ub824 \ud558\uc600\ub2e4. \uac08\ub9b4\ub808\uc774\uc758 \uc774\ub7ec\ud55c \uacbd\ud5a5\uc131\uc740 15\uc138\uae30 \ub9d0\uacbd\ubd80\ud130 \uc218\ud559\uc801\uc778 \uc138\uacc4\uad00\uc744 \uac15\uc870\ud558\ub294 \ud50c\ub77c\ud1a4\uc801 \ubd84\uc704\uae30\uac00 \uc774\ud0c8\ub9ac\uc544\ub97c \uc911\uc2ec\uc73c\ub85c \ud37c\uc838 \uc788\ub358 \uc5ed\uc0ac\uc801 \ubc30\uacbd\uc5d0 \ud06c\uac8c \uc601\ud5a5\uc744 \ubc1b\uc558\uc744 \uac83\uc774\ub2e4. \uadf8\ub9ac\uace0 \uac08\ub9b4\ub808\uc774\ub294 \uc624\ub298\ub0a0\uc758 \ud559\uc790\ub4e4\uacfc \uac19\uc740 \ubaa9\uc801\uc73c\ub85c \uc5f0\uad6c\ud558\uace0 \uae00\uc744 \uc4f0\uc9c0 \uc54a\uc558\ub2e4. \uc5f0\uad6c \uacb0\uacfc\ub97c \ud559\uc220\uc9c0\uc5d0 \ucd9c\ud310\ud558\uc5ec \ub3d9\ub8cc\ub4e4\uc758 \uc778\uc815\uc744 \ubc1b\uae30 \uc704\ud574\uc11c\ub3c4 \uc544\ub2c8\uc5c8\ub2e4. \uc790\uc5f0 \ud604\uc0c1\uc5d0 \ub300\ud55c \ud638\uae30\uc2ec\uc5d0 \ub354\ud574\uc11c \uac08\ub9b4\ub808\uc774\ub294 \ub2f9\uc2dc \uc790\uc2e0\uc744 \uc7ac\uc815\uc801, \uc815\uce58\uc801\uc73c\ub85c \ud6c4\uc6d0\ud574 \uc904 \uc218 \uc788\ub294 \uad8c\ub825\uc790\ub4e4\uc744 \uc5fc\ub450\uc5d0 \ub450\uace0 \uae00\uc744 \uc37c\ub2e4. \ubaa9\uc131 \uc8fc\ubcc0\uc744 \ub3c4\ub294 4\uac1c\uc758 \uc704\uc131\uc744 \ub9dd\uc6d0\uacbd\uc73c\ub85c \uad00\ucc30\ud588\uc744 \ub54c, \uadf8\ub294 \uc774\ub4e4\uc744 \u2018\uba54\ub514\uce58\uc758 \ubcc4\ub4e4\u2019\uc774\ub77c\uace0 \ubd80\ub97c \uac83\uc744 \uc81c\uc548\ud558\uae30\ub3c4 \ud558\uc600\ub2e4. \uba54\ub514\uce58\uac00\ub294 \uc774\ub7ec\ud55c \uac08\ub9b4\ub808\uc774\ub97c \uad81\uc815 \uc218\ud559\uc790\ub85c \ucc44\uc6a9\ud558\uc5ec \uc88b\uc740 \uc870\uac74\uc5d0\uc11c \uc5f0\uad6c\uc5d0 \ubab0\ub450\ud558\ub3c4\ub85d \ubc30\ub824\ud558\uc600\ub2e4.", "paragraph_answer": " \uac08\ub9b4\ub808\uc774 \ub294 \u201c\uc790\uc5f0\uc740 \uc218\ud559\uc774\ub77c\ub294 \uc5b8\uc5b4\ub85c \uc50c\uc5ec\uc9c4 \ucc45\uc774\ub2e4.\u201d\ub77c\uace0 \ub9d0\ud558\uba74\uc11c \uc790\uc720 \ub099\ud558 \ud604\uc0c1\uc744 \uc218\ud559\uc801\uc73c\ub85c \uae30\uc220\ud558\ub824 \ud558\uc600\ub2e4. \uac08\ub9b4\ub808\uc774\uc758 \uc774\ub7ec\ud55c \uacbd\ud5a5\uc131\uc740 15\uc138\uae30 \ub9d0\uacbd\ubd80\ud130 \uc218\ud559\uc801\uc778 \uc138\uacc4\uad00\uc744 \uac15\uc870\ud558\ub294 \ud50c\ub77c\ud1a4\uc801 \ubd84\uc704\uae30\uac00 \uc774\ud0c8\ub9ac\uc544\ub97c \uc911\uc2ec\uc73c\ub85c \ud37c\uc838 \uc788\ub358 \uc5ed\uc0ac\uc801 \ubc30\uacbd\uc5d0 \ud06c\uac8c \uc601\ud5a5\uc744 \ubc1b\uc558\uc744 \uac83\uc774\ub2e4. \uadf8\ub9ac\uace0 \uac08\ub9b4\ub808\uc774\ub294 \uc624\ub298\ub0a0\uc758 \ud559\uc790\ub4e4\uacfc \uac19\uc740 \ubaa9\uc801\uc73c\ub85c \uc5f0\uad6c\ud558\uace0 \uae00\uc744 \uc4f0\uc9c0 \uc54a\uc558\ub2e4. \uc5f0\uad6c \uacb0\uacfc\ub97c \ud559\uc220\uc9c0\uc5d0 \ucd9c\ud310\ud558\uc5ec \ub3d9\ub8cc\ub4e4\uc758 \uc778\uc815\uc744 \ubc1b\uae30 \uc704\ud574\uc11c\ub3c4 \uc544\ub2c8\uc5c8\ub2e4. \uc790\uc5f0 \ud604\uc0c1\uc5d0 \ub300\ud55c \ud638\uae30\uc2ec\uc5d0 \ub354\ud574\uc11c \uac08\ub9b4\ub808\uc774\ub294 \ub2f9\uc2dc \uc790\uc2e0\uc744 \uc7ac\uc815\uc801, \uc815\uce58\uc801\uc73c\ub85c \ud6c4\uc6d0\ud574 \uc904 \uc218 \uc788\ub294 \uad8c\ub825\uc790\ub4e4\uc744 \uc5fc\ub450\uc5d0 \ub450\uace0 \uae00\uc744 \uc37c\ub2e4. \ubaa9\uc131 \uc8fc\ubcc0\uc744 \ub3c4\ub294 4\uac1c\uc758 \uc704\uc131\uc744 \ub9dd\uc6d0\uacbd\uc73c\ub85c \uad00\ucc30\ud588\uc744 \ub54c, \uadf8\ub294 \uc774\ub4e4\uc744 \u2018\uba54\ub514\uce58\uc758 \ubcc4\ub4e4\u2019\uc774\ub77c\uace0 \ubd80\ub97c \uac83\uc744 \uc81c\uc548\ud558\uae30\ub3c4 \ud558\uc600\ub2e4. \uba54\ub514\uce58\uac00\ub294 \uc774\ub7ec\ud55c \uac08\ub9b4\ub808\uc774\ub97c \uad81\uc815 \uc218\ud559\uc790\ub85c \ucc44\uc6a9\ud558\uc5ec \uc88b\uc740 \uc870\uac74\uc5d0\uc11c \uc5f0\uad6c\uc5d0 \ubab0\ub450\ud558\ub3c4\ub85d \ubc30\ub824\ud558\uc600\ub2e4.", "sentence_answer": " \uac08\ub9b4\ub808\uc774 \ub294 \u201c\uc790\uc5f0\uc740 \uc218\ud559\uc774\ub77c\ub294 \uc5b8\uc5b4\ub85c \uc50c\uc5ec\uc9c4 \ucc45\uc774\ub2e4.", "paragraph_id": "6471943-36-0", "paragraph_question": "question: \"\uc790\uc5f0\uc740 \uc218\ud559\uc774\ub77c\ub294 \uc5b8\uc5b4\ub85c \uc50c\uc5ec\uc9c4 \ucc45\uc774\ub2e4\"\ub77c\uace0 \ub9d0\ud55c \uc0ac\ub78c\uc740 \ub204\uad6c\uc778\uac00?, context: \uac08\ub9b4\ub808\uc774\ub294 \u201c\uc790\uc5f0\uc740 \uc218\ud559\uc774\ub77c\ub294 \uc5b8\uc5b4\ub85c \uc50c\uc5ec\uc9c4 \ucc45\uc774\ub2e4.\u201d\ub77c\uace0 \ub9d0\ud558\uba74\uc11c \uc790\uc720 \ub099\ud558 \ud604\uc0c1\uc744 \uc218\ud559\uc801\uc73c\ub85c \uae30\uc220\ud558\ub824 \ud558\uc600\ub2e4. \uac08\ub9b4\ub808\uc774\uc758 \uc774\ub7ec\ud55c \uacbd\ud5a5\uc131\uc740 15\uc138\uae30 \ub9d0\uacbd\ubd80\ud130 \uc218\ud559\uc801\uc778 \uc138\uacc4\uad00\uc744 \uac15\uc870\ud558\ub294 \ud50c\ub77c\ud1a4\uc801 \ubd84\uc704\uae30\uac00 \uc774\ud0c8\ub9ac\uc544\ub97c \uc911\uc2ec\uc73c\ub85c \ud37c\uc838 \uc788\ub358 \uc5ed\uc0ac\uc801 \ubc30\uacbd\uc5d0 \ud06c\uac8c \uc601\ud5a5\uc744 \ubc1b\uc558\uc744 \uac83\uc774\ub2e4. \uadf8\ub9ac\uace0 \uac08\ub9b4\ub808\uc774\ub294 \uc624\ub298\ub0a0\uc758 \ud559\uc790\ub4e4\uacfc \uac19\uc740 \ubaa9\uc801\uc73c\ub85c \uc5f0\uad6c\ud558\uace0 \uae00\uc744 \uc4f0\uc9c0 \uc54a\uc558\ub2e4. \uc5f0\uad6c \uacb0\uacfc\ub97c \ud559\uc220\uc9c0\uc5d0 \ucd9c\ud310\ud558\uc5ec \ub3d9\ub8cc\ub4e4\uc758 \uc778\uc815\uc744 \ubc1b\uae30 \uc704\ud574\uc11c\ub3c4 \uc544\ub2c8\uc5c8\ub2e4. \uc790\uc5f0 \ud604\uc0c1\uc5d0 \ub300\ud55c \ud638\uae30\uc2ec\uc5d0 \ub354\ud574\uc11c \uac08\ub9b4\ub808\uc774\ub294 \ub2f9\uc2dc \uc790\uc2e0\uc744 \uc7ac\uc815\uc801, \uc815\uce58\uc801\uc73c\ub85c \ud6c4\uc6d0\ud574 \uc904 \uc218 \uc788\ub294 \uad8c\ub825\uc790\ub4e4\uc744 \uc5fc\ub450\uc5d0 \ub450\uace0 \uae00\uc744 \uc37c\ub2e4. \ubaa9\uc131 \uc8fc\ubcc0\uc744 \ub3c4\ub294 4\uac1c\uc758 \uc704\uc131\uc744 \ub9dd\uc6d0\uacbd\uc73c\ub85c \uad00\ucc30\ud588\uc744 \ub54c, \uadf8\ub294 \uc774\ub4e4\uc744 \u2018\uba54\ub514\uce58\uc758 \ubcc4\ub4e4\u2019\uc774\ub77c\uace0 \ubd80\ub97c \uac83\uc744 \uc81c\uc548\ud558\uae30\ub3c4 \ud558\uc600\ub2e4. \uba54\ub514\uce58\uac00\ub294 \uc774\ub7ec\ud55c \uac08\ub9b4\ub808\uc774\ub97c \uad81\uc815 \uc218\ud559\uc790\ub85c \ucc44\uc6a9\ud558\uc5ec \uc88b\uc740 \uc870\uac74\uc5d0\uc11c \uc5f0\uad6c\uc5d0 \ubab0\ub450\ud558\ub3c4\ub85d \ubc30\ub824\ud558\uc600\ub2e4."} {"question": "\uc774\ud1a0 \ud788\ub85c\ubd80\ubbf8\uac00 \ud53c\uaca9 \uc9c1\ud6c4 \uc62e\uaca8\uc9c4 \uacf3\uc740?", "paragraph": "\uc800\uaca9 30\ubd84\ub9cc\uc778 \uc624\uc804 10\uc2dc\uacbd, \uc774\ud1a0 \ud788\ub85c\ubd80\ubbf8\ub294 \ud53c\uaca9\ub2f9\ud55c \uc9c1\ud6c4 \uc5f4\ucc28\ub85c \uc62e\uaca8\uc84c\ub2e4. \uc8fd\uae30 \uc9c1\uc804\uc5d0 \ube0c\ub79c\ub514(\uc61b\ub0a0\uc5d0\ub294 \uac01\uc131\uc81c\ub85c \uc885\uc885 \uc0ac\uc6a9) \ud55c \ubaa8\uae08 \ub9c8\uc2dc\uace0 \"\ubc94\uc778\uc740 \uc870\uc120\uc778\uc778\uac00?\"\ud558\uace0 \ubb3c\uc5c8\uc73c\uba70, \uc8fc\ubcc0\uc5d0\uc11c \uadf8\ub807\ub2e4\uace0 \ub300\ub2f5\ud558\uc790 \"\ubc14\ubcf4 \uac19\uc740...\"\uc774\ub77c\uace0 \ub1cc\uae4c\ub9ac\uba70 \uc8fd\uc5c8\ub2e4\uace0 \ud55c\ub2e4. \uc774\ub294 \ub2f9\uc2dc \uc0ac\uac74 \ud604\uc7a5\uc5d0 \uc788\ub358 \uc8fc\ubcc0\uc778\ubb3c\ub4e4\uc758 \uc99d\uc5b8\uc5d0 \uc758\ud55c \uac83\uc774\uc9c0\ub9cc \uc870\uc791\uc774\ub77c\ub294 \uc8fc\uc7a5\uc774 \uc788\ub2e4. \ucd1d\uc744 \uc138 \ubc1c\uc774\ub098 \ub9de\uace0 \uadf8\ub7f0 \ub9d0\uc744 \ub0a8\uae38 \uc218 \uc788\ub290\ub0d0\ub294 \uac83\uc774\ub2e4. \ud558\uc9c0\ub9cc \ub2f9\uc2dc \ub3d9\ud589\ud55c \uc758\uc0ac\uc758 \uc99d\uc5b8\uc73c\ub85c\ub294 \ubd84\uba85\ud788 \uc5f4\ucc28 \ub0b4\ub85c \uc62e\uae38 \ub54c\uae4c\uc9c0 \uc0b4\uc544\uc788\uc5c8\ub2e4\uace0 \ud55c\ub2e4. \ub2e4\ub9cc, \ub2e4\ub978 \ucc45\uc5d0\uc11c\ub294 \uc8fd\uc744 \ub2f9\uc2dc\uc5d0 \"\ub09c \ud2c0\ub838\ub2e4... \ub2e4\ub978 \ubd80\uc0c1\uc790\ub294?\"\uc774\ub780 \ub9d0\uc744 \ub0a8\uae30\uace0 \uc8fd\uc5c8\ub2e4\uace0\ub3c4 \uae30\ub85d\ub418\uc5b4 \uc788\uc5b4 \uc804\ubc18\uc801\uc73c\ub85c \uc815\ud655\ud558\uc9c0 \uc54a\uc740 \uac10\uc774 \uc788\uae30 \ub54c\ubb38\uc5d0 \ub354 \ub9ce\uc740 \uc5f0\uad6c\uac00 \ud544\uc694\ud558\ub2e4.", "answer": "\uc5f4\ucc28", "sentence": "\uc800\uaca9 30\ubd84\ub9cc\uc778 \uc624\uc804 10\uc2dc\uacbd, \uc774\ud1a0 \ud788\ub85c\ubd80\ubbf8\ub294 \ud53c\uaca9\ub2f9\ud55c \uc9c1\ud6c4 \uc5f4\ucc28 \ub85c \uc62e\uaca8\uc84c\ub2e4.", "paragraph_sentence": " \uc800\uaca9 30\ubd84\ub9cc\uc778 \uc624\uc804 10\uc2dc\uacbd, \uc774\ud1a0 \ud788\ub85c\ubd80\ubbf8\ub294 \ud53c\uaca9\ub2f9\ud55c \uc9c1\ud6c4 \uc5f4\ucc28 \ub85c \uc62e\uaca8\uc84c\ub2e4. \uc8fd\uae30 \uc9c1\uc804\uc5d0 \ube0c\ub79c\ub514(\uc61b\ub0a0\uc5d0\ub294 \uac01\uc131\uc81c\ub85c \uc885\uc885 \uc0ac\uc6a9) \ud55c \ubaa8\uae08 \ub9c8\uc2dc\uace0 \"\ubc94\uc778\uc740 \uc870\uc120\uc778\uc778\uac00?\"\ud558\uace0 \ubb3c\uc5c8\uc73c\uba70, \uc8fc\ubcc0\uc5d0\uc11c \uadf8\ub807\ub2e4\uace0 \ub300\ub2f5\ud558\uc790 \"\ubc14\ubcf4 \uac19\uc740...\"\uc774\ub77c\uace0 \ub1cc\uae4c\ub9ac\uba70 \uc8fd\uc5c8\ub2e4\uace0 \ud55c\ub2e4. \uc774\ub294 \ub2f9\uc2dc \uc0ac\uac74 \ud604\uc7a5\uc5d0 \uc788\ub358 \uc8fc\ubcc0\uc778\ubb3c\ub4e4\uc758 \uc99d\uc5b8\uc5d0 \uc758\ud55c \uac83\uc774\uc9c0\ub9cc \uc870\uc791\uc774\ub77c\ub294 \uc8fc\uc7a5\uc774 \uc788\ub2e4. \ucd1d\uc744 \uc138 \ubc1c\uc774\ub098 \ub9de\uace0 \uadf8\ub7f0 \ub9d0\uc744 \ub0a8\uae38 \uc218 \uc788\ub290\ub0d0\ub294 \uac83\uc774\ub2e4. \ud558\uc9c0\ub9cc \ub2f9\uc2dc \ub3d9\ud589\ud55c \uc758\uc0ac\uc758 \uc99d\uc5b8\uc73c\ub85c\ub294 \ubd84\uba85\ud788 \uc5f4\ucc28 \ub0b4\ub85c \uc62e\uae38 \ub54c\uae4c\uc9c0 \uc0b4\uc544\uc788\uc5c8\ub2e4\uace0 \ud55c\ub2e4. \ub2e4\ub9cc, \ub2e4\ub978 \ucc45\uc5d0\uc11c\ub294 \uc8fd\uc744 \ub2f9\uc2dc\uc5d0 \"\ub09c \ud2c0\ub838\ub2e4... \ub2e4\ub978 \ubd80\uc0c1\uc790\ub294?\"\uc774\ub780 \ub9d0\uc744 \ub0a8\uae30\uace0 \uc8fd\uc5c8\ub2e4\uace0\ub3c4 \uae30\ub85d\ub418\uc5b4 \uc788\uc5b4 \uc804\ubc18\uc801\uc73c\ub85c \uc815\ud655\ud558\uc9c0 \uc54a\uc740 \uac10\uc774 \uc788\uae30 \ub54c\ubb38\uc5d0 \ub354 \ub9ce\uc740 \uc5f0\uad6c\uac00 \ud544\uc694\ud558\ub2e4.", "paragraph_answer": "\uc800\uaca9 30\ubd84\ub9cc\uc778 \uc624\uc804 10\uc2dc\uacbd, \uc774\ud1a0 \ud788\ub85c\ubd80\ubbf8\ub294 \ud53c\uaca9\ub2f9\ud55c \uc9c1\ud6c4 \uc5f4\ucc28 \ub85c \uc62e\uaca8\uc84c\ub2e4. \uc8fd\uae30 \uc9c1\uc804\uc5d0 \ube0c\ub79c\ub514(\uc61b\ub0a0\uc5d0\ub294 \uac01\uc131\uc81c\ub85c \uc885\uc885 \uc0ac\uc6a9) \ud55c \ubaa8\uae08 \ub9c8\uc2dc\uace0 \"\ubc94\uc778\uc740 \uc870\uc120\uc778\uc778\uac00?\"\ud558\uace0 \ubb3c\uc5c8\uc73c\uba70, \uc8fc\ubcc0\uc5d0\uc11c \uadf8\ub807\ub2e4\uace0 \ub300\ub2f5\ud558\uc790 \"\ubc14\ubcf4 \uac19\uc740...\"\uc774\ub77c\uace0 \ub1cc\uae4c\ub9ac\uba70 \uc8fd\uc5c8\ub2e4\uace0 \ud55c\ub2e4. \uc774\ub294 \ub2f9\uc2dc \uc0ac\uac74 \ud604\uc7a5\uc5d0 \uc788\ub358 \uc8fc\ubcc0\uc778\ubb3c\ub4e4\uc758 \uc99d\uc5b8\uc5d0 \uc758\ud55c \uac83\uc774\uc9c0\ub9cc \uc870\uc791\uc774\ub77c\ub294 \uc8fc\uc7a5\uc774 \uc788\ub2e4. \ucd1d\uc744 \uc138 \ubc1c\uc774\ub098 \ub9de\uace0 \uadf8\ub7f0 \ub9d0\uc744 \ub0a8\uae38 \uc218 \uc788\ub290\ub0d0\ub294 \uac83\uc774\ub2e4. \ud558\uc9c0\ub9cc \ub2f9\uc2dc \ub3d9\ud589\ud55c \uc758\uc0ac\uc758 \uc99d\uc5b8\uc73c\ub85c\ub294 \ubd84\uba85\ud788 \uc5f4\ucc28 \ub0b4\ub85c \uc62e\uae38 \ub54c\uae4c\uc9c0 \uc0b4\uc544\uc788\uc5c8\ub2e4\uace0 \ud55c\ub2e4. \ub2e4\ub9cc, \ub2e4\ub978 \ucc45\uc5d0\uc11c\ub294 \uc8fd\uc744 \ub2f9\uc2dc\uc5d0 \"\ub09c \ud2c0\ub838\ub2e4... \ub2e4\ub978 \ubd80\uc0c1\uc790\ub294?\"\uc774\ub780 \ub9d0\uc744 \ub0a8\uae30\uace0 \uc8fd\uc5c8\ub2e4\uace0\ub3c4 \uae30\ub85d\ub418\uc5b4 \uc788\uc5b4 \uc804\ubc18\uc801\uc73c\ub85c \uc815\ud655\ud558\uc9c0 \uc54a\uc740 \uac10\uc774 \uc788\uae30 \ub54c\ubb38\uc5d0 \ub354 \ub9ce\uc740 \uc5f0\uad6c\uac00 \ud544\uc694\ud558\ub2e4.", "sentence_answer": "\uc800\uaca9 30\ubd84\ub9cc\uc778 \uc624\uc804 10\uc2dc\uacbd, \uc774\ud1a0 \ud788\ub85c\ubd80\ubbf8\ub294 \ud53c\uaca9\ub2f9\ud55c \uc9c1\ud6c4 \uc5f4\ucc28 \ub85c \uc62e\uaca8\uc84c\ub2e4.", "paragraph_id": "6538987-8-0", "paragraph_question": "question: \uc774\ud1a0 \ud788\ub85c\ubd80\ubbf8\uac00 \ud53c\uaca9 \uc9c1\ud6c4 \uc62e\uaca8\uc9c4 \uacf3\uc740?, context: \uc800\uaca9 30\ubd84\ub9cc\uc778 \uc624\uc804 10\uc2dc\uacbd, \uc774\ud1a0 \ud788\ub85c\ubd80\ubbf8\ub294 \ud53c\uaca9\ub2f9\ud55c \uc9c1\ud6c4 \uc5f4\ucc28\ub85c \uc62e\uaca8\uc84c\ub2e4. \uc8fd\uae30 \uc9c1\uc804\uc5d0 \ube0c\ub79c\ub514(\uc61b\ub0a0\uc5d0\ub294 \uac01\uc131\uc81c\ub85c \uc885\uc885 \uc0ac\uc6a9) \ud55c \ubaa8\uae08 \ub9c8\uc2dc\uace0 \"\ubc94\uc778\uc740 \uc870\uc120\uc778\uc778\uac00?\"\ud558\uace0 \ubb3c\uc5c8\uc73c\uba70, \uc8fc\ubcc0\uc5d0\uc11c \uadf8\ub807\ub2e4\uace0 \ub300\ub2f5\ud558\uc790 \"\ubc14\ubcf4 \uac19\uc740...\"\uc774\ub77c\uace0 \ub1cc\uae4c\ub9ac\uba70 \uc8fd\uc5c8\ub2e4\uace0 \ud55c\ub2e4. \uc774\ub294 \ub2f9\uc2dc \uc0ac\uac74 \ud604\uc7a5\uc5d0 \uc788\ub358 \uc8fc\ubcc0\uc778\ubb3c\ub4e4\uc758 \uc99d\uc5b8\uc5d0 \uc758\ud55c \uac83\uc774\uc9c0\ub9cc \uc870\uc791\uc774\ub77c\ub294 \uc8fc\uc7a5\uc774 \uc788\ub2e4. \ucd1d\uc744 \uc138 \ubc1c\uc774\ub098 \ub9de\uace0 \uadf8\ub7f0 \ub9d0\uc744 \ub0a8\uae38 \uc218 \uc788\ub290\ub0d0\ub294 \uac83\uc774\ub2e4. \ud558\uc9c0\ub9cc \ub2f9\uc2dc \ub3d9\ud589\ud55c \uc758\uc0ac\uc758 \uc99d\uc5b8\uc73c\ub85c\ub294 \ubd84\uba85\ud788 \uc5f4\ucc28 \ub0b4\ub85c \uc62e\uae38 \ub54c\uae4c\uc9c0 \uc0b4\uc544\uc788\uc5c8\ub2e4\uace0 \ud55c\ub2e4. \ub2e4\ub9cc, \ub2e4\ub978 \ucc45\uc5d0\uc11c\ub294 \uc8fd\uc744 \ub2f9\uc2dc\uc5d0 \"\ub09c \ud2c0\ub838\ub2e4... \ub2e4\ub978 \ubd80\uc0c1\uc790\ub294?\"\uc774\ub780 \ub9d0\uc744 \ub0a8\uae30\uace0 \uc8fd\uc5c8\ub2e4\uace0\ub3c4 \uae30\ub85d\ub418\uc5b4 \uc788\uc5b4 \uc804\ubc18\uc801\uc73c\ub85c \uc815\ud655\ud558\uc9c0 \uc54a\uc740 \uac10\uc774 \uc788\uae30 \ub54c\ubb38\uc5d0 \ub354 \ub9ce\uc740 \uc5f0\uad6c\uac00 \ud544\uc694\ud558\ub2e4."} {"question": "\ub85c\uc774\ud0b4\uc744 \uc288\ud37c\uc2a4\ud0c0K4\uc758 3\ucc28\uc608\uc120\uc5d0\uc11c \ubd88\ud569\uaca9\uc2dc\ud0a8 \uc2ec\uc0ac\uc704\uc6d0\uc740?", "paragraph": "2012\ub144 \ub300\ud559 \uc785\ud559 \uc804 Gap year\ub97c \uc774\uc6a9\ud558\uc5ec \uc288\ud37c\uc2a4\ud0c0K4\uc5d0 \ubcf8\uba85 \uae40\uc0c1\uc6b0\ub85c \ucc38\uac00\ud588\ub358 \ub85c\uc774\ud0b4\uc740 3\ucc28 \uc608\uc120\uc5d0\uc11c \ub2f9\uc2dc \uc2ec\uc0ac\uc704\uc6d0\uc774\uc5c8\ub358 \uc774\uc2b9\ucca0, \ubc31\uc9c0\uc601\uc5d0\uac8c \ubd88\ud569\uaca9\uc744 \ubc1b\uc558\uc9c0\ub9cc \ub610\ub2e4\ub978 \uc2ec\uc0ac\uc704\uc6d0 \uc774\ud558\ub298\uc758 \uc288\ud37c\ud328\uc2a4\ub85c \uc288\ud37c\uc704\ud06c\uc5d0 \uc9c4\ucd9c\ud558\uac8c \ub41c\ub2e4. \uc288\ud37c\uc704\ud06c \ub77c\uc774\ubc8c \ubbf8\uc158\uc5d0\uc11c \uc815\uc900\uc601\uacfc \ud568\uaed8 \uae40\uad11\uc11d\uc758 \u2018\uba3c\uc9c0\uac00 \ub418\uc5b4\u2019\ub97c \uc18c\ud654\ud574 \ud070 \uc778\uae30\ub97c \uc5bb\uc5c8\uc73c\uba70, TOP 12 \ud569\uaca9 \ud6c4 \ub9e4\uc8fc \uc0dd\ubc29\uc1a1 \ubb34\ub300\uc5d0\uc11c \ub2e4\uc591\ud55c \ubb34\ub300\uc640 \uc120\uace1\uc744 \uc120\ubcf4\uc600\ub2e4. \ud2b9\ud788 \uc900\uacb0\uc2b9\uc804\uc5d0\uc11c \ubd80\ub978 \uc724\uac74\uc758 '\ud790\ub9c1\uc774 \ud544\uc694\ud574'\ub294\u300a\uc288\ud37c\uc2a4\ud0c0K 4 \uc0dd\ubc29\uc1a1 \ubbf8\uc158\uace1\ub4e4 \uc911 \uc720\uc77c\ud558\uac8c \ub300\ud55c\ubbfc\uad6d \uc74c\uc6d0 \uc804 \ucc28\ud2b8\uc5d0\uc11c \uc2e4\uc2dc\uac04 1\uc704\ub97c \uae30\ub85d\ud558\uba70 \ub85c\uc774\ud0b4\uc740 \uc288\ud37c\uc2a4\ud0c0K\u300b\uc0ac\uc0c1 \ucd5c\uace0\uc758 \uacbd\uc7c1\ub960\uc778 2,083,447\uba85\uc758 \ucc38\uac00\uc2e0\uccad\uc790\ub97c \uc81c\uce58\uace0\u300a\uc288\ud37c\uc2a4\ud0c0K 4\u300b\uc758 \uc6b0\uc2b9\uc790\uac00 \ub418\uc5c8\ub2e4.", "answer": "\uc774\uc2b9\ucca0, \ubc31\uc9c0\uc601", "sentence": "2012\ub144 \ub300\ud559 \uc785\ud559 \uc804 Gap year\ub97c \uc774\uc6a9\ud558\uc5ec \uc288\ud37c\uc2a4\ud0c0K4\uc5d0 \ubcf8\uba85 \uae40\uc0c1\uc6b0\ub85c \ucc38\uac00\ud588\ub358 \ub85c\uc774\ud0b4\uc740 3\ucc28 \uc608\uc120\uc5d0\uc11c \ub2f9\uc2dc \uc2ec\uc0ac\uc704\uc6d0\uc774\uc5c8\ub358 \uc774\uc2b9\ucca0, \ubc31\uc9c0\uc601 \uc5d0\uac8c \ubd88\ud569\uaca9\uc744 \ubc1b\uc558\uc9c0\ub9cc \ub610\ub2e4\ub978 \uc2ec\uc0ac\uc704\uc6d0 \uc774\ud558\ub298\uc758 \uc288\ud37c\ud328\uc2a4\ub85c \uc288\ud37c\uc704\ud06c\uc5d0 \uc9c4\ucd9c\ud558\uac8c \ub41c\ub2e4.", "paragraph_sentence": " 2012\ub144 \ub300\ud559 \uc785\ud559 \uc804 Gap year\ub97c \uc774\uc6a9\ud558\uc5ec \uc288\ud37c\uc2a4\ud0c0K4\uc5d0 \ubcf8\uba85 \uae40\uc0c1\uc6b0\ub85c \ucc38\uac00\ud588\ub358 \ub85c\uc774\ud0b4\uc740 3\ucc28 \uc608\uc120\uc5d0\uc11c \ub2f9\uc2dc \uc2ec\uc0ac\uc704\uc6d0\uc774\uc5c8\ub358 \uc774\uc2b9\ucca0, \ubc31\uc9c0\uc601 \uc5d0\uac8c \ubd88\ud569\uaca9\uc744 \ubc1b\uc558\uc9c0\ub9cc \ub610\ub2e4\ub978 \uc2ec\uc0ac\uc704\uc6d0 \uc774\ud558\ub298\uc758 \uc288\ud37c\ud328\uc2a4\ub85c \uc288\ud37c\uc704\ud06c\uc5d0 \uc9c4\ucd9c\ud558\uac8c \ub41c\ub2e4. \uc288\ud37c\uc704\ud06c \ub77c\uc774\ubc8c \ubbf8\uc158\uc5d0\uc11c \uc815\uc900\uc601\uacfc \ud568\uaed8 \uae40\uad11\uc11d\uc758 \u2018\uba3c\uc9c0\uac00 \ub418\uc5b4\u2019\ub97c \uc18c\ud654\ud574 \ud070 \uc778\uae30\ub97c \uc5bb\uc5c8\uc73c\uba70, TOP 12 \ud569\uaca9 \ud6c4 \ub9e4\uc8fc \uc0dd\ubc29\uc1a1 \ubb34\ub300\uc5d0\uc11c \ub2e4\uc591\ud55c \ubb34\ub300\uc640 \uc120\uace1\uc744 \uc120\ubcf4\uc600\ub2e4. \ud2b9\ud788 \uc900\uacb0\uc2b9\uc804\uc5d0\uc11c \ubd80\ub978 \uc724\uac74\uc758 '\ud790\ub9c1\uc774 \ud544\uc694\ud574'\ub294\u300a\uc288\ud37c\uc2a4\ud0c0K 4 \uc0dd\ubc29\uc1a1 \ubbf8\uc158\uace1\ub4e4 \uc911 \uc720\uc77c\ud558\uac8c \ub300\ud55c\ubbfc\uad6d \uc74c\uc6d0 \uc804 \ucc28\ud2b8\uc5d0\uc11c \uc2e4\uc2dc\uac04 1\uc704\ub97c \uae30\ub85d\ud558\uba70 \ub85c\uc774\ud0b4\uc740 \uc288\ud37c\uc2a4\ud0c0K\u300b\uc0ac\uc0c1 \ucd5c\uace0\uc758 \uacbd\uc7c1\ub960\uc778 2,083,447\uba85\uc758 \ucc38\uac00\uc2e0\uccad\uc790\ub97c \uc81c\uce58\uace0\u300a\uc288\ud37c\uc2a4\ud0c0K 4\u300b\uc758 \uc6b0\uc2b9\uc790\uac00 \ub418\uc5c8\ub2e4.", "paragraph_answer": "2012\ub144 \ub300\ud559 \uc785\ud559 \uc804 Gap year\ub97c \uc774\uc6a9\ud558\uc5ec \uc288\ud37c\uc2a4\ud0c0K4\uc5d0 \ubcf8\uba85 \uae40\uc0c1\uc6b0\ub85c \ucc38\uac00\ud588\ub358 \ub85c\uc774\ud0b4\uc740 3\ucc28 \uc608\uc120\uc5d0\uc11c \ub2f9\uc2dc \uc2ec\uc0ac\uc704\uc6d0\uc774\uc5c8\ub358 \uc774\uc2b9\ucca0, \ubc31\uc9c0\uc601 \uc5d0\uac8c \ubd88\ud569\uaca9\uc744 \ubc1b\uc558\uc9c0\ub9cc \ub610\ub2e4\ub978 \uc2ec\uc0ac\uc704\uc6d0 \uc774\ud558\ub298\uc758 \uc288\ud37c\ud328\uc2a4\ub85c \uc288\ud37c\uc704\ud06c\uc5d0 \uc9c4\ucd9c\ud558\uac8c \ub41c\ub2e4. \uc288\ud37c\uc704\ud06c \ub77c\uc774\ubc8c \ubbf8\uc158\uc5d0\uc11c \uc815\uc900\uc601\uacfc \ud568\uaed8 \uae40\uad11\uc11d\uc758 \u2018\uba3c\uc9c0\uac00 \ub418\uc5b4\u2019\ub97c \uc18c\ud654\ud574 \ud070 \uc778\uae30\ub97c \uc5bb\uc5c8\uc73c\uba70, TOP 12 \ud569\uaca9 \ud6c4 \ub9e4\uc8fc \uc0dd\ubc29\uc1a1 \ubb34\ub300\uc5d0\uc11c \ub2e4\uc591\ud55c \ubb34\ub300\uc640 \uc120\uace1\uc744 \uc120\ubcf4\uc600\ub2e4. \ud2b9\ud788 \uc900\uacb0\uc2b9\uc804\uc5d0\uc11c \ubd80\ub978 \uc724\uac74\uc758 '\ud790\ub9c1\uc774 \ud544\uc694\ud574'\ub294\u300a\uc288\ud37c\uc2a4\ud0c0K 4 \uc0dd\ubc29\uc1a1 \ubbf8\uc158\uace1\ub4e4 \uc911 \uc720\uc77c\ud558\uac8c \ub300\ud55c\ubbfc\uad6d \uc74c\uc6d0 \uc804 \ucc28\ud2b8\uc5d0\uc11c \uc2e4\uc2dc\uac04 1\uc704\ub97c \uae30\ub85d\ud558\uba70 \ub85c\uc774\ud0b4\uc740 \uc288\ud37c\uc2a4\ud0c0K\u300b\uc0ac\uc0c1 \ucd5c\uace0\uc758 \uacbd\uc7c1\ub960\uc778 2,083,447\uba85\uc758 \ucc38\uac00\uc2e0\uccad\uc790\ub97c \uc81c\uce58\uace0\u300a\uc288\ud37c\uc2a4\ud0c0K 4\u300b\uc758 \uc6b0\uc2b9\uc790\uac00 \ub418\uc5c8\ub2e4.", "sentence_answer": "2012\ub144 \ub300\ud559 \uc785\ud559 \uc804 Gap year\ub97c \uc774\uc6a9\ud558\uc5ec \uc288\ud37c\uc2a4\ud0c0K4\uc5d0 \ubcf8\uba85 \uae40\uc0c1\uc6b0\ub85c \ucc38\uac00\ud588\ub358 \ub85c\uc774\ud0b4\uc740 3\ucc28 \uc608\uc120\uc5d0\uc11c \ub2f9\uc2dc \uc2ec\uc0ac\uc704\uc6d0\uc774\uc5c8\ub358 \uc774\uc2b9\ucca0, \ubc31\uc9c0\uc601 \uc5d0\uac8c \ubd88\ud569\uaca9\uc744 \ubc1b\uc558\uc9c0\ub9cc \ub610\ub2e4\ub978 \uc2ec\uc0ac\uc704\uc6d0 \uc774\ud558\ub298\uc758 \uc288\ud37c\ud328\uc2a4\ub85c \uc288\ud37c\uc704\ud06c\uc5d0 \uc9c4\ucd9c\ud558\uac8c \ub41c\ub2e4.", "paragraph_id": "6056504-1-2", "paragraph_question": "question: \ub85c\uc774\ud0b4\uc744 \uc288\ud37c\uc2a4\ud0c0K4\uc758 3\ucc28\uc608\uc120\uc5d0\uc11c \ubd88\ud569\uaca9\uc2dc\ud0a8 \uc2ec\uc0ac\uc704\uc6d0\uc740?, context: 2012\ub144 \ub300\ud559 \uc785\ud559 \uc804 Gap year\ub97c \uc774\uc6a9\ud558\uc5ec \uc288\ud37c\uc2a4\ud0c0K4\uc5d0 \ubcf8\uba85 \uae40\uc0c1\uc6b0\ub85c \ucc38\uac00\ud588\ub358 \ub85c\uc774\ud0b4\uc740 3\ucc28 \uc608\uc120\uc5d0\uc11c \ub2f9\uc2dc \uc2ec\uc0ac\uc704\uc6d0\uc774\uc5c8\ub358 \uc774\uc2b9\ucca0, \ubc31\uc9c0\uc601\uc5d0\uac8c \ubd88\ud569\uaca9\uc744 \ubc1b\uc558\uc9c0\ub9cc \ub610\ub2e4\ub978 \uc2ec\uc0ac\uc704\uc6d0 \uc774\ud558\ub298\uc758 \uc288\ud37c\ud328\uc2a4\ub85c \uc288\ud37c\uc704\ud06c\uc5d0 \uc9c4\ucd9c\ud558\uac8c \ub41c\ub2e4. \uc288\ud37c\uc704\ud06c \ub77c\uc774\ubc8c \ubbf8\uc158\uc5d0\uc11c \uc815\uc900\uc601\uacfc \ud568\uaed8 \uae40\uad11\uc11d\uc758 \u2018\uba3c\uc9c0\uac00 \ub418\uc5b4\u2019\ub97c \uc18c\ud654\ud574 \ud070 \uc778\uae30\ub97c \uc5bb\uc5c8\uc73c\uba70, TOP 12 \ud569\uaca9 \ud6c4 \ub9e4\uc8fc \uc0dd\ubc29\uc1a1 \ubb34\ub300\uc5d0\uc11c \ub2e4\uc591\ud55c \ubb34\ub300\uc640 \uc120\uace1\uc744 \uc120\ubcf4\uc600\ub2e4. \ud2b9\ud788 \uc900\uacb0\uc2b9\uc804\uc5d0\uc11c \ubd80\ub978 \uc724\uac74\uc758 '\ud790\ub9c1\uc774 \ud544\uc694\ud574'\ub294\u300a\uc288\ud37c\uc2a4\ud0c0K 4 \uc0dd\ubc29\uc1a1 \ubbf8\uc158\uace1\ub4e4 \uc911 \uc720\uc77c\ud558\uac8c \ub300\ud55c\ubbfc\uad6d \uc74c\uc6d0 \uc804 \ucc28\ud2b8\uc5d0\uc11c \uc2e4\uc2dc\uac04 1\uc704\ub97c \uae30\ub85d\ud558\uba70 \ub85c\uc774\ud0b4\uc740 \uc288\ud37c\uc2a4\ud0c0K\u300b\uc0ac\uc0c1 \ucd5c\uace0\uc758 \uacbd\uc7c1\ub960\uc778 2,083,447\uba85\uc758 \ucc38\uac00\uc2e0\uccad\uc790\ub97c \uc81c\uce58\uace0\u300a\uc288\ud37c\uc2a4\ud0c0K 4\u300b\uc758 \uc6b0\uc2b9\uc790\uac00 \ub418\uc5c8\ub2e4."} {"question": "\uace0\ubcc4\uae30\uc790\ud68c\uacac\uc5d0\uc11c \uae40\ub300\uc911\uc744 \ud06c\uac8c \uce6d\ucc2c\ud55c IMF \ucd1d\uc7ac\ub294?", "paragraph": "\uc81c\uc784\uc2a4 \uc6b8\ud380\uc2a8 \uc804 \uc138\uacc4 \uc740\ud589 \ucd1d\uc7ac\ub294 \uae40\ub300\uc911\uc5d0 \ub300\ud574 \"\uc5c4\uccad\ub09c \ud0c4\uc555\uc5d0\ub3c4 \uc88c\uc808\ud558\uc9c0 \uc54a\uace0 \ub9c8\uce68\ub0b4 \uc790\uc2e0\uc758 \ub73b\uc744 \uc774\ub904\ub0b8 \uae40\ub300\ud1b5\ub839\uc740 \ub9cc\ub378\ub77c\uc758 \uc0b6\uc744 \uc5f0\uc0c1\uc2dc\ud0b5\ub2c8\ub2e4. \ubbfc\uc8fc\ud654\ubfd0 \uc544\ub2c8\ub77c \uacbd\uc81c\uc758 \uc7ac\uad6c\ucd95\uc5d0 \uc131\uacf5\ud558\uace0 \uc788\ub294 \uae40\ub300\ud1b5\ub839\uc740 \uc138\uacc4\ub97c \uac10\ub3d9\uc2dc\ud0a4\ub294 \uc9c0\ub3c4\uc790 \uc785\ub2c8\ub2e4\"\ub77c\uace0 \ud3c9\uac00\ud588\ub2e4. \ubbf8\uc178 \uce89\ub4dc\uc26c \uc804 IMF \ucd1d\uc7ac\ub294 \uace0\ubcc4\uae30\uc790 \ud68c\uacac\uc5d0\uc11c \"\ud55c\uad6d\uc740 \uae40\ub300\uc911\uc528\ub97c \ub300\ud1b5\ub839\uc73c\ub85c \ub450\uace0 \uc788\ub2e4\ub294 \uc790\uccb4\ub97c \ud589\uc6b4\uc73c\ub85c \ubc1b\uc544\ub4e4\uc5ec\uc57c \ud560 \uac83\uc774\ub2e4. \uacbd\uc81c\uc704\uae30\ub97c \ub9de\uc544 \uba87\uac00\uc9c0 \uace0\ube44\uac00 \uc788\uc5c8\uc9c0\ub9cc IMF\uc640 \ubbf8\uad6d \uadf8\ub9ac\uace0 \uad6d\uc81c\uc0ac\ud68c \uc804\ubc18\uc740 \uae40 \ub300\ud1b5\ub839\uc744 \ubbff\uace0 \ud55c\uad6d\uc744 \ubc00\uc5b4\uc900 \uce21\uba74\uc774 \uc801\uc9c0 \uc54a\uc740\uac8c \uc0ac\uc2e4\uc774\ub2e4\"\ub77c\uace0 \ubc1d\ud614\ub2e4. '\ub808\uc774\uac74 \uc2e0\ud654'\uc758 \uc8fc\uc5ed \ub9c8\uc774\ud06c \ub514\ubc84 \uc804 \ub300\ud1b5\ub839 \ud2b9\ubcf4\ub294 \"\uae40 \ub300\ud1b5\ub839\uc740 \uba54\uc774\ud06c \uc5c5\uc774 \ud544\uc694 \uc5c6\ub294 \ub17c\ud53d\uc158 \ub4dc\ub77c\ub9c8\uc758 \uc778\uc0dd\uc744 \uc0b4\uc544\uc654\ub2e4\"\uba70 \"\uc774\ub984 \uc11d\uc790\ub9cc \uac16\uace0\ub3c4 \uad6d\uc81c\uc0ac\ud68c\uc5d0 \uc601\ud5a5\uc744\uc904 \uc218 \ube0c\ub79c\ub4dc\"\ub77c\uace0 \uadf9\ucc2c\ud588\ub2e4. \ubbf8\uad6d \ub9dd\uba85 \uc2dc\uc808\uc5d0 \ud558\ubc84\ub4dc \ub300\ud559\uad50\uc5d0\uc11c \ud3b4\ub0b8 \u300a\ub300\uc911\uacbd\uc81c\ub860\u300b\uacfc \u300a3\ub2e8\uacc4\ud1b5\uc77c\ub860\u300b\ub294 \ud558\ubc84\ub4dc \ub300\ud559\uad50\uc758 \uad50\uc7ac\uc6a9\uc73c\ub85c \ubc88\uc5ed \ucd9c\uac04\ub418\uc5c8\ub2e4.", "answer": "\ubbf8\uc178 \uce89\ub4dc\uc26c", "sentence": "\ubbf8\uc178 \uce89\ub4dc\uc26c \uc804 IMF \ucd1d\uc7ac\ub294 \uace0\ubcc4\uae30\uc790 \ud68c\uacac\uc5d0\uc11c \"\ud55c\uad6d\uc740 \uae40\ub300\uc911\uc528\ub97c \ub300\ud1b5\ub839\uc73c\ub85c \ub450\uace0 \uc788\ub2e4\ub294 \uc790\uccb4\ub97c \ud589\uc6b4\uc73c\ub85c \ubc1b\uc544\ub4e4\uc5ec\uc57c \ud560 \uac83\uc774\ub2e4.", "paragraph_sentence": "\uc81c\uc784\uc2a4 \uc6b8\ud380\uc2a8 \uc804 \uc138\uacc4 \uc740\ud589 \ucd1d\uc7ac\ub294 \uae40\ub300\uc911\uc5d0 \ub300\ud574 \"\uc5c4\uccad\ub09c \ud0c4\uc555\uc5d0\ub3c4 \uc88c\uc808\ud558\uc9c0 \uc54a\uace0 \ub9c8\uce68\ub0b4 \uc790\uc2e0\uc758 \ub73b\uc744 \uc774\ub904\ub0b8 \uae40\ub300\ud1b5\ub839\uc740 \ub9cc\ub378\ub77c\uc758 \uc0b6\uc744 \uc5f0\uc0c1\uc2dc\ud0b5\ub2c8\ub2e4. \ubbfc\uc8fc\ud654\ubfd0 \uc544\ub2c8\ub77c \uacbd\uc81c\uc758 \uc7ac\uad6c\ucd95\uc5d0 \uc131\uacf5\ud558\uace0 \uc788\ub294 \uae40\ub300\ud1b5\ub839\uc740 \uc138\uacc4\ub97c \uac10\ub3d9\uc2dc\ud0a4\ub294 \uc9c0\ub3c4\uc790 \uc785\ub2c8\ub2e4\"\ub77c\uace0 \ud3c9\uac00\ud588\ub2e4. \ubbf8\uc178 \uce89\ub4dc\uc26c \uc804 IMF \ucd1d\uc7ac\ub294 \uace0\ubcc4\uae30\uc790 \ud68c\uacac\uc5d0\uc11c \"\ud55c\uad6d\uc740 \uae40\ub300\uc911\uc528\ub97c \ub300\ud1b5\ub839\uc73c\ub85c \ub450\uace0 \uc788\ub2e4\ub294 \uc790\uccb4\ub97c \ud589\uc6b4\uc73c\ub85c \ubc1b\uc544\ub4e4\uc5ec\uc57c \ud560 \uac83\uc774\ub2e4. \uacbd\uc81c\uc704\uae30\ub97c \ub9de\uc544 \uba87\uac00\uc9c0 \uace0\ube44\uac00 \uc788\uc5c8\uc9c0\ub9cc IMF\uc640 \ubbf8\uad6d \uadf8\ub9ac\uace0 \uad6d\uc81c\uc0ac\ud68c \uc804\ubc18\uc740 \uae40 \ub300\ud1b5\ub839\uc744 \ubbff\uace0 \ud55c\uad6d\uc744 \ubc00\uc5b4\uc900 \uce21\uba74\uc774 \uc801\uc9c0 \uc54a\uc740\uac8c \uc0ac\uc2e4\uc774\ub2e4\"\ub77c\uace0 \ubc1d\ud614\ub2e4. '\ub808\uc774\uac74 \uc2e0\ud654'\uc758 \uc8fc\uc5ed \ub9c8\uc774\ud06c \ub514\ubc84 \uc804 \ub300\ud1b5\ub839 \ud2b9\ubcf4\ub294 \"\uae40 \ub300\ud1b5\ub839\uc740 \uba54\uc774\ud06c \uc5c5\uc774 \ud544\uc694 \uc5c6\ub294 \ub17c\ud53d\uc158 \ub4dc\ub77c\ub9c8\uc758 \uc778\uc0dd\uc744 \uc0b4\uc544\uc654\ub2e4\"\uba70 \"\uc774\ub984 \uc11d\uc790\ub9cc \uac16\uace0\ub3c4 \uad6d\uc81c\uc0ac\ud68c\uc5d0 \uc601\ud5a5\uc744\uc904 \uc218 \ube0c\ub79c\ub4dc\"\ub77c\uace0 \uadf9\ucc2c\ud588\ub2e4. \ubbf8\uad6d \ub9dd\uba85 \uc2dc\uc808\uc5d0 \ud558\ubc84\ub4dc \ub300\ud559\uad50\uc5d0\uc11c \ud3b4\ub0b8 \u300a\ub300\uc911\uacbd\uc81c\ub860\u300b\uacfc \u300a3\ub2e8\uacc4\ud1b5\uc77c\ub860\u300b\ub294 \ud558\ubc84\ub4dc \ub300\ud559\uad50\uc758 \uad50\uc7ac\uc6a9\uc73c\ub85c \ubc88\uc5ed \ucd9c\uac04\ub418\uc5c8\ub2e4.", "paragraph_answer": "\uc81c\uc784\uc2a4 \uc6b8\ud380\uc2a8 \uc804 \uc138\uacc4 \uc740\ud589 \ucd1d\uc7ac\ub294 \uae40\ub300\uc911\uc5d0 \ub300\ud574 \"\uc5c4\uccad\ub09c \ud0c4\uc555\uc5d0\ub3c4 \uc88c\uc808\ud558\uc9c0 \uc54a\uace0 \ub9c8\uce68\ub0b4 \uc790\uc2e0\uc758 \ub73b\uc744 \uc774\ub904\ub0b8 \uae40\ub300\ud1b5\ub839\uc740 \ub9cc\ub378\ub77c\uc758 \uc0b6\uc744 \uc5f0\uc0c1\uc2dc\ud0b5\ub2c8\ub2e4. \ubbfc\uc8fc\ud654\ubfd0 \uc544\ub2c8\ub77c \uacbd\uc81c\uc758 \uc7ac\uad6c\ucd95\uc5d0 \uc131\uacf5\ud558\uace0 \uc788\ub294 \uae40\ub300\ud1b5\ub839\uc740 \uc138\uacc4\ub97c \uac10\ub3d9\uc2dc\ud0a4\ub294 \uc9c0\ub3c4\uc790 \uc785\ub2c8\ub2e4\"\ub77c\uace0 \ud3c9\uac00\ud588\ub2e4. \ubbf8\uc178 \uce89\ub4dc\uc26c \uc804 IMF \ucd1d\uc7ac\ub294 \uace0\ubcc4\uae30\uc790 \ud68c\uacac\uc5d0\uc11c \"\ud55c\uad6d\uc740 \uae40\ub300\uc911\uc528\ub97c \ub300\ud1b5\ub839\uc73c\ub85c \ub450\uace0 \uc788\ub2e4\ub294 \uc790\uccb4\ub97c \ud589\uc6b4\uc73c\ub85c \ubc1b\uc544\ub4e4\uc5ec\uc57c \ud560 \uac83\uc774\ub2e4. \uacbd\uc81c\uc704\uae30\ub97c \ub9de\uc544 \uba87\uac00\uc9c0 \uace0\ube44\uac00 \uc788\uc5c8\uc9c0\ub9cc IMF\uc640 \ubbf8\uad6d \uadf8\ub9ac\uace0 \uad6d\uc81c\uc0ac\ud68c \uc804\ubc18\uc740 \uae40 \ub300\ud1b5\ub839\uc744 \ubbff\uace0 \ud55c\uad6d\uc744 \ubc00\uc5b4\uc900 \uce21\uba74\uc774 \uc801\uc9c0 \uc54a\uc740\uac8c \uc0ac\uc2e4\uc774\ub2e4\"\ub77c\uace0 \ubc1d\ud614\ub2e4. '\ub808\uc774\uac74 \uc2e0\ud654'\uc758 \uc8fc\uc5ed \ub9c8\uc774\ud06c \ub514\ubc84 \uc804 \ub300\ud1b5\ub839 \ud2b9\ubcf4\ub294 \"\uae40 \ub300\ud1b5\ub839\uc740 \uba54\uc774\ud06c \uc5c5\uc774 \ud544\uc694 \uc5c6\ub294 \ub17c\ud53d\uc158 \ub4dc\ub77c\ub9c8\uc758 \uc778\uc0dd\uc744 \uc0b4\uc544\uc654\ub2e4\"\uba70 \"\uc774\ub984 \uc11d\uc790\ub9cc \uac16\uace0\ub3c4 \uad6d\uc81c\uc0ac\ud68c\uc5d0 \uc601\ud5a5\uc744\uc904 \uc218 \ube0c\ub79c\ub4dc\"\ub77c\uace0 \uadf9\ucc2c\ud588\ub2e4. \ubbf8\uad6d \ub9dd\uba85 \uc2dc\uc808\uc5d0 \ud558\ubc84\ub4dc \ub300\ud559\uad50\uc5d0\uc11c \ud3b4\ub0b8 \u300a\ub300\uc911\uacbd\uc81c\ub860\u300b\uacfc \u300a3\ub2e8\uacc4\ud1b5\uc77c\ub860\u300b\ub294 \ud558\ubc84\ub4dc \ub300\ud559\uad50\uc758 \uad50\uc7ac\uc6a9\uc73c\ub85c \ubc88\uc5ed \ucd9c\uac04\ub418\uc5c8\ub2e4.", "sentence_answer": " \ubbf8\uc178 \uce89\ub4dc\uc26c \uc804 IMF \ucd1d\uc7ac\ub294 \uace0\ubcc4\uae30\uc790 \ud68c\uacac\uc5d0\uc11c \"\ud55c\uad6d\uc740 \uae40\ub300\uc911\uc528\ub97c \ub300\ud1b5\ub839\uc73c\ub85c \ub450\uace0 \uc788\ub2e4\ub294 \uc790\uccb4\ub97c \ud589\uc6b4\uc73c\ub85c \ubc1b\uc544\ub4e4\uc5ec\uc57c \ud560 \uac83\uc774\ub2e4.", "paragraph_id": "6488203-62-0", "paragraph_question": "question: \uace0\ubcc4\uae30\uc790\ud68c\uacac\uc5d0\uc11c \uae40\ub300\uc911\uc744 \ud06c\uac8c \uce6d\ucc2c\ud55c IMF \ucd1d\uc7ac\ub294?, context: \uc81c\uc784\uc2a4 \uc6b8\ud380\uc2a8 \uc804 \uc138\uacc4 \uc740\ud589 \ucd1d\uc7ac\ub294 \uae40\ub300\uc911\uc5d0 \ub300\ud574 \"\uc5c4\uccad\ub09c \ud0c4\uc555\uc5d0\ub3c4 \uc88c\uc808\ud558\uc9c0 \uc54a\uace0 \ub9c8\uce68\ub0b4 \uc790\uc2e0\uc758 \ub73b\uc744 \uc774\ub904\ub0b8 \uae40\ub300\ud1b5\ub839\uc740 \ub9cc\ub378\ub77c\uc758 \uc0b6\uc744 \uc5f0\uc0c1\uc2dc\ud0b5\ub2c8\ub2e4. \ubbfc\uc8fc\ud654\ubfd0 \uc544\ub2c8\ub77c \uacbd\uc81c\uc758 \uc7ac\uad6c\ucd95\uc5d0 \uc131\uacf5\ud558\uace0 \uc788\ub294 \uae40\ub300\ud1b5\ub839\uc740 \uc138\uacc4\ub97c \uac10\ub3d9\uc2dc\ud0a4\ub294 \uc9c0\ub3c4\uc790 \uc785\ub2c8\ub2e4\"\ub77c\uace0 \ud3c9\uac00\ud588\ub2e4. \ubbf8\uc178 \uce89\ub4dc\uc26c \uc804 IMF \ucd1d\uc7ac\ub294 \uace0\ubcc4\uae30\uc790 \ud68c\uacac\uc5d0\uc11c \"\ud55c\uad6d\uc740 \uae40\ub300\uc911\uc528\ub97c \ub300\ud1b5\ub839\uc73c\ub85c \ub450\uace0 \uc788\ub2e4\ub294 \uc790\uccb4\ub97c \ud589\uc6b4\uc73c\ub85c \ubc1b\uc544\ub4e4\uc5ec\uc57c \ud560 \uac83\uc774\ub2e4. \uacbd\uc81c\uc704\uae30\ub97c \ub9de\uc544 \uba87\uac00\uc9c0 \uace0\ube44\uac00 \uc788\uc5c8\uc9c0\ub9cc IMF\uc640 \ubbf8\uad6d \uadf8\ub9ac\uace0 \uad6d\uc81c\uc0ac\ud68c \uc804\ubc18\uc740 \uae40 \ub300\ud1b5\ub839\uc744 \ubbff\uace0 \ud55c\uad6d\uc744 \ubc00\uc5b4\uc900 \uce21\uba74\uc774 \uc801\uc9c0 \uc54a\uc740\uac8c \uc0ac\uc2e4\uc774\ub2e4\"\ub77c\uace0 \ubc1d\ud614\ub2e4. '\ub808\uc774\uac74 \uc2e0\ud654'\uc758 \uc8fc\uc5ed \ub9c8\uc774\ud06c \ub514\ubc84 \uc804 \ub300\ud1b5\ub839 \ud2b9\ubcf4\ub294 \"\uae40 \ub300\ud1b5\ub839\uc740 \uba54\uc774\ud06c \uc5c5\uc774 \ud544\uc694 \uc5c6\ub294 \ub17c\ud53d\uc158 \ub4dc\ub77c\ub9c8\uc758 \uc778\uc0dd\uc744 \uc0b4\uc544\uc654\ub2e4\"\uba70 \"\uc774\ub984 \uc11d\uc790\ub9cc \uac16\uace0\ub3c4 \uad6d\uc81c\uc0ac\ud68c\uc5d0 \uc601\ud5a5\uc744\uc904 \uc218 \ube0c\ub79c\ub4dc\"\ub77c\uace0 \uadf9\ucc2c\ud588\ub2e4. \ubbf8\uad6d \ub9dd\uba85 \uc2dc\uc808\uc5d0 \ud558\ubc84\ub4dc \ub300\ud559\uad50\uc5d0\uc11c \ud3b4\ub0b8 \u300a\ub300\uc911\uacbd\uc81c\ub860\u300b\uacfc \u300a3\ub2e8\uacc4\ud1b5\uc77c\ub860\u300b\ub294 \ud558\ubc84\ub4dc \ub300\ud559\uad50\uc758 \uad50\uc7ac\uc6a9\uc73c\ub85c \ubc88\uc5ed \ucd9c\uac04\ub418\uc5c8\ub2e4."} {"question": "\uc694\ud06c\uc5d0\uc11c \ubb34\uc7a5\ubcd1\ub825\uc744 \ub3c4\uc2dc \uc548\uc73c\ub85c \ub4e4\uc5ec\uc640 \uc0c1\ud669\uc744 \ud1b5\uc81c\ud558\ub824 \ud55c \uc0ac\ub78c\uc740?", "paragraph": "6\uc6d4 17\uc77c\uc5d0\ub294 \ub7f0\ub358\uc5d0\uc11c \uc77c\uc5b4\ub09c \uc77c\uc774 \uc694\ud06c\uc5d0 \ucc98\uc74c \uc54c\ub824\uc84c\uace0, \ub3c4\ubbf8\ub2c8\ud06c \uc218\ub3c4\ud68c\uc640 \ud504\ub780\uce58\uc2a4\ucf54 \uc218\ub3c4\ud68c\ub97c \ube44\ub86f\ud55c \uc885\uad50\uae30\uad00\uc5d0 \ub300\ud55c \uacf5\uaca9\uc774 \uc77c\uc5b4\ub0ac\ub2e4. \ub2e4\uc74c \ud55c \uc8fc \ub3d9\uc548 \ud3ed\ub825\uc0ac\ud0dc\uac00 \uacc4\uc18d\ub418\uc5c8\uace0, 7\uc6d4 1\uc77c\uc5d0 \uc874 \ub514\uae30\uc988\ubc88(John de Gisbourne)\uc774 \uc9c0\ud718\ud558\ub294 \ubb34\uc7a5\ubcd1\ub825\uc774 \ub3c4\uc2dc \uc548\uc73c\ub85c \ub4e4\uc5b4\uc640 \uc0c1\ud669\uc744 \ud1b5\uc81c\ud558\ub824 \uc2dc\ub3c4\ud588\ub2e4. \uc2dc\uc7a5 \uc0ac\uc774\uba3c \ub514\ud035\uc2ac\ub808\uc774(Simon de Quixlay)\ub294 \uc810\ucc28\uc801\uc73c\ub85c \uc678\uad00\uc0c1\uc758 \uad8c\uc704\ub97c \ub418\ucc3e\uc558\uc73c\ub098, \uc9c8\uc11c\ub294 1382\ub144 \uc774\uc804\uae4c\uc9c0 \uc644\uc804\ud788 \ubcf5\uad6c\ub418\uc9c0 \uc54a\uc558\ub2e4. \uc2a4\uce74\ubc84\ub7ec\uc5d0\uc11c\ub3c4 \ub0a8\ucabd\uc758 \ubc18\ub780 \uc18c\uc2dd\uc774 \ub4e4\ub824\uc624\uace0, 6\uc6d4 23\uc77c\uc5d0 \uc5d8\ub9ac\ud2b8\ub4e4\uc5d0\uac8c \ubc18\ub300\ud558\ub294 \ubc18\ub780\uc774 \ubc1c\uc0dd\ud588\ub2e4. \uc774\uacf3\uc758 \ub09c\ubbfc\ub4e4\uc740 \ub4a4\uc5d0 \ubd89\uc740\uc0c9 \uaf2c\ub9ac\uac00 \ub2ec\ub9b0 \ud770\uc0c9 \ub450\uac74\uc744 \uc37c\ub2e4. \uc9c0\uc5ed \uc815\ubd80 \uad6c\uc131\uc6d0\ub4e4\uc740 \uad00\uc800\uc5d0\uc11c \ucad3\uaca8\ub0ac\uace0, \uc138\uae08\uc9d5\uc218\uc6d0 \ud558\ub098\uac00 \ub9b0\uce58\ub97c \ub2f9\ud588\ub2e4. \ud558\uc9c0\ub9cc \uc5d8\ub9ac\ud2b8\ub4e4\uc740 1382\ub144\uae4c\uc9c0 \uad8c\ub825\uc744 \ub2e4\uc2dc \ub418\ucc3e\uc558\ub2e4.", "answer": "\uc874 \ub514\uae30\uc988\ubc88", "sentence": "\ub2e4\uc74c \ud55c \uc8fc \ub3d9\uc548 \ud3ed\ub825\uc0ac\ud0dc\uac00 \uacc4\uc18d\ub418\uc5c8\uace0, 7\uc6d4 1\uc77c\uc5d0 \uc874 \ub514\uae30\uc988\ubc88 (John de Gisbourne)", "paragraph_sentence": "6\uc6d4 17\uc77c\uc5d0\ub294 \ub7f0\ub358\uc5d0\uc11c \uc77c\uc5b4\ub09c \uc77c\uc774 \uc694\ud06c\uc5d0 \ucc98\uc74c \uc54c\ub824\uc84c\uace0, \ub3c4\ubbf8\ub2c8\ud06c \uc218\ub3c4\ud68c\uc640 \ud504\ub780\uce58\uc2a4\ucf54 \uc218\ub3c4\ud68c\ub97c \ube44\ub86f\ud55c \uc885\uad50\uae30\uad00\uc5d0 \ub300\ud55c \uacf5\uaca9\uc774 \uc77c\uc5b4\ub0ac\ub2e4. \ub2e4\uc74c \ud55c \uc8fc \ub3d9\uc548 \ud3ed\ub825\uc0ac\ud0dc\uac00 \uacc4\uc18d\ub418\uc5c8\uace0, 7\uc6d4 1\uc77c\uc5d0 \uc874 \ub514\uae30\uc988\ubc88 (John de Gisbourne) \uc774 \uc9c0\ud718\ud558\ub294 \ubb34\uc7a5\ubcd1\ub825\uc774 \ub3c4\uc2dc \uc548\uc73c\ub85c \ub4e4\uc5b4\uc640 \uc0c1\ud669\uc744 \ud1b5\uc81c\ud558\ub824 \uc2dc\ub3c4\ud588\ub2e4. \uc2dc\uc7a5 \uc0ac\uc774\uba3c \ub514\ud035\uc2ac\ub808\uc774(Simon de Quixlay)\ub294 \uc810\ucc28\uc801\uc73c\ub85c \uc678\uad00\uc0c1\uc758 \uad8c\uc704\ub97c \ub418\ucc3e\uc558\uc73c\ub098, \uc9c8\uc11c\ub294 1382\ub144 \uc774\uc804\uae4c\uc9c0 \uc644\uc804\ud788 \ubcf5\uad6c\ub418\uc9c0 \uc54a\uc558\ub2e4. \uc2a4\uce74\ubc84\ub7ec\uc5d0\uc11c\ub3c4 \ub0a8\ucabd\uc758 \ubc18\ub780 \uc18c\uc2dd\uc774 \ub4e4\ub824\uc624\uace0, 6\uc6d4 23\uc77c\uc5d0 \uc5d8\ub9ac\ud2b8\ub4e4\uc5d0\uac8c \ubc18\ub300\ud558\ub294 \ubc18\ub780\uc774 \ubc1c\uc0dd\ud588\ub2e4. \uc774\uacf3\uc758 \ub09c\ubbfc\ub4e4\uc740 \ub4a4\uc5d0 \ubd89\uc740\uc0c9 \uaf2c\ub9ac\uac00 \ub2ec\ub9b0 \ud770\uc0c9 \ub450\uac74\uc744 \uc37c\ub2e4. \uc9c0\uc5ed \uc815\ubd80 \uad6c\uc131\uc6d0\ub4e4\uc740 \uad00\uc800\uc5d0\uc11c \ucad3\uaca8\ub0ac\uace0, \uc138\uae08\uc9d5\uc218\uc6d0 \ud558\ub098\uac00 \ub9b0\uce58\ub97c \ub2f9\ud588\ub2e4. \ud558\uc9c0\ub9cc \uc5d8\ub9ac\ud2b8\ub4e4\uc740 1382\ub144\uae4c\uc9c0 \uad8c\ub825\uc744 \ub2e4\uc2dc \ub418\ucc3e\uc558\ub2e4.", "paragraph_answer": "6\uc6d4 17\uc77c\uc5d0\ub294 \ub7f0\ub358\uc5d0\uc11c \uc77c\uc5b4\ub09c \uc77c\uc774 \uc694\ud06c\uc5d0 \ucc98\uc74c \uc54c\ub824\uc84c\uace0, \ub3c4\ubbf8\ub2c8\ud06c \uc218\ub3c4\ud68c\uc640 \ud504\ub780\uce58\uc2a4\ucf54 \uc218\ub3c4\ud68c\ub97c \ube44\ub86f\ud55c \uc885\uad50\uae30\uad00\uc5d0 \ub300\ud55c \uacf5\uaca9\uc774 \uc77c\uc5b4\ub0ac\ub2e4. \ub2e4\uc74c \ud55c \uc8fc \ub3d9\uc548 \ud3ed\ub825\uc0ac\ud0dc\uac00 \uacc4\uc18d\ub418\uc5c8\uace0, 7\uc6d4 1\uc77c\uc5d0 \uc874 \ub514\uae30\uc988\ubc88 (John de Gisbourne)\uc774 \uc9c0\ud718\ud558\ub294 \ubb34\uc7a5\ubcd1\ub825\uc774 \ub3c4\uc2dc \uc548\uc73c\ub85c \ub4e4\uc5b4\uc640 \uc0c1\ud669\uc744 \ud1b5\uc81c\ud558\ub824 \uc2dc\ub3c4\ud588\ub2e4. \uc2dc\uc7a5 \uc0ac\uc774\uba3c \ub514\ud035\uc2ac\ub808\uc774(Simon de Quixlay)\ub294 \uc810\ucc28\uc801\uc73c\ub85c \uc678\uad00\uc0c1\uc758 \uad8c\uc704\ub97c \ub418\ucc3e\uc558\uc73c\ub098, \uc9c8\uc11c\ub294 1382\ub144 \uc774\uc804\uae4c\uc9c0 \uc644\uc804\ud788 \ubcf5\uad6c\ub418\uc9c0 \uc54a\uc558\ub2e4. \uc2a4\uce74\ubc84\ub7ec\uc5d0\uc11c\ub3c4 \ub0a8\ucabd\uc758 \ubc18\ub780 \uc18c\uc2dd\uc774 \ub4e4\ub824\uc624\uace0, 6\uc6d4 23\uc77c\uc5d0 \uc5d8\ub9ac\ud2b8\ub4e4\uc5d0\uac8c \ubc18\ub300\ud558\ub294 \ubc18\ub780\uc774 \ubc1c\uc0dd\ud588\ub2e4. \uc774\uacf3\uc758 \ub09c\ubbfc\ub4e4\uc740 \ub4a4\uc5d0 \ubd89\uc740\uc0c9 \uaf2c\ub9ac\uac00 \ub2ec\ub9b0 \ud770\uc0c9 \ub450\uac74\uc744 \uc37c\ub2e4. \uc9c0\uc5ed \uc815\ubd80 \uad6c\uc131\uc6d0\ub4e4\uc740 \uad00\uc800\uc5d0\uc11c \ucad3\uaca8\ub0ac\uace0, \uc138\uae08\uc9d5\uc218\uc6d0 \ud558\ub098\uac00 \ub9b0\uce58\ub97c \ub2f9\ud588\ub2e4. \ud558\uc9c0\ub9cc \uc5d8\ub9ac\ud2b8\ub4e4\uc740 1382\ub144\uae4c\uc9c0 \uad8c\ub825\uc744 \ub2e4\uc2dc \ub418\ucc3e\uc558\ub2e4.", "sentence_answer": "\ub2e4\uc74c \ud55c \uc8fc \ub3d9\uc548 \ud3ed\ub825\uc0ac\ud0dc\uac00 \uacc4\uc18d\ub418\uc5c8\uace0, 7\uc6d4 1\uc77c\uc5d0 \uc874 \ub514\uae30\uc988\ubc88 (John de Gisbourne)", "paragraph_id": "6541477-28-0", "paragraph_question": "question: \uc694\ud06c\uc5d0\uc11c \ubb34\uc7a5\ubcd1\ub825\uc744 \ub3c4\uc2dc \uc548\uc73c\ub85c \ub4e4\uc5ec\uc640 \uc0c1\ud669\uc744 \ud1b5\uc81c\ud558\ub824 \ud55c \uc0ac\ub78c\uc740?, context: 6\uc6d4 17\uc77c\uc5d0\ub294 \ub7f0\ub358\uc5d0\uc11c \uc77c\uc5b4\ub09c \uc77c\uc774 \uc694\ud06c\uc5d0 \ucc98\uc74c \uc54c\ub824\uc84c\uace0, \ub3c4\ubbf8\ub2c8\ud06c \uc218\ub3c4\ud68c\uc640 \ud504\ub780\uce58\uc2a4\ucf54 \uc218\ub3c4\ud68c\ub97c \ube44\ub86f\ud55c \uc885\uad50\uae30\uad00\uc5d0 \ub300\ud55c \uacf5\uaca9\uc774 \uc77c\uc5b4\ub0ac\ub2e4. \ub2e4\uc74c \ud55c \uc8fc \ub3d9\uc548 \ud3ed\ub825\uc0ac\ud0dc\uac00 \uacc4\uc18d\ub418\uc5c8\uace0, 7\uc6d4 1\uc77c\uc5d0 \uc874 \ub514\uae30\uc988\ubc88(John de Gisbourne)\uc774 \uc9c0\ud718\ud558\ub294 \ubb34\uc7a5\ubcd1\ub825\uc774 \ub3c4\uc2dc \uc548\uc73c\ub85c \ub4e4\uc5b4\uc640 \uc0c1\ud669\uc744 \ud1b5\uc81c\ud558\ub824 \uc2dc\ub3c4\ud588\ub2e4. \uc2dc\uc7a5 \uc0ac\uc774\uba3c \ub514\ud035\uc2ac\ub808\uc774(Simon de Quixlay)\ub294 \uc810\ucc28\uc801\uc73c\ub85c \uc678\uad00\uc0c1\uc758 \uad8c\uc704\ub97c \ub418\ucc3e\uc558\uc73c\ub098, \uc9c8\uc11c\ub294 1382\ub144 \uc774\uc804\uae4c\uc9c0 \uc644\uc804\ud788 \ubcf5\uad6c\ub418\uc9c0 \uc54a\uc558\ub2e4. \uc2a4\uce74\ubc84\ub7ec\uc5d0\uc11c\ub3c4 \ub0a8\ucabd\uc758 \ubc18\ub780 \uc18c\uc2dd\uc774 \ub4e4\ub824\uc624\uace0, 6\uc6d4 23\uc77c\uc5d0 \uc5d8\ub9ac\ud2b8\ub4e4\uc5d0\uac8c \ubc18\ub300\ud558\ub294 \ubc18\ub780\uc774 \ubc1c\uc0dd\ud588\ub2e4. \uc774\uacf3\uc758 \ub09c\ubbfc\ub4e4\uc740 \ub4a4\uc5d0 \ubd89\uc740\uc0c9 \uaf2c\ub9ac\uac00 \ub2ec\ub9b0 \ud770\uc0c9 \ub450\uac74\uc744 \uc37c\ub2e4. \uc9c0\uc5ed \uc815\ubd80 \uad6c\uc131\uc6d0\ub4e4\uc740 \uad00\uc800\uc5d0\uc11c \ucad3\uaca8\ub0ac\uace0, \uc138\uae08\uc9d5\uc218\uc6d0 \ud558\ub098\uac00 \ub9b0\uce58\ub97c \ub2f9\ud588\ub2e4. \ud558\uc9c0\ub9cc \uc5d8\ub9ac\ud2b8\ub4e4\uc740 1382\ub144\uae4c\uc9c0 \uad8c\ub825\uc744 \ub2e4\uc2dc \ub418\ucc3e\uc558\ub2e4."} {"question": "\ucf58\ub3c8\uc774 \uc81c\uc77c \ucc98\uc74c \ubc30\uc6b4 \uc545\uae30\ub294?", "paragraph": "\ucf58\ub3c8\uc774 \uc81c\uc77c \ucc98\uc74c \ubc30\uc6b4 \uc545\uae30\ub294 \ud2b8\ub7fc\ud3ab\uc774\uc5c8\uc73c\uba70, \uc591\ucabd \uc190\ubaa9\uc5d0 \ud504\ub80c\uce58 \ud638\ub978 \ubaa8\uc591\uc758 \ubb38\uc2e0\uc744 \uc0c8\uae38 \uc815\ub3c4\ub85c \ube0c\ub77c\uc2a4 \uc545\uae30\ub4e4\uc5d0 \uc5f4\uad11\ud588\ub2e4. \uc5f4\uc77c\uacf1 \uc0b4 \ub54c \ud314 \uc218\uc220\uc744 \ud574\uc11c \uae30\ud0c0\ub97c \uc5f0\uc8fc\ud560 \uc218 \uc5c6\uac8c \ub418\uc790 \uacb0\uad6d \ud2b8\ub7fc\ud3ab\uacfc \ub9cc\ub3cc\ub9b0, \uc6b0\ucfe8\ub810\ub808\uac00 \uadf8\uc758 \uc8fc\ub41c \uc545\uae30\ub85c \uc790\ub9ac \uc7a1\uc558\uace0, \uadf8 \uc545\uae30\ub4e4\uc774 \uc8fc\ucd95\uc774 \ub418\uc5b4 \uc18c\ub144\uc758 \uc5bc\uad74\uc5d0 \ub178\uc778\uc758 \uac10\uc131\uc744 \uc9c0\ub2cc \ub4ef\ud55c \ucf58\ub3c8\uc740 \ubca0\uc774\ub8e8\ud2b8\ub77c\ub294 \uc774\ub984\uacfc \ud568\uaed8 \uc2a4\ubb34 \uc0b4\uc774\ub77c\ub294 \ub098\uc774\uac00 \ubbff\uae30\uc9c0 \uc54a\uc744 \uc815\ub3c4\ub85c \uc644\uc131\ub3c4 \uc788\ub294 \uccab \uc568\ubc94 Gulag Orkestar\ub97c \ubc1c\ud45c\ud55c\ub2e4. \uadf8\ub294 \uc218\ub9ce\uc740 \ube14\ub85c\uadf8\ub4e4\uc744 \ud1b5\ud574 \ud37c\uc838\ub098\uac04 \uc778\uae30\uc640 \ud568\uaed8 \ud3c9\ub2e8\uc5d0\uc11c\ub3c4 \ud3c9\uac00\ub97c \uc5bb\uc73c\uba70 \uc13c\uc138\uc774\uc154\ub110\ud558\uac8c \ub370\ubdd4\ud588\uace0, \ubc14\ub85c \ub2e4\uc74c \ud574\uc5d0 \ub450 \ubc88\uc9f8 \uc568\ubc94 The Flying Club Cup\uc744 \ubc1c\ud45c\ud558\uba70 \uc80a\uace0 \uc655\uc131\ud55c \ucc3d\uc791\uc5f4\uc744 \uc774\uc5b4\uac14\ub2e4. \ubca0\uc774\ub8e8\ud2b8\ub294 \ubcf8\uc778\uc774 \uac00\uc7a5 \uc88b\uc544\ud558\ub294 \uc74c\uc545 \uc7a5\ub974\uc758 \ub124 \uac00\uc9c0\ub85c \ubc1c\uce78 \ube0c\ub77c\uc2a4\uc640 \uc720\ub85c \ud558\uc6b0\uc2a4, \ub365, \uadf8\ub9ac\uace0 \ube0c\ub77c\uc9c8\uc758 \ud2b8\ub85c\ud53c\uce7c\ub9ac\uc544\ub97c \uaf3d\uc558\ub294\ub370, 1\uc9d1 Gulag Orkestar\uc5d0\uc11c\ub294 \ubc1c\uce78 \ube0c\ub77c\uc2a4\uc758 \uc601\ud5a5\uc774 \uac00\uc7a5 \ud06c\uac8c \ub450\ub4dc\ub7ec\uc84c\uace0, \ud504\ub791\uc2a4\ub85c \uc74c\uc545\uc801 \uadfc\uac70\uc9c0\ub97c \uc62e\uae34 2\uc9d1\uc5d0\uc11c\ub294 1\uc9d1\uc758 \uae30\ubc18\uc5d0\uc11c \uc0f9\uc1a1\uc758 \ub85c\ub9e8\ud2f0\uc2dc\uc998\uc774 \ub354\ud574\uc84c\ub2e4. \uc5f4\uc5ec\uc12f \uc0b4 \ub54c\ubd80\ud130 \uc0f9\uc1a1\uc744 \uc990\uaca8\ub4e4\uc5c8\ub358 \ucf58\ub3c8\uc740 \ud2b9\ud788 \uc790\ub044 \ube0c\ub810\uc758 \uc74c\uc545\uc744 \uc990\uaca8 \ub4e4\uc5c8\uc73c\uba70, \ud30c\ub9ac\ub97c \uc74c\uc545\uc801\uc778 \uc758\ubbf8\uc5d0\uc11c \uc81c 2\uc758 \uace0\ud5a5\uc73c\ub85c \uc5ec\uae30\ub294 \ub4ef\ud558\ub2e4.", "answer": "\ud2b8\ub7fc\ud3ab", "sentence": "\ucf58\ub3c8\uc774 \uc81c\uc77c \ucc98\uc74c \ubc30\uc6b4 \uc545\uae30\ub294 \ud2b8\ub7fc\ud3ab \uc774\uc5c8\uc73c\uba70, \uc591\ucabd \uc190\ubaa9\uc5d0 \ud504\ub80c\uce58 \ud638\ub978 \ubaa8\uc591\uc758 \ubb38\uc2e0\uc744 \uc0c8\uae38 \uc815\ub3c4\ub85c \ube0c\ub77c\uc2a4 \uc545\uae30\ub4e4\uc5d0 \uc5f4\uad11\ud588\ub2e4.", "paragraph_sentence": " \ucf58\ub3c8\uc774 \uc81c\uc77c \ucc98\uc74c \ubc30\uc6b4 \uc545\uae30\ub294 \ud2b8\ub7fc\ud3ab \uc774\uc5c8\uc73c\uba70, \uc591\ucabd \uc190\ubaa9\uc5d0 \ud504\ub80c\uce58 \ud638\ub978 \ubaa8\uc591\uc758 \ubb38\uc2e0\uc744 \uc0c8\uae38 \uc815\ub3c4\ub85c \ube0c\ub77c\uc2a4 \uc545\uae30\ub4e4\uc5d0 \uc5f4\uad11\ud588\ub2e4. \uc5f4\uc77c\uacf1 \uc0b4 \ub54c \ud314 \uc218\uc220\uc744 \ud574\uc11c \uae30\ud0c0\ub97c \uc5f0\uc8fc\ud560 \uc218 \uc5c6\uac8c \ub418\uc790 \uacb0\uad6d \ud2b8\ub7fc\ud3ab\uacfc \ub9cc\ub3cc\ub9b0, \uc6b0\ucfe8\ub810\ub808\uac00 \uadf8\uc758 \uc8fc\ub41c \uc545\uae30\ub85c \uc790\ub9ac \uc7a1\uc558\uace0, \uadf8 \uc545\uae30\ub4e4\uc774 \uc8fc\ucd95\uc774 \ub418\uc5b4 \uc18c\ub144\uc758 \uc5bc\uad74\uc5d0 \ub178\uc778\uc758 \uac10\uc131\uc744 \uc9c0\ub2cc \ub4ef\ud55c \ucf58\ub3c8\uc740 \ubca0\uc774\ub8e8\ud2b8\ub77c\ub294 \uc774\ub984\uacfc \ud568\uaed8 \uc2a4\ubb34 \uc0b4\uc774\ub77c\ub294 \ub098\uc774\uac00 \ubbff\uae30\uc9c0 \uc54a\uc744 \uc815\ub3c4\ub85c \uc644\uc131\ub3c4 \uc788\ub294 \uccab \uc568\ubc94 Gulag Orkestar\ub97c \ubc1c\ud45c\ud55c\ub2e4. \uadf8\ub294 \uc218\ub9ce\uc740 \ube14\ub85c\uadf8\ub4e4\uc744 \ud1b5\ud574 \ud37c\uc838\ub098\uac04 \uc778\uae30\uc640 \ud568\uaed8 \ud3c9\ub2e8\uc5d0\uc11c\ub3c4 \ud3c9\uac00\ub97c \uc5bb\uc73c\uba70 \uc13c\uc138\uc774\uc154\ub110\ud558\uac8c \ub370\ubdd4\ud588\uace0, \ubc14\ub85c \ub2e4\uc74c \ud574\uc5d0 \ub450 \ubc88\uc9f8 \uc568\ubc94 The Flying Club Cup\uc744 \ubc1c\ud45c\ud558\uba70 \uc80a\uace0 \uc655\uc131\ud55c \ucc3d\uc791\uc5f4\uc744 \uc774\uc5b4\uac14\ub2e4. \ubca0\uc774\ub8e8\ud2b8\ub294 \ubcf8\uc778\uc774 \uac00\uc7a5 \uc88b\uc544\ud558\ub294 \uc74c\uc545 \uc7a5\ub974\uc758 \ub124 \uac00\uc9c0\ub85c \ubc1c\uce78 \ube0c\ub77c\uc2a4\uc640 \uc720\ub85c \ud558\uc6b0\uc2a4, \ub365, \uadf8\ub9ac\uace0 \ube0c\ub77c\uc9c8\uc758 \ud2b8\ub85c\ud53c\uce7c\ub9ac\uc544\ub97c \uaf3d\uc558\ub294\ub370, 1\uc9d1 Gulag Orkestar\uc5d0\uc11c\ub294 \ubc1c\uce78 \ube0c\ub77c\uc2a4\uc758 \uc601\ud5a5\uc774 \uac00\uc7a5 \ud06c\uac8c \ub450\ub4dc\ub7ec\uc84c\uace0, \ud504\ub791\uc2a4\ub85c \uc74c\uc545\uc801 \uadfc\uac70\uc9c0\ub97c \uc62e\uae34 2\uc9d1\uc5d0\uc11c\ub294 1\uc9d1\uc758 \uae30\ubc18\uc5d0\uc11c \uc0f9\uc1a1\uc758 \ub85c\ub9e8\ud2f0\uc2dc\uc998\uc774 \ub354\ud574\uc84c\ub2e4. \uc5f4\uc5ec\uc12f \uc0b4 \ub54c\ubd80\ud130 \uc0f9\uc1a1\uc744 \uc990\uaca8\ub4e4\uc5c8\ub358 \ucf58\ub3c8\uc740 \ud2b9\ud788 \uc790\ub044 \ube0c\ub810\uc758 \uc74c\uc545\uc744 \uc990\uaca8 \ub4e4\uc5c8\uc73c\uba70, \ud30c\ub9ac\ub97c \uc74c\uc545\uc801\uc778 \uc758\ubbf8\uc5d0\uc11c \uc81c 2\uc758 \uace0\ud5a5\uc73c\ub85c \uc5ec\uae30\ub294 \ub4ef\ud558\ub2e4.", "paragraph_answer": "\ucf58\ub3c8\uc774 \uc81c\uc77c \ucc98\uc74c \ubc30\uc6b4 \uc545\uae30\ub294 \ud2b8\ub7fc\ud3ab \uc774\uc5c8\uc73c\uba70, \uc591\ucabd \uc190\ubaa9\uc5d0 \ud504\ub80c\uce58 \ud638\ub978 \ubaa8\uc591\uc758 \ubb38\uc2e0\uc744 \uc0c8\uae38 \uc815\ub3c4\ub85c \ube0c\ub77c\uc2a4 \uc545\uae30\ub4e4\uc5d0 \uc5f4\uad11\ud588\ub2e4. \uc5f4\uc77c\uacf1 \uc0b4 \ub54c \ud314 \uc218\uc220\uc744 \ud574\uc11c \uae30\ud0c0\ub97c \uc5f0\uc8fc\ud560 \uc218 \uc5c6\uac8c \ub418\uc790 \uacb0\uad6d \ud2b8\ub7fc\ud3ab\uacfc \ub9cc\ub3cc\ub9b0, \uc6b0\ucfe8\ub810\ub808\uac00 \uadf8\uc758 \uc8fc\ub41c \uc545\uae30\ub85c \uc790\ub9ac \uc7a1\uc558\uace0, \uadf8 \uc545\uae30\ub4e4\uc774 \uc8fc\ucd95\uc774 \ub418\uc5b4 \uc18c\ub144\uc758 \uc5bc\uad74\uc5d0 \ub178\uc778\uc758 \uac10\uc131\uc744 \uc9c0\ub2cc \ub4ef\ud55c \ucf58\ub3c8\uc740 \ubca0\uc774\ub8e8\ud2b8\ub77c\ub294 \uc774\ub984\uacfc \ud568\uaed8 \uc2a4\ubb34 \uc0b4\uc774\ub77c\ub294 \ub098\uc774\uac00 \ubbff\uae30\uc9c0 \uc54a\uc744 \uc815\ub3c4\ub85c \uc644\uc131\ub3c4 \uc788\ub294 \uccab \uc568\ubc94 Gulag Orkestar\ub97c \ubc1c\ud45c\ud55c\ub2e4. \uadf8\ub294 \uc218\ub9ce\uc740 \ube14\ub85c\uadf8\ub4e4\uc744 \ud1b5\ud574 \ud37c\uc838\ub098\uac04 \uc778\uae30\uc640 \ud568\uaed8 \ud3c9\ub2e8\uc5d0\uc11c\ub3c4 \ud3c9\uac00\ub97c \uc5bb\uc73c\uba70 \uc13c\uc138\uc774\uc154\ub110\ud558\uac8c \ub370\ubdd4\ud588\uace0, \ubc14\ub85c \ub2e4\uc74c \ud574\uc5d0 \ub450 \ubc88\uc9f8 \uc568\ubc94 The Flying Club Cup\uc744 \ubc1c\ud45c\ud558\uba70 \uc80a\uace0 \uc655\uc131\ud55c \ucc3d\uc791\uc5f4\uc744 \uc774\uc5b4\uac14\ub2e4. \ubca0\uc774\ub8e8\ud2b8\ub294 \ubcf8\uc778\uc774 \uac00\uc7a5 \uc88b\uc544\ud558\ub294 \uc74c\uc545 \uc7a5\ub974\uc758 \ub124 \uac00\uc9c0\ub85c \ubc1c\uce78 \ube0c\ub77c\uc2a4\uc640 \uc720\ub85c \ud558\uc6b0\uc2a4, \ub365, \uadf8\ub9ac\uace0 \ube0c\ub77c\uc9c8\uc758 \ud2b8\ub85c\ud53c\uce7c\ub9ac\uc544\ub97c \uaf3d\uc558\ub294\ub370, 1\uc9d1 Gulag Orkestar\uc5d0\uc11c\ub294 \ubc1c\uce78 \ube0c\ub77c\uc2a4\uc758 \uc601\ud5a5\uc774 \uac00\uc7a5 \ud06c\uac8c \ub450\ub4dc\ub7ec\uc84c\uace0, \ud504\ub791\uc2a4\ub85c \uc74c\uc545\uc801 \uadfc\uac70\uc9c0\ub97c \uc62e\uae34 2\uc9d1\uc5d0\uc11c\ub294 1\uc9d1\uc758 \uae30\ubc18\uc5d0\uc11c \uc0f9\uc1a1\uc758 \ub85c\ub9e8\ud2f0\uc2dc\uc998\uc774 \ub354\ud574\uc84c\ub2e4. \uc5f4\uc5ec\uc12f \uc0b4 \ub54c\ubd80\ud130 \uc0f9\uc1a1\uc744 \uc990\uaca8\ub4e4\uc5c8\ub358 \ucf58\ub3c8\uc740 \ud2b9\ud788 \uc790\ub044 \ube0c\ub810\uc758 \uc74c\uc545\uc744 \uc990\uaca8 \ub4e4\uc5c8\uc73c\uba70, \ud30c\ub9ac\ub97c \uc74c\uc545\uc801\uc778 \uc758\ubbf8\uc5d0\uc11c \uc81c 2\uc758 \uace0\ud5a5\uc73c\ub85c \uc5ec\uae30\ub294 \ub4ef\ud558\ub2e4.", "sentence_answer": "\ucf58\ub3c8\uc774 \uc81c\uc77c \ucc98\uc74c \ubc30\uc6b4 \uc545\uae30\ub294 \ud2b8\ub7fc\ud3ab \uc774\uc5c8\uc73c\uba70, \uc591\ucabd \uc190\ubaa9\uc5d0 \ud504\ub80c\uce58 \ud638\ub978 \ubaa8\uc591\uc758 \ubb38\uc2e0\uc744 \uc0c8\uae38 \uc815\ub3c4\ub85c \ube0c\ub77c\uc2a4 \uc545\uae30\ub4e4\uc5d0 \uc5f4\uad11\ud588\ub2e4.", "paragraph_id": "6418825-1-1", "paragraph_question": "question: \ucf58\ub3c8\uc774 \uc81c\uc77c \ucc98\uc74c \ubc30\uc6b4 \uc545\uae30\ub294?, context: \ucf58\ub3c8\uc774 \uc81c\uc77c \ucc98\uc74c \ubc30\uc6b4 \uc545\uae30\ub294 \ud2b8\ub7fc\ud3ab\uc774\uc5c8\uc73c\uba70, \uc591\ucabd \uc190\ubaa9\uc5d0 \ud504\ub80c\uce58 \ud638\ub978 \ubaa8\uc591\uc758 \ubb38\uc2e0\uc744 \uc0c8\uae38 \uc815\ub3c4\ub85c \ube0c\ub77c\uc2a4 \uc545\uae30\ub4e4\uc5d0 \uc5f4\uad11\ud588\ub2e4. \uc5f4\uc77c\uacf1 \uc0b4 \ub54c \ud314 \uc218\uc220\uc744 \ud574\uc11c \uae30\ud0c0\ub97c \uc5f0\uc8fc\ud560 \uc218 \uc5c6\uac8c \ub418\uc790 \uacb0\uad6d \ud2b8\ub7fc\ud3ab\uacfc \ub9cc\ub3cc\ub9b0, \uc6b0\ucfe8\ub810\ub808\uac00 \uadf8\uc758 \uc8fc\ub41c \uc545\uae30\ub85c \uc790\ub9ac \uc7a1\uc558\uace0, \uadf8 \uc545\uae30\ub4e4\uc774 \uc8fc\ucd95\uc774 \ub418\uc5b4 \uc18c\ub144\uc758 \uc5bc\uad74\uc5d0 \ub178\uc778\uc758 \uac10\uc131\uc744 \uc9c0\ub2cc \ub4ef\ud55c \ucf58\ub3c8\uc740 \ubca0\uc774\ub8e8\ud2b8\ub77c\ub294 \uc774\ub984\uacfc \ud568\uaed8 \uc2a4\ubb34 \uc0b4\uc774\ub77c\ub294 \ub098\uc774\uac00 \ubbff\uae30\uc9c0 \uc54a\uc744 \uc815\ub3c4\ub85c \uc644\uc131\ub3c4 \uc788\ub294 \uccab \uc568\ubc94 Gulag Orkestar\ub97c \ubc1c\ud45c\ud55c\ub2e4. \uadf8\ub294 \uc218\ub9ce\uc740 \ube14\ub85c\uadf8\ub4e4\uc744 \ud1b5\ud574 \ud37c\uc838\ub098\uac04 \uc778\uae30\uc640 \ud568\uaed8 \ud3c9\ub2e8\uc5d0\uc11c\ub3c4 \ud3c9\uac00\ub97c \uc5bb\uc73c\uba70 \uc13c\uc138\uc774\uc154\ub110\ud558\uac8c \ub370\ubdd4\ud588\uace0, \ubc14\ub85c \ub2e4\uc74c \ud574\uc5d0 \ub450 \ubc88\uc9f8 \uc568\ubc94 The Flying Club Cup\uc744 \ubc1c\ud45c\ud558\uba70 \uc80a\uace0 \uc655\uc131\ud55c \ucc3d\uc791\uc5f4\uc744 \uc774\uc5b4\uac14\ub2e4. \ubca0\uc774\ub8e8\ud2b8\ub294 \ubcf8\uc778\uc774 \uac00\uc7a5 \uc88b\uc544\ud558\ub294 \uc74c\uc545 \uc7a5\ub974\uc758 \ub124 \uac00\uc9c0\ub85c \ubc1c\uce78 \ube0c\ub77c\uc2a4\uc640 \uc720\ub85c \ud558\uc6b0\uc2a4, \ub365, \uadf8\ub9ac\uace0 \ube0c\ub77c\uc9c8\uc758 \ud2b8\ub85c\ud53c\uce7c\ub9ac\uc544\ub97c \uaf3d\uc558\ub294\ub370, 1\uc9d1 Gulag Orkestar\uc5d0\uc11c\ub294 \ubc1c\uce78 \ube0c\ub77c\uc2a4\uc758 \uc601\ud5a5\uc774 \uac00\uc7a5 \ud06c\uac8c \ub450\ub4dc\ub7ec\uc84c\uace0, \ud504\ub791\uc2a4\ub85c \uc74c\uc545\uc801 \uadfc\uac70\uc9c0\ub97c \uc62e\uae34 2\uc9d1\uc5d0\uc11c\ub294 1\uc9d1\uc758 \uae30\ubc18\uc5d0\uc11c \uc0f9\uc1a1\uc758 \ub85c\ub9e8\ud2f0\uc2dc\uc998\uc774 \ub354\ud574\uc84c\ub2e4. \uc5f4\uc5ec\uc12f \uc0b4 \ub54c\ubd80\ud130 \uc0f9\uc1a1\uc744 \uc990\uaca8\ub4e4\uc5c8\ub358 \ucf58\ub3c8\uc740 \ud2b9\ud788 \uc790\ub044 \ube0c\ub810\uc758 \uc74c\uc545\uc744 \uc990\uaca8 \ub4e4\uc5c8\uc73c\uba70, \ud30c\ub9ac\ub97c \uc74c\uc545\uc801\uc778 \uc758\ubbf8\uc5d0\uc11c \uc81c 2\uc758 \uace0\ud5a5\uc73c\ub85c \uc5ec\uae30\ub294 \ub4ef\ud558\ub2e4."} {"question": "\uc720\uae30\uccb4 \uc0ac\uc0c1\uacfc \uc9c4\ubcf4\uc8fc\uc758 \uc138\uacc4\uad00\uc758 \uc601\ud5a5\uc744 \ubc1b\uc740 \uce74\uc774\ud37c\uc758 \ubb38\ud654\uad00\uc744 \uc774\ub974\ub294 \ub9d0\uc740?", "paragraph": "\uce74\uc774\ud37c\uc758 \ubb38\ud654\uad00\uc5d0\uc11c \uc911\uc694\ud55c \ud558\ub098\ub294 \u2018\uc528(kiem)\u2019 \uc0ac\uc0c1\uc774\ub2e4. \ubcc0\uc885\uae38\uc5d0 \ub530\ub974\uba74 \uc774\u2018\uc528\u2019\ub294 \uc601\uc6d0 \uc804\uc5d0 \uc608\uc815\ub418\uc5c8\uc73c\uba70, \ucc3d\uc870\uc2dc\uc5d0 \uc778\ub958\uc758 \uc870\uc0c1\uc5d0\uac8c \ub300\ud45c\uc801\uc73c\ub85c \uc8fc\uc5b4\uc84c\uc73c\uba70, \ucc98\uc74c\ubd80\ud130 \uc0ac\ud68c\uc801\uc131\uaca9\uc744 \uc9c0\ub2c8\uace0 \uc788\uae30 \ub54c\ubb38\uc5d0 \uc5ed\uc0ac\uc758 \ubc1c\uc804\uc5d0 \ub530\ub77c \ubc1c\uc544, \ubc1c\uc804\ub420 \uc218\ubc16\uc5d0 \uc5c6\ub2e4\uace0 \ud55c\ub2e4. \uce74\uc774\ud37c\uc758 \ubb38\ud654\uad00\uc758 \ud575\uc2ec\uc740 \u2018\uacc4\ubc1c \uc0ac\uc0c1(ontluikings gedachte)\u2019\uacfc \u2018\uc720\uae30\uccb4 \uc0ac\uc0c1(organische gedachte)\u2019\uc774 \uc788\ub294\ub370 \u2018\uacc4\ubc1c \uc0ac\uc0c1\u2019\uc774\ub780 \ucc3d\uc870\uc2dc\uc5d0 \ud558\ub098\ub2d8\uc774 \ubd80\uc5ec\ud558\uc2e0 \ub2a5\ub825\ub4e4\uc744 \ucd5c\ub300\ud55c\uc73c\ub85c \ubc1c\ud718\ud558\uace0 \ubc1c\uc804\uc2dc\ucf1c\uc57c \ud55c\ub2e4\ub294 \uc0ac\uc0c1\uc774\ub77c\uace0 \ud55c\ub2e4. \u2018\uc720\uae30\uccb4 \uc0ac\uc0c1\u2019\uc774\ub780 \u201c19\uc138\uae30\uc758 \ub0ad\ub9cc\uc8fc\uc758(H\u00f6lderlin \ub4f1)\uc640 \uc774\uc0c1\uc8fc\uc758 \ucca0\ud559(Schelling\uacfc Hegel \ub4f1)\uc5d0\uc11c \ube44\ub86f\ub41c \uc0ac\uc0c1\uc778\ub370, \uc885\uc804\uc758 \uae30\uacc4\uc801 \uc6b0\uc8fc\uad00\uc5d0 \ubc18\ub300\ud558\uace0 \uc2e4\uc7ac(\u5be6\u5728)\ub97c \uc0dd\uba85 \uc788\ub294 \uc720\uae30\uccb4\ub85c \ud30c\uc545\ud558\ub294 \uac83\u201d\uc774\ub77c\uace0 \ud55c\ub2e4. \uc774\ub7ec\ud55c \uc0ac\uc0c1\uc5d0 \uc758\ud558\uba74, \uc720\uae30\uccb4\ub294 \uc0dd\uba85\uc774 \uc788\uc73c\ubbc0\ub85c \uc131\uc7a5\ud558\uace0 \ubc1c\uc804\ud558\uba70, \uc720\uae30\uccb4\ub294 \ub2e8\uc21c\ud788 \uac01 \ubd80\ubd84\ub4e4\uc758 \uc5f0\uacb0\uc774 \uc544\ub2c8\ub77c \uc804\uccb4\ub85c\uc11c \uc870\ud654\uc640 \ud1b5\uc77c\uc131\uc744 \uc774\ub8e8\uace0 \uc788\ub2e4. \ub530\ub77c\uc11c \uc774\ub7f0 \uc0ac\uc0c1\uc744 \uac00\uc9c4 \uc0ac\ub78c\uc740 \ub099\uad00\uc8fc\uc758, \uc9c4\ubcf4\uc8fc\uc758 \uc138\uacc4\uad00\uc744 \uac00\uc9c0\uac8c \ub41c\ub2e4. \uce74\uc774\ud37c\ub294 \ubc14\ub85c \uc774\ub7ec\ud55c 19\uc138\uae30 \ud6c4\ubc18\uc758 \uc720\uae30\uccb4 \uc0ac\uc0c1\uacfc \uc9c4\ubcf4\uc8fc\uc758 \uc138\uacc4\uad00\uc758 \uc601\ud5a5\uc744 \ubc1b\uc544\uc11c \u2018\ub099\uad00\uc801\uc778 \ubb38\ud654\uad00\u2019\uc744 \uc804\uac1c\ud558\uc600\ub2e4\uace0 \ubcc0\uc885\uae38\uc740 \ud3c9\uac00\ud55c\ub2e4. \uc774\ub7f0 \uadf8\uc758 \ud3c9\uac00\ub294 \uc77c\ubc18\uc801\uc73c\ub85c \ubc1b\uc544\ub4e4\uc5ec\uc9c0\uace0 \uc788\ub2e4.", "answer": "\ub099\uad00\uc801\uc778 \ubb38\ud654\uad00", "sentence": "\uce74\uc774\ud37c\ub294 \ubc14\ub85c \uc774\ub7ec\ud55c 19\uc138\uae30 \ud6c4\ubc18\uc758 \uc720\uae30\uccb4 \uc0ac\uc0c1\uacfc \uc9c4\ubcf4\uc8fc\uc758 \uc138\uacc4\uad00\uc758 \uc601\ud5a5\uc744 \ubc1b\uc544\uc11c \u2018 \ub099\uad00\uc801\uc778 \ubb38\ud654\uad00 \u2019\uc744 \uc804\uac1c\ud558\uc600\ub2e4\uace0 \ubcc0\uc885\uae38\uc740 \ud3c9\uac00\ud55c\ub2e4.", "paragraph_sentence": "\uce74\uc774\ud37c\uc758 \ubb38\ud654\uad00\uc5d0\uc11c \uc911\uc694\ud55c \ud558\ub098\ub294 \u2018\uc528(kiem)\u2019 \uc0ac\uc0c1\uc774\ub2e4. \ubcc0\uc885\uae38\uc5d0 \ub530\ub974\uba74 \uc774\u2018\uc528\u2019\ub294 \uc601\uc6d0 \uc804\uc5d0 \uc608\uc815\ub418\uc5c8\uc73c\uba70, \ucc3d\uc870\uc2dc\uc5d0 \uc778\ub958\uc758 \uc870\uc0c1\uc5d0\uac8c \ub300\ud45c\uc801\uc73c\ub85c \uc8fc\uc5b4\uc84c\uc73c\uba70, \ucc98\uc74c\ubd80\ud130 \uc0ac\ud68c\uc801\uc131\uaca9\uc744 \uc9c0\ub2c8\uace0 \uc788\uae30 \ub54c\ubb38\uc5d0 \uc5ed\uc0ac\uc758 \ubc1c\uc804\uc5d0 \ub530\ub77c \ubc1c\uc544, \ubc1c\uc804\ub420 \uc218\ubc16\uc5d0 \uc5c6\ub2e4\uace0 \ud55c\ub2e4. \uce74\uc774\ud37c\uc758 \ubb38\ud654\uad00\uc758 \ud575\uc2ec\uc740 \u2018\uacc4\ubc1c \uc0ac\uc0c1(ontluikings gedachte)\u2019\uacfc \u2018\uc720\uae30\uccb4 \uc0ac\uc0c1(organische gedachte)\u2019\uc774 \uc788\ub294\ub370 \u2018\uacc4\ubc1c \uc0ac\uc0c1\u2019\uc774\ub780 \ucc3d\uc870\uc2dc\uc5d0 \ud558\ub098\ub2d8\uc774 \ubd80\uc5ec\ud558\uc2e0 \ub2a5\ub825\ub4e4\uc744 \ucd5c\ub300\ud55c\uc73c\ub85c \ubc1c\ud718\ud558\uace0 \ubc1c\uc804\uc2dc\ucf1c\uc57c \ud55c\ub2e4\ub294 \uc0ac\uc0c1\uc774\ub77c\uace0 \ud55c\ub2e4. \u2018\uc720\uae30\uccb4 \uc0ac\uc0c1\u2019\uc774\ub780 \u201c19\uc138\uae30\uc758 \ub0ad\ub9cc\uc8fc\uc758(H\u00f6lderlin \ub4f1)\uc640 \uc774\uc0c1\uc8fc\uc758 \ucca0\ud559(Schelling\uacfc Hegel \ub4f1)\uc5d0\uc11c \ube44\ub86f\ub41c \uc0ac\uc0c1\uc778\ub370, \uc885\uc804\uc758 \uae30\uacc4\uc801 \uc6b0\uc8fc\uad00\uc5d0 \ubc18\ub300\ud558\uace0 \uc2e4\uc7ac(\u5be6\u5728)\ub97c \uc0dd\uba85 \uc788\ub294 \uc720\uae30\uccb4\ub85c \ud30c\uc545\ud558\ub294 \uac83\u201d\uc774\ub77c\uace0 \ud55c\ub2e4. \uc774\ub7ec\ud55c \uc0ac\uc0c1\uc5d0 \uc758\ud558\uba74, \uc720\uae30\uccb4\ub294 \uc0dd\uba85\uc774 \uc788\uc73c\ubbc0\ub85c \uc131\uc7a5\ud558\uace0 \ubc1c\uc804\ud558\uba70, \uc720\uae30\uccb4\ub294 \ub2e8\uc21c\ud788 \uac01 \ubd80\ubd84\ub4e4\uc758 \uc5f0\uacb0\uc774 \uc544\ub2c8\ub77c \uc804\uccb4\ub85c\uc11c \uc870\ud654\uc640 \ud1b5\uc77c\uc131\uc744 \uc774\ub8e8\uace0 \uc788\ub2e4. \ub530\ub77c\uc11c \uc774\ub7f0 \uc0ac\uc0c1\uc744 \uac00\uc9c4 \uc0ac\ub78c\uc740 \ub099\uad00\uc8fc\uc758, \uc9c4\ubcf4\uc8fc\uc758 \uc138\uacc4\uad00\uc744 \uac00\uc9c0\uac8c \ub41c\ub2e4. \uce74\uc774\ud37c\ub294 \ubc14\ub85c \uc774\ub7ec\ud55c 19\uc138\uae30 \ud6c4\ubc18\uc758 \uc720\uae30\uccb4 \uc0ac\uc0c1\uacfc \uc9c4\ubcf4\uc8fc\uc758 \uc138\uacc4\uad00\uc758 \uc601\ud5a5\uc744 \ubc1b\uc544\uc11c \u2018 \ub099\uad00\uc801\uc778 \ubb38\ud654\uad00 \u2019\uc744 \uc804\uac1c\ud558\uc600\ub2e4\uace0 \ubcc0\uc885\uae38\uc740 \ud3c9\uac00\ud55c\ub2e4. \uc774\ub7f0 \uadf8\uc758 \ud3c9\uac00\ub294 \uc77c\ubc18\uc801\uc73c\ub85c \ubc1b\uc544\ub4e4\uc5ec\uc9c0\uace0 \uc788\ub2e4.", "paragraph_answer": "\uce74\uc774\ud37c\uc758 \ubb38\ud654\uad00\uc5d0\uc11c \uc911\uc694\ud55c \ud558\ub098\ub294 \u2018\uc528(kiem)\u2019 \uc0ac\uc0c1\uc774\ub2e4. \ubcc0\uc885\uae38\uc5d0 \ub530\ub974\uba74 \uc774\u2018\uc528\u2019\ub294 \uc601\uc6d0 \uc804\uc5d0 \uc608\uc815\ub418\uc5c8\uc73c\uba70, \ucc3d\uc870\uc2dc\uc5d0 \uc778\ub958\uc758 \uc870\uc0c1\uc5d0\uac8c \ub300\ud45c\uc801\uc73c\ub85c \uc8fc\uc5b4\uc84c\uc73c\uba70, \ucc98\uc74c\ubd80\ud130 \uc0ac\ud68c\uc801\uc131\uaca9\uc744 \uc9c0\ub2c8\uace0 \uc788\uae30 \ub54c\ubb38\uc5d0 \uc5ed\uc0ac\uc758 \ubc1c\uc804\uc5d0 \ub530\ub77c \ubc1c\uc544, \ubc1c\uc804\ub420 \uc218\ubc16\uc5d0 \uc5c6\ub2e4\uace0 \ud55c\ub2e4. \uce74\uc774\ud37c\uc758 \ubb38\ud654\uad00\uc758 \ud575\uc2ec\uc740 \u2018\uacc4\ubc1c \uc0ac\uc0c1(ontluikings gedachte)\u2019\uacfc \u2018\uc720\uae30\uccb4 \uc0ac\uc0c1(organische gedachte)\u2019\uc774 \uc788\ub294\ub370 \u2018\uacc4\ubc1c \uc0ac\uc0c1\u2019\uc774\ub780 \ucc3d\uc870\uc2dc\uc5d0 \ud558\ub098\ub2d8\uc774 \ubd80\uc5ec\ud558\uc2e0 \ub2a5\ub825\ub4e4\uc744 \ucd5c\ub300\ud55c\uc73c\ub85c \ubc1c\ud718\ud558\uace0 \ubc1c\uc804\uc2dc\ucf1c\uc57c \ud55c\ub2e4\ub294 \uc0ac\uc0c1\uc774\ub77c\uace0 \ud55c\ub2e4. \u2018\uc720\uae30\uccb4 \uc0ac\uc0c1\u2019\uc774\ub780 \u201c19\uc138\uae30\uc758 \ub0ad\ub9cc\uc8fc\uc758(H\u00f6lderlin \ub4f1)\uc640 \uc774\uc0c1\uc8fc\uc758 \ucca0\ud559(Schelling\uacfc Hegel \ub4f1)\uc5d0\uc11c \ube44\ub86f\ub41c \uc0ac\uc0c1\uc778\ub370, \uc885\uc804\uc758 \uae30\uacc4\uc801 \uc6b0\uc8fc\uad00\uc5d0 \ubc18\ub300\ud558\uace0 \uc2e4\uc7ac(\u5be6\u5728)\ub97c \uc0dd\uba85 \uc788\ub294 \uc720\uae30\uccb4\ub85c \ud30c\uc545\ud558\ub294 \uac83\u201d\uc774\ub77c\uace0 \ud55c\ub2e4. \uc774\ub7ec\ud55c \uc0ac\uc0c1\uc5d0 \uc758\ud558\uba74, \uc720\uae30\uccb4\ub294 \uc0dd\uba85\uc774 \uc788\uc73c\ubbc0\ub85c \uc131\uc7a5\ud558\uace0 \ubc1c\uc804\ud558\uba70, \uc720\uae30\uccb4\ub294 \ub2e8\uc21c\ud788 \uac01 \ubd80\ubd84\ub4e4\uc758 \uc5f0\uacb0\uc774 \uc544\ub2c8\ub77c \uc804\uccb4\ub85c\uc11c \uc870\ud654\uc640 \ud1b5\uc77c\uc131\uc744 \uc774\ub8e8\uace0 \uc788\ub2e4. \ub530\ub77c\uc11c \uc774\ub7f0 \uc0ac\uc0c1\uc744 \uac00\uc9c4 \uc0ac\ub78c\uc740 \ub099\uad00\uc8fc\uc758, \uc9c4\ubcf4\uc8fc\uc758 \uc138\uacc4\uad00\uc744 \uac00\uc9c0\uac8c \ub41c\ub2e4. \uce74\uc774\ud37c\ub294 \ubc14\ub85c \uc774\ub7ec\ud55c 19\uc138\uae30 \ud6c4\ubc18\uc758 \uc720\uae30\uccb4 \uc0ac\uc0c1\uacfc \uc9c4\ubcf4\uc8fc\uc758 \uc138\uacc4\uad00\uc758 \uc601\ud5a5\uc744 \ubc1b\uc544\uc11c \u2018 \ub099\uad00\uc801\uc778 \ubb38\ud654\uad00 \u2019\uc744 \uc804\uac1c\ud558\uc600\ub2e4\uace0 \ubcc0\uc885\uae38\uc740 \ud3c9\uac00\ud55c\ub2e4. \uc774\ub7f0 \uadf8\uc758 \ud3c9\uac00\ub294 \uc77c\ubc18\uc801\uc73c\ub85c \ubc1b\uc544\ub4e4\uc5ec\uc9c0\uace0 \uc788\ub2e4.", "sentence_answer": "\uce74\uc774\ud37c\ub294 \ubc14\ub85c \uc774\ub7ec\ud55c 19\uc138\uae30 \ud6c4\ubc18\uc758 \uc720\uae30\uccb4 \uc0ac\uc0c1\uacfc \uc9c4\ubcf4\uc8fc\uc758 \uc138\uacc4\uad00\uc758 \uc601\ud5a5\uc744 \ubc1b\uc544\uc11c \u2018 \ub099\uad00\uc801\uc778 \ubb38\ud654\uad00 \u2019\uc744 \uc804\uac1c\ud558\uc600\ub2e4\uace0 \ubcc0\uc885\uae38\uc740 \ud3c9\uac00\ud55c\ub2e4.", "paragraph_id": "6505013-1-1", "paragraph_question": "question: \uc720\uae30\uccb4 \uc0ac\uc0c1\uacfc \uc9c4\ubcf4\uc8fc\uc758 \uc138\uacc4\uad00\uc758 \uc601\ud5a5\uc744 \ubc1b\uc740 \uce74\uc774\ud37c\uc758 \ubb38\ud654\uad00\uc744 \uc774\ub974\ub294 \ub9d0\uc740?, context: \uce74\uc774\ud37c\uc758 \ubb38\ud654\uad00\uc5d0\uc11c \uc911\uc694\ud55c \ud558\ub098\ub294 \u2018\uc528(kiem)\u2019 \uc0ac\uc0c1\uc774\ub2e4. \ubcc0\uc885\uae38\uc5d0 \ub530\ub974\uba74 \uc774\u2018\uc528\u2019\ub294 \uc601\uc6d0 \uc804\uc5d0 \uc608\uc815\ub418\uc5c8\uc73c\uba70, \ucc3d\uc870\uc2dc\uc5d0 \uc778\ub958\uc758 \uc870\uc0c1\uc5d0\uac8c \ub300\ud45c\uc801\uc73c\ub85c \uc8fc\uc5b4\uc84c\uc73c\uba70, \ucc98\uc74c\ubd80\ud130 \uc0ac\ud68c\uc801\uc131\uaca9\uc744 \uc9c0\ub2c8\uace0 \uc788\uae30 \ub54c\ubb38\uc5d0 \uc5ed\uc0ac\uc758 \ubc1c\uc804\uc5d0 \ub530\ub77c \ubc1c\uc544, \ubc1c\uc804\ub420 \uc218\ubc16\uc5d0 \uc5c6\ub2e4\uace0 \ud55c\ub2e4. \uce74\uc774\ud37c\uc758 \ubb38\ud654\uad00\uc758 \ud575\uc2ec\uc740 \u2018\uacc4\ubc1c \uc0ac\uc0c1(ontluikings gedachte)\u2019\uacfc \u2018\uc720\uae30\uccb4 \uc0ac\uc0c1(organische gedachte)\u2019\uc774 \uc788\ub294\ub370 \u2018\uacc4\ubc1c \uc0ac\uc0c1\u2019\uc774\ub780 \ucc3d\uc870\uc2dc\uc5d0 \ud558\ub098\ub2d8\uc774 \ubd80\uc5ec\ud558\uc2e0 \ub2a5\ub825\ub4e4\uc744 \ucd5c\ub300\ud55c\uc73c\ub85c \ubc1c\ud718\ud558\uace0 \ubc1c\uc804\uc2dc\ucf1c\uc57c \ud55c\ub2e4\ub294 \uc0ac\uc0c1\uc774\ub77c\uace0 \ud55c\ub2e4. \u2018\uc720\uae30\uccb4 \uc0ac\uc0c1\u2019\uc774\ub780 \u201c19\uc138\uae30\uc758 \ub0ad\ub9cc\uc8fc\uc758(H\u00f6lderlin \ub4f1)\uc640 \uc774\uc0c1\uc8fc\uc758 \ucca0\ud559(Schelling\uacfc Hegel \ub4f1)\uc5d0\uc11c \ube44\ub86f\ub41c \uc0ac\uc0c1\uc778\ub370, \uc885\uc804\uc758 \uae30\uacc4\uc801 \uc6b0\uc8fc\uad00\uc5d0 \ubc18\ub300\ud558\uace0 \uc2e4\uc7ac(\u5be6\u5728)\ub97c \uc0dd\uba85 \uc788\ub294 \uc720\uae30\uccb4\ub85c \ud30c\uc545\ud558\ub294 \uac83\u201d\uc774\ub77c\uace0 \ud55c\ub2e4. \uc774\ub7ec\ud55c \uc0ac\uc0c1\uc5d0 \uc758\ud558\uba74, \uc720\uae30\uccb4\ub294 \uc0dd\uba85\uc774 \uc788\uc73c\ubbc0\ub85c \uc131\uc7a5\ud558\uace0 \ubc1c\uc804\ud558\uba70, \uc720\uae30\uccb4\ub294 \ub2e8\uc21c\ud788 \uac01 \ubd80\ubd84\ub4e4\uc758 \uc5f0\uacb0\uc774 \uc544\ub2c8\ub77c \uc804\uccb4\ub85c\uc11c \uc870\ud654\uc640 \ud1b5\uc77c\uc131\uc744 \uc774\ub8e8\uace0 \uc788\ub2e4. \ub530\ub77c\uc11c \uc774\ub7f0 \uc0ac\uc0c1\uc744 \uac00\uc9c4 \uc0ac\ub78c\uc740 \ub099\uad00\uc8fc\uc758, \uc9c4\ubcf4\uc8fc\uc758 \uc138\uacc4\uad00\uc744 \uac00\uc9c0\uac8c \ub41c\ub2e4. \uce74\uc774\ud37c\ub294 \ubc14\ub85c \uc774\ub7ec\ud55c 19\uc138\uae30 \ud6c4\ubc18\uc758 \uc720\uae30\uccb4 \uc0ac\uc0c1\uacfc \uc9c4\ubcf4\uc8fc\uc758 \uc138\uacc4\uad00\uc758 \uc601\ud5a5\uc744 \ubc1b\uc544\uc11c \u2018\ub099\uad00\uc801\uc778 \ubb38\ud654\uad00\u2019\uc744 \uc804\uac1c\ud558\uc600\ub2e4\uace0 \ubcc0\uc885\uae38\uc740 \ud3c9\uac00\ud55c\ub2e4. \uc774\ub7f0 \uadf8\uc758 \ud3c9\uac00\ub294 \uc77c\ubc18\uc801\uc73c\ub85c \ubc1b\uc544\ub4e4\uc5ec\uc9c0\uace0 \uc788\ub2e4."} {"question": "\ub300\ubd80\ubd84\uc758 \uacbd\uc6b0\uac00 \uc544\ub2cc \uc77c\ubd80 \ub524\ubbf8\ub4e4\uc740 \ubb34\uc5c7\uc744 \ud0dd\ud558\uc5ec \ubaa9\uc228\uc744 \uc783\uae30\ub3c4 \ud558\uc600\ub294\uac00?", "paragraph": "\ub9ce\uc740 \ub524\ubbf8\ub4e4\uc774 \ubd88\uacbd\uc744 \ubc94\ud588\ub2e4\ub294 \uc774\uc720\ub85c \ubaa9\uc228\uc744 \uc783\uc5c8\ub2e4. \uc77c\ubd80 \ub524\ubbf8\ub4e4\uc774 \uc21c\uad50\ub97c \ud0dd\ud558\uae30\ub3c4 \ud588\uc9c0\ub9cc \ub300\ubd80\ubd84 \uc220\uc774 \ucde8\ud558\uac70\ub098 \uc815\uc2e0 \uc774\uc0c1\uc790\uc758 \uacbd\uc6b0\uac00 \ub9ce\uc558\ub2e4\uace0 \ud558\ub294\ub370 \uc815\uce58\uc801\uc778 \uc774\uc720 \ud639\uc740 \uc0ac\uc801\uc778 \uc6d0\ud55c \ub4f1\uc73c\ub85c \uc77c\ubc29\uc801\uc778 \uc0ac\ud615\uc744 \uc218\ud589\ud558\ub294 \uacbd\uc6b0\ub294 \ud754\uce58 \uc54a\uc558\ub2e4. \uc0ac\ud615\uac10\uc778 \ubaa8\ub3c5 \ud589\uc704\ub294 \ubb34\uc2ac\ub9bc\ub4e4\uc5d0\uac8c \uc544\ucca8\uc744 \ud558\uac70\ub098 \ub450\ub824\uc6c0\uc5d0 \ub5a8\uc5b4\uc57c \ud588\ub358 \uc774\uc720\uc600\ub2e4. \uc5d0\ub4dc\uc6cc\ub4dc \uc70c\ub9ac\uc5c4 \ub808\uc778\uc774 \uc774\uc9d1\ud2b8 \uc5ec\ud589\uae30\uc5d0\uc11c \uc37c\ub4ef\uc774 \ub2f9\uc2dc \uc720\ub300\uc778\ub4e4\uc740 \uacb0\ucf54 \ub9d0\uc744 \ud568\ubd80\ub85c \ub0b4\ubc49\uc9c0 \uc54a\uc558\uc73c\uba70 \ud22c\ub974\ud06c\uc871\uacfc \uc544\ub78d\uc778\uc744 \ud5a5\ud55c \ubaa8\uc695\uc801\uc778 \ub9d0\uc744 \uc785\uc5d0 \ub2f4\uc9c0\ub3c4 \uc54a\uc558\ub2e4. \uc774\ub294 \uc218\ub9ce\uc740 \uc0ac\ub78c\ub4e4\uc774 \uc624\ud574\ub97c \uc0ac\uac70\ub098, \uc2e4\uc218\ub85c \uad81\uc9c0\uc5d0 \ubab0\ub824 \uafb8\ub780\uc774\ub098 \uc608\uc5b8\uc790\ub97c \uc695\ub418\uac8c \ud588\ub2e4\ub294 \uc774\uc720\ub85c \uc0ac\ud615\ub2f9\ud588\uae30 \ub54c\ubb38\uc774\ub2e4. \ubd88\uacbd\ud55c \ud589\uc704\uc5d0 \ub300\ud55c \uc751\uc9d5\uc740 \ub3c4\ucc98\uc5d0\uc11c \uc77c\uc5b4\ub0ac\ub2e4 : \ud280\ub2c8\uc2a4(1876\ub144), \ud558\ub9c8\ub2e8(1876\ub144), \uc54c\ub808\ud3ec(1889\ub144), \ud14c\ud5e4\ub780(1895\ub144), \ubaa8\uc220(1911\ub144)", "answer": "\uc21c\uad50", "sentence": "\uc77c\ubd80 \ub524\ubbf8\ub4e4\uc774 \uc21c\uad50 \ub97c \ud0dd\ud558\uae30\ub3c4 \ud588\uc9c0\ub9cc \ub300\ubd80\ubd84 \uc220\uc774 \ucde8\ud558\uac70\ub098 \uc815\uc2e0 \uc774\uc0c1\uc790\uc758 \uacbd\uc6b0\uac00 \ub9ce\uc558\ub2e4\uace0 \ud558\ub294\ub370 \uc815\uce58\uc801\uc778 \uc774\uc720 \ud639\uc740 \uc0ac\uc801\uc778 \uc6d0\ud55c \ub4f1\uc73c\ub85c \uc77c\ubc29\uc801\uc778 \uc0ac\ud615\uc744 \uc218\ud589\ud558\ub294 \uacbd\uc6b0\ub294 \ud754\uce58 \uc54a\uc558\ub2e4.", "paragraph_sentence": "\ub9ce\uc740 \ub524\ubbf8\ub4e4\uc774 \ubd88\uacbd\uc744 \ubc94\ud588\ub2e4\ub294 \uc774\uc720\ub85c \ubaa9\uc228\uc744 \uc783\uc5c8\ub2e4. \uc77c\ubd80 \ub524\ubbf8\ub4e4\uc774 \uc21c\uad50 \ub97c \ud0dd\ud558\uae30\ub3c4 \ud588\uc9c0\ub9cc \ub300\ubd80\ubd84 \uc220\uc774 \ucde8\ud558\uac70\ub098 \uc815\uc2e0 \uc774\uc0c1\uc790\uc758 \uacbd\uc6b0\uac00 \ub9ce\uc558\ub2e4\uace0 \ud558\ub294\ub370 \uc815\uce58\uc801\uc778 \uc774\uc720 \ud639\uc740 \uc0ac\uc801\uc778 \uc6d0\ud55c \ub4f1\uc73c\ub85c \uc77c\ubc29\uc801\uc778 \uc0ac\ud615\uc744 \uc218\ud589\ud558\ub294 \uacbd\uc6b0\ub294 \ud754\uce58 \uc54a\uc558\ub2e4. \uc0ac\ud615\uac10\uc778 \ubaa8\ub3c5 \ud589\uc704\ub294 \ubb34\uc2ac\ub9bc\ub4e4\uc5d0\uac8c \uc544\ucca8\uc744 \ud558\uac70\ub098 \ub450\ub824\uc6c0\uc5d0 \ub5a8\uc5b4\uc57c \ud588\ub358 \uc774\uc720\uc600\ub2e4. \uc5d0\ub4dc\uc6cc\ub4dc \uc70c\ub9ac\uc5c4 \ub808\uc778\uc774 \uc774\uc9d1\ud2b8 \uc5ec\ud589\uae30\uc5d0\uc11c \uc37c\ub4ef\uc774 \ub2f9\uc2dc \uc720\ub300\uc778\ub4e4\uc740 \uacb0\ucf54 \ub9d0\uc744 \ud568\ubd80\ub85c \ub0b4\ubc49\uc9c0 \uc54a\uc558\uc73c\uba70 \ud22c\ub974\ud06c\uc871\uacfc \uc544\ub78d\uc778\uc744 \ud5a5\ud55c \ubaa8\uc695\uc801\uc778 \ub9d0\uc744 \uc785\uc5d0 \ub2f4\uc9c0\ub3c4 \uc54a\uc558\ub2e4. \uc774\ub294 \uc218\ub9ce\uc740 \uc0ac\ub78c\ub4e4\uc774 \uc624\ud574\ub97c \uc0ac\uac70\ub098, \uc2e4\uc218\ub85c \uad81\uc9c0\uc5d0 \ubab0\ub824 \uafb8\ub780\uc774\ub098 \uc608\uc5b8\uc790\ub97c \uc695\ub418\uac8c \ud588\ub2e4\ub294 \uc774\uc720\ub85c \uc0ac\ud615\ub2f9\ud588\uae30 \ub54c\ubb38\uc774\ub2e4. \ubd88\uacbd\ud55c \ud589\uc704\uc5d0 \ub300\ud55c \uc751\uc9d5\uc740 \ub3c4\ucc98\uc5d0\uc11c \uc77c\uc5b4\ub0ac\ub2e4 : \ud280\ub2c8\uc2a4(1876\ub144), \ud558\ub9c8\ub2e8(1876\ub144), \uc54c\ub808\ud3ec(1889\ub144), \ud14c\ud5e4\ub780(1895\ub144), \ubaa8\uc220(1911\ub144)", "paragraph_answer": "\ub9ce\uc740 \ub524\ubbf8\ub4e4\uc774 \ubd88\uacbd\uc744 \ubc94\ud588\ub2e4\ub294 \uc774\uc720\ub85c \ubaa9\uc228\uc744 \uc783\uc5c8\ub2e4. \uc77c\ubd80 \ub524\ubbf8\ub4e4\uc774 \uc21c\uad50 \ub97c \ud0dd\ud558\uae30\ub3c4 \ud588\uc9c0\ub9cc \ub300\ubd80\ubd84 \uc220\uc774 \ucde8\ud558\uac70\ub098 \uc815\uc2e0 \uc774\uc0c1\uc790\uc758 \uacbd\uc6b0\uac00 \ub9ce\uc558\ub2e4\uace0 \ud558\ub294\ub370 \uc815\uce58\uc801\uc778 \uc774\uc720 \ud639\uc740 \uc0ac\uc801\uc778 \uc6d0\ud55c \ub4f1\uc73c\ub85c \uc77c\ubc29\uc801\uc778 \uc0ac\ud615\uc744 \uc218\ud589\ud558\ub294 \uacbd\uc6b0\ub294 \ud754\uce58 \uc54a\uc558\ub2e4. \uc0ac\ud615\uac10\uc778 \ubaa8\ub3c5 \ud589\uc704\ub294 \ubb34\uc2ac\ub9bc\ub4e4\uc5d0\uac8c \uc544\ucca8\uc744 \ud558\uac70\ub098 \ub450\ub824\uc6c0\uc5d0 \ub5a8\uc5b4\uc57c \ud588\ub358 \uc774\uc720\uc600\ub2e4. \uc5d0\ub4dc\uc6cc\ub4dc \uc70c\ub9ac\uc5c4 \ub808\uc778\uc774 \uc774\uc9d1\ud2b8 \uc5ec\ud589\uae30\uc5d0\uc11c \uc37c\ub4ef\uc774 \ub2f9\uc2dc \uc720\ub300\uc778\ub4e4\uc740 \uacb0\ucf54 \ub9d0\uc744 \ud568\ubd80\ub85c \ub0b4\ubc49\uc9c0 \uc54a\uc558\uc73c\uba70 \ud22c\ub974\ud06c\uc871\uacfc \uc544\ub78d\uc778\uc744 \ud5a5\ud55c \ubaa8\uc695\uc801\uc778 \ub9d0\uc744 \uc785\uc5d0 \ub2f4\uc9c0\ub3c4 \uc54a\uc558\ub2e4. \uc774\ub294 \uc218\ub9ce\uc740 \uc0ac\ub78c\ub4e4\uc774 \uc624\ud574\ub97c \uc0ac\uac70\ub098, \uc2e4\uc218\ub85c \uad81\uc9c0\uc5d0 \ubab0\ub824 \uafb8\ub780\uc774\ub098 \uc608\uc5b8\uc790\ub97c \uc695\ub418\uac8c \ud588\ub2e4\ub294 \uc774\uc720\ub85c \uc0ac\ud615\ub2f9\ud588\uae30 \ub54c\ubb38\uc774\ub2e4. \ubd88\uacbd\ud55c \ud589\uc704\uc5d0 \ub300\ud55c \uc751\uc9d5\uc740 \ub3c4\ucc98\uc5d0\uc11c \uc77c\uc5b4\ub0ac\ub2e4 : \ud280\ub2c8\uc2a4(1876\ub144), \ud558\ub9c8\ub2e8(1876\ub144), \uc54c\ub808\ud3ec(1889\ub144), \ud14c\ud5e4\ub780(1895\ub144), \ubaa8\uc220(1911\ub144)", "sentence_answer": "\uc77c\ubd80 \ub524\ubbf8\ub4e4\uc774 \uc21c\uad50 \ub97c \ud0dd\ud558\uae30\ub3c4 \ud588\uc9c0\ub9cc \ub300\ubd80\ubd84 \uc220\uc774 \ucde8\ud558\uac70\ub098 \uc815\uc2e0 \uc774\uc0c1\uc790\uc758 \uacbd\uc6b0\uac00 \ub9ce\uc558\ub2e4\uace0 \ud558\ub294\ub370 \uc815\uce58\uc801\uc778 \uc774\uc720 \ud639\uc740 \uc0ac\uc801\uc778 \uc6d0\ud55c \ub4f1\uc73c\ub85c \uc77c\ubc29\uc801\uc778 \uc0ac\ud615\uc744 \uc218\ud589\ud558\ub294 \uacbd\uc6b0\ub294 \ud754\uce58 \uc54a\uc558\ub2e4.", "paragraph_id": "6528850-9-0", "paragraph_question": "question: \ub300\ubd80\ubd84\uc758 \uacbd\uc6b0\uac00 \uc544\ub2cc \uc77c\ubd80 \ub524\ubbf8\ub4e4\uc740 \ubb34\uc5c7\uc744 \ud0dd\ud558\uc5ec \ubaa9\uc228\uc744 \uc783\uae30\ub3c4 \ud558\uc600\ub294\uac00?, context: \ub9ce\uc740 \ub524\ubbf8\ub4e4\uc774 \ubd88\uacbd\uc744 \ubc94\ud588\ub2e4\ub294 \uc774\uc720\ub85c \ubaa9\uc228\uc744 \uc783\uc5c8\ub2e4. \uc77c\ubd80 \ub524\ubbf8\ub4e4\uc774 \uc21c\uad50\ub97c \ud0dd\ud558\uae30\ub3c4 \ud588\uc9c0\ub9cc \ub300\ubd80\ubd84 \uc220\uc774 \ucde8\ud558\uac70\ub098 \uc815\uc2e0 \uc774\uc0c1\uc790\uc758 \uacbd\uc6b0\uac00 \ub9ce\uc558\ub2e4\uace0 \ud558\ub294\ub370 \uc815\uce58\uc801\uc778 \uc774\uc720 \ud639\uc740 \uc0ac\uc801\uc778 \uc6d0\ud55c \ub4f1\uc73c\ub85c \uc77c\ubc29\uc801\uc778 \uc0ac\ud615\uc744 \uc218\ud589\ud558\ub294 \uacbd\uc6b0\ub294 \ud754\uce58 \uc54a\uc558\ub2e4. \uc0ac\ud615\uac10\uc778 \ubaa8\ub3c5 \ud589\uc704\ub294 \ubb34\uc2ac\ub9bc\ub4e4\uc5d0\uac8c \uc544\ucca8\uc744 \ud558\uac70\ub098 \ub450\ub824\uc6c0\uc5d0 \ub5a8\uc5b4\uc57c \ud588\ub358 \uc774\uc720\uc600\ub2e4. \uc5d0\ub4dc\uc6cc\ub4dc \uc70c\ub9ac\uc5c4 \ub808\uc778\uc774 \uc774\uc9d1\ud2b8 \uc5ec\ud589\uae30\uc5d0\uc11c \uc37c\ub4ef\uc774 \ub2f9\uc2dc \uc720\ub300\uc778\ub4e4\uc740 \uacb0\ucf54 \ub9d0\uc744 \ud568\ubd80\ub85c \ub0b4\ubc49\uc9c0 \uc54a\uc558\uc73c\uba70 \ud22c\ub974\ud06c\uc871\uacfc \uc544\ub78d\uc778\uc744 \ud5a5\ud55c \ubaa8\uc695\uc801\uc778 \ub9d0\uc744 \uc785\uc5d0 \ub2f4\uc9c0\ub3c4 \uc54a\uc558\ub2e4. \uc774\ub294 \uc218\ub9ce\uc740 \uc0ac\ub78c\ub4e4\uc774 \uc624\ud574\ub97c \uc0ac\uac70\ub098, \uc2e4\uc218\ub85c \uad81\uc9c0\uc5d0 \ubab0\ub824 \uafb8\ub780\uc774\ub098 \uc608\uc5b8\uc790\ub97c \uc695\ub418\uac8c \ud588\ub2e4\ub294 \uc774\uc720\ub85c \uc0ac\ud615\ub2f9\ud588\uae30 \ub54c\ubb38\uc774\ub2e4. \ubd88\uacbd\ud55c \ud589\uc704\uc5d0 \ub300\ud55c \uc751\uc9d5\uc740 \ub3c4\ucc98\uc5d0\uc11c \uc77c\uc5b4\ub0ac\ub2e4 : \ud280\ub2c8\uc2a4(1876\ub144), \ud558\ub9c8\ub2e8(1876\ub144), \uc54c\ub808\ud3ec(1889\ub144), \ud14c\ud5e4\ub780(1895\ub144), \ubaa8\uc220(1911\ub144)"} {"question": "\uc2a4\ud39c\uc2a4 D.\ub294 Single Ladies\uac00 \uc5b4\ub5a4 \ud48d\uc758 \ub290\ub08c\uc744 \ub2f4\uace0 \uc788\ub2e4\uace0 \ud588\ub294\uac00?", "paragraph": "\ud3c9\ub860\uac00\ub4e4\uc740 \uace1\uc758 \ub304\uc2a4 \ube44\ud2b8\ub97c \uce6d\ucc2c\ud588\ub294\ub370, PopMatters\uc758 \ucf5c\ub9b0 \ub9e5\uacfc\uc774\uc5b4\ub294 \u3008Single Ladies\u3009\ub97c \ube44\uc698\uc138 \ucd5c\uace0\uc758 \ub304\uc2a4 \ud2b8\ub799 \uc911 \ud558\ub098\ub77c\uba70 \ud638\ud3c9\ud588\ub2e4. Spence D. of IGN \ubba4\uc9c1\uc758 \uc2a4\ud39c\uc2a4 D.\ub294 \"\uc774 \uace1\uc774 \uce74\ub9ac\ube0c \ud48d\uc758 \ub290\ub08c\uc744 \ub2f4\uace0 \uc788\uc73c\uba70 \ucda4\uc744 \uc990\uae30\uc9c0 \uc54a\ub294 \uc0ac\ub78c\ub3c4 \uc5c9\ub369\uc774\ub97c \ud754\ub4e4\uba70 \ud765\uc5d0 \ucde8\ud574 \ubaa9\uc73c\ub85c \ub9ac\ub4ec\uc744 \ud0c8 \uc815\ub3c4\"\ub77c\uace0 \uc124\uba85\ud588\ub2e4. \u300a\ud734\uc2a4\ud134\ud06c\ub85c\ub2c8\ud074\u300b\uc758 \uc870\uc774 \uac8c\ub77c\ub294 \"\uc5c9\ub369\uc774\ub97c \ud754\ub4e4\uac8c \ud558\ub294 \ud074\ub7fd\" \ub178\ub798\ub77c\ub294 \uc810\uc5d0\uc11c \"Check on It\"\uacfc \ube44\uc2b7\ud558\ub2e4\uace0 \ud588\ub2e4. \u300a\uc5d4\ud130\ud14c\uc778\uba3c\ud2b8 \uc704\ud074\ub9ac\u300b\uc758 \ub808\uc544 \uadf8\ub9b0\ube14\ub7ab\uc740 \u3008Single Ladies\u3009\ub294 \"\uc560\uc778\uc5d0\uac8c \uac77\uc5b4\ucc28\uc774\uace0 \ub192\uc740 \uc904\ub118\uae30\ub97c \ud558\ub294 \uac83\uc774 \uc11e\uc778 \uc544\ucc14\ud55c \ub6f0\uae30\"\uac19\ub2e4\uace0 \uc37c\ub2e4. \u300a\uc6cc\uc2f1\ud134 \ud3ec\uc2a4\ud2b8\u300b\uc758 \uc544\ub2f4 \ub9c8\uc988\ub9e4\ub2c8\uc548\uc740 \ube44\uc698\uc138\ub294 \"\uc9c4\uc2ec\uc73c\ub85c \ubc18\ud56d\uc801\uc774\uba70 \ub3c5\ub9bd\uc801\uc778 \ubaa9\uc18c\ub9ac\"\ub85c \ubb34\ub300\uc5d0\uc11c \ub178\ub798\ub97c \ubd80\ub97c \ub54c \uc5ec\uc790\ub4e4\ub85c\ubd80\ud130 \uc9c0\uc9c0 \ubc1b\uc744 \uc218 \uc788\ub3c4\ub85d \uacc4\ud68d\ud588\ub2e4\uace0 \ub9ac\ubdf0\ud588\ub2e4. \ubc18\uba74 \uc5b4\ub5a4 \ud3c9\ub860\uac00\ub4e4\uc740 \u3008Single Ladies\u3009\ub85c \ubd80\ud130 \ubcc4\ub85c \uac10\uba85\ubc1b\uc9c0 \ubabb\ud588\ub2e4\ub294 \ud3c9\uc744 \ub0b4\ub193\uc558\ub294\ub370 \uc77c\ub840\ub85c \u300a\ube4c\ubcf4\ub4dc\u300b\uc758 \ub9c8\ub9ac\uc5d8 \ucf58\uc138\ud504\uc2dc\uc628\uc740 \"\ud3c9\ubc94\ud558\uac8c \ucff5\ucff5\uac70\ub9ac\uace0 \uadc0\uc5d0 \uac70\uc2ac\ub9ac\ub294 \uc18c\ub9ac\ub97c \ub0b4\ub294 \ub178\ub798\"\ub77c\uace0 \ub9ac\ubdf0\ud588\ub2e4. The Observer\uc758 \uc544\ub2f4 \ub9c8\ud14c\ub77c\ub294 \u3008Single Ladies\u3009\uc640 \"Diva\"\ub294 \uc5ec\uc7a5\ub0a8\uc790\uc758 \uc7a0\uc7ac\uc801\uc778 \uae30\uc6d0\uc774\uc790 \uc601\uac10\uc774 \ub418\ub294 \uace1\uc774\ub77c\uace0 \ubcf4\uc558\ub2e4. \uadf8\ub7ec\ub098 \u300a\uc2ac\ub79c\ud2b8 \ub9e4\uac70\uc9c4\u300b\uc758 \uc0b4 \uc2e0\ucf00\ub9c8\ub2c8\ub294 B'Day\uc758 \"\uc794\uc874\ubb3c\"\uc77c\ubfd0\uc774\ub77c\uba70 \ube44\ud310\ud588\ub2e4.", "answer": "\uce74\ub9ac\ube0c \ud48d", "sentence": "Spence D. of IGN \ubba4\uc9c1\uc758 \uc2a4\ud39c\uc2a4 D.\ub294 \"\uc774 \uace1\uc774 \uce74\ub9ac\ube0c \ud48d \uc758 \ub290\ub08c\uc744 \ub2f4\uace0 \uc788\uc73c\uba70 \ucda4\uc744 \uc990\uae30\uc9c0 \uc54a\ub294 \uc0ac\ub78c\ub3c4 \uc5c9\ub369\uc774\ub97c \ud754\ub4e4\uba70 \ud765\uc5d0 \ucde8\ud574 \ubaa9\uc73c\ub85c \ub9ac\ub4ec\uc744 \ud0c8 \uc815\ub3c4\"\ub77c\uace0 \uc124\uba85\ud588\ub2e4.", "paragraph_sentence": "\ud3c9\ub860\uac00\ub4e4\uc740 \uace1\uc758 \ub304\uc2a4 \ube44\ud2b8\ub97c \uce6d\ucc2c\ud588\ub294\ub370, PopMatters\uc758 \ucf5c\ub9b0 \ub9e5\uacfc\uc774\uc5b4\ub294 \u3008Single Ladies\u3009\ub97c \ube44\uc698\uc138 \ucd5c\uace0\uc758 \ub304\uc2a4 \ud2b8\ub799 \uc911 \ud558\ub098\ub77c\uba70 \ud638\ud3c9\ud588\ub2e4. Spence D. of IGN \ubba4\uc9c1\uc758 \uc2a4\ud39c\uc2a4 D.\ub294 \"\uc774 \uace1\uc774 \uce74\ub9ac\ube0c \ud48d \uc758 \ub290\ub08c\uc744 \ub2f4\uace0 \uc788\uc73c\uba70 \ucda4\uc744 \uc990\uae30\uc9c0 \uc54a\ub294 \uc0ac\ub78c\ub3c4 \uc5c9\ub369\uc774\ub97c \ud754\ub4e4\uba70 \ud765\uc5d0 \ucde8\ud574 \ubaa9\uc73c\ub85c \ub9ac\ub4ec\uc744 \ud0c8 \uc815\ub3c4\"\ub77c\uace0 \uc124\uba85\ud588\ub2e4. \u300a\ud734\uc2a4\ud134\ud06c\ub85c\ub2c8\ud074\u300b\uc758 \uc870\uc774 \uac8c\ub77c\ub294 \"\uc5c9\ub369\uc774\ub97c \ud754\ub4e4\uac8c \ud558\ub294 \ud074\ub7fd\" \ub178\ub798\ub77c\ub294 \uc810\uc5d0\uc11c \"Check on It\"\uacfc \ube44\uc2b7\ud558\ub2e4\uace0 \ud588\ub2e4. \u300a\uc5d4\ud130\ud14c\uc778\uba3c\ud2b8 \uc704\ud074\ub9ac\u300b\uc758 \ub808\uc544 \uadf8\ub9b0\ube14\ub7ab\uc740 \u3008Single Ladies\u3009\ub294 \"\uc560\uc778\uc5d0\uac8c \uac77\uc5b4\ucc28\uc774\uace0 \ub192\uc740 \uc904\ub118\uae30\ub97c \ud558\ub294 \uac83\uc774 \uc11e\uc778 \uc544\ucc14\ud55c \ub6f0\uae30\"\uac19\ub2e4\uace0 \uc37c\ub2e4. \u300a\uc6cc\uc2f1\ud134 \ud3ec\uc2a4\ud2b8\u300b\uc758 \uc544\ub2f4 \ub9c8\uc988\ub9e4\ub2c8\uc548\uc740 \ube44\uc698\uc138\ub294 \"\uc9c4\uc2ec\uc73c\ub85c \ubc18\ud56d\uc801\uc774\uba70 \ub3c5\ub9bd\uc801\uc778 \ubaa9\uc18c\ub9ac\"\ub85c \ubb34\ub300\uc5d0\uc11c \ub178\ub798\ub97c \ubd80\ub97c \ub54c \uc5ec\uc790\ub4e4\ub85c\ubd80\ud130 \uc9c0\uc9c0 \ubc1b\uc744 \uc218 \uc788\ub3c4\ub85d \uacc4\ud68d\ud588\ub2e4\uace0 \ub9ac\ubdf0\ud588\ub2e4. \ubc18\uba74 \uc5b4\ub5a4 \ud3c9\ub860\uac00\ub4e4\uc740 \u3008Single Ladies\u3009\ub85c \ubd80\ud130 \ubcc4\ub85c \uac10\uba85\ubc1b\uc9c0 \ubabb\ud588\ub2e4\ub294 \ud3c9\uc744 \ub0b4\ub193\uc558\ub294\ub370 \uc77c\ub840\ub85c \u300a\ube4c\ubcf4\ub4dc\u300b\uc758 \ub9c8\ub9ac\uc5d8 \ucf58\uc138\ud504\uc2dc\uc628\uc740 \"\ud3c9\ubc94\ud558\uac8c \ucff5\ucff5\uac70\ub9ac\uace0 \uadc0\uc5d0 \uac70\uc2ac\ub9ac\ub294 \uc18c\ub9ac\ub97c \ub0b4\ub294 \ub178\ub798\"\ub77c\uace0 \ub9ac\ubdf0\ud588\ub2e4. The Observer\uc758 \uc544\ub2f4 \ub9c8\ud14c\ub77c\ub294 \u3008Single Ladies\u3009\uc640 \"Diva\"\ub294 \uc5ec\uc7a5\ub0a8\uc790\uc758 \uc7a0\uc7ac\uc801\uc778 \uae30\uc6d0\uc774\uc790 \uc601\uac10\uc774 \ub418\ub294 \uace1\uc774\ub77c\uace0 \ubcf4\uc558\ub2e4. \uadf8\ub7ec\ub098 \u300a\uc2ac\ub79c\ud2b8 \ub9e4\uac70\uc9c4\u300b\uc758 \uc0b4 \uc2e0\ucf00\ub9c8\ub2c8\ub294 B'Day\uc758 \"\uc794\uc874\ubb3c\"\uc77c\ubfd0\uc774\ub77c\uba70 \ube44\ud310\ud588\ub2e4.", "paragraph_answer": "\ud3c9\ub860\uac00\ub4e4\uc740 \uace1\uc758 \ub304\uc2a4 \ube44\ud2b8\ub97c \uce6d\ucc2c\ud588\ub294\ub370, PopMatters\uc758 \ucf5c\ub9b0 \ub9e5\uacfc\uc774\uc5b4\ub294 \u3008Single Ladies\u3009\ub97c \ube44\uc698\uc138 \ucd5c\uace0\uc758 \ub304\uc2a4 \ud2b8\ub799 \uc911 \ud558\ub098\ub77c\uba70 \ud638\ud3c9\ud588\ub2e4. Spence D. of IGN \ubba4\uc9c1\uc758 \uc2a4\ud39c\uc2a4 D.\ub294 \"\uc774 \uace1\uc774 \uce74\ub9ac\ube0c \ud48d \uc758 \ub290\ub08c\uc744 \ub2f4\uace0 \uc788\uc73c\uba70 \ucda4\uc744 \uc990\uae30\uc9c0 \uc54a\ub294 \uc0ac\ub78c\ub3c4 \uc5c9\ub369\uc774\ub97c \ud754\ub4e4\uba70 \ud765\uc5d0 \ucde8\ud574 \ubaa9\uc73c\ub85c \ub9ac\ub4ec\uc744 \ud0c8 \uc815\ub3c4\"\ub77c\uace0 \uc124\uba85\ud588\ub2e4. \u300a\ud734\uc2a4\ud134\ud06c\ub85c\ub2c8\ud074\u300b\uc758 \uc870\uc774 \uac8c\ub77c\ub294 \"\uc5c9\ub369\uc774\ub97c \ud754\ub4e4\uac8c \ud558\ub294 \ud074\ub7fd\" \ub178\ub798\ub77c\ub294 \uc810\uc5d0\uc11c \"Check on It\"\uacfc \ube44\uc2b7\ud558\ub2e4\uace0 \ud588\ub2e4. \u300a\uc5d4\ud130\ud14c\uc778\uba3c\ud2b8 \uc704\ud074\ub9ac\u300b\uc758 \ub808\uc544 \uadf8\ub9b0\ube14\ub7ab\uc740 \u3008Single Ladies\u3009\ub294 \"\uc560\uc778\uc5d0\uac8c \uac77\uc5b4\ucc28\uc774\uace0 \ub192\uc740 \uc904\ub118\uae30\ub97c \ud558\ub294 \uac83\uc774 \uc11e\uc778 \uc544\ucc14\ud55c \ub6f0\uae30\"\uac19\ub2e4\uace0 \uc37c\ub2e4. \u300a\uc6cc\uc2f1\ud134 \ud3ec\uc2a4\ud2b8\u300b\uc758 \uc544\ub2f4 \ub9c8\uc988\ub9e4\ub2c8\uc548\uc740 \ube44\uc698\uc138\ub294 \"\uc9c4\uc2ec\uc73c\ub85c \ubc18\ud56d\uc801\uc774\uba70 \ub3c5\ub9bd\uc801\uc778 \ubaa9\uc18c\ub9ac\"\ub85c \ubb34\ub300\uc5d0\uc11c \ub178\ub798\ub97c \ubd80\ub97c \ub54c \uc5ec\uc790\ub4e4\ub85c\ubd80\ud130 \uc9c0\uc9c0 \ubc1b\uc744 \uc218 \uc788\ub3c4\ub85d \uacc4\ud68d\ud588\ub2e4\uace0 \ub9ac\ubdf0\ud588\ub2e4. \ubc18\uba74 \uc5b4\ub5a4 \ud3c9\ub860\uac00\ub4e4\uc740 \u3008Single Ladies\u3009\ub85c \ubd80\ud130 \ubcc4\ub85c \uac10\uba85\ubc1b\uc9c0 \ubabb\ud588\ub2e4\ub294 \ud3c9\uc744 \ub0b4\ub193\uc558\ub294\ub370 \uc77c\ub840\ub85c \u300a\ube4c\ubcf4\ub4dc\u300b\uc758 \ub9c8\ub9ac\uc5d8 \ucf58\uc138\ud504\uc2dc\uc628\uc740 \"\ud3c9\ubc94\ud558\uac8c \ucff5\ucff5\uac70\ub9ac\uace0 \uadc0\uc5d0 \uac70\uc2ac\ub9ac\ub294 \uc18c\ub9ac\ub97c \ub0b4\ub294 \ub178\ub798\"\ub77c\uace0 \ub9ac\ubdf0\ud588\ub2e4. The Observer\uc758 \uc544\ub2f4 \ub9c8\ud14c\ub77c\ub294 \u3008Single Ladies\u3009\uc640 \"Diva\"\ub294 \uc5ec\uc7a5\ub0a8\uc790\uc758 \uc7a0\uc7ac\uc801\uc778 \uae30\uc6d0\uc774\uc790 \uc601\uac10\uc774 \ub418\ub294 \uace1\uc774\ub77c\uace0 \ubcf4\uc558\ub2e4. \uadf8\ub7ec\ub098 \u300a\uc2ac\ub79c\ud2b8 \ub9e4\uac70\uc9c4\u300b\uc758 \uc0b4 \uc2e0\ucf00\ub9c8\ub2c8\ub294 B'Day\uc758 \"\uc794\uc874\ubb3c\"\uc77c\ubfd0\uc774\ub77c\uba70 \ube44\ud310\ud588\ub2e4.", "sentence_answer": "Spence D. of IGN \ubba4\uc9c1\uc758 \uc2a4\ud39c\uc2a4 D.\ub294 \"\uc774 \uace1\uc774 \uce74\ub9ac\ube0c \ud48d \uc758 \ub290\ub08c\uc744 \ub2f4\uace0 \uc788\uc73c\uba70 \ucda4\uc744 \uc990\uae30\uc9c0 \uc54a\ub294 \uc0ac\ub78c\ub3c4 \uc5c9\ub369\uc774\ub97c \ud754\ub4e4\uba70 \ud765\uc5d0 \ucde8\ud574 \ubaa9\uc73c\ub85c \ub9ac\ub4ec\uc744 \ud0c8 \uc815\ub3c4\"\ub77c\uace0 \uc124\uba85\ud588\ub2e4.", "paragraph_id": "5808934-10-0", "paragraph_question": "question: \uc2a4\ud39c\uc2a4 D.\ub294 Single Ladies\uac00 \uc5b4\ub5a4 \ud48d\uc758 \ub290\ub08c\uc744 \ub2f4\uace0 \uc788\ub2e4\uace0 \ud588\ub294\uac00?, context: \ud3c9\ub860\uac00\ub4e4\uc740 \uace1\uc758 \ub304\uc2a4 \ube44\ud2b8\ub97c \uce6d\ucc2c\ud588\ub294\ub370, PopMatters\uc758 \ucf5c\ub9b0 \ub9e5\uacfc\uc774\uc5b4\ub294 \u3008Single Ladies\u3009\ub97c \ube44\uc698\uc138 \ucd5c\uace0\uc758 \ub304\uc2a4 \ud2b8\ub799 \uc911 \ud558\ub098\ub77c\uba70 \ud638\ud3c9\ud588\ub2e4. Spence D. of IGN \ubba4\uc9c1\uc758 \uc2a4\ud39c\uc2a4 D.\ub294 \"\uc774 \uace1\uc774 \uce74\ub9ac\ube0c \ud48d\uc758 \ub290\ub08c\uc744 \ub2f4\uace0 \uc788\uc73c\uba70 \ucda4\uc744 \uc990\uae30\uc9c0 \uc54a\ub294 \uc0ac\ub78c\ub3c4 \uc5c9\ub369\uc774\ub97c \ud754\ub4e4\uba70 \ud765\uc5d0 \ucde8\ud574 \ubaa9\uc73c\ub85c \ub9ac\ub4ec\uc744 \ud0c8 \uc815\ub3c4\"\ub77c\uace0 \uc124\uba85\ud588\ub2e4. \u300a\ud734\uc2a4\ud134\ud06c\ub85c\ub2c8\ud074\u300b\uc758 \uc870\uc774 \uac8c\ub77c\ub294 \"\uc5c9\ub369\uc774\ub97c \ud754\ub4e4\uac8c \ud558\ub294 \ud074\ub7fd\" \ub178\ub798\ub77c\ub294 \uc810\uc5d0\uc11c \"Check on It\"\uacfc \ube44\uc2b7\ud558\ub2e4\uace0 \ud588\ub2e4. \u300a\uc5d4\ud130\ud14c\uc778\uba3c\ud2b8 \uc704\ud074\ub9ac\u300b\uc758 \ub808\uc544 \uadf8\ub9b0\ube14\ub7ab\uc740 \u3008Single Ladies\u3009\ub294 \"\uc560\uc778\uc5d0\uac8c \uac77\uc5b4\ucc28\uc774\uace0 \ub192\uc740 \uc904\ub118\uae30\ub97c \ud558\ub294 \uac83\uc774 \uc11e\uc778 \uc544\ucc14\ud55c \ub6f0\uae30\"\uac19\ub2e4\uace0 \uc37c\ub2e4. \u300a\uc6cc\uc2f1\ud134 \ud3ec\uc2a4\ud2b8\u300b\uc758 \uc544\ub2f4 \ub9c8\uc988\ub9e4\ub2c8\uc548\uc740 \ube44\uc698\uc138\ub294 \"\uc9c4\uc2ec\uc73c\ub85c \ubc18\ud56d\uc801\uc774\uba70 \ub3c5\ub9bd\uc801\uc778 \ubaa9\uc18c\ub9ac\"\ub85c \ubb34\ub300\uc5d0\uc11c \ub178\ub798\ub97c \ubd80\ub97c \ub54c \uc5ec\uc790\ub4e4\ub85c\ubd80\ud130 \uc9c0\uc9c0 \ubc1b\uc744 \uc218 \uc788\ub3c4\ub85d \uacc4\ud68d\ud588\ub2e4\uace0 \ub9ac\ubdf0\ud588\ub2e4. \ubc18\uba74 \uc5b4\ub5a4 \ud3c9\ub860\uac00\ub4e4\uc740 \u3008Single Ladies\u3009\ub85c \ubd80\ud130 \ubcc4\ub85c \uac10\uba85\ubc1b\uc9c0 \ubabb\ud588\ub2e4\ub294 \ud3c9\uc744 \ub0b4\ub193\uc558\ub294\ub370 \uc77c\ub840\ub85c \u300a\ube4c\ubcf4\ub4dc\u300b\uc758 \ub9c8\ub9ac\uc5d8 \ucf58\uc138\ud504\uc2dc\uc628\uc740 \"\ud3c9\ubc94\ud558\uac8c \ucff5\ucff5\uac70\ub9ac\uace0 \uadc0\uc5d0 \uac70\uc2ac\ub9ac\ub294 \uc18c\ub9ac\ub97c \ub0b4\ub294 \ub178\ub798\"\ub77c\uace0 \ub9ac\ubdf0\ud588\ub2e4. The Observer\uc758 \uc544\ub2f4 \ub9c8\ud14c\ub77c\ub294 \u3008Single Ladies\u3009\uc640 \"Diva\"\ub294 \uc5ec\uc7a5\ub0a8\uc790\uc758 \uc7a0\uc7ac\uc801\uc778 \uae30\uc6d0\uc774\uc790 \uc601\uac10\uc774 \ub418\ub294 \uace1\uc774\ub77c\uace0 \ubcf4\uc558\ub2e4. \uadf8\ub7ec\ub098 \u300a\uc2ac\ub79c\ud2b8 \ub9e4\uac70\uc9c4\u300b\uc758 \uc0b4 \uc2e0\ucf00\ub9c8\ub2c8\ub294 B'Day\uc758 \"\uc794\uc874\ubb3c\"\uc77c\ubfd0\uc774\ub77c\uba70 \ube44\ud310\ud588\ub2e4."} {"question": "\ud3f4\ub9c1\uc774 \ucc3d\uc2dc\uc790\uac00 \ub41c \uc0c8\ub85c\uc6b4 \uacfc\ud559\ubd84\uc57c\ub294?", "paragraph": "\uadf8\uc758 \ub9c8\uc9c0\ub9c9 2\ub144\uac04\uc758 \ub300\ud559\uc0dd\ud65c\uc5d0\uc11c \ud3f4\ub9c1\uc740 \uc6d0\uc790\uc640 \ubd84\uc790\uc758 \uc804\uc790\uad6c\uc870\uc5d0 \ub300\ud55c \uc77c\uc744 \ud588\ub358 Gilbert N. Lewis\uc640 Irving Langmuir\uc758 \uc5c5\uc801\uc5d0 \ub300\ud574 \uc54c\uac8c \ub418\uc5c8\ub2e4. \uadf8\ub294 \uadf8\uc758 \uc5f0\uad6c\ub97c \ubb3c\ub9ac\uc801\uc774\uace0 \ud654\ud559\uc801\uc778 \ubb3c\uc9c8\uc758 \uc131\uc9c8\uc744 \uc774\uc6a9\ud558\uc5ec \uc5b4\ub5bb\uac8c \uadf8\uac83\ub4e4\uc774 \ubd84\uc790\ub97c \uc774\ub8e8\ub294 \uc6d0\uc790\ub4e4\uc758 \uad6c\uc870\uc640 \uad00\ub828\ub418\uc5b4\uc788\ub294\uc9c0\uc5d0 \ub300\ud574 \ucd08\uc810\uc744 \ub9de\ucd94\uae30\ub85c \ud588\ub2e4. \uadf8\ub9ac\ud558\uc5ec \uadf8\ub294 \uc591\uc790\ud654\ud559\uc774\ub77c\ub294 \uc0c8\ub85c\uc6b4\uacfc\ud559\ubd84\uc57c\uc758 \ucc3d\uc2dc\uc790\uac00 \ub418\uc5c8\ub2e4. \ud3f4\ub9c1\uc740 \uc778\ub958\ud559 \uc0ac\ud68c\ud559\uc5d0 \ub300\ud55c \uacf5\ubd80\ub97c \ubb34\uc2dc\ud558\uae30 \uc2dc\uc791\ud588\ub2e4. \uadf8\ub294 \uac8c\ub2e4\uac00 \ubb3c\ub9ac, \uc218\ud559\ubd80\uc758 \ud544\uc218\uacfc\ubaa9\uc5d0 \ub300\ud574\uc11c\ub3c4 \ud655\uc2e4\ud558\uac8c \ub2e4\ub8e8\uc5c8\ub2e4. Samuel Graf\uad50\uc218\ub294 \ud3f4\ub9c1\uc744 \ub192\uc740 \uc218\uc900\uc758 \uc218\ud559\uacfc\uc815\uc5d0\uc11c \uadf8\uc758 \uc870\uad50\ub85c\uc11c \uc120\ubc1c\ud588\ub2e4. 4\ud559\ub144 \uaca8\uc6b8 \ubb34\ub835, \ud3f4\ub9c1\uc740 \uac00\uc815\uacbd\uc81c\ud559\uc744 \uc804\uacf5\uc73c\ub85c \ud558\ub294 \ud559\uc0dd\ub4e4\uc744 \ub300\uc0c1\uc73c\ub85c \ud654\ud559\ubd80 \uacfc\ubaa9\uc744 \uac00\ub974\uce58\ub294 \uac83\uc744 \uc81c\uc548\ubc1b\uc558\ub2e4. \uadf8\uacf3\uc5d0\uc11c\ub294 \ud3f4\ub9c1\uc758 \ubbf8\ub798 \uc544\ub0b4\uac00 \ub420 \uc0ac\ub78c\uc778 Ava Helen Miller\uac00 \uc788\uc5c8\ub2e4.", "answer": "\uc591\uc790\ud654\ud559", "sentence": "\uadf8\ub294 \uc591\uc790\ud654\ud559 \uc774\ub77c\ub294 \uc0c8\ub85c\uc6b4\uacfc\ud559\ubd84\uc57c\uc758 \ucc3d\uc2dc\uc790\uac00 \ub418\uc5c8\ub2e4.", "paragraph_sentence": "\uadf8\uc758 \ub9c8\uc9c0\ub9c9 2\ub144\uac04\uc758 \ub300\ud559\uc0dd\ud65c\uc5d0\uc11c \ud3f4\ub9c1\uc740 \uc6d0\uc790\uc640 \ubd84\uc790\uc758 \uc804\uc790\uad6c\uc870\uc5d0 \ub300\ud55c \uc77c\uc744 \ud588\ub358 Gilbert N. Lewis\uc640 Irving Langmuir\uc758 \uc5c5\uc801\uc5d0 \ub300\ud574 \uc54c\uac8c \ub418\uc5c8\ub2e4. \uadf8\ub294 \uadf8\uc758 \uc5f0\uad6c\ub97c \ubb3c\ub9ac\uc801\uc774\uace0 \ud654\ud559\uc801\uc778 \ubb3c\uc9c8\uc758 \uc131\uc9c8\uc744 \uc774\uc6a9\ud558\uc5ec \uc5b4\ub5bb\uac8c \uadf8\uac83\ub4e4\uc774 \ubd84\uc790\ub97c \uc774\ub8e8\ub294 \uc6d0\uc790\ub4e4\uc758 \uad6c\uc870\uc640 \uad00\ub828\ub418\uc5b4\uc788\ub294\uc9c0\uc5d0 \ub300\ud574 \ucd08\uc810\uc744 \ub9de\ucd94\uae30\ub85c \ud588\ub2e4. \uadf8\ub9ac\ud558\uc5ec \uadf8\ub294 \uc591\uc790\ud654\ud559 \uc774\ub77c\ub294 \uc0c8\ub85c\uc6b4\uacfc\ud559\ubd84\uc57c\uc758 \ucc3d\uc2dc\uc790\uac00 \ub418\uc5c8\ub2e4. \ud3f4\ub9c1\uc740 \uc778\ub958\ud559 \uc0ac\ud68c\ud559\uc5d0 \ub300\ud55c \uacf5\ubd80\ub97c \ubb34\uc2dc\ud558\uae30 \uc2dc\uc791\ud588\ub2e4. \uadf8\ub294 \uac8c\ub2e4\uac00 \ubb3c\ub9ac, \uc218\ud559\ubd80\uc758 \ud544\uc218\uacfc\ubaa9\uc5d0 \ub300\ud574\uc11c\ub3c4 \ud655\uc2e4\ud558\uac8c \ub2e4\ub8e8\uc5c8\ub2e4. Samuel Graf\uad50\uc218\ub294 \ud3f4\ub9c1\uc744 \ub192\uc740 \uc218\uc900\uc758 \uc218\ud559\uacfc\uc815\uc5d0\uc11c \uadf8\uc758 \uc870\uad50\ub85c\uc11c \uc120\ubc1c\ud588\ub2e4. 4\ud559\ub144 \uaca8\uc6b8 \ubb34\ub835, \ud3f4\ub9c1\uc740 \uac00\uc815\uacbd\uc81c\ud559\uc744 \uc804\uacf5\uc73c\ub85c \ud558\ub294 \ud559\uc0dd\ub4e4\uc744 \ub300\uc0c1\uc73c\ub85c \ud654\ud559\ubd80 \uacfc\ubaa9\uc744 \uac00\ub974\uce58\ub294 \uac83\uc744 \uc81c\uc548\ubc1b\uc558\ub2e4. \uadf8\uacf3\uc5d0\uc11c\ub294 \ud3f4\ub9c1\uc758 \ubbf8\ub798 \uc544\ub0b4\uac00 \ub420 \uc0ac\ub78c\uc778 Ava Helen Miller\uac00 \uc788\uc5c8\ub2e4.", "paragraph_answer": "\uadf8\uc758 \ub9c8\uc9c0\ub9c9 2\ub144\uac04\uc758 \ub300\ud559\uc0dd\ud65c\uc5d0\uc11c \ud3f4\ub9c1\uc740 \uc6d0\uc790\uc640 \ubd84\uc790\uc758 \uc804\uc790\uad6c\uc870\uc5d0 \ub300\ud55c \uc77c\uc744 \ud588\ub358 Gilbert N. Lewis\uc640 Irving Langmuir\uc758 \uc5c5\uc801\uc5d0 \ub300\ud574 \uc54c\uac8c \ub418\uc5c8\ub2e4. \uadf8\ub294 \uadf8\uc758 \uc5f0\uad6c\ub97c \ubb3c\ub9ac\uc801\uc774\uace0 \ud654\ud559\uc801\uc778 \ubb3c\uc9c8\uc758 \uc131\uc9c8\uc744 \uc774\uc6a9\ud558\uc5ec \uc5b4\ub5bb\uac8c \uadf8\uac83\ub4e4\uc774 \ubd84\uc790\ub97c \uc774\ub8e8\ub294 \uc6d0\uc790\ub4e4\uc758 \uad6c\uc870\uc640 \uad00\ub828\ub418\uc5b4\uc788\ub294\uc9c0\uc5d0 \ub300\ud574 \ucd08\uc810\uc744 \ub9de\ucd94\uae30\ub85c \ud588\ub2e4. \uadf8\ub9ac\ud558\uc5ec \uadf8\ub294 \uc591\uc790\ud654\ud559 \uc774\ub77c\ub294 \uc0c8\ub85c\uc6b4\uacfc\ud559\ubd84\uc57c\uc758 \ucc3d\uc2dc\uc790\uac00 \ub418\uc5c8\ub2e4. \ud3f4\ub9c1\uc740 \uc778\ub958\ud559 \uc0ac\ud68c\ud559\uc5d0 \ub300\ud55c \uacf5\ubd80\ub97c \ubb34\uc2dc\ud558\uae30 \uc2dc\uc791\ud588\ub2e4. \uadf8\ub294 \uac8c\ub2e4\uac00 \ubb3c\ub9ac, \uc218\ud559\ubd80\uc758 \ud544\uc218\uacfc\ubaa9\uc5d0 \ub300\ud574\uc11c\ub3c4 \ud655\uc2e4\ud558\uac8c \ub2e4\ub8e8\uc5c8\ub2e4. Samuel Graf\uad50\uc218\ub294 \ud3f4\ub9c1\uc744 \ub192\uc740 \uc218\uc900\uc758 \uc218\ud559\uacfc\uc815\uc5d0\uc11c \uadf8\uc758 \uc870\uad50\ub85c\uc11c \uc120\ubc1c\ud588\ub2e4. 4\ud559\ub144 \uaca8\uc6b8 \ubb34\ub835, \ud3f4\ub9c1\uc740 \uac00\uc815\uacbd\uc81c\ud559\uc744 \uc804\uacf5\uc73c\ub85c \ud558\ub294 \ud559\uc0dd\ub4e4\uc744 \ub300\uc0c1\uc73c\ub85c \ud654\ud559\ubd80 \uacfc\ubaa9\uc744 \uac00\ub974\uce58\ub294 \uac83\uc744 \uc81c\uc548\ubc1b\uc558\ub2e4. \uadf8\uacf3\uc5d0\uc11c\ub294 \ud3f4\ub9c1\uc758 \ubbf8\ub798 \uc544\ub0b4\uac00 \ub420 \uc0ac\ub78c\uc778 Ava Helen Miller\uac00 \uc788\uc5c8\ub2e4.", "sentence_answer": "\uadf8\ub294 \uc591\uc790\ud654\ud559 \uc774\ub77c\ub294 \uc0c8\ub85c\uc6b4\uacfc\ud559\ubd84\uc57c\uc758 \ucc3d\uc2dc\uc790\uac00 \ub418\uc5c8\ub2e4.", "paragraph_id": "6579230-5-0", "paragraph_question": "question: \ud3f4\ub9c1\uc774 \ucc3d\uc2dc\uc790\uac00 \ub41c \uc0c8\ub85c\uc6b4 \uacfc\ud559\ubd84\uc57c\ub294?, context: \uadf8\uc758 \ub9c8\uc9c0\ub9c9 2\ub144\uac04\uc758 \ub300\ud559\uc0dd\ud65c\uc5d0\uc11c \ud3f4\ub9c1\uc740 \uc6d0\uc790\uc640 \ubd84\uc790\uc758 \uc804\uc790\uad6c\uc870\uc5d0 \ub300\ud55c \uc77c\uc744 \ud588\ub358 Gilbert N. Lewis\uc640 Irving Langmuir\uc758 \uc5c5\uc801\uc5d0 \ub300\ud574 \uc54c\uac8c \ub418\uc5c8\ub2e4. \uadf8\ub294 \uadf8\uc758 \uc5f0\uad6c\ub97c \ubb3c\ub9ac\uc801\uc774\uace0 \ud654\ud559\uc801\uc778 \ubb3c\uc9c8\uc758 \uc131\uc9c8\uc744 \uc774\uc6a9\ud558\uc5ec \uc5b4\ub5bb\uac8c \uadf8\uac83\ub4e4\uc774 \ubd84\uc790\ub97c \uc774\ub8e8\ub294 \uc6d0\uc790\ub4e4\uc758 \uad6c\uc870\uc640 \uad00\ub828\ub418\uc5b4\uc788\ub294\uc9c0\uc5d0 \ub300\ud574 \ucd08\uc810\uc744 \ub9de\ucd94\uae30\ub85c \ud588\ub2e4. \uadf8\ub9ac\ud558\uc5ec \uadf8\ub294 \uc591\uc790\ud654\ud559\uc774\ub77c\ub294 \uc0c8\ub85c\uc6b4\uacfc\ud559\ubd84\uc57c\uc758 \ucc3d\uc2dc\uc790\uac00 \ub418\uc5c8\ub2e4. \ud3f4\ub9c1\uc740 \uc778\ub958\ud559 \uc0ac\ud68c\ud559\uc5d0 \ub300\ud55c \uacf5\ubd80\ub97c \ubb34\uc2dc\ud558\uae30 \uc2dc\uc791\ud588\ub2e4. \uadf8\ub294 \uac8c\ub2e4\uac00 \ubb3c\ub9ac, \uc218\ud559\ubd80\uc758 \ud544\uc218\uacfc\ubaa9\uc5d0 \ub300\ud574\uc11c\ub3c4 \ud655\uc2e4\ud558\uac8c \ub2e4\ub8e8\uc5c8\ub2e4. Samuel Graf\uad50\uc218\ub294 \ud3f4\ub9c1\uc744 \ub192\uc740 \uc218\uc900\uc758 \uc218\ud559\uacfc\uc815\uc5d0\uc11c \uadf8\uc758 \uc870\uad50\ub85c\uc11c \uc120\ubc1c\ud588\ub2e4. 4\ud559\ub144 \uaca8\uc6b8 \ubb34\ub835, \ud3f4\ub9c1\uc740 \uac00\uc815\uacbd\uc81c\ud559\uc744 \uc804\uacf5\uc73c\ub85c \ud558\ub294 \ud559\uc0dd\ub4e4\uc744 \ub300\uc0c1\uc73c\ub85c \ud654\ud559\ubd80 \uacfc\ubaa9\uc744 \uac00\ub974\uce58\ub294 \uac83\uc744 \uc81c\uc548\ubc1b\uc558\ub2e4. \uadf8\uacf3\uc5d0\uc11c\ub294 \ud3f4\ub9c1\uc758 \ubbf8\ub798 \uc544\ub0b4\uac00 \ub420 \uc0ac\ub78c\uc778 Ava Helen Miller\uac00 \uc788\uc5c8\ub2e4."} {"question": "\uc720\ub098\uc774\ud2f0\ub4dc \ud56d\uacf5 175\ud3b8\uc740 \ucd5c\ub300 \uba87 \uba85\uc758 \uc2b9\uac1d\uc744 \uc2e4\uc744 \uc218 \uc788\uc5c8\ub294\uac00?", "paragraph": "\uc774 \ud56d\uacf5\ud3b8\uc740 1983\ub144 \ucc98\uc74c \uc6b4\ud56d\uc744 \uc2dc\uc791\ud55c \ubcf4\uc789 767-200\uc73c\ub85c, \ud56d\uacf5\uae30 \ub4f1\ub85d\ubc88\ud638\ub294 N612UA\uc774\uba70 \ucd5c\ub300 168\uba85\uc758 \uc2b9\uac1d\uc744 \uc2e3\uc744 \uc218 \uc788\ub2e4. \uc774 \ud56d\uacf5\uae30\ub294 1\ub4f1\uc11d 10\uc11d, \ube44\uc988\ub2c8\uc2a4 \ud074\ub798\uc2a4 32\uc11d, \uc774\ucf54\ub178\ubbf8 \ud074\ub798\uc2a4 126\uc11d \ub4f1 \ucd1d 168\uc11d\uc774 \uc788\uc5c8\ub2e4. \ud14c\ub7ec \ub2f9\uc77c\uc778 2001\ub144 9\uc6d4 11\uc77c\uc5d0\ub294 \uc2b9\uac1d 56\uba85\uacfc \uc2b9\ubb34\uc6d0 9\uba85\ub9cc \ud0d1\uc2b9\ud574 \uc788\ub358 \uc0c1\ud0dc\ub85c, \uc5ec\uac1d\uc218\uc1a1\uc728\uc774 33%\ub85c 9\uc6d4 11\uc77c \uc774\uc804 \uc9c0\ub09c 3\uac1c\uc6d4\uac04\uc758 \ud3c9\uade0 \uc5ec\uac1d\uc218\uc1a1\uc728 49%\ub97c \ubc11\ub3c4\ub294 \uc0c1\ud0dc\uc600\ub2e4. \uc774 \uc5ec\uac1d\uae30\uc5d0 \ud0d1\uc2b9\ud588\ub358 \uc2b9\ubb34\uc6d0\uc740 \uae30\uc7a5 \ube45\ud1a0\ub974 \uc0ac\ub77c\uc2dc\ub2c8, \ubd80\uae30\uc7a5 \ub9c8\uc774\ud074 \ud638\ub85d\uc2a4, \uc2b9\ubb34\uc6d0 \ub85c\ubc84\ud2b8 \ud33d\ub9e8, \uc560\uc774\ubbf8 \uc790\ub81b, \uc560\uc774\ubbf8 \ud0b9, \uce90\uc11c\ub9b0 \ub798\ubcf4\ub9b0, \uc54c\ud504\ub798\ub4dc \ub9c8\ucc48\ub4dc, \ub9c8\uc774\ud074 \ud0c0\ub85c\uc6b0, \uc568\ub9ac\uc0e4 \ud2f0\ud22c\uc2a4\uc600\ub2e4. \ub0a9\uce58\ubc94\ub4e4\uc744 \uc81c\uc678\ud55c \ube44\ud589\uae30 \ub0b4 \uc2b9\uac1d\uc740 \ub0a8\uc131 35\uba85, \uc5ec\uc131 12\uba85, 5\uc138 \uc774\ud558\uc758 \uc544\ub3d9\uc774 5\uba85 \ud0d1\uc2b9\ud574 \uc788\uc5c8\ub2e4. \ub610\ud55c, \ud0d1\uc2b9\uac1d \uc911\uc5d0\ub294 \ub0b4\uc154\ub110 \ud558\ud0a4 \ub9ac\uadf8\uc758 \uc804 \uc120\uc218\uc774\uc790 \ub85c\uc2a4\uc564\uc824\ub808\uc2a4 \ud0b9\uc2a4\uc758 \ud504\ub85c\uc288\ud305 \uac10\ub3c5\uc774\uc5c8\ub358 \uadf8\ub791 \ubc1c\ub808\uc774\ub3c4 \uc788\uc5c8\ub2e4.", "answer": "168\uba85", "sentence": "\uc774 \ud56d\uacf5\ud3b8\uc740 1983\ub144 \ucc98\uc74c \uc6b4\ud56d\uc744 \uc2dc\uc791\ud55c \ubcf4\uc789 767-200\uc73c\ub85c, \ud56d\uacf5\uae30 \ub4f1\ub85d\ubc88\ud638\ub294 N612UA\uc774\uba70 \ucd5c\ub300 168\uba85 \uc758 \uc2b9\uac1d\uc744 \uc2e3\uc744 \uc218 \uc788\ub2e4.", "paragraph_sentence": " \uc774 \ud56d\uacf5\ud3b8\uc740 1983\ub144 \ucc98\uc74c \uc6b4\ud56d\uc744 \uc2dc\uc791\ud55c \ubcf4\uc789 767-200\uc73c\ub85c, \ud56d\uacf5\uae30 \ub4f1\ub85d\ubc88\ud638\ub294 N612UA\uc774\uba70 \ucd5c\ub300 168\uba85 \uc758 \uc2b9\uac1d\uc744 \uc2e3\uc744 \uc218 \uc788\ub2e4. \uc774 \ud56d\uacf5\uae30\ub294 1\ub4f1\uc11d 10\uc11d, \ube44\uc988\ub2c8\uc2a4 \ud074\ub798\uc2a4 32\uc11d, \uc774\ucf54\ub178\ubbf8 \ud074\ub798\uc2a4 126\uc11d \ub4f1 \ucd1d 168\uc11d\uc774 \uc788\uc5c8\ub2e4. \ud14c\ub7ec \ub2f9\uc77c\uc778 2001\ub144 9\uc6d4 11\uc77c\uc5d0\ub294 \uc2b9\uac1d 56\uba85\uacfc \uc2b9\ubb34\uc6d0 9\uba85\ub9cc \ud0d1\uc2b9\ud574 \uc788\ub358 \uc0c1\ud0dc\ub85c, \uc5ec\uac1d\uc218\uc1a1\uc728\uc774 33%\ub85c 9\uc6d4 11\uc77c \uc774\uc804 \uc9c0\ub09c 3\uac1c\uc6d4\uac04\uc758 \ud3c9\uade0 \uc5ec\uac1d\uc218\uc1a1\uc728 49%\ub97c \ubc11\ub3c4\ub294 \uc0c1\ud0dc\uc600\ub2e4. \uc774 \uc5ec\uac1d\uae30\uc5d0 \ud0d1\uc2b9\ud588\ub358 \uc2b9\ubb34\uc6d0\uc740 \uae30\uc7a5 \ube45\ud1a0\ub974 \uc0ac\ub77c\uc2dc\ub2c8, \ubd80\uae30\uc7a5 \ub9c8\uc774\ud074 \ud638\ub85d\uc2a4, \uc2b9\ubb34\uc6d0 \ub85c\ubc84\ud2b8 \ud33d\ub9e8, \uc560\uc774\ubbf8 \uc790\ub81b, \uc560\uc774\ubbf8 \ud0b9, \uce90\uc11c\ub9b0 \ub798\ubcf4\ub9b0, \uc54c\ud504\ub798\ub4dc \ub9c8\ucc48\ub4dc, \ub9c8\uc774\ud074 \ud0c0\ub85c\uc6b0, \uc568\ub9ac\uc0e4 \ud2f0\ud22c\uc2a4\uc600\ub2e4. \ub0a9\uce58\ubc94\ub4e4\uc744 \uc81c\uc678\ud55c \ube44\ud589\uae30 \ub0b4 \uc2b9\uac1d\uc740 \ub0a8\uc131 35\uba85, \uc5ec\uc131 12\uba85, 5\uc138 \uc774\ud558\uc758 \uc544\ub3d9\uc774 5\uba85 \ud0d1\uc2b9\ud574 \uc788\uc5c8\ub2e4. \ub610\ud55c, \ud0d1\uc2b9\uac1d \uc911\uc5d0\ub294 \ub0b4\uc154\ub110 \ud558\ud0a4 \ub9ac\uadf8\uc758 \uc804 \uc120\uc218\uc774\uc790 \ub85c\uc2a4\uc564\uc824\ub808\uc2a4 \ud0b9\uc2a4\uc758 \ud504\ub85c\uc288\ud305 \uac10\ub3c5\uc774\uc5c8\ub358 \uadf8\ub791 \ubc1c\ub808\uc774\ub3c4 \uc788\uc5c8\ub2e4.", "paragraph_answer": "\uc774 \ud56d\uacf5\ud3b8\uc740 1983\ub144 \ucc98\uc74c \uc6b4\ud56d\uc744 \uc2dc\uc791\ud55c \ubcf4\uc789 767-200\uc73c\ub85c, \ud56d\uacf5\uae30 \ub4f1\ub85d\ubc88\ud638\ub294 N612UA\uc774\uba70 \ucd5c\ub300 168\uba85 \uc758 \uc2b9\uac1d\uc744 \uc2e3\uc744 \uc218 \uc788\ub2e4. \uc774 \ud56d\uacf5\uae30\ub294 1\ub4f1\uc11d 10\uc11d, \ube44\uc988\ub2c8\uc2a4 \ud074\ub798\uc2a4 32\uc11d, \uc774\ucf54\ub178\ubbf8 \ud074\ub798\uc2a4 126\uc11d \ub4f1 \ucd1d 168\uc11d\uc774 \uc788\uc5c8\ub2e4. \ud14c\ub7ec \ub2f9\uc77c\uc778 2001\ub144 9\uc6d4 11\uc77c\uc5d0\ub294 \uc2b9\uac1d 56\uba85\uacfc \uc2b9\ubb34\uc6d0 9\uba85\ub9cc \ud0d1\uc2b9\ud574 \uc788\ub358 \uc0c1\ud0dc\ub85c, \uc5ec\uac1d\uc218\uc1a1\uc728\uc774 33%\ub85c 9\uc6d4 11\uc77c \uc774\uc804 \uc9c0\ub09c 3\uac1c\uc6d4\uac04\uc758 \ud3c9\uade0 \uc5ec\uac1d\uc218\uc1a1\uc728 49%\ub97c \ubc11\ub3c4\ub294 \uc0c1\ud0dc\uc600\ub2e4. \uc774 \uc5ec\uac1d\uae30\uc5d0 \ud0d1\uc2b9\ud588\ub358 \uc2b9\ubb34\uc6d0\uc740 \uae30\uc7a5 \ube45\ud1a0\ub974 \uc0ac\ub77c\uc2dc\ub2c8, \ubd80\uae30\uc7a5 \ub9c8\uc774\ud074 \ud638\ub85d\uc2a4, \uc2b9\ubb34\uc6d0 \ub85c\ubc84\ud2b8 \ud33d\ub9e8, \uc560\uc774\ubbf8 \uc790\ub81b, \uc560\uc774\ubbf8 \ud0b9, \uce90\uc11c\ub9b0 \ub798\ubcf4\ub9b0, \uc54c\ud504\ub798\ub4dc \ub9c8\ucc48\ub4dc, \ub9c8\uc774\ud074 \ud0c0\ub85c\uc6b0, \uc568\ub9ac\uc0e4 \ud2f0\ud22c\uc2a4\uc600\ub2e4. \ub0a9\uce58\ubc94\ub4e4\uc744 \uc81c\uc678\ud55c \ube44\ud589\uae30 \ub0b4 \uc2b9\uac1d\uc740 \ub0a8\uc131 35\uba85, \uc5ec\uc131 12\uba85, 5\uc138 \uc774\ud558\uc758 \uc544\ub3d9\uc774 5\uba85 \ud0d1\uc2b9\ud574 \uc788\uc5c8\ub2e4. \ub610\ud55c, \ud0d1\uc2b9\uac1d \uc911\uc5d0\ub294 \ub0b4\uc154\ub110 \ud558\ud0a4 \ub9ac\uadf8\uc758 \uc804 \uc120\uc218\uc774\uc790 \ub85c\uc2a4\uc564\uc824\ub808\uc2a4 \ud0b9\uc2a4\uc758 \ud504\ub85c\uc288\ud305 \uac10\ub3c5\uc774\uc5c8\ub358 \uadf8\ub791 \ubc1c\ub808\uc774\ub3c4 \uc788\uc5c8\ub2e4.", "sentence_answer": "\uc774 \ud56d\uacf5\ud3b8\uc740 1983\ub144 \ucc98\uc74c \uc6b4\ud56d\uc744 \uc2dc\uc791\ud55c \ubcf4\uc789 767-200\uc73c\ub85c, \ud56d\uacf5\uae30 \ub4f1\ub85d\ubc88\ud638\ub294 N612UA\uc774\uba70 \ucd5c\ub300 168\uba85 \uc758 \uc2b9\uac1d\uc744 \uc2e3\uc744 \uc218 \uc788\ub2e4.", "paragraph_id": "6527029-2-0", "paragraph_question": "question: \uc720\ub098\uc774\ud2f0\ub4dc \ud56d\uacf5 175\ud3b8\uc740 \ucd5c\ub300 \uba87 \uba85\uc758 \uc2b9\uac1d\uc744 \uc2e4\uc744 \uc218 \uc788\uc5c8\ub294\uac00?, context: \uc774 \ud56d\uacf5\ud3b8\uc740 1983\ub144 \ucc98\uc74c \uc6b4\ud56d\uc744 \uc2dc\uc791\ud55c \ubcf4\uc789 767-200\uc73c\ub85c, \ud56d\uacf5\uae30 \ub4f1\ub85d\ubc88\ud638\ub294 N612UA\uc774\uba70 \ucd5c\ub300 168\uba85\uc758 \uc2b9\uac1d\uc744 \uc2e3\uc744 \uc218 \uc788\ub2e4. \uc774 \ud56d\uacf5\uae30\ub294 1\ub4f1\uc11d 10\uc11d, \ube44\uc988\ub2c8\uc2a4 \ud074\ub798\uc2a4 32\uc11d, \uc774\ucf54\ub178\ubbf8 \ud074\ub798\uc2a4 126\uc11d \ub4f1 \ucd1d 168\uc11d\uc774 \uc788\uc5c8\ub2e4. \ud14c\ub7ec \ub2f9\uc77c\uc778 2001\ub144 9\uc6d4 11\uc77c\uc5d0\ub294 \uc2b9\uac1d 56\uba85\uacfc \uc2b9\ubb34\uc6d0 9\uba85\ub9cc \ud0d1\uc2b9\ud574 \uc788\ub358 \uc0c1\ud0dc\ub85c, \uc5ec\uac1d\uc218\uc1a1\uc728\uc774 33%\ub85c 9\uc6d4 11\uc77c \uc774\uc804 \uc9c0\ub09c 3\uac1c\uc6d4\uac04\uc758 \ud3c9\uade0 \uc5ec\uac1d\uc218\uc1a1\uc728 49%\ub97c \ubc11\ub3c4\ub294 \uc0c1\ud0dc\uc600\ub2e4. \uc774 \uc5ec\uac1d\uae30\uc5d0 \ud0d1\uc2b9\ud588\ub358 \uc2b9\ubb34\uc6d0\uc740 \uae30\uc7a5 \ube45\ud1a0\ub974 \uc0ac\ub77c\uc2dc\ub2c8, \ubd80\uae30\uc7a5 \ub9c8\uc774\ud074 \ud638\ub85d\uc2a4, \uc2b9\ubb34\uc6d0 \ub85c\ubc84\ud2b8 \ud33d\ub9e8, \uc560\uc774\ubbf8 \uc790\ub81b, \uc560\uc774\ubbf8 \ud0b9, \uce90\uc11c\ub9b0 \ub798\ubcf4\ub9b0, \uc54c\ud504\ub798\ub4dc \ub9c8\ucc48\ub4dc, \ub9c8\uc774\ud074 \ud0c0\ub85c\uc6b0, \uc568\ub9ac\uc0e4 \ud2f0\ud22c\uc2a4\uc600\ub2e4. \ub0a9\uce58\ubc94\ub4e4\uc744 \uc81c\uc678\ud55c \ube44\ud589\uae30 \ub0b4 \uc2b9\uac1d\uc740 \ub0a8\uc131 35\uba85, \uc5ec\uc131 12\uba85, 5\uc138 \uc774\ud558\uc758 \uc544\ub3d9\uc774 5\uba85 \ud0d1\uc2b9\ud574 \uc788\uc5c8\ub2e4. \ub610\ud55c, \ud0d1\uc2b9\uac1d \uc911\uc5d0\ub294 \ub0b4\uc154\ub110 \ud558\ud0a4 \ub9ac\uadf8\uc758 \uc804 \uc120\uc218\uc774\uc790 \ub85c\uc2a4\uc564\uc824\ub808\uc2a4 \ud0b9\uc2a4\uc758 \ud504\ub85c\uc288\ud305 \uac10\ub3c5\uc774\uc5c8\ub358 \uadf8\ub791 \ubc1c\ub808\uc774\ub3c4 \uc788\uc5c8\ub2e4."} {"question": "\uc74c\uc545 \ud504\ub85c\uadf8\ub7a8\uc5d0\uc11c \uc120\uc608 \ud63c\uc790 \uacf5\uc5f0\ud558\uc600\ub358 \ud55c\uc131\ubcc4\uace1 OST \uc218\ub85d\uace1\uc758 \uc81c\ubaa9\uc740 \ubb34\uc5c7\uc778\uac00?", "paragraph": "\uc6d0\ub354\uac78\uc2a4\ub294 \"Irony\"\ub85c \ud65c\ubc1c\ud55c \ud65c\ub3d9\uc744 \ud588\uc73c\ub098 \uc774\ud6c4 \ub9ce\uc740 \ubd88\uc6b4\uc744 \uacaa\uac8c \ub41c\ub2e4. 6\uc6d4 19\uc77c \uba64\ubc84 \uc18c\ud76c\uac00 \uc601\ud654 \u300a\ub728\uac70\uc6b4 \uac83\uc774 \uc88b\uc544\u300b \ucd2c\uc601 \uc911 \uc624\ud1a0\ubc14\uc774 \uc0ac\uace0\ub85c \ubb34\ub98e\uc774 \ucc22\uc5b4\uc838 \uac01\uc885 \ud589\uc0ac \ubb34\ub300\uc5d0\uc11c \uc57d \ud55c \ub2ec\uac04 4\uc778 \uccb4\uc81c\ub85c \ud65c\ub3d9\ud558\uc600\uace0, \uc774\ud6c4 \ubb34\ub300\uc5d0\uc11c\ub294 \ud604\uc544\uae4c\uc9c0 \ube60\uc9c4 \uc138 \uba85\uc774 \ud65c\ub3d9\ud558\ub294 \ubaa8\uc2b5\uc744 \ubcf4\uc5ec \uc8fc\uc5c8\ub2e4. \uc758\ub958 \uad11\uace0 \ud654\ubcf4 \ucd2c\uc601\uc5d0\uc11c\ub3c4 \ud604\uc544\ub9cc \ube60\uc838 \uc788\uc5b4\uc11c \ud32c\ub4e4\uc758 \uad81\uae08\uc99d\uc744 \uc790\uc544\ub0b4\ub2e4\uac00, \ub9c8\uce68\ub0b4 7\uc6d4 30\uc77c \uba64\ubc84 \ud604\uc544\uc758 \ud0c8\ud1f4\uac00 \ubc1c\ud45c\ub418\uc5c8\ub2e4. \uc2b5\uad00\uc131 \uc7a5\uc5fc\uc73c\ub85c 3\uc6d4\uacfc 4\uc6d4\uc5d0 \uc7a0\uc2dc \uc785\uc6d0\ud558\ub294 \ub4f1 \ud65c\ub3d9 \ucd08\uae30\ubd80\ud130 \ucc28\uc9c8\uc744 \ube5a\uc5c8\ub358 \uadf8\ub140\ub294, \uc815\uc0c1\uc801\uc778 \ud65c\ub3d9\uc774 \ubd88\uac00\ub2a5\ud558\ub2e4\uace0 \ud310\ub2e8\ud55c \uc18c\uc18d\uc0ac \uce21\uc758 \ud0c8\ud1f4 \uacb0\uc815\uc744 \ubc1b\uc544\ub4e4\uc600\ub2e4. (\ud604\uc544\ub294 \ud604\uc7ac \uc194\ub85c\ub85c \ud65c\ub3d9\ud558\uace0 \uc788\ub2e4.) \uc774\ud6c4 \uc6d0\ub354\uac78\uc2a4\ub294 \ub0a8\uc740 \uba64\ubc84 \uc138 \uba85\uc5d0 \ubc31\ub304\uc11c\ub97c \ub300\ub3d9\ud558\uc5ec \ud65c\ub3d9\ud558\ub294 \ub4f1 \uc2dc\ub828\uc744 \uacaa\uc5b4\uc57c \ud588\ub2e4. \uc774 \uc2dc\uae30\uc5d0 \uc74c\uc545 \ud504\ub85c\uadf8\ub7a8\uc5d0\uc11c \ud55c\uc131\ubcc4\uace1 OST \uc218\ub85d\uace1 \u3008\uc77c\uc6d4\uc9c0\uac00\u3009\ub97c \uc120\uc608 \ud640\ub85c \uacf5\uc5f0\ud560 \uae30\ud68c\uac00 \uc788\uc5c8\ub2e4. \ub2f9\uc2dc \uacf5\uc5f0\uc7a5\uc5d0 \uc788\ub358 \ud32c\ub4e4\uc774 \uac04\uc8fc \ubd80\ubd84\uc5d0\uc11c \u2018\uc78a\uc9c0 \uc54a\uace0 \uae30\ub2e4\ub9b4\uac8c \uc6d0\ub354\uac78\uc2a4\u2019\ub77c\uace0 \uc678\uce58\ub294 \uad6c\ud638 \uc18c\ub9ac\uc5d0 \ub300\uae30\uc2e4\ub85c \ub3cc\uc544\uc640 \ub9ce\uc774 \uc6b8\uc5c8\ub2e4\uace0 \ubc1d\ud788\uae30\ub3c4 \ud588\ub2e4. \uc6d0\ub354\uac78\uc2a4\ub294 \uc774 \uc2dc\uae30\uac00 \uac00\uc7a5 \ud798\ub4e4\uc5c8\ub358 \uc2dc\uae30\ub77c\uace0 \ud68c\uc0c1\ud588\ub2e4.", "answer": "\uc77c\uc6d4\uc9c0\uac00", "sentence": "\uc774 \uc2dc\uae30\uc5d0 \uc74c\uc545 \ud504\ub85c\uadf8\ub7a8\uc5d0\uc11c \ud55c\uc131\ubcc4\uace1 OST \uc218\ub85d\uace1 \u3008 \uc77c\uc6d4\uc9c0\uac00 \u3009\ub97c \uc120\uc608 \ud640\ub85c \uacf5\uc5f0\ud560 \uae30\ud68c\uac00 \uc788\uc5c8\ub2e4.", "paragraph_sentence": "\uc6d0\ub354\uac78\uc2a4\ub294 \"Irony\"\ub85c \ud65c\ubc1c\ud55c \ud65c\ub3d9\uc744 \ud588\uc73c\ub098 \uc774\ud6c4 \ub9ce\uc740 \ubd88\uc6b4\uc744 \uacaa\uac8c \ub41c\ub2e4. 6\uc6d4 19\uc77c \uba64\ubc84 \uc18c\ud76c\uac00 \uc601\ud654 \u300a\ub728\uac70\uc6b4 \uac83\uc774 \uc88b\uc544\u300b \ucd2c\uc601 \uc911 \uc624\ud1a0\ubc14\uc774 \uc0ac\uace0\ub85c \ubb34\ub98e\uc774 \ucc22\uc5b4\uc838 \uac01\uc885 \ud589\uc0ac \ubb34\ub300\uc5d0\uc11c \uc57d \ud55c \ub2ec\uac04 4\uc778 \uccb4\uc81c\ub85c \ud65c\ub3d9\ud558\uc600\uace0, \uc774\ud6c4 \ubb34\ub300\uc5d0\uc11c\ub294 \ud604\uc544\uae4c\uc9c0 \ube60\uc9c4 \uc138 \uba85\uc774 \ud65c\ub3d9\ud558\ub294 \ubaa8\uc2b5\uc744 \ubcf4\uc5ec \uc8fc\uc5c8\ub2e4. \uc758\ub958 \uad11\uace0 \ud654\ubcf4 \ucd2c\uc601\uc5d0\uc11c\ub3c4 \ud604\uc544\ub9cc \ube60\uc838 \uc788\uc5b4\uc11c \ud32c\ub4e4\uc758 \uad81\uae08\uc99d\uc744 \uc790\uc544\ub0b4\ub2e4\uac00, \ub9c8\uce68\ub0b4 7\uc6d4 30\uc77c \uba64\ubc84 \ud604\uc544\uc758 \ud0c8\ud1f4\uac00 \ubc1c\ud45c\ub418\uc5c8\ub2e4. \uc2b5\uad00\uc131 \uc7a5\uc5fc\uc73c\ub85c 3\uc6d4\uacfc 4\uc6d4\uc5d0 \uc7a0\uc2dc \uc785\uc6d0\ud558\ub294 \ub4f1 \ud65c\ub3d9 \ucd08\uae30\ubd80\ud130 \ucc28\uc9c8\uc744 \ube5a\uc5c8\ub358 \uadf8\ub140\ub294, \uc815\uc0c1\uc801\uc778 \ud65c\ub3d9\uc774 \ubd88\uac00\ub2a5\ud558\ub2e4\uace0 \ud310\ub2e8\ud55c \uc18c\uc18d\uc0ac \uce21\uc758 \ud0c8\ud1f4 \uacb0\uc815\uc744 \ubc1b\uc544\ub4e4\uc600\ub2e4. (\ud604\uc544\ub294 \ud604\uc7ac \uc194\ub85c\ub85c \ud65c\ub3d9\ud558\uace0 \uc788\ub2e4. ) \uc774\ud6c4 \uc6d0\ub354\uac78\uc2a4\ub294 \ub0a8\uc740 \uba64\ubc84 \uc138 \uba85\uc5d0 \ubc31\ub304\uc11c\ub97c \ub300\ub3d9\ud558\uc5ec \ud65c\ub3d9\ud558\ub294 \ub4f1 \uc2dc\ub828\uc744 \uacaa\uc5b4\uc57c \ud588\ub2e4. \uc774 \uc2dc\uae30\uc5d0 \uc74c\uc545 \ud504\ub85c\uadf8\ub7a8\uc5d0\uc11c \ud55c\uc131\ubcc4\uace1 OST \uc218\ub85d\uace1 \u3008 \uc77c\uc6d4\uc9c0\uac00 \u3009\ub97c \uc120\uc608 \ud640\ub85c \uacf5\uc5f0\ud560 \uae30\ud68c\uac00 \uc788\uc5c8\ub2e4. \ub2f9\uc2dc \uacf5\uc5f0\uc7a5\uc5d0 \uc788\ub358 \ud32c\ub4e4\uc774 \uac04\uc8fc \ubd80\ubd84\uc5d0\uc11c \u2018\uc78a\uc9c0 \uc54a\uace0 \uae30\ub2e4\ub9b4\uac8c \uc6d0\ub354\uac78\uc2a4\u2019\ub77c\uace0 \uc678\uce58\ub294 \uad6c\ud638 \uc18c\ub9ac\uc5d0 \ub300\uae30\uc2e4\ub85c \ub3cc\uc544\uc640 \ub9ce\uc774 \uc6b8\uc5c8\ub2e4\uace0 \ubc1d\ud788\uae30\ub3c4 \ud588\ub2e4. \uc6d0\ub354\uac78\uc2a4\ub294 \uc774 \uc2dc\uae30\uac00 \uac00\uc7a5 \ud798\ub4e4\uc5c8\ub358 \uc2dc\uae30\ub77c\uace0 \ud68c\uc0c1\ud588\ub2e4.", "paragraph_answer": "\uc6d0\ub354\uac78\uc2a4\ub294 \"Irony\"\ub85c \ud65c\ubc1c\ud55c \ud65c\ub3d9\uc744 \ud588\uc73c\ub098 \uc774\ud6c4 \ub9ce\uc740 \ubd88\uc6b4\uc744 \uacaa\uac8c \ub41c\ub2e4. 6\uc6d4 19\uc77c \uba64\ubc84 \uc18c\ud76c\uac00 \uc601\ud654 \u300a\ub728\uac70\uc6b4 \uac83\uc774 \uc88b\uc544\u300b \ucd2c\uc601 \uc911 \uc624\ud1a0\ubc14\uc774 \uc0ac\uace0\ub85c \ubb34\ub98e\uc774 \ucc22\uc5b4\uc838 \uac01\uc885 \ud589\uc0ac \ubb34\ub300\uc5d0\uc11c \uc57d \ud55c \ub2ec\uac04 4\uc778 \uccb4\uc81c\ub85c \ud65c\ub3d9\ud558\uc600\uace0, \uc774\ud6c4 \ubb34\ub300\uc5d0\uc11c\ub294 \ud604\uc544\uae4c\uc9c0 \ube60\uc9c4 \uc138 \uba85\uc774 \ud65c\ub3d9\ud558\ub294 \ubaa8\uc2b5\uc744 \ubcf4\uc5ec \uc8fc\uc5c8\ub2e4. \uc758\ub958 \uad11\uace0 \ud654\ubcf4 \ucd2c\uc601\uc5d0\uc11c\ub3c4 \ud604\uc544\ub9cc \ube60\uc838 \uc788\uc5b4\uc11c \ud32c\ub4e4\uc758 \uad81\uae08\uc99d\uc744 \uc790\uc544\ub0b4\ub2e4\uac00, \ub9c8\uce68\ub0b4 7\uc6d4 30\uc77c \uba64\ubc84 \ud604\uc544\uc758 \ud0c8\ud1f4\uac00 \ubc1c\ud45c\ub418\uc5c8\ub2e4. \uc2b5\uad00\uc131 \uc7a5\uc5fc\uc73c\ub85c 3\uc6d4\uacfc 4\uc6d4\uc5d0 \uc7a0\uc2dc \uc785\uc6d0\ud558\ub294 \ub4f1 \ud65c\ub3d9 \ucd08\uae30\ubd80\ud130 \ucc28\uc9c8\uc744 \ube5a\uc5c8\ub358 \uadf8\ub140\ub294, \uc815\uc0c1\uc801\uc778 \ud65c\ub3d9\uc774 \ubd88\uac00\ub2a5\ud558\ub2e4\uace0 \ud310\ub2e8\ud55c \uc18c\uc18d\uc0ac \uce21\uc758 \ud0c8\ud1f4 \uacb0\uc815\uc744 \ubc1b\uc544\ub4e4\uc600\ub2e4. (\ud604\uc544\ub294 \ud604\uc7ac \uc194\ub85c\ub85c \ud65c\ub3d9\ud558\uace0 \uc788\ub2e4.) \uc774\ud6c4 \uc6d0\ub354\uac78\uc2a4\ub294 \ub0a8\uc740 \uba64\ubc84 \uc138 \uba85\uc5d0 \ubc31\ub304\uc11c\ub97c \ub300\ub3d9\ud558\uc5ec \ud65c\ub3d9\ud558\ub294 \ub4f1 \uc2dc\ub828\uc744 \uacaa\uc5b4\uc57c \ud588\ub2e4. \uc774 \uc2dc\uae30\uc5d0 \uc74c\uc545 \ud504\ub85c\uadf8\ub7a8\uc5d0\uc11c \ud55c\uc131\ubcc4\uace1 OST \uc218\ub85d\uace1 \u3008 \uc77c\uc6d4\uc9c0\uac00 \u3009\ub97c \uc120\uc608 \ud640\ub85c \uacf5\uc5f0\ud560 \uae30\ud68c\uac00 \uc788\uc5c8\ub2e4. \ub2f9\uc2dc \uacf5\uc5f0\uc7a5\uc5d0 \uc788\ub358 \ud32c\ub4e4\uc774 \uac04\uc8fc \ubd80\ubd84\uc5d0\uc11c \u2018\uc78a\uc9c0 \uc54a\uace0 \uae30\ub2e4\ub9b4\uac8c \uc6d0\ub354\uac78\uc2a4\u2019\ub77c\uace0 \uc678\uce58\ub294 \uad6c\ud638 \uc18c\ub9ac\uc5d0 \ub300\uae30\uc2e4\ub85c \ub3cc\uc544\uc640 \ub9ce\uc774 \uc6b8\uc5c8\ub2e4\uace0 \ubc1d\ud788\uae30\ub3c4 \ud588\ub2e4. \uc6d0\ub354\uac78\uc2a4\ub294 \uc774 \uc2dc\uae30\uac00 \uac00\uc7a5 \ud798\ub4e4\uc5c8\ub358 \uc2dc\uae30\ub77c\uace0 \ud68c\uc0c1\ud588\ub2e4.", "sentence_answer": "\uc774 \uc2dc\uae30\uc5d0 \uc74c\uc545 \ud504\ub85c\uadf8\ub7a8\uc5d0\uc11c \ud55c\uc131\ubcc4\uace1 OST \uc218\ub85d\uace1 \u3008 \uc77c\uc6d4\uc9c0\uac00 \u3009\ub97c \uc120\uc608 \ud640\ub85c \uacf5\uc5f0\ud560 \uae30\ud68c\uac00 \uc788\uc5c8\ub2e4.", "paragraph_id": "6542307-5-1", "paragraph_question": "question: \uc74c\uc545 \ud504\ub85c\uadf8\ub7a8\uc5d0\uc11c \uc120\uc608 \ud63c\uc790 \uacf5\uc5f0\ud558\uc600\ub358 \ud55c\uc131\ubcc4\uace1 OST \uc218\ub85d\uace1\uc758 \uc81c\ubaa9\uc740 \ubb34\uc5c7\uc778\uac00?, context: \uc6d0\ub354\uac78\uc2a4\ub294 \"Irony\"\ub85c \ud65c\ubc1c\ud55c \ud65c\ub3d9\uc744 \ud588\uc73c\ub098 \uc774\ud6c4 \ub9ce\uc740 \ubd88\uc6b4\uc744 \uacaa\uac8c \ub41c\ub2e4. 6\uc6d4 19\uc77c \uba64\ubc84 \uc18c\ud76c\uac00 \uc601\ud654 \u300a\ub728\uac70\uc6b4 \uac83\uc774 \uc88b\uc544\u300b \ucd2c\uc601 \uc911 \uc624\ud1a0\ubc14\uc774 \uc0ac\uace0\ub85c \ubb34\ub98e\uc774 \ucc22\uc5b4\uc838 \uac01\uc885 \ud589\uc0ac \ubb34\ub300\uc5d0\uc11c \uc57d \ud55c \ub2ec\uac04 4\uc778 \uccb4\uc81c\ub85c \ud65c\ub3d9\ud558\uc600\uace0, \uc774\ud6c4 \ubb34\ub300\uc5d0\uc11c\ub294 \ud604\uc544\uae4c\uc9c0 \ube60\uc9c4 \uc138 \uba85\uc774 \ud65c\ub3d9\ud558\ub294 \ubaa8\uc2b5\uc744 \ubcf4\uc5ec \uc8fc\uc5c8\ub2e4. \uc758\ub958 \uad11\uace0 \ud654\ubcf4 \ucd2c\uc601\uc5d0\uc11c\ub3c4 \ud604\uc544\ub9cc \ube60\uc838 \uc788\uc5b4\uc11c \ud32c\ub4e4\uc758 \uad81\uae08\uc99d\uc744 \uc790\uc544\ub0b4\ub2e4\uac00, \ub9c8\uce68\ub0b4 7\uc6d4 30\uc77c \uba64\ubc84 \ud604\uc544\uc758 \ud0c8\ud1f4\uac00 \ubc1c\ud45c\ub418\uc5c8\ub2e4. \uc2b5\uad00\uc131 \uc7a5\uc5fc\uc73c\ub85c 3\uc6d4\uacfc 4\uc6d4\uc5d0 \uc7a0\uc2dc \uc785\uc6d0\ud558\ub294 \ub4f1 \ud65c\ub3d9 \ucd08\uae30\ubd80\ud130 \ucc28\uc9c8\uc744 \ube5a\uc5c8\ub358 \uadf8\ub140\ub294, \uc815\uc0c1\uc801\uc778 \ud65c\ub3d9\uc774 \ubd88\uac00\ub2a5\ud558\ub2e4\uace0 \ud310\ub2e8\ud55c \uc18c\uc18d\uc0ac \uce21\uc758 \ud0c8\ud1f4 \uacb0\uc815\uc744 \ubc1b\uc544\ub4e4\uc600\ub2e4. (\ud604\uc544\ub294 \ud604\uc7ac \uc194\ub85c\ub85c \ud65c\ub3d9\ud558\uace0 \uc788\ub2e4.) \uc774\ud6c4 \uc6d0\ub354\uac78\uc2a4\ub294 \ub0a8\uc740 \uba64\ubc84 \uc138 \uba85\uc5d0 \ubc31\ub304\uc11c\ub97c \ub300\ub3d9\ud558\uc5ec \ud65c\ub3d9\ud558\ub294 \ub4f1 \uc2dc\ub828\uc744 \uacaa\uc5b4\uc57c \ud588\ub2e4. \uc774 \uc2dc\uae30\uc5d0 \uc74c\uc545 \ud504\ub85c\uadf8\ub7a8\uc5d0\uc11c \ud55c\uc131\ubcc4\uace1 OST \uc218\ub85d\uace1 \u3008\uc77c\uc6d4\uc9c0\uac00\u3009\ub97c \uc120\uc608 \ud640\ub85c \uacf5\uc5f0\ud560 \uae30\ud68c\uac00 \uc788\uc5c8\ub2e4. \ub2f9\uc2dc \uacf5\uc5f0\uc7a5\uc5d0 \uc788\ub358 \ud32c\ub4e4\uc774 \uac04\uc8fc \ubd80\ubd84\uc5d0\uc11c \u2018\uc78a\uc9c0 \uc54a\uace0 \uae30\ub2e4\ub9b4\uac8c \uc6d0\ub354\uac78\uc2a4\u2019\ub77c\uace0 \uc678\uce58\ub294 \uad6c\ud638 \uc18c\ub9ac\uc5d0 \ub300\uae30\uc2e4\ub85c \ub3cc\uc544\uc640 \ub9ce\uc774 \uc6b8\uc5c8\ub2e4\uace0 \ubc1d\ud788\uae30\ub3c4 \ud588\ub2e4. \uc6d0\ub354\uac78\uc2a4\ub294 \uc774 \uc2dc\uae30\uac00 \uac00\uc7a5 \ud798\ub4e4\uc5c8\ub358 \uc2dc\uae30\ub77c\uace0 \ud68c\uc0c1\ud588\ub2e4."} {"question": "\uad50\uc6d0\uc591\uc131\uacfc \uad50\uc721\uc5f0\uad6c\uc758 \uc911\uc2ec\uc801 \uc5ed\ud560\uc744 \ud560 \uc218 \uc788\ub3c4\ub85d \ud55c\uad6d\uad50\uc721\uac1c\ubc1c\uc6d0\uc774 \uc81c\uc548\ud55c \ub300\ud559\uad50\uba85\uc740?", "paragraph": "1978\ub144 12\uc6d4 \ud55c\uad6d\uad50\uc721\uac1c\ubc1c\uc6d0\uc774 \ud55c\uad6d\uad50\uc721 30\ub144\uc744 \uae30\ub150\ud558\uc5ec \uac01\uacc4\uc758 \uc5ec\ub860\uacfc \ud559\uc790\ub4e4\uc758 \uc5f0\uad6c\ub97c \ud1a0\ub300\ub85c \uc791\uc131\ud55c \u300a\uad50\uc721\ubc1c\uc804\uc758 \uc804\ub9dd\uacfc \uacfc\uc81c\u300b \ubcf4\uace0\uc11c\uc5d0\uc11c \u2018\uad50\uc721\ub300\ud559\uacfc \uc0ac\ubc94\ub300\ud559\uc744 \ud1b5\ud569 \ubc1c\uc804\uc2dc\ucf1c \uc815\uc608 \uad50\uc6d0\uc591\uc131\uacfc \uad50\uc721\uc5f0\uad6c\uc758 \uc911\uc2ec\uc801 \uc5ed\ud560\uc744 \ud560 \uc218 \uc788\ub294 \uc885\ud569\uad50\uc6d0\uad50\uc721\uae30\uad00\u2019\uc758 \uc124\ub9bd \ud544\uc694\uc131\uc744 \uc81c\uae30\ud558\uc600\ub2e4. 1980\ub144 12\uc6d4\uc5d0\ub294 \ub300\ud1b5\ub839 \uc0b0\ud558 \ud2b9\uc218\uc790\ubb38\uae30\uad00\uc778 \uacbd\uc81c\uacfc\ud559\uc2ec\uc758\ud68c\uc758\uc5d0\uc11c \ud504\ub791\uc2a4\uc758 \uace0\ub4f1\uc0ac\ubc94\ud559\uad50\ub97c \ubaa8\ub378\ub85c \ud558\ub294 \uad6d\ub9bd\uc0ac\ubc94\ub300\ud559\uad50\uc758 \uc124\ub9bd\uc774 \uac74\uc758\ub418\uc5c8\ub2e4. \uc774\uc640 \uad00\ub828\ud574 \ud55c\uad6d\uad50\uc721\uac1c\ubc1c\uc6d0\uc774 1981\ub144 6\uc6d4 \uc5f4\ub9b0 \u300a\uad50\uc6d0\uc778\uc0ac\uc81c\ub3c4\uc758 \uac1c\uc120\ubc29\ud5a5 \ud0d0\uc0c9\uc744 \uc704\ud55c \uc138\ubbf8\ub098\u300b\uc5d0\uc11c \u2018\uad50\uc6d0\uad50\uc721\uc885\ud569\ub300\ud559\uad50(\uac00\uce6d)\u2019\uc758 \uc124\ub9bd\uc744 \uc815\uc2dd\uc73c\ub85c \uc81c\uc548\ud558\uc600\uace0, \uc815\ubd80\ub294 \uc774\ub7ec\ud55c \uad6c\uc0c1\uc744 \uac19\uc740 \ud574 12\uc6d4 \uc81c5\ucc28 \uacbd\uc81c\uc0ac\ud68c\ubc1c\uc804 5\uac1c\ub144 \uacc4\ud68d \uad50\uc721 \ubd80\ubb38\uc5d0 \ubc18\uc601\ud558\uc600\ub2e4.", "answer": "\uad50\uc6d0\uad50\uc721\uc885\ud569\ub300\ud559\uad50", "sentence": "\uc774\uc640 \uad00\ub828\ud574 \ud55c\uad6d\uad50\uc721\uac1c\ubc1c\uc6d0\uc774 1981\ub144 6\uc6d4 \uc5f4\ub9b0 \u300a\uad50\uc6d0\uc778\uc0ac\uc81c\ub3c4\uc758 \uac1c\uc120\ubc29\ud5a5 \ud0d0\uc0c9\uc744 \uc704\ud55c \uc138\ubbf8\ub098\u300b\uc5d0\uc11c \u2018 \uad50\uc6d0\uad50\uc721\uc885\ud569\ub300\ud559\uad50 (\uac00\uce6d)\u2019\uc758 \uc124\ub9bd\uc744 \uc815\uc2dd\uc73c\ub85c \uc81c\uc548\ud558\uc600\uace0, \uc815\ubd80\ub294 \uc774\ub7ec\ud55c \uad6c\uc0c1\uc744 \uac19\uc740 \ud574 12\uc6d4 \uc81c5\ucc28 \uacbd\uc81c\uc0ac\ud68c\ubc1c\uc804 5\uac1c\ub144 \uacc4\ud68d \uad50\uc721 \ubd80\ubb38\uc5d0 \ubc18\uc601\ud558\uc600\ub2e4.", "paragraph_sentence": "1978\ub144 12\uc6d4 \ud55c\uad6d\uad50\uc721\uac1c\ubc1c\uc6d0\uc774 \ud55c\uad6d\uad50\uc721 30\ub144\uc744 \uae30\ub150\ud558\uc5ec \uac01\uacc4\uc758 \uc5ec\ub860\uacfc \ud559\uc790\ub4e4\uc758 \uc5f0\uad6c\ub97c \ud1a0\ub300\ub85c \uc791\uc131\ud55c \u300a\uad50\uc721\ubc1c\uc804\uc758 \uc804\ub9dd\uacfc \uacfc\uc81c\u300b \ubcf4\uace0\uc11c\uc5d0\uc11c \u2018\uad50\uc721\ub300\ud559\uacfc \uc0ac\ubc94\ub300\ud559\uc744 \ud1b5\ud569 \ubc1c\uc804\uc2dc\ucf1c \uc815\uc608 \uad50\uc6d0\uc591\uc131\uacfc \uad50\uc721\uc5f0\uad6c\uc758 \uc911\uc2ec\uc801 \uc5ed\ud560\uc744 \ud560 \uc218 \uc788\ub294 \uc885\ud569\uad50\uc6d0\uad50\uc721\uae30\uad00\u2019\uc758 \uc124\ub9bd \ud544\uc694\uc131\uc744 \uc81c\uae30\ud558\uc600\ub2e4. 1980\ub144 12\uc6d4\uc5d0\ub294 \ub300\ud1b5\ub839 \uc0b0\ud558 \ud2b9\uc218\uc790\ubb38\uae30\uad00\uc778 \uacbd\uc81c\uacfc\ud559\uc2ec\uc758\ud68c\uc758\uc5d0\uc11c \ud504\ub791\uc2a4\uc758 \uace0\ub4f1\uc0ac\ubc94\ud559\uad50\ub97c \ubaa8\ub378\ub85c \ud558\ub294 \uad6d\ub9bd\uc0ac\ubc94\ub300\ud559\uad50\uc758 \uc124\ub9bd\uc774 \uac74\uc758\ub418\uc5c8\ub2e4. \uc774\uc640 \uad00\ub828\ud574 \ud55c\uad6d\uad50\uc721\uac1c\ubc1c\uc6d0\uc774 1981\ub144 6\uc6d4 \uc5f4\ub9b0 \u300a\uad50\uc6d0\uc778\uc0ac\uc81c\ub3c4\uc758 \uac1c\uc120\ubc29\ud5a5 \ud0d0\uc0c9\uc744 \uc704\ud55c \uc138\ubbf8\ub098\u300b\uc5d0\uc11c \u2018 \uad50\uc6d0\uad50\uc721\uc885\ud569\ub300\ud559\uad50 (\uac00\uce6d)\u2019\uc758 \uc124\ub9bd\uc744 \uc815\uc2dd\uc73c\ub85c \uc81c\uc548\ud558\uc600\uace0, \uc815\ubd80\ub294 \uc774\ub7ec\ud55c \uad6c\uc0c1\uc744 \uac19\uc740 \ud574 12\uc6d4 \uc81c5\ucc28 \uacbd\uc81c\uc0ac\ud68c\ubc1c\uc804 5\uac1c\ub144 \uacc4\ud68d \uad50\uc721 \ubd80\ubb38\uc5d0 \ubc18\uc601\ud558\uc600\ub2e4. ", "paragraph_answer": "1978\ub144 12\uc6d4 \ud55c\uad6d\uad50\uc721\uac1c\ubc1c\uc6d0\uc774 \ud55c\uad6d\uad50\uc721 30\ub144\uc744 \uae30\ub150\ud558\uc5ec \uac01\uacc4\uc758 \uc5ec\ub860\uacfc \ud559\uc790\ub4e4\uc758 \uc5f0\uad6c\ub97c \ud1a0\ub300\ub85c \uc791\uc131\ud55c \u300a\uad50\uc721\ubc1c\uc804\uc758 \uc804\ub9dd\uacfc \uacfc\uc81c\u300b \ubcf4\uace0\uc11c\uc5d0\uc11c \u2018\uad50\uc721\ub300\ud559\uacfc \uc0ac\ubc94\ub300\ud559\uc744 \ud1b5\ud569 \ubc1c\uc804\uc2dc\ucf1c \uc815\uc608 \uad50\uc6d0\uc591\uc131\uacfc \uad50\uc721\uc5f0\uad6c\uc758 \uc911\uc2ec\uc801 \uc5ed\ud560\uc744 \ud560 \uc218 \uc788\ub294 \uc885\ud569\uad50\uc6d0\uad50\uc721\uae30\uad00\u2019\uc758 \uc124\ub9bd \ud544\uc694\uc131\uc744 \uc81c\uae30\ud558\uc600\ub2e4. 1980\ub144 12\uc6d4\uc5d0\ub294 \ub300\ud1b5\ub839 \uc0b0\ud558 \ud2b9\uc218\uc790\ubb38\uae30\uad00\uc778 \uacbd\uc81c\uacfc\ud559\uc2ec\uc758\ud68c\uc758\uc5d0\uc11c \ud504\ub791\uc2a4\uc758 \uace0\ub4f1\uc0ac\ubc94\ud559\uad50\ub97c \ubaa8\ub378\ub85c \ud558\ub294 \uad6d\ub9bd\uc0ac\ubc94\ub300\ud559\uad50\uc758 \uc124\ub9bd\uc774 \uac74\uc758\ub418\uc5c8\ub2e4. \uc774\uc640 \uad00\ub828\ud574 \ud55c\uad6d\uad50\uc721\uac1c\ubc1c\uc6d0\uc774 1981\ub144 6\uc6d4 \uc5f4\ub9b0 \u300a\uad50\uc6d0\uc778\uc0ac\uc81c\ub3c4\uc758 \uac1c\uc120\ubc29\ud5a5 \ud0d0\uc0c9\uc744 \uc704\ud55c \uc138\ubbf8\ub098\u300b\uc5d0\uc11c \u2018 \uad50\uc6d0\uad50\uc721\uc885\ud569\ub300\ud559\uad50 (\uac00\uce6d)\u2019\uc758 \uc124\ub9bd\uc744 \uc815\uc2dd\uc73c\ub85c \uc81c\uc548\ud558\uc600\uace0, \uc815\ubd80\ub294 \uc774\ub7ec\ud55c \uad6c\uc0c1\uc744 \uac19\uc740 \ud574 12\uc6d4 \uc81c5\ucc28 \uacbd\uc81c\uc0ac\ud68c\ubc1c\uc804 5\uac1c\ub144 \uacc4\ud68d \uad50\uc721 \ubd80\ubb38\uc5d0 \ubc18\uc601\ud558\uc600\ub2e4.", "sentence_answer": "\uc774\uc640 \uad00\ub828\ud574 \ud55c\uad6d\uad50\uc721\uac1c\ubc1c\uc6d0\uc774 1981\ub144 6\uc6d4 \uc5f4\ub9b0 \u300a\uad50\uc6d0\uc778\uc0ac\uc81c\ub3c4\uc758 \uac1c\uc120\ubc29\ud5a5 \ud0d0\uc0c9\uc744 \uc704\ud55c \uc138\ubbf8\ub098\u300b\uc5d0\uc11c \u2018 \uad50\uc6d0\uad50\uc721\uc885\ud569\ub300\ud559\uad50 (\uac00\uce6d)\u2019\uc758 \uc124\ub9bd\uc744 \uc815\uc2dd\uc73c\ub85c \uc81c\uc548\ud558\uc600\uace0, \uc815\ubd80\ub294 \uc774\ub7ec\ud55c \uad6c\uc0c1\uc744 \uac19\uc740 \ud574 12\uc6d4 \uc81c5\ucc28 \uacbd\uc81c\uc0ac\ud68c\ubc1c\uc804 5\uac1c\ub144 \uacc4\ud68d \uad50\uc721 \ubd80\ubb38\uc5d0 \ubc18\uc601\ud558\uc600\ub2e4.", "paragraph_id": "6569877-0-1", "paragraph_question": "question: \uad50\uc6d0\uc591\uc131\uacfc \uad50\uc721\uc5f0\uad6c\uc758 \uc911\uc2ec\uc801 \uc5ed\ud560\uc744 \ud560 \uc218 \uc788\ub3c4\ub85d \ud55c\uad6d\uad50\uc721\uac1c\ubc1c\uc6d0\uc774 \uc81c\uc548\ud55c \ub300\ud559\uad50\uba85\uc740?, context: 1978\ub144 12\uc6d4 \ud55c\uad6d\uad50\uc721\uac1c\ubc1c\uc6d0\uc774 \ud55c\uad6d\uad50\uc721 30\ub144\uc744 \uae30\ub150\ud558\uc5ec \uac01\uacc4\uc758 \uc5ec\ub860\uacfc \ud559\uc790\ub4e4\uc758 \uc5f0\uad6c\ub97c \ud1a0\ub300\ub85c \uc791\uc131\ud55c \u300a\uad50\uc721\ubc1c\uc804\uc758 \uc804\ub9dd\uacfc \uacfc\uc81c\u300b \ubcf4\uace0\uc11c\uc5d0\uc11c \u2018\uad50\uc721\ub300\ud559\uacfc \uc0ac\ubc94\ub300\ud559\uc744 \ud1b5\ud569 \ubc1c\uc804\uc2dc\ucf1c \uc815\uc608 \uad50\uc6d0\uc591\uc131\uacfc \uad50\uc721\uc5f0\uad6c\uc758 \uc911\uc2ec\uc801 \uc5ed\ud560\uc744 \ud560 \uc218 \uc788\ub294 \uc885\ud569\uad50\uc6d0\uad50\uc721\uae30\uad00\u2019\uc758 \uc124\ub9bd \ud544\uc694\uc131\uc744 \uc81c\uae30\ud558\uc600\ub2e4. 1980\ub144 12\uc6d4\uc5d0\ub294 \ub300\ud1b5\ub839 \uc0b0\ud558 \ud2b9\uc218\uc790\ubb38\uae30\uad00\uc778 \uacbd\uc81c\uacfc\ud559\uc2ec\uc758\ud68c\uc758\uc5d0\uc11c \ud504\ub791\uc2a4\uc758 \uace0\ub4f1\uc0ac\ubc94\ud559\uad50\ub97c \ubaa8\ub378\ub85c \ud558\ub294 \uad6d\ub9bd\uc0ac\ubc94\ub300\ud559\uad50\uc758 \uc124\ub9bd\uc774 \uac74\uc758\ub418\uc5c8\ub2e4. \uc774\uc640 \uad00\ub828\ud574 \ud55c\uad6d\uad50\uc721\uac1c\ubc1c\uc6d0\uc774 1981\ub144 6\uc6d4 \uc5f4\ub9b0 \u300a\uad50\uc6d0\uc778\uc0ac\uc81c\ub3c4\uc758 \uac1c\uc120\ubc29\ud5a5 \ud0d0\uc0c9\uc744 \uc704\ud55c \uc138\ubbf8\ub098\u300b\uc5d0\uc11c \u2018\uad50\uc6d0\uad50\uc721\uc885\ud569\ub300\ud559\uad50(\uac00\uce6d)\u2019\uc758 \uc124\ub9bd\uc744 \uc815\uc2dd\uc73c\ub85c \uc81c\uc548\ud558\uc600\uace0, \uc815\ubd80\ub294 \uc774\ub7ec\ud55c \uad6c\uc0c1\uc744 \uac19\uc740 \ud574 12\uc6d4 \uc81c5\ucc28 \uacbd\uc81c\uc0ac\ud68c\ubc1c\uc804 5\uac1c\ub144 \uacc4\ud68d \uad50\uc721 \ubd80\ubb38\uc5d0 \ubc18\uc601\ud558\uc600\ub2e4."} {"question": "\ub300\uaddc\ubaa8 \uc6d0\uc720\ub294 \uc5b4\ub290 \ubc14\ub2e4 \uc804\uccb4\ub85c \ud37c\uc838\ub098\uac08 \uac83\uc774\ub77c\ub294 \uc804\ub9dd\uc774\uc788\uc5c8\ub294\uac00?", "paragraph": "\uc720\ucd9c \uc6d0\uc720\uc758 \uc9c1\uc811\uc801 \uc81c1\ucc28 \ud53c\ud574\uc9c0\uc5ed\ub4e4\uc778 \ubbf8\uad6d\uc758 \uba55\uc2dc\ucf54\ub9cc \uc5f0\uc548 5\uac1c \uc8fc \uc911 4\uac1c \ubbf8\uc2dc\uc2dc\ud53c \uc8fc, \uc568\ub77c\ubc30\ub9c8 \uc8fc, \ub8e8\uc774\uc9c0\uc560\ub098 \uc8fc, \ud50c\ub85c\ub9ac\ub2e4 \uc8fc\ub4f1\uc5d0 \uae09\uc18d\ud788 \ud37c\uc838\uac14\ub2e4. \ud2b9\ud788 \ub8e8\uc774\uc9c0\uc560\ub098\ub85c\uc758 \uc6d0\uc720 \uc0c1\ub959\uc740 \ud604\uc9c0 \uc2dc\uac01 \uae30\uc900\uc73c\ub85c 5\uc6d4 2\uc77c\ub85c \uc608\uce21\ud588\uc73c\uba70 \uc774\ub97c \uc2dc\uc791\uc73c\ub85c \ub300\uaddc\ubaa8 \uc6d0\uc720\uac00 \uba55\uc2dc\ucf54 \ub9cc \uc804\uc5ed\uc744 \uac15\ud0c0\ud560 \uac83\uc73c\ub85c \ubcf4\uc778\ub2e4. \ud2b9\ud788 \uba55\uc2dc\ucf54 \ub9cc\ub958\uac00 \uc870\ub958\uc758 \ud750\ub984\uc774 \uac70\uc13c \uac83\uc73c\ub85c \uc720\uba85\ud55c \ud574\ub958\uc5ec\uc11c, \uc5c4\uccad\ub09c \uc18d\ub3c4\uc758 \uc624\uc5fc \ud655\uc0b0\uc774 \uc6b0\ub824\ub418\uace0 \uc788\ub2e4. \uc5ec\uae30\uc5d0 \ub300\uc11c\uc591 \uc804\uccb4\ub85c \uae09\uc18d\ud788 \ud37c\uc838\ub098\uac08 \uac70\ub77c\ub294 \uc804\ub9dd\uc774 \uc788\uc5b4 \uc0ac\ud0dc\uac00 \ub9e4\uc6b0 \uc2ec\uac01\ud558\ub2e4. \uba55\uc2dc\ucf54\ub9cc\uc740 \uc774 \uc9c0\uc5ed \ucd5c\ub300 \uc5b4\uc7a5\uc911 \ud558\ub098\uc774\ub2e4. \uc11d\uc720\uac00 \uc774\uacf3\uc5d0 \uc0dd\uaca8\uc640 \ud574\uc548\uc744 \ud615\uc131\ud558\uba74\uc11c \uc808\ub9dd\uc774 \ucee4\uc838\uac14\ub2e4. \uc0ac\uace0\ub97c \uc218\uc2b5\ud558\uae30 \uc704\ud574 \ubbf8\uad6d \uc815\ubd80\ub294 BP\ub97c \uc555\ubc15\ud588\uace0 BP\ub294 \ub9ce\uc740 \ube44\ud310\uc744 \ubc1b\uc558\ub2e4.", "answer": "\ub300\uc11c\uc591", "sentence": "\uc5ec\uae30\uc5d0 \ub300\uc11c\uc591 \uc804\uccb4\ub85c \uae09\uc18d\ud788 \ud37c\uc838\ub098\uac08 \uac70\ub77c\ub294 \uc804\ub9dd\uc774 \uc788\uc5b4 \uc0ac\ud0dc\uac00 \ub9e4\uc6b0 \uc2ec\uac01\ud558\ub2e4.", "paragraph_sentence": "\uc720\ucd9c \uc6d0\uc720\uc758 \uc9c1\uc811\uc801 \uc81c1\ucc28 \ud53c\ud574\uc9c0\uc5ed\ub4e4\uc778 \ubbf8\uad6d\uc758 \uba55\uc2dc\ucf54\ub9cc \uc5f0\uc548 5\uac1c \uc8fc \uc911 4\uac1c \ubbf8\uc2dc\uc2dc\ud53c \uc8fc, \uc568\ub77c\ubc30\ub9c8 \uc8fc, \ub8e8\uc774\uc9c0\uc560\ub098 \uc8fc, \ud50c\ub85c\ub9ac\ub2e4 \uc8fc\ub4f1\uc5d0 \uae09\uc18d\ud788 \ud37c\uc838\uac14\ub2e4. \ud2b9\ud788 \ub8e8\uc774\uc9c0\uc560\ub098\ub85c\uc758 \uc6d0\uc720 \uc0c1\ub959\uc740 \ud604\uc9c0 \uc2dc\uac01 \uae30\uc900\uc73c\ub85c 5\uc6d4 2\uc77c\ub85c \uc608\uce21\ud588\uc73c\uba70 \uc774\ub97c \uc2dc\uc791\uc73c\ub85c \ub300\uaddc\ubaa8 \uc6d0\uc720\uac00 \uba55\uc2dc\ucf54 \ub9cc \uc804\uc5ed\uc744 \uac15\ud0c0\ud560 \uac83\uc73c\ub85c \ubcf4\uc778\ub2e4. \ud2b9\ud788 \uba55\uc2dc\ucf54 \ub9cc\ub958\uac00 \uc870\ub958\uc758 \ud750\ub984\uc774 \uac70\uc13c \uac83\uc73c\ub85c \uc720\uba85\ud55c \ud574\ub958\uc5ec\uc11c, \uc5c4\uccad\ub09c \uc18d\ub3c4\uc758 \uc624\uc5fc \ud655\uc0b0\uc774 \uc6b0\ub824\ub418\uace0 \uc788\ub2e4. \uc5ec\uae30\uc5d0 \ub300\uc11c\uc591 \uc804\uccb4\ub85c \uae09\uc18d\ud788 \ud37c\uc838\ub098\uac08 \uac70\ub77c\ub294 \uc804\ub9dd\uc774 \uc788\uc5b4 \uc0ac\ud0dc\uac00 \ub9e4\uc6b0 \uc2ec\uac01\ud558\ub2e4. \uba55\uc2dc\ucf54\ub9cc\uc740 \uc774 \uc9c0\uc5ed \ucd5c\ub300 \uc5b4\uc7a5\uc911 \ud558\ub098\uc774\ub2e4. \uc11d\uc720\uac00 \uc774\uacf3\uc5d0 \uc0dd\uaca8\uc640 \ud574\uc548\uc744 \ud615\uc131\ud558\uba74\uc11c \uc808\ub9dd\uc774 \ucee4\uc838\uac14\ub2e4. \uc0ac\uace0\ub97c \uc218\uc2b5\ud558\uae30 \uc704\ud574 \ubbf8\uad6d \uc815\ubd80\ub294 BP\ub97c \uc555\ubc15\ud588\uace0 BP\ub294 \ub9ce\uc740 \ube44\ud310\uc744 \ubc1b\uc558\ub2e4.", "paragraph_answer": "\uc720\ucd9c \uc6d0\uc720\uc758 \uc9c1\uc811\uc801 \uc81c1\ucc28 \ud53c\ud574\uc9c0\uc5ed\ub4e4\uc778 \ubbf8\uad6d\uc758 \uba55\uc2dc\ucf54\ub9cc \uc5f0\uc548 5\uac1c \uc8fc \uc911 4\uac1c \ubbf8\uc2dc\uc2dc\ud53c \uc8fc, \uc568\ub77c\ubc30\ub9c8 \uc8fc, \ub8e8\uc774\uc9c0\uc560\ub098 \uc8fc, \ud50c\ub85c\ub9ac\ub2e4 \uc8fc\ub4f1\uc5d0 \uae09\uc18d\ud788 \ud37c\uc838\uac14\ub2e4. \ud2b9\ud788 \ub8e8\uc774\uc9c0\uc560\ub098\ub85c\uc758 \uc6d0\uc720 \uc0c1\ub959\uc740 \ud604\uc9c0 \uc2dc\uac01 \uae30\uc900\uc73c\ub85c 5\uc6d4 2\uc77c\ub85c \uc608\uce21\ud588\uc73c\uba70 \uc774\ub97c \uc2dc\uc791\uc73c\ub85c \ub300\uaddc\ubaa8 \uc6d0\uc720\uac00 \uba55\uc2dc\ucf54 \ub9cc \uc804\uc5ed\uc744 \uac15\ud0c0\ud560 \uac83\uc73c\ub85c \ubcf4\uc778\ub2e4. \ud2b9\ud788 \uba55\uc2dc\ucf54 \ub9cc\ub958\uac00 \uc870\ub958\uc758 \ud750\ub984\uc774 \uac70\uc13c \uac83\uc73c\ub85c \uc720\uba85\ud55c \ud574\ub958\uc5ec\uc11c, \uc5c4\uccad\ub09c \uc18d\ub3c4\uc758 \uc624\uc5fc \ud655\uc0b0\uc774 \uc6b0\ub824\ub418\uace0 \uc788\ub2e4. \uc5ec\uae30\uc5d0 \ub300\uc11c\uc591 \uc804\uccb4\ub85c \uae09\uc18d\ud788 \ud37c\uc838\ub098\uac08 \uac70\ub77c\ub294 \uc804\ub9dd\uc774 \uc788\uc5b4 \uc0ac\ud0dc\uac00 \ub9e4\uc6b0 \uc2ec\uac01\ud558\ub2e4. \uba55\uc2dc\ucf54\ub9cc\uc740 \uc774 \uc9c0\uc5ed \ucd5c\ub300 \uc5b4\uc7a5\uc911 \ud558\ub098\uc774\ub2e4. \uc11d\uc720\uac00 \uc774\uacf3\uc5d0 \uc0dd\uaca8\uc640 \ud574\uc548\uc744 \ud615\uc131\ud558\uba74\uc11c \uc808\ub9dd\uc774 \ucee4\uc838\uac14\ub2e4. \uc0ac\uace0\ub97c \uc218\uc2b5\ud558\uae30 \uc704\ud574 \ubbf8\uad6d \uc815\ubd80\ub294 BP\ub97c \uc555\ubc15\ud588\uace0 BP\ub294 \ub9ce\uc740 \ube44\ud310\uc744 \ubc1b\uc558\ub2e4.", "sentence_answer": "\uc5ec\uae30\uc5d0 \ub300\uc11c\uc591 \uc804\uccb4\ub85c \uae09\uc18d\ud788 \ud37c\uc838\ub098\uac08 \uac70\ub77c\ub294 \uc804\ub9dd\uc774 \uc788\uc5b4 \uc0ac\ud0dc\uac00 \ub9e4\uc6b0 \uc2ec\uac01\ud558\ub2e4.", "paragraph_id": "6513252-4-1", "paragraph_question": "question: \ub300\uaddc\ubaa8 \uc6d0\uc720\ub294 \uc5b4\ub290 \ubc14\ub2e4 \uc804\uccb4\ub85c \ud37c\uc838\ub098\uac08 \uac83\uc774\ub77c\ub294 \uc804\ub9dd\uc774\uc788\uc5c8\ub294\uac00?, context: \uc720\ucd9c \uc6d0\uc720\uc758 \uc9c1\uc811\uc801 \uc81c1\ucc28 \ud53c\ud574\uc9c0\uc5ed\ub4e4\uc778 \ubbf8\uad6d\uc758 \uba55\uc2dc\ucf54\ub9cc \uc5f0\uc548 5\uac1c \uc8fc \uc911 4\uac1c \ubbf8\uc2dc\uc2dc\ud53c \uc8fc, \uc568\ub77c\ubc30\ub9c8 \uc8fc, \ub8e8\uc774\uc9c0\uc560\ub098 \uc8fc, \ud50c\ub85c\ub9ac\ub2e4 \uc8fc\ub4f1\uc5d0 \uae09\uc18d\ud788 \ud37c\uc838\uac14\ub2e4. \ud2b9\ud788 \ub8e8\uc774\uc9c0\uc560\ub098\ub85c\uc758 \uc6d0\uc720 \uc0c1\ub959\uc740 \ud604\uc9c0 \uc2dc\uac01 \uae30\uc900\uc73c\ub85c 5\uc6d4 2\uc77c\ub85c \uc608\uce21\ud588\uc73c\uba70 \uc774\ub97c \uc2dc\uc791\uc73c\ub85c \ub300\uaddc\ubaa8 \uc6d0\uc720\uac00 \uba55\uc2dc\ucf54 \ub9cc \uc804\uc5ed\uc744 \uac15\ud0c0\ud560 \uac83\uc73c\ub85c \ubcf4\uc778\ub2e4. \ud2b9\ud788 \uba55\uc2dc\ucf54 \ub9cc\ub958\uac00 \uc870\ub958\uc758 \ud750\ub984\uc774 \uac70\uc13c \uac83\uc73c\ub85c \uc720\uba85\ud55c \ud574\ub958\uc5ec\uc11c, \uc5c4\uccad\ub09c \uc18d\ub3c4\uc758 \uc624\uc5fc \ud655\uc0b0\uc774 \uc6b0\ub824\ub418\uace0 \uc788\ub2e4. \uc5ec\uae30\uc5d0 \ub300\uc11c\uc591 \uc804\uccb4\ub85c \uae09\uc18d\ud788 \ud37c\uc838\ub098\uac08 \uac70\ub77c\ub294 \uc804\ub9dd\uc774 \uc788\uc5b4 \uc0ac\ud0dc\uac00 \ub9e4\uc6b0 \uc2ec\uac01\ud558\ub2e4. \uba55\uc2dc\ucf54\ub9cc\uc740 \uc774 \uc9c0\uc5ed \ucd5c\ub300 \uc5b4\uc7a5\uc911 \ud558\ub098\uc774\ub2e4. \uc11d\uc720\uac00 \uc774\uacf3\uc5d0 \uc0dd\uaca8\uc640 \ud574\uc548\uc744 \ud615\uc131\ud558\uba74\uc11c \uc808\ub9dd\uc774 \ucee4\uc838\uac14\ub2e4. \uc0ac\uace0\ub97c \uc218\uc2b5\ud558\uae30 \uc704\ud574 \ubbf8\uad6d \uc815\ubd80\ub294 BP\ub97c \uc555\ubc15\ud588\uace0 BP\ub294 \ub9ce\uc740 \ube44\ud310\uc744 \ubc1b\uc558\ub2e4."} {"question": "\ub9b4 \uc6e8\uc778\uc758 2008\ub144 \uc194\ub85c\uc568\ubc94 \uc218\ub85d\uace1\uc778 'Got Money'\uc758 \ube4c\ubcf4\ub4dc \ucc28\ud2b8 \uc21c\uc704\ub294?", "paragraph": "2008\ub144 \uadf8\uc758 \uc194\ub85c\uc568\ubc94 [Tha Carter 3]\uc744 \ubc1c\ub9e4, \uccab \ubc88\uc9f8 \uc2f1\uae00 \u3008Lollipop\u3009\uc740 \ubc1c\ub9e4\ub418\uc790\ub9c8\uc790 \uc628\ub77c\uc778 \ub2e4\uc6b4\ub85c\ub4dc 1,700,000 \uac74\uc744 \uae30\ub85d\ud558\ub294 \ud3ed\ubc1c\uc801\uc778 \ud788\ud2b8\ub97c \uae30\ub85d\ud588\ub2e4. \ubbf8\uad6d\uc5d0\uc11c\ub294 300\ub9cc \uc7a5 \uac00\uae4c\uc774 \ud314\uc544\ub4e4\uc600\uc73c\uba70, \uadf8 \ubc16\uc5d0\ub3c4 \ud788\ud2b8 \uc2f1\uae00\ub4e4\uc778 'Got Money'\ub294 \ube4c\ubcf4\ub4dc \ucc28\ud2b8 13\uc704\ub97c \ucc28\uc9c0\ud588\uace0, \uccab\uc2f1\uae00\uc778 'Lollipop'\uc740 \uc804 \uc138\uacc4 \uac01\uad6d\uc5d0\uc11c 1\uc704\ub97c \ucc28\uc9c0\ud588\ub2e4. \ub450 \ubc88\uc9f8 \uc2f1\uae00\uc778 'A Milli'\ub294 \ubbf8\uad6d \ubaa8\ub4e0 \ud799\ud569 \ucc28\ud2b8\uc5d0\uc11c 1\uc704\ub97c \uae30\ub85d\ud558\uace0 \uce90\ub098\ub2e4 \ucc28\ud2b8\uc5d0\uc11c\ub294 54\uc704\ub97c \ud588\ub2e4. \ub9c8\uc9c0\ub9c9 \uc2f1\uae00\uc778 'Mrs.Officer'\uc740 R&B\uac00\uc218 \ubc14\ube44 \ubc1c\ub80c\ud2f0\ub178\uac00 \ucc38\uc5ec\ud588\uc73c\uba70, \ub7a9\ucc28\ud2b8\uc5d0\uc11c 2\uc704\ub97c \uae30\ub85d\ud588\ub2e4. \uc774\uc678\uc5d0\ub3c4 \ud2b8\ub799 \uc911\uc758 \ud558\ub098\uc774\uba70 \uc81c\uc774 \uc9c0\uac00 \ucc38\uc5ec\ud55c 'Mr. Carter'\uc740 \ub7a9\ucc28\ud2b8\uc5d0\uc11c 13\uc704\ub97c \uac70\ub450\uc5c8\uace0 \uc774\ub4ec\ud574 \uadf8\ub798\ubbf8 \uc5b4\uc6cc\ub4dc\uc5d0\uc11c '\ubca0\uc2a4\ud2b8 \ub4c0\uc624 \ud799\ud569 \uc0c1'\uc744 \ubc1b\uc558\ub2e4.", "answer": "13\uc704", "sentence": "'Got Money'\ub294 \ube4c\ubcf4\ub4dc \ucc28\ud2b8 13\uc704 \ub97c \ucc28\uc9c0\ud588\uace0, \uccab\uc2f1\uae00\uc778 'Lollipop'\uc740 \uc804 \uc138\uacc4 \uac01\uad6d\uc5d0\uc11c 1\uc704\ub97c \ucc28\uc9c0\ud588\ub2e4.", "paragraph_sentence": "2008\ub144 \uadf8\uc758 \uc194\ub85c\uc568\ubc94 [Tha Carter 3]\uc744 \ubc1c\ub9e4, \uccab \ubc88\uc9f8 \uc2f1\uae00 \u3008Lollipop\u3009\uc740 \ubc1c\ub9e4\ub418\uc790\ub9c8\uc790 \uc628\ub77c\uc778 \ub2e4\uc6b4\ub85c\ub4dc 1,700,000 \uac74\uc744 \uae30\ub85d\ud558\ub294 \ud3ed\ubc1c\uc801\uc778 \ud788\ud2b8\ub97c \uae30\ub85d\ud588\ub2e4. \ubbf8\uad6d\uc5d0\uc11c\ub294 300\ub9cc \uc7a5 \uac00\uae4c\uc774 \ud314\uc544\ub4e4\uc600\uc73c\uba70, \uadf8 \ubc16\uc5d0\ub3c4 \ud788\ud2b8 \uc2f1\uae00\ub4e4\uc778 'Got Money'\ub294 \ube4c\ubcf4\ub4dc \ucc28\ud2b8 13\uc704 \ub97c \ucc28\uc9c0\ud588\uace0, \uccab\uc2f1\uae00\uc778 'Lollipop'\uc740 \uc804 \uc138\uacc4 \uac01\uad6d\uc5d0\uc11c 1\uc704\ub97c \ucc28\uc9c0\ud588\ub2e4. \ub450 \ubc88\uc9f8 \uc2f1\uae00\uc778 'A Milli'\ub294 \ubbf8\uad6d \ubaa8\ub4e0 \ud799\ud569 \ucc28\ud2b8\uc5d0\uc11c 1\uc704\ub97c \uae30\ub85d\ud558\uace0 \uce90\ub098\ub2e4 \ucc28\ud2b8\uc5d0\uc11c\ub294 54\uc704\ub97c \ud588\ub2e4. \ub9c8\uc9c0\ub9c9 \uc2f1\uae00\uc778 'Mrs. Officer'\uc740 R&B\uac00\uc218 \ubc14\ube44 \ubc1c\ub80c\ud2f0\ub178\uac00 \ucc38\uc5ec\ud588\uc73c\uba70, \ub7a9\ucc28\ud2b8\uc5d0\uc11c 2\uc704\ub97c \uae30\ub85d\ud588\ub2e4. \uc774\uc678\uc5d0\ub3c4 \ud2b8\ub799 \uc911\uc758 \ud558\ub098\uc774\uba70 \uc81c\uc774 \uc9c0\uac00 \ucc38\uc5ec\ud55c 'Mr. Carter'\uc740 \ub7a9\ucc28\ud2b8\uc5d0\uc11c 13\uc704\ub97c \uac70\ub450\uc5c8\uace0 \uc774\ub4ec\ud574 \uadf8\ub798\ubbf8 \uc5b4\uc6cc\ub4dc\uc5d0\uc11c '\ubca0\uc2a4\ud2b8 \ub4c0\uc624 \ud799\ud569 \uc0c1'\uc744 \ubc1b\uc558\ub2e4.", "paragraph_answer": "2008\ub144 \uadf8\uc758 \uc194\ub85c\uc568\ubc94 [Tha Carter 3]\uc744 \ubc1c\ub9e4, \uccab \ubc88\uc9f8 \uc2f1\uae00 \u3008Lollipop\u3009\uc740 \ubc1c\ub9e4\ub418\uc790\ub9c8\uc790 \uc628\ub77c\uc778 \ub2e4\uc6b4\ub85c\ub4dc 1,700,000 \uac74\uc744 \uae30\ub85d\ud558\ub294 \ud3ed\ubc1c\uc801\uc778 \ud788\ud2b8\ub97c \uae30\ub85d\ud588\ub2e4. \ubbf8\uad6d\uc5d0\uc11c\ub294 300\ub9cc \uc7a5 \uac00\uae4c\uc774 \ud314\uc544\ub4e4\uc600\uc73c\uba70, \uadf8 \ubc16\uc5d0\ub3c4 \ud788\ud2b8 \uc2f1\uae00\ub4e4\uc778 'Got Money'\ub294 \ube4c\ubcf4\ub4dc \ucc28\ud2b8 13\uc704 \ub97c \ucc28\uc9c0\ud588\uace0, \uccab\uc2f1\uae00\uc778 'Lollipop'\uc740 \uc804 \uc138\uacc4 \uac01\uad6d\uc5d0\uc11c 1\uc704\ub97c \ucc28\uc9c0\ud588\ub2e4. \ub450 \ubc88\uc9f8 \uc2f1\uae00\uc778 'A Milli'\ub294 \ubbf8\uad6d \ubaa8\ub4e0 \ud799\ud569 \ucc28\ud2b8\uc5d0\uc11c 1\uc704\ub97c \uae30\ub85d\ud558\uace0 \uce90\ub098\ub2e4 \ucc28\ud2b8\uc5d0\uc11c\ub294 54\uc704\ub97c \ud588\ub2e4. \ub9c8\uc9c0\ub9c9 \uc2f1\uae00\uc778 'Mrs.Officer'\uc740 R&B\uac00\uc218 \ubc14\ube44 \ubc1c\ub80c\ud2f0\ub178\uac00 \ucc38\uc5ec\ud588\uc73c\uba70, \ub7a9\ucc28\ud2b8\uc5d0\uc11c 2\uc704\ub97c \uae30\ub85d\ud588\ub2e4. \uc774\uc678\uc5d0\ub3c4 \ud2b8\ub799 \uc911\uc758 \ud558\ub098\uc774\uba70 \uc81c\uc774 \uc9c0\uac00 \ucc38\uc5ec\ud55c 'Mr. Carter'\uc740 \ub7a9\ucc28\ud2b8\uc5d0\uc11c 13\uc704\ub97c \uac70\ub450\uc5c8\uace0 \uc774\ub4ec\ud574 \uadf8\ub798\ubbf8 \uc5b4\uc6cc\ub4dc\uc5d0\uc11c '\ubca0\uc2a4\ud2b8 \ub4c0\uc624 \ud799\ud569 \uc0c1'\uc744 \ubc1b\uc558\ub2e4.", "sentence_answer": "'Got Money'\ub294 \ube4c\ubcf4\ub4dc \ucc28\ud2b8 13\uc704 \ub97c \ucc28\uc9c0\ud588\uace0, \uccab\uc2f1\uae00\uc778 'Lollipop'\uc740 \uc804 \uc138\uacc4 \uac01\uad6d\uc5d0\uc11c 1\uc704\ub97c \ucc28\uc9c0\ud588\ub2e4.", "paragraph_id": "6602591-0-2", "paragraph_question": "question: \ub9b4 \uc6e8\uc778\uc758 2008\ub144 \uc194\ub85c\uc568\ubc94 \uc218\ub85d\uace1\uc778 'Got Money'\uc758 \ube4c\ubcf4\ub4dc \ucc28\ud2b8 \uc21c\uc704\ub294?, context: 2008\ub144 \uadf8\uc758 \uc194\ub85c\uc568\ubc94 [Tha Carter 3]\uc744 \ubc1c\ub9e4, \uccab \ubc88\uc9f8 \uc2f1\uae00 \u3008Lollipop\u3009\uc740 \ubc1c\ub9e4\ub418\uc790\ub9c8\uc790 \uc628\ub77c\uc778 \ub2e4\uc6b4\ub85c\ub4dc 1,700,000 \uac74\uc744 \uae30\ub85d\ud558\ub294 \ud3ed\ubc1c\uc801\uc778 \ud788\ud2b8\ub97c \uae30\ub85d\ud588\ub2e4. \ubbf8\uad6d\uc5d0\uc11c\ub294 300\ub9cc \uc7a5 \uac00\uae4c\uc774 \ud314\uc544\ub4e4\uc600\uc73c\uba70, \uadf8 \ubc16\uc5d0\ub3c4 \ud788\ud2b8 \uc2f1\uae00\ub4e4\uc778 'Got Money'\ub294 \ube4c\ubcf4\ub4dc \ucc28\ud2b8 13\uc704\ub97c \ucc28\uc9c0\ud588\uace0, \uccab\uc2f1\uae00\uc778 'Lollipop'\uc740 \uc804 \uc138\uacc4 \uac01\uad6d\uc5d0\uc11c 1\uc704\ub97c \ucc28\uc9c0\ud588\ub2e4. \ub450 \ubc88\uc9f8 \uc2f1\uae00\uc778 'A Milli'\ub294 \ubbf8\uad6d \ubaa8\ub4e0 \ud799\ud569 \ucc28\ud2b8\uc5d0\uc11c 1\uc704\ub97c \uae30\ub85d\ud558\uace0 \uce90\ub098\ub2e4 \ucc28\ud2b8\uc5d0\uc11c\ub294 54\uc704\ub97c \ud588\ub2e4. \ub9c8\uc9c0\ub9c9 \uc2f1\uae00\uc778 'Mrs.Officer'\uc740 R&B\uac00\uc218 \ubc14\ube44 \ubc1c\ub80c\ud2f0\ub178\uac00 \ucc38\uc5ec\ud588\uc73c\uba70, \ub7a9\ucc28\ud2b8\uc5d0\uc11c 2\uc704\ub97c \uae30\ub85d\ud588\ub2e4. \uc774\uc678\uc5d0\ub3c4 \ud2b8\ub799 \uc911\uc758 \ud558\ub098\uc774\uba70 \uc81c\uc774 \uc9c0\uac00 \ucc38\uc5ec\ud55c 'Mr. Carter'\uc740 \ub7a9\ucc28\ud2b8\uc5d0\uc11c 13\uc704\ub97c \uac70\ub450\uc5c8\uace0 \uc774\ub4ec\ud574 \uadf8\ub798\ubbf8 \uc5b4\uc6cc\ub4dc\uc5d0\uc11c '\ubca0\uc2a4\ud2b8 \ub4c0\uc624 \ud799\ud569 \uc0c1'\uc744 \ubc1b\uc558\ub2e4."} {"question": "\ub450 \ubc88\uc9f8 \uc608\uc220\ub2e8 \ubc29\ubb38\uc744 \ud55c \uccad\ubd09\uc545\ub2e8\uc758 \uac00\uc218\ub294 \ub204\uad6c\uc785\ub2c8\uae4c?", "paragraph": "\uccad\ubd09\uc545\ub2e8\uc758 \uac00\uc218\ub4e4\uc778 \uae40\uc8fc\ud5a5, \uc1a1\uc601, \ub9ac\uc218\uacbd, \uae40\uccad, \uae40\uc131\uc2ec, \ub85c\uacbd\ubbf8, \uad8c\ud5a5\ub9bc \uadf8\ub9ac\uace0 \uccad\ubd09\uc545\ub2e8\uacfc \ubaa8\ub780\ubd09\uc545\ub2e8\uc744 \uc624\uac00\uba70 \ub9f9\ub82c\ud55c \ud65c\uc57d\uc744 \ubcf4\uc5ec\uc8fc\uace0 \uc788\ub294 \uc911\uacac\uac00\uc218 \uae40\uc625\uc8fc\ub294 \uc5ed\uc2dc \uc9c4\ub2ec\ub798\ube5b \ud55c\ubcf5\uc744 \uc785\uace0 \ub4f1\uc7a5\ud558\uc600\ub294\ub370 \ubd81\uce21 \ub178\ub798 \u3008\ubc18\uac11\uc2b5\ub2c8\ub2e4\u3009\ub85c \uacf5\uc5f0\uc744 \uc2dc\uc791\ud558\uc5ec \ubd81\uce21 \uc0c8\ud574 \uacf5\uc5f0\uc5d0\uc11c \ub298 \ube60\uc9d0\uc5c6\uc774 \ubd80\ub974\ub294 \u3008\uc124\ub208\uc544 \ub0b4\ub824\ub77c\u3009\ub85c \uc774\uc5b4\uc84c\ub2e4. \ub610\ud55c \uae40\uc625\uc8fc\uc640 \uc1a1\uc601\uc740 \ub0a8\uce21 \ub178\ub798\uc778 \u3008J\uc5d0\uac8c\u3009\ub97c \ud568\uaed8 \ubd88\ub800\uc73c\uba70, \uae40\uc625\uc8fc\ub294 \u3008\uc5ec\uc815\u3009\uacfc \u3008\uc0c8\ubcc4\u3009\uc744 \ub3c5\ucc3d\uc73c\ub85c \ubd88\ub7ec\uc11c \uc65c \uadf8\ub140\uac00 \uccad\ubd09\uc545\ub2e8\uacfc \ubaa8\ub780\ubd09\uc545\ub2e8\uc744 \ub204\ube44\uba70 \ud65c\uc57d\ud558\uace0 \uc788\ub294\uc9c0 \ub2e8\uc801\uc73c\ub85c \uc99d\uba85\ud574\uc8fc\uc5c8\ub2e4. \uccad\ubd09\uc545\ub2e8\uc758 \uac00\uc218\uc778 \uae40\uc8fc\ud5a5\uc740 \uc774\ubc88\uc774 \ub450 \ubc88\uc9f8 \uc608\uc220\ub2e8 \ubc29\ubb38\uc73c\ub85c \uc9c0\ub09c 2000\ub144 5\uc6d4 \ud3c9\uc591\ud559\uc0dd\uc18c\ub144\uc608\uc220\ub2e8\uc758 \uc77c\uc6d0\uc73c\ub85c \uc11c\uc6b8 \uc608\uc220\uc758\uc804\ub2f9 \uc624\ud398\ub77c\uadf9\uc7a5 \uacf5\uc5f0\uc5d0\uc11c \ub178\ub798\ub97c \ubd80\ub978 \uc801\uc774 \uc788\uc5c8\ub294\ub370, \ub2f9\uc2dc \uae40\uc8fc\ud5a5\uc758 \ub098\uc774\ub294 8\uc0b4\uc774\uc5c8\ub2e4.", "answer": "\uae40\uc8fc\ud5a5\uc740", "sentence": "\uccad\ubd09\uc545\ub2e8\uc758 \uac00\uc218\uc778 \uae40\uc8fc\ud5a5\uc740 \uc774\ubc88\uc774 \ub450 \ubc88\uc9f8 \uc608\uc220\ub2e8 \ubc29\ubb38\uc73c\ub85c \uc9c0\ub09c 2000\ub144 5\uc6d4 \ud3c9\uc591\ud559\uc0dd\uc18c\ub144\uc608\uc220\ub2e8\uc758 \uc77c\uc6d0\uc73c\ub85c \uc11c\uc6b8 \uc608\uc220\uc758\uc804\ub2f9 \uc624\ud398\ub77c\uadf9\uc7a5 \uacf5\uc5f0\uc5d0\uc11c \ub178\ub798\ub97c \ubd80\ub978 \uc801\uc774 \uc788\uc5c8\ub294\ub370, \ub2f9\uc2dc \uae40\uc8fc\ud5a5\uc758 \ub098\uc774\ub294 8\uc0b4\uc774\uc5c8\ub2e4.", "paragraph_sentence": "\uccad\ubd09\uc545\ub2e8\uc758 \uac00\uc218\ub4e4\uc778 \uae40\uc8fc\ud5a5, \uc1a1\uc601, \ub9ac\uc218\uacbd, \uae40\uccad, \uae40\uc131\uc2ec, \ub85c\uacbd\ubbf8, \uad8c\ud5a5\ub9bc \uadf8\ub9ac\uace0 \uccad\ubd09\uc545\ub2e8\uacfc \ubaa8\ub780\ubd09\uc545\ub2e8\uc744 \uc624\uac00\uba70 \ub9f9\ub82c\ud55c \ud65c\uc57d\uc744 \ubcf4\uc5ec\uc8fc\uace0 \uc788\ub294 \uc911\uacac\uac00\uc218 \uae40\uc625\uc8fc\ub294 \uc5ed\uc2dc \uc9c4\ub2ec\ub798\ube5b \ud55c\ubcf5\uc744 \uc785\uace0 \ub4f1\uc7a5\ud558\uc600\ub294\ub370 \ubd81\uce21 \ub178\ub798 \u3008\ubc18\uac11\uc2b5\ub2c8\ub2e4\u3009\ub85c \uacf5\uc5f0\uc744 \uc2dc\uc791\ud558\uc5ec \ubd81\uce21 \uc0c8\ud574 \uacf5\uc5f0\uc5d0\uc11c \ub298 \ube60\uc9d0\uc5c6\uc774 \ubd80\ub974\ub294 \u3008\uc124\ub208\uc544 \ub0b4\ub824\ub77c\u3009\ub85c \uc774\uc5b4\uc84c\ub2e4. \ub610\ud55c \uae40\uc625\uc8fc\uc640 \uc1a1\uc601\uc740 \ub0a8\uce21 \ub178\ub798\uc778 \u3008J\uc5d0\uac8c\u3009\ub97c \ud568\uaed8 \ubd88\ub800\uc73c\uba70, \uae40\uc625\uc8fc\ub294 \u3008\uc5ec\uc815\u3009\uacfc \u3008\uc0c8\ubcc4\u3009\uc744 \ub3c5\ucc3d\uc73c\ub85c \ubd88\ub7ec\uc11c \uc65c \uadf8\ub140\uac00 \uccad\ubd09\uc545\ub2e8\uacfc \ubaa8\ub780\ubd09\uc545\ub2e8\uc744 \ub204\ube44\uba70 \ud65c\uc57d\ud558\uace0 \uc788\ub294\uc9c0 \ub2e8\uc801\uc73c\ub85c \uc99d\uba85\ud574\uc8fc\uc5c8\ub2e4. \uccad\ubd09\uc545\ub2e8\uc758 \uac00\uc218\uc778 \uae40\uc8fc\ud5a5\uc740 \uc774\ubc88\uc774 \ub450 \ubc88\uc9f8 \uc608\uc220\ub2e8 \ubc29\ubb38\uc73c\ub85c \uc9c0\ub09c 2000\ub144 5\uc6d4 \ud3c9\uc591\ud559\uc0dd\uc18c\ub144\uc608\uc220\ub2e8\uc758 \uc77c\uc6d0\uc73c\ub85c \uc11c\uc6b8 \uc608\uc220\uc758\uc804\ub2f9 \uc624\ud398\ub77c\uadf9\uc7a5 \uacf5\uc5f0\uc5d0\uc11c \ub178\ub798\ub97c \ubd80\ub978 \uc801\uc774 \uc788\uc5c8\ub294\ub370, \ub2f9\uc2dc \uae40\uc8fc\ud5a5\uc758 \ub098\uc774\ub294 8\uc0b4\uc774\uc5c8\ub2e4. ", "paragraph_answer": "\uccad\ubd09\uc545\ub2e8\uc758 \uac00\uc218\ub4e4\uc778 \uae40\uc8fc\ud5a5, \uc1a1\uc601, \ub9ac\uc218\uacbd, \uae40\uccad, \uae40\uc131\uc2ec, \ub85c\uacbd\ubbf8, \uad8c\ud5a5\ub9bc \uadf8\ub9ac\uace0 \uccad\ubd09\uc545\ub2e8\uacfc \ubaa8\ub780\ubd09\uc545\ub2e8\uc744 \uc624\uac00\uba70 \ub9f9\ub82c\ud55c \ud65c\uc57d\uc744 \ubcf4\uc5ec\uc8fc\uace0 \uc788\ub294 \uc911\uacac\uac00\uc218 \uae40\uc625\uc8fc\ub294 \uc5ed\uc2dc \uc9c4\ub2ec\ub798\ube5b \ud55c\ubcf5\uc744 \uc785\uace0 \ub4f1\uc7a5\ud558\uc600\ub294\ub370 \ubd81\uce21 \ub178\ub798 \u3008\ubc18\uac11\uc2b5\ub2c8\ub2e4\u3009\ub85c \uacf5\uc5f0\uc744 \uc2dc\uc791\ud558\uc5ec \ubd81\uce21 \uc0c8\ud574 \uacf5\uc5f0\uc5d0\uc11c \ub298 \ube60\uc9d0\uc5c6\uc774 \ubd80\ub974\ub294 \u3008\uc124\ub208\uc544 \ub0b4\ub824\ub77c\u3009\ub85c \uc774\uc5b4\uc84c\ub2e4. \ub610\ud55c \uae40\uc625\uc8fc\uc640 \uc1a1\uc601\uc740 \ub0a8\uce21 \ub178\ub798\uc778 \u3008J\uc5d0\uac8c\u3009\ub97c \ud568\uaed8 \ubd88\ub800\uc73c\uba70, \uae40\uc625\uc8fc\ub294 \u3008\uc5ec\uc815\u3009\uacfc \u3008\uc0c8\ubcc4\u3009\uc744 \ub3c5\ucc3d\uc73c\ub85c \ubd88\ub7ec\uc11c \uc65c \uadf8\ub140\uac00 \uccad\ubd09\uc545\ub2e8\uacfc \ubaa8\ub780\ubd09\uc545\ub2e8\uc744 \ub204\ube44\uba70 \ud65c\uc57d\ud558\uace0 \uc788\ub294\uc9c0 \ub2e8\uc801\uc73c\ub85c \uc99d\uba85\ud574\uc8fc\uc5c8\ub2e4. \uccad\ubd09\uc545\ub2e8\uc758 \uac00\uc218\uc778 \uae40\uc8fc\ud5a5\uc740 \uc774\ubc88\uc774 \ub450 \ubc88\uc9f8 \uc608\uc220\ub2e8 \ubc29\ubb38\uc73c\ub85c \uc9c0\ub09c 2000\ub144 5\uc6d4 \ud3c9\uc591\ud559\uc0dd\uc18c\ub144\uc608\uc220\ub2e8\uc758 \uc77c\uc6d0\uc73c\ub85c \uc11c\uc6b8 \uc608\uc220\uc758\uc804\ub2f9 \uc624\ud398\ub77c\uadf9\uc7a5 \uacf5\uc5f0\uc5d0\uc11c \ub178\ub798\ub97c \ubd80\ub978 \uc801\uc774 \uc788\uc5c8\ub294\ub370, \ub2f9\uc2dc \uae40\uc8fc\ud5a5\uc758 \ub098\uc774\ub294 8\uc0b4\uc774\uc5c8\ub2e4.", "sentence_answer": "\uccad\ubd09\uc545\ub2e8\uc758 \uac00\uc218\uc778 \uae40\uc8fc\ud5a5\uc740 \uc774\ubc88\uc774 \ub450 \ubc88\uc9f8 \uc608\uc220\ub2e8 \ubc29\ubb38\uc73c\ub85c \uc9c0\ub09c 2000\ub144 5\uc6d4 \ud3c9\uc591\ud559\uc0dd\uc18c\ub144\uc608\uc220\ub2e8\uc758 \uc77c\uc6d0\uc73c\ub85c \uc11c\uc6b8 \uc608\uc220\uc758\uc804\ub2f9 \uc624\ud398\ub77c\uadf9\uc7a5 \uacf5\uc5f0\uc5d0\uc11c \ub178\ub798\ub97c \ubd80\ub978 \uc801\uc774 \uc788\uc5c8\ub294\ub370, \ub2f9\uc2dc \uae40\uc8fc\ud5a5\uc758 \ub098\uc774\ub294 8\uc0b4\uc774\uc5c8\ub2e4.", "paragraph_id": "6136088-1-1", "paragraph_question": "question: \ub450 \ubc88\uc9f8 \uc608\uc220\ub2e8 \ubc29\ubb38\uc744 \ud55c \uccad\ubd09\uc545\ub2e8\uc758 \uac00\uc218\ub294 \ub204\uad6c\uc785\ub2c8\uae4c?, context: \uccad\ubd09\uc545\ub2e8\uc758 \uac00\uc218\ub4e4\uc778 \uae40\uc8fc\ud5a5, \uc1a1\uc601, \ub9ac\uc218\uacbd, \uae40\uccad, \uae40\uc131\uc2ec, \ub85c\uacbd\ubbf8, \uad8c\ud5a5\ub9bc \uadf8\ub9ac\uace0 \uccad\ubd09\uc545\ub2e8\uacfc \ubaa8\ub780\ubd09\uc545\ub2e8\uc744 \uc624\uac00\uba70 \ub9f9\ub82c\ud55c \ud65c\uc57d\uc744 \ubcf4\uc5ec\uc8fc\uace0 \uc788\ub294 \uc911\uacac\uac00\uc218 \uae40\uc625\uc8fc\ub294 \uc5ed\uc2dc \uc9c4\ub2ec\ub798\ube5b \ud55c\ubcf5\uc744 \uc785\uace0 \ub4f1\uc7a5\ud558\uc600\ub294\ub370 \ubd81\uce21 \ub178\ub798 \u3008\ubc18\uac11\uc2b5\ub2c8\ub2e4\u3009\ub85c \uacf5\uc5f0\uc744 \uc2dc\uc791\ud558\uc5ec \ubd81\uce21 \uc0c8\ud574 \uacf5\uc5f0\uc5d0\uc11c \ub298 \ube60\uc9d0\uc5c6\uc774 \ubd80\ub974\ub294 \u3008\uc124\ub208\uc544 \ub0b4\ub824\ub77c\u3009\ub85c \uc774\uc5b4\uc84c\ub2e4. \ub610\ud55c \uae40\uc625\uc8fc\uc640 \uc1a1\uc601\uc740 \ub0a8\uce21 \ub178\ub798\uc778 \u3008J\uc5d0\uac8c\u3009\ub97c \ud568\uaed8 \ubd88\ub800\uc73c\uba70, \uae40\uc625\uc8fc\ub294 \u3008\uc5ec\uc815\u3009\uacfc \u3008\uc0c8\ubcc4\u3009\uc744 \ub3c5\ucc3d\uc73c\ub85c \ubd88\ub7ec\uc11c \uc65c \uadf8\ub140\uac00 \uccad\ubd09\uc545\ub2e8\uacfc \ubaa8\ub780\ubd09\uc545\ub2e8\uc744 \ub204\ube44\uba70 \ud65c\uc57d\ud558\uace0 \uc788\ub294\uc9c0 \ub2e8\uc801\uc73c\ub85c \uc99d\uba85\ud574\uc8fc\uc5c8\ub2e4. \uccad\ubd09\uc545\ub2e8\uc758 \uac00\uc218\uc778 \uae40\uc8fc\ud5a5\uc740 \uc774\ubc88\uc774 \ub450 \ubc88\uc9f8 \uc608\uc220\ub2e8 \ubc29\ubb38\uc73c\ub85c \uc9c0\ub09c 2000\ub144 5\uc6d4 \ud3c9\uc591\ud559\uc0dd\uc18c\ub144\uc608\uc220\ub2e8\uc758 \uc77c\uc6d0\uc73c\ub85c \uc11c\uc6b8 \uc608\uc220\uc758\uc804\ub2f9 \uc624\ud398\ub77c\uadf9\uc7a5 \uacf5\uc5f0\uc5d0\uc11c \ub178\ub798\ub97c \ubd80\ub978 \uc801\uc774 \uc788\uc5c8\ub294\ub370, \ub2f9\uc2dc \uae40\uc8fc\ud5a5\uc758 \ub098\uc774\ub294 8\uc0b4\uc774\uc5c8\ub2e4."} {"question": "\uae40\ub300\uc911\uc774 1960\ub144 \uc81c5\ub300 \ucd1d\uc120\uc5d0\uc11c \ucd9c\ub9c8\ud55c \uac15\uc6d0\ub3c4\uc758 \uc9c0\uc5ed \uc774\ub984\uc740?", "paragraph": "1960\ub144\uc5d0 \uce58\ub904\uc9c4 \uc81c 5\ub300 \ucd1d\uc120 \uc778\uc81c\uc9c0\uad6c\uc5d0\uc120 \uc790\uc720\ub2f9\uc758 \uc804\ud615\uc0b0 \ud6c4\ubcf4\uc5d0 \ud328\ud588\ub2e4. \uc774\ud6c4 \ucd1d\uc120\uc5d0\uc11c \uc2b9\ub9ac\ud558\uc5ec \uc815\uad8c\uc744 \uc7a1\uc740 \ubbfc\uc8fc\ub2f9 \uce21\uc5d0\uc11c \uacf5\ubbfc\uad8c \uc81c\ud55c \ub300\uc0c1\uc790\ub85c \uc804\ud615\uc0b0 \uc758\uc6d0\uc744 \uc120\uc815\ud558\uc5ec \uc758\uc6d0 \uc790\uaca9\uc744 \ubc15\ud0c8\ud558\uc600\uace0, \uc774\ub4ec\ud574 1961\ub144 5\uc6d4 14\uc77c\uc5d0 \uac15\uc6d0\ub3c4 \uc778\uc81c\uc5d0\uc120 \uc7ac\ubcf4\uad90\uc120\uac70\uac00 \uce58\ub904\uc9c0\uac8c \ub41c\ub2e4. \uae40\ub300\uc911\uc740 \uc7ac\ubcf4\uad90\uc120\uac70\uc5d0 \ucd9c\ub9c8\ud558\uc5ec \ubbfc\uc758\uc6d0\uc5d0 \ub2f9\uc120\ub418\uc9c0\ub9cc \uc774\ud2c0 \ub4a4\uc5d0 5\u00b716 \ucfe0\ub370\ud0c0\uac00 \uc77c\uc5b4\ub098 \uad6d\ud68c\uac00 \ud574\uc0b0\ub418\ub294 \ubc14\ub78c\uc5d0 \uacb0\uad6d \uc758\uc6d0 \ud65c\ub3d9\uc740 \ud558\uc9c0 \ubabb\ud588\ub2e4. 5\u00b716 \ucfe0\ub370\ud0c0 \uc774\ud6c4 \uc774\ucca0\uc2b9 \ub4f1 \uc77c\ubd80\uac00 \uc815\uce58\uc815\ud654\ubc95\uc5d0 \ubb36\uc5ec\uc788\uc744 \ub54c \ubbfc\uc8fc\ub2f9 \uc120\uc804\ubd80\uc7a5, \ub2f9 \ub300\ubcc0\uc778 \ub4f1\uc744 \uc5ed\uc784\ud588\ub2e4. \uc774\ud6c4 1951\ub144 \ubd80\uc0b0\uc5d0\uc11c \ub9cc\ub09c\uc801\uc774 \uc788\ub358 \uc774\ud76c\ud638\uc640 \ub2e4\uc2dc \ub9cc\ub098 \uc7ac\ud63c\ud588\uace0, 1963\ub144\uc5d0 \ubbfc\uc8fc\ub2f9 \uc18c\uc18d\uc73c\ub85c \uace0\ud5a5 \ubaa9\ud3ec\uc5d0\uc11c \uc81c6\ub300 \uad6d\ud68c\uc758\uc6d0\uc73c\ub85c \ub2f9\uc120\ub418\uba74\uc11c \uc7ac\uc120 \uad6d\ud68c\uc758\uc6d0\uc774 \ub41c\ub2e4.", "answer": "\uc778\uc81c", "sentence": "1960\ub144\uc5d0 \uce58\ub904\uc9c4 \uc81c 5\ub300 \ucd1d\uc120 \uc778\uc81c \uc9c0\uad6c\uc5d0\uc120 \uc790\uc720\ub2f9\uc758 \uc804\ud615\uc0b0 \ud6c4\ubcf4\uc5d0 \ud328\ud588\ub2e4.", "paragraph_sentence": " 1960\ub144\uc5d0 \uce58\ub904\uc9c4 \uc81c 5\ub300 \ucd1d\uc120 \uc778\uc81c \uc9c0\uad6c\uc5d0\uc120 \uc790\uc720\ub2f9\uc758 \uc804\ud615\uc0b0 \ud6c4\ubcf4\uc5d0 \ud328\ud588\ub2e4. \uc774\ud6c4 \ucd1d\uc120\uc5d0\uc11c \uc2b9\ub9ac\ud558\uc5ec \uc815\uad8c\uc744 \uc7a1\uc740 \ubbfc\uc8fc\ub2f9 \uce21\uc5d0\uc11c \uacf5\ubbfc\uad8c \uc81c\ud55c \ub300\uc0c1\uc790\ub85c \uc804\ud615\uc0b0 \uc758\uc6d0\uc744 \uc120\uc815\ud558\uc5ec \uc758\uc6d0 \uc790\uaca9\uc744 \ubc15\ud0c8\ud558\uc600\uace0, \uc774\ub4ec\ud574 1961\ub144 5\uc6d4 14\uc77c\uc5d0 \uac15\uc6d0\ub3c4 \uc778\uc81c\uc5d0\uc120 \uc7ac\ubcf4\uad90\uc120\uac70\uac00 \uce58\ub904\uc9c0\uac8c \ub41c\ub2e4. \uae40\ub300\uc911\uc740 \uc7ac\ubcf4\uad90\uc120\uac70\uc5d0 \ucd9c\ub9c8\ud558\uc5ec \ubbfc\uc758\uc6d0\uc5d0 \ub2f9\uc120\ub418\uc9c0\ub9cc \uc774\ud2c0 \ub4a4\uc5d0 5\u00b716 \ucfe0\ub370\ud0c0\uac00 \uc77c\uc5b4\ub098 \uad6d\ud68c\uac00 \ud574\uc0b0\ub418\ub294 \ubc14\ub78c\uc5d0 \uacb0\uad6d \uc758\uc6d0 \ud65c\ub3d9\uc740 \ud558\uc9c0 \ubabb\ud588\ub2e4. 5\u00b716 \ucfe0\ub370\ud0c0 \uc774\ud6c4 \uc774\ucca0\uc2b9 \ub4f1 \uc77c\ubd80\uac00 \uc815\uce58\uc815\ud654\ubc95\uc5d0 \ubb36\uc5ec\uc788\uc744 \ub54c \ubbfc\uc8fc\ub2f9 \uc120\uc804\ubd80\uc7a5, \ub2f9 \ub300\ubcc0\uc778 \ub4f1\uc744 \uc5ed\uc784\ud588\ub2e4. \uc774\ud6c4 1951\ub144 \ubd80\uc0b0\uc5d0\uc11c \ub9cc\ub09c\uc801\uc774 \uc788\ub358 \uc774\ud76c\ud638\uc640 \ub2e4\uc2dc \ub9cc\ub098 \uc7ac\ud63c\ud588\uace0, 1963\ub144\uc5d0 \ubbfc\uc8fc\ub2f9 \uc18c\uc18d\uc73c\ub85c \uace0\ud5a5 \ubaa9\ud3ec\uc5d0\uc11c \uc81c6\ub300 \uad6d\ud68c\uc758\uc6d0\uc73c\ub85c \ub2f9\uc120\ub418\uba74\uc11c \uc7ac\uc120 \uad6d\ud68c\uc758\uc6d0\uc774 \ub41c\ub2e4.", "paragraph_answer": "1960\ub144\uc5d0 \uce58\ub904\uc9c4 \uc81c 5\ub300 \ucd1d\uc120 \uc778\uc81c \uc9c0\uad6c\uc5d0\uc120 \uc790\uc720\ub2f9\uc758 \uc804\ud615\uc0b0 \ud6c4\ubcf4\uc5d0 \ud328\ud588\ub2e4. \uc774\ud6c4 \ucd1d\uc120\uc5d0\uc11c \uc2b9\ub9ac\ud558\uc5ec \uc815\uad8c\uc744 \uc7a1\uc740 \ubbfc\uc8fc\ub2f9 \uce21\uc5d0\uc11c \uacf5\ubbfc\uad8c \uc81c\ud55c \ub300\uc0c1\uc790\ub85c \uc804\ud615\uc0b0 \uc758\uc6d0\uc744 \uc120\uc815\ud558\uc5ec \uc758\uc6d0 \uc790\uaca9\uc744 \ubc15\ud0c8\ud558\uc600\uace0, \uc774\ub4ec\ud574 1961\ub144 5\uc6d4 14\uc77c\uc5d0 \uac15\uc6d0\ub3c4 \uc778\uc81c\uc5d0\uc120 \uc7ac\ubcf4\uad90\uc120\uac70\uac00 \uce58\ub904\uc9c0\uac8c \ub41c\ub2e4. \uae40\ub300\uc911\uc740 \uc7ac\ubcf4\uad90\uc120\uac70\uc5d0 \ucd9c\ub9c8\ud558\uc5ec \ubbfc\uc758\uc6d0\uc5d0 \ub2f9\uc120\ub418\uc9c0\ub9cc \uc774\ud2c0 \ub4a4\uc5d0 5\u00b716 \ucfe0\ub370\ud0c0\uac00 \uc77c\uc5b4\ub098 \uad6d\ud68c\uac00 \ud574\uc0b0\ub418\ub294 \ubc14\ub78c\uc5d0 \uacb0\uad6d \uc758\uc6d0 \ud65c\ub3d9\uc740 \ud558\uc9c0 \ubabb\ud588\ub2e4. 5\u00b716 \ucfe0\ub370\ud0c0 \uc774\ud6c4 \uc774\ucca0\uc2b9 \ub4f1 \uc77c\ubd80\uac00 \uc815\uce58\uc815\ud654\ubc95\uc5d0 \ubb36\uc5ec\uc788\uc744 \ub54c \ubbfc\uc8fc\ub2f9 \uc120\uc804\ubd80\uc7a5, \ub2f9 \ub300\ubcc0\uc778 \ub4f1\uc744 \uc5ed\uc784\ud588\ub2e4. \uc774\ud6c4 1951\ub144 \ubd80\uc0b0\uc5d0\uc11c \ub9cc\ub09c\uc801\uc774 \uc788\ub358 \uc774\ud76c\ud638\uc640 \ub2e4\uc2dc \ub9cc\ub098 \uc7ac\ud63c\ud588\uace0, 1963\ub144\uc5d0 \ubbfc\uc8fc\ub2f9 \uc18c\uc18d\uc73c\ub85c \uace0\ud5a5 \ubaa9\ud3ec\uc5d0\uc11c \uc81c6\ub300 \uad6d\ud68c\uc758\uc6d0\uc73c\ub85c \ub2f9\uc120\ub418\uba74\uc11c \uc7ac\uc120 \uad6d\ud68c\uc758\uc6d0\uc774 \ub41c\ub2e4.", "sentence_answer": "1960\ub144\uc5d0 \uce58\ub904\uc9c4 \uc81c 5\ub300 \ucd1d\uc120 \uc778\uc81c \uc9c0\uad6c\uc5d0\uc120 \uc790\uc720\ub2f9\uc758 \uc804\ud615\uc0b0 \ud6c4\ubcf4\uc5d0 \ud328\ud588\ub2e4.", "paragraph_id": "6488203-10-0", "paragraph_question": "question: \uae40\ub300\uc911\uc774 1960\ub144 \uc81c5\ub300 \ucd1d\uc120\uc5d0\uc11c \ucd9c\ub9c8\ud55c \uac15\uc6d0\ub3c4\uc758 \uc9c0\uc5ed \uc774\ub984\uc740?, context: 1960\ub144\uc5d0 \uce58\ub904\uc9c4 \uc81c 5\ub300 \ucd1d\uc120 \uc778\uc81c\uc9c0\uad6c\uc5d0\uc120 \uc790\uc720\ub2f9\uc758 \uc804\ud615\uc0b0 \ud6c4\ubcf4\uc5d0 \ud328\ud588\ub2e4. \uc774\ud6c4 \ucd1d\uc120\uc5d0\uc11c \uc2b9\ub9ac\ud558\uc5ec \uc815\uad8c\uc744 \uc7a1\uc740 \ubbfc\uc8fc\ub2f9 \uce21\uc5d0\uc11c \uacf5\ubbfc\uad8c \uc81c\ud55c \ub300\uc0c1\uc790\ub85c \uc804\ud615\uc0b0 \uc758\uc6d0\uc744 \uc120\uc815\ud558\uc5ec \uc758\uc6d0 \uc790\uaca9\uc744 \ubc15\ud0c8\ud558\uc600\uace0, \uc774\ub4ec\ud574 1961\ub144 5\uc6d4 14\uc77c\uc5d0 \uac15\uc6d0\ub3c4 \uc778\uc81c\uc5d0\uc120 \uc7ac\ubcf4\uad90\uc120\uac70\uac00 \uce58\ub904\uc9c0\uac8c \ub41c\ub2e4. \uae40\ub300\uc911\uc740 \uc7ac\ubcf4\uad90\uc120\uac70\uc5d0 \ucd9c\ub9c8\ud558\uc5ec \ubbfc\uc758\uc6d0\uc5d0 \ub2f9\uc120\ub418\uc9c0\ub9cc \uc774\ud2c0 \ub4a4\uc5d0 5\u00b716 \ucfe0\ub370\ud0c0\uac00 \uc77c\uc5b4\ub098 \uad6d\ud68c\uac00 \ud574\uc0b0\ub418\ub294 \ubc14\ub78c\uc5d0 \uacb0\uad6d \uc758\uc6d0 \ud65c\ub3d9\uc740 \ud558\uc9c0 \ubabb\ud588\ub2e4. 5\u00b716 \ucfe0\ub370\ud0c0 \uc774\ud6c4 \uc774\ucca0\uc2b9 \ub4f1 \uc77c\ubd80\uac00 \uc815\uce58\uc815\ud654\ubc95\uc5d0 \ubb36\uc5ec\uc788\uc744 \ub54c \ubbfc\uc8fc\ub2f9 \uc120\uc804\ubd80\uc7a5, \ub2f9 \ub300\ubcc0\uc778 \ub4f1\uc744 \uc5ed\uc784\ud588\ub2e4. \uc774\ud6c4 1951\ub144 \ubd80\uc0b0\uc5d0\uc11c \ub9cc\ub09c\uc801\uc774 \uc788\ub358 \uc774\ud76c\ud638\uc640 \ub2e4\uc2dc \ub9cc\ub098 \uc7ac\ud63c\ud588\uace0, 1963\ub144\uc5d0 \ubbfc\uc8fc\ub2f9 \uc18c\uc18d\uc73c\ub85c \uace0\ud5a5 \ubaa9\ud3ec\uc5d0\uc11c \uc81c6\ub300 \uad6d\ud68c\uc758\uc6d0\uc73c\ub85c \ub2f9\uc120\ub418\uba74\uc11c \uc7ac\uc120 \uad6d\ud68c\uc758\uc6d0\uc774 \ub41c\ub2e4."} {"question": "2017-2018 \uad6d\uc81c\ube59\uc0c1\uacbd\uae30\uc5f0\ub9f9 \uc6d4\ub4dc\ucef5 \ud30c\uacac \ub300\ud45c \uc120\ubc1c\uc804 3\ucc9cm\uc5d0\uc11c \uae40\ubcf4\ub984\uc758 \uacbd\uae30 \uae30\ub85d\uc740?", "paragraph": "2017 \ub144 10 \uc6d4 19\uc77c \uc11c\uc6b8 \ud0dc\ub989\uad6d\uc81c\uc2a4\ucf00\uc774\ud2b8\uc7a5\uc5d0\uc11c \uc5f4\ub9b0 \uc81c52\ud68c \uc804\uad6d\ub0a8\ub140 \uc885\ubaa9\ubcc4\uc2a4\ud53c\ub4dc\uc2a4\ucf00\uc774\ud305\uc120\uc218\uad8c\ub300\ud68c \uacb8 2017-2018 \uad6d\uc81c\ube59\uc0c1\uacbd\uae30\uc5f0\ub9f9 \uc6d4\ub4dc\ucef5 \ud30c\uacac \ub300\ud45c \uc120\ubc1c\uc804 3000m\uc5d0\uc11c \uae40\ubcf4\ub984(\uac15\uc6d0\ub3c4\uccad)\uc740 4 \ubd84 15 \ucd08 45\uc758 \uae30\ub85d\uc73c\ub85c \uacb0\uc2b9\uc120\uc744 \ud1b5\uacfc\ud588\ub2e4. \uc790\uc2e0\uc774 \uc138\uc6b4 \ub300\ud68c\uc2e0\uae30\ub85d 4 \ubd84 17 \ucd08 63\ubcf4\ub2e4 2\ucd08 \uc774\uc0c1 \ube60\ub978 \uae30\ub85d\uc774\ub2e4. \uc774\ub0a0 7 \uc870 \uc778\ucf54\uc2a4\uc5d0\uc11c \ucd9c\ubc1c\ud574 200m\ub97c 21 \ucd08 18\uc5d0 \uc8fc\ud30c\ud55c \uae40\ubcf4\ub984\uc740 \uc774\ud6c4 \ub9c8\uc9c0\ub9c9 \ubc14\ud034\ub97c \uc81c\uc678\ud558\uace4 \ub9e4 \ubc14\ud034\ub97c 32-33 \ucd08\ub300\ub85c \ub3cc\uba70 \uc904\uace7 \uc120\ub450\ub97c \uc9c0\ucf30\ub2e4. \uc774\ub0a0 3000m 2 \uc704\ub294 \ubc15\uc9c0\uc6b0(\ud55c\uad6d\uccb4\ub300)\ub85c 4 \ubd84 16 \ucd08 78\uc744 \uae30\ub85d\ud588\ub2e4. \uae40\ubcf4\ub984\uc740 20 \uc77c \uc5f4\ub9b0 1500m\uc5d0\uc11c 2 \ubd84 3 \ucd08 31\uc758 \uae30\ub85d\uc73c\ub85c \ub178\uc120\uc601(\ucf5c\ud551\ud300-2 \ubd84 2 \ucd08 99)\uc5d0 \uc774\uc5b4 2 \uc704\ub97c \uae30\ub85d\ud588\ub2e4. \uae40\ubcf4\ub984\uc740 7 \uc870 \uc544\uc6c3\ucf54\uc2a4\uc5d0\uc11c \ubc15\ub3c4\uc601(\ub3d9\ub450\ucc9c\uc2dc\uccad)\uacfc \ub808\uc774\uc2f1\uc744 \ud3bc\ucce4\ub2e4. \uccab 300m \uae30\ub85d\uc740 \ub2e4\uc18c \ub290\ub838\ub2e4. \uae40\ubcf4\ub984\uc740 28 \ucd08 17\ub85c \uc8fc\ud30c\ud574 \ucd9c\uc804 \uc120\uc218 \uc911 10 \uc704\uc5d0 \uadf8\ucce4\ub2e4. \uadf8\ub7ec\ub098 \uac00\uc18d\uc774 \ubd99\uae30 \uc2dc\uc791\ud55c 700m \uad6c\uac04\ubd80\ud130 \uc2a4\ud53c\ub4dc\ub97c \uc62c\ub824 \uc804\uccb4 2 \uc704\ub85c \uacb0\uc2b9\uc120\uc744 \ub04a\uc5c8\ub2e4. \uae40\ubcf4\ub984\uc740 \uc774\ubbf8 \uc138\uacc4\uc885\ubaa9\ubcc4\uc120\uc218\uad8c\ub300\ud68c\uc5d0\uc11c \uc6b0\uc2b9\uc744 \ucc28\uc9c0\ud574 \ub9e4\uc2a4\uc2a4\ud0c0\ud2b8 \uc885\ubaa9 \uc120\ubc1c\uad8c\uc744 \ub544\uace0 \uc804\ub0a0 \uc5ec\uc790 3000m\uc640 5000m, \ud300 \ucd94\uc6d4 \uc885\ubaa9 \ucd9c\uc804\uad8c\ub3c4 \ud655\ubcf4\ud588\ub2e4. \uc5ec\uc790 1500m \uc885\ubaa9\uae4c\uc9c0 \ucd9c\uc804\uad8c\uc744 \ub530\ub0b8 \uae40\ubcf4\ub984\uc740 ISU \uc6d4\ub4dc\ucef5\uc5d0\uc11c \uac01 \uc885\ubaa9 2018 \ud3c9\ucc3d\ub3d9\uacc4\uc62c\ub9bc\ud53d \ucd9c\uc804\uad8c \ud68d\ub4e3\uc5d0 \ub3c4\uc804\ud55c\ub2e4.", "answer": "4 \ubd84 15 \ucd08 45", "sentence": "2017 \ub144 10 \uc6d4 19\uc77c \uc11c\uc6b8 \ud0dc\ub989\uad6d\uc81c\uc2a4\ucf00\uc774\ud2b8\uc7a5\uc5d0\uc11c \uc5f4\ub9b0 \uc81c52\ud68c \uc804\uad6d\ub0a8\ub140 \uc885\ubaa9\ubcc4\uc2a4\ud53c\ub4dc\uc2a4\ucf00\uc774\ud305\uc120\uc218\uad8c\ub300\ud68c \uacb8 2017-2018 \uad6d\uc81c\ube59\uc0c1\uacbd\uae30\uc5f0\ub9f9 \uc6d4\ub4dc\ucef5 \ud30c\uacac \ub300\ud45c \uc120\ubc1c\uc804 3000m\uc5d0\uc11c \uae40\ubcf4\ub984(\uac15\uc6d0\ub3c4\uccad)\uc740 4 \ubd84 15 \ucd08 45 \uc758 \uae30\ub85d\uc73c\ub85c \uacb0\uc2b9\uc120\uc744 \ud1b5\uacfc\ud588\ub2e4.", "paragraph_sentence": " 2017 \ub144 10 \uc6d4 19\uc77c \uc11c\uc6b8 \ud0dc\ub989\uad6d\uc81c\uc2a4\ucf00\uc774\ud2b8\uc7a5\uc5d0\uc11c \uc5f4\ub9b0 \uc81c52\ud68c \uc804\uad6d\ub0a8\ub140 \uc885\ubaa9\ubcc4\uc2a4\ud53c\ub4dc\uc2a4\ucf00\uc774\ud305\uc120\uc218\uad8c\ub300\ud68c \uacb8 2017-2018 \uad6d\uc81c\ube59\uc0c1\uacbd\uae30\uc5f0\ub9f9 \uc6d4\ub4dc\ucef5 \ud30c\uacac \ub300\ud45c \uc120\ubc1c\uc804 3000m\uc5d0\uc11c \uae40\ubcf4\ub984(\uac15\uc6d0\ub3c4\uccad)\uc740 4 \ubd84 15 \ucd08 45 \uc758 \uae30\ub85d\uc73c\ub85c \uacb0\uc2b9\uc120\uc744 \ud1b5\uacfc\ud588\ub2e4. \uc790\uc2e0\uc774 \uc138\uc6b4 \ub300\ud68c\uc2e0\uae30\ub85d 4 \ubd84 17 \ucd08 63\ubcf4\ub2e4 2\ucd08 \uc774\uc0c1 \ube60\ub978 \uae30\ub85d\uc774\ub2e4. \uc774\ub0a0 7 \uc870 \uc778\ucf54\uc2a4\uc5d0\uc11c \ucd9c\ubc1c\ud574 200m\ub97c 21 \ucd08 18\uc5d0 \uc8fc\ud30c\ud55c \uae40\ubcf4\ub984\uc740 \uc774\ud6c4 \ub9c8\uc9c0\ub9c9 \ubc14\ud034\ub97c \uc81c\uc678\ud558\uace4 \ub9e4 \ubc14\ud034\ub97c 32-33 \ucd08\ub300\ub85c \ub3cc\uba70 \uc904\uace7 \uc120\ub450\ub97c \uc9c0\ucf30\ub2e4. \uc774\ub0a0 3000m 2 \uc704\ub294 \ubc15\uc9c0\uc6b0(\ud55c\uad6d\uccb4\ub300)\ub85c 4 \ubd84 16 \ucd08 78\uc744 \uae30\ub85d\ud588\ub2e4. \uae40\ubcf4\ub984\uc740 20 \uc77c \uc5f4\ub9b0 1500m\uc5d0\uc11c 2 \ubd84 3 \ucd08 31\uc758 \uae30\ub85d\uc73c\ub85c \ub178\uc120\uc601(\ucf5c\ud551\ud300-2 \ubd84 2 \ucd08 99)\uc5d0 \uc774\uc5b4 2 \uc704\ub97c \uae30\ub85d\ud588\ub2e4. \uae40\ubcf4\ub984\uc740 7 \uc870 \uc544\uc6c3\ucf54\uc2a4\uc5d0\uc11c \ubc15\ub3c4\uc601(\ub3d9\ub450\ucc9c\uc2dc\uccad)\uacfc \ub808\uc774\uc2f1\uc744 \ud3bc\ucce4\ub2e4. \uccab 300m \uae30\ub85d\uc740 \ub2e4\uc18c \ub290\ub838\ub2e4. \uae40\ubcf4\ub984\uc740 28 \ucd08 17\ub85c \uc8fc\ud30c\ud574 \ucd9c\uc804 \uc120\uc218 \uc911 10 \uc704\uc5d0 \uadf8\ucce4\ub2e4. \uadf8\ub7ec\ub098 \uac00\uc18d\uc774 \ubd99\uae30 \uc2dc\uc791\ud55c 700m \uad6c\uac04\ubd80\ud130 \uc2a4\ud53c\ub4dc\ub97c \uc62c\ub824 \uc804\uccb4 2 \uc704\ub85c \uacb0\uc2b9\uc120\uc744 \ub04a\uc5c8\ub2e4. \uae40\ubcf4\ub984\uc740 \uc774\ubbf8 \uc138\uacc4\uc885\ubaa9\ubcc4\uc120\uc218\uad8c\ub300\ud68c\uc5d0\uc11c \uc6b0\uc2b9\uc744 \ucc28\uc9c0\ud574 \ub9e4\uc2a4\uc2a4\ud0c0\ud2b8 \uc885\ubaa9 \uc120\ubc1c\uad8c\uc744 \ub544\uace0 \uc804\ub0a0 \uc5ec\uc790 3000m\uc640 5000m, \ud300 \ucd94\uc6d4 \uc885\ubaa9 \ucd9c\uc804\uad8c\ub3c4 \ud655\ubcf4\ud588\ub2e4. \uc5ec\uc790 1500m \uc885\ubaa9\uae4c\uc9c0 \ucd9c\uc804\uad8c\uc744 \ub530\ub0b8 \uae40\ubcf4\ub984\uc740 ISU \uc6d4\ub4dc\ucef5\uc5d0\uc11c \uac01 \uc885\ubaa9 2018 \ud3c9\ucc3d\ub3d9\uacc4\uc62c\ub9bc\ud53d \ucd9c\uc804\uad8c \ud68d\ub4e3\uc5d0 \ub3c4\uc804\ud55c\ub2e4.", "paragraph_answer": "2017 \ub144 10 \uc6d4 19\uc77c \uc11c\uc6b8 \ud0dc\ub989\uad6d\uc81c\uc2a4\ucf00\uc774\ud2b8\uc7a5\uc5d0\uc11c \uc5f4\ub9b0 \uc81c52\ud68c \uc804\uad6d\ub0a8\ub140 \uc885\ubaa9\ubcc4\uc2a4\ud53c\ub4dc\uc2a4\ucf00\uc774\ud305\uc120\uc218\uad8c\ub300\ud68c \uacb8 2017-2018 \uad6d\uc81c\ube59\uc0c1\uacbd\uae30\uc5f0\ub9f9 \uc6d4\ub4dc\ucef5 \ud30c\uacac \ub300\ud45c \uc120\ubc1c\uc804 3000m\uc5d0\uc11c \uae40\ubcf4\ub984(\uac15\uc6d0\ub3c4\uccad)\uc740 4 \ubd84 15 \ucd08 45 \uc758 \uae30\ub85d\uc73c\ub85c \uacb0\uc2b9\uc120\uc744 \ud1b5\uacfc\ud588\ub2e4. \uc790\uc2e0\uc774 \uc138\uc6b4 \ub300\ud68c\uc2e0\uae30\ub85d 4 \ubd84 17 \ucd08 63\ubcf4\ub2e4 2\ucd08 \uc774\uc0c1 \ube60\ub978 \uae30\ub85d\uc774\ub2e4. \uc774\ub0a0 7 \uc870 \uc778\ucf54\uc2a4\uc5d0\uc11c \ucd9c\ubc1c\ud574 200m\ub97c 21 \ucd08 18\uc5d0 \uc8fc\ud30c\ud55c \uae40\ubcf4\ub984\uc740 \uc774\ud6c4 \ub9c8\uc9c0\ub9c9 \ubc14\ud034\ub97c \uc81c\uc678\ud558\uace4 \ub9e4 \ubc14\ud034\ub97c 32-33 \ucd08\ub300\ub85c \ub3cc\uba70 \uc904\uace7 \uc120\ub450\ub97c \uc9c0\ucf30\ub2e4. \uc774\ub0a0 3000m 2 \uc704\ub294 \ubc15\uc9c0\uc6b0(\ud55c\uad6d\uccb4\ub300)\ub85c 4 \ubd84 16 \ucd08 78\uc744 \uae30\ub85d\ud588\ub2e4. \uae40\ubcf4\ub984\uc740 20 \uc77c \uc5f4\ub9b0 1500m\uc5d0\uc11c 2 \ubd84 3 \ucd08 31\uc758 \uae30\ub85d\uc73c\ub85c \ub178\uc120\uc601(\ucf5c\ud551\ud300-2 \ubd84 2 \ucd08 99)\uc5d0 \uc774\uc5b4 2 \uc704\ub97c \uae30\ub85d\ud588\ub2e4. \uae40\ubcf4\ub984\uc740 7 \uc870 \uc544\uc6c3\ucf54\uc2a4\uc5d0\uc11c \ubc15\ub3c4\uc601(\ub3d9\ub450\ucc9c\uc2dc\uccad)\uacfc \ub808\uc774\uc2f1\uc744 \ud3bc\ucce4\ub2e4. \uccab 300m \uae30\ub85d\uc740 \ub2e4\uc18c \ub290\ub838\ub2e4. \uae40\ubcf4\ub984\uc740 28 \ucd08 17\ub85c \uc8fc\ud30c\ud574 \ucd9c\uc804 \uc120\uc218 \uc911 10 \uc704\uc5d0 \uadf8\ucce4\ub2e4. \uadf8\ub7ec\ub098 \uac00\uc18d\uc774 \ubd99\uae30 \uc2dc\uc791\ud55c 700m \uad6c\uac04\ubd80\ud130 \uc2a4\ud53c\ub4dc\ub97c \uc62c\ub824 \uc804\uccb4 2 \uc704\ub85c \uacb0\uc2b9\uc120\uc744 \ub04a\uc5c8\ub2e4. \uae40\ubcf4\ub984\uc740 \uc774\ubbf8 \uc138\uacc4\uc885\ubaa9\ubcc4\uc120\uc218\uad8c\ub300\ud68c\uc5d0\uc11c \uc6b0\uc2b9\uc744 \ucc28\uc9c0\ud574 \ub9e4\uc2a4\uc2a4\ud0c0\ud2b8 \uc885\ubaa9 \uc120\ubc1c\uad8c\uc744 \ub544\uace0 \uc804\ub0a0 \uc5ec\uc790 3000m\uc640 5000m, \ud300 \ucd94\uc6d4 \uc885\ubaa9 \ucd9c\uc804\uad8c\ub3c4 \ud655\ubcf4\ud588\ub2e4. \uc5ec\uc790 1500m \uc885\ubaa9\uae4c\uc9c0 \ucd9c\uc804\uad8c\uc744 \ub530\ub0b8 \uae40\ubcf4\ub984\uc740 ISU \uc6d4\ub4dc\ucef5\uc5d0\uc11c \uac01 \uc885\ubaa9 2018 \ud3c9\ucc3d\ub3d9\uacc4\uc62c\ub9bc\ud53d \ucd9c\uc804\uad8c \ud68d\ub4e3\uc5d0 \ub3c4\uc804\ud55c\ub2e4.", "sentence_answer": "2017 \ub144 10 \uc6d4 19\uc77c \uc11c\uc6b8 \ud0dc\ub989\uad6d\uc81c\uc2a4\ucf00\uc774\ud2b8\uc7a5\uc5d0\uc11c \uc5f4\ub9b0 \uc81c52\ud68c \uc804\uad6d\ub0a8\ub140 \uc885\ubaa9\ubcc4\uc2a4\ud53c\ub4dc\uc2a4\ucf00\uc774\ud305\uc120\uc218\uad8c\ub300\ud68c \uacb8 2017-2018 \uad6d\uc81c\ube59\uc0c1\uacbd\uae30\uc5f0\ub9f9 \uc6d4\ub4dc\ucef5 \ud30c\uacac \ub300\ud45c \uc120\ubc1c\uc804 3000m\uc5d0\uc11c \uae40\ubcf4\ub984(\uac15\uc6d0\ub3c4\uccad)\uc740 4 \ubd84 15 \ucd08 45 \uc758 \uae30\ub85d\uc73c\ub85c \uacb0\uc2b9\uc120\uc744 \ud1b5\uacfc\ud588\ub2e4.", "paragraph_id": "6488273-20-0", "paragraph_question": "question: 2017-2018 \uad6d\uc81c\ube59\uc0c1\uacbd\uae30\uc5f0\ub9f9 \uc6d4\ub4dc\ucef5 \ud30c\uacac \ub300\ud45c \uc120\ubc1c\uc804 3\ucc9cm\uc5d0\uc11c \uae40\ubcf4\ub984\uc758 \uacbd\uae30 \uae30\ub85d\uc740?, context: 2017 \ub144 10 \uc6d4 19\uc77c \uc11c\uc6b8 \ud0dc\ub989\uad6d\uc81c\uc2a4\ucf00\uc774\ud2b8\uc7a5\uc5d0\uc11c \uc5f4\ub9b0 \uc81c52\ud68c \uc804\uad6d\ub0a8\ub140 \uc885\ubaa9\ubcc4\uc2a4\ud53c\ub4dc\uc2a4\ucf00\uc774\ud305\uc120\uc218\uad8c\ub300\ud68c \uacb8 2017-2018 \uad6d\uc81c\ube59\uc0c1\uacbd\uae30\uc5f0\ub9f9 \uc6d4\ub4dc\ucef5 \ud30c\uacac \ub300\ud45c \uc120\ubc1c\uc804 3000m\uc5d0\uc11c \uae40\ubcf4\ub984(\uac15\uc6d0\ub3c4\uccad)\uc740 4 \ubd84 15 \ucd08 45\uc758 \uae30\ub85d\uc73c\ub85c \uacb0\uc2b9\uc120\uc744 \ud1b5\uacfc\ud588\ub2e4. \uc790\uc2e0\uc774 \uc138\uc6b4 \ub300\ud68c\uc2e0\uae30\ub85d 4 \ubd84 17 \ucd08 63\ubcf4\ub2e4 2\ucd08 \uc774\uc0c1 \ube60\ub978 \uae30\ub85d\uc774\ub2e4. \uc774\ub0a0 7 \uc870 \uc778\ucf54\uc2a4\uc5d0\uc11c \ucd9c\ubc1c\ud574 200m\ub97c 21 \ucd08 18\uc5d0 \uc8fc\ud30c\ud55c \uae40\ubcf4\ub984\uc740 \uc774\ud6c4 \ub9c8\uc9c0\ub9c9 \ubc14\ud034\ub97c \uc81c\uc678\ud558\uace4 \ub9e4 \ubc14\ud034\ub97c 32-33 \ucd08\ub300\ub85c \ub3cc\uba70 \uc904\uace7 \uc120\ub450\ub97c \uc9c0\ucf30\ub2e4. \uc774\ub0a0 3000m 2 \uc704\ub294 \ubc15\uc9c0\uc6b0(\ud55c\uad6d\uccb4\ub300)\ub85c 4 \ubd84 16 \ucd08 78\uc744 \uae30\ub85d\ud588\ub2e4. \uae40\ubcf4\ub984\uc740 20 \uc77c \uc5f4\ub9b0 1500m\uc5d0\uc11c 2 \ubd84 3 \ucd08 31\uc758 \uae30\ub85d\uc73c\ub85c \ub178\uc120\uc601(\ucf5c\ud551\ud300-2 \ubd84 2 \ucd08 99)\uc5d0 \uc774\uc5b4 2 \uc704\ub97c \uae30\ub85d\ud588\ub2e4. \uae40\ubcf4\ub984\uc740 7 \uc870 \uc544\uc6c3\ucf54\uc2a4\uc5d0\uc11c \ubc15\ub3c4\uc601(\ub3d9\ub450\ucc9c\uc2dc\uccad)\uacfc \ub808\uc774\uc2f1\uc744 \ud3bc\ucce4\ub2e4. \uccab 300m \uae30\ub85d\uc740 \ub2e4\uc18c \ub290\ub838\ub2e4. \uae40\ubcf4\ub984\uc740 28 \ucd08 17\ub85c \uc8fc\ud30c\ud574 \ucd9c\uc804 \uc120\uc218 \uc911 10 \uc704\uc5d0 \uadf8\ucce4\ub2e4. \uadf8\ub7ec\ub098 \uac00\uc18d\uc774 \ubd99\uae30 \uc2dc\uc791\ud55c 700m \uad6c\uac04\ubd80\ud130 \uc2a4\ud53c\ub4dc\ub97c \uc62c\ub824 \uc804\uccb4 2 \uc704\ub85c \uacb0\uc2b9\uc120\uc744 \ub04a\uc5c8\ub2e4. \uae40\ubcf4\ub984\uc740 \uc774\ubbf8 \uc138\uacc4\uc885\ubaa9\ubcc4\uc120\uc218\uad8c\ub300\ud68c\uc5d0\uc11c \uc6b0\uc2b9\uc744 \ucc28\uc9c0\ud574 \ub9e4\uc2a4\uc2a4\ud0c0\ud2b8 \uc885\ubaa9 \uc120\ubc1c\uad8c\uc744 \ub544\uace0 \uc804\ub0a0 \uc5ec\uc790 3000m\uc640 5000m, \ud300 \ucd94\uc6d4 \uc885\ubaa9 \ucd9c\uc804\uad8c\ub3c4 \ud655\ubcf4\ud588\ub2e4. \uc5ec\uc790 1500m \uc885\ubaa9\uae4c\uc9c0 \ucd9c\uc804\uad8c\uc744 \ub530\ub0b8 \uae40\ubcf4\ub984\uc740 ISU \uc6d4\ub4dc\ucef5\uc5d0\uc11c \uac01 \uc885\ubaa9 2018 \ud3c9\ucc3d\ub3d9\uacc4\uc62c\ub9bc\ud53d \ucd9c\uc804\uad8c \ud68d\ub4e3\uc5d0 \ub3c4\uc804\ud55c\ub2e4."} {"question": "1996\ub144 \uc560\ud2c0\ub780\ud0c0 \ud558\uacc4 \uc62c\ub9bc\ud53d \uc911 \ud3ed\ubc1c \uc0ac\uac74\uc774 \uc77c\uc5b4\ub09c \uacf3\uc740?", "paragraph": "\ud14c\ub7ec\ub3c4 \uc62c\ub9bc\ud53d\uc5d0\uc11c \uacf5\ud3ec\uc758 \ub300\uc0c1\uc774\uc5c8\ub2e4. \ubb8c\ud5e8 \ucc38\uc0ac\ub85c \uc54c\ub824\uc9c4 1972\ub144\uc5d0 \uc11c\ub3c5 \ubc14\uc774\uc5d0\ub978\uc758 \ubb8c\ud5e8\uc5d0\uc11c \uc5f4\ub9b0 \ud558\uacc4 \uc62c\ub9bc\ud53d\ub54c\uc758 \uc0ac\uac74\uc740 \ud14c\ub7ec\ub9ac\uc2a4\ud2b8\uc778 \uac80\uc740 9\uc6d4\ub2e8\uc774 \uc77c\uc73c\ud0a8 \uc0ac\uac74\uc73c\ub85c\uc11c \uc774\uc2a4\ub77c\uc5d8 \uc120\uc218 11\uba85\uc744 \uc778\uc9c8\ub85c \ubd99\uc7a1\uc558\ub2e4\uac00 \uc804\uc6d0\uc774 \uc0ac\ub9dd\ud55c \uc0ac\uac74\uc774\ub2e4. \ub2f9\uc2dc \ubbf8\uc219\ud55c \uc9c4\uc555\uc73c\ub85c \uc778\ud574 \uc778\uc9c8 9\uba85(\uc120\uc218 1\uba85\uacfc \ucf54\uce58 1\uba85\uc740 \uc778\uc9c8\ub85c \uc7a1\uae30 \uc774\uc804\uc5d0 \uc0b4\ud574), \ud14c\ub7ec\ubc94 5\uba85, \ub3c5\uc77c \uacbd\ucc30\uad00 1\uba85\uc774 \uc0ac\ub9dd\ud588\uc73c\uba70 \uc774 \uc9c4\uc555 \uc791\uc804 \uc774\uc804\uc5d0\ub294 \uc778\uc9c8\ub4e4\uc740 \ub2e8 \ud55c \uba85\ub3c4 \uc8fd\uc9c0 \uc54a\uc558\ub2e4. \uc560\ud2c0\ub780\ud0c0\uc5d0\uc11c \uc5f4\ub9b0 1996\ub144 \ud558\uacc4 \uc62c\ub9bc\ud53d \ub54c\ub294 \uc13c\ud14c\ub2c8\uc5bc \uc62c\ub9bc\ud53d \uacf5\uc6d0(Centennial Olympic Park)\uc5d0\uc11c \ud3ed\ubc1c \uc0ac\uac74\uc774 \uc77c\uc5b4\ub098 2\uba85\uc774 \uc8fd\uace0 111\uba85\uc774 \ub2e4\uce58\ub294 \uc0ac\uac74\uc774 \ubc1c\uc0dd\ud588\ub2e4. \uc774 \uc0ac\uac74\uc758 \uc8fc\ubaa8\uc790 \uc5d0\ub9ad \ub85c\ubc84\ud2b8 \ub8e8\ub3cc\ud504\ub294 \uc885\uc2e0\ud615\uc744 \uc120\uace0\ubc1b\uc558\ub2e4. \ucc38\uace0\ub85c \ub9c8\ub77c\ud1a4 \uc5ed\uc2dc \uc804\uc7c1\uc5d0\uc11c \uc720\ub798\ud55c \uac83\uc774\ub2e4.", "answer": "\uc13c\ud14c\ub2c8\uc5bc \uc62c\ub9bc\ud53d \uacf5\uc6d0", "sentence": "\uc560\ud2c0\ub780\ud0c0\uc5d0\uc11c \uc5f4\ub9b0 1996\ub144 \ud558\uacc4 \uc62c\ub9bc\ud53d \ub54c\ub294 \uc13c\ud14c\ub2c8\uc5bc \uc62c\ub9bc\ud53d \uacf5\uc6d0 (Centennial Olympic Park)", "paragraph_sentence": "\ud14c\ub7ec\ub3c4 \uc62c\ub9bc\ud53d\uc5d0\uc11c \uacf5\ud3ec\uc758 \ub300\uc0c1\uc774\uc5c8\ub2e4. \ubb8c\ud5e8 \ucc38\uc0ac\ub85c \uc54c\ub824\uc9c4 1972\ub144\uc5d0 \uc11c\ub3c5 \ubc14\uc774\uc5d0\ub978\uc758 \ubb8c\ud5e8\uc5d0\uc11c \uc5f4\ub9b0 \ud558\uacc4 \uc62c\ub9bc\ud53d\ub54c\uc758 \uc0ac\uac74\uc740 \ud14c\ub7ec\ub9ac\uc2a4\ud2b8\uc778 \uac80\uc740 9\uc6d4\ub2e8\uc774 \uc77c\uc73c\ud0a8 \uc0ac\uac74\uc73c\ub85c\uc11c \uc774\uc2a4\ub77c\uc5d8 \uc120\uc218 11\uba85\uc744 \uc778\uc9c8\ub85c \ubd99\uc7a1\uc558\ub2e4\uac00 \uc804\uc6d0\uc774 \uc0ac\ub9dd\ud55c \uc0ac\uac74\uc774\ub2e4. \ub2f9\uc2dc \ubbf8\uc219\ud55c \uc9c4\uc555\uc73c\ub85c \uc778\ud574 \uc778\uc9c8 9\uba85(\uc120\uc218 1\uba85\uacfc \ucf54\uce58 1\uba85\uc740 \uc778\uc9c8\ub85c \uc7a1\uae30 \uc774\uc804\uc5d0 \uc0b4\ud574), \ud14c\ub7ec\ubc94 5\uba85, \ub3c5\uc77c \uacbd\ucc30\uad00 1\uba85\uc774 \uc0ac\ub9dd\ud588\uc73c\uba70 \uc774 \uc9c4\uc555 \uc791\uc804 \uc774\uc804\uc5d0\ub294 \uc778\uc9c8\ub4e4\uc740 \ub2e8 \ud55c \uba85\ub3c4 \uc8fd\uc9c0 \uc54a\uc558\ub2e4. \uc560\ud2c0\ub780\ud0c0\uc5d0\uc11c \uc5f4\ub9b0 1996\ub144 \ud558\uacc4 \uc62c\ub9bc\ud53d \ub54c\ub294 \uc13c\ud14c\ub2c8\uc5bc \uc62c\ub9bc\ud53d \uacf5\uc6d0 (Centennial Olympic Park) \uc5d0\uc11c \ud3ed\ubc1c \uc0ac\uac74\uc774 \uc77c\uc5b4\ub098 2\uba85\uc774 \uc8fd\uace0 111\uba85\uc774 \ub2e4\uce58\ub294 \uc0ac\uac74\uc774 \ubc1c\uc0dd\ud588\ub2e4. \uc774 \uc0ac\uac74\uc758 \uc8fc\ubaa8\uc790 \uc5d0\ub9ad \ub85c\ubc84\ud2b8 \ub8e8\ub3cc\ud504\ub294 \uc885\uc2e0\ud615\uc744 \uc120\uace0\ubc1b\uc558\ub2e4. \ucc38\uace0\ub85c \ub9c8\ub77c\ud1a4 \uc5ed\uc2dc \uc804\uc7c1\uc5d0\uc11c \uc720\ub798\ud55c \uac83\uc774\ub2e4.", "paragraph_answer": "\ud14c\ub7ec\ub3c4 \uc62c\ub9bc\ud53d\uc5d0\uc11c \uacf5\ud3ec\uc758 \ub300\uc0c1\uc774\uc5c8\ub2e4. \ubb8c\ud5e8 \ucc38\uc0ac\ub85c \uc54c\ub824\uc9c4 1972\ub144\uc5d0 \uc11c\ub3c5 \ubc14\uc774\uc5d0\ub978\uc758 \ubb8c\ud5e8\uc5d0\uc11c \uc5f4\ub9b0 \ud558\uacc4 \uc62c\ub9bc\ud53d\ub54c\uc758 \uc0ac\uac74\uc740 \ud14c\ub7ec\ub9ac\uc2a4\ud2b8\uc778 \uac80\uc740 9\uc6d4\ub2e8\uc774 \uc77c\uc73c\ud0a8 \uc0ac\uac74\uc73c\ub85c\uc11c \uc774\uc2a4\ub77c\uc5d8 \uc120\uc218 11\uba85\uc744 \uc778\uc9c8\ub85c \ubd99\uc7a1\uc558\ub2e4\uac00 \uc804\uc6d0\uc774 \uc0ac\ub9dd\ud55c \uc0ac\uac74\uc774\ub2e4. \ub2f9\uc2dc \ubbf8\uc219\ud55c \uc9c4\uc555\uc73c\ub85c \uc778\ud574 \uc778\uc9c8 9\uba85(\uc120\uc218 1\uba85\uacfc \ucf54\uce58 1\uba85\uc740 \uc778\uc9c8\ub85c \uc7a1\uae30 \uc774\uc804\uc5d0 \uc0b4\ud574), \ud14c\ub7ec\ubc94 5\uba85, \ub3c5\uc77c \uacbd\ucc30\uad00 1\uba85\uc774 \uc0ac\ub9dd\ud588\uc73c\uba70 \uc774 \uc9c4\uc555 \uc791\uc804 \uc774\uc804\uc5d0\ub294 \uc778\uc9c8\ub4e4\uc740 \ub2e8 \ud55c \uba85\ub3c4 \uc8fd\uc9c0 \uc54a\uc558\ub2e4. \uc560\ud2c0\ub780\ud0c0\uc5d0\uc11c \uc5f4\ub9b0 1996\ub144 \ud558\uacc4 \uc62c\ub9bc\ud53d \ub54c\ub294 \uc13c\ud14c\ub2c8\uc5bc \uc62c\ub9bc\ud53d \uacf5\uc6d0 (Centennial Olympic Park)\uc5d0\uc11c \ud3ed\ubc1c \uc0ac\uac74\uc774 \uc77c\uc5b4\ub098 2\uba85\uc774 \uc8fd\uace0 111\uba85\uc774 \ub2e4\uce58\ub294 \uc0ac\uac74\uc774 \ubc1c\uc0dd\ud588\ub2e4. \uc774 \uc0ac\uac74\uc758 \uc8fc\ubaa8\uc790 \uc5d0\ub9ad \ub85c\ubc84\ud2b8 \ub8e8\ub3cc\ud504\ub294 \uc885\uc2e0\ud615\uc744 \uc120\uace0\ubc1b\uc558\ub2e4. \ucc38\uace0\ub85c \ub9c8\ub77c\ud1a4 \uc5ed\uc2dc \uc804\uc7c1\uc5d0\uc11c \uc720\ub798\ud55c \uac83\uc774\ub2e4.", "sentence_answer": "\uc560\ud2c0\ub780\ud0c0\uc5d0\uc11c \uc5f4\ub9b0 1996\ub144 \ud558\uacc4 \uc62c\ub9bc\ud53d \ub54c\ub294 \uc13c\ud14c\ub2c8\uc5bc \uc62c\ub9bc\ud53d \uacf5\uc6d0 (Centennial Olympic Park)", "paragraph_id": "6525547-29-2", "paragraph_question": "question: 1996\ub144 \uc560\ud2c0\ub780\ud0c0 \ud558\uacc4 \uc62c\ub9bc\ud53d \uc911 \ud3ed\ubc1c \uc0ac\uac74\uc774 \uc77c\uc5b4\ub09c \uacf3\uc740?, context: \ud14c\ub7ec\ub3c4 \uc62c\ub9bc\ud53d\uc5d0\uc11c \uacf5\ud3ec\uc758 \ub300\uc0c1\uc774\uc5c8\ub2e4. \ubb8c\ud5e8 \ucc38\uc0ac\ub85c \uc54c\ub824\uc9c4 1972\ub144\uc5d0 \uc11c\ub3c5 \ubc14\uc774\uc5d0\ub978\uc758 \ubb8c\ud5e8\uc5d0\uc11c \uc5f4\ub9b0 \ud558\uacc4 \uc62c\ub9bc\ud53d\ub54c\uc758 \uc0ac\uac74\uc740 \ud14c\ub7ec\ub9ac\uc2a4\ud2b8\uc778 \uac80\uc740 9\uc6d4\ub2e8\uc774 \uc77c\uc73c\ud0a8 \uc0ac\uac74\uc73c\ub85c\uc11c \uc774\uc2a4\ub77c\uc5d8 \uc120\uc218 11\uba85\uc744 \uc778\uc9c8\ub85c \ubd99\uc7a1\uc558\ub2e4\uac00 \uc804\uc6d0\uc774 \uc0ac\ub9dd\ud55c \uc0ac\uac74\uc774\ub2e4. \ub2f9\uc2dc \ubbf8\uc219\ud55c \uc9c4\uc555\uc73c\ub85c \uc778\ud574 \uc778\uc9c8 9\uba85(\uc120\uc218 1\uba85\uacfc \ucf54\uce58 1\uba85\uc740 \uc778\uc9c8\ub85c \uc7a1\uae30 \uc774\uc804\uc5d0 \uc0b4\ud574), \ud14c\ub7ec\ubc94 5\uba85, \ub3c5\uc77c \uacbd\ucc30\uad00 1\uba85\uc774 \uc0ac\ub9dd\ud588\uc73c\uba70 \uc774 \uc9c4\uc555 \uc791\uc804 \uc774\uc804\uc5d0\ub294 \uc778\uc9c8\ub4e4\uc740 \ub2e8 \ud55c \uba85\ub3c4 \uc8fd\uc9c0 \uc54a\uc558\ub2e4. \uc560\ud2c0\ub780\ud0c0\uc5d0\uc11c \uc5f4\ub9b0 1996\ub144 \ud558\uacc4 \uc62c\ub9bc\ud53d \ub54c\ub294 \uc13c\ud14c\ub2c8\uc5bc \uc62c\ub9bc\ud53d \uacf5\uc6d0(Centennial Olympic Park)\uc5d0\uc11c \ud3ed\ubc1c \uc0ac\uac74\uc774 \uc77c\uc5b4\ub098 2\uba85\uc774 \uc8fd\uace0 111\uba85\uc774 \ub2e4\uce58\ub294 \uc0ac\uac74\uc774 \ubc1c\uc0dd\ud588\ub2e4. \uc774 \uc0ac\uac74\uc758 \uc8fc\ubaa8\uc790 \uc5d0\ub9ad \ub85c\ubc84\ud2b8 \ub8e8\ub3cc\ud504\ub294 \uc885\uc2e0\ud615\uc744 \uc120\uace0\ubc1b\uc558\ub2e4. \ucc38\uace0\ub85c \ub9c8\ub77c\ud1a4 \uc5ed\uc2dc \uc804\uc7c1\uc5d0\uc11c \uc720\ub798\ud55c \uac83\uc774\ub2e4."} {"question": "\ub77c\ubd80\uc544\uc9c0\uc5d0 \ud074\ub85c\ub4dc \ub8e8\uc774 \ubca0\ub974\ud1a8\ub808 \ub4f1\uc774 \ud568\uaed8 \uac1c\uc815\ud55c \ubc95\uc758 \uc774\ub984\uc740 \ubb34\uc5c7\uc77c\uae4c?", "paragraph": "\uc774\uc804\uc5d0\ub294 \ud654\ud569\ubb3c\uc758 \uc774\ub984\uc744 \ubd99\uc774\ub294 \ub370 \uc788\uc5b4\uc11c \uae30\uc900\uc774 \uc81c\uac01\uac01\uc774\ub77c, \uc774\ub984\uc774 \ub9e4\uc6b0 \ud63c\ub780\uc2a4\ub7ec\uc6b8 \uc218\ubc16\uc5d0 \uc5c6\uc5c8\ub2e4. \uc774\ub97c \ud574\uacb0\ud558\uae30 \uc704\ud574\uc11c \ub77c\ubd80\uc544\uc9c0\uc5d0\u00b7\ud074\ub85c\ub4dc \ub8e8\uc774 \ubca0\ub974\ud1a8\ub808\u00b7\ub8e8\uc774 \ubca0\ub974\ub098\ub974 \uae30\ud1b5 \ub4dc \ubaa8\ub974\ubcf4\u00b7\uc559\ud22c\uc548 \ud504\ub791\uc218\uc544 \ud478\ub974\ud06c\ub8e8\uc544 \ub4f1\uc774 \uc8fc\ucd95\uc774 \ub418\uc5b4 \ud654\ud559\uc801 \uba85\uba85\ubc95\uc744 \uac1c\uc815\ud558\uc600\uace0, \uc774\ub97c 1787\ub144\uc5d0 M\u00e9thode de nomenclature chimique\ub77c\ub294 \uc774\ub984\uc73c\ub85c \ub0b4\ub193\uc558\ub2e4. \uc774\ub4e4\uc740 \ud654\ud559 \ubb3c\uc9c8\uc740 \ud658\uacbd\uc5d0 \ub530\ub77c\uc11c \ubcc0\ud558\uc9c0 \uc54a\ub294 \uc774\ub984\uc744 \uac00\uc9c0\uace0, \uc774\ub984\uc740 \uadf8 \ubb3c\uc9c8\uc758 \uc131\uc9c8\uc744 \uc798 \ub4dc\ub7ec\ub0b4\uba70, \uc774\ub984\uc740 \uadf8\ub9ac\uc2a4\uc5b4 \ub610\ub294 \ub77c\ud2f4\uc5b4\uc5d0 \uadfc\uac04\uc744 \ub454\ub2e4\ub294 \ub4f1\uc758 \uc6d0\uce59\uc5d0 \uae30\ubc18\uc744 \ub450\uc5b4 \ud654\ud559 \ubb3c\uc9c8\ub4e4\uc744 \uc0c8\ub86d\uac8c \uba85\uba85\ud558\uc600\ub2e4. \uc774\ud6c4 \ub9c8\ub974\ud2f4 \ud558\uc778\ub9ac\ud788 \ud074\ub77c\ud504\ub85c\ud2b8\ub97c \uc8fc\ucd95\uc73c\ub85c \ud55c \ub3c5\uc77c\uc758 \ud654\ud559\uc790, \uadf8\ub9ac\uace0 \uc790\uc2e0\uc758 \uad00\uce21\uc744 \ub354 \uc911\uc2dc\ud588\ub358 \ud5e8\ub9ac \uce90\ubc88\ub514\uc2dc\uc640 \uc608\uc804\uc758 \uc774\ub860\uc5d0 \uc9d1\ucc29\ud588\ub358 \uc870\uc9c0\ud504 \ud504\ub9ac\uc2a4\ud2c0\ub9ac\ub97c \uc81c\uc678\ud55c \ub300\ubd80\ubd84\uc758 \uc601\uad6d\uc758 \ud654\ud559\uc790\ub3c4 \ud504\ub791\uc2a4 \ud654\ud559\uc790\uc758 \uc6c0\uc9c1\uc784\uc5d0 \ub3d9\ucc38\ud558\uc600\ub2e4. M\u00e9thode de nomenclature chimique\uc740 \ud504\ub791\uc2a4\ubfd0\ub9cc\uc774 \uc544\ub2c8\ub77c, \ub3c5\uc77c\u00b7\uc601\uad6d\u00b7\uc2a4\ud398\uc778\u00b7\uc774\ud0c8\ub9ac\uc544 \ub4f1 \uc720\ub7fd \uac01\uad6d\uc5d0\uc11c \ubc88\uc5ed\ub418\uc5b4 \uc0ac\uc6a9\ub418\uc5c8\ub2e4. \uacfc\ud559\uc0ac \ud559\uc790\ub4e4\uc740 \uc774\ub7ec\ud55c \ud654\ud559\uc801 \uba85\uba85\ubc95 \uac1c\uc815\uc740 \ud1b5\uc77c\ub41c \ud654\ud559\uc801 \uc5b8\uc5b4\ub97c \ub9cc\ub4dc\ub294 \ub370 \ub3c4\uc6c0\uc744 \uc8fc\uc5b4, \ud654\ud559\uc758 \ubc1c\uc804\uc5d0 \uae30\uc5ec\ud558\uc600\ub2e4\uace0 \ud558\uc5ec \ud070 \uc758\uc758\ub97c \ub450\uace0 \uc788\ub2e4.", "answer": "\ud654\ud559\uc801 \uba85\uba85\ubc95", "sentence": "\uc774\ub97c \ud574\uacb0\ud558\uae30 \uc704\ud574\uc11c \ub77c\ubd80\uc544\uc9c0\uc5d0\u00b7\ud074\ub85c\ub4dc \ub8e8\uc774 \ubca0\ub974\ud1a8\ub808\u00b7\ub8e8\uc774 \ubca0\ub974\ub098\ub974 \uae30\ud1b5 \ub4dc \ubaa8\ub974\ubcf4\u00b7\uc559\ud22c\uc548 \ud504\ub791\uc218\uc544 \ud478\ub974\ud06c\ub8e8\uc544 \ub4f1\uc774 \uc8fc\ucd95\uc774 \ub418\uc5b4 \ud654\ud559\uc801 \uba85\uba85\ubc95 \uc744 \uac1c\uc815\ud558\uc600\uace0, \uc774\ub97c 1787\ub144\uc5d0 M\u00e9thode de nomenclature chimique\ub77c\ub294 \uc774\ub984\uc73c\ub85c \ub0b4\ub193\uc558\ub2e4.", "paragraph_sentence": "\uc774\uc804\uc5d0\ub294 \ud654\ud569\ubb3c\uc758 \uc774\ub984\uc744 \ubd99\uc774\ub294 \ub370 \uc788\uc5b4\uc11c \uae30\uc900\uc774 \uc81c\uac01\uac01\uc774\ub77c, \uc774\ub984\uc774 \ub9e4\uc6b0 \ud63c\ub780\uc2a4\ub7ec\uc6b8 \uc218\ubc16\uc5d0 \uc5c6\uc5c8\ub2e4. \uc774\ub97c \ud574\uacb0\ud558\uae30 \uc704\ud574\uc11c \ub77c\ubd80\uc544\uc9c0\uc5d0\u00b7\ud074\ub85c\ub4dc \ub8e8\uc774 \ubca0\ub974\ud1a8\ub808\u00b7\ub8e8\uc774 \ubca0\ub974\ub098\ub974 \uae30\ud1b5 \ub4dc \ubaa8\ub974\ubcf4\u00b7\uc559\ud22c\uc548 \ud504\ub791\uc218\uc544 \ud478\ub974\ud06c\ub8e8\uc544 \ub4f1\uc774 \uc8fc\ucd95\uc774 \ub418\uc5b4 \ud654\ud559\uc801 \uba85\uba85\ubc95 \uc744 \uac1c\uc815\ud558\uc600\uace0, \uc774\ub97c 1787\ub144\uc5d0 M\u00e9thode de nomenclature chimique\ub77c\ub294 \uc774\ub984\uc73c\ub85c \ub0b4\ub193\uc558\ub2e4. \uc774\ub4e4\uc740 \ud654\ud559 \ubb3c\uc9c8\uc740 \ud658\uacbd\uc5d0 \ub530\ub77c\uc11c \ubcc0\ud558\uc9c0 \uc54a\ub294 \uc774\ub984\uc744 \uac00\uc9c0\uace0, \uc774\ub984\uc740 \uadf8 \ubb3c\uc9c8\uc758 \uc131\uc9c8\uc744 \uc798 \ub4dc\ub7ec\ub0b4\uba70, \uc774\ub984\uc740 \uadf8\ub9ac\uc2a4\uc5b4 \ub610\ub294 \ub77c\ud2f4\uc5b4\uc5d0 \uadfc\uac04\uc744 \ub454\ub2e4\ub294 \ub4f1\uc758 \uc6d0\uce59\uc5d0 \uae30\ubc18\uc744 \ub450\uc5b4 \ud654\ud559 \ubb3c\uc9c8\ub4e4\uc744 \uc0c8\ub86d\uac8c \uba85\uba85\ud558\uc600\ub2e4. \uc774\ud6c4 \ub9c8\ub974\ud2f4 \ud558\uc778\ub9ac\ud788 \ud074\ub77c\ud504\ub85c\ud2b8\ub97c \uc8fc\ucd95\uc73c\ub85c \ud55c \ub3c5\uc77c\uc758 \ud654\ud559\uc790, \uadf8\ub9ac\uace0 \uc790\uc2e0\uc758 \uad00\uce21\uc744 \ub354 \uc911\uc2dc\ud588\ub358 \ud5e8\ub9ac \uce90\ubc88\ub514\uc2dc\uc640 \uc608\uc804\uc758 \uc774\ub860\uc5d0 \uc9d1\ucc29\ud588\ub358 \uc870\uc9c0\ud504 \ud504\ub9ac\uc2a4\ud2c0\ub9ac\ub97c \uc81c\uc678\ud55c \ub300\ubd80\ubd84\uc758 \uc601\uad6d\uc758 \ud654\ud559\uc790\ub3c4 \ud504\ub791\uc2a4 \ud654\ud559\uc790\uc758 \uc6c0\uc9c1\uc784\uc5d0 \ub3d9\ucc38\ud558\uc600\ub2e4. M\u00e9thode de nomenclature chimique\uc740 \ud504\ub791\uc2a4\ubfd0\ub9cc\uc774 \uc544\ub2c8\ub77c, \ub3c5\uc77c\u00b7\uc601\uad6d\u00b7\uc2a4\ud398\uc778\u00b7\uc774\ud0c8\ub9ac\uc544 \ub4f1 \uc720\ub7fd \uac01\uad6d\uc5d0\uc11c \ubc88\uc5ed\ub418\uc5b4 \uc0ac\uc6a9\ub418\uc5c8\ub2e4. \uacfc\ud559\uc0ac \ud559\uc790\ub4e4\uc740 \uc774\ub7ec\ud55c \ud654\ud559\uc801 \uba85\uba85\ubc95 \uac1c\uc815\uc740 \ud1b5\uc77c\ub41c \ud654\ud559\uc801 \uc5b8\uc5b4\ub97c \ub9cc\ub4dc\ub294 \ub370 \ub3c4\uc6c0\uc744 \uc8fc\uc5b4, \ud654\ud559\uc758 \ubc1c\uc804\uc5d0 \uae30\uc5ec\ud558\uc600\ub2e4\uace0 \ud558\uc5ec \ud070 \uc758\uc758\ub97c \ub450\uace0 \uc788\ub2e4.", "paragraph_answer": "\uc774\uc804\uc5d0\ub294 \ud654\ud569\ubb3c\uc758 \uc774\ub984\uc744 \ubd99\uc774\ub294 \ub370 \uc788\uc5b4\uc11c \uae30\uc900\uc774 \uc81c\uac01\uac01\uc774\ub77c, \uc774\ub984\uc774 \ub9e4\uc6b0 \ud63c\ub780\uc2a4\ub7ec\uc6b8 \uc218\ubc16\uc5d0 \uc5c6\uc5c8\ub2e4. \uc774\ub97c \ud574\uacb0\ud558\uae30 \uc704\ud574\uc11c \ub77c\ubd80\uc544\uc9c0\uc5d0\u00b7\ud074\ub85c\ub4dc \ub8e8\uc774 \ubca0\ub974\ud1a8\ub808\u00b7\ub8e8\uc774 \ubca0\ub974\ub098\ub974 \uae30\ud1b5 \ub4dc \ubaa8\ub974\ubcf4\u00b7\uc559\ud22c\uc548 \ud504\ub791\uc218\uc544 \ud478\ub974\ud06c\ub8e8\uc544 \ub4f1\uc774 \uc8fc\ucd95\uc774 \ub418\uc5b4 \ud654\ud559\uc801 \uba85\uba85\ubc95 \uc744 \uac1c\uc815\ud558\uc600\uace0, \uc774\ub97c 1787\ub144\uc5d0 M\u00e9thode de nomenclature chimique\ub77c\ub294 \uc774\ub984\uc73c\ub85c \ub0b4\ub193\uc558\ub2e4. \uc774\ub4e4\uc740 \ud654\ud559 \ubb3c\uc9c8\uc740 \ud658\uacbd\uc5d0 \ub530\ub77c\uc11c \ubcc0\ud558\uc9c0 \uc54a\ub294 \uc774\ub984\uc744 \uac00\uc9c0\uace0, \uc774\ub984\uc740 \uadf8 \ubb3c\uc9c8\uc758 \uc131\uc9c8\uc744 \uc798 \ub4dc\ub7ec\ub0b4\uba70, \uc774\ub984\uc740 \uadf8\ub9ac\uc2a4\uc5b4 \ub610\ub294 \ub77c\ud2f4\uc5b4\uc5d0 \uadfc\uac04\uc744 \ub454\ub2e4\ub294 \ub4f1\uc758 \uc6d0\uce59\uc5d0 \uae30\ubc18\uc744 \ub450\uc5b4 \ud654\ud559 \ubb3c\uc9c8\ub4e4\uc744 \uc0c8\ub86d\uac8c \uba85\uba85\ud558\uc600\ub2e4. \uc774\ud6c4 \ub9c8\ub974\ud2f4 \ud558\uc778\ub9ac\ud788 \ud074\ub77c\ud504\ub85c\ud2b8\ub97c \uc8fc\ucd95\uc73c\ub85c \ud55c \ub3c5\uc77c\uc758 \ud654\ud559\uc790, \uadf8\ub9ac\uace0 \uc790\uc2e0\uc758 \uad00\uce21\uc744 \ub354 \uc911\uc2dc\ud588\ub358 \ud5e8\ub9ac \uce90\ubc88\ub514\uc2dc\uc640 \uc608\uc804\uc758 \uc774\ub860\uc5d0 \uc9d1\ucc29\ud588\ub358 \uc870\uc9c0\ud504 \ud504\ub9ac\uc2a4\ud2c0\ub9ac\ub97c \uc81c\uc678\ud55c \ub300\ubd80\ubd84\uc758 \uc601\uad6d\uc758 \ud654\ud559\uc790\ub3c4 \ud504\ub791\uc2a4 \ud654\ud559\uc790\uc758 \uc6c0\uc9c1\uc784\uc5d0 \ub3d9\ucc38\ud558\uc600\ub2e4. M\u00e9thode de nomenclature chimique\uc740 \ud504\ub791\uc2a4\ubfd0\ub9cc\uc774 \uc544\ub2c8\ub77c, \ub3c5\uc77c\u00b7\uc601\uad6d\u00b7\uc2a4\ud398\uc778\u00b7\uc774\ud0c8\ub9ac\uc544 \ub4f1 \uc720\ub7fd \uac01\uad6d\uc5d0\uc11c \ubc88\uc5ed\ub418\uc5b4 \uc0ac\uc6a9\ub418\uc5c8\ub2e4. \uacfc\ud559\uc0ac \ud559\uc790\ub4e4\uc740 \uc774\ub7ec\ud55c \ud654\ud559\uc801 \uba85\uba85\ubc95 \uac1c\uc815\uc740 \ud1b5\uc77c\ub41c \ud654\ud559\uc801 \uc5b8\uc5b4\ub97c \ub9cc\ub4dc\ub294 \ub370 \ub3c4\uc6c0\uc744 \uc8fc\uc5b4, \ud654\ud559\uc758 \ubc1c\uc804\uc5d0 \uae30\uc5ec\ud558\uc600\ub2e4\uace0 \ud558\uc5ec \ud070 \uc758\uc758\ub97c \ub450\uace0 \uc788\ub2e4.", "sentence_answer": "\uc774\ub97c \ud574\uacb0\ud558\uae30 \uc704\ud574\uc11c \ub77c\ubd80\uc544\uc9c0\uc5d0\u00b7\ud074\ub85c\ub4dc \ub8e8\uc774 \ubca0\ub974\ud1a8\ub808\u00b7\ub8e8\uc774 \ubca0\ub974\ub098\ub974 \uae30\ud1b5 \ub4dc \ubaa8\ub974\ubcf4\u00b7\uc559\ud22c\uc548 \ud504\ub791\uc218\uc544 \ud478\ub974\ud06c\ub8e8\uc544 \ub4f1\uc774 \uc8fc\ucd95\uc774 \ub418\uc5b4 \ud654\ud559\uc801 \uba85\uba85\ubc95 \uc744 \uac1c\uc815\ud558\uc600\uace0, \uc774\ub97c 1787\ub144\uc5d0 M\u00e9thode de nomenclature chimique\ub77c\ub294 \uc774\ub984\uc73c\ub85c \ub0b4\ub193\uc558\ub2e4.", "paragraph_id": "6540101-11-0", "paragraph_question": "question: \ub77c\ubd80\uc544\uc9c0\uc5d0 \ud074\ub85c\ub4dc \ub8e8\uc774 \ubca0\ub974\ud1a8\ub808 \ub4f1\uc774 \ud568\uaed8 \uac1c\uc815\ud55c \ubc95\uc758 \uc774\ub984\uc740 \ubb34\uc5c7\uc77c\uae4c?, context: \uc774\uc804\uc5d0\ub294 \ud654\ud569\ubb3c\uc758 \uc774\ub984\uc744 \ubd99\uc774\ub294 \ub370 \uc788\uc5b4\uc11c \uae30\uc900\uc774 \uc81c\uac01\uac01\uc774\ub77c, \uc774\ub984\uc774 \ub9e4\uc6b0 \ud63c\ub780\uc2a4\ub7ec\uc6b8 \uc218\ubc16\uc5d0 \uc5c6\uc5c8\ub2e4. \uc774\ub97c \ud574\uacb0\ud558\uae30 \uc704\ud574\uc11c \ub77c\ubd80\uc544\uc9c0\uc5d0\u00b7\ud074\ub85c\ub4dc \ub8e8\uc774 \ubca0\ub974\ud1a8\ub808\u00b7\ub8e8\uc774 \ubca0\ub974\ub098\ub974 \uae30\ud1b5 \ub4dc \ubaa8\ub974\ubcf4\u00b7\uc559\ud22c\uc548 \ud504\ub791\uc218\uc544 \ud478\ub974\ud06c\ub8e8\uc544 \ub4f1\uc774 \uc8fc\ucd95\uc774 \ub418\uc5b4 \ud654\ud559\uc801 \uba85\uba85\ubc95\uc744 \uac1c\uc815\ud558\uc600\uace0, \uc774\ub97c 1787\ub144\uc5d0 M\u00e9thode de nomenclature chimique\ub77c\ub294 \uc774\ub984\uc73c\ub85c \ub0b4\ub193\uc558\ub2e4. \uc774\ub4e4\uc740 \ud654\ud559 \ubb3c\uc9c8\uc740 \ud658\uacbd\uc5d0 \ub530\ub77c\uc11c \ubcc0\ud558\uc9c0 \uc54a\ub294 \uc774\ub984\uc744 \uac00\uc9c0\uace0, \uc774\ub984\uc740 \uadf8 \ubb3c\uc9c8\uc758 \uc131\uc9c8\uc744 \uc798 \ub4dc\ub7ec\ub0b4\uba70, \uc774\ub984\uc740 \uadf8\ub9ac\uc2a4\uc5b4 \ub610\ub294 \ub77c\ud2f4\uc5b4\uc5d0 \uadfc\uac04\uc744 \ub454\ub2e4\ub294 \ub4f1\uc758 \uc6d0\uce59\uc5d0 \uae30\ubc18\uc744 \ub450\uc5b4 \ud654\ud559 \ubb3c\uc9c8\ub4e4\uc744 \uc0c8\ub86d\uac8c \uba85\uba85\ud558\uc600\ub2e4. \uc774\ud6c4 \ub9c8\ub974\ud2f4 \ud558\uc778\ub9ac\ud788 \ud074\ub77c\ud504\ub85c\ud2b8\ub97c \uc8fc\ucd95\uc73c\ub85c \ud55c \ub3c5\uc77c\uc758 \ud654\ud559\uc790, \uadf8\ub9ac\uace0 \uc790\uc2e0\uc758 \uad00\uce21\uc744 \ub354 \uc911\uc2dc\ud588\ub358 \ud5e8\ub9ac \uce90\ubc88\ub514\uc2dc\uc640 \uc608\uc804\uc758 \uc774\ub860\uc5d0 \uc9d1\ucc29\ud588\ub358 \uc870\uc9c0\ud504 \ud504\ub9ac\uc2a4\ud2c0\ub9ac\ub97c \uc81c\uc678\ud55c \ub300\ubd80\ubd84\uc758 \uc601\uad6d\uc758 \ud654\ud559\uc790\ub3c4 \ud504\ub791\uc2a4 \ud654\ud559\uc790\uc758 \uc6c0\uc9c1\uc784\uc5d0 \ub3d9\ucc38\ud558\uc600\ub2e4. M\u00e9thode de nomenclature chimique\uc740 \ud504\ub791\uc2a4\ubfd0\ub9cc\uc774 \uc544\ub2c8\ub77c, \ub3c5\uc77c\u00b7\uc601\uad6d\u00b7\uc2a4\ud398\uc778\u00b7\uc774\ud0c8\ub9ac\uc544 \ub4f1 \uc720\ub7fd \uac01\uad6d\uc5d0\uc11c \ubc88\uc5ed\ub418\uc5b4 \uc0ac\uc6a9\ub418\uc5c8\ub2e4. \uacfc\ud559\uc0ac \ud559\uc790\ub4e4\uc740 \uc774\ub7ec\ud55c \ud654\ud559\uc801 \uba85\uba85\ubc95 \uac1c\uc815\uc740 \ud1b5\uc77c\ub41c \ud654\ud559\uc801 \uc5b8\uc5b4\ub97c \ub9cc\ub4dc\ub294 \ub370 \ub3c4\uc6c0\uc744 \uc8fc\uc5b4, \ud654\ud559\uc758 \ubc1c\uc804\uc5d0 \uae30\uc5ec\ud558\uc600\ub2e4\uace0 \ud558\uc5ec \ud070 \uc758\uc758\ub97c \ub450\uace0 \uc788\ub2e4."} {"question": "\ubc38\ub7ec\ub9ac \uc194\ub77c\ub098\uc2a4\uac00 \ube4c\ub9ac\uc9c0 \ubcf4\uc774\uc2a4\uc5d0\uc11c \uc790\uc11c\uc804\uc744 \uc4f0\uaca0\ub2e4\uace0 \uc778\ud130\ubdf0\ud55c \ud574\ub294?", "paragraph": "\uc194\ub77c\ub098\uc2a4\ub294 \uc790\uc11c\uc804\uc744 \uc4f8 \uc0dd\uac01\ub3c4 \ud588\uc5c8\ub358 \uac83\uc73c\ub85c \ubcf4\uc778\ub2e4. 1977\ub144 \u300a\ube4c\ub9ac\uc9c0 \ubcf4\uc774\uc2a4\u300b\uc640\uc758 \uc778\ud130\ubdf0\uc5d0\uc11c, \uc194\ub77c\ub098\uc2a4\ub294 \uc790\uae30 \uc774\ub984\uc744 \uc81c\ubaa9\uc73c\ub85c \ubd99\uc778 \ucc45\uc744 \uc4f8 \uac70\ub77c\uace0 \uc120\uc5b8\ud588\ub2e4. \uc774 \ucc45\uc740 \uc5b4\uca4c\uba74 \ud328\ub7ec\ub514 \ucc45\uc774\uba70 \uc790\uae30\uac00 \uc218\uac10\ub418\uac8c \ub41c \uac83\uc744 \uc74c\ubaa8\ub860\uc801\uc73c\ub85c \ub2e4\ub8e8\ub294 \uac83\uc77c \uc218 \uc788\ub2e4\uace0 \uc0dd\uac01\ub418\uc5c8\ub2e4. \uadf8\ub7f0\ub370 \ub098\uc911\uc5d0 1977\ub144 \u300a\ube4c\ub9ac\uc9c0 \ubcf4\uc774\uc2a4\u300b \uba74\ub2f4 \ub0b4\uc6a9\uc744 \ubc14\ub85c\uc7a1\uc73c\uba74\uc11c \uc194\ub77c\ub098\uc2a4\ub294 \ucc45\uc774 \uc77c\ubd80\ubd84 \uc790\uc804\uc801\uc77c \uc218 \uc788\uc73c\ub098 \uc790\uc11c\uc804\uc740 \uc544\ub2d0 \uac83\uc774\uba70 \ub9ce\uc740 \uac83\uc744 \ub2e4\ub8f0 \uac83\uc774\ub77c\uace0 \ub9d0\ud588\ub2e4. \ub2e4\ub8f0 \ub0b4\uc6a9\uc73c\ub85c\ub294 \uc120\uc5b8\uc11c\uc758 \uc131\uba85\ub4e4\uc744 \uc99d\uba85\ud558\ub294 \uac83\uacfc, \u201c\uac1c\ud5db\uc18c\ub9ac\ub4e4\uc5d0 \ub300\ud55c \ub9e4\uc6b0 \ucca0\uc800\ud55c \ub300\ucc98(deal very intensively with the subject of bullshit)\u201d \ub4f1\uc774 \uc788\ub2e4 \ud588\ub294\ub370, \ud328\ub7ec\ub514\uc5d0 \uad00\ud574\uc11c\ub294 \uc544\ubb34 \ub9d0\ub3c4 \ud558\uc9c0 \uc54a\uc558\ub2e4.", "answer": "1977\ub144", "sentence": "1977\ub144 \u300a\ube4c\ub9ac\uc9c0 \ubcf4\uc774\uc2a4\u300b\uc640\uc758 \uc778\ud130\ubdf0\uc5d0\uc11c, \uc194\ub77c\ub098\uc2a4\ub294 \uc790\uae30 \uc774\ub984\uc744 \uc81c\ubaa9\uc73c\ub85c \ubd99\uc778 \ucc45\uc744 \uc4f8 \uac70\ub77c\uace0 \uc120\uc5b8\ud588\ub2e4.", "paragraph_sentence": "\uc194\ub77c\ub098\uc2a4\ub294 \uc790\uc11c\uc804\uc744 \uc4f8 \uc0dd\uac01\ub3c4 \ud588\uc5c8\ub358 \uac83\uc73c\ub85c \ubcf4\uc778\ub2e4. 1977\ub144 \u300a\ube4c\ub9ac\uc9c0 \ubcf4\uc774\uc2a4\u300b\uc640\uc758 \uc778\ud130\ubdf0\uc5d0\uc11c, \uc194\ub77c\ub098\uc2a4\ub294 \uc790\uae30 \uc774\ub984\uc744 \uc81c\ubaa9\uc73c\ub85c \ubd99\uc778 \ucc45\uc744 \uc4f8 \uac70\ub77c\uace0 \uc120\uc5b8\ud588\ub2e4. \uc774 \ucc45\uc740 \uc5b4\uca4c\uba74 \ud328\ub7ec\ub514 \ucc45\uc774\uba70 \uc790\uae30\uac00 \uc218\uac10\ub418\uac8c \ub41c \uac83\uc744 \uc74c\ubaa8\ub860\uc801\uc73c\ub85c \ub2e4\ub8e8\ub294 \uac83\uc77c \uc218 \uc788\ub2e4\uace0 \uc0dd\uac01\ub418\uc5c8\ub2e4. \uadf8\ub7f0\ub370 \ub098\uc911\uc5d0 1977\ub144 \u300a\ube4c\ub9ac\uc9c0 \ubcf4\uc774\uc2a4\u300b \uba74\ub2f4 \ub0b4\uc6a9\uc744 \ubc14\ub85c\uc7a1\uc73c\uba74\uc11c \uc194\ub77c\ub098\uc2a4\ub294 \ucc45\uc774 \uc77c\ubd80\ubd84 \uc790\uc804\uc801\uc77c \uc218 \uc788\uc73c\ub098 \uc790\uc11c\uc804\uc740 \uc544\ub2d0 \uac83\uc774\uba70 \ub9ce\uc740 \uac83\uc744 \ub2e4\ub8f0 \uac83\uc774\ub77c\uace0 \ub9d0\ud588\ub2e4. \ub2e4\ub8f0 \ub0b4\uc6a9\uc73c\ub85c\ub294 \uc120\uc5b8\uc11c\uc758 \uc131\uba85\ub4e4\uc744 \uc99d\uba85\ud558\ub294 \uac83\uacfc, \u201c\uac1c\ud5db\uc18c\ub9ac\ub4e4\uc5d0 \ub300\ud55c \ub9e4\uc6b0 \ucca0\uc800\ud55c \ub300\ucc98(deal very intensively with the subject of bullshit)\u201d \ub4f1\uc774 \uc788\ub2e4 \ud588\ub294\ub370, \ud328\ub7ec\ub514\uc5d0 \uad00\ud574\uc11c\ub294 \uc544\ubb34 \ub9d0\ub3c4 \ud558\uc9c0 \uc54a\uc558\ub2e4.", "paragraph_answer": "\uc194\ub77c\ub098\uc2a4\ub294 \uc790\uc11c\uc804\uc744 \uc4f8 \uc0dd\uac01\ub3c4 \ud588\uc5c8\ub358 \uac83\uc73c\ub85c \ubcf4\uc778\ub2e4. 1977\ub144 \u300a\ube4c\ub9ac\uc9c0 \ubcf4\uc774\uc2a4\u300b\uc640\uc758 \uc778\ud130\ubdf0\uc5d0\uc11c, \uc194\ub77c\ub098\uc2a4\ub294 \uc790\uae30 \uc774\ub984\uc744 \uc81c\ubaa9\uc73c\ub85c \ubd99\uc778 \ucc45\uc744 \uc4f8 \uac70\ub77c\uace0 \uc120\uc5b8\ud588\ub2e4. \uc774 \ucc45\uc740 \uc5b4\uca4c\uba74 \ud328\ub7ec\ub514 \ucc45\uc774\uba70 \uc790\uae30\uac00 \uc218\uac10\ub418\uac8c \ub41c \uac83\uc744 \uc74c\ubaa8\ub860\uc801\uc73c\ub85c \ub2e4\ub8e8\ub294 \uac83\uc77c \uc218 \uc788\ub2e4\uace0 \uc0dd\uac01\ub418\uc5c8\ub2e4. \uadf8\ub7f0\ub370 \ub098\uc911\uc5d0 1977\ub144 \u300a\ube4c\ub9ac\uc9c0 \ubcf4\uc774\uc2a4\u300b \uba74\ub2f4 \ub0b4\uc6a9\uc744 \ubc14\ub85c\uc7a1\uc73c\uba74\uc11c \uc194\ub77c\ub098\uc2a4\ub294 \ucc45\uc774 \uc77c\ubd80\ubd84 \uc790\uc804\uc801\uc77c \uc218 \uc788\uc73c\ub098 \uc790\uc11c\uc804\uc740 \uc544\ub2d0 \uac83\uc774\uba70 \ub9ce\uc740 \uac83\uc744 \ub2e4\ub8f0 \uac83\uc774\ub77c\uace0 \ub9d0\ud588\ub2e4. \ub2e4\ub8f0 \ub0b4\uc6a9\uc73c\ub85c\ub294 \uc120\uc5b8\uc11c\uc758 \uc131\uba85\ub4e4\uc744 \uc99d\uba85\ud558\ub294 \uac83\uacfc, \u201c\uac1c\ud5db\uc18c\ub9ac\ub4e4\uc5d0 \ub300\ud55c \ub9e4\uc6b0 \ucca0\uc800\ud55c \ub300\ucc98(deal very intensively with the subject of bullshit)\u201d \ub4f1\uc774 \uc788\ub2e4 \ud588\ub294\ub370, \ud328\ub7ec\ub514\uc5d0 \uad00\ud574\uc11c\ub294 \uc544\ubb34 \ub9d0\ub3c4 \ud558\uc9c0 \uc54a\uc558\ub2e4.", "sentence_answer": " 1977\ub144 \u300a\ube4c\ub9ac\uc9c0 \ubcf4\uc774\uc2a4\u300b\uc640\uc758 \uc778\ud130\ubdf0\uc5d0\uc11c, \uc194\ub77c\ub098\uc2a4\ub294 \uc790\uae30 \uc774\ub984\uc744 \uc81c\ubaa9\uc73c\ub85c \ubd99\uc778 \ucc45\uc744 \uc4f8 \uac70\ub77c\uace0 \uc120\uc5b8\ud588\ub2e4.", "paragraph_id": "6535070-11-0", "paragraph_question": "question: \ubc38\ub7ec\ub9ac \uc194\ub77c\ub098\uc2a4\uac00 \ube4c\ub9ac\uc9c0 \ubcf4\uc774\uc2a4\uc5d0\uc11c \uc790\uc11c\uc804\uc744 \uc4f0\uaca0\ub2e4\uace0 \uc778\ud130\ubdf0\ud55c \ud574\ub294?, context: \uc194\ub77c\ub098\uc2a4\ub294 \uc790\uc11c\uc804\uc744 \uc4f8 \uc0dd\uac01\ub3c4 \ud588\uc5c8\ub358 \uac83\uc73c\ub85c \ubcf4\uc778\ub2e4. 1977\ub144 \u300a\ube4c\ub9ac\uc9c0 \ubcf4\uc774\uc2a4\u300b\uc640\uc758 \uc778\ud130\ubdf0\uc5d0\uc11c, \uc194\ub77c\ub098\uc2a4\ub294 \uc790\uae30 \uc774\ub984\uc744 \uc81c\ubaa9\uc73c\ub85c \ubd99\uc778 \ucc45\uc744 \uc4f8 \uac70\ub77c\uace0 \uc120\uc5b8\ud588\ub2e4. \uc774 \ucc45\uc740 \uc5b4\uca4c\uba74 \ud328\ub7ec\ub514 \ucc45\uc774\uba70 \uc790\uae30\uac00 \uc218\uac10\ub418\uac8c \ub41c \uac83\uc744 \uc74c\ubaa8\ub860\uc801\uc73c\ub85c \ub2e4\ub8e8\ub294 \uac83\uc77c \uc218 \uc788\ub2e4\uace0 \uc0dd\uac01\ub418\uc5c8\ub2e4. \uadf8\ub7f0\ub370 \ub098\uc911\uc5d0 1977\ub144 \u300a\ube4c\ub9ac\uc9c0 \ubcf4\uc774\uc2a4\u300b \uba74\ub2f4 \ub0b4\uc6a9\uc744 \ubc14\ub85c\uc7a1\uc73c\uba74\uc11c \uc194\ub77c\ub098\uc2a4\ub294 \ucc45\uc774 \uc77c\ubd80\ubd84 \uc790\uc804\uc801\uc77c \uc218 \uc788\uc73c\ub098 \uc790\uc11c\uc804\uc740 \uc544\ub2d0 \uac83\uc774\uba70 \ub9ce\uc740 \uac83\uc744 \ub2e4\ub8f0 \uac83\uc774\ub77c\uace0 \ub9d0\ud588\ub2e4. \ub2e4\ub8f0 \ub0b4\uc6a9\uc73c\ub85c\ub294 \uc120\uc5b8\uc11c\uc758 \uc131\uba85\ub4e4\uc744 \uc99d\uba85\ud558\ub294 \uac83\uacfc, \u201c\uac1c\ud5db\uc18c\ub9ac\ub4e4\uc5d0 \ub300\ud55c \ub9e4\uc6b0 \ucca0\uc800\ud55c \ub300\ucc98(deal very intensively with the subject of bullshit)\u201d \ub4f1\uc774 \uc788\ub2e4 \ud588\ub294\ub370, \ud328\ub7ec\ub514\uc5d0 \uad00\ud574\uc11c\ub294 \uc544\ubb34 \ub9d0\ub3c4 \ud558\uc9c0 \uc54a\uc558\ub2e4."} {"question": "1951\ub144 2\uc6d4 2\uc77c \uc5f4\ub9b0 \uacbd\uae30\uc5d0\uc11c \ud504\ub791\uc2a4\uc758 \uacbd\uae30\uacb0\uacfc\ub294?", "paragraph": "1951\ub144 2\uc6d4 2\uc77c, \ud504\ub791\uc2a4\ub294 \uc720\uace0\uc2ac\ub77c\ube44\uc544\uc5d0 2-1\ub85c \uc2b9\ub9ac\ud574 \uc124\uc695\ud558\uc600\ub2e4. 1950\ub144\ub300\uc5d0 \ub4e4\uc5b4 \uc950\uc2a4\ud2b8 \ud401\ud150, \ub808\ubabd \ucf54\ud30c, \uc7a5 \ubc45\uc0c1, \ub85c\ubca0\ub974 \uc885\ucf00, \ub9c8\ub7c9 \uc704\uc2a4\ub2c8\uc5d0\uc2a4\ud0a4, \ud0c0\ub370 \uc2dc\uc18c\uc6b0\uc2a4\ud0a4, \uadf8\ub9ac\uace0 \uc544\ub974\ub9dd \ud321\ubca0\ub978\uc774 \uad6d\uac00\ub300\ud45c\ud300\uc774 \ubc1c\ud0c1\ub418\uc5c8\ub2e4. \ub2e4\uc218\uc758 \uc120\uc218\ub4e4\uc740 \ub9b4, \uc0dd\ud14c\ud2f0\uc5d4, \uadf8\ub9ac\uace0 \uc2a4\ud0c0\ub4dc \ub7ad\uc2a4\uc640 \uac19\uc740 \ud504\ub791\uc2a4 \ud074\ub7fd\uc758 \uc120\uc218\ub4e4\ub85c \uad6c\uc131\ub418\uc5b4 \uc788\uc5c8\ub2e4. \uc7a5 \ubc14\ub77c\ud2b8 \uc8fc\uc7a5\uacfc \ub85c\uc81c \ub9c8\uc288\uc758 \uc9c0\ud718 \ud558\uc5d0, \ud504\ub791\uc2a4\ub294 1952\ub144 4\uc6d4 20\uc77c \ud3ec\ub974\ud22c\uac08\uc804 2-0 \uc2b9\ub9ac\ub97c \uc2dc\uc791\uc73c\ub85c 6\uacbd\uae30 \ubb34\ud328 \ud589\uc9c4\uc744 \ubc8c\uc600\ub2e4. \uc774 \uae30\uac04\ub3d9\uc548, \ud504\ub791\uc2a4\ub294 \uc11c\ub3c5\uc744 \uc0c1\ub300\ud574 3-1\ub85c \uc774\uae30\uae30\ub3c4 \ud588\ub2e4. \ubb34\ud328 \ud589\uc9c4\uc740 1952\ub144 \uc131\ud0c4\uc808\uc5d0 \ubca8\uae30\uc5d0\ud55c\ud14c 0-1\ub85c \ud328\ud558\uba74\uc11c \ub05d\ub0ac\ub2e4. \ud504\ub791\uc2a4\ub294 1952\ub144 FIFA \uc6d4\ub4dc\ucef5 \uc608\uc120\uc804\uc5d0\uc11c \uc870 \uc0c1\ub300\uc778 \uc544\uc77c\ub79c\ub4dc\uc640 \ub8e9\uc148\ubd80\ub974\ud06c\ub97c \uc0c1\ub300\ub85c \uc21c\ud56d\ud558\uc600\ub2e4. \ud504\ub791\uc2a4\ub294 \uc608\uc120 \uc870\uc5d0\uc11c \ubb34\ud328\ub97c \uae30\ub85d\ud558\uc600\uace0, 20\uace8\uc744 \ub4dd\uc810\ud558\uace0 \ub2e8 4\uc2e4\uc810\ub9cc \ud5c8\uc6a9\ud558\uc600\ub2e4. \uc870\ubcc4 \ub9ac\uadf8\uc81c\uac00 \ubd80\ud65c\ud55c \uc2a4\uc704\uc2a4\uc5d0\uc11c \uc5f4\ub9b0 FIFA \uc6d4\ub4dc\ucef5 \ubcf8\uc120\uc5d0\uc11c, \ud504\ub791\uc2a4\ub294 \ube0c\ub77c\uc9c8, \uc720\uace0\uc2ac\ub77c\ube44\uc544, \uadf8\ub9ac\uace0 \uba55\uc2dc\ucf54\uc640 \ud55c \uc870\uc5d0 \ud3b8\uc131\ub418\uc5c8\uc73c\ub098, \ud6c4\uc790\uc758 2\ud300\ub9cc \uc0c1\ub300\ud558\uc600\ub2e4. \uc870\ubcc4 \ub9ac\uadf8 1\ucc28\uc804\uc5d0\uc11c \ud504\ub791\uc2a4\ub294 \uc720\uace0\uc2ac\ub77c\ube44\uc544\uc5d0\uac8c 0-1\ub85c \uc84c\ub2e4. \uadf8\ub7ec\ub098 \uba55\uc2dc\ucf54\uc640\uc758 2\ucc28\uc804\uc5d0\uc11c\ub294 3-2 \uc2b9\ub9ac\ub97c \uac70\ub450\uc5c8\uc73c\ub098, \uc720\uace0\uc2ac\ub77c\ube44\uc544\uc640\uc758 \uc0c1\ub300 \uc804\uc801\uc5d0\uc11c \ubc00\ub9ac\ub294 \ubc14\ub78c\uc5d0 8\uac15 \uc9c4\ucd9c\uad8c\uc744 \ub0b4\uc8fc\uc5c8\ub2e4.", "answer": "2-1", "sentence": "1951\ub144 2\uc6d4 2\uc77c, \ud504\ub791\uc2a4\ub294 \uc720\uace0\uc2ac\ub77c\ube44\uc544\uc5d0 2-1 \ub85c \uc2b9\ub9ac\ud574 \uc124\uc695\ud558\uc600\ub2e4.", "paragraph_sentence": " 1951\ub144 2\uc6d4 2\uc77c, \ud504\ub791\uc2a4\ub294 \uc720\uace0\uc2ac\ub77c\ube44\uc544\uc5d0 2-1 \ub85c \uc2b9\ub9ac\ud574 \uc124\uc695\ud558\uc600\ub2e4. 1950\ub144\ub300\uc5d0 \ub4e4\uc5b4 \uc950\uc2a4\ud2b8 \ud401\ud150, \ub808\ubabd \ucf54\ud30c, \uc7a5 \ubc45\uc0c1, \ub85c\ubca0\ub974 \uc885\ucf00, \ub9c8\ub7c9 \uc704\uc2a4\ub2c8\uc5d0\uc2a4\ud0a4, \ud0c0\ub370 \uc2dc\uc18c\uc6b0\uc2a4\ud0a4, \uadf8\ub9ac\uace0 \uc544\ub974\ub9dd \ud321\ubca0\ub978\uc774 \uad6d\uac00\ub300\ud45c\ud300\uc774 \ubc1c\ud0c1\ub418\uc5c8\ub2e4. \ub2e4\uc218\uc758 \uc120\uc218\ub4e4\uc740 \ub9b4, \uc0dd\ud14c\ud2f0\uc5d4, \uadf8\ub9ac\uace0 \uc2a4\ud0c0\ub4dc \ub7ad\uc2a4\uc640 \uac19\uc740 \ud504\ub791\uc2a4 \ud074\ub7fd\uc758 \uc120\uc218\ub4e4\ub85c \uad6c\uc131\ub418\uc5b4 \uc788\uc5c8\ub2e4. \uc7a5 \ubc14\ub77c\ud2b8 \uc8fc\uc7a5\uacfc \ub85c\uc81c \ub9c8\uc288\uc758 \uc9c0\ud718 \ud558\uc5d0, \ud504\ub791\uc2a4\ub294 1952\ub144 4\uc6d4 20\uc77c \ud3ec\ub974\ud22c\uac08\uc804 2-0 \uc2b9\ub9ac\ub97c \uc2dc\uc791\uc73c\ub85c 6\uacbd\uae30 \ubb34\ud328 \ud589\uc9c4\uc744 \ubc8c\uc600\ub2e4. \uc774 \uae30\uac04\ub3d9\uc548, \ud504\ub791\uc2a4\ub294 \uc11c\ub3c5\uc744 \uc0c1\ub300\ud574 3-1\ub85c \uc774\uae30\uae30\ub3c4 \ud588\ub2e4. \ubb34\ud328 \ud589\uc9c4\uc740 1952\ub144 \uc131\ud0c4\uc808\uc5d0 \ubca8\uae30\uc5d0\ud55c\ud14c 0-1\ub85c \ud328\ud558\uba74\uc11c \ub05d\ub0ac\ub2e4. \ud504\ub791\uc2a4\ub294 1952\ub144 FIFA \uc6d4\ub4dc\ucef5 \uc608\uc120\uc804\uc5d0\uc11c \uc870 \uc0c1\ub300\uc778 \uc544\uc77c\ub79c\ub4dc\uc640 \ub8e9\uc148\ubd80\ub974\ud06c\ub97c \uc0c1\ub300\ub85c \uc21c\ud56d\ud558\uc600\ub2e4. \ud504\ub791\uc2a4\ub294 \uc608\uc120 \uc870\uc5d0\uc11c \ubb34\ud328\ub97c \uae30\ub85d\ud558\uc600\uace0, 20\uace8\uc744 \ub4dd\uc810\ud558\uace0 \ub2e8 4\uc2e4\uc810\ub9cc \ud5c8\uc6a9\ud558\uc600\ub2e4. \uc870\ubcc4 \ub9ac\uadf8\uc81c\uac00 \ubd80\ud65c\ud55c \uc2a4\uc704\uc2a4\uc5d0\uc11c \uc5f4\ub9b0 FIFA \uc6d4\ub4dc\ucef5 \ubcf8\uc120\uc5d0\uc11c, \ud504\ub791\uc2a4\ub294 \ube0c\ub77c\uc9c8, \uc720\uace0\uc2ac\ub77c\ube44\uc544, \uadf8\ub9ac\uace0 \uba55\uc2dc\ucf54\uc640 \ud55c \uc870\uc5d0 \ud3b8\uc131\ub418\uc5c8\uc73c\ub098, \ud6c4\uc790\uc758 2\ud300\ub9cc \uc0c1\ub300\ud558\uc600\ub2e4. \uc870\ubcc4 \ub9ac\uadf8 1\ucc28\uc804\uc5d0\uc11c \ud504\ub791\uc2a4\ub294 \uc720\uace0\uc2ac\ub77c\ube44\uc544\uc5d0\uac8c 0-1\ub85c \uc84c\ub2e4. \uadf8\ub7ec\ub098 \uba55\uc2dc\ucf54\uc640\uc758 2\ucc28\uc804\uc5d0\uc11c\ub294 3-2 \uc2b9\ub9ac\ub97c \uac70\ub450\uc5c8\uc73c\ub098, \uc720\uace0\uc2ac\ub77c\ube44\uc544\uc640\uc758 \uc0c1\ub300 \uc804\uc801\uc5d0\uc11c \ubc00\ub9ac\ub294 \ubc14\ub78c\uc5d0 8\uac15 \uc9c4\ucd9c\uad8c\uc744 \ub0b4\uc8fc\uc5c8\ub2e4.", "paragraph_answer": "1951\ub144 2\uc6d4 2\uc77c, \ud504\ub791\uc2a4\ub294 \uc720\uace0\uc2ac\ub77c\ube44\uc544\uc5d0 2-1 \ub85c \uc2b9\ub9ac\ud574 \uc124\uc695\ud558\uc600\ub2e4. 1950\ub144\ub300\uc5d0 \ub4e4\uc5b4 \uc950\uc2a4\ud2b8 \ud401\ud150, \ub808\ubabd \ucf54\ud30c, \uc7a5 \ubc45\uc0c1, \ub85c\ubca0\ub974 \uc885\ucf00, \ub9c8\ub7c9 \uc704\uc2a4\ub2c8\uc5d0\uc2a4\ud0a4, \ud0c0\ub370 \uc2dc\uc18c\uc6b0\uc2a4\ud0a4, \uadf8\ub9ac\uace0 \uc544\ub974\ub9dd \ud321\ubca0\ub978\uc774 \uad6d\uac00\ub300\ud45c\ud300\uc774 \ubc1c\ud0c1\ub418\uc5c8\ub2e4. \ub2e4\uc218\uc758 \uc120\uc218\ub4e4\uc740 \ub9b4, \uc0dd\ud14c\ud2f0\uc5d4, \uadf8\ub9ac\uace0 \uc2a4\ud0c0\ub4dc \ub7ad\uc2a4\uc640 \uac19\uc740 \ud504\ub791\uc2a4 \ud074\ub7fd\uc758 \uc120\uc218\ub4e4\ub85c \uad6c\uc131\ub418\uc5b4 \uc788\uc5c8\ub2e4. \uc7a5 \ubc14\ub77c\ud2b8 \uc8fc\uc7a5\uacfc \ub85c\uc81c \ub9c8\uc288\uc758 \uc9c0\ud718 \ud558\uc5d0, \ud504\ub791\uc2a4\ub294 1952\ub144 4\uc6d4 20\uc77c \ud3ec\ub974\ud22c\uac08\uc804 2-0 \uc2b9\ub9ac\ub97c \uc2dc\uc791\uc73c\ub85c 6\uacbd\uae30 \ubb34\ud328 \ud589\uc9c4\uc744 \ubc8c\uc600\ub2e4. \uc774 \uae30\uac04\ub3d9\uc548, \ud504\ub791\uc2a4\ub294 \uc11c\ub3c5\uc744 \uc0c1\ub300\ud574 3-1\ub85c \uc774\uae30\uae30\ub3c4 \ud588\ub2e4. \ubb34\ud328 \ud589\uc9c4\uc740 1952\ub144 \uc131\ud0c4\uc808\uc5d0 \ubca8\uae30\uc5d0\ud55c\ud14c 0-1\ub85c \ud328\ud558\uba74\uc11c \ub05d\ub0ac\ub2e4. \ud504\ub791\uc2a4\ub294 1952\ub144 FIFA \uc6d4\ub4dc\ucef5 \uc608\uc120\uc804\uc5d0\uc11c \uc870 \uc0c1\ub300\uc778 \uc544\uc77c\ub79c\ub4dc\uc640 \ub8e9\uc148\ubd80\ub974\ud06c\ub97c \uc0c1\ub300\ub85c \uc21c\ud56d\ud558\uc600\ub2e4. \ud504\ub791\uc2a4\ub294 \uc608\uc120 \uc870\uc5d0\uc11c \ubb34\ud328\ub97c \uae30\ub85d\ud558\uc600\uace0, 20\uace8\uc744 \ub4dd\uc810\ud558\uace0 \ub2e8 4\uc2e4\uc810\ub9cc \ud5c8\uc6a9\ud558\uc600\ub2e4. \uc870\ubcc4 \ub9ac\uadf8\uc81c\uac00 \ubd80\ud65c\ud55c \uc2a4\uc704\uc2a4\uc5d0\uc11c \uc5f4\ub9b0 FIFA \uc6d4\ub4dc\ucef5 \ubcf8\uc120\uc5d0\uc11c, \ud504\ub791\uc2a4\ub294 \ube0c\ub77c\uc9c8, \uc720\uace0\uc2ac\ub77c\ube44\uc544, \uadf8\ub9ac\uace0 \uba55\uc2dc\ucf54\uc640 \ud55c \uc870\uc5d0 \ud3b8\uc131\ub418\uc5c8\uc73c\ub098, \ud6c4\uc790\uc758 2\ud300\ub9cc \uc0c1\ub300\ud558\uc600\ub2e4. \uc870\ubcc4 \ub9ac\uadf8 1\ucc28\uc804\uc5d0\uc11c \ud504\ub791\uc2a4\ub294 \uc720\uace0\uc2ac\ub77c\ube44\uc544\uc5d0\uac8c 0-1\ub85c \uc84c\ub2e4. \uadf8\ub7ec\ub098 \uba55\uc2dc\ucf54\uc640\uc758 2\ucc28\uc804\uc5d0\uc11c\ub294 3-2 \uc2b9\ub9ac\ub97c \uac70\ub450\uc5c8\uc73c\ub098, \uc720\uace0\uc2ac\ub77c\ube44\uc544\uc640\uc758 \uc0c1\ub300 \uc804\uc801\uc5d0\uc11c \ubc00\ub9ac\ub294 \ubc14\ub78c\uc5d0 8\uac15 \uc9c4\ucd9c\uad8c\uc744 \ub0b4\uc8fc\uc5c8\ub2e4.", "sentence_answer": "1951\ub144 2\uc6d4 2\uc77c, \ud504\ub791\uc2a4\ub294 \uc720\uace0\uc2ac\ub77c\ube44\uc544\uc5d0 2-1 \ub85c \uc2b9\ub9ac\ud574 \uc124\uc695\ud558\uc600\ub2e4.", "paragraph_id": "6355377-7-0", "paragraph_question": "question: 1951\ub144 2\uc6d4 2\uc77c \uc5f4\ub9b0 \uacbd\uae30\uc5d0\uc11c \ud504\ub791\uc2a4\uc758 \uacbd\uae30\uacb0\uacfc\ub294?, context: 1951\ub144 2\uc6d4 2\uc77c, \ud504\ub791\uc2a4\ub294 \uc720\uace0\uc2ac\ub77c\ube44\uc544\uc5d0 2-1\ub85c \uc2b9\ub9ac\ud574 \uc124\uc695\ud558\uc600\ub2e4. 1950\ub144\ub300\uc5d0 \ub4e4\uc5b4 \uc950\uc2a4\ud2b8 \ud401\ud150, \ub808\ubabd \ucf54\ud30c, \uc7a5 \ubc45\uc0c1, \ub85c\ubca0\ub974 \uc885\ucf00, \ub9c8\ub7c9 \uc704\uc2a4\ub2c8\uc5d0\uc2a4\ud0a4, \ud0c0\ub370 \uc2dc\uc18c\uc6b0\uc2a4\ud0a4, \uadf8\ub9ac\uace0 \uc544\ub974\ub9dd \ud321\ubca0\ub978\uc774 \uad6d\uac00\ub300\ud45c\ud300\uc774 \ubc1c\ud0c1\ub418\uc5c8\ub2e4. \ub2e4\uc218\uc758 \uc120\uc218\ub4e4\uc740 \ub9b4, \uc0dd\ud14c\ud2f0\uc5d4, \uadf8\ub9ac\uace0 \uc2a4\ud0c0\ub4dc \ub7ad\uc2a4\uc640 \uac19\uc740 \ud504\ub791\uc2a4 \ud074\ub7fd\uc758 \uc120\uc218\ub4e4\ub85c \uad6c\uc131\ub418\uc5b4 \uc788\uc5c8\ub2e4. \uc7a5 \ubc14\ub77c\ud2b8 \uc8fc\uc7a5\uacfc \ub85c\uc81c \ub9c8\uc288\uc758 \uc9c0\ud718 \ud558\uc5d0, \ud504\ub791\uc2a4\ub294 1952\ub144 4\uc6d4 20\uc77c \ud3ec\ub974\ud22c\uac08\uc804 2-0 \uc2b9\ub9ac\ub97c \uc2dc\uc791\uc73c\ub85c 6\uacbd\uae30 \ubb34\ud328 \ud589\uc9c4\uc744 \ubc8c\uc600\ub2e4. \uc774 \uae30\uac04\ub3d9\uc548, \ud504\ub791\uc2a4\ub294 \uc11c\ub3c5\uc744 \uc0c1\ub300\ud574 3-1\ub85c \uc774\uae30\uae30\ub3c4 \ud588\ub2e4. \ubb34\ud328 \ud589\uc9c4\uc740 1952\ub144 \uc131\ud0c4\uc808\uc5d0 \ubca8\uae30\uc5d0\ud55c\ud14c 0-1\ub85c \ud328\ud558\uba74\uc11c \ub05d\ub0ac\ub2e4. \ud504\ub791\uc2a4\ub294 1952\ub144 FIFA \uc6d4\ub4dc\ucef5 \uc608\uc120\uc804\uc5d0\uc11c \uc870 \uc0c1\ub300\uc778 \uc544\uc77c\ub79c\ub4dc\uc640 \ub8e9\uc148\ubd80\ub974\ud06c\ub97c \uc0c1\ub300\ub85c \uc21c\ud56d\ud558\uc600\ub2e4. \ud504\ub791\uc2a4\ub294 \uc608\uc120 \uc870\uc5d0\uc11c \ubb34\ud328\ub97c \uae30\ub85d\ud558\uc600\uace0, 20\uace8\uc744 \ub4dd\uc810\ud558\uace0 \ub2e8 4\uc2e4\uc810\ub9cc \ud5c8\uc6a9\ud558\uc600\ub2e4. \uc870\ubcc4 \ub9ac\uadf8\uc81c\uac00 \ubd80\ud65c\ud55c \uc2a4\uc704\uc2a4\uc5d0\uc11c \uc5f4\ub9b0 FIFA \uc6d4\ub4dc\ucef5 \ubcf8\uc120\uc5d0\uc11c, \ud504\ub791\uc2a4\ub294 \ube0c\ub77c\uc9c8, \uc720\uace0\uc2ac\ub77c\ube44\uc544, \uadf8\ub9ac\uace0 \uba55\uc2dc\ucf54\uc640 \ud55c \uc870\uc5d0 \ud3b8\uc131\ub418\uc5c8\uc73c\ub098, \ud6c4\uc790\uc758 2\ud300\ub9cc \uc0c1\ub300\ud558\uc600\ub2e4. \uc870\ubcc4 \ub9ac\uadf8 1\ucc28\uc804\uc5d0\uc11c \ud504\ub791\uc2a4\ub294 \uc720\uace0\uc2ac\ub77c\ube44\uc544\uc5d0\uac8c 0-1\ub85c \uc84c\ub2e4. \uadf8\ub7ec\ub098 \uba55\uc2dc\ucf54\uc640\uc758 2\ucc28\uc804\uc5d0\uc11c\ub294 3-2 \uc2b9\ub9ac\ub97c \uac70\ub450\uc5c8\uc73c\ub098, \uc720\uace0\uc2ac\ub77c\ube44\uc544\uc640\uc758 \uc0c1\ub300 \uc804\uc801\uc5d0\uc11c \ubc00\ub9ac\ub294 \ubc14\ub78c\uc5d0 8\uac15 \uc9c4\ucd9c\uad8c\uc744 \ub0b4\uc8fc\uc5c8\ub2e4."} {"question": "\ub098\uc77c\ub860\uc744 \uc591\ubd84\uc73c\ub85c \ubcc0\ud654\uc2dc\ud0a4\ub294 \ud6a8\uc18c\ub97c \ub9cc\ub4e4\uc5b4 \ub0b8\uac83\uc740?", "paragraph": "\uc801\uc751\uc740 \uc774\ub85c\uc6b4 \ud2b9\uc9d5\uc758 \ubc1c\ud604\uc740 \ucd09\uc9c4\ub418\uace0 \ubd88\ub9ac\ud55c \ud2b9\uc9d5\uc758 \ubc1c\ud604\uc740 \uc904\uc5b4\ub4e4\uba74\uc11c \uc774\ub8e8\uc5b4\uc9c4\ub2e4. \ubc15\ud14c\ub9ac\uc544\uac00 \ud56d\uc0dd\ubb3c\uc9c8\uc5d0 \uc801\uc751\ud558\uc5ec \uc9c4\ud654\ud558\ub294 \uac83\uc744 \uc608\ub85c \ub4e4 \uc218 \uc788\ub2e4. \ubc15\ud14c\ub9ac\uc544\ub294 \uc57d\ubb3c\uc774 \uac1c\uccb4\uc5d0 \uc8fc\ub294 \uc601\ud5a5\ub825\uc744 \uac10\uc18c\uc2dc\ud0a4\uace0 \uc57d\ubb3c \uc790\uccb4\ub97c \uccb4\uc678\ub85c \ub0b4\ubcf4\ub0bc \uc218 \uc788\ub294 \uad6c\uc870\ub85c \ubcc0\ud654\ud558\uc5ec \uacb0\uad6d \ud56d\uc0dd\uc81c \ub0b4\uc131\uc774 \uc0dd\uae30\uac8c \ub41c\ub2e4. \ubcf4\ub2e4 \uadf9\ub2e8\uc801\uc778 \uc0ac\ub840\ub4e4\ub3c4 \uc788\ub2e4. \ub300\uc7a5\uade0 \uc7a5\uae30 \uc9c4\ud654 \uc2e4\ud5d8\uc5d0\uc11c \ub300\uc7a5\uade0\uc740 \uc2dc\ud2b8\ub974\uc0b0\uc744 \uc591\ubd84\uc73c\ub85c \uc0ac\uc6a9\ud560 \uc218 \uc788\uc5c8\ub2e4. \uc2ec\uc9c0\uc5b4 \ud50c\ub77c\ubcf4\ubc15\ud14c\ub9ac\uc6c0\uc740 \ub098\uc77c\ub860\uc744 \uc591\ubd84\uc73c\ub85c \ubcc0\ud654\uc2dc\ud0a4\ub294 \ud6a8\uc18c\ub97c \ub9cc\ub4e4\uc5b4 \ub0b4\uc5c8\ub2e4. \ud1a0\uc591 \ubc15\ud14c\ub9ac\uc544\uc758 \uc77c\uc885\uc778 \uc2a4\ud551\uace0\ube44\uc6c0\uc740 \ub18d\uc57d\uc73c\ub85c \uc0ac\uc6a9\ub41c \ud39c\ud0c0\ud074\ub85c\ub85c\ud398\ub180\uc758 \ub3c5\uc131\uc744 \uc57d\ud654\uc2dc\ud0a4\uae30\ub294 \uc0c8\ub85c\uc6b4 \ub300\uc0ac\ud68c\ub85c\ub97c \uac16\ub294 \ud615\ud0dc\ub85c \uc801\uc751\ud558\uc600\ub2e4. \uc5ec\uc804\ud788 \ub17c\ub780\uc774 \uc788\uc73c\ub098 \uc720\uc804\uc790 \ub2e4\uc591\uc131\uc744 \ubc1c\ud604\uc2dc\ud0a4\ub294 \uc0dd\ubb3c\uc758 \ub2a5\ub825\uc774 \uc801\uc751\uc758 \uc131\uacf5\uacfc \ubc00\uc811\ud55c \uad00\ub828\uc774 \uc788\ub2e4\ub294 \uc774\ub860\uc774 \uc810\ucc28 \uc9c0\uc9c0\ub97c \uc5bb\uc5b4\uac00\uace0 \uc788\ub2e4. \uc5bc\ub9c8\ub098 \uc131\uacf5\uc801\uc73c\ub85c \uc801\uc751\ud560 \uc218 \uc788\ub294\uc9c0\ub97c \ub098\ud0c0\ub0b4\ub294 \ucc99\ub3c4\ub85c \uc9c4\ud654\ub3c4\ub780 \uac1c\ub150\uc774 \uc81c\uc2dc\ub418\uc5c8\ub2e4.", "answer": "\ud50c\ub77c\ubcf4\ubc15\ud14c\ub9ac\uc6c0", "sentence": "\uc2ec\uc9c0\uc5b4 \ud50c\ub77c\ubcf4\ubc15\ud14c\ub9ac\uc6c0 \uc740 \ub098\uc77c\ub860\uc744 \uc591\ubd84\uc73c\ub85c \ubcc0\ud654\uc2dc\ud0a4\ub294 \ud6a8\uc18c\ub97c \ub9cc\ub4e4\uc5b4 \ub0b4\uc5c8\ub2e4.", "paragraph_sentence": "\uc801\uc751\uc740 \uc774\ub85c\uc6b4 \ud2b9\uc9d5\uc758 \ubc1c\ud604\uc740 \ucd09\uc9c4\ub418\uace0 \ubd88\ub9ac\ud55c \ud2b9\uc9d5\uc758 \ubc1c\ud604\uc740 \uc904\uc5b4\ub4e4\uba74\uc11c \uc774\ub8e8\uc5b4\uc9c4\ub2e4. \ubc15\ud14c\ub9ac\uc544\uac00 \ud56d\uc0dd\ubb3c\uc9c8\uc5d0 \uc801\uc751\ud558\uc5ec \uc9c4\ud654\ud558\ub294 \uac83\uc744 \uc608\ub85c \ub4e4 \uc218 \uc788\ub2e4. \ubc15\ud14c\ub9ac\uc544\ub294 \uc57d\ubb3c\uc774 \uac1c\uccb4\uc5d0 \uc8fc\ub294 \uc601\ud5a5\ub825\uc744 \uac10\uc18c\uc2dc\ud0a4\uace0 \uc57d\ubb3c \uc790\uccb4\ub97c \uccb4\uc678\ub85c \ub0b4\ubcf4\ub0bc \uc218 \uc788\ub294 \uad6c\uc870\ub85c \ubcc0\ud654\ud558\uc5ec \uacb0\uad6d \ud56d\uc0dd\uc81c \ub0b4\uc131\uc774 \uc0dd\uae30\uac8c \ub41c\ub2e4. \ubcf4\ub2e4 \uadf9\ub2e8\uc801\uc778 \uc0ac\ub840\ub4e4\ub3c4 \uc788\ub2e4. \ub300\uc7a5\uade0 \uc7a5\uae30 \uc9c4\ud654 \uc2e4\ud5d8\uc5d0\uc11c \ub300\uc7a5\uade0\uc740 \uc2dc\ud2b8\ub974\uc0b0\uc744 \uc591\ubd84\uc73c\ub85c \uc0ac\uc6a9\ud560 \uc218 \uc788\uc5c8\ub2e4. \uc2ec\uc9c0\uc5b4 \ud50c\ub77c\ubcf4\ubc15\ud14c\ub9ac\uc6c0 \uc740 \ub098\uc77c\ub860\uc744 \uc591\ubd84\uc73c\ub85c \ubcc0\ud654\uc2dc\ud0a4\ub294 \ud6a8\uc18c\ub97c \ub9cc\ub4e4\uc5b4 \ub0b4\uc5c8\ub2e4. \ud1a0\uc591 \ubc15\ud14c\ub9ac\uc544\uc758 \uc77c\uc885\uc778 \uc2a4\ud551\uace0\ube44\uc6c0\uc740 \ub18d\uc57d\uc73c\ub85c \uc0ac\uc6a9\ub41c \ud39c\ud0c0\ud074\ub85c\ub85c\ud398\ub180\uc758 \ub3c5\uc131\uc744 \uc57d\ud654\uc2dc\ud0a4\uae30\ub294 \uc0c8\ub85c\uc6b4 \ub300\uc0ac\ud68c\ub85c\ub97c \uac16\ub294 \ud615\ud0dc\ub85c \uc801\uc751\ud558\uc600\ub2e4. \uc5ec\uc804\ud788 \ub17c\ub780\uc774 \uc788\uc73c\ub098 \uc720\uc804\uc790 \ub2e4\uc591\uc131\uc744 \ubc1c\ud604\uc2dc\ud0a4\ub294 \uc0dd\ubb3c\uc758 \ub2a5\ub825\uc774 \uc801\uc751\uc758 \uc131\uacf5\uacfc \ubc00\uc811\ud55c \uad00\ub828\uc774 \uc788\ub2e4\ub294 \uc774\ub860\uc774 \uc810\ucc28 \uc9c0\uc9c0\ub97c \uc5bb\uc5b4\uac00\uace0 \uc788\ub2e4. \uc5bc\ub9c8\ub098 \uc131\uacf5\uc801\uc73c\ub85c \uc801\uc751\ud560 \uc218 \uc788\ub294\uc9c0\ub97c \ub098\ud0c0\ub0b4\ub294 \ucc99\ub3c4\ub85c \uc9c4\ud654\ub3c4\ub780 \uac1c\ub150\uc774 \uc81c\uc2dc\ub418\uc5c8\ub2e4.", "paragraph_answer": "\uc801\uc751\uc740 \uc774\ub85c\uc6b4 \ud2b9\uc9d5\uc758 \ubc1c\ud604\uc740 \ucd09\uc9c4\ub418\uace0 \ubd88\ub9ac\ud55c \ud2b9\uc9d5\uc758 \ubc1c\ud604\uc740 \uc904\uc5b4\ub4e4\uba74\uc11c \uc774\ub8e8\uc5b4\uc9c4\ub2e4. \ubc15\ud14c\ub9ac\uc544\uac00 \ud56d\uc0dd\ubb3c\uc9c8\uc5d0 \uc801\uc751\ud558\uc5ec \uc9c4\ud654\ud558\ub294 \uac83\uc744 \uc608\ub85c \ub4e4 \uc218 \uc788\ub2e4. \ubc15\ud14c\ub9ac\uc544\ub294 \uc57d\ubb3c\uc774 \uac1c\uccb4\uc5d0 \uc8fc\ub294 \uc601\ud5a5\ub825\uc744 \uac10\uc18c\uc2dc\ud0a4\uace0 \uc57d\ubb3c \uc790\uccb4\ub97c \uccb4\uc678\ub85c \ub0b4\ubcf4\ub0bc \uc218 \uc788\ub294 \uad6c\uc870\ub85c \ubcc0\ud654\ud558\uc5ec \uacb0\uad6d \ud56d\uc0dd\uc81c \ub0b4\uc131\uc774 \uc0dd\uae30\uac8c \ub41c\ub2e4. \ubcf4\ub2e4 \uadf9\ub2e8\uc801\uc778 \uc0ac\ub840\ub4e4\ub3c4 \uc788\ub2e4. \ub300\uc7a5\uade0 \uc7a5\uae30 \uc9c4\ud654 \uc2e4\ud5d8\uc5d0\uc11c \ub300\uc7a5\uade0\uc740 \uc2dc\ud2b8\ub974\uc0b0\uc744 \uc591\ubd84\uc73c\ub85c \uc0ac\uc6a9\ud560 \uc218 \uc788\uc5c8\ub2e4. \uc2ec\uc9c0\uc5b4 \ud50c\ub77c\ubcf4\ubc15\ud14c\ub9ac\uc6c0 \uc740 \ub098\uc77c\ub860\uc744 \uc591\ubd84\uc73c\ub85c \ubcc0\ud654\uc2dc\ud0a4\ub294 \ud6a8\uc18c\ub97c \ub9cc\ub4e4\uc5b4 \ub0b4\uc5c8\ub2e4. \ud1a0\uc591 \ubc15\ud14c\ub9ac\uc544\uc758 \uc77c\uc885\uc778 \uc2a4\ud551\uace0\ube44\uc6c0\uc740 \ub18d\uc57d\uc73c\ub85c \uc0ac\uc6a9\ub41c \ud39c\ud0c0\ud074\ub85c\ub85c\ud398\ub180\uc758 \ub3c5\uc131\uc744 \uc57d\ud654\uc2dc\ud0a4\uae30\ub294 \uc0c8\ub85c\uc6b4 \ub300\uc0ac\ud68c\ub85c\ub97c \uac16\ub294 \ud615\ud0dc\ub85c \uc801\uc751\ud558\uc600\ub2e4. \uc5ec\uc804\ud788 \ub17c\ub780\uc774 \uc788\uc73c\ub098 \uc720\uc804\uc790 \ub2e4\uc591\uc131\uc744 \ubc1c\ud604\uc2dc\ud0a4\ub294 \uc0dd\ubb3c\uc758 \ub2a5\ub825\uc774 \uc801\uc751\uc758 \uc131\uacf5\uacfc \ubc00\uc811\ud55c \uad00\ub828\uc774 \uc788\ub2e4\ub294 \uc774\ub860\uc774 \uc810\ucc28 \uc9c0\uc9c0\ub97c \uc5bb\uc5b4\uac00\uace0 \uc788\ub2e4. \uc5bc\ub9c8\ub098 \uc131\uacf5\uc801\uc73c\ub85c \uc801\uc751\ud560 \uc218 \uc788\ub294\uc9c0\ub97c \ub098\ud0c0\ub0b4\ub294 \ucc99\ub3c4\ub85c \uc9c4\ud654\ub3c4\ub780 \uac1c\ub150\uc774 \uc81c\uc2dc\ub418\uc5c8\ub2e4.", "sentence_answer": "\uc2ec\uc9c0\uc5b4 \ud50c\ub77c\ubcf4\ubc15\ud14c\ub9ac\uc6c0 \uc740 \ub098\uc77c\ub860\uc744 \uc591\ubd84\uc73c\ub85c \ubcc0\ud654\uc2dc\ud0a4\ub294 \ud6a8\uc18c\ub97c \ub9cc\ub4e4\uc5b4 \ub0b4\uc5c8\ub2e4.", "paragraph_id": "6557516-13-2", "paragraph_question": "question: \ub098\uc77c\ub860\uc744 \uc591\ubd84\uc73c\ub85c \ubcc0\ud654\uc2dc\ud0a4\ub294 \ud6a8\uc18c\ub97c \ub9cc\ub4e4\uc5b4 \ub0b8\uac83\uc740?, context: \uc801\uc751\uc740 \uc774\ub85c\uc6b4 \ud2b9\uc9d5\uc758 \ubc1c\ud604\uc740 \ucd09\uc9c4\ub418\uace0 \ubd88\ub9ac\ud55c \ud2b9\uc9d5\uc758 \ubc1c\ud604\uc740 \uc904\uc5b4\ub4e4\uba74\uc11c \uc774\ub8e8\uc5b4\uc9c4\ub2e4. \ubc15\ud14c\ub9ac\uc544\uac00 \ud56d\uc0dd\ubb3c\uc9c8\uc5d0 \uc801\uc751\ud558\uc5ec \uc9c4\ud654\ud558\ub294 \uac83\uc744 \uc608\ub85c \ub4e4 \uc218 \uc788\ub2e4. \ubc15\ud14c\ub9ac\uc544\ub294 \uc57d\ubb3c\uc774 \uac1c\uccb4\uc5d0 \uc8fc\ub294 \uc601\ud5a5\ub825\uc744 \uac10\uc18c\uc2dc\ud0a4\uace0 \uc57d\ubb3c \uc790\uccb4\ub97c \uccb4\uc678\ub85c \ub0b4\ubcf4\ub0bc \uc218 \uc788\ub294 \uad6c\uc870\ub85c \ubcc0\ud654\ud558\uc5ec \uacb0\uad6d \ud56d\uc0dd\uc81c \ub0b4\uc131\uc774 \uc0dd\uae30\uac8c \ub41c\ub2e4. \ubcf4\ub2e4 \uadf9\ub2e8\uc801\uc778 \uc0ac\ub840\ub4e4\ub3c4 \uc788\ub2e4. \ub300\uc7a5\uade0 \uc7a5\uae30 \uc9c4\ud654 \uc2e4\ud5d8\uc5d0\uc11c \ub300\uc7a5\uade0\uc740 \uc2dc\ud2b8\ub974\uc0b0\uc744 \uc591\ubd84\uc73c\ub85c \uc0ac\uc6a9\ud560 \uc218 \uc788\uc5c8\ub2e4. \uc2ec\uc9c0\uc5b4 \ud50c\ub77c\ubcf4\ubc15\ud14c\ub9ac\uc6c0\uc740 \ub098\uc77c\ub860\uc744 \uc591\ubd84\uc73c\ub85c \ubcc0\ud654\uc2dc\ud0a4\ub294 \ud6a8\uc18c\ub97c \ub9cc\ub4e4\uc5b4 \ub0b4\uc5c8\ub2e4. \ud1a0\uc591 \ubc15\ud14c\ub9ac\uc544\uc758 \uc77c\uc885\uc778 \uc2a4\ud551\uace0\ube44\uc6c0\uc740 \ub18d\uc57d\uc73c\ub85c \uc0ac\uc6a9\ub41c \ud39c\ud0c0\ud074\ub85c\ub85c\ud398\ub180\uc758 \ub3c5\uc131\uc744 \uc57d\ud654\uc2dc\ud0a4\uae30\ub294 \uc0c8\ub85c\uc6b4 \ub300\uc0ac\ud68c\ub85c\ub97c \uac16\ub294 \ud615\ud0dc\ub85c \uc801\uc751\ud558\uc600\ub2e4. \uc5ec\uc804\ud788 \ub17c\ub780\uc774 \uc788\uc73c\ub098 \uc720\uc804\uc790 \ub2e4\uc591\uc131\uc744 \ubc1c\ud604\uc2dc\ud0a4\ub294 \uc0dd\ubb3c\uc758 \ub2a5\ub825\uc774 \uc801\uc751\uc758 \uc131\uacf5\uacfc \ubc00\uc811\ud55c \uad00\ub828\uc774 \uc788\ub2e4\ub294 \uc774\ub860\uc774 \uc810\ucc28 \uc9c0\uc9c0\ub97c \uc5bb\uc5b4\uac00\uace0 \uc788\ub2e4. \uc5bc\ub9c8\ub098 \uc131\uacf5\uc801\uc73c\ub85c \uc801\uc751\ud560 \uc218 \uc788\ub294\uc9c0\ub97c \ub098\ud0c0\ub0b4\ub294 \ucc99\ub3c4\ub85c \uc9c4\ud654\ub3c4\ub780 \uac1c\ub150\uc774 \uc81c\uc2dc\ub418\uc5c8\ub2e4."} {"question": "\ud5c8\ub9ac\ucf00\uc778 \ub9b0\ub2e4 \uc774\uc804\uc5d0 \uce98\ub9ac\ud3ec\ub2c8\uc544 \uc8fc \ud574\uc548\uc5d0 \ud5c8\ub9ac\ucf00\uc778\uc774 \uba87\ub144\ub3c4\uc5d0 \uc0c1\ub959\ud588\uc5c8\ub098?", "paragraph": "\ud5c8\ub9ac\ucf00\uc778 \ub9b0\ub2e4\uac00 \uc11c\ubd81\uc11c\ucabd\uc73c\ub85c \uc774\ub3d9\ud558\uae30 \uc804\uc5d0 \uc88c\uce21\uc758 \uadf8\ub9bc\uacfc \uac19\uc774 \uc608\ubcf4\ub294 \uce98\ub9ac\ud3ec\ub2c8\uc544 \uc8fc \ub0a8\ubd80\uc5d0 \uc5f4\ub300 \ud3ed\ud48d\uc758 \uc138\ub825\uc73c\ub85c \ub9b0\ub2e4\uac00 \ub3c4\ub2ec\ud560 \uac83\uc774\ub77c\uace0 \ub098\uc640\uc788\ub2e4. \uc774\uc804\uc5d0 \uce98\ub9ac\ud3ec\ub2c8\uc544 \uc8fc \ud574\uc548\uc5d0 \uc0c1\ub959\ud55c \ud5c8\ub9ac\ucf00\uc778\uc740 1939\ub144\uc758 1939\ub144 \uce98\ub9ac\ud3ec\ub2c8\uc544 \ud5c8\ub9ac\ucf00\uc778\uc73c\ub85c, 59\ub144 \ub9cc\uc5d0 \ubbf8\uad6d \ud574\uc548\uc5d0 \uc0c1\ub959\ud560 \uac83\uc774\ub77c\uace0 \ubc1c\ud45c\ud588\uc5c8\ub2e4. \ubbf8\uad6d \uce98\ub9ac\ud3ec\ub2c8\uc544 \uc8fc \uae30\uc0c1 \uc11c\ube44\uc2a4 \uc13c\ud130\uc5d0\uc11c\ub294 \uce98\ub9ac\ud3ec\ub2c8\uc544 \uc8fc \ub0a8\ubd80\uc5d0 \ub9ce\uc740 \uc601\ud5a5\uc744 \ub07c\uce60 \uc218 \uc788\ub2e4\uace0 \ubc1c\ud45c\ud558\uace0 \uacbd\uace0\ub839\uc744 \ub0b4\ub838\uc73c\ub098 \ud655\uc2e4\ud558\uc9c0\ub294 \uc54a\uc544 \ub9e4\uc2a4\ucef4\ud55c\ud14c\ub294 \ud5c8\ub9ac\ucf00\uc778\uc744 \uacfc\uc7a5\ud574 \ubcf4\ub3c4\ud558\uc9c0 \ub9d0\uc544\ub2ec\ub77c\uace0 \ubd80\ud0c1\ud588\ub2e4. \uae30\uc0c1 \uc11c\ube44\uc2a4 \uc13c\ud130\uc5d0\uc11c\ub294 \ud3ed\uc6b0\ub85c \uc778\ud574 \ud64d\uc218\uac00 \uc77c\uc5b4\ub0a0 \uc218 \uc788\uace0 \ub192\uc740 \ud30c\ub3c4\uac00 \ud574\uc548\uac00\ub97c \ub36e\uce60 \uc218 \uc788\ub2e4\ub294 \uacbd\uace0\ub97c \ud588\uace0 \uc774\uc5d0 \ub300\uc751\ud574 \uc791\uc5c5\uc790\ub4e4\uc740 \ubc30\uc218\uad6c\ub97c \uc815\ube44\ud558\uace0 \ud574\uc548\uac00 \uce68\uc218\ub97c \ub9c9\uae30 \uc704\ud574 \ubaa8\ub798\uc8fc\uba38\ub2c8\ub97c \uc900\ube44\ud558\uae30 \uc2dc\uc791\ud588\ub2e4.", "answer": "1939\ub144", "sentence": "\uc774\uc804\uc5d0 \uce98\ub9ac\ud3ec\ub2c8\uc544 \uc8fc \ud574\uc548\uc5d0 \uc0c1\ub959\ud55c \ud5c8\ub9ac\ucf00\uc778\uc740 1939\ub144 \uc758 1939\ub144 \uce98\ub9ac\ud3ec\ub2c8\uc544 \ud5c8\ub9ac\ucf00\uc778\uc73c\ub85c, 59\ub144 \ub9cc\uc5d0 \ubbf8\uad6d \ud574\uc548\uc5d0 \uc0c1\ub959\ud560 \uac83\uc774\ub77c\uace0 \ubc1c\ud45c\ud588\uc5c8\ub2e4.", "paragraph_sentence": "\ud5c8\ub9ac\ucf00\uc778 \ub9b0\ub2e4\uac00 \uc11c\ubd81\uc11c\ucabd\uc73c\ub85c \uc774\ub3d9\ud558\uae30 \uc804\uc5d0 \uc88c\uce21\uc758 \uadf8\ub9bc\uacfc \uac19\uc774 \uc608\ubcf4\ub294 \uce98\ub9ac\ud3ec\ub2c8\uc544 \uc8fc \ub0a8\ubd80\uc5d0 \uc5f4\ub300 \ud3ed\ud48d\uc758 \uc138\ub825\uc73c\ub85c \ub9b0\ub2e4\uac00 \ub3c4\ub2ec\ud560 \uac83\uc774\ub77c\uace0 \ub098\uc640\uc788\ub2e4. \uc774\uc804\uc5d0 \uce98\ub9ac\ud3ec\ub2c8\uc544 \uc8fc \ud574\uc548\uc5d0 \uc0c1\ub959\ud55c \ud5c8\ub9ac\ucf00\uc778\uc740 1939\ub144 \uc758 1939\ub144 \uce98\ub9ac\ud3ec\ub2c8\uc544 \ud5c8\ub9ac\ucf00\uc778\uc73c\ub85c, 59\ub144 \ub9cc\uc5d0 \ubbf8\uad6d \ud574\uc548\uc5d0 \uc0c1\ub959\ud560 \uac83\uc774\ub77c\uace0 \ubc1c\ud45c\ud588\uc5c8\ub2e4. \ubbf8\uad6d \uce98\ub9ac\ud3ec\ub2c8\uc544 \uc8fc \uae30\uc0c1 \uc11c\ube44\uc2a4 \uc13c\ud130\uc5d0\uc11c\ub294 \uce98\ub9ac\ud3ec\ub2c8\uc544 \uc8fc \ub0a8\ubd80\uc5d0 \ub9ce\uc740 \uc601\ud5a5\uc744 \ub07c\uce60 \uc218 \uc788\ub2e4\uace0 \ubc1c\ud45c\ud558\uace0 \uacbd\uace0\ub839\uc744 \ub0b4\ub838\uc73c\ub098 \ud655\uc2e4\ud558\uc9c0\ub294 \uc54a\uc544 \ub9e4\uc2a4\ucef4\ud55c\ud14c\ub294 \ud5c8\ub9ac\ucf00\uc778\uc744 \uacfc\uc7a5\ud574 \ubcf4\ub3c4\ud558\uc9c0 \ub9d0\uc544\ub2ec\ub77c\uace0 \ubd80\ud0c1\ud588\ub2e4. \uae30\uc0c1 \uc11c\ube44\uc2a4 \uc13c\ud130\uc5d0\uc11c\ub294 \ud3ed\uc6b0\ub85c \uc778\ud574 \ud64d\uc218\uac00 \uc77c\uc5b4\ub0a0 \uc218 \uc788\uace0 \ub192\uc740 \ud30c\ub3c4\uac00 \ud574\uc548\uac00\ub97c \ub36e\uce60 \uc218 \uc788\ub2e4\ub294 \uacbd\uace0\ub97c \ud588\uace0 \uc774\uc5d0 \ub300\uc751\ud574 \uc791\uc5c5\uc790\ub4e4\uc740 \ubc30\uc218\uad6c\ub97c \uc815\ube44\ud558\uace0 \ud574\uc548\uac00 \uce68\uc218\ub97c \ub9c9\uae30 \uc704\ud574 \ubaa8\ub798\uc8fc\uba38\ub2c8\ub97c \uc900\ube44\ud558\uae30 \uc2dc\uc791\ud588\ub2e4.", "paragraph_answer": "\ud5c8\ub9ac\ucf00\uc778 \ub9b0\ub2e4\uac00 \uc11c\ubd81\uc11c\ucabd\uc73c\ub85c \uc774\ub3d9\ud558\uae30 \uc804\uc5d0 \uc88c\uce21\uc758 \uadf8\ub9bc\uacfc \uac19\uc774 \uc608\ubcf4\ub294 \uce98\ub9ac\ud3ec\ub2c8\uc544 \uc8fc \ub0a8\ubd80\uc5d0 \uc5f4\ub300 \ud3ed\ud48d\uc758 \uc138\ub825\uc73c\ub85c \ub9b0\ub2e4\uac00 \ub3c4\ub2ec\ud560 \uac83\uc774\ub77c\uace0 \ub098\uc640\uc788\ub2e4. \uc774\uc804\uc5d0 \uce98\ub9ac\ud3ec\ub2c8\uc544 \uc8fc \ud574\uc548\uc5d0 \uc0c1\ub959\ud55c \ud5c8\ub9ac\ucf00\uc778\uc740 1939\ub144 \uc758 1939\ub144 \uce98\ub9ac\ud3ec\ub2c8\uc544 \ud5c8\ub9ac\ucf00\uc778\uc73c\ub85c, 59\ub144 \ub9cc\uc5d0 \ubbf8\uad6d \ud574\uc548\uc5d0 \uc0c1\ub959\ud560 \uac83\uc774\ub77c\uace0 \ubc1c\ud45c\ud588\uc5c8\ub2e4. \ubbf8\uad6d \uce98\ub9ac\ud3ec\ub2c8\uc544 \uc8fc \uae30\uc0c1 \uc11c\ube44\uc2a4 \uc13c\ud130\uc5d0\uc11c\ub294 \uce98\ub9ac\ud3ec\ub2c8\uc544 \uc8fc \ub0a8\ubd80\uc5d0 \ub9ce\uc740 \uc601\ud5a5\uc744 \ub07c\uce60 \uc218 \uc788\ub2e4\uace0 \ubc1c\ud45c\ud558\uace0 \uacbd\uace0\ub839\uc744 \ub0b4\ub838\uc73c\ub098 \ud655\uc2e4\ud558\uc9c0\ub294 \uc54a\uc544 \ub9e4\uc2a4\ucef4\ud55c\ud14c\ub294 \ud5c8\ub9ac\ucf00\uc778\uc744 \uacfc\uc7a5\ud574 \ubcf4\ub3c4\ud558\uc9c0 \ub9d0\uc544\ub2ec\ub77c\uace0 \ubd80\ud0c1\ud588\ub2e4. \uae30\uc0c1 \uc11c\ube44\uc2a4 \uc13c\ud130\uc5d0\uc11c\ub294 \ud3ed\uc6b0\ub85c \uc778\ud574 \ud64d\uc218\uac00 \uc77c\uc5b4\ub0a0 \uc218 \uc788\uace0 \ub192\uc740 \ud30c\ub3c4\uac00 \ud574\uc548\uac00\ub97c \ub36e\uce60 \uc218 \uc788\ub2e4\ub294 \uacbd\uace0\ub97c \ud588\uace0 \uc774\uc5d0 \ub300\uc751\ud574 \uc791\uc5c5\uc790\ub4e4\uc740 \ubc30\uc218\uad6c\ub97c \uc815\ube44\ud558\uace0 \ud574\uc548\uac00 \uce68\uc218\ub97c \ub9c9\uae30 \uc704\ud574 \ubaa8\ub798\uc8fc\uba38\ub2c8\ub97c \uc900\ube44\ud558\uae30 \uc2dc\uc791\ud588\ub2e4.", "sentence_answer": "\uc774\uc804\uc5d0 \uce98\ub9ac\ud3ec\ub2c8\uc544 \uc8fc \ud574\uc548\uc5d0 \uc0c1\ub959\ud55c \ud5c8\ub9ac\ucf00\uc778\uc740 1939\ub144 \uc758 1939\ub144 \uce98\ub9ac\ud3ec\ub2c8\uc544 \ud5c8\ub9ac\ucf00\uc778\uc73c\ub85c, 59\ub144 \ub9cc\uc5d0 \ubbf8\uad6d \ud574\uc548\uc5d0 \uc0c1\ub959\ud560 \uac83\uc774\ub77c\uace0 \ubc1c\ud45c\ud588\uc5c8\ub2e4.", "paragraph_id": "6571170-3-1", "paragraph_question": "question: \ud5c8\ub9ac\ucf00\uc778 \ub9b0\ub2e4 \uc774\uc804\uc5d0 \uce98\ub9ac\ud3ec\ub2c8\uc544 \uc8fc \ud574\uc548\uc5d0 \ud5c8\ub9ac\ucf00\uc778\uc774 \uba87\ub144\ub3c4\uc5d0 \uc0c1\ub959\ud588\uc5c8\ub098?, context: \ud5c8\ub9ac\ucf00\uc778 \ub9b0\ub2e4\uac00 \uc11c\ubd81\uc11c\ucabd\uc73c\ub85c \uc774\ub3d9\ud558\uae30 \uc804\uc5d0 \uc88c\uce21\uc758 \uadf8\ub9bc\uacfc \uac19\uc774 \uc608\ubcf4\ub294 \uce98\ub9ac\ud3ec\ub2c8\uc544 \uc8fc \ub0a8\ubd80\uc5d0 \uc5f4\ub300 \ud3ed\ud48d\uc758 \uc138\ub825\uc73c\ub85c \ub9b0\ub2e4\uac00 \ub3c4\ub2ec\ud560 \uac83\uc774\ub77c\uace0 \ub098\uc640\uc788\ub2e4. \uc774\uc804\uc5d0 \uce98\ub9ac\ud3ec\ub2c8\uc544 \uc8fc \ud574\uc548\uc5d0 \uc0c1\ub959\ud55c \ud5c8\ub9ac\ucf00\uc778\uc740 1939\ub144\uc758 1939\ub144 \uce98\ub9ac\ud3ec\ub2c8\uc544 \ud5c8\ub9ac\ucf00\uc778\uc73c\ub85c, 59\ub144 \ub9cc\uc5d0 \ubbf8\uad6d \ud574\uc548\uc5d0 \uc0c1\ub959\ud560 \uac83\uc774\ub77c\uace0 \ubc1c\ud45c\ud588\uc5c8\ub2e4. \ubbf8\uad6d \uce98\ub9ac\ud3ec\ub2c8\uc544 \uc8fc \uae30\uc0c1 \uc11c\ube44\uc2a4 \uc13c\ud130\uc5d0\uc11c\ub294 \uce98\ub9ac\ud3ec\ub2c8\uc544 \uc8fc \ub0a8\ubd80\uc5d0 \ub9ce\uc740 \uc601\ud5a5\uc744 \ub07c\uce60 \uc218 \uc788\ub2e4\uace0 \ubc1c\ud45c\ud558\uace0 \uacbd\uace0\ub839\uc744 \ub0b4\ub838\uc73c\ub098 \ud655\uc2e4\ud558\uc9c0\ub294 \uc54a\uc544 \ub9e4\uc2a4\ucef4\ud55c\ud14c\ub294 \ud5c8\ub9ac\ucf00\uc778\uc744 \uacfc\uc7a5\ud574 \ubcf4\ub3c4\ud558\uc9c0 \ub9d0\uc544\ub2ec\ub77c\uace0 \ubd80\ud0c1\ud588\ub2e4. \uae30\uc0c1 \uc11c\ube44\uc2a4 \uc13c\ud130\uc5d0\uc11c\ub294 \ud3ed\uc6b0\ub85c \uc778\ud574 \ud64d\uc218\uac00 \uc77c\uc5b4\ub0a0 \uc218 \uc788\uace0 \ub192\uc740 \ud30c\ub3c4\uac00 \ud574\uc548\uac00\ub97c \ub36e\uce60 \uc218 \uc788\ub2e4\ub294 \uacbd\uace0\ub97c \ud588\uace0 \uc774\uc5d0 \ub300\uc751\ud574 \uc791\uc5c5\uc790\ub4e4\uc740 \ubc30\uc218\uad6c\ub97c \uc815\ube44\ud558\uace0 \ud574\uc548\uac00 \uce68\uc218\ub97c \ub9c9\uae30 \uc704\ud574 \ubaa8\ub798\uc8fc\uba38\ub2c8\ub97c \uc900\ube44\ud558\uae30 \uc2dc\uc791\ud588\ub2e4."} {"question": "\uc564\ub4dc\ub958 \uac00\ud544\ub4dc\ub294 \uc5b4\ub514\uc5d0\uc11c \ud0dc\uc5b4\ub0ac\ub098?", "paragraph": "\uc564\ub4dc\ub958 \ub7ec\uc140 \uac00\ud544\ub4dc(\uc601\uc5b4: Andrew Russell Garfield \uc564\ub4dc\ub8e8 \ub7ec\uc140 \uac00\ud544\ub4dc, 1983\ub144 8\uc6d4 20\uc77c ~ )\ub294 \ubbf8\uad6d\uacfc \uc601\uad6d\uc758 \uad6d\uc801\uc744 \uac00\uc9c4 \ubc30\uc6b0\uc774\ub2e4. \ubbf8\uad6d \ub85c\uc2a4\uc564\uc824\ub808\uc2a4\uc5d0\uc11c \ud0dc\uc5b4\ub098 \uc601\uad6d \uc11c\ub9ac \uc8fc\uc5d0\uc11c \uc790\ub790\ub2e4. \uc601\uad6d \uc5f0\uadf9 \ubb34\ub300\uc640 \ud154\ub808\ube44\uc804 \ud504\ub85c\uadf8\ub7a8 \uc81c\uc791\uc744 \ud558\uba74\uc11c \uacbd\ub825\uc744 \uc313\uae30 \uc2dc\uc791\ud588\uc73c\uba70, 2007\ub144 \ub4dc\ub77c\ub9c8 \uc601\ud654 \u300a\ub85c\uc2a4\ud2b8 \ub77c\uc774\uc5b8\uc988\u300b\uc5d0 \ucd9c\uc5f0\ud558\uba74\uc11c \uc601\ud654 \ub370\ubdd4\ub97c \ud588\ub2e4. 2010\ub144 \u300a\uc18c\uc15c \ub124\ud2b8\uc6cc\ud06c\u300b\uc5d0\uc11c \uc5d0\ub4dc\uc640\ub3c4 \uc0c8\ubc84\ub9b0 \uc5ed\uc744 \uc5f0\uae30\ud574 \uc8fc\ubaa9\uc744 \ubc1b\uc558\uace0, \ube44\ud3c9\uac00\ub4e4\uc5d0 \ucc2c\uc0ac\ub97c \ubc1b\uc544 \uace8\ub4e0 \uae00\ub85c\ube0c\uc0c1\uacfc \uc601\uad6d \uc544\uce74\ub370\ubbf8\uc0c1(BAFTA)\uc5d0 \ud6c4\ubcf4 \uc9c0\uba85\ub418\uc5c8\ub2e4. \uc774\ud6c4 \uc2a4\ud30c\uc774\ub354\ub9e8 \uc601\ud654 \uc2dc\ub9ac\uc988\uc758 \ub9ac\ubd80\ud2b8 \uc791\ud488\uc778 \u300a\uc5b4\uba54\uc774\uc9d5 \uc2a4\ud30c\uc774\ub354\ub9e8\u300b(2012)\uc5d0\uc11c \uc8fc\uc5f0 \uc2a4\ud30c\uc774\ub354\ub9e8 / \ud53c\ud130 \ud30c\ucee4 \uc5ed\uc73c\ub85c \ucd9c\uc5f0\ud574 \ub300\uccb4\ub85c \uc88b\uc740 \ud3c9\uac00\ub97c \ubc1b\uc558\ub2e4. 2014\ub144 4\uc6d4\uc5d0 \uacf5\uac1c\ub41c \u300a\uc5b4\uba54\uc774\uc9d5 \uc2a4\ud30c\uc774\ub354\ub9e8 2\u300b\uc5d0\uc11c\ub3c4 \ub2e4\uc2dc \ud55c \ubc88 \uc2a4\ud30c\uc774\ub354\ub9e8 \uc5ed\uc73c\ub85c \ucd9c\uc5f0\ud558\uc600\ub2e4. \uadf8\ub294 \ub610\ud55c 2012\ub144 \ube0c\ub85c\ub4dc\uc6e8\uc774\uc5d0\uc11c \uacf5\uc5f0 \ub41c \uc544\uc11c \ubc00\ub7ec\uc758 \ub3d9\uba85 \uc18c\uc124\uc744 \uae30\ubc18\uc73c\ub85c \ud55c \uc5f0\uadf9 \u300a\uc138\uc77c\uc988\ub9e8\uc758 \uc8fd\uc74c\u300b\uc5d0\uc11c \ube44\ud504 \ub85c\uba3c \uc5ed\uc744 \uc5f0\uae30\ud574, \uc774 \uc791\ud488\uc740 \ud1a0\ub2c8\uc0c1 \uc5f0\uadf9 \ub0a8\uc6b0\uc870\uc5f0\uc0c1\uc5d0 \ud6c4\ubcf4 \uc9c0\uba85 \ub418\uc5c8\ub2e4. 2014\ub144\uc5d0 \uadf8\ub294 \uc2a4\ub9b4\ub7ec \ub4dc\ub77c\ub9c8 \u300a99 \ud648\uc988\u300b\uc5d0\uc11c \uacf5\ub3d9 \uc81c\uc791\uacfc \uc8fc\uc5f0\uc744 \ub9e1\uc558\ub2e4. 2016\ub144 \uac00\ud544\ub4dc\ub294 \ub9c8\ud2f4 \uc2a4\ucf54\uc138\uc774\uc9c0\uc758 \u300a\uc0ac\uc77c\ub7f0\uc2a4\u300b\uc640 \uba5c \uae41\uc2a8\uc758 \u300a\ud575\uc18c \uace0\uc9c0\u300b \ub4f1 \ub450 \uc791\ud488\uc5d0\uc11c \ube44\ud3c9\uc801\uc73c\ub85c \uc720\uba85\ud55c \uc2dc\ub300\uadf9 \ub4dc\ub77c\ub9c8\uc5d0 \ucd9c\uc5f0\ud588\ub2e4. \uadf8\ub294 \ub370\uc988\uba3c\ub4dc \ub3c4\uc2a4 \uc5ed\uc758 \uc5f0\uae30\ub85c \ud638\ud3c9\uc744 \ubc1b\uc544 \ubbf8\uad6d \uc544\uce74\ub370\ubbf8\uc0c1, \uace8\ub4e0\uae00\ub85c\ube0c\uc0c1, \uc601\uad6d \uc544\uce74\ub370\ubbf8 \uc601\ud654\uc0c1 \ubc0f \ubbf8\uad6d \ubc30\uc6b0 \uc870\ud569\uc0c1\uc5d0 \ud6c4\ubcf4 \uc9c0\uba85\ub418\uc5c8\ub2e4.", "answer": "\ubbf8\uad6d \ub85c\uc2a4\uc564\uc824\ub808\uc2a4", "sentence": "\ubbf8\uad6d \ub85c\uc2a4\uc564\uc824\ub808\uc2a4 \uc5d0\uc11c \ud0dc\uc5b4\ub098 \uc601\uad6d \uc11c\ub9ac \uc8fc\uc5d0\uc11c \uc790\ub790\ub2e4.", "paragraph_sentence": "\uc564\ub4dc\ub958 \ub7ec\uc140 \uac00\ud544\ub4dc(\uc601\uc5b4: Andrew Russell Garfield \uc564\ub4dc\ub8e8 \ub7ec\uc140 \uac00\ud544\ub4dc, 1983\ub144 8\uc6d4 20\uc77c ~ )\ub294 \ubbf8\uad6d\uacfc \uc601\uad6d\uc758 \uad6d\uc801\uc744 \uac00\uc9c4 \ubc30\uc6b0\uc774\ub2e4. \ubbf8\uad6d \ub85c\uc2a4\uc564\uc824\ub808\uc2a4 \uc5d0\uc11c \ud0dc\uc5b4\ub098 \uc601\uad6d \uc11c\ub9ac \uc8fc\uc5d0\uc11c \uc790\ub790\ub2e4. \uc601\uad6d \uc5f0\uadf9 \ubb34\ub300\uc640 \ud154\ub808\ube44\uc804 \ud504\ub85c\uadf8\ub7a8 \uc81c\uc791\uc744 \ud558\uba74\uc11c \uacbd\ub825\uc744 \uc313\uae30 \uc2dc\uc791\ud588\uc73c\uba70, 2007\ub144 \ub4dc\ub77c\ub9c8 \uc601\ud654 \u300a\ub85c\uc2a4\ud2b8 \ub77c\uc774\uc5b8\uc988\u300b\uc5d0 \ucd9c\uc5f0\ud558\uba74\uc11c \uc601\ud654 \ub370\ubdd4\ub97c \ud588\ub2e4. 2010\ub144 \u300a\uc18c\uc15c \ub124\ud2b8\uc6cc\ud06c\u300b\uc5d0\uc11c \uc5d0\ub4dc\uc640\ub3c4 \uc0c8\ubc84\ub9b0 \uc5ed\uc744 \uc5f0\uae30\ud574 \uc8fc\ubaa9\uc744 \ubc1b\uc558\uace0, \ube44\ud3c9\uac00\ub4e4\uc5d0 \ucc2c\uc0ac\ub97c \ubc1b\uc544 \uace8\ub4e0 \uae00\ub85c\ube0c\uc0c1\uacfc \uc601\uad6d \uc544\uce74\ub370\ubbf8\uc0c1(BAFTA)\uc5d0 \ud6c4\ubcf4 \uc9c0\uba85\ub418\uc5c8\ub2e4. \uc774\ud6c4 \uc2a4\ud30c\uc774\ub354\ub9e8 \uc601\ud654 \uc2dc\ub9ac\uc988\uc758 \ub9ac\ubd80\ud2b8 \uc791\ud488\uc778 \u300a\uc5b4\uba54\uc774\uc9d5 \uc2a4\ud30c\uc774\ub354\ub9e8\u300b(2012)\uc5d0\uc11c \uc8fc\uc5f0 \uc2a4\ud30c\uc774\ub354\ub9e8 / \ud53c\ud130 \ud30c\ucee4 \uc5ed\uc73c\ub85c \ucd9c\uc5f0\ud574 \ub300\uccb4\ub85c \uc88b\uc740 \ud3c9\uac00\ub97c \ubc1b\uc558\ub2e4. 2014\ub144 4\uc6d4\uc5d0 \uacf5\uac1c\ub41c \u300a\uc5b4\uba54\uc774\uc9d5 \uc2a4\ud30c\uc774\ub354\ub9e8 2\u300b\uc5d0\uc11c\ub3c4 \ub2e4\uc2dc \ud55c \ubc88 \uc2a4\ud30c\uc774\ub354\ub9e8 \uc5ed\uc73c\ub85c \ucd9c\uc5f0\ud558\uc600\ub2e4. \uadf8\ub294 \ub610\ud55c 2012\ub144 \ube0c\ub85c\ub4dc\uc6e8\uc774\uc5d0\uc11c \uacf5\uc5f0 \ub41c \uc544\uc11c \ubc00\ub7ec\uc758 \ub3d9\uba85 \uc18c\uc124\uc744 \uae30\ubc18\uc73c\ub85c \ud55c \uc5f0\uadf9 \u300a\uc138\uc77c\uc988\ub9e8\uc758 \uc8fd\uc74c\u300b\uc5d0\uc11c \ube44\ud504 \ub85c\uba3c \uc5ed\uc744 \uc5f0\uae30\ud574, \uc774 \uc791\ud488\uc740 \ud1a0\ub2c8\uc0c1 \uc5f0\uadf9 \ub0a8\uc6b0\uc870\uc5f0\uc0c1\uc5d0 \ud6c4\ubcf4 \uc9c0\uba85 \ub418\uc5c8\ub2e4. 2014\ub144\uc5d0 \uadf8\ub294 \uc2a4\ub9b4\ub7ec \ub4dc\ub77c\ub9c8 \u300a99 \ud648\uc988\u300b\uc5d0\uc11c \uacf5\ub3d9 \uc81c\uc791\uacfc \uc8fc\uc5f0\uc744 \ub9e1\uc558\ub2e4. 2016\ub144 \uac00\ud544\ub4dc\ub294 \ub9c8\ud2f4 \uc2a4\ucf54\uc138\uc774\uc9c0\uc758 \u300a\uc0ac\uc77c\ub7f0\uc2a4\u300b\uc640 \uba5c \uae41\uc2a8\uc758 \u300a\ud575\uc18c \uace0\uc9c0\u300b \ub4f1 \ub450 \uc791\ud488\uc5d0\uc11c \ube44\ud3c9\uc801\uc73c\ub85c \uc720\uba85\ud55c \uc2dc\ub300\uadf9 \ub4dc\ub77c\ub9c8\uc5d0 \ucd9c\uc5f0\ud588\ub2e4. \uadf8\ub294 \ub370\uc988\uba3c\ub4dc \ub3c4\uc2a4 \uc5ed\uc758 \uc5f0\uae30\ub85c \ud638\ud3c9\uc744 \ubc1b\uc544 \ubbf8\uad6d \uc544\uce74\ub370\ubbf8\uc0c1, \uace8\ub4e0\uae00\ub85c\ube0c\uc0c1, \uc601\uad6d \uc544\uce74\ub370\ubbf8 \uc601\ud654\uc0c1 \ubc0f \ubbf8\uad6d \ubc30\uc6b0 \uc870\ud569\uc0c1\uc5d0 \ud6c4\ubcf4 \uc9c0\uba85\ub418\uc5c8\ub2e4.", "paragraph_answer": "\uc564\ub4dc\ub958 \ub7ec\uc140 \uac00\ud544\ub4dc(\uc601\uc5b4: Andrew Russell Garfield \uc564\ub4dc\ub8e8 \ub7ec\uc140 \uac00\ud544\ub4dc, 1983\ub144 8\uc6d4 20\uc77c ~ )\ub294 \ubbf8\uad6d\uacfc \uc601\uad6d\uc758 \uad6d\uc801\uc744 \uac00\uc9c4 \ubc30\uc6b0\uc774\ub2e4. \ubbf8\uad6d \ub85c\uc2a4\uc564\uc824\ub808\uc2a4 \uc5d0\uc11c \ud0dc\uc5b4\ub098 \uc601\uad6d \uc11c\ub9ac \uc8fc\uc5d0\uc11c \uc790\ub790\ub2e4. \uc601\uad6d \uc5f0\uadf9 \ubb34\ub300\uc640 \ud154\ub808\ube44\uc804 \ud504\ub85c\uadf8\ub7a8 \uc81c\uc791\uc744 \ud558\uba74\uc11c \uacbd\ub825\uc744 \uc313\uae30 \uc2dc\uc791\ud588\uc73c\uba70, 2007\ub144 \ub4dc\ub77c\ub9c8 \uc601\ud654 \u300a\ub85c\uc2a4\ud2b8 \ub77c\uc774\uc5b8\uc988\u300b\uc5d0 \ucd9c\uc5f0\ud558\uba74\uc11c \uc601\ud654 \ub370\ubdd4\ub97c \ud588\ub2e4. 2010\ub144 \u300a\uc18c\uc15c \ub124\ud2b8\uc6cc\ud06c\u300b\uc5d0\uc11c \uc5d0\ub4dc\uc640\ub3c4 \uc0c8\ubc84\ub9b0 \uc5ed\uc744 \uc5f0\uae30\ud574 \uc8fc\ubaa9\uc744 \ubc1b\uc558\uace0, \ube44\ud3c9\uac00\ub4e4\uc5d0 \ucc2c\uc0ac\ub97c \ubc1b\uc544 \uace8\ub4e0 \uae00\ub85c\ube0c\uc0c1\uacfc \uc601\uad6d \uc544\uce74\ub370\ubbf8\uc0c1(BAFTA)\uc5d0 \ud6c4\ubcf4 \uc9c0\uba85\ub418\uc5c8\ub2e4. \uc774\ud6c4 \uc2a4\ud30c\uc774\ub354\ub9e8 \uc601\ud654 \uc2dc\ub9ac\uc988\uc758 \ub9ac\ubd80\ud2b8 \uc791\ud488\uc778 \u300a\uc5b4\uba54\uc774\uc9d5 \uc2a4\ud30c\uc774\ub354\ub9e8\u300b(2012)\uc5d0\uc11c \uc8fc\uc5f0 \uc2a4\ud30c\uc774\ub354\ub9e8 / \ud53c\ud130 \ud30c\ucee4 \uc5ed\uc73c\ub85c \ucd9c\uc5f0\ud574 \ub300\uccb4\ub85c \uc88b\uc740 \ud3c9\uac00\ub97c \ubc1b\uc558\ub2e4. 2014\ub144 4\uc6d4\uc5d0 \uacf5\uac1c\ub41c \u300a\uc5b4\uba54\uc774\uc9d5 \uc2a4\ud30c\uc774\ub354\ub9e8 2\u300b\uc5d0\uc11c\ub3c4 \ub2e4\uc2dc \ud55c \ubc88 \uc2a4\ud30c\uc774\ub354\ub9e8 \uc5ed\uc73c\ub85c \ucd9c\uc5f0\ud558\uc600\ub2e4. \uadf8\ub294 \ub610\ud55c 2012\ub144 \ube0c\ub85c\ub4dc\uc6e8\uc774\uc5d0\uc11c \uacf5\uc5f0 \ub41c \uc544\uc11c \ubc00\ub7ec\uc758 \ub3d9\uba85 \uc18c\uc124\uc744 \uae30\ubc18\uc73c\ub85c \ud55c \uc5f0\uadf9 \u300a\uc138\uc77c\uc988\ub9e8\uc758 \uc8fd\uc74c\u300b\uc5d0\uc11c \ube44\ud504 \ub85c\uba3c \uc5ed\uc744 \uc5f0\uae30\ud574, \uc774 \uc791\ud488\uc740 \ud1a0\ub2c8\uc0c1 \uc5f0\uadf9 \ub0a8\uc6b0\uc870\uc5f0\uc0c1\uc5d0 \ud6c4\ubcf4 \uc9c0\uba85 \ub418\uc5c8\ub2e4. 2014\ub144\uc5d0 \uadf8\ub294 \uc2a4\ub9b4\ub7ec \ub4dc\ub77c\ub9c8 \u300a99 \ud648\uc988\u300b\uc5d0\uc11c \uacf5\ub3d9 \uc81c\uc791\uacfc \uc8fc\uc5f0\uc744 \ub9e1\uc558\ub2e4. 2016\ub144 \uac00\ud544\ub4dc\ub294 \ub9c8\ud2f4 \uc2a4\ucf54\uc138\uc774\uc9c0\uc758 \u300a\uc0ac\uc77c\ub7f0\uc2a4\u300b\uc640 \uba5c \uae41\uc2a8\uc758 \u300a\ud575\uc18c \uace0\uc9c0\u300b \ub4f1 \ub450 \uc791\ud488\uc5d0\uc11c \ube44\ud3c9\uc801\uc73c\ub85c \uc720\uba85\ud55c \uc2dc\ub300\uadf9 \ub4dc\ub77c\ub9c8\uc5d0 \ucd9c\uc5f0\ud588\ub2e4. \uadf8\ub294 \ub370\uc988\uba3c\ub4dc \ub3c4\uc2a4 \uc5ed\uc758 \uc5f0\uae30\ub85c \ud638\ud3c9\uc744 \ubc1b\uc544 \ubbf8\uad6d \uc544\uce74\ub370\ubbf8\uc0c1, \uace8\ub4e0\uae00\ub85c\ube0c\uc0c1, \uc601\uad6d \uc544\uce74\ub370\ubbf8 \uc601\ud654\uc0c1 \ubc0f \ubbf8\uad6d \ubc30\uc6b0 \uc870\ud569\uc0c1\uc5d0 \ud6c4\ubcf4 \uc9c0\uba85\ub418\uc5c8\ub2e4.", "sentence_answer": " \ubbf8\uad6d \ub85c\uc2a4\uc564\uc824\ub808\uc2a4 \uc5d0\uc11c \ud0dc\uc5b4\ub098 \uc601\uad6d \uc11c\ub9ac \uc8fc\uc5d0\uc11c \uc790\ub790\ub2e4.", "paragraph_id": "6540241-0-1", "paragraph_question": "question: \uc564\ub4dc\ub958 \uac00\ud544\ub4dc\ub294 \uc5b4\ub514\uc5d0\uc11c \ud0dc\uc5b4\ub0ac\ub098?, context: \uc564\ub4dc\ub958 \ub7ec\uc140 \uac00\ud544\ub4dc(\uc601\uc5b4: Andrew Russell Garfield \uc564\ub4dc\ub8e8 \ub7ec\uc140 \uac00\ud544\ub4dc, 1983\ub144 8\uc6d4 20\uc77c ~ )\ub294 \ubbf8\uad6d\uacfc \uc601\uad6d\uc758 \uad6d\uc801\uc744 \uac00\uc9c4 \ubc30\uc6b0\uc774\ub2e4. \ubbf8\uad6d \ub85c\uc2a4\uc564\uc824\ub808\uc2a4\uc5d0\uc11c \ud0dc\uc5b4\ub098 \uc601\uad6d \uc11c\ub9ac \uc8fc\uc5d0\uc11c \uc790\ub790\ub2e4. \uc601\uad6d \uc5f0\uadf9 \ubb34\ub300\uc640 \ud154\ub808\ube44\uc804 \ud504\ub85c\uadf8\ub7a8 \uc81c\uc791\uc744 \ud558\uba74\uc11c \uacbd\ub825\uc744 \uc313\uae30 \uc2dc\uc791\ud588\uc73c\uba70, 2007\ub144 \ub4dc\ub77c\ub9c8 \uc601\ud654 \u300a\ub85c\uc2a4\ud2b8 \ub77c\uc774\uc5b8\uc988\u300b\uc5d0 \ucd9c\uc5f0\ud558\uba74\uc11c \uc601\ud654 \ub370\ubdd4\ub97c \ud588\ub2e4. 2010\ub144 \u300a\uc18c\uc15c \ub124\ud2b8\uc6cc\ud06c\u300b\uc5d0\uc11c \uc5d0\ub4dc\uc640\ub3c4 \uc0c8\ubc84\ub9b0 \uc5ed\uc744 \uc5f0\uae30\ud574 \uc8fc\ubaa9\uc744 \ubc1b\uc558\uace0, \ube44\ud3c9\uac00\ub4e4\uc5d0 \ucc2c\uc0ac\ub97c \ubc1b\uc544 \uace8\ub4e0 \uae00\ub85c\ube0c\uc0c1\uacfc \uc601\uad6d \uc544\uce74\ub370\ubbf8\uc0c1(BAFTA)\uc5d0 \ud6c4\ubcf4 \uc9c0\uba85\ub418\uc5c8\ub2e4. \uc774\ud6c4 \uc2a4\ud30c\uc774\ub354\ub9e8 \uc601\ud654 \uc2dc\ub9ac\uc988\uc758 \ub9ac\ubd80\ud2b8 \uc791\ud488\uc778 \u300a\uc5b4\uba54\uc774\uc9d5 \uc2a4\ud30c\uc774\ub354\ub9e8\u300b(2012)\uc5d0\uc11c \uc8fc\uc5f0 \uc2a4\ud30c\uc774\ub354\ub9e8 / \ud53c\ud130 \ud30c\ucee4 \uc5ed\uc73c\ub85c \ucd9c\uc5f0\ud574 \ub300\uccb4\ub85c \uc88b\uc740 \ud3c9\uac00\ub97c \ubc1b\uc558\ub2e4. 2014\ub144 4\uc6d4\uc5d0 \uacf5\uac1c\ub41c \u300a\uc5b4\uba54\uc774\uc9d5 \uc2a4\ud30c\uc774\ub354\ub9e8 2\u300b\uc5d0\uc11c\ub3c4 \ub2e4\uc2dc \ud55c \ubc88 \uc2a4\ud30c\uc774\ub354\ub9e8 \uc5ed\uc73c\ub85c \ucd9c\uc5f0\ud558\uc600\ub2e4. \uadf8\ub294 \ub610\ud55c 2012\ub144 \ube0c\ub85c\ub4dc\uc6e8\uc774\uc5d0\uc11c \uacf5\uc5f0 \ub41c \uc544\uc11c \ubc00\ub7ec\uc758 \ub3d9\uba85 \uc18c\uc124\uc744 \uae30\ubc18\uc73c\ub85c \ud55c \uc5f0\uadf9 \u300a\uc138\uc77c\uc988\ub9e8\uc758 \uc8fd\uc74c\u300b\uc5d0\uc11c \ube44\ud504 \ub85c\uba3c \uc5ed\uc744 \uc5f0\uae30\ud574, \uc774 \uc791\ud488\uc740 \ud1a0\ub2c8\uc0c1 \uc5f0\uadf9 \ub0a8\uc6b0\uc870\uc5f0\uc0c1\uc5d0 \ud6c4\ubcf4 \uc9c0\uba85 \ub418\uc5c8\ub2e4. 2014\ub144\uc5d0 \uadf8\ub294 \uc2a4\ub9b4\ub7ec \ub4dc\ub77c\ub9c8 \u300a99 \ud648\uc988\u300b\uc5d0\uc11c \uacf5\ub3d9 \uc81c\uc791\uacfc \uc8fc\uc5f0\uc744 \ub9e1\uc558\ub2e4. 2016\ub144 \uac00\ud544\ub4dc\ub294 \ub9c8\ud2f4 \uc2a4\ucf54\uc138\uc774\uc9c0\uc758 \u300a\uc0ac\uc77c\ub7f0\uc2a4\u300b\uc640 \uba5c \uae41\uc2a8\uc758 \u300a\ud575\uc18c \uace0\uc9c0\u300b \ub4f1 \ub450 \uc791\ud488\uc5d0\uc11c \ube44\ud3c9\uc801\uc73c\ub85c \uc720\uba85\ud55c \uc2dc\ub300\uadf9 \ub4dc\ub77c\ub9c8\uc5d0 \ucd9c\uc5f0\ud588\ub2e4. \uadf8\ub294 \ub370\uc988\uba3c\ub4dc \ub3c4\uc2a4 \uc5ed\uc758 \uc5f0\uae30\ub85c \ud638\ud3c9\uc744 \ubc1b\uc544 \ubbf8\uad6d \uc544\uce74\ub370\ubbf8\uc0c1, \uace8\ub4e0\uae00\ub85c\ube0c\uc0c1, \uc601\uad6d \uc544\uce74\ub370\ubbf8 \uc601\ud654\uc0c1 \ubc0f \ubbf8\uad6d \ubc30\uc6b0 \uc870\ud569\uc0c1\uc5d0 \ud6c4\ubcf4 \uc9c0\uba85\ub418\uc5c8\ub2e4."} {"question": "\uc720\ub300\uc778\uacfc \uae30\ub3c5\uad50\uc778\uc744 \ud559\ub300\ud558\ub294 \uc790\ub294 \uc2ec\ud310 \ub54c \ub0b4\uac00 \uc9c1\uc811 \uace0\uc18c\uc778\uc73c\ub85c \ub098\uc124 \uac83\uc774\ub2e4 \ub77c\uace0 \ubc1d\ud78c \uc608\uc5b8\uc790\uc758 \uc774\ub984\uc740?", "paragraph": "\uc608\uc5b8\uc790 \ubb34\ud568\ub9c8\ub4dc\ub294 \"\uc720\ub300\uc778\uacfc \uae30\ub3c5\uad50\uc778\uc744 \ud559\ub300\ud558\ub294 \uc790\ub294 \uc2ec\ud310 \ub54c \ub0b4\uac00 \uc9c1\uc811 \uace0\uc18c\uc778\uc73c\ub85c \ub098\uc124 \uac83\uc774\ub2e4\"\ub77c\uace0 \ubc1d\ud614\ub2e4. \uc774\ub97c \uc778\uc6a9\ud55c \ubc84\ub098\ub4dc \ub8e8\uc774\uc2a4\ub294 \"\uacc4\uc57d\uc73c\ub85c \ubd80\ub9ac\ub294 \uc0ac\ub78c\uc744 \uc8fd\uc778 \uc0ac\ub78c\uc740 \ucc9c\uad6d\uc758 \ud5a5\uc870\ucc28\ub3c4 \ub204\ub9b4 \uc218 \uc5c6\uc744 \uac83\uc774\ub2e4\"\ub77c\ub294 \ud558\ub514\uc2a4\uc758 \ubb38\uad6c\ub97c \uc778\uc6a9\ud588\ub2e4. \ucc45\uc758 \uc0ac\ub78c\ub4e4\uc5d0 \ud574\ub2f9\ud558\ub294 \uc720\ub300\uc778, \uae30\ub3c5\uad50\uc778\uc5d0 \ub300\ud55c \ubcf4\ud638\ub97c \uc0c1\uae30\ud558\ub294 \uae30\ucd08 \uc790\ub8cc\uc774\ub2e4. \uc774\ub77c\ud06c\uacc4 \ubbf8\uad6d\uc778\uc774\uc790 \uc5ec\ub7ec \ud559\uad50\uc5d0\uc11c \uc911\ub3d9\ud559\uc744 \uac00\ub974\uce5c \ubc14 \uc788\ub294 \ub9c8\uc774\ub4dc \uce74\ub450\ub9ac \uad50\uc218\ub294 \ube44\uc2b7\ud55c \ud558\ub514\uc2a4 \uad6c\uc808\uc778 \"\uacc4\uc57d \uad00\uacc4(\ub524\ubbf8)\ud558\uc5d0 \uc788\ub294 \uc790\ub4e4\uc5d0\uac8c \ud53c\ud574\ub97c \uc785\ud788\uba74 \ud6d7\ub0a0 \uadf8\uac00 \uac00\uc9c4 \ubaa8\ub4e0 \uac83\uc744 \ubab0\uc218\ud560 \uac83\uc774\uba70 \ub0b4\uac00 \uc2a4\uc2a4\ub85c \uc7ac\ud310\uc5d0 \ud568\uaed8\ud560 \uac83\uc774\ub2e4.\"\ub97c \uc778\uc6a9\ud588\ub2e4. \uadf8\ub7ec\ub098 \uc774\uc2ac\ub78c \uad6d\uac00\uc5d0\uc11c \ub524\ubbf8\uc758 \uc548\uc804\uc740 \ubd88\uc548\ud558\uae30 \uadf8\uc9c0 \uc5c6\uc5c8\ub2e4.", "answer": "\ubb34\ud568\ub9c8\ub4dc", "sentence": "\uc608\uc5b8\uc790 \ubb34\ud568\ub9c8\ub4dc \ub294 \"\uc720\ub300\uc778\uacfc \uae30\ub3c5\uad50\uc778\uc744 \ud559\ub300\ud558\ub294 \uc790\ub294 \uc2ec\ud310 \ub54c \ub0b4\uac00 \uc9c1\uc811 \uace0\uc18c\uc778\uc73c\ub85c \ub098\uc124 \uac83\uc774\ub2e4\"\ub77c\uace0 \ubc1d\ud614\ub2e4.", "paragraph_sentence": " \uc608\uc5b8\uc790 \ubb34\ud568\ub9c8\ub4dc \ub294 \"\uc720\ub300\uc778\uacfc \uae30\ub3c5\uad50\uc778\uc744 \ud559\ub300\ud558\ub294 \uc790\ub294 \uc2ec\ud310 \ub54c \ub0b4\uac00 \uc9c1\uc811 \uace0\uc18c\uc778\uc73c\ub85c \ub098\uc124 \uac83\uc774\ub2e4\"\ub77c\uace0 \ubc1d\ud614\ub2e4. \uc774\ub97c \uc778\uc6a9\ud55c \ubc84\ub098\ub4dc \ub8e8\uc774\uc2a4\ub294 \"\uacc4\uc57d\uc73c\ub85c \ubd80\ub9ac\ub294 \uc0ac\ub78c\uc744 \uc8fd\uc778 \uc0ac\ub78c\uc740 \ucc9c\uad6d\uc758 \ud5a5\uc870\ucc28\ub3c4 \ub204\ub9b4 \uc218 \uc5c6\uc744 \uac83\uc774\ub2e4\"\ub77c\ub294 \ud558\ub514\uc2a4\uc758 \ubb38\uad6c\ub97c \uc778\uc6a9\ud588\ub2e4. \ucc45\uc758 \uc0ac\ub78c\ub4e4\uc5d0 \ud574\ub2f9\ud558\ub294 \uc720\ub300\uc778, \uae30\ub3c5\uad50\uc778\uc5d0 \ub300\ud55c \ubcf4\ud638\ub97c \uc0c1\uae30\ud558\ub294 \uae30\ucd08 \uc790\ub8cc\uc774\ub2e4. \uc774\ub77c\ud06c\uacc4 \ubbf8\uad6d\uc778\uc774\uc790 \uc5ec\ub7ec \ud559\uad50\uc5d0\uc11c \uc911\ub3d9\ud559\uc744 \uac00\ub974\uce5c \ubc14 \uc788\ub294 \ub9c8\uc774\ub4dc \uce74\ub450\ub9ac \uad50\uc218\ub294 \ube44\uc2b7\ud55c \ud558\ub514\uc2a4 \uad6c\uc808\uc778 \"\uacc4\uc57d \uad00\uacc4(\ub524\ubbf8)\ud558\uc5d0 \uc788\ub294 \uc790\ub4e4\uc5d0\uac8c \ud53c\ud574\ub97c \uc785\ud788\uba74 \ud6d7\ub0a0 \uadf8\uac00 \uac00\uc9c4 \ubaa8\ub4e0 \uac83\uc744 \ubab0\uc218\ud560 \uac83\uc774\uba70 \ub0b4\uac00 \uc2a4\uc2a4\ub85c \uc7ac\ud310\uc5d0 \ud568\uaed8\ud560 \uac83\uc774\ub2e4. \"\ub97c \uc778\uc6a9\ud588\ub2e4. \uadf8\ub7ec\ub098 \uc774\uc2ac\ub78c \uad6d\uac00\uc5d0\uc11c \ub524\ubbf8\uc758 \uc548\uc804\uc740 \ubd88\uc548\ud558\uae30 \uadf8\uc9c0 \uc5c6\uc5c8\ub2e4.", "paragraph_answer": "\uc608\uc5b8\uc790 \ubb34\ud568\ub9c8\ub4dc \ub294 \"\uc720\ub300\uc778\uacfc \uae30\ub3c5\uad50\uc778\uc744 \ud559\ub300\ud558\ub294 \uc790\ub294 \uc2ec\ud310 \ub54c \ub0b4\uac00 \uc9c1\uc811 \uace0\uc18c\uc778\uc73c\ub85c \ub098\uc124 \uac83\uc774\ub2e4\"\ub77c\uace0 \ubc1d\ud614\ub2e4. \uc774\ub97c \uc778\uc6a9\ud55c \ubc84\ub098\ub4dc \ub8e8\uc774\uc2a4\ub294 \"\uacc4\uc57d\uc73c\ub85c \ubd80\ub9ac\ub294 \uc0ac\ub78c\uc744 \uc8fd\uc778 \uc0ac\ub78c\uc740 \ucc9c\uad6d\uc758 \ud5a5\uc870\ucc28\ub3c4 \ub204\ub9b4 \uc218 \uc5c6\uc744 \uac83\uc774\ub2e4\"\ub77c\ub294 \ud558\ub514\uc2a4\uc758 \ubb38\uad6c\ub97c \uc778\uc6a9\ud588\ub2e4. \ucc45\uc758 \uc0ac\ub78c\ub4e4\uc5d0 \ud574\ub2f9\ud558\ub294 \uc720\ub300\uc778, \uae30\ub3c5\uad50\uc778\uc5d0 \ub300\ud55c \ubcf4\ud638\ub97c \uc0c1\uae30\ud558\ub294 \uae30\ucd08 \uc790\ub8cc\uc774\ub2e4. \uc774\ub77c\ud06c\uacc4 \ubbf8\uad6d\uc778\uc774\uc790 \uc5ec\ub7ec \ud559\uad50\uc5d0\uc11c \uc911\ub3d9\ud559\uc744 \uac00\ub974\uce5c \ubc14 \uc788\ub294 \ub9c8\uc774\ub4dc \uce74\ub450\ub9ac \uad50\uc218\ub294 \ube44\uc2b7\ud55c \ud558\ub514\uc2a4 \uad6c\uc808\uc778 \"\uacc4\uc57d \uad00\uacc4(\ub524\ubbf8)\ud558\uc5d0 \uc788\ub294 \uc790\ub4e4\uc5d0\uac8c \ud53c\ud574\ub97c \uc785\ud788\uba74 \ud6d7\ub0a0 \uadf8\uac00 \uac00\uc9c4 \ubaa8\ub4e0 \uac83\uc744 \ubab0\uc218\ud560 \uac83\uc774\uba70 \ub0b4\uac00 \uc2a4\uc2a4\ub85c \uc7ac\ud310\uc5d0 \ud568\uaed8\ud560 \uac83\uc774\ub2e4.\"\ub97c \uc778\uc6a9\ud588\ub2e4. \uadf8\ub7ec\ub098 \uc774\uc2ac\ub78c \uad6d\uac00\uc5d0\uc11c \ub524\ubbf8\uc758 \uc548\uc804\uc740 \ubd88\uc548\ud558\uae30 \uadf8\uc9c0 \uc5c6\uc5c8\ub2e4.", "sentence_answer": "\uc608\uc5b8\uc790 \ubb34\ud568\ub9c8\ub4dc \ub294 \"\uc720\ub300\uc778\uacfc \uae30\ub3c5\uad50\uc778\uc744 \ud559\ub300\ud558\ub294 \uc790\ub294 \uc2ec\ud310 \ub54c \ub0b4\uac00 \uc9c1\uc811 \uace0\uc18c\uc778\uc73c\ub85c \ub098\uc124 \uac83\uc774\ub2e4\"\ub77c\uace0 \ubc1d\ud614\ub2e4.", "paragraph_id": "6513164-2-0", "paragraph_question": "question: \uc720\ub300\uc778\uacfc \uae30\ub3c5\uad50\uc778\uc744 \ud559\ub300\ud558\ub294 \uc790\ub294 \uc2ec\ud310 \ub54c \ub0b4\uac00 \uc9c1\uc811 \uace0\uc18c\uc778\uc73c\ub85c \ub098\uc124 \uac83\uc774\ub2e4 \ub77c\uace0 \ubc1d\ud78c \uc608\uc5b8\uc790\uc758 \uc774\ub984\uc740?, context: \uc608\uc5b8\uc790 \ubb34\ud568\ub9c8\ub4dc\ub294 \"\uc720\ub300\uc778\uacfc \uae30\ub3c5\uad50\uc778\uc744 \ud559\ub300\ud558\ub294 \uc790\ub294 \uc2ec\ud310 \ub54c \ub0b4\uac00 \uc9c1\uc811 \uace0\uc18c\uc778\uc73c\ub85c \ub098\uc124 \uac83\uc774\ub2e4\"\ub77c\uace0 \ubc1d\ud614\ub2e4. \uc774\ub97c \uc778\uc6a9\ud55c \ubc84\ub098\ub4dc \ub8e8\uc774\uc2a4\ub294 \"\uacc4\uc57d\uc73c\ub85c \ubd80\ub9ac\ub294 \uc0ac\ub78c\uc744 \uc8fd\uc778 \uc0ac\ub78c\uc740 \ucc9c\uad6d\uc758 \ud5a5\uc870\ucc28\ub3c4 \ub204\ub9b4 \uc218 \uc5c6\uc744 \uac83\uc774\ub2e4\"\ub77c\ub294 \ud558\ub514\uc2a4\uc758 \ubb38\uad6c\ub97c \uc778\uc6a9\ud588\ub2e4. \ucc45\uc758 \uc0ac\ub78c\ub4e4\uc5d0 \ud574\ub2f9\ud558\ub294 \uc720\ub300\uc778, \uae30\ub3c5\uad50\uc778\uc5d0 \ub300\ud55c \ubcf4\ud638\ub97c \uc0c1\uae30\ud558\ub294 \uae30\ucd08 \uc790\ub8cc\uc774\ub2e4. \uc774\ub77c\ud06c\uacc4 \ubbf8\uad6d\uc778\uc774\uc790 \uc5ec\ub7ec \ud559\uad50\uc5d0\uc11c \uc911\ub3d9\ud559\uc744 \uac00\ub974\uce5c \ubc14 \uc788\ub294 \ub9c8\uc774\ub4dc \uce74\ub450\ub9ac \uad50\uc218\ub294 \ube44\uc2b7\ud55c \ud558\ub514\uc2a4 \uad6c\uc808\uc778 \"\uacc4\uc57d \uad00\uacc4(\ub524\ubbf8)\ud558\uc5d0 \uc788\ub294 \uc790\ub4e4\uc5d0\uac8c \ud53c\ud574\ub97c \uc785\ud788\uba74 \ud6d7\ub0a0 \uadf8\uac00 \uac00\uc9c4 \ubaa8\ub4e0 \uac83\uc744 \ubab0\uc218\ud560 \uac83\uc774\uba70 \ub0b4\uac00 \uc2a4\uc2a4\ub85c \uc7ac\ud310\uc5d0 \ud568\uaed8\ud560 \uac83\uc774\ub2e4.\"\ub97c \uc778\uc6a9\ud588\ub2e4. \uadf8\ub7ec\ub098 \uc774\uc2ac\ub78c \uad6d\uac00\uc5d0\uc11c \ub524\ubbf8\uc758 \uc548\uc804\uc740 \ubd88\uc548\ud558\uae30 \uadf8\uc9c0 \uc5c6\uc5c8\ub2e4."} {"question": "1\uc2ec\uc5d0\uc11c \uc131\uae34\uc5ec\ub300 \uc9c1\uc6d0\uc774 \uc790\ubc1c\uc801\uc73c\ub85c \ucc38\uc11d\ud588\ub2e4\uba70 \uc870 \uad50\uc218\uc5d0\uac8c \ub0b4\ub824\uc9c4 \ubc8c\uae08\ud615\uc740 \uc5bc\ub9c8\uc778\uac00?", "paragraph": "2017\ub144 2\uc6d44\uc77c \ub354\ubd88\uc5b4\ubbfc\uc8fc\ub2f9 \ubb38\uc7ac\uc778 \uc804 \ub300\ud45c\uc758 \ub300\uc120 \ucea0\ud504\uc5d0 \ud569\ub958\ud55c \uc804\uc778\ubc94 \uc804 \ud2b9\uc804\uc0ac\ub839\uad00\uc774 \uc544\ub0b4\uac00 \ucd1d\uc7a5\uc73c\ub85c \uc788\ub294 \uc131\uc2e0\uc5ec\ub300\uc758 \uad50\uc9c1\uc6d0\ub4e4\uc744 \uc0ac\uc801\uc73c\ub85c \uc774\uc6a9\ud55c \uac83\uc73c\ub85c \ub4dc\ub7ec\ub0ac\ub2e4. \ub300\ubc95\uc6d0\uc740 9\uc77c \uc804 \uc804 \uc0ac\ub839\uad00\uc774 \uc131\uc2e0\uc5ec\ub300 \uc804 \ubd80\ucd1d\uc7a5 \uc870 \ubaa8 \uad50\uc218\ub97c \uace0\uc18c\ud55c \uba85\uc608\ud6fc\uc190 \uc0ac\uac74 \uc0c1\uace0\uc2ec\uc5d0\uc11c \uc870 \uad50\uc218\uc758 \uc758\ud639 \uc81c\uae30\uac00 \uc77c\ubd80 \uc0ac\uc2e4\uc774\ub77c\uace0 \ubcf8 2\uc2ec \ud310\ub2e8\uc774 \uc815\ub2f9\ud558\ub2e4\uace0 \ud310\ub2e8\ud588\ub2e4. \uc870 \uad50\uc218\ub294 \uc9c0\ub09c 2009\ub144 \uc804 \uc804 \uc0ac\ub839\uad00\uc774 \uac15\uc6d0\ub3c4 \ud654\ucc9c\uc5d0\uc11c \uc18c\uc7a5 \uc2b9\uc9c4 \ucd95\ud558\ud30c\ud2f0\ub97c \uc5f4\uba74\uc11c \uc131\uc2e0\uc5ec\ub300 \uad50\uc9c1\uc6d0 20\uc5ec \uba85\uc744 \ud30c\ud2f0\uc6a9 \uc74c\uc2dd \uc900\ube44\uc640 \uc11c\ube59 \ub4f1 \ud589\uc0ac \uc694\uc6d0\uc73c\ub85c \ub3d9\uc6d0\ud588\ub2e4\ub294 \uc758\ud639\uc744 2013\ub144 \uc5b8\ub860 \ub4f1\uc5d0 \uc81c\uae30\ud588\ub2e4. 1\uc2ec\uc740 \uc131\uc2e0\uc5ec\ub300 \uc9c1\uc6d0\uc774 \uc790\ubc1c\uc801\uc73c\ub85c \ucc38\uc11d\ud588\ub2e4\uba70 \uc870 \uad50\uc218\uc5d0\uac8c \ubc8c\uae08 300\ub9cc\uc6d0\uc744 \uc120\uace0\ud588\uc9c0\ub9cc 2\uc2ec\uc740 \uc77c\ubd80 \uc0ac\uc2e4\uc5d0 \ucc28\uc774\uac00 \uc788\ub354\ub77c\ub3c4 \uc2b9\uc9c4 \ud30c\ud2f0\uc5d0 \uc9c1\uc6d0\uacfc \ud559\uc0dd\uc744 \ub3d9\uc6d0\ud588\ub2e4\ub294 \ub0b4\uc6a9\uc740 \uc911\uc694 \ubd80\ubd84\uc774 \uc778\uc815\ub41c\ub2e4\uba70 1\uc2ec\uc744 \ud30c\uae30\ud558\uace0 \uc774 \ubd80\ubd84\uc744 \ubb34\uc8c4\ub85c \ubd24\ub2e4.", "answer": "\ubc8c\uae08 300\ub9cc\uc6d0", "sentence": "1\uc2ec\uc740 \uc131\uc2e0\uc5ec\ub300 \uc9c1\uc6d0\uc774 \uc790\ubc1c\uc801\uc73c\ub85c \ucc38\uc11d\ud588\ub2e4\uba70 \uc870 \uad50\uc218\uc5d0\uac8c \ubc8c\uae08 300\ub9cc\uc6d0 \uc744 \uc120\uace0\ud588\uc9c0\ub9cc 2\uc2ec\uc740 \uc77c\ubd80 \uc0ac\uc2e4\uc5d0 \ucc28\uc774\uac00 \uc788\ub354\ub77c\ub3c4 \uc2b9\uc9c4 \ud30c\ud2f0\uc5d0 \uc9c1\uc6d0\uacfc \ud559\uc0dd\uc744 \ub3d9\uc6d0\ud588\ub2e4\ub294 \ub0b4\uc6a9\uc740 \uc911\uc694 \ubd80\ubd84\uc774 \uc778\uc815\ub41c\ub2e4\uba70 1\uc2ec\uc744 \ud30c\uae30\ud558\uace0 \uc774 \ubd80\ubd84\uc744 \ubb34\uc8c4\ub85c \ubd24\ub2e4.", "paragraph_sentence": "2017\ub144 2\uc6d44\uc77c \ub354\ubd88\uc5b4\ubbfc\uc8fc\ub2f9 \ubb38\uc7ac\uc778 \uc804 \ub300\ud45c\uc758 \ub300\uc120 \ucea0\ud504\uc5d0 \ud569\ub958\ud55c \uc804\uc778\ubc94 \uc804 \ud2b9\uc804\uc0ac\ub839\uad00\uc774 \uc544\ub0b4\uac00 \ucd1d\uc7a5\uc73c\ub85c \uc788\ub294 \uc131\uc2e0\uc5ec\ub300\uc758 \uad50\uc9c1\uc6d0\ub4e4\uc744 \uc0ac\uc801\uc73c\ub85c \uc774\uc6a9\ud55c \uac83\uc73c\ub85c \ub4dc\ub7ec\ub0ac\ub2e4. \ub300\ubc95\uc6d0\uc740 9\uc77c \uc804 \uc804 \uc0ac\ub839\uad00\uc774 \uc131\uc2e0\uc5ec\ub300 \uc804 \ubd80\ucd1d\uc7a5 \uc870 \ubaa8 \uad50\uc218\ub97c \uace0\uc18c\ud55c \uba85\uc608\ud6fc\uc190 \uc0ac\uac74 \uc0c1\uace0\uc2ec\uc5d0\uc11c \uc870 \uad50\uc218\uc758 \uc758\ud639 \uc81c\uae30\uac00 \uc77c\ubd80 \uc0ac\uc2e4\uc774\ub77c\uace0 \ubcf8 2\uc2ec \ud310\ub2e8\uc774 \uc815\ub2f9\ud558\ub2e4\uace0 \ud310\ub2e8\ud588\ub2e4. \uc870 \uad50\uc218\ub294 \uc9c0\ub09c 2009\ub144 \uc804 \uc804 \uc0ac\ub839\uad00\uc774 \uac15\uc6d0\ub3c4 \ud654\ucc9c\uc5d0\uc11c \uc18c\uc7a5 \uc2b9\uc9c4 \ucd95\ud558\ud30c\ud2f0\ub97c \uc5f4\uba74\uc11c \uc131\uc2e0\uc5ec\ub300 \uad50\uc9c1\uc6d0 20\uc5ec \uba85\uc744 \ud30c\ud2f0\uc6a9 \uc74c\uc2dd \uc900\ube44\uc640 \uc11c\ube59 \ub4f1 \ud589\uc0ac \uc694\uc6d0\uc73c\ub85c \ub3d9\uc6d0\ud588\ub2e4\ub294 \uc758\ud639\uc744 2013\ub144 \uc5b8\ub860 \ub4f1\uc5d0 \uc81c\uae30\ud588\ub2e4. 1\uc2ec\uc740 \uc131\uc2e0\uc5ec\ub300 \uc9c1\uc6d0\uc774 \uc790\ubc1c\uc801\uc73c\ub85c \ucc38\uc11d\ud588\ub2e4\uba70 \uc870 \uad50\uc218\uc5d0\uac8c \ubc8c\uae08 300\ub9cc\uc6d0 \uc744 \uc120\uace0\ud588\uc9c0\ub9cc 2\uc2ec\uc740 \uc77c\ubd80 \uc0ac\uc2e4\uc5d0 \ucc28\uc774\uac00 \uc788\ub354\ub77c\ub3c4 \uc2b9\uc9c4 \ud30c\ud2f0\uc5d0 \uc9c1\uc6d0\uacfc \ud559\uc0dd\uc744 \ub3d9\uc6d0\ud588\ub2e4\ub294 \ub0b4\uc6a9\uc740 \uc911\uc694 \ubd80\ubd84\uc774 \uc778\uc815\ub41c\ub2e4\uba70 1\uc2ec\uc744 \ud30c\uae30\ud558\uace0 \uc774 \ubd80\ubd84\uc744 \ubb34\uc8c4\ub85c \ubd24\ub2e4. ", "paragraph_answer": "2017\ub144 2\uc6d44\uc77c \ub354\ubd88\uc5b4\ubbfc\uc8fc\ub2f9 \ubb38\uc7ac\uc778 \uc804 \ub300\ud45c\uc758 \ub300\uc120 \ucea0\ud504\uc5d0 \ud569\ub958\ud55c \uc804\uc778\ubc94 \uc804 \ud2b9\uc804\uc0ac\ub839\uad00\uc774 \uc544\ub0b4\uac00 \ucd1d\uc7a5\uc73c\ub85c \uc788\ub294 \uc131\uc2e0\uc5ec\ub300\uc758 \uad50\uc9c1\uc6d0\ub4e4\uc744 \uc0ac\uc801\uc73c\ub85c \uc774\uc6a9\ud55c \uac83\uc73c\ub85c \ub4dc\ub7ec\ub0ac\ub2e4. \ub300\ubc95\uc6d0\uc740 9\uc77c \uc804 \uc804 \uc0ac\ub839\uad00\uc774 \uc131\uc2e0\uc5ec\ub300 \uc804 \ubd80\ucd1d\uc7a5 \uc870 \ubaa8 \uad50\uc218\ub97c \uace0\uc18c\ud55c \uba85\uc608\ud6fc\uc190 \uc0ac\uac74 \uc0c1\uace0\uc2ec\uc5d0\uc11c \uc870 \uad50\uc218\uc758 \uc758\ud639 \uc81c\uae30\uac00 \uc77c\ubd80 \uc0ac\uc2e4\uc774\ub77c\uace0 \ubcf8 2\uc2ec \ud310\ub2e8\uc774 \uc815\ub2f9\ud558\ub2e4\uace0 \ud310\ub2e8\ud588\ub2e4. \uc870 \uad50\uc218\ub294 \uc9c0\ub09c 2009\ub144 \uc804 \uc804 \uc0ac\ub839\uad00\uc774 \uac15\uc6d0\ub3c4 \ud654\ucc9c\uc5d0\uc11c \uc18c\uc7a5 \uc2b9\uc9c4 \ucd95\ud558\ud30c\ud2f0\ub97c \uc5f4\uba74\uc11c \uc131\uc2e0\uc5ec\ub300 \uad50\uc9c1\uc6d0 20\uc5ec \uba85\uc744 \ud30c\ud2f0\uc6a9 \uc74c\uc2dd \uc900\ube44\uc640 \uc11c\ube59 \ub4f1 \ud589\uc0ac \uc694\uc6d0\uc73c\ub85c \ub3d9\uc6d0\ud588\ub2e4\ub294 \uc758\ud639\uc744 2013\ub144 \uc5b8\ub860 \ub4f1\uc5d0 \uc81c\uae30\ud588\ub2e4. 1\uc2ec\uc740 \uc131\uc2e0\uc5ec\ub300 \uc9c1\uc6d0\uc774 \uc790\ubc1c\uc801\uc73c\ub85c \ucc38\uc11d\ud588\ub2e4\uba70 \uc870 \uad50\uc218\uc5d0\uac8c \ubc8c\uae08 300\ub9cc\uc6d0 \uc744 \uc120\uace0\ud588\uc9c0\ub9cc 2\uc2ec\uc740 \uc77c\ubd80 \uc0ac\uc2e4\uc5d0 \ucc28\uc774\uac00 \uc788\ub354\ub77c\ub3c4 \uc2b9\uc9c4 \ud30c\ud2f0\uc5d0 \uc9c1\uc6d0\uacfc \ud559\uc0dd\uc744 \ub3d9\uc6d0\ud588\ub2e4\ub294 \ub0b4\uc6a9\uc740 \uc911\uc694 \ubd80\ubd84\uc774 \uc778\uc815\ub41c\ub2e4\uba70 1\uc2ec\uc744 \ud30c\uae30\ud558\uace0 \uc774 \ubd80\ubd84\uc744 \ubb34\uc8c4\ub85c \ubd24\ub2e4.", "sentence_answer": "1\uc2ec\uc740 \uc131\uc2e0\uc5ec\ub300 \uc9c1\uc6d0\uc774 \uc790\ubc1c\uc801\uc73c\ub85c \ucc38\uc11d\ud588\ub2e4\uba70 \uc870 \uad50\uc218\uc5d0\uac8c \ubc8c\uae08 300\ub9cc\uc6d0 \uc744 \uc120\uace0\ud588\uc9c0\ub9cc 2\uc2ec\uc740 \uc77c\ubd80 \uc0ac\uc2e4\uc5d0 \ucc28\uc774\uac00 \uc788\ub354\ub77c\ub3c4 \uc2b9\uc9c4 \ud30c\ud2f0\uc5d0 \uc9c1\uc6d0\uacfc \ud559\uc0dd\uc744 \ub3d9\uc6d0\ud588\ub2e4\ub294 \ub0b4\uc6a9\uc740 \uc911\uc694 \ubd80\ubd84\uc774 \uc778\uc815\ub41c\ub2e4\uba70 1\uc2ec\uc744 \ud30c\uae30\ud558\uace0 \uc774 \ubd80\ubd84\uc744 \ubb34\uc8c4\ub85c \ubd24\ub2e4.", "paragraph_id": "6457513-0-2", "paragraph_question": "question: 1\uc2ec\uc5d0\uc11c \uc131\uae34\uc5ec\ub300 \uc9c1\uc6d0\uc774 \uc790\ubc1c\uc801\uc73c\ub85c \ucc38\uc11d\ud588\ub2e4\uba70 \uc870 \uad50\uc218\uc5d0\uac8c \ub0b4\ub824\uc9c4 \ubc8c\uae08\ud615\uc740 \uc5bc\ub9c8\uc778\uac00?, context: 2017\ub144 2\uc6d44\uc77c \ub354\ubd88\uc5b4\ubbfc\uc8fc\ub2f9 \ubb38\uc7ac\uc778 \uc804 \ub300\ud45c\uc758 \ub300\uc120 \ucea0\ud504\uc5d0 \ud569\ub958\ud55c \uc804\uc778\ubc94 \uc804 \ud2b9\uc804\uc0ac\ub839\uad00\uc774 \uc544\ub0b4\uac00 \ucd1d\uc7a5\uc73c\ub85c \uc788\ub294 \uc131\uc2e0\uc5ec\ub300\uc758 \uad50\uc9c1\uc6d0\ub4e4\uc744 \uc0ac\uc801\uc73c\ub85c \uc774\uc6a9\ud55c \uac83\uc73c\ub85c \ub4dc\ub7ec\ub0ac\ub2e4. \ub300\ubc95\uc6d0\uc740 9\uc77c \uc804 \uc804 \uc0ac\ub839\uad00\uc774 \uc131\uc2e0\uc5ec\ub300 \uc804 \ubd80\ucd1d\uc7a5 \uc870 \ubaa8 \uad50\uc218\ub97c \uace0\uc18c\ud55c \uba85\uc608\ud6fc\uc190 \uc0ac\uac74 \uc0c1\uace0\uc2ec\uc5d0\uc11c \uc870 \uad50\uc218\uc758 \uc758\ud639 \uc81c\uae30\uac00 \uc77c\ubd80 \uc0ac\uc2e4\uc774\ub77c\uace0 \ubcf8 2\uc2ec \ud310\ub2e8\uc774 \uc815\ub2f9\ud558\ub2e4\uace0 \ud310\ub2e8\ud588\ub2e4. \uc870 \uad50\uc218\ub294 \uc9c0\ub09c 2009\ub144 \uc804 \uc804 \uc0ac\ub839\uad00\uc774 \uac15\uc6d0\ub3c4 \ud654\ucc9c\uc5d0\uc11c \uc18c\uc7a5 \uc2b9\uc9c4 \ucd95\ud558\ud30c\ud2f0\ub97c \uc5f4\uba74\uc11c \uc131\uc2e0\uc5ec\ub300 \uad50\uc9c1\uc6d0 20\uc5ec \uba85\uc744 \ud30c\ud2f0\uc6a9 \uc74c\uc2dd \uc900\ube44\uc640 \uc11c\ube59 \ub4f1 \ud589\uc0ac \uc694\uc6d0\uc73c\ub85c \ub3d9\uc6d0\ud588\ub2e4\ub294 \uc758\ud639\uc744 2013\ub144 \uc5b8\ub860 \ub4f1\uc5d0 \uc81c\uae30\ud588\ub2e4. 1\uc2ec\uc740 \uc131\uc2e0\uc5ec\ub300 \uc9c1\uc6d0\uc774 \uc790\ubc1c\uc801\uc73c\ub85c \ucc38\uc11d\ud588\ub2e4\uba70 \uc870 \uad50\uc218\uc5d0\uac8c \ubc8c\uae08 300\ub9cc\uc6d0\uc744 \uc120\uace0\ud588\uc9c0\ub9cc 2\uc2ec\uc740 \uc77c\ubd80 \uc0ac\uc2e4\uc5d0 \ucc28\uc774\uac00 \uc788\ub354\ub77c\ub3c4 \uc2b9\uc9c4 \ud30c\ud2f0\uc5d0 \uc9c1\uc6d0\uacfc \ud559\uc0dd\uc744 \ub3d9\uc6d0\ud588\ub2e4\ub294 \ub0b4\uc6a9\uc740 \uc911\uc694 \ubd80\ubd84\uc774 \uc778\uc815\ub41c\ub2e4\uba70 1\uc2ec\uc744 \ud30c\uae30\ud558\uace0 \uc774 \ubd80\ubd84\uc744 \ubb34\uc8c4\ub85c \ubd24\ub2e4."} {"question": "\uc2a4\ud0a4\uc810\ud504\ub294 \uc5b8\uc81c\ubd80\ud130 \ub3d9\uacc4 \uc62c\ub9bc\ud53d\uc758 \uc815\uc2dd \uc885\ubaa9\uc778\uac00?", "paragraph": "\uc62c\ub9bc\ud53d \uc885\ubaa9\uc740 \ucd1d 33\uac1c\ubd80\ubb38 52\uac1c \uc885\ubaa9\uc5d0\uc11c \uc57d 400\uac1c\uc758 \uacbd\uae30\ub85c \uc774\ub8e8\uc5b4\uc838\uc788\ub2e4. \uc608\ub97c \ub4e4\uc5b4\uc11c \ud558\uacc4 \uc62c\ub9bc\ud53d \ubd80\ubb38\uc778 \ub808\uc2ac\ub9c1\uc740 \uc790\uc720\ud615\uacfc \uadf8\ub808\ucf54\ub85c\ub9cc\ud615\uc758 \ub450 \uc885\ubaa9\uc73c\ub85c \ub098\ub25c\ub2e4. \uc5ec\uae30\uc5d0\uc11c 10\uacbd\uae30\ub294 \ub0a8\uc790\ubd80, 4\uacbd\uae30\ub294 \uc5ec\uc790\ubd80\ub85c \uc5f4\ub9ac\uba70 \ubd84\ub958\uae30\uc900\uc740 \uccb4\uc911\uc774\ub2e4. \ud558\uacc4 \uc62c\ub9bc\ud53d\uc740 26\uac1c, \ub3d9\uacc4 \uc62c\ub9bc\ud53d\uc740 7\uac1c \ubd80\ubb38\uc73c\ub85c \uc774\ub8e8\uc5b4\uc838\uc788\ub2e4. \ud558\uacc4 \uc62c\ub9bc\ud53d\uc5d0\uc11c\ub294 \uc721\uc0c1, \uc218\uc601, \ud39c\uc2f1, \uccb4\uc870\uac00 1\ud68c \ub300\ud68c\ub54c\ubd80\ud130 \ud55c\ubc88\ub3c4 \ube60\uc9d0\uc5c6\uc774 \uc815\uc2dd\uc885\ubaa9\uc774\uc5c8\uc73c\uba70, \ub3d9\uacc4 \uc62c\ub9bc\ud53d\uc5d0\uc11c\ub294 \ud06c\ub85c\uc2a4\ucee8\ud2b8\ub9ac, \ud53c\uaca8 \uc2a4\ucf00\uc774\ud305, \uc544\uc774\uc2a4 \ud558\ud0a4, \ub178\ub974\ub515 \ubcf5\ud569, \uc2a4\ud0a4 \uc810\ud504, \uc2a4\ud53c\ub4dc \uc2a4\ucf00\uc774\ud305\uc774 1924\ub144 \ub3d9\uacc4 \uc62c\ub9bc\ud53d\ubd80\ud130 \ube60\uc9d0\uc5c6\uc774 \uc815\uc2dd\uc885\ubaa9\uc774\uc5c8\ub2e4. \ubc30\ub4dc\ubbfc\ud134, \ub18d\uad6c, \ubc30\uad6c\uc640 \uac19\uc740 \uc815\uc2dd\uc885\ubaa9\ub4e4\uc740 \ucc98\uc74c\uc5d0\ub294 \uc2dc\ubc94\uc885\ubaa9\uc774\uc5c8\uc73c\uba70 \uadf8 \ud6c4\uc5d0 \uc815\uc2dd\uc885\ubaa9\uc73c\ub85c \uc2b9\uc778 \ub418\uc5c8\ub2e4. \uc57c\uad6c\ucc98\ub7fc \uc608\uc804\uc5d0\ub294 \uc815\uc2dd\uc885\ubaa9 \uc774\uc5c8\uc9c0\ub9cc \uc9c0\uae08\uc740 \uc815\uc2dd \uc885\ubaa9\uc5d0\uc11c \ube60\uc9c4 \uc885\ubaa9\ub3c4 \uc788\ub2e4.", "answer": "1924\ub144", "sentence": "\ub3d9\uacc4 \uc62c\ub9bc\ud53d\uc5d0\uc11c\ub294 \ud06c\ub85c\uc2a4\ucee8\ud2b8\ub9ac, \ud53c\uaca8 \uc2a4\ucf00\uc774\ud305, \uc544\uc774\uc2a4 \ud558\ud0a4, \ub178\ub974\ub515 \ubcf5\ud569, \uc2a4\ud0a4 \uc810\ud504, \uc2a4\ud53c\ub4dc \uc2a4\ucf00\uc774\ud305\uc774 1924\ub144 \ub3d9\uacc4 \uc62c\ub9bc\ud53d\ubd80\ud130 \ube60\uc9d0\uc5c6\uc774 \uc815\uc2dd\uc885\ubaa9\uc774\uc5c8\ub2e4.", "paragraph_sentence": "\uc62c\ub9bc\ud53d \uc885\ubaa9\uc740 \ucd1d 33\uac1c\ubd80\ubb38 52\uac1c \uc885\ubaa9\uc5d0\uc11c \uc57d 400\uac1c\uc758 \uacbd\uae30\ub85c \uc774\ub8e8\uc5b4\uc838\uc788\ub2e4. \uc608\ub97c \ub4e4\uc5b4\uc11c \ud558\uacc4 \uc62c\ub9bc\ud53d \ubd80\ubb38\uc778 \ub808\uc2ac\ub9c1\uc740 \uc790\uc720\ud615\uacfc \uadf8\ub808\ucf54\ub85c\ub9cc\ud615\uc758 \ub450 \uc885\ubaa9\uc73c\ub85c \ub098\ub25c\ub2e4. \uc5ec\uae30\uc5d0\uc11c 10\uacbd\uae30\ub294 \ub0a8\uc790\ubd80, 4\uacbd\uae30\ub294 \uc5ec\uc790\ubd80\ub85c \uc5f4\ub9ac\uba70 \ubd84\ub958\uae30\uc900\uc740 \uccb4\uc911\uc774\ub2e4. \ud558\uacc4 \uc62c\ub9bc\ud53d\uc740 26\uac1c, \ub3d9\uacc4 \uc62c\ub9bc\ud53d\uc740 7\uac1c \ubd80\ubb38\uc73c\ub85c \uc774\ub8e8\uc5b4\uc838\uc788\ub2e4. \ud558\uacc4 \uc62c\ub9bc\ud53d\uc5d0\uc11c\ub294 \uc721\uc0c1, \uc218\uc601, \ud39c\uc2f1, \uccb4\uc870\uac00 1\ud68c \ub300\ud68c\ub54c\ubd80\ud130 \ud55c\ubc88\ub3c4 \ube60\uc9d0\uc5c6\uc774 \uc815\uc2dd\uc885\ubaa9\uc774\uc5c8\uc73c\uba70, \ub3d9\uacc4 \uc62c\ub9bc\ud53d\uc5d0\uc11c\ub294 \ud06c\ub85c\uc2a4\ucee8\ud2b8\ub9ac, \ud53c\uaca8 \uc2a4\ucf00\uc774\ud305, \uc544\uc774\uc2a4 \ud558\ud0a4, \ub178\ub974\ub515 \ubcf5\ud569, \uc2a4\ud0a4 \uc810\ud504, \uc2a4\ud53c\ub4dc \uc2a4\ucf00\uc774\ud305\uc774 1924\ub144 \ub3d9\uacc4 \uc62c\ub9bc\ud53d\ubd80\ud130 \ube60\uc9d0\uc5c6\uc774 \uc815\uc2dd\uc885\ubaa9\uc774\uc5c8\ub2e4. \ubc30\ub4dc\ubbfc\ud134, \ub18d\uad6c, \ubc30\uad6c\uc640 \uac19\uc740 \uc815\uc2dd\uc885\ubaa9\ub4e4\uc740 \ucc98\uc74c\uc5d0\ub294 \uc2dc\ubc94\uc885\ubaa9\uc774\uc5c8\uc73c\uba70 \uadf8 \ud6c4\uc5d0 \uc815\uc2dd\uc885\ubaa9\uc73c\ub85c \uc2b9\uc778 \ub418\uc5c8\ub2e4. \uc57c\uad6c\ucc98\ub7fc \uc608\uc804\uc5d0\ub294 \uc815\uc2dd\uc885\ubaa9 \uc774\uc5c8\uc9c0\ub9cc \uc9c0\uae08\uc740 \uc815\uc2dd \uc885\ubaa9\uc5d0\uc11c \ube60\uc9c4 \uc885\ubaa9\ub3c4 \uc788\ub2e4.", "paragraph_answer": "\uc62c\ub9bc\ud53d \uc885\ubaa9\uc740 \ucd1d 33\uac1c\ubd80\ubb38 52\uac1c \uc885\ubaa9\uc5d0\uc11c \uc57d 400\uac1c\uc758 \uacbd\uae30\ub85c \uc774\ub8e8\uc5b4\uc838\uc788\ub2e4. \uc608\ub97c \ub4e4\uc5b4\uc11c \ud558\uacc4 \uc62c\ub9bc\ud53d \ubd80\ubb38\uc778 \ub808\uc2ac\ub9c1\uc740 \uc790\uc720\ud615\uacfc \uadf8\ub808\ucf54\ub85c\ub9cc\ud615\uc758 \ub450 \uc885\ubaa9\uc73c\ub85c \ub098\ub25c\ub2e4. \uc5ec\uae30\uc5d0\uc11c 10\uacbd\uae30\ub294 \ub0a8\uc790\ubd80, 4\uacbd\uae30\ub294 \uc5ec\uc790\ubd80\ub85c \uc5f4\ub9ac\uba70 \ubd84\ub958\uae30\uc900\uc740 \uccb4\uc911\uc774\ub2e4. \ud558\uacc4 \uc62c\ub9bc\ud53d\uc740 26\uac1c, \ub3d9\uacc4 \uc62c\ub9bc\ud53d\uc740 7\uac1c \ubd80\ubb38\uc73c\ub85c \uc774\ub8e8\uc5b4\uc838\uc788\ub2e4. \ud558\uacc4 \uc62c\ub9bc\ud53d\uc5d0\uc11c\ub294 \uc721\uc0c1, \uc218\uc601, \ud39c\uc2f1, \uccb4\uc870\uac00 1\ud68c \ub300\ud68c\ub54c\ubd80\ud130 \ud55c\ubc88\ub3c4 \ube60\uc9d0\uc5c6\uc774 \uc815\uc2dd\uc885\ubaa9\uc774\uc5c8\uc73c\uba70, \ub3d9\uacc4 \uc62c\ub9bc\ud53d\uc5d0\uc11c\ub294 \ud06c\ub85c\uc2a4\ucee8\ud2b8\ub9ac, \ud53c\uaca8 \uc2a4\ucf00\uc774\ud305, \uc544\uc774\uc2a4 \ud558\ud0a4, \ub178\ub974\ub515 \ubcf5\ud569, \uc2a4\ud0a4 \uc810\ud504, \uc2a4\ud53c\ub4dc \uc2a4\ucf00\uc774\ud305\uc774 1924\ub144 \ub3d9\uacc4 \uc62c\ub9bc\ud53d\ubd80\ud130 \ube60\uc9d0\uc5c6\uc774 \uc815\uc2dd\uc885\ubaa9\uc774\uc5c8\ub2e4. \ubc30\ub4dc\ubbfc\ud134, \ub18d\uad6c, \ubc30\uad6c\uc640 \uac19\uc740 \uc815\uc2dd\uc885\ubaa9\ub4e4\uc740 \ucc98\uc74c\uc5d0\ub294 \uc2dc\ubc94\uc885\ubaa9\uc774\uc5c8\uc73c\uba70 \uadf8 \ud6c4\uc5d0 \uc815\uc2dd\uc885\ubaa9\uc73c\ub85c \uc2b9\uc778 \ub418\uc5c8\ub2e4. \uc57c\uad6c\ucc98\ub7fc \uc608\uc804\uc5d0\ub294 \uc815\uc2dd\uc885\ubaa9 \uc774\uc5c8\uc9c0\ub9cc \uc9c0\uae08\uc740 \uc815\uc2dd \uc885\ubaa9\uc5d0\uc11c \ube60\uc9c4 \uc885\ubaa9\ub3c4 \uc788\ub2e4.", "sentence_answer": "\ub3d9\uacc4 \uc62c\ub9bc\ud53d\uc5d0\uc11c\ub294 \ud06c\ub85c\uc2a4\ucee8\ud2b8\ub9ac, \ud53c\uaca8 \uc2a4\ucf00\uc774\ud305, \uc544\uc774\uc2a4 \ud558\ud0a4, \ub178\ub974\ub515 \ubcf5\ud569, \uc2a4\ud0a4 \uc810\ud504, \uc2a4\ud53c\ub4dc \uc2a4\ucf00\uc774\ud305\uc774 1924\ub144 \ub3d9\uacc4 \uc62c\ub9bc\ud53d\ubd80\ud130 \ube60\uc9d0\uc5c6\uc774 \uc815\uc2dd\uc885\ubaa9\uc774\uc5c8\ub2e4.", "paragraph_id": "6588341-20-1", "paragraph_question": "question: \uc2a4\ud0a4\uc810\ud504\ub294 \uc5b8\uc81c\ubd80\ud130 \ub3d9\uacc4 \uc62c\ub9bc\ud53d\uc758 \uc815\uc2dd \uc885\ubaa9\uc778\uac00?, context: \uc62c\ub9bc\ud53d \uc885\ubaa9\uc740 \ucd1d 33\uac1c\ubd80\ubb38 52\uac1c \uc885\ubaa9\uc5d0\uc11c \uc57d 400\uac1c\uc758 \uacbd\uae30\ub85c \uc774\ub8e8\uc5b4\uc838\uc788\ub2e4. \uc608\ub97c \ub4e4\uc5b4\uc11c \ud558\uacc4 \uc62c\ub9bc\ud53d \ubd80\ubb38\uc778 \ub808\uc2ac\ub9c1\uc740 \uc790\uc720\ud615\uacfc \uadf8\ub808\ucf54\ub85c\ub9cc\ud615\uc758 \ub450 \uc885\ubaa9\uc73c\ub85c \ub098\ub25c\ub2e4. \uc5ec\uae30\uc5d0\uc11c 10\uacbd\uae30\ub294 \ub0a8\uc790\ubd80, 4\uacbd\uae30\ub294 \uc5ec\uc790\ubd80\ub85c \uc5f4\ub9ac\uba70 \ubd84\ub958\uae30\uc900\uc740 \uccb4\uc911\uc774\ub2e4. \ud558\uacc4 \uc62c\ub9bc\ud53d\uc740 26\uac1c, \ub3d9\uacc4 \uc62c\ub9bc\ud53d\uc740 7\uac1c \ubd80\ubb38\uc73c\ub85c \uc774\ub8e8\uc5b4\uc838\uc788\ub2e4. \ud558\uacc4 \uc62c\ub9bc\ud53d\uc5d0\uc11c\ub294 \uc721\uc0c1, \uc218\uc601, \ud39c\uc2f1, \uccb4\uc870\uac00 1\ud68c \ub300\ud68c\ub54c\ubd80\ud130 \ud55c\ubc88\ub3c4 \ube60\uc9d0\uc5c6\uc774 \uc815\uc2dd\uc885\ubaa9\uc774\uc5c8\uc73c\uba70, \ub3d9\uacc4 \uc62c\ub9bc\ud53d\uc5d0\uc11c\ub294 \ud06c\ub85c\uc2a4\ucee8\ud2b8\ub9ac, \ud53c\uaca8 \uc2a4\ucf00\uc774\ud305, \uc544\uc774\uc2a4 \ud558\ud0a4, \ub178\ub974\ub515 \ubcf5\ud569, \uc2a4\ud0a4 \uc810\ud504, \uc2a4\ud53c\ub4dc \uc2a4\ucf00\uc774\ud305\uc774 1924\ub144 \ub3d9\uacc4 \uc62c\ub9bc\ud53d\ubd80\ud130 \ube60\uc9d0\uc5c6\uc774 \uc815\uc2dd\uc885\ubaa9\uc774\uc5c8\ub2e4. \ubc30\ub4dc\ubbfc\ud134, \ub18d\uad6c, \ubc30\uad6c\uc640 \uac19\uc740 \uc815\uc2dd\uc885\ubaa9\ub4e4\uc740 \ucc98\uc74c\uc5d0\ub294 \uc2dc\ubc94\uc885\ubaa9\uc774\uc5c8\uc73c\uba70 \uadf8 \ud6c4\uc5d0 \uc815\uc2dd\uc885\ubaa9\uc73c\ub85c \uc2b9\uc778 \ub418\uc5c8\ub2e4. \uc57c\uad6c\ucc98\ub7fc \uc608\uc804\uc5d0\ub294 \uc815\uc2dd\uc885\ubaa9 \uc774\uc5c8\uc9c0\ub9cc \uc9c0\uae08\uc740 \uc815\uc2dd \uc885\ubaa9\uc5d0\uc11c \ube60\uc9c4 \uc885\ubaa9\ub3c4 \uc788\ub2e4."} {"question": "\ud3ec\ucf13\ubaac \uac8c\uc784\uc5d0\uc11c \ubc84\ud37c\uc5d0 \uc9c0\uc815\ub41c \uac12\uc774 \uc5c6\uae30 \ub54c\ubb38\uc5d0 \ubbf8\uc2f1\ub178 \ud604\uc0c1\uc774 \ubc1c\uc0dd\ud558\ub294 \uc7a5\uc18c\ub294?", "paragraph": "\ubbf8\uc2f1\ub178\ub294 \uc138 \uac1c\uc758 \uc5f0\uc0b0 \uc774\ubca4\ud2b8\uc758 \uacb0\uacfc\ub85c\uc11c \ub098\ud0c0\ub098\ub294 \ud604\uc0c1\uc774\ub2e4. \uccab \ubc88\uc9f8 \uc694\uc18c\ub294 \uac8c\uc784\uc758 \ub79c\ub364 \ub300\uc804 \uc2dc\uc2a4\ud15c\uc774\ub2e4. \ud50c\ub808\uc774\uc5b4\uac00 \uc57c\uc0dd \ud3ec\ucf13\ubaac\uacfc \uc870\uc6b0\ud560 \ub54c \uc811\uadfc\ud558\ub294 \ub370\uc774\ud130 \ubc84\ud37c \ub0b4\uc5d0\uc11c \uac01 \uc9c0\uc5ed\ubcc4 \ud3ec\ucf13\ubaac\uc5d0 \ud2b9\uc815 \uac12\uc744 \uc9c0\uc815\ud55c\ub2e4. \uadf8\ub7ec\ub098 \ud64d\ub828\uc12c \ub3d9\ucabd \ud574\uc548\uc5d0\uc11c\ub294 \uc774 \ubc84\ud37c\uc5d0 \uc9c0\uc815\ub41c \uac12\uc774 \uc5c6\uae30 \ub54c\ubb38\uc5d0, \uc774\uc804\uc5d0 \ubc29\ubb38\ud588\ub358 \uc9c0\uc5ed\uc758 \uc815\ubcf4\ub97c \ub300\uc2e0 \uc0ac\uc6a9\ud558\uac8c \ub41c\ub2e4. \ub450 \ubc88\uc9f8 \uc694\uc18c\ub294 \ud29c\ud1a0\ub9ac\uc5bc\uc774\ub2e4. \ud29c\ud1a0\ub9ac\uc5bc\uc5d0\uc11c\ub294 \ud50c\ub808\uc774\uc5b4\uc758 \uc774\ub984\uc774 \ub370\uc774\ud130 \ubc84\ud37c\uc5d0 \uc784\uc2dc\ub85c \uc800\uc7a5\ub41c\ub2e4. \uc774 \ub54c\ubb38\uc5d0 \ud64d\ub828\uc12c\uc5d0\uc11c \ud3ec\ucf13\ubaac\uc744 \uc870\uc6b0\ud560 \ub54c 16\uc9c4\uc218 \uac12\uc778 \ud50c\ub808\uc774\uc5b4 \uc774\ub984\uc5d0 \uc561\uc138\uc2a4\ud558\uac8c \ub41c\ub2e4. \uc138 \ubc88\uc9f8 \uc694\uc18c\ub294 \uac8c\uc784\uc758 \uc5d0\ub7ec \ucc98\ub9ac \uc2dc\uc2a4\ud15c\uc5d0 \uc788\ub2e4. \uc2e4\uc874\ud558\ub294 \ud3ec\ucf13\ubaac\uc774 \uc544\ub2cc \ub370\uc774\ud130 \ubc84\ud37c \uac12\uc774 \uc120\ud0dd\ub418\ub294 \uacbd\uc6b0, \ub204\ub77d\ub41c \ubc88\ud638\ub97c \uc758\ubbf8\ud558\ub294 \u201c\ubbf8\uc2f1\ub178\u201d\ub97c \uc774\ub984\uc73c\ub85c \ud55c \ud3ec\ucf13\ubaac\uc774 \ub098\ud0c0\ub098\uac8c \ub418\ub294 \uc11c\ube0c\ub8e8\ud2f4\uc774 \uad6c\ub3d9\ub41c\ub2e4.", "answer": "\ud64d\ub828\uc12c \ub3d9\ucabd \ud574\uc548", "sentence": "\uadf8\ub7ec\ub098 \ud64d\ub828\uc12c \ub3d9\ucabd \ud574\uc548 \uc5d0\uc11c\ub294 \uc774 \ubc84\ud37c\uc5d0 \uc9c0\uc815\ub41c \uac12\uc774 \uc5c6\uae30 \ub54c\ubb38\uc5d0, \uc774\uc804\uc5d0 \ubc29\ubb38\ud588\ub358 \uc9c0\uc5ed\uc758 \uc815\ubcf4\ub97c \ub300\uc2e0 \uc0ac\uc6a9\ud558\uac8c \ub41c\ub2e4.", "paragraph_sentence": "\ubbf8\uc2f1\ub178\ub294 \uc138 \uac1c\uc758 \uc5f0\uc0b0 \uc774\ubca4\ud2b8\uc758 \uacb0\uacfc\ub85c\uc11c \ub098\ud0c0\ub098\ub294 \ud604\uc0c1\uc774\ub2e4. \uccab \ubc88\uc9f8 \uc694\uc18c\ub294 \uac8c\uc784\uc758 \ub79c\ub364 \ub300\uc804 \uc2dc\uc2a4\ud15c\uc774\ub2e4. \ud50c\ub808\uc774\uc5b4\uac00 \uc57c\uc0dd \ud3ec\ucf13\ubaac\uacfc \uc870\uc6b0\ud560 \ub54c \uc811\uadfc\ud558\ub294 \ub370\uc774\ud130 \ubc84\ud37c \ub0b4\uc5d0\uc11c \uac01 \uc9c0\uc5ed\ubcc4 \ud3ec\ucf13\ubaac\uc5d0 \ud2b9\uc815 \uac12\uc744 \uc9c0\uc815\ud55c\ub2e4. \uadf8\ub7ec\ub098 \ud64d\ub828\uc12c \ub3d9\ucabd \ud574\uc548 \uc5d0\uc11c\ub294 \uc774 \ubc84\ud37c\uc5d0 \uc9c0\uc815\ub41c \uac12\uc774 \uc5c6\uae30 \ub54c\ubb38\uc5d0, \uc774\uc804\uc5d0 \ubc29\ubb38\ud588\ub358 \uc9c0\uc5ed\uc758 \uc815\ubcf4\ub97c \ub300\uc2e0 \uc0ac\uc6a9\ud558\uac8c \ub41c\ub2e4. \ub450 \ubc88\uc9f8 \uc694\uc18c\ub294 \ud29c\ud1a0\ub9ac\uc5bc\uc774\ub2e4. \ud29c\ud1a0\ub9ac\uc5bc\uc5d0\uc11c\ub294 \ud50c\ub808\uc774\uc5b4\uc758 \uc774\ub984\uc774 \ub370\uc774\ud130 \ubc84\ud37c\uc5d0 \uc784\uc2dc\ub85c \uc800\uc7a5\ub41c\ub2e4. \uc774 \ub54c\ubb38\uc5d0 \ud64d\ub828\uc12c\uc5d0\uc11c \ud3ec\ucf13\ubaac\uc744 \uc870\uc6b0\ud560 \ub54c 16\uc9c4\uc218 \uac12\uc778 \ud50c\ub808\uc774\uc5b4 \uc774\ub984\uc5d0 \uc561\uc138\uc2a4\ud558\uac8c \ub41c\ub2e4. \uc138 \ubc88\uc9f8 \uc694\uc18c\ub294 \uac8c\uc784\uc758 \uc5d0\ub7ec \ucc98\ub9ac \uc2dc\uc2a4\ud15c\uc5d0 \uc788\ub2e4. \uc2e4\uc874\ud558\ub294 \ud3ec\ucf13\ubaac\uc774 \uc544\ub2cc \ub370\uc774\ud130 \ubc84\ud37c \uac12\uc774 \uc120\ud0dd\ub418\ub294 \uacbd\uc6b0, \ub204\ub77d\ub41c \ubc88\ud638\ub97c \uc758\ubbf8\ud558\ub294 \u201c\ubbf8\uc2f1\ub178\u201d\ub97c \uc774\ub984\uc73c\ub85c \ud55c \ud3ec\ucf13\ubaac\uc774 \ub098\ud0c0\ub098\uac8c \ub418\ub294 \uc11c\ube0c\ub8e8\ud2f4\uc774 \uad6c\ub3d9\ub41c\ub2e4.", "paragraph_answer": "\ubbf8\uc2f1\ub178\ub294 \uc138 \uac1c\uc758 \uc5f0\uc0b0 \uc774\ubca4\ud2b8\uc758 \uacb0\uacfc\ub85c\uc11c \ub098\ud0c0\ub098\ub294 \ud604\uc0c1\uc774\ub2e4. \uccab \ubc88\uc9f8 \uc694\uc18c\ub294 \uac8c\uc784\uc758 \ub79c\ub364 \ub300\uc804 \uc2dc\uc2a4\ud15c\uc774\ub2e4. \ud50c\ub808\uc774\uc5b4\uac00 \uc57c\uc0dd \ud3ec\ucf13\ubaac\uacfc \uc870\uc6b0\ud560 \ub54c \uc811\uadfc\ud558\ub294 \ub370\uc774\ud130 \ubc84\ud37c \ub0b4\uc5d0\uc11c \uac01 \uc9c0\uc5ed\ubcc4 \ud3ec\ucf13\ubaac\uc5d0 \ud2b9\uc815 \uac12\uc744 \uc9c0\uc815\ud55c\ub2e4. \uadf8\ub7ec\ub098 \ud64d\ub828\uc12c \ub3d9\ucabd \ud574\uc548 \uc5d0\uc11c\ub294 \uc774 \ubc84\ud37c\uc5d0 \uc9c0\uc815\ub41c \uac12\uc774 \uc5c6\uae30 \ub54c\ubb38\uc5d0, \uc774\uc804\uc5d0 \ubc29\ubb38\ud588\ub358 \uc9c0\uc5ed\uc758 \uc815\ubcf4\ub97c \ub300\uc2e0 \uc0ac\uc6a9\ud558\uac8c \ub41c\ub2e4. \ub450 \ubc88\uc9f8 \uc694\uc18c\ub294 \ud29c\ud1a0\ub9ac\uc5bc\uc774\ub2e4. \ud29c\ud1a0\ub9ac\uc5bc\uc5d0\uc11c\ub294 \ud50c\ub808\uc774\uc5b4\uc758 \uc774\ub984\uc774 \ub370\uc774\ud130 \ubc84\ud37c\uc5d0 \uc784\uc2dc\ub85c \uc800\uc7a5\ub41c\ub2e4. \uc774 \ub54c\ubb38\uc5d0 \ud64d\ub828\uc12c\uc5d0\uc11c \ud3ec\ucf13\ubaac\uc744 \uc870\uc6b0\ud560 \ub54c 16\uc9c4\uc218 \uac12\uc778 \ud50c\ub808\uc774\uc5b4 \uc774\ub984\uc5d0 \uc561\uc138\uc2a4\ud558\uac8c \ub41c\ub2e4. \uc138 \ubc88\uc9f8 \uc694\uc18c\ub294 \uac8c\uc784\uc758 \uc5d0\ub7ec \ucc98\ub9ac \uc2dc\uc2a4\ud15c\uc5d0 \uc788\ub2e4. \uc2e4\uc874\ud558\ub294 \ud3ec\ucf13\ubaac\uc774 \uc544\ub2cc \ub370\uc774\ud130 \ubc84\ud37c \uac12\uc774 \uc120\ud0dd\ub418\ub294 \uacbd\uc6b0, \ub204\ub77d\ub41c \ubc88\ud638\ub97c \uc758\ubbf8\ud558\ub294 \u201c\ubbf8\uc2f1\ub178\u201d\ub97c \uc774\ub984\uc73c\ub85c \ud55c \ud3ec\ucf13\ubaac\uc774 \ub098\ud0c0\ub098\uac8c \ub418\ub294 \uc11c\ube0c\ub8e8\ud2f4\uc774 \uad6c\ub3d9\ub41c\ub2e4.", "sentence_answer": "\uadf8\ub7ec\ub098 \ud64d\ub828\uc12c \ub3d9\ucabd \ud574\uc548 \uc5d0\uc11c\ub294 \uc774 \ubc84\ud37c\uc5d0 \uc9c0\uc815\ub41c \uac12\uc774 \uc5c6\uae30 \ub54c\ubb38\uc5d0, \uc774\uc804\uc5d0 \ubc29\ubb38\ud588\ub358 \uc9c0\uc5ed\uc758 \uc815\ubcf4\ub97c \ub300\uc2e0 \uc0ac\uc6a9\ud558\uac8c \ub41c\ub2e4.", "paragraph_id": "6505619-1-0", "paragraph_question": "question: \ud3ec\ucf13\ubaac \uac8c\uc784\uc5d0\uc11c \ubc84\ud37c\uc5d0 \uc9c0\uc815\ub41c \uac12\uc774 \uc5c6\uae30 \ub54c\ubb38\uc5d0 \ubbf8\uc2f1\ub178 \ud604\uc0c1\uc774 \ubc1c\uc0dd\ud558\ub294 \uc7a5\uc18c\ub294?, context: \ubbf8\uc2f1\ub178\ub294 \uc138 \uac1c\uc758 \uc5f0\uc0b0 \uc774\ubca4\ud2b8\uc758 \uacb0\uacfc\ub85c\uc11c \ub098\ud0c0\ub098\ub294 \ud604\uc0c1\uc774\ub2e4. \uccab \ubc88\uc9f8 \uc694\uc18c\ub294 \uac8c\uc784\uc758 \ub79c\ub364 \ub300\uc804 \uc2dc\uc2a4\ud15c\uc774\ub2e4. \ud50c\ub808\uc774\uc5b4\uac00 \uc57c\uc0dd \ud3ec\ucf13\ubaac\uacfc \uc870\uc6b0\ud560 \ub54c \uc811\uadfc\ud558\ub294 \ub370\uc774\ud130 \ubc84\ud37c \ub0b4\uc5d0\uc11c \uac01 \uc9c0\uc5ed\ubcc4 \ud3ec\ucf13\ubaac\uc5d0 \ud2b9\uc815 \uac12\uc744 \uc9c0\uc815\ud55c\ub2e4. \uadf8\ub7ec\ub098 \ud64d\ub828\uc12c \ub3d9\ucabd \ud574\uc548\uc5d0\uc11c\ub294 \uc774 \ubc84\ud37c\uc5d0 \uc9c0\uc815\ub41c \uac12\uc774 \uc5c6\uae30 \ub54c\ubb38\uc5d0, \uc774\uc804\uc5d0 \ubc29\ubb38\ud588\ub358 \uc9c0\uc5ed\uc758 \uc815\ubcf4\ub97c \ub300\uc2e0 \uc0ac\uc6a9\ud558\uac8c \ub41c\ub2e4. \ub450 \ubc88\uc9f8 \uc694\uc18c\ub294 \ud29c\ud1a0\ub9ac\uc5bc\uc774\ub2e4. \ud29c\ud1a0\ub9ac\uc5bc\uc5d0\uc11c\ub294 \ud50c\ub808\uc774\uc5b4\uc758 \uc774\ub984\uc774 \ub370\uc774\ud130 \ubc84\ud37c\uc5d0 \uc784\uc2dc\ub85c \uc800\uc7a5\ub41c\ub2e4. \uc774 \ub54c\ubb38\uc5d0 \ud64d\ub828\uc12c\uc5d0\uc11c \ud3ec\ucf13\ubaac\uc744 \uc870\uc6b0\ud560 \ub54c 16\uc9c4\uc218 \uac12\uc778 \ud50c\ub808\uc774\uc5b4 \uc774\ub984\uc5d0 \uc561\uc138\uc2a4\ud558\uac8c \ub41c\ub2e4. \uc138 \ubc88\uc9f8 \uc694\uc18c\ub294 \uac8c\uc784\uc758 \uc5d0\ub7ec \ucc98\ub9ac \uc2dc\uc2a4\ud15c\uc5d0 \uc788\ub2e4. \uc2e4\uc874\ud558\ub294 \ud3ec\ucf13\ubaac\uc774 \uc544\ub2cc \ub370\uc774\ud130 \ubc84\ud37c \uac12\uc774 \uc120\ud0dd\ub418\ub294 \uacbd\uc6b0, \ub204\ub77d\ub41c \ubc88\ud638\ub97c \uc758\ubbf8\ud558\ub294 \u201c\ubbf8\uc2f1\ub178\u201d\ub97c \uc774\ub984\uc73c\ub85c \ud55c \ud3ec\ucf13\ubaac\uc774 \ub098\ud0c0\ub098\uac8c \ub418\ub294 \uc11c\ube0c\ub8e8\ud2f4\uc774 \uad6c\ub3d9\ub41c\ub2e4."} {"question": "\uadf8\uc758 2013\ub144 \uc2dc\uc98c \uc5f0\ubd09\uc740 2012\ub144 \ub300\ube44\ud558\uc5ec \uc5bc\ub9c8\ub9cc\ud07c \uc904\uc5c8\ub294\uac00?", "paragraph": "\uc2dc\uc98c \ucd08\ubc18\uc778 4\uc6d4\uc5d0 \ud0c0\uc728 0.306, 19\uc548\ud0c0 10\ub4dd\uc810\uc73c\ub85c \ud300\uc758 \uacf5\uaca9\uc744 \uc774\ub04c\uc5c8\uc73c\ub098, \uc774\ud6c4 \ud0c0\uc728\uc774 5\uc6d4\uc5d0\ub294 0.202, 6\uc6d4\uc5d0\ub294 \ubb34\ub98e \ubd80\uc0c1\ub4f1\uc774 \uacb9\uce58\uba70 0.143\uae4c\uc9c0 \ub0b4\ub824\uac14\ub2e4. \uadf9\uc2ec\ud55c \ubd80\uc9c4\uc73c\ub85c \ud558\uc704\ud0c0\uc21c\uc73c\ub85c \ubc00\ub824\ub0ac\uace0, \ub2f9\uc2dc \uac10\ub3c5\uc774\uc5c8\ub358 \uae40\uc9c4\uc6b1\uc744 \ube44\ub86f\ud574 \uc774\ud1a0 \uc4f0\ud1a0\ubb34 \uc218\uc11d \ucf54\uce58 \ub4f1\uc774 \ubd80\uc9c4 \ud0c8\ucd9c\uc744 \uc704\ud574 \ub9ce\uc740 \ub178\ub825\uc744 \uae30\uc6b8\uc600\uc73c\ub098 \ubd80\uc9c4\uc740 \uae38\uc5b4\uc84c\ub2e4. \uacb0\uad6d 6\uc6d4 \ub9d0\uc5d0 \uadf8\ub3d9\uc548 \uadf8\ub97c \uafb8\uc900\ud788 \uc9c0\ub3c4\ud574\uc654\ub358 \ub2f9\uc2dc 2\uad70 \uac10\ub3c5\uc774\uc5c8\ub358 \uc1a1\uc7ac\ubc15\uc774 1\uad70 \ud0c0\uaca9 \ucf54\uce58\ub85c \uc62c\ub77c\uc654\uace0, \ub178\uc2a4\ud15d \uc790\uc138\uc640 \uc5b4\ud37c \uc2a4\uc719 \ub300\uc2e0 \uc9e7\uac8c \ub04a\uc5b4\uce58\ub294 \ub545\ubcfc \uc704\uc8fc\uc758 \ud0c0\uaca9\uc5d0 \uc8fc\ub825\ud588\ub2e4. \ud55c\ud3b8 6\uc6d4 29\uc77c \ubd80\uc0c1\uc73c\ub85c \ud300 \uc804\ub825\uc5d0\uc11c \uc81c\uc678\ub41c \uc784\uc7ac\ucca0\uc744 \ub300\uc2e0\ud574 \uc8fc\uc7a5\uc744 \ub9e1\uc740 \uadf8\ub294, 7\uc6d4 0.247, 8\uc6d4 0.262\ub85c \ud0c0\uc728\uc774 \uc0c1\uc2b9\ud558\ub294 \ud398\uc774\uc2a4\ub97c \ubcf4\uc5ec\uc8fc\uc5c8\ub2e4. 7\uc6d4 27\uc77c\uc5d0\ub294 \uc5ed\uc804 \ub05d\ub0b4\uae30 \uc548\ud0c0\ub85c \ud504\ub85c \ub370\ubdd4 \ud6c4 \uccab \ub05d\ub0b4\uae30 \uc548\ud0c0\ub97c \uccd0\ub0c8\ub2e4. \uc62c \uc2dc\uc98c 121\uacbd\uae30\uc5d0\uc11c 2\ud560\ub300 \ud0c0\uc728(\ud0c0\uc728 \ucd5c\ud558\uc704), 21\ub3c4\ub8e8\ub97c \uae30\ub85d\ud558\uba70 \uc9c0\uae08\uae4c\uc9c0 \uce58\ub800\ub358 \uc2dc\uc98c \uc911 \uac00\uc7a5 \ubd80\uc9c4\ud55c \ubaa8\uc2b5\uc744 \ubcf4\uc600\ub358 \uadf8\ub294, \"\uc0ac\uc2e4 \ud06c\uac8c \uc544\ud508 \ub370\ub294 \uc5c6\uc5c8\ub2e4. \uc774\uc0c1\ud558\uac8c \ubc29\ub9dd\uc774\uac00 \ub9de\uc9c0 \uc54a\ub354\ub77c. \uc88b\uc740 \uacbd\ud5d8\uc774\uc5c8\ub2e4. \uc774\ub807\uac8c \ud558\uba74 \uc548 \ub41c\ub2e4\ub294 \uac78 \uae68\ub2eb\uac8c \ub410\ub2e4. \uc57c\uad6c\uac00 \ucc38 \uc5b4\ub835\ub2e4\"\uace0 \ub9d0\ud588\ub2e4. \ub450\uc0b0 \ubca0\uc5b4\uc2a4\ub294 \ud398\ub10c\ud2b8\ub808\uc774\uc2a4 3\uc704\ub97c \uae30\ub85d\ud558\uba70, \uc900\ud50c\ub808\uc774\uc624\ud504\uc5d0\uc11c 4\uc704 \ub86f\ub370 \uc790\uc774\uc5b8\uce20\uc640 \ub9de\ubd99\uc5b4, \uccab \ub450 \uacbd\uae30\uc5d0\uc11c 2\uc5f0\ud328\ub97c \ub2f9\ud588\ub2e4. \uadf8\ub7ec\ub098 3\ucc28\uc804\uc5d0\uc11c \uadf8\uac00 1\ud68c\ucd08 \uc120\ub450 \ud0c0\uc790\ub85c \ub098\uc11c \ubab8\uc5d0 \ub9de\ub294 \ubcfc\ub85c \ucd9c\ub8e8\ud55c \ub4a4, 2\ub8e8 \ub3c4\ub8e8\uc5d0 \uc131\uacf5\ud558\uace0, \ud6c4\uc18d\ud0c0 \ub54c \ub4dd\uc810\uc744 \ud574 \ubd80\uc0c1 \ud22c\ud63c\uc744 \ubcf4\uc5ec\uc8fc\uc5c8\uace0, \uc774\ub7f0 \ud65c\uc57d \ub355\uc5d0 \ub450\uc0b0 \ubca0\uc5b4\uc2a4\ub294 3\ucc28\uc804\uc744 \uc2b9\ub9ac\ud588\ub2e4. \ud558\uc9c0\ub9cc, \uc2ec\uac01\ud55c \uc885\uc544\ub9ac \ubd80\uc0c1\uc73c\ub85c \uc778\ud574 \ub0a8\uc740 \uc2dc\ub9ac\uc988\uc5d0 \ucd9c\uc804\ud558\uc9c0 \ubabb\ud588\uace0, \ub450\uc0b0 \ubca0\uc5b4\uc2a4\ub294 \ub86f\ub370 \uc790\uc774\uc5b8\uce20\uc5d0\uac8c 4\ucc28\uc804\uc744 \ub0b4\uc8fc\uba74\uc11c \ud50c\ub808\uc774\uc624\ud504 \uc9c4\ucd9c\uc5d0 \uc2e4\ud328\ud588\ub2e4. \uc62c \uc2dc\uc98c \ubd80\uc9c4\uc73c\ub85c \uc778\ud574 800\ub9cc\uc6d0\uc774 \uae4e\uc778 1\uc5b5 9700\ub9cc\uc6d0\uc5d0 2013 \uc2dc\uc98c \uc5f0\ubd09\uc744 \uacc4\uc57d\ud588\ub2e4.", "answer": "800\ub9cc\uc6d0", "sentence": "\uc62c \uc2dc\uc98c \ubd80\uc9c4\uc73c\ub85c \uc778\ud574 800\ub9cc\uc6d0 \uc774 \uae4e\uc778 1\uc5b5 9700\ub9cc\uc6d0\uc5d0 2013 \uc2dc\uc98c \uc5f0\ubd09\uc744 \uacc4\uc57d\ud588\ub2e4.", "paragraph_sentence": "\uc2dc\uc98c \ucd08\ubc18\uc778 4\uc6d4\uc5d0 \ud0c0\uc728 0.306, 19\uc548\ud0c0 10\ub4dd\uc810\uc73c\ub85c \ud300\uc758 \uacf5\uaca9\uc744 \uc774\ub04c\uc5c8\uc73c\ub098, \uc774\ud6c4 \ud0c0\uc728\uc774 5\uc6d4\uc5d0\ub294 0.202, 6\uc6d4\uc5d0\ub294 \ubb34\ub98e \ubd80\uc0c1\ub4f1\uc774 \uacb9\uce58\uba70 0.143\uae4c\uc9c0 \ub0b4\ub824\uac14\ub2e4. \uadf9\uc2ec\ud55c \ubd80\uc9c4\uc73c\ub85c \ud558\uc704\ud0c0\uc21c\uc73c\ub85c \ubc00\ub824\ub0ac\uace0, \ub2f9\uc2dc \uac10\ub3c5\uc774\uc5c8\ub358 \uae40\uc9c4\uc6b1\uc744 \ube44\ub86f\ud574 \uc774\ud1a0 \uc4f0\ud1a0\ubb34 \uc218\uc11d \ucf54\uce58 \ub4f1\uc774 \ubd80\uc9c4 \ud0c8\ucd9c\uc744 \uc704\ud574 \ub9ce\uc740 \ub178\ub825\uc744 \uae30\uc6b8\uc600\uc73c\ub098 \ubd80\uc9c4\uc740 \uae38\uc5b4\uc84c\ub2e4. \uacb0\uad6d 6\uc6d4 \ub9d0\uc5d0 \uadf8\ub3d9\uc548 \uadf8\ub97c \uafb8\uc900\ud788 \uc9c0\ub3c4\ud574\uc654\ub358 \ub2f9\uc2dc 2\uad70 \uac10\ub3c5\uc774\uc5c8\ub358 \uc1a1\uc7ac\ubc15\uc774 1\uad70 \ud0c0\uaca9 \ucf54\uce58\ub85c \uc62c\ub77c\uc654\uace0, \ub178\uc2a4\ud15d \uc790\uc138\uc640 \uc5b4\ud37c \uc2a4\uc719 \ub300\uc2e0 \uc9e7\uac8c \ub04a\uc5b4\uce58\ub294 \ub545\ubcfc \uc704\uc8fc\uc758 \ud0c0\uaca9\uc5d0 \uc8fc\ub825\ud588\ub2e4. \ud55c\ud3b8 6\uc6d4 29\uc77c \ubd80\uc0c1\uc73c\ub85c \ud300 \uc804\ub825\uc5d0\uc11c \uc81c\uc678\ub41c \uc784\uc7ac\ucca0\uc744 \ub300\uc2e0\ud574 \uc8fc\uc7a5\uc744 \ub9e1\uc740 \uadf8\ub294, 7\uc6d4 0.247, 8\uc6d4 0.262\ub85c \ud0c0\uc728\uc774 \uc0c1\uc2b9\ud558\ub294 \ud398\uc774\uc2a4\ub97c \ubcf4\uc5ec\uc8fc\uc5c8\ub2e4. 7\uc6d4 27\uc77c\uc5d0\ub294 \uc5ed\uc804 \ub05d\ub0b4\uae30 \uc548\ud0c0\ub85c \ud504\ub85c \ub370\ubdd4 \ud6c4 \uccab \ub05d\ub0b4\uae30 \uc548\ud0c0\ub97c \uccd0\ub0c8\ub2e4. \uc62c \uc2dc\uc98c 121\uacbd\uae30\uc5d0\uc11c 2\ud560\ub300 \ud0c0\uc728(\ud0c0\uc728 \ucd5c\ud558\uc704), 21\ub3c4\ub8e8\ub97c \uae30\ub85d\ud558\uba70 \uc9c0\uae08\uae4c\uc9c0 \uce58\ub800\ub358 \uc2dc\uc98c \uc911 \uac00\uc7a5 \ubd80\uc9c4\ud55c \ubaa8\uc2b5\uc744 \ubcf4\uc600\ub358 \uadf8\ub294, \"\uc0ac\uc2e4 \ud06c\uac8c \uc544\ud508 \ub370\ub294 \uc5c6\uc5c8\ub2e4. \uc774\uc0c1\ud558\uac8c \ubc29\ub9dd\uc774\uac00 \ub9de\uc9c0 \uc54a\ub354\ub77c. \uc88b\uc740 \uacbd\ud5d8\uc774\uc5c8\ub2e4. \uc774\ub807\uac8c \ud558\uba74 \uc548 \ub41c\ub2e4\ub294 \uac78 \uae68\ub2eb\uac8c \ub410\ub2e4. \uc57c\uad6c\uac00 \ucc38 \uc5b4\ub835\ub2e4\"\uace0 \ub9d0\ud588\ub2e4. \ub450\uc0b0 \ubca0\uc5b4\uc2a4\ub294 \ud398\ub10c\ud2b8\ub808\uc774\uc2a4 3\uc704\ub97c \uae30\ub85d\ud558\uba70, \uc900\ud50c\ub808\uc774\uc624\ud504\uc5d0\uc11c 4\uc704 \ub86f\ub370 \uc790\uc774\uc5b8\uce20\uc640 \ub9de\ubd99\uc5b4, \uccab \ub450 \uacbd\uae30\uc5d0\uc11c 2\uc5f0\ud328\ub97c \ub2f9\ud588\ub2e4. \uadf8\ub7ec\ub098 3\ucc28\uc804\uc5d0\uc11c \uadf8\uac00 1\ud68c\ucd08 \uc120\ub450 \ud0c0\uc790\ub85c \ub098\uc11c \ubab8\uc5d0 \ub9de\ub294 \ubcfc\ub85c \ucd9c\ub8e8\ud55c \ub4a4, 2\ub8e8 \ub3c4\ub8e8\uc5d0 \uc131\uacf5\ud558\uace0, \ud6c4\uc18d\ud0c0 \ub54c \ub4dd\uc810\uc744 \ud574 \ubd80\uc0c1 \ud22c\ud63c\uc744 \ubcf4\uc5ec\uc8fc\uc5c8\uace0, \uc774\ub7f0 \ud65c\uc57d \ub355\uc5d0 \ub450\uc0b0 \ubca0\uc5b4\uc2a4\ub294 3\ucc28\uc804\uc744 \uc2b9\ub9ac\ud588\ub2e4. \ud558\uc9c0\ub9cc, \uc2ec\uac01\ud55c \uc885\uc544\ub9ac \ubd80\uc0c1\uc73c\ub85c \uc778\ud574 \ub0a8\uc740 \uc2dc\ub9ac\uc988\uc5d0 \ucd9c\uc804\ud558\uc9c0 \ubabb\ud588\uace0, \ub450\uc0b0 \ubca0\uc5b4\uc2a4\ub294 \ub86f\ub370 \uc790\uc774\uc5b8\uce20\uc5d0\uac8c 4\ucc28\uc804\uc744 \ub0b4\uc8fc\uba74\uc11c \ud50c\ub808\uc774\uc624\ud504 \uc9c4\ucd9c\uc5d0 \uc2e4\ud328\ud588\ub2e4. \uc62c \uc2dc\uc98c \ubd80\uc9c4\uc73c\ub85c \uc778\ud574 800\ub9cc\uc6d0 \uc774 \uae4e\uc778 1\uc5b5 9700\ub9cc\uc6d0\uc5d0 2013 \uc2dc\uc98c \uc5f0\ubd09\uc744 \uacc4\uc57d\ud588\ub2e4. ", "paragraph_answer": "\uc2dc\uc98c \ucd08\ubc18\uc778 4\uc6d4\uc5d0 \ud0c0\uc728 0.306, 19\uc548\ud0c0 10\ub4dd\uc810\uc73c\ub85c \ud300\uc758 \uacf5\uaca9\uc744 \uc774\ub04c\uc5c8\uc73c\ub098, \uc774\ud6c4 \ud0c0\uc728\uc774 5\uc6d4\uc5d0\ub294 0.202, 6\uc6d4\uc5d0\ub294 \ubb34\ub98e \ubd80\uc0c1\ub4f1\uc774 \uacb9\uce58\uba70 0.143\uae4c\uc9c0 \ub0b4\ub824\uac14\ub2e4. \uadf9\uc2ec\ud55c \ubd80\uc9c4\uc73c\ub85c \ud558\uc704\ud0c0\uc21c\uc73c\ub85c \ubc00\ub824\ub0ac\uace0, \ub2f9\uc2dc \uac10\ub3c5\uc774\uc5c8\ub358 \uae40\uc9c4\uc6b1\uc744 \ube44\ub86f\ud574 \uc774\ud1a0 \uc4f0\ud1a0\ubb34 \uc218\uc11d \ucf54\uce58 \ub4f1\uc774 \ubd80\uc9c4 \ud0c8\ucd9c\uc744 \uc704\ud574 \ub9ce\uc740 \ub178\ub825\uc744 \uae30\uc6b8\uc600\uc73c\ub098 \ubd80\uc9c4\uc740 \uae38\uc5b4\uc84c\ub2e4. \uacb0\uad6d 6\uc6d4 \ub9d0\uc5d0 \uadf8\ub3d9\uc548 \uadf8\ub97c \uafb8\uc900\ud788 \uc9c0\ub3c4\ud574\uc654\ub358 \ub2f9\uc2dc 2\uad70 \uac10\ub3c5\uc774\uc5c8\ub358 \uc1a1\uc7ac\ubc15\uc774 1\uad70 \ud0c0\uaca9 \ucf54\uce58\ub85c \uc62c\ub77c\uc654\uace0, \ub178\uc2a4\ud15d \uc790\uc138\uc640 \uc5b4\ud37c \uc2a4\uc719 \ub300\uc2e0 \uc9e7\uac8c \ub04a\uc5b4\uce58\ub294 \ub545\ubcfc \uc704\uc8fc\uc758 \ud0c0\uaca9\uc5d0 \uc8fc\ub825\ud588\ub2e4. \ud55c\ud3b8 6\uc6d4 29\uc77c \ubd80\uc0c1\uc73c\ub85c \ud300 \uc804\ub825\uc5d0\uc11c \uc81c\uc678\ub41c \uc784\uc7ac\ucca0\uc744 \ub300\uc2e0\ud574 \uc8fc\uc7a5\uc744 \ub9e1\uc740 \uadf8\ub294, 7\uc6d4 0.247, 8\uc6d4 0.262\ub85c \ud0c0\uc728\uc774 \uc0c1\uc2b9\ud558\ub294 \ud398\uc774\uc2a4\ub97c \ubcf4\uc5ec\uc8fc\uc5c8\ub2e4. 7\uc6d4 27\uc77c\uc5d0\ub294 \uc5ed\uc804 \ub05d\ub0b4\uae30 \uc548\ud0c0\ub85c \ud504\ub85c \ub370\ubdd4 \ud6c4 \uccab \ub05d\ub0b4\uae30 \uc548\ud0c0\ub97c \uccd0\ub0c8\ub2e4. \uc62c \uc2dc\uc98c 121\uacbd\uae30\uc5d0\uc11c 2\ud560\ub300 \ud0c0\uc728(\ud0c0\uc728 \ucd5c\ud558\uc704), 21\ub3c4\ub8e8\ub97c \uae30\ub85d\ud558\uba70 \uc9c0\uae08\uae4c\uc9c0 \uce58\ub800\ub358 \uc2dc\uc98c \uc911 \uac00\uc7a5 \ubd80\uc9c4\ud55c \ubaa8\uc2b5\uc744 \ubcf4\uc600\ub358 \uadf8\ub294, \"\uc0ac\uc2e4 \ud06c\uac8c \uc544\ud508 \ub370\ub294 \uc5c6\uc5c8\ub2e4. \uc774\uc0c1\ud558\uac8c \ubc29\ub9dd\uc774\uac00 \ub9de\uc9c0 \uc54a\ub354\ub77c. \uc88b\uc740 \uacbd\ud5d8\uc774\uc5c8\ub2e4. \uc774\ub807\uac8c \ud558\uba74 \uc548 \ub41c\ub2e4\ub294 \uac78 \uae68\ub2eb\uac8c \ub410\ub2e4. \uc57c\uad6c\uac00 \ucc38 \uc5b4\ub835\ub2e4\"\uace0 \ub9d0\ud588\ub2e4. \ub450\uc0b0 \ubca0\uc5b4\uc2a4\ub294 \ud398\ub10c\ud2b8\ub808\uc774\uc2a4 3\uc704\ub97c \uae30\ub85d\ud558\uba70, \uc900\ud50c\ub808\uc774\uc624\ud504\uc5d0\uc11c 4\uc704 \ub86f\ub370 \uc790\uc774\uc5b8\uce20\uc640 \ub9de\ubd99\uc5b4, \uccab \ub450 \uacbd\uae30\uc5d0\uc11c 2\uc5f0\ud328\ub97c \ub2f9\ud588\ub2e4. \uadf8\ub7ec\ub098 3\ucc28\uc804\uc5d0\uc11c \uadf8\uac00 1\ud68c\ucd08 \uc120\ub450 \ud0c0\uc790\ub85c \ub098\uc11c \ubab8\uc5d0 \ub9de\ub294 \ubcfc\ub85c \ucd9c\ub8e8\ud55c \ub4a4, 2\ub8e8 \ub3c4\ub8e8\uc5d0 \uc131\uacf5\ud558\uace0, \ud6c4\uc18d\ud0c0 \ub54c \ub4dd\uc810\uc744 \ud574 \ubd80\uc0c1 \ud22c\ud63c\uc744 \ubcf4\uc5ec\uc8fc\uc5c8\uace0, \uc774\ub7f0 \ud65c\uc57d \ub355\uc5d0 \ub450\uc0b0 \ubca0\uc5b4\uc2a4\ub294 3\ucc28\uc804\uc744 \uc2b9\ub9ac\ud588\ub2e4. \ud558\uc9c0\ub9cc, \uc2ec\uac01\ud55c \uc885\uc544\ub9ac \ubd80\uc0c1\uc73c\ub85c \uc778\ud574 \ub0a8\uc740 \uc2dc\ub9ac\uc988\uc5d0 \ucd9c\uc804\ud558\uc9c0 \ubabb\ud588\uace0, \ub450\uc0b0 \ubca0\uc5b4\uc2a4\ub294 \ub86f\ub370 \uc790\uc774\uc5b8\uce20\uc5d0\uac8c 4\ucc28\uc804\uc744 \ub0b4\uc8fc\uba74\uc11c \ud50c\ub808\uc774\uc624\ud504 \uc9c4\ucd9c\uc5d0 \uc2e4\ud328\ud588\ub2e4. \uc62c \uc2dc\uc98c \ubd80\uc9c4\uc73c\ub85c \uc778\ud574 800\ub9cc\uc6d0 \uc774 \uae4e\uc778 1\uc5b5 9700\ub9cc\uc6d0\uc5d0 2013 \uc2dc\uc98c \uc5f0\ubd09\uc744 \uacc4\uc57d\ud588\ub2e4.", "sentence_answer": "\uc62c \uc2dc\uc98c \ubd80\uc9c4\uc73c\ub85c \uc778\ud574 800\ub9cc\uc6d0 \uc774 \uae4e\uc778 1\uc5b5 9700\ub9cc\uc6d0\uc5d0 2013 \uc2dc\uc98c \uc5f0\ubd09\uc744 \uacc4\uc57d\ud588\ub2e4.", "paragraph_id": "6547804-7-2", "paragraph_question": "question: \uadf8\uc758 2013\ub144 \uc2dc\uc98c \uc5f0\ubd09\uc740 2012\ub144 \ub300\ube44\ud558\uc5ec \uc5bc\ub9c8\ub9cc\ud07c \uc904\uc5c8\ub294\uac00?, context: \uc2dc\uc98c \ucd08\ubc18\uc778 4\uc6d4\uc5d0 \ud0c0\uc728 0.306, 19\uc548\ud0c0 10\ub4dd\uc810\uc73c\ub85c \ud300\uc758 \uacf5\uaca9\uc744 \uc774\ub04c\uc5c8\uc73c\ub098, \uc774\ud6c4 \ud0c0\uc728\uc774 5\uc6d4\uc5d0\ub294 0.202, 6\uc6d4\uc5d0\ub294 \ubb34\ub98e \ubd80\uc0c1\ub4f1\uc774 \uacb9\uce58\uba70 0.143\uae4c\uc9c0 \ub0b4\ub824\uac14\ub2e4. \uadf9\uc2ec\ud55c \ubd80\uc9c4\uc73c\ub85c \ud558\uc704\ud0c0\uc21c\uc73c\ub85c \ubc00\ub824\ub0ac\uace0, \ub2f9\uc2dc \uac10\ub3c5\uc774\uc5c8\ub358 \uae40\uc9c4\uc6b1\uc744 \ube44\ub86f\ud574 \uc774\ud1a0 \uc4f0\ud1a0\ubb34 \uc218\uc11d \ucf54\uce58 \ub4f1\uc774 \ubd80\uc9c4 \ud0c8\ucd9c\uc744 \uc704\ud574 \ub9ce\uc740 \ub178\ub825\uc744 \uae30\uc6b8\uc600\uc73c\ub098 \ubd80\uc9c4\uc740 \uae38\uc5b4\uc84c\ub2e4. \uacb0\uad6d 6\uc6d4 \ub9d0\uc5d0 \uadf8\ub3d9\uc548 \uadf8\ub97c \uafb8\uc900\ud788 \uc9c0\ub3c4\ud574\uc654\ub358 \ub2f9\uc2dc 2\uad70 \uac10\ub3c5\uc774\uc5c8\ub358 \uc1a1\uc7ac\ubc15\uc774 1\uad70 \ud0c0\uaca9 \ucf54\uce58\ub85c \uc62c\ub77c\uc654\uace0, \ub178\uc2a4\ud15d \uc790\uc138\uc640 \uc5b4\ud37c \uc2a4\uc719 \ub300\uc2e0 \uc9e7\uac8c \ub04a\uc5b4\uce58\ub294 \ub545\ubcfc \uc704\uc8fc\uc758 \ud0c0\uaca9\uc5d0 \uc8fc\ub825\ud588\ub2e4. \ud55c\ud3b8 6\uc6d4 29\uc77c \ubd80\uc0c1\uc73c\ub85c \ud300 \uc804\ub825\uc5d0\uc11c \uc81c\uc678\ub41c \uc784\uc7ac\ucca0\uc744 \ub300\uc2e0\ud574 \uc8fc\uc7a5\uc744 \ub9e1\uc740 \uadf8\ub294, 7\uc6d4 0.247, 8\uc6d4 0.262\ub85c \ud0c0\uc728\uc774 \uc0c1\uc2b9\ud558\ub294 \ud398\uc774\uc2a4\ub97c \ubcf4\uc5ec\uc8fc\uc5c8\ub2e4. 7\uc6d4 27\uc77c\uc5d0\ub294 \uc5ed\uc804 \ub05d\ub0b4\uae30 \uc548\ud0c0\ub85c \ud504\ub85c \ub370\ubdd4 \ud6c4 \uccab \ub05d\ub0b4\uae30 \uc548\ud0c0\ub97c \uccd0\ub0c8\ub2e4. \uc62c \uc2dc\uc98c 121\uacbd\uae30\uc5d0\uc11c 2\ud560\ub300 \ud0c0\uc728(\ud0c0\uc728 \ucd5c\ud558\uc704), 21\ub3c4\ub8e8\ub97c \uae30\ub85d\ud558\uba70 \uc9c0\uae08\uae4c\uc9c0 \uce58\ub800\ub358 \uc2dc\uc98c \uc911 \uac00\uc7a5 \ubd80\uc9c4\ud55c \ubaa8\uc2b5\uc744 \ubcf4\uc600\ub358 \uadf8\ub294, \"\uc0ac\uc2e4 \ud06c\uac8c \uc544\ud508 \ub370\ub294 \uc5c6\uc5c8\ub2e4. \uc774\uc0c1\ud558\uac8c \ubc29\ub9dd\uc774\uac00 \ub9de\uc9c0 \uc54a\ub354\ub77c. \uc88b\uc740 \uacbd\ud5d8\uc774\uc5c8\ub2e4. \uc774\ub807\uac8c \ud558\uba74 \uc548 \ub41c\ub2e4\ub294 \uac78 \uae68\ub2eb\uac8c \ub410\ub2e4. \uc57c\uad6c\uac00 \ucc38 \uc5b4\ub835\ub2e4\"\uace0 \ub9d0\ud588\ub2e4. \ub450\uc0b0 \ubca0\uc5b4\uc2a4\ub294 \ud398\ub10c\ud2b8\ub808\uc774\uc2a4 3\uc704\ub97c \uae30\ub85d\ud558\uba70, \uc900\ud50c\ub808\uc774\uc624\ud504\uc5d0\uc11c 4\uc704 \ub86f\ub370 \uc790\uc774\uc5b8\uce20\uc640 \ub9de\ubd99\uc5b4, \uccab \ub450 \uacbd\uae30\uc5d0\uc11c 2\uc5f0\ud328\ub97c \ub2f9\ud588\ub2e4. \uadf8\ub7ec\ub098 3\ucc28\uc804\uc5d0\uc11c \uadf8\uac00 1\ud68c\ucd08 \uc120\ub450 \ud0c0\uc790\ub85c \ub098\uc11c \ubab8\uc5d0 \ub9de\ub294 \ubcfc\ub85c \ucd9c\ub8e8\ud55c \ub4a4, 2\ub8e8 \ub3c4\ub8e8\uc5d0 \uc131\uacf5\ud558\uace0, \ud6c4\uc18d\ud0c0 \ub54c \ub4dd\uc810\uc744 \ud574 \ubd80\uc0c1 \ud22c\ud63c\uc744 \ubcf4\uc5ec\uc8fc\uc5c8\uace0, \uc774\ub7f0 \ud65c\uc57d \ub355\uc5d0 \ub450\uc0b0 \ubca0\uc5b4\uc2a4\ub294 3\ucc28\uc804\uc744 \uc2b9\ub9ac\ud588\ub2e4. \ud558\uc9c0\ub9cc, \uc2ec\uac01\ud55c \uc885\uc544\ub9ac \ubd80\uc0c1\uc73c\ub85c \uc778\ud574 \ub0a8\uc740 \uc2dc\ub9ac\uc988\uc5d0 \ucd9c\uc804\ud558\uc9c0 \ubabb\ud588\uace0, \ub450\uc0b0 \ubca0\uc5b4\uc2a4\ub294 \ub86f\ub370 \uc790\uc774\uc5b8\uce20\uc5d0\uac8c 4\ucc28\uc804\uc744 \ub0b4\uc8fc\uba74\uc11c \ud50c\ub808\uc774\uc624\ud504 \uc9c4\ucd9c\uc5d0 \uc2e4\ud328\ud588\ub2e4. \uc62c \uc2dc\uc98c \ubd80\uc9c4\uc73c\ub85c \uc778\ud574 800\ub9cc\uc6d0\uc774 \uae4e\uc778 1\uc5b5 9700\ub9cc\uc6d0\uc5d0 2013 \uc2dc\uc98c \uc5f0\ubd09\uc744 \uacc4\uc57d\ud588\ub2e4."} {"question": "\uc591\uc720\ucc2c\uc758 \ud6c4\uc784\uc73c\ub85c \ub300\ub9ac\ub300\uc0ac\ub85c \uc9c0\uba85\ub41c \uc0ac\ub78c\uc740?", "paragraph": "4\u00b719 \ud601\uba85 \uc9c1\ud6c4\uc5d0 \uc591\uc720\ucc2c\uc740 \uc0ac\uc9c1\ud558\uc600\ub2e4. \uacfc\ub3c4 \uad6d\ubb34\uc6d0\uc740 4\uc6d4 29\uc77c \uc0c1\uc624 10\uc2dc\ubd80\ud130 \uc5f4\ub9b0 \uccab \uad6d\ubb34\ud68c\uc758\uc5d0\uc11c \uc0ac\ud45c\ub97c \ub0b8 \uc591\uc720\ucc2c \ub300\uc0ac\ub97c \uba74\uc9c1\ud558\uae30\ub85c \uc815\uc2dd\uc758\uacb0\ud558\uc600\ub2e4. 1960\ub144 4\uc6d4 29\uc77c, \uc591\uc720\ucc2c\uc740 \uc6cc\uc2f1\ud134\uc5d0\uc11c \uac1c\uc778\uc0ac\uc5c5\uc744 \ud558\ub824\uace0 \uc0dd\uac01\ud558\uace0 \uc788\ub2e4\uace0 \ub9d0\ud558\uc600\ub2e4. \uc591\uc720\ucc2c\uc740 \uc544\uc9c1 \ud55c\uad6d \uc784\uc2dc\uc815\ubd80\ub85c\ubd80\ud130 \ub204\uac00 \uadf8\uc758 \ud6c4\uc784\uc774 \ub420 \uac83\uc778\uac00\uc5d0 \uad00\ud558\uc5ec \uc544\ubb34\ub7f0 \uc2dc\uc0ac\ub3c4 \ubc1b\uc9c0 \uc54a\uc558\ub2e4. \uadf8\ub294 \uc544\uc9c1 \uba85\ud655\ud55c \uacc4\ud68d\uc740 \uc5c6\uc73c\ub098 \ud55c\uad6d \uc815\uacc4\uc5d0 \ud22c\uc2e0\ud558\uac70\ub098 \uc758\uc5c5\uc744 \uc7ac\uacc4\ud560 \uc0dd\uac01\uc740 \uc5c6\ub2e4\uace0 \ub9d0\ud558\uc600\ub2e4. 5\uc6d4 2\uc77c, \uc591\uc720\ucc2c\uc740 \uc0c8\ub85c\uc6b4 \ud55c\uad6d\uc815\ubd80\uac00 \uc774\ub0a0 \uadf8 \uc0ac\ud45c\ub97c \uc218\ub9ac\ud588\ub2e4\uace0 \ubc1d\ud614\ub2e4. \uc591\uc720\ucc2c\uc740 \"\ub098\ub294 \uadf8\ub4e4\uc774 \ub098\uc758 \uc0ac\ud45c\ub97c \uc218\ub9ac\ud574\uc11c \uae30\uc058\uba70 \uc9d0\uc774 \uac00\ubcbc\uc6cc\uc84c\ub2e4\"\uace0 \ub9d0\ud558\uc600\ub2e4. \ud55c\ud3b8 \ud55c\uad6d \ub300\uc0ac\uad00\uc758 \uc81c2\uc790(\u7b2c\u4e8c\u8005)\uc774\uba70 \ub178\ub828\ud55c \uc678\uad50\uad00\uc778 \ud55c\ud45c\uc6b1 \uacf5\uc0ac\uac00 \ub300\ub9ac\ub300\uc0ac\ub85c \uc9c0\uba85\ub418\uc5c8\ub2e4. \uc774\uc640 \uac19\uc740 \ubcc0\uacbd\uc5d0 \uad00\ud55c \uc815\uc2dd \ud1b5\uace0\ubb38\uc774 2\uc77c \ubbf8\uad6d \uad6d\ubb34\uc131\uacfc \uc678\uad50\ub2e8\uc5d0 \uc804\ub2ec\ub418\uc5c8\ub2e4. \uc591\uc720\ucc2c\uc758 \uc678\uad50\ud65c\ub3d9\uc740 \uc678\ubb34\ubcf8\ubd80\uc758 \uccb4\uacc4\ub97c \ubb34\uc2dc\ud55c \uacbd\ubb34\ub300 \uc9c1\ud1b5 \uc678\uad50\ub85c \ube44\ub09c\uc744 \ubc1b\uc558\ub2e4. \uc678\uad50 \uc635\uc368\ubc84\ub4e4\uc740 \uc591\uc720\ucc2c\uc774 \ud55c\uacb0\uac19\uc774 \uc774\uc2b9\ub9cc \uc815\uad8c\uc758 \ub3c5\uc120\uc801\uc778 \uc678\uad50\ub97c \ud569\ub9ac\ud654\uc2dc\ud0a4\ub294 \ub370 \uc804\ub825\uc744 \uacbd\uc8fc\ud558\uc600\uc73c\uba70 \uc9c0\ub09c 3\u2219 15 \uc815\u2219 \ubd80\ud1b5\ub839\uc120\uac70\uc5d0 \ub300\ud558\uc5ec\ub3c4 \uc790\uc720\uc138\uacc4\uc5d0 \ubaa8\ubc94\uc774 \ub41c \uacf5\uc815\ud55c \uc120\uac70\uc600\ub2e4\ub294 \uac83\uc744 \uc6b0\ubc29 \uc81c\uad6d\uc5d0 \uc65c\uace1 \uc120\uc804\ud558\ub294 \ud55c\ud3b8 \ub9c8\uc0b0\uc5d0\uc11c\uc758 \uad6d\ubbfc\uc758\uac70\ub294 \uacf5\uc0b0\uc624\uc5f4(\u4e94\u5217)\uc758 \uac1c\uc785 \uc6b4\uc6b4\uc73c\ub85c \uc774\uc2b9\ub9cc \uc815\uad8c\uc758 \ubd80\ud328\uc0c1\uc744 \ubb35\uc778, \uad6d\uc81c\uc5ec\ub860\uc744 \uc624\ub3c4\ucf00 \ud558\uc600\uc74c\uc744 \uc0c1\uae30\uc2dc\ucf30\ub2e4. \uc591\uc720\ucc2c\uc740 \uc870\uc6a9\ud55c \uc8fc\ud0dd\uc9c0\uc778 \uba54\uc774\ud50c\ubc00\uac00(\u8857)\uc5d0 \uc788\ub294 \uc790\ud0dd\uc5d0\uc11c \uc790\uae30\uc640 \ud55c(\u97d3) \uacf5\uc0ac\uac00 \uc774\uc2b9\ub9cc\uc758 \ub3c8\uc744 \uc2a4\uc704\uc2a4 \uc740\ud589\uc5d0 \uc608\uae08\ud558\uc600\ub2e4\ub294 \ubaa8 \uc2e0\ubb38\uae30\uc0ac\uc5d0 \uad00\ud558\uc5ec \ud384\ud384\ub6f0\uba74\uc11c \uc774\uac83\uc740 \uc0ac\uc2e4\ubb34\uadfc\uc774\ub77c\uace0 1960\ub144 7\uc6d4 13\uc77c \ubc18\ubc15\ud558\uc600\ub2e4.", "answer": "\ud55c\ud45c\uc6b1", "sentence": "\ud55c\ud3b8 \ud55c\uad6d \ub300\uc0ac\uad00\uc758 \uc81c2\uc790(\u7b2c\u4e8c\u8005)\uc774\uba70 \ub178\ub828\ud55c \uc678\uad50\uad00\uc778 \ud55c\ud45c\uc6b1 \uacf5\uc0ac\uac00 \ub300\ub9ac\ub300\uc0ac\ub85c \uc9c0\uba85\ub418\uc5c8\ub2e4.", "paragraph_sentence": "4\u00b719 \ud601\uba85 \uc9c1\ud6c4\uc5d0 \uc591\uc720\ucc2c\uc740 \uc0ac\uc9c1\ud558\uc600\ub2e4. \uacfc\ub3c4 \uad6d\ubb34\uc6d0\uc740 4\uc6d4 29\uc77c \uc0c1\uc624 10\uc2dc\ubd80\ud130 \uc5f4\ub9b0 \uccab \uad6d\ubb34\ud68c\uc758\uc5d0\uc11c \uc0ac\ud45c\ub97c \ub0b8 \uc591\uc720\ucc2c \ub300\uc0ac\ub97c \uba74\uc9c1\ud558\uae30\ub85c \uc815\uc2dd\uc758\uacb0\ud558\uc600\ub2e4. 1960\ub144 4\uc6d4 29\uc77c, \uc591\uc720\ucc2c\uc740 \uc6cc\uc2f1\ud134\uc5d0\uc11c \uac1c\uc778\uc0ac\uc5c5\uc744 \ud558\ub824\uace0 \uc0dd\uac01\ud558\uace0 \uc788\ub2e4\uace0 \ub9d0\ud558\uc600\ub2e4. \uc591\uc720\ucc2c\uc740 \uc544\uc9c1 \ud55c\uad6d \uc784\uc2dc\uc815\ubd80\ub85c\ubd80\ud130 \ub204\uac00 \uadf8\uc758 \ud6c4\uc784\uc774 \ub420 \uac83\uc778\uac00\uc5d0 \uad00\ud558\uc5ec \uc544\ubb34\ub7f0 \uc2dc\uc0ac\ub3c4 \ubc1b\uc9c0 \uc54a\uc558\ub2e4. \uadf8\ub294 \uc544\uc9c1 \uba85\ud655\ud55c \uacc4\ud68d\uc740 \uc5c6\uc73c\ub098 \ud55c\uad6d \uc815\uacc4\uc5d0 \ud22c\uc2e0\ud558\uac70\ub098 \uc758\uc5c5\uc744 \uc7ac\uacc4\ud560 \uc0dd\uac01\uc740 \uc5c6\ub2e4\uace0 \ub9d0\ud558\uc600\ub2e4. 5\uc6d4 2\uc77c, \uc591\uc720\ucc2c\uc740 \uc0c8\ub85c\uc6b4 \ud55c\uad6d\uc815\ubd80\uac00 \uc774\ub0a0 \uadf8 \uc0ac\ud45c\ub97c \uc218\ub9ac\ud588\ub2e4\uace0 \ubc1d\ud614\ub2e4. \uc591\uc720\ucc2c\uc740 \"\ub098\ub294 \uadf8\ub4e4\uc774 \ub098\uc758 \uc0ac\ud45c\ub97c \uc218\ub9ac\ud574\uc11c \uae30\uc058\uba70 \uc9d0\uc774 \uac00\ubcbc\uc6cc\uc84c\ub2e4\"\uace0 \ub9d0\ud558\uc600\ub2e4. \ud55c\ud3b8 \ud55c\uad6d \ub300\uc0ac\uad00\uc758 \uc81c2\uc790(\u7b2c\u4e8c\u8005)\uc774\uba70 \ub178\ub828\ud55c \uc678\uad50\uad00\uc778 \ud55c\ud45c\uc6b1 \uacf5\uc0ac\uac00 \ub300\ub9ac\ub300\uc0ac\ub85c \uc9c0\uba85\ub418\uc5c8\ub2e4. \uc774\uc640 \uac19\uc740 \ubcc0\uacbd\uc5d0 \uad00\ud55c \uc815\uc2dd \ud1b5\uace0\ubb38\uc774 2\uc77c \ubbf8\uad6d \uad6d\ubb34\uc131\uacfc \uc678\uad50\ub2e8\uc5d0 \uc804\ub2ec\ub418\uc5c8\ub2e4. \uc591\uc720\ucc2c\uc758 \uc678\uad50\ud65c\ub3d9\uc740 \uc678\ubb34\ubcf8\ubd80\uc758 \uccb4\uacc4\ub97c \ubb34\uc2dc\ud55c \uacbd\ubb34\ub300 \uc9c1\ud1b5 \uc678\uad50\ub85c \ube44\ub09c\uc744 \ubc1b\uc558\ub2e4. \uc678\uad50 \uc635\uc368\ubc84\ub4e4\uc740 \uc591\uc720\ucc2c\uc774 \ud55c\uacb0\uac19\uc774 \uc774\uc2b9\ub9cc \uc815\uad8c\uc758 \ub3c5\uc120\uc801\uc778 \uc678\uad50\ub97c \ud569\ub9ac\ud654\uc2dc\ud0a4\ub294 \ub370 \uc804\ub825\uc744 \uacbd\uc8fc\ud558\uc600\uc73c\uba70 \uc9c0\ub09c 3\u2219 15 \uc815\u2219 \ubd80\ud1b5\ub839\uc120\uac70\uc5d0 \ub300\ud558\uc5ec\ub3c4 \uc790\uc720\uc138\uacc4\uc5d0 \ubaa8\ubc94\uc774 \ub41c \uacf5\uc815\ud55c \uc120\uac70\uc600\ub2e4\ub294 \uac83\uc744 \uc6b0\ubc29 \uc81c\uad6d\uc5d0 \uc65c\uace1 \uc120\uc804\ud558\ub294 \ud55c\ud3b8 \ub9c8\uc0b0\uc5d0\uc11c\uc758 \uad6d\ubbfc\uc758\uac70\ub294 \uacf5\uc0b0\uc624\uc5f4(\u4e94\u5217)\uc758 \uac1c\uc785 \uc6b4\uc6b4\uc73c\ub85c \uc774\uc2b9\ub9cc \uc815\uad8c\uc758 \ubd80\ud328\uc0c1\uc744 \ubb35\uc778, \uad6d\uc81c\uc5ec\ub860\uc744 \uc624\ub3c4\ucf00 \ud558\uc600\uc74c\uc744 \uc0c1\uae30\uc2dc\ucf30\ub2e4. \uc591\uc720\ucc2c\uc740 \uc870\uc6a9\ud55c \uc8fc\ud0dd\uc9c0\uc778 \uba54\uc774\ud50c\ubc00\uac00(\u8857)\uc5d0 \uc788\ub294 \uc790\ud0dd\uc5d0\uc11c \uc790\uae30\uc640 \ud55c(\u97d3) \uacf5\uc0ac\uac00 \uc774\uc2b9\ub9cc\uc758 \ub3c8\uc744 \uc2a4\uc704\uc2a4 \uc740\ud589\uc5d0 \uc608\uae08\ud558\uc600\ub2e4\ub294 \ubaa8 \uc2e0\ubb38\uae30\uc0ac\uc5d0 \uad00\ud558\uc5ec \ud384\ud384\ub6f0\uba74\uc11c \uc774\uac83\uc740 \uc0ac\uc2e4\ubb34\uadfc\uc774\ub77c\uace0 1960\ub144 7\uc6d4 13\uc77c \ubc18\ubc15\ud558\uc600\ub2e4.", "paragraph_answer": "4\u00b719 \ud601\uba85 \uc9c1\ud6c4\uc5d0 \uc591\uc720\ucc2c\uc740 \uc0ac\uc9c1\ud558\uc600\ub2e4. \uacfc\ub3c4 \uad6d\ubb34\uc6d0\uc740 4\uc6d4 29\uc77c \uc0c1\uc624 10\uc2dc\ubd80\ud130 \uc5f4\ub9b0 \uccab \uad6d\ubb34\ud68c\uc758\uc5d0\uc11c \uc0ac\ud45c\ub97c \ub0b8 \uc591\uc720\ucc2c \ub300\uc0ac\ub97c \uba74\uc9c1\ud558\uae30\ub85c \uc815\uc2dd\uc758\uacb0\ud558\uc600\ub2e4. 1960\ub144 4\uc6d4 29\uc77c, \uc591\uc720\ucc2c\uc740 \uc6cc\uc2f1\ud134\uc5d0\uc11c \uac1c\uc778\uc0ac\uc5c5\uc744 \ud558\ub824\uace0 \uc0dd\uac01\ud558\uace0 \uc788\ub2e4\uace0 \ub9d0\ud558\uc600\ub2e4. \uc591\uc720\ucc2c\uc740 \uc544\uc9c1 \ud55c\uad6d \uc784\uc2dc\uc815\ubd80\ub85c\ubd80\ud130 \ub204\uac00 \uadf8\uc758 \ud6c4\uc784\uc774 \ub420 \uac83\uc778\uac00\uc5d0 \uad00\ud558\uc5ec \uc544\ubb34\ub7f0 \uc2dc\uc0ac\ub3c4 \ubc1b\uc9c0 \uc54a\uc558\ub2e4. \uadf8\ub294 \uc544\uc9c1 \uba85\ud655\ud55c \uacc4\ud68d\uc740 \uc5c6\uc73c\ub098 \ud55c\uad6d \uc815\uacc4\uc5d0 \ud22c\uc2e0\ud558\uac70\ub098 \uc758\uc5c5\uc744 \uc7ac\uacc4\ud560 \uc0dd\uac01\uc740 \uc5c6\ub2e4\uace0 \ub9d0\ud558\uc600\ub2e4. 5\uc6d4 2\uc77c, \uc591\uc720\ucc2c\uc740 \uc0c8\ub85c\uc6b4 \ud55c\uad6d\uc815\ubd80\uac00 \uc774\ub0a0 \uadf8 \uc0ac\ud45c\ub97c \uc218\ub9ac\ud588\ub2e4\uace0 \ubc1d\ud614\ub2e4. \uc591\uc720\ucc2c\uc740 \"\ub098\ub294 \uadf8\ub4e4\uc774 \ub098\uc758 \uc0ac\ud45c\ub97c \uc218\ub9ac\ud574\uc11c \uae30\uc058\uba70 \uc9d0\uc774 \uac00\ubcbc\uc6cc\uc84c\ub2e4\"\uace0 \ub9d0\ud558\uc600\ub2e4. \ud55c\ud3b8 \ud55c\uad6d \ub300\uc0ac\uad00\uc758 \uc81c2\uc790(\u7b2c\u4e8c\u8005)\uc774\uba70 \ub178\ub828\ud55c \uc678\uad50\uad00\uc778 \ud55c\ud45c\uc6b1 \uacf5\uc0ac\uac00 \ub300\ub9ac\ub300\uc0ac\ub85c \uc9c0\uba85\ub418\uc5c8\ub2e4. \uc774\uc640 \uac19\uc740 \ubcc0\uacbd\uc5d0 \uad00\ud55c \uc815\uc2dd \ud1b5\uace0\ubb38\uc774 2\uc77c \ubbf8\uad6d \uad6d\ubb34\uc131\uacfc \uc678\uad50\ub2e8\uc5d0 \uc804\ub2ec\ub418\uc5c8\ub2e4. \uc591\uc720\ucc2c\uc758 \uc678\uad50\ud65c\ub3d9\uc740 \uc678\ubb34\ubcf8\ubd80\uc758 \uccb4\uacc4\ub97c \ubb34\uc2dc\ud55c \uacbd\ubb34\ub300 \uc9c1\ud1b5 \uc678\uad50\ub85c \ube44\ub09c\uc744 \ubc1b\uc558\ub2e4. \uc678\uad50 \uc635\uc368\ubc84\ub4e4\uc740 \uc591\uc720\ucc2c\uc774 \ud55c\uacb0\uac19\uc774 \uc774\uc2b9\ub9cc \uc815\uad8c\uc758 \ub3c5\uc120\uc801\uc778 \uc678\uad50\ub97c \ud569\ub9ac\ud654\uc2dc\ud0a4\ub294 \ub370 \uc804\ub825\uc744 \uacbd\uc8fc\ud558\uc600\uc73c\uba70 \uc9c0\ub09c 3\u2219 15 \uc815\u2219 \ubd80\ud1b5\ub839\uc120\uac70\uc5d0 \ub300\ud558\uc5ec\ub3c4 \uc790\uc720\uc138\uacc4\uc5d0 \ubaa8\ubc94\uc774 \ub41c \uacf5\uc815\ud55c \uc120\uac70\uc600\ub2e4\ub294 \uac83\uc744 \uc6b0\ubc29 \uc81c\uad6d\uc5d0 \uc65c\uace1 \uc120\uc804\ud558\ub294 \ud55c\ud3b8 \ub9c8\uc0b0\uc5d0\uc11c\uc758 \uad6d\ubbfc\uc758\uac70\ub294 \uacf5\uc0b0\uc624\uc5f4(\u4e94\u5217)\uc758 \uac1c\uc785 \uc6b4\uc6b4\uc73c\ub85c \uc774\uc2b9\ub9cc \uc815\uad8c\uc758 \ubd80\ud328\uc0c1\uc744 \ubb35\uc778, \uad6d\uc81c\uc5ec\ub860\uc744 \uc624\ub3c4\ucf00 \ud558\uc600\uc74c\uc744 \uc0c1\uae30\uc2dc\ucf30\ub2e4. \uc591\uc720\ucc2c\uc740 \uc870\uc6a9\ud55c \uc8fc\ud0dd\uc9c0\uc778 \uba54\uc774\ud50c\ubc00\uac00(\u8857)\uc5d0 \uc788\ub294 \uc790\ud0dd\uc5d0\uc11c \uc790\uae30\uc640 \ud55c(\u97d3) \uacf5\uc0ac\uac00 \uc774\uc2b9\ub9cc\uc758 \ub3c8\uc744 \uc2a4\uc704\uc2a4 \uc740\ud589\uc5d0 \uc608\uae08\ud558\uc600\ub2e4\ub294 \ubaa8 \uc2e0\ubb38\uae30\uc0ac\uc5d0 \uad00\ud558\uc5ec \ud384\ud384\ub6f0\uba74\uc11c \uc774\uac83\uc740 \uc0ac\uc2e4\ubb34\uadfc\uc774\ub77c\uace0 1960\ub144 7\uc6d4 13\uc77c \ubc18\ubc15\ud558\uc600\ub2e4.", "sentence_answer": "\ud55c\ud3b8 \ud55c\uad6d \ub300\uc0ac\uad00\uc758 \uc81c2\uc790(\u7b2c\u4e8c\u8005)\uc774\uba70 \ub178\ub828\ud55c \uc678\uad50\uad00\uc778 \ud55c\ud45c\uc6b1 \uacf5\uc0ac\uac00 \ub300\ub9ac\ub300\uc0ac\ub85c \uc9c0\uba85\ub418\uc5c8\ub2e4.", "paragraph_id": "6540543-3-2", "paragraph_question": "question: \uc591\uc720\ucc2c\uc758 \ud6c4\uc784\uc73c\ub85c \ub300\ub9ac\ub300\uc0ac\ub85c \uc9c0\uba85\ub41c \uc0ac\ub78c\uc740?, context: 4\u00b719 \ud601\uba85 \uc9c1\ud6c4\uc5d0 \uc591\uc720\ucc2c\uc740 \uc0ac\uc9c1\ud558\uc600\ub2e4. \uacfc\ub3c4 \uad6d\ubb34\uc6d0\uc740 4\uc6d4 29\uc77c \uc0c1\uc624 10\uc2dc\ubd80\ud130 \uc5f4\ub9b0 \uccab \uad6d\ubb34\ud68c\uc758\uc5d0\uc11c \uc0ac\ud45c\ub97c \ub0b8 \uc591\uc720\ucc2c \ub300\uc0ac\ub97c \uba74\uc9c1\ud558\uae30\ub85c \uc815\uc2dd\uc758\uacb0\ud558\uc600\ub2e4. 1960\ub144 4\uc6d4 29\uc77c, \uc591\uc720\ucc2c\uc740 \uc6cc\uc2f1\ud134\uc5d0\uc11c \uac1c\uc778\uc0ac\uc5c5\uc744 \ud558\ub824\uace0 \uc0dd\uac01\ud558\uace0 \uc788\ub2e4\uace0 \ub9d0\ud558\uc600\ub2e4. \uc591\uc720\ucc2c\uc740 \uc544\uc9c1 \ud55c\uad6d \uc784\uc2dc\uc815\ubd80\ub85c\ubd80\ud130 \ub204\uac00 \uadf8\uc758 \ud6c4\uc784\uc774 \ub420 \uac83\uc778\uac00\uc5d0 \uad00\ud558\uc5ec \uc544\ubb34\ub7f0 \uc2dc\uc0ac\ub3c4 \ubc1b\uc9c0 \uc54a\uc558\ub2e4. \uadf8\ub294 \uc544\uc9c1 \uba85\ud655\ud55c \uacc4\ud68d\uc740 \uc5c6\uc73c\ub098 \ud55c\uad6d \uc815\uacc4\uc5d0 \ud22c\uc2e0\ud558\uac70\ub098 \uc758\uc5c5\uc744 \uc7ac\uacc4\ud560 \uc0dd\uac01\uc740 \uc5c6\ub2e4\uace0 \ub9d0\ud558\uc600\ub2e4. 5\uc6d4 2\uc77c, \uc591\uc720\ucc2c\uc740 \uc0c8\ub85c\uc6b4 \ud55c\uad6d\uc815\ubd80\uac00 \uc774\ub0a0 \uadf8 \uc0ac\ud45c\ub97c \uc218\ub9ac\ud588\ub2e4\uace0 \ubc1d\ud614\ub2e4. \uc591\uc720\ucc2c\uc740 \"\ub098\ub294 \uadf8\ub4e4\uc774 \ub098\uc758 \uc0ac\ud45c\ub97c \uc218\ub9ac\ud574\uc11c \uae30\uc058\uba70 \uc9d0\uc774 \uac00\ubcbc\uc6cc\uc84c\ub2e4\"\uace0 \ub9d0\ud558\uc600\ub2e4. \ud55c\ud3b8 \ud55c\uad6d \ub300\uc0ac\uad00\uc758 \uc81c2\uc790(\u7b2c\u4e8c\u8005)\uc774\uba70 \ub178\ub828\ud55c \uc678\uad50\uad00\uc778 \ud55c\ud45c\uc6b1 \uacf5\uc0ac\uac00 \ub300\ub9ac\ub300\uc0ac\ub85c \uc9c0\uba85\ub418\uc5c8\ub2e4. \uc774\uc640 \uac19\uc740 \ubcc0\uacbd\uc5d0 \uad00\ud55c \uc815\uc2dd \ud1b5\uace0\ubb38\uc774 2\uc77c \ubbf8\uad6d \uad6d\ubb34\uc131\uacfc \uc678\uad50\ub2e8\uc5d0 \uc804\ub2ec\ub418\uc5c8\ub2e4. \uc591\uc720\ucc2c\uc758 \uc678\uad50\ud65c\ub3d9\uc740 \uc678\ubb34\ubcf8\ubd80\uc758 \uccb4\uacc4\ub97c \ubb34\uc2dc\ud55c \uacbd\ubb34\ub300 \uc9c1\ud1b5 \uc678\uad50\ub85c \ube44\ub09c\uc744 \ubc1b\uc558\ub2e4. \uc678\uad50 \uc635\uc368\ubc84\ub4e4\uc740 \uc591\uc720\ucc2c\uc774 \ud55c\uacb0\uac19\uc774 \uc774\uc2b9\ub9cc \uc815\uad8c\uc758 \ub3c5\uc120\uc801\uc778 \uc678\uad50\ub97c \ud569\ub9ac\ud654\uc2dc\ud0a4\ub294 \ub370 \uc804\ub825\uc744 \uacbd\uc8fc\ud558\uc600\uc73c\uba70 \uc9c0\ub09c 3\u2219 15 \uc815\u2219 \ubd80\ud1b5\ub839\uc120\uac70\uc5d0 \ub300\ud558\uc5ec\ub3c4 \uc790\uc720\uc138\uacc4\uc5d0 \ubaa8\ubc94\uc774 \ub41c \uacf5\uc815\ud55c \uc120\uac70\uc600\ub2e4\ub294 \uac83\uc744 \uc6b0\ubc29 \uc81c\uad6d\uc5d0 \uc65c\uace1 \uc120\uc804\ud558\ub294 \ud55c\ud3b8 \ub9c8\uc0b0\uc5d0\uc11c\uc758 \uad6d\ubbfc\uc758\uac70\ub294 \uacf5\uc0b0\uc624\uc5f4(\u4e94\u5217)\uc758 \uac1c\uc785 \uc6b4\uc6b4\uc73c\ub85c \uc774\uc2b9\ub9cc \uc815\uad8c\uc758 \ubd80\ud328\uc0c1\uc744 \ubb35\uc778, \uad6d\uc81c\uc5ec\ub860\uc744 \uc624\ub3c4\ucf00 \ud558\uc600\uc74c\uc744 \uc0c1\uae30\uc2dc\ucf30\ub2e4. \uc591\uc720\ucc2c\uc740 \uc870\uc6a9\ud55c \uc8fc\ud0dd\uc9c0\uc778 \uba54\uc774\ud50c\ubc00\uac00(\u8857)\uc5d0 \uc788\ub294 \uc790\ud0dd\uc5d0\uc11c \uc790\uae30\uc640 \ud55c(\u97d3) \uacf5\uc0ac\uac00 \uc774\uc2b9\ub9cc\uc758 \ub3c8\uc744 \uc2a4\uc704\uc2a4 \uc740\ud589\uc5d0 \uc608\uae08\ud558\uc600\ub2e4\ub294 \ubaa8 \uc2e0\ubb38\uae30\uc0ac\uc5d0 \uad00\ud558\uc5ec \ud384\ud384\ub6f0\uba74\uc11c \uc774\uac83\uc740 \uc0ac\uc2e4\ubb34\uadfc\uc774\ub77c\uace0 1960\ub144 7\uc6d4 13\uc77c \ubc18\ubc15\ud558\uc600\ub2e4."} {"question": "\uc2e0\ucc9c\uc9c0\uc758 \uc2e0\ub3c4\ub4e4\uc774 \uad50\uc8fc\uc778 \uc774\ub9cc\ud76c\ub294 \ubb34\uc5c7\uc744 \ud560 \uc218 \uc788\ub2e4\uace0 \ubbff\uace0 \uc788\ub098?", "paragraph": "\uc2e0\ucc9c\uc9c0\ub97c \ube44\ud310\ud558\ub294 \ubc29\uc1a1\uc774 2007\ub144\uacfc 2008\ub144\uc5d0 \uac78\uccd0 \uc5ec\ub7ec \ucc28\ub840 \ubc29\uc1a1\uc5d0 \ubcf4\ub3c4\ub41c \ubc14 \uc788\ub2e4. MBC\uc5d0\uc11c\ub294 2007\ub144 5\uc6d4 8\uc77c\uacfc 2007\ub144 12\uc6d4 25\uc77c \ub450 \ucc28\ub840\uc5d0 \uac78\uccd0 PD\uc218\ucca9\uc744 \ud1b5\ud574 \uc2e0\ucc9c\uc9c0\uc5d0 \ub300\ud55c \ubc29\uc1a1\uc744 \ubc29\uc601\ud558\uc600\ub2e4. \ubc29\uc1a1\uc5d0\uc11c\ub294 \uc2e0\ucc9c\uc9c0\uac00 \"\uc2e0\ub3c4\uc758 \uac00\uc871\uc5d0\uac8c \uc885\uad50\ub97c \uac15\uc694\ud558\uc600\ub2e4\", \"\uac00\uc815\uc744 \ud30c\uad34\ud558\ub3c4\ub85d \uc870\uc7a5\ud558\uc600\ub2e4\", \"\uc774\ub9cc\ud76c \ubcf8\uc778\uc740 \ubc29\uc1a1\uc5d0\uc11c \uc601\uc0dd\uc744 \ubd80\uc778\ud588\uc73c\ub098 \uc2e4\uc81c\ub85c\ub294 \uc2e0\ub3c4\ub4e4\uc774 \uc774\ub9cc\ud76c\uc528\uac00 \uc601\uc0dd\uc744 \ud55c\ub2e4\uace0 \ubbff\uace0 \uc788\uc73c\uba70, \uc774\ub9cc\ud76c\uc528\ub3c4 \uc2e0\ub3c4\uc5d0\uac8c \uc601\uc0dd\uad8c\uc744 \uc368 \uc900 \uc0ac\uc2e4\uc774 \uc788\ub2e4\"\ub294 \ub4f1\uc758 \ub0b4\uc6a9\uc73c\ub85c \uc2e0\ucc9c\uc9c0\ub97c \ube44\ud310\ud558\uc600\ub2e4. \uadf8\ub9ac\uace0 \uc774 \ub54c\ubd80\ud130 \uc2e0\ucc9c\uc9c0\uc608\uc218\uad50 \uc99d\uac70\uc7a5\ub9c9\uc131\uc804\uc774 \ubcf8\uaca9\uc801\uc73c\ub85c \ub300\ud55c\ubbfc\uad6d\uc5d0\uc11c \ub17c\ub780\uc774 \ub418\uae30 \uc2dc\uc791\ud588\ub2e4. \uc774 \ubc29\uc1a1\uacfc \uad00\ub828\ud558\uc5ec \uc2e0\ucc9c\uc9c0 \ubcf8\ubd80\uce21\uc758 MBC \ubb38\ud654\ubc29\uc1a1\uc5d0 \ub300\ud55c \ubc95\uc815 \uc18c\uc1a1\uacb0\uacfc, 2009\ub144 10\uc6d4 12\uc77c \uc11c\uc6b8\uace0\ub4f1\ubc95\uc6d0\uc758 \ubc95\uc6d0\uc870\uc815 \ud310\uacb0\uc5d0 \ub530\ub77c MBC \ubb38\ud654\ubc29\uc1a1\uc774 2009\ub144 10\uc6d4 20\uc77c PD\uc218\ucca9 \ubc29\uc1a1 \ud504\ub85c\uadf8\ub7a8 \uccab\uba38\ub9ac\uc5d0 '\uc815\uc815\ubcf4\ub3c4 1\uac74 \ubc0f \ubc18\ub860\ubcf4\ub3c4\ubb38'\uc744 \ubc29\uc1a1\ud558\uc600\ub2e4.", "answer": "\uc601\uc0dd", "sentence": "\ubc29\uc1a1\uc5d0\uc11c\ub294 \uc2e0\ucc9c\uc9c0\uac00 \"\uc2e0\ub3c4\uc758 \uac00\uc871\uc5d0\uac8c \uc885\uad50\ub97c \uac15\uc694\ud558\uc600\ub2e4\", \"\uac00\uc815\uc744 \ud30c\uad34\ud558\ub3c4\ub85d \uc870\uc7a5\ud558\uc600\ub2e4\", \"\uc774\ub9cc\ud76c \ubcf8\uc778\uc740 \ubc29\uc1a1\uc5d0\uc11c \uc601\uc0dd \uc744 \ubd80\uc778\ud588\uc73c\ub098 \uc2e4\uc81c\ub85c\ub294 \uc2e0\ub3c4\ub4e4\uc774 \uc774\ub9cc\ud76c\uc528\uac00 \uc601\uc0dd\uc744 \ud55c\ub2e4\uace0 \ubbff\uace0 \uc788\uc73c\uba70, \uc774\ub9cc\ud76c\uc528\ub3c4 \uc2e0\ub3c4\uc5d0\uac8c \uc601\uc0dd\uad8c\uc744 \uc368 \uc900 \uc0ac\uc2e4\uc774 \uc788\ub2e4\"\ub294 \ub4f1\uc758 \ub0b4\uc6a9\uc73c\ub85c \uc2e0\ucc9c\uc9c0\ub97c \ube44\ud310\ud558\uc600\ub2e4.", "paragraph_sentence": "\uc2e0\ucc9c\uc9c0\ub97c \ube44\ud310\ud558\ub294 \ubc29\uc1a1\uc774 2007\ub144\uacfc 2008\ub144\uc5d0 \uac78\uccd0 \uc5ec\ub7ec \ucc28\ub840 \ubc29\uc1a1\uc5d0 \ubcf4\ub3c4\ub41c \ubc14 \uc788\ub2e4. MBC\uc5d0\uc11c\ub294 2007\ub144 5\uc6d4 8\uc77c\uacfc 2007\ub144 12\uc6d4 25\uc77c \ub450 \ucc28\ub840\uc5d0 \uac78\uccd0 PD\uc218\ucca9\uc744 \ud1b5\ud574 \uc2e0\ucc9c\uc9c0\uc5d0 \ub300\ud55c \ubc29\uc1a1\uc744 \ubc29\uc601\ud558\uc600\ub2e4. \ubc29\uc1a1\uc5d0\uc11c\ub294 \uc2e0\ucc9c\uc9c0\uac00 \"\uc2e0\ub3c4\uc758 \uac00\uc871\uc5d0\uac8c \uc885\uad50\ub97c \uac15\uc694\ud558\uc600\ub2e4\", \"\uac00\uc815\uc744 \ud30c\uad34\ud558\ub3c4\ub85d \uc870\uc7a5\ud558\uc600\ub2e4\", \"\uc774\ub9cc\ud76c \ubcf8\uc778\uc740 \ubc29\uc1a1\uc5d0\uc11c \uc601\uc0dd \uc744 \ubd80\uc778\ud588\uc73c\ub098 \uc2e4\uc81c\ub85c\ub294 \uc2e0\ub3c4\ub4e4\uc774 \uc774\ub9cc\ud76c\uc528\uac00 \uc601\uc0dd\uc744 \ud55c\ub2e4\uace0 \ubbff\uace0 \uc788\uc73c\uba70, \uc774\ub9cc\ud76c\uc528\ub3c4 \uc2e0\ub3c4\uc5d0\uac8c \uc601\uc0dd\uad8c\uc744 \uc368 \uc900 \uc0ac\uc2e4\uc774 \uc788\ub2e4\"\ub294 \ub4f1\uc758 \ub0b4\uc6a9\uc73c\ub85c \uc2e0\ucc9c\uc9c0\ub97c \ube44\ud310\ud558\uc600\ub2e4. \uadf8\ub9ac\uace0 \uc774 \ub54c\ubd80\ud130 \uc2e0\ucc9c\uc9c0\uc608\uc218\uad50 \uc99d\uac70\uc7a5\ub9c9\uc131\uc804\uc774 \ubcf8\uaca9\uc801\uc73c\ub85c \ub300\ud55c\ubbfc\uad6d\uc5d0\uc11c \ub17c\ub780\uc774 \ub418\uae30 \uc2dc\uc791\ud588\ub2e4. \uc774 \ubc29\uc1a1\uacfc \uad00\ub828\ud558\uc5ec \uc2e0\ucc9c\uc9c0 \ubcf8\ubd80\uce21\uc758 MBC \ubb38\ud654\ubc29\uc1a1\uc5d0 \ub300\ud55c \ubc95\uc815 \uc18c\uc1a1\uacb0\uacfc, 2009\ub144 10\uc6d4 12\uc77c \uc11c\uc6b8\uace0\ub4f1\ubc95\uc6d0\uc758 \ubc95\uc6d0\uc870\uc815 \ud310\uacb0\uc5d0 \ub530\ub77c MBC \ubb38\ud654\ubc29\uc1a1\uc774 2009\ub144 10\uc6d4 20\uc77c PD\uc218\ucca9 \ubc29\uc1a1 \ud504\ub85c\uadf8\ub7a8 \uccab\uba38\ub9ac\uc5d0 '\uc815\uc815\ubcf4\ub3c4 1\uac74 \ubc0f \ubc18\ub860\ubcf4\ub3c4\ubb38'\uc744 \ubc29\uc1a1\ud558\uc600\ub2e4.", "paragraph_answer": "\uc2e0\ucc9c\uc9c0\ub97c \ube44\ud310\ud558\ub294 \ubc29\uc1a1\uc774 2007\ub144\uacfc 2008\ub144\uc5d0 \uac78\uccd0 \uc5ec\ub7ec \ucc28\ub840 \ubc29\uc1a1\uc5d0 \ubcf4\ub3c4\ub41c \ubc14 \uc788\ub2e4. MBC\uc5d0\uc11c\ub294 2007\ub144 5\uc6d4 8\uc77c\uacfc 2007\ub144 12\uc6d4 25\uc77c \ub450 \ucc28\ub840\uc5d0 \uac78\uccd0 PD\uc218\ucca9\uc744 \ud1b5\ud574 \uc2e0\ucc9c\uc9c0\uc5d0 \ub300\ud55c \ubc29\uc1a1\uc744 \ubc29\uc601\ud558\uc600\ub2e4. \ubc29\uc1a1\uc5d0\uc11c\ub294 \uc2e0\ucc9c\uc9c0\uac00 \"\uc2e0\ub3c4\uc758 \uac00\uc871\uc5d0\uac8c \uc885\uad50\ub97c \uac15\uc694\ud558\uc600\ub2e4\", \"\uac00\uc815\uc744 \ud30c\uad34\ud558\ub3c4\ub85d \uc870\uc7a5\ud558\uc600\ub2e4\", \"\uc774\ub9cc\ud76c \ubcf8\uc778\uc740 \ubc29\uc1a1\uc5d0\uc11c \uc601\uc0dd \uc744 \ubd80\uc778\ud588\uc73c\ub098 \uc2e4\uc81c\ub85c\ub294 \uc2e0\ub3c4\ub4e4\uc774 \uc774\ub9cc\ud76c\uc528\uac00 \uc601\uc0dd\uc744 \ud55c\ub2e4\uace0 \ubbff\uace0 \uc788\uc73c\uba70, \uc774\ub9cc\ud76c\uc528\ub3c4 \uc2e0\ub3c4\uc5d0\uac8c \uc601\uc0dd\uad8c\uc744 \uc368 \uc900 \uc0ac\uc2e4\uc774 \uc788\ub2e4\"\ub294 \ub4f1\uc758 \ub0b4\uc6a9\uc73c\ub85c \uc2e0\ucc9c\uc9c0\ub97c \ube44\ud310\ud558\uc600\ub2e4. \uadf8\ub9ac\uace0 \uc774 \ub54c\ubd80\ud130 \uc2e0\ucc9c\uc9c0\uc608\uc218\uad50 \uc99d\uac70\uc7a5\ub9c9\uc131\uc804\uc774 \ubcf8\uaca9\uc801\uc73c\ub85c \ub300\ud55c\ubbfc\uad6d\uc5d0\uc11c \ub17c\ub780\uc774 \ub418\uae30 \uc2dc\uc791\ud588\ub2e4. \uc774 \ubc29\uc1a1\uacfc \uad00\ub828\ud558\uc5ec \uc2e0\ucc9c\uc9c0 \ubcf8\ubd80\uce21\uc758 MBC \ubb38\ud654\ubc29\uc1a1\uc5d0 \ub300\ud55c \ubc95\uc815 \uc18c\uc1a1\uacb0\uacfc, 2009\ub144 10\uc6d4 12\uc77c \uc11c\uc6b8\uace0\ub4f1\ubc95\uc6d0\uc758 \ubc95\uc6d0\uc870\uc815 \ud310\uacb0\uc5d0 \ub530\ub77c MBC \ubb38\ud654\ubc29\uc1a1\uc774 2009\ub144 10\uc6d4 20\uc77c PD\uc218\ucca9 \ubc29\uc1a1 \ud504\ub85c\uadf8\ub7a8 \uccab\uba38\ub9ac\uc5d0 '\uc815\uc815\ubcf4\ub3c4 1\uac74 \ubc0f \ubc18\ub860\ubcf4\ub3c4\ubb38'\uc744 \ubc29\uc1a1\ud558\uc600\ub2e4.", "sentence_answer": "\ubc29\uc1a1\uc5d0\uc11c\ub294 \uc2e0\ucc9c\uc9c0\uac00 \"\uc2e0\ub3c4\uc758 \uac00\uc871\uc5d0\uac8c \uc885\uad50\ub97c \uac15\uc694\ud558\uc600\ub2e4\", \"\uac00\uc815\uc744 \ud30c\uad34\ud558\ub3c4\ub85d \uc870\uc7a5\ud558\uc600\ub2e4\", \"\uc774\ub9cc\ud76c \ubcf8\uc778\uc740 \ubc29\uc1a1\uc5d0\uc11c \uc601\uc0dd \uc744 \ubd80\uc778\ud588\uc73c\ub098 \uc2e4\uc81c\ub85c\ub294 \uc2e0\ub3c4\ub4e4\uc774 \uc774\ub9cc\ud76c\uc528\uac00 \uc601\uc0dd\uc744 \ud55c\ub2e4\uace0 \ubbff\uace0 \uc788\uc73c\uba70, \uc774\ub9cc\ud76c\uc528\ub3c4 \uc2e0\ub3c4\uc5d0\uac8c \uc601\uc0dd\uad8c\uc744 \uc368 \uc900 \uc0ac\uc2e4\uc774 \uc788\ub2e4\"\ub294 \ub4f1\uc758 \ub0b4\uc6a9\uc73c\ub85c \uc2e0\ucc9c\uc9c0\ub97c \ube44\ud310\ud558\uc600\ub2e4.", "paragraph_id": "6538210-2-1", "paragraph_question": "question: \uc2e0\ucc9c\uc9c0\uc758 \uc2e0\ub3c4\ub4e4\uc774 \uad50\uc8fc\uc778 \uc774\ub9cc\ud76c\ub294 \ubb34\uc5c7\uc744 \ud560 \uc218 \uc788\ub2e4\uace0 \ubbff\uace0 \uc788\ub098?, context: \uc2e0\ucc9c\uc9c0\ub97c \ube44\ud310\ud558\ub294 \ubc29\uc1a1\uc774 2007\ub144\uacfc 2008\ub144\uc5d0 \uac78\uccd0 \uc5ec\ub7ec \ucc28\ub840 \ubc29\uc1a1\uc5d0 \ubcf4\ub3c4\ub41c \ubc14 \uc788\ub2e4. MBC\uc5d0\uc11c\ub294 2007\ub144 5\uc6d4 8\uc77c\uacfc 2007\ub144 12\uc6d4 25\uc77c \ub450 \ucc28\ub840\uc5d0 \uac78\uccd0 PD\uc218\ucca9\uc744 \ud1b5\ud574 \uc2e0\ucc9c\uc9c0\uc5d0 \ub300\ud55c \ubc29\uc1a1\uc744 \ubc29\uc601\ud558\uc600\ub2e4. \ubc29\uc1a1\uc5d0\uc11c\ub294 \uc2e0\ucc9c\uc9c0\uac00 \"\uc2e0\ub3c4\uc758 \uac00\uc871\uc5d0\uac8c \uc885\uad50\ub97c \uac15\uc694\ud558\uc600\ub2e4\", \"\uac00\uc815\uc744 \ud30c\uad34\ud558\ub3c4\ub85d \uc870\uc7a5\ud558\uc600\ub2e4\", \"\uc774\ub9cc\ud76c \ubcf8\uc778\uc740 \ubc29\uc1a1\uc5d0\uc11c \uc601\uc0dd\uc744 \ubd80\uc778\ud588\uc73c\ub098 \uc2e4\uc81c\ub85c\ub294 \uc2e0\ub3c4\ub4e4\uc774 \uc774\ub9cc\ud76c\uc528\uac00 \uc601\uc0dd\uc744 \ud55c\ub2e4\uace0 \ubbff\uace0 \uc788\uc73c\uba70, \uc774\ub9cc\ud76c\uc528\ub3c4 \uc2e0\ub3c4\uc5d0\uac8c \uc601\uc0dd\uad8c\uc744 \uc368 \uc900 \uc0ac\uc2e4\uc774 \uc788\ub2e4\"\ub294 \ub4f1\uc758 \ub0b4\uc6a9\uc73c\ub85c \uc2e0\ucc9c\uc9c0\ub97c \ube44\ud310\ud558\uc600\ub2e4. \uadf8\ub9ac\uace0 \uc774 \ub54c\ubd80\ud130 \uc2e0\ucc9c\uc9c0\uc608\uc218\uad50 \uc99d\uac70\uc7a5\ub9c9\uc131\uc804\uc774 \ubcf8\uaca9\uc801\uc73c\ub85c \ub300\ud55c\ubbfc\uad6d\uc5d0\uc11c \ub17c\ub780\uc774 \ub418\uae30 \uc2dc\uc791\ud588\ub2e4. \uc774 \ubc29\uc1a1\uacfc \uad00\ub828\ud558\uc5ec \uc2e0\ucc9c\uc9c0 \ubcf8\ubd80\uce21\uc758 MBC \ubb38\ud654\ubc29\uc1a1\uc5d0 \ub300\ud55c \ubc95\uc815 \uc18c\uc1a1\uacb0\uacfc, 2009\ub144 10\uc6d4 12\uc77c \uc11c\uc6b8\uace0\ub4f1\ubc95\uc6d0\uc758 \ubc95\uc6d0\uc870\uc815 \ud310\uacb0\uc5d0 \ub530\ub77c MBC \ubb38\ud654\ubc29\uc1a1\uc774 2009\ub144 10\uc6d4 20\uc77c PD\uc218\ucca9 \ubc29\uc1a1 \ud504\ub85c\uadf8\ub7a8 \uccab\uba38\ub9ac\uc5d0 '\uc815\uc815\ubcf4\ub3c4 1\uac74 \ubc0f \ubc18\ub860\ubcf4\ub3c4\ubb38'\uc744 \ubc29\uc1a1\ud558\uc600\ub2e4."} {"question": "\uc778\ub3c4\uc758 \ubb38\ud559\uc740 \uc5b4\ub290 \uc5b8\uc5b4 \uacc4\ud1b5\uc73c\ub85c \uc11c\uc220\ud558\ub294\uac00?", "paragraph": "\uc778\ub3c4(\u5370\u5ea6)\ub294 \uc6d0\ub798 \uc778\uad6c\uac00 \ub9ce\uace0 \uad6d\ud1a0\uac00 \ub113\uc740\ub370 \uc774\uc5d0 \ub530\ub978 \ubbfc\uc871\uc758 \uc774\ub3d9 \u00b7 \uc885\uad50\uc758 \ubcf5\uc7a1\uc131 \u00b7 \ub2e4\uc591\ud55c \uc5b8\uc5b4 \ub4f1\uc758 \uc694\uc778\uc73c\ub85c \uc778\ud574 \uc778\ub3c4\uc758 \ubb38\ud559(\u5370\u5ea6\uc758 \u6587\u5b78)\ub3c4 \ub2e8\uc21c\ud558\uc9c0\uac00 \uc54a\ub2e4. \uc885\uad50\uc758 \uc885\ub958\ub3c4 \ube0c\ub77c\ub9cc\uad50 \u00b7 \ubd88\uad50 \u00b7 \uc790\uc774\ub098\uad50 \u00b7 \ud78c\ub450\uad50 \ub4f1\uc758 \uc778\ub3c4 \uace0\uc720\uc758 \uac83\uc774 \uc788\uace0 \uc678\ubd80\ub85c\ubd80\ud130 \uc720\uc785\ub41c \uc774\uc2ac\ub78c\uad50 \u00b7 \uc870\ub85c\uc544\uc2a4\ud130\uad50 \ub4f1\uc774 \uc788\uc73c\uba70, \uc774\ub4e4 \uc885\uad50\ub4e4 \uc0c1\ud638\uac04\uc5d0\ub294 \ucda9\ub3cc\uacfc \uad50\ub958\uac00 \uc788\uc5c8\ub2e4. \uc778\ub3c4 \ubb38\ud559(\u5370\u5ea6\u6587\u5b78)\uc758 \uc5b8\uc5b4\uba74\uc5d0\uc11c\ub294, \uc778\ub3c4\uc758 \uc5b8\uc5b4\ub294 \uc778\ub3c4 \uc544\ub9ac\uc544\uc5b4 \uacc4\ud1b5(\"\uc778\ub3c4\uc720\ub7fd\uc5b4\uc871-\uc778\ub3c4\uc774\ub780\uc5b4\ud30c-\uc778\ub3c4\uc544\ub9ac\uc544\uc5b4\uad70\")\uacfc \ub4dc\ub77c\ube44\ub2e4\uc5b4 \uacc4\ud1b5(\"\ub4dc\ub77c\ube44\ub2e4\uc5b4\uc871\")\uc73c\ub85c \ub098\ub25c\ub2e4. \uc778\ub3c4 \uc544\ub9ac\uc544\uc5b4 \uacc4\ud1b5\uc5d0\ub294 \ubca0\ub2e4 \uc0b0\uc2a4\ud06c\ub9ac\ud2b8\uc5b4\uc640 \uace0\uc804 \uc0b0\uc2a4\ud06c\ub9ac\ud2b8\uc5b4\uac00 \uc18d\ud55c\ub2e4. \uc5ec\uae30\uc11c\ub294 \uc8fc\ub85c \uc804\uc790\uc758 \uc778\ub3c4\uc544\ub9ac\uc544\uc5b4 \uacc4\ud1b5 \uc5b8\uc5b4\ub4e4\uc758 \ubb38\ud559\uc5d0 \ub300\ud574 \uc11c\uc220\ud55c\ub2e4.", "answer": "\uc778\ub3c4\uc544\ub9ac\uc544\uc5b4 \uacc4\ud1b5", "sentence": "\uc5ec\uae30\uc11c\ub294 \uc8fc\ub85c \uc804\uc790\uc758 \uc778\ub3c4\uc544\ub9ac\uc544\uc5b4 \uacc4\ud1b5 \uc5b8\uc5b4\ub4e4\uc758 \ubb38\ud559\uc5d0 \ub300\ud574 \uc11c\uc220\ud55c\ub2e4.", "paragraph_sentence": "\uc778\ub3c4(\u5370\u5ea6)\ub294 \uc6d0\ub798 \uc778\uad6c\uac00 \ub9ce\uace0 \uad6d\ud1a0\uac00 \ub113\uc740\ub370 \uc774\uc5d0 \ub530\ub978 \ubbfc\uc871\uc758 \uc774\ub3d9 \u00b7 \uc885\uad50\uc758 \ubcf5\uc7a1\uc131 \u00b7 \ub2e4\uc591\ud55c \uc5b8\uc5b4 \ub4f1\uc758 \uc694\uc778\uc73c\ub85c \uc778\ud574 \uc778\ub3c4\uc758 \ubb38\ud559(\u5370\u5ea6\uc758 \u6587\u5b78)\ub3c4 \ub2e8\uc21c\ud558\uc9c0\uac00 \uc54a\ub2e4. \uc885\uad50\uc758 \uc885\ub958\ub3c4 \ube0c\ub77c\ub9cc\uad50 \u00b7 \ubd88\uad50 \u00b7 \uc790\uc774\ub098\uad50 \u00b7 \ud78c\ub450\uad50 \ub4f1\uc758 \uc778\ub3c4 \uace0\uc720\uc758 \uac83\uc774 \uc788\uace0 \uc678\ubd80\ub85c\ubd80\ud130 \uc720\uc785\ub41c \uc774\uc2ac\ub78c\uad50 \u00b7 \uc870\ub85c\uc544\uc2a4\ud130\uad50 \ub4f1\uc774 \uc788\uc73c\uba70, \uc774\ub4e4 \uc885\uad50\ub4e4 \uc0c1\ud638\uac04\uc5d0\ub294 \ucda9\ub3cc\uacfc \uad50\ub958\uac00 \uc788\uc5c8\ub2e4. \uc778\ub3c4 \ubb38\ud559(\u5370\u5ea6\u6587\u5b78)\uc758 \uc5b8\uc5b4\uba74\uc5d0\uc11c\ub294, \uc778\ub3c4\uc758 \uc5b8\uc5b4\ub294 \uc778\ub3c4 \uc544\ub9ac\uc544\uc5b4 \uacc4\ud1b5(\"\uc778\ub3c4\uc720\ub7fd\uc5b4\uc871-\uc778\ub3c4\uc774\ub780\uc5b4\ud30c-\uc778\ub3c4\uc544\ub9ac\uc544\uc5b4\uad70\")\uacfc \ub4dc\ub77c\ube44\ub2e4\uc5b4 \uacc4\ud1b5(\"\ub4dc\ub77c\ube44\ub2e4\uc5b4\uc871\")\uc73c\ub85c \ub098\ub25c\ub2e4. \uc778\ub3c4 \uc544\ub9ac\uc544\uc5b4 \uacc4\ud1b5\uc5d0\ub294 \ubca0\ub2e4 \uc0b0\uc2a4\ud06c\ub9ac\ud2b8\uc5b4\uc640 \uace0\uc804 \uc0b0\uc2a4\ud06c\ub9ac\ud2b8\uc5b4\uac00 \uc18d\ud55c\ub2e4. \uc5ec\uae30\uc11c\ub294 \uc8fc\ub85c \uc804\uc790\uc758 \uc778\ub3c4\uc544\ub9ac\uc544\uc5b4 \uacc4\ud1b5 \uc5b8\uc5b4\ub4e4\uc758 \ubb38\ud559\uc5d0 \ub300\ud574 \uc11c\uc220\ud55c\ub2e4. ", "paragraph_answer": "\uc778\ub3c4(\u5370\u5ea6)\ub294 \uc6d0\ub798 \uc778\uad6c\uac00 \ub9ce\uace0 \uad6d\ud1a0\uac00 \ub113\uc740\ub370 \uc774\uc5d0 \ub530\ub978 \ubbfc\uc871\uc758 \uc774\ub3d9 \u00b7 \uc885\uad50\uc758 \ubcf5\uc7a1\uc131 \u00b7 \ub2e4\uc591\ud55c \uc5b8\uc5b4 \ub4f1\uc758 \uc694\uc778\uc73c\ub85c \uc778\ud574 \uc778\ub3c4\uc758 \ubb38\ud559(\u5370\u5ea6\uc758 \u6587\u5b78)\ub3c4 \ub2e8\uc21c\ud558\uc9c0\uac00 \uc54a\ub2e4. \uc885\uad50\uc758 \uc885\ub958\ub3c4 \ube0c\ub77c\ub9cc\uad50 \u00b7 \ubd88\uad50 \u00b7 \uc790\uc774\ub098\uad50 \u00b7 \ud78c\ub450\uad50 \ub4f1\uc758 \uc778\ub3c4 \uace0\uc720\uc758 \uac83\uc774 \uc788\uace0 \uc678\ubd80\ub85c\ubd80\ud130 \uc720\uc785\ub41c \uc774\uc2ac\ub78c\uad50 \u00b7 \uc870\ub85c\uc544\uc2a4\ud130\uad50 \ub4f1\uc774 \uc788\uc73c\uba70, \uc774\ub4e4 \uc885\uad50\ub4e4 \uc0c1\ud638\uac04\uc5d0\ub294 \ucda9\ub3cc\uacfc \uad50\ub958\uac00 \uc788\uc5c8\ub2e4. \uc778\ub3c4 \ubb38\ud559(\u5370\u5ea6\u6587\u5b78)\uc758 \uc5b8\uc5b4\uba74\uc5d0\uc11c\ub294, \uc778\ub3c4\uc758 \uc5b8\uc5b4\ub294 \uc778\ub3c4 \uc544\ub9ac\uc544\uc5b4 \uacc4\ud1b5(\"\uc778\ub3c4\uc720\ub7fd\uc5b4\uc871-\uc778\ub3c4\uc774\ub780\uc5b4\ud30c-\uc778\ub3c4\uc544\ub9ac\uc544\uc5b4\uad70\")\uacfc \ub4dc\ub77c\ube44\ub2e4\uc5b4 \uacc4\ud1b5(\"\ub4dc\ub77c\ube44\ub2e4\uc5b4\uc871\")\uc73c\ub85c \ub098\ub25c\ub2e4. \uc778\ub3c4 \uc544\ub9ac\uc544\uc5b4 \uacc4\ud1b5\uc5d0\ub294 \ubca0\ub2e4 \uc0b0\uc2a4\ud06c\ub9ac\ud2b8\uc5b4\uc640 \uace0\uc804 \uc0b0\uc2a4\ud06c\ub9ac\ud2b8\uc5b4\uac00 \uc18d\ud55c\ub2e4. \uc5ec\uae30\uc11c\ub294 \uc8fc\ub85c \uc804\uc790\uc758 \uc778\ub3c4\uc544\ub9ac\uc544\uc5b4 \uacc4\ud1b5 \uc5b8\uc5b4\ub4e4\uc758 \ubb38\ud559\uc5d0 \ub300\ud574 \uc11c\uc220\ud55c\ub2e4.", "sentence_answer": "\uc5ec\uae30\uc11c\ub294 \uc8fc\ub85c \uc804\uc790\uc758 \uc778\ub3c4\uc544\ub9ac\uc544\uc5b4 \uacc4\ud1b5 \uc5b8\uc5b4\ub4e4\uc758 \ubb38\ud559\uc5d0 \ub300\ud574 \uc11c\uc220\ud55c\ub2e4.", "paragraph_id": "6548115-0-0", "paragraph_question": "question: \uc778\ub3c4\uc758 \ubb38\ud559\uc740 \uc5b4\ub290 \uc5b8\uc5b4 \uacc4\ud1b5\uc73c\ub85c \uc11c\uc220\ud558\ub294\uac00?, context: \uc778\ub3c4(\u5370\u5ea6)\ub294 \uc6d0\ub798 \uc778\uad6c\uac00 \ub9ce\uace0 \uad6d\ud1a0\uac00 \ub113\uc740\ub370 \uc774\uc5d0 \ub530\ub978 \ubbfc\uc871\uc758 \uc774\ub3d9 \u00b7 \uc885\uad50\uc758 \ubcf5\uc7a1\uc131 \u00b7 \ub2e4\uc591\ud55c \uc5b8\uc5b4 \ub4f1\uc758 \uc694\uc778\uc73c\ub85c \uc778\ud574 \uc778\ub3c4\uc758 \ubb38\ud559(\u5370\u5ea6\uc758 \u6587\u5b78)\ub3c4 \ub2e8\uc21c\ud558\uc9c0\uac00 \uc54a\ub2e4. \uc885\uad50\uc758 \uc885\ub958\ub3c4 \ube0c\ub77c\ub9cc\uad50 \u00b7 \ubd88\uad50 \u00b7 \uc790\uc774\ub098\uad50 \u00b7 \ud78c\ub450\uad50 \ub4f1\uc758 \uc778\ub3c4 \uace0\uc720\uc758 \uac83\uc774 \uc788\uace0 \uc678\ubd80\ub85c\ubd80\ud130 \uc720\uc785\ub41c \uc774\uc2ac\ub78c\uad50 \u00b7 \uc870\ub85c\uc544\uc2a4\ud130\uad50 \ub4f1\uc774 \uc788\uc73c\uba70, \uc774\ub4e4 \uc885\uad50\ub4e4 \uc0c1\ud638\uac04\uc5d0\ub294 \ucda9\ub3cc\uacfc \uad50\ub958\uac00 \uc788\uc5c8\ub2e4. \uc778\ub3c4 \ubb38\ud559(\u5370\u5ea6\u6587\u5b78)\uc758 \uc5b8\uc5b4\uba74\uc5d0\uc11c\ub294, \uc778\ub3c4\uc758 \uc5b8\uc5b4\ub294 \uc778\ub3c4 \uc544\ub9ac\uc544\uc5b4 \uacc4\ud1b5(\"\uc778\ub3c4\uc720\ub7fd\uc5b4\uc871-\uc778\ub3c4\uc774\ub780\uc5b4\ud30c-\uc778\ub3c4\uc544\ub9ac\uc544\uc5b4\uad70\")\uacfc \ub4dc\ub77c\ube44\ub2e4\uc5b4 \uacc4\ud1b5(\"\ub4dc\ub77c\ube44\ub2e4\uc5b4\uc871\")\uc73c\ub85c \ub098\ub25c\ub2e4. \uc778\ub3c4 \uc544\ub9ac\uc544\uc5b4 \uacc4\ud1b5\uc5d0\ub294 \ubca0\ub2e4 \uc0b0\uc2a4\ud06c\ub9ac\ud2b8\uc5b4\uc640 \uace0\uc804 \uc0b0\uc2a4\ud06c\ub9ac\ud2b8\uc5b4\uac00 \uc18d\ud55c\ub2e4. \uc5ec\uae30\uc11c\ub294 \uc8fc\ub85c \uc804\uc790\uc758 \uc778\ub3c4\uc544\ub9ac\uc544\uc5b4 \uacc4\ud1b5 \uc5b8\uc5b4\ub4e4\uc758 \ubb38\ud559\uc5d0 \ub300\ud574 \uc11c\uc220\ud55c\ub2e4."} {"question": "\ud29c\ub294 \uacbd\ub9e4\uac00 \ub05d\ub098\uae30 \uc0ac\ud758 \uc804 \uc5b4\ub514 \ub2e8\uccb4\ub85c\ubd80\ud130 \uc804\uc790 \uba54\uc77c\uc744 \ubc1b\uc558\ub294\uac00?", "paragraph": "1,000\uac1c \ud654\uc18c \uacbd\ub9e4\uac00 \ub05d\ub098\uae30 \uc0ac\ud758 \uc55e\ub454 2006\ub144 1\uc6d4 7\uc77c\uc5d0 \ud29c\ub294 \ub354 \ub2e4\ud06c \uadf8\ub8f9(The Dark Group)\uc774\ub77c\ub294 \ub2e8\uccb4\ub85c\ubd80\ud130 \uc804\uc790 \uba54\uc77c\uc744 \ubc1b\uc558\ub2e4. \uc5ec\uae30\uc11c 5,000\ub2ec\ub7ec \ubc30\uc0c1\uae08\uc744 1\uc6d4 10\uc77c\uae4c\uc9c0 \uccad\uad6c\ud558\uc9c0 \uc54a\uc73c\uba74 \ubc00\ub9ac\uc5b8 \ub2ec\ub7ec \ud648\ud398\uc774\uc9c0\uac00 \ubd84\uc0b0 \uc11c\ube44\uc2a4 \uac70\ubd80 \uacf5\uaca9 (DDoS)\uc744 \ubc1b\uc744 \uac83\uc774\ub77c\ub294 \uc774\uc57c\uae30\uac00 \uc801\ud600 \uc788\uc5c8\ub2e4. \uc704\ud611\uc774 \uac00\uc9dc\uc77c \uac83\uc774\ub77c\uace0 \uadf8\ub294 \ubbff\uc5b4 \uc774\ub97c \ubcf4\uace0 \ub118\uc5b4\uac14\uc9c0\ub9cc \ud55c \uc8fc\uac00 \uc9c0\ub098 \ub450 \ubc88\uc9f8 \uc804\uc790 \uba54\uc77c \ud611\ubc15\uc744 \ubc1b\uc558\ub2e4: \"\uc548\ub155 \ub124 \uc6f9\uc0ac\uc774\ud2b8\uac00 DDoS \uacf5\uaca9\uc5d0 \uc2dc\ub2ec\ub9ac\uc9c0 \uc54a\uac8c \ud558\ub824\uba74 50000 \ub2ec\ub7ec\ub97c \uc6b0\ub9ac\uc5d0\uac8c \ubcf4\ub0b4\ub77c\"(\"Hello u website is under us atack to stop the DDoS send us 50000$.\"). \ub610, \uadf8\ub294 \uc774 \ud611\ubc15\uc744 \ubb34\uc2dc\ud588\ub294\ub370 \uc6f9\uc0ac\uc774\ud2b8\ub294 \uc5c4\uccad\ub09c \ud2b8\ub798\ud53d\uacfc \uc804\uc790 \uba54\uc77c\ub85c \uace4\ud639\uc744 \uce58\ub974\ub2e4\uac00 \ucef4\ud4e8\ud130 \ucda9\ub3cc\ub85c\uae4c\uc9c0 \uc774\uc5b4\uc84c\ub2e4. \ud29c\ub294 \"\uc804 \uc774 \uc0ac\ub78c\ub4e4 \uc5b4\ub290 \ub204\uad6c\uc5d0\uac8c\ub3c4 \ub300\ub2f5\ud558\uc9c0 \uc54a\uc558\uc2b5\ub2c8\ub2e4. \uc774\ub4e4\uc744 \ub9cc\uc871\uc2dc\ud0a4\uace0 \uc2f6\uc9c0\ub3c4 \uc54a\uc558\uace0 \uadf8 \uc0ac\ub78c\ub4e4\uc5d0\uac8c \uc5b4\ub5a0\ud55c \ub3c8\ub3c4 \uc904 \uc0dd\uac01\uc774 \uc5c6\uc5c8\uc2b5\ub2c8\ub2e4. \uc81c \uc6f9\uc0ac\uc774\ud2b8\uc5d0 \uc77c\uc5b4\ub09c \uc77c\uc740 \ub9c8\uce58 \ud14c\ub7ec \uac19\uc558\uc2b5\ub2c8\ub2e4. \uc5ec\ub7ec\ubd84\uc774 \uadf8\ub4e4\uc5d0\uac8c \ub3c8\uc744 \uc900\ub2e4\uba74 \ub610 \uacf5\uaca9\uc774 \uc2dc\uc791\ub420 \uac83\uc785\ub2c8\ub2e4.\"\ub77c\uace0 \ub9d0\ud558\uc600\ub2e4.", "answer": "\ub354 \ub2e4\ud06c \uadf8\ub8f9", "sentence": "1,000\uac1c \ud654\uc18c \uacbd\ub9e4\uac00 \ub05d\ub098\uae30 \uc0ac\ud758 \uc55e\ub454 2006\ub144 1\uc6d4 7\uc77c\uc5d0 \ud29c\ub294 \ub354 \ub2e4\ud06c \uadf8\ub8f9 (The Dark Group)", "paragraph_sentence": " 1,000\uac1c \ud654\uc18c \uacbd\ub9e4\uac00 \ub05d\ub098\uae30 \uc0ac\ud758 \uc55e\ub454 2006\ub144 1\uc6d4 7\uc77c\uc5d0 \ud29c\ub294 \ub354 \ub2e4\ud06c \uadf8\ub8f9 (The Dark Group) \uc774\ub77c\ub294 \ub2e8\uccb4\ub85c\ubd80\ud130 \uc804\uc790 \uba54\uc77c\uc744 \ubc1b\uc558\ub2e4. \uc5ec\uae30\uc11c 5,000\ub2ec\ub7ec \ubc30\uc0c1\uae08\uc744 1\uc6d4 10\uc77c\uae4c\uc9c0 \uccad\uad6c\ud558\uc9c0 \uc54a\uc73c\uba74 \ubc00\ub9ac\uc5b8 \ub2ec\ub7ec \ud648\ud398\uc774\uc9c0\uac00 \ubd84\uc0b0 \uc11c\ube44\uc2a4 \uac70\ubd80 \uacf5\uaca9 (DDoS)\uc744 \ubc1b\uc744 \uac83\uc774\ub77c\ub294 \uc774\uc57c\uae30\uac00 \uc801\ud600 \uc788\uc5c8\ub2e4. \uc704\ud611\uc774 \uac00\uc9dc\uc77c \uac83\uc774\ub77c\uace0 \uadf8\ub294 \ubbff\uc5b4 \uc774\ub97c \ubcf4\uace0 \ub118\uc5b4\uac14\uc9c0\ub9cc \ud55c \uc8fc\uac00 \uc9c0\ub098 \ub450 \ubc88\uc9f8 \uc804\uc790 \uba54\uc77c \ud611\ubc15\uc744 \ubc1b\uc558\ub2e4: \"\uc548\ub155 \ub124 \uc6f9\uc0ac\uc774\ud2b8\uac00 DDoS \uacf5\uaca9\uc5d0 \uc2dc\ub2ec\ub9ac\uc9c0 \uc54a\uac8c \ud558\ub824\uba74 50000 \ub2ec\ub7ec\ub97c \uc6b0\ub9ac\uc5d0\uac8c \ubcf4\ub0b4\ub77c\"(\"Hello u website is under us atack to stop the DDoS send us 50000$.\"). \ub610, \uadf8\ub294 \uc774 \ud611\ubc15\uc744 \ubb34\uc2dc\ud588\ub294\ub370 \uc6f9\uc0ac\uc774\ud2b8\ub294 \uc5c4\uccad\ub09c \ud2b8\ub798\ud53d\uacfc \uc804\uc790 \uba54\uc77c\ub85c \uace4\ud639\uc744 \uce58\ub974\ub2e4\uac00 \ucef4\ud4e8\ud130 \ucda9\ub3cc\ub85c\uae4c\uc9c0 \uc774\uc5b4\uc84c\ub2e4. \ud29c\ub294 \"\uc804 \uc774 \uc0ac\ub78c\ub4e4 \uc5b4\ub290 \ub204\uad6c\uc5d0\uac8c\ub3c4 \ub300\ub2f5\ud558\uc9c0 \uc54a\uc558\uc2b5\ub2c8\ub2e4. \uc774\ub4e4\uc744 \ub9cc\uc871\uc2dc\ud0a4\uace0 \uc2f6\uc9c0\ub3c4 \uc54a\uc558\uace0 \uadf8 \uc0ac\ub78c\ub4e4\uc5d0\uac8c \uc5b4\ub5a0\ud55c \ub3c8\ub3c4 \uc904 \uc0dd\uac01\uc774 \uc5c6\uc5c8\uc2b5\ub2c8\ub2e4. \uc81c \uc6f9\uc0ac\uc774\ud2b8\uc5d0 \uc77c\uc5b4\ub09c \uc77c\uc740 \ub9c8\uce58 \ud14c\ub7ec \uac19\uc558\uc2b5\ub2c8\ub2e4. \uc5ec\ub7ec\ubd84\uc774 \uadf8\ub4e4\uc5d0\uac8c \ub3c8\uc744 \uc900\ub2e4\uba74 \ub610 \uacf5\uaca9\uc774 \uc2dc\uc791\ub420 \uac83\uc785\ub2c8\ub2e4. \"\ub77c\uace0 \ub9d0\ud558\uc600\ub2e4.", "paragraph_answer": "1,000\uac1c \ud654\uc18c \uacbd\ub9e4\uac00 \ub05d\ub098\uae30 \uc0ac\ud758 \uc55e\ub454 2006\ub144 1\uc6d4 7\uc77c\uc5d0 \ud29c\ub294 \ub354 \ub2e4\ud06c \uadf8\ub8f9 (The Dark Group)\uc774\ub77c\ub294 \ub2e8\uccb4\ub85c\ubd80\ud130 \uc804\uc790 \uba54\uc77c\uc744 \ubc1b\uc558\ub2e4. \uc5ec\uae30\uc11c 5,000\ub2ec\ub7ec \ubc30\uc0c1\uae08\uc744 1\uc6d4 10\uc77c\uae4c\uc9c0 \uccad\uad6c\ud558\uc9c0 \uc54a\uc73c\uba74 \ubc00\ub9ac\uc5b8 \ub2ec\ub7ec \ud648\ud398\uc774\uc9c0\uac00 \ubd84\uc0b0 \uc11c\ube44\uc2a4 \uac70\ubd80 \uacf5\uaca9 (DDoS)\uc744 \ubc1b\uc744 \uac83\uc774\ub77c\ub294 \uc774\uc57c\uae30\uac00 \uc801\ud600 \uc788\uc5c8\ub2e4. \uc704\ud611\uc774 \uac00\uc9dc\uc77c \uac83\uc774\ub77c\uace0 \uadf8\ub294 \ubbff\uc5b4 \uc774\ub97c \ubcf4\uace0 \ub118\uc5b4\uac14\uc9c0\ub9cc \ud55c \uc8fc\uac00 \uc9c0\ub098 \ub450 \ubc88\uc9f8 \uc804\uc790 \uba54\uc77c \ud611\ubc15\uc744 \ubc1b\uc558\ub2e4: \"\uc548\ub155 \ub124 \uc6f9\uc0ac\uc774\ud2b8\uac00 DDoS \uacf5\uaca9\uc5d0 \uc2dc\ub2ec\ub9ac\uc9c0 \uc54a\uac8c \ud558\ub824\uba74 50000 \ub2ec\ub7ec\ub97c \uc6b0\ub9ac\uc5d0\uac8c \ubcf4\ub0b4\ub77c\"(\"Hello u website is under us atack to stop the DDoS send us 50000$.\"). \ub610, \uadf8\ub294 \uc774 \ud611\ubc15\uc744 \ubb34\uc2dc\ud588\ub294\ub370 \uc6f9\uc0ac\uc774\ud2b8\ub294 \uc5c4\uccad\ub09c \ud2b8\ub798\ud53d\uacfc \uc804\uc790 \uba54\uc77c\ub85c \uace4\ud639\uc744 \uce58\ub974\ub2e4\uac00 \ucef4\ud4e8\ud130 \ucda9\ub3cc\ub85c\uae4c\uc9c0 \uc774\uc5b4\uc84c\ub2e4. \ud29c\ub294 \"\uc804 \uc774 \uc0ac\ub78c\ub4e4 \uc5b4\ub290 \ub204\uad6c\uc5d0\uac8c\ub3c4 \ub300\ub2f5\ud558\uc9c0 \uc54a\uc558\uc2b5\ub2c8\ub2e4. \uc774\ub4e4\uc744 \ub9cc\uc871\uc2dc\ud0a4\uace0 \uc2f6\uc9c0\ub3c4 \uc54a\uc558\uace0 \uadf8 \uc0ac\ub78c\ub4e4\uc5d0\uac8c \uc5b4\ub5a0\ud55c \ub3c8\ub3c4 \uc904 \uc0dd\uac01\uc774 \uc5c6\uc5c8\uc2b5\ub2c8\ub2e4. \uc81c \uc6f9\uc0ac\uc774\ud2b8\uc5d0 \uc77c\uc5b4\ub09c \uc77c\uc740 \ub9c8\uce58 \ud14c\ub7ec \uac19\uc558\uc2b5\ub2c8\ub2e4. \uc5ec\ub7ec\ubd84\uc774 \uadf8\ub4e4\uc5d0\uac8c \ub3c8\uc744 \uc900\ub2e4\uba74 \ub610 \uacf5\uaca9\uc774 \uc2dc\uc791\ub420 \uac83\uc785\ub2c8\ub2e4.\"\ub77c\uace0 \ub9d0\ud558\uc600\ub2e4.", "sentence_answer": "1,000\uac1c \ud654\uc18c \uacbd\ub9e4\uac00 \ub05d\ub098\uae30 \uc0ac\ud758 \uc55e\ub454 2006\ub144 1\uc6d4 7\uc77c\uc5d0 \ud29c\ub294 \ub354 \ub2e4\ud06c \uadf8\ub8f9 (The Dark Group)", "paragraph_id": "6603055-6-0", "paragraph_question": "question: \ud29c\ub294 \uacbd\ub9e4\uac00 \ub05d\ub098\uae30 \uc0ac\ud758 \uc804 \uc5b4\ub514 \ub2e8\uccb4\ub85c\ubd80\ud130 \uc804\uc790 \uba54\uc77c\uc744 \ubc1b\uc558\ub294\uac00?, context: 1,000\uac1c \ud654\uc18c \uacbd\ub9e4\uac00 \ub05d\ub098\uae30 \uc0ac\ud758 \uc55e\ub454 2006\ub144 1\uc6d4 7\uc77c\uc5d0 \ud29c\ub294 \ub354 \ub2e4\ud06c \uadf8\ub8f9(The Dark Group)\uc774\ub77c\ub294 \ub2e8\uccb4\ub85c\ubd80\ud130 \uc804\uc790 \uba54\uc77c\uc744 \ubc1b\uc558\ub2e4. \uc5ec\uae30\uc11c 5,000\ub2ec\ub7ec \ubc30\uc0c1\uae08\uc744 1\uc6d4 10\uc77c\uae4c\uc9c0 \uccad\uad6c\ud558\uc9c0 \uc54a\uc73c\uba74 \ubc00\ub9ac\uc5b8 \ub2ec\ub7ec \ud648\ud398\uc774\uc9c0\uac00 \ubd84\uc0b0 \uc11c\ube44\uc2a4 \uac70\ubd80 \uacf5\uaca9 (DDoS)\uc744 \ubc1b\uc744 \uac83\uc774\ub77c\ub294 \uc774\uc57c\uae30\uac00 \uc801\ud600 \uc788\uc5c8\ub2e4. \uc704\ud611\uc774 \uac00\uc9dc\uc77c \uac83\uc774\ub77c\uace0 \uadf8\ub294 \ubbff\uc5b4 \uc774\ub97c \ubcf4\uace0 \ub118\uc5b4\uac14\uc9c0\ub9cc \ud55c \uc8fc\uac00 \uc9c0\ub098 \ub450 \ubc88\uc9f8 \uc804\uc790 \uba54\uc77c \ud611\ubc15\uc744 \ubc1b\uc558\ub2e4: \"\uc548\ub155 \ub124 \uc6f9\uc0ac\uc774\ud2b8\uac00 DDoS \uacf5\uaca9\uc5d0 \uc2dc\ub2ec\ub9ac\uc9c0 \uc54a\uac8c \ud558\ub824\uba74 50000 \ub2ec\ub7ec\ub97c \uc6b0\ub9ac\uc5d0\uac8c \ubcf4\ub0b4\ub77c\"(\"Hello u website is under us atack to stop the DDoS send us 50000$.\"). \ub610, \uadf8\ub294 \uc774 \ud611\ubc15\uc744 \ubb34\uc2dc\ud588\ub294\ub370 \uc6f9\uc0ac\uc774\ud2b8\ub294 \uc5c4\uccad\ub09c \ud2b8\ub798\ud53d\uacfc \uc804\uc790 \uba54\uc77c\ub85c \uace4\ud639\uc744 \uce58\ub974\ub2e4\uac00 \ucef4\ud4e8\ud130 \ucda9\ub3cc\ub85c\uae4c\uc9c0 \uc774\uc5b4\uc84c\ub2e4. \ud29c\ub294 \"\uc804 \uc774 \uc0ac\ub78c\ub4e4 \uc5b4\ub290 \ub204\uad6c\uc5d0\uac8c\ub3c4 \ub300\ub2f5\ud558\uc9c0 \uc54a\uc558\uc2b5\ub2c8\ub2e4. \uc774\ub4e4\uc744 \ub9cc\uc871\uc2dc\ud0a4\uace0 \uc2f6\uc9c0\ub3c4 \uc54a\uc558\uace0 \uadf8 \uc0ac\ub78c\ub4e4\uc5d0\uac8c \uc5b4\ub5a0\ud55c \ub3c8\ub3c4 \uc904 \uc0dd\uac01\uc774 \uc5c6\uc5c8\uc2b5\ub2c8\ub2e4. \uc81c \uc6f9\uc0ac\uc774\ud2b8\uc5d0 \uc77c\uc5b4\ub09c \uc77c\uc740 \ub9c8\uce58 \ud14c\ub7ec \uac19\uc558\uc2b5\ub2c8\ub2e4. \uc5ec\ub7ec\ubd84\uc774 \uadf8\ub4e4\uc5d0\uac8c \ub3c8\uc744 \uc900\ub2e4\uba74 \ub610 \uacf5\uaca9\uc774 \uc2dc\uc791\ub420 \uac83\uc785\ub2c8\ub2e4.\"\ub77c\uace0 \ub9d0\ud558\uc600\ub2e4."} {"question": "\uad50\uc721\ubd80 \uc7a5\uad00 \ud1f4\uc9c4 \uc11c\uba85\uc6b4\ub3d9\uc740 \uc5b4\ub514\uc11c \ubc8c\uc778\uac83\uc778\uac00?", "paragraph": "\uc7a5\uad00 \uc7ac\uc9c1 \uc911 \ud559\uae09\ub2f9 50\uba85, 60\uba85 \uc774\uc0c1\uc758 \uacfc\ubc00 \ud559\uae09\uc218\ub97c \uc904\uc774\ub824\ub294 \ub178\ub825\uc744 \ucd94\uc9c4\ud558\uc600\uc73c\ub098 \uc2e4\ud328\ud558\uc600\ub2e4. \uadf8\ub7ec\ub098 \uadf8\uc758 \uc2dc\ub3c4\ub294 \ud6c4\uc77c '2\ub144 \ub9cc\uc5d0 \ucd08, \uc911\ub4f1\ud559\uad50 \ud559\uae09\ub2f9 \uc815\uc6d0\uc218 35\uba85\uc73c\ub85c \uc904\uc774\ub294' \ud6a8\uacfc\ub97c \ub0c8\ub2e4. \uadf8\ub7ec\ub098 \uc57c\uac04 \uc790\uc728\ud559\uc2b5\uc744 \uc5c6\uc560\uace0 \uc6d4\uac04 \ubaa8\uc758\uace0\uc0ac\ub97c \ucd95\uc18c\ud558\uba74\uc11c \"\ud2b9\uae30 \ud558\ub098\ub9cc \uc788\uc73c\uba74 \ub300\ud559\uc5d0 \uac08 \uc218 \uc788\ub294 \uc81c\ub3c4\ub97c \ub9cc\ub4e4\uaca0\ub2e4\"\uace0 \uc120\ud3ec\ud574 \uace0\uad50 \ud559\uc2b5\uc774 \uc804\ubc18\uc801\uc73c\ub85c \ub290\uc2a8\ud574\uc84c\uace0, \uc774\ub85c \uc778\ud574 \ud559\ub825 \uc800\ud558 \uc2dc\ube44\uac00 \uc77c\uc5c8\ub2e4. \uc774\ub294 \uc77c\ubcf8\uc758 \uc720\ud1a0\ub9ac \uad50\uc721\uc744 \ud749\ub0b4 \ub0b8 \uc81c\ub3c4\ub77c\uace0 \ube44\ud310\ubc1b\uc558\ub2e4. \ud55c\uad6d\uad50\uc6d0\ub2e8\uccb4\ucd1d\uc5f0\ud569\ud68c(\uad50\ucd1d)\uc740 \uad50\uc721\ubd80 \uc7a5\uad00 \ud1f4\uc9c4 \uc11c\uba85\uc6b4\ub3d9\uc744 \ubc8c\uc600\uc73c\uba70, \uc804\uad6d\uad50\uc9c1\uc6d0\ub178\ub3d9\uc870\ud569(\uc804\uad50\uc870)\uc740 \uc11c\uba85\uc5d0 \ub3d9\ucc38\ud558\uc9c0 \uc54a\ub294 \ud0dc\ub3c4\ub97c \ucde8\ud588\ub2e4. \uadf8\ub7ec\ub098 \uc77c\ubd80 \uad50\uc6d0\ub4e4\uc740 \ubc29\uad00\ud558\uac70\ub098 \ucc38\uc5ec\ud558\uae30\ub3c4 \ud588\ub2e4.", "answer": "\ud55c\uad6d\uad50\uc6d0\ub2e8\uccb4\ucd1d\uc5f0\ud569\ud68c", "sentence": "\ud55c\uad6d\uad50\uc6d0\ub2e8\uccb4\ucd1d\uc5f0\ud569\ud68c (\uad50\ucd1d)", "paragraph_sentence": "\uc7a5\uad00 \uc7ac\uc9c1 \uc911 \ud559\uae09\ub2f9 50\uba85, 60\uba85 \uc774\uc0c1\uc758 \uacfc\ubc00 \ud559\uae09\uc218\ub97c \uc904\uc774\ub824\ub294 \ub178\ub825\uc744 \ucd94\uc9c4\ud558\uc600\uc73c\ub098 \uc2e4\ud328\ud558\uc600\ub2e4. \uadf8\ub7ec\ub098 \uadf8\uc758 \uc2dc\ub3c4\ub294 \ud6c4\uc77c '2\ub144 \ub9cc\uc5d0 \ucd08, \uc911\ub4f1\ud559\uad50 \ud559\uae09\ub2f9 \uc815\uc6d0\uc218 35\uba85\uc73c\ub85c \uc904\uc774\ub294' \ud6a8\uacfc\ub97c \ub0c8\ub2e4. \uadf8\ub7ec\ub098 \uc57c\uac04 \uc790\uc728\ud559\uc2b5\uc744 \uc5c6\uc560\uace0 \uc6d4\uac04 \ubaa8\uc758\uace0\uc0ac\ub97c \ucd95\uc18c\ud558\uba74\uc11c \"\ud2b9\uae30 \ud558\ub098\ub9cc \uc788\uc73c\uba74 \ub300\ud559\uc5d0 \uac08 \uc218 \uc788\ub294 \uc81c\ub3c4\ub97c \ub9cc\ub4e4\uaca0\ub2e4\"\uace0 \uc120\ud3ec\ud574 \uace0\uad50 \ud559\uc2b5\uc774 \uc804\ubc18\uc801\uc73c\ub85c \ub290\uc2a8\ud574\uc84c\uace0, \uc774\ub85c \uc778\ud574 \ud559\ub825 \uc800\ud558 \uc2dc\ube44\uac00 \uc77c\uc5c8\ub2e4. \uc774\ub294 \uc77c\ubcf8\uc758 \uc720\ud1a0\ub9ac \uad50\uc721\uc744 \ud749\ub0b4 \ub0b8 \uc81c\ub3c4\ub77c\uace0 \ube44\ud310\ubc1b\uc558\ub2e4. \ud55c\uad6d\uad50\uc6d0\ub2e8\uccb4\ucd1d\uc5f0\ud569\ud68c (\uad50\ucd1d) \uc740 \uad50\uc721\ubd80 \uc7a5\uad00 \ud1f4\uc9c4 \uc11c\uba85\uc6b4\ub3d9\uc744 \ubc8c\uc600\uc73c\uba70, \uc804\uad6d\uad50\uc9c1\uc6d0\ub178\ub3d9\uc870\ud569(\uc804\uad50\uc870)\uc740 \uc11c\uba85\uc5d0 \ub3d9\ucc38\ud558\uc9c0 \uc54a\ub294 \ud0dc\ub3c4\ub97c \ucde8\ud588\ub2e4. \uadf8\ub7ec\ub098 \uc77c\ubd80 \uad50\uc6d0\ub4e4\uc740 \ubc29\uad00\ud558\uac70\ub098 \ucc38\uc5ec\ud558\uae30\ub3c4 \ud588\ub2e4.", "paragraph_answer": "\uc7a5\uad00 \uc7ac\uc9c1 \uc911 \ud559\uae09\ub2f9 50\uba85, 60\uba85 \uc774\uc0c1\uc758 \uacfc\ubc00 \ud559\uae09\uc218\ub97c \uc904\uc774\ub824\ub294 \ub178\ub825\uc744 \ucd94\uc9c4\ud558\uc600\uc73c\ub098 \uc2e4\ud328\ud558\uc600\ub2e4. \uadf8\ub7ec\ub098 \uadf8\uc758 \uc2dc\ub3c4\ub294 \ud6c4\uc77c '2\ub144 \ub9cc\uc5d0 \ucd08, \uc911\ub4f1\ud559\uad50 \ud559\uae09\ub2f9 \uc815\uc6d0\uc218 35\uba85\uc73c\ub85c \uc904\uc774\ub294' \ud6a8\uacfc\ub97c \ub0c8\ub2e4. \uadf8\ub7ec\ub098 \uc57c\uac04 \uc790\uc728\ud559\uc2b5\uc744 \uc5c6\uc560\uace0 \uc6d4\uac04 \ubaa8\uc758\uace0\uc0ac\ub97c \ucd95\uc18c\ud558\uba74\uc11c \"\ud2b9\uae30 \ud558\ub098\ub9cc \uc788\uc73c\uba74 \ub300\ud559\uc5d0 \uac08 \uc218 \uc788\ub294 \uc81c\ub3c4\ub97c \ub9cc\ub4e4\uaca0\ub2e4\"\uace0 \uc120\ud3ec\ud574 \uace0\uad50 \ud559\uc2b5\uc774 \uc804\ubc18\uc801\uc73c\ub85c \ub290\uc2a8\ud574\uc84c\uace0, \uc774\ub85c \uc778\ud574 \ud559\ub825 \uc800\ud558 \uc2dc\ube44\uac00 \uc77c\uc5c8\ub2e4. \uc774\ub294 \uc77c\ubcf8\uc758 \uc720\ud1a0\ub9ac \uad50\uc721\uc744 \ud749\ub0b4 \ub0b8 \uc81c\ub3c4\ub77c\uace0 \ube44\ud310\ubc1b\uc558\ub2e4. \ud55c\uad6d\uad50\uc6d0\ub2e8\uccb4\ucd1d\uc5f0\ud569\ud68c (\uad50\ucd1d)\uc740 \uad50\uc721\ubd80 \uc7a5\uad00 \ud1f4\uc9c4 \uc11c\uba85\uc6b4\ub3d9\uc744 \ubc8c\uc600\uc73c\uba70, \uc804\uad6d\uad50\uc9c1\uc6d0\ub178\ub3d9\uc870\ud569(\uc804\uad50\uc870)\uc740 \uc11c\uba85\uc5d0 \ub3d9\ucc38\ud558\uc9c0 \uc54a\ub294 \ud0dc\ub3c4\ub97c \ucde8\ud588\ub2e4. \uadf8\ub7ec\ub098 \uc77c\ubd80 \uad50\uc6d0\ub4e4\uc740 \ubc29\uad00\ud558\uac70\ub098 \ucc38\uc5ec\ud558\uae30\ub3c4 \ud588\ub2e4.", "sentence_answer": " \ud55c\uad6d\uad50\uc6d0\ub2e8\uccb4\ucd1d\uc5f0\ud569\ud68c (\uad50\ucd1d)", "paragraph_id": "5849814-3-1", "paragraph_question": "question: \uad50\uc721\ubd80 \uc7a5\uad00 \ud1f4\uc9c4 \uc11c\uba85\uc6b4\ub3d9\uc740 \uc5b4\ub514\uc11c \ubc8c\uc778\uac83\uc778\uac00?, context: \uc7a5\uad00 \uc7ac\uc9c1 \uc911 \ud559\uae09\ub2f9 50\uba85, 60\uba85 \uc774\uc0c1\uc758 \uacfc\ubc00 \ud559\uae09\uc218\ub97c \uc904\uc774\ub824\ub294 \ub178\ub825\uc744 \ucd94\uc9c4\ud558\uc600\uc73c\ub098 \uc2e4\ud328\ud558\uc600\ub2e4. \uadf8\ub7ec\ub098 \uadf8\uc758 \uc2dc\ub3c4\ub294 \ud6c4\uc77c '2\ub144 \ub9cc\uc5d0 \ucd08, \uc911\ub4f1\ud559\uad50 \ud559\uae09\ub2f9 \uc815\uc6d0\uc218 35\uba85\uc73c\ub85c \uc904\uc774\ub294' \ud6a8\uacfc\ub97c \ub0c8\ub2e4. \uadf8\ub7ec\ub098 \uc57c\uac04 \uc790\uc728\ud559\uc2b5\uc744 \uc5c6\uc560\uace0 \uc6d4\uac04 \ubaa8\uc758\uace0\uc0ac\ub97c \ucd95\uc18c\ud558\uba74\uc11c \"\ud2b9\uae30 \ud558\ub098\ub9cc \uc788\uc73c\uba74 \ub300\ud559\uc5d0 \uac08 \uc218 \uc788\ub294 \uc81c\ub3c4\ub97c \ub9cc\ub4e4\uaca0\ub2e4\"\uace0 \uc120\ud3ec\ud574 \uace0\uad50 \ud559\uc2b5\uc774 \uc804\ubc18\uc801\uc73c\ub85c \ub290\uc2a8\ud574\uc84c\uace0, \uc774\ub85c \uc778\ud574 \ud559\ub825 \uc800\ud558 \uc2dc\ube44\uac00 \uc77c\uc5c8\ub2e4. \uc774\ub294 \uc77c\ubcf8\uc758 \uc720\ud1a0\ub9ac \uad50\uc721\uc744 \ud749\ub0b4 \ub0b8 \uc81c\ub3c4\ub77c\uace0 \ube44\ud310\ubc1b\uc558\ub2e4. \ud55c\uad6d\uad50\uc6d0\ub2e8\uccb4\ucd1d\uc5f0\ud569\ud68c(\uad50\ucd1d)\uc740 \uad50\uc721\ubd80 \uc7a5\uad00 \ud1f4\uc9c4 \uc11c\uba85\uc6b4\ub3d9\uc744 \ubc8c\uc600\uc73c\uba70, \uc804\uad6d\uad50\uc9c1\uc6d0\ub178\ub3d9\uc870\ud569(\uc804\uad50\uc870)\uc740 \uc11c\uba85\uc5d0 \ub3d9\ucc38\ud558\uc9c0 \uc54a\ub294 \ud0dc\ub3c4\ub97c \ucde8\ud588\ub2e4. \uadf8\ub7ec\ub098 \uc77c\ubd80 \uad50\uc6d0\ub4e4\uc740 \ubc29\uad00\ud558\uac70\ub098 \ucc38\uc5ec\ud558\uae30\ub3c4 \ud588\ub2e4."} {"question": "\uac00\uacbd\uc81c\uac00 \ud0dc\uc5b4\ub0ac\uc744 \ub54c \uac74\ub96d\uc81c\uc758 \ub098\uc774\ub294?", "paragraph": "1760\ub144(\uac74\ub96d 25\ub144) 11\uc6d4 13\uc77c\uc5d0 \uc774\uad81 \uc6d0\uba85\uc6d0\uc5d0\uc11c \uac74\ub96d\uc81c\uc640 \uadf8\uc758 \ud6c4\uad81\uc778 \uc601\ube44(\u4ee4\u5983) \uc704\uac00\uc528\uc758 \uc544\ub4e4\ub85c \ud0dc\uc5b4\ub0ac\ub2e4. \uc601\uc5fc\uc774 \ud0dc\uc5b4\ub0a0 \ub54c \uac74\ub96d\uc81c\uc758 \ub098\uc774\ub294 \uc774\ubbf8 50\uc138\uc5ec\uc11c \ud669\uc790\ub4e4 \uc911\uc5d0\uc11c \uc720\ub2ec\ub9ac \ucd1d\uc560\ud558\uc600\ub2e4\uace0 \ud55c\ub2e4. \ub610\ud55c \uc601\uc5fc\uc740 \ud6a8\uc2ec\uc774 \uae4a\uc5b4 \ud669\ud0dc\ud6c4\uc778 \ud6a8\uc131\ud5cc\ud669\ud6c4\uc5d0\uac8c \ubb38\uc548\uc744 \uac8c\uc744\ub9ac\ud558\uc9c0 \uc54a\uc558\uace0 \uc544\ubc84\uc9c0\uc778 \uac74\ub96d\uc81c\uc5d0\uac8c\ub3c4 \ud6a8\ub97c \ub2e4\ud558\uc790 1773\ub144(\uac74\ub96d 38\ub144), \ub0a8\ub2ec\ub9ac \uc601\uc5fc\uc744 \ub9c8\uc74c\uc5d0 \ub450\uc5c8\ub358 \uac74\ub96d\uc81c\ub294 \uc800\uc704\ube44\uac74\ubc95\uc758 \uc6d0\uce59\uc5d0 \ub530\ub77c \uc601\uc5fc\uc744 \ud669\ud0dc\uc790\uc5d0 \ube44\ubc00\ub9ac\uc5d0 \ucc45\ubd09\ud558\uc600\ub2e4. \ubcf8\ub798 \uac74\ub96d\uc81c\ub294 \uc801\uc7a5\uc790\uc778 \uc81c2\ud669\uc790 \uc601\ub828(\u6c38\u7489)\uc744 \ud669\ud0dc\uc790\uc5d0 \ubd09\ud558\uc600\uc73c\ub098 \uc601\ub828\uc774 \uc694\uc808\ud558\uc600\uace0 \ub2e4\uc2dc \uc801\ucc28\uc790\uc778 \uc81c7\ud669\uc790 \uc601\uc885(\u6c38\u742e)\uc73c\ub85c \ucc45\ubd09\ud558\uc600\uc73c\ub098 \uc601\uc885\ub9c8\uc800\ub3c4 3\uc138\uc5d0 \uc694\uc808\ud55c \uc774\ud6c4\ub85c \uac74\ub96d\uc81c\ub294 30\ub144 \uac00\uae4c\uc774 \ud669\ud0dc\uc790\ub97c \uc138\uc6b0\uc9c0 \uc54a\uc558\ub2e4. \uc774 \uc774\ud6c4\ub85c\ub294 \uc870\uc815\uc758 \uc2e0\ub9dd\uc744 \uc5bb\uc740 \uc81c5\ud669\uc790 \uc601\uce5c\uc655 \uc601\uae30\uac00 \uc788\uc5c8\uc9c0\ub9cc \uadf8\ub9c8\uc800\ub3c4 1766\ub144(\uac74\ub96d 31\ub144)\uc5d0 \uc0ac\ub9dd\ud55c \ub4a4\ub85c\ub294 \ub531\ud788 \uac74\ub96d\uc81c\uc640 \uc870\uc815\uc5d0\uc11c \uc8fc\ubaa9\ud558\ub294 \ud669\uc790\uac00 \uc5c6\uc5c8\ub2e4. \ud558\uc9c0\ub9cc \uac74\ub96d\uc81c\uac00 \uc5b4\ub9b0 \uc601\uc5fc\uc744 \ube44\ubc00\ub9ac\uc5d0 \ud669\ud0dc\uc790\uc5d0 \ucc45\ubd09\ud55c \uac83\uc740 \uac74\ub96d\uc81c\uac00 \uc601\uc5fc\uacfc \uadf8\uc758 \uc5b4\uba38\ub2c8\uc778 \uc601\uadc0\ube44 \uc704\uac00\uc528\ub97c \ub9e4\uc6b0 \uc544\uaf08\ub2e4\ub294 \uac83\uc744 \ubcf4\uc5ec\uc8fc\ub294 \ub300\ubaa9\uc774\ub2e4. 1775\ub144(\uac74\ub96d 40\ub144), \uc601\uc5fc\uc758 \uc5b4\uba38\ub2c8\uc778 \uc601\uadc0\ube44\uac00 \uc8fd\uc790 \uac74\ub96d\uc81c\ub294 \ub9e4\uc6b0 \uc2ac\ud37c\ud558\uc5ec \uadf8\ub140\uc5d0\uac8c \uc601\uc758\ud669\uadc0\ube44(\u4ee4\u61ff\u7687\u8cb4\u5983)\ub780 \uc2dc\ud638\ub97c \ub0b4\ub9ac\uace0 \uc601\uc5fc\uacfc \uadf8\uc758 \ub3d9\ubcf5 \ud615\uc81c\ub4e4\uc744 \ub354\uc6b1 \uacc1\uc5d0 \ub450\uace0 \ucd1d\uc560\ud558\uc600\ub2e4. 1789\ub144(\uac74\ub96d 54\ub144)\uc5d0 \uac74\ub96d\uc81c\ub294 \uc870\uc815\ud68c\uc758\ub97c \uc5f4\uace0 \uc81c15\ud669\uc790 \uc601\uc5fc\uc744 \ud654\uc11d\uce5c\uc655(\u548c\u78a9\u89aa\u738b)\uc5d0 \ubd09\ud558\uc5ec \uac00\uce5c\uc655(\u5609\u89aa\u738b)\uc774\ub77c \ubd80\ub974\uac8c \ud558\uace0 \uc5b4\uc804\ud68c\uc758\uc5d0 \ucc38\uc11d\ud558\uc5ec \uc5ec\ub7ec \ub300\uc2e0\ub4e4\uacfc \uc815\uc0ac\ub97c \ub17c\uc758\ud560 \uc218 \uc788\ub3c4\ub85d \ud5c8\ub77d\ud558\uc600\ub2e4.", "answer": "50\uc138", "sentence": "\uc774\ubbf8 50\uc138 \uc5ec\uc11c \ud669\uc790\ub4e4 \uc911\uc5d0\uc11c \uc720\ub2ec\ub9ac \ucd1d\uc560\ud558\uc600\ub2e4\uace0 \ud55c\ub2e4.", "paragraph_sentence": "1760\ub144(\uac74\ub96d 25\ub144) 11\uc6d4 13\uc77c\uc5d0 \uc774\uad81 \uc6d0\uba85\uc6d0\uc5d0\uc11c \uac74\ub96d\uc81c\uc640 \uadf8\uc758 \ud6c4\uad81\uc778 \uc601\ube44(\u4ee4\u5983) \uc704\uac00\uc528\uc758 \uc544\ub4e4\ub85c \ud0dc\uc5b4\ub0ac\ub2e4. \uc601\uc5fc\uc774 \ud0dc\uc5b4\ub0a0 \ub54c \uac74\ub96d\uc81c\uc758 \ub098\uc774\ub294 \uc774\ubbf8 50\uc138 \uc5ec\uc11c \ud669\uc790\ub4e4 \uc911\uc5d0\uc11c \uc720\ub2ec\ub9ac \ucd1d\uc560\ud558\uc600\ub2e4\uace0 \ud55c\ub2e4. \ub610\ud55c \uc601\uc5fc\uc740 \ud6a8\uc2ec\uc774 \uae4a\uc5b4 \ud669\ud0dc\ud6c4\uc778 \ud6a8\uc131\ud5cc\ud669\ud6c4\uc5d0\uac8c \ubb38\uc548\uc744 \uac8c\uc744\ub9ac\ud558\uc9c0 \uc54a\uc558\uace0 \uc544\ubc84\uc9c0\uc778 \uac74\ub96d\uc81c\uc5d0\uac8c\ub3c4 \ud6a8\ub97c \ub2e4\ud558\uc790 1773\ub144(\uac74\ub96d 38\ub144), \ub0a8\ub2ec\ub9ac \uc601\uc5fc\uc744 \ub9c8\uc74c\uc5d0 \ub450\uc5c8\ub358 \uac74\ub96d\uc81c\ub294 \uc800\uc704\ube44\uac74\ubc95\uc758 \uc6d0\uce59\uc5d0 \ub530\ub77c \uc601\uc5fc\uc744 \ud669\ud0dc\uc790\uc5d0 \ube44\ubc00\ub9ac\uc5d0 \ucc45\ubd09\ud558\uc600\ub2e4. \ubcf8\ub798 \uac74\ub96d\uc81c\ub294 \uc801\uc7a5\uc790\uc778 \uc81c2\ud669\uc790 \uc601\ub828(\u6c38\u7489)\uc744 \ud669\ud0dc\uc790\uc5d0 \ubd09\ud558\uc600\uc73c\ub098 \uc601\ub828\uc774 \uc694\uc808\ud558\uc600\uace0 \ub2e4\uc2dc \uc801\ucc28\uc790\uc778 \uc81c7\ud669\uc790 \uc601\uc885(\u6c38\u742e)\uc73c\ub85c \ucc45\ubd09\ud558\uc600\uc73c\ub098 \uc601\uc885\ub9c8\uc800\ub3c4 3\uc138\uc5d0 \uc694\uc808\ud55c \uc774\ud6c4\ub85c \uac74\ub96d\uc81c\ub294 30\ub144 \uac00\uae4c\uc774 \ud669\ud0dc\uc790\ub97c \uc138\uc6b0\uc9c0 \uc54a\uc558\ub2e4. \uc774 \uc774\ud6c4\ub85c\ub294 \uc870\uc815\uc758 \uc2e0\ub9dd\uc744 \uc5bb\uc740 \uc81c5\ud669\uc790 \uc601\uce5c\uc655 \uc601\uae30\uac00 \uc788\uc5c8\uc9c0\ub9cc \uadf8\ub9c8\uc800\ub3c4 1766\ub144(\uac74\ub96d 31\ub144)\uc5d0 \uc0ac\ub9dd\ud55c \ub4a4\ub85c\ub294 \ub531\ud788 \uac74\ub96d\uc81c\uc640 \uc870\uc815\uc5d0\uc11c \uc8fc\ubaa9\ud558\ub294 \ud669\uc790\uac00 \uc5c6\uc5c8\ub2e4. \ud558\uc9c0\ub9cc \uac74\ub96d\uc81c\uac00 \uc5b4\ub9b0 \uc601\uc5fc\uc744 \ube44\ubc00\ub9ac\uc5d0 \ud669\ud0dc\uc790\uc5d0 \ucc45\ubd09\ud55c \uac83\uc740 \uac74\ub96d\uc81c\uac00 \uc601\uc5fc\uacfc \uadf8\uc758 \uc5b4\uba38\ub2c8\uc778 \uc601\uadc0\ube44 \uc704\uac00\uc528\ub97c \ub9e4\uc6b0 \uc544\uaf08\ub2e4\ub294 \uac83\uc744 \ubcf4\uc5ec\uc8fc\ub294 \ub300\ubaa9\uc774\ub2e4. 1775\ub144(\uac74\ub96d 40\ub144), \uc601\uc5fc\uc758 \uc5b4\uba38\ub2c8\uc778 \uc601\uadc0\ube44\uac00 \uc8fd\uc790 \uac74\ub96d\uc81c\ub294 \ub9e4\uc6b0 \uc2ac\ud37c\ud558\uc5ec \uadf8\ub140\uc5d0\uac8c \uc601\uc758\ud669\uadc0\ube44(\u4ee4\u61ff\u7687\u8cb4\u5983)\ub780 \uc2dc\ud638\ub97c \ub0b4\ub9ac\uace0 \uc601\uc5fc\uacfc \uadf8\uc758 \ub3d9\ubcf5 \ud615\uc81c\ub4e4\uc744 \ub354\uc6b1 \uacc1\uc5d0 \ub450\uace0 \ucd1d\uc560\ud558\uc600\ub2e4. 1789\ub144(\uac74\ub96d 54\ub144)\uc5d0 \uac74\ub96d\uc81c\ub294 \uc870\uc815\ud68c\uc758\ub97c \uc5f4\uace0 \uc81c15\ud669\uc790 \uc601\uc5fc\uc744 \ud654\uc11d\uce5c\uc655(\u548c\u78a9\u89aa\u738b)\uc5d0 \ubd09\ud558\uc5ec \uac00\uce5c\uc655(\u5609\u89aa\u738b)\uc774\ub77c \ubd80\ub974\uac8c \ud558\uace0 \uc5b4\uc804\ud68c\uc758\uc5d0 \ucc38\uc11d\ud558\uc5ec \uc5ec\ub7ec \ub300\uc2e0\ub4e4\uacfc \uc815\uc0ac\ub97c \ub17c\uc758\ud560 \uc218 \uc788\ub3c4\ub85d \ud5c8\ub77d\ud558\uc600\ub2e4.", "paragraph_answer": "1760\ub144(\uac74\ub96d 25\ub144) 11\uc6d4 13\uc77c\uc5d0 \uc774\uad81 \uc6d0\uba85\uc6d0\uc5d0\uc11c \uac74\ub96d\uc81c\uc640 \uadf8\uc758 \ud6c4\uad81\uc778 \uc601\ube44(\u4ee4\u5983) \uc704\uac00\uc528\uc758 \uc544\ub4e4\ub85c \ud0dc\uc5b4\ub0ac\ub2e4. \uc601\uc5fc\uc774 \ud0dc\uc5b4\ub0a0 \ub54c \uac74\ub96d\uc81c\uc758 \ub098\uc774\ub294 \uc774\ubbf8 50\uc138 \uc5ec\uc11c \ud669\uc790\ub4e4 \uc911\uc5d0\uc11c \uc720\ub2ec\ub9ac \ucd1d\uc560\ud558\uc600\ub2e4\uace0 \ud55c\ub2e4. \ub610\ud55c \uc601\uc5fc\uc740 \ud6a8\uc2ec\uc774 \uae4a\uc5b4 \ud669\ud0dc\ud6c4\uc778 \ud6a8\uc131\ud5cc\ud669\ud6c4\uc5d0\uac8c \ubb38\uc548\uc744 \uac8c\uc744\ub9ac\ud558\uc9c0 \uc54a\uc558\uace0 \uc544\ubc84\uc9c0\uc778 \uac74\ub96d\uc81c\uc5d0\uac8c\ub3c4 \ud6a8\ub97c \ub2e4\ud558\uc790 1773\ub144(\uac74\ub96d 38\ub144), \ub0a8\ub2ec\ub9ac \uc601\uc5fc\uc744 \ub9c8\uc74c\uc5d0 \ub450\uc5c8\ub358 \uac74\ub96d\uc81c\ub294 \uc800\uc704\ube44\uac74\ubc95\uc758 \uc6d0\uce59\uc5d0 \ub530\ub77c \uc601\uc5fc\uc744 \ud669\ud0dc\uc790\uc5d0 \ube44\ubc00\ub9ac\uc5d0 \ucc45\ubd09\ud558\uc600\ub2e4. \ubcf8\ub798 \uac74\ub96d\uc81c\ub294 \uc801\uc7a5\uc790\uc778 \uc81c2\ud669\uc790 \uc601\ub828(\u6c38\u7489)\uc744 \ud669\ud0dc\uc790\uc5d0 \ubd09\ud558\uc600\uc73c\ub098 \uc601\ub828\uc774 \uc694\uc808\ud558\uc600\uace0 \ub2e4\uc2dc \uc801\ucc28\uc790\uc778 \uc81c7\ud669\uc790 \uc601\uc885(\u6c38\u742e)\uc73c\ub85c \ucc45\ubd09\ud558\uc600\uc73c\ub098 \uc601\uc885\ub9c8\uc800\ub3c4 3\uc138\uc5d0 \uc694\uc808\ud55c \uc774\ud6c4\ub85c \uac74\ub96d\uc81c\ub294 30\ub144 \uac00\uae4c\uc774 \ud669\ud0dc\uc790\ub97c \uc138\uc6b0\uc9c0 \uc54a\uc558\ub2e4. \uc774 \uc774\ud6c4\ub85c\ub294 \uc870\uc815\uc758 \uc2e0\ub9dd\uc744 \uc5bb\uc740 \uc81c5\ud669\uc790 \uc601\uce5c\uc655 \uc601\uae30\uac00 \uc788\uc5c8\uc9c0\ub9cc \uadf8\ub9c8\uc800\ub3c4 1766\ub144(\uac74\ub96d 31\ub144)\uc5d0 \uc0ac\ub9dd\ud55c \ub4a4\ub85c\ub294 \ub531\ud788 \uac74\ub96d\uc81c\uc640 \uc870\uc815\uc5d0\uc11c \uc8fc\ubaa9\ud558\ub294 \ud669\uc790\uac00 \uc5c6\uc5c8\ub2e4. \ud558\uc9c0\ub9cc \uac74\ub96d\uc81c\uac00 \uc5b4\ub9b0 \uc601\uc5fc\uc744 \ube44\ubc00\ub9ac\uc5d0 \ud669\ud0dc\uc790\uc5d0 \ucc45\ubd09\ud55c \uac83\uc740 \uac74\ub96d\uc81c\uac00 \uc601\uc5fc\uacfc \uadf8\uc758 \uc5b4\uba38\ub2c8\uc778 \uc601\uadc0\ube44 \uc704\uac00\uc528\ub97c \ub9e4\uc6b0 \uc544\uaf08\ub2e4\ub294 \uac83\uc744 \ubcf4\uc5ec\uc8fc\ub294 \ub300\ubaa9\uc774\ub2e4. 1775\ub144(\uac74\ub96d 40\ub144), \uc601\uc5fc\uc758 \uc5b4\uba38\ub2c8\uc778 \uc601\uadc0\ube44\uac00 \uc8fd\uc790 \uac74\ub96d\uc81c\ub294 \ub9e4\uc6b0 \uc2ac\ud37c\ud558\uc5ec \uadf8\ub140\uc5d0\uac8c \uc601\uc758\ud669\uadc0\ube44(\u4ee4\u61ff\u7687\u8cb4\u5983)\ub780 \uc2dc\ud638\ub97c \ub0b4\ub9ac\uace0 \uc601\uc5fc\uacfc \uadf8\uc758 \ub3d9\ubcf5 \ud615\uc81c\ub4e4\uc744 \ub354\uc6b1 \uacc1\uc5d0 \ub450\uace0 \ucd1d\uc560\ud558\uc600\ub2e4. 1789\ub144(\uac74\ub96d 54\ub144)\uc5d0 \uac74\ub96d\uc81c\ub294 \uc870\uc815\ud68c\uc758\ub97c \uc5f4\uace0 \uc81c15\ud669\uc790 \uc601\uc5fc\uc744 \ud654\uc11d\uce5c\uc655(\u548c\u78a9\u89aa\u738b)\uc5d0 \ubd09\ud558\uc5ec \uac00\uce5c\uc655(\u5609\u89aa\u738b)\uc774\ub77c \ubd80\ub974\uac8c \ud558\uace0 \uc5b4\uc804\ud68c\uc758\uc5d0 \ucc38\uc11d\ud558\uc5ec \uc5ec\ub7ec \ub300\uc2e0\ub4e4\uacfc \uc815\uc0ac\ub97c \ub17c\uc758\ud560 \uc218 \uc788\ub3c4\ub85d \ud5c8\ub77d\ud558\uc600\ub2e4.", "sentence_answer": "\uc774\ubbf8 50\uc138 \uc5ec\uc11c \ud669\uc790\ub4e4 \uc911\uc5d0\uc11c \uc720\ub2ec\ub9ac \ucd1d\uc560\ud558\uc600\ub2e4\uace0 \ud55c\ub2e4.", "paragraph_id": "6500266-2-0", "paragraph_question": "question: \uac00\uacbd\uc81c\uac00 \ud0dc\uc5b4\ub0ac\uc744 \ub54c \uac74\ub96d\uc81c\uc758 \ub098\uc774\ub294?, context: 1760\ub144(\uac74\ub96d 25\ub144) 11\uc6d4 13\uc77c\uc5d0 \uc774\uad81 \uc6d0\uba85\uc6d0\uc5d0\uc11c \uac74\ub96d\uc81c\uc640 \uadf8\uc758 \ud6c4\uad81\uc778 \uc601\ube44(\u4ee4\u5983) \uc704\uac00\uc528\uc758 \uc544\ub4e4\ub85c \ud0dc\uc5b4\ub0ac\ub2e4. \uc601\uc5fc\uc774 \ud0dc\uc5b4\ub0a0 \ub54c \uac74\ub96d\uc81c\uc758 \ub098\uc774\ub294 \uc774\ubbf8 50\uc138\uc5ec\uc11c \ud669\uc790\ub4e4 \uc911\uc5d0\uc11c \uc720\ub2ec\ub9ac \ucd1d\uc560\ud558\uc600\ub2e4\uace0 \ud55c\ub2e4. \ub610\ud55c \uc601\uc5fc\uc740 \ud6a8\uc2ec\uc774 \uae4a\uc5b4 \ud669\ud0dc\ud6c4\uc778 \ud6a8\uc131\ud5cc\ud669\ud6c4\uc5d0\uac8c \ubb38\uc548\uc744 \uac8c\uc744\ub9ac\ud558\uc9c0 \uc54a\uc558\uace0 \uc544\ubc84\uc9c0\uc778 \uac74\ub96d\uc81c\uc5d0\uac8c\ub3c4 \ud6a8\ub97c \ub2e4\ud558\uc790 1773\ub144(\uac74\ub96d 38\ub144), \ub0a8\ub2ec\ub9ac \uc601\uc5fc\uc744 \ub9c8\uc74c\uc5d0 \ub450\uc5c8\ub358 \uac74\ub96d\uc81c\ub294 \uc800\uc704\ube44\uac74\ubc95\uc758 \uc6d0\uce59\uc5d0 \ub530\ub77c \uc601\uc5fc\uc744 \ud669\ud0dc\uc790\uc5d0 \ube44\ubc00\ub9ac\uc5d0 \ucc45\ubd09\ud558\uc600\ub2e4. \ubcf8\ub798 \uac74\ub96d\uc81c\ub294 \uc801\uc7a5\uc790\uc778 \uc81c2\ud669\uc790 \uc601\ub828(\u6c38\u7489)\uc744 \ud669\ud0dc\uc790\uc5d0 \ubd09\ud558\uc600\uc73c\ub098 \uc601\ub828\uc774 \uc694\uc808\ud558\uc600\uace0 \ub2e4\uc2dc \uc801\ucc28\uc790\uc778 \uc81c7\ud669\uc790 \uc601\uc885(\u6c38\u742e)\uc73c\ub85c \ucc45\ubd09\ud558\uc600\uc73c\ub098 \uc601\uc885\ub9c8\uc800\ub3c4 3\uc138\uc5d0 \uc694\uc808\ud55c \uc774\ud6c4\ub85c \uac74\ub96d\uc81c\ub294 30\ub144 \uac00\uae4c\uc774 \ud669\ud0dc\uc790\ub97c \uc138\uc6b0\uc9c0 \uc54a\uc558\ub2e4. \uc774 \uc774\ud6c4\ub85c\ub294 \uc870\uc815\uc758 \uc2e0\ub9dd\uc744 \uc5bb\uc740 \uc81c5\ud669\uc790 \uc601\uce5c\uc655 \uc601\uae30\uac00 \uc788\uc5c8\uc9c0\ub9cc \uadf8\ub9c8\uc800\ub3c4 1766\ub144(\uac74\ub96d 31\ub144)\uc5d0 \uc0ac\ub9dd\ud55c \ub4a4\ub85c\ub294 \ub531\ud788 \uac74\ub96d\uc81c\uc640 \uc870\uc815\uc5d0\uc11c \uc8fc\ubaa9\ud558\ub294 \ud669\uc790\uac00 \uc5c6\uc5c8\ub2e4. \ud558\uc9c0\ub9cc \uac74\ub96d\uc81c\uac00 \uc5b4\ub9b0 \uc601\uc5fc\uc744 \ube44\ubc00\ub9ac\uc5d0 \ud669\ud0dc\uc790\uc5d0 \ucc45\ubd09\ud55c \uac83\uc740 \uac74\ub96d\uc81c\uac00 \uc601\uc5fc\uacfc \uadf8\uc758 \uc5b4\uba38\ub2c8\uc778 \uc601\uadc0\ube44 \uc704\uac00\uc528\ub97c \ub9e4\uc6b0 \uc544\uaf08\ub2e4\ub294 \uac83\uc744 \ubcf4\uc5ec\uc8fc\ub294 \ub300\ubaa9\uc774\ub2e4. 1775\ub144(\uac74\ub96d 40\ub144), \uc601\uc5fc\uc758 \uc5b4\uba38\ub2c8\uc778 \uc601\uadc0\ube44\uac00 \uc8fd\uc790 \uac74\ub96d\uc81c\ub294 \ub9e4\uc6b0 \uc2ac\ud37c\ud558\uc5ec \uadf8\ub140\uc5d0\uac8c \uc601\uc758\ud669\uadc0\ube44(\u4ee4\u61ff\u7687\u8cb4\u5983)\ub780 \uc2dc\ud638\ub97c \ub0b4\ub9ac\uace0 \uc601\uc5fc\uacfc \uadf8\uc758 \ub3d9\ubcf5 \ud615\uc81c\ub4e4\uc744 \ub354\uc6b1 \uacc1\uc5d0 \ub450\uace0 \ucd1d\uc560\ud558\uc600\ub2e4. 1789\ub144(\uac74\ub96d 54\ub144)\uc5d0 \uac74\ub96d\uc81c\ub294 \uc870\uc815\ud68c\uc758\ub97c \uc5f4\uace0 \uc81c15\ud669\uc790 \uc601\uc5fc\uc744 \ud654\uc11d\uce5c\uc655(\u548c\u78a9\u89aa\u738b)\uc5d0 \ubd09\ud558\uc5ec \uac00\uce5c\uc655(\u5609\u89aa\u738b)\uc774\ub77c \ubd80\ub974\uac8c \ud558\uace0 \uc5b4\uc804\ud68c\uc758\uc5d0 \ucc38\uc11d\ud558\uc5ec \uc5ec\ub7ec \ub300\uc2e0\ub4e4\uacfc \uc815\uc0ac\ub97c \ub17c\uc758\ud560 \uc218 \uc788\ub3c4\ub85d \ud5c8\ub77d\ud558\uc600\ub2e4."} {"question": "1987\ub144 \uac1c\uc815\ub41c \ub300\ud1b5\ub839 \uc120\uac70 \uad00\ub828\ubc95\uc740 \ud5cc\ubc95 \uba87 \uc870\uc778\uac00?", "paragraph": "1987\ub144 10\uc6d4 29\uc77c \uac1c\uc815\ub41c \ud5cc\ubc95 \uc81c68\uc870\uc5d0 \ub530\ub974\uba74 \ub300\ud1b5\ub839 \uc120\uac70\ub294 \uc784\uae30 \ub9cc\ub8cc\uc77c 70\uc77c \uc804\ubd80\ud130 40\uc77c \uc774\uc804\uc5d0 \uc2e4\uc2dc\ud558\uc5ec\uc57c \ud55c\ub2e4. 2004\ub144 3\uc6d4 12\uc77c \uc77c\ubd80\uac1c\uc815\ub41c \uacf5\uc9c1\uc120\uac70\ubc95 \uc81c34\uc870\uc5d0\ub294 \uc784\uae30 \ub9cc\ub8cc\uc77c 70\uc77c \uc804 \uc774\ud6c4 \uccab \ubc88\uc9f8 \uc218\uc694\uc77c\ub85c \uaddc\uc815\ub418\uc5b4 \uc788\ub2e4. \ud604\ud589 \ud5cc\ubc95\uc758 \ubd80\uce59\uc5d0 \uc758\ud558\uc5ec \ub300\ud1b5\ub839\uc758 \uc784\uae30\ub294 \ud5cc\ubc95\uc758 \uc2dc\ud589\uc77c\ub85c\ubd80\ud130 \uac1c\uc2dc\ub418\ub3c4\ub85d \uc815\ud574\uc838 \uc788\ub2e4. \ud604\ud589 \ud5cc\ubc95\uc774 1988\ub144 2\uc6d4 25\uc77c\uc5d0 \uc2dc\ud589\ub418\uc5c8\uace0 \uc911\uac04\uc5d0 \uad90\uc704\ub41c \ub300\ud1b5\ub839\uc774 \uc5c6\uc73c\ubbc0\ub85c \ub300\ud1b5\ub839\uc758 \uc784\uae30\ub294 \ucde8\uc784 \ud6c4 5\ub144\uc774 \uacbd\uacfc\ud55c \ud574\uc758 2\uc6d4 24\uc77c\uc5d0 \ub9cc\ub8cc\ub41c\ub2e4. 1987\ub144 \uc774\ud6c4 \ub300\ud1b5\ub839 \uc120\uac70\ub294 5\ub144\ub9c8\ub2e4 12\uc6d4 16\uc77c(1987\ub144), 12\uc6d4 18\uc77c(1992\ub144, 1997\ub144), 12\uc6d4 19\uc77c(2002\ub144, 2007\ub144)\uc5d0 \uc2e4\uc2dc\ub418\uc5c8\ub2e4.", "answer": "\uc81c68\uc870", "sentence": "1987\ub144 10\uc6d4 29\uc77c \uac1c\uc815\ub41c \ud5cc\ubc95 \uc81c68\uc870 \uc5d0", "paragraph_sentence": " 1987\ub144 10\uc6d4 29\uc77c \uac1c\uc815\ub41c \ud5cc\ubc95 \uc81c68\uc870 \uc5d0 \ub530\ub974\uba74 \ub300\ud1b5\ub839 \uc120\uac70\ub294 \uc784\uae30 \ub9cc\ub8cc\uc77c 70\uc77c \uc804\ubd80\ud130 40\uc77c \uc774\uc804\uc5d0 \uc2e4\uc2dc\ud558\uc5ec\uc57c \ud55c\ub2e4. 2004\ub144 3\uc6d4 12\uc77c \uc77c\ubd80\uac1c\uc815\ub41c \uacf5\uc9c1\uc120\uac70\ubc95 \uc81c34\uc870\uc5d0\ub294 \uc784\uae30 \ub9cc\ub8cc\uc77c 70\uc77c \uc804 \uc774\ud6c4 \uccab \ubc88\uc9f8 \uc218\uc694\uc77c\ub85c \uaddc\uc815\ub418\uc5b4 \uc788\ub2e4. \ud604\ud589 \ud5cc\ubc95\uc758 \ubd80\uce59\uc5d0 \uc758\ud558\uc5ec \ub300\ud1b5\ub839\uc758 \uc784\uae30\ub294 \ud5cc\ubc95\uc758 \uc2dc\ud589\uc77c\ub85c\ubd80\ud130 \uac1c\uc2dc\ub418\ub3c4\ub85d \uc815\ud574\uc838 \uc788\ub2e4. \ud604\ud589 \ud5cc\ubc95\uc774 1988\ub144 2\uc6d4 25\uc77c\uc5d0 \uc2dc\ud589\ub418\uc5c8\uace0 \uc911\uac04\uc5d0 \uad90\uc704\ub41c \ub300\ud1b5\ub839\uc774 \uc5c6\uc73c\ubbc0\ub85c \ub300\ud1b5\ub839\uc758 \uc784\uae30\ub294 \ucde8\uc784 \ud6c4 5\ub144\uc774 \uacbd\uacfc\ud55c \ud574\uc758 2\uc6d4 24\uc77c\uc5d0 \ub9cc\ub8cc\ub41c\ub2e4. 1987\ub144 \uc774\ud6c4 \ub300\ud1b5\ub839 \uc120\uac70\ub294 5\ub144\ub9c8\ub2e4 12\uc6d4 16\uc77c(1987\ub144), 12\uc6d4 18\uc77c(1992\ub144, 1997\ub144), 12\uc6d4 19\uc77c(2002\ub144, 2007\ub144)\uc5d0 \uc2e4\uc2dc\ub418\uc5c8\ub2e4.", "paragraph_answer": "1987\ub144 10\uc6d4 29\uc77c \uac1c\uc815\ub41c \ud5cc\ubc95 \uc81c68\uc870 \uc5d0 \ub530\ub974\uba74 \ub300\ud1b5\ub839 \uc120\uac70\ub294 \uc784\uae30 \ub9cc\ub8cc\uc77c 70\uc77c \uc804\ubd80\ud130 40\uc77c \uc774\uc804\uc5d0 \uc2e4\uc2dc\ud558\uc5ec\uc57c \ud55c\ub2e4. 2004\ub144 3\uc6d4 12\uc77c \uc77c\ubd80\uac1c\uc815\ub41c \uacf5\uc9c1\uc120\uac70\ubc95 \uc81c34\uc870\uc5d0\ub294 \uc784\uae30 \ub9cc\ub8cc\uc77c 70\uc77c \uc804 \uc774\ud6c4 \uccab \ubc88\uc9f8 \uc218\uc694\uc77c\ub85c \uaddc\uc815\ub418\uc5b4 \uc788\ub2e4. \ud604\ud589 \ud5cc\ubc95\uc758 \ubd80\uce59\uc5d0 \uc758\ud558\uc5ec \ub300\ud1b5\ub839\uc758 \uc784\uae30\ub294 \ud5cc\ubc95\uc758 \uc2dc\ud589\uc77c\ub85c\ubd80\ud130 \uac1c\uc2dc\ub418\ub3c4\ub85d \uc815\ud574\uc838 \uc788\ub2e4. \ud604\ud589 \ud5cc\ubc95\uc774 1988\ub144 2\uc6d4 25\uc77c\uc5d0 \uc2dc\ud589\ub418\uc5c8\uace0 \uc911\uac04\uc5d0 \uad90\uc704\ub41c \ub300\ud1b5\ub839\uc774 \uc5c6\uc73c\ubbc0\ub85c \ub300\ud1b5\ub839\uc758 \uc784\uae30\ub294 \ucde8\uc784 \ud6c4 5\ub144\uc774 \uacbd\uacfc\ud55c \ud574\uc758 2\uc6d4 24\uc77c\uc5d0 \ub9cc\ub8cc\ub41c\ub2e4. 1987\ub144 \uc774\ud6c4 \ub300\ud1b5\ub839 \uc120\uac70\ub294 5\ub144\ub9c8\ub2e4 12\uc6d4 16\uc77c(1987\ub144), 12\uc6d4 18\uc77c(1992\ub144, 1997\ub144), 12\uc6d4 19\uc77c(2002\ub144, 2007\ub144)\uc5d0 \uc2e4\uc2dc\ub418\uc5c8\ub2e4.", "sentence_answer": "1987\ub144 10\uc6d4 29\uc77c \uac1c\uc815\ub41c \ud5cc\ubc95 \uc81c68\uc870 \uc5d0", "paragraph_id": "6656873-0-1", "paragraph_question": "question: 1987\ub144 \uac1c\uc815\ub41c \ub300\ud1b5\ub839 \uc120\uac70 \uad00\ub828\ubc95\uc740 \ud5cc\ubc95 \uba87 \uc870\uc778\uac00?, context: 1987\ub144 10\uc6d4 29\uc77c \uac1c\uc815\ub41c \ud5cc\ubc95 \uc81c68\uc870\uc5d0 \ub530\ub974\uba74 \ub300\ud1b5\ub839 \uc120\uac70\ub294 \uc784\uae30 \ub9cc\ub8cc\uc77c 70\uc77c \uc804\ubd80\ud130 40\uc77c \uc774\uc804\uc5d0 \uc2e4\uc2dc\ud558\uc5ec\uc57c \ud55c\ub2e4. 2004\ub144 3\uc6d4 12\uc77c \uc77c\ubd80\uac1c\uc815\ub41c \uacf5\uc9c1\uc120\uac70\ubc95 \uc81c34\uc870\uc5d0\ub294 \uc784\uae30 \ub9cc\ub8cc\uc77c 70\uc77c \uc804 \uc774\ud6c4 \uccab \ubc88\uc9f8 \uc218\uc694\uc77c\ub85c \uaddc\uc815\ub418\uc5b4 \uc788\ub2e4. \ud604\ud589 \ud5cc\ubc95\uc758 \ubd80\uce59\uc5d0 \uc758\ud558\uc5ec \ub300\ud1b5\ub839\uc758 \uc784\uae30\ub294 \ud5cc\ubc95\uc758 \uc2dc\ud589\uc77c\ub85c\ubd80\ud130 \uac1c\uc2dc\ub418\ub3c4\ub85d \uc815\ud574\uc838 \uc788\ub2e4. \ud604\ud589 \ud5cc\ubc95\uc774 1988\ub144 2\uc6d4 25\uc77c\uc5d0 \uc2dc\ud589\ub418\uc5c8\uace0 \uc911\uac04\uc5d0 \uad90\uc704\ub41c \ub300\ud1b5\ub839\uc774 \uc5c6\uc73c\ubbc0\ub85c \ub300\ud1b5\ub839\uc758 \uc784\uae30\ub294 \ucde8\uc784 \ud6c4 5\ub144\uc774 \uacbd\uacfc\ud55c \ud574\uc758 2\uc6d4 24\uc77c\uc5d0 \ub9cc\ub8cc\ub41c\ub2e4. 1987\ub144 \uc774\ud6c4 \ub300\ud1b5\ub839 \uc120\uac70\ub294 5\ub144\ub9c8\ub2e4 12\uc6d4 16\uc77c(1987\ub144), 12\uc6d4 18\uc77c(1992\ub144, 1997\ub144), 12\uc6d4 19\uc77c(2002\ub144, 2007\ub144)\uc5d0 \uc2e4\uc2dc\ub418\uc5c8\ub2e4."} {"question": "\uad50\uc721\uc758 \uc591\uc801 \uc99d\ub300\ub97c \uc720\ubc1c\ud558\ub294 \uc808\ub300\uc801\uc778 \uc694\uc778\uc774 \ub418\ub294\uac83\uc740?", "paragraph": "\uacbd\uc81c\ubc1c\uc804\uc5d0 \ub530\ub978 \uc0b0\uc5c5\ud654\uc758 \uacfc\uc815\uc5d0\uc11c \uc804\ubb38\uc801\uc778 \uc9c0\uc2dd\u00b7\uae30\ub2a5\u00b7\uae30\uc220\uc744 \uac00\uc9c4 \uc778\ub825\uc5d0 \ub300\ud55c \uc218\uc694\uac00 \uc99d\uac00\ud574 \uc654\ub2e4\ub294 \uac83\uc774\ub2e4. \uc0b0\uc5c5\uad6c\uc870\uc758 \uace0\ub3c4\ud654\ub294 \uc0dd\uc0b0\uc9c1\uc5d0\uc11c\uc870\ucc28 \uc911\ub4f1\uad50\uc721 \uc774\uc0c1\uc758 \uccb4\uacc4\uc801\uc778 \uc9c0\uc2dd\uacfc \uc804\ubb38\uc801\uc778 \uae30\uc220\u00b7\uae30\ub2a5\uc744 \uc694\uad6c\ud558\uac8c \ub418\uc5c8\uace0 \uadf8 \uacb0\uacfc \uc0c1\uae09\uc758 \uad50\uc721\uc740 \uc0ac\ud68c\u00b7\uacbd\uc81c\uc801\uc73c\ub85c \uc720\ub9ac\ud55c \ucde8\uc5c5\uc870\uac74\uc774 \ub418\uc5b4 \uad50\uc721\uc758 \uc591\uc801\u00b7\uc9c8\uc801 \uc99d\ub300\ub97c \ucd08\ub798\ud558\uc600\ub2e4. \ub610, \uc804\uccb4\uc778\uad6c\uc758 \uc99d\uac00\ub97c \ub4e4 \uc218 \uc788\ub2e4. \uc778\uad6c\uc758 \uc99d\uac00 \uc790\uccb4\ub294 \ucde8\ud559\ub960\uacfc \ubb34\uad00\ud558\uc9c0\ub9cc \uad50\uc721\uc778\uad6c\uc758 \uc808\ub300\uc218\ub97c \uc99d\uac00\uc2dc\ud0a4\ub294 \uc9c1\uc811\uc801\uc778 \uc694\uc778\uc774\uba70 \ub530\ub77c\uc11c \uad50\uc721\uc758 \uc591\uc801 \uc99d\ub300\ub97c \uc720\ubc1c\ud558\ub294 \uc808\ub300\uc801 \uc694\uc778\uc774 \ub41c\ub2e4. \uad50\uc721\uc758 \uc591\uc801\u00b7\uc9c8\uc801 \uc99d\ub300\ub294 \ub2e4\uc18c\uc758 \ubb38\uc81c\ub294 \uc788\uc73c\ub098 \uc9c0\uadf9\ud788 \ubc14\ub78c\uc9c1\ud55c \uac83\uc774\ub2e4. \uadf8\ub7ec\ub098 \ud55c\uad6d\uc758 \uad50\uc721\uc740 \uad50\uc721\uc7ac\uc815\u00b7\uad50\uc721\ud658\uacbd\uba74\uc5d0\uc11c, \uadf8\ub9ac\uace0 \uad6c\uc870\uc801\uc73c\ub85c \ub9ce\uc740 \ubb38\uc81c\ub97c \uc548\uace0 \uc788\ub2e4. \ub2e8\uc801\uc73c\ub85c 3\ub144\uc758 \uc911\ub4f1\uad50\uc721 \uc758\ubb34\uad50\uc721\uc81c\ub3c4\ub97c \ucc44\ud0dd\ud558\uc600\uc73c\uba74\uc11c\ub3c4 \uc77c\ubd80 \uc2e4\uc2dc\ud560 \uc218\ubc16\uc5d0 \uc5c6\ub294 \uae4c\ub2ed\uc774 \uadf8\uac83\uc774\uba70, \uc77c\ubc18\uacc4 \uace0\ub4f1\ud559\uad50\uc758 \uc9c0\ub098\uce5c \ube44\ub300\uc640 \uc9c1\uc5c5\uad50\uc721\uc758 \ubd80\uc7ac\ub85c \uc7ac\uc218\uc0dd\uc774 \uc99d\uac00\ud558\uace0 \ucde8\uc5c5\ub09c\uc744 \ube5a\uace0 \uc788\ub294 \ud604\uc0c1\uc774 \uadf8\uac83\uc774\ub2e4.", "answer": "\uc778\uad6c\uc758 \uc99d\uac00", "sentence": "\ub610, \uc804\uccb4 \uc778\uad6c\uc758 \uc99d\uac00 \ub97c \ub4e4 \uc218 \uc788\ub2e4.", "paragraph_sentence": "\uacbd\uc81c\ubc1c\uc804\uc5d0 \ub530\ub978 \uc0b0\uc5c5\ud654\uc758 \uacfc\uc815\uc5d0\uc11c \uc804\ubb38\uc801\uc778 \uc9c0\uc2dd\u00b7\uae30\ub2a5\u00b7\uae30\uc220\uc744 \uac00\uc9c4 \uc778\ub825\uc5d0 \ub300\ud55c \uc218\uc694\uac00 \uc99d\uac00\ud574 \uc654\ub2e4\ub294 \uac83\uc774\ub2e4. \uc0b0\uc5c5\uad6c\uc870\uc758 \uace0\ub3c4\ud654\ub294 \uc0dd\uc0b0\uc9c1\uc5d0\uc11c\uc870\ucc28 \uc911\ub4f1\uad50\uc721 \uc774\uc0c1\uc758 \uccb4\uacc4\uc801\uc778 \uc9c0\uc2dd\uacfc \uc804\ubb38\uc801\uc778 \uae30\uc220\u00b7\uae30\ub2a5\uc744 \uc694\uad6c\ud558\uac8c \ub418\uc5c8\uace0 \uadf8 \uacb0\uacfc \uc0c1\uae09\uc758 \uad50\uc721\uc740 \uc0ac\ud68c\u00b7\uacbd\uc81c\uc801\uc73c\ub85c \uc720\ub9ac\ud55c \ucde8\uc5c5\uc870\uac74\uc774 \ub418\uc5b4 \uad50\uc721\uc758 \uc591\uc801\u00b7\uc9c8\uc801 \uc99d\ub300\ub97c \ucd08\ub798\ud558\uc600\ub2e4. \ub610, \uc804\uccb4 \uc778\uad6c\uc758 \uc99d\uac00 \ub97c \ub4e4 \uc218 \uc788\ub2e4. \uc778\uad6c\uc758 \uc99d\uac00 \uc790\uccb4\ub294 \ucde8\ud559\ub960\uacfc \ubb34\uad00\ud558\uc9c0\ub9cc \uad50\uc721\uc778\uad6c\uc758 \uc808\ub300\uc218\ub97c \uc99d\uac00\uc2dc\ud0a4\ub294 \uc9c1\uc811\uc801\uc778 \uc694\uc778\uc774\uba70 \ub530\ub77c\uc11c \uad50\uc721\uc758 \uc591\uc801 \uc99d\ub300\ub97c \uc720\ubc1c\ud558\ub294 \uc808\ub300\uc801 \uc694\uc778\uc774 \ub41c\ub2e4. \uad50\uc721\uc758 \uc591\uc801\u00b7\uc9c8\uc801 \uc99d\ub300\ub294 \ub2e4\uc18c\uc758 \ubb38\uc81c\ub294 \uc788\uc73c\ub098 \uc9c0\uadf9\ud788 \ubc14\ub78c\uc9c1\ud55c \uac83\uc774\ub2e4. \uadf8\ub7ec\ub098 \ud55c\uad6d\uc758 \uad50\uc721\uc740 \uad50\uc721\uc7ac\uc815\u00b7\uad50\uc721\ud658\uacbd\uba74\uc5d0\uc11c, \uadf8\ub9ac\uace0 \uad6c\uc870\uc801\uc73c\ub85c \ub9ce\uc740 \ubb38\uc81c\ub97c \uc548\uace0 \uc788\ub2e4. \ub2e8\uc801\uc73c\ub85c 3\ub144\uc758 \uc911\ub4f1\uad50\uc721 \uc758\ubb34\uad50\uc721\uc81c\ub3c4\ub97c \ucc44\ud0dd\ud558\uc600\uc73c\uba74\uc11c\ub3c4 \uc77c\ubd80 \uc2e4\uc2dc\ud560 \uc218\ubc16\uc5d0 \uc5c6\ub294 \uae4c\ub2ed\uc774 \uadf8\uac83\uc774\uba70, \uc77c\ubc18\uacc4 \uace0\ub4f1\ud559\uad50\uc758 \uc9c0\ub098\uce5c \ube44\ub300\uc640 \uc9c1\uc5c5\uad50\uc721\uc758 \ubd80\uc7ac\ub85c \uc7ac\uc218\uc0dd\uc774 \uc99d\uac00\ud558\uace0 \ucde8\uc5c5\ub09c\uc744 \ube5a\uace0 \uc788\ub294 \ud604\uc0c1\uc774 \uadf8\uac83\uc774\ub2e4.", "paragraph_answer": "\uacbd\uc81c\ubc1c\uc804\uc5d0 \ub530\ub978 \uc0b0\uc5c5\ud654\uc758 \uacfc\uc815\uc5d0\uc11c \uc804\ubb38\uc801\uc778 \uc9c0\uc2dd\u00b7\uae30\ub2a5\u00b7\uae30\uc220\uc744 \uac00\uc9c4 \uc778\ub825\uc5d0 \ub300\ud55c \uc218\uc694\uac00 \uc99d\uac00\ud574 \uc654\ub2e4\ub294 \uac83\uc774\ub2e4. \uc0b0\uc5c5\uad6c\uc870\uc758 \uace0\ub3c4\ud654\ub294 \uc0dd\uc0b0\uc9c1\uc5d0\uc11c\uc870\ucc28 \uc911\ub4f1\uad50\uc721 \uc774\uc0c1\uc758 \uccb4\uacc4\uc801\uc778 \uc9c0\uc2dd\uacfc \uc804\ubb38\uc801\uc778 \uae30\uc220\u00b7\uae30\ub2a5\uc744 \uc694\uad6c\ud558\uac8c \ub418\uc5c8\uace0 \uadf8 \uacb0\uacfc \uc0c1\uae09\uc758 \uad50\uc721\uc740 \uc0ac\ud68c\u00b7\uacbd\uc81c\uc801\uc73c\ub85c \uc720\ub9ac\ud55c \ucde8\uc5c5\uc870\uac74\uc774 \ub418\uc5b4 \uad50\uc721\uc758 \uc591\uc801\u00b7\uc9c8\uc801 \uc99d\ub300\ub97c \ucd08\ub798\ud558\uc600\ub2e4. \ub610, \uc804\uccb4 \uc778\uad6c\uc758 \uc99d\uac00 \ub97c \ub4e4 \uc218 \uc788\ub2e4. \uc778\uad6c\uc758 \uc99d\uac00 \uc790\uccb4\ub294 \ucde8\ud559\ub960\uacfc \ubb34\uad00\ud558\uc9c0\ub9cc \uad50\uc721\uc778\uad6c\uc758 \uc808\ub300\uc218\ub97c \uc99d\uac00\uc2dc\ud0a4\ub294 \uc9c1\uc811\uc801\uc778 \uc694\uc778\uc774\uba70 \ub530\ub77c\uc11c \uad50\uc721\uc758 \uc591\uc801 \uc99d\ub300\ub97c \uc720\ubc1c\ud558\ub294 \uc808\ub300\uc801 \uc694\uc778\uc774 \ub41c\ub2e4. \uad50\uc721\uc758 \uc591\uc801\u00b7\uc9c8\uc801 \uc99d\ub300\ub294 \ub2e4\uc18c\uc758 \ubb38\uc81c\ub294 \uc788\uc73c\ub098 \uc9c0\uadf9\ud788 \ubc14\ub78c\uc9c1\ud55c \uac83\uc774\ub2e4. \uadf8\ub7ec\ub098 \ud55c\uad6d\uc758 \uad50\uc721\uc740 \uad50\uc721\uc7ac\uc815\u00b7\uad50\uc721\ud658\uacbd\uba74\uc5d0\uc11c, \uadf8\ub9ac\uace0 \uad6c\uc870\uc801\uc73c\ub85c \ub9ce\uc740 \ubb38\uc81c\ub97c \uc548\uace0 \uc788\ub2e4. \ub2e8\uc801\uc73c\ub85c 3\ub144\uc758 \uc911\ub4f1\uad50\uc721 \uc758\ubb34\uad50\uc721\uc81c\ub3c4\ub97c \ucc44\ud0dd\ud558\uc600\uc73c\uba74\uc11c\ub3c4 \uc77c\ubd80 \uc2e4\uc2dc\ud560 \uc218\ubc16\uc5d0 \uc5c6\ub294 \uae4c\ub2ed\uc774 \uadf8\uac83\uc774\uba70, \uc77c\ubc18\uacc4 \uace0\ub4f1\ud559\uad50\uc758 \uc9c0\ub098\uce5c \ube44\ub300\uc640 \uc9c1\uc5c5\uad50\uc721\uc758 \ubd80\uc7ac\ub85c \uc7ac\uc218\uc0dd\uc774 \uc99d\uac00\ud558\uace0 \ucde8\uc5c5\ub09c\uc744 \ube5a\uace0 \uc788\ub294 \ud604\uc0c1\uc774 \uadf8\uac83\uc774\ub2e4.", "sentence_answer": "\ub610, \uc804\uccb4 \uc778\uad6c\uc758 \uc99d\uac00 \ub97c \ub4e4 \uc218 \uc788\ub2e4.", "paragraph_id": "6528089-2-0", "paragraph_question": "question: \uad50\uc721\uc758 \uc591\uc801 \uc99d\ub300\ub97c \uc720\ubc1c\ud558\ub294 \uc808\ub300\uc801\uc778 \uc694\uc778\uc774 \ub418\ub294\uac83\uc740?, context: \uacbd\uc81c\ubc1c\uc804\uc5d0 \ub530\ub978 \uc0b0\uc5c5\ud654\uc758 \uacfc\uc815\uc5d0\uc11c \uc804\ubb38\uc801\uc778 \uc9c0\uc2dd\u00b7\uae30\ub2a5\u00b7\uae30\uc220\uc744 \uac00\uc9c4 \uc778\ub825\uc5d0 \ub300\ud55c \uc218\uc694\uac00 \uc99d\uac00\ud574 \uc654\ub2e4\ub294 \uac83\uc774\ub2e4. \uc0b0\uc5c5\uad6c\uc870\uc758 \uace0\ub3c4\ud654\ub294 \uc0dd\uc0b0\uc9c1\uc5d0\uc11c\uc870\ucc28 \uc911\ub4f1\uad50\uc721 \uc774\uc0c1\uc758 \uccb4\uacc4\uc801\uc778 \uc9c0\uc2dd\uacfc \uc804\ubb38\uc801\uc778 \uae30\uc220\u00b7\uae30\ub2a5\uc744 \uc694\uad6c\ud558\uac8c \ub418\uc5c8\uace0 \uadf8 \uacb0\uacfc \uc0c1\uae09\uc758 \uad50\uc721\uc740 \uc0ac\ud68c\u00b7\uacbd\uc81c\uc801\uc73c\ub85c \uc720\ub9ac\ud55c \ucde8\uc5c5\uc870\uac74\uc774 \ub418\uc5b4 \uad50\uc721\uc758 \uc591\uc801\u00b7\uc9c8\uc801 \uc99d\ub300\ub97c \ucd08\ub798\ud558\uc600\ub2e4. \ub610, \uc804\uccb4\uc778\uad6c\uc758 \uc99d\uac00\ub97c \ub4e4 \uc218 \uc788\ub2e4. \uc778\uad6c\uc758 \uc99d\uac00 \uc790\uccb4\ub294 \ucde8\ud559\ub960\uacfc \ubb34\uad00\ud558\uc9c0\ub9cc \uad50\uc721\uc778\uad6c\uc758 \uc808\ub300\uc218\ub97c \uc99d\uac00\uc2dc\ud0a4\ub294 \uc9c1\uc811\uc801\uc778 \uc694\uc778\uc774\uba70 \ub530\ub77c\uc11c \uad50\uc721\uc758 \uc591\uc801 \uc99d\ub300\ub97c \uc720\ubc1c\ud558\ub294 \uc808\ub300\uc801 \uc694\uc778\uc774 \ub41c\ub2e4. \uad50\uc721\uc758 \uc591\uc801\u00b7\uc9c8\uc801 \uc99d\ub300\ub294 \ub2e4\uc18c\uc758 \ubb38\uc81c\ub294 \uc788\uc73c\ub098 \uc9c0\uadf9\ud788 \ubc14\ub78c\uc9c1\ud55c \uac83\uc774\ub2e4. \uadf8\ub7ec\ub098 \ud55c\uad6d\uc758 \uad50\uc721\uc740 \uad50\uc721\uc7ac\uc815\u00b7\uad50\uc721\ud658\uacbd\uba74\uc5d0\uc11c, \uadf8\ub9ac\uace0 \uad6c\uc870\uc801\uc73c\ub85c \ub9ce\uc740 \ubb38\uc81c\ub97c \uc548\uace0 \uc788\ub2e4. \ub2e8\uc801\uc73c\ub85c 3\ub144\uc758 \uc911\ub4f1\uad50\uc721 \uc758\ubb34\uad50\uc721\uc81c\ub3c4\ub97c \ucc44\ud0dd\ud558\uc600\uc73c\uba74\uc11c\ub3c4 \uc77c\ubd80 \uc2e4\uc2dc\ud560 \uc218\ubc16\uc5d0 \uc5c6\ub294 \uae4c\ub2ed\uc774 \uadf8\uac83\uc774\uba70, \uc77c\ubc18\uacc4 \uace0\ub4f1\ud559\uad50\uc758 \uc9c0\ub098\uce5c \ube44\ub300\uc640 \uc9c1\uc5c5\uad50\uc721\uc758 \ubd80\uc7ac\ub85c \uc7ac\uc218\uc0dd\uc774 \uc99d\uac00\ud558\uace0 \ucde8\uc5c5\ub09c\uc744 \ube5a\uace0 \uc788\ub294 \ud604\uc0c1\uc774 \uadf8\uac83\uc774\ub2e4."} {"question": "\uace0\uc885\uc774 \uc815\uac15\uc758 \ubcc0 \uc9c1\ud6c4 \ub0a8\ucabd\uc73c\ub85c \ud30c\ucc9c\ud558\uc5ec \uc1a1 \ud669\uc870\ub97c \uc7ac\uac74\ud55c \uc9c0\uc5ed\uc740 \uc5b4\ub514\uc778\uac00?", "paragraph": "\ud718\uc885\uc758 \uc544\ud649 \ubc88\uc9f8 \uc544\ub4e4\uc774\uc790 \ud760\uc885\uc758 \uc774\ubcf5\ub3d9\uc0dd\uc778 \uace0\uc885\uc740 \uc815\uac15\uc758 \ubcc0 \uc9c1\ud6c4 \ub0a8\ucabd\uc73c\ub85c \ud30c\ucc9c\ud558\uc5ec \uc9c0\uae08\uc758 \ub09c\uc9d5\uc778 \uac74\uac15\uc5d0\uc11c \uc1a1 \ud669\uc870\ub97c \uc7ac\uac74\ud558\uc600\ub2e4. \uace0\uc885\uc740 1129\ub144\uc5d0 \uc9c0\uae08\uc758 \ud56d\uc800\uc6b0\uc778 \uc784\uc548(\u81e8\u5b89)\uc5d0 \uc784\uc2dc \ub3c4\uc74d\uc744 \uac74\uc124\ud558\uc600\uace0, 3\ub144 \ub4a4\uc778 1132\ub144\uc5d0\ub294 \uc784\uc548\uc744 \uc81c\uad6d\uc758 \uacf5\uc2dd \uc218\ub3c4\ub85c \ucc9c\uba85\ud588\ub2e4. \ud55c\ud3b8, \ubd81\ucabd\uc758 \uae08\ub098\ub77c\ub294 \ub0a8\uc1a1\uc744 \uc815\ubcf5\ud558\uae30 \uc704\ud574 \uc5ec\ub7ec\ubc88 \uc815\ubc8c\uad70\uc744 \ud3b8\uc131\ud558\uc600\uc73c\ub098 \uc2e4\ud328\ub85c \ub05d\ub0ac\uace0, \uacb0\uad6d 1165\ub144, \ub0a8\uc1a1\uc758 \ud6a8\uc885\uacfc \uae08 \uc138\uc885\uc740 \ud654\uce5c\uc744 \ub9fa\uc73c\uba70 \uace0\uc555\uc801\uc774\uc5c8\ub358 \uc591\uad6d\uac04\uc758 \uad00\uacc4\ub97c \uc644\ud654\uc2dc\ucf30\ub2e4. \ub0a8\uc1a1\uc740 \uc774\uc5b4\uc11c \uc911\uad6d \ub300\ub959 \ub0a8\ubc18\ubd80\ub97c \uc9c0\ubc30\ud558\uc600\uc73c\ub098, 1279\ub144 \uc6d0\ub098\ub77c\uc758 \ud669\uc81c \ucfe0\ube4c\ub77c\uc774 \uce78\uc774 \ub300\ub300\uc801\uc778 \uc815\ubc8c\uad70\uc744 \ud3b8\uc131\ud558\uc5ec \ub0a8\uc1a1\uc744 \uce68\ub7b5, \uace7 \uba78\ub9dd\uc2dc\ucf30\ub2e4. \ub0a8\uc1a1\uc758 \ub9c8\uc9c0\ub9c9 \ud669\uc81c\ub294 \uc870\ubcd1\uc73c\ub85c \uc2dc\ud638\uc778 \uc18c\uc81c(\u5c11\u5e1d) \ub610\ub294 \ud669\uc81c\uc5d0 \uc624\ub974\uae30 \uc804\uc758 \uc791\ud638\uc600\ub358 \uc704\uc655(\u885b\u738b)\uc73c\ub85c \uc54c\ub824\uc838 \uc788\ub2e4. \uc870\ubcd1\uc740 1279\ub144 3\uc6d4 27\uc77c, \uc9c0\uae08\uc758 \uad11\ub465 \uc131 \uc7a5\uba3c \uc2dc \uc57c\uba3c \uc9c4\uc5d0\uc11c \ubc8c\uc5b4\uc9c4 \uc560\uc0b0 \uc804\ud22c\uc5d0\uc11c \ud328\ud558\uc5ec \uc88c\uc2b9\uc0c1 \uc721\uc218\ubd80\uc640 \ud568\uaed8 \ubc14\ub2e4\ub85c \ud22c\uc2e0, \uc775\uc0ac\ud558\uc600\ub2e4.", "answer": "\uac74\uac15", "sentence": "\ud718\uc885\uc758 \uc544\ud649 \ubc88\uc9f8 \uc544\ub4e4\uc774\uc790 \ud760\uc885\uc758 \uc774\ubcf5\ub3d9\uc0dd\uc778 \uace0\uc885\uc740 \uc815\uac15\uc758 \ubcc0 \uc9c1\ud6c4 \ub0a8\ucabd\uc73c\ub85c \ud30c\ucc9c\ud558\uc5ec \uc9c0\uae08\uc758 \ub09c\uc9d5\uc778 \uac74\uac15 \uc5d0\uc11c \uc1a1 \ud669\uc870\ub97c \uc7ac\uac74\ud558\uc600\ub2e4.", "paragraph_sentence": " \ud718\uc885\uc758 \uc544\ud649 \ubc88\uc9f8 \uc544\ub4e4\uc774\uc790 \ud760\uc885\uc758 \uc774\ubcf5\ub3d9\uc0dd\uc778 \uace0\uc885\uc740 \uc815\uac15\uc758 \ubcc0 \uc9c1\ud6c4 \ub0a8\ucabd\uc73c\ub85c \ud30c\ucc9c\ud558\uc5ec \uc9c0\uae08\uc758 \ub09c\uc9d5\uc778 \uac74\uac15 \uc5d0\uc11c \uc1a1 \ud669\uc870\ub97c \uc7ac\uac74\ud558\uc600\ub2e4. \uace0\uc885\uc740 1129\ub144\uc5d0 \uc9c0\uae08\uc758 \ud56d\uc800\uc6b0\uc778 \uc784\uc548(\u81e8\u5b89)\uc5d0 \uc784\uc2dc \ub3c4\uc74d\uc744 \uac74\uc124\ud558\uc600\uace0, 3\ub144 \ub4a4\uc778 1132\ub144\uc5d0\ub294 \uc784\uc548\uc744 \uc81c\uad6d\uc758 \uacf5\uc2dd \uc218\ub3c4\ub85c \ucc9c\uba85\ud588\ub2e4. \ud55c\ud3b8, \ubd81\ucabd\uc758 \uae08\ub098\ub77c\ub294 \ub0a8\uc1a1\uc744 \uc815\ubcf5\ud558\uae30 \uc704\ud574 \uc5ec\ub7ec\ubc88 \uc815\ubc8c\uad70\uc744 \ud3b8\uc131\ud558\uc600\uc73c\ub098 \uc2e4\ud328\ub85c \ub05d\ub0ac\uace0, \uacb0\uad6d 1165\ub144, \ub0a8\uc1a1\uc758 \ud6a8\uc885\uacfc \uae08 \uc138\uc885\uc740 \ud654\uce5c\uc744 \ub9fa\uc73c\uba70 \uace0\uc555\uc801\uc774\uc5c8\ub358 \uc591\uad6d\uac04\uc758 \uad00\uacc4\ub97c \uc644\ud654\uc2dc\ucf30\ub2e4. \ub0a8\uc1a1\uc740 \uc774\uc5b4\uc11c \uc911\uad6d \ub300\ub959 \ub0a8\ubc18\ubd80\ub97c \uc9c0\ubc30\ud558\uc600\uc73c\ub098, 1279\ub144 \uc6d0\ub098\ub77c\uc758 \ud669\uc81c \ucfe0\ube4c\ub77c\uc774 \uce78\uc774 \ub300\ub300\uc801\uc778 \uc815\ubc8c\uad70\uc744 \ud3b8\uc131\ud558\uc5ec \ub0a8\uc1a1\uc744 \uce68\ub7b5, \uace7 \uba78\ub9dd\uc2dc\ucf30\ub2e4. \ub0a8\uc1a1\uc758 \ub9c8\uc9c0\ub9c9 \ud669\uc81c\ub294 \uc870\ubcd1\uc73c\ub85c \uc2dc\ud638\uc778 \uc18c\uc81c(\u5c11\u5e1d) \ub610\ub294 \ud669\uc81c\uc5d0 \uc624\ub974\uae30 \uc804\uc758 \uc791\ud638\uc600\ub358 \uc704\uc655(\u885b\u738b)\uc73c\ub85c \uc54c\ub824\uc838 \uc788\ub2e4. \uc870\ubcd1\uc740 1279\ub144 3\uc6d4 27\uc77c, \uc9c0\uae08\uc758 \uad11\ub465 \uc131 \uc7a5\uba3c \uc2dc \uc57c\uba3c \uc9c4\uc5d0\uc11c \ubc8c\uc5b4\uc9c4 \uc560\uc0b0 \uc804\ud22c\uc5d0\uc11c \ud328\ud558\uc5ec \uc88c\uc2b9\uc0c1 \uc721\uc218\ubd80\uc640 \ud568\uaed8 \ubc14\ub2e4\ub85c \ud22c\uc2e0, \uc775\uc0ac\ud558\uc600\ub2e4.", "paragraph_answer": "\ud718\uc885\uc758 \uc544\ud649 \ubc88\uc9f8 \uc544\ub4e4\uc774\uc790 \ud760\uc885\uc758 \uc774\ubcf5\ub3d9\uc0dd\uc778 \uace0\uc885\uc740 \uc815\uac15\uc758 \ubcc0 \uc9c1\ud6c4 \ub0a8\ucabd\uc73c\ub85c \ud30c\ucc9c\ud558\uc5ec \uc9c0\uae08\uc758 \ub09c\uc9d5\uc778 \uac74\uac15 \uc5d0\uc11c \uc1a1 \ud669\uc870\ub97c \uc7ac\uac74\ud558\uc600\ub2e4. \uace0\uc885\uc740 1129\ub144\uc5d0 \uc9c0\uae08\uc758 \ud56d\uc800\uc6b0\uc778 \uc784\uc548(\u81e8\u5b89)\uc5d0 \uc784\uc2dc \ub3c4\uc74d\uc744 \uac74\uc124\ud558\uc600\uace0, 3\ub144 \ub4a4\uc778 1132\ub144\uc5d0\ub294 \uc784\uc548\uc744 \uc81c\uad6d\uc758 \uacf5\uc2dd \uc218\ub3c4\ub85c \ucc9c\uba85\ud588\ub2e4. \ud55c\ud3b8, \ubd81\ucabd\uc758 \uae08\ub098\ub77c\ub294 \ub0a8\uc1a1\uc744 \uc815\ubcf5\ud558\uae30 \uc704\ud574 \uc5ec\ub7ec\ubc88 \uc815\ubc8c\uad70\uc744 \ud3b8\uc131\ud558\uc600\uc73c\ub098 \uc2e4\ud328\ub85c \ub05d\ub0ac\uace0, \uacb0\uad6d 1165\ub144, \ub0a8\uc1a1\uc758 \ud6a8\uc885\uacfc \uae08 \uc138\uc885\uc740 \ud654\uce5c\uc744 \ub9fa\uc73c\uba70 \uace0\uc555\uc801\uc774\uc5c8\ub358 \uc591\uad6d\uac04\uc758 \uad00\uacc4\ub97c \uc644\ud654\uc2dc\ucf30\ub2e4. \ub0a8\uc1a1\uc740 \uc774\uc5b4\uc11c \uc911\uad6d \ub300\ub959 \ub0a8\ubc18\ubd80\ub97c \uc9c0\ubc30\ud558\uc600\uc73c\ub098, 1279\ub144 \uc6d0\ub098\ub77c\uc758 \ud669\uc81c \ucfe0\ube4c\ub77c\uc774 \uce78\uc774 \ub300\ub300\uc801\uc778 \uc815\ubc8c\uad70\uc744 \ud3b8\uc131\ud558\uc5ec \ub0a8\uc1a1\uc744 \uce68\ub7b5, \uace7 \uba78\ub9dd\uc2dc\ucf30\ub2e4. \ub0a8\uc1a1\uc758 \ub9c8\uc9c0\ub9c9 \ud669\uc81c\ub294 \uc870\ubcd1\uc73c\ub85c \uc2dc\ud638\uc778 \uc18c\uc81c(\u5c11\u5e1d) \ub610\ub294 \ud669\uc81c\uc5d0 \uc624\ub974\uae30 \uc804\uc758 \uc791\ud638\uc600\ub358 \uc704\uc655(\u885b\u738b)\uc73c\ub85c \uc54c\ub824\uc838 \uc788\ub2e4. \uc870\ubcd1\uc740 1279\ub144 3\uc6d4 27\uc77c, \uc9c0\uae08\uc758 \uad11\ub465 \uc131 \uc7a5\uba3c \uc2dc \uc57c\uba3c \uc9c4\uc5d0\uc11c \ubc8c\uc5b4\uc9c4 \uc560\uc0b0 \uc804\ud22c\uc5d0\uc11c \ud328\ud558\uc5ec \uc88c\uc2b9\uc0c1 \uc721\uc218\ubd80\uc640 \ud568\uaed8 \ubc14\ub2e4\ub85c \ud22c\uc2e0, \uc775\uc0ac\ud558\uc600\ub2e4.", "sentence_answer": "\ud718\uc885\uc758 \uc544\ud649 \ubc88\uc9f8 \uc544\ub4e4\uc774\uc790 \ud760\uc885\uc758 \uc774\ubcf5\ub3d9\uc0dd\uc778 \uace0\uc885\uc740 \uc815\uac15\uc758 \ubcc0 \uc9c1\ud6c4 \ub0a8\ucabd\uc73c\ub85c \ud30c\ucc9c\ud558\uc5ec \uc9c0\uae08\uc758 \ub09c\uc9d5\uc778 \uac74\uac15 \uc5d0\uc11c \uc1a1 \ud669\uc870\ub97c \uc7ac\uac74\ud558\uc600\ub2e4.", "paragraph_id": "6518410-1-0", "paragraph_question": "question: \uace0\uc885\uc774 \uc815\uac15\uc758 \ubcc0 \uc9c1\ud6c4 \ub0a8\ucabd\uc73c\ub85c \ud30c\ucc9c\ud558\uc5ec \uc1a1 \ud669\uc870\ub97c \uc7ac\uac74\ud55c \uc9c0\uc5ed\uc740 \uc5b4\ub514\uc778\uac00?, context: \ud718\uc885\uc758 \uc544\ud649 \ubc88\uc9f8 \uc544\ub4e4\uc774\uc790 \ud760\uc885\uc758 \uc774\ubcf5\ub3d9\uc0dd\uc778 \uace0\uc885\uc740 \uc815\uac15\uc758 \ubcc0 \uc9c1\ud6c4 \ub0a8\ucabd\uc73c\ub85c \ud30c\ucc9c\ud558\uc5ec \uc9c0\uae08\uc758 \ub09c\uc9d5\uc778 \uac74\uac15\uc5d0\uc11c \uc1a1 \ud669\uc870\ub97c \uc7ac\uac74\ud558\uc600\ub2e4. \uace0\uc885\uc740 1129\ub144\uc5d0 \uc9c0\uae08\uc758 \ud56d\uc800\uc6b0\uc778 \uc784\uc548(\u81e8\u5b89)\uc5d0 \uc784\uc2dc \ub3c4\uc74d\uc744 \uac74\uc124\ud558\uc600\uace0, 3\ub144 \ub4a4\uc778 1132\ub144\uc5d0\ub294 \uc784\uc548\uc744 \uc81c\uad6d\uc758 \uacf5\uc2dd \uc218\ub3c4\ub85c \ucc9c\uba85\ud588\ub2e4. \ud55c\ud3b8, \ubd81\ucabd\uc758 \uae08\ub098\ub77c\ub294 \ub0a8\uc1a1\uc744 \uc815\ubcf5\ud558\uae30 \uc704\ud574 \uc5ec\ub7ec\ubc88 \uc815\ubc8c\uad70\uc744 \ud3b8\uc131\ud558\uc600\uc73c\ub098 \uc2e4\ud328\ub85c \ub05d\ub0ac\uace0, \uacb0\uad6d 1165\ub144, \ub0a8\uc1a1\uc758 \ud6a8\uc885\uacfc \uae08 \uc138\uc885\uc740 \ud654\uce5c\uc744 \ub9fa\uc73c\uba70 \uace0\uc555\uc801\uc774\uc5c8\ub358 \uc591\uad6d\uac04\uc758 \uad00\uacc4\ub97c \uc644\ud654\uc2dc\ucf30\ub2e4. \ub0a8\uc1a1\uc740 \uc774\uc5b4\uc11c \uc911\uad6d \ub300\ub959 \ub0a8\ubc18\ubd80\ub97c \uc9c0\ubc30\ud558\uc600\uc73c\ub098, 1279\ub144 \uc6d0\ub098\ub77c\uc758 \ud669\uc81c \ucfe0\ube4c\ub77c\uc774 \uce78\uc774 \ub300\ub300\uc801\uc778 \uc815\ubc8c\uad70\uc744 \ud3b8\uc131\ud558\uc5ec \ub0a8\uc1a1\uc744 \uce68\ub7b5, \uace7 \uba78\ub9dd\uc2dc\ucf30\ub2e4. \ub0a8\uc1a1\uc758 \ub9c8\uc9c0\ub9c9 \ud669\uc81c\ub294 \uc870\ubcd1\uc73c\ub85c \uc2dc\ud638\uc778 \uc18c\uc81c(\u5c11\u5e1d) \ub610\ub294 \ud669\uc81c\uc5d0 \uc624\ub974\uae30 \uc804\uc758 \uc791\ud638\uc600\ub358 \uc704\uc655(\u885b\u738b)\uc73c\ub85c \uc54c\ub824\uc838 \uc788\ub2e4. \uc870\ubcd1\uc740 1279\ub144 3\uc6d4 27\uc77c, \uc9c0\uae08\uc758 \uad11\ub465 \uc131 \uc7a5\uba3c \uc2dc \uc57c\uba3c \uc9c4\uc5d0\uc11c \ubc8c\uc5b4\uc9c4 \uc560\uc0b0 \uc804\ud22c\uc5d0\uc11c \ud328\ud558\uc5ec \uc88c\uc2b9\uc0c1 \uc721\uc218\ubd80\uc640 \ud568\uaed8 \ubc14\ub2e4\ub85c \ud22c\uc2e0, \uc775\uc0ac\ud558\uc600\ub2e4."} {"question": "1920\ub144 \uc6d4\ub4dc \uc2dc\ub9ac\uc988\uc5d0\uc11c \ucf54\ubca8\ub808\uc2a4\ud0a4\uac00 \uae30\ub85d\ud55c \ud3c9\uade0\uc790\ucc45\uc810\uc740 \uc5bc\ub9c8\uc778\uac00?", "paragraph": "1920\ub144 \uc2dc\uc98c \ud074\ub9ac\ube14\ub79c\ub4dc \uc778\ub514\uc5b8\uc2a4\ub294 \uc544\uba54\ub9ac\uce78 \ub9ac\uadf8 \uc6b0\uc2b9\uc744 \ucc28\uc9c0\ud588\uace0, \ucf54\ubca8\ub808\uc2a4\ud0a4\ub3c4 \uc774\uc5d0 \ud55c\ubaab\uc744 \ud588\ub2e4. \uc778\ub514\uc5b8\uc2a4\ub294 \uc6d4\ub4dc \uc2dc\ub9ac\uc988\uc5d0 \uc9c4\ucd9c\ud574 \ube0c\ub8e8\ud074\ub9b0 \ub85c\ube48\uc2a4\uc640 \ub9de\ubd99\uc5c8\ub2e4. 1920\ub144 \uc6d4\ub4dc \uc2dc\ub9ac\uc988\uc758 \uc2a4\ud0c0\ub294 \uc138 \uacbd\uae30\uc5d0\uc11c \uc644\ud22c\uc2b9\uc744 \uac70\ub454 \ucf54\ubca8\ub808\uc2a4\ud0a4\uc600\ub2e4. 1\ucc28\uc804\uc5d0\uc11c \ub8e8\ube0c \ub9c8\ucffc\ub4dc\uc640 \uc120\ubc1c \ub9de\ub300\uacb0\uc744 \ubc8c\uc5ec 5\ud53c\uc548\ud0c0 1\uc2e4\uc810\ud558\uba70 \ud300\uc758 3-1 \uc2b9\ub9ac\ub97c \uc774\ub04c\uc5c8\ub2e4. \ub098\ud758 \ub4a4 \uce58\ub7ec\uc9c4 4\ucc28\uc804\uc5d0\uc11c\ub294 5\ud53c\uc548\ud0c0 1\uc2e4\uc810\ud588\uace0 \ud300\uc740 5-1\ub85c \uc2b9\ub9ac\ud588\ub2e4. \uc2dc\ub9ac\uc988 \ub9c8\uc9c0\ub9c9 \uacbd\uae30 7\ucc28\uc804\uc5d0\uc11c \ub2e4\uc2dc \uc120\ubc1c \ub4f1\ud310\ud574 \uc2a4\ud54f\ubcfc\ub7ec \ubc8c\ub9ac \uadf8\ub77c\uc784\uc2a4\uc640 \ub9de\ub300\uacb0\uc744 \ud3bc\ucce4\uace0, 5\ud53c\uc548\ud0c0 \uc644\ubd09\uc2b9\uc744 \uac70\ub450\uba74\uc11c \ud300\uc758 3-0 \uc2b9\ub9ac\ub97c \uc774\ub04c\uc5c8\ub2e4. \ud074\ub9ac\ube14\ub79c\ub4dc \uc778\ub514\uc5b8\uc2a4\ub294 \ud504\ub79c\ucc28\uc774\uc988 \uc5ed\uc0ac\uc0c1 \uccab \uc6d4\ub4dc \uc2dc\ub9ac\uc988 \uc6b0\uc2b9\uc744 \ucc28\uc9c0\ud588\ub2e4. \ucf54\ubca8\ub808\uc2a4\ud0a4\ub294 \uc774\ubc88 \uc2dc\ub9ac\uc988\uc5d0\uc11c 0.67\uc758 \ud3c9\uade0\uc790\ucc45\uc810\uc744 \uae30\ub85d\ud588\ub294\ub370, \uc774\ub294 \uc6d4\ub4dc \uc2dc\ub9ac\uc988 \uae30\ub85d\uc73c\ub85c \ub0a8\uc544\uc788\ub2e4.", "answer": "0.67", "sentence": "\ucf54\ubca8\ub808\uc2a4\ud0a4\ub294 \uc774\ubc88 \uc2dc\ub9ac\uc988\uc5d0\uc11c 0.67 \uc758 \ud3c9\uade0\uc790\ucc45\uc810\uc744 \uae30\ub85d\ud588\ub294\ub370, \uc774\ub294 \uc6d4\ub4dc \uc2dc\ub9ac\uc988 \uae30\ub85d\uc73c\ub85c \ub0a8\uc544\uc788\ub2e4.", "paragraph_sentence": "1920\ub144 \uc2dc\uc98c \ud074\ub9ac\ube14\ub79c\ub4dc \uc778\ub514\uc5b8\uc2a4\ub294 \uc544\uba54\ub9ac\uce78 \ub9ac\uadf8 \uc6b0\uc2b9\uc744 \ucc28\uc9c0\ud588\uace0, \ucf54\ubca8\ub808\uc2a4\ud0a4\ub3c4 \uc774\uc5d0 \ud55c\ubaab\uc744 \ud588\ub2e4. \uc778\ub514\uc5b8\uc2a4\ub294 \uc6d4\ub4dc \uc2dc\ub9ac\uc988\uc5d0 \uc9c4\ucd9c\ud574 \ube0c\ub8e8\ud074\ub9b0 \ub85c\ube48\uc2a4\uc640 \ub9de\ubd99\uc5c8\ub2e4. 1920\ub144 \uc6d4\ub4dc \uc2dc\ub9ac\uc988\uc758 \uc2a4\ud0c0\ub294 \uc138 \uacbd\uae30\uc5d0\uc11c \uc644\ud22c\uc2b9\uc744 \uac70\ub454 \ucf54\ubca8\ub808\uc2a4\ud0a4\uc600\ub2e4. 1\ucc28\uc804\uc5d0\uc11c \ub8e8\ube0c \ub9c8\ucffc\ub4dc\uc640 \uc120\ubc1c \ub9de\ub300\uacb0\uc744 \ubc8c\uc5ec 5\ud53c\uc548\ud0c0 1\uc2e4\uc810\ud558\uba70 \ud300\uc758 3-1 \uc2b9\ub9ac\ub97c \uc774\ub04c\uc5c8\ub2e4. \ub098\ud758 \ub4a4 \uce58\ub7ec\uc9c4 4\ucc28\uc804\uc5d0\uc11c\ub294 5\ud53c\uc548\ud0c0 1\uc2e4\uc810\ud588\uace0 \ud300\uc740 5-1\ub85c \uc2b9\ub9ac\ud588\ub2e4. \uc2dc\ub9ac\uc988 \ub9c8\uc9c0\ub9c9 \uacbd\uae30 7\ucc28\uc804\uc5d0\uc11c \ub2e4\uc2dc \uc120\ubc1c \ub4f1\ud310\ud574 \uc2a4\ud54f\ubcfc\ub7ec \ubc8c\ub9ac \uadf8\ub77c\uc784\uc2a4\uc640 \ub9de\ub300\uacb0\uc744 \ud3bc\ucce4\uace0, 5\ud53c\uc548\ud0c0 \uc644\ubd09\uc2b9\uc744 \uac70\ub450\uba74\uc11c \ud300\uc758 3-0 \uc2b9\ub9ac\ub97c \uc774\ub04c\uc5c8\ub2e4. \ud074\ub9ac\ube14\ub79c\ub4dc \uc778\ub514\uc5b8\uc2a4\ub294 \ud504\ub79c\ucc28\uc774\uc988 \uc5ed\uc0ac\uc0c1 \uccab \uc6d4\ub4dc \uc2dc\ub9ac\uc988 \uc6b0\uc2b9\uc744 \ucc28\uc9c0\ud588\ub2e4. \ucf54\ubca8\ub808\uc2a4\ud0a4\ub294 \uc774\ubc88 \uc2dc\ub9ac\uc988\uc5d0\uc11c 0.67 \uc758 \ud3c9\uade0\uc790\ucc45\uc810\uc744 \uae30\ub85d\ud588\ub294\ub370, \uc774\ub294 \uc6d4\ub4dc \uc2dc\ub9ac\uc988 \uae30\ub85d\uc73c\ub85c \ub0a8\uc544\uc788\ub2e4. ", "paragraph_answer": "1920\ub144 \uc2dc\uc98c \ud074\ub9ac\ube14\ub79c\ub4dc \uc778\ub514\uc5b8\uc2a4\ub294 \uc544\uba54\ub9ac\uce78 \ub9ac\uadf8 \uc6b0\uc2b9\uc744 \ucc28\uc9c0\ud588\uace0, \ucf54\ubca8\ub808\uc2a4\ud0a4\ub3c4 \uc774\uc5d0 \ud55c\ubaab\uc744 \ud588\ub2e4. \uc778\ub514\uc5b8\uc2a4\ub294 \uc6d4\ub4dc \uc2dc\ub9ac\uc988\uc5d0 \uc9c4\ucd9c\ud574 \ube0c\ub8e8\ud074\ub9b0 \ub85c\ube48\uc2a4\uc640 \ub9de\ubd99\uc5c8\ub2e4. 1920\ub144 \uc6d4\ub4dc \uc2dc\ub9ac\uc988\uc758 \uc2a4\ud0c0\ub294 \uc138 \uacbd\uae30\uc5d0\uc11c \uc644\ud22c\uc2b9\uc744 \uac70\ub454 \ucf54\ubca8\ub808\uc2a4\ud0a4\uc600\ub2e4. 1\ucc28\uc804\uc5d0\uc11c \ub8e8\ube0c \ub9c8\ucffc\ub4dc\uc640 \uc120\ubc1c \ub9de\ub300\uacb0\uc744 \ubc8c\uc5ec 5\ud53c\uc548\ud0c0 1\uc2e4\uc810\ud558\uba70 \ud300\uc758 3-1 \uc2b9\ub9ac\ub97c \uc774\ub04c\uc5c8\ub2e4. \ub098\ud758 \ub4a4 \uce58\ub7ec\uc9c4 4\ucc28\uc804\uc5d0\uc11c\ub294 5\ud53c\uc548\ud0c0 1\uc2e4\uc810\ud588\uace0 \ud300\uc740 5-1\ub85c \uc2b9\ub9ac\ud588\ub2e4. \uc2dc\ub9ac\uc988 \ub9c8\uc9c0\ub9c9 \uacbd\uae30 7\ucc28\uc804\uc5d0\uc11c \ub2e4\uc2dc \uc120\ubc1c \ub4f1\ud310\ud574 \uc2a4\ud54f\ubcfc\ub7ec \ubc8c\ub9ac \uadf8\ub77c\uc784\uc2a4\uc640 \ub9de\ub300\uacb0\uc744 \ud3bc\ucce4\uace0, 5\ud53c\uc548\ud0c0 \uc644\ubd09\uc2b9\uc744 \uac70\ub450\uba74\uc11c \ud300\uc758 3-0 \uc2b9\ub9ac\ub97c \uc774\ub04c\uc5c8\ub2e4. \ud074\ub9ac\ube14\ub79c\ub4dc \uc778\ub514\uc5b8\uc2a4\ub294 \ud504\ub79c\ucc28\uc774\uc988 \uc5ed\uc0ac\uc0c1 \uccab \uc6d4\ub4dc \uc2dc\ub9ac\uc988 \uc6b0\uc2b9\uc744 \ucc28\uc9c0\ud588\ub2e4. \ucf54\ubca8\ub808\uc2a4\ud0a4\ub294 \uc774\ubc88 \uc2dc\ub9ac\uc988\uc5d0\uc11c 0.67 \uc758 \ud3c9\uade0\uc790\ucc45\uc810\uc744 \uae30\ub85d\ud588\ub294\ub370, \uc774\ub294 \uc6d4\ub4dc \uc2dc\ub9ac\uc988 \uae30\ub85d\uc73c\ub85c \ub0a8\uc544\uc788\ub2e4.", "sentence_answer": "\ucf54\ubca8\ub808\uc2a4\ud0a4\ub294 \uc774\ubc88 \uc2dc\ub9ac\uc988\uc5d0\uc11c 0.67 \uc758 \ud3c9\uade0\uc790\ucc45\uc810\uc744 \uae30\ub85d\ud588\ub294\ub370, \uc774\ub294 \uc6d4\ub4dc \uc2dc\ub9ac\uc988 \uae30\ub85d\uc73c\ub85c \ub0a8\uc544\uc788\ub2e4.", "paragraph_id": "6584358-7-1", "paragraph_question": "question: 1920\ub144 \uc6d4\ub4dc \uc2dc\ub9ac\uc988\uc5d0\uc11c \ucf54\ubca8\ub808\uc2a4\ud0a4\uac00 \uae30\ub85d\ud55c \ud3c9\uade0\uc790\ucc45\uc810\uc740 \uc5bc\ub9c8\uc778\uac00?, context: 1920\ub144 \uc2dc\uc98c \ud074\ub9ac\ube14\ub79c\ub4dc \uc778\ub514\uc5b8\uc2a4\ub294 \uc544\uba54\ub9ac\uce78 \ub9ac\uadf8 \uc6b0\uc2b9\uc744 \ucc28\uc9c0\ud588\uace0, \ucf54\ubca8\ub808\uc2a4\ud0a4\ub3c4 \uc774\uc5d0 \ud55c\ubaab\uc744 \ud588\ub2e4. \uc778\ub514\uc5b8\uc2a4\ub294 \uc6d4\ub4dc \uc2dc\ub9ac\uc988\uc5d0 \uc9c4\ucd9c\ud574 \ube0c\ub8e8\ud074\ub9b0 \ub85c\ube48\uc2a4\uc640 \ub9de\ubd99\uc5c8\ub2e4. 1920\ub144 \uc6d4\ub4dc \uc2dc\ub9ac\uc988\uc758 \uc2a4\ud0c0\ub294 \uc138 \uacbd\uae30\uc5d0\uc11c \uc644\ud22c\uc2b9\uc744 \uac70\ub454 \ucf54\ubca8\ub808\uc2a4\ud0a4\uc600\ub2e4. 1\ucc28\uc804\uc5d0\uc11c \ub8e8\ube0c \ub9c8\ucffc\ub4dc\uc640 \uc120\ubc1c \ub9de\ub300\uacb0\uc744 \ubc8c\uc5ec 5\ud53c\uc548\ud0c0 1\uc2e4\uc810\ud558\uba70 \ud300\uc758 3-1 \uc2b9\ub9ac\ub97c \uc774\ub04c\uc5c8\ub2e4. \ub098\ud758 \ub4a4 \uce58\ub7ec\uc9c4 4\ucc28\uc804\uc5d0\uc11c\ub294 5\ud53c\uc548\ud0c0 1\uc2e4\uc810\ud588\uace0 \ud300\uc740 5-1\ub85c \uc2b9\ub9ac\ud588\ub2e4. \uc2dc\ub9ac\uc988 \ub9c8\uc9c0\ub9c9 \uacbd\uae30 7\ucc28\uc804\uc5d0\uc11c \ub2e4\uc2dc \uc120\ubc1c \ub4f1\ud310\ud574 \uc2a4\ud54f\ubcfc\ub7ec \ubc8c\ub9ac \uadf8\ub77c\uc784\uc2a4\uc640 \ub9de\ub300\uacb0\uc744 \ud3bc\ucce4\uace0, 5\ud53c\uc548\ud0c0 \uc644\ubd09\uc2b9\uc744 \uac70\ub450\uba74\uc11c \ud300\uc758 3-0 \uc2b9\ub9ac\ub97c \uc774\ub04c\uc5c8\ub2e4. \ud074\ub9ac\ube14\ub79c\ub4dc \uc778\ub514\uc5b8\uc2a4\ub294 \ud504\ub79c\ucc28\uc774\uc988 \uc5ed\uc0ac\uc0c1 \uccab \uc6d4\ub4dc \uc2dc\ub9ac\uc988 \uc6b0\uc2b9\uc744 \ucc28\uc9c0\ud588\ub2e4. \ucf54\ubca8\ub808\uc2a4\ud0a4\ub294 \uc774\ubc88 \uc2dc\ub9ac\uc988\uc5d0\uc11c 0.67\uc758 \ud3c9\uade0\uc790\ucc45\uc810\uc744 \uae30\ub85d\ud588\ub294\ub370, \uc774\ub294 \uc6d4\ub4dc \uc2dc\ub9ac\uc988 \uae30\ub85d\uc73c\ub85c \ub0a8\uc544\uc788\ub2e4."} {"question": "\ud56d\uc131\uc758 \ubcf5\uc0ac\uc555\uc740 \uc5b4\ub5a4 \ubc29\ud5a5\uc73c\ub85c \uc791\uc6a9\ud558\ub294\uac00", "paragraph": "\ud56d\uc131\uc740 \uc218\uc18c \ubc0f \ud5ec\ub968, \uae30\ud0c0 \uc911\uc6d0\uc18c\ub85c \uc774\ub8e8\uc5b4\uc9c4 \uc131\uac04 \uad6c\ub984\uc774 \ubd95\uad34\ud558\uba74\uc11c \ud0c4\uc0dd\ud55c\ub2e4. \uc911\uc2ec\ud575\uc774 \ucda9\ubd84\ud788 \ub728\uac70\uc6cc\uc9c0\uba74 \uc218\uc18c \uc911 \uc77c\ubd80\uac00 \ud575\uc735\ud569 \uc791\uc6a9\uc744 \ud1b5\ud558\uc5ec \ud5ec\ub968\uc73c\ub85c \uc804\ud658\ub418\uae30 \uc2dc\uc791\ud55c\ub2e4. \ub098\uba38\uc9c0 \uc218\uc18c \ubb3c\uc9c8\uc740 \ub300\ub958 \ubc0f \ubcf5\uc0ac \uacfc\uc815\uc744 \ud1b5\ud558\uc5ec \uc911\uc2ec\ud575\uc5d0\uc11c \uc0dd\uc131\ub41c \ubcf5\uc0ac \uc5d0\ub108\uc9c0\ub97c \ubc14\uae65\ucabd\uc73c\ub85c \uc62e\uae34\ub2e4. \ud56d\uc131\uc740 \ub0b4\ubd80\uc5d0\uc11c \ubc14\uae65\ucabd\uc73c\ub85c \uc791\uc6a9\ud558\ub294 \ubcf5\uc0ac\uc555\uacfc \uc790\uccb4 \uc911\ub825\uc774 \uade0\ud615\uc744 \uc774\ub8ec \uc0c1\ud0dc\uc5d0 \uc788\ub2e4. \uc911\uc2ec\ud575\uc5d0 \uc788\ub294 \uc218\uc18c\uac00 \ubaa8\ub450 \uc18c\uc9c4\ub418\uba74 \ud0dc\uc591 \uc9c8\ub7c9\uc758 0.4\ubc30 \uac00 \ub118\ub294 \ud56d\uc131\uc740 \uc801\uc0c9 \uac70\uc131\uc73c\ub85c \uc9c4\ud654\ud558\uba70, \uc774 \ub2e8\uacc4\uc5d0\uc11c \ud56d\uc131\uc740 \uc5ec\ub7ec \uc911\uc6d0\uc18c\ub97c \uc911\uc2ec\ud575 \ub610\ub294 \uc911\uc2ec\ud575 \uc8fc\ubcc0\uc5d0\uc11c \ud0dc\uc6b4\ub2e4. \ud56d\uc131\uc740 \uc0dd\uc758 \ub9c8\uc9c0\ub9c9\uc5d0 \uc790\uc2e0\uc774 \uc9c0\ub2cc \uc9c8\ub7c9\uc744 \uc6b0\uc8fc \uacf5\uac04\uc73c\ub85c \ubc29\ucd9c\ud558\uba70 \ucd95\ud1f4\ub41c\ub2e4. \ubc29\ucd9c\ub41c \ubb3c\uc9c8\uc740 \uc774\uc804\ubcf4\ub2e4 \uc911\uc6d0\uc18c \ud568\ub7c9\uc774 \ub354 \ub9ce\uc73c\uba70, \uc774\ub294 \uc0c8\ub85c\uc6b4 \ubcc4\uc744 \ud0c4\uc0dd\uc2dc\ud0a4\ub294 \uc7ac\ub8cc\ub85c \uc7ac\ud65c\uc6a9\ub41c\ub2e4.", "answer": "\ub0b4\ubd80\uc5d0\uc11c \ubc14\uae65\ucabd\uc73c\ub85c", "sentence": "\ud56d\uc131\uc740 \ub0b4\ubd80\uc5d0\uc11c \ubc14\uae65\ucabd\uc73c\ub85c \uc791\uc6a9\ud558\ub294 \ubcf5\uc0ac\uc555\uacfc \uc790\uccb4 \uc911\ub825\uc774 \uade0\ud615\uc744 \uc774\ub8ec \uc0c1\ud0dc\uc5d0 \uc788\ub2e4.", "paragraph_sentence": "\ud56d\uc131\uc740 \uc218\uc18c \ubc0f \ud5ec\ub968, \uae30\ud0c0 \uc911\uc6d0\uc18c\ub85c \uc774\ub8e8\uc5b4\uc9c4 \uc131\uac04 \uad6c\ub984\uc774 \ubd95\uad34\ud558\uba74\uc11c \ud0c4\uc0dd\ud55c\ub2e4. \uc911\uc2ec\ud575\uc774 \ucda9\ubd84\ud788 \ub728\uac70\uc6cc\uc9c0\uba74 \uc218\uc18c \uc911 \uc77c\ubd80\uac00 \ud575\uc735\ud569 \uc791\uc6a9\uc744 \ud1b5\ud558\uc5ec \ud5ec\ub968\uc73c\ub85c \uc804\ud658\ub418\uae30 \uc2dc\uc791\ud55c\ub2e4. \ub098\uba38\uc9c0 \uc218\uc18c \ubb3c\uc9c8\uc740 \ub300\ub958 \ubc0f \ubcf5\uc0ac \uacfc\uc815\uc744 \ud1b5\ud558\uc5ec \uc911\uc2ec\ud575\uc5d0\uc11c \uc0dd\uc131\ub41c \ubcf5\uc0ac \uc5d0\ub108\uc9c0\ub97c \ubc14\uae65\ucabd\uc73c\ub85c \uc62e\uae34\ub2e4. \ud56d\uc131\uc740 \ub0b4\ubd80\uc5d0\uc11c \ubc14\uae65\ucabd\uc73c\ub85c \uc791\uc6a9\ud558\ub294 \ubcf5\uc0ac\uc555\uacfc \uc790\uccb4 \uc911\ub825\uc774 \uade0\ud615\uc744 \uc774\ub8ec \uc0c1\ud0dc\uc5d0 \uc788\ub2e4. \uc911\uc2ec\ud575\uc5d0 \uc788\ub294 \uc218\uc18c\uac00 \ubaa8\ub450 \uc18c\uc9c4\ub418\uba74 \ud0dc\uc591 \uc9c8\ub7c9\uc758 0.4\ubc30 \uac00 \ub118\ub294 \ud56d\uc131\uc740 \uc801\uc0c9 \uac70\uc131\uc73c\ub85c \uc9c4\ud654\ud558\uba70, \uc774 \ub2e8\uacc4\uc5d0\uc11c \ud56d\uc131\uc740 \uc5ec\ub7ec \uc911\uc6d0\uc18c\ub97c \uc911\uc2ec\ud575 \ub610\ub294 \uc911\uc2ec\ud575 \uc8fc\ubcc0\uc5d0\uc11c \ud0dc\uc6b4\ub2e4. \ud56d\uc131\uc740 \uc0dd\uc758 \ub9c8\uc9c0\ub9c9\uc5d0 \uc790\uc2e0\uc774 \uc9c0\ub2cc \uc9c8\ub7c9\uc744 \uc6b0\uc8fc \uacf5\uac04\uc73c\ub85c \ubc29\ucd9c\ud558\uba70 \ucd95\ud1f4\ub41c\ub2e4. \ubc29\ucd9c\ub41c \ubb3c\uc9c8\uc740 \uc774\uc804\ubcf4\ub2e4 \uc911\uc6d0\uc18c \ud568\ub7c9\uc774 \ub354 \ub9ce\uc73c\uba70, \uc774\ub294 \uc0c8\ub85c\uc6b4 \ubcc4\uc744 \ud0c4\uc0dd\uc2dc\ud0a4\ub294 \uc7ac\ub8cc\ub85c \uc7ac\ud65c\uc6a9\ub41c\ub2e4.", "paragraph_answer": "\ud56d\uc131\uc740 \uc218\uc18c \ubc0f \ud5ec\ub968, \uae30\ud0c0 \uc911\uc6d0\uc18c\ub85c \uc774\ub8e8\uc5b4\uc9c4 \uc131\uac04 \uad6c\ub984\uc774 \ubd95\uad34\ud558\uba74\uc11c \ud0c4\uc0dd\ud55c\ub2e4. \uc911\uc2ec\ud575\uc774 \ucda9\ubd84\ud788 \ub728\uac70\uc6cc\uc9c0\uba74 \uc218\uc18c \uc911 \uc77c\ubd80\uac00 \ud575\uc735\ud569 \uc791\uc6a9\uc744 \ud1b5\ud558\uc5ec \ud5ec\ub968\uc73c\ub85c \uc804\ud658\ub418\uae30 \uc2dc\uc791\ud55c\ub2e4. \ub098\uba38\uc9c0 \uc218\uc18c \ubb3c\uc9c8\uc740 \ub300\ub958 \ubc0f \ubcf5\uc0ac \uacfc\uc815\uc744 \ud1b5\ud558\uc5ec \uc911\uc2ec\ud575\uc5d0\uc11c \uc0dd\uc131\ub41c \ubcf5\uc0ac \uc5d0\ub108\uc9c0\ub97c \ubc14\uae65\ucabd\uc73c\ub85c \uc62e\uae34\ub2e4. \ud56d\uc131\uc740 \ub0b4\ubd80\uc5d0\uc11c \ubc14\uae65\ucabd\uc73c\ub85c \uc791\uc6a9\ud558\ub294 \ubcf5\uc0ac\uc555\uacfc \uc790\uccb4 \uc911\ub825\uc774 \uade0\ud615\uc744 \uc774\ub8ec \uc0c1\ud0dc\uc5d0 \uc788\ub2e4. \uc911\uc2ec\ud575\uc5d0 \uc788\ub294 \uc218\uc18c\uac00 \ubaa8\ub450 \uc18c\uc9c4\ub418\uba74 \ud0dc\uc591 \uc9c8\ub7c9\uc758 0.4\ubc30 \uac00 \ub118\ub294 \ud56d\uc131\uc740 \uc801\uc0c9 \uac70\uc131\uc73c\ub85c \uc9c4\ud654\ud558\uba70, \uc774 \ub2e8\uacc4\uc5d0\uc11c \ud56d\uc131\uc740 \uc5ec\ub7ec \uc911\uc6d0\uc18c\ub97c \uc911\uc2ec\ud575 \ub610\ub294 \uc911\uc2ec\ud575 \uc8fc\ubcc0\uc5d0\uc11c \ud0dc\uc6b4\ub2e4. \ud56d\uc131\uc740 \uc0dd\uc758 \ub9c8\uc9c0\ub9c9\uc5d0 \uc790\uc2e0\uc774 \uc9c0\ub2cc \uc9c8\ub7c9\uc744 \uc6b0\uc8fc \uacf5\uac04\uc73c\ub85c \ubc29\ucd9c\ud558\uba70 \ucd95\ud1f4\ub41c\ub2e4. \ubc29\ucd9c\ub41c \ubb3c\uc9c8\uc740 \uc774\uc804\ubcf4\ub2e4 \uc911\uc6d0\uc18c \ud568\ub7c9\uc774 \ub354 \ub9ce\uc73c\uba70, \uc774\ub294 \uc0c8\ub85c\uc6b4 \ubcc4\uc744 \ud0c4\uc0dd\uc2dc\ud0a4\ub294 \uc7ac\ub8cc\ub85c \uc7ac\ud65c\uc6a9\ub41c\ub2e4.", "sentence_answer": "\ud56d\uc131\uc740 \ub0b4\ubd80\uc5d0\uc11c \ubc14\uae65\ucabd\uc73c\ub85c \uc791\uc6a9\ud558\ub294 \ubcf5\uc0ac\uc555\uacfc \uc790\uccb4 \uc911\ub825\uc774 \uade0\ud615\uc744 \uc774\ub8ec \uc0c1\ud0dc\uc5d0 \uc788\ub2e4.", "paragraph_id": "5940964-0-1", "paragraph_question": "question: \ud56d\uc131\uc758 \ubcf5\uc0ac\uc555\uc740 \uc5b4\ub5a4 \ubc29\ud5a5\uc73c\ub85c \uc791\uc6a9\ud558\ub294\uac00, context: \ud56d\uc131\uc740 \uc218\uc18c \ubc0f \ud5ec\ub968, \uae30\ud0c0 \uc911\uc6d0\uc18c\ub85c \uc774\ub8e8\uc5b4\uc9c4 \uc131\uac04 \uad6c\ub984\uc774 \ubd95\uad34\ud558\uba74\uc11c \ud0c4\uc0dd\ud55c\ub2e4. \uc911\uc2ec\ud575\uc774 \ucda9\ubd84\ud788 \ub728\uac70\uc6cc\uc9c0\uba74 \uc218\uc18c \uc911 \uc77c\ubd80\uac00 \ud575\uc735\ud569 \uc791\uc6a9\uc744 \ud1b5\ud558\uc5ec \ud5ec\ub968\uc73c\ub85c \uc804\ud658\ub418\uae30 \uc2dc\uc791\ud55c\ub2e4. \ub098\uba38\uc9c0 \uc218\uc18c \ubb3c\uc9c8\uc740 \ub300\ub958 \ubc0f \ubcf5\uc0ac \uacfc\uc815\uc744 \ud1b5\ud558\uc5ec \uc911\uc2ec\ud575\uc5d0\uc11c \uc0dd\uc131\ub41c \ubcf5\uc0ac \uc5d0\ub108\uc9c0\ub97c \ubc14\uae65\ucabd\uc73c\ub85c \uc62e\uae34\ub2e4. \ud56d\uc131\uc740 \ub0b4\ubd80\uc5d0\uc11c \ubc14\uae65\ucabd\uc73c\ub85c \uc791\uc6a9\ud558\ub294 \ubcf5\uc0ac\uc555\uacfc \uc790\uccb4 \uc911\ub825\uc774 \uade0\ud615\uc744 \uc774\ub8ec \uc0c1\ud0dc\uc5d0 \uc788\ub2e4. \uc911\uc2ec\ud575\uc5d0 \uc788\ub294 \uc218\uc18c\uac00 \ubaa8\ub450 \uc18c\uc9c4\ub418\uba74 \ud0dc\uc591 \uc9c8\ub7c9\uc758 0.4\ubc30 \uac00 \ub118\ub294 \ud56d\uc131\uc740 \uc801\uc0c9 \uac70\uc131\uc73c\ub85c \uc9c4\ud654\ud558\uba70, \uc774 \ub2e8\uacc4\uc5d0\uc11c \ud56d\uc131\uc740 \uc5ec\ub7ec \uc911\uc6d0\uc18c\ub97c \uc911\uc2ec\ud575 \ub610\ub294 \uc911\uc2ec\ud575 \uc8fc\ubcc0\uc5d0\uc11c \ud0dc\uc6b4\ub2e4. \ud56d\uc131\uc740 \uc0dd\uc758 \ub9c8\uc9c0\ub9c9\uc5d0 \uc790\uc2e0\uc774 \uc9c0\ub2cc \uc9c8\ub7c9\uc744 \uc6b0\uc8fc \uacf5\uac04\uc73c\ub85c \ubc29\ucd9c\ud558\uba70 \ucd95\ud1f4\ub41c\ub2e4. \ubc29\ucd9c\ub41c \ubb3c\uc9c8\uc740 \uc774\uc804\ubcf4\ub2e4 \uc911\uc6d0\uc18c \ud568\ub7c9\uc774 \ub354 \ub9ce\uc73c\uba70, \uc774\ub294 \uc0c8\ub85c\uc6b4 \ubcc4\uc744 \ud0c4\uc0dd\uc2dc\ud0a4\ub294 \uc7ac\ub8cc\ub85c \uc7ac\ud65c\uc6a9\ub41c\ub2e4."} {"question": "2004\ub144 \ud3f4\ub780\ub4dc\uac00 \uac00\uc785\ud55c \ub2e8\uccb4\ub294?", "paragraph": "\uc6d0\ub798 \uc9c0\uae08\uc758 \ud3f4\ub780\ub4dc \ub545\uc5d0\ub294 \uac8c\ub974\ub9cc\uc871\ub4e4\uc774 \uc0b4\uace0 \uc788\uc5c8\ub2e4. 10\uc138\uae30 \uacbd, \ud3f4\ub77c\ub2c8\uc5d0\uc871\uc744 \uc911\uc2ec\uc73c\ub85c \ub098\ub77c\uac00 \ud615\uc131\ub418\uc5b4 \ubbfc\uc871\uacfc \uad6d\ud1a0\uc758 \uba85\uce6d\uc774 \uc0dd\uaca8\ub0ac\ub2e4. 966\ub144 \uac00\ud1a8\ub9ad\uc744 \ubc1b\uc544\ub4e4\uc774\uba70 \ud53c\uc544\uc2a4\ud2b8 \uc655\uc870\uac00 \uc131\ub9bd\ub418\uc5c8\uace0 \uadf8\ub2c8\uc5d0\uc988\ub178\uc640 \ud06c\ub77c\ucfe0\ud504\uac00 \uc815\uce58\uc801 \uc911\uc2ec\uc9c0\ub85c \ubc1c\uc804\ud588\ub2e4. 200\uc5ec\ub144\uc758 \uacf5\uad6d \ubd84\ud560 \uc2dc\ub300\ub97c \uac70\uccd0 \uc911\uc559 \uc9d1\uad8c \uad6d\uac00\ub97c \uc774\ub8e8\uc5c8\ub2e4. 1385\ub144, \ud53c\uc544\uc2a4\ud2b8 \uc655\uc870\uc5d0 \uc774\uc5b4 \ud3f4\ub780\ub4dc-\ub9ac\ud22c\uc544\ub2c8\uc544\uc758 \uc5f0\ud569 \uc655\uc870\uc778 \uc57c\uae30\uc5d0\uc624 \uc655\uc870(14\u223c16\uc138\uae30, \uc218\ub3c4 \ud06c\ub77c\ucfe0\ud504)\uac00 \ud0c4\uc0dd\ud558\uc600\ub2e4. 1410\ub144 \uadf8\ub8ec\ubc1c\ud2b8 \uc804\ud22c\uc5d0\uc11c \ub3c5\uc77c\uad70\uc744 \uaca9\ud30c, \ubc1c\ud2b8\ud574\ub85c \ud1b5\ud558\ub294 \uae38\uc774 \uc5f4\ub838\ub2e4. 16\uc138\uae30\uc5d0\ub294 \uc720\ub7fd\uc758 \uace1\ucc3d \uc9c0\ub300\ub85c \ucd5c\ub300 \uc804\uc131\uae30\ub97c \ub9de\uc558\ub2e4. 1573\ub144 \uc57c\uae30\uc5d0\uc624 \uc655\uc870\uac00 \ub05d\ub098\uace0, \uadc0\uc871\ub4e4\uc774 \uad6d\uc655\uc744 \uc120\ucd9c\ud558\ub294 \uc77c\uc885\uc758 \uadc0\uc871 \uacf5\ud654\uc815\uc774 \ub4f1\uc7a5\ud558\uc600\uace0 1596\ub144\uc5d0 \uc218\ub3c4\ub97c \ud06c\ub77c\ucfe0\ud504\uc5d0\uc11c \ubc14\ub974\uc0e4\ubc14\ub85c \uc774\uc804\ud588\ub2e4. \uadf8\ub7ec\ub098 \uc120\uac70\uc655\uc81c(\u9078\u64e7\u738b\u5236)\uc758 \ucc44\uc6a9\uacfc \ubc14\ub974\uc0e4\ubc14\ub85c \ucc9c\ub3c4\ud55c \ud6c4, \uadc0\uc871\uacc4\uae09\uc758 \uac15\ub300\ud654\uc640 \ud22c\ub974\ud06c, \uc2a4\uc6e8\ub374\uacfc \uc804\uc7c1 \ub4f1\uc73c\ub85c \uad6d\ub825\uc774 \uc1e0\ud1f4\ud558\uc5ec \uad6d\uc6b4\uc774 \uae30\uc6b8\uc5b4\uc84c\ub2e4. 18\uc138\uae30 \ud6c4\ubc18\uc5d0 \uc655\uad8c\uc758 \uac15\ud654\uc640 \uad6d\uac00 \uac1c\ud601\uc774 \uc2dc\ub3c4\ub418\uc5c8\uc73c\ub098 \ud504\ub85c\uc774\uc13c, \ub7ec\uc2dc\uc544, \uc624\uc2a4\ud2b8\ub9ac\uc544 3\uad6d\uc774 \uc810\uc9c4\uc801\uc73c\ub85c \ud3f4\ub780\ub4dc\ub97c \uce68\uc785\ud574 1795\ub144 \ud3f4\ub780\ub4dc\ub97c \ubd84\ud560\ud558\uc5ec, \ub098\ud3f4\ub808\uc639\uc5d0 \uc758\ud55c \ubc14\ub974\uc0e4\ubc14 \uacf5\uad6d \uc2dc\ub300(1807~1815) \uc678\uc5d0\ub294 1795\ub144\ubd80\ud130 1918\ub144\uae4c\uc9c0 3\uad6d\uc758 \uc9c0\ubc30\ub97c \ubc1b\uac8c \ub418\uc5c8\ub2e4. \uadf8\uc758 \ub9de\uc11c 1830\ub144 \ub3c5\ub9bd\uc744 \uc704\ud55c \ud601\uba85 \uc815\ubd80\ub97c \uc870\uc9c1, \ubd09\uae30\ub85c \uc774\uc5b4\uc84c\uc73c\ub098 \ub3c5\ub9bd\ud22c\uc7c1\uc740 \uc2e4\ud328\ud588\ub2e4. \uc774\ud6c4 1\ucc28 \uc138\uacc4 \ub300\uc804 \uc911 \uc6d4\uc2a8 \ub300\ud1b5\ub839\uc774 \uc81c\ucc3d\ud55c \ubbfc\uc871\uc790\uacb0\uc8fc\uc758 \uc6d0\uce59\uc5d0 \ub530\ub77c 1918\ub144 \ub3c5\ub9bd\ud558\uc600\ub2e4. 19\uc138\uae30\uc758 \ub3c5\ub9bd\uc6b4\ub3d9\uc758 \uc804\ud1b5\uacfc \uc81c1\ucc28 \uc138\uacc4 \ub300\uc804\uc744 \uae30\ud68c\ub85c \uad6d\uac00 \uc7ac\uac74\uc744 \uc218\ud589\ud55c \uac83\uc774\ub2e4. \uadf8 \ub4a4 \ubc14\ub974\ubc14\ub2c8\ud30c\ub974\ud1a0 \uc18c\uc7a5\uc774 \uc9c0\ud718\ud558\ub294 \uad70\uc778\uc774 \ucfe0\ub370\ud0c0\ub97c \uc77c\uc73c\ud0a8\ub2e4. 1939\ub144 \ub098\uce58 \ub3c5\uc77c\uacfc \uc18c\ub828\uc758 \uce68\ub7b5\uc744 \ubc1b\uace0 \uc11c\ubd80\uc9c0\uc5ed\uc740 \ub098\uce58 \ub3c5\uc77c\uc5d0, \ub3d9\ubd80\ub294 \uc18c\ub828\uc5d0 \ubd84\ud560 \uc810\ub839\ub418\uc5c8\ub2e4\uac00 1945\ub144 \ud574\ubc29\ub418\uc5c8\ub2e4. \ud574\ubc29 \ud6c4 1947\ub144 \ucd1d\uc120 \uacb0\uacfc \ub178\ub3d9\uc790\ub2f9\uc758 \uc555\uc2b9\uc73c\ub85c \uacf5\uc0b0\ub2f9 \uc815\ubd80\uac00 \uc218\ub9bd\ub418\uc5c8\uace0, \ub0c9\uc804\uc744 \uac70\uce58\uba70 \uc18c\ub828\uc758 \uac15\ud55c \uc601\ud5a5\ub825 \ud558\uc5d0 \uc788\uc5c8\ub2e4. \uadf8\ub7ec\ub098 \uacbd\uc81c \uc2e4\uc815\uacfc \uc9c0\ub3c4\uce35\uc758 \ubd80\ud328\ub85c \ub178\ub3d9\uc790 \ud30c\uc5c5\uc774 \uc77c\uc5b4\ub0ac\uace0, 1981\ub144 \ubc14\uc6ec\uc0ac\uac00 \uc774\ub044\ub294 \uc790\uc720 \ub178\uc870\ub294 \uc804\uad6d\uc801\uc73c\ub85c \ud655\uc0b0\ub418\uc5c8\ub2e4. \uc774\ud6c4 \ubbfc\uc8fc\ud654\ub97c \uc774\ub8e8\uba70 1990\ub144 \ubc14\uc6ec\uc0ac\uac00 \uccab \ubbfc\uc120 \ub300\ud1b5\ub839\uc774 \ub418\uc5c8\ub2e4. 1999\ub144\uc5d0 NATO, 2004\ub144\uc5d0 EU\uc5d0 \uac01\uac01 \uac00\uc785\ud558\uc600\ub2e4.", "answer": "EU", "sentence": "1999\ub144\uc5d0 NATO, 2004\ub144\uc5d0 EU \uc5d0", "paragraph_sentence": "\uc6d0\ub798 \uc9c0\uae08\uc758 \ud3f4\ub780\ub4dc \ub545\uc5d0\ub294 \uac8c\ub974\ub9cc\uc871\ub4e4\uc774 \uc0b4\uace0 \uc788\uc5c8\ub2e4. 10\uc138\uae30 \uacbd, \ud3f4\ub77c\ub2c8\uc5d0\uc871\uc744 \uc911\uc2ec\uc73c\ub85c \ub098\ub77c\uac00 \ud615\uc131\ub418\uc5b4 \ubbfc\uc871\uacfc \uad6d\ud1a0\uc758 \uba85\uce6d\uc774 \uc0dd\uaca8\ub0ac\ub2e4. 966\ub144 \uac00\ud1a8\ub9ad\uc744 \ubc1b\uc544\ub4e4\uc774\uba70 \ud53c\uc544\uc2a4\ud2b8 \uc655\uc870\uac00 \uc131\ub9bd\ub418\uc5c8\uace0 \uadf8\ub2c8\uc5d0\uc988\ub178\uc640 \ud06c\ub77c\ucfe0\ud504\uac00 \uc815\uce58\uc801 \uc911\uc2ec\uc9c0\ub85c \ubc1c\uc804\ud588\ub2e4. 200\uc5ec\ub144\uc758 \uacf5\uad6d \ubd84\ud560 \uc2dc\ub300\ub97c \uac70\uccd0 \uc911\uc559 \uc9d1\uad8c \uad6d\uac00\ub97c \uc774\ub8e8\uc5c8\ub2e4. 1385\ub144, \ud53c\uc544\uc2a4\ud2b8 \uc655\uc870\uc5d0 \uc774\uc5b4 \ud3f4\ub780\ub4dc-\ub9ac\ud22c\uc544\ub2c8\uc544\uc758 \uc5f0\ud569 \uc655\uc870\uc778 \uc57c\uae30\uc5d0\uc624 \uc655\uc870(14\u223c16\uc138\uae30, \uc218\ub3c4 \ud06c\ub77c\ucfe0\ud504)\uac00 \ud0c4\uc0dd\ud558\uc600\ub2e4. 1410\ub144 \uadf8\ub8ec\ubc1c\ud2b8 \uc804\ud22c\uc5d0\uc11c \ub3c5\uc77c\uad70\uc744 \uaca9\ud30c, \ubc1c\ud2b8\ud574\ub85c \ud1b5\ud558\ub294 \uae38\uc774 \uc5f4\ub838\ub2e4. 16\uc138\uae30\uc5d0\ub294 \uc720\ub7fd\uc758 \uace1\ucc3d \uc9c0\ub300\ub85c \ucd5c\ub300 \uc804\uc131\uae30\ub97c \ub9de\uc558\ub2e4. 1573\ub144 \uc57c\uae30\uc5d0\uc624 \uc655\uc870\uac00 \ub05d\ub098\uace0, \uadc0\uc871\ub4e4\uc774 \uad6d\uc655\uc744 \uc120\ucd9c\ud558\ub294 \uc77c\uc885\uc758 \uadc0\uc871 \uacf5\ud654\uc815\uc774 \ub4f1\uc7a5\ud558\uc600\uace0 1596\ub144\uc5d0 \uc218\ub3c4\ub97c \ud06c\ub77c\ucfe0\ud504\uc5d0\uc11c \ubc14\ub974\uc0e4\ubc14\ub85c \uc774\uc804\ud588\ub2e4. \uadf8\ub7ec\ub098 \uc120\uac70\uc655\uc81c(\u9078\u64e7\u738b\u5236)\uc758 \ucc44\uc6a9\uacfc \ubc14\ub974\uc0e4\ubc14\ub85c \ucc9c\ub3c4\ud55c \ud6c4, \uadc0\uc871\uacc4\uae09\uc758 \uac15\ub300\ud654\uc640 \ud22c\ub974\ud06c, \uc2a4\uc6e8\ub374\uacfc \uc804\uc7c1 \ub4f1\uc73c\ub85c \uad6d\ub825\uc774 \uc1e0\ud1f4\ud558\uc5ec \uad6d\uc6b4\uc774 \uae30\uc6b8\uc5b4\uc84c\ub2e4. 18\uc138\uae30 \ud6c4\ubc18\uc5d0 \uc655\uad8c\uc758 \uac15\ud654\uc640 \uad6d\uac00 \uac1c\ud601\uc774 \uc2dc\ub3c4\ub418\uc5c8\uc73c\ub098 \ud504\ub85c\uc774\uc13c, \ub7ec\uc2dc\uc544, \uc624\uc2a4\ud2b8\ub9ac\uc544 3\uad6d\uc774 \uc810\uc9c4\uc801\uc73c\ub85c \ud3f4\ub780\ub4dc\ub97c \uce68\uc785\ud574 1795\ub144 \ud3f4\ub780\ub4dc\ub97c \ubd84\ud560\ud558\uc5ec, \ub098\ud3f4\ub808\uc639\uc5d0 \uc758\ud55c \ubc14\ub974\uc0e4\ubc14 \uacf5\uad6d \uc2dc\ub300(1807~1815) \uc678\uc5d0\ub294 1795\ub144\ubd80\ud130 1918\ub144\uae4c\uc9c0 3\uad6d\uc758 \uc9c0\ubc30\ub97c \ubc1b\uac8c \ub418\uc5c8\ub2e4. \uadf8\uc758 \ub9de\uc11c 1830\ub144 \ub3c5\ub9bd\uc744 \uc704\ud55c \ud601\uba85 \uc815\ubd80\ub97c \uc870\uc9c1, \ubd09\uae30\ub85c \uc774\uc5b4\uc84c\uc73c\ub098 \ub3c5\ub9bd\ud22c\uc7c1\uc740 \uc2e4\ud328\ud588\ub2e4. \uc774\ud6c4 1\ucc28 \uc138\uacc4 \ub300\uc804 \uc911 \uc6d4\uc2a8 \ub300\ud1b5\ub839\uc774 \uc81c\ucc3d\ud55c \ubbfc\uc871\uc790\uacb0\uc8fc\uc758 \uc6d0\uce59\uc5d0 \ub530\ub77c 1918\ub144 \ub3c5\ub9bd\ud558\uc600\ub2e4. 19\uc138\uae30\uc758 \ub3c5\ub9bd\uc6b4\ub3d9\uc758 \uc804\ud1b5\uacfc \uc81c1\ucc28 \uc138\uacc4 \ub300\uc804\uc744 \uae30\ud68c\ub85c \uad6d\uac00 \uc7ac\uac74\uc744 \uc218\ud589\ud55c \uac83\uc774\ub2e4. \uadf8 \ub4a4 \ubc14\ub974\ubc14\ub2c8\ud30c\ub974\ud1a0 \uc18c\uc7a5\uc774 \uc9c0\ud718\ud558\ub294 \uad70\uc778\uc774 \ucfe0\ub370\ud0c0\ub97c \uc77c\uc73c\ud0a8\ub2e4. 1939\ub144 \ub098\uce58 \ub3c5\uc77c\uacfc \uc18c\ub828\uc758 \uce68\ub7b5\uc744 \ubc1b\uace0 \uc11c\ubd80\uc9c0\uc5ed\uc740 \ub098\uce58 \ub3c5\uc77c\uc5d0, \ub3d9\ubd80\ub294 \uc18c\ub828\uc5d0 \ubd84\ud560 \uc810\ub839\ub418\uc5c8\ub2e4\uac00 1945\ub144 \ud574\ubc29\ub418\uc5c8\ub2e4. \ud574\ubc29 \ud6c4 1947\ub144 \ucd1d\uc120 \uacb0\uacfc \ub178\ub3d9\uc790\ub2f9\uc758 \uc555\uc2b9\uc73c\ub85c \uacf5\uc0b0\ub2f9 \uc815\ubd80\uac00 \uc218\ub9bd\ub418\uc5c8\uace0, \ub0c9\uc804\uc744 \uac70\uce58\uba70 \uc18c\ub828\uc758 \uac15\ud55c \uc601\ud5a5\ub825 \ud558\uc5d0 \uc788\uc5c8\ub2e4. \uadf8\ub7ec\ub098 \uacbd\uc81c \uc2e4\uc815\uacfc \uc9c0\ub3c4\uce35\uc758 \ubd80\ud328\ub85c \ub178\ub3d9\uc790 \ud30c\uc5c5\uc774 \uc77c\uc5b4\ub0ac\uace0, 1981\ub144 \ubc14\uc6ec\uc0ac\uac00 \uc774\ub044\ub294 \uc790\uc720 \ub178\uc870\ub294 \uc804\uad6d\uc801\uc73c\ub85c \ud655\uc0b0\ub418\uc5c8\ub2e4. \uc774\ud6c4 \ubbfc\uc8fc\ud654\ub97c \uc774\ub8e8\uba70 1990\ub144 \ubc14\uc6ec\uc0ac\uac00 \uccab \ubbfc\uc120 \ub300\ud1b5\ub839\uc774 \ub418\uc5c8\ub2e4. 1999\ub144\uc5d0 NATO, 2004\ub144\uc5d0 EU \uc5d0 \uac01\uac01 \uac00\uc785\ud558\uc600\ub2e4.", "paragraph_answer": "\uc6d0\ub798 \uc9c0\uae08\uc758 \ud3f4\ub780\ub4dc \ub545\uc5d0\ub294 \uac8c\ub974\ub9cc\uc871\ub4e4\uc774 \uc0b4\uace0 \uc788\uc5c8\ub2e4. 10\uc138\uae30 \uacbd, \ud3f4\ub77c\ub2c8\uc5d0\uc871\uc744 \uc911\uc2ec\uc73c\ub85c \ub098\ub77c\uac00 \ud615\uc131\ub418\uc5b4 \ubbfc\uc871\uacfc \uad6d\ud1a0\uc758 \uba85\uce6d\uc774 \uc0dd\uaca8\ub0ac\ub2e4. 966\ub144 \uac00\ud1a8\ub9ad\uc744 \ubc1b\uc544\ub4e4\uc774\uba70 \ud53c\uc544\uc2a4\ud2b8 \uc655\uc870\uac00 \uc131\ub9bd\ub418\uc5c8\uace0 \uadf8\ub2c8\uc5d0\uc988\ub178\uc640 \ud06c\ub77c\ucfe0\ud504\uac00 \uc815\uce58\uc801 \uc911\uc2ec\uc9c0\ub85c \ubc1c\uc804\ud588\ub2e4. 200\uc5ec\ub144\uc758 \uacf5\uad6d \ubd84\ud560 \uc2dc\ub300\ub97c \uac70\uccd0 \uc911\uc559 \uc9d1\uad8c \uad6d\uac00\ub97c \uc774\ub8e8\uc5c8\ub2e4. 1385\ub144, \ud53c\uc544\uc2a4\ud2b8 \uc655\uc870\uc5d0 \uc774\uc5b4 \ud3f4\ub780\ub4dc-\ub9ac\ud22c\uc544\ub2c8\uc544\uc758 \uc5f0\ud569 \uc655\uc870\uc778 \uc57c\uae30\uc5d0\uc624 \uc655\uc870(14\u223c16\uc138\uae30, \uc218\ub3c4 \ud06c\ub77c\ucfe0\ud504)\uac00 \ud0c4\uc0dd\ud558\uc600\ub2e4. 1410\ub144 \uadf8\ub8ec\ubc1c\ud2b8 \uc804\ud22c\uc5d0\uc11c \ub3c5\uc77c\uad70\uc744 \uaca9\ud30c, \ubc1c\ud2b8\ud574\ub85c \ud1b5\ud558\ub294 \uae38\uc774 \uc5f4\ub838\ub2e4. 16\uc138\uae30\uc5d0\ub294 \uc720\ub7fd\uc758 \uace1\ucc3d \uc9c0\ub300\ub85c \ucd5c\ub300 \uc804\uc131\uae30\ub97c \ub9de\uc558\ub2e4. 1573\ub144 \uc57c\uae30\uc5d0\uc624 \uc655\uc870\uac00 \ub05d\ub098\uace0, \uadc0\uc871\ub4e4\uc774 \uad6d\uc655\uc744 \uc120\ucd9c\ud558\ub294 \uc77c\uc885\uc758 \uadc0\uc871 \uacf5\ud654\uc815\uc774 \ub4f1\uc7a5\ud558\uc600\uace0 1596\ub144\uc5d0 \uc218\ub3c4\ub97c \ud06c\ub77c\ucfe0\ud504\uc5d0\uc11c \ubc14\ub974\uc0e4\ubc14\ub85c \uc774\uc804\ud588\ub2e4. \uadf8\ub7ec\ub098 \uc120\uac70\uc655\uc81c(\u9078\u64e7\u738b\u5236)\uc758 \ucc44\uc6a9\uacfc \ubc14\ub974\uc0e4\ubc14\ub85c \ucc9c\ub3c4\ud55c \ud6c4, \uadc0\uc871\uacc4\uae09\uc758 \uac15\ub300\ud654\uc640 \ud22c\ub974\ud06c, \uc2a4\uc6e8\ub374\uacfc \uc804\uc7c1 \ub4f1\uc73c\ub85c \uad6d\ub825\uc774 \uc1e0\ud1f4\ud558\uc5ec \uad6d\uc6b4\uc774 \uae30\uc6b8\uc5b4\uc84c\ub2e4. 18\uc138\uae30 \ud6c4\ubc18\uc5d0 \uc655\uad8c\uc758 \uac15\ud654\uc640 \uad6d\uac00 \uac1c\ud601\uc774 \uc2dc\ub3c4\ub418\uc5c8\uc73c\ub098 \ud504\ub85c\uc774\uc13c, \ub7ec\uc2dc\uc544, \uc624\uc2a4\ud2b8\ub9ac\uc544 3\uad6d\uc774 \uc810\uc9c4\uc801\uc73c\ub85c \ud3f4\ub780\ub4dc\ub97c \uce68\uc785\ud574 1795\ub144 \ud3f4\ub780\ub4dc\ub97c \ubd84\ud560\ud558\uc5ec, \ub098\ud3f4\ub808\uc639\uc5d0 \uc758\ud55c \ubc14\ub974\uc0e4\ubc14 \uacf5\uad6d \uc2dc\ub300(1807~1815) \uc678\uc5d0\ub294 1795\ub144\ubd80\ud130 1918\ub144\uae4c\uc9c0 3\uad6d\uc758 \uc9c0\ubc30\ub97c \ubc1b\uac8c \ub418\uc5c8\ub2e4. \uadf8\uc758 \ub9de\uc11c 1830\ub144 \ub3c5\ub9bd\uc744 \uc704\ud55c \ud601\uba85 \uc815\ubd80\ub97c \uc870\uc9c1, \ubd09\uae30\ub85c \uc774\uc5b4\uc84c\uc73c\ub098 \ub3c5\ub9bd\ud22c\uc7c1\uc740 \uc2e4\ud328\ud588\ub2e4. \uc774\ud6c4 1\ucc28 \uc138\uacc4 \ub300\uc804 \uc911 \uc6d4\uc2a8 \ub300\ud1b5\ub839\uc774 \uc81c\ucc3d\ud55c \ubbfc\uc871\uc790\uacb0\uc8fc\uc758 \uc6d0\uce59\uc5d0 \ub530\ub77c 1918\ub144 \ub3c5\ub9bd\ud558\uc600\ub2e4. 19\uc138\uae30\uc758 \ub3c5\ub9bd\uc6b4\ub3d9\uc758 \uc804\ud1b5\uacfc \uc81c1\ucc28 \uc138\uacc4 \ub300\uc804\uc744 \uae30\ud68c\ub85c \uad6d\uac00 \uc7ac\uac74\uc744 \uc218\ud589\ud55c \uac83\uc774\ub2e4. \uadf8 \ub4a4 \ubc14\ub974\ubc14\ub2c8\ud30c\ub974\ud1a0 \uc18c\uc7a5\uc774 \uc9c0\ud718\ud558\ub294 \uad70\uc778\uc774 \ucfe0\ub370\ud0c0\ub97c \uc77c\uc73c\ud0a8\ub2e4. 1939\ub144 \ub098\uce58 \ub3c5\uc77c\uacfc \uc18c\ub828\uc758 \uce68\ub7b5\uc744 \ubc1b\uace0 \uc11c\ubd80\uc9c0\uc5ed\uc740 \ub098\uce58 \ub3c5\uc77c\uc5d0, \ub3d9\ubd80\ub294 \uc18c\ub828\uc5d0 \ubd84\ud560 \uc810\ub839\ub418\uc5c8\ub2e4\uac00 1945\ub144 \ud574\ubc29\ub418\uc5c8\ub2e4. \ud574\ubc29 \ud6c4 1947\ub144 \ucd1d\uc120 \uacb0\uacfc \ub178\ub3d9\uc790\ub2f9\uc758 \uc555\uc2b9\uc73c\ub85c \uacf5\uc0b0\ub2f9 \uc815\ubd80\uac00 \uc218\ub9bd\ub418\uc5c8\uace0, \ub0c9\uc804\uc744 \uac70\uce58\uba70 \uc18c\ub828\uc758 \uac15\ud55c \uc601\ud5a5\ub825 \ud558\uc5d0 \uc788\uc5c8\ub2e4. \uadf8\ub7ec\ub098 \uacbd\uc81c \uc2e4\uc815\uacfc \uc9c0\ub3c4\uce35\uc758 \ubd80\ud328\ub85c \ub178\ub3d9\uc790 \ud30c\uc5c5\uc774 \uc77c\uc5b4\ub0ac\uace0, 1981\ub144 \ubc14\uc6ec\uc0ac\uac00 \uc774\ub044\ub294 \uc790\uc720 \ub178\uc870\ub294 \uc804\uad6d\uc801\uc73c\ub85c \ud655\uc0b0\ub418\uc5c8\ub2e4. \uc774\ud6c4 \ubbfc\uc8fc\ud654\ub97c \uc774\ub8e8\uba70 1990\ub144 \ubc14\uc6ec\uc0ac\uac00 \uccab \ubbfc\uc120 \ub300\ud1b5\ub839\uc774 \ub418\uc5c8\ub2e4. 1999\ub144\uc5d0 NATO, 2004\ub144\uc5d0 EU \uc5d0 \uac01\uac01 \uac00\uc785\ud558\uc600\ub2e4.", "sentence_answer": "1999\ub144\uc5d0 NATO, 2004\ub144\uc5d0 EU \uc5d0", "paragraph_id": "6600375-0-2", "paragraph_question": "question: 2004\ub144 \ud3f4\ub780\ub4dc\uac00 \uac00\uc785\ud55c \ub2e8\uccb4\ub294?, context: \uc6d0\ub798 \uc9c0\uae08\uc758 \ud3f4\ub780\ub4dc \ub545\uc5d0\ub294 \uac8c\ub974\ub9cc\uc871\ub4e4\uc774 \uc0b4\uace0 \uc788\uc5c8\ub2e4. 10\uc138\uae30 \uacbd, \ud3f4\ub77c\ub2c8\uc5d0\uc871\uc744 \uc911\uc2ec\uc73c\ub85c \ub098\ub77c\uac00 \ud615\uc131\ub418\uc5b4 \ubbfc\uc871\uacfc \uad6d\ud1a0\uc758 \uba85\uce6d\uc774 \uc0dd\uaca8\ub0ac\ub2e4. 966\ub144 \uac00\ud1a8\ub9ad\uc744 \ubc1b\uc544\ub4e4\uc774\uba70 \ud53c\uc544\uc2a4\ud2b8 \uc655\uc870\uac00 \uc131\ub9bd\ub418\uc5c8\uace0 \uadf8\ub2c8\uc5d0\uc988\ub178\uc640 \ud06c\ub77c\ucfe0\ud504\uac00 \uc815\uce58\uc801 \uc911\uc2ec\uc9c0\ub85c \ubc1c\uc804\ud588\ub2e4. 200\uc5ec\ub144\uc758 \uacf5\uad6d \ubd84\ud560 \uc2dc\ub300\ub97c \uac70\uccd0 \uc911\uc559 \uc9d1\uad8c \uad6d\uac00\ub97c \uc774\ub8e8\uc5c8\ub2e4. 1385\ub144, \ud53c\uc544\uc2a4\ud2b8 \uc655\uc870\uc5d0 \uc774\uc5b4 \ud3f4\ub780\ub4dc-\ub9ac\ud22c\uc544\ub2c8\uc544\uc758 \uc5f0\ud569 \uc655\uc870\uc778 \uc57c\uae30\uc5d0\uc624 \uc655\uc870(14\u223c16\uc138\uae30, \uc218\ub3c4 \ud06c\ub77c\ucfe0\ud504)\uac00 \ud0c4\uc0dd\ud558\uc600\ub2e4. 1410\ub144 \uadf8\ub8ec\ubc1c\ud2b8 \uc804\ud22c\uc5d0\uc11c \ub3c5\uc77c\uad70\uc744 \uaca9\ud30c, \ubc1c\ud2b8\ud574\ub85c \ud1b5\ud558\ub294 \uae38\uc774 \uc5f4\ub838\ub2e4. 16\uc138\uae30\uc5d0\ub294 \uc720\ub7fd\uc758 \uace1\ucc3d \uc9c0\ub300\ub85c \ucd5c\ub300 \uc804\uc131\uae30\ub97c \ub9de\uc558\ub2e4. 1573\ub144 \uc57c\uae30\uc5d0\uc624 \uc655\uc870\uac00 \ub05d\ub098\uace0, \uadc0\uc871\ub4e4\uc774 \uad6d\uc655\uc744 \uc120\ucd9c\ud558\ub294 \uc77c\uc885\uc758 \uadc0\uc871 \uacf5\ud654\uc815\uc774 \ub4f1\uc7a5\ud558\uc600\uace0 1596\ub144\uc5d0 \uc218\ub3c4\ub97c \ud06c\ub77c\ucfe0\ud504\uc5d0\uc11c \ubc14\ub974\uc0e4\ubc14\ub85c \uc774\uc804\ud588\ub2e4. \uadf8\ub7ec\ub098 \uc120\uac70\uc655\uc81c(\u9078\u64e7\u738b\u5236)\uc758 \ucc44\uc6a9\uacfc \ubc14\ub974\uc0e4\ubc14\ub85c \ucc9c\ub3c4\ud55c \ud6c4, \uadc0\uc871\uacc4\uae09\uc758 \uac15\ub300\ud654\uc640 \ud22c\ub974\ud06c, \uc2a4\uc6e8\ub374\uacfc \uc804\uc7c1 \ub4f1\uc73c\ub85c \uad6d\ub825\uc774 \uc1e0\ud1f4\ud558\uc5ec \uad6d\uc6b4\uc774 \uae30\uc6b8\uc5b4\uc84c\ub2e4. 18\uc138\uae30 \ud6c4\ubc18\uc5d0 \uc655\uad8c\uc758 \uac15\ud654\uc640 \uad6d\uac00 \uac1c\ud601\uc774 \uc2dc\ub3c4\ub418\uc5c8\uc73c\ub098 \ud504\ub85c\uc774\uc13c, \ub7ec\uc2dc\uc544, \uc624\uc2a4\ud2b8\ub9ac\uc544 3\uad6d\uc774 \uc810\uc9c4\uc801\uc73c\ub85c \ud3f4\ub780\ub4dc\ub97c \uce68\uc785\ud574 1795\ub144 \ud3f4\ub780\ub4dc\ub97c \ubd84\ud560\ud558\uc5ec, \ub098\ud3f4\ub808\uc639\uc5d0 \uc758\ud55c \ubc14\ub974\uc0e4\ubc14 \uacf5\uad6d \uc2dc\ub300(1807~1815) \uc678\uc5d0\ub294 1795\ub144\ubd80\ud130 1918\ub144\uae4c\uc9c0 3\uad6d\uc758 \uc9c0\ubc30\ub97c \ubc1b\uac8c \ub418\uc5c8\ub2e4. \uadf8\uc758 \ub9de\uc11c 1830\ub144 \ub3c5\ub9bd\uc744 \uc704\ud55c \ud601\uba85 \uc815\ubd80\ub97c \uc870\uc9c1, \ubd09\uae30\ub85c \uc774\uc5b4\uc84c\uc73c\ub098 \ub3c5\ub9bd\ud22c\uc7c1\uc740 \uc2e4\ud328\ud588\ub2e4. \uc774\ud6c4 1\ucc28 \uc138\uacc4 \ub300\uc804 \uc911 \uc6d4\uc2a8 \ub300\ud1b5\ub839\uc774 \uc81c\ucc3d\ud55c \ubbfc\uc871\uc790\uacb0\uc8fc\uc758 \uc6d0\uce59\uc5d0 \ub530\ub77c 1918\ub144 \ub3c5\ub9bd\ud558\uc600\ub2e4. 19\uc138\uae30\uc758 \ub3c5\ub9bd\uc6b4\ub3d9\uc758 \uc804\ud1b5\uacfc \uc81c1\ucc28 \uc138\uacc4 \ub300\uc804\uc744 \uae30\ud68c\ub85c \uad6d\uac00 \uc7ac\uac74\uc744 \uc218\ud589\ud55c \uac83\uc774\ub2e4. \uadf8 \ub4a4 \ubc14\ub974\ubc14\ub2c8\ud30c\ub974\ud1a0 \uc18c\uc7a5\uc774 \uc9c0\ud718\ud558\ub294 \uad70\uc778\uc774 \ucfe0\ub370\ud0c0\ub97c \uc77c\uc73c\ud0a8\ub2e4. 1939\ub144 \ub098\uce58 \ub3c5\uc77c\uacfc \uc18c\ub828\uc758 \uce68\ub7b5\uc744 \ubc1b\uace0 \uc11c\ubd80\uc9c0\uc5ed\uc740 \ub098\uce58 \ub3c5\uc77c\uc5d0, \ub3d9\ubd80\ub294 \uc18c\ub828\uc5d0 \ubd84\ud560 \uc810\ub839\ub418\uc5c8\ub2e4\uac00 1945\ub144 \ud574\ubc29\ub418\uc5c8\ub2e4. \ud574\ubc29 \ud6c4 1947\ub144 \ucd1d\uc120 \uacb0\uacfc \ub178\ub3d9\uc790\ub2f9\uc758 \uc555\uc2b9\uc73c\ub85c \uacf5\uc0b0\ub2f9 \uc815\ubd80\uac00 \uc218\ub9bd\ub418\uc5c8\uace0, \ub0c9\uc804\uc744 \uac70\uce58\uba70 \uc18c\ub828\uc758 \uac15\ud55c \uc601\ud5a5\ub825 \ud558\uc5d0 \uc788\uc5c8\ub2e4. \uadf8\ub7ec\ub098 \uacbd\uc81c \uc2e4\uc815\uacfc \uc9c0\ub3c4\uce35\uc758 \ubd80\ud328\ub85c \ub178\ub3d9\uc790 \ud30c\uc5c5\uc774 \uc77c\uc5b4\ub0ac\uace0, 1981\ub144 \ubc14\uc6ec\uc0ac\uac00 \uc774\ub044\ub294 \uc790\uc720 \ub178\uc870\ub294 \uc804\uad6d\uc801\uc73c\ub85c \ud655\uc0b0\ub418\uc5c8\ub2e4. \uc774\ud6c4 \ubbfc\uc8fc\ud654\ub97c \uc774\ub8e8\uba70 1990\ub144 \ubc14\uc6ec\uc0ac\uac00 \uccab \ubbfc\uc120 \ub300\ud1b5\ub839\uc774 \ub418\uc5c8\ub2e4. 1999\ub144\uc5d0 NATO, 2004\ub144\uc5d0 EU\uc5d0 \uac01\uac01 \uac00\uc785\ud558\uc600\ub2e4."} {"question": "\ud788\ub9d0\ub77c\uc57c\uc5d0\uc11c \ud574\ubc1c 7200m\ub97c \ub118\ub294 \uace0\ubd09\uc740 \uba87\uac1c?", "paragraph": "\ucd5c\ucd08 \ub4f1\ubc18\uc790 \uba54\uc2a4\ub108\ubd80\ud130 \uc608\uc9c0 \ucfe0\ucfe0\uce58\uce74, \uc5d0\ub974\ud750\ub974\ud2b8 \ub85c\ub808\ud0c4\uae4c\uc9c0 14\uc88c\ub97c \uc644\ub4f1\ud55c \ub2e4\ub978 \uc0b0\uc545\uc778\ub4e4\uc740 \uc704\uc131\ubd09\uc778 \uc54c\ub8fd\uce89\uacfc \ub85c\uccb4\uc0e4\ub974\uae4c\uc9c0 \ub9c8\uc800 \ub4f1\uc815\ud574 16\uc88c \uc644\ub4f1\uc744 \uc774\ub8e8\ub824\uace0 \uc2dc\ub3c4\ud55c \uc801\uc774 \uc5c6\ub2e4. \ubb3c\ub860 \uc2a4\ud3f0\uc11c\uac00 \ud544\uc218\uc801\uc774\uace0 \uc2a4\ud3f0\uc11c\ub97c \ubc1b\uae30 \uc704\ud574 \ud64d\ubcf4\uac00 \ud544\uc694\ud558\ubbc0\ub85c \uc5b4\ub290\uc815\ub3c4\uc758 '\uc2a4\ud1a0\ub9ac'\ub97c \ub9cc\ub4dc\ub294 \uac83\uc740 \uc5b4\uca54 \uc218 \uc5c6\ub294 \uc77c\uc774\ub77c \uc0dd\uac01\ud560 \uc218\ub3c4 \uc788\uaca0\uc73c\ub098 \uadf8 \uc21c\uac04 \ud55c\uad6d\uc778 \uac00\uc6b4\ub370 \ucd5c\ucd08\ub85c '14\uc88c'\ub97c \uc644\ub4f1\ud55c \ubc15\uc601\uc11d\uc774\ub098 \ub610 \ub2e4\ub978 \uc644\ub4f1\uc790 \ud55c\uc655\uc6a9\uc740 16\uc88c\ub97c \uc644\ub4f1\ud560 \ub2a5\ub825\uc774 \ubd80\uc871\ud55c \uc0ac\ub78c\uc774 \ub418\uc5b4 \ubc84\ub9ac\ub294 \ubb38\uc81c\uac00 \uc788\uace0 \uc2e4\uc81c \uc774\ub4e4 \uc911\uc5d0\ub294 \"\ub4a4\ud1b5\uc218\ub97c \ub9de\uc558\ub2e4\"\uba70 \uc12d\uc12d\ud568\uc744 \ud1a0\ub85c\ud558\uae30\ub3c4 \ud588\ub2e4. \uc2e4\uc81c \ud574\uc678 \uc0b0\uc545\ub9e4\uccb4\uc5d0\uc11c\ub3c4 8000m \ubd09\uc6b0\ub9ac\ub294 Everest\uc640 K2\ub97c \ube44\ub86f\ud55c 14\uacf3\ub9cc \uaddc\uc815\ud55c\ub2e4. \ud788\ub9d0\ub77c\uc57c\uc5d0\ub294 \ud574\ubc1c 7200m\uac00 \ub118\ub294 \uace0\ubd09 100\uc5ec\uac1c\uac00 \uc19f\uc544 \uc788\uace0 \uadf8\uc911\uc5d0\uc11c\ub3c4 \ucd5c\uace0\ubd09\uc778 \uc5d0\ubca0\ub808\uc2a4\ud2b8(8848m)\ub97c \ud3ec\ud568\ud574 \ub192\uc774 8000m\uac00 \ub118\ub294 14\uac1c\uc758 \ubd09\uc6b0\ub9ac\ub97c \ud1b5\uc0c1 14\uc88c\ub77c\uace0 \uce6d\ud558\uba70 16\uc88c\ub77c\ub294 \ub9d0\uc740 \ud55c\uad6d\uc5d0\uc11c\ub9cc \uc0ac\uc6a9\ub418\ub294 \uac1c\ub150\uc774\ub2e4.", "answer": "\uace0\ubd09 100\uc5ec\uac1c", "sentence": "\ud788\ub9d0\ub77c\uc57c\uc5d0\ub294 \ud574\ubc1c 7200m\uac00 \ub118\ub294 \uace0\ubd09 100\uc5ec\uac1c \uac00 \uc19f\uc544 \uc788\uace0 \uadf8\uc911\uc5d0\uc11c\ub3c4 \ucd5c\uace0\ubd09\uc778 \uc5d0\ubca0\ub808\uc2a4\ud2b8(8848m)\ub97c \ud3ec\ud568\ud574 \ub192\uc774 8000m\uac00 \ub118\ub294 14\uac1c\uc758 \ubd09\uc6b0\ub9ac\ub97c \ud1b5\uc0c1 14\uc88c\ub77c\uace0 \uce6d\ud558\uba70 16\uc88c\ub77c\ub294 \ub9d0\uc740 \ud55c\uad6d\uc5d0\uc11c\ub9cc \uc0ac\uc6a9\ub418\ub294 \uac1c\ub150\uc774\ub2e4.", "paragraph_sentence": "\ucd5c\ucd08 \ub4f1\ubc18\uc790 \uba54\uc2a4\ub108\ubd80\ud130 \uc608\uc9c0 \ucfe0\ucfe0\uce58\uce74, \uc5d0\ub974\ud750\ub974\ud2b8 \ub85c\ub808\ud0c4\uae4c\uc9c0 14\uc88c\ub97c \uc644\ub4f1\ud55c \ub2e4\ub978 \uc0b0\uc545\uc778\ub4e4\uc740 \uc704\uc131\ubd09\uc778 \uc54c\ub8fd\uce89\uacfc \ub85c\uccb4\uc0e4\ub974\uae4c\uc9c0 \ub9c8\uc800 \ub4f1\uc815\ud574 16\uc88c \uc644\ub4f1\uc744 \uc774\ub8e8\ub824\uace0 \uc2dc\ub3c4\ud55c \uc801\uc774 \uc5c6\ub2e4. \ubb3c\ub860 \uc2a4\ud3f0\uc11c\uac00 \ud544\uc218\uc801\uc774\uace0 \uc2a4\ud3f0\uc11c\ub97c \ubc1b\uae30 \uc704\ud574 \ud64d\ubcf4\uac00 \ud544\uc694\ud558\ubbc0\ub85c \uc5b4\ub290\uc815\ub3c4\uc758 '\uc2a4\ud1a0\ub9ac'\ub97c \ub9cc\ub4dc\ub294 \uac83\uc740 \uc5b4\uca54 \uc218 \uc5c6\ub294 \uc77c\uc774\ub77c \uc0dd\uac01\ud560 \uc218\ub3c4 \uc788\uaca0\uc73c\ub098 \uadf8 \uc21c\uac04 \ud55c\uad6d\uc778 \uac00\uc6b4\ub370 \ucd5c\ucd08\ub85c '14\uc88c'\ub97c \uc644\ub4f1\ud55c \ubc15\uc601\uc11d\uc774\ub098 \ub610 \ub2e4\ub978 \uc644\ub4f1\uc790 \ud55c\uc655\uc6a9\uc740 16\uc88c\ub97c \uc644\ub4f1\ud560 \ub2a5\ub825\uc774 \ubd80\uc871\ud55c \uc0ac\ub78c\uc774 \ub418\uc5b4 \ubc84\ub9ac\ub294 \ubb38\uc81c\uac00 \uc788\uace0 \uc2e4\uc81c \uc774\ub4e4 \uc911\uc5d0\ub294 \"\ub4a4\ud1b5\uc218\ub97c \ub9de\uc558\ub2e4\"\uba70 \uc12d\uc12d\ud568\uc744 \ud1a0\ub85c\ud558\uae30\ub3c4 \ud588\ub2e4. \uc2e4\uc81c \ud574\uc678 \uc0b0\uc545\ub9e4\uccb4\uc5d0\uc11c\ub3c4 8000m \ubd09\uc6b0\ub9ac\ub294 Everest\uc640 K2\ub97c \ube44\ub86f\ud55c 14\uacf3\ub9cc \uaddc\uc815\ud55c\ub2e4. \ud788\ub9d0\ub77c\uc57c\uc5d0\ub294 \ud574\ubc1c 7200m\uac00 \ub118\ub294 \uace0\ubd09 100\uc5ec\uac1c \uac00 \uc19f\uc544 \uc788\uace0 \uadf8\uc911\uc5d0\uc11c\ub3c4 \ucd5c\uace0\ubd09\uc778 \uc5d0\ubca0\ub808\uc2a4\ud2b8(8848m)\ub97c \ud3ec\ud568\ud574 \ub192\uc774 8000m\uac00 \ub118\ub294 14\uac1c\uc758 \ubd09\uc6b0\ub9ac\ub97c \ud1b5\uc0c1 14\uc88c\ub77c\uace0 \uce6d\ud558\uba70 16\uc88c\ub77c\ub294 \ub9d0\uc740 \ud55c\uad6d\uc5d0\uc11c\ub9cc \uc0ac\uc6a9\ub418\ub294 \uac1c\ub150\uc774\ub2e4. ", "paragraph_answer": "\ucd5c\ucd08 \ub4f1\ubc18\uc790 \uba54\uc2a4\ub108\ubd80\ud130 \uc608\uc9c0 \ucfe0\ucfe0\uce58\uce74, \uc5d0\ub974\ud750\ub974\ud2b8 \ub85c\ub808\ud0c4\uae4c\uc9c0 14\uc88c\ub97c \uc644\ub4f1\ud55c \ub2e4\ub978 \uc0b0\uc545\uc778\ub4e4\uc740 \uc704\uc131\ubd09\uc778 \uc54c\ub8fd\uce89\uacfc \ub85c\uccb4\uc0e4\ub974\uae4c\uc9c0 \ub9c8\uc800 \ub4f1\uc815\ud574 16\uc88c \uc644\ub4f1\uc744 \uc774\ub8e8\ub824\uace0 \uc2dc\ub3c4\ud55c \uc801\uc774 \uc5c6\ub2e4. \ubb3c\ub860 \uc2a4\ud3f0\uc11c\uac00 \ud544\uc218\uc801\uc774\uace0 \uc2a4\ud3f0\uc11c\ub97c \ubc1b\uae30 \uc704\ud574 \ud64d\ubcf4\uac00 \ud544\uc694\ud558\ubbc0\ub85c \uc5b4\ub290\uc815\ub3c4\uc758 '\uc2a4\ud1a0\ub9ac'\ub97c \ub9cc\ub4dc\ub294 \uac83\uc740 \uc5b4\uca54 \uc218 \uc5c6\ub294 \uc77c\uc774\ub77c \uc0dd\uac01\ud560 \uc218\ub3c4 \uc788\uaca0\uc73c\ub098 \uadf8 \uc21c\uac04 \ud55c\uad6d\uc778 \uac00\uc6b4\ub370 \ucd5c\ucd08\ub85c '14\uc88c'\ub97c \uc644\ub4f1\ud55c \ubc15\uc601\uc11d\uc774\ub098 \ub610 \ub2e4\ub978 \uc644\ub4f1\uc790 \ud55c\uc655\uc6a9\uc740 16\uc88c\ub97c \uc644\ub4f1\ud560 \ub2a5\ub825\uc774 \ubd80\uc871\ud55c \uc0ac\ub78c\uc774 \ub418\uc5b4 \ubc84\ub9ac\ub294 \ubb38\uc81c\uac00 \uc788\uace0 \uc2e4\uc81c \uc774\ub4e4 \uc911\uc5d0\ub294 \"\ub4a4\ud1b5\uc218\ub97c \ub9de\uc558\ub2e4\"\uba70 \uc12d\uc12d\ud568\uc744 \ud1a0\ub85c\ud558\uae30\ub3c4 \ud588\ub2e4. \uc2e4\uc81c \ud574\uc678 \uc0b0\uc545\ub9e4\uccb4\uc5d0\uc11c\ub3c4 8000m \ubd09\uc6b0\ub9ac\ub294 Everest\uc640 K2\ub97c \ube44\ub86f\ud55c 14\uacf3\ub9cc \uaddc\uc815\ud55c\ub2e4. \ud788\ub9d0\ub77c\uc57c\uc5d0\ub294 \ud574\ubc1c 7200m\uac00 \ub118\ub294 \uace0\ubd09 100\uc5ec\uac1c \uac00 \uc19f\uc544 \uc788\uace0 \uadf8\uc911\uc5d0\uc11c\ub3c4 \ucd5c\uace0\ubd09\uc778 \uc5d0\ubca0\ub808\uc2a4\ud2b8(8848m)\ub97c \ud3ec\ud568\ud574 \ub192\uc774 8000m\uac00 \ub118\ub294 14\uac1c\uc758 \ubd09\uc6b0\ub9ac\ub97c \ud1b5\uc0c1 14\uc88c\ub77c\uace0 \uce6d\ud558\uba70 16\uc88c\ub77c\ub294 \ub9d0\uc740 \ud55c\uad6d\uc5d0\uc11c\ub9cc \uc0ac\uc6a9\ub418\ub294 \uac1c\ub150\uc774\ub2e4.", "sentence_answer": "\ud788\ub9d0\ub77c\uc57c\uc5d0\ub294 \ud574\ubc1c 7200m\uac00 \ub118\ub294 \uace0\ubd09 100\uc5ec\uac1c \uac00 \uc19f\uc544 \uc788\uace0 \uadf8\uc911\uc5d0\uc11c\ub3c4 \ucd5c\uace0\ubd09\uc778 \uc5d0\ubca0\ub808\uc2a4\ud2b8(8848m)\ub97c \ud3ec\ud568\ud574 \ub192\uc774 8000m\uac00 \ub118\ub294 14\uac1c\uc758 \ubd09\uc6b0\ub9ac\ub97c \ud1b5\uc0c1 14\uc88c\ub77c\uace0 \uce6d\ud558\uba70 16\uc88c\ub77c\ub294 \ub9d0\uc740 \ud55c\uad6d\uc5d0\uc11c\ub9cc \uc0ac\uc6a9\ub418\ub294 \uac1c\ub150\uc774\ub2e4.", "paragraph_id": "6522972-2-0", "paragraph_question": "question: \ud788\ub9d0\ub77c\uc57c\uc5d0\uc11c \ud574\ubc1c 7200m\ub97c \ub118\ub294 \uace0\ubd09\uc740 \uba87\uac1c?, context: \ucd5c\ucd08 \ub4f1\ubc18\uc790 \uba54\uc2a4\ub108\ubd80\ud130 \uc608\uc9c0 \ucfe0\ucfe0\uce58\uce74, \uc5d0\ub974\ud750\ub974\ud2b8 \ub85c\ub808\ud0c4\uae4c\uc9c0 14\uc88c\ub97c \uc644\ub4f1\ud55c \ub2e4\ub978 \uc0b0\uc545\uc778\ub4e4\uc740 \uc704\uc131\ubd09\uc778 \uc54c\ub8fd\uce89\uacfc \ub85c\uccb4\uc0e4\ub974\uae4c\uc9c0 \ub9c8\uc800 \ub4f1\uc815\ud574 16\uc88c \uc644\ub4f1\uc744 \uc774\ub8e8\ub824\uace0 \uc2dc\ub3c4\ud55c \uc801\uc774 \uc5c6\ub2e4. \ubb3c\ub860 \uc2a4\ud3f0\uc11c\uac00 \ud544\uc218\uc801\uc774\uace0 \uc2a4\ud3f0\uc11c\ub97c \ubc1b\uae30 \uc704\ud574 \ud64d\ubcf4\uac00 \ud544\uc694\ud558\ubbc0\ub85c \uc5b4\ub290\uc815\ub3c4\uc758 '\uc2a4\ud1a0\ub9ac'\ub97c \ub9cc\ub4dc\ub294 \uac83\uc740 \uc5b4\uca54 \uc218 \uc5c6\ub294 \uc77c\uc774\ub77c \uc0dd\uac01\ud560 \uc218\ub3c4 \uc788\uaca0\uc73c\ub098 \uadf8 \uc21c\uac04 \ud55c\uad6d\uc778 \uac00\uc6b4\ub370 \ucd5c\ucd08\ub85c '14\uc88c'\ub97c \uc644\ub4f1\ud55c \ubc15\uc601\uc11d\uc774\ub098 \ub610 \ub2e4\ub978 \uc644\ub4f1\uc790 \ud55c\uc655\uc6a9\uc740 16\uc88c\ub97c \uc644\ub4f1\ud560 \ub2a5\ub825\uc774 \ubd80\uc871\ud55c \uc0ac\ub78c\uc774 \ub418\uc5b4 \ubc84\ub9ac\ub294 \ubb38\uc81c\uac00 \uc788\uace0 \uc2e4\uc81c \uc774\ub4e4 \uc911\uc5d0\ub294 \"\ub4a4\ud1b5\uc218\ub97c \ub9de\uc558\ub2e4\"\uba70 \uc12d\uc12d\ud568\uc744 \ud1a0\ub85c\ud558\uae30\ub3c4 \ud588\ub2e4. \uc2e4\uc81c \ud574\uc678 \uc0b0\uc545\ub9e4\uccb4\uc5d0\uc11c\ub3c4 8000m \ubd09\uc6b0\ub9ac\ub294 Everest\uc640 K2\ub97c \ube44\ub86f\ud55c 14\uacf3\ub9cc \uaddc\uc815\ud55c\ub2e4. \ud788\ub9d0\ub77c\uc57c\uc5d0\ub294 \ud574\ubc1c 7200m\uac00 \ub118\ub294 \uace0\ubd09 100\uc5ec\uac1c\uac00 \uc19f\uc544 \uc788\uace0 \uadf8\uc911\uc5d0\uc11c\ub3c4 \ucd5c\uace0\ubd09\uc778 \uc5d0\ubca0\ub808\uc2a4\ud2b8(8848m)\ub97c \ud3ec\ud568\ud574 \ub192\uc774 8000m\uac00 \ub118\ub294 14\uac1c\uc758 \ubd09\uc6b0\ub9ac\ub97c \ud1b5\uc0c1 14\uc88c\ub77c\uace0 \uce6d\ud558\uba70 16\uc88c\ub77c\ub294 \ub9d0\uc740 \ud55c\uad6d\uc5d0\uc11c\ub9cc \uc0ac\uc6a9\ub418\ub294 \uac1c\ub150\uc774\ub2e4."} {"question": "\ub85c\ub9c8 \uc81c\uc815 \ucd08\uae30\ub294 \uc5b4\ub5a4 \ud615\ud0dc\uc758 \uc815\uce58\uccb4\uc81c\ub97c \ud45c\uba74\uc801\uc73c\ub85c \ub0b4\uc138\uc6e0\ub098?", "paragraph": "\uc81c\uc815 \ucd08\uae30\ub294 \uc815\ubd80\ub294 \uacf5\ud654\uc815\uc758 \ud615\ud0dc\ub97c \uac00\uc7a5\ud558\uc600\ub2e4. \ub85c\ub9c8 \ud669\uc81c\ub294 \uadf8\uc800 \uc81c1\uc2dc\ubbfc(priceps)\uc77c \ubfd0\uc774\uc5c8\uc73c\uba70, \uc6d0\ub85c\uc6d0\uc740 \uacfc\uac70 \ubbfc\ud68c\uac00 \ubcf4\uc720\ud558\ub358 \uc785\ubc95\uad8c\uacfc \ubaa8\ub4e0 \ubc95\uc801 \uad8c\ud55c\uc744 \uac00\uc9c0\uac8c \ub41c\ub2e4. \uadf8\ub7ec\ub098 \uc2dc\uac04\uc774 \uac08\uc218\ub85d \ud669\uc81c\uc758 \uad8c\ub825\uc740 \uc810\ucc28 \uc804\uc81c \uad8c\ub825\uc73c\ub85c \ubc1c\uc804\ud588\uc73c\uba70, \uc6d0\ub85c\uc6d0\uc740 \ud669\uc81c\uac00 \uc784\uba85\ud558\ub294 \uc790\ubb38 \uae30\uad6c\ub85c \uc804\ub77d\ud558\uc600\ub2e4. \ub85c\ub9c8 \uc81c\uad6d\uc740 \uacf5\ud654\uc815 \uc2dc\ub300\uc758 \ud589\uc815 \uccb4\uc81c\ub97c \ubb3c\ub824\ubc1b\uc9c0 \uc54a\uc558\ub294\ub370, \uacf5\ud654\uc815\uc5d0\ub294 \uc6d0\ub85c\uc6d0 \uc678\uc5d0 \uc601\uc18d\uc801\uc778 \ud589\uc815 \uae30\uad6c\uac00 \uc5c6\uc5c8\uae30 \ub54c\ubb38\uc774\uc5c8\ub2e4. \uc544\uc6b0\uad6c\uc2a4\ud22c\uc2a4\ub294 \ucd5c\uace0\uc704 \ud589\uc815\uac00\uc640 \uc6d0\ub85c\uc6d0 \uc758\uc6d0, \uc790\uc2e0\uc758 \uce5c\uad6c, \uc804\ubb38 \ubc95\ub960\uac00 \ub4f1\uc744 \ubaa8\uc544 \uc870\uc5b8\uc744 \uad6c\ud558\uc600\uc73c\uba70, \uc6d0\ub85c\uc6d0 \uc758\uc6d0, \uae30\uc0ac, \ud53c\ud574\ubc29\uc778, \uc2ec\uc9c0\uc5b4 \ub178\uc608\uae4c\uc9c0 \uac01\uacc4 \uac01\uce35\uc758 \uc0ac\ub78c\ub4e4\uc744 \uacf5\ubb34\uc6d0\uc73c\ub85c \uae30\uc6a9\ud558\uc5ec \uace1\ubb3c, \uc218\ub3c4, \uce58\uc548, \ubc95, \uc7ac\uc815 \ub4f1 \uc77c\uc885\uc758 \ud589\uc815 \uc870\uc9c1\uc744 \uad6c\uc131\ud558\uc600\ub2e4.", "answer": "\uacf5\ud654\uc815", "sentence": "\uc81c\uc815 \ucd08\uae30\ub294 \uc815\ubd80\ub294 \uacf5\ud654\uc815 \uc758 \ud615\ud0dc\ub97c \uac00\uc7a5\ud558\uc600\ub2e4.", "paragraph_sentence": " \uc81c\uc815 \ucd08\uae30\ub294 \uc815\ubd80\ub294 \uacf5\ud654\uc815 \uc758 \ud615\ud0dc\ub97c \uac00\uc7a5\ud558\uc600\ub2e4. \ub85c\ub9c8 \ud669\uc81c\ub294 \uadf8\uc800 \uc81c1\uc2dc\ubbfc(priceps)\uc77c \ubfd0\uc774\uc5c8\uc73c\uba70, \uc6d0\ub85c\uc6d0\uc740 \uacfc\uac70 \ubbfc\ud68c\uac00 \ubcf4\uc720\ud558\ub358 \uc785\ubc95\uad8c\uacfc \ubaa8\ub4e0 \ubc95\uc801 \uad8c\ud55c\uc744 \uac00\uc9c0\uac8c \ub41c\ub2e4. \uadf8\ub7ec\ub098 \uc2dc\uac04\uc774 \uac08\uc218\ub85d \ud669\uc81c\uc758 \uad8c\ub825\uc740 \uc810\ucc28 \uc804\uc81c \uad8c\ub825\uc73c\ub85c \ubc1c\uc804\ud588\uc73c\uba70, \uc6d0\ub85c\uc6d0\uc740 \ud669\uc81c\uac00 \uc784\uba85\ud558\ub294 \uc790\ubb38 \uae30\uad6c\ub85c \uc804\ub77d\ud558\uc600\ub2e4. \ub85c\ub9c8 \uc81c\uad6d\uc740 \uacf5\ud654\uc815 \uc2dc\ub300\uc758 \ud589\uc815 \uccb4\uc81c\ub97c \ubb3c\ub824\ubc1b\uc9c0 \uc54a\uc558\ub294\ub370, \uacf5\ud654\uc815\uc5d0\ub294 \uc6d0\ub85c\uc6d0 \uc678\uc5d0 \uc601\uc18d\uc801\uc778 \ud589\uc815 \uae30\uad6c\uac00 \uc5c6\uc5c8\uae30 \ub54c\ubb38\uc774\uc5c8\ub2e4. \uc544\uc6b0\uad6c\uc2a4\ud22c\uc2a4\ub294 \ucd5c\uace0\uc704 \ud589\uc815\uac00\uc640 \uc6d0\ub85c\uc6d0 \uc758\uc6d0, \uc790\uc2e0\uc758 \uce5c\uad6c, \uc804\ubb38 \ubc95\ub960\uac00 \ub4f1\uc744 \ubaa8\uc544 \uc870\uc5b8\uc744 \uad6c\ud558\uc600\uc73c\uba70, \uc6d0\ub85c\uc6d0 \uc758\uc6d0, \uae30\uc0ac, \ud53c\ud574\ubc29\uc778, \uc2ec\uc9c0\uc5b4 \ub178\uc608\uae4c\uc9c0 \uac01\uacc4 \uac01\uce35\uc758 \uc0ac\ub78c\ub4e4\uc744 \uacf5\ubb34\uc6d0\uc73c\ub85c \uae30\uc6a9\ud558\uc5ec \uace1\ubb3c, \uc218\ub3c4, \uce58\uc548, \ubc95, \uc7ac\uc815 \ub4f1 \uc77c\uc885\uc758 \ud589\uc815 \uc870\uc9c1\uc744 \uad6c\uc131\ud558\uc600\ub2e4.", "paragraph_answer": "\uc81c\uc815 \ucd08\uae30\ub294 \uc815\ubd80\ub294 \uacf5\ud654\uc815 \uc758 \ud615\ud0dc\ub97c \uac00\uc7a5\ud558\uc600\ub2e4. \ub85c\ub9c8 \ud669\uc81c\ub294 \uadf8\uc800 \uc81c1\uc2dc\ubbfc(priceps)\uc77c \ubfd0\uc774\uc5c8\uc73c\uba70, \uc6d0\ub85c\uc6d0\uc740 \uacfc\uac70 \ubbfc\ud68c\uac00 \ubcf4\uc720\ud558\ub358 \uc785\ubc95\uad8c\uacfc \ubaa8\ub4e0 \ubc95\uc801 \uad8c\ud55c\uc744 \uac00\uc9c0\uac8c \ub41c\ub2e4. \uadf8\ub7ec\ub098 \uc2dc\uac04\uc774 \uac08\uc218\ub85d \ud669\uc81c\uc758 \uad8c\ub825\uc740 \uc810\ucc28 \uc804\uc81c \uad8c\ub825\uc73c\ub85c \ubc1c\uc804\ud588\uc73c\uba70, \uc6d0\ub85c\uc6d0\uc740 \ud669\uc81c\uac00 \uc784\uba85\ud558\ub294 \uc790\ubb38 \uae30\uad6c\ub85c \uc804\ub77d\ud558\uc600\ub2e4. \ub85c\ub9c8 \uc81c\uad6d\uc740 \uacf5\ud654\uc815 \uc2dc\ub300\uc758 \ud589\uc815 \uccb4\uc81c\ub97c \ubb3c\ub824\ubc1b\uc9c0 \uc54a\uc558\ub294\ub370, \uacf5\ud654\uc815\uc5d0\ub294 \uc6d0\ub85c\uc6d0 \uc678\uc5d0 \uc601\uc18d\uc801\uc778 \ud589\uc815 \uae30\uad6c\uac00 \uc5c6\uc5c8\uae30 \ub54c\ubb38\uc774\uc5c8\ub2e4. \uc544\uc6b0\uad6c\uc2a4\ud22c\uc2a4\ub294 \ucd5c\uace0\uc704 \ud589\uc815\uac00\uc640 \uc6d0\ub85c\uc6d0 \uc758\uc6d0, \uc790\uc2e0\uc758 \uce5c\uad6c, \uc804\ubb38 \ubc95\ub960\uac00 \ub4f1\uc744 \ubaa8\uc544 \uc870\uc5b8\uc744 \uad6c\ud558\uc600\uc73c\uba70, \uc6d0\ub85c\uc6d0 \uc758\uc6d0, \uae30\uc0ac, \ud53c\ud574\ubc29\uc778, \uc2ec\uc9c0\uc5b4 \ub178\uc608\uae4c\uc9c0 \uac01\uacc4 \uac01\uce35\uc758 \uc0ac\ub78c\ub4e4\uc744 \uacf5\ubb34\uc6d0\uc73c\ub85c \uae30\uc6a9\ud558\uc5ec \uace1\ubb3c, \uc218\ub3c4, \uce58\uc548, \ubc95, \uc7ac\uc815 \ub4f1 \uc77c\uc885\uc758 \ud589\uc815 \uc870\uc9c1\uc744 \uad6c\uc131\ud558\uc600\ub2e4.", "sentence_answer": "\uc81c\uc815 \ucd08\uae30\ub294 \uc815\ubd80\ub294 \uacf5\ud654\uc815 \uc758 \ud615\ud0dc\ub97c \uac00\uc7a5\ud558\uc600\ub2e4.", "paragraph_id": "6466082-14-1", "paragraph_question": "question: \ub85c\ub9c8 \uc81c\uc815 \ucd08\uae30\ub294 \uc5b4\ub5a4 \ud615\ud0dc\uc758 \uc815\uce58\uccb4\uc81c\ub97c \ud45c\uba74\uc801\uc73c\ub85c \ub0b4\uc138\uc6e0\ub098?, context: \uc81c\uc815 \ucd08\uae30\ub294 \uc815\ubd80\ub294 \uacf5\ud654\uc815\uc758 \ud615\ud0dc\ub97c \uac00\uc7a5\ud558\uc600\ub2e4. \ub85c\ub9c8 \ud669\uc81c\ub294 \uadf8\uc800 \uc81c1\uc2dc\ubbfc(priceps)\uc77c \ubfd0\uc774\uc5c8\uc73c\uba70, \uc6d0\ub85c\uc6d0\uc740 \uacfc\uac70 \ubbfc\ud68c\uac00 \ubcf4\uc720\ud558\ub358 \uc785\ubc95\uad8c\uacfc \ubaa8\ub4e0 \ubc95\uc801 \uad8c\ud55c\uc744 \uac00\uc9c0\uac8c \ub41c\ub2e4. \uadf8\ub7ec\ub098 \uc2dc\uac04\uc774 \uac08\uc218\ub85d \ud669\uc81c\uc758 \uad8c\ub825\uc740 \uc810\ucc28 \uc804\uc81c \uad8c\ub825\uc73c\ub85c \ubc1c\uc804\ud588\uc73c\uba70, \uc6d0\ub85c\uc6d0\uc740 \ud669\uc81c\uac00 \uc784\uba85\ud558\ub294 \uc790\ubb38 \uae30\uad6c\ub85c \uc804\ub77d\ud558\uc600\ub2e4. \ub85c\ub9c8 \uc81c\uad6d\uc740 \uacf5\ud654\uc815 \uc2dc\ub300\uc758 \ud589\uc815 \uccb4\uc81c\ub97c \ubb3c\ub824\ubc1b\uc9c0 \uc54a\uc558\ub294\ub370, \uacf5\ud654\uc815\uc5d0\ub294 \uc6d0\ub85c\uc6d0 \uc678\uc5d0 \uc601\uc18d\uc801\uc778 \ud589\uc815 \uae30\uad6c\uac00 \uc5c6\uc5c8\uae30 \ub54c\ubb38\uc774\uc5c8\ub2e4. \uc544\uc6b0\uad6c\uc2a4\ud22c\uc2a4\ub294 \ucd5c\uace0\uc704 \ud589\uc815\uac00\uc640 \uc6d0\ub85c\uc6d0 \uc758\uc6d0, \uc790\uc2e0\uc758 \uce5c\uad6c, \uc804\ubb38 \ubc95\ub960\uac00 \ub4f1\uc744 \ubaa8\uc544 \uc870\uc5b8\uc744 \uad6c\ud558\uc600\uc73c\uba70, \uc6d0\ub85c\uc6d0 \uc758\uc6d0, \uae30\uc0ac, \ud53c\ud574\ubc29\uc778, \uc2ec\uc9c0\uc5b4 \ub178\uc608\uae4c\uc9c0 \uac01\uacc4 \uac01\uce35\uc758 \uc0ac\ub78c\ub4e4\uc744 \uacf5\ubb34\uc6d0\uc73c\ub85c \uae30\uc6a9\ud558\uc5ec \uace1\ubb3c, \uc218\ub3c4, \uce58\uc548, \ubc95, \uc7ac\uc815 \ub4f1 \uc77c\uc885\uc758 \ud589\uc815 \uc870\uc9c1\uc744 \uad6c\uc131\ud558\uc600\ub2e4."} {"question": "1882\ub144 3\uc6d4 1\uc77c\ubd80\ud130 \uc77c\ubcf8\uc5d0 \uc9c0\uc9c0\uc2e0\ubcf4 \uc77c\uac04\uc9c0\ub97c \ucc3d\uac04\ud55c \uc778\ubb3c\uc740?", "paragraph": "\ud55c\ud3b8 \ubbf8\uad6d\uacfc \uc218\uad50\ub97c \uccb4\uacb0\ud55c \uc870\uc120\uc740 1882\ub144(\uace0\uc885 19) \uc8fc\uc870\uc120 \ubbf8\uad6d \uacf5\uc0ac \ub8e8\uc2dc\uc5b4\uc2a4 H. \ud478\ud2b8(L. H. Foote)\uac00 \ubc29\ud55c\ud558\uc790 \uc870\uc120\ub3c4 \ubbf8\uad6d\uc5d0 \uce5c\uc120\uc0ac\uc808\ub2e8(\ubcf4\ube59\uc0ac)\uc744 \ud30c\uacac\ud558\uc600\uace0, \uc720\uae38\uc900\uc740 \ud6c4\ucfe0\uc790\uc640 \uc720\ud0a4\uce58\uc5d0\uac8c \ubc31\uc778\uc744 \uc0c1\ub300\ud558\ub294 \ubc29\ubc95\uc744 \ubb38\uc758\ud558\uc600\ub2e4. \uc774 \ubb34\ub835 \ud6c4\ucfe0\uc790\uc640\ub294 \ub9ce\uc740 \uc800\uc791\ubb3c\uc744 \ubc1c\ud45c\ud568\uacfc \ub3d9\uc2dc\uc5d0 1882\ub144 3\uc6d4 1\uc77c\ubd80\ud130 \uc77c\ubcf8\uc5d0 \uc9c0\uc9c0\uc2e0\ubcf4(\u6642\u4e8b\u65b0\u5831)\ub77c\ub294 \uc77c\uac04\uc9c0\ub97c \ucc3d\uac04\ud588\ub2e4. \uc77c\ubcf8\uc758 \ud6c4\ucfe0\uc790\uc640\ub85c\ubd80\ud130 \ud3b8\uc9c0\ub97c \uc804\ub2ec\ubc1b\uace0 \uc6d0\uace0 \uc694\uccad\uc774 \uc788\uc790 \uadf8\ub294 \uac10\uaca9, \uc77c\ubcf8 \uc0ac\ud68c\uc5d0\uc11c \uc2e0\ubb38\uc774 \uc5bc\ub9c8\ub098 \ud070 \uc5ed\ud560\uc744 \ud558\uace0 \uc788\ub294\uac00\uc5d0 \ub300\ud55c \ub0b4\uc6a9\uc744 \uadf8\ud574 4\uc6d4 21\uc77c '\uc2e0\ubb38\uc758 \uae30\ub825\uc744 \ub17c \ud568'\uc774\ub77c\ub294 \uc81c\ubaa9\uc73c\ub85c \ubcf4\ub0c8\uace0, \ud6c4\ucfe0\uc790\uc640 \uc720\ud0a4\uce58\ub294 \uc774\ub97c \uba85\ubb38\uc774\ub77c\uba70 \uc790\uc2e0\uc758 \uc9c0\uc9c0\uc2e0\ubcf4\uc5d0 \uac8c\uc7ac\ud588\ub2e4.", "answer": "\ud6c4\ucfe0\uc790\uc640", "sentence": "\uc720\uae38\uc900\uc740 \ud6c4\ucfe0\uc790\uc640 \uc720\ud0a4\uce58\uc5d0\uac8c \ubc31\uc778\uc744 \uc0c1\ub300\ud558\ub294 \ubc29\ubc95\uc744 \ubb38\uc758\ud558\uc600\ub2e4.", "paragraph_sentence": "\ud55c\ud3b8 \ubbf8\uad6d\uacfc \uc218\uad50\ub97c \uccb4\uacb0\ud55c \uc870\uc120\uc740 1882\ub144(\uace0\uc885 19) \uc8fc\uc870\uc120 \ubbf8\uad6d \uacf5\uc0ac \ub8e8\uc2dc\uc5b4\uc2a4 H. \ud478\ud2b8(L. H. Foote)\uac00 \ubc29\ud55c\ud558\uc790 \uc870\uc120\ub3c4 \ubbf8\uad6d\uc5d0 \uce5c\uc120\uc0ac\uc808\ub2e8(\ubcf4\ube59\uc0ac)\uc744 \ud30c\uacac\ud558\uc600\uace0, \uc720\uae38\uc900\uc740 \ud6c4\ucfe0\uc790\uc640 \uc720\ud0a4\uce58\uc5d0\uac8c \ubc31\uc778\uc744 \uc0c1\ub300\ud558\ub294 \ubc29\ubc95\uc744 \ubb38\uc758\ud558\uc600\ub2e4. \uc774 \ubb34\ub835 \ud6c4\ucfe0\uc790\uc640\ub294 \ub9ce\uc740 \uc800\uc791\ubb3c\uc744 \ubc1c\ud45c\ud568\uacfc \ub3d9\uc2dc\uc5d0 1882\ub144 3\uc6d4 1\uc77c\ubd80\ud130 \uc77c\ubcf8\uc5d0 \uc9c0\uc9c0\uc2e0\ubcf4(\u6642\u4e8b\u65b0\u5831)\ub77c\ub294 \uc77c\uac04\uc9c0\ub97c \ucc3d\uac04\ud588\ub2e4. \uc77c\ubcf8\uc758 \ud6c4\ucfe0\uc790\uc640\ub85c\ubd80\ud130 \ud3b8\uc9c0\ub97c \uc804\ub2ec\ubc1b\uace0 \uc6d0\uace0 \uc694\uccad\uc774 \uc788\uc790 \uadf8\ub294 \uac10\uaca9, \uc77c\ubcf8 \uc0ac\ud68c\uc5d0\uc11c \uc2e0\ubb38\uc774 \uc5bc\ub9c8\ub098 \ud070 \uc5ed\ud560\uc744 \ud558\uace0 \uc788\ub294\uac00\uc5d0 \ub300\ud55c \ub0b4\uc6a9\uc744 \uadf8\ud574 4\uc6d4 21\uc77c '\uc2e0\ubb38\uc758 \uae30\ub825\uc744 \ub17c \ud568'\uc774\ub77c\ub294 \uc81c\ubaa9\uc73c\ub85c \ubcf4\ub0c8\uace0, \ud6c4\ucfe0\uc790\uc640 \uc720\ud0a4\uce58\ub294 \uc774\ub97c \uba85\ubb38\uc774\ub77c\uba70 \uc790\uc2e0\uc758 \uc9c0\uc9c0\uc2e0\ubcf4\uc5d0 \uac8c\uc7ac\ud588\ub2e4.", "paragraph_answer": "\ud55c\ud3b8 \ubbf8\uad6d\uacfc \uc218\uad50\ub97c \uccb4\uacb0\ud55c \uc870\uc120\uc740 1882\ub144(\uace0\uc885 19) \uc8fc\uc870\uc120 \ubbf8\uad6d \uacf5\uc0ac \ub8e8\uc2dc\uc5b4\uc2a4 H. \ud478\ud2b8(L. H. Foote)\uac00 \ubc29\ud55c\ud558\uc790 \uc870\uc120\ub3c4 \ubbf8\uad6d\uc5d0 \uce5c\uc120\uc0ac\uc808\ub2e8(\ubcf4\ube59\uc0ac)\uc744 \ud30c\uacac\ud558\uc600\uace0, \uc720\uae38\uc900\uc740 \ud6c4\ucfe0\uc790\uc640 \uc720\ud0a4\uce58\uc5d0\uac8c \ubc31\uc778\uc744 \uc0c1\ub300\ud558\ub294 \ubc29\ubc95\uc744 \ubb38\uc758\ud558\uc600\ub2e4. \uc774 \ubb34\ub835 \ud6c4\ucfe0\uc790\uc640\ub294 \ub9ce\uc740 \uc800\uc791\ubb3c\uc744 \ubc1c\ud45c\ud568\uacfc \ub3d9\uc2dc\uc5d0 1882\ub144 3\uc6d4 1\uc77c\ubd80\ud130 \uc77c\ubcf8\uc5d0 \uc9c0\uc9c0\uc2e0\ubcf4(\u6642\u4e8b\u65b0\u5831)\ub77c\ub294 \uc77c\uac04\uc9c0\ub97c \ucc3d\uac04\ud588\ub2e4. \uc77c\ubcf8\uc758 \ud6c4\ucfe0\uc790\uc640\ub85c\ubd80\ud130 \ud3b8\uc9c0\ub97c \uc804\ub2ec\ubc1b\uace0 \uc6d0\uace0 \uc694\uccad\uc774 \uc788\uc790 \uadf8\ub294 \uac10\uaca9, \uc77c\ubcf8 \uc0ac\ud68c\uc5d0\uc11c \uc2e0\ubb38\uc774 \uc5bc\ub9c8\ub098 \ud070 \uc5ed\ud560\uc744 \ud558\uace0 \uc788\ub294\uac00\uc5d0 \ub300\ud55c \ub0b4\uc6a9\uc744 \uadf8\ud574 4\uc6d4 21\uc77c '\uc2e0\ubb38\uc758 \uae30\ub825\uc744 \ub17c \ud568'\uc774\ub77c\ub294 \uc81c\ubaa9\uc73c\ub85c \ubcf4\ub0c8\uace0, \ud6c4\ucfe0\uc790\uc640 \uc720\ud0a4\uce58\ub294 \uc774\ub97c \uba85\ubb38\uc774\ub77c\uba70 \uc790\uc2e0\uc758 \uc9c0\uc9c0\uc2e0\ubcf4\uc5d0 \uac8c\uc7ac\ud588\ub2e4.", "sentence_answer": "\uc720\uae38\uc900\uc740 \ud6c4\ucfe0\uc790\uc640 \uc720\ud0a4\uce58\uc5d0\uac8c \ubc31\uc778\uc744 \uc0c1\ub300\ud558\ub294 \ubc29\ubc95\uc744 \ubb38\uc758\ud558\uc600\ub2e4.", "paragraph_id": "6542752-7-1", "paragraph_question": "question: 1882\ub144 3\uc6d4 1\uc77c\ubd80\ud130 \uc77c\ubcf8\uc5d0 \uc9c0\uc9c0\uc2e0\ubcf4 \uc77c\uac04\uc9c0\ub97c \ucc3d\uac04\ud55c \uc778\ubb3c\uc740?, context: \ud55c\ud3b8 \ubbf8\uad6d\uacfc \uc218\uad50\ub97c \uccb4\uacb0\ud55c \uc870\uc120\uc740 1882\ub144(\uace0\uc885 19) \uc8fc\uc870\uc120 \ubbf8\uad6d \uacf5\uc0ac \ub8e8\uc2dc\uc5b4\uc2a4 H. \ud478\ud2b8(L. H. Foote)\uac00 \ubc29\ud55c\ud558\uc790 \uc870\uc120\ub3c4 \ubbf8\uad6d\uc5d0 \uce5c\uc120\uc0ac\uc808\ub2e8(\ubcf4\ube59\uc0ac)\uc744 \ud30c\uacac\ud558\uc600\uace0, \uc720\uae38\uc900\uc740 \ud6c4\ucfe0\uc790\uc640 \uc720\ud0a4\uce58\uc5d0\uac8c \ubc31\uc778\uc744 \uc0c1\ub300\ud558\ub294 \ubc29\ubc95\uc744 \ubb38\uc758\ud558\uc600\ub2e4. \uc774 \ubb34\ub835 \ud6c4\ucfe0\uc790\uc640\ub294 \ub9ce\uc740 \uc800\uc791\ubb3c\uc744 \ubc1c\ud45c\ud568\uacfc \ub3d9\uc2dc\uc5d0 1882\ub144 3\uc6d4 1\uc77c\ubd80\ud130 \uc77c\ubcf8\uc5d0 \uc9c0\uc9c0\uc2e0\ubcf4(\u6642\u4e8b\u65b0\u5831)\ub77c\ub294 \uc77c\uac04\uc9c0\ub97c \ucc3d\uac04\ud588\ub2e4. \uc77c\ubcf8\uc758 \ud6c4\ucfe0\uc790\uc640\ub85c\ubd80\ud130 \ud3b8\uc9c0\ub97c \uc804\ub2ec\ubc1b\uace0 \uc6d0\uace0 \uc694\uccad\uc774 \uc788\uc790 \uadf8\ub294 \uac10\uaca9, \uc77c\ubcf8 \uc0ac\ud68c\uc5d0\uc11c \uc2e0\ubb38\uc774 \uc5bc\ub9c8\ub098 \ud070 \uc5ed\ud560\uc744 \ud558\uace0 \uc788\ub294\uac00\uc5d0 \ub300\ud55c \ub0b4\uc6a9\uc744 \uadf8\ud574 4\uc6d4 21\uc77c '\uc2e0\ubb38\uc758 \uae30\ub825\uc744 \ub17c \ud568'\uc774\ub77c\ub294 \uc81c\ubaa9\uc73c\ub85c \ubcf4\ub0c8\uace0, \ud6c4\ucfe0\uc790\uc640 \uc720\ud0a4\uce58\ub294 \uc774\ub97c \uba85\ubb38\uc774\ub77c\uba70 \uc790\uc2e0\uc758 \uc9c0\uc9c0\uc2e0\ubcf4\uc5d0 \uac8c\uc7ac\ud588\ub2e4."} {"question": "UEFA \ucc54\ud53c\uc5b8\uc2a4\ub9ac\uadf8\uc5d0\uc11c \ud30c\ub128\uce74 \ud0a5\uc744 \uc131\uacf5\uc2dc\ud0a8 \uccbc\uc2dc FC \uc120\uc218\ub294?", "paragraph": "\uc774\ud6c4, UEFA \uc720\ub85c 2000\uc5d0\uc11c \ud504\ub780\uccb4\uc2a4\ucf54 \ud1a0\ud2f0\uac00 \ub124\ub35c\ub780\ub4dc\uc640\uc758 \uc900\uacb0\uc2b9\uc804 \uc2b9\ubd80\ucc28\uae30\uc5d0\uc11c \uc774 \uc29b\uc744 \uc131\uacf5\uc2dc\ucf30\ub2e4. \uadf8\ub9ac\uace0 2006\ub144 FIFA \uc6d4\ub4dc\ucef5\uc5d0\uc11c \uc9c0\ub124\ub518 \uc9c0\ub2e8\uc774 \uc774\ud0c8\ub9ac\uc544\uc640\uc758 \uacb0\uc2b9\uc804\uc5d0\uc11c \uc9c0\uc548\ub8e8\uc774\uc9c0 \ubd80\ud3f0\uc744 \uc0c1\ub300\ub85c \uc774 \uc29b\uc744 \uc131\uacf5\uc2dc\ucf30\uace0, \ub514\ub514\uc5d0 \ub4dc\ub85c\uadf8\ubc14\ub294 UEFA \ucc54\ud53c\uc5b8\uc2a4\ub9ac\uadf8 \uacb0\uc2b9\uc804\uc5d0\uc11c \uccbc\uc2dc FC\uc758 \ub9c8\uc9c0\ub9c9 \uc2b9\ubd80\ucc28\uae30 \ud0a4\ucee4\ub85c \ub098\uc11c \ud30c\ub128\uce74 \ud0a5\uc73c\ub85c \uc131\uacf5\uc2dc\ud0a4\uba70 FC \ubc14\uc774\uc5d0\ub978 \ubb8c\ud5e8\uc744 \uc2b9\ubd80\ucc28\uae30\uc5d0\uc11c 4-3\uc73c\ub85c \uaebe\uace0 \uccbc\uc2dc\uc758 \ucc54\ud53c\uc5b8\uc2a4\ub9ac\uadf8 \uccab \uc6b0\uc2b9\uc744 \uc774\ub04c\uc5c8\ub2e4. UEFA \uc720\ub85c 2012\uc5d0\uc11c\ub294 \uc774\ud0c8\ub9ac\uc544-\uc789\uae00\ub79c\ub4dc\uc758 8\uac15\uc804\uacfc \uc2a4\ud398\uc778-\ud3ec\ub974\ud22c\uac08\uc758 4\uac15\uc804\uc5d0\uc11c \uc2b9\ubd80\ucc28\uae30\uac00 \ubc8c\uc5b4\uc84c\ub2e4. \uc774\ud0c8\ub9ac\uc544-\uc789\uae00\ub79c\ub4dc\uc640\uc758 8\uac15\uc804 \uc2b9\ubd80\ucc28\uae30\uc5d0\uc11c\ub294 \uc548\ub4dc\ub808\uc544 \ud53c\ub97c\ub85c\uac00 \ud30c\ub128\uce74 \ud0a5\uc744 \uc2dc\ub3c4\ud574 \uc131\uacf5\uc2dc\ucf30\uace0, \uc2a4\ud398\uc778-\ud3ec\ub974\ud22c\uac08\uacfc\uc758 4\uac15\uc804 \uc2b9\ubd80\ucc28\uae30\uc5d0\uc11c\ub294 \uc138\ub974\ud788\uc624 \ub77c\ubaa8\uc2a4\uac00 \ud30c\ub128\uce74 \ud0a5\uc744 \uc2dc\ub3c4\ud574 \uc5ed\uc2dc \uc131\uacf5\ud558\uc600\ub2e4. \ud765\ubbf8\ub86d\uac8c\ub3c4, UEFA \uc720\ub85c 1976\uc5d0\uc11c \ud30c\ub128\uce74\uac00 \ud30c\ub128\uce74 \ud0a5\uc744 \uc131\uacf5\ud55c \uccb4\ucf54\uc2ac\ub85c\ubc14\ud0a4\uc544, \ud53c\ub97c\ub85c\uac00 \uc131\uacf5\ud55c \uc774\ud0c8\ub9ac\uc544, \ub77c\ubaa8\uc2a4\uac00 \uc131\uacf5\ud55c \uc2a4\ud398\uc778 \ubaa8\ub450 \uadf8 \uc2b9\ubd80\ucc28\uae30\uc5d0\uc11c \uc774\uacbc\ub2e4.", "answer": "\ub514\ub514\uc5d0 \ub4dc\ub85c\uadf8\ubc14", "sentence": "\uadf8\ub9ac\uace0 2006\ub144 FIFA \uc6d4\ub4dc\ucef5\uc5d0\uc11c \uc9c0\ub124\ub518 \uc9c0\ub2e8\uc774 \uc774\ud0c8\ub9ac\uc544\uc640\uc758 \uacb0\uc2b9\uc804\uc5d0\uc11c \uc9c0\uc548\ub8e8\uc774\uc9c0 \ubd80\ud3f0\uc744 \uc0c1\ub300\ub85c \uc774 \uc29b\uc744 \uc131\uacf5\uc2dc\ucf30\uace0, \ub514\ub514\uc5d0 \ub4dc\ub85c\uadf8\ubc14 \ub294 UEFA \ucc54\ud53c\uc5b8\uc2a4\ub9ac\uadf8 \uacb0\uc2b9\uc804\uc5d0\uc11c \uccbc\uc2dc FC\uc758 \ub9c8\uc9c0\ub9c9 \uc2b9\ubd80\ucc28\uae30 \ud0a4\ucee4\ub85c \ub098\uc11c \ud30c\ub128\uce74 \ud0a5\uc73c\ub85c \uc131\uacf5\uc2dc\ud0a4\uba70 FC \ubc14\uc774\uc5d0\ub978 \ubb8c\ud5e8\uc744 \uc2b9\ubd80\ucc28\uae30\uc5d0\uc11c 4-3\uc73c\ub85c \uaebe\uace0 \uccbc\uc2dc\uc758 \ucc54\ud53c\uc5b8\uc2a4\ub9ac\uadf8 \uccab \uc6b0\uc2b9\uc744 \uc774\ub04c\uc5c8\ub2e4.", "paragraph_sentence": "\uc774\ud6c4, UEFA \uc720\ub85c 2000\uc5d0\uc11c \ud504\ub780\uccb4\uc2a4\ucf54 \ud1a0\ud2f0\uac00 \ub124\ub35c\ub780\ub4dc\uc640\uc758 \uc900\uacb0\uc2b9\uc804 \uc2b9\ubd80\ucc28\uae30\uc5d0\uc11c \uc774 \uc29b\uc744 \uc131\uacf5\uc2dc\ucf30\ub2e4. \uadf8\ub9ac\uace0 2006\ub144 FIFA \uc6d4\ub4dc\ucef5\uc5d0\uc11c \uc9c0\ub124\ub518 \uc9c0\ub2e8\uc774 \uc774\ud0c8\ub9ac\uc544\uc640\uc758 \uacb0\uc2b9\uc804\uc5d0\uc11c \uc9c0\uc548\ub8e8\uc774\uc9c0 \ubd80\ud3f0\uc744 \uc0c1\ub300\ub85c \uc774 \uc29b\uc744 \uc131\uacf5\uc2dc\ucf30\uace0, \ub514\ub514\uc5d0 \ub4dc\ub85c\uadf8\ubc14 \ub294 UEFA \ucc54\ud53c\uc5b8\uc2a4\ub9ac\uadf8 \uacb0\uc2b9\uc804\uc5d0\uc11c \uccbc\uc2dc FC\uc758 \ub9c8\uc9c0\ub9c9 \uc2b9\ubd80\ucc28\uae30 \ud0a4\ucee4\ub85c \ub098\uc11c \ud30c\ub128\uce74 \ud0a5\uc73c\ub85c \uc131\uacf5\uc2dc\ud0a4\uba70 FC \ubc14\uc774\uc5d0\ub978 \ubb8c\ud5e8\uc744 \uc2b9\ubd80\ucc28\uae30\uc5d0\uc11c 4-3\uc73c\ub85c \uaebe\uace0 \uccbc\uc2dc\uc758 \ucc54\ud53c\uc5b8\uc2a4\ub9ac\uadf8 \uccab \uc6b0\uc2b9\uc744 \uc774\ub04c\uc5c8\ub2e4. UEFA \uc720\ub85c 2012\uc5d0\uc11c\ub294 \uc774\ud0c8\ub9ac\uc544-\uc789\uae00\ub79c\ub4dc\uc758 8\uac15\uc804\uacfc \uc2a4\ud398\uc778-\ud3ec\ub974\ud22c\uac08\uc758 4\uac15\uc804\uc5d0\uc11c \uc2b9\ubd80\ucc28\uae30\uac00 \ubc8c\uc5b4\uc84c\ub2e4. \uc774\ud0c8\ub9ac\uc544-\uc789\uae00\ub79c\ub4dc\uc640\uc758 8\uac15\uc804 \uc2b9\ubd80\ucc28\uae30\uc5d0\uc11c\ub294 \uc548\ub4dc\ub808\uc544 \ud53c\ub97c\ub85c\uac00 \ud30c\ub128\uce74 \ud0a5\uc744 \uc2dc\ub3c4\ud574 \uc131\uacf5\uc2dc\ucf30\uace0, \uc2a4\ud398\uc778-\ud3ec\ub974\ud22c\uac08\uacfc\uc758 4\uac15\uc804 \uc2b9\ubd80\ucc28\uae30\uc5d0\uc11c\ub294 \uc138\ub974\ud788\uc624 \ub77c\ubaa8\uc2a4\uac00 \ud30c\ub128\uce74 \ud0a5\uc744 \uc2dc\ub3c4\ud574 \uc5ed\uc2dc \uc131\uacf5\ud558\uc600\ub2e4. \ud765\ubbf8\ub86d\uac8c\ub3c4, UEFA \uc720\ub85c 1976\uc5d0\uc11c \ud30c\ub128\uce74\uac00 \ud30c\ub128\uce74 \ud0a5\uc744 \uc131\uacf5\ud55c \uccb4\ucf54\uc2ac\ub85c\ubc14\ud0a4\uc544, \ud53c\ub97c\ub85c\uac00 \uc131\uacf5\ud55c \uc774\ud0c8\ub9ac\uc544, \ub77c\ubaa8\uc2a4\uac00 \uc131\uacf5\ud55c \uc2a4\ud398\uc778 \ubaa8\ub450 \uadf8 \uc2b9\ubd80\ucc28\uae30\uc5d0\uc11c \uc774\uacbc\ub2e4.", "paragraph_answer": "\uc774\ud6c4, UEFA \uc720\ub85c 2000\uc5d0\uc11c \ud504\ub780\uccb4\uc2a4\ucf54 \ud1a0\ud2f0\uac00 \ub124\ub35c\ub780\ub4dc\uc640\uc758 \uc900\uacb0\uc2b9\uc804 \uc2b9\ubd80\ucc28\uae30\uc5d0\uc11c \uc774 \uc29b\uc744 \uc131\uacf5\uc2dc\ucf30\ub2e4. \uadf8\ub9ac\uace0 2006\ub144 FIFA \uc6d4\ub4dc\ucef5\uc5d0\uc11c \uc9c0\ub124\ub518 \uc9c0\ub2e8\uc774 \uc774\ud0c8\ub9ac\uc544\uc640\uc758 \uacb0\uc2b9\uc804\uc5d0\uc11c \uc9c0\uc548\ub8e8\uc774\uc9c0 \ubd80\ud3f0\uc744 \uc0c1\ub300\ub85c \uc774 \uc29b\uc744 \uc131\uacf5\uc2dc\ucf30\uace0, \ub514\ub514\uc5d0 \ub4dc\ub85c\uadf8\ubc14 \ub294 UEFA \ucc54\ud53c\uc5b8\uc2a4\ub9ac\uadf8 \uacb0\uc2b9\uc804\uc5d0\uc11c \uccbc\uc2dc FC\uc758 \ub9c8\uc9c0\ub9c9 \uc2b9\ubd80\ucc28\uae30 \ud0a4\ucee4\ub85c \ub098\uc11c \ud30c\ub128\uce74 \ud0a5\uc73c\ub85c \uc131\uacf5\uc2dc\ud0a4\uba70 FC \ubc14\uc774\uc5d0\ub978 \ubb8c\ud5e8\uc744 \uc2b9\ubd80\ucc28\uae30\uc5d0\uc11c 4-3\uc73c\ub85c \uaebe\uace0 \uccbc\uc2dc\uc758 \ucc54\ud53c\uc5b8\uc2a4\ub9ac\uadf8 \uccab \uc6b0\uc2b9\uc744 \uc774\ub04c\uc5c8\ub2e4. UEFA \uc720\ub85c 2012\uc5d0\uc11c\ub294 \uc774\ud0c8\ub9ac\uc544-\uc789\uae00\ub79c\ub4dc\uc758 8\uac15\uc804\uacfc \uc2a4\ud398\uc778-\ud3ec\ub974\ud22c\uac08\uc758 4\uac15\uc804\uc5d0\uc11c \uc2b9\ubd80\ucc28\uae30\uac00 \ubc8c\uc5b4\uc84c\ub2e4. \uc774\ud0c8\ub9ac\uc544-\uc789\uae00\ub79c\ub4dc\uc640\uc758 8\uac15\uc804 \uc2b9\ubd80\ucc28\uae30\uc5d0\uc11c\ub294 \uc548\ub4dc\ub808\uc544 \ud53c\ub97c\ub85c\uac00 \ud30c\ub128\uce74 \ud0a5\uc744 \uc2dc\ub3c4\ud574 \uc131\uacf5\uc2dc\ucf30\uace0, \uc2a4\ud398\uc778-\ud3ec\ub974\ud22c\uac08\uacfc\uc758 4\uac15\uc804 \uc2b9\ubd80\ucc28\uae30\uc5d0\uc11c\ub294 \uc138\ub974\ud788\uc624 \ub77c\ubaa8\uc2a4\uac00 \ud30c\ub128\uce74 \ud0a5\uc744 \uc2dc\ub3c4\ud574 \uc5ed\uc2dc \uc131\uacf5\ud558\uc600\ub2e4. \ud765\ubbf8\ub86d\uac8c\ub3c4, UEFA \uc720\ub85c 1976\uc5d0\uc11c \ud30c\ub128\uce74\uac00 \ud30c\ub128\uce74 \ud0a5\uc744 \uc131\uacf5\ud55c \uccb4\ucf54\uc2ac\ub85c\ubc14\ud0a4\uc544, \ud53c\ub97c\ub85c\uac00 \uc131\uacf5\ud55c \uc774\ud0c8\ub9ac\uc544, \ub77c\ubaa8\uc2a4\uac00 \uc131\uacf5\ud55c \uc2a4\ud398\uc778 \ubaa8\ub450 \uadf8 \uc2b9\ubd80\ucc28\uae30\uc5d0\uc11c \uc774\uacbc\ub2e4.", "sentence_answer": "\uadf8\ub9ac\uace0 2006\ub144 FIFA \uc6d4\ub4dc\ucef5\uc5d0\uc11c \uc9c0\ub124\ub518 \uc9c0\ub2e8\uc774 \uc774\ud0c8\ub9ac\uc544\uc640\uc758 \uacb0\uc2b9\uc804\uc5d0\uc11c \uc9c0\uc548\ub8e8\uc774\uc9c0 \ubd80\ud3f0\uc744 \uc0c1\ub300\ub85c \uc774 \uc29b\uc744 \uc131\uacf5\uc2dc\ucf30\uace0, \ub514\ub514\uc5d0 \ub4dc\ub85c\uadf8\ubc14 \ub294 UEFA \ucc54\ud53c\uc5b8\uc2a4\ub9ac\uadf8 \uacb0\uc2b9\uc804\uc5d0\uc11c \uccbc\uc2dc FC\uc758 \ub9c8\uc9c0\ub9c9 \uc2b9\ubd80\ucc28\uae30 \ud0a4\ucee4\ub85c \ub098\uc11c \ud30c\ub128\uce74 \ud0a5\uc73c\ub85c \uc131\uacf5\uc2dc\ud0a4\uba70 FC \ubc14\uc774\uc5d0\ub978 \ubb8c\ud5e8\uc744 \uc2b9\ubd80\ucc28\uae30\uc5d0\uc11c 4-3\uc73c\ub85c \uaebe\uace0 \uccbc\uc2dc\uc758 \ucc54\ud53c\uc5b8\uc2a4\ub9ac\uadf8 \uccab \uc6b0\uc2b9\uc744 \uc774\ub04c\uc5c8\ub2e4.", "paragraph_id": "6457763-2-2", "paragraph_question": "question: UEFA \ucc54\ud53c\uc5b8\uc2a4\ub9ac\uadf8\uc5d0\uc11c \ud30c\ub128\uce74 \ud0a5\uc744 \uc131\uacf5\uc2dc\ud0a8 \uccbc\uc2dc FC \uc120\uc218\ub294?, context: \uc774\ud6c4, UEFA \uc720\ub85c 2000\uc5d0\uc11c \ud504\ub780\uccb4\uc2a4\ucf54 \ud1a0\ud2f0\uac00 \ub124\ub35c\ub780\ub4dc\uc640\uc758 \uc900\uacb0\uc2b9\uc804 \uc2b9\ubd80\ucc28\uae30\uc5d0\uc11c \uc774 \uc29b\uc744 \uc131\uacf5\uc2dc\ucf30\ub2e4. \uadf8\ub9ac\uace0 2006\ub144 FIFA \uc6d4\ub4dc\ucef5\uc5d0\uc11c \uc9c0\ub124\ub518 \uc9c0\ub2e8\uc774 \uc774\ud0c8\ub9ac\uc544\uc640\uc758 \uacb0\uc2b9\uc804\uc5d0\uc11c \uc9c0\uc548\ub8e8\uc774\uc9c0 \ubd80\ud3f0\uc744 \uc0c1\ub300\ub85c \uc774 \uc29b\uc744 \uc131\uacf5\uc2dc\ucf30\uace0, \ub514\ub514\uc5d0 \ub4dc\ub85c\uadf8\ubc14\ub294 UEFA \ucc54\ud53c\uc5b8\uc2a4\ub9ac\uadf8 \uacb0\uc2b9\uc804\uc5d0\uc11c \uccbc\uc2dc FC\uc758 \ub9c8\uc9c0\ub9c9 \uc2b9\ubd80\ucc28\uae30 \ud0a4\ucee4\ub85c \ub098\uc11c \ud30c\ub128\uce74 \ud0a5\uc73c\ub85c \uc131\uacf5\uc2dc\ud0a4\uba70 FC \ubc14\uc774\uc5d0\ub978 \ubb8c\ud5e8\uc744 \uc2b9\ubd80\ucc28\uae30\uc5d0\uc11c 4-3\uc73c\ub85c \uaebe\uace0 \uccbc\uc2dc\uc758 \ucc54\ud53c\uc5b8\uc2a4\ub9ac\uadf8 \uccab \uc6b0\uc2b9\uc744 \uc774\ub04c\uc5c8\ub2e4. UEFA \uc720\ub85c 2012\uc5d0\uc11c\ub294 \uc774\ud0c8\ub9ac\uc544-\uc789\uae00\ub79c\ub4dc\uc758 8\uac15\uc804\uacfc \uc2a4\ud398\uc778-\ud3ec\ub974\ud22c\uac08\uc758 4\uac15\uc804\uc5d0\uc11c \uc2b9\ubd80\ucc28\uae30\uac00 \ubc8c\uc5b4\uc84c\ub2e4. \uc774\ud0c8\ub9ac\uc544-\uc789\uae00\ub79c\ub4dc\uc640\uc758 8\uac15\uc804 \uc2b9\ubd80\ucc28\uae30\uc5d0\uc11c\ub294 \uc548\ub4dc\ub808\uc544 \ud53c\ub97c\ub85c\uac00 \ud30c\ub128\uce74 \ud0a5\uc744 \uc2dc\ub3c4\ud574 \uc131\uacf5\uc2dc\ucf30\uace0, \uc2a4\ud398\uc778-\ud3ec\ub974\ud22c\uac08\uacfc\uc758 4\uac15\uc804 \uc2b9\ubd80\ucc28\uae30\uc5d0\uc11c\ub294 \uc138\ub974\ud788\uc624 \ub77c\ubaa8\uc2a4\uac00 \ud30c\ub128\uce74 \ud0a5\uc744 \uc2dc\ub3c4\ud574 \uc5ed\uc2dc \uc131\uacf5\ud558\uc600\ub2e4. \ud765\ubbf8\ub86d\uac8c\ub3c4, UEFA \uc720\ub85c 1976\uc5d0\uc11c \ud30c\ub128\uce74\uac00 \ud30c\ub128\uce74 \ud0a5\uc744 \uc131\uacf5\ud55c \uccb4\ucf54\uc2ac\ub85c\ubc14\ud0a4\uc544, \ud53c\ub97c\ub85c\uac00 \uc131\uacf5\ud55c \uc774\ud0c8\ub9ac\uc544, \ub77c\ubaa8\uc2a4\uac00 \uc131\uacf5\ud55c \uc2a4\ud398\uc778 \ubaa8\ub450 \uadf8 \uc2b9\ubd80\ucc28\uae30\uc5d0\uc11c \uc774\uacbc\ub2e4."} {"question": "\ud558\uc6d0\uc774 \uc5f0\ubc29 \uace0\uc704 \uacf5\ubb34\uc6d0\uc744 \ud0c4\ud575\ud558\uace0\uc790 \ud560 \ub54c \ud0c4\ud575\uc7ac\ud310\uc744 \uc5f4 \uc218 \uc788\ub294 \uad8c\ud55c\uc744 \uac16\ub294 \uac83\uc740?", "paragraph": "\ubbf8\uad6d \ud5cc\ubc95 \uc81c1\uc870\ub294 \uc5f0\ubc29\ubc95\uc744 \uc81c\uc815\ud558\uace0, \ud3d0\uc9c0\ud558\uba70 \uac1c\uc815\ud558\ub294 \ubaa8\ub4e0 \uc785\ubc95\uad8c\uc744 \uc758\ud68c\uc5d0 \ubd80\uc5ec\ud55c\ub2e4. \ub610 \uc5f0\ubc29 \uc138\uae08\uc758 \uaddc\ubaa8\ub97c \uacb0\uc815\ud558\uace0, \uc815\ubd80\uc758 \uc608\uc0b0\uc744 \uc2b9\uc778\ud55c\ub2e4. \ud558\uc6d0\uacfc \uc0c1\uc6d0\uc740 \uc785\ubc95 \uacfc\uc815\uc5d0 \uc788\uc5b4 \ub300\ub4f1\ud55c \ud30c\ud2b8\ub108\uc774\ub2e4. (\ubc95\ub960\uc740 \uc591\uc6d0\uc758 \ub3d9\uc758 \uc5c6\uc774\ub294 \uc81c\uc815\ub418\uc9c0 \uc54a\ub294\ub2e4.) \ud558\uc9c0\ub9cc, \ubbf8\uad6d \ud5cc\ubc95\uc740 \uac01 \uc6d0\uc5d0 \ub3c5\ud2b9\ud55c \uad8c\ud55c\uc744 \ubd80\uc5ec\ud558\uace0 \uc788\ub2e4. \uc0c1\uc6d0\uc740 \uba87 \uac00\uc9c0 \uace0\uc720\ud55c \uad8c\ud55c\uc744 \uac00\uc9c4\ub2e4. \uccab\uc9f8, \ud558\uc6d0\uc774 \uc5f0\ubc29 \uace0\uc704 \uacf5\ubb34\uc6d0\uc744 \ud0c4\ud575\ud558\uace0\uc790 \ud560 \ub54c \ud0c4\ud575\uc7ac\ud310\uc744 \uc5f4 \uc218 \uc788\ub294 \uad8c\ud55c\uc774 \uc788\ub2e4. \ub458\uc9f8, \ub300\ud1b5\ub839\uc774 \uc9c0\uba85\ud558\ub294 \uc5f0\ubc29 \uace0\uc704 \uacf5\ubb34\uc6d0\uc5d0 \ub300\ud55c \uc2b9\uc778\uad8c\uc744 \uac00\uc9c4\ub2e4. \uc14b\uc9f8, \ubbf8\uad6d\uc774 \uccb4\uacb0\ud558\ub294 \uc870\uc57d\uc5d0 \ub300\ud55c \uc2b9\uc778\uad8c\uc774 \uc788\ub2e4. \ubc18\uba74, \uc138\uc785 \uc9d5\uc218\uc5d0 \uad00\ud55c \ubc95\ub960\uc548\uc740 \ud558\uc6d0\uc5d0\uc11c \uba3c\uc800 \uc81c\uc548\ub418\uc5b4\uc57c \ud55c\ub2e4. \ub610\ud55c \ud558\uc6d0\uc774 \uc5f0\ubc29 \uace0\uc704 \uacf5\ubb34\uc6d0\uc5d0 \ub300\ud55c \ud0c4\ud575 \uad8c\ud55c\uc744 \uc804\uc720\ud558\ub294\ub370 \ub300\ud574, \uc0c1\uc6d0\uc740 \ud0c4\ud575 \uc2ec\ud310\uad8c\uc744 \uc804\uc720\ud55c\ub2e4. \uc758\ud68c\uc5d0\ub294 \ud574\uc0b0\uc774 \uc5c6\uc73c\uba70, 2\ub144\ub9c8\ub2e4 \uc804\uc6d0 \uac1c\uc120\ub418\ub294 \ud558\uc6d0\uc758\uc6d0\uc758 \uc784\uae30\uc5d0 \ub9de\ucd94\uc5b4\uc11c \ud68c\uae30\uac00 \uacb0\uc815\ub41c\ub2e4. \uc758\uc6d0\ub4e4\uc740 \uc0c1\uc6d0 15\uac1c, \ud558\uc6d0 22\uac1c\uc758 \uc0c1\uc784\uc704\uc6d0\ud68c \uc911 \uc5b4\ub290 \uc704\uc6d0\ud68c\uc5d0 \uc18c\uc18d\ud558\uba70, \uc2ec\uc758\ub294 \uc704\uc6d0\ud68c \uc911\uc2ec\uc73c\ub85c \uc9c4\ud589\ub41c\ub2e4. \uc758\ud68c\ub294 \uc6cc\uc2f1\ud134 D.C.\uc758 \ubbf8\uad6d \uc758\ud68c\uc758\uc0ac\ub2f9\uc5d0\uc11c \uc5f4\ub9b0\ub2e4.", "answer": "\uc0c1\uc6d0", "sentence": "\ud558\uc6d0\uacfc \uc0c1\uc6d0 \uc740 \uc785\ubc95 \uacfc\uc815\uc5d0 \uc788\uc5b4 \ub300\ub4f1\ud55c \ud30c\ud2b8\ub108\uc774\ub2e4.", "paragraph_sentence": "\ubbf8\uad6d \ud5cc\ubc95 \uc81c1\uc870\ub294 \uc5f0\ubc29\ubc95\uc744 \uc81c\uc815\ud558\uace0, \ud3d0\uc9c0\ud558\uba70 \uac1c\uc815\ud558\ub294 \ubaa8\ub4e0 \uc785\ubc95\uad8c\uc744 \uc758\ud68c\uc5d0 \ubd80\uc5ec\ud55c\ub2e4. \ub610 \uc5f0\ubc29 \uc138\uae08\uc758 \uaddc\ubaa8\ub97c \uacb0\uc815\ud558\uace0, \uc815\ubd80\uc758 \uc608\uc0b0\uc744 \uc2b9\uc778\ud55c\ub2e4. \ud558\uc6d0\uacfc \uc0c1\uc6d0 \uc740 \uc785\ubc95 \uacfc\uc815\uc5d0 \uc788\uc5b4 \ub300\ub4f1\ud55c \ud30c\ud2b8\ub108\uc774\ub2e4. (\ubc95\ub960\uc740 \uc591\uc6d0\uc758 \ub3d9\uc758 \uc5c6\uc774\ub294 \uc81c\uc815\ub418\uc9c0 \uc54a\ub294\ub2e4.) \ud558\uc9c0\ub9cc, \ubbf8\uad6d \ud5cc\ubc95\uc740 \uac01 \uc6d0\uc5d0 \ub3c5\ud2b9\ud55c \uad8c\ud55c\uc744 \ubd80\uc5ec\ud558\uace0 \uc788\ub2e4. \uc0c1\uc6d0\uc740 \uba87 \uac00\uc9c0 \uace0\uc720\ud55c \uad8c\ud55c\uc744 \uac00\uc9c4\ub2e4. \uccab\uc9f8, \ud558\uc6d0\uc774 \uc5f0\ubc29 \uace0\uc704 \uacf5\ubb34\uc6d0\uc744 \ud0c4\ud575\ud558\uace0\uc790 \ud560 \ub54c \ud0c4\ud575\uc7ac\ud310\uc744 \uc5f4 \uc218 \uc788\ub294 \uad8c\ud55c\uc774 \uc788\ub2e4. \ub458\uc9f8, \ub300\ud1b5\ub839\uc774 \uc9c0\uba85\ud558\ub294 \uc5f0\ubc29 \uace0\uc704 \uacf5\ubb34\uc6d0\uc5d0 \ub300\ud55c \uc2b9\uc778\uad8c\uc744 \uac00\uc9c4\ub2e4. \uc14b\uc9f8, \ubbf8\uad6d\uc774 \uccb4\uacb0\ud558\ub294 \uc870\uc57d\uc5d0 \ub300\ud55c \uc2b9\uc778\uad8c\uc774 \uc788\ub2e4. \ubc18\uba74, \uc138\uc785 \uc9d5\uc218\uc5d0 \uad00\ud55c \ubc95\ub960\uc548\uc740 \ud558\uc6d0\uc5d0\uc11c \uba3c\uc800 \uc81c\uc548\ub418\uc5b4\uc57c \ud55c\ub2e4. \ub610\ud55c \ud558\uc6d0\uc774 \uc5f0\ubc29 \uace0\uc704 \uacf5\ubb34\uc6d0\uc5d0 \ub300\ud55c \ud0c4\ud575 \uad8c\ud55c\uc744 \uc804\uc720\ud558\ub294\ub370 \ub300\ud574, \uc0c1\uc6d0\uc740 \ud0c4\ud575 \uc2ec\ud310\uad8c\uc744 \uc804\uc720\ud55c\ub2e4. \uc758\ud68c\uc5d0\ub294 \ud574\uc0b0\uc774 \uc5c6\uc73c\uba70, 2\ub144\ub9c8\ub2e4 \uc804\uc6d0 \uac1c\uc120\ub418\ub294 \ud558\uc6d0\uc758\uc6d0\uc758 \uc784\uae30\uc5d0 \ub9de\ucd94\uc5b4\uc11c \ud68c\uae30\uac00 \uacb0\uc815\ub41c\ub2e4. \uc758\uc6d0\ub4e4\uc740 \uc0c1\uc6d0 15\uac1c, \ud558\uc6d0 22\uac1c\uc758 \uc0c1\uc784\uc704\uc6d0\ud68c \uc911 \uc5b4\ub290 \uc704\uc6d0\ud68c\uc5d0 \uc18c\uc18d\ud558\uba70, \uc2ec\uc758\ub294 \uc704\uc6d0\ud68c \uc911\uc2ec\uc73c\ub85c \uc9c4\ud589\ub41c\ub2e4. \uc758\ud68c\ub294 \uc6cc\uc2f1\ud134 D.C.\uc758 \ubbf8\uad6d \uc758\ud68c\uc758\uc0ac\ub2f9\uc5d0\uc11c \uc5f4\ub9b0\ub2e4.", "paragraph_answer": "\ubbf8\uad6d \ud5cc\ubc95 \uc81c1\uc870\ub294 \uc5f0\ubc29\ubc95\uc744 \uc81c\uc815\ud558\uace0, \ud3d0\uc9c0\ud558\uba70 \uac1c\uc815\ud558\ub294 \ubaa8\ub4e0 \uc785\ubc95\uad8c\uc744 \uc758\ud68c\uc5d0 \ubd80\uc5ec\ud55c\ub2e4. \ub610 \uc5f0\ubc29 \uc138\uae08\uc758 \uaddc\ubaa8\ub97c \uacb0\uc815\ud558\uace0, \uc815\ubd80\uc758 \uc608\uc0b0\uc744 \uc2b9\uc778\ud55c\ub2e4. \ud558\uc6d0\uacfc \uc0c1\uc6d0 \uc740 \uc785\ubc95 \uacfc\uc815\uc5d0 \uc788\uc5b4 \ub300\ub4f1\ud55c \ud30c\ud2b8\ub108\uc774\ub2e4. (\ubc95\ub960\uc740 \uc591\uc6d0\uc758 \ub3d9\uc758 \uc5c6\uc774\ub294 \uc81c\uc815\ub418\uc9c0 \uc54a\ub294\ub2e4.) \ud558\uc9c0\ub9cc, \ubbf8\uad6d \ud5cc\ubc95\uc740 \uac01 \uc6d0\uc5d0 \ub3c5\ud2b9\ud55c \uad8c\ud55c\uc744 \ubd80\uc5ec\ud558\uace0 \uc788\ub2e4. \uc0c1\uc6d0\uc740 \uba87 \uac00\uc9c0 \uace0\uc720\ud55c \uad8c\ud55c\uc744 \uac00\uc9c4\ub2e4. \uccab\uc9f8, \ud558\uc6d0\uc774 \uc5f0\ubc29 \uace0\uc704 \uacf5\ubb34\uc6d0\uc744 \ud0c4\ud575\ud558\uace0\uc790 \ud560 \ub54c \ud0c4\ud575\uc7ac\ud310\uc744 \uc5f4 \uc218 \uc788\ub294 \uad8c\ud55c\uc774 \uc788\ub2e4. \ub458\uc9f8, \ub300\ud1b5\ub839\uc774 \uc9c0\uba85\ud558\ub294 \uc5f0\ubc29 \uace0\uc704 \uacf5\ubb34\uc6d0\uc5d0 \ub300\ud55c \uc2b9\uc778\uad8c\uc744 \uac00\uc9c4\ub2e4. \uc14b\uc9f8, \ubbf8\uad6d\uc774 \uccb4\uacb0\ud558\ub294 \uc870\uc57d\uc5d0 \ub300\ud55c \uc2b9\uc778\uad8c\uc774 \uc788\ub2e4. \ubc18\uba74, \uc138\uc785 \uc9d5\uc218\uc5d0 \uad00\ud55c \ubc95\ub960\uc548\uc740 \ud558\uc6d0\uc5d0\uc11c \uba3c\uc800 \uc81c\uc548\ub418\uc5b4\uc57c \ud55c\ub2e4. \ub610\ud55c \ud558\uc6d0\uc774 \uc5f0\ubc29 \uace0\uc704 \uacf5\ubb34\uc6d0\uc5d0 \ub300\ud55c \ud0c4\ud575 \uad8c\ud55c\uc744 \uc804\uc720\ud558\ub294\ub370 \ub300\ud574, \uc0c1\uc6d0\uc740 \ud0c4\ud575 \uc2ec\ud310\uad8c\uc744 \uc804\uc720\ud55c\ub2e4. \uc758\ud68c\uc5d0\ub294 \ud574\uc0b0\uc774 \uc5c6\uc73c\uba70, 2\ub144\ub9c8\ub2e4 \uc804\uc6d0 \uac1c\uc120\ub418\ub294 \ud558\uc6d0\uc758\uc6d0\uc758 \uc784\uae30\uc5d0 \ub9de\ucd94\uc5b4\uc11c \ud68c\uae30\uac00 \uacb0\uc815\ub41c\ub2e4. \uc758\uc6d0\ub4e4\uc740 \uc0c1\uc6d0 15\uac1c, \ud558\uc6d0 22\uac1c\uc758 \uc0c1\uc784\uc704\uc6d0\ud68c \uc911 \uc5b4\ub290 \uc704\uc6d0\ud68c\uc5d0 \uc18c\uc18d\ud558\uba70, \uc2ec\uc758\ub294 \uc704\uc6d0\ud68c \uc911\uc2ec\uc73c\ub85c \uc9c4\ud589\ub41c\ub2e4. \uc758\ud68c\ub294 \uc6cc\uc2f1\ud134 D.C.\uc758 \ubbf8\uad6d \uc758\ud68c\uc758\uc0ac\ub2f9\uc5d0\uc11c \uc5f4\ub9b0\ub2e4.", "sentence_answer": "\ud558\uc6d0\uacfc \uc0c1\uc6d0 \uc740 \uc785\ubc95 \uacfc\uc815\uc5d0 \uc788\uc5b4 \ub300\ub4f1\ud55c \ud30c\ud2b8\ub108\uc774\ub2e4.", "paragraph_id": "6534652-10-0", "paragraph_question": "question: \ud558\uc6d0\uc774 \uc5f0\ubc29 \uace0\uc704 \uacf5\ubb34\uc6d0\uc744 \ud0c4\ud575\ud558\uace0\uc790 \ud560 \ub54c \ud0c4\ud575\uc7ac\ud310\uc744 \uc5f4 \uc218 \uc788\ub294 \uad8c\ud55c\uc744 \uac16\ub294 \uac83\uc740?, context: \ubbf8\uad6d \ud5cc\ubc95 \uc81c1\uc870\ub294 \uc5f0\ubc29\ubc95\uc744 \uc81c\uc815\ud558\uace0, \ud3d0\uc9c0\ud558\uba70 \uac1c\uc815\ud558\ub294 \ubaa8\ub4e0 \uc785\ubc95\uad8c\uc744 \uc758\ud68c\uc5d0 \ubd80\uc5ec\ud55c\ub2e4. \ub610 \uc5f0\ubc29 \uc138\uae08\uc758 \uaddc\ubaa8\ub97c \uacb0\uc815\ud558\uace0, \uc815\ubd80\uc758 \uc608\uc0b0\uc744 \uc2b9\uc778\ud55c\ub2e4. \ud558\uc6d0\uacfc \uc0c1\uc6d0\uc740 \uc785\ubc95 \uacfc\uc815\uc5d0 \uc788\uc5b4 \ub300\ub4f1\ud55c \ud30c\ud2b8\ub108\uc774\ub2e4. (\ubc95\ub960\uc740 \uc591\uc6d0\uc758 \ub3d9\uc758 \uc5c6\uc774\ub294 \uc81c\uc815\ub418\uc9c0 \uc54a\ub294\ub2e4.) \ud558\uc9c0\ub9cc, \ubbf8\uad6d \ud5cc\ubc95\uc740 \uac01 \uc6d0\uc5d0 \ub3c5\ud2b9\ud55c \uad8c\ud55c\uc744 \ubd80\uc5ec\ud558\uace0 \uc788\ub2e4. \uc0c1\uc6d0\uc740 \uba87 \uac00\uc9c0 \uace0\uc720\ud55c \uad8c\ud55c\uc744 \uac00\uc9c4\ub2e4. \uccab\uc9f8, \ud558\uc6d0\uc774 \uc5f0\ubc29 \uace0\uc704 \uacf5\ubb34\uc6d0\uc744 \ud0c4\ud575\ud558\uace0\uc790 \ud560 \ub54c \ud0c4\ud575\uc7ac\ud310\uc744 \uc5f4 \uc218 \uc788\ub294 \uad8c\ud55c\uc774 \uc788\ub2e4. \ub458\uc9f8, \ub300\ud1b5\ub839\uc774 \uc9c0\uba85\ud558\ub294 \uc5f0\ubc29 \uace0\uc704 \uacf5\ubb34\uc6d0\uc5d0 \ub300\ud55c \uc2b9\uc778\uad8c\uc744 \uac00\uc9c4\ub2e4. \uc14b\uc9f8, \ubbf8\uad6d\uc774 \uccb4\uacb0\ud558\ub294 \uc870\uc57d\uc5d0 \ub300\ud55c \uc2b9\uc778\uad8c\uc774 \uc788\ub2e4. \ubc18\uba74, \uc138\uc785 \uc9d5\uc218\uc5d0 \uad00\ud55c \ubc95\ub960\uc548\uc740 \ud558\uc6d0\uc5d0\uc11c \uba3c\uc800 \uc81c\uc548\ub418\uc5b4\uc57c \ud55c\ub2e4. \ub610\ud55c \ud558\uc6d0\uc774 \uc5f0\ubc29 \uace0\uc704 \uacf5\ubb34\uc6d0\uc5d0 \ub300\ud55c \ud0c4\ud575 \uad8c\ud55c\uc744 \uc804\uc720\ud558\ub294\ub370 \ub300\ud574, \uc0c1\uc6d0\uc740 \ud0c4\ud575 \uc2ec\ud310\uad8c\uc744 \uc804\uc720\ud55c\ub2e4. \uc758\ud68c\uc5d0\ub294 \ud574\uc0b0\uc774 \uc5c6\uc73c\uba70, 2\ub144\ub9c8\ub2e4 \uc804\uc6d0 \uac1c\uc120\ub418\ub294 \ud558\uc6d0\uc758\uc6d0\uc758 \uc784\uae30\uc5d0 \ub9de\ucd94\uc5b4\uc11c \ud68c\uae30\uac00 \uacb0\uc815\ub41c\ub2e4. \uc758\uc6d0\ub4e4\uc740 \uc0c1\uc6d0 15\uac1c, \ud558\uc6d0 22\uac1c\uc758 \uc0c1\uc784\uc704\uc6d0\ud68c \uc911 \uc5b4\ub290 \uc704\uc6d0\ud68c\uc5d0 \uc18c\uc18d\ud558\uba70, \uc2ec\uc758\ub294 \uc704\uc6d0\ud68c \uc911\uc2ec\uc73c\ub85c \uc9c4\ud589\ub41c\ub2e4. \uc758\ud68c\ub294 \uc6cc\uc2f1\ud134 D.C.\uc758 \ubbf8\uad6d \uc758\ud68c\uc758\uc0ac\ub2f9\uc5d0\uc11c \uc5f4\ub9b0\ub2e4."} {"question": "\uad6d\uc815\uc6d0 \uc9c1\uc6d0\uc758 \uad6d\uc815\uc6d0\ubc95 \uc704\ubc18\uc73c\ub85c \uad00\ub828\uc790 \uc77c\ubd80\uac00 \uac80\ucc30\uc5d0 \uae30\uc18c\ub41c \ub0a0\uc740?", "paragraph": "\ubbfc\uc8fc\ud1b5\ud569\ub2f9\uc740 2012\ub144 12\uc6d4 11\uc77c \ub300\ud55c\ubbfc\uad6d \uad6d\uac00\uc815\ubcf4\uc6d0 \uc18c\uc18d \uc2ec\ub9ac\uc815\ubcf4\uad6d \uacf5\ubb34\uc6d0\ub4e4\uc774 \uad6d\uac00\uc815\ubcf4\uc6d0\uc758 \uc9c0\uc2dc\uc5d0 \ub530\ub77c \uc778\ud130\ub137\uc5d0 \uac8c\uc2dc\uae00\uc744 \ub0a8\uaca8\uc11c \uc120\uac70\uc5d0 \uc601\ud5a5\uc744 \ubbf8\uce58\uace0 \uc788\ub2e4\uace0 \uc8fc\uc7a5\ud588\ub2e4. \uc774\uc5b4\uc11c \ud574\ub2f9 \ud65c\ub3d9\uc5d0 \ucc38\uc5ec\ud55c \uad6d\uac00\uc815\ubcf4\uc6d0 \uc9c1\uc6d0\uc911 1\uba85\uc744 \uc9c0\ubaa9\ud558\uace0 \ubd88\ubc95\uc73c\ub85c \uc2a4\ud1a0\ud0b9 \ubc0f \uc0ac\ucc30, \uc624\ud53c\uc2a4\ud154 \uc810\uac70\ud589\uc704\ub97c \ubc8c\uc778\ub2e4. \uadf8\ub7ec\ub098 \ud574\ub2f9 \uc9c1\uc6d0\uc740 \uc624\ud53c\uc2a4\ud154\uc5d0\uc11c \ub098\uc624\uc9c0 \uc54a\uc558\uace0 \uadf8\ub7f0 \uc77c\uc744 \ud55c \uc801\uc774 \uc5c6\ub2e4\uace0 \uc8fc\uc7a5\ud558\uc600\ub2e4. \uacbd\ucc30\uc774 \uc218\uc0ac\uc5d0 \ub4e4\uc5b4\uac14\uc73c\ub098 \ubc15\uadfc\ud61c\uc640 \ubb38\uc7ac\uc778\uac04\uc758 \ub9c8\uc9c0\ub9c9 TV\ud1a0\ub860 \uc9c1\ud6c4\uc778 12\uc6d4 16\uc77c \uc77c\uc694\uc77c \uc624\ud6c4 11\uc2dc \uc99d\uac70\uac00 \ubc1c\uacac\ub418\uc9c0 \uc54a\uc558\ub2e4\uace0 \ubc1c\ud45c\ud558\uc600\uace0, \ubc15\uadfc\ud61c\uac00 \ub300\ud1b5\ub839\uc120\uac70\uc5d0\uc11c \uc2b9\ub9ac\ud558\uc600\ub2e4. \uadf8\ub7ec\ub098 \ubc15\uadfc\ud61c\uac00 \ucde8\uc784\ud55c \uc774\ud6c4 \ubb38\uc81c\uc758 \uc9c1\uc6d0\uc744 \ud3ec\ud568\ud55c \uad6d\uac00\uc815\ubcf4\uc6d0\uc758 \uc9c1\uc6d0\ub4e4\uc774 '\uc624\ub298\uc758 \uc720\uba38' \ub4f1\uc758 \uc0ac\uc774\ud2b8\uc5d0 \ub313\uae00 \uc791\uc131\uc744 \ud558\uc600\uc73c\uba70 \ub313\uae00 \ucd1d 224\uac1c\uc911\uc5d0 3\uac1c\uc758 \ub313\uae00\uc774 \ud2b9\uc815 \ub300\uc120 \ud6c4\ubcf4\uc5d0 \uc720\ub9ac\ud558\uac8c \ud65c\ub3d9\ud55c \uc0ac\uc2e4\uc774 \ud655\uc778\ub418\uc5c8\ub2e4. \uc774\ub294 \uad6d\uc815\uc6d0 \uc9c1\uc6d0\uc758 \uc815\uce58\uad00\uc5ec\ub97c \uae08\uc9c0\ud55c \uad6d\uc815\uc6d0\ubc95\uc744 \uc704\ubc18\ud55c \ud589\uc704\ub85c\uc11c, 2013\ub144 6\uc6d4 \uac80\ucc30\uc5d0 \uc758\ud558\uc5ec \uad00\ub828\uc790 \uc77c\ubd80\uac00 \uae30\uc18c\ub418\uc5c8\uace0, \ud604\uc7ac 1\uc2ec\uc774 \uc9c4\ud589 \uc911\uc774\ub2e4.", "answer": "2013\ub144 6\uc6d4", "sentence": "\uc774\ub294 \uad6d\uc815\uc6d0 \uc9c1\uc6d0\uc758 \uc815\uce58\uad00\uc5ec\ub97c \uae08\uc9c0\ud55c \uad6d\uc815\uc6d0\ubc95\uc744 \uc704\ubc18\ud55c \ud589\uc704\ub85c\uc11c, 2013\ub144 6\uc6d4 \uac80\ucc30\uc5d0 \uc758\ud558\uc5ec \uad00\ub828\uc790 \uc77c\ubd80\uac00 \uae30\uc18c\ub418\uc5c8\uace0, \ud604\uc7ac 1\uc2ec\uc774 \uc9c4\ud589 \uc911\uc774\ub2e4.", "paragraph_sentence": "\ubbfc\uc8fc\ud1b5\ud569\ub2f9\uc740 2012\ub144 12\uc6d4 11\uc77c \ub300\ud55c\ubbfc\uad6d \uad6d\uac00\uc815\ubcf4\uc6d0 \uc18c\uc18d \uc2ec\ub9ac\uc815\ubcf4\uad6d \uacf5\ubb34\uc6d0\ub4e4\uc774 \uad6d\uac00\uc815\ubcf4\uc6d0\uc758 \uc9c0\uc2dc\uc5d0 \ub530\ub77c \uc778\ud130\ub137\uc5d0 \uac8c\uc2dc\uae00\uc744 \ub0a8\uaca8\uc11c \uc120\uac70\uc5d0 \uc601\ud5a5\uc744 \ubbf8\uce58\uace0 \uc788\ub2e4\uace0 \uc8fc\uc7a5\ud588\ub2e4. \uc774\uc5b4\uc11c \ud574\ub2f9 \ud65c\ub3d9\uc5d0 \ucc38\uc5ec\ud55c \uad6d\uac00\uc815\ubcf4\uc6d0 \uc9c1\uc6d0\uc911 1\uba85\uc744 \uc9c0\ubaa9\ud558\uace0 \ubd88\ubc95\uc73c\ub85c \uc2a4\ud1a0\ud0b9 \ubc0f \uc0ac\ucc30, \uc624\ud53c\uc2a4\ud154 \uc810\uac70\ud589\uc704\ub97c \ubc8c\uc778\ub2e4. \uadf8\ub7ec\ub098 \ud574\ub2f9 \uc9c1\uc6d0\uc740 \uc624\ud53c\uc2a4\ud154\uc5d0\uc11c \ub098\uc624\uc9c0 \uc54a\uc558\uace0 \uadf8\ub7f0 \uc77c\uc744 \ud55c \uc801\uc774 \uc5c6\ub2e4\uace0 \uc8fc\uc7a5\ud558\uc600\ub2e4. \uacbd\ucc30\uc774 \uc218\uc0ac\uc5d0 \ub4e4\uc5b4\uac14\uc73c\ub098 \ubc15\uadfc\ud61c\uc640 \ubb38\uc7ac\uc778\uac04\uc758 \ub9c8\uc9c0\ub9c9 TV\ud1a0\ub860 \uc9c1\ud6c4\uc778 12\uc6d4 16\uc77c \uc77c\uc694\uc77c \uc624\ud6c4 11\uc2dc \uc99d\uac70\uac00 \ubc1c\uacac\ub418\uc9c0 \uc54a\uc558\ub2e4\uace0 \ubc1c\ud45c\ud558\uc600\uace0, \ubc15\uadfc\ud61c\uac00 \ub300\ud1b5\ub839\uc120\uac70\uc5d0\uc11c \uc2b9\ub9ac\ud558\uc600\ub2e4. \uadf8\ub7ec\ub098 \ubc15\uadfc\ud61c\uac00 \ucde8\uc784\ud55c \uc774\ud6c4 \ubb38\uc81c\uc758 \uc9c1\uc6d0\uc744 \ud3ec\ud568\ud55c \uad6d\uac00\uc815\ubcf4\uc6d0\uc758 \uc9c1\uc6d0\ub4e4\uc774 '\uc624\ub298\uc758 \uc720\uba38' \ub4f1\uc758 \uc0ac\uc774\ud2b8\uc5d0 \ub313\uae00 \uc791\uc131\uc744 \ud558\uc600\uc73c\uba70 \ub313\uae00 \ucd1d 224\uac1c\uc911\uc5d0 3\uac1c\uc758 \ub313\uae00\uc774 \ud2b9\uc815 \ub300\uc120 \ud6c4\ubcf4\uc5d0 \uc720\ub9ac\ud558\uac8c \ud65c\ub3d9\ud55c \uc0ac\uc2e4\uc774 \ud655\uc778\ub418\uc5c8\ub2e4. \uc774\ub294 \uad6d\uc815\uc6d0 \uc9c1\uc6d0\uc758 \uc815\uce58\uad00\uc5ec\ub97c \uae08\uc9c0\ud55c \uad6d\uc815\uc6d0\ubc95\uc744 \uc704\ubc18\ud55c \ud589\uc704\ub85c\uc11c, 2013\ub144 6\uc6d4 \uac80\ucc30\uc5d0 \uc758\ud558\uc5ec \uad00\ub828\uc790 \uc77c\ubd80\uac00 \uae30\uc18c\ub418\uc5c8\uace0, \ud604\uc7ac 1\uc2ec\uc774 \uc9c4\ud589 \uc911\uc774\ub2e4. ", "paragraph_answer": "\ubbfc\uc8fc\ud1b5\ud569\ub2f9\uc740 2012\ub144 12\uc6d4 11\uc77c \ub300\ud55c\ubbfc\uad6d \uad6d\uac00\uc815\ubcf4\uc6d0 \uc18c\uc18d \uc2ec\ub9ac\uc815\ubcf4\uad6d \uacf5\ubb34\uc6d0\ub4e4\uc774 \uad6d\uac00\uc815\ubcf4\uc6d0\uc758 \uc9c0\uc2dc\uc5d0 \ub530\ub77c \uc778\ud130\ub137\uc5d0 \uac8c\uc2dc\uae00\uc744 \ub0a8\uaca8\uc11c \uc120\uac70\uc5d0 \uc601\ud5a5\uc744 \ubbf8\uce58\uace0 \uc788\ub2e4\uace0 \uc8fc\uc7a5\ud588\ub2e4. \uc774\uc5b4\uc11c \ud574\ub2f9 \ud65c\ub3d9\uc5d0 \ucc38\uc5ec\ud55c \uad6d\uac00\uc815\ubcf4\uc6d0 \uc9c1\uc6d0\uc911 1\uba85\uc744 \uc9c0\ubaa9\ud558\uace0 \ubd88\ubc95\uc73c\ub85c \uc2a4\ud1a0\ud0b9 \ubc0f \uc0ac\ucc30, \uc624\ud53c\uc2a4\ud154 \uc810\uac70\ud589\uc704\ub97c \ubc8c\uc778\ub2e4. \uadf8\ub7ec\ub098 \ud574\ub2f9 \uc9c1\uc6d0\uc740 \uc624\ud53c\uc2a4\ud154\uc5d0\uc11c \ub098\uc624\uc9c0 \uc54a\uc558\uace0 \uadf8\ub7f0 \uc77c\uc744 \ud55c \uc801\uc774 \uc5c6\ub2e4\uace0 \uc8fc\uc7a5\ud558\uc600\ub2e4. \uacbd\ucc30\uc774 \uc218\uc0ac\uc5d0 \ub4e4\uc5b4\uac14\uc73c\ub098 \ubc15\uadfc\ud61c\uc640 \ubb38\uc7ac\uc778\uac04\uc758 \ub9c8\uc9c0\ub9c9 TV\ud1a0\ub860 \uc9c1\ud6c4\uc778 12\uc6d4 16\uc77c \uc77c\uc694\uc77c \uc624\ud6c4 11\uc2dc \uc99d\uac70\uac00 \ubc1c\uacac\ub418\uc9c0 \uc54a\uc558\ub2e4\uace0 \ubc1c\ud45c\ud558\uc600\uace0, \ubc15\uadfc\ud61c\uac00 \ub300\ud1b5\ub839\uc120\uac70\uc5d0\uc11c \uc2b9\ub9ac\ud558\uc600\ub2e4. \uadf8\ub7ec\ub098 \ubc15\uadfc\ud61c\uac00 \ucde8\uc784\ud55c \uc774\ud6c4 \ubb38\uc81c\uc758 \uc9c1\uc6d0\uc744 \ud3ec\ud568\ud55c \uad6d\uac00\uc815\ubcf4\uc6d0\uc758 \uc9c1\uc6d0\ub4e4\uc774 '\uc624\ub298\uc758 \uc720\uba38' \ub4f1\uc758 \uc0ac\uc774\ud2b8\uc5d0 \ub313\uae00 \uc791\uc131\uc744 \ud558\uc600\uc73c\uba70 \ub313\uae00 \ucd1d 224\uac1c\uc911\uc5d0 3\uac1c\uc758 \ub313\uae00\uc774 \ud2b9\uc815 \ub300\uc120 \ud6c4\ubcf4\uc5d0 \uc720\ub9ac\ud558\uac8c \ud65c\ub3d9\ud55c \uc0ac\uc2e4\uc774 \ud655\uc778\ub418\uc5c8\ub2e4. \uc774\ub294 \uad6d\uc815\uc6d0 \uc9c1\uc6d0\uc758 \uc815\uce58\uad00\uc5ec\ub97c \uae08\uc9c0\ud55c \uad6d\uc815\uc6d0\ubc95\uc744 \uc704\ubc18\ud55c \ud589\uc704\ub85c\uc11c, 2013\ub144 6\uc6d4 \uac80\ucc30\uc5d0 \uc758\ud558\uc5ec \uad00\ub828\uc790 \uc77c\ubd80\uac00 \uae30\uc18c\ub418\uc5c8\uace0, \ud604\uc7ac 1\uc2ec\uc774 \uc9c4\ud589 \uc911\uc774\ub2e4.", "sentence_answer": "\uc774\ub294 \uad6d\uc815\uc6d0 \uc9c1\uc6d0\uc758 \uc815\uce58\uad00\uc5ec\ub97c \uae08\uc9c0\ud55c \uad6d\uc815\uc6d0\ubc95\uc744 \uc704\ubc18\ud55c \ud589\uc704\ub85c\uc11c, 2013\ub144 6\uc6d4 \uac80\ucc30\uc5d0 \uc758\ud558\uc5ec \uad00\ub828\uc790 \uc77c\ubd80\uac00 \uae30\uc18c\ub418\uc5c8\uace0, \ud604\uc7ac 1\uc2ec\uc774 \uc9c4\ud589 \uc911\uc774\ub2e4.", "paragraph_id": "6656873-2-2", "paragraph_question": "question: \uad6d\uc815\uc6d0 \uc9c1\uc6d0\uc758 \uad6d\uc815\uc6d0\ubc95 \uc704\ubc18\uc73c\ub85c \uad00\ub828\uc790 \uc77c\ubd80\uac00 \uac80\ucc30\uc5d0 \uae30\uc18c\ub41c \ub0a0\uc740?, context: \ubbfc\uc8fc\ud1b5\ud569\ub2f9\uc740 2012\ub144 12\uc6d4 11\uc77c \ub300\ud55c\ubbfc\uad6d \uad6d\uac00\uc815\ubcf4\uc6d0 \uc18c\uc18d \uc2ec\ub9ac\uc815\ubcf4\uad6d \uacf5\ubb34\uc6d0\ub4e4\uc774 \uad6d\uac00\uc815\ubcf4\uc6d0\uc758 \uc9c0\uc2dc\uc5d0 \ub530\ub77c \uc778\ud130\ub137\uc5d0 \uac8c\uc2dc\uae00\uc744 \ub0a8\uaca8\uc11c \uc120\uac70\uc5d0 \uc601\ud5a5\uc744 \ubbf8\uce58\uace0 \uc788\ub2e4\uace0 \uc8fc\uc7a5\ud588\ub2e4. \uc774\uc5b4\uc11c \ud574\ub2f9 \ud65c\ub3d9\uc5d0 \ucc38\uc5ec\ud55c \uad6d\uac00\uc815\ubcf4\uc6d0 \uc9c1\uc6d0\uc911 1\uba85\uc744 \uc9c0\ubaa9\ud558\uace0 \ubd88\ubc95\uc73c\ub85c \uc2a4\ud1a0\ud0b9 \ubc0f \uc0ac\ucc30, \uc624\ud53c\uc2a4\ud154 \uc810\uac70\ud589\uc704\ub97c \ubc8c\uc778\ub2e4. \uadf8\ub7ec\ub098 \ud574\ub2f9 \uc9c1\uc6d0\uc740 \uc624\ud53c\uc2a4\ud154\uc5d0\uc11c \ub098\uc624\uc9c0 \uc54a\uc558\uace0 \uadf8\ub7f0 \uc77c\uc744 \ud55c \uc801\uc774 \uc5c6\ub2e4\uace0 \uc8fc\uc7a5\ud558\uc600\ub2e4. \uacbd\ucc30\uc774 \uc218\uc0ac\uc5d0 \ub4e4\uc5b4\uac14\uc73c\ub098 \ubc15\uadfc\ud61c\uc640 \ubb38\uc7ac\uc778\uac04\uc758 \ub9c8\uc9c0\ub9c9 TV\ud1a0\ub860 \uc9c1\ud6c4\uc778 12\uc6d4 16\uc77c \uc77c\uc694\uc77c \uc624\ud6c4 11\uc2dc \uc99d\uac70\uac00 \ubc1c\uacac\ub418\uc9c0 \uc54a\uc558\ub2e4\uace0 \ubc1c\ud45c\ud558\uc600\uace0, \ubc15\uadfc\ud61c\uac00 \ub300\ud1b5\ub839\uc120\uac70\uc5d0\uc11c \uc2b9\ub9ac\ud558\uc600\ub2e4. \uadf8\ub7ec\ub098 \ubc15\uadfc\ud61c\uac00 \ucde8\uc784\ud55c \uc774\ud6c4 \ubb38\uc81c\uc758 \uc9c1\uc6d0\uc744 \ud3ec\ud568\ud55c \uad6d\uac00\uc815\ubcf4\uc6d0\uc758 \uc9c1\uc6d0\ub4e4\uc774 '\uc624\ub298\uc758 \uc720\uba38' \ub4f1\uc758 \uc0ac\uc774\ud2b8\uc5d0 \ub313\uae00 \uc791\uc131\uc744 \ud558\uc600\uc73c\uba70 \ub313\uae00 \ucd1d 224\uac1c\uc911\uc5d0 3\uac1c\uc758 \ub313\uae00\uc774 \ud2b9\uc815 \ub300\uc120 \ud6c4\ubcf4\uc5d0 \uc720\ub9ac\ud558\uac8c \ud65c\ub3d9\ud55c \uc0ac\uc2e4\uc774 \ud655\uc778\ub418\uc5c8\ub2e4. \uc774\ub294 \uad6d\uc815\uc6d0 \uc9c1\uc6d0\uc758 \uc815\uce58\uad00\uc5ec\ub97c \uae08\uc9c0\ud55c \uad6d\uc815\uc6d0\ubc95\uc744 \uc704\ubc18\ud55c \ud589\uc704\ub85c\uc11c, 2013\ub144 6\uc6d4 \uac80\ucc30\uc5d0 \uc758\ud558\uc5ec \uad00\ub828\uc790 \uc77c\ubd80\uac00 \uae30\uc18c\ub418\uc5c8\uace0, \ud604\uc7ac 1\uc2ec\uc774 \uc9c4\ud589 \uc911\uc774\ub2e4."} {"question": "\ud55c\uad6d\uc774 PSI \ud611\ub825\ubc29\uc548\uc758 \ud611\uc870\ubc29\uce68\uc744 \uacb0\uc815\ud574 \ub450\uace0\ub3c4 \uc774\ub97c \uc26c\uc26c\ud588\uc744 \ub54c \uc774 \uc0ac\uc2e4\uc744 \ud3ed\ub85c\ud55c \ub2f9\uc2dc \uc5f4\ub9b0\uc6b0\ub9ac\ub2f9 \uc758\uc6d0\uc740?", "paragraph": "\ud55c\uad6d\uc774 PSI\uc5d0 \ucc38\uc5ec\ud560 \uac83\uc774\ub0d0 \ub9d0 \uac83\uc774\ub0d0, \ucc38\uc5ec\ud55c\ub2e4\uba74 \uc5b4\ub290 \uc815\ub3c4 \ucc38\uc5ec\ud560 \uac83\uc774\ub0d0 \ud558\ub294 \ubb38\uc81c\ub294 \ud06c\uac8c \uc138 \uac00\uc9c0 \uc694\uc18c \ub54c\ubb38\uc5d0 \ub17c\ub780\uc774 \ub41c\ub2e4. \uccab \ubc88\uc9f8\ub294 \ud55c\ubc18\ub3c4 \uc8fc\ubcc0\uc5d0\uc11c\uc758 \uad70\uc0ac\uc801 \ucda9\ub3cc \uac1c\uc5f0\uc131, \ub450 \ubc88\uc9f8\ub294 \uad6d\uc81c\ubc95\uc801\uc778 \uce21\uba74, \ub9c8\uc9c0\ub9c9\uc73c\ub85c\ub294 \ud55c-\ubbf8 \ub3d9\ub9f9\uc758 \ubb38\uc81c\ub2e4. 2006\ub144\uc758 \uc0c1\ud669\uc744 \uc694\uc57d\ud574 \ubcf4\uc790\uba74, \ud55c\uad6d\uc740 2005\ub144 \ubbf8\uad6d\uc73c\ub85c\ubd80\ud130 PSI\uc5d0 \ucc38\uc5ec\ud558\ub77c\ub294 \uc555\ubc15\uc744 \ubc1b\uace0 \uc788\uc5c8\ub2e4. \ubd81\ud55c\uc744 \uc758\uc2dd\ud558\uc9c0 \uc54a\uc744 \uc218 \uc5c6\uc5c8\ub358 \ud55c\uad6d\uc740 2005\ub144 12\uc6d4 29\uc77c PSI 8\uac1c \ud611\ub825\ubc29\uc548 \uac00\uc6b4\ub370 5\uac1c \ubd84\uc57c \ud611\uc870\ubc29\uce68\uc744 \uacb0\uc815\ud574 \ub193\uace0\ub3c4 \uc774\ub97c \uc26c\uc26c \ud588\uc73c\uba70 \ub2f9\uc2dc \uc5f4\ub9b0\uc6b0\ub9ac\ub2f9 \ucd5c\uc7ac\ucc9c \uc758\uc6d0\uc774 \uc774 \uac19\uc740 \uc0ac\uc2e4\uc744 \ud3ed\ub85c\ud558\uc790 \uc678\uad50\ud1b5\uc0c1\ubd80\ub294 2006\ub144 2\uc6d424\uc77c \uc774 \uac19\uc740 \uc0ac\uc2e4\uc744 \ubc1d\ud614\ub2e4. 2005\ub144 \ub9d0 \ud55c\uad6d \uc815\ubd80\uac00 \uacb0\uc815\ud55c PSI \ucc38\uc5ec \ubc94\uc704\ub294 \ub2e4\uc74c\uacfc \uac19\ub2e4.", "answer": "\ucd5c\uc7ac\ucc9c", "sentence": "\ubd81\ud55c\uc744 \uc758\uc2dd\ud558\uc9c0 \uc54a\uc744 \uc218 \uc5c6\uc5c8\ub358 \ud55c\uad6d\uc740 2005\ub144 12\uc6d4 29\uc77c PSI 8\uac1c \ud611\ub825\ubc29\uc548 \uac00\uc6b4\ub370 5\uac1c \ubd84\uc57c \ud611\uc870\ubc29\uce68\uc744 \uacb0\uc815\ud574 \ub193\uace0\ub3c4 \uc774\ub97c \uc26c\uc26c \ud588\uc73c\uba70 \ub2f9\uc2dc \uc5f4\ub9b0\uc6b0\ub9ac\ub2f9 \ucd5c\uc7ac\ucc9c \uc758\uc6d0\uc774 \uc774 \uac19\uc740 \uc0ac\uc2e4\uc744 \ud3ed\ub85c\ud558\uc790 \uc678\uad50\ud1b5\uc0c1\ubd80\ub294 2006\ub144 2\uc6d424\uc77c \uc774 \uac19\uc740 \uc0ac\uc2e4\uc744 \ubc1d\ud614\ub2e4.", "paragraph_sentence": "\ud55c\uad6d\uc774 PSI\uc5d0 \ucc38\uc5ec\ud560 \uac83\uc774\ub0d0 \ub9d0 \uac83\uc774\ub0d0, \ucc38\uc5ec\ud55c\ub2e4\uba74 \uc5b4\ub290 \uc815\ub3c4 \ucc38\uc5ec\ud560 \uac83\uc774\ub0d0 \ud558\ub294 \ubb38\uc81c\ub294 \ud06c\uac8c \uc138 \uac00\uc9c0 \uc694\uc18c \ub54c\ubb38\uc5d0 \ub17c\ub780\uc774 \ub41c\ub2e4. \uccab \ubc88\uc9f8\ub294 \ud55c\ubc18\ub3c4 \uc8fc\ubcc0\uc5d0\uc11c\uc758 \uad70\uc0ac\uc801 \ucda9\ub3cc \uac1c\uc5f0\uc131, \ub450 \ubc88\uc9f8\ub294 \uad6d\uc81c\ubc95\uc801\uc778 \uce21\uba74, \ub9c8\uc9c0\ub9c9\uc73c\ub85c\ub294 \ud55c-\ubbf8 \ub3d9\ub9f9\uc758 \ubb38\uc81c\ub2e4. 2006\ub144\uc758 \uc0c1\ud669\uc744 \uc694\uc57d\ud574 \ubcf4\uc790\uba74, \ud55c\uad6d\uc740 2005\ub144 \ubbf8\uad6d\uc73c\ub85c\ubd80\ud130 PSI\uc5d0 \ucc38\uc5ec\ud558\ub77c\ub294 \uc555\ubc15\uc744 \ubc1b\uace0 \uc788\uc5c8\ub2e4. \ubd81\ud55c\uc744 \uc758\uc2dd\ud558\uc9c0 \uc54a\uc744 \uc218 \uc5c6\uc5c8\ub358 \ud55c\uad6d\uc740 2005\ub144 12\uc6d4 29\uc77c PSI 8\uac1c \ud611\ub825\ubc29\uc548 \uac00\uc6b4\ub370 5\uac1c \ubd84\uc57c \ud611\uc870\ubc29\uce68\uc744 \uacb0\uc815\ud574 \ub193\uace0\ub3c4 \uc774\ub97c \uc26c\uc26c \ud588\uc73c\uba70 \ub2f9\uc2dc \uc5f4\ub9b0\uc6b0\ub9ac\ub2f9 \ucd5c\uc7ac\ucc9c \uc758\uc6d0\uc774 \uc774 \uac19\uc740 \uc0ac\uc2e4\uc744 \ud3ed\ub85c\ud558\uc790 \uc678\uad50\ud1b5\uc0c1\ubd80\ub294 2006\ub144 2\uc6d424\uc77c \uc774 \uac19\uc740 \uc0ac\uc2e4\uc744 \ubc1d\ud614\ub2e4. 2005\ub144 \ub9d0 \ud55c\uad6d \uc815\ubd80\uac00 \uacb0\uc815\ud55c PSI \ucc38\uc5ec \ubc94\uc704\ub294 \ub2e4\uc74c\uacfc \uac19\ub2e4.", "paragraph_answer": "\ud55c\uad6d\uc774 PSI\uc5d0 \ucc38\uc5ec\ud560 \uac83\uc774\ub0d0 \ub9d0 \uac83\uc774\ub0d0, \ucc38\uc5ec\ud55c\ub2e4\uba74 \uc5b4\ub290 \uc815\ub3c4 \ucc38\uc5ec\ud560 \uac83\uc774\ub0d0 \ud558\ub294 \ubb38\uc81c\ub294 \ud06c\uac8c \uc138 \uac00\uc9c0 \uc694\uc18c \ub54c\ubb38\uc5d0 \ub17c\ub780\uc774 \ub41c\ub2e4. \uccab \ubc88\uc9f8\ub294 \ud55c\ubc18\ub3c4 \uc8fc\ubcc0\uc5d0\uc11c\uc758 \uad70\uc0ac\uc801 \ucda9\ub3cc \uac1c\uc5f0\uc131, \ub450 \ubc88\uc9f8\ub294 \uad6d\uc81c\ubc95\uc801\uc778 \uce21\uba74, \ub9c8\uc9c0\ub9c9\uc73c\ub85c\ub294 \ud55c-\ubbf8 \ub3d9\ub9f9\uc758 \ubb38\uc81c\ub2e4. 2006\ub144\uc758 \uc0c1\ud669\uc744 \uc694\uc57d\ud574 \ubcf4\uc790\uba74, \ud55c\uad6d\uc740 2005\ub144 \ubbf8\uad6d\uc73c\ub85c\ubd80\ud130 PSI\uc5d0 \ucc38\uc5ec\ud558\ub77c\ub294 \uc555\ubc15\uc744 \ubc1b\uace0 \uc788\uc5c8\ub2e4. \ubd81\ud55c\uc744 \uc758\uc2dd\ud558\uc9c0 \uc54a\uc744 \uc218 \uc5c6\uc5c8\ub358 \ud55c\uad6d\uc740 2005\ub144 12\uc6d4 29\uc77c PSI 8\uac1c \ud611\ub825\ubc29\uc548 \uac00\uc6b4\ub370 5\uac1c \ubd84\uc57c \ud611\uc870\ubc29\uce68\uc744 \uacb0\uc815\ud574 \ub193\uace0\ub3c4 \uc774\ub97c \uc26c\uc26c \ud588\uc73c\uba70 \ub2f9\uc2dc \uc5f4\ub9b0\uc6b0\ub9ac\ub2f9 \ucd5c\uc7ac\ucc9c \uc758\uc6d0\uc774 \uc774 \uac19\uc740 \uc0ac\uc2e4\uc744 \ud3ed\ub85c\ud558\uc790 \uc678\uad50\ud1b5\uc0c1\ubd80\ub294 2006\ub144 2\uc6d424\uc77c \uc774 \uac19\uc740 \uc0ac\uc2e4\uc744 \ubc1d\ud614\ub2e4. 2005\ub144 \ub9d0 \ud55c\uad6d \uc815\ubd80\uac00 \uacb0\uc815\ud55c PSI \ucc38\uc5ec \ubc94\uc704\ub294 \ub2e4\uc74c\uacfc \uac19\ub2e4.", "sentence_answer": "\ubd81\ud55c\uc744 \uc758\uc2dd\ud558\uc9c0 \uc54a\uc744 \uc218 \uc5c6\uc5c8\ub358 \ud55c\uad6d\uc740 2005\ub144 12\uc6d4 29\uc77c PSI 8\uac1c \ud611\ub825\ubc29\uc548 \uac00\uc6b4\ub370 5\uac1c \ubd84\uc57c \ud611\uc870\ubc29\uce68\uc744 \uacb0\uc815\ud574 \ub193\uace0\ub3c4 \uc774\ub97c \uc26c\uc26c \ud588\uc73c\uba70 \ub2f9\uc2dc \uc5f4\ub9b0\uc6b0\ub9ac\ub2f9 \ucd5c\uc7ac\ucc9c \uc758\uc6d0\uc774 \uc774 \uac19\uc740 \uc0ac\uc2e4\uc744 \ud3ed\ub85c\ud558\uc790 \uc678\uad50\ud1b5\uc0c1\ubd80\ub294 2006\ub144 2\uc6d424\uc77c \uc774 \uac19\uc740 \uc0ac\uc2e4\uc744 \ubc1d\ud614\ub2e4.", "paragraph_id": "6490854-6-1", "paragraph_question": "question: \ud55c\uad6d\uc774 PSI \ud611\ub825\ubc29\uc548\uc758 \ud611\uc870\ubc29\uce68\uc744 \uacb0\uc815\ud574 \ub450\uace0\ub3c4 \uc774\ub97c \uc26c\uc26c\ud588\uc744 \ub54c \uc774 \uc0ac\uc2e4\uc744 \ud3ed\ub85c\ud55c \ub2f9\uc2dc \uc5f4\ub9b0\uc6b0\ub9ac\ub2f9 \uc758\uc6d0\uc740?, context: \ud55c\uad6d\uc774 PSI\uc5d0 \ucc38\uc5ec\ud560 \uac83\uc774\ub0d0 \ub9d0 \uac83\uc774\ub0d0, \ucc38\uc5ec\ud55c\ub2e4\uba74 \uc5b4\ub290 \uc815\ub3c4 \ucc38\uc5ec\ud560 \uac83\uc774\ub0d0 \ud558\ub294 \ubb38\uc81c\ub294 \ud06c\uac8c \uc138 \uac00\uc9c0 \uc694\uc18c \ub54c\ubb38\uc5d0 \ub17c\ub780\uc774 \ub41c\ub2e4. \uccab \ubc88\uc9f8\ub294 \ud55c\ubc18\ub3c4 \uc8fc\ubcc0\uc5d0\uc11c\uc758 \uad70\uc0ac\uc801 \ucda9\ub3cc \uac1c\uc5f0\uc131, \ub450 \ubc88\uc9f8\ub294 \uad6d\uc81c\ubc95\uc801\uc778 \uce21\uba74, \ub9c8\uc9c0\ub9c9\uc73c\ub85c\ub294 \ud55c-\ubbf8 \ub3d9\ub9f9\uc758 \ubb38\uc81c\ub2e4. 2006\ub144\uc758 \uc0c1\ud669\uc744 \uc694\uc57d\ud574 \ubcf4\uc790\uba74, \ud55c\uad6d\uc740 2005\ub144 \ubbf8\uad6d\uc73c\ub85c\ubd80\ud130 PSI\uc5d0 \ucc38\uc5ec\ud558\ub77c\ub294 \uc555\ubc15\uc744 \ubc1b\uace0 \uc788\uc5c8\ub2e4. \ubd81\ud55c\uc744 \uc758\uc2dd\ud558\uc9c0 \uc54a\uc744 \uc218 \uc5c6\uc5c8\ub358 \ud55c\uad6d\uc740 2005\ub144 12\uc6d4 29\uc77c PSI 8\uac1c \ud611\ub825\ubc29\uc548 \uac00\uc6b4\ub370 5\uac1c \ubd84\uc57c \ud611\uc870\ubc29\uce68\uc744 \uacb0\uc815\ud574 \ub193\uace0\ub3c4 \uc774\ub97c \uc26c\uc26c \ud588\uc73c\uba70 \ub2f9\uc2dc \uc5f4\ub9b0\uc6b0\ub9ac\ub2f9 \ucd5c\uc7ac\ucc9c \uc758\uc6d0\uc774 \uc774 \uac19\uc740 \uc0ac\uc2e4\uc744 \ud3ed\ub85c\ud558\uc790 \uc678\uad50\ud1b5\uc0c1\ubd80\ub294 2006\ub144 2\uc6d424\uc77c \uc774 \uac19\uc740 \uc0ac\uc2e4\uc744 \ubc1d\ud614\ub2e4. 2005\ub144 \ub9d0 \ud55c\uad6d \uc815\ubd80\uac00 \uacb0\uc815\ud55c PSI \ucc38\uc5ec \ubc94\uc704\ub294 \ub2e4\uc74c\uacfc \uac19\ub2e4."} {"question": "\ub85c\ud2b8\ub108\uac00 \ud2b8\uc640\uc77c\ub77c\uc787\uc758 \uc624\ub514\uc158\uc744 \ubcf8\uac83\uc740 \uc5b8\uc81c\uc778\uac00?", "paragraph": "2007\ub144, \uc601\ud654 \uc81c\uc791\uc790\ub4e4\uc740 \u300a\ud2b8\uc640\uc77c\ub77c\uc787 \uc601\ud654 \uc2dc\ub9ac\uc988\u300b\uc758 \uccab \ubc88\uc9f8 \uc791\ud488\uc778 \u300a\ud2b8\uc640\uc77c\ub77c\uc787\u300b\uc5d0\uc11c \ubca8\ub77c \uc2a4\uc644\uc758 \uc6d0\uc8fc\ubbfc \uce5c\uad6c \uc81c\uc774\ucf65 \ube14\ub799\uc744 \ub9e1\uc744 \ubc30\uc6b0\ub97c \ucc3e\uace0 \uc788\uc5c8\ub2e4. 2008\ub144 1\uc6d4, \ud3ec\ud2c0\ub79c\ub4dc\uc5d0\uc11c \uc624\ub514\uc158\uc774 \uc5f4\ub838\uace0, \ub85c\ud2b8\ub108\ub294 \u300a\ud2b8\uc640\uc77c\ub77c\uc787\u300b\uc5d0 \ub300\ud574 \uc804\ud600 \ubab0\ub790\uc9c0\ub9cc \uae30\ud68d\uc0ac\uc5d0 \uc758\ud574 \uc624\ub514\uc158\uc744 \ubcf4\uac8c \ub418\uc5c8\ub2e4. \uc624\ub514\uc158\uc5d0\uc11c \uc774\ubbf8 \ubca8\ub77c \uc5ed\uc73c\ub85c \uce90\uc2a4\ud305\ub41c \ud06c\ub9ac\uc2a4\ud2f4 \uc2a4\ud29c\uc5b4\ud2b8\uc640 \ub300\uc0ac\ub97c \uc77d\uc5c8\uace0 \uadf8\ub4e4\uc740 \u300a\ub274\ubb38\u300b, \u300a\uc774\ud074\ub9bd\uc2a4\u300b\uc758 \uc7a5\uba74\ub4e4\uc744 \uc5f0\uae30\ud558\uc600\ub2e4. \uacb0\uacfc\ub294 \uc81c\uc774\ucf65 \ube14\ub799 \uc5ed\uc73c\ub85c \uce90\uc2a4\ud305\ub418\uc5c8\uace0, \uc601\ud654\ub294 2008\ub144\uc5d0 \uac1c\ubd09\ud558\uc600\ub2e4. \uc601\ud654\ub294 \uc131\uacf5\uc801\uc774\uc600\uace0, \uccab \uc8fc\uc5d0 6\ucc9c 9\ubc31\ub9cc\ub2ec\ub7ec\ub97c \ubc8c\uc5c8\uc73c\uba70, \uc804\uc138\uacc4\ub85c\ub294 3\uc5b5 9\ucc9c 2\ubc31\ub9cc\ub2ec\ub7ec\uc758 \uc218\uc785\uc744 \ub0c8\ub2e4. \ub85c\ud2bc \ud1a0\ub9c8\ud1a0\uc5d0\uc11c\ub294 10\uc810 \uc911 5.5\uc810\uc744 \ud68d\ub4dd\ud558\uc600\uace0, \uba54\ud0c0\ud06c\ub9ac\ud2f1\uc5d0\uc11c\ub3c4 \ud3c9\uade0\uc801\uc778 \ud3c9\uc744 \ubc1b\uc558\ub2e4. 2009\ub144 MTV \ubb34\ube44 \uc5b4\uc6cc\ub4dc\uc5d0\uc11c \ub85c\ud2b8\ub108\ub294 \u300a\ud2b8\uc640\uc77c\ub77c\uc787\u300b\uc5d0 \uac19\uc774 \ucd9c\uc5f0\ud55c \ub85c\ubc84\ud2b8 \ud328\ud2f4\uc2a8\uacfc \ub0a8\uc6b0\uc2e0\uc778\uc0c1\uc5d0 \ud6c4\ubcf4\ub85c \uc62c\ub790\uc73c\ub098, \ud328\ud2f4\uc2a8\uc774 \uc218\uc0c1\ud558\uc600\ub2e4.", "answer": "2008\ub144 1\uc6d4", "sentence": "2008\ub144 1\uc6d4 ,", "paragraph_sentence": "2007\ub144, \uc601\ud654 \uc81c\uc791\uc790\ub4e4\uc740 \u300a\ud2b8\uc640\uc77c\ub77c\uc787 \uc601\ud654 \uc2dc\ub9ac\uc988\u300b\uc758 \uccab \ubc88\uc9f8 \uc791\ud488\uc778 \u300a\ud2b8\uc640\uc77c\ub77c\uc787\u300b\uc5d0\uc11c \ubca8\ub77c \uc2a4\uc644\uc758 \uc6d0\uc8fc\ubbfc \uce5c\uad6c \uc81c\uc774\ucf65 \ube14\ub799\uc744 \ub9e1\uc744 \ubc30\uc6b0\ub97c \ucc3e\uace0 \uc788\uc5c8\ub2e4. 2008\ub144 1\uc6d4 , \ud3ec\ud2c0\ub79c\ub4dc\uc5d0\uc11c \uc624\ub514\uc158\uc774 \uc5f4\ub838\uace0, \ub85c\ud2b8\ub108\ub294 \u300a\ud2b8\uc640\uc77c\ub77c\uc787\u300b\uc5d0 \ub300\ud574 \uc804\ud600 \ubab0\ub790\uc9c0\ub9cc \uae30\ud68d\uc0ac\uc5d0 \uc758\ud574 \uc624\ub514\uc158\uc744 \ubcf4\uac8c \ub418\uc5c8\ub2e4. \uc624\ub514\uc158\uc5d0\uc11c \uc774\ubbf8 \ubca8\ub77c \uc5ed\uc73c\ub85c \uce90\uc2a4\ud305\ub41c \ud06c\ub9ac\uc2a4\ud2f4 \uc2a4\ud29c\uc5b4\ud2b8\uc640 \ub300\uc0ac\ub97c \uc77d\uc5c8\uace0 \uadf8\ub4e4\uc740 \u300a\ub274\ubb38\u300b, \u300a\uc774\ud074\ub9bd\uc2a4\u300b\uc758 \uc7a5\uba74\ub4e4\uc744 \uc5f0\uae30\ud558\uc600\ub2e4. \uacb0\uacfc\ub294 \uc81c\uc774\ucf65 \ube14\ub799 \uc5ed\uc73c\ub85c \uce90\uc2a4\ud305\ub418\uc5c8\uace0, \uc601\ud654\ub294 2008\ub144\uc5d0 \uac1c\ubd09\ud558\uc600\ub2e4. \uc601\ud654\ub294 \uc131\uacf5\uc801\uc774\uc600\uace0, \uccab \uc8fc\uc5d0 6\ucc9c 9\ubc31\ub9cc\ub2ec\ub7ec\ub97c \ubc8c\uc5c8\uc73c\uba70, \uc804\uc138\uacc4\ub85c\ub294 3\uc5b5 9\ucc9c 2\ubc31\ub9cc\ub2ec\ub7ec\uc758 \uc218\uc785\uc744 \ub0c8\ub2e4. \ub85c\ud2bc \ud1a0\ub9c8\ud1a0\uc5d0\uc11c\ub294 10\uc810 \uc911 5.5\uc810\uc744 \ud68d\ub4dd\ud558\uc600\uace0, \uba54\ud0c0\ud06c\ub9ac\ud2f1\uc5d0\uc11c\ub3c4 \ud3c9\uade0\uc801\uc778 \ud3c9\uc744 \ubc1b\uc558\ub2e4. 2009\ub144 MTV \ubb34\ube44 \uc5b4\uc6cc\ub4dc\uc5d0\uc11c \ub85c\ud2b8\ub108\ub294 \u300a\ud2b8\uc640\uc77c\ub77c\uc787\u300b\uc5d0 \uac19\uc774 \ucd9c\uc5f0\ud55c \ub85c\ubc84\ud2b8 \ud328\ud2f4\uc2a8\uacfc \ub0a8\uc6b0\uc2e0\uc778\uc0c1\uc5d0 \ud6c4\ubcf4\ub85c \uc62c\ub790\uc73c\ub098, \ud328\ud2f4\uc2a8\uc774 \uc218\uc0c1\ud558\uc600\ub2e4.", "paragraph_answer": "2007\ub144, \uc601\ud654 \uc81c\uc791\uc790\ub4e4\uc740 \u300a\ud2b8\uc640\uc77c\ub77c\uc787 \uc601\ud654 \uc2dc\ub9ac\uc988\u300b\uc758 \uccab \ubc88\uc9f8 \uc791\ud488\uc778 \u300a\ud2b8\uc640\uc77c\ub77c\uc787\u300b\uc5d0\uc11c \ubca8\ub77c \uc2a4\uc644\uc758 \uc6d0\uc8fc\ubbfc \uce5c\uad6c \uc81c\uc774\ucf65 \ube14\ub799\uc744 \ub9e1\uc744 \ubc30\uc6b0\ub97c \ucc3e\uace0 \uc788\uc5c8\ub2e4. 2008\ub144 1\uc6d4 , \ud3ec\ud2c0\ub79c\ub4dc\uc5d0\uc11c \uc624\ub514\uc158\uc774 \uc5f4\ub838\uace0, \ub85c\ud2b8\ub108\ub294 \u300a\ud2b8\uc640\uc77c\ub77c\uc787\u300b\uc5d0 \ub300\ud574 \uc804\ud600 \ubab0\ub790\uc9c0\ub9cc \uae30\ud68d\uc0ac\uc5d0 \uc758\ud574 \uc624\ub514\uc158\uc744 \ubcf4\uac8c \ub418\uc5c8\ub2e4. \uc624\ub514\uc158\uc5d0\uc11c \uc774\ubbf8 \ubca8\ub77c \uc5ed\uc73c\ub85c \uce90\uc2a4\ud305\ub41c \ud06c\ub9ac\uc2a4\ud2f4 \uc2a4\ud29c\uc5b4\ud2b8\uc640 \ub300\uc0ac\ub97c \uc77d\uc5c8\uace0 \uadf8\ub4e4\uc740 \u300a\ub274\ubb38\u300b, \u300a\uc774\ud074\ub9bd\uc2a4\u300b\uc758 \uc7a5\uba74\ub4e4\uc744 \uc5f0\uae30\ud558\uc600\ub2e4. \uacb0\uacfc\ub294 \uc81c\uc774\ucf65 \ube14\ub799 \uc5ed\uc73c\ub85c \uce90\uc2a4\ud305\ub418\uc5c8\uace0, \uc601\ud654\ub294 2008\ub144\uc5d0 \uac1c\ubd09\ud558\uc600\ub2e4. \uc601\ud654\ub294 \uc131\uacf5\uc801\uc774\uc600\uace0, \uccab \uc8fc\uc5d0 6\ucc9c 9\ubc31\ub9cc\ub2ec\ub7ec\ub97c \ubc8c\uc5c8\uc73c\uba70, \uc804\uc138\uacc4\ub85c\ub294 3\uc5b5 9\ucc9c 2\ubc31\ub9cc\ub2ec\ub7ec\uc758 \uc218\uc785\uc744 \ub0c8\ub2e4. \ub85c\ud2bc \ud1a0\ub9c8\ud1a0\uc5d0\uc11c\ub294 10\uc810 \uc911 5.5\uc810\uc744 \ud68d\ub4dd\ud558\uc600\uace0, \uba54\ud0c0\ud06c\ub9ac\ud2f1\uc5d0\uc11c\ub3c4 \ud3c9\uade0\uc801\uc778 \ud3c9\uc744 \ubc1b\uc558\ub2e4. 2009\ub144 MTV \ubb34\ube44 \uc5b4\uc6cc\ub4dc\uc5d0\uc11c \ub85c\ud2b8\ub108\ub294 \u300a\ud2b8\uc640\uc77c\ub77c\uc787\u300b\uc5d0 \uac19\uc774 \ucd9c\uc5f0\ud55c \ub85c\ubc84\ud2b8 \ud328\ud2f4\uc2a8\uacfc \ub0a8\uc6b0\uc2e0\uc778\uc0c1\uc5d0 \ud6c4\ubcf4\ub85c \uc62c\ub790\uc73c\ub098, \ud328\ud2f4\uc2a8\uc774 \uc218\uc0c1\ud558\uc600\ub2e4.", "sentence_answer": " 2008\ub144 1\uc6d4 ,", "paragraph_id": "6565991-1-0", "paragraph_question": "question: \ub85c\ud2b8\ub108\uac00 \ud2b8\uc640\uc77c\ub77c\uc787\uc758 \uc624\ub514\uc158\uc744 \ubcf8\uac83\uc740 \uc5b8\uc81c\uc778\uac00?, context: 2007\ub144, \uc601\ud654 \uc81c\uc791\uc790\ub4e4\uc740 \u300a\ud2b8\uc640\uc77c\ub77c\uc787 \uc601\ud654 \uc2dc\ub9ac\uc988\u300b\uc758 \uccab \ubc88\uc9f8 \uc791\ud488\uc778 \u300a\ud2b8\uc640\uc77c\ub77c\uc787\u300b\uc5d0\uc11c \ubca8\ub77c \uc2a4\uc644\uc758 \uc6d0\uc8fc\ubbfc \uce5c\uad6c \uc81c\uc774\ucf65 \ube14\ub799\uc744 \ub9e1\uc744 \ubc30\uc6b0\ub97c \ucc3e\uace0 \uc788\uc5c8\ub2e4. 2008\ub144 1\uc6d4, \ud3ec\ud2c0\ub79c\ub4dc\uc5d0\uc11c \uc624\ub514\uc158\uc774 \uc5f4\ub838\uace0, \ub85c\ud2b8\ub108\ub294 \u300a\ud2b8\uc640\uc77c\ub77c\uc787\u300b\uc5d0 \ub300\ud574 \uc804\ud600 \ubab0\ub790\uc9c0\ub9cc \uae30\ud68d\uc0ac\uc5d0 \uc758\ud574 \uc624\ub514\uc158\uc744 \ubcf4\uac8c \ub418\uc5c8\ub2e4. \uc624\ub514\uc158\uc5d0\uc11c \uc774\ubbf8 \ubca8\ub77c \uc5ed\uc73c\ub85c \uce90\uc2a4\ud305\ub41c \ud06c\ub9ac\uc2a4\ud2f4 \uc2a4\ud29c\uc5b4\ud2b8\uc640 \ub300\uc0ac\ub97c \uc77d\uc5c8\uace0 \uadf8\ub4e4\uc740 \u300a\ub274\ubb38\u300b, \u300a\uc774\ud074\ub9bd\uc2a4\u300b\uc758 \uc7a5\uba74\ub4e4\uc744 \uc5f0\uae30\ud558\uc600\ub2e4. \uacb0\uacfc\ub294 \uc81c\uc774\ucf65 \ube14\ub799 \uc5ed\uc73c\ub85c \uce90\uc2a4\ud305\ub418\uc5c8\uace0, \uc601\ud654\ub294 2008\ub144\uc5d0 \uac1c\ubd09\ud558\uc600\ub2e4. \uc601\ud654\ub294 \uc131\uacf5\uc801\uc774\uc600\uace0, \uccab \uc8fc\uc5d0 6\ucc9c 9\ubc31\ub9cc\ub2ec\ub7ec\ub97c \ubc8c\uc5c8\uc73c\uba70, \uc804\uc138\uacc4\ub85c\ub294 3\uc5b5 9\ucc9c 2\ubc31\ub9cc\ub2ec\ub7ec\uc758 \uc218\uc785\uc744 \ub0c8\ub2e4. \ub85c\ud2bc \ud1a0\ub9c8\ud1a0\uc5d0\uc11c\ub294 10\uc810 \uc911 5.5\uc810\uc744 \ud68d\ub4dd\ud558\uc600\uace0, \uba54\ud0c0\ud06c\ub9ac\ud2f1\uc5d0\uc11c\ub3c4 \ud3c9\uade0\uc801\uc778 \ud3c9\uc744 \ubc1b\uc558\ub2e4. 2009\ub144 MTV \ubb34\ube44 \uc5b4\uc6cc\ub4dc\uc5d0\uc11c \ub85c\ud2b8\ub108\ub294 \u300a\ud2b8\uc640\uc77c\ub77c\uc787\u300b\uc5d0 \uac19\uc774 \ucd9c\uc5f0\ud55c \ub85c\ubc84\ud2b8 \ud328\ud2f4\uc2a8\uacfc \ub0a8\uc6b0\uc2e0\uc778\uc0c1\uc5d0 \ud6c4\ubcf4\ub85c \uc62c\ub790\uc73c\ub098, \ud328\ud2f4\uc2a8\uc774 \uc218\uc0c1\ud558\uc600\ub2e4."} {"question": "\uc678\ub534 \uc9c0\uc5ed\uc774\ub098 \uc6d0\uac70\ub9ac\uc5d0 \uc0b4\uace0 \uc788\ub294 \ud658\uc790\ub4e4\uc5d0\uac8c \uc720\uc775\ud55c \uc758\ub8cc\uc758 \ubc29\ubc95\uc740?", "paragraph": "\uc6d0\uaca9 \uc758\ub8cc\ub294 \uc678\ub534 \uc9c0\uc5ed\uc774\ub098 \uc6d0\uac70\ub9ac\uc5d0 \uc0b4\uace0 \uc788\ub294 \ud658\uc790\ub4e4\uc5d0\uac8c \uc720\uc775\ud558\ub2e4. \uadf8\ub4e4\uc740 \uc9c1\uc811 \uc758\uc0ac\ub098 \uc804\ubb38\uc758\ub4e4\uc744 \ub9cc\ub098\ub7ec \uba40\ub9ac\uae4c\uc9c0 \uac08 \ud544\uc694 \uc5c6\uc774 \uce58\ub8cc\ub97c \ubc1b\uc744 \uc218 \uc788\ub2e4. \ucd5c\uadfc\uc758 \ubaa8\ubc14\uc77c \ud611\uc5c5 \uae30\uc220\uc758 \ubc1c\uc804\uc740 \uac19\uc740 \uc9c0\uc5ed\uc758 \uacbd\uc6b0, \ub2e4\uc591\ud55c \uc7a5\uc18c\uc5d0 \uc788\ub294 \uc758\ub8cc\uc778\ub4e4\uc774 \uc815\ubcf4\ub97c \uacf5\uc720\ud558\uace0 \ud658\uc790\ub4e4\uc758 \ubb38\uc81c\uc5d0 \uad00\ud574 \uc758\ub17c\ud558\ub294 \uac83\uc744 \uac00\ub2a5\ud558\uac8c \ub9cc\ub4e4\uc5c8\ub2e4. \ubaa8\ubc14\uc77c \uae30\uc220\uc744 \uc774\uc6a9\ud55c \uc6d0\uaca9 \ud658\uc790 \ubaa8\ub2c8\ud130\ub9c1\uc740 \uc678\ub798 \ud658\uc790\ub4e4\uc758 \ubc29\ubb38 \ud544\uc694\uc131\uc744 \uc904\uc5ec\uc8fc\uace0, \uc6d0\uaca9 \ucc98\ubc29\uc758 \uac80\uc99d\uacfc \uc57d\ubb3c \ud22c\uc5ec \uad00\ub9ac\ub97c \uac00\ub2a5\ud558\uac8c \ud558\uc5ec \uac74\uac15 \uad00\ub9ac\uc758 \uc804\ubc18\uc801\uc778 \ube44\uc6a9\uc744 \uc7a0\uc7ac\uc801\uc73c\ub85c \uc0c1\ub2f9\ud788 \uc904\uc77c \uc218 \uc788\uac8c \ub41c\ub2e4. \ub610, \uc6d0\uaca9\uc758\ub8cc\ub294 \uadfc\ubb34\uc790\ub4e4\uc5d0\uac8c \uc804\ubb38\uac00\ub4e4\uc744 \ubcf4\uace0 \ubc30\uc6b8 \uc218 \uc788\uac8c \ud558\uace0, \ucd5c\uc801\uc758 \uc2e4\uc2b5\uc744 \ud6e8\uc52c \uc27d\uac8c \uacf5\uc720\ud568\uc73c\ub85c\uc368, \uc758\ud559 \uad50\uc721 \ud6a8\uacfc\ub3c4 \uc788\ub2e4.", "answer": "\uc6d0\uaca9 \uc758\ub8cc", "sentence": "\uc6d0\uaca9 \uc758\ub8cc \ub294 \uc678\ub534 \uc9c0\uc5ed\uc774\ub098 \uc6d0\uac70\ub9ac\uc5d0 \uc0b4\uace0 \uc788\ub294 \ud658\uc790\ub4e4\uc5d0\uac8c \uc720\uc775\ud558\ub2e4.", "paragraph_sentence": " \uc6d0\uaca9 \uc758\ub8cc \ub294 \uc678\ub534 \uc9c0\uc5ed\uc774\ub098 \uc6d0\uac70\ub9ac\uc5d0 \uc0b4\uace0 \uc788\ub294 \ud658\uc790\ub4e4\uc5d0\uac8c \uc720\uc775\ud558\ub2e4. \uadf8\ub4e4\uc740 \uc9c1\uc811 \uc758\uc0ac\ub098 \uc804\ubb38\uc758\ub4e4\uc744 \ub9cc\ub098\ub7ec \uba40\ub9ac\uae4c\uc9c0 \uac08 \ud544\uc694 \uc5c6\uc774 \uce58\ub8cc\ub97c \ubc1b\uc744 \uc218 \uc788\ub2e4. \ucd5c\uadfc\uc758 \ubaa8\ubc14\uc77c \ud611\uc5c5 \uae30\uc220\uc758 \ubc1c\uc804\uc740 \uac19\uc740 \uc9c0\uc5ed\uc758 \uacbd\uc6b0, \ub2e4\uc591\ud55c \uc7a5\uc18c\uc5d0 \uc788\ub294 \uc758\ub8cc\uc778\ub4e4\uc774 \uc815\ubcf4\ub97c \uacf5\uc720\ud558\uace0 \ud658\uc790\ub4e4\uc758 \ubb38\uc81c\uc5d0 \uad00\ud574 \uc758\ub17c\ud558\ub294 \uac83\uc744 \uac00\ub2a5\ud558\uac8c \ub9cc\ub4e4\uc5c8\ub2e4. \ubaa8\ubc14\uc77c \uae30\uc220\uc744 \uc774\uc6a9\ud55c \uc6d0\uaca9 \ud658\uc790 \ubaa8\ub2c8\ud130\ub9c1\uc740 \uc678\ub798 \ud658\uc790\ub4e4\uc758 \ubc29\ubb38 \ud544\uc694\uc131\uc744 \uc904\uc5ec\uc8fc\uace0, \uc6d0\uaca9 \ucc98\ubc29\uc758 \uac80\uc99d\uacfc \uc57d\ubb3c \ud22c\uc5ec \uad00\ub9ac\ub97c \uac00\ub2a5\ud558\uac8c \ud558\uc5ec \uac74\uac15 \uad00\ub9ac\uc758 \uc804\ubc18\uc801\uc778 \ube44\uc6a9\uc744 \uc7a0\uc7ac\uc801\uc73c\ub85c \uc0c1\ub2f9\ud788 \uc904\uc77c \uc218 \uc788\uac8c \ub41c\ub2e4. \ub610, \uc6d0\uaca9\uc758\ub8cc\ub294 \uadfc\ubb34\uc790\ub4e4\uc5d0\uac8c \uc804\ubb38\uac00\ub4e4\uc744 \ubcf4\uace0 \ubc30\uc6b8 \uc218 \uc788\uac8c \ud558\uace0, \ucd5c\uc801\uc758 \uc2e4\uc2b5\uc744 \ud6e8\uc52c \uc27d\uac8c \uacf5\uc720\ud568\uc73c\ub85c\uc368, \uc758\ud559 \uad50\uc721 \ud6a8\uacfc\ub3c4 \uc788\ub2e4.", "paragraph_answer": " \uc6d0\uaca9 \uc758\ub8cc \ub294 \uc678\ub534 \uc9c0\uc5ed\uc774\ub098 \uc6d0\uac70\ub9ac\uc5d0 \uc0b4\uace0 \uc788\ub294 \ud658\uc790\ub4e4\uc5d0\uac8c \uc720\uc775\ud558\ub2e4. \uadf8\ub4e4\uc740 \uc9c1\uc811 \uc758\uc0ac\ub098 \uc804\ubb38\uc758\ub4e4\uc744 \ub9cc\ub098\ub7ec \uba40\ub9ac\uae4c\uc9c0 \uac08 \ud544\uc694 \uc5c6\uc774 \uce58\ub8cc\ub97c \ubc1b\uc744 \uc218 \uc788\ub2e4. \ucd5c\uadfc\uc758 \ubaa8\ubc14\uc77c \ud611\uc5c5 \uae30\uc220\uc758 \ubc1c\uc804\uc740 \uac19\uc740 \uc9c0\uc5ed\uc758 \uacbd\uc6b0, \ub2e4\uc591\ud55c \uc7a5\uc18c\uc5d0 \uc788\ub294 \uc758\ub8cc\uc778\ub4e4\uc774 \uc815\ubcf4\ub97c \uacf5\uc720\ud558\uace0 \ud658\uc790\ub4e4\uc758 \ubb38\uc81c\uc5d0 \uad00\ud574 \uc758\ub17c\ud558\ub294 \uac83\uc744 \uac00\ub2a5\ud558\uac8c \ub9cc\ub4e4\uc5c8\ub2e4. \ubaa8\ubc14\uc77c \uae30\uc220\uc744 \uc774\uc6a9\ud55c \uc6d0\uaca9 \ud658\uc790 \ubaa8\ub2c8\ud130\ub9c1\uc740 \uc678\ub798 \ud658\uc790\ub4e4\uc758 \ubc29\ubb38 \ud544\uc694\uc131\uc744 \uc904\uc5ec\uc8fc\uace0, \uc6d0\uaca9 \ucc98\ubc29\uc758 \uac80\uc99d\uacfc \uc57d\ubb3c \ud22c\uc5ec \uad00\ub9ac\ub97c \uac00\ub2a5\ud558\uac8c \ud558\uc5ec \uac74\uac15 \uad00\ub9ac\uc758 \uc804\ubc18\uc801\uc778 \ube44\uc6a9\uc744 \uc7a0\uc7ac\uc801\uc73c\ub85c \uc0c1\ub2f9\ud788 \uc904\uc77c \uc218 \uc788\uac8c \ub41c\ub2e4. \ub610, \uc6d0\uaca9\uc758\ub8cc\ub294 \uadfc\ubb34\uc790\ub4e4\uc5d0\uac8c \uc804\ubb38\uac00\ub4e4\uc744 \ubcf4\uace0 \ubc30\uc6b8 \uc218 \uc788\uac8c \ud558\uace0, \ucd5c\uc801\uc758 \uc2e4\uc2b5\uc744 \ud6e8\uc52c \uc27d\uac8c \uacf5\uc720\ud568\uc73c\ub85c\uc368, \uc758\ud559 \uad50\uc721 \ud6a8\uacfc\ub3c4 \uc788\ub2e4.", "sentence_answer": " \uc6d0\uaca9 \uc758\ub8cc \ub294 \uc678\ub534 \uc9c0\uc5ed\uc774\ub098 \uc6d0\uac70\ub9ac\uc5d0 \uc0b4\uace0 \uc788\ub294 \ud658\uc790\ub4e4\uc5d0\uac8c \uc720\uc775\ud558\ub2e4.", "paragraph_id": "6526552-0-0", "paragraph_question": "question: \uc678\ub534 \uc9c0\uc5ed\uc774\ub098 \uc6d0\uac70\ub9ac\uc5d0 \uc0b4\uace0 \uc788\ub294 \ud658\uc790\ub4e4\uc5d0\uac8c \uc720\uc775\ud55c \uc758\ub8cc\uc758 \ubc29\ubc95\uc740?, context: \uc6d0\uaca9 \uc758\ub8cc\ub294 \uc678\ub534 \uc9c0\uc5ed\uc774\ub098 \uc6d0\uac70\ub9ac\uc5d0 \uc0b4\uace0 \uc788\ub294 \ud658\uc790\ub4e4\uc5d0\uac8c \uc720\uc775\ud558\ub2e4. \uadf8\ub4e4\uc740 \uc9c1\uc811 \uc758\uc0ac\ub098 \uc804\ubb38\uc758\ub4e4\uc744 \ub9cc\ub098\ub7ec \uba40\ub9ac\uae4c\uc9c0 \uac08 \ud544\uc694 \uc5c6\uc774 \uce58\ub8cc\ub97c \ubc1b\uc744 \uc218 \uc788\ub2e4. \ucd5c\uadfc\uc758 \ubaa8\ubc14\uc77c \ud611\uc5c5 \uae30\uc220\uc758 \ubc1c\uc804\uc740 \uac19\uc740 \uc9c0\uc5ed\uc758 \uacbd\uc6b0, \ub2e4\uc591\ud55c \uc7a5\uc18c\uc5d0 \uc788\ub294 \uc758\ub8cc\uc778\ub4e4\uc774 \uc815\ubcf4\ub97c \uacf5\uc720\ud558\uace0 \ud658\uc790\ub4e4\uc758 \ubb38\uc81c\uc5d0 \uad00\ud574 \uc758\ub17c\ud558\ub294 \uac83\uc744 \uac00\ub2a5\ud558\uac8c \ub9cc\ub4e4\uc5c8\ub2e4. \ubaa8\ubc14\uc77c \uae30\uc220\uc744 \uc774\uc6a9\ud55c \uc6d0\uaca9 \ud658\uc790 \ubaa8\ub2c8\ud130\ub9c1\uc740 \uc678\ub798 \ud658\uc790\ub4e4\uc758 \ubc29\ubb38 \ud544\uc694\uc131\uc744 \uc904\uc5ec\uc8fc\uace0, \uc6d0\uaca9 \ucc98\ubc29\uc758 \uac80\uc99d\uacfc \uc57d\ubb3c \ud22c\uc5ec \uad00\ub9ac\ub97c \uac00\ub2a5\ud558\uac8c \ud558\uc5ec \uac74\uac15 \uad00\ub9ac\uc758 \uc804\ubc18\uc801\uc778 \ube44\uc6a9\uc744 \uc7a0\uc7ac\uc801\uc73c\ub85c \uc0c1\ub2f9\ud788 \uc904\uc77c \uc218 \uc788\uac8c \ub41c\ub2e4. \ub610, \uc6d0\uaca9\uc758\ub8cc\ub294 \uadfc\ubb34\uc790\ub4e4\uc5d0\uac8c \uc804\ubb38\uac00\ub4e4\uc744 \ubcf4\uace0 \ubc30\uc6b8 \uc218 \uc788\uac8c \ud558\uace0, \ucd5c\uc801\uc758 \uc2e4\uc2b5\uc744 \ud6e8\uc52c \uc27d\uac8c \uacf5\uc720\ud568\uc73c\ub85c\uc368, \uc758\ud559 \uad50\uc721 \ud6a8\uacfc\ub3c4 \uc788\ub2e4."} {"question": "\ub9c8\ub974\ucf54\uac00 \uc74c\uc545\uc5d0 \ube60\uc9c0\uace0 \ub098\uc11c \ud569\ub958\ud55c \ud300\uc758 \uc774\ub984\uc740?", "paragraph": "\ub9c8\ub974\ucf54\ub294 \uc5b4\ub824\uc11c\ubd80\ud130 \ud53c\uc544\ub178\ub97c \ubc30\uc6b0\uba74\uc11c \uc74c\uc545\uc5d0 \ube60\uc9c0\uac8c \ub418\uc5c8\uace0, \uc2e0\ub514\uc0ac\uc774\uc800\ub97c \uc811\ud558\uba74\uc11c \uc791\uace1\uc5d0 \uad00\uc2ec\uc744 \uac00\uc9c0\uac8c \ub418\uc5c8\ub2e4. \uc774\ub97c \ud1b5\ud574 AN \ud328\ubc00\ub9ac \ub4f1\uc758 \ud300 \ud65c\ub3d9\uc744 \ud558\uae30\ub3c4 \ud588\ub2e4. \uadf8\ub7ec\ub358 \uc911 \uace0\ub4f1\ud559\uad50 2\ud559\ub144 \ub54c \ub9cc\ub09c Lugar\uc640 \ud568\uaed8 \ub4c0\uc624 mo'REAL\uc744 \uacb0\uc131\ud558\uba74\uc11c \ud799\ud569\uc744 \ubcf8\uaca9\uc801\uc73c\ub85c \uc2dc\uc791\ud558\uac8c \ub418\uc5c8\ub2e4. mo'REAL\uc740 \uc555\uad6c\uc815\ub3d9\uc5d0 \uc704\uce58\ud55c \ud074\ub7fd Krazyie\uc640 \uc2e0\ucd0c \ud074\ub7fd \uc815\uae00\uc744 \uc8fc\ubb34\ub300\ub85c \uacf5\uc5f0 \ud65c\ub3d9\uc744 \ud558\uc600\ub2e4. \ub610 \uc628\ub77c\uc778\uc744 \ud1b5\ud574 \uce90\ub098\ub2e4\uc5d0 \uac70\uc8fc \uc911\uc778 \ub798\ud37c \uacb8 \ud504\ub85c\ub4c0\uc11c juNi\uc640 \ub9cc\ub098\uac8c \ub418\uc5c8\uc73c\uba70, \uadf8 \uc678 \uc815\uacac (\ud604 J'Kyun), Freestarr, LEGO\uc640 \ud568\uaed8 Diamond Tribe\ub77c\ub294 \ud06c\ub8e8\ub97c \uacb0\uc131\ud558\uc600\ub2e4. \uc774\ub97c \uacc4\uae30\ub85c Marco\ub294 juNi\uc758 \uc568\ubc94 Made in Canada\uc5d0 \ucc38\uc5ec\ud558\uc5ec \uc790\uc2e0\uc758 \ubaa9\uc18c\ub9ac\ub97c \ucc98\uc74c \uc2e4\uc5c8\ub2e4. \uc774\ud6c4 \uc774\ub4e4\uc740 \ub2f9\uc2dc \uc2e0\uc0dd \ub808\uc774\ube14\uc774\uc5c8\ub358 \ube45\ub51c \ub808\ucf54\ub4dc\uc5d0 \ud569\ub958\ud558\uac8c \ub418\uc5c8\uc73c\ub098, 2004\ub144 mo'REAL\uc758 \ub450 \ubc88\uc9f8 EP\uac00 \ubc1c\ud45c\ub41c \ud6c4 Marco\uac00 \uc785\ub300\ub97c \ud558\uba74\uc11c mo'REAL\uc740 \uc774\ub807\ub2e4\ud560 \ud65c\ub3d9\uc744 \ud558\uc9c0 \ubabb \ud558\uc600\ub2e4. 2006\ub144 \uc81c\ub300 \ud6c4 Marco\ub294 \ub2e4\uc2dc mo'REAL \ud65c\ub3d9\uc744 \ud558\uae30\ub97c \uafc8\uafe8\uc73c\ub098 Lugar\uac00 \uc74c\uc545\uc744 \uc26c\uace0 \uc2f6\ub2e4\ub294 \uc758\uc0ac\ub97c \ud45c\uba85\ud558\uba74\uc11c mo'REAL\uc740 \ud574\uccb4\uc758 \uc218\uc21c\uc744 \ubc1f\uc558\ub2e4. \uc790\uc5f0\uc2a4\ub7fd\uac8c Marco\ub294 \uc194\ub85c \ud65c\ub3d9\uc744 \uc900\ube44\ud558\uae30 \uc2dc\uc791\ud558\uc600\ub2e4.", "answer": "AN \ud328\ubc00\ub9ac", "sentence": "\uc774\ub97c \ud1b5\ud574 AN \ud328\ubc00\ub9ac \ub4f1\uc758 \ud300 \ud65c\ub3d9\uc744 \ud558\uae30\ub3c4 \ud588\ub2e4.", "paragraph_sentence": "\ub9c8\ub974\ucf54\ub294 \uc5b4\ub824\uc11c\ubd80\ud130 \ud53c\uc544\ub178\ub97c \ubc30\uc6b0\uba74\uc11c \uc74c\uc545\uc5d0 \ube60\uc9c0\uac8c \ub418\uc5c8\uace0, \uc2e0\ub514\uc0ac\uc774\uc800\ub97c \uc811\ud558\uba74\uc11c \uc791\uace1\uc5d0 \uad00\uc2ec\uc744 \uac00\uc9c0\uac8c \ub418\uc5c8\ub2e4. \uc774\ub97c \ud1b5\ud574 AN \ud328\ubc00\ub9ac \ub4f1\uc758 \ud300 \ud65c\ub3d9\uc744 \ud558\uae30\ub3c4 \ud588\ub2e4. \uadf8\ub7ec\ub358 \uc911 \uace0\ub4f1\ud559\uad50 2\ud559\ub144 \ub54c \ub9cc\ub09c Lugar\uc640 \ud568\uaed8 \ub4c0\uc624 mo'REAL\uc744 \uacb0\uc131\ud558\uba74\uc11c \ud799\ud569\uc744 \ubcf8\uaca9\uc801\uc73c\ub85c \uc2dc\uc791\ud558\uac8c \ub418\uc5c8\ub2e4. mo'REAL\uc740 \uc555\uad6c\uc815\ub3d9\uc5d0 \uc704\uce58\ud55c \ud074\ub7fd Krazyie\uc640 \uc2e0\ucd0c \ud074\ub7fd \uc815\uae00\uc744 \uc8fc\ubb34\ub300\ub85c \uacf5\uc5f0 \ud65c\ub3d9\uc744 \ud558\uc600\ub2e4. \ub610 \uc628\ub77c\uc778\uc744 \ud1b5\ud574 \uce90\ub098\ub2e4\uc5d0 \uac70\uc8fc \uc911\uc778 \ub798\ud37c \uacb8 \ud504\ub85c\ub4c0\uc11c juNi\uc640 \ub9cc\ub098\uac8c \ub418\uc5c8\uc73c\uba70, \uadf8 \uc678 \uc815\uacac (\ud604 J'Kyun), Freestarr, LEGO\uc640 \ud568\uaed8 Diamond Tribe\ub77c\ub294 \ud06c\ub8e8\ub97c \uacb0\uc131\ud558\uc600\ub2e4. \uc774\ub97c \uacc4\uae30\ub85c Marco\ub294 juNi\uc758 \uc568\ubc94 Made in Canada\uc5d0 \ucc38\uc5ec\ud558\uc5ec \uc790\uc2e0\uc758 \ubaa9\uc18c\ub9ac\ub97c \ucc98\uc74c \uc2e4\uc5c8\ub2e4. \uc774\ud6c4 \uc774\ub4e4\uc740 \ub2f9\uc2dc \uc2e0\uc0dd \ub808\uc774\ube14\uc774\uc5c8\ub358 \ube45\ub51c \ub808\ucf54\ub4dc\uc5d0 \ud569\ub958\ud558\uac8c \ub418\uc5c8\uc73c\ub098, 2004\ub144 mo'REAL\uc758 \ub450 \ubc88\uc9f8 EP\uac00 \ubc1c\ud45c\ub41c \ud6c4 Marco\uac00 \uc785\ub300\ub97c \ud558\uba74\uc11c mo'REAL\uc740 \uc774\ub807\ub2e4\ud560 \ud65c\ub3d9\uc744 \ud558\uc9c0 \ubabb \ud558\uc600\ub2e4. 2006\ub144 \uc81c\ub300 \ud6c4 Marco\ub294 \ub2e4\uc2dc mo'REAL \ud65c\ub3d9\uc744 \ud558\uae30\ub97c \uafc8\uafe8\uc73c\ub098 Lugar\uac00 \uc74c\uc545\uc744 \uc26c\uace0 \uc2f6\ub2e4\ub294 \uc758\uc0ac\ub97c \ud45c\uba85\ud558\uba74\uc11c mo'REAL\uc740 \ud574\uccb4\uc758 \uc218\uc21c\uc744 \ubc1f\uc558\ub2e4. \uc790\uc5f0\uc2a4\ub7fd\uac8c Marco\ub294 \uc194\ub85c \ud65c\ub3d9\uc744 \uc900\ube44\ud558\uae30 \uc2dc\uc791\ud558\uc600\ub2e4.", "paragraph_answer": "\ub9c8\ub974\ucf54\ub294 \uc5b4\ub824\uc11c\ubd80\ud130 \ud53c\uc544\ub178\ub97c \ubc30\uc6b0\uba74\uc11c \uc74c\uc545\uc5d0 \ube60\uc9c0\uac8c \ub418\uc5c8\uace0, \uc2e0\ub514\uc0ac\uc774\uc800\ub97c \uc811\ud558\uba74\uc11c \uc791\uace1\uc5d0 \uad00\uc2ec\uc744 \uac00\uc9c0\uac8c \ub418\uc5c8\ub2e4. \uc774\ub97c \ud1b5\ud574 AN \ud328\ubc00\ub9ac \ub4f1\uc758 \ud300 \ud65c\ub3d9\uc744 \ud558\uae30\ub3c4 \ud588\ub2e4. \uadf8\ub7ec\ub358 \uc911 \uace0\ub4f1\ud559\uad50 2\ud559\ub144 \ub54c \ub9cc\ub09c Lugar\uc640 \ud568\uaed8 \ub4c0\uc624 mo'REAL\uc744 \uacb0\uc131\ud558\uba74\uc11c \ud799\ud569\uc744 \ubcf8\uaca9\uc801\uc73c\ub85c \uc2dc\uc791\ud558\uac8c \ub418\uc5c8\ub2e4. mo'REAL\uc740 \uc555\uad6c\uc815\ub3d9\uc5d0 \uc704\uce58\ud55c \ud074\ub7fd Krazyie\uc640 \uc2e0\ucd0c \ud074\ub7fd \uc815\uae00\uc744 \uc8fc\ubb34\ub300\ub85c \uacf5\uc5f0 \ud65c\ub3d9\uc744 \ud558\uc600\ub2e4. \ub610 \uc628\ub77c\uc778\uc744 \ud1b5\ud574 \uce90\ub098\ub2e4\uc5d0 \uac70\uc8fc \uc911\uc778 \ub798\ud37c \uacb8 \ud504\ub85c\ub4c0\uc11c juNi\uc640 \ub9cc\ub098\uac8c \ub418\uc5c8\uc73c\uba70, \uadf8 \uc678 \uc815\uacac (\ud604 J'Kyun), Freestarr, LEGO\uc640 \ud568\uaed8 Diamond Tribe\ub77c\ub294 \ud06c\ub8e8\ub97c \uacb0\uc131\ud558\uc600\ub2e4. \uc774\ub97c \uacc4\uae30\ub85c Marco\ub294 juNi\uc758 \uc568\ubc94 Made in Canada\uc5d0 \ucc38\uc5ec\ud558\uc5ec \uc790\uc2e0\uc758 \ubaa9\uc18c\ub9ac\ub97c \ucc98\uc74c \uc2e4\uc5c8\ub2e4. \uc774\ud6c4 \uc774\ub4e4\uc740 \ub2f9\uc2dc \uc2e0\uc0dd \ub808\uc774\ube14\uc774\uc5c8\ub358 \ube45\ub51c \ub808\ucf54\ub4dc\uc5d0 \ud569\ub958\ud558\uac8c \ub418\uc5c8\uc73c\ub098, 2004\ub144 mo'REAL\uc758 \ub450 \ubc88\uc9f8 EP\uac00 \ubc1c\ud45c\ub41c \ud6c4 Marco\uac00 \uc785\ub300\ub97c \ud558\uba74\uc11c mo'REAL\uc740 \uc774\ub807\ub2e4\ud560 \ud65c\ub3d9\uc744 \ud558\uc9c0 \ubabb \ud558\uc600\ub2e4. 2006\ub144 \uc81c\ub300 \ud6c4 Marco\ub294 \ub2e4\uc2dc mo'REAL \ud65c\ub3d9\uc744 \ud558\uae30\ub97c \uafc8\uafe8\uc73c\ub098 Lugar\uac00 \uc74c\uc545\uc744 \uc26c\uace0 \uc2f6\ub2e4\ub294 \uc758\uc0ac\ub97c \ud45c\uba85\ud558\uba74\uc11c mo'REAL\uc740 \ud574\uccb4\uc758 \uc218\uc21c\uc744 \ubc1f\uc558\ub2e4. \uc790\uc5f0\uc2a4\ub7fd\uac8c Marco\ub294 \uc194\ub85c \ud65c\ub3d9\uc744 \uc900\ube44\ud558\uae30 \uc2dc\uc791\ud558\uc600\ub2e4.", "sentence_answer": "\uc774\ub97c \ud1b5\ud574 AN \ud328\ubc00\ub9ac \ub4f1\uc758 \ud300 \ud65c\ub3d9\uc744 \ud558\uae30\ub3c4 \ud588\ub2e4.", "paragraph_id": "6068228-0-0", "paragraph_question": "question: \ub9c8\ub974\ucf54\uac00 \uc74c\uc545\uc5d0 \ube60\uc9c0\uace0 \ub098\uc11c \ud569\ub958\ud55c \ud300\uc758 \uc774\ub984\uc740?, context: \ub9c8\ub974\ucf54\ub294 \uc5b4\ub824\uc11c\ubd80\ud130 \ud53c\uc544\ub178\ub97c \ubc30\uc6b0\uba74\uc11c \uc74c\uc545\uc5d0 \ube60\uc9c0\uac8c \ub418\uc5c8\uace0, \uc2e0\ub514\uc0ac\uc774\uc800\ub97c \uc811\ud558\uba74\uc11c \uc791\uace1\uc5d0 \uad00\uc2ec\uc744 \uac00\uc9c0\uac8c \ub418\uc5c8\ub2e4. \uc774\ub97c \ud1b5\ud574 AN \ud328\ubc00\ub9ac \ub4f1\uc758 \ud300 \ud65c\ub3d9\uc744 \ud558\uae30\ub3c4 \ud588\ub2e4. \uadf8\ub7ec\ub358 \uc911 \uace0\ub4f1\ud559\uad50 2\ud559\ub144 \ub54c \ub9cc\ub09c Lugar\uc640 \ud568\uaed8 \ub4c0\uc624 mo'REAL\uc744 \uacb0\uc131\ud558\uba74\uc11c \ud799\ud569\uc744 \ubcf8\uaca9\uc801\uc73c\ub85c \uc2dc\uc791\ud558\uac8c \ub418\uc5c8\ub2e4. mo'REAL\uc740 \uc555\uad6c\uc815\ub3d9\uc5d0 \uc704\uce58\ud55c \ud074\ub7fd Krazyie\uc640 \uc2e0\ucd0c \ud074\ub7fd \uc815\uae00\uc744 \uc8fc\ubb34\ub300\ub85c \uacf5\uc5f0 \ud65c\ub3d9\uc744 \ud558\uc600\ub2e4. \ub610 \uc628\ub77c\uc778\uc744 \ud1b5\ud574 \uce90\ub098\ub2e4\uc5d0 \uac70\uc8fc \uc911\uc778 \ub798\ud37c \uacb8 \ud504\ub85c\ub4c0\uc11c juNi\uc640 \ub9cc\ub098\uac8c \ub418\uc5c8\uc73c\uba70, \uadf8 \uc678 \uc815\uacac (\ud604 J'Kyun), Freestarr, LEGO\uc640 \ud568\uaed8 Diamond Tribe\ub77c\ub294 \ud06c\ub8e8\ub97c \uacb0\uc131\ud558\uc600\ub2e4. \uc774\ub97c \uacc4\uae30\ub85c Marco\ub294 juNi\uc758 \uc568\ubc94 Made in Canada\uc5d0 \ucc38\uc5ec\ud558\uc5ec \uc790\uc2e0\uc758 \ubaa9\uc18c\ub9ac\ub97c \ucc98\uc74c \uc2e4\uc5c8\ub2e4. \uc774\ud6c4 \uc774\ub4e4\uc740 \ub2f9\uc2dc \uc2e0\uc0dd \ub808\uc774\ube14\uc774\uc5c8\ub358 \ube45\ub51c \ub808\ucf54\ub4dc\uc5d0 \ud569\ub958\ud558\uac8c \ub418\uc5c8\uc73c\ub098, 2004\ub144 mo'REAL\uc758 \ub450 \ubc88\uc9f8 EP\uac00 \ubc1c\ud45c\ub41c \ud6c4 Marco\uac00 \uc785\ub300\ub97c \ud558\uba74\uc11c mo'REAL\uc740 \uc774\ub807\ub2e4\ud560 \ud65c\ub3d9\uc744 \ud558\uc9c0 \ubabb \ud558\uc600\ub2e4. 2006\ub144 \uc81c\ub300 \ud6c4 Marco\ub294 \ub2e4\uc2dc mo'REAL \ud65c\ub3d9\uc744 \ud558\uae30\ub97c \uafc8\uafe8\uc73c\ub098 Lugar\uac00 \uc74c\uc545\uc744 \uc26c\uace0 \uc2f6\ub2e4\ub294 \uc758\uc0ac\ub97c \ud45c\uba85\ud558\uba74\uc11c mo'REAL\uc740 \ud574\uccb4\uc758 \uc218\uc21c\uc744 \ubc1f\uc558\ub2e4. \uc790\uc5f0\uc2a4\ub7fd\uac8c Marco\ub294 \uc194\ub85c \ud65c\ub3d9\uc744 \uc900\ube44\ud558\uae30 \uc2dc\uc791\ud558\uc600\ub2e4."} {"question": "\ub9ac\ube44\ud2b8\ub294 \ubb34\uc5c7\ub54c\ubb38\uc5d0 \uc0ac\ub9dd\ud588\ub098?", "paragraph": "\uc6b0\uc8fc\uac00 \ud33d\ucc3d\ud55c\ub2e4\ub294 \uac83\uc744 \ubc1d\ud600\ub0b8 \ubbf8\uad6d \ucc9c\ubb38\ud559\uc790 \uc5d0\ub4dc\uc708 \ud5c8\ube14\uc758 \uc5c5\uc801 \uc5ed\uc2dc \ub9ac\ube44\ud2b8\uc758 \uc2e0\uae30\uc6d0\uc744 \uc774\ub8ec \uc5f0\uad6c\uac00 \uc788\uc5c8\uae30\uc5d0 \uac00\ub2a5\ud588\ub358 \uac83\uc774\ub2e4. \ud5c8\ube14\uc740 \ub9ac\ube44\ud2b8\uc758 \uc5c5\uc801\uc744 \uadf8\uc758 \uc800\uc11c\uc5d0\uc11c\u201c\ud5e8\ub9ac\uc5d0\ud0c0 \ub9ac\ube44\ud2b8\uac00 \uc6b0\uc8fc\uc758 \ud06c\uae30\ub97c \uacb0\uc815\ud560 \uc218 \uc788\ub294 \uc5f4\uc1e0\ub97c \ub9cc\ub4e4\uc5b4\ub0c8\ub2e4\uba74, \uc5d0\ub4dc\uc708 \ud5c8\ube14\uc774 \uadf8 \uc5f4\uc1e0\ub97c \uc790\ubb3c\uc1e0\uc5d0 \uc464\uc154\ub123\uace0 \ub4a4\uc774\uc5b4 \uadf8 \uc5f4\uc1e0\uac00 \ub3cc\uc544\uac00\uac8c\ub054 \ud558\ub294 \uad00\uce21\uc0ac\uc2e4\uc744 \uc81c\uacf5\ud558\uc600\ub2e4\u201d\ub77c\uace0 \uc5b8\uae09\ud588\ub2e4. \uc774\ucc98\ub7fc \ud5c8\ube14 \ubcf8\uc778\uc740 \ub9ac\ube44\ud2b8\uc758 \uc5c5\uc801\uc744 \uc778\uc815\ud558\uba70 \ub9ac\ube44\ud2b8\ub294 \ub178\ubca8\uc0c1\uc744 \ubc1b\uc744 \uc790\uaca9\uc774 \uc788\ub2e4\uace0 \uc790\uc8fc \ub9d0\ud558\uace4 \ud588\ub2e4. \uc2a4\uc6e8\ub374 \ud55c\ub9bc\uc6d0\uc758 \uad34\uc2a4\ud0c0 \ubbf8\ud0c0\uadf8\ub808\ud50c\ub7ec\ub294 \ub9ac\ube44\ud2b8\ub97c \ub178\ubca8 \ubb3c\ub9ac\ud559\uc0c1 \ud6c4\ubcf4\ub85c \uc62c\ub9b4 \uc0dd\uac01\uc744 \ud558\uace0 \ud558\ubc84\ub4dc \ucc9c\ubb38\ub300\uc7a5 \uc100\ud50c\ub9ac\uc5d0\uac8c \ub9ac\ube44\ud2b8\uc758 \uc138\ud398\uc774\ub4dc\ud615 \ubcc0\uad11\uc131 \uc5f0\uad6c\uc5d0 \uad00\ud55c \uc815\ubcf4\ub97c \uc694\uccad\ud588\ub2e4. \ud558\uc9c0\ub9cc \ubbf8\ud0c0\uadf8\ub808\ud50c\ub7ec\ub294 \uadf8\ubcf4\ub2e4 3\ub144 \uc804\uc5d0 \ub9ac\ube44\ud2b8\uac00 \uc554\uc73c\ub85c \uc0ac\ub9dd\ud588\ub2e4\ub294 \uc0ac\uc2e4\ub9cc \uc54c\uac8c \ub418\uc5c8\ub2e4. \ub178\ubca8\uc0c1\uc740 \uc0ac\uc790\uc5d0\uac8c\ub294 \uc218\uc5ec\ub418\uc9c0 \uc54a\ub294\ub2e4.", "answer": "\uc554", "sentence": "\ud558\uc9c0\ub9cc \ubbf8\ud0c0\uadf8\ub808\ud50c\ub7ec\ub294 \uadf8\ubcf4\ub2e4 3\ub144 \uc804\uc5d0 \ub9ac\ube44\ud2b8\uac00 \uc554 \uc73c\ub85c \uc0ac\ub9dd\ud588\ub2e4\ub294 \uc0ac\uc2e4\ub9cc \uc54c\uac8c \ub418\uc5c8\ub2e4.", "paragraph_sentence": "\uc6b0\uc8fc\uac00 \ud33d\ucc3d\ud55c\ub2e4\ub294 \uac83\uc744 \ubc1d\ud600\ub0b8 \ubbf8\uad6d \ucc9c\ubb38\ud559\uc790 \uc5d0\ub4dc\uc708 \ud5c8\ube14\uc758 \uc5c5\uc801 \uc5ed\uc2dc \ub9ac\ube44\ud2b8\uc758 \uc2e0\uae30\uc6d0\uc744 \uc774\ub8ec \uc5f0\uad6c\uac00 \uc788\uc5c8\uae30\uc5d0 \uac00\ub2a5\ud588\ub358 \uac83\uc774\ub2e4. \ud5c8\ube14\uc740 \ub9ac\ube44\ud2b8\uc758 \uc5c5\uc801\uc744 \uadf8\uc758 \uc800\uc11c\uc5d0\uc11c\u201c\ud5e8\ub9ac\uc5d0\ud0c0 \ub9ac\ube44\ud2b8\uac00 \uc6b0\uc8fc\uc758 \ud06c\uae30\ub97c \uacb0\uc815\ud560 \uc218 \uc788\ub294 \uc5f4\uc1e0\ub97c \ub9cc\ub4e4\uc5b4\ub0c8\ub2e4\uba74, \uc5d0\ub4dc\uc708 \ud5c8\ube14\uc774 \uadf8 \uc5f4\uc1e0\ub97c \uc790\ubb3c\uc1e0\uc5d0 \uc464\uc154\ub123\uace0 \ub4a4\uc774\uc5b4 \uadf8 \uc5f4\uc1e0\uac00 \ub3cc\uc544\uac00\uac8c\ub054 \ud558\ub294 \uad00\uce21\uc0ac\uc2e4\uc744 \uc81c\uacf5\ud558\uc600\ub2e4\u201d\ub77c\uace0 \uc5b8\uae09\ud588\ub2e4. \uc774\ucc98\ub7fc \ud5c8\ube14 \ubcf8\uc778\uc740 \ub9ac\ube44\ud2b8\uc758 \uc5c5\uc801\uc744 \uc778\uc815\ud558\uba70 \ub9ac\ube44\ud2b8\ub294 \ub178\ubca8\uc0c1\uc744 \ubc1b\uc744 \uc790\uaca9\uc774 \uc788\ub2e4\uace0 \uc790\uc8fc \ub9d0\ud558\uace4 \ud588\ub2e4. \uc2a4\uc6e8\ub374 \ud55c\ub9bc\uc6d0\uc758 \uad34\uc2a4\ud0c0 \ubbf8\ud0c0\uadf8\ub808\ud50c\ub7ec\ub294 \ub9ac\ube44\ud2b8\ub97c \ub178\ubca8 \ubb3c\ub9ac\ud559\uc0c1 \ud6c4\ubcf4\ub85c \uc62c\ub9b4 \uc0dd\uac01\uc744 \ud558\uace0 \ud558\ubc84\ub4dc \ucc9c\ubb38\ub300\uc7a5 \uc100\ud50c\ub9ac\uc5d0\uac8c \ub9ac\ube44\ud2b8\uc758 \uc138\ud398\uc774\ub4dc\ud615 \ubcc0\uad11\uc131 \uc5f0\uad6c\uc5d0 \uad00\ud55c \uc815\ubcf4\ub97c \uc694\uccad\ud588\ub2e4. \ud558\uc9c0\ub9cc \ubbf8\ud0c0\uadf8\ub808\ud50c\ub7ec\ub294 \uadf8\ubcf4\ub2e4 3\ub144 \uc804\uc5d0 \ub9ac\ube44\ud2b8\uac00 \uc554 \uc73c\ub85c \uc0ac\ub9dd\ud588\ub2e4\ub294 \uc0ac\uc2e4\ub9cc \uc54c\uac8c \ub418\uc5c8\ub2e4. \ub178\ubca8\uc0c1\uc740 \uc0ac\uc790\uc5d0\uac8c\ub294 \uc218\uc5ec\ub418\uc9c0 \uc54a\ub294\ub2e4.", "paragraph_answer": "\uc6b0\uc8fc\uac00 \ud33d\ucc3d\ud55c\ub2e4\ub294 \uac83\uc744 \ubc1d\ud600\ub0b8 \ubbf8\uad6d \ucc9c\ubb38\ud559\uc790 \uc5d0\ub4dc\uc708 \ud5c8\ube14\uc758 \uc5c5\uc801 \uc5ed\uc2dc \ub9ac\ube44\ud2b8\uc758 \uc2e0\uae30\uc6d0\uc744 \uc774\ub8ec \uc5f0\uad6c\uac00 \uc788\uc5c8\uae30\uc5d0 \uac00\ub2a5\ud588\ub358 \uac83\uc774\ub2e4. \ud5c8\ube14\uc740 \ub9ac\ube44\ud2b8\uc758 \uc5c5\uc801\uc744 \uadf8\uc758 \uc800\uc11c\uc5d0\uc11c\u201c\ud5e8\ub9ac\uc5d0\ud0c0 \ub9ac\ube44\ud2b8\uac00 \uc6b0\uc8fc\uc758 \ud06c\uae30\ub97c \uacb0\uc815\ud560 \uc218 \uc788\ub294 \uc5f4\uc1e0\ub97c \ub9cc\ub4e4\uc5b4\ub0c8\ub2e4\uba74, \uc5d0\ub4dc\uc708 \ud5c8\ube14\uc774 \uadf8 \uc5f4\uc1e0\ub97c \uc790\ubb3c\uc1e0\uc5d0 \uc464\uc154\ub123\uace0 \ub4a4\uc774\uc5b4 \uadf8 \uc5f4\uc1e0\uac00 \ub3cc\uc544\uac00\uac8c\ub054 \ud558\ub294 \uad00\uce21\uc0ac\uc2e4\uc744 \uc81c\uacf5\ud558\uc600\ub2e4\u201d\ub77c\uace0 \uc5b8\uae09\ud588\ub2e4. \uc774\ucc98\ub7fc \ud5c8\ube14 \ubcf8\uc778\uc740 \ub9ac\ube44\ud2b8\uc758 \uc5c5\uc801\uc744 \uc778\uc815\ud558\uba70 \ub9ac\ube44\ud2b8\ub294 \ub178\ubca8\uc0c1\uc744 \ubc1b\uc744 \uc790\uaca9\uc774 \uc788\ub2e4\uace0 \uc790\uc8fc \ub9d0\ud558\uace4 \ud588\ub2e4. \uc2a4\uc6e8\ub374 \ud55c\ub9bc\uc6d0\uc758 \uad34\uc2a4\ud0c0 \ubbf8\ud0c0\uadf8\ub808\ud50c\ub7ec\ub294 \ub9ac\ube44\ud2b8\ub97c \ub178\ubca8 \ubb3c\ub9ac\ud559\uc0c1 \ud6c4\ubcf4\ub85c \uc62c\ub9b4 \uc0dd\uac01\uc744 \ud558\uace0 \ud558\ubc84\ub4dc \ucc9c\ubb38\ub300\uc7a5 \uc100\ud50c\ub9ac\uc5d0\uac8c \ub9ac\ube44\ud2b8\uc758 \uc138\ud398\uc774\ub4dc\ud615 \ubcc0\uad11\uc131 \uc5f0\uad6c\uc5d0 \uad00\ud55c \uc815\ubcf4\ub97c \uc694\uccad\ud588\ub2e4. \ud558\uc9c0\ub9cc \ubbf8\ud0c0\uadf8\ub808\ud50c\ub7ec\ub294 \uadf8\ubcf4\ub2e4 3\ub144 \uc804\uc5d0 \ub9ac\ube44\ud2b8\uac00 \uc554 \uc73c\ub85c \uc0ac\ub9dd\ud588\ub2e4\ub294 \uc0ac\uc2e4\ub9cc \uc54c\uac8c \ub418\uc5c8\ub2e4. \ub178\ubca8\uc0c1\uc740 \uc0ac\uc790\uc5d0\uac8c\ub294 \uc218\uc5ec\ub418\uc9c0 \uc54a\ub294\ub2e4.", "sentence_answer": "\ud558\uc9c0\ub9cc \ubbf8\ud0c0\uadf8\ub808\ud50c\ub7ec\ub294 \uadf8\ubcf4\ub2e4 3\ub144 \uc804\uc5d0 \ub9ac\ube44\ud2b8\uac00 \uc554 \uc73c\ub85c \uc0ac\ub9dd\ud588\ub2e4\ub294 \uc0ac\uc2e4\ub9cc \uc54c\uac8c \ub418\uc5c8\ub2e4.", "paragraph_id": "6571435-2-2", "paragraph_question": "question: \ub9ac\ube44\ud2b8\ub294 \ubb34\uc5c7\ub54c\ubb38\uc5d0 \uc0ac\ub9dd\ud588\ub098?, context: \uc6b0\uc8fc\uac00 \ud33d\ucc3d\ud55c\ub2e4\ub294 \uac83\uc744 \ubc1d\ud600\ub0b8 \ubbf8\uad6d \ucc9c\ubb38\ud559\uc790 \uc5d0\ub4dc\uc708 \ud5c8\ube14\uc758 \uc5c5\uc801 \uc5ed\uc2dc \ub9ac\ube44\ud2b8\uc758 \uc2e0\uae30\uc6d0\uc744 \uc774\ub8ec \uc5f0\uad6c\uac00 \uc788\uc5c8\uae30\uc5d0 \uac00\ub2a5\ud588\ub358 \uac83\uc774\ub2e4. \ud5c8\ube14\uc740 \ub9ac\ube44\ud2b8\uc758 \uc5c5\uc801\uc744 \uadf8\uc758 \uc800\uc11c\uc5d0\uc11c\u201c\ud5e8\ub9ac\uc5d0\ud0c0 \ub9ac\ube44\ud2b8\uac00 \uc6b0\uc8fc\uc758 \ud06c\uae30\ub97c \uacb0\uc815\ud560 \uc218 \uc788\ub294 \uc5f4\uc1e0\ub97c \ub9cc\ub4e4\uc5b4\ub0c8\ub2e4\uba74, \uc5d0\ub4dc\uc708 \ud5c8\ube14\uc774 \uadf8 \uc5f4\uc1e0\ub97c \uc790\ubb3c\uc1e0\uc5d0 \uc464\uc154\ub123\uace0 \ub4a4\uc774\uc5b4 \uadf8 \uc5f4\uc1e0\uac00 \ub3cc\uc544\uac00\uac8c\ub054 \ud558\ub294 \uad00\uce21\uc0ac\uc2e4\uc744 \uc81c\uacf5\ud558\uc600\ub2e4\u201d\ub77c\uace0 \uc5b8\uae09\ud588\ub2e4. \uc774\ucc98\ub7fc \ud5c8\ube14 \ubcf8\uc778\uc740 \ub9ac\ube44\ud2b8\uc758 \uc5c5\uc801\uc744 \uc778\uc815\ud558\uba70 \ub9ac\ube44\ud2b8\ub294 \ub178\ubca8\uc0c1\uc744 \ubc1b\uc744 \uc790\uaca9\uc774 \uc788\ub2e4\uace0 \uc790\uc8fc \ub9d0\ud558\uace4 \ud588\ub2e4. \uc2a4\uc6e8\ub374 \ud55c\ub9bc\uc6d0\uc758 \uad34\uc2a4\ud0c0 \ubbf8\ud0c0\uadf8\ub808\ud50c\ub7ec\ub294 \ub9ac\ube44\ud2b8\ub97c \ub178\ubca8 \ubb3c\ub9ac\ud559\uc0c1 \ud6c4\ubcf4\ub85c \uc62c\ub9b4 \uc0dd\uac01\uc744 \ud558\uace0 \ud558\ubc84\ub4dc \ucc9c\ubb38\ub300\uc7a5 \uc100\ud50c\ub9ac\uc5d0\uac8c \ub9ac\ube44\ud2b8\uc758 \uc138\ud398\uc774\ub4dc\ud615 \ubcc0\uad11\uc131 \uc5f0\uad6c\uc5d0 \uad00\ud55c \uc815\ubcf4\ub97c \uc694\uccad\ud588\ub2e4. \ud558\uc9c0\ub9cc \ubbf8\ud0c0\uadf8\ub808\ud50c\ub7ec\ub294 \uadf8\ubcf4\ub2e4 3\ub144 \uc804\uc5d0 \ub9ac\ube44\ud2b8\uac00 \uc554\uc73c\ub85c \uc0ac\ub9dd\ud588\ub2e4\ub294 \uc0ac\uc2e4\ub9cc \uc54c\uac8c \ub418\uc5c8\ub2e4. \ub178\ubca8\uc0c1\uc740 \uc0ac\uc790\uc5d0\uac8c\ub294 \uc218\uc5ec\ub418\uc9c0 \uc54a\ub294\ub2e4."} {"question": "1995\ub144 \uc9c0\ubc29\uc120\uac70\uc5d0\uc11c \uc5ec\ub2f9\uc744 \ubc00\uc5b4\ub0b4\uace0 \uc11c\uc6b8\uc2dc\uc7a5\uc744 \ucc28\uc9c0\ud55c \ub2f9\uc740?", "paragraph": "1992\ub144 5\uc6d4 26\uc77c \uc81c14\ub300 \ub300\ud1b5\ub839\ud6c4\ubcf4 \uacbd\uc120\uc5d0 \ucd9c\ub9c8\ud558\uc600\uc73c\ub098, \uacbd\uc120\uc5d0\uc11c \uae40\ub300\uc911 \ub300\ud45c\ucd5c\uace0\uc704\uc6d0\uc5d0\uac8c \uc11d\ud328\ud558\uc600\ub2e4. 12\uc6d4 \uae40\ub300\uc911\uc774 \ub300\ud1b5\ub839 \uc120\uac70\uc5d0\uc11c \ub099\uc120\ud558\uace0 \uc815\uacc4\ub97c \uc740\ud1f4\ud558\uc790, \uc77c\uc57d \uc57c\ub2f9\ucd1d\uc218\uac00 \ub418\uc5c8\ub2e4. 1993-1994\ub144\uc5d0\ub294 \uc81c1\uc57c\ub2f9\uc758 \ucd1d\uc218\ub85c\uc11c \uc804\uc131\uae30\ub97c \ub204\ub838\uc73c\ub098, \ub2f9\ub0b4 \ucd5c\ub300 \uc9c0\ubd84\uc744 \uc18c\uc720\ud558\uace0 \uc788\ub358 \ub3d9\uad50\ub3d9\uacc4\uc640 \ubc88\ubc88\uc774 \uc2e0\uacbd\uc804\uc744 \ubc8c\uc774\uba70 \uc8fc\ub3c4\uad8c \ub2e4\ud23c\uc744 \ud558\uc600\ub2e4. 1995\ub144 \uc9c0\ubc29\uc120\uac70\uc5d0\uc11c \uacf5\ucc9c\uad8c\uc744 \ub458\ub7ec\uc2f8\uace0 \ub3d9\uad50\ub3d9\uacc4\uc640 \uc7a6\uc740 \ucda9\ub3cc \ub05d\uc5d0 \uacb0\uad6d \ub418\ub3cc\uc544\uc62c \uc218 \uc5c6\ub294 \uac15\uc744 \uac74\ub11c\ub2e4. \ubbfc\uc8fc\ub2f9\uc740 \uc5ec\ub2f9\uc744 \ubc00\uc5b4\ub0b4\uace0 \uc11c\uc6b8\uc2dc\uc7a5\uc744 \ucc28\uc9c0\ud558\ub294\ub4f1 \uc9c0\ubc29\uc120\uac70\uc5d0\uc11c\ub294 \uc0ac\uc2e4\uc0c1 \uc2b9\ub9ac\ud558\uc600\uc9c0\ub9cc, \uc774\ubbf8 \uc774\uae30\ud0dd\uc744 \ubc30\uc81c\ud558\uae30\ub85c \ub9c8\uc74c\uba39\uc740 \uae40\ub300\uc911\uc740 \ub3d9\uad50\ub3d9\uacc4 \uc758\uc6d0 \uc804\uc6d0\uc744 \ubbfc\uc8fc\ub2f9\uc5d0\uc11c \ud0c8\ub2f9\uc2dc\ucf1c \uc0c8\uc815\uce58\uad6d\ubbfc\ud68c\uc758\ub97c \ucc3d\ub2f9\ud558\uace0 \uc815\uacc4\uc5d0 \ubcf5\uadc0\ud558\uc600\ub2e4.", "answer": "\ubbfc\uc8fc\ub2f9", "sentence": "\ubbfc\uc8fc\ub2f9 \uc740 \uc5ec\ub2f9\uc744 \ubc00\uc5b4\ub0b4\uace0 \uc11c\uc6b8\uc2dc\uc7a5\uc744 \ucc28\uc9c0\ud558\ub294\ub4f1 \uc9c0\ubc29\uc120\uac70\uc5d0\uc11c\ub294 \uc0ac\uc2e4\uc0c1 \uc2b9\ub9ac\ud558\uc600\uc9c0\ub9cc, \uc774\ubbf8 \uc774\uae30\ud0dd\uc744 \ubc30\uc81c\ud558\uae30\ub85c \ub9c8\uc74c\uba39\uc740 \uae40\ub300\uc911\uc740 \ub3d9\uad50\ub3d9\uacc4 \uc758\uc6d0 \uc804\uc6d0\uc744 \ubbfc\uc8fc\ub2f9\uc5d0\uc11c \ud0c8\ub2f9\uc2dc\ucf1c \uc0c8\uc815\uce58\uad6d\ubbfc\ud68c\uc758\ub97c \ucc3d\ub2f9\ud558\uace0 \uc815\uacc4\uc5d0 \ubcf5\uadc0\ud558\uc600\ub2e4.", "paragraph_sentence": "1992\ub144 5\uc6d4 26\uc77c \uc81c14\ub300 \ub300\ud1b5\ub839\ud6c4\ubcf4 \uacbd\uc120\uc5d0 \ucd9c\ub9c8\ud558\uc600\uc73c\ub098, \uacbd\uc120\uc5d0\uc11c \uae40\ub300\uc911 \ub300\ud45c\ucd5c\uace0\uc704\uc6d0\uc5d0\uac8c \uc11d\ud328\ud558\uc600\ub2e4. 12\uc6d4 \uae40\ub300\uc911\uc774 \ub300\ud1b5\ub839 \uc120\uac70\uc5d0\uc11c \ub099\uc120\ud558\uace0 \uc815\uacc4\ub97c \uc740\ud1f4\ud558\uc790, \uc77c\uc57d \uc57c\ub2f9\ucd1d\uc218\uac00 \ub418\uc5c8\ub2e4. 1993-1994\ub144\uc5d0\ub294 \uc81c1\uc57c\ub2f9\uc758 \ucd1d\uc218\ub85c\uc11c \uc804\uc131\uae30\ub97c \ub204\ub838\uc73c\ub098, \ub2f9\ub0b4 \ucd5c\ub300 \uc9c0\ubd84\uc744 \uc18c\uc720\ud558\uace0 \uc788\ub358 \ub3d9\uad50\ub3d9\uacc4\uc640 \ubc88\ubc88\uc774 \uc2e0\uacbd\uc804\uc744 \ubc8c\uc774\uba70 \uc8fc\ub3c4\uad8c \ub2e4\ud23c\uc744 \ud558\uc600\ub2e4. 1995\ub144 \uc9c0\ubc29\uc120\uac70\uc5d0\uc11c \uacf5\ucc9c\uad8c\uc744 \ub458\ub7ec\uc2f8\uace0 \ub3d9\uad50\ub3d9\uacc4\uc640 \uc7a6\uc740 \ucda9\ub3cc \ub05d\uc5d0 \uacb0\uad6d \ub418\ub3cc\uc544\uc62c \uc218 \uc5c6\ub294 \uac15\uc744 \uac74\ub11c\ub2e4. \ubbfc\uc8fc\ub2f9 \uc740 \uc5ec\ub2f9\uc744 \ubc00\uc5b4\ub0b4\uace0 \uc11c\uc6b8\uc2dc\uc7a5\uc744 \ucc28\uc9c0\ud558\ub294\ub4f1 \uc9c0\ubc29\uc120\uac70\uc5d0\uc11c\ub294 \uc0ac\uc2e4\uc0c1 \uc2b9\ub9ac\ud558\uc600\uc9c0\ub9cc, \uc774\ubbf8 \uc774\uae30\ud0dd\uc744 \ubc30\uc81c\ud558\uae30\ub85c \ub9c8\uc74c\uba39\uc740 \uae40\ub300\uc911\uc740 \ub3d9\uad50\ub3d9\uacc4 \uc758\uc6d0 \uc804\uc6d0\uc744 \ubbfc\uc8fc\ub2f9\uc5d0\uc11c \ud0c8\ub2f9\uc2dc\ucf1c \uc0c8\uc815\uce58\uad6d\ubbfc\ud68c\uc758\ub97c \ucc3d\ub2f9\ud558\uace0 \uc815\uacc4\uc5d0 \ubcf5\uadc0\ud558\uc600\ub2e4. ", "paragraph_answer": "1992\ub144 5\uc6d4 26\uc77c \uc81c14\ub300 \ub300\ud1b5\ub839\ud6c4\ubcf4 \uacbd\uc120\uc5d0 \ucd9c\ub9c8\ud558\uc600\uc73c\ub098, \uacbd\uc120\uc5d0\uc11c \uae40\ub300\uc911 \ub300\ud45c\ucd5c\uace0\uc704\uc6d0\uc5d0\uac8c \uc11d\ud328\ud558\uc600\ub2e4. 12\uc6d4 \uae40\ub300\uc911\uc774 \ub300\ud1b5\ub839 \uc120\uac70\uc5d0\uc11c \ub099\uc120\ud558\uace0 \uc815\uacc4\ub97c \uc740\ud1f4\ud558\uc790, \uc77c\uc57d \uc57c\ub2f9\ucd1d\uc218\uac00 \ub418\uc5c8\ub2e4. 1993-1994\ub144\uc5d0\ub294 \uc81c1\uc57c\ub2f9\uc758 \ucd1d\uc218\ub85c\uc11c \uc804\uc131\uae30\ub97c \ub204\ub838\uc73c\ub098, \ub2f9\ub0b4 \ucd5c\ub300 \uc9c0\ubd84\uc744 \uc18c\uc720\ud558\uace0 \uc788\ub358 \ub3d9\uad50\ub3d9\uacc4\uc640 \ubc88\ubc88\uc774 \uc2e0\uacbd\uc804\uc744 \ubc8c\uc774\uba70 \uc8fc\ub3c4\uad8c \ub2e4\ud23c\uc744 \ud558\uc600\ub2e4. 1995\ub144 \uc9c0\ubc29\uc120\uac70\uc5d0\uc11c \uacf5\ucc9c\uad8c\uc744 \ub458\ub7ec\uc2f8\uace0 \ub3d9\uad50\ub3d9\uacc4\uc640 \uc7a6\uc740 \ucda9\ub3cc \ub05d\uc5d0 \uacb0\uad6d \ub418\ub3cc\uc544\uc62c \uc218 \uc5c6\ub294 \uac15\uc744 \uac74\ub11c\ub2e4. \ubbfc\uc8fc\ub2f9 \uc740 \uc5ec\ub2f9\uc744 \ubc00\uc5b4\ub0b4\uace0 \uc11c\uc6b8\uc2dc\uc7a5\uc744 \ucc28\uc9c0\ud558\ub294\ub4f1 \uc9c0\ubc29\uc120\uac70\uc5d0\uc11c\ub294 \uc0ac\uc2e4\uc0c1 \uc2b9\ub9ac\ud558\uc600\uc9c0\ub9cc, \uc774\ubbf8 \uc774\uae30\ud0dd\uc744 \ubc30\uc81c\ud558\uae30\ub85c \ub9c8\uc74c\uba39\uc740 \uae40\ub300\uc911\uc740 \ub3d9\uad50\ub3d9\uacc4 \uc758\uc6d0 \uc804\uc6d0\uc744 \ubbfc\uc8fc\ub2f9\uc5d0\uc11c \ud0c8\ub2f9\uc2dc\ucf1c \uc0c8\uc815\uce58\uad6d\ubbfc\ud68c\uc758\ub97c \ucc3d\ub2f9\ud558\uace0 \uc815\uacc4\uc5d0 \ubcf5\uadc0\ud558\uc600\ub2e4.", "sentence_answer": " \ubbfc\uc8fc\ub2f9 \uc740 \uc5ec\ub2f9\uc744 \ubc00\uc5b4\ub0b4\uace0 \uc11c\uc6b8\uc2dc\uc7a5\uc744 \ucc28\uc9c0\ud558\ub294\ub4f1 \uc9c0\ubc29\uc120\uac70\uc5d0\uc11c\ub294 \uc0ac\uc2e4\uc0c1 \uc2b9\ub9ac\ud558\uc600\uc9c0\ub9cc, \uc774\ubbf8 \uc774\uae30\ud0dd\uc744 \ubc30\uc81c\ud558\uae30\ub85c \ub9c8\uc74c\uba39\uc740 \uae40\ub300\uc911\uc740 \ub3d9\uad50\ub3d9\uacc4 \uc758\uc6d0 \uc804\uc6d0\uc744 \ubbfc\uc8fc\ub2f9\uc5d0\uc11c \ud0c8\ub2f9\uc2dc\ucf1c \uc0c8\uc815\uce58\uad6d\ubbfc\ud68c\uc758\ub97c \ucc3d\ub2f9\ud558\uace0 \uc815\uacc4\uc5d0 \ubcf5\uadc0\ud558\uc600\ub2e4.", "paragraph_id": "6589939-1-1", "paragraph_question": "question: 1995\ub144 \uc9c0\ubc29\uc120\uac70\uc5d0\uc11c \uc5ec\ub2f9\uc744 \ubc00\uc5b4\ub0b4\uace0 \uc11c\uc6b8\uc2dc\uc7a5\uc744 \ucc28\uc9c0\ud55c \ub2f9\uc740?, context: 1992\ub144 5\uc6d4 26\uc77c \uc81c14\ub300 \ub300\ud1b5\ub839\ud6c4\ubcf4 \uacbd\uc120\uc5d0 \ucd9c\ub9c8\ud558\uc600\uc73c\ub098, \uacbd\uc120\uc5d0\uc11c \uae40\ub300\uc911 \ub300\ud45c\ucd5c\uace0\uc704\uc6d0\uc5d0\uac8c \uc11d\ud328\ud558\uc600\ub2e4. 12\uc6d4 \uae40\ub300\uc911\uc774 \ub300\ud1b5\ub839 \uc120\uac70\uc5d0\uc11c \ub099\uc120\ud558\uace0 \uc815\uacc4\ub97c \uc740\ud1f4\ud558\uc790, \uc77c\uc57d \uc57c\ub2f9\ucd1d\uc218\uac00 \ub418\uc5c8\ub2e4. 1993-1994\ub144\uc5d0\ub294 \uc81c1\uc57c\ub2f9\uc758 \ucd1d\uc218\ub85c\uc11c \uc804\uc131\uae30\ub97c \ub204\ub838\uc73c\ub098, \ub2f9\ub0b4 \ucd5c\ub300 \uc9c0\ubd84\uc744 \uc18c\uc720\ud558\uace0 \uc788\ub358 \ub3d9\uad50\ub3d9\uacc4\uc640 \ubc88\ubc88\uc774 \uc2e0\uacbd\uc804\uc744 \ubc8c\uc774\uba70 \uc8fc\ub3c4\uad8c \ub2e4\ud23c\uc744 \ud558\uc600\ub2e4. 1995\ub144 \uc9c0\ubc29\uc120\uac70\uc5d0\uc11c \uacf5\ucc9c\uad8c\uc744 \ub458\ub7ec\uc2f8\uace0 \ub3d9\uad50\ub3d9\uacc4\uc640 \uc7a6\uc740 \ucda9\ub3cc \ub05d\uc5d0 \uacb0\uad6d \ub418\ub3cc\uc544\uc62c \uc218 \uc5c6\ub294 \uac15\uc744 \uac74\ub11c\ub2e4. \ubbfc\uc8fc\ub2f9\uc740 \uc5ec\ub2f9\uc744 \ubc00\uc5b4\ub0b4\uace0 \uc11c\uc6b8\uc2dc\uc7a5\uc744 \ucc28\uc9c0\ud558\ub294\ub4f1 \uc9c0\ubc29\uc120\uac70\uc5d0\uc11c\ub294 \uc0ac\uc2e4\uc0c1 \uc2b9\ub9ac\ud558\uc600\uc9c0\ub9cc, \uc774\ubbf8 \uc774\uae30\ud0dd\uc744 \ubc30\uc81c\ud558\uae30\ub85c \ub9c8\uc74c\uba39\uc740 \uae40\ub300\uc911\uc740 \ub3d9\uad50\ub3d9\uacc4 \uc758\uc6d0 \uc804\uc6d0\uc744 \ubbfc\uc8fc\ub2f9\uc5d0\uc11c \ud0c8\ub2f9\uc2dc\ucf1c \uc0c8\uc815\uce58\uad6d\ubbfc\ud68c\uc758\ub97c \ucc3d\ub2f9\ud558\uace0 \uc815\uacc4\uc5d0 \ubcf5\uadc0\ud558\uc600\ub2e4."} {"question": "\ubcd1\ub825\uc774 16000\uba85\uc5d0\uc11c 8000\uba85\uae4c\uc9c0 \uc904\uc5b4\ub4e4\uc5b4 \ud070 \ud53c\ud574\ub97c \uc785\uc740 \uc870\uc120\uc778\ubbfc\uad70 \uad70\ub2e8\uc740?", "paragraph": "1\uc6d4 \ub9d0, \uc870\uc120\uc778\ubbfc\uad70 \uc81c2\uad70\ub2e8\uc740 \uc720\uaca9\uc804 \ub3d9\uc548 \ud070 \ud53c\ud574\ub97c \uc785\uc5c8\uc73c\uba70 \ubcd1\ub825\uc774 16,000\uba85\uc5d0\uc11c 8,000\uba85\uc73c\ub85c \uc904\uc5b4\ub4e0 \uac83\uc73c\ub85c \ucd94\uc815\ub41c\ub2e4. \uc870\uc120\uc778\ubbfc\uad70 \uc81c5\uad70\ub2e8\uc774 \uc6d0\uc8fc\ub97c \uc810\ub839\ud558\ub824\uace0 \ud588\ub358 \uc2dc\ub3c4 \ub610\ud55c \uc0ac\uc0c1\uc790\uac00 \uc99d\uac00\ud558\ub294 \uacb0\uacfc\ub97c \ub0b3\uc558\uc73c\uba70, \uc81c5\uad70\ub2e8\uc740 32,000\uba85\uc5d0\uc11c 22,000\uba85\uc73c\ub85c \ubcd1\ub825\uc774 \uac10\uc18c\ub41c \uac83\uc73c\ub85c \ucd94\uc0b0\ub418\uc5c8\ub2e4. \ub300\uc870\uc801\uc73c\ub85c \uc720\uc5d4\uad70\uc758 \uc190\uc2e4\uc740 \uac19\uc740 \uae30\uac04\uc5d0 \ube44\uad50\uc801 \uc801\uc740 \ud53c\ud574\ub97c \uc785\uc5c8\ub2e4. \ubd81\ud55c\uad70\uc758 \ud328\ubc30\ub85c \uc720\uc5d4\uad70\uc740 \ud55c\ubc18\ub3c4\uc758 \uc911\ubd80 \uc804\uc120 \ubc0f \ub3d9\ubd80 \uc804\uc120\uc744 \uac15\ud654\uc2dc\ud0ac \uc218 \uc788\uc5c8\uace0, \uadf8\ub4e4\uc758 \uc804\ubc29\uacfc \ub3d9\uce21\ubc29\uc774 \ud655\ubcf4\ub41c \uc774\ud6c4 \uc11c\ubd80 \uc804\uc120\uc5d0\uc11c \ud1f4\uac01\ud558\ub358 \ubbf8\uad6d \uc81c8\uad70\uc740 \ub9c8\uce68\ub0b4 \uacf5\uc138\ub85c \uc804\ud658\ud560 \uc218 \uc788\uac8c \ub418\uc5c8\ub2e4. \ubbf8\uad6d \uc81c10\uad70\ub2e8\uc774 1\uc6d4 \ub9d0 \uc911\ubd80 \uc804\uc120\uc744 \uc644\uc804\ud788 \uc7a5\uc545\ud55c \uc774\ud6c4 \ub9ac\uc9c0\uc6e8\uc774\ub294 \uace7\ubc14\ub85c \ubbf8\uad6d \uc81c8\uad70\uc5d0\uac8c 1951\ub144 1\uc6d4 25\uc77c \uc911\uacf5\uad70\uacfc \ubd81\ud55c\uad70\uc5d0 \ub9de\uc11c \uc120\ub354\ubcfc\ud2b8 \uc791\uc804\uc744 \uac1c\uc2dc\ud560 \uac83\uc744 \uba85\ub839\ud588\ub2e4.", "answer": "\uc870\uc120\uc778\ubbfc\uad70 \uc81c2\uad70\ub2e8", "sentence": "1\uc6d4 \ub9d0, \uc870\uc120\uc778\ubbfc\uad70 \uc81c2\uad70\ub2e8 \uc740 \uc720\uaca9\uc804 \ub3d9\uc548 \ud070 \ud53c\ud574\ub97c \uc785\uc5c8\uc73c\uba70 \ubcd1\ub825\uc774 16,000\uba85\uc5d0\uc11c 8,000\uba85\uc73c\ub85c \uc904\uc5b4\ub4e0 \uac83\uc73c\ub85c \ucd94\uc815\ub41c\ub2e4.", "paragraph_sentence": " 1\uc6d4 \ub9d0, \uc870\uc120\uc778\ubbfc\uad70 \uc81c2\uad70\ub2e8 \uc740 \uc720\uaca9\uc804 \ub3d9\uc548 \ud070 \ud53c\ud574\ub97c \uc785\uc5c8\uc73c\uba70 \ubcd1\ub825\uc774 16,000\uba85\uc5d0\uc11c 8,000\uba85\uc73c\ub85c \uc904\uc5b4\ub4e0 \uac83\uc73c\ub85c \ucd94\uc815\ub41c\ub2e4. \uc870\uc120\uc778\ubbfc\uad70 \uc81c5\uad70\ub2e8\uc774 \uc6d0\uc8fc\ub97c \uc810\ub839\ud558\ub824\uace0 \ud588\ub358 \uc2dc\ub3c4 \ub610\ud55c \uc0ac\uc0c1\uc790\uac00 \uc99d\uac00\ud558\ub294 \uacb0\uacfc\ub97c \ub0b3\uc558\uc73c\uba70, \uc81c5\uad70\ub2e8\uc740 32,000\uba85\uc5d0\uc11c 22,000\uba85\uc73c\ub85c \ubcd1\ub825\uc774 \uac10\uc18c\ub41c \uac83\uc73c\ub85c \ucd94\uc0b0\ub418\uc5c8\ub2e4. \ub300\uc870\uc801\uc73c\ub85c \uc720\uc5d4\uad70\uc758 \uc190\uc2e4\uc740 \uac19\uc740 \uae30\uac04\uc5d0 \ube44\uad50\uc801 \uc801\uc740 \ud53c\ud574\ub97c \uc785\uc5c8\ub2e4. \ubd81\ud55c\uad70\uc758 \ud328\ubc30\ub85c \uc720\uc5d4\uad70\uc740 \ud55c\ubc18\ub3c4\uc758 \uc911\ubd80 \uc804\uc120 \ubc0f \ub3d9\ubd80 \uc804\uc120\uc744 \uac15\ud654\uc2dc\ud0ac \uc218 \uc788\uc5c8\uace0, \uadf8\ub4e4\uc758 \uc804\ubc29\uacfc \ub3d9\uce21\ubc29\uc774 \ud655\ubcf4\ub41c \uc774\ud6c4 \uc11c\ubd80 \uc804\uc120\uc5d0\uc11c \ud1f4\uac01\ud558\ub358 \ubbf8\uad6d \uc81c8\uad70\uc740 \ub9c8\uce68\ub0b4 \uacf5\uc138\ub85c \uc804\ud658\ud560 \uc218 \uc788\uac8c \ub418\uc5c8\ub2e4. \ubbf8\uad6d \uc81c10\uad70\ub2e8\uc774 1\uc6d4 \ub9d0 \uc911\ubd80 \uc804\uc120\uc744 \uc644\uc804\ud788 \uc7a5\uc545\ud55c \uc774\ud6c4 \ub9ac\uc9c0\uc6e8\uc774\ub294 \uace7\ubc14\ub85c \ubbf8\uad6d \uc81c8\uad70\uc5d0\uac8c 1951\ub144 1\uc6d4 25\uc77c \uc911\uacf5\uad70\uacfc \ubd81\ud55c\uad70\uc5d0 \ub9de\uc11c \uc120\ub354\ubcfc\ud2b8 \uc791\uc804\uc744 \uac1c\uc2dc\ud560 \uac83\uc744 \uba85\ub839\ud588\ub2e4.", "paragraph_answer": "1\uc6d4 \ub9d0, \uc870\uc120\uc778\ubbfc\uad70 \uc81c2\uad70\ub2e8 \uc740 \uc720\uaca9\uc804 \ub3d9\uc548 \ud070 \ud53c\ud574\ub97c \uc785\uc5c8\uc73c\uba70 \ubcd1\ub825\uc774 16,000\uba85\uc5d0\uc11c 8,000\uba85\uc73c\ub85c \uc904\uc5b4\ub4e0 \uac83\uc73c\ub85c \ucd94\uc815\ub41c\ub2e4. \uc870\uc120\uc778\ubbfc\uad70 \uc81c5\uad70\ub2e8\uc774 \uc6d0\uc8fc\ub97c \uc810\ub839\ud558\ub824\uace0 \ud588\ub358 \uc2dc\ub3c4 \ub610\ud55c \uc0ac\uc0c1\uc790\uac00 \uc99d\uac00\ud558\ub294 \uacb0\uacfc\ub97c \ub0b3\uc558\uc73c\uba70, \uc81c5\uad70\ub2e8\uc740 32,000\uba85\uc5d0\uc11c 22,000\uba85\uc73c\ub85c \ubcd1\ub825\uc774 \uac10\uc18c\ub41c \uac83\uc73c\ub85c \ucd94\uc0b0\ub418\uc5c8\ub2e4. \ub300\uc870\uc801\uc73c\ub85c \uc720\uc5d4\uad70\uc758 \uc190\uc2e4\uc740 \uac19\uc740 \uae30\uac04\uc5d0 \ube44\uad50\uc801 \uc801\uc740 \ud53c\ud574\ub97c \uc785\uc5c8\ub2e4. \ubd81\ud55c\uad70\uc758 \ud328\ubc30\ub85c \uc720\uc5d4\uad70\uc740 \ud55c\ubc18\ub3c4\uc758 \uc911\ubd80 \uc804\uc120 \ubc0f \ub3d9\ubd80 \uc804\uc120\uc744 \uac15\ud654\uc2dc\ud0ac \uc218 \uc788\uc5c8\uace0, \uadf8\ub4e4\uc758 \uc804\ubc29\uacfc \ub3d9\uce21\ubc29\uc774 \ud655\ubcf4\ub41c \uc774\ud6c4 \uc11c\ubd80 \uc804\uc120\uc5d0\uc11c \ud1f4\uac01\ud558\ub358 \ubbf8\uad6d \uc81c8\uad70\uc740 \ub9c8\uce68\ub0b4 \uacf5\uc138\ub85c \uc804\ud658\ud560 \uc218 \uc788\uac8c \ub418\uc5c8\ub2e4. \ubbf8\uad6d \uc81c10\uad70\ub2e8\uc774 1\uc6d4 \ub9d0 \uc911\ubd80 \uc804\uc120\uc744 \uc644\uc804\ud788 \uc7a5\uc545\ud55c \uc774\ud6c4 \ub9ac\uc9c0\uc6e8\uc774\ub294 \uace7\ubc14\ub85c \ubbf8\uad6d \uc81c8\uad70\uc5d0\uac8c 1951\ub144 1\uc6d4 25\uc77c \uc911\uacf5\uad70\uacfc \ubd81\ud55c\uad70\uc5d0 \ub9de\uc11c \uc120\ub354\ubcfc\ud2b8 \uc791\uc804\uc744 \uac1c\uc2dc\ud560 \uac83\uc744 \uba85\ub839\ud588\ub2e4.", "sentence_answer": "1\uc6d4 \ub9d0, \uc870\uc120\uc778\ubbfc\uad70 \uc81c2\uad70\ub2e8 \uc740 \uc720\uaca9\uc804 \ub3d9\uc548 \ud070 \ud53c\ud574\ub97c \uc785\uc5c8\uc73c\uba70 \ubcd1\ub825\uc774 16,000\uba85\uc5d0\uc11c 8,000\uba85\uc73c\ub85c \uc904\uc5b4\ub4e0 \uac83\uc73c\ub85c \ucd94\uc815\ub41c\ub2e4.", "paragraph_id": "6459762-13-1", "paragraph_question": "question: \ubcd1\ub825\uc774 16000\uba85\uc5d0\uc11c 8000\uba85\uae4c\uc9c0 \uc904\uc5b4\ub4e4\uc5b4 \ud070 \ud53c\ud574\ub97c \uc785\uc740 \uc870\uc120\uc778\ubbfc\uad70 \uad70\ub2e8\uc740?, context: 1\uc6d4 \ub9d0, \uc870\uc120\uc778\ubbfc\uad70 \uc81c2\uad70\ub2e8\uc740 \uc720\uaca9\uc804 \ub3d9\uc548 \ud070 \ud53c\ud574\ub97c \uc785\uc5c8\uc73c\uba70 \ubcd1\ub825\uc774 16,000\uba85\uc5d0\uc11c 8,000\uba85\uc73c\ub85c \uc904\uc5b4\ub4e0 \uac83\uc73c\ub85c \ucd94\uc815\ub41c\ub2e4. \uc870\uc120\uc778\ubbfc\uad70 \uc81c5\uad70\ub2e8\uc774 \uc6d0\uc8fc\ub97c \uc810\ub839\ud558\ub824\uace0 \ud588\ub358 \uc2dc\ub3c4 \ub610\ud55c \uc0ac\uc0c1\uc790\uac00 \uc99d\uac00\ud558\ub294 \uacb0\uacfc\ub97c \ub0b3\uc558\uc73c\uba70, \uc81c5\uad70\ub2e8\uc740 32,000\uba85\uc5d0\uc11c 22,000\uba85\uc73c\ub85c \ubcd1\ub825\uc774 \uac10\uc18c\ub41c \uac83\uc73c\ub85c \ucd94\uc0b0\ub418\uc5c8\ub2e4. \ub300\uc870\uc801\uc73c\ub85c \uc720\uc5d4\uad70\uc758 \uc190\uc2e4\uc740 \uac19\uc740 \uae30\uac04\uc5d0 \ube44\uad50\uc801 \uc801\uc740 \ud53c\ud574\ub97c \uc785\uc5c8\ub2e4. \ubd81\ud55c\uad70\uc758 \ud328\ubc30\ub85c \uc720\uc5d4\uad70\uc740 \ud55c\ubc18\ub3c4\uc758 \uc911\ubd80 \uc804\uc120 \ubc0f \ub3d9\ubd80 \uc804\uc120\uc744 \uac15\ud654\uc2dc\ud0ac \uc218 \uc788\uc5c8\uace0, \uadf8\ub4e4\uc758 \uc804\ubc29\uacfc \ub3d9\uce21\ubc29\uc774 \ud655\ubcf4\ub41c \uc774\ud6c4 \uc11c\ubd80 \uc804\uc120\uc5d0\uc11c \ud1f4\uac01\ud558\ub358 \ubbf8\uad6d \uc81c8\uad70\uc740 \ub9c8\uce68\ub0b4 \uacf5\uc138\ub85c \uc804\ud658\ud560 \uc218 \uc788\uac8c \ub418\uc5c8\ub2e4. \ubbf8\uad6d \uc81c10\uad70\ub2e8\uc774 1\uc6d4 \ub9d0 \uc911\ubd80 \uc804\uc120\uc744 \uc644\uc804\ud788 \uc7a5\uc545\ud55c \uc774\ud6c4 \ub9ac\uc9c0\uc6e8\uc774\ub294 \uace7\ubc14\ub85c \ubbf8\uad6d \uc81c8\uad70\uc5d0\uac8c 1951\ub144 1\uc6d4 25\uc77c \uc911\uacf5\uad70\uacfc \ubd81\ud55c\uad70\uc5d0 \ub9de\uc11c \uc120\ub354\ubcfc\ud2b8 \uc791\uc804\uc744 \uac1c\uc2dc\ud560 \uac83\uc744 \uba85\ub839\ud588\ub2e4."} {"question": "\ub048 \uc774\ub860\uc774 \ucd08\ub300\uce6d\uc744 \ud3ec\ud568\ud558\uc9c0 \uc54a\ub294 \uacbd\uc6b0\ub97c \ubb34\uc5c7\uc774\ub77c \ubd80\ub974\ub098?", "paragraph": "\ucd08\ub300\uce6d\uc744 \ud3ec\ud568\ud558\uc9c0 \uc54a\ub294 \ub048 \uc774\ub860\uc744 \ubcf4\uc190 \ub048 \uc774\ub860\uc774\ub77c \ud55c\ub2e4. \uc774 \uacbd\uc6b0 \uc5f4\ub9b0 \ub048\uc744 \ud5c8\uc6a9\ud560 \uc218 \uc788\uace0, \ud5c8\uc6a9\ud558\uc9c0 \uc54a\uc544\ub3c4 \ub41c\ub2e4. \ubcf4\uc190 \ub048 \uc774\ub860\uc740 \uadf8\ub7ec\ub098 \ucd08\ub300\uce6d\uc774 \uc5c6\uc73c\ubbc0\ub85c \ud398\ub974\ubbf8\uc628\uc744 \ud3ec\ud568\ud558\uc9c0 \uc54a\ub294\ub2e4. \ub610\ud55c \uc774\ub4e4\uc740 \ud0c0\ud0a4\uc628\uc744 \uc9c0\ub140, \uac74\ub4dc\ub9bc\uc774\ub860\uc744 \ubbff\uc744 \uc218 \uc5c6\uace0, \ud0c0\ud0a4\uc628 \uc751\ucd95\uc744 \ubd84\uc11d\ud558\uc5ec \ucc38 \uc9c4\uacf5\uc774 \uc874\uc7ac\ud558\ub294\uc9c0, \uc874\uc7ac\ud55c\ub2e4\uba74 \uc5b4\ub514\uc778\uc9c0\ub97c \ucc3e\uc544\uc57c \ud55c\ub2e4. \uc5f4\ub9b0 \ub048\uc758 \uacbd\uc6b0\uc5d0\ub294 \ub048 \uc7a5\ub860\uc744 \uc368\uc11c \ucc38 \uc9c4\uacf5\uc774 \uc874\uc7ac\ud55c\ub2e4\ub294 \uc0ac\uc2e4\uc744 \ubc1d\ud614\uc73c\ub098, \uc544\uc9c1 \ub2eb\ud78c \ub048\uc740 \ucc38 \uc9c4\uacf5\uc774 \uc5b4\ub514\uc778\uc9c0 \uc54c\uc9c0 \ubabb\ud55c\ub2e4. \ubcf4\uc190 \ub048 \uc774\ub860\uc740 26\ucc28\uc6d0\uc5d0\uc11c \uc874\uc7ac\ud55c\ub2e4. \uc5f4\ub9b0 \ub048 \uc774\ub860\uc740 \ubcc0\uce59\uc744 \ud53c\ud558\uae30 \uc704\ud558\uc5ec SO(8192)\uc758 \uac8c\uc774\uc9c0 \ub300\uce6d\uc744 \uc9c0\ub140\uc57c \ud55c\ub2e4. \uc774\ub294 8192\uac1c\uc758 D-\ub9c9\uc774 \uc11c\ub85c \uacb9\uccd0 \uc788\ub294 \uac83\uc73c\ub85c \uc774\ud574\ud560 \uc218 \uc788\ub2e4.", "answer": "\ubcf4\uc190 \ub048 \uc774\ub860", "sentence": "\ucd08\ub300\uce6d\uc744 \ud3ec\ud568\ud558\uc9c0 \uc54a\ub294 \ub048 \uc774\ub860\uc744 \ubcf4\uc190 \ub048 \uc774\ub860 \uc774\ub77c \ud55c\ub2e4.", "paragraph_sentence": " \ucd08\ub300\uce6d\uc744 \ud3ec\ud568\ud558\uc9c0 \uc54a\ub294 \ub048 \uc774\ub860\uc744 \ubcf4\uc190 \ub048 \uc774\ub860 \uc774\ub77c \ud55c\ub2e4. \uc774 \uacbd\uc6b0 \uc5f4\ub9b0 \ub048\uc744 \ud5c8\uc6a9\ud560 \uc218 \uc788\uace0, \ud5c8\uc6a9\ud558\uc9c0 \uc54a\uc544\ub3c4 \ub41c\ub2e4. \ubcf4\uc190 \ub048 \uc774\ub860\uc740 \uadf8\ub7ec\ub098 \ucd08\ub300\uce6d\uc774 \uc5c6\uc73c\ubbc0\ub85c \ud398\ub974\ubbf8\uc628\uc744 \ud3ec\ud568\ud558\uc9c0 \uc54a\ub294\ub2e4. \ub610\ud55c \uc774\ub4e4\uc740 \ud0c0\ud0a4\uc628\uc744 \uc9c0\ub140, \uac74\ub4dc\ub9bc\uc774\ub860\uc744 \ubbff\uc744 \uc218 \uc5c6\uace0, \ud0c0\ud0a4\uc628 \uc751\ucd95\uc744 \ubd84\uc11d\ud558\uc5ec \ucc38 \uc9c4\uacf5\uc774 \uc874\uc7ac\ud558\ub294\uc9c0, \uc874\uc7ac\ud55c\ub2e4\uba74 \uc5b4\ub514\uc778\uc9c0\ub97c \ucc3e\uc544\uc57c \ud55c\ub2e4. \uc5f4\ub9b0 \ub048\uc758 \uacbd\uc6b0\uc5d0\ub294 \ub048 \uc7a5\ub860\uc744 \uc368\uc11c \ucc38 \uc9c4\uacf5\uc774 \uc874\uc7ac\ud55c\ub2e4\ub294 \uc0ac\uc2e4\uc744 \ubc1d\ud614\uc73c\ub098, \uc544\uc9c1 \ub2eb\ud78c \ub048\uc740 \ucc38 \uc9c4\uacf5\uc774 \uc5b4\ub514\uc778\uc9c0 \uc54c\uc9c0 \ubabb\ud55c\ub2e4. \ubcf4\uc190 \ub048 \uc774\ub860\uc740 26\ucc28\uc6d0\uc5d0\uc11c \uc874\uc7ac\ud55c\ub2e4. \uc5f4\ub9b0 \ub048 \uc774\ub860\uc740 \ubcc0\uce59\uc744 \ud53c\ud558\uae30 \uc704\ud558\uc5ec SO(8192)\uc758 \uac8c\uc774\uc9c0 \ub300\uce6d\uc744 \uc9c0\ub140\uc57c \ud55c\ub2e4. \uc774\ub294 8192\uac1c\uc758 D-\ub9c9\uc774 \uc11c\ub85c \uacb9\uccd0 \uc788\ub294 \uac83\uc73c\ub85c \uc774\ud574\ud560 \uc218 \uc788\ub2e4.", "paragraph_answer": "\ucd08\ub300\uce6d\uc744 \ud3ec\ud568\ud558\uc9c0 \uc54a\ub294 \ub048 \uc774\ub860\uc744 \ubcf4\uc190 \ub048 \uc774\ub860 \uc774\ub77c \ud55c\ub2e4. \uc774 \uacbd\uc6b0 \uc5f4\ub9b0 \ub048\uc744 \ud5c8\uc6a9\ud560 \uc218 \uc788\uace0, \ud5c8\uc6a9\ud558\uc9c0 \uc54a\uc544\ub3c4 \ub41c\ub2e4. \ubcf4\uc190 \ub048 \uc774\ub860\uc740 \uadf8\ub7ec\ub098 \ucd08\ub300\uce6d\uc774 \uc5c6\uc73c\ubbc0\ub85c \ud398\ub974\ubbf8\uc628\uc744 \ud3ec\ud568\ud558\uc9c0 \uc54a\ub294\ub2e4. \ub610\ud55c \uc774\ub4e4\uc740 \ud0c0\ud0a4\uc628\uc744 \uc9c0\ub140, \uac74\ub4dc\ub9bc\uc774\ub860\uc744 \ubbff\uc744 \uc218 \uc5c6\uace0, \ud0c0\ud0a4\uc628 \uc751\ucd95\uc744 \ubd84\uc11d\ud558\uc5ec \ucc38 \uc9c4\uacf5\uc774 \uc874\uc7ac\ud558\ub294\uc9c0, \uc874\uc7ac\ud55c\ub2e4\uba74 \uc5b4\ub514\uc778\uc9c0\ub97c \ucc3e\uc544\uc57c \ud55c\ub2e4. \uc5f4\ub9b0 \ub048\uc758 \uacbd\uc6b0\uc5d0\ub294 \ub048 \uc7a5\ub860\uc744 \uc368\uc11c \ucc38 \uc9c4\uacf5\uc774 \uc874\uc7ac\ud55c\ub2e4\ub294 \uc0ac\uc2e4\uc744 \ubc1d\ud614\uc73c\ub098, \uc544\uc9c1 \ub2eb\ud78c \ub048\uc740 \ucc38 \uc9c4\uacf5\uc774 \uc5b4\ub514\uc778\uc9c0 \uc54c\uc9c0 \ubabb\ud55c\ub2e4. \ubcf4\uc190 \ub048 \uc774\ub860\uc740 26\ucc28\uc6d0\uc5d0\uc11c \uc874\uc7ac\ud55c\ub2e4. \uc5f4\ub9b0 \ub048 \uc774\ub860\uc740 \ubcc0\uce59\uc744 \ud53c\ud558\uae30 \uc704\ud558\uc5ec SO(8192)\uc758 \uac8c\uc774\uc9c0 \ub300\uce6d\uc744 \uc9c0\ub140\uc57c \ud55c\ub2e4. \uc774\ub294 8192\uac1c\uc758 D-\ub9c9\uc774 \uc11c\ub85c \uacb9\uccd0 \uc788\ub294 \uac83\uc73c\ub85c \uc774\ud574\ud560 \uc218 \uc788\ub2e4.", "sentence_answer": "\ucd08\ub300\uce6d\uc744 \ud3ec\ud568\ud558\uc9c0 \uc54a\ub294 \ub048 \uc774\ub860\uc744 \ubcf4\uc190 \ub048 \uc774\ub860 \uc774\ub77c \ud55c\ub2e4.", "paragraph_id": "6488659-1-0", "paragraph_question": "question: \ub048 \uc774\ub860\uc774 \ucd08\ub300\uce6d\uc744 \ud3ec\ud568\ud558\uc9c0 \uc54a\ub294 \uacbd\uc6b0\ub97c \ubb34\uc5c7\uc774\ub77c \ubd80\ub974\ub098?, context: \ucd08\ub300\uce6d\uc744 \ud3ec\ud568\ud558\uc9c0 \uc54a\ub294 \ub048 \uc774\ub860\uc744 \ubcf4\uc190 \ub048 \uc774\ub860\uc774\ub77c \ud55c\ub2e4. \uc774 \uacbd\uc6b0 \uc5f4\ub9b0 \ub048\uc744 \ud5c8\uc6a9\ud560 \uc218 \uc788\uace0, \ud5c8\uc6a9\ud558\uc9c0 \uc54a\uc544\ub3c4 \ub41c\ub2e4. \ubcf4\uc190 \ub048 \uc774\ub860\uc740 \uadf8\ub7ec\ub098 \ucd08\ub300\uce6d\uc774 \uc5c6\uc73c\ubbc0\ub85c \ud398\ub974\ubbf8\uc628\uc744 \ud3ec\ud568\ud558\uc9c0 \uc54a\ub294\ub2e4. \ub610\ud55c \uc774\ub4e4\uc740 \ud0c0\ud0a4\uc628\uc744 \uc9c0\ub140, \uac74\ub4dc\ub9bc\uc774\ub860\uc744 \ubbff\uc744 \uc218 \uc5c6\uace0, \ud0c0\ud0a4\uc628 \uc751\ucd95\uc744 \ubd84\uc11d\ud558\uc5ec \ucc38 \uc9c4\uacf5\uc774 \uc874\uc7ac\ud558\ub294\uc9c0, \uc874\uc7ac\ud55c\ub2e4\uba74 \uc5b4\ub514\uc778\uc9c0\ub97c \ucc3e\uc544\uc57c \ud55c\ub2e4. \uc5f4\ub9b0 \ub048\uc758 \uacbd\uc6b0\uc5d0\ub294 \ub048 \uc7a5\ub860\uc744 \uc368\uc11c \ucc38 \uc9c4\uacf5\uc774 \uc874\uc7ac\ud55c\ub2e4\ub294 \uc0ac\uc2e4\uc744 \ubc1d\ud614\uc73c\ub098, \uc544\uc9c1 \ub2eb\ud78c \ub048\uc740 \ucc38 \uc9c4\uacf5\uc774 \uc5b4\ub514\uc778\uc9c0 \uc54c\uc9c0 \ubabb\ud55c\ub2e4. \ubcf4\uc190 \ub048 \uc774\ub860\uc740 26\ucc28\uc6d0\uc5d0\uc11c \uc874\uc7ac\ud55c\ub2e4. \uc5f4\ub9b0 \ub048 \uc774\ub860\uc740 \ubcc0\uce59\uc744 \ud53c\ud558\uae30 \uc704\ud558\uc5ec SO(8192)\uc758 \uac8c\uc774\uc9c0 \ub300\uce6d\uc744 \uc9c0\ub140\uc57c \ud55c\ub2e4. \uc774\ub294 8192\uac1c\uc758 D-\ub9c9\uc774 \uc11c\ub85c \uacb9\uccd0 \uc788\ub294 \uac83\uc73c\ub85c \uc774\ud574\ud560 \uc218 \uc788\ub2e4."} {"question": "\uae40\ubcf4\ub984 \uc120\uc218\uc758 \uc804\ud5a5\uc5d0 \uc601\ud5a5\uc744 \ubbf8\uce5c \uc120\uc218\ub294?", "paragraph": "\ucd08\ub4f1\ud559\uad50 5\ud559\ub144 \ub54c \uc1fc\ud2b8\ud2b8\ub799 \uc120\uc218\ub85c \uc785\ubb38\ud55c \uae40\ubcf4\ub984\uc740 \ub450\uac01\uc744 \ub098\ud0c0\ub0b4\uc9c0 \ubabb\ud588\uace0, \uacb0\uad6d \uace0\ub4f1\ud559\uad50 2\ud559\ub144 \ub54c \uc2a4\ud53c\ub4dc \uc2a4\ucf00\uc774\ud305\uc73c\ub85c \uc804\ud5a5\ud588\ub2e4. \"\uc544\ubb34\ub798\ub3c4 \uc62c\ub9bc\ud53d\uc5d0\uc11c \uc774\uc2b9\ud6c8 \uc120\uc218\uac00 \ud65c\uc57d\ud588\ub358 \uac8c \uc601\ud5a5\uc774 \uac00\uc7a5 \ucef8\uc8e0. \ud558\uc9c0\ub9cc 2010\ub144\ub3c4 \uc804\ubd80\ud130 \uc2a4\ud53c\ub4dc\ub97c \ud0c0\uace0 \uc2f6\ub2e4\ub294 \uc0dd\uac01\uc740 \ud588\uc5c8\uc5b4\uc694.\" \"\uc2a4\ud53c\ub4dc \uc2a4\ucf00\uc774\ud305\uc758 \ud63c\uc790\ub9cc\uc758 \uc2f8\uc6c0\uc774 \ub9e4\ub825\uc801\uc774\ub77c \uc804\ud5a5 \uacb0\uc2ec\ud588\uc2b5\ub2c8\ub2e4.\" \"\ub531\ud788 \uacc4\uae30\ub77c\uace0 \ud560 \uac74 \uc5c6\uace0 \uc6b4\ub3d9\uc744 \uadf8\ub9cc\ub450\ub824\uace0 \ud588\uc5b4\uc694. \uadf8\ub7ec\ub2e4\uac00 \ub9c8\uc9c0\ub9c9\uc73c\ub85c '\uc2a4\ud53c\ub4dc \uc2a4\ucf00\uc774\ud305 \ud55c\ubc88 \ud574\ubcf4\uace0 \uadf8\ub9cc\ub450\uc790'\ud574\uc11c \uc804\ud5a5\uc744 \ud588\ub294\ub370 \uc9c0\uae08 \uc5ec\uae30\uae4c\uc9c0 \uc628 \uac70\uc8e0. \uc1fc\ud2b8\ud2b8\ub799 \ud560 \ub54c \uadf8\ub807\uac8c \uc798 \ud0d4\ub358 \uc120\uc218\ub3c4 \uc544\ub2c8\uc5c8\uace0 \uc131\uc801\ub3c4 \uacc4\uc18d \uc548\ub098\uc654\uace0 \ub300\ud559\uc5d0 \uac00\uc11c \uc9c4\ub85c \uacb0\uc815\uc744 \ud655\uc2e4\ud788 \ud574\uc57c \ud558\ub358 \uc2dc\uae30\uc5ec\uc11c \uc804\ud5a5\uc744 \ud558\uac8c \ub410\uc5b4\uc694. \"\uc544\ubb34\ub798\ub3c4 \uc2dc\uc791\uc774 \ub2a6\ub2e4 \ubcf4\ub2c8 \uc758\uc695\uacfc\ub294 \ub2ec\ub9ac \ub610\ub798 \uce5c\uad6c\ub4e4\uacfc\uc758 \uaca9\ucc28\ub97c \uc904\uc774\uae30\uac00 \ubc85\ucc28\ub354\ub77c\uace0\uc694. \uacb0\uad6d\uc5d4 \uc1fc\ud2b8\ud2b8\ub799\uc5d0 \ud765\ubbf8\ub97c \uc783\uc5c8\uace0, \uc911\ud559\uad50 \uc785\ud559 \ud6c4\uc5d0\ub294 \uc6b4\ub3d9\uc744 \uadf8\ub9cc \ub450\uac8c \ub40f\uc8e0. \ub2e4\uc2dc \uc1fc\ud2b8\ud2b8\ub799\uc744 \uc2dc\uc791\ud558\uba74\uc11c \uc774\uc804\ubcf4\ub2e4\ub294 \ud655\uc2e4\ud788 \uad1c\ucc2e\uc558\uc5b4\uc694. 2009\ub144 \uc804\uad6d\ub3d9\uacc4\uccb4\uc721\ub300\ud68c \uc1fc\ud2b8\ud2b8\ub799 1500m \ubd80\ubb38\uc5d0\uc11c \ub3d9\uba54\ub2ec\uc744 \ub534 \uc801\ub3c4 \uc788\uc5c8\uac70\ub4e0\uc694. \uadf8\ub807\uc9c0\ub9cc \uc800\ub294 \uc1fc\ud2b8\ud2b8\ub799 \uc120\uc218\ub85c\uc11c\uc758 \uac00\ub2a5\uc131\uc774 \uadf8\ub807\uac8c \ud070 \uc120\uc218\ub294 \uc544\ub2c8\uc5c8\uc8e0. \uc1fc\ud2b8\ud2b8\ub799\uc73c\ub85c\ub294 \ud55c\uacc4\uac00 \uc788\ub2e4\uace0 \uc0dd\uac01\uc744 \ud588\uc5b4\uc694\" \"\uc804\ud5a5\uc5d0 \ub300\ud574 \uc8fc\uc704\uc5d0\uc11c \ubc18\ub300\ub97c \ub9ce\uc774 \ud588\uc5b4\uc694. \uc5f4 \uba85\uc774\uba74 \uc5f4 \uba85 \ub2e4 \uc88b\uc9c0 \uc54a\uac8c \ubcf4\uc2dc\ub354\ub77c\uace0\uc694. \ub204\uad70\uac00 '\uadf8\ub0e5 \uc1fc\ud2b8\ud2b8\ub799 \ud0c0'\ub77c\uace0 \ub9d0\ud588\ub2e4\uba74 \uc624\ud788\ub824 \uad1c\ucc2e\uc558\uc744 \uac70\uc608\uc694. \uadf8\ub7f0\ub370 \ub2e4\ub4e4 '\ub124\uac00 \ud560 \uc218 \uc788\uc744 \uac83 \uac19\ub0d0'\uace0 \ud558\uc2dc\ub354\ub77c\uace0\uc694. \uc2a4\ud53c\ub4dc\uc2a4\ucf00\uc774\ud305\uc5d0\uc11c\ub3c4 \uadf8\uc800\uadf8\ub7f0 \uc120\uc218\ub85c \ub0a8\uc73c\uba74 \uc800\ub97c \ube44\ub09c\ud588\ub358 \uc0ac\ub78c\ub4e4\uc758 \uc0dd\uac01\ucc98\ub7fc \ub418\ub294 \uac70\uc796\uc544\uc694. \ubb54\uac00 \uc624\uae30, \ubaa9\ud45c\uac00 \uc0dd\uacbc\uc5b4\uc694.\" \"\uc1fc\ud2b8\ud2b8\ub799\uc5d0 \ub2a6\uac8c \uc785\ubb38\ud588\uace0, \uc2a4\ud53c\ub4dc\uc2a4\ucf00\uc774\ud305\ub3c4 \uc5c4\uccad\ub098\uac8c \ub2a6\uac8c \uc2dc\uc791\ud588\uc9c0\ub9cc, \uc660\uc9c0 \ubaa8\ub974\uac8c \ub9c8\uc74c\uc774 \ud3b8\ud588\ub2e4. \ub9c8\uc9c0\ub9c9 \uad81\uc9c0\uc5d0 \ubab0\ub9b0 \ud130\ub77c \ub354 \uadf8\ub7ac\ub358 \uac83 \uac19\ub2e4.\" \"\uc774\uc2b9\ud6c8\uc740 \uc1fc\ud2b8\ud2b8\ub799\uc5d0\uc11c\ub3c4 \uad6d\uac00\ub300\ud45c\ub97c \uc9c0\ub0c8\uc9c0\ub9cc \uae40\ubcf4\ub984\uc740 \ub2e4\ub974\ub2e4. \uae40\ubcf4\ub984\uc740 \uc1fc\ud2b8\ud2b8\ub799\uc5d0\uc11c \ud06c\uac8c \ub450\uac01\uc744 \ubcf4\uc774\uc9c0 \ubabb\ud558\uace0 \uc885\ubaa9\uc744 \uc804\ud658\ud574 \uc131\uacf5\ud588\ub2e4. \ub354 \ub9ce\uc740 \uc120\uc218\ub4e4\uc5d0\uac8c \ud76c\ub9dd\uc744 \uc8fc\uae30\ub3c4 \ud55c\ub2e4.\" \uc2e4\ud328\ud588\ub358 \uae30\uac04\uc73c\ub85c \ubcf4\uc600\ub358 \uc1fc\ud2b8\ud2b8\ub799 \uacbd\ud5d8\uc774 \ud6d7\ub0a0 \uc8fc\uc885\ubaa9\uc774 \ub41c \ub9e4\uc2a4\uc2a4\ud0c0\ud2b8\ub97c \uc704\ud55c \uae40\ubcf4\ub984\uc758 \uac00\uc7a5 \uadc0\ud55c \uc790\uc0b0\uc774 \ub41c\ub2e4. \"\uc1fc\ud2b8\ud2b8\ub799 \uacbd\ud5d8\uc774 \ub9e4\uc2a4\uc2a4\ud0c0\ud2b8\uc5d0 \ub9ce\uc740 \ub3c4\uc6c0\uc744 \uc900\ub2e4.\"", "answer": "\uc774\uc2b9\ud6c8 \uc120\uc218", "sentence": "\"\uc544\ubb34\ub798\ub3c4 \uc62c\ub9bc\ud53d\uc5d0\uc11c \uc774\uc2b9\ud6c8 \uc120\uc218 \uac00 \ud65c\uc57d\ud588\ub358 \uac8c \uc601\ud5a5\uc774 \uac00\uc7a5 \ucef8\uc8e0.", "paragraph_sentence": "\ucd08\ub4f1\ud559\uad50 5\ud559\ub144 \ub54c \uc1fc\ud2b8\ud2b8\ub799 \uc120\uc218\ub85c \uc785\ubb38\ud55c \uae40\ubcf4\ub984\uc740 \ub450\uac01\uc744 \ub098\ud0c0\ub0b4\uc9c0 \ubabb\ud588\uace0, \uacb0\uad6d \uace0\ub4f1\ud559\uad50 2\ud559\ub144 \ub54c \uc2a4\ud53c\ub4dc \uc2a4\ucf00\uc774\ud305\uc73c\ub85c \uc804\ud5a5\ud588\ub2e4. \"\uc544\ubb34\ub798\ub3c4 \uc62c\ub9bc\ud53d\uc5d0\uc11c \uc774\uc2b9\ud6c8 \uc120\uc218 \uac00 \ud65c\uc57d\ud588\ub358 \uac8c \uc601\ud5a5\uc774 \uac00\uc7a5 \ucef8\uc8e0. \ud558\uc9c0\ub9cc 2010\ub144\ub3c4 \uc804\ubd80\ud130 \uc2a4\ud53c\ub4dc\ub97c \ud0c0\uace0 \uc2f6\ub2e4\ub294 \uc0dd\uac01\uc740 \ud588\uc5c8\uc5b4\uc694.\" \"\uc2a4\ud53c\ub4dc \uc2a4\ucf00\uc774\ud305\uc758 \ud63c\uc790\ub9cc\uc758 \uc2f8\uc6c0\uc774 \ub9e4\ub825\uc801\uc774\ub77c \uc804\ud5a5 \uacb0\uc2ec\ud588\uc2b5\ub2c8\ub2e4. \" \"\ub531\ud788 \uacc4\uae30\ub77c\uace0 \ud560 \uac74 \uc5c6\uace0 \uc6b4\ub3d9\uc744 \uadf8\ub9cc\ub450\ub824\uace0 \ud588\uc5b4\uc694. \uadf8\ub7ec\ub2e4\uac00 \ub9c8\uc9c0\ub9c9\uc73c\ub85c '\uc2a4\ud53c\ub4dc \uc2a4\ucf00\uc774\ud305 \ud55c\ubc88 \ud574\ubcf4\uace0 \uadf8\ub9cc\ub450\uc790'\ud574\uc11c \uc804\ud5a5\uc744 \ud588\ub294\ub370 \uc9c0\uae08 \uc5ec\uae30\uae4c\uc9c0 \uc628 \uac70\uc8e0. \uc1fc\ud2b8\ud2b8\ub799 \ud560 \ub54c \uadf8\ub807\uac8c \uc798 \ud0d4\ub358 \uc120\uc218\ub3c4 \uc544\ub2c8\uc5c8\uace0 \uc131\uc801\ub3c4 \uacc4\uc18d \uc548\ub098\uc654\uace0 \ub300\ud559\uc5d0 \uac00\uc11c \uc9c4\ub85c \uacb0\uc815\uc744 \ud655\uc2e4\ud788 \ud574\uc57c \ud558\ub358 \uc2dc\uae30\uc5ec\uc11c \uc804\ud5a5\uc744 \ud558\uac8c \ub410\uc5b4\uc694. \"\uc544\ubb34\ub798\ub3c4 \uc2dc\uc791\uc774 \ub2a6\ub2e4 \ubcf4\ub2c8 \uc758\uc695\uacfc\ub294 \ub2ec\ub9ac \ub610\ub798 \uce5c\uad6c\ub4e4\uacfc\uc758 \uaca9\ucc28\ub97c \uc904\uc774\uae30\uac00 \ubc85\ucc28\ub354\ub77c\uace0\uc694. \uacb0\uad6d\uc5d4 \uc1fc\ud2b8\ud2b8\ub799\uc5d0 \ud765\ubbf8\ub97c \uc783\uc5c8\uace0, \uc911\ud559\uad50 \uc785\ud559 \ud6c4\uc5d0\ub294 \uc6b4\ub3d9\uc744 \uadf8\ub9cc \ub450\uac8c \ub40f\uc8e0. \ub2e4\uc2dc \uc1fc\ud2b8\ud2b8\ub799\uc744 \uc2dc\uc791\ud558\uba74\uc11c \uc774\uc804\ubcf4\ub2e4\ub294 \ud655\uc2e4\ud788 \uad1c\ucc2e\uc558\uc5b4\uc694. 2009\ub144 \uc804\uad6d\ub3d9\uacc4\uccb4\uc721\ub300\ud68c \uc1fc\ud2b8\ud2b8\ub799 1500m \ubd80\ubb38\uc5d0\uc11c \ub3d9\uba54\ub2ec\uc744 \ub534 \uc801\ub3c4 \uc788\uc5c8\uac70\ub4e0\uc694. \uadf8\ub807\uc9c0\ub9cc \uc800\ub294 \uc1fc\ud2b8\ud2b8\ub799 \uc120\uc218\ub85c\uc11c\uc758 \uac00\ub2a5\uc131\uc774 \uadf8\ub807\uac8c \ud070 \uc120\uc218\ub294 \uc544\ub2c8\uc5c8\uc8e0. \uc1fc\ud2b8\ud2b8\ub799\uc73c\ub85c\ub294 \ud55c\uacc4\uac00 \uc788\ub2e4\uace0 \uc0dd\uac01\uc744 \ud588\uc5b4\uc694\" \"\uc804\ud5a5\uc5d0 \ub300\ud574 \uc8fc\uc704\uc5d0\uc11c \ubc18\ub300\ub97c \ub9ce\uc774 \ud588\uc5b4\uc694. \uc5f4 \uba85\uc774\uba74 \uc5f4 \uba85 \ub2e4 \uc88b\uc9c0 \uc54a\uac8c \ubcf4\uc2dc\ub354\ub77c\uace0\uc694. \ub204\uad70\uac00 '\uadf8\ub0e5 \uc1fc\ud2b8\ud2b8\ub799 \ud0c0'\ub77c\uace0 \ub9d0\ud588\ub2e4\uba74 \uc624\ud788\ub824 \uad1c\ucc2e\uc558\uc744 \uac70\uc608\uc694. \uadf8\ub7f0\ub370 \ub2e4\ub4e4 '\ub124\uac00 \ud560 \uc218 \uc788\uc744 \uac83 \uac19\ub0d0'\uace0 \ud558\uc2dc\ub354\ub77c\uace0\uc694. \uc2a4\ud53c\ub4dc\uc2a4\ucf00\uc774\ud305\uc5d0\uc11c\ub3c4 \uadf8\uc800\uadf8\ub7f0 \uc120\uc218\ub85c \ub0a8\uc73c\uba74 \uc800\ub97c \ube44\ub09c\ud588\ub358 \uc0ac\ub78c\ub4e4\uc758 \uc0dd\uac01\ucc98\ub7fc \ub418\ub294 \uac70\uc796\uc544\uc694. \ubb54\uac00 \uc624\uae30, \ubaa9\ud45c\uac00 \uc0dd\uacbc\uc5b4\uc694. \" \"\uc1fc\ud2b8\ud2b8\ub799\uc5d0 \ub2a6\uac8c \uc785\ubb38\ud588\uace0, \uc2a4\ud53c\ub4dc\uc2a4\ucf00\uc774\ud305\ub3c4 \uc5c4\uccad\ub098\uac8c \ub2a6\uac8c \uc2dc\uc791\ud588\uc9c0\ub9cc, \uc660\uc9c0 \ubaa8\ub974\uac8c \ub9c8\uc74c\uc774 \ud3b8\ud588\ub2e4. \ub9c8\uc9c0\ub9c9 \uad81\uc9c0\uc5d0 \ubab0\ub9b0 \ud130\ub77c \ub354 \uadf8\ub7ac\ub358 \uac83 \uac19\ub2e4. \" \"\uc774\uc2b9\ud6c8\uc740 \uc1fc\ud2b8\ud2b8\ub799\uc5d0\uc11c\ub3c4 \uad6d\uac00\ub300\ud45c\ub97c \uc9c0\ub0c8\uc9c0\ub9cc \uae40\ubcf4\ub984\uc740 \ub2e4\ub974\ub2e4. \uae40\ubcf4\ub984\uc740 \uc1fc\ud2b8\ud2b8\ub799\uc5d0\uc11c \ud06c\uac8c \ub450\uac01\uc744 \ubcf4\uc774\uc9c0 \ubabb\ud558\uace0 \uc885\ubaa9\uc744 \uc804\ud658\ud574 \uc131\uacf5\ud588\ub2e4. \ub354 \ub9ce\uc740 \uc120\uc218\ub4e4\uc5d0\uac8c \ud76c\ub9dd\uc744 \uc8fc\uae30\ub3c4 \ud55c\ub2e4.\" \uc2e4\ud328\ud588\ub358 \uae30\uac04\uc73c\ub85c \ubcf4\uc600\ub358 \uc1fc\ud2b8\ud2b8\ub799 \uacbd\ud5d8\uc774 \ud6d7\ub0a0 \uc8fc\uc885\ubaa9\uc774 \ub41c \ub9e4\uc2a4\uc2a4\ud0c0\ud2b8\ub97c \uc704\ud55c \uae40\ubcf4\ub984\uc758 \uac00\uc7a5 \uadc0\ud55c \uc790\uc0b0\uc774 \ub41c\ub2e4. \"\uc1fc\ud2b8\ud2b8\ub799 \uacbd\ud5d8\uc774 \ub9e4\uc2a4\uc2a4\ud0c0\ud2b8\uc5d0 \ub9ce\uc740 \ub3c4\uc6c0\uc744 \uc900\ub2e4. \"", "paragraph_answer": "\ucd08\ub4f1\ud559\uad50 5\ud559\ub144 \ub54c \uc1fc\ud2b8\ud2b8\ub799 \uc120\uc218\ub85c \uc785\ubb38\ud55c \uae40\ubcf4\ub984\uc740 \ub450\uac01\uc744 \ub098\ud0c0\ub0b4\uc9c0 \ubabb\ud588\uace0, \uacb0\uad6d \uace0\ub4f1\ud559\uad50 2\ud559\ub144 \ub54c \uc2a4\ud53c\ub4dc \uc2a4\ucf00\uc774\ud305\uc73c\ub85c \uc804\ud5a5\ud588\ub2e4. \"\uc544\ubb34\ub798\ub3c4 \uc62c\ub9bc\ud53d\uc5d0\uc11c \uc774\uc2b9\ud6c8 \uc120\uc218 \uac00 \ud65c\uc57d\ud588\ub358 \uac8c \uc601\ud5a5\uc774 \uac00\uc7a5 \ucef8\uc8e0. \ud558\uc9c0\ub9cc 2010\ub144\ub3c4 \uc804\ubd80\ud130 \uc2a4\ud53c\ub4dc\ub97c \ud0c0\uace0 \uc2f6\ub2e4\ub294 \uc0dd\uac01\uc740 \ud588\uc5c8\uc5b4\uc694.\" \"\uc2a4\ud53c\ub4dc \uc2a4\ucf00\uc774\ud305\uc758 \ud63c\uc790\ub9cc\uc758 \uc2f8\uc6c0\uc774 \ub9e4\ub825\uc801\uc774\ub77c \uc804\ud5a5 \uacb0\uc2ec\ud588\uc2b5\ub2c8\ub2e4.\" \"\ub531\ud788 \uacc4\uae30\ub77c\uace0 \ud560 \uac74 \uc5c6\uace0 \uc6b4\ub3d9\uc744 \uadf8\ub9cc\ub450\ub824\uace0 \ud588\uc5b4\uc694. \uadf8\ub7ec\ub2e4\uac00 \ub9c8\uc9c0\ub9c9\uc73c\ub85c '\uc2a4\ud53c\ub4dc \uc2a4\ucf00\uc774\ud305 \ud55c\ubc88 \ud574\ubcf4\uace0 \uadf8\ub9cc\ub450\uc790'\ud574\uc11c \uc804\ud5a5\uc744 \ud588\ub294\ub370 \uc9c0\uae08 \uc5ec\uae30\uae4c\uc9c0 \uc628 \uac70\uc8e0. \uc1fc\ud2b8\ud2b8\ub799 \ud560 \ub54c \uadf8\ub807\uac8c \uc798 \ud0d4\ub358 \uc120\uc218\ub3c4 \uc544\ub2c8\uc5c8\uace0 \uc131\uc801\ub3c4 \uacc4\uc18d \uc548\ub098\uc654\uace0 \ub300\ud559\uc5d0 \uac00\uc11c \uc9c4\ub85c \uacb0\uc815\uc744 \ud655\uc2e4\ud788 \ud574\uc57c \ud558\ub358 \uc2dc\uae30\uc5ec\uc11c \uc804\ud5a5\uc744 \ud558\uac8c \ub410\uc5b4\uc694. \"\uc544\ubb34\ub798\ub3c4 \uc2dc\uc791\uc774 \ub2a6\ub2e4 \ubcf4\ub2c8 \uc758\uc695\uacfc\ub294 \ub2ec\ub9ac \ub610\ub798 \uce5c\uad6c\ub4e4\uacfc\uc758 \uaca9\ucc28\ub97c \uc904\uc774\uae30\uac00 \ubc85\ucc28\ub354\ub77c\uace0\uc694. \uacb0\uad6d\uc5d4 \uc1fc\ud2b8\ud2b8\ub799\uc5d0 \ud765\ubbf8\ub97c \uc783\uc5c8\uace0, \uc911\ud559\uad50 \uc785\ud559 \ud6c4\uc5d0\ub294 \uc6b4\ub3d9\uc744 \uadf8\ub9cc \ub450\uac8c \ub40f\uc8e0. \ub2e4\uc2dc \uc1fc\ud2b8\ud2b8\ub799\uc744 \uc2dc\uc791\ud558\uba74\uc11c \uc774\uc804\ubcf4\ub2e4\ub294 \ud655\uc2e4\ud788 \uad1c\ucc2e\uc558\uc5b4\uc694. 2009\ub144 \uc804\uad6d\ub3d9\uacc4\uccb4\uc721\ub300\ud68c \uc1fc\ud2b8\ud2b8\ub799 1500m \ubd80\ubb38\uc5d0\uc11c \ub3d9\uba54\ub2ec\uc744 \ub534 \uc801\ub3c4 \uc788\uc5c8\uac70\ub4e0\uc694. \uadf8\ub807\uc9c0\ub9cc \uc800\ub294 \uc1fc\ud2b8\ud2b8\ub799 \uc120\uc218\ub85c\uc11c\uc758 \uac00\ub2a5\uc131\uc774 \uadf8\ub807\uac8c \ud070 \uc120\uc218\ub294 \uc544\ub2c8\uc5c8\uc8e0. \uc1fc\ud2b8\ud2b8\ub799\uc73c\ub85c\ub294 \ud55c\uacc4\uac00 \uc788\ub2e4\uace0 \uc0dd\uac01\uc744 \ud588\uc5b4\uc694\" \"\uc804\ud5a5\uc5d0 \ub300\ud574 \uc8fc\uc704\uc5d0\uc11c \ubc18\ub300\ub97c \ub9ce\uc774 \ud588\uc5b4\uc694. \uc5f4 \uba85\uc774\uba74 \uc5f4 \uba85 \ub2e4 \uc88b\uc9c0 \uc54a\uac8c \ubcf4\uc2dc\ub354\ub77c\uace0\uc694. \ub204\uad70\uac00 '\uadf8\ub0e5 \uc1fc\ud2b8\ud2b8\ub799 \ud0c0'\ub77c\uace0 \ub9d0\ud588\ub2e4\uba74 \uc624\ud788\ub824 \uad1c\ucc2e\uc558\uc744 \uac70\uc608\uc694. \uadf8\ub7f0\ub370 \ub2e4\ub4e4 '\ub124\uac00 \ud560 \uc218 \uc788\uc744 \uac83 \uac19\ub0d0'\uace0 \ud558\uc2dc\ub354\ub77c\uace0\uc694. \uc2a4\ud53c\ub4dc\uc2a4\ucf00\uc774\ud305\uc5d0\uc11c\ub3c4 \uadf8\uc800\uadf8\ub7f0 \uc120\uc218\ub85c \ub0a8\uc73c\uba74 \uc800\ub97c \ube44\ub09c\ud588\ub358 \uc0ac\ub78c\ub4e4\uc758 \uc0dd\uac01\ucc98\ub7fc \ub418\ub294 \uac70\uc796\uc544\uc694. \ubb54\uac00 \uc624\uae30, \ubaa9\ud45c\uac00 \uc0dd\uacbc\uc5b4\uc694.\" \"\uc1fc\ud2b8\ud2b8\ub799\uc5d0 \ub2a6\uac8c \uc785\ubb38\ud588\uace0, \uc2a4\ud53c\ub4dc\uc2a4\ucf00\uc774\ud305\ub3c4 \uc5c4\uccad\ub098\uac8c \ub2a6\uac8c \uc2dc\uc791\ud588\uc9c0\ub9cc, \uc660\uc9c0 \ubaa8\ub974\uac8c \ub9c8\uc74c\uc774 \ud3b8\ud588\ub2e4. \ub9c8\uc9c0\ub9c9 \uad81\uc9c0\uc5d0 \ubab0\ub9b0 \ud130\ub77c \ub354 \uadf8\ub7ac\ub358 \uac83 \uac19\ub2e4.\" \"\uc774\uc2b9\ud6c8\uc740 \uc1fc\ud2b8\ud2b8\ub799\uc5d0\uc11c\ub3c4 \uad6d\uac00\ub300\ud45c\ub97c \uc9c0\ub0c8\uc9c0\ub9cc \uae40\ubcf4\ub984\uc740 \ub2e4\ub974\ub2e4. \uae40\ubcf4\ub984\uc740 \uc1fc\ud2b8\ud2b8\ub799\uc5d0\uc11c \ud06c\uac8c \ub450\uac01\uc744 \ubcf4\uc774\uc9c0 \ubabb\ud558\uace0 \uc885\ubaa9\uc744 \uc804\ud658\ud574 \uc131\uacf5\ud588\ub2e4. \ub354 \ub9ce\uc740 \uc120\uc218\ub4e4\uc5d0\uac8c \ud76c\ub9dd\uc744 \uc8fc\uae30\ub3c4 \ud55c\ub2e4.\" \uc2e4\ud328\ud588\ub358 \uae30\uac04\uc73c\ub85c \ubcf4\uc600\ub358 \uc1fc\ud2b8\ud2b8\ub799 \uacbd\ud5d8\uc774 \ud6d7\ub0a0 \uc8fc\uc885\ubaa9\uc774 \ub41c \ub9e4\uc2a4\uc2a4\ud0c0\ud2b8\ub97c \uc704\ud55c \uae40\ubcf4\ub984\uc758 \uac00\uc7a5 \uadc0\ud55c \uc790\uc0b0\uc774 \ub41c\ub2e4. \"\uc1fc\ud2b8\ud2b8\ub799 \uacbd\ud5d8\uc774 \ub9e4\uc2a4\uc2a4\ud0c0\ud2b8\uc5d0 \ub9ce\uc740 \ub3c4\uc6c0\uc744 \uc900\ub2e4.\"", "sentence_answer": "\"\uc544\ubb34\ub798\ub3c4 \uc62c\ub9bc\ud53d\uc5d0\uc11c \uc774\uc2b9\ud6c8 \uc120\uc218 \uac00 \ud65c\uc57d\ud588\ub358 \uac8c \uc601\ud5a5\uc774 \uac00\uc7a5 \ucef8\uc8e0.", "paragraph_id": "6488273-0-1", "paragraph_question": "question: \uae40\ubcf4\ub984 \uc120\uc218\uc758 \uc804\ud5a5\uc5d0 \uc601\ud5a5\uc744 \ubbf8\uce5c \uc120\uc218\ub294?, context: \ucd08\ub4f1\ud559\uad50 5\ud559\ub144 \ub54c \uc1fc\ud2b8\ud2b8\ub799 \uc120\uc218\ub85c \uc785\ubb38\ud55c \uae40\ubcf4\ub984\uc740 \ub450\uac01\uc744 \ub098\ud0c0\ub0b4\uc9c0 \ubabb\ud588\uace0, \uacb0\uad6d \uace0\ub4f1\ud559\uad50 2\ud559\ub144 \ub54c \uc2a4\ud53c\ub4dc \uc2a4\ucf00\uc774\ud305\uc73c\ub85c \uc804\ud5a5\ud588\ub2e4. \"\uc544\ubb34\ub798\ub3c4 \uc62c\ub9bc\ud53d\uc5d0\uc11c \uc774\uc2b9\ud6c8 \uc120\uc218\uac00 \ud65c\uc57d\ud588\ub358 \uac8c \uc601\ud5a5\uc774 \uac00\uc7a5 \ucef8\uc8e0. \ud558\uc9c0\ub9cc 2010\ub144\ub3c4 \uc804\ubd80\ud130 \uc2a4\ud53c\ub4dc\ub97c \ud0c0\uace0 \uc2f6\ub2e4\ub294 \uc0dd\uac01\uc740 \ud588\uc5c8\uc5b4\uc694.\" \"\uc2a4\ud53c\ub4dc \uc2a4\ucf00\uc774\ud305\uc758 \ud63c\uc790\ub9cc\uc758 \uc2f8\uc6c0\uc774 \ub9e4\ub825\uc801\uc774\ub77c \uc804\ud5a5 \uacb0\uc2ec\ud588\uc2b5\ub2c8\ub2e4.\" \"\ub531\ud788 \uacc4\uae30\ub77c\uace0 \ud560 \uac74 \uc5c6\uace0 \uc6b4\ub3d9\uc744 \uadf8\ub9cc\ub450\ub824\uace0 \ud588\uc5b4\uc694. \uadf8\ub7ec\ub2e4\uac00 \ub9c8\uc9c0\ub9c9\uc73c\ub85c '\uc2a4\ud53c\ub4dc \uc2a4\ucf00\uc774\ud305 \ud55c\ubc88 \ud574\ubcf4\uace0 \uadf8\ub9cc\ub450\uc790'\ud574\uc11c \uc804\ud5a5\uc744 \ud588\ub294\ub370 \uc9c0\uae08 \uc5ec\uae30\uae4c\uc9c0 \uc628 \uac70\uc8e0. \uc1fc\ud2b8\ud2b8\ub799 \ud560 \ub54c \uadf8\ub807\uac8c \uc798 \ud0d4\ub358 \uc120\uc218\ub3c4 \uc544\ub2c8\uc5c8\uace0 \uc131\uc801\ub3c4 \uacc4\uc18d \uc548\ub098\uc654\uace0 \ub300\ud559\uc5d0 \uac00\uc11c \uc9c4\ub85c \uacb0\uc815\uc744 \ud655\uc2e4\ud788 \ud574\uc57c \ud558\ub358 \uc2dc\uae30\uc5ec\uc11c \uc804\ud5a5\uc744 \ud558\uac8c \ub410\uc5b4\uc694. \"\uc544\ubb34\ub798\ub3c4 \uc2dc\uc791\uc774 \ub2a6\ub2e4 \ubcf4\ub2c8 \uc758\uc695\uacfc\ub294 \ub2ec\ub9ac \ub610\ub798 \uce5c\uad6c\ub4e4\uacfc\uc758 \uaca9\ucc28\ub97c \uc904\uc774\uae30\uac00 \ubc85\ucc28\ub354\ub77c\uace0\uc694. \uacb0\uad6d\uc5d4 \uc1fc\ud2b8\ud2b8\ub799\uc5d0 \ud765\ubbf8\ub97c \uc783\uc5c8\uace0, \uc911\ud559\uad50 \uc785\ud559 \ud6c4\uc5d0\ub294 \uc6b4\ub3d9\uc744 \uadf8\ub9cc \ub450\uac8c \ub40f\uc8e0. \ub2e4\uc2dc \uc1fc\ud2b8\ud2b8\ub799\uc744 \uc2dc\uc791\ud558\uba74\uc11c \uc774\uc804\ubcf4\ub2e4\ub294 \ud655\uc2e4\ud788 \uad1c\ucc2e\uc558\uc5b4\uc694. 2009\ub144 \uc804\uad6d\ub3d9\uacc4\uccb4\uc721\ub300\ud68c \uc1fc\ud2b8\ud2b8\ub799 1500m \ubd80\ubb38\uc5d0\uc11c \ub3d9\uba54\ub2ec\uc744 \ub534 \uc801\ub3c4 \uc788\uc5c8\uac70\ub4e0\uc694. \uadf8\ub807\uc9c0\ub9cc \uc800\ub294 \uc1fc\ud2b8\ud2b8\ub799 \uc120\uc218\ub85c\uc11c\uc758 \uac00\ub2a5\uc131\uc774 \uadf8\ub807\uac8c \ud070 \uc120\uc218\ub294 \uc544\ub2c8\uc5c8\uc8e0. \uc1fc\ud2b8\ud2b8\ub799\uc73c\ub85c\ub294 \ud55c\uacc4\uac00 \uc788\ub2e4\uace0 \uc0dd\uac01\uc744 \ud588\uc5b4\uc694\" \"\uc804\ud5a5\uc5d0 \ub300\ud574 \uc8fc\uc704\uc5d0\uc11c \ubc18\ub300\ub97c \ub9ce\uc774 \ud588\uc5b4\uc694. \uc5f4 \uba85\uc774\uba74 \uc5f4 \uba85 \ub2e4 \uc88b\uc9c0 \uc54a\uac8c \ubcf4\uc2dc\ub354\ub77c\uace0\uc694. \ub204\uad70\uac00 '\uadf8\ub0e5 \uc1fc\ud2b8\ud2b8\ub799 \ud0c0'\ub77c\uace0 \ub9d0\ud588\ub2e4\uba74 \uc624\ud788\ub824 \uad1c\ucc2e\uc558\uc744 \uac70\uc608\uc694. \uadf8\ub7f0\ub370 \ub2e4\ub4e4 '\ub124\uac00 \ud560 \uc218 \uc788\uc744 \uac83 \uac19\ub0d0'\uace0 \ud558\uc2dc\ub354\ub77c\uace0\uc694. \uc2a4\ud53c\ub4dc\uc2a4\ucf00\uc774\ud305\uc5d0\uc11c\ub3c4 \uadf8\uc800\uadf8\ub7f0 \uc120\uc218\ub85c \ub0a8\uc73c\uba74 \uc800\ub97c \ube44\ub09c\ud588\ub358 \uc0ac\ub78c\ub4e4\uc758 \uc0dd\uac01\ucc98\ub7fc \ub418\ub294 \uac70\uc796\uc544\uc694. \ubb54\uac00 \uc624\uae30, \ubaa9\ud45c\uac00 \uc0dd\uacbc\uc5b4\uc694.\" \"\uc1fc\ud2b8\ud2b8\ub799\uc5d0 \ub2a6\uac8c \uc785\ubb38\ud588\uace0, \uc2a4\ud53c\ub4dc\uc2a4\ucf00\uc774\ud305\ub3c4 \uc5c4\uccad\ub098\uac8c \ub2a6\uac8c \uc2dc\uc791\ud588\uc9c0\ub9cc, \uc660\uc9c0 \ubaa8\ub974\uac8c \ub9c8\uc74c\uc774 \ud3b8\ud588\ub2e4. \ub9c8\uc9c0\ub9c9 \uad81\uc9c0\uc5d0 \ubab0\ub9b0 \ud130\ub77c \ub354 \uadf8\ub7ac\ub358 \uac83 \uac19\ub2e4.\" \"\uc774\uc2b9\ud6c8\uc740 \uc1fc\ud2b8\ud2b8\ub799\uc5d0\uc11c\ub3c4 \uad6d\uac00\ub300\ud45c\ub97c \uc9c0\ub0c8\uc9c0\ub9cc \uae40\ubcf4\ub984\uc740 \ub2e4\ub974\ub2e4. \uae40\ubcf4\ub984\uc740 \uc1fc\ud2b8\ud2b8\ub799\uc5d0\uc11c \ud06c\uac8c \ub450\uac01\uc744 \ubcf4\uc774\uc9c0 \ubabb\ud558\uace0 \uc885\ubaa9\uc744 \uc804\ud658\ud574 \uc131\uacf5\ud588\ub2e4. \ub354 \ub9ce\uc740 \uc120\uc218\ub4e4\uc5d0\uac8c \ud76c\ub9dd\uc744 \uc8fc\uae30\ub3c4 \ud55c\ub2e4.\" \uc2e4\ud328\ud588\ub358 \uae30\uac04\uc73c\ub85c \ubcf4\uc600\ub358 \uc1fc\ud2b8\ud2b8\ub799 \uacbd\ud5d8\uc774 \ud6d7\ub0a0 \uc8fc\uc885\ubaa9\uc774 \ub41c \ub9e4\uc2a4\uc2a4\ud0c0\ud2b8\ub97c \uc704\ud55c \uae40\ubcf4\ub984\uc758 \uac00\uc7a5 \uadc0\ud55c \uc790\uc0b0\uc774 \ub41c\ub2e4. \"\uc1fc\ud2b8\ud2b8\ub799 \uacbd\ud5d8\uc774 \ub9e4\uc2a4\uc2a4\ud0c0\ud2b8\uc5d0 \ub9ce\uc740 \ub3c4\uc6c0\uc744 \uc900\ub2e4.\""} {"question": "\uc288\ub8b0\ub354\uac00 \ub0b8 \ud68c\uace0\ub85d\uc740?", "paragraph": "\uc790\ud06c \uc2dc\ub77c\ud06c \ub300\ud1b5\ub839\uacfc \uc6b0\ud638\uc801\uc778 \uad00\uacc4\uc5d0 \ucd94\uac00\ub85c \uc288\ub8b0\ub354\ub294 \ub7ec\uc2dc\uc544\uc640 \ub3c5\uc77c \uc0ac\uc774\uc5d0 \ubc1c\ud2b8\ud574\uc5d0 \uac00\uc2a4 \ud30c\uc774\ud504\ub77c\uc778\uc744 \uac1c\uc7a5\ud55c \uac83\uc744 \ud3ec\ud568\ud55c \ubca0\ub97c\ub9b0\uacfc \ubaa8\uc2a4\ud06c\ubc14 \uc0ac\uc774\uc758 \uc804\ub7b5 \ud611\ub825\uc744 \uac15\ud654\ud558\ub294 \uc2dc\ub3c4\uc5d0\uc11c \ub7ec\uc2dc\uc544\uc758 \ube14\ub77c\ub514\ubbf8\ub974 \ud478\ud2f4 \ub300\ud1b5\ub839\uacfc \uac00\uae4c\uc6b4 \uad00\uacc4\ub97c \uae4a\uac8c \ud558\uc600\ub2e4. \uc288\ub8b0\ub354\ub294 \ud478\ud2f4\uc774 \uc6b0\ud06c\ub77c\uc774\ub098\uc5d0\uc11c \uc624\ub80c\uc9c0 \ud601\uba85\uc774 \uc77c\uc5b4\ub098\ub294 \ub3d9\uc548\uc5d0 \ube45\ud1a0\ub974 \uc57c\ub204\ucf54\ube44\uce58\ub97c \ucd95\ud558\ud558\uae30 \uba70\uce60 \uc804 11\uc6d4 22\uc77c \ud478\ud2f4\uc744 \"\uc644\ubcbd\ud55c \ubbfc\uc8fc\uc8fc\uc758\uc790\"\ub85c \ubd80\ub978 \uc774\uc720\ub85c \ub300\uc911 \ub9e4\uccb4\uc5d0\uc11c, \uadf8\ub9ac\uace0 \uc774\uc5b4\uc11c \uc559\uac94\ub77c \uba54\ub974\ucf08\uc5d0 \uc758\ud558\uc5ec \ube44\ub09c\uc744 \ubc1b\uc558\ub2e4. \uc790\uc2e0\uc758 \uc218\uc0c1 \uc784\uae30\uac00 \uc2dc\uc791\ub41c \uc9c0 \uba70\uce60 \ub9cc\uc5d0 \uc288\ub8b0\ub354\ub294 \ud569\ub3d9 \ud22c\uae30\uc758 \uc774\uc0ac\uad6d\uc5d0 \uac00\uc785\ud558\uc5ec \uc790\uc2e0\uc758 \uc774\uc804\uc758 \uac1d\uad00\uc131\uc5d0 \uad00\ud55c \uc0c8\ub85c\uc6b4 \ud22c\uae30\ub4e4\uc744 \uac00\uc838\uc654\ub2e4. \uc790\uc2e0\uc758 \ud68c\uace0\ub85d \u300a\uacb0\uc815\ub4e4:\uc815\uce58\uc5d0\uc11c \ub098\uc758 \uc778\uc0dd\u300b\uc5d0\uc11c \uc288\ub8b0\ub354\ub294 \uc544\uc9c1\ub3c4 \uc790\uc2e0\uc758 \uce5c\uad6c\uc774\uc790 \uc815\uce58\uc801 \uc81c\ud734 \ud478\ud2f4\uc5d0 \uc758\uc9c0\ud558\uace0 \"\uad6d\ub0b4\uc801 \uc815\uce58 \uac1c\ud601\uacfc \ubbfc\uc8fc\uc801 \uac1c\ubc1c\uc758 \ube44\uc728\uc5d0 \uc62c \ub54c \ub7ec\uc2dc\uc544\uc5d0 \uacfc\ub3c4\ud55c \uc694\uad6c\ub4e4\uc744 \ub193\uac70\ub098 \uccb4\uccb8 \uc804\uc7c1\uc758 \uae30\ucd08\uc5d0 \uc644\uace0\ud558\uac8c \uc7ac\ud310\ud558\ub294 \ub370 \uc798\ubabb\ub418\uc5c8\uc744 \uac83\uc774\ub2e4\"\ub77c\uace0 \uc9c4\uc220\ud558\uc600\ub2e4.", "answer": "\uacb0\uc815\ub4e4:\uc815\uce58\uc5d0\uc11c \ub098\uc758 \uc778\uc0dd", "sentence": "\uc790\uc2e0\uc758 \ud68c\uace0\ub85d \u300a \uacb0\uc815\ub4e4:\uc815\uce58\uc5d0\uc11c \ub098\uc758 \uc778\uc0dd \u300b\uc5d0\uc11c \uc288\ub8b0\ub354\ub294 \uc544\uc9c1\ub3c4 \uc790\uc2e0\uc758 \uce5c\uad6c\uc774\uc790 \uc815\uce58\uc801 \uc81c\ud734 \ud478\ud2f4\uc5d0 \uc758\uc9c0\ud558\uace0 \"\uad6d\ub0b4\uc801 \uc815\uce58 \uac1c\ud601\uacfc \ubbfc\uc8fc\uc801 \uac1c\ubc1c\uc758 \ube44\uc728\uc5d0 \uc62c \ub54c \ub7ec\uc2dc\uc544\uc5d0 \uacfc\ub3c4\ud55c \uc694\uad6c\ub4e4\uc744 \ub193\uac70\ub098 \uccb4\uccb8 \uc804\uc7c1\uc758 \uae30\ucd08\uc5d0 \uc644\uace0\ud558\uac8c \uc7ac\ud310\ud558\ub294 \ub370 \uc798\ubabb\ub418\uc5c8\uc744 \uac83\uc774\ub2e4\"\ub77c\uace0 \uc9c4\uc220\ud558\uc600\ub2e4.", "paragraph_sentence": "\uc790\ud06c \uc2dc\ub77c\ud06c \ub300\ud1b5\ub839\uacfc \uc6b0\ud638\uc801\uc778 \uad00\uacc4\uc5d0 \ucd94\uac00\ub85c \uc288\ub8b0\ub354\ub294 \ub7ec\uc2dc\uc544\uc640 \ub3c5\uc77c \uc0ac\uc774\uc5d0 \ubc1c\ud2b8\ud574\uc5d0 \uac00\uc2a4 \ud30c\uc774\ud504\ub77c\uc778\uc744 \uac1c\uc7a5\ud55c \uac83\uc744 \ud3ec\ud568\ud55c \ubca0\ub97c\ub9b0\uacfc \ubaa8\uc2a4\ud06c\ubc14 \uc0ac\uc774\uc758 \uc804\ub7b5 \ud611\ub825\uc744 \uac15\ud654\ud558\ub294 \uc2dc\ub3c4\uc5d0\uc11c \ub7ec\uc2dc\uc544\uc758 \ube14\ub77c\ub514\ubbf8\ub974 \ud478\ud2f4 \ub300\ud1b5\ub839\uacfc \uac00\uae4c\uc6b4 \uad00\uacc4\ub97c \uae4a\uac8c \ud558\uc600\ub2e4. \uc288\ub8b0\ub354\ub294 \ud478\ud2f4\uc774 \uc6b0\ud06c\ub77c\uc774\ub098\uc5d0\uc11c \uc624\ub80c\uc9c0 \ud601\uba85\uc774 \uc77c\uc5b4\ub098\ub294 \ub3d9\uc548\uc5d0 \ube45\ud1a0\ub974 \uc57c\ub204\ucf54\ube44\uce58\ub97c \ucd95\ud558\ud558\uae30 \uba70\uce60 \uc804 11\uc6d4 22\uc77c \ud478\ud2f4\uc744 \"\uc644\ubcbd\ud55c \ubbfc\uc8fc\uc8fc\uc758\uc790\"\ub85c \ubd80\ub978 \uc774\uc720\ub85c \ub300\uc911 \ub9e4\uccb4\uc5d0\uc11c, \uadf8\ub9ac\uace0 \uc774\uc5b4\uc11c \uc559\uac94\ub77c \uba54\ub974\ucf08\uc5d0 \uc758\ud558\uc5ec \ube44\ub09c\uc744 \ubc1b\uc558\ub2e4. \uc790\uc2e0\uc758 \uc218\uc0c1 \uc784\uae30\uac00 \uc2dc\uc791\ub41c \uc9c0 \uba70\uce60 \ub9cc\uc5d0 \uc288\ub8b0\ub354\ub294 \ud569\ub3d9 \ud22c\uae30\uc758 \uc774\uc0ac\uad6d\uc5d0 \uac00\uc785\ud558\uc5ec \uc790\uc2e0\uc758 \uc774\uc804\uc758 \uac1d\uad00\uc131\uc5d0 \uad00\ud55c \uc0c8\ub85c\uc6b4 \ud22c\uae30\ub4e4\uc744 \uac00\uc838\uc654\ub2e4. \uc790\uc2e0\uc758 \ud68c\uace0\ub85d \u300a \uacb0\uc815\ub4e4:\uc815\uce58\uc5d0\uc11c \ub098\uc758 \uc778\uc0dd \u300b\uc5d0\uc11c \uc288\ub8b0\ub354\ub294 \uc544\uc9c1\ub3c4 \uc790\uc2e0\uc758 \uce5c\uad6c\uc774\uc790 \uc815\uce58\uc801 \uc81c\ud734 \ud478\ud2f4\uc5d0 \uc758\uc9c0\ud558\uace0 \"\uad6d\ub0b4\uc801 \uc815\uce58 \uac1c\ud601\uacfc \ubbfc\uc8fc\uc801 \uac1c\ubc1c\uc758 \ube44\uc728\uc5d0 \uc62c \ub54c \ub7ec\uc2dc\uc544\uc5d0 \uacfc\ub3c4\ud55c \uc694\uad6c\ub4e4\uc744 \ub193\uac70\ub098 \uccb4\uccb8 \uc804\uc7c1\uc758 \uae30\ucd08\uc5d0 \uc644\uace0\ud558\uac8c \uc7ac\ud310\ud558\ub294 \ub370 \uc798\ubabb\ub418\uc5c8\uc744 \uac83\uc774\ub2e4\"\ub77c\uace0 \uc9c4\uc220\ud558\uc600\ub2e4. ", "paragraph_answer": "\uc790\ud06c \uc2dc\ub77c\ud06c \ub300\ud1b5\ub839\uacfc \uc6b0\ud638\uc801\uc778 \uad00\uacc4\uc5d0 \ucd94\uac00\ub85c \uc288\ub8b0\ub354\ub294 \ub7ec\uc2dc\uc544\uc640 \ub3c5\uc77c \uc0ac\uc774\uc5d0 \ubc1c\ud2b8\ud574\uc5d0 \uac00\uc2a4 \ud30c\uc774\ud504\ub77c\uc778\uc744 \uac1c\uc7a5\ud55c \uac83\uc744 \ud3ec\ud568\ud55c \ubca0\ub97c\ub9b0\uacfc \ubaa8\uc2a4\ud06c\ubc14 \uc0ac\uc774\uc758 \uc804\ub7b5 \ud611\ub825\uc744 \uac15\ud654\ud558\ub294 \uc2dc\ub3c4\uc5d0\uc11c \ub7ec\uc2dc\uc544\uc758 \ube14\ub77c\ub514\ubbf8\ub974 \ud478\ud2f4 \ub300\ud1b5\ub839\uacfc \uac00\uae4c\uc6b4 \uad00\uacc4\ub97c \uae4a\uac8c \ud558\uc600\ub2e4. \uc288\ub8b0\ub354\ub294 \ud478\ud2f4\uc774 \uc6b0\ud06c\ub77c\uc774\ub098\uc5d0\uc11c \uc624\ub80c\uc9c0 \ud601\uba85\uc774 \uc77c\uc5b4\ub098\ub294 \ub3d9\uc548\uc5d0 \ube45\ud1a0\ub974 \uc57c\ub204\ucf54\ube44\uce58\ub97c \ucd95\ud558\ud558\uae30 \uba70\uce60 \uc804 11\uc6d4 22\uc77c \ud478\ud2f4\uc744 \"\uc644\ubcbd\ud55c \ubbfc\uc8fc\uc8fc\uc758\uc790\"\ub85c \ubd80\ub978 \uc774\uc720\ub85c \ub300\uc911 \ub9e4\uccb4\uc5d0\uc11c, \uadf8\ub9ac\uace0 \uc774\uc5b4\uc11c \uc559\uac94\ub77c \uba54\ub974\ucf08\uc5d0 \uc758\ud558\uc5ec \ube44\ub09c\uc744 \ubc1b\uc558\ub2e4. \uc790\uc2e0\uc758 \uc218\uc0c1 \uc784\uae30\uac00 \uc2dc\uc791\ub41c \uc9c0 \uba70\uce60 \ub9cc\uc5d0 \uc288\ub8b0\ub354\ub294 \ud569\ub3d9 \ud22c\uae30\uc758 \uc774\uc0ac\uad6d\uc5d0 \uac00\uc785\ud558\uc5ec \uc790\uc2e0\uc758 \uc774\uc804\uc758 \uac1d\uad00\uc131\uc5d0 \uad00\ud55c \uc0c8\ub85c\uc6b4 \ud22c\uae30\ub4e4\uc744 \uac00\uc838\uc654\ub2e4. \uc790\uc2e0\uc758 \ud68c\uace0\ub85d \u300a \uacb0\uc815\ub4e4:\uc815\uce58\uc5d0\uc11c \ub098\uc758 \uc778\uc0dd \u300b\uc5d0\uc11c \uc288\ub8b0\ub354\ub294 \uc544\uc9c1\ub3c4 \uc790\uc2e0\uc758 \uce5c\uad6c\uc774\uc790 \uc815\uce58\uc801 \uc81c\ud734 \ud478\ud2f4\uc5d0 \uc758\uc9c0\ud558\uace0 \"\uad6d\ub0b4\uc801 \uc815\uce58 \uac1c\ud601\uacfc \ubbfc\uc8fc\uc801 \uac1c\ubc1c\uc758 \ube44\uc728\uc5d0 \uc62c \ub54c \ub7ec\uc2dc\uc544\uc5d0 \uacfc\ub3c4\ud55c \uc694\uad6c\ub4e4\uc744 \ub193\uac70\ub098 \uccb4\uccb8 \uc804\uc7c1\uc758 \uae30\ucd08\uc5d0 \uc644\uace0\ud558\uac8c \uc7ac\ud310\ud558\ub294 \ub370 \uc798\ubabb\ub418\uc5c8\uc744 \uac83\uc774\ub2e4\"\ub77c\uace0 \uc9c4\uc220\ud558\uc600\ub2e4.", "sentence_answer": "\uc790\uc2e0\uc758 \ud68c\uace0\ub85d \u300a \uacb0\uc815\ub4e4:\uc815\uce58\uc5d0\uc11c \ub098\uc758 \uc778\uc0dd \u300b\uc5d0\uc11c \uc288\ub8b0\ub354\ub294 \uc544\uc9c1\ub3c4 \uc790\uc2e0\uc758 \uce5c\uad6c\uc774\uc790 \uc815\uce58\uc801 \uc81c\ud734 \ud478\ud2f4\uc5d0 \uc758\uc9c0\ud558\uace0 \"\uad6d\ub0b4\uc801 \uc815\uce58 \uac1c\ud601\uacfc \ubbfc\uc8fc\uc801 \uac1c\ubc1c\uc758 \ube44\uc728\uc5d0 \uc62c \ub54c \ub7ec\uc2dc\uc544\uc5d0 \uacfc\ub3c4\ud55c \uc694\uad6c\ub4e4\uc744 \ub193\uac70\ub098 \uccb4\uccb8 \uc804\uc7c1\uc758 \uae30\ucd08\uc5d0 \uc644\uace0\ud558\uac8c \uc7ac\ud310\ud558\ub294 \ub370 \uc798\ubabb\ub418\uc5c8\uc744 \uac83\uc774\ub2e4\"\ub77c\uace0 \uc9c4\uc220\ud558\uc600\ub2e4.", "paragraph_id": "6601078-2-1", "paragraph_question": "question: \uc288\ub8b0\ub354\uac00 \ub0b8 \ud68c\uace0\ub85d\uc740?, context: \uc790\ud06c \uc2dc\ub77c\ud06c \ub300\ud1b5\ub839\uacfc \uc6b0\ud638\uc801\uc778 \uad00\uacc4\uc5d0 \ucd94\uac00\ub85c \uc288\ub8b0\ub354\ub294 \ub7ec\uc2dc\uc544\uc640 \ub3c5\uc77c \uc0ac\uc774\uc5d0 \ubc1c\ud2b8\ud574\uc5d0 \uac00\uc2a4 \ud30c\uc774\ud504\ub77c\uc778\uc744 \uac1c\uc7a5\ud55c \uac83\uc744 \ud3ec\ud568\ud55c \ubca0\ub97c\ub9b0\uacfc \ubaa8\uc2a4\ud06c\ubc14 \uc0ac\uc774\uc758 \uc804\ub7b5 \ud611\ub825\uc744 \uac15\ud654\ud558\ub294 \uc2dc\ub3c4\uc5d0\uc11c \ub7ec\uc2dc\uc544\uc758 \ube14\ub77c\ub514\ubbf8\ub974 \ud478\ud2f4 \ub300\ud1b5\ub839\uacfc \uac00\uae4c\uc6b4 \uad00\uacc4\ub97c \uae4a\uac8c \ud558\uc600\ub2e4. \uc288\ub8b0\ub354\ub294 \ud478\ud2f4\uc774 \uc6b0\ud06c\ub77c\uc774\ub098\uc5d0\uc11c \uc624\ub80c\uc9c0 \ud601\uba85\uc774 \uc77c\uc5b4\ub098\ub294 \ub3d9\uc548\uc5d0 \ube45\ud1a0\ub974 \uc57c\ub204\ucf54\ube44\uce58\ub97c \ucd95\ud558\ud558\uae30 \uba70\uce60 \uc804 11\uc6d4 22\uc77c \ud478\ud2f4\uc744 \"\uc644\ubcbd\ud55c \ubbfc\uc8fc\uc8fc\uc758\uc790\"\ub85c \ubd80\ub978 \uc774\uc720\ub85c \ub300\uc911 \ub9e4\uccb4\uc5d0\uc11c, \uadf8\ub9ac\uace0 \uc774\uc5b4\uc11c \uc559\uac94\ub77c \uba54\ub974\ucf08\uc5d0 \uc758\ud558\uc5ec \ube44\ub09c\uc744 \ubc1b\uc558\ub2e4. \uc790\uc2e0\uc758 \uc218\uc0c1 \uc784\uae30\uac00 \uc2dc\uc791\ub41c \uc9c0 \uba70\uce60 \ub9cc\uc5d0 \uc288\ub8b0\ub354\ub294 \ud569\ub3d9 \ud22c\uae30\uc758 \uc774\uc0ac\uad6d\uc5d0 \uac00\uc785\ud558\uc5ec \uc790\uc2e0\uc758 \uc774\uc804\uc758 \uac1d\uad00\uc131\uc5d0 \uad00\ud55c \uc0c8\ub85c\uc6b4 \ud22c\uae30\ub4e4\uc744 \uac00\uc838\uc654\ub2e4. \uc790\uc2e0\uc758 \ud68c\uace0\ub85d \u300a\uacb0\uc815\ub4e4:\uc815\uce58\uc5d0\uc11c \ub098\uc758 \uc778\uc0dd\u300b\uc5d0\uc11c \uc288\ub8b0\ub354\ub294 \uc544\uc9c1\ub3c4 \uc790\uc2e0\uc758 \uce5c\uad6c\uc774\uc790 \uc815\uce58\uc801 \uc81c\ud734 \ud478\ud2f4\uc5d0 \uc758\uc9c0\ud558\uace0 \"\uad6d\ub0b4\uc801 \uc815\uce58 \uac1c\ud601\uacfc \ubbfc\uc8fc\uc801 \uac1c\ubc1c\uc758 \ube44\uc728\uc5d0 \uc62c \ub54c \ub7ec\uc2dc\uc544\uc5d0 \uacfc\ub3c4\ud55c \uc694\uad6c\ub4e4\uc744 \ub193\uac70\ub098 \uccb4\uccb8 \uc804\uc7c1\uc758 \uae30\ucd08\uc5d0 \uc644\uace0\ud558\uac8c \uc7ac\ud310\ud558\ub294 \ub370 \uc798\ubabb\ub418\uc5c8\uc744 \uac83\uc774\ub2e4\"\ub77c\uace0 \uc9c4\uc220\ud558\uc600\ub2e4."} {"question": "\ub9e5\uba38\ub2c8\uac00 \ud544\uc694\ud55c \ubd80\ud488\uc744 \uc815\ud655\ud55c \uc790\ub9ac\uc5d0 \uac16\ucd94\ub3c4\ub85d \ub514\uc790\uc778 \ub418\uae30 \uc704\ud574 \uac00\uc9c0\uac8c \ub41c \ucde8\uc57d\uc810\uc740?", "paragraph": "\ub9e5\ubbf8\ub2c8\ub294 \uac00\uaca9\ub300\ube44 \uc131\ub2a5\ube44\ub97c \uac10\uc548\ud55c \uc564\ud2b8\ub9ac \ub808\ubca8 \ub370\uc2a4\ud06c\ud1b1\uc774\uc5c8\uc73c\ub098, 2011\ub144\uc5d0 \uc774\ub974\ub7ec\uc11c\ub294 \uc800\uc804\ub825 \ucf54\uc5b4\ub97c \ud0d1\uc7ac\ud558\uc600\uc73c\ub098 \ub2e4\ub978 \ucef4\ud4e8\ud130\uc640 \ub2e4\ub974\uc9c0 \uc54a\uc740 \uc131\ub2a5\uc744 \uac16\ucd94\uae30 \uc2dc\uc791\ud588\ub2e4. \ud558\uc9c0\ub9cc \uc5ec\uc804\ud788 \uc77c\ubc18\uc801\uc778 \ub370\uc2a4\ud06c\ud1b1 \ucef4\ud4e8\ud130\uc640 \ube44\uad50\ud558\uae30\ub294 \ud798\ub4e4\ub2e4. 3.5\uc778\uce58 \ud558\ub4dc \ub514\uc2a4\ud06c \ub4dc\ub77c\uc774\ube0c\uc640 \uc560\ud50c\uc774 \uc124\uacc4\ud55c \uc560\ud50c \uc804\uc6a9 \ub7a9\ud0d1 \ubd80\ud488\uc744 \uc774\uc6a9\ud55c\ub2e4. \uadf8\ub807\uae30 \ub54c\ubb38\uc5d0 \ub9e5\ubbf8\ub2c8\uc5d0\ub294 \ubc18\ub4dc\uc2dc \ud544\uc694\ud55c \ubd80\ud488\uc774 \uc644\ubcbd\ud55c \uc790\ub9ac\ub97c \uac16\ucd94\ub3c4\ub85d \uc124\uacc4\ub418\uc5c8\uc73c\uba70, \uc774 \ub54c\ubb38\uc5d0 \ubc1c\uc5f4 \ubb38\uc81c\uc5d0 \ucde8\uc57d\ud55c \ucf64\ud329\ud2b8\ud55c \ub514\uc790\uc778\uc744 \uac00\uc9c0\uace0 \uc788\ub2e4. \uc774\uc804\uc758 \ubaa8\ub378\ub4e4\uc758 \uacbd\uc6b0 \ubd80\ud488\uc744 \uc870\uae08\uc529 \uc120\ud0dd\ud560 \uc218 \uc788\uc5c8\uc73c\uba70, \uc57d\uac04 \ub290\ub9b0 \ub370\uc2a4\ud06c\ud1b1 \ucef4\ud4e8\ud130 \uc815\ub3c4\ub85c \ucde8\uae09\ub418\uc5c8\ub2e4. \uadf8\ub807\uae30 \ub54c\ubb38\uc5d02011\ub144 \uc0ac\uc591\ubd80\ud130\ub294 \uc774\ub7ec\ud55c \ubd88\ud3b8\uc0ac\ud56d\uc744 \ubc1b\uc544\ub4e4\uc5ec \ub2e4\ub978 \uc77c\ubc18\uc801\uc778 \ub370\uc2a4\ud06c\ud1b1\uacfc \ube44\uad50\ud560\ub9cc\ud55c \uc800\uc7a5\uc7a5\uce58\uc640 \uba54\ubaa8\ub9ac\ub97c \uac16\ucd94\ub3c4\ub85d \uac1c\uc120\ub418\uc5c8\ub2e4.", "answer": "\ubc1c\uc5f4 \ubb38\uc81c", "sentence": "\uc774 \ub54c\ubb38\uc5d0 \ubc1c\uc5f4 \ubb38\uc81c \uc5d0", "paragraph_sentence": "\ub9e5\ubbf8\ub2c8\ub294 \uac00\uaca9\ub300\ube44 \uc131\ub2a5\ube44\ub97c \uac10\uc548\ud55c \uc564\ud2b8\ub9ac \ub808\ubca8 \ub370\uc2a4\ud06c\ud1b1\uc774\uc5c8\uc73c\ub098, 2011\ub144\uc5d0 \uc774\ub974\ub7ec\uc11c\ub294 \uc800\uc804\ub825 \ucf54\uc5b4\ub97c \ud0d1\uc7ac\ud558\uc600\uc73c\ub098 \ub2e4\ub978 \ucef4\ud4e8\ud130\uc640 \ub2e4\ub974\uc9c0 \uc54a\uc740 \uc131\ub2a5\uc744 \uac16\ucd94\uae30 \uc2dc\uc791\ud588\ub2e4. \ud558\uc9c0\ub9cc \uc5ec\uc804\ud788 \uc77c\ubc18\uc801\uc778 \ub370\uc2a4\ud06c\ud1b1 \ucef4\ud4e8\ud130\uc640 \ube44\uad50\ud558\uae30\ub294 \ud798\ub4e4\ub2e4. 3.5\uc778\uce58 \ud558\ub4dc \ub514\uc2a4\ud06c \ub4dc\ub77c\uc774\ube0c\uc640 \uc560\ud50c\uc774 \uc124\uacc4\ud55c \uc560\ud50c \uc804\uc6a9 \ub7a9\ud0d1 \ubd80\ud488\uc744 \uc774\uc6a9\ud55c\ub2e4. \uadf8\ub807\uae30 \ub54c\ubb38\uc5d0 \ub9e5\ubbf8\ub2c8\uc5d0\ub294 \ubc18\ub4dc\uc2dc \ud544\uc694\ud55c \ubd80\ud488\uc774 \uc644\ubcbd\ud55c \uc790\ub9ac\ub97c \uac16\ucd94\ub3c4\ub85d \uc124\uacc4\ub418\uc5c8\uc73c\uba70, \uc774 \ub54c\ubb38\uc5d0 \ubc1c\uc5f4 \ubb38\uc81c \uc5d0 \ucde8\uc57d\ud55c \ucf64\ud329\ud2b8\ud55c \ub514\uc790\uc778\uc744 \uac00\uc9c0\uace0 \uc788\ub2e4. \uc774\uc804\uc758 \ubaa8\ub378\ub4e4\uc758 \uacbd\uc6b0 \ubd80\ud488\uc744 \uc870\uae08\uc529 \uc120\ud0dd\ud560 \uc218 \uc788\uc5c8\uc73c\uba70, \uc57d\uac04 \ub290\ub9b0 \ub370\uc2a4\ud06c\ud1b1 \ucef4\ud4e8\ud130 \uc815\ub3c4\ub85c \ucde8\uae09\ub418\uc5c8\ub2e4. \uadf8\ub807\uae30 \ub54c\ubb38\uc5d02011\ub144 \uc0ac\uc591\ubd80\ud130\ub294 \uc774\ub7ec\ud55c \ubd88\ud3b8\uc0ac\ud56d\uc744 \ubc1b\uc544\ub4e4\uc5ec \ub2e4\ub978 \uc77c\ubc18\uc801\uc778 \ub370\uc2a4\ud06c\ud1b1\uacfc \ube44\uad50\ud560\ub9cc\ud55c \uc800\uc7a5\uc7a5\uce58\uc640 \uba54\ubaa8\ub9ac\ub97c \uac16\ucd94\ub3c4\ub85d \uac1c\uc120\ub418\uc5c8\ub2e4.", "paragraph_answer": "\ub9e5\ubbf8\ub2c8\ub294 \uac00\uaca9\ub300\ube44 \uc131\ub2a5\ube44\ub97c \uac10\uc548\ud55c \uc564\ud2b8\ub9ac \ub808\ubca8 \ub370\uc2a4\ud06c\ud1b1\uc774\uc5c8\uc73c\ub098, 2011\ub144\uc5d0 \uc774\ub974\ub7ec\uc11c\ub294 \uc800\uc804\ub825 \ucf54\uc5b4\ub97c \ud0d1\uc7ac\ud558\uc600\uc73c\ub098 \ub2e4\ub978 \ucef4\ud4e8\ud130\uc640 \ub2e4\ub974\uc9c0 \uc54a\uc740 \uc131\ub2a5\uc744 \uac16\ucd94\uae30 \uc2dc\uc791\ud588\ub2e4. \ud558\uc9c0\ub9cc \uc5ec\uc804\ud788 \uc77c\ubc18\uc801\uc778 \ub370\uc2a4\ud06c\ud1b1 \ucef4\ud4e8\ud130\uc640 \ube44\uad50\ud558\uae30\ub294 \ud798\ub4e4\ub2e4. 3.5\uc778\uce58 \ud558\ub4dc \ub514\uc2a4\ud06c \ub4dc\ub77c\uc774\ube0c\uc640 \uc560\ud50c\uc774 \uc124\uacc4\ud55c \uc560\ud50c \uc804\uc6a9 \ub7a9\ud0d1 \ubd80\ud488\uc744 \uc774\uc6a9\ud55c\ub2e4. \uadf8\ub807\uae30 \ub54c\ubb38\uc5d0 \ub9e5\ubbf8\ub2c8\uc5d0\ub294 \ubc18\ub4dc\uc2dc \ud544\uc694\ud55c \ubd80\ud488\uc774 \uc644\ubcbd\ud55c \uc790\ub9ac\ub97c \uac16\ucd94\ub3c4\ub85d \uc124\uacc4\ub418\uc5c8\uc73c\uba70, \uc774 \ub54c\ubb38\uc5d0 \ubc1c\uc5f4 \ubb38\uc81c \uc5d0 \ucde8\uc57d\ud55c \ucf64\ud329\ud2b8\ud55c \ub514\uc790\uc778\uc744 \uac00\uc9c0\uace0 \uc788\ub2e4. \uc774\uc804\uc758 \ubaa8\ub378\ub4e4\uc758 \uacbd\uc6b0 \ubd80\ud488\uc744 \uc870\uae08\uc529 \uc120\ud0dd\ud560 \uc218 \uc788\uc5c8\uc73c\uba70, \uc57d\uac04 \ub290\ub9b0 \ub370\uc2a4\ud06c\ud1b1 \ucef4\ud4e8\ud130 \uc815\ub3c4\ub85c \ucde8\uae09\ub418\uc5c8\ub2e4. \uadf8\ub807\uae30 \ub54c\ubb38\uc5d02011\ub144 \uc0ac\uc591\ubd80\ud130\ub294 \uc774\ub7ec\ud55c \ubd88\ud3b8\uc0ac\ud56d\uc744 \ubc1b\uc544\ub4e4\uc5ec \ub2e4\ub978 \uc77c\ubc18\uc801\uc778 \ub370\uc2a4\ud06c\ud1b1\uacfc \ube44\uad50\ud560\ub9cc\ud55c \uc800\uc7a5\uc7a5\uce58\uc640 \uba54\ubaa8\ub9ac\ub97c \uac16\ucd94\ub3c4\ub85d \uac1c\uc120\ub418\uc5c8\ub2e4.", "sentence_answer": "\uc774 \ub54c\ubb38\uc5d0 \ubc1c\uc5f4 \ubb38\uc81c \uc5d0", "paragraph_id": "6075058-0-1", "paragraph_question": "question: \ub9e5\uba38\ub2c8\uac00 \ud544\uc694\ud55c \ubd80\ud488\uc744 \uc815\ud655\ud55c \uc790\ub9ac\uc5d0 \uac16\ucd94\ub3c4\ub85d \ub514\uc790\uc778 \ub418\uae30 \uc704\ud574 \uac00\uc9c0\uac8c \ub41c \ucde8\uc57d\uc810\uc740?, context: \ub9e5\ubbf8\ub2c8\ub294 \uac00\uaca9\ub300\ube44 \uc131\ub2a5\ube44\ub97c \uac10\uc548\ud55c \uc564\ud2b8\ub9ac \ub808\ubca8 \ub370\uc2a4\ud06c\ud1b1\uc774\uc5c8\uc73c\ub098, 2011\ub144\uc5d0 \uc774\ub974\ub7ec\uc11c\ub294 \uc800\uc804\ub825 \ucf54\uc5b4\ub97c \ud0d1\uc7ac\ud558\uc600\uc73c\ub098 \ub2e4\ub978 \ucef4\ud4e8\ud130\uc640 \ub2e4\ub974\uc9c0 \uc54a\uc740 \uc131\ub2a5\uc744 \uac16\ucd94\uae30 \uc2dc\uc791\ud588\ub2e4. \ud558\uc9c0\ub9cc \uc5ec\uc804\ud788 \uc77c\ubc18\uc801\uc778 \ub370\uc2a4\ud06c\ud1b1 \ucef4\ud4e8\ud130\uc640 \ube44\uad50\ud558\uae30\ub294 \ud798\ub4e4\ub2e4. 3.5\uc778\uce58 \ud558\ub4dc \ub514\uc2a4\ud06c \ub4dc\ub77c\uc774\ube0c\uc640 \uc560\ud50c\uc774 \uc124\uacc4\ud55c \uc560\ud50c \uc804\uc6a9 \ub7a9\ud0d1 \ubd80\ud488\uc744 \uc774\uc6a9\ud55c\ub2e4. \uadf8\ub807\uae30 \ub54c\ubb38\uc5d0 \ub9e5\ubbf8\ub2c8\uc5d0\ub294 \ubc18\ub4dc\uc2dc \ud544\uc694\ud55c \ubd80\ud488\uc774 \uc644\ubcbd\ud55c \uc790\ub9ac\ub97c \uac16\ucd94\ub3c4\ub85d \uc124\uacc4\ub418\uc5c8\uc73c\uba70, \uc774 \ub54c\ubb38\uc5d0 \ubc1c\uc5f4 \ubb38\uc81c\uc5d0 \ucde8\uc57d\ud55c \ucf64\ud329\ud2b8\ud55c \ub514\uc790\uc778\uc744 \uac00\uc9c0\uace0 \uc788\ub2e4. \uc774\uc804\uc758 \ubaa8\ub378\ub4e4\uc758 \uacbd\uc6b0 \ubd80\ud488\uc744 \uc870\uae08\uc529 \uc120\ud0dd\ud560 \uc218 \uc788\uc5c8\uc73c\uba70, \uc57d\uac04 \ub290\ub9b0 \ub370\uc2a4\ud06c\ud1b1 \ucef4\ud4e8\ud130 \uc815\ub3c4\ub85c \ucde8\uae09\ub418\uc5c8\ub2e4. \uadf8\ub807\uae30 \ub54c\ubb38\uc5d02011\ub144 \uc0ac\uc591\ubd80\ud130\ub294 \uc774\ub7ec\ud55c \ubd88\ud3b8\uc0ac\ud56d\uc744 \ubc1b\uc544\ub4e4\uc5ec \ub2e4\ub978 \uc77c\ubc18\uc801\uc778 \ub370\uc2a4\ud06c\ud1b1\uacfc \ube44\uad50\ud560\ub9cc\ud55c \uc800\uc7a5\uc7a5\uce58\uc640 \uba54\ubaa8\ub9ac\ub97c \uac16\ucd94\ub3c4\ub85d \uac1c\uc120\ub418\uc5c8\ub2e4."} {"question": "\ud398\ub974\uc18c\ub098\uac00 \ub2e4\uc591\ud55c \uc870\uc9c1\uc5d0\uc11c \uc0ac\uc6a9\uc790 \uc774\ud574\ub97c \uc704\ud574 \uc5b4\ub5a0\ud55c \ub3c4\uad6c\ub85c \uc774\uc6a9\ub420 \uc218 \uc788\uc744\uae4c?", "paragraph": "\ud398\ub974\uc18c\ub098\uc758 \ubc29\ubc95\ub860\uc758 \uc120\uad6c\uc790\uc778 \ud504\ub8e8\uc774\ud2b8\uc640 \uc544\ub4dc\ub9b0\uc5d0 \ub530\ub974\uba74 \ud398\ub974\uc18c\ub098\uc758 \uc0ac\uc6a9\uc740 \uc81c\ud488 \uac1c\ubc1c\uc5d0 \uc788\uc5b4\uc11c \uba87\uac00\uc9c0 \uc911\uc694\ud55c \uc774\uc810\uc744 \uac00\uc9c0\ub294 \uc5f0\uad6c \ubc29\ubc95\uc774\ub2e4. \uc774 \ubc29\ubc95\uc744 \uc0ac\uc6a9\ud558\uba74 \ubcf5\uc7a1\ud55c \ub370\uc774\ud130\ub098 \ud1b5\uacc4\uc790\ub8cc\uc5d0 \uc758\uc874\ud558\uc5ec \uc2dc\uc7a5\uacfc \uc0ac\uc6a9\uc790\ub97c \uc774\ud574\ud558\ub294 \ub300\uc2e0 \uc778\uac04\uc758 \uc5bc\uad74\uc744 \uac00\uc9c4 \ud55c \uac1c\uc778\uc801 \uc778\uaca9\uccb4\ub85c\uc11c \ubcf4\ub2e4 \uc778\uc9c0\uc801\uc73c\ub85c \uac00\uae5d\uac8c \ub290\ub084 \uc218 \uc788\uac8c \ub41c\ub2e4. \uc5b4\ub5a4 \uac00\uc0c1\uc801 \uc778\ubb3c\uc774 \uac00\uc9c8 \ud544\uc694\uc5d0 \ub300\ud574 \uc0dd\uac01\ud558\uac8c \ub418\uba74 \ub514\uc790\uc774\ub108\ub294 \uc2e4\uc81c \uc0ac\uc6a9\uc790\ub4e4\uc774 \ud544\uc694\ud558\uace0 \ub290\ub084\ub9cc\ud55c \uac83\uc5d0 \ub300\ud574 \ubcf4\ub2e4 \uc27d\uac8c \uc774\ud574\ud558\uace0 \uc811\uadfc\ud560 \uc218 \uc788\uac8c \ub41c\ub2e4. \ud398\ub974\uc18c\ub098\ub294 \ube0c\ub808\uc778 \uc2a4\ud1a0\ubc0d\uc774\ub098 \uc720\uc2a4\ucf00\uc774\uc2a4 \ubd84\uc11d \ud639\uc740 \uae30\ub2a5 \uc815\uc758\ub4f1\uc758 \uac1c\ubc1c\uacfc\uc815\uc5d0 \ub2e4\uc591\ud558\uac8c \uc4f0\uc77c \uc218 \uc788\ub2e4. \ub610\ud55c \ud398\ub974\uc18c\ub098\ub294 \ub9c8\ucf00\ud305, \ub514\uc790\uc778, \ud310\ub9e4\ub4f1 \ub2e4\uc591\ud55c \uc870\uc9c1\uac04\uc5d0 \uc0ac\uc6a9\uc790 \uc774\ud574\ub97c \uc704\ud55c \uc0c1\ud638\uc18c\ud1b5\uc758 \ub3c4\uad6c\ub85c\uc11c \uc720\uc6a9\ud558\uac8c \uc774\uc6a9\ud560 \uc218 \uc788\ub2e4.", "answer": "\uc0c1\ud638\uc18c\ud1b5\uc758 \ub3c4\uad6c", "sentence": "\ub610\ud55c \ud398\ub974\uc18c\ub098\ub294 \ub9c8\ucf00\ud305, \ub514\uc790\uc778, \ud310\ub9e4\ub4f1 \ub2e4\uc591\ud55c \uc870\uc9c1\uac04\uc5d0 \uc0ac\uc6a9\uc790 \uc774\ud574\ub97c \uc704\ud55c \uc0c1\ud638\uc18c\ud1b5\uc758 \ub3c4\uad6c \ub85c\uc11c \uc720\uc6a9\ud558\uac8c \uc774\uc6a9\ud560 \uc218 \uc788\ub2e4.", "paragraph_sentence": "\ud398\ub974\uc18c\ub098\uc758 \ubc29\ubc95\ub860\uc758 \uc120\uad6c\uc790\uc778 \ud504\ub8e8\uc774\ud2b8\uc640 \uc544\ub4dc\ub9b0\uc5d0 \ub530\ub974\uba74 \ud398\ub974\uc18c\ub098\uc758 \uc0ac\uc6a9\uc740 \uc81c\ud488 \uac1c\ubc1c\uc5d0 \uc788\uc5b4\uc11c \uba87\uac00\uc9c0 \uc911\uc694\ud55c \uc774\uc810\uc744 \uac00\uc9c0\ub294 \uc5f0\uad6c \ubc29\ubc95\uc774\ub2e4. \uc774 \ubc29\ubc95\uc744 \uc0ac\uc6a9\ud558\uba74 \ubcf5\uc7a1\ud55c \ub370\uc774\ud130\ub098 \ud1b5\uacc4\uc790\ub8cc\uc5d0 \uc758\uc874\ud558\uc5ec \uc2dc\uc7a5\uacfc \uc0ac\uc6a9\uc790\ub97c \uc774\ud574\ud558\ub294 \ub300\uc2e0 \uc778\uac04\uc758 \uc5bc\uad74\uc744 \uac00\uc9c4 \ud55c \uac1c\uc778\uc801 \uc778\uaca9\uccb4\ub85c\uc11c \ubcf4\ub2e4 \uc778\uc9c0\uc801\uc73c\ub85c \uac00\uae5d\uac8c \ub290\ub084 \uc218 \uc788\uac8c \ub41c\ub2e4. \uc5b4\ub5a4 \uac00\uc0c1\uc801 \uc778\ubb3c\uc774 \uac00\uc9c8 \ud544\uc694\uc5d0 \ub300\ud574 \uc0dd\uac01\ud558\uac8c \ub418\uba74 \ub514\uc790\uc774\ub108\ub294 \uc2e4\uc81c \uc0ac\uc6a9\uc790\ub4e4\uc774 \ud544\uc694\ud558\uace0 \ub290\ub084\ub9cc\ud55c \uac83\uc5d0 \ub300\ud574 \ubcf4\ub2e4 \uc27d\uac8c \uc774\ud574\ud558\uace0 \uc811\uadfc\ud560 \uc218 \uc788\uac8c \ub41c\ub2e4. \ud398\ub974\uc18c\ub098\ub294 \ube0c\ub808\uc778 \uc2a4\ud1a0\ubc0d\uc774\ub098 \uc720\uc2a4\ucf00\uc774\uc2a4 \ubd84\uc11d \ud639\uc740 \uae30\ub2a5 \uc815\uc758\ub4f1\uc758 \uac1c\ubc1c\uacfc\uc815\uc5d0 \ub2e4\uc591\ud558\uac8c \uc4f0\uc77c \uc218 \uc788\ub2e4. \ub610\ud55c \ud398\ub974\uc18c\ub098\ub294 \ub9c8\ucf00\ud305, \ub514\uc790\uc778, \ud310\ub9e4\ub4f1 \ub2e4\uc591\ud55c \uc870\uc9c1\uac04\uc5d0 \uc0ac\uc6a9\uc790 \uc774\ud574\ub97c \uc704\ud55c \uc0c1\ud638\uc18c\ud1b5\uc758 \ub3c4\uad6c \ub85c\uc11c \uc720\uc6a9\ud558\uac8c \uc774\uc6a9\ud560 \uc218 \uc788\ub2e4. ", "paragraph_answer": "\ud398\ub974\uc18c\ub098\uc758 \ubc29\ubc95\ub860\uc758 \uc120\uad6c\uc790\uc778 \ud504\ub8e8\uc774\ud2b8\uc640 \uc544\ub4dc\ub9b0\uc5d0 \ub530\ub974\uba74 \ud398\ub974\uc18c\ub098\uc758 \uc0ac\uc6a9\uc740 \uc81c\ud488 \uac1c\ubc1c\uc5d0 \uc788\uc5b4\uc11c \uba87\uac00\uc9c0 \uc911\uc694\ud55c \uc774\uc810\uc744 \uac00\uc9c0\ub294 \uc5f0\uad6c \ubc29\ubc95\uc774\ub2e4. \uc774 \ubc29\ubc95\uc744 \uc0ac\uc6a9\ud558\uba74 \ubcf5\uc7a1\ud55c \ub370\uc774\ud130\ub098 \ud1b5\uacc4\uc790\ub8cc\uc5d0 \uc758\uc874\ud558\uc5ec \uc2dc\uc7a5\uacfc \uc0ac\uc6a9\uc790\ub97c \uc774\ud574\ud558\ub294 \ub300\uc2e0 \uc778\uac04\uc758 \uc5bc\uad74\uc744 \uac00\uc9c4 \ud55c \uac1c\uc778\uc801 \uc778\uaca9\uccb4\ub85c\uc11c \ubcf4\ub2e4 \uc778\uc9c0\uc801\uc73c\ub85c \uac00\uae5d\uac8c \ub290\ub084 \uc218 \uc788\uac8c \ub41c\ub2e4. \uc5b4\ub5a4 \uac00\uc0c1\uc801 \uc778\ubb3c\uc774 \uac00\uc9c8 \ud544\uc694\uc5d0 \ub300\ud574 \uc0dd\uac01\ud558\uac8c \ub418\uba74 \ub514\uc790\uc774\ub108\ub294 \uc2e4\uc81c \uc0ac\uc6a9\uc790\ub4e4\uc774 \ud544\uc694\ud558\uace0 \ub290\ub084\ub9cc\ud55c \uac83\uc5d0 \ub300\ud574 \ubcf4\ub2e4 \uc27d\uac8c \uc774\ud574\ud558\uace0 \uc811\uadfc\ud560 \uc218 \uc788\uac8c \ub41c\ub2e4. \ud398\ub974\uc18c\ub098\ub294 \ube0c\ub808\uc778 \uc2a4\ud1a0\ubc0d\uc774\ub098 \uc720\uc2a4\ucf00\uc774\uc2a4 \ubd84\uc11d \ud639\uc740 \uae30\ub2a5 \uc815\uc758\ub4f1\uc758 \uac1c\ubc1c\uacfc\uc815\uc5d0 \ub2e4\uc591\ud558\uac8c \uc4f0\uc77c \uc218 \uc788\ub2e4. \ub610\ud55c \ud398\ub974\uc18c\ub098\ub294 \ub9c8\ucf00\ud305, \ub514\uc790\uc778, \ud310\ub9e4\ub4f1 \ub2e4\uc591\ud55c \uc870\uc9c1\uac04\uc5d0 \uc0ac\uc6a9\uc790 \uc774\ud574\ub97c \uc704\ud55c \uc0c1\ud638\uc18c\ud1b5\uc758 \ub3c4\uad6c \ub85c\uc11c \uc720\uc6a9\ud558\uac8c \uc774\uc6a9\ud560 \uc218 \uc788\ub2e4.", "sentence_answer": "\ub610\ud55c \ud398\ub974\uc18c\ub098\ub294 \ub9c8\ucf00\ud305, \ub514\uc790\uc778, \ud310\ub9e4\ub4f1 \ub2e4\uc591\ud55c \uc870\uc9c1\uac04\uc5d0 \uc0ac\uc6a9\uc790 \uc774\ud574\ub97c \uc704\ud55c \uc0c1\ud638\uc18c\ud1b5\uc758 \ub3c4\uad6c \ub85c\uc11c \uc720\uc6a9\ud558\uac8c \uc774\uc6a9\ud560 \uc218 \uc788\ub2e4.", "paragraph_id": "6567116-2-0", "paragraph_question": "question: \ud398\ub974\uc18c\ub098\uac00 \ub2e4\uc591\ud55c \uc870\uc9c1\uc5d0\uc11c \uc0ac\uc6a9\uc790 \uc774\ud574\ub97c \uc704\ud574 \uc5b4\ub5a0\ud55c \ub3c4\uad6c\ub85c \uc774\uc6a9\ub420 \uc218 \uc788\uc744\uae4c?, context: \ud398\ub974\uc18c\ub098\uc758 \ubc29\ubc95\ub860\uc758 \uc120\uad6c\uc790\uc778 \ud504\ub8e8\uc774\ud2b8\uc640 \uc544\ub4dc\ub9b0\uc5d0 \ub530\ub974\uba74 \ud398\ub974\uc18c\ub098\uc758 \uc0ac\uc6a9\uc740 \uc81c\ud488 \uac1c\ubc1c\uc5d0 \uc788\uc5b4\uc11c \uba87\uac00\uc9c0 \uc911\uc694\ud55c \uc774\uc810\uc744 \uac00\uc9c0\ub294 \uc5f0\uad6c \ubc29\ubc95\uc774\ub2e4. \uc774 \ubc29\ubc95\uc744 \uc0ac\uc6a9\ud558\uba74 \ubcf5\uc7a1\ud55c \ub370\uc774\ud130\ub098 \ud1b5\uacc4\uc790\ub8cc\uc5d0 \uc758\uc874\ud558\uc5ec \uc2dc\uc7a5\uacfc \uc0ac\uc6a9\uc790\ub97c \uc774\ud574\ud558\ub294 \ub300\uc2e0 \uc778\uac04\uc758 \uc5bc\uad74\uc744 \uac00\uc9c4 \ud55c \uac1c\uc778\uc801 \uc778\uaca9\uccb4\ub85c\uc11c \ubcf4\ub2e4 \uc778\uc9c0\uc801\uc73c\ub85c \uac00\uae5d\uac8c \ub290\ub084 \uc218 \uc788\uac8c \ub41c\ub2e4. \uc5b4\ub5a4 \uac00\uc0c1\uc801 \uc778\ubb3c\uc774 \uac00\uc9c8 \ud544\uc694\uc5d0 \ub300\ud574 \uc0dd\uac01\ud558\uac8c \ub418\uba74 \ub514\uc790\uc774\ub108\ub294 \uc2e4\uc81c \uc0ac\uc6a9\uc790\ub4e4\uc774 \ud544\uc694\ud558\uace0 \ub290\ub084\ub9cc\ud55c \uac83\uc5d0 \ub300\ud574 \ubcf4\ub2e4 \uc27d\uac8c \uc774\ud574\ud558\uace0 \uc811\uadfc\ud560 \uc218 \uc788\uac8c \ub41c\ub2e4. \ud398\ub974\uc18c\ub098\ub294 \ube0c\ub808\uc778 \uc2a4\ud1a0\ubc0d\uc774\ub098 \uc720\uc2a4\ucf00\uc774\uc2a4 \ubd84\uc11d \ud639\uc740 \uae30\ub2a5 \uc815\uc758\ub4f1\uc758 \uac1c\ubc1c\uacfc\uc815\uc5d0 \ub2e4\uc591\ud558\uac8c \uc4f0\uc77c \uc218 \uc788\ub2e4. \ub610\ud55c \ud398\ub974\uc18c\ub098\ub294 \ub9c8\ucf00\ud305, \ub514\uc790\uc778, \ud310\ub9e4\ub4f1 \ub2e4\uc591\ud55c \uc870\uc9c1\uac04\uc5d0 \uc0ac\uc6a9\uc790 \uc774\ud574\ub97c \uc704\ud55c \uc0c1\ud638\uc18c\ud1b5\uc758 \ub3c4\uad6c\ub85c\uc11c \uc720\uc6a9\ud558\uac8c \uc774\uc6a9\ud560 \uc218 \uc788\ub2e4."} {"question": "\uc544\uba54\ub9ac\uce78 \uac15\uac00\uc5d0 \uac74\uc124 \uc911\uc774\ub358 \uc81c\uc7ac\uc18c\uc758 \ubc29\uc218\ub85c\uc5d0\uc11c \ubc1c\uacac\ub41c\uac83\uc73c?", "paragraph": "\uace8\ub4dc \ub7ec\uc2dc\ub294 \uce98\ub9ac\ud3ec\ub2c8\uc544 \ucf5c\ub85c\ub9c8\uc5d0 \uac00\uae4c\uc6b4 \uc11c\ud130\uc2a4 \ubc00\uc5d0\uc11c \uc2dc\uc791\ub418\uc5c8\ub2e4. 1848\ub144 1\uc6d4 24\uc77c, \uc0c8\ud06c\ub77c\uba58\ud1a0\uc758 \uac1c\ucc99\uc790 \uc874 \uc11c\ud130\uac00 \uace0\uc6a9\ud55c \ud604\uc7a5 \uac10\ub3c5\uc774\uc5c8\ub358 \uc81c\uc784\uc2a4 \ub9c8\uc15c\uc740 \uc544\uba54\ub9ac\uce78 \uac15\uac00\uc5d0 \uac74\uc124\ud558\uace0 \uc788\uc5c8\ub358 \uc81c\uc7ac\uc18c\uc758 \ubc29\uc218\ub85c\uc5d0\uc11c \ube5b\ub098\ub294 \uc0ac\uae08\uc744 \ubc1c\uacac\ud588\ub2e4. \ub9c8\uc15c\uc740 \uc790\uc2e0\uc774 \ubc1c\uacac\ud55c \uac83\uc744 \uc870\uc6a9\ud788 \uc154\ud130\uc5d0\uac8c \uac00\uc838 \uac14\uace0 \ub450 \uba85\uc740 \uc740\ubc00\ud558\uac8c \ubc1c\uacac\ubb3c\uc744 \uc870\uc0ac\ud588\ub2e4. \uc774 \uc870\uc0ac\ub85c \ub9c8\uc15c\uc774 \uac00\uc838\uc628 \uc870\uac01\uc774 \uae08\uc778 \uac83\uc744 \uc54c\uac8c \ub418\uc5c8\ub2e4. \uc154\ud130\ub294 \uae08\uc758 \ub300\ub300\uc801\uc778 \ud0d0\uc0ac\uac00 \uc2dc\uc791\ub418\uba74 \ub300\uaddc\ubaa8 \ub18d\uacbd\uc744 \ud558\ub824\ud588\ub358 \uc790\uc2e0\uc758 \uacc4\ud68d\uc5d0 \ucc28\uc9c8\uc744 \ube5a\uc744\uae4c \ub450\ub824\uc6cc\ud588\uae30 \ub54c\ubb38\uc5d0 \ub2f9\ud669\ud588\uace0, \uc774 \uc0ac\uc2e4\uc744 \ube44\ubc00\uc5d0 \ubd80\ucce4\ub2e4. \uadf8\ub7ec\ub098 \uc758\ub3c4\uc640\ub294 \ub2ec\ub9ac \uc18c\ubb38\uc740 \ud37c\uc9c0\uae30 \uc2dc\uc791\ud588\uace0, 1848\ub144 3\uc6d4 \uc0cc\ud504\ub780\uc2dc\uc2a4\ucf54\uc758 \uc2e0\ubb38\uc0ac \uc0ac\uc8fc\ub85c \uc788\uc5c8\ub358 \uc0c1\uc778 \uc0c8\ubba4\uc5bc \ube0c\ub798\ub10c\uc5d0 \uc758\ud574\uc11c \ud655\uc778\ub418\uc5c8\ub2e4. \uadf8\ub294 \uae08\ud0d0\uc0ac\uc6a9 \ubb3c\uc790\ub97c \ud30c\ub294 \uc0c1\uc810\uc744 \uc11c\ub458\ub7ec \ucc28\ub9b0 \uc774\ud6c4\uc5d0 \uae08\uc744 \ub123\uc740 \uc791\uc740 \ubcd1\uc744 \ub4e4\uace0\uc11c, \uc0cc\ud504\ub780\uc2dc\uc2a4\ucf54 \ub300\ub85c\ub97c \ud65c\ubcf4\ud558\uba74\uc11c, \u201c\uae08\uc774\ub2e4! \uae08\uc774\ub2e4! \uc544\uba54\ub9ac\uce74 \uac15\uc5d0\uc11c \uae08\uc774 \ubc1c\uacac\ub418\uc5c8\ub2e4!\u201d\ub77c\uace0 \uc678\uce58\uace0 \ub2e4\ub154\ub2e4. \uc774\uac83\uc740 \uc774\ud6c4\uc758 \uadf8\uc758 \uac00\uc7a5 \uc720\uba85\ud55c \uc778\uc6a9\uad6c\uac00 \ub418\uc5c8\ub2e4. \uae08\uc774 \ubc1c\uacac\ub418\ub358 \ub2f9\uc2dc \uce98\ub9ac\ud3ec\ub2c8\uc544\ub294 \uba55\uc2dc\ucf54\uc758 \uc601\ud1a0\uc778 \uc54c\ud0c0 \uce98\ub9ac\ud3ec\ub2c8\uc544\uc758 \uc77c\ubd80\uc600\uace0, \uae08 \ubc1c\uacac\uc758 \uc9c1\ud6c4 1848\ub144 2\uc6d4 2\uc77c \ubbf8\uad6d-\uba55\uc2dc\ucf54 \uc804\uc7c1\uc744 \ub05d\ub0b4\uba74\uc11c \ub9fa\uc740 \uacfc\ub2ec\ub8e8\ud398-\uc774\ub2ec\uace0 \uc870\uc57d\uc5d0 \uc758\ud574 \uc774 \uc9c0\uc5ed\uacfc \uc54c\ud0c0 \uce98\ub9ac\ud3ec\ub2c8\uc544\uc758 \ub098\uba38\uc9c0 \ubd80\ubd84\uc774 \ubbf8\ud569\uc911\uad6d\uc5d0 \ud560\uc591 \ub418\uc5c8\ub2e4.", "answer": "\uc0ac\uae08", "sentence": "1848\ub144 1\uc6d4 24\uc77c, \uc0c8\ud06c\ub77c\uba58\ud1a0\uc758 \uac1c\ucc99\uc790 \uc874 \uc11c\ud130\uac00 \uace0\uc6a9\ud55c \ud604\uc7a5 \uac10\ub3c5\uc774\uc5c8\ub358 \uc81c\uc784\uc2a4 \ub9c8\uc15c\uc740 \uc544\uba54\ub9ac\uce78 \uac15\uac00\uc5d0 \uac74\uc124\ud558\uace0 \uc788\uc5c8\ub358 \uc81c\uc7ac\uc18c\uc758 \ubc29\uc218\ub85c\uc5d0\uc11c \ube5b\ub098\ub294 \uc0ac\uae08 \uc744 \ubc1c\uacac\ud588\ub2e4.", "paragraph_sentence": "\uace8\ub4dc \ub7ec\uc2dc\ub294 \uce98\ub9ac\ud3ec\ub2c8\uc544 \ucf5c\ub85c\ub9c8\uc5d0 \uac00\uae4c\uc6b4 \uc11c\ud130\uc2a4 \ubc00\uc5d0\uc11c \uc2dc\uc791\ub418\uc5c8\ub2e4. 1848\ub144 1\uc6d4 24\uc77c, \uc0c8\ud06c\ub77c\uba58\ud1a0\uc758 \uac1c\ucc99\uc790 \uc874 \uc11c\ud130\uac00 \uace0\uc6a9\ud55c \ud604\uc7a5 \uac10\ub3c5\uc774\uc5c8\ub358 \uc81c\uc784\uc2a4 \ub9c8\uc15c\uc740 \uc544\uba54\ub9ac\uce78 \uac15\uac00\uc5d0 \uac74\uc124\ud558\uace0 \uc788\uc5c8\ub358 \uc81c\uc7ac\uc18c\uc758 \ubc29\uc218\ub85c\uc5d0\uc11c \ube5b\ub098\ub294 \uc0ac\uae08 \uc744 \ubc1c\uacac\ud588\ub2e4. \ub9c8\uc15c\uc740 \uc790\uc2e0\uc774 \ubc1c\uacac\ud55c \uac83\uc744 \uc870\uc6a9\ud788 \uc154\ud130\uc5d0\uac8c \uac00\uc838 \uac14\uace0 \ub450 \uba85\uc740 \uc740\ubc00\ud558\uac8c \ubc1c\uacac\ubb3c\uc744 \uc870\uc0ac\ud588\ub2e4. \uc774 \uc870\uc0ac\ub85c \ub9c8\uc15c\uc774 \uac00\uc838\uc628 \uc870\uac01\uc774 \uae08\uc778 \uac83\uc744 \uc54c\uac8c \ub418\uc5c8\ub2e4. \uc154\ud130\ub294 \uae08\uc758 \ub300\ub300\uc801\uc778 \ud0d0\uc0ac\uac00 \uc2dc\uc791\ub418\uba74 \ub300\uaddc\ubaa8 \ub18d\uacbd\uc744 \ud558\ub824\ud588\ub358 \uc790\uc2e0\uc758 \uacc4\ud68d\uc5d0 \ucc28\uc9c8\uc744 \ube5a\uc744\uae4c \ub450\ub824\uc6cc\ud588\uae30 \ub54c\ubb38\uc5d0 \ub2f9\ud669\ud588\uace0, \uc774 \uc0ac\uc2e4\uc744 \ube44\ubc00\uc5d0 \ubd80\ucce4\ub2e4. \uadf8\ub7ec\ub098 \uc758\ub3c4\uc640\ub294 \ub2ec\ub9ac \uc18c\ubb38\uc740 \ud37c\uc9c0\uae30 \uc2dc\uc791\ud588\uace0, 1848\ub144 3\uc6d4 \uc0cc\ud504\ub780\uc2dc\uc2a4\ucf54\uc758 \uc2e0\ubb38\uc0ac \uc0ac\uc8fc\ub85c \uc788\uc5c8\ub358 \uc0c1\uc778 \uc0c8\ubba4\uc5bc \ube0c\ub798\ub10c\uc5d0 \uc758\ud574\uc11c \ud655\uc778\ub418\uc5c8\ub2e4. \uadf8\ub294 \uae08\ud0d0\uc0ac\uc6a9 \ubb3c\uc790\ub97c \ud30c\ub294 \uc0c1\uc810\uc744 \uc11c\ub458\ub7ec \ucc28\ub9b0 \uc774\ud6c4\uc5d0 \uae08\uc744 \ub123\uc740 \uc791\uc740 \ubcd1\uc744 \ub4e4\uace0\uc11c, \uc0cc\ud504\ub780\uc2dc\uc2a4\ucf54 \ub300\ub85c\ub97c \ud65c\ubcf4\ud558\uba74\uc11c, \u201c\uae08\uc774\ub2e4! \uae08\uc774\ub2e4! \uc544\uba54\ub9ac\uce74 \uac15\uc5d0\uc11c \uae08\uc774 \ubc1c\uacac\ub418\uc5c8\ub2e4!\u201d\ub77c\uace0 \uc678\uce58\uace0 \ub2e4\ub154\ub2e4. \uc774\uac83\uc740 \uc774\ud6c4\uc758 \uadf8\uc758 \uac00\uc7a5 \uc720\uba85\ud55c \uc778\uc6a9\uad6c\uac00 \ub418\uc5c8\ub2e4. \uae08\uc774 \ubc1c\uacac\ub418\ub358 \ub2f9\uc2dc \uce98\ub9ac\ud3ec\ub2c8\uc544\ub294 \uba55\uc2dc\ucf54\uc758 \uc601\ud1a0\uc778 \uc54c\ud0c0 \uce98\ub9ac\ud3ec\ub2c8\uc544\uc758 \uc77c\ubd80\uc600\uace0, \uae08 \ubc1c\uacac\uc758 \uc9c1\ud6c4 1848\ub144 2\uc6d4 2\uc77c \ubbf8\uad6d-\uba55\uc2dc\ucf54 \uc804\uc7c1\uc744 \ub05d\ub0b4\uba74\uc11c \ub9fa\uc740 \uacfc\ub2ec\ub8e8\ud398-\uc774\ub2ec\uace0 \uc870\uc57d\uc5d0 \uc758\ud574 \uc774 \uc9c0\uc5ed\uacfc \uc54c\ud0c0 \uce98\ub9ac\ud3ec\ub2c8\uc544\uc758 \ub098\uba38\uc9c0 \ubd80\ubd84\uc774 \ubbf8\ud569\uc911\uad6d\uc5d0 \ud560\uc591 \ub418\uc5c8\ub2e4.", "paragraph_answer": "\uace8\ub4dc \ub7ec\uc2dc\ub294 \uce98\ub9ac\ud3ec\ub2c8\uc544 \ucf5c\ub85c\ub9c8\uc5d0 \uac00\uae4c\uc6b4 \uc11c\ud130\uc2a4 \ubc00\uc5d0\uc11c \uc2dc\uc791\ub418\uc5c8\ub2e4. 1848\ub144 1\uc6d4 24\uc77c, \uc0c8\ud06c\ub77c\uba58\ud1a0\uc758 \uac1c\ucc99\uc790 \uc874 \uc11c\ud130\uac00 \uace0\uc6a9\ud55c \ud604\uc7a5 \uac10\ub3c5\uc774\uc5c8\ub358 \uc81c\uc784\uc2a4 \ub9c8\uc15c\uc740 \uc544\uba54\ub9ac\uce78 \uac15\uac00\uc5d0 \uac74\uc124\ud558\uace0 \uc788\uc5c8\ub358 \uc81c\uc7ac\uc18c\uc758 \ubc29\uc218\ub85c\uc5d0\uc11c \ube5b\ub098\ub294 \uc0ac\uae08 \uc744 \ubc1c\uacac\ud588\ub2e4. \ub9c8\uc15c\uc740 \uc790\uc2e0\uc774 \ubc1c\uacac\ud55c \uac83\uc744 \uc870\uc6a9\ud788 \uc154\ud130\uc5d0\uac8c \uac00\uc838 \uac14\uace0 \ub450 \uba85\uc740 \uc740\ubc00\ud558\uac8c \ubc1c\uacac\ubb3c\uc744 \uc870\uc0ac\ud588\ub2e4. \uc774 \uc870\uc0ac\ub85c \ub9c8\uc15c\uc774 \uac00\uc838\uc628 \uc870\uac01\uc774 \uae08\uc778 \uac83\uc744 \uc54c\uac8c \ub418\uc5c8\ub2e4. \uc154\ud130\ub294 \uae08\uc758 \ub300\ub300\uc801\uc778 \ud0d0\uc0ac\uac00 \uc2dc\uc791\ub418\uba74 \ub300\uaddc\ubaa8 \ub18d\uacbd\uc744 \ud558\ub824\ud588\ub358 \uc790\uc2e0\uc758 \uacc4\ud68d\uc5d0 \ucc28\uc9c8\uc744 \ube5a\uc744\uae4c \ub450\ub824\uc6cc\ud588\uae30 \ub54c\ubb38\uc5d0 \ub2f9\ud669\ud588\uace0, \uc774 \uc0ac\uc2e4\uc744 \ube44\ubc00\uc5d0 \ubd80\ucce4\ub2e4. \uadf8\ub7ec\ub098 \uc758\ub3c4\uc640\ub294 \ub2ec\ub9ac \uc18c\ubb38\uc740 \ud37c\uc9c0\uae30 \uc2dc\uc791\ud588\uace0, 1848\ub144 3\uc6d4 \uc0cc\ud504\ub780\uc2dc\uc2a4\ucf54\uc758 \uc2e0\ubb38\uc0ac \uc0ac\uc8fc\ub85c \uc788\uc5c8\ub358 \uc0c1\uc778 \uc0c8\ubba4\uc5bc \ube0c\ub798\ub10c\uc5d0 \uc758\ud574\uc11c \ud655\uc778\ub418\uc5c8\ub2e4. \uadf8\ub294 \uae08\ud0d0\uc0ac\uc6a9 \ubb3c\uc790\ub97c \ud30c\ub294 \uc0c1\uc810\uc744 \uc11c\ub458\ub7ec \ucc28\ub9b0 \uc774\ud6c4\uc5d0 \uae08\uc744 \ub123\uc740 \uc791\uc740 \ubcd1\uc744 \ub4e4\uace0\uc11c, \uc0cc\ud504\ub780\uc2dc\uc2a4\ucf54 \ub300\ub85c\ub97c \ud65c\ubcf4\ud558\uba74\uc11c, \u201c\uae08\uc774\ub2e4! \uae08\uc774\ub2e4! \uc544\uba54\ub9ac\uce74 \uac15\uc5d0\uc11c \uae08\uc774 \ubc1c\uacac\ub418\uc5c8\ub2e4!\u201d\ub77c\uace0 \uc678\uce58\uace0 \ub2e4\ub154\ub2e4. \uc774\uac83\uc740 \uc774\ud6c4\uc758 \uadf8\uc758 \uac00\uc7a5 \uc720\uba85\ud55c \uc778\uc6a9\uad6c\uac00 \ub418\uc5c8\ub2e4. \uae08\uc774 \ubc1c\uacac\ub418\ub358 \ub2f9\uc2dc \uce98\ub9ac\ud3ec\ub2c8\uc544\ub294 \uba55\uc2dc\ucf54\uc758 \uc601\ud1a0\uc778 \uc54c\ud0c0 \uce98\ub9ac\ud3ec\ub2c8\uc544\uc758 \uc77c\ubd80\uc600\uace0, \uae08 \ubc1c\uacac\uc758 \uc9c1\ud6c4 1848\ub144 2\uc6d4 2\uc77c \ubbf8\uad6d-\uba55\uc2dc\ucf54 \uc804\uc7c1\uc744 \ub05d\ub0b4\uba74\uc11c \ub9fa\uc740 \uacfc\ub2ec\ub8e8\ud398-\uc774\ub2ec\uace0 \uc870\uc57d\uc5d0 \uc758\ud574 \uc774 \uc9c0\uc5ed\uacfc \uc54c\ud0c0 \uce98\ub9ac\ud3ec\ub2c8\uc544\uc758 \ub098\uba38\uc9c0 \ubd80\ubd84\uc774 \ubbf8\ud569\uc911\uad6d\uc5d0 \ud560\uc591 \ub418\uc5c8\ub2e4.", "sentence_answer": "1848\ub144 1\uc6d4 24\uc77c, \uc0c8\ud06c\ub77c\uba58\ud1a0\uc758 \uac1c\ucc99\uc790 \uc874 \uc11c\ud130\uac00 \uace0\uc6a9\ud55c \ud604\uc7a5 \uac10\ub3c5\uc774\uc5c8\ub358 \uc81c\uc784\uc2a4 \ub9c8\uc15c\uc740 \uc544\uba54\ub9ac\uce78 \uac15\uac00\uc5d0 \uac74\uc124\ud558\uace0 \uc788\uc5c8\ub358 \uc81c\uc7ac\uc18c\uc758 \ubc29\uc218\ub85c\uc5d0\uc11c \ube5b\ub098\ub294 \uc0ac\uae08 \uc744 \ubc1c\uacac\ud588\ub2e4.", "paragraph_id": "6601248-2-2", "paragraph_question": "question: \uc544\uba54\ub9ac\uce78 \uac15\uac00\uc5d0 \uac74\uc124 \uc911\uc774\ub358 \uc81c\uc7ac\uc18c\uc758 \ubc29\uc218\ub85c\uc5d0\uc11c \ubc1c\uacac\ub41c\uac83\uc73c?, context: \uace8\ub4dc \ub7ec\uc2dc\ub294 \uce98\ub9ac\ud3ec\ub2c8\uc544 \ucf5c\ub85c\ub9c8\uc5d0 \uac00\uae4c\uc6b4 \uc11c\ud130\uc2a4 \ubc00\uc5d0\uc11c \uc2dc\uc791\ub418\uc5c8\ub2e4. 1848\ub144 1\uc6d4 24\uc77c, \uc0c8\ud06c\ub77c\uba58\ud1a0\uc758 \uac1c\ucc99\uc790 \uc874 \uc11c\ud130\uac00 \uace0\uc6a9\ud55c \ud604\uc7a5 \uac10\ub3c5\uc774\uc5c8\ub358 \uc81c\uc784\uc2a4 \ub9c8\uc15c\uc740 \uc544\uba54\ub9ac\uce78 \uac15\uac00\uc5d0 \uac74\uc124\ud558\uace0 \uc788\uc5c8\ub358 \uc81c\uc7ac\uc18c\uc758 \ubc29\uc218\ub85c\uc5d0\uc11c \ube5b\ub098\ub294 \uc0ac\uae08\uc744 \ubc1c\uacac\ud588\ub2e4. \ub9c8\uc15c\uc740 \uc790\uc2e0\uc774 \ubc1c\uacac\ud55c \uac83\uc744 \uc870\uc6a9\ud788 \uc154\ud130\uc5d0\uac8c \uac00\uc838 \uac14\uace0 \ub450 \uba85\uc740 \uc740\ubc00\ud558\uac8c \ubc1c\uacac\ubb3c\uc744 \uc870\uc0ac\ud588\ub2e4. \uc774 \uc870\uc0ac\ub85c \ub9c8\uc15c\uc774 \uac00\uc838\uc628 \uc870\uac01\uc774 \uae08\uc778 \uac83\uc744 \uc54c\uac8c \ub418\uc5c8\ub2e4. \uc154\ud130\ub294 \uae08\uc758 \ub300\ub300\uc801\uc778 \ud0d0\uc0ac\uac00 \uc2dc\uc791\ub418\uba74 \ub300\uaddc\ubaa8 \ub18d\uacbd\uc744 \ud558\ub824\ud588\ub358 \uc790\uc2e0\uc758 \uacc4\ud68d\uc5d0 \ucc28\uc9c8\uc744 \ube5a\uc744\uae4c \ub450\ub824\uc6cc\ud588\uae30 \ub54c\ubb38\uc5d0 \ub2f9\ud669\ud588\uace0, \uc774 \uc0ac\uc2e4\uc744 \ube44\ubc00\uc5d0 \ubd80\ucce4\ub2e4. \uadf8\ub7ec\ub098 \uc758\ub3c4\uc640\ub294 \ub2ec\ub9ac \uc18c\ubb38\uc740 \ud37c\uc9c0\uae30 \uc2dc\uc791\ud588\uace0, 1848\ub144 3\uc6d4 \uc0cc\ud504\ub780\uc2dc\uc2a4\ucf54\uc758 \uc2e0\ubb38\uc0ac \uc0ac\uc8fc\ub85c \uc788\uc5c8\ub358 \uc0c1\uc778 \uc0c8\ubba4\uc5bc \ube0c\ub798\ub10c\uc5d0 \uc758\ud574\uc11c \ud655\uc778\ub418\uc5c8\ub2e4. \uadf8\ub294 \uae08\ud0d0\uc0ac\uc6a9 \ubb3c\uc790\ub97c \ud30c\ub294 \uc0c1\uc810\uc744 \uc11c\ub458\ub7ec \ucc28\ub9b0 \uc774\ud6c4\uc5d0 \uae08\uc744 \ub123\uc740 \uc791\uc740 \ubcd1\uc744 \ub4e4\uace0\uc11c, \uc0cc\ud504\ub780\uc2dc\uc2a4\ucf54 \ub300\ub85c\ub97c \ud65c\ubcf4\ud558\uba74\uc11c, \u201c\uae08\uc774\ub2e4! \uae08\uc774\ub2e4! \uc544\uba54\ub9ac\uce74 \uac15\uc5d0\uc11c \uae08\uc774 \ubc1c\uacac\ub418\uc5c8\ub2e4!\u201d\ub77c\uace0 \uc678\uce58\uace0 \ub2e4\ub154\ub2e4. \uc774\uac83\uc740 \uc774\ud6c4\uc758 \uadf8\uc758 \uac00\uc7a5 \uc720\uba85\ud55c \uc778\uc6a9\uad6c\uac00 \ub418\uc5c8\ub2e4. \uae08\uc774 \ubc1c\uacac\ub418\ub358 \ub2f9\uc2dc \uce98\ub9ac\ud3ec\ub2c8\uc544\ub294 \uba55\uc2dc\ucf54\uc758 \uc601\ud1a0\uc778 \uc54c\ud0c0 \uce98\ub9ac\ud3ec\ub2c8\uc544\uc758 \uc77c\ubd80\uc600\uace0, \uae08 \ubc1c\uacac\uc758 \uc9c1\ud6c4 1848\ub144 2\uc6d4 2\uc77c \ubbf8\uad6d-\uba55\uc2dc\ucf54 \uc804\uc7c1\uc744 \ub05d\ub0b4\uba74\uc11c \ub9fa\uc740 \uacfc\ub2ec\ub8e8\ud398-\uc774\ub2ec\uace0 \uc870\uc57d\uc5d0 \uc758\ud574 \uc774 \uc9c0\uc5ed\uacfc \uc54c\ud0c0 \uce98\ub9ac\ud3ec\ub2c8\uc544\uc758 \ub098\uba38\uc9c0 \ubd80\ubd84\uc774 \ubbf8\ud569\uc911\uad6d\uc5d0 \ud560\uc591 \ub418\uc5c8\ub2e4."} {"question": "1958\ub144 \ub274\ud3ec\ud2b8 \uc7ac\uc988 \ud398\uc2a4\ud2f0\ubc8c\uc5d0\uc11c \uacf5\uc5f0\ud558\ub294 \ubaa8\uc2b5\uc774 \ucd2c\uc601\ub418\uc5b4 \uc81c\uc791\ub41c \uc601\ud654\ub294 \ubb34\uc5c7\uc778\uac00?", "paragraph": "1957\ub144 \ub9d0 \ubca0\ub9ac\ub294 \uc568\ub7f0 \ud504\ub9ac\ub4dc\uac00 \uae30\ud68d\ud55c \"Biggest Show of Stars for 1957\"\uc758 \ud55c \uc790\ub9ac\ub97c \ucc28\uc9c0\ud558\uac8c \ub41c\ub2e4. \uc774 \ud22c\uc5b4\uc5d0\ub294 \uc5d0\ubc8c\ub9ac \ube0c\ub77c\ub354\uc2a4\uc640 \ubc84\ub514 \ud640\ub9ac \ub4f1\uc774 \ub3d9\ud589\ud588\ub2e4. ABC\uc758 \u300a\uac00\uc774 \ubbf8\uccbc \uc1fc\u300b\uc5d0\uc11c\ub3c4 \uac8c\uc2a4\ud2b8\ub85c \ub4f1\uc7a5, \ud788\ud2b8\uace1 \u3008Rock 'n' Roll Music\u3009\uc744 \ubd88\ub800\ub2e4. \uadf8\uc758 \ud788\ud2b8\ub294 1957\ub144\uc5d0\uc11c 1959\ub144\uae4c\uc9c0 \uacc4\uc18d\ub418\uc5c8\uace0, \uc774 \uc2dc\uae30 24\uac1c\uac00 \ub118\ub294 \uace1\uc744 \ucc28\ud2b8\uc5d0 \uc9c4\uc785\uc2dc\ud0a4\ub294\ub370 \uc131\uacf5\ud55c\ub2e4. \ubbf8\uad6d \ucc28\ud2b8 10\uc704\uad8c\uc5d0\ub294 \u3008School Days\u3009, \u3008Rock and Roll Music\u3009, \u3008Sweet Little Sixteen\u3009, \u3008Johnny B. Goode\u3009 \ub4f1\uc774 \uc62c\ub790\ub2e4. \uc774\ubc16\uc5d0\ub3c4 \ub450 \ud3b8\uc758 \ucd08\uae30 \ub85c\ud070\ub864 \uc601\ud654\uc5d0 \ucd9c\uc5f0\ud55c \uadf8\ub294 \u300a\ub85d, \ub85d, \ub85d\u300b\uc5d0\uc11c \u3008You Can't Catch Me\u3009\ub97c \uacf5\uc5f0\ud588\uace0, \u300a\uace0, \uc790\ub2c8, \uace0!\u300b\uc5d0\uc11c \ubcf8\uc778\uc73c\ub85c \ucd9c\uc5f0\ud558\uc5ec \u3008Johnny B. Goode\u3009, \u3008Memphis, Tennessee\u3009, \u3008Little Queenie\u3009\ub97c \ubd88\ub800\ub2e4. 1958\ub144 \ub274\ud3ec\ud2b8 \uc7ac\uc988 \ud398\uc2a4\ud2f0\ubc8c\uc5d0\uc11c \u3008Sweet Little Sixteen\u3009\uc744 \uacf5\uc5f0\ud558\ub294 \ubaa8\uc2b5\uc774 \ucd2c\uc601\ub418\uc5b4 \uc601\ud654 \u300a\uc7ac\uc988 \uc628 \uc5b4 \uc11c\uba38\uc988 \ub370\uc774\u300b\ub85c \uc81c\uc791\ub418\uc5c8\ub2e4.", "answer": "\uc7ac\uc988 \uc628 \uc5b4 \uc11c\uba38\uc988 \ub370\uc774", "sentence": "1958\ub144 \ub274\ud3ec\ud2b8 \uc7ac\uc988 \ud398\uc2a4\ud2f0\ubc8c\uc5d0\uc11c \u3008Sweet Little Sixteen\u3009\uc744 \uacf5\uc5f0\ud558\ub294 \ubaa8\uc2b5\uc774 \ucd2c\uc601\ub418\uc5b4 \uc601\ud654 \u300a \uc7ac\uc988 \uc628 \uc5b4 \uc11c\uba38\uc988 \ub370\uc774 \u300b\ub85c \uc81c\uc791\ub418\uc5c8\ub2e4.", "paragraph_sentence": "1957\ub144 \ub9d0 \ubca0\ub9ac\ub294 \uc568\ub7f0 \ud504\ub9ac\ub4dc\uac00 \uae30\ud68d\ud55c \"Biggest Show of Stars for 1957\"\uc758 \ud55c \uc790\ub9ac\ub97c \ucc28\uc9c0\ud558\uac8c \ub41c\ub2e4. \uc774 \ud22c\uc5b4\uc5d0\ub294 \uc5d0\ubc8c\ub9ac \ube0c\ub77c\ub354\uc2a4\uc640 \ubc84\ub514 \ud640\ub9ac \ub4f1\uc774 \ub3d9\ud589\ud588\ub2e4. ABC\uc758 \u300a\uac00\uc774 \ubbf8\uccbc \uc1fc\u300b\uc5d0\uc11c\ub3c4 \uac8c\uc2a4\ud2b8\ub85c \ub4f1\uc7a5, \ud788\ud2b8\uace1 \u3008Rock 'n' Roll Music\u3009\uc744 \ubd88\ub800\ub2e4. \uadf8\uc758 \ud788\ud2b8\ub294 1957\ub144\uc5d0\uc11c 1959\ub144\uae4c\uc9c0 \uacc4\uc18d\ub418\uc5c8\uace0, \uc774 \uc2dc\uae30 24\uac1c\uac00 \ub118\ub294 \uace1\uc744 \ucc28\ud2b8\uc5d0 \uc9c4\uc785\uc2dc\ud0a4\ub294\ub370 \uc131\uacf5\ud55c\ub2e4. \ubbf8\uad6d \ucc28\ud2b8 10\uc704\uad8c\uc5d0\ub294 \u3008School Days\u3009, \u3008Rock and Roll Music\u3009, \u3008Sweet Little Sixteen\u3009, \u3008Johnny B. Goode\u3009 \ub4f1\uc774 \uc62c\ub790\ub2e4. \uc774\ubc16\uc5d0\ub3c4 \ub450 \ud3b8\uc758 \ucd08\uae30 \ub85c\ud070\ub864 \uc601\ud654\uc5d0 \ucd9c\uc5f0\ud55c \uadf8\ub294 \u300a\ub85d, \ub85d, \ub85d\u300b\uc5d0\uc11c \u3008You Can't Catch Me\u3009\ub97c \uacf5\uc5f0\ud588\uace0, \u300a\uace0, \uc790\ub2c8, \uace0!\u300b\uc5d0\uc11c \ubcf8\uc778\uc73c\ub85c \ucd9c\uc5f0\ud558\uc5ec \u3008Johnny B. Goode\u3009, \u3008Memphis, Tennessee\u3009, \u3008Little Queenie\u3009\ub97c \ubd88\ub800\ub2e4. 1958\ub144 \ub274\ud3ec\ud2b8 \uc7ac\uc988 \ud398\uc2a4\ud2f0\ubc8c\uc5d0\uc11c \u3008Sweet Little Sixteen\u3009\uc744 \uacf5\uc5f0\ud558\ub294 \ubaa8\uc2b5\uc774 \ucd2c\uc601\ub418\uc5b4 \uc601\ud654 \u300a \uc7ac\uc988 \uc628 \uc5b4 \uc11c\uba38\uc988 \ub370\uc774 \u300b\ub85c \uc81c\uc791\ub418\uc5c8\ub2e4. ", "paragraph_answer": "1957\ub144 \ub9d0 \ubca0\ub9ac\ub294 \uc568\ub7f0 \ud504\ub9ac\ub4dc\uac00 \uae30\ud68d\ud55c \"Biggest Show of Stars for 1957\"\uc758 \ud55c \uc790\ub9ac\ub97c \ucc28\uc9c0\ud558\uac8c \ub41c\ub2e4. \uc774 \ud22c\uc5b4\uc5d0\ub294 \uc5d0\ubc8c\ub9ac \ube0c\ub77c\ub354\uc2a4\uc640 \ubc84\ub514 \ud640\ub9ac \ub4f1\uc774 \ub3d9\ud589\ud588\ub2e4. ABC\uc758 \u300a\uac00\uc774 \ubbf8\uccbc \uc1fc\u300b\uc5d0\uc11c\ub3c4 \uac8c\uc2a4\ud2b8\ub85c \ub4f1\uc7a5, \ud788\ud2b8\uace1 \u3008Rock 'n' Roll Music\u3009\uc744 \ubd88\ub800\ub2e4. \uadf8\uc758 \ud788\ud2b8\ub294 1957\ub144\uc5d0\uc11c 1959\ub144\uae4c\uc9c0 \uacc4\uc18d\ub418\uc5c8\uace0, \uc774 \uc2dc\uae30 24\uac1c\uac00 \ub118\ub294 \uace1\uc744 \ucc28\ud2b8\uc5d0 \uc9c4\uc785\uc2dc\ud0a4\ub294\ub370 \uc131\uacf5\ud55c\ub2e4. \ubbf8\uad6d \ucc28\ud2b8 10\uc704\uad8c\uc5d0\ub294 \u3008School Days\u3009, \u3008Rock and Roll Music\u3009, \u3008Sweet Little Sixteen\u3009, \u3008Johnny B. Goode\u3009 \ub4f1\uc774 \uc62c\ub790\ub2e4. \uc774\ubc16\uc5d0\ub3c4 \ub450 \ud3b8\uc758 \ucd08\uae30 \ub85c\ud070\ub864 \uc601\ud654\uc5d0 \ucd9c\uc5f0\ud55c \uadf8\ub294 \u300a\ub85d, \ub85d, \ub85d\u300b\uc5d0\uc11c \u3008You Can't Catch Me\u3009\ub97c \uacf5\uc5f0\ud588\uace0, \u300a\uace0, \uc790\ub2c8, \uace0!\u300b\uc5d0\uc11c \ubcf8\uc778\uc73c\ub85c \ucd9c\uc5f0\ud558\uc5ec \u3008Johnny B. Goode\u3009, \u3008Memphis, Tennessee\u3009, \u3008Little Queenie\u3009\ub97c \ubd88\ub800\ub2e4. 1958\ub144 \ub274\ud3ec\ud2b8 \uc7ac\uc988 \ud398\uc2a4\ud2f0\ubc8c\uc5d0\uc11c \u3008Sweet Little Sixteen\u3009\uc744 \uacf5\uc5f0\ud558\ub294 \ubaa8\uc2b5\uc774 \ucd2c\uc601\ub418\uc5b4 \uc601\ud654 \u300a \uc7ac\uc988 \uc628 \uc5b4 \uc11c\uba38\uc988 \ub370\uc774 \u300b\ub85c \uc81c\uc791\ub418\uc5c8\ub2e4.", "sentence_answer": "1958\ub144 \ub274\ud3ec\ud2b8 \uc7ac\uc988 \ud398\uc2a4\ud2f0\ubc8c\uc5d0\uc11c \u3008Sweet Little Sixteen\u3009\uc744 \uacf5\uc5f0\ud558\ub294 \ubaa8\uc2b5\uc774 \ucd2c\uc601\ub418\uc5b4 \uc601\ud654 \u300a \uc7ac\uc988 \uc628 \uc5b4 \uc11c\uba38\uc988 \ub370\uc774 \u300b\ub85c \uc81c\uc791\ub418\uc5c8\ub2e4.", "paragraph_id": "6596458-7-2", "paragraph_question": "question: 1958\ub144 \ub274\ud3ec\ud2b8 \uc7ac\uc988 \ud398\uc2a4\ud2f0\ubc8c\uc5d0\uc11c \uacf5\uc5f0\ud558\ub294 \ubaa8\uc2b5\uc774 \ucd2c\uc601\ub418\uc5b4 \uc81c\uc791\ub41c \uc601\ud654\ub294 \ubb34\uc5c7\uc778\uac00?, context: 1957\ub144 \ub9d0 \ubca0\ub9ac\ub294 \uc568\ub7f0 \ud504\ub9ac\ub4dc\uac00 \uae30\ud68d\ud55c \"Biggest Show of Stars for 1957\"\uc758 \ud55c \uc790\ub9ac\ub97c \ucc28\uc9c0\ud558\uac8c \ub41c\ub2e4. \uc774 \ud22c\uc5b4\uc5d0\ub294 \uc5d0\ubc8c\ub9ac \ube0c\ub77c\ub354\uc2a4\uc640 \ubc84\ub514 \ud640\ub9ac \ub4f1\uc774 \ub3d9\ud589\ud588\ub2e4. ABC\uc758 \u300a\uac00\uc774 \ubbf8\uccbc \uc1fc\u300b\uc5d0\uc11c\ub3c4 \uac8c\uc2a4\ud2b8\ub85c \ub4f1\uc7a5, \ud788\ud2b8\uace1 \u3008Rock 'n' Roll Music\u3009\uc744 \ubd88\ub800\ub2e4. \uadf8\uc758 \ud788\ud2b8\ub294 1957\ub144\uc5d0\uc11c 1959\ub144\uae4c\uc9c0 \uacc4\uc18d\ub418\uc5c8\uace0, \uc774 \uc2dc\uae30 24\uac1c\uac00 \ub118\ub294 \uace1\uc744 \ucc28\ud2b8\uc5d0 \uc9c4\uc785\uc2dc\ud0a4\ub294\ub370 \uc131\uacf5\ud55c\ub2e4. \ubbf8\uad6d \ucc28\ud2b8 10\uc704\uad8c\uc5d0\ub294 \u3008School Days\u3009, \u3008Rock and Roll Music\u3009, \u3008Sweet Little Sixteen\u3009, \u3008Johnny B. Goode\u3009 \ub4f1\uc774 \uc62c\ub790\ub2e4. \uc774\ubc16\uc5d0\ub3c4 \ub450 \ud3b8\uc758 \ucd08\uae30 \ub85c\ud070\ub864 \uc601\ud654\uc5d0 \ucd9c\uc5f0\ud55c \uadf8\ub294 \u300a\ub85d, \ub85d, \ub85d\u300b\uc5d0\uc11c \u3008You Can't Catch Me\u3009\ub97c \uacf5\uc5f0\ud588\uace0, \u300a\uace0, \uc790\ub2c8, \uace0!\u300b\uc5d0\uc11c \ubcf8\uc778\uc73c\ub85c \ucd9c\uc5f0\ud558\uc5ec \u3008Johnny B. Goode\u3009, \u3008Memphis, Tennessee\u3009, \u3008Little Queenie\u3009\ub97c \ubd88\ub800\ub2e4. 1958\ub144 \ub274\ud3ec\ud2b8 \uc7ac\uc988 \ud398\uc2a4\ud2f0\ubc8c\uc5d0\uc11c \u3008Sweet Little Sixteen\u3009\uc744 \uacf5\uc5f0\ud558\ub294 \ubaa8\uc2b5\uc774 \ucd2c\uc601\ub418\uc5b4 \uc601\ud654 \u300a\uc7ac\uc988 \uc628 \uc5b4 \uc11c\uba38\uc988 \ub370\uc774\u300b\ub85c \uc81c\uc791\ub418\uc5c8\ub2e4."} {"question": "\uc74c\uc545\uc0ac\ud559\uc758 \ubaa8\ub4e0 \uc601\uc5ed\uc5d0 \ub300\ud55c \uae30\ub85d\uc774 \uce58\ubc00\ud558\uace0 \ubc29\ub300\ud558\uac8c \ub0a8\uc544\uc788\ub294 \uc5ed\uc0ac\uc11c\ub294?", "paragraph": "\ub3d9\uc591\uc5d0\uc11c \uc74c\uc545\uc5f0\uad6c\uc758 \uc5ed\uc0ac\ub294 \uc11c\uc591\uc74c\uc545\uc0ac\ud559\uacfc \ub9c8\ucc2c\uac00\uc9c0\ub85c \ub9e4\uc6b0 \uc7a5\uad6c\ud55c\ub370, \uc911\uad6d \uc8fc\ub098\ub77c \ub54c \uccad\uc18c\ub144\uc758 \ud559\uc2b5\uacfc\ubaa9\uc778 \uc721\uc608(\u516d\u79ae) \uac00\uc6b4\ub370 \uc545(\u6a02)\uc774 \ub4e4\uc5b4\uc788\uc5c8\uace0, \uc720\uac00(\u5112\u5bb6)\uc758 \uacbd\uc804\uc778 \uc721\uacbd \uac00\uc6b4\ub370\uc5d0\ub3c4 \uc545\uacbd(\u6a02\u7d93)\uc774 \ub4e4\uc5b4\uc788\uc5c8\ub2e4. \ub3d9\uc591\uc5d0\uc11c\uc758 \uc74c\uc545\ub3c4 \uc11c\uc591\uacfc \ub9c8\ucc2c\uac00\uc9c0\ub85c \uc778\uac04 \uc0dd\ud65c\uc758 \ud544\uc218 \uc694\uc18c \uc911 \ud558\ub098\uc600\uae30 \ub54c\ubb38\uc5d0 \uc815\uc2e0\uc218\uc591, \uc608\uc220, \uc758\ub840, \uc624\ub77d\uc758 \uae30\ub2a5\uc744 \ub118\uc5b4\uc11c \uad6d\uac00\uac04 \uc678\uad50\uc758 \ud604\uc7a5\uc5d0\uc11c \uc545\ub300\uc758 \ubc18\uc8fc\ub85c \uc2dc(\u8a69)\ub97c \ub178\ub798\ud558\uc5ec \uc11c\ub85c \uc758\uc0ac\ub97c \ud0c0\uc9c4\ud558\uc600\ub2e4\ub294 \uae30\ub85d\uc774 \uc788\uc744 \ub9cc\ud07c \uc74c\uc545\uc758 \uc704\uce58\ub294 \ub3d9\uc591\uc5d0\uc11c \ud2b9\ubcc4\ud55c \uac83\uc774\uc5c8\ub2e4. \uc608\uae30(\u79ae\u8a18) \uc911 \uc545\uae30\u6a02\u8a18\ub294 \uc774\ub7ec\ud55c \uc911\uad6d \uace0\ub300\uc0ac\uc5d0\uc11c \uc74c\uc545\uc774 \uc608\uc220\uc758 \ubc94\uc704\ub97c \ub118\uc5b4 \ud1b5\uce58\uc640 \uad50\uc721, \uc5f0\uad6c\uc758 \uc911\uc694\ud55c \uc218\ub2e8\uc774\uc790 \uadf8 \uad6c\ud604\uccb4\ub85c \uae30\uc220\ub41c \uc810\uc774 \uc774\ub97c \ub4b7\ubc1b\uce68\ud55c\ub2e4. \uc774\ub7ec\ud55c \uc804\ud1b5 \uc544\ub798 \ub3d9\uc591\uc758 \uc5ed\uc0ac\uac00\ub4e4\uc740 \uc74c\uc545\uc0ac\ud559\uc744 \ub2e8\ub3c5\ud559\ubb38\uc73c\ub85c \ub3c5\ub9bd\uc2dc\ucf1c \ub530\ub85c \uc11c\uc220\ud558\uae30 \uc2dc\uc791\ud558\uc600\ub2e4. \uc774\uc2ed\uc0ac\uc0ac\uc5d0\uc11c\ub294 '\uc545\uc9c0\u6a02\u5fd7' \ub4f1\uc758 \uc774\ub984\uc73c\ub85c \uc74c\uc545\uc758 \uc0ac\uc0c1, \uc774\ub150, \uc815\ucc45, \uae30\uad00, \uc774\ub860, \uc545\uae30, \uc791\ud488, \uc74c\uc545\uac00 \ub4f1 \uc74c\uc545\uc0ac\ud559 \uc804 \uc601\uc5ed\uc5d0 \uac78\uccd0 \uce58\ubc00\ud558\uace0 \ubc29\ub300\ud55c \uae30\ub85d\uc774 \ub0a8\uc544\uc788\ub2e4.", "answer": "\uc774\uc2ed\uc0ac\uc0ac", "sentence": "\uc774\uc2ed\uc0ac\uc0ac \uc5d0\uc11c\ub294 '\uc545\uc9c0\u6a02\u5fd7' \ub4f1\uc758 \uc774\ub984\uc73c\ub85c \uc74c\uc545\uc758 \uc0ac\uc0c1, \uc774\ub150, \uc815\ucc45, \uae30\uad00, \uc774\ub860, \uc545\uae30, \uc791\ud488, \uc74c\uc545\uac00 \ub4f1 \uc74c\uc545\uc0ac\ud559 \uc804 \uc601\uc5ed\uc5d0 \uac78\uccd0 \uce58\ubc00\ud558\uace0 \ubc29\ub300\ud55c \uae30\ub85d\uc774 \ub0a8\uc544\uc788\ub2e4.", "paragraph_sentence": "\ub3d9\uc591\uc5d0\uc11c \uc74c\uc545\uc5f0\uad6c\uc758 \uc5ed\uc0ac\ub294 \uc11c\uc591\uc74c\uc545\uc0ac\ud559\uacfc \ub9c8\ucc2c\uac00\uc9c0\ub85c \ub9e4\uc6b0 \uc7a5\uad6c\ud55c\ub370, \uc911\uad6d \uc8fc\ub098\ub77c \ub54c \uccad\uc18c\ub144\uc758 \ud559\uc2b5\uacfc\ubaa9\uc778 \uc721\uc608(\u516d\u79ae) \uac00\uc6b4\ub370 \uc545(\u6a02)\uc774 \ub4e4\uc5b4\uc788\uc5c8\uace0, \uc720\uac00(\u5112\u5bb6)\uc758 \uacbd\uc804\uc778 \uc721\uacbd \uac00\uc6b4\ub370\uc5d0\ub3c4 \uc545\uacbd(\u6a02\u7d93)\uc774 \ub4e4\uc5b4\uc788\uc5c8\ub2e4. \ub3d9\uc591\uc5d0\uc11c\uc758 \uc74c\uc545\ub3c4 \uc11c\uc591\uacfc \ub9c8\ucc2c\uac00\uc9c0\ub85c \uc778\uac04 \uc0dd\ud65c\uc758 \ud544\uc218 \uc694\uc18c \uc911 \ud558\ub098\uc600\uae30 \ub54c\ubb38\uc5d0 \uc815\uc2e0\uc218\uc591, \uc608\uc220, \uc758\ub840, \uc624\ub77d\uc758 \uae30\ub2a5\uc744 \ub118\uc5b4\uc11c \uad6d\uac00\uac04 \uc678\uad50\uc758 \ud604\uc7a5\uc5d0\uc11c \uc545\ub300\uc758 \ubc18\uc8fc\ub85c \uc2dc(\u8a69)\ub97c \ub178\ub798\ud558\uc5ec \uc11c\ub85c \uc758\uc0ac\ub97c \ud0c0\uc9c4\ud558\uc600\ub2e4\ub294 \uae30\ub85d\uc774 \uc788\uc744 \ub9cc\ud07c \uc74c\uc545\uc758 \uc704\uce58\ub294 \ub3d9\uc591\uc5d0\uc11c \ud2b9\ubcc4\ud55c \uac83\uc774\uc5c8\ub2e4. \uc608\uae30(\u79ae\u8a18) \uc911 \uc545\uae30\u6a02\u8a18\ub294 \uc774\ub7ec\ud55c \uc911\uad6d \uace0\ub300\uc0ac\uc5d0\uc11c \uc74c\uc545\uc774 \uc608\uc220\uc758 \ubc94\uc704\ub97c \ub118\uc5b4 \ud1b5\uce58\uc640 \uad50\uc721, \uc5f0\uad6c\uc758 \uc911\uc694\ud55c \uc218\ub2e8\uc774\uc790 \uadf8 \uad6c\ud604\uccb4\ub85c \uae30\uc220\ub41c \uc810\uc774 \uc774\ub97c \ub4b7\ubc1b\uce68\ud55c\ub2e4. \uc774\ub7ec\ud55c \uc804\ud1b5 \uc544\ub798 \ub3d9\uc591\uc758 \uc5ed\uc0ac\uac00\ub4e4\uc740 \uc74c\uc545\uc0ac\ud559\uc744 \ub2e8\ub3c5\ud559\ubb38\uc73c\ub85c \ub3c5\ub9bd\uc2dc\ucf1c \ub530\ub85c \uc11c\uc220\ud558\uae30 \uc2dc\uc791\ud558\uc600\ub2e4. \uc774\uc2ed\uc0ac\uc0ac \uc5d0\uc11c\ub294 '\uc545\uc9c0\u6a02\u5fd7' \ub4f1\uc758 \uc774\ub984\uc73c\ub85c \uc74c\uc545\uc758 \uc0ac\uc0c1, \uc774\ub150, \uc815\ucc45, \uae30\uad00, \uc774\ub860, \uc545\uae30, \uc791\ud488, \uc74c\uc545\uac00 \ub4f1 \uc74c\uc545\uc0ac\ud559 \uc804 \uc601\uc5ed\uc5d0 \uac78\uccd0 \uce58\ubc00\ud558\uace0 \ubc29\ub300\ud55c \uae30\ub85d\uc774 \ub0a8\uc544\uc788\ub2e4. ", "paragraph_answer": "\ub3d9\uc591\uc5d0\uc11c \uc74c\uc545\uc5f0\uad6c\uc758 \uc5ed\uc0ac\ub294 \uc11c\uc591\uc74c\uc545\uc0ac\ud559\uacfc \ub9c8\ucc2c\uac00\uc9c0\ub85c \ub9e4\uc6b0 \uc7a5\uad6c\ud55c\ub370, \uc911\uad6d \uc8fc\ub098\ub77c \ub54c \uccad\uc18c\ub144\uc758 \ud559\uc2b5\uacfc\ubaa9\uc778 \uc721\uc608(\u516d\u79ae) \uac00\uc6b4\ub370 \uc545(\u6a02)\uc774 \ub4e4\uc5b4\uc788\uc5c8\uace0, \uc720\uac00(\u5112\u5bb6)\uc758 \uacbd\uc804\uc778 \uc721\uacbd \uac00\uc6b4\ub370\uc5d0\ub3c4 \uc545\uacbd(\u6a02\u7d93)\uc774 \ub4e4\uc5b4\uc788\uc5c8\ub2e4. \ub3d9\uc591\uc5d0\uc11c\uc758 \uc74c\uc545\ub3c4 \uc11c\uc591\uacfc \ub9c8\ucc2c\uac00\uc9c0\ub85c \uc778\uac04 \uc0dd\ud65c\uc758 \ud544\uc218 \uc694\uc18c \uc911 \ud558\ub098\uc600\uae30 \ub54c\ubb38\uc5d0 \uc815\uc2e0\uc218\uc591, \uc608\uc220, \uc758\ub840, \uc624\ub77d\uc758 \uae30\ub2a5\uc744 \ub118\uc5b4\uc11c \uad6d\uac00\uac04 \uc678\uad50\uc758 \ud604\uc7a5\uc5d0\uc11c \uc545\ub300\uc758 \ubc18\uc8fc\ub85c \uc2dc(\u8a69)\ub97c \ub178\ub798\ud558\uc5ec \uc11c\ub85c \uc758\uc0ac\ub97c \ud0c0\uc9c4\ud558\uc600\ub2e4\ub294 \uae30\ub85d\uc774 \uc788\uc744 \ub9cc\ud07c \uc74c\uc545\uc758 \uc704\uce58\ub294 \ub3d9\uc591\uc5d0\uc11c \ud2b9\ubcc4\ud55c \uac83\uc774\uc5c8\ub2e4. \uc608\uae30(\u79ae\u8a18) \uc911 \uc545\uae30\u6a02\u8a18\ub294 \uc774\ub7ec\ud55c \uc911\uad6d \uace0\ub300\uc0ac\uc5d0\uc11c \uc74c\uc545\uc774 \uc608\uc220\uc758 \ubc94\uc704\ub97c \ub118\uc5b4 \ud1b5\uce58\uc640 \uad50\uc721, \uc5f0\uad6c\uc758 \uc911\uc694\ud55c \uc218\ub2e8\uc774\uc790 \uadf8 \uad6c\ud604\uccb4\ub85c \uae30\uc220\ub41c \uc810\uc774 \uc774\ub97c \ub4b7\ubc1b\uce68\ud55c\ub2e4. \uc774\ub7ec\ud55c \uc804\ud1b5 \uc544\ub798 \ub3d9\uc591\uc758 \uc5ed\uc0ac\uac00\ub4e4\uc740 \uc74c\uc545\uc0ac\ud559\uc744 \ub2e8\ub3c5\ud559\ubb38\uc73c\ub85c \ub3c5\ub9bd\uc2dc\ucf1c \ub530\ub85c \uc11c\uc220\ud558\uae30 \uc2dc\uc791\ud558\uc600\ub2e4. \uc774\uc2ed\uc0ac\uc0ac \uc5d0\uc11c\ub294 '\uc545\uc9c0\u6a02\u5fd7' \ub4f1\uc758 \uc774\ub984\uc73c\ub85c \uc74c\uc545\uc758 \uc0ac\uc0c1, \uc774\ub150, \uc815\ucc45, \uae30\uad00, \uc774\ub860, \uc545\uae30, \uc791\ud488, \uc74c\uc545\uac00 \ub4f1 \uc74c\uc545\uc0ac\ud559 \uc804 \uc601\uc5ed\uc5d0 \uac78\uccd0 \uce58\ubc00\ud558\uace0 \ubc29\ub300\ud55c \uae30\ub85d\uc774 \ub0a8\uc544\uc788\ub2e4.", "sentence_answer": " \uc774\uc2ed\uc0ac\uc0ac \uc5d0\uc11c\ub294 '\uc545\uc9c0\u6a02\u5fd7' \ub4f1\uc758 \uc774\ub984\uc73c\ub85c \uc74c\uc545\uc758 \uc0ac\uc0c1, \uc774\ub150, \uc815\ucc45, \uae30\uad00, \uc774\ub860, \uc545\uae30, \uc791\ud488, \uc74c\uc545\uac00 \ub4f1 \uc74c\uc545\uc0ac\ud559 \uc804 \uc601\uc5ed\uc5d0 \uac78\uccd0 \uce58\ubc00\ud558\uace0 \ubc29\ub300\ud55c \uae30\ub85d\uc774 \ub0a8\uc544\uc788\ub2e4.", "paragraph_id": "6544214-2-1", "paragraph_question": "question: \uc74c\uc545\uc0ac\ud559\uc758 \ubaa8\ub4e0 \uc601\uc5ed\uc5d0 \ub300\ud55c \uae30\ub85d\uc774 \uce58\ubc00\ud558\uace0 \ubc29\ub300\ud558\uac8c \ub0a8\uc544\uc788\ub294 \uc5ed\uc0ac\uc11c\ub294?, context: \ub3d9\uc591\uc5d0\uc11c \uc74c\uc545\uc5f0\uad6c\uc758 \uc5ed\uc0ac\ub294 \uc11c\uc591\uc74c\uc545\uc0ac\ud559\uacfc \ub9c8\ucc2c\uac00\uc9c0\ub85c \ub9e4\uc6b0 \uc7a5\uad6c\ud55c\ub370, \uc911\uad6d \uc8fc\ub098\ub77c \ub54c \uccad\uc18c\ub144\uc758 \ud559\uc2b5\uacfc\ubaa9\uc778 \uc721\uc608(\u516d\u79ae) \uac00\uc6b4\ub370 \uc545(\u6a02)\uc774 \ub4e4\uc5b4\uc788\uc5c8\uace0, \uc720\uac00(\u5112\u5bb6)\uc758 \uacbd\uc804\uc778 \uc721\uacbd \uac00\uc6b4\ub370\uc5d0\ub3c4 \uc545\uacbd(\u6a02\u7d93)\uc774 \ub4e4\uc5b4\uc788\uc5c8\ub2e4. \ub3d9\uc591\uc5d0\uc11c\uc758 \uc74c\uc545\ub3c4 \uc11c\uc591\uacfc \ub9c8\ucc2c\uac00\uc9c0\ub85c \uc778\uac04 \uc0dd\ud65c\uc758 \ud544\uc218 \uc694\uc18c \uc911 \ud558\ub098\uc600\uae30 \ub54c\ubb38\uc5d0 \uc815\uc2e0\uc218\uc591, \uc608\uc220, \uc758\ub840, \uc624\ub77d\uc758 \uae30\ub2a5\uc744 \ub118\uc5b4\uc11c \uad6d\uac00\uac04 \uc678\uad50\uc758 \ud604\uc7a5\uc5d0\uc11c \uc545\ub300\uc758 \ubc18\uc8fc\ub85c \uc2dc(\u8a69)\ub97c \ub178\ub798\ud558\uc5ec \uc11c\ub85c \uc758\uc0ac\ub97c \ud0c0\uc9c4\ud558\uc600\ub2e4\ub294 \uae30\ub85d\uc774 \uc788\uc744 \ub9cc\ud07c \uc74c\uc545\uc758 \uc704\uce58\ub294 \ub3d9\uc591\uc5d0\uc11c \ud2b9\ubcc4\ud55c \uac83\uc774\uc5c8\ub2e4. \uc608\uae30(\u79ae\u8a18) \uc911 \uc545\uae30\u6a02\u8a18\ub294 \uc774\ub7ec\ud55c \uc911\uad6d \uace0\ub300\uc0ac\uc5d0\uc11c \uc74c\uc545\uc774 \uc608\uc220\uc758 \ubc94\uc704\ub97c \ub118\uc5b4 \ud1b5\uce58\uc640 \uad50\uc721, \uc5f0\uad6c\uc758 \uc911\uc694\ud55c \uc218\ub2e8\uc774\uc790 \uadf8 \uad6c\ud604\uccb4\ub85c \uae30\uc220\ub41c \uc810\uc774 \uc774\ub97c \ub4b7\ubc1b\uce68\ud55c\ub2e4. \uc774\ub7ec\ud55c \uc804\ud1b5 \uc544\ub798 \ub3d9\uc591\uc758 \uc5ed\uc0ac\uac00\ub4e4\uc740 \uc74c\uc545\uc0ac\ud559\uc744 \ub2e8\ub3c5\ud559\ubb38\uc73c\ub85c \ub3c5\ub9bd\uc2dc\ucf1c \ub530\ub85c \uc11c\uc220\ud558\uae30 \uc2dc\uc791\ud558\uc600\ub2e4. \uc774\uc2ed\uc0ac\uc0ac\uc5d0\uc11c\ub294 '\uc545\uc9c0\u6a02\u5fd7' \ub4f1\uc758 \uc774\ub984\uc73c\ub85c \uc74c\uc545\uc758 \uc0ac\uc0c1, \uc774\ub150, \uc815\ucc45, \uae30\uad00, \uc774\ub860, \uc545\uae30, \uc791\ud488, \uc74c\uc545\uac00 \ub4f1 \uc74c\uc545\uc0ac\ud559 \uc804 \uc601\uc5ed\uc5d0 \uac78\uccd0 \uce58\ubc00\ud558\uace0 \ubc29\ub300\ud55c \uae30\ub85d\uc774 \ub0a8\uc544\uc788\ub2e4."} {"question": "\uc790\uc720\ud615\uacfc \uadf8\ub808\ucf54\ub85c\ub9cc\ud615\uc758 \ub450 \uc885\ubaa9\uc73c\ub85c \ub098\ub258\ub294 \uac83\uc740 \ubb34\uc5c7\uc778\uac00?", "paragraph": "\uc62c\ub9bc\ud53d \uc885\ubaa9\uc740 \ucd1d 33\uac1c\ubd80\ubb38 52\uac1c \uc885\ubaa9\uc5d0\uc11c \uc57d 400\uac1c\uc758 \uacbd\uae30\ub85c \uc774\ub8e8\uc5b4\uc838\uc788\ub2e4. \uc608\ub97c \ub4e4\uc5b4\uc11c \ud558\uacc4 \uc62c\ub9bc\ud53d \ubd80\ubb38\uc778 \ub808\uc2ac\ub9c1\uc740 \uc790\uc720\ud615\uacfc \uadf8\ub808\ucf54\ub85c\ub9cc\ud615\uc758 \ub450 \uc885\ubaa9\uc73c\ub85c \ub098\ub25c\ub2e4. \uc5ec\uae30\uc5d0\uc11c 10\uacbd\uae30\ub294 \ub0a8\uc790\ubd80, 4\uacbd\uae30\ub294 \uc5ec\uc790\ubd80\ub85c \uc5f4\ub9ac\uba70 \ubd84\ub958\uae30\uc900\uc740 \uccb4\uc911\uc774\ub2e4. \ud558\uacc4 \uc62c\ub9bc\ud53d\uc740 26\uac1c, \ub3d9\uacc4 \uc62c\ub9bc\ud53d\uc740 7\uac1c \ubd80\ubb38\uc73c\ub85c \uc774\ub8e8\uc5b4\uc838\uc788\ub2e4. \ud558\uacc4 \uc62c\ub9bc\ud53d\uc5d0\uc11c\ub294 \uc721\uc0c1, \uc218\uc601, \ud39c\uc2f1, \uccb4\uc870\uac00 1\ud68c \ub300\ud68c\ub54c\ubd80\ud130 \ud55c\ubc88\ub3c4 \ube60\uc9d0\uc5c6\uc774 \uc815\uc2dd\uc885\ubaa9\uc774\uc5c8\uc73c\uba70, \ub3d9\uacc4 \uc62c\ub9bc\ud53d\uc5d0\uc11c\ub294 \ud06c\ub85c\uc2a4\ucee8\ud2b8\ub9ac, \ud53c\uaca8 \uc2a4\ucf00\uc774\ud305, \uc544\uc774\uc2a4 \ud558\ud0a4, \ub178\ub974\ub515 \ubcf5\ud569, \uc2a4\ud0a4 \uc810\ud504, \uc2a4\ud53c\ub4dc \uc2a4\ucf00\uc774\ud305\uc774 1924\ub144 \ub3d9\uacc4 \uc62c\ub9bc\ud53d\ubd80\ud130 \ube60\uc9d0\uc5c6\uc774 \uc815\uc2dd\uc885\ubaa9\uc774\uc5c8\ub2e4. \ubc30\ub4dc\ubbfc\ud134, \ub18d\uad6c, \ubc30\uad6c\uc640 \uac19\uc740 \uc815\uc2dd\uc885\ubaa9\ub4e4\uc740 \ucc98\uc74c\uc5d0\ub294 \uc2dc\ubc94\uc885\ubaa9\uc774\uc5c8\uc73c\uba70 \uadf8 \ud6c4\uc5d0 \uc815\uc2dd\uc885\ubaa9\uc73c\ub85c \uc2b9\uc778 \ub418\uc5c8\ub2e4. \uc57c\uad6c\ucc98\ub7fc \uc608\uc804\uc5d0\ub294 \uc815\uc2dd\uc885\ubaa9 \uc774\uc5c8\uc9c0\ub9cc \uc9c0\uae08\uc740 \uc815\uc2dd \uc885\ubaa9\uc5d0\uc11c \ube60\uc9c4 \uc885\ubaa9\ub3c4 \uc788\ub2e4.", "answer": "\ub808\uc2ac\ub9c1", "sentence": "\uc608\ub97c \ub4e4\uc5b4\uc11c \ud558\uacc4 \uc62c\ub9bc\ud53d \ubd80\ubb38\uc778 \ub808\uc2ac\ub9c1 \uc740 \uc790\uc720\ud615\uacfc \uadf8\ub808\ucf54\ub85c\ub9cc\ud615\uc758 \ub450 \uc885\ubaa9\uc73c\ub85c \ub098\ub25c\ub2e4.", "paragraph_sentence": "\uc62c\ub9bc\ud53d \uc885\ubaa9\uc740 \ucd1d 33\uac1c\ubd80\ubb38 52\uac1c \uc885\ubaa9\uc5d0\uc11c \uc57d 400\uac1c\uc758 \uacbd\uae30\ub85c \uc774\ub8e8\uc5b4\uc838\uc788\ub2e4. \uc608\ub97c \ub4e4\uc5b4\uc11c \ud558\uacc4 \uc62c\ub9bc\ud53d \ubd80\ubb38\uc778 \ub808\uc2ac\ub9c1 \uc740 \uc790\uc720\ud615\uacfc \uadf8\ub808\ucf54\ub85c\ub9cc\ud615\uc758 \ub450 \uc885\ubaa9\uc73c\ub85c \ub098\ub25c\ub2e4. \uc5ec\uae30\uc5d0\uc11c 10\uacbd\uae30\ub294 \ub0a8\uc790\ubd80, 4\uacbd\uae30\ub294 \uc5ec\uc790\ubd80\ub85c \uc5f4\ub9ac\uba70 \ubd84\ub958\uae30\uc900\uc740 \uccb4\uc911\uc774\ub2e4. \ud558\uacc4 \uc62c\ub9bc\ud53d\uc740 26\uac1c, \ub3d9\uacc4 \uc62c\ub9bc\ud53d\uc740 7\uac1c \ubd80\ubb38\uc73c\ub85c \uc774\ub8e8\uc5b4\uc838\uc788\ub2e4. \ud558\uacc4 \uc62c\ub9bc\ud53d\uc5d0\uc11c\ub294 \uc721\uc0c1, \uc218\uc601, \ud39c\uc2f1, \uccb4\uc870\uac00 1\ud68c \ub300\ud68c\ub54c\ubd80\ud130 \ud55c\ubc88\ub3c4 \ube60\uc9d0\uc5c6\uc774 \uc815\uc2dd\uc885\ubaa9\uc774\uc5c8\uc73c\uba70, \ub3d9\uacc4 \uc62c\ub9bc\ud53d\uc5d0\uc11c\ub294 \ud06c\ub85c\uc2a4\ucee8\ud2b8\ub9ac, \ud53c\uaca8 \uc2a4\ucf00\uc774\ud305, \uc544\uc774\uc2a4 \ud558\ud0a4, \ub178\ub974\ub515 \ubcf5\ud569, \uc2a4\ud0a4 \uc810\ud504, \uc2a4\ud53c\ub4dc \uc2a4\ucf00\uc774\ud305\uc774 1924\ub144 \ub3d9\uacc4 \uc62c\ub9bc\ud53d\ubd80\ud130 \ube60\uc9d0\uc5c6\uc774 \uc815\uc2dd\uc885\ubaa9\uc774\uc5c8\ub2e4. \ubc30\ub4dc\ubbfc\ud134, \ub18d\uad6c, \ubc30\uad6c\uc640 \uac19\uc740 \uc815\uc2dd\uc885\ubaa9\ub4e4\uc740 \ucc98\uc74c\uc5d0\ub294 \uc2dc\ubc94\uc885\ubaa9\uc774\uc5c8\uc73c\uba70 \uadf8 \ud6c4\uc5d0 \uc815\uc2dd\uc885\ubaa9\uc73c\ub85c \uc2b9\uc778 \ub418\uc5c8\ub2e4. \uc57c\uad6c\ucc98\ub7fc \uc608\uc804\uc5d0\ub294 \uc815\uc2dd\uc885\ubaa9 \uc774\uc5c8\uc9c0\ub9cc \uc9c0\uae08\uc740 \uc815\uc2dd \uc885\ubaa9\uc5d0\uc11c \ube60\uc9c4 \uc885\ubaa9\ub3c4 \uc788\ub2e4.", "paragraph_answer": "\uc62c\ub9bc\ud53d \uc885\ubaa9\uc740 \ucd1d 33\uac1c\ubd80\ubb38 52\uac1c \uc885\ubaa9\uc5d0\uc11c \uc57d 400\uac1c\uc758 \uacbd\uae30\ub85c \uc774\ub8e8\uc5b4\uc838\uc788\ub2e4. \uc608\ub97c \ub4e4\uc5b4\uc11c \ud558\uacc4 \uc62c\ub9bc\ud53d \ubd80\ubb38\uc778 \ub808\uc2ac\ub9c1 \uc740 \uc790\uc720\ud615\uacfc \uadf8\ub808\ucf54\ub85c\ub9cc\ud615\uc758 \ub450 \uc885\ubaa9\uc73c\ub85c \ub098\ub25c\ub2e4. \uc5ec\uae30\uc5d0\uc11c 10\uacbd\uae30\ub294 \ub0a8\uc790\ubd80, 4\uacbd\uae30\ub294 \uc5ec\uc790\ubd80\ub85c \uc5f4\ub9ac\uba70 \ubd84\ub958\uae30\uc900\uc740 \uccb4\uc911\uc774\ub2e4. \ud558\uacc4 \uc62c\ub9bc\ud53d\uc740 26\uac1c, \ub3d9\uacc4 \uc62c\ub9bc\ud53d\uc740 7\uac1c \ubd80\ubb38\uc73c\ub85c \uc774\ub8e8\uc5b4\uc838\uc788\ub2e4. \ud558\uacc4 \uc62c\ub9bc\ud53d\uc5d0\uc11c\ub294 \uc721\uc0c1, \uc218\uc601, \ud39c\uc2f1, \uccb4\uc870\uac00 1\ud68c \ub300\ud68c\ub54c\ubd80\ud130 \ud55c\ubc88\ub3c4 \ube60\uc9d0\uc5c6\uc774 \uc815\uc2dd\uc885\ubaa9\uc774\uc5c8\uc73c\uba70, \ub3d9\uacc4 \uc62c\ub9bc\ud53d\uc5d0\uc11c\ub294 \ud06c\ub85c\uc2a4\ucee8\ud2b8\ub9ac, \ud53c\uaca8 \uc2a4\ucf00\uc774\ud305, \uc544\uc774\uc2a4 \ud558\ud0a4, \ub178\ub974\ub515 \ubcf5\ud569, \uc2a4\ud0a4 \uc810\ud504, \uc2a4\ud53c\ub4dc \uc2a4\ucf00\uc774\ud305\uc774 1924\ub144 \ub3d9\uacc4 \uc62c\ub9bc\ud53d\ubd80\ud130 \ube60\uc9d0\uc5c6\uc774 \uc815\uc2dd\uc885\ubaa9\uc774\uc5c8\ub2e4. \ubc30\ub4dc\ubbfc\ud134, \ub18d\uad6c, \ubc30\uad6c\uc640 \uac19\uc740 \uc815\uc2dd\uc885\ubaa9\ub4e4\uc740 \ucc98\uc74c\uc5d0\ub294 \uc2dc\ubc94\uc885\ubaa9\uc774\uc5c8\uc73c\uba70 \uadf8 \ud6c4\uc5d0 \uc815\uc2dd\uc885\ubaa9\uc73c\ub85c \uc2b9\uc778 \ub418\uc5c8\ub2e4. \uc57c\uad6c\ucc98\ub7fc \uc608\uc804\uc5d0\ub294 \uc815\uc2dd\uc885\ubaa9 \uc774\uc5c8\uc9c0\ub9cc \uc9c0\uae08\uc740 \uc815\uc2dd \uc885\ubaa9\uc5d0\uc11c \ube60\uc9c4 \uc885\ubaa9\ub3c4 \uc788\ub2e4.", "sentence_answer": "\uc608\ub97c \ub4e4\uc5b4\uc11c \ud558\uacc4 \uc62c\ub9bc\ud53d \ubd80\ubb38\uc778 \ub808\uc2ac\ub9c1 \uc740 \uc790\uc720\ud615\uacfc \uadf8\ub808\ucf54\ub85c\ub9cc\ud615\uc758 \ub450 \uc885\ubaa9\uc73c\ub85c \ub098\ub25c\ub2e4.", "paragraph_id": "6588307-20-0", "paragraph_question": "question: \uc790\uc720\ud615\uacfc \uadf8\ub808\ucf54\ub85c\ub9cc\ud615\uc758 \ub450 \uc885\ubaa9\uc73c\ub85c \ub098\ub258\ub294 \uac83\uc740 \ubb34\uc5c7\uc778\uac00?, context: \uc62c\ub9bc\ud53d \uc885\ubaa9\uc740 \ucd1d 33\uac1c\ubd80\ubb38 52\uac1c \uc885\ubaa9\uc5d0\uc11c \uc57d 400\uac1c\uc758 \uacbd\uae30\ub85c \uc774\ub8e8\uc5b4\uc838\uc788\ub2e4. \uc608\ub97c \ub4e4\uc5b4\uc11c \ud558\uacc4 \uc62c\ub9bc\ud53d \ubd80\ubb38\uc778 \ub808\uc2ac\ub9c1\uc740 \uc790\uc720\ud615\uacfc \uadf8\ub808\ucf54\ub85c\ub9cc\ud615\uc758 \ub450 \uc885\ubaa9\uc73c\ub85c \ub098\ub25c\ub2e4. \uc5ec\uae30\uc5d0\uc11c 10\uacbd\uae30\ub294 \ub0a8\uc790\ubd80, 4\uacbd\uae30\ub294 \uc5ec\uc790\ubd80\ub85c \uc5f4\ub9ac\uba70 \ubd84\ub958\uae30\uc900\uc740 \uccb4\uc911\uc774\ub2e4. \ud558\uacc4 \uc62c\ub9bc\ud53d\uc740 26\uac1c, \ub3d9\uacc4 \uc62c\ub9bc\ud53d\uc740 7\uac1c \ubd80\ubb38\uc73c\ub85c \uc774\ub8e8\uc5b4\uc838\uc788\ub2e4. \ud558\uacc4 \uc62c\ub9bc\ud53d\uc5d0\uc11c\ub294 \uc721\uc0c1, \uc218\uc601, \ud39c\uc2f1, \uccb4\uc870\uac00 1\ud68c \ub300\ud68c\ub54c\ubd80\ud130 \ud55c\ubc88\ub3c4 \ube60\uc9d0\uc5c6\uc774 \uc815\uc2dd\uc885\ubaa9\uc774\uc5c8\uc73c\uba70, \ub3d9\uacc4 \uc62c\ub9bc\ud53d\uc5d0\uc11c\ub294 \ud06c\ub85c\uc2a4\ucee8\ud2b8\ub9ac, \ud53c\uaca8 \uc2a4\ucf00\uc774\ud305, \uc544\uc774\uc2a4 \ud558\ud0a4, \ub178\ub974\ub515 \ubcf5\ud569, \uc2a4\ud0a4 \uc810\ud504, \uc2a4\ud53c\ub4dc \uc2a4\ucf00\uc774\ud305\uc774 1924\ub144 \ub3d9\uacc4 \uc62c\ub9bc\ud53d\ubd80\ud130 \ube60\uc9d0\uc5c6\uc774 \uc815\uc2dd\uc885\ubaa9\uc774\uc5c8\ub2e4. \ubc30\ub4dc\ubbfc\ud134, \ub18d\uad6c, \ubc30\uad6c\uc640 \uac19\uc740 \uc815\uc2dd\uc885\ubaa9\ub4e4\uc740 \ucc98\uc74c\uc5d0\ub294 \uc2dc\ubc94\uc885\ubaa9\uc774\uc5c8\uc73c\uba70 \uadf8 \ud6c4\uc5d0 \uc815\uc2dd\uc885\ubaa9\uc73c\ub85c \uc2b9\uc778 \ub418\uc5c8\ub2e4. \uc57c\uad6c\ucc98\ub7fc \uc608\uc804\uc5d0\ub294 \uc815\uc2dd\uc885\ubaa9 \uc774\uc5c8\uc9c0\ub9cc \uc9c0\uae08\uc740 \uc815\uc2dd \uc885\ubaa9\uc5d0\uc11c \ube60\uc9c4 \uc885\ubaa9\ub3c4 \uc788\ub2e4."} {"question": "\uae40\uc7ac\uaddc\uc5d0 \ub530\ub974\uba74 \uac70\uc0ac\uac00 \uc5c6\ub2e4\uba74 \ubd80\ub9c8\ud56d\uc7c1\uc774 \uc5b4\ub514\ub85c \ud655\ub300\ub418\ub294 \uac83\uc744 \uc6b0\ub824\ud588\ub294\uac00?", "paragraph": "\uae40\uc7ac\uaddc\ub294 \u2018\ub0b4\uac00 (\uac70\uc0ac\ub97c) \uc548 \ud558\uba74 \ud2c0\ub9bc\uc5c6\uc774 \ubd80\ub9c8\ud56d\uc7c1\uc774 5\ub300\ub3c4\uc2dc\ub85c \ud655\ub300\ub3fc\uc11c 4\u00b719\ubcf4\ub2e4 \ub354 \ud070 \uc0ac\ud0dc\uac00 \uc77c\uc5b4\ub0a0 \uac83\uc774\ub2e4\u2019\uace0 \ud310\ub2e8\ud588\ub2e4. \uc774\uc2b9\ub9cc\uc740 \ubb3c\ub7ec\ub0a0 \uc904 \uc54c\uc558\uc9c0\ub9cc \ubc15\uc815\ud76c\ub294 \uc808\ub300 \ubb3c\ub7ec\ub0a0 \uc131\uaca9\uc774 \uc544\ub2c8\ub77c\ub294 \ud310\ub2e8\uc744 \ud588\ub2e4. \ucc28\uc9c0\ucca0\uc740 \u2018\uce84\ubcf4\ub514\uc544\uc5d0\uc11c 300\ub9cc\uc744 \uc8fd\uc600\ub294\ub370 \uc6b0\ub9ac\uac00 100\ub9cc~200\ub9cc \uba85 \ubabb \uc8fd\uc774\uaca0\ub290\ub0d0\u2019\uace0 \ud588\ub2e4. \uadf8\ub7f0 \ucc38\ubaa8\uac00 \uc606\uc5d0 \uc788\uace0 \ubc15\uc815\ud76c\ub3c4 \u2018\uc61b\ub0a0 \ucd5c\uc778\uaddc\uc640 \uacfd\uc601\uc8fc\uac00 \uc8fd\uc740 \uac74 \uc790\uae30\ub4e4\uc774 \ubc1c\ud3ec \uba85\ub839\uc744 \ub0b4\ub838\uae30 \ub54c\ubb38\uc778\ub370 \ub0b4\uac00 \uc9c1\uc811 \ubc1c\ud3ec \uba85\ub839\uc744 \ub0b4\ub9ac\uba74 \ub098\ub97c \ucd1d\uc0b4\uc2dc\ud0ac \uc0ac\ub78c\uc774 \ub204\uac00 \uc788\ub290\ub0d0\u2019\ub77c\uace0 \ub9d0\uc744 \ud588\ub2e4. \uc774\uc5d0 \uae40\uc7ac\uaddc\ub294 \ub354 \ud070 \ud76c\uc0dd\uc744 \ub9c9\uae30\uc704\ud574\uc11c \ud588\ub2e4\uace0 \ud55c\ub2e4. \ud558\uc9c0\ub9cc \ucc28\uc9c0\ucca0\uacfc \ubd84\uc7c1\uc774 \uc788\uae30 \uc804\uae4c\uc9c0\ub294 \ubc15\uc815\ud76c\uc758 \ucda9\uc2e0\uc774\uc5c8\ub2e4\ub294 \uc810\uc5d0\uc11c \uadf8\uac00 \uae09\uc870\ud55c \uac83\uc774\ub77c\ub294 \uc8fc\uc7a5\uc774 \uc788\ub2e4.", "answer": "5\ub300\ub3c4\uc2dc", "sentence": "\uae40\uc7ac\uaddc\ub294 \u2018\ub0b4\uac00 (\uac70\uc0ac\ub97c) \uc548 \ud558\uba74 \ud2c0\ub9bc\uc5c6\uc774 \ubd80\ub9c8\ud56d\uc7c1\uc774 5\ub300\ub3c4\uc2dc \ub85c \ud655\ub300\ub3fc\uc11c 4\u00b719\ubcf4\ub2e4 \ub354 \ud070 \uc0ac\ud0dc\uac00 \uc77c\uc5b4\ub0a0 \uac83\uc774\ub2e4\u2019\uace0 \ud310\ub2e8\ud588\ub2e4.", "paragraph_sentence": " \uae40\uc7ac\uaddc\ub294 \u2018\ub0b4\uac00 (\uac70\uc0ac\ub97c) \uc548 \ud558\uba74 \ud2c0\ub9bc\uc5c6\uc774 \ubd80\ub9c8\ud56d\uc7c1\uc774 5\ub300\ub3c4\uc2dc \ub85c \ud655\ub300\ub3fc\uc11c 4\u00b719\ubcf4\ub2e4 \ub354 \ud070 \uc0ac\ud0dc\uac00 \uc77c\uc5b4\ub0a0 \uac83\uc774\ub2e4\u2019\uace0 \ud310\ub2e8\ud588\ub2e4. \uc774\uc2b9\ub9cc\uc740 \ubb3c\ub7ec\ub0a0 \uc904 \uc54c\uc558\uc9c0\ub9cc \ubc15\uc815\ud76c\ub294 \uc808\ub300 \ubb3c\ub7ec\ub0a0 \uc131\uaca9\uc774 \uc544\ub2c8\ub77c\ub294 \ud310\ub2e8\uc744 \ud588\ub2e4. \ucc28\uc9c0\ucca0\uc740 \u2018\uce84\ubcf4\ub514\uc544\uc5d0\uc11c 300\ub9cc\uc744 \uc8fd\uc600\ub294\ub370 \uc6b0\ub9ac\uac00 100\ub9cc~200\ub9cc \uba85 \ubabb \uc8fd\uc774\uaca0\ub290\ub0d0\u2019\uace0 \ud588\ub2e4. \uadf8\ub7f0 \ucc38\ubaa8\uac00 \uc606\uc5d0 \uc788\uace0 \ubc15\uc815\ud76c\ub3c4 \u2018\uc61b\ub0a0 \ucd5c\uc778\uaddc\uc640 \uacfd\uc601\uc8fc\uac00 \uc8fd\uc740 \uac74 \uc790\uae30\ub4e4\uc774 \ubc1c\ud3ec \uba85\ub839\uc744 \ub0b4\ub838\uae30 \ub54c\ubb38\uc778\ub370 \ub0b4\uac00 \uc9c1\uc811 \ubc1c\ud3ec \uba85\ub839\uc744 \ub0b4\ub9ac\uba74 \ub098\ub97c \ucd1d\uc0b4\uc2dc\ud0ac \uc0ac\ub78c\uc774 \ub204\uac00 \uc788\ub290\ub0d0\u2019\ub77c\uace0 \ub9d0\uc744 \ud588\ub2e4. \uc774\uc5d0 \uae40\uc7ac\uaddc\ub294 \ub354 \ud070 \ud76c\uc0dd\uc744 \ub9c9\uae30\uc704\ud574\uc11c \ud588\ub2e4\uace0 \ud55c\ub2e4. \ud558\uc9c0\ub9cc \ucc28\uc9c0\ucca0\uacfc \ubd84\uc7c1\uc774 \uc788\uae30 \uc804\uae4c\uc9c0\ub294 \ubc15\uc815\ud76c\uc758 \ucda9\uc2e0\uc774\uc5c8\ub2e4\ub294 \uc810\uc5d0\uc11c \uadf8\uac00 \uae09\uc870\ud55c \uac83\uc774\ub77c\ub294 \uc8fc\uc7a5\uc774 \uc788\ub2e4.", "paragraph_answer": "\uae40\uc7ac\uaddc\ub294 \u2018\ub0b4\uac00 (\uac70\uc0ac\ub97c) \uc548 \ud558\uba74 \ud2c0\ub9bc\uc5c6\uc774 \ubd80\ub9c8\ud56d\uc7c1\uc774 5\ub300\ub3c4\uc2dc \ub85c \ud655\ub300\ub3fc\uc11c 4\u00b719\ubcf4\ub2e4 \ub354 \ud070 \uc0ac\ud0dc\uac00 \uc77c\uc5b4\ub0a0 \uac83\uc774\ub2e4\u2019\uace0 \ud310\ub2e8\ud588\ub2e4. \uc774\uc2b9\ub9cc\uc740 \ubb3c\ub7ec\ub0a0 \uc904 \uc54c\uc558\uc9c0\ub9cc \ubc15\uc815\ud76c\ub294 \uc808\ub300 \ubb3c\ub7ec\ub0a0 \uc131\uaca9\uc774 \uc544\ub2c8\ub77c\ub294 \ud310\ub2e8\uc744 \ud588\ub2e4. \ucc28\uc9c0\ucca0\uc740 \u2018\uce84\ubcf4\ub514\uc544\uc5d0\uc11c 300\ub9cc\uc744 \uc8fd\uc600\ub294\ub370 \uc6b0\ub9ac\uac00 100\ub9cc~200\ub9cc \uba85 \ubabb \uc8fd\uc774\uaca0\ub290\ub0d0\u2019\uace0 \ud588\ub2e4. \uadf8\ub7f0 \ucc38\ubaa8\uac00 \uc606\uc5d0 \uc788\uace0 \ubc15\uc815\ud76c\ub3c4 \u2018\uc61b\ub0a0 \ucd5c\uc778\uaddc\uc640 \uacfd\uc601\uc8fc\uac00 \uc8fd\uc740 \uac74 \uc790\uae30\ub4e4\uc774 \ubc1c\ud3ec \uba85\ub839\uc744 \ub0b4\ub838\uae30 \ub54c\ubb38\uc778\ub370 \ub0b4\uac00 \uc9c1\uc811 \ubc1c\ud3ec \uba85\ub839\uc744 \ub0b4\ub9ac\uba74 \ub098\ub97c \ucd1d\uc0b4\uc2dc\ud0ac \uc0ac\ub78c\uc774 \ub204\uac00 \uc788\ub290\ub0d0\u2019\ub77c\uace0 \ub9d0\uc744 \ud588\ub2e4. \uc774\uc5d0 \uae40\uc7ac\uaddc\ub294 \ub354 \ud070 \ud76c\uc0dd\uc744 \ub9c9\uae30\uc704\ud574\uc11c \ud588\ub2e4\uace0 \ud55c\ub2e4. \ud558\uc9c0\ub9cc \ucc28\uc9c0\ucca0\uacfc \ubd84\uc7c1\uc774 \uc788\uae30 \uc804\uae4c\uc9c0\ub294 \ubc15\uc815\ud76c\uc758 \ucda9\uc2e0\uc774\uc5c8\ub2e4\ub294 \uc810\uc5d0\uc11c \uadf8\uac00 \uae09\uc870\ud55c \uac83\uc774\ub77c\ub294 \uc8fc\uc7a5\uc774 \uc788\ub2e4.", "sentence_answer": "\uae40\uc7ac\uaddc\ub294 \u2018\ub0b4\uac00 (\uac70\uc0ac\ub97c) \uc548 \ud558\uba74 \ud2c0\ub9bc\uc5c6\uc774 \ubd80\ub9c8\ud56d\uc7c1\uc774 5\ub300\ub3c4\uc2dc \ub85c \ud655\ub300\ub3fc\uc11c 4\u00b719\ubcf4\ub2e4 \ub354 \ud070 \uc0ac\ud0dc\uac00 \uc77c\uc5b4\ub0a0 \uac83\uc774\ub2e4\u2019\uace0 \ud310\ub2e8\ud588\ub2e4.", "paragraph_id": "6269946-0-2", "paragraph_question": "question: \uae40\uc7ac\uaddc\uc5d0 \ub530\ub974\uba74 \uac70\uc0ac\uac00 \uc5c6\ub2e4\uba74 \ubd80\ub9c8\ud56d\uc7c1\uc774 \uc5b4\ub514\ub85c \ud655\ub300\ub418\ub294 \uac83\uc744 \uc6b0\ub824\ud588\ub294\uac00?, context: \uae40\uc7ac\uaddc\ub294 \u2018\ub0b4\uac00 (\uac70\uc0ac\ub97c) \uc548 \ud558\uba74 \ud2c0\ub9bc\uc5c6\uc774 \ubd80\ub9c8\ud56d\uc7c1\uc774 5\ub300\ub3c4\uc2dc\ub85c \ud655\ub300\ub3fc\uc11c 4\u00b719\ubcf4\ub2e4 \ub354 \ud070 \uc0ac\ud0dc\uac00 \uc77c\uc5b4\ub0a0 \uac83\uc774\ub2e4\u2019\uace0 \ud310\ub2e8\ud588\ub2e4. \uc774\uc2b9\ub9cc\uc740 \ubb3c\ub7ec\ub0a0 \uc904 \uc54c\uc558\uc9c0\ub9cc \ubc15\uc815\ud76c\ub294 \uc808\ub300 \ubb3c\ub7ec\ub0a0 \uc131\uaca9\uc774 \uc544\ub2c8\ub77c\ub294 \ud310\ub2e8\uc744 \ud588\ub2e4. \ucc28\uc9c0\ucca0\uc740 \u2018\uce84\ubcf4\ub514\uc544\uc5d0\uc11c 300\ub9cc\uc744 \uc8fd\uc600\ub294\ub370 \uc6b0\ub9ac\uac00 100\ub9cc~200\ub9cc \uba85 \ubabb \uc8fd\uc774\uaca0\ub290\ub0d0\u2019\uace0 \ud588\ub2e4. \uadf8\ub7f0 \ucc38\ubaa8\uac00 \uc606\uc5d0 \uc788\uace0 \ubc15\uc815\ud76c\ub3c4 \u2018\uc61b\ub0a0 \ucd5c\uc778\uaddc\uc640 \uacfd\uc601\uc8fc\uac00 \uc8fd\uc740 \uac74 \uc790\uae30\ub4e4\uc774 \ubc1c\ud3ec \uba85\ub839\uc744 \ub0b4\ub838\uae30 \ub54c\ubb38\uc778\ub370 \ub0b4\uac00 \uc9c1\uc811 \ubc1c\ud3ec \uba85\ub839\uc744 \ub0b4\ub9ac\uba74 \ub098\ub97c \ucd1d\uc0b4\uc2dc\ud0ac \uc0ac\ub78c\uc774 \ub204\uac00 \uc788\ub290\ub0d0\u2019\ub77c\uace0 \ub9d0\uc744 \ud588\ub2e4. \uc774\uc5d0 \uae40\uc7ac\uaddc\ub294 \ub354 \ud070 \ud76c\uc0dd\uc744 \ub9c9\uae30\uc704\ud574\uc11c \ud588\ub2e4\uace0 \ud55c\ub2e4. \ud558\uc9c0\ub9cc \ucc28\uc9c0\ucca0\uacfc \ubd84\uc7c1\uc774 \uc788\uae30 \uc804\uae4c\uc9c0\ub294 \ubc15\uc815\ud76c\uc758 \ucda9\uc2e0\uc774\uc5c8\ub2e4\ub294 \uc810\uc5d0\uc11c \uadf8\uac00 \uae09\uc870\ud55c \uac83\uc774\ub77c\ub294 \uc8fc\uc7a5\uc774 \uc788\ub2e4."} {"question": "\ud0c0\ube14\ub85c\uac00 \ud559\ub825 \ub17c\ub780\uc73c\ub85c \ud65c\ub3d9\uc744 \uc911\ub2e8\ud55c \ub54c\ub294?", "paragraph": "\u300a\uc5f4\uaf43\u300b\uc5d0 \uc218\ub85d\ub41c \uace1\ub4e4\uc740 2010\ub144 5\uc6d4\ubd80\ud130 \ub2e4\uc2dc\uae08 \ubd88\uac70\uc9c4 \ud559\ub825 \ub17c\ub780\uc73c\ub85c \uc778\ud574 \ud65c\ub3d9\uc744 \uc804\uba74 \uc911\ub2e8\ud55c \ud0c0\ube14\ub85c\uac00 2010\ub144 7\uc6d4\ubd80\ud130 2011\ub144 \ucd08\uae4c\uc9c0 \ud640\ub85c \uc791\uc5c5\ud55c \uace1\ub4e4\uc778\ub370, \uc791\uc5c5 \ub2f9\uc2dc \ud559\ub825 \ub17c\ub780\uc73c\ub85c \uc778\ud574 \ub69c\ub837\ud55c \uacc4\ud68d \ubc0f \ubaa9\ud45c\uc5c6\uc774 \uc791\uc5c5\ud55c \uace1\ub4e4\uc774\ub77c\uace0 \ubc1d\ud614\ub2e4. \ud65c\ub3d9 \uc911\ub2e8 \uc774\ud6c4 \uc57d 1\ub144\uc774 \uc9c0\ub09c 2011\ub144 5\uc6d4\ubd80\ud130 \ub450 \ucc28\ub840\uc758 \uac15\uc5f0\uc744 \ud568\uc73c\ub85c\uc368 \ub300\uc678\uc801\uc778 \ud65c\ub3d9\uc744 \uc2dc\uc791\ud558\uba74\uc11c\ub3c4 \uc74c\uc545 \ud65c\ub3d9\uc5d0 \ub300\ud55c \uad6c\uccb4\uc801\uc778 \uc5b8\uae09\uc740 \ud53c\ud588\ub2e4. \uadf8 \ub54c\ubd80\ud130 YG \uc5d4\ud130\ud14c\uc778\uba3c\ud2b8\uac00 \ub9e4\ub2c8\uc800 \ubc0f \ucc28\ub7c9\uc744 \uc81c\uacf5\ud558\ub294 \ub4f1 \ud0c0\ube14\ub85c\uac00 \ud65c\ub3d9\ud558\ub294 \ub370 \ub3c4\uc6c0\uc744 \uc8fc\uc5c8\uace0 \ub610\ud55c \ud0c0\ube14\ub85c\uc758 \uc791\uc5c5 \uace1\ub4e4\uc744 \ub4e4\uc740 \uc591\ud604\uc11d\uc774 \uc74c\ubc18 \ubc1c\ub9e4 \uc81c\uc548\uc744 \ud558\uba70 \uc774\ud6c4 9\uc6d4 27\uc77c, YG \uc5d4\ud130\ud14c\uc778\uba3c\ud2b8\uc640 4\ub144 \uacc4\uc57d\uc744 \ub9fa\uace0 11\uc6d4 1\uc77c, \uc194\ub85c \uc74c\ubc18\uc744 \ubc1c\ub9e4\ud55c\ub2e4\uace0 \ubc1d\ud614\ub2e4.", "answer": "2010\ub144 5\uc6d4", "sentence": "\u300a\uc5f4\uaf43\u300b\uc5d0 \uc218\ub85d\ub41c \uace1\ub4e4\uc740 2010\ub144 5\uc6d4 \ubd80\ud130 \ub2e4\uc2dc\uae08 \ubd88\uac70\uc9c4 \ud559\ub825 \ub17c\ub780\uc73c\ub85c \uc778\ud574 \ud65c\ub3d9\uc744 \uc804\uba74 \uc911\ub2e8\ud55c \ud0c0\ube14\ub85c\uac00 2010\ub144 7\uc6d4\ubd80\ud130 2011\ub144 \ucd08\uae4c\uc9c0 \ud640\ub85c \uc791\uc5c5\ud55c \uace1\ub4e4\uc778\ub370, \uc791\uc5c5 \ub2f9\uc2dc \ud559\ub825 \ub17c\ub780\uc73c\ub85c \uc778\ud574 \ub69c\ub837\ud55c \uacc4\ud68d \ubc0f \ubaa9\ud45c\uc5c6\uc774 \uc791\uc5c5\ud55c \uace1\ub4e4\uc774\ub77c\uace0 \ubc1d\ud614\ub2e4.", "paragraph_sentence": " \u300a\uc5f4\uaf43\u300b\uc5d0 \uc218\ub85d\ub41c \uace1\ub4e4\uc740 2010\ub144 5\uc6d4 \ubd80\ud130 \ub2e4\uc2dc\uae08 \ubd88\uac70\uc9c4 \ud559\ub825 \ub17c\ub780\uc73c\ub85c \uc778\ud574 \ud65c\ub3d9\uc744 \uc804\uba74 \uc911\ub2e8\ud55c \ud0c0\ube14\ub85c\uac00 2010\ub144 7\uc6d4\ubd80\ud130 2011\ub144 \ucd08\uae4c\uc9c0 \ud640\ub85c \uc791\uc5c5\ud55c \uace1\ub4e4\uc778\ub370, \uc791\uc5c5 \ub2f9\uc2dc \ud559\ub825 \ub17c\ub780\uc73c\ub85c \uc778\ud574 \ub69c\ub837\ud55c \uacc4\ud68d \ubc0f \ubaa9\ud45c\uc5c6\uc774 \uc791\uc5c5\ud55c \uace1\ub4e4\uc774\ub77c\uace0 \ubc1d\ud614\ub2e4. \ud65c\ub3d9 \uc911\ub2e8 \uc774\ud6c4 \uc57d 1\ub144\uc774 \uc9c0\ub09c 2011\ub144 5\uc6d4\ubd80\ud130 \ub450 \ucc28\ub840\uc758 \uac15\uc5f0\uc744 \ud568\uc73c\ub85c\uc368 \ub300\uc678\uc801\uc778 \ud65c\ub3d9\uc744 \uc2dc\uc791\ud558\uba74\uc11c\ub3c4 \uc74c\uc545 \ud65c\ub3d9\uc5d0 \ub300\ud55c \uad6c\uccb4\uc801\uc778 \uc5b8\uae09\uc740 \ud53c\ud588\ub2e4. \uadf8 \ub54c\ubd80\ud130 YG \uc5d4\ud130\ud14c\uc778\uba3c\ud2b8\uac00 \ub9e4\ub2c8\uc800 \ubc0f \ucc28\ub7c9\uc744 \uc81c\uacf5\ud558\ub294 \ub4f1 \ud0c0\ube14\ub85c\uac00 \ud65c\ub3d9\ud558\ub294 \ub370 \ub3c4\uc6c0\uc744 \uc8fc\uc5c8\uace0 \ub610\ud55c \ud0c0\ube14\ub85c\uc758 \uc791\uc5c5 \uace1\ub4e4\uc744 \ub4e4\uc740 \uc591\ud604\uc11d\uc774 \uc74c\ubc18 \ubc1c\ub9e4 \uc81c\uc548\uc744 \ud558\uba70 \uc774\ud6c4 9\uc6d4 27\uc77c, YG \uc5d4\ud130\ud14c\uc778\uba3c\ud2b8\uc640 4\ub144 \uacc4\uc57d\uc744 \ub9fa\uace0 11\uc6d4 1\uc77c, \uc194\ub85c \uc74c\ubc18\uc744 \ubc1c\ub9e4\ud55c\ub2e4\uace0 \ubc1d\ud614\ub2e4.", "paragraph_answer": "\u300a\uc5f4\uaf43\u300b\uc5d0 \uc218\ub85d\ub41c \uace1\ub4e4\uc740 2010\ub144 5\uc6d4 \ubd80\ud130 \ub2e4\uc2dc\uae08 \ubd88\uac70\uc9c4 \ud559\ub825 \ub17c\ub780\uc73c\ub85c \uc778\ud574 \ud65c\ub3d9\uc744 \uc804\uba74 \uc911\ub2e8\ud55c \ud0c0\ube14\ub85c\uac00 2010\ub144 7\uc6d4\ubd80\ud130 2011\ub144 \ucd08\uae4c\uc9c0 \ud640\ub85c \uc791\uc5c5\ud55c \uace1\ub4e4\uc778\ub370, \uc791\uc5c5 \ub2f9\uc2dc \ud559\ub825 \ub17c\ub780\uc73c\ub85c \uc778\ud574 \ub69c\ub837\ud55c \uacc4\ud68d \ubc0f \ubaa9\ud45c\uc5c6\uc774 \uc791\uc5c5\ud55c \uace1\ub4e4\uc774\ub77c\uace0 \ubc1d\ud614\ub2e4. \ud65c\ub3d9 \uc911\ub2e8 \uc774\ud6c4 \uc57d 1\ub144\uc774 \uc9c0\ub09c 2011\ub144 5\uc6d4\ubd80\ud130 \ub450 \ucc28\ub840\uc758 \uac15\uc5f0\uc744 \ud568\uc73c\ub85c\uc368 \ub300\uc678\uc801\uc778 \ud65c\ub3d9\uc744 \uc2dc\uc791\ud558\uba74\uc11c\ub3c4 \uc74c\uc545 \ud65c\ub3d9\uc5d0 \ub300\ud55c \uad6c\uccb4\uc801\uc778 \uc5b8\uae09\uc740 \ud53c\ud588\ub2e4. \uadf8 \ub54c\ubd80\ud130 YG \uc5d4\ud130\ud14c\uc778\uba3c\ud2b8\uac00 \ub9e4\ub2c8\uc800 \ubc0f \ucc28\ub7c9\uc744 \uc81c\uacf5\ud558\ub294 \ub4f1 \ud0c0\ube14\ub85c\uac00 \ud65c\ub3d9\ud558\ub294 \ub370 \ub3c4\uc6c0\uc744 \uc8fc\uc5c8\uace0 \ub610\ud55c \ud0c0\ube14\ub85c\uc758 \uc791\uc5c5 \uace1\ub4e4\uc744 \ub4e4\uc740 \uc591\ud604\uc11d\uc774 \uc74c\ubc18 \ubc1c\ub9e4 \uc81c\uc548\uc744 \ud558\uba70 \uc774\ud6c4 9\uc6d4 27\uc77c, YG \uc5d4\ud130\ud14c\uc778\uba3c\ud2b8\uc640 4\ub144 \uacc4\uc57d\uc744 \ub9fa\uace0 11\uc6d4 1\uc77c, \uc194\ub85c \uc74c\ubc18\uc744 \ubc1c\ub9e4\ud55c\ub2e4\uace0 \ubc1d\ud614\ub2e4.", "sentence_answer": "\u300a\uc5f4\uaf43\u300b\uc5d0 \uc218\ub85d\ub41c \uace1\ub4e4\uc740 2010\ub144 5\uc6d4 \ubd80\ud130 \ub2e4\uc2dc\uae08 \ubd88\uac70\uc9c4 \ud559\ub825 \ub17c\ub780\uc73c\ub85c \uc778\ud574 \ud65c\ub3d9\uc744 \uc804\uba74 \uc911\ub2e8\ud55c \ud0c0\ube14\ub85c\uac00 2010\ub144 7\uc6d4\ubd80\ud130 2011\ub144 \ucd08\uae4c\uc9c0 \ud640\ub85c \uc791\uc5c5\ud55c \uace1\ub4e4\uc778\ub370, \uc791\uc5c5 \ub2f9\uc2dc \ud559\ub825 \ub17c\ub780\uc73c\ub85c \uc778\ud574 \ub69c\ub837\ud55c \uacc4\ud68d \ubc0f \ubaa9\ud45c\uc5c6\uc774 \uc791\uc5c5\ud55c \uace1\ub4e4\uc774\ub77c\uace0 \ubc1d\ud614\ub2e4.", "paragraph_id": "6524103-6-0", "paragraph_question": "question: \ud0c0\ube14\ub85c\uac00 \ud559\ub825 \ub17c\ub780\uc73c\ub85c \ud65c\ub3d9\uc744 \uc911\ub2e8\ud55c \ub54c\ub294?, context: \u300a\uc5f4\uaf43\u300b\uc5d0 \uc218\ub85d\ub41c \uace1\ub4e4\uc740 2010\ub144 5\uc6d4\ubd80\ud130 \ub2e4\uc2dc\uae08 \ubd88\uac70\uc9c4 \ud559\ub825 \ub17c\ub780\uc73c\ub85c \uc778\ud574 \ud65c\ub3d9\uc744 \uc804\uba74 \uc911\ub2e8\ud55c \ud0c0\ube14\ub85c\uac00 2010\ub144 7\uc6d4\ubd80\ud130 2011\ub144 \ucd08\uae4c\uc9c0 \ud640\ub85c \uc791\uc5c5\ud55c \uace1\ub4e4\uc778\ub370, \uc791\uc5c5 \ub2f9\uc2dc \ud559\ub825 \ub17c\ub780\uc73c\ub85c \uc778\ud574 \ub69c\ub837\ud55c \uacc4\ud68d \ubc0f \ubaa9\ud45c\uc5c6\uc774 \uc791\uc5c5\ud55c \uace1\ub4e4\uc774\ub77c\uace0 \ubc1d\ud614\ub2e4. \ud65c\ub3d9 \uc911\ub2e8 \uc774\ud6c4 \uc57d 1\ub144\uc774 \uc9c0\ub09c 2011\ub144 5\uc6d4\ubd80\ud130 \ub450 \ucc28\ub840\uc758 \uac15\uc5f0\uc744 \ud568\uc73c\ub85c\uc368 \ub300\uc678\uc801\uc778 \ud65c\ub3d9\uc744 \uc2dc\uc791\ud558\uba74\uc11c\ub3c4 \uc74c\uc545 \ud65c\ub3d9\uc5d0 \ub300\ud55c \uad6c\uccb4\uc801\uc778 \uc5b8\uae09\uc740 \ud53c\ud588\ub2e4. \uadf8 \ub54c\ubd80\ud130 YG \uc5d4\ud130\ud14c\uc778\uba3c\ud2b8\uac00 \ub9e4\ub2c8\uc800 \ubc0f \ucc28\ub7c9\uc744 \uc81c\uacf5\ud558\ub294 \ub4f1 \ud0c0\ube14\ub85c\uac00 \ud65c\ub3d9\ud558\ub294 \ub370 \ub3c4\uc6c0\uc744 \uc8fc\uc5c8\uace0 \ub610\ud55c \ud0c0\ube14\ub85c\uc758 \uc791\uc5c5 \uace1\ub4e4\uc744 \ub4e4\uc740 \uc591\ud604\uc11d\uc774 \uc74c\ubc18 \ubc1c\ub9e4 \uc81c\uc548\uc744 \ud558\uba70 \uc774\ud6c4 9\uc6d4 27\uc77c, YG \uc5d4\ud130\ud14c\uc778\uba3c\ud2b8\uc640 4\ub144 \uacc4\uc57d\uc744 \ub9fa\uace0 11\uc6d4 1\uc77c, \uc194\ub85c \uc74c\ubc18\uc744 \ubc1c\ub9e4\ud55c\ub2e4\uace0 \ubc1d\ud614\ub2e4."} {"question": "\uc54c\ubca0\ub974\ud1a0 \ubaac\ub514\ub294 \uac15\uc6d0\ub300\ud559\uad50\uc5d0\uc11c \uc5b4\ub5a4 \ud559\ubb38\uc73c\ub85c \uc11d\uc0ac\ud559\uc704\ub97c \ucde8\ub4dd\ud588\ub294\uac00?", "paragraph": "6\uc138 \ub54c\ubd80\ud130 \ucd95\uad6c\ub97c \uc2dc\uc791\ud558\uc5ec 21\uc0b4\uae4c\uc9c0 \uc138\ub9ac\uc5d0 D \ucd95\uad6c \uc120\uc218\uc600\ub2e4\uac00 \ub808\uac00 \ud504\ub85c\ub85c \ud300\uc774 \uc2b9\uaca9\ud560 \ud655\ub960\uc774 \uc5c6\ub2e4\uace0 \ud310\ub2e8\ud558\uc5ec \ucd95\uad6c\ub97c \uadf8\ub9cc\ub450\uc5c8\ub2e4. 2003\ub144\ubd80\ud130 2007\ub144\uae4c\uc9c0 \uce74 \ud3ec\uc2a4\uce74\ub9ac \ubca0\ub124\uce58\uc544 \ub300\ud559\uad50 \ub3d9\uc544\uc2dc\uc544\ubb38\ud654\ud559 \uc911\uc5b4\uc911\ubb38\ud559 \uc804\uacf5\uc744 \uacf5\ubd80\ud558\uc600\uc73c\uba70, \ub300\ud559\uc0dd \uc2dc\uc808\uc5d0 1\ub144 \uac04 \uc911\uad6d\uc758 \ub2e4\ub844 \uc678\uad6d\uc5b4 \ub300\ud559\uad50\uc5d0\uc11c \uad50\ud658 \ud559\uc0dd\uc73c\ub85c \uacf5\ubd80\ub97c \ud588\ub2e4. \uac70\uae30\uc11c \uc9c0\uae08\uc758 \ud55c\uad6d\uc778 \uc544\ub0b4\ub97c \ub9cc\ub0ac\ub2e4. \ub300\ud559\uad50\ub97c \uc878\uc5c5\ud55c \ub4a4 \ub300\ud55c\ubbfc\uad6d\uc5d0 \ub4e4\uc5b4\uc640 2008\ub144 1\uc6d4\ubd80\ud130 4\uc6d4\uae4c\uc9c0 \uc8fc\ud55c \uc774\ud0c8\ub9ac\uc544 \ub300\uc0ac\uad00\uc5d0\uc11c \uc778\ud134\uc73c\ub85c \uadfc\ubb34\ud588\ub2e4. \uc774\ud6c4 2008\ub144\ubd80\ud130 2010\ub144\uae4c\uc9c0 \uac15\uc6d0\ub300\ud559\uad50\uc5d0\uc11c \uacbd\uc81c\ud559\uacfc \uc11d\uc0ac\uacfc\uc815\uc744 \ub9c8\uccd0 \uacbd\uc81c\ud559 \uc11d\uc0ac\ud559\uc704(\ud559\uc704\ub17c\ubb38\uba85 - Oil Shocks and the World Rice Market Puzzle: a VAR Analysis)\ub97c \ucde8\ub4dd\ud588\uace0, 2010\ub144\uc5d0\ub294 \ud55c\uad6d\uc870\uc138\uc5f0\uad6c\uc6d0\uc5d0\uc11c \ub300\uc678 \ud611\ub825 \uc5c5\ubb34\ub97c \ub2f4\ub2f9\ud558\uba74\uc11c \ud559\ud68c \ubc0f \uad50\ub958 \ud65c\ub3d9\uc744 \uc9c0\uc6d0\ud558\uae30\ub3c4 \ud588\ub2e4. 2011\ub144\ubd80\ud130 2013\ub144\uc5d0\ub294 \uc774\ud0c8\ub9ac\uc544\uc758 \ub9e5\uc8fc \ud68c\uc0ac\uc778 SAB \ubc00\ub7ec\uc5d0\uc11c \uc77c\ud558\uba70, \ub300\ud55c\ubbfc\uad6d\uc5d0 \uc774\ud0c8\ub9ac\uc544 \ub9e5\uc8fc\ub97c \ub860\uce6d\ud558\ub294 \uc77c\ub3c4 \ub2f4\ub2f9\ud588\ub2e4. 2013\ub144\ubd80\ud130 \uc8fc\ud55c \uc774\ud0c8\ub9ac\uc544 \uc0c1\uacf5\ud68c\uc758\uc18c\uc758 \uc774\uc0ac\ud68c\uc6d0\uc73c\ub85c \uc77c\ud558\uace0 \uc788\uc73c\uba70, \ub610 2013\ub144 8\uc6d4\uc5d0 \ud53c\uc544\ud2b8 \ud06c\ub77c\uc774\uc2ac\ub7ec\uc5d0 \uc785\uc0ac\ud55c \ud6c4, \ud68c\uc0ac\uc5d0\uc11c \ub51c\ub7ec \ubc0f \uc804\uc2dc\uc7a5 \uc601\uc5c5 \uc0ac\uc6d0\uc744 \uad00\ub9ac\ud558\ub294 \uc9c1\ubb34\ub97c \ub9e1\uace0 \uc788\ub2e4.", "answer": "\uacbd\uc81c\ud559", "sentence": "\uc774\ud6c4 2008\ub144\ubd80\ud130 2010\ub144\uae4c\uc9c0 \uac15\uc6d0\ub300\ud559\uad50\uc5d0\uc11c \uacbd\uc81c\ud559 \uacfc \uc11d\uc0ac\uacfc\uc815\uc744 \ub9c8\uccd0 \uacbd\uc81c\ud559 \uc11d\uc0ac\ud559\uc704(\ud559\uc704\ub17c\ubb38\uba85 - Oil Shocks and the World Rice Market Puzzle: a VAR Analysis)\ub97c \ucde8\ub4dd\ud588\uace0, 2010\ub144\uc5d0\ub294 \ud55c\uad6d\uc870\uc138\uc5f0\uad6c\uc6d0\uc5d0\uc11c \ub300\uc678 \ud611\ub825 \uc5c5\ubb34\ub97c \ub2f4\ub2f9\ud558\uba74\uc11c \ud559\ud68c \ubc0f \uad50\ub958 \ud65c\ub3d9\uc744 \uc9c0\uc6d0\ud558\uae30\ub3c4 \ud588\ub2e4.", "paragraph_sentence": "6\uc138 \ub54c\ubd80\ud130 \ucd95\uad6c\ub97c \uc2dc\uc791\ud558\uc5ec 21\uc0b4\uae4c\uc9c0 \uc138\ub9ac\uc5d0 D \ucd95\uad6c \uc120\uc218\uc600\ub2e4\uac00 \ub808\uac00 \ud504\ub85c\ub85c \ud300\uc774 \uc2b9\uaca9\ud560 \ud655\ub960\uc774 \uc5c6\ub2e4\uace0 \ud310\ub2e8\ud558\uc5ec \ucd95\uad6c\ub97c \uadf8\ub9cc\ub450\uc5c8\ub2e4. 2003\ub144\ubd80\ud130 2007\ub144\uae4c\uc9c0 \uce74 \ud3ec\uc2a4\uce74\ub9ac \ubca0\ub124\uce58\uc544 \ub300\ud559\uad50 \ub3d9\uc544\uc2dc\uc544\ubb38\ud654\ud559 \uc911\uc5b4\uc911\ubb38\ud559 \uc804\uacf5\uc744 \uacf5\ubd80\ud558\uc600\uc73c\uba70, \ub300\ud559\uc0dd \uc2dc\uc808\uc5d0 1\ub144 \uac04 \uc911\uad6d\uc758 \ub2e4\ub844 \uc678\uad6d\uc5b4 \ub300\ud559\uad50\uc5d0\uc11c \uad50\ud658 \ud559\uc0dd\uc73c\ub85c \uacf5\ubd80\ub97c \ud588\ub2e4. \uac70\uae30\uc11c \uc9c0\uae08\uc758 \ud55c\uad6d\uc778 \uc544\ub0b4\ub97c \ub9cc\ub0ac\ub2e4. \ub300\ud559\uad50\ub97c \uc878\uc5c5\ud55c \ub4a4 \ub300\ud55c\ubbfc\uad6d\uc5d0 \ub4e4\uc5b4\uc640 2008\ub144 1\uc6d4\ubd80\ud130 4\uc6d4\uae4c\uc9c0 \uc8fc\ud55c \uc774\ud0c8\ub9ac\uc544 \ub300\uc0ac\uad00\uc5d0\uc11c \uc778\ud134\uc73c\ub85c \uadfc\ubb34\ud588\ub2e4. \uc774\ud6c4 2008\ub144\ubd80\ud130 2010\ub144\uae4c\uc9c0 \uac15\uc6d0\ub300\ud559\uad50\uc5d0\uc11c \uacbd\uc81c\ud559 \uacfc \uc11d\uc0ac\uacfc\uc815\uc744 \ub9c8\uccd0 \uacbd\uc81c\ud559 \uc11d\uc0ac\ud559\uc704(\ud559\uc704\ub17c\ubb38\uba85 - Oil Shocks and the World Rice Market Puzzle: a VAR Analysis)\ub97c \ucde8\ub4dd\ud588\uace0, 2010\ub144\uc5d0\ub294 \ud55c\uad6d\uc870\uc138\uc5f0\uad6c\uc6d0\uc5d0\uc11c \ub300\uc678 \ud611\ub825 \uc5c5\ubb34\ub97c \ub2f4\ub2f9\ud558\uba74\uc11c \ud559\ud68c \ubc0f \uad50\ub958 \ud65c\ub3d9\uc744 \uc9c0\uc6d0\ud558\uae30\ub3c4 \ud588\ub2e4. 2011\ub144\ubd80\ud130 2013\ub144\uc5d0\ub294 \uc774\ud0c8\ub9ac\uc544\uc758 \ub9e5\uc8fc \ud68c\uc0ac\uc778 SAB \ubc00\ub7ec\uc5d0\uc11c \uc77c\ud558\uba70, \ub300\ud55c\ubbfc\uad6d\uc5d0 \uc774\ud0c8\ub9ac\uc544 \ub9e5\uc8fc\ub97c \ub860\uce6d\ud558\ub294 \uc77c\ub3c4 \ub2f4\ub2f9\ud588\ub2e4. 2013\ub144\ubd80\ud130 \uc8fc\ud55c \uc774\ud0c8\ub9ac\uc544 \uc0c1\uacf5\ud68c\uc758\uc18c\uc758 \uc774\uc0ac\ud68c\uc6d0\uc73c\ub85c \uc77c\ud558\uace0 \uc788\uc73c\uba70, \ub610 2013\ub144 8\uc6d4\uc5d0 \ud53c\uc544\ud2b8 \ud06c\ub77c\uc774\uc2ac\ub7ec\uc5d0 \uc785\uc0ac\ud55c \ud6c4, \ud68c\uc0ac\uc5d0\uc11c \ub51c\ub7ec \ubc0f \uc804\uc2dc\uc7a5 \uc601\uc5c5 \uc0ac\uc6d0\uc744 \uad00\ub9ac\ud558\ub294 \uc9c1\ubb34\ub97c \ub9e1\uace0 \uc788\ub2e4.", "paragraph_answer": "6\uc138 \ub54c\ubd80\ud130 \ucd95\uad6c\ub97c \uc2dc\uc791\ud558\uc5ec 21\uc0b4\uae4c\uc9c0 \uc138\ub9ac\uc5d0 D \ucd95\uad6c \uc120\uc218\uc600\ub2e4\uac00 \ub808\uac00 \ud504\ub85c\ub85c \ud300\uc774 \uc2b9\uaca9\ud560 \ud655\ub960\uc774 \uc5c6\ub2e4\uace0 \ud310\ub2e8\ud558\uc5ec \ucd95\uad6c\ub97c \uadf8\ub9cc\ub450\uc5c8\ub2e4. 2003\ub144\ubd80\ud130 2007\ub144\uae4c\uc9c0 \uce74 \ud3ec\uc2a4\uce74\ub9ac \ubca0\ub124\uce58\uc544 \ub300\ud559\uad50 \ub3d9\uc544\uc2dc\uc544\ubb38\ud654\ud559 \uc911\uc5b4\uc911\ubb38\ud559 \uc804\uacf5\uc744 \uacf5\ubd80\ud558\uc600\uc73c\uba70, \ub300\ud559\uc0dd \uc2dc\uc808\uc5d0 1\ub144 \uac04 \uc911\uad6d\uc758 \ub2e4\ub844 \uc678\uad6d\uc5b4 \ub300\ud559\uad50\uc5d0\uc11c \uad50\ud658 \ud559\uc0dd\uc73c\ub85c \uacf5\ubd80\ub97c \ud588\ub2e4. \uac70\uae30\uc11c \uc9c0\uae08\uc758 \ud55c\uad6d\uc778 \uc544\ub0b4\ub97c \ub9cc\ub0ac\ub2e4. \ub300\ud559\uad50\ub97c \uc878\uc5c5\ud55c \ub4a4 \ub300\ud55c\ubbfc\uad6d\uc5d0 \ub4e4\uc5b4\uc640 2008\ub144 1\uc6d4\ubd80\ud130 4\uc6d4\uae4c\uc9c0 \uc8fc\ud55c \uc774\ud0c8\ub9ac\uc544 \ub300\uc0ac\uad00\uc5d0\uc11c \uc778\ud134\uc73c\ub85c \uadfc\ubb34\ud588\ub2e4. \uc774\ud6c4 2008\ub144\ubd80\ud130 2010\ub144\uae4c\uc9c0 \uac15\uc6d0\ub300\ud559\uad50\uc5d0\uc11c \uacbd\uc81c\ud559 \uacfc \uc11d\uc0ac\uacfc\uc815\uc744 \ub9c8\uccd0 \uacbd\uc81c\ud559 \uc11d\uc0ac\ud559\uc704(\ud559\uc704\ub17c\ubb38\uba85 - Oil Shocks and the World Rice Market Puzzle: a VAR Analysis)\ub97c \ucde8\ub4dd\ud588\uace0, 2010\ub144\uc5d0\ub294 \ud55c\uad6d\uc870\uc138\uc5f0\uad6c\uc6d0\uc5d0\uc11c \ub300\uc678 \ud611\ub825 \uc5c5\ubb34\ub97c \ub2f4\ub2f9\ud558\uba74\uc11c \ud559\ud68c \ubc0f \uad50\ub958 \ud65c\ub3d9\uc744 \uc9c0\uc6d0\ud558\uae30\ub3c4 \ud588\ub2e4. 2011\ub144\ubd80\ud130 2013\ub144\uc5d0\ub294 \uc774\ud0c8\ub9ac\uc544\uc758 \ub9e5\uc8fc \ud68c\uc0ac\uc778 SAB \ubc00\ub7ec\uc5d0\uc11c \uc77c\ud558\uba70, \ub300\ud55c\ubbfc\uad6d\uc5d0 \uc774\ud0c8\ub9ac\uc544 \ub9e5\uc8fc\ub97c \ub860\uce6d\ud558\ub294 \uc77c\ub3c4 \ub2f4\ub2f9\ud588\ub2e4. 2013\ub144\ubd80\ud130 \uc8fc\ud55c \uc774\ud0c8\ub9ac\uc544 \uc0c1\uacf5\ud68c\uc758\uc18c\uc758 \uc774\uc0ac\ud68c\uc6d0\uc73c\ub85c \uc77c\ud558\uace0 \uc788\uc73c\uba70, \ub610 2013\ub144 8\uc6d4\uc5d0 \ud53c\uc544\ud2b8 \ud06c\ub77c\uc774\uc2ac\ub7ec\uc5d0 \uc785\uc0ac\ud55c \ud6c4, \ud68c\uc0ac\uc5d0\uc11c \ub51c\ub7ec \ubc0f \uc804\uc2dc\uc7a5 \uc601\uc5c5 \uc0ac\uc6d0\uc744 \uad00\ub9ac\ud558\ub294 \uc9c1\ubb34\ub97c \ub9e1\uace0 \uc788\ub2e4.", "sentence_answer": "\uc774\ud6c4 2008\ub144\ubd80\ud130 2010\ub144\uae4c\uc9c0 \uac15\uc6d0\ub300\ud559\uad50\uc5d0\uc11c \uacbd\uc81c\ud559 \uacfc \uc11d\uc0ac\uacfc\uc815\uc744 \ub9c8\uccd0 \uacbd\uc81c\ud559 \uc11d\uc0ac\ud559\uc704(\ud559\uc704\ub17c\ubb38\uba85 - Oil Shocks and the World Rice Market Puzzle: a VAR Analysis)\ub97c \ucde8\ub4dd\ud588\uace0, 2010\ub144\uc5d0\ub294 \ud55c\uad6d\uc870\uc138\uc5f0\uad6c\uc6d0\uc5d0\uc11c \ub300\uc678 \ud611\ub825 \uc5c5\ubb34\ub97c \ub2f4\ub2f9\ud558\uba74\uc11c \ud559\ud68c \ubc0f \uad50\ub958 \ud65c\ub3d9\uc744 \uc9c0\uc6d0\ud558\uae30\ub3c4 \ud588\ub2e4.", "paragraph_id": "6521870-0-1", "paragraph_question": "question: \uc54c\ubca0\ub974\ud1a0 \ubaac\ub514\ub294 \uac15\uc6d0\ub300\ud559\uad50\uc5d0\uc11c \uc5b4\ub5a4 \ud559\ubb38\uc73c\ub85c \uc11d\uc0ac\ud559\uc704\ub97c \ucde8\ub4dd\ud588\ub294\uac00?, context: 6\uc138 \ub54c\ubd80\ud130 \ucd95\uad6c\ub97c \uc2dc\uc791\ud558\uc5ec 21\uc0b4\uae4c\uc9c0 \uc138\ub9ac\uc5d0 D \ucd95\uad6c \uc120\uc218\uc600\ub2e4\uac00 \ub808\uac00 \ud504\ub85c\ub85c \ud300\uc774 \uc2b9\uaca9\ud560 \ud655\ub960\uc774 \uc5c6\ub2e4\uace0 \ud310\ub2e8\ud558\uc5ec \ucd95\uad6c\ub97c \uadf8\ub9cc\ub450\uc5c8\ub2e4. 2003\ub144\ubd80\ud130 2007\ub144\uae4c\uc9c0 \uce74 \ud3ec\uc2a4\uce74\ub9ac \ubca0\ub124\uce58\uc544 \ub300\ud559\uad50 \ub3d9\uc544\uc2dc\uc544\ubb38\ud654\ud559 \uc911\uc5b4\uc911\ubb38\ud559 \uc804\uacf5\uc744 \uacf5\ubd80\ud558\uc600\uc73c\uba70, \ub300\ud559\uc0dd \uc2dc\uc808\uc5d0 1\ub144 \uac04 \uc911\uad6d\uc758 \ub2e4\ub844 \uc678\uad6d\uc5b4 \ub300\ud559\uad50\uc5d0\uc11c \uad50\ud658 \ud559\uc0dd\uc73c\ub85c \uacf5\ubd80\ub97c \ud588\ub2e4. \uac70\uae30\uc11c \uc9c0\uae08\uc758 \ud55c\uad6d\uc778 \uc544\ub0b4\ub97c \ub9cc\ub0ac\ub2e4. \ub300\ud559\uad50\ub97c \uc878\uc5c5\ud55c \ub4a4 \ub300\ud55c\ubbfc\uad6d\uc5d0 \ub4e4\uc5b4\uc640 2008\ub144 1\uc6d4\ubd80\ud130 4\uc6d4\uae4c\uc9c0 \uc8fc\ud55c \uc774\ud0c8\ub9ac\uc544 \ub300\uc0ac\uad00\uc5d0\uc11c \uc778\ud134\uc73c\ub85c \uadfc\ubb34\ud588\ub2e4. \uc774\ud6c4 2008\ub144\ubd80\ud130 2010\ub144\uae4c\uc9c0 \uac15\uc6d0\ub300\ud559\uad50\uc5d0\uc11c \uacbd\uc81c\ud559\uacfc \uc11d\uc0ac\uacfc\uc815\uc744 \ub9c8\uccd0 \uacbd\uc81c\ud559 \uc11d\uc0ac\ud559\uc704(\ud559\uc704\ub17c\ubb38\uba85 - Oil Shocks and the World Rice Market Puzzle: a VAR Analysis)\ub97c \ucde8\ub4dd\ud588\uace0, 2010\ub144\uc5d0\ub294 \ud55c\uad6d\uc870\uc138\uc5f0\uad6c\uc6d0\uc5d0\uc11c \ub300\uc678 \ud611\ub825 \uc5c5\ubb34\ub97c \ub2f4\ub2f9\ud558\uba74\uc11c \ud559\ud68c \ubc0f \uad50\ub958 \ud65c\ub3d9\uc744 \uc9c0\uc6d0\ud558\uae30\ub3c4 \ud588\ub2e4. 2011\ub144\ubd80\ud130 2013\ub144\uc5d0\ub294 \uc774\ud0c8\ub9ac\uc544\uc758 \ub9e5\uc8fc \ud68c\uc0ac\uc778 SAB \ubc00\ub7ec\uc5d0\uc11c \uc77c\ud558\uba70, \ub300\ud55c\ubbfc\uad6d\uc5d0 \uc774\ud0c8\ub9ac\uc544 \ub9e5\uc8fc\ub97c \ub860\uce6d\ud558\ub294 \uc77c\ub3c4 \ub2f4\ub2f9\ud588\ub2e4. 2013\ub144\ubd80\ud130 \uc8fc\ud55c \uc774\ud0c8\ub9ac\uc544 \uc0c1\uacf5\ud68c\uc758\uc18c\uc758 \uc774\uc0ac\ud68c\uc6d0\uc73c\ub85c \uc77c\ud558\uace0 \uc788\uc73c\uba70, \ub610 2013\ub144 8\uc6d4\uc5d0 \ud53c\uc544\ud2b8 \ud06c\ub77c\uc774\uc2ac\ub7ec\uc5d0 \uc785\uc0ac\ud55c \ud6c4, \ud68c\uc0ac\uc5d0\uc11c \ub51c\ub7ec \ubc0f \uc804\uc2dc\uc7a5 \uc601\uc5c5 \uc0ac\uc6d0\uc744 \uad00\ub9ac\ud558\ub294 \uc9c1\ubb34\ub97c \ub9e1\uace0 \uc788\ub2e4."} {"question": "\uc81c19\ub300 \ub300\ud1b5\ub839 \uc120\uac70\uc5d0\uc11c \ubc14\ub978\uc815\ub2f9\uc758 \ud6c4\ubcf4\ub85c \uc785\ud6c4\ubcf4\ud574 \uc720\uc2b9\ubbfc \uc758\uc6d0\uacfc 2\ud30c\uc804\uc744 \ud615\uc131\ud588\uc73c\ub098 \ud0c8\ub77d\ud55c \ub2f9\uc2dc \uacbd\uae30\ub3c4\uc9c0\uc0ac\uc758 \uc774\ub984\uc740?", "paragraph": "\ubc14\ub978\uc815\ub2f9\uc740 2017\ub144 3\uc6d4 16\uc77c\ubd80\ud130 3\uc6d4 17\uc77c\uae4c\uc9c0 \ub300\ud1b5\ub839 \uc120\uac70 \uc608\ube44\ud6c4\ubcf4 \ub4f1\ub85d\uc744 \uc9c4\ud589\ud558\uc600\ub2e4. \ucd5c\uc885\uc801\uc73c\ub85c \uc720\uc2b9\ubbfc \uc758\uc6d0\uacfc \ub0a8\uacbd\ud544 \uacbd\uae30\ub3c4\uc9c0\uc0ac\uac00 \uc785\ud6c4\ubcf4\ud558\uc5ec 2\ud30c\uc804\uc774 \ub418\uc5c8\ub2e4. \uc774\ud6c4 \ud6c4\ubcf4\ud1a0\ub860\ud68c \uc77c\uc815\uc744 \uac1c\uc2dc\ud558\uc5ec, 3\uc6d4 18\uc77c \uad11\uc8fc\uc5d0\uc11c \ud638\ub0a8\uad8c \ud1a0\ub860\ud68c, 3\uc6d4 21\uc77c\uc5d0\ub294 \ubd80\uc0b0\uc5d0\uc11c \uc601\ub0a8\uad8c \ud1a0\ub860\ud68c\ub97c \uac1c\ucd5c\ud558\uc600\ub2e4. 3\uc6d4 23\uc77c\uc5d0\ub294 \ub300\uc804\uc5d0\uc11c \ucda9\uccad\u00b7\uac15\uc6d0\uad8c \ud1a0\ub860\ud68c\ub97c, 3\uc6d4 25\uc77c\uc5d0\ub294 \uc11c\uc6b8\uc5d0\uc11c \uc218\ub3c4\uad8c \ud1a0\ub860\ud68c\ub97c \uc9c4\ud589\ud558\uc600\ub2e4. \ud638\ub0a8\uad8c \ud1a0\ub860\ud68c\uc640 \uc218\ub3c4\uad8c \ud1a0\ub860\ud68c\ub294 \uac01\uac01 \uad11\uc8fcMBC\uc640 KBS\uac00 \ud154\ub808\ube44\uc804 \ub179\ud654\u00b7\uc0dd\uc911\uacc4\ub97c \uc9c4\ud589\ud558\uc600\ub2e4. \uc774\ud6c4 2017\ub144 3\uc6d4 28\uc77c, \ubc14\ub978\uc815\ub2f9\uc740 \uad6d\ubbfc\uc815\ucc45\ud3c9\uac00\ub2e8 \uc804\ud654\uba74\uc811\ud22c\ud45c 40%, \uc804\ub2f9\uc6d0 \ud22c\ud45c 30%, \uc77c\ubc18\uad6d\ubbfc \uc5ec\ub860\uc870\uc0ac 30%\ub97c \ubc18\uc601\ud55c \uacbd\uc120 \ud22c\ud45c \uacb0\uacfc, \uc720\uc2b9\ubbfc \ubc14\ub978\uc815\ub2f9 \uc0c1\uc784\uace0\ubb38\uc774 36,593\ud45c (62.9%)\ub97c \ucc28\uc9c0\ud574 \ub0a8\uacbd\ud544 \uacbd\uae30\ub3c4\uc9c0\uc0ac (21,625\ud45c, 37.1%)\ub97c \ub204\ub974\uace0 \ub300\ud55c\ubbfc\uad6d \uc81c19\ub300 \ub300\ud1b5\ub839 \uc120\uac70 \ud6c4\ubcf4\uc5d0 \uc120\ucd9c\ub418\uc5c8\ub2e4. \uc720\uc2b9\ubbfc \ud6c4\ubcf4\ub294 \uc218\ub77d\uc5f0\uc124\uc5d0\uc11c \"\uc0c8\ub85c\uc6b4 \ubcf4\uc218\uc758 \ud76c\ub9dd\uc774 \ub418\uaca0\ub2e4\"\uba74\uc11c \"\ubcf4\uc218\uc758 \uc7ac\uac74\uc744 \ubc14\ub77c\ub294 \uad6d\ubbfc\uc758 \uc5ec\ub9dd\uc744 \ubaa8\uc544 \uc774\ubc88 \ub300\ud1b5\ub839\uc120\uac70\uc5d0\uc11c \ub2f9\ub2f9\ud558\uac8c \uad6d\ubbfc\uc758 \uc120\ud0dd\uc744 \ubc1b\uaca0\ub2e4\"\uace0 \ubc1d\ud614\ub2e4.", "answer": "\ub0a8\uacbd\ud544", "sentence": "\ucd5c\uc885\uc801\uc73c\ub85c \uc720\uc2b9\ubbfc \uc758\uc6d0\uacfc \ub0a8\uacbd\ud544 \uacbd\uae30\ub3c4\uc9c0\uc0ac\uac00 \uc785\ud6c4\ubcf4\ud558\uc5ec 2\ud30c\uc804\uc774 \ub418\uc5c8\ub2e4.", "paragraph_sentence": "\ubc14\ub978\uc815\ub2f9\uc740 2017\ub144 3\uc6d4 16\uc77c\ubd80\ud130 3\uc6d4 17\uc77c\uae4c\uc9c0 \ub300\ud1b5\ub839 \uc120\uac70 \uc608\ube44\ud6c4\ubcf4 \ub4f1\ub85d\uc744 \uc9c4\ud589\ud558\uc600\ub2e4. \ucd5c\uc885\uc801\uc73c\ub85c \uc720\uc2b9\ubbfc \uc758\uc6d0\uacfc \ub0a8\uacbd\ud544 \uacbd\uae30\ub3c4\uc9c0\uc0ac\uac00 \uc785\ud6c4\ubcf4\ud558\uc5ec 2\ud30c\uc804\uc774 \ub418\uc5c8\ub2e4. \uc774\ud6c4 \ud6c4\ubcf4\ud1a0\ub860\ud68c \uc77c\uc815\uc744 \uac1c\uc2dc\ud558\uc5ec, 3\uc6d4 18\uc77c \uad11\uc8fc\uc5d0\uc11c \ud638\ub0a8\uad8c \ud1a0\ub860\ud68c, 3\uc6d4 21\uc77c\uc5d0\ub294 \ubd80\uc0b0\uc5d0\uc11c \uc601\ub0a8\uad8c \ud1a0\ub860\ud68c\ub97c \uac1c\ucd5c\ud558\uc600\ub2e4. 3\uc6d4 23\uc77c\uc5d0\ub294 \ub300\uc804\uc5d0\uc11c \ucda9\uccad\u00b7\uac15\uc6d0\uad8c \ud1a0\ub860\ud68c\ub97c, 3\uc6d4 25\uc77c\uc5d0\ub294 \uc11c\uc6b8\uc5d0\uc11c \uc218\ub3c4\uad8c \ud1a0\ub860\ud68c\ub97c \uc9c4\ud589\ud558\uc600\ub2e4. \ud638\ub0a8\uad8c \ud1a0\ub860\ud68c\uc640 \uc218\ub3c4\uad8c \ud1a0\ub860\ud68c\ub294 \uac01\uac01 \uad11\uc8fcMBC\uc640 KBS\uac00 \ud154\ub808\ube44\uc804 \ub179\ud654\u00b7\uc0dd\uc911\uacc4\ub97c \uc9c4\ud589\ud558\uc600\ub2e4. \uc774\ud6c4 2017\ub144 3\uc6d4 28\uc77c, \ubc14\ub978\uc815\ub2f9\uc740 \uad6d\ubbfc\uc815\ucc45\ud3c9\uac00\ub2e8 \uc804\ud654\uba74\uc811\ud22c\ud45c 40%, \uc804\ub2f9\uc6d0 \ud22c\ud45c 30%, \uc77c\ubc18\uad6d\ubbfc \uc5ec\ub860\uc870\uc0ac 30%\ub97c \ubc18\uc601\ud55c \uacbd\uc120 \ud22c\ud45c \uacb0\uacfc, \uc720\uc2b9\ubbfc \ubc14\ub978\uc815\ub2f9 \uc0c1\uc784\uace0\ubb38\uc774 36,593\ud45c (62.9%)\ub97c \ucc28\uc9c0\ud574 \ub0a8\uacbd\ud544 \uacbd\uae30\ub3c4\uc9c0\uc0ac (21,625\ud45c, 37.1%)\ub97c \ub204\ub974\uace0 \ub300\ud55c\ubbfc\uad6d \uc81c19\ub300 \ub300\ud1b5\ub839 \uc120\uac70 \ud6c4\ubcf4\uc5d0 \uc120\ucd9c\ub418\uc5c8\ub2e4. \uc720\uc2b9\ubbfc \ud6c4\ubcf4\ub294 \uc218\ub77d\uc5f0\uc124\uc5d0\uc11c \"\uc0c8\ub85c\uc6b4 \ubcf4\uc218\uc758 \ud76c\ub9dd\uc774 \ub418\uaca0\ub2e4\"\uba74\uc11c \"\ubcf4\uc218\uc758 \uc7ac\uac74\uc744 \ubc14\ub77c\ub294 \uad6d\ubbfc\uc758 \uc5ec\ub9dd\uc744 \ubaa8\uc544 \uc774\ubc88 \ub300\ud1b5\ub839\uc120\uac70\uc5d0\uc11c \ub2f9\ub2f9\ud558\uac8c \uad6d\ubbfc\uc758 \uc120\ud0dd\uc744 \ubc1b\uaca0\ub2e4\"\uace0 \ubc1d\ud614\ub2e4.", "paragraph_answer": "\ubc14\ub978\uc815\ub2f9\uc740 2017\ub144 3\uc6d4 16\uc77c\ubd80\ud130 3\uc6d4 17\uc77c\uae4c\uc9c0 \ub300\ud1b5\ub839 \uc120\uac70 \uc608\ube44\ud6c4\ubcf4 \ub4f1\ub85d\uc744 \uc9c4\ud589\ud558\uc600\ub2e4. \ucd5c\uc885\uc801\uc73c\ub85c \uc720\uc2b9\ubbfc \uc758\uc6d0\uacfc \ub0a8\uacbd\ud544 \uacbd\uae30\ub3c4\uc9c0\uc0ac\uac00 \uc785\ud6c4\ubcf4\ud558\uc5ec 2\ud30c\uc804\uc774 \ub418\uc5c8\ub2e4. \uc774\ud6c4 \ud6c4\ubcf4\ud1a0\ub860\ud68c \uc77c\uc815\uc744 \uac1c\uc2dc\ud558\uc5ec, 3\uc6d4 18\uc77c \uad11\uc8fc\uc5d0\uc11c \ud638\ub0a8\uad8c \ud1a0\ub860\ud68c, 3\uc6d4 21\uc77c\uc5d0\ub294 \ubd80\uc0b0\uc5d0\uc11c \uc601\ub0a8\uad8c \ud1a0\ub860\ud68c\ub97c \uac1c\ucd5c\ud558\uc600\ub2e4. 3\uc6d4 23\uc77c\uc5d0\ub294 \ub300\uc804\uc5d0\uc11c \ucda9\uccad\u00b7\uac15\uc6d0\uad8c \ud1a0\ub860\ud68c\ub97c, 3\uc6d4 25\uc77c\uc5d0\ub294 \uc11c\uc6b8\uc5d0\uc11c \uc218\ub3c4\uad8c \ud1a0\ub860\ud68c\ub97c \uc9c4\ud589\ud558\uc600\ub2e4. \ud638\ub0a8\uad8c \ud1a0\ub860\ud68c\uc640 \uc218\ub3c4\uad8c \ud1a0\ub860\ud68c\ub294 \uac01\uac01 \uad11\uc8fcMBC\uc640 KBS\uac00 \ud154\ub808\ube44\uc804 \ub179\ud654\u00b7\uc0dd\uc911\uacc4\ub97c \uc9c4\ud589\ud558\uc600\ub2e4. \uc774\ud6c4 2017\ub144 3\uc6d4 28\uc77c, \ubc14\ub978\uc815\ub2f9\uc740 \uad6d\ubbfc\uc815\ucc45\ud3c9\uac00\ub2e8 \uc804\ud654\uba74\uc811\ud22c\ud45c 40%, \uc804\ub2f9\uc6d0 \ud22c\ud45c 30%, \uc77c\ubc18\uad6d\ubbfc \uc5ec\ub860\uc870\uc0ac 30%\ub97c \ubc18\uc601\ud55c \uacbd\uc120 \ud22c\ud45c \uacb0\uacfc, \uc720\uc2b9\ubbfc \ubc14\ub978\uc815\ub2f9 \uc0c1\uc784\uace0\ubb38\uc774 36,593\ud45c (62.9%)\ub97c \ucc28\uc9c0\ud574 \ub0a8\uacbd\ud544 \uacbd\uae30\ub3c4\uc9c0\uc0ac (21,625\ud45c, 37.1%)\ub97c \ub204\ub974\uace0 \ub300\ud55c\ubbfc\uad6d \uc81c19\ub300 \ub300\ud1b5\ub839 \uc120\uac70 \ud6c4\ubcf4\uc5d0 \uc120\ucd9c\ub418\uc5c8\ub2e4. \uc720\uc2b9\ubbfc \ud6c4\ubcf4\ub294 \uc218\ub77d\uc5f0\uc124\uc5d0\uc11c \"\uc0c8\ub85c\uc6b4 \ubcf4\uc218\uc758 \ud76c\ub9dd\uc774 \ub418\uaca0\ub2e4\"\uba74\uc11c \"\ubcf4\uc218\uc758 \uc7ac\uac74\uc744 \ubc14\ub77c\ub294 \uad6d\ubbfc\uc758 \uc5ec\ub9dd\uc744 \ubaa8\uc544 \uc774\ubc88 \ub300\ud1b5\ub839\uc120\uac70\uc5d0\uc11c \ub2f9\ub2f9\ud558\uac8c \uad6d\ubbfc\uc758 \uc120\ud0dd\uc744 \ubc1b\uaca0\ub2e4\"\uace0 \ubc1d\ud614\ub2e4.", "sentence_answer": "\ucd5c\uc885\uc801\uc73c\ub85c \uc720\uc2b9\ubbfc \uc758\uc6d0\uacfc \ub0a8\uacbd\ud544 \uacbd\uae30\ub3c4\uc9c0\uc0ac\uac00 \uc785\ud6c4\ubcf4\ud558\uc5ec 2\ud30c\uc804\uc774 \ub418\uc5c8\ub2e4.", "paragraph_id": "6490942-4-1", "paragraph_question": "question: \uc81c19\ub300 \ub300\ud1b5\ub839 \uc120\uac70\uc5d0\uc11c \ubc14\ub978\uc815\ub2f9\uc758 \ud6c4\ubcf4\ub85c \uc785\ud6c4\ubcf4\ud574 \uc720\uc2b9\ubbfc \uc758\uc6d0\uacfc 2\ud30c\uc804\uc744 \ud615\uc131\ud588\uc73c\ub098 \ud0c8\ub77d\ud55c \ub2f9\uc2dc \uacbd\uae30\ub3c4\uc9c0\uc0ac\uc758 \uc774\ub984\uc740?, context: \ubc14\ub978\uc815\ub2f9\uc740 2017\ub144 3\uc6d4 16\uc77c\ubd80\ud130 3\uc6d4 17\uc77c\uae4c\uc9c0 \ub300\ud1b5\ub839 \uc120\uac70 \uc608\ube44\ud6c4\ubcf4 \ub4f1\ub85d\uc744 \uc9c4\ud589\ud558\uc600\ub2e4. \ucd5c\uc885\uc801\uc73c\ub85c \uc720\uc2b9\ubbfc \uc758\uc6d0\uacfc \ub0a8\uacbd\ud544 \uacbd\uae30\ub3c4\uc9c0\uc0ac\uac00 \uc785\ud6c4\ubcf4\ud558\uc5ec 2\ud30c\uc804\uc774 \ub418\uc5c8\ub2e4. \uc774\ud6c4 \ud6c4\ubcf4\ud1a0\ub860\ud68c \uc77c\uc815\uc744 \uac1c\uc2dc\ud558\uc5ec, 3\uc6d4 18\uc77c \uad11\uc8fc\uc5d0\uc11c \ud638\ub0a8\uad8c \ud1a0\ub860\ud68c, 3\uc6d4 21\uc77c\uc5d0\ub294 \ubd80\uc0b0\uc5d0\uc11c \uc601\ub0a8\uad8c \ud1a0\ub860\ud68c\ub97c \uac1c\ucd5c\ud558\uc600\ub2e4. 3\uc6d4 23\uc77c\uc5d0\ub294 \ub300\uc804\uc5d0\uc11c \ucda9\uccad\u00b7\uac15\uc6d0\uad8c \ud1a0\ub860\ud68c\ub97c, 3\uc6d4 25\uc77c\uc5d0\ub294 \uc11c\uc6b8\uc5d0\uc11c \uc218\ub3c4\uad8c \ud1a0\ub860\ud68c\ub97c \uc9c4\ud589\ud558\uc600\ub2e4. \ud638\ub0a8\uad8c \ud1a0\ub860\ud68c\uc640 \uc218\ub3c4\uad8c \ud1a0\ub860\ud68c\ub294 \uac01\uac01 \uad11\uc8fcMBC\uc640 KBS\uac00 \ud154\ub808\ube44\uc804 \ub179\ud654\u00b7\uc0dd\uc911\uacc4\ub97c \uc9c4\ud589\ud558\uc600\ub2e4. \uc774\ud6c4 2017\ub144 3\uc6d4 28\uc77c, \ubc14\ub978\uc815\ub2f9\uc740 \uad6d\ubbfc\uc815\ucc45\ud3c9\uac00\ub2e8 \uc804\ud654\uba74\uc811\ud22c\ud45c 40%, \uc804\ub2f9\uc6d0 \ud22c\ud45c 30%, \uc77c\ubc18\uad6d\ubbfc \uc5ec\ub860\uc870\uc0ac 30%\ub97c \ubc18\uc601\ud55c \uacbd\uc120 \ud22c\ud45c \uacb0\uacfc, \uc720\uc2b9\ubbfc \ubc14\ub978\uc815\ub2f9 \uc0c1\uc784\uace0\ubb38\uc774 36,593\ud45c (62.9%)\ub97c \ucc28\uc9c0\ud574 \ub0a8\uacbd\ud544 \uacbd\uae30\ub3c4\uc9c0\uc0ac (21,625\ud45c, 37.1%)\ub97c \ub204\ub974\uace0 \ub300\ud55c\ubbfc\uad6d \uc81c19\ub300 \ub300\ud1b5\ub839 \uc120\uac70 \ud6c4\ubcf4\uc5d0 \uc120\ucd9c\ub418\uc5c8\ub2e4. \uc720\uc2b9\ubbfc \ud6c4\ubcf4\ub294 \uc218\ub77d\uc5f0\uc124\uc5d0\uc11c \"\uc0c8\ub85c\uc6b4 \ubcf4\uc218\uc758 \ud76c\ub9dd\uc774 \ub418\uaca0\ub2e4\"\uba74\uc11c \"\ubcf4\uc218\uc758 \uc7ac\uac74\uc744 \ubc14\ub77c\ub294 \uad6d\ubbfc\uc758 \uc5ec\ub9dd\uc744 \ubaa8\uc544 \uc774\ubc88 \ub300\ud1b5\ub839\uc120\uac70\uc5d0\uc11c \ub2f9\ub2f9\ud558\uac8c \uad6d\ubbfc\uc758 \uc120\ud0dd\uc744 \ubc1b\uaca0\ub2e4\"\uace0 \ubc1d\ud614\ub2e4."} {"question": "\uc544\ud504\ub9ac\uce74\uc758 \uc8fc\uc694 \uc218\uc1a1\ud1b5\ub85c\ub294?", "paragraph": "\ud55c\ud3b8, \ucca0\ub3c4\uc640 \uc218\uc6b4(\u6c34\u904b)\uc758 \uc774\uc6a9\uc774 \ud55c\uc815\ub41c \uc544\ud504\ub9ac\uce74\uc5d0\uc11c\ub294 \ub3c4\ub85c\uac00 \ub9e4\uc6b0 \uc911\uc694\ud55c \uc5ed\ud560\uc744 \ub2f4\ub2f9\ud558\uace0 \uc788\ub2e4. \ub18d\uc0b0\ud488\uc758 \uc9d1\ud558\u00b7\uc6b4\uc1a1\uacfc \uc18c\ube44\ubb3c\uc790 \ubc0f \uc2b9\uac1d\uc758 \uc218\uc1a1\uc5d0 \ud3b8\ub9ac\ud55c \uc18c\ud615\ud2b8\ub7ed\uc744 \ub9ce\uc774 \uc774\uc6a9\ud558\uace0 \uc788\uc9c0\ub9cc, \ub3c4\ub85c\ud3ec\uc7a5\ub960\uc774 \ub0ae\uace0 \ub3c4\ub85c\uc758 \uc720\uc9c0\u00b7\ubcf4\uc218 \ub4f1\uc774 \uc644\uc804\ud558\uc9c0 \ubabb\ud558\ub2e4. \uc774\ub7ec\ud55c \ubb38\uc81c\uc810\uc744 \ud574\uacb0\ud558\uae30 \uc704\ud574 UN\uc758 \uc544\ud504\ub9ac\uce74\uacbd\uc81c\uc704\uc6d0\ud68c\ub294 \ubab8\ubc14\uc0ac\uc640 \ub77c\uace0\uc2a4\ub97c \uc787\ub294 \uc0ac\ud558\ub77c\ud6a1\ub2e8\uace0\uc18d\ub3c4\ub85c\uc640 \uc11c\uc544\ud504\ub9ac\uce74\uc758 \ub77c\uace0\uc2a4\uc5d0\uc11c \ub2e4\uce74\ub974\ub97c \uc787\ub294 \uace0\uc18d\ub3c4\ub85c\uc758 \uac74\uc124\uc744 \uacc4\ud68d\ud558\uace0 \uc788\ub2e4.\ub3c4\uc2dc\uac04(\u9593)\uc758 \uad50\ud1b5\uc740 \uc790\ub3d9\ucc28\ub97c \uc774\uc6a9\ud558\uace0 \uc788\uc73c\ub098, \uc9c0\uc5ed\uac04 \uc6b4\ubc18 \uad50\ud1b5\uc73c\ub85c\ub294 \ubd81\ubd80\uc9c0\ubc29\uc5d0\uc11c\ub294 \ub099\ud0c0, \uc911\ubd80\uc9c0\ubc29\uc5d0\uc11c\ub294 \ud751\uc778, \ub0a8\ubd80\uc5d0\uc11c\ub294 \uc18c\ub97c \ub9ce\uc774 \uc774\uc6a9\ud55c\ub2e4. \ub3c4\ub85c\ub9dd\uc758 \ubc1c\ub2ec\uc740 \uc2dd\ubbfc\uc9c0 \ud1b5\uce58\ub97c \uc704\ud55c \uc815\uce58\uc801\u00b7\uad70\uc0ac\uc801 \ud544\uc694\uc131\uc5d0\uc11c \uc2dc\uc791\ub418\uc5c8\uc73c\uba70, \uacbd\uc81c \uc0b0\uc5c5\uc0c1\uc758 \ubaa9\uc801\uc740 \uadf8 \ub2e4\uc74c\uc774\uc5c8\ub2e4. \ub530\ub77c\uc11c \ub3c5\ub9bd\uc774 \ub41c \uc9c0\uae08\uc740 \ub3c4\ub85c\uac00 \ud06c\uac8c \ubd80\uc871\ud558\ub2e4.", "answer": "\ub3c4\ub85c", "sentence": "\ucca0\ub3c4\uc640 \uc218\uc6b4(\u6c34\u904b)\uc758 \uc774\uc6a9\uc774 \ud55c\uc815\ub41c \uc544\ud504\ub9ac\uce74\uc5d0\uc11c\ub294 \ub3c4\ub85c \uac00 \ub9e4\uc6b0 \uc911\uc694\ud55c \uc5ed\ud560\uc744 \ub2f4\ub2f9\ud558\uace0 \uc788\ub2e4.", "paragraph_sentence": "\ud55c\ud3b8, \ucca0\ub3c4\uc640 \uc218\uc6b4(\u6c34\u904b)\uc758 \uc774\uc6a9\uc774 \ud55c\uc815\ub41c \uc544\ud504\ub9ac\uce74\uc5d0\uc11c\ub294 \ub3c4\ub85c \uac00 \ub9e4\uc6b0 \uc911\uc694\ud55c \uc5ed\ud560\uc744 \ub2f4\ub2f9\ud558\uace0 \uc788\ub2e4. \ub18d\uc0b0\ud488\uc758 \uc9d1\ud558\u00b7\uc6b4\uc1a1\uacfc \uc18c\ube44\ubb3c\uc790 \ubc0f \uc2b9\uac1d\uc758 \uc218\uc1a1\uc5d0 \ud3b8\ub9ac\ud55c \uc18c\ud615\ud2b8\ub7ed\uc744 \ub9ce\uc774 \uc774\uc6a9\ud558\uace0 \uc788\uc9c0\ub9cc, \ub3c4\ub85c\ud3ec\uc7a5\ub960\uc774 \ub0ae\uace0 \ub3c4\ub85c\uc758 \uc720\uc9c0\u00b7\ubcf4\uc218 \ub4f1\uc774 \uc644\uc804\ud558\uc9c0 \ubabb\ud558\ub2e4. \uc774\ub7ec\ud55c \ubb38\uc81c\uc810\uc744 \ud574\uacb0\ud558\uae30 \uc704\ud574 UN\uc758 \uc544\ud504\ub9ac\uce74\uacbd\uc81c\uc704\uc6d0\ud68c\ub294 \ubab8\ubc14\uc0ac\uc640 \ub77c\uace0\uc2a4\ub97c \uc787\ub294 \uc0ac\ud558\ub77c\ud6a1\ub2e8\uace0\uc18d\ub3c4\ub85c\uc640 \uc11c\uc544\ud504\ub9ac\uce74\uc758 \ub77c\uace0\uc2a4\uc5d0\uc11c \ub2e4\uce74\ub974\ub97c \uc787\ub294 \uace0\uc18d\ub3c4\ub85c\uc758 \uac74\uc124\uc744 \uacc4\ud68d\ud558\uace0 \uc788\ub2e4. \ub3c4\uc2dc\uac04(\u9593)\uc758 \uad50\ud1b5\uc740 \uc790\ub3d9\ucc28\ub97c \uc774\uc6a9\ud558\uace0 \uc788\uc73c\ub098, \uc9c0\uc5ed\uac04 \uc6b4\ubc18 \uad50\ud1b5\uc73c\ub85c\ub294 \ubd81\ubd80\uc9c0\ubc29\uc5d0\uc11c\ub294 \ub099\ud0c0, \uc911\ubd80\uc9c0\ubc29\uc5d0\uc11c\ub294 \ud751\uc778, \ub0a8\ubd80\uc5d0\uc11c\ub294 \uc18c\ub97c \ub9ce\uc774 \uc774\uc6a9\ud55c\ub2e4. \ub3c4\ub85c\ub9dd\uc758 \ubc1c\ub2ec\uc740 \uc2dd\ubbfc\uc9c0 \ud1b5\uce58\ub97c \uc704\ud55c \uc815\uce58\uc801\u00b7\uad70\uc0ac\uc801 \ud544\uc694\uc131\uc5d0\uc11c \uc2dc\uc791\ub418\uc5c8\uc73c\uba70, \uacbd\uc81c \uc0b0\uc5c5\uc0c1\uc758 \ubaa9\uc801\uc740 \uadf8 \ub2e4\uc74c\uc774\uc5c8\ub2e4. \ub530\ub77c\uc11c \ub3c5\ub9bd\uc774 \ub41c \uc9c0\uae08\uc740 \ub3c4\ub85c\uac00 \ud06c\uac8c \ubd80\uc871\ud558\ub2e4.", "paragraph_answer": "\ud55c\ud3b8, \ucca0\ub3c4\uc640 \uc218\uc6b4(\u6c34\u904b)\uc758 \uc774\uc6a9\uc774 \ud55c\uc815\ub41c \uc544\ud504\ub9ac\uce74\uc5d0\uc11c\ub294 \ub3c4\ub85c \uac00 \ub9e4\uc6b0 \uc911\uc694\ud55c \uc5ed\ud560\uc744 \ub2f4\ub2f9\ud558\uace0 \uc788\ub2e4. \ub18d\uc0b0\ud488\uc758 \uc9d1\ud558\u00b7\uc6b4\uc1a1\uacfc \uc18c\ube44\ubb3c\uc790 \ubc0f \uc2b9\uac1d\uc758 \uc218\uc1a1\uc5d0 \ud3b8\ub9ac\ud55c \uc18c\ud615\ud2b8\ub7ed\uc744 \ub9ce\uc774 \uc774\uc6a9\ud558\uace0 \uc788\uc9c0\ub9cc, \ub3c4\ub85c\ud3ec\uc7a5\ub960\uc774 \ub0ae\uace0 \ub3c4\ub85c\uc758 \uc720\uc9c0\u00b7\ubcf4\uc218 \ub4f1\uc774 \uc644\uc804\ud558\uc9c0 \ubabb\ud558\ub2e4. \uc774\ub7ec\ud55c \ubb38\uc81c\uc810\uc744 \ud574\uacb0\ud558\uae30 \uc704\ud574 UN\uc758 \uc544\ud504\ub9ac\uce74\uacbd\uc81c\uc704\uc6d0\ud68c\ub294 \ubab8\ubc14\uc0ac\uc640 \ub77c\uace0\uc2a4\ub97c \uc787\ub294 \uc0ac\ud558\ub77c\ud6a1\ub2e8\uace0\uc18d\ub3c4\ub85c\uc640 \uc11c\uc544\ud504\ub9ac\uce74\uc758 \ub77c\uace0\uc2a4\uc5d0\uc11c \ub2e4\uce74\ub974\ub97c \uc787\ub294 \uace0\uc18d\ub3c4\ub85c\uc758 \uac74\uc124\uc744 \uacc4\ud68d\ud558\uace0 \uc788\ub2e4.\ub3c4\uc2dc\uac04(\u9593)\uc758 \uad50\ud1b5\uc740 \uc790\ub3d9\ucc28\ub97c \uc774\uc6a9\ud558\uace0 \uc788\uc73c\ub098, \uc9c0\uc5ed\uac04 \uc6b4\ubc18 \uad50\ud1b5\uc73c\ub85c\ub294 \ubd81\ubd80\uc9c0\ubc29\uc5d0\uc11c\ub294 \ub099\ud0c0, \uc911\ubd80\uc9c0\ubc29\uc5d0\uc11c\ub294 \ud751\uc778, \ub0a8\ubd80\uc5d0\uc11c\ub294 \uc18c\ub97c \ub9ce\uc774 \uc774\uc6a9\ud55c\ub2e4. \ub3c4\ub85c\ub9dd\uc758 \ubc1c\ub2ec\uc740 \uc2dd\ubbfc\uc9c0 \ud1b5\uce58\ub97c \uc704\ud55c \uc815\uce58\uc801\u00b7\uad70\uc0ac\uc801 \ud544\uc694\uc131\uc5d0\uc11c \uc2dc\uc791\ub418\uc5c8\uc73c\uba70, \uacbd\uc81c \uc0b0\uc5c5\uc0c1\uc758 \ubaa9\uc801\uc740 \uadf8 \ub2e4\uc74c\uc774\uc5c8\ub2e4. \ub530\ub77c\uc11c \ub3c5\ub9bd\uc774 \ub41c \uc9c0\uae08\uc740 \ub3c4\ub85c\uac00 \ud06c\uac8c \ubd80\uc871\ud558\ub2e4.", "sentence_answer": "\ucca0\ub3c4\uc640 \uc218\uc6b4(\u6c34\u904b)\uc758 \uc774\uc6a9\uc774 \ud55c\uc815\ub41c \uc544\ud504\ub9ac\uce74\uc5d0\uc11c\ub294 \ub3c4\ub85c \uac00 \ub9e4\uc6b0 \uc911\uc694\ud55c \uc5ed\ud560\uc744 \ub2f4\ub2f9\ud558\uace0 \uc788\ub2e4.", "paragraph_id": "6538810-8-0", "paragraph_question": "question: \uc544\ud504\ub9ac\uce74\uc758 \uc8fc\uc694 \uc218\uc1a1\ud1b5\ub85c\ub294?, context: \ud55c\ud3b8, \ucca0\ub3c4\uc640 \uc218\uc6b4(\u6c34\u904b)\uc758 \uc774\uc6a9\uc774 \ud55c\uc815\ub41c \uc544\ud504\ub9ac\uce74\uc5d0\uc11c\ub294 \ub3c4\ub85c\uac00 \ub9e4\uc6b0 \uc911\uc694\ud55c \uc5ed\ud560\uc744 \ub2f4\ub2f9\ud558\uace0 \uc788\ub2e4. \ub18d\uc0b0\ud488\uc758 \uc9d1\ud558\u00b7\uc6b4\uc1a1\uacfc \uc18c\ube44\ubb3c\uc790 \ubc0f \uc2b9\uac1d\uc758 \uc218\uc1a1\uc5d0 \ud3b8\ub9ac\ud55c \uc18c\ud615\ud2b8\ub7ed\uc744 \ub9ce\uc774 \uc774\uc6a9\ud558\uace0 \uc788\uc9c0\ub9cc, \ub3c4\ub85c\ud3ec\uc7a5\ub960\uc774 \ub0ae\uace0 \ub3c4\ub85c\uc758 \uc720\uc9c0\u00b7\ubcf4\uc218 \ub4f1\uc774 \uc644\uc804\ud558\uc9c0 \ubabb\ud558\ub2e4. \uc774\ub7ec\ud55c \ubb38\uc81c\uc810\uc744 \ud574\uacb0\ud558\uae30 \uc704\ud574 UN\uc758 \uc544\ud504\ub9ac\uce74\uacbd\uc81c\uc704\uc6d0\ud68c\ub294 \ubab8\ubc14\uc0ac\uc640 \ub77c\uace0\uc2a4\ub97c \uc787\ub294 \uc0ac\ud558\ub77c\ud6a1\ub2e8\uace0\uc18d\ub3c4\ub85c\uc640 \uc11c\uc544\ud504\ub9ac\uce74\uc758 \ub77c\uace0\uc2a4\uc5d0\uc11c \ub2e4\uce74\ub974\ub97c \uc787\ub294 \uace0\uc18d\ub3c4\ub85c\uc758 \uac74\uc124\uc744 \uacc4\ud68d\ud558\uace0 \uc788\ub2e4.\ub3c4\uc2dc\uac04(\u9593)\uc758 \uad50\ud1b5\uc740 \uc790\ub3d9\ucc28\ub97c \uc774\uc6a9\ud558\uace0 \uc788\uc73c\ub098, \uc9c0\uc5ed\uac04 \uc6b4\ubc18 \uad50\ud1b5\uc73c\ub85c\ub294 \ubd81\ubd80\uc9c0\ubc29\uc5d0\uc11c\ub294 \ub099\ud0c0, \uc911\ubd80\uc9c0\ubc29\uc5d0\uc11c\ub294 \ud751\uc778, \ub0a8\ubd80\uc5d0\uc11c\ub294 \uc18c\ub97c \ub9ce\uc774 \uc774\uc6a9\ud55c\ub2e4. \ub3c4\ub85c\ub9dd\uc758 \ubc1c\ub2ec\uc740 \uc2dd\ubbfc\uc9c0 \ud1b5\uce58\ub97c \uc704\ud55c \uc815\uce58\uc801\u00b7\uad70\uc0ac\uc801 \ud544\uc694\uc131\uc5d0\uc11c \uc2dc\uc791\ub418\uc5c8\uc73c\uba70, \uacbd\uc81c \uc0b0\uc5c5\uc0c1\uc758 \ubaa9\uc801\uc740 \uadf8 \ub2e4\uc74c\uc774\uc5c8\ub2e4. \ub530\ub77c\uc11c \ub3c5\ub9bd\uc774 \ub41c \uc9c0\uae08\uc740 \ub3c4\ub85c\uac00 \ud06c\uac8c \ubd80\uc871\ud558\ub2e4."} {"question": "2014\ub144 FIFA \uc6d4\ub4dc\ucef5 \uc720\ub7fd \uc9c0\uc5ed \uc608\uc120\uc5d0\uc11c \ud1b1\uc2dc\ub4dc\uac00 \ub41c \uad6d\uac00\ub85c\uc368, \uc544\uc774\uc2ac\ub780\ub4dc\uc640 \uc2a4\uc704\uc2a4\uc5d0\uac8c \uc5f0\ub2ec\uc5d0 \ud328\ud55c \uad6d\uac00\ub294?", "paragraph": "\ud2b9\uc815 \ub300\ub959\uc5d0\uac8c \uc720\ub9ac\ud558\uac8c \uc8fc\uc5b4\uc9c0\ub294 FIFA \uc810\uc218\uc640 \ud2b9\uc815 \ub300\ud68c\uc5d0\ub9cc \ube44\uc911\uc774 \ub192\uc740 \ub7ad\ud0b9\uc0b0\uc815\ubc29\uc2dd\uc73c\ub85c \uc778\ud558\uc5ec \uc720\ub7fd\uc758 \uc57d\ud300\uc774 \ud0c0\ub300\ub959\uc758 \uc9c0\uc5ed\uac15\ud300\ubcf4\ub2e4 \ub354 \ub7ad\ud0b9\uc774 \ub192\uc740 \uacbd\uc6b0\ub3c4 \ubc1c\uc0dd\ud55c\ub2e4. \uc77c\ub840\ub85c \uadf8\ub9ac\uc2a4\ub294 \ub300\ud55c\ubbfc\uad6d\uacfc \ub9de\ubd99\uae30\ub9cc \ud588\ub2e4 \ud558\uba74 \ud328\ud558\ub294 \ub370\ub2e4\uac00 UEFA \uc720\ub85c 2004 \uc6b0\uc2b9 \uc774\uc678\uc5d0\ub294 \uc774\ub807\ub2e4\ud560 \uad6d\uc81c\ub300\ud68c \uc131\uc801\uc744 \uac70\ub454\uac8c \uc5c6\uc74c\uc5d0\ub3c4 \ubd88\uad6c\ud558\uace0 10\uc704\uad8c\uc778 \ubc18\uba74 \uc774\ud0c8\ub9ac\uc544\ub294 2006\ub144 FIFA \uc6d4\ub4dc\ucef5 \uc6b0\uc2b9\uacfc UEFA \uc720\ub85c 2012 \uc900\uc6b0\uc2b9\uc744 \uac70\ub450\uace0\ub3c4 \ud53c\ud30c\ub7ad\ud0b9\uc774 \ub0ae\ub2e4. \ub610\ud55c \ud53c\ud30c\ub7ad\ud0b9\uc0c1 \ub192\uac8c \ucc45\uc815\ub418\uc5b4 2014\ub144 FIFA \uc6d4\ub4dc\ucef5 \uc720\ub7fd \uc9c0\uc5ed \uc608\uc120\uc5d0\uc11c \ud1b1\uc2dc\ub4dc\uac00 \ub41c \ub178\ub974\uc6e8\uc774\ub294 \uadf8 \uc870\uc5d0\uc11c 6\ubc88\uc2dc\ub4dc\uc778 \uc544\uc774\uc2ac\ub780\ub4dc\uc640 2\ubc88 \uc2dc\ub4dc\uc778 \uc2a4\uc704\uc2a4\uc5d0\uac8c \uc5f0\ub2ec\uc544 \ud328\ud558\uace0 \ud558\uc704\uad8c\uc73c\ub85c \uc9c0\uc5ed\uc608\uc120 \ud0c8\ub77d\ud55c \uac83\ub3c4 \ubaa8\uc790\ub77c \ud53c\ud30c\ub7ad\ud0b9\uc774 \ub0ae\uc544\uc11c 2\ubc88\uc2dc\ub4dc\ub85c \ubc30\uc815\ubc1b\uc740 \ud504\ub791\uc2a4\uc640\uc758 \ud3c9\uac00\uc804\uc5d0\uc11c 0-4\ub85c \ub300\ud328\ub97c \ub2f9\ud558\ub294 \uc218\ubaa8\ub97c \uacaa\uae30\uae4c\uc9c0 \ud588\ub2e4. \uc774\ub7ec\ud55c \ubb38\uc81c\uc788\ub294 \uc0b0\uc815\ubc29\uc2dd \ub54c\ubb38\uc5d0 \ubca8\uae30\uc5d0\uac00 \uc774\ud0c8\ub9ac\uc544\ub97c \uc81c\uce58\uace0 2014\ub144 FIFA \uc6d4\ub4dc\ucef5 \ubcf8\uc120\uc5d0\uc11c \ud1b1\uc2dc\ub4dc\ub85c \ud560\ub2f9\ubc1b\uc558\ub2e4.", "answer": "\ub178\ub974\uc6e8\uc774", "sentence": "\ub610\ud55c \ud53c\ud30c\ub7ad\ud0b9\uc0c1 \ub192\uac8c \ucc45\uc815\ub418\uc5b4 2014\ub144 FIFA \uc6d4\ub4dc\ucef5 \uc720\ub7fd \uc9c0\uc5ed \uc608\uc120\uc5d0\uc11c \ud1b1\uc2dc\ub4dc\uac00 \ub41c \ub178\ub974\uc6e8\uc774 \ub294 \uadf8 \uc870\uc5d0\uc11c 6\ubc88\uc2dc\ub4dc\uc778 \uc544\uc774\uc2ac\ub780\ub4dc\uc640 2\ubc88 \uc2dc\ub4dc\uc778 \uc2a4\uc704\uc2a4\uc5d0\uac8c \uc5f0\ub2ec\uc544 \ud328\ud558\uace0 \ud558\uc704\uad8c\uc73c\ub85c \uc9c0\uc5ed\uc608\uc120 \ud0c8\ub77d\ud55c \uac83\ub3c4 \ubaa8\uc790\ub77c \ud53c\ud30c\ub7ad\ud0b9\uc774 \ub0ae\uc544\uc11c 2\ubc88\uc2dc\ub4dc\ub85c \ubc30\uc815\ubc1b\uc740 \ud504\ub791\uc2a4\uc640\uc758 \ud3c9\uac00\uc804\uc5d0\uc11c 0-4\ub85c \ub300\ud328\ub97c \ub2f9\ud558\ub294 \uc218\ubaa8\ub97c \uacaa\uae30\uae4c\uc9c0 \ud588\ub2e4.", "paragraph_sentence": "\ud2b9\uc815 \ub300\ub959\uc5d0\uac8c \uc720\ub9ac\ud558\uac8c \uc8fc\uc5b4\uc9c0\ub294 FIFA \uc810\uc218\uc640 \ud2b9\uc815 \ub300\ud68c\uc5d0\ub9cc \ube44\uc911\uc774 \ub192\uc740 \ub7ad\ud0b9\uc0b0\uc815\ubc29\uc2dd\uc73c\ub85c \uc778\ud558\uc5ec \uc720\ub7fd\uc758 \uc57d\ud300\uc774 \ud0c0\ub300\ub959\uc758 \uc9c0\uc5ed\uac15\ud300\ubcf4\ub2e4 \ub354 \ub7ad\ud0b9\uc774 \ub192\uc740 \uacbd\uc6b0\ub3c4 \ubc1c\uc0dd\ud55c\ub2e4. \uc77c\ub840\ub85c \uadf8\ub9ac\uc2a4\ub294 \ub300\ud55c\ubbfc\uad6d\uacfc \ub9de\ubd99\uae30\ub9cc \ud588\ub2e4 \ud558\uba74 \ud328\ud558\ub294 \ub370\ub2e4\uac00 UEFA \uc720\ub85c 2004 \uc6b0\uc2b9 \uc774\uc678\uc5d0\ub294 \uc774\ub807\ub2e4\ud560 \uad6d\uc81c\ub300\ud68c \uc131\uc801\uc744 \uac70\ub454\uac8c \uc5c6\uc74c\uc5d0\ub3c4 \ubd88\uad6c\ud558\uace0 10\uc704\uad8c\uc778 \ubc18\uba74 \uc774\ud0c8\ub9ac\uc544\ub294 2006\ub144 FIFA \uc6d4\ub4dc\ucef5 \uc6b0\uc2b9\uacfc UEFA \uc720\ub85c 2012 \uc900\uc6b0\uc2b9\uc744 \uac70\ub450\uace0\ub3c4 \ud53c\ud30c\ub7ad\ud0b9\uc774 \ub0ae\ub2e4. \ub610\ud55c \ud53c\ud30c\ub7ad\ud0b9\uc0c1 \ub192\uac8c \ucc45\uc815\ub418\uc5b4 2014\ub144 FIFA \uc6d4\ub4dc\ucef5 \uc720\ub7fd \uc9c0\uc5ed \uc608\uc120\uc5d0\uc11c \ud1b1\uc2dc\ub4dc\uac00 \ub41c \ub178\ub974\uc6e8\uc774 \ub294 \uadf8 \uc870\uc5d0\uc11c 6\ubc88\uc2dc\ub4dc\uc778 \uc544\uc774\uc2ac\ub780\ub4dc\uc640 2\ubc88 \uc2dc\ub4dc\uc778 \uc2a4\uc704\uc2a4\uc5d0\uac8c \uc5f0\ub2ec\uc544 \ud328\ud558\uace0 \ud558\uc704\uad8c\uc73c\ub85c \uc9c0\uc5ed\uc608\uc120 \ud0c8\ub77d\ud55c \uac83\ub3c4 \ubaa8\uc790\ub77c \ud53c\ud30c\ub7ad\ud0b9\uc774 \ub0ae\uc544\uc11c 2\ubc88\uc2dc\ub4dc\ub85c \ubc30\uc815\ubc1b\uc740 \ud504\ub791\uc2a4\uc640\uc758 \ud3c9\uac00\uc804\uc5d0\uc11c 0-4\ub85c \ub300\ud328\ub97c \ub2f9\ud558\ub294 \uc218\ubaa8\ub97c \uacaa\uae30\uae4c\uc9c0 \ud588\ub2e4. \uc774\ub7ec\ud55c \ubb38\uc81c\uc788\ub294 \uc0b0\uc815\ubc29\uc2dd \ub54c\ubb38\uc5d0 \ubca8\uae30\uc5d0\uac00 \uc774\ud0c8\ub9ac\uc544\ub97c \uc81c\uce58\uace0 2014\ub144 FIFA \uc6d4\ub4dc\ucef5 \ubcf8\uc120\uc5d0\uc11c \ud1b1\uc2dc\ub4dc\ub85c \ud560\ub2f9\ubc1b\uc558\ub2e4.", "paragraph_answer": "\ud2b9\uc815 \ub300\ub959\uc5d0\uac8c \uc720\ub9ac\ud558\uac8c \uc8fc\uc5b4\uc9c0\ub294 FIFA \uc810\uc218\uc640 \ud2b9\uc815 \ub300\ud68c\uc5d0\ub9cc \ube44\uc911\uc774 \ub192\uc740 \ub7ad\ud0b9\uc0b0\uc815\ubc29\uc2dd\uc73c\ub85c \uc778\ud558\uc5ec \uc720\ub7fd\uc758 \uc57d\ud300\uc774 \ud0c0\ub300\ub959\uc758 \uc9c0\uc5ed\uac15\ud300\ubcf4\ub2e4 \ub354 \ub7ad\ud0b9\uc774 \ub192\uc740 \uacbd\uc6b0\ub3c4 \ubc1c\uc0dd\ud55c\ub2e4. \uc77c\ub840\ub85c \uadf8\ub9ac\uc2a4\ub294 \ub300\ud55c\ubbfc\uad6d\uacfc \ub9de\ubd99\uae30\ub9cc \ud588\ub2e4 \ud558\uba74 \ud328\ud558\ub294 \ub370\ub2e4\uac00 UEFA \uc720\ub85c 2004 \uc6b0\uc2b9 \uc774\uc678\uc5d0\ub294 \uc774\ub807\ub2e4\ud560 \uad6d\uc81c\ub300\ud68c \uc131\uc801\uc744 \uac70\ub454\uac8c \uc5c6\uc74c\uc5d0\ub3c4 \ubd88\uad6c\ud558\uace0 10\uc704\uad8c\uc778 \ubc18\uba74 \uc774\ud0c8\ub9ac\uc544\ub294 2006\ub144 FIFA \uc6d4\ub4dc\ucef5 \uc6b0\uc2b9\uacfc UEFA \uc720\ub85c 2012 \uc900\uc6b0\uc2b9\uc744 \uac70\ub450\uace0\ub3c4 \ud53c\ud30c\ub7ad\ud0b9\uc774 \ub0ae\ub2e4. \ub610\ud55c \ud53c\ud30c\ub7ad\ud0b9\uc0c1 \ub192\uac8c \ucc45\uc815\ub418\uc5b4 2014\ub144 FIFA \uc6d4\ub4dc\ucef5 \uc720\ub7fd \uc9c0\uc5ed \uc608\uc120\uc5d0\uc11c \ud1b1\uc2dc\ub4dc\uac00 \ub41c \ub178\ub974\uc6e8\uc774 \ub294 \uadf8 \uc870\uc5d0\uc11c 6\ubc88\uc2dc\ub4dc\uc778 \uc544\uc774\uc2ac\ub780\ub4dc\uc640 2\ubc88 \uc2dc\ub4dc\uc778 \uc2a4\uc704\uc2a4\uc5d0\uac8c \uc5f0\ub2ec\uc544 \ud328\ud558\uace0 \ud558\uc704\uad8c\uc73c\ub85c \uc9c0\uc5ed\uc608\uc120 \ud0c8\ub77d\ud55c \uac83\ub3c4 \ubaa8\uc790\ub77c \ud53c\ud30c\ub7ad\ud0b9\uc774 \ub0ae\uc544\uc11c 2\ubc88\uc2dc\ub4dc\ub85c \ubc30\uc815\ubc1b\uc740 \ud504\ub791\uc2a4\uc640\uc758 \ud3c9\uac00\uc804\uc5d0\uc11c 0-4\ub85c \ub300\ud328\ub97c \ub2f9\ud558\ub294 \uc218\ubaa8\ub97c \uacaa\uae30\uae4c\uc9c0 \ud588\ub2e4. \uc774\ub7ec\ud55c \ubb38\uc81c\uc788\ub294 \uc0b0\uc815\ubc29\uc2dd \ub54c\ubb38\uc5d0 \ubca8\uae30\uc5d0\uac00 \uc774\ud0c8\ub9ac\uc544\ub97c \uc81c\uce58\uace0 2014\ub144 FIFA \uc6d4\ub4dc\ucef5 \ubcf8\uc120\uc5d0\uc11c \ud1b1\uc2dc\ub4dc\ub85c \ud560\ub2f9\ubc1b\uc558\ub2e4.", "sentence_answer": "\ub610\ud55c \ud53c\ud30c\ub7ad\ud0b9\uc0c1 \ub192\uac8c \ucc45\uc815\ub418\uc5b4 2014\ub144 FIFA \uc6d4\ub4dc\ucef5 \uc720\ub7fd \uc9c0\uc5ed \uc608\uc120\uc5d0\uc11c \ud1b1\uc2dc\ub4dc\uac00 \ub41c \ub178\ub974\uc6e8\uc774 \ub294 \uadf8 \uc870\uc5d0\uc11c 6\ubc88\uc2dc\ub4dc\uc778 \uc544\uc774\uc2ac\ub780\ub4dc\uc640 2\ubc88 \uc2dc\ub4dc\uc778 \uc2a4\uc704\uc2a4\uc5d0\uac8c \uc5f0\ub2ec\uc544 \ud328\ud558\uace0 \ud558\uc704\uad8c\uc73c\ub85c \uc9c0\uc5ed\uc608\uc120 \ud0c8\ub77d\ud55c \uac83\ub3c4 \ubaa8\uc790\ub77c \ud53c\ud30c\ub7ad\ud0b9\uc774 \ub0ae\uc544\uc11c 2\ubc88\uc2dc\ub4dc\ub85c \ubc30\uc815\ubc1b\uc740 \ud504\ub791\uc2a4\uc640\uc758 \ud3c9\uac00\uc804\uc5d0\uc11c 0-4\ub85c \ub300\ud328\ub97c \ub2f9\ud558\ub294 \uc218\ubaa8\ub97c \uacaa\uae30\uae4c\uc9c0 \ud588\ub2e4.", "paragraph_id": "5782910-2-1", "paragraph_question": "question: 2014\ub144 FIFA \uc6d4\ub4dc\ucef5 \uc720\ub7fd \uc9c0\uc5ed \uc608\uc120\uc5d0\uc11c \ud1b1\uc2dc\ub4dc\uac00 \ub41c \uad6d\uac00\ub85c\uc368, \uc544\uc774\uc2ac\ub780\ub4dc\uc640 \uc2a4\uc704\uc2a4\uc5d0\uac8c \uc5f0\ub2ec\uc5d0 \ud328\ud55c \uad6d\uac00\ub294?, context: \ud2b9\uc815 \ub300\ub959\uc5d0\uac8c \uc720\ub9ac\ud558\uac8c \uc8fc\uc5b4\uc9c0\ub294 FIFA \uc810\uc218\uc640 \ud2b9\uc815 \ub300\ud68c\uc5d0\ub9cc \ube44\uc911\uc774 \ub192\uc740 \ub7ad\ud0b9\uc0b0\uc815\ubc29\uc2dd\uc73c\ub85c \uc778\ud558\uc5ec \uc720\ub7fd\uc758 \uc57d\ud300\uc774 \ud0c0\ub300\ub959\uc758 \uc9c0\uc5ed\uac15\ud300\ubcf4\ub2e4 \ub354 \ub7ad\ud0b9\uc774 \ub192\uc740 \uacbd\uc6b0\ub3c4 \ubc1c\uc0dd\ud55c\ub2e4. \uc77c\ub840\ub85c \uadf8\ub9ac\uc2a4\ub294 \ub300\ud55c\ubbfc\uad6d\uacfc \ub9de\ubd99\uae30\ub9cc \ud588\ub2e4 \ud558\uba74 \ud328\ud558\ub294 \ub370\ub2e4\uac00 UEFA \uc720\ub85c 2004 \uc6b0\uc2b9 \uc774\uc678\uc5d0\ub294 \uc774\ub807\ub2e4\ud560 \uad6d\uc81c\ub300\ud68c \uc131\uc801\uc744 \uac70\ub454\uac8c \uc5c6\uc74c\uc5d0\ub3c4 \ubd88\uad6c\ud558\uace0 10\uc704\uad8c\uc778 \ubc18\uba74 \uc774\ud0c8\ub9ac\uc544\ub294 2006\ub144 FIFA \uc6d4\ub4dc\ucef5 \uc6b0\uc2b9\uacfc UEFA \uc720\ub85c 2012 \uc900\uc6b0\uc2b9\uc744 \uac70\ub450\uace0\ub3c4 \ud53c\ud30c\ub7ad\ud0b9\uc774 \ub0ae\ub2e4. \ub610\ud55c \ud53c\ud30c\ub7ad\ud0b9\uc0c1 \ub192\uac8c \ucc45\uc815\ub418\uc5b4 2014\ub144 FIFA \uc6d4\ub4dc\ucef5 \uc720\ub7fd \uc9c0\uc5ed \uc608\uc120\uc5d0\uc11c \ud1b1\uc2dc\ub4dc\uac00 \ub41c \ub178\ub974\uc6e8\uc774\ub294 \uadf8 \uc870\uc5d0\uc11c 6\ubc88\uc2dc\ub4dc\uc778 \uc544\uc774\uc2ac\ub780\ub4dc\uc640 2\ubc88 \uc2dc\ub4dc\uc778 \uc2a4\uc704\uc2a4\uc5d0\uac8c \uc5f0\ub2ec\uc544 \ud328\ud558\uace0 \ud558\uc704\uad8c\uc73c\ub85c \uc9c0\uc5ed\uc608\uc120 \ud0c8\ub77d\ud55c \uac83\ub3c4 \ubaa8\uc790\ub77c \ud53c\ud30c\ub7ad\ud0b9\uc774 \ub0ae\uc544\uc11c 2\ubc88\uc2dc\ub4dc\ub85c \ubc30\uc815\ubc1b\uc740 \ud504\ub791\uc2a4\uc640\uc758 \ud3c9\uac00\uc804\uc5d0\uc11c 0-4\ub85c \ub300\ud328\ub97c \ub2f9\ud558\ub294 \uc218\ubaa8\ub97c \uacaa\uae30\uae4c\uc9c0 \ud588\ub2e4. \uc774\ub7ec\ud55c \ubb38\uc81c\uc788\ub294 \uc0b0\uc815\ubc29\uc2dd \ub54c\ubb38\uc5d0 \ubca8\uae30\uc5d0\uac00 \uc774\ud0c8\ub9ac\uc544\ub97c \uc81c\uce58\uace0 2014\ub144 FIFA \uc6d4\ub4dc\ucef5 \ubcf8\uc120\uc5d0\uc11c \ud1b1\uc2dc\ub4dc\ub85c \ud560\ub2f9\ubc1b\uc558\ub2e4."} {"question": "\ucfe0 \ud6cc\ub9b0\uc758 \uc678\uc219\ubaa8\uc774\uc790 \ucf58\ucf54\ubc14\ub974 \uc655\uc758 \uc544\ub0b4\ub294?", "paragraph": "\uc774\uba58 \ub9c8\ud558\uc758 \uc5b4\ub290 \ub0a0, \ucfe0 \ud6cc\ub9b0\uc740 \uce74\ud750\ubc14\ub4dc\uac00 \ud559\uc0dd\ub4e4\uc744 \uac00\ub974\uce58\ub294 \uac83\uc744 \uc6b0\uc5f0\ud788 \ub4e3\uac8c \ub418\uc5c8\ub2e4. \ud55c \ud559\uc0dd\uc774 \uce74\ud750\ubc14\ub4dc\uc5d0\uac8c \uc624\ub298\uc774 \uc5b4\ub5a4 \uc885\ub958\uc758 \uc0c1\uc11c\ub85c\uc6b4 \ub0a0\uc774\ub0d0\uace0 \ubb3c\uc5c8\ub2e4. \uce74\ud750\ubc14\ub4dc\ub294 \uc624\ub298 \ubb34\uae30\ub97c \uac16\uac8c \ub418\ub294 \uc804\uc0ac\ub294 \ubd88\uba78\uc758 \uba85\uc131\uc744 \uac00\uc9c0\uac8c \ub420 \uac83\uc774\ub77c\uace0 \ub300\ub2f5\ud588\ub2e4. \uc774\ub54c \uace0\uc791 7\uc0b4\uc774\uc5c8\ub358 \ucfe0 \ud6cc\ub9b0\uc740 \ucf58\ucf54\ubc14\ub974 \uc655\uc5d0\uac8c \uac00\uc11c \ubb34\uae30\ub97c \ub2ec\ub77c\uace0 \ud588\ub2e4. \ubb34\uae30\ub4e4\uc774 \ucfe0 \ud6cc\ub9b0\uc758 \ud798\uc744 \uacac\ub38c\ub0b4\uc9c0 \ubabb\ud558\uc790 \ucf58\ucf54\ubc14\ub974\ub294 \uc790\uae30 \ubb34\uae30\ub97c \uadf8\uc5d0\uac8c \uc8fc\uc5c8\ub2e4. \uadf8\ub7ec\ub098 \uc774 \uaf34\uc744 \ubcf8 \uce74\ud750\ubc14\ub4dc\ub294 \uc560\ud1b5\ud574\ud588\ub2e4. \ucfe0 \ud6cc\ub9b0\uc740 \uce74\ud750\ubc14\ub4dc\uc758 \uc608\uc5b8\uc744 \uc55e\ubd80\ubd84\ub9cc \ub4e3\uace0 \uac00\ubc84\ub838\ub358 \uac83\uc774\ub2e4. \uc608\uc5b8\uc758 \uc804\uccb4 \ub0b4\uc6a9\uc740 \uc624\ub298 \ubb34\uae30\ub97c \uac16\uac8c \ub418\ub294 \uc804\uc0ac\ub294 \ubd88\uba78\uc758 \uba85\uc131\uc744 \uac00\uc9c0\uac8c \ub420 \uac83\uc774\ub098 \uadf8\uc758 \uc218\uba85\uc740 \uc9e7\uc744 \uac83\uc774\ub77c\ub294 \uac83\uc774\uc5c8\ub2e4. \uc5bc\ub9c8 \ub4a4 \uce74\ud750\ubc14\ub4dc\uc758 \ub2e4\ub978 \uc608\uc5b8\uc744 \ub610 \ub4e4\uc740 \ucfe0 \ud6cc\ub9b0\uc740 \ucf58\ucf54\ubc14\ub974\uc5d0\uac8c \uc804\ucc28\ub97c \ub2ec\ub77c\uace0 \ud588\uace0, \uc774\ubc88\uc5d0\ub3c4 \ucf58\ucf54\ubc14\ub974 \uc655\uc758 \uc804\ucc28\ub9cc\uc774 \uadf8\uc758 \ud798\uc744 \uacac\ub38c\ub0bc \uc218 \uc788\uc5c8\ub2e4. \ubb34\uae30\uc640 \ud0c8\uac83\uc744 \uac00\uc9c0\uac8c \ub41c \ucfe0 \ud6cc\ub9b0\uc740 \u2018\uc9c0\uae08 \uc0b4\uc544\uc788\ub294 \uc6b8\ub77c\uc778\ub4e4\ubcf4\ub2e4 \ub354 \ub9ce\uc740 \uc6b8\ub77c\uc778\ub4e4\uc744 \uc8fd\uc600\ub2e4\u2019\uace0 \uc790\ub791\ud558\uace0 \ub2e4\ub2c8\ub358 \ub124\ud06c\ud0c4 \uc2a4\ucf00\ub124\uc758 \uc138 \uc544\ub4e4\ub4e4\uc744 \uae09\uc2b5\ud574\uc11c \ubaa8\uc870\ub9ac \uc8fd\uc600\ub2e4. \uadf8\ub294 \uc804\ub9ac\ud488\uc744 \ucc59\uaca8\uc11c \uc774\uba58 \ub9c8\ud558\ub85c \ub3cc\uc544\uc654\uace0, \uc6b8\ub77c\uc778\ub4e4\uc740 \ub9ac\uc2a4\ud2b8\ub77c\ub4dc \uc0c1\ud0dc\uc5d0 \ube60\uc9c4 \ucfe0 \ud6cc\ub9b0\uc774 \uc790\uc2e0\ub4e4\uae4c\uc9c0 \ubaa8\ub450 \uc8fd\uc774\uc9c0 \uc54a\uc744\uc9c0 \ub450\ub824\uc6cc\ud588\ub2e4. \ucf58\ucf54\ubc14\ub974 \uc655\uc758 \uc544\ub0b4(\uc989 \ucfe0 \ud6cc\ub9b0\uc5d0\uac8c\ub294 \uc678\uc219\ubaa8) \ubb34\uac04 \uc655\ube44\uac00 \uc774\uba58 \ub9c8\ud558\uc758 \uc5ec\uc790\ub4e4\uc744 \uc774\ub04c\uace0 \ub098\uc640 \uc6c3\uc637\uc744 \uae4c\uace0 \uc816\uac00\uc2b4\uc744 \ub4dc\ub7ec\ub0b4 \ubcf4\uc600\ub2e4. \uc774\uac83\uc744 \ubcf8 \ucfe0 \ud6cc\ub9b0\uc740 \ubd80\ub044\ub7ec\uc6c0\uc5d0 \ub208\uc744 \ub3cc\ub838\uace0, \uc774\ub54c \ub0a8\uc790\ub4e4\uc774 \ub2ec\ub824\ub4e4\uc5b4 \ubab8\uc2f8\uc6c0 \ub05d\uc5d0 \uadf8\ub97c \ub0c9\uc218\uac00 \ub4e4\uc5b4\uc788\ub294 \ubb3c\ud1b5\uc5d0 \ucc98\ubc15\uc558\ub2e4. \ucfe0 \ud6cc\ub9b0\uc774 \ubc1c\uc0b0\ud558\ub294 \uad11\ub780\uc758 \uc5f4\uae30 \ub54c\ubb38\uc5d0 \ubb3c\ud1b5\uc774 \ud130\uc838 \ubc84\ub838\ub2e4. \uc6b8\ub77c \ub0a8\uc790\ub4e4\uc740 \ub450 \ubc88\uc9f8 \ubb3c\ud1b5\uc5d0 \ub2e4\uc2dc \uadf8\ub97c \ucc98\ubc15\uc558\uace0, \uc774\ubc88\uc5d0\ub294 \ubb3c\uc774 \ud384\ud384 \ub053\uc5b4\uc62c\ub790\ub2e4. \uc138 \ubc88\uc9f8 \ubb3c\ud1b5\uc5d0 \ub2e4\uc2dc \ucc98\ubc15\uc790 \uaca8\uc6b0 \uc801\ub2f9\ud55c \uc628\ub3c4\ub85c \uc5f4\uc744 \uc2dd\ud790 \uc218 \uc788\uc5c8\ub2e4.", "answer": "\ubb34\uac04 \uc655\ube44", "sentence": "\ucf58\ucf54\ubc14\ub974 \uc655\uc758 \uc544\ub0b4(\uc989 \ucfe0 \ud6cc\ub9b0\uc5d0\uac8c\ub294 \uc678\uc219\ubaa8) \ubb34\uac04 \uc655\ube44 \uac00 \uc774\uba58 \ub9c8\ud558\uc758 \uc5ec\uc790\ub4e4\uc744 \uc774\ub04c\uace0 \ub098\uc640 \uc6c3\uc637\uc744 \uae4c\uace0 \uc816\uac00\uc2b4\uc744 \ub4dc\ub7ec\ub0b4 \ubcf4\uc600\ub2e4.", "paragraph_sentence": "\uc774\uba58 \ub9c8\ud558\uc758 \uc5b4\ub290 \ub0a0, \ucfe0 \ud6cc\ub9b0\uc740 \uce74\ud750\ubc14\ub4dc\uac00 \ud559\uc0dd\ub4e4\uc744 \uac00\ub974\uce58\ub294 \uac83\uc744 \uc6b0\uc5f0\ud788 \ub4e3\uac8c \ub418\uc5c8\ub2e4. \ud55c \ud559\uc0dd\uc774 \uce74\ud750\ubc14\ub4dc\uc5d0\uac8c \uc624\ub298\uc774 \uc5b4\ub5a4 \uc885\ub958\uc758 \uc0c1\uc11c\ub85c\uc6b4 \ub0a0\uc774\ub0d0\uace0 \ubb3c\uc5c8\ub2e4. \uce74\ud750\ubc14\ub4dc\ub294 \uc624\ub298 \ubb34\uae30\ub97c \uac16\uac8c \ub418\ub294 \uc804\uc0ac\ub294 \ubd88\uba78\uc758 \uba85\uc131\uc744 \uac00\uc9c0\uac8c \ub420 \uac83\uc774\ub77c\uace0 \ub300\ub2f5\ud588\ub2e4. \uc774\ub54c \uace0\uc791 7\uc0b4\uc774\uc5c8\ub358 \ucfe0 \ud6cc\ub9b0\uc740 \ucf58\ucf54\ubc14\ub974 \uc655\uc5d0\uac8c \uac00\uc11c \ubb34\uae30\ub97c \ub2ec\ub77c\uace0 \ud588\ub2e4. \ubb34\uae30\ub4e4\uc774 \ucfe0 \ud6cc\ub9b0\uc758 \ud798\uc744 \uacac\ub38c\ub0b4\uc9c0 \ubabb\ud558\uc790 \ucf58\ucf54\ubc14\ub974\ub294 \uc790\uae30 \ubb34\uae30\ub97c \uadf8\uc5d0\uac8c \uc8fc\uc5c8\ub2e4. \uadf8\ub7ec\ub098 \uc774 \uaf34\uc744 \ubcf8 \uce74\ud750\ubc14\ub4dc\ub294 \uc560\ud1b5\ud574\ud588\ub2e4. \ucfe0 \ud6cc\ub9b0\uc740 \uce74\ud750\ubc14\ub4dc\uc758 \uc608\uc5b8\uc744 \uc55e\ubd80\ubd84\ub9cc \ub4e3\uace0 \uac00\ubc84\ub838\ub358 \uac83\uc774\ub2e4. \uc608\uc5b8\uc758 \uc804\uccb4 \ub0b4\uc6a9\uc740 \uc624\ub298 \ubb34\uae30\ub97c \uac16\uac8c \ub418\ub294 \uc804\uc0ac\ub294 \ubd88\uba78\uc758 \uba85\uc131\uc744 \uac00\uc9c0\uac8c \ub420 \uac83\uc774\ub098 \uadf8\uc758 \uc218\uba85\uc740 \uc9e7\uc744 \uac83\uc774\ub77c\ub294 \uac83\uc774\uc5c8\ub2e4. \uc5bc\ub9c8 \ub4a4 \uce74\ud750\ubc14\ub4dc\uc758 \ub2e4\ub978 \uc608\uc5b8\uc744 \ub610 \ub4e4\uc740 \ucfe0 \ud6cc\ub9b0\uc740 \ucf58\ucf54\ubc14\ub974\uc5d0\uac8c \uc804\ucc28\ub97c \ub2ec\ub77c\uace0 \ud588\uace0, \uc774\ubc88\uc5d0\ub3c4 \ucf58\ucf54\ubc14\ub974 \uc655\uc758 \uc804\ucc28\ub9cc\uc774 \uadf8\uc758 \ud798\uc744 \uacac\ub38c\ub0bc \uc218 \uc788\uc5c8\ub2e4. \ubb34\uae30\uc640 \ud0c8\uac83\uc744 \uac00\uc9c0\uac8c \ub41c \ucfe0 \ud6cc\ub9b0\uc740 \u2018\uc9c0\uae08 \uc0b4\uc544\uc788\ub294 \uc6b8\ub77c\uc778\ub4e4\ubcf4\ub2e4 \ub354 \ub9ce\uc740 \uc6b8\ub77c\uc778\ub4e4\uc744 \uc8fd\uc600\ub2e4\u2019\uace0 \uc790\ub791\ud558\uace0 \ub2e4\ub2c8\ub358 \ub124\ud06c\ud0c4 \uc2a4\ucf00\ub124\uc758 \uc138 \uc544\ub4e4\ub4e4\uc744 \uae09\uc2b5\ud574\uc11c \ubaa8\uc870\ub9ac \uc8fd\uc600\ub2e4. \uadf8\ub294 \uc804\ub9ac\ud488\uc744 \ucc59\uaca8\uc11c \uc774\uba58 \ub9c8\ud558\ub85c \ub3cc\uc544\uc654\uace0, \uc6b8\ub77c\uc778\ub4e4\uc740 \ub9ac\uc2a4\ud2b8\ub77c\ub4dc \uc0c1\ud0dc\uc5d0 \ube60\uc9c4 \ucfe0 \ud6cc\ub9b0\uc774 \uc790\uc2e0\ub4e4\uae4c\uc9c0 \ubaa8\ub450 \uc8fd\uc774\uc9c0 \uc54a\uc744\uc9c0 \ub450\ub824\uc6cc\ud588\ub2e4. \ucf58\ucf54\ubc14\ub974 \uc655\uc758 \uc544\ub0b4(\uc989 \ucfe0 \ud6cc\ub9b0\uc5d0\uac8c\ub294 \uc678\uc219\ubaa8) \ubb34\uac04 \uc655\ube44 \uac00 \uc774\uba58 \ub9c8\ud558\uc758 \uc5ec\uc790\ub4e4\uc744 \uc774\ub04c\uace0 \ub098\uc640 \uc6c3\uc637\uc744 \uae4c\uace0 \uc816\uac00\uc2b4\uc744 \ub4dc\ub7ec\ub0b4 \ubcf4\uc600\ub2e4. \uc774\uac83\uc744 \ubcf8 \ucfe0 \ud6cc\ub9b0\uc740 \ubd80\ub044\ub7ec\uc6c0\uc5d0 \ub208\uc744 \ub3cc\ub838\uace0, \uc774\ub54c \ub0a8\uc790\ub4e4\uc774 \ub2ec\ub824\ub4e4\uc5b4 \ubab8\uc2f8\uc6c0 \ub05d\uc5d0 \uadf8\ub97c \ub0c9\uc218\uac00 \ub4e4\uc5b4\uc788\ub294 \ubb3c\ud1b5\uc5d0 \ucc98\ubc15\uc558\ub2e4. \ucfe0 \ud6cc\ub9b0\uc774 \ubc1c\uc0b0\ud558\ub294 \uad11\ub780\uc758 \uc5f4\uae30 \ub54c\ubb38\uc5d0 \ubb3c\ud1b5\uc774 \ud130\uc838 \ubc84\ub838\ub2e4. \uc6b8\ub77c \ub0a8\uc790\ub4e4\uc740 \ub450 \ubc88\uc9f8 \ubb3c\ud1b5\uc5d0 \ub2e4\uc2dc \uadf8\ub97c \ucc98\ubc15\uc558\uace0, \uc774\ubc88\uc5d0\ub294 \ubb3c\uc774 \ud384\ud384 \ub053\uc5b4\uc62c\ub790\ub2e4. \uc138 \ubc88\uc9f8 \ubb3c\ud1b5\uc5d0 \ub2e4\uc2dc \ucc98\ubc15\uc790 \uaca8\uc6b0 \uc801\ub2f9\ud55c \uc628\ub3c4\ub85c \uc5f4\uc744 \uc2dd\ud790 \uc218 \uc788\uc5c8\ub2e4.", "paragraph_answer": "\uc774\uba58 \ub9c8\ud558\uc758 \uc5b4\ub290 \ub0a0, \ucfe0 \ud6cc\ub9b0\uc740 \uce74\ud750\ubc14\ub4dc\uac00 \ud559\uc0dd\ub4e4\uc744 \uac00\ub974\uce58\ub294 \uac83\uc744 \uc6b0\uc5f0\ud788 \ub4e3\uac8c \ub418\uc5c8\ub2e4. \ud55c \ud559\uc0dd\uc774 \uce74\ud750\ubc14\ub4dc\uc5d0\uac8c \uc624\ub298\uc774 \uc5b4\ub5a4 \uc885\ub958\uc758 \uc0c1\uc11c\ub85c\uc6b4 \ub0a0\uc774\ub0d0\uace0 \ubb3c\uc5c8\ub2e4. \uce74\ud750\ubc14\ub4dc\ub294 \uc624\ub298 \ubb34\uae30\ub97c \uac16\uac8c \ub418\ub294 \uc804\uc0ac\ub294 \ubd88\uba78\uc758 \uba85\uc131\uc744 \uac00\uc9c0\uac8c \ub420 \uac83\uc774\ub77c\uace0 \ub300\ub2f5\ud588\ub2e4. \uc774\ub54c \uace0\uc791 7\uc0b4\uc774\uc5c8\ub358 \ucfe0 \ud6cc\ub9b0\uc740 \ucf58\ucf54\ubc14\ub974 \uc655\uc5d0\uac8c \uac00\uc11c \ubb34\uae30\ub97c \ub2ec\ub77c\uace0 \ud588\ub2e4. \ubb34\uae30\ub4e4\uc774 \ucfe0 \ud6cc\ub9b0\uc758 \ud798\uc744 \uacac\ub38c\ub0b4\uc9c0 \ubabb\ud558\uc790 \ucf58\ucf54\ubc14\ub974\ub294 \uc790\uae30 \ubb34\uae30\ub97c \uadf8\uc5d0\uac8c \uc8fc\uc5c8\ub2e4. \uadf8\ub7ec\ub098 \uc774 \uaf34\uc744 \ubcf8 \uce74\ud750\ubc14\ub4dc\ub294 \uc560\ud1b5\ud574\ud588\ub2e4. \ucfe0 \ud6cc\ub9b0\uc740 \uce74\ud750\ubc14\ub4dc\uc758 \uc608\uc5b8\uc744 \uc55e\ubd80\ubd84\ub9cc \ub4e3\uace0 \uac00\ubc84\ub838\ub358 \uac83\uc774\ub2e4. \uc608\uc5b8\uc758 \uc804\uccb4 \ub0b4\uc6a9\uc740 \uc624\ub298 \ubb34\uae30\ub97c \uac16\uac8c \ub418\ub294 \uc804\uc0ac\ub294 \ubd88\uba78\uc758 \uba85\uc131\uc744 \uac00\uc9c0\uac8c \ub420 \uac83\uc774\ub098 \uadf8\uc758 \uc218\uba85\uc740 \uc9e7\uc744 \uac83\uc774\ub77c\ub294 \uac83\uc774\uc5c8\ub2e4. \uc5bc\ub9c8 \ub4a4 \uce74\ud750\ubc14\ub4dc\uc758 \ub2e4\ub978 \uc608\uc5b8\uc744 \ub610 \ub4e4\uc740 \ucfe0 \ud6cc\ub9b0\uc740 \ucf58\ucf54\ubc14\ub974\uc5d0\uac8c \uc804\ucc28\ub97c \ub2ec\ub77c\uace0 \ud588\uace0, \uc774\ubc88\uc5d0\ub3c4 \ucf58\ucf54\ubc14\ub974 \uc655\uc758 \uc804\ucc28\ub9cc\uc774 \uadf8\uc758 \ud798\uc744 \uacac\ub38c\ub0bc \uc218 \uc788\uc5c8\ub2e4. \ubb34\uae30\uc640 \ud0c8\uac83\uc744 \uac00\uc9c0\uac8c \ub41c \ucfe0 \ud6cc\ub9b0\uc740 \u2018\uc9c0\uae08 \uc0b4\uc544\uc788\ub294 \uc6b8\ub77c\uc778\ub4e4\ubcf4\ub2e4 \ub354 \ub9ce\uc740 \uc6b8\ub77c\uc778\ub4e4\uc744 \uc8fd\uc600\ub2e4\u2019\uace0 \uc790\ub791\ud558\uace0 \ub2e4\ub2c8\ub358 \ub124\ud06c\ud0c4 \uc2a4\ucf00\ub124\uc758 \uc138 \uc544\ub4e4\ub4e4\uc744 \uae09\uc2b5\ud574\uc11c \ubaa8\uc870\ub9ac \uc8fd\uc600\ub2e4. \uadf8\ub294 \uc804\ub9ac\ud488\uc744 \ucc59\uaca8\uc11c \uc774\uba58 \ub9c8\ud558\ub85c \ub3cc\uc544\uc654\uace0, \uc6b8\ub77c\uc778\ub4e4\uc740 \ub9ac\uc2a4\ud2b8\ub77c\ub4dc \uc0c1\ud0dc\uc5d0 \ube60\uc9c4 \ucfe0 \ud6cc\ub9b0\uc774 \uc790\uc2e0\ub4e4\uae4c\uc9c0 \ubaa8\ub450 \uc8fd\uc774\uc9c0 \uc54a\uc744\uc9c0 \ub450\ub824\uc6cc\ud588\ub2e4. \ucf58\ucf54\ubc14\ub974 \uc655\uc758 \uc544\ub0b4(\uc989 \ucfe0 \ud6cc\ub9b0\uc5d0\uac8c\ub294 \uc678\uc219\ubaa8) \ubb34\uac04 \uc655\ube44 \uac00 \uc774\uba58 \ub9c8\ud558\uc758 \uc5ec\uc790\ub4e4\uc744 \uc774\ub04c\uace0 \ub098\uc640 \uc6c3\uc637\uc744 \uae4c\uace0 \uc816\uac00\uc2b4\uc744 \ub4dc\ub7ec\ub0b4 \ubcf4\uc600\ub2e4. \uc774\uac83\uc744 \ubcf8 \ucfe0 \ud6cc\ub9b0\uc740 \ubd80\ub044\ub7ec\uc6c0\uc5d0 \ub208\uc744 \ub3cc\ub838\uace0, \uc774\ub54c \ub0a8\uc790\ub4e4\uc774 \ub2ec\ub824\ub4e4\uc5b4 \ubab8\uc2f8\uc6c0 \ub05d\uc5d0 \uadf8\ub97c \ub0c9\uc218\uac00 \ub4e4\uc5b4\uc788\ub294 \ubb3c\ud1b5\uc5d0 \ucc98\ubc15\uc558\ub2e4. \ucfe0 \ud6cc\ub9b0\uc774 \ubc1c\uc0b0\ud558\ub294 \uad11\ub780\uc758 \uc5f4\uae30 \ub54c\ubb38\uc5d0 \ubb3c\ud1b5\uc774 \ud130\uc838 \ubc84\ub838\ub2e4. \uc6b8\ub77c \ub0a8\uc790\ub4e4\uc740 \ub450 \ubc88\uc9f8 \ubb3c\ud1b5\uc5d0 \ub2e4\uc2dc \uadf8\ub97c \ucc98\ubc15\uc558\uace0, \uc774\ubc88\uc5d0\ub294 \ubb3c\uc774 \ud384\ud384 \ub053\uc5b4\uc62c\ub790\ub2e4. \uc138 \ubc88\uc9f8 \ubb3c\ud1b5\uc5d0 \ub2e4\uc2dc \ucc98\ubc15\uc790 \uaca8\uc6b0 \uc801\ub2f9\ud55c \uc628\ub3c4\ub85c \uc5f4\uc744 \uc2dd\ud790 \uc218 \uc788\uc5c8\ub2e4.", "sentence_answer": "\ucf58\ucf54\ubc14\ub974 \uc655\uc758 \uc544\ub0b4(\uc989 \ucfe0 \ud6cc\ub9b0\uc5d0\uac8c\ub294 \uc678\uc219\ubaa8) \ubb34\uac04 \uc655\ube44 \uac00 \uc774\uba58 \ub9c8\ud558\uc758 \uc5ec\uc790\ub4e4\uc744 \uc774\ub04c\uace0 \ub098\uc640 \uc6c3\uc637\uc744 \uae4c\uace0 \uc816\uac00\uc2b4\uc744 \ub4dc\ub7ec\ub0b4 \ubcf4\uc600\ub2e4.", "paragraph_id": "6562782-5-2", "paragraph_question": "question: \ucfe0 \ud6cc\ub9b0\uc758 \uc678\uc219\ubaa8\uc774\uc790 \ucf58\ucf54\ubc14\ub974 \uc655\uc758 \uc544\ub0b4\ub294?, context: \uc774\uba58 \ub9c8\ud558\uc758 \uc5b4\ub290 \ub0a0, \ucfe0 \ud6cc\ub9b0\uc740 \uce74\ud750\ubc14\ub4dc\uac00 \ud559\uc0dd\ub4e4\uc744 \uac00\ub974\uce58\ub294 \uac83\uc744 \uc6b0\uc5f0\ud788 \ub4e3\uac8c \ub418\uc5c8\ub2e4. \ud55c \ud559\uc0dd\uc774 \uce74\ud750\ubc14\ub4dc\uc5d0\uac8c \uc624\ub298\uc774 \uc5b4\ub5a4 \uc885\ub958\uc758 \uc0c1\uc11c\ub85c\uc6b4 \ub0a0\uc774\ub0d0\uace0 \ubb3c\uc5c8\ub2e4. \uce74\ud750\ubc14\ub4dc\ub294 \uc624\ub298 \ubb34\uae30\ub97c \uac16\uac8c \ub418\ub294 \uc804\uc0ac\ub294 \ubd88\uba78\uc758 \uba85\uc131\uc744 \uac00\uc9c0\uac8c \ub420 \uac83\uc774\ub77c\uace0 \ub300\ub2f5\ud588\ub2e4. \uc774\ub54c \uace0\uc791 7\uc0b4\uc774\uc5c8\ub358 \ucfe0 \ud6cc\ub9b0\uc740 \ucf58\ucf54\ubc14\ub974 \uc655\uc5d0\uac8c \uac00\uc11c \ubb34\uae30\ub97c \ub2ec\ub77c\uace0 \ud588\ub2e4. \ubb34\uae30\ub4e4\uc774 \ucfe0 \ud6cc\ub9b0\uc758 \ud798\uc744 \uacac\ub38c\ub0b4\uc9c0 \ubabb\ud558\uc790 \ucf58\ucf54\ubc14\ub974\ub294 \uc790\uae30 \ubb34\uae30\ub97c \uadf8\uc5d0\uac8c \uc8fc\uc5c8\ub2e4. \uadf8\ub7ec\ub098 \uc774 \uaf34\uc744 \ubcf8 \uce74\ud750\ubc14\ub4dc\ub294 \uc560\ud1b5\ud574\ud588\ub2e4. \ucfe0 \ud6cc\ub9b0\uc740 \uce74\ud750\ubc14\ub4dc\uc758 \uc608\uc5b8\uc744 \uc55e\ubd80\ubd84\ub9cc \ub4e3\uace0 \uac00\ubc84\ub838\ub358 \uac83\uc774\ub2e4. \uc608\uc5b8\uc758 \uc804\uccb4 \ub0b4\uc6a9\uc740 \uc624\ub298 \ubb34\uae30\ub97c \uac16\uac8c \ub418\ub294 \uc804\uc0ac\ub294 \ubd88\uba78\uc758 \uba85\uc131\uc744 \uac00\uc9c0\uac8c \ub420 \uac83\uc774\ub098 \uadf8\uc758 \uc218\uba85\uc740 \uc9e7\uc744 \uac83\uc774\ub77c\ub294 \uac83\uc774\uc5c8\ub2e4. \uc5bc\ub9c8 \ub4a4 \uce74\ud750\ubc14\ub4dc\uc758 \ub2e4\ub978 \uc608\uc5b8\uc744 \ub610 \ub4e4\uc740 \ucfe0 \ud6cc\ub9b0\uc740 \ucf58\ucf54\ubc14\ub974\uc5d0\uac8c \uc804\ucc28\ub97c \ub2ec\ub77c\uace0 \ud588\uace0, \uc774\ubc88\uc5d0\ub3c4 \ucf58\ucf54\ubc14\ub974 \uc655\uc758 \uc804\ucc28\ub9cc\uc774 \uadf8\uc758 \ud798\uc744 \uacac\ub38c\ub0bc \uc218 \uc788\uc5c8\ub2e4. \ubb34\uae30\uc640 \ud0c8\uac83\uc744 \uac00\uc9c0\uac8c \ub41c \ucfe0 \ud6cc\ub9b0\uc740 \u2018\uc9c0\uae08 \uc0b4\uc544\uc788\ub294 \uc6b8\ub77c\uc778\ub4e4\ubcf4\ub2e4 \ub354 \ub9ce\uc740 \uc6b8\ub77c\uc778\ub4e4\uc744 \uc8fd\uc600\ub2e4\u2019\uace0 \uc790\ub791\ud558\uace0 \ub2e4\ub2c8\ub358 \ub124\ud06c\ud0c4 \uc2a4\ucf00\ub124\uc758 \uc138 \uc544\ub4e4\ub4e4\uc744 \uae09\uc2b5\ud574\uc11c \ubaa8\uc870\ub9ac \uc8fd\uc600\ub2e4. \uadf8\ub294 \uc804\ub9ac\ud488\uc744 \ucc59\uaca8\uc11c \uc774\uba58 \ub9c8\ud558\ub85c \ub3cc\uc544\uc654\uace0, \uc6b8\ub77c\uc778\ub4e4\uc740 \ub9ac\uc2a4\ud2b8\ub77c\ub4dc \uc0c1\ud0dc\uc5d0 \ube60\uc9c4 \ucfe0 \ud6cc\ub9b0\uc774 \uc790\uc2e0\ub4e4\uae4c\uc9c0 \ubaa8\ub450 \uc8fd\uc774\uc9c0 \uc54a\uc744\uc9c0 \ub450\ub824\uc6cc\ud588\ub2e4. \ucf58\ucf54\ubc14\ub974 \uc655\uc758 \uc544\ub0b4(\uc989 \ucfe0 \ud6cc\ub9b0\uc5d0\uac8c\ub294 \uc678\uc219\ubaa8) \ubb34\uac04 \uc655\ube44\uac00 \uc774\uba58 \ub9c8\ud558\uc758 \uc5ec\uc790\ub4e4\uc744 \uc774\ub04c\uace0 \ub098\uc640 \uc6c3\uc637\uc744 \uae4c\uace0 \uc816\uac00\uc2b4\uc744 \ub4dc\ub7ec\ub0b4 \ubcf4\uc600\ub2e4. \uc774\uac83\uc744 \ubcf8 \ucfe0 \ud6cc\ub9b0\uc740 \ubd80\ub044\ub7ec\uc6c0\uc5d0 \ub208\uc744 \ub3cc\ub838\uace0, \uc774\ub54c \ub0a8\uc790\ub4e4\uc774 \ub2ec\ub824\ub4e4\uc5b4 \ubab8\uc2f8\uc6c0 \ub05d\uc5d0 \uadf8\ub97c \ub0c9\uc218\uac00 \ub4e4\uc5b4\uc788\ub294 \ubb3c\ud1b5\uc5d0 \ucc98\ubc15\uc558\ub2e4. \ucfe0 \ud6cc\ub9b0\uc774 \ubc1c\uc0b0\ud558\ub294 \uad11\ub780\uc758 \uc5f4\uae30 \ub54c\ubb38\uc5d0 \ubb3c\ud1b5\uc774 \ud130\uc838 \ubc84\ub838\ub2e4. \uc6b8\ub77c \ub0a8\uc790\ub4e4\uc740 \ub450 \ubc88\uc9f8 \ubb3c\ud1b5\uc5d0 \ub2e4\uc2dc \uadf8\ub97c \ucc98\ubc15\uc558\uace0, \uc774\ubc88\uc5d0\ub294 \ubb3c\uc774 \ud384\ud384 \ub053\uc5b4\uc62c\ub790\ub2e4. \uc138 \ubc88\uc9f8 \ubb3c\ud1b5\uc5d0 \ub2e4\uc2dc \ucc98\ubc15\uc790 \uaca8\uc6b0 \uc801\ub2f9\ud55c \uc628\ub3c4\ub85c \uc5f4\uc744 \uc2dd\ud790 \uc218 \uc788\uc5c8\ub2e4."} {"question": "\ub7ec\uc2dc\uc544 \uc804\uc120\uc758 \uace1\uc0ac\ud3ec \uc5f0\ub300\uc5d0\uc11c \ube44\ud2b8\uac90\uc288\ud0c0\uc778\uc774 \ubc1b\uc740 \ud6c8\uc7a5\uc740?", "paragraph": "\uc81c1\ucc28 \uc138\uacc4\ub300\uc804\uc774 \uc77c\uc5b4\ub09c 1914\ub144\uc5d0 \ube44\ud2b8\uac90\uc288\ud0c0\uc778\uc740 1\ub144\uc9f8 \uc740\ub454 \uc0dd\ud65c\ud558\uace0 \uc788\uc5c8\ub2e4. \ube44\ud2b8\uac90\uc288\ud0c0\uc778\uc740 \uc804\uc7c1\uc774 \uc77c\uc5b4\ub0ac\ub2e4\ub294 \uc18c\uc2dd\uc5d0 \ud06c\uac8c \ucda9\uaca9\ubc1b\uc558\uace0 \uc624\uc2a4\ud2b8\ub9ac\uc544 \ud5dd\uac00\ub9ac \uc81c\uad6d \uc721\uad70\uc5d0 \uc790\uc6d0\ud558\uc600\ub2e4. \uc785\ub300 \ucd08\uae30\uc5d0\ub294 \ud568\uc120\uc5d0\uc11c \uadfc\ubb34\ud558\uc600\uc73c\ub098 \uc5bc\ub9c8 \ud6c4 \ud3ec\ubcd1\ub300\ub85c \uc804\uadfc\ud558\uc600\ub2e4. 1916\ub144 3\uc6d4\uc5d0 \ube44\ud2b8\uac90\uc288\ud0c0\uc778\uc740 \uc624\uc2a4\ud2b8\ub9ac\uc544 7\uad70\uc5d0 \uc790\uc6d0\ud558\uc5ec \ube0c\ub8e8\uc2e4\ub85c\ud504 \uacf5\uc138\ub97c \ubc29\uc5b4\ud558\ub294 \ucd5c\uc804\ubc29 \uc870\uc885\uc0ac\ub85c \ucc38\uc804\ud558\uc600\ub2e4. \uc601\uad6d\uad70\uacfc\uc758 \uc804\ud22c \ud6c4\uc5d0 \ube44\ud2b8\uac90\uc288\ud0c0\uc778\uc740 \"\uc6a9\uac10\ud55c \ud589\ub3d9\uacfc \uce68\ucc29\ud558\uace0 \ub0c9\uc815\ud55c \uc601\uc6c5\ub2e4\uc6b4 \ud65c\uc57d\"\uc73c\ub85c \ub9d0\ubbf8\uc554\uc544 \uc218\ud6c8\ud588\ub2e4. 1917\ub144 1\uc6d4 \ube44\ud2b8\uac90\uc288\ud0c0\uc778\uc740 \ub7ec\uc2dc\uc544 \uc804\uc120\uc758 \uace1\uc0ac\ud3ec \uc5f0\ub300\ub85c \uc804\uadfc\ub418\uc5c8\uace0 \uc5ec\uae30\uc11c \ub2e4\uc2dc \uc6a9\uac10\ud55c \ud589\ub3d9\uc73c\ub85c \uc740\uc7a5\ubb34\uacf5\ud6c8\uc7a5\uc744 \ube44\ub86f\ud574 \ud6c8\uc7a5 \uc5ec\ub7ec \uac1c\ub97c \ubc1b\uc558\ub2e4. 1918\ub144 \ube44\ud2b8\uac90\uc288\ud0c0\uc778\uc740 \ub300\uc704\ub85c \uc2b9\uc9c4\ud558\uc600\uace0 \uc624\ub298\ub0a0 \ud2b8\ub80c\ud2f0\ub178\ub85c \ubd88\ub9ac\ub294 \uc774\ud0c8\ub9ac\uc544 \ud2f0\ub864 \ub0a8\ubd80 \uc804\uc5ed\uc5d0\uc11c \uc0b0\uc545 \ud3ec\ubcd1 \uc5f0\ub300\uc5d0 \ubc30\uc18d\ub418\uc5c8\ub2e4. 1918\ub144 6\uc6d4 \uc624\uc2a4\ud2b8\ub9ac\uc544\uc758 \uacf5\uc138\uc5d0\uc11c \uc804\ud6c8\uc744 \uc138\uc6b4 \ube44\ud2b8\uac90\uc288\ud0c0\uc778\uc740 \ucd5c\uace0 \ud6c8\uc7a5\uc778 \uae08\uc7a5\ubb34\uacf5\ud6c8\uc7a5\uc5d0 \ucd94\ucc9c\ub418\uc5c8\uace0 \uadf8 \uc544\ub798 \ub2e8\uacc4\uc778 \uac80\uc758 \ubb34\uacf5\ud6c8\uc7a5\uc744 \ubc1b\uc558\ub2e4. \ube44\ud2b8\uac90\uc288\ud0c0\uc778\uc740 1918\ub144 11\uc6d4 \ud2b8\ub80c\ud1a0 \uadfc\uad50\uc5d0\uc11c \uc774\ud0c8\ub9ac\uc544\uad70\uc758 \ud3ec\ub85c\uac00 \ub418\uc5c8\ub2e4.", "answer": "\uc740\uc7a5\ubb34\uacf5\ud6c8\uc7a5", "sentence": "\uc5ec\uae30\uc11c \ub2e4\uc2dc \uc6a9\uac10\ud55c \ud589\ub3d9\uc73c\ub85c \uc740\uc7a5\ubb34\uacf5\ud6c8\uc7a5 \uc744 \ube44\ub86f\ud574 \ud6c8\uc7a5 \uc5ec\ub7ec \uac1c\ub97c \ubc1b\uc558\ub2e4.", "paragraph_sentence": "\uc81c1\ucc28 \uc138\uacc4\ub300\uc804\uc774 \uc77c\uc5b4\ub09c 1914\ub144\uc5d0 \ube44\ud2b8\uac90\uc288\ud0c0\uc778\uc740 1\ub144\uc9f8 \uc740\ub454 \uc0dd\ud65c\ud558\uace0 \uc788\uc5c8\ub2e4. \ube44\ud2b8\uac90\uc288\ud0c0\uc778\uc740 \uc804\uc7c1\uc774 \uc77c\uc5b4\ub0ac\ub2e4\ub294 \uc18c\uc2dd\uc5d0 \ud06c\uac8c \ucda9\uaca9\ubc1b\uc558\uace0 \uc624\uc2a4\ud2b8\ub9ac\uc544 \ud5dd\uac00\ub9ac \uc81c\uad6d \uc721\uad70\uc5d0 \uc790\uc6d0\ud558\uc600\ub2e4. \uc785\ub300 \ucd08\uae30\uc5d0\ub294 \ud568\uc120\uc5d0\uc11c \uadfc\ubb34\ud558\uc600\uc73c\ub098 \uc5bc\ub9c8 \ud6c4 \ud3ec\ubcd1\ub300\ub85c \uc804\uadfc\ud558\uc600\ub2e4. 1916\ub144 3\uc6d4\uc5d0 \ube44\ud2b8\uac90\uc288\ud0c0\uc778\uc740 \uc624\uc2a4\ud2b8\ub9ac\uc544 7\uad70\uc5d0 \uc790\uc6d0\ud558\uc5ec \ube0c\ub8e8\uc2e4\ub85c\ud504 \uacf5\uc138\ub97c \ubc29\uc5b4\ud558\ub294 \ucd5c\uc804\ubc29 \uc870\uc885\uc0ac\ub85c \ucc38\uc804\ud558\uc600\ub2e4. \uc601\uad6d\uad70\uacfc\uc758 \uc804\ud22c \ud6c4\uc5d0 \ube44\ud2b8\uac90\uc288\ud0c0\uc778\uc740 \"\uc6a9\uac10\ud55c \ud589\ub3d9\uacfc \uce68\ucc29\ud558\uace0 \ub0c9\uc815\ud55c \uc601\uc6c5\ub2e4\uc6b4 \ud65c\uc57d\"\uc73c\ub85c \ub9d0\ubbf8\uc554\uc544 \uc218\ud6c8\ud588\ub2e4. 1917\ub144 1\uc6d4 \ube44\ud2b8\uac90\uc288\ud0c0\uc778\uc740 \ub7ec\uc2dc\uc544 \uc804\uc120\uc758 \uace1\uc0ac\ud3ec \uc5f0\ub300\ub85c \uc804\uadfc\ub418\uc5c8\uace0 \uc5ec\uae30\uc11c \ub2e4\uc2dc \uc6a9\uac10\ud55c \ud589\ub3d9\uc73c\ub85c \uc740\uc7a5\ubb34\uacf5\ud6c8\uc7a5 \uc744 \ube44\ub86f\ud574 \ud6c8\uc7a5 \uc5ec\ub7ec \uac1c\ub97c \ubc1b\uc558\ub2e4. 1918\ub144 \ube44\ud2b8\uac90\uc288\ud0c0\uc778\uc740 \ub300\uc704\ub85c \uc2b9\uc9c4\ud558\uc600\uace0 \uc624\ub298\ub0a0 \ud2b8\ub80c\ud2f0\ub178\ub85c \ubd88\ub9ac\ub294 \uc774\ud0c8\ub9ac\uc544 \ud2f0\ub864 \ub0a8\ubd80 \uc804\uc5ed\uc5d0\uc11c \uc0b0\uc545 \ud3ec\ubcd1 \uc5f0\ub300\uc5d0 \ubc30\uc18d\ub418\uc5c8\ub2e4. 1918\ub144 6\uc6d4 \uc624\uc2a4\ud2b8\ub9ac\uc544\uc758 \uacf5\uc138\uc5d0\uc11c \uc804\ud6c8\uc744 \uc138\uc6b4 \ube44\ud2b8\uac90\uc288\ud0c0\uc778\uc740 \ucd5c\uace0 \ud6c8\uc7a5\uc778 \uae08\uc7a5\ubb34\uacf5\ud6c8\uc7a5\uc5d0 \ucd94\ucc9c\ub418\uc5c8\uace0 \uadf8 \uc544\ub798 \ub2e8\uacc4\uc778 \uac80\uc758 \ubb34\uacf5\ud6c8\uc7a5\uc744 \ubc1b\uc558\ub2e4. \ube44\ud2b8\uac90\uc288\ud0c0\uc778\uc740 1918\ub144 11\uc6d4 \ud2b8\ub80c\ud1a0 \uadfc\uad50\uc5d0\uc11c \uc774\ud0c8\ub9ac\uc544\uad70\uc758 \ud3ec\ub85c\uac00 \ub418\uc5c8\ub2e4.", "paragraph_answer": "\uc81c1\ucc28 \uc138\uacc4\ub300\uc804\uc774 \uc77c\uc5b4\ub09c 1914\ub144\uc5d0 \ube44\ud2b8\uac90\uc288\ud0c0\uc778\uc740 1\ub144\uc9f8 \uc740\ub454 \uc0dd\ud65c\ud558\uace0 \uc788\uc5c8\ub2e4. \ube44\ud2b8\uac90\uc288\ud0c0\uc778\uc740 \uc804\uc7c1\uc774 \uc77c\uc5b4\ub0ac\ub2e4\ub294 \uc18c\uc2dd\uc5d0 \ud06c\uac8c \ucda9\uaca9\ubc1b\uc558\uace0 \uc624\uc2a4\ud2b8\ub9ac\uc544 \ud5dd\uac00\ub9ac \uc81c\uad6d \uc721\uad70\uc5d0 \uc790\uc6d0\ud558\uc600\ub2e4. \uc785\ub300 \ucd08\uae30\uc5d0\ub294 \ud568\uc120\uc5d0\uc11c \uadfc\ubb34\ud558\uc600\uc73c\ub098 \uc5bc\ub9c8 \ud6c4 \ud3ec\ubcd1\ub300\ub85c \uc804\uadfc\ud558\uc600\ub2e4. 1916\ub144 3\uc6d4\uc5d0 \ube44\ud2b8\uac90\uc288\ud0c0\uc778\uc740 \uc624\uc2a4\ud2b8\ub9ac\uc544 7\uad70\uc5d0 \uc790\uc6d0\ud558\uc5ec \ube0c\ub8e8\uc2e4\ub85c\ud504 \uacf5\uc138\ub97c \ubc29\uc5b4\ud558\ub294 \ucd5c\uc804\ubc29 \uc870\uc885\uc0ac\ub85c \ucc38\uc804\ud558\uc600\ub2e4. \uc601\uad6d\uad70\uacfc\uc758 \uc804\ud22c \ud6c4\uc5d0 \ube44\ud2b8\uac90\uc288\ud0c0\uc778\uc740 \"\uc6a9\uac10\ud55c \ud589\ub3d9\uacfc \uce68\ucc29\ud558\uace0 \ub0c9\uc815\ud55c \uc601\uc6c5\ub2e4\uc6b4 \ud65c\uc57d\"\uc73c\ub85c \ub9d0\ubbf8\uc554\uc544 \uc218\ud6c8\ud588\ub2e4. 1917\ub144 1\uc6d4 \ube44\ud2b8\uac90\uc288\ud0c0\uc778\uc740 \ub7ec\uc2dc\uc544 \uc804\uc120\uc758 \uace1\uc0ac\ud3ec \uc5f0\ub300\ub85c \uc804\uadfc\ub418\uc5c8\uace0 \uc5ec\uae30\uc11c \ub2e4\uc2dc \uc6a9\uac10\ud55c \ud589\ub3d9\uc73c\ub85c \uc740\uc7a5\ubb34\uacf5\ud6c8\uc7a5 \uc744 \ube44\ub86f\ud574 \ud6c8\uc7a5 \uc5ec\ub7ec \uac1c\ub97c \ubc1b\uc558\ub2e4. 1918\ub144 \ube44\ud2b8\uac90\uc288\ud0c0\uc778\uc740 \ub300\uc704\ub85c \uc2b9\uc9c4\ud558\uc600\uace0 \uc624\ub298\ub0a0 \ud2b8\ub80c\ud2f0\ub178\ub85c \ubd88\ub9ac\ub294 \uc774\ud0c8\ub9ac\uc544 \ud2f0\ub864 \ub0a8\ubd80 \uc804\uc5ed\uc5d0\uc11c \uc0b0\uc545 \ud3ec\ubcd1 \uc5f0\ub300\uc5d0 \ubc30\uc18d\ub418\uc5c8\ub2e4. 1918\ub144 6\uc6d4 \uc624\uc2a4\ud2b8\ub9ac\uc544\uc758 \uacf5\uc138\uc5d0\uc11c \uc804\ud6c8\uc744 \uc138\uc6b4 \ube44\ud2b8\uac90\uc288\ud0c0\uc778\uc740 \ucd5c\uace0 \ud6c8\uc7a5\uc778 \uae08\uc7a5\ubb34\uacf5\ud6c8\uc7a5\uc5d0 \ucd94\ucc9c\ub418\uc5c8\uace0 \uadf8 \uc544\ub798 \ub2e8\uacc4\uc778 \uac80\uc758 \ubb34\uacf5\ud6c8\uc7a5\uc744 \ubc1b\uc558\ub2e4. \ube44\ud2b8\uac90\uc288\ud0c0\uc778\uc740 1918\ub144 11\uc6d4 \ud2b8\ub80c\ud1a0 \uadfc\uad50\uc5d0\uc11c \uc774\ud0c8\ub9ac\uc544\uad70\uc758 \ud3ec\ub85c\uac00 \ub418\uc5c8\ub2e4.", "sentence_answer": "\uc5ec\uae30\uc11c \ub2e4\uc2dc \uc6a9\uac10\ud55c \ud589\ub3d9\uc73c\ub85c \uc740\uc7a5\ubb34\uacf5\ud6c8\uc7a5 \uc744 \ube44\ub86f\ud574 \ud6c8\uc7a5 \uc5ec\ub7ec \uac1c\ub97c \ubc1b\uc558\ub2e4.", "paragraph_id": "6497900-4-2", "paragraph_question": "question: \ub7ec\uc2dc\uc544 \uc804\uc120\uc758 \uace1\uc0ac\ud3ec \uc5f0\ub300\uc5d0\uc11c \ube44\ud2b8\uac90\uc288\ud0c0\uc778\uc774 \ubc1b\uc740 \ud6c8\uc7a5\uc740?, context: \uc81c1\ucc28 \uc138\uacc4\ub300\uc804\uc774 \uc77c\uc5b4\ub09c 1914\ub144\uc5d0 \ube44\ud2b8\uac90\uc288\ud0c0\uc778\uc740 1\ub144\uc9f8 \uc740\ub454 \uc0dd\ud65c\ud558\uace0 \uc788\uc5c8\ub2e4. \ube44\ud2b8\uac90\uc288\ud0c0\uc778\uc740 \uc804\uc7c1\uc774 \uc77c\uc5b4\ub0ac\ub2e4\ub294 \uc18c\uc2dd\uc5d0 \ud06c\uac8c \ucda9\uaca9\ubc1b\uc558\uace0 \uc624\uc2a4\ud2b8\ub9ac\uc544 \ud5dd\uac00\ub9ac \uc81c\uad6d \uc721\uad70\uc5d0 \uc790\uc6d0\ud558\uc600\ub2e4. \uc785\ub300 \ucd08\uae30\uc5d0\ub294 \ud568\uc120\uc5d0\uc11c \uadfc\ubb34\ud558\uc600\uc73c\ub098 \uc5bc\ub9c8 \ud6c4 \ud3ec\ubcd1\ub300\ub85c \uc804\uadfc\ud558\uc600\ub2e4. 1916\ub144 3\uc6d4\uc5d0 \ube44\ud2b8\uac90\uc288\ud0c0\uc778\uc740 \uc624\uc2a4\ud2b8\ub9ac\uc544 7\uad70\uc5d0 \uc790\uc6d0\ud558\uc5ec \ube0c\ub8e8\uc2e4\ub85c\ud504 \uacf5\uc138\ub97c \ubc29\uc5b4\ud558\ub294 \ucd5c\uc804\ubc29 \uc870\uc885\uc0ac\ub85c \ucc38\uc804\ud558\uc600\ub2e4. \uc601\uad6d\uad70\uacfc\uc758 \uc804\ud22c \ud6c4\uc5d0 \ube44\ud2b8\uac90\uc288\ud0c0\uc778\uc740 \"\uc6a9\uac10\ud55c \ud589\ub3d9\uacfc \uce68\ucc29\ud558\uace0 \ub0c9\uc815\ud55c \uc601\uc6c5\ub2e4\uc6b4 \ud65c\uc57d\"\uc73c\ub85c \ub9d0\ubbf8\uc554\uc544 \uc218\ud6c8\ud588\ub2e4. 1917\ub144 1\uc6d4 \ube44\ud2b8\uac90\uc288\ud0c0\uc778\uc740 \ub7ec\uc2dc\uc544 \uc804\uc120\uc758 \uace1\uc0ac\ud3ec \uc5f0\ub300\ub85c \uc804\uadfc\ub418\uc5c8\uace0 \uc5ec\uae30\uc11c \ub2e4\uc2dc \uc6a9\uac10\ud55c \ud589\ub3d9\uc73c\ub85c \uc740\uc7a5\ubb34\uacf5\ud6c8\uc7a5\uc744 \ube44\ub86f\ud574 \ud6c8\uc7a5 \uc5ec\ub7ec \uac1c\ub97c \ubc1b\uc558\ub2e4. 1918\ub144 \ube44\ud2b8\uac90\uc288\ud0c0\uc778\uc740 \ub300\uc704\ub85c \uc2b9\uc9c4\ud558\uc600\uace0 \uc624\ub298\ub0a0 \ud2b8\ub80c\ud2f0\ub178\ub85c \ubd88\ub9ac\ub294 \uc774\ud0c8\ub9ac\uc544 \ud2f0\ub864 \ub0a8\ubd80 \uc804\uc5ed\uc5d0\uc11c \uc0b0\uc545 \ud3ec\ubcd1 \uc5f0\ub300\uc5d0 \ubc30\uc18d\ub418\uc5c8\ub2e4. 1918\ub144 6\uc6d4 \uc624\uc2a4\ud2b8\ub9ac\uc544\uc758 \uacf5\uc138\uc5d0\uc11c \uc804\ud6c8\uc744 \uc138\uc6b4 \ube44\ud2b8\uac90\uc288\ud0c0\uc778\uc740 \ucd5c\uace0 \ud6c8\uc7a5\uc778 \uae08\uc7a5\ubb34\uacf5\ud6c8\uc7a5\uc5d0 \ucd94\ucc9c\ub418\uc5c8\uace0 \uadf8 \uc544\ub798 \ub2e8\uacc4\uc778 \uac80\uc758 \ubb34\uacf5\ud6c8\uc7a5\uc744 \ubc1b\uc558\ub2e4. \ube44\ud2b8\uac90\uc288\ud0c0\uc778\uc740 1918\ub144 11\uc6d4 \ud2b8\ub80c\ud1a0 \uadfc\uad50\uc5d0\uc11c \uc774\ud0c8\ub9ac\uc544\uad70\uc758 \ud3ec\ub85c\uac00 \ub418\uc5c8\ub2e4."} {"question": "\ub178\ub3d9\uc790 \ud074\ub7fd\uc744 \ub9cc\ub4e0 \uc54c\ubca0\ub974 \ub4dc\ubb11 \ubc31\uc791\uc758 \uace0\ud574 \uc2e0\ubd80 \uc774\ub984\uc740?", "paragraph": "\ub610 \ub4dc\ub808\ud4cc\uc2a4 \uc0ac\uac74\uc744 \ucd5c\ucd08\ub85c \ubcf4\ub3c4\ud55c \uc2e0\ubb38\uc740 \uc608\uc218\ud68c\uac00 \uc790\uae08\uc744 \uc9c0\uc6d0\ud55c <\ub77c \ub9ac\ube0c\ub974 \ube60\ub864(La Libre Parole)>\uc9c0\uc600\uace0 \ubc14\uc77c\ub9ac \uc2e0\ubd80\uac00 \ucc3d\uac04\ud55c<\ub77c \ud06c\ub85c\uc640>\ub4f1 \uc5ec\ud0c0 \uac00\ud1a8\ub9ad \uacc4\uc5f4 \uc2e0\ubb38\ub4e4\uc774 \uc55e\uc7a5\uc11c\uc11c \ub4dc\ub808\ud4cc\uc2a4\uc758 \uc720\uc8c4\ub97c \uc8fc\uc7a5\ud588\uace0 \ubc18\uc720\ud0dc\uc778 \uc815\uc11c\ub97c \ub0a0\ub9c8\ub2e4 \ub418\ud480\uc774\ud588\ub2e4. \uc774 <\ub77c \ub9ac\ube0c\ub974 \ube60\ub864>\uc9c0\uc758 \ubc1c\ud589\uc778 \uc5d0\ub450\uc544\ub974 \ub4dc\ub93c\ubabd(\u00c9douard Drumont)\uc5d0\uac8c \uc790\uae08\uc744 \ub300\uc900 \uc0ac\ub78c\uc774 \uc608\uc218\ud68c\uc758 \ub4a4\ub77d(Du Lac) \uc2e0\ubd80\uc600\ub2e4. \uadf8\ub294 \ub610 \ub2f9\uc2dc \uad6d\ud68c \ub0b4\uc758 \ucda9\uc2e4\ud55c \uac00\ud1a8\ub9ad \uad50\ud68c \uc639\ud638\uc790\uc774\uc790 '\ub178\ub3d9\uc790 \ud074\ub7fd'(\uc601\ubb38:The Society of Catholic Worker Circles, \ubd88\uc5b4:Oeuvre Circles Catholiques d'Ouvriers)\uc744 \ub9cc\ub4e0 \uc54c\ubca0\ub974 \ub4dc\ubb11(Albert de Mun)\ubc31\uc791\uacfc \ucc38\ubaa8\ubcf8\ubd80\uc758 \ucd1d\ucc38\ubaa8\uc7a5 \ub4dc \ube0c\uc640\ub370\ud504\ub974\uc758 \uace0\ud574\uc2e0\ubd80\uc774\uae30\ub3c4 \ud588\ub2e4.", "answer": "\ub4a4\ub77d", "sentence": "\uc774 <\ub77c \ub9ac\ube0c\ub974 \ube60\ub864>\uc9c0\uc758 \ubc1c\ud589\uc778 \uc5d0\ub450\uc544\ub974 \ub4dc\ub93c\ubabd(\u00c9douard Drumont)\uc5d0\uac8c \uc790\uae08\uc744 \ub300\uc900 \uc0ac\ub78c\uc774 \uc608\uc218\ud68c\uc758 \ub4a4\ub77d (Du Lac) \uc2e0\ubd80\uc600\ub2e4.", "paragraph_sentence": "\ub610 \ub4dc\ub808\ud4cc\uc2a4 \uc0ac\uac74\uc744 \ucd5c\ucd08\ub85c \ubcf4\ub3c4\ud55c \uc2e0\ubb38\uc740 \uc608\uc218\ud68c\uac00 \uc790\uae08\uc744 \uc9c0\uc6d0\ud55c <\ub77c \ub9ac\ube0c\ub974 \ube60\ub864(La Libre Parole)>\uc9c0\uc600\uace0 \ubc14\uc77c\ub9ac \uc2e0\ubd80\uac00 \ucc3d\uac04\ud55c<\ub77c \ud06c\ub85c\uc640>\ub4f1 \uc5ec\ud0c0 \uac00\ud1a8\ub9ad \uacc4\uc5f4 \uc2e0\ubb38\ub4e4\uc774 \uc55e\uc7a5\uc11c\uc11c \ub4dc\ub808\ud4cc\uc2a4\uc758 \uc720\uc8c4\ub97c \uc8fc\uc7a5\ud588\uace0 \ubc18\uc720\ud0dc\uc778 \uc815\uc11c\ub97c \ub0a0\ub9c8\ub2e4 \ub418\ud480\uc774\ud588\ub2e4. \uc774 <\ub77c \ub9ac\ube0c\ub974 \ube60\ub864>\uc9c0\uc758 \ubc1c\ud589\uc778 \uc5d0\ub450\uc544\ub974 \ub4dc\ub93c\ubabd(\u00c9douard Drumont)\uc5d0\uac8c \uc790\uae08\uc744 \ub300\uc900 \uc0ac\ub78c\uc774 \uc608\uc218\ud68c\uc758 \ub4a4\ub77d (Du Lac) \uc2e0\ubd80\uc600\ub2e4. \uadf8\ub294 \ub610 \ub2f9\uc2dc \uad6d\ud68c \ub0b4\uc758 \ucda9\uc2e4\ud55c \uac00\ud1a8\ub9ad \uad50\ud68c \uc639\ud638\uc790\uc774\uc790 '\ub178\ub3d9\uc790 \ud074\ub7fd'(\uc601\ubb38:The Society of Catholic Worker Circles, \ubd88\uc5b4:Oeuvre Circles Catholiques d'Ouvriers)\uc744 \ub9cc\ub4e0 \uc54c\ubca0\ub974 \ub4dc\ubb11(Albert de Mun)\ubc31\uc791\uacfc \ucc38\ubaa8\ubcf8\ubd80\uc758 \ucd1d\ucc38\ubaa8\uc7a5 \ub4dc \ube0c\uc640\ub370\ud504\ub974\uc758 \uace0\ud574\uc2e0\ubd80\uc774\uae30\ub3c4 \ud588\ub2e4.", "paragraph_answer": "\ub610 \ub4dc\ub808\ud4cc\uc2a4 \uc0ac\uac74\uc744 \ucd5c\ucd08\ub85c \ubcf4\ub3c4\ud55c \uc2e0\ubb38\uc740 \uc608\uc218\ud68c\uac00 \uc790\uae08\uc744 \uc9c0\uc6d0\ud55c <\ub77c \ub9ac\ube0c\ub974 \ube60\ub864(La Libre Parole)>\uc9c0\uc600\uace0 \ubc14\uc77c\ub9ac \uc2e0\ubd80\uac00 \ucc3d\uac04\ud55c<\ub77c \ud06c\ub85c\uc640>\ub4f1 \uc5ec\ud0c0 \uac00\ud1a8\ub9ad \uacc4\uc5f4 \uc2e0\ubb38\ub4e4\uc774 \uc55e\uc7a5\uc11c\uc11c \ub4dc\ub808\ud4cc\uc2a4\uc758 \uc720\uc8c4\ub97c \uc8fc\uc7a5\ud588\uace0 \ubc18\uc720\ud0dc\uc778 \uc815\uc11c\ub97c \ub0a0\ub9c8\ub2e4 \ub418\ud480\uc774\ud588\ub2e4. \uc774 <\ub77c \ub9ac\ube0c\ub974 \ube60\ub864>\uc9c0\uc758 \ubc1c\ud589\uc778 \uc5d0\ub450\uc544\ub974 \ub4dc\ub93c\ubabd(\u00c9douard Drumont)\uc5d0\uac8c \uc790\uae08\uc744 \ub300\uc900 \uc0ac\ub78c\uc774 \uc608\uc218\ud68c\uc758 \ub4a4\ub77d (Du Lac) \uc2e0\ubd80\uc600\ub2e4. \uadf8\ub294 \ub610 \ub2f9\uc2dc \uad6d\ud68c \ub0b4\uc758 \ucda9\uc2e4\ud55c \uac00\ud1a8\ub9ad \uad50\ud68c \uc639\ud638\uc790\uc774\uc790 '\ub178\ub3d9\uc790 \ud074\ub7fd'(\uc601\ubb38:The Society of Catholic Worker Circles, \ubd88\uc5b4:Oeuvre Circles Catholiques d'Ouvriers)\uc744 \ub9cc\ub4e0 \uc54c\ubca0\ub974 \ub4dc\ubb11(Albert de Mun)\ubc31\uc791\uacfc \ucc38\ubaa8\ubcf8\ubd80\uc758 \ucd1d\ucc38\ubaa8\uc7a5 \ub4dc \ube0c\uc640\ub370\ud504\ub974\uc758 \uace0\ud574\uc2e0\ubd80\uc774\uae30\ub3c4 \ud588\ub2e4.", "sentence_answer": "\uc774 <\ub77c \ub9ac\ube0c\ub974 \ube60\ub864>\uc9c0\uc758 \ubc1c\ud589\uc778 \uc5d0\ub450\uc544\ub974 \ub4dc\ub93c\ubabd(\u00c9douard Drumont)\uc5d0\uac8c \uc790\uae08\uc744 \ub300\uc900 \uc0ac\ub78c\uc774 \uc608\uc218\ud68c\uc758 \ub4a4\ub77d (Du Lac) \uc2e0\ubd80\uc600\ub2e4.", "paragraph_id": "6579200-5-1", "paragraph_question": "question: \ub178\ub3d9\uc790 \ud074\ub7fd\uc744 \ub9cc\ub4e0 \uc54c\ubca0\ub974 \ub4dc\ubb11 \ubc31\uc791\uc758 \uace0\ud574 \uc2e0\ubd80 \uc774\ub984\uc740?, context: \ub610 \ub4dc\ub808\ud4cc\uc2a4 \uc0ac\uac74\uc744 \ucd5c\ucd08\ub85c \ubcf4\ub3c4\ud55c \uc2e0\ubb38\uc740 \uc608\uc218\ud68c\uac00 \uc790\uae08\uc744 \uc9c0\uc6d0\ud55c <\ub77c \ub9ac\ube0c\ub974 \ube60\ub864(La Libre Parole)>\uc9c0\uc600\uace0 \ubc14\uc77c\ub9ac \uc2e0\ubd80\uac00 \ucc3d\uac04\ud55c<\ub77c \ud06c\ub85c\uc640>\ub4f1 \uc5ec\ud0c0 \uac00\ud1a8\ub9ad \uacc4\uc5f4 \uc2e0\ubb38\ub4e4\uc774 \uc55e\uc7a5\uc11c\uc11c \ub4dc\ub808\ud4cc\uc2a4\uc758 \uc720\uc8c4\ub97c \uc8fc\uc7a5\ud588\uace0 \ubc18\uc720\ud0dc\uc778 \uc815\uc11c\ub97c \ub0a0\ub9c8\ub2e4 \ub418\ud480\uc774\ud588\ub2e4. \uc774 <\ub77c \ub9ac\ube0c\ub974 \ube60\ub864>\uc9c0\uc758 \ubc1c\ud589\uc778 \uc5d0\ub450\uc544\ub974 \ub4dc\ub93c\ubabd(\u00c9douard Drumont)\uc5d0\uac8c \uc790\uae08\uc744 \ub300\uc900 \uc0ac\ub78c\uc774 \uc608\uc218\ud68c\uc758 \ub4a4\ub77d(Du Lac) \uc2e0\ubd80\uc600\ub2e4. \uadf8\ub294 \ub610 \ub2f9\uc2dc \uad6d\ud68c \ub0b4\uc758 \ucda9\uc2e4\ud55c \uac00\ud1a8\ub9ad \uad50\ud68c \uc639\ud638\uc790\uc774\uc790 '\ub178\ub3d9\uc790 \ud074\ub7fd'(\uc601\ubb38:The Society of Catholic Worker Circles, \ubd88\uc5b4:Oeuvre Circles Catholiques d'Ouvriers)\uc744 \ub9cc\ub4e0 \uc54c\ubca0\ub974 \ub4dc\ubb11(Albert de Mun)\ubc31\uc791\uacfc \ucc38\ubaa8\ubcf8\ubd80\uc758 \ucd1d\ucc38\ubaa8\uc7a5 \ub4dc \ube0c\uc640\ub370\ud504\ub974\uc758 \uace0\ud574\uc2e0\ubd80\uc774\uae30\ub3c4 \ud588\ub2e4."} {"question": "\ub450 \ubc88\uc9f8 \uc2dc\uc98c\uc5d0 \ucc38\uc5ec\ud55c \uac8c\uc2a4\ud2b8 \uc560\ub2c8\uba54\uc774\ud130\ub294?", "paragraph": "\uc77c\ubd80 \uc5d0\ud53c\uc18c\ub4dc\uc5d0\uc11c\ub294 \uac8c\uc2a4\ud2b8 \uc560\ub2c8\uba54\uc774\ud130\uac00 \uc81c\uc791\uc5d0 \ucc38\uc5ec\ud55c\ub2e4. \ub450 \ubc88\uc9f8 \uc2dc\uc98c\uc758 \u3008Guardians of Sunshine\u3009\uc5d0\uc11c\ub294 \ube44\ubaa8 \ubab8 \uc18d\uc758 \uac8c\uc784 \uc7a5\uba74\uc5d0\uc11c \ucee4 \uc7a5(Ke Jiang)\uc758 3D \uc560\ub2c8\uba54\uc774\uc158\uc774 \uc0ac\uc6a9\ub418\uc5c8\ub2e4. \ub2e4\uc12f \ubc88\uc9f8 \uc2dc\uc98c\uc758 \u3008A Glitch is a Glitch\u3009\ub294 \uc544\uc77c\ub79c\ub4dc\uc758 \uc601\ud654 \uc81c\uc791\uc790\uc778 \ub370\uc774\ube44\ub4dc \uc624\ub77c\uc77c\ub9ac(David OReilly)\uac00 3D \uc560\ub2c8\uba54\uc774\uc158\uc744 \uc774\uc6a9\ud558\uc5ec \uc81c\uc791\ud588\ub2e4. \uc560\ub2c8\uba54\uc774\ud130 \uc81c\uc784\uc2a4 \ubca1\uc2a4\ud130(James Baxter)\ub294 \ub2e4\uc12f \ubc88\uc9f8 \uc2dc\uc98c \u3008James Baxter The Horse\u3009\uc640 \uc5ec\ub35f \ubc88\uc9f8 \uc2dc\uc98c \u3008Horse & Ball\u3009 \uc81c\uc791\uc5d0 \ucc38\uc5ec\ud588\ub2e4. \uc5ec\uc12f \ubc88\uc9f8 \uc2dc\uc98c\uc758 \u3008Food Chain\u3009\uc740 \uc77c\ubcf8\uc758 \uc720\uc544\uc0ac \ub9c8\uc0ac\uc544\ud0a4\uac00 \uc790\uc2e0\uc758 \uc5d0\ud53c\uc18c\ub4dc \uc2a4\ud29c\ub514\uc624\uc5d0\uc11c \uc5d0\ud53c\uc18c\ub4dc \uc804\uccb4\ub97c \uc81c\uc791\ud558\uc600\ub2e4. \uac19\uc740 \uc2dc\uc98c\uc758 \u3008Water Park Prank\u3009\uc5d0\uc11c\ub294 \ub370\uc774\ube44\ub4dc \ud37c\uac70\uc2a8(David Ferguson)\uc758 \ud50c\ub798\uc2dc \uc560\ub2c8\uba54\uc774\uc158\uc774 \uc0ac\uc6a9\ub418\uc5c8\ub2e4. \uc77c\uacf1 \ubc88\uc9f8 \uc2dc\uc98c\uc5d0\uc11c\ub294 \ucee4\uc2a4\ud2f4 \ub808\ud3ec\uc5b4\uac00 \uc81c\uc791\ud55c \uc2a4\ud1b1\ubaa8\uc158 \uc5d0\ud53c\uc18c\ub4dc\uc778 \u3008Bad Jubies\u3009\uac00 \ubc29\uc601\ub418\uc5c8\ub2e4. \uac19\uc740 \uc2dc\uc98c \u3008Beyond the Grotto\u3009\uc5d0\uc11c\ub294 \uc6f9 \uc560\ub2c8\uba54\uc774\uc158 \u300a\ubc30\ub9e8 \ud30c\uc774\ub354\ub9e8\u300b(Baman Piderman)\uc73c\ub85c \uc720\uba85\ud55c \uc54c\ub809\uc2a4 \ubd80\ud14c\ub77c(Alex Butera)\uc640 \ub9b0\uc9c0 \uc2a4\ubab0(Lindsay Small)\uc774 \uc81c\uc791\uc5d0 \ucc38\uc5ec\ud558\uc600\ub2e4.", "answer": "\ucee4 \uc7a5", "sentence": "\ub450 \ubc88\uc9f8 \uc2dc\uc98c\uc758 \u3008Guardians of Sunshine\u3009\uc5d0\uc11c\ub294 \ube44\ubaa8 \ubab8 \uc18d\uc758 \uac8c\uc784 \uc7a5\uba74\uc5d0\uc11c \ucee4 \uc7a5 (Ke Jiang)", "paragraph_sentence": "\uc77c\ubd80 \uc5d0\ud53c\uc18c\ub4dc\uc5d0\uc11c\ub294 \uac8c\uc2a4\ud2b8 \uc560\ub2c8\uba54\uc774\ud130\uac00 \uc81c\uc791\uc5d0 \ucc38\uc5ec\ud55c\ub2e4. \ub450 \ubc88\uc9f8 \uc2dc\uc98c\uc758 \u3008Guardians of Sunshine\u3009\uc5d0\uc11c\ub294 \ube44\ubaa8 \ubab8 \uc18d\uc758 \uac8c\uc784 \uc7a5\uba74\uc5d0\uc11c \ucee4 \uc7a5 (Ke Jiang) \uc758 3D \uc560\ub2c8\uba54\uc774\uc158\uc774 \uc0ac\uc6a9\ub418\uc5c8\ub2e4. \ub2e4\uc12f \ubc88\uc9f8 \uc2dc\uc98c\uc758 \u3008A Glitch is a Glitch\u3009\ub294 \uc544\uc77c\ub79c\ub4dc\uc758 \uc601\ud654 \uc81c\uc791\uc790\uc778 \ub370\uc774\ube44\ub4dc \uc624\ub77c\uc77c\ub9ac(David OReilly)\uac00 3D \uc560\ub2c8\uba54\uc774\uc158\uc744 \uc774\uc6a9\ud558\uc5ec \uc81c\uc791\ud588\ub2e4. \uc560\ub2c8\uba54\uc774\ud130 \uc81c\uc784\uc2a4 \ubca1\uc2a4\ud130(James Baxter)\ub294 \ub2e4\uc12f \ubc88\uc9f8 \uc2dc\uc98c \u3008James Baxter The Horse\u3009\uc640 \uc5ec\ub35f \ubc88\uc9f8 \uc2dc\uc98c \u3008Horse & Ball\u3009 \uc81c\uc791\uc5d0 \ucc38\uc5ec\ud588\ub2e4. \uc5ec\uc12f \ubc88\uc9f8 \uc2dc\uc98c\uc758 \u3008Food Chain\u3009\uc740 \uc77c\ubcf8\uc758 \uc720\uc544\uc0ac \ub9c8\uc0ac\uc544\ud0a4\uac00 \uc790\uc2e0\uc758 \uc5d0\ud53c\uc18c\ub4dc \uc2a4\ud29c\ub514\uc624\uc5d0\uc11c \uc5d0\ud53c\uc18c\ub4dc \uc804\uccb4\ub97c \uc81c\uc791\ud558\uc600\ub2e4. \uac19\uc740 \uc2dc\uc98c\uc758 \u3008Water Park Prank\u3009\uc5d0\uc11c\ub294 \ub370\uc774\ube44\ub4dc \ud37c\uac70\uc2a8(David Ferguson)\uc758 \ud50c\ub798\uc2dc \uc560\ub2c8\uba54\uc774\uc158\uc774 \uc0ac\uc6a9\ub418\uc5c8\ub2e4. \uc77c\uacf1 \ubc88\uc9f8 \uc2dc\uc98c\uc5d0\uc11c\ub294 \ucee4\uc2a4\ud2f4 \ub808\ud3ec\uc5b4\uac00 \uc81c\uc791\ud55c \uc2a4\ud1b1\ubaa8\uc158 \uc5d0\ud53c\uc18c\ub4dc\uc778 \u3008Bad Jubies\u3009\uac00 \ubc29\uc601\ub418\uc5c8\ub2e4. \uac19\uc740 \uc2dc\uc98c \u3008Beyond the Grotto\u3009\uc5d0\uc11c\ub294 \uc6f9 \uc560\ub2c8\uba54\uc774\uc158 \u300a\ubc30\ub9e8 \ud30c\uc774\ub354\ub9e8\u300b(Baman Piderman)\uc73c\ub85c \uc720\uba85\ud55c \uc54c\ub809\uc2a4 \ubd80\ud14c\ub77c(Alex Butera)\uc640 \ub9b0\uc9c0 \uc2a4\ubab0(Lindsay Small)\uc774 \uc81c\uc791\uc5d0 \ucc38\uc5ec\ud558\uc600\ub2e4.", "paragraph_answer": "\uc77c\ubd80 \uc5d0\ud53c\uc18c\ub4dc\uc5d0\uc11c\ub294 \uac8c\uc2a4\ud2b8 \uc560\ub2c8\uba54\uc774\ud130\uac00 \uc81c\uc791\uc5d0 \ucc38\uc5ec\ud55c\ub2e4. \ub450 \ubc88\uc9f8 \uc2dc\uc98c\uc758 \u3008Guardians of Sunshine\u3009\uc5d0\uc11c\ub294 \ube44\ubaa8 \ubab8 \uc18d\uc758 \uac8c\uc784 \uc7a5\uba74\uc5d0\uc11c \ucee4 \uc7a5 (Ke Jiang)\uc758 3D \uc560\ub2c8\uba54\uc774\uc158\uc774 \uc0ac\uc6a9\ub418\uc5c8\ub2e4. \ub2e4\uc12f \ubc88\uc9f8 \uc2dc\uc98c\uc758 \u3008A Glitch is a Glitch\u3009\ub294 \uc544\uc77c\ub79c\ub4dc\uc758 \uc601\ud654 \uc81c\uc791\uc790\uc778 \ub370\uc774\ube44\ub4dc \uc624\ub77c\uc77c\ub9ac(David OReilly)\uac00 3D \uc560\ub2c8\uba54\uc774\uc158\uc744 \uc774\uc6a9\ud558\uc5ec \uc81c\uc791\ud588\ub2e4. \uc560\ub2c8\uba54\uc774\ud130 \uc81c\uc784\uc2a4 \ubca1\uc2a4\ud130(James Baxter)\ub294 \ub2e4\uc12f \ubc88\uc9f8 \uc2dc\uc98c \u3008James Baxter The Horse\u3009\uc640 \uc5ec\ub35f \ubc88\uc9f8 \uc2dc\uc98c \u3008Horse & Ball\u3009 \uc81c\uc791\uc5d0 \ucc38\uc5ec\ud588\ub2e4. \uc5ec\uc12f \ubc88\uc9f8 \uc2dc\uc98c\uc758 \u3008Food Chain\u3009\uc740 \uc77c\ubcf8\uc758 \uc720\uc544\uc0ac \ub9c8\uc0ac\uc544\ud0a4\uac00 \uc790\uc2e0\uc758 \uc5d0\ud53c\uc18c\ub4dc \uc2a4\ud29c\ub514\uc624\uc5d0\uc11c \uc5d0\ud53c\uc18c\ub4dc \uc804\uccb4\ub97c \uc81c\uc791\ud558\uc600\ub2e4. \uac19\uc740 \uc2dc\uc98c\uc758 \u3008Water Park Prank\u3009\uc5d0\uc11c\ub294 \ub370\uc774\ube44\ub4dc \ud37c\uac70\uc2a8(David Ferguson)\uc758 \ud50c\ub798\uc2dc \uc560\ub2c8\uba54\uc774\uc158\uc774 \uc0ac\uc6a9\ub418\uc5c8\ub2e4. \uc77c\uacf1 \ubc88\uc9f8 \uc2dc\uc98c\uc5d0\uc11c\ub294 \ucee4\uc2a4\ud2f4 \ub808\ud3ec\uc5b4\uac00 \uc81c\uc791\ud55c \uc2a4\ud1b1\ubaa8\uc158 \uc5d0\ud53c\uc18c\ub4dc\uc778 \u3008Bad Jubies\u3009\uac00 \ubc29\uc601\ub418\uc5c8\ub2e4. \uac19\uc740 \uc2dc\uc98c \u3008Beyond the Grotto\u3009\uc5d0\uc11c\ub294 \uc6f9 \uc560\ub2c8\uba54\uc774\uc158 \u300a\ubc30\ub9e8 \ud30c\uc774\ub354\ub9e8\u300b(Baman Piderman)\uc73c\ub85c \uc720\uba85\ud55c \uc54c\ub809\uc2a4 \ubd80\ud14c\ub77c(Alex Butera)\uc640 \ub9b0\uc9c0 \uc2a4\ubab0(Lindsay Small)\uc774 \uc81c\uc791\uc5d0 \ucc38\uc5ec\ud558\uc600\ub2e4.", "sentence_answer": "\ub450 \ubc88\uc9f8 \uc2dc\uc98c\uc758 \u3008Guardians of Sunshine\u3009\uc5d0\uc11c\ub294 \ube44\ubaa8 \ubab8 \uc18d\uc758 \uac8c\uc784 \uc7a5\uba74\uc5d0\uc11c \ucee4 \uc7a5 (Ke Jiang)", "paragraph_id": "6569103-9-0", "paragraph_question": "question: \ub450 \ubc88\uc9f8 \uc2dc\uc98c\uc5d0 \ucc38\uc5ec\ud55c \uac8c\uc2a4\ud2b8 \uc560\ub2c8\uba54\uc774\ud130\ub294?, context: \uc77c\ubd80 \uc5d0\ud53c\uc18c\ub4dc\uc5d0\uc11c\ub294 \uac8c\uc2a4\ud2b8 \uc560\ub2c8\uba54\uc774\ud130\uac00 \uc81c\uc791\uc5d0 \ucc38\uc5ec\ud55c\ub2e4. \ub450 \ubc88\uc9f8 \uc2dc\uc98c\uc758 \u3008Guardians of Sunshine\u3009\uc5d0\uc11c\ub294 \ube44\ubaa8 \ubab8 \uc18d\uc758 \uac8c\uc784 \uc7a5\uba74\uc5d0\uc11c \ucee4 \uc7a5(Ke Jiang)\uc758 3D \uc560\ub2c8\uba54\uc774\uc158\uc774 \uc0ac\uc6a9\ub418\uc5c8\ub2e4. \ub2e4\uc12f \ubc88\uc9f8 \uc2dc\uc98c\uc758 \u3008A Glitch is a Glitch\u3009\ub294 \uc544\uc77c\ub79c\ub4dc\uc758 \uc601\ud654 \uc81c\uc791\uc790\uc778 \ub370\uc774\ube44\ub4dc \uc624\ub77c\uc77c\ub9ac(David OReilly)\uac00 3D \uc560\ub2c8\uba54\uc774\uc158\uc744 \uc774\uc6a9\ud558\uc5ec \uc81c\uc791\ud588\ub2e4. \uc560\ub2c8\uba54\uc774\ud130 \uc81c\uc784\uc2a4 \ubca1\uc2a4\ud130(James Baxter)\ub294 \ub2e4\uc12f \ubc88\uc9f8 \uc2dc\uc98c \u3008James Baxter The Horse\u3009\uc640 \uc5ec\ub35f \ubc88\uc9f8 \uc2dc\uc98c \u3008Horse & Ball\u3009 \uc81c\uc791\uc5d0 \ucc38\uc5ec\ud588\ub2e4. \uc5ec\uc12f \ubc88\uc9f8 \uc2dc\uc98c\uc758 \u3008Food Chain\u3009\uc740 \uc77c\ubcf8\uc758 \uc720\uc544\uc0ac \ub9c8\uc0ac\uc544\ud0a4\uac00 \uc790\uc2e0\uc758 \uc5d0\ud53c\uc18c\ub4dc \uc2a4\ud29c\ub514\uc624\uc5d0\uc11c \uc5d0\ud53c\uc18c\ub4dc \uc804\uccb4\ub97c \uc81c\uc791\ud558\uc600\ub2e4. \uac19\uc740 \uc2dc\uc98c\uc758 \u3008Water Park Prank\u3009\uc5d0\uc11c\ub294 \ub370\uc774\ube44\ub4dc \ud37c\uac70\uc2a8(David Ferguson)\uc758 \ud50c\ub798\uc2dc \uc560\ub2c8\uba54\uc774\uc158\uc774 \uc0ac\uc6a9\ub418\uc5c8\ub2e4. \uc77c\uacf1 \ubc88\uc9f8 \uc2dc\uc98c\uc5d0\uc11c\ub294 \ucee4\uc2a4\ud2f4 \ub808\ud3ec\uc5b4\uac00 \uc81c\uc791\ud55c \uc2a4\ud1b1\ubaa8\uc158 \uc5d0\ud53c\uc18c\ub4dc\uc778 \u3008Bad Jubies\u3009\uac00 \ubc29\uc601\ub418\uc5c8\ub2e4. \uac19\uc740 \uc2dc\uc98c \u3008Beyond the Grotto\u3009\uc5d0\uc11c\ub294 \uc6f9 \uc560\ub2c8\uba54\uc774\uc158 \u300a\ubc30\ub9e8 \ud30c\uc774\ub354\ub9e8\u300b(Baman Piderman)\uc73c\ub85c \uc720\uba85\ud55c \uc54c\ub809\uc2a4 \ubd80\ud14c\ub77c(Alex Butera)\uc640 \ub9b0\uc9c0 \uc2a4\ubab0(Lindsay Small)\uc774 \uc81c\uc791\uc5d0 \ucc38\uc5ec\ud558\uc600\ub2e4."} {"question": "1990\ub144 \uc2a4\ud504\ub9c1 \ucea0\ud504\uc5d0\uc11c \ud314\ub85c \uc2a4\ub8e8\ub97c \uc9c0\ub3c4\ud574\uc900 \ucf54\uce58\ub294?", "paragraph": "1990\ub144, \uc2a4\ud504\ub9c1 \ucea0\ud504\uc5d0\uc11c \ud22c\uc218 \ucf54\uce58\uc600\ub358 \uc774\ucf00\ub2e4 \ud788\ub370\ud1a0\uc2dc\uc5d0\uac8c\uc11c \ud314\ub85c \uc2a4\ub8e8 \ub4f1\uc744 \uc9c0\ub3c4\ubc1b\uc544 \uc9c1\uad6c\uc758 \uc131\uc7a5\uacfc \ubcc0\ud654\uad6c\uc758 \uaebe\uc784\uc774 \uac1c\uc120\ub410\uace0 \ub610\ud55c \uc2f1\ucee4\uc758 \uc2b5\ub4dd\uc5d0\ub3c4 \ub3c4\uc804\ud588\ub2e4. 3\uc6d4 4\uc77c\uc5d0 \uc788\uc740 \uc218\ube44 \uc5f0\uc2b5\uc5d0\uc11c \ub178\ud06c\ub41c \uacf5\uc744 \ubb34\ub98e\uc5d0 \ub9de\uc544 \ud22c\uad6c\ub294 \ucee4\ub155 \uce90\uce58\ubcfc\uc870\ucc28 \ud558\uc9c0 \ubabb\ud558\ub294 \ub0a0\uc774 \uc774\uc5b4\uc84c\uc9c0\ub9cc 3\uc6d4 24\uc77c \ub3c4\uc694\ud558\uc2dc \uc2dc\ubbfc \uad6c\uc7a5\uc5d0\uc11c \uc5f4\ub9b0 \ud6c4\ucfe0\uc624\uce74 \ub2e4\uc774\uc5d0 \ud638\ud06c\uc2a4\uc640\uc758 \uc2dc\ubc94 \uacbd\uae30\uc5d0\uc11c \ubb34\ub77c\ud0c0 \uac00\uc4f0\uc694\uc2dc\uc640 \ud22c\uc218\uc804\uc744 \ud3bc\uccd0 5\uc774\ub2dd \ub3d9\uc548 61\uac1c\uc758 \uacf5\uc744 \ub358\uc9c0\uba74\uc11c 2\ud53c\uc548\ud0c0 \ubb34\ubcfc\ub137\uacfc \ubb34\uc2e4\uc810\uc758 \ud638\ud22c\ub97c \ubcf4\uc600\ub2e4. \uac1c\ub9c9 \uc774\ud6c4\uc5d0\ub294 \uc7a5\ub798 \uc88c\uc644 \uc5d0\uc774\uc2a4\ub85c\uc11c\uc758 \uae30\ub300\ub97c \ud55c\ubab8\uc5d0 \ubc1b\uc544\uc11c \uc8fc\ub85c \uc120\ubc1c\ub85c\uc11c \uae30\uc6a9\ub410\ub2e4. \ub2f9\uc2dc\uc5d0\ub294 \uc544\uc9c1 \ub9c9\uc5f0\ud558\uac8c \ub358\uc9c0\ub294 \ub4ef\ud55c \uc810\uc774 \uc788\uc5c8\uc9c0\ub9cc \uc120\ubc1c \ub85c\ud14c\uc774\uc158\uc5d0 \uc815\ucc29\ud558\uc5ec 6\ucc28\ub840\uc758 \uc644\ud22c \ub4f1 \uaddc\uc815 \ud22c\uad6c \uc774\ub2dd\uc5d0 \ub3c4\ub2ec\ud574\uc11c 10\uc2b9(6\ud328)\uc744 \uae30\ub85d\ud588\ub2e4.", "answer": "\uc774\ucf00\ub2e4 \ud788\ub370\ud1a0\uc2dc", "sentence": "1990\ub144, \uc2a4\ud504\ub9c1 \ucea0\ud504\uc5d0\uc11c \ud22c\uc218 \ucf54\uce58\uc600\ub358 \uc774\ucf00\ub2e4 \ud788\ub370\ud1a0\uc2dc \uc5d0\uac8c\uc11c \ud314\ub85c \uc2a4\ub8e8 \ub4f1\uc744 \uc9c0\ub3c4\ubc1b\uc544 \uc9c1\uad6c\uc758 \uc131\uc7a5\uacfc \ubcc0\ud654\uad6c\uc758 \uaebe\uc784\uc774 \uac1c\uc120\ub410\uace0 \ub610\ud55c \uc2f1\ucee4\uc758 \uc2b5\ub4dd\uc5d0\ub3c4 \ub3c4\uc804\ud588\ub2e4.", "paragraph_sentence": " 1990\ub144, \uc2a4\ud504\ub9c1 \ucea0\ud504\uc5d0\uc11c \ud22c\uc218 \ucf54\uce58\uc600\ub358 \uc774\ucf00\ub2e4 \ud788\ub370\ud1a0\uc2dc \uc5d0\uac8c\uc11c \ud314\ub85c \uc2a4\ub8e8 \ub4f1\uc744 \uc9c0\ub3c4\ubc1b\uc544 \uc9c1\uad6c\uc758 \uc131\uc7a5\uacfc \ubcc0\ud654\uad6c\uc758 \uaebe\uc784\uc774 \uac1c\uc120\ub410\uace0 \ub610\ud55c \uc2f1\ucee4\uc758 \uc2b5\ub4dd\uc5d0\ub3c4 \ub3c4\uc804\ud588\ub2e4. 3\uc6d4 4\uc77c\uc5d0 \uc788\uc740 \uc218\ube44 \uc5f0\uc2b5\uc5d0\uc11c \ub178\ud06c\ub41c \uacf5\uc744 \ubb34\ub98e\uc5d0 \ub9de\uc544 \ud22c\uad6c\ub294 \ucee4\ub155 \uce90\uce58\ubcfc\uc870\ucc28 \ud558\uc9c0 \ubabb\ud558\ub294 \ub0a0\uc774 \uc774\uc5b4\uc84c\uc9c0\ub9cc 3\uc6d4 24\uc77c \ub3c4\uc694\ud558\uc2dc \uc2dc\ubbfc \uad6c\uc7a5\uc5d0\uc11c \uc5f4\ub9b0 \ud6c4\ucfe0\uc624\uce74 \ub2e4\uc774\uc5d0 \ud638\ud06c\uc2a4\uc640\uc758 \uc2dc\ubc94 \uacbd\uae30\uc5d0\uc11c \ubb34\ub77c\ud0c0 \uac00\uc4f0\uc694\uc2dc\uc640 \ud22c\uc218\uc804\uc744 \ud3bc\uccd0 5\uc774\ub2dd \ub3d9\uc548 61\uac1c\uc758 \uacf5\uc744 \ub358\uc9c0\uba74\uc11c 2\ud53c\uc548\ud0c0 \ubb34\ubcfc\ub137\uacfc \ubb34\uc2e4\uc810\uc758 \ud638\ud22c\ub97c \ubcf4\uc600\ub2e4. \uac1c\ub9c9 \uc774\ud6c4\uc5d0\ub294 \uc7a5\ub798 \uc88c\uc644 \uc5d0\uc774\uc2a4\ub85c\uc11c\uc758 \uae30\ub300\ub97c \ud55c\ubab8\uc5d0 \ubc1b\uc544\uc11c \uc8fc\ub85c \uc120\ubc1c\ub85c\uc11c \uae30\uc6a9\ub410\ub2e4. \ub2f9\uc2dc\uc5d0\ub294 \uc544\uc9c1 \ub9c9\uc5f0\ud558\uac8c \ub358\uc9c0\ub294 \ub4ef\ud55c \uc810\uc774 \uc788\uc5c8\uc9c0\ub9cc \uc120\ubc1c \ub85c\ud14c\uc774\uc158\uc5d0 \uc815\ucc29\ud558\uc5ec 6\ucc28\ub840\uc758 \uc644\ud22c \ub4f1 \uaddc\uc815 \ud22c\uad6c \uc774\ub2dd\uc5d0 \ub3c4\ub2ec\ud574\uc11c 10\uc2b9(6\ud328)\uc744 \uae30\ub85d\ud588\ub2e4.", "paragraph_answer": "1990\ub144, \uc2a4\ud504\ub9c1 \ucea0\ud504\uc5d0\uc11c \ud22c\uc218 \ucf54\uce58\uc600\ub358 \uc774\ucf00\ub2e4 \ud788\ub370\ud1a0\uc2dc \uc5d0\uac8c\uc11c \ud314\ub85c \uc2a4\ub8e8 \ub4f1\uc744 \uc9c0\ub3c4\ubc1b\uc544 \uc9c1\uad6c\uc758 \uc131\uc7a5\uacfc \ubcc0\ud654\uad6c\uc758 \uaebe\uc784\uc774 \uac1c\uc120\ub410\uace0 \ub610\ud55c \uc2f1\ucee4\uc758 \uc2b5\ub4dd\uc5d0\ub3c4 \ub3c4\uc804\ud588\ub2e4. 3\uc6d4 4\uc77c\uc5d0 \uc788\uc740 \uc218\ube44 \uc5f0\uc2b5\uc5d0\uc11c \ub178\ud06c\ub41c \uacf5\uc744 \ubb34\ub98e\uc5d0 \ub9de\uc544 \ud22c\uad6c\ub294 \ucee4\ub155 \uce90\uce58\ubcfc\uc870\ucc28 \ud558\uc9c0 \ubabb\ud558\ub294 \ub0a0\uc774 \uc774\uc5b4\uc84c\uc9c0\ub9cc 3\uc6d4 24\uc77c \ub3c4\uc694\ud558\uc2dc \uc2dc\ubbfc \uad6c\uc7a5\uc5d0\uc11c \uc5f4\ub9b0 \ud6c4\ucfe0\uc624\uce74 \ub2e4\uc774\uc5d0 \ud638\ud06c\uc2a4\uc640\uc758 \uc2dc\ubc94 \uacbd\uae30\uc5d0\uc11c \ubb34\ub77c\ud0c0 \uac00\uc4f0\uc694\uc2dc\uc640 \ud22c\uc218\uc804\uc744 \ud3bc\uccd0 5\uc774\ub2dd \ub3d9\uc548 61\uac1c\uc758 \uacf5\uc744 \ub358\uc9c0\uba74\uc11c 2\ud53c\uc548\ud0c0 \ubb34\ubcfc\ub137\uacfc \ubb34\uc2e4\uc810\uc758 \ud638\ud22c\ub97c \ubcf4\uc600\ub2e4. \uac1c\ub9c9 \uc774\ud6c4\uc5d0\ub294 \uc7a5\ub798 \uc88c\uc644 \uc5d0\uc774\uc2a4\ub85c\uc11c\uc758 \uae30\ub300\ub97c \ud55c\ubab8\uc5d0 \ubc1b\uc544\uc11c \uc8fc\ub85c \uc120\ubc1c\ub85c\uc11c \uae30\uc6a9\ub410\ub2e4. \ub2f9\uc2dc\uc5d0\ub294 \uc544\uc9c1 \ub9c9\uc5f0\ud558\uac8c \ub358\uc9c0\ub294 \ub4ef\ud55c \uc810\uc774 \uc788\uc5c8\uc9c0\ub9cc \uc120\ubc1c \ub85c\ud14c\uc774\uc158\uc5d0 \uc815\ucc29\ud558\uc5ec 6\ucc28\ub840\uc758 \uc644\ud22c \ub4f1 \uaddc\uc815 \ud22c\uad6c \uc774\ub2dd\uc5d0 \ub3c4\ub2ec\ud574\uc11c 10\uc2b9(6\ud328)\uc744 \uae30\ub85d\ud588\ub2e4.", "sentence_answer": "1990\ub144, \uc2a4\ud504\ub9c1 \ucea0\ud504\uc5d0\uc11c \ud22c\uc218 \ucf54\uce58\uc600\ub358 \uc774\ucf00\ub2e4 \ud788\ub370\ud1a0\uc2dc \uc5d0\uac8c\uc11c \ud314\ub85c \uc2a4\ub8e8 \ub4f1\uc744 \uc9c0\ub3c4\ubc1b\uc544 \uc9c1\uad6c\uc758 \uc131\uc7a5\uacfc \ubcc0\ud654\uad6c\uc758 \uaebe\uc784\uc774 \uac1c\uc120\ub410\uace0 \ub610\ud55c \uc2f1\ucee4\uc758 \uc2b5\ub4dd\uc5d0\ub3c4 \ub3c4\uc804\ud588\ub2e4.", "paragraph_id": "6545142-4-0", "paragraph_question": "question: 1990\ub144 \uc2a4\ud504\ub9c1 \ucea0\ud504\uc5d0\uc11c \ud314\ub85c \uc2a4\ub8e8\ub97c \uc9c0\ub3c4\ud574\uc900 \ucf54\uce58\ub294?, context: 1990\ub144, \uc2a4\ud504\ub9c1 \ucea0\ud504\uc5d0\uc11c \ud22c\uc218 \ucf54\uce58\uc600\ub358 \uc774\ucf00\ub2e4 \ud788\ub370\ud1a0\uc2dc\uc5d0\uac8c\uc11c \ud314\ub85c \uc2a4\ub8e8 \ub4f1\uc744 \uc9c0\ub3c4\ubc1b\uc544 \uc9c1\uad6c\uc758 \uc131\uc7a5\uacfc \ubcc0\ud654\uad6c\uc758 \uaebe\uc784\uc774 \uac1c\uc120\ub410\uace0 \ub610\ud55c \uc2f1\ucee4\uc758 \uc2b5\ub4dd\uc5d0\ub3c4 \ub3c4\uc804\ud588\ub2e4. 3\uc6d4 4\uc77c\uc5d0 \uc788\uc740 \uc218\ube44 \uc5f0\uc2b5\uc5d0\uc11c \ub178\ud06c\ub41c \uacf5\uc744 \ubb34\ub98e\uc5d0 \ub9de\uc544 \ud22c\uad6c\ub294 \ucee4\ub155 \uce90\uce58\ubcfc\uc870\ucc28 \ud558\uc9c0 \ubabb\ud558\ub294 \ub0a0\uc774 \uc774\uc5b4\uc84c\uc9c0\ub9cc 3\uc6d4 24\uc77c \ub3c4\uc694\ud558\uc2dc \uc2dc\ubbfc \uad6c\uc7a5\uc5d0\uc11c \uc5f4\ub9b0 \ud6c4\ucfe0\uc624\uce74 \ub2e4\uc774\uc5d0 \ud638\ud06c\uc2a4\uc640\uc758 \uc2dc\ubc94 \uacbd\uae30\uc5d0\uc11c \ubb34\ub77c\ud0c0 \uac00\uc4f0\uc694\uc2dc\uc640 \ud22c\uc218\uc804\uc744 \ud3bc\uccd0 5\uc774\ub2dd \ub3d9\uc548 61\uac1c\uc758 \uacf5\uc744 \ub358\uc9c0\uba74\uc11c 2\ud53c\uc548\ud0c0 \ubb34\ubcfc\ub137\uacfc \ubb34\uc2e4\uc810\uc758 \ud638\ud22c\ub97c \ubcf4\uc600\ub2e4. \uac1c\ub9c9 \uc774\ud6c4\uc5d0\ub294 \uc7a5\ub798 \uc88c\uc644 \uc5d0\uc774\uc2a4\ub85c\uc11c\uc758 \uae30\ub300\ub97c \ud55c\ubab8\uc5d0 \ubc1b\uc544\uc11c \uc8fc\ub85c \uc120\ubc1c\ub85c\uc11c \uae30\uc6a9\ub410\ub2e4. \ub2f9\uc2dc\uc5d0\ub294 \uc544\uc9c1 \ub9c9\uc5f0\ud558\uac8c \ub358\uc9c0\ub294 \ub4ef\ud55c \uc810\uc774 \uc788\uc5c8\uc9c0\ub9cc \uc120\ubc1c \ub85c\ud14c\uc774\uc158\uc5d0 \uc815\ucc29\ud558\uc5ec 6\ucc28\ub840\uc758 \uc644\ud22c \ub4f1 \uaddc\uc815 \ud22c\uad6c \uc774\ub2dd\uc5d0 \ub3c4\ub2ec\ud574\uc11c 10\uc2b9(6\ud328)\uc744 \uae30\ub85d\ud588\ub2e4."} {"question": "\uc77c\ubcf8\uc778\uc758 2009\ub144 \ub0a8\uc131\uc758 \ud3c9\uade0 \uc218\uba85\uc740?", "paragraph": "\uc77c\ubcf8\uc778\uc758 \ud3c9\uade0 \uc218\uba85\uc740 2009\ub144 \uae30\uc900 \ub0a8\uc131\uc774 79.29\uc138\uc774\uba70 \uc5ec\uc131\uc740 \uc138\uacc4\uc5d0\uc11c \uac00\uc7a5 \ub192\uc740 \uc218\uce58\uc778 86.05\uc138\ub97c \uae30\ub85d\ud558\uc600\ub2e4. \ud2b9\ud788 \uc5ec\uc131\uc758 \ud3c9\uade0 \uc218\uba85\uc740 1985\ub144 \uc774\ud6c4 \uacc4\uc18d \uc138\uacc4 1\uc704\ub97c \uc720\uc9c0\ud558\uace0 \uc788\ub2e4. \uc77c\ubcf8\uc778\uc758 \uc8fc\ub41c \uc0ac\uc778\uc740 1996\ub144 \uc870\uc0ac\uc5d0 \uc758\ud558\uba74 \uc554\uc774 \uac00\uc7a5 \ub9ce\uace0, \ub2e4\uc74c\uc73c\ub85c\ub294 \uc2ec\uc7a5\ubcd1, \ub1cc\uc878\uc911 \ub4f1\uc758 \uc131\uc778\ubcd1\uacfc \uc790\uc0b4, \uad50\ud1b5\uc0ac\uace0 \ub4f1\uc774 \uadf8 \ub4a4\ub97c \uc774\uc5c8\ub2e4. \ucd9c\uc0dd\ub960\uc774 \uacc4\uc18d \uc904\uc5b4\ub4e4\uace0 \uc788\uc9c0\ub9cc \uc81c2\ucc28 \uc138\uacc4 \ub300\uc804 \uc774\ud6c4 \uc758\ud559\uc774 \uae09\uc18d\ub3c4\ub85c \ubc1c\uc804\ud558\uc5ec \uc0ac\ub9dd\ub960 \ub610\ud55c \ud604\uc800\ud788 \uc904\uc5b4\ub4e4\uc5b4 \ub178\uc778 \uacc4\uce35\uc774 \uc99d\uac00\ud574 \uc774\ubbf8 \ucd08\uace0\ub839 \uc0ac\ud68c\uc5d0 \uc9c4\uc785\ud55c \uc0c1\ud0dc\uc774\ub2e4. \ubbf8\uad6d \uc9c8\ubcd1\ud1b5\uc81c\uc608\ubc29\uc13c\ud130\ub294 \uc77c\ubcf8\uc778\uc758 \uae30\ub300 \uc218\uba85\uc740 2009\ub144 \uae30\uc900 \uc2f1\uac00\ud3ec\ub974\uc640 \ub354\ubd88\uc5b4 \uc57d 82\uc138 \uc815\ub3c4\ub77c\uace0 \ubc1c\ud45c\ud558\uc600\ub2e4. \ud558\uc9c0\ub9cc \uc790\uc0b4\ub960\uc774 OECD \uad6d\uac00 \uc911 \ub300\ud55c\ubbfc\uad6d\uc5d0 \uc774\uc5b4 2\uc704\ub85c \ub192\uc544 \uc2ec\uac01\ud55c \uc0ac\ud68c \ubb38\uc81c\ub85c \ub300\ub450\ub418\uace0 \uc788\ub2e4.", "answer": "79.29\uc138", "sentence": "\uc77c\ubcf8\uc778\uc758 \ud3c9\uade0 \uc218\uba85\uc740 2009\ub144 \uae30\uc900 \ub0a8\uc131\uc774 79.29\uc138 \uc774\uba70 \uc5ec\uc131\uc740 \uc138\uacc4\uc5d0\uc11c \uac00\uc7a5 \ub192\uc740 \uc218\uce58\uc778 86.05\uc138\ub97c \uae30\ub85d\ud558\uc600\ub2e4.", "paragraph_sentence": " \uc77c\ubcf8\uc778\uc758 \ud3c9\uade0 \uc218\uba85\uc740 2009\ub144 \uae30\uc900 \ub0a8\uc131\uc774 79.29\uc138 \uc774\uba70 \uc5ec\uc131\uc740 \uc138\uacc4\uc5d0\uc11c \uac00\uc7a5 \ub192\uc740 \uc218\uce58\uc778 86.05\uc138\ub97c \uae30\ub85d\ud558\uc600\ub2e4. \ud2b9\ud788 \uc5ec\uc131\uc758 \ud3c9\uade0 \uc218\uba85\uc740 1985\ub144 \uc774\ud6c4 \uacc4\uc18d \uc138\uacc4 1\uc704\ub97c \uc720\uc9c0\ud558\uace0 \uc788\ub2e4. \uc77c\ubcf8\uc778\uc758 \uc8fc\ub41c \uc0ac\uc778\uc740 1996\ub144 \uc870\uc0ac\uc5d0 \uc758\ud558\uba74 \uc554\uc774 \uac00\uc7a5 \ub9ce\uace0, \ub2e4\uc74c\uc73c\ub85c\ub294 \uc2ec\uc7a5\ubcd1, \ub1cc\uc878\uc911 \ub4f1\uc758 \uc131\uc778\ubcd1\uacfc \uc790\uc0b4, \uad50\ud1b5\uc0ac\uace0 \ub4f1\uc774 \uadf8 \ub4a4\ub97c \uc774\uc5c8\ub2e4. \ucd9c\uc0dd\ub960\uc774 \uacc4\uc18d \uc904\uc5b4\ub4e4\uace0 \uc788\uc9c0\ub9cc \uc81c2\ucc28 \uc138\uacc4 \ub300\uc804 \uc774\ud6c4 \uc758\ud559\uc774 \uae09\uc18d\ub3c4\ub85c \ubc1c\uc804\ud558\uc5ec \uc0ac\ub9dd\ub960 \ub610\ud55c \ud604\uc800\ud788 \uc904\uc5b4\ub4e4\uc5b4 \ub178\uc778 \uacc4\uce35\uc774 \uc99d\uac00\ud574 \uc774\ubbf8 \ucd08\uace0\ub839 \uc0ac\ud68c\uc5d0 \uc9c4\uc785\ud55c \uc0c1\ud0dc\uc774\ub2e4. \ubbf8\uad6d \uc9c8\ubcd1\ud1b5\uc81c\uc608\ubc29\uc13c\ud130\ub294 \uc77c\ubcf8\uc778\uc758 \uae30\ub300 \uc218\uba85\uc740 2009\ub144 \uae30\uc900 \uc2f1\uac00\ud3ec\ub974\uc640 \ub354\ubd88\uc5b4 \uc57d 82\uc138 \uc815\ub3c4\ub77c\uace0 \ubc1c\ud45c\ud558\uc600\ub2e4. \ud558\uc9c0\ub9cc \uc790\uc0b4\ub960\uc774 OECD \uad6d\uac00 \uc911 \ub300\ud55c\ubbfc\uad6d\uc5d0 \uc774\uc5b4 2\uc704\ub85c \ub192\uc544 \uc2ec\uac01\ud55c \uc0ac\ud68c \ubb38\uc81c\ub85c \ub300\ub450\ub418\uace0 \uc788\ub2e4.", "paragraph_answer": "\uc77c\ubcf8\uc778\uc758 \ud3c9\uade0 \uc218\uba85\uc740 2009\ub144 \uae30\uc900 \ub0a8\uc131\uc774 79.29\uc138 \uc774\uba70 \uc5ec\uc131\uc740 \uc138\uacc4\uc5d0\uc11c \uac00\uc7a5 \ub192\uc740 \uc218\uce58\uc778 86.05\uc138\ub97c \uae30\ub85d\ud558\uc600\ub2e4. \ud2b9\ud788 \uc5ec\uc131\uc758 \ud3c9\uade0 \uc218\uba85\uc740 1985\ub144 \uc774\ud6c4 \uacc4\uc18d \uc138\uacc4 1\uc704\ub97c \uc720\uc9c0\ud558\uace0 \uc788\ub2e4. \uc77c\ubcf8\uc778\uc758 \uc8fc\ub41c \uc0ac\uc778\uc740 1996\ub144 \uc870\uc0ac\uc5d0 \uc758\ud558\uba74 \uc554\uc774 \uac00\uc7a5 \ub9ce\uace0, \ub2e4\uc74c\uc73c\ub85c\ub294 \uc2ec\uc7a5\ubcd1, \ub1cc\uc878\uc911 \ub4f1\uc758 \uc131\uc778\ubcd1\uacfc \uc790\uc0b4, \uad50\ud1b5\uc0ac\uace0 \ub4f1\uc774 \uadf8 \ub4a4\ub97c \uc774\uc5c8\ub2e4. \ucd9c\uc0dd\ub960\uc774 \uacc4\uc18d \uc904\uc5b4\ub4e4\uace0 \uc788\uc9c0\ub9cc \uc81c2\ucc28 \uc138\uacc4 \ub300\uc804 \uc774\ud6c4 \uc758\ud559\uc774 \uae09\uc18d\ub3c4\ub85c \ubc1c\uc804\ud558\uc5ec \uc0ac\ub9dd\ub960 \ub610\ud55c \ud604\uc800\ud788 \uc904\uc5b4\ub4e4\uc5b4 \ub178\uc778 \uacc4\uce35\uc774 \uc99d\uac00\ud574 \uc774\ubbf8 \ucd08\uace0\ub839 \uc0ac\ud68c\uc5d0 \uc9c4\uc785\ud55c \uc0c1\ud0dc\uc774\ub2e4. \ubbf8\uad6d \uc9c8\ubcd1\ud1b5\uc81c\uc608\ubc29\uc13c\ud130\ub294 \uc77c\ubcf8\uc778\uc758 \uae30\ub300 \uc218\uba85\uc740 2009\ub144 \uae30\uc900 \uc2f1\uac00\ud3ec\ub974\uc640 \ub354\ubd88\uc5b4 \uc57d 82\uc138 \uc815\ub3c4\ub77c\uace0 \ubc1c\ud45c\ud558\uc600\ub2e4. \ud558\uc9c0\ub9cc \uc790\uc0b4\ub960\uc774 OECD \uad6d\uac00 \uc911 \ub300\ud55c\ubbfc\uad6d\uc5d0 \uc774\uc5b4 2\uc704\ub85c \ub192\uc544 \uc2ec\uac01\ud55c \uc0ac\ud68c \ubb38\uc81c\ub85c \ub300\ub450\ub418\uace0 \uc788\ub2e4.", "sentence_answer": "\uc77c\ubcf8\uc778\uc758 \ud3c9\uade0 \uc218\uba85\uc740 2009\ub144 \uae30\uc900 \ub0a8\uc131\uc774 79.29\uc138 \uc774\uba70 \uc5ec\uc131\uc740 \uc138\uacc4\uc5d0\uc11c \uac00\uc7a5 \ub192\uc740 \uc218\uce58\uc778 86.05\uc138\ub97c \uae30\ub85d\ud558\uc600\ub2e4.", "paragraph_id": "6173085-27-0", "paragraph_question": "question: \uc77c\ubcf8\uc778\uc758 2009\ub144 \ub0a8\uc131\uc758 \ud3c9\uade0 \uc218\uba85\uc740?, context: \uc77c\ubcf8\uc778\uc758 \ud3c9\uade0 \uc218\uba85\uc740 2009\ub144 \uae30\uc900 \ub0a8\uc131\uc774 79.29\uc138\uc774\uba70 \uc5ec\uc131\uc740 \uc138\uacc4\uc5d0\uc11c \uac00\uc7a5 \ub192\uc740 \uc218\uce58\uc778 86.05\uc138\ub97c \uae30\ub85d\ud558\uc600\ub2e4. \ud2b9\ud788 \uc5ec\uc131\uc758 \ud3c9\uade0 \uc218\uba85\uc740 1985\ub144 \uc774\ud6c4 \uacc4\uc18d \uc138\uacc4 1\uc704\ub97c \uc720\uc9c0\ud558\uace0 \uc788\ub2e4. \uc77c\ubcf8\uc778\uc758 \uc8fc\ub41c \uc0ac\uc778\uc740 1996\ub144 \uc870\uc0ac\uc5d0 \uc758\ud558\uba74 \uc554\uc774 \uac00\uc7a5 \ub9ce\uace0, \ub2e4\uc74c\uc73c\ub85c\ub294 \uc2ec\uc7a5\ubcd1, \ub1cc\uc878\uc911 \ub4f1\uc758 \uc131\uc778\ubcd1\uacfc \uc790\uc0b4, \uad50\ud1b5\uc0ac\uace0 \ub4f1\uc774 \uadf8 \ub4a4\ub97c \uc774\uc5c8\ub2e4. \ucd9c\uc0dd\ub960\uc774 \uacc4\uc18d \uc904\uc5b4\ub4e4\uace0 \uc788\uc9c0\ub9cc \uc81c2\ucc28 \uc138\uacc4 \ub300\uc804 \uc774\ud6c4 \uc758\ud559\uc774 \uae09\uc18d\ub3c4\ub85c \ubc1c\uc804\ud558\uc5ec \uc0ac\ub9dd\ub960 \ub610\ud55c \ud604\uc800\ud788 \uc904\uc5b4\ub4e4\uc5b4 \ub178\uc778 \uacc4\uce35\uc774 \uc99d\uac00\ud574 \uc774\ubbf8 \ucd08\uace0\ub839 \uc0ac\ud68c\uc5d0 \uc9c4\uc785\ud55c \uc0c1\ud0dc\uc774\ub2e4. \ubbf8\uad6d \uc9c8\ubcd1\ud1b5\uc81c\uc608\ubc29\uc13c\ud130\ub294 \uc77c\ubcf8\uc778\uc758 \uae30\ub300 \uc218\uba85\uc740 2009\ub144 \uae30\uc900 \uc2f1\uac00\ud3ec\ub974\uc640 \ub354\ubd88\uc5b4 \uc57d 82\uc138 \uc815\ub3c4\ub77c\uace0 \ubc1c\ud45c\ud558\uc600\ub2e4. \ud558\uc9c0\ub9cc \uc790\uc0b4\ub960\uc774 OECD \uad6d\uac00 \uc911 \ub300\ud55c\ubbfc\uad6d\uc5d0 \uc774\uc5b4 2\uc704\ub85c \ub192\uc544 \uc2ec\uac01\ud55c \uc0ac\ud68c \ubb38\uc81c\ub85c \ub300\ub450\ub418\uace0 \uc788\ub2e4."} {"question": "2007\ub144 \uc774\ub3d9\uac74\uacfc \ud568\uaed8 \ucd9c\uc5f0\ud55c 4\ubd80\uc791 \ubba4\uc9c1 \ub4dc\ub77c\ub9c8\uc758 \uc81c\ubaa9\uc740?", "paragraph": "\uc774\ud6a8\ub9ac\ub294 2\uc9d1 \ud65c\ub3d9 \uc911\uc5d0 4\uc6d4\ubd80\ud130 KBS \u300a\ud574\ud53c\ud22c\uac8c\ub354\u300b MC\ub97c 3\ub144\ub9cc\uc5d0 \ub2e4\uc2dc \ub9e1\uc544 \ucd9c\uc5f0\ud588\ub2e4. \ud55c\ud3b8 2006\ub144 11\uc6d4, \uc774\ud6a8\ub9ac\ub294 DSP \uc5d4\ud130\ud14c\uc778\uba3c\ud2b8\ub97c \ub5a0\ub098 \uacc4\uc57d\uae08 \uc57d 25\uc5b5\uc5d0 \uc5e0\ub137\ubbf8\ub514\uc5b4\uc640 3\ub144\uac04 \uacc4\uc57d\ud588\ub2e4. \uadf8 \ud6c4 2007\ub144 2\uc6d4, \uc774\ub3d9\uac74\uacfc \ud568\uaed8 \uc790\uc2e0\uc758 \uc18c\uc18d\uc0ac \uc5e0\ub137\ubbf8\ub514\uc5b4\uc5d0\uc11c \uc81c\uc791\ud55c \u300a\uc0ac\ub791\ud55c\ub2e4\uba74 \uc774\ub4e4\ucc98\ub7fc\u300b \uc774\ub77c\ub294 \uc81c\ubaa9\uc758 4\ubd80\uc791 \ubba4\uc9c1 \ub4dc\ub77c\ub9c8\uc5d0 \ucd9c\uc5f0\ud558\uc600\ub2e4. \uadf8\ub7ec\ub098 \uc774\uc804 \ub4dc\ub77c\ub9c8 \ucd9c\uc5f0\uc791 \u300a\uc138\uc78e\ud074\ub85c\ubc84\u300b \uc2e4\ud328\ub85c \uc5b8\ub860\uc740 \ub610 \ub2e4\uc2dc \uc2e4\ud328\ud560 \uac83\uc774\ub77c\ub294 \ubd80\uc815\uc801\uc778 \uae30\uc0ac\ub97c \ub0b4\ubcf4\ub0c8\uc73c\uba70, \ucd2c\uc601 \uae30\uac04 \uc911 \uc774\ud6a8\ub9ac\ub294 \ubd80\uc0c1\uc744 \ub2f9\ud558\uae30\ub3c4 \ud558\uc600\ub2e4. SBS\uc5d0\uc11c \ubc29\uc601\ub41c \uc774 \ub4dc\ub77c\ub9c8\ub294 \uacfc\ub3c4\ud55c PPL\uacfc \uc9c4\ubd80\ud55c \uc2a4\ud1a0\ub9ac\ub85c \ubd80\uc815\uc801\uc778 \ud3c9\uac00\ub97c \ubc1b\uc73c\uba70 \ud06c\uac8c \ud765\ud589\ud558\uc9c0\ub294 \ubabb\ud558\uc600\ub2e4. \ub610\ud55c, \uc77c\ubcf8 \ud6c4\uc9c0TV\uc5d0\uc11c\ub3c4 \ubc29\uc601\ub418\uc5c8\ub2e4. 2\uc6d4 25\uc77c \ub4dc\ub77c\ub9c8 \ubc29\uc601 \uc2dc\uae30\uc5d0 \ub9de\ucdb0 \uc2f1\uae00 \"Toc Toc Toc\" \uc744 \ubc1c\ub9e4\ud574 8\uac1c\uc6d4\ub9cc\uc5d0 SBS \u300a\uc778\uae30\uac00\uc694\u300b\ub97c \ud1b5\ud574 \uac00\uc218\ub85c\uc11c \ucef4\ubc31\ud588\uace0, \uae30\uc874\uc758 \uc774\ubbf8\uc9c0\ub97c \ubc97\uae30 \uc704\ud574 \"\uc794\uc18c\ub9ac\" \ub77c\ub294 \ubc1c\ub77c\ub4dc \uace1\ub3c4 \uc218\ub85d\ub418\uc5b4 \uc788\ub2e4.", "answer": "\uc0ac\ub791\ud55c\ub2e4\uba74 \uc774\ub4e4\ucc98\ub7fc", "sentence": "\uadf8 \ud6c4 2007\ub144 2\uc6d4, \uc774\ub3d9\uac74\uacfc \ud568\uaed8 \uc790\uc2e0\uc758 \uc18c\uc18d\uc0ac \uc5e0\ub137\ubbf8\ub514\uc5b4\uc5d0\uc11c \uc81c\uc791\ud55c \u300a \uc0ac\ub791\ud55c\ub2e4\uba74 \uc774\ub4e4\ucc98\ub7fc \u300b \uc774\ub77c\ub294 \uc81c\ubaa9\uc758 4\ubd80\uc791 \ubba4\uc9c1 \ub4dc\ub77c\ub9c8\uc5d0 \ucd9c\uc5f0\ud558\uc600\ub2e4.", "paragraph_sentence": "\uc774\ud6a8\ub9ac\ub294 2\uc9d1 \ud65c\ub3d9 \uc911\uc5d0 4\uc6d4\ubd80\ud130 KBS \u300a\ud574\ud53c\ud22c\uac8c\ub354\u300b MC\ub97c 3\ub144\ub9cc\uc5d0 \ub2e4\uc2dc \ub9e1\uc544 \ucd9c\uc5f0\ud588\ub2e4. \ud55c\ud3b8 2006\ub144 11\uc6d4, \uc774\ud6a8\ub9ac\ub294 DSP \uc5d4\ud130\ud14c\uc778\uba3c\ud2b8\ub97c \ub5a0\ub098 \uacc4\uc57d\uae08 \uc57d 25\uc5b5\uc5d0 \uc5e0\ub137\ubbf8\ub514\uc5b4\uc640 3\ub144\uac04 \uacc4\uc57d\ud588\ub2e4. \uadf8 \ud6c4 2007\ub144 2\uc6d4, \uc774\ub3d9\uac74\uacfc \ud568\uaed8 \uc790\uc2e0\uc758 \uc18c\uc18d\uc0ac \uc5e0\ub137\ubbf8\ub514\uc5b4\uc5d0\uc11c \uc81c\uc791\ud55c \u300a \uc0ac\ub791\ud55c\ub2e4\uba74 \uc774\ub4e4\ucc98\ub7fc \u300b \uc774\ub77c\ub294 \uc81c\ubaa9\uc758 4\ubd80\uc791 \ubba4\uc9c1 \ub4dc\ub77c\ub9c8\uc5d0 \ucd9c\uc5f0\ud558\uc600\ub2e4. \uadf8\ub7ec\ub098 \uc774\uc804 \ub4dc\ub77c\ub9c8 \ucd9c\uc5f0\uc791 \u300a\uc138\uc78e\ud074\ub85c\ubc84\u300b \uc2e4\ud328\ub85c \uc5b8\ub860\uc740 \ub610 \ub2e4\uc2dc \uc2e4\ud328\ud560 \uac83\uc774\ub77c\ub294 \ubd80\uc815\uc801\uc778 \uae30\uc0ac\ub97c \ub0b4\ubcf4\ub0c8\uc73c\uba70, \ucd2c\uc601 \uae30\uac04 \uc911 \uc774\ud6a8\ub9ac\ub294 \ubd80\uc0c1\uc744 \ub2f9\ud558\uae30\ub3c4 \ud558\uc600\ub2e4. SBS\uc5d0\uc11c \ubc29\uc601\ub41c \uc774 \ub4dc\ub77c\ub9c8\ub294 \uacfc\ub3c4\ud55c PPL\uacfc \uc9c4\ubd80\ud55c \uc2a4\ud1a0\ub9ac\ub85c \ubd80\uc815\uc801\uc778 \ud3c9\uac00\ub97c \ubc1b\uc73c\uba70 \ud06c\uac8c \ud765\ud589\ud558\uc9c0\ub294 \ubabb\ud558\uc600\ub2e4. \ub610\ud55c, \uc77c\ubcf8 \ud6c4\uc9c0TV\uc5d0\uc11c\ub3c4 \ubc29\uc601\ub418\uc5c8\ub2e4. 2\uc6d4 25\uc77c \ub4dc\ub77c\ub9c8 \ubc29\uc601 \uc2dc\uae30\uc5d0 \ub9de\ucdb0 \uc2f1\uae00 \"Toc Toc Toc\" \uc744 \ubc1c\ub9e4\ud574 8\uac1c\uc6d4\ub9cc\uc5d0 SBS \u300a\uc778\uae30\uac00\uc694\u300b\ub97c \ud1b5\ud574 \uac00\uc218\ub85c\uc11c \ucef4\ubc31\ud588\uace0, \uae30\uc874\uc758 \uc774\ubbf8\uc9c0\ub97c \ubc97\uae30 \uc704\ud574 \"\uc794\uc18c\ub9ac\" \ub77c\ub294 \ubc1c\ub77c\ub4dc \uace1\ub3c4 \uc218\ub85d\ub418\uc5b4 \uc788\ub2e4.", "paragraph_answer": "\uc774\ud6a8\ub9ac\ub294 2\uc9d1 \ud65c\ub3d9 \uc911\uc5d0 4\uc6d4\ubd80\ud130 KBS \u300a\ud574\ud53c\ud22c\uac8c\ub354\u300b MC\ub97c 3\ub144\ub9cc\uc5d0 \ub2e4\uc2dc \ub9e1\uc544 \ucd9c\uc5f0\ud588\ub2e4. \ud55c\ud3b8 2006\ub144 11\uc6d4, \uc774\ud6a8\ub9ac\ub294 DSP \uc5d4\ud130\ud14c\uc778\uba3c\ud2b8\ub97c \ub5a0\ub098 \uacc4\uc57d\uae08 \uc57d 25\uc5b5\uc5d0 \uc5e0\ub137\ubbf8\ub514\uc5b4\uc640 3\ub144\uac04 \uacc4\uc57d\ud588\ub2e4. \uadf8 \ud6c4 2007\ub144 2\uc6d4, \uc774\ub3d9\uac74\uacfc \ud568\uaed8 \uc790\uc2e0\uc758 \uc18c\uc18d\uc0ac \uc5e0\ub137\ubbf8\ub514\uc5b4\uc5d0\uc11c \uc81c\uc791\ud55c \u300a \uc0ac\ub791\ud55c\ub2e4\uba74 \uc774\ub4e4\ucc98\ub7fc \u300b \uc774\ub77c\ub294 \uc81c\ubaa9\uc758 4\ubd80\uc791 \ubba4\uc9c1 \ub4dc\ub77c\ub9c8\uc5d0 \ucd9c\uc5f0\ud558\uc600\ub2e4. \uadf8\ub7ec\ub098 \uc774\uc804 \ub4dc\ub77c\ub9c8 \ucd9c\uc5f0\uc791 \u300a\uc138\uc78e\ud074\ub85c\ubc84\u300b \uc2e4\ud328\ub85c \uc5b8\ub860\uc740 \ub610 \ub2e4\uc2dc \uc2e4\ud328\ud560 \uac83\uc774\ub77c\ub294 \ubd80\uc815\uc801\uc778 \uae30\uc0ac\ub97c \ub0b4\ubcf4\ub0c8\uc73c\uba70, \ucd2c\uc601 \uae30\uac04 \uc911 \uc774\ud6a8\ub9ac\ub294 \ubd80\uc0c1\uc744 \ub2f9\ud558\uae30\ub3c4 \ud558\uc600\ub2e4. SBS\uc5d0\uc11c \ubc29\uc601\ub41c \uc774 \ub4dc\ub77c\ub9c8\ub294 \uacfc\ub3c4\ud55c PPL\uacfc \uc9c4\ubd80\ud55c \uc2a4\ud1a0\ub9ac\ub85c \ubd80\uc815\uc801\uc778 \ud3c9\uac00\ub97c \ubc1b\uc73c\uba70 \ud06c\uac8c \ud765\ud589\ud558\uc9c0\ub294 \ubabb\ud558\uc600\ub2e4. \ub610\ud55c, \uc77c\ubcf8 \ud6c4\uc9c0TV\uc5d0\uc11c\ub3c4 \ubc29\uc601\ub418\uc5c8\ub2e4. 2\uc6d4 25\uc77c \ub4dc\ub77c\ub9c8 \ubc29\uc601 \uc2dc\uae30\uc5d0 \ub9de\ucdb0 \uc2f1\uae00 \"Toc Toc Toc\" \uc744 \ubc1c\ub9e4\ud574 8\uac1c\uc6d4\ub9cc\uc5d0 SBS \u300a\uc778\uae30\uac00\uc694\u300b\ub97c \ud1b5\ud574 \uac00\uc218\ub85c\uc11c \ucef4\ubc31\ud588\uace0, \uae30\uc874\uc758 \uc774\ubbf8\uc9c0\ub97c \ubc97\uae30 \uc704\ud574 \"\uc794\uc18c\ub9ac\" \ub77c\ub294 \ubc1c\ub77c\ub4dc \uace1\ub3c4 \uc218\ub85d\ub418\uc5b4 \uc788\ub2e4.", "sentence_answer": "\uadf8 \ud6c4 2007\ub144 2\uc6d4, \uc774\ub3d9\uac74\uacfc \ud568\uaed8 \uc790\uc2e0\uc758 \uc18c\uc18d\uc0ac \uc5e0\ub137\ubbf8\ub514\uc5b4\uc5d0\uc11c \uc81c\uc791\ud55c \u300a \uc0ac\ub791\ud55c\ub2e4\uba74 \uc774\ub4e4\ucc98\ub7fc \u300b \uc774\ub77c\ub294 \uc81c\ubaa9\uc758 4\ubd80\uc791 \ubba4\uc9c1 \ub4dc\ub77c\ub9c8\uc5d0 \ucd9c\uc5f0\ud558\uc600\ub2e4.", "paragraph_id": "6547976-4-1", "paragraph_question": "question: 2007\ub144 \uc774\ub3d9\uac74\uacfc \ud568\uaed8 \ucd9c\uc5f0\ud55c 4\ubd80\uc791 \ubba4\uc9c1 \ub4dc\ub77c\ub9c8\uc758 \uc81c\ubaa9\uc740?, context: \uc774\ud6a8\ub9ac\ub294 2\uc9d1 \ud65c\ub3d9 \uc911\uc5d0 4\uc6d4\ubd80\ud130 KBS \u300a\ud574\ud53c\ud22c\uac8c\ub354\u300b MC\ub97c 3\ub144\ub9cc\uc5d0 \ub2e4\uc2dc \ub9e1\uc544 \ucd9c\uc5f0\ud588\ub2e4. \ud55c\ud3b8 2006\ub144 11\uc6d4, \uc774\ud6a8\ub9ac\ub294 DSP \uc5d4\ud130\ud14c\uc778\uba3c\ud2b8\ub97c \ub5a0\ub098 \uacc4\uc57d\uae08 \uc57d 25\uc5b5\uc5d0 \uc5e0\ub137\ubbf8\ub514\uc5b4\uc640 3\ub144\uac04 \uacc4\uc57d\ud588\ub2e4. \uadf8 \ud6c4 2007\ub144 2\uc6d4, \uc774\ub3d9\uac74\uacfc \ud568\uaed8 \uc790\uc2e0\uc758 \uc18c\uc18d\uc0ac \uc5e0\ub137\ubbf8\ub514\uc5b4\uc5d0\uc11c \uc81c\uc791\ud55c \u300a\uc0ac\ub791\ud55c\ub2e4\uba74 \uc774\ub4e4\ucc98\ub7fc\u300b \uc774\ub77c\ub294 \uc81c\ubaa9\uc758 4\ubd80\uc791 \ubba4\uc9c1 \ub4dc\ub77c\ub9c8\uc5d0 \ucd9c\uc5f0\ud558\uc600\ub2e4. \uadf8\ub7ec\ub098 \uc774\uc804 \ub4dc\ub77c\ub9c8 \ucd9c\uc5f0\uc791 \u300a\uc138\uc78e\ud074\ub85c\ubc84\u300b \uc2e4\ud328\ub85c \uc5b8\ub860\uc740 \ub610 \ub2e4\uc2dc \uc2e4\ud328\ud560 \uac83\uc774\ub77c\ub294 \ubd80\uc815\uc801\uc778 \uae30\uc0ac\ub97c \ub0b4\ubcf4\ub0c8\uc73c\uba70, \ucd2c\uc601 \uae30\uac04 \uc911 \uc774\ud6a8\ub9ac\ub294 \ubd80\uc0c1\uc744 \ub2f9\ud558\uae30\ub3c4 \ud558\uc600\ub2e4. SBS\uc5d0\uc11c \ubc29\uc601\ub41c \uc774 \ub4dc\ub77c\ub9c8\ub294 \uacfc\ub3c4\ud55c PPL\uacfc \uc9c4\ubd80\ud55c \uc2a4\ud1a0\ub9ac\ub85c \ubd80\uc815\uc801\uc778 \ud3c9\uac00\ub97c \ubc1b\uc73c\uba70 \ud06c\uac8c \ud765\ud589\ud558\uc9c0\ub294 \ubabb\ud558\uc600\ub2e4. \ub610\ud55c, \uc77c\ubcf8 \ud6c4\uc9c0TV\uc5d0\uc11c\ub3c4 \ubc29\uc601\ub418\uc5c8\ub2e4. 2\uc6d4 25\uc77c \ub4dc\ub77c\ub9c8 \ubc29\uc601 \uc2dc\uae30\uc5d0 \ub9de\ucdb0 \uc2f1\uae00 \"Toc Toc Toc\" \uc744 \ubc1c\ub9e4\ud574 8\uac1c\uc6d4\ub9cc\uc5d0 SBS \u300a\uc778\uae30\uac00\uc694\u300b\ub97c \ud1b5\ud574 \uac00\uc218\ub85c\uc11c \ucef4\ubc31\ud588\uace0, \uae30\uc874\uc758 \uc774\ubbf8\uc9c0\ub97c \ubc97\uae30 \uc704\ud574 \"\uc794\uc18c\ub9ac\" \ub77c\ub294 \ubc1c\ub77c\ub4dc \uace1\ub3c4 \uc218\ub85d\ub418\uc5b4 \uc788\ub2e4."} {"question": "\ubaa8\ube0c\uc138\uc2a4 \uce74\uac04\uce74\ub4dc\ubc14\ud2b8\uc2dc\ub294 \uc5b4\ub290 \uc9c0\uc5ed\uc758 \uc5ed\uc0ac\uac00\uc778\uac00?", "paragraph": "\uadf8 \uc9c0\uc5ed \uc544\ub974\uba54\ub2c8\uc544\uc758 \uc5ed\uc0ac\uac00\ub85c, \uc624\uc9c1 \uac70\uc758 \uc644\uc804\ud55c \uc5ed\uc0ac\uc801 \uae30\ub85d\ub9cc \ub0a8\uae34 \ubaa8\ube0c\uc138\uc2a4 \uce74\uac04\uce74\ub4dc\ubc14\ud2b8\uc2dc\ub294 \uce85\uce74\uc2a4 \uc54c\ubc14\ub2c8\uc544\uc758 \uccab \ubc88\uc9f8 \ud1b5\uce58\uc790 \uc544\ub974\ub780 \ub610\ub294 \uc544\ub974\ud55c\uc758 \uc560\uce6d\uc774\ub77c\uace0 \ub9d0\ud55c, \uc544\ub780\uc758 \uc778\uc790\ud55c \uc131\ud488\uc744 \ub73b\ud558\ub294 \ub2e8\uc5b4 \uc544\uadf8 \ud6c4(\uc544\ub974\uba54\ub2c8\uc544\uc5b4\ub85c \uc0c1\ub0e5\ud55c, \uc21c\ud55c, \ub2e4\uc815\ud55c)\uc758 \uc720\ub798\uc5b4\ub85c \uc544\uadf8\ubc18\ud06c\uc758 \uc774\ub984\uc744 \uc124\uba85\ud55c\ub2e4. \uce7c\ub780\uce74\ud280\ud06c\uc758 \ubaa8\uc138\uc2a4\uc640 \ub2e4\ub978 \uace0\ub300 \uc790\ub8cc\ub4e4\uc740 \uce85\uce74\uc2a4 \uc54c\ubc14\ub2c8\uc544\uc758 \uc804\uc124\uc801\uc778 \ucc3d\uc2dc\uc790\uc758 \uc774\ub984\uc73c\ub85c, \uc544\ub2c8\uba74, \uba87\uba87 \ubc88\uc5ed\uc11c\uc5d0\uc11c \ub178\uc544\uc758 \uc544\ub4e4 \uc57c\ud3ab(\uc790\ud3ab)\uc758 \uc544\ub4e4\uc778 \uc54c\ub780(\uc54c\ub77c\ub2c8)\uc73c\ub85c \uc54c\ub824\uc9c4 \uc774\ub780\uc758 \ubd80\uc871 \uc774\ub984 \uae4c\uc9c0\ub3c4\ub85c\uc758, \uc544\ub974\ub780 \ub610\ub294 \uc544\ub974\ud55c\uc744 \uc124\uba85\ud55c\ub2e4.. \uc544\ubca0\uc2a4\ud0c0\uc758 \ubc88\uc5ed\uac00 \uc790\uba54\uc2a4 \ub2e4\ub974\uba54\uc2a4\ud14c\ub974\ub294 \uc544\ub974\ub780\uacfc, \ud604\ub300 \uc774\ub860\ub4e4\uc5d0\uc11c\ub294 \uc774\ub780\uc758 \ub3d9\ubd80\uc5d0 \uadf8 \uc9c0\uc5ed\uc774 \uc704\uce58\ud558\ub294 \uacbd\ud5a5\uc774 \uc788\ub294\ub370\ub3c4 \ubd88\uad6c\ud558\uace0, \uadf8\uac00 \uc544\ub77d\uc138\uc2a4-\uc544\ub77c\ub77c\ud2b8 \uc9c0\uc5ed \uc5d0 \uc788\ub2e4\uace0 \uc5ec\uae30\uae30\ub3c4 \ud588\ub358, \uc544\uc774\ub974\uc57c\ub098 \ubc14\uc5d0\uace0\ub97c \ube44\uad50\ud55c\ub2e4.", "answer": "\uc544\ub974\uba54\ub2c8\uc544", "sentence": "\uadf8 \uc9c0\uc5ed \uc544\ub974\uba54\ub2c8\uc544 \uc758 \uc5ed\uc0ac\uac00\ub85c, \uc624\uc9c1 \uac70\uc758 \uc644\uc804\ud55c \uc5ed\uc0ac\uc801 \uae30\ub85d\ub9cc \ub0a8\uae34 \ubaa8\ube0c\uc138\uc2a4 \uce74\uac04\uce74\ub4dc\ubc14\ud2b8\uc2dc\ub294 \uce85\uce74\uc2a4 \uc54c\ubc14\ub2c8\uc544\uc758 \uccab \ubc88\uc9f8 \ud1b5\uce58\uc790 \uc544\ub974\ub780 \ub610\ub294 \uc544\ub974\ud55c\uc758 \uc560\uce6d\uc774\ub77c\uace0 \ub9d0\ud55c, \uc544\ub780\uc758 \uc778\uc790\ud55c \uc131\ud488\uc744 \ub73b\ud558\ub294 \ub2e8\uc5b4 \uc544\uadf8 \ud6c4(\uc544\ub974\uba54\ub2c8\uc544\uc5b4\ub85c \uc0c1\ub0e5\ud55c, \uc21c\ud55c, \ub2e4\uc815\ud55c)\uc758 \uc720\ub798\uc5b4\ub85c \uc544\uadf8\ubc18\ud06c\uc758 \uc774\ub984\uc744 \uc124\uba85\ud55c\ub2e4.", "paragraph_sentence": " \uadf8 \uc9c0\uc5ed \uc544\ub974\uba54\ub2c8\uc544 \uc758 \uc5ed\uc0ac\uac00\ub85c, \uc624\uc9c1 \uac70\uc758 \uc644\uc804\ud55c \uc5ed\uc0ac\uc801 \uae30\ub85d\ub9cc \ub0a8\uae34 \ubaa8\ube0c\uc138\uc2a4 \uce74\uac04\uce74\ub4dc\ubc14\ud2b8\uc2dc\ub294 \uce85\uce74\uc2a4 \uc54c\ubc14\ub2c8\uc544\uc758 \uccab \ubc88\uc9f8 \ud1b5\uce58\uc790 \uc544\ub974\ub780 \ub610\ub294 \uc544\ub974\ud55c\uc758 \uc560\uce6d\uc774\ub77c\uace0 \ub9d0\ud55c, \uc544\ub780\uc758 \uc778\uc790\ud55c \uc131\ud488\uc744 \ub73b\ud558\ub294 \ub2e8\uc5b4 \uc544\uadf8 \ud6c4(\uc544\ub974\uba54\ub2c8\uc544\uc5b4\ub85c \uc0c1\ub0e5\ud55c, \uc21c\ud55c, \ub2e4\uc815\ud55c)\uc758 \uc720\ub798\uc5b4\ub85c \uc544\uadf8\ubc18\ud06c\uc758 \uc774\ub984\uc744 \uc124\uba85\ud55c\ub2e4. \uce7c\ub780\uce74\ud280\ud06c\uc758 \ubaa8\uc138\uc2a4\uc640 \ub2e4\ub978 \uace0\ub300 \uc790\ub8cc\ub4e4\uc740 \uce85\uce74\uc2a4 \uc54c\ubc14\ub2c8\uc544\uc758 \uc804\uc124\uc801\uc778 \ucc3d\uc2dc\uc790\uc758 \uc774\ub984\uc73c\ub85c, \uc544\ub2c8\uba74, \uba87\uba87 \ubc88\uc5ed\uc11c\uc5d0\uc11c \ub178\uc544\uc758 \uc544\ub4e4 \uc57c\ud3ab(\uc790\ud3ab)\uc758 \uc544\ub4e4\uc778 \uc54c\ub780(\uc54c\ub77c\ub2c8)\uc73c\ub85c \uc54c\ub824\uc9c4 \uc774\ub780\uc758 \ubd80\uc871 \uc774\ub984 \uae4c\uc9c0\ub3c4\ub85c\uc758, \uc544\ub974\ub780 \ub610\ub294 \uc544\ub974\ud55c\uc744 \uc124\uba85\ud55c\ub2e4.. \uc544\ubca0\uc2a4\ud0c0\uc758 \ubc88\uc5ed\uac00 \uc790\uba54\uc2a4 \ub2e4\ub974\uba54\uc2a4\ud14c\ub974\ub294 \uc544\ub974\ub780\uacfc, \ud604\ub300 \uc774\ub860\ub4e4\uc5d0\uc11c\ub294 \uc774\ub780\uc758 \ub3d9\ubd80\uc5d0 \uadf8 \uc9c0\uc5ed\uc774 \uc704\uce58\ud558\ub294 \uacbd\ud5a5\uc774 \uc788\ub294\ub370\ub3c4 \ubd88\uad6c\ud558\uace0, \uadf8\uac00 \uc544\ub77d\uc138\uc2a4-\uc544\ub77c\ub77c\ud2b8 \uc9c0\uc5ed \uc5d0 \uc788\ub2e4\uace0 \uc5ec\uae30\uae30\ub3c4 \ud588\ub358, \uc544\uc774\ub974\uc57c\ub098 \ubc14\uc5d0\uace0\ub97c \ube44\uad50\ud55c\ub2e4.", "paragraph_answer": "\uadf8 \uc9c0\uc5ed \uc544\ub974\uba54\ub2c8\uc544 \uc758 \uc5ed\uc0ac\uac00\ub85c, \uc624\uc9c1 \uac70\uc758 \uc644\uc804\ud55c \uc5ed\uc0ac\uc801 \uae30\ub85d\ub9cc \ub0a8\uae34 \ubaa8\ube0c\uc138\uc2a4 \uce74\uac04\uce74\ub4dc\ubc14\ud2b8\uc2dc\ub294 \uce85\uce74\uc2a4 \uc54c\ubc14\ub2c8\uc544\uc758 \uccab \ubc88\uc9f8 \ud1b5\uce58\uc790 \uc544\ub974\ub780 \ub610\ub294 \uc544\ub974\ud55c\uc758 \uc560\uce6d\uc774\ub77c\uace0 \ub9d0\ud55c, \uc544\ub780\uc758 \uc778\uc790\ud55c \uc131\ud488\uc744 \ub73b\ud558\ub294 \ub2e8\uc5b4 \uc544\uadf8 \ud6c4(\uc544\ub974\uba54\ub2c8\uc544\uc5b4\ub85c \uc0c1\ub0e5\ud55c, \uc21c\ud55c, \ub2e4\uc815\ud55c)\uc758 \uc720\ub798\uc5b4\ub85c \uc544\uadf8\ubc18\ud06c\uc758 \uc774\ub984\uc744 \uc124\uba85\ud55c\ub2e4. \uce7c\ub780\uce74\ud280\ud06c\uc758 \ubaa8\uc138\uc2a4\uc640 \ub2e4\ub978 \uace0\ub300 \uc790\ub8cc\ub4e4\uc740 \uce85\uce74\uc2a4 \uc54c\ubc14\ub2c8\uc544\uc758 \uc804\uc124\uc801\uc778 \ucc3d\uc2dc\uc790\uc758 \uc774\ub984\uc73c\ub85c, \uc544\ub2c8\uba74, \uba87\uba87 \ubc88\uc5ed\uc11c\uc5d0\uc11c \ub178\uc544\uc758 \uc544\ub4e4 \uc57c\ud3ab(\uc790\ud3ab)\uc758 \uc544\ub4e4\uc778 \uc54c\ub780(\uc54c\ub77c\ub2c8)\uc73c\ub85c \uc54c\ub824\uc9c4 \uc774\ub780\uc758 \ubd80\uc871 \uc774\ub984 \uae4c\uc9c0\ub3c4\ub85c\uc758, \uc544\ub974\ub780 \ub610\ub294 \uc544\ub974\ud55c\uc744 \uc124\uba85\ud55c\ub2e4.. \uc544\ubca0\uc2a4\ud0c0\uc758 \ubc88\uc5ed\uac00 \uc790\uba54\uc2a4 \ub2e4\ub974\uba54\uc2a4\ud14c\ub974\ub294 \uc544\ub974\ub780\uacfc, \ud604\ub300 \uc774\ub860\ub4e4\uc5d0\uc11c\ub294 \uc774\ub780\uc758 \ub3d9\ubd80\uc5d0 \uadf8 \uc9c0\uc5ed\uc774 \uc704\uce58\ud558\ub294 \uacbd\ud5a5\uc774 \uc788\ub294\ub370\ub3c4 \ubd88\uad6c\ud558\uace0, \uadf8\uac00 \uc544\ub77d\uc138\uc2a4-\uc544\ub77c\ub77c\ud2b8 \uc9c0\uc5ed \uc5d0 \uc788\ub2e4\uace0 \uc5ec\uae30\uae30\ub3c4 \ud588\ub358, \uc544\uc774\ub974\uc57c\ub098 \ubc14\uc5d0\uace0\ub97c \ube44\uad50\ud55c\ub2e4.", "sentence_answer": "\uadf8 \uc9c0\uc5ed \uc544\ub974\uba54\ub2c8\uc544 \uc758 \uc5ed\uc0ac\uac00\ub85c, \uc624\uc9c1 \uac70\uc758 \uc644\uc804\ud55c \uc5ed\uc0ac\uc801 \uae30\ub85d\ub9cc \ub0a8\uae34 \ubaa8\ube0c\uc138\uc2a4 \uce74\uac04\uce74\ub4dc\ubc14\ud2b8\uc2dc\ub294 \uce85\uce74\uc2a4 \uc54c\ubc14\ub2c8\uc544\uc758 \uccab \ubc88\uc9f8 \ud1b5\uce58\uc790 \uc544\ub974\ub780 \ub610\ub294 \uc544\ub974\ud55c\uc758 \uc560\uce6d\uc774\ub77c\uace0 \ub9d0\ud55c, \uc544\ub780\uc758 \uc778\uc790\ud55c \uc131\ud488\uc744 \ub73b\ud558\ub294 \ub2e8\uc5b4 \uc544\uadf8 \ud6c4(\uc544\ub974\uba54\ub2c8\uc544\uc5b4\ub85c \uc0c1\ub0e5\ud55c, \uc21c\ud55c, \ub2e4\uc815\ud55c)\uc758 \uc720\ub798\uc5b4\ub85c \uc544\uadf8\ubc18\ud06c\uc758 \uc774\ub984\uc744 \uc124\uba85\ud55c\ub2e4.", "paragraph_id": "6208903-0-2", "paragraph_question": "question: \ubaa8\ube0c\uc138\uc2a4 \uce74\uac04\uce74\ub4dc\ubc14\ud2b8\uc2dc\ub294 \uc5b4\ub290 \uc9c0\uc5ed\uc758 \uc5ed\uc0ac\uac00\uc778\uac00?, context: \uadf8 \uc9c0\uc5ed \uc544\ub974\uba54\ub2c8\uc544\uc758 \uc5ed\uc0ac\uac00\ub85c, \uc624\uc9c1 \uac70\uc758 \uc644\uc804\ud55c \uc5ed\uc0ac\uc801 \uae30\ub85d\ub9cc \ub0a8\uae34 \ubaa8\ube0c\uc138\uc2a4 \uce74\uac04\uce74\ub4dc\ubc14\ud2b8\uc2dc\ub294 \uce85\uce74\uc2a4 \uc54c\ubc14\ub2c8\uc544\uc758 \uccab \ubc88\uc9f8 \ud1b5\uce58\uc790 \uc544\ub974\ub780 \ub610\ub294 \uc544\ub974\ud55c\uc758 \uc560\uce6d\uc774\ub77c\uace0 \ub9d0\ud55c, \uc544\ub780\uc758 \uc778\uc790\ud55c \uc131\ud488\uc744 \ub73b\ud558\ub294 \ub2e8\uc5b4 \uc544\uadf8 \ud6c4(\uc544\ub974\uba54\ub2c8\uc544\uc5b4\ub85c \uc0c1\ub0e5\ud55c, \uc21c\ud55c, \ub2e4\uc815\ud55c)\uc758 \uc720\ub798\uc5b4\ub85c \uc544\uadf8\ubc18\ud06c\uc758 \uc774\ub984\uc744 \uc124\uba85\ud55c\ub2e4. \uce7c\ub780\uce74\ud280\ud06c\uc758 \ubaa8\uc138\uc2a4\uc640 \ub2e4\ub978 \uace0\ub300 \uc790\ub8cc\ub4e4\uc740 \uce85\uce74\uc2a4 \uc54c\ubc14\ub2c8\uc544\uc758 \uc804\uc124\uc801\uc778 \ucc3d\uc2dc\uc790\uc758 \uc774\ub984\uc73c\ub85c, \uc544\ub2c8\uba74, \uba87\uba87 \ubc88\uc5ed\uc11c\uc5d0\uc11c \ub178\uc544\uc758 \uc544\ub4e4 \uc57c\ud3ab(\uc790\ud3ab)\uc758 \uc544\ub4e4\uc778 \uc54c\ub780(\uc54c\ub77c\ub2c8)\uc73c\ub85c \uc54c\ub824\uc9c4 \uc774\ub780\uc758 \ubd80\uc871 \uc774\ub984 \uae4c\uc9c0\ub3c4\ub85c\uc758, \uc544\ub974\ub780 \ub610\ub294 \uc544\ub974\ud55c\uc744 \uc124\uba85\ud55c\ub2e4.. \uc544\ubca0\uc2a4\ud0c0\uc758 \ubc88\uc5ed\uac00 \uc790\uba54\uc2a4 \ub2e4\ub974\uba54\uc2a4\ud14c\ub974\ub294 \uc544\ub974\ub780\uacfc, \ud604\ub300 \uc774\ub860\ub4e4\uc5d0\uc11c\ub294 \uc774\ub780\uc758 \ub3d9\ubd80\uc5d0 \uadf8 \uc9c0\uc5ed\uc774 \uc704\uce58\ud558\ub294 \uacbd\ud5a5\uc774 \uc788\ub294\ub370\ub3c4 \ubd88\uad6c\ud558\uace0, \uadf8\uac00 \uc544\ub77d\uc138\uc2a4-\uc544\ub77c\ub77c\ud2b8 \uc9c0\uc5ed \uc5d0 \uc788\ub2e4\uace0 \uc5ec\uae30\uae30\ub3c4 \ud588\ub358, \uc544\uc774\ub974\uc57c\ub098 \ubc14\uc5d0\uace0\ub97c \ube44\uad50\ud55c\ub2e4."} {"question": "2011\ub144 \uc5c9\uc131\ud55c \uc2dc\ub098\ub9ac\uc624\uc758 \ud639\ud3c9\uc73c\ub85c \ud765\ud589\uc5d0 \uc2e4\ud328\ud55c \uc791\ud488\uc740?", "paragraph": "\ud558\uc9c0\uc6d0\uc740 \uc655\uc131\ud558\uac8c \ud65c\uc57d\ud574 \uc628 \uc2a4\ud06c\ub9b0\uc5d0\uc11c 2010\ub144 11\uc6d4 \uccab \ubc29\uc601 \ub41c \ub4dc\ub77c\ub9c8 \u300a\uc2dc\ud06c\ub9bf \uac00\ub4e0\u300b\uc73c\ub85c 4\ub144\uc5ec\ub9cc\uc5d0 \ube0c\ub77c\uc6b4\uad00\uc5d0 \ubcf5\uadc0\ud558\uc600\ub2e4. \ub4dc\ub77c\ub9c8\ub294 \uc2a4\ud0c0 \ucf64\ube44 \uae40\uc740\uc219 \uc791\uac00\uc640 \uc2e0\uc6b0\ucca0 \uac10\ub3c5\uc758 \ucc28\uae30\uc791\uc73c\ub85c \ubc29\uc601\uc804\ubd80\ud130 \ud654\uc81c\ub97c \ubd88\ub7ec \uc77c\uc73c\ucf30\uace0, \ud558\uc9c0\uc6d0\uc740 \uae4c\uce60\ud55c \ubc31\ub9cc\uc7a5\uc790 \ubc31\ud654\uc810 \uc0ac\uc7a5 \uae40\uc8fc\uc6d0(\ud604\ube48 \ubd84)\uacfc \uc601\ud63c\uc774 \ubc14\ub00c\ub294 \ubb34\uc220\uac10\ub3c5\uc744 \uafc8\uafb8\ub294 \uc2a4\ud134\ud2b8 \uc6b0\uba3c \uae38\ub77c\uc784 \uc5ed\uc744 \ub9e1\uc544 \uc20f\ucee4\ud2b8\uc640 \uc218\ub354\ubd84\ud55c \uc637\ucc28\ub9bc\uc744 \uc120\ubcf4\uc774\uba70 \uce90\ub9ad\ud130\uc640 \ub180\ub77c\uc6b4 \uc2f1\ud06c\ub85c\uc728\uc744 \uc790\ub791\ud558\uba70 \uc640\uc774\uc5b4 \uc5f0\uae30\ubd80\ud130 \uc790\uc804\uac70 \ucd94\uaca9\uc2e0\uae4c\uc9c0 \uc9c1\uc811 \uc18c\ud654\ud558\uba70 \ucc2c\uc0ac\ub97c \ubc1b\uc558\ub2e4. \ub610\ud55c \ub4dc\ub77c\ub9c8\ub294 \ucd5c\uace0\uc2dc\uccad\ub960 35.2%\ub97c \uae30\ub85d\ud558\uba70 \uc2e0\ub4dc\ub86c\uae09 \uc778\uae30\ub97c \uc5bb\uc5c8\ub2e4. \uc774 \uc791\ud488\uc73c\ub85c SBS \uc5f0\uae30\ub300\uc0c1 \ub124\ud2f0\uc98c\uc778\uae30\uc0c1, \ubca0\uc2a4\ud2b8\ucee4\ud50c\uc0c1, 10\ub300\uc2a4\ud0c0\uc0c1, \ub4dc\ub77c\ub9c8\uc2a4\ud398\uc15c\ubd80\ubb38 \ucd5c\uc6b0\uc218\uc5f0\uae30\uc0c1\uacfc \uc81c24\ud68c \uadf8\ub9ac\uba54\uc0c1 \uc5ec\uc790\uc5ec\uc790\ucd5c\uc6b0\uc218\uc5f0\uae30\uc0c1\uc744 \uc218\uc0c1\ud588\ub2e4. 2011\ub144 8\uc6d4 \uac1c\ubd09 \ub41c \uc21c\uc218 \uc6b0\ub9ac \uae30\uc220\uc9c4\uc774 \ub9cc\ub4e4\uc5b4\ub0b8 3D \uc601\ud654\ub85c \uac1c\ubd09 \uc804\ubd80\ud130 \ub728\uac70\uc6b4 \ud654\uc81c\ub97c \ubaa8\uc740 \uad34\uc218 \uc601\ud654 \u300a7\uad11\uad6c\u300b\uc5d0 \ucd9c\uc5f0\ud588\uc9c0\ub9cc \uc601\ud654\ub294 \ubd80\uc871\ud55c \uc5c9\uc131\ud55c \uc2dc\ub098\ub9ac\uc624\uac00 \ud3c9\ub2e8\uacfc \ub300\uc911\uc73c\ub85c\ubd80\ud130 \ud639\ud3c9\uc744 \ubc1b\uc73c\uba70 \ud765\ud589\uc5d0 \ucc38\ud328\ud588\ub2e4.", "answer": "7\uad11\uad6c", "sentence": "2011\ub144 8\uc6d4 \uac1c\ubd09 \ub41c \uc21c\uc218 \uc6b0\ub9ac \uae30\uc220\uc9c4\uc774 \ub9cc\ub4e4\uc5b4\ub0b8 3D \uc601\ud654\ub85c \uac1c\ubd09 \uc804\ubd80\ud130 \ub728\uac70\uc6b4 \ud654\uc81c\ub97c \ubaa8\uc740 \uad34\uc218 \uc601\ud654 \u300a 7\uad11\uad6c \u300b\uc5d0 \ucd9c\uc5f0\ud588\uc9c0\ub9cc \uc601\ud654\ub294 \ubd80\uc871\ud55c \uc5c9\uc131\ud55c \uc2dc\ub098\ub9ac\uc624\uac00 \ud3c9\ub2e8\uacfc \ub300\uc911\uc73c\ub85c\ubd80\ud130 \ud639\ud3c9\uc744 \ubc1b\uc73c\uba70 \ud765\ud589\uc5d0 \ucc38\ud328\ud588\ub2e4.", "paragraph_sentence": "\ud558\uc9c0\uc6d0\uc740 \uc655\uc131\ud558\uac8c \ud65c\uc57d\ud574 \uc628 \uc2a4\ud06c\ub9b0\uc5d0\uc11c 2010\ub144 11\uc6d4 \uccab \ubc29\uc601 \ub41c \ub4dc\ub77c\ub9c8 \u300a\uc2dc\ud06c\ub9bf \uac00\ub4e0\u300b\uc73c\ub85c 4\ub144\uc5ec\ub9cc\uc5d0 \ube0c\ub77c\uc6b4\uad00\uc5d0 \ubcf5\uadc0\ud558\uc600\ub2e4. \ub4dc\ub77c\ub9c8\ub294 \uc2a4\ud0c0 \ucf64\ube44 \uae40\uc740\uc219 \uc791\uac00\uc640 \uc2e0\uc6b0\ucca0 \uac10\ub3c5\uc758 \ucc28\uae30\uc791\uc73c\ub85c \ubc29\uc601\uc804\ubd80\ud130 \ud654\uc81c\ub97c \ubd88\ub7ec \uc77c\uc73c\ucf30\uace0, \ud558\uc9c0\uc6d0\uc740 \uae4c\uce60\ud55c \ubc31\ub9cc\uc7a5\uc790 \ubc31\ud654\uc810 \uc0ac\uc7a5 \uae40\uc8fc\uc6d0(\ud604\ube48 \ubd84)\uacfc \uc601\ud63c\uc774 \ubc14\ub00c\ub294 \ubb34\uc220\uac10\ub3c5\uc744 \uafc8\uafb8\ub294 \uc2a4\ud134\ud2b8 \uc6b0\uba3c \uae38\ub77c\uc784 \uc5ed\uc744 \ub9e1\uc544 \uc20f\ucee4\ud2b8\uc640 \uc218\ub354\ubd84\ud55c \uc637\ucc28\ub9bc\uc744 \uc120\ubcf4\uc774\uba70 \uce90\ub9ad\ud130\uc640 \ub180\ub77c\uc6b4 \uc2f1\ud06c\ub85c\uc728\uc744 \uc790\ub791\ud558\uba70 \uc640\uc774\uc5b4 \uc5f0\uae30\ubd80\ud130 \uc790\uc804\uac70 \ucd94\uaca9\uc2e0\uae4c\uc9c0 \uc9c1\uc811 \uc18c\ud654\ud558\uba70 \ucc2c\uc0ac\ub97c \ubc1b\uc558\ub2e4. \ub610\ud55c \ub4dc\ub77c\ub9c8\ub294 \ucd5c\uace0\uc2dc\uccad\ub960 35.2%\ub97c \uae30\ub85d\ud558\uba70 \uc2e0\ub4dc\ub86c\uae09 \uc778\uae30\ub97c \uc5bb\uc5c8\ub2e4. \uc774 \uc791\ud488\uc73c\ub85c SBS \uc5f0\uae30\ub300\uc0c1 \ub124\ud2f0\uc98c\uc778\uae30\uc0c1, \ubca0\uc2a4\ud2b8\ucee4\ud50c\uc0c1, 10\ub300\uc2a4\ud0c0\uc0c1, \ub4dc\ub77c\ub9c8\uc2a4\ud398\uc15c\ubd80\ubb38 \ucd5c\uc6b0\uc218\uc5f0\uae30\uc0c1\uacfc \uc81c24\ud68c \uadf8\ub9ac\uba54\uc0c1 \uc5ec\uc790\uc5ec\uc790\ucd5c\uc6b0\uc218\uc5f0\uae30\uc0c1\uc744 \uc218\uc0c1\ud588\ub2e4. 2011\ub144 8\uc6d4 \uac1c\ubd09 \ub41c \uc21c\uc218 \uc6b0\ub9ac \uae30\uc220\uc9c4\uc774 \ub9cc\ub4e4\uc5b4\ub0b8 3D \uc601\ud654\ub85c \uac1c\ubd09 \uc804\ubd80\ud130 \ub728\uac70\uc6b4 \ud654\uc81c\ub97c \ubaa8\uc740 \uad34\uc218 \uc601\ud654 \u300a 7\uad11\uad6c \u300b\uc5d0 \ucd9c\uc5f0\ud588\uc9c0\ub9cc \uc601\ud654\ub294 \ubd80\uc871\ud55c \uc5c9\uc131\ud55c \uc2dc\ub098\ub9ac\uc624\uac00 \ud3c9\ub2e8\uacfc \ub300\uc911\uc73c\ub85c\ubd80\ud130 \ud639\ud3c9\uc744 \ubc1b\uc73c\uba70 \ud765\ud589\uc5d0 \ucc38\ud328\ud588\ub2e4. ", "paragraph_answer": "\ud558\uc9c0\uc6d0\uc740 \uc655\uc131\ud558\uac8c \ud65c\uc57d\ud574 \uc628 \uc2a4\ud06c\ub9b0\uc5d0\uc11c 2010\ub144 11\uc6d4 \uccab \ubc29\uc601 \ub41c \ub4dc\ub77c\ub9c8 \u300a\uc2dc\ud06c\ub9bf \uac00\ub4e0\u300b\uc73c\ub85c 4\ub144\uc5ec\ub9cc\uc5d0 \ube0c\ub77c\uc6b4\uad00\uc5d0 \ubcf5\uadc0\ud558\uc600\ub2e4. \ub4dc\ub77c\ub9c8\ub294 \uc2a4\ud0c0 \ucf64\ube44 \uae40\uc740\uc219 \uc791\uac00\uc640 \uc2e0\uc6b0\ucca0 \uac10\ub3c5\uc758 \ucc28\uae30\uc791\uc73c\ub85c \ubc29\uc601\uc804\ubd80\ud130 \ud654\uc81c\ub97c \ubd88\ub7ec \uc77c\uc73c\ucf30\uace0, \ud558\uc9c0\uc6d0\uc740 \uae4c\uce60\ud55c \ubc31\ub9cc\uc7a5\uc790 \ubc31\ud654\uc810 \uc0ac\uc7a5 \uae40\uc8fc\uc6d0(\ud604\ube48 \ubd84)\uacfc \uc601\ud63c\uc774 \ubc14\ub00c\ub294 \ubb34\uc220\uac10\ub3c5\uc744 \uafc8\uafb8\ub294 \uc2a4\ud134\ud2b8 \uc6b0\uba3c \uae38\ub77c\uc784 \uc5ed\uc744 \ub9e1\uc544 \uc20f\ucee4\ud2b8\uc640 \uc218\ub354\ubd84\ud55c \uc637\ucc28\ub9bc\uc744 \uc120\ubcf4\uc774\uba70 \uce90\ub9ad\ud130\uc640 \ub180\ub77c\uc6b4 \uc2f1\ud06c\ub85c\uc728\uc744 \uc790\ub791\ud558\uba70 \uc640\uc774\uc5b4 \uc5f0\uae30\ubd80\ud130 \uc790\uc804\uac70 \ucd94\uaca9\uc2e0\uae4c\uc9c0 \uc9c1\uc811 \uc18c\ud654\ud558\uba70 \ucc2c\uc0ac\ub97c \ubc1b\uc558\ub2e4. \ub610\ud55c \ub4dc\ub77c\ub9c8\ub294 \ucd5c\uace0\uc2dc\uccad\ub960 35.2%\ub97c \uae30\ub85d\ud558\uba70 \uc2e0\ub4dc\ub86c\uae09 \uc778\uae30\ub97c \uc5bb\uc5c8\ub2e4. \uc774 \uc791\ud488\uc73c\ub85c SBS \uc5f0\uae30\ub300\uc0c1 \ub124\ud2f0\uc98c\uc778\uae30\uc0c1, \ubca0\uc2a4\ud2b8\ucee4\ud50c\uc0c1, 10\ub300\uc2a4\ud0c0\uc0c1, \ub4dc\ub77c\ub9c8\uc2a4\ud398\uc15c\ubd80\ubb38 \ucd5c\uc6b0\uc218\uc5f0\uae30\uc0c1\uacfc \uc81c24\ud68c \uadf8\ub9ac\uba54\uc0c1 \uc5ec\uc790\uc5ec\uc790\ucd5c\uc6b0\uc218\uc5f0\uae30\uc0c1\uc744 \uc218\uc0c1\ud588\ub2e4. 2011\ub144 8\uc6d4 \uac1c\ubd09 \ub41c \uc21c\uc218 \uc6b0\ub9ac \uae30\uc220\uc9c4\uc774 \ub9cc\ub4e4\uc5b4\ub0b8 3D \uc601\ud654\ub85c \uac1c\ubd09 \uc804\ubd80\ud130 \ub728\uac70\uc6b4 \ud654\uc81c\ub97c \ubaa8\uc740 \uad34\uc218 \uc601\ud654 \u300a 7\uad11\uad6c \u300b\uc5d0 \ucd9c\uc5f0\ud588\uc9c0\ub9cc \uc601\ud654\ub294 \ubd80\uc871\ud55c \uc5c9\uc131\ud55c \uc2dc\ub098\ub9ac\uc624\uac00 \ud3c9\ub2e8\uacfc \ub300\uc911\uc73c\ub85c\ubd80\ud130 \ud639\ud3c9\uc744 \ubc1b\uc73c\uba70 \ud765\ud589\uc5d0 \ucc38\ud328\ud588\ub2e4.", "sentence_answer": "2011\ub144 8\uc6d4 \uac1c\ubd09 \ub41c \uc21c\uc218 \uc6b0\ub9ac \uae30\uc220\uc9c4\uc774 \ub9cc\ub4e4\uc5b4\ub0b8 3D \uc601\ud654\ub85c \uac1c\ubd09 \uc804\ubd80\ud130 \ub728\uac70\uc6b4 \ud654\uc81c\ub97c \ubaa8\uc740 \uad34\uc218 \uc601\ud654 \u300a 7\uad11\uad6c \u300b\uc5d0 \ucd9c\uc5f0\ud588\uc9c0\ub9cc \uc601\ud654\ub294 \ubd80\uc871\ud55c \uc5c9\uc131\ud55c \uc2dc\ub098\ub9ac\uc624\uac00 \ud3c9\ub2e8\uacfc \ub300\uc911\uc73c\ub85c\ubd80\ud130 \ud639\ud3c9\uc744 \ubc1b\uc73c\uba70 \ud765\ud589\uc5d0 \ucc38\ud328\ud588\ub2e4.", "paragraph_id": "5934842-4-1", "paragraph_question": "question: 2011\ub144 \uc5c9\uc131\ud55c \uc2dc\ub098\ub9ac\uc624\uc758 \ud639\ud3c9\uc73c\ub85c \ud765\ud589\uc5d0 \uc2e4\ud328\ud55c \uc791\ud488\uc740?, context: \ud558\uc9c0\uc6d0\uc740 \uc655\uc131\ud558\uac8c \ud65c\uc57d\ud574 \uc628 \uc2a4\ud06c\ub9b0\uc5d0\uc11c 2010\ub144 11\uc6d4 \uccab \ubc29\uc601 \ub41c \ub4dc\ub77c\ub9c8 \u300a\uc2dc\ud06c\ub9bf \uac00\ub4e0\u300b\uc73c\ub85c 4\ub144\uc5ec\ub9cc\uc5d0 \ube0c\ub77c\uc6b4\uad00\uc5d0 \ubcf5\uadc0\ud558\uc600\ub2e4. \ub4dc\ub77c\ub9c8\ub294 \uc2a4\ud0c0 \ucf64\ube44 \uae40\uc740\uc219 \uc791\uac00\uc640 \uc2e0\uc6b0\ucca0 \uac10\ub3c5\uc758 \ucc28\uae30\uc791\uc73c\ub85c \ubc29\uc601\uc804\ubd80\ud130 \ud654\uc81c\ub97c \ubd88\ub7ec \uc77c\uc73c\ucf30\uace0, \ud558\uc9c0\uc6d0\uc740 \uae4c\uce60\ud55c \ubc31\ub9cc\uc7a5\uc790 \ubc31\ud654\uc810 \uc0ac\uc7a5 \uae40\uc8fc\uc6d0(\ud604\ube48 \ubd84)\uacfc \uc601\ud63c\uc774 \ubc14\ub00c\ub294 \ubb34\uc220\uac10\ub3c5\uc744 \uafc8\uafb8\ub294 \uc2a4\ud134\ud2b8 \uc6b0\uba3c \uae38\ub77c\uc784 \uc5ed\uc744 \ub9e1\uc544 \uc20f\ucee4\ud2b8\uc640 \uc218\ub354\ubd84\ud55c \uc637\ucc28\ub9bc\uc744 \uc120\ubcf4\uc774\uba70 \uce90\ub9ad\ud130\uc640 \ub180\ub77c\uc6b4 \uc2f1\ud06c\ub85c\uc728\uc744 \uc790\ub791\ud558\uba70 \uc640\uc774\uc5b4 \uc5f0\uae30\ubd80\ud130 \uc790\uc804\uac70 \ucd94\uaca9\uc2e0\uae4c\uc9c0 \uc9c1\uc811 \uc18c\ud654\ud558\uba70 \ucc2c\uc0ac\ub97c \ubc1b\uc558\ub2e4. \ub610\ud55c \ub4dc\ub77c\ub9c8\ub294 \ucd5c\uace0\uc2dc\uccad\ub960 35.2%\ub97c \uae30\ub85d\ud558\uba70 \uc2e0\ub4dc\ub86c\uae09 \uc778\uae30\ub97c \uc5bb\uc5c8\ub2e4. \uc774 \uc791\ud488\uc73c\ub85c SBS \uc5f0\uae30\ub300\uc0c1 \ub124\ud2f0\uc98c\uc778\uae30\uc0c1, \ubca0\uc2a4\ud2b8\ucee4\ud50c\uc0c1, 10\ub300\uc2a4\ud0c0\uc0c1, \ub4dc\ub77c\ub9c8\uc2a4\ud398\uc15c\ubd80\ubb38 \ucd5c\uc6b0\uc218\uc5f0\uae30\uc0c1\uacfc \uc81c24\ud68c \uadf8\ub9ac\uba54\uc0c1 \uc5ec\uc790\uc5ec\uc790\ucd5c\uc6b0\uc218\uc5f0\uae30\uc0c1\uc744 \uc218\uc0c1\ud588\ub2e4. 2011\ub144 8\uc6d4 \uac1c\ubd09 \ub41c \uc21c\uc218 \uc6b0\ub9ac \uae30\uc220\uc9c4\uc774 \ub9cc\ub4e4\uc5b4\ub0b8 3D \uc601\ud654\ub85c \uac1c\ubd09 \uc804\ubd80\ud130 \ub728\uac70\uc6b4 \ud654\uc81c\ub97c \ubaa8\uc740 \uad34\uc218 \uc601\ud654 \u300a7\uad11\uad6c\u300b\uc5d0 \ucd9c\uc5f0\ud588\uc9c0\ub9cc \uc601\ud654\ub294 \ubd80\uc871\ud55c \uc5c9\uc131\ud55c \uc2dc\ub098\ub9ac\uc624\uac00 \ud3c9\ub2e8\uacfc \ub300\uc911\uc73c\ub85c\ubd80\ud130 \ud639\ud3c9\uc744 \ubc1b\uc73c\uba70 \ud765\ud589\uc5d0 \ucc38\ud328\ud588\ub2e4."} {"question": "\ud0dc\uc591\uc774\ub098 \ubaa9\uc131\uc73c\ub85c\ubd80\ud130 \uc640\uc11c \uce7c\ub9ac\uc2a4\ud1a0\uac00 \uc804\ub3c4\ud558\ub294 \uad6c\ucc98\ub7fc \ubc18\uc751\ud558\uac8c \ub9cc\ub4dc\ub294 \uac83\uc740?", "paragraph": "\uce7c\ub9ac\uc2a4\ud1a0\uc758 \uac70\uce5c \ud45c\uba74\uc740 \uc57d 80~150 km \ub450\uaed8\uc758 \ucc28\uac11\uace0, \uacbd\uc9c1\ub418\uc5b4 \uc788\uc73c\uba70, \uc5bc\uc5b4\ubd99\uc740 \uc554\uad8c \uc704\uc5d0 \uc788\ub2e4. \ub610\ud55c, \ubaa9\uc131\uacfc \uadf8 \uc8fc\ubcc0 \uc704\uc131\ub4e4\uc758 \uc790\uae30\uc7a5\uc744 \uc5f0\uad6c\ud55c \uacb0\uacfc\ub85c\uc11c, \uce7c\ub9ac\uc2a4\ud1a0\uc5d0 50~200 km \uae4a\uc774\uc758 \uc5fc\ubd84\uc774 \ud3ec\ud568\ub41c \ubc14\ub2e4\uac00 \uc9c0\uac01 \ubc11\uc5d0 \uc874\uc7ac\ud560 \uac83\uc73c\ub85c \ucd94\uce21\ub41c\ub2e4. \uce7c\ub9ac\uc2a4\ud1a0\uac00 \ubaa9\uc131\uc758 \uc9c0\uc18d\uc801\uc73c\ub85c \ubcc0\ud654\ud558\ub294 \uc790\uae30\uc7a5\uc5d0 \ub9c8\uce58 \uc644\ubcbd\ud788 \uc804\ub3c4\ud558\ub294 \uad6c\ucc98\ub7fc \ubc18\uc751\ud55c\ub2e4\ub294 \uac83\uc774 \ubc1d\ud600\uc84c\ub294\ub370, \uc774\ub294 \uc790\uae30\uc7a5\uc774 \uce7c\ub9ac\uc2a4\ud1a0\uc758 \ub0b4\ubd80\ub97c \ub6ab\uace0 \ub4e4\uc5b4\uac08 \uc218 \uc5c6\ub2e4\ub294 \uac83\uc744 \uc758\ubbf8\ud558\uba70, \uc774\uc5d0 \ub530\ub77c \uce7c\ub9ac\uc2a4\ud1a0 \ub0b4\ubd80\uc5d0 \ucd5c\uc18c\ud55c \uae4a\uc774\uac00 10 km \uc774\uc0c1\uc778, \ub9e4\uc6b0 \uc804\ub3c4\uc131\uc774 \ub192\uc740 \uce35\uc774 \uc788\ub2e4\ub294 \uc774\ub860\uc774 \uc81c\uae30\ub418\uc5c8\ub2e4. \uc774\ub7ec\ud55c \ud604\uc0c1\uc740 \ud0dc\uc591\uc774\ub098 \ubaa9\uc131\uc73c\ub85c\ubd80\ud130 \ubd88\uc5b4\uc624\ub294 \ud558\uc804\uc785\uc790 \ub54c\ubb38\uc5d0\ub3c4 \ubc1c\uc0dd\ud560 \uc218 \uc788\uc73c\ub098, \uce7c\ub9ac\uc2a4\ud1a0\uc758 \ub300\uae30\ub294 \ub9e4\uc6b0 \uc595\uae30 \ub54c\ubb38\uc5d0 \uac00\ub2a5\uc131\uc774 \ub0ae\ub2e4\uace0 \uc5ec\uaca8\uc9c4\ub2e4. \ubc14\ub2e4\uc758 \uc554\ubaa8\ub2c8\uc544\ub098 \uae30\ud0c0 \ubd80\ub3d9\uc561\ub4e4\uc758 \ud568\ub7c9\uc740 \uc804\uccb4 \uc9c8\ub7c9\uc758 5% \uc815\ub3c4\uc774\ub9ac\ub77c\uace0 \uc5ec\uaca8\uc9c4\ub2e4. \uc774 \uacbd\uc6b0\uc5d0\ub294 \ubb3c\uacfc \uc5bc\uc74c\uc758 \uce35\uc774 250~300 km \ub450\uaed8\ub97c \uac00\uc9c8 \uc218 \uc788\ub2e4. \ub9cc\uc57d \ubc14\ub2e4\uac00 \uc5c6\ub2e4\uba74, \uc5bc\uc74c\uc73c\ub85c \uc774\ub8e8\uc5b4\uc9c4 \uc554\uad8c\uc740 \uc57d 300 km \uc815\ub3c4\uc758 \ub2e4\uc18c \ub450\uaebc\uc6b8 \uac83\uc73c\ub85c \uc0dd\uac01\ub41c\ub2e4.", "answer": "\ud558\uc804\uc785\uc790", "sentence": "\uc774\ub7ec\ud55c \ud604\uc0c1\uc740 \ud0dc\uc591\uc774\ub098 \ubaa9\uc131\uc73c\ub85c\ubd80\ud130 \ubd88\uc5b4\uc624\ub294 \ud558\uc804\uc785\uc790 \ub54c\ubb38\uc5d0\ub3c4 \ubc1c\uc0dd\ud560 \uc218 \uc788\uc73c\ub098, \uce7c\ub9ac\uc2a4\ud1a0\uc758 \ub300\uae30\ub294 \ub9e4\uc6b0 \uc595\uae30 \ub54c\ubb38\uc5d0 \uac00\ub2a5\uc131\uc774 \ub0ae\ub2e4\uace0 \uc5ec\uaca8\uc9c4\ub2e4.", "paragraph_sentence": "\uce7c\ub9ac\uc2a4\ud1a0\uc758 \uac70\uce5c \ud45c\uba74\uc740 \uc57d 80~150 km \ub450\uaed8\uc758 \ucc28\uac11\uace0, \uacbd\uc9c1\ub418\uc5b4 \uc788\uc73c\uba70, \uc5bc\uc5b4\ubd99\uc740 \uc554\uad8c \uc704\uc5d0 \uc788\ub2e4. \ub610\ud55c, \ubaa9\uc131\uacfc \uadf8 \uc8fc\ubcc0 \uc704\uc131\ub4e4\uc758 \uc790\uae30\uc7a5\uc744 \uc5f0\uad6c\ud55c \uacb0\uacfc\ub85c\uc11c, \uce7c\ub9ac\uc2a4\ud1a0\uc5d0 50~200 km \uae4a\uc774\uc758 \uc5fc\ubd84\uc774 \ud3ec\ud568\ub41c \ubc14\ub2e4\uac00 \uc9c0\uac01 \ubc11\uc5d0 \uc874\uc7ac\ud560 \uac83\uc73c\ub85c \ucd94\uce21\ub41c\ub2e4. \uce7c\ub9ac\uc2a4\ud1a0\uac00 \ubaa9\uc131\uc758 \uc9c0\uc18d\uc801\uc73c\ub85c \ubcc0\ud654\ud558\ub294 \uc790\uae30\uc7a5\uc5d0 \ub9c8\uce58 \uc644\ubcbd\ud788 \uc804\ub3c4\ud558\ub294 \uad6c\ucc98\ub7fc \ubc18\uc751\ud55c\ub2e4\ub294 \uac83\uc774 \ubc1d\ud600\uc84c\ub294\ub370, \uc774\ub294 \uc790\uae30\uc7a5\uc774 \uce7c\ub9ac\uc2a4\ud1a0\uc758 \ub0b4\ubd80\ub97c \ub6ab\uace0 \ub4e4\uc5b4\uac08 \uc218 \uc5c6\ub2e4\ub294 \uac83\uc744 \uc758\ubbf8\ud558\uba70, \uc774\uc5d0 \ub530\ub77c \uce7c\ub9ac\uc2a4\ud1a0 \ub0b4\ubd80\uc5d0 \ucd5c\uc18c\ud55c \uae4a\uc774\uac00 10 km \uc774\uc0c1\uc778, \ub9e4\uc6b0 \uc804\ub3c4\uc131\uc774 \ub192\uc740 \uce35\uc774 \uc788\ub2e4\ub294 \uc774\ub860\uc774 \uc81c\uae30\ub418\uc5c8\ub2e4. \uc774\ub7ec\ud55c \ud604\uc0c1\uc740 \ud0dc\uc591\uc774\ub098 \ubaa9\uc131\uc73c\ub85c\ubd80\ud130 \ubd88\uc5b4\uc624\ub294 \ud558\uc804\uc785\uc790 \ub54c\ubb38\uc5d0\ub3c4 \ubc1c\uc0dd\ud560 \uc218 \uc788\uc73c\ub098, \uce7c\ub9ac\uc2a4\ud1a0\uc758 \ub300\uae30\ub294 \ub9e4\uc6b0 \uc595\uae30 \ub54c\ubb38\uc5d0 \uac00\ub2a5\uc131\uc774 \ub0ae\ub2e4\uace0 \uc5ec\uaca8\uc9c4\ub2e4. \ubc14\ub2e4\uc758 \uc554\ubaa8\ub2c8\uc544\ub098 \uae30\ud0c0 \ubd80\ub3d9\uc561\ub4e4\uc758 \ud568\ub7c9\uc740 \uc804\uccb4 \uc9c8\ub7c9\uc758 5% \uc815\ub3c4\uc774\ub9ac\ub77c\uace0 \uc5ec\uaca8\uc9c4\ub2e4. \uc774 \uacbd\uc6b0\uc5d0\ub294 \ubb3c\uacfc \uc5bc\uc74c\uc758 \uce35\uc774 250~300 km \ub450\uaed8\ub97c \uac00\uc9c8 \uc218 \uc788\ub2e4. \ub9cc\uc57d \ubc14\ub2e4\uac00 \uc5c6\ub2e4\uba74, \uc5bc\uc74c\uc73c\ub85c \uc774\ub8e8\uc5b4\uc9c4 \uc554\uad8c\uc740 \uc57d 300 km \uc815\ub3c4\uc758 \ub2e4\uc18c \ub450\uaebc\uc6b8 \uac83\uc73c\ub85c \uc0dd\uac01\ub41c\ub2e4.", "paragraph_answer": "\uce7c\ub9ac\uc2a4\ud1a0\uc758 \uac70\uce5c \ud45c\uba74\uc740 \uc57d 80~150 km \ub450\uaed8\uc758 \ucc28\uac11\uace0, \uacbd\uc9c1\ub418\uc5b4 \uc788\uc73c\uba70, \uc5bc\uc5b4\ubd99\uc740 \uc554\uad8c \uc704\uc5d0 \uc788\ub2e4. \ub610\ud55c, \ubaa9\uc131\uacfc \uadf8 \uc8fc\ubcc0 \uc704\uc131\ub4e4\uc758 \uc790\uae30\uc7a5\uc744 \uc5f0\uad6c\ud55c \uacb0\uacfc\ub85c\uc11c, \uce7c\ub9ac\uc2a4\ud1a0\uc5d0 50~200 km \uae4a\uc774\uc758 \uc5fc\ubd84\uc774 \ud3ec\ud568\ub41c \ubc14\ub2e4\uac00 \uc9c0\uac01 \ubc11\uc5d0 \uc874\uc7ac\ud560 \uac83\uc73c\ub85c \ucd94\uce21\ub41c\ub2e4. \uce7c\ub9ac\uc2a4\ud1a0\uac00 \ubaa9\uc131\uc758 \uc9c0\uc18d\uc801\uc73c\ub85c \ubcc0\ud654\ud558\ub294 \uc790\uae30\uc7a5\uc5d0 \ub9c8\uce58 \uc644\ubcbd\ud788 \uc804\ub3c4\ud558\ub294 \uad6c\ucc98\ub7fc \ubc18\uc751\ud55c\ub2e4\ub294 \uac83\uc774 \ubc1d\ud600\uc84c\ub294\ub370, \uc774\ub294 \uc790\uae30\uc7a5\uc774 \uce7c\ub9ac\uc2a4\ud1a0\uc758 \ub0b4\ubd80\ub97c \ub6ab\uace0 \ub4e4\uc5b4\uac08 \uc218 \uc5c6\ub2e4\ub294 \uac83\uc744 \uc758\ubbf8\ud558\uba70, \uc774\uc5d0 \ub530\ub77c \uce7c\ub9ac\uc2a4\ud1a0 \ub0b4\ubd80\uc5d0 \ucd5c\uc18c\ud55c \uae4a\uc774\uac00 10 km \uc774\uc0c1\uc778, \ub9e4\uc6b0 \uc804\ub3c4\uc131\uc774 \ub192\uc740 \uce35\uc774 \uc788\ub2e4\ub294 \uc774\ub860\uc774 \uc81c\uae30\ub418\uc5c8\ub2e4. \uc774\ub7ec\ud55c \ud604\uc0c1\uc740 \ud0dc\uc591\uc774\ub098 \ubaa9\uc131\uc73c\ub85c\ubd80\ud130 \ubd88\uc5b4\uc624\ub294 \ud558\uc804\uc785\uc790 \ub54c\ubb38\uc5d0\ub3c4 \ubc1c\uc0dd\ud560 \uc218 \uc788\uc73c\ub098, \uce7c\ub9ac\uc2a4\ud1a0\uc758 \ub300\uae30\ub294 \ub9e4\uc6b0 \uc595\uae30 \ub54c\ubb38\uc5d0 \uac00\ub2a5\uc131\uc774 \ub0ae\ub2e4\uace0 \uc5ec\uaca8\uc9c4\ub2e4. \ubc14\ub2e4\uc758 \uc554\ubaa8\ub2c8\uc544\ub098 \uae30\ud0c0 \ubd80\ub3d9\uc561\ub4e4\uc758 \ud568\ub7c9\uc740 \uc804\uccb4 \uc9c8\ub7c9\uc758 5% \uc815\ub3c4\uc774\ub9ac\ub77c\uace0 \uc5ec\uaca8\uc9c4\ub2e4. \uc774 \uacbd\uc6b0\uc5d0\ub294 \ubb3c\uacfc \uc5bc\uc74c\uc758 \uce35\uc774 250~300 km \ub450\uaed8\ub97c \uac00\uc9c8 \uc218 \uc788\ub2e4. \ub9cc\uc57d \ubc14\ub2e4\uac00 \uc5c6\ub2e4\uba74, \uc5bc\uc74c\uc73c\ub85c \uc774\ub8e8\uc5b4\uc9c4 \uc554\uad8c\uc740 \uc57d 300 km \uc815\ub3c4\uc758 \ub2e4\uc18c \ub450\uaebc\uc6b8 \uac83\uc73c\ub85c \uc0dd\uac01\ub41c\ub2e4.", "sentence_answer": "\uc774\ub7ec\ud55c \ud604\uc0c1\uc740 \ud0dc\uc591\uc774\ub098 \ubaa9\uc131\uc73c\ub85c\ubd80\ud130 \ubd88\uc5b4\uc624\ub294 \ud558\uc804\uc785\uc790 \ub54c\ubb38\uc5d0\ub3c4 \ubc1c\uc0dd\ud560 \uc218 \uc788\uc73c\ub098, \uce7c\ub9ac\uc2a4\ud1a0\uc758 \ub300\uae30\ub294 \ub9e4\uc6b0 \uc595\uae30 \ub54c\ubb38\uc5d0 \uac00\ub2a5\uc131\uc774 \ub0ae\ub2e4\uace0 \uc5ec\uaca8\uc9c4\ub2e4.", "paragraph_id": "6561311-3-1", "paragraph_question": "question: \ud0dc\uc591\uc774\ub098 \ubaa9\uc131\uc73c\ub85c\ubd80\ud130 \uc640\uc11c \uce7c\ub9ac\uc2a4\ud1a0\uac00 \uc804\ub3c4\ud558\ub294 \uad6c\ucc98\ub7fc \ubc18\uc751\ud558\uac8c \ub9cc\ub4dc\ub294 \uac83\uc740?, context: \uce7c\ub9ac\uc2a4\ud1a0\uc758 \uac70\uce5c \ud45c\uba74\uc740 \uc57d 80~150 km \ub450\uaed8\uc758 \ucc28\uac11\uace0, \uacbd\uc9c1\ub418\uc5b4 \uc788\uc73c\uba70, \uc5bc\uc5b4\ubd99\uc740 \uc554\uad8c \uc704\uc5d0 \uc788\ub2e4. \ub610\ud55c, \ubaa9\uc131\uacfc \uadf8 \uc8fc\ubcc0 \uc704\uc131\ub4e4\uc758 \uc790\uae30\uc7a5\uc744 \uc5f0\uad6c\ud55c \uacb0\uacfc\ub85c\uc11c, \uce7c\ub9ac\uc2a4\ud1a0\uc5d0 50~200 km \uae4a\uc774\uc758 \uc5fc\ubd84\uc774 \ud3ec\ud568\ub41c \ubc14\ub2e4\uac00 \uc9c0\uac01 \ubc11\uc5d0 \uc874\uc7ac\ud560 \uac83\uc73c\ub85c \ucd94\uce21\ub41c\ub2e4. \uce7c\ub9ac\uc2a4\ud1a0\uac00 \ubaa9\uc131\uc758 \uc9c0\uc18d\uc801\uc73c\ub85c \ubcc0\ud654\ud558\ub294 \uc790\uae30\uc7a5\uc5d0 \ub9c8\uce58 \uc644\ubcbd\ud788 \uc804\ub3c4\ud558\ub294 \uad6c\ucc98\ub7fc \ubc18\uc751\ud55c\ub2e4\ub294 \uac83\uc774 \ubc1d\ud600\uc84c\ub294\ub370, \uc774\ub294 \uc790\uae30\uc7a5\uc774 \uce7c\ub9ac\uc2a4\ud1a0\uc758 \ub0b4\ubd80\ub97c \ub6ab\uace0 \ub4e4\uc5b4\uac08 \uc218 \uc5c6\ub2e4\ub294 \uac83\uc744 \uc758\ubbf8\ud558\uba70, \uc774\uc5d0 \ub530\ub77c \uce7c\ub9ac\uc2a4\ud1a0 \ub0b4\ubd80\uc5d0 \ucd5c\uc18c\ud55c \uae4a\uc774\uac00 10 km \uc774\uc0c1\uc778, \ub9e4\uc6b0 \uc804\ub3c4\uc131\uc774 \ub192\uc740 \uce35\uc774 \uc788\ub2e4\ub294 \uc774\ub860\uc774 \uc81c\uae30\ub418\uc5c8\ub2e4. \uc774\ub7ec\ud55c \ud604\uc0c1\uc740 \ud0dc\uc591\uc774\ub098 \ubaa9\uc131\uc73c\ub85c\ubd80\ud130 \ubd88\uc5b4\uc624\ub294 \ud558\uc804\uc785\uc790 \ub54c\ubb38\uc5d0\ub3c4 \ubc1c\uc0dd\ud560 \uc218 \uc788\uc73c\ub098, \uce7c\ub9ac\uc2a4\ud1a0\uc758 \ub300\uae30\ub294 \ub9e4\uc6b0 \uc595\uae30 \ub54c\ubb38\uc5d0 \uac00\ub2a5\uc131\uc774 \ub0ae\ub2e4\uace0 \uc5ec\uaca8\uc9c4\ub2e4. \ubc14\ub2e4\uc758 \uc554\ubaa8\ub2c8\uc544\ub098 \uae30\ud0c0 \ubd80\ub3d9\uc561\ub4e4\uc758 \ud568\ub7c9\uc740 \uc804\uccb4 \uc9c8\ub7c9\uc758 5% \uc815\ub3c4\uc774\ub9ac\ub77c\uace0 \uc5ec\uaca8\uc9c4\ub2e4. \uc774 \uacbd\uc6b0\uc5d0\ub294 \ubb3c\uacfc \uc5bc\uc74c\uc758 \uce35\uc774 250~300 km \ub450\uaed8\ub97c \uac00\uc9c8 \uc218 \uc788\ub2e4. \ub9cc\uc57d \ubc14\ub2e4\uac00 \uc5c6\ub2e4\uba74, \uc5bc\uc74c\uc73c\ub85c \uc774\ub8e8\uc5b4\uc9c4 \uc554\uad8c\uc740 \uc57d 300 km \uc815\ub3c4\uc758 \ub2e4\uc18c \ub450\uaebc\uc6b8 \uac83\uc73c\ub85c \uc0dd\uac01\ub41c\ub2e4."} {"question": "1996\ub144 \uc774\uc601\ud559\uc740 \uc911\ud559\uad50 \uba87 \ud559\ub144\uc774\uc5c8\ub294\uac00?", "paragraph": "\uc774\uc601\ud559\uc740 \uacbd\uae30\ub3c4 \uc758\uc815\ubd80\uc2dc\uc5d0\uc11c \uc911\ud559\uad50\ub97c \ub2e4\ub154\ub2e4. \uc774\uc601\ud559\uc758 \uc911\ud559\uad50 \uad50\uc0ac\ub294 \"1996\ub144 \uc774\uc601\ud559\uc774 \uc911\ud559\uad50 2\ud559\ub144 \ub54c \ubab8\uc5d0 \ud53c\ub97c \ubb3b\ud788\uace0 \ub4f1\uad50\ud574 \ud53c\ud574 \uc5ec\ud559\uc0dd\uc758 \ud53c\ub77c\uba74\uc11c \ub3d9\uae09\uc0dd\ub4e4\uc5d0\uac8c \uc5ec\ud559\uc0dd \uc131\ud3ed\ud589 \uc0ac\uc2e4\uc744 \uc790\ub791\ud558\uace0 \ub2e4\ub140 \uc870\uc0ac\ud588\ub354\ub2c8 \uc2dc\uc778\ud588\ub2e4\"\uba70 \"\ud1f4\ud559\uc744 \uc2dc\ud0a4\ub824 \ud588\uc73c\ub098 \uad50\uc7a5\uc758 \ubc18\ub300\ub85c \uacbd\ubbf8\ud55c \uc9d5\uacc4\uc5d0 \uadf8\ucce4\ub2e4\"\uace0 \ud68c\uc0c1\ud588\ub2e4. \"\uc774\uc601\ud559\uc740 \ubb34\ub2e8\uacb0\uc11d \uc77c\uc218\uac00 \uc218\uc5c5\uc77c\uc218 1/3\uc744 \ucd08\uacfc\ud574 \uc878\uc5c5\uc774 \ubd88\uac00\ud55c \uc0c1\ud669\uc774\uc5c8\ub294\ub370 \uad50\uc7a5\uc774 \uc9c1\uad8c\uc73c\ub85c \uc0c1\uc704\ubc95\uc744 \uc704\ubc18\ud558\uba74\uc11c \uc740\ud3d0\ud574 \uc878\uc5c5\uc2dc\ucf30\ub2e4\"\uba70 \"\ub2f9\uc2dc \uc774\uc601\ud559\uc758 \ubd80\uce5c\uc774 \uc758\uc815\ubd80\uc758 \uc7ac\ub825\uac00\uc600\ub294\ub370 \uc5b4\uba38\ub2c8\uc758 \uce58\ub9db\ubc14\ub78c\uc774 \uc14c\ub2e4\"\uace0 \ub367\ubd99\uc600\ub2e4. \uc774\uc601\ud559\uc758 \uc911\ud559\uad50 \ub3d9\ucc3d\ub4e4\uc740 \"\uc774\uc601\ud559\uc774 \ub610\ub798 \uce5c\uad6c 3\uba85\uacfc \ud55c \uc5ec\uc131\uc744 \uc131\ud3ed\ud589\ud55c \ubaa8\uc2b5\uc744 \uc9c1\uc811 \ubaa9\uaca9\ud588\ub2e4\"\uba70 \ucd08\ub4f1\ud559\uc0dd\uc5d0\uac8c \uacfc\uc790\ub97c \uc8fc\uba70 \ub370\ub824\uac00 \uc131\ud3ed\ud589\ud588\ub2e4\uace0\ub3c4 \ub9d0\ud588\ub2e4. \uc774\uc601\ud559\uc758 \uc5b4\uba38\ub2c8 \uae40\ubaa8\uc528\ub294 \uc774\uc601\ud559\uc758 \uc544\ubc84\uc9c0\uac00 \uc0ac\uc5c5\uc5d0 \uc2e4\ud328\ud558\uba74\uc11c \uc774\ud63c\ud588\ub2e4.", "answer": "2\ud559\ub144", "sentence": "\uc774\uc601\ud559\uc758 \uc911\ud559\uad50 \uad50\uc0ac\ub294 \"1996\ub144 \uc774\uc601\ud559\uc774 \uc911\ud559\uad50 2\ud559\ub144 \ub54c \ubab8\uc5d0 \ud53c\ub97c \ubb3b\ud788\uace0 \ub4f1\uad50\ud574 \ud53c\ud574 \uc5ec\ud559\uc0dd\uc758 \ud53c\ub77c\uba74\uc11c \ub3d9\uae09\uc0dd\ub4e4\uc5d0\uac8c \uc5ec\ud559\uc0dd \uc131\ud3ed\ud589 \uc0ac\uc2e4\uc744 \uc790\ub791\ud558\uace0 \ub2e4\ub140 \uc870\uc0ac\ud588\ub354\ub2c8 \uc2dc\uc778\ud588\ub2e4\"\uba70 \"\ud1f4\ud559\uc744 \uc2dc\ud0a4\ub824 \ud588\uc73c\ub098 \uad50\uc7a5\uc758 \ubc18\ub300\ub85c \uacbd\ubbf8\ud55c \uc9d5\uacc4\uc5d0 \uadf8\ucce4\ub2e4\"\uace0 \ud68c\uc0c1\ud588\ub2e4.", "paragraph_sentence": "\uc774\uc601\ud559\uc740 \uacbd\uae30\ub3c4 \uc758\uc815\ubd80\uc2dc\uc5d0\uc11c \uc911\ud559\uad50\ub97c \ub2e4\ub154\ub2e4. \uc774\uc601\ud559\uc758 \uc911\ud559\uad50 \uad50\uc0ac\ub294 \"1996\ub144 \uc774\uc601\ud559\uc774 \uc911\ud559\uad50 2\ud559\ub144 \ub54c \ubab8\uc5d0 \ud53c\ub97c \ubb3b\ud788\uace0 \ub4f1\uad50\ud574 \ud53c\ud574 \uc5ec\ud559\uc0dd\uc758 \ud53c\ub77c\uba74\uc11c \ub3d9\uae09\uc0dd\ub4e4\uc5d0\uac8c \uc5ec\ud559\uc0dd \uc131\ud3ed\ud589 \uc0ac\uc2e4\uc744 \uc790\ub791\ud558\uace0 \ub2e4\ub140 \uc870\uc0ac\ud588\ub354\ub2c8 \uc2dc\uc778\ud588\ub2e4\"\uba70 \"\ud1f4\ud559\uc744 \uc2dc\ud0a4\ub824 \ud588\uc73c\ub098 \uad50\uc7a5\uc758 \ubc18\ub300\ub85c \uacbd\ubbf8\ud55c \uc9d5\uacc4\uc5d0 \uadf8\ucce4\ub2e4\"\uace0 \ud68c\uc0c1\ud588\ub2e4. \"\uc774\uc601\ud559\uc740 \ubb34\ub2e8\uacb0\uc11d \uc77c\uc218\uac00 \uc218\uc5c5\uc77c\uc218 1/3\uc744 \ucd08\uacfc\ud574 \uc878\uc5c5\uc774 \ubd88\uac00\ud55c \uc0c1\ud669\uc774\uc5c8\ub294\ub370 \uad50\uc7a5\uc774 \uc9c1\uad8c\uc73c\ub85c \uc0c1\uc704\ubc95\uc744 \uc704\ubc18\ud558\uba74\uc11c \uc740\ud3d0\ud574 \uc878\uc5c5\uc2dc\ucf30\ub2e4\"\uba70 \"\ub2f9\uc2dc \uc774\uc601\ud559\uc758 \ubd80\uce5c\uc774 \uc758\uc815\ubd80\uc758 \uc7ac\ub825\uac00\uc600\ub294\ub370 \uc5b4\uba38\ub2c8\uc758 \uce58\ub9db\ubc14\ub78c\uc774 \uc14c\ub2e4\"\uace0 \ub367\ubd99\uc600\ub2e4. \uc774\uc601\ud559\uc758 \uc911\ud559\uad50 \ub3d9\ucc3d\ub4e4\uc740 \"\uc774\uc601\ud559\uc774 \ub610\ub798 \uce5c\uad6c 3\uba85\uacfc \ud55c \uc5ec\uc131\uc744 \uc131\ud3ed\ud589\ud55c \ubaa8\uc2b5\uc744 \uc9c1\uc811 \ubaa9\uaca9\ud588\ub2e4\"\uba70 \ucd08\ub4f1\ud559\uc0dd\uc5d0\uac8c \uacfc\uc790\ub97c \uc8fc\uba70 \ub370\ub824\uac00 \uc131\ud3ed\ud589\ud588\ub2e4\uace0\ub3c4 \ub9d0\ud588\ub2e4. \uc774\uc601\ud559\uc758 \uc5b4\uba38\ub2c8 \uae40\ubaa8\uc528\ub294 \uc774\uc601\ud559\uc758 \uc544\ubc84\uc9c0\uac00 \uc0ac\uc5c5\uc5d0 \uc2e4\ud328\ud558\uba74\uc11c \uc774\ud63c\ud588\ub2e4.", "paragraph_answer": "\uc774\uc601\ud559\uc740 \uacbd\uae30\ub3c4 \uc758\uc815\ubd80\uc2dc\uc5d0\uc11c \uc911\ud559\uad50\ub97c \ub2e4\ub154\ub2e4. \uc774\uc601\ud559\uc758 \uc911\ud559\uad50 \uad50\uc0ac\ub294 \"1996\ub144 \uc774\uc601\ud559\uc774 \uc911\ud559\uad50 2\ud559\ub144 \ub54c \ubab8\uc5d0 \ud53c\ub97c \ubb3b\ud788\uace0 \ub4f1\uad50\ud574 \ud53c\ud574 \uc5ec\ud559\uc0dd\uc758 \ud53c\ub77c\uba74\uc11c \ub3d9\uae09\uc0dd\ub4e4\uc5d0\uac8c \uc5ec\ud559\uc0dd \uc131\ud3ed\ud589 \uc0ac\uc2e4\uc744 \uc790\ub791\ud558\uace0 \ub2e4\ub140 \uc870\uc0ac\ud588\ub354\ub2c8 \uc2dc\uc778\ud588\ub2e4\"\uba70 \"\ud1f4\ud559\uc744 \uc2dc\ud0a4\ub824 \ud588\uc73c\ub098 \uad50\uc7a5\uc758 \ubc18\ub300\ub85c \uacbd\ubbf8\ud55c \uc9d5\uacc4\uc5d0 \uadf8\ucce4\ub2e4\"\uace0 \ud68c\uc0c1\ud588\ub2e4. \"\uc774\uc601\ud559\uc740 \ubb34\ub2e8\uacb0\uc11d \uc77c\uc218\uac00 \uc218\uc5c5\uc77c\uc218 1/3\uc744 \ucd08\uacfc\ud574 \uc878\uc5c5\uc774 \ubd88\uac00\ud55c \uc0c1\ud669\uc774\uc5c8\ub294\ub370 \uad50\uc7a5\uc774 \uc9c1\uad8c\uc73c\ub85c \uc0c1\uc704\ubc95\uc744 \uc704\ubc18\ud558\uba74\uc11c \uc740\ud3d0\ud574 \uc878\uc5c5\uc2dc\ucf30\ub2e4\"\uba70 \"\ub2f9\uc2dc \uc774\uc601\ud559\uc758 \ubd80\uce5c\uc774 \uc758\uc815\ubd80\uc758 \uc7ac\ub825\uac00\uc600\ub294\ub370 \uc5b4\uba38\ub2c8\uc758 \uce58\ub9db\ubc14\ub78c\uc774 \uc14c\ub2e4\"\uace0 \ub367\ubd99\uc600\ub2e4. \uc774\uc601\ud559\uc758 \uc911\ud559\uad50 \ub3d9\ucc3d\ub4e4\uc740 \"\uc774\uc601\ud559\uc774 \ub610\ub798 \uce5c\uad6c 3\uba85\uacfc \ud55c \uc5ec\uc131\uc744 \uc131\ud3ed\ud589\ud55c \ubaa8\uc2b5\uc744 \uc9c1\uc811 \ubaa9\uaca9\ud588\ub2e4\"\uba70 \ucd08\ub4f1\ud559\uc0dd\uc5d0\uac8c \uacfc\uc790\ub97c \uc8fc\uba70 \ub370\ub824\uac00 \uc131\ud3ed\ud589\ud588\ub2e4\uace0\ub3c4 \ub9d0\ud588\ub2e4. \uc774\uc601\ud559\uc758 \uc5b4\uba38\ub2c8 \uae40\ubaa8\uc528\ub294 \uc774\uc601\ud559\uc758 \uc544\ubc84\uc9c0\uac00 \uc0ac\uc5c5\uc5d0 \uc2e4\ud328\ud558\uba74\uc11c \uc774\ud63c\ud588\ub2e4.", "sentence_answer": "\uc774\uc601\ud559\uc758 \uc911\ud559\uad50 \uad50\uc0ac\ub294 \"1996\ub144 \uc774\uc601\ud559\uc774 \uc911\ud559\uad50 2\ud559\ub144 \ub54c \ubab8\uc5d0 \ud53c\ub97c \ubb3b\ud788\uace0 \ub4f1\uad50\ud574 \ud53c\ud574 \uc5ec\ud559\uc0dd\uc758 \ud53c\ub77c\uba74\uc11c \ub3d9\uae09\uc0dd\ub4e4\uc5d0\uac8c \uc5ec\ud559\uc0dd \uc131\ud3ed\ud589 \uc0ac\uc2e4\uc744 \uc790\ub791\ud558\uace0 \ub2e4\ub140 \uc870\uc0ac\ud588\ub354\ub2c8 \uc2dc\uc778\ud588\ub2e4\"\uba70 \"\ud1f4\ud559\uc744 \uc2dc\ud0a4\ub824 \ud588\uc73c\ub098 \uad50\uc7a5\uc758 \ubc18\ub300\ub85c \uacbd\ubbf8\ud55c \uc9d5\uacc4\uc5d0 \uadf8\ucce4\ub2e4\"\uace0 \ud68c\uc0c1\ud588\ub2e4.", "paragraph_id": "6545802-0-0", "paragraph_question": "question: 1996\ub144 \uc774\uc601\ud559\uc740 \uc911\ud559\uad50 \uba87 \ud559\ub144\uc774\uc5c8\ub294\uac00?, context: \uc774\uc601\ud559\uc740 \uacbd\uae30\ub3c4 \uc758\uc815\ubd80\uc2dc\uc5d0\uc11c \uc911\ud559\uad50\ub97c \ub2e4\ub154\ub2e4. \uc774\uc601\ud559\uc758 \uc911\ud559\uad50 \uad50\uc0ac\ub294 \"1996\ub144 \uc774\uc601\ud559\uc774 \uc911\ud559\uad50 2\ud559\ub144 \ub54c \ubab8\uc5d0 \ud53c\ub97c \ubb3b\ud788\uace0 \ub4f1\uad50\ud574 \ud53c\ud574 \uc5ec\ud559\uc0dd\uc758 \ud53c\ub77c\uba74\uc11c \ub3d9\uae09\uc0dd\ub4e4\uc5d0\uac8c \uc5ec\ud559\uc0dd \uc131\ud3ed\ud589 \uc0ac\uc2e4\uc744 \uc790\ub791\ud558\uace0 \ub2e4\ub140 \uc870\uc0ac\ud588\ub354\ub2c8 \uc2dc\uc778\ud588\ub2e4\"\uba70 \"\ud1f4\ud559\uc744 \uc2dc\ud0a4\ub824 \ud588\uc73c\ub098 \uad50\uc7a5\uc758 \ubc18\ub300\ub85c \uacbd\ubbf8\ud55c \uc9d5\uacc4\uc5d0 \uadf8\ucce4\ub2e4\"\uace0 \ud68c\uc0c1\ud588\ub2e4. \"\uc774\uc601\ud559\uc740 \ubb34\ub2e8\uacb0\uc11d \uc77c\uc218\uac00 \uc218\uc5c5\uc77c\uc218 1/3\uc744 \ucd08\uacfc\ud574 \uc878\uc5c5\uc774 \ubd88\uac00\ud55c \uc0c1\ud669\uc774\uc5c8\ub294\ub370 \uad50\uc7a5\uc774 \uc9c1\uad8c\uc73c\ub85c \uc0c1\uc704\ubc95\uc744 \uc704\ubc18\ud558\uba74\uc11c \uc740\ud3d0\ud574 \uc878\uc5c5\uc2dc\ucf30\ub2e4\"\uba70 \"\ub2f9\uc2dc \uc774\uc601\ud559\uc758 \ubd80\uce5c\uc774 \uc758\uc815\ubd80\uc758 \uc7ac\ub825\uac00\uc600\ub294\ub370 \uc5b4\uba38\ub2c8\uc758 \uce58\ub9db\ubc14\ub78c\uc774 \uc14c\ub2e4\"\uace0 \ub367\ubd99\uc600\ub2e4. \uc774\uc601\ud559\uc758 \uc911\ud559\uad50 \ub3d9\ucc3d\ub4e4\uc740 \"\uc774\uc601\ud559\uc774 \ub610\ub798 \uce5c\uad6c 3\uba85\uacfc \ud55c \uc5ec\uc131\uc744 \uc131\ud3ed\ud589\ud55c \ubaa8\uc2b5\uc744 \uc9c1\uc811 \ubaa9\uaca9\ud588\ub2e4\"\uba70 \ucd08\ub4f1\ud559\uc0dd\uc5d0\uac8c \uacfc\uc790\ub97c \uc8fc\uba70 \ub370\ub824\uac00 \uc131\ud3ed\ud589\ud588\ub2e4\uace0\ub3c4 \ub9d0\ud588\ub2e4. \uc774\uc601\ud559\uc758 \uc5b4\uba38\ub2c8 \uae40\ubaa8\uc528\ub294 \uc774\uc601\ud559\uc758 \uc544\ubc84\uc9c0\uac00 \uc0ac\uc5c5\uc5d0 \uc2e4\ud328\ud558\uba74\uc11c \uc774\ud63c\ud588\ub2e4."} {"question": "\uc2e0\uadf9\uc801\uc778 \ub300\uc0ac\ub294 \ucc3d\uadf9\uc5d0 \uc801\ud569\uc9c0 \uc54a\uace0 \ub300\uccb4\ub85c \ubb34\uc5c7\uc73c\ub85c \uc0ac\uc6a9\ud558\ub294\uac00?", "paragraph": "\ubb34\ub300\uc7a5\uce58\ub294 \ub2e4\ubaa9\uc801 \uc0ac\uc6a9\ubb34\ub300\ub85c, \uc989 \uac04\ub7b5\ud558\ub098\ub9c8 \ub3d9\uc591\ud654\uc801\uc778 \ud654\ud48d\uc5d0 \ud55c \uc7a5\uba74\uc774 \ub2e4\uc74c \uc7a5\uba74\uc5d0\uae4c\uc9c0 \uc751\uc6a9\ub418\uba70 \ub610\ub294 \ud55c\ub450 \ucabd\uc758 \ubcc0\ud654\ub85c \uac04\ub2e8\ud788 \ub2e4\ub978 \uc7a5\uc18c\ub85c \ubcc0\uacbd\uc2dc\ud0ac \uc218 \uc788\uc5c8\uc73c\uba70 \ubb34\ub300 \uc804\uba74\uc758 \uc88c\uce21\uc5d0\ub294 \uc545\uc0ac\uc11d, \uc6b0\uce21\uc740 \ub3c4\ucc3d\uc11d(\u5c0e\u5531\u5e2d)\uc73c\ub85c \uc124\uc815\ud558\uc600\ub2e4. \ub3c4\ucc3d\uc774\ub77c \ud568\uc740 \ucc3d\uadf9\uc758 \uc2dc\ucd08\uc640 \ub05d\uc744 \ucc3d\uacfc \uc544\ub2c8\ub9ac\uc640 \ubc1c\ub9bc\uc73c\ub85c \uc9c4\ud589\uc2dc\ud0a4\uace0, \uadf9 \uc0ac\uc774 \uc0ac\uc774\uc5d0 \uc124\uba85\uacfc \ubb18\uc0ac\ub97c \ud558\uc5ec \ub2e4\uac01\ub3c4\ub85c \ubcc0\ud558\ub294 \ubb34\ub300\uc7a5\uba74\uc744 \uc720\uae30\uc801\uc73c\ub85c \ub04c\uc5b4\uac00\ub294 \uc5ed\ud560\uc774\ub2e4. \uc5ec\uae30\uc5d0\uc11c \ucc3d\uadf9 \uc5f0\uae30\uc790\ub4e4\uc758 \uc5f0\uae30 \ubc29\ubc95\uc740 \uc18c\uc704 \uc2e0\uadf9\uc801\uc778 \uc5f0\uae30\uc5d0\uc11c \uc644\uc804\ud788 \ubc97\uc5b4\ub098 \ud310\uc18c\ub9ac\uc5d0 \uc788\ub294 \uc544\ub2c8\ub9ac(\ub300\uc0ac)\uc640 \ubc1c\ub9bc(\ud615\uc6a9\ub3d9\uc791)\uc744 \uc8fc\ub85c \uadf8 \uc5f0\uae30\uba74\uc5d0 \uc0ac\uc6a9\ud55c\ub2e4. \uadf8\ub7ec\ub098 \uc544\ub2c8\ub9ac\uc640 \ubc1c\ub9bc(\ud639\uc740 \ub108\ub984\uc0c8)\ub9cc\uc73c\ub85c\ub294 \ucc3d\uadf9 \ud45c\ud604\uc774 \ubd80\uc871\ud558\ub2e4. \ucc3d\uadf9\uc774 \uc6b0\ub9ac\ub098\ub77c \uace0\uc720\uc758 \uc5f0\ucd9c\uacfc \uc5f0\uae30\ub97c \ubc1c\ud718\ud558\ub824\uba74 \ud55c\uad6d\uc801\uc778 \ubbfc\uc18d\uc5d0\uc11c \ucc3d\uadf9\uc758 \uadf9\uc801 \uc694\uc18c\uc640 \uc5f0\uae30\uba74\uc744 \ubcf4\ucda9\ud560 \uc218 \uc788\uc5b4\uc57c \ud560 \uc904 \uc548\ub2e4. \uadf8\ub807\ub2e4\uba74 \ud0c8\ucda4\u00b7\uac00\uba74\uadf9\u00b7\uc778\ud615\uadf9\uc758 \uadf9\uc801 \ud615\uc2dd\uc744 \uac00\ubbf8\uc2dc\ud0b4\uc73c\ub85c\uc368 \ucc3d\uadf9\uc740 \uc11c\uad6c\uc801\uc778 \uc5f0\uadf9 \ud615\ud0dc\ub97c \ube4c\uc9c0 \uc54a\ub294 \ub3c5\ud2b9\ud55c \uace0\uc720\uc758 \uc6b0\ub9ac \ucc3d\uadf9\uc73c\ub85c \ubb34\ub300\ud654\ud560 \uc218 \uc788\uc5b4\uc57c \ud558\uba70, \uc2e0\uadf9\uc801\uc778 \ub300\uc0ac\ub294 \ucc3d\uadf9\uc5d0\ub294 \uc801\ud569\uc9c0 \uc54a\uace0 \ub300\uccb4\ub85c \uc544\ub2c8\ub9ac \uc870(\u8abf)\ub85c \ub300\uc0ac\ub97c \uc0ac\uc6a9\ud558\uc5ec\uc57c \ud55c\ub2e4.", "answer": "\uc544\ub2c8\ub9ac \uc870", "sentence": "\ub300\uccb4\ub85c \uc544\ub2c8\ub9ac \uc870 (\u8abf)", "paragraph_sentence": "\ubb34\ub300\uc7a5\uce58\ub294 \ub2e4\ubaa9\uc801 \uc0ac\uc6a9\ubb34\ub300\ub85c, \uc989 \uac04\ub7b5\ud558\ub098\ub9c8 \ub3d9\uc591\ud654\uc801\uc778 \ud654\ud48d\uc5d0 \ud55c \uc7a5\uba74\uc774 \ub2e4\uc74c \uc7a5\uba74\uc5d0\uae4c\uc9c0 \uc751\uc6a9\ub418\uba70 \ub610\ub294 \ud55c\ub450 \ucabd\uc758 \ubcc0\ud654\ub85c \uac04\ub2e8\ud788 \ub2e4\ub978 \uc7a5\uc18c\ub85c \ubcc0\uacbd\uc2dc\ud0ac \uc218 \uc788\uc5c8\uc73c\uba70 \ubb34\ub300 \uc804\uba74\uc758 \uc88c\uce21\uc5d0\ub294 \uc545\uc0ac\uc11d, \uc6b0\uce21\uc740 \ub3c4\ucc3d\uc11d(\u5c0e\u5531\u5e2d)\uc73c\ub85c \uc124\uc815\ud558\uc600\ub2e4. \ub3c4\ucc3d\uc774\ub77c \ud568\uc740 \ucc3d\uadf9\uc758 \uc2dc\ucd08\uc640 \ub05d\uc744 \ucc3d\uacfc \uc544\ub2c8\ub9ac\uc640 \ubc1c\ub9bc\uc73c\ub85c \uc9c4\ud589\uc2dc\ud0a4\uace0, \uadf9 \uc0ac\uc774 \uc0ac\uc774\uc5d0 \uc124\uba85\uacfc \ubb18\uc0ac\ub97c \ud558\uc5ec \ub2e4\uac01\ub3c4\ub85c \ubcc0\ud558\ub294 \ubb34\ub300\uc7a5\uba74\uc744 \uc720\uae30\uc801\uc73c\ub85c \ub04c\uc5b4\uac00\ub294 \uc5ed\ud560\uc774\ub2e4. \uc5ec\uae30\uc5d0\uc11c \ucc3d\uadf9 \uc5f0\uae30\uc790\ub4e4\uc758 \uc5f0\uae30 \ubc29\ubc95\uc740 \uc18c\uc704 \uc2e0\uadf9\uc801\uc778 \uc5f0\uae30\uc5d0\uc11c \uc644\uc804\ud788 \ubc97\uc5b4\ub098 \ud310\uc18c\ub9ac\uc5d0 \uc788\ub294 \uc544\ub2c8\ub9ac(\ub300\uc0ac)\uc640 \ubc1c\ub9bc(\ud615\uc6a9\ub3d9\uc791)\uc744 \uc8fc\ub85c \uadf8 \uc5f0\uae30\uba74\uc5d0 \uc0ac\uc6a9\ud55c\ub2e4. \uadf8\ub7ec\ub098 \uc544\ub2c8\ub9ac\uc640 \ubc1c\ub9bc(\ud639\uc740 \ub108\ub984\uc0c8)\ub9cc\uc73c\ub85c\ub294 \ucc3d\uadf9 \ud45c\ud604\uc774 \ubd80\uc871\ud558\ub2e4. \ucc3d\uadf9\uc774 \uc6b0\ub9ac\ub098\ub77c \uace0\uc720\uc758 \uc5f0\ucd9c\uacfc \uc5f0\uae30\ub97c \ubc1c\ud718\ud558\ub824\uba74 \ud55c\uad6d\uc801\uc778 \ubbfc\uc18d\uc5d0\uc11c \ucc3d\uadf9\uc758 \uadf9\uc801 \uc694\uc18c\uc640 \uc5f0\uae30\uba74\uc744 \ubcf4\ucda9\ud560 \uc218 \uc788\uc5b4\uc57c \ud560 \uc904 \uc548\ub2e4. \uadf8\ub807\ub2e4\uba74 \ud0c8\ucda4\u00b7\uac00\uba74\uadf9\u00b7\uc778\ud615\uadf9\uc758 \uadf9\uc801 \ud615\uc2dd\uc744 \uac00\ubbf8\uc2dc\ud0b4\uc73c\ub85c\uc368 \ucc3d\uadf9\uc740 \uc11c\uad6c\uc801\uc778 \uc5f0\uadf9 \ud615\ud0dc\ub97c \ube4c\uc9c0 \uc54a\ub294 \ub3c5\ud2b9\ud55c \uace0\uc720\uc758 \uc6b0\ub9ac \ucc3d\uadf9\uc73c\ub85c \ubb34\ub300\ud654\ud560 \uc218 \uc788\uc5b4\uc57c \ud558\uba70, \uc2e0\uadf9\uc801\uc778 \ub300\uc0ac\ub294 \ucc3d\uadf9\uc5d0\ub294 \uc801\ud569\uc9c0 \uc54a\uace0 \ub300\uccb4\ub85c \uc544\ub2c8\ub9ac \uc870 (\u8abf) \ub85c \ub300\uc0ac\ub97c \uc0ac\uc6a9\ud558\uc5ec\uc57c \ud55c\ub2e4.", "paragraph_answer": "\ubb34\ub300\uc7a5\uce58\ub294 \ub2e4\ubaa9\uc801 \uc0ac\uc6a9\ubb34\ub300\ub85c, \uc989 \uac04\ub7b5\ud558\ub098\ub9c8 \ub3d9\uc591\ud654\uc801\uc778 \ud654\ud48d\uc5d0 \ud55c \uc7a5\uba74\uc774 \ub2e4\uc74c \uc7a5\uba74\uc5d0\uae4c\uc9c0 \uc751\uc6a9\ub418\uba70 \ub610\ub294 \ud55c\ub450 \ucabd\uc758 \ubcc0\ud654\ub85c \uac04\ub2e8\ud788 \ub2e4\ub978 \uc7a5\uc18c\ub85c \ubcc0\uacbd\uc2dc\ud0ac \uc218 \uc788\uc5c8\uc73c\uba70 \ubb34\ub300 \uc804\uba74\uc758 \uc88c\uce21\uc5d0\ub294 \uc545\uc0ac\uc11d, \uc6b0\uce21\uc740 \ub3c4\ucc3d\uc11d(\u5c0e\u5531\u5e2d)\uc73c\ub85c \uc124\uc815\ud558\uc600\ub2e4. \ub3c4\ucc3d\uc774\ub77c \ud568\uc740 \ucc3d\uadf9\uc758 \uc2dc\ucd08\uc640 \ub05d\uc744 \ucc3d\uacfc \uc544\ub2c8\ub9ac\uc640 \ubc1c\ub9bc\uc73c\ub85c \uc9c4\ud589\uc2dc\ud0a4\uace0, \uadf9 \uc0ac\uc774 \uc0ac\uc774\uc5d0 \uc124\uba85\uacfc \ubb18\uc0ac\ub97c \ud558\uc5ec \ub2e4\uac01\ub3c4\ub85c \ubcc0\ud558\ub294 \ubb34\ub300\uc7a5\uba74\uc744 \uc720\uae30\uc801\uc73c\ub85c \ub04c\uc5b4\uac00\ub294 \uc5ed\ud560\uc774\ub2e4. \uc5ec\uae30\uc5d0\uc11c \ucc3d\uadf9 \uc5f0\uae30\uc790\ub4e4\uc758 \uc5f0\uae30 \ubc29\ubc95\uc740 \uc18c\uc704 \uc2e0\uadf9\uc801\uc778 \uc5f0\uae30\uc5d0\uc11c \uc644\uc804\ud788 \ubc97\uc5b4\ub098 \ud310\uc18c\ub9ac\uc5d0 \uc788\ub294 \uc544\ub2c8\ub9ac(\ub300\uc0ac)\uc640 \ubc1c\ub9bc(\ud615\uc6a9\ub3d9\uc791)\uc744 \uc8fc\ub85c \uadf8 \uc5f0\uae30\uba74\uc5d0 \uc0ac\uc6a9\ud55c\ub2e4. \uadf8\ub7ec\ub098 \uc544\ub2c8\ub9ac\uc640 \ubc1c\ub9bc(\ud639\uc740 \ub108\ub984\uc0c8)\ub9cc\uc73c\ub85c\ub294 \ucc3d\uadf9 \ud45c\ud604\uc774 \ubd80\uc871\ud558\ub2e4. \ucc3d\uadf9\uc774 \uc6b0\ub9ac\ub098\ub77c \uace0\uc720\uc758 \uc5f0\ucd9c\uacfc \uc5f0\uae30\ub97c \ubc1c\ud718\ud558\ub824\uba74 \ud55c\uad6d\uc801\uc778 \ubbfc\uc18d\uc5d0\uc11c \ucc3d\uadf9\uc758 \uadf9\uc801 \uc694\uc18c\uc640 \uc5f0\uae30\uba74\uc744 \ubcf4\ucda9\ud560 \uc218 \uc788\uc5b4\uc57c \ud560 \uc904 \uc548\ub2e4. \uadf8\ub807\ub2e4\uba74 \ud0c8\ucda4\u00b7\uac00\uba74\uadf9\u00b7\uc778\ud615\uadf9\uc758 \uadf9\uc801 \ud615\uc2dd\uc744 \uac00\ubbf8\uc2dc\ud0b4\uc73c\ub85c\uc368 \ucc3d\uadf9\uc740 \uc11c\uad6c\uc801\uc778 \uc5f0\uadf9 \ud615\ud0dc\ub97c \ube4c\uc9c0 \uc54a\ub294 \ub3c5\ud2b9\ud55c \uace0\uc720\uc758 \uc6b0\ub9ac \ucc3d\uadf9\uc73c\ub85c \ubb34\ub300\ud654\ud560 \uc218 \uc788\uc5b4\uc57c \ud558\uba70, \uc2e0\uadf9\uc801\uc778 \ub300\uc0ac\ub294 \ucc3d\uadf9\uc5d0\ub294 \uc801\ud569\uc9c0 \uc54a\uace0 \ub300\uccb4\ub85c \uc544\ub2c8\ub9ac \uc870 (\u8abf)\ub85c \ub300\uc0ac\ub97c \uc0ac\uc6a9\ud558\uc5ec\uc57c \ud55c\ub2e4.", "sentence_answer": "\ub300\uccb4\ub85c \uc544\ub2c8\ub9ac \uc870 (\u8abf)", "paragraph_id": "6557875-3-1", "paragraph_question": "question: \uc2e0\uadf9\uc801\uc778 \ub300\uc0ac\ub294 \ucc3d\uadf9\uc5d0 \uc801\ud569\uc9c0 \uc54a\uace0 \ub300\uccb4\ub85c \ubb34\uc5c7\uc73c\ub85c \uc0ac\uc6a9\ud558\ub294\uac00?, context: \ubb34\ub300\uc7a5\uce58\ub294 \ub2e4\ubaa9\uc801 \uc0ac\uc6a9\ubb34\ub300\ub85c, \uc989 \uac04\ub7b5\ud558\ub098\ub9c8 \ub3d9\uc591\ud654\uc801\uc778 \ud654\ud48d\uc5d0 \ud55c \uc7a5\uba74\uc774 \ub2e4\uc74c \uc7a5\uba74\uc5d0\uae4c\uc9c0 \uc751\uc6a9\ub418\uba70 \ub610\ub294 \ud55c\ub450 \ucabd\uc758 \ubcc0\ud654\ub85c \uac04\ub2e8\ud788 \ub2e4\ub978 \uc7a5\uc18c\ub85c \ubcc0\uacbd\uc2dc\ud0ac \uc218 \uc788\uc5c8\uc73c\uba70 \ubb34\ub300 \uc804\uba74\uc758 \uc88c\uce21\uc5d0\ub294 \uc545\uc0ac\uc11d, \uc6b0\uce21\uc740 \ub3c4\ucc3d\uc11d(\u5c0e\u5531\u5e2d)\uc73c\ub85c \uc124\uc815\ud558\uc600\ub2e4. \ub3c4\ucc3d\uc774\ub77c \ud568\uc740 \ucc3d\uadf9\uc758 \uc2dc\ucd08\uc640 \ub05d\uc744 \ucc3d\uacfc \uc544\ub2c8\ub9ac\uc640 \ubc1c\ub9bc\uc73c\ub85c \uc9c4\ud589\uc2dc\ud0a4\uace0, \uadf9 \uc0ac\uc774 \uc0ac\uc774\uc5d0 \uc124\uba85\uacfc \ubb18\uc0ac\ub97c \ud558\uc5ec \ub2e4\uac01\ub3c4\ub85c \ubcc0\ud558\ub294 \ubb34\ub300\uc7a5\uba74\uc744 \uc720\uae30\uc801\uc73c\ub85c \ub04c\uc5b4\uac00\ub294 \uc5ed\ud560\uc774\ub2e4. \uc5ec\uae30\uc5d0\uc11c \ucc3d\uadf9 \uc5f0\uae30\uc790\ub4e4\uc758 \uc5f0\uae30 \ubc29\ubc95\uc740 \uc18c\uc704 \uc2e0\uadf9\uc801\uc778 \uc5f0\uae30\uc5d0\uc11c \uc644\uc804\ud788 \ubc97\uc5b4\ub098 \ud310\uc18c\ub9ac\uc5d0 \uc788\ub294 \uc544\ub2c8\ub9ac(\ub300\uc0ac)\uc640 \ubc1c\ub9bc(\ud615\uc6a9\ub3d9\uc791)\uc744 \uc8fc\ub85c \uadf8 \uc5f0\uae30\uba74\uc5d0 \uc0ac\uc6a9\ud55c\ub2e4. \uadf8\ub7ec\ub098 \uc544\ub2c8\ub9ac\uc640 \ubc1c\ub9bc(\ud639\uc740 \ub108\ub984\uc0c8)\ub9cc\uc73c\ub85c\ub294 \ucc3d\uadf9 \ud45c\ud604\uc774 \ubd80\uc871\ud558\ub2e4. \ucc3d\uadf9\uc774 \uc6b0\ub9ac\ub098\ub77c \uace0\uc720\uc758 \uc5f0\ucd9c\uacfc \uc5f0\uae30\ub97c \ubc1c\ud718\ud558\ub824\uba74 \ud55c\uad6d\uc801\uc778 \ubbfc\uc18d\uc5d0\uc11c \ucc3d\uadf9\uc758 \uadf9\uc801 \uc694\uc18c\uc640 \uc5f0\uae30\uba74\uc744 \ubcf4\ucda9\ud560 \uc218 \uc788\uc5b4\uc57c \ud560 \uc904 \uc548\ub2e4. \uadf8\ub807\ub2e4\uba74 \ud0c8\ucda4\u00b7\uac00\uba74\uadf9\u00b7\uc778\ud615\uadf9\uc758 \uadf9\uc801 \ud615\uc2dd\uc744 \uac00\ubbf8\uc2dc\ud0b4\uc73c\ub85c\uc368 \ucc3d\uadf9\uc740 \uc11c\uad6c\uc801\uc778 \uc5f0\uadf9 \ud615\ud0dc\ub97c \ube4c\uc9c0 \uc54a\ub294 \ub3c5\ud2b9\ud55c \uace0\uc720\uc758 \uc6b0\ub9ac \ucc3d\uadf9\uc73c\ub85c \ubb34\ub300\ud654\ud560 \uc218 \uc788\uc5b4\uc57c \ud558\uba70, \uc2e0\uadf9\uc801\uc778 \ub300\uc0ac\ub294 \ucc3d\uadf9\uc5d0\ub294 \uc801\ud569\uc9c0 \uc54a\uace0 \ub300\uccb4\ub85c \uc544\ub2c8\ub9ac \uc870(\u8abf)\ub85c \ub300\uc0ac\ub97c \uc0ac\uc6a9\ud558\uc5ec\uc57c \ud55c\ub2e4."} {"question": "2003\ub144 10\uc6d4 5\uc77c \ub9c8\uc9c0\ub9c9\uc73c\ub85c \ubb34\ub300\ub97c \uc624\ub974\uac8c \ub41c \ud398\uc2a4\ud2f0\ubc8c \uc774\ub984\uc740?", "paragraph": "2003\ub144 \ud558\ud604\uc6b0, \uc804\uaddc\ud638, \uc774\uc815\uae38\uc740 \ubc34\ub4dc\uba85\uc744 The C.O.M(\ub354 \ucef4)\uc73c\ub85c \ubcc0\uacbd\ud55c\ub2e4. \ubc15\ucca0\uc601\uc744 \uc0c8\ub85c\uc6b4 \ubca0\uc774\uc2dc\uc2a4\ud2b8\ub85c \uc601\uc785\ud558\uace0 \ub77c\uc774\ube0c\ud074\ub7fd \uc2ac\ub7ec\uac70, \uc7ac\uba38\uc2a4 \ub4f1\uc9c0\uc5d0\uc11c \uacf5\uc5f0\uc744 \ud558\uba70 \ud65c\ub3d9\uc744 \uc2dc\uc791\ud55c\ub2e4. \uadf8\ub7ec\ub098 \uadf8\ud574 8\uc6d4, \uad70\uc785\ub300\ubb38\uc81c\ub85c \uc778\ud574 \ud300\uc740 \ud574\uccb4\ud558\uac8c \ub41c\ub2e4. 2003\ub144 10\uc6d4 5\uc77c \uc308\uc9c0 \uc0ac\uc6b4\ub4dc \ud398\uc2a4\ud2f0\ubc8c \uc228\uc740\uace0\uc218\uc5d0 \uc120\uc815\ub41c\ub2e4. \ud300\uc740 \uc774\ubbf8 \ud574\uccb4\ud55c \uc0c1\ud0dc\uc600\uc9c0\ub9cc \uc308\uc9c0 \uc0ac\uc6b4\ub4dc \ud398\uc2a4\ud2f0\ubc8c\uc744 \ub9c8\uc9c0\ub9c9\uc73c\ub85c \ubb34\ub300\uc5d0 \uc624\ub974\uac8c \ub41c\ub2e4. 2004\ub144 \ud558\ud604\uc6b0, \uc774\uc815\uae38\uc740 \uad70\uc785\ub300\ub97c \ud558\uc600\uace0 \uc804\uaddc\ud638\ub294 \uace0\ud5a5\uc778 \uac15\uc6d0\ub3c4\ub85c \ub3cc\uc544\uac04\ub2e4. 2006\ub144 4\uc6d4, \ud558\ud604\uc6b0\ub294 \uad70\uc81c\ub300\ub97c \ud558\uace0 \uc804\uaddc\ud638\ub97c \ub9cc\ub098 \ubc34\ub4dc\ub97c \ub2e4\uc2dc \ud558\uae30\ub85c\ud55c\ub2e4. \uac15\uc6d0\ub3c4\uc5d0 \uc788\ub294 \uc804\uaddc\ud638\uc758 \ubd80\ubaa8\ub2d8\uc774 \uc6b4\uc601\ud558\ub294 \ud39c\uc158\uc5d0\uc11c \ud568\uaed8 \uc0b4\uba74\uc11c 7\uac1c\uc6d4 \ub3d9\uc548 \uace1\uc744 \uc4f0\uac8c \ub41c\ub2e4.", "answer": "\uc308\uc9c0 \uc0ac\uc6b4\ub4dc \ud398\uc2a4\ud2f0\ubc8c", "sentence": "2003\ub144 10\uc6d4 5\uc77c \uc308\uc9c0 \uc0ac\uc6b4\ub4dc \ud398\uc2a4\ud2f0\ubc8c \uc228\uc740\uace0\uc218\uc5d0 \uc120\uc815\ub41c\ub2e4.", "paragraph_sentence": "2003\ub144 \ud558\ud604\uc6b0, \uc804\uaddc\ud638, \uc774\uc815\uae38\uc740 \ubc34\ub4dc\uba85\uc744 The C.O.M(\ub354 \ucef4)\uc73c\ub85c \ubcc0\uacbd\ud55c\ub2e4. \ubc15\ucca0\uc601\uc744 \uc0c8\ub85c\uc6b4 \ubca0\uc774\uc2dc\uc2a4\ud2b8\ub85c \uc601\uc785\ud558\uace0 \ub77c\uc774\ube0c\ud074\ub7fd \uc2ac\ub7ec\uac70, \uc7ac\uba38\uc2a4 \ub4f1\uc9c0\uc5d0\uc11c \uacf5\uc5f0\uc744 \ud558\uba70 \ud65c\ub3d9\uc744 \uc2dc\uc791\ud55c\ub2e4. \uadf8\ub7ec\ub098 \uadf8\ud574 8\uc6d4, \uad70\uc785\ub300\ubb38\uc81c\ub85c \uc778\ud574 \ud300\uc740 \ud574\uccb4\ud558\uac8c \ub41c\ub2e4. 2003\ub144 10\uc6d4 5\uc77c \uc308\uc9c0 \uc0ac\uc6b4\ub4dc \ud398\uc2a4\ud2f0\ubc8c \uc228\uc740\uace0\uc218\uc5d0 \uc120\uc815\ub41c\ub2e4. \ud300\uc740 \uc774\ubbf8 \ud574\uccb4\ud55c \uc0c1\ud0dc\uc600\uc9c0\ub9cc \uc308\uc9c0 \uc0ac\uc6b4\ub4dc \ud398\uc2a4\ud2f0\ubc8c\uc744 \ub9c8\uc9c0\ub9c9\uc73c\ub85c \ubb34\ub300\uc5d0 \uc624\ub974\uac8c \ub41c\ub2e4. 2004\ub144 \ud558\ud604\uc6b0, \uc774\uc815\uae38\uc740 \uad70\uc785\ub300\ub97c \ud558\uc600\uace0 \uc804\uaddc\ud638\ub294 \uace0\ud5a5\uc778 \uac15\uc6d0\ub3c4\ub85c \ub3cc\uc544\uac04\ub2e4. 2006\ub144 4\uc6d4, \ud558\ud604\uc6b0\ub294 \uad70\uc81c\ub300\ub97c \ud558\uace0 \uc804\uaddc\ud638\ub97c \ub9cc\ub098 \ubc34\ub4dc\ub97c \ub2e4\uc2dc \ud558\uae30\ub85c\ud55c\ub2e4. \uac15\uc6d0\ub3c4\uc5d0 \uc788\ub294 \uc804\uaddc\ud638\uc758 \ubd80\ubaa8\ub2d8\uc774 \uc6b4\uc601\ud558\ub294 \ud39c\uc158\uc5d0\uc11c \ud568\uaed8 \uc0b4\uba74\uc11c 7\uac1c\uc6d4 \ub3d9\uc548 \uace1\uc744 \uc4f0\uac8c \ub41c\ub2e4.", "paragraph_answer": "2003\ub144 \ud558\ud604\uc6b0, \uc804\uaddc\ud638, \uc774\uc815\uae38\uc740 \ubc34\ub4dc\uba85\uc744 The C.O.M(\ub354 \ucef4)\uc73c\ub85c \ubcc0\uacbd\ud55c\ub2e4. \ubc15\ucca0\uc601\uc744 \uc0c8\ub85c\uc6b4 \ubca0\uc774\uc2dc\uc2a4\ud2b8\ub85c \uc601\uc785\ud558\uace0 \ub77c\uc774\ube0c\ud074\ub7fd \uc2ac\ub7ec\uac70, \uc7ac\uba38\uc2a4 \ub4f1\uc9c0\uc5d0\uc11c \uacf5\uc5f0\uc744 \ud558\uba70 \ud65c\ub3d9\uc744 \uc2dc\uc791\ud55c\ub2e4. \uadf8\ub7ec\ub098 \uadf8\ud574 8\uc6d4, \uad70\uc785\ub300\ubb38\uc81c\ub85c \uc778\ud574 \ud300\uc740 \ud574\uccb4\ud558\uac8c \ub41c\ub2e4. 2003\ub144 10\uc6d4 5\uc77c \uc308\uc9c0 \uc0ac\uc6b4\ub4dc \ud398\uc2a4\ud2f0\ubc8c \uc228\uc740\uace0\uc218\uc5d0 \uc120\uc815\ub41c\ub2e4. \ud300\uc740 \uc774\ubbf8 \ud574\uccb4\ud55c \uc0c1\ud0dc\uc600\uc9c0\ub9cc \uc308\uc9c0 \uc0ac\uc6b4\ub4dc \ud398\uc2a4\ud2f0\ubc8c\uc744 \ub9c8\uc9c0\ub9c9\uc73c\ub85c \ubb34\ub300\uc5d0 \uc624\ub974\uac8c \ub41c\ub2e4. 2004\ub144 \ud558\ud604\uc6b0, \uc774\uc815\uae38\uc740 \uad70\uc785\ub300\ub97c \ud558\uc600\uace0 \uc804\uaddc\ud638\ub294 \uace0\ud5a5\uc778 \uac15\uc6d0\ub3c4\ub85c \ub3cc\uc544\uac04\ub2e4. 2006\ub144 4\uc6d4, \ud558\ud604\uc6b0\ub294 \uad70\uc81c\ub300\ub97c \ud558\uace0 \uc804\uaddc\ud638\ub97c \ub9cc\ub098 \ubc34\ub4dc\ub97c \ub2e4\uc2dc \ud558\uae30\ub85c\ud55c\ub2e4. \uac15\uc6d0\ub3c4\uc5d0 \uc788\ub294 \uc804\uaddc\ud638\uc758 \ubd80\ubaa8\ub2d8\uc774 \uc6b4\uc601\ud558\ub294 \ud39c\uc158\uc5d0\uc11c \ud568\uaed8 \uc0b4\uba74\uc11c 7\uac1c\uc6d4 \ub3d9\uc548 \uace1\uc744 \uc4f0\uac8c \ub41c\ub2e4.", "sentence_answer": "2003\ub144 10\uc6d4 5\uc77c \uc308\uc9c0 \uc0ac\uc6b4\ub4dc \ud398\uc2a4\ud2f0\ubc8c \uc228\uc740\uace0\uc218\uc5d0 \uc120\uc815\ub41c\ub2e4.", "paragraph_id": "6477505-1-0", "paragraph_question": "question: 2003\ub144 10\uc6d4 5\uc77c \ub9c8\uc9c0\ub9c9\uc73c\ub85c \ubb34\ub300\ub97c \uc624\ub974\uac8c \ub41c \ud398\uc2a4\ud2f0\ubc8c \uc774\ub984\uc740?, context: 2003\ub144 \ud558\ud604\uc6b0, \uc804\uaddc\ud638, \uc774\uc815\uae38\uc740 \ubc34\ub4dc\uba85\uc744 The C.O.M(\ub354 \ucef4)\uc73c\ub85c \ubcc0\uacbd\ud55c\ub2e4. \ubc15\ucca0\uc601\uc744 \uc0c8\ub85c\uc6b4 \ubca0\uc774\uc2dc\uc2a4\ud2b8\ub85c \uc601\uc785\ud558\uace0 \ub77c\uc774\ube0c\ud074\ub7fd \uc2ac\ub7ec\uac70, \uc7ac\uba38\uc2a4 \ub4f1\uc9c0\uc5d0\uc11c \uacf5\uc5f0\uc744 \ud558\uba70 \ud65c\ub3d9\uc744 \uc2dc\uc791\ud55c\ub2e4. \uadf8\ub7ec\ub098 \uadf8\ud574 8\uc6d4, \uad70\uc785\ub300\ubb38\uc81c\ub85c \uc778\ud574 \ud300\uc740 \ud574\uccb4\ud558\uac8c \ub41c\ub2e4. 2003\ub144 10\uc6d4 5\uc77c \uc308\uc9c0 \uc0ac\uc6b4\ub4dc \ud398\uc2a4\ud2f0\ubc8c \uc228\uc740\uace0\uc218\uc5d0 \uc120\uc815\ub41c\ub2e4. \ud300\uc740 \uc774\ubbf8 \ud574\uccb4\ud55c \uc0c1\ud0dc\uc600\uc9c0\ub9cc \uc308\uc9c0 \uc0ac\uc6b4\ub4dc \ud398\uc2a4\ud2f0\ubc8c\uc744 \ub9c8\uc9c0\ub9c9\uc73c\ub85c \ubb34\ub300\uc5d0 \uc624\ub974\uac8c \ub41c\ub2e4. 2004\ub144 \ud558\ud604\uc6b0, \uc774\uc815\uae38\uc740 \uad70\uc785\ub300\ub97c \ud558\uc600\uace0 \uc804\uaddc\ud638\ub294 \uace0\ud5a5\uc778 \uac15\uc6d0\ub3c4\ub85c \ub3cc\uc544\uac04\ub2e4. 2006\ub144 4\uc6d4, \ud558\ud604\uc6b0\ub294 \uad70\uc81c\ub300\ub97c \ud558\uace0 \uc804\uaddc\ud638\ub97c \ub9cc\ub098 \ubc34\ub4dc\ub97c \ub2e4\uc2dc \ud558\uae30\ub85c\ud55c\ub2e4. \uac15\uc6d0\ub3c4\uc5d0 \uc788\ub294 \uc804\uaddc\ud638\uc758 \ubd80\ubaa8\ub2d8\uc774 \uc6b4\uc601\ud558\ub294 \ud39c\uc158\uc5d0\uc11c \ud568\uaed8 \uc0b4\uba74\uc11c 7\uac1c\uc6d4 \ub3d9\uc548 \uace1\uc744 \uc4f0\uac8c \ub41c\ub2e4."} {"question": "\uc138\ub4dc\ub098\uc758 \ud45c\uba74\uc5d0\uc11c \ubcf4\uc774\ub294 \ud0dc\uc591\uc758 \ubc1d\uae30\ub294 \uc9c0\uad6c\uc5d0\uc11c \ubcf4\ub294 \ubcf4\ub984\ub2ec\uc758 \ubc1d\uae30\uc758 \uba87 \ubc30\uc778\uac00?", "paragraph": "\uc138\ub4dc\ub098\ub294 \uc9c0\uae08\uae4c\uc9c0 \ud0dc\uc591\uacc4\uc5d0\uc11c \ubc1c\uacac\ub41c \ub300\ud615 \ucc9c\uccb4 \uc911 \uacf5\uc804 \uc8fc\uae30\uac00 \uac00\uc7a5 \uae38\uc5b4, 11,400 \ub144 \uc804\ud6c4\ub85c \uacc4\uc0b0\ub41c\ub2e4. \uadf8 \uada4\ub3c4\ub294 \uc774\uc2ec\ub960\uc774 \uadf9\ub3c4\ub85c \ucee4\uc11c \uc6d0\uc77c\uc810 \uac70\ub9ac\uac00 \uc57d 937 AU\uc774\uace0 \uadfc\uc77c\uc810 \uac70\ub9ac\uac00 \uc57d 76 AU \uc774\ub2e4. \uc774 \uc6d0\uc77c\uc810 \uac70\ub9ac\ub294 2012\ub144 2012 VP113\uc774 \ubc1c\uacac\ub418\uae30 \uc804\uae4c\uc9c0 \ud0dc\uc591\uacc4 \ucc9c\uccb4 \uc911 \uac00\uc7a5 \uba3c \uac70\ub9ac\uc600\ub2e4. \uc138\ub4dc\ub098\uac00 \ubc1c\uacac\ub418\uc5c8\uc744 \ub2f9\uc2dc \ud0dc\uc591\uc5d0\uc11c\uc758 \uac70\ub9ac\ub294 89.6 AU\uc600\uc73c\uba70 \uadfc\uc77c\uc810\uc744 \ud5a5\ud574 \uc6c0\uc9c1\uc774\uace0 \uc788\uc5c8\ub2e4. \uc774\ud6c4 \uac19\uc740 \ud0d0\uc0ac \uacfc\uc815\uc5d0\uc11c \ud0dc\uc591\uc73c\ub85c\ubd80\ud130 97 AU \ub5a8\uc5b4\uc9c4 \uadfc\uc77c\uc810 \uadfc\ucc98\uc5d0\uc11c \uc5d0\ub9ac\uc2a4\uac00 \uac10\uc9c0\ub418\uc5c8\ub2e4. \uc138\ub4dc\ub098\ubcf4\ub2e4 \uada4\ub3c4\uac00 \ub354 \uba3c \ucc9c\uccb4\ub294 \uc77c\ubd80 \uc7a5\uc8fc\uae30 \ud61c\uc131\ub4e4\ubc16\uc5d0 \uc5c6\ub2e4. \uadf8\ub9ac\uace0 \ud61c\uc131\ub4e4\uc740 \ub108\ubb34 \uc5b4\ub450\uc6cc\uc11c \ub0b4\ubd80 \ud0dc\uc591\uacc4\ub85c \ub4e4\uc5b4\uc640 \uadfc\uc77c\uc810\uc744 \ud5a5\ud560 \ub54c\uac00 \uc544\ub2c8\uba74 \ubcf4\uc774\uc9c0\ub3c4 \uc54a\ub294\ub2e4. \uc138\ub4dc\ub098\ub3c4 2076\ub144 \uc911\uc21c\uc5d0 \uadfc\uc77c\uc810\uc5d0 \ub2e4\ub2e4\ub974\uaca0\uc9c0\ub9cc, \ub108\ubb34 \uba40\uc5b4\uc11c \uc721\uc548\uc73c\ub85c\ub294 \uc6d0\ubc18\uc0c1\uc744 \ubcfc \uc218 \uc5c6\uc744 \uac83\uc774\ub2e4. \uc138\ub4dc\ub098 \ud45c\uba74\uc5d0\uc11c \ud558\ub298\uc744 \ubcf4\uba74 \ud0dc\uc591\uc774 \uadf8\uc800 \ub9e4\uc6b0 \ubc1d\uc740(\ubcf4\ub984\ub2ec\uc758 100\ubc30 \ubc1d\uae30) \ud56d\uc131 \uc911 \ud558\ub098\ub85c \ubcf4\uc77c \uac83\uc774\ub2e4. \uc9c0\uad6c\uc5d0\uc11c \ubcf4\uc774\ub294 \ud0dc\uc591\uc758 \ubc1d\uae30\ub294 \ubcf4\ub984\ub2ec\uc758 400,000 \ubc30 \ubc1d\uae30\uc774\ubbc0\ub85c \uc774\ub97c \ub2a5\ud788 \ube44\uad50\ud560 \uc218 \uc788\ub2e4.", "answer": "100\ubc30", "sentence": "\uc138\ub4dc\ub098 \ud45c\uba74\uc5d0\uc11c \ud558\ub298\uc744 \ubcf4\uba74 \ud0dc\uc591\uc774 \uadf8\uc800 \ub9e4\uc6b0 \ubc1d\uc740(\ubcf4\ub984\ub2ec\uc758 100\ubc30 \ubc1d\uae30)", "paragraph_sentence": "\uc138\ub4dc\ub098\ub294 \uc9c0\uae08\uae4c\uc9c0 \ud0dc\uc591\uacc4\uc5d0\uc11c \ubc1c\uacac\ub41c \ub300\ud615 \ucc9c\uccb4 \uc911 \uacf5\uc804 \uc8fc\uae30\uac00 \uac00\uc7a5 \uae38\uc5b4, 11,400 \ub144 \uc804\ud6c4\ub85c \uacc4\uc0b0\ub41c\ub2e4. \uadf8 \uada4\ub3c4\ub294 \uc774\uc2ec\ub960\uc774 \uadf9\ub3c4\ub85c \ucee4\uc11c \uc6d0\uc77c\uc810 \uac70\ub9ac\uac00 \uc57d 937 AU\uc774\uace0 \uadfc\uc77c\uc810 \uac70\ub9ac\uac00 \uc57d 76 AU \uc774\ub2e4. \uc774 \uc6d0\uc77c\uc810 \uac70\ub9ac\ub294 2012\ub144 2012 VP113\uc774 \ubc1c\uacac\ub418\uae30 \uc804\uae4c\uc9c0 \ud0dc\uc591\uacc4 \ucc9c\uccb4 \uc911 \uac00\uc7a5 \uba3c \uac70\ub9ac\uc600\ub2e4. \uc138\ub4dc\ub098\uac00 \ubc1c\uacac\ub418\uc5c8\uc744 \ub2f9\uc2dc \ud0dc\uc591\uc5d0\uc11c\uc758 \uac70\ub9ac\ub294 89.6 AU\uc600\uc73c\uba70 \uadfc\uc77c\uc810\uc744 \ud5a5\ud574 \uc6c0\uc9c1\uc774\uace0 \uc788\uc5c8\ub2e4. \uc774\ud6c4 \uac19\uc740 \ud0d0\uc0ac \uacfc\uc815\uc5d0\uc11c \ud0dc\uc591\uc73c\ub85c\ubd80\ud130 97 AU \ub5a8\uc5b4\uc9c4 \uadfc\uc77c\uc810 \uadfc\ucc98\uc5d0\uc11c \uc5d0\ub9ac\uc2a4\uac00 \uac10\uc9c0\ub418\uc5c8\ub2e4. \uc138\ub4dc\ub098\ubcf4\ub2e4 \uada4\ub3c4\uac00 \ub354 \uba3c \ucc9c\uccb4\ub294 \uc77c\ubd80 \uc7a5\uc8fc\uae30 \ud61c\uc131\ub4e4\ubc16\uc5d0 \uc5c6\ub2e4. \uadf8\ub9ac\uace0 \ud61c\uc131\ub4e4\uc740 \ub108\ubb34 \uc5b4\ub450\uc6cc\uc11c \ub0b4\ubd80 \ud0dc\uc591\uacc4\ub85c \ub4e4\uc5b4\uc640 \uadfc\uc77c\uc810\uc744 \ud5a5\ud560 \ub54c\uac00 \uc544\ub2c8\uba74 \ubcf4\uc774\uc9c0\ub3c4 \uc54a\ub294\ub2e4. \uc138\ub4dc\ub098\ub3c4 2076\ub144 \uc911\uc21c\uc5d0 \uadfc\uc77c\uc810\uc5d0 \ub2e4\ub2e4\ub974\uaca0\uc9c0\ub9cc, \ub108\ubb34 \uba40\uc5b4\uc11c \uc721\uc548\uc73c\ub85c\ub294 \uc6d0\ubc18\uc0c1\uc744 \ubcfc \uc218 \uc5c6\uc744 \uac83\uc774\ub2e4. \uc138\ub4dc\ub098 \ud45c\uba74\uc5d0\uc11c \ud558\ub298\uc744 \ubcf4\uba74 \ud0dc\uc591\uc774 \uadf8\uc800 \ub9e4\uc6b0 \ubc1d\uc740(\ubcf4\ub984\ub2ec\uc758 100\ubc30 \ubc1d\uae30) \ud56d\uc131 \uc911 \ud558\ub098\ub85c \ubcf4\uc77c \uac83\uc774\ub2e4. \uc9c0\uad6c\uc5d0\uc11c \ubcf4\uc774\ub294 \ud0dc\uc591\uc758 \ubc1d\uae30\ub294 \ubcf4\ub984\ub2ec\uc758 400,000 \ubc30 \ubc1d\uae30\uc774\ubbc0\ub85c \uc774\ub97c \ub2a5\ud788 \ube44\uad50\ud560 \uc218 \uc788\ub2e4.", "paragraph_answer": "\uc138\ub4dc\ub098\ub294 \uc9c0\uae08\uae4c\uc9c0 \ud0dc\uc591\uacc4\uc5d0\uc11c \ubc1c\uacac\ub41c \ub300\ud615 \ucc9c\uccb4 \uc911 \uacf5\uc804 \uc8fc\uae30\uac00 \uac00\uc7a5 \uae38\uc5b4, 11,400 \ub144 \uc804\ud6c4\ub85c \uacc4\uc0b0\ub41c\ub2e4. \uadf8 \uada4\ub3c4\ub294 \uc774\uc2ec\ub960\uc774 \uadf9\ub3c4\ub85c \ucee4\uc11c \uc6d0\uc77c\uc810 \uac70\ub9ac\uac00 \uc57d 937 AU\uc774\uace0 \uadfc\uc77c\uc810 \uac70\ub9ac\uac00 \uc57d 76 AU \uc774\ub2e4. \uc774 \uc6d0\uc77c\uc810 \uac70\ub9ac\ub294 2012\ub144 2012 VP113\uc774 \ubc1c\uacac\ub418\uae30 \uc804\uae4c\uc9c0 \ud0dc\uc591\uacc4 \ucc9c\uccb4 \uc911 \uac00\uc7a5 \uba3c \uac70\ub9ac\uc600\ub2e4. \uc138\ub4dc\ub098\uac00 \ubc1c\uacac\ub418\uc5c8\uc744 \ub2f9\uc2dc \ud0dc\uc591\uc5d0\uc11c\uc758 \uac70\ub9ac\ub294 89.6 AU\uc600\uc73c\uba70 \uadfc\uc77c\uc810\uc744 \ud5a5\ud574 \uc6c0\uc9c1\uc774\uace0 \uc788\uc5c8\ub2e4. \uc774\ud6c4 \uac19\uc740 \ud0d0\uc0ac \uacfc\uc815\uc5d0\uc11c \ud0dc\uc591\uc73c\ub85c\ubd80\ud130 97 AU \ub5a8\uc5b4\uc9c4 \uadfc\uc77c\uc810 \uadfc\ucc98\uc5d0\uc11c \uc5d0\ub9ac\uc2a4\uac00 \uac10\uc9c0\ub418\uc5c8\ub2e4. \uc138\ub4dc\ub098\ubcf4\ub2e4 \uada4\ub3c4\uac00 \ub354 \uba3c \ucc9c\uccb4\ub294 \uc77c\ubd80 \uc7a5\uc8fc\uae30 \ud61c\uc131\ub4e4\ubc16\uc5d0 \uc5c6\ub2e4. \uadf8\ub9ac\uace0 \ud61c\uc131\ub4e4\uc740 \ub108\ubb34 \uc5b4\ub450\uc6cc\uc11c \ub0b4\ubd80 \ud0dc\uc591\uacc4\ub85c \ub4e4\uc5b4\uc640 \uadfc\uc77c\uc810\uc744 \ud5a5\ud560 \ub54c\uac00 \uc544\ub2c8\uba74 \ubcf4\uc774\uc9c0\ub3c4 \uc54a\ub294\ub2e4. \uc138\ub4dc\ub098\ub3c4 2076\ub144 \uc911\uc21c\uc5d0 \uadfc\uc77c\uc810\uc5d0 \ub2e4\ub2e4\ub974\uaca0\uc9c0\ub9cc, \ub108\ubb34 \uba40\uc5b4\uc11c \uc721\uc548\uc73c\ub85c\ub294 \uc6d0\ubc18\uc0c1\uc744 \ubcfc \uc218 \uc5c6\uc744 \uac83\uc774\ub2e4. \uc138\ub4dc\ub098 \ud45c\uba74\uc5d0\uc11c \ud558\ub298\uc744 \ubcf4\uba74 \ud0dc\uc591\uc774 \uadf8\uc800 \ub9e4\uc6b0 \ubc1d\uc740(\ubcf4\ub984\ub2ec\uc758 100\ubc30 \ubc1d\uae30) \ud56d\uc131 \uc911 \ud558\ub098\ub85c \ubcf4\uc77c \uac83\uc774\ub2e4. \uc9c0\uad6c\uc5d0\uc11c \ubcf4\uc774\ub294 \ud0dc\uc591\uc758 \ubc1d\uae30\ub294 \ubcf4\ub984\ub2ec\uc758 400,000 \ubc30 \ubc1d\uae30\uc774\ubbc0\ub85c \uc774\ub97c \ub2a5\ud788 \ube44\uad50\ud560 \uc218 \uc788\ub2e4.", "sentence_answer": "\uc138\ub4dc\ub098 \ud45c\uba74\uc5d0\uc11c \ud558\ub298\uc744 \ubcf4\uba74 \ud0dc\uc591\uc774 \uadf8\uc800 \ub9e4\uc6b0 \ubc1d\uc740(\ubcf4\ub984\ub2ec\uc758 100\ubc30 \ubc1d\uae30)", "paragraph_id": "6568339-3-0", "paragraph_question": "question: \uc138\ub4dc\ub098\uc758 \ud45c\uba74\uc5d0\uc11c \ubcf4\uc774\ub294 \ud0dc\uc591\uc758 \ubc1d\uae30\ub294 \uc9c0\uad6c\uc5d0\uc11c \ubcf4\ub294 \ubcf4\ub984\ub2ec\uc758 \ubc1d\uae30\uc758 \uba87 \ubc30\uc778\uac00?, context: \uc138\ub4dc\ub098\ub294 \uc9c0\uae08\uae4c\uc9c0 \ud0dc\uc591\uacc4\uc5d0\uc11c \ubc1c\uacac\ub41c \ub300\ud615 \ucc9c\uccb4 \uc911 \uacf5\uc804 \uc8fc\uae30\uac00 \uac00\uc7a5 \uae38\uc5b4, 11,400 \ub144 \uc804\ud6c4\ub85c \uacc4\uc0b0\ub41c\ub2e4. \uadf8 \uada4\ub3c4\ub294 \uc774\uc2ec\ub960\uc774 \uadf9\ub3c4\ub85c \ucee4\uc11c \uc6d0\uc77c\uc810 \uac70\ub9ac\uac00 \uc57d 937 AU\uc774\uace0 \uadfc\uc77c\uc810 \uac70\ub9ac\uac00 \uc57d 76 AU \uc774\ub2e4. \uc774 \uc6d0\uc77c\uc810 \uac70\ub9ac\ub294 2012\ub144 2012 VP113\uc774 \ubc1c\uacac\ub418\uae30 \uc804\uae4c\uc9c0 \ud0dc\uc591\uacc4 \ucc9c\uccb4 \uc911 \uac00\uc7a5 \uba3c \uac70\ub9ac\uc600\ub2e4. \uc138\ub4dc\ub098\uac00 \ubc1c\uacac\ub418\uc5c8\uc744 \ub2f9\uc2dc \ud0dc\uc591\uc5d0\uc11c\uc758 \uac70\ub9ac\ub294 89.6 AU\uc600\uc73c\uba70 \uadfc\uc77c\uc810\uc744 \ud5a5\ud574 \uc6c0\uc9c1\uc774\uace0 \uc788\uc5c8\ub2e4. \uc774\ud6c4 \uac19\uc740 \ud0d0\uc0ac \uacfc\uc815\uc5d0\uc11c \ud0dc\uc591\uc73c\ub85c\ubd80\ud130 97 AU \ub5a8\uc5b4\uc9c4 \uadfc\uc77c\uc810 \uadfc\ucc98\uc5d0\uc11c \uc5d0\ub9ac\uc2a4\uac00 \uac10\uc9c0\ub418\uc5c8\ub2e4. \uc138\ub4dc\ub098\ubcf4\ub2e4 \uada4\ub3c4\uac00 \ub354 \uba3c \ucc9c\uccb4\ub294 \uc77c\ubd80 \uc7a5\uc8fc\uae30 \ud61c\uc131\ub4e4\ubc16\uc5d0 \uc5c6\ub2e4. \uadf8\ub9ac\uace0 \ud61c\uc131\ub4e4\uc740 \ub108\ubb34 \uc5b4\ub450\uc6cc\uc11c \ub0b4\ubd80 \ud0dc\uc591\uacc4\ub85c \ub4e4\uc5b4\uc640 \uadfc\uc77c\uc810\uc744 \ud5a5\ud560 \ub54c\uac00 \uc544\ub2c8\uba74 \ubcf4\uc774\uc9c0\ub3c4 \uc54a\ub294\ub2e4. \uc138\ub4dc\ub098\ub3c4 2076\ub144 \uc911\uc21c\uc5d0 \uadfc\uc77c\uc810\uc5d0 \ub2e4\ub2e4\ub974\uaca0\uc9c0\ub9cc, \ub108\ubb34 \uba40\uc5b4\uc11c \uc721\uc548\uc73c\ub85c\ub294 \uc6d0\ubc18\uc0c1\uc744 \ubcfc \uc218 \uc5c6\uc744 \uac83\uc774\ub2e4. \uc138\ub4dc\ub098 \ud45c\uba74\uc5d0\uc11c \ud558\ub298\uc744 \ubcf4\uba74 \ud0dc\uc591\uc774 \uadf8\uc800 \ub9e4\uc6b0 \ubc1d\uc740(\ubcf4\ub984\ub2ec\uc758 100\ubc30 \ubc1d\uae30) \ud56d\uc131 \uc911 \ud558\ub098\ub85c \ubcf4\uc77c \uac83\uc774\ub2e4. \uc9c0\uad6c\uc5d0\uc11c \ubcf4\uc774\ub294 \ud0dc\uc591\uc758 \ubc1d\uae30\ub294 \ubcf4\ub984\ub2ec\uc758 400,000 \ubc30 \ubc1d\uae30\uc774\ubbc0\ub85c \uc774\ub97c \ub2a5\ud788 \ube44\uad50\ud560 \uc218 \uc788\ub2e4."} {"question": "The Boys \ubc34\ub4dc\ubc84\uc804\uc744 \ubd80\ub978 \uacf3\uc740?", "paragraph": "2012\ub144 1\uc6d4 31\uc77c\uc5d0\ub294 \uc18c\ub140\uc2dc\ub300\uac00 \ucc98\uc74c\uc73c\ub85c \ubbf8\uad6d \uc9c0\uc0c1\ud30c CBS\uc758 \u300a\ub370\uc774\ube44\ub4dc \ub808\ud130\ub9e8\uc1fc\u300b\uc5d0 \ucd9c\uc5f0\ud574 \"The Boys\" \ubc34\ub4dc \ubc84\uc804\uc73c\ub85c \ubd88\ub800\uace0, \uc88b\uc740 \ubc18\uc751\uc744 \uc5bb\uc5c8\ub2e4. \uc774\ud6c4 2\uc6d4 1\uc77c\uc5d0\ub294 \ub610\ub2e4\ub978 \uc9c0\uc0c1\ud30c ABC\uc758 \u300a\ub77c\uc774\ube0c! \uc704\ub4dc \ucf08\ub9ac \uc564\ub4dc \ub9c8\uc774\ud074\u300b\uc5d0\ub3c4 \ucd9c\uc5f0\ud574 \uacf5\uc5f0\ud588\ub2e4. 2012\ub144 2\uc6d4\uc5d0\ub294 \ud504\ub791\uc2a4 \ucc44\ub110 'CANAL plus'\uc758 \ud1a0\ud06c\uc1fc \u300a\ub974 \uadf8\ub791 \uc8fc\ub974\ub0a0\u300b\uc5d0 \uc720\ub7fd \ubc29\uc1a1\uc5d0\uc11c \ucd5c\ucd08\ub85c \"The Boys\" \uacf5\uc5f0\uc744 \ud588\uace0, \uba54\uc778\ub274\uc2a4 'Le Journal De 20H'\uc5d0\uc11c \uc778\ud130\ubdf0\ub3c4 \ud588\ub2e4. \ub610\ud55c 2012\ub144 8\uc6d4 18\uc77c \ubc29\uc601\ub41c \ud504\ub791\uc2a4 \ud30c\ub9ac \ubca0\uc528 \uc2a4\ud0c0\ub514\uc6c0\uc5d0\uc11c \uc5f4\ub9b0 \u300a\ubba4\uc9c1\ubc45\ud06c \uc778 \ud30c\ub9ac\u300b\uc5d0 \ucd9c\uc5f0\ud574 \uc5ec\ub7ec \ud788\ud2b8\uace1\ub4e4\uacfc \"The Boys\", \"Mr. Taxi\"\ub97c \uacf5\uc5f0\ud588\ub2e4.", "answer": "\ub370\uc774\ube44\ub4dc \ub808\ud130\ub9e8\uc1fc", "sentence": "2012\ub144 1\uc6d4 31\uc77c\uc5d0\ub294 \uc18c\ub140\uc2dc\ub300\uac00 \ucc98\uc74c\uc73c\ub85c \ubbf8\uad6d \uc9c0\uc0c1\ud30c CBS\uc758 \u300a \ub370\uc774\ube44\ub4dc \ub808\ud130\ub9e8\uc1fc \u300b\uc5d0 \ucd9c\uc5f0\ud574 \"The Boys\" \ubc34\ub4dc \ubc84\uc804\uc73c\ub85c \ubd88\ub800\uace0, \uc88b\uc740 \ubc18\uc751\uc744 \uc5bb\uc5c8\ub2e4.", "paragraph_sentence": " 2012\ub144 1\uc6d4 31\uc77c\uc5d0\ub294 \uc18c\ub140\uc2dc\ub300\uac00 \ucc98\uc74c\uc73c\ub85c \ubbf8\uad6d \uc9c0\uc0c1\ud30c CBS\uc758 \u300a \ub370\uc774\ube44\ub4dc \ub808\ud130\ub9e8\uc1fc \u300b\uc5d0 \ucd9c\uc5f0\ud574 \"The Boys\" \ubc34\ub4dc \ubc84\uc804\uc73c\ub85c \ubd88\ub800\uace0, \uc88b\uc740 \ubc18\uc751\uc744 \uc5bb\uc5c8\ub2e4. \uc774\ud6c4 2\uc6d4 1\uc77c\uc5d0\ub294 \ub610\ub2e4\ub978 \uc9c0\uc0c1\ud30c ABC\uc758 \u300a\ub77c\uc774\ube0c! \uc704\ub4dc \ucf08\ub9ac \uc564\ub4dc \ub9c8\uc774\ud074\u300b\uc5d0\ub3c4 \ucd9c\uc5f0\ud574 \uacf5\uc5f0\ud588\ub2e4. 2012\ub144 2\uc6d4\uc5d0\ub294 \ud504\ub791\uc2a4 \ucc44\ub110 'CANAL plus'\uc758 \ud1a0\ud06c\uc1fc \u300a\ub974 \uadf8\ub791 \uc8fc\ub974\ub0a0\u300b\uc5d0 \uc720\ub7fd \ubc29\uc1a1\uc5d0\uc11c \ucd5c\ucd08\ub85c \"The Boys\" \uacf5\uc5f0\uc744 \ud588\uace0, \uba54\uc778\ub274\uc2a4 'Le Journal De 20H'\uc5d0\uc11c \uc778\ud130\ubdf0\ub3c4 \ud588\ub2e4. \ub610\ud55c 2012\ub144 8\uc6d4 18\uc77c \ubc29\uc601\ub41c \ud504\ub791\uc2a4 \ud30c\ub9ac \ubca0\uc528 \uc2a4\ud0c0\ub514\uc6c0\uc5d0\uc11c \uc5f4\ub9b0 \u300a\ubba4\uc9c1\ubc45\ud06c \uc778 \ud30c\ub9ac\u300b\uc5d0 \ucd9c\uc5f0\ud574 \uc5ec\ub7ec \ud788\ud2b8\uace1\ub4e4\uacfc \"The Boys\", \"Mr. Taxi\"\ub97c \uacf5\uc5f0\ud588\ub2e4.", "paragraph_answer": "2012\ub144 1\uc6d4 31\uc77c\uc5d0\ub294 \uc18c\ub140\uc2dc\ub300\uac00 \ucc98\uc74c\uc73c\ub85c \ubbf8\uad6d \uc9c0\uc0c1\ud30c CBS\uc758 \u300a \ub370\uc774\ube44\ub4dc \ub808\ud130\ub9e8\uc1fc \u300b\uc5d0 \ucd9c\uc5f0\ud574 \"The Boys\" \ubc34\ub4dc \ubc84\uc804\uc73c\ub85c \ubd88\ub800\uace0, \uc88b\uc740 \ubc18\uc751\uc744 \uc5bb\uc5c8\ub2e4. \uc774\ud6c4 2\uc6d4 1\uc77c\uc5d0\ub294 \ub610\ub2e4\ub978 \uc9c0\uc0c1\ud30c ABC\uc758 \u300a\ub77c\uc774\ube0c! \uc704\ub4dc \ucf08\ub9ac \uc564\ub4dc \ub9c8\uc774\ud074\u300b\uc5d0\ub3c4 \ucd9c\uc5f0\ud574 \uacf5\uc5f0\ud588\ub2e4. 2012\ub144 2\uc6d4\uc5d0\ub294 \ud504\ub791\uc2a4 \ucc44\ub110 'CANAL plus'\uc758 \ud1a0\ud06c\uc1fc \u300a\ub974 \uadf8\ub791 \uc8fc\ub974\ub0a0\u300b\uc5d0 \uc720\ub7fd \ubc29\uc1a1\uc5d0\uc11c \ucd5c\ucd08\ub85c \"The Boys\" \uacf5\uc5f0\uc744 \ud588\uace0, \uba54\uc778\ub274\uc2a4 'Le Journal De 20H'\uc5d0\uc11c \uc778\ud130\ubdf0\ub3c4 \ud588\ub2e4. \ub610\ud55c 2012\ub144 8\uc6d4 18\uc77c \ubc29\uc601\ub41c \ud504\ub791\uc2a4 \ud30c\ub9ac \ubca0\uc528 \uc2a4\ud0c0\ub514\uc6c0\uc5d0\uc11c \uc5f4\ub9b0 \u300a\ubba4\uc9c1\ubc45\ud06c \uc778 \ud30c\ub9ac\u300b\uc5d0 \ucd9c\uc5f0\ud574 \uc5ec\ub7ec \ud788\ud2b8\uace1\ub4e4\uacfc \"The Boys\", \"Mr. Taxi\"\ub97c \uacf5\uc5f0\ud588\ub2e4.", "sentence_answer": "2012\ub144 1\uc6d4 31\uc77c\uc5d0\ub294 \uc18c\ub140\uc2dc\ub300\uac00 \ucc98\uc74c\uc73c\ub85c \ubbf8\uad6d \uc9c0\uc0c1\ud30c CBS\uc758 \u300a \ub370\uc774\ube44\ub4dc \ub808\ud130\ub9e8\uc1fc \u300b\uc5d0 \ucd9c\uc5f0\ud574 \"The Boys\" \ubc34\ub4dc \ubc84\uc804\uc73c\ub85c \ubd88\ub800\uace0, \uc88b\uc740 \ubc18\uc751\uc744 \uc5bb\uc5c8\ub2e4.", "paragraph_id": "6042513-8-0", "paragraph_question": "question: The Boys \ubc34\ub4dc\ubc84\uc804\uc744 \ubd80\ub978 \uacf3\uc740?, context: 2012\ub144 1\uc6d4 31\uc77c\uc5d0\ub294 \uc18c\ub140\uc2dc\ub300\uac00 \ucc98\uc74c\uc73c\ub85c \ubbf8\uad6d \uc9c0\uc0c1\ud30c CBS\uc758 \u300a\ub370\uc774\ube44\ub4dc \ub808\ud130\ub9e8\uc1fc\u300b\uc5d0 \ucd9c\uc5f0\ud574 \"The Boys\" \ubc34\ub4dc \ubc84\uc804\uc73c\ub85c \ubd88\ub800\uace0, \uc88b\uc740 \ubc18\uc751\uc744 \uc5bb\uc5c8\ub2e4. \uc774\ud6c4 2\uc6d4 1\uc77c\uc5d0\ub294 \ub610\ub2e4\ub978 \uc9c0\uc0c1\ud30c ABC\uc758 \u300a\ub77c\uc774\ube0c! \uc704\ub4dc \ucf08\ub9ac \uc564\ub4dc \ub9c8\uc774\ud074\u300b\uc5d0\ub3c4 \ucd9c\uc5f0\ud574 \uacf5\uc5f0\ud588\ub2e4. 2012\ub144 2\uc6d4\uc5d0\ub294 \ud504\ub791\uc2a4 \ucc44\ub110 'CANAL plus'\uc758 \ud1a0\ud06c\uc1fc \u300a\ub974 \uadf8\ub791 \uc8fc\ub974\ub0a0\u300b\uc5d0 \uc720\ub7fd \ubc29\uc1a1\uc5d0\uc11c \ucd5c\ucd08\ub85c \"The Boys\" \uacf5\uc5f0\uc744 \ud588\uace0, \uba54\uc778\ub274\uc2a4 'Le Journal De 20H'\uc5d0\uc11c \uc778\ud130\ubdf0\ub3c4 \ud588\ub2e4. \ub610\ud55c 2012\ub144 8\uc6d4 18\uc77c \ubc29\uc601\ub41c \ud504\ub791\uc2a4 \ud30c\ub9ac \ubca0\uc528 \uc2a4\ud0c0\ub514\uc6c0\uc5d0\uc11c \uc5f4\ub9b0 \u300a\ubba4\uc9c1\ubc45\ud06c \uc778 \ud30c\ub9ac\u300b\uc5d0 \ucd9c\uc5f0\ud574 \uc5ec\ub7ec \ud788\ud2b8\uace1\ub4e4\uacfc \"The Boys\", \"Mr. Taxi\"\ub97c \uacf5\uc5f0\ud588\ub2e4."} {"question": "\uc218\uc9c1\uc73c\ub85c \ub0b4\ub9ac\ucb10\ub294 \ud587\ube5b\uc73c\ub85c \uc5bc\uc74c\uc774 \ub179\uc544\uc11c \uc0dd\uae34\uac83\uc744 \ubb34\uc5c7\uc774\ub77c\uace0 \ubd80\ub974\ub294\uac00?", "paragraph": "\uc720\ub85c\ud30c\ub294 \uc0b0\uc774\ub098 \ucda9\ub3cc\uad6c\uac00 \uc801\uc5b4 \ud0dc\uc591\uacc4\uc5d0\uc11c \uac00\uc7a5 \ub9e4\ub044\ub7ec\uc6b4 \ucc9c\uccb4 \uc911 \ud558\ub098\uc774\ub2e4. \uadf8\ub7ec\ub098 \uc720\ub85c\ud30c\uc758 \uc801\ub3c4\uc5d0 10m\uc758 \ub192\uc774\ub97c \uac00\uc9c4 \uc5bc\uc74c \uc2a4\ud30c\uc774\ud06c\uac00 \uc788\ub294\ub370, \uc218\uc9c1\uc73c\ub85c \ub0b4\ub9ac\ucb10\ub294 \ud587\ube5b\uc758 \uc601\ud5a5\uc73c\ub85c \uc5bc\uc74c\uc774 \ub179\uc544\uc11c \uade0\uc5f4\uc774 \uc0dd\uae34 \uac83\uc774\ub77c\uace0 \ucd94\uce21\ub41c\ub2e4. \uc774\ub294 \ud398\ub2c8\ud150\ud2b8\ub77c \ubd88\ub9b0\ub2e4. \uc720\ub85c\ud30c\uc758 \uc2ed\uc790 \uc9c0\ud615\uc740 \uc54c\ubca0\ub3c4 \uc9c0\ud615\uc778 \uac83\uc73c\ub85c \ubcf4\uc5ec\uc9c0\uace0, \uc774\ub294 \ud398\ub2c8\ud150\ud2b8\uac00 \ub0ae\uc740 \uacf3\uc5d0 \uc788\uc74c\uc744 \uac00\ub9ac\ud0a8\ub2e4. \ud45c\uba74\uc5d0 \uc788\ub294 \ucda9\ub3cc\uad6c\ub4e4\uc740 \uc0c1\ub300\uc801\uc73c\ub85c \uc80a\uc740\ub370, \ud45c\uba74\uc774 \ud310 \uad6c\uc870\ub860\uacfc \uac19\uc740 \uc6d0\ub9ac\ub85c \ub04a\uc784\uc5c6\uc774 \ud65c\ub3d9\ud558\uae30 \ub54c\ubb38\uc774\ub77c\uace0 \ucd94\uce21\ub418\uace0 \uc788\ub2e4. \uc720\ub85c\ud30c\uc758 \uc5bc\uc74c \uc9c0\uac01\uc758 \ubc18\uc0ac\uc728\uc740 0.64\uc774\ub2e4. \uc720\ub85c\ud30c\ub294 \ud0dc\uc591\uacc4\uc5d0\uc11c \ubc18\uc0ac\uc728\uc774 \ub192\uc740 \ucc9c\uccb4\ub4e4 \uc911 \ud558\ub098\uc774\ub2e4. \ud61c\uc131\uc774 \uc720\ub85c\ud30c\uc5d0 \ucda9\ub3cc\ud560 \ub54c \ubc1c\uc0dd\ud558\ub294 \ucda9\ub3cc \uc8fc\ud30c\uc218\ub85c \ub098\uc774\ub97c \uacc4\uc0b0\ud558\uba74, \ud45c\uba74\uc740 \uc57d 20\ub9cc~1\uc5b5 8\ucc9c\ub9cc \ub144 \uc815\ub3c4 \ub418\uc5c8\ub2e4\ub294 \uacc4\uc0b0\uc774 \ub098\uc628\ub2e4. \uc720\ub85c\ud30c\uc758 \ud45c\uba74 \ud65c\ub3d9\uc5d0 \uad00\ud55c \uc774\ub860\uc740 \ub54c\ub85c\ub294 \ubaa8\uc21c\ub418\uae30\ub3c4 \ud558\uba70, \ud1b5\uc77c\ub41c \uacac\ud574\ub098 \uc774\ub860\uc740 \uc5c6\ub2e4.", "answer": "\ud398\ub2c8\ud150\ud2b8", "sentence": "\uc774\ub294 \ud398\ub2c8\ud150\ud2b8 \ub77c \ubd88\ub9b0\ub2e4.", "paragraph_sentence": "\uc720\ub85c\ud30c\ub294 \uc0b0\uc774\ub098 \ucda9\ub3cc\uad6c\uac00 \uc801\uc5b4 \ud0dc\uc591\uacc4\uc5d0\uc11c \uac00\uc7a5 \ub9e4\ub044\ub7ec\uc6b4 \ucc9c\uccb4 \uc911 \ud558\ub098\uc774\ub2e4. \uadf8\ub7ec\ub098 \uc720\ub85c\ud30c\uc758 \uc801\ub3c4\uc5d0 10m\uc758 \ub192\uc774\ub97c \uac00\uc9c4 \uc5bc\uc74c \uc2a4\ud30c\uc774\ud06c\uac00 \uc788\ub294\ub370, \uc218\uc9c1\uc73c\ub85c \ub0b4\ub9ac\ucb10\ub294 \ud587\ube5b\uc758 \uc601\ud5a5\uc73c\ub85c \uc5bc\uc74c\uc774 \ub179\uc544\uc11c \uade0\uc5f4\uc774 \uc0dd\uae34 \uac83\uc774\ub77c\uace0 \ucd94\uce21\ub41c\ub2e4. \uc774\ub294 \ud398\ub2c8\ud150\ud2b8 \ub77c \ubd88\ub9b0\ub2e4. \uc720\ub85c\ud30c\uc758 \uc2ed\uc790 \uc9c0\ud615\uc740 \uc54c\ubca0\ub3c4 \uc9c0\ud615\uc778 \uac83\uc73c\ub85c \ubcf4\uc5ec\uc9c0\uace0, \uc774\ub294 \ud398\ub2c8\ud150\ud2b8\uac00 \ub0ae\uc740 \uacf3\uc5d0 \uc788\uc74c\uc744 \uac00\ub9ac\ud0a8\ub2e4. \ud45c\uba74\uc5d0 \uc788\ub294 \ucda9\ub3cc\uad6c\ub4e4\uc740 \uc0c1\ub300\uc801\uc73c\ub85c \uc80a\uc740\ub370, \ud45c\uba74\uc774 \ud310 \uad6c\uc870\ub860\uacfc \uac19\uc740 \uc6d0\ub9ac\ub85c \ub04a\uc784\uc5c6\uc774 \ud65c\ub3d9\ud558\uae30 \ub54c\ubb38\uc774\ub77c\uace0 \ucd94\uce21\ub418\uace0 \uc788\ub2e4. \uc720\ub85c\ud30c\uc758 \uc5bc\uc74c \uc9c0\uac01\uc758 \ubc18\uc0ac\uc728\uc740 0.64\uc774\ub2e4. \uc720\ub85c\ud30c\ub294 \ud0dc\uc591\uacc4\uc5d0\uc11c \ubc18\uc0ac\uc728\uc774 \ub192\uc740 \ucc9c\uccb4\ub4e4 \uc911 \ud558\ub098\uc774\ub2e4. \ud61c\uc131\uc774 \uc720\ub85c\ud30c\uc5d0 \ucda9\ub3cc\ud560 \ub54c \ubc1c\uc0dd\ud558\ub294 \ucda9\ub3cc \uc8fc\ud30c\uc218\ub85c \ub098\uc774\ub97c \uacc4\uc0b0\ud558\uba74, \ud45c\uba74\uc740 \uc57d 20\ub9cc~1\uc5b5 8\ucc9c\ub9cc \ub144 \uc815\ub3c4 \ub418\uc5c8\ub2e4\ub294 \uacc4\uc0b0\uc774 \ub098\uc628\ub2e4. \uc720\ub85c\ud30c\uc758 \ud45c\uba74 \ud65c\ub3d9\uc5d0 \uad00\ud55c \uc774\ub860\uc740 \ub54c\ub85c\ub294 \ubaa8\uc21c\ub418\uae30\ub3c4 \ud558\uba70, \ud1b5\uc77c\ub41c \uacac\ud574\ub098 \uc774\ub860\uc740 \uc5c6\ub2e4.", "paragraph_answer": "\uc720\ub85c\ud30c\ub294 \uc0b0\uc774\ub098 \ucda9\ub3cc\uad6c\uac00 \uc801\uc5b4 \ud0dc\uc591\uacc4\uc5d0\uc11c \uac00\uc7a5 \ub9e4\ub044\ub7ec\uc6b4 \ucc9c\uccb4 \uc911 \ud558\ub098\uc774\ub2e4. \uadf8\ub7ec\ub098 \uc720\ub85c\ud30c\uc758 \uc801\ub3c4\uc5d0 10m\uc758 \ub192\uc774\ub97c \uac00\uc9c4 \uc5bc\uc74c \uc2a4\ud30c\uc774\ud06c\uac00 \uc788\ub294\ub370, \uc218\uc9c1\uc73c\ub85c \ub0b4\ub9ac\ucb10\ub294 \ud587\ube5b\uc758 \uc601\ud5a5\uc73c\ub85c \uc5bc\uc74c\uc774 \ub179\uc544\uc11c \uade0\uc5f4\uc774 \uc0dd\uae34 \uac83\uc774\ub77c\uace0 \ucd94\uce21\ub41c\ub2e4. \uc774\ub294 \ud398\ub2c8\ud150\ud2b8 \ub77c \ubd88\ub9b0\ub2e4. \uc720\ub85c\ud30c\uc758 \uc2ed\uc790 \uc9c0\ud615\uc740 \uc54c\ubca0\ub3c4 \uc9c0\ud615\uc778 \uac83\uc73c\ub85c \ubcf4\uc5ec\uc9c0\uace0, \uc774\ub294 \ud398\ub2c8\ud150\ud2b8\uac00 \ub0ae\uc740 \uacf3\uc5d0 \uc788\uc74c\uc744 \uac00\ub9ac\ud0a8\ub2e4. \ud45c\uba74\uc5d0 \uc788\ub294 \ucda9\ub3cc\uad6c\ub4e4\uc740 \uc0c1\ub300\uc801\uc73c\ub85c \uc80a\uc740\ub370, \ud45c\uba74\uc774 \ud310 \uad6c\uc870\ub860\uacfc \uac19\uc740 \uc6d0\ub9ac\ub85c \ub04a\uc784\uc5c6\uc774 \ud65c\ub3d9\ud558\uae30 \ub54c\ubb38\uc774\ub77c\uace0 \ucd94\uce21\ub418\uace0 \uc788\ub2e4. \uc720\ub85c\ud30c\uc758 \uc5bc\uc74c \uc9c0\uac01\uc758 \ubc18\uc0ac\uc728\uc740 0.64\uc774\ub2e4. \uc720\ub85c\ud30c\ub294 \ud0dc\uc591\uacc4\uc5d0\uc11c \ubc18\uc0ac\uc728\uc774 \ub192\uc740 \ucc9c\uccb4\ub4e4 \uc911 \ud558\ub098\uc774\ub2e4. \ud61c\uc131\uc774 \uc720\ub85c\ud30c\uc5d0 \ucda9\ub3cc\ud560 \ub54c \ubc1c\uc0dd\ud558\ub294 \ucda9\ub3cc \uc8fc\ud30c\uc218\ub85c \ub098\uc774\ub97c \uacc4\uc0b0\ud558\uba74, \ud45c\uba74\uc740 \uc57d 20\ub9cc~1\uc5b5 8\ucc9c\ub9cc \ub144 \uc815\ub3c4 \ub418\uc5c8\ub2e4\ub294 \uacc4\uc0b0\uc774 \ub098\uc628\ub2e4. \uc720\ub85c\ud30c\uc758 \ud45c\uba74 \ud65c\ub3d9\uc5d0 \uad00\ud55c \uc774\ub860\uc740 \ub54c\ub85c\ub294 \ubaa8\uc21c\ub418\uae30\ub3c4 \ud558\uba70, \ud1b5\uc77c\ub41c \uacac\ud574\ub098 \uc774\ub860\uc740 \uc5c6\ub2e4.", "sentence_answer": "\uc774\ub294 \ud398\ub2c8\ud150\ud2b8 \ub77c \ubd88\ub9b0\ub2e4.", "paragraph_id": "6657470-4-1", "paragraph_question": "question: \uc218\uc9c1\uc73c\ub85c \ub0b4\ub9ac\ucb10\ub294 \ud587\ube5b\uc73c\ub85c \uc5bc\uc74c\uc774 \ub179\uc544\uc11c \uc0dd\uae34\uac83\uc744 \ubb34\uc5c7\uc774\ub77c\uace0 \ubd80\ub974\ub294\uac00?, context: \uc720\ub85c\ud30c\ub294 \uc0b0\uc774\ub098 \ucda9\ub3cc\uad6c\uac00 \uc801\uc5b4 \ud0dc\uc591\uacc4\uc5d0\uc11c \uac00\uc7a5 \ub9e4\ub044\ub7ec\uc6b4 \ucc9c\uccb4 \uc911 \ud558\ub098\uc774\ub2e4. \uadf8\ub7ec\ub098 \uc720\ub85c\ud30c\uc758 \uc801\ub3c4\uc5d0 10m\uc758 \ub192\uc774\ub97c \uac00\uc9c4 \uc5bc\uc74c \uc2a4\ud30c\uc774\ud06c\uac00 \uc788\ub294\ub370, \uc218\uc9c1\uc73c\ub85c \ub0b4\ub9ac\ucb10\ub294 \ud587\ube5b\uc758 \uc601\ud5a5\uc73c\ub85c \uc5bc\uc74c\uc774 \ub179\uc544\uc11c \uade0\uc5f4\uc774 \uc0dd\uae34 \uac83\uc774\ub77c\uace0 \ucd94\uce21\ub41c\ub2e4. \uc774\ub294 \ud398\ub2c8\ud150\ud2b8\ub77c \ubd88\ub9b0\ub2e4. \uc720\ub85c\ud30c\uc758 \uc2ed\uc790 \uc9c0\ud615\uc740 \uc54c\ubca0\ub3c4 \uc9c0\ud615\uc778 \uac83\uc73c\ub85c \ubcf4\uc5ec\uc9c0\uace0, \uc774\ub294 \ud398\ub2c8\ud150\ud2b8\uac00 \ub0ae\uc740 \uacf3\uc5d0 \uc788\uc74c\uc744 \uac00\ub9ac\ud0a8\ub2e4. \ud45c\uba74\uc5d0 \uc788\ub294 \ucda9\ub3cc\uad6c\ub4e4\uc740 \uc0c1\ub300\uc801\uc73c\ub85c \uc80a\uc740\ub370, \ud45c\uba74\uc774 \ud310 \uad6c\uc870\ub860\uacfc \uac19\uc740 \uc6d0\ub9ac\ub85c \ub04a\uc784\uc5c6\uc774 \ud65c\ub3d9\ud558\uae30 \ub54c\ubb38\uc774\ub77c\uace0 \ucd94\uce21\ub418\uace0 \uc788\ub2e4. \uc720\ub85c\ud30c\uc758 \uc5bc\uc74c \uc9c0\uac01\uc758 \ubc18\uc0ac\uc728\uc740 0.64\uc774\ub2e4. \uc720\ub85c\ud30c\ub294 \ud0dc\uc591\uacc4\uc5d0\uc11c \ubc18\uc0ac\uc728\uc774 \ub192\uc740 \ucc9c\uccb4\ub4e4 \uc911 \ud558\ub098\uc774\ub2e4. \ud61c\uc131\uc774 \uc720\ub85c\ud30c\uc5d0 \ucda9\ub3cc\ud560 \ub54c \ubc1c\uc0dd\ud558\ub294 \ucda9\ub3cc \uc8fc\ud30c\uc218\ub85c \ub098\uc774\ub97c \uacc4\uc0b0\ud558\uba74, \ud45c\uba74\uc740 \uc57d 20\ub9cc~1\uc5b5 8\ucc9c\ub9cc \ub144 \uc815\ub3c4 \ub418\uc5c8\ub2e4\ub294 \uacc4\uc0b0\uc774 \ub098\uc628\ub2e4. \uc720\ub85c\ud30c\uc758 \ud45c\uba74 \ud65c\ub3d9\uc5d0 \uad00\ud55c \uc774\ub860\uc740 \ub54c\ub85c\ub294 \ubaa8\uc21c\ub418\uae30\ub3c4 \ud558\uba70, \ud1b5\uc77c\ub41c \uacac\ud574\ub098 \uc774\ub860\uc740 \uc5c6\ub2e4."} {"question": "\uc0bc\uc131\uc804\uc790\uc5d0\uc11c \uac24\ub7ed\uc2dc \ub178\ud2b87\uc758 \ud3ed\ubc1c \uc6d0\uc778\uc744 \ubb34\uc5c7\uc774\ub77c \ud574\uba85\ud588\ub098?", "paragraph": "\uadf8\ub7ec\ub098 8\uc6d4 24\uc77c\ubd80\ud130 \uc0bc\uc131 \uac24\ub7ed\uc2dc \ub178\ud2b87\uc758 \ud3ed\ubc1c \uc0ac\uace0\uac00 \uacc4\uc18d \ubcf4\uace0\ub418\uc790 \uc0bc\uc131\uc804\uc790\ub294 \ucd9c\uc2dc 13\uc77c\uc774 \ub418\ub294 9\uc6d4 2\uc77c \ub9ac\ucf5c\uc744 \ubc1c\ud45c\ud558\uc600\ub2e4. \ud558\uc9c0\ub9cc \ub9ac\ucf5c \ud6c4\uc5d0\ub3c4 \uacc4\uc18d\ub41c \ub178\ud2b8 7\uc758 \ud3ed\ubc1c \uc0ac\uace0\ub85c \uc778\ud574 \uc0bc\uc131\uc804\uc790\ub294 \uac24\ub7ed\uc2dc \ub178\ud2b8 7\uc758 \ud3ed\ubc1c \uc6d0\uc778\uc744 \ubc30\ud130\ub9ac \uc124\uacc4 \uacb0\ud568\uc73c\ub85c \ud574\uba85\ud55c \ucc44, \ubc30\ud130\ub9ac\ub97c ATL\uc0ac \ubc30\ud130\ub9ac\ub85c \uc804\ub7c9 \uad50\uccb4\ud588\uc73c\ub098 \ubc30\ud130\ub9ac\ub97c \uad50\uccb4\ud55c \ub178\ud2b8 7\uc5d0\uc11c\ub3c4 \ud3ed\ubc1c\uc774 \uc5f0\uac70\ud478 \uacc4\uc18d\ub418\uc5b4, \uae09\uae30\uc57c 2016\ub144 10\uc6d4 11\uc77c\uc5d0 \ub2e8\uc885\ub418\uc5c8\ub2e4. \uc774\ub85c \uc778\ud574 \uc6d0\ub798 2017\ub144 \uc0c1\ubc18\uae30 \uc608\uc815\uc774\uc5c8\ub358 \uc548\ub4dc\ub85c\uc774\ub4dc 7.0 \ub204\uac00 \uc5c5\ub370\uc774\ud2b8 \ub610\ud55c \ucde8\uc18c\ub418\uc5c8\ub2e4. 2017\ub144 5\uc6d4, \ub178\ud2b87 \uc7ac\ud310\ub9e4 \uac00\ub2a5\uc131\uc774 \ub192\uc544\uc838 \uad00\uc2ec\uc744 \ub04c\uace0\uc788\ub2e4. \uae30\uc874\uc758 \ub178\ud2b87\uc5d0\uc11c \ubc30\ud130\ub9ac \uc6a9\ub7c9\uc744 3200mAh\ub85c \uc904\uc774\uace0 \uc0bc\uc131 \uac24\ub7ed\uc2dc S8\uc758 UI\uc640 \ud568\uae68 \ube45\uc2a4\ube44 (\uc0bc\uc131 \uac24\ub7ed\uc2dc S8\uc758 AI \ube44\uc11c \uae30\ub2a5\uc73c\ub85c, \ube45\uc2a4\ube44 \ube44\uc804, \ube45\uc2a4\ube44 \ubcf4\uc774\uc2a4, \ube45\uc2a4\ube44 \ud648, \ube45\uc2a4\ube44 \ub9ac\ub9c8\uc778\ub354\uac00 \uc788\ub2e4.) \uc758 \ub450\uac00\uc9c0 \uae30\ub2a5\uc778 \ube45\uc2a4\ube44 \ud648\uacfc \ub9ac\ub9c8\uc778\ub354\uac00 \ucd94\uac00\ub41c \uc0c1\ud0dc\ub85c \uc7ac\ud310\ub9e4\ud558\ub294 \uac83\uc774\ub2e4 (UI\uc640 \uc18c\ud504\ud2b8\uc6e8\uc5b4\ub294 \uc804\ubd80 \uc548\uc804 \uc54c\uace0\ub9ac\uc998\uc744 \ucd94\uac00\ud574 \uc0c8\ub85c \ub9cc\ub4e4\uc5c8\ub2e4). \uc774\ub984\ub3c4 \ub610\ud55c \"\uac24\ub7ed\uc2dc \ub178\ud2b8 Fan Edition\" \uc73c\ub85c \ubc14\uafb8\uace0, \ud55c\uc815\ud310\uc73c\ub85c 40\ub9cc\ub300\uac00 \uad6d\ub0b4\uc5d0\uc11c\ub9cc \ucd9c\uc2dc\ud588\ub2e4. \ub610\ud55c, \uc548\ub4dc\ub85c\uc774\ub4dc \ubc84\uc804\uc744 7.0 (\ub204\uac00) \uc73c\ub85c \ub192\uc5ec \ucd9c\uc2dc\ud588\ub2e4.", "answer": "\ubc30\ud130\ub9ac \uc124\uacc4 \uacb0\ud568", "sentence": "\ud558\uc9c0\ub9cc \ub9ac\ucf5c \ud6c4\uc5d0\ub3c4 \uacc4\uc18d\ub41c \ub178\ud2b8 7\uc758 \ud3ed\ubc1c \uc0ac\uace0\ub85c \uc778\ud574 \uc0bc\uc131\uc804\uc790\ub294 \uac24\ub7ed\uc2dc \ub178\ud2b8 7\uc758 \ud3ed\ubc1c \uc6d0\uc778\uc744 \ubc30\ud130\ub9ac \uc124\uacc4 \uacb0\ud568 \uc73c\ub85c \ud574\uba85\ud55c \ucc44, \ubc30\ud130\ub9ac\ub97c ATL\uc0ac \ubc30\ud130\ub9ac\ub85c \uc804\ub7c9 \uad50\uccb4\ud588\uc73c\ub098 \ubc30\ud130\ub9ac\ub97c \uad50\uccb4\ud55c \ub178\ud2b8 7\uc5d0\uc11c\ub3c4 \ud3ed\ubc1c\uc774 \uc5f0\uac70\ud478 \uacc4\uc18d\ub418\uc5b4, \uae09\uae30\uc57c 2016\ub144 10\uc6d4 11\uc77c\uc5d0 \ub2e8\uc885\ub418\uc5c8\ub2e4.", "paragraph_sentence": "\uadf8\ub7ec\ub098 8\uc6d4 24\uc77c\ubd80\ud130 \uc0bc\uc131 \uac24\ub7ed\uc2dc \ub178\ud2b87\uc758 \ud3ed\ubc1c \uc0ac\uace0\uac00 \uacc4\uc18d \ubcf4\uace0\ub418\uc790 \uc0bc\uc131\uc804\uc790\ub294 \ucd9c\uc2dc 13\uc77c\uc774 \ub418\ub294 9\uc6d4 2\uc77c \ub9ac\ucf5c\uc744 \ubc1c\ud45c\ud558\uc600\ub2e4. \ud558\uc9c0\ub9cc \ub9ac\ucf5c \ud6c4\uc5d0\ub3c4 \uacc4\uc18d\ub41c \ub178\ud2b8 7\uc758 \ud3ed\ubc1c \uc0ac\uace0\ub85c \uc778\ud574 \uc0bc\uc131\uc804\uc790\ub294 \uac24\ub7ed\uc2dc \ub178\ud2b8 7\uc758 \ud3ed\ubc1c \uc6d0\uc778\uc744 \ubc30\ud130\ub9ac \uc124\uacc4 \uacb0\ud568 \uc73c\ub85c \ud574\uba85\ud55c \ucc44, \ubc30\ud130\ub9ac\ub97c ATL\uc0ac \ubc30\ud130\ub9ac\ub85c \uc804\ub7c9 \uad50\uccb4\ud588\uc73c\ub098 \ubc30\ud130\ub9ac\ub97c \uad50\uccb4\ud55c \ub178\ud2b8 7\uc5d0\uc11c\ub3c4 \ud3ed\ubc1c\uc774 \uc5f0\uac70\ud478 \uacc4\uc18d\ub418\uc5b4, \uae09\uae30\uc57c 2016\ub144 10\uc6d4 11\uc77c\uc5d0 \ub2e8\uc885\ub418\uc5c8\ub2e4. \uc774\ub85c \uc778\ud574 \uc6d0\ub798 2017\ub144 \uc0c1\ubc18\uae30 \uc608\uc815\uc774\uc5c8\ub358 \uc548\ub4dc\ub85c\uc774\ub4dc 7.0 \ub204\uac00 \uc5c5\ub370\uc774\ud2b8 \ub610\ud55c \ucde8\uc18c\ub418\uc5c8\ub2e4. 2017\ub144 5\uc6d4, \ub178\ud2b87 \uc7ac\ud310\ub9e4 \uac00\ub2a5\uc131\uc774 \ub192\uc544\uc838 \uad00\uc2ec\uc744 \ub04c\uace0\uc788\ub2e4. \uae30\uc874\uc758 \ub178\ud2b87\uc5d0\uc11c \ubc30\ud130\ub9ac \uc6a9\ub7c9\uc744 3200mAh\ub85c \uc904\uc774\uace0 \uc0bc\uc131 \uac24\ub7ed\uc2dc S8\uc758 UI\uc640 \ud568\uae68 \ube45\uc2a4\ube44 (\uc0bc\uc131 \uac24\ub7ed\uc2dc S8\uc758 AI \ube44\uc11c \uae30\ub2a5\uc73c\ub85c, \ube45\uc2a4\ube44 \ube44\uc804, \ube45\uc2a4\ube44 \ubcf4\uc774\uc2a4, \ube45\uc2a4\ube44 \ud648, \ube45\uc2a4\ube44 \ub9ac\ub9c8\uc778\ub354\uac00 \uc788\ub2e4. ) \uc758 \ub450\uac00\uc9c0 \uae30\ub2a5\uc778 \ube45\uc2a4\ube44 \ud648\uacfc \ub9ac\ub9c8\uc778\ub354\uac00 \ucd94\uac00\ub41c \uc0c1\ud0dc\ub85c \uc7ac\ud310\ub9e4\ud558\ub294 \uac83\uc774\ub2e4 (UI\uc640 \uc18c\ud504\ud2b8\uc6e8\uc5b4\ub294 \uc804\ubd80 \uc548\uc804 \uc54c\uace0\ub9ac\uc998\uc744 \ucd94\uac00\ud574 \uc0c8\ub85c \ub9cc\ub4e4\uc5c8\ub2e4). \uc774\ub984\ub3c4 \ub610\ud55c \"\uac24\ub7ed\uc2dc \ub178\ud2b8 Fan Edition\" \uc73c\ub85c \ubc14\uafb8\uace0, \ud55c\uc815\ud310\uc73c\ub85c 40\ub9cc\ub300\uac00 \uad6d\ub0b4\uc5d0\uc11c\ub9cc \ucd9c\uc2dc\ud588\ub2e4. \ub610\ud55c, \uc548\ub4dc\ub85c\uc774\ub4dc \ubc84\uc804\uc744 7.0 (\ub204\uac00) \uc73c\ub85c \ub192\uc5ec \ucd9c\uc2dc\ud588\ub2e4.", "paragraph_answer": "\uadf8\ub7ec\ub098 8\uc6d4 24\uc77c\ubd80\ud130 \uc0bc\uc131 \uac24\ub7ed\uc2dc \ub178\ud2b87\uc758 \ud3ed\ubc1c \uc0ac\uace0\uac00 \uacc4\uc18d \ubcf4\uace0\ub418\uc790 \uc0bc\uc131\uc804\uc790\ub294 \ucd9c\uc2dc 13\uc77c\uc774 \ub418\ub294 9\uc6d4 2\uc77c \ub9ac\ucf5c\uc744 \ubc1c\ud45c\ud558\uc600\ub2e4. \ud558\uc9c0\ub9cc \ub9ac\ucf5c \ud6c4\uc5d0\ub3c4 \uacc4\uc18d\ub41c \ub178\ud2b8 7\uc758 \ud3ed\ubc1c \uc0ac\uace0\ub85c \uc778\ud574 \uc0bc\uc131\uc804\uc790\ub294 \uac24\ub7ed\uc2dc \ub178\ud2b8 7\uc758 \ud3ed\ubc1c \uc6d0\uc778\uc744 \ubc30\ud130\ub9ac \uc124\uacc4 \uacb0\ud568 \uc73c\ub85c \ud574\uba85\ud55c \ucc44, \ubc30\ud130\ub9ac\ub97c ATL\uc0ac \ubc30\ud130\ub9ac\ub85c \uc804\ub7c9 \uad50\uccb4\ud588\uc73c\ub098 \ubc30\ud130\ub9ac\ub97c \uad50\uccb4\ud55c \ub178\ud2b8 7\uc5d0\uc11c\ub3c4 \ud3ed\ubc1c\uc774 \uc5f0\uac70\ud478 \uacc4\uc18d\ub418\uc5b4, \uae09\uae30\uc57c 2016\ub144 10\uc6d4 11\uc77c\uc5d0 \ub2e8\uc885\ub418\uc5c8\ub2e4. \uc774\ub85c \uc778\ud574 \uc6d0\ub798 2017\ub144 \uc0c1\ubc18\uae30 \uc608\uc815\uc774\uc5c8\ub358 \uc548\ub4dc\ub85c\uc774\ub4dc 7.0 \ub204\uac00 \uc5c5\ub370\uc774\ud2b8 \ub610\ud55c \ucde8\uc18c\ub418\uc5c8\ub2e4. 2017\ub144 5\uc6d4, \ub178\ud2b87 \uc7ac\ud310\ub9e4 \uac00\ub2a5\uc131\uc774 \ub192\uc544\uc838 \uad00\uc2ec\uc744 \ub04c\uace0\uc788\ub2e4. \uae30\uc874\uc758 \ub178\ud2b87\uc5d0\uc11c \ubc30\ud130\ub9ac \uc6a9\ub7c9\uc744 3200mAh\ub85c \uc904\uc774\uace0 \uc0bc\uc131 \uac24\ub7ed\uc2dc S8\uc758 UI\uc640 \ud568\uae68 \ube45\uc2a4\ube44 (\uc0bc\uc131 \uac24\ub7ed\uc2dc S8\uc758 AI \ube44\uc11c \uae30\ub2a5\uc73c\ub85c, \ube45\uc2a4\ube44 \ube44\uc804, \ube45\uc2a4\ube44 \ubcf4\uc774\uc2a4, \ube45\uc2a4\ube44 \ud648, \ube45\uc2a4\ube44 \ub9ac\ub9c8\uc778\ub354\uac00 \uc788\ub2e4.) \uc758 \ub450\uac00\uc9c0 \uae30\ub2a5\uc778 \ube45\uc2a4\ube44 \ud648\uacfc \ub9ac\ub9c8\uc778\ub354\uac00 \ucd94\uac00\ub41c \uc0c1\ud0dc\ub85c \uc7ac\ud310\ub9e4\ud558\ub294 \uac83\uc774\ub2e4 (UI\uc640 \uc18c\ud504\ud2b8\uc6e8\uc5b4\ub294 \uc804\ubd80 \uc548\uc804 \uc54c\uace0\ub9ac\uc998\uc744 \ucd94\uac00\ud574 \uc0c8\ub85c \ub9cc\ub4e4\uc5c8\ub2e4). \uc774\ub984\ub3c4 \ub610\ud55c \"\uac24\ub7ed\uc2dc \ub178\ud2b8 Fan Edition\" \uc73c\ub85c \ubc14\uafb8\uace0, \ud55c\uc815\ud310\uc73c\ub85c 40\ub9cc\ub300\uac00 \uad6d\ub0b4\uc5d0\uc11c\ub9cc \ucd9c\uc2dc\ud588\ub2e4. \ub610\ud55c, \uc548\ub4dc\ub85c\uc774\ub4dc \ubc84\uc804\uc744 7.0 (\ub204\uac00) \uc73c\ub85c \ub192\uc5ec \ucd9c\uc2dc\ud588\ub2e4.", "sentence_answer": "\ud558\uc9c0\ub9cc \ub9ac\ucf5c \ud6c4\uc5d0\ub3c4 \uacc4\uc18d\ub41c \ub178\ud2b8 7\uc758 \ud3ed\ubc1c \uc0ac\uace0\ub85c \uc778\ud574 \uc0bc\uc131\uc804\uc790\ub294 \uac24\ub7ed\uc2dc \ub178\ud2b8 7\uc758 \ud3ed\ubc1c \uc6d0\uc778\uc744 \ubc30\ud130\ub9ac \uc124\uacc4 \uacb0\ud568 \uc73c\ub85c \ud574\uba85\ud55c \ucc44, \ubc30\ud130\ub9ac\ub97c ATL\uc0ac \ubc30\ud130\ub9ac\ub85c \uc804\ub7c9 \uad50\uccb4\ud588\uc73c\ub098 \ubc30\ud130\ub9ac\ub97c \uad50\uccb4\ud55c \ub178\ud2b8 7\uc5d0\uc11c\ub3c4 \ud3ed\ubc1c\uc774 \uc5f0\uac70\ud478 \uacc4\uc18d\ub418\uc5b4, \uae09\uae30\uc57c 2016\ub144 10\uc6d4 11\uc77c\uc5d0 \ub2e8\uc885\ub418\uc5c8\ub2e4.", "paragraph_id": "6531575-0-1", "paragraph_question": "question: \uc0bc\uc131\uc804\uc790\uc5d0\uc11c \uac24\ub7ed\uc2dc \ub178\ud2b87\uc758 \ud3ed\ubc1c \uc6d0\uc778\uc744 \ubb34\uc5c7\uc774\ub77c \ud574\uba85\ud588\ub098?, context: \uadf8\ub7ec\ub098 8\uc6d4 24\uc77c\ubd80\ud130 \uc0bc\uc131 \uac24\ub7ed\uc2dc \ub178\ud2b87\uc758 \ud3ed\ubc1c \uc0ac\uace0\uac00 \uacc4\uc18d \ubcf4\uace0\ub418\uc790 \uc0bc\uc131\uc804\uc790\ub294 \ucd9c\uc2dc 13\uc77c\uc774 \ub418\ub294 9\uc6d4 2\uc77c \ub9ac\ucf5c\uc744 \ubc1c\ud45c\ud558\uc600\ub2e4. \ud558\uc9c0\ub9cc \ub9ac\ucf5c \ud6c4\uc5d0\ub3c4 \uacc4\uc18d\ub41c \ub178\ud2b8 7\uc758 \ud3ed\ubc1c \uc0ac\uace0\ub85c \uc778\ud574 \uc0bc\uc131\uc804\uc790\ub294 \uac24\ub7ed\uc2dc \ub178\ud2b8 7\uc758 \ud3ed\ubc1c \uc6d0\uc778\uc744 \ubc30\ud130\ub9ac \uc124\uacc4 \uacb0\ud568\uc73c\ub85c \ud574\uba85\ud55c \ucc44, \ubc30\ud130\ub9ac\ub97c ATL\uc0ac \ubc30\ud130\ub9ac\ub85c \uc804\ub7c9 \uad50\uccb4\ud588\uc73c\ub098 \ubc30\ud130\ub9ac\ub97c \uad50\uccb4\ud55c \ub178\ud2b8 7\uc5d0\uc11c\ub3c4 \ud3ed\ubc1c\uc774 \uc5f0\uac70\ud478 \uacc4\uc18d\ub418\uc5b4, \uae09\uae30\uc57c 2016\ub144 10\uc6d4 11\uc77c\uc5d0 \ub2e8\uc885\ub418\uc5c8\ub2e4. \uc774\ub85c \uc778\ud574 \uc6d0\ub798 2017\ub144 \uc0c1\ubc18\uae30 \uc608\uc815\uc774\uc5c8\ub358 \uc548\ub4dc\ub85c\uc774\ub4dc 7.0 \ub204\uac00 \uc5c5\ub370\uc774\ud2b8 \ub610\ud55c \ucde8\uc18c\ub418\uc5c8\ub2e4. 2017\ub144 5\uc6d4, \ub178\ud2b87 \uc7ac\ud310\ub9e4 \uac00\ub2a5\uc131\uc774 \ub192\uc544\uc838 \uad00\uc2ec\uc744 \ub04c\uace0\uc788\ub2e4. \uae30\uc874\uc758 \ub178\ud2b87\uc5d0\uc11c \ubc30\ud130\ub9ac \uc6a9\ub7c9\uc744 3200mAh\ub85c \uc904\uc774\uace0 \uc0bc\uc131 \uac24\ub7ed\uc2dc S8\uc758 UI\uc640 \ud568\uae68 \ube45\uc2a4\ube44 (\uc0bc\uc131 \uac24\ub7ed\uc2dc S8\uc758 AI \ube44\uc11c \uae30\ub2a5\uc73c\ub85c, \ube45\uc2a4\ube44 \ube44\uc804, \ube45\uc2a4\ube44 \ubcf4\uc774\uc2a4, \ube45\uc2a4\ube44 \ud648, \ube45\uc2a4\ube44 \ub9ac\ub9c8\uc778\ub354\uac00 \uc788\ub2e4.) \uc758 \ub450\uac00\uc9c0 \uae30\ub2a5\uc778 \ube45\uc2a4\ube44 \ud648\uacfc \ub9ac\ub9c8\uc778\ub354\uac00 \ucd94\uac00\ub41c \uc0c1\ud0dc\ub85c \uc7ac\ud310\ub9e4\ud558\ub294 \uac83\uc774\ub2e4 (UI\uc640 \uc18c\ud504\ud2b8\uc6e8\uc5b4\ub294 \uc804\ubd80 \uc548\uc804 \uc54c\uace0\ub9ac\uc998\uc744 \ucd94\uac00\ud574 \uc0c8\ub85c \ub9cc\ub4e4\uc5c8\ub2e4). \uc774\ub984\ub3c4 \ub610\ud55c \"\uac24\ub7ed\uc2dc \ub178\ud2b8 Fan Edition\" \uc73c\ub85c \ubc14\uafb8\uace0, \ud55c\uc815\ud310\uc73c\ub85c 40\ub9cc\ub300\uac00 \uad6d\ub0b4\uc5d0\uc11c\ub9cc \ucd9c\uc2dc\ud588\ub2e4. \ub610\ud55c, \uc548\ub4dc\ub85c\uc774\ub4dc \ubc84\uc804\uc744 7.0 (\ub204\uac00) \uc73c\ub85c \ub192\uc5ec \ucd9c\uc2dc\ud588\ub2e4."} {"question": "\ub9c8\ub204\uc5d8 \ub370 \uace0\ub3c4\uc774\uc758 \ucd9c\uc0dd\uc9c0\ub294?", "paragraph": "\uace0\ub3c4\uc774\ub294 1767\ub144 \ubc14\ub2e4\ud638\uc2a4 \uc9c0\ubc29\uc758 \uce74\uc2a4\ud22c\uc5d0\ub77c(Castuera)\uc5d0\uc11c \uac00\ub09c\ud55c \uadc0\uc871\uc758 \uc544\ub4e4\ub85c \ud0dc\uc5b4\ub0ac\ub2e4. 1784\ub144 \uadfc\uc704\ubd80\ub300\uc5d0 \uc785\ub300\ud574 \ub9c8\ub4dc\ub9ac\ub4dc\ub85c \uc0c1\uacbd\ud55c \uace0\ub3c4\uc774\ub294 1788\ub144 \uc2a4\ud398\uc778\uc758 \uc655\uc704 \uacc4\uc2b9\uc790\uc778 \uce74\ub97c\ub85c\uc2a4\ub97c \ub9cc\ub098\uac8c \ub418\uc5c8\ub2e4. \uce74\ub97c\ub85c\uc2a4\uc640 \uadf8 \uc544\ub0b4 \ub9c8\ub9ac\uc544 \ub8e8\uc774\uc0ac\ub294 \uace0\ub3c4\uc774\ub97c \ubb34\ucc99 \ucd1d\uc560\ud558\uc5ec \uadf8\ub294 \uae09\uc18d\ub3c4\ub85c \uc2b9\uc9c4\uc744 \uac70\ub4ed\ud574 1790\ub144 8\uc6d4, \uc9c0\ud718\uad00\uc774 \ub418\uc5c8\uc73c\uba70 \uc774\ub4ec\ud574 7\uc6d4\uc5d0\ub294 \uc721\uad70 \uc911\uc7a5\uc774 \ub418\uc5c8\ub2e4. 1791\ub144 \ucd1d\ub9ac \ud50c\ub85c\ub9ac\ub2e4\ube14\ub791\uce74(Floridablanca)\ub294 \uace0\ub3c4\uc774\uc640 \uc655\ube44\uac00 \uc560\uc778 \uad00\uacc4\uc5d0 \uc788\ub2e4\uace0 \ud3ed\ub85c\ud558\uc600\uc73c\ub098 \uadf8\ub294 \uc774\ub4ec\ud574 \ucd08 \uc2e4\uac01\ud588\ub2e4. \ud6c4\uc784 \uc544\ub780\ub2e4(Aranda) \ucd1d\ub9ac \ub610\ud55c 10\uac1c\uc6d4\ub9cc\uc5d0 \uc2e4\uac01\ud558\uace0 1792\ub144 11\uc6d4 15\uc77c, \uace0\ub3c4\uc774\ub294 \ucd1d\ub9ac\uac00 \ub418\uc5c8\ub2e4. \uadf8\ub294 \uc54c\ucfe0\ub514\uc544(Alc\u00fadia) \uacf5\uc791\uc758 \uc791\uc704\ub97c \ubc1b\uc558\uc73c\uba70 \ud669\uae08\uc591\ubaa8 \uae30\uc0ac\ub2e8\uc758 \ub2e8\uc6d0\uc774 \ub418\uc5c8\ub2e4. \ub610 \uc774\ub4ec\ud574\uc5d0\ub294 \ucd1d\uc0ac\ub839\uad00, \uc2a4\uc5d0\uce74 \uacf5\uc791, \uc54c\ubc14\ub808\uc988 \ud6c4\uc791, \uc18c\ud1a0 \ub370 \ub85c\ub9c8(Soto de Roma) \uacbd\uc5d0 \ubd09\ud574\uc84c\ub2e4.", "answer": "\uce74\uc2a4\ud22c\uc5d0\ub77c", "sentence": "\uace0\ub3c4\uc774\ub294 1767\ub144 \ubc14\ub2e4\ud638\uc2a4 \uc9c0\ubc29\uc758 \uce74\uc2a4\ud22c\uc5d0\ub77c (Castuera)", "paragraph_sentence": " \uace0\ub3c4\uc774\ub294 1767\ub144 \ubc14\ub2e4\ud638\uc2a4 \uc9c0\ubc29\uc758 \uce74\uc2a4\ud22c\uc5d0\ub77c (Castuera) \uc5d0\uc11c \uac00\ub09c\ud55c \uadc0\uc871\uc758 \uc544\ub4e4\ub85c \ud0dc\uc5b4\ub0ac\ub2e4. 1784\ub144 \uadfc\uc704\ubd80\ub300\uc5d0 \uc785\ub300\ud574 \ub9c8\ub4dc\ub9ac\ub4dc\ub85c \uc0c1\uacbd\ud55c \uace0\ub3c4\uc774\ub294 1788\ub144 \uc2a4\ud398\uc778\uc758 \uc655\uc704 \uacc4\uc2b9\uc790\uc778 \uce74\ub97c\ub85c\uc2a4\ub97c \ub9cc\ub098\uac8c \ub418\uc5c8\ub2e4. \uce74\ub97c\ub85c\uc2a4\uc640 \uadf8 \uc544\ub0b4 \ub9c8\ub9ac\uc544 \ub8e8\uc774\uc0ac\ub294 \uace0\ub3c4\uc774\ub97c \ubb34\ucc99 \ucd1d\uc560\ud558\uc5ec \uadf8\ub294 \uae09\uc18d\ub3c4\ub85c \uc2b9\uc9c4\uc744 \uac70\ub4ed\ud574 1790\ub144 8\uc6d4, \uc9c0\ud718\uad00\uc774 \ub418\uc5c8\uc73c\uba70 \uc774\ub4ec\ud574 7\uc6d4\uc5d0\ub294 \uc721\uad70 \uc911\uc7a5\uc774 \ub418\uc5c8\ub2e4. 1791\ub144 \ucd1d\ub9ac \ud50c\ub85c\ub9ac\ub2e4\ube14\ub791\uce74(Floridablanca)\ub294 \uace0\ub3c4\uc774\uc640 \uc655\ube44\uac00 \uc560\uc778 \uad00\uacc4\uc5d0 \uc788\ub2e4\uace0 \ud3ed\ub85c\ud558\uc600\uc73c\ub098 \uadf8\ub294 \uc774\ub4ec\ud574 \ucd08 \uc2e4\uac01\ud588\ub2e4. \ud6c4\uc784 \uc544\ub780\ub2e4(Aranda) \ucd1d\ub9ac \ub610\ud55c 10\uac1c\uc6d4\ub9cc\uc5d0 \uc2e4\uac01\ud558\uace0 1792\ub144 11\uc6d4 15\uc77c, \uace0\ub3c4\uc774\ub294 \ucd1d\ub9ac\uac00 \ub418\uc5c8\ub2e4. \uadf8\ub294 \uc54c\ucfe0\ub514\uc544(Alc\u00fadia) \uacf5\uc791\uc758 \uc791\uc704\ub97c \ubc1b\uc558\uc73c\uba70 \ud669\uae08\uc591\ubaa8 \uae30\uc0ac\ub2e8\uc758 \ub2e8\uc6d0\uc774 \ub418\uc5c8\ub2e4. \ub610 \uc774\ub4ec\ud574\uc5d0\ub294 \ucd1d\uc0ac\ub839\uad00, \uc2a4\uc5d0\uce74 \uacf5\uc791, \uc54c\ubc14\ub808\uc988 \ud6c4\uc791, \uc18c\ud1a0 \ub370 \ub85c\ub9c8(Soto de Roma) \uacbd\uc5d0 \ubd09\ud574\uc84c\ub2e4.", "paragraph_answer": "\uace0\ub3c4\uc774\ub294 1767\ub144 \ubc14\ub2e4\ud638\uc2a4 \uc9c0\ubc29\uc758 \uce74\uc2a4\ud22c\uc5d0\ub77c (Castuera)\uc5d0\uc11c \uac00\ub09c\ud55c \uadc0\uc871\uc758 \uc544\ub4e4\ub85c \ud0dc\uc5b4\ub0ac\ub2e4. 1784\ub144 \uadfc\uc704\ubd80\ub300\uc5d0 \uc785\ub300\ud574 \ub9c8\ub4dc\ub9ac\ub4dc\ub85c \uc0c1\uacbd\ud55c \uace0\ub3c4\uc774\ub294 1788\ub144 \uc2a4\ud398\uc778\uc758 \uc655\uc704 \uacc4\uc2b9\uc790\uc778 \uce74\ub97c\ub85c\uc2a4\ub97c \ub9cc\ub098\uac8c \ub418\uc5c8\ub2e4. \uce74\ub97c\ub85c\uc2a4\uc640 \uadf8 \uc544\ub0b4 \ub9c8\ub9ac\uc544 \ub8e8\uc774\uc0ac\ub294 \uace0\ub3c4\uc774\ub97c \ubb34\ucc99 \ucd1d\uc560\ud558\uc5ec \uadf8\ub294 \uae09\uc18d\ub3c4\ub85c \uc2b9\uc9c4\uc744 \uac70\ub4ed\ud574 1790\ub144 8\uc6d4, \uc9c0\ud718\uad00\uc774 \ub418\uc5c8\uc73c\uba70 \uc774\ub4ec\ud574 7\uc6d4\uc5d0\ub294 \uc721\uad70 \uc911\uc7a5\uc774 \ub418\uc5c8\ub2e4. 1791\ub144 \ucd1d\ub9ac \ud50c\ub85c\ub9ac\ub2e4\ube14\ub791\uce74(Floridablanca)\ub294 \uace0\ub3c4\uc774\uc640 \uc655\ube44\uac00 \uc560\uc778 \uad00\uacc4\uc5d0 \uc788\ub2e4\uace0 \ud3ed\ub85c\ud558\uc600\uc73c\ub098 \uadf8\ub294 \uc774\ub4ec\ud574 \ucd08 \uc2e4\uac01\ud588\ub2e4. \ud6c4\uc784 \uc544\ub780\ub2e4(Aranda) \ucd1d\ub9ac \ub610\ud55c 10\uac1c\uc6d4\ub9cc\uc5d0 \uc2e4\uac01\ud558\uace0 1792\ub144 11\uc6d4 15\uc77c, \uace0\ub3c4\uc774\ub294 \ucd1d\ub9ac\uac00 \ub418\uc5c8\ub2e4. \uadf8\ub294 \uc54c\ucfe0\ub514\uc544(Alc\u00fadia) \uacf5\uc791\uc758 \uc791\uc704\ub97c \ubc1b\uc558\uc73c\uba70 \ud669\uae08\uc591\ubaa8 \uae30\uc0ac\ub2e8\uc758 \ub2e8\uc6d0\uc774 \ub418\uc5c8\ub2e4. \ub610 \uc774\ub4ec\ud574\uc5d0\ub294 \ucd1d\uc0ac\ub839\uad00, \uc2a4\uc5d0\uce74 \uacf5\uc791, \uc54c\ubc14\ub808\uc988 \ud6c4\uc791, \uc18c\ud1a0 \ub370 \ub85c\ub9c8(Soto de Roma) \uacbd\uc5d0 \ubd09\ud574\uc84c\ub2e4.", "sentence_answer": "\uace0\ub3c4\uc774\ub294 1767\ub144 \ubc14\ub2e4\ud638\uc2a4 \uc9c0\ubc29\uc758 \uce74\uc2a4\ud22c\uc5d0\ub77c (Castuera)", "paragraph_id": "6498958-0-0", "paragraph_question": "question: \ub9c8\ub204\uc5d8 \ub370 \uace0\ub3c4\uc774\uc758 \ucd9c\uc0dd\uc9c0\ub294?, context: \uace0\ub3c4\uc774\ub294 1767\ub144 \ubc14\ub2e4\ud638\uc2a4 \uc9c0\ubc29\uc758 \uce74\uc2a4\ud22c\uc5d0\ub77c(Castuera)\uc5d0\uc11c \uac00\ub09c\ud55c \uadc0\uc871\uc758 \uc544\ub4e4\ub85c \ud0dc\uc5b4\ub0ac\ub2e4. 1784\ub144 \uadfc\uc704\ubd80\ub300\uc5d0 \uc785\ub300\ud574 \ub9c8\ub4dc\ub9ac\ub4dc\ub85c \uc0c1\uacbd\ud55c \uace0\ub3c4\uc774\ub294 1788\ub144 \uc2a4\ud398\uc778\uc758 \uc655\uc704 \uacc4\uc2b9\uc790\uc778 \uce74\ub97c\ub85c\uc2a4\ub97c \ub9cc\ub098\uac8c \ub418\uc5c8\ub2e4. \uce74\ub97c\ub85c\uc2a4\uc640 \uadf8 \uc544\ub0b4 \ub9c8\ub9ac\uc544 \ub8e8\uc774\uc0ac\ub294 \uace0\ub3c4\uc774\ub97c \ubb34\ucc99 \ucd1d\uc560\ud558\uc5ec \uadf8\ub294 \uae09\uc18d\ub3c4\ub85c \uc2b9\uc9c4\uc744 \uac70\ub4ed\ud574 1790\ub144 8\uc6d4, \uc9c0\ud718\uad00\uc774 \ub418\uc5c8\uc73c\uba70 \uc774\ub4ec\ud574 7\uc6d4\uc5d0\ub294 \uc721\uad70 \uc911\uc7a5\uc774 \ub418\uc5c8\ub2e4. 1791\ub144 \ucd1d\ub9ac \ud50c\ub85c\ub9ac\ub2e4\ube14\ub791\uce74(Floridablanca)\ub294 \uace0\ub3c4\uc774\uc640 \uc655\ube44\uac00 \uc560\uc778 \uad00\uacc4\uc5d0 \uc788\ub2e4\uace0 \ud3ed\ub85c\ud558\uc600\uc73c\ub098 \uadf8\ub294 \uc774\ub4ec\ud574 \ucd08 \uc2e4\uac01\ud588\ub2e4. \ud6c4\uc784 \uc544\ub780\ub2e4(Aranda) \ucd1d\ub9ac \ub610\ud55c 10\uac1c\uc6d4\ub9cc\uc5d0 \uc2e4\uac01\ud558\uace0 1792\ub144 11\uc6d4 15\uc77c, \uace0\ub3c4\uc774\ub294 \ucd1d\ub9ac\uac00 \ub418\uc5c8\ub2e4. \uadf8\ub294 \uc54c\ucfe0\ub514\uc544(Alc\u00fadia) \uacf5\uc791\uc758 \uc791\uc704\ub97c \ubc1b\uc558\uc73c\uba70 \ud669\uae08\uc591\ubaa8 \uae30\uc0ac\ub2e8\uc758 \ub2e8\uc6d0\uc774 \ub418\uc5c8\ub2e4. \ub610 \uc774\ub4ec\ud574\uc5d0\ub294 \ucd1d\uc0ac\ub839\uad00, \uc2a4\uc5d0\uce74 \uacf5\uc791, \uc54c\ubc14\ub808\uc988 \ud6c4\uc791, \uc18c\ud1a0 \ub370 \ub85c\ub9c8(Soto de Roma) \uacbd\uc5d0 \ubd09\ud574\uc84c\ub2e4."} {"question": "\ub9cc\uc288\ud0c0\uc778\uc740 \ud3f0\ub780\ub4dc\uad70\uc744 \uc5b4\ub290 \uac15\uc758 \uc11c\uc548\uc5d0 \ud3ec\uc704\uc2dc\ud0a4\ub824\uace0 \uacc4\ud68d \ud588\ub294\uac00?", "paragraph": "1939\ub144 8\uc6d4 18\uc77c, \ud3f4\ub780\ub4dc\ub97c \uce68\uacf5\ud558\ub294 \ubc31\uc0c9 \uc791\uc804\uc744 \uc900\ube44\ud558\ub358 \ub9cc\uc288\ud0c0\uc778\uc740 \uac8c\ub974\ud2b8 \ud3f0 \ub8ec\ud2b8\uc288\ud14c\ud2b8\uc758 \ub0a8\ubd80 \uc9d1\ub2e8\uad70 \ucc38\ubaa8\uc7a5\uc73c\ub85c \ubd80\uc784\ud55c\ub2e4. \ub9cc\uc288\ud0c0\uc778\uc740 \ub8ec\ud2b8\uc288\ud14c\ud2b8\uc758 \uc791\uc804\ucc38\ubaa8 \uadc4\ud130 \ube14\ub8e8\uba58\ud2b8\ub9ac\ud2b8 \ub300\ub839\uacfc \ud568\uaed8 \uc791\uc804\uacc4\ud68d \uc218\ub9bd\uc5d0 \ucc38\uc5ec\ud588\ub2e4. \ub9cc\uc288\ud0c0\uc778\uc740 \uc9d1\ub2e8\uad70\uc758 \uae30\uac11\ubcd1\ub825 \ub2e4\uc218\ub97c \ubc1c\ud130 \ud3f0 \ub77c\uc774\ud5e4\ub098\uc6b0\uc758 \uc81c10\uad70\uc5d0 \uc9d1\uc911\uc2dc\ucf1c \uc8fc\uba74\uc11c \ub3cc\ud30c\uad6c\ub97c \ub9c8\ub828, \ud3f4\ub780\ub4dc\uad70\uc744 \ube44\uc2a4\ud234\ub77c \uac15 \uc11c\uc548\uc5d0 \ud3ec\uc704\uc2dc\ud0a4\uace0, \ub77c\uc774\ud5e4\ub098\uc6b0\uc758 \uc81c10\uad70\uc774 \uae30\uac11\uae30\ub3d9\uc758 \ud798\uc73c\ub85c \ud3f4\ub780\ub4dc\uc758 \uc218\ub3c4 \ubc14\ub974\uc0e4\ubc14\uae4c\uc9c0 \ub3cc\uc9c4\ud558\ub294 \ub3d9\uc548 \ube4c\ud5ec\ub984 \ub9ac\uc2a4\ud2b8\uc758 \uc81c14\uad70\uacfc \uc694\ud558\ub124\uc2a4 \ube14\ub77c\ucf54\ube44\uce20\uc758 \uc81c8\uad70\uc740 \uce21\uba74\uc9c0\uc6d0\uc744 \ub9e1\ub294\ub2e4\ub294 \uacc4\ud68d\uc744 \uc791\uc131\ud588\uace0, \ub8ec\ud2b8\uc288\ud14c\ud2b8\ub294 \uc774\ub97c \ubc1b\uc544\ub4e4\uc600\ub2e4. \uc0ac\uc2e4 \ub9cc\uc288\ud0c0\uc778\uc740 \uac1c\uc778\uc801\uc73c\ub85c \ud3f4\ub780\ub4dc\ub97c \ub3c5\uc77c\uacfc \uc18c\ub828 \uc0ac\uc774\uc758 \uc644\ucda9\uc9c0\ub300\ub85c \ub0a8\uaca8\ub450\ub294 \ud3b8\uc774 \uc88b\ub2e4\uace0 \uc0dd\uac01\ud588\uae30\uc5d0 \ub300\ud3f4\ub780\ub4dc \uad70\uc0ac \uc791\uc804\uc5d0 \ubbf8\uc628\uc801\uc774\uc5c8\ub2e4. \ubfd0\ub9cc \uc544\ub2c8\ub77c \ud3f4\ub780\ub4dc \uce68\uacf5\uc774 \uc644\ub8cc\ub418\uc9c0 \uc54a\uc558\uc744 \ub54c \uc11c\ucabd\uc5d0\uc11c \uc5f0\ud569\uad70\uc774 \ub4a4\ud1b5\uc218\ub97c \uccd0\uc11c \ub3c5\uc77c\uc774 \uc591\uba74\uc804\uc7c1\uc5d0 \ube60\uc9c0\uac8c \ub418\ub294 \uac83\ub3c4 \uac71\uc815\uac70\ub9ac\uc600\ub2e4.", "answer": "\ube44\uc2a4\ud234\ub77c \uac15", "sentence": "\ub9cc\uc288\ud0c0\uc778\uc740 \uc9d1\ub2e8\uad70\uc758 \uae30\uac11\ubcd1\ub825 \ub2e4\uc218\ub97c \ubc1c\ud130 \ud3f0 \ub77c\uc774\ud5e4\ub098\uc6b0\uc758 \uc81c10\uad70\uc5d0 \uc9d1\uc911\uc2dc\ucf1c \uc8fc\uba74\uc11c \ub3cc\ud30c\uad6c\ub97c \ub9c8\ub828, \ud3f4\ub780\ub4dc\uad70\uc744 \ube44\uc2a4\ud234\ub77c \uac15 \uc11c\uc548\uc5d0 \ud3ec\uc704\uc2dc\ud0a4\uace0, \ub77c\uc774\ud5e4\ub098\uc6b0\uc758 \uc81c10\uad70\uc774 \uae30\uac11\uae30\ub3d9\uc758 \ud798\uc73c\ub85c \ud3f4\ub780\ub4dc\uc758 \uc218\ub3c4 \ubc14\ub974\uc0e4\ubc14\uae4c\uc9c0 \ub3cc\uc9c4\ud558\ub294 \ub3d9\uc548 \ube4c\ud5ec\ub984 \ub9ac\uc2a4\ud2b8\uc758 \uc81c14\uad70\uacfc \uc694\ud558\ub124\uc2a4 \ube14\ub77c\ucf54\ube44\uce20\uc758 \uc81c8\uad70\uc740 \uce21\uba74\uc9c0\uc6d0\uc744 \ub9e1\ub294\ub2e4\ub294 \uacc4\ud68d\uc744 \uc791\uc131\ud588\uace0, \ub8ec\ud2b8\uc288\ud14c\ud2b8\ub294 \uc774\ub97c \ubc1b\uc544\ub4e4\uc600\ub2e4.", "paragraph_sentence": "1939\ub144 8\uc6d4 18\uc77c, \ud3f4\ub780\ub4dc\ub97c \uce68\uacf5\ud558\ub294 \ubc31\uc0c9 \uc791\uc804\uc744 \uc900\ube44\ud558\ub358 \ub9cc\uc288\ud0c0\uc778\uc740 \uac8c\ub974\ud2b8 \ud3f0 \ub8ec\ud2b8\uc288\ud14c\ud2b8\uc758 \ub0a8\ubd80 \uc9d1\ub2e8\uad70 \ucc38\ubaa8\uc7a5\uc73c\ub85c \ubd80\uc784\ud55c\ub2e4. \ub9cc\uc288\ud0c0\uc778\uc740 \ub8ec\ud2b8\uc288\ud14c\ud2b8\uc758 \uc791\uc804\ucc38\ubaa8 \uadc4\ud130 \ube14\ub8e8\uba58\ud2b8\ub9ac\ud2b8 \ub300\ub839\uacfc \ud568\uaed8 \uc791\uc804\uacc4\ud68d \uc218\ub9bd\uc5d0 \ucc38\uc5ec\ud588\ub2e4. \ub9cc\uc288\ud0c0\uc778\uc740 \uc9d1\ub2e8\uad70\uc758 \uae30\uac11\ubcd1\ub825 \ub2e4\uc218\ub97c \ubc1c\ud130 \ud3f0 \ub77c\uc774\ud5e4\ub098\uc6b0\uc758 \uc81c10\uad70\uc5d0 \uc9d1\uc911\uc2dc\ucf1c \uc8fc\uba74\uc11c \ub3cc\ud30c\uad6c\ub97c \ub9c8\ub828, \ud3f4\ub780\ub4dc\uad70\uc744 \ube44\uc2a4\ud234\ub77c \uac15 \uc11c\uc548\uc5d0 \ud3ec\uc704\uc2dc\ud0a4\uace0, \ub77c\uc774\ud5e4\ub098\uc6b0\uc758 \uc81c10\uad70\uc774 \uae30\uac11\uae30\ub3d9\uc758 \ud798\uc73c\ub85c \ud3f4\ub780\ub4dc\uc758 \uc218\ub3c4 \ubc14\ub974\uc0e4\ubc14\uae4c\uc9c0 \ub3cc\uc9c4\ud558\ub294 \ub3d9\uc548 \ube4c\ud5ec\ub984 \ub9ac\uc2a4\ud2b8\uc758 \uc81c14\uad70\uacfc \uc694\ud558\ub124\uc2a4 \ube14\ub77c\ucf54\ube44\uce20\uc758 \uc81c8\uad70\uc740 \uce21\uba74\uc9c0\uc6d0\uc744 \ub9e1\ub294\ub2e4\ub294 \uacc4\ud68d\uc744 \uc791\uc131\ud588\uace0, \ub8ec\ud2b8\uc288\ud14c\ud2b8\ub294 \uc774\ub97c \ubc1b\uc544\ub4e4\uc600\ub2e4. \uc0ac\uc2e4 \ub9cc\uc288\ud0c0\uc778\uc740 \uac1c\uc778\uc801\uc73c\ub85c \ud3f4\ub780\ub4dc\ub97c \ub3c5\uc77c\uacfc \uc18c\ub828 \uc0ac\uc774\uc758 \uc644\ucda9\uc9c0\ub300\ub85c \ub0a8\uaca8\ub450\ub294 \ud3b8\uc774 \uc88b\ub2e4\uace0 \uc0dd\uac01\ud588\uae30\uc5d0 \ub300\ud3f4\ub780\ub4dc \uad70\uc0ac \uc791\uc804\uc5d0 \ubbf8\uc628\uc801\uc774\uc5c8\ub2e4. \ubfd0\ub9cc \uc544\ub2c8\ub77c \ud3f4\ub780\ub4dc \uce68\uacf5\uc774 \uc644\ub8cc\ub418\uc9c0 \uc54a\uc558\uc744 \ub54c \uc11c\ucabd\uc5d0\uc11c \uc5f0\ud569\uad70\uc774 \ub4a4\ud1b5\uc218\ub97c \uccd0\uc11c \ub3c5\uc77c\uc774 \uc591\uba74\uc804\uc7c1\uc5d0 \ube60\uc9c0\uac8c \ub418\ub294 \uac83\ub3c4 \uac71\uc815\uac70\ub9ac\uc600\ub2e4.", "paragraph_answer": "1939\ub144 8\uc6d4 18\uc77c, \ud3f4\ub780\ub4dc\ub97c \uce68\uacf5\ud558\ub294 \ubc31\uc0c9 \uc791\uc804\uc744 \uc900\ube44\ud558\ub358 \ub9cc\uc288\ud0c0\uc778\uc740 \uac8c\ub974\ud2b8 \ud3f0 \ub8ec\ud2b8\uc288\ud14c\ud2b8\uc758 \ub0a8\ubd80 \uc9d1\ub2e8\uad70 \ucc38\ubaa8\uc7a5\uc73c\ub85c \ubd80\uc784\ud55c\ub2e4. \ub9cc\uc288\ud0c0\uc778\uc740 \ub8ec\ud2b8\uc288\ud14c\ud2b8\uc758 \uc791\uc804\ucc38\ubaa8 \uadc4\ud130 \ube14\ub8e8\uba58\ud2b8\ub9ac\ud2b8 \ub300\ub839\uacfc \ud568\uaed8 \uc791\uc804\uacc4\ud68d \uc218\ub9bd\uc5d0 \ucc38\uc5ec\ud588\ub2e4. \ub9cc\uc288\ud0c0\uc778\uc740 \uc9d1\ub2e8\uad70\uc758 \uae30\uac11\ubcd1\ub825 \ub2e4\uc218\ub97c \ubc1c\ud130 \ud3f0 \ub77c\uc774\ud5e4\ub098\uc6b0\uc758 \uc81c10\uad70\uc5d0 \uc9d1\uc911\uc2dc\ucf1c \uc8fc\uba74\uc11c \ub3cc\ud30c\uad6c\ub97c \ub9c8\ub828, \ud3f4\ub780\ub4dc\uad70\uc744 \ube44\uc2a4\ud234\ub77c \uac15 \uc11c\uc548\uc5d0 \ud3ec\uc704\uc2dc\ud0a4\uace0, \ub77c\uc774\ud5e4\ub098\uc6b0\uc758 \uc81c10\uad70\uc774 \uae30\uac11\uae30\ub3d9\uc758 \ud798\uc73c\ub85c \ud3f4\ub780\ub4dc\uc758 \uc218\ub3c4 \ubc14\ub974\uc0e4\ubc14\uae4c\uc9c0 \ub3cc\uc9c4\ud558\ub294 \ub3d9\uc548 \ube4c\ud5ec\ub984 \ub9ac\uc2a4\ud2b8\uc758 \uc81c14\uad70\uacfc \uc694\ud558\ub124\uc2a4 \ube14\ub77c\ucf54\ube44\uce20\uc758 \uc81c8\uad70\uc740 \uce21\uba74\uc9c0\uc6d0\uc744 \ub9e1\ub294\ub2e4\ub294 \uacc4\ud68d\uc744 \uc791\uc131\ud588\uace0, \ub8ec\ud2b8\uc288\ud14c\ud2b8\ub294 \uc774\ub97c \ubc1b\uc544\ub4e4\uc600\ub2e4. \uc0ac\uc2e4 \ub9cc\uc288\ud0c0\uc778\uc740 \uac1c\uc778\uc801\uc73c\ub85c \ud3f4\ub780\ub4dc\ub97c \ub3c5\uc77c\uacfc \uc18c\ub828 \uc0ac\uc774\uc758 \uc644\ucda9\uc9c0\ub300\ub85c \ub0a8\uaca8\ub450\ub294 \ud3b8\uc774 \uc88b\ub2e4\uace0 \uc0dd\uac01\ud588\uae30\uc5d0 \ub300\ud3f4\ub780\ub4dc \uad70\uc0ac \uc791\uc804\uc5d0 \ubbf8\uc628\uc801\uc774\uc5c8\ub2e4. \ubfd0\ub9cc \uc544\ub2c8\ub77c \ud3f4\ub780\ub4dc \uce68\uacf5\uc774 \uc644\ub8cc\ub418\uc9c0 \uc54a\uc558\uc744 \ub54c \uc11c\ucabd\uc5d0\uc11c \uc5f0\ud569\uad70\uc774 \ub4a4\ud1b5\uc218\ub97c \uccd0\uc11c \ub3c5\uc77c\uc774 \uc591\uba74\uc804\uc7c1\uc5d0 \ube60\uc9c0\uac8c \ub418\ub294 \uac83\ub3c4 \uac71\uc815\uac70\ub9ac\uc600\ub2e4.", "sentence_answer": "\ub9cc\uc288\ud0c0\uc778\uc740 \uc9d1\ub2e8\uad70\uc758 \uae30\uac11\ubcd1\ub825 \ub2e4\uc218\ub97c \ubc1c\ud130 \ud3f0 \ub77c\uc774\ud5e4\ub098\uc6b0\uc758 \uc81c10\uad70\uc5d0 \uc9d1\uc911\uc2dc\ucf1c \uc8fc\uba74\uc11c \ub3cc\ud30c\uad6c\ub97c \ub9c8\ub828, \ud3f4\ub780\ub4dc\uad70\uc744 \ube44\uc2a4\ud234\ub77c \uac15 \uc11c\uc548\uc5d0 \ud3ec\uc704\uc2dc\ud0a4\uace0, \ub77c\uc774\ud5e4\ub098\uc6b0\uc758 \uc81c10\uad70\uc774 \uae30\uac11\uae30\ub3d9\uc758 \ud798\uc73c\ub85c \ud3f4\ub780\ub4dc\uc758 \uc218\ub3c4 \ubc14\ub974\uc0e4\ubc14\uae4c\uc9c0 \ub3cc\uc9c4\ud558\ub294 \ub3d9\uc548 \ube4c\ud5ec\ub984 \ub9ac\uc2a4\ud2b8\uc758 \uc81c14\uad70\uacfc \uc694\ud558\ub124\uc2a4 \ube14\ub77c\ucf54\ube44\uce20\uc758 \uc81c8\uad70\uc740 \uce21\uba74\uc9c0\uc6d0\uc744 \ub9e1\ub294\ub2e4\ub294 \uacc4\ud68d\uc744 \uc791\uc131\ud588\uace0, \ub8ec\ud2b8\uc288\ud14c\ud2b8\ub294 \uc774\ub97c \ubc1b\uc544\ub4e4\uc600\ub2e4.", "paragraph_id": "6586570-9-2", "paragraph_question": "question: \ub9cc\uc288\ud0c0\uc778\uc740 \ud3f0\ub780\ub4dc\uad70\uc744 \uc5b4\ub290 \uac15\uc758 \uc11c\uc548\uc5d0 \ud3ec\uc704\uc2dc\ud0a4\ub824\uace0 \uacc4\ud68d \ud588\ub294\uac00?, context: 1939\ub144 8\uc6d4 18\uc77c, \ud3f4\ub780\ub4dc\ub97c \uce68\uacf5\ud558\ub294 \ubc31\uc0c9 \uc791\uc804\uc744 \uc900\ube44\ud558\ub358 \ub9cc\uc288\ud0c0\uc778\uc740 \uac8c\ub974\ud2b8 \ud3f0 \ub8ec\ud2b8\uc288\ud14c\ud2b8\uc758 \ub0a8\ubd80 \uc9d1\ub2e8\uad70 \ucc38\ubaa8\uc7a5\uc73c\ub85c \ubd80\uc784\ud55c\ub2e4. \ub9cc\uc288\ud0c0\uc778\uc740 \ub8ec\ud2b8\uc288\ud14c\ud2b8\uc758 \uc791\uc804\ucc38\ubaa8 \uadc4\ud130 \ube14\ub8e8\uba58\ud2b8\ub9ac\ud2b8 \ub300\ub839\uacfc \ud568\uaed8 \uc791\uc804\uacc4\ud68d \uc218\ub9bd\uc5d0 \ucc38\uc5ec\ud588\ub2e4. \ub9cc\uc288\ud0c0\uc778\uc740 \uc9d1\ub2e8\uad70\uc758 \uae30\uac11\ubcd1\ub825 \ub2e4\uc218\ub97c \ubc1c\ud130 \ud3f0 \ub77c\uc774\ud5e4\ub098\uc6b0\uc758 \uc81c10\uad70\uc5d0 \uc9d1\uc911\uc2dc\ucf1c \uc8fc\uba74\uc11c \ub3cc\ud30c\uad6c\ub97c \ub9c8\ub828, \ud3f4\ub780\ub4dc\uad70\uc744 \ube44\uc2a4\ud234\ub77c \uac15 \uc11c\uc548\uc5d0 \ud3ec\uc704\uc2dc\ud0a4\uace0, \ub77c\uc774\ud5e4\ub098\uc6b0\uc758 \uc81c10\uad70\uc774 \uae30\uac11\uae30\ub3d9\uc758 \ud798\uc73c\ub85c \ud3f4\ub780\ub4dc\uc758 \uc218\ub3c4 \ubc14\ub974\uc0e4\ubc14\uae4c\uc9c0 \ub3cc\uc9c4\ud558\ub294 \ub3d9\uc548 \ube4c\ud5ec\ub984 \ub9ac\uc2a4\ud2b8\uc758 \uc81c14\uad70\uacfc \uc694\ud558\ub124\uc2a4 \ube14\ub77c\ucf54\ube44\uce20\uc758 \uc81c8\uad70\uc740 \uce21\uba74\uc9c0\uc6d0\uc744 \ub9e1\ub294\ub2e4\ub294 \uacc4\ud68d\uc744 \uc791\uc131\ud588\uace0, \ub8ec\ud2b8\uc288\ud14c\ud2b8\ub294 \uc774\ub97c \ubc1b\uc544\ub4e4\uc600\ub2e4. \uc0ac\uc2e4 \ub9cc\uc288\ud0c0\uc778\uc740 \uac1c\uc778\uc801\uc73c\ub85c \ud3f4\ub780\ub4dc\ub97c \ub3c5\uc77c\uacfc \uc18c\ub828 \uc0ac\uc774\uc758 \uc644\ucda9\uc9c0\ub300\ub85c \ub0a8\uaca8\ub450\ub294 \ud3b8\uc774 \uc88b\ub2e4\uace0 \uc0dd\uac01\ud588\uae30\uc5d0 \ub300\ud3f4\ub780\ub4dc \uad70\uc0ac \uc791\uc804\uc5d0 \ubbf8\uc628\uc801\uc774\uc5c8\ub2e4. \ubfd0\ub9cc \uc544\ub2c8\ub77c \ud3f4\ub780\ub4dc \uce68\uacf5\uc774 \uc644\ub8cc\ub418\uc9c0 \uc54a\uc558\uc744 \ub54c \uc11c\ucabd\uc5d0\uc11c \uc5f0\ud569\uad70\uc774 \ub4a4\ud1b5\uc218\ub97c \uccd0\uc11c \ub3c5\uc77c\uc774 \uc591\uba74\uc804\uc7c1\uc5d0 \ube60\uc9c0\uac8c \ub418\ub294 \uac83\ub3c4 \uac71\uc815\uac70\ub9ac\uc600\ub2e4."} {"question": "\ubbf8\ud558\uc77c \uace0\ub974\ubc14\ucd08\ud504\uac00 \ub300\ud1b5\ub839\uc744 \uc5ed\uc784\ud55c \uad6d\uac00\ub294?", "paragraph": "1990\ub144 \ub178\ubca8 \ud3c9\ud654\uc0c1 \uc218\uc0c1\uc790\uc778 \ubbf8\ud558\uc77c \uace0\ub974\ubc14\ucd08\ud504 \uc804 \uc18c\ub828 \ub300\ud1b5\ub839\uc740 \uc11c\ud55c\uc73c\ub85c \"\uc800\uba85\ud55c \uc815\uce58\uc778\uc774\uc790 \ub178\ubca8\ud3c9\ud654\uc0c1 \uc218\uc0c1\uc790\uc778 \uc804 \uae40 \uc804 \ub300\ud1b5\ub839\uc758 \uc11c\uac70 \uc18c\uc2dd\uc744 \uc804\ud574 \ub4e4\uc5b4 \ub9e4\uc6b0 \uc560\ud1b5\ud558\ub2e4\"\ub77c\uace0 \ub9d0\ud588\ub2e4. \uace0\ub974\ubc14\ucd08\ud504 \uc804 \ub300\ud1b5\ub839\uc740 \uce5c\ud544\ub85c \uc11c\uba85\ud55c \uc774 \uc11c\ud55c\uc5d0\uc11c \"\uae40 \uc804 \ub300\ud1b5\ub839\uc740 \ud55c\uad6d\uc758 \uc778\uad8c\uacfc \ubbfc\uc8fc\uc8fc\uc758 \ud615\uc131, \ud55c\ubc18\ub3c4\uc758 \ud3c9\ud654\uc870\uc131\uc5d0 \ud070 \uae30\uc5ec\ub97c \ud588\ub2e4. \uadf8\uc758 \uc815\uce58\uc801 \ud589\ubcf4\ub294 \ud3c9\ud654, \ubbfc\uc8fc\uc8fc\uc758, \uc790\uc720, \uc778\uad8c\uc744 \ubc14\ud0d5\uc73c\ub85c \ud55c \uac83\uc73c\ub85c, \uadf8\ub7ac\uae30\uc5d0 \uadf8\ub294 \ub300\ud1b5\ub839\uc5d0 \ub2f9\uc120\ub41c \uac83\"\uc774\ub77c\uace0 \uc804\ud588\ub2e4. \ud2b9\ud788 \"\uc624\ub798\uc804\ubd80\ud130 \uae40 \uc804 \ub300\ud1b5\ub839\uc744 \uc54c\uc544\uc654\uace0 \uadf8\uc758 \uc6a9\uae30\uc640 \uc120\uacac\uc9c0\uba85\uc744 \ub9e4\uc6b0 \ub192\uac8c \uc0ac \uc654\ub2e4\"\uba70 \"\uadf8\ub294 \ub300\ud1b5\ub839 \uc784\uae30 \ub9d0 \uac74\uac15\uc5d0 \ubb38\uc81c\ub97c \uacaa\uc73c\uba74\uc11c\ub3c4 \ud55c\ubc18\ub3c4\ub97c \uc7ac\uacb0\ud569\uc2dc\ud0a4\uba74\uc11c \ub0a8\ubd81\ubb38\uc81c \ud574\uacb0\ucc45\uc744 \uacc4\uc18d\ud574\uc11c \ucc3e\uc558\ub2e4\"\ub77c\uace0 \ud68c\uace0\ud588\ub2e4. \uadf8\ub294 \ub610 \"\uae40 \uc804 \ub300\ud1b5\ub839\uc740 \uc6b0\ub9ac\uc758 \ub178\ubca8\ud3c9\ud654\uc0c1 \uc218\uc0c1\uc790 \ud3ec\ub7fc \uacf5\ub3d9 \uc5c5\ubb34\uc5d0\ub3c4 \ub9ce\uc740 \ud798\uc744 \uc3df\uc558\ub2e4\"\uba70 \"\uae40 \uc804 \ub300\ud1b5\ub839\uc758 \uc720\uc871\uacfc \uc9c0\uc778\ub4e4, \ubaa8\ub4e0 \ud55c\uad6d \uad6d\ubbfc\uc5d0\uac8c \uae4a\uc740 \uc560\ub3c4\uc758 \ub73b\uc744 \ud45c\ud55c\ub2e4\u201d\uace0 \ub367\ubd99\uc600\ub2e4.", "answer": "\uc18c\ub828", "sentence": "1990\ub144 \ub178\ubca8 \ud3c9\ud654\uc0c1 \uc218\uc0c1\uc790\uc778 \ubbf8\ud558\uc77c \uace0\ub974\ubc14\ucd08\ud504 \uc804 \uc18c\ub828 \ub300\ud1b5\ub839\uc740 \uc11c\ud55c\uc73c\ub85c \"\uc800\uba85\ud55c \uc815\uce58\uc778\uc774\uc790 \ub178\ubca8\ud3c9\ud654\uc0c1 \uc218\uc0c1\uc790\uc778 \uc804 \uae40 \uc804 \ub300\ud1b5\ub839\uc758 \uc11c\uac70 \uc18c\uc2dd\uc744 \uc804\ud574 \ub4e4\uc5b4 \ub9e4\uc6b0 \uc560\ud1b5\ud558\ub2e4\"\ub77c\uace0 \ub9d0\ud588\ub2e4.", "paragraph_sentence": " 1990\ub144 \ub178\ubca8 \ud3c9\ud654\uc0c1 \uc218\uc0c1\uc790\uc778 \ubbf8\ud558\uc77c \uace0\ub974\ubc14\ucd08\ud504 \uc804 \uc18c\ub828 \ub300\ud1b5\ub839\uc740 \uc11c\ud55c\uc73c\ub85c \"\uc800\uba85\ud55c \uc815\uce58\uc778\uc774\uc790 \ub178\ubca8\ud3c9\ud654\uc0c1 \uc218\uc0c1\uc790\uc778 \uc804 \uae40 \uc804 \ub300\ud1b5\ub839\uc758 \uc11c\uac70 \uc18c\uc2dd\uc744 \uc804\ud574 \ub4e4\uc5b4 \ub9e4\uc6b0 \uc560\ud1b5\ud558\ub2e4\"\ub77c\uace0 \ub9d0\ud588\ub2e4. \uace0\ub974\ubc14\ucd08\ud504 \uc804 \ub300\ud1b5\ub839\uc740 \uce5c\ud544\ub85c \uc11c\uba85\ud55c \uc774 \uc11c\ud55c\uc5d0\uc11c \"\uae40 \uc804 \ub300\ud1b5\ub839\uc740 \ud55c\uad6d\uc758 \uc778\uad8c\uacfc \ubbfc\uc8fc\uc8fc\uc758 \ud615\uc131, \ud55c\ubc18\ub3c4\uc758 \ud3c9\ud654\uc870\uc131\uc5d0 \ud070 \uae30\uc5ec\ub97c \ud588\ub2e4. \uadf8\uc758 \uc815\uce58\uc801 \ud589\ubcf4\ub294 \ud3c9\ud654, \ubbfc\uc8fc\uc8fc\uc758, \uc790\uc720, \uc778\uad8c\uc744 \ubc14\ud0d5\uc73c\ub85c \ud55c \uac83\uc73c\ub85c, \uadf8\ub7ac\uae30\uc5d0 \uadf8\ub294 \ub300\ud1b5\ub839\uc5d0 \ub2f9\uc120\ub41c \uac83\"\uc774\ub77c\uace0 \uc804\ud588\ub2e4. \ud2b9\ud788 \"\uc624\ub798\uc804\ubd80\ud130 \uae40 \uc804 \ub300\ud1b5\ub839\uc744 \uc54c\uc544\uc654\uace0 \uadf8\uc758 \uc6a9\uae30\uc640 \uc120\uacac\uc9c0\uba85\uc744 \ub9e4\uc6b0 \ub192\uac8c \uc0ac \uc654\ub2e4\"\uba70 \"\uadf8\ub294 \ub300\ud1b5\ub839 \uc784\uae30 \ub9d0 \uac74\uac15\uc5d0 \ubb38\uc81c\ub97c \uacaa\uc73c\uba74\uc11c\ub3c4 \ud55c\ubc18\ub3c4\ub97c \uc7ac\uacb0\ud569\uc2dc\ud0a4\uba74\uc11c \ub0a8\ubd81\ubb38\uc81c \ud574\uacb0\ucc45\uc744 \uacc4\uc18d\ud574\uc11c \ucc3e\uc558\ub2e4\"\ub77c\uace0 \ud68c\uace0\ud588\ub2e4. \uadf8\ub294 \ub610 \"\uae40 \uc804 \ub300\ud1b5\ub839\uc740 \uc6b0\ub9ac\uc758 \ub178\ubca8\ud3c9\ud654\uc0c1 \uc218\uc0c1\uc790 \ud3ec\ub7fc \uacf5\ub3d9 \uc5c5\ubb34\uc5d0\ub3c4 \ub9ce\uc740 \ud798\uc744 \uc3df\uc558\ub2e4\"\uba70 \"\uae40 \uc804 \ub300\ud1b5\ub839\uc758 \uc720\uc871\uacfc \uc9c0\uc778\ub4e4, \ubaa8\ub4e0 \ud55c\uad6d \uad6d\ubbfc\uc5d0\uac8c \uae4a\uc740 \uc560\ub3c4\uc758 \ub73b\uc744 \ud45c\ud55c\ub2e4\u201d\uace0 \ub367\ubd99\uc600\ub2e4.", "paragraph_answer": "1990\ub144 \ub178\ubca8 \ud3c9\ud654\uc0c1 \uc218\uc0c1\uc790\uc778 \ubbf8\ud558\uc77c \uace0\ub974\ubc14\ucd08\ud504 \uc804 \uc18c\ub828 \ub300\ud1b5\ub839\uc740 \uc11c\ud55c\uc73c\ub85c \"\uc800\uba85\ud55c \uc815\uce58\uc778\uc774\uc790 \ub178\ubca8\ud3c9\ud654\uc0c1 \uc218\uc0c1\uc790\uc778 \uc804 \uae40 \uc804 \ub300\ud1b5\ub839\uc758 \uc11c\uac70 \uc18c\uc2dd\uc744 \uc804\ud574 \ub4e4\uc5b4 \ub9e4\uc6b0 \uc560\ud1b5\ud558\ub2e4\"\ub77c\uace0 \ub9d0\ud588\ub2e4. \uace0\ub974\ubc14\ucd08\ud504 \uc804 \ub300\ud1b5\ub839\uc740 \uce5c\ud544\ub85c \uc11c\uba85\ud55c \uc774 \uc11c\ud55c\uc5d0\uc11c \"\uae40 \uc804 \ub300\ud1b5\ub839\uc740 \ud55c\uad6d\uc758 \uc778\uad8c\uacfc \ubbfc\uc8fc\uc8fc\uc758 \ud615\uc131, \ud55c\ubc18\ub3c4\uc758 \ud3c9\ud654\uc870\uc131\uc5d0 \ud070 \uae30\uc5ec\ub97c \ud588\ub2e4. \uadf8\uc758 \uc815\uce58\uc801 \ud589\ubcf4\ub294 \ud3c9\ud654, \ubbfc\uc8fc\uc8fc\uc758, \uc790\uc720, \uc778\uad8c\uc744 \ubc14\ud0d5\uc73c\ub85c \ud55c \uac83\uc73c\ub85c, \uadf8\ub7ac\uae30\uc5d0 \uadf8\ub294 \ub300\ud1b5\ub839\uc5d0 \ub2f9\uc120\ub41c \uac83\"\uc774\ub77c\uace0 \uc804\ud588\ub2e4. \ud2b9\ud788 \"\uc624\ub798\uc804\ubd80\ud130 \uae40 \uc804 \ub300\ud1b5\ub839\uc744 \uc54c\uc544\uc654\uace0 \uadf8\uc758 \uc6a9\uae30\uc640 \uc120\uacac\uc9c0\uba85\uc744 \ub9e4\uc6b0 \ub192\uac8c \uc0ac \uc654\ub2e4\"\uba70 \"\uadf8\ub294 \ub300\ud1b5\ub839 \uc784\uae30 \ub9d0 \uac74\uac15\uc5d0 \ubb38\uc81c\ub97c \uacaa\uc73c\uba74\uc11c\ub3c4 \ud55c\ubc18\ub3c4\ub97c \uc7ac\uacb0\ud569\uc2dc\ud0a4\uba74\uc11c \ub0a8\ubd81\ubb38\uc81c \ud574\uacb0\ucc45\uc744 \uacc4\uc18d\ud574\uc11c \ucc3e\uc558\ub2e4\"\ub77c\uace0 \ud68c\uace0\ud588\ub2e4. \uadf8\ub294 \ub610 \"\uae40 \uc804 \ub300\ud1b5\ub839\uc740 \uc6b0\ub9ac\uc758 \ub178\ubca8\ud3c9\ud654\uc0c1 \uc218\uc0c1\uc790 \ud3ec\ub7fc \uacf5\ub3d9 \uc5c5\ubb34\uc5d0\ub3c4 \ub9ce\uc740 \ud798\uc744 \uc3df\uc558\ub2e4\"\uba70 \"\uae40 \uc804 \ub300\ud1b5\ub839\uc758 \uc720\uc871\uacfc \uc9c0\uc778\ub4e4, \ubaa8\ub4e0 \ud55c\uad6d \uad6d\ubbfc\uc5d0\uac8c \uae4a\uc740 \uc560\ub3c4\uc758 \ub73b\uc744 \ud45c\ud55c\ub2e4\u201d\uace0 \ub367\ubd99\uc600\ub2e4.", "sentence_answer": "1990\ub144 \ub178\ubca8 \ud3c9\ud654\uc0c1 \uc218\uc0c1\uc790\uc778 \ubbf8\ud558\uc77c \uace0\ub974\ubc14\ucd08\ud504 \uc804 \uc18c\ub828 \ub300\ud1b5\ub839\uc740 \uc11c\ud55c\uc73c\ub85c \"\uc800\uba85\ud55c \uc815\uce58\uc778\uc774\uc790 \ub178\ubca8\ud3c9\ud654\uc0c1 \uc218\uc0c1\uc790\uc778 \uc804 \uae40 \uc804 \ub300\ud1b5\ub839\uc758 \uc11c\uac70 \uc18c\uc2dd\uc744 \uc804\ud574 \ub4e4\uc5b4 \ub9e4\uc6b0 \uc560\ud1b5\ud558\ub2e4\"\ub77c\uace0 \ub9d0\ud588\ub2e4.", "paragraph_id": "6488203-48-1", "paragraph_question": "question: \ubbf8\ud558\uc77c \uace0\ub974\ubc14\ucd08\ud504\uac00 \ub300\ud1b5\ub839\uc744 \uc5ed\uc784\ud55c \uad6d\uac00\ub294?, context: 1990\ub144 \ub178\ubca8 \ud3c9\ud654\uc0c1 \uc218\uc0c1\uc790\uc778 \ubbf8\ud558\uc77c \uace0\ub974\ubc14\ucd08\ud504 \uc804 \uc18c\ub828 \ub300\ud1b5\ub839\uc740 \uc11c\ud55c\uc73c\ub85c \"\uc800\uba85\ud55c \uc815\uce58\uc778\uc774\uc790 \ub178\ubca8\ud3c9\ud654\uc0c1 \uc218\uc0c1\uc790\uc778 \uc804 \uae40 \uc804 \ub300\ud1b5\ub839\uc758 \uc11c\uac70 \uc18c\uc2dd\uc744 \uc804\ud574 \ub4e4\uc5b4 \ub9e4\uc6b0 \uc560\ud1b5\ud558\ub2e4\"\ub77c\uace0 \ub9d0\ud588\ub2e4. \uace0\ub974\ubc14\ucd08\ud504 \uc804 \ub300\ud1b5\ub839\uc740 \uce5c\ud544\ub85c \uc11c\uba85\ud55c \uc774 \uc11c\ud55c\uc5d0\uc11c \"\uae40 \uc804 \ub300\ud1b5\ub839\uc740 \ud55c\uad6d\uc758 \uc778\uad8c\uacfc \ubbfc\uc8fc\uc8fc\uc758 \ud615\uc131, \ud55c\ubc18\ub3c4\uc758 \ud3c9\ud654\uc870\uc131\uc5d0 \ud070 \uae30\uc5ec\ub97c \ud588\ub2e4. \uadf8\uc758 \uc815\uce58\uc801 \ud589\ubcf4\ub294 \ud3c9\ud654, \ubbfc\uc8fc\uc8fc\uc758, \uc790\uc720, \uc778\uad8c\uc744 \ubc14\ud0d5\uc73c\ub85c \ud55c \uac83\uc73c\ub85c, \uadf8\ub7ac\uae30\uc5d0 \uadf8\ub294 \ub300\ud1b5\ub839\uc5d0 \ub2f9\uc120\ub41c \uac83\"\uc774\ub77c\uace0 \uc804\ud588\ub2e4. \ud2b9\ud788 \"\uc624\ub798\uc804\ubd80\ud130 \uae40 \uc804 \ub300\ud1b5\ub839\uc744 \uc54c\uc544\uc654\uace0 \uadf8\uc758 \uc6a9\uae30\uc640 \uc120\uacac\uc9c0\uba85\uc744 \ub9e4\uc6b0 \ub192\uac8c \uc0ac \uc654\ub2e4\"\uba70 \"\uadf8\ub294 \ub300\ud1b5\ub839 \uc784\uae30 \ub9d0 \uac74\uac15\uc5d0 \ubb38\uc81c\ub97c \uacaa\uc73c\uba74\uc11c\ub3c4 \ud55c\ubc18\ub3c4\ub97c \uc7ac\uacb0\ud569\uc2dc\ud0a4\uba74\uc11c \ub0a8\ubd81\ubb38\uc81c \ud574\uacb0\ucc45\uc744 \uacc4\uc18d\ud574\uc11c \ucc3e\uc558\ub2e4\"\ub77c\uace0 \ud68c\uace0\ud588\ub2e4. \uadf8\ub294 \ub610 \"\uae40 \uc804 \ub300\ud1b5\ub839\uc740 \uc6b0\ub9ac\uc758 \ub178\ubca8\ud3c9\ud654\uc0c1 \uc218\uc0c1\uc790 \ud3ec\ub7fc \uacf5\ub3d9 \uc5c5\ubb34\uc5d0\ub3c4 \ub9ce\uc740 \ud798\uc744 \uc3df\uc558\ub2e4\"\uba70 \"\uae40 \uc804 \ub300\ud1b5\ub839\uc758 \uc720\uc871\uacfc \uc9c0\uc778\ub4e4, \ubaa8\ub4e0 \ud55c\uad6d \uad6d\ubbfc\uc5d0\uac8c \uae4a\uc740 \uc560\ub3c4\uc758 \ub73b\uc744 \ud45c\ud55c\ub2e4\u201d\uace0 \ub367\ubd99\uc600\ub2e4."} {"question": "\uc601\uad6d\uc774 \uc720\ub7fd\uacbd\uc81c\uacf5\ub3d9\uccb4\uc5d0 \uc815\uc2dd \uac00\uc785\ud55c \uac83\uc740 \uc5b8\uc81c\ubd80\ud130 \uc778\uac00?", "paragraph": "\uc774 \ub098\ub77c\ub294 \ub2e4\ub978 \uc5ec\ub7ec \ub098\ub77c\uc5d0 \uc55e\uc11c\uc11c \uc0b0\uc5c5\ud601\uba85\uc744 \uc131\ucde8\ud558\uc5ec \uc624\ub7ab\ub3d9\uc548 \uc138\uacc4 \uacbd\uc81c\uc5d0 \uad70\ub9bc\ud574 \uc654\ub2e4. \uadf8\ub7ec\ub098 20\uc138\uae30\uc5d0 \ub4e4\uc5b4\uc640\uc11c\ub294 \uc720\ub7fd \ub300\ub959\uacfc \ubd81\uc544\uba54\ub9ac\uce74\uc758 \uacf5\uc5c5\uc774 \uae09\uc18d\ub3c4\ub85c \ubc1c\ub2ec\ud558\uc5ec 1\ucc28\ub300\uc804 \uc774\ud6c4 \uc138\uacc4 \uacbd\uc81c\uc5d0 \uc788\uc5b4\uc11c\uc758 \uc9c0\uc704\ub294 \ucc28\ucc28 \uc800\ud558\ud588\ub2e4. \ud574\uc678 \ud22c\uc790\ub85c\ubd80\ud130\uc758 \uc218\uc785\uc73c\ub85c \uacbd\uc81c\uc758 \uc548\uc815\uc744 \uaf80\ud588\uc73c\ub098 2\ucc28\ub300\uc804\uc5d0 \uc758\ud574 \ud070 \ud0c0\uaca9\uc744 \ubc1b\uace0 \uc7ac\ud3b8\uc131\ud558\uae30\uc5d0 \uc774\ub974\ub800\ub2e4. \uadf8\uac83\uc740 \uae30\uac04\uc0b0\uc5c5\uc758 \uad6d\uc720\ud654\ub97c \ube44\ub86f\ud55c \uacc4\ud68d\uacbd\uc81c\uc758 \ub3c4\uc785\uc73c\ub85c \uad6d\uc81c\uc218\uc9c0\ub97c \uac1c\uc120\ud558\uace0 \uc0ac\ud68c\ubcf4\uc7a5\uc81c\ub3c4\ub97c \ud655\ucda9\ud558\uc5ec \ubcf5\uc9c0 \uad6d\uac00\uc758 \uc2e4\ud604\uc744 \uaf80\ud558\ub294 \uac83\uc744 \uc8fc\uc694 \ubaa9\uc801\uc73c\ub85c \ud558\ub294 \uacbd\uc81c \uccb4\uc81c\uc774\ub2e4. \uadf8\ub7ec\ub098 \ub192\uc740 \uace0\uc6a9\uc218\uc900\uc744 \uc720\uc9c0\ud558\uba74\uc11c\ub3c4 \ub300\uc678\uc801\uc73c\ub85c\ub294 \uad6d\uc81c \uc218\uc9c0\uc758 \uc545\ud654, \ub300\ub0b4\uc801\uc73c\ub85c\ub294 \ucf54\uc2a4\ud2b8 \uc778\ud50c\ub808\uc774\uc158\uc5d0 \uc758\ud55c \uad34\ub85c\uc6c0\uc744 \ub2f9\ud558\uace0 \uc788\ub2e4. \uc774\ub7ec\ud55c \uc0c1\ud669\uc744 \ud0c0\uac1c\ud558\uae30 \uc704\ud574 \uc601\uad6d\uc740 1961\ub144\ubd80\ud130 \uc720\ub7fd\uacbd\uc81c\uacf5\ub3d9\uccb4(EEC) \uac00\uc785\uc744 \uc2e0\uccad\ud588\uc73c\ub098 \ud504\ub791\uc2a4\uc758 \uac70\ubd80\ub85c \uc2e4\ud604\uc744 \ubcf4\uc9c0 \ubabb\ud558\ub2e4\uac00 1971\ub144 6\uc6d4 \ub9c8\uce68\ub0b4 \uc218\ucd9c\uc785\uc815\ucc45 \ubc0f \ud30c\uc6b4\ub4dc\ud654(\u8ca8) \ubb38\uc81c\uc5d0 \ub9e4\ub4ed\uc744 \uc9d3\uace0 \uc601\u00b7\ubd88 \uc591\uad6d\uac04\uc5d0 \ucd5c\uc885 \ud569\uc758\uc5d0 \ub3c4\ub2ec\ud568\uc73c\ub85c\uc368 1973\ub144 1\uc6d4\ubd80\ud130 \uc815\uc2dd \uac00\uc785(\uacbd\uacfc\uae30\uac04 5\ub144\uc744 \uacc4\uc0b0\ud558\uba74 \uc2e4\uc9c8\uc801\uc778 \uac00\uc785\uc740 1978\ub144\ubd80\ud130\uac00 \ub428)\ud558\uac8c \ub418\uc5c8\ub2e4.", "answer": "1973\ub144 1\uc6d4", "sentence": "\uc774\ub7ec\ud55c \uc0c1\ud669\uc744 \ud0c0\uac1c\ud558\uae30 \uc704\ud574 \uc601\uad6d\uc740 1961\ub144\ubd80\ud130 \uc720\ub7fd\uacbd\uc81c\uacf5\ub3d9\uccb4(EEC) \uac00\uc785\uc744 \uc2e0\uccad\ud588\uc73c\ub098 \ud504\ub791\uc2a4\uc758 \uac70\ubd80\ub85c \uc2e4\ud604\uc744 \ubcf4\uc9c0 \ubabb\ud558\ub2e4\uac00 1971\ub144 6\uc6d4 \ub9c8\uce68\ub0b4 \uc218\ucd9c\uc785\uc815\ucc45 \ubc0f \ud30c\uc6b4\ub4dc\ud654(\u8ca8) \ubb38\uc81c\uc5d0 \ub9e4\ub4ed\uc744 \uc9d3\uace0 \uc601\u00b7\ubd88 \uc591\uad6d\uac04\uc5d0 \ucd5c\uc885 \ud569\uc758\uc5d0 \ub3c4\ub2ec\ud568\uc73c\ub85c\uc368 1973\ub144 1\uc6d4 \ubd80\ud130 \uc815\uc2dd \uac00\uc785(\uacbd\uacfc\uae30\uac04 5\ub144\uc744 \uacc4\uc0b0\ud558\uba74 \uc2e4\uc9c8\uc801\uc778 \uac00\uc785\uc740 1978\ub144\ubd80\ud130\uac00 \ub428)\ud558\uac8c \ub418\uc5c8\ub2e4.", "paragraph_sentence": "\uc774 \ub098\ub77c\ub294 \ub2e4\ub978 \uc5ec\ub7ec \ub098\ub77c\uc5d0 \uc55e\uc11c\uc11c \uc0b0\uc5c5\ud601\uba85\uc744 \uc131\ucde8\ud558\uc5ec \uc624\ub7ab\ub3d9\uc548 \uc138\uacc4 \uacbd\uc81c\uc5d0 \uad70\ub9bc\ud574 \uc654\ub2e4. \uadf8\ub7ec\ub098 20\uc138\uae30\uc5d0 \ub4e4\uc5b4\uc640\uc11c\ub294 \uc720\ub7fd \ub300\ub959\uacfc \ubd81\uc544\uba54\ub9ac\uce74\uc758 \uacf5\uc5c5\uc774 \uae09\uc18d\ub3c4\ub85c \ubc1c\ub2ec\ud558\uc5ec 1\ucc28\ub300\uc804 \uc774\ud6c4 \uc138\uacc4 \uacbd\uc81c\uc5d0 \uc788\uc5b4\uc11c\uc758 \uc9c0\uc704\ub294 \ucc28\ucc28 \uc800\ud558\ud588\ub2e4. \ud574\uc678 \ud22c\uc790\ub85c\ubd80\ud130\uc758 \uc218\uc785\uc73c\ub85c \uacbd\uc81c\uc758 \uc548\uc815\uc744 \uaf80\ud588\uc73c\ub098 2\ucc28\ub300\uc804\uc5d0 \uc758\ud574 \ud070 \ud0c0\uaca9\uc744 \ubc1b\uace0 \uc7ac\ud3b8\uc131\ud558\uae30\uc5d0 \uc774\ub974\ub800\ub2e4. \uadf8\uac83\uc740 \uae30\uac04\uc0b0\uc5c5\uc758 \uad6d\uc720\ud654\ub97c \ube44\ub86f\ud55c \uacc4\ud68d\uacbd\uc81c\uc758 \ub3c4\uc785\uc73c\ub85c \uad6d\uc81c\uc218\uc9c0\ub97c \uac1c\uc120\ud558\uace0 \uc0ac\ud68c\ubcf4\uc7a5\uc81c\ub3c4\ub97c \ud655\ucda9\ud558\uc5ec \ubcf5\uc9c0 \uad6d\uac00\uc758 \uc2e4\ud604\uc744 \uaf80\ud558\ub294 \uac83\uc744 \uc8fc\uc694 \ubaa9\uc801\uc73c\ub85c \ud558\ub294 \uacbd\uc81c \uccb4\uc81c\uc774\ub2e4. \uadf8\ub7ec\ub098 \ub192\uc740 \uace0\uc6a9\uc218\uc900\uc744 \uc720\uc9c0\ud558\uba74\uc11c\ub3c4 \ub300\uc678\uc801\uc73c\ub85c\ub294 \uad6d\uc81c \uc218\uc9c0\uc758 \uc545\ud654, \ub300\ub0b4\uc801\uc73c\ub85c\ub294 \ucf54\uc2a4\ud2b8 \uc778\ud50c\ub808\uc774\uc158\uc5d0 \uc758\ud55c \uad34\ub85c\uc6c0\uc744 \ub2f9\ud558\uace0 \uc788\ub2e4. \uc774\ub7ec\ud55c \uc0c1\ud669\uc744 \ud0c0\uac1c\ud558\uae30 \uc704\ud574 \uc601\uad6d\uc740 1961\ub144\ubd80\ud130 \uc720\ub7fd\uacbd\uc81c\uacf5\ub3d9\uccb4(EEC) \uac00\uc785\uc744 \uc2e0\uccad\ud588\uc73c\ub098 \ud504\ub791\uc2a4\uc758 \uac70\ubd80\ub85c \uc2e4\ud604\uc744 \ubcf4\uc9c0 \ubabb\ud558\ub2e4\uac00 1971\ub144 6\uc6d4 \ub9c8\uce68\ub0b4 \uc218\ucd9c\uc785\uc815\ucc45 \ubc0f \ud30c\uc6b4\ub4dc\ud654(\u8ca8) \ubb38\uc81c\uc5d0 \ub9e4\ub4ed\uc744 \uc9d3\uace0 \uc601\u00b7\ubd88 \uc591\uad6d\uac04\uc5d0 \ucd5c\uc885 \ud569\uc758\uc5d0 \ub3c4\ub2ec\ud568\uc73c\ub85c\uc368 1973\ub144 1\uc6d4 \ubd80\ud130 \uc815\uc2dd \uac00\uc785(\uacbd\uacfc\uae30\uac04 5\ub144\uc744 \uacc4\uc0b0\ud558\uba74 \uc2e4\uc9c8\uc801\uc778 \uac00\uc785\uc740 1978\ub144\ubd80\ud130\uac00 \ub428)\ud558\uac8c \ub418\uc5c8\ub2e4. ", "paragraph_answer": "\uc774 \ub098\ub77c\ub294 \ub2e4\ub978 \uc5ec\ub7ec \ub098\ub77c\uc5d0 \uc55e\uc11c\uc11c \uc0b0\uc5c5\ud601\uba85\uc744 \uc131\ucde8\ud558\uc5ec \uc624\ub7ab\ub3d9\uc548 \uc138\uacc4 \uacbd\uc81c\uc5d0 \uad70\ub9bc\ud574 \uc654\ub2e4. \uadf8\ub7ec\ub098 20\uc138\uae30\uc5d0 \ub4e4\uc5b4\uc640\uc11c\ub294 \uc720\ub7fd \ub300\ub959\uacfc \ubd81\uc544\uba54\ub9ac\uce74\uc758 \uacf5\uc5c5\uc774 \uae09\uc18d\ub3c4\ub85c \ubc1c\ub2ec\ud558\uc5ec 1\ucc28\ub300\uc804 \uc774\ud6c4 \uc138\uacc4 \uacbd\uc81c\uc5d0 \uc788\uc5b4\uc11c\uc758 \uc9c0\uc704\ub294 \ucc28\ucc28 \uc800\ud558\ud588\ub2e4. \ud574\uc678 \ud22c\uc790\ub85c\ubd80\ud130\uc758 \uc218\uc785\uc73c\ub85c \uacbd\uc81c\uc758 \uc548\uc815\uc744 \uaf80\ud588\uc73c\ub098 2\ucc28\ub300\uc804\uc5d0 \uc758\ud574 \ud070 \ud0c0\uaca9\uc744 \ubc1b\uace0 \uc7ac\ud3b8\uc131\ud558\uae30\uc5d0 \uc774\ub974\ub800\ub2e4. \uadf8\uac83\uc740 \uae30\uac04\uc0b0\uc5c5\uc758 \uad6d\uc720\ud654\ub97c \ube44\ub86f\ud55c \uacc4\ud68d\uacbd\uc81c\uc758 \ub3c4\uc785\uc73c\ub85c \uad6d\uc81c\uc218\uc9c0\ub97c \uac1c\uc120\ud558\uace0 \uc0ac\ud68c\ubcf4\uc7a5\uc81c\ub3c4\ub97c \ud655\ucda9\ud558\uc5ec \ubcf5\uc9c0 \uad6d\uac00\uc758 \uc2e4\ud604\uc744 \uaf80\ud558\ub294 \uac83\uc744 \uc8fc\uc694 \ubaa9\uc801\uc73c\ub85c \ud558\ub294 \uacbd\uc81c \uccb4\uc81c\uc774\ub2e4. \uadf8\ub7ec\ub098 \ub192\uc740 \uace0\uc6a9\uc218\uc900\uc744 \uc720\uc9c0\ud558\uba74\uc11c\ub3c4 \ub300\uc678\uc801\uc73c\ub85c\ub294 \uad6d\uc81c \uc218\uc9c0\uc758 \uc545\ud654, \ub300\ub0b4\uc801\uc73c\ub85c\ub294 \ucf54\uc2a4\ud2b8 \uc778\ud50c\ub808\uc774\uc158\uc5d0 \uc758\ud55c \uad34\ub85c\uc6c0\uc744 \ub2f9\ud558\uace0 \uc788\ub2e4. \uc774\ub7ec\ud55c \uc0c1\ud669\uc744 \ud0c0\uac1c\ud558\uae30 \uc704\ud574 \uc601\uad6d\uc740 1961\ub144\ubd80\ud130 \uc720\ub7fd\uacbd\uc81c\uacf5\ub3d9\uccb4(EEC) \uac00\uc785\uc744 \uc2e0\uccad\ud588\uc73c\ub098 \ud504\ub791\uc2a4\uc758 \uac70\ubd80\ub85c \uc2e4\ud604\uc744 \ubcf4\uc9c0 \ubabb\ud558\ub2e4\uac00 1971\ub144 6\uc6d4 \ub9c8\uce68\ub0b4 \uc218\ucd9c\uc785\uc815\ucc45 \ubc0f \ud30c\uc6b4\ub4dc\ud654(\u8ca8) \ubb38\uc81c\uc5d0 \ub9e4\ub4ed\uc744 \uc9d3\uace0 \uc601\u00b7\ubd88 \uc591\uad6d\uac04\uc5d0 \ucd5c\uc885 \ud569\uc758\uc5d0 \ub3c4\ub2ec\ud568\uc73c\ub85c\uc368 1973\ub144 1\uc6d4 \ubd80\ud130 \uc815\uc2dd \uac00\uc785(\uacbd\uacfc\uae30\uac04 5\ub144\uc744 \uacc4\uc0b0\ud558\uba74 \uc2e4\uc9c8\uc801\uc778 \uac00\uc785\uc740 1978\ub144\ubd80\ud130\uac00 \ub428)\ud558\uac8c \ub418\uc5c8\ub2e4.", "sentence_answer": "\uc774\ub7ec\ud55c \uc0c1\ud669\uc744 \ud0c0\uac1c\ud558\uae30 \uc704\ud574 \uc601\uad6d\uc740 1961\ub144\ubd80\ud130 \uc720\ub7fd\uacbd\uc81c\uacf5\ub3d9\uccb4(EEC) \uac00\uc785\uc744 \uc2e0\uccad\ud588\uc73c\ub098 \ud504\ub791\uc2a4\uc758 \uac70\ubd80\ub85c \uc2e4\ud604\uc744 \ubcf4\uc9c0 \ubabb\ud558\ub2e4\uac00 1971\ub144 6\uc6d4 \ub9c8\uce68\ub0b4 \uc218\ucd9c\uc785\uc815\ucc45 \ubc0f \ud30c\uc6b4\ub4dc\ud654(\u8ca8) \ubb38\uc81c\uc5d0 \ub9e4\ub4ed\uc744 \uc9d3\uace0 \uc601\u00b7\ubd88 \uc591\uad6d\uac04\uc5d0 \ucd5c\uc885 \ud569\uc758\uc5d0 \ub3c4\ub2ec\ud568\uc73c\ub85c\uc368 1973\ub144 1\uc6d4 \ubd80\ud130 \uc815\uc2dd \uac00\uc785(\uacbd\uacfc\uae30\uac04 5\ub144\uc744 \uacc4\uc0b0\ud558\uba74 \uc2e4\uc9c8\uc801\uc778 \uac00\uc785\uc740 1978\ub144\ubd80\ud130\uac00 \ub428)\ud558\uac8c \ub418\uc5c8\ub2e4.", "paragraph_id": "6458616-15-2", "paragraph_question": "question: \uc601\uad6d\uc774 \uc720\ub7fd\uacbd\uc81c\uacf5\ub3d9\uccb4\uc5d0 \uc815\uc2dd \uac00\uc785\ud55c \uac83\uc740 \uc5b8\uc81c\ubd80\ud130 \uc778\uac00?, context: \uc774 \ub098\ub77c\ub294 \ub2e4\ub978 \uc5ec\ub7ec \ub098\ub77c\uc5d0 \uc55e\uc11c\uc11c \uc0b0\uc5c5\ud601\uba85\uc744 \uc131\ucde8\ud558\uc5ec \uc624\ub7ab\ub3d9\uc548 \uc138\uacc4 \uacbd\uc81c\uc5d0 \uad70\ub9bc\ud574 \uc654\ub2e4. \uadf8\ub7ec\ub098 20\uc138\uae30\uc5d0 \ub4e4\uc5b4\uc640\uc11c\ub294 \uc720\ub7fd \ub300\ub959\uacfc \ubd81\uc544\uba54\ub9ac\uce74\uc758 \uacf5\uc5c5\uc774 \uae09\uc18d\ub3c4\ub85c \ubc1c\ub2ec\ud558\uc5ec 1\ucc28\ub300\uc804 \uc774\ud6c4 \uc138\uacc4 \uacbd\uc81c\uc5d0 \uc788\uc5b4\uc11c\uc758 \uc9c0\uc704\ub294 \ucc28\ucc28 \uc800\ud558\ud588\ub2e4. \ud574\uc678 \ud22c\uc790\ub85c\ubd80\ud130\uc758 \uc218\uc785\uc73c\ub85c \uacbd\uc81c\uc758 \uc548\uc815\uc744 \uaf80\ud588\uc73c\ub098 2\ucc28\ub300\uc804\uc5d0 \uc758\ud574 \ud070 \ud0c0\uaca9\uc744 \ubc1b\uace0 \uc7ac\ud3b8\uc131\ud558\uae30\uc5d0 \uc774\ub974\ub800\ub2e4. \uadf8\uac83\uc740 \uae30\uac04\uc0b0\uc5c5\uc758 \uad6d\uc720\ud654\ub97c \ube44\ub86f\ud55c \uacc4\ud68d\uacbd\uc81c\uc758 \ub3c4\uc785\uc73c\ub85c \uad6d\uc81c\uc218\uc9c0\ub97c \uac1c\uc120\ud558\uace0 \uc0ac\ud68c\ubcf4\uc7a5\uc81c\ub3c4\ub97c \ud655\ucda9\ud558\uc5ec \ubcf5\uc9c0 \uad6d\uac00\uc758 \uc2e4\ud604\uc744 \uaf80\ud558\ub294 \uac83\uc744 \uc8fc\uc694 \ubaa9\uc801\uc73c\ub85c \ud558\ub294 \uacbd\uc81c \uccb4\uc81c\uc774\ub2e4. \uadf8\ub7ec\ub098 \ub192\uc740 \uace0\uc6a9\uc218\uc900\uc744 \uc720\uc9c0\ud558\uba74\uc11c\ub3c4 \ub300\uc678\uc801\uc73c\ub85c\ub294 \uad6d\uc81c \uc218\uc9c0\uc758 \uc545\ud654, \ub300\ub0b4\uc801\uc73c\ub85c\ub294 \ucf54\uc2a4\ud2b8 \uc778\ud50c\ub808\uc774\uc158\uc5d0 \uc758\ud55c \uad34\ub85c\uc6c0\uc744 \ub2f9\ud558\uace0 \uc788\ub2e4. \uc774\ub7ec\ud55c \uc0c1\ud669\uc744 \ud0c0\uac1c\ud558\uae30 \uc704\ud574 \uc601\uad6d\uc740 1961\ub144\ubd80\ud130 \uc720\ub7fd\uacbd\uc81c\uacf5\ub3d9\uccb4(EEC) \uac00\uc785\uc744 \uc2e0\uccad\ud588\uc73c\ub098 \ud504\ub791\uc2a4\uc758 \uac70\ubd80\ub85c \uc2e4\ud604\uc744 \ubcf4\uc9c0 \ubabb\ud558\ub2e4\uac00 1971\ub144 6\uc6d4 \ub9c8\uce68\ub0b4 \uc218\ucd9c\uc785\uc815\ucc45 \ubc0f \ud30c\uc6b4\ub4dc\ud654(\u8ca8) \ubb38\uc81c\uc5d0 \ub9e4\ub4ed\uc744 \uc9d3\uace0 \uc601\u00b7\ubd88 \uc591\uad6d\uac04\uc5d0 \ucd5c\uc885 \ud569\uc758\uc5d0 \ub3c4\ub2ec\ud568\uc73c\ub85c\uc368 1973\ub144 1\uc6d4\ubd80\ud130 \uc815\uc2dd \uac00\uc785(\uacbd\uacfc\uae30\uac04 5\ub144\uc744 \uacc4\uc0b0\ud558\uba74 \uc2e4\uc9c8\uc801\uc778 \uac00\uc785\uc740 1978\ub144\ubd80\ud130\uac00 \ub428)\ud558\uac8c \ub418\uc5c8\ub2e4."} {"question": "\ud55c\uc591\ub300\ud559\uad50 \uc11c\uc6b8\ucea0\ud37c\uc2a4\uc5d0\ub294 \uba87 \uacf3\uc5d0 ATM\uc774 \uc788\ub294\uac00?", "paragraph": "\ud55c\uc591\ub300\ud559\uad50 \uc11c\uc6b8\ucea0\ud37c\uc2a4\uc5d0\ub294 \ud559\uc0dd\ubcf5\uc9c0\uc640 \uad00\ub828\ud55c \uc5ec\ub7ec \uac00\uc9c0 \ud3b8\uc758 \uc2dc\uc124\ub4e4\uc774 \uc788\ub2e4. \uae08\uc735\uad00\ub828 \uc2dc\uc124\ub85c\ub294 \uc2e0\ud55c\uc740\ud589\uacfc \uad6d\ubbfc\uc740\ud589 ATM\uc774 \uc77c\uacf1 \uacf3\uc5d0 \uc788\uc73c\uba70, \uc2e0\ud55c\uc740\ud589 \ub450 \uac1c \uc9c0\uc810\uc774 \uc788\ub2e4. \ubcf5\uc9c0 \uc2dc\uc124 \uc911\uc2ec\uc758 \uac74\ubb3c\uc740 \ud559\uc0dd\ud68c\uad00, \ud55c\uc591\ud50c\ub77c\uc790, \ud589\uc6d0\ud30c\ud06c\uac00 \uc788\ub2e4. \ub610\ud55c \uc870\uacbd \uad00\ub828 \uc2dc\uc124\ub85c\ub294 \ubd84\uc218 \uc138 \uac1c, \uc778\uacf5 \ud3ed\ud3ec \ud55c \uac1c\uac00 \uc788\uc73c\uba70, \uc18c\uaddc\ubaa8\uc758 \ub179\uc9c0\uc2dc\uc124\ub4e4\uc774 \ucea0\ud37c\uc2a4 \uacf3\uacf3\uc5d0 \ub4e4\uc5b4\uc11c \uc788\ub2e4. \ub610\ud55c 158\uacc4\ub2e8 \uc27c\ud130\uc5d0\ub294 \ubc15\ubaa9\uc6d4 \uc2dc\ube44\uc640 \ud568\uaed8 \ubbfc\uc8fc\uc5f4\uc0ac \uae30\ub150\ube44\uac00 \uc788\ub2e4. \uac04\uc2dd\uad00\ub828 \ubcf5\uc9c0 \uc2dc\uc124\ub85c\ub294 \ucee4\ud53c\uc20d\uc774 \uc5f4\ud55c \uacf3\uc774 \uc788\uc73c\uba70, 24\uc2dc\uac04 \ud3b8\uc758\uc810 \ub124 \uacf3\uc744 \ud3ec\ud568\ud574 \ucd1d \uc5f4 \uac1c\uc758 \ub9e4\uc810\uc744 \uc6b4\uc601\ud558\uace0 \uc788\ub2e4. \uadf8 \uc678\uc5d0 \uc704\uc0dd \uc2dc\uc124\uc778 \uc628\uc218\uac00\ub3d9 \uc0e4\uc6cc\uc2e4\uc774 \uac74\ubb3c \uc5ec\uc12f \uacf3\uc5d0 \uc6b4\uc601\ub418\uace0 \uc788\uc73c\uba70 \uc11c\uc810 \ub450 \uacf3, \ubb38\uad6c\uc810 \uc138 \uacf3 , \ubcf5\uc0ac\uc2e4 / \uc5ec\ud589\uc0ac / \uc0ac\uc9c4\uad00 / \uae30\ub150\ud488\uc810 \ub4f1\uc774 \uc788\ub2e4.", "answer": "\uc77c\uacf1 \uacf3", "sentence": "\uae08\uc735\uad00\ub828 \uc2dc\uc124\ub85c\ub294 \uc2e0\ud55c\uc740\ud589\uacfc \uad6d\ubbfc\uc740\ud589 ATM\uc774 \uc77c\uacf1 \uacf3 \uc5d0", "paragraph_sentence": "\ud55c\uc591\ub300\ud559\uad50 \uc11c\uc6b8\ucea0\ud37c\uc2a4\uc5d0\ub294 \ud559\uc0dd\ubcf5\uc9c0\uc640 \uad00\ub828\ud55c \uc5ec\ub7ec \uac00\uc9c0 \ud3b8\uc758 \uc2dc\uc124\ub4e4\uc774 \uc788\ub2e4. \uae08\uc735\uad00\ub828 \uc2dc\uc124\ub85c\ub294 \uc2e0\ud55c\uc740\ud589\uacfc \uad6d\ubbfc\uc740\ud589 ATM\uc774 \uc77c\uacf1 \uacf3 \uc5d0 \uc788\uc73c\uba70, \uc2e0\ud55c\uc740\ud589 \ub450 \uac1c \uc9c0\uc810\uc774 \uc788\ub2e4. \ubcf5\uc9c0 \uc2dc\uc124 \uc911\uc2ec\uc758 \uac74\ubb3c\uc740 \ud559\uc0dd\ud68c\uad00, \ud55c\uc591\ud50c\ub77c\uc790, \ud589\uc6d0\ud30c\ud06c\uac00 \uc788\ub2e4. \ub610\ud55c \uc870\uacbd \uad00\ub828 \uc2dc\uc124\ub85c\ub294 \ubd84\uc218 \uc138 \uac1c, \uc778\uacf5 \ud3ed\ud3ec \ud55c \uac1c\uac00 \uc788\uc73c\uba70, \uc18c\uaddc\ubaa8\uc758 \ub179\uc9c0\uc2dc\uc124\ub4e4\uc774 \ucea0\ud37c\uc2a4 \uacf3\uacf3\uc5d0 \ub4e4\uc5b4\uc11c \uc788\ub2e4. \ub610\ud55c 158\uacc4\ub2e8 \uc27c\ud130\uc5d0\ub294 \ubc15\ubaa9\uc6d4 \uc2dc\ube44\uc640 \ud568\uaed8 \ubbfc\uc8fc\uc5f4\uc0ac \uae30\ub150\ube44\uac00 \uc788\ub2e4. \uac04\uc2dd\uad00\ub828 \ubcf5\uc9c0 \uc2dc\uc124\ub85c\ub294 \ucee4\ud53c\uc20d\uc774 \uc5f4\ud55c \uacf3\uc774 \uc788\uc73c\uba70, 24\uc2dc\uac04 \ud3b8\uc758\uc810 \ub124 \uacf3\uc744 \ud3ec\ud568\ud574 \ucd1d \uc5f4 \uac1c\uc758 \ub9e4\uc810\uc744 \uc6b4\uc601\ud558\uace0 \uc788\ub2e4. \uadf8 \uc678\uc5d0 \uc704\uc0dd \uc2dc\uc124\uc778 \uc628\uc218\uac00\ub3d9 \uc0e4\uc6cc\uc2e4\uc774 \uac74\ubb3c \uc5ec\uc12f \uacf3\uc5d0 \uc6b4\uc601\ub418\uace0 \uc788\uc73c\uba70 \uc11c\uc810 \ub450 \uacf3, \ubb38\uad6c\uc810 \uc138 \uacf3 , \ubcf5\uc0ac\uc2e4 / \uc5ec\ud589\uc0ac / \uc0ac\uc9c4\uad00 / \uae30\ub150\ud488\uc810 \ub4f1\uc774 \uc788\ub2e4.", "paragraph_answer": "\ud55c\uc591\ub300\ud559\uad50 \uc11c\uc6b8\ucea0\ud37c\uc2a4\uc5d0\ub294 \ud559\uc0dd\ubcf5\uc9c0\uc640 \uad00\ub828\ud55c \uc5ec\ub7ec \uac00\uc9c0 \ud3b8\uc758 \uc2dc\uc124\ub4e4\uc774 \uc788\ub2e4. \uae08\uc735\uad00\ub828 \uc2dc\uc124\ub85c\ub294 \uc2e0\ud55c\uc740\ud589\uacfc \uad6d\ubbfc\uc740\ud589 ATM\uc774 \uc77c\uacf1 \uacf3 \uc5d0 \uc788\uc73c\uba70, \uc2e0\ud55c\uc740\ud589 \ub450 \uac1c \uc9c0\uc810\uc774 \uc788\ub2e4. \ubcf5\uc9c0 \uc2dc\uc124 \uc911\uc2ec\uc758 \uac74\ubb3c\uc740 \ud559\uc0dd\ud68c\uad00, \ud55c\uc591\ud50c\ub77c\uc790, \ud589\uc6d0\ud30c\ud06c\uac00 \uc788\ub2e4. \ub610\ud55c \uc870\uacbd \uad00\ub828 \uc2dc\uc124\ub85c\ub294 \ubd84\uc218 \uc138 \uac1c, \uc778\uacf5 \ud3ed\ud3ec \ud55c \uac1c\uac00 \uc788\uc73c\uba70, \uc18c\uaddc\ubaa8\uc758 \ub179\uc9c0\uc2dc\uc124\ub4e4\uc774 \ucea0\ud37c\uc2a4 \uacf3\uacf3\uc5d0 \ub4e4\uc5b4\uc11c \uc788\ub2e4. \ub610\ud55c 158\uacc4\ub2e8 \uc27c\ud130\uc5d0\ub294 \ubc15\ubaa9\uc6d4 \uc2dc\ube44\uc640 \ud568\uaed8 \ubbfc\uc8fc\uc5f4\uc0ac \uae30\ub150\ube44\uac00 \uc788\ub2e4. \uac04\uc2dd\uad00\ub828 \ubcf5\uc9c0 \uc2dc\uc124\ub85c\ub294 \ucee4\ud53c\uc20d\uc774 \uc5f4\ud55c \uacf3\uc774 \uc788\uc73c\uba70, 24\uc2dc\uac04 \ud3b8\uc758\uc810 \ub124 \uacf3\uc744 \ud3ec\ud568\ud574 \ucd1d \uc5f4 \uac1c\uc758 \ub9e4\uc810\uc744 \uc6b4\uc601\ud558\uace0 \uc788\ub2e4. \uadf8 \uc678\uc5d0 \uc704\uc0dd \uc2dc\uc124\uc778 \uc628\uc218\uac00\ub3d9 \uc0e4\uc6cc\uc2e4\uc774 \uac74\ubb3c \uc5ec\uc12f \uacf3\uc5d0 \uc6b4\uc601\ub418\uace0 \uc788\uc73c\uba70 \uc11c\uc810 \ub450 \uacf3, \ubb38\uad6c\uc810 \uc138 \uacf3 , \ubcf5\uc0ac\uc2e4 / \uc5ec\ud589\uc0ac / \uc0ac\uc9c4\uad00 / \uae30\ub150\ud488\uc810 \ub4f1\uc774 \uc788\ub2e4.", "sentence_answer": "\uae08\uc735\uad00\ub828 \uc2dc\uc124\ub85c\ub294 \uc2e0\ud55c\uc740\ud589\uacfc \uad6d\ubbfc\uc740\ud589 ATM\uc774 \uc77c\uacf1 \uacf3 \uc5d0", "paragraph_id": "6570118-11-0", "paragraph_question": "question: \ud55c\uc591\ub300\ud559\uad50 \uc11c\uc6b8\ucea0\ud37c\uc2a4\uc5d0\ub294 \uba87 \uacf3\uc5d0 ATM\uc774 \uc788\ub294\uac00?, context: \ud55c\uc591\ub300\ud559\uad50 \uc11c\uc6b8\ucea0\ud37c\uc2a4\uc5d0\ub294 \ud559\uc0dd\ubcf5\uc9c0\uc640 \uad00\ub828\ud55c \uc5ec\ub7ec \uac00\uc9c0 \ud3b8\uc758 \uc2dc\uc124\ub4e4\uc774 \uc788\ub2e4. \uae08\uc735\uad00\ub828 \uc2dc\uc124\ub85c\ub294 \uc2e0\ud55c\uc740\ud589\uacfc \uad6d\ubbfc\uc740\ud589 ATM\uc774 \uc77c\uacf1 \uacf3\uc5d0 \uc788\uc73c\uba70, \uc2e0\ud55c\uc740\ud589 \ub450 \uac1c \uc9c0\uc810\uc774 \uc788\ub2e4. \ubcf5\uc9c0 \uc2dc\uc124 \uc911\uc2ec\uc758 \uac74\ubb3c\uc740 \ud559\uc0dd\ud68c\uad00, \ud55c\uc591\ud50c\ub77c\uc790, \ud589\uc6d0\ud30c\ud06c\uac00 \uc788\ub2e4. \ub610\ud55c \uc870\uacbd \uad00\ub828 \uc2dc\uc124\ub85c\ub294 \ubd84\uc218 \uc138 \uac1c, \uc778\uacf5 \ud3ed\ud3ec \ud55c \uac1c\uac00 \uc788\uc73c\uba70, \uc18c\uaddc\ubaa8\uc758 \ub179\uc9c0\uc2dc\uc124\ub4e4\uc774 \ucea0\ud37c\uc2a4 \uacf3\uacf3\uc5d0 \ub4e4\uc5b4\uc11c \uc788\ub2e4. \ub610\ud55c 158\uacc4\ub2e8 \uc27c\ud130\uc5d0\ub294 \ubc15\ubaa9\uc6d4 \uc2dc\ube44\uc640 \ud568\uaed8 \ubbfc\uc8fc\uc5f4\uc0ac \uae30\ub150\ube44\uac00 \uc788\ub2e4. \uac04\uc2dd\uad00\ub828 \ubcf5\uc9c0 \uc2dc\uc124\ub85c\ub294 \ucee4\ud53c\uc20d\uc774 \uc5f4\ud55c \uacf3\uc774 \uc788\uc73c\uba70, 24\uc2dc\uac04 \ud3b8\uc758\uc810 \ub124 \uacf3\uc744 \ud3ec\ud568\ud574 \ucd1d \uc5f4 \uac1c\uc758 \ub9e4\uc810\uc744 \uc6b4\uc601\ud558\uace0 \uc788\ub2e4. \uadf8 \uc678\uc5d0 \uc704\uc0dd \uc2dc\uc124\uc778 \uc628\uc218\uac00\ub3d9 \uc0e4\uc6cc\uc2e4\uc774 \uac74\ubb3c \uc5ec\uc12f \uacf3\uc5d0 \uc6b4\uc601\ub418\uace0 \uc788\uc73c\uba70 \uc11c\uc810 \ub450 \uacf3, \ubb38\uad6c\uc810 \uc138 \uacf3 , \ubcf5\uc0ac\uc2e4 / \uc5ec\ud589\uc0ac / \uc0ac\uc9c4\uad00 / \uae30\ub150\ud488\uc810 \ub4f1\uc774 \uc788\ub2e4."} {"question": "\uc544\uc6b0\ub460\ube14\ub77c\uac00 \ud565\uc740 \uc5bc\uc74c \uc18d\uc5d0\uc11c \ub4dc\ub7ec\ub09c \ub0a8\uc790\uc758 \uc774\ub984\uc740?", "paragraph": "\uac15\uae00\ub808\ub9ac\ub294 \uc704\ubbf8\ub974\uac00 \uc5b4\ub514\uc5d0 \uc0b4\uc558\uace0 \ubb34\uc5c7\uc744 \uba39\uace0 \uc0b4\uc558\ub0d0\uace0 \ubb3b\ub294\ub2e4. \ub192\uc73c\uc2e0 \ubd84\uc740 \ub3c5 \ubb3c\ubc29\uc6b8\uc5d0\uc11c \uc704\ubbf8\ub974\uc640 \ud568\uaed8 \uc544\uc6b0\ub460\ube14\ub77c\ub77c\ub294 \uc554\uc18c\uac00 \ud0dc\uc5b4\ub0ac\ub2e4\uace0 \ub9d0\ud55c\ub2e4. \uc544\uc6b0\ub460\ube14\ub77c\uc758 \uc816\ud1b5\uc5d0\uc11c \uc6b0\uc720\uac00 \ub124 \uc904\uae30\uc758 \uac15\uc744 \uc774\ub8e8\uba70 \ud758\ub800\uace0, \uc704\ubbf8\ub974\ub294 \uadf8 \uc6b0\uc720\ub97c \uba39\uc5c8\ub2e4. \uac15\uae00\ub808\ub9ac\ub294 \uadf8 \uc554\uc18c\ub294 \ubb34\uc5c7\uc744 \uba39\uc5c8\ub0d0\uace0 \ubb3b\ub294\ub2e4. \ub192\uc73c\uc2e0 \ubd84\uc740 \uc554\uc18c\ub294 \uc18c\uae08\uae30 \uc788\ub294 \uc5bc\uc74c \ub369\uc5b4\ub9ac\ub97c \ud565\uc558\ub2e4\uace0 \ub2f5\ud55c\ub2e4. \uc544\uc6b0\ub460\ube14\ub77c\uac00 \uc5bc\uc74c \ub369\uc5b4\ub9ac\ub97c \ud565\uc740 \uccab \ub0a0\uc5d0\ub294 \uc5bc\uc74c \uc18d\uc5d0\uc11c \ub0a8\uc790\uc758 \uba38\ub9ac\uce74\ub77d\uc774 \ubcf4\uc600\uace0, \ub458\uc9f8 \ub0a0\uc5d0\ub294 \uba38\ub9ac\uac00 \ubcf4\uc600\uace0, \uc14b\uc9f8 \ub0a0\uc5d0\ub294 \uc804\uccb4 \ubab8\uc774 \ub4dc\ub7ec\ub0ac\ub2e4. \uc774 \ub0a8\uc790\uc758 \uc774\ub984\uc740 \ubd80\ub9ac\uc600\ub2e4. \uadf8 \uc5ed\uc2dc \uac70\ub300\ud558\uace0 \uac15\ub825\ud588\uc73c\uba70, \uc678\ubaa8\ub294 \uc544\ub984\ub2e4\uc6e0\ub2e4\uace0 \ub9d0\ud55c\ub2e4. \ubd80\ub9ac\ub294 \ubcf4\ub974\ub97c \ub0b3\uc558\uace0 \ubcf4\ub974\ub294 \ubcfc\uc18c\ub978\uc758 \ub538\uc778 \uc5ec\uc131 \uc694\ud230 \ubca0\uc2a4\ud2c0\ub77c\uc640 \uacb0\ud63c\ud558\uc5ec \uc624\ub518, \ube4c\ub9ac, \ubca0\uc774\ub77c\ub294 3\ud615\uc81c\ub97c \ub0b3\uc558\ub2e4. \ub192\uc73c\uc2e0 \ubd84\uc740 \u201c\uc624\ub518\uacfc \uadf8\uc758 \ud615\uc81c\ub4e4\uc740 \ud558\ub298\uacfc \ub545\uc758 \uc9c0\ubc30\uc790\uac00 \ub418\uc5b4\uc57c\ub9cc \ud588\ub2e4. \uadf8\ub294 \uadf8\ub807\uac8c \ub418\uc5c8\uc5b4\uc57c \ud55c\ub2e4\ub294 \uac83\uc774 \uc6b0\ub9ac\uc758 \uc758\uacac\uc774\ub2e4. \uc624\ub518\uc774\ub780 \uc6b0\ub9ac\uac00 \uc544\ub294 \uac00\uc7a5 \uc704\ub300\ud558\uace0 \uc601\uad11\uc2a4\ub7ec\uc6b4 \uc874\uc7ac\uc758 \uc774\ub984\uc774\uba70, \uadf8\ub300 \uc5ed\uc2dc \uadf8\ub97c \uadf8\ub807\uac8c \ubd80\ub974\ub294 \uac83\uc5d0 \ub300\ud574 \uae30\uaebc\uc774 \ub3d9\uc758\ud558\ub9ac\ub77c \ubcf8\ub2e4\u201d\uace0 \ub9d0\ud55c\ub2e4.", "answer": "\ubd80\ub9ac", "sentence": "\uc774 \ub0a8\uc790\uc758 \uc774\ub984\uc740 \ubd80\ub9ac \uc600\ub2e4.", "paragraph_sentence": "\uac15\uae00\ub808\ub9ac\ub294 \uc704\ubbf8\ub974\uac00 \uc5b4\ub514\uc5d0 \uc0b4\uc558\uace0 \ubb34\uc5c7\uc744 \uba39\uace0 \uc0b4\uc558\ub0d0\uace0 \ubb3b\ub294\ub2e4. \ub192\uc73c\uc2e0 \ubd84\uc740 \ub3c5 \ubb3c\ubc29\uc6b8\uc5d0\uc11c \uc704\ubbf8\ub974\uc640 \ud568\uaed8 \uc544\uc6b0\ub460\ube14\ub77c\ub77c\ub294 \uc554\uc18c\uac00 \ud0dc\uc5b4\ub0ac\ub2e4\uace0 \ub9d0\ud55c\ub2e4. \uc544\uc6b0\ub460\ube14\ub77c\uc758 \uc816\ud1b5\uc5d0\uc11c \uc6b0\uc720\uac00 \ub124 \uc904\uae30\uc758 \uac15\uc744 \uc774\ub8e8\uba70 \ud758\ub800\uace0, \uc704\ubbf8\ub974\ub294 \uadf8 \uc6b0\uc720\ub97c \uba39\uc5c8\ub2e4. \uac15\uae00\ub808\ub9ac\ub294 \uadf8 \uc554\uc18c\ub294 \ubb34\uc5c7\uc744 \uba39\uc5c8\ub0d0\uace0 \ubb3b\ub294\ub2e4. \ub192\uc73c\uc2e0 \ubd84\uc740 \uc554\uc18c\ub294 \uc18c\uae08\uae30 \uc788\ub294 \uc5bc\uc74c \ub369\uc5b4\ub9ac\ub97c \ud565\uc558\ub2e4\uace0 \ub2f5\ud55c\ub2e4. \uc544\uc6b0\ub460\ube14\ub77c\uac00 \uc5bc\uc74c \ub369\uc5b4\ub9ac\ub97c \ud565\uc740 \uccab \ub0a0\uc5d0\ub294 \uc5bc\uc74c \uc18d\uc5d0\uc11c \ub0a8\uc790\uc758 \uba38\ub9ac\uce74\ub77d\uc774 \ubcf4\uc600\uace0, \ub458\uc9f8 \ub0a0\uc5d0\ub294 \uba38\ub9ac\uac00 \ubcf4\uc600\uace0, \uc14b\uc9f8 \ub0a0\uc5d0\ub294 \uc804\uccb4 \ubab8\uc774 \ub4dc\ub7ec\ub0ac\ub2e4. \uc774 \ub0a8\uc790\uc758 \uc774\ub984\uc740 \ubd80\ub9ac \uc600\ub2e4. \uadf8 \uc5ed\uc2dc \uac70\ub300\ud558\uace0 \uac15\ub825\ud588\uc73c\uba70, \uc678\ubaa8\ub294 \uc544\ub984\ub2e4\uc6e0\ub2e4\uace0 \ub9d0\ud55c\ub2e4. \ubd80\ub9ac\ub294 \ubcf4\ub974\ub97c \ub0b3\uc558\uace0 \ubcf4\ub974\ub294 \ubcfc\uc18c\ub978\uc758 \ub538\uc778 \uc5ec\uc131 \uc694\ud230 \ubca0\uc2a4\ud2c0\ub77c\uc640 \uacb0\ud63c\ud558\uc5ec \uc624\ub518, \ube4c\ub9ac, \ubca0\uc774\ub77c\ub294 3\ud615\uc81c\ub97c \ub0b3\uc558\ub2e4. \ub192\uc73c\uc2e0 \ubd84\uc740 \u201c\uc624\ub518\uacfc \uadf8\uc758 \ud615\uc81c\ub4e4\uc740 \ud558\ub298\uacfc \ub545\uc758 \uc9c0\ubc30\uc790\uac00 \ub418\uc5b4\uc57c\ub9cc \ud588\ub2e4. \uadf8\ub294 \uadf8\ub807\uac8c \ub418\uc5c8\uc5b4\uc57c \ud55c\ub2e4\ub294 \uac83\uc774 \uc6b0\ub9ac\uc758 \uc758\uacac\uc774\ub2e4. \uc624\ub518\uc774\ub780 \uc6b0\ub9ac\uac00 \uc544\ub294 \uac00\uc7a5 \uc704\ub300\ud558\uace0 \uc601\uad11\uc2a4\ub7ec\uc6b4 \uc874\uc7ac\uc758 \uc774\ub984\uc774\uba70, \uadf8\ub300 \uc5ed\uc2dc \uadf8\ub97c \uadf8\ub807\uac8c \ubd80\ub974\ub294 \uac83\uc5d0 \ub300\ud574 \uae30\uaebc\uc774 \ub3d9\uc758\ud558\ub9ac\ub77c \ubcf8\ub2e4\u201d\uace0 \ub9d0\ud55c\ub2e4.", "paragraph_answer": "\uac15\uae00\ub808\ub9ac\ub294 \uc704\ubbf8\ub974\uac00 \uc5b4\ub514\uc5d0 \uc0b4\uc558\uace0 \ubb34\uc5c7\uc744 \uba39\uace0 \uc0b4\uc558\ub0d0\uace0 \ubb3b\ub294\ub2e4. \ub192\uc73c\uc2e0 \ubd84\uc740 \ub3c5 \ubb3c\ubc29\uc6b8\uc5d0\uc11c \uc704\ubbf8\ub974\uc640 \ud568\uaed8 \uc544\uc6b0\ub460\ube14\ub77c\ub77c\ub294 \uc554\uc18c\uac00 \ud0dc\uc5b4\ub0ac\ub2e4\uace0 \ub9d0\ud55c\ub2e4. \uc544\uc6b0\ub460\ube14\ub77c\uc758 \uc816\ud1b5\uc5d0\uc11c \uc6b0\uc720\uac00 \ub124 \uc904\uae30\uc758 \uac15\uc744 \uc774\ub8e8\uba70 \ud758\ub800\uace0, \uc704\ubbf8\ub974\ub294 \uadf8 \uc6b0\uc720\ub97c \uba39\uc5c8\ub2e4. \uac15\uae00\ub808\ub9ac\ub294 \uadf8 \uc554\uc18c\ub294 \ubb34\uc5c7\uc744 \uba39\uc5c8\ub0d0\uace0 \ubb3b\ub294\ub2e4. \ub192\uc73c\uc2e0 \ubd84\uc740 \uc554\uc18c\ub294 \uc18c\uae08\uae30 \uc788\ub294 \uc5bc\uc74c \ub369\uc5b4\ub9ac\ub97c \ud565\uc558\ub2e4\uace0 \ub2f5\ud55c\ub2e4. \uc544\uc6b0\ub460\ube14\ub77c\uac00 \uc5bc\uc74c \ub369\uc5b4\ub9ac\ub97c \ud565\uc740 \uccab \ub0a0\uc5d0\ub294 \uc5bc\uc74c \uc18d\uc5d0\uc11c \ub0a8\uc790\uc758 \uba38\ub9ac\uce74\ub77d\uc774 \ubcf4\uc600\uace0, \ub458\uc9f8 \ub0a0\uc5d0\ub294 \uba38\ub9ac\uac00 \ubcf4\uc600\uace0, \uc14b\uc9f8 \ub0a0\uc5d0\ub294 \uc804\uccb4 \ubab8\uc774 \ub4dc\ub7ec\ub0ac\ub2e4. \uc774 \ub0a8\uc790\uc758 \uc774\ub984\uc740 \ubd80\ub9ac \uc600\ub2e4. \uadf8 \uc5ed\uc2dc \uac70\ub300\ud558\uace0 \uac15\ub825\ud588\uc73c\uba70, \uc678\ubaa8\ub294 \uc544\ub984\ub2e4\uc6e0\ub2e4\uace0 \ub9d0\ud55c\ub2e4. \ubd80\ub9ac\ub294 \ubcf4\ub974\ub97c \ub0b3\uc558\uace0 \ubcf4\ub974\ub294 \ubcfc\uc18c\ub978\uc758 \ub538\uc778 \uc5ec\uc131 \uc694\ud230 \ubca0\uc2a4\ud2c0\ub77c\uc640 \uacb0\ud63c\ud558\uc5ec \uc624\ub518, \ube4c\ub9ac, \ubca0\uc774\ub77c\ub294 3\ud615\uc81c\ub97c \ub0b3\uc558\ub2e4. \ub192\uc73c\uc2e0 \ubd84\uc740 \u201c\uc624\ub518\uacfc \uadf8\uc758 \ud615\uc81c\ub4e4\uc740 \ud558\ub298\uacfc \ub545\uc758 \uc9c0\ubc30\uc790\uac00 \ub418\uc5b4\uc57c\ub9cc \ud588\ub2e4. \uadf8\ub294 \uadf8\ub807\uac8c \ub418\uc5c8\uc5b4\uc57c \ud55c\ub2e4\ub294 \uac83\uc774 \uc6b0\ub9ac\uc758 \uc758\uacac\uc774\ub2e4. \uc624\ub518\uc774\ub780 \uc6b0\ub9ac\uac00 \uc544\ub294 \uac00\uc7a5 \uc704\ub300\ud558\uace0 \uc601\uad11\uc2a4\ub7ec\uc6b4 \uc874\uc7ac\uc758 \uc774\ub984\uc774\uba70, \uadf8\ub300 \uc5ed\uc2dc \uadf8\ub97c \uadf8\ub807\uac8c \ubd80\ub974\ub294 \uac83\uc5d0 \ub300\ud574 \uae30\uaebc\uc774 \ub3d9\uc758\ud558\ub9ac\ub77c \ubcf8\ub2e4\u201d\uace0 \ub9d0\ud55c\ub2e4.", "sentence_answer": "\uc774 \ub0a8\uc790\uc758 \uc774\ub984\uc740 \ubd80\ub9ac \uc600\ub2e4.", "paragraph_id": "6526951-3-0", "paragraph_question": "question: \uc544\uc6b0\ub460\ube14\ub77c\uac00 \ud565\uc740 \uc5bc\uc74c \uc18d\uc5d0\uc11c \ub4dc\ub7ec\ub09c \ub0a8\uc790\uc758 \uc774\ub984\uc740?, context: \uac15\uae00\ub808\ub9ac\ub294 \uc704\ubbf8\ub974\uac00 \uc5b4\ub514\uc5d0 \uc0b4\uc558\uace0 \ubb34\uc5c7\uc744 \uba39\uace0 \uc0b4\uc558\ub0d0\uace0 \ubb3b\ub294\ub2e4. \ub192\uc73c\uc2e0 \ubd84\uc740 \ub3c5 \ubb3c\ubc29\uc6b8\uc5d0\uc11c \uc704\ubbf8\ub974\uc640 \ud568\uaed8 \uc544\uc6b0\ub460\ube14\ub77c\ub77c\ub294 \uc554\uc18c\uac00 \ud0dc\uc5b4\ub0ac\ub2e4\uace0 \ub9d0\ud55c\ub2e4. \uc544\uc6b0\ub460\ube14\ub77c\uc758 \uc816\ud1b5\uc5d0\uc11c \uc6b0\uc720\uac00 \ub124 \uc904\uae30\uc758 \uac15\uc744 \uc774\ub8e8\uba70 \ud758\ub800\uace0, \uc704\ubbf8\ub974\ub294 \uadf8 \uc6b0\uc720\ub97c \uba39\uc5c8\ub2e4. \uac15\uae00\ub808\ub9ac\ub294 \uadf8 \uc554\uc18c\ub294 \ubb34\uc5c7\uc744 \uba39\uc5c8\ub0d0\uace0 \ubb3b\ub294\ub2e4. \ub192\uc73c\uc2e0 \ubd84\uc740 \uc554\uc18c\ub294 \uc18c\uae08\uae30 \uc788\ub294 \uc5bc\uc74c \ub369\uc5b4\ub9ac\ub97c \ud565\uc558\ub2e4\uace0 \ub2f5\ud55c\ub2e4. \uc544\uc6b0\ub460\ube14\ub77c\uac00 \uc5bc\uc74c \ub369\uc5b4\ub9ac\ub97c \ud565\uc740 \uccab \ub0a0\uc5d0\ub294 \uc5bc\uc74c \uc18d\uc5d0\uc11c \ub0a8\uc790\uc758 \uba38\ub9ac\uce74\ub77d\uc774 \ubcf4\uc600\uace0, \ub458\uc9f8 \ub0a0\uc5d0\ub294 \uba38\ub9ac\uac00 \ubcf4\uc600\uace0, \uc14b\uc9f8 \ub0a0\uc5d0\ub294 \uc804\uccb4 \ubab8\uc774 \ub4dc\ub7ec\ub0ac\ub2e4. \uc774 \ub0a8\uc790\uc758 \uc774\ub984\uc740 \ubd80\ub9ac\uc600\ub2e4. \uadf8 \uc5ed\uc2dc \uac70\ub300\ud558\uace0 \uac15\ub825\ud588\uc73c\uba70, \uc678\ubaa8\ub294 \uc544\ub984\ub2e4\uc6e0\ub2e4\uace0 \ub9d0\ud55c\ub2e4. \ubd80\ub9ac\ub294 \ubcf4\ub974\ub97c \ub0b3\uc558\uace0 \ubcf4\ub974\ub294 \ubcfc\uc18c\ub978\uc758 \ub538\uc778 \uc5ec\uc131 \uc694\ud230 \ubca0\uc2a4\ud2c0\ub77c\uc640 \uacb0\ud63c\ud558\uc5ec \uc624\ub518, \ube4c\ub9ac, \ubca0\uc774\ub77c\ub294 3\ud615\uc81c\ub97c \ub0b3\uc558\ub2e4. \ub192\uc73c\uc2e0 \ubd84\uc740 \u201c\uc624\ub518\uacfc \uadf8\uc758 \ud615\uc81c\ub4e4\uc740 \ud558\ub298\uacfc \ub545\uc758 \uc9c0\ubc30\uc790\uac00 \ub418\uc5b4\uc57c\ub9cc \ud588\ub2e4. \uadf8\ub294 \uadf8\ub807\uac8c \ub418\uc5c8\uc5b4\uc57c \ud55c\ub2e4\ub294 \uac83\uc774 \uc6b0\ub9ac\uc758 \uc758\uacac\uc774\ub2e4. \uc624\ub518\uc774\ub780 \uc6b0\ub9ac\uac00 \uc544\ub294 \uac00\uc7a5 \uc704\ub300\ud558\uace0 \uc601\uad11\uc2a4\ub7ec\uc6b4 \uc874\uc7ac\uc758 \uc774\ub984\uc774\uba70, \uadf8\ub300 \uc5ed\uc2dc \uadf8\ub97c \uadf8\ub807\uac8c \ubd80\ub974\ub294 \uac83\uc5d0 \ub300\ud574 \uae30\uaebc\uc774 \ub3d9\uc758\ud558\ub9ac\ub77c \ubcf8\ub2e4\u201d\uace0 \ub9d0\ud55c\ub2e4."} {"question": "\uc624\ub178\uac00 \ub300\ud559 \uc2dc\uc808 \ud070 \uc601\ud5a5\uc744 \ubc1b\uc558\ub358 \uc2e4\uc874\uc8fc\uc758 \uc0ac\uc0c1\uac00\ub294?", "paragraph": "\uc624\ub178\ub294 \uac1c\ub150 \ubbf8\uc220\uacfc \ud589\uc704 \uc608\uc220\uc744 \ud0d0\uad6c\ud558\uc600\ub2e4. 1964\ub144 \ub3c4\ucfc4\uc758 \uc18c\uac8c\uce20 \uc544\ud2b8 \uc13c\ud130\uc5d0\uc11c \uacf5\uc5f0\ub41c \"\uc870\uac01\ub0b4\uae30\"(\uc601\uc5b4 Cut piece)\ub294 \uadf8\ub140\uc758 \ub300\ud45c\uc801\uc778 \ud589\uc704 \uc608\uc220 \uc911 \ud558\ub098\ub85c \uc774\ub984 \ub9cc\ud07c\uc774\ub098 \ud30c\uad34\uc801\uc778 \ub0b4\uc6a9\uc744 \ub2f4\uace0 \uc788\uc5c8\ub2e4. \uadf8\ub140\ub294 \ubb34\ub300\ub97c \uac77\ub2e4 \uc637\uc790\ub77d\uc5d0 \uac78\ub824 \ubb34\ub98e\uc744 \uafc7\uace0 \uad00\uc911\uc758 \uc694\uad6c\uc5d0 \ub530\ub77c \uc54c\ubab8\uc774 \ub420 \ub54c\uae4c\uc9c0 \uac78\uce5c \uc637\uc744 \uc798\ub77c\ub0b4\uc5c8\ub2e4. \uc624\ub178\ub294 \ub300\ud559 \uc2dc\uc808 \uc7a5 \ud3f4 \uc0ac\ub974\ud2b8\ub974\uc758 \uc2e4\uc874\uc8fc\uc758 \uc0ac\uc0c1\uc73c\ub85c\ubd80\ud130 \ud070 \uc601\ud5a5\uc744 \ubc1b\uc558\uc73c\uba70, \uc790\uae30 \uc2a4\uc2a4\ub85c\uc758 \uc778\uac04\uc801 \uace0\ub1cc\ub97c \ub2ec\ub798\ub294 \uc791\ud488\uc744 \uac08\ub9dd\ud574\uc654\ub2e4. \uadf8\ub7f0 \uc774\uc720\ub85c \uc624\ub178\ub294 \uc790\uc2e0\uc758 \uc608\uc220 \uc791\ud488\uc744 \ud1b5\ud574 \uc790\uc2e0\uc758 \uad00\uc810\uacfc \uac1c\uc131\uc744 \uc628\uc804\ud788 \ud45c\ucd9c\ud558\uace0\uc790 \ud558\uc600\ub2e4. \uadf8\ub140\uc758 \ub300\ud45c\uc791 \uc911 \ud558\ub098\uac00\ub41c \"\uc870\uac01\ub0b4\uae30\"\uc758 \uacbd\uc6b0, \uc624\ub178\ub294 \uc791\ud488\uc744 \ud1b5\ud574 \uc0ac\ud68c\uc801 \uc5f0\ub300\uc640 \uc0ac\ub791\uc744 \uc8fc\uc694 \uc8fc\uc81c\ub85c \ud558\uc5ec \uc131(\u6027)\uacfc \uc131\ucc28\ubcc4, \uadf8\ub9ac\uace0 \uc778\uac04\uc758 \uace0\ub1cc\uc640 \uace0\ub3c5\uc73c\ub85c \uc778\ud55c \uac70\ub300\ud55c \uc6b0\uc8fc\uc801\uc778 \uace0\ud1b5\uc744 \ub2e4\ub8e8\uace0 \uc788\ub2e4. \uc624\ub178\ub294 \"\uc870\uac01\ub0b4\uae30\"\ub97c \ub7f0\ub358\uc5d0\uc11c\ub3c4 \uacf5\uc5f0\ud558\uc600\ub2e4. \uc77c\ubcf8\uc758 \uad00\uac1d\uc774 \uc18c\uadf9\uc801\uc774\uace0 \uc870\uc2ec\uc2a4\ub7ec\uc6e0\ub358 \ub370 \ubc18\ud574 \uc601\uad6d\uc758 \uad00\uac1d\ub4e4\uc740 \uc801\uadf9\uc801\uc73c\ub85c \ud37c\ud3ec\uba3c\uc2a4\uc5d0 \ucc38\uc5ec\ud558\uba70 \uc5f4\uad11\ud558\uc600\ub2e4. \ub7f0\ub358\uc5d0\uc11c\ub294 \uad00\uac1d\ub4e4\uc774 \uc798\ub824\ub098\uac04 \uc624\ub178\uc758 \uc637\uc744 \uc9d1\uae30\uc704\ud574 \ubab0\ub824\ub4e4\uc5b4 \uc624\ub178\ub294 \ubcf4\uc548\uc694\uc6d0\uc758 \ubcf4\ud638\ub97c \ubc1b\uc544\uc57c \ud560 \uc815\ub3c4\uc600\ub2e4.", "answer": "\uc7a5 \ud3f4 \uc0ac\ub974\ud2b8\ub974", "sentence": "\uc624\ub178\ub294 \ub300\ud559 \uc2dc\uc808 \uc7a5 \ud3f4 \uc0ac\ub974\ud2b8\ub974 \uc758 \uc2e4\uc874\uc8fc\uc758 \uc0ac\uc0c1\uc73c\ub85c\ubd80\ud130 \ud070 \uc601\ud5a5\uc744 \ubc1b\uc558\uc73c\uba70, \uc790\uae30 \uc2a4\uc2a4\ub85c\uc758 \uc778\uac04\uc801 \uace0\ub1cc\ub97c \ub2ec\ub798\ub294 \uc791\ud488\uc744 \uac08\ub9dd\ud574\uc654\ub2e4.", "paragraph_sentence": "\uc624\ub178\ub294 \uac1c\ub150 \ubbf8\uc220\uacfc \ud589\uc704 \uc608\uc220\uc744 \ud0d0\uad6c\ud558\uc600\ub2e4. 1964\ub144 \ub3c4\ucfc4\uc758 \uc18c\uac8c\uce20 \uc544\ud2b8 \uc13c\ud130\uc5d0\uc11c \uacf5\uc5f0\ub41c \"\uc870\uac01\ub0b4\uae30\"(\uc601\uc5b4 Cut piece)\ub294 \uadf8\ub140\uc758 \ub300\ud45c\uc801\uc778 \ud589\uc704 \uc608\uc220 \uc911 \ud558\ub098\ub85c \uc774\ub984 \ub9cc\ud07c\uc774\ub098 \ud30c\uad34\uc801\uc778 \ub0b4\uc6a9\uc744 \ub2f4\uace0 \uc788\uc5c8\ub2e4. \uadf8\ub140\ub294 \ubb34\ub300\ub97c \uac77\ub2e4 \uc637\uc790\ub77d\uc5d0 \uac78\ub824 \ubb34\ub98e\uc744 \uafc7\uace0 \uad00\uc911\uc758 \uc694\uad6c\uc5d0 \ub530\ub77c \uc54c\ubab8\uc774 \ub420 \ub54c\uae4c\uc9c0 \uac78\uce5c \uc637\uc744 \uc798\ub77c\ub0b4\uc5c8\ub2e4. \uc624\ub178\ub294 \ub300\ud559 \uc2dc\uc808 \uc7a5 \ud3f4 \uc0ac\ub974\ud2b8\ub974 \uc758 \uc2e4\uc874\uc8fc\uc758 \uc0ac\uc0c1\uc73c\ub85c\ubd80\ud130 \ud070 \uc601\ud5a5\uc744 \ubc1b\uc558\uc73c\uba70, \uc790\uae30 \uc2a4\uc2a4\ub85c\uc758 \uc778\uac04\uc801 \uace0\ub1cc\ub97c \ub2ec\ub798\ub294 \uc791\ud488\uc744 \uac08\ub9dd\ud574\uc654\ub2e4. \uadf8\ub7f0 \uc774\uc720\ub85c \uc624\ub178\ub294 \uc790\uc2e0\uc758 \uc608\uc220 \uc791\ud488\uc744 \ud1b5\ud574 \uc790\uc2e0\uc758 \uad00\uc810\uacfc \uac1c\uc131\uc744 \uc628\uc804\ud788 \ud45c\ucd9c\ud558\uace0\uc790 \ud558\uc600\ub2e4. \uadf8\ub140\uc758 \ub300\ud45c\uc791 \uc911 \ud558\ub098\uac00\ub41c \"\uc870\uac01\ub0b4\uae30\"\uc758 \uacbd\uc6b0, \uc624\ub178\ub294 \uc791\ud488\uc744 \ud1b5\ud574 \uc0ac\ud68c\uc801 \uc5f0\ub300\uc640 \uc0ac\ub791\uc744 \uc8fc\uc694 \uc8fc\uc81c\ub85c \ud558\uc5ec \uc131(\u6027)\uacfc \uc131\ucc28\ubcc4, \uadf8\ub9ac\uace0 \uc778\uac04\uc758 \uace0\ub1cc\uc640 \uace0\ub3c5\uc73c\ub85c \uc778\ud55c \uac70\ub300\ud55c \uc6b0\uc8fc\uc801\uc778 \uace0\ud1b5\uc744 \ub2e4\ub8e8\uace0 \uc788\ub2e4. \uc624\ub178\ub294 \"\uc870\uac01\ub0b4\uae30\"\ub97c \ub7f0\ub358\uc5d0\uc11c\ub3c4 \uacf5\uc5f0\ud558\uc600\ub2e4. \uc77c\ubcf8\uc758 \uad00\uac1d\uc774 \uc18c\uadf9\uc801\uc774\uace0 \uc870\uc2ec\uc2a4\ub7ec\uc6e0\ub358 \ub370 \ubc18\ud574 \uc601\uad6d\uc758 \uad00\uac1d\ub4e4\uc740 \uc801\uadf9\uc801\uc73c\ub85c \ud37c\ud3ec\uba3c\uc2a4\uc5d0 \ucc38\uc5ec\ud558\uba70 \uc5f4\uad11\ud558\uc600\ub2e4. \ub7f0\ub358\uc5d0\uc11c\ub294 \uad00\uac1d\ub4e4\uc774 \uc798\ub824\ub098\uac04 \uc624\ub178\uc758 \uc637\uc744 \uc9d1\uae30\uc704\ud574 \ubab0\ub824\ub4e4\uc5b4 \uc624\ub178\ub294 \ubcf4\uc548\uc694\uc6d0\uc758 \ubcf4\ud638\ub97c \ubc1b\uc544\uc57c \ud560 \uc815\ub3c4\uc600\ub2e4.", "paragraph_answer": "\uc624\ub178\ub294 \uac1c\ub150 \ubbf8\uc220\uacfc \ud589\uc704 \uc608\uc220\uc744 \ud0d0\uad6c\ud558\uc600\ub2e4. 1964\ub144 \ub3c4\ucfc4\uc758 \uc18c\uac8c\uce20 \uc544\ud2b8 \uc13c\ud130\uc5d0\uc11c \uacf5\uc5f0\ub41c \"\uc870\uac01\ub0b4\uae30\"(\uc601\uc5b4 Cut piece)\ub294 \uadf8\ub140\uc758 \ub300\ud45c\uc801\uc778 \ud589\uc704 \uc608\uc220 \uc911 \ud558\ub098\ub85c \uc774\ub984 \ub9cc\ud07c\uc774\ub098 \ud30c\uad34\uc801\uc778 \ub0b4\uc6a9\uc744 \ub2f4\uace0 \uc788\uc5c8\ub2e4. \uadf8\ub140\ub294 \ubb34\ub300\ub97c \uac77\ub2e4 \uc637\uc790\ub77d\uc5d0 \uac78\ub824 \ubb34\ub98e\uc744 \uafc7\uace0 \uad00\uc911\uc758 \uc694\uad6c\uc5d0 \ub530\ub77c \uc54c\ubab8\uc774 \ub420 \ub54c\uae4c\uc9c0 \uac78\uce5c \uc637\uc744 \uc798\ub77c\ub0b4\uc5c8\ub2e4. \uc624\ub178\ub294 \ub300\ud559 \uc2dc\uc808 \uc7a5 \ud3f4 \uc0ac\ub974\ud2b8\ub974 \uc758 \uc2e4\uc874\uc8fc\uc758 \uc0ac\uc0c1\uc73c\ub85c\ubd80\ud130 \ud070 \uc601\ud5a5\uc744 \ubc1b\uc558\uc73c\uba70, \uc790\uae30 \uc2a4\uc2a4\ub85c\uc758 \uc778\uac04\uc801 \uace0\ub1cc\ub97c \ub2ec\ub798\ub294 \uc791\ud488\uc744 \uac08\ub9dd\ud574\uc654\ub2e4. \uadf8\ub7f0 \uc774\uc720\ub85c \uc624\ub178\ub294 \uc790\uc2e0\uc758 \uc608\uc220 \uc791\ud488\uc744 \ud1b5\ud574 \uc790\uc2e0\uc758 \uad00\uc810\uacfc \uac1c\uc131\uc744 \uc628\uc804\ud788 \ud45c\ucd9c\ud558\uace0\uc790 \ud558\uc600\ub2e4. \uadf8\ub140\uc758 \ub300\ud45c\uc791 \uc911 \ud558\ub098\uac00\ub41c \"\uc870\uac01\ub0b4\uae30\"\uc758 \uacbd\uc6b0, \uc624\ub178\ub294 \uc791\ud488\uc744 \ud1b5\ud574 \uc0ac\ud68c\uc801 \uc5f0\ub300\uc640 \uc0ac\ub791\uc744 \uc8fc\uc694 \uc8fc\uc81c\ub85c \ud558\uc5ec \uc131(\u6027)\uacfc \uc131\ucc28\ubcc4, \uadf8\ub9ac\uace0 \uc778\uac04\uc758 \uace0\ub1cc\uc640 \uace0\ub3c5\uc73c\ub85c \uc778\ud55c \uac70\ub300\ud55c \uc6b0\uc8fc\uc801\uc778 \uace0\ud1b5\uc744 \ub2e4\ub8e8\uace0 \uc788\ub2e4. \uc624\ub178\ub294 \"\uc870\uac01\ub0b4\uae30\"\ub97c \ub7f0\ub358\uc5d0\uc11c\ub3c4 \uacf5\uc5f0\ud558\uc600\ub2e4. \uc77c\ubcf8\uc758 \uad00\uac1d\uc774 \uc18c\uadf9\uc801\uc774\uace0 \uc870\uc2ec\uc2a4\ub7ec\uc6e0\ub358 \ub370 \ubc18\ud574 \uc601\uad6d\uc758 \uad00\uac1d\ub4e4\uc740 \uc801\uadf9\uc801\uc73c\ub85c \ud37c\ud3ec\uba3c\uc2a4\uc5d0 \ucc38\uc5ec\ud558\uba70 \uc5f4\uad11\ud558\uc600\ub2e4. \ub7f0\ub358\uc5d0\uc11c\ub294 \uad00\uac1d\ub4e4\uc774 \uc798\ub824\ub098\uac04 \uc624\ub178\uc758 \uc637\uc744 \uc9d1\uae30\uc704\ud574 \ubab0\ub824\ub4e4\uc5b4 \uc624\ub178\ub294 \ubcf4\uc548\uc694\uc6d0\uc758 \ubcf4\ud638\ub97c \ubc1b\uc544\uc57c \ud560 \uc815\ub3c4\uc600\ub2e4.", "sentence_answer": "\uc624\ub178\ub294 \ub300\ud559 \uc2dc\uc808 \uc7a5 \ud3f4 \uc0ac\ub974\ud2b8\ub974 \uc758 \uc2e4\uc874\uc8fc\uc758 \uc0ac\uc0c1\uc73c\ub85c\ubd80\ud130 \ud070 \uc601\ud5a5\uc744 \ubc1b\uc558\uc73c\uba70, \uc790\uae30 \uc2a4\uc2a4\ub85c\uc758 \uc778\uac04\uc801 \uace0\ub1cc\ub97c \ub2ec\ub798\ub294 \uc791\ud488\uc744 \uac08\ub9dd\ud574\uc654\ub2e4.", "paragraph_id": "6541260-0-1", "paragraph_question": "question: \uc624\ub178\uac00 \ub300\ud559 \uc2dc\uc808 \ud070 \uc601\ud5a5\uc744 \ubc1b\uc558\ub358 \uc2e4\uc874\uc8fc\uc758 \uc0ac\uc0c1\uac00\ub294?, context: \uc624\ub178\ub294 \uac1c\ub150 \ubbf8\uc220\uacfc \ud589\uc704 \uc608\uc220\uc744 \ud0d0\uad6c\ud558\uc600\ub2e4. 1964\ub144 \ub3c4\ucfc4\uc758 \uc18c\uac8c\uce20 \uc544\ud2b8 \uc13c\ud130\uc5d0\uc11c \uacf5\uc5f0\ub41c \"\uc870\uac01\ub0b4\uae30\"(\uc601\uc5b4 Cut piece)\ub294 \uadf8\ub140\uc758 \ub300\ud45c\uc801\uc778 \ud589\uc704 \uc608\uc220 \uc911 \ud558\ub098\ub85c \uc774\ub984 \ub9cc\ud07c\uc774\ub098 \ud30c\uad34\uc801\uc778 \ub0b4\uc6a9\uc744 \ub2f4\uace0 \uc788\uc5c8\ub2e4. \uadf8\ub140\ub294 \ubb34\ub300\ub97c \uac77\ub2e4 \uc637\uc790\ub77d\uc5d0 \uac78\ub824 \ubb34\ub98e\uc744 \uafc7\uace0 \uad00\uc911\uc758 \uc694\uad6c\uc5d0 \ub530\ub77c \uc54c\ubab8\uc774 \ub420 \ub54c\uae4c\uc9c0 \uac78\uce5c \uc637\uc744 \uc798\ub77c\ub0b4\uc5c8\ub2e4. \uc624\ub178\ub294 \ub300\ud559 \uc2dc\uc808 \uc7a5 \ud3f4 \uc0ac\ub974\ud2b8\ub974\uc758 \uc2e4\uc874\uc8fc\uc758 \uc0ac\uc0c1\uc73c\ub85c\ubd80\ud130 \ud070 \uc601\ud5a5\uc744 \ubc1b\uc558\uc73c\uba70, \uc790\uae30 \uc2a4\uc2a4\ub85c\uc758 \uc778\uac04\uc801 \uace0\ub1cc\ub97c \ub2ec\ub798\ub294 \uc791\ud488\uc744 \uac08\ub9dd\ud574\uc654\ub2e4. \uadf8\ub7f0 \uc774\uc720\ub85c \uc624\ub178\ub294 \uc790\uc2e0\uc758 \uc608\uc220 \uc791\ud488\uc744 \ud1b5\ud574 \uc790\uc2e0\uc758 \uad00\uc810\uacfc \uac1c\uc131\uc744 \uc628\uc804\ud788 \ud45c\ucd9c\ud558\uace0\uc790 \ud558\uc600\ub2e4. \uadf8\ub140\uc758 \ub300\ud45c\uc791 \uc911 \ud558\ub098\uac00\ub41c \"\uc870\uac01\ub0b4\uae30\"\uc758 \uacbd\uc6b0, \uc624\ub178\ub294 \uc791\ud488\uc744 \ud1b5\ud574 \uc0ac\ud68c\uc801 \uc5f0\ub300\uc640 \uc0ac\ub791\uc744 \uc8fc\uc694 \uc8fc\uc81c\ub85c \ud558\uc5ec \uc131(\u6027)\uacfc \uc131\ucc28\ubcc4, \uadf8\ub9ac\uace0 \uc778\uac04\uc758 \uace0\ub1cc\uc640 \uace0\ub3c5\uc73c\ub85c \uc778\ud55c \uac70\ub300\ud55c \uc6b0\uc8fc\uc801\uc778 \uace0\ud1b5\uc744 \ub2e4\ub8e8\uace0 \uc788\ub2e4. \uc624\ub178\ub294 \"\uc870\uac01\ub0b4\uae30\"\ub97c \ub7f0\ub358\uc5d0\uc11c\ub3c4 \uacf5\uc5f0\ud558\uc600\ub2e4. \uc77c\ubcf8\uc758 \uad00\uac1d\uc774 \uc18c\uadf9\uc801\uc774\uace0 \uc870\uc2ec\uc2a4\ub7ec\uc6e0\ub358 \ub370 \ubc18\ud574 \uc601\uad6d\uc758 \uad00\uac1d\ub4e4\uc740 \uc801\uadf9\uc801\uc73c\ub85c \ud37c\ud3ec\uba3c\uc2a4\uc5d0 \ucc38\uc5ec\ud558\uba70 \uc5f4\uad11\ud558\uc600\ub2e4. \ub7f0\ub358\uc5d0\uc11c\ub294 \uad00\uac1d\ub4e4\uc774 \uc798\ub824\ub098\uac04 \uc624\ub178\uc758 \uc637\uc744 \uc9d1\uae30\uc704\ud574 \ubab0\ub824\ub4e4\uc5b4 \uc624\ub178\ub294 \ubcf4\uc548\uc694\uc6d0\uc758 \ubcf4\ud638\ub97c \ubc1b\uc544\uc57c \ud560 \uc815\ub3c4\uc600\ub2e4."} {"question": "\ud1a0\uc9c0\uacf5\uac1c\ub150\uc774 \uc18c\uc124\uc5d0\ub098 \ub098\uc624\ub294 \ub9d0\uc774\ub77c\uba70 \ubc18\ub300 \uc758\uacac\uc744 \ud53c\ub825\ud588\ub358 \uc778\ubb3c\uc740?", "paragraph": "\ub178\ud0dc\uc6b0 \uc815\ubd80 \uc2dc\uc808 \uc9d1\uac12,\ub545\uac12 \uae09\ub4f1\uc744 \ub9c9\uae30 \uc704\ud574 \ud1a0\uc9c0\uacf5\uac1c\ub150\uc81c\uac00 \uc2dc\ub3c4\ub410\uc73c\ub098 \uc904\uc904\uc774 \uc704\ud5cc \uc18c\uc1a1 \ub4f1\uc73c\ub85c \uc88c\ucd08\ub410\ub2e4. \ub2f9\uc2dc \uccad\uc640\ub300 \uacbd\uc81c\uc218\uc11d\uc744 \uc9c0\ub0b8 \uae40\uc885\uc778\uc740 \ud1a0\uc9c0\uacf5\uac1c\ub150\uc744 \ub3c4\uc785\ud55c \uc8fc\uccb4\ub85c \uc5ec\uaca8\uc84c\uc5c8\ub2e4. \uadf8\ub7ec\ub098 \uc774\ub7f0 \uc138\uac04\uc758 \uc778\uc2dd\uacfc\ub294 \ub2ec\ub9ac \uae40\uc885\uc778\uc740, 2005\ub144 \ubd80\ub3d9\uc0b0 \ud22c\uae30 \uc5f4\ud48d \ub54c \uc5f4\ub9b0\uc6b0\ub9ac\ub2f9\uc774 \uc7ac\ub3c4\uc785\ud558\ub824\ub358 \ud1a0\uc9c0\uacf5\uac1c\ub150\uc774 \uad81\uc5ec\uc9c0\ucc45\uc774\ub77c \ud3c4\ud558\ud558\uace0 \ub2e4\uc74c\uacfc \uac19\uc774 \ub9d0\ud588\ub2e4. \"\ud1a0\uc9c0\uacf5\uac1c\ub150\uc740 1989\ub144 \ub3c4\uc785 \ub17c\uc758 \ub2f9\uc2dc \uacbd\uc81c\uae30\ud68d\uc6d0\uc774 \ucc3d\uc791\ud55c \ub2e8\uc5b4\ub85c \uacbd\uc81c\ud559\uc5d0\ub3c4 \uc5c6\ub294 \uac1c\ub150\uc774\uba70, \uad6d\uc720\uc9c0\u00b7\uc0ac\uc720\uc9c0\ub294 \uc788\uc744 \uc218 \uc788\uc5b4\ub3c4 \ud1a0\uc9c0\uacf5\uac1c\ub150\uc740 \uc18c\uc124 \uc18d\uc5d0\uc11c\ub098 \ub098\uc62c \uc218 \uc788\ub294 \ub9d0\"\uc774\ub77c\uace0 \ubc1d\ud614\ub2e4. \uc774\uc5b4 \"\ub098\ub294 \ubcf4\uac74\uc0ac\ud68c\ubd80 \uc7a5\uad00\uc73c\ub85c \uc788\uc5c8\ub294\ub370 \ud1a0\uc9c0\uacf5\uac1c\ub150\uc740 \uc808\ub300\ub85c \uc548\ub41c\ub2e4\uace0 \ubc18\ub300\ud588\uc5c8\ub2e4\"\uba70 \"\ub3c4\uc785\uc744 \uc8fc\uc7a5\ud588\ub358 \uc0ac\ub78c\ub3c4 \uadfc\uac70\ub97c \uc81c\ub300\ub85c \uc124\uba85\uc740 \ubabb\ud55c \ucc44 \ud1a0\uc9c0\uacf5\uae09 \ud655\ub300\uac00 \uc5b4\ub835\ub2e4\ub294 \uba85\ubd84\ub9cc \ub0b4\uc138\uc6e0\ub2e4\"\uace0 \ub367\ubd99\uc600\ub2e4. \"\ud1a0\uc9c0\uacf5\uac1c\ub150\uc740 \uc138\uc81c\uc815\ucc45\uc73c\ub85c \uc77c\uc2dc\uc801 \ud6a8\uacfc\ub97c \uac70\ub458 \uc218\ub294 \uc788\uaca0\uc9c0\ub9cc \uacb0\uad6d\uc2dc\uc7a5\uc774 \uc801\uc751\ud558\ub294 \ubc29\ubc95\uc744 \ucc3e\uac8c \ub41c\ub2e4\"\uba70 \"\ud1a0\uc9c0\ucd08\uacfc\uc774\ub4dd\uc138 \ub4f1\uc740 \uc808\ub300\ub85c \ubd80\uacfc\ud560 \uc218 \uc5c6\ub294 \uc138\uae08\uc73c\ub85c \uc704\ud5cc \uc18c\uc9c0\uac00 \uc788\uc73c\uba70 '\uac1c\ubc1c\uc774\uc775\ud658\uc218\uc81c'\ub3c4 \uacb0\uad6d \ud1a0\uc9c0\uac12\uc73c\ub85c \uc804\uac00\ub418\uac8c \ub41c\ub2e4\"\uace0 \uac15\uc870\ud588\ub2e4. \uadf8\ub294 \ubd80\ub3d9\uc0b0 \ud22c\uae30\ub97c \uc5b5\uc81c\ud560 \uadfc\ubcf8 \ubc29\ucc45\uc740 \"\uc138\uc81c\ub85c \ud22c\uae30\uc5b5\uc81c\ub97c \ud558\uaca0\ub2e4\ub294 \ubc1c\uc0c1\uc740 \uc798\ubabb\ub410\uace0 \uae08\ub9ac\uc778\uc0c1 \ub4f1 \uacbd\uc81c\uc815\ucc45 \uc804\ubc18\uc744 \uc798 \uc870\uc728\ud574\uc11c \ud574\ubc95\uc744 \ucc3e\uc544\ub098\uac00\uc57c \ud55c\ub2e4\"\uace0 \uc8fc\uc7a5\ud588\ub2e4. \ub610\ud55c \uae08\uc735\uc2e4\uba85\uc81c\ub3c4 \uac15\ub825 \ubc18\ub300\ud55c \uac83\uc73c\ub85c \ubcf4\ub3c4\ub41c \ubc14 \uc788\ub2e4.", "answer": "\uae40\uc885\uc778", "sentence": "\ub2f9\uc2dc \uccad\uc640\ub300 \uacbd\uc81c\uc218\uc11d\uc744 \uc9c0\ub0b8 \uae40\uc885\uc778 \uc740 \ud1a0\uc9c0\uacf5\uac1c\ub150\uc744 \ub3c4\uc785\ud55c \uc8fc\uccb4\ub85c \uc5ec\uaca8\uc84c\uc5c8\ub2e4.", "paragraph_sentence": "\ub178\ud0dc\uc6b0 \uc815\ubd80 \uc2dc\uc808 \uc9d1\uac12,\ub545\uac12 \uae09\ub4f1\uc744 \ub9c9\uae30 \uc704\ud574 \ud1a0\uc9c0\uacf5\uac1c\ub150\uc81c\uac00 \uc2dc\ub3c4\ub410\uc73c\ub098 \uc904\uc904\uc774 \uc704\ud5cc \uc18c\uc1a1 \ub4f1\uc73c\ub85c \uc88c\ucd08\ub410\ub2e4. \ub2f9\uc2dc \uccad\uc640\ub300 \uacbd\uc81c\uc218\uc11d\uc744 \uc9c0\ub0b8 \uae40\uc885\uc778 \uc740 \ud1a0\uc9c0\uacf5\uac1c\ub150\uc744 \ub3c4\uc785\ud55c \uc8fc\uccb4\ub85c \uc5ec\uaca8\uc84c\uc5c8\ub2e4. \uadf8\ub7ec\ub098 \uc774\ub7f0 \uc138\uac04\uc758 \uc778\uc2dd\uacfc\ub294 \ub2ec\ub9ac \uae40\uc885\uc778\uc740, 2005\ub144 \ubd80\ub3d9\uc0b0 \ud22c\uae30 \uc5f4\ud48d \ub54c \uc5f4\ub9b0\uc6b0\ub9ac\ub2f9\uc774 \uc7ac\ub3c4\uc785\ud558\ub824\ub358 \ud1a0\uc9c0\uacf5\uac1c\ub150\uc774 \uad81\uc5ec\uc9c0\ucc45\uc774\ub77c \ud3c4\ud558\ud558\uace0 \ub2e4\uc74c\uacfc \uac19\uc774 \ub9d0\ud588\ub2e4. \"\ud1a0\uc9c0\uacf5\uac1c\ub150\uc740 1989\ub144 \ub3c4\uc785 \ub17c\uc758 \ub2f9\uc2dc \uacbd\uc81c\uae30\ud68d\uc6d0\uc774 \ucc3d\uc791\ud55c \ub2e8\uc5b4\ub85c \uacbd\uc81c\ud559\uc5d0\ub3c4 \uc5c6\ub294 \uac1c\ub150\uc774\uba70, \uad6d\uc720\uc9c0\u00b7\uc0ac\uc720\uc9c0\ub294 \uc788\uc744 \uc218 \uc788\uc5b4\ub3c4 \ud1a0\uc9c0\uacf5\uac1c\ub150\uc740 \uc18c\uc124 \uc18d\uc5d0\uc11c\ub098 \ub098\uc62c \uc218 \uc788\ub294 \ub9d0\"\uc774\ub77c\uace0 \ubc1d\ud614\ub2e4. \uc774\uc5b4 \"\ub098\ub294 \ubcf4\uac74\uc0ac\ud68c\ubd80 \uc7a5\uad00\uc73c\ub85c \uc788\uc5c8\ub294\ub370 \ud1a0\uc9c0\uacf5\uac1c\ub150\uc740 \uc808\ub300\ub85c \uc548\ub41c\ub2e4\uace0 \ubc18\ub300\ud588\uc5c8\ub2e4\"\uba70 \"\ub3c4\uc785\uc744 \uc8fc\uc7a5\ud588\ub358 \uc0ac\ub78c\ub3c4 \uadfc\uac70\ub97c \uc81c\ub300\ub85c \uc124\uba85\uc740 \ubabb\ud55c \ucc44 \ud1a0\uc9c0\uacf5\uae09 \ud655\ub300\uac00 \uc5b4\ub835\ub2e4\ub294 \uba85\ubd84\ub9cc \ub0b4\uc138\uc6e0\ub2e4\"\uace0 \ub367\ubd99\uc600\ub2e4. \"\ud1a0\uc9c0\uacf5\uac1c\ub150\uc740 \uc138\uc81c\uc815\ucc45\uc73c\ub85c \uc77c\uc2dc\uc801 \ud6a8\uacfc\ub97c \uac70\ub458 \uc218\ub294 \uc788\uaca0\uc9c0\ub9cc \uacb0\uad6d\uc2dc\uc7a5\uc774 \uc801\uc751\ud558\ub294 \ubc29\ubc95\uc744 \ucc3e\uac8c \ub41c\ub2e4\"\uba70 \"\ud1a0\uc9c0\ucd08\uacfc\uc774\ub4dd\uc138 \ub4f1\uc740 \uc808\ub300\ub85c \ubd80\uacfc\ud560 \uc218 \uc5c6\ub294 \uc138\uae08\uc73c\ub85c \uc704\ud5cc \uc18c\uc9c0\uac00 \uc788\uc73c\uba70 '\uac1c\ubc1c\uc774\uc775\ud658\uc218\uc81c'\ub3c4 \uacb0\uad6d \ud1a0\uc9c0\uac12\uc73c\ub85c \uc804\uac00\ub418\uac8c \ub41c\ub2e4\"\uace0 \uac15\uc870\ud588\ub2e4. \uadf8\ub294 \ubd80\ub3d9\uc0b0 \ud22c\uae30\ub97c \uc5b5\uc81c\ud560 \uadfc\ubcf8 \ubc29\ucc45\uc740 \"\uc138\uc81c\ub85c \ud22c\uae30\uc5b5\uc81c\ub97c \ud558\uaca0\ub2e4\ub294 \ubc1c\uc0c1\uc740 \uc798\ubabb\ub410\uace0 \uae08\ub9ac\uc778\uc0c1 \ub4f1 \uacbd\uc81c\uc815\ucc45 \uc804\ubc18\uc744 \uc798 \uc870\uc728\ud574\uc11c \ud574\ubc95\uc744 \ucc3e\uc544\ub098\uac00\uc57c \ud55c\ub2e4\"\uace0 \uc8fc\uc7a5\ud588\ub2e4. \ub610\ud55c \uae08\uc735\uc2e4\uba85\uc81c\ub3c4 \uac15\ub825 \ubc18\ub300\ud55c \uac83\uc73c\ub85c \ubcf4\ub3c4\ub41c \ubc14 \uc788\ub2e4.", "paragraph_answer": "\ub178\ud0dc\uc6b0 \uc815\ubd80 \uc2dc\uc808 \uc9d1\uac12,\ub545\uac12 \uae09\ub4f1\uc744 \ub9c9\uae30 \uc704\ud574 \ud1a0\uc9c0\uacf5\uac1c\ub150\uc81c\uac00 \uc2dc\ub3c4\ub410\uc73c\ub098 \uc904\uc904\uc774 \uc704\ud5cc \uc18c\uc1a1 \ub4f1\uc73c\ub85c \uc88c\ucd08\ub410\ub2e4. \ub2f9\uc2dc \uccad\uc640\ub300 \uacbd\uc81c\uc218\uc11d\uc744 \uc9c0\ub0b8 \uae40\uc885\uc778 \uc740 \ud1a0\uc9c0\uacf5\uac1c\ub150\uc744 \ub3c4\uc785\ud55c \uc8fc\uccb4\ub85c \uc5ec\uaca8\uc84c\uc5c8\ub2e4. \uadf8\ub7ec\ub098 \uc774\ub7f0 \uc138\uac04\uc758 \uc778\uc2dd\uacfc\ub294 \ub2ec\ub9ac \uae40\uc885\uc778\uc740, 2005\ub144 \ubd80\ub3d9\uc0b0 \ud22c\uae30 \uc5f4\ud48d \ub54c \uc5f4\ub9b0\uc6b0\ub9ac\ub2f9\uc774 \uc7ac\ub3c4\uc785\ud558\ub824\ub358 \ud1a0\uc9c0\uacf5\uac1c\ub150\uc774 \uad81\uc5ec\uc9c0\ucc45\uc774\ub77c \ud3c4\ud558\ud558\uace0 \ub2e4\uc74c\uacfc \uac19\uc774 \ub9d0\ud588\ub2e4. \"\ud1a0\uc9c0\uacf5\uac1c\ub150\uc740 1989\ub144 \ub3c4\uc785 \ub17c\uc758 \ub2f9\uc2dc \uacbd\uc81c\uae30\ud68d\uc6d0\uc774 \ucc3d\uc791\ud55c \ub2e8\uc5b4\ub85c \uacbd\uc81c\ud559\uc5d0\ub3c4 \uc5c6\ub294 \uac1c\ub150\uc774\uba70, \uad6d\uc720\uc9c0\u00b7\uc0ac\uc720\uc9c0\ub294 \uc788\uc744 \uc218 \uc788\uc5b4\ub3c4 \ud1a0\uc9c0\uacf5\uac1c\ub150\uc740 \uc18c\uc124 \uc18d\uc5d0\uc11c\ub098 \ub098\uc62c \uc218 \uc788\ub294 \ub9d0\"\uc774\ub77c\uace0 \ubc1d\ud614\ub2e4. \uc774\uc5b4 \"\ub098\ub294 \ubcf4\uac74\uc0ac\ud68c\ubd80 \uc7a5\uad00\uc73c\ub85c \uc788\uc5c8\ub294\ub370 \ud1a0\uc9c0\uacf5\uac1c\ub150\uc740 \uc808\ub300\ub85c \uc548\ub41c\ub2e4\uace0 \ubc18\ub300\ud588\uc5c8\ub2e4\"\uba70 \"\ub3c4\uc785\uc744 \uc8fc\uc7a5\ud588\ub358 \uc0ac\ub78c\ub3c4 \uadfc\uac70\ub97c \uc81c\ub300\ub85c \uc124\uba85\uc740 \ubabb\ud55c \ucc44 \ud1a0\uc9c0\uacf5\uae09 \ud655\ub300\uac00 \uc5b4\ub835\ub2e4\ub294 \uba85\ubd84\ub9cc \ub0b4\uc138\uc6e0\ub2e4\"\uace0 \ub367\ubd99\uc600\ub2e4. \"\ud1a0\uc9c0\uacf5\uac1c\ub150\uc740 \uc138\uc81c\uc815\ucc45\uc73c\ub85c \uc77c\uc2dc\uc801 \ud6a8\uacfc\ub97c \uac70\ub458 \uc218\ub294 \uc788\uaca0\uc9c0\ub9cc \uacb0\uad6d\uc2dc\uc7a5\uc774 \uc801\uc751\ud558\ub294 \ubc29\ubc95\uc744 \ucc3e\uac8c \ub41c\ub2e4\"\uba70 \"\ud1a0\uc9c0\ucd08\uacfc\uc774\ub4dd\uc138 \ub4f1\uc740 \uc808\ub300\ub85c \ubd80\uacfc\ud560 \uc218 \uc5c6\ub294 \uc138\uae08\uc73c\ub85c \uc704\ud5cc \uc18c\uc9c0\uac00 \uc788\uc73c\uba70 '\uac1c\ubc1c\uc774\uc775\ud658\uc218\uc81c'\ub3c4 \uacb0\uad6d \ud1a0\uc9c0\uac12\uc73c\ub85c \uc804\uac00\ub418\uac8c \ub41c\ub2e4\"\uace0 \uac15\uc870\ud588\ub2e4. \uadf8\ub294 \ubd80\ub3d9\uc0b0 \ud22c\uae30\ub97c \uc5b5\uc81c\ud560 \uadfc\ubcf8 \ubc29\ucc45\uc740 \"\uc138\uc81c\ub85c \ud22c\uae30\uc5b5\uc81c\ub97c \ud558\uaca0\ub2e4\ub294 \ubc1c\uc0c1\uc740 \uc798\ubabb\ub410\uace0 \uae08\ub9ac\uc778\uc0c1 \ub4f1 \uacbd\uc81c\uc815\ucc45 \uc804\ubc18\uc744 \uc798 \uc870\uc728\ud574\uc11c \ud574\ubc95\uc744 \ucc3e\uc544\ub098\uac00\uc57c \ud55c\ub2e4\"\uace0 \uc8fc\uc7a5\ud588\ub2e4. \ub610\ud55c \uae08\uc735\uc2e4\uba85\uc81c\ub3c4 \uac15\ub825 \ubc18\ub300\ud55c \uac83\uc73c\ub85c \ubcf4\ub3c4\ub41c \ubc14 \uc788\ub2e4.", "sentence_answer": "\ub2f9\uc2dc \uccad\uc640\ub300 \uacbd\uc81c\uc218\uc11d\uc744 \uc9c0\ub0b8 \uae40\uc885\uc778 \uc740 \ud1a0\uc9c0\uacf5\uac1c\ub150\uc744 \ub3c4\uc785\ud55c \uc8fc\uccb4\ub85c \uc5ec\uaca8\uc84c\uc5c8\ub2e4.", "paragraph_id": "6488514-1-1", "paragraph_question": "question: \ud1a0\uc9c0\uacf5\uac1c\ub150\uc774 \uc18c\uc124\uc5d0\ub098 \ub098\uc624\ub294 \ub9d0\uc774\ub77c\uba70 \ubc18\ub300 \uc758\uacac\uc744 \ud53c\ub825\ud588\ub358 \uc778\ubb3c\uc740?, context: \ub178\ud0dc\uc6b0 \uc815\ubd80 \uc2dc\uc808 \uc9d1\uac12,\ub545\uac12 \uae09\ub4f1\uc744 \ub9c9\uae30 \uc704\ud574 \ud1a0\uc9c0\uacf5\uac1c\ub150\uc81c\uac00 \uc2dc\ub3c4\ub410\uc73c\ub098 \uc904\uc904\uc774 \uc704\ud5cc \uc18c\uc1a1 \ub4f1\uc73c\ub85c \uc88c\ucd08\ub410\ub2e4. \ub2f9\uc2dc \uccad\uc640\ub300 \uacbd\uc81c\uc218\uc11d\uc744 \uc9c0\ub0b8 \uae40\uc885\uc778\uc740 \ud1a0\uc9c0\uacf5\uac1c\ub150\uc744 \ub3c4\uc785\ud55c \uc8fc\uccb4\ub85c \uc5ec\uaca8\uc84c\uc5c8\ub2e4. \uadf8\ub7ec\ub098 \uc774\ub7f0 \uc138\uac04\uc758 \uc778\uc2dd\uacfc\ub294 \ub2ec\ub9ac \uae40\uc885\uc778\uc740, 2005\ub144 \ubd80\ub3d9\uc0b0 \ud22c\uae30 \uc5f4\ud48d \ub54c \uc5f4\ub9b0\uc6b0\ub9ac\ub2f9\uc774 \uc7ac\ub3c4\uc785\ud558\ub824\ub358 \ud1a0\uc9c0\uacf5\uac1c\ub150\uc774 \uad81\uc5ec\uc9c0\ucc45\uc774\ub77c \ud3c4\ud558\ud558\uace0 \ub2e4\uc74c\uacfc \uac19\uc774 \ub9d0\ud588\ub2e4. \"\ud1a0\uc9c0\uacf5\uac1c\ub150\uc740 1989\ub144 \ub3c4\uc785 \ub17c\uc758 \ub2f9\uc2dc \uacbd\uc81c\uae30\ud68d\uc6d0\uc774 \ucc3d\uc791\ud55c \ub2e8\uc5b4\ub85c \uacbd\uc81c\ud559\uc5d0\ub3c4 \uc5c6\ub294 \uac1c\ub150\uc774\uba70, \uad6d\uc720\uc9c0\u00b7\uc0ac\uc720\uc9c0\ub294 \uc788\uc744 \uc218 \uc788\uc5b4\ub3c4 \ud1a0\uc9c0\uacf5\uac1c\ub150\uc740 \uc18c\uc124 \uc18d\uc5d0\uc11c\ub098 \ub098\uc62c \uc218 \uc788\ub294 \ub9d0\"\uc774\ub77c\uace0 \ubc1d\ud614\ub2e4. \uc774\uc5b4 \"\ub098\ub294 \ubcf4\uac74\uc0ac\ud68c\ubd80 \uc7a5\uad00\uc73c\ub85c \uc788\uc5c8\ub294\ub370 \ud1a0\uc9c0\uacf5\uac1c\ub150\uc740 \uc808\ub300\ub85c \uc548\ub41c\ub2e4\uace0 \ubc18\ub300\ud588\uc5c8\ub2e4\"\uba70 \"\ub3c4\uc785\uc744 \uc8fc\uc7a5\ud588\ub358 \uc0ac\ub78c\ub3c4 \uadfc\uac70\ub97c \uc81c\ub300\ub85c \uc124\uba85\uc740 \ubabb\ud55c \ucc44 \ud1a0\uc9c0\uacf5\uae09 \ud655\ub300\uac00 \uc5b4\ub835\ub2e4\ub294 \uba85\ubd84\ub9cc \ub0b4\uc138\uc6e0\ub2e4\"\uace0 \ub367\ubd99\uc600\ub2e4. \"\ud1a0\uc9c0\uacf5\uac1c\ub150\uc740 \uc138\uc81c\uc815\ucc45\uc73c\ub85c \uc77c\uc2dc\uc801 \ud6a8\uacfc\ub97c \uac70\ub458 \uc218\ub294 \uc788\uaca0\uc9c0\ub9cc \uacb0\uad6d\uc2dc\uc7a5\uc774 \uc801\uc751\ud558\ub294 \ubc29\ubc95\uc744 \ucc3e\uac8c \ub41c\ub2e4\"\uba70 \"\ud1a0\uc9c0\ucd08\uacfc\uc774\ub4dd\uc138 \ub4f1\uc740 \uc808\ub300\ub85c \ubd80\uacfc\ud560 \uc218 \uc5c6\ub294 \uc138\uae08\uc73c\ub85c \uc704\ud5cc \uc18c\uc9c0\uac00 \uc788\uc73c\uba70 '\uac1c\ubc1c\uc774\uc775\ud658\uc218\uc81c'\ub3c4 \uacb0\uad6d \ud1a0\uc9c0\uac12\uc73c\ub85c \uc804\uac00\ub418\uac8c \ub41c\ub2e4\"\uace0 \uac15\uc870\ud588\ub2e4. \uadf8\ub294 \ubd80\ub3d9\uc0b0 \ud22c\uae30\ub97c \uc5b5\uc81c\ud560 \uadfc\ubcf8 \ubc29\ucc45\uc740 \"\uc138\uc81c\ub85c \ud22c\uae30\uc5b5\uc81c\ub97c \ud558\uaca0\ub2e4\ub294 \ubc1c\uc0c1\uc740 \uc798\ubabb\ub410\uace0 \uae08\ub9ac\uc778\uc0c1 \ub4f1 \uacbd\uc81c\uc815\ucc45 \uc804\ubc18\uc744 \uc798 \uc870\uc728\ud574\uc11c \ud574\ubc95\uc744 \ucc3e\uc544\ub098\uac00\uc57c \ud55c\ub2e4\"\uace0 \uc8fc\uc7a5\ud588\ub2e4. \ub610\ud55c \uae08\uc735\uc2e4\uba85\uc81c\ub3c4 \uac15\ub825 \ubc18\ub300\ud55c \uac83\uc73c\ub85c \ubcf4\ub3c4\ub41c \ubc14 \uc788\ub2e4."} {"question": "\uad6d\ub0b4 \uc8fc\uc694 \ub85c\ud38c\uc5d0\uc11c \uc778\ud130\uc2ed\uc744 \uac70\uce5c \ub85c\uc2a4\ucfe8\uc0dd\uc758 \uc218\ub294?", "paragraph": "\ubc29\ud559\uae30\uac04\uc744 \uc774\uc6a9 \uae40\uc564\uc7a5, \uad11\uc7a5, \ud0dc\ud3c9\uc591, \uc9c0\ud3c9, \uc728\ucd0c, \uc138\uc885, KCL, \ubc14\ub978, \ub300\uc885 \ub4f1 \ud55c\uad6d\uc758 \uc8fc\uc694 \ub85c\ud38c\uc5d0\uc11c \uad6d\uc81c\uc911\uc7ac\uacfc \uac19\uc740 \uad50\uc721 \ubc0f \uc2e4\uc2b5\uc744 \ud55c\ub2e4. \uc778\ud134\uc2ed \uae30\uad00\uc73c\ub85c\ub294 \uc11c\uc6b8\ud2b9\ubcc4\uc2dc, \ubc95\uc81c\uc5f0\uad6c\uc6d0, \ub300\ud55c\uc0c1\uc0ac\uc911\uc7ac\uc6d0, \ubc95\uc81c\ucc98, \ucc38\uc5ec\uc5f0\ub300, \ubc95\uc6d0 \ub4f1\uc774 \uc788\ub2e4. \ubbfc\uc8fc\uc0ac\ud68c\ub97c \uc704\ud55c \ubcc0\ud638\uc0ac\ubaa8\uc784 \ub4f1\ub3c4 \uc788\ub2e4. \uacf5\uacf5\uae30\uad00\uc73c\ub85c\ub294 \uad70\ubc95\ubb34\uad00 \uc218\uc2b5\uc774 \uc788\uc73c\uba70 \uc774\ub54c \uad70\ubc95\ubb34\uad00 \uc870\uc9c1 \ubc0f \uc5c5\ubb34 \ub4f1\uc744 \ubc30\uc6b4\ub2e4. \uae30\uac04\uc774 \ub108\ubb34 \uc9e7\uace0 \uae4a\uc774\uac00 \uc5c6\uc5b4 \uac00\uce58\uac00 \uc5c6\ub2e4\ub294 \uc778\ud134\uc2ed \ubb34\uc6a9\ub860\ub3c4 \uc788\ub2e4.2011\ub144 \ud55c\ub098\ub77c\ub2f9 \uc8fc\uad11\ub355 \uc758\uc6d0\uc774 \uad50\uc721\uacfc\ud559\uae30\uc220\ubd80\ub85c\ubd80\ud130 \uc81c\ucd9c\ubc1b\uc740 \ub85c\uc2a4\ucfe8 \ub85c\ud38c \uc2e4\ubb34 \uc218\uc2b5 \ud604\ud669\uc5d0 \ub530\ub974\uba74 \uae40\uc564\uc7a5, \ud0dc\ud3c9\uc591, \uad11\uc7a5, \uc138\uc885 \ub4f1 \uad6d\ub0b4 \uc8fc\uc694 \ub85c\ud38c\uc758 \uc2e4\ubb34 \uc218\uc2b5 \uae30\ud68c \ub300\ubd80\ubd84\uc774 \uc11c\uc6b8 \uc18c\uc7ac \ub300\ud559\uc778 \ub370\ub2e4 \uc11c\uc6b8\ub300\uc640 \uace0\ub824\ub300, \uc5f0\uc138\ub300 \ub4f1 \uc774\ub978\ubc14 'SKY \ub300\ud559'\uc774 \ub300\ubd80\ubd84\uc744 \ucc28\uc9c0\ud558\ub294 \ud3b8\uc911\ud604\uc0c1\uc774 \ub098\ud0c0\ub0ac\ub2e4- \uc9c0\ub09c 2009\ub144\ubd80\ud130 \ud604\uc7ac\uae4c\uc9c0 8\ub300 \uad6d\ub0b4 \uc8fc\uc694 \ub85c\ud38c(\ubcc0\ud638\uc0ac\uc218 100\uba85\uc774\uc0c1, \ub300\ud55c\ubcc0\ud611 2011\ub1443\uc6d4 \uc790\ub8cc)\uc5d0\uc11c \uc2e4\ubb34 \uc218\uc2b5\uc744 \uac70\uce5c \ub85c\uc2a4\ucfe8\uc0dd 1543\uba85\uc774 \ub85c\ud38c\uc5d0\uc11c \uc2e4\ubb34 \uc218\uc2b5\uc744 \ud55c \uac00\uc6b4\ub370 \uc774 \uac00\uc6b4\ub370 \uc11c\uc6b8\ub300 525\uba85, \uace0\ub824\ub300 257\uba85, \uc5f0\uc138\ub300 204\uba85\uc774\ub098 \ucda9\ub0a8\ub300\ub294 24\uba85, \ucda9\ubd81\ub300\ub294 4\uba85 \ub9cc\uc774 \ub300\ud615 \ub85c\ud38c\uc758 \uc2e4\ubb34 \uc218\uc2b5 \uae30\ud68c\ub97c \uac00\uc84c\ub2e4. \uc81c\uc8fc\ub300\ub294 \ub2e8 \ud55c\uba85\ub3c4 \ub300\ud615 \ub85c\ud38c\uc5d0\uc11c \uc2e4\ubb34 \uc218\uc2b5 \uae30\ud68c\ub97c \uac16\uc9c0 \ubabb\ud588\uace0, \uac15\uc6d0\ub300 5\uba85, \uc6d0\uad11\ub300\ub294 \ub2e8 3\uba85 \ub9cc\uc774 \ub300\ud615 \ub85c\ud38c\uc5d0\uc11c \uc2e4\ubb34 \uc218\uc2b5\uc744 \ud55c \uac83\uc73c\ub85c \uc9d1\uacc4\ub410\ub2e4.", "answer": "1543\uba85", "sentence": "\uae30\uac04\uc774 \ub108\ubb34 \uc9e7\uace0 \uae4a\uc774\uac00 \uc5c6\uc5b4 \uac00\uce58\uac00 \uc5c6\ub2e4\ub294 \uc778\ud134\uc2ed \ubb34\uc6a9\ub860\ub3c4 \uc788\ub2e4.2011\ub144 \ud55c\ub098\ub77c\ub2f9 \uc8fc\uad11\ub355 \uc758\uc6d0\uc774 \uad50\uc721\uacfc\ud559\uae30\uc220\ubd80\ub85c\ubd80\ud130 \uc81c\ucd9c\ubc1b\uc740 \ub85c\uc2a4\ucfe8 \ub85c\ud38c \uc2e4\ubb34 \uc218\uc2b5 \ud604\ud669\uc5d0 \ub530\ub974\uba74 \uae40\uc564\uc7a5, \ud0dc\ud3c9\uc591, \uad11\uc7a5, \uc138\uc885 \ub4f1 \uad6d\ub0b4 \uc8fc\uc694 \ub85c\ud38c\uc758 \uc2e4\ubb34 \uc218\uc2b5 \uae30\ud68c \ub300\ubd80\ubd84\uc774 \uc11c\uc6b8 \uc18c\uc7ac \ub300\ud559\uc778 \ub370\ub2e4 \uc11c\uc6b8\ub300\uc640 \uace0\ub824\ub300, \uc5f0\uc138\ub300 \ub4f1 \uc774\ub978\ubc14 'SKY \ub300\ud559'\uc774 \ub300\ubd80\ubd84\uc744 \ucc28\uc9c0\ud558\ub294 \ud3b8\uc911\ud604\uc0c1\uc774 \ub098\ud0c0\ub0ac\ub2e4- \uc9c0\ub09c 2009\ub144\ubd80\ud130 \ud604\uc7ac\uae4c\uc9c0 8\ub300 \uad6d\ub0b4 \uc8fc\uc694 \ub85c\ud38c(\ubcc0\ud638\uc0ac\uc218 100\uba85\uc774\uc0c1, \ub300\ud55c\ubcc0\ud611 2011\ub1443\uc6d4 \uc790\ub8cc)\uc5d0\uc11c \uc2e4\ubb34 \uc218\uc2b5\uc744 \uac70\uce5c \ub85c\uc2a4\ucfe8\uc0dd 1543\uba85 \uc774 \ub85c\ud38c\uc5d0\uc11c \uc2e4\ubb34 \uc218\uc2b5\uc744 \ud55c \uac00\uc6b4\ub370 \uc774 \uac00\uc6b4\ub370 \uc11c\uc6b8\ub300 525\uba85, \uace0\ub824\ub300 257\uba85, \uc5f0\uc138\ub300 204\uba85\uc774\ub098 \ucda9\ub0a8\ub300\ub294 24\uba85, \ucda9\ubd81\ub300\ub294 4\uba85 \ub9cc\uc774 \ub300\ud615 \ub85c\ud38c\uc758 \uc2e4\ubb34 \uc218\uc2b5 \uae30\ud68c\ub97c \uac00\uc84c\ub2e4.", "paragraph_sentence": "\ubc29\ud559\uae30\uac04\uc744 \uc774\uc6a9 \uae40\uc564\uc7a5, \uad11\uc7a5, \ud0dc\ud3c9\uc591, \uc9c0\ud3c9, \uc728\ucd0c, \uc138\uc885, KCL, \ubc14\ub978, \ub300\uc885 \ub4f1 \ud55c\uad6d\uc758 \uc8fc\uc694 \ub85c\ud38c\uc5d0\uc11c \uad6d\uc81c\uc911\uc7ac\uacfc \uac19\uc740 \uad50\uc721 \ubc0f \uc2e4\uc2b5\uc744 \ud55c\ub2e4. \uc778\ud134\uc2ed \uae30\uad00\uc73c\ub85c\ub294 \uc11c\uc6b8\ud2b9\ubcc4\uc2dc, \ubc95\uc81c\uc5f0\uad6c\uc6d0, \ub300\ud55c\uc0c1\uc0ac\uc911\uc7ac\uc6d0, \ubc95\uc81c\ucc98, \ucc38\uc5ec\uc5f0\ub300, \ubc95\uc6d0 \ub4f1\uc774 \uc788\ub2e4. \ubbfc\uc8fc\uc0ac\ud68c\ub97c \uc704\ud55c \ubcc0\ud638\uc0ac\ubaa8\uc784 \ub4f1\ub3c4 \uc788\ub2e4. \uacf5\uacf5\uae30\uad00\uc73c\ub85c\ub294 \uad70\ubc95\ubb34\uad00 \uc218\uc2b5\uc774 \uc788\uc73c\uba70 \uc774\ub54c \uad70\ubc95\ubb34\uad00 \uc870\uc9c1 \ubc0f \uc5c5\ubb34 \ub4f1\uc744 \ubc30\uc6b4\ub2e4. \uae30\uac04\uc774 \ub108\ubb34 \uc9e7\uace0 \uae4a\uc774\uac00 \uc5c6\uc5b4 \uac00\uce58\uac00 \uc5c6\ub2e4\ub294 \uc778\ud134\uc2ed \ubb34\uc6a9\ub860\ub3c4 \uc788\ub2e4.2011\ub144 \ud55c\ub098\ub77c\ub2f9 \uc8fc\uad11\ub355 \uc758\uc6d0\uc774 \uad50\uc721\uacfc\ud559\uae30\uc220\ubd80\ub85c\ubd80\ud130 \uc81c\ucd9c\ubc1b\uc740 \ub85c\uc2a4\ucfe8 \ub85c\ud38c \uc2e4\ubb34 \uc218\uc2b5 \ud604\ud669\uc5d0 \ub530\ub974\uba74 \uae40\uc564\uc7a5, \ud0dc\ud3c9\uc591, \uad11\uc7a5, \uc138\uc885 \ub4f1 \uad6d\ub0b4 \uc8fc\uc694 \ub85c\ud38c\uc758 \uc2e4\ubb34 \uc218\uc2b5 \uae30\ud68c \ub300\ubd80\ubd84\uc774 \uc11c\uc6b8 \uc18c\uc7ac \ub300\ud559\uc778 \ub370\ub2e4 \uc11c\uc6b8\ub300\uc640 \uace0\ub824\ub300, \uc5f0\uc138\ub300 \ub4f1 \uc774\ub978\ubc14 'SKY \ub300\ud559'\uc774 \ub300\ubd80\ubd84\uc744 \ucc28\uc9c0\ud558\ub294 \ud3b8\uc911\ud604\uc0c1\uc774 \ub098\ud0c0\ub0ac\ub2e4- \uc9c0\ub09c 2009\ub144\ubd80\ud130 \ud604\uc7ac\uae4c\uc9c0 8\ub300 \uad6d\ub0b4 \uc8fc\uc694 \ub85c\ud38c(\ubcc0\ud638\uc0ac\uc218 100\uba85\uc774\uc0c1, \ub300\ud55c\ubcc0\ud611 2011\ub1443\uc6d4 \uc790\ub8cc)\uc5d0\uc11c \uc2e4\ubb34 \uc218\uc2b5\uc744 \uac70\uce5c \ub85c\uc2a4\ucfe8\uc0dd 1543\uba85 \uc774 \ub85c\ud38c\uc5d0\uc11c \uc2e4\ubb34 \uc218\uc2b5\uc744 \ud55c \uac00\uc6b4\ub370 \uc774 \uac00\uc6b4\ub370 \uc11c\uc6b8\ub300 525\uba85, \uace0\ub824\ub300 257\uba85, \uc5f0\uc138\ub300 204\uba85\uc774\ub098 \ucda9\ub0a8\ub300\ub294 24\uba85, \ucda9\ubd81\ub300\ub294 4\uba85 \ub9cc\uc774 \ub300\ud615 \ub85c\ud38c\uc758 \uc2e4\ubb34 \uc218\uc2b5 \uae30\ud68c\ub97c \uac00\uc84c\ub2e4. \uc81c\uc8fc\ub300\ub294 \ub2e8 \ud55c\uba85\ub3c4 \ub300\ud615 \ub85c\ud38c\uc5d0\uc11c \uc2e4\ubb34 \uc218\uc2b5 \uae30\ud68c\ub97c \uac16\uc9c0 \ubabb\ud588\uace0, \uac15\uc6d0\ub300 5\uba85, \uc6d0\uad11\ub300\ub294 \ub2e8 3\uba85 \ub9cc\uc774 \ub300\ud615 \ub85c\ud38c\uc5d0\uc11c \uc2e4\ubb34 \uc218\uc2b5\uc744 \ud55c \uac83\uc73c\ub85c \uc9d1\uacc4\ub410\ub2e4.", "paragraph_answer": "\ubc29\ud559\uae30\uac04\uc744 \uc774\uc6a9 \uae40\uc564\uc7a5, \uad11\uc7a5, \ud0dc\ud3c9\uc591, \uc9c0\ud3c9, \uc728\ucd0c, \uc138\uc885, KCL, \ubc14\ub978, \ub300\uc885 \ub4f1 \ud55c\uad6d\uc758 \uc8fc\uc694 \ub85c\ud38c\uc5d0\uc11c \uad6d\uc81c\uc911\uc7ac\uacfc \uac19\uc740 \uad50\uc721 \ubc0f \uc2e4\uc2b5\uc744 \ud55c\ub2e4. \uc778\ud134\uc2ed \uae30\uad00\uc73c\ub85c\ub294 \uc11c\uc6b8\ud2b9\ubcc4\uc2dc, \ubc95\uc81c\uc5f0\uad6c\uc6d0, \ub300\ud55c\uc0c1\uc0ac\uc911\uc7ac\uc6d0, \ubc95\uc81c\ucc98, \ucc38\uc5ec\uc5f0\ub300, \ubc95\uc6d0 \ub4f1\uc774 \uc788\ub2e4. \ubbfc\uc8fc\uc0ac\ud68c\ub97c \uc704\ud55c \ubcc0\ud638\uc0ac\ubaa8\uc784 \ub4f1\ub3c4 \uc788\ub2e4. \uacf5\uacf5\uae30\uad00\uc73c\ub85c\ub294 \uad70\ubc95\ubb34\uad00 \uc218\uc2b5\uc774 \uc788\uc73c\uba70 \uc774\ub54c \uad70\ubc95\ubb34\uad00 \uc870\uc9c1 \ubc0f \uc5c5\ubb34 \ub4f1\uc744 \ubc30\uc6b4\ub2e4. \uae30\uac04\uc774 \ub108\ubb34 \uc9e7\uace0 \uae4a\uc774\uac00 \uc5c6\uc5b4 \uac00\uce58\uac00 \uc5c6\ub2e4\ub294 \uc778\ud134\uc2ed \ubb34\uc6a9\ub860\ub3c4 \uc788\ub2e4.2011\ub144 \ud55c\ub098\ub77c\ub2f9 \uc8fc\uad11\ub355 \uc758\uc6d0\uc774 \uad50\uc721\uacfc\ud559\uae30\uc220\ubd80\ub85c\ubd80\ud130 \uc81c\ucd9c\ubc1b\uc740 \ub85c\uc2a4\ucfe8 \ub85c\ud38c \uc2e4\ubb34 \uc218\uc2b5 \ud604\ud669\uc5d0 \ub530\ub974\uba74 \uae40\uc564\uc7a5, \ud0dc\ud3c9\uc591, \uad11\uc7a5, \uc138\uc885 \ub4f1 \uad6d\ub0b4 \uc8fc\uc694 \ub85c\ud38c\uc758 \uc2e4\ubb34 \uc218\uc2b5 \uae30\ud68c \ub300\ubd80\ubd84\uc774 \uc11c\uc6b8 \uc18c\uc7ac \ub300\ud559\uc778 \ub370\ub2e4 \uc11c\uc6b8\ub300\uc640 \uace0\ub824\ub300, \uc5f0\uc138\ub300 \ub4f1 \uc774\ub978\ubc14 'SKY \ub300\ud559'\uc774 \ub300\ubd80\ubd84\uc744 \ucc28\uc9c0\ud558\ub294 \ud3b8\uc911\ud604\uc0c1\uc774 \ub098\ud0c0\ub0ac\ub2e4- \uc9c0\ub09c 2009\ub144\ubd80\ud130 \ud604\uc7ac\uae4c\uc9c0 8\ub300 \uad6d\ub0b4 \uc8fc\uc694 \ub85c\ud38c(\ubcc0\ud638\uc0ac\uc218 100\uba85\uc774\uc0c1, \ub300\ud55c\ubcc0\ud611 2011\ub1443\uc6d4 \uc790\ub8cc)\uc5d0\uc11c \uc2e4\ubb34 \uc218\uc2b5\uc744 \uac70\uce5c \ub85c\uc2a4\ucfe8\uc0dd 1543\uba85 \uc774 \ub85c\ud38c\uc5d0\uc11c \uc2e4\ubb34 \uc218\uc2b5\uc744 \ud55c \uac00\uc6b4\ub370 \uc774 \uac00\uc6b4\ub370 \uc11c\uc6b8\ub300 525\uba85, \uace0\ub824\ub300 257\uba85, \uc5f0\uc138\ub300 204\uba85\uc774\ub098 \ucda9\ub0a8\ub300\ub294 24\uba85, \ucda9\ubd81\ub300\ub294 4\uba85 \ub9cc\uc774 \ub300\ud615 \ub85c\ud38c\uc758 \uc2e4\ubb34 \uc218\uc2b5 \uae30\ud68c\ub97c \uac00\uc84c\ub2e4. \uc81c\uc8fc\ub300\ub294 \ub2e8 \ud55c\uba85\ub3c4 \ub300\ud615 \ub85c\ud38c\uc5d0\uc11c \uc2e4\ubb34 \uc218\uc2b5 \uae30\ud68c\ub97c \uac16\uc9c0 \ubabb\ud588\uace0, \uac15\uc6d0\ub300 5\uba85, \uc6d0\uad11\ub300\ub294 \ub2e8 3\uba85 \ub9cc\uc774 \ub300\ud615 \ub85c\ud38c\uc5d0\uc11c \uc2e4\ubb34 \uc218\uc2b5\uc744 \ud55c \uac83\uc73c\ub85c \uc9d1\uacc4\ub410\ub2e4.", "sentence_answer": "\uae30\uac04\uc774 \ub108\ubb34 \uc9e7\uace0 \uae4a\uc774\uac00 \uc5c6\uc5b4 \uac00\uce58\uac00 \uc5c6\ub2e4\ub294 \uc778\ud134\uc2ed \ubb34\uc6a9\ub860\ub3c4 \uc788\ub2e4.2011\ub144 \ud55c\ub098\ub77c\ub2f9 \uc8fc\uad11\ub355 \uc758\uc6d0\uc774 \uad50\uc721\uacfc\ud559\uae30\uc220\ubd80\ub85c\ubd80\ud130 \uc81c\ucd9c\ubc1b\uc740 \ub85c\uc2a4\ucfe8 \ub85c\ud38c \uc2e4\ubb34 \uc218\uc2b5 \ud604\ud669\uc5d0 \ub530\ub974\uba74 \uae40\uc564\uc7a5, \ud0dc\ud3c9\uc591, \uad11\uc7a5, \uc138\uc885 \ub4f1 \uad6d\ub0b4 \uc8fc\uc694 \ub85c\ud38c\uc758 \uc2e4\ubb34 \uc218\uc2b5 \uae30\ud68c \ub300\ubd80\ubd84\uc774 \uc11c\uc6b8 \uc18c\uc7ac \ub300\ud559\uc778 \ub370\ub2e4 \uc11c\uc6b8\ub300\uc640 \uace0\ub824\ub300, \uc5f0\uc138\ub300 \ub4f1 \uc774\ub978\ubc14 'SKY \ub300\ud559'\uc774 \ub300\ubd80\ubd84\uc744 \ucc28\uc9c0\ud558\ub294 \ud3b8\uc911\ud604\uc0c1\uc774 \ub098\ud0c0\ub0ac\ub2e4- \uc9c0\ub09c 2009\ub144\ubd80\ud130 \ud604\uc7ac\uae4c\uc9c0 8\ub300 \uad6d\ub0b4 \uc8fc\uc694 \ub85c\ud38c(\ubcc0\ud638\uc0ac\uc218 100\uba85\uc774\uc0c1, \ub300\ud55c\ubcc0\ud611 2011\ub1443\uc6d4 \uc790\ub8cc)\uc5d0\uc11c \uc2e4\ubb34 \uc218\uc2b5\uc744 \uac70\uce5c \ub85c\uc2a4\ucfe8\uc0dd 1543\uba85 \uc774 \ub85c\ud38c\uc5d0\uc11c \uc2e4\ubb34 \uc218\uc2b5\uc744 \ud55c \uac00\uc6b4\ub370 \uc774 \uac00\uc6b4\ub370 \uc11c\uc6b8\ub300 525\uba85, \uace0\ub824\ub300 257\uba85, \uc5f0\uc138\ub300 204\uba85\uc774\ub098 \ucda9\ub0a8\ub300\ub294 24\uba85, \ucda9\ubd81\ub300\ub294 4\uba85 \ub9cc\uc774 \ub300\ud615 \ub85c\ud38c\uc758 \uc2e4\ubb34 \uc218\uc2b5 \uae30\ud68c\ub97c \uac00\uc84c\ub2e4.", "paragraph_id": "6512325-7-1", "paragraph_question": "question: \uad6d\ub0b4 \uc8fc\uc694 \ub85c\ud38c\uc5d0\uc11c \uc778\ud130\uc2ed\uc744 \uac70\uce5c \ub85c\uc2a4\ucfe8\uc0dd\uc758 \uc218\ub294?, context: \ubc29\ud559\uae30\uac04\uc744 \uc774\uc6a9 \uae40\uc564\uc7a5, \uad11\uc7a5, \ud0dc\ud3c9\uc591, \uc9c0\ud3c9, \uc728\ucd0c, \uc138\uc885, KCL, \ubc14\ub978, \ub300\uc885 \ub4f1 \ud55c\uad6d\uc758 \uc8fc\uc694 \ub85c\ud38c\uc5d0\uc11c \uad6d\uc81c\uc911\uc7ac\uacfc \uac19\uc740 \uad50\uc721 \ubc0f \uc2e4\uc2b5\uc744 \ud55c\ub2e4. \uc778\ud134\uc2ed \uae30\uad00\uc73c\ub85c\ub294 \uc11c\uc6b8\ud2b9\ubcc4\uc2dc, \ubc95\uc81c\uc5f0\uad6c\uc6d0, \ub300\ud55c\uc0c1\uc0ac\uc911\uc7ac\uc6d0, \ubc95\uc81c\ucc98, \ucc38\uc5ec\uc5f0\ub300, \ubc95\uc6d0 \ub4f1\uc774 \uc788\ub2e4. \ubbfc\uc8fc\uc0ac\ud68c\ub97c \uc704\ud55c \ubcc0\ud638\uc0ac\ubaa8\uc784 \ub4f1\ub3c4 \uc788\ub2e4. \uacf5\uacf5\uae30\uad00\uc73c\ub85c\ub294 \uad70\ubc95\ubb34\uad00 \uc218\uc2b5\uc774 \uc788\uc73c\uba70 \uc774\ub54c \uad70\ubc95\ubb34\uad00 \uc870\uc9c1 \ubc0f \uc5c5\ubb34 \ub4f1\uc744 \ubc30\uc6b4\ub2e4. \uae30\uac04\uc774 \ub108\ubb34 \uc9e7\uace0 \uae4a\uc774\uac00 \uc5c6\uc5b4 \uac00\uce58\uac00 \uc5c6\ub2e4\ub294 \uc778\ud134\uc2ed \ubb34\uc6a9\ub860\ub3c4 \uc788\ub2e4.2011\ub144 \ud55c\ub098\ub77c\ub2f9 \uc8fc\uad11\ub355 \uc758\uc6d0\uc774 \uad50\uc721\uacfc\ud559\uae30\uc220\ubd80\ub85c\ubd80\ud130 \uc81c\ucd9c\ubc1b\uc740 \ub85c\uc2a4\ucfe8 \ub85c\ud38c \uc2e4\ubb34 \uc218\uc2b5 \ud604\ud669\uc5d0 \ub530\ub974\uba74 \uae40\uc564\uc7a5, \ud0dc\ud3c9\uc591, \uad11\uc7a5, \uc138\uc885 \ub4f1 \uad6d\ub0b4 \uc8fc\uc694 \ub85c\ud38c\uc758 \uc2e4\ubb34 \uc218\uc2b5 \uae30\ud68c \ub300\ubd80\ubd84\uc774 \uc11c\uc6b8 \uc18c\uc7ac \ub300\ud559\uc778 \ub370\ub2e4 \uc11c\uc6b8\ub300\uc640 \uace0\ub824\ub300, \uc5f0\uc138\ub300 \ub4f1 \uc774\ub978\ubc14 'SKY \ub300\ud559'\uc774 \ub300\ubd80\ubd84\uc744 \ucc28\uc9c0\ud558\ub294 \ud3b8\uc911\ud604\uc0c1\uc774 \ub098\ud0c0\ub0ac\ub2e4- \uc9c0\ub09c 2009\ub144\ubd80\ud130 \ud604\uc7ac\uae4c\uc9c0 8\ub300 \uad6d\ub0b4 \uc8fc\uc694 \ub85c\ud38c(\ubcc0\ud638\uc0ac\uc218 100\uba85\uc774\uc0c1, \ub300\ud55c\ubcc0\ud611 2011\ub1443\uc6d4 \uc790\ub8cc)\uc5d0\uc11c \uc2e4\ubb34 \uc218\uc2b5\uc744 \uac70\uce5c \ub85c\uc2a4\ucfe8\uc0dd 1543\uba85\uc774 \ub85c\ud38c\uc5d0\uc11c \uc2e4\ubb34 \uc218\uc2b5\uc744 \ud55c \uac00\uc6b4\ub370 \uc774 \uac00\uc6b4\ub370 \uc11c\uc6b8\ub300 525\uba85, \uace0\ub824\ub300 257\uba85, \uc5f0\uc138\ub300 204\uba85\uc774\ub098 \ucda9\ub0a8\ub300\ub294 24\uba85, \ucda9\ubd81\ub300\ub294 4\uba85 \ub9cc\uc774 \ub300\ud615 \ub85c\ud38c\uc758 \uc2e4\ubb34 \uc218\uc2b5 \uae30\ud68c\ub97c \uac00\uc84c\ub2e4. \uc81c\uc8fc\ub300\ub294 \ub2e8 \ud55c\uba85\ub3c4 \ub300\ud615 \ub85c\ud38c\uc5d0\uc11c \uc2e4\ubb34 \uc218\uc2b5 \uae30\ud68c\ub97c \uac16\uc9c0 \ubabb\ud588\uace0, \uac15\uc6d0\ub300 5\uba85, \uc6d0\uad11\ub300\ub294 \ub2e8 3\uba85 \ub9cc\uc774 \ub300\ud615 \ub85c\ud38c\uc5d0\uc11c \uc2e4\ubb34 \uc218\uc2b5\uc744 \ud55c \uac83\uc73c\ub85c \uc9d1\uacc4\ub410\ub2e4."} {"question": "\uc778\ud130\ub137\uacfc \uc5f0\uc608\ub9e4\uccb4\ub4e4\uc774 \uc774\ud63c\uacfc\uc815\uc744 \uc2e4\uc2dc\uac04 \uc911\uacc4\ud588\ub2e4\uace0 \ub9d0\ud55c \uc5f0\ucd9c\uac00\ub294?", "paragraph": "\ucd5c\uc9c4\uc2e4\uc758 \uc5b4\uba38\ub2c8 \uc815\uc625\uc219\uc740 \uc790\uc2e0\uc758 \uc800\uc11c \u300a\uc5c4\ub9c8\uac00, \ubbf8\uc548\ud574 \uadf8\ub9ac\uace0 \uc0ac\ub791\ud574\u300b(2011\ub144)\uc5d0\uc11c \"\uc5b8\ub860\uc740 \uc784\uc2e0\ud55c \uc0ac\ub78c\uc744, \uc544\uc774 \uc5b4\uba38\ub2c8\ub97c \ubc30\ub824\ud558\uc9c0 \uc54a\uc558\ub2e4\"\ub77c\uace0 \uc9c0\uc801\ud558\uba70 \uae30\uc790\ub4e4\uc758 \uc724\ub9ac \uc758\uc2dd\uc5d0 \uc758\uad6c\uc2ec\uc744 \ud45c\ud604\ud588\ub2e4. \ucd5c\uc9c4\uc2e4\uc740 \uc774\ud63c \uacfc\uc815\uc5d0 \uc788\uc744 \ub54c \uc784\uc2e0\ud55c \uc0c1\ud0dc\uc600\ub294\ub370, \uae30\uc790\ub4e4\uc774 \ucd5c\uc9c4\uc2e4\uc758 \uc9d1 \uc55e\uc5d0 \uc9c4\uc744 \uce58\uace0 \uc788\uc5b4\uc11c \ucd5c\uc9c4\uc2e4\uc740 \uc815\uae30 \uac80\uc9c4\uc744 \ubc1b\ub294 \ub0a0 \uc678\uc5d0\ub294 \uac70\uc758 \uc9d1\uc5d0 \uac07\ud600 \uc788\uc5b4\uc57c \ud588\uace0, \uc774\ud63c \uacfc\uc815\uc744 \uc0dd\uc911\uacc4\ud558\ub294 \ub4ef\ud55c \uc120\uc815\uc801\uc774\uace0 \ubb34\ubd84\ubcc4\ud55c \uae30\uc0ac\uc5d0 \uace0\ud1b5\uc744 \uacaa\uae30\ub3c4 \ud558\uc600\ub2e4. \ucd5c\uc9c4\uc2e4\uc740 2\ub144\uc5ec \ub3d9\uc548\uc758 \uc774\ud63c \uacfc\uc815\uc5d0\uc11c \ud5c8\uc704\uc0ac\uc2e4\uc774\ub098 \uc0ac\uc2e4 \uc65c\uace1\uc744 \ubc14\ub85c\uc7a1\uae30 \uc704\ud574 \ubc18\ubc15\uc131\u00b7\ud574\uba85\uc131 \uc778\ud130\ubdf0\ub97c \ud558\uac70\ub098 \ubc95\uc801 \uc870\uce58\ub97c \ucde8\ud558\uae30\ub3c4 \ud558\uc600\ub294\ub370, \uc774\ub294 \uc0ac\uc0dd\ud65c\uc774 \ub354 \uc774\uc0c1 \uc138\uc0c1\uc5d0 \uc54c\ub824\uc9c0\uace0 \uc65c\uace1\ub418\ub294 \uac83\uc744 \ub9c9\uaca0\ub2e4\ub294 \ucde8\uc9c0\uc600\ub2e4. \uadf8 \ubc95\uc801 \uc870\uce58\uc758 \ub300\uc0c1\uc5d0\ub294 \uc5b8\ub860\ub3c4 \ud3ec\ud568\ub418\uc5b4 \uc788\uc5c8\ub2e4. \uc778\ud130\ubdf0\ub97c \uc790\uccad\ud558\uc5ec \ud5c8\uc704\uc0ac\uc2e4\uc744 \uc720\ud3ec\ud55c \uc790\uac00 \uc788\uc5c8\uace0 \uadf8 \uc778\ud130\ubdf0\ub97c \uc9c4\ud589\ud55c \uae30\uc790\ub294 \uadf8 \uc720\ud3ec \ub0b4\uc6a9\uc744 \uadf8\ub300\ub85c \ubc1b\uc544\uc801\uc5b4 \uae30\uc0ac\ud654\ud558\uc600\ub294\ub370, \uc774\ub294 \uae30\uc790\uac00 \ud5c8\uc704\uc0ac\uc2e4\uc720\ud3ec \ud589\uc704\uc5d0 \ub3d9\ucc38\ud55c \uac83\uc774\uba74\uc11c \ud5c8\uc704\uc0ac\uc2e4 \uc720\ud3ec\uc790\uc640 \uae30\uc790 \ubaa8\ub450 \uc774\ud63c\uc740 \ud63c\uc778 \ub2f9\uc0ac\uc790\uc640 \uad6d\uac00\uac00 \ud574\uacb0\ubd10\uc57c \ud558\ub294 \uc0ac\uc548\uc774\ub77c\ub294 \uc0ac\uc2e4\uc744 \ub9dd\uac01\ud55c \ud589\uc704\uc774\uae30\ub3c4 \ud558\uc600\ub2e4. \uc774\ub7f0 \ud5c8\uc704\uc0ac\uc2e4\uc720\ud3ec \ud589\uc704\uc5d0 \ubc29\uc1a1\uc0ac\ub3c4 \ub3d9\ucc38\ud588\ub294\ub370, \uc774\ub97c \ud1b5\ud574 \ud5c8\uc704\uc0ac\uc2e4\uc740 \ud65c\uc790\uc640 \uc601\uc0c1\uc744 \ud1b5\ud574 \ub110\ub9ac \ud37c\uc838 \ub098\uac14\uace0, \uac70\uc758 \uc804 \uad6d\ubbfc\uc801\uc778 \uc778\uc9c0\ub3c4\ub97c \uac00\uc9c4 \ucd5c\uc9c4\uc2e4\uc774 \ubc1b\uc740 \ud0c0\uaca9\uc740 \uc0c1\ub2f9\ud788 \ucef8\uc73c\uba70, \ud6d7\ub0a0 \ucd5c\uc9c4\uc2e4\uc740 \uc774 \ub54c\uc758 \uc0c1\ud669\uc5d0 \ub300\ud574 \"\uad6d\ubbfc\uc744 '\ubc30\uc2ec\uc6d0'\uc73c\ub85c \uc549\ud600\ub193\uace0 2~3\ub144\uac04 \uc774\ud63c \ubb38\uc81c\ub85c \uc2dc\ub044\ub7fd\uac8c \ub5a0\ub4e4\uc5c8\ub2e4. \uadf8 \uacfc\uc815\uc5d0\uc11c \uce58\ubd80\ub97c \uc0dd\uc0dd\ud558\uac8c \ubcf4\uc5ec\uc918 \ub098\ub3c4 \uc544\uc774\ub4e4\ub3c4 \uce58\uba85\uc801\uc778 \uc0c1\ucc98\ub97c \uc785\uc5c8\ub2e4\"\ub77c\uace0 \ub9d0\ud558\uae30\ub3c4 \ud558\uc600\ub2e4. \uadf8 \ub2f9\uc2dc \uc5b8\ub860 \uc0c1\ud669\uc5d0 \ub300\ud574 \uc5f0\ucd9c\uac00 \uc815\uae38\ud654\ub294 \"IT \uac15\uad6d \ud55c\uad6d\uc758 \uc778\ud130\ub137\uacfc \ub09c\ub9bd\ud55c \uc5f0\uc608 \uc800\ub110\ub9ac\uc998\uc740 \uadf8 \ubaa8\ub4e0 \uc0c1\ud669\uc744 \uac70\uc758 \uc2e4\uc2dc\uac04\uc73c\ub85c \uc911\uacc4\ud588\ub2e4\"\ub77c\uace0 \ud45c\ud604\ud588\ub2e4. \uae30\uc790 \uc8fc\uc9c4\uc6b0\ub294 \"\ucd5c\uc9c4\uc2e4\uc740 \uadc0\uc5fd\uace0 \ubc1c\ub784\ud55c \uc5ec\uc131\uc758 \ub300\uba85\uc0ac\uc600\ub2e4. \uc774\ud63c \ub54c\ubb38\uc5d0 \uae54\ub054\ud55c \uc774\ubbf8\uc9c0\uac00 \uc190\uc0c1\ub418\uba74\uc11c \ucd5c\uc9c4\uc2e4\uc740 \ub098\ub77d\uc5d0 \ub5a8\uc5b4\uc84c\ub2e4. \uc5f0\uc608\uacc4\uc5d0\uc11c \uadf8\ub97c \ucc3e\ub294 \uacf3\uc740 \uc544\ubb34 \ub370\ub3c4 \uc5c6\uc5c8\ub2e4. 2004\ub144\uc5d0 MBC\uc758 \ud55c \uad6d\uc7a5\uc740 '\ucd5c\uc9c4\uc2e4\uc740 \ub05d\ub0ac\ub2e4. \uadc0\uc5fd\uace0 \ubc1c\ub784\ud568\uc73c\ub85c \ub5a0\uc624\ub974\ub358 \ucd5c\uc9c4\uc2e4\uc774 \uc774\uc81c\ub294 \uac00\uc815\ud3ed\ub825\uacfc \uc774\ud63c\uc758 \ub300\uba85\uc0ac\uac00 \ub418\uc5c8\ub294\ub370, \ucd5c\uc9c4\uc2e4\uc744 \uc5b4\ub514\uc5d0\ub2e4 \uc4f0\uaca0\ub294\uac00'\ub77c\uace0 \ub9d0\ud588\ub2e4\"\ub77c\uace0 \ub9d0\ud588\ub2e4. \uc774\ub294 \uac00\uc815\ud3ed\ub825\uc774\ub77c\ub294 \ubc94\uc8c4 \ud589\uc704\uc758 \ud53c\ud574\uc790\uc5d0\uac8c \uc624\ud788\ub824 \uadf8 \uc6b4\uc2e0\uc758 \ud3ed\uc744 \uc81c\ud55c\ud558\ub294 \ub300\ud55c\ubbfc\uad6d \ubc29\uc1a1\uacc4\uc758 \uc778\uc2dd \uc218\uc900\uc744 \ub2e8\uc801\uc73c\ub85c \ubcf4\uc5ec\uc8fc\ub294 \uac83\uc774\uc5c8\uace0, \uacb0\ud63c\uc744 \ud558\uba74 \uc774\ud63c\ud560 \uac00\ub2a5\uc131\uc740 \ub298 \uc788\ub294\ub370, \ud569\ubc95\uc801\uc778 \ud589\uc704\uc778 \uc774\ud63c\uc744 \ud588\ub2e4\ub294 \uc774\uc720\ub85c \uc0ac\ud68c\ud65c\ub3d9\uc744 \uc81c\ud55c\ubc1b\uc744 \uc218 \uc788\ub2e4\ub294 \uc0ac\ud68c \ubd84\uc704\uae30\ub97c \ub4dc\ub7ec\ub0b4\ub294 \uac83\uc774\uae30\ub3c4 \ud558\uc600\ub2e4. \ud3ed\ub825\ubc94\uc774 \uc720\uba85\uc778\uc744 \uc0c1\ub300\ub85c \ub300\ub193\uace0 \ud3ed\ub825 \ud589\uc704\ub97c \uc800\uc9c0\ub978 \uac83\uc740 \uc774\ub7f0 \uc0ac\ud68c \ubd84\uc704\uae30\uc640 \ubb34\uad00\ud558\uc9c0 \uc54a\uace0 \uc774\ub7f0 \uc0ac\ud68c \ubd84\uc704\uae30\ub294 \uc624\ud788\ub824 \uac00\uc815\ud3ed\ub825\uc744 \uc870\uc7a5\ud558\ub294 \uba74\uc774 \uc5c6\uc9c0 \uc54a\uc558\ub294\ub370, \ucd5c\uc9c4\uc2e4\uc740 \uc774\ud63c \uc219\ub824 \uae30\uac04\uc744 \uac00\uc9c0\ub294 \ub3d9\uc548\uc5d0 \uc54c\ub824\uc9c4 \uac83\ub9cc\uc73c\ub85c\ub3c4 \uac19\uc740 \uac00\ud574\uc790\ub85c\ubd80\ud130 \ub450 \ubc88\uc758 \uac00\uc815\ud3ed\ub825 \ud589\uc704\ub97c \ub2f9\ud588\uc5c8\uace0, \uccab \ubc88\uc9f8 \ud3ed\ub825 \ud589\uc704\ub294 \uc784\uc2e0 \uc911\uc5d0 \ub2f9\ud55c \uac83\uc774\uae30 \ub54c\ubb38\uc5d0 \ud0dc\uc544\ub3c4 \uadf8 \ud589\uc704\uc758 \ud53c\ud574\uc790\uc600\ub2e4. \uc774\ud63c \uacfc\uc815\uc5d0\uc11c \ucd5c\uc9c4\uc2e4\uc740 \ubbfc\uc0ac\uc0c1\u00b7\ud615\uc0ac\uc0c1 \ud53c\ud574\uc790\uc600\uc9c0\ub9cc, \uac1c\uc778\uc758 \uc544\ud508 \uc0ac\uc0dd\ud65c\uc5d0 \ub300\ud574\uc11c \uadf8\ub9ac\uace0 \uadf8 \ud53c\ud574\uc790\uc5d0 \ub300\ud574\uc11c \ud765\ubbf8 \uc704\uc8fc \uac00\uc2ed\uc73c\ub85c \ud314\uc544\uba39\ub294 \uc77c\ubd80 \uc5b8\ub860\uc758 '\uc0c1\ud63c' \ub4f1\uc73c\ub85c \uc774\uc911\uc73c\ub85c \uace0\ud1b5\uc744 \ubc1b\uc544\uc57c \ud588\ub2e4.", "answer": "\uc815\uae38\ud654", "sentence": "\uadf8 \ub2f9\uc2dc \uc5b8\ub860 \uc0c1\ud669\uc5d0 \ub300\ud574 \uc5f0\ucd9c\uac00 \uc815\uae38\ud654 \ub294 \"IT \uac15\uad6d \ud55c\uad6d\uc758 \uc778\ud130\ub137\uacfc \ub09c\ub9bd\ud55c \uc5f0\uc608 \uc800\ub110\ub9ac\uc998\uc740 \uadf8 \ubaa8\ub4e0 \uc0c1\ud669\uc744 \uac70\uc758 \uc2e4\uc2dc\uac04\uc73c\ub85c \uc911\uacc4\ud588\ub2e4\"\ub77c\uace0 \ud45c\ud604\ud588\ub2e4.", "paragraph_sentence": "\ucd5c\uc9c4\uc2e4\uc758 \uc5b4\uba38\ub2c8 \uc815\uc625\uc219\uc740 \uc790\uc2e0\uc758 \uc800\uc11c \u300a\uc5c4\ub9c8\uac00, \ubbf8\uc548\ud574 \uadf8\ub9ac\uace0 \uc0ac\ub791\ud574\u300b(2011\ub144)\uc5d0\uc11c \"\uc5b8\ub860\uc740 \uc784\uc2e0\ud55c \uc0ac\ub78c\uc744, \uc544\uc774 \uc5b4\uba38\ub2c8\ub97c \ubc30\ub824\ud558\uc9c0 \uc54a\uc558\ub2e4\"\ub77c\uace0 \uc9c0\uc801\ud558\uba70 \uae30\uc790\ub4e4\uc758 \uc724\ub9ac \uc758\uc2dd\uc5d0 \uc758\uad6c\uc2ec\uc744 \ud45c\ud604\ud588\ub2e4. \ucd5c\uc9c4\uc2e4\uc740 \uc774\ud63c \uacfc\uc815\uc5d0 \uc788\uc744 \ub54c \uc784\uc2e0\ud55c \uc0c1\ud0dc\uc600\ub294\ub370, \uae30\uc790\ub4e4\uc774 \ucd5c\uc9c4\uc2e4\uc758 \uc9d1 \uc55e\uc5d0 \uc9c4\uc744 \uce58\uace0 \uc788\uc5b4\uc11c \ucd5c\uc9c4\uc2e4\uc740 \uc815\uae30 \uac80\uc9c4\uc744 \ubc1b\ub294 \ub0a0 \uc678\uc5d0\ub294 \uac70\uc758 \uc9d1\uc5d0 \uac07\ud600 \uc788\uc5b4\uc57c \ud588\uace0, \uc774\ud63c \uacfc\uc815\uc744 \uc0dd\uc911\uacc4\ud558\ub294 \ub4ef\ud55c \uc120\uc815\uc801\uc774\uace0 \ubb34\ubd84\ubcc4\ud55c \uae30\uc0ac\uc5d0 \uace0\ud1b5\uc744 \uacaa\uae30\ub3c4 \ud558\uc600\ub2e4. \ucd5c\uc9c4\uc2e4\uc740 2\ub144\uc5ec \ub3d9\uc548\uc758 \uc774\ud63c \uacfc\uc815\uc5d0\uc11c \ud5c8\uc704\uc0ac\uc2e4\uc774\ub098 \uc0ac\uc2e4 \uc65c\uace1\uc744 \ubc14\ub85c\uc7a1\uae30 \uc704\ud574 \ubc18\ubc15\uc131\u00b7\ud574\uba85\uc131 \uc778\ud130\ubdf0\ub97c \ud558\uac70\ub098 \ubc95\uc801 \uc870\uce58\ub97c \ucde8\ud558\uae30\ub3c4 \ud558\uc600\ub294\ub370, \uc774\ub294 \uc0ac\uc0dd\ud65c\uc774 \ub354 \uc774\uc0c1 \uc138\uc0c1\uc5d0 \uc54c\ub824\uc9c0\uace0 \uc65c\uace1\ub418\ub294 \uac83\uc744 \ub9c9\uaca0\ub2e4\ub294 \ucde8\uc9c0\uc600\ub2e4. \uadf8 \ubc95\uc801 \uc870\uce58\uc758 \ub300\uc0c1\uc5d0\ub294 \uc5b8\ub860\ub3c4 \ud3ec\ud568\ub418\uc5b4 \uc788\uc5c8\ub2e4. \uc778\ud130\ubdf0\ub97c \uc790\uccad\ud558\uc5ec \ud5c8\uc704\uc0ac\uc2e4\uc744 \uc720\ud3ec\ud55c \uc790\uac00 \uc788\uc5c8\uace0 \uadf8 \uc778\ud130\ubdf0\ub97c \uc9c4\ud589\ud55c \uae30\uc790\ub294 \uadf8 \uc720\ud3ec \ub0b4\uc6a9\uc744 \uadf8\ub300\ub85c \ubc1b\uc544\uc801\uc5b4 \uae30\uc0ac\ud654\ud558\uc600\ub294\ub370, \uc774\ub294 \uae30\uc790\uac00 \ud5c8\uc704\uc0ac\uc2e4\uc720\ud3ec \ud589\uc704\uc5d0 \ub3d9\ucc38\ud55c \uac83\uc774\uba74\uc11c \ud5c8\uc704\uc0ac\uc2e4 \uc720\ud3ec\uc790\uc640 \uae30\uc790 \ubaa8\ub450 \uc774\ud63c\uc740 \ud63c\uc778 \ub2f9\uc0ac\uc790\uc640 \uad6d\uac00\uac00 \ud574\uacb0\ubd10\uc57c \ud558\ub294 \uc0ac\uc548\uc774\ub77c\ub294 \uc0ac\uc2e4\uc744 \ub9dd\uac01\ud55c \ud589\uc704\uc774\uae30\ub3c4 \ud558\uc600\ub2e4. \uc774\ub7f0 \ud5c8\uc704\uc0ac\uc2e4\uc720\ud3ec \ud589\uc704\uc5d0 \ubc29\uc1a1\uc0ac\ub3c4 \ub3d9\ucc38\ud588\ub294\ub370, \uc774\ub97c \ud1b5\ud574 \ud5c8\uc704\uc0ac\uc2e4\uc740 \ud65c\uc790\uc640 \uc601\uc0c1\uc744 \ud1b5\ud574 \ub110\ub9ac \ud37c\uc838 \ub098\uac14\uace0, \uac70\uc758 \uc804 \uad6d\ubbfc\uc801\uc778 \uc778\uc9c0\ub3c4\ub97c \uac00\uc9c4 \ucd5c\uc9c4\uc2e4\uc774 \ubc1b\uc740 \ud0c0\uaca9\uc740 \uc0c1\ub2f9\ud788 \ucef8\uc73c\uba70, \ud6d7\ub0a0 \ucd5c\uc9c4\uc2e4\uc740 \uc774 \ub54c\uc758 \uc0c1\ud669\uc5d0 \ub300\ud574 \"\uad6d\ubbfc\uc744 '\ubc30\uc2ec\uc6d0'\uc73c\ub85c \uc549\ud600\ub193\uace0 2~3\ub144\uac04 \uc774\ud63c \ubb38\uc81c\ub85c \uc2dc\ub044\ub7fd\uac8c \ub5a0\ub4e4\uc5c8\ub2e4. \uadf8 \uacfc\uc815\uc5d0\uc11c \uce58\ubd80\ub97c \uc0dd\uc0dd\ud558\uac8c \ubcf4\uc5ec\uc918 \ub098\ub3c4 \uc544\uc774\ub4e4\ub3c4 \uce58\uba85\uc801\uc778 \uc0c1\ucc98\ub97c \uc785\uc5c8\ub2e4\"\ub77c\uace0 \ub9d0\ud558\uae30\ub3c4 \ud558\uc600\ub2e4. \uadf8 \ub2f9\uc2dc \uc5b8\ub860 \uc0c1\ud669\uc5d0 \ub300\ud574 \uc5f0\ucd9c\uac00 \uc815\uae38\ud654 \ub294 \"IT \uac15\uad6d \ud55c\uad6d\uc758 \uc778\ud130\ub137\uacfc \ub09c\ub9bd\ud55c \uc5f0\uc608 \uc800\ub110\ub9ac\uc998\uc740 \uadf8 \ubaa8\ub4e0 \uc0c1\ud669\uc744 \uac70\uc758 \uc2e4\uc2dc\uac04\uc73c\ub85c \uc911\uacc4\ud588\ub2e4\"\ub77c\uace0 \ud45c\ud604\ud588\ub2e4. \uae30\uc790 \uc8fc\uc9c4\uc6b0\ub294 \"\ucd5c\uc9c4\uc2e4\uc740 \uadc0\uc5fd\uace0 \ubc1c\ub784\ud55c \uc5ec\uc131\uc758 \ub300\uba85\uc0ac\uc600\ub2e4. \uc774\ud63c \ub54c\ubb38\uc5d0 \uae54\ub054\ud55c \uc774\ubbf8\uc9c0\uac00 \uc190\uc0c1\ub418\uba74\uc11c \ucd5c\uc9c4\uc2e4\uc740 \ub098\ub77d\uc5d0 \ub5a8\uc5b4\uc84c\ub2e4. \uc5f0\uc608\uacc4\uc5d0\uc11c \uadf8\ub97c \ucc3e\ub294 \uacf3\uc740 \uc544\ubb34 \ub370\ub3c4 \uc5c6\uc5c8\ub2e4. 2004\ub144\uc5d0 MBC\uc758 \ud55c \uad6d\uc7a5\uc740 '\ucd5c\uc9c4\uc2e4\uc740 \ub05d\ub0ac\ub2e4. \uadc0\uc5fd\uace0 \ubc1c\ub784\ud568\uc73c\ub85c \ub5a0\uc624\ub974\ub358 \ucd5c\uc9c4\uc2e4\uc774 \uc774\uc81c\ub294 \uac00\uc815\ud3ed\ub825\uacfc \uc774\ud63c\uc758 \ub300\uba85\uc0ac\uac00 \ub418\uc5c8\ub294\ub370, \ucd5c\uc9c4\uc2e4\uc744 \uc5b4\ub514\uc5d0\ub2e4 \uc4f0\uaca0\ub294\uac00'\ub77c\uace0 \ub9d0\ud588\ub2e4\"\ub77c\uace0 \ub9d0\ud588\ub2e4. \uc774\ub294 \uac00\uc815\ud3ed\ub825\uc774\ub77c\ub294 \ubc94\uc8c4 \ud589\uc704\uc758 \ud53c\ud574\uc790\uc5d0\uac8c \uc624\ud788\ub824 \uadf8 \uc6b4\uc2e0\uc758 \ud3ed\uc744 \uc81c\ud55c\ud558\ub294 \ub300\ud55c\ubbfc\uad6d \ubc29\uc1a1\uacc4\uc758 \uc778\uc2dd \uc218\uc900\uc744 \ub2e8\uc801\uc73c\ub85c \ubcf4\uc5ec\uc8fc\ub294 \uac83\uc774\uc5c8\uace0, \uacb0\ud63c\uc744 \ud558\uba74 \uc774\ud63c\ud560 \uac00\ub2a5\uc131\uc740 \ub298 \uc788\ub294\ub370, \ud569\ubc95\uc801\uc778 \ud589\uc704\uc778 \uc774\ud63c\uc744 \ud588\ub2e4\ub294 \uc774\uc720\ub85c \uc0ac\ud68c\ud65c\ub3d9\uc744 \uc81c\ud55c\ubc1b\uc744 \uc218 \uc788\ub2e4\ub294 \uc0ac\ud68c \ubd84\uc704\uae30\ub97c \ub4dc\ub7ec\ub0b4\ub294 \uac83\uc774\uae30\ub3c4 \ud558\uc600\ub2e4. \ud3ed\ub825\ubc94\uc774 \uc720\uba85\uc778\uc744 \uc0c1\ub300\ub85c \ub300\ub193\uace0 \ud3ed\ub825 \ud589\uc704\ub97c \uc800\uc9c0\ub978 \uac83\uc740 \uc774\ub7f0 \uc0ac\ud68c \ubd84\uc704\uae30\uc640 \ubb34\uad00\ud558\uc9c0 \uc54a\uace0 \uc774\ub7f0 \uc0ac\ud68c \ubd84\uc704\uae30\ub294 \uc624\ud788\ub824 \uac00\uc815\ud3ed\ub825\uc744 \uc870\uc7a5\ud558\ub294 \uba74\uc774 \uc5c6\uc9c0 \uc54a\uc558\ub294\ub370, \ucd5c\uc9c4\uc2e4\uc740 \uc774\ud63c \uc219\ub824 \uae30\uac04\uc744 \uac00\uc9c0\ub294 \ub3d9\uc548\uc5d0 \uc54c\ub824\uc9c4 \uac83\ub9cc\uc73c\ub85c\ub3c4 \uac19\uc740 \uac00\ud574\uc790\ub85c\ubd80\ud130 \ub450 \ubc88\uc758 \uac00\uc815\ud3ed\ub825 \ud589\uc704\ub97c \ub2f9\ud588\uc5c8\uace0, \uccab \ubc88\uc9f8 \ud3ed\ub825 \ud589\uc704\ub294 \uc784\uc2e0 \uc911\uc5d0 \ub2f9\ud55c \uac83\uc774\uae30 \ub54c\ubb38\uc5d0 \ud0dc\uc544\ub3c4 \uadf8 \ud589\uc704\uc758 \ud53c\ud574\uc790\uc600\ub2e4. \uc774\ud63c \uacfc\uc815\uc5d0\uc11c \ucd5c\uc9c4\uc2e4\uc740 \ubbfc\uc0ac\uc0c1\u00b7\ud615\uc0ac\uc0c1 \ud53c\ud574\uc790\uc600\uc9c0\ub9cc, \uac1c\uc778\uc758 \uc544\ud508 \uc0ac\uc0dd\ud65c\uc5d0 \ub300\ud574\uc11c \uadf8\ub9ac\uace0 \uadf8 \ud53c\ud574\uc790\uc5d0 \ub300\ud574\uc11c \ud765\ubbf8 \uc704\uc8fc \uac00\uc2ed\uc73c\ub85c \ud314\uc544\uba39\ub294 \uc77c\ubd80 \uc5b8\ub860\uc758 '\uc0c1\ud63c' \ub4f1\uc73c\ub85c \uc774\uc911\uc73c\ub85c \uace0\ud1b5\uc744 \ubc1b\uc544\uc57c \ud588\ub2e4.", "paragraph_answer": "\ucd5c\uc9c4\uc2e4\uc758 \uc5b4\uba38\ub2c8 \uc815\uc625\uc219\uc740 \uc790\uc2e0\uc758 \uc800\uc11c \u300a\uc5c4\ub9c8\uac00, \ubbf8\uc548\ud574 \uadf8\ub9ac\uace0 \uc0ac\ub791\ud574\u300b(2011\ub144)\uc5d0\uc11c \"\uc5b8\ub860\uc740 \uc784\uc2e0\ud55c \uc0ac\ub78c\uc744, \uc544\uc774 \uc5b4\uba38\ub2c8\ub97c \ubc30\ub824\ud558\uc9c0 \uc54a\uc558\ub2e4\"\ub77c\uace0 \uc9c0\uc801\ud558\uba70 \uae30\uc790\ub4e4\uc758 \uc724\ub9ac \uc758\uc2dd\uc5d0 \uc758\uad6c\uc2ec\uc744 \ud45c\ud604\ud588\ub2e4. \ucd5c\uc9c4\uc2e4\uc740 \uc774\ud63c \uacfc\uc815\uc5d0 \uc788\uc744 \ub54c \uc784\uc2e0\ud55c \uc0c1\ud0dc\uc600\ub294\ub370, \uae30\uc790\ub4e4\uc774 \ucd5c\uc9c4\uc2e4\uc758 \uc9d1 \uc55e\uc5d0 \uc9c4\uc744 \uce58\uace0 \uc788\uc5b4\uc11c \ucd5c\uc9c4\uc2e4\uc740 \uc815\uae30 \uac80\uc9c4\uc744 \ubc1b\ub294 \ub0a0 \uc678\uc5d0\ub294 \uac70\uc758 \uc9d1\uc5d0 \uac07\ud600 \uc788\uc5b4\uc57c \ud588\uace0, \uc774\ud63c \uacfc\uc815\uc744 \uc0dd\uc911\uacc4\ud558\ub294 \ub4ef\ud55c \uc120\uc815\uc801\uc774\uace0 \ubb34\ubd84\ubcc4\ud55c \uae30\uc0ac\uc5d0 \uace0\ud1b5\uc744 \uacaa\uae30\ub3c4 \ud558\uc600\ub2e4. \ucd5c\uc9c4\uc2e4\uc740 2\ub144\uc5ec \ub3d9\uc548\uc758 \uc774\ud63c \uacfc\uc815\uc5d0\uc11c \ud5c8\uc704\uc0ac\uc2e4\uc774\ub098 \uc0ac\uc2e4 \uc65c\uace1\uc744 \ubc14\ub85c\uc7a1\uae30 \uc704\ud574 \ubc18\ubc15\uc131\u00b7\ud574\uba85\uc131 \uc778\ud130\ubdf0\ub97c \ud558\uac70\ub098 \ubc95\uc801 \uc870\uce58\ub97c \ucde8\ud558\uae30\ub3c4 \ud558\uc600\ub294\ub370, \uc774\ub294 \uc0ac\uc0dd\ud65c\uc774 \ub354 \uc774\uc0c1 \uc138\uc0c1\uc5d0 \uc54c\ub824\uc9c0\uace0 \uc65c\uace1\ub418\ub294 \uac83\uc744 \ub9c9\uaca0\ub2e4\ub294 \ucde8\uc9c0\uc600\ub2e4. \uadf8 \ubc95\uc801 \uc870\uce58\uc758 \ub300\uc0c1\uc5d0\ub294 \uc5b8\ub860\ub3c4 \ud3ec\ud568\ub418\uc5b4 \uc788\uc5c8\ub2e4. \uc778\ud130\ubdf0\ub97c \uc790\uccad\ud558\uc5ec \ud5c8\uc704\uc0ac\uc2e4\uc744 \uc720\ud3ec\ud55c \uc790\uac00 \uc788\uc5c8\uace0 \uadf8 \uc778\ud130\ubdf0\ub97c \uc9c4\ud589\ud55c \uae30\uc790\ub294 \uadf8 \uc720\ud3ec \ub0b4\uc6a9\uc744 \uadf8\ub300\ub85c \ubc1b\uc544\uc801\uc5b4 \uae30\uc0ac\ud654\ud558\uc600\ub294\ub370, \uc774\ub294 \uae30\uc790\uac00 \ud5c8\uc704\uc0ac\uc2e4\uc720\ud3ec \ud589\uc704\uc5d0 \ub3d9\ucc38\ud55c \uac83\uc774\uba74\uc11c \ud5c8\uc704\uc0ac\uc2e4 \uc720\ud3ec\uc790\uc640 \uae30\uc790 \ubaa8\ub450 \uc774\ud63c\uc740 \ud63c\uc778 \ub2f9\uc0ac\uc790\uc640 \uad6d\uac00\uac00 \ud574\uacb0\ubd10\uc57c \ud558\ub294 \uc0ac\uc548\uc774\ub77c\ub294 \uc0ac\uc2e4\uc744 \ub9dd\uac01\ud55c \ud589\uc704\uc774\uae30\ub3c4 \ud558\uc600\ub2e4. \uc774\ub7f0 \ud5c8\uc704\uc0ac\uc2e4\uc720\ud3ec \ud589\uc704\uc5d0 \ubc29\uc1a1\uc0ac\ub3c4 \ub3d9\ucc38\ud588\ub294\ub370, \uc774\ub97c \ud1b5\ud574 \ud5c8\uc704\uc0ac\uc2e4\uc740 \ud65c\uc790\uc640 \uc601\uc0c1\uc744 \ud1b5\ud574 \ub110\ub9ac \ud37c\uc838 \ub098\uac14\uace0, \uac70\uc758 \uc804 \uad6d\ubbfc\uc801\uc778 \uc778\uc9c0\ub3c4\ub97c \uac00\uc9c4 \ucd5c\uc9c4\uc2e4\uc774 \ubc1b\uc740 \ud0c0\uaca9\uc740 \uc0c1\ub2f9\ud788 \ucef8\uc73c\uba70, \ud6d7\ub0a0 \ucd5c\uc9c4\uc2e4\uc740 \uc774 \ub54c\uc758 \uc0c1\ud669\uc5d0 \ub300\ud574 \"\uad6d\ubbfc\uc744 '\ubc30\uc2ec\uc6d0'\uc73c\ub85c \uc549\ud600\ub193\uace0 2~3\ub144\uac04 \uc774\ud63c \ubb38\uc81c\ub85c \uc2dc\ub044\ub7fd\uac8c \ub5a0\ub4e4\uc5c8\ub2e4. \uadf8 \uacfc\uc815\uc5d0\uc11c \uce58\ubd80\ub97c \uc0dd\uc0dd\ud558\uac8c \ubcf4\uc5ec\uc918 \ub098\ub3c4 \uc544\uc774\ub4e4\ub3c4 \uce58\uba85\uc801\uc778 \uc0c1\ucc98\ub97c \uc785\uc5c8\ub2e4\"\ub77c\uace0 \ub9d0\ud558\uae30\ub3c4 \ud558\uc600\ub2e4. \uadf8 \ub2f9\uc2dc \uc5b8\ub860 \uc0c1\ud669\uc5d0 \ub300\ud574 \uc5f0\ucd9c\uac00 \uc815\uae38\ud654 \ub294 \"IT \uac15\uad6d \ud55c\uad6d\uc758 \uc778\ud130\ub137\uacfc \ub09c\ub9bd\ud55c \uc5f0\uc608 \uc800\ub110\ub9ac\uc998\uc740 \uadf8 \ubaa8\ub4e0 \uc0c1\ud669\uc744 \uac70\uc758 \uc2e4\uc2dc\uac04\uc73c\ub85c \uc911\uacc4\ud588\ub2e4\"\ub77c\uace0 \ud45c\ud604\ud588\ub2e4. \uae30\uc790 \uc8fc\uc9c4\uc6b0\ub294 \"\ucd5c\uc9c4\uc2e4\uc740 \uadc0\uc5fd\uace0 \ubc1c\ub784\ud55c \uc5ec\uc131\uc758 \ub300\uba85\uc0ac\uc600\ub2e4. \uc774\ud63c \ub54c\ubb38\uc5d0 \uae54\ub054\ud55c \uc774\ubbf8\uc9c0\uac00 \uc190\uc0c1\ub418\uba74\uc11c \ucd5c\uc9c4\uc2e4\uc740 \ub098\ub77d\uc5d0 \ub5a8\uc5b4\uc84c\ub2e4. \uc5f0\uc608\uacc4\uc5d0\uc11c \uadf8\ub97c \ucc3e\ub294 \uacf3\uc740 \uc544\ubb34 \ub370\ub3c4 \uc5c6\uc5c8\ub2e4. 2004\ub144\uc5d0 MBC\uc758 \ud55c \uad6d\uc7a5\uc740 '\ucd5c\uc9c4\uc2e4\uc740 \ub05d\ub0ac\ub2e4. \uadc0\uc5fd\uace0 \ubc1c\ub784\ud568\uc73c\ub85c \ub5a0\uc624\ub974\ub358 \ucd5c\uc9c4\uc2e4\uc774 \uc774\uc81c\ub294 \uac00\uc815\ud3ed\ub825\uacfc \uc774\ud63c\uc758 \ub300\uba85\uc0ac\uac00 \ub418\uc5c8\ub294\ub370, \ucd5c\uc9c4\uc2e4\uc744 \uc5b4\ub514\uc5d0\ub2e4 \uc4f0\uaca0\ub294\uac00'\ub77c\uace0 \ub9d0\ud588\ub2e4\"\ub77c\uace0 \ub9d0\ud588\ub2e4. \uc774\ub294 \uac00\uc815\ud3ed\ub825\uc774\ub77c\ub294 \ubc94\uc8c4 \ud589\uc704\uc758 \ud53c\ud574\uc790\uc5d0\uac8c \uc624\ud788\ub824 \uadf8 \uc6b4\uc2e0\uc758 \ud3ed\uc744 \uc81c\ud55c\ud558\ub294 \ub300\ud55c\ubbfc\uad6d \ubc29\uc1a1\uacc4\uc758 \uc778\uc2dd \uc218\uc900\uc744 \ub2e8\uc801\uc73c\ub85c \ubcf4\uc5ec\uc8fc\ub294 \uac83\uc774\uc5c8\uace0, \uacb0\ud63c\uc744 \ud558\uba74 \uc774\ud63c\ud560 \uac00\ub2a5\uc131\uc740 \ub298 \uc788\ub294\ub370, \ud569\ubc95\uc801\uc778 \ud589\uc704\uc778 \uc774\ud63c\uc744 \ud588\ub2e4\ub294 \uc774\uc720\ub85c \uc0ac\ud68c\ud65c\ub3d9\uc744 \uc81c\ud55c\ubc1b\uc744 \uc218 \uc788\ub2e4\ub294 \uc0ac\ud68c \ubd84\uc704\uae30\ub97c \ub4dc\ub7ec\ub0b4\ub294 \uac83\uc774\uae30\ub3c4 \ud558\uc600\ub2e4. \ud3ed\ub825\ubc94\uc774 \uc720\uba85\uc778\uc744 \uc0c1\ub300\ub85c \ub300\ub193\uace0 \ud3ed\ub825 \ud589\uc704\ub97c \uc800\uc9c0\ub978 \uac83\uc740 \uc774\ub7f0 \uc0ac\ud68c \ubd84\uc704\uae30\uc640 \ubb34\uad00\ud558\uc9c0 \uc54a\uace0 \uc774\ub7f0 \uc0ac\ud68c \ubd84\uc704\uae30\ub294 \uc624\ud788\ub824 \uac00\uc815\ud3ed\ub825\uc744 \uc870\uc7a5\ud558\ub294 \uba74\uc774 \uc5c6\uc9c0 \uc54a\uc558\ub294\ub370, \ucd5c\uc9c4\uc2e4\uc740 \uc774\ud63c \uc219\ub824 \uae30\uac04\uc744 \uac00\uc9c0\ub294 \ub3d9\uc548\uc5d0 \uc54c\ub824\uc9c4 \uac83\ub9cc\uc73c\ub85c\ub3c4 \uac19\uc740 \uac00\ud574\uc790\ub85c\ubd80\ud130 \ub450 \ubc88\uc758 \uac00\uc815\ud3ed\ub825 \ud589\uc704\ub97c \ub2f9\ud588\uc5c8\uace0, \uccab \ubc88\uc9f8 \ud3ed\ub825 \ud589\uc704\ub294 \uc784\uc2e0 \uc911\uc5d0 \ub2f9\ud55c \uac83\uc774\uae30 \ub54c\ubb38\uc5d0 \ud0dc\uc544\ub3c4 \uadf8 \ud589\uc704\uc758 \ud53c\ud574\uc790\uc600\ub2e4. \uc774\ud63c \uacfc\uc815\uc5d0\uc11c \ucd5c\uc9c4\uc2e4\uc740 \ubbfc\uc0ac\uc0c1\u00b7\ud615\uc0ac\uc0c1 \ud53c\ud574\uc790\uc600\uc9c0\ub9cc, \uac1c\uc778\uc758 \uc544\ud508 \uc0ac\uc0dd\ud65c\uc5d0 \ub300\ud574\uc11c \uadf8\ub9ac\uace0 \uadf8 \ud53c\ud574\uc790\uc5d0 \ub300\ud574\uc11c \ud765\ubbf8 \uc704\uc8fc \uac00\uc2ed\uc73c\ub85c \ud314\uc544\uba39\ub294 \uc77c\ubd80 \uc5b8\ub860\uc758 '\uc0c1\ud63c' \ub4f1\uc73c\ub85c \uc774\uc911\uc73c\ub85c \uace0\ud1b5\uc744 \ubc1b\uc544\uc57c \ud588\ub2e4.", "sentence_answer": "\uadf8 \ub2f9\uc2dc \uc5b8\ub860 \uc0c1\ud669\uc5d0 \ub300\ud574 \uc5f0\ucd9c\uac00 \uc815\uae38\ud654 \ub294 \"IT \uac15\uad6d \ud55c\uad6d\uc758 \uc778\ud130\ub137\uacfc \ub09c\ub9bd\ud55c \uc5f0\uc608 \uc800\ub110\ub9ac\uc998\uc740 \uadf8 \ubaa8\ub4e0 \uc0c1\ud669\uc744 \uac70\uc758 \uc2e4\uc2dc\uac04\uc73c\ub85c \uc911\uacc4\ud588\ub2e4\"\ub77c\uace0 \ud45c\ud604\ud588\ub2e4.", "paragraph_id": "6560193-11-1", "paragraph_question": "question: \uc778\ud130\ub137\uacfc \uc5f0\uc608\ub9e4\uccb4\ub4e4\uc774 \uc774\ud63c\uacfc\uc815\uc744 \uc2e4\uc2dc\uac04 \uc911\uacc4\ud588\ub2e4\uace0 \ub9d0\ud55c \uc5f0\ucd9c\uac00\ub294?, context: \ucd5c\uc9c4\uc2e4\uc758 \uc5b4\uba38\ub2c8 \uc815\uc625\uc219\uc740 \uc790\uc2e0\uc758 \uc800\uc11c \u300a\uc5c4\ub9c8\uac00, \ubbf8\uc548\ud574 \uadf8\ub9ac\uace0 \uc0ac\ub791\ud574\u300b(2011\ub144)\uc5d0\uc11c \"\uc5b8\ub860\uc740 \uc784\uc2e0\ud55c \uc0ac\ub78c\uc744, \uc544\uc774 \uc5b4\uba38\ub2c8\ub97c \ubc30\ub824\ud558\uc9c0 \uc54a\uc558\ub2e4\"\ub77c\uace0 \uc9c0\uc801\ud558\uba70 \uae30\uc790\ub4e4\uc758 \uc724\ub9ac \uc758\uc2dd\uc5d0 \uc758\uad6c\uc2ec\uc744 \ud45c\ud604\ud588\ub2e4. \ucd5c\uc9c4\uc2e4\uc740 \uc774\ud63c \uacfc\uc815\uc5d0 \uc788\uc744 \ub54c \uc784\uc2e0\ud55c \uc0c1\ud0dc\uc600\ub294\ub370, \uae30\uc790\ub4e4\uc774 \ucd5c\uc9c4\uc2e4\uc758 \uc9d1 \uc55e\uc5d0 \uc9c4\uc744 \uce58\uace0 \uc788\uc5b4\uc11c \ucd5c\uc9c4\uc2e4\uc740 \uc815\uae30 \uac80\uc9c4\uc744 \ubc1b\ub294 \ub0a0 \uc678\uc5d0\ub294 \uac70\uc758 \uc9d1\uc5d0 \uac07\ud600 \uc788\uc5b4\uc57c \ud588\uace0, \uc774\ud63c \uacfc\uc815\uc744 \uc0dd\uc911\uacc4\ud558\ub294 \ub4ef\ud55c \uc120\uc815\uc801\uc774\uace0 \ubb34\ubd84\ubcc4\ud55c \uae30\uc0ac\uc5d0 \uace0\ud1b5\uc744 \uacaa\uae30\ub3c4 \ud558\uc600\ub2e4. \ucd5c\uc9c4\uc2e4\uc740 2\ub144\uc5ec \ub3d9\uc548\uc758 \uc774\ud63c \uacfc\uc815\uc5d0\uc11c \ud5c8\uc704\uc0ac\uc2e4\uc774\ub098 \uc0ac\uc2e4 \uc65c\uace1\uc744 \ubc14\ub85c\uc7a1\uae30 \uc704\ud574 \ubc18\ubc15\uc131\u00b7\ud574\uba85\uc131 \uc778\ud130\ubdf0\ub97c \ud558\uac70\ub098 \ubc95\uc801 \uc870\uce58\ub97c \ucde8\ud558\uae30\ub3c4 \ud558\uc600\ub294\ub370, \uc774\ub294 \uc0ac\uc0dd\ud65c\uc774 \ub354 \uc774\uc0c1 \uc138\uc0c1\uc5d0 \uc54c\ub824\uc9c0\uace0 \uc65c\uace1\ub418\ub294 \uac83\uc744 \ub9c9\uaca0\ub2e4\ub294 \ucde8\uc9c0\uc600\ub2e4. \uadf8 \ubc95\uc801 \uc870\uce58\uc758 \ub300\uc0c1\uc5d0\ub294 \uc5b8\ub860\ub3c4 \ud3ec\ud568\ub418\uc5b4 \uc788\uc5c8\ub2e4. \uc778\ud130\ubdf0\ub97c \uc790\uccad\ud558\uc5ec \ud5c8\uc704\uc0ac\uc2e4\uc744 \uc720\ud3ec\ud55c \uc790\uac00 \uc788\uc5c8\uace0 \uadf8 \uc778\ud130\ubdf0\ub97c \uc9c4\ud589\ud55c \uae30\uc790\ub294 \uadf8 \uc720\ud3ec \ub0b4\uc6a9\uc744 \uadf8\ub300\ub85c \ubc1b\uc544\uc801\uc5b4 \uae30\uc0ac\ud654\ud558\uc600\ub294\ub370, \uc774\ub294 \uae30\uc790\uac00 \ud5c8\uc704\uc0ac\uc2e4\uc720\ud3ec \ud589\uc704\uc5d0 \ub3d9\ucc38\ud55c \uac83\uc774\uba74\uc11c \ud5c8\uc704\uc0ac\uc2e4 \uc720\ud3ec\uc790\uc640 \uae30\uc790 \ubaa8\ub450 \uc774\ud63c\uc740 \ud63c\uc778 \ub2f9\uc0ac\uc790\uc640 \uad6d\uac00\uac00 \ud574\uacb0\ubd10\uc57c \ud558\ub294 \uc0ac\uc548\uc774\ub77c\ub294 \uc0ac\uc2e4\uc744 \ub9dd\uac01\ud55c \ud589\uc704\uc774\uae30\ub3c4 \ud558\uc600\ub2e4. \uc774\ub7f0 \ud5c8\uc704\uc0ac\uc2e4\uc720\ud3ec \ud589\uc704\uc5d0 \ubc29\uc1a1\uc0ac\ub3c4 \ub3d9\ucc38\ud588\ub294\ub370, \uc774\ub97c \ud1b5\ud574 \ud5c8\uc704\uc0ac\uc2e4\uc740 \ud65c\uc790\uc640 \uc601\uc0c1\uc744 \ud1b5\ud574 \ub110\ub9ac \ud37c\uc838 \ub098\uac14\uace0, \uac70\uc758 \uc804 \uad6d\ubbfc\uc801\uc778 \uc778\uc9c0\ub3c4\ub97c \uac00\uc9c4 \ucd5c\uc9c4\uc2e4\uc774 \ubc1b\uc740 \ud0c0\uaca9\uc740 \uc0c1\ub2f9\ud788 \ucef8\uc73c\uba70, \ud6d7\ub0a0 \ucd5c\uc9c4\uc2e4\uc740 \uc774 \ub54c\uc758 \uc0c1\ud669\uc5d0 \ub300\ud574 \"\uad6d\ubbfc\uc744 '\ubc30\uc2ec\uc6d0'\uc73c\ub85c \uc549\ud600\ub193\uace0 2~3\ub144\uac04 \uc774\ud63c \ubb38\uc81c\ub85c \uc2dc\ub044\ub7fd\uac8c \ub5a0\ub4e4\uc5c8\ub2e4. \uadf8 \uacfc\uc815\uc5d0\uc11c \uce58\ubd80\ub97c \uc0dd\uc0dd\ud558\uac8c \ubcf4\uc5ec\uc918 \ub098\ub3c4 \uc544\uc774\ub4e4\ub3c4 \uce58\uba85\uc801\uc778 \uc0c1\ucc98\ub97c \uc785\uc5c8\ub2e4\"\ub77c\uace0 \ub9d0\ud558\uae30\ub3c4 \ud558\uc600\ub2e4. \uadf8 \ub2f9\uc2dc \uc5b8\ub860 \uc0c1\ud669\uc5d0 \ub300\ud574 \uc5f0\ucd9c\uac00 \uc815\uae38\ud654\ub294 \"IT \uac15\uad6d \ud55c\uad6d\uc758 \uc778\ud130\ub137\uacfc \ub09c\ub9bd\ud55c \uc5f0\uc608 \uc800\ub110\ub9ac\uc998\uc740 \uadf8 \ubaa8\ub4e0 \uc0c1\ud669\uc744 \uac70\uc758 \uc2e4\uc2dc\uac04\uc73c\ub85c \uc911\uacc4\ud588\ub2e4\"\ub77c\uace0 \ud45c\ud604\ud588\ub2e4. \uae30\uc790 \uc8fc\uc9c4\uc6b0\ub294 \"\ucd5c\uc9c4\uc2e4\uc740 \uadc0\uc5fd\uace0 \ubc1c\ub784\ud55c \uc5ec\uc131\uc758 \ub300\uba85\uc0ac\uc600\ub2e4. \uc774\ud63c \ub54c\ubb38\uc5d0 \uae54\ub054\ud55c \uc774\ubbf8\uc9c0\uac00 \uc190\uc0c1\ub418\uba74\uc11c \ucd5c\uc9c4\uc2e4\uc740 \ub098\ub77d\uc5d0 \ub5a8\uc5b4\uc84c\ub2e4. \uc5f0\uc608\uacc4\uc5d0\uc11c \uadf8\ub97c \ucc3e\ub294 \uacf3\uc740 \uc544\ubb34 \ub370\ub3c4 \uc5c6\uc5c8\ub2e4. 2004\ub144\uc5d0 MBC\uc758 \ud55c \uad6d\uc7a5\uc740 '\ucd5c\uc9c4\uc2e4\uc740 \ub05d\ub0ac\ub2e4. \uadc0\uc5fd\uace0 \ubc1c\ub784\ud568\uc73c\ub85c \ub5a0\uc624\ub974\ub358 \ucd5c\uc9c4\uc2e4\uc774 \uc774\uc81c\ub294 \uac00\uc815\ud3ed\ub825\uacfc \uc774\ud63c\uc758 \ub300\uba85\uc0ac\uac00 \ub418\uc5c8\ub294\ub370, \ucd5c\uc9c4\uc2e4\uc744 \uc5b4\ub514\uc5d0\ub2e4 \uc4f0\uaca0\ub294\uac00'\ub77c\uace0 \ub9d0\ud588\ub2e4\"\ub77c\uace0 \ub9d0\ud588\ub2e4. \uc774\ub294 \uac00\uc815\ud3ed\ub825\uc774\ub77c\ub294 \ubc94\uc8c4 \ud589\uc704\uc758 \ud53c\ud574\uc790\uc5d0\uac8c \uc624\ud788\ub824 \uadf8 \uc6b4\uc2e0\uc758 \ud3ed\uc744 \uc81c\ud55c\ud558\ub294 \ub300\ud55c\ubbfc\uad6d \ubc29\uc1a1\uacc4\uc758 \uc778\uc2dd \uc218\uc900\uc744 \ub2e8\uc801\uc73c\ub85c \ubcf4\uc5ec\uc8fc\ub294 \uac83\uc774\uc5c8\uace0, \uacb0\ud63c\uc744 \ud558\uba74 \uc774\ud63c\ud560 \uac00\ub2a5\uc131\uc740 \ub298 \uc788\ub294\ub370, \ud569\ubc95\uc801\uc778 \ud589\uc704\uc778 \uc774\ud63c\uc744 \ud588\ub2e4\ub294 \uc774\uc720\ub85c \uc0ac\ud68c\ud65c\ub3d9\uc744 \uc81c\ud55c\ubc1b\uc744 \uc218 \uc788\ub2e4\ub294 \uc0ac\ud68c \ubd84\uc704\uae30\ub97c \ub4dc\ub7ec\ub0b4\ub294 \uac83\uc774\uae30\ub3c4 \ud558\uc600\ub2e4. \ud3ed\ub825\ubc94\uc774 \uc720\uba85\uc778\uc744 \uc0c1\ub300\ub85c \ub300\ub193\uace0 \ud3ed\ub825 \ud589\uc704\ub97c \uc800\uc9c0\ub978 \uac83\uc740 \uc774\ub7f0 \uc0ac\ud68c \ubd84\uc704\uae30\uc640 \ubb34\uad00\ud558\uc9c0 \uc54a\uace0 \uc774\ub7f0 \uc0ac\ud68c \ubd84\uc704\uae30\ub294 \uc624\ud788\ub824 \uac00\uc815\ud3ed\ub825\uc744 \uc870\uc7a5\ud558\ub294 \uba74\uc774 \uc5c6\uc9c0 \uc54a\uc558\ub294\ub370, \ucd5c\uc9c4\uc2e4\uc740 \uc774\ud63c \uc219\ub824 \uae30\uac04\uc744 \uac00\uc9c0\ub294 \ub3d9\uc548\uc5d0 \uc54c\ub824\uc9c4 \uac83\ub9cc\uc73c\ub85c\ub3c4 \uac19\uc740 \uac00\ud574\uc790\ub85c\ubd80\ud130 \ub450 \ubc88\uc758 \uac00\uc815\ud3ed\ub825 \ud589\uc704\ub97c \ub2f9\ud588\uc5c8\uace0, \uccab \ubc88\uc9f8 \ud3ed\ub825 \ud589\uc704\ub294 \uc784\uc2e0 \uc911\uc5d0 \ub2f9\ud55c \uac83\uc774\uae30 \ub54c\ubb38\uc5d0 \ud0dc\uc544\ub3c4 \uadf8 \ud589\uc704\uc758 \ud53c\ud574\uc790\uc600\ub2e4. \uc774\ud63c \uacfc\uc815\uc5d0\uc11c \ucd5c\uc9c4\uc2e4\uc740 \ubbfc\uc0ac\uc0c1\u00b7\ud615\uc0ac\uc0c1 \ud53c\ud574\uc790\uc600\uc9c0\ub9cc, \uac1c\uc778\uc758 \uc544\ud508 \uc0ac\uc0dd\ud65c\uc5d0 \ub300\ud574\uc11c \uadf8\ub9ac\uace0 \uadf8 \ud53c\ud574\uc790\uc5d0 \ub300\ud574\uc11c \ud765\ubbf8 \uc704\uc8fc \uac00\uc2ed\uc73c\ub85c \ud314\uc544\uba39\ub294 \uc77c\ubd80 \uc5b8\ub860\uc758 '\uc0c1\ud63c' \ub4f1\uc73c\ub85c \uc774\uc911\uc73c\ub85c \uace0\ud1b5\uc744 \ubc1b\uc544\uc57c \ud588\ub2e4."} {"question": "\uccad\uacc4\ucc9c \uc8fc\ubcc0\uc9c0\uc5ed \uc0c1\uc778\uc744 \ub300\uc0c1\uc73c\ub85c \uc2e4\uc2dc\ud55c \uc124\ubb38\uc870\uc0ac\uc5d0\uc11c \ubcf5\uc6d0 \uc0ac\uc5c5\uc774 \uc798 \uc9c4\ud589\ub410\ub2e4\uace0 \ud3c9\uac00\ud55c \ube44\uc728\uc740?", "paragraph": "\uc5ec\ub7ec \uae30\uad00\uc11c \uccad\uacc4\ucc9c \ubcf5\uc6d0\uc5d0 \ub300\ud574 \uc124\ubb38\uc870\uc0ac\ub97c \ud558\uc600\ub294\ub370 \uc11c\uc6b8\ud559\uc5f0\uad6c\uc18c\uac00 \uc2dc\ubbfc\uacfc \uc804\ubb38\uac00\ub4e4\uc744 \ub300\uc0c1\uc73c\ub85c \uc2e4\uc2dc\ud55c \uc124\ubb38\uc870\uc0ac\uc5d0\uc11c\ub294 1995\ub144\ubd80\ud130 2010\ub144\uae4c\uc9c0 \uc11c\uc6b8 \ub3c4\uc2ec\uc5d0\uc11c \uc2dc\ud589\ub41c \uc8fc\uc694 20\uac1c \uc0ac\uc5c5 \uac00\uc6b4\ub370 '\uccad\uacc4\ucc9c \ubcf5\uc6d0'\uc774 \ub9cc\uc871\ub3c4\uc640 \uae30\uc5ec\ub3c4 \ud3c9\uac00\uc5d0\uc11c 5\uc810 \ub9cc\uc810\uc5d0 4.02\uc810\uc744 \ubc1b\uc544 1\uc704\uc5d0 \uc120\uc815\ub418\uc5c8\ub2e4. \uc11c\uc6b8\uc2dc\uc815\uac1c\ubc1c\uc5f0\uad6c\uc6d0\uc5d0\uc11c \uc2dc\ubbfc\uc744 \ub300\uc0c1\uc73c\ub85c \uc2e4\uc2dc\ud55c \uc124\ubb38\uc870\uc0ac\uc5d0\uc11c\ub294 \uccad\uacc4\ucc9c \ubcf5\uc6d0\uc758 \ucd5c\ub300 \uc131\uacfc\ub85c \uc0dd\ud0dc\ud658\uacbd\uc758 \ud68c\ubcf5\uc744 \uaf3d\uc558\uc73c\uba70 \uc11c\uc6b8\uc758 \ub79c\ub4dc\ub9c8\ud06c\ub97c \ubb3b\ub294 \uc9c8\ubb38\uc5d0 \uccad\uacc4\ucc9c\uc740 4\uc704\uc5d0 \uc62c\ub790\ub2e4. \ub610\ud55c \uc11c\uc6b8\uc2dc\uc640 \uc11c\uc6b8\uc2dc\uc815\uac1c\ubc1c\uc5f0\uad6c\uc6d0\uc774 \uccad\uacc4\ucc9c \uc8fc\ubcc0\uc9c0\uc5ed \uc0c1\uc778\uc744 \ub300\uc0c1\uc73c\ub85c \uc2e4\uc2dc\ud55c \uc124\ubb38\uc870\uc0ac\uc5d0\uc11c\ub294 \uccad\uacc4\ucc9c \ubcf5\uc6d0 \uc0ac\uc5c5\uc758 \uc9c4\ud589 \uc0c1\ud669\uc5d0 \ub300\ud574 `\uc798 \uc9c4\ud589\ub410\ub2e4'\ub294 \ud3c9\uac00\uac00 66.8%\uc778 \uac83\uc73c\ub85c \ub098\ud0c0\ub0ac\uc73c\uba70 \ubcf5\uc6d0 \uc0ac\uc5c5\uc758 \ud5a5\ud6c4 \uc804\ub9dd\uc5d0 \ub300\ud574\uc11c\ub3c4 `\uc798 \uc9c4\ud589\ub420 \uac83'\uc774\ub77c\ub294 \uc804\ub9dd\uc774 77.6%\uc5d0 \ub2ec\ud558\ub294 \uac83\uc73c\ub85c \uc870\uc0ac\ub418\uc5c8\ub2e4.", "answer": "66.8%", "sentence": "\uc11c\uc6b8\uc2dc\uc640 \uc11c\uc6b8\uc2dc\uc815\uac1c\ubc1c\uc5f0\uad6c\uc6d0\uc774 \uccad\uacc4\ucc9c \uc8fc\ubcc0\uc9c0\uc5ed \uc0c1\uc778\uc744 \ub300\uc0c1\uc73c\ub85c \uc2e4\uc2dc\ud55c \uc124\ubb38\uc870\uc0ac\uc5d0\uc11c\ub294 \uccad\uacc4\ucc9c \ubcf5\uc6d0 \uc0ac\uc5c5\uc758 \uc9c4\ud589 \uc0c1\ud669\uc5d0 \ub300\ud574 `\uc798 \uc9c4\ud589\ub410\ub2e4'\ub294 \ud3c9\uac00\uac00 66.8% \uc778 \uac83\uc73c\ub85c \ub098\ud0c0\ub0ac\uc73c\uba70 \ubcf5\uc6d0 \uc0ac\uc5c5\uc758 \ud5a5\ud6c4 \uc804\ub9dd\uc5d0 \ub300\ud574\uc11c\ub3c4 `\uc798 \uc9c4\ud589\ub420 \uac83'\uc774\ub77c\ub294 \uc804\ub9dd\uc774 77.6%\uc5d0 \ub2ec\ud558\ub294 \uac83\uc73c\ub85c \uc870\uc0ac\ub418\uc5c8\ub2e4.", "paragraph_sentence": "\uc5ec\ub7ec \uae30\uad00\uc11c \uccad\uacc4\ucc9c \ubcf5\uc6d0\uc5d0 \ub300\ud574 \uc124\ubb38\uc870\uc0ac\ub97c \ud558\uc600\ub294\ub370 \uc11c\uc6b8\ud559\uc5f0\uad6c\uc18c\uac00 \uc2dc\ubbfc\uacfc \uc804\ubb38\uac00\ub4e4\uc744 \ub300\uc0c1\uc73c\ub85c \uc2e4\uc2dc\ud55c \uc124\ubb38\uc870\uc0ac\uc5d0\uc11c\ub294 1995\ub144\ubd80\ud130 2010\ub144\uae4c\uc9c0 \uc11c\uc6b8 \ub3c4\uc2ec\uc5d0\uc11c \uc2dc\ud589\ub41c \uc8fc\uc694 20\uac1c \uc0ac\uc5c5 \uac00\uc6b4\ub370 '\uccad\uacc4\ucc9c \ubcf5\uc6d0'\uc774 \ub9cc\uc871\ub3c4\uc640 \uae30\uc5ec\ub3c4 \ud3c9\uac00\uc5d0\uc11c 5\uc810 \ub9cc\uc810\uc5d0 4.02\uc810\uc744 \ubc1b\uc544 1\uc704\uc5d0 \uc120\uc815\ub418\uc5c8\ub2e4. \uc11c\uc6b8\uc2dc\uc815\uac1c\ubc1c\uc5f0\uad6c\uc6d0\uc5d0\uc11c \uc2dc\ubbfc\uc744 \ub300\uc0c1\uc73c\ub85c \uc2e4\uc2dc\ud55c \uc124\ubb38\uc870\uc0ac\uc5d0\uc11c\ub294 \uccad\uacc4\ucc9c \ubcf5\uc6d0\uc758 \ucd5c\ub300 \uc131\uacfc\ub85c \uc0dd\ud0dc\ud658\uacbd\uc758 \ud68c\ubcf5\uc744 \uaf3d\uc558\uc73c\uba70 \uc11c\uc6b8\uc758 \ub79c\ub4dc\ub9c8\ud06c\ub97c \ubb3b\ub294 \uc9c8\ubb38\uc5d0 \uccad\uacc4\ucc9c\uc740 4\uc704\uc5d0 \uc62c\ub790\ub2e4. \ub610\ud55c \uc11c\uc6b8\uc2dc\uc640 \uc11c\uc6b8\uc2dc\uc815\uac1c\ubc1c\uc5f0\uad6c\uc6d0\uc774 \uccad\uacc4\ucc9c \uc8fc\ubcc0\uc9c0\uc5ed \uc0c1\uc778\uc744 \ub300\uc0c1\uc73c\ub85c \uc2e4\uc2dc\ud55c \uc124\ubb38\uc870\uc0ac\uc5d0\uc11c\ub294 \uccad\uacc4\ucc9c \ubcf5\uc6d0 \uc0ac\uc5c5\uc758 \uc9c4\ud589 \uc0c1\ud669\uc5d0 \ub300\ud574 `\uc798 \uc9c4\ud589\ub410\ub2e4'\ub294 \ud3c9\uac00\uac00 66.8% \uc778 \uac83\uc73c\ub85c \ub098\ud0c0\ub0ac\uc73c\uba70 \ubcf5\uc6d0 \uc0ac\uc5c5\uc758 \ud5a5\ud6c4 \uc804\ub9dd\uc5d0 \ub300\ud574\uc11c\ub3c4 `\uc798 \uc9c4\ud589\ub420 \uac83'\uc774\ub77c\ub294 \uc804\ub9dd\uc774 77.6%\uc5d0 \ub2ec\ud558\ub294 \uac83\uc73c\ub85c \uc870\uc0ac\ub418\uc5c8\ub2e4. ", "paragraph_answer": "\uc5ec\ub7ec \uae30\uad00\uc11c \uccad\uacc4\ucc9c \ubcf5\uc6d0\uc5d0 \ub300\ud574 \uc124\ubb38\uc870\uc0ac\ub97c \ud558\uc600\ub294\ub370 \uc11c\uc6b8\ud559\uc5f0\uad6c\uc18c\uac00 \uc2dc\ubbfc\uacfc \uc804\ubb38\uac00\ub4e4\uc744 \ub300\uc0c1\uc73c\ub85c \uc2e4\uc2dc\ud55c \uc124\ubb38\uc870\uc0ac\uc5d0\uc11c\ub294 1995\ub144\ubd80\ud130 2010\ub144\uae4c\uc9c0 \uc11c\uc6b8 \ub3c4\uc2ec\uc5d0\uc11c \uc2dc\ud589\ub41c \uc8fc\uc694 20\uac1c \uc0ac\uc5c5 \uac00\uc6b4\ub370 '\uccad\uacc4\ucc9c \ubcf5\uc6d0'\uc774 \ub9cc\uc871\ub3c4\uc640 \uae30\uc5ec\ub3c4 \ud3c9\uac00\uc5d0\uc11c 5\uc810 \ub9cc\uc810\uc5d0 4.02\uc810\uc744 \ubc1b\uc544 1\uc704\uc5d0 \uc120\uc815\ub418\uc5c8\ub2e4. \uc11c\uc6b8\uc2dc\uc815\uac1c\ubc1c\uc5f0\uad6c\uc6d0\uc5d0\uc11c \uc2dc\ubbfc\uc744 \ub300\uc0c1\uc73c\ub85c \uc2e4\uc2dc\ud55c \uc124\ubb38\uc870\uc0ac\uc5d0\uc11c\ub294 \uccad\uacc4\ucc9c \ubcf5\uc6d0\uc758 \ucd5c\ub300 \uc131\uacfc\ub85c \uc0dd\ud0dc\ud658\uacbd\uc758 \ud68c\ubcf5\uc744 \uaf3d\uc558\uc73c\uba70 \uc11c\uc6b8\uc758 \ub79c\ub4dc\ub9c8\ud06c\ub97c \ubb3b\ub294 \uc9c8\ubb38\uc5d0 \uccad\uacc4\ucc9c\uc740 4\uc704\uc5d0 \uc62c\ub790\ub2e4. \ub610\ud55c \uc11c\uc6b8\uc2dc\uc640 \uc11c\uc6b8\uc2dc\uc815\uac1c\ubc1c\uc5f0\uad6c\uc6d0\uc774 \uccad\uacc4\ucc9c \uc8fc\ubcc0\uc9c0\uc5ed \uc0c1\uc778\uc744 \ub300\uc0c1\uc73c\ub85c \uc2e4\uc2dc\ud55c \uc124\ubb38\uc870\uc0ac\uc5d0\uc11c\ub294 \uccad\uacc4\ucc9c \ubcf5\uc6d0 \uc0ac\uc5c5\uc758 \uc9c4\ud589 \uc0c1\ud669\uc5d0 \ub300\ud574 `\uc798 \uc9c4\ud589\ub410\ub2e4'\ub294 \ud3c9\uac00\uac00 66.8% \uc778 \uac83\uc73c\ub85c \ub098\ud0c0\ub0ac\uc73c\uba70 \ubcf5\uc6d0 \uc0ac\uc5c5\uc758 \ud5a5\ud6c4 \uc804\ub9dd\uc5d0 \ub300\ud574\uc11c\ub3c4 `\uc798 \uc9c4\ud589\ub420 \uac83'\uc774\ub77c\ub294 \uc804\ub9dd\uc774 77.6%\uc5d0 \ub2ec\ud558\ub294 \uac83\uc73c\ub85c \uc870\uc0ac\ub418\uc5c8\ub2e4.", "sentence_answer": "\uc11c\uc6b8\uc2dc\uc640 \uc11c\uc6b8\uc2dc\uc815\uac1c\ubc1c\uc5f0\uad6c\uc6d0\uc774 \uccad\uacc4\ucc9c \uc8fc\ubcc0\uc9c0\uc5ed \uc0c1\uc778\uc744 \ub300\uc0c1\uc73c\ub85c \uc2e4\uc2dc\ud55c \uc124\ubb38\uc870\uc0ac\uc5d0\uc11c\ub294 \uccad\uacc4\ucc9c \ubcf5\uc6d0 \uc0ac\uc5c5\uc758 \uc9c4\ud589 \uc0c1\ud669\uc5d0 \ub300\ud574 `\uc798 \uc9c4\ud589\ub410\ub2e4'\ub294 \ud3c9\uac00\uac00 66.8% \uc778 \uac83\uc73c\ub85c \ub098\ud0c0\ub0ac\uc73c\uba70 \ubcf5\uc6d0 \uc0ac\uc5c5\uc758 \ud5a5\ud6c4 \uc804\ub9dd\uc5d0 \ub300\ud574\uc11c\ub3c4 `\uc798 \uc9c4\ud589\ub420 \uac83'\uc774\ub77c\ub294 \uc804\ub9dd\uc774 77.6%\uc5d0 \ub2ec\ud558\ub294 \uac83\uc73c\ub85c \uc870\uc0ac\ub418\uc5c8\ub2e4.", "paragraph_id": "6559272-1-2", "paragraph_question": "question: \uccad\uacc4\ucc9c \uc8fc\ubcc0\uc9c0\uc5ed \uc0c1\uc778\uc744 \ub300\uc0c1\uc73c\ub85c \uc2e4\uc2dc\ud55c \uc124\ubb38\uc870\uc0ac\uc5d0\uc11c \ubcf5\uc6d0 \uc0ac\uc5c5\uc774 \uc798 \uc9c4\ud589\ub410\ub2e4\uace0 \ud3c9\uac00\ud55c \ube44\uc728\uc740?, context: \uc5ec\ub7ec \uae30\uad00\uc11c \uccad\uacc4\ucc9c \ubcf5\uc6d0\uc5d0 \ub300\ud574 \uc124\ubb38\uc870\uc0ac\ub97c \ud558\uc600\ub294\ub370 \uc11c\uc6b8\ud559\uc5f0\uad6c\uc18c\uac00 \uc2dc\ubbfc\uacfc \uc804\ubb38\uac00\ub4e4\uc744 \ub300\uc0c1\uc73c\ub85c \uc2e4\uc2dc\ud55c \uc124\ubb38\uc870\uc0ac\uc5d0\uc11c\ub294 1995\ub144\ubd80\ud130 2010\ub144\uae4c\uc9c0 \uc11c\uc6b8 \ub3c4\uc2ec\uc5d0\uc11c \uc2dc\ud589\ub41c \uc8fc\uc694 20\uac1c \uc0ac\uc5c5 \uac00\uc6b4\ub370 '\uccad\uacc4\ucc9c \ubcf5\uc6d0'\uc774 \ub9cc\uc871\ub3c4\uc640 \uae30\uc5ec\ub3c4 \ud3c9\uac00\uc5d0\uc11c 5\uc810 \ub9cc\uc810\uc5d0 4.02\uc810\uc744 \ubc1b\uc544 1\uc704\uc5d0 \uc120\uc815\ub418\uc5c8\ub2e4. \uc11c\uc6b8\uc2dc\uc815\uac1c\ubc1c\uc5f0\uad6c\uc6d0\uc5d0\uc11c \uc2dc\ubbfc\uc744 \ub300\uc0c1\uc73c\ub85c \uc2e4\uc2dc\ud55c \uc124\ubb38\uc870\uc0ac\uc5d0\uc11c\ub294 \uccad\uacc4\ucc9c \ubcf5\uc6d0\uc758 \ucd5c\ub300 \uc131\uacfc\ub85c \uc0dd\ud0dc\ud658\uacbd\uc758 \ud68c\ubcf5\uc744 \uaf3d\uc558\uc73c\uba70 \uc11c\uc6b8\uc758 \ub79c\ub4dc\ub9c8\ud06c\ub97c \ubb3b\ub294 \uc9c8\ubb38\uc5d0 \uccad\uacc4\ucc9c\uc740 4\uc704\uc5d0 \uc62c\ub790\ub2e4. \ub610\ud55c \uc11c\uc6b8\uc2dc\uc640 \uc11c\uc6b8\uc2dc\uc815\uac1c\ubc1c\uc5f0\uad6c\uc6d0\uc774 \uccad\uacc4\ucc9c \uc8fc\ubcc0\uc9c0\uc5ed \uc0c1\uc778\uc744 \ub300\uc0c1\uc73c\ub85c \uc2e4\uc2dc\ud55c \uc124\ubb38\uc870\uc0ac\uc5d0\uc11c\ub294 \uccad\uacc4\ucc9c \ubcf5\uc6d0 \uc0ac\uc5c5\uc758 \uc9c4\ud589 \uc0c1\ud669\uc5d0 \ub300\ud574 `\uc798 \uc9c4\ud589\ub410\ub2e4'\ub294 \ud3c9\uac00\uac00 66.8%\uc778 \uac83\uc73c\ub85c \ub098\ud0c0\ub0ac\uc73c\uba70 \ubcf5\uc6d0 \uc0ac\uc5c5\uc758 \ud5a5\ud6c4 \uc804\ub9dd\uc5d0 \ub300\ud574\uc11c\ub3c4 `\uc798 \uc9c4\ud589\ub420 \uac83'\uc774\ub77c\ub294 \uc804\ub9dd\uc774 77.6%\uc5d0 \ub2ec\ud558\ub294 \uac83\uc73c\ub85c \uc870\uc0ac\ub418\uc5c8\ub2e4."} {"question": "\ubc84\uceec\ub9ac \uc5f0\uad6c\uc18c\ub294 \ucd08\uc2e0\uc131\uc758 \ud3ed\ubc1c\ub85c \ubb34\uc5c7\uc774 \uba78\uc885\ud588\uc744 \uac83\uc73c\ub85c \uc8fc\uc7a5\ud588\ub098?", "paragraph": "\ucd08\uc2e0\uc131 \ud3ed\ubc1c \uc2dc \ubc29\ucd9c\ub418\ub294 \uac15\ub825\ud55c \uc6b0\uc8fc\uc120\uc5d0 \uc758\ud55c \uba78\uc885\uc5d0 \ub354\ud558\uc5ec \ucd08\uc2e0\uc131\uc758 \ud3ed\ubc1c \uc794\ud574\uac00 \uc9c0\uad6c\uc5d0 \ucda9\ub3cc\ud558\uc5ec \uc0dd\ubb3c\uc758 \uba78\uc885\uc73c\ub85c \uc774\uc5b4\uc9c8 \uc218\ub3c4 \uc788\ub2e4\ub294 \uc8fc\uc7a5\ub3c4 \ub098\uc654\ub2e4. 2005\ub144 \ubc84\uceec\ub9ac\uc5f0\uad6c\uc18c(U.S. Department of Energy\u2019s Lawrence Berkeley National Laboratory)\uc758 \ud575\ubb3c\ub9ac\ud559\uc790 \ud30c\uc774\uc5b4\uc2a4\ud1a4(Richard Firestone)\uacfc \uc560\ub9ac\uc870\ub098 \uc9c0\uc9c8\ud559\uc790 \uc6e8\uc2a4\ud2b8(Allen West)\ub294 \ucd08\uc2e0\uc131 \ud3ed\ubc1c\ub85c \uc778\ud574 \ub9e4\uba38\ub4dc\uac00 \uba78\uc885\ud588\uc744 \uc218 \uc788\ub2e4\uace0 \uc8fc\uc7a5\ud588\ub2e4. \uadf8\ub4e4\uc758 \uc8fc\uc7a5\uc5d0 \uc758\ud558\uba74 \uc9c0\uae08\ubd80\ud130 41,000\ub144 \uc804, \uc9c0\uad6c\ub85c\ubd80\ud130 \uc57d 250\uad11\ub144 \ub5a8\uc5b4\uc9c4 \ucd08\uc2e0\uc131\uc774 \ud3ed\ubc1c\ud588\ub2e4. \uc774\ub54c \ubc1c\uc0dd\ud55c \ucd08\uae30 \ucda9\uaca9\ud30c\uac00 \ucd08\uc18d 10,000km\uc758 \uc18d\ub3c4\ub85c 7,000\ub144 \ub3d9\uc548 \uc6b0\uc8fc\uacf5\uac04\uc744 \uac00\ub85c\uc9c8\ub7ec \uc640\uc11c \uc9c0\uad6c\uc758 \ub9e4\uba38\ub4dc\ub4e4\uc744 \uc8fd\uc600\uc73c\uba70, \uc774\uc5b4 \uadf8\ub85c\ubd80\ud130 21,000\ub144 \ud6c4\uc778 13,000\ub144 \uc804\uc5d0\ub294 \ucd08\uc2e0\uc131 \ud3ed\ubc1c\uc758 \uc794\ud574\ub85c \uc774\ub8e8\uc5b4\uc9c4 \uc9c1\uacbd 10km \uc815\ub3c4\uc758 \ud61c\uc131\uacfc \uac19\uc740 \uc800\ubc00\ub3c4 \ubb3c\uccb4\uac00 \uc9c0\uad6c\uc5d0 \ucda9\ub3cc\ud558\uc5ec \ubd81\ubbf8\uc8fc \ub300\ub959\uc5d0 \uc11c\uc2dd\ud558\ub358 \ub9e4\uba38\ub4dc\ub4e4\uc744 \ubaa8\ub450 \uba78\uc885\uc2dc\ucf30\ub2e4\uace0 \ud55c\ub2e4.", "answer": "\ub9e4\uba38\ub4dc", "sentence": "2005\ub144 \ubc84\uceec\ub9ac\uc5f0\uad6c\uc18c(U.S. Department of Energy\u2019s Lawrence Berkeley National Laboratory)\uc758 \ud575\ubb3c\ub9ac\ud559\uc790 \ud30c\uc774\uc5b4\uc2a4\ud1a4(Richard Firestone)\uacfc \uc560\ub9ac\uc870\ub098 \uc9c0\uc9c8\ud559\uc790 \uc6e8\uc2a4\ud2b8(Allen West)\ub294 \ucd08\uc2e0\uc131 \ud3ed\ubc1c\ub85c \uc778\ud574 \ub9e4\uba38\ub4dc \uac00 \uba78\uc885\ud588\uc744 \uc218 \uc788\ub2e4\uace0 \uc8fc\uc7a5\ud588\ub2e4.", "paragraph_sentence": "\ucd08\uc2e0\uc131 \ud3ed\ubc1c \uc2dc \ubc29\ucd9c\ub418\ub294 \uac15\ub825\ud55c \uc6b0\uc8fc\uc120\uc5d0 \uc758\ud55c \uba78\uc885\uc5d0 \ub354\ud558\uc5ec \ucd08\uc2e0\uc131\uc758 \ud3ed\ubc1c \uc794\ud574\uac00 \uc9c0\uad6c\uc5d0 \ucda9\ub3cc\ud558\uc5ec \uc0dd\ubb3c\uc758 \uba78\uc885\uc73c\ub85c \uc774\uc5b4\uc9c8 \uc218\ub3c4 \uc788\ub2e4\ub294 \uc8fc\uc7a5\ub3c4 \ub098\uc654\ub2e4. 2005\ub144 \ubc84\uceec\ub9ac\uc5f0\uad6c\uc18c(U.S. Department of Energy\u2019s Lawrence Berkeley National Laboratory)\uc758 \ud575\ubb3c\ub9ac\ud559\uc790 \ud30c\uc774\uc5b4\uc2a4\ud1a4(Richard Firestone)\uacfc \uc560\ub9ac\uc870\ub098 \uc9c0\uc9c8\ud559\uc790 \uc6e8\uc2a4\ud2b8(Allen West)\ub294 \ucd08\uc2e0\uc131 \ud3ed\ubc1c\ub85c \uc778\ud574 \ub9e4\uba38\ub4dc \uac00 \uba78\uc885\ud588\uc744 \uc218 \uc788\ub2e4\uace0 \uc8fc\uc7a5\ud588\ub2e4. \uadf8\ub4e4\uc758 \uc8fc\uc7a5\uc5d0 \uc758\ud558\uba74 \uc9c0\uae08\ubd80\ud130 41,000\ub144 \uc804, \uc9c0\uad6c\ub85c\ubd80\ud130 \uc57d 250\uad11\ub144 \ub5a8\uc5b4\uc9c4 \ucd08\uc2e0\uc131\uc774 \ud3ed\ubc1c\ud588\ub2e4. \uc774\ub54c \ubc1c\uc0dd\ud55c \ucd08\uae30 \ucda9\uaca9\ud30c\uac00 \ucd08\uc18d 10,000km\uc758 \uc18d\ub3c4\ub85c 7,000\ub144 \ub3d9\uc548 \uc6b0\uc8fc\uacf5\uac04\uc744 \uac00\ub85c\uc9c8\ub7ec \uc640\uc11c \uc9c0\uad6c\uc758 \ub9e4\uba38\ub4dc\ub4e4\uc744 \uc8fd\uc600\uc73c\uba70, \uc774\uc5b4 \uadf8\ub85c\ubd80\ud130 21,000\ub144 \ud6c4\uc778 13,000\ub144 \uc804\uc5d0\ub294 \ucd08\uc2e0\uc131 \ud3ed\ubc1c\uc758 \uc794\ud574\ub85c \uc774\ub8e8\uc5b4\uc9c4 \uc9c1\uacbd 10km \uc815\ub3c4\uc758 \ud61c\uc131\uacfc \uac19\uc740 \uc800\ubc00\ub3c4 \ubb3c\uccb4\uac00 \uc9c0\uad6c\uc5d0 \ucda9\ub3cc\ud558\uc5ec \ubd81\ubbf8\uc8fc \ub300\ub959\uc5d0 \uc11c\uc2dd\ud558\ub358 \ub9e4\uba38\ub4dc\ub4e4\uc744 \ubaa8\ub450 \uba78\uc885\uc2dc\ucf30\ub2e4\uace0 \ud55c\ub2e4.", "paragraph_answer": "\ucd08\uc2e0\uc131 \ud3ed\ubc1c \uc2dc \ubc29\ucd9c\ub418\ub294 \uac15\ub825\ud55c \uc6b0\uc8fc\uc120\uc5d0 \uc758\ud55c \uba78\uc885\uc5d0 \ub354\ud558\uc5ec \ucd08\uc2e0\uc131\uc758 \ud3ed\ubc1c \uc794\ud574\uac00 \uc9c0\uad6c\uc5d0 \ucda9\ub3cc\ud558\uc5ec \uc0dd\ubb3c\uc758 \uba78\uc885\uc73c\ub85c \uc774\uc5b4\uc9c8 \uc218\ub3c4 \uc788\ub2e4\ub294 \uc8fc\uc7a5\ub3c4 \ub098\uc654\ub2e4. 2005\ub144 \ubc84\uceec\ub9ac\uc5f0\uad6c\uc18c(U.S. Department of Energy\u2019s Lawrence Berkeley National Laboratory)\uc758 \ud575\ubb3c\ub9ac\ud559\uc790 \ud30c\uc774\uc5b4\uc2a4\ud1a4(Richard Firestone)\uacfc \uc560\ub9ac\uc870\ub098 \uc9c0\uc9c8\ud559\uc790 \uc6e8\uc2a4\ud2b8(Allen West)\ub294 \ucd08\uc2e0\uc131 \ud3ed\ubc1c\ub85c \uc778\ud574 \ub9e4\uba38\ub4dc \uac00 \uba78\uc885\ud588\uc744 \uc218 \uc788\ub2e4\uace0 \uc8fc\uc7a5\ud588\ub2e4. \uadf8\ub4e4\uc758 \uc8fc\uc7a5\uc5d0 \uc758\ud558\uba74 \uc9c0\uae08\ubd80\ud130 41,000\ub144 \uc804, \uc9c0\uad6c\ub85c\ubd80\ud130 \uc57d 250\uad11\ub144 \ub5a8\uc5b4\uc9c4 \ucd08\uc2e0\uc131\uc774 \ud3ed\ubc1c\ud588\ub2e4. \uc774\ub54c \ubc1c\uc0dd\ud55c \ucd08\uae30 \ucda9\uaca9\ud30c\uac00 \ucd08\uc18d 10,000km\uc758 \uc18d\ub3c4\ub85c 7,000\ub144 \ub3d9\uc548 \uc6b0\uc8fc\uacf5\uac04\uc744 \uac00\ub85c\uc9c8\ub7ec \uc640\uc11c \uc9c0\uad6c\uc758 \ub9e4\uba38\ub4dc\ub4e4\uc744 \uc8fd\uc600\uc73c\uba70, \uc774\uc5b4 \uadf8\ub85c\ubd80\ud130 21,000\ub144 \ud6c4\uc778 13,000\ub144 \uc804\uc5d0\ub294 \ucd08\uc2e0\uc131 \ud3ed\ubc1c\uc758 \uc794\ud574\ub85c \uc774\ub8e8\uc5b4\uc9c4 \uc9c1\uacbd 10km \uc815\ub3c4\uc758 \ud61c\uc131\uacfc \uac19\uc740 \uc800\ubc00\ub3c4 \ubb3c\uccb4\uac00 \uc9c0\uad6c\uc5d0 \ucda9\ub3cc\ud558\uc5ec \ubd81\ubbf8\uc8fc \ub300\ub959\uc5d0 \uc11c\uc2dd\ud558\ub358 \ub9e4\uba38\ub4dc\ub4e4\uc744 \ubaa8\ub450 \uba78\uc885\uc2dc\ucf30\ub2e4\uace0 \ud55c\ub2e4.", "sentence_answer": "2005\ub144 \ubc84\uceec\ub9ac\uc5f0\uad6c\uc18c(U.S. Department of Energy\u2019s Lawrence Berkeley National Laboratory)\uc758 \ud575\ubb3c\ub9ac\ud559\uc790 \ud30c\uc774\uc5b4\uc2a4\ud1a4(Richard Firestone)\uacfc \uc560\ub9ac\uc870\ub098 \uc9c0\uc9c8\ud559\uc790 \uc6e8\uc2a4\ud2b8(Allen West)\ub294 \ucd08\uc2e0\uc131 \ud3ed\ubc1c\ub85c \uc778\ud574 \ub9e4\uba38\ub4dc \uac00 \uba78\uc885\ud588\uc744 \uc218 \uc788\ub2e4\uace0 \uc8fc\uc7a5\ud588\ub2e4.", "paragraph_id": "6472464-6-0", "paragraph_question": "question: \ubc84\uceec\ub9ac \uc5f0\uad6c\uc18c\ub294 \ucd08\uc2e0\uc131\uc758 \ud3ed\ubc1c\ub85c \ubb34\uc5c7\uc774 \uba78\uc885\ud588\uc744 \uac83\uc73c\ub85c \uc8fc\uc7a5\ud588\ub098?, context: \ucd08\uc2e0\uc131 \ud3ed\ubc1c \uc2dc \ubc29\ucd9c\ub418\ub294 \uac15\ub825\ud55c \uc6b0\uc8fc\uc120\uc5d0 \uc758\ud55c \uba78\uc885\uc5d0 \ub354\ud558\uc5ec \ucd08\uc2e0\uc131\uc758 \ud3ed\ubc1c \uc794\ud574\uac00 \uc9c0\uad6c\uc5d0 \ucda9\ub3cc\ud558\uc5ec \uc0dd\ubb3c\uc758 \uba78\uc885\uc73c\ub85c \uc774\uc5b4\uc9c8 \uc218\ub3c4 \uc788\ub2e4\ub294 \uc8fc\uc7a5\ub3c4 \ub098\uc654\ub2e4. 2005\ub144 \ubc84\uceec\ub9ac\uc5f0\uad6c\uc18c(U.S. Department of Energy\u2019s Lawrence Berkeley National Laboratory)\uc758 \ud575\ubb3c\ub9ac\ud559\uc790 \ud30c\uc774\uc5b4\uc2a4\ud1a4(Richard Firestone)\uacfc \uc560\ub9ac\uc870\ub098 \uc9c0\uc9c8\ud559\uc790 \uc6e8\uc2a4\ud2b8(Allen West)\ub294 \ucd08\uc2e0\uc131 \ud3ed\ubc1c\ub85c \uc778\ud574 \ub9e4\uba38\ub4dc\uac00 \uba78\uc885\ud588\uc744 \uc218 \uc788\ub2e4\uace0 \uc8fc\uc7a5\ud588\ub2e4. \uadf8\ub4e4\uc758 \uc8fc\uc7a5\uc5d0 \uc758\ud558\uba74 \uc9c0\uae08\ubd80\ud130 41,000\ub144 \uc804, \uc9c0\uad6c\ub85c\ubd80\ud130 \uc57d 250\uad11\ub144 \ub5a8\uc5b4\uc9c4 \ucd08\uc2e0\uc131\uc774 \ud3ed\ubc1c\ud588\ub2e4. \uc774\ub54c \ubc1c\uc0dd\ud55c \ucd08\uae30 \ucda9\uaca9\ud30c\uac00 \ucd08\uc18d 10,000km\uc758 \uc18d\ub3c4\ub85c 7,000\ub144 \ub3d9\uc548 \uc6b0\uc8fc\uacf5\uac04\uc744 \uac00\ub85c\uc9c8\ub7ec \uc640\uc11c \uc9c0\uad6c\uc758 \ub9e4\uba38\ub4dc\ub4e4\uc744 \uc8fd\uc600\uc73c\uba70, \uc774\uc5b4 \uadf8\ub85c\ubd80\ud130 21,000\ub144 \ud6c4\uc778 13,000\ub144 \uc804\uc5d0\ub294 \ucd08\uc2e0\uc131 \ud3ed\ubc1c\uc758 \uc794\ud574\ub85c \uc774\ub8e8\uc5b4\uc9c4 \uc9c1\uacbd 10km \uc815\ub3c4\uc758 \ud61c\uc131\uacfc \uac19\uc740 \uc800\ubc00\ub3c4 \ubb3c\uccb4\uac00 \uc9c0\uad6c\uc5d0 \ucda9\ub3cc\ud558\uc5ec \ubd81\ubbf8\uc8fc \ub300\ub959\uc5d0 \uc11c\uc2dd\ud558\ub358 \ub9e4\uba38\ub4dc\ub4e4\uc744 \ubaa8\ub450 \uba78\uc885\uc2dc\ucf30\ub2e4\uace0 \ud55c\ub2e4."} {"question": "\uad6d\ud68c\uc758\uc0ac\ub2f9\uc774 \ud3ed\ud30c \ub420 \ub54c \ubd88\uaf43\ub180\uc774\uc758 \ubaa8\uc591\uc740 \ubb34\uc5c7\uc778\uac00?", "paragraph": "\uc601\ud654\ub3c4 \ub9cc\ud654\ucc45\uacfc \ube44\uc2b7\ud558\uac8c \uae00\uc790 V\uc640 \uc22b\uc790 5\uac00 \ubc18\ubcf5\ub418\ub294 \ud14c\ub9c8\ub85c \uc774\uc6a9\ub418\uc5c8\ub2e4. \uc608\ub97c \ub4e4\uc5b4, \ube0c\uc774\uac00 \uc774\ube44\ub97c \ucc98\uc74c \ub9cc\ub0ac\uc744\ub54c \ub298\uc5b4\ub193\uc740 \uc18c\uac1c\ub9d0\uc5d0\uc11c\ub294 V\ub85c \uc2dc\uc791\ud558\ub294 50\uac1c\uc758 \ub2e8\uc5b4\uac00 \ub450\uc6b4\uc744 \ub9de\ucdb0 \uc5f4\uac70\ub418\uc5b4 \uc788\uace0, \ubb38\uc790 V \uc790\uccb4\ub85c\ub294 54\uac1c\uac00 \ub098\uc628\ub2e4. \uc774\ube44\uac00 \ube0c\uc774\uc5d0\uac8c \uc790\uae30 \uc774\ub984\uc744 \ub9d0\ud558\uc790 \ube0c\uc774\ub294 \u201c\uc774\u2026 \ube0c\uc774\u201d(\"E... V\")\ub77c\uace0 \ub290\ub9bf\ub290\ub9bf \ubc18\ubcf5\ud558\ub294\ub370, E\ub294 \uc54c\ud30c\ubcb3\uc758 \ub2e4\uc12f \ubc88\uc9f8 \ubb38\uc790\uc774\ub2e4. \ub77c\ud06c\ud790\uc5d0 \uad6c\ub958\ub418\uc5b4 \uc788\ub41c \ub3d9\uc548, \ube0c\uc774\ub294 V\ubc88, \uc989 \ub85c\ub9c8 \uc22b\uc790\uc758 5\ubc88 \ubc29\uc5d0 \uac07\ud600 \uc788\uc5c8\uace0, \uc774\ube44\uac00 \uac70\uc9d3 \ud22c\uc625\uc744 \ub2f9\ud588\uc744 \ub54c\ub3c4 5\ubc88 \ubc29\uc774\uc5c8\ub2e4. \ube0c\uc774\uc758 \uc11c\uba85\uc740 \uc870\ub85c\uac00 \uce7c\uc790\uad6d\uc73c\ub85c Z\ub97c \ub0a8\uae30\ub294 \uac83\ucc98\ub7fc \uce7c\ub85c V\uc790\ub97c \uadf8\ub9ac\ub294 \uac83\uc774\ub2e4. \uc911\uc559 \ud615\uc0ac \uc7ac\ud310\uc18c\uac00 \ud3ed\ud30c\ub420 \ub54c \ubd88\uaf43\ub180\uc774\uc758 \ubd88\uaf43\uc774 \ubd89\uc740\uc0c9\uc758 V \ubaa8\uc591\uc744 \ub9cc\ub4e0\ub2e4. \ub465\uadfc \ubd88\uaf43\uc774 \ub354\ud574\uc9c0\uba74 \ubd88\uaf43\uc758 \ubaa8\uc591\uc740 V \ubaa8\uc591 \ubfd0 \uc544\ub2c8\ub77c \ube0c\uc774 \ud3ec \ubca4\ub370\ud0c0 \ub85c\uace0\uc640\ub3c4 \ub2ee\uc558\ub2e4. \ube0c\uc774\uc758 \uc88c\uc6b0\uba85\uc778 \u201c\uc9c4\uc2e4\uc758 \ud798\uc73c\ub85c \uc0b4\uba74\uc11c \uc6b0\uc8fc\ub97c \uc815\ubcf5\ud588\ub2e4\u201d\ub294 \uc6d0\ub798 \ub77c\ud2f4\uc5b4 \u201c\ube44 \ubca0\ub9ac \ubca0\ub2c8\ubca0\ub974\uc228 \ube44\ubd80\uc2a4 \ube44\ud0a4\u201d(\"Vi Veri Veniversum Vivus Vici\")\ub85c, 5\uac1c\uc758 V\uac00 \ub4e4\uc5b4\uac00 \uc788\ub2e4. \uc774\ube44\uc640 \ucda4 \ucd9c \ub54c \ube0c\uc774\uac00 \uace0\ub974\ub294 \ub178\ub798\ub294 \uc8fc\ud06c\ubc15\uc2a4\uc758 5\ubc88 \ub178\ub798\uc774\ub2e4. \uc0ac\uc2e4 \ubaa8\ub4e0 \ub178\ub798\ub4e4\uc758 \ub118\ubc84\uac00 5\ubc88\uc774\ub2e4. \ud06c\ub9ac\ub514\uc758 \uc9d1\uc5d0\uc11c \ud06c\ub9ac\ub514\uc640 \ub300\uacb0\ud560 \ub54c \ube0c\uc774\uac00 \ud2c0\uc5b4\ub193\uc740 \uc74c\uc545\uc740 \u300a\uc6b4\uba85 \uad50\ud5a5\uace1\u300b, \uc989 \ubca0\ud1a0\ubca4 \u30085\ubc88\u3009 \uad50\ud5a5\uace1\uc73c\ub85c, \uc774 \ub178\ub798\uc758 \ucd08\ubc18\ubd80\uc758 \ub9ac\ub4ec \ud328\ud134\uc740 \ubaa8\uc2a4 \ubd80\ud638\uc758 V(\u00b7\u00b7\u00b7\u2013)\uc640 \ub2ee\uc558\ub2e4. \uc601\ud654\uc758 \uc81c\ubaa9 \uc790\uccb4\ub3c4 \ube0c\uc774 \uc0ac\uc778\uc744 \uc5f0\uc0c1\ud558\uac8c \ud55c\ub2e4. \ube0c\uc774\ub294 \ubd95\uad34\uc758 \ubc24\uc744 \uae30\ub2e4\ub9ac\uba74\uc11c \uac80\uc740\uc0c9\uacfc \ubd89\uc740\uc0c9\uc758 \ub3c4\ubbf8\ub178\ub85c \ubcf5\uc7a1\ud55c \ud328\ud134\uc744 \ub9cc\ub4e4\uace0, \ub3c4\ubbf8\ub178\uac00 \ub2e4 \uc4f0\ub7ec\uc9c0\uace0 \ub098\uc790 V \ub85c\uace0 \ubaa8\uc591\uc774 \ub098\ud0c0\ub09c\ub2e4. \ud06c\ub9ac\ub514\uc640 \uadf8 \ubd80\ud558\ub4e4\uacfc \ubc8c\uc778 \ucd5c\ud6c4 \uacb0\uc804\uc740 \ub7f0\ub358 \uc9c0\ud558\ucca0 \ube45\ud1a0\ub9ac\uc544\uc5ed \uc5d0\uc11c \ubc8c\uc5b4\uc84c\uace0, \uc774\ub54c \ube0c\uc774\ub294 6\uac1c\uc758 \ub2e8\uac80 \uc911 5\uac1c\ub97c \uc0ac\uc6a9\ud558\uba70, \ub2e8\uac80\uc744 \ub358\uc9c0\uae30 \uc9c1\uc804 \ub2e8\uac80\uc73c\ub85c V \ubaa8\uc591\uc744 \ub9cc\ub4e0\ub2e4. \ube0c\uc774\uac00 \ubd80\ud558\ub4e4\uc744 \ub2e4 \uc8fd\uc774\uace0 \ub098\uc11c, \ud06c\ub9ac\ub514\ub294 \ube0c\uc774\uc5d0\uac8c \ub9ac\ubcfc\ubc84\ub97c \ub2e4\uc12f \ubc1c \uc3dc\ub2e4. \uc2f8\uc6c0\uc774 \ub05d\ub09c \ub4a4, \uce58\uba85\uc0c1\uc744 \uc785\uc740 \ube0c\uc774\ub294 \uc5ed\uc744 \ub5a0\ub098\uba74\uc11c \ubcbd\uc5d0 \ubd80\ub51b\ud788\ub294\ub370, \uc774\ub54c V \ubaa8\uc591\uc758 \ud54f\uc790\uad6d\uc774 \uc0dd\uae34\ub2e4. \ube0c\uc774\uc640 \ud06c\ub9ac\ub514\uc758 \uc2f8\uc6c0\uc774 \ub05d\ub098\uc790 \ube45\ubca4\uc774 11\uc2dc 5\ubd84\uc744 \uac00\ub9ac\ud0a4\ub294\ub370, \uc774\ub54c \uc2dc\uce68\uacfc \ubd84\uce68\uc758 \ubaa8\uc591\uc774 V \ubaa8\uc591\uc774\ub2e4(11\uc6d4 5\uc77c\uc744 \uc5f0\uc0c1\ud558\uac8c \ud55c\ub2e4). \uad6d\ud68c\uc758\uc0ac\ub2f9\uc774 \ud3ed\ud30c\ub420 \ub54c\uc758 \ubd88\uaf43\ub180\uc774\ub3c4 V \ubaa8\uc591\uc774\uc790 \ubb34\uc815\ubd80\uc8fc\uc758\uc758 \uc0c1\uc9d5\uc778 Circle-A\uac00 \ub4a4\uc9d1\ud78c \ubaa8\uc591\uc744 \ub9cc\ub4e0\ub2e4.", "answer": "V \ubaa8\uc591", "sentence": "\uc911\uc559 \ud615\uc0ac \uc7ac\ud310\uc18c\uac00 \ud3ed\ud30c\ub420 \ub54c \ubd88\uaf43\ub180\uc774\uc758 \ubd88\uaf43\uc774 \ubd89\uc740\uc0c9\uc758 V \ubaa8\uc591 \uc744 \ub9cc\ub4e0\ub2e4.", "paragraph_sentence": "\uc601\ud654\ub3c4 \ub9cc\ud654\ucc45\uacfc \ube44\uc2b7\ud558\uac8c \uae00\uc790 V\uc640 \uc22b\uc790 5\uac00 \ubc18\ubcf5\ub418\ub294 \ud14c\ub9c8\ub85c \uc774\uc6a9\ub418\uc5c8\ub2e4. \uc608\ub97c \ub4e4\uc5b4, \ube0c\uc774\uac00 \uc774\ube44\ub97c \ucc98\uc74c \ub9cc\ub0ac\uc744\ub54c \ub298\uc5b4\ub193\uc740 \uc18c\uac1c\ub9d0\uc5d0\uc11c\ub294 V\ub85c \uc2dc\uc791\ud558\ub294 50\uac1c\uc758 \ub2e8\uc5b4\uac00 \ub450\uc6b4\uc744 \ub9de\ucdb0 \uc5f4\uac70\ub418\uc5b4 \uc788\uace0, \ubb38\uc790 V \uc790\uccb4\ub85c\ub294 54\uac1c\uac00 \ub098\uc628\ub2e4. \uc774\ube44\uac00 \ube0c\uc774\uc5d0\uac8c \uc790\uae30 \uc774\ub984\uc744 \ub9d0\ud558\uc790 \ube0c\uc774\ub294 \u201c\uc774\u2026 \ube0c\uc774\u201d(\"E... V\")\ub77c\uace0 \ub290\ub9bf\ub290\ub9bf \ubc18\ubcf5\ud558\ub294\ub370, E\ub294 \uc54c\ud30c\ubcb3\uc758 \ub2e4\uc12f \ubc88\uc9f8 \ubb38\uc790\uc774\ub2e4. \ub77c\ud06c\ud790\uc5d0 \uad6c\ub958\ub418\uc5b4 \uc788\ub41c \ub3d9\uc548, \ube0c\uc774\ub294 V\ubc88, \uc989 \ub85c\ub9c8 \uc22b\uc790\uc758 5\ubc88 \ubc29\uc5d0 \uac07\ud600 \uc788\uc5c8\uace0, \uc774\ube44\uac00 \uac70\uc9d3 \ud22c\uc625\uc744 \ub2f9\ud588\uc744 \ub54c\ub3c4 5\ubc88 \ubc29\uc774\uc5c8\ub2e4. \ube0c\uc774\uc758 \uc11c\uba85\uc740 \uc870\ub85c\uac00 \uce7c\uc790\uad6d\uc73c\ub85c Z\ub97c \ub0a8\uae30\ub294 \uac83\ucc98\ub7fc \uce7c\ub85c V\uc790\ub97c \uadf8\ub9ac\ub294 \uac83\uc774\ub2e4. \uc911\uc559 \ud615\uc0ac \uc7ac\ud310\uc18c\uac00 \ud3ed\ud30c\ub420 \ub54c \ubd88\uaf43\ub180\uc774\uc758 \ubd88\uaf43\uc774 \ubd89\uc740\uc0c9\uc758 V \ubaa8\uc591 \uc744 \ub9cc\ub4e0\ub2e4. \ub465\uadfc \ubd88\uaf43\uc774 \ub354\ud574\uc9c0\uba74 \ubd88\uaf43\uc758 \ubaa8\uc591\uc740 V \ubaa8\uc591 \ubfd0 \uc544\ub2c8\ub77c \ube0c\uc774 \ud3ec \ubca4\ub370\ud0c0 \ub85c\uace0\uc640\ub3c4 \ub2ee\uc558\ub2e4. \ube0c\uc774\uc758 \uc88c\uc6b0\uba85\uc778 \u201c\uc9c4\uc2e4\uc758 \ud798\uc73c\ub85c \uc0b4\uba74\uc11c \uc6b0\uc8fc\ub97c \uc815\ubcf5\ud588\ub2e4\u201d\ub294 \uc6d0\ub798 \ub77c\ud2f4\uc5b4 \u201c\ube44 \ubca0\ub9ac \ubca0\ub2c8\ubca0\ub974\uc228 \ube44\ubd80\uc2a4 \ube44\ud0a4\u201d(\"Vi Veri Veniversum Vivus Vici\")\ub85c, 5\uac1c\uc758 V\uac00 \ub4e4\uc5b4\uac00 \uc788\ub2e4. \uc774\ube44\uc640 \ucda4 \ucd9c \ub54c \ube0c\uc774\uac00 \uace0\ub974\ub294 \ub178\ub798\ub294 \uc8fc\ud06c\ubc15\uc2a4\uc758 5\ubc88 \ub178\ub798\uc774\ub2e4. \uc0ac\uc2e4 \ubaa8\ub4e0 \ub178\ub798\ub4e4\uc758 \ub118\ubc84\uac00 5\ubc88\uc774\ub2e4. \ud06c\ub9ac\ub514\uc758 \uc9d1\uc5d0\uc11c \ud06c\ub9ac\ub514\uc640 \ub300\uacb0\ud560 \ub54c \ube0c\uc774\uac00 \ud2c0\uc5b4\ub193\uc740 \uc74c\uc545\uc740 \u300a\uc6b4\uba85 \uad50\ud5a5\uace1\u300b, \uc989 \ubca0\ud1a0\ubca4 \u30085\ubc88\u3009 \uad50\ud5a5\uace1\uc73c\ub85c, \uc774 \ub178\ub798\uc758 \ucd08\ubc18\ubd80\uc758 \ub9ac\ub4ec \ud328\ud134\uc740 \ubaa8\uc2a4 \ubd80\ud638\uc758 V(\u00b7\u00b7\u00b7\u2013)\uc640 \ub2ee\uc558\ub2e4. \uc601\ud654\uc758 \uc81c\ubaa9 \uc790\uccb4\ub3c4 \ube0c\uc774 \uc0ac\uc778\uc744 \uc5f0\uc0c1\ud558\uac8c \ud55c\ub2e4. \ube0c\uc774\ub294 \ubd95\uad34\uc758 \ubc24\uc744 \uae30\ub2e4\ub9ac\uba74\uc11c \uac80\uc740\uc0c9\uacfc \ubd89\uc740\uc0c9\uc758 \ub3c4\ubbf8\ub178\ub85c \ubcf5\uc7a1\ud55c \ud328\ud134\uc744 \ub9cc\ub4e4\uace0, \ub3c4\ubbf8\ub178\uac00 \ub2e4 \uc4f0\ub7ec\uc9c0\uace0 \ub098\uc790 V \ub85c\uace0 \ubaa8\uc591\uc774 \ub098\ud0c0\ub09c\ub2e4. \ud06c\ub9ac\ub514\uc640 \uadf8 \ubd80\ud558\ub4e4\uacfc \ubc8c\uc778 \ucd5c\ud6c4 \uacb0\uc804\uc740 \ub7f0\ub358 \uc9c0\ud558\ucca0 \ube45\ud1a0\ub9ac\uc544\uc5ed \uc5d0\uc11c \ubc8c\uc5b4\uc84c\uace0, \uc774\ub54c \ube0c\uc774\ub294 6\uac1c\uc758 \ub2e8\uac80 \uc911 5\uac1c\ub97c \uc0ac\uc6a9\ud558\uba70, \ub2e8\uac80\uc744 \ub358\uc9c0\uae30 \uc9c1\uc804 \ub2e8\uac80\uc73c\ub85c V \ubaa8\uc591\uc744 \ub9cc\ub4e0\ub2e4. \ube0c\uc774\uac00 \ubd80\ud558\ub4e4\uc744 \ub2e4 \uc8fd\uc774\uace0 \ub098\uc11c, \ud06c\ub9ac\ub514\ub294 \ube0c\uc774\uc5d0\uac8c \ub9ac\ubcfc\ubc84\ub97c \ub2e4\uc12f \ubc1c \uc3dc\ub2e4. \uc2f8\uc6c0\uc774 \ub05d\ub09c \ub4a4, \uce58\uba85\uc0c1\uc744 \uc785\uc740 \ube0c\uc774\ub294 \uc5ed\uc744 \ub5a0\ub098\uba74\uc11c \ubcbd\uc5d0 \ubd80\ub51b\ud788\ub294\ub370, \uc774\ub54c V \ubaa8\uc591\uc758 \ud54f\uc790\uad6d\uc774 \uc0dd\uae34\ub2e4. \ube0c\uc774\uc640 \ud06c\ub9ac\ub514\uc758 \uc2f8\uc6c0\uc774 \ub05d\ub098\uc790 \ube45\ubca4\uc774 11\uc2dc 5\ubd84\uc744 \uac00\ub9ac\ud0a4\ub294\ub370, \uc774\ub54c \uc2dc\uce68\uacfc \ubd84\uce68\uc758 \ubaa8\uc591\uc774 V \ubaa8\uc591\uc774\ub2e4(11\uc6d4 5\uc77c\uc744 \uc5f0\uc0c1\ud558\uac8c \ud55c\ub2e4). \uad6d\ud68c\uc758\uc0ac\ub2f9\uc774 \ud3ed\ud30c\ub420 \ub54c\uc758 \ubd88\uaf43\ub180\uc774\ub3c4 V \ubaa8\uc591\uc774\uc790 \ubb34\uc815\ubd80\uc8fc\uc758\uc758 \uc0c1\uc9d5\uc778 Circle-A\uac00 \ub4a4\uc9d1\ud78c \ubaa8\uc591\uc744 \ub9cc\ub4e0\ub2e4.", "paragraph_answer": "\uc601\ud654\ub3c4 \ub9cc\ud654\ucc45\uacfc \ube44\uc2b7\ud558\uac8c \uae00\uc790 V\uc640 \uc22b\uc790 5\uac00 \ubc18\ubcf5\ub418\ub294 \ud14c\ub9c8\ub85c \uc774\uc6a9\ub418\uc5c8\ub2e4. \uc608\ub97c \ub4e4\uc5b4, \ube0c\uc774\uac00 \uc774\ube44\ub97c \ucc98\uc74c \ub9cc\ub0ac\uc744\ub54c \ub298\uc5b4\ub193\uc740 \uc18c\uac1c\ub9d0\uc5d0\uc11c\ub294 V\ub85c \uc2dc\uc791\ud558\ub294 50\uac1c\uc758 \ub2e8\uc5b4\uac00 \ub450\uc6b4\uc744 \ub9de\ucdb0 \uc5f4\uac70\ub418\uc5b4 \uc788\uace0, \ubb38\uc790 V \uc790\uccb4\ub85c\ub294 54\uac1c\uac00 \ub098\uc628\ub2e4. \uc774\ube44\uac00 \ube0c\uc774\uc5d0\uac8c \uc790\uae30 \uc774\ub984\uc744 \ub9d0\ud558\uc790 \ube0c\uc774\ub294 \u201c\uc774\u2026 \ube0c\uc774\u201d(\"E... V\")\ub77c\uace0 \ub290\ub9bf\ub290\ub9bf \ubc18\ubcf5\ud558\ub294\ub370, E\ub294 \uc54c\ud30c\ubcb3\uc758 \ub2e4\uc12f \ubc88\uc9f8 \ubb38\uc790\uc774\ub2e4. \ub77c\ud06c\ud790\uc5d0 \uad6c\ub958\ub418\uc5b4 \uc788\ub41c \ub3d9\uc548, \ube0c\uc774\ub294 V\ubc88, \uc989 \ub85c\ub9c8 \uc22b\uc790\uc758 5\ubc88 \ubc29\uc5d0 \uac07\ud600 \uc788\uc5c8\uace0, \uc774\ube44\uac00 \uac70\uc9d3 \ud22c\uc625\uc744 \ub2f9\ud588\uc744 \ub54c\ub3c4 5\ubc88 \ubc29\uc774\uc5c8\ub2e4. \ube0c\uc774\uc758 \uc11c\uba85\uc740 \uc870\ub85c\uac00 \uce7c\uc790\uad6d\uc73c\ub85c Z\ub97c \ub0a8\uae30\ub294 \uac83\ucc98\ub7fc \uce7c\ub85c V\uc790\ub97c \uadf8\ub9ac\ub294 \uac83\uc774\ub2e4. \uc911\uc559 \ud615\uc0ac \uc7ac\ud310\uc18c\uac00 \ud3ed\ud30c\ub420 \ub54c \ubd88\uaf43\ub180\uc774\uc758 \ubd88\uaf43\uc774 \ubd89\uc740\uc0c9\uc758 V \ubaa8\uc591 \uc744 \ub9cc\ub4e0\ub2e4. \ub465\uadfc \ubd88\uaf43\uc774 \ub354\ud574\uc9c0\uba74 \ubd88\uaf43\uc758 \ubaa8\uc591\uc740 V \ubaa8\uc591 \ubfd0 \uc544\ub2c8\ub77c \ube0c\uc774 \ud3ec \ubca4\ub370\ud0c0 \ub85c\uace0\uc640\ub3c4 \ub2ee\uc558\ub2e4. \ube0c\uc774\uc758 \uc88c\uc6b0\uba85\uc778 \u201c\uc9c4\uc2e4\uc758 \ud798\uc73c\ub85c \uc0b4\uba74\uc11c \uc6b0\uc8fc\ub97c \uc815\ubcf5\ud588\ub2e4\u201d\ub294 \uc6d0\ub798 \ub77c\ud2f4\uc5b4 \u201c\ube44 \ubca0\ub9ac \ubca0\ub2c8\ubca0\ub974\uc228 \ube44\ubd80\uc2a4 \ube44\ud0a4\u201d(\"Vi Veri Veniversum Vivus Vici\")\ub85c, 5\uac1c\uc758 V\uac00 \ub4e4\uc5b4\uac00 \uc788\ub2e4. \uc774\ube44\uc640 \ucda4 \ucd9c \ub54c \ube0c\uc774\uac00 \uace0\ub974\ub294 \ub178\ub798\ub294 \uc8fc\ud06c\ubc15\uc2a4\uc758 5\ubc88 \ub178\ub798\uc774\ub2e4. \uc0ac\uc2e4 \ubaa8\ub4e0 \ub178\ub798\ub4e4\uc758 \ub118\ubc84\uac00 5\ubc88\uc774\ub2e4. \ud06c\ub9ac\ub514\uc758 \uc9d1\uc5d0\uc11c \ud06c\ub9ac\ub514\uc640 \ub300\uacb0\ud560 \ub54c \ube0c\uc774\uac00 \ud2c0\uc5b4\ub193\uc740 \uc74c\uc545\uc740 \u300a\uc6b4\uba85 \uad50\ud5a5\uace1\u300b, \uc989 \ubca0\ud1a0\ubca4 \u30085\ubc88\u3009 \uad50\ud5a5\uace1\uc73c\ub85c, \uc774 \ub178\ub798\uc758 \ucd08\ubc18\ubd80\uc758 \ub9ac\ub4ec \ud328\ud134\uc740 \ubaa8\uc2a4 \ubd80\ud638\uc758 V(\u00b7\u00b7\u00b7\u2013)\uc640 \ub2ee\uc558\ub2e4. \uc601\ud654\uc758 \uc81c\ubaa9 \uc790\uccb4\ub3c4 \ube0c\uc774 \uc0ac\uc778\uc744 \uc5f0\uc0c1\ud558\uac8c \ud55c\ub2e4. \ube0c\uc774\ub294 \ubd95\uad34\uc758 \ubc24\uc744 \uae30\ub2e4\ub9ac\uba74\uc11c \uac80\uc740\uc0c9\uacfc \ubd89\uc740\uc0c9\uc758 \ub3c4\ubbf8\ub178\ub85c \ubcf5\uc7a1\ud55c \ud328\ud134\uc744 \ub9cc\ub4e4\uace0, \ub3c4\ubbf8\ub178\uac00 \ub2e4 \uc4f0\ub7ec\uc9c0\uace0 \ub098\uc790 V \ub85c\uace0 \ubaa8\uc591\uc774 \ub098\ud0c0\ub09c\ub2e4. \ud06c\ub9ac\ub514\uc640 \uadf8 \ubd80\ud558\ub4e4\uacfc \ubc8c\uc778 \ucd5c\ud6c4 \uacb0\uc804\uc740 \ub7f0\ub358 \uc9c0\ud558\ucca0 \ube45\ud1a0\ub9ac\uc544\uc5ed \uc5d0\uc11c \ubc8c\uc5b4\uc84c\uace0, \uc774\ub54c \ube0c\uc774\ub294 6\uac1c\uc758 \ub2e8\uac80 \uc911 5\uac1c\ub97c \uc0ac\uc6a9\ud558\uba70, \ub2e8\uac80\uc744 \ub358\uc9c0\uae30 \uc9c1\uc804 \ub2e8\uac80\uc73c\ub85c V \ubaa8\uc591\uc744 \ub9cc\ub4e0\ub2e4. \ube0c\uc774\uac00 \ubd80\ud558\ub4e4\uc744 \ub2e4 \uc8fd\uc774\uace0 \ub098\uc11c, \ud06c\ub9ac\ub514\ub294 \ube0c\uc774\uc5d0\uac8c \ub9ac\ubcfc\ubc84\ub97c \ub2e4\uc12f \ubc1c \uc3dc\ub2e4. \uc2f8\uc6c0\uc774 \ub05d\ub09c \ub4a4, \uce58\uba85\uc0c1\uc744 \uc785\uc740 \ube0c\uc774\ub294 \uc5ed\uc744 \ub5a0\ub098\uba74\uc11c \ubcbd\uc5d0 \ubd80\ub51b\ud788\ub294\ub370, \uc774\ub54c V \ubaa8\uc591\uc758 \ud54f\uc790\uad6d\uc774 \uc0dd\uae34\ub2e4. \ube0c\uc774\uc640 \ud06c\ub9ac\ub514\uc758 \uc2f8\uc6c0\uc774 \ub05d\ub098\uc790 \ube45\ubca4\uc774 11\uc2dc 5\ubd84\uc744 \uac00\ub9ac\ud0a4\ub294\ub370, \uc774\ub54c \uc2dc\uce68\uacfc \ubd84\uce68\uc758 \ubaa8\uc591\uc774 V \ubaa8\uc591\uc774\ub2e4(11\uc6d4 5\uc77c\uc744 \uc5f0\uc0c1\ud558\uac8c \ud55c\ub2e4). \uad6d\ud68c\uc758\uc0ac\ub2f9\uc774 \ud3ed\ud30c\ub420 \ub54c\uc758 \ubd88\uaf43\ub180\uc774\ub3c4 V \ubaa8\uc591\uc774\uc790 \ubb34\uc815\ubd80\uc8fc\uc758\uc758 \uc0c1\uc9d5\uc778 Circle-A\uac00 \ub4a4\uc9d1\ud78c \ubaa8\uc591\uc744 \ub9cc\ub4e0\ub2e4.", "sentence_answer": "\uc911\uc559 \ud615\uc0ac \uc7ac\ud310\uc18c\uac00 \ud3ed\ud30c\ub420 \ub54c \ubd88\uaf43\ub180\uc774\uc758 \ubd88\uaf43\uc774 \ubd89\uc740\uc0c9\uc758 V \ubaa8\uc591 \uc744 \ub9cc\ub4e0\ub2e4.", "paragraph_id": "6516998-15-2", "paragraph_question": "question: \uad6d\ud68c\uc758\uc0ac\ub2f9\uc774 \ud3ed\ud30c \ub420 \ub54c \ubd88\uaf43\ub180\uc774\uc758 \ubaa8\uc591\uc740 \ubb34\uc5c7\uc778\uac00?, context: \uc601\ud654\ub3c4 \ub9cc\ud654\ucc45\uacfc \ube44\uc2b7\ud558\uac8c \uae00\uc790 V\uc640 \uc22b\uc790 5\uac00 \ubc18\ubcf5\ub418\ub294 \ud14c\ub9c8\ub85c \uc774\uc6a9\ub418\uc5c8\ub2e4. \uc608\ub97c \ub4e4\uc5b4, \ube0c\uc774\uac00 \uc774\ube44\ub97c \ucc98\uc74c \ub9cc\ub0ac\uc744\ub54c \ub298\uc5b4\ub193\uc740 \uc18c\uac1c\ub9d0\uc5d0\uc11c\ub294 V\ub85c \uc2dc\uc791\ud558\ub294 50\uac1c\uc758 \ub2e8\uc5b4\uac00 \ub450\uc6b4\uc744 \ub9de\ucdb0 \uc5f4\uac70\ub418\uc5b4 \uc788\uace0, \ubb38\uc790 V \uc790\uccb4\ub85c\ub294 54\uac1c\uac00 \ub098\uc628\ub2e4. \uc774\ube44\uac00 \ube0c\uc774\uc5d0\uac8c \uc790\uae30 \uc774\ub984\uc744 \ub9d0\ud558\uc790 \ube0c\uc774\ub294 \u201c\uc774\u2026 \ube0c\uc774\u201d(\"E... V\")\ub77c\uace0 \ub290\ub9bf\ub290\ub9bf \ubc18\ubcf5\ud558\ub294\ub370, E\ub294 \uc54c\ud30c\ubcb3\uc758 \ub2e4\uc12f \ubc88\uc9f8 \ubb38\uc790\uc774\ub2e4. \ub77c\ud06c\ud790\uc5d0 \uad6c\ub958\ub418\uc5b4 \uc788\ub41c \ub3d9\uc548, \ube0c\uc774\ub294 V\ubc88, \uc989 \ub85c\ub9c8 \uc22b\uc790\uc758 5\ubc88 \ubc29\uc5d0 \uac07\ud600 \uc788\uc5c8\uace0, \uc774\ube44\uac00 \uac70\uc9d3 \ud22c\uc625\uc744 \ub2f9\ud588\uc744 \ub54c\ub3c4 5\ubc88 \ubc29\uc774\uc5c8\ub2e4. \ube0c\uc774\uc758 \uc11c\uba85\uc740 \uc870\ub85c\uac00 \uce7c\uc790\uad6d\uc73c\ub85c Z\ub97c \ub0a8\uae30\ub294 \uac83\ucc98\ub7fc \uce7c\ub85c V\uc790\ub97c \uadf8\ub9ac\ub294 \uac83\uc774\ub2e4. \uc911\uc559 \ud615\uc0ac \uc7ac\ud310\uc18c\uac00 \ud3ed\ud30c\ub420 \ub54c \ubd88\uaf43\ub180\uc774\uc758 \ubd88\uaf43\uc774 \ubd89\uc740\uc0c9\uc758 V \ubaa8\uc591\uc744 \ub9cc\ub4e0\ub2e4. \ub465\uadfc \ubd88\uaf43\uc774 \ub354\ud574\uc9c0\uba74 \ubd88\uaf43\uc758 \ubaa8\uc591\uc740 V \ubaa8\uc591 \ubfd0 \uc544\ub2c8\ub77c \ube0c\uc774 \ud3ec \ubca4\ub370\ud0c0 \ub85c\uace0\uc640\ub3c4 \ub2ee\uc558\ub2e4. \ube0c\uc774\uc758 \uc88c\uc6b0\uba85\uc778 \u201c\uc9c4\uc2e4\uc758 \ud798\uc73c\ub85c \uc0b4\uba74\uc11c \uc6b0\uc8fc\ub97c \uc815\ubcf5\ud588\ub2e4\u201d\ub294 \uc6d0\ub798 \ub77c\ud2f4\uc5b4 \u201c\ube44 \ubca0\ub9ac \ubca0\ub2c8\ubca0\ub974\uc228 \ube44\ubd80\uc2a4 \ube44\ud0a4\u201d(\"Vi Veri Veniversum Vivus Vici\")\ub85c, 5\uac1c\uc758 V\uac00 \ub4e4\uc5b4\uac00 \uc788\ub2e4. \uc774\ube44\uc640 \ucda4 \ucd9c \ub54c \ube0c\uc774\uac00 \uace0\ub974\ub294 \ub178\ub798\ub294 \uc8fc\ud06c\ubc15\uc2a4\uc758 5\ubc88 \ub178\ub798\uc774\ub2e4. \uc0ac\uc2e4 \ubaa8\ub4e0 \ub178\ub798\ub4e4\uc758 \ub118\ubc84\uac00 5\ubc88\uc774\ub2e4. \ud06c\ub9ac\ub514\uc758 \uc9d1\uc5d0\uc11c \ud06c\ub9ac\ub514\uc640 \ub300\uacb0\ud560 \ub54c \ube0c\uc774\uac00 \ud2c0\uc5b4\ub193\uc740 \uc74c\uc545\uc740 \u300a\uc6b4\uba85 \uad50\ud5a5\uace1\u300b, \uc989 \ubca0\ud1a0\ubca4 \u30085\ubc88\u3009 \uad50\ud5a5\uace1\uc73c\ub85c, \uc774 \ub178\ub798\uc758 \ucd08\ubc18\ubd80\uc758 \ub9ac\ub4ec \ud328\ud134\uc740 \ubaa8\uc2a4 \ubd80\ud638\uc758 V(\u00b7\u00b7\u00b7\u2013)\uc640 \ub2ee\uc558\ub2e4. \uc601\ud654\uc758 \uc81c\ubaa9 \uc790\uccb4\ub3c4 \ube0c\uc774 \uc0ac\uc778\uc744 \uc5f0\uc0c1\ud558\uac8c \ud55c\ub2e4. \ube0c\uc774\ub294 \ubd95\uad34\uc758 \ubc24\uc744 \uae30\ub2e4\ub9ac\uba74\uc11c \uac80\uc740\uc0c9\uacfc \ubd89\uc740\uc0c9\uc758 \ub3c4\ubbf8\ub178\ub85c \ubcf5\uc7a1\ud55c \ud328\ud134\uc744 \ub9cc\ub4e4\uace0, \ub3c4\ubbf8\ub178\uac00 \ub2e4 \uc4f0\ub7ec\uc9c0\uace0 \ub098\uc790 V \ub85c\uace0 \ubaa8\uc591\uc774 \ub098\ud0c0\ub09c\ub2e4. \ud06c\ub9ac\ub514\uc640 \uadf8 \ubd80\ud558\ub4e4\uacfc \ubc8c\uc778 \ucd5c\ud6c4 \uacb0\uc804\uc740 \ub7f0\ub358 \uc9c0\ud558\ucca0 \ube45\ud1a0\ub9ac\uc544\uc5ed \uc5d0\uc11c \ubc8c\uc5b4\uc84c\uace0, \uc774\ub54c \ube0c\uc774\ub294 6\uac1c\uc758 \ub2e8\uac80 \uc911 5\uac1c\ub97c \uc0ac\uc6a9\ud558\uba70, \ub2e8\uac80\uc744 \ub358\uc9c0\uae30 \uc9c1\uc804 \ub2e8\uac80\uc73c\ub85c V \ubaa8\uc591\uc744 \ub9cc\ub4e0\ub2e4. \ube0c\uc774\uac00 \ubd80\ud558\ub4e4\uc744 \ub2e4 \uc8fd\uc774\uace0 \ub098\uc11c, \ud06c\ub9ac\ub514\ub294 \ube0c\uc774\uc5d0\uac8c \ub9ac\ubcfc\ubc84\ub97c \ub2e4\uc12f \ubc1c \uc3dc\ub2e4. \uc2f8\uc6c0\uc774 \ub05d\ub09c \ub4a4, \uce58\uba85\uc0c1\uc744 \uc785\uc740 \ube0c\uc774\ub294 \uc5ed\uc744 \ub5a0\ub098\uba74\uc11c \ubcbd\uc5d0 \ubd80\ub51b\ud788\ub294\ub370, \uc774\ub54c V \ubaa8\uc591\uc758 \ud54f\uc790\uad6d\uc774 \uc0dd\uae34\ub2e4. \ube0c\uc774\uc640 \ud06c\ub9ac\ub514\uc758 \uc2f8\uc6c0\uc774 \ub05d\ub098\uc790 \ube45\ubca4\uc774 11\uc2dc 5\ubd84\uc744 \uac00\ub9ac\ud0a4\ub294\ub370, \uc774\ub54c \uc2dc\uce68\uacfc \ubd84\uce68\uc758 \ubaa8\uc591\uc774 V \ubaa8\uc591\uc774\ub2e4(11\uc6d4 5\uc77c\uc744 \uc5f0\uc0c1\ud558\uac8c \ud55c\ub2e4). \uad6d\ud68c\uc758\uc0ac\ub2f9\uc774 \ud3ed\ud30c\ub420 \ub54c\uc758 \ubd88\uaf43\ub180\uc774\ub3c4 V \ubaa8\uc591\uc774\uc790 \ubb34\uc815\ubd80\uc8fc\uc758\uc758 \uc0c1\uc9d5\uc778 Circle-A\uac00 \ub4a4\uc9d1\ud78c \ubaa8\uc591\uc744 \ub9cc\ub4e0\ub2e4."} {"question": "\uc804\uc8fc\uc2e0\ud765\uace0\ub4f1\ud559\uad50 \uad50\uac00\uc758 \uc6d0\uace0\uc73c\ub85c \uc54c\ub824\uc9c4 \uace1\uc740?", "paragraph": "\uc804\uc8fc\uc2e0\ud765\uace0\ub4f1\ud559\uad50(\uc804\uc8fc\uc2e0\ud765\uc911\ud559\uad50)\uc758 \uad50\uac00\ub294 1910\ub144\ub300 \ub9cc\uc8fc\uc5d0\uc11c \ud65c\uc57d\ud558\ub358 \ub3c5\ub9bd\uad70\ub4e4\uc774\ubd80\ub974\ub358 \ub300\ud45c\uc801 \uad70\uac00\uc758 \ud558\ub098\uc778 '\uc6a9\uc9c4\uac00'\uc640 \ub3d9\uc77c\ud558\uace0 \ud6c4\ub834\ubd80 \uac00\uc0ac\uac00 \ube44\uc2b7\ud558\ub2e4. 1910\ub144\ub300 \uc804\uc8fc\uc758 \uc2e0\ud765\ud559\uad50\uc5d0\uc11c\ub3c4 \ub3d9\uc77c\ud55c \uad50\uac00\uac00 \ubd88\ub838\ub2e4\ub294 \ub2f9\uc2dc \uc7ac\ud559\uc0dd\uc758 \uc99d\uc5b8\uc774 \uc788\uc73c\uba70, 1910\ub144\ub300 \uc804\uc8fc \uc2e0\ud765\ud559\uad50\uc5d0 \uc7ac\uc9c1\ud588\ub358 \uad50\uc0ac\ub4e4 \uc911 \ubbfc\uc871\uc758\uc2dd\uc774 \uac15\ud588\ub358 \uc624\uc0b0\ud559\uad50\uc640 \uc22d\uc2e4\ud559\uad50 \ucd9c\uc2e0\uc774 \ub9ce\uc558\uc73c\ubbc0\ub85c \ub9cc\uc8fc\uc9c0\uc5ed \ub3c5\ub9bd\uad70\uacfc \uc9c1\uac04\uc811\uc801\uc73c\ub85c \uc5f0\uacb0\ub418\uc5b4 \uc788\uc5c8\uc744 \uac83\uc774\ub77c \ucd94\uc815\ud558\uace0 \uc788\ub2e4. \ub3c5\ub9bd\uad70\uc758 \uad70\uac00\uc600\ub358 \uc6a9\uc9c4\uac00\ub294 1911\ub144 \ub9cc\uc8fc \uc11c\uac04\ub3c4\uc5d0\uc11c \uac1c\uad50\ud55c \uc2e0\ud765\ubb34\uad00\ud559\uad50(\uacbd\ud76c\ub300\ud559\uad50\uc758 \uc804\uc2e0)\uc758 \uad50\uac00\ub85c\ub3c4 \ubcc0\ud615\ub418\uc5b4 \uc0ac\uc6a9\ub418\uc5c8\uc73c\uba70, \uac00\uc0ac\uac00 \ubcc0\ud615\ub418\uc5b4 \ubd81\ud55c\uc5d0\uc11c \uad70\uac00\ub85c\ub3c4 \uc0ac\uc6a9\ub418\uace0 \uc788\ub2e4. 2000\ub144 6\uc6d4 13\uc77c \ubc29\ubd81\ud55c \uae40\ub300\uc911 \ub300\ud1b5\ub839\uc744 \ud658\uc601\ud558\uae30 \uc704\ud574 \ubd81\ud55c \uad70\uc545\ub300\uac00 \uc21c\uc548\uacf5\ud56d\uc5d0\uc11c \uc5f0\uc8fc\ud55c \uace1\uc774 \uc804\uc8fc\uc2e0\ud765\uace0\ub4f1\ud559\uad50 \uad50\uac00\uc640 \uc720\uc0ac\ud558\ub2e4\ub294 \uac83\uc774 \ud654\uc81c \uac00 \ub418\uc5c8\ub2e4. \uc774 \uace1\ub4e4\uc758 \uc6d0\uace1\uc740 \ubbf8\uad6d \ub0a8\ubd81\uc804\uc7c1\uc774 \ub05d\ub0a0 \ubb34\ub835\uc778 1865\ub144 \ud5e8\ub9ac \ud074\ub808\uc774 \uc6cc\ud06c\uac00 \uc9c0\uc740 \uc870\uc9c0\uc544\ud589\uc9c4\uace1('Marching Through Georgia')\uc73c\ub85c \uc54c\ub824\uc838 \uc788\ub2e4.", "answer": "\uc870\uc9c0\uc544\ud589\uc9c4\uace1", "sentence": "\uc774 \uace1\ub4e4\uc758 \uc6d0\uace1\uc740 \ubbf8\uad6d \ub0a8\ubd81\uc804\uc7c1\uc774 \ub05d\ub0a0 \ubb34\ub835\uc778 1865\ub144 \ud5e8\ub9ac \ud074\ub808\uc774 \uc6cc\ud06c\uac00 \uc9c0\uc740 \uc870\uc9c0\uc544\ud589\uc9c4\uace1 ('Marching Through Georgia')", "paragraph_sentence": "\uc804\uc8fc\uc2e0\ud765\uace0\ub4f1\ud559\uad50(\uc804\uc8fc\uc2e0\ud765\uc911\ud559\uad50)\uc758 \uad50\uac00\ub294 1910\ub144\ub300 \ub9cc\uc8fc\uc5d0\uc11c \ud65c\uc57d\ud558\ub358 \ub3c5\ub9bd\uad70\ub4e4\uc774\ubd80\ub974\ub358 \ub300\ud45c\uc801 \uad70\uac00\uc758 \ud558\ub098\uc778 '\uc6a9\uc9c4\uac00'\uc640 \ub3d9\uc77c\ud558\uace0 \ud6c4\ub834\ubd80 \uac00\uc0ac\uac00 \ube44\uc2b7\ud558\ub2e4. 1910\ub144\ub300 \uc804\uc8fc\uc758 \uc2e0\ud765\ud559\uad50\uc5d0\uc11c\ub3c4 \ub3d9\uc77c\ud55c \uad50\uac00\uac00 \ubd88\ub838\ub2e4\ub294 \ub2f9\uc2dc \uc7ac\ud559\uc0dd\uc758 \uc99d\uc5b8\uc774 \uc788\uc73c\uba70, 1910\ub144\ub300 \uc804\uc8fc \uc2e0\ud765\ud559\uad50\uc5d0 \uc7ac\uc9c1\ud588\ub358 \uad50\uc0ac\ub4e4 \uc911 \ubbfc\uc871\uc758\uc2dd\uc774 \uac15\ud588\ub358 \uc624\uc0b0\ud559\uad50\uc640 \uc22d\uc2e4\ud559\uad50 \ucd9c\uc2e0\uc774 \ub9ce\uc558\uc73c\ubbc0\ub85c \ub9cc\uc8fc\uc9c0\uc5ed \ub3c5\ub9bd\uad70\uacfc \uc9c1\uac04\uc811\uc801\uc73c\ub85c \uc5f0\uacb0\ub418\uc5b4 \uc788\uc5c8\uc744 \uac83\uc774\ub77c \ucd94\uc815\ud558\uace0 \uc788\ub2e4. \ub3c5\ub9bd\uad70\uc758 \uad70\uac00\uc600\ub358 \uc6a9\uc9c4\uac00\ub294 1911\ub144 \ub9cc\uc8fc \uc11c\uac04\ub3c4\uc5d0\uc11c \uac1c\uad50\ud55c \uc2e0\ud765\ubb34\uad00\ud559\uad50(\uacbd\ud76c\ub300\ud559\uad50\uc758 \uc804\uc2e0)\uc758 \uad50\uac00\ub85c\ub3c4 \ubcc0\ud615\ub418\uc5b4 \uc0ac\uc6a9\ub418\uc5c8\uc73c\uba70, \uac00\uc0ac\uac00 \ubcc0\ud615\ub418\uc5b4 \ubd81\ud55c\uc5d0\uc11c \uad70\uac00\ub85c\ub3c4 \uc0ac\uc6a9\ub418\uace0 \uc788\ub2e4. 2000\ub144 6\uc6d4 13\uc77c \ubc29\ubd81\ud55c \uae40\ub300\uc911 \ub300\ud1b5\ub839\uc744 \ud658\uc601\ud558\uae30 \uc704\ud574 \ubd81\ud55c \uad70\uc545\ub300\uac00 \uc21c\uc548\uacf5\ud56d\uc5d0\uc11c \uc5f0\uc8fc\ud55c \uace1\uc774 \uc804\uc8fc\uc2e0\ud765\uace0\ub4f1\ud559\uad50 \uad50\uac00\uc640 \uc720\uc0ac\ud558\ub2e4\ub294 \uac83\uc774 \ud654\uc81c \uac00 \ub418\uc5c8\ub2e4. \uc774 \uace1\ub4e4\uc758 \uc6d0\uace1\uc740 \ubbf8\uad6d \ub0a8\ubd81\uc804\uc7c1\uc774 \ub05d\ub0a0 \ubb34\ub835\uc778 1865\ub144 \ud5e8\ub9ac \ud074\ub808\uc774 \uc6cc\ud06c\uac00 \uc9c0\uc740 \uc870\uc9c0\uc544\ud589\uc9c4\uace1 ('Marching Through Georgia') \uc73c\ub85c \uc54c\ub824\uc838 \uc788\ub2e4.", "paragraph_answer": "\uc804\uc8fc\uc2e0\ud765\uace0\ub4f1\ud559\uad50(\uc804\uc8fc\uc2e0\ud765\uc911\ud559\uad50)\uc758 \uad50\uac00\ub294 1910\ub144\ub300 \ub9cc\uc8fc\uc5d0\uc11c \ud65c\uc57d\ud558\ub358 \ub3c5\ub9bd\uad70\ub4e4\uc774\ubd80\ub974\ub358 \ub300\ud45c\uc801 \uad70\uac00\uc758 \ud558\ub098\uc778 '\uc6a9\uc9c4\uac00'\uc640 \ub3d9\uc77c\ud558\uace0 \ud6c4\ub834\ubd80 \uac00\uc0ac\uac00 \ube44\uc2b7\ud558\ub2e4. 1910\ub144\ub300 \uc804\uc8fc\uc758 \uc2e0\ud765\ud559\uad50\uc5d0\uc11c\ub3c4 \ub3d9\uc77c\ud55c \uad50\uac00\uac00 \ubd88\ub838\ub2e4\ub294 \ub2f9\uc2dc \uc7ac\ud559\uc0dd\uc758 \uc99d\uc5b8\uc774 \uc788\uc73c\uba70, 1910\ub144\ub300 \uc804\uc8fc \uc2e0\ud765\ud559\uad50\uc5d0 \uc7ac\uc9c1\ud588\ub358 \uad50\uc0ac\ub4e4 \uc911 \ubbfc\uc871\uc758\uc2dd\uc774 \uac15\ud588\ub358 \uc624\uc0b0\ud559\uad50\uc640 \uc22d\uc2e4\ud559\uad50 \ucd9c\uc2e0\uc774 \ub9ce\uc558\uc73c\ubbc0\ub85c \ub9cc\uc8fc\uc9c0\uc5ed \ub3c5\ub9bd\uad70\uacfc \uc9c1\uac04\uc811\uc801\uc73c\ub85c \uc5f0\uacb0\ub418\uc5b4 \uc788\uc5c8\uc744 \uac83\uc774\ub77c \ucd94\uc815\ud558\uace0 \uc788\ub2e4. \ub3c5\ub9bd\uad70\uc758 \uad70\uac00\uc600\ub358 \uc6a9\uc9c4\uac00\ub294 1911\ub144 \ub9cc\uc8fc \uc11c\uac04\ub3c4\uc5d0\uc11c \uac1c\uad50\ud55c \uc2e0\ud765\ubb34\uad00\ud559\uad50(\uacbd\ud76c\ub300\ud559\uad50\uc758 \uc804\uc2e0)\uc758 \uad50\uac00\ub85c\ub3c4 \ubcc0\ud615\ub418\uc5b4 \uc0ac\uc6a9\ub418\uc5c8\uc73c\uba70, \uac00\uc0ac\uac00 \ubcc0\ud615\ub418\uc5b4 \ubd81\ud55c\uc5d0\uc11c \uad70\uac00\ub85c\ub3c4 \uc0ac\uc6a9\ub418\uace0 \uc788\ub2e4. 2000\ub144 6\uc6d4 13\uc77c \ubc29\ubd81\ud55c \uae40\ub300\uc911 \ub300\ud1b5\ub839\uc744 \ud658\uc601\ud558\uae30 \uc704\ud574 \ubd81\ud55c \uad70\uc545\ub300\uac00 \uc21c\uc548\uacf5\ud56d\uc5d0\uc11c \uc5f0\uc8fc\ud55c \uace1\uc774 \uc804\uc8fc\uc2e0\ud765\uace0\ub4f1\ud559\uad50 \uad50\uac00\uc640 \uc720\uc0ac\ud558\ub2e4\ub294 \uac83\uc774 \ud654\uc81c \uac00 \ub418\uc5c8\ub2e4. \uc774 \uace1\ub4e4\uc758 \uc6d0\uace1\uc740 \ubbf8\uad6d \ub0a8\ubd81\uc804\uc7c1\uc774 \ub05d\ub0a0 \ubb34\ub835\uc778 1865\ub144 \ud5e8\ub9ac \ud074\ub808\uc774 \uc6cc\ud06c\uac00 \uc9c0\uc740 \uc870\uc9c0\uc544\ud589\uc9c4\uace1 ('Marching Through Georgia')\uc73c\ub85c \uc54c\ub824\uc838 \uc788\ub2e4.", "sentence_answer": "\uc774 \uace1\ub4e4\uc758 \uc6d0\uace1\uc740 \ubbf8\uad6d \ub0a8\ubd81\uc804\uc7c1\uc774 \ub05d\ub0a0 \ubb34\ub835\uc778 1865\ub144 \ud5e8\ub9ac \ud074\ub808\uc774 \uc6cc\ud06c\uac00 \uc9c0\uc740 \uc870\uc9c0\uc544\ud589\uc9c4\uace1 ('Marching Through Georgia')", "paragraph_id": "5865150-4-1", "paragraph_question": "question: \uc804\uc8fc\uc2e0\ud765\uace0\ub4f1\ud559\uad50 \uad50\uac00\uc758 \uc6d0\uace0\uc73c\ub85c \uc54c\ub824\uc9c4 \uace1\uc740?, context: \uc804\uc8fc\uc2e0\ud765\uace0\ub4f1\ud559\uad50(\uc804\uc8fc\uc2e0\ud765\uc911\ud559\uad50)\uc758 \uad50\uac00\ub294 1910\ub144\ub300 \ub9cc\uc8fc\uc5d0\uc11c \ud65c\uc57d\ud558\ub358 \ub3c5\ub9bd\uad70\ub4e4\uc774\ubd80\ub974\ub358 \ub300\ud45c\uc801 \uad70\uac00\uc758 \ud558\ub098\uc778 '\uc6a9\uc9c4\uac00'\uc640 \ub3d9\uc77c\ud558\uace0 \ud6c4\ub834\ubd80 \uac00\uc0ac\uac00 \ube44\uc2b7\ud558\ub2e4. 1910\ub144\ub300 \uc804\uc8fc\uc758 \uc2e0\ud765\ud559\uad50\uc5d0\uc11c\ub3c4 \ub3d9\uc77c\ud55c \uad50\uac00\uac00 \ubd88\ub838\ub2e4\ub294 \ub2f9\uc2dc \uc7ac\ud559\uc0dd\uc758 \uc99d\uc5b8\uc774 \uc788\uc73c\uba70, 1910\ub144\ub300 \uc804\uc8fc \uc2e0\ud765\ud559\uad50\uc5d0 \uc7ac\uc9c1\ud588\ub358 \uad50\uc0ac\ub4e4 \uc911 \ubbfc\uc871\uc758\uc2dd\uc774 \uac15\ud588\ub358 \uc624\uc0b0\ud559\uad50\uc640 \uc22d\uc2e4\ud559\uad50 \ucd9c\uc2e0\uc774 \ub9ce\uc558\uc73c\ubbc0\ub85c \ub9cc\uc8fc\uc9c0\uc5ed \ub3c5\ub9bd\uad70\uacfc \uc9c1\uac04\uc811\uc801\uc73c\ub85c \uc5f0\uacb0\ub418\uc5b4 \uc788\uc5c8\uc744 \uac83\uc774\ub77c \ucd94\uc815\ud558\uace0 \uc788\ub2e4. \ub3c5\ub9bd\uad70\uc758 \uad70\uac00\uc600\ub358 \uc6a9\uc9c4\uac00\ub294 1911\ub144 \ub9cc\uc8fc \uc11c\uac04\ub3c4\uc5d0\uc11c \uac1c\uad50\ud55c \uc2e0\ud765\ubb34\uad00\ud559\uad50(\uacbd\ud76c\ub300\ud559\uad50\uc758 \uc804\uc2e0)\uc758 \uad50\uac00\ub85c\ub3c4 \ubcc0\ud615\ub418\uc5b4 \uc0ac\uc6a9\ub418\uc5c8\uc73c\uba70, \uac00\uc0ac\uac00 \ubcc0\ud615\ub418\uc5b4 \ubd81\ud55c\uc5d0\uc11c \uad70\uac00\ub85c\ub3c4 \uc0ac\uc6a9\ub418\uace0 \uc788\ub2e4. 2000\ub144 6\uc6d4 13\uc77c \ubc29\ubd81\ud55c \uae40\ub300\uc911 \ub300\ud1b5\ub839\uc744 \ud658\uc601\ud558\uae30 \uc704\ud574 \ubd81\ud55c \uad70\uc545\ub300\uac00 \uc21c\uc548\uacf5\ud56d\uc5d0\uc11c \uc5f0\uc8fc\ud55c \uace1\uc774 \uc804\uc8fc\uc2e0\ud765\uace0\ub4f1\ud559\uad50 \uad50\uac00\uc640 \uc720\uc0ac\ud558\ub2e4\ub294 \uac83\uc774 \ud654\uc81c \uac00 \ub418\uc5c8\ub2e4. \uc774 \uace1\ub4e4\uc758 \uc6d0\uace1\uc740 \ubbf8\uad6d \ub0a8\ubd81\uc804\uc7c1\uc774 \ub05d\ub0a0 \ubb34\ub835\uc778 1865\ub144 \ud5e8\ub9ac \ud074\ub808\uc774 \uc6cc\ud06c\uac00 \uc9c0\uc740 \uc870\uc9c0\uc544\ud589\uc9c4\uace1('Marching Through Georgia')\uc73c\ub85c \uc54c\ub824\uc838 \uc788\ub2e4."} {"question": "\ud30c\uc5c5, \ud0dc\uc5c5, \uc0ac\ubcf4\ud0c0\uc9c0 \ub4f1\uc73c\ub85c 1947\ub144 \ud558\ub8e8 \uc6b4\ud589\uc5d0 \ub098\uc11c\ub294 \uc804\ucc28 \uc218\ub294?", "paragraph": "\uc77c\uc81c \ud328\ub9dd\uacfc \ud568\uaed8 \uacbd\uc131\uc804\uae30\uc8fc\uc2dd\ud68c\uc0ac\uc758 \uc804\ucc28\uc0ac\uc5c5 \uc5ed\uc2dc \ub2e4\ub978 \uc0ac\uc5c5\uacfc \ud568\uaed8 \ud55c\uad6d\uc778\uc758 \uc190\uc73c\ub85c \uc6b4\uc601\ud558\uac8c \ub418\uc5c8\ub2e4. \uadf8\ub7ec\ub098 \ud55c\ud3b8\uc73c\ub85c, \uc804\uc2dc\ub3d9\uc6d0\uae30\uc758 \ud639\uc0ac\uc640 \uad11\ubcf5\uc758 \uae30\uc068\uc5d0 \ud658\ud638\ud558\ub294 \uc2dc\ubbfc\ub4e4\uc758 \ub09c\ub3d9\uc73c\ub85c \uc778\ud574 \uc804\ucc28\ub294 \uc218\ub09c\uc744 \uacaa\uac8c \ub418\uc5c8\ub2e4. \ud574\ubc29 \ud6c4 \ubcf4\ub984 \ub3d9\uc548 \ucc28\ub7c9\uc758 \uac00\ub3d9\uc0c1\ud0dc\ub294 101\ub300\uc5d0\uc11c 59\ub300\uae4c\uc9c0 \uae09\uac10\ud558\uae30\uc5d0 \uc774\ub974\ub800\ub2e4. \ub610\ud55c, \uc11c\uc6b8\uc2dc\ub294 \uadc0\uad6d\ub3d9\ud3ec\uc640 \ubd81\ud55c\uc73c\ub85c\ubd80\ud130\uc758 \uc6d4\ub0a8\uc790 \uc99d\uac00\ub85c \ud06c\uac8c \ubc00\ub3c4\uac00 \uc99d\uac00\ud574\uc11c, \uc804\ucc28\uc6b4\ud589\uc740 \uac00\uc77c\uce35 \uac00\ud639\ud55c \uc0c1\ud669\uc774\uc5c8\ub2e4. \uc5ec\uae30\uc5d0 \uc88c\uc6b0\uc775\uc758 \ub300\ub9bd\uc5d0 \ub530\ub978 \ud63c\ub780\uc73c\ub85c \ub178\ub3d9\uc870\ud569\uc758 \ud30c\uc5c5, \ud0dc\uc5c5, \uc0ac\ubcf4\ud0c0\uc9c0 \ub4f1\uc774 \uc18d\ucd9c\ud558\uc5ec 1947\ub144\uc5d0 \uc774\ub974\ub7ec\uc11c\ub294 \ud558\ub8e8 27\ub300\ub9cc \uc6b4\ud589\ud560 \uc815\ub3c4\ub85c \ud63c\ub780\uc758 \uadf9\uc5d0 \ub2ec\ud588\ub2e4. \uc774\ud6c4, \ub2e8\uccb4\ud611\uc57d\ub4f1\uc744 \ud1b5\ud574 \uc0c1\ud669\uc774 \ud638\uc804\ub418\uba74\uc11c 27\ub300\ub97c \uae30\uc810\uc73c\ub85c \ub2e4\uc2dc \uac00\ub3d9\ub960\uc774 \ud68c\ubcf5\ub418\uc5b4, 1948\ub144\uc5d0 \uc774\ub974\ub7ec\uc11c\ub294 133\ub300\uac00 \uc6b4\ud589\ub418\uae30\uc5d0 \uc774\ub978\ub2e4.", "answer": "27\ub300", "sentence": "1947\ub144\uc5d0 \uc774\ub974\ub7ec\uc11c\ub294 \ud558\ub8e8 27\ub300 \ub9cc \uc6b4\ud589\ud560 \uc815\ub3c4\ub85c \ud63c\ub780\uc758 \uadf9\uc5d0 \ub2ec\ud588\ub2e4.", "paragraph_sentence": "\uc77c\uc81c \ud328\ub9dd\uacfc \ud568\uaed8 \uacbd\uc131\uc804\uae30\uc8fc\uc2dd\ud68c\uc0ac\uc758 \uc804\ucc28\uc0ac\uc5c5 \uc5ed\uc2dc \ub2e4\ub978 \uc0ac\uc5c5\uacfc \ud568\uaed8 \ud55c\uad6d\uc778\uc758 \uc190\uc73c\ub85c \uc6b4\uc601\ud558\uac8c \ub418\uc5c8\ub2e4. \uadf8\ub7ec\ub098 \ud55c\ud3b8\uc73c\ub85c, \uc804\uc2dc\ub3d9\uc6d0\uae30\uc758 \ud639\uc0ac\uc640 \uad11\ubcf5\uc758 \uae30\uc068\uc5d0 \ud658\ud638\ud558\ub294 \uc2dc\ubbfc\ub4e4\uc758 \ub09c\ub3d9\uc73c\ub85c \uc778\ud574 \uc804\ucc28\ub294 \uc218\ub09c\uc744 \uacaa\uac8c \ub418\uc5c8\ub2e4. \ud574\ubc29 \ud6c4 \ubcf4\ub984 \ub3d9\uc548 \ucc28\ub7c9\uc758 \uac00\ub3d9\uc0c1\ud0dc\ub294 101\ub300\uc5d0\uc11c 59\ub300\uae4c\uc9c0 \uae09\uac10\ud558\uae30\uc5d0 \uc774\ub974\ub800\ub2e4. \ub610\ud55c, \uc11c\uc6b8\uc2dc\ub294 \uadc0\uad6d\ub3d9\ud3ec\uc640 \ubd81\ud55c\uc73c\ub85c\ubd80\ud130\uc758 \uc6d4\ub0a8\uc790 \uc99d\uac00\ub85c \ud06c\uac8c \ubc00\ub3c4\uac00 \uc99d\uac00\ud574\uc11c, \uc804\ucc28\uc6b4\ud589\uc740 \uac00\uc77c\uce35 \uac00\ud639\ud55c \uc0c1\ud669\uc774\uc5c8\ub2e4. \uc5ec\uae30\uc5d0 \uc88c\uc6b0\uc775\uc758 \ub300\ub9bd\uc5d0 \ub530\ub978 \ud63c\ub780\uc73c\ub85c \ub178\ub3d9\uc870\ud569\uc758 \ud30c\uc5c5, \ud0dc\uc5c5, \uc0ac\ubcf4\ud0c0\uc9c0 \ub4f1\uc774 \uc18d\ucd9c\ud558\uc5ec 1947\ub144\uc5d0 \uc774\ub974\ub7ec\uc11c\ub294 \ud558\ub8e8 27\ub300 \ub9cc \uc6b4\ud589\ud560 \uc815\ub3c4\ub85c \ud63c\ub780\uc758 \uadf9\uc5d0 \ub2ec\ud588\ub2e4. \uc774\ud6c4, \ub2e8\uccb4\ud611\uc57d\ub4f1\uc744 \ud1b5\ud574 \uc0c1\ud669\uc774 \ud638\uc804\ub418\uba74\uc11c 27\ub300\ub97c \uae30\uc810\uc73c\ub85c \ub2e4\uc2dc \uac00\ub3d9\ub960\uc774 \ud68c\ubcf5\ub418\uc5b4, 1948\ub144\uc5d0 \uc774\ub974\ub7ec\uc11c\ub294 133\ub300\uac00 \uc6b4\ud589\ub418\uae30\uc5d0 \uc774\ub978\ub2e4.", "paragraph_answer": "\uc77c\uc81c \ud328\ub9dd\uacfc \ud568\uaed8 \uacbd\uc131\uc804\uae30\uc8fc\uc2dd\ud68c\uc0ac\uc758 \uc804\ucc28\uc0ac\uc5c5 \uc5ed\uc2dc \ub2e4\ub978 \uc0ac\uc5c5\uacfc \ud568\uaed8 \ud55c\uad6d\uc778\uc758 \uc190\uc73c\ub85c \uc6b4\uc601\ud558\uac8c \ub418\uc5c8\ub2e4. \uadf8\ub7ec\ub098 \ud55c\ud3b8\uc73c\ub85c, \uc804\uc2dc\ub3d9\uc6d0\uae30\uc758 \ud639\uc0ac\uc640 \uad11\ubcf5\uc758 \uae30\uc068\uc5d0 \ud658\ud638\ud558\ub294 \uc2dc\ubbfc\ub4e4\uc758 \ub09c\ub3d9\uc73c\ub85c \uc778\ud574 \uc804\ucc28\ub294 \uc218\ub09c\uc744 \uacaa\uac8c \ub418\uc5c8\ub2e4. \ud574\ubc29 \ud6c4 \ubcf4\ub984 \ub3d9\uc548 \ucc28\ub7c9\uc758 \uac00\ub3d9\uc0c1\ud0dc\ub294 101\ub300\uc5d0\uc11c 59\ub300\uae4c\uc9c0 \uae09\uac10\ud558\uae30\uc5d0 \uc774\ub974\ub800\ub2e4. \ub610\ud55c, \uc11c\uc6b8\uc2dc\ub294 \uadc0\uad6d\ub3d9\ud3ec\uc640 \ubd81\ud55c\uc73c\ub85c\ubd80\ud130\uc758 \uc6d4\ub0a8\uc790 \uc99d\uac00\ub85c \ud06c\uac8c \ubc00\ub3c4\uac00 \uc99d\uac00\ud574\uc11c, \uc804\ucc28\uc6b4\ud589\uc740 \uac00\uc77c\uce35 \uac00\ud639\ud55c \uc0c1\ud669\uc774\uc5c8\ub2e4. \uc5ec\uae30\uc5d0 \uc88c\uc6b0\uc775\uc758 \ub300\ub9bd\uc5d0 \ub530\ub978 \ud63c\ub780\uc73c\ub85c \ub178\ub3d9\uc870\ud569\uc758 \ud30c\uc5c5, \ud0dc\uc5c5, \uc0ac\ubcf4\ud0c0\uc9c0 \ub4f1\uc774 \uc18d\ucd9c\ud558\uc5ec 1947\ub144\uc5d0 \uc774\ub974\ub7ec\uc11c\ub294 \ud558\ub8e8 27\ub300 \ub9cc \uc6b4\ud589\ud560 \uc815\ub3c4\ub85c \ud63c\ub780\uc758 \uadf9\uc5d0 \ub2ec\ud588\ub2e4. \uc774\ud6c4, \ub2e8\uccb4\ud611\uc57d\ub4f1\uc744 \ud1b5\ud574 \uc0c1\ud669\uc774 \ud638\uc804\ub418\uba74\uc11c 27\ub300\ub97c \uae30\uc810\uc73c\ub85c \ub2e4\uc2dc \uac00\ub3d9\ub960\uc774 \ud68c\ubcf5\ub418\uc5b4, 1948\ub144\uc5d0 \uc774\ub974\ub7ec\uc11c\ub294 133\ub300\uac00 \uc6b4\ud589\ub418\uae30\uc5d0 \uc774\ub978\ub2e4.", "sentence_answer": "1947\ub144\uc5d0 \uc774\ub974\ub7ec\uc11c\ub294 \ud558\ub8e8 27\ub300 \ub9cc \uc6b4\ud589\ud560 \uc815\ub3c4\ub85c \ud63c\ub780\uc758 \uadf9\uc5d0 \ub2ec\ud588\ub2e4.", "paragraph_id": "6517594-6-2", "paragraph_question": "question: \ud30c\uc5c5, \ud0dc\uc5c5, \uc0ac\ubcf4\ud0c0\uc9c0 \ub4f1\uc73c\ub85c 1947\ub144 \ud558\ub8e8 \uc6b4\ud589\uc5d0 \ub098\uc11c\ub294 \uc804\ucc28 \uc218\ub294?, context: \uc77c\uc81c \ud328\ub9dd\uacfc \ud568\uaed8 \uacbd\uc131\uc804\uae30\uc8fc\uc2dd\ud68c\uc0ac\uc758 \uc804\ucc28\uc0ac\uc5c5 \uc5ed\uc2dc \ub2e4\ub978 \uc0ac\uc5c5\uacfc \ud568\uaed8 \ud55c\uad6d\uc778\uc758 \uc190\uc73c\ub85c \uc6b4\uc601\ud558\uac8c \ub418\uc5c8\ub2e4. \uadf8\ub7ec\ub098 \ud55c\ud3b8\uc73c\ub85c, \uc804\uc2dc\ub3d9\uc6d0\uae30\uc758 \ud639\uc0ac\uc640 \uad11\ubcf5\uc758 \uae30\uc068\uc5d0 \ud658\ud638\ud558\ub294 \uc2dc\ubbfc\ub4e4\uc758 \ub09c\ub3d9\uc73c\ub85c \uc778\ud574 \uc804\ucc28\ub294 \uc218\ub09c\uc744 \uacaa\uac8c \ub418\uc5c8\ub2e4. \ud574\ubc29 \ud6c4 \ubcf4\ub984 \ub3d9\uc548 \ucc28\ub7c9\uc758 \uac00\ub3d9\uc0c1\ud0dc\ub294 101\ub300\uc5d0\uc11c 59\ub300\uae4c\uc9c0 \uae09\uac10\ud558\uae30\uc5d0 \uc774\ub974\ub800\ub2e4. \ub610\ud55c, \uc11c\uc6b8\uc2dc\ub294 \uadc0\uad6d\ub3d9\ud3ec\uc640 \ubd81\ud55c\uc73c\ub85c\ubd80\ud130\uc758 \uc6d4\ub0a8\uc790 \uc99d\uac00\ub85c \ud06c\uac8c \ubc00\ub3c4\uac00 \uc99d\uac00\ud574\uc11c, \uc804\ucc28\uc6b4\ud589\uc740 \uac00\uc77c\uce35 \uac00\ud639\ud55c \uc0c1\ud669\uc774\uc5c8\ub2e4. \uc5ec\uae30\uc5d0 \uc88c\uc6b0\uc775\uc758 \ub300\ub9bd\uc5d0 \ub530\ub978 \ud63c\ub780\uc73c\ub85c \ub178\ub3d9\uc870\ud569\uc758 \ud30c\uc5c5, \ud0dc\uc5c5, \uc0ac\ubcf4\ud0c0\uc9c0 \ub4f1\uc774 \uc18d\ucd9c\ud558\uc5ec 1947\ub144\uc5d0 \uc774\ub974\ub7ec\uc11c\ub294 \ud558\ub8e8 27\ub300\ub9cc \uc6b4\ud589\ud560 \uc815\ub3c4\ub85c \ud63c\ub780\uc758 \uadf9\uc5d0 \ub2ec\ud588\ub2e4. \uc774\ud6c4, \ub2e8\uccb4\ud611\uc57d\ub4f1\uc744 \ud1b5\ud574 \uc0c1\ud669\uc774 \ud638\uc804\ub418\uba74\uc11c 27\ub300\ub97c \uae30\uc810\uc73c\ub85c \ub2e4\uc2dc \uac00\ub3d9\ub960\uc774 \ud68c\ubcf5\ub418\uc5b4, 1948\ub144\uc5d0 \uc774\ub974\ub7ec\uc11c\ub294 133\ub300\uac00 \uc6b4\ud589\ub418\uae30\uc5d0 \uc774\ub978\ub2e4."} {"question": "\ubcc4\uc758 \ucee4\ube44\uc758 \ud734\uc2dd\uacf5\uac04\uc5d0 \ub098\uc624\ub294 \uce74\ud53c \uc5d0\uc13c\uc2a4 \uc911 \ud558\ub098\ub294 \uaf2d \ubb34\uc5c7\uc785\ub2c8\uae4c?", "paragraph": "\ubb3c\ub860 \uc774\uac83\ub3c4 EX \ubaa8\ub4dc \ubc84\uc804\uc774 \ub530\ub85c \uc788\ub2e4. \ub2e4\ub9cc \uccb4\ub825 \uc0ac\uc591\uc774 EX \ubaa8\ub4dc\ucc98\ub7fc \uc808\ubc18 \uc218\uc900\uc774 \uc544\ub2cc \uba54\uc778\ubaa8\ub4dc \ub9cc\ud07c\uc774\uba70, \ud68c\ubcf5\uc218\ub2e8\ub3c4 3\uac1c\ubfd0\uc774\uc9c0\ub9cc \ub9e5\uc2ec\ud1a0\ub9c8\ud1a0\uac00 \uc8fc\uc5b4\uc9c4\ub2e4. \ub300\uc2e0 \ud734\uc2dd\uacf5\uac04\uc5d0\uc11c \uc8fc\uc5b4\uc9c0\ub294 \uce74\ud53c \uc5d0\uc13c\uc2a4\uc911 \ud558\ub098\uac00 \ubc18\ub4dc\uc2dc \uc2ac\ub9bd\uc778\uac74 \uadf8\ub300\ub85c\ub2e4. \uc2a4\ud14c\uc774\uc9c0\uc218\ub294 HR-D3(\uba54\ud0c8 \uc81c\ub808\ub110 2\ucc28\uc804\uacfc 3\ucc28\uc804)\uc2a4\ud14c\uc774\uc9c0\uc640 \uac24\ub7ed\ud2f1 \ub098\uc774\ud2b8(\ub9e4\uc6b0 \ud76c\uadc0)\uc2a4\ud14c\uc774\uc9c0\uac00 \ucd94\uac00\ub418\uc5b4 \ucd1d 15\uc2a4\ud14c\uc774\uc9c0. 10\ubc88\uc9f8 \uae4c\uc9c0\ub294 \ub79c\ub364\uc73c\ub85c \ub4f1\uc7a5\ud558\uace0 \uadf8 \ub4a4 5\uc2a4\ud14c\uc774\uc9c0\ub294 \ub85c\uc544 - HR-D3 - \uac24\ub7ed\ud2f1 \ub098\uc774\ud2b8 - \ub9c8\ubc84\ub85c\uc544 - \ub9c8\ubc84\ub85c\uc544 \uc18c\uc6b8\ub85c \uace0\uc815\ub418\uba70, \uc2a4\ud14c\uc774\uc9c0\ub85c \uc785\uc7a5\ud558\ub294 \ubb38\uc774 \uc774\uacf5\uac04 \ud3ec\ud138\ub85c \ubc14\ub010\ub2e4. \uc989 \ud1a0\ub9c8\ud1a0 \uc0c1\uc790\ub97c \uc4f8 \uc218 \uc5c6\ub2e4. \uadf8 \uc678\uc758 \ucc28\uc774\uc810\uc740 \uc911\ubcf4\uc2a4 \ub7ec\uc2dc \uc2a4\ud14c\uc774\uc9c0\uc5d0\uc11c \ub9c8\uc9c0\ub9c9 2\ub9c8\ub9ac\uac00 \ub3d9\uc2dc\uc5d0 \ub098\uc628\ub2e4. \uc288\ud37c \ub2a5\ub825\uc740 \ub098\uc624\uc9c0 \uc54a\ub294\ub2e4. \ub05d", "answer": "\uc2ac\ub9bd", "sentence": "\ub300\uc2e0 \ud734\uc2dd\uacf5\uac04\uc5d0\uc11c \uc8fc\uc5b4\uc9c0\ub294 \uce74\ud53c \uc5d0\uc13c\uc2a4\uc911 \ud558\ub098\uac00 \ubc18\ub4dc\uc2dc \uc2ac\ub9bd \uc778\uac74 \uadf8\ub300\ub85c\ub2e4.", "paragraph_sentence": "\ubb3c\ub860 \uc774\uac83\ub3c4 EX \ubaa8\ub4dc \ubc84\uc804\uc774 \ub530\ub85c \uc788\ub2e4. \ub2e4\ub9cc \uccb4\ub825 \uc0ac\uc591\uc774 EX \ubaa8\ub4dc\ucc98\ub7fc \uc808\ubc18 \uc218\uc900\uc774 \uc544\ub2cc \uba54\uc778\ubaa8\ub4dc \ub9cc\ud07c\uc774\uba70, \ud68c\ubcf5\uc218\ub2e8\ub3c4 3\uac1c\ubfd0\uc774\uc9c0\ub9cc \ub9e5\uc2ec\ud1a0\ub9c8\ud1a0\uac00 \uc8fc\uc5b4\uc9c4\ub2e4. \ub300\uc2e0 \ud734\uc2dd\uacf5\uac04\uc5d0\uc11c \uc8fc\uc5b4\uc9c0\ub294 \uce74\ud53c \uc5d0\uc13c\uc2a4\uc911 \ud558\ub098\uac00 \ubc18\ub4dc\uc2dc \uc2ac\ub9bd \uc778\uac74 \uadf8\ub300\ub85c\ub2e4. \uc2a4\ud14c\uc774\uc9c0\uc218\ub294 HR-D3(\uba54\ud0c8 \uc81c\ub808\ub110 2\ucc28\uc804\uacfc 3\ucc28\uc804)\uc2a4\ud14c\uc774\uc9c0\uc640 \uac24\ub7ed\ud2f1 \ub098\uc774\ud2b8(\ub9e4\uc6b0 \ud76c\uadc0)\uc2a4\ud14c\uc774\uc9c0\uac00 \ucd94\uac00\ub418\uc5b4 \ucd1d 15\uc2a4\ud14c\uc774\uc9c0. 10\ubc88\uc9f8 \uae4c\uc9c0\ub294 \ub79c\ub364\uc73c\ub85c \ub4f1\uc7a5\ud558\uace0 \uadf8 \ub4a4 5\uc2a4\ud14c\uc774\uc9c0\ub294 \ub85c\uc544 - HR-D3 - \uac24\ub7ed\ud2f1 \ub098\uc774\ud2b8 - \ub9c8\ubc84\ub85c\uc544 - \ub9c8\ubc84\ub85c\uc544 \uc18c\uc6b8\ub85c \uace0\uc815\ub418\uba70, \uc2a4\ud14c\uc774\uc9c0\ub85c \uc785\uc7a5\ud558\ub294 \ubb38\uc774 \uc774\uacf5\uac04 \ud3ec\ud138\ub85c \ubc14\ub010\ub2e4. \uc989 \ud1a0\ub9c8\ud1a0 \uc0c1\uc790\ub97c \uc4f8 \uc218 \uc5c6\ub2e4. \uadf8 \uc678\uc758 \ucc28\uc774\uc810\uc740 \uc911\ubcf4\uc2a4 \ub7ec\uc2dc \uc2a4\ud14c\uc774\uc9c0\uc5d0\uc11c \ub9c8\uc9c0\ub9c9 2\ub9c8\ub9ac\uac00 \ub3d9\uc2dc\uc5d0 \ub098\uc628\ub2e4. \uc288\ud37c \ub2a5\ub825\uc740 \ub098\uc624\uc9c0 \uc54a\ub294\ub2e4. \ub05d", "paragraph_answer": "\ubb3c\ub860 \uc774\uac83\ub3c4 EX \ubaa8\ub4dc \ubc84\uc804\uc774 \ub530\ub85c \uc788\ub2e4. \ub2e4\ub9cc \uccb4\ub825 \uc0ac\uc591\uc774 EX \ubaa8\ub4dc\ucc98\ub7fc \uc808\ubc18 \uc218\uc900\uc774 \uc544\ub2cc \uba54\uc778\ubaa8\ub4dc \ub9cc\ud07c\uc774\uba70, \ud68c\ubcf5\uc218\ub2e8\ub3c4 3\uac1c\ubfd0\uc774\uc9c0\ub9cc \ub9e5\uc2ec\ud1a0\ub9c8\ud1a0\uac00 \uc8fc\uc5b4\uc9c4\ub2e4. \ub300\uc2e0 \ud734\uc2dd\uacf5\uac04\uc5d0\uc11c \uc8fc\uc5b4\uc9c0\ub294 \uce74\ud53c \uc5d0\uc13c\uc2a4\uc911 \ud558\ub098\uac00 \ubc18\ub4dc\uc2dc \uc2ac\ub9bd \uc778\uac74 \uadf8\ub300\ub85c\ub2e4. \uc2a4\ud14c\uc774\uc9c0\uc218\ub294 HR-D3(\uba54\ud0c8 \uc81c\ub808\ub110 2\ucc28\uc804\uacfc 3\ucc28\uc804)\uc2a4\ud14c\uc774\uc9c0\uc640 \uac24\ub7ed\ud2f1 \ub098\uc774\ud2b8(\ub9e4\uc6b0 \ud76c\uadc0)\uc2a4\ud14c\uc774\uc9c0\uac00 \ucd94\uac00\ub418\uc5b4 \ucd1d 15\uc2a4\ud14c\uc774\uc9c0. 10\ubc88\uc9f8 \uae4c\uc9c0\ub294 \ub79c\ub364\uc73c\ub85c \ub4f1\uc7a5\ud558\uace0 \uadf8 \ub4a4 5\uc2a4\ud14c\uc774\uc9c0\ub294 \ub85c\uc544 - HR-D3 - \uac24\ub7ed\ud2f1 \ub098\uc774\ud2b8 - \ub9c8\ubc84\ub85c\uc544 - \ub9c8\ubc84\ub85c\uc544 \uc18c\uc6b8\ub85c \uace0\uc815\ub418\uba70, \uc2a4\ud14c\uc774\uc9c0\ub85c \uc785\uc7a5\ud558\ub294 \ubb38\uc774 \uc774\uacf5\uac04 \ud3ec\ud138\ub85c \ubc14\ub010\ub2e4. \uc989 \ud1a0\ub9c8\ud1a0 \uc0c1\uc790\ub97c \uc4f8 \uc218 \uc5c6\ub2e4. \uadf8 \uc678\uc758 \ucc28\uc774\uc810\uc740 \uc911\ubcf4\uc2a4 \ub7ec\uc2dc \uc2a4\ud14c\uc774\uc9c0\uc5d0\uc11c \ub9c8\uc9c0\ub9c9 2\ub9c8\ub9ac\uac00 \ub3d9\uc2dc\uc5d0 \ub098\uc628\ub2e4. \uc288\ud37c \ub2a5\ub825\uc740 \ub098\uc624\uc9c0 \uc54a\ub294\ub2e4. \ub05d", "sentence_answer": "\ub300\uc2e0 \ud734\uc2dd\uacf5\uac04\uc5d0\uc11c \uc8fc\uc5b4\uc9c0\ub294 \uce74\ud53c \uc5d0\uc13c\uc2a4\uc911 \ud558\ub098\uac00 \ubc18\ub4dc\uc2dc \uc2ac\ub9bd \uc778\uac74 \uadf8\ub300\ub85c\ub2e4.", "paragraph_id": "6508600-4-0", "paragraph_question": "question: \ubcc4\uc758 \ucee4\ube44\uc758 \ud734\uc2dd\uacf5\uac04\uc5d0 \ub098\uc624\ub294 \uce74\ud53c \uc5d0\uc13c\uc2a4 \uc911 \ud558\ub098\ub294 \uaf2d \ubb34\uc5c7\uc785\ub2c8\uae4c?, context: \ubb3c\ub860 \uc774\uac83\ub3c4 EX \ubaa8\ub4dc \ubc84\uc804\uc774 \ub530\ub85c \uc788\ub2e4. \ub2e4\ub9cc \uccb4\ub825 \uc0ac\uc591\uc774 EX \ubaa8\ub4dc\ucc98\ub7fc \uc808\ubc18 \uc218\uc900\uc774 \uc544\ub2cc \uba54\uc778\ubaa8\ub4dc \ub9cc\ud07c\uc774\uba70, \ud68c\ubcf5\uc218\ub2e8\ub3c4 3\uac1c\ubfd0\uc774\uc9c0\ub9cc \ub9e5\uc2ec\ud1a0\ub9c8\ud1a0\uac00 \uc8fc\uc5b4\uc9c4\ub2e4. \ub300\uc2e0 \ud734\uc2dd\uacf5\uac04\uc5d0\uc11c \uc8fc\uc5b4\uc9c0\ub294 \uce74\ud53c \uc5d0\uc13c\uc2a4\uc911 \ud558\ub098\uac00 \ubc18\ub4dc\uc2dc \uc2ac\ub9bd\uc778\uac74 \uadf8\ub300\ub85c\ub2e4. \uc2a4\ud14c\uc774\uc9c0\uc218\ub294 HR-D3(\uba54\ud0c8 \uc81c\ub808\ub110 2\ucc28\uc804\uacfc 3\ucc28\uc804)\uc2a4\ud14c\uc774\uc9c0\uc640 \uac24\ub7ed\ud2f1 \ub098\uc774\ud2b8(\ub9e4\uc6b0 \ud76c\uadc0)\uc2a4\ud14c\uc774\uc9c0\uac00 \ucd94\uac00\ub418\uc5b4 \ucd1d 15\uc2a4\ud14c\uc774\uc9c0. 10\ubc88\uc9f8 \uae4c\uc9c0\ub294 \ub79c\ub364\uc73c\ub85c \ub4f1\uc7a5\ud558\uace0 \uadf8 \ub4a4 5\uc2a4\ud14c\uc774\uc9c0\ub294 \ub85c\uc544 - HR-D3 - \uac24\ub7ed\ud2f1 \ub098\uc774\ud2b8 - \ub9c8\ubc84\ub85c\uc544 - \ub9c8\ubc84\ub85c\uc544 \uc18c\uc6b8\ub85c \uace0\uc815\ub418\uba70, \uc2a4\ud14c\uc774\uc9c0\ub85c \uc785\uc7a5\ud558\ub294 \ubb38\uc774 \uc774\uacf5\uac04 \ud3ec\ud138\ub85c \ubc14\ub010\ub2e4. \uc989 \ud1a0\ub9c8\ud1a0 \uc0c1\uc790\ub97c \uc4f8 \uc218 \uc5c6\ub2e4. \uadf8 \uc678\uc758 \ucc28\uc774\uc810\uc740 \uc911\ubcf4\uc2a4 \ub7ec\uc2dc \uc2a4\ud14c\uc774\uc9c0\uc5d0\uc11c \ub9c8\uc9c0\ub9c9 2\ub9c8\ub9ac\uac00 \ub3d9\uc2dc\uc5d0 \ub098\uc628\ub2e4. \uc288\ud37c \ub2a5\ub825\uc740 \ub098\uc624\uc9c0 \uc54a\ub294\ub2e4. \ub05d"} {"question": "\ub3c8 \ud6c4\uc548 \ub370 \uc544\uc6b0\uc2a4\ud2b8\ub9ac\uc544 \uc9c1\uc811 \ud1b5\uc194\ud55c \ud568\ub300 \uc218\ub294?", "paragraph": "\uae30\ub3c5\uad50 \uc5f0\ud569 \ud568\ub300\ub294 \ubd81\ucabd\uc5d0\uc11c \ub0a8\ucabd\uae4c\uc9c0 4\uac1c\uc758 \ubd84\ub300\ub85c \ub098\ub258\uc5c8\ub2e4. \ud574\uc548\uacfc \uadfc\uc811\ud55c \ubd81\ucabd\uc758 \ub044\ud2b8\uba38\ub9ac\ub294 53\ucc99\uc758 \uac24\ub9ac\uc120\uc774 \uc88c\uc775\uc744 \uad6c\uc131\ud558\uc600\ub2e4. \ubca0\ub124\uce58\uc544\uc758 \uc544\uace0\uc2a4\ud2f0\ub178 \ubc14\ub974\ubc14\ub9ac\uace0\uacfc \ub9c8\ub974\ucf54 \ud034\ub9ac\ub2c8\uac00 \uc548\ud1a0\ub2c8\uc624 \ub2e4 \uce74\ub0a0\ub808\uc758 \uc9c0\uc6d0\uc744 \ubc1b\uc73c\uba74\uc11c \uc8fc\ub85c \ubca0\ub124\uce58\uc544 \ud568\ub300\ub85c \uc774\ub8e8\uc5b4\uc9c4 \uc88c\uc775\uc744 \ud1b5\uc194\ud558\uc600\ub2e4. \uc911\uc559\uc5d0\ub294 \ub3c8 \ud6c4\uc548 \ub370 \uc544\uc6b0\uc2a4\ud2b8\ub9ac\uc544\uac00 \uc9c1\uc811 \uae30\ud568 \ub808\uc54c\uc5d0\uc11c 62\ucc99\uc758 \ud568\ub300\ub97c \ud1b5\uc194\ud588\ub294\ub370, \uc5ec\uae30\uc5d0\ub294 \ud6d7\ub0a0 \ubca0\ub124\uce58\uc544\uc758 \ub3c4\uc81c\uac00 \ub41c \uc138\ubc14\uc2a4\ud2f0\uc544\ub178 \ubca0\ub2c8\uc5d0\ub974\uc640 \ub9c8\ub974\uce78\ud1a0\ub2c8\uc624 \ucf5c\ub85c\ub098\uac00 \uadf8\ub97c \ubcf4\uc88c\ud588\ub2e4. \uc6b0\uc775\uc5d0\uc11c\ub294 \uc720\uba85\ud55c \uc548\ub4dc\ub808\uc544 \ub3c4\ub9ac\uc544\uc758 \uc870\uce74\uc190\uc790\uc778 \uc81c\ub178\ubc14\uc758 \uc9c0\uc624\ubc18\ub2c8 \uc548\ub4dc\ub808\uc544 \ub3c4\ub9ac\uc544\uac00 \ub2e4\ub978 53\ucc99\uc758 \uc9c0\ud718\ub97c \ub9e1\uc558\ub2e4. \uc88c\uc775\uacfc \uc911\uc545 \uc6b0\uc775\uc758 \uc55e\ubd80\ubd84\uc5d0\ub294 \ub450 \ucc99\uc758 \uac24\ub9ac\uc5b4\uc2a4\uac00 \ubc30\uce58\ub418\uc5b4 \uc788\uc5c8\ub294\ub370, \ubbf8\uac94 \ub370 \uc138\ub974\ubc18\ud14c\uc2a4 \uc5d0 \ub530\ub974\uba74, \uc774\ub807\uac8c \ubc30\uce58\ud55c \ubaa9\uc801\uc740 \uc624\uc2a4\ub9cc\uad70\uc758 \ud568\uc120\uc774 \uc791\uc740 \ubc30\ub85c \uae30\uc2b5\ud558\uc5ec, \uae30\ub3c5\uad50 \ud568\uc120\uc5d0 \uc62c\ub77c\ud0c0 \uacf5\uaca9\ud558\ub294 \uac83\uc744 \ubc29\uc5b4\ud558\uae30 \uc704\ud574\uc11c\uc600\ub2e4. \ub098\uba38\uc9c0 \ud55c \ubd80\ub300\ub294 \uc911\uc559\ubd80\ub300\uc758 \ud6c4\ubc29\uc5d0 \uc608\ube44\ub300\ub85c\uc368 \ubc30\uce58\ub418\uc5c8\ub294\ub370, \ud544\uc694\ud55c \uacf3\uc5d0 \uc5b8\uc81c\ub4e0\uc9c0 \ub2ec\ub824\uac00 \uc9c0\uc6d0\uc744 \ud558\uae30 \uc704\ud574\uc11c\uc600\ub2e4. \uc774 \ud6c4\ubc29 \ubd80\ub300\ub294 38\ucc99\uc758 \uac24\ub9ac\uc120\uc73c\ub85c \ub418\uc5b4 \uc788\uc5c8\ub294\ub370, 30\ucc99\uc740 \uc911\uc559\ubd80\ubd84\uc758 \ud6c4\uc704\uc5d0 \ubc30\uce58\ub418\uc5b4 \uc54c\ubc14\ub85c \ub370 \ubc14\uc794\uc774 \ub2f4\ub2f9\ud558\uc600\uace0, 4\ucc99\uc740 \uac01 \ub0a0\uac1c\uc758 \ud6c4\ubc29\uc5d0 \ubc30\uce58\ub418\uc5b4 \uc788\uc5c8\ub2e4. \uc815\ucc30\ubd80\ub300\ub3c4 \uad6c\uc131\ub418\uc5c8\ub294\ub370, \uc6b0\uc775\uc5d0\uc11c \ub450 \ucc99\uacfc \ud6c4\ubc29\ubd80\ub300\uc5d0\uc11c 6\ucc99\uc758 \uac24\ub9ac\uc120\uc774 \ucc28\ucd9c\ub418\uc5c8\ub2e4. \uae30\ub3c5\uad50 \ud568\ub300\ub294 \ub290\ub9ac\uac8c \uc2a4\ud06c\ub85c\ud30c \uc9c0\uc810\uc758 \uc8fc\ubcc0\uc744 \ub3cc\uc558\uace0, \ud574\uc548\uac00\uc5d0\uc11c \ub5a8\uc5b4\uc9c4 \uacf3\uc5d0 \uc788\ub358 \ub3c4\ub9ac\uc544\uac00 \uc9c0\ud718\ud558\ub294 \uc6b0\uc775\uc740 \uc804\ud22c\ub97c \uc2dc\uc791\ud558\ub294 \uac8c \ub2a6\uc5b4 \uc6b0\uc775\uc758 \uac24\ub9ac\uc5b4\uc2a4\ub294 \uc790\ub9ac\ub97c \uc7a1\uc9c0 \ubabb\ud588\ub2e4.", "answer": "62\ucc99", "sentence": "\uc911\uc559\uc5d0\ub294 \ub3c8 \ud6c4\uc548 \ub370 \uc544\uc6b0\uc2a4\ud2b8\ub9ac\uc544\uac00 \uc9c1\uc811 \uae30\ud568 \ub808\uc54c\uc5d0\uc11c 62\ucc99 \uc758 \ud568\ub300\ub97c \ud1b5\uc194\ud588\ub294\ub370, \uc5ec\uae30\uc5d0\ub294 \ud6d7\ub0a0 \ubca0\ub124\uce58\uc544\uc758 \ub3c4\uc81c\uac00 \ub41c \uc138\ubc14\uc2a4\ud2f0\uc544\ub178 \ubca0\ub2c8\uc5d0\ub974\uc640 \ub9c8\ub974\uce78\ud1a0\ub2c8\uc624 \ucf5c\ub85c\ub098\uac00 \uadf8\ub97c \ubcf4\uc88c\ud588\ub2e4.", "paragraph_sentence": "\uae30\ub3c5\uad50 \uc5f0\ud569 \ud568\ub300\ub294 \ubd81\ucabd\uc5d0\uc11c \ub0a8\ucabd\uae4c\uc9c0 4\uac1c\uc758 \ubd84\ub300\ub85c \ub098\ub258\uc5c8\ub2e4. \ud574\uc548\uacfc \uadfc\uc811\ud55c \ubd81\ucabd\uc758 \ub044\ud2b8\uba38\ub9ac\ub294 53\ucc99\uc758 \uac24\ub9ac\uc120\uc774 \uc88c\uc775\uc744 \uad6c\uc131\ud558\uc600\ub2e4. \ubca0\ub124\uce58\uc544\uc758 \uc544\uace0\uc2a4\ud2f0\ub178 \ubc14\ub974\ubc14\ub9ac\uace0\uacfc \ub9c8\ub974\ucf54 \ud034\ub9ac\ub2c8\uac00 \uc548\ud1a0\ub2c8\uc624 \ub2e4 \uce74\ub0a0\ub808\uc758 \uc9c0\uc6d0\uc744 \ubc1b\uc73c\uba74\uc11c \uc8fc\ub85c \ubca0\ub124\uce58\uc544 \ud568\ub300\ub85c \uc774\ub8e8\uc5b4\uc9c4 \uc88c\uc775\uc744 \ud1b5\uc194\ud558\uc600\ub2e4. \uc911\uc559\uc5d0\ub294 \ub3c8 \ud6c4\uc548 \ub370 \uc544\uc6b0\uc2a4\ud2b8\ub9ac\uc544\uac00 \uc9c1\uc811 \uae30\ud568 \ub808\uc54c\uc5d0\uc11c 62\ucc99 \uc758 \ud568\ub300\ub97c \ud1b5\uc194\ud588\ub294\ub370, \uc5ec\uae30\uc5d0\ub294 \ud6d7\ub0a0 \ubca0\ub124\uce58\uc544\uc758 \ub3c4\uc81c\uac00 \ub41c \uc138\ubc14\uc2a4\ud2f0\uc544\ub178 \ubca0\ub2c8\uc5d0\ub974\uc640 \ub9c8\ub974\uce78\ud1a0\ub2c8\uc624 \ucf5c\ub85c\ub098\uac00 \uadf8\ub97c \ubcf4\uc88c\ud588\ub2e4. \uc6b0\uc775\uc5d0\uc11c\ub294 \uc720\uba85\ud55c \uc548\ub4dc\ub808\uc544 \ub3c4\ub9ac\uc544\uc758 \uc870\uce74\uc190\uc790\uc778 \uc81c\ub178\ubc14\uc758 \uc9c0\uc624\ubc18\ub2c8 \uc548\ub4dc\ub808\uc544 \ub3c4\ub9ac\uc544\uac00 \ub2e4\ub978 53\ucc99\uc758 \uc9c0\ud718\ub97c \ub9e1\uc558\ub2e4. \uc88c\uc775\uacfc \uc911\uc545 \uc6b0\uc775\uc758 \uc55e\ubd80\ubd84\uc5d0\ub294 \ub450 \ucc99\uc758 \uac24\ub9ac\uc5b4\uc2a4\uac00 \ubc30\uce58\ub418\uc5b4 \uc788\uc5c8\ub294\ub370, \ubbf8\uac94 \ub370 \uc138\ub974\ubc18\ud14c\uc2a4 \uc5d0 \ub530\ub974\uba74, \uc774\ub807\uac8c \ubc30\uce58\ud55c \ubaa9\uc801\uc740 \uc624\uc2a4\ub9cc\uad70\uc758 \ud568\uc120\uc774 \uc791\uc740 \ubc30\ub85c \uae30\uc2b5\ud558\uc5ec, \uae30\ub3c5\uad50 \ud568\uc120\uc5d0 \uc62c\ub77c\ud0c0 \uacf5\uaca9\ud558\ub294 \uac83\uc744 \ubc29\uc5b4\ud558\uae30 \uc704\ud574\uc11c\uc600\ub2e4. \ub098\uba38\uc9c0 \ud55c \ubd80\ub300\ub294 \uc911\uc559\ubd80\ub300\uc758 \ud6c4\ubc29\uc5d0 \uc608\ube44\ub300\ub85c\uc368 \ubc30\uce58\ub418\uc5c8\ub294\ub370, \ud544\uc694\ud55c \uacf3\uc5d0 \uc5b8\uc81c\ub4e0\uc9c0 \ub2ec\ub824\uac00 \uc9c0\uc6d0\uc744 \ud558\uae30 \uc704\ud574\uc11c\uc600\ub2e4. \uc774 \ud6c4\ubc29 \ubd80\ub300\ub294 38\ucc99\uc758 \uac24\ub9ac\uc120\uc73c\ub85c \ub418\uc5b4 \uc788\uc5c8\ub294\ub370, 30\ucc99\uc740 \uc911\uc559\ubd80\ubd84\uc758 \ud6c4\uc704\uc5d0 \ubc30\uce58\ub418\uc5b4 \uc54c\ubc14\ub85c \ub370 \ubc14\uc794\uc774 \ub2f4\ub2f9\ud558\uc600\uace0, 4\ucc99\uc740 \uac01 \ub0a0\uac1c\uc758 \ud6c4\ubc29\uc5d0 \ubc30\uce58\ub418\uc5b4 \uc788\uc5c8\ub2e4. \uc815\ucc30\ubd80\ub300\ub3c4 \uad6c\uc131\ub418\uc5c8\ub294\ub370, \uc6b0\uc775\uc5d0\uc11c \ub450 \ucc99\uacfc \ud6c4\ubc29\ubd80\ub300\uc5d0\uc11c 6\ucc99\uc758 \uac24\ub9ac\uc120\uc774 \ucc28\ucd9c\ub418\uc5c8\ub2e4. \uae30\ub3c5\uad50 \ud568\ub300\ub294 \ub290\ub9ac\uac8c \uc2a4\ud06c\ub85c\ud30c \uc9c0\uc810\uc758 \uc8fc\ubcc0\uc744 \ub3cc\uc558\uace0, \ud574\uc548\uac00\uc5d0\uc11c \ub5a8\uc5b4\uc9c4 \uacf3\uc5d0 \uc788\ub358 \ub3c4\ub9ac\uc544\uac00 \uc9c0\ud718\ud558\ub294 \uc6b0\uc775\uc740 \uc804\ud22c\ub97c \uc2dc\uc791\ud558\ub294 \uac8c \ub2a6\uc5b4 \uc6b0\uc775\uc758 \uac24\ub9ac\uc5b4\uc2a4\ub294 \uc790\ub9ac\ub97c \uc7a1\uc9c0 \ubabb\ud588\ub2e4.", "paragraph_answer": "\uae30\ub3c5\uad50 \uc5f0\ud569 \ud568\ub300\ub294 \ubd81\ucabd\uc5d0\uc11c \ub0a8\ucabd\uae4c\uc9c0 4\uac1c\uc758 \ubd84\ub300\ub85c \ub098\ub258\uc5c8\ub2e4. \ud574\uc548\uacfc \uadfc\uc811\ud55c \ubd81\ucabd\uc758 \ub044\ud2b8\uba38\ub9ac\ub294 53\ucc99\uc758 \uac24\ub9ac\uc120\uc774 \uc88c\uc775\uc744 \uad6c\uc131\ud558\uc600\ub2e4. \ubca0\ub124\uce58\uc544\uc758 \uc544\uace0\uc2a4\ud2f0\ub178 \ubc14\ub974\ubc14\ub9ac\uace0\uacfc \ub9c8\ub974\ucf54 \ud034\ub9ac\ub2c8\uac00 \uc548\ud1a0\ub2c8\uc624 \ub2e4 \uce74\ub0a0\ub808\uc758 \uc9c0\uc6d0\uc744 \ubc1b\uc73c\uba74\uc11c \uc8fc\ub85c \ubca0\ub124\uce58\uc544 \ud568\ub300\ub85c \uc774\ub8e8\uc5b4\uc9c4 \uc88c\uc775\uc744 \ud1b5\uc194\ud558\uc600\ub2e4. \uc911\uc559\uc5d0\ub294 \ub3c8 \ud6c4\uc548 \ub370 \uc544\uc6b0\uc2a4\ud2b8\ub9ac\uc544\uac00 \uc9c1\uc811 \uae30\ud568 \ub808\uc54c\uc5d0\uc11c 62\ucc99 \uc758 \ud568\ub300\ub97c \ud1b5\uc194\ud588\ub294\ub370, \uc5ec\uae30\uc5d0\ub294 \ud6d7\ub0a0 \ubca0\ub124\uce58\uc544\uc758 \ub3c4\uc81c\uac00 \ub41c \uc138\ubc14\uc2a4\ud2f0\uc544\ub178 \ubca0\ub2c8\uc5d0\ub974\uc640 \ub9c8\ub974\uce78\ud1a0\ub2c8\uc624 \ucf5c\ub85c\ub098\uac00 \uadf8\ub97c \ubcf4\uc88c\ud588\ub2e4. \uc6b0\uc775\uc5d0\uc11c\ub294 \uc720\uba85\ud55c \uc548\ub4dc\ub808\uc544 \ub3c4\ub9ac\uc544\uc758 \uc870\uce74\uc190\uc790\uc778 \uc81c\ub178\ubc14\uc758 \uc9c0\uc624\ubc18\ub2c8 \uc548\ub4dc\ub808\uc544 \ub3c4\ub9ac\uc544\uac00 \ub2e4\ub978 53\ucc99\uc758 \uc9c0\ud718\ub97c \ub9e1\uc558\ub2e4. \uc88c\uc775\uacfc \uc911\uc545 \uc6b0\uc775\uc758 \uc55e\ubd80\ubd84\uc5d0\ub294 \ub450 \ucc99\uc758 \uac24\ub9ac\uc5b4\uc2a4\uac00 \ubc30\uce58\ub418\uc5b4 \uc788\uc5c8\ub294\ub370, \ubbf8\uac94 \ub370 \uc138\ub974\ubc18\ud14c\uc2a4 \uc5d0 \ub530\ub974\uba74, \uc774\ub807\uac8c \ubc30\uce58\ud55c \ubaa9\uc801\uc740 \uc624\uc2a4\ub9cc\uad70\uc758 \ud568\uc120\uc774 \uc791\uc740 \ubc30\ub85c \uae30\uc2b5\ud558\uc5ec, \uae30\ub3c5\uad50 \ud568\uc120\uc5d0 \uc62c\ub77c\ud0c0 \uacf5\uaca9\ud558\ub294 \uac83\uc744 \ubc29\uc5b4\ud558\uae30 \uc704\ud574\uc11c\uc600\ub2e4. \ub098\uba38\uc9c0 \ud55c \ubd80\ub300\ub294 \uc911\uc559\ubd80\ub300\uc758 \ud6c4\ubc29\uc5d0 \uc608\ube44\ub300\ub85c\uc368 \ubc30\uce58\ub418\uc5c8\ub294\ub370, \ud544\uc694\ud55c \uacf3\uc5d0 \uc5b8\uc81c\ub4e0\uc9c0 \ub2ec\ub824\uac00 \uc9c0\uc6d0\uc744 \ud558\uae30 \uc704\ud574\uc11c\uc600\ub2e4. \uc774 \ud6c4\ubc29 \ubd80\ub300\ub294 38\ucc99\uc758 \uac24\ub9ac\uc120\uc73c\ub85c \ub418\uc5b4 \uc788\uc5c8\ub294\ub370, 30\ucc99\uc740 \uc911\uc559\ubd80\ubd84\uc758 \ud6c4\uc704\uc5d0 \ubc30\uce58\ub418\uc5b4 \uc54c\ubc14\ub85c \ub370 \ubc14\uc794\uc774 \ub2f4\ub2f9\ud558\uc600\uace0, 4\ucc99\uc740 \uac01 \ub0a0\uac1c\uc758 \ud6c4\ubc29\uc5d0 \ubc30\uce58\ub418\uc5b4 \uc788\uc5c8\ub2e4. \uc815\ucc30\ubd80\ub300\ub3c4 \uad6c\uc131\ub418\uc5c8\ub294\ub370, \uc6b0\uc775\uc5d0\uc11c \ub450 \ucc99\uacfc \ud6c4\ubc29\ubd80\ub300\uc5d0\uc11c 6\ucc99\uc758 \uac24\ub9ac\uc120\uc774 \ucc28\ucd9c\ub418\uc5c8\ub2e4. \uae30\ub3c5\uad50 \ud568\ub300\ub294 \ub290\ub9ac\uac8c \uc2a4\ud06c\ub85c\ud30c \uc9c0\uc810\uc758 \uc8fc\ubcc0\uc744 \ub3cc\uc558\uace0, \ud574\uc548\uac00\uc5d0\uc11c \ub5a8\uc5b4\uc9c4 \uacf3\uc5d0 \uc788\ub358 \ub3c4\ub9ac\uc544\uac00 \uc9c0\ud718\ud558\ub294 \uc6b0\uc775\uc740 \uc804\ud22c\ub97c \uc2dc\uc791\ud558\ub294 \uac8c \ub2a6\uc5b4 \uc6b0\uc775\uc758 \uac24\ub9ac\uc5b4\uc2a4\ub294 \uc790\ub9ac\ub97c \uc7a1\uc9c0 \ubabb\ud588\ub2e4.", "sentence_answer": "\uc911\uc559\uc5d0\ub294 \ub3c8 \ud6c4\uc548 \ub370 \uc544\uc6b0\uc2a4\ud2b8\ub9ac\uc544\uac00 \uc9c1\uc811 \uae30\ud568 \ub808\uc54c\uc5d0\uc11c 62\ucc99 \uc758 \ud568\ub300\ub97c \ud1b5\uc194\ud588\ub294\ub370, \uc5ec\uae30\uc5d0\ub294 \ud6d7\ub0a0 \ubca0\ub124\uce58\uc544\uc758 \ub3c4\uc81c\uac00 \ub41c \uc138\ubc14\uc2a4\ud2f0\uc544\ub178 \ubca0\ub2c8\uc5d0\ub974\uc640 \ub9c8\ub974\uce78\ud1a0\ub2c8\uc624 \ucf5c\ub85c\ub098\uac00 \uadf8\ub97c \ubcf4\uc88c\ud588\ub2e4.", "paragraph_id": "6306274-3-0", "paragraph_question": "question: \ub3c8 \ud6c4\uc548 \ub370 \uc544\uc6b0\uc2a4\ud2b8\ub9ac\uc544 \uc9c1\uc811 \ud1b5\uc194\ud55c \ud568\ub300 \uc218\ub294?, context: \uae30\ub3c5\uad50 \uc5f0\ud569 \ud568\ub300\ub294 \ubd81\ucabd\uc5d0\uc11c \ub0a8\ucabd\uae4c\uc9c0 4\uac1c\uc758 \ubd84\ub300\ub85c \ub098\ub258\uc5c8\ub2e4. \ud574\uc548\uacfc \uadfc\uc811\ud55c \ubd81\ucabd\uc758 \ub044\ud2b8\uba38\ub9ac\ub294 53\ucc99\uc758 \uac24\ub9ac\uc120\uc774 \uc88c\uc775\uc744 \uad6c\uc131\ud558\uc600\ub2e4. \ubca0\ub124\uce58\uc544\uc758 \uc544\uace0\uc2a4\ud2f0\ub178 \ubc14\ub974\ubc14\ub9ac\uace0\uacfc \ub9c8\ub974\ucf54 \ud034\ub9ac\ub2c8\uac00 \uc548\ud1a0\ub2c8\uc624 \ub2e4 \uce74\ub0a0\ub808\uc758 \uc9c0\uc6d0\uc744 \ubc1b\uc73c\uba74\uc11c \uc8fc\ub85c \ubca0\ub124\uce58\uc544 \ud568\ub300\ub85c \uc774\ub8e8\uc5b4\uc9c4 \uc88c\uc775\uc744 \ud1b5\uc194\ud558\uc600\ub2e4. \uc911\uc559\uc5d0\ub294 \ub3c8 \ud6c4\uc548 \ub370 \uc544\uc6b0\uc2a4\ud2b8\ub9ac\uc544\uac00 \uc9c1\uc811 \uae30\ud568 \ub808\uc54c\uc5d0\uc11c 62\ucc99\uc758 \ud568\ub300\ub97c \ud1b5\uc194\ud588\ub294\ub370, \uc5ec\uae30\uc5d0\ub294 \ud6d7\ub0a0 \ubca0\ub124\uce58\uc544\uc758 \ub3c4\uc81c\uac00 \ub41c \uc138\ubc14\uc2a4\ud2f0\uc544\ub178 \ubca0\ub2c8\uc5d0\ub974\uc640 \ub9c8\ub974\uce78\ud1a0\ub2c8\uc624 \ucf5c\ub85c\ub098\uac00 \uadf8\ub97c \ubcf4\uc88c\ud588\ub2e4. \uc6b0\uc775\uc5d0\uc11c\ub294 \uc720\uba85\ud55c \uc548\ub4dc\ub808\uc544 \ub3c4\ub9ac\uc544\uc758 \uc870\uce74\uc190\uc790\uc778 \uc81c\ub178\ubc14\uc758 \uc9c0\uc624\ubc18\ub2c8 \uc548\ub4dc\ub808\uc544 \ub3c4\ub9ac\uc544\uac00 \ub2e4\ub978 53\ucc99\uc758 \uc9c0\ud718\ub97c \ub9e1\uc558\ub2e4. \uc88c\uc775\uacfc \uc911\uc545 \uc6b0\uc775\uc758 \uc55e\ubd80\ubd84\uc5d0\ub294 \ub450 \ucc99\uc758 \uac24\ub9ac\uc5b4\uc2a4\uac00 \ubc30\uce58\ub418\uc5b4 \uc788\uc5c8\ub294\ub370, \ubbf8\uac94 \ub370 \uc138\ub974\ubc18\ud14c\uc2a4 \uc5d0 \ub530\ub974\uba74, \uc774\ub807\uac8c \ubc30\uce58\ud55c \ubaa9\uc801\uc740 \uc624\uc2a4\ub9cc\uad70\uc758 \ud568\uc120\uc774 \uc791\uc740 \ubc30\ub85c \uae30\uc2b5\ud558\uc5ec, \uae30\ub3c5\uad50 \ud568\uc120\uc5d0 \uc62c\ub77c\ud0c0 \uacf5\uaca9\ud558\ub294 \uac83\uc744 \ubc29\uc5b4\ud558\uae30 \uc704\ud574\uc11c\uc600\ub2e4. \ub098\uba38\uc9c0 \ud55c \ubd80\ub300\ub294 \uc911\uc559\ubd80\ub300\uc758 \ud6c4\ubc29\uc5d0 \uc608\ube44\ub300\ub85c\uc368 \ubc30\uce58\ub418\uc5c8\ub294\ub370, \ud544\uc694\ud55c \uacf3\uc5d0 \uc5b8\uc81c\ub4e0\uc9c0 \ub2ec\ub824\uac00 \uc9c0\uc6d0\uc744 \ud558\uae30 \uc704\ud574\uc11c\uc600\ub2e4. \uc774 \ud6c4\ubc29 \ubd80\ub300\ub294 38\ucc99\uc758 \uac24\ub9ac\uc120\uc73c\ub85c \ub418\uc5b4 \uc788\uc5c8\ub294\ub370, 30\ucc99\uc740 \uc911\uc559\ubd80\ubd84\uc758 \ud6c4\uc704\uc5d0 \ubc30\uce58\ub418\uc5b4 \uc54c\ubc14\ub85c \ub370 \ubc14\uc794\uc774 \ub2f4\ub2f9\ud558\uc600\uace0, 4\ucc99\uc740 \uac01 \ub0a0\uac1c\uc758 \ud6c4\ubc29\uc5d0 \ubc30\uce58\ub418\uc5b4 \uc788\uc5c8\ub2e4. \uc815\ucc30\ubd80\ub300\ub3c4 \uad6c\uc131\ub418\uc5c8\ub294\ub370, \uc6b0\uc775\uc5d0\uc11c \ub450 \ucc99\uacfc \ud6c4\ubc29\ubd80\ub300\uc5d0\uc11c 6\ucc99\uc758 \uac24\ub9ac\uc120\uc774 \ucc28\ucd9c\ub418\uc5c8\ub2e4. \uae30\ub3c5\uad50 \ud568\ub300\ub294 \ub290\ub9ac\uac8c \uc2a4\ud06c\ub85c\ud30c \uc9c0\uc810\uc758 \uc8fc\ubcc0\uc744 \ub3cc\uc558\uace0, \ud574\uc548\uac00\uc5d0\uc11c \ub5a8\uc5b4\uc9c4 \uacf3\uc5d0 \uc788\ub358 \ub3c4\ub9ac\uc544\uac00 \uc9c0\ud718\ud558\ub294 \uc6b0\uc775\uc740 \uc804\ud22c\ub97c \uc2dc\uc791\ud558\ub294 \uac8c \ub2a6\uc5b4 \uc6b0\uc775\uc758 \uac24\ub9ac\uc5b4\uc2a4\ub294 \uc790\ub9ac\ub97c \uc7a1\uc9c0 \ubabb\ud588\ub2e4."} {"question": "\uac80\uc740 9\uc6d4\ub2e8\uc774 \uc778\uc9c8\ub85c \ubd99\uc7a1\uc558\ub358 \uc774\uc2a4\ub77c\uc5d8 \uc120\uc218 11\uba85\uc740 \uacb0\uad6d\n \uc5b4\ub5bb\uac8c \ub418\uc5c8\ub294\uac00?", "paragraph": "\ud14c\ub7ec\ub3c4 \uc62c\ub9bc\ud53d\uc5d0\uc11c \uacf5\ud3ec\uc758 \ub300\uc0c1\uc774\uc5c8\ub2e4. \ubb8c\ud5e8 \ucc38\uc0ac\ub85c \uc54c\ub824\uc9c4 1972\ub144\uc5d0 \uc11c\ub3c5 \ubc14\uc774\uc5d0\ub978\uc758 \ubb8c\ud5e8\uc5d0\uc11c \uc5f4\ub9b0 \ud558\uacc4 \uc62c\ub9bc\ud53d\ub54c\uc758 \uc0ac\uac74\uc740 \ud14c\ub7ec\ub9ac\uc2a4\ud2b8\uc778 \uac80\uc740 9\uc6d4\ub2e8\uc774 \uc77c\uc73c\ud0a8 \uc0ac\uac74\uc73c\ub85c\uc11c \uc774\uc2a4\ub77c\uc5d8 \uc120\uc218 11\uba85\uc744 \uc778\uc9c8\ub85c \ubd99\uc7a1\uc558\ub2e4\uac00 \uc804\uc6d0\uc774 \uc0ac\ub9dd\ud55c \uc0ac\uac74\uc774\ub2e4. \ub2f9\uc2dc \ubbf8\uc219\ud55c \uc9c4\uc555\uc73c\ub85c \uc778\ud574 \uc778\uc9c8 9\uba85(\uc120\uc218 1\uba85\uacfc \ucf54\uce58 1\uba85\uc740 \uc778\uc9c8\ub85c \uc7a1\uae30 \uc774\uc804\uc5d0 \uc0b4\ud574), \ud14c\ub7ec\ubc94 5\uba85, \ub3c5\uc77c \uacbd\ucc30\uad00 1\uba85\uc774 \uc0ac\ub9dd\ud588\uc73c\uba70 \uc774 \uc9c4\uc555 \uc791\uc804 \uc774\uc804\uc5d0\ub294 \uc778\uc9c8\ub4e4\uc740 \ub2e8 \ud55c \uba85\ub3c4 \uc8fd\uc9c0 \uc54a\uc558\ub2e4. \uc560\ud2c0\ub780\ud0c0\uc5d0\uc11c \uc5f4\ub9b0 1996\ub144 \ud558\uacc4 \uc62c\ub9bc\ud53d \ub54c\ub294 \uc13c\ud14c\ub2c8\uc5bc \uc62c\ub9bc\ud53d \uacf5\uc6d0(Centennial Olympic Park)\uc5d0\uc11c \ud3ed\ubc1c \uc0ac\uac74\uc774 \uc77c\uc5b4\ub098 2\uba85\uc774 \uc8fd\uace0 111\uba85\uc774 \ub2e4\uce58\ub294 \uc0ac\uac74\uc774 \ubc1c\uc0dd\ud588\ub2e4. \uc774 \uc0ac\uac74\uc758 \uc8fc\ubaa8\uc790 \uc5d0\ub9ad \ub85c\ubc84\ud2b8 \ub8e8\ub3cc\ud504\ub294 \uc885\uc2e0\ud615\uc744 \uc120\uace0\ubc1b\uc558\ub2e4. \ucc38\uace0\ub85c \ub9c8\ub77c\ud1a4 \uc5ed\uc2dc \uc804\uc7c1\uc5d0\uc11c \uc720\ub798\ud55c \uac83\uc774\ub2e4.", "answer": "\uc804\uc6d0\uc774 \uc0ac\ub9dd", "sentence": "\ubb8c\ud5e8 \ucc38\uc0ac\ub85c \uc54c\ub824\uc9c4 1972\ub144\uc5d0 \uc11c\ub3c5 \ubc14\uc774\uc5d0\ub978\uc758 \ubb8c\ud5e8\uc5d0\uc11c \uc5f4\ub9b0 \ud558\uacc4 \uc62c\ub9bc\ud53d\ub54c\uc758 \uc0ac\uac74\uc740 \ud14c\ub7ec\ub9ac\uc2a4\ud2b8\uc778 \uac80\uc740 9\uc6d4\ub2e8\uc774 \uc77c\uc73c\ud0a8 \uc0ac\uac74\uc73c\ub85c\uc11c \uc774\uc2a4\ub77c\uc5d8 \uc120\uc218 11\uba85\uc744 \uc778\uc9c8\ub85c \ubd99\uc7a1\uc558\ub2e4\uac00 \uc804\uc6d0\uc774 \uc0ac\ub9dd \ud55c \uc0ac\uac74\uc774\ub2e4.", "paragraph_sentence": "\ud14c\ub7ec\ub3c4 \uc62c\ub9bc\ud53d\uc5d0\uc11c \uacf5\ud3ec\uc758 \ub300\uc0c1\uc774\uc5c8\ub2e4. \ubb8c\ud5e8 \ucc38\uc0ac\ub85c \uc54c\ub824\uc9c4 1972\ub144\uc5d0 \uc11c\ub3c5 \ubc14\uc774\uc5d0\ub978\uc758 \ubb8c\ud5e8\uc5d0\uc11c \uc5f4\ub9b0 \ud558\uacc4 \uc62c\ub9bc\ud53d\ub54c\uc758 \uc0ac\uac74\uc740 \ud14c\ub7ec\ub9ac\uc2a4\ud2b8\uc778 \uac80\uc740 9\uc6d4\ub2e8\uc774 \uc77c\uc73c\ud0a8 \uc0ac\uac74\uc73c\ub85c\uc11c \uc774\uc2a4\ub77c\uc5d8 \uc120\uc218 11\uba85\uc744 \uc778\uc9c8\ub85c \ubd99\uc7a1\uc558\ub2e4\uac00 \uc804\uc6d0\uc774 \uc0ac\ub9dd \ud55c \uc0ac\uac74\uc774\ub2e4. \ub2f9\uc2dc \ubbf8\uc219\ud55c \uc9c4\uc555\uc73c\ub85c \uc778\ud574 \uc778\uc9c8 9\uba85(\uc120\uc218 1\uba85\uacfc \ucf54\uce58 1\uba85\uc740 \uc778\uc9c8\ub85c \uc7a1\uae30 \uc774\uc804\uc5d0 \uc0b4\ud574), \ud14c\ub7ec\ubc94 5\uba85, \ub3c5\uc77c \uacbd\ucc30\uad00 1\uba85\uc774 \uc0ac\ub9dd\ud588\uc73c\uba70 \uc774 \uc9c4\uc555 \uc791\uc804 \uc774\uc804\uc5d0\ub294 \uc778\uc9c8\ub4e4\uc740 \ub2e8 \ud55c \uba85\ub3c4 \uc8fd\uc9c0 \uc54a\uc558\ub2e4. \uc560\ud2c0\ub780\ud0c0\uc5d0\uc11c \uc5f4\ub9b0 1996\ub144 \ud558\uacc4 \uc62c\ub9bc\ud53d \ub54c\ub294 \uc13c\ud14c\ub2c8\uc5bc \uc62c\ub9bc\ud53d \uacf5\uc6d0(Centennial Olympic Park)\uc5d0\uc11c \ud3ed\ubc1c \uc0ac\uac74\uc774 \uc77c\uc5b4\ub098 2\uba85\uc774 \uc8fd\uace0 111\uba85\uc774 \ub2e4\uce58\ub294 \uc0ac\uac74\uc774 \ubc1c\uc0dd\ud588\ub2e4. \uc774 \uc0ac\uac74\uc758 \uc8fc\ubaa8\uc790 \uc5d0\ub9ad \ub85c\ubc84\ud2b8 \ub8e8\ub3cc\ud504\ub294 \uc885\uc2e0\ud615\uc744 \uc120\uace0\ubc1b\uc558\ub2e4. \ucc38\uace0\ub85c \ub9c8\ub77c\ud1a4 \uc5ed\uc2dc \uc804\uc7c1\uc5d0\uc11c \uc720\ub798\ud55c \uac83\uc774\ub2e4.", "paragraph_answer": "\ud14c\ub7ec\ub3c4 \uc62c\ub9bc\ud53d\uc5d0\uc11c \uacf5\ud3ec\uc758 \ub300\uc0c1\uc774\uc5c8\ub2e4. \ubb8c\ud5e8 \ucc38\uc0ac\ub85c \uc54c\ub824\uc9c4 1972\ub144\uc5d0 \uc11c\ub3c5 \ubc14\uc774\uc5d0\ub978\uc758 \ubb8c\ud5e8\uc5d0\uc11c \uc5f4\ub9b0 \ud558\uacc4 \uc62c\ub9bc\ud53d\ub54c\uc758 \uc0ac\uac74\uc740 \ud14c\ub7ec\ub9ac\uc2a4\ud2b8\uc778 \uac80\uc740 9\uc6d4\ub2e8\uc774 \uc77c\uc73c\ud0a8 \uc0ac\uac74\uc73c\ub85c\uc11c \uc774\uc2a4\ub77c\uc5d8 \uc120\uc218 11\uba85\uc744 \uc778\uc9c8\ub85c \ubd99\uc7a1\uc558\ub2e4\uac00 \uc804\uc6d0\uc774 \uc0ac\ub9dd \ud55c \uc0ac\uac74\uc774\ub2e4. \ub2f9\uc2dc \ubbf8\uc219\ud55c \uc9c4\uc555\uc73c\ub85c \uc778\ud574 \uc778\uc9c8 9\uba85(\uc120\uc218 1\uba85\uacfc \ucf54\uce58 1\uba85\uc740 \uc778\uc9c8\ub85c \uc7a1\uae30 \uc774\uc804\uc5d0 \uc0b4\ud574), \ud14c\ub7ec\ubc94 5\uba85, \ub3c5\uc77c \uacbd\ucc30\uad00 1\uba85\uc774 \uc0ac\ub9dd\ud588\uc73c\uba70 \uc774 \uc9c4\uc555 \uc791\uc804 \uc774\uc804\uc5d0\ub294 \uc778\uc9c8\ub4e4\uc740 \ub2e8 \ud55c \uba85\ub3c4 \uc8fd\uc9c0 \uc54a\uc558\ub2e4. \uc560\ud2c0\ub780\ud0c0\uc5d0\uc11c \uc5f4\ub9b0 1996\ub144 \ud558\uacc4 \uc62c\ub9bc\ud53d \ub54c\ub294 \uc13c\ud14c\ub2c8\uc5bc \uc62c\ub9bc\ud53d \uacf5\uc6d0(Centennial Olympic Park)\uc5d0\uc11c \ud3ed\ubc1c \uc0ac\uac74\uc774 \uc77c\uc5b4\ub098 2\uba85\uc774 \uc8fd\uace0 111\uba85\uc774 \ub2e4\uce58\ub294 \uc0ac\uac74\uc774 \ubc1c\uc0dd\ud588\ub2e4. \uc774 \uc0ac\uac74\uc758 \uc8fc\ubaa8\uc790 \uc5d0\ub9ad \ub85c\ubc84\ud2b8 \ub8e8\ub3cc\ud504\ub294 \uc885\uc2e0\ud615\uc744 \uc120\uace0\ubc1b\uc558\ub2e4. \ucc38\uace0\ub85c \ub9c8\ub77c\ud1a4 \uc5ed\uc2dc \uc804\uc7c1\uc5d0\uc11c \uc720\ub798\ud55c \uac83\uc774\ub2e4.", "sentence_answer": "\ubb8c\ud5e8 \ucc38\uc0ac\ub85c \uc54c\ub824\uc9c4 1972\ub144\uc5d0 \uc11c\ub3c5 \ubc14\uc774\uc5d0\ub978\uc758 \ubb8c\ud5e8\uc5d0\uc11c \uc5f4\ub9b0 \ud558\uacc4 \uc62c\ub9bc\ud53d\ub54c\uc758 \uc0ac\uac74\uc740 \ud14c\ub7ec\ub9ac\uc2a4\ud2b8\uc778 \uac80\uc740 9\uc6d4\ub2e8\uc774 \uc77c\uc73c\ud0a8 \uc0ac\uac74\uc73c\ub85c\uc11c \uc774\uc2a4\ub77c\uc5d8 \uc120\uc218 11\uba85\uc744 \uc778\uc9c8\ub85c \ubd99\uc7a1\uc558\ub2e4\uac00 \uc804\uc6d0\uc774 \uc0ac\ub9dd \ud55c \uc0ac\uac74\uc774\ub2e4.", "paragraph_id": "6588341-29-1", "paragraph_question": "question: \uac80\uc740 9\uc6d4\ub2e8\uc774 \uc778\uc9c8\ub85c \ubd99\uc7a1\uc558\ub358 \uc774\uc2a4\ub77c\uc5d8 \uc120\uc218 11\uba85\uc740 \uacb0\uad6d\n \uc5b4\ub5bb\uac8c \ub418\uc5c8\ub294\uac00?, context: \ud14c\ub7ec\ub3c4 \uc62c\ub9bc\ud53d\uc5d0\uc11c \uacf5\ud3ec\uc758 \ub300\uc0c1\uc774\uc5c8\ub2e4. \ubb8c\ud5e8 \ucc38\uc0ac\ub85c \uc54c\ub824\uc9c4 1972\ub144\uc5d0 \uc11c\ub3c5 \ubc14\uc774\uc5d0\ub978\uc758 \ubb8c\ud5e8\uc5d0\uc11c \uc5f4\ub9b0 \ud558\uacc4 \uc62c\ub9bc\ud53d\ub54c\uc758 \uc0ac\uac74\uc740 \ud14c\ub7ec\ub9ac\uc2a4\ud2b8\uc778 \uac80\uc740 9\uc6d4\ub2e8\uc774 \uc77c\uc73c\ud0a8 \uc0ac\uac74\uc73c\ub85c\uc11c \uc774\uc2a4\ub77c\uc5d8 \uc120\uc218 11\uba85\uc744 \uc778\uc9c8\ub85c \ubd99\uc7a1\uc558\ub2e4\uac00 \uc804\uc6d0\uc774 \uc0ac\ub9dd\ud55c \uc0ac\uac74\uc774\ub2e4. \ub2f9\uc2dc \ubbf8\uc219\ud55c \uc9c4\uc555\uc73c\ub85c \uc778\ud574 \uc778\uc9c8 9\uba85(\uc120\uc218 1\uba85\uacfc \ucf54\uce58 1\uba85\uc740 \uc778\uc9c8\ub85c \uc7a1\uae30 \uc774\uc804\uc5d0 \uc0b4\ud574), \ud14c\ub7ec\ubc94 5\uba85, \ub3c5\uc77c \uacbd\ucc30\uad00 1\uba85\uc774 \uc0ac\ub9dd\ud588\uc73c\uba70 \uc774 \uc9c4\uc555 \uc791\uc804 \uc774\uc804\uc5d0\ub294 \uc778\uc9c8\ub4e4\uc740 \ub2e8 \ud55c \uba85\ub3c4 \uc8fd\uc9c0 \uc54a\uc558\ub2e4. \uc560\ud2c0\ub780\ud0c0\uc5d0\uc11c \uc5f4\ub9b0 1996\ub144 \ud558\uacc4 \uc62c\ub9bc\ud53d \ub54c\ub294 \uc13c\ud14c\ub2c8\uc5bc \uc62c\ub9bc\ud53d \uacf5\uc6d0(Centennial Olympic Park)\uc5d0\uc11c \ud3ed\ubc1c \uc0ac\uac74\uc774 \uc77c\uc5b4\ub098 2\uba85\uc774 \uc8fd\uace0 111\uba85\uc774 \ub2e4\uce58\ub294 \uc0ac\uac74\uc774 \ubc1c\uc0dd\ud588\ub2e4. \uc774 \uc0ac\uac74\uc758 \uc8fc\ubaa8\uc790 \uc5d0\ub9ad \ub85c\ubc84\ud2b8 \ub8e8\ub3cc\ud504\ub294 \uc885\uc2e0\ud615\uc744 \uc120\uace0\ubc1b\uc558\ub2e4. \ucc38\uace0\ub85c \ub9c8\ub77c\ud1a4 \uc5ed\uc2dc \uc804\uc7c1\uc5d0\uc11c \uc720\ub798\ud55c \uac83\uc774\ub2e4."} {"question": "\ud544\ub9ac\ud540\uc5d0\uc11c \uc624\ud1a0\ubc14\uc774\uc640 \uad00\ub828\ub41c \uc0ac\uace0 \uac00\ub2a5\uc131\uc740 \uc804\uccb4 \uc790\ub3d9\ucc28 \uc0ac\uace0\uc758 1%\ub3c4 \ub418\uc9c0 \uc54a\ub294\ub2e4\ub294 \ud1b5\uacc4\ub294 \uc5b4\ub290 \uae30\uad00\uc758 \uc870\uc0ac\uc778\uac00?", "paragraph": "2001\ub144 1\uc6d4, \ud544\ub9ac\ud540\uc758 \uba87\uba87 \ubaa8\ud130\uc0ac\uc774\ud074 \ud074\ub7fd\uc774 \ub9c8\uce74\ud2f0(Makati City) \ubc95\uc6d0\uc5d0 \uc720\ub8cc\ub3c4\ub85c\uc758 \uc6b4\ud589\uc744 \uae08\uc9c0\ud558\ub294 \ud1b5\ud589\uaddc\uc81c\uc704\uc6d0\ud68c \uaddc\uce59\uc5d0 \ub300\ud574 \uc2dc\ubbfc \uc18c\uc1a1\uc744 \uc81c\uae30\ud558\uc600\ub2e4. \uccad\uc6d0\uc790\ub294 \ud1b5\ud589\uae08\uc9c0\uac00 \ud604\ud589 \ubc95\ub960\uc744 \uc704\ubc18\ud560 \ubfd0 \uc544\ub2c8\ub77c \ubaa8\ub4e0 \uc624\ud1a0\ubc14\uc774 \uc6b4\uc804\uc790\uac00 \uad50\ud1b5\uc0ac\uace0 \uc8fc \uc6d0\uc778\uc73c\ub85c\ubd80\ud130 \uc790\uc720\ub85c\uc6b4 \uc720\ub8cc\ub3c4\ub85c\ub97c \uc774\uc6a9\ud558\uc9c0 \ubabb\ud558\ub3c4\ub85d \uac15\uc81c\ud558\uc5ec \uacf5\uacf5 \uc548\uc804\uc744 \uc704\ud611\ud55c\ub2e4\uace0 \uc8fc\uc7a5\ud558\uc600\ub2e4. \uc721\uc0c1\uc6b4\uc1a1\uad6d \ud1b5\uacc4\uc5d0 \ub530\ub974\uba74 \uc624\ud1a0\ubc14\uc774\uc640 \uad00\ub828\ub41c \uc0ac\uace0 \uac00\ub2a5\uc131\uc740 \uc804\uccb4 \uc790\ub3d9\ucc28 \uc0ac\uace0\uc758 1%\ub3c4 \uc548\ub418\ubbc0\ub85c \ud1b5\ud589\uae08\uc9c0\ub97c \ud574\uc81c\ud558\uba74 \uc0ac\uace0\uac00 \ub354\uc6b1 \ub354 \uc904\uc5b4\ub4e4 \uac83\uc73c\ub85c \uc608\uc0c1\ub418\uc5c8\ub2e4. \ud544\ub9ac\ud540 \uc804\uc5ed\uc758 \uc624\ud1a0\ubc14\uc774 \uc0ac\uc6a9\uc790\ub294 \uc804\uccb4 \uc790\ub3d9\ucc28 \uc778\uad6c\uc758 \uc57d 1/3\ub85c \uc218\ubc31\ub9cc\uba85\uc5d0\uac8c\ub294 \uadf9\ud788 \uc911\uc694\ud55c \uc6b4\uc1a1\uc218\ub2e8\uc774\ub2e4. \uc774 \uc0ac\uac74\uc758 \uc8fc\uc694 \ub17c\uc810\uc740 \uc811\uadfc\uc81c\ud55c\uacf5\ub3c4\ubc95\uc73c\ub85c \uc54c\ub824\uc9c4 \ud544\ub9ac\ud540 \uacf5\ud654\uad6d\ubc95\ub839 2000\ud638\uac00 \uc624\ub85c\uc9c0 \ud2b8\ub7ed, \ubc84\uc2a4, \uae30\ud0c0 \uc0c1\uc5c5\uc6a9 \ucc28\ub7c9\uc758 \uc720\ub8cc\ub3c4\ub85c \ud1b5\ud589\uc744 \uc81c\uc678\ud558\uace0 \uc788\ub2e4\ub294 \uc0ac\uc2e4\uc774\ub2e4. \uadf8\ub7ec\ub098 \uccad\uc6d0\uc790\ub4e4\uc740 \ud1b5\ud589\uaddc\uc81c\uc704\uc6d0\ud68c\uac00 \uc624\uc9c1 \uc624\ud1a0\ubc14\uc774\ub9cc \ucd94\ubc29\ud558\uc5ec \ubc95\uc744 \uc704\ubc18\ud558\ub294 \uac83\uc744 \uad00\ucc30\ud558\uc600\ub2e4.", "answer": "\uc721\uc0c1\uc6b4\uc1a1\uad6d", "sentence": "\uc721\uc0c1\uc6b4\uc1a1\uad6d \ud1b5\uacc4\uc5d0 \ub530\ub974\uba74 \uc624\ud1a0\ubc14\uc774\uc640 \uad00\ub828\ub41c \uc0ac\uace0 \uac00\ub2a5\uc131\uc740 \uc804\uccb4 \uc790\ub3d9\ucc28 \uc0ac\uace0\uc758 1%\ub3c4 \uc548\ub418\ubbc0\ub85c \ud1b5\ud589\uae08\uc9c0\ub97c \ud574\uc81c\ud558\uba74 \uc0ac\uace0\uac00 \ub354\uc6b1 \ub354 \uc904\uc5b4\ub4e4 \uac83\uc73c\ub85c \uc608\uc0c1\ub418\uc5c8\ub2e4.", "paragraph_sentence": "2001\ub144 1\uc6d4, \ud544\ub9ac\ud540\uc758 \uba87\uba87 \ubaa8\ud130\uc0ac\uc774\ud074 \ud074\ub7fd\uc774 \ub9c8\uce74\ud2f0(Makati City) \ubc95\uc6d0\uc5d0 \uc720\ub8cc\ub3c4\ub85c\uc758 \uc6b4\ud589\uc744 \uae08\uc9c0\ud558\ub294 \ud1b5\ud589\uaddc\uc81c\uc704\uc6d0\ud68c \uaddc\uce59\uc5d0 \ub300\ud574 \uc2dc\ubbfc \uc18c\uc1a1\uc744 \uc81c\uae30\ud558\uc600\ub2e4. \uccad\uc6d0\uc790\ub294 \ud1b5\ud589\uae08\uc9c0\uac00 \ud604\ud589 \ubc95\ub960\uc744 \uc704\ubc18\ud560 \ubfd0 \uc544\ub2c8\ub77c \ubaa8\ub4e0 \uc624\ud1a0\ubc14\uc774 \uc6b4\uc804\uc790\uac00 \uad50\ud1b5\uc0ac\uace0 \uc8fc \uc6d0\uc778\uc73c\ub85c\ubd80\ud130 \uc790\uc720\ub85c\uc6b4 \uc720\ub8cc\ub3c4\ub85c\ub97c \uc774\uc6a9\ud558\uc9c0 \ubabb\ud558\ub3c4\ub85d \uac15\uc81c\ud558\uc5ec \uacf5\uacf5 \uc548\uc804\uc744 \uc704\ud611\ud55c\ub2e4\uace0 \uc8fc\uc7a5\ud558\uc600\ub2e4. \uc721\uc0c1\uc6b4\uc1a1\uad6d \ud1b5\uacc4\uc5d0 \ub530\ub974\uba74 \uc624\ud1a0\ubc14\uc774\uc640 \uad00\ub828\ub41c \uc0ac\uace0 \uac00\ub2a5\uc131\uc740 \uc804\uccb4 \uc790\ub3d9\ucc28 \uc0ac\uace0\uc758 1%\ub3c4 \uc548\ub418\ubbc0\ub85c \ud1b5\ud589\uae08\uc9c0\ub97c \ud574\uc81c\ud558\uba74 \uc0ac\uace0\uac00 \ub354\uc6b1 \ub354 \uc904\uc5b4\ub4e4 \uac83\uc73c\ub85c \uc608\uc0c1\ub418\uc5c8\ub2e4. \ud544\ub9ac\ud540 \uc804\uc5ed\uc758 \uc624\ud1a0\ubc14\uc774 \uc0ac\uc6a9\uc790\ub294 \uc804\uccb4 \uc790\ub3d9\ucc28 \uc778\uad6c\uc758 \uc57d 1/3\ub85c \uc218\ubc31\ub9cc\uba85\uc5d0\uac8c\ub294 \uadf9\ud788 \uc911\uc694\ud55c \uc6b4\uc1a1\uc218\ub2e8\uc774\ub2e4. \uc774 \uc0ac\uac74\uc758 \uc8fc\uc694 \ub17c\uc810\uc740 \uc811\uadfc\uc81c\ud55c\uacf5\ub3c4\ubc95\uc73c\ub85c \uc54c\ub824\uc9c4 \ud544\ub9ac\ud540 \uacf5\ud654\uad6d\ubc95\ub839 2000\ud638\uac00 \uc624\ub85c\uc9c0 \ud2b8\ub7ed, \ubc84\uc2a4, \uae30\ud0c0 \uc0c1\uc5c5\uc6a9 \ucc28\ub7c9\uc758 \uc720\ub8cc\ub3c4\ub85c \ud1b5\ud589\uc744 \uc81c\uc678\ud558\uace0 \uc788\ub2e4\ub294 \uc0ac\uc2e4\uc774\ub2e4. \uadf8\ub7ec\ub098 \uccad\uc6d0\uc790\ub4e4\uc740 \ud1b5\ud589\uaddc\uc81c\uc704\uc6d0\ud68c\uac00 \uc624\uc9c1 \uc624\ud1a0\ubc14\uc774\ub9cc \ucd94\ubc29\ud558\uc5ec \ubc95\uc744 \uc704\ubc18\ud558\ub294 \uac83\uc744 \uad00\ucc30\ud558\uc600\ub2e4.", "paragraph_answer": "2001\ub144 1\uc6d4, \ud544\ub9ac\ud540\uc758 \uba87\uba87 \ubaa8\ud130\uc0ac\uc774\ud074 \ud074\ub7fd\uc774 \ub9c8\uce74\ud2f0(Makati City) \ubc95\uc6d0\uc5d0 \uc720\ub8cc\ub3c4\ub85c\uc758 \uc6b4\ud589\uc744 \uae08\uc9c0\ud558\ub294 \ud1b5\ud589\uaddc\uc81c\uc704\uc6d0\ud68c \uaddc\uce59\uc5d0 \ub300\ud574 \uc2dc\ubbfc \uc18c\uc1a1\uc744 \uc81c\uae30\ud558\uc600\ub2e4. \uccad\uc6d0\uc790\ub294 \ud1b5\ud589\uae08\uc9c0\uac00 \ud604\ud589 \ubc95\ub960\uc744 \uc704\ubc18\ud560 \ubfd0 \uc544\ub2c8\ub77c \ubaa8\ub4e0 \uc624\ud1a0\ubc14\uc774 \uc6b4\uc804\uc790\uac00 \uad50\ud1b5\uc0ac\uace0 \uc8fc \uc6d0\uc778\uc73c\ub85c\ubd80\ud130 \uc790\uc720\ub85c\uc6b4 \uc720\ub8cc\ub3c4\ub85c\ub97c \uc774\uc6a9\ud558\uc9c0 \ubabb\ud558\ub3c4\ub85d \uac15\uc81c\ud558\uc5ec \uacf5\uacf5 \uc548\uc804\uc744 \uc704\ud611\ud55c\ub2e4\uace0 \uc8fc\uc7a5\ud558\uc600\ub2e4. \uc721\uc0c1\uc6b4\uc1a1\uad6d \ud1b5\uacc4\uc5d0 \ub530\ub974\uba74 \uc624\ud1a0\ubc14\uc774\uc640 \uad00\ub828\ub41c \uc0ac\uace0 \uac00\ub2a5\uc131\uc740 \uc804\uccb4 \uc790\ub3d9\ucc28 \uc0ac\uace0\uc758 1%\ub3c4 \uc548\ub418\ubbc0\ub85c \ud1b5\ud589\uae08\uc9c0\ub97c \ud574\uc81c\ud558\uba74 \uc0ac\uace0\uac00 \ub354\uc6b1 \ub354 \uc904\uc5b4\ub4e4 \uac83\uc73c\ub85c \uc608\uc0c1\ub418\uc5c8\ub2e4. \ud544\ub9ac\ud540 \uc804\uc5ed\uc758 \uc624\ud1a0\ubc14\uc774 \uc0ac\uc6a9\uc790\ub294 \uc804\uccb4 \uc790\ub3d9\ucc28 \uc778\uad6c\uc758 \uc57d 1/3\ub85c \uc218\ubc31\ub9cc\uba85\uc5d0\uac8c\ub294 \uadf9\ud788 \uc911\uc694\ud55c \uc6b4\uc1a1\uc218\ub2e8\uc774\ub2e4. \uc774 \uc0ac\uac74\uc758 \uc8fc\uc694 \ub17c\uc810\uc740 \uc811\uadfc\uc81c\ud55c\uacf5\ub3c4\ubc95\uc73c\ub85c \uc54c\ub824\uc9c4 \ud544\ub9ac\ud540 \uacf5\ud654\uad6d\ubc95\ub839 2000\ud638\uac00 \uc624\ub85c\uc9c0 \ud2b8\ub7ed, \ubc84\uc2a4, \uae30\ud0c0 \uc0c1\uc5c5\uc6a9 \ucc28\ub7c9\uc758 \uc720\ub8cc\ub3c4\ub85c \ud1b5\ud589\uc744 \uc81c\uc678\ud558\uace0 \uc788\ub2e4\ub294 \uc0ac\uc2e4\uc774\ub2e4. \uadf8\ub7ec\ub098 \uccad\uc6d0\uc790\ub4e4\uc740 \ud1b5\ud589\uaddc\uc81c\uc704\uc6d0\ud68c\uac00 \uc624\uc9c1 \uc624\ud1a0\ubc14\uc774\ub9cc \ucd94\ubc29\ud558\uc5ec \ubc95\uc744 \uc704\ubc18\ud558\ub294 \uac83\uc744 \uad00\ucc30\ud558\uc600\ub2e4.", "sentence_answer": " \uc721\uc0c1\uc6b4\uc1a1\uad6d \ud1b5\uacc4\uc5d0 \ub530\ub974\uba74 \uc624\ud1a0\ubc14\uc774\uc640 \uad00\ub828\ub41c \uc0ac\uace0 \uac00\ub2a5\uc131\uc740 \uc804\uccb4 \uc790\ub3d9\ucc28 \uc0ac\uace0\uc758 1%\ub3c4 \uc548\ub418\ubbc0\ub85c \ud1b5\ud589\uae08\uc9c0\ub97c \ud574\uc81c\ud558\uba74 \uc0ac\uace0\uac00 \ub354\uc6b1 \ub354 \uc904\uc5b4\ub4e4 \uac83\uc73c\ub85c \uc608\uc0c1\ub418\uc5c8\ub2e4.", "paragraph_id": "6578976-2-1", "paragraph_question": "question: \ud544\ub9ac\ud540\uc5d0\uc11c \uc624\ud1a0\ubc14\uc774\uc640 \uad00\ub828\ub41c \uc0ac\uace0 \uac00\ub2a5\uc131\uc740 \uc804\uccb4 \uc790\ub3d9\ucc28 \uc0ac\uace0\uc758 1%\ub3c4 \ub418\uc9c0 \uc54a\ub294\ub2e4\ub294 \ud1b5\uacc4\ub294 \uc5b4\ub290 \uae30\uad00\uc758 \uc870\uc0ac\uc778\uac00?, context: 2001\ub144 1\uc6d4, \ud544\ub9ac\ud540\uc758 \uba87\uba87 \ubaa8\ud130\uc0ac\uc774\ud074 \ud074\ub7fd\uc774 \ub9c8\uce74\ud2f0(Makati City) \ubc95\uc6d0\uc5d0 \uc720\ub8cc\ub3c4\ub85c\uc758 \uc6b4\ud589\uc744 \uae08\uc9c0\ud558\ub294 \ud1b5\ud589\uaddc\uc81c\uc704\uc6d0\ud68c \uaddc\uce59\uc5d0 \ub300\ud574 \uc2dc\ubbfc \uc18c\uc1a1\uc744 \uc81c\uae30\ud558\uc600\ub2e4. \uccad\uc6d0\uc790\ub294 \ud1b5\ud589\uae08\uc9c0\uac00 \ud604\ud589 \ubc95\ub960\uc744 \uc704\ubc18\ud560 \ubfd0 \uc544\ub2c8\ub77c \ubaa8\ub4e0 \uc624\ud1a0\ubc14\uc774 \uc6b4\uc804\uc790\uac00 \uad50\ud1b5\uc0ac\uace0 \uc8fc \uc6d0\uc778\uc73c\ub85c\ubd80\ud130 \uc790\uc720\ub85c\uc6b4 \uc720\ub8cc\ub3c4\ub85c\ub97c \uc774\uc6a9\ud558\uc9c0 \ubabb\ud558\ub3c4\ub85d \uac15\uc81c\ud558\uc5ec \uacf5\uacf5 \uc548\uc804\uc744 \uc704\ud611\ud55c\ub2e4\uace0 \uc8fc\uc7a5\ud558\uc600\ub2e4. \uc721\uc0c1\uc6b4\uc1a1\uad6d \ud1b5\uacc4\uc5d0 \ub530\ub974\uba74 \uc624\ud1a0\ubc14\uc774\uc640 \uad00\ub828\ub41c \uc0ac\uace0 \uac00\ub2a5\uc131\uc740 \uc804\uccb4 \uc790\ub3d9\ucc28 \uc0ac\uace0\uc758 1%\ub3c4 \uc548\ub418\ubbc0\ub85c \ud1b5\ud589\uae08\uc9c0\ub97c \ud574\uc81c\ud558\uba74 \uc0ac\uace0\uac00 \ub354\uc6b1 \ub354 \uc904\uc5b4\ub4e4 \uac83\uc73c\ub85c \uc608\uc0c1\ub418\uc5c8\ub2e4. \ud544\ub9ac\ud540 \uc804\uc5ed\uc758 \uc624\ud1a0\ubc14\uc774 \uc0ac\uc6a9\uc790\ub294 \uc804\uccb4 \uc790\ub3d9\ucc28 \uc778\uad6c\uc758 \uc57d 1/3\ub85c \uc218\ubc31\ub9cc\uba85\uc5d0\uac8c\ub294 \uadf9\ud788 \uc911\uc694\ud55c \uc6b4\uc1a1\uc218\ub2e8\uc774\ub2e4. \uc774 \uc0ac\uac74\uc758 \uc8fc\uc694 \ub17c\uc810\uc740 \uc811\uadfc\uc81c\ud55c\uacf5\ub3c4\ubc95\uc73c\ub85c \uc54c\ub824\uc9c4 \ud544\ub9ac\ud540 \uacf5\ud654\uad6d\ubc95\ub839 2000\ud638\uac00 \uc624\ub85c\uc9c0 \ud2b8\ub7ed, \ubc84\uc2a4, \uae30\ud0c0 \uc0c1\uc5c5\uc6a9 \ucc28\ub7c9\uc758 \uc720\ub8cc\ub3c4\ub85c \ud1b5\ud589\uc744 \uc81c\uc678\ud558\uace0 \uc788\ub2e4\ub294 \uc0ac\uc2e4\uc774\ub2e4. \uadf8\ub7ec\ub098 \uccad\uc6d0\uc790\ub4e4\uc740 \ud1b5\ud589\uaddc\uc81c\uc704\uc6d0\ud68c\uac00 \uc624\uc9c1 \uc624\ud1a0\ubc14\uc774\ub9cc \ucd94\ubc29\ud558\uc5ec \ubc95\uc744 \uc704\ubc18\ud558\ub294 \uac83\uc744 \uad00\ucc30\ud558\uc600\ub2e4."} {"question": "\ubd81\ub300\uc804IC\uc2dc\uc678\ubc84\uc2a4\uc815\ub958\uc18c \uc124\uce58\uc640 \ubcc4\uac1c\ub85c \ub300\uc804\uc2dc\uc5d0\uc11c\ub294 \ubb34\uc5c7\uc744 \uc2e0\ucd95\ud558\uace0\uc790 \ucd94\uc9c4 \uc911\uc774\uc5c8\ub294\uac00?", "paragraph": "\ubd81\ub300\uc804IC\uc2dc\uc678\ubc84\uc2a4\uc815\ub958\uc18c\ub294 \uc124\uce58\ub97c \ucd94\uc9c4\ud560 \ub54c\ubd80\ud130 \ub9d0\uc774 \ub9ce\uc558\ub2e4. \ubd81\ub300\uc804 \ub098\ub4e4\ubaa9 \uc778\uadfc \ub3c4\ub85c\ub294 \ud3ed\uc774 \ud3b8\ub3c4 3\ucc28\ub85c\uc5d0 \uace0\uc18d\uc73c\ub85c \uc8fc\ud589\ud558\ub294 \ucc28\ub7c9\uc774 \ub9ce\uc544 \uad50\ud1b5\uc0ac\uace0 \ub4f1\uc758 \uc704\ud5d8 \uc694\uc18c\uac00 \ud56d\uc0c1 \ub3c4\uc0ac\ub9ac\uace0 \uc788\uc73c\uba70, \uc774\ub7ec\ud55c \ub3c4\ub85c\uc5d0 \ub300\ud615 \ucc28\ub7c9\uc778 \uc2dc\uc678\ubc84\uc2a4\uac00 \uc815\ub958\uc7a5\uc5d0 \uc815\ucc28\ud558\uae30 \uc704\ud574 \uc18d\ub3c4\ub97c \uc904\uc774\uac8c \ub418\uba74 \uc2ec\uac01\ud55c \uc815\uccb4 \ud604\uc0c1\uc73c\ub85c \uad50\ud1b5\ud63c\uc7a1\uc774 \uc608\uc0c1\ub418\uc5b4 \ub17c\ub780\uc774 \uc77c\uc5c8\ub2e4. \ub610\ud55c \ub300\uc804\uc2dc\uc5d0\uc11c\ub294 \uc774\uc640 \ubcc4\uac1c\ub85c \uae30\uc874\uc758 \uc720\uc131\uc2dc\uc678\ubc84\uc2a4\uc815\ub958\uc18c\uc640 \uc720\uc131\uace0\uc18d\ubc84\uc2a4\ud130\ubbf8\ub110\uc744 \ud1b5\ud569\ud574 \uc720\uc131\ubcf5\ud569\ud130\ubbf8\ub110 \uc2e0\ucd95\uc744 \ucd94\uc9c4\ud558\uace0 \uc788\uae30 \ub54c\ubb38\uc5d0 \uc0c8\ub85c\uc6b4 \uac04\uc774\uc815\ub958\uc18c\ub97c \uc124\uce58\ud558\ub294 \uac83\ubcf4\ub2e4\ub294 \uc720\uc131\ubcf5\ud569\ud130\ubbf8\ub110\uacfc \ub300\uc804\ubcf5\ud569\ud130\ubbf8\ub110 \uc911\uc2ec\uc73c\ub85c \uc2dc\uc678\ubc84\uc2a4 \uc6b4\uc601\uc744 \uc7ac\uc815\ube44\ud558\uae30\uae4c\uc9c0 \ub2e4\uc18c \ubd88\ud3b8\ud568\uc774 \uc788\ub354\ub77c\ub3c4 \uc778\uadfc\uc758 \uc815\ub958\uc7a5\uc744 \uc774\uc6a9\ud560 \uc218 \uc788\uac8c \uc720\ub3c4\ud558\ub294 \uac83\uc774 \uac80\ud1a0\ub418\uc5b4\uc57c \ud55c\ub2e4\ub294 \uc8fc\uc7a5\ub3c4 \uc81c\uae30\ub418\uc5c8\ub2e4.", "answer": "\uc720\uc131\ubcf5\ud569\ud130\ubbf8\ub110", "sentence": "\ub610\ud55c \ub300\uc804\uc2dc\uc5d0\uc11c\ub294 \uc774\uc640 \ubcc4\uac1c\ub85c \uae30\uc874\uc758 \uc720\uc131\uc2dc\uc678\ubc84\uc2a4\uc815\ub958\uc18c\uc640 \uc720\uc131\uace0\uc18d\ubc84\uc2a4\ud130\ubbf8\ub110\uc744 \ud1b5\ud569\ud574 \uc720\uc131\ubcf5\ud569\ud130\ubbf8\ub110 \uc2e0\ucd95\uc744 \ucd94\uc9c4\ud558\uace0 \uc788\uae30 \ub54c\ubb38\uc5d0 \uc0c8\ub85c\uc6b4 \uac04\uc774\uc815\ub958\uc18c\ub97c \uc124\uce58\ud558\ub294 \uac83\ubcf4\ub2e4\ub294 \uc720\uc131\ubcf5\ud569\ud130\ubbf8\ub110\uacfc \ub300\uc804\ubcf5\ud569\ud130\ubbf8\ub110 \uc911\uc2ec\uc73c\ub85c \uc2dc\uc678\ubc84\uc2a4 \uc6b4\uc601\uc744 \uc7ac\uc815\ube44\ud558\uae30\uae4c\uc9c0 \ub2e4\uc18c \ubd88\ud3b8\ud568\uc774 \uc788\ub354\ub77c\ub3c4 \uc778\uadfc\uc758 \uc815\ub958\uc7a5\uc744 \uc774\uc6a9\ud560 \uc218 \uc788\uac8c \uc720\ub3c4\ud558\ub294 \uac83\uc774 \uac80\ud1a0\ub418\uc5b4\uc57c \ud55c\ub2e4\ub294 \uc8fc\uc7a5\ub3c4 \uc81c\uae30\ub418\uc5c8\ub2e4.", "paragraph_sentence": "\ubd81\ub300\uc804IC\uc2dc\uc678\ubc84\uc2a4\uc815\ub958\uc18c\ub294 \uc124\uce58\ub97c \ucd94\uc9c4\ud560 \ub54c\ubd80\ud130 \ub9d0\uc774 \ub9ce\uc558\ub2e4. \ubd81\ub300\uc804 \ub098\ub4e4\ubaa9 \uc778\uadfc \ub3c4\ub85c\ub294 \ud3ed\uc774 \ud3b8\ub3c4 3\ucc28\ub85c\uc5d0 \uace0\uc18d\uc73c\ub85c \uc8fc\ud589\ud558\ub294 \ucc28\ub7c9\uc774 \ub9ce\uc544 \uad50\ud1b5\uc0ac\uace0 \ub4f1\uc758 \uc704\ud5d8 \uc694\uc18c\uac00 \ud56d\uc0c1 \ub3c4\uc0ac\ub9ac\uace0 \uc788\uc73c\uba70, \uc774\ub7ec\ud55c \ub3c4\ub85c\uc5d0 \ub300\ud615 \ucc28\ub7c9\uc778 \uc2dc\uc678\ubc84\uc2a4\uac00 \uc815\ub958\uc7a5\uc5d0 \uc815\ucc28\ud558\uae30 \uc704\ud574 \uc18d\ub3c4\ub97c \uc904\uc774\uac8c \ub418\uba74 \uc2ec\uac01\ud55c \uc815\uccb4 \ud604\uc0c1\uc73c\ub85c \uad50\ud1b5\ud63c\uc7a1\uc774 \uc608\uc0c1\ub418\uc5b4 \ub17c\ub780\uc774 \uc77c\uc5c8\ub2e4. \ub610\ud55c \ub300\uc804\uc2dc\uc5d0\uc11c\ub294 \uc774\uc640 \ubcc4\uac1c\ub85c \uae30\uc874\uc758 \uc720\uc131\uc2dc\uc678\ubc84\uc2a4\uc815\ub958\uc18c\uc640 \uc720\uc131\uace0\uc18d\ubc84\uc2a4\ud130\ubbf8\ub110\uc744 \ud1b5\ud569\ud574 \uc720\uc131\ubcf5\ud569\ud130\ubbf8\ub110 \uc2e0\ucd95\uc744 \ucd94\uc9c4\ud558\uace0 \uc788\uae30 \ub54c\ubb38\uc5d0 \uc0c8\ub85c\uc6b4 \uac04\uc774\uc815\ub958\uc18c\ub97c \uc124\uce58\ud558\ub294 \uac83\ubcf4\ub2e4\ub294 \uc720\uc131\ubcf5\ud569\ud130\ubbf8\ub110\uacfc \ub300\uc804\ubcf5\ud569\ud130\ubbf8\ub110 \uc911\uc2ec\uc73c\ub85c \uc2dc\uc678\ubc84\uc2a4 \uc6b4\uc601\uc744 \uc7ac\uc815\ube44\ud558\uae30\uae4c\uc9c0 \ub2e4\uc18c \ubd88\ud3b8\ud568\uc774 \uc788\ub354\ub77c\ub3c4 \uc778\uadfc\uc758 \uc815\ub958\uc7a5\uc744 \uc774\uc6a9\ud560 \uc218 \uc788\uac8c \uc720\ub3c4\ud558\ub294 \uac83\uc774 \uac80\ud1a0\ub418\uc5b4\uc57c \ud55c\ub2e4\ub294 \uc8fc\uc7a5\ub3c4 \uc81c\uae30\ub418\uc5c8\ub2e4. ", "paragraph_answer": "\ubd81\ub300\uc804IC\uc2dc\uc678\ubc84\uc2a4\uc815\ub958\uc18c\ub294 \uc124\uce58\ub97c \ucd94\uc9c4\ud560 \ub54c\ubd80\ud130 \ub9d0\uc774 \ub9ce\uc558\ub2e4. \ubd81\ub300\uc804 \ub098\ub4e4\ubaa9 \uc778\uadfc \ub3c4\ub85c\ub294 \ud3ed\uc774 \ud3b8\ub3c4 3\ucc28\ub85c\uc5d0 \uace0\uc18d\uc73c\ub85c \uc8fc\ud589\ud558\ub294 \ucc28\ub7c9\uc774 \ub9ce\uc544 \uad50\ud1b5\uc0ac\uace0 \ub4f1\uc758 \uc704\ud5d8 \uc694\uc18c\uac00 \ud56d\uc0c1 \ub3c4\uc0ac\ub9ac\uace0 \uc788\uc73c\uba70, \uc774\ub7ec\ud55c \ub3c4\ub85c\uc5d0 \ub300\ud615 \ucc28\ub7c9\uc778 \uc2dc\uc678\ubc84\uc2a4\uac00 \uc815\ub958\uc7a5\uc5d0 \uc815\ucc28\ud558\uae30 \uc704\ud574 \uc18d\ub3c4\ub97c \uc904\uc774\uac8c \ub418\uba74 \uc2ec\uac01\ud55c \uc815\uccb4 \ud604\uc0c1\uc73c\ub85c \uad50\ud1b5\ud63c\uc7a1\uc774 \uc608\uc0c1\ub418\uc5b4 \ub17c\ub780\uc774 \uc77c\uc5c8\ub2e4. \ub610\ud55c \ub300\uc804\uc2dc\uc5d0\uc11c\ub294 \uc774\uc640 \ubcc4\uac1c\ub85c \uae30\uc874\uc758 \uc720\uc131\uc2dc\uc678\ubc84\uc2a4\uc815\ub958\uc18c\uc640 \uc720\uc131\uace0\uc18d\ubc84\uc2a4\ud130\ubbf8\ub110\uc744 \ud1b5\ud569\ud574 \uc720\uc131\ubcf5\ud569\ud130\ubbf8\ub110 \uc2e0\ucd95\uc744 \ucd94\uc9c4\ud558\uace0 \uc788\uae30 \ub54c\ubb38\uc5d0 \uc0c8\ub85c\uc6b4 \uac04\uc774\uc815\ub958\uc18c\ub97c \uc124\uce58\ud558\ub294 \uac83\ubcf4\ub2e4\ub294 \uc720\uc131\ubcf5\ud569\ud130\ubbf8\ub110\uacfc \ub300\uc804\ubcf5\ud569\ud130\ubbf8\ub110 \uc911\uc2ec\uc73c\ub85c \uc2dc\uc678\ubc84\uc2a4 \uc6b4\uc601\uc744 \uc7ac\uc815\ube44\ud558\uae30\uae4c\uc9c0 \ub2e4\uc18c \ubd88\ud3b8\ud568\uc774 \uc788\ub354\ub77c\ub3c4 \uc778\uadfc\uc758 \uc815\ub958\uc7a5\uc744 \uc774\uc6a9\ud560 \uc218 \uc788\uac8c \uc720\ub3c4\ud558\ub294 \uac83\uc774 \uac80\ud1a0\ub418\uc5b4\uc57c \ud55c\ub2e4\ub294 \uc8fc\uc7a5\ub3c4 \uc81c\uae30\ub418\uc5c8\ub2e4.", "sentence_answer": "\ub610\ud55c \ub300\uc804\uc2dc\uc5d0\uc11c\ub294 \uc774\uc640 \ubcc4\uac1c\ub85c \uae30\uc874\uc758 \uc720\uc131\uc2dc\uc678\ubc84\uc2a4\uc815\ub958\uc18c\uc640 \uc720\uc131\uace0\uc18d\ubc84\uc2a4\ud130\ubbf8\ub110\uc744 \ud1b5\ud569\ud574 \uc720\uc131\ubcf5\ud569\ud130\ubbf8\ub110 \uc2e0\ucd95\uc744 \ucd94\uc9c4\ud558\uace0 \uc788\uae30 \ub54c\ubb38\uc5d0 \uc0c8\ub85c\uc6b4 \uac04\uc774\uc815\ub958\uc18c\ub97c \uc124\uce58\ud558\ub294 \uac83\ubcf4\ub2e4\ub294 \uc720\uc131\ubcf5\ud569\ud130\ubbf8\ub110\uacfc \ub300\uc804\ubcf5\ud569\ud130\ubbf8\ub110 \uc911\uc2ec\uc73c\ub85c \uc2dc\uc678\ubc84\uc2a4 \uc6b4\uc601\uc744 \uc7ac\uc815\ube44\ud558\uae30\uae4c\uc9c0 \ub2e4\uc18c \ubd88\ud3b8\ud568\uc774 \uc788\ub354\ub77c\ub3c4 \uc778\uadfc\uc758 \uc815\ub958\uc7a5\uc744 \uc774\uc6a9\ud560 \uc218 \uc788\uac8c \uc720\ub3c4\ud558\ub294 \uac83\uc774 \uac80\ud1a0\ub418\uc5b4\uc57c \ud55c\ub2e4\ub294 \uc8fc\uc7a5\ub3c4 \uc81c\uae30\ub418\uc5c8\ub2e4.", "paragraph_id": "6582407-0-0", "paragraph_question": "question: \ubd81\ub300\uc804IC\uc2dc\uc678\ubc84\uc2a4\uc815\ub958\uc18c \uc124\uce58\uc640 \ubcc4\uac1c\ub85c \ub300\uc804\uc2dc\uc5d0\uc11c\ub294 \ubb34\uc5c7\uc744 \uc2e0\ucd95\ud558\uace0\uc790 \ucd94\uc9c4 \uc911\uc774\uc5c8\ub294\uac00?, context: \ubd81\ub300\uc804IC\uc2dc\uc678\ubc84\uc2a4\uc815\ub958\uc18c\ub294 \uc124\uce58\ub97c \ucd94\uc9c4\ud560 \ub54c\ubd80\ud130 \ub9d0\uc774 \ub9ce\uc558\ub2e4. \ubd81\ub300\uc804 \ub098\ub4e4\ubaa9 \uc778\uadfc \ub3c4\ub85c\ub294 \ud3ed\uc774 \ud3b8\ub3c4 3\ucc28\ub85c\uc5d0 \uace0\uc18d\uc73c\ub85c \uc8fc\ud589\ud558\ub294 \ucc28\ub7c9\uc774 \ub9ce\uc544 \uad50\ud1b5\uc0ac\uace0 \ub4f1\uc758 \uc704\ud5d8 \uc694\uc18c\uac00 \ud56d\uc0c1 \ub3c4\uc0ac\ub9ac\uace0 \uc788\uc73c\uba70, \uc774\ub7ec\ud55c \ub3c4\ub85c\uc5d0 \ub300\ud615 \ucc28\ub7c9\uc778 \uc2dc\uc678\ubc84\uc2a4\uac00 \uc815\ub958\uc7a5\uc5d0 \uc815\ucc28\ud558\uae30 \uc704\ud574 \uc18d\ub3c4\ub97c \uc904\uc774\uac8c \ub418\uba74 \uc2ec\uac01\ud55c \uc815\uccb4 \ud604\uc0c1\uc73c\ub85c \uad50\ud1b5\ud63c\uc7a1\uc774 \uc608\uc0c1\ub418\uc5b4 \ub17c\ub780\uc774 \uc77c\uc5c8\ub2e4. \ub610\ud55c \ub300\uc804\uc2dc\uc5d0\uc11c\ub294 \uc774\uc640 \ubcc4\uac1c\ub85c \uae30\uc874\uc758 \uc720\uc131\uc2dc\uc678\ubc84\uc2a4\uc815\ub958\uc18c\uc640 \uc720\uc131\uace0\uc18d\ubc84\uc2a4\ud130\ubbf8\ub110\uc744 \ud1b5\ud569\ud574 \uc720\uc131\ubcf5\ud569\ud130\ubbf8\ub110 \uc2e0\ucd95\uc744 \ucd94\uc9c4\ud558\uace0 \uc788\uae30 \ub54c\ubb38\uc5d0 \uc0c8\ub85c\uc6b4 \uac04\uc774\uc815\ub958\uc18c\ub97c \uc124\uce58\ud558\ub294 \uac83\ubcf4\ub2e4\ub294 \uc720\uc131\ubcf5\ud569\ud130\ubbf8\ub110\uacfc \ub300\uc804\ubcf5\ud569\ud130\ubbf8\ub110 \uc911\uc2ec\uc73c\ub85c \uc2dc\uc678\ubc84\uc2a4 \uc6b4\uc601\uc744 \uc7ac\uc815\ube44\ud558\uae30\uae4c\uc9c0 \ub2e4\uc18c \ubd88\ud3b8\ud568\uc774 \uc788\ub354\ub77c\ub3c4 \uc778\uadfc\uc758 \uc815\ub958\uc7a5\uc744 \uc774\uc6a9\ud560 \uc218 \uc788\uac8c \uc720\ub3c4\ud558\ub294 \uac83\uc774 \uac80\ud1a0\ub418\uc5b4\uc57c \ud55c\ub2e4\ub294 \uc8fc\uc7a5\ub3c4 \uc81c\uae30\ub418\uc5c8\ub2e4."} {"question": "\uc911\uc559\uad11\uc7a5\uc744 \ucd9c\ubc1c\ud558\uc5ec \uc218\uc720\ub3d9 \uad6d\ub9bd 4.19 \ubb18\uc9c0\uae4c\uc9c0 \ud589\uc9c4\ud55c \ub2e4\uc74c 4.19 \uae30\ub150\ud0d1\uc5d0\uc11c \ucc38\ubc30\ud55c \ud6c4 \uace0\ub824\ub300\ud559\uad50\ub85c \ub2e4\uc2dc \ub3cc\uc544\uc624\ub294 \ud615\uc2dd\uc744 \ub744\ub294 \uc774 \ud589\uc0ac\ub294 \uba87\ub144\ub3c4\uc5d0 \ucc98\uc74c \uac1c\ucd5c \ub418\uc5c8\ub294\uac00?", "paragraph": "\ub9e4\ub144 4\uc6d4 18\uc77c\uc5d0\ub294 4\u00b719 \ud601\uba85\uc758 \ub3c4\ud654\uc120\uc774 \ub410\ub358 4\u00b718 \uc758\uac70\ub97c \uae30\ub150\ud558\uae30 \uc704\ud574 4\u00b718 \uae30\ub150 \ub9c8\ub77c\ud1a4\uacfc \uad6c\uad6d\ub300\uc7a5\uc815\uc744 \uc9c4\ud589\ud55c\ub2e4. \ud5cc\ud654 \ud589\uc0ac\uc5d0 \uc774\uc5b4 \uc624\uc804\uc5d0\ub294 \uae30\ub150 \ub9c8\ub77c\ud1a4\uc774, \uc624\ud6c4\uc5d0\ub294 \uad6c\uad6d\ub300\uc7a5\uc815\uc774 \uc9c4\ud589\ub41c\ub2e4. \uae30\ub150 \ub9c8\ub77c\ud1a4\uc740 1969\ub144 \ub2f9\uc2dc \ucd1d\ud559\uc0dd\ud68c\uc5d0\uc11c \ucc98\uc74c\uc73c\ub85c \ub9cc\ub4e0 \uc774\ub798 \ud604\uc7ac\uae4c\uc9c0 \uafb8\uc900\ud788 \uc774\uc5b4\uc838 \ub0b4\ub824\uc624\uace0 \uc788\uc73c\uba70, \ud559\uad50\ub97c \ucd9c\ubc1c\ud558\uc5ec \uad6d\ub9bd 4\u00b719 \ubb18\uc9c0\ub97c \uac70\uccd0 \ub2e4\uc2dc \ud559\uad50\ub85c \ub3cc\uc544\uc624\ub294 \ucd1d 16.2km\uc758 \ucf54\uc2a4\ub85c \uad6c\uc131\ub3fc \uc788\ub2e4. \ucd5c\uc885 \ubaa9\uc801\uc9c0\uac00 \uad6d\ub9bd 4\u00b719 \ubb18\uc9c0\ub85c \uc124\uc815\ub418\uc5b4 \uc788\ub294 \uac83\uc5d0\ub294 4\u00b718\uc774 4\u00b719 \uc815\uc2e0\uc73c\ub85c \uc774\uc5b4\uc9c4\ub2e4\ub294 \uc758\ubbf8\uac00 \ub2f4\uaca8 \uc788\ub2e4. \uc774\uc640 \uac19\uc740 \ud589\uc0ac\ub294 1980\ub144\ub300 \uad70\uc0ac\uc815\uad8c \uc2dc\uc808\uc5d0\ub3c4 \uc774\uc5b4\uc838 \ub9c8\ub77c\ud1a4\uc774\ub77c\ub294 \ud615\uc2dd\uc744 \ube4c\ub824 \ubd88\uc758\uc5d0 \ub9de\uc11c \uc2f8\uc6b0\ub294 \uae30\ub2a5 \ub610\ud55c \ub2f4\ub2f9\ud558\uc600\ub2e4. \uad6c\uad6d\ub300\uc7a5\uc815\uc740 \uac01 \ub2e8\uacfc\ub300\ubcc4\ub85c \ubaa8\uc5ec \uad50\ub0b4 \uc911\uc559\uad11\uc7a5\uc744 \ucd9c\ubc1c\ud558\uc5ec \uc218\uc720\ub3d9 \uad6d\ub9bd 4\u00b719 \ubb18\uc9c0\uae4c\uc9c0 \ud589\uc9c4\ud55c \ub2e4\uc74c, 4\u00b719 \uae30\ub150\ud0d1\uc5d0\uc11c \ucc38\ubc30\ud55c \ud6c4 \ud559\uad50\ub85c \ub2e4\uc2dc \ub3cc\uc544\uc624\ub294 \ud615\uc2dd\uc744 \ub744\ub294 \ud589\uc0ac\ub85c, \uc774 \ud589\uc0ac \ub610\ud55c 1969\ub144\uc5d0 \ucc98\uc74c\uc73c\ub85c \uac1c\ucd5c\ub410\ub2e4. \ud589\uc0ac \ucd08\uae30\uc5d0\ub294 \uc2dc\ubbfc\ub4e4\uc758 \ubd88\ud3b8\uc744 \uc6b0\ub824\ud558\uc5ec \ucc38\uac00\uc790 \uc218\uc5d0 \uc81c\ud55c\uc744 \ub450\uc5c8\uc73c\ub098 1984\ub144\ubd80\ud130 \uc774 \uc81c\ud55c\uc774 \ud3d0\uc9c0\ub410\ub2e4. \uc774 \ud589\uc0ac\uc5d0\uc11c\uc758 \uad6c\ud638\ub294 \ud574\ub2f9\uc5f0\ub3c4\uc758 \uc774\uc288 \ub610\ub294 \uc2dc\ub300\uc801\uc778 \ubcc0\ud654\uac00 \ubc18\uc601\ub41c \uac83\uc73c\ub85c, \uc774\ub97c \uad6c\ud638\ubfd0\ub9cc \uc544\ub2c8\ub77c \ud31c\ud50c\ub81b\uc744 \ud1b5\ud574\uc11c\ub3c4 \uc2dc\ubbfc\ub4e4\uacfc \uacf5\uc720\ud558\uace0 \uc788\ub2e4. \uc120\ubc30\ub4e4\uc758 \uc815\uc2e0\uc744 \uacc4\uc2b9\ud55c\ub2e4\uae30\ubcf4\ub2e4\ub294 \uace0\ub300\uc0dd\uc774 \ub418\uae30 \uc704\ud55c \ud558\ub098\uc758 \uc808\ucc28 \uc815\ub3c4\ub85c \uc778\uc2dd\ub418\ub294 \uac83\uacfc \uad00\ub828\ud558\uc5ec \uc54c\ub9f9\uc774\uac00 \ube60\uc838 \uc788\ub2e4\ub294 \uc758\uacac\ub4e4\uc774 \uc874\uc7ac\ud558\ub294 \ubc18\uba74, \uafb8\uc900\ud788 \ud589\uc0ac\ub97c \uc774\uc5b4\ub098\uac00\uace0 \uc788\ub2e4\ub294 \uc810\uc744 \uae0d\uc815\uc801\uc73c\ub85c \ubcf4\ub294 \uc2dc\uac01 \ub610\ud55c \uc874\uc7ac\ud55c\ub2e4.", "answer": "1969\ub144", "sentence": "\uae30\ub150 \ub9c8\ub77c\ud1a4\uc740 1969\ub144 \ub2f9\uc2dc \ucd1d\ud559\uc0dd\ud68c\uc5d0\uc11c \ucc98\uc74c\uc73c\ub85c \ub9cc\ub4e0 \uc774\ub798 \ud604\uc7ac\uae4c\uc9c0 \uafb8\uc900\ud788 \uc774\uc5b4\uc838 \ub0b4\ub824\uc624\uace0 \uc788\uc73c\uba70, \ud559\uad50\ub97c \ucd9c\ubc1c\ud558\uc5ec \uad6d\ub9bd 4\u00b719 \ubb18\uc9c0\ub97c \uac70\uccd0 \ub2e4\uc2dc \ud559\uad50\ub85c \ub3cc\uc544\uc624\ub294 \ucd1d 16.2km\uc758 \ucf54\uc2a4\ub85c \uad6c\uc131\ub3fc \uc788\ub2e4.", "paragraph_sentence": "\ub9e4\ub144 4\uc6d4 18\uc77c\uc5d0\ub294 4\u00b719 \ud601\uba85\uc758 \ub3c4\ud654\uc120\uc774 \ub410\ub358 4\u00b718 \uc758\uac70\ub97c \uae30\ub150\ud558\uae30 \uc704\ud574 4\u00b718 \uae30\ub150 \ub9c8\ub77c\ud1a4\uacfc \uad6c\uad6d\ub300\uc7a5\uc815\uc744 \uc9c4\ud589\ud55c\ub2e4. \ud5cc\ud654 \ud589\uc0ac\uc5d0 \uc774\uc5b4 \uc624\uc804\uc5d0\ub294 \uae30\ub150 \ub9c8\ub77c\ud1a4\uc774, \uc624\ud6c4\uc5d0\ub294 \uad6c\uad6d\ub300\uc7a5\uc815\uc774 \uc9c4\ud589\ub41c\ub2e4. \uae30\ub150 \ub9c8\ub77c\ud1a4\uc740 1969\ub144 \ub2f9\uc2dc \ucd1d\ud559\uc0dd\ud68c\uc5d0\uc11c \ucc98\uc74c\uc73c\ub85c \ub9cc\ub4e0 \uc774\ub798 \ud604\uc7ac\uae4c\uc9c0 \uafb8\uc900\ud788 \uc774\uc5b4\uc838 \ub0b4\ub824\uc624\uace0 \uc788\uc73c\uba70, \ud559\uad50\ub97c \ucd9c\ubc1c\ud558\uc5ec \uad6d\ub9bd 4\u00b719 \ubb18\uc9c0\ub97c \uac70\uccd0 \ub2e4\uc2dc \ud559\uad50\ub85c \ub3cc\uc544\uc624\ub294 \ucd1d 16.2km\uc758 \ucf54\uc2a4\ub85c \uad6c\uc131\ub3fc \uc788\ub2e4. \ucd5c\uc885 \ubaa9\uc801\uc9c0\uac00 \uad6d\ub9bd 4\u00b719 \ubb18\uc9c0\ub85c \uc124\uc815\ub418\uc5b4 \uc788\ub294 \uac83\uc5d0\ub294 4\u00b718\uc774 4\u00b719 \uc815\uc2e0\uc73c\ub85c \uc774\uc5b4\uc9c4\ub2e4\ub294 \uc758\ubbf8\uac00 \ub2f4\uaca8 \uc788\ub2e4. \uc774\uc640 \uac19\uc740 \ud589\uc0ac\ub294 1980\ub144\ub300 \uad70\uc0ac\uc815\uad8c \uc2dc\uc808\uc5d0\ub3c4 \uc774\uc5b4\uc838 \ub9c8\ub77c\ud1a4\uc774\ub77c\ub294 \ud615\uc2dd\uc744 \ube4c\ub824 \ubd88\uc758\uc5d0 \ub9de\uc11c \uc2f8\uc6b0\ub294 \uae30\ub2a5 \ub610\ud55c \ub2f4\ub2f9\ud558\uc600\ub2e4. \uad6c\uad6d\ub300\uc7a5\uc815\uc740 \uac01 \ub2e8\uacfc\ub300\ubcc4\ub85c \ubaa8\uc5ec \uad50\ub0b4 \uc911\uc559\uad11\uc7a5\uc744 \ucd9c\ubc1c\ud558\uc5ec \uc218\uc720\ub3d9 \uad6d\ub9bd 4\u00b719 \ubb18\uc9c0\uae4c\uc9c0 \ud589\uc9c4\ud55c \ub2e4\uc74c, 4\u00b719 \uae30\ub150\ud0d1\uc5d0\uc11c \ucc38\ubc30\ud55c \ud6c4 \ud559\uad50\ub85c \ub2e4\uc2dc \ub3cc\uc544\uc624\ub294 \ud615\uc2dd\uc744 \ub744\ub294 \ud589\uc0ac\ub85c, \uc774 \ud589\uc0ac \ub610\ud55c 1969\ub144\uc5d0 \ucc98\uc74c\uc73c\ub85c \uac1c\ucd5c\ub410\ub2e4. \ud589\uc0ac \ucd08\uae30\uc5d0\ub294 \uc2dc\ubbfc\ub4e4\uc758 \ubd88\ud3b8\uc744 \uc6b0\ub824\ud558\uc5ec \ucc38\uac00\uc790 \uc218\uc5d0 \uc81c\ud55c\uc744 \ub450\uc5c8\uc73c\ub098 1984\ub144\ubd80\ud130 \uc774 \uc81c\ud55c\uc774 \ud3d0\uc9c0\ub410\ub2e4. \uc774 \ud589\uc0ac\uc5d0\uc11c\uc758 \uad6c\ud638\ub294 \ud574\ub2f9\uc5f0\ub3c4\uc758 \uc774\uc288 \ub610\ub294 \uc2dc\ub300\uc801\uc778 \ubcc0\ud654\uac00 \ubc18\uc601\ub41c \uac83\uc73c\ub85c, \uc774\ub97c \uad6c\ud638\ubfd0\ub9cc \uc544\ub2c8\ub77c \ud31c\ud50c\ub81b\uc744 \ud1b5\ud574\uc11c\ub3c4 \uc2dc\ubbfc\ub4e4\uacfc \uacf5\uc720\ud558\uace0 \uc788\ub2e4. \uc120\ubc30\ub4e4\uc758 \uc815\uc2e0\uc744 \uacc4\uc2b9\ud55c\ub2e4\uae30\ubcf4\ub2e4\ub294 \uace0\ub300\uc0dd\uc774 \ub418\uae30 \uc704\ud55c \ud558\ub098\uc758 \uc808\ucc28 \uc815\ub3c4\ub85c \uc778\uc2dd\ub418\ub294 \uac83\uacfc \uad00\ub828\ud558\uc5ec \uc54c\ub9f9\uc774\uac00 \ube60\uc838 \uc788\ub2e4\ub294 \uc758\uacac\ub4e4\uc774 \uc874\uc7ac\ud558\ub294 \ubc18\uba74, \uafb8\uc900\ud788 \ud589\uc0ac\ub97c \uc774\uc5b4\ub098\uac00\uace0 \uc788\ub2e4\ub294 \uc810\uc744 \uae0d\uc815\uc801\uc73c\ub85c \ubcf4\ub294 \uc2dc\uac01 \ub610\ud55c \uc874\uc7ac\ud55c\ub2e4.", "paragraph_answer": "\ub9e4\ub144 4\uc6d4 18\uc77c\uc5d0\ub294 4\u00b719 \ud601\uba85\uc758 \ub3c4\ud654\uc120\uc774 \ub410\ub358 4\u00b718 \uc758\uac70\ub97c \uae30\ub150\ud558\uae30 \uc704\ud574 4\u00b718 \uae30\ub150 \ub9c8\ub77c\ud1a4\uacfc \uad6c\uad6d\ub300\uc7a5\uc815\uc744 \uc9c4\ud589\ud55c\ub2e4. \ud5cc\ud654 \ud589\uc0ac\uc5d0 \uc774\uc5b4 \uc624\uc804\uc5d0\ub294 \uae30\ub150 \ub9c8\ub77c\ud1a4\uc774, \uc624\ud6c4\uc5d0\ub294 \uad6c\uad6d\ub300\uc7a5\uc815\uc774 \uc9c4\ud589\ub41c\ub2e4. \uae30\ub150 \ub9c8\ub77c\ud1a4\uc740 1969\ub144 \ub2f9\uc2dc \ucd1d\ud559\uc0dd\ud68c\uc5d0\uc11c \ucc98\uc74c\uc73c\ub85c \ub9cc\ub4e0 \uc774\ub798 \ud604\uc7ac\uae4c\uc9c0 \uafb8\uc900\ud788 \uc774\uc5b4\uc838 \ub0b4\ub824\uc624\uace0 \uc788\uc73c\uba70, \ud559\uad50\ub97c \ucd9c\ubc1c\ud558\uc5ec \uad6d\ub9bd 4\u00b719 \ubb18\uc9c0\ub97c \uac70\uccd0 \ub2e4\uc2dc \ud559\uad50\ub85c \ub3cc\uc544\uc624\ub294 \ucd1d 16.2km\uc758 \ucf54\uc2a4\ub85c \uad6c\uc131\ub3fc \uc788\ub2e4. \ucd5c\uc885 \ubaa9\uc801\uc9c0\uac00 \uad6d\ub9bd 4\u00b719 \ubb18\uc9c0\ub85c \uc124\uc815\ub418\uc5b4 \uc788\ub294 \uac83\uc5d0\ub294 4\u00b718\uc774 4\u00b719 \uc815\uc2e0\uc73c\ub85c \uc774\uc5b4\uc9c4\ub2e4\ub294 \uc758\ubbf8\uac00 \ub2f4\uaca8 \uc788\ub2e4. \uc774\uc640 \uac19\uc740 \ud589\uc0ac\ub294 1980\ub144\ub300 \uad70\uc0ac\uc815\uad8c \uc2dc\uc808\uc5d0\ub3c4 \uc774\uc5b4\uc838 \ub9c8\ub77c\ud1a4\uc774\ub77c\ub294 \ud615\uc2dd\uc744 \ube4c\ub824 \ubd88\uc758\uc5d0 \ub9de\uc11c \uc2f8\uc6b0\ub294 \uae30\ub2a5 \ub610\ud55c \ub2f4\ub2f9\ud558\uc600\ub2e4. \uad6c\uad6d\ub300\uc7a5\uc815\uc740 \uac01 \ub2e8\uacfc\ub300\ubcc4\ub85c \ubaa8\uc5ec \uad50\ub0b4 \uc911\uc559\uad11\uc7a5\uc744 \ucd9c\ubc1c\ud558\uc5ec \uc218\uc720\ub3d9 \uad6d\ub9bd 4\u00b719 \ubb18\uc9c0\uae4c\uc9c0 \ud589\uc9c4\ud55c \ub2e4\uc74c, 4\u00b719 \uae30\ub150\ud0d1\uc5d0\uc11c \ucc38\ubc30\ud55c \ud6c4 \ud559\uad50\ub85c \ub2e4\uc2dc \ub3cc\uc544\uc624\ub294 \ud615\uc2dd\uc744 \ub744\ub294 \ud589\uc0ac\ub85c, \uc774 \ud589\uc0ac \ub610\ud55c 1969\ub144\uc5d0 \ucc98\uc74c\uc73c\ub85c \uac1c\ucd5c\ub410\ub2e4. \ud589\uc0ac \ucd08\uae30\uc5d0\ub294 \uc2dc\ubbfc\ub4e4\uc758 \ubd88\ud3b8\uc744 \uc6b0\ub824\ud558\uc5ec \ucc38\uac00\uc790 \uc218\uc5d0 \uc81c\ud55c\uc744 \ub450\uc5c8\uc73c\ub098 1984\ub144\ubd80\ud130 \uc774 \uc81c\ud55c\uc774 \ud3d0\uc9c0\ub410\ub2e4. \uc774 \ud589\uc0ac\uc5d0\uc11c\uc758 \uad6c\ud638\ub294 \ud574\ub2f9\uc5f0\ub3c4\uc758 \uc774\uc288 \ub610\ub294 \uc2dc\ub300\uc801\uc778 \ubcc0\ud654\uac00 \ubc18\uc601\ub41c \uac83\uc73c\ub85c, \uc774\ub97c \uad6c\ud638\ubfd0\ub9cc \uc544\ub2c8\ub77c \ud31c\ud50c\ub81b\uc744 \ud1b5\ud574\uc11c\ub3c4 \uc2dc\ubbfc\ub4e4\uacfc \uacf5\uc720\ud558\uace0 \uc788\ub2e4. \uc120\ubc30\ub4e4\uc758 \uc815\uc2e0\uc744 \uacc4\uc2b9\ud55c\ub2e4\uae30\ubcf4\ub2e4\ub294 \uace0\ub300\uc0dd\uc774 \ub418\uae30 \uc704\ud55c \ud558\ub098\uc758 \uc808\ucc28 \uc815\ub3c4\ub85c \uc778\uc2dd\ub418\ub294 \uac83\uacfc \uad00\ub828\ud558\uc5ec \uc54c\ub9f9\uc774\uac00 \ube60\uc838 \uc788\ub2e4\ub294 \uc758\uacac\ub4e4\uc774 \uc874\uc7ac\ud558\ub294 \ubc18\uba74, \uafb8\uc900\ud788 \ud589\uc0ac\ub97c \uc774\uc5b4\ub098\uac00\uace0 \uc788\ub2e4\ub294 \uc810\uc744 \uae0d\uc815\uc801\uc73c\ub85c \ubcf4\ub294 \uc2dc\uac01 \ub610\ud55c \uc874\uc7ac\ud55c\ub2e4.", "sentence_answer": "\uae30\ub150 \ub9c8\ub77c\ud1a4\uc740 1969\ub144 \ub2f9\uc2dc \ucd1d\ud559\uc0dd\ud68c\uc5d0\uc11c \ucc98\uc74c\uc73c\ub85c \ub9cc\ub4e0 \uc774\ub798 \ud604\uc7ac\uae4c\uc9c0 \uafb8\uc900\ud788 \uc774\uc5b4\uc838 \ub0b4\ub824\uc624\uace0 \uc788\uc73c\uba70, \ud559\uad50\ub97c \ucd9c\ubc1c\ud558\uc5ec \uad6d\ub9bd 4\u00b719 \ubb18\uc9c0\ub97c \uac70\uccd0 \ub2e4\uc2dc \ud559\uad50\ub85c \ub3cc\uc544\uc624\ub294 \ucd1d 16.2km\uc758 \ucf54\uc2a4\ub85c \uad6c\uc131\ub3fc \uc788\ub2e4.", "paragraph_id": "6474998-11-1", "paragraph_question": "question: \uc911\uc559\uad11\uc7a5\uc744 \ucd9c\ubc1c\ud558\uc5ec \uc218\uc720\ub3d9 \uad6d\ub9bd 4.19 \ubb18\uc9c0\uae4c\uc9c0 \ud589\uc9c4\ud55c \ub2e4\uc74c 4.19 \uae30\ub150\ud0d1\uc5d0\uc11c \ucc38\ubc30\ud55c \ud6c4 \uace0\ub824\ub300\ud559\uad50\ub85c \ub2e4\uc2dc \ub3cc\uc544\uc624\ub294 \ud615\uc2dd\uc744 \ub744\ub294 \uc774 \ud589\uc0ac\ub294 \uba87\ub144\ub3c4\uc5d0 \ucc98\uc74c \uac1c\ucd5c \ub418\uc5c8\ub294\uac00?, context: \ub9e4\ub144 4\uc6d4 18\uc77c\uc5d0\ub294 4\u00b719 \ud601\uba85\uc758 \ub3c4\ud654\uc120\uc774 \ub410\ub358 4\u00b718 \uc758\uac70\ub97c \uae30\ub150\ud558\uae30 \uc704\ud574 4\u00b718 \uae30\ub150 \ub9c8\ub77c\ud1a4\uacfc \uad6c\uad6d\ub300\uc7a5\uc815\uc744 \uc9c4\ud589\ud55c\ub2e4. \ud5cc\ud654 \ud589\uc0ac\uc5d0 \uc774\uc5b4 \uc624\uc804\uc5d0\ub294 \uae30\ub150 \ub9c8\ub77c\ud1a4\uc774, \uc624\ud6c4\uc5d0\ub294 \uad6c\uad6d\ub300\uc7a5\uc815\uc774 \uc9c4\ud589\ub41c\ub2e4. \uae30\ub150 \ub9c8\ub77c\ud1a4\uc740 1969\ub144 \ub2f9\uc2dc \ucd1d\ud559\uc0dd\ud68c\uc5d0\uc11c \ucc98\uc74c\uc73c\ub85c \ub9cc\ub4e0 \uc774\ub798 \ud604\uc7ac\uae4c\uc9c0 \uafb8\uc900\ud788 \uc774\uc5b4\uc838 \ub0b4\ub824\uc624\uace0 \uc788\uc73c\uba70, \ud559\uad50\ub97c \ucd9c\ubc1c\ud558\uc5ec \uad6d\ub9bd 4\u00b719 \ubb18\uc9c0\ub97c \uac70\uccd0 \ub2e4\uc2dc \ud559\uad50\ub85c \ub3cc\uc544\uc624\ub294 \ucd1d 16.2km\uc758 \ucf54\uc2a4\ub85c \uad6c\uc131\ub3fc \uc788\ub2e4. \ucd5c\uc885 \ubaa9\uc801\uc9c0\uac00 \uad6d\ub9bd 4\u00b719 \ubb18\uc9c0\ub85c \uc124\uc815\ub418\uc5b4 \uc788\ub294 \uac83\uc5d0\ub294 4\u00b718\uc774 4\u00b719 \uc815\uc2e0\uc73c\ub85c \uc774\uc5b4\uc9c4\ub2e4\ub294 \uc758\ubbf8\uac00 \ub2f4\uaca8 \uc788\ub2e4. \uc774\uc640 \uac19\uc740 \ud589\uc0ac\ub294 1980\ub144\ub300 \uad70\uc0ac\uc815\uad8c \uc2dc\uc808\uc5d0\ub3c4 \uc774\uc5b4\uc838 \ub9c8\ub77c\ud1a4\uc774\ub77c\ub294 \ud615\uc2dd\uc744 \ube4c\ub824 \ubd88\uc758\uc5d0 \ub9de\uc11c \uc2f8\uc6b0\ub294 \uae30\ub2a5 \ub610\ud55c \ub2f4\ub2f9\ud558\uc600\ub2e4. \uad6c\uad6d\ub300\uc7a5\uc815\uc740 \uac01 \ub2e8\uacfc\ub300\ubcc4\ub85c \ubaa8\uc5ec \uad50\ub0b4 \uc911\uc559\uad11\uc7a5\uc744 \ucd9c\ubc1c\ud558\uc5ec \uc218\uc720\ub3d9 \uad6d\ub9bd 4\u00b719 \ubb18\uc9c0\uae4c\uc9c0 \ud589\uc9c4\ud55c \ub2e4\uc74c, 4\u00b719 \uae30\ub150\ud0d1\uc5d0\uc11c \ucc38\ubc30\ud55c \ud6c4 \ud559\uad50\ub85c \ub2e4\uc2dc \ub3cc\uc544\uc624\ub294 \ud615\uc2dd\uc744 \ub744\ub294 \ud589\uc0ac\ub85c, \uc774 \ud589\uc0ac \ub610\ud55c 1969\ub144\uc5d0 \ucc98\uc74c\uc73c\ub85c \uac1c\ucd5c\ub410\ub2e4. \ud589\uc0ac \ucd08\uae30\uc5d0\ub294 \uc2dc\ubbfc\ub4e4\uc758 \ubd88\ud3b8\uc744 \uc6b0\ub824\ud558\uc5ec \ucc38\uac00\uc790 \uc218\uc5d0 \uc81c\ud55c\uc744 \ub450\uc5c8\uc73c\ub098 1984\ub144\ubd80\ud130 \uc774 \uc81c\ud55c\uc774 \ud3d0\uc9c0\ub410\ub2e4. \uc774 \ud589\uc0ac\uc5d0\uc11c\uc758 \uad6c\ud638\ub294 \ud574\ub2f9\uc5f0\ub3c4\uc758 \uc774\uc288 \ub610\ub294 \uc2dc\ub300\uc801\uc778 \ubcc0\ud654\uac00 \ubc18\uc601\ub41c \uac83\uc73c\ub85c, \uc774\ub97c \uad6c\ud638\ubfd0\ub9cc \uc544\ub2c8\ub77c \ud31c\ud50c\ub81b\uc744 \ud1b5\ud574\uc11c\ub3c4 \uc2dc\ubbfc\ub4e4\uacfc \uacf5\uc720\ud558\uace0 \uc788\ub2e4. \uc120\ubc30\ub4e4\uc758 \uc815\uc2e0\uc744 \uacc4\uc2b9\ud55c\ub2e4\uae30\ubcf4\ub2e4\ub294 \uace0\ub300\uc0dd\uc774 \ub418\uae30 \uc704\ud55c \ud558\ub098\uc758 \uc808\ucc28 \uc815\ub3c4\ub85c \uc778\uc2dd\ub418\ub294 \uac83\uacfc \uad00\ub828\ud558\uc5ec \uc54c\ub9f9\uc774\uac00 \ube60\uc838 \uc788\ub2e4\ub294 \uc758\uacac\ub4e4\uc774 \uc874\uc7ac\ud558\ub294 \ubc18\uba74, \uafb8\uc900\ud788 \ud589\uc0ac\ub97c \uc774\uc5b4\ub098\uac00\uace0 \uc788\ub2e4\ub294 \uc810\uc744 \uae0d\uc815\uc801\uc73c\ub85c \ubcf4\ub294 \uc2dc\uac01 \ub610\ud55c \uc874\uc7ac\ud55c\ub2e4."} {"question": "\uae40\ubd80\ub9ac\uc758 \ub300\uc655\uac01\uc5d0 \uc2e0\uc73c\ub85c \ubaa8\uc154\uc9c0\uace0 \uc788\ub294 \uc778\ubb3c\uc740?", "paragraph": "\ub2e4\ub9cc \ub9c8\uc758 \ud0dc\uc790(\u9ebb\u8863 \u592a\u5b50)\uac00 \uc740\uac70\ud588\ub358 \uae08\uac15\uc0b0 \uc8fc\ubcc0\uc744 \uc911\uc2ec\uc73c\ub85c \ub9c8\uc758 \ud0dc\uc790\uc640 \uad00\ub828\ud55c \ubbfc\uac04 \uc804\uc2b9\uc774 \ub9ce\uc774 \uc804\ud574\uc9c0\uace0 \uc788\ub2e4. \ube44\ub85c\ubd09 \ubc14\ub85c \uc544\ub798\uc5d0\ub294 \ub9c8\uc758 \ud0dc\uc790\uc758 \ubb34\ub364\uc774\ub77c \uc804\ud558\ub294 '\uc2e0\ub77c\ub9c8\uc758\ud0dc\uc790\ub989(\u65b0\u7f85\u9ebb\u8863\u592a\u5b50\u9675)'\uc774\ub77c\ub294 \ube44\uc11d\uc774 \uc0c8\uaca8\uc9c4 \ubb34\ub364\uc774 \uc788\uace0, \ubc14\ub85c \uc606\uc5d0 \uadf8\uac00 \ud0c0\uace0 \ub2e4\ub2c8\ub358 \uc6a9\ub9c8(\u9f8d\u99ac)\uac00 \ubcc0\ud574 \ub3cc\uc774 \ub418\uc5c8\ub2e4\ub294 \uc804\uc124\uc774 \uc788\ub294 \uc6a9\ub9c8\uc11d(\u9f8d\u99ac\u77f3)\ub3c4 \uc788\ub2e4. \ubb34\ub364\uc740 \ub458\ub808 \uc57d 10\ubbf8\ud130, \ub192\uc774 1.5\ubbf8\ud130\ub85c \ubcf4\ud1b5 \ubb34\ub364\ubcf4\ub2e4 \uc870\uae08 \ud06c\ub2e4. \uac15\uc6d0\ub3c4 \uc778\uc81c\uad70\uc5d0\ub294 \uc0c1\ub0a8\uba74 \uc625\uc0c8\ubc14\uc704(\ub9c8\uc758\ud0dc\uc790\uac00 \uc625\uc0c8\ub97c \uc228\uacbc\ub2e4\uace0 \uc804\ud558\ub294 \ubc14\uc704), \uae40\ubd80\ub9ac(\u91d1\u5bcc\u5927\u738b\u6d1e) \uc5b4\uadc0\uc758 \uc218\uac70\ub108\uba38(\ub9c8\uc758\ud0dc\uc790\uac00 \uc218\ub808\ub97c \ud0c0\uace0 \ub118\uc5c8\ub2e4\ub294 \uace0\uac1c) \ub4f1 \ub9c8\uc758\ud0dc\uc790\uc640 \uad00\ub828\ub41c \uc9c0\uba85\ub4e4\uc774 \ub9ce\uc774 \ub0a8\uc544 \uc788\ub2e4. \uae40\ubd80\ub9ac\ub77c\ub294 \uc9c0\uba85\uc740 \u300a\uc624\uc8fc\uc5f0\ubb38\uc7a5\uc804\uc0b0\uace0\u300b(\u4e94\u6d32\u884d\u6587\u9577\u7b8b\u6563\u7a3f)\uc5d0 \uc2e0\ub77c \uacbd\uc21c\uc655(\uae40\ubd80\ub300\uc655)\uc774 \uc0b4\uc558\ub358 \ub370\uc11c \uc720\ub798\ud55c\ub2e4\uace0 \uc18c\uac1c\ub418\uc5b4 \uc788\uc9c0\ub9cc, \uae40\ubd80\ub9ac\uc758 \ub300\uc655\uac01(\u5927\u738b\u95a3)\uc5d0\ub294 \uacbd\uc21c\uc655\uc758 \ud0dc\uc790\ub77c\ub294 \uc778\ubb3c\uc774 \uc2e0\uc73c\ub85c \ubaa8\uc154\uc9c0\uace0 \uc788\uc73c\uba70, \ub300\uc655\uac01 \uc548\uc758 \uc704\ud328\uc5d0\ub294 \u300c\uc2e0\ub77c\uacbd\uc21c\ub300\uc655\ud0dc\uc790\uae40\uacf5\uc77c\uc9c0\uc2e0\uc704(\u65b0\u7f85\u656c\u9806\u5927\u738b\u592a\u5b50\u91d1\u516c\u93b0\u4e4b\u795e\u4f4d)\u300d\ub77c\uace0 \uc0c8\uaca8\uc838 \uc788\ub294\ub370, '\uae40\uc77c'(\u91d1\u93b0)\uc774\ub77c\ub294 \uc774\ub984\uc740 1940\ub144\ub300 \uc774\ud6c4\uc5d0\uc57c \uc704\ud328\uc5d0 \uc0c8\ub85c \ucd94\uac00\ud55c \uac83\uc774\ub2e4.", "answer": "\ud0dc\uc790", "sentence": "\ub2e4\ub9cc \ub9c8\uc758 \ud0dc\uc790 (\u9ebb\u8863 \u592a\u5b50)\uac00 \uc740\uac70\ud588\ub358 \uae08\uac15\uc0b0 \uc8fc\ubcc0\uc744 \uc911\uc2ec\uc73c\ub85c \ub9c8\uc758 \ud0dc\uc790\uc640 \uad00\ub828\ud55c \ubbfc\uac04 \uc804\uc2b9\uc774 \ub9ce\uc774 \uc804\ud574\uc9c0\uace0 \uc788\ub2e4.", "paragraph_sentence": " \ub2e4\ub9cc \ub9c8\uc758 \ud0dc\uc790 (\u9ebb\u8863 \u592a\u5b50)\uac00 \uc740\uac70\ud588\ub358 \uae08\uac15\uc0b0 \uc8fc\ubcc0\uc744 \uc911\uc2ec\uc73c\ub85c \ub9c8\uc758 \ud0dc\uc790\uc640 \uad00\ub828\ud55c \ubbfc\uac04 \uc804\uc2b9\uc774 \ub9ce\uc774 \uc804\ud574\uc9c0\uace0 \uc788\ub2e4. \ube44\ub85c\ubd09 \ubc14\ub85c \uc544\ub798\uc5d0\ub294 \ub9c8\uc758 \ud0dc\uc790\uc758 \ubb34\ub364\uc774\ub77c \uc804\ud558\ub294 '\uc2e0\ub77c\ub9c8\uc758\ud0dc\uc790\ub989(\u65b0\u7f85\u9ebb\u8863\u592a\u5b50\u9675)'\uc774\ub77c\ub294 \ube44\uc11d\uc774 \uc0c8\uaca8\uc9c4 \ubb34\ub364\uc774 \uc788\uace0, \ubc14\ub85c \uc606\uc5d0 \uadf8\uac00 \ud0c0\uace0 \ub2e4\ub2c8\ub358 \uc6a9\ub9c8(\u9f8d\u99ac)\uac00 \ubcc0\ud574 \ub3cc\uc774 \ub418\uc5c8\ub2e4\ub294 \uc804\uc124\uc774 \uc788\ub294 \uc6a9\ub9c8\uc11d(\u9f8d\u99ac\u77f3)\ub3c4 \uc788\ub2e4. \ubb34\ub364\uc740 \ub458\ub808 \uc57d 10\ubbf8\ud130, \ub192\uc774 1.5\ubbf8\ud130\ub85c \ubcf4\ud1b5 \ubb34\ub364\ubcf4\ub2e4 \uc870\uae08 \ud06c\ub2e4. \uac15\uc6d0\ub3c4 \uc778\uc81c\uad70\uc5d0\ub294 \uc0c1\ub0a8\uba74 \uc625\uc0c8\ubc14\uc704(\ub9c8\uc758\ud0dc\uc790\uac00 \uc625\uc0c8\ub97c \uc228\uacbc\ub2e4\uace0 \uc804\ud558\ub294 \ubc14\uc704), \uae40\ubd80\ub9ac(\u91d1\u5bcc\u5927\u738b\u6d1e) \uc5b4\uadc0\uc758 \uc218\uac70\ub108\uba38(\ub9c8\uc758\ud0dc\uc790\uac00 \uc218\ub808\ub97c \ud0c0\uace0 \ub118\uc5c8\ub2e4\ub294 \uace0\uac1c) \ub4f1 \ub9c8\uc758\ud0dc\uc790\uc640 \uad00\ub828\ub41c \uc9c0\uba85\ub4e4\uc774 \ub9ce\uc774 \ub0a8\uc544 \uc788\ub2e4. \uae40\ubd80\ub9ac\ub77c\ub294 \uc9c0\uba85\uc740 \u300a\uc624\uc8fc\uc5f0\ubb38\uc7a5\uc804\uc0b0\uace0\u300b(\u4e94\u6d32\u884d\u6587\u9577\u7b8b\u6563\u7a3f)\uc5d0 \uc2e0\ub77c \uacbd\uc21c\uc655(\uae40\ubd80\ub300\uc655)\uc774 \uc0b4\uc558\ub358 \ub370\uc11c \uc720\ub798\ud55c\ub2e4\uace0 \uc18c\uac1c\ub418\uc5b4 \uc788\uc9c0\ub9cc, \uae40\ubd80\ub9ac\uc758 \ub300\uc655\uac01(\u5927\u738b\u95a3)\uc5d0\ub294 \uacbd\uc21c\uc655\uc758 \ud0dc\uc790\ub77c\ub294 \uc778\ubb3c\uc774 \uc2e0\uc73c\ub85c \ubaa8\uc154\uc9c0\uace0 \uc788\uc73c\uba70, \ub300\uc655\uac01 \uc548\uc758 \uc704\ud328\uc5d0\ub294 \u300c\uc2e0\ub77c\uacbd\uc21c\ub300\uc655\ud0dc\uc790\uae40\uacf5\uc77c\uc9c0\uc2e0\uc704(\u65b0\u7f85\u656c\u9806\u5927\u738b\u592a\u5b50\u91d1\u516c\u93b0\u4e4b\u795e\u4f4d)\u300d\ub77c\uace0 \uc0c8\uaca8\uc838 \uc788\ub294\ub370, '\uae40\uc77c'(\u91d1\u93b0)\uc774\ub77c\ub294 \uc774\ub984\uc740 1940\ub144\ub300 \uc774\ud6c4\uc5d0\uc57c \uc704\ud328\uc5d0 \uc0c8\ub85c \ucd94\uac00\ud55c \uac83\uc774\ub2e4.", "paragraph_answer": "\ub2e4\ub9cc \ub9c8\uc758 \ud0dc\uc790 (\u9ebb\u8863 \u592a\u5b50)\uac00 \uc740\uac70\ud588\ub358 \uae08\uac15\uc0b0 \uc8fc\ubcc0\uc744 \uc911\uc2ec\uc73c\ub85c \ub9c8\uc758 \ud0dc\uc790\uc640 \uad00\ub828\ud55c \ubbfc\uac04 \uc804\uc2b9\uc774 \ub9ce\uc774 \uc804\ud574\uc9c0\uace0 \uc788\ub2e4. \ube44\ub85c\ubd09 \ubc14\ub85c \uc544\ub798\uc5d0\ub294 \ub9c8\uc758 \ud0dc\uc790\uc758 \ubb34\ub364\uc774\ub77c \uc804\ud558\ub294 '\uc2e0\ub77c\ub9c8\uc758\ud0dc\uc790\ub989(\u65b0\u7f85\u9ebb\u8863\u592a\u5b50\u9675)'\uc774\ub77c\ub294 \ube44\uc11d\uc774 \uc0c8\uaca8\uc9c4 \ubb34\ub364\uc774 \uc788\uace0, \ubc14\ub85c \uc606\uc5d0 \uadf8\uac00 \ud0c0\uace0 \ub2e4\ub2c8\ub358 \uc6a9\ub9c8(\u9f8d\u99ac)\uac00 \ubcc0\ud574 \ub3cc\uc774 \ub418\uc5c8\ub2e4\ub294 \uc804\uc124\uc774 \uc788\ub294 \uc6a9\ub9c8\uc11d(\u9f8d\u99ac\u77f3)\ub3c4 \uc788\ub2e4. \ubb34\ub364\uc740 \ub458\ub808 \uc57d 10\ubbf8\ud130, \ub192\uc774 1.5\ubbf8\ud130\ub85c \ubcf4\ud1b5 \ubb34\ub364\ubcf4\ub2e4 \uc870\uae08 \ud06c\ub2e4. \uac15\uc6d0\ub3c4 \uc778\uc81c\uad70\uc5d0\ub294 \uc0c1\ub0a8\uba74 \uc625\uc0c8\ubc14\uc704(\ub9c8\uc758\ud0dc\uc790\uac00 \uc625\uc0c8\ub97c \uc228\uacbc\ub2e4\uace0 \uc804\ud558\ub294 \ubc14\uc704), \uae40\ubd80\ub9ac(\u91d1\u5bcc\u5927\u738b\u6d1e) \uc5b4\uadc0\uc758 \uc218\uac70\ub108\uba38(\ub9c8\uc758\ud0dc\uc790\uac00 \uc218\ub808\ub97c \ud0c0\uace0 \ub118\uc5c8\ub2e4\ub294 \uace0\uac1c) \ub4f1 \ub9c8\uc758\ud0dc\uc790\uc640 \uad00\ub828\ub41c \uc9c0\uba85\ub4e4\uc774 \ub9ce\uc774 \ub0a8\uc544 \uc788\ub2e4. \uae40\ubd80\ub9ac\ub77c\ub294 \uc9c0\uba85\uc740 \u300a\uc624\uc8fc\uc5f0\ubb38\uc7a5\uc804\uc0b0\uace0\u300b(\u4e94\u6d32\u884d\u6587\u9577\u7b8b\u6563\u7a3f)\uc5d0 \uc2e0\ub77c \uacbd\uc21c\uc655(\uae40\ubd80\ub300\uc655)\uc774 \uc0b4\uc558\ub358 \ub370\uc11c \uc720\ub798\ud55c\ub2e4\uace0 \uc18c\uac1c\ub418\uc5b4 \uc788\uc9c0\ub9cc, \uae40\ubd80\ub9ac\uc758 \ub300\uc655\uac01(\u5927\u738b\u95a3)\uc5d0\ub294 \uacbd\uc21c\uc655\uc758 \ud0dc\uc790\ub77c\ub294 \uc778\ubb3c\uc774 \uc2e0\uc73c\ub85c \ubaa8\uc154\uc9c0\uace0 \uc788\uc73c\uba70, \ub300\uc655\uac01 \uc548\uc758 \uc704\ud328\uc5d0\ub294 \u300c\uc2e0\ub77c\uacbd\uc21c\ub300\uc655\ud0dc\uc790\uae40\uacf5\uc77c\uc9c0\uc2e0\uc704(\u65b0\u7f85\u656c\u9806\u5927\u738b\u592a\u5b50\u91d1\u516c\u93b0\u4e4b\u795e\u4f4d)\u300d\ub77c\uace0 \uc0c8\uaca8\uc838 \uc788\ub294\ub370, '\uae40\uc77c'(\u91d1\u93b0)\uc774\ub77c\ub294 \uc774\ub984\uc740 1940\ub144\ub300 \uc774\ud6c4\uc5d0\uc57c \uc704\ud328\uc5d0 \uc0c8\ub85c \ucd94\uac00\ud55c \uac83\uc774\ub2e4.", "sentence_answer": "\ub2e4\ub9cc \ub9c8\uc758 \ud0dc\uc790 (\u9ebb\u8863 \u592a\u5b50)\uac00 \uc740\uac70\ud588\ub358 \uae08\uac15\uc0b0 \uc8fc\ubcc0\uc744 \uc911\uc2ec\uc73c\ub85c \ub9c8\uc758 \ud0dc\uc790\uc640 \uad00\ub828\ud55c \ubbfc\uac04 \uc804\uc2b9\uc774 \ub9ce\uc774 \uc804\ud574\uc9c0\uace0 \uc788\ub2e4.", "paragraph_id": "6500216-0-0", "paragraph_question": "question: \uae40\ubd80\ub9ac\uc758 \ub300\uc655\uac01\uc5d0 \uc2e0\uc73c\ub85c \ubaa8\uc154\uc9c0\uace0 \uc788\ub294 \uc778\ubb3c\uc740?, context: \ub2e4\ub9cc \ub9c8\uc758 \ud0dc\uc790(\u9ebb\u8863 \u592a\u5b50)\uac00 \uc740\uac70\ud588\ub358 \uae08\uac15\uc0b0 \uc8fc\ubcc0\uc744 \uc911\uc2ec\uc73c\ub85c \ub9c8\uc758 \ud0dc\uc790\uc640 \uad00\ub828\ud55c \ubbfc\uac04 \uc804\uc2b9\uc774 \ub9ce\uc774 \uc804\ud574\uc9c0\uace0 \uc788\ub2e4. \ube44\ub85c\ubd09 \ubc14\ub85c \uc544\ub798\uc5d0\ub294 \ub9c8\uc758 \ud0dc\uc790\uc758 \ubb34\ub364\uc774\ub77c \uc804\ud558\ub294 '\uc2e0\ub77c\ub9c8\uc758\ud0dc\uc790\ub989(\u65b0\u7f85\u9ebb\u8863\u592a\u5b50\u9675)'\uc774\ub77c\ub294 \ube44\uc11d\uc774 \uc0c8\uaca8\uc9c4 \ubb34\ub364\uc774 \uc788\uace0, \ubc14\ub85c \uc606\uc5d0 \uadf8\uac00 \ud0c0\uace0 \ub2e4\ub2c8\ub358 \uc6a9\ub9c8(\u9f8d\u99ac)\uac00 \ubcc0\ud574 \ub3cc\uc774 \ub418\uc5c8\ub2e4\ub294 \uc804\uc124\uc774 \uc788\ub294 \uc6a9\ub9c8\uc11d(\u9f8d\u99ac\u77f3)\ub3c4 \uc788\ub2e4. \ubb34\ub364\uc740 \ub458\ub808 \uc57d 10\ubbf8\ud130, \ub192\uc774 1.5\ubbf8\ud130\ub85c \ubcf4\ud1b5 \ubb34\ub364\ubcf4\ub2e4 \uc870\uae08 \ud06c\ub2e4. \uac15\uc6d0\ub3c4 \uc778\uc81c\uad70\uc5d0\ub294 \uc0c1\ub0a8\uba74 \uc625\uc0c8\ubc14\uc704(\ub9c8\uc758\ud0dc\uc790\uac00 \uc625\uc0c8\ub97c \uc228\uacbc\ub2e4\uace0 \uc804\ud558\ub294 \ubc14\uc704), \uae40\ubd80\ub9ac(\u91d1\u5bcc\u5927\u738b\u6d1e) \uc5b4\uadc0\uc758 \uc218\uac70\ub108\uba38(\ub9c8\uc758\ud0dc\uc790\uac00 \uc218\ub808\ub97c \ud0c0\uace0 \ub118\uc5c8\ub2e4\ub294 \uace0\uac1c) \ub4f1 \ub9c8\uc758\ud0dc\uc790\uc640 \uad00\ub828\ub41c \uc9c0\uba85\ub4e4\uc774 \ub9ce\uc774 \ub0a8\uc544 \uc788\ub2e4. \uae40\ubd80\ub9ac\ub77c\ub294 \uc9c0\uba85\uc740 \u300a\uc624\uc8fc\uc5f0\ubb38\uc7a5\uc804\uc0b0\uace0\u300b(\u4e94\u6d32\u884d\u6587\u9577\u7b8b\u6563\u7a3f)\uc5d0 \uc2e0\ub77c \uacbd\uc21c\uc655(\uae40\ubd80\ub300\uc655)\uc774 \uc0b4\uc558\ub358 \ub370\uc11c \uc720\ub798\ud55c\ub2e4\uace0 \uc18c\uac1c\ub418\uc5b4 \uc788\uc9c0\ub9cc, \uae40\ubd80\ub9ac\uc758 \ub300\uc655\uac01(\u5927\u738b\u95a3)\uc5d0\ub294 \uacbd\uc21c\uc655\uc758 \ud0dc\uc790\ub77c\ub294 \uc778\ubb3c\uc774 \uc2e0\uc73c\ub85c \ubaa8\uc154\uc9c0\uace0 \uc788\uc73c\uba70, \ub300\uc655\uac01 \uc548\uc758 \uc704\ud328\uc5d0\ub294 \u300c\uc2e0\ub77c\uacbd\uc21c\ub300\uc655\ud0dc\uc790\uae40\uacf5\uc77c\uc9c0\uc2e0\uc704(\u65b0\u7f85\u656c\u9806\u5927\u738b\u592a\u5b50\u91d1\u516c\u93b0\u4e4b\u795e\u4f4d)\u300d\ub77c\uace0 \uc0c8\uaca8\uc838 \uc788\ub294\ub370, '\uae40\uc77c'(\u91d1\u93b0)\uc774\ub77c\ub294 \uc774\ub984\uc740 1940\ub144\ub300 \uc774\ud6c4\uc5d0\uc57c \uc704\ud328\uc5d0 \uc0c8\ub85c \ucd94\uac00\ud55c \uac83\uc774\ub2e4."} {"question": "1697\ub144 \uac1c\uc2dc\ub97c \uc5f4\uc5b4 \uccad\ub098\ub77c\uc758 \uace1\uc2dd\uc744 \uc218\uc785\ud560 \uac83\uc744 \uc8fc\uc7a5\ud55c \uc0ac\ub78c\uc740?", "paragraph": "\ud55c\ud3b8 \uc219\uc885\uc740 \uccad\ub098\ub77c \uc870\uc815\uc5d0 \uccad\ud558\uc5ec \uc870\uc120\uacfc\uc758 \uad50\uc5ed\ub7c9\uc744 \ub298\ub9ac\uace0 \uc2dd\ub7c9\uc758 \uc9c0\uc6d0\uc744 \uc694\uccad\ud55c\ub2e4. \uad6d\ub0b4\uc5d0 \uace1\uc2dd\uc774 \uc5c6\uc73c\uba74 \uc784\uc9c4\uc65c\ub780 \ub54c \uc720\uc131\ub8e1\uc774 \uc555\ub85d\uac15 \uc911\uac15\uc9c4\uc5d0 \uad6d\uc81c \ubb34\uc5ed\uc2dc\uc7a5\uc778 \uc911\uac15\uac1c\uc2dc\ub97c \uc5f4\uc5b4 \uba85(\u660e)\uc758 \uace1\ubb3c\uc744 \ub4e4\uc5ec\uc628 \uac83\ucc98\ub7fc \uccad\uc758 \uace1\uc2dd\uc744 \ub4e4\uc5ec\uc640 \uae30\ubbfc\uc744 \uad6c\uc81c\ud574\uc57c \ud588\ub2e4. 1697\ub144(\uc219\uc885 23\ub144) 5\uc6d4 12\uc77c \ub300\uc0ac\uac04 \ubc15\ud0dc\uc21c\uc774 \uac1c\uc2dc\ub97c \uc5f4\uc5b4 \uccad\ub098\ub77c\uc758 \uace1\uc2dd\uc744 \uc218\uc785\ud560 \uac83\uc744 \uc8fc\uc7a5\ud588\uc73c\ub098 4\uac1c\uc6d4 \ud6c4\uc778 9\uc6d4 21\uc77c\uc5d0\uc57c \uc774 \ubb38\uc81c\uac00 \uc870\uc815\uc5d0\uc11c \ub17c\uc758\ub418\uc5c8\ub2e4. \ucc2c\ubc18 \uc591\ub860\uc774 \uac08\ub824 \uac11\ub860\uc744\ubc15\ud558\ub2e4\uac00 \ubcf8\uaca9\uc801\uc778 \uad50\uc5ed\uc740 \ub098\ub77c\uac00 \u2018\uc18c\uc2dd(\u8607\u606f:\uc228\ud1b5\uc774 \ud2b8\uc784)\ub418\uae30\ub97c \uae30\ub2e4\ub824 \ud558\uc790\u2019\uace0 \uc720\ubcf4\ud558\uba74\uc11c\ub3c4 \uc77c\ub2e8 \uc7ac\uc790\uad00(\uc0ac\uc2e0\uc758 \uc77c\uc885)\uc744 \ud30c\uacac\ud574 \uace1\uc2dd\ub9cc \uba3c\uc800 \uad50\uc5ed\ud558\uc790\uace0 \uccad\ud55c \uac83\uc774 \ub2e4\ud589\uc774\uc5c8\ub2e4. \uadf8\ub798\uc11c 1698\ub144(\uc219\uc885 24\ub144) 1\uc6d4 \uccad\ub098\ub77c\uc5d0\uc11c \uc881\uc300 4\ub9cc \uc11d\uc774 \ub4e4\uc5b4\uc640 \uc11c\uc6b8\u00b7\uacbd\uae30\u00b7\ucda9\uccad\u00b7\uc11c\ub85c(\ud3c9\uc548\ub3c4\u00b7\ud669\ud574\ub3c4)\uc5d0 1\ub9cc \uc11d\uc529\uc744 \ub098\ub204\uc5b4\uc8fc\uc5b4 \uae09\ud55c \ubd88\uc744 \ub04c \uc218 \uc788\uc5c8\ub2e4. \uadf8\ub7ec\ub098 \ud749\ub144\uacfc \ud749\uc791\uc740 \uacc4\uc18d\ub418\uc5c8\uace0, \ub3d9\uc2dc\uc5d0 \uc591\ubc18\ub4e4\uacfc \ud1a0\ud638\ub4e4\uc758 \uc218\ud0c8\uacfc \ud6a1\ud3ec\ub3c4 \uc2ec\ud574\uc838 \ubbfc\ub780\uacfc \ub3c4\uc8fc \uc0ac\ud0dc\uac00 \ubc1c\uc0dd\ud558\uac8c \ub41c\ub2e4.", "answer": "\ubc15\ud0dc\uc21c", "sentence": "1697\ub144(\uc219\uc885 23\ub144) 5\uc6d4 12\uc77c \ub300\uc0ac\uac04 \ubc15\ud0dc\uc21c \uc774 \uac1c\uc2dc\ub97c \uc5f4\uc5b4 \uccad\ub098\ub77c\uc758 \uace1\uc2dd\uc744 \uc218\uc785\ud560 \uac83\uc744 \uc8fc\uc7a5\ud588\uc73c\ub098 4\uac1c\uc6d4 \ud6c4\uc778 9\uc6d4 21\uc77c\uc5d0\uc57c \uc774 \ubb38\uc81c\uac00 \uc870\uc815\uc5d0\uc11c \ub17c\uc758\ub418\uc5c8\ub2e4.", "paragraph_sentence": "\ud55c\ud3b8 \uc219\uc885\uc740 \uccad\ub098\ub77c \uc870\uc815\uc5d0 \uccad\ud558\uc5ec \uc870\uc120\uacfc\uc758 \uad50\uc5ed\ub7c9\uc744 \ub298\ub9ac\uace0 \uc2dd\ub7c9\uc758 \uc9c0\uc6d0\uc744 \uc694\uccad\ud55c\ub2e4. \uad6d\ub0b4\uc5d0 \uace1\uc2dd\uc774 \uc5c6\uc73c\uba74 \uc784\uc9c4\uc65c\ub780 \ub54c \uc720\uc131\ub8e1\uc774 \uc555\ub85d\uac15 \uc911\uac15\uc9c4\uc5d0 \uad6d\uc81c \ubb34\uc5ed\uc2dc\uc7a5\uc778 \uc911\uac15\uac1c\uc2dc\ub97c \uc5f4\uc5b4 \uba85(\u660e)\uc758 \uace1\ubb3c\uc744 \ub4e4\uc5ec\uc628 \uac83\ucc98\ub7fc \uccad\uc758 \uace1\uc2dd\uc744 \ub4e4\uc5ec\uc640 \uae30\ubbfc\uc744 \uad6c\uc81c\ud574\uc57c \ud588\ub2e4. 1697\ub144(\uc219\uc885 23\ub144) 5\uc6d4 12\uc77c \ub300\uc0ac\uac04 \ubc15\ud0dc\uc21c \uc774 \uac1c\uc2dc\ub97c \uc5f4\uc5b4 \uccad\ub098\ub77c\uc758 \uace1\uc2dd\uc744 \uc218\uc785\ud560 \uac83\uc744 \uc8fc\uc7a5\ud588\uc73c\ub098 4\uac1c\uc6d4 \ud6c4\uc778 9\uc6d4 21\uc77c\uc5d0\uc57c \uc774 \ubb38\uc81c\uac00 \uc870\uc815\uc5d0\uc11c \ub17c\uc758\ub418\uc5c8\ub2e4. \ucc2c\ubc18 \uc591\ub860\uc774 \uac08\ub824 \uac11\ub860\uc744\ubc15\ud558\ub2e4\uac00 \ubcf8\uaca9\uc801\uc778 \uad50\uc5ed\uc740 \ub098\ub77c\uac00 \u2018\uc18c\uc2dd(\u8607\u606f:\uc228\ud1b5\uc774 \ud2b8\uc784)\ub418\uae30\ub97c \uae30\ub2e4\ub824 \ud558\uc790\u2019\uace0 \uc720\ubcf4\ud558\uba74\uc11c\ub3c4 \uc77c\ub2e8 \uc7ac\uc790\uad00(\uc0ac\uc2e0\uc758 \uc77c\uc885)\uc744 \ud30c\uacac\ud574 \uace1\uc2dd\ub9cc \uba3c\uc800 \uad50\uc5ed\ud558\uc790\uace0 \uccad\ud55c \uac83\uc774 \ub2e4\ud589\uc774\uc5c8\ub2e4. \uadf8\ub798\uc11c 1698\ub144(\uc219\uc885 24\ub144) 1\uc6d4 \uccad\ub098\ub77c\uc5d0\uc11c \uc881\uc300 4\ub9cc \uc11d\uc774 \ub4e4\uc5b4\uc640 \uc11c\uc6b8\u00b7\uacbd\uae30\u00b7\ucda9\uccad\u00b7\uc11c\ub85c(\ud3c9\uc548\ub3c4\u00b7\ud669\ud574\ub3c4)\uc5d0 1\ub9cc \uc11d\uc529\uc744 \ub098\ub204\uc5b4\uc8fc\uc5b4 \uae09\ud55c \ubd88\uc744 \ub04c \uc218 \uc788\uc5c8\ub2e4. \uadf8\ub7ec\ub098 \ud749\ub144\uacfc \ud749\uc791\uc740 \uacc4\uc18d\ub418\uc5c8\uace0, \ub3d9\uc2dc\uc5d0 \uc591\ubc18\ub4e4\uacfc \ud1a0\ud638\ub4e4\uc758 \uc218\ud0c8\uacfc \ud6a1\ud3ec\ub3c4 \uc2ec\ud574\uc838 \ubbfc\ub780\uacfc \ub3c4\uc8fc \uc0ac\ud0dc\uac00 \ubc1c\uc0dd\ud558\uac8c \ub41c\ub2e4.", "paragraph_answer": "\ud55c\ud3b8 \uc219\uc885\uc740 \uccad\ub098\ub77c \uc870\uc815\uc5d0 \uccad\ud558\uc5ec \uc870\uc120\uacfc\uc758 \uad50\uc5ed\ub7c9\uc744 \ub298\ub9ac\uace0 \uc2dd\ub7c9\uc758 \uc9c0\uc6d0\uc744 \uc694\uccad\ud55c\ub2e4. \uad6d\ub0b4\uc5d0 \uace1\uc2dd\uc774 \uc5c6\uc73c\uba74 \uc784\uc9c4\uc65c\ub780 \ub54c \uc720\uc131\ub8e1\uc774 \uc555\ub85d\uac15 \uc911\uac15\uc9c4\uc5d0 \uad6d\uc81c \ubb34\uc5ed\uc2dc\uc7a5\uc778 \uc911\uac15\uac1c\uc2dc\ub97c \uc5f4\uc5b4 \uba85(\u660e)\uc758 \uace1\ubb3c\uc744 \ub4e4\uc5ec\uc628 \uac83\ucc98\ub7fc \uccad\uc758 \uace1\uc2dd\uc744 \ub4e4\uc5ec\uc640 \uae30\ubbfc\uc744 \uad6c\uc81c\ud574\uc57c \ud588\ub2e4. 1697\ub144(\uc219\uc885 23\ub144) 5\uc6d4 12\uc77c \ub300\uc0ac\uac04 \ubc15\ud0dc\uc21c \uc774 \uac1c\uc2dc\ub97c \uc5f4\uc5b4 \uccad\ub098\ub77c\uc758 \uace1\uc2dd\uc744 \uc218\uc785\ud560 \uac83\uc744 \uc8fc\uc7a5\ud588\uc73c\ub098 4\uac1c\uc6d4 \ud6c4\uc778 9\uc6d4 21\uc77c\uc5d0\uc57c \uc774 \ubb38\uc81c\uac00 \uc870\uc815\uc5d0\uc11c \ub17c\uc758\ub418\uc5c8\ub2e4. \ucc2c\ubc18 \uc591\ub860\uc774 \uac08\ub824 \uac11\ub860\uc744\ubc15\ud558\ub2e4\uac00 \ubcf8\uaca9\uc801\uc778 \uad50\uc5ed\uc740 \ub098\ub77c\uac00 \u2018\uc18c\uc2dd(\u8607\u606f:\uc228\ud1b5\uc774 \ud2b8\uc784)\ub418\uae30\ub97c \uae30\ub2e4\ub824 \ud558\uc790\u2019\uace0 \uc720\ubcf4\ud558\uba74\uc11c\ub3c4 \uc77c\ub2e8 \uc7ac\uc790\uad00(\uc0ac\uc2e0\uc758 \uc77c\uc885)\uc744 \ud30c\uacac\ud574 \uace1\uc2dd\ub9cc \uba3c\uc800 \uad50\uc5ed\ud558\uc790\uace0 \uccad\ud55c \uac83\uc774 \ub2e4\ud589\uc774\uc5c8\ub2e4. \uadf8\ub798\uc11c 1698\ub144(\uc219\uc885 24\ub144) 1\uc6d4 \uccad\ub098\ub77c\uc5d0\uc11c \uc881\uc300 4\ub9cc \uc11d\uc774 \ub4e4\uc5b4\uc640 \uc11c\uc6b8\u00b7\uacbd\uae30\u00b7\ucda9\uccad\u00b7\uc11c\ub85c(\ud3c9\uc548\ub3c4\u00b7\ud669\ud574\ub3c4)\uc5d0 1\ub9cc \uc11d\uc529\uc744 \ub098\ub204\uc5b4\uc8fc\uc5b4 \uae09\ud55c \ubd88\uc744 \ub04c \uc218 \uc788\uc5c8\ub2e4. \uadf8\ub7ec\ub098 \ud749\ub144\uacfc \ud749\uc791\uc740 \uacc4\uc18d\ub418\uc5c8\uace0, \ub3d9\uc2dc\uc5d0 \uc591\ubc18\ub4e4\uacfc \ud1a0\ud638\ub4e4\uc758 \uc218\ud0c8\uacfc \ud6a1\ud3ec\ub3c4 \uc2ec\ud574\uc838 \ubbfc\ub780\uacfc \ub3c4\uc8fc \uc0ac\ud0dc\uac00 \ubc1c\uc0dd\ud558\uac8c \ub41c\ub2e4.", "sentence_answer": "1697\ub144(\uc219\uc885 23\ub144) 5\uc6d4 12\uc77c \ub300\uc0ac\uac04 \ubc15\ud0dc\uc21c \uc774 \uac1c\uc2dc\ub97c \uc5f4\uc5b4 \uccad\ub098\ub77c\uc758 \uace1\uc2dd\uc744 \uc218\uc785\ud560 \uac83\uc744 \uc8fc\uc7a5\ud588\uc73c\ub098 4\uac1c\uc6d4 \ud6c4\uc778 9\uc6d4 21\uc77c\uc5d0\uc57c \uc774 \ubb38\uc81c\uac00 \uc870\uc815\uc5d0\uc11c \ub17c\uc758\ub418\uc5c8\ub2e4.", "paragraph_id": "5876576-7-1", "paragraph_question": "question: 1697\ub144 \uac1c\uc2dc\ub97c \uc5f4\uc5b4 \uccad\ub098\ub77c\uc758 \uace1\uc2dd\uc744 \uc218\uc785\ud560 \uac83\uc744 \uc8fc\uc7a5\ud55c \uc0ac\ub78c\uc740?, context: \ud55c\ud3b8 \uc219\uc885\uc740 \uccad\ub098\ub77c \uc870\uc815\uc5d0 \uccad\ud558\uc5ec \uc870\uc120\uacfc\uc758 \uad50\uc5ed\ub7c9\uc744 \ub298\ub9ac\uace0 \uc2dd\ub7c9\uc758 \uc9c0\uc6d0\uc744 \uc694\uccad\ud55c\ub2e4. \uad6d\ub0b4\uc5d0 \uace1\uc2dd\uc774 \uc5c6\uc73c\uba74 \uc784\uc9c4\uc65c\ub780 \ub54c \uc720\uc131\ub8e1\uc774 \uc555\ub85d\uac15 \uc911\uac15\uc9c4\uc5d0 \uad6d\uc81c \ubb34\uc5ed\uc2dc\uc7a5\uc778 \uc911\uac15\uac1c\uc2dc\ub97c \uc5f4\uc5b4 \uba85(\u660e)\uc758 \uace1\ubb3c\uc744 \ub4e4\uc5ec\uc628 \uac83\ucc98\ub7fc \uccad\uc758 \uace1\uc2dd\uc744 \ub4e4\uc5ec\uc640 \uae30\ubbfc\uc744 \uad6c\uc81c\ud574\uc57c \ud588\ub2e4. 1697\ub144(\uc219\uc885 23\ub144) 5\uc6d4 12\uc77c \ub300\uc0ac\uac04 \ubc15\ud0dc\uc21c\uc774 \uac1c\uc2dc\ub97c \uc5f4\uc5b4 \uccad\ub098\ub77c\uc758 \uace1\uc2dd\uc744 \uc218\uc785\ud560 \uac83\uc744 \uc8fc\uc7a5\ud588\uc73c\ub098 4\uac1c\uc6d4 \ud6c4\uc778 9\uc6d4 21\uc77c\uc5d0\uc57c \uc774 \ubb38\uc81c\uac00 \uc870\uc815\uc5d0\uc11c \ub17c\uc758\ub418\uc5c8\ub2e4. \ucc2c\ubc18 \uc591\ub860\uc774 \uac08\ub824 \uac11\ub860\uc744\ubc15\ud558\ub2e4\uac00 \ubcf8\uaca9\uc801\uc778 \uad50\uc5ed\uc740 \ub098\ub77c\uac00 \u2018\uc18c\uc2dd(\u8607\u606f:\uc228\ud1b5\uc774 \ud2b8\uc784)\ub418\uae30\ub97c \uae30\ub2e4\ub824 \ud558\uc790\u2019\uace0 \uc720\ubcf4\ud558\uba74\uc11c\ub3c4 \uc77c\ub2e8 \uc7ac\uc790\uad00(\uc0ac\uc2e0\uc758 \uc77c\uc885)\uc744 \ud30c\uacac\ud574 \uace1\uc2dd\ub9cc \uba3c\uc800 \uad50\uc5ed\ud558\uc790\uace0 \uccad\ud55c \uac83\uc774 \ub2e4\ud589\uc774\uc5c8\ub2e4. \uadf8\ub798\uc11c 1698\ub144(\uc219\uc885 24\ub144) 1\uc6d4 \uccad\ub098\ub77c\uc5d0\uc11c \uc881\uc300 4\ub9cc \uc11d\uc774 \ub4e4\uc5b4\uc640 \uc11c\uc6b8\u00b7\uacbd\uae30\u00b7\ucda9\uccad\u00b7\uc11c\ub85c(\ud3c9\uc548\ub3c4\u00b7\ud669\ud574\ub3c4)\uc5d0 1\ub9cc \uc11d\uc529\uc744 \ub098\ub204\uc5b4\uc8fc\uc5b4 \uae09\ud55c \ubd88\uc744 \ub04c \uc218 \uc788\uc5c8\ub2e4. \uadf8\ub7ec\ub098 \ud749\ub144\uacfc \ud749\uc791\uc740 \uacc4\uc18d\ub418\uc5c8\uace0, \ub3d9\uc2dc\uc5d0 \uc591\ubc18\ub4e4\uacfc \ud1a0\ud638\ub4e4\uc758 \uc218\ud0c8\uacfc \ud6a1\ud3ec\ub3c4 \uc2ec\ud574\uc838 \ubbfc\ub780\uacfc \ub3c4\uc8fc \uc0ac\ud0dc\uac00 \ubc1c\uc0dd\ud558\uac8c \ub41c\ub2e4."} {"question": "\uc8fc\uad50\uc7a5\uc758 \ub2e8\uc7a5\uc740?", "paragraph": ". . . \uadf8\ub9ac\uace0 \ud558\ub290\ub2d8\uc774\uc2e0 \uad6c\uc138\uc8fc\uaed8\uc11c \ub2f9\uc2e0 \uad50\ud68c\uac00 \uc2e0\uc559\uacfc \ub3c4\ub355\uc5d0 \uad00\ud55c \uad50\ub9ac\uc758 \uacb0\uc815\uc5d0\uc11c \uc624\ub958\uac00 \uc5c6\uae30\ub97c \ubc14\ub77c\uc168\ub358 \uc774 \ubb34\ub958\uc131\uc740 \uad50\ud68c\uac00 \uac70\ub8e9\ud558\uac8c \ubcf4\uc804\ud558\uace0 \ucda9\uc2e4\ud788 \uc124\uba85\ud558\uc5ec\uc57c \ud560 \ud558\ub290\ub2d8 \uacc4\uc2dc\uc758 \uc704\ud0c1\uc774 \ud3bc\uccd0\uc9c0\ub294 \uadf8\ub9cc\ud07c \ud3bc\uccd0\uc9c4\ub2e4. \uc8fc\uad50\ub2e8\uc758 \ub2e8\uc7a5\uc778 \uad50\ud669\uc740 \ucc38\uc73c\ub85c \uc2e0\uc559 \uc548\uc5d0\uc11c \uc790\uae30 \ud615\uc81c\ub4e4\uc758 \ud798\uc744 \ubd81\ub3cb\uc6cc \uc8fc\ub294 \uc0ac\ub78c\uc774\ubbc0\ub85c(\ub8e8\uce74 22,32 \ucc38\uc870), \ubaa8\ub4e0 \uadf8\ub9ac\uc2a4\ub3c4\uc778\uc758 \ucd5c\uace0 \ubaa9\uc790\uc774\uba70 \uc2a4\uc2b9\uc73c\ub85c\uc11c \uc2e0\uc559\uacfc \ub3c4\ub355\uc5d0 \uad00\ud55c \uad50\ub9ac\ub97c \ud655\uc815\uc801 \ud589\uc704\ub85c \uc120\uc5b8\ud558\ub294 \ub54c\uc5d0, \uad50\ud669\uc740 \uc790\uae30 \uc784\ubb34\uc5d0 \ub530\ub77c \uadf8 \ubb34\ub958\uc131\uc744 \uc9c0\ub2cc\ub2e4. \uadf8\ub7ec\ubbc0\ub85c \uad50\ud669\uc758 \uacb0\uc815\uc740 \uad50\ud68c\uc758 \ub3d9\uc758 \ub54c\ubb38\uc774 \uc544\ub2c8\ub77c \uadf8 \uc790\uccb4\ub85c\uc11c \ub9c8\ub545\ud788 \ubc14\ub014 \uc218 \uc5c6\ub294 \uac83\uc73c\ub85c \uc5ec\uaca8\uc9c4\ub2e4. \uadf8\uac83\uc740 \ubcf5\ub41c \ubca0\ub4dc\ub85c \uc548\uc5d0\uc11c \uad50\ud669\uc5d0\uac8c \uc57d\uc18d\ub41c \uc131\ub839\uc758 \ub3c4\uc6c0\uc744 \ubc1b\uc544 \uc120\ud3ec\ub41c \uac83\uc774\uae30 \ub54c\ubb38\uc774\ub2e4. \ub610\ud55c \uadf8 \uacb0\uc815\uc740 \uacb0\ucf54 \ub2e4\ub978 \ub204\uad6c\uc758 \uc2b9\uc778\ub3c4 \ud544\uc694\ud558\uc9c0 \uc54a\uace0 \ub2e4\ub978 \ud310\ub2e8\uc744 \uc694\uad6c\ud558\ub294 \uc5b4\ub5a0\ud55c \uc0c1\uc18c\ub3c4 \ud5c8\uc6a9\ub418\uc9c0 \uc54a\ub294\ub2e4. \uadf8\ub7ec\ud560 \ub54c\uc5d0 \uad50\ud669\uc740 \ud55c \uac1c\uc778\uc73c\ub85c\uc11c \ud310\ub2e8\uc744 \ub0b4\ub9ac\ub294 \uac83\uc774 \uc544\ub2c8\ub77c \ubc14\ub85c \uad50\ud68c \uc790\uccb4\uc758 \ubb34\ub958\uc131\uc758 \uc740\uc0ac\ub97c \ud2b9\ubcc4\ud788 \uc9c0\ub2c8\uace0 \uc788\ub294 \ubcf4\ud3b8 \uad50\ud68c\uc758 \ucd5c\uace0 \uc2a4\uc2b9\uc73c\ub85c\uc11c \uac00\ud1a8\ub9ad \uc2e0\uc559\uc758 \uad50\ub9ac\ub97c \uc124\uba85\ud558\uace0 \uc639\ud638\ud558\ub294 \uac83\uc774\ub2e4. \uad50\ud68c\uc5d0 \uc57d\uc18d\ub41c \ubb34\ub958\uc131\uc740 \uc8fc\uad50\ub2e8\uc774 \ubca0\ub4dc\ub85c\uc758 \ud6c4\uacc4\uc790\uc640 \ub354\ubd88\uc5b4 \ucd5c\uace0 \uad50\ub3c4\uad8c\uc744 \ud589\uc0ac\ud560 \ub54c\uc5d0 \uc8fc\uad50\ub2e8 \uc548\uc5d0\ub3c4 \ub0b4\uc7ac\ud55c\ub2e4. \uc774\ub7ec\ud55c \uacb0\uc815\uc5d0 \ub300\ud558\uc5ec \uad50\ud68c\uc758 \ub3d9\uc758\uac00 \uacb0\ucf54 \uc5c6\uc744 \uc218 \uc5c6\ub2e4. \ub611\uac19\uc740 \uc131\ub839\uc758 \ud65c\ub3d9\uc73c\ub85c \uadf8\ub9ac\uc2a4\ub3c4\uc758 \ubaa8\ub4e0 \uc591 \ub5bc\uac00 \uc2e0\uc559\uc758 \uc77c\uce58 \uc548\uc5d0\uc11c \ubcf4\uc804\ub418\uace0 \uc9c4\ubcf4\ud558\uae30 \ub54c\ubb38\uc774\ub2e4.", "answer": "\uad50\ud669", "sentence": "\uc8fc\uad50\ub2e8\uc758 \ub2e8\uc7a5\uc778 \uad50\ud669 \uc740 \ucc38\uc73c\ub85c \uc2e0\uc559 \uc548\uc5d0\uc11c \uc790\uae30 \ud615\uc81c\ub4e4\uc758 \ud798\uc744 \ubd81\ub3cb\uc6cc \uc8fc\ub294 \uc0ac\ub78c\uc774\ubbc0\ub85c(\ub8e8\uce74 22,32 \ucc38\uc870), \ubaa8\ub4e0 \uadf8\ub9ac\uc2a4\ub3c4\uc778\uc758 \ucd5c\uace0 \ubaa9\uc790\uc774\uba70 \uc2a4\uc2b9\uc73c\ub85c\uc11c \uc2e0\uc559\uacfc \ub3c4\ub355\uc5d0 \uad00\ud55c \uad50\ub9ac\ub97c \ud655\uc815\uc801 \ud589\uc704\ub85c \uc120\uc5b8\ud558\ub294 \ub54c\uc5d0, \uad50\ud669\uc740 \uc790\uae30 \uc784\ubb34\uc5d0 \ub530\ub77c \uadf8 \ubb34\ub958\uc131\uc744 \uc9c0\ub2cc\ub2e4.", "paragraph_sentence": ". . . \uadf8\ub9ac\uace0 \ud558\ub290\ub2d8\uc774\uc2e0 \uad6c\uc138\uc8fc\uaed8\uc11c \ub2f9\uc2e0 \uad50\ud68c\uac00 \uc2e0\uc559\uacfc \ub3c4\ub355\uc5d0 \uad00\ud55c \uad50\ub9ac\uc758 \uacb0\uc815\uc5d0\uc11c \uc624\ub958\uac00 \uc5c6\uae30\ub97c \ubc14\ub77c\uc168\ub358 \uc774 \ubb34\ub958\uc131\uc740 \uad50\ud68c\uac00 \uac70\ub8e9\ud558\uac8c \ubcf4\uc804\ud558\uace0 \ucda9\uc2e4\ud788 \uc124\uba85\ud558\uc5ec\uc57c \ud560 \ud558\ub290\ub2d8 \uacc4\uc2dc\uc758 \uc704\ud0c1\uc774 \ud3bc\uccd0\uc9c0\ub294 \uadf8\ub9cc\ud07c \ud3bc\uccd0\uc9c4\ub2e4. \uc8fc\uad50\ub2e8\uc758 \ub2e8\uc7a5\uc778 \uad50\ud669 \uc740 \ucc38\uc73c\ub85c \uc2e0\uc559 \uc548\uc5d0\uc11c \uc790\uae30 \ud615\uc81c\ub4e4\uc758 \ud798\uc744 \ubd81\ub3cb\uc6cc \uc8fc\ub294 \uc0ac\ub78c\uc774\ubbc0\ub85c(\ub8e8\uce74 22,32 \ucc38\uc870), \ubaa8\ub4e0 \uadf8\ub9ac\uc2a4\ub3c4\uc778\uc758 \ucd5c\uace0 \ubaa9\uc790\uc774\uba70 \uc2a4\uc2b9\uc73c\ub85c\uc11c \uc2e0\uc559\uacfc \ub3c4\ub355\uc5d0 \uad00\ud55c \uad50\ub9ac\ub97c \ud655\uc815\uc801 \ud589\uc704\ub85c \uc120\uc5b8\ud558\ub294 \ub54c\uc5d0, \uad50\ud669\uc740 \uc790\uae30 \uc784\ubb34\uc5d0 \ub530\ub77c \uadf8 \ubb34\ub958\uc131\uc744 \uc9c0\ub2cc\ub2e4. \uadf8\ub7ec\ubbc0\ub85c \uad50\ud669\uc758 \uacb0\uc815\uc740 \uad50\ud68c\uc758 \ub3d9\uc758 \ub54c\ubb38\uc774 \uc544\ub2c8\ub77c \uadf8 \uc790\uccb4\ub85c\uc11c \ub9c8\ub545\ud788 \ubc14\ub014 \uc218 \uc5c6\ub294 \uac83\uc73c\ub85c \uc5ec\uaca8\uc9c4\ub2e4. \uadf8\uac83\uc740 \ubcf5\ub41c \ubca0\ub4dc\ub85c \uc548\uc5d0\uc11c \uad50\ud669\uc5d0\uac8c \uc57d\uc18d\ub41c \uc131\ub839\uc758 \ub3c4\uc6c0\uc744 \ubc1b\uc544 \uc120\ud3ec\ub41c \uac83\uc774\uae30 \ub54c\ubb38\uc774\ub2e4. \ub610\ud55c \uadf8 \uacb0\uc815\uc740 \uacb0\ucf54 \ub2e4\ub978 \ub204\uad6c\uc758 \uc2b9\uc778\ub3c4 \ud544\uc694\ud558\uc9c0 \uc54a\uace0 \ub2e4\ub978 \ud310\ub2e8\uc744 \uc694\uad6c\ud558\ub294 \uc5b4\ub5a0\ud55c \uc0c1\uc18c\ub3c4 \ud5c8\uc6a9\ub418\uc9c0 \uc54a\ub294\ub2e4. \uadf8\ub7ec\ud560 \ub54c\uc5d0 \uad50\ud669\uc740 \ud55c \uac1c\uc778\uc73c\ub85c\uc11c \ud310\ub2e8\uc744 \ub0b4\ub9ac\ub294 \uac83\uc774 \uc544\ub2c8\ub77c \ubc14\ub85c \uad50\ud68c \uc790\uccb4\uc758 \ubb34\ub958\uc131\uc758 \uc740\uc0ac\ub97c \ud2b9\ubcc4\ud788 \uc9c0\ub2c8\uace0 \uc788\ub294 \ubcf4\ud3b8 \uad50\ud68c\uc758 \ucd5c\uace0 \uc2a4\uc2b9\uc73c\ub85c\uc11c \uac00\ud1a8\ub9ad \uc2e0\uc559\uc758 \uad50\ub9ac\ub97c \uc124\uba85\ud558\uace0 \uc639\ud638\ud558\ub294 \uac83\uc774\ub2e4. \uad50\ud68c\uc5d0 \uc57d\uc18d\ub41c \ubb34\ub958\uc131\uc740 \uc8fc\uad50\ub2e8\uc774 \ubca0\ub4dc\ub85c\uc758 \ud6c4\uacc4\uc790\uc640 \ub354\ubd88\uc5b4 \ucd5c\uace0 \uad50\ub3c4\uad8c\uc744 \ud589\uc0ac\ud560 \ub54c\uc5d0 \uc8fc\uad50\ub2e8 \uc548\uc5d0\ub3c4 \ub0b4\uc7ac\ud55c\ub2e4. \uc774\ub7ec\ud55c \uacb0\uc815\uc5d0 \ub300\ud558\uc5ec \uad50\ud68c\uc758 \ub3d9\uc758\uac00 \uacb0\ucf54 \uc5c6\uc744 \uc218 \uc5c6\ub2e4. \ub611\uac19\uc740 \uc131\ub839\uc758 \ud65c\ub3d9\uc73c\ub85c \uadf8\ub9ac\uc2a4\ub3c4\uc758 \ubaa8\ub4e0 \uc591 \ub5bc\uac00 \uc2e0\uc559\uc758 \uc77c\uce58 \uc548\uc5d0\uc11c \ubcf4\uc804\ub418\uace0 \uc9c4\ubcf4\ud558\uae30 \ub54c\ubb38\uc774\ub2e4.", "paragraph_answer": ". . . \uadf8\ub9ac\uace0 \ud558\ub290\ub2d8\uc774\uc2e0 \uad6c\uc138\uc8fc\uaed8\uc11c \ub2f9\uc2e0 \uad50\ud68c\uac00 \uc2e0\uc559\uacfc \ub3c4\ub355\uc5d0 \uad00\ud55c \uad50\ub9ac\uc758 \uacb0\uc815\uc5d0\uc11c \uc624\ub958\uac00 \uc5c6\uae30\ub97c \ubc14\ub77c\uc168\ub358 \uc774 \ubb34\ub958\uc131\uc740 \uad50\ud68c\uac00 \uac70\ub8e9\ud558\uac8c \ubcf4\uc804\ud558\uace0 \ucda9\uc2e4\ud788 \uc124\uba85\ud558\uc5ec\uc57c \ud560 \ud558\ub290\ub2d8 \uacc4\uc2dc\uc758 \uc704\ud0c1\uc774 \ud3bc\uccd0\uc9c0\ub294 \uadf8\ub9cc\ud07c \ud3bc\uccd0\uc9c4\ub2e4. \uc8fc\uad50\ub2e8\uc758 \ub2e8\uc7a5\uc778 \uad50\ud669 \uc740 \ucc38\uc73c\ub85c \uc2e0\uc559 \uc548\uc5d0\uc11c \uc790\uae30 \ud615\uc81c\ub4e4\uc758 \ud798\uc744 \ubd81\ub3cb\uc6cc \uc8fc\ub294 \uc0ac\ub78c\uc774\ubbc0\ub85c(\ub8e8\uce74 22,32 \ucc38\uc870), \ubaa8\ub4e0 \uadf8\ub9ac\uc2a4\ub3c4\uc778\uc758 \ucd5c\uace0 \ubaa9\uc790\uc774\uba70 \uc2a4\uc2b9\uc73c\ub85c\uc11c \uc2e0\uc559\uacfc \ub3c4\ub355\uc5d0 \uad00\ud55c \uad50\ub9ac\ub97c \ud655\uc815\uc801 \ud589\uc704\ub85c \uc120\uc5b8\ud558\ub294 \ub54c\uc5d0, \uad50\ud669\uc740 \uc790\uae30 \uc784\ubb34\uc5d0 \ub530\ub77c \uadf8 \ubb34\ub958\uc131\uc744 \uc9c0\ub2cc\ub2e4. \uadf8\ub7ec\ubbc0\ub85c \uad50\ud669\uc758 \uacb0\uc815\uc740 \uad50\ud68c\uc758 \ub3d9\uc758 \ub54c\ubb38\uc774 \uc544\ub2c8\ub77c \uadf8 \uc790\uccb4\ub85c\uc11c \ub9c8\ub545\ud788 \ubc14\ub014 \uc218 \uc5c6\ub294 \uac83\uc73c\ub85c \uc5ec\uaca8\uc9c4\ub2e4. \uadf8\uac83\uc740 \ubcf5\ub41c \ubca0\ub4dc\ub85c \uc548\uc5d0\uc11c \uad50\ud669\uc5d0\uac8c \uc57d\uc18d\ub41c \uc131\ub839\uc758 \ub3c4\uc6c0\uc744 \ubc1b\uc544 \uc120\ud3ec\ub41c \uac83\uc774\uae30 \ub54c\ubb38\uc774\ub2e4. \ub610\ud55c \uadf8 \uacb0\uc815\uc740 \uacb0\ucf54 \ub2e4\ub978 \ub204\uad6c\uc758 \uc2b9\uc778\ub3c4 \ud544\uc694\ud558\uc9c0 \uc54a\uace0 \ub2e4\ub978 \ud310\ub2e8\uc744 \uc694\uad6c\ud558\ub294 \uc5b4\ub5a0\ud55c \uc0c1\uc18c\ub3c4 \ud5c8\uc6a9\ub418\uc9c0 \uc54a\ub294\ub2e4. \uadf8\ub7ec\ud560 \ub54c\uc5d0 \uad50\ud669\uc740 \ud55c \uac1c\uc778\uc73c\ub85c\uc11c \ud310\ub2e8\uc744 \ub0b4\ub9ac\ub294 \uac83\uc774 \uc544\ub2c8\ub77c \ubc14\ub85c \uad50\ud68c \uc790\uccb4\uc758 \ubb34\ub958\uc131\uc758 \uc740\uc0ac\ub97c \ud2b9\ubcc4\ud788 \uc9c0\ub2c8\uace0 \uc788\ub294 \ubcf4\ud3b8 \uad50\ud68c\uc758 \ucd5c\uace0 \uc2a4\uc2b9\uc73c\ub85c\uc11c \uac00\ud1a8\ub9ad \uc2e0\uc559\uc758 \uad50\ub9ac\ub97c \uc124\uba85\ud558\uace0 \uc639\ud638\ud558\ub294 \uac83\uc774\ub2e4. \uad50\ud68c\uc5d0 \uc57d\uc18d\ub41c \ubb34\ub958\uc131\uc740 \uc8fc\uad50\ub2e8\uc774 \ubca0\ub4dc\ub85c\uc758 \ud6c4\uacc4\uc790\uc640 \ub354\ubd88\uc5b4 \ucd5c\uace0 \uad50\ub3c4\uad8c\uc744 \ud589\uc0ac\ud560 \ub54c\uc5d0 \uc8fc\uad50\ub2e8 \uc548\uc5d0\ub3c4 \ub0b4\uc7ac\ud55c\ub2e4. \uc774\ub7ec\ud55c \uacb0\uc815\uc5d0 \ub300\ud558\uc5ec \uad50\ud68c\uc758 \ub3d9\uc758\uac00 \uacb0\ucf54 \uc5c6\uc744 \uc218 \uc5c6\ub2e4. \ub611\uac19\uc740 \uc131\ub839\uc758 \ud65c\ub3d9\uc73c\ub85c \uadf8\ub9ac\uc2a4\ub3c4\uc758 \ubaa8\ub4e0 \uc591 \ub5bc\uac00 \uc2e0\uc559\uc758 \uc77c\uce58 \uc548\uc5d0\uc11c \ubcf4\uc804\ub418\uace0 \uc9c4\ubcf4\ud558\uae30 \ub54c\ubb38\uc774\ub2e4.", "sentence_answer": "\uc8fc\uad50\ub2e8\uc758 \ub2e8\uc7a5\uc778 \uad50\ud669 \uc740 \ucc38\uc73c\ub85c \uc2e0\uc559 \uc548\uc5d0\uc11c \uc790\uae30 \ud615\uc81c\ub4e4\uc758 \ud798\uc744 \ubd81\ub3cb\uc6cc \uc8fc\ub294 \uc0ac\ub78c\uc774\ubbc0\ub85c(\ub8e8\uce74 22,32 \ucc38\uc870), \ubaa8\ub4e0 \uadf8\ub9ac\uc2a4\ub3c4\uc778\uc758 \ucd5c\uace0 \ubaa9\uc790\uc774\uba70 \uc2a4\uc2b9\uc73c\ub85c\uc11c \uc2e0\uc559\uacfc \ub3c4\ub355\uc5d0 \uad00\ud55c \uad50\ub9ac\ub97c \ud655\uc815\uc801 \ud589\uc704\ub85c \uc120\uc5b8\ud558\ub294 \ub54c\uc5d0, \uad50\ud669\uc740 \uc790\uae30 \uc784\ubb34\uc5d0 \ub530\ub77c \uadf8 \ubb34\ub958\uc131\uc744 \uc9c0\ub2cc\ub2e4.", "paragraph_id": "6475503-15-0", "paragraph_question": "question: \uc8fc\uad50\uc7a5\uc758 \ub2e8\uc7a5\uc740?, context: . . . \uadf8\ub9ac\uace0 \ud558\ub290\ub2d8\uc774\uc2e0 \uad6c\uc138\uc8fc\uaed8\uc11c \ub2f9\uc2e0 \uad50\ud68c\uac00 \uc2e0\uc559\uacfc \ub3c4\ub355\uc5d0 \uad00\ud55c \uad50\ub9ac\uc758 \uacb0\uc815\uc5d0\uc11c \uc624\ub958\uac00 \uc5c6\uae30\ub97c \ubc14\ub77c\uc168\ub358 \uc774 \ubb34\ub958\uc131\uc740 \uad50\ud68c\uac00 \uac70\ub8e9\ud558\uac8c \ubcf4\uc804\ud558\uace0 \ucda9\uc2e4\ud788 \uc124\uba85\ud558\uc5ec\uc57c \ud560 \ud558\ub290\ub2d8 \uacc4\uc2dc\uc758 \uc704\ud0c1\uc774 \ud3bc\uccd0\uc9c0\ub294 \uadf8\ub9cc\ud07c \ud3bc\uccd0\uc9c4\ub2e4. \uc8fc\uad50\ub2e8\uc758 \ub2e8\uc7a5\uc778 \uad50\ud669\uc740 \ucc38\uc73c\ub85c \uc2e0\uc559 \uc548\uc5d0\uc11c \uc790\uae30 \ud615\uc81c\ub4e4\uc758 \ud798\uc744 \ubd81\ub3cb\uc6cc \uc8fc\ub294 \uc0ac\ub78c\uc774\ubbc0\ub85c(\ub8e8\uce74 22,32 \ucc38\uc870), \ubaa8\ub4e0 \uadf8\ub9ac\uc2a4\ub3c4\uc778\uc758 \ucd5c\uace0 \ubaa9\uc790\uc774\uba70 \uc2a4\uc2b9\uc73c\ub85c\uc11c \uc2e0\uc559\uacfc \ub3c4\ub355\uc5d0 \uad00\ud55c \uad50\ub9ac\ub97c \ud655\uc815\uc801 \ud589\uc704\ub85c \uc120\uc5b8\ud558\ub294 \ub54c\uc5d0, \uad50\ud669\uc740 \uc790\uae30 \uc784\ubb34\uc5d0 \ub530\ub77c \uadf8 \ubb34\ub958\uc131\uc744 \uc9c0\ub2cc\ub2e4. \uadf8\ub7ec\ubbc0\ub85c \uad50\ud669\uc758 \uacb0\uc815\uc740 \uad50\ud68c\uc758 \ub3d9\uc758 \ub54c\ubb38\uc774 \uc544\ub2c8\ub77c \uadf8 \uc790\uccb4\ub85c\uc11c \ub9c8\ub545\ud788 \ubc14\ub014 \uc218 \uc5c6\ub294 \uac83\uc73c\ub85c \uc5ec\uaca8\uc9c4\ub2e4. \uadf8\uac83\uc740 \ubcf5\ub41c \ubca0\ub4dc\ub85c \uc548\uc5d0\uc11c \uad50\ud669\uc5d0\uac8c \uc57d\uc18d\ub41c \uc131\ub839\uc758 \ub3c4\uc6c0\uc744 \ubc1b\uc544 \uc120\ud3ec\ub41c \uac83\uc774\uae30 \ub54c\ubb38\uc774\ub2e4. \ub610\ud55c \uadf8 \uacb0\uc815\uc740 \uacb0\ucf54 \ub2e4\ub978 \ub204\uad6c\uc758 \uc2b9\uc778\ub3c4 \ud544\uc694\ud558\uc9c0 \uc54a\uace0 \ub2e4\ub978 \ud310\ub2e8\uc744 \uc694\uad6c\ud558\ub294 \uc5b4\ub5a0\ud55c \uc0c1\uc18c\ub3c4 \ud5c8\uc6a9\ub418\uc9c0 \uc54a\ub294\ub2e4. \uadf8\ub7ec\ud560 \ub54c\uc5d0 \uad50\ud669\uc740 \ud55c \uac1c\uc778\uc73c\ub85c\uc11c \ud310\ub2e8\uc744 \ub0b4\ub9ac\ub294 \uac83\uc774 \uc544\ub2c8\ub77c \ubc14\ub85c \uad50\ud68c \uc790\uccb4\uc758 \ubb34\ub958\uc131\uc758 \uc740\uc0ac\ub97c \ud2b9\ubcc4\ud788 \uc9c0\ub2c8\uace0 \uc788\ub294 \ubcf4\ud3b8 \uad50\ud68c\uc758 \ucd5c\uace0 \uc2a4\uc2b9\uc73c\ub85c\uc11c \uac00\ud1a8\ub9ad \uc2e0\uc559\uc758 \uad50\ub9ac\ub97c \uc124\uba85\ud558\uace0 \uc639\ud638\ud558\ub294 \uac83\uc774\ub2e4. \uad50\ud68c\uc5d0 \uc57d\uc18d\ub41c \ubb34\ub958\uc131\uc740 \uc8fc\uad50\ub2e8\uc774 \ubca0\ub4dc\ub85c\uc758 \ud6c4\uacc4\uc790\uc640 \ub354\ubd88\uc5b4 \ucd5c\uace0 \uad50\ub3c4\uad8c\uc744 \ud589\uc0ac\ud560 \ub54c\uc5d0 \uc8fc\uad50\ub2e8 \uc548\uc5d0\ub3c4 \ub0b4\uc7ac\ud55c\ub2e4. \uc774\ub7ec\ud55c \uacb0\uc815\uc5d0 \ub300\ud558\uc5ec \uad50\ud68c\uc758 \ub3d9\uc758\uac00 \uacb0\ucf54 \uc5c6\uc744 \uc218 \uc5c6\ub2e4. \ub611\uac19\uc740 \uc131\ub839\uc758 \ud65c\ub3d9\uc73c\ub85c \uadf8\ub9ac\uc2a4\ub3c4\uc758 \ubaa8\ub4e0 \uc591 \ub5bc\uac00 \uc2e0\uc559\uc758 \uc77c\uce58 \uc548\uc5d0\uc11c \ubcf4\uc804\ub418\uace0 \uc9c4\ubcf4\ud558\uae30 \ub54c\ubb38\uc774\ub2e4."} {"question": "\ub808\uc624\ub12c\ub85c\ub294 1444\ub144 5\uc6d4 \ub204\uad6c\uc640 \uacb0\ud63c\ud588\ub294\uac00?", "paragraph": "\ud6cc\ub96d\ud55c \ud559\ud48d\uc801 \ubc30\uacbd\uc73c\ub85c, \ub808\uc624\ub12c\ub85c\ub294 \ub2c8\ucf5c\ub85c 3\uc138 \uc774\ud6c4\uc5d0 \uadf8\uac00 \uc790\ub9ac\ub97c \uc7a1\uc740 \ud6c4 \ud398\ub77c\ub77c\uc5d0 \uc804\ub840\uc5c6\ub294 \uacbd\uc81c\uc801, \uc815\uce58\uc801, \ubb38\ud654\uc801 \ubcc0\ud654\ub97c \uac00\uc838\uc654\ub2e4. 1444\ub144 5\uc6d4, \ub808\uc624\ub12c\ub85c\ub294 \uc54c\ud3f0\uc18c 5\uc138 \ub370 \uc544\ub77c\uace4\uc758 \uc0ac\uc0dd\uc544 \ub538\uc778 19\uc0b4\uc758 \ub9c8\ub9ac\uc544 \ub2e4\ub77c\uace0\ub098\uc640 \ud63c\uc778\ud558\uc600\ub2e4. \uc774 \ud63c\uc778\uc740 \uc815\uce58\uc801\uc778 \ud63c\uc778\uc774\uc600\ub2e4. \ud398\ub77c\ub77c \ub0b4\ubd80\uc758 \uc815\uce58\uc801 \uc548\uc815\uc131\uc744 \ubcf4\uc7a5\ud558\uae30 \uc704\ud574, \ub808\uc624\ub12c\ub85c\ub294 \ubc00\ub77c\ub178\uc640 \ubca0\ub124\uce58\uc544 \uc0ac\uc774\uc758 \uc815\uce58\uc801 \uade0\ud615\uc5d0\uc11c \uc911\ub9bd\uc744 \uc720\uc9c0\ud558\uc600\ub2e4. \uadf8\ub7ec\ub098 \ub808\uc624\ub12c\ub85c\ub294 \uad8c\uc704\uc640 \ud798\uc744 \ud0a4\uc6b0\ub294\ub4f1 \ud398\ub77c\ub77c\uc758 \uccad\uc0ac\uc9c4 \ub610\ud55c \uc9c0\ub2c8\uace0 \uc788\uc5c8\ub2e4. \ub9c8\ub974\uac8c\ub9ac\ud0c0 \uace4\ucc28\uac00 \uc0ac\ub9dd \ud6c4, \ub808\uc624\ub12c\ub85c\ub294 \uce5c\uc871\uc73c\ub85c \uac15\ud654\ub41c \ud398\ub77c\ub77c\uc758 \ud798\uc744 \ud1b5\ud574 \uc774\uc6c3 \uc9c0\uc5ed\ub4e4\uacfc\uc758 \ub3d9\ub9f9\uc744 \uc774\ub8f0 \uae30\ud68c\ub97c \ud3ec\ucc29\ud588\ub2e4. \ub098\ud3f4\ub9ac\uc5d0\uc11c \ub9c8\ub9ac\uc544 \ub2e4\ub77c\uace0\ub098\uc758 \uc544\ubc84\uc9c0\uc778 \uc54c\ud3f0\uc18c 5\uc138\uc758 \uc2b9\ub9ac\ub294 \ub808\uc624\ub12c\ub85c\uc5d0\uac8c \uc678\uad50\uc801 \ud63c\uc778\uacfc \ud398\ub77c\ub77c\uc5d0\uac8c\ub294 \uae30\ud68c\ub85c\uc11c \ucd09\ub9e4\uc81c\ub85c \uc791\uc6a9\ud588\ub2e4.", "answer": "\ub9c8\ub9ac\uc544 \ub2e4\ub77c\uace0\ub098", "sentence": "1444\ub144 5\uc6d4, \ub808\uc624\ub12c\ub85c\ub294 \uc54c\ud3f0\uc18c 5\uc138 \ub370 \uc544\ub77c\uace4\uc758 \uc0ac\uc0dd\uc544 \ub538\uc778 19\uc0b4\uc758 \ub9c8\ub9ac\uc544 \ub2e4\ub77c\uace0\ub098 \uc640 \ud63c\uc778\ud558\uc600\ub2e4.", "paragraph_sentence": "\ud6cc\ub96d\ud55c \ud559\ud48d\uc801 \ubc30\uacbd\uc73c\ub85c, \ub808\uc624\ub12c\ub85c\ub294 \ub2c8\ucf5c\ub85c 3\uc138 \uc774\ud6c4\uc5d0 \uadf8\uac00 \uc790\ub9ac\ub97c \uc7a1\uc740 \ud6c4 \ud398\ub77c\ub77c\uc5d0 \uc804\ub840\uc5c6\ub294 \uacbd\uc81c\uc801, \uc815\uce58\uc801, \ubb38\ud654\uc801 \ubcc0\ud654\ub97c \uac00\uc838\uc654\ub2e4. 1444\ub144 5\uc6d4, \ub808\uc624\ub12c\ub85c\ub294 \uc54c\ud3f0\uc18c 5\uc138 \ub370 \uc544\ub77c\uace4\uc758 \uc0ac\uc0dd\uc544 \ub538\uc778 19\uc0b4\uc758 \ub9c8\ub9ac\uc544 \ub2e4\ub77c\uace0\ub098 \uc640 \ud63c\uc778\ud558\uc600\ub2e4. \uc774 \ud63c\uc778\uc740 \uc815\uce58\uc801\uc778 \ud63c\uc778\uc774\uc600\ub2e4. \ud398\ub77c\ub77c \ub0b4\ubd80\uc758 \uc815\uce58\uc801 \uc548\uc815\uc131\uc744 \ubcf4\uc7a5\ud558\uae30 \uc704\ud574, \ub808\uc624\ub12c\ub85c\ub294 \ubc00\ub77c\ub178\uc640 \ubca0\ub124\uce58\uc544 \uc0ac\uc774\uc758 \uc815\uce58\uc801 \uade0\ud615\uc5d0\uc11c \uc911\ub9bd\uc744 \uc720\uc9c0\ud558\uc600\ub2e4. \uadf8\ub7ec\ub098 \ub808\uc624\ub12c\ub85c\ub294 \uad8c\uc704\uc640 \ud798\uc744 \ud0a4\uc6b0\ub294\ub4f1 \ud398\ub77c\ub77c\uc758 \uccad\uc0ac\uc9c4 \ub610\ud55c \uc9c0\ub2c8\uace0 \uc788\uc5c8\ub2e4. \ub9c8\ub974\uac8c\ub9ac\ud0c0 \uace4\ucc28\uac00 \uc0ac\ub9dd \ud6c4, \ub808\uc624\ub12c\ub85c\ub294 \uce5c\uc871\uc73c\ub85c \uac15\ud654\ub41c \ud398\ub77c\ub77c\uc758 \ud798\uc744 \ud1b5\ud574 \uc774\uc6c3 \uc9c0\uc5ed\ub4e4\uacfc\uc758 \ub3d9\ub9f9\uc744 \uc774\ub8f0 \uae30\ud68c\ub97c \ud3ec\ucc29\ud588\ub2e4. \ub098\ud3f4\ub9ac\uc5d0\uc11c \ub9c8\ub9ac\uc544 \ub2e4\ub77c\uace0\ub098\uc758 \uc544\ubc84\uc9c0\uc778 \uc54c\ud3f0\uc18c 5\uc138\uc758 \uc2b9\ub9ac\ub294 \ub808\uc624\ub12c\ub85c\uc5d0\uac8c \uc678\uad50\uc801 \ud63c\uc778\uacfc \ud398\ub77c\ub77c\uc5d0\uac8c\ub294 \uae30\ud68c\ub85c\uc11c \ucd09\ub9e4\uc81c\ub85c \uc791\uc6a9\ud588\ub2e4.", "paragraph_answer": "\ud6cc\ub96d\ud55c \ud559\ud48d\uc801 \ubc30\uacbd\uc73c\ub85c, \ub808\uc624\ub12c\ub85c\ub294 \ub2c8\ucf5c\ub85c 3\uc138 \uc774\ud6c4\uc5d0 \uadf8\uac00 \uc790\ub9ac\ub97c \uc7a1\uc740 \ud6c4 \ud398\ub77c\ub77c\uc5d0 \uc804\ub840\uc5c6\ub294 \uacbd\uc81c\uc801, \uc815\uce58\uc801, \ubb38\ud654\uc801 \ubcc0\ud654\ub97c \uac00\uc838\uc654\ub2e4. 1444\ub144 5\uc6d4, \ub808\uc624\ub12c\ub85c\ub294 \uc54c\ud3f0\uc18c 5\uc138 \ub370 \uc544\ub77c\uace4\uc758 \uc0ac\uc0dd\uc544 \ub538\uc778 19\uc0b4\uc758 \ub9c8\ub9ac\uc544 \ub2e4\ub77c\uace0\ub098 \uc640 \ud63c\uc778\ud558\uc600\ub2e4. \uc774 \ud63c\uc778\uc740 \uc815\uce58\uc801\uc778 \ud63c\uc778\uc774\uc600\ub2e4. \ud398\ub77c\ub77c \ub0b4\ubd80\uc758 \uc815\uce58\uc801 \uc548\uc815\uc131\uc744 \ubcf4\uc7a5\ud558\uae30 \uc704\ud574, \ub808\uc624\ub12c\ub85c\ub294 \ubc00\ub77c\ub178\uc640 \ubca0\ub124\uce58\uc544 \uc0ac\uc774\uc758 \uc815\uce58\uc801 \uade0\ud615\uc5d0\uc11c \uc911\ub9bd\uc744 \uc720\uc9c0\ud558\uc600\ub2e4. \uadf8\ub7ec\ub098 \ub808\uc624\ub12c\ub85c\ub294 \uad8c\uc704\uc640 \ud798\uc744 \ud0a4\uc6b0\ub294\ub4f1 \ud398\ub77c\ub77c\uc758 \uccad\uc0ac\uc9c4 \ub610\ud55c \uc9c0\ub2c8\uace0 \uc788\uc5c8\ub2e4. \ub9c8\ub974\uac8c\ub9ac\ud0c0 \uace4\ucc28\uac00 \uc0ac\ub9dd \ud6c4, \ub808\uc624\ub12c\ub85c\ub294 \uce5c\uc871\uc73c\ub85c \uac15\ud654\ub41c \ud398\ub77c\ub77c\uc758 \ud798\uc744 \ud1b5\ud574 \uc774\uc6c3 \uc9c0\uc5ed\ub4e4\uacfc\uc758 \ub3d9\ub9f9\uc744 \uc774\ub8f0 \uae30\ud68c\ub97c \ud3ec\ucc29\ud588\ub2e4. \ub098\ud3f4\ub9ac\uc5d0\uc11c \ub9c8\ub9ac\uc544 \ub2e4\ub77c\uace0\ub098\uc758 \uc544\ubc84\uc9c0\uc778 \uc54c\ud3f0\uc18c 5\uc138\uc758 \uc2b9\ub9ac\ub294 \ub808\uc624\ub12c\ub85c\uc5d0\uac8c \uc678\uad50\uc801 \ud63c\uc778\uacfc \ud398\ub77c\ub77c\uc5d0\uac8c\ub294 \uae30\ud68c\ub85c\uc11c \ucd09\ub9e4\uc81c\ub85c \uc791\uc6a9\ud588\ub2e4.", "sentence_answer": "1444\ub144 5\uc6d4, \ub808\uc624\ub12c\ub85c\ub294 \uc54c\ud3f0\uc18c 5\uc138 \ub370 \uc544\ub77c\uace4\uc758 \uc0ac\uc0dd\uc544 \ub538\uc778 19\uc0b4\uc758 \ub9c8\ub9ac\uc544 \ub2e4\ub77c\uace0\ub098 \uc640 \ud63c\uc778\ud558\uc600\ub2e4.", "paragraph_id": "6306261-1-0", "paragraph_question": "question: \ub808\uc624\ub12c\ub85c\ub294 1444\ub144 5\uc6d4 \ub204\uad6c\uc640 \uacb0\ud63c\ud588\ub294\uac00?, context: \ud6cc\ub96d\ud55c \ud559\ud48d\uc801 \ubc30\uacbd\uc73c\ub85c, \ub808\uc624\ub12c\ub85c\ub294 \ub2c8\ucf5c\ub85c 3\uc138 \uc774\ud6c4\uc5d0 \uadf8\uac00 \uc790\ub9ac\ub97c \uc7a1\uc740 \ud6c4 \ud398\ub77c\ub77c\uc5d0 \uc804\ub840\uc5c6\ub294 \uacbd\uc81c\uc801, \uc815\uce58\uc801, \ubb38\ud654\uc801 \ubcc0\ud654\ub97c \uac00\uc838\uc654\ub2e4. 1444\ub144 5\uc6d4, \ub808\uc624\ub12c\ub85c\ub294 \uc54c\ud3f0\uc18c 5\uc138 \ub370 \uc544\ub77c\uace4\uc758 \uc0ac\uc0dd\uc544 \ub538\uc778 19\uc0b4\uc758 \ub9c8\ub9ac\uc544 \ub2e4\ub77c\uace0\ub098\uc640 \ud63c\uc778\ud558\uc600\ub2e4. \uc774 \ud63c\uc778\uc740 \uc815\uce58\uc801\uc778 \ud63c\uc778\uc774\uc600\ub2e4. \ud398\ub77c\ub77c \ub0b4\ubd80\uc758 \uc815\uce58\uc801 \uc548\uc815\uc131\uc744 \ubcf4\uc7a5\ud558\uae30 \uc704\ud574, \ub808\uc624\ub12c\ub85c\ub294 \ubc00\ub77c\ub178\uc640 \ubca0\ub124\uce58\uc544 \uc0ac\uc774\uc758 \uc815\uce58\uc801 \uade0\ud615\uc5d0\uc11c \uc911\ub9bd\uc744 \uc720\uc9c0\ud558\uc600\ub2e4. \uadf8\ub7ec\ub098 \ub808\uc624\ub12c\ub85c\ub294 \uad8c\uc704\uc640 \ud798\uc744 \ud0a4\uc6b0\ub294\ub4f1 \ud398\ub77c\ub77c\uc758 \uccad\uc0ac\uc9c4 \ub610\ud55c \uc9c0\ub2c8\uace0 \uc788\uc5c8\ub2e4. \ub9c8\ub974\uac8c\ub9ac\ud0c0 \uace4\ucc28\uac00 \uc0ac\ub9dd \ud6c4, \ub808\uc624\ub12c\ub85c\ub294 \uce5c\uc871\uc73c\ub85c \uac15\ud654\ub41c \ud398\ub77c\ub77c\uc758 \ud798\uc744 \ud1b5\ud574 \uc774\uc6c3 \uc9c0\uc5ed\ub4e4\uacfc\uc758 \ub3d9\ub9f9\uc744 \uc774\ub8f0 \uae30\ud68c\ub97c \ud3ec\ucc29\ud588\ub2e4. \ub098\ud3f4\ub9ac\uc5d0\uc11c \ub9c8\ub9ac\uc544 \ub2e4\ub77c\uace0\ub098\uc758 \uc544\ubc84\uc9c0\uc778 \uc54c\ud3f0\uc18c 5\uc138\uc758 \uc2b9\ub9ac\ub294 \ub808\uc624\ub12c\ub85c\uc5d0\uac8c \uc678\uad50\uc801 \ud63c\uc778\uacfc \ud398\ub77c\ub77c\uc5d0\uac8c\ub294 \uae30\ud68c\ub85c\uc11c \ucd09\ub9e4\uc81c\ub85c \uc791\uc6a9\ud588\ub2e4."} {"question": "\uc77c\ubcf8\uc758 \uac74\uac15\ubcf4\ud5d8\uc81c\ub3c4\uc758 \ud2b9\uc9d5 \uc911 \ud558\ub098\ub294 \ubb34\uc5c7\uc778\uac00?", "paragraph": "\uc77c\ubcf8\uc5d0\uc11c \uac74\uac15\ubcf4\ud5d8\uc81c\ub3c4\uc758 \ud61c\ud0dd\uc744 \ubc1b\uc73c\ub824\uba74 \uc77c\ubcf8\uc5d0 \uac70\uc8fc\ud558\uc5ec\uc57c \ud55c\ub2e4. \ub610\ud55c \uac74\uac15\ubcf4\ud5d8\uc81c\ub3c4\ub294 \uad6d\uac00 \ubc0f \uc9c0\ubc29 \uc790\uce58 \ub2e8\uccb4\uc5d0 \uc758\ud574 \uc81c\uacf5\ub41c\ub2e4. 1961\ub144\uc5d0 \uac1c\uc778 \uc758\ub8cc \uc11c\ube44\uc2a4\uac00 \uc2dc\uc791\ub418\uc5c8\ub2e4. \uc0ac\ub78c\ub4e4\uc774 \uac74\uac15\ubcf4\ud5d8\uc758 \ud61c\ud0dd\uc744 \ubc1b\uae30 \uc704\ud574\uc120 \ubc18\ub4dc\uc2dc \uc5b4\ub5a4 \uc885\ub958\uc758 \uac74\uac15\ubcf4\ud5d8\uacc4\ud68d\uc5d0 \uac00\uc785\ud558\uc5ec\uc57c \ud55c\ub2e4. \uacf5\ubb34\uc6d0\uc774\ub098 \uac1c\uc778 \ud68c\uc0ac\uc758 \ub178\ub3d9\uc790\ub4e4\uc740 \uadfc\ub85c\uc790 \ubcf4\ud5d8 \ub610\ub294 \ud68c\uc0ac\uac74\uac15 \ubcf4\ud5d8\uacc4\ud68d \uc911 \ud558\ub098\uc5d0 \uac00\uc785\ud574\uc57c \ud558\uba70 \uc790\uc601\uc5c5\uc790\uc640 \ud1f4\uc9c1\uc790\ub4e4\uc740 \uc9c0\ubc29\ub2e8\uccb4\uac00 \uc6b4\uc601\ud558\ub294 \uad6d\uac00\uac74\uac15\ubcf4\ud5d8\uacc4\ud68d\uc5d0 \uac00\uc785\ud574\uc57c \ud55c\ub2e4. \uc77c\ubcf8\uc758 \uac74\uac15 \ubcf4\ud5d8\uc81c\ub3c4\uc758 \ud2b9\uc9d5\uc73c\ub85c\ub294 \u2018\uc131\uacfc\uae09 \uc81c\ub3c4\u2019\uac00 \uc788\ub2e4. \uc774 \uc81c\ub3c4\ub294 \uc758\ub8cc\uae30\uad00\ub4e4\uc774 \ud658\uc790\ub4e4\uc5d0\uac8c \uc758\ub8cc \uc11c\ube44\uc2a4\ub97c \uc81c\uacf5\ud558\uc600\uc744 \ub54c \uac74\uac15\ubcf4\ud5d8\uacc4\ud68d\uc5d0 \uc758\ud558\uc5ec \ubc30\uc0c1\uae08\uc744 \uc9c0\uae09\ubc1b\ub294 \uc81c\ub3c4\uc774\ub2e4. \uac74\uac15\ubcf4\ud5d8\uacc4\ud68d\uc740 \uc758\uc0ac\ub4e4\uc5d0\uac8c \uac1c\uc778 \ubcd1\uc6d0\uc744 \uc544\ubb34 \uacf3\uc774\ub4e0 \uac1c\uc5c5\ud560 \uc218 \uc788\ub294 \uc790\uc720\ub97c \ubd80\uc5ec\ud558\uc600\ub2e4. \uac74\uac15\uc99d\uc9c4\ubc95\uc774 \uc788\uc5b4 \ucca0\ub3c4\uc5ed \ub4f1\uc5d0\uc11c \ud761\uc5f0\uc774 \uae08\uc9c0\ub418\uc5b4 \uc788\ub2e4.", "answer": "\uc131\uacfc\uae09 \uc81c\ub3c4", "sentence": "\uc77c\ubcf8\uc758 \uac74\uac15 \ubcf4\ud5d8\uc81c\ub3c4\uc758 \ud2b9\uc9d5\uc73c\ub85c\ub294 \u2018 \uc131\uacfc\uae09 \uc81c\ub3c4 \u2019\uac00 \uc788\ub2e4.", "paragraph_sentence": "\uc77c\ubcf8\uc5d0\uc11c \uac74\uac15\ubcf4\ud5d8\uc81c\ub3c4\uc758 \ud61c\ud0dd\uc744 \ubc1b\uc73c\ub824\uba74 \uc77c\ubcf8\uc5d0 \uac70\uc8fc\ud558\uc5ec\uc57c \ud55c\ub2e4. \ub610\ud55c \uac74\uac15\ubcf4\ud5d8\uc81c\ub3c4\ub294 \uad6d\uac00 \ubc0f \uc9c0\ubc29 \uc790\uce58 \ub2e8\uccb4\uc5d0 \uc758\ud574 \uc81c\uacf5\ub41c\ub2e4. 1961\ub144\uc5d0 \uac1c\uc778 \uc758\ub8cc \uc11c\ube44\uc2a4\uac00 \uc2dc\uc791\ub418\uc5c8\ub2e4. \uc0ac\ub78c\ub4e4\uc774 \uac74\uac15\ubcf4\ud5d8\uc758 \ud61c\ud0dd\uc744 \ubc1b\uae30 \uc704\ud574\uc120 \ubc18\ub4dc\uc2dc \uc5b4\ub5a4 \uc885\ub958\uc758 \uac74\uac15\ubcf4\ud5d8\uacc4\ud68d\uc5d0 \uac00\uc785\ud558\uc5ec\uc57c \ud55c\ub2e4. \uacf5\ubb34\uc6d0\uc774\ub098 \uac1c\uc778 \ud68c\uc0ac\uc758 \ub178\ub3d9\uc790\ub4e4\uc740 \uadfc\ub85c\uc790 \ubcf4\ud5d8 \ub610\ub294 \ud68c\uc0ac\uac74\uac15 \ubcf4\ud5d8\uacc4\ud68d \uc911 \ud558\ub098\uc5d0 \uac00\uc785\ud574\uc57c \ud558\uba70 \uc790\uc601\uc5c5\uc790\uc640 \ud1f4\uc9c1\uc790\ub4e4\uc740 \uc9c0\ubc29\ub2e8\uccb4\uac00 \uc6b4\uc601\ud558\ub294 \uad6d\uac00\uac74\uac15\ubcf4\ud5d8\uacc4\ud68d\uc5d0 \uac00\uc785\ud574\uc57c \ud55c\ub2e4. \uc77c\ubcf8\uc758 \uac74\uac15 \ubcf4\ud5d8\uc81c\ub3c4\uc758 \ud2b9\uc9d5\uc73c\ub85c\ub294 \u2018 \uc131\uacfc\uae09 \uc81c\ub3c4 \u2019\uac00 \uc788\ub2e4. \uc774 \uc81c\ub3c4\ub294 \uc758\ub8cc\uae30\uad00\ub4e4\uc774 \ud658\uc790\ub4e4\uc5d0\uac8c \uc758\ub8cc \uc11c\ube44\uc2a4\ub97c \uc81c\uacf5\ud558\uc600\uc744 \ub54c \uac74\uac15\ubcf4\ud5d8\uacc4\ud68d\uc5d0 \uc758\ud558\uc5ec \ubc30\uc0c1\uae08\uc744 \uc9c0\uae09\ubc1b\ub294 \uc81c\ub3c4\uc774\ub2e4. \uac74\uac15\ubcf4\ud5d8\uacc4\ud68d\uc740 \uc758\uc0ac\ub4e4\uc5d0\uac8c \uac1c\uc778 \ubcd1\uc6d0\uc744 \uc544\ubb34 \uacf3\uc774\ub4e0 \uac1c\uc5c5\ud560 \uc218 \uc788\ub294 \uc790\uc720\ub97c \ubd80\uc5ec\ud558\uc600\ub2e4. \uac74\uac15\uc99d\uc9c4\ubc95\uc774 \uc788\uc5b4 \ucca0\ub3c4\uc5ed \ub4f1\uc5d0\uc11c \ud761\uc5f0\uc774 \uae08\uc9c0\ub418\uc5b4 \uc788\ub2e4.", "paragraph_answer": "\uc77c\ubcf8\uc5d0\uc11c \uac74\uac15\ubcf4\ud5d8\uc81c\ub3c4\uc758 \ud61c\ud0dd\uc744 \ubc1b\uc73c\ub824\uba74 \uc77c\ubcf8\uc5d0 \uac70\uc8fc\ud558\uc5ec\uc57c \ud55c\ub2e4. \ub610\ud55c \uac74\uac15\ubcf4\ud5d8\uc81c\ub3c4\ub294 \uad6d\uac00 \ubc0f \uc9c0\ubc29 \uc790\uce58 \ub2e8\uccb4\uc5d0 \uc758\ud574 \uc81c\uacf5\ub41c\ub2e4. 1961\ub144\uc5d0 \uac1c\uc778 \uc758\ub8cc \uc11c\ube44\uc2a4\uac00 \uc2dc\uc791\ub418\uc5c8\ub2e4. \uc0ac\ub78c\ub4e4\uc774 \uac74\uac15\ubcf4\ud5d8\uc758 \ud61c\ud0dd\uc744 \ubc1b\uae30 \uc704\ud574\uc120 \ubc18\ub4dc\uc2dc \uc5b4\ub5a4 \uc885\ub958\uc758 \uac74\uac15\ubcf4\ud5d8\uacc4\ud68d\uc5d0 \uac00\uc785\ud558\uc5ec\uc57c \ud55c\ub2e4. \uacf5\ubb34\uc6d0\uc774\ub098 \uac1c\uc778 \ud68c\uc0ac\uc758 \ub178\ub3d9\uc790\ub4e4\uc740 \uadfc\ub85c\uc790 \ubcf4\ud5d8 \ub610\ub294 \ud68c\uc0ac\uac74\uac15 \ubcf4\ud5d8\uacc4\ud68d \uc911 \ud558\ub098\uc5d0 \uac00\uc785\ud574\uc57c \ud558\uba70 \uc790\uc601\uc5c5\uc790\uc640 \ud1f4\uc9c1\uc790\ub4e4\uc740 \uc9c0\ubc29\ub2e8\uccb4\uac00 \uc6b4\uc601\ud558\ub294 \uad6d\uac00\uac74\uac15\ubcf4\ud5d8\uacc4\ud68d\uc5d0 \uac00\uc785\ud574\uc57c \ud55c\ub2e4. \uc77c\ubcf8\uc758 \uac74\uac15 \ubcf4\ud5d8\uc81c\ub3c4\uc758 \ud2b9\uc9d5\uc73c\ub85c\ub294 \u2018 \uc131\uacfc\uae09 \uc81c\ub3c4 \u2019\uac00 \uc788\ub2e4. \uc774 \uc81c\ub3c4\ub294 \uc758\ub8cc\uae30\uad00\ub4e4\uc774 \ud658\uc790\ub4e4\uc5d0\uac8c \uc758\ub8cc \uc11c\ube44\uc2a4\ub97c \uc81c\uacf5\ud558\uc600\uc744 \ub54c \uac74\uac15\ubcf4\ud5d8\uacc4\ud68d\uc5d0 \uc758\ud558\uc5ec \ubc30\uc0c1\uae08\uc744 \uc9c0\uae09\ubc1b\ub294 \uc81c\ub3c4\uc774\ub2e4. \uac74\uac15\ubcf4\ud5d8\uacc4\ud68d\uc740 \uc758\uc0ac\ub4e4\uc5d0\uac8c \uac1c\uc778 \ubcd1\uc6d0\uc744 \uc544\ubb34 \uacf3\uc774\ub4e0 \uac1c\uc5c5\ud560 \uc218 \uc788\ub294 \uc790\uc720\ub97c \ubd80\uc5ec\ud558\uc600\ub2e4. \uac74\uac15\uc99d\uc9c4\ubc95\uc774 \uc788\uc5b4 \ucca0\ub3c4\uc5ed \ub4f1\uc5d0\uc11c \ud761\uc5f0\uc774 \uae08\uc9c0\ub418\uc5b4 \uc788\ub2e4.", "sentence_answer": "\uc77c\ubcf8\uc758 \uac74\uac15 \ubcf4\ud5d8\uc81c\ub3c4\uc758 \ud2b9\uc9d5\uc73c\ub85c\ub294 \u2018 \uc131\uacfc\uae09 \uc81c\ub3c4 \u2019\uac00 \uc788\ub2e4.", "paragraph_id": "6591477-32-2", "paragraph_question": "question: \uc77c\ubcf8\uc758 \uac74\uac15\ubcf4\ud5d8\uc81c\ub3c4\uc758 \ud2b9\uc9d5 \uc911 \ud558\ub098\ub294 \ubb34\uc5c7\uc778\uac00?, context: \uc77c\ubcf8\uc5d0\uc11c \uac74\uac15\ubcf4\ud5d8\uc81c\ub3c4\uc758 \ud61c\ud0dd\uc744 \ubc1b\uc73c\ub824\uba74 \uc77c\ubcf8\uc5d0 \uac70\uc8fc\ud558\uc5ec\uc57c \ud55c\ub2e4. \ub610\ud55c \uac74\uac15\ubcf4\ud5d8\uc81c\ub3c4\ub294 \uad6d\uac00 \ubc0f \uc9c0\ubc29 \uc790\uce58 \ub2e8\uccb4\uc5d0 \uc758\ud574 \uc81c\uacf5\ub41c\ub2e4. 1961\ub144\uc5d0 \uac1c\uc778 \uc758\ub8cc \uc11c\ube44\uc2a4\uac00 \uc2dc\uc791\ub418\uc5c8\ub2e4. \uc0ac\ub78c\ub4e4\uc774 \uac74\uac15\ubcf4\ud5d8\uc758 \ud61c\ud0dd\uc744 \ubc1b\uae30 \uc704\ud574\uc120 \ubc18\ub4dc\uc2dc \uc5b4\ub5a4 \uc885\ub958\uc758 \uac74\uac15\ubcf4\ud5d8\uacc4\ud68d\uc5d0 \uac00\uc785\ud558\uc5ec\uc57c \ud55c\ub2e4. \uacf5\ubb34\uc6d0\uc774\ub098 \uac1c\uc778 \ud68c\uc0ac\uc758 \ub178\ub3d9\uc790\ub4e4\uc740 \uadfc\ub85c\uc790 \ubcf4\ud5d8 \ub610\ub294 \ud68c\uc0ac\uac74\uac15 \ubcf4\ud5d8\uacc4\ud68d \uc911 \ud558\ub098\uc5d0 \uac00\uc785\ud574\uc57c \ud558\uba70 \uc790\uc601\uc5c5\uc790\uc640 \ud1f4\uc9c1\uc790\ub4e4\uc740 \uc9c0\ubc29\ub2e8\uccb4\uac00 \uc6b4\uc601\ud558\ub294 \uad6d\uac00\uac74\uac15\ubcf4\ud5d8\uacc4\ud68d\uc5d0 \uac00\uc785\ud574\uc57c \ud55c\ub2e4. \uc77c\ubcf8\uc758 \uac74\uac15 \ubcf4\ud5d8\uc81c\ub3c4\uc758 \ud2b9\uc9d5\uc73c\ub85c\ub294 \u2018\uc131\uacfc\uae09 \uc81c\ub3c4\u2019\uac00 \uc788\ub2e4. \uc774 \uc81c\ub3c4\ub294 \uc758\ub8cc\uae30\uad00\ub4e4\uc774 \ud658\uc790\ub4e4\uc5d0\uac8c \uc758\ub8cc \uc11c\ube44\uc2a4\ub97c \uc81c\uacf5\ud558\uc600\uc744 \ub54c \uac74\uac15\ubcf4\ud5d8\uacc4\ud68d\uc5d0 \uc758\ud558\uc5ec \ubc30\uc0c1\uae08\uc744 \uc9c0\uae09\ubc1b\ub294 \uc81c\ub3c4\uc774\ub2e4. \uac74\uac15\ubcf4\ud5d8\uacc4\ud68d\uc740 \uc758\uc0ac\ub4e4\uc5d0\uac8c \uac1c\uc778 \ubcd1\uc6d0\uc744 \uc544\ubb34 \uacf3\uc774\ub4e0 \uac1c\uc5c5\ud560 \uc218 \uc788\ub294 \uc790\uc720\ub97c \ubd80\uc5ec\ud558\uc600\ub2e4. \uac74\uac15\uc99d\uc9c4\ubc95\uc774 \uc788\uc5b4 \ucca0\ub3c4\uc5ed \ub4f1\uc5d0\uc11c \ud761\uc5f0\uc774 \uae08\uc9c0\ub418\uc5b4 \uc788\ub2e4."} {"question": "\uccab \ubc88\uc9f8 \uc5d0\ud53c\uc18c\ub4dc\ub97c \uac00\ub9ac\ucf1c \uba85\uc791\uc774\ub77c\uace0 \ud45c\ud604\ud55c IGN\uc758 \uc791\uac00\ub294 \ub204\uad6c\uc778\uac00?", "paragraph": "\uccab \uc5d0\ud53c\uc18c\ub4dc\uc778 \u3008It Came from the Nightosphere\u3009\ub294 \ub300\uccb4\ub85c \ud638\ud3c9\uc744 \ubc1b\uc558\ub2e4. \u300aDVD \ud1a0\ud06c\u300b(DVD Talk)\uc758 \ud0c0\uc77c\ub7ec \ud3ec\uc2a4\ud130(Tyler Foster)\ub294 \ub9c8\ub974\uc140\ub9b0\uacfc \uadf8\uc758 \uc544\ube60 \uc0ac\uc774\uc758 \uac08\ub4f1\uc774 \ub2e4\ub8e8\uc5b4\uc9c0\ub294 \ubc29\uc2dd\uacfc \uadf8 \ub3c4\uc911\uc5d0 \ub098\uc628 \ud3ad\uadc4\uc5d0 \uad00\ud55c \uac1c\uadf8\uac00 \uc88b\uc558\ub2e4\uace0 \uc758\uacac\uc744 \ubc1d\ud614\ub2e4. \ub610\ud55c \u201c\ub0b4 \uc758\uacac\uc73c\ub85c\ub294 \uc5b4\ub5a4 \uc5d0\ud53c\uc18c\ub4dc\uc5d0 \ub178\ub798\uac00 \ub4e4\uc5b4\uac00\ub358 \ud50c\ub7ec\uc2a4 \uc694\uc18c\uac00 \ub420 \uac83\uc774\ub2e4.\u201d\ub77c\uace0 \ub9d0\ud588\ub2e4. \u300aMTV Geek\u300b\uc758 \ucc30\uc2a4 \uc6e8\ube0c(Charles Webb)\ub294 \ub3d9\uba85\uc758 DVD\uc5d0 \uc218\ub85d\ub41c \uc5d0\ud53c\uc18c\ub4dc \uc911\uc5d0\uc11c \uc774 \ud3b8\uc774 \u2018\ucd5c\uace0\uc758 \ud558\uc774\ub77c\uc774\ud2b8\u2019\ub77c\uace0 \uce6d\ud588\ub2e4. IGN\uc758 \uc791\uac00 \ub9e4\ud2b8 \ud30c\uc6b8\ub7ec(Matt Fowler)\ub294 \uc774 \uc5d0\ud53c\uc18c\ub4dc\ub97c \uac00\ub9ac\ucf1c \u2018\uba85\uc791\u2019(classic)\uc774\ub77c\ub294 \ud45c\ud604\uc744 \uc0ac\uc6a9\ud588\ub2e4. \ub610\ud55c 2011\ub144 \ud504\ub77c\uc784\ud0c0\uc784 \uc5d0\ubbf8\uc0c1 \u2018\ub6f0\uc5b4\ub09c \ub2e8\ud3b8 \uc560\ub2c8\uba54\uc774\uc158 \ud504\ub85c\uadf8\ub7a8\u2019\uc758 \ud6c4\ubcf4\uc5d0 \uc624\ub974\uae30\ub3c4 \ud558\uc600\ub2e4.", "answer": "\ub9e4\ud2b8 \ud30c\uc6b8\ub7ec", "sentence": "IGN\uc758 \uc791\uac00 \ub9e4\ud2b8 \ud30c\uc6b8\ub7ec (Matt Fowler)\ub294 \uc774 \uc5d0\ud53c\uc18c\ub4dc\ub97c \uac00\ub9ac\ucf1c \u2018\uba85\uc791\u2019(classic)\uc774\ub77c\ub294 \ud45c\ud604\uc744 \uc0ac\uc6a9\ud588\ub2e4.", "paragraph_sentence": "\uccab \uc5d0\ud53c\uc18c\ub4dc\uc778 \u3008It Came from the Nightosphere\u3009\ub294 \ub300\uccb4\ub85c \ud638\ud3c9\uc744 \ubc1b\uc558\ub2e4. \u300aDVD \ud1a0\ud06c\u300b(DVD Talk)\uc758 \ud0c0\uc77c\ub7ec \ud3ec\uc2a4\ud130(Tyler Foster)\ub294 \ub9c8\ub974\uc140\ub9b0\uacfc \uadf8\uc758 \uc544\ube60 \uc0ac\uc774\uc758 \uac08\ub4f1\uc774 \ub2e4\ub8e8\uc5b4\uc9c0\ub294 \ubc29\uc2dd\uacfc \uadf8 \ub3c4\uc911\uc5d0 \ub098\uc628 \ud3ad\uadc4\uc5d0 \uad00\ud55c \uac1c\uadf8\uac00 \uc88b\uc558\ub2e4\uace0 \uc758\uacac\uc744 \ubc1d\ud614\ub2e4. \ub610\ud55c \u201c\ub0b4 \uc758\uacac\uc73c\ub85c\ub294 \uc5b4\ub5a4 \uc5d0\ud53c\uc18c\ub4dc\uc5d0 \ub178\ub798\uac00 \ub4e4\uc5b4\uac00\ub358 \ud50c\ub7ec\uc2a4 \uc694\uc18c\uac00 \ub420 \uac83\uc774\ub2e4. \u201d\ub77c\uace0 \ub9d0\ud588\ub2e4. \u300aMTV Geek\u300b\uc758 \ucc30\uc2a4 \uc6e8\ube0c(Charles Webb)\ub294 \ub3d9\uba85\uc758 DVD\uc5d0 \uc218\ub85d\ub41c \uc5d0\ud53c\uc18c\ub4dc \uc911\uc5d0\uc11c \uc774 \ud3b8\uc774 \u2018\ucd5c\uace0\uc758 \ud558\uc774\ub77c\uc774\ud2b8\u2019\ub77c\uace0 \uce6d\ud588\ub2e4. IGN\uc758 \uc791\uac00 \ub9e4\ud2b8 \ud30c\uc6b8\ub7ec (Matt Fowler)\ub294 \uc774 \uc5d0\ud53c\uc18c\ub4dc\ub97c \uac00\ub9ac\ucf1c \u2018\uba85\uc791\u2019(classic)\uc774\ub77c\ub294 \ud45c\ud604\uc744 \uc0ac\uc6a9\ud588\ub2e4. \ub610\ud55c 2011\ub144 \ud504\ub77c\uc784\ud0c0\uc784 \uc5d0\ubbf8\uc0c1 \u2018\ub6f0\uc5b4\ub09c \ub2e8\ud3b8 \uc560\ub2c8\uba54\uc774\uc158 \ud504\ub85c\uadf8\ub7a8\u2019\uc758 \ud6c4\ubcf4\uc5d0 \uc624\ub974\uae30\ub3c4 \ud558\uc600\ub2e4.", "paragraph_answer": "\uccab \uc5d0\ud53c\uc18c\ub4dc\uc778 \u3008It Came from the Nightosphere\u3009\ub294 \ub300\uccb4\ub85c \ud638\ud3c9\uc744 \ubc1b\uc558\ub2e4. \u300aDVD \ud1a0\ud06c\u300b(DVD Talk)\uc758 \ud0c0\uc77c\ub7ec \ud3ec\uc2a4\ud130(Tyler Foster)\ub294 \ub9c8\ub974\uc140\ub9b0\uacfc \uadf8\uc758 \uc544\ube60 \uc0ac\uc774\uc758 \uac08\ub4f1\uc774 \ub2e4\ub8e8\uc5b4\uc9c0\ub294 \ubc29\uc2dd\uacfc \uadf8 \ub3c4\uc911\uc5d0 \ub098\uc628 \ud3ad\uadc4\uc5d0 \uad00\ud55c \uac1c\uadf8\uac00 \uc88b\uc558\ub2e4\uace0 \uc758\uacac\uc744 \ubc1d\ud614\ub2e4. \ub610\ud55c \u201c\ub0b4 \uc758\uacac\uc73c\ub85c\ub294 \uc5b4\ub5a4 \uc5d0\ud53c\uc18c\ub4dc\uc5d0 \ub178\ub798\uac00 \ub4e4\uc5b4\uac00\ub358 \ud50c\ub7ec\uc2a4 \uc694\uc18c\uac00 \ub420 \uac83\uc774\ub2e4.\u201d\ub77c\uace0 \ub9d0\ud588\ub2e4. \u300aMTV Geek\u300b\uc758 \ucc30\uc2a4 \uc6e8\ube0c(Charles Webb)\ub294 \ub3d9\uba85\uc758 DVD\uc5d0 \uc218\ub85d\ub41c \uc5d0\ud53c\uc18c\ub4dc \uc911\uc5d0\uc11c \uc774 \ud3b8\uc774 \u2018\ucd5c\uace0\uc758 \ud558\uc774\ub77c\uc774\ud2b8\u2019\ub77c\uace0 \uce6d\ud588\ub2e4. IGN\uc758 \uc791\uac00 \ub9e4\ud2b8 \ud30c\uc6b8\ub7ec (Matt Fowler)\ub294 \uc774 \uc5d0\ud53c\uc18c\ub4dc\ub97c \uac00\ub9ac\ucf1c \u2018\uba85\uc791\u2019(classic)\uc774\ub77c\ub294 \ud45c\ud604\uc744 \uc0ac\uc6a9\ud588\ub2e4. \ub610\ud55c 2011\ub144 \ud504\ub77c\uc784\ud0c0\uc784 \uc5d0\ubbf8\uc0c1 \u2018\ub6f0\uc5b4\ub09c \ub2e8\ud3b8 \uc560\ub2c8\uba54\uc774\uc158 \ud504\ub85c\uadf8\ub7a8\u2019\uc758 \ud6c4\ubcf4\uc5d0 \uc624\ub974\uae30\ub3c4 \ud558\uc600\ub2e4.", "sentence_answer": "IGN\uc758 \uc791\uac00 \ub9e4\ud2b8 \ud30c\uc6b8\ub7ec (Matt Fowler)\ub294 \uc774 \uc5d0\ud53c\uc18c\ub4dc\ub97c \uac00\ub9ac\ucf1c \u2018\uba85\uc791\u2019(classic)\uc774\ub77c\ub294 \ud45c\ud604\uc744 \uc0ac\uc6a9\ud588\ub2e4.", "paragraph_id": "6569379-7-1", "paragraph_question": "question: \uccab \ubc88\uc9f8 \uc5d0\ud53c\uc18c\ub4dc\ub97c \uac00\ub9ac\ucf1c \uba85\uc791\uc774\ub77c\uace0 \ud45c\ud604\ud55c IGN\uc758 \uc791\uac00\ub294 \ub204\uad6c\uc778\uac00?, context: \uccab \uc5d0\ud53c\uc18c\ub4dc\uc778 \u3008It Came from the Nightosphere\u3009\ub294 \ub300\uccb4\ub85c \ud638\ud3c9\uc744 \ubc1b\uc558\ub2e4. \u300aDVD \ud1a0\ud06c\u300b(DVD Talk)\uc758 \ud0c0\uc77c\ub7ec \ud3ec\uc2a4\ud130(Tyler Foster)\ub294 \ub9c8\ub974\uc140\ub9b0\uacfc \uadf8\uc758 \uc544\ube60 \uc0ac\uc774\uc758 \uac08\ub4f1\uc774 \ub2e4\ub8e8\uc5b4\uc9c0\ub294 \ubc29\uc2dd\uacfc \uadf8 \ub3c4\uc911\uc5d0 \ub098\uc628 \ud3ad\uadc4\uc5d0 \uad00\ud55c \uac1c\uadf8\uac00 \uc88b\uc558\ub2e4\uace0 \uc758\uacac\uc744 \ubc1d\ud614\ub2e4. \ub610\ud55c \u201c\ub0b4 \uc758\uacac\uc73c\ub85c\ub294 \uc5b4\ub5a4 \uc5d0\ud53c\uc18c\ub4dc\uc5d0 \ub178\ub798\uac00 \ub4e4\uc5b4\uac00\ub358 \ud50c\ub7ec\uc2a4 \uc694\uc18c\uac00 \ub420 \uac83\uc774\ub2e4.\u201d\ub77c\uace0 \ub9d0\ud588\ub2e4. \u300aMTV Geek\u300b\uc758 \ucc30\uc2a4 \uc6e8\ube0c(Charles Webb)\ub294 \ub3d9\uba85\uc758 DVD\uc5d0 \uc218\ub85d\ub41c \uc5d0\ud53c\uc18c\ub4dc \uc911\uc5d0\uc11c \uc774 \ud3b8\uc774 \u2018\ucd5c\uace0\uc758 \ud558\uc774\ub77c\uc774\ud2b8\u2019\ub77c\uace0 \uce6d\ud588\ub2e4. IGN\uc758 \uc791\uac00 \ub9e4\ud2b8 \ud30c\uc6b8\ub7ec(Matt Fowler)\ub294 \uc774 \uc5d0\ud53c\uc18c\ub4dc\ub97c \uac00\ub9ac\ucf1c \u2018\uba85\uc791\u2019(classic)\uc774\ub77c\ub294 \ud45c\ud604\uc744 \uc0ac\uc6a9\ud588\ub2e4. \ub610\ud55c 2011\ub144 \ud504\ub77c\uc784\ud0c0\uc784 \uc5d0\ubbf8\uc0c1 \u2018\ub6f0\uc5b4\ub09c \ub2e8\ud3b8 \uc560\ub2c8\uba54\uc774\uc158 \ud504\ub85c\uadf8\ub7a8\u2019\uc758 \ud6c4\ubcf4\uc5d0 \uc624\ub974\uae30\ub3c4 \ud558\uc600\ub2e4."} {"question": "\uc131\uc120 \ubc0f \ud56d\uacf5\uae30 \ud1b5\uc2e0, \uc544\ub9c8\ucd94\uc5b4 \ubb34\uc120\ud1b5\uc2e0\uc5d0\uc11c\ub294 \uc5b4\ub5a4 \uc608\ubcf4\uc5d0 \uad00\uc2ec\uc744 \uac16\ub294\uac00?", "paragraph": "\uc804\ud30c \ud1b5\uc2e0\uc758 \uc0c1\uacf5\ud30c \ubaa8\ub4dc\ub294 \uc774\uc628\uad8c\uc744 \ud1b5\uacfc\ud560 \ub54c \uad74\uc808\ud558\ub294 \uc804\ud30c (\uc804\uc790\uae30\ud30c)\uc758 \uc758\ud574 \ubc1c\uc0dd\ud55c\ub2e4. \ud0dc\uc591 \uc8fc\uae30\uc758 \u201c\uc808\uc815\u201d\uc5d0\uc11c \uc774\uc628\uad8c\uc740 \ud0dc\uc591\uc5d0\uc11c \uc624\ub294 \uad11\uc790\uc640 \uc6b0\uc8fc\uc120\uc785\uc790\uc5d0 \uc758\ud574 \uc774\uc628\ud654\ub418\ub294 \uc815\ub3c4\uac00 \uc99d\uac00\ud55c\ub2e4. \uc774\ub294 \uad6d\uc9c0\uc801\uc778 \uc7a5\uac70\ub9ac \ud1b5\uc2e0\uc744 \uc774\uc6a9\uac00\ub2a5\ud558\uac8c \ud558\uac70\ub098 \ub610\ub294 \ubc29\ud574\ud558\ub294 \uc2dd\uc758 \ubcf5\uc7a1\ud55c \ubc29\ubc95\uc73c\ub85c \uc804\ud30c \uc9c4\ud589 \uacbd\ub85c\uc5d0 \uc601\ud5a5\uc744 \uc900\ub2e4. \uc0c1\uacf5\ud30c \ubaa8\ub4dc\uc758 \uc608\ubcf4\ub294 \uc0c1\uc120 \ubc0f \ud56d\uacf5\uae30 \ud1b5\uc2e0, \uc544\ub9c8\ucd94\uc5b4 \ubb34\uc120\ud1b5\uc2e0, \ub2e8\ud30c \ubc29\uc1a1\uc0b0\uc5c5 \ubd84\uc57c\uc5d0\uc11c \uc0c1\ub2f9\ud55c \uad00\uc2ec\uc744 \ubc1b\uace0 \uc788\ub2e4. \uc774 \ubd84\uc57c\uc758 \uc885\uc0ac\uc790\ub4e4\uc740 \uc774\ub7ec\ud55c \ud0dc\uc591\uacfc \uc774\uc628\uad8c\uc5d0\uc11c\uc758 \ubcc0\ud654\uc5d0 \uac00\uc7a5 \uc601\ud5a5\uc744 \ub9ce\uc774 \ubc1b\ub294 \uace0\uc8fc\ud30c(HF) \uc2a4\ud399\ud2b8\ub7fc\ub0b4\uc758 \uc8fc\ud30c\uc218\ub4e4\uc744 \uc2e4\uc6a9\ud654\ud55c\ub2e4. \ud0dc\uc591 \ubc29\ucd9c\ub7c9\uc5d0\uc11c\uc758 \ubcc0\ud654\ub294 \ucd5c\ub300\uc0ac\uc6a9\uc8fc\ud30c\uc218(\ud1b5\uc2e0\uc5d0 \uc774\uc6a9\uac00\ub2a5\ud55c \uac00\uc7a5 \ub192\uc740 \uc8fc\ud30c\uc218\uc758 \ud55c\uacc4\uce58) \uc5d0 \uc601\ud5a5\uc744 \uc900\ub2e4.", "answer": "\uc0c1\uacf5\ud30c \ubaa8\ub4dc\uc758 \uc608\ubcf4", "sentence": "\uc0c1\uacf5\ud30c \ubaa8\ub4dc\uc758 \uc608\ubcf4 \ub294 \uc0c1\uc120 \ubc0f \ud56d\uacf5\uae30 \ud1b5\uc2e0, \uc544\ub9c8\ucd94\uc5b4 \ubb34\uc120\ud1b5\uc2e0, \ub2e8\ud30c \ubc29\uc1a1\uc0b0\uc5c5 \ubd84\uc57c\uc5d0\uc11c \uc0c1\ub2f9\ud55c \uad00\uc2ec\uc744 \ubc1b\uace0 \uc788\ub2e4.", "paragraph_sentence": "\uc804\ud30c \ud1b5\uc2e0\uc758 \uc0c1\uacf5\ud30c \ubaa8\ub4dc\ub294 \uc774\uc628\uad8c\uc744 \ud1b5\uacfc\ud560 \ub54c \uad74\uc808\ud558\ub294 \uc804\ud30c (\uc804\uc790\uae30\ud30c)\uc758 \uc758\ud574 \ubc1c\uc0dd\ud55c\ub2e4. \ud0dc\uc591 \uc8fc\uae30\uc758 \u201c\uc808\uc815\u201d\uc5d0\uc11c \uc774\uc628\uad8c\uc740 \ud0dc\uc591\uc5d0\uc11c \uc624\ub294 \uad11\uc790\uc640 \uc6b0\uc8fc\uc120\uc785\uc790\uc5d0 \uc758\ud574 \uc774\uc628\ud654\ub418\ub294 \uc815\ub3c4\uac00 \uc99d\uac00\ud55c\ub2e4. \uc774\ub294 \uad6d\uc9c0\uc801\uc778 \uc7a5\uac70\ub9ac \ud1b5\uc2e0\uc744 \uc774\uc6a9\uac00\ub2a5\ud558\uac8c \ud558\uac70\ub098 \ub610\ub294 \ubc29\ud574\ud558\ub294 \uc2dd\uc758 \ubcf5\uc7a1\ud55c \ubc29\ubc95\uc73c\ub85c \uc804\ud30c \uc9c4\ud589 \uacbd\ub85c\uc5d0 \uc601\ud5a5\uc744 \uc900\ub2e4. \uc0c1\uacf5\ud30c \ubaa8\ub4dc\uc758 \uc608\ubcf4 \ub294 \uc0c1\uc120 \ubc0f \ud56d\uacf5\uae30 \ud1b5\uc2e0, \uc544\ub9c8\ucd94\uc5b4 \ubb34\uc120\ud1b5\uc2e0, \ub2e8\ud30c \ubc29\uc1a1\uc0b0\uc5c5 \ubd84\uc57c\uc5d0\uc11c \uc0c1\ub2f9\ud55c \uad00\uc2ec\uc744 \ubc1b\uace0 \uc788\ub2e4. \uc774 \ubd84\uc57c\uc758 \uc885\uc0ac\uc790\ub4e4\uc740 \uc774\ub7ec\ud55c \ud0dc\uc591\uacfc \uc774\uc628\uad8c\uc5d0\uc11c\uc758 \ubcc0\ud654\uc5d0 \uac00\uc7a5 \uc601\ud5a5\uc744 \ub9ce\uc774 \ubc1b\ub294 \uace0\uc8fc\ud30c(HF) \uc2a4\ud399\ud2b8\ub7fc\ub0b4\uc758 \uc8fc\ud30c\uc218\ub4e4\uc744 \uc2e4\uc6a9\ud654\ud55c\ub2e4. \ud0dc\uc591 \ubc29\ucd9c\ub7c9\uc5d0\uc11c\uc758 \ubcc0\ud654\ub294 \ucd5c\ub300\uc0ac\uc6a9\uc8fc\ud30c\uc218(\ud1b5\uc2e0\uc5d0 \uc774\uc6a9\uac00\ub2a5\ud55c \uac00\uc7a5 \ub192\uc740 \uc8fc\ud30c\uc218\uc758 \ud55c\uacc4\uce58) \uc5d0 \uc601\ud5a5\uc744 \uc900\ub2e4.", "paragraph_answer": "\uc804\ud30c \ud1b5\uc2e0\uc758 \uc0c1\uacf5\ud30c \ubaa8\ub4dc\ub294 \uc774\uc628\uad8c\uc744 \ud1b5\uacfc\ud560 \ub54c \uad74\uc808\ud558\ub294 \uc804\ud30c (\uc804\uc790\uae30\ud30c)\uc758 \uc758\ud574 \ubc1c\uc0dd\ud55c\ub2e4. \ud0dc\uc591 \uc8fc\uae30\uc758 \u201c\uc808\uc815\u201d\uc5d0\uc11c \uc774\uc628\uad8c\uc740 \ud0dc\uc591\uc5d0\uc11c \uc624\ub294 \uad11\uc790\uc640 \uc6b0\uc8fc\uc120\uc785\uc790\uc5d0 \uc758\ud574 \uc774\uc628\ud654\ub418\ub294 \uc815\ub3c4\uac00 \uc99d\uac00\ud55c\ub2e4. \uc774\ub294 \uad6d\uc9c0\uc801\uc778 \uc7a5\uac70\ub9ac \ud1b5\uc2e0\uc744 \uc774\uc6a9\uac00\ub2a5\ud558\uac8c \ud558\uac70\ub098 \ub610\ub294 \ubc29\ud574\ud558\ub294 \uc2dd\uc758 \ubcf5\uc7a1\ud55c \ubc29\ubc95\uc73c\ub85c \uc804\ud30c \uc9c4\ud589 \uacbd\ub85c\uc5d0 \uc601\ud5a5\uc744 \uc900\ub2e4. \uc0c1\uacf5\ud30c \ubaa8\ub4dc\uc758 \uc608\ubcf4 \ub294 \uc0c1\uc120 \ubc0f \ud56d\uacf5\uae30 \ud1b5\uc2e0, \uc544\ub9c8\ucd94\uc5b4 \ubb34\uc120\ud1b5\uc2e0, \ub2e8\ud30c \ubc29\uc1a1\uc0b0\uc5c5 \ubd84\uc57c\uc5d0\uc11c \uc0c1\ub2f9\ud55c \uad00\uc2ec\uc744 \ubc1b\uace0 \uc788\ub2e4. \uc774 \ubd84\uc57c\uc758 \uc885\uc0ac\uc790\ub4e4\uc740 \uc774\ub7ec\ud55c \ud0dc\uc591\uacfc \uc774\uc628\uad8c\uc5d0\uc11c\uc758 \ubcc0\ud654\uc5d0 \uac00\uc7a5 \uc601\ud5a5\uc744 \ub9ce\uc774 \ubc1b\ub294 \uace0\uc8fc\ud30c(HF) \uc2a4\ud399\ud2b8\ub7fc\ub0b4\uc758 \uc8fc\ud30c\uc218\ub4e4\uc744 \uc2e4\uc6a9\ud654\ud55c\ub2e4. \ud0dc\uc591 \ubc29\ucd9c\ub7c9\uc5d0\uc11c\uc758 \ubcc0\ud654\ub294 \ucd5c\ub300\uc0ac\uc6a9\uc8fc\ud30c\uc218(\ud1b5\uc2e0\uc5d0 \uc774\uc6a9\uac00\ub2a5\ud55c \uac00\uc7a5 \ub192\uc740 \uc8fc\ud30c\uc218\uc758 \ud55c\uacc4\uce58) \uc5d0 \uc601\ud5a5\uc744 \uc900\ub2e4.", "sentence_answer": " \uc0c1\uacf5\ud30c \ubaa8\ub4dc\uc758 \uc608\ubcf4 \ub294 \uc0c1\uc120 \ubc0f \ud56d\uacf5\uae30 \ud1b5\uc2e0, \uc544\ub9c8\ucd94\uc5b4 \ubb34\uc120\ud1b5\uc2e0, \ub2e8\ud30c \ubc29\uc1a1\uc0b0\uc5c5 \ubd84\uc57c\uc5d0\uc11c \uc0c1\ub2f9\ud55c \uad00\uc2ec\uc744 \ubc1b\uace0 \uc788\ub2e4.", "paragraph_id": "6565092-2-1", "paragraph_question": "question: \uc131\uc120 \ubc0f \ud56d\uacf5\uae30 \ud1b5\uc2e0, \uc544\ub9c8\ucd94\uc5b4 \ubb34\uc120\ud1b5\uc2e0\uc5d0\uc11c\ub294 \uc5b4\ub5a4 \uc608\ubcf4\uc5d0 \uad00\uc2ec\uc744 \uac16\ub294\uac00?, context: \uc804\ud30c \ud1b5\uc2e0\uc758 \uc0c1\uacf5\ud30c \ubaa8\ub4dc\ub294 \uc774\uc628\uad8c\uc744 \ud1b5\uacfc\ud560 \ub54c \uad74\uc808\ud558\ub294 \uc804\ud30c (\uc804\uc790\uae30\ud30c)\uc758 \uc758\ud574 \ubc1c\uc0dd\ud55c\ub2e4. \ud0dc\uc591 \uc8fc\uae30\uc758 \u201c\uc808\uc815\u201d\uc5d0\uc11c \uc774\uc628\uad8c\uc740 \ud0dc\uc591\uc5d0\uc11c \uc624\ub294 \uad11\uc790\uc640 \uc6b0\uc8fc\uc120\uc785\uc790\uc5d0 \uc758\ud574 \uc774\uc628\ud654\ub418\ub294 \uc815\ub3c4\uac00 \uc99d\uac00\ud55c\ub2e4. \uc774\ub294 \uad6d\uc9c0\uc801\uc778 \uc7a5\uac70\ub9ac \ud1b5\uc2e0\uc744 \uc774\uc6a9\uac00\ub2a5\ud558\uac8c \ud558\uac70\ub098 \ub610\ub294 \ubc29\ud574\ud558\ub294 \uc2dd\uc758 \ubcf5\uc7a1\ud55c \ubc29\ubc95\uc73c\ub85c \uc804\ud30c \uc9c4\ud589 \uacbd\ub85c\uc5d0 \uc601\ud5a5\uc744 \uc900\ub2e4. \uc0c1\uacf5\ud30c \ubaa8\ub4dc\uc758 \uc608\ubcf4\ub294 \uc0c1\uc120 \ubc0f \ud56d\uacf5\uae30 \ud1b5\uc2e0, \uc544\ub9c8\ucd94\uc5b4 \ubb34\uc120\ud1b5\uc2e0, \ub2e8\ud30c \ubc29\uc1a1\uc0b0\uc5c5 \ubd84\uc57c\uc5d0\uc11c \uc0c1\ub2f9\ud55c \uad00\uc2ec\uc744 \ubc1b\uace0 \uc788\ub2e4. \uc774 \ubd84\uc57c\uc758 \uc885\uc0ac\uc790\ub4e4\uc740 \uc774\ub7ec\ud55c \ud0dc\uc591\uacfc \uc774\uc628\uad8c\uc5d0\uc11c\uc758 \ubcc0\ud654\uc5d0 \uac00\uc7a5 \uc601\ud5a5\uc744 \ub9ce\uc774 \ubc1b\ub294 \uace0\uc8fc\ud30c(HF) \uc2a4\ud399\ud2b8\ub7fc\ub0b4\uc758 \uc8fc\ud30c\uc218\ub4e4\uc744 \uc2e4\uc6a9\ud654\ud55c\ub2e4. \ud0dc\uc591 \ubc29\ucd9c\ub7c9\uc5d0\uc11c\uc758 \ubcc0\ud654\ub294 \ucd5c\ub300\uc0ac\uc6a9\uc8fc\ud30c\uc218(\ud1b5\uc2e0\uc5d0 \uc774\uc6a9\uac00\ub2a5\ud55c \uac00\uc7a5 \ub192\uc740 \uc8fc\ud30c\uc218\uc758 \ud55c\uacc4\uce58) \uc5d0 \uc601\ud5a5\uc744 \uc900\ub2e4."} {"question": "\ub098\uce74\ubaa8\ub9ac \uc544\ud0a4\ub098\ub97c \ud76c\ub300\uc758 \ubcf4\uceec\ub9ac\uc2a4\ud2b8\uc774\uba70 \ub300\uc911\ub4e4\uc774 \uc54c\uace0 \uc788\ub294 \ud754\ud55c \uc0ac\uc2e4\uc744 \uc74c\ubc18\uc5d0\uc11c \uae30\ub300 \uc774\uc0c1\uc73c\ub85c \ub04c\uc5b4\uc62c\ub9b0\ub2e4\uace0 \ub9d0\ud55c \uc0ac\ub78c\uc740 \ub204\uad6c\uc77c\uae4c?", "paragraph": "\uc74c\uc545 \ub77c\uc774\ud130\uc778 \ub9c8\uce74\uc774\ub178 \ubaa8\ud1a0\ud788\ub85c\ub294 \ub098\uce74\ubaa8\ub9ac \uc544\ud0a4\ub098\uc758 \ucd5c\ub300 \ud2b9\uc9d5\uc740 \ubc14\ub85c \u201c\ub178\uc120\u201d\uc744 \uc815\ud558\uc9c0 \uc54a\uc558\ub2e4\ub294 \uac83\uc73c\ub85c\uc11c \uac00\ucc3d\ub825\uc758 \uc5ed\ub7c9\ubcf4\ub2e4\ub294 \ud48d\ubd80\ud55c \uac10\uc815\ud45c\ud604\uc73c\ub85c \uc131\ub7c9\uc744 \uae30\uc5b4 \uccb4\uc778\uc9c0\uc640 \uac19\uc774 \ubc14\uafb8\uc5b4 \ud2b9\uc720\uc758 \ube44\ube0c\ub77c\ud1a0 \ucc3d\ubc95\uc744 \ud655\ub9bd\ud558\uc600\ub2e4\uace0 \uc9c0\uc801\ud558\uc600\ub2e4. \uadf8\ub7ec\ub098 \u300aCD\uc800\ub110\u300b\uc740 \uadf8\ub140\uac00 \ub2f9\uc2dc \uc218\ub9ce\uc740 \uc544\uc774\ub3cc \uac00\uc218 \uc911\uc5d0\uc11c\ub3c4 \ucd5c\uace0\uc758 \uac00\ucc3d\ub825\uacfc \ud3c9\uac00\ub97c \ubc1b\uc558\ub2e4\uace0 \ud638\ud3c9\ud588\ub2e4. \ub610\ud55c 1989\ub144 \uc790\uc0b4\ubbf8\uc218 \uc0ac\uac74 \uc774\ud6c4 \ubcf5\uadc0\ud558\uace0 \uc120\ubcf4\uc778 \uc74c\ubc18 \u300aUNBALANCE+BALANCE\u300b\uc5d0\uc11c\ub294 \uadf8\ub140\uac00 \uc5ed\uc2dc \ub178\ub798\ub97c \uc798 \ubd80\ub974\ub294 \uc0ac\ub78c\uc774\uc5c8\ub2e4\ub294 \uac83\uc744 \ub2e4\uc2dc \uc778\uc2dd\uc2dc\ucf1c\uc92c\ub2e4\uace0 \ud3c9\ud588\ub2e4. \uac19\uc740 \uc7a1\uc9c0\uc758 \ud3c9\ub860\uac00 \uace0\ud1a0 \uc720\ud0a4\ud788\ub85c \uc5ed\uc2dc \uc11c\uc591 \uc74c\uc545\uc801\uc778 \uc694\uc18c \uc18d\uc5d0\uc11c \uc77c\ubcf8\uc5b4 \uac00\uc0ac\ub97c \uc774\ub807\uac8c \ub2e4\uc591\ud558\uba74\uc11c\ub3c4 \uc544\ub984\ub2f5\uac8c \uc6b8\ub9ac\uac8c \ud558\ub294 \uac00\uc218\ub294 \uc544\ud0a4\ub098\uac00 \ub9c8\uc9c0\ub9c9\uc774\ub77c \ud558\uba70 \uae0d\uc815\uc801\uc778 \ud3c9\uac00\ub97c \ub0b4\ub838\ub2e4. \uc74c\uc545\ud3c9\ub860\uac00 \ub178\uc9c0 \uc720\ucf54\ub294 \uc74c\uc545 \uc7a1\uc9c0 \u300aWHAT'S IN?\u300b\uc5d0\uc11c \uadf8\ub140\uc758 \ucee4\ubc84 \uc74c\ubc18 \u300a\uac00\ud76c\u300b\ub97c \ub450\uace0 \ub514\ubc14\ub85c\uc11c\uc758 \ud65c\ub825\uc774 \uc628\ubab8\uc5d0 \ub118\uce58\ub294 \uac83 \uac19\ub2e4\uace0 \uc9c0\uc801, \uae30\ub7c9 \uc774\uc804\uc5d0 \uc7ac\ub2a5\uc774 \ub118\uce58\ub294 \uac00\uc218\ub85c \ud3c9\ud558\uc600\ub2e4. \uc74c\uc545 \uc7a1\uc9c0 \u300a\ubba4\uc9c1 \ub9e4\uac70\uc9c4\u300b\uc758 \uce7c\ub7fc\ub2c8\uc2a4\ud2b8 \ud638\ubcf4 \ub2e4\uc774\uc790\ubd80\ub85c\ub294 \uadf8\ub140\ub97c \ub450\uace0 \uc601\ud63c\uc758 \uc2ec\uc5f0\uc5d0\uc11c \uad49\uc7a5\ud55c \ud761\uc785\ub825\uc73c\ub85c \ub4e3\ub294 \uc0ac\ub78c\uc744 \ub04c\uc5b4\ub2f9\uae30\uac8c \ud55c\ub2e4\uace0 \ub9d0\ud588\ub2e4. \ub610\ud55c \uc774\ub9c8\ubaa8\ud1a0 \ub3c4\uc694\uc2dc\ub294 \ub098\uce74\ubaa8\ub9ac\ub97c \uc77c\ub7ec \ud76c\ub300\uc758 \ubcf4\uceec\ub9ac\uc2a4\ud2b8\ub85c, \ub300\uc911\ub4e4\uc774 \uc54c\uace0 \uc788\ub294 \uadf8 \ud754\ud55c \uc0ac\uc2e4\uc744 \uc74c\ubc18\uc5d0\uc11c\ub294 \uae30\ub300 \uc774\uc0c1\uc73c\ub85c \ub04c\uc5b4\uc62c\ub9ac\ub294 \ub2a5\ub825\uc744 \uac16\ucd94\uc5c8\ub2e4\uace0 \ube44\ud3c9\ud558\uc600\ub2e4. MusicVoice\uc758 \ud5e4\uc774\ud0a4\uce58 \ucf04\uc9c0(\u5e73\u5409\u8ce2\u6cbb)\ub294 \ub098\uce74\ubaa8\ub9ac\uac00 \u2018\ub178\ub798\uc758 \uad6c\uc131\ub825\u2019\uc774 \uc788\uc5b4 \ub9c8\uc74c\uc744 \uc6b8\ub9ac\ub294 \uc124\ub4dd\ub825\uc73c\ub85c \uc74c\uc545\uc744 \ub4e3\ub294 \uc0ac\ub78c\ub4e4\uc5d0\uac8c \ub354 \uae4a\uc740 \uac10\uc815\uc744 \ud638\uc18c\ub825\uc788\uac8c \uc804\ub2ec\ud558\uace0, \uace1\uc758 \uc808\uc815\uc5d0\ub294 \u2018\uac10\uc815\ud45c\ud604\uacfc \uc644\uae09\uc870\uc808\uc758 \ubc30\ubd84\u2019\uc774 \uba85\ud655\ud558\uc5ec \uc801\uc808\ud55c \ubd80\ubd84\uacfc \ud0c0\uc774\ubc0d\uc5d0 \uccad\uc911\uc744 \uaf3c\uc9dd \ubabb\ud558\uac8c \ub9e4\ub8cc\uc2dc\ud0a8\ub2e4\uace0 \ud638\ud3c9\ud558\uc600\ub2e4.", "answer": "\uc774\ub9c8\ubaa8\ud1a0 \ub3c4\uc694\uc2dc", "sentence": "\ub610\ud55c \uc774\ub9c8\ubaa8\ud1a0 \ub3c4\uc694\uc2dc \ub294 \ub098\uce74\ubaa8\ub9ac\ub97c \uc77c\ub7ec \ud76c\ub300\uc758 \ubcf4\uceec\ub9ac\uc2a4\ud2b8\ub85c, \ub300\uc911\ub4e4\uc774 \uc54c\uace0 \uc788\ub294 \uadf8 \ud754\ud55c \uc0ac\uc2e4\uc744 \uc74c\ubc18\uc5d0\uc11c\ub294 \uae30\ub300 \uc774\uc0c1\uc73c\ub85c \ub04c\uc5b4\uc62c\ub9ac\ub294 \ub2a5\ub825\uc744 \uac16\ucd94\uc5c8\ub2e4\uace0 \ube44\ud3c9\ud558\uc600\ub2e4.", "paragraph_sentence": "\uc74c\uc545 \ub77c\uc774\ud130\uc778 \ub9c8\uce74\uc774\ub178 \ubaa8\ud1a0\ud788\ub85c\ub294 \ub098\uce74\ubaa8\ub9ac \uc544\ud0a4\ub098\uc758 \ucd5c\ub300 \ud2b9\uc9d5\uc740 \ubc14\ub85c \u201c\ub178\uc120\u201d\uc744 \uc815\ud558\uc9c0 \uc54a\uc558\ub2e4\ub294 \uac83\uc73c\ub85c\uc11c \uac00\ucc3d\ub825\uc758 \uc5ed\ub7c9\ubcf4\ub2e4\ub294 \ud48d\ubd80\ud55c \uac10\uc815\ud45c\ud604\uc73c\ub85c \uc131\ub7c9\uc744 \uae30\uc5b4 \uccb4\uc778\uc9c0\uc640 \uac19\uc774 \ubc14\uafb8\uc5b4 \ud2b9\uc720\uc758 \ube44\ube0c\ub77c\ud1a0 \ucc3d\ubc95\uc744 \ud655\ub9bd\ud558\uc600\ub2e4\uace0 \uc9c0\uc801\ud558\uc600\ub2e4. \uadf8\ub7ec\ub098 \u300aCD\uc800\ub110\u300b\uc740 \uadf8\ub140\uac00 \ub2f9\uc2dc \uc218\ub9ce\uc740 \uc544\uc774\ub3cc \uac00\uc218 \uc911\uc5d0\uc11c\ub3c4 \ucd5c\uace0\uc758 \uac00\ucc3d\ub825\uacfc \ud3c9\uac00\ub97c \ubc1b\uc558\ub2e4\uace0 \ud638\ud3c9\ud588\ub2e4. \ub610\ud55c 1989\ub144 \uc790\uc0b4\ubbf8\uc218 \uc0ac\uac74 \uc774\ud6c4 \ubcf5\uadc0\ud558\uace0 \uc120\ubcf4\uc778 \uc74c\ubc18 \u300aUNBALANCE+BALANCE\u300b\uc5d0\uc11c\ub294 \uadf8\ub140\uac00 \uc5ed\uc2dc \ub178\ub798\ub97c \uc798 \ubd80\ub974\ub294 \uc0ac\ub78c\uc774\uc5c8\ub2e4\ub294 \uac83\uc744 \ub2e4\uc2dc \uc778\uc2dd\uc2dc\ucf1c\uc92c\ub2e4\uace0 \ud3c9\ud588\ub2e4. \uac19\uc740 \uc7a1\uc9c0\uc758 \ud3c9\ub860\uac00 \uace0\ud1a0 \uc720\ud0a4\ud788\ub85c \uc5ed\uc2dc \uc11c\uc591 \uc74c\uc545\uc801\uc778 \uc694\uc18c \uc18d\uc5d0\uc11c \uc77c\ubcf8\uc5b4 \uac00\uc0ac\ub97c \uc774\ub807\uac8c \ub2e4\uc591\ud558\uba74\uc11c\ub3c4 \uc544\ub984\ub2f5\uac8c \uc6b8\ub9ac\uac8c \ud558\ub294 \uac00\uc218\ub294 \uc544\ud0a4\ub098\uac00 \ub9c8\uc9c0\ub9c9\uc774\ub77c \ud558\uba70 \uae0d\uc815\uc801\uc778 \ud3c9\uac00\ub97c \ub0b4\ub838\ub2e4. \uc74c\uc545\ud3c9\ub860\uac00 \ub178\uc9c0 \uc720\ucf54\ub294 \uc74c\uc545 \uc7a1\uc9c0 \u300aWHAT'S IN?\u300b\uc5d0\uc11c \uadf8\ub140\uc758 \ucee4\ubc84 \uc74c\ubc18 \u300a\uac00\ud76c\u300b\ub97c \ub450\uace0 \ub514\ubc14\ub85c\uc11c\uc758 \ud65c\ub825\uc774 \uc628\ubab8\uc5d0 \ub118\uce58\ub294 \uac83 \uac19\ub2e4\uace0 \uc9c0\uc801, \uae30\ub7c9 \uc774\uc804\uc5d0 \uc7ac\ub2a5\uc774 \ub118\uce58\ub294 \uac00\uc218\ub85c \ud3c9\ud558\uc600\ub2e4. \uc74c\uc545 \uc7a1\uc9c0 \u300a\ubba4\uc9c1 \ub9e4\uac70\uc9c4\u300b\uc758 \uce7c\ub7fc\ub2c8\uc2a4\ud2b8 \ud638\ubcf4 \ub2e4\uc774\uc790\ubd80\ub85c\ub294 \uadf8\ub140\ub97c \ub450\uace0 \uc601\ud63c\uc758 \uc2ec\uc5f0\uc5d0\uc11c \uad49\uc7a5\ud55c \ud761\uc785\ub825\uc73c\ub85c \ub4e3\ub294 \uc0ac\ub78c\uc744 \ub04c\uc5b4\ub2f9\uae30\uac8c \ud55c\ub2e4\uace0 \ub9d0\ud588\ub2e4. \ub610\ud55c \uc774\ub9c8\ubaa8\ud1a0 \ub3c4\uc694\uc2dc \ub294 \ub098\uce74\ubaa8\ub9ac\ub97c \uc77c\ub7ec \ud76c\ub300\uc758 \ubcf4\uceec\ub9ac\uc2a4\ud2b8\ub85c, \ub300\uc911\ub4e4\uc774 \uc54c\uace0 \uc788\ub294 \uadf8 \ud754\ud55c \uc0ac\uc2e4\uc744 \uc74c\ubc18\uc5d0\uc11c\ub294 \uae30\ub300 \uc774\uc0c1\uc73c\ub85c \ub04c\uc5b4\uc62c\ub9ac\ub294 \ub2a5\ub825\uc744 \uac16\ucd94\uc5c8\ub2e4\uace0 \ube44\ud3c9\ud558\uc600\ub2e4. MusicVoice\uc758 \ud5e4\uc774\ud0a4\uce58 \ucf04\uc9c0(\u5e73\u5409\u8ce2\u6cbb)\ub294 \ub098\uce74\ubaa8\ub9ac\uac00 \u2018\ub178\ub798\uc758 \uad6c\uc131\ub825\u2019\uc774 \uc788\uc5b4 \ub9c8\uc74c\uc744 \uc6b8\ub9ac\ub294 \uc124\ub4dd\ub825\uc73c\ub85c \uc74c\uc545\uc744 \ub4e3\ub294 \uc0ac\ub78c\ub4e4\uc5d0\uac8c \ub354 \uae4a\uc740 \uac10\uc815\uc744 \ud638\uc18c\ub825\uc788\uac8c \uc804\ub2ec\ud558\uace0, \uace1\uc758 \uc808\uc815\uc5d0\ub294 \u2018\uac10\uc815\ud45c\ud604\uacfc \uc644\uae09\uc870\uc808\uc758 \ubc30\ubd84\u2019\uc774 \uba85\ud655\ud558\uc5ec \uc801\uc808\ud55c \ubd80\ubd84\uacfc \ud0c0\uc774\ubc0d\uc5d0 \uccad\uc911\uc744 \uaf3c\uc9dd \ubabb\ud558\uac8c \ub9e4\ub8cc\uc2dc\ud0a8\ub2e4\uace0 \ud638\ud3c9\ud558\uc600\ub2e4.", "paragraph_answer": "\uc74c\uc545 \ub77c\uc774\ud130\uc778 \ub9c8\uce74\uc774\ub178 \ubaa8\ud1a0\ud788\ub85c\ub294 \ub098\uce74\ubaa8\ub9ac \uc544\ud0a4\ub098\uc758 \ucd5c\ub300 \ud2b9\uc9d5\uc740 \ubc14\ub85c \u201c\ub178\uc120\u201d\uc744 \uc815\ud558\uc9c0 \uc54a\uc558\ub2e4\ub294 \uac83\uc73c\ub85c\uc11c \uac00\ucc3d\ub825\uc758 \uc5ed\ub7c9\ubcf4\ub2e4\ub294 \ud48d\ubd80\ud55c \uac10\uc815\ud45c\ud604\uc73c\ub85c \uc131\ub7c9\uc744 \uae30\uc5b4 \uccb4\uc778\uc9c0\uc640 \uac19\uc774 \ubc14\uafb8\uc5b4 \ud2b9\uc720\uc758 \ube44\ube0c\ub77c\ud1a0 \ucc3d\ubc95\uc744 \ud655\ub9bd\ud558\uc600\ub2e4\uace0 \uc9c0\uc801\ud558\uc600\ub2e4. \uadf8\ub7ec\ub098 \u300aCD\uc800\ub110\u300b\uc740 \uadf8\ub140\uac00 \ub2f9\uc2dc \uc218\ub9ce\uc740 \uc544\uc774\ub3cc \uac00\uc218 \uc911\uc5d0\uc11c\ub3c4 \ucd5c\uace0\uc758 \uac00\ucc3d\ub825\uacfc \ud3c9\uac00\ub97c \ubc1b\uc558\ub2e4\uace0 \ud638\ud3c9\ud588\ub2e4. \ub610\ud55c 1989\ub144 \uc790\uc0b4\ubbf8\uc218 \uc0ac\uac74 \uc774\ud6c4 \ubcf5\uadc0\ud558\uace0 \uc120\ubcf4\uc778 \uc74c\ubc18 \u300aUNBALANCE+BALANCE\u300b\uc5d0\uc11c\ub294 \uadf8\ub140\uac00 \uc5ed\uc2dc \ub178\ub798\ub97c \uc798 \ubd80\ub974\ub294 \uc0ac\ub78c\uc774\uc5c8\ub2e4\ub294 \uac83\uc744 \ub2e4\uc2dc \uc778\uc2dd\uc2dc\ucf1c\uc92c\ub2e4\uace0 \ud3c9\ud588\ub2e4. \uac19\uc740 \uc7a1\uc9c0\uc758 \ud3c9\ub860\uac00 \uace0\ud1a0 \uc720\ud0a4\ud788\ub85c \uc5ed\uc2dc \uc11c\uc591 \uc74c\uc545\uc801\uc778 \uc694\uc18c \uc18d\uc5d0\uc11c \uc77c\ubcf8\uc5b4 \uac00\uc0ac\ub97c \uc774\ub807\uac8c \ub2e4\uc591\ud558\uba74\uc11c\ub3c4 \uc544\ub984\ub2f5\uac8c \uc6b8\ub9ac\uac8c \ud558\ub294 \uac00\uc218\ub294 \uc544\ud0a4\ub098\uac00 \ub9c8\uc9c0\ub9c9\uc774\ub77c \ud558\uba70 \uae0d\uc815\uc801\uc778 \ud3c9\uac00\ub97c \ub0b4\ub838\ub2e4. \uc74c\uc545\ud3c9\ub860\uac00 \ub178\uc9c0 \uc720\ucf54\ub294 \uc74c\uc545 \uc7a1\uc9c0 \u300aWHAT'S IN?\u300b\uc5d0\uc11c \uadf8\ub140\uc758 \ucee4\ubc84 \uc74c\ubc18 \u300a\uac00\ud76c\u300b\ub97c \ub450\uace0 \ub514\ubc14\ub85c\uc11c\uc758 \ud65c\ub825\uc774 \uc628\ubab8\uc5d0 \ub118\uce58\ub294 \uac83 \uac19\ub2e4\uace0 \uc9c0\uc801, \uae30\ub7c9 \uc774\uc804\uc5d0 \uc7ac\ub2a5\uc774 \ub118\uce58\ub294 \uac00\uc218\ub85c \ud3c9\ud558\uc600\ub2e4. \uc74c\uc545 \uc7a1\uc9c0 \u300a\ubba4\uc9c1 \ub9e4\uac70\uc9c4\u300b\uc758 \uce7c\ub7fc\ub2c8\uc2a4\ud2b8 \ud638\ubcf4 \ub2e4\uc774\uc790\ubd80\ub85c\ub294 \uadf8\ub140\ub97c \ub450\uace0 \uc601\ud63c\uc758 \uc2ec\uc5f0\uc5d0\uc11c \uad49\uc7a5\ud55c \ud761\uc785\ub825\uc73c\ub85c \ub4e3\ub294 \uc0ac\ub78c\uc744 \ub04c\uc5b4\ub2f9\uae30\uac8c \ud55c\ub2e4\uace0 \ub9d0\ud588\ub2e4. \ub610\ud55c \uc774\ub9c8\ubaa8\ud1a0 \ub3c4\uc694\uc2dc \ub294 \ub098\uce74\ubaa8\ub9ac\ub97c \uc77c\ub7ec \ud76c\ub300\uc758 \ubcf4\uceec\ub9ac\uc2a4\ud2b8\ub85c, \ub300\uc911\ub4e4\uc774 \uc54c\uace0 \uc788\ub294 \uadf8 \ud754\ud55c \uc0ac\uc2e4\uc744 \uc74c\ubc18\uc5d0\uc11c\ub294 \uae30\ub300 \uc774\uc0c1\uc73c\ub85c \ub04c\uc5b4\uc62c\ub9ac\ub294 \ub2a5\ub825\uc744 \uac16\ucd94\uc5c8\ub2e4\uace0 \ube44\ud3c9\ud558\uc600\ub2e4. MusicVoice\uc758 \ud5e4\uc774\ud0a4\uce58 \ucf04\uc9c0(\u5e73\u5409\u8ce2\u6cbb)\ub294 \ub098\uce74\ubaa8\ub9ac\uac00 \u2018\ub178\ub798\uc758 \uad6c\uc131\ub825\u2019\uc774 \uc788\uc5b4 \ub9c8\uc74c\uc744 \uc6b8\ub9ac\ub294 \uc124\ub4dd\ub825\uc73c\ub85c \uc74c\uc545\uc744 \ub4e3\ub294 \uc0ac\ub78c\ub4e4\uc5d0\uac8c \ub354 \uae4a\uc740 \uac10\uc815\uc744 \ud638\uc18c\ub825\uc788\uac8c \uc804\ub2ec\ud558\uace0, \uace1\uc758 \uc808\uc815\uc5d0\ub294 \u2018\uac10\uc815\ud45c\ud604\uacfc \uc644\uae09\uc870\uc808\uc758 \ubc30\ubd84\u2019\uc774 \uba85\ud655\ud558\uc5ec \uc801\uc808\ud55c \ubd80\ubd84\uacfc \ud0c0\uc774\ubc0d\uc5d0 \uccad\uc911\uc744 \uaf3c\uc9dd \ubabb\ud558\uac8c \ub9e4\ub8cc\uc2dc\ud0a8\ub2e4\uace0 \ud638\ud3c9\ud558\uc600\ub2e4.", "sentence_answer": "\ub610\ud55c \uc774\ub9c8\ubaa8\ud1a0 \ub3c4\uc694\uc2dc \ub294 \ub098\uce74\ubaa8\ub9ac\ub97c \uc77c\ub7ec \ud76c\ub300\uc758 \ubcf4\uceec\ub9ac\uc2a4\ud2b8\ub85c, \ub300\uc911\ub4e4\uc774 \uc54c\uace0 \uc788\ub294 \uadf8 \ud754\ud55c \uc0ac\uc2e4\uc744 \uc74c\ubc18\uc5d0\uc11c\ub294 \uae30\ub300 \uc774\uc0c1\uc73c\ub85c \ub04c\uc5b4\uc62c\ub9ac\ub294 \ub2a5\ub825\uc744 \uac16\ucd94\uc5c8\ub2e4\uace0 \ube44\ud3c9\ud558\uc600\ub2e4.", "paragraph_id": "6533918-27-0", "paragraph_question": "question: \ub098\uce74\ubaa8\ub9ac \uc544\ud0a4\ub098\ub97c \ud76c\ub300\uc758 \ubcf4\uceec\ub9ac\uc2a4\ud2b8\uc774\uba70 \ub300\uc911\ub4e4\uc774 \uc54c\uace0 \uc788\ub294 \ud754\ud55c \uc0ac\uc2e4\uc744 \uc74c\ubc18\uc5d0\uc11c \uae30\ub300 \uc774\uc0c1\uc73c\ub85c \ub04c\uc5b4\uc62c\ub9b0\ub2e4\uace0 \ub9d0\ud55c \uc0ac\ub78c\uc740 \ub204\uad6c\uc77c\uae4c?, context: \uc74c\uc545 \ub77c\uc774\ud130\uc778 \ub9c8\uce74\uc774\ub178 \ubaa8\ud1a0\ud788\ub85c\ub294 \ub098\uce74\ubaa8\ub9ac \uc544\ud0a4\ub098\uc758 \ucd5c\ub300 \ud2b9\uc9d5\uc740 \ubc14\ub85c \u201c\ub178\uc120\u201d\uc744 \uc815\ud558\uc9c0 \uc54a\uc558\ub2e4\ub294 \uac83\uc73c\ub85c\uc11c \uac00\ucc3d\ub825\uc758 \uc5ed\ub7c9\ubcf4\ub2e4\ub294 \ud48d\ubd80\ud55c \uac10\uc815\ud45c\ud604\uc73c\ub85c \uc131\ub7c9\uc744 \uae30\uc5b4 \uccb4\uc778\uc9c0\uc640 \uac19\uc774 \ubc14\uafb8\uc5b4 \ud2b9\uc720\uc758 \ube44\ube0c\ub77c\ud1a0 \ucc3d\ubc95\uc744 \ud655\ub9bd\ud558\uc600\ub2e4\uace0 \uc9c0\uc801\ud558\uc600\ub2e4. \uadf8\ub7ec\ub098 \u300aCD\uc800\ub110\u300b\uc740 \uadf8\ub140\uac00 \ub2f9\uc2dc \uc218\ub9ce\uc740 \uc544\uc774\ub3cc \uac00\uc218 \uc911\uc5d0\uc11c\ub3c4 \ucd5c\uace0\uc758 \uac00\ucc3d\ub825\uacfc \ud3c9\uac00\ub97c \ubc1b\uc558\ub2e4\uace0 \ud638\ud3c9\ud588\ub2e4. \ub610\ud55c 1989\ub144 \uc790\uc0b4\ubbf8\uc218 \uc0ac\uac74 \uc774\ud6c4 \ubcf5\uadc0\ud558\uace0 \uc120\ubcf4\uc778 \uc74c\ubc18 \u300aUNBALANCE+BALANCE\u300b\uc5d0\uc11c\ub294 \uadf8\ub140\uac00 \uc5ed\uc2dc \ub178\ub798\ub97c \uc798 \ubd80\ub974\ub294 \uc0ac\ub78c\uc774\uc5c8\ub2e4\ub294 \uac83\uc744 \ub2e4\uc2dc \uc778\uc2dd\uc2dc\ucf1c\uc92c\ub2e4\uace0 \ud3c9\ud588\ub2e4. \uac19\uc740 \uc7a1\uc9c0\uc758 \ud3c9\ub860\uac00 \uace0\ud1a0 \uc720\ud0a4\ud788\ub85c \uc5ed\uc2dc \uc11c\uc591 \uc74c\uc545\uc801\uc778 \uc694\uc18c \uc18d\uc5d0\uc11c \uc77c\ubcf8\uc5b4 \uac00\uc0ac\ub97c \uc774\ub807\uac8c \ub2e4\uc591\ud558\uba74\uc11c\ub3c4 \uc544\ub984\ub2f5\uac8c \uc6b8\ub9ac\uac8c \ud558\ub294 \uac00\uc218\ub294 \uc544\ud0a4\ub098\uac00 \ub9c8\uc9c0\ub9c9\uc774\ub77c \ud558\uba70 \uae0d\uc815\uc801\uc778 \ud3c9\uac00\ub97c \ub0b4\ub838\ub2e4. \uc74c\uc545\ud3c9\ub860\uac00 \ub178\uc9c0 \uc720\ucf54\ub294 \uc74c\uc545 \uc7a1\uc9c0 \u300aWHAT'S IN?\u300b\uc5d0\uc11c \uadf8\ub140\uc758 \ucee4\ubc84 \uc74c\ubc18 \u300a\uac00\ud76c\u300b\ub97c \ub450\uace0 \ub514\ubc14\ub85c\uc11c\uc758 \ud65c\ub825\uc774 \uc628\ubab8\uc5d0 \ub118\uce58\ub294 \uac83 \uac19\ub2e4\uace0 \uc9c0\uc801, \uae30\ub7c9 \uc774\uc804\uc5d0 \uc7ac\ub2a5\uc774 \ub118\uce58\ub294 \uac00\uc218\ub85c \ud3c9\ud558\uc600\ub2e4. \uc74c\uc545 \uc7a1\uc9c0 \u300a\ubba4\uc9c1 \ub9e4\uac70\uc9c4\u300b\uc758 \uce7c\ub7fc\ub2c8\uc2a4\ud2b8 \ud638\ubcf4 \ub2e4\uc774\uc790\ubd80\ub85c\ub294 \uadf8\ub140\ub97c \ub450\uace0 \uc601\ud63c\uc758 \uc2ec\uc5f0\uc5d0\uc11c \uad49\uc7a5\ud55c \ud761\uc785\ub825\uc73c\ub85c \ub4e3\ub294 \uc0ac\ub78c\uc744 \ub04c\uc5b4\ub2f9\uae30\uac8c \ud55c\ub2e4\uace0 \ub9d0\ud588\ub2e4. \ub610\ud55c \uc774\ub9c8\ubaa8\ud1a0 \ub3c4\uc694\uc2dc\ub294 \ub098\uce74\ubaa8\ub9ac\ub97c \uc77c\ub7ec \ud76c\ub300\uc758 \ubcf4\uceec\ub9ac\uc2a4\ud2b8\ub85c, \ub300\uc911\ub4e4\uc774 \uc54c\uace0 \uc788\ub294 \uadf8 \ud754\ud55c \uc0ac\uc2e4\uc744 \uc74c\ubc18\uc5d0\uc11c\ub294 \uae30\ub300 \uc774\uc0c1\uc73c\ub85c \ub04c\uc5b4\uc62c\ub9ac\ub294 \ub2a5\ub825\uc744 \uac16\ucd94\uc5c8\ub2e4\uace0 \ube44\ud3c9\ud558\uc600\ub2e4. MusicVoice\uc758 \ud5e4\uc774\ud0a4\uce58 \ucf04\uc9c0(\u5e73\u5409\u8ce2\u6cbb)\ub294 \ub098\uce74\ubaa8\ub9ac\uac00 \u2018\ub178\ub798\uc758 \uad6c\uc131\ub825\u2019\uc774 \uc788\uc5b4 \ub9c8\uc74c\uc744 \uc6b8\ub9ac\ub294 \uc124\ub4dd\ub825\uc73c\ub85c \uc74c\uc545\uc744 \ub4e3\ub294 \uc0ac\ub78c\ub4e4\uc5d0\uac8c \ub354 \uae4a\uc740 \uac10\uc815\uc744 \ud638\uc18c\ub825\uc788\uac8c \uc804\ub2ec\ud558\uace0, \uace1\uc758 \uc808\uc815\uc5d0\ub294 \u2018\uac10\uc815\ud45c\ud604\uacfc \uc644\uae09\uc870\uc808\uc758 \ubc30\ubd84\u2019\uc774 \uba85\ud655\ud558\uc5ec \uc801\uc808\ud55c \ubd80\ubd84\uacfc \ud0c0\uc774\ubc0d\uc5d0 \uccad\uc911\uc744 \uaf3c\uc9dd \ubabb\ud558\uac8c \ub9e4\ub8cc\uc2dc\ud0a8\ub2e4\uace0 \ud638\ud3c9\ud558\uc600\ub2e4."} {"question": "2017\ub144 1\uc6d4 \uc804\uad6d \ub3d9\uacc4\uccb4\uc804\uc5d0\uc11c \uae40\uc544\ub791\uc774 \ub178\ub3c4\ud76c\uc640 \ud568\uaed8 \ub118\uc5b4\uc9c0\uba70 \uc5bc\uad74\uc774 \ucc22\uae30\ub294 \ubd80\uc0c1\uc744 \uc785\uc740 \ucf54\uc2a4\ub294 \uba87 \ubbf8\ud130\uc778\uac00", "paragraph": "\uae40\uc544\ub791\uc740 2016\ub144 4\uc6d4\uc5d0 \uc5f4\ub9b0 2016-2017 \uc2dc\uc98c \uc1fc\ud2b8\ud2b8\ub799 \uad6d\uac00\ub300\ud45c \uc120\ubc1c\ub300\ud68c\uc5d0\uc11c \ud0c8\ub77d\ud558\uc5ec 3\ub144 \ub9cc\uc5d0 \ud0dc\uadf9 \ub9c8\ud06c\ub97c \ub0b4\ub824\ub193\uc544\uc57c \ud588\ub2e4. \ubc1c\ubaa9\uacfc \ud5c8\ub9ac \ubd80\uc0c1\uc73c\ub85c \uc778\ud558\uc5ec \ud6c8\ub828\uc744 \uc81c\ub300\ub85c \ud560 \uc218 \uc5c6\uc5c8\uace0 \uadf8\uac83\uc740 \uace7 \uae30\ub7c9 \uc800\ud558\ub85c \uc774\uc5b4\uc84c\ub2e4\ub294 \ud3c9\uac00\uac00 \uc788\uc5c8\ub2e4. \ub610\ud55c 2017\ub144 1\uc6d4 19\uc77c \uc804\uad6d \ub3d9\uacc4\uccb4\uc804 \uc5ec\uc790 \ub300\ud559\ubd80 3000\ubbf8\ud130 \uacbd\uae30\uc5d0 \ucd9c\uc804\ud55c \uae40\uc544\ub791\uc740 \uc0c1\ub300 \uc120\uc218\uc758 \uc2a4\ucf00\uc774\ud2b8 \ub0a0\uc5d0 \uc5bc\uad74\uc774 \ucc22\uae30\ub294 \uc0c1\ucc98\ub97c \uc785\uc5c8\ub2e4. \uc544\uc6c3\ucf54\uc2a4\ub85c \ucd94\uc6d4\ud558\ub358 \uc911 \ub178\ub3c4\ud76c\uc640 \ud568\uaed8 \ub118\uc5b4\uc9c0\uba74\uc11c \uc67c\ucabd \ubea8\uc744 \ubca0\uc778 \uac83\uc774\uc5c8\ub2e4. \ud558\uc9c0\ub9cc \uae40\uc544\ub791\uc740 \ubd88\uacfc \ud55c \ub2ec \ud6c4 2017\ub144 2\uc6d4 \uce74\uc790\ud750\uc2a4\ud0c4 \uc54c\ub9c8\ud2f0\uc5d0\uc11c \uc5f4\ub9b0 \uc81c28\ud68c \ub3d9\uacc4 \uc720\ub2c8\ubc84\uc2dc\uc544\ub4dc \uc5ec\uc790 \uc1fc\ud2b8\ud2b8\ub799 1500\ubbf8\ud130 \uacbd\uae30\uc5d0 \ucd9c\uc804\ud558\uc5ec \uc740\uba54\ub2ec\uc744 \ub544\uc73c\uba70, 3000\ubbf8\ud130 \uacc4\uc8fc\uc5d0\uc11c\ub3c4 \uae08\uba54\ub2ec\uc744 \ucc28\uc9c0\ud558\uc600\ub2e4. \ubd80\uc0c1\uc758 \ud2b8\ub77c\uc6b0\ub9c8\ub97c \uc5b4\ub290 \uc815\ub3c4 \uadf9\ubcf5\ud55c \uac83\uc73c\ub85c \ubcf4\uc600\ub2e4. \ub610\ud55c \uae40\uc544\ub791\uc740 \ud3c9\ucc3d \uc62c\ub9bc\ud53d\uc774 \ub05d\ub09c \ud6c4 \uc5b4\ub290 \uc778\ud130\ubdf0\uc5d0\uc11c, \uc0c1\ucc98\ub97c \uc785\uc5c8\uc744 \ub54c \ub2e4\ub978 \ubd84\ub4e4\uc740 \uc5ec\uc790\ub85c\uc11c \uc5bc\uad74\uc758 \uc0c1\ucc98\ub97c \uac71\uc815\ud588\uc9c0\ub9cc \uc790\uc2e0\uc740 \uc6d0\ub798 \ubd80\uc0c1\uc774 \uc788\uc5c8\ub358 \ubc1c\ubaa9 \ub610\ub294 \ud5c8\ub9ac\uac00 \uc544\ud504\uc9c0 \uc54a\uc744\uae4c \uac71\uc815\uc744 \ud588\ub2e4\uba70 \uc790\uc2e0\uc774 \uc120\uc218\ub85c\uc11c \ub9c8\uc74c\uac00\uc9d0\uc744 \ub2e4\uc2dc \ub3cc\uc544\ubcf4\ub294 \uacc4\uae30\uac00 \ub418\uc5c8\ub2e4\uace0 \ub9d0\ud558\uc600\ub2e4.", "answer": "3000\ubbf8\ud130", "sentence": "\ub610\ud55c 2017\ub144 1\uc6d4 19\uc77c \uc804\uad6d \ub3d9\uacc4\uccb4\uc804 \uc5ec\uc790 \ub300\ud559\ubd80 3000\ubbf8\ud130 \uacbd\uae30\uc5d0 \ucd9c\uc804\ud55c \uae40\uc544\ub791\uc740 \uc0c1\ub300 \uc120\uc218\uc758 \uc2a4\ucf00\uc774\ud2b8 \ub0a0\uc5d0 \uc5bc\uad74\uc774 \ucc22\uae30\ub294 \uc0c1\ucc98\ub97c \uc785\uc5c8\ub2e4.", "paragraph_sentence": "\uae40\uc544\ub791\uc740 2016\ub144 4\uc6d4\uc5d0 \uc5f4\ub9b0 2016-2017 \uc2dc\uc98c \uc1fc\ud2b8\ud2b8\ub799 \uad6d\uac00\ub300\ud45c \uc120\ubc1c\ub300\ud68c\uc5d0\uc11c \ud0c8\ub77d\ud558\uc5ec 3\ub144 \ub9cc\uc5d0 \ud0dc\uadf9 \ub9c8\ud06c\ub97c \ub0b4\ub824\ub193\uc544\uc57c \ud588\ub2e4. \ubc1c\ubaa9\uacfc \ud5c8\ub9ac \ubd80\uc0c1\uc73c\ub85c \uc778\ud558\uc5ec \ud6c8\ub828\uc744 \uc81c\ub300\ub85c \ud560 \uc218 \uc5c6\uc5c8\uace0 \uadf8\uac83\uc740 \uace7 \uae30\ub7c9 \uc800\ud558\ub85c \uc774\uc5b4\uc84c\ub2e4\ub294 \ud3c9\uac00\uac00 \uc788\uc5c8\ub2e4. \ub610\ud55c 2017\ub144 1\uc6d4 19\uc77c \uc804\uad6d \ub3d9\uacc4\uccb4\uc804 \uc5ec\uc790 \ub300\ud559\ubd80 3000\ubbf8\ud130 \uacbd\uae30\uc5d0 \ucd9c\uc804\ud55c \uae40\uc544\ub791\uc740 \uc0c1\ub300 \uc120\uc218\uc758 \uc2a4\ucf00\uc774\ud2b8 \ub0a0\uc5d0 \uc5bc\uad74\uc774 \ucc22\uae30\ub294 \uc0c1\ucc98\ub97c \uc785\uc5c8\ub2e4. \uc544\uc6c3\ucf54\uc2a4\ub85c \ucd94\uc6d4\ud558\ub358 \uc911 \ub178\ub3c4\ud76c\uc640 \ud568\uaed8 \ub118\uc5b4\uc9c0\uba74\uc11c \uc67c\ucabd \ubea8\uc744 \ubca0\uc778 \uac83\uc774\uc5c8\ub2e4. \ud558\uc9c0\ub9cc \uae40\uc544\ub791\uc740 \ubd88\uacfc \ud55c \ub2ec \ud6c4 2017\ub144 2\uc6d4 \uce74\uc790\ud750\uc2a4\ud0c4 \uc54c\ub9c8\ud2f0\uc5d0\uc11c \uc5f4\ub9b0 \uc81c28\ud68c \ub3d9\uacc4 \uc720\ub2c8\ubc84\uc2dc\uc544\ub4dc \uc5ec\uc790 \uc1fc\ud2b8\ud2b8\ub799 1500\ubbf8\ud130 \uacbd\uae30\uc5d0 \ucd9c\uc804\ud558\uc5ec \uc740\uba54\ub2ec\uc744 \ub544\uc73c\uba70, 3000\ubbf8\ud130 \uacc4\uc8fc\uc5d0\uc11c\ub3c4 \uae08\uba54\ub2ec\uc744 \ucc28\uc9c0\ud558\uc600\ub2e4. \ubd80\uc0c1\uc758 \ud2b8\ub77c\uc6b0\ub9c8\ub97c \uc5b4\ub290 \uc815\ub3c4 \uadf9\ubcf5\ud55c \uac83\uc73c\ub85c \ubcf4\uc600\ub2e4. \ub610\ud55c \uae40\uc544\ub791\uc740 \ud3c9\ucc3d \uc62c\ub9bc\ud53d\uc774 \ub05d\ub09c \ud6c4 \uc5b4\ub290 \uc778\ud130\ubdf0\uc5d0\uc11c, \uc0c1\ucc98\ub97c \uc785\uc5c8\uc744 \ub54c \ub2e4\ub978 \ubd84\ub4e4\uc740 \uc5ec\uc790\ub85c\uc11c \uc5bc\uad74\uc758 \uc0c1\ucc98\ub97c \uac71\uc815\ud588\uc9c0\ub9cc \uc790\uc2e0\uc740 \uc6d0\ub798 \ubd80\uc0c1\uc774 \uc788\uc5c8\ub358 \ubc1c\ubaa9 \ub610\ub294 \ud5c8\ub9ac\uac00 \uc544\ud504\uc9c0 \uc54a\uc744\uae4c \uac71\uc815\uc744 \ud588\ub2e4\uba70 \uc790\uc2e0\uc774 \uc120\uc218\ub85c\uc11c \ub9c8\uc74c\uac00\uc9d0\uc744 \ub2e4\uc2dc \ub3cc\uc544\ubcf4\ub294 \uacc4\uae30\uac00 \ub418\uc5c8\ub2e4\uace0 \ub9d0\ud558\uc600\ub2e4.", "paragraph_answer": "\uae40\uc544\ub791\uc740 2016\ub144 4\uc6d4\uc5d0 \uc5f4\ub9b0 2016-2017 \uc2dc\uc98c \uc1fc\ud2b8\ud2b8\ub799 \uad6d\uac00\ub300\ud45c \uc120\ubc1c\ub300\ud68c\uc5d0\uc11c \ud0c8\ub77d\ud558\uc5ec 3\ub144 \ub9cc\uc5d0 \ud0dc\uadf9 \ub9c8\ud06c\ub97c \ub0b4\ub824\ub193\uc544\uc57c \ud588\ub2e4. \ubc1c\ubaa9\uacfc \ud5c8\ub9ac \ubd80\uc0c1\uc73c\ub85c \uc778\ud558\uc5ec \ud6c8\ub828\uc744 \uc81c\ub300\ub85c \ud560 \uc218 \uc5c6\uc5c8\uace0 \uadf8\uac83\uc740 \uace7 \uae30\ub7c9 \uc800\ud558\ub85c \uc774\uc5b4\uc84c\ub2e4\ub294 \ud3c9\uac00\uac00 \uc788\uc5c8\ub2e4. \ub610\ud55c 2017\ub144 1\uc6d4 19\uc77c \uc804\uad6d \ub3d9\uacc4\uccb4\uc804 \uc5ec\uc790 \ub300\ud559\ubd80 3000\ubbf8\ud130 \uacbd\uae30\uc5d0 \ucd9c\uc804\ud55c \uae40\uc544\ub791\uc740 \uc0c1\ub300 \uc120\uc218\uc758 \uc2a4\ucf00\uc774\ud2b8 \ub0a0\uc5d0 \uc5bc\uad74\uc774 \ucc22\uae30\ub294 \uc0c1\ucc98\ub97c \uc785\uc5c8\ub2e4. \uc544\uc6c3\ucf54\uc2a4\ub85c \ucd94\uc6d4\ud558\ub358 \uc911 \ub178\ub3c4\ud76c\uc640 \ud568\uaed8 \ub118\uc5b4\uc9c0\uba74\uc11c \uc67c\ucabd \ubea8\uc744 \ubca0\uc778 \uac83\uc774\uc5c8\ub2e4. \ud558\uc9c0\ub9cc \uae40\uc544\ub791\uc740 \ubd88\uacfc \ud55c \ub2ec \ud6c4 2017\ub144 2\uc6d4 \uce74\uc790\ud750\uc2a4\ud0c4 \uc54c\ub9c8\ud2f0\uc5d0\uc11c \uc5f4\ub9b0 \uc81c28\ud68c \ub3d9\uacc4 \uc720\ub2c8\ubc84\uc2dc\uc544\ub4dc \uc5ec\uc790 \uc1fc\ud2b8\ud2b8\ub799 1500\ubbf8\ud130 \uacbd\uae30\uc5d0 \ucd9c\uc804\ud558\uc5ec \uc740\uba54\ub2ec\uc744 \ub544\uc73c\uba70, 3000\ubbf8\ud130 \uacc4\uc8fc\uc5d0\uc11c\ub3c4 \uae08\uba54\ub2ec\uc744 \ucc28\uc9c0\ud558\uc600\ub2e4. \ubd80\uc0c1\uc758 \ud2b8\ub77c\uc6b0\ub9c8\ub97c \uc5b4\ub290 \uc815\ub3c4 \uadf9\ubcf5\ud55c \uac83\uc73c\ub85c \ubcf4\uc600\ub2e4. \ub610\ud55c \uae40\uc544\ub791\uc740 \ud3c9\ucc3d \uc62c\ub9bc\ud53d\uc774 \ub05d\ub09c \ud6c4 \uc5b4\ub290 \uc778\ud130\ubdf0\uc5d0\uc11c, \uc0c1\ucc98\ub97c \uc785\uc5c8\uc744 \ub54c \ub2e4\ub978 \ubd84\ub4e4\uc740 \uc5ec\uc790\ub85c\uc11c \uc5bc\uad74\uc758 \uc0c1\ucc98\ub97c \uac71\uc815\ud588\uc9c0\ub9cc \uc790\uc2e0\uc740 \uc6d0\ub798 \ubd80\uc0c1\uc774 \uc788\uc5c8\ub358 \ubc1c\ubaa9 \ub610\ub294 \ud5c8\ub9ac\uac00 \uc544\ud504\uc9c0 \uc54a\uc744\uae4c \uac71\uc815\uc744 \ud588\ub2e4\uba70 \uc790\uc2e0\uc774 \uc120\uc218\ub85c\uc11c \ub9c8\uc74c\uac00\uc9d0\uc744 \ub2e4\uc2dc \ub3cc\uc544\ubcf4\ub294 \uacc4\uae30\uac00 \ub418\uc5c8\ub2e4\uace0 \ub9d0\ud558\uc600\ub2e4.", "sentence_answer": "\ub610\ud55c 2017\ub144 1\uc6d4 19\uc77c \uc804\uad6d \ub3d9\uacc4\uccb4\uc804 \uc5ec\uc790 \ub300\ud559\ubd80 3000\ubbf8\ud130 \uacbd\uae30\uc5d0 \ucd9c\uc804\ud55c \uae40\uc544\ub791\uc740 \uc0c1\ub300 \uc120\uc218\uc758 \uc2a4\ucf00\uc774\ud2b8 \ub0a0\uc5d0 \uc5bc\uad74\uc774 \ucc22\uae30\ub294 \uc0c1\ucc98\ub97c \uc785\uc5c8\ub2e4.", "paragraph_id": "5998192-4-1", "paragraph_question": "question: 2017\ub144 1\uc6d4 \uc804\uad6d \ub3d9\uacc4\uccb4\uc804\uc5d0\uc11c \uae40\uc544\ub791\uc774 \ub178\ub3c4\ud76c\uc640 \ud568\uaed8 \ub118\uc5b4\uc9c0\uba70 \uc5bc\uad74\uc774 \ucc22\uae30\ub294 \ubd80\uc0c1\uc744 \uc785\uc740 \ucf54\uc2a4\ub294 \uba87 \ubbf8\ud130\uc778\uac00, context: \uae40\uc544\ub791\uc740 2016\ub144 4\uc6d4\uc5d0 \uc5f4\ub9b0 2016-2017 \uc2dc\uc98c \uc1fc\ud2b8\ud2b8\ub799 \uad6d\uac00\ub300\ud45c \uc120\ubc1c\ub300\ud68c\uc5d0\uc11c \ud0c8\ub77d\ud558\uc5ec 3\ub144 \ub9cc\uc5d0 \ud0dc\uadf9 \ub9c8\ud06c\ub97c \ub0b4\ub824\ub193\uc544\uc57c \ud588\ub2e4. \ubc1c\ubaa9\uacfc \ud5c8\ub9ac \ubd80\uc0c1\uc73c\ub85c \uc778\ud558\uc5ec \ud6c8\ub828\uc744 \uc81c\ub300\ub85c \ud560 \uc218 \uc5c6\uc5c8\uace0 \uadf8\uac83\uc740 \uace7 \uae30\ub7c9 \uc800\ud558\ub85c \uc774\uc5b4\uc84c\ub2e4\ub294 \ud3c9\uac00\uac00 \uc788\uc5c8\ub2e4. \ub610\ud55c 2017\ub144 1\uc6d4 19\uc77c \uc804\uad6d \ub3d9\uacc4\uccb4\uc804 \uc5ec\uc790 \ub300\ud559\ubd80 3000\ubbf8\ud130 \uacbd\uae30\uc5d0 \ucd9c\uc804\ud55c \uae40\uc544\ub791\uc740 \uc0c1\ub300 \uc120\uc218\uc758 \uc2a4\ucf00\uc774\ud2b8 \ub0a0\uc5d0 \uc5bc\uad74\uc774 \ucc22\uae30\ub294 \uc0c1\ucc98\ub97c \uc785\uc5c8\ub2e4. \uc544\uc6c3\ucf54\uc2a4\ub85c \ucd94\uc6d4\ud558\ub358 \uc911 \ub178\ub3c4\ud76c\uc640 \ud568\uaed8 \ub118\uc5b4\uc9c0\uba74\uc11c \uc67c\ucabd \ubea8\uc744 \ubca0\uc778 \uac83\uc774\uc5c8\ub2e4. \ud558\uc9c0\ub9cc \uae40\uc544\ub791\uc740 \ubd88\uacfc \ud55c \ub2ec \ud6c4 2017\ub144 2\uc6d4 \uce74\uc790\ud750\uc2a4\ud0c4 \uc54c\ub9c8\ud2f0\uc5d0\uc11c \uc5f4\ub9b0 \uc81c28\ud68c \ub3d9\uacc4 \uc720\ub2c8\ubc84\uc2dc\uc544\ub4dc \uc5ec\uc790 \uc1fc\ud2b8\ud2b8\ub799 1500\ubbf8\ud130 \uacbd\uae30\uc5d0 \ucd9c\uc804\ud558\uc5ec \uc740\uba54\ub2ec\uc744 \ub544\uc73c\uba70, 3000\ubbf8\ud130 \uacc4\uc8fc\uc5d0\uc11c\ub3c4 \uae08\uba54\ub2ec\uc744 \ucc28\uc9c0\ud558\uc600\ub2e4. \ubd80\uc0c1\uc758 \ud2b8\ub77c\uc6b0\ub9c8\ub97c \uc5b4\ub290 \uc815\ub3c4 \uadf9\ubcf5\ud55c \uac83\uc73c\ub85c \ubcf4\uc600\ub2e4. \ub610\ud55c \uae40\uc544\ub791\uc740 \ud3c9\ucc3d \uc62c\ub9bc\ud53d\uc774 \ub05d\ub09c \ud6c4 \uc5b4\ub290 \uc778\ud130\ubdf0\uc5d0\uc11c, \uc0c1\ucc98\ub97c \uc785\uc5c8\uc744 \ub54c \ub2e4\ub978 \ubd84\ub4e4\uc740 \uc5ec\uc790\ub85c\uc11c \uc5bc\uad74\uc758 \uc0c1\ucc98\ub97c \uac71\uc815\ud588\uc9c0\ub9cc \uc790\uc2e0\uc740 \uc6d0\ub798 \ubd80\uc0c1\uc774 \uc788\uc5c8\ub358 \ubc1c\ubaa9 \ub610\ub294 \ud5c8\ub9ac\uac00 \uc544\ud504\uc9c0 \uc54a\uc744\uae4c \uac71\uc815\uc744 \ud588\ub2e4\uba70 \uc790\uc2e0\uc774 \uc120\uc218\ub85c\uc11c \ub9c8\uc74c\uac00\uc9d0\uc744 \ub2e4\uc2dc \ub3cc\uc544\ubcf4\ub294 \uacc4\uae30\uac00 \ub418\uc5c8\ub2e4\uace0 \ub9d0\ud558\uc600\ub2e4."} {"question": "\ub098\ub77c \uc2dc\ub300 \ucd08\uae30 \uc815\uad8c\uc744 \uc950\uace0 \ud6c4\uc9c0\uc640\ub77c \uc528 \ubc1c\uc804\uc758 \uae30\ucd08\ub97c \ub2e6\uc740 \uc774\ub294 \ub204\uad6c\uc778\uac00?", "paragraph": "\ub098\ub77c \uc2dc\ub300 \ucd08\uae30\uc5d0\ub294 \ub098\uce74\ud1a0\ubbf8\ub178 \uac00\ub9c8\ud0c0\ub9ac\uc758 \uc544\ub4e4 \ud6c4\uc9c0\uc640\ub77c \ud6c4\ud788\ud1a0\uac00 \uc815\uad8c\uc744 \uc950\uace0 \uc728\ub839\uc81c\ub3c4\uc758 \ud655\ub9bd\uc5d0 \uc9c4\ub825\ud558\ub294 \ub3d9\uc2dc\uc5d0 \ud669\uc2e4\uacfc \uc778\ucc99\uad00\uacc4\ub97c \ub9fa\uc73c\uba70 \ud6c4\uc9c0\uc640\ub77c \uc528 \ubc1c\uc804\uc758 \uae30\ucd08\ub97c \ub2e6\uc558\ub2e4. \ud6c4\ud788\ud1a0 \uc0ac\ud6c4\uc5d0\ub294 \ub2e4\ucf00\uce58 \ud669\uc790\uc758 \uc544\ub4e4\ub85c \ub374\ubb34 \ucc9c\ud669\uc758 \uc190\uc790\uc778 \ub098\uac00\uc57c \uc655\uc774 \uc6b0\ub2e4\uc774\uc9c4\uc774 \ub418\uc5b4 \uc815\uad8c\uc744 \ub2f4\ub2f9\ud558\uc600\ub2e4. \uc804\ub300\ubd80\ud130 \ub204\uc801\ub418\uc5b4 \uc628 \uacfc\uc911\ud55c \ubd80\ub2f4\uc744 \uacac\ub514\uc9c0 \ubabb\ud55c \ub18d\ubbfc\ub4e4\uc758 \uc720\ub791\uacfc \ub3c4\ub9dd\uc774 \uae09\uaca9\ud788 \ub298\uc5b4\ub098 \uc0ac\ud68c \ubd88\uc548\uc774 \ud45c\uba74\ud654\ub418\uc5c8\uae30 \ub54c\ubb38\uc5d0, \uc815\ubd80\ub294 \uc7ac\uc6d0\ud655\ubcf4\ub97c \uc704\ud574 723\ub144(\uc694\ub85c 7\ub144)\uc5d0\ub294 \uc0bc\uc138\uc77c\uc2e0\ubc95(\u4e09\u4e16\u4e00\u8eab\u6cd5)\uc744 \uc2dc\ud589\ud558\uc5ec \uac1c\uac04\uc744 \uc7a5\ub824\ud558\uc600\ub2e4. \ubaac\ubb34 \ucc9c\ud669\uacfc \ud6c4\uc9c0\uc640\ub77c \ud6c4\ud788\ud1a0\uc758 \ub538\uc778 \ud6c4\uc9c0\uc640\ub77c \ubbf8\uc57c\ucf54(\u85e4\u539f\u5bae\u5b50) \uc0ac\uc774\uc5d0\uc11c \ud0dc\uc5b4\ub09c \uc1fc\ubb34 \ucc9c\ud669\uc774 \uc989\uc704\ud55c 724\ub144(\uc9c4\ud0a4 \uc6d0\ub144) \uc988\uc74c\ubd80\ud130 \ud6c4\ud788\ud1a0\uc758 \ub124 \uc544\ub4e4 \ubb34\uce58\ub9c8\ub85c(\u6b66\u667a\u9ebb\u5442), \ud6c4\uc0ac\uc0ac\ud0a4(\u623f\u524d), \uc6b0\ub9c8\uce74\uc774(\u5b87\u5408), \ub9c8\ub85c(\u9ebb\u5442)\uac00 \uc815\uacc4\uc5d0 \uc9c4\ucd9c\ud558\uc600\ub2e4. 729\ub144(\uc9c4\ud0a4 6\ub144), \ub098\uac00\uc57c \uc655\uc774 \uc0ac\uc2e4\uc0c1 \uc815\uacc4 \ucd5c\uace0\uc9c1\uc5d0 \ud574\ub2f9\ud558\ub294 \uc0ac\ub2e4\uc774\uc9c4\uc5d0 \ucde8\uc784\ud558\uc790, \uc774\uc5d0 \uc704\uae30\uac10\uc744 \ub290\ub080 \ud6c4\uc9c0\uc640\ub77c 4\ud615\uc81c\ub294 \u2018\uc88c\ub3c4(\u5de6\u9053)\uc5d0 \ube60\uc838 \uad6d\uc815\uc744 \uae30\uc6b8\uac8c \ud55c\ub2e4\u2019\uace0 \ucc38\uc18c\ud558\uc5ec \ub098\uac00\uc57c \uc655\uc744 \uc790\uacb0\ub85c \ubab0\uc544\ub123\uace0(\ub098\uac00\uc57c \uc655\uc758 \uc815\ubcc0), \uc815\uad8c\uc744 \uc7a5\uc545\ud588\ub2e4. \uc815\ubcc0 \uc9c1\ud6c4, \ud6c4\uc9c0\uc640\ub77c \uc528\ub294 \ud6c4\ud788\ud1a0\uc758 \ub538\uc778 \uace0\ubb18\uc2dc(\u5149\u660e\u5b50, \uace0\ubb18 \ud669\ud6c4)\ub97c \uc1fc\ubb34 \ucc9c\ud669\uc758 \ud669\ud6c4\ub85c \ucd94\ub300\ud558\ub294 \ub370 \uc131\uacf5\ud558\uc5ec, \uace0\ubb18\uc2dc\ub294 \uc2e0\ud558\ub85c\uc11c\ub294 \ucd5c\ucd08\uc758 \ud669\ud6c4\uac00 \ub418\uc5c8\ub2e4.", "answer": "\ud6c4\uc9c0\uc640\ub77c \ud6c4\ud788\ud1a0", "sentence": "\ub098\ub77c \uc2dc\ub300 \ucd08\uae30\uc5d0\ub294 \ub098\uce74\ud1a0\ubbf8\ub178 \uac00\ub9c8\ud0c0\ub9ac\uc758 \uc544\ub4e4 \ud6c4\uc9c0\uc640\ub77c \ud6c4\ud788\ud1a0 \uac00 \uc815\uad8c\uc744 \uc950\uace0 \uc728\ub839\uc81c\ub3c4\uc758 \ud655\ub9bd\uc5d0 \uc9c4\ub825\ud558\ub294 \ub3d9\uc2dc\uc5d0 \ud669\uc2e4\uacfc \uc778\ucc99\uad00\uacc4\ub97c \ub9fa\uc73c\uba70 \ud6c4\uc9c0\uc640\ub77c \uc528 \ubc1c\uc804\uc758 \uae30\ucd08\ub97c \ub2e6\uc558\ub2e4.", "paragraph_sentence": " \ub098\ub77c \uc2dc\ub300 \ucd08\uae30\uc5d0\ub294 \ub098\uce74\ud1a0\ubbf8\ub178 \uac00\ub9c8\ud0c0\ub9ac\uc758 \uc544\ub4e4 \ud6c4\uc9c0\uc640\ub77c \ud6c4\ud788\ud1a0 \uac00 \uc815\uad8c\uc744 \uc950\uace0 \uc728\ub839\uc81c\ub3c4\uc758 \ud655\ub9bd\uc5d0 \uc9c4\ub825\ud558\ub294 \ub3d9\uc2dc\uc5d0 \ud669\uc2e4\uacfc \uc778\ucc99\uad00\uacc4\ub97c \ub9fa\uc73c\uba70 \ud6c4\uc9c0\uc640\ub77c \uc528 \ubc1c\uc804\uc758 \uae30\ucd08\ub97c \ub2e6\uc558\ub2e4. \ud6c4\ud788\ud1a0 \uc0ac\ud6c4\uc5d0\ub294 \ub2e4\ucf00\uce58 \ud669\uc790\uc758 \uc544\ub4e4\ub85c \ub374\ubb34 \ucc9c\ud669\uc758 \uc190\uc790\uc778 \ub098\uac00\uc57c \uc655\uc774 \uc6b0\ub2e4\uc774\uc9c4\uc774 \ub418\uc5b4 \uc815\uad8c\uc744 \ub2f4\ub2f9\ud558\uc600\ub2e4. \uc804\ub300\ubd80\ud130 \ub204\uc801\ub418\uc5b4 \uc628 \uacfc\uc911\ud55c \ubd80\ub2f4\uc744 \uacac\ub514\uc9c0 \ubabb\ud55c \ub18d\ubbfc\ub4e4\uc758 \uc720\ub791\uacfc \ub3c4\ub9dd\uc774 \uae09\uaca9\ud788 \ub298\uc5b4\ub098 \uc0ac\ud68c \ubd88\uc548\uc774 \ud45c\uba74\ud654\ub418\uc5c8\uae30 \ub54c\ubb38\uc5d0, \uc815\ubd80\ub294 \uc7ac\uc6d0\ud655\ubcf4\ub97c \uc704\ud574 723\ub144(\uc694\ub85c 7\ub144)\uc5d0\ub294 \uc0bc\uc138\uc77c\uc2e0\ubc95(\u4e09\u4e16\u4e00\u8eab\u6cd5)\uc744 \uc2dc\ud589\ud558\uc5ec \uac1c\uac04\uc744 \uc7a5\ub824\ud558\uc600\ub2e4. \ubaac\ubb34 \ucc9c\ud669\uacfc \ud6c4\uc9c0\uc640\ub77c \ud6c4\ud788\ud1a0\uc758 \ub538\uc778 \ud6c4\uc9c0\uc640\ub77c \ubbf8\uc57c\ucf54(\u85e4\u539f\u5bae\u5b50) \uc0ac\uc774\uc5d0\uc11c \ud0dc\uc5b4\ub09c \uc1fc\ubb34 \ucc9c\ud669\uc774 \uc989\uc704\ud55c 724\ub144(\uc9c4\ud0a4 \uc6d0\ub144) \uc988\uc74c\ubd80\ud130 \ud6c4\ud788\ud1a0\uc758 \ub124 \uc544\ub4e4 \ubb34\uce58\ub9c8\ub85c(\u6b66\u667a\u9ebb\u5442), \ud6c4\uc0ac\uc0ac\ud0a4(\u623f\u524d), \uc6b0\ub9c8\uce74\uc774(\u5b87\u5408), \ub9c8\ub85c(\u9ebb\u5442)\uac00 \uc815\uacc4\uc5d0 \uc9c4\ucd9c\ud558\uc600\ub2e4. 729\ub144(\uc9c4\ud0a4 6\ub144), \ub098\uac00\uc57c \uc655\uc774 \uc0ac\uc2e4\uc0c1 \uc815\uacc4 \ucd5c\uace0\uc9c1\uc5d0 \ud574\ub2f9\ud558\ub294 \uc0ac\ub2e4\uc774\uc9c4\uc5d0 \ucde8\uc784\ud558\uc790, \uc774\uc5d0 \uc704\uae30\uac10\uc744 \ub290\ub080 \ud6c4\uc9c0\uc640\ub77c 4\ud615\uc81c\ub294 \u2018\uc88c\ub3c4(\u5de6\u9053)\uc5d0 \ube60\uc838 \uad6d\uc815\uc744 \uae30\uc6b8\uac8c \ud55c\ub2e4\u2019\uace0 \ucc38\uc18c\ud558\uc5ec \ub098\uac00\uc57c \uc655\uc744 \uc790\uacb0\ub85c \ubab0\uc544\ub123\uace0(\ub098\uac00\uc57c \uc655\uc758 \uc815\ubcc0), \uc815\uad8c\uc744 \uc7a5\uc545\ud588\ub2e4. \uc815\ubcc0 \uc9c1\ud6c4, \ud6c4\uc9c0\uc640\ub77c \uc528\ub294 \ud6c4\ud788\ud1a0\uc758 \ub538\uc778 \uace0\ubb18\uc2dc(\u5149\u660e\u5b50, \uace0\ubb18 \ud669\ud6c4)\ub97c \uc1fc\ubb34 \ucc9c\ud669\uc758 \ud669\ud6c4\ub85c \ucd94\ub300\ud558\ub294 \ub370 \uc131\uacf5\ud558\uc5ec, \uace0\ubb18\uc2dc\ub294 \uc2e0\ud558\ub85c\uc11c\ub294 \ucd5c\ucd08\uc758 \ud669\ud6c4\uac00 \ub418\uc5c8\ub2e4.", "paragraph_answer": "\ub098\ub77c \uc2dc\ub300 \ucd08\uae30\uc5d0\ub294 \ub098\uce74\ud1a0\ubbf8\ub178 \uac00\ub9c8\ud0c0\ub9ac\uc758 \uc544\ub4e4 \ud6c4\uc9c0\uc640\ub77c \ud6c4\ud788\ud1a0 \uac00 \uc815\uad8c\uc744 \uc950\uace0 \uc728\ub839\uc81c\ub3c4\uc758 \ud655\ub9bd\uc5d0 \uc9c4\ub825\ud558\ub294 \ub3d9\uc2dc\uc5d0 \ud669\uc2e4\uacfc \uc778\ucc99\uad00\uacc4\ub97c \ub9fa\uc73c\uba70 \ud6c4\uc9c0\uc640\ub77c \uc528 \ubc1c\uc804\uc758 \uae30\ucd08\ub97c \ub2e6\uc558\ub2e4. \ud6c4\ud788\ud1a0 \uc0ac\ud6c4\uc5d0\ub294 \ub2e4\ucf00\uce58 \ud669\uc790\uc758 \uc544\ub4e4\ub85c \ub374\ubb34 \ucc9c\ud669\uc758 \uc190\uc790\uc778 \ub098\uac00\uc57c \uc655\uc774 \uc6b0\ub2e4\uc774\uc9c4\uc774 \ub418\uc5b4 \uc815\uad8c\uc744 \ub2f4\ub2f9\ud558\uc600\ub2e4. \uc804\ub300\ubd80\ud130 \ub204\uc801\ub418\uc5b4 \uc628 \uacfc\uc911\ud55c \ubd80\ub2f4\uc744 \uacac\ub514\uc9c0 \ubabb\ud55c \ub18d\ubbfc\ub4e4\uc758 \uc720\ub791\uacfc \ub3c4\ub9dd\uc774 \uae09\uaca9\ud788 \ub298\uc5b4\ub098 \uc0ac\ud68c \ubd88\uc548\uc774 \ud45c\uba74\ud654\ub418\uc5c8\uae30 \ub54c\ubb38\uc5d0, \uc815\ubd80\ub294 \uc7ac\uc6d0\ud655\ubcf4\ub97c \uc704\ud574 723\ub144(\uc694\ub85c 7\ub144)\uc5d0\ub294 \uc0bc\uc138\uc77c\uc2e0\ubc95(\u4e09\u4e16\u4e00\u8eab\u6cd5)\uc744 \uc2dc\ud589\ud558\uc5ec \uac1c\uac04\uc744 \uc7a5\ub824\ud558\uc600\ub2e4. \ubaac\ubb34 \ucc9c\ud669\uacfc \ud6c4\uc9c0\uc640\ub77c \ud6c4\ud788\ud1a0\uc758 \ub538\uc778 \ud6c4\uc9c0\uc640\ub77c \ubbf8\uc57c\ucf54(\u85e4\u539f\u5bae\u5b50) \uc0ac\uc774\uc5d0\uc11c \ud0dc\uc5b4\ub09c \uc1fc\ubb34 \ucc9c\ud669\uc774 \uc989\uc704\ud55c 724\ub144(\uc9c4\ud0a4 \uc6d0\ub144) \uc988\uc74c\ubd80\ud130 \ud6c4\ud788\ud1a0\uc758 \ub124 \uc544\ub4e4 \ubb34\uce58\ub9c8\ub85c(\u6b66\u667a\u9ebb\u5442), \ud6c4\uc0ac\uc0ac\ud0a4(\u623f\u524d), \uc6b0\ub9c8\uce74\uc774(\u5b87\u5408), \ub9c8\ub85c(\u9ebb\u5442)\uac00 \uc815\uacc4\uc5d0 \uc9c4\ucd9c\ud558\uc600\ub2e4. 729\ub144(\uc9c4\ud0a4 6\ub144), \ub098\uac00\uc57c \uc655\uc774 \uc0ac\uc2e4\uc0c1 \uc815\uacc4 \ucd5c\uace0\uc9c1\uc5d0 \ud574\ub2f9\ud558\ub294 \uc0ac\ub2e4\uc774\uc9c4\uc5d0 \ucde8\uc784\ud558\uc790, \uc774\uc5d0 \uc704\uae30\uac10\uc744 \ub290\ub080 \ud6c4\uc9c0\uc640\ub77c 4\ud615\uc81c\ub294 \u2018\uc88c\ub3c4(\u5de6\u9053)\uc5d0 \ube60\uc838 \uad6d\uc815\uc744 \uae30\uc6b8\uac8c \ud55c\ub2e4\u2019\uace0 \ucc38\uc18c\ud558\uc5ec \ub098\uac00\uc57c \uc655\uc744 \uc790\uacb0\ub85c \ubab0\uc544\ub123\uace0(\ub098\uac00\uc57c \uc655\uc758 \uc815\ubcc0), \uc815\uad8c\uc744 \uc7a5\uc545\ud588\ub2e4. \uc815\ubcc0 \uc9c1\ud6c4, \ud6c4\uc9c0\uc640\ub77c \uc528\ub294 \ud6c4\ud788\ud1a0\uc758 \ub538\uc778 \uace0\ubb18\uc2dc(\u5149\u660e\u5b50, \uace0\ubb18 \ud669\ud6c4)\ub97c \uc1fc\ubb34 \ucc9c\ud669\uc758 \ud669\ud6c4\ub85c \ucd94\ub300\ud558\ub294 \ub370 \uc131\uacf5\ud558\uc5ec, \uace0\ubb18\uc2dc\ub294 \uc2e0\ud558\ub85c\uc11c\ub294 \ucd5c\ucd08\uc758 \ud669\ud6c4\uac00 \ub418\uc5c8\ub2e4.", "sentence_answer": "\ub098\ub77c \uc2dc\ub300 \ucd08\uae30\uc5d0\ub294 \ub098\uce74\ud1a0\ubbf8\ub178 \uac00\ub9c8\ud0c0\ub9ac\uc758 \uc544\ub4e4 \ud6c4\uc9c0\uc640\ub77c \ud6c4\ud788\ud1a0 \uac00 \uc815\uad8c\uc744 \uc950\uace0 \uc728\ub839\uc81c\ub3c4\uc758 \ud655\ub9bd\uc5d0 \uc9c4\ub825\ud558\ub294 \ub3d9\uc2dc\uc5d0 \ud669\uc2e4\uacfc \uc778\ucc99\uad00\uacc4\ub97c \ub9fa\uc73c\uba70 \ud6c4\uc9c0\uc640\ub77c \uc528 \ubc1c\uc804\uc758 \uae30\ucd08\ub97c \ub2e6\uc558\ub2e4.", "paragraph_id": "6461620-1-0", "paragraph_question": "question: \ub098\ub77c \uc2dc\ub300 \ucd08\uae30 \uc815\uad8c\uc744 \uc950\uace0 \ud6c4\uc9c0\uc640\ub77c \uc528 \ubc1c\uc804\uc758 \uae30\ucd08\ub97c \ub2e6\uc740 \uc774\ub294 \ub204\uad6c\uc778\uac00?, context: \ub098\ub77c \uc2dc\ub300 \ucd08\uae30\uc5d0\ub294 \ub098\uce74\ud1a0\ubbf8\ub178 \uac00\ub9c8\ud0c0\ub9ac\uc758 \uc544\ub4e4 \ud6c4\uc9c0\uc640\ub77c \ud6c4\ud788\ud1a0\uac00 \uc815\uad8c\uc744 \uc950\uace0 \uc728\ub839\uc81c\ub3c4\uc758 \ud655\ub9bd\uc5d0 \uc9c4\ub825\ud558\ub294 \ub3d9\uc2dc\uc5d0 \ud669\uc2e4\uacfc \uc778\ucc99\uad00\uacc4\ub97c \ub9fa\uc73c\uba70 \ud6c4\uc9c0\uc640\ub77c \uc528 \ubc1c\uc804\uc758 \uae30\ucd08\ub97c \ub2e6\uc558\ub2e4. \ud6c4\ud788\ud1a0 \uc0ac\ud6c4\uc5d0\ub294 \ub2e4\ucf00\uce58 \ud669\uc790\uc758 \uc544\ub4e4\ub85c \ub374\ubb34 \ucc9c\ud669\uc758 \uc190\uc790\uc778 \ub098\uac00\uc57c \uc655\uc774 \uc6b0\ub2e4\uc774\uc9c4\uc774 \ub418\uc5b4 \uc815\uad8c\uc744 \ub2f4\ub2f9\ud558\uc600\ub2e4. \uc804\ub300\ubd80\ud130 \ub204\uc801\ub418\uc5b4 \uc628 \uacfc\uc911\ud55c \ubd80\ub2f4\uc744 \uacac\ub514\uc9c0 \ubabb\ud55c \ub18d\ubbfc\ub4e4\uc758 \uc720\ub791\uacfc \ub3c4\ub9dd\uc774 \uae09\uaca9\ud788 \ub298\uc5b4\ub098 \uc0ac\ud68c \ubd88\uc548\uc774 \ud45c\uba74\ud654\ub418\uc5c8\uae30 \ub54c\ubb38\uc5d0, \uc815\ubd80\ub294 \uc7ac\uc6d0\ud655\ubcf4\ub97c \uc704\ud574 723\ub144(\uc694\ub85c 7\ub144)\uc5d0\ub294 \uc0bc\uc138\uc77c\uc2e0\ubc95(\u4e09\u4e16\u4e00\u8eab\u6cd5)\uc744 \uc2dc\ud589\ud558\uc5ec \uac1c\uac04\uc744 \uc7a5\ub824\ud558\uc600\ub2e4. \ubaac\ubb34 \ucc9c\ud669\uacfc \ud6c4\uc9c0\uc640\ub77c \ud6c4\ud788\ud1a0\uc758 \ub538\uc778 \ud6c4\uc9c0\uc640\ub77c \ubbf8\uc57c\ucf54(\u85e4\u539f\u5bae\u5b50) \uc0ac\uc774\uc5d0\uc11c \ud0dc\uc5b4\ub09c \uc1fc\ubb34 \ucc9c\ud669\uc774 \uc989\uc704\ud55c 724\ub144(\uc9c4\ud0a4 \uc6d0\ub144) \uc988\uc74c\ubd80\ud130 \ud6c4\ud788\ud1a0\uc758 \ub124 \uc544\ub4e4 \ubb34\uce58\ub9c8\ub85c(\u6b66\u667a\u9ebb\u5442), \ud6c4\uc0ac\uc0ac\ud0a4(\u623f\u524d), \uc6b0\ub9c8\uce74\uc774(\u5b87\u5408), \ub9c8\ub85c(\u9ebb\u5442)\uac00 \uc815\uacc4\uc5d0 \uc9c4\ucd9c\ud558\uc600\ub2e4. 729\ub144(\uc9c4\ud0a4 6\ub144), \ub098\uac00\uc57c \uc655\uc774 \uc0ac\uc2e4\uc0c1 \uc815\uacc4 \ucd5c\uace0\uc9c1\uc5d0 \ud574\ub2f9\ud558\ub294 \uc0ac\ub2e4\uc774\uc9c4\uc5d0 \ucde8\uc784\ud558\uc790, \uc774\uc5d0 \uc704\uae30\uac10\uc744 \ub290\ub080 \ud6c4\uc9c0\uc640\ub77c 4\ud615\uc81c\ub294 \u2018\uc88c\ub3c4(\u5de6\u9053)\uc5d0 \ube60\uc838 \uad6d\uc815\uc744 \uae30\uc6b8\uac8c \ud55c\ub2e4\u2019\uace0 \ucc38\uc18c\ud558\uc5ec \ub098\uac00\uc57c \uc655\uc744 \uc790\uacb0\ub85c \ubab0\uc544\ub123\uace0(\ub098\uac00\uc57c \uc655\uc758 \uc815\ubcc0), \uc815\uad8c\uc744 \uc7a5\uc545\ud588\ub2e4. \uc815\ubcc0 \uc9c1\ud6c4, \ud6c4\uc9c0\uc640\ub77c \uc528\ub294 \ud6c4\ud788\ud1a0\uc758 \ub538\uc778 \uace0\ubb18\uc2dc(\u5149\u660e\u5b50, \uace0\ubb18 \ud669\ud6c4)\ub97c \uc1fc\ubb34 \ucc9c\ud669\uc758 \ud669\ud6c4\ub85c \ucd94\ub300\ud558\ub294 \ub370 \uc131\uacf5\ud558\uc5ec, \uace0\ubb18\uc2dc\ub294 \uc2e0\ud558\ub85c\uc11c\ub294 \ucd5c\ucd08\uc758 \ud669\ud6c4\uac00 \ub418\uc5c8\ub2e4."} {"question": "300\ub300 \uac00\uae4c\uc774 \ub418\ub294 \uc804\uc8fc\uc758 \uc2dc\ub0b4\ubc84\uc2a4\uac00 \uc2dc\uc704\uc5d0 \ucc38\uc5ec\ud55c \uc2dc\uac04\uc740 \uc5b4\ub290\uc815\ub3c4\uc778\uac00?", "paragraph": "10\uc6d4 29\uc77c \ub2f9\uc77c \uc624\ud6c4 4\uc2dc, \uc11c\uc6b8 \uc9d1\ud68c\uc5d0 \uc55e\uc11c \uc804\ub77c\ubd81\ub3c4 \uc804\uc8fc\uc2dc\uc5d0\uc11c\ub294 \uc804\ubd81\ubc84\uc2a4\ub178\ub3d9\uc870\ud569 \uc18c\uc18d \ubc84\uc2a4\uae30\uc0ac\ub4e4\uc774 \ubc15\uadfc\ud61c \ud1f4\uc9c4\uc5d0 \ub3d9\uc758\ud55c\ub2e4\ub294 \ub73b\uc73c\ub85c \uc815\uac01\uc744 \uae30\ud574 \uc6b4\ud589 \uc911 \uacbd\uc801\uc744 \uc6b8\ub9ac\ub294 \uc2dc\uc704\ub97c \ubc8c\uc600\ub2e4. \ucd1d 300\uc5ec\ub300\uc758 \uc804\uc8fc \uc2dc\ub0b4\ubc84\uc2a4\uac00 \uc57d 3\ubd84\uc5ec\uac04 \uc2dc\uc704\uc5d0 \ucc38\uc5ec\ud558\uc600\uc73c\uba70, \uc2dc\uc704 \uad00\ub828 \ud53c\ucf13\uc744 \ubc84\uc2a4 \ub0b4\uc5d0 \ubd80\ucc29\ud574 \uc804\uc8fc \uc2dc\ubbfc\ub4e4\uc5d0\uac8c \uc548\ub0b4\ud558\uc600\ub2e4. \uacbd\uc801 \uc2dc\uc704\ub294 \uc9c0\ub09c 1987\ub144 6\uc6d4 \ud56d\uc7c1 \ub2f9\uc2dc \ubbfc\uc8fc\ud654 \uc6b4\ub3d9\uc758 \uc0c1\uc9d5\uc774\uae30\ub3c4 \ud588\ub2e4. \ub610 \uc804\uc8fc \uc138\uc774\ube0c\uc874 \uc55e\uc5d0\uc11c \ucd1b\ubd88\uc9d1\ud68c\ub97c \uc2dc\uc791, \uacbd\uae30\uc804 \uc55e\uae4c\uc9c0 \ub300\uaddc\ubaa8 \ud589\uc9c4\uc744 \uc9c4\ud589\ud558\uc600\ub2e4. \uac19\uc740 \uc2dc\uac01 \uad11\uc8fc\uad11\uc5ed\uc2dc \uae08\ub0a8\ub85c 5\u00b718\ubbfc\uc8fc\uad11\uc7a5 \uc55e\uc5d0\uc11c\ub294 '\uad6d\uc815\ub18d\ub2e8 \ubc15\uadfc\ud61c \ud1f4\uc9c4 \ucd09\uad6c'\uac00 \uc5f4\ub824 \uc9d1\ud68c\ub97c \uac00\uc9c4 \ub4a4, \uae08\ub0a8\ub85c\ub97c \ub530\ub77c \uc591\ub3d9\uc2dc\uc7a5\uae4c\uc9c0 \uc57d 4km \uac70\ub9ac\ub97c \ud589\uc9c4\ud558\uc600\ub2e4. \uc774\ub0a0 \uad11\uc8fc \uc9d1\ud68c\uc5d0\uc11c\ub294 \uc2dc\ubbfc\uacfc \uc9c4\ubcf4\ub2e8\uccb4 \ub4f1 200\uc5ec\uba85\uc774 \ucc38\uac00\ud558\uc600\ub2e4.", "answer": "3\ubd84", "sentence": "\ucd1d 300\uc5ec\ub300\uc758 \uc804\uc8fc \uc2dc\ub0b4\ubc84\uc2a4\uac00 \uc57d 3\ubd84 \uc5ec\uac04 \uc2dc\uc704\uc5d0 \ucc38\uc5ec\ud558\uc600\uc73c\uba70, \uc2dc\uc704 \uad00\ub828 \ud53c\ucf13\uc744 \ubc84\uc2a4 \ub0b4\uc5d0 \ubd80\ucc29\ud574 \uc804\uc8fc \uc2dc\ubbfc\ub4e4\uc5d0\uac8c \uc548\ub0b4\ud558\uc600\ub2e4.", "paragraph_sentence": "10\uc6d4 29\uc77c \ub2f9\uc77c \uc624\ud6c4 4\uc2dc, \uc11c\uc6b8 \uc9d1\ud68c\uc5d0 \uc55e\uc11c \uc804\ub77c\ubd81\ub3c4 \uc804\uc8fc\uc2dc\uc5d0\uc11c\ub294 \uc804\ubd81\ubc84\uc2a4\ub178\ub3d9\uc870\ud569 \uc18c\uc18d \ubc84\uc2a4\uae30\uc0ac\ub4e4\uc774 \ubc15\uadfc\ud61c \ud1f4\uc9c4\uc5d0 \ub3d9\uc758\ud55c\ub2e4\ub294 \ub73b\uc73c\ub85c \uc815\uac01\uc744 \uae30\ud574 \uc6b4\ud589 \uc911 \uacbd\uc801\uc744 \uc6b8\ub9ac\ub294 \uc2dc\uc704\ub97c \ubc8c\uc600\ub2e4. \ucd1d 300\uc5ec\ub300\uc758 \uc804\uc8fc \uc2dc\ub0b4\ubc84\uc2a4\uac00 \uc57d 3\ubd84 \uc5ec\uac04 \uc2dc\uc704\uc5d0 \ucc38\uc5ec\ud558\uc600\uc73c\uba70, \uc2dc\uc704 \uad00\ub828 \ud53c\ucf13\uc744 \ubc84\uc2a4 \ub0b4\uc5d0 \ubd80\ucc29\ud574 \uc804\uc8fc \uc2dc\ubbfc\ub4e4\uc5d0\uac8c \uc548\ub0b4\ud558\uc600\ub2e4. \uacbd\uc801 \uc2dc\uc704\ub294 \uc9c0\ub09c 1987\ub144 6\uc6d4 \ud56d\uc7c1 \ub2f9\uc2dc \ubbfc\uc8fc\ud654 \uc6b4\ub3d9\uc758 \uc0c1\uc9d5\uc774\uae30\ub3c4 \ud588\ub2e4. \ub610 \uc804\uc8fc \uc138\uc774\ube0c\uc874 \uc55e\uc5d0\uc11c \ucd1b\ubd88\uc9d1\ud68c\ub97c \uc2dc\uc791, \uacbd\uae30\uc804 \uc55e\uae4c\uc9c0 \ub300\uaddc\ubaa8 \ud589\uc9c4\uc744 \uc9c4\ud589\ud558\uc600\ub2e4. \uac19\uc740 \uc2dc\uac01 \uad11\uc8fc\uad11\uc5ed\uc2dc \uae08\ub0a8\ub85c 5\u00b718\ubbfc\uc8fc\uad11\uc7a5 \uc55e\uc5d0\uc11c\ub294 '\uad6d\uc815\ub18d\ub2e8 \ubc15\uadfc\ud61c \ud1f4\uc9c4 \ucd09\uad6c'\uac00 \uc5f4\ub824 \uc9d1\ud68c\ub97c \uac00\uc9c4 \ub4a4, \uae08\ub0a8\ub85c\ub97c \ub530\ub77c \uc591\ub3d9\uc2dc\uc7a5\uae4c\uc9c0 \uc57d 4km \uac70\ub9ac\ub97c \ud589\uc9c4\ud558\uc600\ub2e4. \uc774\ub0a0 \uad11\uc8fc \uc9d1\ud68c\uc5d0\uc11c\ub294 \uc2dc\ubbfc\uacfc \uc9c4\ubcf4\ub2e8\uccb4 \ub4f1 200\uc5ec\uba85\uc774 \ucc38\uac00\ud558\uc600\ub2e4.", "paragraph_answer": "10\uc6d4 29\uc77c \ub2f9\uc77c \uc624\ud6c4 4\uc2dc, \uc11c\uc6b8 \uc9d1\ud68c\uc5d0 \uc55e\uc11c \uc804\ub77c\ubd81\ub3c4 \uc804\uc8fc\uc2dc\uc5d0\uc11c\ub294 \uc804\ubd81\ubc84\uc2a4\ub178\ub3d9\uc870\ud569 \uc18c\uc18d \ubc84\uc2a4\uae30\uc0ac\ub4e4\uc774 \ubc15\uadfc\ud61c \ud1f4\uc9c4\uc5d0 \ub3d9\uc758\ud55c\ub2e4\ub294 \ub73b\uc73c\ub85c \uc815\uac01\uc744 \uae30\ud574 \uc6b4\ud589 \uc911 \uacbd\uc801\uc744 \uc6b8\ub9ac\ub294 \uc2dc\uc704\ub97c \ubc8c\uc600\ub2e4. \ucd1d 300\uc5ec\ub300\uc758 \uc804\uc8fc \uc2dc\ub0b4\ubc84\uc2a4\uac00 \uc57d 3\ubd84 \uc5ec\uac04 \uc2dc\uc704\uc5d0 \ucc38\uc5ec\ud558\uc600\uc73c\uba70, \uc2dc\uc704 \uad00\ub828 \ud53c\ucf13\uc744 \ubc84\uc2a4 \ub0b4\uc5d0 \ubd80\ucc29\ud574 \uc804\uc8fc \uc2dc\ubbfc\ub4e4\uc5d0\uac8c \uc548\ub0b4\ud558\uc600\ub2e4. \uacbd\uc801 \uc2dc\uc704\ub294 \uc9c0\ub09c 1987\ub144 6\uc6d4 \ud56d\uc7c1 \ub2f9\uc2dc \ubbfc\uc8fc\ud654 \uc6b4\ub3d9\uc758 \uc0c1\uc9d5\uc774\uae30\ub3c4 \ud588\ub2e4. \ub610 \uc804\uc8fc \uc138\uc774\ube0c\uc874 \uc55e\uc5d0\uc11c \ucd1b\ubd88\uc9d1\ud68c\ub97c \uc2dc\uc791, \uacbd\uae30\uc804 \uc55e\uae4c\uc9c0 \ub300\uaddc\ubaa8 \ud589\uc9c4\uc744 \uc9c4\ud589\ud558\uc600\ub2e4. \uac19\uc740 \uc2dc\uac01 \uad11\uc8fc\uad11\uc5ed\uc2dc \uae08\ub0a8\ub85c 5\u00b718\ubbfc\uc8fc\uad11\uc7a5 \uc55e\uc5d0\uc11c\ub294 '\uad6d\uc815\ub18d\ub2e8 \ubc15\uadfc\ud61c \ud1f4\uc9c4 \ucd09\uad6c'\uac00 \uc5f4\ub824 \uc9d1\ud68c\ub97c \uac00\uc9c4 \ub4a4, \uae08\ub0a8\ub85c\ub97c \ub530\ub77c \uc591\ub3d9\uc2dc\uc7a5\uae4c\uc9c0 \uc57d 4km \uac70\ub9ac\ub97c \ud589\uc9c4\ud558\uc600\ub2e4. \uc774\ub0a0 \uad11\uc8fc \uc9d1\ud68c\uc5d0\uc11c\ub294 \uc2dc\ubbfc\uacfc \uc9c4\ubcf4\ub2e8\uccb4 \ub4f1 200\uc5ec\uba85\uc774 \ucc38\uac00\ud558\uc600\ub2e4.", "sentence_answer": "\ucd1d 300\uc5ec\ub300\uc758 \uc804\uc8fc \uc2dc\ub0b4\ubc84\uc2a4\uac00 \uc57d 3\ubd84 \uc5ec\uac04 \uc2dc\uc704\uc5d0 \ucc38\uc5ec\ud558\uc600\uc73c\uba70, \uc2dc\uc704 \uad00\ub828 \ud53c\ucf13\uc744 \ubc84\uc2a4 \ub0b4\uc5d0 \ubd80\ucc29\ud574 \uc804\uc8fc \uc2dc\ubbfc\ub4e4\uc5d0\uac8c \uc548\ub0b4\ud558\uc600\ub2e4.", "paragraph_id": "6534298-5-1", "paragraph_question": "question: 300\ub300 \uac00\uae4c\uc774 \ub418\ub294 \uc804\uc8fc\uc758 \uc2dc\ub0b4\ubc84\uc2a4\uac00 \uc2dc\uc704\uc5d0 \ucc38\uc5ec\ud55c \uc2dc\uac04\uc740 \uc5b4\ub290\uc815\ub3c4\uc778\uac00?, context: 10\uc6d4 29\uc77c \ub2f9\uc77c \uc624\ud6c4 4\uc2dc, \uc11c\uc6b8 \uc9d1\ud68c\uc5d0 \uc55e\uc11c \uc804\ub77c\ubd81\ub3c4 \uc804\uc8fc\uc2dc\uc5d0\uc11c\ub294 \uc804\ubd81\ubc84\uc2a4\ub178\ub3d9\uc870\ud569 \uc18c\uc18d \ubc84\uc2a4\uae30\uc0ac\ub4e4\uc774 \ubc15\uadfc\ud61c \ud1f4\uc9c4\uc5d0 \ub3d9\uc758\ud55c\ub2e4\ub294 \ub73b\uc73c\ub85c \uc815\uac01\uc744 \uae30\ud574 \uc6b4\ud589 \uc911 \uacbd\uc801\uc744 \uc6b8\ub9ac\ub294 \uc2dc\uc704\ub97c \ubc8c\uc600\ub2e4. \ucd1d 300\uc5ec\ub300\uc758 \uc804\uc8fc \uc2dc\ub0b4\ubc84\uc2a4\uac00 \uc57d 3\ubd84\uc5ec\uac04 \uc2dc\uc704\uc5d0 \ucc38\uc5ec\ud558\uc600\uc73c\uba70, \uc2dc\uc704 \uad00\ub828 \ud53c\ucf13\uc744 \ubc84\uc2a4 \ub0b4\uc5d0 \ubd80\ucc29\ud574 \uc804\uc8fc \uc2dc\ubbfc\ub4e4\uc5d0\uac8c \uc548\ub0b4\ud558\uc600\ub2e4. \uacbd\uc801 \uc2dc\uc704\ub294 \uc9c0\ub09c 1987\ub144 6\uc6d4 \ud56d\uc7c1 \ub2f9\uc2dc \ubbfc\uc8fc\ud654 \uc6b4\ub3d9\uc758 \uc0c1\uc9d5\uc774\uae30\ub3c4 \ud588\ub2e4. \ub610 \uc804\uc8fc \uc138\uc774\ube0c\uc874 \uc55e\uc5d0\uc11c \ucd1b\ubd88\uc9d1\ud68c\ub97c \uc2dc\uc791, \uacbd\uae30\uc804 \uc55e\uae4c\uc9c0 \ub300\uaddc\ubaa8 \ud589\uc9c4\uc744 \uc9c4\ud589\ud558\uc600\ub2e4. \uac19\uc740 \uc2dc\uac01 \uad11\uc8fc\uad11\uc5ed\uc2dc \uae08\ub0a8\ub85c 5\u00b718\ubbfc\uc8fc\uad11\uc7a5 \uc55e\uc5d0\uc11c\ub294 '\uad6d\uc815\ub18d\ub2e8 \ubc15\uadfc\ud61c \ud1f4\uc9c4 \ucd09\uad6c'\uac00 \uc5f4\ub824 \uc9d1\ud68c\ub97c \uac00\uc9c4 \ub4a4, \uae08\ub0a8\ub85c\ub97c \ub530\ub77c \uc591\ub3d9\uc2dc\uc7a5\uae4c\uc9c0 \uc57d 4km \uac70\ub9ac\ub97c \ud589\uc9c4\ud558\uc600\ub2e4. \uc774\ub0a0 \uad11\uc8fc \uc9d1\ud68c\uc5d0\uc11c\ub294 \uc2dc\ubbfc\uacfc \uc9c4\ubcf4\ub2e8\uccb4 \ub4f1 200\uc5ec\uba85\uc774 \ucc38\uac00\ud558\uc600\ub2e4."} {"question": "\uac00\ubb3c\uce58\uac00 \ubc1c\uacac\ub41c \uadfc\ucc98 \uc5f0\ubabb\uc5d0 \uc0b4\ud3ec\ud55c \uac83\uc740?", "paragraph": "\ubbf8\uad6d \ud604\uc9c0\uc5d0\uc11c \uccab \ubc88\uc9f8\ub85c \uac00\ubb3c\uce58\uac00 \ub274\uc2a4\uc5d0 \uc2e4\ub9b0 \uac83\uc740 2002\ub144 \uc5ec\ub984 \uba54\ub9b4\ub79c\ub4dc \uc8fc \ud06c\ub85c\ud504\ud134\uc5d0\uc11c\uc600\ub2e4. \uac00\ubb3c\uce58\uac00 \uccb4\uc11c\ud53c\ud06c \ub9cc \uc720\uc5ed\uc5d0 \ud574\ub97c \uc785\ud788\uac8c \ub418\uc790 \ub2f9\uc9c0\uc758 \uacf5\ubb34\uc6d0\ub4e4\uc774 \uc7ac\ube68\ub9ac \ub300\ucc98\ud574 \uc5f0\ubabb\uc758 \ubb3c\uc744 \ud37c\ub0b4\ub294 \ub4f1 \uac00\ubb3c\uce58\ub97c \ubfcc\ub9ac \ubf51\ub294 \uc791\uc5c5\uc744 \uc2e4\uc2dc\ud588\uace0, \uadf8 \uacb0\uacfc 2\ub9c8\ub9ac\uc758 \uc131\uccb4 \uac00\ubb3c\uce58\uc640 100\ub9c8\ub9ac\uac00 \ub118\ub294 \uce58\uc5b4\ub97c \ubc15\uba78\ud560 \uc218 \uc788\uc5c8\ub2e4. \uac00\ubb3c\uce58 \ucd9c\ud604\uc758 \uc6d0\uc778\uc740 \ud55c \ub0a8\uc131\uc774 \uc2dc\uc7a5\uc5d0\uc11c \uc0ac\uc628 \uac00\ubb3c\uce58\ub97c \uc5f0\ubabb\uc5d0 \ud480\uc5b4\ub193\uc740 \uac83\uc774\uc5c8\ub2e4. \uc774 \ub54c \ub2f9\uad6d\uc5d0\uc11c\ub294 \uac00\ubb3c\uce58\uac00 \ubc1c\uacac\ub41c \uadfc\ucc98\uc758 \uc138 \uc5f0\ubabb\uc5d0 \uc81c\ucd08\uc81c \ubc0f \uc0b4\uc5b4\uc81c\uc778 \ub85c\ud14c\ub17c\uc744 \uc0b4\ud3ec\ud588\ub294\ub370, \uc218\uc911\uc5d0\uc11c 1~3\uc77c \uc815\ub3c4\ub85c \ube60\ub978 \ubc18\uac10\uae30\ub97c \uc9c0\ub2cc \uc774 \ubb3c\uc9c8\uc740 \uadf9\uc131\uc2a4\ub7ec\uc6b4 \uac00\ubb3c\uce58\uc758 \uce68\uc785\uc744 \ub9c9\uae30 \uc704\ud574 \uc5f0\ubabb\uc758 \ubaa8\ub4e0 \ubb3c\uace0\uae30\ub97c \uc8fd\uc774\ub294 \ub370\uc5d0 \uc0ac\uc6a9\ub418\uc5c8\ub2e4.", "answer": "\uc81c\ucd08\uc81c \ubc0f \uc0b4\uc5b4\uc81c", "sentence": "\uc774 \ub54c \ub2f9\uad6d\uc5d0\uc11c\ub294 \uac00\ubb3c\uce58\uac00 \ubc1c\uacac\ub41c \uadfc\ucc98\uc758 \uc138 \uc5f0\ubabb\uc5d0 \uc81c\ucd08\uc81c \ubc0f \uc0b4\uc5b4\uc81c \uc778 \ub85c\ud14c\ub17c\uc744 \uc0b4\ud3ec\ud588\ub294\ub370, \uc218\uc911\uc5d0\uc11c 1~3\uc77c \uc815\ub3c4\ub85c \ube60\ub978 \ubc18\uac10\uae30\ub97c \uc9c0\ub2cc \uc774 \ubb3c\uc9c8\uc740 \uadf9\uc131\uc2a4\ub7ec\uc6b4 \uac00\ubb3c\uce58\uc758 \uce68\uc785\uc744 \ub9c9\uae30 \uc704\ud574 \uc5f0\ubabb\uc758 \ubaa8\ub4e0 \ubb3c\uace0\uae30\ub97c \uc8fd\uc774\ub294 \ub370\uc5d0 \uc0ac\uc6a9\ub418\uc5c8\ub2e4.", "paragraph_sentence": "\ubbf8\uad6d \ud604\uc9c0\uc5d0\uc11c \uccab \ubc88\uc9f8\ub85c \uac00\ubb3c\uce58\uac00 \ub274\uc2a4\uc5d0 \uc2e4\ub9b0 \uac83\uc740 2002\ub144 \uc5ec\ub984 \uba54\ub9b4\ub79c\ub4dc \uc8fc \ud06c\ub85c\ud504\ud134\uc5d0\uc11c\uc600\ub2e4. \uac00\ubb3c\uce58\uac00 \uccb4\uc11c\ud53c\ud06c \ub9cc \uc720\uc5ed\uc5d0 \ud574\ub97c \uc785\ud788\uac8c \ub418\uc790 \ub2f9\uc9c0\uc758 \uacf5\ubb34\uc6d0\ub4e4\uc774 \uc7ac\ube68\ub9ac \ub300\ucc98\ud574 \uc5f0\ubabb\uc758 \ubb3c\uc744 \ud37c\ub0b4\ub294 \ub4f1 \uac00\ubb3c\uce58\ub97c \ubfcc\ub9ac \ubf51\ub294 \uc791\uc5c5\uc744 \uc2e4\uc2dc\ud588\uace0, \uadf8 \uacb0\uacfc 2\ub9c8\ub9ac\uc758 \uc131\uccb4 \uac00\ubb3c\uce58\uc640 100\ub9c8\ub9ac\uac00 \ub118\ub294 \uce58\uc5b4\ub97c \ubc15\uba78\ud560 \uc218 \uc788\uc5c8\ub2e4. \uac00\ubb3c\uce58 \ucd9c\ud604\uc758 \uc6d0\uc778\uc740 \ud55c \ub0a8\uc131\uc774 \uc2dc\uc7a5\uc5d0\uc11c \uc0ac\uc628 \uac00\ubb3c\uce58\ub97c \uc5f0\ubabb\uc5d0 \ud480\uc5b4\ub193\uc740 \uac83\uc774\uc5c8\ub2e4. \uc774 \ub54c \ub2f9\uad6d\uc5d0\uc11c\ub294 \uac00\ubb3c\uce58\uac00 \ubc1c\uacac\ub41c \uadfc\ucc98\uc758 \uc138 \uc5f0\ubabb\uc5d0 \uc81c\ucd08\uc81c \ubc0f \uc0b4\uc5b4\uc81c \uc778 \ub85c\ud14c\ub17c\uc744 \uc0b4\ud3ec\ud588\ub294\ub370, \uc218\uc911\uc5d0\uc11c 1~3\uc77c \uc815\ub3c4\ub85c \ube60\ub978 \ubc18\uac10\uae30\ub97c \uc9c0\ub2cc \uc774 \ubb3c\uc9c8\uc740 \uadf9\uc131\uc2a4\ub7ec\uc6b4 \uac00\ubb3c\uce58\uc758 \uce68\uc785\uc744 \ub9c9\uae30 \uc704\ud574 \uc5f0\ubabb\uc758 \ubaa8\ub4e0 \ubb3c\uace0\uae30\ub97c \uc8fd\uc774\ub294 \ub370\uc5d0 \uc0ac\uc6a9\ub418\uc5c8\ub2e4. ", "paragraph_answer": "\ubbf8\uad6d \ud604\uc9c0\uc5d0\uc11c \uccab \ubc88\uc9f8\ub85c \uac00\ubb3c\uce58\uac00 \ub274\uc2a4\uc5d0 \uc2e4\ub9b0 \uac83\uc740 2002\ub144 \uc5ec\ub984 \uba54\ub9b4\ub79c\ub4dc \uc8fc \ud06c\ub85c\ud504\ud134\uc5d0\uc11c\uc600\ub2e4. \uac00\ubb3c\uce58\uac00 \uccb4\uc11c\ud53c\ud06c \ub9cc \uc720\uc5ed\uc5d0 \ud574\ub97c \uc785\ud788\uac8c \ub418\uc790 \ub2f9\uc9c0\uc758 \uacf5\ubb34\uc6d0\ub4e4\uc774 \uc7ac\ube68\ub9ac \ub300\ucc98\ud574 \uc5f0\ubabb\uc758 \ubb3c\uc744 \ud37c\ub0b4\ub294 \ub4f1 \uac00\ubb3c\uce58\ub97c \ubfcc\ub9ac \ubf51\ub294 \uc791\uc5c5\uc744 \uc2e4\uc2dc\ud588\uace0, \uadf8 \uacb0\uacfc 2\ub9c8\ub9ac\uc758 \uc131\uccb4 \uac00\ubb3c\uce58\uc640 100\ub9c8\ub9ac\uac00 \ub118\ub294 \uce58\uc5b4\ub97c \ubc15\uba78\ud560 \uc218 \uc788\uc5c8\ub2e4. \uac00\ubb3c\uce58 \ucd9c\ud604\uc758 \uc6d0\uc778\uc740 \ud55c \ub0a8\uc131\uc774 \uc2dc\uc7a5\uc5d0\uc11c \uc0ac\uc628 \uac00\ubb3c\uce58\ub97c \uc5f0\ubabb\uc5d0 \ud480\uc5b4\ub193\uc740 \uac83\uc774\uc5c8\ub2e4. \uc774 \ub54c \ub2f9\uad6d\uc5d0\uc11c\ub294 \uac00\ubb3c\uce58\uac00 \ubc1c\uacac\ub41c \uadfc\ucc98\uc758 \uc138 \uc5f0\ubabb\uc5d0 \uc81c\ucd08\uc81c \ubc0f \uc0b4\uc5b4\uc81c \uc778 \ub85c\ud14c\ub17c\uc744 \uc0b4\ud3ec\ud588\ub294\ub370, \uc218\uc911\uc5d0\uc11c 1~3\uc77c \uc815\ub3c4\ub85c \ube60\ub978 \ubc18\uac10\uae30\ub97c \uc9c0\ub2cc \uc774 \ubb3c\uc9c8\uc740 \uadf9\uc131\uc2a4\ub7ec\uc6b4 \uac00\ubb3c\uce58\uc758 \uce68\uc785\uc744 \ub9c9\uae30 \uc704\ud574 \uc5f0\ubabb\uc758 \ubaa8\ub4e0 \ubb3c\uace0\uae30\ub97c \uc8fd\uc774\ub294 \ub370\uc5d0 \uc0ac\uc6a9\ub418\uc5c8\ub2e4.", "sentence_answer": "\uc774 \ub54c \ub2f9\uad6d\uc5d0\uc11c\ub294 \uac00\ubb3c\uce58\uac00 \ubc1c\uacac\ub41c \uadfc\ucc98\uc758 \uc138 \uc5f0\ubabb\uc5d0 \uc81c\ucd08\uc81c \ubc0f \uc0b4\uc5b4\uc81c \uc778 \ub85c\ud14c\ub17c\uc744 \uc0b4\ud3ec\ud588\ub294\ub370, \uc218\uc911\uc5d0\uc11c 1~3\uc77c \uc815\ub3c4\ub85c \ube60\ub978 \ubc18\uac10\uae30\ub97c \uc9c0\ub2cc \uc774 \ubb3c\uc9c8\uc740 \uadf9\uc131\uc2a4\ub7ec\uc6b4 \uac00\ubb3c\uce58\uc758 \uce68\uc785\uc744 \ub9c9\uae30 \uc704\ud574 \uc5f0\ubabb\uc758 \ubaa8\ub4e0 \ubb3c\uace0\uae30\ub97c \uc8fd\uc774\ub294 \ub370\uc5d0 \uc0ac\uc6a9\ub418\uc5c8\ub2e4.", "paragraph_id": "6603220-2-2", "paragraph_question": "question: \uac00\ubb3c\uce58\uac00 \ubc1c\uacac\ub41c \uadfc\ucc98 \uc5f0\ubabb\uc5d0 \uc0b4\ud3ec\ud55c \uac83\uc740?, context: \ubbf8\uad6d \ud604\uc9c0\uc5d0\uc11c \uccab \ubc88\uc9f8\ub85c \uac00\ubb3c\uce58\uac00 \ub274\uc2a4\uc5d0 \uc2e4\ub9b0 \uac83\uc740 2002\ub144 \uc5ec\ub984 \uba54\ub9b4\ub79c\ub4dc \uc8fc \ud06c\ub85c\ud504\ud134\uc5d0\uc11c\uc600\ub2e4. \uac00\ubb3c\uce58\uac00 \uccb4\uc11c\ud53c\ud06c \ub9cc \uc720\uc5ed\uc5d0 \ud574\ub97c \uc785\ud788\uac8c \ub418\uc790 \ub2f9\uc9c0\uc758 \uacf5\ubb34\uc6d0\ub4e4\uc774 \uc7ac\ube68\ub9ac \ub300\ucc98\ud574 \uc5f0\ubabb\uc758 \ubb3c\uc744 \ud37c\ub0b4\ub294 \ub4f1 \uac00\ubb3c\uce58\ub97c \ubfcc\ub9ac \ubf51\ub294 \uc791\uc5c5\uc744 \uc2e4\uc2dc\ud588\uace0, \uadf8 \uacb0\uacfc 2\ub9c8\ub9ac\uc758 \uc131\uccb4 \uac00\ubb3c\uce58\uc640 100\ub9c8\ub9ac\uac00 \ub118\ub294 \uce58\uc5b4\ub97c \ubc15\uba78\ud560 \uc218 \uc788\uc5c8\ub2e4. \uac00\ubb3c\uce58 \ucd9c\ud604\uc758 \uc6d0\uc778\uc740 \ud55c \ub0a8\uc131\uc774 \uc2dc\uc7a5\uc5d0\uc11c \uc0ac\uc628 \uac00\ubb3c\uce58\ub97c \uc5f0\ubabb\uc5d0 \ud480\uc5b4\ub193\uc740 \uac83\uc774\uc5c8\ub2e4. \uc774 \ub54c \ub2f9\uad6d\uc5d0\uc11c\ub294 \uac00\ubb3c\uce58\uac00 \ubc1c\uacac\ub41c \uadfc\ucc98\uc758 \uc138 \uc5f0\ubabb\uc5d0 \uc81c\ucd08\uc81c \ubc0f \uc0b4\uc5b4\uc81c\uc778 \ub85c\ud14c\ub17c\uc744 \uc0b4\ud3ec\ud588\ub294\ub370, \uc218\uc911\uc5d0\uc11c 1~3\uc77c \uc815\ub3c4\ub85c \ube60\ub978 \ubc18\uac10\uae30\ub97c \uc9c0\ub2cc \uc774 \ubb3c\uc9c8\uc740 \uadf9\uc131\uc2a4\ub7ec\uc6b4 \uac00\ubb3c\uce58\uc758 \uce68\uc785\uc744 \ub9c9\uae30 \uc704\ud574 \uc5f0\ubabb\uc758 \ubaa8\ub4e0 \ubb3c\uace0\uae30\ub97c \uc8fd\uc774\ub294 \ub370\uc5d0 \uc0ac\uc6a9\ub418\uc5c8\ub2e4."} {"question": "\ub77c\uba55\uc774 \uc218\uace0\ub86d\uac8c \uc77c\ud558\ub294 \uc6b0\ub9ac\ub97c \uc774 \uc544\ub4e4\uc774 \uc548\uc704\ud558\ub9ac\ub77c\ud558\uace0 \uc9c0\uc740 \uc544\ub4e4 \uc774\ub984\uc740?", "paragraph": "\uc5d0\ub178\uc2a4\ub294 \uad6c\uc2ed \uc138\uc5d0 \uac8c\ub09c\uc744 \ub0b3\uc558\uace0 \uac8c\ub09c\uc744 \ub0b3\uc740 \ud6c4 \ud314\ubc31\uc2ed\uc624 \ub144\uc744 \uc9c0\ub0b4\uba70 \uc790\ub140\ub4e4\uc744 \ub0b3\uc558\uc73c\uba70 \uadf8\ub294 \uad6c\ubc31\uc624 \uc138\ub97c \uc0b4\uace0 \uc8fd\uc5c8\ub354\ub77c \uac8c\ub09c\uc740 \uce60\uc2ed \uc138\uc5d0 \ub9c8\ud560\ub784\ub810\uc744 \ub0b3\uc558\uace0 \ub9c8\ud560\ub784\ub810\uc744 \ub0b3\uc740 \ud6c4 \ud314\ubc31\uc0ac\uc2ed \ub144\uc744 \uc9c0\ub0b4\uba70 \uc790\ub140\ub4e4\uc744 \ub0b3\uc558\uc73c\uba70 \uadf8\ub294 \uad6c\ubc31\uc2ed \uc138\ub97c \uc0b4\uace0 \uc8fd\uc5c8\ub354\ub77c \ub9c8\ud560\ub784\ub810\uc740 \uc721\uc2ed\uc624 \uc138\uc5d0 \uc57c\ub81b\uc744 \ub0b3\uc558\uace0 \uc57c\ub81b\uc744 \ub0b3\uc740 \ud6c4 \ud314\ubc31\uc0bc\uc2ed \ub144\uc744 \uc9c0\ub0b4\uba70 \uc790\ub140\ub97c \ub0b3\uc558\uc73c\uba70 \uadf8\ub294 \ud314\ubc31\uad6c\uc2ed\uc624 \uc138\ub97c \uc0b4\uace0 \uc8fd\uc5c8\ub354\ub77c \uc57c\ub81b\uc740 \ubc31\uc721\uc2ed\uc774 \uc138\uc5d0 \uc5d0\ub179\uc744 \ub0b3\uc558\uace0 \uc5d0\ub179\uc744 \ub0b3\uc740 \ud6c4 \ud314\ubc31 \ub144\uc744 \uc9c0\ub0b4\uba70 \uc790\ub140\ub4e4\uc744 \ub0b3\uc558\uc73c\uba70 \uadf8\ub294 \uad6c\ubc31\uc721\uc2ed\uc774 \uc138\ub97c \uc0b4\uace0 \uc8fd\uc5c8\ub354\ub77c \uc5d0\ub179\uc740 \uc721\uc2ed\uc624 \uc138\uc5d0 \ubbc0\ub450\uc140\ub77c\ub97c \ub0b3\uc558\uace0 \ubbc0\ub450\uc140\ub77c\ub97c \ub0b3\uc740 \ud6c4 \uc0bc\ubc31 \ub144\uc744 \ud558\ub098\ub2d8\uacfc \ub3d9\ud589\ud558\uba70 \uc790\ub140\ub4e4\uc744 \ub0b3\uc558\uc73c\uba70 \uadf8\ub294 \uc0bc\ubc31\uc721\uc2ed\uc624 \uc138\ub97c \uc0b4\uc558\ub354\ub77c \uc5d0\ub179\uc774 \ud558\ub098\ub2d8\uacfc \ub3d9\ud589\ud558\ub354\ub2c8 \ud558\ub098\ub2d8\uc774 \uadf8\ub97c \ub370\ub824\uac00\uc2dc\ubbc0\ub85c \uc138\uc0c1\uc5d0 \uc788\uc9c0 \uc544\ub2c8\ud558\uc600\ub354\ub77c \ubbc0\ub450\uc140\ub77c\ub294 \ubc31\ud314\uc2ed\uce60 \uc138\uc5d0 \ub77c\uba55\uc744 \ub0b3\uc558\uace0 \ub77c\uba55\uc744 \ub0b3\uc740 \ud6c4 \uce60\ubc31\ud314\uc2ed\uc774 \ub144\uc744 \uc9c0\ub0b4\uba70 \uc790\ub140\ub97c \ub0b3\uc558\uc73c\uba70 \uadf8\ub294 \uad6c\ubc31\uc721\uc2ed\uad6c \uc138\ub97c \uc0b4\uace0 \uc8fd\uc5c8\ub354\ub77c \ub77c\uba55\uc740 \ubc31\ud314\uc2ed\uc774 \uc138\uc5d0 \uc544\ub4e4\uc744 \ub0b3\uace0 \uc774\ub984\uc744 \ub178\uc544\ub77c \ud558\uc5ec \uc774\ub974\ub418 \uc5ec\ud638\uc640\uaed8\uc11c \ub545\uc744 \uc800\uc8fc\ud558\uc2dc\ubbc0\ub85c \uc218\uace0\ub86d\uac8c \uc77c\ud558\ub294 \uc6b0\ub9ac\ub97c \uc774 \uc544\ub4e4\uc774 \uc548\uc704\ud558\ub9ac\ub77c \ud558\uc600\ub354\ub77c \ub77c\uba55\uc740 \ub178\uc544\ub97c \ub0b3\uc740 \ud6c4 \uc624\ubc31\uad6c\uc2ed\uc624 \ub144\uc744 \uc9c0\ub0b4\uba70 \uc790\ub140\ub4e4\uc744 \ub0b3\uc558\uc73c\uba70", "answer": "\ub178\uc544", "sentence": "\uc5d0\ub178\uc2a4\ub294 \uad6c\uc2ed \uc138\uc5d0 \uac8c\ub09c\uc744 \ub0b3\uc558\uace0 \uac8c\ub09c\uc744 \ub0b3\uc740 \ud6c4 \ud314\ubc31\uc2ed\uc624 \ub144\uc744 \uc9c0\ub0b4\uba70 \uc790\ub140\ub4e4\uc744 \ub0b3\uc558\uc73c\uba70 \uadf8\ub294 \uad6c\ubc31\uc624 \uc138\ub97c \uc0b4\uace0 \uc8fd\uc5c8\ub354\ub77c \uac8c\ub09c\uc740 \uce60\uc2ed \uc138\uc5d0 \ub9c8\ud560\ub784\ub810\uc744 \ub0b3\uc558\uace0 \ub9c8\ud560\ub784\ub810\uc744 \ub0b3\uc740 \ud6c4 \ud314\ubc31\uc0ac\uc2ed \ub144\uc744 \uc9c0\ub0b4\uba70 \uc790\ub140\ub4e4\uc744 \ub0b3\uc558\uc73c\uba70 \uadf8\ub294 \uad6c\ubc31\uc2ed \uc138\ub97c \uc0b4\uace0 \uc8fd\uc5c8\ub354\ub77c \ub9c8\ud560\ub784\ub810\uc740 \uc721\uc2ed\uc624 \uc138\uc5d0 \uc57c\ub81b\uc744 \ub0b3\uc558\uace0 \uc57c\ub81b\uc744 \ub0b3\uc740 \ud6c4 \ud314\ubc31\uc0bc\uc2ed \ub144\uc744 \uc9c0\ub0b4\uba70 \uc790\ub140\ub97c \ub0b3\uc558\uc73c\uba70 \uadf8\ub294 \ud314\ubc31\uad6c\uc2ed\uc624 \uc138\ub97c \uc0b4\uace0 \uc8fd\uc5c8\ub354\ub77c \uc57c\ub81b\uc740 \ubc31\uc721\uc2ed\uc774 \uc138\uc5d0 \uc5d0\ub179\uc744 \ub0b3\uc558\uace0 \uc5d0\ub179\uc744 \ub0b3\uc740 \ud6c4 \ud314\ubc31 \ub144\uc744 \uc9c0\ub0b4\uba70 \uc790\ub140\ub4e4\uc744 \ub0b3\uc558\uc73c\uba70 \uadf8\ub294 \uad6c\ubc31\uc721\uc2ed\uc774 \uc138\ub97c \uc0b4\uace0 \uc8fd\uc5c8\ub354\ub77c \uc5d0\ub179\uc740 \uc721\uc2ed\uc624 \uc138\uc5d0 \ubbc0\ub450\uc140\ub77c\ub97c \ub0b3\uc558\uace0 \ubbc0\ub450\uc140\ub77c\ub97c \ub0b3\uc740 \ud6c4 \uc0bc\ubc31 \ub144\uc744 \ud558\ub098\ub2d8\uacfc \ub3d9\ud589\ud558\uba70 \uc790\ub140\ub4e4\uc744 \ub0b3\uc558\uc73c\uba70 \uadf8\ub294 \uc0bc\ubc31\uc721\uc2ed\uc624 \uc138\ub97c \uc0b4\uc558\ub354\ub77c \uc5d0\ub179\uc774 \ud558\ub098\ub2d8\uacfc \ub3d9\ud589\ud558\ub354\ub2c8 \ud558\ub098\ub2d8\uc774 \uadf8\ub97c \ub370\ub824\uac00\uc2dc\ubbc0\ub85c \uc138\uc0c1\uc5d0 \uc788\uc9c0 \uc544\ub2c8\ud558\uc600\ub354\ub77c \ubbc0\ub450\uc140\ub77c\ub294 \ubc31\ud314\uc2ed\uce60 \uc138\uc5d0 \ub77c\uba55\uc744 \ub0b3\uc558\uace0 \ub77c\uba55\uc744 \ub0b3\uc740 \ud6c4 \uce60\ubc31\ud314\uc2ed\uc774 \ub144\uc744 \uc9c0\ub0b4\uba70 \uc790\ub140\ub97c \ub0b3\uc558\uc73c\uba70 \uadf8\ub294 \uad6c\ubc31\uc721\uc2ed\uad6c \uc138\ub97c \uc0b4\uace0 \uc8fd\uc5c8\ub354\ub77c \ub77c\uba55\uc740 \ubc31\ud314\uc2ed\uc774 \uc138\uc5d0 \uc544\ub4e4\uc744 \ub0b3\uace0 \uc774\ub984\uc744 \ub178\uc544 \ub77c \ud558\uc5ec \uc774\ub974\ub418 \uc5ec\ud638\uc640\uaed8\uc11c \ub545\uc744 \uc800\uc8fc\ud558\uc2dc\ubbc0\ub85c \uc218\uace0\ub86d\uac8c \uc77c\ud558\ub294 \uc6b0\ub9ac\ub97c \uc774 \uc544\ub4e4\uc774 \uc548\uc704\ud558\ub9ac\ub77c \ud558\uc600\ub354\ub77c \ub77c\uba55\uc740 \ub178\uc544\ub97c \ub0b3\uc740 \ud6c4 \uc624\ubc31\uad6c\uc2ed\uc624 \ub144\uc744 \uc9c0\ub0b4\uba70 \uc790\ub140\ub4e4\uc744 \ub0b3\uc558\uc73c\uba70", "paragraph_sentence": " \uc5d0\ub178\uc2a4\ub294 \uad6c\uc2ed \uc138\uc5d0 \uac8c\ub09c\uc744 \ub0b3\uc558\uace0 \uac8c\ub09c\uc744 \ub0b3\uc740 \ud6c4 \ud314\ubc31\uc2ed\uc624 \ub144\uc744 \uc9c0\ub0b4\uba70 \uc790\ub140\ub4e4\uc744 \ub0b3\uc558\uc73c\uba70 \uadf8\ub294 \uad6c\ubc31\uc624 \uc138\ub97c \uc0b4\uace0 \uc8fd\uc5c8\ub354\ub77c \uac8c\ub09c\uc740 \uce60\uc2ed \uc138\uc5d0 \ub9c8\ud560\ub784\ub810\uc744 \ub0b3\uc558\uace0 \ub9c8\ud560\ub784\ub810\uc744 \ub0b3\uc740 \ud6c4 \ud314\ubc31\uc0ac\uc2ed \ub144\uc744 \uc9c0\ub0b4\uba70 \uc790\ub140\ub4e4\uc744 \ub0b3\uc558\uc73c\uba70 \uadf8\ub294 \uad6c\ubc31\uc2ed \uc138\ub97c \uc0b4\uace0 \uc8fd\uc5c8\ub354\ub77c \ub9c8\ud560\ub784\ub810\uc740 \uc721\uc2ed\uc624 \uc138\uc5d0 \uc57c\ub81b\uc744 \ub0b3\uc558\uace0 \uc57c\ub81b\uc744 \ub0b3\uc740 \ud6c4 \ud314\ubc31\uc0bc\uc2ed \ub144\uc744 \uc9c0\ub0b4\uba70 \uc790\ub140\ub97c \ub0b3\uc558\uc73c\uba70 \uadf8\ub294 \ud314\ubc31\uad6c\uc2ed\uc624 \uc138\ub97c \uc0b4\uace0 \uc8fd\uc5c8\ub354\ub77c \uc57c\ub81b\uc740 \ubc31\uc721\uc2ed\uc774 \uc138\uc5d0 \uc5d0\ub179\uc744 \ub0b3\uc558\uace0 \uc5d0\ub179\uc744 \ub0b3\uc740 \ud6c4 \ud314\ubc31 \ub144\uc744 \uc9c0\ub0b4\uba70 \uc790\ub140\ub4e4\uc744 \ub0b3\uc558\uc73c\uba70 \uadf8\ub294 \uad6c\ubc31\uc721\uc2ed\uc774 \uc138\ub97c \uc0b4\uace0 \uc8fd\uc5c8\ub354\ub77c \uc5d0\ub179\uc740 \uc721\uc2ed\uc624 \uc138\uc5d0 \ubbc0\ub450\uc140\ub77c\ub97c \ub0b3\uc558\uace0 \ubbc0\ub450\uc140\ub77c\ub97c \ub0b3\uc740 \ud6c4 \uc0bc\ubc31 \ub144\uc744 \ud558\ub098\ub2d8\uacfc \ub3d9\ud589\ud558\uba70 \uc790\ub140\ub4e4\uc744 \ub0b3\uc558\uc73c\uba70 \uadf8\ub294 \uc0bc\ubc31\uc721\uc2ed\uc624 \uc138\ub97c \uc0b4\uc558\ub354\ub77c \uc5d0\ub179\uc774 \ud558\ub098\ub2d8\uacfc \ub3d9\ud589\ud558\ub354\ub2c8 \ud558\ub098\ub2d8\uc774 \uadf8\ub97c \ub370\ub824\uac00\uc2dc\ubbc0\ub85c \uc138\uc0c1\uc5d0 \uc788\uc9c0 \uc544\ub2c8\ud558\uc600\ub354\ub77c \ubbc0\ub450\uc140\ub77c\ub294 \ubc31\ud314\uc2ed\uce60 \uc138\uc5d0 \ub77c\uba55\uc744 \ub0b3\uc558\uace0 \ub77c\uba55\uc744 \ub0b3\uc740 \ud6c4 \uce60\ubc31\ud314\uc2ed\uc774 \ub144\uc744 \uc9c0\ub0b4\uba70 \uc790\ub140\ub97c \ub0b3\uc558\uc73c\uba70 \uadf8\ub294 \uad6c\ubc31\uc721\uc2ed\uad6c \uc138\ub97c \uc0b4\uace0 \uc8fd\uc5c8\ub354\ub77c \ub77c\uba55\uc740 \ubc31\ud314\uc2ed\uc774 \uc138\uc5d0 \uc544\ub4e4\uc744 \ub0b3\uace0 \uc774\ub984\uc744 \ub178\uc544 \ub77c \ud558\uc5ec \uc774\ub974\ub418 \uc5ec\ud638\uc640\uaed8\uc11c \ub545\uc744 \uc800\uc8fc\ud558\uc2dc\ubbc0\ub85c \uc218\uace0\ub86d\uac8c \uc77c\ud558\ub294 \uc6b0\ub9ac\ub97c \uc774 \uc544\ub4e4\uc774 \uc548\uc704\ud558\ub9ac\ub77c \ud558\uc600\ub354\ub77c \ub77c\uba55\uc740 \ub178\uc544\ub97c \ub0b3\uc740 \ud6c4 \uc624\ubc31\uad6c\uc2ed\uc624 \ub144\uc744 \uc9c0\ub0b4\uba70 \uc790\ub140\ub4e4\uc744 \ub0b3\uc558\uc73c\uba70 ", "paragraph_answer": "\uc5d0\ub178\uc2a4\ub294 \uad6c\uc2ed \uc138\uc5d0 \uac8c\ub09c\uc744 \ub0b3\uc558\uace0 \uac8c\ub09c\uc744 \ub0b3\uc740 \ud6c4 \ud314\ubc31\uc2ed\uc624 \ub144\uc744 \uc9c0\ub0b4\uba70 \uc790\ub140\ub4e4\uc744 \ub0b3\uc558\uc73c\uba70 \uadf8\ub294 \uad6c\ubc31\uc624 \uc138\ub97c \uc0b4\uace0 \uc8fd\uc5c8\ub354\ub77c \uac8c\ub09c\uc740 \uce60\uc2ed \uc138\uc5d0 \ub9c8\ud560\ub784\ub810\uc744 \ub0b3\uc558\uace0 \ub9c8\ud560\ub784\ub810\uc744 \ub0b3\uc740 \ud6c4 \ud314\ubc31\uc0ac\uc2ed \ub144\uc744 \uc9c0\ub0b4\uba70 \uc790\ub140\ub4e4\uc744 \ub0b3\uc558\uc73c\uba70 \uadf8\ub294 \uad6c\ubc31\uc2ed \uc138\ub97c \uc0b4\uace0 \uc8fd\uc5c8\ub354\ub77c \ub9c8\ud560\ub784\ub810\uc740 \uc721\uc2ed\uc624 \uc138\uc5d0 \uc57c\ub81b\uc744 \ub0b3\uc558\uace0 \uc57c\ub81b\uc744 \ub0b3\uc740 \ud6c4 \ud314\ubc31\uc0bc\uc2ed \ub144\uc744 \uc9c0\ub0b4\uba70 \uc790\ub140\ub97c \ub0b3\uc558\uc73c\uba70 \uadf8\ub294 \ud314\ubc31\uad6c\uc2ed\uc624 \uc138\ub97c \uc0b4\uace0 \uc8fd\uc5c8\ub354\ub77c \uc57c\ub81b\uc740 \ubc31\uc721\uc2ed\uc774 \uc138\uc5d0 \uc5d0\ub179\uc744 \ub0b3\uc558\uace0 \uc5d0\ub179\uc744 \ub0b3\uc740 \ud6c4 \ud314\ubc31 \ub144\uc744 \uc9c0\ub0b4\uba70 \uc790\ub140\ub4e4\uc744 \ub0b3\uc558\uc73c\uba70 \uadf8\ub294 \uad6c\ubc31\uc721\uc2ed\uc774 \uc138\ub97c \uc0b4\uace0 \uc8fd\uc5c8\ub354\ub77c \uc5d0\ub179\uc740 \uc721\uc2ed\uc624 \uc138\uc5d0 \ubbc0\ub450\uc140\ub77c\ub97c \ub0b3\uc558\uace0 \ubbc0\ub450\uc140\ub77c\ub97c \ub0b3\uc740 \ud6c4 \uc0bc\ubc31 \ub144\uc744 \ud558\ub098\ub2d8\uacfc \ub3d9\ud589\ud558\uba70 \uc790\ub140\ub4e4\uc744 \ub0b3\uc558\uc73c\uba70 \uadf8\ub294 \uc0bc\ubc31\uc721\uc2ed\uc624 \uc138\ub97c \uc0b4\uc558\ub354\ub77c \uc5d0\ub179\uc774 \ud558\ub098\ub2d8\uacfc \ub3d9\ud589\ud558\ub354\ub2c8 \ud558\ub098\ub2d8\uc774 \uadf8\ub97c \ub370\ub824\uac00\uc2dc\ubbc0\ub85c \uc138\uc0c1\uc5d0 \uc788\uc9c0 \uc544\ub2c8\ud558\uc600\ub354\ub77c \ubbc0\ub450\uc140\ub77c\ub294 \ubc31\ud314\uc2ed\uce60 \uc138\uc5d0 \ub77c\uba55\uc744 \ub0b3\uc558\uace0 \ub77c\uba55\uc744 \ub0b3\uc740 \ud6c4 \uce60\ubc31\ud314\uc2ed\uc774 \ub144\uc744 \uc9c0\ub0b4\uba70 \uc790\ub140\ub97c \ub0b3\uc558\uc73c\uba70 \uadf8\ub294 \uad6c\ubc31\uc721\uc2ed\uad6c \uc138\ub97c \uc0b4\uace0 \uc8fd\uc5c8\ub354\ub77c \ub77c\uba55\uc740 \ubc31\ud314\uc2ed\uc774 \uc138\uc5d0 \uc544\ub4e4\uc744 \ub0b3\uace0 \uc774\ub984\uc744 \ub178\uc544 \ub77c \ud558\uc5ec \uc774\ub974\ub418 \uc5ec\ud638\uc640\uaed8\uc11c \ub545\uc744 \uc800\uc8fc\ud558\uc2dc\ubbc0\ub85c \uc218\uace0\ub86d\uac8c \uc77c\ud558\ub294 \uc6b0\ub9ac\ub97c \uc774 \uc544\ub4e4\uc774 \uc548\uc704\ud558\ub9ac\ub77c \ud558\uc600\ub354\ub77c \ub77c\uba55\uc740 \ub178\uc544\ub97c \ub0b3\uc740 \ud6c4 \uc624\ubc31\uad6c\uc2ed\uc624 \ub144\uc744 \uc9c0\ub0b4\uba70 \uc790\ub140\ub4e4\uc744 \ub0b3\uc558\uc73c\uba70", "sentence_answer": "\uc5d0\ub178\uc2a4\ub294 \uad6c\uc2ed \uc138\uc5d0 \uac8c\ub09c\uc744 \ub0b3\uc558\uace0 \uac8c\ub09c\uc744 \ub0b3\uc740 \ud6c4 \ud314\ubc31\uc2ed\uc624 \ub144\uc744 \uc9c0\ub0b4\uba70 \uc790\ub140\ub4e4\uc744 \ub0b3\uc558\uc73c\uba70 \uadf8\ub294 \uad6c\ubc31\uc624 \uc138\ub97c \uc0b4\uace0 \uc8fd\uc5c8\ub354\ub77c \uac8c\ub09c\uc740 \uce60\uc2ed \uc138\uc5d0 \ub9c8\ud560\ub784\ub810\uc744 \ub0b3\uc558\uace0 \ub9c8\ud560\ub784\ub810\uc744 \ub0b3\uc740 \ud6c4 \ud314\ubc31\uc0ac\uc2ed \ub144\uc744 \uc9c0\ub0b4\uba70 \uc790\ub140\ub4e4\uc744 \ub0b3\uc558\uc73c\uba70 \uadf8\ub294 \uad6c\ubc31\uc2ed \uc138\ub97c \uc0b4\uace0 \uc8fd\uc5c8\ub354\ub77c \ub9c8\ud560\ub784\ub810\uc740 \uc721\uc2ed\uc624 \uc138\uc5d0 \uc57c\ub81b\uc744 \ub0b3\uc558\uace0 \uc57c\ub81b\uc744 \ub0b3\uc740 \ud6c4 \ud314\ubc31\uc0bc\uc2ed \ub144\uc744 \uc9c0\ub0b4\uba70 \uc790\ub140\ub97c \ub0b3\uc558\uc73c\uba70 \uadf8\ub294 \ud314\ubc31\uad6c\uc2ed\uc624 \uc138\ub97c \uc0b4\uace0 \uc8fd\uc5c8\ub354\ub77c \uc57c\ub81b\uc740 \ubc31\uc721\uc2ed\uc774 \uc138\uc5d0 \uc5d0\ub179\uc744 \ub0b3\uc558\uace0 \uc5d0\ub179\uc744 \ub0b3\uc740 \ud6c4 \ud314\ubc31 \ub144\uc744 \uc9c0\ub0b4\uba70 \uc790\ub140\ub4e4\uc744 \ub0b3\uc558\uc73c\uba70 \uadf8\ub294 \uad6c\ubc31\uc721\uc2ed\uc774 \uc138\ub97c \uc0b4\uace0 \uc8fd\uc5c8\ub354\ub77c \uc5d0\ub179\uc740 \uc721\uc2ed\uc624 \uc138\uc5d0 \ubbc0\ub450\uc140\ub77c\ub97c \ub0b3\uc558\uace0 \ubbc0\ub450\uc140\ub77c\ub97c \ub0b3\uc740 \ud6c4 \uc0bc\ubc31 \ub144\uc744 \ud558\ub098\ub2d8\uacfc \ub3d9\ud589\ud558\uba70 \uc790\ub140\ub4e4\uc744 \ub0b3\uc558\uc73c\uba70 \uadf8\ub294 \uc0bc\ubc31\uc721\uc2ed\uc624 \uc138\ub97c \uc0b4\uc558\ub354\ub77c \uc5d0\ub179\uc774 \ud558\ub098\ub2d8\uacfc \ub3d9\ud589\ud558\ub354\ub2c8 \ud558\ub098\ub2d8\uc774 \uadf8\ub97c \ub370\ub824\uac00\uc2dc\ubbc0\ub85c \uc138\uc0c1\uc5d0 \uc788\uc9c0 \uc544\ub2c8\ud558\uc600\ub354\ub77c \ubbc0\ub450\uc140\ub77c\ub294 \ubc31\ud314\uc2ed\uce60 \uc138\uc5d0 \ub77c\uba55\uc744 \ub0b3\uc558\uace0 \ub77c\uba55\uc744 \ub0b3\uc740 \ud6c4 \uce60\ubc31\ud314\uc2ed\uc774 \ub144\uc744 \uc9c0\ub0b4\uba70 \uc790\ub140\ub97c \ub0b3\uc558\uc73c\uba70 \uadf8\ub294 \uad6c\ubc31\uc721\uc2ed\uad6c \uc138\ub97c \uc0b4\uace0 \uc8fd\uc5c8\ub354\ub77c \ub77c\uba55\uc740 \ubc31\ud314\uc2ed\uc774 \uc138\uc5d0 \uc544\ub4e4\uc744 \ub0b3\uace0 \uc774\ub984\uc744 \ub178\uc544 \ub77c \ud558\uc5ec \uc774\ub974\ub418 \uc5ec\ud638\uc640\uaed8\uc11c \ub545\uc744 \uc800\uc8fc\ud558\uc2dc\ubbc0\ub85c \uc218\uace0\ub86d\uac8c \uc77c\ud558\ub294 \uc6b0\ub9ac\ub97c \uc774 \uc544\ub4e4\uc774 \uc548\uc704\ud558\ub9ac\ub77c \ud558\uc600\ub354\ub77c \ub77c\uba55\uc740 \ub178\uc544\ub97c \ub0b3\uc740 \ud6c4 \uc624\ubc31\uad6c\uc2ed\uc624 \ub144\uc744 \uc9c0\ub0b4\uba70 \uc790\ub140\ub4e4\uc744 \ub0b3\uc558\uc73c\uba70", "paragraph_id": "6026533-0-1", "paragraph_question": "question: \ub77c\uba55\uc774 \uc218\uace0\ub86d\uac8c \uc77c\ud558\ub294 \uc6b0\ub9ac\ub97c \uc774 \uc544\ub4e4\uc774 \uc548\uc704\ud558\ub9ac\ub77c\ud558\uace0 \uc9c0\uc740 \uc544\ub4e4 \uc774\ub984\uc740?, context: \uc5d0\ub178\uc2a4\ub294 \uad6c\uc2ed \uc138\uc5d0 \uac8c\ub09c\uc744 \ub0b3\uc558\uace0 \uac8c\ub09c\uc744 \ub0b3\uc740 \ud6c4 \ud314\ubc31\uc2ed\uc624 \ub144\uc744 \uc9c0\ub0b4\uba70 \uc790\ub140\ub4e4\uc744 \ub0b3\uc558\uc73c\uba70 \uadf8\ub294 \uad6c\ubc31\uc624 \uc138\ub97c \uc0b4\uace0 \uc8fd\uc5c8\ub354\ub77c \uac8c\ub09c\uc740 \uce60\uc2ed \uc138\uc5d0 \ub9c8\ud560\ub784\ub810\uc744 \ub0b3\uc558\uace0 \ub9c8\ud560\ub784\ub810\uc744 \ub0b3\uc740 \ud6c4 \ud314\ubc31\uc0ac\uc2ed \ub144\uc744 \uc9c0\ub0b4\uba70 \uc790\ub140\ub4e4\uc744 \ub0b3\uc558\uc73c\uba70 \uadf8\ub294 \uad6c\ubc31\uc2ed \uc138\ub97c \uc0b4\uace0 \uc8fd\uc5c8\ub354\ub77c \ub9c8\ud560\ub784\ub810\uc740 \uc721\uc2ed\uc624 \uc138\uc5d0 \uc57c\ub81b\uc744 \ub0b3\uc558\uace0 \uc57c\ub81b\uc744 \ub0b3\uc740 \ud6c4 \ud314\ubc31\uc0bc\uc2ed \ub144\uc744 \uc9c0\ub0b4\uba70 \uc790\ub140\ub97c \ub0b3\uc558\uc73c\uba70 \uadf8\ub294 \ud314\ubc31\uad6c\uc2ed\uc624 \uc138\ub97c \uc0b4\uace0 \uc8fd\uc5c8\ub354\ub77c \uc57c\ub81b\uc740 \ubc31\uc721\uc2ed\uc774 \uc138\uc5d0 \uc5d0\ub179\uc744 \ub0b3\uc558\uace0 \uc5d0\ub179\uc744 \ub0b3\uc740 \ud6c4 \ud314\ubc31 \ub144\uc744 \uc9c0\ub0b4\uba70 \uc790\ub140\ub4e4\uc744 \ub0b3\uc558\uc73c\uba70 \uadf8\ub294 \uad6c\ubc31\uc721\uc2ed\uc774 \uc138\ub97c \uc0b4\uace0 \uc8fd\uc5c8\ub354\ub77c \uc5d0\ub179\uc740 \uc721\uc2ed\uc624 \uc138\uc5d0 \ubbc0\ub450\uc140\ub77c\ub97c \ub0b3\uc558\uace0 \ubbc0\ub450\uc140\ub77c\ub97c \ub0b3\uc740 \ud6c4 \uc0bc\ubc31 \ub144\uc744 \ud558\ub098\ub2d8\uacfc \ub3d9\ud589\ud558\uba70 \uc790\ub140\ub4e4\uc744 \ub0b3\uc558\uc73c\uba70 \uadf8\ub294 \uc0bc\ubc31\uc721\uc2ed\uc624 \uc138\ub97c \uc0b4\uc558\ub354\ub77c \uc5d0\ub179\uc774 \ud558\ub098\ub2d8\uacfc \ub3d9\ud589\ud558\ub354\ub2c8 \ud558\ub098\ub2d8\uc774 \uadf8\ub97c \ub370\ub824\uac00\uc2dc\ubbc0\ub85c \uc138\uc0c1\uc5d0 \uc788\uc9c0 \uc544\ub2c8\ud558\uc600\ub354\ub77c \ubbc0\ub450\uc140\ub77c\ub294 \ubc31\ud314\uc2ed\uce60 \uc138\uc5d0 \ub77c\uba55\uc744 \ub0b3\uc558\uace0 \ub77c\uba55\uc744 \ub0b3\uc740 \ud6c4 \uce60\ubc31\ud314\uc2ed\uc774 \ub144\uc744 \uc9c0\ub0b4\uba70 \uc790\ub140\ub97c \ub0b3\uc558\uc73c\uba70 \uadf8\ub294 \uad6c\ubc31\uc721\uc2ed\uad6c \uc138\ub97c \uc0b4\uace0 \uc8fd\uc5c8\ub354\ub77c \ub77c\uba55\uc740 \ubc31\ud314\uc2ed\uc774 \uc138\uc5d0 \uc544\ub4e4\uc744 \ub0b3\uace0 \uc774\ub984\uc744 \ub178\uc544\ub77c \ud558\uc5ec \uc774\ub974\ub418 \uc5ec\ud638\uc640\uaed8\uc11c \ub545\uc744 \uc800\uc8fc\ud558\uc2dc\ubbc0\ub85c \uc218\uace0\ub86d\uac8c \uc77c\ud558\ub294 \uc6b0\ub9ac\ub97c \uc774 \uc544\ub4e4\uc774 \uc548\uc704\ud558\ub9ac\ub77c \ud558\uc600\ub354\ub77c \ub77c\uba55\uc740 \ub178\uc544\ub97c \ub0b3\uc740 \ud6c4 \uc624\ubc31\uad6c\uc2ed\uc624 \ub144\uc744 \uc9c0\ub0b4\uba70 \uc790\ub140\ub4e4\uc744 \ub0b3\uc558\uc73c\uba70"} {"question": "\ubc15\ud0dc\ud658\uc774 \ud22c\uc57d\ud55c \uac83\uc73c\ub85c \ub4dc\ub7ec\ub09c \uc57d\ubb3c\uc740?", "paragraph": "2014\ub144 \uc778\ucc9c \uc544\uc2dc\uc548\uac8c\uc784 \uc774\uc804 \uc2a4\ud14c\ub85c\uc774\ub4dc \uacc4\uc5f4\uc758 \uc57d\ubb3c\uc778 \ub124\ube44\ub3c4\ub97c \ud22c\uc57d\ud55c \uac83\uc73c\ub85c \ub4dc\ub7ec\ub098 \ud070 \ub17c\ub780\uc744 \ubd88\ub7ec\uc77c\uc73c\ucf30\uc73c\uba70 \uc774\ub85c\uc368 \uc778\ucc9c \uc544\uc2dc\uc548\uac8c\uc784\uc5d0\uc11c \ud68d\ub4dd\ud55c \ubaa8\ub4e0 \uba54\ub2ec\uc774 \ubc15\ud0c8\ub418\uc5c8\ub2e4. \uc9d5\uacc4\uae30\uac04\uc740 2014\ub144 9\uc6d4 3\uc77c\ubd80\ud130 2016\ub144 3\uc6d4 2\uc77c\uae4c\uc9c0\uc774\uba70 \uc790\uaca9\uc815\uc9c0\ub85c \uc778\ud574 \uc120\uc218\uc790\uaca9\uc73c\ub85c \ud6c8\ub828\uc744 \ud560 \uc218 \uc5c6\uc5b4 \uc77c\ubc18\uc778 \uc790\uaca9\uc73c\ub85c \ub178\ubbfc\uc0c1 \uc218\uc601\uad50\uc2e4\uc5d0 \ub4f1\ub85d\ud574 \uc11c\uc6b8 \ubc29\uc774\ub3d9 \uc62c\ub9bc\ud53d\uacf5\uc6d0 \uc62c\ub9bc\ud53d\uc218\uc601\uc7a5\uc5d0\uc11c \ud6c8\ub828\uc744 \uc7ac\uac1c\ud588\ub2e4. \uadf8\ub7ec\ub098 \ub300\ud55c\uccb4\uc721\ud68c \uaddc\uc815\uc0c1 \uae08\uc9c0 \uc57d\ubb3c \uc9d5\uacc4\ub97c \ubc1b\uc740 \uc120\uc218\ub294 \uc9d5\uacc4 \ub9cc\ub8cc\uc77c\ub85c\ubd80\ud130 3\ub144 \uac04 \ub300\ud45c \uc120\uc218\ub85c \ub6f8 \uc218 \uc5c6\uae30 \ub54c\ubb38\uc5d0 \ud604 \ub300\ud55c\uccb4\uc721\ud68c \uaddc\uc815\uc0c1 2019\ub144\uae4c\uc9c0 \ub300\ud55c\ubbfc\uad6d \uad6d\uac00\ub300\ud45c\ud300 \ubc1c\ud0c1\uc774 \ubd88\uac00\ub2a5\ud574 \ub9ac\uc6b0 \uc62c\ub9bc\ud53d\uc5d0 \ucd9c\uc804\ud560 \uc218 \uc5c6\uc5c8\ub2e4. \uc774\uc911\ucc98\ubc8c \ub17c\ub780\uc774 \uc77c\uc790 \ub300\ud55c\uccb4\uc721\ud68c\ub294 \uad6d\ubbfc\uc0dd\ud65c\uccb4\uc721\ud68c\uc640 \ud1b5\ud569 \uc791\uc5c5\uc774 \ub9c8\ubb34\ub9ac\ub418\ub294 2016\ub144 3\uc6d4 \uc774\ud6c4 \ubc95\ub960 \uac80\ud1a0\ub97c \uac70\uccd0 \ubc15\ud0dc\ud658\uc758 \ucd9c\uc804\uc744 \ud5c8\uc6a9\ud560 \uc9c0 \uc5ec\ubd80\ub97c \uacb0\uc815\ud558\uae30\ub85c \ud588\ub2e4. \ubc95\uc6d0\uc740 \ub300\ud55c\uccb4\uc721\ud68c\uc640 \uc218\uc601\uc5f0\ub9f9\uc744 \uc0c1\ub300\ub85c \ub0b8 \uac00\ucc98\ubd84\uc2e0\uccad\uc744 \ubc1b\uc544\ub4e4\uc600\ub2e4. \ubc15\ud0dc\ud658\uc740 \uad6d\uc81c\uc2a4\ud3ec\uce20 \uc911\uc7ac\uc7ac\ud310\uc18c(CAS)\uc5d0 \ub300\ud55c\uccb4\uc721\ud68c\uc640 \uc218\uc601\uc5f0\ub9f9\uc744 \uc0c1\ub300\ub85c \uc774\uc911\ucc98\ubc8c \ucca0\ud68c\uc5d0 \ub300\ud55c \uc18c\uc1a1\uc744 \uac78\uc5c8\uace0 \uacb0\uad6d \uad6d\uc81c\uc2a4\ud3ec\uce20 \uc911\uc7ac\uc7ac\ud310\uc18c(CAS)\uac00 \ubc15\ud0dc\ud658\uc758 \uc190\uc744 \ub4e4\uc5b4\uc8fc\uba74\uc11c \ubc15\ud0dc\ud658\uc740 \uc6b0\uc5ec\uace1\uc808 \ub05d\uc5d0 \ub9ac\uc6b0 \uc62c\ub9bc\ud53d\uc5d0 \ucd9c\uc804\ud560 \uc218 \uc788\uac8c \ub418\uc5c8\ub2e4.", "answer": "\ub124\ube44\ub3c4", "sentence": "2014\ub144 \uc778\ucc9c \uc544\uc2dc\uc548\uac8c\uc784 \uc774\uc804 \uc2a4\ud14c\ub85c\uc774\ub4dc \uacc4\uc5f4\uc758 \uc57d\ubb3c\uc778 \ub124\ube44\ub3c4 \ub97c \ud22c\uc57d\ud55c \uac83\uc73c\ub85c \ub4dc\ub7ec\ub098 \ud070 \ub17c\ub780\uc744 \ubd88\ub7ec\uc77c\uc73c\ucf30\uc73c\uba70 \uc774\ub85c\uc368 \uc778\ucc9c \uc544\uc2dc\uc548\uac8c\uc784\uc5d0\uc11c \ud68d\ub4dd\ud55c \ubaa8\ub4e0 \uba54\ub2ec\uc774 \ubc15\ud0c8\ub418\uc5c8\ub2e4.", "paragraph_sentence": " 2014\ub144 \uc778\ucc9c \uc544\uc2dc\uc548\uac8c\uc784 \uc774\uc804 \uc2a4\ud14c\ub85c\uc774\ub4dc \uacc4\uc5f4\uc758 \uc57d\ubb3c\uc778 \ub124\ube44\ub3c4 \ub97c \ud22c\uc57d\ud55c \uac83\uc73c\ub85c \ub4dc\ub7ec\ub098 \ud070 \ub17c\ub780\uc744 \ubd88\ub7ec\uc77c\uc73c\ucf30\uc73c\uba70 \uc774\ub85c\uc368 \uc778\ucc9c \uc544\uc2dc\uc548\uac8c\uc784\uc5d0\uc11c \ud68d\ub4dd\ud55c \ubaa8\ub4e0 \uba54\ub2ec\uc774 \ubc15\ud0c8\ub418\uc5c8\ub2e4. \uc9d5\uacc4\uae30\uac04\uc740 2014\ub144 9\uc6d4 3\uc77c\ubd80\ud130 2016\ub144 3\uc6d4 2\uc77c\uae4c\uc9c0\uc774\uba70 \uc790\uaca9\uc815\uc9c0\ub85c \uc778\ud574 \uc120\uc218\uc790\uaca9\uc73c\ub85c \ud6c8\ub828\uc744 \ud560 \uc218 \uc5c6\uc5b4 \uc77c\ubc18\uc778 \uc790\uaca9\uc73c\ub85c \ub178\ubbfc\uc0c1 \uc218\uc601\uad50\uc2e4\uc5d0 \ub4f1\ub85d\ud574 \uc11c\uc6b8 \ubc29\uc774\ub3d9 \uc62c\ub9bc\ud53d\uacf5\uc6d0 \uc62c\ub9bc\ud53d\uc218\uc601\uc7a5\uc5d0\uc11c \ud6c8\ub828\uc744 \uc7ac\uac1c\ud588\ub2e4. \uadf8\ub7ec\ub098 \ub300\ud55c\uccb4\uc721\ud68c \uaddc\uc815\uc0c1 \uae08\uc9c0 \uc57d\ubb3c \uc9d5\uacc4\ub97c \ubc1b\uc740 \uc120\uc218\ub294 \uc9d5\uacc4 \ub9cc\ub8cc\uc77c\ub85c\ubd80\ud130 3\ub144 \uac04 \ub300\ud45c \uc120\uc218\ub85c \ub6f8 \uc218 \uc5c6\uae30 \ub54c\ubb38\uc5d0 \ud604 \ub300\ud55c\uccb4\uc721\ud68c \uaddc\uc815\uc0c1 2019\ub144\uae4c\uc9c0 \ub300\ud55c\ubbfc\uad6d \uad6d\uac00\ub300\ud45c\ud300 \ubc1c\ud0c1\uc774 \ubd88\uac00\ub2a5\ud574 \ub9ac\uc6b0 \uc62c\ub9bc\ud53d\uc5d0 \ucd9c\uc804\ud560 \uc218 \uc5c6\uc5c8\ub2e4. \uc774\uc911\ucc98\ubc8c \ub17c\ub780\uc774 \uc77c\uc790 \ub300\ud55c\uccb4\uc721\ud68c\ub294 \uad6d\ubbfc\uc0dd\ud65c\uccb4\uc721\ud68c\uc640 \ud1b5\ud569 \uc791\uc5c5\uc774 \ub9c8\ubb34\ub9ac\ub418\ub294 2016\ub144 3\uc6d4 \uc774\ud6c4 \ubc95\ub960 \uac80\ud1a0\ub97c \uac70\uccd0 \ubc15\ud0dc\ud658\uc758 \ucd9c\uc804\uc744 \ud5c8\uc6a9\ud560 \uc9c0 \uc5ec\ubd80\ub97c \uacb0\uc815\ud558\uae30\ub85c \ud588\ub2e4. \ubc95\uc6d0\uc740 \ub300\ud55c\uccb4\uc721\ud68c\uc640 \uc218\uc601\uc5f0\ub9f9\uc744 \uc0c1\ub300\ub85c \ub0b8 \uac00\ucc98\ubd84\uc2e0\uccad\uc744 \ubc1b\uc544\ub4e4\uc600\ub2e4. \ubc15\ud0dc\ud658\uc740 \uad6d\uc81c\uc2a4\ud3ec\uce20 \uc911\uc7ac\uc7ac\ud310\uc18c(CAS)\uc5d0 \ub300\ud55c\uccb4\uc721\ud68c\uc640 \uc218\uc601\uc5f0\ub9f9\uc744 \uc0c1\ub300\ub85c \uc774\uc911\ucc98\ubc8c \ucca0\ud68c\uc5d0 \ub300\ud55c \uc18c\uc1a1\uc744 \uac78\uc5c8\uace0 \uacb0\uad6d \uad6d\uc81c\uc2a4\ud3ec\uce20 \uc911\uc7ac\uc7ac\ud310\uc18c(CAS)\uac00 \ubc15\ud0dc\ud658\uc758 \uc190\uc744 \ub4e4\uc5b4\uc8fc\uba74\uc11c \ubc15\ud0dc\ud658\uc740 \uc6b0\uc5ec\uace1\uc808 \ub05d\uc5d0 \ub9ac\uc6b0 \uc62c\ub9bc\ud53d\uc5d0 \ucd9c\uc804\ud560 \uc218 \uc788\uac8c \ub418\uc5c8\ub2e4.", "paragraph_answer": "2014\ub144 \uc778\ucc9c \uc544\uc2dc\uc548\uac8c\uc784 \uc774\uc804 \uc2a4\ud14c\ub85c\uc774\ub4dc \uacc4\uc5f4\uc758 \uc57d\ubb3c\uc778 \ub124\ube44\ub3c4 \ub97c \ud22c\uc57d\ud55c \uac83\uc73c\ub85c \ub4dc\ub7ec\ub098 \ud070 \ub17c\ub780\uc744 \ubd88\ub7ec\uc77c\uc73c\ucf30\uc73c\uba70 \uc774\ub85c\uc368 \uc778\ucc9c \uc544\uc2dc\uc548\uac8c\uc784\uc5d0\uc11c \ud68d\ub4dd\ud55c \ubaa8\ub4e0 \uba54\ub2ec\uc774 \ubc15\ud0c8\ub418\uc5c8\ub2e4. \uc9d5\uacc4\uae30\uac04\uc740 2014\ub144 9\uc6d4 3\uc77c\ubd80\ud130 2016\ub144 3\uc6d4 2\uc77c\uae4c\uc9c0\uc774\uba70 \uc790\uaca9\uc815\uc9c0\ub85c \uc778\ud574 \uc120\uc218\uc790\uaca9\uc73c\ub85c \ud6c8\ub828\uc744 \ud560 \uc218 \uc5c6\uc5b4 \uc77c\ubc18\uc778 \uc790\uaca9\uc73c\ub85c \ub178\ubbfc\uc0c1 \uc218\uc601\uad50\uc2e4\uc5d0 \ub4f1\ub85d\ud574 \uc11c\uc6b8 \ubc29\uc774\ub3d9 \uc62c\ub9bc\ud53d\uacf5\uc6d0 \uc62c\ub9bc\ud53d\uc218\uc601\uc7a5\uc5d0\uc11c \ud6c8\ub828\uc744 \uc7ac\uac1c\ud588\ub2e4. \uadf8\ub7ec\ub098 \ub300\ud55c\uccb4\uc721\ud68c \uaddc\uc815\uc0c1 \uae08\uc9c0 \uc57d\ubb3c \uc9d5\uacc4\ub97c \ubc1b\uc740 \uc120\uc218\ub294 \uc9d5\uacc4 \ub9cc\ub8cc\uc77c\ub85c\ubd80\ud130 3\ub144 \uac04 \ub300\ud45c \uc120\uc218\ub85c \ub6f8 \uc218 \uc5c6\uae30 \ub54c\ubb38\uc5d0 \ud604 \ub300\ud55c\uccb4\uc721\ud68c \uaddc\uc815\uc0c1 2019\ub144\uae4c\uc9c0 \ub300\ud55c\ubbfc\uad6d \uad6d\uac00\ub300\ud45c\ud300 \ubc1c\ud0c1\uc774 \ubd88\uac00\ub2a5\ud574 \ub9ac\uc6b0 \uc62c\ub9bc\ud53d\uc5d0 \ucd9c\uc804\ud560 \uc218 \uc5c6\uc5c8\ub2e4. \uc774\uc911\ucc98\ubc8c \ub17c\ub780\uc774 \uc77c\uc790 \ub300\ud55c\uccb4\uc721\ud68c\ub294 \uad6d\ubbfc\uc0dd\ud65c\uccb4\uc721\ud68c\uc640 \ud1b5\ud569 \uc791\uc5c5\uc774 \ub9c8\ubb34\ub9ac\ub418\ub294 2016\ub144 3\uc6d4 \uc774\ud6c4 \ubc95\ub960 \uac80\ud1a0\ub97c \uac70\uccd0 \ubc15\ud0dc\ud658\uc758 \ucd9c\uc804\uc744 \ud5c8\uc6a9\ud560 \uc9c0 \uc5ec\ubd80\ub97c \uacb0\uc815\ud558\uae30\ub85c \ud588\ub2e4. \ubc95\uc6d0\uc740 \ub300\ud55c\uccb4\uc721\ud68c\uc640 \uc218\uc601\uc5f0\ub9f9\uc744 \uc0c1\ub300\ub85c \ub0b8 \uac00\ucc98\ubd84\uc2e0\uccad\uc744 \ubc1b\uc544\ub4e4\uc600\ub2e4. \ubc15\ud0dc\ud658\uc740 \uad6d\uc81c\uc2a4\ud3ec\uce20 \uc911\uc7ac\uc7ac\ud310\uc18c(CAS)\uc5d0 \ub300\ud55c\uccb4\uc721\ud68c\uc640 \uc218\uc601\uc5f0\ub9f9\uc744 \uc0c1\ub300\ub85c \uc774\uc911\ucc98\ubc8c \ucca0\ud68c\uc5d0 \ub300\ud55c \uc18c\uc1a1\uc744 \uac78\uc5c8\uace0 \uacb0\uad6d \uad6d\uc81c\uc2a4\ud3ec\uce20 \uc911\uc7ac\uc7ac\ud310\uc18c(CAS)\uac00 \ubc15\ud0dc\ud658\uc758 \uc190\uc744 \ub4e4\uc5b4\uc8fc\uba74\uc11c \ubc15\ud0dc\ud658\uc740 \uc6b0\uc5ec\uace1\uc808 \ub05d\uc5d0 \ub9ac\uc6b0 \uc62c\ub9bc\ud53d\uc5d0 \ucd9c\uc804\ud560 \uc218 \uc788\uac8c \ub418\uc5c8\ub2e4.", "sentence_answer": "2014\ub144 \uc778\ucc9c \uc544\uc2dc\uc548\uac8c\uc784 \uc774\uc804 \uc2a4\ud14c\ub85c\uc774\ub4dc \uacc4\uc5f4\uc758 \uc57d\ubb3c\uc778 \ub124\ube44\ub3c4 \ub97c \ud22c\uc57d\ud55c \uac83\uc73c\ub85c \ub4dc\ub7ec\ub098 \ud070 \ub17c\ub780\uc744 \ubd88\ub7ec\uc77c\uc73c\ucf30\uc73c\uba70 \uc774\ub85c\uc368 \uc778\ucc9c \uc544\uc2dc\uc548\uac8c\uc784\uc5d0\uc11c \ud68d\ub4dd\ud55c \ubaa8\ub4e0 \uba54\ub2ec\uc774 \ubc15\ud0c8\ub418\uc5c8\ub2e4.", "paragraph_id": "6530244-2-0", "paragraph_question": "question: \ubc15\ud0dc\ud658\uc774 \ud22c\uc57d\ud55c \uac83\uc73c\ub85c \ub4dc\ub7ec\ub09c \uc57d\ubb3c\uc740?, context: 2014\ub144 \uc778\ucc9c \uc544\uc2dc\uc548\uac8c\uc784 \uc774\uc804 \uc2a4\ud14c\ub85c\uc774\ub4dc \uacc4\uc5f4\uc758 \uc57d\ubb3c\uc778 \ub124\ube44\ub3c4\ub97c \ud22c\uc57d\ud55c \uac83\uc73c\ub85c \ub4dc\ub7ec\ub098 \ud070 \ub17c\ub780\uc744 \ubd88\ub7ec\uc77c\uc73c\ucf30\uc73c\uba70 \uc774\ub85c\uc368 \uc778\ucc9c \uc544\uc2dc\uc548\uac8c\uc784\uc5d0\uc11c \ud68d\ub4dd\ud55c \ubaa8\ub4e0 \uba54\ub2ec\uc774 \ubc15\ud0c8\ub418\uc5c8\ub2e4. \uc9d5\uacc4\uae30\uac04\uc740 2014\ub144 9\uc6d4 3\uc77c\ubd80\ud130 2016\ub144 3\uc6d4 2\uc77c\uae4c\uc9c0\uc774\uba70 \uc790\uaca9\uc815\uc9c0\ub85c \uc778\ud574 \uc120\uc218\uc790\uaca9\uc73c\ub85c \ud6c8\ub828\uc744 \ud560 \uc218 \uc5c6\uc5b4 \uc77c\ubc18\uc778 \uc790\uaca9\uc73c\ub85c \ub178\ubbfc\uc0c1 \uc218\uc601\uad50\uc2e4\uc5d0 \ub4f1\ub85d\ud574 \uc11c\uc6b8 \ubc29\uc774\ub3d9 \uc62c\ub9bc\ud53d\uacf5\uc6d0 \uc62c\ub9bc\ud53d\uc218\uc601\uc7a5\uc5d0\uc11c \ud6c8\ub828\uc744 \uc7ac\uac1c\ud588\ub2e4. \uadf8\ub7ec\ub098 \ub300\ud55c\uccb4\uc721\ud68c \uaddc\uc815\uc0c1 \uae08\uc9c0 \uc57d\ubb3c \uc9d5\uacc4\ub97c \ubc1b\uc740 \uc120\uc218\ub294 \uc9d5\uacc4 \ub9cc\ub8cc\uc77c\ub85c\ubd80\ud130 3\ub144 \uac04 \ub300\ud45c \uc120\uc218\ub85c \ub6f8 \uc218 \uc5c6\uae30 \ub54c\ubb38\uc5d0 \ud604 \ub300\ud55c\uccb4\uc721\ud68c \uaddc\uc815\uc0c1 2019\ub144\uae4c\uc9c0 \ub300\ud55c\ubbfc\uad6d \uad6d\uac00\ub300\ud45c\ud300 \ubc1c\ud0c1\uc774 \ubd88\uac00\ub2a5\ud574 \ub9ac\uc6b0 \uc62c\ub9bc\ud53d\uc5d0 \ucd9c\uc804\ud560 \uc218 \uc5c6\uc5c8\ub2e4. \uc774\uc911\ucc98\ubc8c \ub17c\ub780\uc774 \uc77c\uc790 \ub300\ud55c\uccb4\uc721\ud68c\ub294 \uad6d\ubbfc\uc0dd\ud65c\uccb4\uc721\ud68c\uc640 \ud1b5\ud569 \uc791\uc5c5\uc774 \ub9c8\ubb34\ub9ac\ub418\ub294 2016\ub144 3\uc6d4 \uc774\ud6c4 \ubc95\ub960 \uac80\ud1a0\ub97c \uac70\uccd0 \ubc15\ud0dc\ud658\uc758 \ucd9c\uc804\uc744 \ud5c8\uc6a9\ud560 \uc9c0 \uc5ec\ubd80\ub97c \uacb0\uc815\ud558\uae30\ub85c \ud588\ub2e4. \ubc95\uc6d0\uc740 \ub300\ud55c\uccb4\uc721\ud68c\uc640 \uc218\uc601\uc5f0\ub9f9\uc744 \uc0c1\ub300\ub85c \ub0b8 \uac00\ucc98\ubd84\uc2e0\uccad\uc744 \ubc1b\uc544\ub4e4\uc600\ub2e4. \ubc15\ud0dc\ud658\uc740 \uad6d\uc81c\uc2a4\ud3ec\uce20 \uc911\uc7ac\uc7ac\ud310\uc18c(CAS)\uc5d0 \ub300\ud55c\uccb4\uc721\ud68c\uc640 \uc218\uc601\uc5f0\ub9f9\uc744 \uc0c1\ub300\ub85c \uc774\uc911\ucc98\ubc8c \ucca0\ud68c\uc5d0 \ub300\ud55c \uc18c\uc1a1\uc744 \uac78\uc5c8\uace0 \uacb0\uad6d \uad6d\uc81c\uc2a4\ud3ec\uce20 \uc911\uc7ac\uc7ac\ud310\uc18c(CAS)\uac00 \ubc15\ud0dc\ud658\uc758 \uc190\uc744 \ub4e4\uc5b4\uc8fc\uba74\uc11c \ubc15\ud0dc\ud658\uc740 \uc6b0\uc5ec\uace1\uc808 \ub05d\uc5d0 \ub9ac\uc6b0 \uc62c\ub9bc\ud53d\uc5d0 \ucd9c\uc804\ud560 \uc218 \uc788\uac8c \ub418\uc5c8\ub2e4."} {"question": "\uace0\uc740\uc758 \uc2dc\ub294 \uba87\uac1c\uad6d\uc5d0 \ubc88\uc5ed\ub418\uc5b4 \ub9ce\uc740 \uc601\uac10\uc744 \uc8fc\uace0 \uc788\ub098?", "paragraph": "\uadf8\ub294 \ub178\ubca8 \ubb38\ud559\uc0c1 \ud6c4\ubcf4\ub85c \uc790\uc8fc \uac70\ub860\ub418\uace0 \uc788\ub294\ub370, \uc138\uacc4 \uac01\uad6d\uc5d0\uc11c \uadf8\uac00 \uc798 \uc54c\ub824\uc9c4 \uac83\uc740 \ub178\ubca8\ubb38\ud559\uc0c1 \ud6c4\ubcf4\ub77c\ub294 \uac83 \uc774\uc804\uc5d0 \uadf8\uc758 \uc2dc\uac00 20\uc5ec\uac1c\uad6d\uc5d0 \ubc88\uc5ed\ub418\uc5b4 \ub9ce\uc740 \uc601\uac10\uc744 \uc8fc\uace0 \uc788\ub2e4\ub294 \uc810 \ub54c\ubb38\uc774\ub2e4. \uc2e4\uc81c\ub85c 2005\ub144 \uc774\ud6c4 \uc5ec\ub7ec\ubc88 \ub178\ubca8\ubb38\ud559\uc0c1 \ud6c4\ubcf4\ub85c \uac70\ub860 \ub418\uae30\ub3c4 \ud558\uc600\ub2e4. \uadf8\uc758 \uc800\uba85\uc131\uc740 \ub9e4\ub144 \uc5ec\ub7ec \ub098\ub77c\uc5d0\uc11c \ucd08\uccad\ud558\ub294 \ub4f1 \uadf8\uc758 \uc2dc\uac00 \uad6d\uacbd\uc744 \ucd08\uc6d4\ud558\uace0 \uc788\ub2e4\ub294 \uc810\uc5d0\uc11c \ubcf4\uc5ec\uc8fc\uace0 \uc788\ub2e4. \ud2b9\ud788 \ubd81\uc720\ub7fd\uc5d0\uc11c \uadf8\uc758 \uc2dc\ub294 \ub192\uc740 \ud3c9\uac00\ub97c \ubc1b\ub294\ub2e4. \ub178\ub974\uc6e8\uc774\uc5d0\uc11c\ub294 \uc790\uad6d\uc758 \ub178\ubca8 \ubb38\ud559\uc0c1 \uc218\uc0c1\uc790 \ube44\uc5d0\ub978\uc2a4\ud2f0\uc5d0\ub974\ub124 \ube44\uc5d0\ub978\uc190\uc744 \uae30\ub9b0 \ube44\uc5d0\ub978\uc190 \ud6c8\uc7a5\uc744 \ubc1b\uc558\uace0, \uc2a4\uc6e8\ub374\uc5d0\uc11c\ub3c4 \uc790\uad6d \ub178\ubca8 \ubb38\ud559\uc0c1 \uc218\uc0c1\uc790 \ud558\ub93c \ub9c8\ub974\ud2f4\uc190\uc744 \uae30\ub9ac\ub294 \uc0c1\uc778 \ub3d9\uc2dc\uc5d0 \ub3d9\uc544\uc2dc\uc544\uad8c \ubb38\ud559\uc5d0 \uc8fc\ub294 \uc0c1\uc778 \uc2dc\uce74\ub2e4\uc0c1\uc744 \uc218\uc5ec\ud558\uc600\ub2e4. \uc2a4\uc6e8\ub374\uc758 \ud55c \uc5b8\ub860\ub9e4\uccb4 \uae30\uc0ac\uc5d0\uc11c\ub294 \uadf8\ub97c \uad70\uc0b0\uc758 \uc81c\uc655\uc774\ub77c \uce6d\ud560 \uc815\ub3c4\uc600\ub2e4.", "answer": "20\uc5ec\uac1c\uad6d", "sentence": "\uc138\uacc4 \uac01\uad6d\uc5d0\uc11c \uadf8\uac00 \uc798 \uc54c\ub824\uc9c4 \uac83\uc740 \ub178\ubca8\ubb38\ud559\uc0c1 \ud6c4\ubcf4\ub77c\ub294 \uac83 \uc774\uc804\uc5d0 \uadf8\uc758 \uc2dc\uac00 20\uc5ec\uac1c\uad6d \uc5d0 \ubc88\uc5ed\ub418\uc5b4 \ub9ce\uc740 \uc601\uac10\uc744 \uc8fc\uace0 \uc788\ub2e4\ub294 \uc810 \ub54c\ubb38\uc774\ub2e4.", "paragraph_sentence": "\uadf8\ub294 \ub178\ubca8 \ubb38\ud559\uc0c1 \ud6c4\ubcf4\ub85c \uc790\uc8fc \uac70\ub860\ub418\uace0 \uc788\ub294\ub370, \uc138\uacc4 \uac01\uad6d\uc5d0\uc11c \uadf8\uac00 \uc798 \uc54c\ub824\uc9c4 \uac83\uc740 \ub178\ubca8\ubb38\ud559\uc0c1 \ud6c4\ubcf4\ub77c\ub294 \uac83 \uc774\uc804\uc5d0 \uadf8\uc758 \uc2dc\uac00 20\uc5ec\uac1c\uad6d \uc5d0 \ubc88\uc5ed\ub418\uc5b4 \ub9ce\uc740 \uc601\uac10\uc744 \uc8fc\uace0 \uc788\ub2e4\ub294 \uc810 \ub54c\ubb38\uc774\ub2e4. \uc2e4\uc81c\ub85c 2005\ub144 \uc774\ud6c4 \uc5ec\ub7ec\ubc88 \ub178\ubca8\ubb38\ud559\uc0c1 \ud6c4\ubcf4\ub85c \uac70\ub860 \ub418\uae30\ub3c4 \ud558\uc600\ub2e4. \uadf8\uc758 \uc800\uba85\uc131\uc740 \ub9e4\ub144 \uc5ec\ub7ec \ub098\ub77c\uc5d0\uc11c \ucd08\uccad\ud558\ub294 \ub4f1 \uadf8\uc758 \uc2dc\uac00 \uad6d\uacbd\uc744 \ucd08\uc6d4\ud558\uace0 \uc788\ub2e4\ub294 \uc810\uc5d0\uc11c \ubcf4\uc5ec\uc8fc\uace0 \uc788\ub2e4. \ud2b9\ud788 \ubd81\uc720\ub7fd\uc5d0\uc11c \uadf8\uc758 \uc2dc\ub294 \ub192\uc740 \ud3c9\uac00\ub97c \ubc1b\ub294\ub2e4. \ub178\ub974\uc6e8\uc774\uc5d0\uc11c\ub294 \uc790\uad6d\uc758 \ub178\ubca8 \ubb38\ud559\uc0c1 \uc218\uc0c1\uc790 \ube44\uc5d0\ub978\uc2a4\ud2f0\uc5d0\ub974\ub124 \ube44\uc5d0\ub978\uc190\uc744 \uae30\ub9b0 \ube44\uc5d0\ub978\uc190 \ud6c8\uc7a5\uc744 \ubc1b\uc558\uace0, \uc2a4\uc6e8\ub374\uc5d0\uc11c\ub3c4 \uc790\uad6d \ub178\ubca8 \ubb38\ud559\uc0c1 \uc218\uc0c1\uc790 \ud558\ub93c \ub9c8\ub974\ud2f4\uc190\uc744 \uae30\ub9ac\ub294 \uc0c1\uc778 \ub3d9\uc2dc\uc5d0 \ub3d9\uc544\uc2dc\uc544\uad8c \ubb38\ud559\uc5d0 \uc8fc\ub294 \uc0c1\uc778 \uc2dc\uce74\ub2e4\uc0c1\uc744 \uc218\uc5ec\ud558\uc600\ub2e4. \uc2a4\uc6e8\ub374\uc758 \ud55c \uc5b8\ub860\ub9e4\uccb4 \uae30\uc0ac\uc5d0\uc11c\ub294 \uadf8\ub97c \uad70\uc0b0\uc758 \uc81c\uc655\uc774\ub77c \uce6d\ud560 \uc815\ub3c4\uc600\ub2e4.", "paragraph_answer": "\uadf8\ub294 \ub178\ubca8 \ubb38\ud559\uc0c1 \ud6c4\ubcf4\ub85c \uc790\uc8fc \uac70\ub860\ub418\uace0 \uc788\ub294\ub370, \uc138\uacc4 \uac01\uad6d\uc5d0\uc11c \uadf8\uac00 \uc798 \uc54c\ub824\uc9c4 \uac83\uc740 \ub178\ubca8\ubb38\ud559\uc0c1 \ud6c4\ubcf4\ub77c\ub294 \uac83 \uc774\uc804\uc5d0 \uadf8\uc758 \uc2dc\uac00 20\uc5ec\uac1c\uad6d \uc5d0 \ubc88\uc5ed\ub418\uc5b4 \ub9ce\uc740 \uc601\uac10\uc744 \uc8fc\uace0 \uc788\ub2e4\ub294 \uc810 \ub54c\ubb38\uc774\ub2e4. \uc2e4\uc81c\ub85c 2005\ub144 \uc774\ud6c4 \uc5ec\ub7ec\ubc88 \ub178\ubca8\ubb38\ud559\uc0c1 \ud6c4\ubcf4\ub85c \uac70\ub860 \ub418\uae30\ub3c4 \ud558\uc600\ub2e4. \uadf8\uc758 \uc800\uba85\uc131\uc740 \ub9e4\ub144 \uc5ec\ub7ec \ub098\ub77c\uc5d0\uc11c \ucd08\uccad\ud558\ub294 \ub4f1 \uadf8\uc758 \uc2dc\uac00 \uad6d\uacbd\uc744 \ucd08\uc6d4\ud558\uace0 \uc788\ub2e4\ub294 \uc810\uc5d0\uc11c \ubcf4\uc5ec\uc8fc\uace0 \uc788\ub2e4. \ud2b9\ud788 \ubd81\uc720\ub7fd\uc5d0\uc11c \uadf8\uc758 \uc2dc\ub294 \ub192\uc740 \ud3c9\uac00\ub97c \ubc1b\ub294\ub2e4. \ub178\ub974\uc6e8\uc774\uc5d0\uc11c\ub294 \uc790\uad6d\uc758 \ub178\ubca8 \ubb38\ud559\uc0c1 \uc218\uc0c1\uc790 \ube44\uc5d0\ub978\uc2a4\ud2f0\uc5d0\ub974\ub124 \ube44\uc5d0\ub978\uc190\uc744 \uae30\ub9b0 \ube44\uc5d0\ub978\uc190 \ud6c8\uc7a5\uc744 \ubc1b\uc558\uace0, \uc2a4\uc6e8\ub374\uc5d0\uc11c\ub3c4 \uc790\uad6d \ub178\ubca8 \ubb38\ud559\uc0c1 \uc218\uc0c1\uc790 \ud558\ub93c \ub9c8\ub974\ud2f4\uc190\uc744 \uae30\ub9ac\ub294 \uc0c1\uc778 \ub3d9\uc2dc\uc5d0 \ub3d9\uc544\uc2dc\uc544\uad8c \ubb38\ud559\uc5d0 \uc8fc\ub294 \uc0c1\uc778 \uc2dc\uce74\ub2e4\uc0c1\uc744 \uc218\uc5ec\ud558\uc600\ub2e4. \uc2a4\uc6e8\ub374\uc758 \ud55c \uc5b8\ub860\ub9e4\uccb4 \uae30\uc0ac\uc5d0\uc11c\ub294 \uadf8\ub97c \uad70\uc0b0\uc758 \uc81c\uc655\uc774\ub77c \uce6d\ud560 \uc815\ub3c4\uc600\ub2e4.", "sentence_answer": "\uc138\uacc4 \uac01\uad6d\uc5d0\uc11c \uadf8\uac00 \uc798 \uc54c\ub824\uc9c4 \uac83\uc740 \ub178\ubca8\ubb38\ud559\uc0c1 \ud6c4\ubcf4\ub77c\ub294 \uac83 \uc774\uc804\uc5d0 \uadf8\uc758 \uc2dc\uac00 20\uc5ec\uac1c\uad6d \uc5d0 \ubc88\uc5ed\ub418\uc5b4 \ub9ce\uc740 \uc601\uac10\uc744 \uc8fc\uace0 \uc788\ub2e4\ub294 \uc810 \ub54c\ubb38\uc774\ub2e4.", "paragraph_id": "6471453-0-0", "paragraph_question": "question: \uace0\uc740\uc758 \uc2dc\ub294 \uba87\uac1c\uad6d\uc5d0 \ubc88\uc5ed\ub418\uc5b4 \ub9ce\uc740 \uc601\uac10\uc744 \uc8fc\uace0 \uc788\ub098?, context: \uadf8\ub294 \ub178\ubca8 \ubb38\ud559\uc0c1 \ud6c4\ubcf4\ub85c \uc790\uc8fc \uac70\ub860\ub418\uace0 \uc788\ub294\ub370, \uc138\uacc4 \uac01\uad6d\uc5d0\uc11c \uadf8\uac00 \uc798 \uc54c\ub824\uc9c4 \uac83\uc740 \ub178\ubca8\ubb38\ud559\uc0c1 \ud6c4\ubcf4\ub77c\ub294 \uac83 \uc774\uc804\uc5d0 \uadf8\uc758 \uc2dc\uac00 20\uc5ec\uac1c\uad6d\uc5d0 \ubc88\uc5ed\ub418\uc5b4 \ub9ce\uc740 \uc601\uac10\uc744 \uc8fc\uace0 \uc788\ub2e4\ub294 \uc810 \ub54c\ubb38\uc774\ub2e4. \uc2e4\uc81c\ub85c 2005\ub144 \uc774\ud6c4 \uc5ec\ub7ec\ubc88 \ub178\ubca8\ubb38\ud559\uc0c1 \ud6c4\ubcf4\ub85c \uac70\ub860 \ub418\uae30\ub3c4 \ud558\uc600\ub2e4. \uadf8\uc758 \uc800\uba85\uc131\uc740 \ub9e4\ub144 \uc5ec\ub7ec \ub098\ub77c\uc5d0\uc11c \ucd08\uccad\ud558\ub294 \ub4f1 \uadf8\uc758 \uc2dc\uac00 \uad6d\uacbd\uc744 \ucd08\uc6d4\ud558\uace0 \uc788\ub2e4\ub294 \uc810\uc5d0\uc11c \ubcf4\uc5ec\uc8fc\uace0 \uc788\ub2e4. \ud2b9\ud788 \ubd81\uc720\ub7fd\uc5d0\uc11c \uadf8\uc758 \uc2dc\ub294 \ub192\uc740 \ud3c9\uac00\ub97c \ubc1b\ub294\ub2e4. \ub178\ub974\uc6e8\uc774\uc5d0\uc11c\ub294 \uc790\uad6d\uc758 \ub178\ubca8 \ubb38\ud559\uc0c1 \uc218\uc0c1\uc790 \ube44\uc5d0\ub978\uc2a4\ud2f0\uc5d0\ub974\ub124 \ube44\uc5d0\ub978\uc190\uc744 \uae30\ub9b0 \ube44\uc5d0\ub978\uc190 \ud6c8\uc7a5\uc744 \ubc1b\uc558\uace0, \uc2a4\uc6e8\ub374\uc5d0\uc11c\ub3c4 \uc790\uad6d \ub178\ubca8 \ubb38\ud559\uc0c1 \uc218\uc0c1\uc790 \ud558\ub93c \ub9c8\ub974\ud2f4\uc190\uc744 \uae30\ub9ac\ub294 \uc0c1\uc778 \ub3d9\uc2dc\uc5d0 \ub3d9\uc544\uc2dc\uc544\uad8c \ubb38\ud559\uc5d0 \uc8fc\ub294 \uc0c1\uc778 \uc2dc\uce74\ub2e4\uc0c1\uc744 \uc218\uc5ec\ud558\uc600\ub2e4. \uc2a4\uc6e8\ub374\uc758 \ud55c \uc5b8\ub860\ub9e4\uccb4 \uae30\uc0ac\uc5d0\uc11c\ub294 \uadf8\ub97c \uad70\uc0b0\uc758 \uc81c\uc655\uc774\ub77c \uce6d\ud560 \uc815\ub3c4\uc600\ub2e4."} {"question": "\uccad\ub098\ub77c \ub9d0\uae30\uc5d0 \uc0b0\ub465 \uc9c0\ubc29, \ud654\ubca0\uc774 \uc9c0\uc5ed\uc5d0\uc11c \uc758\ud654\ub2e8\uc774 \uc77c\uc73c\ud0a8 \uc678\uc138 \ubc30\ucc99 \uc6b4\ub3d9\uc740?", "paragraph": "\uc758\ud654\ub2e8 \uc6b4\ub3d9(\u7fa9\u548c\u5718\u904b\u52d5)\uc740 \uccad\ub098\ub77c \ub9d0\uae30 1899\ub144 11\uc6d4 2\uc77c\ubd80\ud130 1901\ub144 9\uc6d4 7\uc77c\uae4c\uc9c0 \uc0b0\ub465 \uc9c0\ubc29, \ud654\ubca0\uc774 \uc9c0\uc5ed\uc5d0\uc11c \uc758\ud654\ub2e8(\u7fa9\u548c\u5718)\uc774 \uc77c\uc73c\ud0a8 \uc678\uc138 \ubc30\ucc99 \uc6b4\ub3d9\uc774\ub2e4. \uc758\ud654\ub2e8\uc758 \ub09c\uc774\ub77c\uace0\ub3c4 \ud558\uba70 1900\ub144, \uc989 \uacbd\uc790\ub144(\u5e9a\u5b50\u5e74)\uc5d0 \uc77c\uc5b4\ub09c \uad50\ub09c\uc774\ub77c\ub294 \uc758\ubbf8\ub85c \uacbd\uc790\uad50\ub09c\uc774\ub77c\uace0 \ubd80\ub974\uae30\ub3c4 \ud55c\ub2e4. \ub610 \uc758\ud654\ub2e8\uc744 \uc8fc\uba39\uc744 \uc4f0\ub294 \ube44\uc801\ub4e4\uc774\ub77c\ub294 \uc758\ubbf8\uc758 \u2018\uad8c\ube44(\u62f3\u532a)\u2019\ub098 \u2018\ub2e8\ube44(\u5718\u532a)\u2019\ub85c \uc9c0\uce6d\ud558\uc600\ub294\ub370, \ub530\ub77c\uc11c \uc758\ud654\ub2e8\uc6b4\ub3d9\uc744 \u2018\uc758\ud654\ub2e8\uc758 \ub09c,\u2019 \u2018\uad8c\ube44\uc758 \ub09c,\u2019 \u2018\ub2e8\ube44\uc758 \ub09c\u2019 \ub4f1\uc73c\ub85c \uc9c0\uce6d\ud558\uae30\ub3c4 \ud558\uc600\ub2e4. \uc0b0\ub465 \uc9c0\uc5ed\uc5d0\uc11c\ub294 \uc77c\ucc0d\uc774 \uc758\ud654\uad8c(\u7fa9\u548c\u62f3)\uc774\ub77c\ub294 \ubbfc\uac04 \uacb0\uc0ac\uac00 \uc0dd\uaca8\ub098 \ubc18\uc678\uc138 \uc6b4\ub3d9\uc744 \ubc8c\uc774\uace0 \uc788\uc5c8\ub294\ub370 1897\ub144 \ub3c5\uc77c\uc774 \uc0b0\ub465\uc131 \uc77c\ub300\ub97c \uc810\ub839\ud558\uc790 \uc758\ud654\uad8c\uc758 \ubc18\uc678\uc138, \ubc18\uae30\ub3c5\uad50 \uc6b4\ub3d9\uc774 \uaca9\ud654\ub410\ub2e4. \uc758\ud654\uad8c\uc740 \ub2e4\ub978 \ubbfc\uac04 \uc790\uc704 \uc870\uc9c1\uc5d0 \uce68\ud22c\ud574 \ud1b5\ud569\uc744 \uc774\ub8e8\uace0\ub294 \uc2a4\uc2a4\ub85c \uc758\ud654\ub2e8\uc774\ub77c\uace0 \uce6d\ud588\ub2e4.", "answer": "\uc758\ud654\ub2e8 \uc6b4\ub3d9", "sentence": "\uc758\ud654\ub2e8 \uc6b4\ub3d9 (\u7fa9\u548c\u5718\u904b\u52d5)", "paragraph_sentence": " \uc758\ud654\ub2e8 \uc6b4\ub3d9 (\u7fa9\u548c\u5718\u904b\u52d5) \uc740 \uccad\ub098\ub77c \ub9d0\uae30 1899\ub144 11\uc6d4 2\uc77c\ubd80\ud130 1901\ub144 9\uc6d4 7\uc77c\uae4c\uc9c0 \uc0b0\ub465 \uc9c0\ubc29, \ud654\ubca0\uc774 \uc9c0\uc5ed\uc5d0\uc11c \uc758\ud654\ub2e8(\u7fa9\u548c\u5718)\uc774 \uc77c\uc73c\ud0a8 \uc678\uc138 \ubc30\ucc99 \uc6b4\ub3d9\uc774\ub2e4. \uc758\ud654\ub2e8\uc758 \ub09c\uc774\ub77c\uace0\ub3c4 \ud558\uba70 1900\ub144, \uc989 \uacbd\uc790\ub144(\u5e9a\u5b50\u5e74)\uc5d0 \uc77c\uc5b4\ub09c \uad50\ub09c\uc774\ub77c\ub294 \uc758\ubbf8\ub85c \uacbd\uc790\uad50\ub09c\uc774\ub77c\uace0 \ubd80\ub974\uae30\ub3c4 \ud55c\ub2e4. \ub610 \uc758\ud654\ub2e8\uc744 \uc8fc\uba39\uc744 \uc4f0\ub294 \ube44\uc801\ub4e4\uc774\ub77c\ub294 \uc758\ubbf8\uc758 \u2018\uad8c\ube44(\u62f3\u532a)\u2019\ub098 \u2018\ub2e8\ube44(\u5718\u532a)\u2019\ub85c \uc9c0\uce6d\ud558\uc600\ub294\ub370, \ub530\ub77c\uc11c \uc758\ud654\ub2e8\uc6b4\ub3d9\uc744 \u2018\uc758\ud654\ub2e8\uc758 \ub09c,\u2019 \u2018\uad8c\ube44\uc758 \ub09c,\u2019 \u2018\ub2e8\ube44\uc758 \ub09c\u2019 \ub4f1\uc73c\ub85c \uc9c0\uce6d\ud558\uae30\ub3c4 \ud558\uc600\ub2e4. \uc0b0\ub465 \uc9c0\uc5ed\uc5d0\uc11c\ub294 \uc77c\ucc0d\uc774 \uc758\ud654\uad8c(\u7fa9\u548c\u62f3)\uc774\ub77c\ub294 \ubbfc\uac04 \uacb0\uc0ac\uac00 \uc0dd\uaca8\ub098 \ubc18\uc678\uc138 \uc6b4\ub3d9\uc744 \ubc8c\uc774\uace0 \uc788\uc5c8\ub294\ub370 1897\ub144 \ub3c5\uc77c\uc774 \uc0b0\ub465\uc131 \uc77c\ub300\ub97c \uc810\ub839\ud558\uc790 \uc758\ud654\uad8c\uc758 \ubc18\uc678\uc138, \ubc18\uae30\ub3c5\uad50 \uc6b4\ub3d9\uc774 \uaca9\ud654\ub410\ub2e4. \uc758\ud654\uad8c\uc740 \ub2e4\ub978 \ubbfc\uac04 \uc790\uc704 \uc870\uc9c1\uc5d0 \uce68\ud22c\ud574 \ud1b5\ud569\uc744 \uc774\ub8e8\uace0\ub294 \uc2a4\uc2a4\ub85c \uc758\ud654\ub2e8\uc774\ub77c\uace0 \uce6d\ud588\ub2e4.", "paragraph_answer": " \uc758\ud654\ub2e8 \uc6b4\ub3d9 (\u7fa9\u548c\u5718\u904b\u52d5)\uc740 \uccad\ub098\ub77c \ub9d0\uae30 1899\ub144 11\uc6d4 2\uc77c\ubd80\ud130 1901\ub144 9\uc6d4 7\uc77c\uae4c\uc9c0 \uc0b0\ub465 \uc9c0\ubc29, \ud654\ubca0\uc774 \uc9c0\uc5ed\uc5d0\uc11c \uc758\ud654\ub2e8(\u7fa9\u548c\u5718)\uc774 \uc77c\uc73c\ud0a8 \uc678\uc138 \ubc30\ucc99 \uc6b4\ub3d9\uc774\ub2e4. \uc758\ud654\ub2e8\uc758 \ub09c\uc774\ub77c\uace0\ub3c4 \ud558\uba70 1900\ub144, \uc989 \uacbd\uc790\ub144(\u5e9a\u5b50\u5e74)\uc5d0 \uc77c\uc5b4\ub09c \uad50\ub09c\uc774\ub77c\ub294 \uc758\ubbf8\ub85c \uacbd\uc790\uad50\ub09c\uc774\ub77c\uace0 \ubd80\ub974\uae30\ub3c4 \ud55c\ub2e4. \ub610 \uc758\ud654\ub2e8\uc744 \uc8fc\uba39\uc744 \uc4f0\ub294 \ube44\uc801\ub4e4\uc774\ub77c\ub294 \uc758\ubbf8\uc758 \u2018\uad8c\ube44(\u62f3\u532a)\u2019\ub098 \u2018\ub2e8\ube44(\u5718\u532a)\u2019\ub85c \uc9c0\uce6d\ud558\uc600\ub294\ub370, \ub530\ub77c\uc11c \uc758\ud654\ub2e8\uc6b4\ub3d9\uc744 \u2018\uc758\ud654\ub2e8\uc758 \ub09c,\u2019 \u2018\uad8c\ube44\uc758 \ub09c,\u2019 \u2018\ub2e8\ube44\uc758 \ub09c\u2019 \ub4f1\uc73c\ub85c \uc9c0\uce6d\ud558\uae30\ub3c4 \ud558\uc600\ub2e4. \uc0b0\ub465 \uc9c0\uc5ed\uc5d0\uc11c\ub294 \uc77c\ucc0d\uc774 \uc758\ud654\uad8c(\u7fa9\u548c\u62f3)\uc774\ub77c\ub294 \ubbfc\uac04 \uacb0\uc0ac\uac00 \uc0dd\uaca8\ub098 \ubc18\uc678\uc138 \uc6b4\ub3d9\uc744 \ubc8c\uc774\uace0 \uc788\uc5c8\ub294\ub370 1897\ub144 \ub3c5\uc77c\uc774 \uc0b0\ub465\uc131 \uc77c\ub300\ub97c \uc810\ub839\ud558\uc790 \uc758\ud654\uad8c\uc758 \ubc18\uc678\uc138, \ubc18\uae30\ub3c5\uad50 \uc6b4\ub3d9\uc774 \uaca9\ud654\ub410\ub2e4. \uc758\ud654\uad8c\uc740 \ub2e4\ub978 \ubbfc\uac04 \uc790\uc704 \uc870\uc9c1\uc5d0 \uce68\ud22c\ud574 \ud1b5\ud569\uc744 \uc774\ub8e8\uace0\ub294 \uc2a4\uc2a4\ub85c \uc758\ud654\ub2e8\uc774\ub77c\uace0 \uce6d\ud588\ub2e4.", "sentence_answer": " \uc758\ud654\ub2e8 \uc6b4\ub3d9 (\u7fa9\u548c\u5718\u904b\u52d5)", "paragraph_id": "6326091-0-0", "paragraph_question": "question: \uccad\ub098\ub77c \ub9d0\uae30\uc5d0 \uc0b0\ub465 \uc9c0\ubc29, \ud654\ubca0\uc774 \uc9c0\uc5ed\uc5d0\uc11c \uc758\ud654\ub2e8\uc774 \uc77c\uc73c\ud0a8 \uc678\uc138 \ubc30\ucc99 \uc6b4\ub3d9\uc740?, context: \uc758\ud654\ub2e8 \uc6b4\ub3d9(\u7fa9\u548c\u5718\u904b\u52d5)\uc740 \uccad\ub098\ub77c \ub9d0\uae30 1899\ub144 11\uc6d4 2\uc77c\ubd80\ud130 1901\ub144 9\uc6d4 7\uc77c\uae4c\uc9c0 \uc0b0\ub465 \uc9c0\ubc29, \ud654\ubca0\uc774 \uc9c0\uc5ed\uc5d0\uc11c \uc758\ud654\ub2e8(\u7fa9\u548c\u5718)\uc774 \uc77c\uc73c\ud0a8 \uc678\uc138 \ubc30\ucc99 \uc6b4\ub3d9\uc774\ub2e4. \uc758\ud654\ub2e8\uc758 \ub09c\uc774\ub77c\uace0\ub3c4 \ud558\uba70 1900\ub144, \uc989 \uacbd\uc790\ub144(\u5e9a\u5b50\u5e74)\uc5d0 \uc77c\uc5b4\ub09c \uad50\ub09c\uc774\ub77c\ub294 \uc758\ubbf8\ub85c \uacbd\uc790\uad50\ub09c\uc774\ub77c\uace0 \ubd80\ub974\uae30\ub3c4 \ud55c\ub2e4. \ub610 \uc758\ud654\ub2e8\uc744 \uc8fc\uba39\uc744 \uc4f0\ub294 \ube44\uc801\ub4e4\uc774\ub77c\ub294 \uc758\ubbf8\uc758 \u2018\uad8c\ube44(\u62f3\u532a)\u2019\ub098 \u2018\ub2e8\ube44(\u5718\u532a)\u2019\ub85c \uc9c0\uce6d\ud558\uc600\ub294\ub370, \ub530\ub77c\uc11c \uc758\ud654\ub2e8\uc6b4\ub3d9\uc744 \u2018\uc758\ud654\ub2e8\uc758 \ub09c,\u2019 \u2018\uad8c\ube44\uc758 \ub09c,\u2019 \u2018\ub2e8\ube44\uc758 \ub09c\u2019 \ub4f1\uc73c\ub85c \uc9c0\uce6d\ud558\uae30\ub3c4 \ud558\uc600\ub2e4. \uc0b0\ub465 \uc9c0\uc5ed\uc5d0\uc11c\ub294 \uc77c\ucc0d\uc774 \uc758\ud654\uad8c(\u7fa9\u548c\u62f3)\uc774\ub77c\ub294 \ubbfc\uac04 \uacb0\uc0ac\uac00 \uc0dd\uaca8\ub098 \ubc18\uc678\uc138 \uc6b4\ub3d9\uc744 \ubc8c\uc774\uace0 \uc788\uc5c8\ub294\ub370 1897\ub144 \ub3c5\uc77c\uc774 \uc0b0\ub465\uc131 \uc77c\ub300\ub97c \uc810\ub839\ud558\uc790 \uc758\ud654\uad8c\uc758 \ubc18\uc678\uc138, \ubc18\uae30\ub3c5\uad50 \uc6b4\ub3d9\uc774 \uaca9\ud654\ub410\ub2e4. \uc758\ud654\uad8c\uc740 \ub2e4\ub978 \ubbfc\uac04 \uc790\uc704 \uc870\uc9c1\uc5d0 \uce68\ud22c\ud574 \ud1b5\ud569\uc744 \uc774\ub8e8\uace0\ub294 \uc2a4\uc2a4\ub85c \uc758\ud654\ub2e8\uc774\ub77c\uace0 \uce6d\ud588\ub2e4."} {"question": "\ub1cc \ubd80\ud53c\uc758 10-20%\ubc16\uc5d0 \ucc28\uc9c0\ud558\uc9c0 \uc54a\uc9c0\ub9cc \ub1cc \uc804\uccb4 \ub274\ub7f0\uc758 \ubc18 \uc774\uc0c1\uc744 \uac00\uc9c0\ub294 \ub1cc\uc758 \uc601\uc5ed\uc740 \uc5b4\ub514\uc778\uac00?", "paragraph": "\ucd5c\uadfc\ub4e4\uc5b4 \uc8fc\uc758, \uc5b8\uc5b4, \uc778\uc9c0 \ubc0f \uc815\uc11c\uc5d0 \uc788\uc5b4 \uc18c\ub1cc\uc758 \uc911\uc694\ud55c \uc5ed\ud560\uc5d0 \ub300\ud55c \uc778\uc2dd\uc774 \uc99d\uac00\ud558\uace0 \uc788\ub2e4. \uc778\uc9c0\uc801, \uc5b8\uc5b4\uc801, \uc0ac\ud68c\ud589\ub3d9\uc801 \ubc0f \uc815\uc11c\uc801 \uc7a5\uc560\uac00 \uc18c\ub1cc \ubc0f \ucda9\ubd80 \ucd9c\ud608\uc2dc \uc8fc\uc694\ud55c \uc784\uc0c1\uc801 \uacb0\uacfc\ub85c \ub098\ud0c0\ub098\uace0 \uc788\ub2e4. \uacc4\ud1b5 \ubc1c\uc0dd\uc801 \uc99d\uac70\ub294 \ucd5c\uadfc \uc218\ubc31\ub9cc \ub144 \ub3d9\uc548\uc758 \uc18c\ub1cc \ubc18\uad6c \ud655\uc7a5\uc774 \uc804\ub450\uc5fd \ud06c\uae30\uc758 \uadf9\uc801\uc778 \uc9c4\ud654\uc801 \uc99d\uac00\uc640 \ubcd1\ud589\ub418\uc5c8\uc74c\uc744 \ubcf4\uc5ec\uc900\ub2e4. \ub1cc \ubd80\ud53c\uc758 10-20%\ub9cc\uc744 \ucc28\uc9c0\ud558\ub294 \uc18c\ub1cc\ub294 \ub1cc\uc804\uccb4 \ub274\ub7f0\uc758 \ubc18 \uc774\uc0c1\uc744 \ubcf4\uc720\ud55c\ub2e4. \uc0c8\ub85c\uc6b4 \uc5f0\uad6c \uacb0\uacfc\ub4e4\uc740 \uc18c\ub1cc\ucda9\ubd80\uc758 \ube44\uc815\uc0c1\uc131\uc740 \uc870\ud604\uc99d, \uc790\ud3d0\uc99d, ADHD \ubc0f \uc591\uadf9\uc131, \ub2e8\uadf9\uc131 \uc6b0\uc6b8\uc99d \ub4f1 \ub9ce\uc740 \uc815\uc2e0\uacfc\uc801 \uc9c8\ud658\uacfc \uad00\ub828\ub418\uc5b4 \ubcf4\uc778\ub2e4. \uc18c\ub1cc\ucda9\ubd80\ub294 norepinephrine \ubc0f \ub3c4\ud30c\ubbfc \ubd84\ucd9c\uc744 \uc704\ud55c \uc138\ud3ec\uccb4 \uc601\uc5ed\uc778 \uccad\ubc18(locus coeruleus), \ubcf5\uce21\ud53c\uac1c\uc601\uc5ed(ventral tegmental area), \ud751\uc9c8(substantia nigra)\uc5d0 \uc788\uc5b4 \uac15\ud55c \uc870\uc808 \ud6a8\uacfc\ub97c \ub098\ud0c0\ub0b8\ub2e4. \uc18c\ub1cc\ub294 \ub192\uc740 \ubc00\ub3c4\uc758 \uae00\ub8e8\ucf54\ucf54\ub974\ud2f0\ucf54\uc774\ub4dc \uc218\uc6a9\uccb4\uc640 CRF \uc218\uc6a9\uccb4\ub97c \ubcf4\uc720\ud558\uba70 \uc2a4\ud2b8\ub808\uc2a4 \ubc18\uc751 \uc870\uc808\uc5d0 \uc911\uc694\ud55c \uae30\ub2a5\uc744 \ud558\ub294 \uac83\uc73c\ub85c \ubcf4\uc778\ub2e4. \uc18c\ub1cc\ub294 \uc2dc\uc2a4\ud15c\uc801 \uc21c\ud658\ub9cc\uc744 \uc870\uc808\ud560 \ubfd0 \uc544\ub2c8\ub77c rCBF(\uad6d\uc18c\ub1cc\ud608\ub958\ub7c9)\uc5d0\ub3c4 \uc0c1\ub2f9\ud55c \uc601\ud5a5\uc744 \ubbf8\uce58\uba70, \ud608\ub958\uc5d0 \ub300\ud55c \uc601\ud5a5\uacfc \ub3c5\ub9bd\uc801\uc73c\ub85c \ud5c8\ud608\uc131 \uc190\uc0c1\uc73c\ub85c\ubd80\ud130\uc758 \ub1cc\uc758 \uc7a5\uae30\uc801 \uc2e0\uacbd\ubcf4\ud638\ub97c \ucd09\ubc1c\ud55c\ub2e4. \uc18c\ub1cc\ucda9\ubd80(vermis)\ub294 \uc815\uc2e0\uac74\uac15 \uc720\uc9c0\uc5d0 \uc788\uc5b4 \uc911\uc694\ud55c \ubd80\uc704\uc774\uba70 \ucd08\uae30 \uc2a4\ud2b8\ub808\uc2a4\ub098 \ubc29\uc784\uc5d0 \uc720\uc758\ud558\uac8c \uc601\ud5a5\uc744 \ubc1b\uc73c\uba70, \ucd08\uae30 \uc2a4\ud2b8\ub808\uc640 \ubc29\uc784\uc73c\ub85c \uc778\ud55c \uc911\uc694\ud55c \uc2e0\uacbd\ud589\ub3d9\ud559\uc801 \uacb0\uacfc\uc758 \uc77c\ubd80\ub97c \uc911\uc7ac\ud558\ub294 \uac83\uc73c\ub85c \ubcf4\uc778\ub2e4.", "answer": "\uc18c\ub1cc", "sentence": "\ucd5c\uadfc\ub4e4\uc5b4 \uc8fc\uc758, \uc5b8\uc5b4, \uc778\uc9c0 \ubc0f \uc815\uc11c\uc5d0 \uc788\uc5b4 \uc18c\ub1cc \uc758 \uc911\uc694\ud55c \uc5ed\ud560\uc5d0 \ub300\ud55c \uc778\uc2dd\uc774 \uc99d\uac00\ud558\uace0 \uc788\ub2e4.", "paragraph_sentence": " \ucd5c\uadfc\ub4e4\uc5b4 \uc8fc\uc758, \uc5b8\uc5b4, \uc778\uc9c0 \ubc0f \uc815\uc11c\uc5d0 \uc788\uc5b4 \uc18c\ub1cc \uc758 \uc911\uc694\ud55c \uc5ed\ud560\uc5d0 \ub300\ud55c \uc778\uc2dd\uc774 \uc99d\uac00\ud558\uace0 \uc788\ub2e4. \uc778\uc9c0\uc801, \uc5b8\uc5b4\uc801, \uc0ac\ud68c\ud589\ub3d9\uc801 \ubc0f \uc815\uc11c\uc801 \uc7a5\uc560\uac00 \uc18c\ub1cc \ubc0f \ucda9\ubd80 \ucd9c\ud608\uc2dc \uc8fc\uc694\ud55c \uc784\uc0c1\uc801 \uacb0\uacfc\ub85c \ub098\ud0c0\ub098\uace0 \uc788\ub2e4. \uacc4\ud1b5 \ubc1c\uc0dd\uc801 \uc99d\uac70\ub294 \ucd5c\uadfc \uc218\ubc31\ub9cc \ub144 \ub3d9\uc548\uc758 \uc18c\ub1cc \ubc18\uad6c \ud655\uc7a5\uc774 \uc804\ub450\uc5fd \ud06c\uae30\uc758 \uadf9\uc801\uc778 \uc9c4\ud654\uc801 \uc99d\uac00\uc640 \ubcd1\ud589\ub418\uc5c8\uc74c\uc744 \ubcf4\uc5ec\uc900\ub2e4. \ub1cc \ubd80\ud53c\uc758 10-20%\ub9cc\uc744 \ucc28\uc9c0\ud558\ub294 \uc18c\ub1cc\ub294 \ub1cc\uc804\uccb4 \ub274\ub7f0\uc758 \ubc18 \uc774\uc0c1\uc744 \ubcf4\uc720\ud55c\ub2e4. \uc0c8\ub85c\uc6b4 \uc5f0\uad6c \uacb0\uacfc\ub4e4\uc740 \uc18c\ub1cc\ucda9\ubd80\uc758 \ube44\uc815\uc0c1\uc131\uc740 \uc870\ud604\uc99d, \uc790\ud3d0\uc99d, ADHD \ubc0f \uc591\uadf9\uc131, \ub2e8\uadf9\uc131 \uc6b0\uc6b8\uc99d \ub4f1 \ub9ce\uc740 \uc815\uc2e0\uacfc\uc801 \uc9c8\ud658\uacfc \uad00\ub828\ub418\uc5b4 \ubcf4\uc778\ub2e4. \uc18c\ub1cc\ucda9\ubd80\ub294 norepinephrine \ubc0f \ub3c4\ud30c\ubbfc \ubd84\ucd9c\uc744 \uc704\ud55c \uc138\ud3ec\uccb4 \uc601\uc5ed\uc778 \uccad\ubc18(locus coeruleus), \ubcf5\uce21\ud53c\uac1c\uc601\uc5ed(ventral tegmental area), \ud751\uc9c8(substantia nigra)\uc5d0 \uc788\uc5b4 \uac15\ud55c \uc870\uc808 \ud6a8\uacfc\ub97c \ub098\ud0c0\ub0b8\ub2e4. \uc18c\ub1cc\ub294 \ub192\uc740 \ubc00\ub3c4\uc758 \uae00\ub8e8\ucf54\ucf54\ub974\ud2f0\ucf54\uc774\ub4dc \uc218\uc6a9\uccb4\uc640 CRF \uc218\uc6a9\uccb4\ub97c \ubcf4\uc720\ud558\uba70 \uc2a4\ud2b8\ub808\uc2a4 \ubc18\uc751 \uc870\uc808\uc5d0 \uc911\uc694\ud55c \uae30\ub2a5\uc744 \ud558\ub294 \uac83\uc73c\ub85c \ubcf4\uc778\ub2e4. \uc18c\ub1cc\ub294 \uc2dc\uc2a4\ud15c\uc801 \uc21c\ud658\ub9cc\uc744 \uc870\uc808\ud560 \ubfd0 \uc544\ub2c8\ub77c rCBF(\uad6d\uc18c\ub1cc\ud608\ub958\ub7c9)\uc5d0\ub3c4 \uc0c1\ub2f9\ud55c \uc601\ud5a5\uc744 \ubbf8\uce58\uba70, \ud608\ub958\uc5d0 \ub300\ud55c \uc601\ud5a5\uacfc \ub3c5\ub9bd\uc801\uc73c\ub85c \ud5c8\ud608\uc131 \uc190\uc0c1\uc73c\ub85c\ubd80\ud130\uc758 \ub1cc\uc758 \uc7a5\uae30\uc801 \uc2e0\uacbd\ubcf4\ud638\ub97c \ucd09\ubc1c\ud55c\ub2e4. \uc18c\ub1cc\ucda9\ubd80(vermis)\ub294 \uc815\uc2e0\uac74\uac15 \uc720\uc9c0\uc5d0 \uc788\uc5b4 \uc911\uc694\ud55c \ubd80\uc704\uc774\uba70 \ucd08\uae30 \uc2a4\ud2b8\ub808\uc2a4\ub098 \ubc29\uc784\uc5d0 \uc720\uc758\ud558\uac8c \uc601\ud5a5\uc744 \ubc1b\uc73c\uba70, \ucd08\uae30 \uc2a4\ud2b8\ub808\uc640 \ubc29\uc784\uc73c\ub85c \uc778\ud55c \uc911\uc694\ud55c \uc2e0\uacbd\ud589\ub3d9\ud559\uc801 \uacb0\uacfc\uc758 \uc77c\ubd80\ub97c \uc911\uc7ac\ud558\ub294 \uac83\uc73c\ub85c \ubcf4\uc778\ub2e4.", "paragraph_answer": "\ucd5c\uadfc\ub4e4\uc5b4 \uc8fc\uc758, \uc5b8\uc5b4, \uc778\uc9c0 \ubc0f \uc815\uc11c\uc5d0 \uc788\uc5b4 \uc18c\ub1cc \uc758 \uc911\uc694\ud55c \uc5ed\ud560\uc5d0 \ub300\ud55c \uc778\uc2dd\uc774 \uc99d\uac00\ud558\uace0 \uc788\ub2e4. \uc778\uc9c0\uc801, \uc5b8\uc5b4\uc801, \uc0ac\ud68c\ud589\ub3d9\uc801 \ubc0f \uc815\uc11c\uc801 \uc7a5\uc560\uac00 \uc18c\ub1cc \ubc0f \ucda9\ubd80 \ucd9c\ud608\uc2dc \uc8fc\uc694\ud55c \uc784\uc0c1\uc801 \uacb0\uacfc\ub85c \ub098\ud0c0\ub098\uace0 \uc788\ub2e4. \uacc4\ud1b5 \ubc1c\uc0dd\uc801 \uc99d\uac70\ub294 \ucd5c\uadfc \uc218\ubc31\ub9cc \ub144 \ub3d9\uc548\uc758 \uc18c\ub1cc \ubc18\uad6c \ud655\uc7a5\uc774 \uc804\ub450\uc5fd \ud06c\uae30\uc758 \uadf9\uc801\uc778 \uc9c4\ud654\uc801 \uc99d\uac00\uc640 \ubcd1\ud589\ub418\uc5c8\uc74c\uc744 \ubcf4\uc5ec\uc900\ub2e4. \ub1cc \ubd80\ud53c\uc758 10-20%\ub9cc\uc744 \ucc28\uc9c0\ud558\ub294 \uc18c\ub1cc\ub294 \ub1cc\uc804\uccb4 \ub274\ub7f0\uc758 \ubc18 \uc774\uc0c1\uc744 \ubcf4\uc720\ud55c\ub2e4. \uc0c8\ub85c\uc6b4 \uc5f0\uad6c \uacb0\uacfc\ub4e4\uc740 \uc18c\ub1cc\ucda9\ubd80\uc758 \ube44\uc815\uc0c1\uc131\uc740 \uc870\ud604\uc99d, \uc790\ud3d0\uc99d, ADHD \ubc0f \uc591\uadf9\uc131, \ub2e8\uadf9\uc131 \uc6b0\uc6b8\uc99d \ub4f1 \ub9ce\uc740 \uc815\uc2e0\uacfc\uc801 \uc9c8\ud658\uacfc \uad00\ub828\ub418\uc5b4 \ubcf4\uc778\ub2e4. \uc18c\ub1cc\ucda9\ubd80\ub294 norepinephrine \ubc0f \ub3c4\ud30c\ubbfc \ubd84\ucd9c\uc744 \uc704\ud55c \uc138\ud3ec\uccb4 \uc601\uc5ed\uc778 \uccad\ubc18(locus coeruleus), \ubcf5\uce21\ud53c\uac1c\uc601\uc5ed(ventral tegmental area), \ud751\uc9c8(substantia nigra)\uc5d0 \uc788\uc5b4 \uac15\ud55c \uc870\uc808 \ud6a8\uacfc\ub97c \ub098\ud0c0\ub0b8\ub2e4. \uc18c\ub1cc\ub294 \ub192\uc740 \ubc00\ub3c4\uc758 \uae00\ub8e8\ucf54\ucf54\ub974\ud2f0\ucf54\uc774\ub4dc \uc218\uc6a9\uccb4\uc640 CRF \uc218\uc6a9\uccb4\ub97c \ubcf4\uc720\ud558\uba70 \uc2a4\ud2b8\ub808\uc2a4 \ubc18\uc751 \uc870\uc808\uc5d0 \uc911\uc694\ud55c \uae30\ub2a5\uc744 \ud558\ub294 \uac83\uc73c\ub85c \ubcf4\uc778\ub2e4. \uc18c\ub1cc\ub294 \uc2dc\uc2a4\ud15c\uc801 \uc21c\ud658\ub9cc\uc744 \uc870\uc808\ud560 \ubfd0 \uc544\ub2c8\ub77c rCBF(\uad6d\uc18c\ub1cc\ud608\ub958\ub7c9)\uc5d0\ub3c4 \uc0c1\ub2f9\ud55c \uc601\ud5a5\uc744 \ubbf8\uce58\uba70, \ud608\ub958\uc5d0 \ub300\ud55c \uc601\ud5a5\uacfc \ub3c5\ub9bd\uc801\uc73c\ub85c \ud5c8\ud608\uc131 \uc190\uc0c1\uc73c\ub85c\ubd80\ud130\uc758 \ub1cc\uc758 \uc7a5\uae30\uc801 \uc2e0\uacbd\ubcf4\ud638\ub97c \ucd09\ubc1c\ud55c\ub2e4. \uc18c\ub1cc\ucda9\ubd80(vermis)\ub294 \uc815\uc2e0\uac74\uac15 \uc720\uc9c0\uc5d0 \uc788\uc5b4 \uc911\uc694\ud55c \ubd80\uc704\uc774\uba70 \ucd08\uae30 \uc2a4\ud2b8\ub808\uc2a4\ub098 \ubc29\uc784\uc5d0 \uc720\uc758\ud558\uac8c \uc601\ud5a5\uc744 \ubc1b\uc73c\uba70, \ucd08\uae30 \uc2a4\ud2b8\ub808\uc640 \ubc29\uc784\uc73c\ub85c \uc778\ud55c \uc911\uc694\ud55c \uc2e0\uacbd\ud589\ub3d9\ud559\uc801 \uacb0\uacfc\uc758 \uc77c\ubd80\ub97c \uc911\uc7ac\ud558\ub294 \uac83\uc73c\ub85c \ubcf4\uc778\ub2e4.", "sentence_answer": "\ucd5c\uadfc\ub4e4\uc5b4 \uc8fc\uc758, \uc5b8\uc5b4, \uc778\uc9c0 \ubc0f \uc815\uc11c\uc5d0 \uc788\uc5b4 \uc18c\ub1cc \uc758 \uc911\uc694\ud55c \uc5ed\ud560\uc5d0 \ub300\ud55c \uc778\uc2dd\uc774 \uc99d\uac00\ud558\uace0 \uc788\ub2e4.", "paragraph_id": "6531662-8-0", "paragraph_question": "question: \ub1cc \ubd80\ud53c\uc758 10-20%\ubc16\uc5d0 \ucc28\uc9c0\ud558\uc9c0 \uc54a\uc9c0\ub9cc \ub1cc \uc804\uccb4 \ub274\ub7f0\uc758 \ubc18 \uc774\uc0c1\uc744 \uac00\uc9c0\ub294 \ub1cc\uc758 \uc601\uc5ed\uc740 \uc5b4\ub514\uc778\uac00?, context: \ucd5c\uadfc\ub4e4\uc5b4 \uc8fc\uc758, \uc5b8\uc5b4, \uc778\uc9c0 \ubc0f \uc815\uc11c\uc5d0 \uc788\uc5b4 \uc18c\ub1cc\uc758 \uc911\uc694\ud55c \uc5ed\ud560\uc5d0 \ub300\ud55c \uc778\uc2dd\uc774 \uc99d\uac00\ud558\uace0 \uc788\ub2e4. \uc778\uc9c0\uc801, \uc5b8\uc5b4\uc801, \uc0ac\ud68c\ud589\ub3d9\uc801 \ubc0f \uc815\uc11c\uc801 \uc7a5\uc560\uac00 \uc18c\ub1cc \ubc0f \ucda9\ubd80 \ucd9c\ud608\uc2dc \uc8fc\uc694\ud55c \uc784\uc0c1\uc801 \uacb0\uacfc\ub85c \ub098\ud0c0\ub098\uace0 \uc788\ub2e4. \uacc4\ud1b5 \ubc1c\uc0dd\uc801 \uc99d\uac70\ub294 \ucd5c\uadfc \uc218\ubc31\ub9cc \ub144 \ub3d9\uc548\uc758 \uc18c\ub1cc \ubc18\uad6c \ud655\uc7a5\uc774 \uc804\ub450\uc5fd \ud06c\uae30\uc758 \uadf9\uc801\uc778 \uc9c4\ud654\uc801 \uc99d\uac00\uc640 \ubcd1\ud589\ub418\uc5c8\uc74c\uc744 \ubcf4\uc5ec\uc900\ub2e4. \ub1cc \ubd80\ud53c\uc758 10-20%\ub9cc\uc744 \ucc28\uc9c0\ud558\ub294 \uc18c\ub1cc\ub294 \ub1cc\uc804\uccb4 \ub274\ub7f0\uc758 \ubc18 \uc774\uc0c1\uc744 \ubcf4\uc720\ud55c\ub2e4. \uc0c8\ub85c\uc6b4 \uc5f0\uad6c \uacb0\uacfc\ub4e4\uc740 \uc18c\ub1cc\ucda9\ubd80\uc758 \ube44\uc815\uc0c1\uc131\uc740 \uc870\ud604\uc99d, \uc790\ud3d0\uc99d, ADHD \ubc0f \uc591\uadf9\uc131, \ub2e8\uadf9\uc131 \uc6b0\uc6b8\uc99d \ub4f1 \ub9ce\uc740 \uc815\uc2e0\uacfc\uc801 \uc9c8\ud658\uacfc \uad00\ub828\ub418\uc5b4 \ubcf4\uc778\ub2e4. \uc18c\ub1cc\ucda9\ubd80\ub294 norepinephrine \ubc0f \ub3c4\ud30c\ubbfc \ubd84\ucd9c\uc744 \uc704\ud55c \uc138\ud3ec\uccb4 \uc601\uc5ed\uc778 \uccad\ubc18(locus coeruleus), \ubcf5\uce21\ud53c\uac1c\uc601\uc5ed(ventral tegmental area), \ud751\uc9c8(substantia nigra)\uc5d0 \uc788\uc5b4 \uac15\ud55c \uc870\uc808 \ud6a8\uacfc\ub97c \ub098\ud0c0\ub0b8\ub2e4. \uc18c\ub1cc\ub294 \ub192\uc740 \ubc00\ub3c4\uc758 \uae00\ub8e8\ucf54\ucf54\ub974\ud2f0\ucf54\uc774\ub4dc \uc218\uc6a9\uccb4\uc640 CRF \uc218\uc6a9\uccb4\ub97c \ubcf4\uc720\ud558\uba70 \uc2a4\ud2b8\ub808\uc2a4 \ubc18\uc751 \uc870\uc808\uc5d0 \uc911\uc694\ud55c \uae30\ub2a5\uc744 \ud558\ub294 \uac83\uc73c\ub85c \ubcf4\uc778\ub2e4. \uc18c\ub1cc\ub294 \uc2dc\uc2a4\ud15c\uc801 \uc21c\ud658\ub9cc\uc744 \uc870\uc808\ud560 \ubfd0 \uc544\ub2c8\ub77c rCBF(\uad6d\uc18c\ub1cc\ud608\ub958\ub7c9)\uc5d0\ub3c4 \uc0c1\ub2f9\ud55c \uc601\ud5a5\uc744 \ubbf8\uce58\uba70, \ud608\ub958\uc5d0 \ub300\ud55c \uc601\ud5a5\uacfc \ub3c5\ub9bd\uc801\uc73c\ub85c \ud5c8\ud608\uc131 \uc190\uc0c1\uc73c\ub85c\ubd80\ud130\uc758 \ub1cc\uc758 \uc7a5\uae30\uc801 \uc2e0\uacbd\ubcf4\ud638\ub97c \ucd09\ubc1c\ud55c\ub2e4. \uc18c\ub1cc\ucda9\ubd80(vermis)\ub294 \uc815\uc2e0\uac74\uac15 \uc720\uc9c0\uc5d0 \uc788\uc5b4 \uc911\uc694\ud55c \ubd80\uc704\uc774\uba70 \ucd08\uae30 \uc2a4\ud2b8\ub808\uc2a4\ub098 \ubc29\uc784\uc5d0 \uc720\uc758\ud558\uac8c \uc601\ud5a5\uc744 \ubc1b\uc73c\uba70, \ucd08\uae30 \uc2a4\ud2b8\ub808\uc640 \ubc29\uc784\uc73c\ub85c \uc778\ud55c \uc911\uc694\ud55c \uc2e0\uacbd\ud589\ub3d9\ud559\uc801 \uacb0\uacfc\uc758 \uc77c\ubd80\ub97c \uc911\uc7ac\ud558\ub294 \uac83\uc73c\ub85c \ubcf4\uc778\ub2e4."} {"question": "\uac1c\ubc29\uc2dc\uac04\uc774 \uc815\ud574\uc838 \uc788\ub358 \uccad\uc640\ub300 \uc55e\uae38\uc744 \ubb38\uc7ac\uc778 \uc815\ubd80\uac00 \ub4e4\uc5b4\uc11c\uba74\uc11c \uac1c\ubc29\uc2dc\uac04\uc774 \uc5b4\ub5bb\uac8c \ubc14\ub00c\uc5c8\ub098?", "paragraph": "\uc774\uc804 \uc815\uad8c\uc758 \ubd88\ud1b5 \uc774\ubbf8\uc9c0\uac00 \uc2ec\uac01\ud588\ub358 \ud0d3\ub3c4 \uc788\uae30\uc5d0, \ubb38\uc7ac\uc778 \uc815\ubd80\ub294 \ucd9c\ubc94\ud55c \ub4a4 \uc18c\ud1b5\uacfc \ud568\uaed8 \ud0c8\uad8c\uc704\ub97c \uac15\uc870\ud588\ub2e4. 5\uc6d4 10\uc77c \ucde8\uc784\uc2dd\uc5d0\ub294 \ud5cc\ubc95\uae30\uad00\uc7a5\uc778 5\ubd80 \uc694\uc778\uacfc \uad6d\ud68c\uc758\uc6d0, \uad6d\ubb34\uc704\uc6d0, \uad70 \uc9c0\ud718\uad00 \ub4f1 300\uc5ec \uba85\ub9cc\uc744 \ucd08\ub300\ud558\uc5ec \uac04\uc18c\ud558\uac8c \uc9c4\ud589\ud588\ub294\ub370 \uc218\ub9cc \uba85\uc744 \ucd08\ub300\ud558\uc5ec \uad6d\ud68c\uc758\uc0ac\ub2f9\uc5d0\uc11c \uc131\ub300\ud558\uac8c \uc5f4\ub9ac\ub358 \uc5ed\ub300 \ucde8\uc784\uc2dd\uacfc\ub294 \ub2e4\ub978 \ud48d\uacbd\uc774\uc5c8\ub2e4. \uc774\ub294 \ucde8\uc784\uc2dd\uc744 \uc900\ube44\ud560 \uc2dc\uac04\uc870\ucc28 \uc5c6\uc5c8\ub358 \ud0d3\ub3c4 \uc788\uc9c0\ub9cc, \uad6d\uc815 \ud63c\ub780\uc744 \ube60\ub974\uac8c \uc218\uc2b5\ud558\uace0 \uc2e0\uc18d\ud788 \ud0c0\uac1c\ud558\uaca0\ub2e4\ub294 \ub300\ud1b5\ub839\uc758 \uc758\uc9c0\uac00 \ubc18\uc601\ub410\ub2e4\ub294 \ud6c4\ubb38\uc774\ub2e4. 11\uc77c\uc5d0\ub294 \uc218\uc11d\uc774 \uc544\ub2cc \ube44\uc11c\uad00\uacfc \uacb8\uc0c1\uc744 \ud558\uace0 \uc2e0\uc784 \uc218\uc11d \ub4f1\uacfc \uaca9\uc758 \uc5c6\ub294 \uc624\ucc2c\uc744 \uac00\uc84c\uc73c\uba70, \uc9c1\ud6c4\uc5d0\ub294 \uacbd\ub0b4\uc5d0\uc11c \ucee4\ud53c \uc0b0\ucc45\uc744 \ud588\ub2e4. \uacbd\ud638\uc5d0 \uc788\uc5b4\uc11c\ub3c4 '\uc5f4\ub9b0 \uacbd\ud638', '\ub0ae\uc740 \uacbd\ud638'\ub97c \uc9c0\ud5a5\ud558\uba70 \uc2dc\ubbfc\ub4e4\uc5d0\uac8c \ub2e4\uac00\uac00\uae30 \uc704\ud55c \ub178\ub825\uc744 \ubcf4\uc600\ub2e4. 6\uc6d4 \ud56d\uc7c1 \uae30\ub150\uc2dd\uc5d0 10\ub144 \ub9cc\uc5d0 \ud604\uc9c1 \ub300\ud1b5\ub839\uc774 \ucc38\uc11d\ud588\uc9c0\ub9cc '\ub300\ud1b5\ub839\uc774 \ucc38\uc11d\ud55c \uc57c\uc678\ud589\uc0ac'\uce58\uace0\ub294 \uacbd\ube44\u00b7\uacbd\ud638 \uc778\ub825\uc758 \uc874\uc7ac\uac00 \ub450\ub4dc\ub7ec\uc9c0\uc9c0 \uc54a\uc558\ub2e4. \uc758\uc2ec\uc2a4\ub7ec\uc6b4 \uc778\ubb3c\uc744 \uac00\ub824\ub0b4\uae30 \uc704\ud574 \uae08\uc18d\ud0d0\uc9c0\uae30\ub85c \uc18c\uc9c0\ud488\uc744 \uac80\uc0ac\ud558\ub294 \uc808\ucc28 \uc815\ub3c4\ub9cc \uac70\uce60 \ubfd0, \ud589\uc0ac\uc7a5\uc5d0 \ubc30\uce58\ub41c \uacbd\ucc30\uad00\ub4e4\uc740 \uc2dc\ubbfc\ub4e4\uc758 \ud1b5\ud589\uc5d0 \uc801\uadf9\uc801\uc73c\ub85c \ud611\uc870\ud558\ub294 \ubaa8\uc2b5\uc744 \ubcf4\uc600\ub2e4. 22\uc77c\uc5d0\ub294 5\uc2dc 30\ubd84\ubd80\ud130 20\uc2dc\uae4c\uc9c0\ub9cc \uac1c\ubc29\ub418\ub358 \uccad\uc640\ub300 \uc55e\uae38\uc744 24\uc2dc\uac04 \uc804\uba74 \uac1c\ubc29\ud558\uae30\ub85c \ud558\uba74\uc11c \ud1b5\ud589\u00b7\uc0ac\uc9c4 \ucd2c\uc601 \ub4f1\uc758 \uc81c\ud55c\uc744 \uc5c6\uc560\uae30\ub85c \ud588\ub2e4. \uccad\uc640\ub300 \ud648\ud398\uc774\uc9c0\uc5d0\ub294 \ub300\ud1b5\ub839\uacbd\ud638\ucc98\uac00 \ub300\ud1b5\ub839 \uacbd\ud638 \uc0ac\uc9c4\uc744 \uacf5\uac1c\ud558\uace0 \uc788\uc73c\uba70 \uc8fc\uc601\ud6c8 \ucc98\uc7a5\ub3c4 \uac1c\uc778 \ud398\uc774\uc2a4\ubd81\uc5d0 2 ~ 3\uc77c\uc5d0 \ud55c \ubc88 \uaf34\ub85c \ud648\ud398\uc774\uc9c0 \uc0ac\uc9c4\uc744 \uacf5\uc720\ud558\uac70\ub098 \uacbd\ud638\ucc98 \uc18c\uc2dd\uc744 \uc804\ud558\uace0 \uc788\ub2e4. \ub2e4\ub9cc, \uc2e4\ub0b4\uc640 \ub2ec\ub9ac \uc57c\uc678\uc640 \uac19\uc740 \uc5f4\ub9b0 \uacf5\uac04\uc5d0\uc11c\ub294 \ub9cc\uc77c\uc758 \uc0ac\ud0dc\uc5d0 \ub300\ube44\ud574\uc57c \ud558\uae30 \ub54c\ubb38\uc5d0 \ub300\ud1b5\ub839\uc758 \uacbd\ud638 \ubc29\uce68\uc744 \uc774\ud589\ud558\uba74\uc11c\ub3c4 \uc808\ub300\uc801\uc73c\ub85c \uc548\uc804\uc744 \ud655\ubcf4\ud55c\ub2e4\ub294 \uc774\uc911 \uacfc\uc81c\ub97c \ub2ec\uc131\ud574\uc57c \ud558\ub294 \uc0c1\ud669\uc5d0 \ub193\uc774\uae30\ub3c4 \ud588\ub2e4. \uacbd\ucc30\uccad\uc740 \"\ucd5c\uc18c \uc778\uc6d0\ub9cc \ub178\ucd9c\ud558\uace0, \uc2dc\ubbfc \ud1b5\uc81c\ub3c4 \uac00\uae09\uc801 \ucd5c\uc18c\ud654\ud558\ub294 \ubc29\ud5a5\uc73c\ub85c \uacbd\ud638 \uacfc\uc815\uacfc \ubc29\ubc95\uc774 \ub2ec\ub77c\uc838\uc57c \ud558\ub294 \uc0c1\ud669\"\uc774\ub77c\uba74\uc11c\ub3c4 \"'\uc808\ub300 \uc548\uc804 \ud655\ubcf4'\ub77c\ub294 \uacbd\ud638 \ubaa9\uc801\uc740 \uc5b4\ub5a4 \uacbd\uc6b0\uc5d0\ub3c4 \ub2ec\uc131\ud574\uc57c \ud55c\ub2e4\"\uace0 \ub9d0\ud588\ub2e4. \uc8fc\uc601\ud6c8 \ucc98\uc7a5 \uc5ed\uc2dc \"\ubb38 \ub300\ud1b5\ub839\uc758 '\uce5c\uadfc\ud55c \uacbd\ud638, \uc5f4\ub9b0 \uacbd\ud638, \ub0ae\uc740 \uacbd\ud638'\ub294 \uac15\ud558\uace0 \uc644\ubcbd\ud55c \uacbd\ud638\ub97c \uadfc\uac04\uc73c\ub85c \ud558\uace0 \uc788\ub2e4\"\uace0 \uc124\uba85\ud588\ub2e4.", "answer": "24\uc2dc\uac04 \uc804\uba74 \uac1c\ubc29", "sentence": "22\uc77c\uc5d0\ub294 5\uc2dc 30\ubd84\ubd80\ud130 20\uc2dc\uae4c\uc9c0\ub9cc \uac1c\ubc29\ub418\ub358 \uccad\uc640\ub300 \uc55e\uae38\uc744 24\uc2dc\uac04 \uc804\uba74 \uac1c\ubc29 \ud558\uae30\ub85c \ud558\uba74\uc11c \ud1b5\ud589\u00b7\uc0ac\uc9c4 \ucd2c\uc601 \ub4f1\uc758 \uc81c\ud55c\uc744 \uc5c6\uc560\uae30\ub85c \ud588\ub2e4.", "paragraph_sentence": "\uc774\uc804 \uc815\uad8c\uc758 \ubd88\ud1b5 \uc774\ubbf8\uc9c0\uac00 \uc2ec\uac01\ud588\ub358 \ud0d3\ub3c4 \uc788\uae30\uc5d0, \ubb38\uc7ac\uc778 \uc815\ubd80\ub294 \ucd9c\ubc94\ud55c \ub4a4 \uc18c\ud1b5\uacfc \ud568\uaed8 \ud0c8\uad8c\uc704\ub97c \uac15\uc870\ud588\ub2e4. 5\uc6d4 10\uc77c \ucde8\uc784\uc2dd\uc5d0\ub294 \ud5cc\ubc95\uae30\uad00\uc7a5\uc778 5\ubd80 \uc694\uc778\uacfc \uad6d\ud68c\uc758\uc6d0, \uad6d\ubb34\uc704\uc6d0, \uad70 \uc9c0\ud718\uad00 \ub4f1 300\uc5ec \uba85\ub9cc\uc744 \ucd08\ub300\ud558\uc5ec \uac04\uc18c\ud558\uac8c \uc9c4\ud589\ud588\ub294\ub370 \uc218\ub9cc \uba85\uc744 \ucd08\ub300\ud558\uc5ec \uad6d\ud68c\uc758\uc0ac\ub2f9\uc5d0\uc11c \uc131\ub300\ud558\uac8c \uc5f4\ub9ac\ub358 \uc5ed\ub300 \ucde8\uc784\uc2dd\uacfc\ub294 \ub2e4\ub978 \ud48d\uacbd\uc774\uc5c8\ub2e4. \uc774\ub294 \ucde8\uc784\uc2dd\uc744 \uc900\ube44\ud560 \uc2dc\uac04\uc870\ucc28 \uc5c6\uc5c8\ub358 \ud0d3\ub3c4 \uc788\uc9c0\ub9cc, \uad6d\uc815 \ud63c\ub780\uc744 \ube60\ub974\uac8c \uc218\uc2b5\ud558\uace0 \uc2e0\uc18d\ud788 \ud0c0\uac1c\ud558\uaca0\ub2e4\ub294 \ub300\ud1b5\ub839\uc758 \uc758\uc9c0\uac00 \ubc18\uc601\ub410\ub2e4\ub294 \ud6c4\ubb38\uc774\ub2e4. 11\uc77c\uc5d0\ub294 \uc218\uc11d\uc774 \uc544\ub2cc \ube44\uc11c\uad00\uacfc \uacb8\uc0c1\uc744 \ud558\uace0 \uc2e0\uc784 \uc218\uc11d \ub4f1\uacfc \uaca9\uc758 \uc5c6\ub294 \uc624\ucc2c\uc744 \uac00\uc84c\uc73c\uba70, \uc9c1\ud6c4\uc5d0\ub294 \uacbd\ub0b4\uc5d0\uc11c \ucee4\ud53c \uc0b0\ucc45\uc744 \ud588\ub2e4. \uacbd\ud638\uc5d0 \uc788\uc5b4\uc11c\ub3c4 '\uc5f4\ub9b0 \uacbd\ud638', '\ub0ae\uc740 \uacbd\ud638'\ub97c \uc9c0\ud5a5\ud558\uba70 \uc2dc\ubbfc\ub4e4\uc5d0\uac8c \ub2e4\uac00\uac00\uae30 \uc704\ud55c \ub178\ub825\uc744 \ubcf4\uc600\ub2e4. 6\uc6d4 \ud56d\uc7c1 \uae30\ub150\uc2dd\uc5d0 10\ub144 \ub9cc\uc5d0 \ud604\uc9c1 \ub300\ud1b5\ub839\uc774 \ucc38\uc11d\ud588\uc9c0\ub9cc '\ub300\ud1b5\ub839\uc774 \ucc38\uc11d\ud55c \uc57c\uc678\ud589\uc0ac'\uce58\uace0\ub294 \uacbd\ube44\u00b7\uacbd\ud638 \uc778\ub825\uc758 \uc874\uc7ac\uac00 \ub450\ub4dc\ub7ec\uc9c0\uc9c0 \uc54a\uc558\ub2e4. \uc758\uc2ec\uc2a4\ub7ec\uc6b4 \uc778\ubb3c\uc744 \uac00\ub824\ub0b4\uae30 \uc704\ud574 \uae08\uc18d\ud0d0\uc9c0\uae30\ub85c \uc18c\uc9c0\ud488\uc744 \uac80\uc0ac\ud558\ub294 \uc808\ucc28 \uc815\ub3c4\ub9cc \uac70\uce60 \ubfd0, \ud589\uc0ac\uc7a5\uc5d0 \ubc30\uce58\ub41c \uacbd\ucc30\uad00\ub4e4\uc740 \uc2dc\ubbfc\ub4e4\uc758 \ud1b5\ud589\uc5d0 \uc801\uadf9\uc801\uc73c\ub85c \ud611\uc870\ud558\ub294 \ubaa8\uc2b5\uc744 \ubcf4\uc600\ub2e4. 22\uc77c\uc5d0\ub294 5\uc2dc 30\ubd84\ubd80\ud130 20\uc2dc\uae4c\uc9c0\ub9cc \uac1c\ubc29\ub418\ub358 \uccad\uc640\ub300 \uc55e\uae38\uc744 24\uc2dc\uac04 \uc804\uba74 \uac1c\ubc29 \ud558\uae30\ub85c \ud558\uba74\uc11c \ud1b5\ud589\u00b7\uc0ac\uc9c4 \ucd2c\uc601 \ub4f1\uc758 \uc81c\ud55c\uc744 \uc5c6\uc560\uae30\ub85c \ud588\ub2e4. \uccad\uc640\ub300 \ud648\ud398\uc774\uc9c0\uc5d0\ub294 \ub300\ud1b5\ub839\uacbd\ud638\ucc98\uac00 \ub300\ud1b5\ub839 \uacbd\ud638 \uc0ac\uc9c4\uc744 \uacf5\uac1c\ud558\uace0 \uc788\uc73c\uba70 \uc8fc\uc601\ud6c8 \ucc98\uc7a5\ub3c4 \uac1c\uc778 \ud398\uc774\uc2a4\ubd81\uc5d0 2 ~ 3\uc77c\uc5d0 \ud55c \ubc88 \uaf34\ub85c \ud648\ud398\uc774\uc9c0 \uc0ac\uc9c4\uc744 \uacf5\uc720\ud558\uac70\ub098 \uacbd\ud638\ucc98 \uc18c\uc2dd\uc744 \uc804\ud558\uace0 \uc788\ub2e4. \ub2e4\ub9cc, \uc2e4\ub0b4\uc640 \ub2ec\ub9ac \uc57c\uc678\uc640 \uac19\uc740 \uc5f4\ub9b0 \uacf5\uac04\uc5d0\uc11c\ub294 \ub9cc\uc77c\uc758 \uc0ac\ud0dc\uc5d0 \ub300\ube44\ud574\uc57c \ud558\uae30 \ub54c\ubb38\uc5d0 \ub300\ud1b5\ub839\uc758 \uacbd\ud638 \ubc29\uce68\uc744 \uc774\ud589\ud558\uba74\uc11c\ub3c4 \uc808\ub300\uc801\uc73c\ub85c \uc548\uc804\uc744 \ud655\ubcf4\ud55c\ub2e4\ub294 \uc774\uc911 \uacfc\uc81c\ub97c \ub2ec\uc131\ud574\uc57c \ud558\ub294 \uc0c1\ud669\uc5d0 \ub193\uc774\uae30\ub3c4 \ud588\ub2e4. \uacbd\ucc30\uccad\uc740 \"\ucd5c\uc18c \uc778\uc6d0\ub9cc \ub178\ucd9c\ud558\uace0, \uc2dc\ubbfc \ud1b5\uc81c\ub3c4 \uac00\uae09\uc801 \ucd5c\uc18c\ud654\ud558\ub294 \ubc29\ud5a5\uc73c\ub85c \uacbd\ud638 \uacfc\uc815\uacfc \ubc29\ubc95\uc774 \ub2ec\ub77c\uc838\uc57c \ud558\ub294 \uc0c1\ud669\"\uc774\ub77c\uba74\uc11c\ub3c4 \"'\uc808\ub300 \uc548\uc804 \ud655\ubcf4'\ub77c\ub294 \uacbd\ud638 \ubaa9\uc801\uc740 \uc5b4\ub5a4 \uacbd\uc6b0\uc5d0\ub3c4 \ub2ec\uc131\ud574\uc57c \ud55c\ub2e4\"\uace0 \ub9d0\ud588\ub2e4. \uc8fc\uc601\ud6c8 \ucc98\uc7a5 \uc5ed\uc2dc \"\ubb38 \ub300\ud1b5\ub839\uc758 '\uce5c\uadfc\ud55c \uacbd\ud638, \uc5f4\ub9b0 \uacbd\ud638, \ub0ae\uc740 \uacbd\ud638'\ub294 \uac15\ud558\uace0 \uc644\ubcbd\ud55c \uacbd\ud638\ub97c \uadfc\uac04\uc73c\ub85c \ud558\uace0 \uc788\ub2e4\"\uace0 \uc124\uba85\ud588\ub2e4.", "paragraph_answer": "\uc774\uc804 \uc815\uad8c\uc758 \ubd88\ud1b5 \uc774\ubbf8\uc9c0\uac00 \uc2ec\uac01\ud588\ub358 \ud0d3\ub3c4 \uc788\uae30\uc5d0, \ubb38\uc7ac\uc778 \uc815\ubd80\ub294 \ucd9c\ubc94\ud55c \ub4a4 \uc18c\ud1b5\uacfc \ud568\uaed8 \ud0c8\uad8c\uc704\ub97c \uac15\uc870\ud588\ub2e4. 5\uc6d4 10\uc77c \ucde8\uc784\uc2dd\uc5d0\ub294 \ud5cc\ubc95\uae30\uad00\uc7a5\uc778 5\ubd80 \uc694\uc778\uacfc \uad6d\ud68c\uc758\uc6d0, \uad6d\ubb34\uc704\uc6d0, \uad70 \uc9c0\ud718\uad00 \ub4f1 300\uc5ec \uba85\ub9cc\uc744 \ucd08\ub300\ud558\uc5ec \uac04\uc18c\ud558\uac8c \uc9c4\ud589\ud588\ub294\ub370 \uc218\ub9cc \uba85\uc744 \ucd08\ub300\ud558\uc5ec \uad6d\ud68c\uc758\uc0ac\ub2f9\uc5d0\uc11c \uc131\ub300\ud558\uac8c \uc5f4\ub9ac\ub358 \uc5ed\ub300 \ucde8\uc784\uc2dd\uacfc\ub294 \ub2e4\ub978 \ud48d\uacbd\uc774\uc5c8\ub2e4. \uc774\ub294 \ucde8\uc784\uc2dd\uc744 \uc900\ube44\ud560 \uc2dc\uac04\uc870\ucc28 \uc5c6\uc5c8\ub358 \ud0d3\ub3c4 \uc788\uc9c0\ub9cc, \uad6d\uc815 \ud63c\ub780\uc744 \ube60\ub974\uac8c \uc218\uc2b5\ud558\uace0 \uc2e0\uc18d\ud788 \ud0c0\uac1c\ud558\uaca0\ub2e4\ub294 \ub300\ud1b5\ub839\uc758 \uc758\uc9c0\uac00 \ubc18\uc601\ub410\ub2e4\ub294 \ud6c4\ubb38\uc774\ub2e4. 11\uc77c\uc5d0\ub294 \uc218\uc11d\uc774 \uc544\ub2cc \ube44\uc11c\uad00\uacfc \uacb8\uc0c1\uc744 \ud558\uace0 \uc2e0\uc784 \uc218\uc11d \ub4f1\uacfc \uaca9\uc758 \uc5c6\ub294 \uc624\ucc2c\uc744 \uac00\uc84c\uc73c\uba70, \uc9c1\ud6c4\uc5d0\ub294 \uacbd\ub0b4\uc5d0\uc11c \ucee4\ud53c \uc0b0\ucc45\uc744 \ud588\ub2e4. \uacbd\ud638\uc5d0 \uc788\uc5b4\uc11c\ub3c4 '\uc5f4\ub9b0 \uacbd\ud638', '\ub0ae\uc740 \uacbd\ud638'\ub97c \uc9c0\ud5a5\ud558\uba70 \uc2dc\ubbfc\ub4e4\uc5d0\uac8c \ub2e4\uac00\uac00\uae30 \uc704\ud55c \ub178\ub825\uc744 \ubcf4\uc600\ub2e4. 6\uc6d4 \ud56d\uc7c1 \uae30\ub150\uc2dd\uc5d0 10\ub144 \ub9cc\uc5d0 \ud604\uc9c1 \ub300\ud1b5\ub839\uc774 \ucc38\uc11d\ud588\uc9c0\ub9cc '\ub300\ud1b5\ub839\uc774 \ucc38\uc11d\ud55c \uc57c\uc678\ud589\uc0ac'\uce58\uace0\ub294 \uacbd\ube44\u00b7\uacbd\ud638 \uc778\ub825\uc758 \uc874\uc7ac\uac00 \ub450\ub4dc\ub7ec\uc9c0\uc9c0 \uc54a\uc558\ub2e4. \uc758\uc2ec\uc2a4\ub7ec\uc6b4 \uc778\ubb3c\uc744 \uac00\ub824\ub0b4\uae30 \uc704\ud574 \uae08\uc18d\ud0d0\uc9c0\uae30\ub85c \uc18c\uc9c0\ud488\uc744 \uac80\uc0ac\ud558\ub294 \uc808\ucc28 \uc815\ub3c4\ub9cc \uac70\uce60 \ubfd0, \ud589\uc0ac\uc7a5\uc5d0 \ubc30\uce58\ub41c \uacbd\ucc30\uad00\ub4e4\uc740 \uc2dc\ubbfc\ub4e4\uc758 \ud1b5\ud589\uc5d0 \uc801\uadf9\uc801\uc73c\ub85c \ud611\uc870\ud558\ub294 \ubaa8\uc2b5\uc744 \ubcf4\uc600\ub2e4. 22\uc77c\uc5d0\ub294 5\uc2dc 30\ubd84\ubd80\ud130 20\uc2dc\uae4c\uc9c0\ub9cc \uac1c\ubc29\ub418\ub358 \uccad\uc640\ub300 \uc55e\uae38\uc744 24\uc2dc\uac04 \uc804\uba74 \uac1c\ubc29 \ud558\uae30\ub85c \ud558\uba74\uc11c \ud1b5\ud589\u00b7\uc0ac\uc9c4 \ucd2c\uc601 \ub4f1\uc758 \uc81c\ud55c\uc744 \uc5c6\uc560\uae30\ub85c \ud588\ub2e4. \uccad\uc640\ub300 \ud648\ud398\uc774\uc9c0\uc5d0\ub294 \ub300\ud1b5\ub839\uacbd\ud638\ucc98\uac00 \ub300\ud1b5\ub839 \uacbd\ud638 \uc0ac\uc9c4\uc744 \uacf5\uac1c\ud558\uace0 \uc788\uc73c\uba70 \uc8fc\uc601\ud6c8 \ucc98\uc7a5\ub3c4 \uac1c\uc778 \ud398\uc774\uc2a4\ubd81\uc5d0 2 ~ 3\uc77c\uc5d0 \ud55c \ubc88 \uaf34\ub85c \ud648\ud398\uc774\uc9c0 \uc0ac\uc9c4\uc744 \uacf5\uc720\ud558\uac70\ub098 \uacbd\ud638\ucc98 \uc18c\uc2dd\uc744 \uc804\ud558\uace0 \uc788\ub2e4. \ub2e4\ub9cc, \uc2e4\ub0b4\uc640 \ub2ec\ub9ac \uc57c\uc678\uc640 \uac19\uc740 \uc5f4\ub9b0 \uacf5\uac04\uc5d0\uc11c\ub294 \ub9cc\uc77c\uc758 \uc0ac\ud0dc\uc5d0 \ub300\ube44\ud574\uc57c \ud558\uae30 \ub54c\ubb38\uc5d0 \ub300\ud1b5\ub839\uc758 \uacbd\ud638 \ubc29\uce68\uc744 \uc774\ud589\ud558\uba74\uc11c\ub3c4 \uc808\ub300\uc801\uc73c\ub85c \uc548\uc804\uc744 \ud655\ubcf4\ud55c\ub2e4\ub294 \uc774\uc911 \uacfc\uc81c\ub97c \ub2ec\uc131\ud574\uc57c \ud558\ub294 \uc0c1\ud669\uc5d0 \ub193\uc774\uae30\ub3c4 \ud588\ub2e4. \uacbd\ucc30\uccad\uc740 \"\ucd5c\uc18c \uc778\uc6d0\ub9cc \ub178\ucd9c\ud558\uace0, \uc2dc\ubbfc \ud1b5\uc81c\ub3c4 \uac00\uae09\uc801 \ucd5c\uc18c\ud654\ud558\ub294 \ubc29\ud5a5\uc73c\ub85c \uacbd\ud638 \uacfc\uc815\uacfc \ubc29\ubc95\uc774 \ub2ec\ub77c\uc838\uc57c \ud558\ub294 \uc0c1\ud669\"\uc774\ub77c\uba74\uc11c\ub3c4 \"'\uc808\ub300 \uc548\uc804 \ud655\ubcf4'\ub77c\ub294 \uacbd\ud638 \ubaa9\uc801\uc740 \uc5b4\ub5a4 \uacbd\uc6b0\uc5d0\ub3c4 \ub2ec\uc131\ud574\uc57c \ud55c\ub2e4\"\uace0 \ub9d0\ud588\ub2e4. \uc8fc\uc601\ud6c8 \ucc98\uc7a5 \uc5ed\uc2dc \"\ubb38 \ub300\ud1b5\ub839\uc758 '\uce5c\uadfc\ud55c \uacbd\ud638, \uc5f4\ub9b0 \uacbd\ud638, \ub0ae\uc740 \uacbd\ud638'\ub294 \uac15\ud558\uace0 \uc644\ubcbd\ud55c \uacbd\ud638\ub97c \uadfc\uac04\uc73c\ub85c \ud558\uace0 \uc788\ub2e4\"\uace0 \uc124\uba85\ud588\ub2e4.", "sentence_answer": "22\uc77c\uc5d0\ub294 5\uc2dc 30\ubd84\ubd80\ud130 20\uc2dc\uae4c\uc9c0\ub9cc \uac1c\ubc29\ub418\ub358 \uccad\uc640\ub300 \uc55e\uae38\uc744 24\uc2dc\uac04 \uc804\uba74 \uac1c\ubc29 \ud558\uae30\ub85c \ud558\uba74\uc11c \ud1b5\ud589\u00b7\uc0ac\uc9c4 \ucd2c\uc601 \ub4f1\uc758 \uc81c\ud55c\uc744 \uc5c6\uc560\uae30\ub85c \ud588\ub2e4.", "paragraph_id": "6546926-58-1", "paragraph_question": "question: \uac1c\ubc29\uc2dc\uac04\uc774 \uc815\ud574\uc838 \uc788\ub358 \uccad\uc640\ub300 \uc55e\uae38\uc744 \ubb38\uc7ac\uc778 \uc815\ubd80\uac00 \ub4e4\uc5b4\uc11c\uba74\uc11c \uac1c\ubc29\uc2dc\uac04\uc774 \uc5b4\ub5bb\uac8c \ubc14\ub00c\uc5c8\ub098?, context: \uc774\uc804 \uc815\uad8c\uc758 \ubd88\ud1b5 \uc774\ubbf8\uc9c0\uac00 \uc2ec\uac01\ud588\ub358 \ud0d3\ub3c4 \uc788\uae30\uc5d0, \ubb38\uc7ac\uc778 \uc815\ubd80\ub294 \ucd9c\ubc94\ud55c \ub4a4 \uc18c\ud1b5\uacfc \ud568\uaed8 \ud0c8\uad8c\uc704\ub97c \uac15\uc870\ud588\ub2e4. 5\uc6d4 10\uc77c \ucde8\uc784\uc2dd\uc5d0\ub294 \ud5cc\ubc95\uae30\uad00\uc7a5\uc778 5\ubd80 \uc694\uc778\uacfc \uad6d\ud68c\uc758\uc6d0, \uad6d\ubb34\uc704\uc6d0, \uad70 \uc9c0\ud718\uad00 \ub4f1 300\uc5ec \uba85\ub9cc\uc744 \ucd08\ub300\ud558\uc5ec \uac04\uc18c\ud558\uac8c \uc9c4\ud589\ud588\ub294\ub370 \uc218\ub9cc \uba85\uc744 \ucd08\ub300\ud558\uc5ec \uad6d\ud68c\uc758\uc0ac\ub2f9\uc5d0\uc11c \uc131\ub300\ud558\uac8c \uc5f4\ub9ac\ub358 \uc5ed\ub300 \ucde8\uc784\uc2dd\uacfc\ub294 \ub2e4\ub978 \ud48d\uacbd\uc774\uc5c8\ub2e4. \uc774\ub294 \ucde8\uc784\uc2dd\uc744 \uc900\ube44\ud560 \uc2dc\uac04\uc870\ucc28 \uc5c6\uc5c8\ub358 \ud0d3\ub3c4 \uc788\uc9c0\ub9cc, \uad6d\uc815 \ud63c\ub780\uc744 \ube60\ub974\uac8c \uc218\uc2b5\ud558\uace0 \uc2e0\uc18d\ud788 \ud0c0\uac1c\ud558\uaca0\ub2e4\ub294 \ub300\ud1b5\ub839\uc758 \uc758\uc9c0\uac00 \ubc18\uc601\ub410\ub2e4\ub294 \ud6c4\ubb38\uc774\ub2e4. 11\uc77c\uc5d0\ub294 \uc218\uc11d\uc774 \uc544\ub2cc \ube44\uc11c\uad00\uacfc \uacb8\uc0c1\uc744 \ud558\uace0 \uc2e0\uc784 \uc218\uc11d \ub4f1\uacfc \uaca9\uc758 \uc5c6\ub294 \uc624\ucc2c\uc744 \uac00\uc84c\uc73c\uba70, \uc9c1\ud6c4\uc5d0\ub294 \uacbd\ub0b4\uc5d0\uc11c \ucee4\ud53c \uc0b0\ucc45\uc744 \ud588\ub2e4. \uacbd\ud638\uc5d0 \uc788\uc5b4\uc11c\ub3c4 '\uc5f4\ub9b0 \uacbd\ud638', '\ub0ae\uc740 \uacbd\ud638'\ub97c \uc9c0\ud5a5\ud558\uba70 \uc2dc\ubbfc\ub4e4\uc5d0\uac8c \ub2e4\uac00\uac00\uae30 \uc704\ud55c \ub178\ub825\uc744 \ubcf4\uc600\ub2e4. 6\uc6d4 \ud56d\uc7c1 \uae30\ub150\uc2dd\uc5d0 10\ub144 \ub9cc\uc5d0 \ud604\uc9c1 \ub300\ud1b5\ub839\uc774 \ucc38\uc11d\ud588\uc9c0\ub9cc '\ub300\ud1b5\ub839\uc774 \ucc38\uc11d\ud55c \uc57c\uc678\ud589\uc0ac'\uce58\uace0\ub294 \uacbd\ube44\u00b7\uacbd\ud638 \uc778\ub825\uc758 \uc874\uc7ac\uac00 \ub450\ub4dc\ub7ec\uc9c0\uc9c0 \uc54a\uc558\ub2e4. \uc758\uc2ec\uc2a4\ub7ec\uc6b4 \uc778\ubb3c\uc744 \uac00\ub824\ub0b4\uae30 \uc704\ud574 \uae08\uc18d\ud0d0\uc9c0\uae30\ub85c \uc18c\uc9c0\ud488\uc744 \uac80\uc0ac\ud558\ub294 \uc808\ucc28 \uc815\ub3c4\ub9cc \uac70\uce60 \ubfd0, \ud589\uc0ac\uc7a5\uc5d0 \ubc30\uce58\ub41c \uacbd\ucc30\uad00\ub4e4\uc740 \uc2dc\ubbfc\ub4e4\uc758 \ud1b5\ud589\uc5d0 \uc801\uadf9\uc801\uc73c\ub85c \ud611\uc870\ud558\ub294 \ubaa8\uc2b5\uc744 \ubcf4\uc600\ub2e4. 22\uc77c\uc5d0\ub294 5\uc2dc 30\ubd84\ubd80\ud130 20\uc2dc\uae4c\uc9c0\ub9cc \uac1c\ubc29\ub418\ub358 \uccad\uc640\ub300 \uc55e\uae38\uc744 24\uc2dc\uac04 \uc804\uba74 \uac1c\ubc29\ud558\uae30\ub85c \ud558\uba74\uc11c \ud1b5\ud589\u00b7\uc0ac\uc9c4 \ucd2c\uc601 \ub4f1\uc758 \uc81c\ud55c\uc744 \uc5c6\uc560\uae30\ub85c \ud588\ub2e4. \uccad\uc640\ub300 \ud648\ud398\uc774\uc9c0\uc5d0\ub294 \ub300\ud1b5\ub839\uacbd\ud638\ucc98\uac00 \ub300\ud1b5\ub839 \uacbd\ud638 \uc0ac\uc9c4\uc744 \uacf5\uac1c\ud558\uace0 \uc788\uc73c\uba70 \uc8fc\uc601\ud6c8 \ucc98\uc7a5\ub3c4 \uac1c\uc778 \ud398\uc774\uc2a4\ubd81\uc5d0 2 ~ 3\uc77c\uc5d0 \ud55c \ubc88 \uaf34\ub85c \ud648\ud398\uc774\uc9c0 \uc0ac\uc9c4\uc744 \uacf5\uc720\ud558\uac70\ub098 \uacbd\ud638\ucc98 \uc18c\uc2dd\uc744 \uc804\ud558\uace0 \uc788\ub2e4. \ub2e4\ub9cc, \uc2e4\ub0b4\uc640 \ub2ec\ub9ac \uc57c\uc678\uc640 \uac19\uc740 \uc5f4\ub9b0 \uacf5\uac04\uc5d0\uc11c\ub294 \ub9cc\uc77c\uc758 \uc0ac\ud0dc\uc5d0 \ub300\ube44\ud574\uc57c \ud558\uae30 \ub54c\ubb38\uc5d0 \ub300\ud1b5\ub839\uc758 \uacbd\ud638 \ubc29\uce68\uc744 \uc774\ud589\ud558\uba74\uc11c\ub3c4 \uc808\ub300\uc801\uc73c\ub85c \uc548\uc804\uc744 \ud655\ubcf4\ud55c\ub2e4\ub294 \uc774\uc911 \uacfc\uc81c\ub97c \ub2ec\uc131\ud574\uc57c \ud558\ub294 \uc0c1\ud669\uc5d0 \ub193\uc774\uae30\ub3c4 \ud588\ub2e4. \uacbd\ucc30\uccad\uc740 \"\ucd5c\uc18c \uc778\uc6d0\ub9cc \ub178\ucd9c\ud558\uace0, \uc2dc\ubbfc \ud1b5\uc81c\ub3c4 \uac00\uae09\uc801 \ucd5c\uc18c\ud654\ud558\ub294 \ubc29\ud5a5\uc73c\ub85c \uacbd\ud638 \uacfc\uc815\uacfc \ubc29\ubc95\uc774 \ub2ec\ub77c\uc838\uc57c \ud558\ub294 \uc0c1\ud669\"\uc774\ub77c\uba74\uc11c\ub3c4 \"'\uc808\ub300 \uc548\uc804 \ud655\ubcf4'\ub77c\ub294 \uacbd\ud638 \ubaa9\uc801\uc740 \uc5b4\ub5a4 \uacbd\uc6b0\uc5d0\ub3c4 \ub2ec\uc131\ud574\uc57c \ud55c\ub2e4\"\uace0 \ub9d0\ud588\ub2e4. \uc8fc\uc601\ud6c8 \ucc98\uc7a5 \uc5ed\uc2dc \"\ubb38 \ub300\ud1b5\ub839\uc758 '\uce5c\uadfc\ud55c \uacbd\ud638, \uc5f4\ub9b0 \uacbd\ud638, \ub0ae\uc740 \uacbd\ud638'\ub294 \uac15\ud558\uace0 \uc644\ubcbd\ud55c \uacbd\ud638\ub97c \uadfc\uac04\uc73c\ub85c \ud558\uace0 \uc788\ub2e4\"\uace0 \uc124\uba85\ud588\ub2e4."} {"question": "\uc138\uacc4 \ucd5c\ucd08\uc758 \uad6d\uc81c \ubc29\uc1a1\ub9dd\uc740 \ubb34\uc5c7\uc778\uac00?", "paragraph": "\uc804\uc7c1\uc5d0\uc11c \ubc97\uc5b4\ub09c \ub4a4\uc5d0 \uc138\uacc4 \ucd5c\ucd08\ub85c \uac70\ub300\ud55c \uad6d\uc81c \ubc29\uc1a1\ub9dd\uc778 BBC\uac00 \uc0dd\uaca8\ub0ac\ub2e4. \uc601\uad6d\uc758 \ub178\ub3d9 \uc6b4\ub3d9\uc740 19\uc138\uae30 \ud6c4\ubc18\ubd80\ud130 \ud655\uc7a5\ud574 \uc654\uace0, 1924\ub144\uc5d0 \uc81c1\ucc28 \ub178\ub3d9\ub2f9 \ub0b4\uac01\uc774 \uc9d1\uad8c\ud558\uc600\ub2e4. \uc81c2\ucc28 \uc138\uacc4 \ub300\uc804 \ub54c \uc601\uad6d\uc740, \uce90\ub098\ub2e4\uc640 \uc624\uc2a4\ud2b8\ub808\uc77c\ub9ac\uc544, \ub274\uc9c8\ub79c\ub4dc, \ub0a8\uc544\ud504\ub9ac\uce74 \uacf5\ud654\uad6d, \uc778\ub3c4\ub97c \ud3ec\ud568\ud55c \uc601\uad6d \uc5f0\ubc29\uacfc \uc5f0\ud569\ud558\uc5ec, \uadf8\ub9ac\uace0 \ub098\uc911\uc5d0\ub294 \uc5f0\ud569\uad6d\uacfc \uc5f0\ud569\ud558\uc5ec \ub098\uce58 \ub3c5\uc77c\uacfc \uc2f8\uc6e0\ub2e4. \uc804\uc2dc \uc9c0\ub3c4\uc790 \uc708\uc2a4\ud134 \ucc98\uce60\uacfc \uadf8\uc758 \ud6c4\uc784 \ud074\ub808\uba3c\ud2b8 \uc560\ud2c0\ub9ac(Clement Attlee)\ub294 3\ub300 \uac15\uad6d\uc758 \ud55c \ucd95\uc73c\ub85c \uc804\ud6c4 \uc138\uacc4\ub97c \uacc4\ud68d\ud55c\ub2e4. \uadf8\ub7ec\ub098 \uc81c2\ucc28 \uc138\uacc4 \ub300\uc804\uc73c\ub85c \uc601\uad6d\uc5d0 \uc7ac\uc815\uc801\uc778 \ud0c0\uaca9\uc744 \uc8fc\uc5c8\ub2e4. \uacbd\uc81c\uc801\uc73c\ub85c \ube44\uc2fc \uc804\uc2dc \ub300\ubd80\uae08, \ubbf8\uad6d\uacfc \uce90\ub098\ub2e4\uc5d0\uc11c 1945\ub144\uc5d0 \ube4c\ub824\uc900 \ub300\ubd80\uae08\uc774 \uc804\ud6c4\uc5d0 \ubbf8\uad6d\uc758 \ub9c8\uc15c \ud50c\ub79c \uc6d0\uc870\uc640 \ud569\uccd0\uc838\uc11c \uc601\uad6d\uc740 \ubcf5\uad6c\ub418\uae30 \uc2dc\uc791\ud558\uc600\ub2e4.", "answer": "BBC", "sentence": "\uc804\uc7c1\uc5d0\uc11c \ubc97\uc5b4\ub09c \ub4a4\uc5d0 \uc138\uacc4 \ucd5c\ucd08\ub85c \uac70\ub300\ud55c \uad6d\uc81c \ubc29\uc1a1\ub9dd\uc778 BBC \uac00 \uc0dd\uaca8\ub0ac\ub2e4.", "paragraph_sentence": " \uc804\uc7c1\uc5d0\uc11c \ubc97\uc5b4\ub09c \ub4a4\uc5d0 \uc138\uacc4 \ucd5c\ucd08\ub85c \uac70\ub300\ud55c \uad6d\uc81c \ubc29\uc1a1\ub9dd\uc778 BBC \uac00 \uc0dd\uaca8\ub0ac\ub2e4. \uc601\uad6d\uc758 \ub178\ub3d9 \uc6b4\ub3d9\uc740 19\uc138\uae30 \ud6c4\ubc18\ubd80\ud130 \ud655\uc7a5\ud574 \uc654\uace0, 1924\ub144\uc5d0 \uc81c1\ucc28 \ub178\ub3d9\ub2f9 \ub0b4\uac01\uc774 \uc9d1\uad8c\ud558\uc600\ub2e4. \uc81c2\ucc28 \uc138\uacc4 \ub300\uc804 \ub54c \uc601\uad6d\uc740, \uce90\ub098\ub2e4\uc640 \uc624\uc2a4\ud2b8\ub808\uc77c\ub9ac\uc544, \ub274\uc9c8\ub79c\ub4dc, \ub0a8\uc544\ud504\ub9ac\uce74 \uacf5\ud654\uad6d, \uc778\ub3c4\ub97c \ud3ec\ud568\ud55c \uc601\uad6d \uc5f0\ubc29\uacfc \uc5f0\ud569\ud558\uc5ec, \uadf8\ub9ac\uace0 \ub098\uc911\uc5d0\ub294 \uc5f0\ud569\uad6d\uacfc \uc5f0\ud569\ud558\uc5ec \ub098\uce58 \ub3c5\uc77c\uacfc \uc2f8\uc6e0\ub2e4. \uc804\uc2dc \uc9c0\ub3c4\uc790 \uc708\uc2a4\ud134 \ucc98\uce60\uacfc \uadf8\uc758 \ud6c4\uc784 \ud074\ub808\uba3c\ud2b8 \uc560\ud2c0\ub9ac(Clement Attlee)\ub294 3\ub300 \uac15\uad6d\uc758 \ud55c \ucd95\uc73c\ub85c \uc804\ud6c4 \uc138\uacc4\ub97c \uacc4\ud68d\ud55c\ub2e4. \uadf8\ub7ec\ub098 \uc81c2\ucc28 \uc138\uacc4 \ub300\uc804\uc73c\ub85c \uc601\uad6d\uc5d0 \uc7ac\uc815\uc801\uc778 \ud0c0\uaca9\uc744 \uc8fc\uc5c8\ub2e4. \uacbd\uc81c\uc801\uc73c\ub85c \ube44\uc2fc \uc804\uc2dc \ub300\ubd80\uae08, \ubbf8\uad6d\uacfc \uce90\ub098\ub2e4\uc5d0\uc11c 1945\ub144\uc5d0 \ube4c\ub824\uc900 \ub300\ubd80\uae08\uc774 \uc804\ud6c4\uc5d0 \ubbf8\uad6d\uc758 \ub9c8\uc15c \ud50c\ub79c \uc6d0\uc870\uc640 \ud569\uccd0\uc838\uc11c \uc601\uad6d\uc740 \ubcf5\uad6c\ub418\uae30 \uc2dc\uc791\ud558\uc600\ub2e4.", "paragraph_answer": "\uc804\uc7c1\uc5d0\uc11c \ubc97\uc5b4\ub09c \ub4a4\uc5d0 \uc138\uacc4 \ucd5c\ucd08\ub85c \uac70\ub300\ud55c \uad6d\uc81c \ubc29\uc1a1\ub9dd\uc778 BBC \uac00 \uc0dd\uaca8\ub0ac\ub2e4. \uc601\uad6d\uc758 \ub178\ub3d9 \uc6b4\ub3d9\uc740 19\uc138\uae30 \ud6c4\ubc18\ubd80\ud130 \ud655\uc7a5\ud574 \uc654\uace0, 1924\ub144\uc5d0 \uc81c1\ucc28 \ub178\ub3d9\ub2f9 \ub0b4\uac01\uc774 \uc9d1\uad8c\ud558\uc600\ub2e4. \uc81c2\ucc28 \uc138\uacc4 \ub300\uc804 \ub54c \uc601\uad6d\uc740, \uce90\ub098\ub2e4\uc640 \uc624\uc2a4\ud2b8\ub808\uc77c\ub9ac\uc544, \ub274\uc9c8\ub79c\ub4dc, \ub0a8\uc544\ud504\ub9ac\uce74 \uacf5\ud654\uad6d, \uc778\ub3c4\ub97c \ud3ec\ud568\ud55c \uc601\uad6d \uc5f0\ubc29\uacfc \uc5f0\ud569\ud558\uc5ec, \uadf8\ub9ac\uace0 \ub098\uc911\uc5d0\ub294 \uc5f0\ud569\uad6d\uacfc \uc5f0\ud569\ud558\uc5ec \ub098\uce58 \ub3c5\uc77c\uacfc \uc2f8\uc6e0\ub2e4. \uc804\uc2dc \uc9c0\ub3c4\uc790 \uc708\uc2a4\ud134 \ucc98\uce60\uacfc \uadf8\uc758 \ud6c4\uc784 \ud074\ub808\uba3c\ud2b8 \uc560\ud2c0\ub9ac(Clement Attlee)\ub294 3\ub300 \uac15\uad6d\uc758 \ud55c \ucd95\uc73c\ub85c \uc804\ud6c4 \uc138\uacc4\ub97c \uacc4\ud68d\ud55c\ub2e4. \uadf8\ub7ec\ub098 \uc81c2\ucc28 \uc138\uacc4 \ub300\uc804\uc73c\ub85c \uc601\uad6d\uc5d0 \uc7ac\uc815\uc801\uc778 \ud0c0\uaca9\uc744 \uc8fc\uc5c8\ub2e4. \uacbd\uc81c\uc801\uc73c\ub85c \ube44\uc2fc \uc804\uc2dc \ub300\ubd80\uae08, \ubbf8\uad6d\uacfc \uce90\ub098\ub2e4\uc5d0\uc11c 1945\ub144\uc5d0 \ube4c\ub824\uc900 \ub300\ubd80\uae08\uc774 \uc804\ud6c4\uc5d0 \ubbf8\uad6d\uc758 \ub9c8\uc15c \ud50c\ub79c \uc6d0\uc870\uc640 \ud569\uccd0\uc838\uc11c \uc601\uad6d\uc740 \ubcf5\uad6c\ub418\uae30 \uc2dc\uc791\ud558\uc600\ub2e4.", "sentence_answer": "\uc804\uc7c1\uc5d0\uc11c \ubc97\uc5b4\ub09c \ub4a4\uc5d0 \uc138\uacc4 \ucd5c\ucd08\ub85c \uac70\ub300\ud55c \uad6d\uc81c \ubc29\uc1a1\ub9dd\uc778 BBC \uac00 \uc0dd\uaca8\ub0ac\ub2e4.", "paragraph_id": "6458616-4-0", "paragraph_question": "question: \uc138\uacc4 \ucd5c\ucd08\uc758 \uad6d\uc81c \ubc29\uc1a1\ub9dd\uc740 \ubb34\uc5c7\uc778\uac00?, context: \uc804\uc7c1\uc5d0\uc11c \ubc97\uc5b4\ub09c \ub4a4\uc5d0 \uc138\uacc4 \ucd5c\ucd08\ub85c \uac70\ub300\ud55c \uad6d\uc81c \ubc29\uc1a1\ub9dd\uc778 BBC\uac00 \uc0dd\uaca8\ub0ac\ub2e4. \uc601\uad6d\uc758 \ub178\ub3d9 \uc6b4\ub3d9\uc740 19\uc138\uae30 \ud6c4\ubc18\ubd80\ud130 \ud655\uc7a5\ud574 \uc654\uace0, 1924\ub144\uc5d0 \uc81c1\ucc28 \ub178\ub3d9\ub2f9 \ub0b4\uac01\uc774 \uc9d1\uad8c\ud558\uc600\ub2e4. \uc81c2\ucc28 \uc138\uacc4 \ub300\uc804 \ub54c \uc601\uad6d\uc740, \uce90\ub098\ub2e4\uc640 \uc624\uc2a4\ud2b8\ub808\uc77c\ub9ac\uc544, \ub274\uc9c8\ub79c\ub4dc, \ub0a8\uc544\ud504\ub9ac\uce74 \uacf5\ud654\uad6d, \uc778\ub3c4\ub97c \ud3ec\ud568\ud55c \uc601\uad6d \uc5f0\ubc29\uacfc \uc5f0\ud569\ud558\uc5ec, \uadf8\ub9ac\uace0 \ub098\uc911\uc5d0\ub294 \uc5f0\ud569\uad6d\uacfc \uc5f0\ud569\ud558\uc5ec \ub098\uce58 \ub3c5\uc77c\uacfc \uc2f8\uc6e0\ub2e4. \uc804\uc2dc \uc9c0\ub3c4\uc790 \uc708\uc2a4\ud134 \ucc98\uce60\uacfc \uadf8\uc758 \ud6c4\uc784 \ud074\ub808\uba3c\ud2b8 \uc560\ud2c0\ub9ac(Clement Attlee)\ub294 3\ub300 \uac15\uad6d\uc758 \ud55c \ucd95\uc73c\ub85c \uc804\ud6c4 \uc138\uacc4\ub97c \uacc4\ud68d\ud55c\ub2e4. \uadf8\ub7ec\ub098 \uc81c2\ucc28 \uc138\uacc4 \ub300\uc804\uc73c\ub85c \uc601\uad6d\uc5d0 \uc7ac\uc815\uc801\uc778 \ud0c0\uaca9\uc744 \uc8fc\uc5c8\ub2e4. \uacbd\uc81c\uc801\uc73c\ub85c \ube44\uc2fc \uc804\uc2dc \ub300\ubd80\uae08, \ubbf8\uad6d\uacfc \uce90\ub098\ub2e4\uc5d0\uc11c 1945\ub144\uc5d0 \ube4c\ub824\uc900 \ub300\ubd80\uae08\uc774 \uc804\ud6c4\uc5d0 \ubbf8\uad6d\uc758 \ub9c8\uc15c \ud50c\ub79c \uc6d0\uc870\uc640 \ud569\uccd0\uc838\uc11c \uc601\uad6d\uc740 \ubcf5\uad6c\ub418\uae30 \uc2dc\uc791\ud558\uc600\ub2e4."} {"question": "\ubca0\uc774\ube44\ubcf5\uc2a4 \uce21\uc740 \ubbf8\uc544\ub9ac\ubcf5\uc2a4\ub77c\ub294 \ube44\ud558 \ub2e8\uc5b4\ub97c \uc0ac\uc6a9\ud55c \uc774\ud558\ub298\uc5d0\uac8c \uc5b8\uc81c\uae4c\uc9c0 \uc0ac\uacfc\ud558\ub77c\uace0 \ucd5c\ud6c4 \ud1b5\ucca9\uc744 \ubcf4\ub0c8\uc2b5\ub2c8\uae4c?", "paragraph": "\ub85c \ud65c\ub3d9\ud558\ub358 \uc911 \ucf00\uc774\ube14TV M.net\uc758 '\ud799\ud569\ub354\ubc14\uc774\ube0c' \ucf54\ub108\uc5d0 \uac8c\uc2a4\ud2b8\ub85c \ucd9c\uc5f0\ud55c \ub3d9\ub8cc \uac00\uc218 DJ DOC\uc758 \uba64\ubc84 \uc774\ud558\ub298\uc774 \ubca0\uc774\ube44\ubcf5\uc2a4\ub97c \ud5a5\ud574 \uc8fd\uc740\uc0ac\ub78c\uc744 \uc0c1\uc5c5\uc801\uc73c\ub85c \uc774\uc6a9\ud55c\ub2e4\uba70 \uacf5\uac1c \ube44\ub09c\ud588\uace0 \uc774\uc5d0 \ubca0\uc774\ube44\ubcf5\uc2a4 \uce21 \ub7a9\ud37c \ud50c\ub85c\uc2a4\ud53c\ub294 \u201c\uc9c0\uc801\uc5d0 \uac10\uc0ac\ud55c\ub2e4. \ubca0\uc774\ube44\ubcf5\uc2a4 \uc74c\uc545\uc5d0 \ucd5c\uc120\uc744 \ub2e4\ud588\ub294\ub370 \ud558\ub298\uc528\uc758 \uc74c\uc545\uc131\uc5d0\ub294 \ubbf8\uce58\uc9c0 \ubabb\ud55c \uac83 \uac19\ub2e4. \ud558\uc9c0\ub9cc DJ DOC\uc758 \ub7a9\ub3c4 \uadf8\uc800 \ubbf8\uad6d\ub7a9\uc744 \ube4c\ub824 \uc4f0\ub294 \uc815\ub3c4\uc5d0 \uadf8\ucce4\ub2e4\uace0 \uc0dd\uac01\ud55c\ub2e4. \uc815\uccb4\uc131\uc774 \uc5c6\uc5b4 \ubcf4\uc778\ub2e4. \uc790\uc2e0\ub3c4 \ub0a8\uc758 \uac83\uc744 \uac00\uc838\ub2e4 \ud65c\uc6a9\ud558\uba74\uc11c \u2018\ub2e4\ub978 \uc0ac\ub78c\uc758 \uac83\uc740 \uc544\ub2c8\ub2e4\u2019\ub77c\uace0 \ube44\ub09c\ud558\ub294 \uac83\uc740 \uacf5\uc778\uc758 \ub3c4\ub9ac\uac00 \uc544\ub2cc \uac83 \uac19\ub2e4\u201d\ub77c\uba70 \uc774\ud558\ub298\uc758 \ubc1c\uc5b8\uc5d0 \ub9de\ub300\uc751\ud588\ub2e4. \uc774 \ud6c4 \uc774\ud558\ub298\uc740 \uc790\uc2e0\uc758 \ud648\ud398\uc774\uc9c0\ub97c \ud1b5\ud574 \uc7a5\ubb38\uc758 \ubca0\uc774\ube44\ubcf5\uc2a4\uc640 \ud50c\ub85c\uc2a4 \ud53c\ub97c \ube44\ubc29\ud558\ub294 \uae00\uc744 \uc62c\ub838\ub294\ub370 \uc774 \uacfc\uc815\uc5d0\uc11c '\ubbf8\uc544\ub9ac\ubcf5\uc2a4'\ub77c\ub294 \uc131\uc801 \ube44\ud558 \ub2e8\uc5b4\ub97c \uc0ac\uc6a9\ud558\uba70 \ud070 \ubb3c\uc758\ub97c \uc77c\uc73c\ucf30\uace0 \ubca0\uc774\ube44\ubcf5\uc2a4\uc5d0\uac8c \ub9c9\ub300\ud55c \uc774\ubbf8\uc9c0 \uc190\uc0c1\uc744 \uc785\ud614\ub2e4. \uc774\uc5d0 \ubca0\uc774\ube44\ubcf5\uc2a4 \uce21\uc740 \uc774\ud558\ub298\uc5d0\uac8c '6\uc6d419\uc77c\uae4c\uc9c0 \uc0ac\uacfc\ud558\ub77c'\ub294 \ucd5c\ud6c4 \ud1b5\ucca9\uc744 \ubcf4\ub0b4\uba70 \uac15\uacbd \ub300\uc751\uc5d0 \ub098\uc120\ub2e4. \uc774\ud558\ub298\uc740 \uacf5\uc2dd \uae30\uc790\ud68c\uacac\uc744 \uc5f4\uc5b4\u2018\ubbf8\uc544\ub9ac\ubcf5\uc2a4\u2019\ub780 \ub2e8\uc5b4 \uc120\ud0dd\uc740 \ub0b4\uac00 \uc0dd\uac01\ud574\ubd10\ub3c4 \uc9c1\uc124\uc801\uc778 \ud45c\ud604\uc774\uc5c8\ub2e4\uba70 \uc0ac\uacfc\ud588\uc9c0\ub9cc \ubca0\uc774\ubca0\ubcf5\uc2a4 \uc18c\uc18d\uc0ac\uc5d0 \ub300\ud574 \u201c\ub2f9\uc2e0\ub4e4\uc774 \ub2f9\ud55c \uba85\uc608\ud6fc\uc190\uacfc \ub2f9\uc2e0\ub4e4\uc774 \uc2e4\ucd94\uc2dc\ud0a8 \ud22c\ud30d\uc758 \uba85\uc608 \uc911 \uc5b4\ub5a4 \uac83\uc774 \ud070\uc9c0\ub97c \uc0dd\uac01\ud574 \ubcf4\ub77c\u201d\uba70 \u201c\ub098\ub294 \ud6c4\uc790\uac00 \ud06c\ub2e4\uace0 \uc0dd\uac01\ud55c\ub2e4\u201d\ub77c\uace0 \ub9d0\ud588\ub294\ub370 \uc774\uc5d0 DR\ubba4\uc9c1\uc758 \uc724\ub4f1\ub8e1 \ub300\ud45c\ub294 \uc774\ud558\ub298\uc5d0 \ub300\ud55c \ud615\uc0ac\uc18c\uc1a1\uc744 \uc81c\uae30\ud558\uae30\uc5d0 \uc774\ub974\ub800\ub2e4. \ub3d9\ub8cc \uc5f0\uc608\uc778 \uae40\uc9c4\ud45c\ub294 \uc790\uc2e0\uc758 \ud648\ud398\uc774\uc9c0\uc5d0 \uae00\uc744 \uc62c\ub824 \uc774\ubc88 \uc0ac\uac74\uc774 \ubca0\uc774\ube44\ubcf5\uc2a4\uac00 \ucd5c\uadfc \uc568\ubc94\uc5d0 \ubbf8\uad6d\uc758 \ub798\ud37c \ud22c\ud30d\uc758 \ub7a9\uc744 \uc0bd\uc785\ud55c \ub370\uc11c \uc2dc\uc791\ub41c \uac83\uc784\uc744 \uc9c0\uc801\ud558\uba70 \"\uc5b4\uca0c\ub4e0 \uc774\ubc88 \uc77c\ub85c \ud22c\ud329\uc774 \uc870\uae08\uc740 \uc6c3\uc744 \uac83 \uac19\uc740 \ub290\ub08c\uc774 \ub4e0\ub2e4\"\ub77c\uace0 \ub9d0\ud588\uace0 \ub9ce\uc740 \ud799\ud569 \uc544\ud2f0\uc2a4\ud2b8\uc758 \ud32c\ub4e4\uc774 \ubca0\uc774\ube44\ubcf5\uc2a4\ub97c \ud5a5\ud574 \uc758\ubbf8\uc2ec\uc7a5\ud55c \ube44\ub09c\uc758 \ub9d0\uc744 \ub358\uc84c\ub2e4. \uc774 \uc0ac\uac74\uc5d0 \ub300\ud574 \uac00\uc694 \uad00\uacc4\uc790 \uac15\ubaa8\uc528\ub294 \"\ud799\ud569 \ubba4\uc9c0\uc158\ub4e4\uc774 \uc544\uc774\ub3cc\uc2a4\ud0c0\ub97c \uacf5\uaca9\ud558\ub294 \uac83\uc740 \ubbf8\uad6d \ud799\ud569\uacc4\uc5d0\uc11c\ub294 \ud754\ud788 \uc788\ub294 '\ub514\uc2a4'\uc758 \ud55c \ud615\ud0dc\"\ub77c\uba70 \"\uc774\ud558\ub298\uc774 \uc5d0\ubbf8\ub134\uc744 \ub530\ub77c\ud558\ub294 \uac83 \uac19\uc740 \ub290\ub08c\ub3c4 \ub4e4\uc9c0\ub9cc \ud2b9\uc815 \uc5f0\uc608\uc778\uc744 \ubb34\ucc28\ubcc4 \uacf5\uaca9\ud588\ub2e4\ub294 \uc810\uc5d0\uc11c \ud30c\uc7a5\uc774 \ud074 \uac83\uc73c\ub85c \uc608\uc0c1\ub41c\ub2e4\"\ub77c\uace0 \ub9d0\ud588\ub2e4.", "answer": "6\uc6d419\uc77c", "sentence": "\uc774\uc5d0 \ubca0\uc774\ube44\ubcf5\uc2a4 \uce21\uc740 \uc774\ud558\ub298\uc5d0\uac8c ' 6\uc6d419\uc77c \uae4c\uc9c0 \uc0ac\uacfc\ud558\ub77c'\ub294 \ucd5c\ud6c4 \ud1b5\ucca9\uc744 \ubcf4\ub0b4\uba70 \uac15\uacbd \ub300\uc751\uc5d0 \ub098\uc120\ub2e4.", "paragraph_sentence": "\ub85c \ud65c\ub3d9\ud558\ub358 \uc911 \ucf00\uc774\ube14TV M.net\uc758 '\ud799\ud569\ub354\ubc14\uc774\ube0c' \ucf54\ub108\uc5d0 \uac8c\uc2a4\ud2b8\ub85c \ucd9c\uc5f0\ud55c \ub3d9\ub8cc \uac00\uc218 DJ DOC\uc758 \uba64\ubc84 \uc774\ud558\ub298\uc774 \ubca0\uc774\ube44\ubcf5\uc2a4\ub97c \ud5a5\ud574 \uc8fd\uc740\uc0ac\ub78c\uc744 \uc0c1\uc5c5\uc801\uc73c\ub85c \uc774\uc6a9\ud55c\ub2e4\uba70 \uacf5\uac1c \ube44\ub09c\ud588\uace0 \uc774\uc5d0 \ubca0\uc774\ube44\ubcf5\uc2a4 \uce21 \ub7a9\ud37c \ud50c\ub85c\uc2a4\ud53c\ub294 \u201c\uc9c0\uc801\uc5d0 \uac10\uc0ac\ud55c\ub2e4. \ubca0\uc774\ube44\ubcf5\uc2a4 \uc74c\uc545\uc5d0 \ucd5c\uc120\uc744 \ub2e4\ud588\ub294\ub370 \ud558\ub298\uc528\uc758 \uc74c\uc545\uc131\uc5d0\ub294 \ubbf8\uce58\uc9c0 \ubabb\ud55c \uac83 \uac19\ub2e4. \ud558\uc9c0\ub9cc DJ DOC\uc758 \ub7a9\ub3c4 \uadf8\uc800 \ubbf8\uad6d\ub7a9\uc744 \ube4c\ub824 \uc4f0\ub294 \uc815\ub3c4\uc5d0 \uadf8\ucce4\ub2e4\uace0 \uc0dd\uac01\ud55c\ub2e4. \uc815\uccb4\uc131\uc774 \uc5c6\uc5b4 \ubcf4\uc778\ub2e4. \uc790\uc2e0\ub3c4 \ub0a8\uc758 \uac83\uc744 \uac00\uc838\ub2e4 \ud65c\uc6a9\ud558\uba74\uc11c \u2018\ub2e4\ub978 \uc0ac\ub78c\uc758 \uac83\uc740 \uc544\ub2c8\ub2e4\u2019\ub77c\uace0 \ube44\ub09c\ud558\ub294 \uac83\uc740 \uacf5\uc778\uc758 \ub3c4\ub9ac\uac00 \uc544\ub2cc \uac83 \uac19\ub2e4\u201d\ub77c\uba70 \uc774\ud558\ub298\uc758 \ubc1c\uc5b8\uc5d0 \ub9de\ub300\uc751\ud588\ub2e4. \uc774 \ud6c4 \uc774\ud558\ub298\uc740 \uc790\uc2e0\uc758 \ud648\ud398\uc774\uc9c0\ub97c \ud1b5\ud574 \uc7a5\ubb38\uc758 \ubca0\uc774\ube44\ubcf5\uc2a4\uc640 \ud50c\ub85c\uc2a4 \ud53c\ub97c \ube44\ubc29\ud558\ub294 \uae00\uc744 \uc62c\ub838\ub294\ub370 \uc774 \uacfc\uc815\uc5d0\uc11c '\ubbf8\uc544\ub9ac\ubcf5\uc2a4'\ub77c\ub294 \uc131\uc801 \ube44\ud558 \ub2e8\uc5b4\ub97c \uc0ac\uc6a9\ud558\uba70 \ud070 \ubb3c\uc758\ub97c \uc77c\uc73c\ucf30\uace0 \ubca0\uc774\ube44\ubcf5\uc2a4\uc5d0\uac8c \ub9c9\ub300\ud55c \uc774\ubbf8\uc9c0 \uc190\uc0c1\uc744 \uc785\ud614\ub2e4. \uc774\uc5d0 \ubca0\uc774\ube44\ubcf5\uc2a4 \uce21\uc740 \uc774\ud558\ub298\uc5d0\uac8c ' 6\uc6d419\uc77c \uae4c\uc9c0 \uc0ac\uacfc\ud558\ub77c'\ub294 \ucd5c\ud6c4 \ud1b5\ucca9\uc744 \ubcf4\ub0b4\uba70 \uac15\uacbd \ub300\uc751\uc5d0 \ub098\uc120\ub2e4. \uc774\ud558\ub298\uc740 \uacf5\uc2dd \uae30\uc790\ud68c\uacac\uc744 \uc5f4\uc5b4\u2018\ubbf8\uc544\ub9ac\ubcf5\uc2a4\u2019\ub780 \ub2e8\uc5b4 \uc120\ud0dd\uc740 \ub0b4\uac00 \uc0dd\uac01\ud574\ubd10\ub3c4 \uc9c1\uc124\uc801\uc778 \ud45c\ud604\uc774\uc5c8\ub2e4\uba70 \uc0ac\uacfc\ud588\uc9c0\ub9cc \ubca0\uc774\ubca0\ubcf5\uc2a4 \uc18c\uc18d\uc0ac\uc5d0 \ub300\ud574 \u201c\ub2f9\uc2e0\ub4e4\uc774 \ub2f9\ud55c \uba85\uc608\ud6fc\uc190\uacfc \ub2f9\uc2e0\ub4e4\uc774 \uc2e4\ucd94\uc2dc\ud0a8 \ud22c\ud30d\uc758 \uba85\uc608 \uc911 \uc5b4\ub5a4 \uac83\uc774 \ud070\uc9c0\ub97c \uc0dd\uac01\ud574 \ubcf4\ub77c\u201d\uba70 \u201c\ub098\ub294 \ud6c4\uc790\uac00 \ud06c\ub2e4\uace0 \uc0dd\uac01\ud55c\ub2e4\u201d\ub77c\uace0 \ub9d0\ud588\ub294\ub370 \uc774\uc5d0 DR\ubba4\uc9c1\uc758 \uc724\ub4f1\ub8e1 \ub300\ud45c\ub294 \uc774\ud558\ub298\uc5d0 \ub300\ud55c \ud615\uc0ac\uc18c\uc1a1\uc744 \uc81c\uae30\ud558\uae30\uc5d0 \uc774\ub974\ub800\ub2e4. \ub3d9\ub8cc \uc5f0\uc608\uc778 \uae40\uc9c4\ud45c\ub294 \uc790\uc2e0\uc758 \ud648\ud398\uc774\uc9c0\uc5d0 \uae00\uc744 \uc62c\ub824 \uc774\ubc88 \uc0ac\uac74\uc774 \ubca0\uc774\ube44\ubcf5\uc2a4\uac00 \ucd5c\uadfc \uc568\ubc94\uc5d0 \ubbf8\uad6d\uc758 \ub798\ud37c \ud22c\ud30d\uc758 \ub7a9\uc744 \uc0bd\uc785\ud55c \ub370\uc11c \uc2dc\uc791\ub41c \uac83\uc784\uc744 \uc9c0\uc801\ud558\uba70 \"\uc5b4\uca0c\ub4e0 \uc774\ubc88 \uc77c\ub85c \ud22c\ud329\uc774 \uc870\uae08\uc740 \uc6c3\uc744 \uac83 \uac19\uc740 \ub290\ub08c\uc774 \ub4e0\ub2e4\"\ub77c\uace0 \ub9d0\ud588\uace0 \ub9ce\uc740 \ud799\ud569 \uc544\ud2f0\uc2a4\ud2b8\uc758 \ud32c\ub4e4\uc774 \ubca0\uc774\ube44\ubcf5\uc2a4\ub97c \ud5a5\ud574 \uc758\ubbf8\uc2ec\uc7a5\ud55c \ube44\ub09c\uc758 \ub9d0\uc744 \ub358\uc84c\ub2e4. \uc774 \uc0ac\uac74\uc5d0 \ub300\ud574 \uac00\uc694 \uad00\uacc4\uc790 \uac15\ubaa8\uc528\ub294 \"\ud799\ud569 \ubba4\uc9c0\uc158\ub4e4\uc774 \uc544\uc774\ub3cc\uc2a4\ud0c0\ub97c \uacf5\uaca9\ud558\ub294 \uac83\uc740 \ubbf8\uad6d \ud799\ud569\uacc4\uc5d0\uc11c\ub294 \ud754\ud788 \uc788\ub294 '\ub514\uc2a4'\uc758 \ud55c \ud615\ud0dc\"\ub77c\uba70 \"\uc774\ud558\ub298\uc774 \uc5d0\ubbf8\ub134\uc744 \ub530\ub77c\ud558\ub294 \uac83 \uac19\uc740 \ub290\ub08c\ub3c4 \ub4e4\uc9c0\ub9cc \ud2b9\uc815 \uc5f0\uc608\uc778\uc744 \ubb34\ucc28\ubcc4 \uacf5\uaca9\ud588\ub2e4\ub294 \uc810\uc5d0\uc11c \ud30c\uc7a5\uc774 \ud074 \uac83\uc73c\ub85c \uc608\uc0c1\ub41c\ub2e4\"\ub77c\uace0 \ub9d0\ud588\ub2e4.", "paragraph_answer": "\ub85c \ud65c\ub3d9\ud558\ub358 \uc911 \ucf00\uc774\ube14TV M.net\uc758 '\ud799\ud569\ub354\ubc14\uc774\ube0c' \ucf54\ub108\uc5d0 \uac8c\uc2a4\ud2b8\ub85c \ucd9c\uc5f0\ud55c \ub3d9\ub8cc \uac00\uc218 DJ DOC\uc758 \uba64\ubc84 \uc774\ud558\ub298\uc774 \ubca0\uc774\ube44\ubcf5\uc2a4\ub97c \ud5a5\ud574 \uc8fd\uc740\uc0ac\ub78c\uc744 \uc0c1\uc5c5\uc801\uc73c\ub85c \uc774\uc6a9\ud55c\ub2e4\uba70 \uacf5\uac1c \ube44\ub09c\ud588\uace0 \uc774\uc5d0 \ubca0\uc774\ube44\ubcf5\uc2a4 \uce21 \ub7a9\ud37c \ud50c\ub85c\uc2a4\ud53c\ub294 \u201c\uc9c0\uc801\uc5d0 \uac10\uc0ac\ud55c\ub2e4. \ubca0\uc774\ube44\ubcf5\uc2a4 \uc74c\uc545\uc5d0 \ucd5c\uc120\uc744 \ub2e4\ud588\ub294\ub370 \ud558\ub298\uc528\uc758 \uc74c\uc545\uc131\uc5d0\ub294 \ubbf8\uce58\uc9c0 \ubabb\ud55c \uac83 \uac19\ub2e4. \ud558\uc9c0\ub9cc DJ DOC\uc758 \ub7a9\ub3c4 \uadf8\uc800 \ubbf8\uad6d\ub7a9\uc744 \ube4c\ub824 \uc4f0\ub294 \uc815\ub3c4\uc5d0 \uadf8\ucce4\ub2e4\uace0 \uc0dd\uac01\ud55c\ub2e4. \uc815\uccb4\uc131\uc774 \uc5c6\uc5b4 \ubcf4\uc778\ub2e4. \uc790\uc2e0\ub3c4 \ub0a8\uc758 \uac83\uc744 \uac00\uc838\ub2e4 \ud65c\uc6a9\ud558\uba74\uc11c \u2018\ub2e4\ub978 \uc0ac\ub78c\uc758 \uac83\uc740 \uc544\ub2c8\ub2e4\u2019\ub77c\uace0 \ube44\ub09c\ud558\ub294 \uac83\uc740 \uacf5\uc778\uc758 \ub3c4\ub9ac\uac00 \uc544\ub2cc \uac83 \uac19\ub2e4\u201d\ub77c\uba70 \uc774\ud558\ub298\uc758 \ubc1c\uc5b8\uc5d0 \ub9de\ub300\uc751\ud588\ub2e4. \uc774 \ud6c4 \uc774\ud558\ub298\uc740 \uc790\uc2e0\uc758 \ud648\ud398\uc774\uc9c0\ub97c \ud1b5\ud574 \uc7a5\ubb38\uc758 \ubca0\uc774\ube44\ubcf5\uc2a4\uc640 \ud50c\ub85c\uc2a4 \ud53c\ub97c \ube44\ubc29\ud558\ub294 \uae00\uc744 \uc62c\ub838\ub294\ub370 \uc774 \uacfc\uc815\uc5d0\uc11c '\ubbf8\uc544\ub9ac\ubcf5\uc2a4'\ub77c\ub294 \uc131\uc801 \ube44\ud558 \ub2e8\uc5b4\ub97c \uc0ac\uc6a9\ud558\uba70 \ud070 \ubb3c\uc758\ub97c \uc77c\uc73c\ucf30\uace0 \ubca0\uc774\ube44\ubcf5\uc2a4\uc5d0\uac8c \ub9c9\ub300\ud55c \uc774\ubbf8\uc9c0 \uc190\uc0c1\uc744 \uc785\ud614\ub2e4. \uc774\uc5d0 \ubca0\uc774\ube44\ubcf5\uc2a4 \uce21\uc740 \uc774\ud558\ub298\uc5d0\uac8c ' 6\uc6d419\uc77c \uae4c\uc9c0 \uc0ac\uacfc\ud558\ub77c'\ub294 \ucd5c\ud6c4 \ud1b5\ucca9\uc744 \ubcf4\ub0b4\uba70 \uac15\uacbd \ub300\uc751\uc5d0 \ub098\uc120\ub2e4. \uc774\ud558\ub298\uc740 \uacf5\uc2dd \uae30\uc790\ud68c\uacac\uc744 \uc5f4\uc5b4\u2018\ubbf8\uc544\ub9ac\ubcf5\uc2a4\u2019\ub780 \ub2e8\uc5b4 \uc120\ud0dd\uc740 \ub0b4\uac00 \uc0dd\uac01\ud574\ubd10\ub3c4 \uc9c1\uc124\uc801\uc778 \ud45c\ud604\uc774\uc5c8\ub2e4\uba70 \uc0ac\uacfc\ud588\uc9c0\ub9cc \ubca0\uc774\ubca0\ubcf5\uc2a4 \uc18c\uc18d\uc0ac\uc5d0 \ub300\ud574 \u201c\ub2f9\uc2e0\ub4e4\uc774 \ub2f9\ud55c \uba85\uc608\ud6fc\uc190\uacfc \ub2f9\uc2e0\ub4e4\uc774 \uc2e4\ucd94\uc2dc\ud0a8 \ud22c\ud30d\uc758 \uba85\uc608 \uc911 \uc5b4\ub5a4 \uac83\uc774 \ud070\uc9c0\ub97c \uc0dd\uac01\ud574 \ubcf4\ub77c\u201d\uba70 \u201c\ub098\ub294 \ud6c4\uc790\uac00 \ud06c\ub2e4\uace0 \uc0dd\uac01\ud55c\ub2e4\u201d\ub77c\uace0 \ub9d0\ud588\ub294\ub370 \uc774\uc5d0 DR\ubba4\uc9c1\uc758 \uc724\ub4f1\ub8e1 \ub300\ud45c\ub294 \uc774\ud558\ub298\uc5d0 \ub300\ud55c \ud615\uc0ac\uc18c\uc1a1\uc744 \uc81c\uae30\ud558\uae30\uc5d0 \uc774\ub974\ub800\ub2e4. \ub3d9\ub8cc \uc5f0\uc608\uc778 \uae40\uc9c4\ud45c\ub294 \uc790\uc2e0\uc758 \ud648\ud398\uc774\uc9c0\uc5d0 \uae00\uc744 \uc62c\ub824 \uc774\ubc88 \uc0ac\uac74\uc774 \ubca0\uc774\ube44\ubcf5\uc2a4\uac00 \ucd5c\uadfc \uc568\ubc94\uc5d0 \ubbf8\uad6d\uc758 \ub798\ud37c \ud22c\ud30d\uc758 \ub7a9\uc744 \uc0bd\uc785\ud55c \ub370\uc11c \uc2dc\uc791\ub41c \uac83\uc784\uc744 \uc9c0\uc801\ud558\uba70 \"\uc5b4\uca0c\ub4e0 \uc774\ubc88 \uc77c\ub85c \ud22c\ud329\uc774 \uc870\uae08\uc740 \uc6c3\uc744 \uac83 \uac19\uc740 \ub290\ub08c\uc774 \ub4e0\ub2e4\"\ub77c\uace0 \ub9d0\ud588\uace0 \ub9ce\uc740 \ud799\ud569 \uc544\ud2f0\uc2a4\ud2b8\uc758 \ud32c\ub4e4\uc774 \ubca0\uc774\ube44\ubcf5\uc2a4\ub97c \ud5a5\ud574 \uc758\ubbf8\uc2ec\uc7a5\ud55c \ube44\ub09c\uc758 \ub9d0\uc744 \ub358\uc84c\ub2e4. \uc774 \uc0ac\uac74\uc5d0 \ub300\ud574 \uac00\uc694 \uad00\uacc4\uc790 \uac15\ubaa8\uc528\ub294 \"\ud799\ud569 \ubba4\uc9c0\uc158\ub4e4\uc774 \uc544\uc774\ub3cc\uc2a4\ud0c0\ub97c \uacf5\uaca9\ud558\ub294 \uac83\uc740 \ubbf8\uad6d \ud799\ud569\uacc4\uc5d0\uc11c\ub294 \ud754\ud788 \uc788\ub294 '\ub514\uc2a4'\uc758 \ud55c \ud615\ud0dc\"\ub77c\uba70 \"\uc774\ud558\ub298\uc774 \uc5d0\ubbf8\ub134\uc744 \ub530\ub77c\ud558\ub294 \uac83 \uac19\uc740 \ub290\ub08c\ub3c4 \ub4e4\uc9c0\ub9cc \ud2b9\uc815 \uc5f0\uc608\uc778\uc744 \ubb34\ucc28\ubcc4 \uacf5\uaca9\ud588\ub2e4\ub294 \uc810\uc5d0\uc11c \ud30c\uc7a5\uc774 \ud074 \uac83\uc73c\ub85c \uc608\uc0c1\ub41c\ub2e4\"\ub77c\uace0 \ub9d0\ud588\ub2e4.", "sentence_answer": "\uc774\uc5d0 \ubca0\uc774\ube44\ubcf5\uc2a4 \uce21\uc740 \uc774\ud558\ub298\uc5d0\uac8c ' 6\uc6d419\uc77c \uae4c\uc9c0 \uc0ac\uacfc\ud558\ub77c'\ub294 \ucd5c\ud6c4 \ud1b5\ucca9\uc744 \ubcf4\ub0b4\uba70 \uac15\uacbd \ub300\uc751\uc5d0 \ub098\uc120\ub2e4.", "paragraph_id": "6508291-27-1", "paragraph_question": "question: \ubca0\uc774\ube44\ubcf5\uc2a4 \uce21\uc740 \ubbf8\uc544\ub9ac\ubcf5\uc2a4\ub77c\ub294 \ube44\ud558 \ub2e8\uc5b4\ub97c \uc0ac\uc6a9\ud55c \uc774\ud558\ub298\uc5d0\uac8c \uc5b8\uc81c\uae4c\uc9c0 \uc0ac\uacfc\ud558\ub77c\uace0 \ucd5c\ud6c4 \ud1b5\ucca9\uc744 \ubcf4\ub0c8\uc2b5\ub2c8\uae4c?, context: \ub85c \ud65c\ub3d9\ud558\ub358 \uc911 \ucf00\uc774\ube14TV M.net\uc758 '\ud799\ud569\ub354\ubc14\uc774\ube0c' \ucf54\ub108\uc5d0 \uac8c\uc2a4\ud2b8\ub85c \ucd9c\uc5f0\ud55c \ub3d9\ub8cc \uac00\uc218 DJ DOC\uc758 \uba64\ubc84 \uc774\ud558\ub298\uc774 \ubca0\uc774\ube44\ubcf5\uc2a4\ub97c \ud5a5\ud574 \uc8fd\uc740\uc0ac\ub78c\uc744 \uc0c1\uc5c5\uc801\uc73c\ub85c \uc774\uc6a9\ud55c\ub2e4\uba70 \uacf5\uac1c \ube44\ub09c\ud588\uace0 \uc774\uc5d0 \ubca0\uc774\ube44\ubcf5\uc2a4 \uce21 \ub7a9\ud37c \ud50c\ub85c\uc2a4\ud53c\ub294 \u201c\uc9c0\uc801\uc5d0 \uac10\uc0ac\ud55c\ub2e4. \ubca0\uc774\ube44\ubcf5\uc2a4 \uc74c\uc545\uc5d0 \ucd5c\uc120\uc744 \ub2e4\ud588\ub294\ub370 \ud558\ub298\uc528\uc758 \uc74c\uc545\uc131\uc5d0\ub294 \ubbf8\uce58\uc9c0 \ubabb\ud55c \uac83 \uac19\ub2e4. \ud558\uc9c0\ub9cc DJ DOC\uc758 \ub7a9\ub3c4 \uadf8\uc800 \ubbf8\uad6d\ub7a9\uc744 \ube4c\ub824 \uc4f0\ub294 \uc815\ub3c4\uc5d0 \uadf8\ucce4\ub2e4\uace0 \uc0dd\uac01\ud55c\ub2e4. \uc815\uccb4\uc131\uc774 \uc5c6\uc5b4 \ubcf4\uc778\ub2e4. \uc790\uc2e0\ub3c4 \ub0a8\uc758 \uac83\uc744 \uac00\uc838\ub2e4 \ud65c\uc6a9\ud558\uba74\uc11c \u2018\ub2e4\ub978 \uc0ac\ub78c\uc758 \uac83\uc740 \uc544\ub2c8\ub2e4\u2019\ub77c\uace0 \ube44\ub09c\ud558\ub294 \uac83\uc740 \uacf5\uc778\uc758 \ub3c4\ub9ac\uac00 \uc544\ub2cc \uac83 \uac19\ub2e4\u201d\ub77c\uba70 \uc774\ud558\ub298\uc758 \ubc1c\uc5b8\uc5d0 \ub9de\ub300\uc751\ud588\ub2e4. \uc774 \ud6c4 \uc774\ud558\ub298\uc740 \uc790\uc2e0\uc758 \ud648\ud398\uc774\uc9c0\ub97c \ud1b5\ud574 \uc7a5\ubb38\uc758 \ubca0\uc774\ube44\ubcf5\uc2a4\uc640 \ud50c\ub85c\uc2a4 \ud53c\ub97c \ube44\ubc29\ud558\ub294 \uae00\uc744 \uc62c\ub838\ub294\ub370 \uc774 \uacfc\uc815\uc5d0\uc11c '\ubbf8\uc544\ub9ac\ubcf5\uc2a4'\ub77c\ub294 \uc131\uc801 \ube44\ud558 \ub2e8\uc5b4\ub97c \uc0ac\uc6a9\ud558\uba70 \ud070 \ubb3c\uc758\ub97c \uc77c\uc73c\ucf30\uace0 \ubca0\uc774\ube44\ubcf5\uc2a4\uc5d0\uac8c \ub9c9\ub300\ud55c \uc774\ubbf8\uc9c0 \uc190\uc0c1\uc744 \uc785\ud614\ub2e4. \uc774\uc5d0 \ubca0\uc774\ube44\ubcf5\uc2a4 \uce21\uc740 \uc774\ud558\ub298\uc5d0\uac8c '6\uc6d419\uc77c\uae4c\uc9c0 \uc0ac\uacfc\ud558\ub77c'\ub294 \ucd5c\ud6c4 \ud1b5\ucca9\uc744 \ubcf4\ub0b4\uba70 \uac15\uacbd \ub300\uc751\uc5d0 \ub098\uc120\ub2e4. \uc774\ud558\ub298\uc740 \uacf5\uc2dd \uae30\uc790\ud68c\uacac\uc744 \uc5f4\uc5b4\u2018\ubbf8\uc544\ub9ac\ubcf5\uc2a4\u2019\ub780 \ub2e8\uc5b4 \uc120\ud0dd\uc740 \ub0b4\uac00 \uc0dd\uac01\ud574\ubd10\ub3c4 \uc9c1\uc124\uc801\uc778 \ud45c\ud604\uc774\uc5c8\ub2e4\uba70 \uc0ac\uacfc\ud588\uc9c0\ub9cc \ubca0\uc774\ubca0\ubcf5\uc2a4 \uc18c\uc18d\uc0ac\uc5d0 \ub300\ud574 \u201c\ub2f9\uc2e0\ub4e4\uc774 \ub2f9\ud55c \uba85\uc608\ud6fc\uc190\uacfc \ub2f9\uc2e0\ub4e4\uc774 \uc2e4\ucd94\uc2dc\ud0a8 \ud22c\ud30d\uc758 \uba85\uc608 \uc911 \uc5b4\ub5a4 \uac83\uc774 \ud070\uc9c0\ub97c \uc0dd\uac01\ud574 \ubcf4\ub77c\u201d\uba70 \u201c\ub098\ub294 \ud6c4\uc790\uac00 \ud06c\ub2e4\uace0 \uc0dd\uac01\ud55c\ub2e4\u201d\ub77c\uace0 \ub9d0\ud588\ub294\ub370 \uc774\uc5d0 DR\ubba4\uc9c1\uc758 \uc724\ub4f1\ub8e1 \ub300\ud45c\ub294 \uc774\ud558\ub298\uc5d0 \ub300\ud55c \ud615\uc0ac\uc18c\uc1a1\uc744 \uc81c\uae30\ud558\uae30\uc5d0 \uc774\ub974\ub800\ub2e4. \ub3d9\ub8cc \uc5f0\uc608\uc778 \uae40\uc9c4\ud45c\ub294 \uc790\uc2e0\uc758 \ud648\ud398\uc774\uc9c0\uc5d0 \uae00\uc744 \uc62c\ub824 \uc774\ubc88 \uc0ac\uac74\uc774 \ubca0\uc774\ube44\ubcf5\uc2a4\uac00 \ucd5c\uadfc \uc568\ubc94\uc5d0 \ubbf8\uad6d\uc758 \ub798\ud37c \ud22c\ud30d\uc758 \ub7a9\uc744 \uc0bd\uc785\ud55c \ub370\uc11c \uc2dc\uc791\ub41c \uac83\uc784\uc744 \uc9c0\uc801\ud558\uba70 \"\uc5b4\uca0c\ub4e0 \uc774\ubc88 \uc77c\ub85c \ud22c\ud329\uc774 \uc870\uae08\uc740 \uc6c3\uc744 \uac83 \uac19\uc740 \ub290\ub08c\uc774 \ub4e0\ub2e4\"\ub77c\uace0 \ub9d0\ud588\uace0 \ub9ce\uc740 \ud799\ud569 \uc544\ud2f0\uc2a4\ud2b8\uc758 \ud32c\ub4e4\uc774 \ubca0\uc774\ube44\ubcf5\uc2a4\ub97c \ud5a5\ud574 \uc758\ubbf8\uc2ec\uc7a5\ud55c \ube44\ub09c\uc758 \ub9d0\uc744 \ub358\uc84c\ub2e4. \uc774 \uc0ac\uac74\uc5d0 \ub300\ud574 \uac00\uc694 \uad00\uacc4\uc790 \uac15\ubaa8\uc528\ub294 \"\ud799\ud569 \ubba4\uc9c0\uc158\ub4e4\uc774 \uc544\uc774\ub3cc\uc2a4\ud0c0\ub97c \uacf5\uaca9\ud558\ub294 \uac83\uc740 \ubbf8\uad6d \ud799\ud569\uacc4\uc5d0\uc11c\ub294 \ud754\ud788 \uc788\ub294 '\ub514\uc2a4'\uc758 \ud55c \ud615\ud0dc\"\ub77c\uba70 \"\uc774\ud558\ub298\uc774 \uc5d0\ubbf8\ub134\uc744 \ub530\ub77c\ud558\ub294 \uac83 \uac19\uc740 \ub290\ub08c\ub3c4 \ub4e4\uc9c0\ub9cc \ud2b9\uc815 \uc5f0\uc608\uc778\uc744 \ubb34\ucc28\ubcc4 \uacf5\uaca9\ud588\ub2e4\ub294 \uc810\uc5d0\uc11c \ud30c\uc7a5\uc774 \ud074 \uac83\uc73c\ub85c \uc608\uc0c1\ub41c\ub2e4\"\ub77c\uace0 \ub9d0\ud588\ub2e4."} {"question": "\uc194\ub9ac\ub4dc \uc2a4\ud14c\uc774\ud2b8 \ub514\uc2a4\ud06c\ub97c \ubcf4\ud1b5 \ubb34\uc5c7\uc774\ub77c\uace0 \uc904\uc5ec \ud45c\ud604\ud558\ub294\uac00?", "paragraph": "SATA\uc758 \ub85c\ub4dc\ub9f5\uc740 SATA 6.0 Gbit/s \uaddc\uaca9\uc744 \ud3ec\ud568\ud558\uace0 \uc788\ub2e4. \ud604\uc7ac\uc758 PC\uc5d0\uc11c SATA 3.0 Gbit/s\ub294 \ubc8c\uc368 \ud558\ub4dc \ub514\uc2a4\ud06c\uc758 \ucd5c\uace0 \uc804\uc1a1 \uc18d\ub3c4(\ubc84\uc2a4\ud2b8 \uc804\uc1a1\uc744 \uc81c\uc678\ud55c)\ub97c \ud6e8\uc52c \ub6f0\uc5b4 \ub118\uc5c8\uae30 \ub54c\ubb38\uc5d0 6.0 Gbit/s \uaddc\uaca9\uc740 \ud558\ub098\uc758 SATA \ud3ec\ud2b8\uc5d0 \uc5ec\ub7ec \uac1c\uc758 \ub4dc\ub77c\uc774\ube0c\ub97c \uc5f0\uacb0\ud558\ub294 \ud3ec\ud2b8 \uba40\ud2f0\ud50c\ub77c\uc774\uc5b4\uc5d0 \uc4f8\ubaa8 \uc788\uc744 \uac83\uc73c\ub85c \ubcf4\uc778\ub2e4. \ub7a8 \ub4dc\ub77c\uc774\ube0c\uc640 \uac19\uc740 \uc194\ub9ac\ub4dc \uc2a4\ud14c\uc774\ud2b8 \ub514\uc2a4\ud06c(SSD) \ub610\ud55c \uba38\uc9c0 \uc54a\uc544 \uc774\ub7ec\ud55c \uc804\uc1a1 \uc18d\ub3c4\ub97c \uac1c\uc120\ud558\ub294 \uc774\ub4dd\uc744 \ubcfc \uac83\uc73c\ub85c \uc608\uc0c1\ub41c\ub2e4. \ud558\ub4dc \ub514\uc2a4\ud06c \uc81c\uc870 \uc5c5\uccb4\uc5d0\uc11c \uc774\ub860\uc801\uc778 \ubc84\uc2a4\ud2b8 \uc804\uc1a1 \uc18d\ub3c4\ub97c \uc798 \ubc1d\ud788\uc9c0 \uc54a\uae30 \ub54c\ubb38\uc5d0 \uc2e4\uc0dd\ud65c\uc5d0\uc11c SATA\ub97c \uc0ac\uc6a9\ud558\uba74\uc11c \uc5bb\ub294 \uc774\ub4dd\uc740 \ub354 \uc801\uc740 \uc804\ub825 \uc18c\ubaa8\uc640 \ucf00\uc774\ube14, \uadf8\ub9ac\uace0 \ud56b \ud50c\ub7ec\uadf8 \uc815\ub3c4\ub2e4. \uc9c0\uae08 \uba87\uba87 \ubcf4\ub4dc\uc5d0\uc11c \uc9c0\uc6d0\ud558\uace0 \uc788\ub2e4.\uc778\ud154\uc758 P67,H67,Z68\uc5d0\uc11c \uc9c0\uc6d0\ud558\uace0 AMD\ub294 SB850,A75 \uc0ac\uc6b0\uc2a4\ube0c\ub9ac\uc9c0\uac00 \ud0d1\uc7ac\ub41c \uba54\uc778\ubcf4\ub4dc\uc5d0\uc11c \ub124\uc774\ud2f0\ube0c\ub85c \uc9c0\uc6d0\ub41c\ub2e4 \uadf8 \uc678 \uba54\uc778\ubcf4\ub4dc\ub294 \ubcc4\ub3c4\uc758 \ucee8\ud2b8\ub864\ub7ec\uc5d0 \uc758\ud55c \uc9c0\uc6d0\uc774\ub2e4.", "answer": "SSD", "sentence": "\ub7a8 \ub4dc\ub77c\uc774\ube0c\uc640 \uac19\uc740 \uc194\ub9ac\ub4dc \uc2a4\ud14c\uc774\ud2b8 \ub514\uc2a4\ud06c( SSD )", "paragraph_sentence": "SATA\uc758 \ub85c\ub4dc\ub9f5\uc740 SATA 6.0 Gbit/s \uaddc\uaca9\uc744 \ud3ec\ud568\ud558\uace0 \uc788\ub2e4. \ud604\uc7ac\uc758 PC\uc5d0\uc11c SATA 3.0 Gbit/s\ub294 \ubc8c\uc368 \ud558\ub4dc \ub514\uc2a4\ud06c\uc758 \ucd5c\uace0 \uc804\uc1a1 \uc18d\ub3c4(\ubc84\uc2a4\ud2b8 \uc804\uc1a1\uc744 \uc81c\uc678\ud55c)\ub97c \ud6e8\uc52c \ub6f0\uc5b4 \ub118\uc5c8\uae30 \ub54c\ubb38\uc5d0 6.0 Gbit/s \uaddc\uaca9\uc740 \ud558\ub098\uc758 SATA \ud3ec\ud2b8\uc5d0 \uc5ec\ub7ec \uac1c\uc758 \ub4dc\ub77c\uc774\ube0c\ub97c \uc5f0\uacb0\ud558\ub294 \ud3ec\ud2b8 \uba40\ud2f0\ud50c\ub77c\uc774\uc5b4\uc5d0 \uc4f8\ubaa8 \uc788\uc744 \uac83\uc73c\ub85c \ubcf4\uc778\ub2e4. \ub7a8 \ub4dc\ub77c\uc774\ube0c\uc640 \uac19\uc740 \uc194\ub9ac\ub4dc \uc2a4\ud14c\uc774\ud2b8 \ub514\uc2a4\ud06c( SSD ) \ub610\ud55c \uba38\uc9c0 \uc54a\uc544 \uc774\ub7ec\ud55c \uc804\uc1a1 \uc18d\ub3c4\ub97c \uac1c\uc120\ud558\ub294 \uc774\ub4dd\uc744 \ubcfc \uac83\uc73c\ub85c \uc608\uc0c1\ub41c\ub2e4. \ud558\ub4dc \ub514\uc2a4\ud06c \uc81c\uc870 \uc5c5\uccb4\uc5d0\uc11c \uc774\ub860\uc801\uc778 \ubc84\uc2a4\ud2b8 \uc804\uc1a1 \uc18d\ub3c4\ub97c \uc798 \ubc1d\ud788\uc9c0 \uc54a\uae30 \ub54c\ubb38\uc5d0 \uc2e4\uc0dd\ud65c\uc5d0\uc11c SATA\ub97c \uc0ac\uc6a9\ud558\uba74\uc11c \uc5bb\ub294 \uc774\ub4dd\uc740 \ub354 \uc801\uc740 \uc804\ub825 \uc18c\ubaa8\uc640 \ucf00\uc774\ube14, \uadf8\ub9ac\uace0 \ud56b \ud50c\ub7ec\uadf8 \uc815\ub3c4\ub2e4. \uc9c0\uae08 \uba87\uba87 \ubcf4\ub4dc\uc5d0\uc11c \uc9c0\uc6d0\ud558\uace0 \uc788\ub2e4.\uc778\ud154\uc758 P67,H67,Z68\uc5d0\uc11c \uc9c0\uc6d0\ud558\uace0 AMD\ub294 SB850,A75 \uc0ac\uc6b0\uc2a4\ube0c\ub9ac\uc9c0\uac00 \ud0d1\uc7ac\ub41c \uba54\uc778\ubcf4\ub4dc\uc5d0\uc11c \ub124\uc774\ud2f0\ube0c\ub85c \uc9c0\uc6d0\ub41c\ub2e4 \uadf8 \uc678 \uba54\uc778\ubcf4\ub4dc\ub294 \ubcc4\ub3c4\uc758 \ucee8\ud2b8\ub864\ub7ec\uc5d0 \uc758\ud55c \uc9c0\uc6d0\uc774\ub2e4.", "paragraph_answer": "SATA\uc758 \ub85c\ub4dc\ub9f5\uc740 SATA 6.0 Gbit/s \uaddc\uaca9\uc744 \ud3ec\ud568\ud558\uace0 \uc788\ub2e4. \ud604\uc7ac\uc758 PC\uc5d0\uc11c SATA 3.0 Gbit/s\ub294 \ubc8c\uc368 \ud558\ub4dc \ub514\uc2a4\ud06c\uc758 \ucd5c\uace0 \uc804\uc1a1 \uc18d\ub3c4(\ubc84\uc2a4\ud2b8 \uc804\uc1a1\uc744 \uc81c\uc678\ud55c)\ub97c \ud6e8\uc52c \ub6f0\uc5b4 \ub118\uc5c8\uae30 \ub54c\ubb38\uc5d0 6.0 Gbit/s \uaddc\uaca9\uc740 \ud558\ub098\uc758 SATA \ud3ec\ud2b8\uc5d0 \uc5ec\ub7ec \uac1c\uc758 \ub4dc\ub77c\uc774\ube0c\ub97c \uc5f0\uacb0\ud558\ub294 \ud3ec\ud2b8 \uba40\ud2f0\ud50c\ub77c\uc774\uc5b4\uc5d0 \uc4f8\ubaa8 \uc788\uc744 \uac83\uc73c\ub85c \ubcf4\uc778\ub2e4. \ub7a8 \ub4dc\ub77c\uc774\ube0c\uc640 \uac19\uc740 \uc194\ub9ac\ub4dc \uc2a4\ud14c\uc774\ud2b8 \ub514\uc2a4\ud06c( SSD ) \ub610\ud55c \uba38\uc9c0 \uc54a\uc544 \uc774\ub7ec\ud55c \uc804\uc1a1 \uc18d\ub3c4\ub97c \uac1c\uc120\ud558\ub294 \uc774\ub4dd\uc744 \ubcfc \uac83\uc73c\ub85c \uc608\uc0c1\ub41c\ub2e4. \ud558\ub4dc \ub514\uc2a4\ud06c \uc81c\uc870 \uc5c5\uccb4\uc5d0\uc11c \uc774\ub860\uc801\uc778 \ubc84\uc2a4\ud2b8 \uc804\uc1a1 \uc18d\ub3c4\ub97c \uc798 \ubc1d\ud788\uc9c0 \uc54a\uae30 \ub54c\ubb38\uc5d0 \uc2e4\uc0dd\ud65c\uc5d0\uc11c SATA\ub97c \uc0ac\uc6a9\ud558\uba74\uc11c \uc5bb\ub294 \uc774\ub4dd\uc740 \ub354 \uc801\uc740 \uc804\ub825 \uc18c\ubaa8\uc640 \ucf00\uc774\ube14, \uadf8\ub9ac\uace0 \ud56b \ud50c\ub7ec\uadf8 \uc815\ub3c4\ub2e4. \uc9c0\uae08 \uba87\uba87 \ubcf4\ub4dc\uc5d0\uc11c \uc9c0\uc6d0\ud558\uace0 \uc788\ub2e4.\uc778\ud154\uc758 P67,H67,Z68\uc5d0\uc11c \uc9c0\uc6d0\ud558\uace0 AMD\ub294 SB850,A75 \uc0ac\uc6b0\uc2a4\ube0c\ub9ac\uc9c0\uac00 \ud0d1\uc7ac\ub41c \uba54\uc778\ubcf4\ub4dc\uc5d0\uc11c \ub124\uc774\ud2f0\ube0c\ub85c \uc9c0\uc6d0\ub41c\ub2e4 \uadf8 \uc678 \uba54\uc778\ubcf4\ub4dc\ub294 \ubcc4\ub3c4\uc758 \ucee8\ud2b8\ub864\ub7ec\uc5d0 \uc758\ud55c \uc9c0\uc6d0\uc774\ub2e4.", "sentence_answer": "\ub7a8 \ub4dc\ub77c\uc774\ube0c\uc640 \uac19\uc740 \uc194\ub9ac\ub4dc \uc2a4\ud14c\uc774\ud2b8 \ub514\uc2a4\ud06c( SSD )", "paragraph_id": "5895415-4-1", "paragraph_question": "question: \uc194\ub9ac\ub4dc \uc2a4\ud14c\uc774\ud2b8 \ub514\uc2a4\ud06c\ub97c \ubcf4\ud1b5 \ubb34\uc5c7\uc774\ub77c\uace0 \uc904\uc5ec \ud45c\ud604\ud558\ub294\uac00?, context: SATA\uc758 \ub85c\ub4dc\ub9f5\uc740 SATA 6.0 Gbit/s \uaddc\uaca9\uc744 \ud3ec\ud568\ud558\uace0 \uc788\ub2e4. \ud604\uc7ac\uc758 PC\uc5d0\uc11c SATA 3.0 Gbit/s\ub294 \ubc8c\uc368 \ud558\ub4dc \ub514\uc2a4\ud06c\uc758 \ucd5c\uace0 \uc804\uc1a1 \uc18d\ub3c4(\ubc84\uc2a4\ud2b8 \uc804\uc1a1\uc744 \uc81c\uc678\ud55c)\ub97c \ud6e8\uc52c \ub6f0\uc5b4 \ub118\uc5c8\uae30 \ub54c\ubb38\uc5d0 6.0 Gbit/s \uaddc\uaca9\uc740 \ud558\ub098\uc758 SATA \ud3ec\ud2b8\uc5d0 \uc5ec\ub7ec \uac1c\uc758 \ub4dc\ub77c\uc774\ube0c\ub97c \uc5f0\uacb0\ud558\ub294 \ud3ec\ud2b8 \uba40\ud2f0\ud50c\ub77c\uc774\uc5b4\uc5d0 \uc4f8\ubaa8 \uc788\uc744 \uac83\uc73c\ub85c \ubcf4\uc778\ub2e4. \ub7a8 \ub4dc\ub77c\uc774\ube0c\uc640 \uac19\uc740 \uc194\ub9ac\ub4dc \uc2a4\ud14c\uc774\ud2b8 \ub514\uc2a4\ud06c(SSD) \ub610\ud55c \uba38\uc9c0 \uc54a\uc544 \uc774\ub7ec\ud55c \uc804\uc1a1 \uc18d\ub3c4\ub97c \uac1c\uc120\ud558\ub294 \uc774\ub4dd\uc744 \ubcfc \uac83\uc73c\ub85c \uc608\uc0c1\ub41c\ub2e4. \ud558\ub4dc \ub514\uc2a4\ud06c \uc81c\uc870 \uc5c5\uccb4\uc5d0\uc11c \uc774\ub860\uc801\uc778 \ubc84\uc2a4\ud2b8 \uc804\uc1a1 \uc18d\ub3c4\ub97c \uc798 \ubc1d\ud788\uc9c0 \uc54a\uae30 \ub54c\ubb38\uc5d0 \uc2e4\uc0dd\ud65c\uc5d0\uc11c SATA\ub97c \uc0ac\uc6a9\ud558\uba74\uc11c \uc5bb\ub294 \uc774\ub4dd\uc740 \ub354 \uc801\uc740 \uc804\ub825 \uc18c\ubaa8\uc640 \ucf00\uc774\ube14, \uadf8\ub9ac\uace0 \ud56b \ud50c\ub7ec\uadf8 \uc815\ub3c4\ub2e4. \uc9c0\uae08 \uba87\uba87 \ubcf4\ub4dc\uc5d0\uc11c \uc9c0\uc6d0\ud558\uace0 \uc788\ub2e4.\uc778\ud154\uc758 P67,H67,Z68\uc5d0\uc11c \uc9c0\uc6d0\ud558\uace0 AMD\ub294 SB850,A75 \uc0ac\uc6b0\uc2a4\ube0c\ub9ac\uc9c0\uac00 \ud0d1\uc7ac\ub41c \uba54\uc778\ubcf4\ub4dc\uc5d0\uc11c \ub124\uc774\ud2f0\ube0c\ub85c \uc9c0\uc6d0\ub41c\ub2e4 \uadf8 \uc678 \uba54\uc778\ubcf4\ub4dc\ub294 \ubcc4\ub3c4\uc758 \ucee8\ud2b8\ub864\ub7ec\uc5d0 \uc758\ud55c \uc9c0\uc6d0\uc774\ub2e4."} {"question": "\uad6d\uac00 \ube44\uc815\ub2f9 \ud68c\uc758\uc758 \uc57d\uc790\ub294 \ubb34\uc5c7\uc778\uac00?", "paragraph": "\ub178\ub9c8 \uc5d8\ub9ac\uc790\ubca0\uc2a4 \ubcf4\uc774\ub4dc\uac00 \uc8fc\ub3c4\ud55c \uc5ec\uc131\ud68c\ub294 1938\ub144 \uc2dc\ubbfc \ubbfc\uc8fc \uad8c\ub9ac\ub97c \uc704\ud55c \uad6d\uac00 \ube44\uc815\ub2f9 \ub85c\ube44(National Non-Partisan Lobby on Civil and Democratic Rights, \uc57d\uc790:NPC)\ub97c \ucc3d\uc124\ud588\uace0, \uc774\ud6c4 NPC\uc758 \uc774\ub984\uc744 \uacf5\ubb34\ub97c \uc704\ud55c \uad6d\uac00 \ube44\uc815\ub2f9 \ud68c\uc758(Non-Partisan Council on Public Affairs, \uc57d\uc790\ub294 \ubcc0\ud568\uc5c6\uc774 NPC)\ub85c \ubc14\uafb8\uc5c8\ub2e4. \uc774\uac83\uc740 \uc18c\uc218\ubbfc\uc871\uc758 \uad8c\ub9ac\ub97c \uc704\ud55c \ucd5c\ucd08\uc758 \uc804\uc784(\u5c08\u4efb) \uad6d\ud68c \ub85c\ube44 \ub2e8\uccb4\uc600\ub2e4. NPC\ub294 \uc874\uc7ac \uae30\uac04 \ub0b4\ub0b4 NAACP, \uc804\uad6d \ub3c4\uc2dc \ub3d9\ub9f9, UOPWA, NAGN, AFC, \uc720\uc0c9 \uc5ec\uc131 \ud074\ub7fd, BSCPA, \ub274\uc695 \ud22c\ud45c\uc790 \ub3d9\ub9f9\uacfc \ud611\ub825\ud588\ub2e4. NPC\ub294 12\ubc88\uc9f8 \ucd5c\uace0 \ubc14\uc2e4\ub808\uc624\uc2a4(Basileus, \uc9c0\ub3c4\uc790) \uc5d0\ub4dc\ub098 \uc624\ubc84 \uadf8\ub808\uc774\ucea0\ubca8\uc5d0 \uc758\ud574 1948\ub144 7\uc6d4 15\uc77c \ud574\uccb4\ub418\uc5c8\ub2e4. 1\ub144 \ud6c4, AKA\ub294 NAACP\uc758 \uccab \ubc88\uc9f8 \uc885\uc2e0\ud68c\uc6d0\uc774 \ub418\uc5c8\ub2e4.", "answer": "NPC", "sentence": "\ub178\ub9c8 \uc5d8\ub9ac\uc790\ubca0\uc2a4 \ubcf4\uc774\ub4dc\uac00 \uc8fc\ub3c4\ud55c \uc5ec\uc131\ud68c\ub294 1938\ub144 \uc2dc\ubbfc \ubbfc\uc8fc \uad8c\ub9ac\ub97c \uc704\ud55c \uad6d\uac00 \ube44\uc815\ub2f9 \ub85c\ube44(National Non-Partisan Lobby on Civil and Democratic Rights, \uc57d\uc790: NPC )", "paragraph_sentence": " \ub178\ub9c8 \uc5d8\ub9ac\uc790\ubca0\uc2a4 \ubcf4\uc774\ub4dc\uac00 \uc8fc\ub3c4\ud55c \uc5ec\uc131\ud68c\ub294 1938\ub144 \uc2dc\ubbfc \ubbfc\uc8fc \uad8c\ub9ac\ub97c \uc704\ud55c \uad6d\uac00 \ube44\uc815\ub2f9 \ub85c\ube44(National Non-Partisan Lobby on Civil and Democratic Rights, \uc57d\uc790: NPC ) \ub97c \ucc3d\uc124\ud588\uace0, \uc774\ud6c4 NPC\uc758 \uc774\ub984\uc744 \uacf5\ubb34\ub97c \uc704\ud55c \uad6d\uac00 \ube44\uc815\ub2f9 \ud68c\uc758(Non-Partisan Council on Public Affairs, \uc57d\uc790\ub294 \ubcc0\ud568\uc5c6\uc774 NPC)\ub85c \ubc14\uafb8\uc5c8\ub2e4. \uc774\uac83\uc740 \uc18c\uc218\ubbfc\uc871\uc758 \uad8c\ub9ac\ub97c \uc704\ud55c \ucd5c\ucd08\uc758 \uc804\uc784(\u5c08\u4efb) \uad6d\ud68c \ub85c\ube44 \ub2e8\uccb4\uc600\ub2e4. NPC\ub294 \uc874\uc7ac \uae30\uac04 \ub0b4\ub0b4 NAACP, \uc804\uad6d \ub3c4\uc2dc \ub3d9\ub9f9, UOPWA, NAGN, AFC, \uc720\uc0c9 \uc5ec\uc131 \ud074\ub7fd, BSCPA, \ub274\uc695 \ud22c\ud45c\uc790 \ub3d9\ub9f9\uacfc \ud611\ub825\ud588\ub2e4. NPC\ub294 12\ubc88\uc9f8 \ucd5c\uace0 \ubc14\uc2e4\ub808\uc624\uc2a4(Basileus, \uc9c0\ub3c4\uc790) \uc5d0\ub4dc\ub098 \uc624\ubc84 \uadf8\ub808\uc774\ucea0\ubca8\uc5d0 \uc758\ud574 1948\ub144 7\uc6d4 15\uc77c \ud574\uccb4\ub418\uc5c8\ub2e4. 1\ub144 \ud6c4, AKA\ub294 NAACP\uc758 \uccab \ubc88\uc9f8 \uc885\uc2e0\ud68c\uc6d0\uc774 \ub418\uc5c8\ub2e4.", "paragraph_answer": "\ub178\ub9c8 \uc5d8\ub9ac\uc790\ubca0\uc2a4 \ubcf4\uc774\ub4dc\uac00 \uc8fc\ub3c4\ud55c \uc5ec\uc131\ud68c\ub294 1938\ub144 \uc2dc\ubbfc \ubbfc\uc8fc \uad8c\ub9ac\ub97c \uc704\ud55c \uad6d\uac00 \ube44\uc815\ub2f9 \ub85c\ube44(National Non-Partisan Lobby on Civil and Democratic Rights, \uc57d\uc790: NPC )\ub97c \ucc3d\uc124\ud588\uace0, \uc774\ud6c4 NPC\uc758 \uc774\ub984\uc744 \uacf5\ubb34\ub97c \uc704\ud55c \uad6d\uac00 \ube44\uc815\ub2f9 \ud68c\uc758(Non-Partisan Council on Public Affairs, \uc57d\uc790\ub294 \ubcc0\ud568\uc5c6\uc774 NPC)\ub85c \ubc14\uafb8\uc5c8\ub2e4. \uc774\uac83\uc740 \uc18c\uc218\ubbfc\uc871\uc758 \uad8c\ub9ac\ub97c \uc704\ud55c \ucd5c\ucd08\uc758 \uc804\uc784(\u5c08\u4efb) \uad6d\ud68c \ub85c\ube44 \ub2e8\uccb4\uc600\ub2e4. NPC\ub294 \uc874\uc7ac \uae30\uac04 \ub0b4\ub0b4 NAACP, \uc804\uad6d \ub3c4\uc2dc \ub3d9\ub9f9, UOPWA, NAGN, AFC, \uc720\uc0c9 \uc5ec\uc131 \ud074\ub7fd, BSCPA, \ub274\uc695 \ud22c\ud45c\uc790 \ub3d9\ub9f9\uacfc \ud611\ub825\ud588\ub2e4. NPC\ub294 12\ubc88\uc9f8 \ucd5c\uace0 \ubc14\uc2e4\ub808\uc624\uc2a4(Basileus, \uc9c0\ub3c4\uc790) \uc5d0\ub4dc\ub098 \uc624\ubc84 \uadf8\ub808\uc774\ucea0\ubca8\uc5d0 \uc758\ud574 1948\ub144 7\uc6d4 15\uc77c \ud574\uccb4\ub418\uc5c8\ub2e4. 1\ub144 \ud6c4, AKA\ub294 NAACP\uc758 \uccab \ubc88\uc9f8 \uc885\uc2e0\ud68c\uc6d0\uc774 \ub418\uc5c8\ub2e4.", "sentence_answer": "\ub178\ub9c8 \uc5d8\ub9ac\uc790\ubca0\uc2a4 \ubcf4\uc774\ub4dc\uac00 \uc8fc\ub3c4\ud55c \uc5ec\uc131\ud68c\ub294 1938\ub144 \uc2dc\ubbfc \ubbfc\uc8fc \uad8c\ub9ac\ub97c \uc704\ud55c \uad6d\uac00 \ube44\uc815\ub2f9 \ub85c\ube44(National Non-Partisan Lobby on Civil and Democratic Rights, \uc57d\uc790: NPC )", "paragraph_id": "6604746-4-1", "paragraph_question": "question: \uad6d\uac00 \ube44\uc815\ub2f9 \ud68c\uc758\uc758 \uc57d\uc790\ub294 \ubb34\uc5c7\uc778\uac00?, context: \ub178\ub9c8 \uc5d8\ub9ac\uc790\ubca0\uc2a4 \ubcf4\uc774\ub4dc\uac00 \uc8fc\ub3c4\ud55c \uc5ec\uc131\ud68c\ub294 1938\ub144 \uc2dc\ubbfc \ubbfc\uc8fc \uad8c\ub9ac\ub97c \uc704\ud55c \uad6d\uac00 \ube44\uc815\ub2f9 \ub85c\ube44(National Non-Partisan Lobby on Civil and Democratic Rights, \uc57d\uc790:NPC)\ub97c \ucc3d\uc124\ud588\uace0, \uc774\ud6c4 NPC\uc758 \uc774\ub984\uc744 \uacf5\ubb34\ub97c \uc704\ud55c \uad6d\uac00 \ube44\uc815\ub2f9 \ud68c\uc758(Non-Partisan Council on Public Affairs, \uc57d\uc790\ub294 \ubcc0\ud568\uc5c6\uc774 NPC)\ub85c \ubc14\uafb8\uc5c8\ub2e4. \uc774\uac83\uc740 \uc18c\uc218\ubbfc\uc871\uc758 \uad8c\ub9ac\ub97c \uc704\ud55c \ucd5c\ucd08\uc758 \uc804\uc784(\u5c08\u4efb) \uad6d\ud68c \ub85c\ube44 \ub2e8\uccb4\uc600\ub2e4. NPC\ub294 \uc874\uc7ac \uae30\uac04 \ub0b4\ub0b4 NAACP, \uc804\uad6d \ub3c4\uc2dc \ub3d9\ub9f9, UOPWA, NAGN, AFC, \uc720\uc0c9 \uc5ec\uc131 \ud074\ub7fd, BSCPA, \ub274\uc695 \ud22c\ud45c\uc790 \ub3d9\ub9f9\uacfc \ud611\ub825\ud588\ub2e4. NPC\ub294 12\ubc88\uc9f8 \ucd5c\uace0 \ubc14\uc2e4\ub808\uc624\uc2a4(Basileus, \uc9c0\ub3c4\uc790) \uc5d0\ub4dc\ub098 \uc624\ubc84 \uadf8\ub808\uc774\ucea0\ubca8\uc5d0 \uc758\ud574 1948\ub144 7\uc6d4 15\uc77c \ud574\uccb4\ub418\uc5c8\ub2e4. 1\ub144 \ud6c4, AKA\ub294 NAACP\uc758 \uccab \ubc88\uc9f8 \uc885\uc2e0\ud68c\uc6d0\uc774 \ub418\uc5c8\ub2e4."} {"question": "\uad6c\uc81c\uc5ed\uc740 \uba87 \uac1c\uc758 \ubc1c\uad7d\uc73c\ub85c \uac08\ub77c\uc9c4 \uac00\ucd95\ub4e4\uc5d0\uac8c \uac10\uc5fc\ub418\ub098?", "paragraph": "\uad6c\uc81c\uc5ed(\u53e3\u8e44\u75ab, \uc601\uc5b4: foot-and-mouth disease, hoof-and-mouth disease, \ud559\uba85: Aphtae epizoot) \ub610\ub294 \uc785\ubc1c\uad7d\ubcd1 \uc740 \uc18c\uc640 \ub3fc\uc9c0 \ub4f1 \uac00\ucd95\uc5d0 \ub300\ud55c \uc804\uc5fc\uc131\uc774 \ub192\uc740 \uae09\uc131 \ubc14\uc774\ub7ec\uc2a4\uc131 \uc804\uc5fc\ubcd1\uc758 \ud558\ub098\uc774\ub2e4. \uc0ac\uc2b4\uc774\ub098 \uc5fc\uc18c, \uc591\uacfc \uae30\ud0c0 \uc18c\uacfc \uc6b0\uc81c\ub958 \uac00\ucd95\ub4e4, \uadf8\ub9ac\uace0 \ucf54\ub07c\ub9ac, \uc950, \uace0\uc2b4\ub3c4\uce58 \ub4f1 \ubc1c\uad7d\uc774 \ub450\uac1c\ub85c \uac08\ub77c\uc9c4 \uac00\ucd95\ub4e4\uc5d0\uac8c \uac10\uc5fc\ub41c\ub2e4. \ub77c\ub9c8, \uc54c\ud30c\uce74\ub3c4 \uac00\ubcbc\uc6b4 \uc99d\uc0c1\uc744 \ubcf4\uc77c \uc218 \uc788\uc73c\ub098 \uc800\ud56d\ub825\uc744 \uac00\uc9c0\uace0 \uc788\uace0 \uac19\uc740 \uc885\uc758 \ub2e4\ub978 \ub3d9\ubb3c\uc5d0 \uc804\uc5fc\uc2dc\ud0a4\uc9c0 \uc54a\ub294\ub2e4. \uc2e4\ud5d8\uc2e4 \uc2e4\ud5d8\uc5d0\uc11c \uc950, \ub2ed\uc5d0 \uc778\uacf5\uc801\uc73c\ub85c \uac10\uc5fc\uc2dc\ud0a4\ub294\ub370 \uc131\uacf5\ud558\uc600\ub2e4 \ud558\ub098 \uc790\uc5f0\uc0c1\ud0dc\uc5d0\uc11c\ub3c4 \uac10\uc5fc\ub418\ub294\uc9c0\ub294 \ud655\uc2e4\ud558\uc9c0 \uc54a\ub2e4. \uc0ac\ub78c\ub3c4 \uade0\uc744 \uc637\uc774\ub098 \ubab8\uc5d0 \ubb3b\ud788\uace0 \ub2e4\ub2d8\uc73c\ub85c\uc368 \ubcd1\uc744 \ud37c\ub728\ub9b4 \uc218 \uc788\uc73c\ub098, \uc774 \ubcd1\uc5d0 \uac78\ub9ac\uc9c0 \uc54a\ub294 \ub3d9\ubb3c\uc740 \uc5b4\ub5a4 \ub3c4\uc6c0\uc774 \uc788\uc5b4\uc57c \uc804\ud30c\uc2dc\ud0a8\ub2e4. 1952\ub144 \uce90\ub098\ub2e4\uc5d0\uc11c\ub294 \uac1c\ub4e4\uc774 \uc8fd\uc740 \ub3d9\ubb3c\ub4e4\uc758 \ubf08\ub97c \uc62e\uae34 \ud6c4\uc5d0 \ud655\uc0b0\uc5d0 \ub2e4\uc2dc \ubd88\uae38\uc774 \ubd99\uc5c8\uace0, \uc61b \uc18c\ub828\uc5d0\uc11c\ub294 \ub291\ub300\uac00 \ube44\uc2b7\ud55c \uc5ed\ud560\uc744 \ud588\ub358 \uac83\uc73c\ub85c \uc0dd\uac01\ub418\uace0 \uc788\ub2e4. \uc0ac\ub78c\uc5d0 \uc601\ud5a5\uc744 \ubbf8\uce58\ub294 \uacbd\uc6b0\ub294 \ub9e4\uc6b0 \ub4dc\ubb3c\uba70, \uc77c\ubc18\uc801\uc73c\ub85c \ubcc4\ub2e4\ub978 \uce58\ub8cc\ub97c \ud558\uc9c0 \uc54a\ub294\ub2e4. \ub300\ud55c\ubbfc\uad6d\uc5d0\uc11c\ub294 \uc81c1\uc885\uac00\ucd95\uc804\uc5fc\ubcd1\uc73c\ub85c \uc9c0\uc815\ub418\uc5b4 \uc788\ub2e4. 1897\ub144\uc5d0 \ud504\ub9ac\ub4dc\ub9ac\ud788 \ub8b0\ud50c\ub7ec\uac00 \uad6c\uc81c\uc5ed\uc758 \uc6d0\uc778\uc740 \ubc14\uc774\ub7ec\uc2a4\ub77c\ub294 \uac83\uc744 \ubc1c\uacac\ud558\uc600\ub2e4. \uadf8\ub294 \uac10\uc5fc\ub41c \ub3d9\ubb3c\uc758 \ud608\uc561\uc744 \ud3ec\uc140\ub9b0 \ud544\ud130\uc5d0 \ud1b5\uacfc\uc2dc\ucf1c \uac78\ub7ec\ub3c4 \uc5ec\uc804\ud788 \ub2e4\ub978 \ub3d9\ubb3c\uc744 \uac10\uc5fc\uc2dc\ud0ac \uc218 \uc788\uc74c\uc744 \ud655\uc778\ud588\ub2e4. \uad6c\uc81c\uc5ed\uc740 \uc138\uacc4 \ub300\ubd80\ubd84\uc758 \uc9c0\uc5ed\uc5d0\uc11c \ubc1c\uc0dd\ud558\uba70, \uc219\uc8fc\uac00 \ub418\ub294 \ub3d9\ubb3c\uc758 \uc885\ub958\uc640 \uac1c\uccb4\uc218\uac00 \ub9ce\uace0 \uc804\uc5fc\uc131\uc774 \ub192\uae30 \ub54c\ubb38\uc5d0, \ud55c\ub3d9\uc548 \uad6c\uc81c\uc5ed\uc774 \ubc1c\uc0dd\ud558\uc9c0 \uc54a\ub358 \uacf3\uc5d0\uc11c\ub3c4 \ubc1c\uc0dd\ud560 \uc218 \uc788\ub2e4.", "answer": "\ub450\uac1c", "sentence": "\uc0ac\uc2b4\uc774\ub098 \uc5fc\uc18c, \uc591\uacfc \uae30\ud0c0 \uc18c\uacfc \uc6b0\uc81c\ub958 \uac00\ucd95\ub4e4, \uadf8\ub9ac\uace0 \ucf54\ub07c\ub9ac, \uc950, \uace0\uc2b4\ub3c4\uce58 \ub4f1 \ubc1c\uad7d\uc774 \ub450\uac1c \ub85c \uac08\ub77c\uc9c4 \uac00\ucd95\ub4e4\uc5d0\uac8c \uac10\uc5fc\ub41c\ub2e4.", "paragraph_sentence": "\uad6c\uc81c\uc5ed(\u53e3\u8e44\u75ab, \uc601\uc5b4: foot-and-mouth disease, hoof-and-mouth disease, \ud559\uba85: Aphtae epizoot) \ub610\ub294 \uc785\ubc1c\uad7d\ubcd1 \uc740 \uc18c\uc640 \ub3fc\uc9c0 \ub4f1 \uac00\ucd95\uc5d0 \ub300\ud55c \uc804\uc5fc\uc131\uc774 \ub192\uc740 \uae09\uc131 \ubc14\uc774\ub7ec\uc2a4\uc131 \uc804\uc5fc\ubcd1\uc758 \ud558\ub098\uc774\ub2e4. \uc0ac\uc2b4\uc774\ub098 \uc5fc\uc18c, \uc591\uacfc \uae30\ud0c0 \uc18c\uacfc \uc6b0\uc81c\ub958 \uac00\ucd95\ub4e4, \uadf8\ub9ac\uace0 \ucf54\ub07c\ub9ac, \uc950, \uace0\uc2b4\ub3c4\uce58 \ub4f1 \ubc1c\uad7d\uc774 \ub450\uac1c \ub85c \uac08\ub77c\uc9c4 \uac00\ucd95\ub4e4\uc5d0\uac8c \uac10\uc5fc\ub41c\ub2e4. \ub77c\ub9c8, \uc54c\ud30c\uce74\ub3c4 \uac00\ubcbc\uc6b4 \uc99d\uc0c1\uc744 \ubcf4\uc77c \uc218 \uc788\uc73c\ub098 \uc800\ud56d\ub825\uc744 \uac00\uc9c0\uace0 \uc788\uace0 \uac19\uc740 \uc885\uc758 \ub2e4\ub978 \ub3d9\ubb3c\uc5d0 \uc804\uc5fc\uc2dc\ud0a4\uc9c0 \uc54a\ub294\ub2e4. \uc2e4\ud5d8\uc2e4 \uc2e4\ud5d8\uc5d0\uc11c \uc950, \ub2ed\uc5d0 \uc778\uacf5\uc801\uc73c\ub85c \uac10\uc5fc\uc2dc\ud0a4\ub294\ub370 \uc131\uacf5\ud558\uc600\ub2e4 \ud558\ub098 \uc790\uc5f0\uc0c1\ud0dc\uc5d0\uc11c\ub3c4 \uac10\uc5fc\ub418\ub294\uc9c0\ub294 \ud655\uc2e4\ud558\uc9c0 \uc54a\ub2e4. \uc0ac\ub78c\ub3c4 \uade0\uc744 \uc637\uc774\ub098 \ubab8\uc5d0 \ubb3b\ud788\uace0 \ub2e4\ub2d8\uc73c\ub85c\uc368 \ubcd1\uc744 \ud37c\ub728\ub9b4 \uc218 \uc788\uc73c\ub098, \uc774 \ubcd1\uc5d0 \uac78\ub9ac\uc9c0 \uc54a\ub294 \ub3d9\ubb3c\uc740 \uc5b4\ub5a4 \ub3c4\uc6c0\uc774 \uc788\uc5b4\uc57c \uc804\ud30c\uc2dc\ud0a8\ub2e4. 1952\ub144 \uce90\ub098\ub2e4\uc5d0\uc11c\ub294 \uac1c\ub4e4\uc774 \uc8fd\uc740 \ub3d9\ubb3c\ub4e4\uc758 \ubf08\ub97c \uc62e\uae34 \ud6c4\uc5d0 \ud655\uc0b0\uc5d0 \ub2e4\uc2dc \ubd88\uae38\uc774 \ubd99\uc5c8\uace0, \uc61b \uc18c\ub828\uc5d0\uc11c\ub294 \ub291\ub300\uac00 \ube44\uc2b7\ud55c \uc5ed\ud560\uc744 \ud588\ub358 \uac83\uc73c\ub85c \uc0dd\uac01\ub418\uace0 \uc788\ub2e4. \uc0ac\ub78c\uc5d0 \uc601\ud5a5\uc744 \ubbf8\uce58\ub294 \uacbd\uc6b0\ub294 \ub9e4\uc6b0 \ub4dc\ubb3c\uba70, \uc77c\ubc18\uc801\uc73c\ub85c \ubcc4\ub2e4\ub978 \uce58\ub8cc\ub97c \ud558\uc9c0 \uc54a\ub294\ub2e4. \ub300\ud55c\ubbfc\uad6d\uc5d0\uc11c\ub294 \uc81c1\uc885\uac00\ucd95\uc804\uc5fc\ubcd1\uc73c\ub85c \uc9c0\uc815\ub418\uc5b4 \uc788\ub2e4. 1897\ub144\uc5d0 \ud504\ub9ac\ub4dc\ub9ac\ud788 \ub8b0\ud50c\ub7ec\uac00 \uad6c\uc81c\uc5ed\uc758 \uc6d0\uc778\uc740 \ubc14\uc774\ub7ec\uc2a4\ub77c\ub294 \uac83\uc744 \ubc1c\uacac\ud558\uc600\ub2e4. \uadf8\ub294 \uac10\uc5fc\ub41c \ub3d9\ubb3c\uc758 \ud608\uc561\uc744 \ud3ec\uc140\ub9b0 \ud544\ud130\uc5d0 \ud1b5\uacfc\uc2dc\ucf1c \uac78\ub7ec\ub3c4 \uc5ec\uc804\ud788 \ub2e4\ub978 \ub3d9\ubb3c\uc744 \uac10\uc5fc\uc2dc\ud0ac \uc218 \uc788\uc74c\uc744 \ud655\uc778\ud588\ub2e4. \uad6c\uc81c\uc5ed\uc740 \uc138\uacc4 \ub300\ubd80\ubd84\uc758 \uc9c0\uc5ed\uc5d0\uc11c \ubc1c\uc0dd\ud558\uba70, \uc219\uc8fc\uac00 \ub418\ub294 \ub3d9\ubb3c\uc758 \uc885\ub958\uc640 \uac1c\uccb4\uc218\uac00 \ub9ce\uace0 \uc804\uc5fc\uc131\uc774 \ub192\uae30 \ub54c\ubb38\uc5d0, \ud55c\ub3d9\uc548 \uad6c\uc81c\uc5ed\uc774 \ubc1c\uc0dd\ud558\uc9c0 \uc54a\ub358 \uacf3\uc5d0\uc11c\ub3c4 \ubc1c\uc0dd\ud560 \uc218 \uc788\ub2e4.", "paragraph_answer": "\uad6c\uc81c\uc5ed(\u53e3\u8e44\u75ab, \uc601\uc5b4: foot-and-mouth disease, hoof-and-mouth disease, \ud559\uba85: Aphtae epizoot) \ub610\ub294 \uc785\ubc1c\uad7d\ubcd1 \uc740 \uc18c\uc640 \ub3fc\uc9c0 \ub4f1 \uac00\ucd95\uc5d0 \ub300\ud55c \uc804\uc5fc\uc131\uc774 \ub192\uc740 \uae09\uc131 \ubc14\uc774\ub7ec\uc2a4\uc131 \uc804\uc5fc\ubcd1\uc758 \ud558\ub098\uc774\ub2e4. \uc0ac\uc2b4\uc774\ub098 \uc5fc\uc18c, \uc591\uacfc \uae30\ud0c0 \uc18c\uacfc \uc6b0\uc81c\ub958 \uac00\ucd95\ub4e4, \uadf8\ub9ac\uace0 \ucf54\ub07c\ub9ac, \uc950, \uace0\uc2b4\ub3c4\uce58 \ub4f1 \ubc1c\uad7d\uc774 \ub450\uac1c \ub85c \uac08\ub77c\uc9c4 \uac00\ucd95\ub4e4\uc5d0\uac8c \uac10\uc5fc\ub41c\ub2e4. \ub77c\ub9c8, \uc54c\ud30c\uce74\ub3c4 \uac00\ubcbc\uc6b4 \uc99d\uc0c1\uc744 \ubcf4\uc77c \uc218 \uc788\uc73c\ub098 \uc800\ud56d\ub825\uc744 \uac00\uc9c0\uace0 \uc788\uace0 \uac19\uc740 \uc885\uc758 \ub2e4\ub978 \ub3d9\ubb3c\uc5d0 \uc804\uc5fc\uc2dc\ud0a4\uc9c0 \uc54a\ub294\ub2e4. \uc2e4\ud5d8\uc2e4 \uc2e4\ud5d8\uc5d0\uc11c \uc950, \ub2ed\uc5d0 \uc778\uacf5\uc801\uc73c\ub85c \uac10\uc5fc\uc2dc\ud0a4\ub294\ub370 \uc131\uacf5\ud558\uc600\ub2e4 \ud558\ub098 \uc790\uc5f0\uc0c1\ud0dc\uc5d0\uc11c\ub3c4 \uac10\uc5fc\ub418\ub294\uc9c0\ub294 \ud655\uc2e4\ud558\uc9c0 \uc54a\ub2e4. \uc0ac\ub78c\ub3c4 \uade0\uc744 \uc637\uc774\ub098 \ubab8\uc5d0 \ubb3b\ud788\uace0 \ub2e4\ub2d8\uc73c\ub85c\uc368 \ubcd1\uc744 \ud37c\ub728\ub9b4 \uc218 \uc788\uc73c\ub098, \uc774 \ubcd1\uc5d0 \uac78\ub9ac\uc9c0 \uc54a\ub294 \ub3d9\ubb3c\uc740 \uc5b4\ub5a4 \ub3c4\uc6c0\uc774 \uc788\uc5b4\uc57c \uc804\ud30c\uc2dc\ud0a8\ub2e4. 1952\ub144 \uce90\ub098\ub2e4\uc5d0\uc11c\ub294 \uac1c\ub4e4\uc774 \uc8fd\uc740 \ub3d9\ubb3c\ub4e4\uc758 \ubf08\ub97c \uc62e\uae34 \ud6c4\uc5d0 \ud655\uc0b0\uc5d0 \ub2e4\uc2dc \ubd88\uae38\uc774 \ubd99\uc5c8\uace0, \uc61b \uc18c\ub828\uc5d0\uc11c\ub294 \ub291\ub300\uac00 \ube44\uc2b7\ud55c \uc5ed\ud560\uc744 \ud588\ub358 \uac83\uc73c\ub85c \uc0dd\uac01\ub418\uace0 \uc788\ub2e4. \uc0ac\ub78c\uc5d0 \uc601\ud5a5\uc744 \ubbf8\uce58\ub294 \uacbd\uc6b0\ub294 \ub9e4\uc6b0 \ub4dc\ubb3c\uba70, \uc77c\ubc18\uc801\uc73c\ub85c \ubcc4\ub2e4\ub978 \uce58\ub8cc\ub97c \ud558\uc9c0 \uc54a\ub294\ub2e4. \ub300\ud55c\ubbfc\uad6d\uc5d0\uc11c\ub294 \uc81c1\uc885\uac00\ucd95\uc804\uc5fc\ubcd1\uc73c\ub85c \uc9c0\uc815\ub418\uc5b4 \uc788\ub2e4. 1897\ub144\uc5d0 \ud504\ub9ac\ub4dc\ub9ac\ud788 \ub8b0\ud50c\ub7ec\uac00 \uad6c\uc81c\uc5ed\uc758 \uc6d0\uc778\uc740 \ubc14\uc774\ub7ec\uc2a4\ub77c\ub294 \uac83\uc744 \ubc1c\uacac\ud558\uc600\ub2e4. \uadf8\ub294 \uac10\uc5fc\ub41c \ub3d9\ubb3c\uc758 \ud608\uc561\uc744 \ud3ec\uc140\ub9b0 \ud544\ud130\uc5d0 \ud1b5\uacfc\uc2dc\ucf1c \uac78\ub7ec\ub3c4 \uc5ec\uc804\ud788 \ub2e4\ub978 \ub3d9\ubb3c\uc744 \uac10\uc5fc\uc2dc\ud0ac \uc218 \uc788\uc74c\uc744 \ud655\uc778\ud588\ub2e4. \uad6c\uc81c\uc5ed\uc740 \uc138\uacc4 \ub300\ubd80\ubd84\uc758 \uc9c0\uc5ed\uc5d0\uc11c \ubc1c\uc0dd\ud558\uba70, \uc219\uc8fc\uac00 \ub418\ub294 \ub3d9\ubb3c\uc758 \uc885\ub958\uc640 \uac1c\uccb4\uc218\uac00 \ub9ce\uace0 \uc804\uc5fc\uc131\uc774 \ub192\uae30 \ub54c\ubb38\uc5d0, \ud55c\ub3d9\uc548 \uad6c\uc81c\uc5ed\uc774 \ubc1c\uc0dd\ud558\uc9c0 \uc54a\ub358 \uacf3\uc5d0\uc11c\ub3c4 \ubc1c\uc0dd\ud560 \uc218 \uc788\ub2e4.", "sentence_answer": "\uc0ac\uc2b4\uc774\ub098 \uc5fc\uc18c, \uc591\uacfc \uae30\ud0c0 \uc18c\uacfc \uc6b0\uc81c\ub958 \uac00\ucd95\ub4e4, \uadf8\ub9ac\uace0 \ucf54\ub07c\ub9ac, \uc950, \uace0\uc2b4\ub3c4\uce58 \ub4f1 \ubc1c\uad7d\uc774 \ub450\uac1c \ub85c \uac08\ub77c\uc9c4 \uac00\ucd95\ub4e4\uc5d0\uac8c \uac10\uc5fc\ub41c\ub2e4.", "paragraph_id": "6477205-0-1", "paragraph_question": "question: \uad6c\uc81c\uc5ed\uc740 \uba87 \uac1c\uc758 \ubc1c\uad7d\uc73c\ub85c \uac08\ub77c\uc9c4 \uac00\ucd95\ub4e4\uc5d0\uac8c \uac10\uc5fc\ub418\ub098?, context: \uad6c\uc81c\uc5ed(\u53e3\u8e44\u75ab, \uc601\uc5b4: foot-and-mouth disease, hoof-and-mouth disease, \ud559\uba85: Aphtae epizoot) \ub610\ub294 \uc785\ubc1c\uad7d\ubcd1 \uc740 \uc18c\uc640 \ub3fc\uc9c0 \ub4f1 \uac00\ucd95\uc5d0 \ub300\ud55c \uc804\uc5fc\uc131\uc774 \ub192\uc740 \uae09\uc131 \ubc14\uc774\ub7ec\uc2a4\uc131 \uc804\uc5fc\ubcd1\uc758 \ud558\ub098\uc774\ub2e4. \uc0ac\uc2b4\uc774\ub098 \uc5fc\uc18c, \uc591\uacfc \uae30\ud0c0 \uc18c\uacfc \uc6b0\uc81c\ub958 \uac00\ucd95\ub4e4, \uadf8\ub9ac\uace0 \ucf54\ub07c\ub9ac, \uc950, \uace0\uc2b4\ub3c4\uce58 \ub4f1 \ubc1c\uad7d\uc774 \ub450\uac1c\ub85c \uac08\ub77c\uc9c4 \uac00\ucd95\ub4e4\uc5d0\uac8c \uac10\uc5fc\ub41c\ub2e4. \ub77c\ub9c8, \uc54c\ud30c\uce74\ub3c4 \uac00\ubcbc\uc6b4 \uc99d\uc0c1\uc744 \ubcf4\uc77c \uc218 \uc788\uc73c\ub098 \uc800\ud56d\ub825\uc744 \uac00\uc9c0\uace0 \uc788\uace0 \uac19\uc740 \uc885\uc758 \ub2e4\ub978 \ub3d9\ubb3c\uc5d0 \uc804\uc5fc\uc2dc\ud0a4\uc9c0 \uc54a\ub294\ub2e4. \uc2e4\ud5d8\uc2e4 \uc2e4\ud5d8\uc5d0\uc11c \uc950, \ub2ed\uc5d0 \uc778\uacf5\uc801\uc73c\ub85c \uac10\uc5fc\uc2dc\ud0a4\ub294\ub370 \uc131\uacf5\ud558\uc600\ub2e4 \ud558\ub098 \uc790\uc5f0\uc0c1\ud0dc\uc5d0\uc11c\ub3c4 \uac10\uc5fc\ub418\ub294\uc9c0\ub294 \ud655\uc2e4\ud558\uc9c0 \uc54a\ub2e4. \uc0ac\ub78c\ub3c4 \uade0\uc744 \uc637\uc774\ub098 \ubab8\uc5d0 \ubb3b\ud788\uace0 \ub2e4\ub2d8\uc73c\ub85c\uc368 \ubcd1\uc744 \ud37c\ub728\ub9b4 \uc218 \uc788\uc73c\ub098, \uc774 \ubcd1\uc5d0 \uac78\ub9ac\uc9c0 \uc54a\ub294 \ub3d9\ubb3c\uc740 \uc5b4\ub5a4 \ub3c4\uc6c0\uc774 \uc788\uc5b4\uc57c \uc804\ud30c\uc2dc\ud0a8\ub2e4. 1952\ub144 \uce90\ub098\ub2e4\uc5d0\uc11c\ub294 \uac1c\ub4e4\uc774 \uc8fd\uc740 \ub3d9\ubb3c\ub4e4\uc758 \ubf08\ub97c \uc62e\uae34 \ud6c4\uc5d0 \ud655\uc0b0\uc5d0 \ub2e4\uc2dc \ubd88\uae38\uc774 \ubd99\uc5c8\uace0, \uc61b \uc18c\ub828\uc5d0\uc11c\ub294 \ub291\ub300\uac00 \ube44\uc2b7\ud55c \uc5ed\ud560\uc744 \ud588\ub358 \uac83\uc73c\ub85c \uc0dd\uac01\ub418\uace0 \uc788\ub2e4. \uc0ac\ub78c\uc5d0 \uc601\ud5a5\uc744 \ubbf8\uce58\ub294 \uacbd\uc6b0\ub294 \ub9e4\uc6b0 \ub4dc\ubb3c\uba70, \uc77c\ubc18\uc801\uc73c\ub85c \ubcc4\ub2e4\ub978 \uce58\ub8cc\ub97c \ud558\uc9c0 \uc54a\ub294\ub2e4. \ub300\ud55c\ubbfc\uad6d\uc5d0\uc11c\ub294 \uc81c1\uc885\uac00\ucd95\uc804\uc5fc\ubcd1\uc73c\ub85c \uc9c0\uc815\ub418\uc5b4 \uc788\ub2e4. 1897\ub144\uc5d0 \ud504\ub9ac\ub4dc\ub9ac\ud788 \ub8b0\ud50c\ub7ec\uac00 \uad6c\uc81c\uc5ed\uc758 \uc6d0\uc778\uc740 \ubc14\uc774\ub7ec\uc2a4\ub77c\ub294 \uac83\uc744 \ubc1c\uacac\ud558\uc600\ub2e4. \uadf8\ub294 \uac10\uc5fc\ub41c \ub3d9\ubb3c\uc758 \ud608\uc561\uc744 \ud3ec\uc140\ub9b0 \ud544\ud130\uc5d0 \ud1b5\uacfc\uc2dc\ucf1c \uac78\ub7ec\ub3c4 \uc5ec\uc804\ud788 \ub2e4\ub978 \ub3d9\ubb3c\uc744 \uac10\uc5fc\uc2dc\ud0ac \uc218 \uc788\uc74c\uc744 \ud655\uc778\ud588\ub2e4. \uad6c\uc81c\uc5ed\uc740 \uc138\uacc4 \ub300\ubd80\ubd84\uc758 \uc9c0\uc5ed\uc5d0\uc11c \ubc1c\uc0dd\ud558\uba70, \uc219\uc8fc\uac00 \ub418\ub294 \ub3d9\ubb3c\uc758 \uc885\ub958\uc640 \uac1c\uccb4\uc218\uac00 \ub9ce\uace0 \uc804\uc5fc\uc131\uc774 \ub192\uae30 \ub54c\ubb38\uc5d0, \ud55c\ub3d9\uc548 \uad6c\uc81c\uc5ed\uc774 \ubc1c\uc0dd\ud558\uc9c0 \uc54a\ub358 \uacf3\uc5d0\uc11c\ub3c4 \ubc1c\uc0dd\ud560 \uc218 \uc788\ub2e4."} {"question": "614\ub144\uacfc 615\ub144\uc5d0 \uc2a4\ud30c\ub2c8\uc544\ub97c \ud06c\uac8c \uacf5\uaca9\ud588\ub358 \uc2a4\ud30c\ub2c8\uc544\ub97c \uc815\ubc8c\ud558\ub294\ub370 \uac00\ub2a5 \ud070 \uc5c5\uc801\uc744 \ub0a8\uae34 \uc655\uc740?", "paragraph": "\ud6c4\ub300 \uc655 \uc911\uc5d0\uc11c \ube44\ud14c\ub9ac\ud06c \uc655\uc740 \uc2a4\ud30c\ub2c8\uc544 \uc138\ub825\uc5d0 \ub300\ud574 \ube48\ubc88\ud558\uac8c \ubc18\ub300 \ud65c\ub3d9\uc744 \ubc8c\uc600\uc73c\uba70 \uadf8\uc758 \ud718\ud558\uc5d0 \uc788\ub358 \uc7a5\uc218\ub4e4\uc740 \ud6e8\uc52c \ub354 \ud070 \ud6a8\uacfc\ub97c \uac70\ub480\ub2e4. \uad70\ub370\ub9c8\ub974 \uc655\uc740 \ud788\uc2a4\ud30c\ub2c8\uc544(Carthaginiensis)\uc758 \uad50\ud669\uccad\uc744 \ube44\uc794\ud2f4 \uce74\ub974\ud0c0\ud5e4\ub098\uc5d0\uc11c \uc11c\uace0\ud2b8\ub839 \ud1a8\ub808\ub3c4\ub85c 610\ub144 \uc62e\uacbc\uc73c\uba70 611\ub144 \uacf5\uc138\ub97c \ud37c\ubd80\uc5b4 \uc2a4\ud30c\ub2c8\uc544\ub97c \ubcf5\uc18d\ud558\ub824 \ub4e4\uc5c8\uc9c0\ub9cc \ud2b9\ubcc4\ud55c \uc601\ud5a5\uc774 \uc788\uc5c8\ub358 \uac83\uc740 \uc544\ub2c8\uc5c8\ub2e4. \uc2dc\uc138\ubd80\ud2b8 \uc655\uc740 \ube44\uc794\ud2f4\ub839 \uc2a4\ud30c\ub2c8\uc544\ub97c \uc815\ubc8c\ud558\ub294 \ub370 \uac00\uc7a5 \ud601\ud601\ud55c \uc5c5\uc801\uc744 \ub0a8\uae34 \uc655\uc774\uc5c8\uc73c\uba70 614\ub144\uacfc 615\ub144\uc5d0 \ub450 \ubc88\uc758 \ud070 \uacf5\uaca9\uc744 \uac10\ud589\ud558\uc5ec 619\ub144 \uc804\uc5d0 \ub9d0\ub77c\uac00\ub97c \uc815\ubc8c\ud558\ub294 \ud55c\ud3b8 \uc9c0\uc911\ud574 \uc5f0\uc548\uae4c\uc9c0 \uc138\ub97c \ud655\uc7a5\ud588\ub2e4. \uc8fc\ubcc0 \ub3c4\uc2dc\ub4e4\uae4c\uc9c0 \ubb34\ub825\ud654\uc2dc\ud0a4\uba70 \uc601\ud1a0\uac00 \ud06c\uac8c \ud655\uc7a5\ub418\uc790 \u300a\ud504\ub808\ub370\uac00\ub974\uc758 \uc5f0\ub300\uae30Chronicle of Fredegar\u300b\uc5d0\ub294 \uadf8\ub97c \ub450\uace0 \uac70\uc758 \ub300\ubd80\ubd84\uc758 \ub85c\ub9c8 \ub3c4\uc2dc\ub97c \ud30c\uad34\ud558\uc5ec \ubc8c\ubc8c \uae30\uac8c \ub9cc\ub4e4\uc5c8\ub2e4\uace0 \uc37c\ub2e4.", "answer": "\uc2dc\uc138\ubd80\ud2b8 \uc655", "sentence": "\uc2dc\uc138\ubd80\ud2b8 \uc655 \uc740 \ube44\uc794\ud2f4\ub839 \uc2a4\ud30c\ub2c8\uc544\ub97c \uc815\ubc8c\ud558\ub294 \ub370 \uac00\uc7a5 \ud601\ud601\ud55c \uc5c5\uc801\uc744 \ub0a8\uae34 \uc655\uc774\uc5c8\uc73c\uba70 614\ub144\uacfc 615\ub144\uc5d0 \ub450 \ubc88\uc758 \ud070 \uacf5\uaca9\uc744 \uac10\ud589\ud558\uc5ec 619\ub144 \uc804\uc5d0 \ub9d0\ub77c\uac00\ub97c \uc815\ubc8c\ud558\ub294 \ud55c\ud3b8 \uc9c0\uc911\ud574 \uc5f0\uc548\uae4c\uc9c0 \uc138\ub97c \ud655\uc7a5\ud588\ub2e4.", "paragraph_sentence": "\ud6c4\ub300 \uc655 \uc911\uc5d0\uc11c \ube44\ud14c\ub9ac\ud06c \uc655\uc740 \uc2a4\ud30c\ub2c8\uc544 \uc138\ub825\uc5d0 \ub300\ud574 \ube48\ubc88\ud558\uac8c \ubc18\ub300 \ud65c\ub3d9\uc744 \ubc8c\uc600\uc73c\uba70 \uadf8\uc758 \ud718\ud558\uc5d0 \uc788\ub358 \uc7a5\uc218\ub4e4\uc740 \ud6e8\uc52c \ub354 \ud070 \ud6a8\uacfc\ub97c \uac70\ub480\ub2e4. \uad70\ub370\ub9c8\ub974 \uc655\uc740 \ud788\uc2a4\ud30c\ub2c8\uc544(Carthaginiensis)\uc758 \uad50\ud669\uccad\uc744 \ube44\uc794\ud2f4 \uce74\ub974\ud0c0\ud5e4\ub098\uc5d0\uc11c \uc11c\uace0\ud2b8\ub839 \ud1a8\ub808\ub3c4\ub85c 610\ub144 \uc62e\uacbc\uc73c\uba70 611\ub144 \uacf5\uc138\ub97c \ud37c\ubd80\uc5b4 \uc2a4\ud30c\ub2c8\uc544\ub97c \ubcf5\uc18d\ud558\ub824 \ub4e4\uc5c8\uc9c0\ub9cc \ud2b9\ubcc4\ud55c \uc601\ud5a5\uc774 \uc788\uc5c8\ub358 \uac83\uc740 \uc544\ub2c8\uc5c8\ub2e4. \uc2dc\uc138\ubd80\ud2b8 \uc655 \uc740 \ube44\uc794\ud2f4\ub839 \uc2a4\ud30c\ub2c8\uc544\ub97c \uc815\ubc8c\ud558\ub294 \ub370 \uac00\uc7a5 \ud601\ud601\ud55c \uc5c5\uc801\uc744 \ub0a8\uae34 \uc655\uc774\uc5c8\uc73c\uba70 614\ub144\uacfc 615\ub144\uc5d0 \ub450 \ubc88\uc758 \ud070 \uacf5\uaca9\uc744 \uac10\ud589\ud558\uc5ec 619\ub144 \uc804\uc5d0 \ub9d0\ub77c\uac00\ub97c \uc815\ubc8c\ud558\ub294 \ud55c\ud3b8 \uc9c0\uc911\ud574 \uc5f0\uc548\uae4c\uc9c0 \uc138\ub97c \ud655\uc7a5\ud588\ub2e4. \uc8fc\ubcc0 \ub3c4\uc2dc\ub4e4\uae4c\uc9c0 \ubb34\ub825\ud654\uc2dc\ud0a4\uba70 \uc601\ud1a0\uac00 \ud06c\uac8c \ud655\uc7a5\ub418\uc790 \u300a\ud504\ub808\ub370\uac00\ub974\uc758 \uc5f0\ub300\uae30Chronicle of Fredegar\u300b\uc5d0\ub294 \uadf8\ub97c \ub450\uace0 \uac70\uc758 \ub300\ubd80\ubd84\uc758 \ub85c\ub9c8 \ub3c4\uc2dc\ub97c \ud30c\uad34\ud558\uc5ec \ubc8c\ubc8c \uae30\uac8c \ub9cc\ub4e4\uc5c8\ub2e4\uace0 \uc37c\ub2e4.", "paragraph_answer": "\ud6c4\ub300 \uc655 \uc911\uc5d0\uc11c \ube44\ud14c\ub9ac\ud06c \uc655\uc740 \uc2a4\ud30c\ub2c8\uc544 \uc138\ub825\uc5d0 \ub300\ud574 \ube48\ubc88\ud558\uac8c \ubc18\ub300 \ud65c\ub3d9\uc744 \ubc8c\uc600\uc73c\uba70 \uadf8\uc758 \ud718\ud558\uc5d0 \uc788\ub358 \uc7a5\uc218\ub4e4\uc740 \ud6e8\uc52c \ub354 \ud070 \ud6a8\uacfc\ub97c \uac70\ub480\ub2e4. \uad70\ub370\ub9c8\ub974 \uc655\uc740 \ud788\uc2a4\ud30c\ub2c8\uc544(Carthaginiensis)\uc758 \uad50\ud669\uccad\uc744 \ube44\uc794\ud2f4 \uce74\ub974\ud0c0\ud5e4\ub098\uc5d0\uc11c \uc11c\uace0\ud2b8\ub839 \ud1a8\ub808\ub3c4\ub85c 610\ub144 \uc62e\uacbc\uc73c\uba70 611\ub144 \uacf5\uc138\ub97c \ud37c\ubd80\uc5b4 \uc2a4\ud30c\ub2c8\uc544\ub97c \ubcf5\uc18d\ud558\ub824 \ub4e4\uc5c8\uc9c0\ub9cc \ud2b9\ubcc4\ud55c \uc601\ud5a5\uc774 \uc788\uc5c8\ub358 \uac83\uc740 \uc544\ub2c8\uc5c8\ub2e4. \uc2dc\uc138\ubd80\ud2b8 \uc655 \uc740 \ube44\uc794\ud2f4\ub839 \uc2a4\ud30c\ub2c8\uc544\ub97c \uc815\ubc8c\ud558\ub294 \ub370 \uac00\uc7a5 \ud601\ud601\ud55c \uc5c5\uc801\uc744 \ub0a8\uae34 \uc655\uc774\uc5c8\uc73c\uba70 614\ub144\uacfc 615\ub144\uc5d0 \ub450 \ubc88\uc758 \ud070 \uacf5\uaca9\uc744 \uac10\ud589\ud558\uc5ec 619\ub144 \uc804\uc5d0 \ub9d0\ub77c\uac00\ub97c \uc815\ubc8c\ud558\ub294 \ud55c\ud3b8 \uc9c0\uc911\ud574 \uc5f0\uc548\uae4c\uc9c0 \uc138\ub97c \ud655\uc7a5\ud588\ub2e4. \uc8fc\ubcc0 \ub3c4\uc2dc\ub4e4\uae4c\uc9c0 \ubb34\ub825\ud654\uc2dc\ud0a4\uba70 \uc601\ud1a0\uac00 \ud06c\uac8c \ud655\uc7a5\ub418\uc790 \u300a\ud504\ub808\ub370\uac00\ub974\uc758 \uc5f0\ub300\uae30Chronicle of Fredegar\u300b\uc5d0\ub294 \uadf8\ub97c \ub450\uace0 \uac70\uc758 \ub300\ubd80\ubd84\uc758 \ub85c\ub9c8 \ub3c4\uc2dc\ub97c \ud30c\uad34\ud558\uc5ec \ubc8c\ubc8c \uae30\uac8c \ub9cc\ub4e4\uc5c8\ub2e4\uace0 \uc37c\ub2e4.", "sentence_answer": " \uc2dc\uc138\ubd80\ud2b8 \uc655 \uc740 \ube44\uc794\ud2f4\ub839 \uc2a4\ud30c\ub2c8\uc544\ub97c \uc815\ubc8c\ud558\ub294 \ub370 \uac00\uc7a5 \ud601\ud601\ud55c \uc5c5\uc801\uc744 \ub0a8\uae34 \uc655\uc774\uc5c8\uc73c\uba70 614\ub144\uacfc 615\ub144\uc5d0 \ub450 \ubc88\uc758 \ud070 \uacf5\uaca9\uc744 \uac10\ud589\ud558\uc5ec 619\ub144 \uc804\uc5d0 \ub9d0\ub77c\uac00\ub97c \uc815\ubc8c\ud558\ub294 \ud55c\ud3b8 \uc9c0\uc911\ud574 \uc5f0\uc548\uae4c\uc9c0 \uc138\ub97c \ud655\uc7a5\ud588\ub2e4.", "paragraph_id": "6457325-2-1", "paragraph_question": "question: 614\ub144\uacfc 615\ub144\uc5d0 \uc2a4\ud30c\ub2c8\uc544\ub97c \ud06c\uac8c \uacf5\uaca9\ud588\ub358 \uc2a4\ud30c\ub2c8\uc544\ub97c \uc815\ubc8c\ud558\ub294\ub370 \uac00\ub2a5 \ud070 \uc5c5\uc801\uc744 \ub0a8\uae34 \uc655\uc740?, context: \ud6c4\ub300 \uc655 \uc911\uc5d0\uc11c \ube44\ud14c\ub9ac\ud06c \uc655\uc740 \uc2a4\ud30c\ub2c8\uc544 \uc138\ub825\uc5d0 \ub300\ud574 \ube48\ubc88\ud558\uac8c \ubc18\ub300 \ud65c\ub3d9\uc744 \ubc8c\uc600\uc73c\uba70 \uadf8\uc758 \ud718\ud558\uc5d0 \uc788\ub358 \uc7a5\uc218\ub4e4\uc740 \ud6e8\uc52c \ub354 \ud070 \ud6a8\uacfc\ub97c \uac70\ub480\ub2e4. \uad70\ub370\ub9c8\ub974 \uc655\uc740 \ud788\uc2a4\ud30c\ub2c8\uc544(Carthaginiensis)\uc758 \uad50\ud669\uccad\uc744 \ube44\uc794\ud2f4 \uce74\ub974\ud0c0\ud5e4\ub098\uc5d0\uc11c \uc11c\uace0\ud2b8\ub839 \ud1a8\ub808\ub3c4\ub85c 610\ub144 \uc62e\uacbc\uc73c\uba70 611\ub144 \uacf5\uc138\ub97c \ud37c\ubd80\uc5b4 \uc2a4\ud30c\ub2c8\uc544\ub97c \ubcf5\uc18d\ud558\ub824 \ub4e4\uc5c8\uc9c0\ub9cc \ud2b9\ubcc4\ud55c \uc601\ud5a5\uc774 \uc788\uc5c8\ub358 \uac83\uc740 \uc544\ub2c8\uc5c8\ub2e4. \uc2dc\uc138\ubd80\ud2b8 \uc655\uc740 \ube44\uc794\ud2f4\ub839 \uc2a4\ud30c\ub2c8\uc544\ub97c \uc815\ubc8c\ud558\ub294 \ub370 \uac00\uc7a5 \ud601\ud601\ud55c \uc5c5\uc801\uc744 \ub0a8\uae34 \uc655\uc774\uc5c8\uc73c\uba70 614\ub144\uacfc 615\ub144\uc5d0 \ub450 \ubc88\uc758 \ud070 \uacf5\uaca9\uc744 \uac10\ud589\ud558\uc5ec 619\ub144 \uc804\uc5d0 \ub9d0\ub77c\uac00\ub97c \uc815\ubc8c\ud558\ub294 \ud55c\ud3b8 \uc9c0\uc911\ud574 \uc5f0\uc548\uae4c\uc9c0 \uc138\ub97c \ud655\uc7a5\ud588\ub2e4. \uc8fc\ubcc0 \ub3c4\uc2dc\ub4e4\uae4c\uc9c0 \ubb34\ub825\ud654\uc2dc\ud0a4\uba70 \uc601\ud1a0\uac00 \ud06c\uac8c \ud655\uc7a5\ub418\uc790 \u300a\ud504\ub808\ub370\uac00\ub974\uc758 \uc5f0\ub300\uae30Chronicle of Fredegar\u300b\uc5d0\ub294 \uadf8\ub97c \ub450\uace0 \uac70\uc758 \ub300\ubd80\ubd84\uc758 \ub85c\ub9c8 \ub3c4\uc2dc\ub97c \ud30c\uad34\ud558\uc5ec \ubc8c\ubc8c \uae30\uac8c \ub9cc\ub4e4\uc5c8\ub2e4\uace0 \uc37c\ub2e4."} {"question": "\uc911\uad6d\uc758 \ub098\uc778\ubd07\uc0ac\uac00 \uc138\uadf8\uc6e8\uc774\ub97c \uac1c\ub7c9\ud55c \uae30\uae30\ub294?", "paragraph": "\ucd08\uae30 \uc138\uadf8\uc6e8\uc774\uc758 \uac00\uaca9\uc740 1000\ub9cc \uc6d0 \ub300\ub97c \ud6cc\uca4d \ub6f0\uc5b4\ub118\uc5c8\ub2e4. \uc77c\ubc18 \ub300\uc911\ub4e4\uc774 \ub808\uc838\uc6a9 \ud639\uc740 \uc77c\uc0c1\uc5d0\uc11c \uc0dd\ud65c\ud558\ub824\uace0 \uad6c\ub9e4\ud558\uae30\uc5d0\ub294 \ubd80\ub2f4\uc2a4\ub7f0\uc6b4 \uae08\uc561\uc774\uc5c8\ub2e4. 2015\ub144 \ud604\uc7ac\uc5d0 \uc774\ub974\uae30\uae4c\uc9c0 \uac00\uaca9\uc774 \uc810\uc810 \ud558\ub77d\ud558\uae30\ub294 \ud588\uc9c0\ub9cc \uc544\uc9c1\uae4c\uc9c0\ub294 \ub300\uc911\ub4e4\uc774 \ubc1b\uc544\ub4e4\uc774\uae30\uc5d0\ub294 \ube44\uc2fc \uac00\uaca9\uc744 \uc720\uc9c0\ud558\uace0 \uc788\ub294 \ud604\uc2e4\uc774\ub2e4. \uc774\uc5d0 \ub300\ud55c \ud0c0\uac1c\ucc45\uc73c\ub85c \uc138\uadf8\uc6e8\uc774\uc0ac\ub97c \uc778\uc218\ud55c \uc911\uad6d\uc758 \ub098\uc778\ubd07\uc0ac\uac00 2015\ub144 10\uc6d4 19\uc77c '\ub098\uc778\ubd07 \ubbf8\ub2c8'\uc774\ub77c\ub294 \uc138\uadf8\uc6e8\uc774\ub97c \uac1c\ub7c9\ud55c \uae30\uae30\ub97c \ubc1c\ub9e4\ud558\uc5ec \uc911\uad6d\ub0b4\uc218\uc2dc\uc7a5\uc5d0\uc11c \ud55c\ud654\ub85c \uc57d 35\ub9cc\uc6d0\uc758 \uac00\uaca9\uc5d0 \ud310\ub9e4\ud558\uace0 \uc788\ub2e4. \uae30\uae30\uc758 \uae30\ub2a5\uc801 \uce21\uba74\uc5d0\uc11c \ud55c\uacc4\uc810\uc73c\ub85c\ub294 \uac00\ub3d9\uac70\ub9ac\ub97c \uaf3d\uc744 \uc218 \uc788\ub2e4. \ud55c \ubc88 \ucda9\uc804\uc744 \ud558\uba74 \ucd5c\ub300 25~30km\uae4c\uc9c0\ubc16\uc5d0 \uc6b4\ud589\uc744 \ud560 \uc218 \uc5c6\uae30 \ub54c\ubb38\uc5d0 \uc774 \ubd80\ubd84\uc5d0\uc11c\ub3c4 \ud55c\uacc4\uc810\uc744 \uc9c0\ub2c8\uace0 \uc788\ub2e4.", "answer": "'\ub098\uc778\ubd07 \ubbf8\ub2c8'", "sentence": "\uc774\uc5d0 \ub300\ud55c \ud0c0\uac1c\ucc45\uc73c\ub85c \uc138\uadf8\uc6e8\uc774\uc0ac\ub97c \uc778\uc218\ud55c \uc911\uad6d\uc758 \ub098\uc778\ubd07\uc0ac\uac00 2015\ub144 10\uc6d4 19\uc77c '\ub098\uc778\ubd07 \ubbf8\ub2c8' \uc774\ub77c\ub294 \uc138\uadf8\uc6e8\uc774\ub97c \uac1c\ub7c9\ud55c \uae30\uae30\ub97c \ubc1c\ub9e4\ud558\uc5ec \uc911\uad6d\ub0b4\uc218\uc2dc\uc7a5\uc5d0\uc11c \ud55c\ud654\ub85c \uc57d 35\ub9cc\uc6d0\uc758 \uac00\uaca9\uc5d0 \ud310\ub9e4\ud558\uace0 \uc788\ub2e4.", "paragraph_sentence": "\ucd08\uae30 \uc138\uadf8\uc6e8\uc774\uc758 \uac00\uaca9\uc740 1000\ub9cc \uc6d0 \ub300\ub97c \ud6cc\uca4d \ub6f0\uc5b4\ub118\uc5c8\ub2e4. \uc77c\ubc18 \ub300\uc911\ub4e4\uc774 \ub808\uc838\uc6a9 \ud639\uc740 \uc77c\uc0c1\uc5d0\uc11c \uc0dd\ud65c\ud558\ub824\uace0 \uad6c\ub9e4\ud558\uae30\uc5d0\ub294 \ubd80\ub2f4\uc2a4\ub7f0\uc6b4 \uae08\uc561\uc774\uc5c8\ub2e4. 2015\ub144 \ud604\uc7ac\uc5d0 \uc774\ub974\uae30\uae4c\uc9c0 \uac00\uaca9\uc774 \uc810\uc810 \ud558\ub77d\ud558\uae30\ub294 \ud588\uc9c0\ub9cc \uc544\uc9c1\uae4c\uc9c0\ub294 \ub300\uc911\ub4e4\uc774 \ubc1b\uc544\ub4e4\uc774\uae30\uc5d0\ub294 \ube44\uc2fc \uac00\uaca9\uc744 \uc720\uc9c0\ud558\uace0 \uc788\ub294 \ud604\uc2e4\uc774\ub2e4. \uc774\uc5d0 \ub300\ud55c \ud0c0\uac1c\ucc45\uc73c\ub85c \uc138\uadf8\uc6e8\uc774\uc0ac\ub97c \uc778\uc218\ud55c \uc911\uad6d\uc758 \ub098\uc778\ubd07\uc0ac\uac00 2015\ub144 10\uc6d4 19\uc77c '\ub098\uc778\ubd07 \ubbf8\ub2c8' \uc774\ub77c\ub294 \uc138\uadf8\uc6e8\uc774\ub97c \uac1c\ub7c9\ud55c \uae30\uae30\ub97c \ubc1c\ub9e4\ud558\uc5ec \uc911\uad6d\ub0b4\uc218\uc2dc\uc7a5\uc5d0\uc11c \ud55c\ud654\ub85c \uc57d 35\ub9cc\uc6d0\uc758 \uac00\uaca9\uc5d0 \ud310\ub9e4\ud558\uace0 \uc788\ub2e4. \uae30\uae30\uc758 \uae30\ub2a5\uc801 \uce21\uba74\uc5d0\uc11c \ud55c\uacc4\uc810\uc73c\ub85c\ub294 \uac00\ub3d9\uac70\ub9ac\ub97c \uaf3d\uc744 \uc218 \uc788\ub2e4. \ud55c \ubc88 \ucda9\uc804\uc744 \ud558\uba74 \ucd5c\ub300 25~30km\uae4c\uc9c0\ubc16\uc5d0 \uc6b4\ud589\uc744 \ud560 \uc218 \uc5c6\uae30 \ub54c\ubb38\uc5d0 \uc774 \ubd80\ubd84\uc5d0\uc11c\ub3c4 \ud55c\uacc4\uc810\uc744 \uc9c0\ub2c8\uace0 \uc788\ub2e4.", "paragraph_answer": "\ucd08\uae30 \uc138\uadf8\uc6e8\uc774\uc758 \uac00\uaca9\uc740 1000\ub9cc \uc6d0 \ub300\ub97c \ud6cc\uca4d \ub6f0\uc5b4\ub118\uc5c8\ub2e4. \uc77c\ubc18 \ub300\uc911\ub4e4\uc774 \ub808\uc838\uc6a9 \ud639\uc740 \uc77c\uc0c1\uc5d0\uc11c \uc0dd\ud65c\ud558\ub824\uace0 \uad6c\ub9e4\ud558\uae30\uc5d0\ub294 \ubd80\ub2f4\uc2a4\ub7f0\uc6b4 \uae08\uc561\uc774\uc5c8\ub2e4. 2015\ub144 \ud604\uc7ac\uc5d0 \uc774\ub974\uae30\uae4c\uc9c0 \uac00\uaca9\uc774 \uc810\uc810 \ud558\ub77d\ud558\uae30\ub294 \ud588\uc9c0\ub9cc \uc544\uc9c1\uae4c\uc9c0\ub294 \ub300\uc911\ub4e4\uc774 \ubc1b\uc544\ub4e4\uc774\uae30\uc5d0\ub294 \ube44\uc2fc \uac00\uaca9\uc744 \uc720\uc9c0\ud558\uace0 \uc788\ub294 \ud604\uc2e4\uc774\ub2e4. \uc774\uc5d0 \ub300\ud55c \ud0c0\uac1c\ucc45\uc73c\ub85c \uc138\uadf8\uc6e8\uc774\uc0ac\ub97c \uc778\uc218\ud55c \uc911\uad6d\uc758 \ub098\uc778\ubd07\uc0ac\uac00 2015\ub144 10\uc6d4 19\uc77c '\ub098\uc778\ubd07 \ubbf8\ub2c8' \uc774\ub77c\ub294 \uc138\uadf8\uc6e8\uc774\ub97c \uac1c\ub7c9\ud55c \uae30\uae30\ub97c \ubc1c\ub9e4\ud558\uc5ec \uc911\uad6d\ub0b4\uc218\uc2dc\uc7a5\uc5d0\uc11c \ud55c\ud654\ub85c \uc57d 35\ub9cc\uc6d0\uc758 \uac00\uaca9\uc5d0 \ud310\ub9e4\ud558\uace0 \uc788\ub2e4. \uae30\uae30\uc758 \uae30\ub2a5\uc801 \uce21\uba74\uc5d0\uc11c \ud55c\uacc4\uc810\uc73c\ub85c\ub294 \uac00\ub3d9\uac70\ub9ac\ub97c \uaf3d\uc744 \uc218 \uc788\ub2e4. \ud55c \ubc88 \ucda9\uc804\uc744 \ud558\uba74 \ucd5c\ub300 25~30km\uae4c\uc9c0\ubc16\uc5d0 \uc6b4\ud589\uc744 \ud560 \uc218 \uc5c6\uae30 \ub54c\ubb38\uc5d0 \uc774 \ubd80\ubd84\uc5d0\uc11c\ub3c4 \ud55c\uacc4\uc810\uc744 \uc9c0\ub2c8\uace0 \uc788\ub2e4.", "sentence_answer": "\uc774\uc5d0 \ub300\ud55c \ud0c0\uac1c\ucc45\uc73c\ub85c \uc138\uadf8\uc6e8\uc774\uc0ac\ub97c \uc778\uc218\ud55c \uc911\uad6d\uc758 \ub098\uc778\ubd07\uc0ac\uac00 2015\ub144 10\uc6d4 19\uc77c '\ub098\uc778\ubd07 \ubbf8\ub2c8' \uc774\ub77c\ub294 \uc138\uadf8\uc6e8\uc774\ub97c \uac1c\ub7c9\ud55c \uae30\uae30\ub97c \ubc1c\ub9e4\ud558\uc5ec \uc911\uad6d\ub0b4\uc218\uc2dc\uc7a5\uc5d0\uc11c \ud55c\ud654\ub85c \uc57d 35\ub9cc\uc6d0\uc758 \uac00\uaca9\uc5d0 \ud310\ub9e4\ud558\uace0 \uc788\ub2e4.", "paragraph_id": "6443282-4-1", "paragraph_question": "question: \uc911\uad6d\uc758 \ub098\uc778\ubd07\uc0ac\uac00 \uc138\uadf8\uc6e8\uc774\ub97c \uac1c\ub7c9\ud55c \uae30\uae30\ub294?, context: \ucd08\uae30 \uc138\uadf8\uc6e8\uc774\uc758 \uac00\uaca9\uc740 1000\ub9cc \uc6d0 \ub300\ub97c \ud6cc\uca4d \ub6f0\uc5b4\ub118\uc5c8\ub2e4. \uc77c\ubc18 \ub300\uc911\ub4e4\uc774 \ub808\uc838\uc6a9 \ud639\uc740 \uc77c\uc0c1\uc5d0\uc11c \uc0dd\ud65c\ud558\ub824\uace0 \uad6c\ub9e4\ud558\uae30\uc5d0\ub294 \ubd80\ub2f4\uc2a4\ub7f0\uc6b4 \uae08\uc561\uc774\uc5c8\ub2e4. 2015\ub144 \ud604\uc7ac\uc5d0 \uc774\ub974\uae30\uae4c\uc9c0 \uac00\uaca9\uc774 \uc810\uc810 \ud558\ub77d\ud558\uae30\ub294 \ud588\uc9c0\ub9cc \uc544\uc9c1\uae4c\uc9c0\ub294 \ub300\uc911\ub4e4\uc774 \ubc1b\uc544\ub4e4\uc774\uae30\uc5d0\ub294 \ube44\uc2fc \uac00\uaca9\uc744 \uc720\uc9c0\ud558\uace0 \uc788\ub294 \ud604\uc2e4\uc774\ub2e4. \uc774\uc5d0 \ub300\ud55c \ud0c0\uac1c\ucc45\uc73c\ub85c \uc138\uadf8\uc6e8\uc774\uc0ac\ub97c \uc778\uc218\ud55c \uc911\uad6d\uc758 \ub098\uc778\ubd07\uc0ac\uac00 2015\ub144 10\uc6d4 19\uc77c '\ub098\uc778\ubd07 \ubbf8\ub2c8'\uc774\ub77c\ub294 \uc138\uadf8\uc6e8\uc774\ub97c \uac1c\ub7c9\ud55c \uae30\uae30\ub97c \ubc1c\ub9e4\ud558\uc5ec \uc911\uad6d\ub0b4\uc218\uc2dc\uc7a5\uc5d0\uc11c \ud55c\ud654\ub85c \uc57d 35\ub9cc\uc6d0\uc758 \uac00\uaca9\uc5d0 \ud310\ub9e4\ud558\uace0 \uc788\ub2e4. \uae30\uae30\uc758 \uae30\ub2a5\uc801 \uce21\uba74\uc5d0\uc11c \ud55c\uacc4\uc810\uc73c\ub85c\ub294 \uac00\ub3d9\uac70\ub9ac\ub97c \uaf3d\uc744 \uc218 \uc788\ub2e4. \ud55c \ubc88 \ucda9\uc804\uc744 \ud558\uba74 \ucd5c\ub300 25~30km\uae4c\uc9c0\ubc16\uc5d0 \uc6b4\ud589\uc744 \ud560 \uc218 \uc5c6\uae30 \ub54c\ubb38\uc5d0 \uc774 \ubd80\ubd84\uc5d0\uc11c\ub3c4 \ud55c\uacc4\uc810\uc744 \uc9c0\ub2c8\uace0 \uc788\ub2e4."} {"question": "\ub354\ucf70\uc774\uc5c7\uc774 \ub124\ubc88\uc9f8 \uc815\uaddc \uc74c\ubc18\uc744 \ubc1c\ud45c\ud55c \ud574\ub294?", "paragraph": "2010\ub144 3\uc6d4, \ub124\ubc88\uc9f8 \uc815\uaddc \uc74c\ubc18 Quiet Storm: a Night Record\ub97c \ubc1c\ud45c \ud6c4 12\uc6d4, \uc18c\uc6b8 \ucef4\ud37c\ub2c8\ub97c \ud0c8\ud1f4. 2011\ub144 1\uc6d4 1\uc77c, \ub3c4\ub07c\uc640 \ud568\uaed8 \uc77c\ub9ac\ub124\uc5b4 \ub808\ucf54\uc988\ub97c \uc124\ub9bd\ud558\uc600\uace0 \uc124\ub9bd \ud6c4, 2011\ub144 Stormy Friday EP, 2013\ub144 AMBITIQN\ub97c \ubc1c\ub9e4\ud558\uc600\ub2e4. 2014\ub144 \uc77c\ub9ac\ub124\uc5b4 \ub808\ucf54\uc988\uc758 \ucef4\ud544\ub808\uc774\uc158 \uc74c\ubc18 11:11\uc744 \ubc1c\ub9e4\ud558\uc600\uc73c\uba70 \uc5e0\ub137 \u300aShow Me The Money 3\u300b\uc5d0 \ud504\ub85c\ub4c0\uc11c\ub85c \ucd9c\uc5f0\ud558\uc600\ub2e4. 2015\ub144 10\uc6d4\uc5d0\ub294 \ub2e4\uc12f\ubc88\uc9f8 \uc815\uaddc \uc74c\ubc18 1 Life 2 Live\ub97c \ubc1c\ub9e4\ud558\uc600\uace0, 2016\ub144\uc5d0\ub294 \uc5e0\ub137 \u300aShow Me The Money 5\u300b\uc5d0 \ud504\ub85c\ub4c0\uc11c\ub85c \ucd9c\uc5f0\ud558\uc600\ub2e4. \uadf8 \uc774\ud6c4 \"still got luv\"\uc640 \"Mission\", \"Beautiful life II\"\ub97c \uc8fc\ub9c8\ub2e4 \ubc1c\ub9e4\ud558\uace0 \uae40\ud6a8\uc740, \ud574\uc26c\uc2a4\uc644, \ucc3d\ubaa8\ub85c \uad6c\uc131\ub41c \"Ambition Music\"\uc744 \uc124\ub9bd\ud558\uc600\ub2e4.", "answer": "2010\ub144", "sentence": "2010\ub144 3\uc6d4, \ub124\ubc88\uc9f8 \uc815\uaddc \uc74c\ubc18 Quiet Storm: a Night Record\ub97c \ubc1c\ud45c \ud6c4 12\uc6d4, \uc18c\uc6b8 \ucef4\ud37c\ub2c8\ub97c \ud0c8\ud1f4.", "paragraph_sentence": " 2010\ub144 3\uc6d4, \ub124\ubc88\uc9f8 \uc815\uaddc \uc74c\ubc18 Quiet Storm: a Night Record\ub97c \ubc1c\ud45c \ud6c4 12\uc6d4, \uc18c\uc6b8 \ucef4\ud37c\ub2c8\ub97c \ud0c8\ud1f4. 2011\ub144 1\uc6d4 1\uc77c, \ub3c4\ub07c\uc640 \ud568\uaed8 \uc77c\ub9ac\ub124\uc5b4 \ub808\ucf54\uc988\ub97c \uc124\ub9bd\ud558\uc600\uace0 \uc124\ub9bd \ud6c4, 2011\ub144 Stormy Friday EP, 2013\ub144 AMBITIQN\ub97c \ubc1c\ub9e4\ud558\uc600\ub2e4. 2014\ub144 \uc77c\ub9ac\ub124\uc5b4 \ub808\ucf54\uc988\uc758 \ucef4\ud544\ub808\uc774\uc158 \uc74c\ubc18 11:11\uc744 \ubc1c\ub9e4\ud558\uc600\uc73c\uba70 \uc5e0\ub137 \u300aShow Me The Money 3\u300b\uc5d0 \ud504\ub85c\ub4c0\uc11c\ub85c \ucd9c\uc5f0\ud558\uc600\ub2e4. 2015\ub144 10\uc6d4\uc5d0\ub294 \ub2e4\uc12f\ubc88\uc9f8 \uc815\uaddc \uc74c\ubc18 1 Life 2 Live\ub97c \ubc1c\ub9e4\ud558\uc600\uace0, 2016\ub144\uc5d0\ub294 \uc5e0\ub137 \u300aShow Me The Money 5\u300b\uc5d0 \ud504\ub85c\ub4c0\uc11c\ub85c \ucd9c\uc5f0\ud558\uc600\ub2e4. \uadf8 \uc774\ud6c4 \"still got luv\"\uc640 \"Mission\", \"Beautiful life II\"\ub97c \uc8fc\ub9c8\ub2e4 \ubc1c\ub9e4\ud558\uace0 \uae40\ud6a8\uc740, \ud574\uc26c\uc2a4\uc644, \ucc3d\ubaa8\ub85c \uad6c\uc131\ub41c \"Ambition Music\"\uc744 \uc124\ub9bd\ud558\uc600\ub2e4.", "paragraph_answer": " 2010\ub144 3\uc6d4, \ub124\ubc88\uc9f8 \uc815\uaddc \uc74c\ubc18 Quiet Storm: a Night Record\ub97c \ubc1c\ud45c \ud6c4 12\uc6d4, \uc18c\uc6b8 \ucef4\ud37c\ub2c8\ub97c \ud0c8\ud1f4. 2011\ub144 1\uc6d4 1\uc77c, \ub3c4\ub07c\uc640 \ud568\uaed8 \uc77c\ub9ac\ub124\uc5b4 \ub808\ucf54\uc988\ub97c \uc124\ub9bd\ud558\uc600\uace0 \uc124\ub9bd \ud6c4, 2011\ub144 Stormy Friday EP, 2013\ub144 AMBITIQN\ub97c \ubc1c\ub9e4\ud558\uc600\ub2e4. 2014\ub144 \uc77c\ub9ac\ub124\uc5b4 \ub808\ucf54\uc988\uc758 \ucef4\ud544\ub808\uc774\uc158 \uc74c\ubc18 11:11\uc744 \ubc1c\ub9e4\ud558\uc600\uc73c\uba70 \uc5e0\ub137 \u300aShow Me The Money 3\u300b\uc5d0 \ud504\ub85c\ub4c0\uc11c\ub85c \ucd9c\uc5f0\ud558\uc600\ub2e4. 2015\ub144 10\uc6d4\uc5d0\ub294 \ub2e4\uc12f\ubc88\uc9f8 \uc815\uaddc \uc74c\ubc18 1 Life 2 Live\ub97c \ubc1c\ub9e4\ud558\uc600\uace0, 2016\ub144\uc5d0\ub294 \uc5e0\ub137 \u300aShow Me The Money 5\u300b\uc5d0 \ud504\ub85c\ub4c0\uc11c\ub85c \ucd9c\uc5f0\ud558\uc600\ub2e4. \uadf8 \uc774\ud6c4 \"still got luv\"\uc640 \"Mission\", \"Beautiful life II\"\ub97c \uc8fc\ub9c8\ub2e4 \ubc1c\ub9e4\ud558\uace0 \uae40\ud6a8\uc740, \ud574\uc26c\uc2a4\uc644, \ucc3d\ubaa8\ub85c \uad6c\uc131\ub41c \"Ambition Music\"\uc744 \uc124\ub9bd\ud558\uc600\ub2e4.", "sentence_answer": " 2010\ub144 3\uc6d4, \ub124\ubc88\uc9f8 \uc815\uaddc \uc74c\ubc18 Quiet Storm: a Night Record\ub97c \ubc1c\ud45c \ud6c4 12\uc6d4, \uc18c\uc6b8 \ucef4\ud37c\ub2c8\ub97c \ud0c8\ud1f4.", "paragraph_id": "6578232-0-0", "paragraph_question": "question: \ub354\ucf70\uc774\uc5c7\uc774 \ub124\ubc88\uc9f8 \uc815\uaddc \uc74c\ubc18\uc744 \ubc1c\ud45c\ud55c \ud574\ub294?, context: 2010\ub144 3\uc6d4, \ub124\ubc88\uc9f8 \uc815\uaddc \uc74c\ubc18 Quiet Storm: a Night Record\ub97c \ubc1c\ud45c \ud6c4 12\uc6d4, \uc18c\uc6b8 \ucef4\ud37c\ub2c8\ub97c \ud0c8\ud1f4. 2011\ub144 1\uc6d4 1\uc77c, \ub3c4\ub07c\uc640 \ud568\uaed8 \uc77c\ub9ac\ub124\uc5b4 \ub808\ucf54\uc988\ub97c \uc124\ub9bd\ud558\uc600\uace0 \uc124\ub9bd \ud6c4, 2011\ub144 Stormy Friday EP, 2013\ub144 AMBITIQN\ub97c \ubc1c\ub9e4\ud558\uc600\ub2e4. 2014\ub144 \uc77c\ub9ac\ub124\uc5b4 \ub808\ucf54\uc988\uc758 \ucef4\ud544\ub808\uc774\uc158 \uc74c\ubc18 11:11\uc744 \ubc1c\ub9e4\ud558\uc600\uc73c\uba70 \uc5e0\ub137 \u300aShow Me The Money 3\u300b\uc5d0 \ud504\ub85c\ub4c0\uc11c\ub85c \ucd9c\uc5f0\ud558\uc600\ub2e4. 2015\ub144 10\uc6d4\uc5d0\ub294 \ub2e4\uc12f\ubc88\uc9f8 \uc815\uaddc \uc74c\ubc18 1 Life 2 Live\ub97c \ubc1c\ub9e4\ud558\uc600\uace0, 2016\ub144\uc5d0\ub294 \uc5e0\ub137 \u300aShow Me The Money 5\u300b\uc5d0 \ud504\ub85c\ub4c0\uc11c\ub85c \ucd9c\uc5f0\ud558\uc600\ub2e4. \uadf8 \uc774\ud6c4 \"still got luv\"\uc640 \"Mission\", \"Beautiful life II\"\ub97c \uc8fc\ub9c8\ub2e4 \ubc1c\ub9e4\ud558\uace0 \uae40\ud6a8\uc740, \ud574\uc26c\uc2a4\uc644, \ucc3d\ubaa8\ub85c \uad6c\uc131\ub41c \"Ambition Music\"\uc744 \uc124\ub9bd\ud558\uc600\ub2e4."} {"question": "2010\ub144 FIFA \uc6d4\ub4dc\ucef5\uc758 \uacb0\uc2b9\uc804\uc5d0\uc11c 14\uc7a5\uc758 \uc610\ub85c\uce74\ub4dc\ub97c \uaebc\ub0b8 \uc2ec\ud310\uc740 \ub204\uad6c\uc778\uac00?", "paragraph": "2010\ub144 FIFA \uc6d4\ub4dc\ucef5\uc5d0\uc11c \uc6f9\uc740 \uc774\uc804\uae4c\uc9c0 3\uacbd\uae30\ub97c \uc8fc\uad00\ud588\uace0, \ubaa8\ub450 \ubb3c\ub77c\ud0a4\uc640 \uce78\uc774 \ubcf4\uc870\ud588\ub2e4. \uc870\ubcc4 \ub9ac\uadf8\uc5d0\uc11c \uadf8\ub294 \uc2a4\ud398\uc778-\uc2a4\uc704\uc2a4 \uacbd\uae30\uc640 \uc2ac\ub85c\ubc14\ud0a4\uc544-\uc774\ud0c8\ub9ac\uc544 \uacbd\uae30\ub97c \uc8fc\uad00\ud558\uc600\uace0, 16\uac15\uc5d0\ub294 \ube0c\ub77c\uc9c8-\uce60\ub808 \uacbd\uae30\ub97c \uc911\uc7ac\ud588\ub2e4. \uc774 \uc138\ubc88\uc758 \uacbd\uae30\uc5d0\uc11c \ub2e8 \ud55c\ubc88\ub3c4 \ub808\ub4dc\uce74\ub4dc\ub97c \uaebc\ub0b4\uac70\ub098 \ud398\ub110\ud2f0\ud0a5\uc744 \uc120\uc5b8\ud558\uc9c0 \uc54a\uc558\uc9c0\ub9cc, \ud3c9\uade0 5.67\uc7a5\uc758 \uc610\ub85c\uce74\ub4dc\ub97c \uaebc\ub0b4 \uc774 \ubd80\ubb38\uc5d0\uc11c \uc218\uce58\uac00 \ub450\ubc88\uc9f8\ub85c \ub192\uc558\ub2e4. \uacb0\uc2b9\uc804\uc5d0\uc11c 14\uc7a5\uc758 \uc610\ub85c \uce74\ub4dc (2\uc7a5\uc758 \uc610\ub85c\uce74\ub4dc\ub97c \ubc1b\uc740 \uc698 \ud5e4\uc774\ud305\uc544\uc5d0\uac8c\ub294 \ub808\ub4dc\uce74\ub4dc\uac00 \ub098\uc654\ub2e4.) \ub97c \uaebc\ub0b4\uba74\uc11c, \uadf8\ub294 \uc885\uc804\uc5d0 1986\ub144 \ub300\ud68c \uacb0\uc2b9\uc804\uc5d0 \uc138\uc6b4 FIFA \uc6d4\ub4dc\ucef5 \uacb0\uc2b9\uc804\uc758 \ucd5c\ub2e4 \uacbd\uace0 \uae30\ub85d\uc778 6\uc7a5\uc744 \uac00\ubccd\uac8c \ub118\uc5b4\uc130\ub2e4. \uc774\uc911 9\uc7a5\uc774 90\ubd84 \uc815\uaddc\uc2dc\uac04 \ub0b4\uc5d0 \ub098\uc654\ub2e4. \uc6f9\uc740 \uc774 \ub300\ud68c\uc5d0\uc11c \ucd1d 31\uc7a5\uc758 \uc610\ub85c\uce74\ub4dc\ub97c \uaebc\ub0b4 \uacbd\uae30\ub2f9 \ud3c9\uade0 7.75\uc7a5\uc744 \uaebc\ub0b8\uac8c \ub418\uc5c8\ub2e4.", "answer": "\uc6f9", "sentence": "2010\ub144 FIFA \uc6d4\ub4dc\ucef5\uc5d0\uc11c \uc6f9 \uc740 \uc774\uc804\uae4c\uc9c0 3\uacbd\uae30\ub97c \uc8fc\uad00\ud588\uace0, \ubaa8\ub450 \ubb3c\ub77c\ud0a4\uc640 \uce78\uc774 \ubcf4\uc870\ud588\ub2e4.", "paragraph_sentence": " 2010\ub144 FIFA \uc6d4\ub4dc\ucef5\uc5d0\uc11c \uc6f9 \uc740 \uc774\uc804\uae4c\uc9c0 3\uacbd\uae30\ub97c \uc8fc\uad00\ud588\uace0, \ubaa8\ub450 \ubb3c\ub77c\ud0a4\uc640 \uce78\uc774 \ubcf4\uc870\ud588\ub2e4. \uc870\ubcc4 \ub9ac\uadf8\uc5d0\uc11c \uadf8\ub294 \uc2a4\ud398\uc778-\uc2a4\uc704\uc2a4 \uacbd\uae30\uc640 \uc2ac\ub85c\ubc14\ud0a4\uc544-\uc774\ud0c8\ub9ac\uc544 \uacbd\uae30\ub97c \uc8fc\uad00\ud558\uc600\uace0, 16\uac15\uc5d0\ub294 \ube0c\ub77c\uc9c8-\uce60\ub808 \uacbd\uae30\ub97c \uc911\uc7ac\ud588\ub2e4. \uc774 \uc138\ubc88\uc758 \uacbd\uae30\uc5d0\uc11c \ub2e8 \ud55c\ubc88\ub3c4 \ub808\ub4dc\uce74\ub4dc\ub97c \uaebc\ub0b4\uac70\ub098 \ud398\ub110\ud2f0\ud0a5\uc744 \uc120\uc5b8\ud558\uc9c0 \uc54a\uc558\uc9c0\ub9cc, \ud3c9\uade0 5.67\uc7a5\uc758 \uc610\ub85c\uce74\ub4dc\ub97c \uaebc\ub0b4 \uc774 \ubd80\ubb38\uc5d0\uc11c \uc218\uce58\uac00 \ub450\ubc88\uc9f8\ub85c \ub192\uc558\ub2e4. \uacb0\uc2b9\uc804\uc5d0\uc11c 14\uc7a5\uc758 \uc610\ub85c \uce74\ub4dc (2\uc7a5\uc758 \uc610\ub85c\uce74\ub4dc\ub97c \ubc1b\uc740 \uc698 \ud5e4\uc774\ud305\uc544\uc5d0\uac8c\ub294 \ub808\ub4dc\uce74\ub4dc\uac00 \ub098\uc654\ub2e4. ) \ub97c \uaebc\ub0b4\uba74\uc11c, \uadf8\ub294 \uc885\uc804\uc5d0 1986\ub144 \ub300\ud68c \uacb0\uc2b9\uc804\uc5d0 \uc138\uc6b4 FIFA \uc6d4\ub4dc\ucef5 \uacb0\uc2b9\uc804\uc758 \ucd5c\ub2e4 \uacbd\uace0 \uae30\ub85d\uc778 6\uc7a5\uc744 \uac00\ubccd\uac8c \ub118\uc5b4\uc130\ub2e4. \uc774\uc911 9\uc7a5\uc774 90\ubd84 \uc815\uaddc\uc2dc\uac04 \ub0b4\uc5d0 \ub098\uc654\ub2e4. \uc6f9\uc740 \uc774 \ub300\ud68c\uc5d0\uc11c \ucd1d 31\uc7a5\uc758 \uc610\ub85c\uce74\ub4dc\ub97c \uaebc\ub0b4 \uacbd\uae30\ub2f9 \ud3c9\uade0 7.75\uc7a5\uc744 \uaebc\ub0b8\uac8c \ub418\uc5c8\ub2e4.", "paragraph_answer": "2010\ub144 FIFA \uc6d4\ub4dc\ucef5\uc5d0\uc11c \uc6f9 \uc740 \uc774\uc804\uae4c\uc9c0 3\uacbd\uae30\ub97c \uc8fc\uad00\ud588\uace0, \ubaa8\ub450 \ubb3c\ub77c\ud0a4\uc640 \uce78\uc774 \ubcf4\uc870\ud588\ub2e4. \uc870\ubcc4 \ub9ac\uadf8\uc5d0\uc11c \uadf8\ub294 \uc2a4\ud398\uc778-\uc2a4\uc704\uc2a4 \uacbd\uae30\uc640 \uc2ac\ub85c\ubc14\ud0a4\uc544-\uc774\ud0c8\ub9ac\uc544 \uacbd\uae30\ub97c \uc8fc\uad00\ud558\uc600\uace0, 16\uac15\uc5d0\ub294 \ube0c\ub77c\uc9c8-\uce60\ub808 \uacbd\uae30\ub97c \uc911\uc7ac\ud588\ub2e4. \uc774 \uc138\ubc88\uc758 \uacbd\uae30\uc5d0\uc11c \ub2e8 \ud55c\ubc88\ub3c4 \ub808\ub4dc\uce74\ub4dc\ub97c \uaebc\ub0b4\uac70\ub098 \ud398\ub110\ud2f0\ud0a5\uc744 \uc120\uc5b8\ud558\uc9c0 \uc54a\uc558\uc9c0\ub9cc, \ud3c9\uade0 5.67\uc7a5\uc758 \uc610\ub85c\uce74\ub4dc\ub97c \uaebc\ub0b4 \uc774 \ubd80\ubb38\uc5d0\uc11c \uc218\uce58\uac00 \ub450\ubc88\uc9f8\ub85c \ub192\uc558\ub2e4. \uacb0\uc2b9\uc804\uc5d0\uc11c 14\uc7a5\uc758 \uc610\ub85c \uce74\ub4dc (2\uc7a5\uc758 \uc610\ub85c\uce74\ub4dc\ub97c \ubc1b\uc740 \uc698 \ud5e4\uc774\ud305\uc544\uc5d0\uac8c\ub294 \ub808\ub4dc\uce74\ub4dc\uac00 \ub098\uc654\ub2e4.) \ub97c \uaebc\ub0b4\uba74\uc11c, \uadf8\ub294 \uc885\uc804\uc5d0 1986\ub144 \ub300\ud68c \uacb0\uc2b9\uc804\uc5d0 \uc138\uc6b4 FIFA \uc6d4\ub4dc\ucef5 \uacb0\uc2b9\uc804\uc758 \ucd5c\ub2e4 \uacbd\uace0 \uae30\ub85d\uc778 6\uc7a5\uc744 \uac00\ubccd\uac8c \ub118\uc5b4\uc130\ub2e4. \uc774\uc911 9\uc7a5\uc774 90\ubd84 \uc815\uaddc\uc2dc\uac04 \ub0b4\uc5d0 \ub098\uc654\ub2e4. \uc6f9\uc740 \uc774 \ub300\ud68c\uc5d0\uc11c \ucd1d 31\uc7a5\uc758 \uc610\ub85c\uce74\ub4dc\ub97c \uaebc\ub0b4 \uacbd\uae30\ub2f9 \ud3c9\uade0 7.75\uc7a5\uc744 \uaebc\ub0b8\uac8c \ub418\uc5c8\ub2e4.", "sentence_answer": "2010\ub144 FIFA \uc6d4\ub4dc\ucef5\uc5d0\uc11c \uc6f9 \uc740 \uc774\uc804\uae4c\uc9c0 3\uacbd\uae30\ub97c \uc8fc\uad00\ud588\uace0, \ubaa8\ub450 \ubb3c\ub77c\ud0a4\uc640 \uce78\uc774 \ubcf4\uc870\ud588\ub2e4.", "paragraph_id": "6533357-1-0", "paragraph_question": "question: 2010\ub144 FIFA \uc6d4\ub4dc\ucef5\uc758 \uacb0\uc2b9\uc804\uc5d0\uc11c 14\uc7a5\uc758 \uc610\ub85c\uce74\ub4dc\ub97c \uaebc\ub0b8 \uc2ec\ud310\uc740 \ub204\uad6c\uc778\uac00?, context: 2010\ub144 FIFA \uc6d4\ub4dc\ucef5\uc5d0\uc11c \uc6f9\uc740 \uc774\uc804\uae4c\uc9c0 3\uacbd\uae30\ub97c \uc8fc\uad00\ud588\uace0, \ubaa8\ub450 \ubb3c\ub77c\ud0a4\uc640 \uce78\uc774 \ubcf4\uc870\ud588\ub2e4. \uc870\ubcc4 \ub9ac\uadf8\uc5d0\uc11c \uadf8\ub294 \uc2a4\ud398\uc778-\uc2a4\uc704\uc2a4 \uacbd\uae30\uc640 \uc2ac\ub85c\ubc14\ud0a4\uc544-\uc774\ud0c8\ub9ac\uc544 \uacbd\uae30\ub97c \uc8fc\uad00\ud558\uc600\uace0, 16\uac15\uc5d0\ub294 \ube0c\ub77c\uc9c8-\uce60\ub808 \uacbd\uae30\ub97c \uc911\uc7ac\ud588\ub2e4. \uc774 \uc138\ubc88\uc758 \uacbd\uae30\uc5d0\uc11c \ub2e8 \ud55c\ubc88\ub3c4 \ub808\ub4dc\uce74\ub4dc\ub97c \uaebc\ub0b4\uac70\ub098 \ud398\ub110\ud2f0\ud0a5\uc744 \uc120\uc5b8\ud558\uc9c0 \uc54a\uc558\uc9c0\ub9cc, \ud3c9\uade0 5.67\uc7a5\uc758 \uc610\ub85c\uce74\ub4dc\ub97c \uaebc\ub0b4 \uc774 \ubd80\ubb38\uc5d0\uc11c \uc218\uce58\uac00 \ub450\ubc88\uc9f8\ub85c \ub192\uc558\ub2e4. \uacb0\uc2b9\uc804\uc5d0\uc11c 14\uc7a5\uc758 \uc610\ub85c \uce74\ub4dc (2\uc7a5\uc758 \uc610\ub85c\uce74\ub4dc\ub97c \ubc1b\uc740 \uc698 \ud5e4\uc774\ud305\uc544\uc5d0\uac8c\ub294 \ub808\ub4dc\uce74\ub4dc\uac00 \ub098\uc654\ub2e4.) \ub97c \uaebc\ub0b4\uba74\uc11c, \uadf8\ub294 \uc885\uc804\uc5d0 1986\ub144 \ub300\ud68c \uacb0\uc2b9\uc804\uc5d0 \uc138\uc6b4 FIFA \uc6d4\ub4dc\ucef5 \uacb0\uc2b9\uc804\uc758 \ucd5c\ub2e4 \uacbd\uace0 \uae30\ub85d\uc778 6\uc7a5\uc744 \uac00\ubccd\uac8c \ub118\uc5b4\uc130\ub2e4. \uc774\uc911 9\uc7a5\uc774 90\ubd84 \uc815\uaddc\uc2dc\uac04 \ub0b4\uc5d0 \ub098\uc654\ub2e4. \uc6f9\uc740 \uc774 \ub300\ud68c\uc5d0\uc11c \ucd1d 31\uc7a5\uc758 \uc610\ub85c\uce74\ub4dc\ub97c \uaebc\ub0b4 \uacbd\uae30\ub2f9 \ud3c9\uade0 7.75\uc7a5\uc744 \uaebc\ub0b8\uac8c \ub418\uc5c8\ub2e4."} {"question": "\ubc30\ub9ac\uc5b4 \ud504\ub9ac\ub97c \uc704\ud574 2\ud638\ucc28\uc5d0 \ud0d1\uc7ac\ud55c \uac83\uc740?", "paragraph": "\uc11c\uc6b8-\uc2e0\ucc3d \uac04 \ub178\uc120\uc758 \uacbd\uc6b0, \uc218\ub3c4\uad8c \uc804\ucca0 \uad6c\uac04\uc5d0\uc11c \uc9d1\uc911\uc801\uc73c\ub85c \uc6b4\uc6a9\ub418\uae30 \ub54c\ubb38\uc5d0, \uc800\uc0c1 \uc2b9\uac15\uc7a5\uacfc \uace0\uc0c1 \uc2b9\uac15\uc7a5 \ubaa8\ub450\uc5d0\uc11c \uc2b9\ud558\ucc28\uac00 \uac00\ub2a5\ud558\ub3c4\ub85d \uc720\uc555\uc2dd \ub9ac\ud504\ud2b8 \uc7a5\uce58\ub97c \ud0d1\uc7ac\ud558\uc600\uace0, \ubc30\ub9ac\uc5b4 \ud504\ub9ac\ub97c \uc704\ud574 2\ud638\ucc28\uc5d0\ub294 \uc7a5\uc560\uc778 \ud0d1\uc2b9\uc6a9 \uacbd\uc0ac\ub85c \uc7a5\uce58\ub97c \ud0d1\uc7ac\ud558\uc600\ub2e4. \uc2e0\ucc3d\uc5ed\uacfc \uc11c\uc6b8\uc5ed\uc744 \uc81c\uc678\ud55c \ubaa8\ub4e0 \uc5ed\uc5d0\uc11c \uc800\uc0c1 \uc2b9\uac15\uc7a5\uc744 \uc774\uc6a9\ud558\ub098 \ubaa8\ub4e0 \uc5f4\ucc28\uac00 \uc11c\uc6b8\uc5ed\uc5d0\uc11c \uace0\uc0c1 \uc2b9\uac15\uc7a5\ub9cc\uc744 \uc0ac\uc6a9\ud558\ub294 \uac83\uc740 \uc544\ub2c8\ub2e4. \uc2b9\uac15\uad6c\uc758 \ucc28\ub0b4 \ubd80\ubd84\uc5d0 \ub178\ub780\uc0c9\uc73c\ub85c \ud45c\uc2dc\ub41c \ubd80\ubd84 \uc548\ucabd\uc5d0\ub294 \uc800\uc0c1 \uc2b9\uac15\uc7a5 \uc815\ucc28 \uc2dc \uacc4\ub2e8\uc774 \ub0b4\ub824\uac00\ub3c4\ub85d \ub41c \ubd80\ubd84\uc73c\ub85c, \uc774 \ubd80\ubd84\uc5d0\ub294 \uc13c\uc11c\uac00 \uc788\uc5b4\uc11c \uc800\uc0c1 \uc2b9\uac15\uc7a5 \uc815\ucc28 \uc2dc \uc7a5\uc560\ubb3c\uc774 \uc788\uc744 \uacbd\uc6b0 \ub0b4\ub824\uac00\uc9c0 \uc54a\uc73c\uba70, \ubc18\ub300\ub85c \ubb38\uc744 \ub2eb\uc744 \ub54c\uc5d0\ub3c4 \uc2b9\uac1d \ub4f1\uc774 \uc11c \uc788\uc744 \uacbd\uc6b0 \uacc4\ub2e8\uc744 \uc62c\ub9ac\uace0 \ubb38\uc744 \ub2eb\uc744 \uc218 \uc5c6\ub2e4. \ud55c\ud3b8, \uc77c\ubc18\uc801\uc778 \ud1b5\uadfc\ud615 \uc804\ub3d9\ucc28\uc640 \ucc28\ub7c9 \uae38\uc774\uac00 \ub2e4\ub974\uace0, \uc2a4\ud06c\ub9b0\ub3c4\uc5b4\uc640 \ubb38\uc758 \uc704\uce58\uac00 \ub2ec\ub77c\uc11c \uc2a4\ud06c\ub9b0\ub3c4\uc5b4\uac00 \uc124\uce58\ub41c \uc2b9\uac15\uc7a5\uc5d0\uc11c\ub294 \uc5ec\uac1d \ucde8\uae09\uc774 \ubd88\uac00\ub2a5\ud558\ub2e4.", "answer": "\uc7a5\uc560\uc778 \ud0d1\uc2b9\uc6a9 \uacbd\uc0ac\ub85c \uc7a5\uce58", "sentence": "\uc11c\uc6b8-\uc2e0\ucc3d \uac04 \ub178\uc120\uc758 \uacbd\uc6b0, \uc218\ub3c4\uad8c \uc804\ucca0 \uad6c\uac04\uc5d0\uc11c \uc9d1\uc911\uc801\uc73c\ub85c \uc6b4\uc6a9\ub418\uae30 \ub54c\ubb38\uc5d0, \uc800\uc0c1 \uc2b9\uac15\uc7a5\uacfc \uace0\uc0c1 \uc2b9\uac15\uc7a5 \ubaa8\ub450\uc5d0\uc11c \uc2b9\ud558\ucc28\uac00 \uac00\ub2a5\ud558\ub3c4\ub85d \uc720\uc555\uc2dd \ub9ac\ud504\ud2b8 \uc7a5\uce58\ub97c \ud0d1\uc7ac\ud558\uc600\uace0, \ubc30\ub9ac\uc5b4 \ud504\ub9ac\ub97c \uc704\ud574 2\ud638\ucc28\uc5d0\ub294 \uc7a5\uc560\uc778 \ud0d1\uc2b9\uc6a9 \uacbd\uc0ac\ub85c \uc7a5\uce58 \ub97c \ud0d1\uc7ac\ud558\uc600\ub2e4.", "paragraph_sentence": " \uc11c\uc6b8-\uc2e0\ucc3d \uac04 \ub178\uc120\uc758 \uacbd\uc6b0, \uc218\ub3c4\uad8c \uc804\ucca0 \uad6c\uac04\uc5d0\uc11c \uc9d1\uc911\uc801\uc73c\ub85c \uc6b4\uc6a9\ub418\uae30 \ub54c\ubb38\uc5d0, \uc800\uc0c1 \uc2b9\uac15\uc7a5\uacfc \uace0\uc0c1 \uc2b9\uac15\uc7a5 \ubaa8\ub450\uc5d0\uc11c \uc2b9\ud558\ucc28\uac00 \uac00\ub2a5\ud558\ub3c4\ub85d \uc720\uc555\uc2dd \ub9ac\ud504\ud2b8 \uc7a5\uce58\ub97c \ud0d1\uc7ac\ud558\uc600\uace0, \ubc30\ub9ac\uc5b4 \ud504\ub9ac\ub97c \uc704\ud574 2\ud638\ucc28\uc5d0\ub294 \uc7a5\uc560\uc778 \ud0d1\uc2b9\uc6a9 \uacbd\uc0ac\ub85c \uc7a5\uce58 \ub97c \ud0d1\uc7ac\ud558\uc600\ub2e4. \uc2e0\ucc3d\uc5ed\uacfc \uc11c\uc6b8\uc5ed\uc744 \uc81c\uc678\ud55c \ubaa8\ub4e0 \uc5ed\uc5d0\uc11c \uc800\uc0c1 \uc2b9\uac15\uc7a5\uc744 \uc774\uc6a9\ud558\ub098 \ubaa8\ub4e0 \uc5f4\ucc28\uac00 \uc11c\uc6b8\uc5ed\uc5d0\uc11c \uace0\uc0c1 \uc2b9\uac15\uc7a5\ub9cc\uc744 \uc0ac\uc6a9\ud558\ub294 \uac83\uc740 \uc544\ub2c8\ub2e4. \uc2b9\uac15\uad6c\uc758 \ucc28\ub0b4 \ubd80\ubd84\uc5d0 \ub178\ub780\uc0c9\uc73c\ub85c \ud45c\uc2dc\ub41c \ubd80\ubd84 \uc548\ucabd\uc5d0\ub294 \uc800\uc0c1 \uc2b9\uac15\uc7a5 \uc815\ucc28 \uc2dc \uacc4\ub2e8\uc774 \ub0b4\ub824\uac00\ub3c4\ub85d \ub41c \ubd80\ubd84\uc73c\ub85c, \uc774 \ubd80\ubd84\uc5d0\ub294 \uc13c\uc11c\uac00 \uc788\uc5b4\uc11c \uc800\uc0c1 \uc2b9\uac15\uc7a5 \uc815\ucc28 \uc2dc \uc7a5\uc560\ubb3c\uc774 \uc788\uc744 \uacbd\uc6b0 \ub0b4\ub824\uac00\uc9c0 \uc54a\uc73c\uba70, \ubc18\ub300\ub85c \ubb38\uc744 \ub2eb\uc744 \ub54c\uc5d0\ub3c4 \uc2b9\uac1d \ub4f1\uc774 \uc11c \uc788\uc744 \uacbd\uc6b0 \uacc4\ub2e8\uc744 \uc62c\ub9ac\uace0 \ubb38\uc744 \ub2eb\uc744 \uc218 \uc5c6\ub2e4. \ud55c\ud3b8, \uc77c\ubc18\uc801\uc778 \ud1b5\uadfc\ud615 \uc804\ub3d9\ucc28\uc640 \ucc28\ub7c9 \uae38\uc774\uac00 \ub2e4\ub974\uace0, \uc2a4\ud06c\ub9b0\ub3c4\uc5b4\uc640 \ubb38\uc758 \uc704\uce58\uac00 \ub2ec\ub77c\uc11c \uc2a4\ud06c\ub9b0\ub3c4\uc5b4\uac00 \uc124\uce58\ub41c \uc2b9\uac15\uc7a5\uc5d0\uc11c\ub294 \uc5ec\uac1d \ucde8\uae09\uc774 \ubd88\uac00\ub2a5\ud558\ub2e4.", "paragraph_answer": "\uc11c\uc6b8-\uc2e0\ucc3d \uac04 \ub178\uc120\uc758 \uacbd\uc6b0, \uc218\ub3c4\uad8c \uc804\ucca0 \uad6c\uac04\uc5d0\uc11c \uc9d1\uc911\uc801\uc73c\ub85c \uc6b4\uc6a9\ub418\uae30 \ub54c\ubb38\uc5d0, \uc800\uc0c1 \uc2b9\uac15\uc7a5\uacfc \uace0\uc0c1 \uc2b9\uac15\uc7a5 \ubaa8\ub450\uc5d0\uc11c \uc2b9\ud558\ucc28\uac00 \uac00\ub2a5\ud558\ub3c4\ub85d \uc720\uc555\uc2dd \ub9ac\ud504\ud2b8 \uc7a5\uce58\ub97c \ud0d1\uc7ac\ud558\uc600\uace0, \ubc30\ub9ac\uc5b4 \ud504\ub9ac\ub97c \uc704\ud574 2\ud638\ucc28\uc5d0\ub294 \uc7a5\uc560\uc778 \ud0d1\uc2b9\uc6a9 \uacbd\uc0ac\ub85c \uc7a5\uce58 \ub97c \ud0d1\uc7ac\ud558\uc600\ub2e4. \uc2e0\ucc3d\uc5ed\uacfc \uc11c\uc6b8\uc5ed\uc744 \uc81c\uc678\ud55c \ubaa8\ub4e0 \uc5ed\uc5d0\uc11c \uc800\uc0c1 \uc2b9\uac15\uc7a5\uc744 \uc774\uc6a9\ud558\ub098 \ubaa8\ub4e0 \uc5f4\ucc28\uac00 \uc11c\uc6b8\uc5ed\uc5d0\uc11c \uace0\uc0c1 \uc2b9\uac15\uc7a5\ub9cc\uc744 \uc0ac\uc6a9\ud558\ub294 \uac83\uc740 \uc544\ub2c8\ub2e4. \uc2b9\uac15\uad6c\uc758 \ucc28\ub0b4 \ubd80\ubd84\uc5d0 \ub178\ub780\uc0c9\uc73c\ub85c \ud45c\uc2dc\ub41c \ubd80\ubd84 \uc548\ucabd\uc5d0\ub294 \uc800\uc0c1 \uc2b9\uac15\uc7a5 \uc815\ucc28 \uc2dc \uacc4\ub2e8\uc774 \ub0b4\ub824\uac00\ub3c4\ub85d \ub41c \ubd80\ubd84\uc73c\ub85c, \uc774 \ubd80\ubd84\uc5d0\ub294 \uc13c\uc11c\uac00 \uc788\uc5b4\uc11c \uc800\uc0c1 \uc2b9\uac15\uc7a5 \uc815\ucc28 \uc2dc \uc7a5\uc560\ubb3c\uc774 \uc788\uc744 \uacbd\uc6b0 \ub0b4\ub824\uac00\uc9c0 \uc54a\uc73c\uba70, \ubc18\ub300\ub85c \ubb38\uc744 \ub2eb\uc744 \ub54c\uc5d0\ub3c4 \uc2b9\uac1d \ub4f1\uc774 \uc11c \uc788\uc744 \uacbd\uc6b0 \uacc4\ub2e8\uc744 \uc62c\ub9ac\uace0 \ubb38\uc744 \ub2eb\uc744 \uc218 \uc5c6\ub2e4. \ud55c\ud3b8, \uc77c\ubc18\uc801\uc778 \ud1b5\uadfc\ud615 \uc804\ub3d9\ucc28\uc640 \ucc28\ub7c9 \uae38\uc774\uac00 \ub2e4\ub974\uace0, \uc2a4\ud06c\ub9b0\ub3c4\uc5b4\uc640 \ubb38\uc758 \uc704\uce58\uac00 \ub2ec\ub77c\uc11c \uc2a4\ud06c\ub9b0\ub3c4\uc5b4\uac00 \uc124\uce58\ub41c \uc2b9\uac15\uc7a5\uc5d0\uc11c\ub294 \uc5ec\uac1d \ucde8\uae09\uc774 \ubd88\uac00\ub2a5\ud558\ub2e4.", "sentence_answer": "\uc11c\uc6b8-\uc2e0\ucc3d \uac04 \ub178\uc120\uc758 \uacbd\uc6b0, \uc218\ub3c4\uad8c \uc804\ucca0 \uad6c\uac04\uc5d0\uc11c \uc9d1\uc911\uc801\uc73c\ub85c \uc6b4\uc6a9\ub418\uae30 \ub54c\ubb38\uc5d0, \uc800\uc0c1 \uc2b9\uac15\uc7a5\uacfc \uace0\uc0c1 \uc2b9\uac15\uc7a5 \ubaa8\ub450\uc5d0\uc11c \uc2b9\ud558\ucc28\uac00 \uac00\ub2a5\ud558\ub3c4\ub85d \uc720\uc555\uc2dd \ub9ac\ud504\ud2b8 \uc7a5\uce58\ub97c \ud0d1\uc7ac\ud558\uc600\uace0, \ubc30\ub9ac\uc5b4 \ud504\ub9ac\ub97c \uc704\ud574 2\ud638\ucc28\uc5d0\ub294 \uc7a5\uc560\uc778 \ud0d1\uc2b9\uc6a9 \uacbd\uc0ac\ub85c \uc7a5\uce58 \ub97c \ud0d1\uc7ac\ud558\uc600\ub2e4.", "paragraph_id": "6569933-3-2", "paragraph_question": "question: \ubc30\ub9ac\uc5b4 \ud504\ub9ac\ub97c \uc704\ud574 2\ud638\ucc28\uc5d0 \ud0d1\uc7ac\ud55c \uac83\uc740?, context: \uc11c\uc6b8-\uc2e0\ucc3d \uac04 \ub178\uc120\uc758 \uacbd\uc6b0, \uc218\ub3c4\uad8c \uc804\ucca0 \uad6c\uac04\uc5d0\uc11c \uc9d1\uc911\uc801\uc73c\ub85c \uc6b4\uc6a9\ub418\uae30 \ub54c\ubb38\uc5d0, \uc800\uc0c1 \uc2b9\uac15\uc7a5\uacfc \uace0\uc0c1 \uc2b9\uac15\uc7a5 \ubaa8\ub450\uc5d0\uc11c \uc2b9\ud558\ucc28\uac00 \uac00\ub2a5\ud558\ub3c4\ub85d \uc720\uc555\uc2dd \ub9ac\ud504\ud2b8 \uc7a5\uce58\ub97c \ud0d1\uc7ac\ud558\uc600\uace0, \ubc30\ub9ac\uc5b4 \ud504\ub9ac\ub97c \uc704\ud574 2\ud638\ucc28\uc5d0\ub294 \uc7a5\uc560\uc778 \ud0d1\uc2b9\uc6a9 \uacbd\uc0ac\ub85c \uc7a5\uce58\ub97c \ud0d1\uc7ac\ud558\uc600\ub2e4. \uc2e0\ucc3d\uc5ed\uacfc \uc11c\uc6b8\uc5ed\uc744 \uc81c\uc678\ud55c \ubaa8\ub4e0 \uc5ed\uc5d0\uc11c \uc800\uc0c1 \uc2b9\uac15\uc7a5\uc744 \uc774\uc6a9\ud558\ub098 \ubaa8\ub4e0 \uc5f4\ucc28\uac00 \uc11c\uc6b8\uc5ed\uc5d0\uc11c \uace0\uc0c1 \uc2b9\uac15\uc7a5\ub9cc\uc744 \uc0ac\uc6a9\ud558\ub294 \uac83\uc740 \uc544\ub2c8\ub2e4. \uc2b9\uac15\uad6c\uc758 \ucc28\ub0b4 \ubd80\ubd84\uc5d0 \ub178\ub780\uc0c9\uc73c\ub85c \ud45c\uc2dc\ub41c \ubd80\ubd84 \uc548\ucabd\uc5d0\ub294 \uc800\uc0c1 \uc2b9\uac15\uc7a5 \uc815\ucc28 \uc2dc \uacc4\ub2e8\uc774 \ub0b4\ub824\uac00\ub3c4\ub85d \ub41c \ubd80\ubd84\uc73c\ub85c, \uc774 \ubd80\ubd84\uc5d0\ub294 \uc13c\uc11c\uac00 \uc788\uc5b4\uc11c \uc800\uc0c1 \uc2b9\uac15\uc7a5 \uc815\ucc28 \uc2dc \uc7a5\uc560\ubb3c\uc774 \uc788\uc744 \uacbd\uc6b0 \ub0b4\ub824\uac00\uc9c0 \uc54a\uc73c\uba70, \ubc18\ub300\ub85c \ubb38\uc744 \ub2eb\uc744 \ub54c\uc5d0\ub3c4 \uc2b9\uac1d \ub4f1\uc774 \uc11c \uc788\uc744 \uacbd\uc6b0 \uacc4\ub2e8\uc744 \uc62c\ub9ac\uace0 \ubb38\uc744 \ub2eb\uc744 \uc218 \uc5c6\ub2e4. \ud55c\ud3b8, \uc77c\ubc18\uc801\uc778 \ud1b5\uadfc\ud615 \uc804\ub3d9\ucc28\uc640 \ucc28\ub7c9 \uae38\uc774\uac00 \ub2e4\ub974\uace0, \uc2a4\ud06c\ub9b0\ub3c4\uc5b4\uc640 \ubb38\uc758 \uc704\uce58\uac00 \ub2ec\ub77c\uc11c \uc2a4\ud06c\ub9b0\ub3c4\uc5b4\uac00 \uc124\uce58\ub41c \uc2b9\uac15\uc7a5\uc5d0\uc11c\ub294 \uc5ec\uac1d \ucde8\uae09\uc774 \ubd88\uac00\ub2a5\ud558\ub2e4."} {"question": "\ube0c\ub8e8\ub178 \ub9c8\uc2a4\uac00 \uc791\uc5c5\ud55c \uc2f1\uae00 \uc568\ubc94 'Get Sexy'\uc758 \uac00\uc218\ub294 \ub204\uad6c\uc778\uac00?", "paragraph": "\uc194\ub85c \uc544\ud2f0\uc2a4\ud2b8\uac00 \ub418\uae30\uc804\uc5d0 \ub9c8\uc2a4\ub294 \uc54c\ub809\uc0b0\ub4dc\ub77c \ubc84\ud06c, \ud2b8\ub798\ube44 \ub9e4\ucf54\uc774, \uc560\ub364 \ub9ac\ubc14\uc778, \ube0c\ub79c\ub514 \ub178\uc6b0\ub4dc, \uc200 \ud0b9\uc2a4\ud134, \ud50c\ub85c \ub77c\uc774\ub2e4\ub85c\ubd80\ud130 \uc74c\uc545 \ud504\ub85c\ub4c0\uc11c \ubc0f \uc1a1\ub77c\uc774\ud130\ub85c \uc778\uc815\uc744 \ubc1b\uc558\ub2e4. \ub610\ud55c, \uc288\uac00\ubca0\uc774\ube44\uc2a4\uc758 \ud788\ud2b8 \uc2f1\uae00 \"Get Sexy\"\ub97c \uc791\uc5c5\ud588\uc73c\uba70, \uadf8\ub4e4\uc758 \uc74c\ubc18 Sweet 7\uc5d0\ub3c4 \ubc31\ubcf4\uceec\ub85c \ucc38\uc5ec\ud558\uc600\ub2e4. \ub9c8\uc2a4\uac00 \ucc98\uc74c\uc73c\ub85c \ub178\ub798\ub97c \ubc1c\ub9e4\ud55c \uac83\uc740 \ud30c \uc774\uc2a4\ud2b8 \ubb34\ube0c\uba3c\ud2b8\uc758 \ub178\ub798 \"3D\"\uc758 \ud53c\ucc98\ub9c1\uc73c\ub85c, 2\uc9d1 \uc74c\ubc18 Animal\uc5d0 \uc218\ub85d\ub41c \uace1\uc774\ub2e4. \ub9c8\uc2a4\ub294 \ub610\ud55c 2009\ub144 8\uc6d4\uc5d0 \ubc1c\ub9e4\ub41c \ubaa9\uc0ac\uc774\uc790 \ud799\ud569 \uc544\ud2f0\uc2a4\ud2b8 \uc81c\uc774\uc2a8 \ub9c8\uc758 \ub370\ubdd4 \uc2f1\uae00 \"Love\"\uc5d0\ub3c4 \ud53c\ucc98\ub9c1\uc744 \ud558\uc600\ub2e4. B.o.B\uc758 \"Nothin' on You\"\uc640 \ud2b8\ub798\ube44 \ub9e4\ucf54\uc774\uc758 \"Billionaire\"\uc5d0 \ucc38\uc5ec\ud55c \ub9c8\uc2a4\ub294 \uc774\ub54c\ubd80\ud130 \uc194\ub85c \uc544\ud2f0\uc2a4\ud2b8\ub85c\uc11c \ubcf8\uaca9\uc801\uc73c\ub85c \uc74c\uc545\uc801 \uba85\uc131\uc744 \ub5a8\ucce4\ub2e4. \uc774 \ub450\uace1\uc740 \uc804\uc138\uacc4\uc758 \ub9ce\uc740 \ub098\ub77c\uc5d0\uc11c \ud06c\uac8c \ud788\ud2b8\ub97c \ucce4\ub2e4. \uc774 \uc131\uacf5\uc5d0 \uc774\uc5b4 \ub9c8\uc2a4\ub294 \ub370\ubdd4 EP It's Better If You Don't Understand\ub97c 2010\ub144 5\uc6d4 11\uc77c \ubc1c\ub9e4\ud558\uc600\ub2e4. \uc774 EP\ub294 \ube4c\ubcf4\ub4dc 200\uc5d0\uc11c 99\uc704\ub97c \ud558\uc600\uc73c\uba70, \uc2f1\uae00 \"The Other Side\"\ub294 \uc2dc \ub85c \uadf8\ub9b0\uacfc B.o.B\uac00 \ud53c\ucc98\ub9c1\uc744 \ud558\uc600\ub2e4. \ub9c8\uc2a4\ub294 2010\ub144 8\uc6d4 \ubc1c\ub9e4\ub41c \uc2dc \ub85c \uadf8\ub9b0\uc758 \uc2f1\uae00 \"Fuck You\"\ub97c \uacf5\ub3d9\uc791\uc5c5\ud558\uae30\ub3c4 \ud558\uc600\ub2e4. 2010\ub144 9\uc6d4 12\uc77c 2010\ub144 MTV \ube44\ub514\uc624 \ubba4\uc9c1 \uc5b4\uc6cc\ub4dc\uc5d0\uc11c \ub9c8\uc2a4\ub294 B.o.B, \ud5e4\uc77c\ub9ac \uc70c\ub9ac\uc5c4\uc2a4\uc640 \ud568\uaed8 \"Nothin' on You\"\uc640 \"Airplanes\"\ub97c \ubd88\ub800\ub2e4.", "answer": "\uc288\uac00\ubca0\uc774\ube44\uc2a4", "sentence": "\ub610\ud55c, \uc288\uac00\ubca0\uc774\ube44\uc2a4 \uc758 \ud788\ud2b8 \uc2f1\uae00 \"Get Sexy\"\ub97c \uc791\uc5c5\ud588\uc73c\uba70, \uadf8\ub4e4\uc758 \uc74c\ubc18 Sweet 7\uc5d0\ub3c4 \ubc31\ubcf4\uceec\ub85c \ucc38\uc5ec\ud558\uc600\ub2e4.", "paragraph_sentence": "\uc194\ub85c \uc544\ud2f0\uc2a4\ud2b8\uac00 \ub418\uae30\uc804\uc5d0 \ub9c8\uc2a4\ub294 \uc54c\ub809\uc0b0\ub4dc\ub77c \ubc84\ud06c, \ud2b8\ub798\ube44 \ub9e4\ucf54\uc774, \uc560\ub364 \ub9ac\ubc14\uc778, \ube0c\ub79c\ub514 \ub178\uc6b0\ub4dc, \uc200 \ud0b9\uc2a4\ud134, \ud50c\ub85c \ub77c\uc774\ub2e4\ub85c\ubd80\ud130 \uc74c\uc545 \ud504\ub85c\ub4c0\uc11c \ubc0f \uc1a1\ub77c\uc774\ud130\ub85c \uc778\uc815\uc744 \ubc1b\uc558\ub2e4. \ub610\ud55c, \uc288\uac00\ubca0\uc774\ube44\uc2a4 \uc758 \ud788\ud2b8 \uc2f1\uae00 \"Get Sexy\"\ub97c \uc791\uc5c5\ud588\uc73c\uba70, \uadf8\ub4e4\uc758 \uc74c\ubc18 Sweet 7\uc5d0\ub3c4 \ubc31\ubcf4\uceec\ub85c \ucc38\uc5ec\ud558\uc600\ub2e4. \ub9c8\uc2a4\uac00 \ucc98\uc74c\uc73c\ub85c \ub178\ub798\ub97c \ubc1c\ub9e4\ud55c \uac83\uc740 \ud30c \uc774\uc2a4\ud2b8 \ubb34\ube0c\uba3c\ud2b8\uc758 \ub178\ub798 \"3D\"\uc758 \ud53c\ucc98\ub9c1\uc73c\ub85c, 2\uc9d1 \uc74c\ubc18 Animal\uc5d0 \uc218\ub85d\ub41c \uace1\uc774\ub2e4. \ub9c8\uc2a4\ub294 \ub610\ud55c 2009\ub144 8\uc6d4\uc5d0 \ubc1c\ub9e4\ub41c \ubaa9\uc0ac\uc774\uc790 \ud799\ud569 \uc544\ud2f0\uc2a4\ud2b8 \uc81c\uc774\uc2a8 \ub9c8\uc758 \ub370\ubdd4 \uc2f1\uae00 \"Love\"\uc5d0\ub3c4 \ud53c\ucc98\ub9c1\uc744 \ud558\uc600\ub2e4. B.o. B\uc758 \"Nothin' on You\"\uc640 \ud2b8\ub798\ube44 \ub9e4\ucf54\uc774\uc758 \"Billionaire\"\uc5d0 \ucc38\uc5ec\ud55c \ub9c8\uc2a4\ub294 \uc774\ub54c\ubd80\ud130 \uc194\ub85c \uc544\ud2f0\uc2a4\ud2b8\ub85c\uc11c \ubcf8\uaca9\uc801\uc73c\ub85c \uc74c\uc545\uc801 \uba85\uc131\uc744 \ub5a8\ucce4\ub2e4. \uc774 \ub450\uace1\uc740 \uc804\uc138\uacc4\uc758 \ub9ce\uc740 \ub098\ub77c\uc5d0\uc11c \ud06c\uac8c \ud788\ud2b8\ub97c \ucce4\ub2e4. \uc774 \uc131\uacf5\uc5d0 \uc774\uc5b4 \ub9c8\uc2a4\ub294 \ub370\ubdd4 EP It's Better If You Don't Understand\ub97c 2010\ub144 5\uc6d4 11\uc77c \ubc1c\ub9e4\ud558\uc600\ub2e4. \uc774 EP\ub294 \ube4c\ubcf4\ub4dc 200\uc5d0\uc11c 99\uc704\ub97c \ud558\uc600\uc73c\uba70, \uc2f1\uae00 \"The Other Side\"\ub294 \uc2dc \ub85c \uadf8\ub9b0\uacfc B.o. B\uac00 \ud53c\ucc98\ub9c1\uc744 \ud558\uc600\ub2e4. \ub9c8\uc2a4\ub294 2010\ub144 8\uc6d4 \ubc1c\ub9e4\ub41c \uc2dc \ub85c \uadf8\ub9b0\uc758 \uc2f1\uae00 \"Fuck You\"\ub97c \uacf5\ub3d9\uc791\uc5c5\ud558\uae30\ub3c4 \ud558\uc600\ub2e4. 2010\ub144 9\uc6d4 12\uc77c 2010\ub144 MTV \ube44\ub514\uc624 \ubba4\uc9c1 \uc5b4\uc6cc\ub4dc\uc5d0\uc11c \ub9c8\uc2a4\ub294 B.o. B, \ud5e4\uc77c\ub9ac \uc70c\ub9ac\uc5c4\uc2a4\uc640 \ud568\uaed8 \"Nothin' on You\"\uc640 \"Airplanes\"\ub97c \ubd88\ub800\ub2e4.", "paragraph_answer": "\uc194\ub85c \uc544\ud2f0\uc2a4\ud2b8\uac00 \ub418\uae30\uc804\uc5d0 \ub9c8\uc2a4\ub294 \uc54c\ub809\uc0b0\ub4dc\ub77c \ubc84\ud06c, \ud2b8\ub798\ube44 \ub9e4\ucf54\uc774, \uc560\ub364 \ub9ac\ubc14\uc778, \ube0c\ub79c\ub514 \ub178\uc6b0\ub4dc, \uc200 \ud0b9\uc2a4\ud134, \ud50c\ub85c \ub77c\uc774\ub2e4\ub85c\ubd80\ud130 \uc74c\uc545 \ud504\ub85c\ub4c0\uc11c \ubc0f \uc1a1\ub77c\uc774\ud130\ub85c \uc778\uc815\uc744 \ubc1b\uc558\ub2e4. \ub610\ud55c, \uc288\uac00\ubca0\uc774\ube44\uc2a4 \uc758 \ud788\ud2b8 \uc2f1\uae00 \"Get Sexy\"\ub97c \uc791\uc5c5\ud588\uc73c\uba70, \uadf8\ub4e4\uc758 \uc74c\ubc18 Sweet 7\uc5d0\ub3c4 \ubc31\ubcf4\uceec\ub85c \ucc38\uc5ec\ud558\uc600\ub2e4. \ub9c8\uc2a4\uac00 \ucc98\uc74c\uc73c\ub85c \ub178\ub798\ub97c \ubc1c\ub9e4\ud55c \uac83\uc740 \ud30c \uc774\uc2a4\ud2b8 \ubb34\ube0c\uba3c\ud2b8\uc758 \ub178\ub798 \"3D\"\uc758 \ud53c\ucc98\ub9c1\uc73c\ub85c, 2\uc9d1 \uc74c\ubc18 Animal\uc5d0 \uc218\ub85d\ub41c \uace1\uc774\ub2e4. \ub9c8\uc2a4\ub294 \ub610\ud55c 2009\ub144 8\uc6d4\uc5d0 \ubc1c\ub9e4\ub41c \ubaa9\uc0ac\uc774\uc790 \ud799\ud569 \uc544\ud2f0\uc2a4\ud2b8 \uc81c\uc774\uc2a8 \ub9c8\uc758 \ub370\ubdd4 \uc2f1\uae00 \"Love\"\uc5d0\ub3c4 \ud53c\ucc98\ub9c1\uc744 \ud558\uc600\ub2e4. B.o.B\uc758 \"Nothin' on You\"\uc640 \ud2b8\ub798\ube44 \ub9e4\ucf54\uc774\uc758 \"Billionaire\"\uc5d0 \ucc38\uc5ec\ud55c \ub9c8\uc2a4\ub294 \uc774\ub54c\ubd80\ud130 \uc194\ub85c \uc544\ud2f0\uc2a4\ud2b8\ub85c\uc11c \ubcf8\uaca9\uc801\uc73c\ub85c \uc74c\uc545\uc801 \uba85\uc131\uc744 \ub5a8\ucce4\ub2e4. \uc774 \ub450\uace1\uc740 \uc804\uc138\uacc4\uc758 \ub9ce\uc740 \ub098\ub77c\uc5d0\uc11c \ud06c\uac8c \ud788\ud2b8\ub97c \ucce4\ub2e4. \uc774 \uc131\uacf5\uc5d0 \uc774\uc5b4 \ub9c8\uc2a4\ub294 \ub370\ubdd4 EP It's Better If You Don't Understand\ub97c 2010\ub144 5\uc6d4 11\uc77c \ubc1c\ub9e4\ud558\uc600\ub2e4. \uc774 EP\ub294 \ube4c\ubcf4\ub4dc 200\uc5d0\uc11c 99\uc704\ub97c \ud558\uc600\uc73c\uba70, \uc2f1\uae00 \"The Other Side\"\ub294 \uc2dc \ub85c \uadf8\ub9b0\uacfc B.o.B\uac00 \ud53c\ucc98\ub9c1\uc744 \ud558\uc600\ub2e4. \ub9c8\uc2a4\ub294 2010\ub144 8\uc6d4 \ubc1c\ub9e4\ub41c \uc2dc \ub85c \uadf8\ub9b0\uc758 \uc2f1\uae00 \"Fuck You\"\ub97c \uacf5\ub3d9\uc791\uc5c5\ud558\uae30\ub3c4 \ud558\uc600\ub2e4. 2010\ub144 9\uc6d4 12\uc77c 2010\ub144 MTV \ube44\ub514\uc624 \ubba4\uc9c1 \uc5b4\uc6cc\ub4dc\uc5d0\uc11c \ub9c8\uc2a4\ub294 B.o.B, \ud5e4\uc77c\ub9ac \uc70c\ub9ac\uc5c4\uc2a4\uc640 \ud568\uaed8 \"Nothin' on You\"\uc640 \"Airplanes\"\ub97c \ubd88\ub800\ub2e4.", "sentence_answer": "\ub610\ud55c, \uc288\uac00\ubca0\uc774\ube44\uc2a4 \uc758 \ud788\ud2b8 \uc2f1\uae00 \"Get Sexy\"\ub97c \uc791\uc5c5\ud588\uc73c\uba70, \uadf8\ub4e4\uc758 \uc74c\ubc18 Sweet 7\uc5d0\ub3c4 \ubc31\ubcf4\uceec\ub85c \ucc38\uc5ec\ud558\uc600\ub2e4.", "paragraph_id": "6530441-5-0", "paragraph_question": "question: \ube0c\ub8e8\ub178 \ub9c8\uc2a4\uac00 \uc791\uc5c5\ud55c \uc2f1\uae00 \uc568\ubc94 'Get Sexy'\uc758 \uac00\uc218\ub294 \ub204\uad6c\uc778\uac00?, context: \uc194\ub85c \uc544\ud2f0\uc2a4\ud2b8\uac00 \ub418\uae30\uc804\uc5d0 \ub9c8\uc2a4\ub294 \uc54c\ub809\uc0b0\ub4dc\ub77c \ubc84\ud06c, \ud2b8\ub798\ube44 \ub9e4\ucf54\uc774, \uc560\ub364 \ub9ac\ubc14\uc778, \ube0c\ub79c\ub514 \ub178\uc6b0\ub4dc, \uc200 \ud0b9\uc2a4\ud134, \ud50c\ub85c \ub77c\uc774\ub2e4\ub85c\ubd80\ud130 \uc74c\uc545 \ud504\ub85c\ub4c0\uc11c \ubc0f \uc1a1\ub77c\uc774\ud130\ub85c \uc778\uc815\uc744 \ubc1b\uc558\ub2e4. \ub610\ud55c, \uc288\uac00\ubca0\uc774\ube44\uc2a4\uc758 \ud788\ud2b8 \uc2f1\uae00 \"Get Sexy\"\ub97c \uc791\uc5c5\ud588\uc73c\uba70, \uadf8\ub4e4\uc758 \uc74c\ubc18 Sweet 7\uc5d0\ub3c4 \ubc31\ubcf4\uceec\ub85c \ucc38\uc5ec\ud558\uc600\ub2e4. \ub9c8\uc2a4\uac00 \ucc98\uc74c\uc73c\ub85c \ub178\ub798\ub97c \ubc1c\ub9e4\ud55c \uac83\uc740 \ud30c \uc774\uc2a4\ud2b8 \ubb34\ube0c\uba3c\ud2b8\uc758 \ub178\ub798 \"3D\"\uc758 \ud53c\ucc98\ub9c1\uc73c\ub85c, 2\uc9d1 \uc74c\ubc18 Animal\uc5d0 \uc218\ub85d\ub41c \uace1\uc774\ub2e4. \ub9c8\uc2a4\ub294 \ub610\ud55c 2009\ub144 8\uc6d4\uc5d0 \ubc1c\ub9e4\ub41c \ubaa9\uc0ac\uc774\uc790 \ud799\ud569 \uc544\ud2f0\uc2a4\ud2b8 \uc81c\uc774\uc2a8 \ub9c8\uc758 \ub370\ubdd4 \uc2f1\uae00 \"Love\"\uc5d0\ub3c4 \ud53c\ucc98\ub9c1\uc744 \ud558\uc600\ub2e4. B.o.B\uc758 \"Nothin' on You\"\uc640 \ud2b8\ub798\ube44 \ub9e4\ucf54\uc774\uc758 \"Billionaire\"\uc5d0 \ucc38\uc5ec\ud55c \ub9c8\uc2a4\ub294 \uc774\ub54c\ubd80\ud130 \uc194\ub85c \uc544\ud2f0\uc2a4\ud2b8\ub85c\uc11c \ubcf8\uaca9\uc801\uc73c\ub85c \uc74c\uc545\uc801 \uba85\uc131\uc744 \ub5a8\ucce4\ub2e4. \uc774 \ub450\uace1\uc740 \uc804\uc138\uacc4\uc758 \ub9ce\uc740 \ub098\ub77c\uc5d0\uc11c \ud06c\uac8c \ud788\ud2b8\ub97c \ucce4\ub2e4. \uc774 \uc131\uacf5\uc5d0 \uc774\uc5b4 \ub9c8\uc2a4\ub294 \ub370\ubdd4 EP It's Better If You Don't Understand\ub97c 2010\ub144 5\uc6d4 11\uc77c \ubc1c\ub9e4\ud558\uc600\ub2e4. \uc774 EP\ub294 \ube4c\ubcf4\ub4dc 200\uc5d0\uc11c 99\uc704\ub97c \ud558\uc600\uc73c\uba70, \uc2f1\uae00 \"The Other Side\"\ub294 \uc2dc \ub85c \uadf8\ub9b0\uacfc B.o.B\uac00 \ud53c\ucc98\ub9c1\uc744 \ud558\uc600\ub2e4. \ub9c8\uc2a4\ub294 2010\ub144 8\uc6d4 \ubc1c\ub9e4\ub41c \uc2dc \ub85c \uadf8\ub9b0\uc758 \uc2f1\uae00 \"Fuck You\"\ub97c \uacf5\ub3d9\uc791\uc5c5\ud558\uae30\ub3c4 \ud558\uc600\ub2e4. 2010\ub144 9\uc6d4 12\uc77c 2010\ub144 MTV \ube44\ub514\uc624 \ubba4\uc9c1 \uc5b4\uc6cc\ub4dc\uc5d0\uc11c \ub9c8\uc2a4\ub294 B.o.B, \ud5e4\uc77c\ub9ac \uc70c\ub9ac\uc5c4\uc2a4\uc640 \ud568\uaed8 \"Nothin' on You\"\uc640 \"Airplanes\"\ub97c \ubd88\ub800\ub2e4."} {"question": "\uc7ac\uc988 \uc6b4\ub3d9\uc758 \ub9ac\ub354\ub4e4 \uc911 \ud55c\uc0ac\ub78c\uc73c\ub85c\uc11c \uc815\uce58\uc801\uc73c\ub85c\ub3c4 \uad00\uc5ec\ud558\uace0 \uc139\uc18c\ud3f0 \uc5f0\uc8fc\uac00 \ube4c\ub9ac \ud558\ud37c\uc640 \ubca0\ub2c8 \ubaa8\ud33d, \ud53c\uc544\ub2c8\uc2a4\ud2b8 \ud574\ub864\ub4dc \uba54\uc774\ubc88, \ubca0\uc774\uc2dc\uc2a4\ud2b8 \uc9c0\ubbf8 \uba54\ub9bf\uacfc \ub4dc\ub7ec\uba38 \ubbf8\ud0a4 \ub85c\ucee4\ub85c \uad6c\uc131\ub41c \ubc34\ub4dc\uc758 \ub9ac\ub354\ub97c \ub9e1\uc740 \uc74c\uc545\uac00\ub294?", "paragraph": "\uc8fd\uae30 \uc804 2\ub144\uac04, \ub9ac \ubaa8\uac74\uc740 \ub2f9\uc2dc \ubc1c\uc0dd\ud588\ub358 \uc7ac\uc988\uc6b4\ub3d9\uc758 \ub9ac\ub354\ub4e4 \uc911 \ud55c\uc0ac\ub78c\uc73c\ub85c\uc11c \uc815\uce58\uc801\uc73c\ub85c \ub354\uc6b1\ub354 \uad00\uc5ec\ud558\uc600\ub2e4. \uc774 \ubaa8\uc784\uc740 \uc778\ud130\ubdf0\ub098 \ubc84\ub77c\uc774\uc5b4\ud2f0\uc1fc \ub4f1\uc5d0\uc11c \" TV\ud504\ub85c\uadf8\ub7a8\uc774 \uc7ac\uc988 \uc5f0\uc8fc\uac00\ub4e4\uc744 \uac8c\uc2a4\ud2b8 \uc544\ud2f0\uc2a4\ud2b8\ub85c \uc12d\uc678\ud558\ub824\uace0 \ud558\uc9c0 \uc54a\ub294 \uac83\uacfc \uc790\uae30 \ud504\ub85c\uadf8\ub7a8\uc758 \ubc34\ub4dc\ub85c \uc4f0\ub824\uace0 \ud558\uc9c0 \uc54a\ub294 \uac83\"\uc5d0 \ud56d\uc758\ud558\uc600\ub2e4. \ud55c\ud3b8 \uc8fd\uae30\uc804 1 \ub144\uac04 \ub9ac \ubaa8\uac74\uc774 \ub9ac\ub354\uac00 \ub41c \ubc34\ub4dc\ub294 \uc139\uc18c\ud3f0 \uc5f0\uc8fc\uac00 \ube4c\ub9ac \ud558\ud37c\uc640 \ubca0\ub2c8 \ubaa8\ud33d, \ud53c\uc544\ub2c8\uc2a4\ud2b8 \ud574\ub864\ub4dc \uba54\uc774\ubc88, \ubca0\uc774\uc2dc\uc2a4\ud2b8 \uc9c0\ubbf8 \uba54\ub9bf\uacfc \ub4dc\ub7ec\uba38 \ubbf8\ud0a4 \ub85c\ucee4(\ud639\uc740 \ud504\ub808\ub514 \uc6e8\uc774\uce20)\ub85c \uad6c\uc131\ub418\uc5b4 \uc788\uc5c8\ub2e4. \ubaa8\ud33d, \uba54\uc774\ubc88, \uba54\ub9bf\uacfc \ub85c\ucee4\uac00 \ucc38\uc5ec\ud55c \uc74c\ubc18 \"Live at the Lighthouse\"\ub294 1970\ub144 \uce98\ub9ac\ud3ec\ub2c8\uc544\uc758 \ud5c8\ubaa8\uc11c \ube44\uce58 \ud074\ub7fd(Hermosa Beach club)\uacfc\uc758 2\uc8fc\uac04 \uacc4\uc57d \ub3d9\uc548 \ub179\uc74c\ub41c \ub77c\uc774\ube0c \uc74c\ubc18\uc73c\ub85c\uc11c \uac78\uc791\uc73c\ub85c \ud3c9\uac00\ubc1b\ub294\ub2e4.", "answer": "\ub9ac \ubaa8\uac74", "sentence": "\uc8fd\uae30 \uc804 2\ub144\uac04, \ub9ac \ubaa8\uac74 \uc740 \ub2f9\uc2dc \ubc1c\uc0dd\ud588\ub358 \uc7ac\uc988\uc6b4\ub3d9\uc758 \ub9ac\ub354\ub4e4 \uc911 \ud55c\uc0ac\ub78c\uc73c\ub85c\uc11c \uc815\uce58\uc801\uc73c\ub85c \ub354\uc6b1\ub354 \uad00\uc5ec\ud558\uc600\ub2e4.", "paragraph_sentence": " \uc8fd\uae30 \uc804 2\ub144\uac04, \ub9ac \ubaa8\uac74 \uc740 \ub2f9\uc2dc \ubc1c\uc0dd\ud588\ub358 \uc7ac\uc988\uc6b4\ub3d9\uc758 \ub9ac\ub354\ub4e4 \uc911 \ud55c\uc0ac\ub78c\uc73c\ub85c\uc11c \uc815\uce58\uc801\uc73c\ub85c \ub354\uc6b1\ub354 \uad00\uc5ec\ud558\uc600\ub2e4. \uc774 \ubaa8\uc784\uc740 \uc778\ud130\ubdf0\ub098 \ubc84\ub77c\uc774\uc5b4\ud2f0\uc1fc \ub4f1\uc5d0\uc11c \" TV\ud504\ub85c\uadf8\ub7a8\uc774 \uc7ac\uc988 \uc5f0\uc8fc\uac00\ub4e4\uc744 \uac8c\uc2a4\ud2b8 \uc544\ud2f0\uc2a4\ud2b8\ub85c \uc12d\uc678\ud558\ub824\uace0 \ud558\uc9c0 \uc54a\ub294 \uac83\uacfc \uc790\uae30 \ud504\ub85c\uadf8\ub7a8\uc758 \ubc34\ub4dc\ub85c \uc4f0\ub824\uace0 \ud558\uc9c0 \uc54a\ub294 \uac83\"\uc5d0 \ud56d\uc758\ud558\uc600\ub2e4. \ud55c\ud3b8 \uc8fd\uae30\uc804 1 \ub144\uac04 \ub9ac \ubaa8\uac74\uc774 \ub9ac\ub354\uac00 \ub41c \ubc34\ub4dc\ub294 \uc139\uc18c\ud3f0 \uc5f0\uc8fc\uac00 \ube4c\ub9ac \ud558\ud37c\uc640 \ubca0\ub2c8 \ubaa8\ud33d, \ud53c\uc544\ub2c8\uc2a4\ud2b8 \ud574\ub864\ub4dc \uba54\uc774\ubc88, \ubca0\uc774\uc2dc\uc2a4\ud2b8 \uc9c0\ubbf8 \uba54\ub9bf\uacfc \ub4dc\ub7ec\uba38 \ubbf8\ud0a4 \ub85c\ucee4(\ud639\uc740 \ud504\ub808\ub514 \uc6e8\uc774\uce20)\ub85c \uad6c\uc131\ub418\uc5b4 \uc788\uc5c8\ub2e4. \ubaa8\ud33d, \uba54\uc774\ubc88, \uba54\ub9bf\uacfc \ub85c\ucee4\uac00 \ucc38\uc5ec\ud55c \uc74c\ubc18 \"Live at the Lighthouse\"\ub294 1970\ub144 \uce98\ub9ac\ud3ec\ub2c8\uc544\uc758 \ud5c8\ubaa8\uc11c \ube44\uce58 \ud074\ub7fd(Hermosa Beach club)\uacfc\uc758 2\uc8fc\uac04 \uacc4\uc57d \ub3d9\uc548 \ub179\uc74c\ub41c \ub77c\uc774\ube0c \uc74c\ubc18\uc73c\ub85c\uc11c \uac78\uc791\uc73c\ub85c \ud3c9\uac00\ubc1b\ub294\ub2e4.", "paragraph_answer": "\uc8fd\uae30 \uc804 2\ub144\uac04, \ub9ac \ubaa8\uac74 \uc740 \ub2f9\uc2dc \ubc1c\uc0dd\ud588\ub358 \uc7ac\uc988\uc6b4\ub3d9\uc758 \ub9ac\ub354\ub4e4 \uc911 \ud55c\uc0ac\ub78c\uc73c\ub85c\uc11c \uc815\uce58\uc801\uc73c\ub85c \ub354\uc6b1\ub354 \uad00\uc5ec\ud558\uc600\ub2e4. \uc774 \ubaa8\uc784\uc740 \uc778\ud130\ubdf0\ub098 \ubc84\ub77c\uc774\uc5b4\ud2f0\uc1fc \ub4f1\uc5d0\uc11c \" TV\ud504\ub85c\uadf8\ub7a8\uc774 \uc7ac\uc988 \uc5f0\uc8fc\uac00\ub4e4\uc744 \uac8c\uc2a4\ud2b8 \uc544\ud2f0\uc2a4\ud2b8\ub85c \uc12d\uc678\ud558\ub824\uace0 \ud558\uc9c0 \uc54a\ub294 \uac83\uacfc \uc790\uae30 \ud504\ub85c\uadf8\ub7a8\uc758 \ubc34\ub4dc\ub85c \uc4f0\ub824\uace0 \ud558\uc9c0 \uc54a\ub294 \uac83\"\uc5d0 \ud56d\uc758\ud558\uc600\ub2e4. \ud55c\ud3b8 \uc8fd\uae30\uc804 1 \ub144\uac04 \ub9ac \ubaa8\uac74\uc774 \ub9ac\ub354\uac00 \ub41c \ubc34\ub4dc\ub294 \uc139\uc18c\ud3f0 \uc5f0\uc8fc\uac00 \ube4c\ub9ac \ud558\ud37c\uc640 \ubca0\ub2c8 \ubaa8\ud33d, \ud53c\uc544\ub2c8\uc2a4\ud2b8 \ud574\ub864\ub4dc \uba54\uc774\ubc88, \ubca0\uc774\uc2dc\uc2a4\ud2b8 \uc9c0\ubbf8 \uba54\ub9bf\uacfc \ub4dc\ub7ec\uba38 \ubbf8\ud0a4 \ub85c\ucee4(\ud639\uc740 \ud504\ub808\ub514 \uc6e8\uc774\uce20)\ub85c \uad6c\uc131\ub418\uc5b4 \uc788\uc5c8\ub2e4. \ubaa8\ud33d, \uba54\uc774\ubc88, \uba54\ub9bf\uacfc \ub85c\ucee4\uac00 \ucc38\uc5ec\ud55c \uc74c\ubc18 \"Live at the Lighthouse\"\ub294 1970\ub144 \uce98\ub9ac\ud3ec\ub2c8\uc544\uc758 \ud5c8\ubaa8\uc11c \ube44\uce58 \ud074\ub7fd(Hermosa Beach club)\uacfc\uc758 2\uc8fc\uac04 \uacc4\uc57d \ub3d9\uc548 \ub179\uc74c\ub41c \ub77c\uc774\ube0c \uc74c\ubc18\uc73c\ub85c\uc11c \uac78\uc791\uc73c\ub85c \ud3c9\uac00\ubc1b\ub294\ub2e4.", "sentence_answer": "\uc8fd\uae30 \uc804 2\ub144\uac04, \ub9ac \ubaa8\uac74 \uc740 \ub2f9\uc2dc \ubc1c\uc0dd\ud588\ub358 \uc7ac\uc988\uc6b4\ub3d9\uc758 \ub9ac\ub354\ub4e4 \uc911 \ud55c\uc0ac\ub78c\uc73c\ub85c\uc11c \uc815\uce58\uc801\uc73c\ub85c \ub354\uc6b1\ub354 \uad00\uc5ec\ud558\uc600\ub2e4.", "paragraph_id": "6579976-2-1", "paragraph_question": "question: \uc7ac\uc988 \uc6b4\ub3d9\uc758 \ub9ac\ub354\ub4e4 \uc911 \ud55c\uc0ac\ub78c\uc73c\ub85c\uc11c \uc815\uce58\uc801\uc73c\ub85c\ub3c4 \uad00\uc5ec\ud558\uace0 \uc139\uc18c\ud3f0 \uc5f0\uc8fc\uac00 \ube4c\ub9ac \ud558\ud37c\uc640 \ubca0\ub2c8 \ubaa8\ud33d, \ud53c\uc544\ub2c8\uc2a4\ud2b8 \ud574\ub864\ub4dc \uba54\uc774\ubc88, \ubca0\uc774\uc2dc\uc2a4\ud2b8 \uc9c0\ubbf8 \uba54\ub9bf\uacfc \ub4dc\ub7ec\uba38 \ubbf8\ud0a4 \ub85c\ucee4\ub85c \uad6c\uc131\ub41c \ubc34\ub4dc\uc758 \ub9ac\ub354\ub97c \ub9e1\uc740 \uc74c\uc545\uac00\ub294?, context: \uc8fd\uae30 \uc804 2\ub144\uac04, \ub9ac \ubaa8\uac74\uc740 \ub2f9\uc2dc \ubc1c\uc0dd\ud588\ub358 \uc7ac\uc988\uc6b4\ub3d9\uc758 \ub9ac\ub354\ub4e4 \uc911 \ud55c\uc0ac\ub78c\uc73c\ub85c\uc11c \uc815\uce58\uc801\uc73c\ub85c \ub354\uc6b1\ub354 \uad00\uc5ec\ud558\uc600\ub2e4. \uc774 \ubaa8\uc784\uc740 \uc778\ud130\ubdf0\ub098 \ubc84\ub77c\uc774\uc5b4\ud2f0\uc1fc \ub4f1\uc5d0\uc11c \" TV\ud504\ub85c\uadf8\ub7a8\uc774 \uc7ac\uc988 \uc5f0\uc8fc\uac00\ub4e4\uc744 \uac8c\uc2a4\ud2b8 \uc544\ud2f0\uc2a4\ud2b8\ub85c \uc12d\uc678\ud558\ub824\uace0 \ud558\uc9c0 \uc54a\ub294 \uac83\uacfc \uc790\uae30 \ud504\ub85c\uadf8\ub7a8\uc758 \ubc34\ub4dc\ub85c \uc4f0\ub824\uace0 \ud558\uc9c0 \uc54a\ub294 \uac83\"\uc5d0 \ud56d\uc758\ud558\uc600\ub2e4. \ud55c\ud3b8 \uc8fd\uae30\uc804 1 \ub144\uac04 \ub9ac \ubaa8\uac74\uc774 \ub9ac\ub354\uac00 \ub41c \ubc34\ub4dc\ub294 \uc139\uc18c\ud3f0 \uc5f0\uc8fc\uac00 \ube4c\ub9ac \ud558\ud37c\uc640 \ubca0\ub2c8 \ubaa8\ud33d, \ud53c\uc544\ub2c8\uc2a4\ud2b8 \ud574\ub864\ub4dc \uba54\uc774\ubc88, \ubca0\uc774\uc2dc\uc2a4\ud2b8 \uc9c0\ubbf8 \uba54\ub9bf\uacfc \ub4dc\ub7ec\uba38 \ubbf8\ud0a4 \ub85c\ucee4(\ud639\uc740 \ud504\ub808\ub514 \uc6e8\uc774\uce20)\ub85c \uad6c\uc131\ub418\uc5b4 \uc788\uc5c8\ub2e4. \ubaa8\ud33d, \uba54\uc774\ubc88, \uba54\ub9bf\uacfc \ub85c\ucee4\uac00 \ucc38\uc5ec\ud55c \uc74c\ubc18 \"Live at the Lighthouse\"\ub294 1970\ub144 \uce98\ub9ac\ud3ec\ub2c8\uc544\uc758 \ud5c8\ubaa8\uc11c \ube44\uce58 \ud074\ub7fd(Hermosa Beach club)\uacfc\uc758 2\uc8fc\uac04 \uacc4\uc57d \ub3d9\uc548 \ub179\uc74c\ub41c \ub77c\uc774\ube0c \uc74c\ubc18\uc73c\ub85c\uc11c \uac78\uc791\uc73c\ub85c \ud3c9\uac00\ubc1b\ub294\ub2e4."} {"question": "\uad50\uc721\ubd80 \uc7a5\uad00 \ud1f4\uc9c4 \uc11c\uba85\uc6b4\ub3d9\uc5d0 \ub3d9\ucc38\ud558\uc9c0 \uc54a\ub294 \uc758\uc0ac\ub97c \ubcf4\uc778 \uacf3\uc740 \uc5b4\ub514\uc778\uac00?", "paragraph": "\uc7a5\uad00 \uc7ac\uc9c1 \uc911 \ud559\uae09\ub2f9 50\uba85, 60\uba85 \uc774\uc0c1\uc758 \uacfc\ubc00 \ud559\uae09\uc218\ub97c \uc904\uc774\ub824\ub294 \ub178\ub825\uc744 \ucd94\uc9c4\ud558\uc600\uc73c\ub098 \uc2e4\ud328\ud558\uc600\ub2e4. \uadf8\ub7ec\ub098 \uadf8\uc758 \uc2dc\ub3c4\ub294 \ud6c4\uc77c '2\ub144 \ub9cc\uc5d0 \ucd08, \uc911\ub4f1\ud559\uad50 \ud559\uae09\ub2f9 \uc815\uc6d0\uc218 35\uba85\uc73c\ub85c \uc904\uc774\ub294' \ud6a8\uacfc\ub97c \ub0c8\ub2e4. \uadf8\ub7ec\ub098 \uc57c\uac04 \uc790\uc728\ud559\uc2b5\uc744 \uc5c6\uc560\uace0 \uc6d4\uac04 \ubaa8\uc758\uace0\uc0ac\ub97c \ucd95\uc18c\ud558\uba74\uc11c \"\ud2b9\uae30 \ud558\ub098\ub9cc \uc788\uc73c\uba74 \ub300\ud559\uc5d0 \uac08 \uc218 \uc788\ub294 \uc81c\ub3c4\ub97c \ub9cc\ub4e4\uaca0\ub2e4\"\uace0 \uc120\ud3ec\ud574 \uace0\uad50 \ud559\uc2b5\uc774 \uc804\ubc18\uc801\uc73c\ub85c \ub290\uc2a8\ud574\uc84c\uace0, \uc774\ub85c \uc778\ud574 \ud559\ub825 \uc800\ud558 \uc2dc\ube44\uac00 \uc77c\uc5c8\ub2e4. \uc774\ub294 \uc77c\ubcf8\uc758 \uc720\ud1a0\ub9ac \uad50\uc721\uc744 \ud749\ub0b4 \ub0b8 \uc81c\ub3c4\ub77c\uace0 \ube44\ud310\ubc1b\uc558\ub2e4. \ud55c\uad6d\uad50\uc6d0\ub2e8\uccb4\ucd1d\uc5f0\ud569\ud68c(\uad50\ucd1d)\uc740 \uad50\uc721\ubd80 \uc7a5\uad00 \ud1f4\uc9c4 \uc11c\uba85\uc6b4\ub3d9\uc744 \ubc8c\uc600\uc73c\uba70, \uc804\uad6d\uad50\uc9c1\uc6d0\ub178\ub3d9\uc870\ud569(\uc804\uad50\uc870)\uc740 \uc11c\uba85\uc5d0 \ub3d9\ucc38\ud558\uc9c0 \uc54a\ub294 \ud0dc\ub3c4\ub97c \ucde8\ud588\ub2e4. \uadf8\ub7ec\ub098 \uc77c\ubd80 \uad50\uc6d0\ub4e4\uc740 \ubc29\uad00\ud558\uac70\ub098 \ucc38\uc5ec\ud558\uae30\ub3c4 \ud588\ub2e4.", "answer": "\uc804\uad6d\uad50\uc9c1\uc6d0\ub178\ub3d9\uc870\ud569", "sentence": "\ud55c\uad6d\uad50\uc6d0\ub2e8\uccb4\ucd1d\uc5f0\ud569\ud68c(\uad50\ucd1d)\uc740 \uad50\uc721\ubd80 \uc7a5\uad00 \ud1f4\uc9c4 \uc11c\uba85\uc6b4\ub3d9\uc744 \ubc8c\uc600\uc73c\uba70, \uc804\uad6d\uad50\uc9c1\uc6d0\ub178\ub3d9\uc870\ud569 (\uc804\uad50\uc870)\uc740 \uc11c\uba85\uc5d0 \ub3d9\ucc38\ud558\uc9c0 \uc54a\ub294 \ud0dc\ub3c4\ub97c \ucde8\ud588\ub2e4.", "paragraph_sentence": "\uc7a5\uad00 \uc7ac\uc9c1 \uc911 \ud559\uae09\ub2f9 50\uba85, 60\uba85 \uc774\uc0c1\uc758 \uacfc\ubc00 \ud559\uae09\uc218\ub97c \uc904\uc774\ub824\ub294 \ub178\ub825\uc744 \ucd94\uc9c4\ud558\uc600\uc73c\ub098 \uc2e4\ud328\ud558\uc600\ub2e4. \uadf8\ub7ec\ub098 \uadf8\uc758 \uc2dc\ub3c4\ub294 \ud6c4\uc77c '2\ub144 \ub9cc\uc5d0 \ucd08, \uc911\ub4f1\ud559\uad50 \ud559\uae09\ub2f9 \uc815\uc6d0\uc218 35\uba85\uc73c\ub85c \uc904\uc774\ub294' \ud6a8\uacfc\ub97c \ub0c8\ub2e4. \uadf8\ub7ec\ub098 \uc57c\uac04 \uc790\uc728\ud559\uc2b5\uc744 \uc5c6\uc560\uace0 \uc6d4\uac04 \ubaa8\uc758\uace0\uc0ac\ub97c \ucd95\uc18c\ud558\uba74\uc11c \"\ud2b9\uae30 \ud558\ub098\ub9cc \uc788\uc73c\uba74 \ub300\ud559\uc5d0 \uac08 \uc218 \uc788\ub294 \uc81c\ub3c4\ub97c \ub9cc\ub4e4\uaca0\ub2e4\"\uace0 \uc120\ud3ec\ud574 \uace0\uad50 \ud559\uc2b5\uc774 \uc804\ubc18\uc801\uc73c\ub85c \ub290\uc2a8\ud574\uc84c\uace0, \uc774\ub85c \uc778\ud574 \ud559\ub825 \uc800\ud558 \uc2dc\ube44\uac00 \uc77c\uc5c8\ub2e4. \uc774\ub294 \uc77c\ubcf8\uc758 \uc720\ud1a0\ub9ac \uad50\uc721\uc744 \ud749\ub0b4 \ub0b8 \uc81c\ub3c4\ub77c\uace0 \ube44\ud310\ubc1b\uc558\ub2e4. \ud55c\uad6d\uad50\uc6d0\ub2e8\uccb4\ucd1d\uc5f0\ud569\ud68c(\uad50\ucd1d)\uc740 \uad50\uc721\ubd80 \uc7a5\uad00 \ud1f4\uc9c4 \uc11c\uba85\uc6b4\ub3d9\uc744 \ubc8c\uc600\uc73c\uba70, \uc804\uad6d\uad50\uc9c1\uc6d0\ub178\ub3d9\uc870\ud569 (\uc804\uad50\uc870)\uc740 \uc11c\uba85\uc5d0 \ub3d9\ucc38\ud558\uc9c0 \uc54a\ub294 \ud0dc\ub3c4\ub97c \ucde8\ud588\ub2e4. \uadf8\ub7ec\ub098 \uc77c\ubd80 \uad50\uc6d0\ub4e4\uc740 \ubc29\uad00\ud558\uac70\ub098 \ucc38\uc5ec\ud558\uae30\ub3c4 \ud588\ub2e4.", "paragraph_answer": "\uc7a5\uad00 \uc7ac\uc9c1 \uc911 \ud559\uae09\ub2f9 50\uba85, 60\uba85 \uc774\uc0c1\uc758 \uacfc\ubc00 \ud559\uae09\uc218\ub97c \uc904\uc774\ub824\ub294 \ub178\ub825\uc744 \ucd94\uc9c4\ud558\uc600\uc73c\ub098 \uc2e4\ud328\ud558\uc600\ub2e4. \uadf8\ub7ec\ub098 \uadf8\uc758 \uc2dc\ub3c4\ub294 \ud6c4\uc77c '2\ub144 \ub9cc\uc5d0 \ucd08, \uc911\ub4f1\ud559\uad50 \ud559\uae09\ub2f9 \uc815\uc6d0\uc218 35\uba85\uc73c\ub85c \uc904\uc774\ub294' \ud6a8\uacfc\ub97c \ub0c8\ub2e4. \uadf8\ub7ec\ub098 \uc57c\uac04 \uc790\uc728\ud559\uc2b5\uc744 \uc5c6\uc560\uace0 \uc6d4\uac04 \ubaa8\uc758\uace0\uc0ac\ub97c \ucd95\uc18c\ud558\uba74\uc11c \"\ud2b9\uae30 \ud558\ub098\ub9cc \uc788\uc73c\uba74 \ub300\ud559\uc5d0 \uac08 \uc218 \uc788\ub294 \uc81c\ub3c4\ub97c \ub9cc\ub4e4\uaca0\ub2e4\"\uace0 \uc120\ud3ec\ud574 \uace0\uad50 \ud559\uc2b5\uc774 \uc804\ubc18\uc801\uc73c\ub85c \ub290\uc2a8\ud574\uc84c\uace0, \uc774\ub85c \uc778\ud574 \ud559\ub825 \uc800\ud558 \uc2dc\ube44\uac00 \uc77c\uc5c8\ub2e4. \uc774\ub294 \uc77c\ubcf8\uc758 \uc720\ud1a0\ub9ac \uad50\uc721\uc744 \ud749\ub0b4 \ub0b8 \uc81c\ub3c4\ub77c\uace0 \ube44\ud310\ubc1b\uc558\ub2e4. \ud55c\uad6d\uad50\uc6d0\ub2e8\uccb4\ucd1d\uc5f0\ud569\ud68c(\uad50\ucd1d)\uc740 \uad50\uc721\ubd80 \uc7a5\uad00 \ud1f4\uc9c4 \uc11c\uba85\uc6b4\ub3d9\uc744 \ubc8c\uc600\uc73c\uba70, \uc804\uad6d\uad50\uc9c1\uc6d0\ub178\ub3d9\uc870\ud569 (\uc804\uad50\uc870)\uc740 \uc11c\uba85\uc5d0 \ub3d9\ucc38\ud558\uc9c0 \uc54a\ub294 \ud0dc\ub3c4\ub97c \ucde8\ud588\ub2e4. \uadf8\ub7ec\ub098 \uc77c\ubd80 \uad50\uc6d0\ub4e4\uc740 \ubc29\uad00\ud558\uac70\ub098 \ucc38\uc5ec\ud558\uae30\ub3c4 \ud588\ub2e4.", "sentence_answer": "\ud55c\uad6d\uad50\uc6d0\ub2e8\uccb4\ucd1d\uc5f0\ud569\ud68c(\uad50\ucd1d)\uc740 \uad50\uc721\ubd80 \uc7a5\uad00 \ud1f4\uc9c4 \uc11c\uba85\uc6b4\ub3d9\uc744 \ubc8c\uc600\uc73c\uba70, \uc804\uad6d\uad50\uc9c1\uc6d0\ub178\ub3d9\uc870\ud569 (\uc804\uad50\uc870)\uc740 \uc11c\uba85\uc5d0 \ub3d9\ucc38\ud558\uc9c0 \uc54a\ub294 \ud0dc\ub3c4\ub97c \ucde8\ud588\ub2e4.", "paragraph_id": "6547929-3-0", "paragraph_question": "question: \uad50\uc721\ubd80 \uc7a5\uad00 \ud1f4\uc9c4 \uc11c\uba85\uc6b4\ub3d9\uc5d0 \ub3d9\ucc38\ud558\uc9c0 \uc54a\ub294 \uc758\uc0ac\ub97c \ubcf4\uc778 \uacf3\uc740 \uc5b4\ub514\uc778\uac00?, context: \uc7a5\uad00 \uc7ac\uc9c1 \uc911 \ud559\uae09\ub2f9 50\uba85, 60\uba85 \uc774\uc0c1\uc758 \uacfc\ubc00 \ud559\uae09\uc218\ub97c \uc904\uc774\ub824\ub294 \ub178\ub825\uc744 \ucd94\uc9c4\ud558\uc600\uc73c\ub098 \uc2e4\ud328\ud558\uc600\ub2e4. \uadf8\ub7ec\ub098 \uadf8\uc758 \uc2dc\ub3c4\ub294 \ud6c4\uc77c '2\ub144 \ub9cc\uc5d0 \ucd08, \uc911\ub4f1\ud559\uad50 \ud559\uae09\ub2f9 \uc815\uc6d0\uc218 35\uba85\uc73c\ub85c \uc904\uc774\ub294' \ud6a8\uacfc\ub97c \ub0c8\ub2e4. \uadf8\ub7ec\ub098 \uc57c\uac04 \uc790\uc728\ud559\uc2b5\uc744 \uc5c6\uc560\uace0 \uc6d4\uac04 \ubaa8\uc758\uace0\uc0ac\ub97c \ucd95\uc18c\ud558\uba74\uc11c \"\ud2b9\uae30 \ud558\ub098\ub9cc \uc788\uc73c\uba74 \ub300\ud559\uc5d0 \uac08 \uc218 \uc788\ub294 \uc81c\ub3c4\ub97c \ub9cc\ub4e4\uaca0\ub2e4\"\uace0 \uc120\ud3ec\ud574 \uace0\uad50 \ud559\uc2b5\uc774 \uc804\ubc18\uc801\uc73c\ub85c \ub290\uc2a8\ud574\uc84c\uace0, \uc774\ub85c \uc778\ud574 \ud559\ub825 \uc800\ud558 \uc2dc\ube44\uac00 \uc77c\uc5c8\ub2e4. \uc774\ub294 \uc77c\ubcf8\uc758 \uc720\ud1a0\ub9ac \uad50\uc721\uc744 \ud749\ub0b4 \ub0b8 \uc81c\ub3c4\ub77c\uace0 \ube44\ud310\ubc1b\uc558\ub2e4. \ud55c\uad6d\uad50\uc6d0\ub2e8\uccb4\ucd1d\uc5f0\ud569\ud68c(\uad50\ucd1d)\uc740 \uad50\uc721\ubd80 \uc7a5\uad00 \ud1f4\uc9c4 \uc11c\uba85\uc6b4\ub3d9\uc744 \ubc8c\uc600\uc73c\uba70, \uc804\uad6d\uad50\uc9c1\uc6d0\ub178\ub3d9\uc870\ud569(\uc804\uad50\uc870)\uc740 \uc11c\uba85\uc5d0 \ub3d9\ucc38\ud558\uc9c0 \uc54a\ub294 \ud0dc\ub3c4\ub97c \ucde8\ud588\ub2e4. \uadf8\ub7ec\ub098 \uc77c\ubd80 \uad50\uc6d0\ub4e4\uc740 \ubc29\uad00\ud558\uac70\ub098 \ucc38\uc5ec\ud558\uae30\ub3c4 \ud588\ub2e4."} {"question": "1999\ub144, \uacf5\uac1c \uc624\ub514\uc158\uc744 \ud1b5\ud558\uc5ec \uba64\ubc84 \uc774\uac00\uc774\uc758 \ube48\uc790\ub9ac\ub97c \ub300\uc2e0\ud558\uac8c \ub41c \uc0c8 \uba64\ubc84\uc758 \uc774\ub984\uc740 \ubb34\uc5c7\uc778\uac00?", "paragraph": "1999\ub144 4\uc6d4, \uba64\ubc84 \uc774\uac00\uc774\uc758 \ube48\uc790\ub9ac\ub97c \uba54\uafc0 \uc0c8 \uba64\ubc84\ub97c \ubf51\ub294 \uacf5\uac1c \uc624\ub514\uc158\uc744 \uac1c\ucd5c\ud55c\ub2e4. \ub2f9\uc2dc \uc911\ud559\uad50 3\ud559\ub144\uc5d0 \uc7ac\ud559\uc911\uc774\ub358 \uc724\uc740\ud61c\uac00 \ud574\ub2f9 \uc624\ub514\uc158\uc5d0\uc11c \uce90\uc2a4\ud305\ub418\uc5c8\uace0 1~2\uc9d1 \ub2f9\uc2dc \ubd88\uc548\uc815\ud588\ub358 \uba64\ubc84 \ub77c\uc778\uc5c5\uc774 \ucd5c\uc885\uc801\uc73c\ub85c \uc644\uc131\ub418\ub294 \uacc4\uae30\uac00 \ub418\uc5c8\ub2e4. \uc724\uc740\ud61c\uac00 \ud569\ub958\ub41c \ubca0\uc774\ube44\ubcf5\uc2a4\ub294 1999\ub144 7\uc6d4\uc5d0 3\uc9d1 \uc758 \uc2f1\uae00 \ub85c \ucef4\ubc31\ud55c\ub2e4. \uc740 \ubc15\uc9c4\uc601\uc774 \uc791\uc0ac\ub97c, \uae40\ud615\uc11d\uc774 \uc791\uace1\uc744 \ub9e1\uc740 \ud391\ud0a4 \uc2a4\ud0c0\uc77c\uc758 \ud31d \ub304\uc2a4\uace1\uc774\ub2e4. \uccab\ubc29\uc1a1\uc5d0\uc11c \uae40\uc774\uc9c0\uc758 \uac00\uc2b4\uc774 \ub178\ucd9c\ub41c \uc120\uc815\uc801\uc778 \uc758\uc0c1\uacfc \ub2e4\ub9ac\ub97c \uc4f8\uc5b4\ub0b4\ub9ac\ub294 \uc548\ubb34\ub4f1\uc774 PC\ud1b5\uc2e0\uc0c1\uc5d0\uc11c \ud06c\uac8c \uc774\uc288\uac00 \ub418\uc5c8\ub2e4. \uc911\ub3c5\uc131 \uc788\ub294 \uac00\uc0ac\uc5d0 \uc27d\uac8c \uadc0\uc5d0\uc775\ub294 \uba5c\ub85c\ub514\ub4f1\uc774 \ub300\uc911\ub4e4\uc5d0\uac8c \ud06c\uac8c \uc5b4\ud544\ub418\uc5b4 \uc2dc\uc7a5\uc5d0\uc11c \uc131\uacf5\uc744 \uac70\ub450\uc5c8\uace0, 1999\ub144 8\uc6d4 3\uc77c, \uc74c\ubc18\uc774 \ubc1c\ub9e4\ub41c\uc9c0 3\uc8fc\ub3c4 \ucc44 \uc9c0\ub098\uc9c0 \uc54a\uc544 KBS \ubba4\uc9c1\ubc45\ud06c\uc5d0\uc11c \ub370\ubdd4 2\ub144\ub9cc\uc5d0 \ucc98\uc74c\uc73c\ub85c 1\uc704\ub97c \uc218\uc0c1\ud558\uba70 \uc815\uc0c1\uc5d0 \ub4f1\uadf9\ud55c\ub2e4.", "answer": "\uc724\uc740\ud61c", "sentence": "\ub2f9\uc2dc \uc911\ud559\uad50 3\ud559\ub144\uc5d0 \uc7ac\ud559\uc911\uc774\ub358 \uc724\uc740\ud61c \uac00 \ud574\ub2f9 \uc624\ub514\uc158\uc5d0\uc11c \uce90\uc2a4\ud305\ub418\uc5c8\uace0 1~2\uc9d1 \ub2f9\uc2dc \ubd88\uc548\uc815\ud588\ub358 \uba64\ubc84 \ub77c\uc778\uc5c5\uc774 \ucd5c\uc885\uc801\uc73c\ub85c \uc644\uc131\ub418\ub294 \uacc4\uae30\uac00 \ub418\uc5c8\ub2e4.", "paragraph_sentence": "1999\ub144 4\uc6d4, \uba64\ubc84 \uc774\uac00\uc774\uc758 \ube48\uc790\ub9ac\ub97c \uba54\uafc0 \uc0c8 \uba64\ubc84\ub97c \ubf51\ub294 \uacf5\uac1c \uc624\ub514\uc158\uc744 \uac1c\ucd5c\ud55c\ub2e4. \ub2f9\uc2dc \uc911\ud559\uad50 3\ud559\ub144\uc5d0 \uc7ac\ud559\uc911\uc774\ub358 \uc724\uc740\ud61c \uac00 \ud574\ub2f9 \uc624\ub514\uc158\uc5d0\uc11c \uce90\uc2a4\ud305\ub418\uc5c8\uace0 1~2\uc9d1 \ub2f9\uc2dc \ubd88\uc548\uc815\ud588\ub358 \uba64\ubc84 \ub77c\uc778\uc5c5\uc774 \ucd5c\uc885\uc801\uc73c\ub85c \uc644\uc131\ub418\ub294 \uacc4\uae30\uac00 \ub418\uc5c8\ub2e4. \uc724\uc740\ud61c\uac00 \ud569\ub958\ub41c \ubca0\uc774\ube44\ubcf5\uc2a4\ub294 1999\ub144 7\uc6d4\uc5d0 3\uc9d1 \uc758 \uc2f1\uae00 \ub85c \ucef4\ubc31\ud55c\ub2e4. \uc740 \ubc15\uc9c4\uc601\uc774 \uc791\uc0ac\ub97c, \uae40\ud615\uc11d\uc774 \uc791\uace1\uc744 \ub9e1\uc740 \ud391\ud0a4 \uc2a4\ud0c0\uc77c\uc758 \ud31d \ub304\uc2a4\uace1\uc774\ub2e4. \uccab\ubc29\uc1a1\uc5d0\uc11c \uae40\uc774\uc9c0\uc758 \uac00\uc2b4\uc774 \ub178\ucd9c\ub41c \uc120\uc815\uc801\uc778 \uc758\uc0c1\uacfc \ub2e4\ub9ac\ub97c \uc4f8\uc5b4\ub0b4\ub9ac\ub294 \uc548\ubb34\ub4f1\uc774 PC\ud1b5\uc2e0\uc0c1\uc5d0\uc11c \ud06c\uac8c \uc774\uc288\uac00 \ub418\uc5c8\ub2e4. \uc911\ub3c5\uc131 \uc788\ub294 \uac00\uc0ac\uc5d0 \uc27d\uac8c \uadc0\uc5d0\uc775\ub294 \uba5c\ub85c\ub514\ub4f1\uc774 \ub300\uc911\ub4e4\uc5d0\uac8c \ud06c\uac8c \uc5b4\ud544\ub418\uc5b4 \uc2dc\uc7a5\uc5d0\uc11c \uc131\uacf5\uc744 \uac70\ub450\uc5c8\uace0, 1999\ub144 8\uc6d4 3\uc77c, \uc74c\ubc18\uc774 \ubc1c\ub9e4\ub41c\uc9c0 3\uc8fc\ub3c4 \ucc44 \uc9c0\ub098\uc9c0 \uc54a\uc544 KBS \ubba4\uc9c1\ubc45\ud06c\uc5d0\uc11c \ub370\ubdd4 2\ub144\ub9cc\uc5d0 \ucc98\uc74c\uc73c\ub85c 1\uc704\ub97c \uc218\uc0c1\ud558\uba70 \uc815\uc0c1\uc5d0 \ub4f1\uadf9\ud55c\ub2e4.", "paragraph_answer": "1999\ub144 4\uc6d4, \uba64\ubc84 \uc774\uac00\uc774\uc758 \ube48\uc790\ub9ac\ub97c \uba54\uafc0 \uc0c8 \uba64\ubc84\ub97c \ubf51\ub294 \uacf5\uac1c \uc624\ub514\uc158\uc744 \uac1c\ucd5c\ud55c\ub2e4. \ub2f9\uc2dc \uc911\ud559\uad50 3\ud559\ub144\uc5d0 \uc7ac\ud559\uc911\uc774\ub358 \uc724\uc740\ud61c \uac00 \ud574\ub2f9 \uc624\ub514\uc158\uc5d0\uc11c \uce90\uc2a4\ud305\ub418\uc5c8\uace0 1~2\uc9d1 \ub2f9\uc2dc \ubd88\uc548\uc815\ud588\ub358 \uba64\ubc84 \ub77c\uc778\uc5c5\uc774 \ucd5c\uc885\uc801\uc73c\ub85c \uc644\uc131\ub418\ub294 \uacc4\uae30\uac00 \ub418\uc5c8\ub2e4. \uc724\uc740\ud61c\uac00 \ud569\ub958\ub41c \ubca0\uc774\ube44\ubcf5\uc2a4\ub294 1999\ub144 7\uc6d4\uc5d0 3\uc9d1 \uc758 \uc2f1\uae00 \ub85c \ucef4\ubc31\ud55c\ub2e4. \uc740 \ubc15\uc9c4\uc601\uc774 \uc791\uc0ac\ub97c, \uae40\ud615\uc11d\uc774 \uc791\uace1\uc744 \ub9e1\uc740 \ud391\ud0a4 \uc2a4\ud0c0\uc77c\uc758 \ud31d \ub304\uc2a4\uace1\uc774\ub2e4. \uccab\ubc29\uc1a1\uc5d0\uc11c \uae40\uc774\uc9c0\uc758 \uac00\uc2b4\uc774 \ub178\ucd9c\ub41c \uc120\uc815\uc801\uc778 \uc758\uc0c1\uacfc \ub2e4\ub9ac\ub97c \uc4f8\uc5b4\ub0b4\ub9ac\ub294 \uc548\ubb34\ub4f1\uc774 PC\ud1b5\uc2e0\uc0c1\uc5d0\uc11c \ud06c\uac8c \uc774\uc288\uac00 \ub418\uc5c8\ub2e4. \uc911\ub3c5\uc131 \uc788\ub294 \uac00\uc0ac\uc5d0 \uc27d\uac8c \uadc0\uc5d0\uc775\ub294 \uba5c\ub85c\ub514\ub4f1\uc774 \ub300\uc911\ub4e4\uc5d0\uac8c \ud06c\uac8c \uc5b4\ud544\ub418\uc5b4 \uc2dc\uc7a5\uc5d0\uc11c \uc131\uacf5\uc744 \uac70\ub450\uc5c8\uace0, 1999\ub144 8\uc6d4 3\uc77c, \uc74c\ubc18\uc774 \ubc1c\ub9e4\ub41c\uc9c0 3\uc8fc\ub3c4 \ucc44 \uc9c0\ub098\uc9c0 \uc54a\uc544 KBS \ubba4\uc9c1\ubc45\ud06c\uc5d0\uc11c \ub370\ubdd4 2\ub144\ub9cc\uc5d0 \ucc98\uc74c\uc73c\ub85c 1\uc704\ub97c \uc218\uc0c1\ud558\uba70 \uc815\uc0c1\uc5d0 \ub4f1\uadf9\ud55c\ub2e4.", "sentence_answer": "\ub2f9\uc2dc \uc911\ud559\uad50 3\ud559\ub144\uc5d0 \uc7ac\ud559\uc911\uc774\ub358 \uc724\uc740\ud61c \uac00 \ud574\ub2f9 \uc624\ub514\uc158\uc5d0\uc11c \uce90\uc2a4\ud305\ub418\uc5c8\uace0 1~2\uc9d1 \ub2f9\uc2dc \ubd88\uc548\uc815\ud588\ub358 \uba64\ubc84 \ub77c\uc778\uc5c5\uc774 \ucd5c\uc885\uc801\uc73c\ub85c \uc644\uc131\ub418\ub294 \uacc4\uae30\uac00 \ub418\uc5c8\ub2e4.", "paragraph_id": "6110374-9-0", "paragraph_question": "question: 1999\ub144, \uacf5\uac1c \uc624\ub514\uc158\uc744 \ud1b5\ud558\uc5ec \uba64\ubc84 \uc774\uac00\uc774\uc758 \ube48\uc790\ub9ac\ub97c \ub300\uc2e0\ud558\uac8c \ub41c \uc0c8 \uba64\ubc84\uc758 \uc774\ub984\uc740 \ubb34\uc5c7\uc778\uac00?, context: 1999\ub144 4\uc6d4, \uba64\ubc84 \uc774\uac00\uc774\uc758 \ube48\uc790\ub9ac\ub97c \uba54\uafc0 \uc0c8 \uba64\ubc84\ub97c \ubf51\ub294 \uacf5\uac1c \uc624\ub514\uc158\uc744 \uac1c\ucd5c\ud55c\ub2e4. \ub2f9\uc2dc \uc911\ud559\uad50 3\ud559\ub144\uc5d0 \uc7ac\ud559\uc911\uc774\ub358 \uc724\uc740\ud61c\uac00 \ud574\ub2f9 \uc624\ub514\uc158\uc5d0\uc11c \uce90\uc2a4\ud305\ub418\uc5c8\uace0 1~2\uc9d1 \ub2f9\uc2dc \ubd88\uc548\uc815\ud588\ub358 \uba64\ubc84 \ub77c\uc778\uc5c5\uc774 \ucd5c\uc885\uc801\uc73c\ub85c \uc644\uc131\ub418\ub294 \uacc4\uae30\uac00 \ub418\uc5c8\ub2e4. \uc724\uc740\ud61c\uac00 \ud569\ub958\ub41c \ubca0\uc774\ube44\ubcf5\uc2a4\ub294 1999\ub144 7\uc6d4\uc5d0 3\uc9d1 \uc758 \uc2f1\uae00 \ub85c \ucef4\ubc31\ud55c\ub2e4. \uc740 \ubc15\uc9c4\uc601\uc774 \uc791\uc0ac\ub97c, \uae40\ud615\uc11d\uc774 \uc791\uace1\uc744 \ub9e1\uc740 \ud391\ud0a4 \uc2a4\ud0c0\uc77c\uc758 \ud31d \ub304\uc2a4\uace1\uc774\ub2e4. \uccab\ubc29\uc1a1\uc5d0\uc11c \uae40\uc774\uc9c0\uc758 \uac00\uc2b4\uc774 \ub178\ucd9c\ub41c \uc120\uc815\uc801\uc778 \uc758\uc0c1\uacfc \ub2e4\ub9ac\ub97c \uc4f8\uc5b4\ub0b4\ub9ac\ub294 \uc548\ubb34\ub4f1\uc774 PC\ud1b5\uc2e0\uc0c1\uc5d0\uc11c \ud06c\uac8c \uc774\uc288\uac00 \ub418\uc5c8\ub2e4. \uc911\ub3c5\uc131 \uc788\ub294 \uac00\uc0ac\uc5d0 \uc27d\uac8c \uadc0\uc5d0\uc775\ub294 \uba5c\ub85c\ub514\ub4f1\uc774 \ub300\uc911\ub4e4\uc5d0\uac8c \ud06c\uac8c \uc5b4\ud544\ub418\uc5b4 \uc2dc\uc7a5\uc5d0\uc11c \uc131\uacf5\uc744 \uac70\ub450\uc5c8\uace0, 1999\ub144 8\uc6d4 3\uc77c, \uc74c\ubc18\uc774 \ubc1c\ub9e4\ub41c\uc9c0 3\uc8fc\ub3c4 \ucc44 \uc9c0\ub098\uc9c0 \uc54a\uc544 KBS \ubba4\uc9c1\ubc45\ud06c\uc5d0\uc11c \ub370\ubdd4 2\ub144\ub9cc\uc5d0 \ucc98\uc74c\uc73c\ub85c 1\uc704\ub97c \uc218\uc0c1\ud558\uba70 \uc815\uc0c1\uc5d0 \ub4f1\uadf9\ud55c\ub2e4."} {"question": "\uce7c\ub9ac\uc2a4\ud1a0 \ub300\uae30\uad8c\uc744 \uad6c\uc131\ud558\ub294 \uc774\uc0b0\ud654\ud0c4\uc18c\ub97c \uc2b9\ud654\uc2dc\ud0a4\ub294 \ubd80\ubd84\uc740?", "paragraph": "\uce7c\ub9ac\uc2a4\ud1a0\ub294 \uc774\uc0b0\ud654 \ud0c4\uc18c\ub85c \uad6c\uc131\ub41c \ub9e4\uc6b0 \ubbf8\uc57d\ud55c \ub300\uae30\uad8c\uc744 \uac00\uc9c0\uace0 \uc788\ub2e4. \uc774\ub294 \uac08\ub9b4\ub808\uc624 \ud0d0\uc0ac\uc120\uc758 \uadfc\uc801\uc678\uc120 \ubd84\uad11 \uc9c0\ub3c4\uc791\uc131\uae30(Near Infrared Mapping Spectrometer, NIMS)\uac00 4.2 \u00b5m \ub300\uc5ed\uc5d0\uc11c \ub300\uae30\uad8c\uc774 \ub9cc\ub4e4\uc5b4 \ub0b8 \ud30c\uc7a5 \ud761\uc218\ub97c \uac10\uc9c0\ud558\uc5ec \ubc1c\uacac\ub41c \uac83\uc774\ub2e4. \uce7c\ub9ac\uc2a4\ud1a0 \ud45c\uba74\uc5d0\uc11c\uc758 \uae30\uc555\uc740 \uc57d 7.5\u00d710^ bar (0.75 \u00b5Pa) \ub85c \ucd94\uc815\ub418\uace0, \uc785\uc790\ub4e4\uc758 \ubc00\ub3c4\ub294 4\u00d710^ cm \uc774\ub2e4. \uc774 \uc815\ub3c4\ub85c \uc605\uc740 \ub300\uae30\uad8c\uc740 4\uc77c \uc774\ub0b4\ub85c \ubaa8\ub450 \uce7c\ub9ac\uc2a4\ud1a0\ub97c \ud0c8\ucd9c\ud574 \ubc84\ub9b4 \uac83\uc774\uae30 \ub54c\ubb38\uc5d0, \ub300\uae30\uad8c\uc740 \uce7c\ub9ac\uc2a4\ud1a0\uc758 \uc5bc\uc74c \uc9c0\uac01\uc5d0\uc11c \ucc9c\ucc9c\ud788 \uc2b9\ud654\ub418\ub294 \uc774\uc0b0\ud654 \ud0c4\uc18c\uc5d0 \uc758\ud574 \uc9c0\uc18d\uc801\uc73c\ub85c \ubcf4\ucda9\ub418\uace0 \uc788\ub2e4\uace0 \uc5ec\uaca8\uc9c4\ub2e4. \uc774 \uc774\ub860\uc740 \ud45c\uba74\uc758 \ud639\ub4e4\uc744 \uc124\uba85\ud558\ub294 \uc2b9\ud654-\ubd84\ud574 \uc774\ub860\uacfc \uc5b4\ub290 \uc815\ub3c4 \uc77c\uce58\ud55c\ub2e4.", "answer": "\uc5bc\uc74c \uc9c0\uac01", "sentence": "\uc774 \uc815\ub3c4\ub85c \uc605\uc740 \ub300\uae30\uad8c\uc740 4\uc77c \uc774\ub0b4\ub85c \ubaa8\ub450 \uce7c\ub9ac\uc2a4\ud1a0\ub97c \ud0c8\ucd9c\ud574 \ubc84\ub9b4 \uac83\uc774\uae30 \ub54c\ubb38\uc5d0, \ub300\uae30\uad8c\uc740 \uce7c\ub9ac\uc2a4\ud1a0\uc758 \uc5bc\uc74c \uc9c0\uac01 \uc5d0\uc11c \ucc9c\ucc9c\ud788 \uc2b9\ud654\ub418\ub294 \uc774\uc0b0\ud654 \ud0c4\uc18c\uc5d0 \uc758\ud574 \uc9c0\uc18d\uc801\uc73c\ub85c \ubcf4\ucda9\ub418\uace0 \uc788\ub2e4\uace0 \uc5ec\uaca8\uc9c4\ub2e4.", "paragraph_sentence": "\uce7c\ub9ac\uc2a4\ud1a0\ub294 \uc774\uc0b0\ud654 \ud0c4\uc18c\ub85c \uad6c\uc131\ub41c \ub9e4\uc6b0 \ubbf8\uc57d\ud55c \ub300\uae30\uad8c\uc744 \uac00\uc9c0\uace0 \uc788\ub2e4. \uc774\ub294 \uac08\ub9b4\ub808\uc624 \ud0d0\uc0ac\uc120\uc758 \uadfc\uc801\uc678\uc120 \ubd84\uad11 \uc9c0\ub3c4\uc791\uc131\uae30(Near Infrared Mapping Spectrometer, NIMS)\uac00 4.2 \u00b5m \ub300\uc5ed\uc5d0\uc11c \ub300\uae30\uad8c\uc774 \ub9cc\ub4e4\uc5b4 \ub0b8 \ud30c\uc7a5 \ud761\uc218\ub97c \uac10\uc9c0\ud558\uc5ec \ubc1c\uacac\ub41c \uac83\uc774\ub2e4. \uce7c\ub9ac\uc2a4\ud1a0 \ud45c\uba74\uc5d0\uc11c\uc758 \uae30\uc555\uc740 \uc57d 7.5\u00d710^ bar (0.75 \u00b5Pa) \ub85c \ucd94\uc815\ub418\uace0, \uc785\uc790\ub4e4\uc758 \ubc00\ub3c4\ub294 4\u00d710^ cm \uc774\ub2e4. \uc774 \uc815\ub3c4\ub85c \uc605\uc740 \ub300\uae30\uad8c\uc740 4\uc77c \uc774\ub0b4\ub85c \ubaa8\ub450 \uce7c\ub9ac\uc2a4\ud1a0\ub97c \ud0c8\ucd9c\ud574 \ubc84\ub9b4 \uac83\uc774\uae30 \ub54c\ubb38\uc5d0, \ub300\uae30\uad8c\uc740 \uce7c\ub9ac\uc2a4\ud1a0\uc758 \uc5bc\uc74c \uc9c0\uac01 \uc5d0\uc11c \ucc9c\ucc9c\ud788 \uc2b9\ud654\ub418\ub294 \uc774\uc0b0\ud654 \ud0c4\uc18c\uc5d0 \uc758\ud574 \uc9c0\uc18d\uc801\uc73c\ub85c \ubcf4\ucda9\ub418\uace0 \uc788\ub2e4\uace0 \uc5ec\uaca8\uc9c4\ub2e4. \uc774 \uc774\ub860\uc740 \ud45c\uba74\uc758 \ud639\ub4e4\uc744 \uc124\uba85\ud558\ub294 \uc2b9\ud654-\ubd84\ud574 \uc774\ub860\uacfc \uc5b4\ub290 \uc815\ub3c4 \uc77c\uce58\ud55c\ub2e4.", "paragraph_answer": "\uce7c\ub9ac\uc2a4\ud1a0\ub294 \uc774\uc0b0\ud654 \ud0c4\uc18c\ub85c \uad6c\uc131\ub41c \ub9e4\uc6b0 \ubbf8\uc57d\ud55c \ub300\uae30\uad8c\uc744 \uac00\uc9c0\uace0 \uc788\ub2e4. \uc774\ub294 \uac08\ub9b4\ub808\uc624 \ud0d0\uc0ac\uc120\uc758 \uadfc\uc801\uc678\uc120 \ubd84\uad11 \uc9c0\ub3c4\uc791\uc131\uae30(Near Infrared Mapping Spectrometer, NIMS)\uac00 4.2 \u00b5m \ub300\uc5ed\uc5d0\uc11c \ub300\uae30\uad8c\uc774 \ub9cc\ub4e4\uc5b4 \ub0b8 \ud30c\uc7a5 \ud761\uc218\ub97c \uac10\uc9c0\ud558\uc5ec \ubc1c\uacac\ub41c \uac83\uc774\ub2e4. \uce7c\ub9ac\uc2a4\ud1a0 \ud45c\uba74\uc5d0\uc11c\uc758 \uae30\uc555\uc740 \uc57d 7.5\u00d710^ bar (0.75 \u00b5Pa) \ub85c \ucd94\uc815\ub418\uace0, \uc785\uc790\ub4e4\uc758 \ubc00\ub3c4\ub294 4\u00d710^ cm \uc774\ub2e4. \uc774 \uc815\ub3c4\ub85c \uc605\uc740 \ub300\uae30\uad8c\uc740 4\uc77c \uc774\ub0b4\ub85c \ubaa8\ub450 \uce7c\ub9ac\uc2a4\ud1a0\ub97c \ud0c8\ucd9c\ud574 \ubc84\ub9b4 \uac83\uc774\uae30 \ub54c\ubb38\uc5d0, \ub300\uae30\uad8c\uc740 \uce7c\ub9ac\uc2a4\ud1a0\uc758 \uc5bc\uc74c \uc9c0\uac01 \uc5d0\uc11c \ucc9c\ucc9c\ud788 \uc2b9\ud654\ub418\ub294 \uc774\uc0b0\ud654 \ud0c4\uc18c\uc5d0 \uc758\ud574 \uc9c0\uc18d\uc801\uc73c\ub85c \ubcf4\ucda9\ub418\uace0 \uc788\ub2e4\uace0 \uc5ec\uaca8\uc9c4\ub2e4. \uc774 \uc774\ub860\uc740 \ud45c\uba74\uc758 \ud639\ub4e4\uc744 \uc124\uba85\ud558\ub294 \uc2b9\ud654-\ubd84\ud574 \uc774\ub860\uacfc \uc5b4\ub290 \uc815\ub3c4 \uc77c\uce58\ud55c\ub2e4.", "sentence_answer": "\uc774 \uc815\ub3c4\ub85c \uc605\uc740 \ub300\uae30\uad8c\uc740 4\uc77c \uc774\ub0b4\ub85c \ubaa8\ub450 \uce7c\ub9ac\uc2a4\ud1a0\ub97c \ud0c8\ucd9c\ud574 \ubc84\ub9b4 \uac83\uc774\uae30 \ub54c\ubb38\uc5d0, \ub300\uae30\uad8c\uc740 \uce7c\ub9ac\uc2a4\ud1a0\uc758 \uc5bc\uc74c \uc9c0\uac01 \uc5d0\uc11c \ucc9c\ucc9c\ud788 \uc2b9\ud654\ub418\ub294 \uc774\uc0b0\ud654 \ud0c4\uc18c\uc5d0 \uc758\ud574 \uc9c0\uc18d\uc801\uc73c\ub85c \ubcf4\ucda9\ub418\uace0 \uc788\ub2e4\uace0 \uc5ec\uaca8\uc9c4\ub2e4.", "paragraph_id": "6561307-8-2", "paragraph_question": "question: \uce7c\ub9ac\uc2a4\ud1a0 \ub300\uae30\uad8c\uc744 \uad6c\uc131\ud558\ub294 \uc774\uc0b0\ud654\ud0c4\uc18c\ub97c \uc2b9\ud654\uc2dc\ud0a4\ub294 \ubd80\ubd84\uc740?, context: \uce7c\ub9ac\uc2a4\ud1a0\ub294 \uc774\uc0b0\ud654 \ud0c4\uc18c\ub85c \uad6c\uc131\ub41c \ub9e4\uc6b0 \ubbf8\uc57d\ud55c \ub300\uae30\uad8c\uc744 \uac00\uc9c0\uace0 \uc788\ub2e4. \uc774\ub294 \uac08\ub9b4\ub808\uc624 \ud0d0\uc0ac\uc120\uc758 \uadfc\uc801\uc678\uc120 \ubd84\uad11 \uc9c0\ub3c4\uc791\uc131\uae30(Near Infrared Mapping Spectrometer, NIMS)\uac00 4.2 \u00b5m \ub300\uc5ed\uc5d0\uc11c \ub300\uae30\uad8c\uc774 \ub9cc\ub4e4\uc5b4 \ub0b8 \ud30c\uc7a5 \ud761\uc218\ub97c \uac10\uc9c0\ud558\uc5ec \ubc1c\uacac\ub41c \uac83\uc774\ub2e4. \uce7c\ub9ac\uc2a4\ud1a0 \ud45c\uba74\uc5d0\uc11c\uc758 \uae30\uc555\uc740 \uc57d 7.5\u00d710^ bar (0.75 \u00b5Pa) \ub85c \ucd94\uc815\ub418\uace0, \uc785\uc790\ub4e4\uc758 \ubc00\ub3c4\ub294 4\u00d710^ cm \uc774\ub2e4. \uc774 \uc815\ub3c4\ub85c \uc605\uc740 \ub300\uae30\uad8c\uc740 4\uc77c \uc774\ub0b4\ub85c \ubaa8\ub450 \uce7c\ub9ac\uc2a4\ud1a0\ub97c \ud0c8\ucd9c\ud574 \ubc84\ub9b4 \uac83\uc774\uae30 \ub54c\ubb38\uc5d0, \ub300\uae30\uad8c\uc740 \uce7c\ub9ac\uc2a4\ud1a0\uc758 \uc5bc\uc74c \uc9c0\uac01\uc5d0\uc11c \ucc9c\ucc9c\ud788 \uc2b9\ud654\ub418\ub294 \uc774\uc0b0\ud654 \ud0c4\uc18c\uc5d0 \uc758\ud574 \uc9c0\uc18d\uc801\uc73c\ub85c \ubcf4\ucda9\ub418\uace0 \uc788\ub2e4\uace0 \uc5ec\uaca8\uc9c4\ub2e4. \uc774 \uc774\ub860\uc740 \ud45c\uba74\uc758 \ud639\ub4e4\uc744 \uc124\uba85\ud558\ub294 \uc2b9\ud654-\ubd84\ud574 \uc774\ub860\uacfc \uc5b4\ub290 \uc815\ub3c4 \uc77c\uce58\ud55c\ub2e4."} {"question": "\uc5ec\ub860\uc870\uc0ac \uc5c5\uccb4 \ub9ac\uc5bc\ubbf8\ud130\uc758 \ub300\ud45c\ub294 \ub204\uad6c\uc778\uac00\uc694?", "paragraph": "\uad6d\uc815\uc6d0\uc758 \ub313\uae00, \ud2b8\uc704\ud130\ub4f1\uc744 \ud1b5\ud55c \ub300\uc120 \uac1c\uc785 \ud65c\ub3d9\uc774 \ub300\ud1b5\ub839 \uc120\uac70 \uacb0\uacfc\uc5d0 \ub07c\uce5c \uc601\ud5a5\uc5d0 \ub300\ud574\uc11c\ub294 \uc758\uacac\uc774 \ubd84\ubd84\ud558\ub2e4. \uadf8\ub7ec\ub098 \uc5ec\ub860\uc870\uc0ac \uacb0\uacfc\uc5d0 \ub530\ub974\uba74 \uae40\uc6a9\ud310\uc758 \uc9c0\uc2dc\uc5d0 \ub530\ub978 \uc11c\uc6b8\uc9c0\ubc29\uacbd\ucc30\uccad\uc758 \ud5c8\uc704\uc218\uc0ac\ubc1c\ud45c\ub294 \ub300\ud1b5\ub839 \uc120\uac70 \uacb0\uacfc\uc5d0 \uc601\ud5a5\uc744 \ubbf8\ucce4\ub2e4. \ub9ac\uc11c\uce58\ubdf0\ub294 \uc804\uad6d \ub4dd\ud45c\uc728\uacfc \uac70\uc758 \ube44\uc2b7\ud558\uac8c \ub098\uc628 \uc778\ucc9c\uad11\uc5ed\uc2dc\uc758 \uc2dc\ubbfc\uc744 \ub300\uc0c1\uc73c\ub85c \uc5ec\ub860\uc870\uc0ac\ub97c \ud588\ub2e4. \uacbd\ucc30\uc774 \uc0ac\uc2e4\ub300\ub85c \ubc1c\ud45c\ud588\uc744 \uacbd\uc6b0 \ubc15\uadfc\ud61c \ud22c\ud45c\uce35\uc758 13.8%\uac00 \ubb38\uc7ac\uc778\uc744 \ucc0d\uc5c8\uc744 \uac83\uc774\ub77c\uace0 \uc751\ub2f5\ud588\ub2e4. \uc774\ub294 \uc804\uccb4 \ub4dd\ud45c\uc728\uc758 7.12%p\uc5d0 \ud574\ub2f9\ub41c\ub2e4. \uc5ec\ub860\uc870\uc0ac \uc5c5\uccb4 \ub9ac\uc5bc\ubbf8\ud130\uc758 \uc774\ud0dd\uc218\ub300\ud45c\ub294 \uae40\uc6a9\ud310\uc758 \ud5c8\uc704\uc218\uc0ac\ubc1c\ud45c\uc9c1\uc804 \ubb38\uc7ac\uc778 \ud6c4\ubcf4\uac00 \uc5ec\ub860\uc870\uc0ac\uc5d0\uc11c \ubc15\uadfc\ud61c \ud6c4\ubcf4\uc758 \uc9c0\uc9c0\uc728\uc744 \ucd94\uc6d4\ud558\uc600\uc9c0\ub9cc, \uacbd\ucc30\uc758 \ubc1c\ud45c \uc774\ud6c4 \ub2e4\uc2dc \uc544\ub798\ub85c \ub0b4\ub824\uac14\ub2e4\uace0 \ubc1d\ud614\ub2e4. \ub9ac\uc11c\uce58\ubdf0\uac00 2013\ub144 10\uc6d4 27\uc77c \uc804\uad6d \ub9cc 19\uc138 \uc774\uc0c1 \uc720\uad8c\uc790 1000\uba85\uc744 \ub300\uc0c1\uc73c\ub85c \uc5ec\ub860\uc870\uc0ac\ub97c \ubc8c\uc778 \uacb0\uacfc\ub3c4, \uacbd\ucc30\uc774 \uc81c\ub300\ub85c \ubc1c\ud45c\ud588\uc5c8\ub2e4\uba74 \ub300\uc120 \uacb0\uacfc\uac00 \ubc14\ub00c\uc5c8\uc744 \uac83\uc774\ub77c\ub294 \ucd94\uc815\uc744 \uac00\ub2a5\ud558\uac8c \ud55c\ub2e4. \ubc15\uadfc\ud61c \ud6c4\ubcf4\ub97c \ucc0d\uc5c8\ub358 \ud22c\ud45c\uce35 \uc911 8.3%\ub294 \uacbd\ucc30\uc774 \uc0ac\uc2e4\ub300\ub85c \ubc1c\ud45c\ud588\uc744 \uacbd\uc6b0 \ubb38\uc7ac\uc778 \ud6c4\ubcf4\ub97c \ucc0d\uc5c8\uc744 \uac83\uc774\ub77c\uace0 \uc751\ub2f5\ud588\ub2e4\ub294 \uac83\uc774\ub2e4. \ub9ac\uc11c\uce58\ubdf0 \uce21\uc740 \"\uc774\ub7ec\ud55c \uacb0\uacfc\ub97c \ub450 \ud6c4\ubcf4\uc758 \ucd5c\uc885\ub4dd\ud45c\uc728\uc5d0 \ubc18\uc601\ud560 \uacbd\uc6b0 \ubc15\uadfc\ud61c \ud6c4\ubcf4\ub294 51.55\u219247.27%, \ubb38\uc7ac\uc778 \ud6c4\ubcf4\ub294 48.02%\u219252.3%\ub85c \ub098\ud0c0\ub0ac\ub2e4\"\ub77c\uace0 \ubc1d\ud614\ub2e4.", "answer": "\uc774\ud0dd\uc218", "sentence": "\uc5ec\ub860\uc870\uc0ac \uc5c5\uccb4 \ub9ac\uc5bc\ubbf8\ud130\uc758 \uc774\ud0dd\uc218 \ub300\ud45c\ub294 \uae40\uc6a9\ud310\uc758 \ud5c8\uc704\uc218\uc0ac\ubc1c\ud45c\uc9c1\uc804 \ubb38\uc7ac\uc778 \ud6c4\ubcf4\uac00 \uc5ec\ub860\uc870\uc0ac\uc5d0\uc11c \ubc15\uadfc\ud61c \ud6c4\ubcf4\uc758 \uc9c0\uc9c0\uc728\uc744 \ucd94\uc6d4\ud558\uc600\uc9c0\ub9cc, \uacbd\ucc30\uc758 \ubc1c\ud45c \uc774\ud6c4 \ub2e4\uc2dc \uc544\ub798\ub85c \ub0b4\ub824\uac14\ub2e4\uace0 \ubc1d\ud614\ub2e4.", "paragraph_sentence": "\uad6d\uc815\uc6d0\uc758 \ub313\uae00, \ud2b8\uc704\ud130\ub4f1\uc744 \ud1b5\ud55c \ub300\uc120 \uac1c\uc785 \ud65c\ub3d9\uc774 \ub300\ud1b5\ub839 \uc120\uac70 \uacb0\uacfc\uc5d0 \ub07c\uce5c \uc601\ud5a5\uc5d0 \ub300\ud574\uc11c\ub294 \uc758\uacac\uc774 \ubd84\ubd84\ud558\ub2e4. \uadf8\ub7ec\ub098 \uc5ec\ub860\uc870\uc0ac \uacb0\uacfc\uc5d0 \ub530\ub974\uba74 \uae40\uc6a9\ud310\uc758 \uc9c0\uc2dc\uc5d0 \ub530\ub978 \uc11c\uc6b8\uc9c0\ubc29\uacbd\ucc30\uccad\uc758 \ud5c8\uc704\uc218\uc0ac\ubc1c\ud45c\ub294 \ub300\ud1b5\ub839 \uc120\uac70 \uacb0\uacfc\uc5d0 \uc601\ud5a5\uc744 \ubbf8\ucce4\ub2e4. \ub9ac\uc11c\uce58\ubdf0\ub294 \uc804\uad6d \ub4dd\ud45c\uc728\uacfc \uac70\uc758 \ube44\uc2b7\ud558\uac8c \ub098\uc628 \uc778\ucc9c\uad11\uc5ed\uc2dc\uc758 \uc2dc\ubbfc\uc744 \ub300\uc0c1\uc73c\ub85c \uc5ec\ub860\uc870\uc0ac\ub97c \ud588\ub2e4. \uacbd\ucc30\uc774 \uc0ac\uc2e4\ub300\ub85c \ubc1c\ud45c\ud588\uc744 \uacbd\uc6b0 \ubc15\uadfc\ud61c \ud22c\ud45c\uce35\uc758 13.8%\uac00 \ubb38\uc7ac\uc778\uc744 \ucc0d\uc5c8\uc744 \uac83\uc774\ub77c\uace0 \uc751\ub2f5\ud588\ub2e4. \uc774\ub294 \uc804\uccb4 \ub4dd\ud45c\uc728\uc758 7.12%p\uc5d0 \ud574\ub2f9\ub41c\ub2e4. \uc5ec\ub860\uc870\uc0ac \uc5c5\uccb4 \ub9ac\uc5bc\ubbf8\ud130\uc758 \uc774\ud0dd\uc218 \ub300\ud45c\ub294 \uae40\uc6a9\ud310\uc758 \ud5c8\uc704\uc218\uc0ac\ubc1c\ud45c\uc9c1\uc804 \ubb38\uc7ac\uc778 \ud6c4\ubcf4\uac00 \uc5ec\ub860\uc870\uc0ac\uc5d0\uc11c \ubc15\uadfc\ud61c \ud6c4\ubcf4\uc758 \uc9c0\uc9c0\uc728\uc744 \ucd94\uc6d4\ud558\uc600\uc9c0\ub9cc, \uacbd\ucc30\uc758 \ubc1c\ud45c \uc774\ud6c4 \ub2e4\uc2dc \uc544\ub798\ub85c \ub0b4\ub824\uac14\ub2e4\uace0 \ubc1d\ud614\ub2e4. \ub9ac\uc11c\uce58\ubdf0\uac00 2013\ub144 10\uc6d4 27\uc77c \uc804\uad6d \ub9cc 19\uc138 \uc774\uc0c1 \uc720\uad8c\uc790 1000\uba85\uc744 \ub300\uc0c1\uc73c\ub85c \uc5ec\ub860\uc870\uc0ac\ub97c \ubc8c\uc778 \uacb0\uacfc\ub3c4, \uacbd\ucc30\uc774 \uc81c\ub300\ub85c \ubc1c\ud45c\ud588\uc5c8\ub2e4\uba74 \ub300\uc120 \uacb0\uacfc\uac00 \ubc14\ub00c\uc5c8\uc744 \uac83\uc774\ub77c\ub294 \ucd94\uc815\uc744 \uac00\ub2a5\ud558\uac8c \ud55c\ub2e4. \ubc15\uadfc\ud61c \ud6c4\ubcf4\ub97c \ucc0d\uc5c8\ub358 \ud22c\ud45c\uce35 \uc911 8.3%\ub294 \uacbd\ucc30\uc774 \uc0ac\uc2e4\ub300\ub85c \ubc1c\ud45c\ud588\uc744 \uacbd\uc6b0 \ubb38\uc7ac\uc778 \ud6c4\ubcf4\ub97c \ucc0d\uc5c8\uc744 \uac83\uc774\ub77c\uace0 \uc751\ub2f5\ud588\ub2e4\ub294 \uac83\uc774\ub2e4. \ub9ac\uc11c\uce58\ubdf0 \uce21\uc740 \"\uc774\ub7ec\ud55c \uacb0\uacfc\ub97c \ub450 \ud6c4\ubcf4\uc758 \ucd5c\uc885\ub4dd\ud45c\uc728\uc5d0 \ubc18\uc601\ud560 \uacbd\uc6b0 \ubc15\uadfc\ud61c \ud6c4\ubcf4\ub294 51.55\u219247.27%, \ubb38\uc7ac\uc778 \ud6c4\ubcf4\ub294 48.02%\u219252.3%\ub85c \ub098\ud0c0\ub0ac\ub2e4\"\ub77c\uace0 \ubc1d\ud614\ub2e4.", "paragraph_answer": "\uad6d\uc815\uc6d0\uc758 \ub313\uae00, \ud2b8\uc704\ud130\ub4f1\uc744 \ud1b5\ud55c \ub300\uc120 \uac1c\uc785 \ud65c\ub3d9\uc774 \ub300\ud1b5\ub839 \uc120\uac70 \uacb0\uacfc\uc5d0 \ub07c\uce5c \uc601\ud5a5\uc5d0 \ub300\ud574\uc11c\ub294 \uc758\uacac\uc774 \ubd84\ubd84\ud558\ub2e4. \uadf8\ub7ec\ub098 \uc5ec\ub860\uc870\uc0ac \uacb0\uacfc\uc5d0 \ub530\ub974\uba74 \uae40\uc6a9\ud310\uc758 \uc9c0\uc2dc\uc5d0 \ub530\ub978 \uc11c\uc6b8\uc9c0\ubc29\uacbd\ucc30\uccad\uc758 \ud5c8\uc704\uc218\uc0ac\ubc1c\ud45c\ub294 \ub300\ud1b5\ub839 \uc120\uac70 \uacb0\uacfc\uc5d0 \uc601\ud5a5\uc744 \ubbf8\ucce4\ub2e4. \ub9ac\uc11c\uce58\ubdf0\ub294 \uc804\uad6d \ub4dd\ud45c\uc728\uacfc \uac70\uc758 \ube44\uc2b7\ud558\uac8c \ub098\uc628 \uc778\ucc9c\uad11\uc5ed\uc2dc\uc758 \uc2dc\ubbfc\uc744 \ub300\uc0c1\uc73c\ub85c \uc5ec\ub860\uc870\uc0ac\ub97c \ud588\ub2e4. \uacbd\ucc30\uc774 \uc0ac\uc2e4\ub300\ub85c \ubc1c\ud45c\ud588\uc744 \uacbd\uc6b0 \ubc15\uadfc\ud61c \ud22c\ud45c\uce35\uc758 13.8%\uac00 \ubb38\uc7ac\uc778\uc744 \ucc0d\uc5c8\uc744 \uac83\uc774\ub77c\uace0 \uc751\ub2f5\ud588\ub2e4. \uc774\ub294 \uc804\uccb4 \ub4dd\ud45c\uc728\uc758 7.12%p\uc5d0 \ud574\ub2f9\ub41c\ub2e4. \uc5ec\ub860\uc870\uc0ac \uc5c5\uccb4 \ub9ac\uc5bc\ubbf8\ud130\uc758 \uc774\ud0dd\uc218 \ub300\ud45c\ub294 \uae40\uc6a9\ud310\uc758 \ud5c8\uc704\uc218\uc0ac\ubc1c\ud45c\uc9c1\uc804 \ubb38\uc7ac\uc778 \ud6c4\ubcf4\uac00 \uc5ec\ub860\uc870\uc0ac\uc5d0\uc11c \ubc15\uadfc\ud61c \ud6c4\ubcf4\uc758 \uc9c0\uc9c0\uc728\uc744 \ucd94\uc6d4\ud558\uc600\uc9c0\ub9cc, \uacbd\ucc30\uc758 \ubc1c\ud45c \uc774\ud6c4 \ub2e4\uc2dc \uc544\ub798\ub85c \ub0b4\ub824\uac14\ub2e4\uace0 \ubc1d\ud614\ub2e4. \ub9ac\uc11c\uce58\ubdf0\uac00 2013\ub144 10\uc6d4 27\uc77c \uc804\uad6d \ub9cc 19\uc138 \uc774\uc0c1 \uc720\uad8c\uc790 1000\uba85\uc744 \ub300\uc0c1\uc73c\ub85c \uc5ec\ub860\uc870\uc0ac\ub97c \ubc8c\uc778 \uacb0\uacfc\ub3c4, \uacbd\ucc30\uc774 \uc81c\ub300\ub85c \ubc1c\ud45c\ud588\uc5c8\ub2e4\uba74 \ub300\uc120 \uacb0\uacfc\uac00 \ubc14\ub00c\uc5c8\uc744 \uac83\uc774\ub77c\ub294 \ucd94\uc815\uc744 \uac00\ub2a5\ud558\uac8c \ud55c\ub2e4. \ubc15\uadfc\ud61c \ud6c4\ubcf4\ub97c \ucc0d\uc5c8\ub358 \ud22c\ud45c\uce35 \uc911 8.3%\ub294 \uacbd\ucc30\uc774 \uc0ac\uc2e4\ub300\ub85c \ubc1c\ud45c\ud588\uc744 \uacbd\uc6b0 \ubb38\uc7ac\uc778 \ud6c4\ubcf4\ub97c \ucc0d\uc5c8\uc744 \uac83\uc774\ub77c\uace0 \uc751\ub2f5\ud588\ub2e4\ub294 \uac83\uc774\ub2e4. \ub9ac\uc11c\uce58\ubdf0 \uce21\uc740 \"\uc774\ub7ec\ud55c \uacb0\uacfc\ub97c \ub450 \ud6c4\ubcf4\uc758 \ucd5c\uc885\ub4dd\ud45c\uc728\uc5d0 \ubc18\uc601\ud560 \uacbd\uc6b0 \ubc15\uadfc\ud61c \ud6c4\ubcf4\ub294 51.55\u219247.27%, \ubb38\uc7ac\uc778 \ud6c4\ubcf4\ub294 48.02%\u219252.3%\ub85c \ub098\ud0c0\ub0ac\ub2e4\"\ub77c\uace0 \ubc1d\ud614\ub2e4.", "sentence_answer": "\uc5ec\ub860\uc870\uc0ac \uc5c5\uccb4 \ub9ac\uc5bc\ubbf8\ud130\uc758 \uc774\ud0dd\uc218 \ub300\ud45c\ub294 \uae40\uc6a9\ud310\uc758 \ud5c8\uc704\uc218\uc0ac\ubc1c\ud45c\uc9c1\uc804 \ubb38\uc7ac\uc778 \ud6c4\ubcf4\uac00 \uc5ec\ub860\uc870\uc0ac\uc5d0\uc11c \ubc15\uadfc\ud61c \ud6c4\ubcf4\uc758 \uc9c0\uc9c0\uc728\uc744 \ucd94\uc6d4\ud558\uc600\uc9c0\ub9cc, \uacbd\ucc30\uc758 \ubc1c\ud45c \uc774\ud6c4 \ub2e4\uc2dc \uc544\ub798\ub85c \ub0b4\ub824\uac14\ub2e4\uace0 \ubc1d\ud614\ub2e4.", "paragraph_id": "6506520-21-0", "paragraph_question": "question: \uc5ec\ub860\uc870\uc0ac \uc5c5\uccb4 \ub9ac\uc5bc\ubbf8\ud130\uc758 \ub300\ud45c\ub294 \ub204\uad6c\uc778\uac00\uc694?, context: \uad6d\uc815\uc6d0\uc758 \ub313\uae00, \ud2b8\uc704\ud130\ub4f1\uc744 \ud1b5\ud55c \ub300\uc120 \uac1c\uc785 \ud65c\ub3d9\uc774 \ub300\ud1b5\ub839 \uc120\uac70 \uacb0\uacfc\uc5d0 \ub07c\uce5c \uc601\ud5a5\uc5d0 \ub300\ud574\uc11c\ub294 \uc758\uacac\uc774 \ubd84\ubd84\ud558\ub2e4. \uadf8\ub7ec\ub098 \uc5ec\ub860\uc870\uc0ac \uacb0\uacfc\uc5d0 \ub530\ub974\uba74 \uae40\uc6a9\ud310\uc758 \uc9c0\uc2dc\uc5d0 \ub530\ub978 \uc11c\uc6b8\uc9c0\ubc29\uacbd\ucc30\uccad\uc758 \ud5c8\uc704\uc218\uc0ac\ubc1c\ud45c\ub294 \ub300\ud1b5\ub839 \uc120\uac70 \uacb0\uacfc\uc5d0 \uc601\ud5a5\uc744 \ubbf8\ucce4\ub2e4. \ub9ac\uc11c\uce58\ubdf0\ub294 \uc804\uad6d \ub4dd\ud45c\uc728\uacfc \uac70\uc758 \ube44\uc2b7\ud558\uac8c \ub098\uc628 \uc778\ucc9c\uad11\uc5ed\uc2dc\uc758 \uc2dc\ubbfc\uc744 \ub300\uc0c1\uc73c\ub85c \uc5ec\ub860\uc870\uc0ac\ub97c \ud588\ub2e4. \uacbd\ucc30\uc774 \uc0ac\uc2e4\ub300\ub85c \ubc1c\ud45c\ud588\uc744 \uacbd\uc6b0 \ubc15\uadfc\ud61c \ud22c\ud45c\uce35\uc758 13.8%\uac00 \ubb38\uc7ac\uc778\uc744 \ucc0d\uc5c8\uc744 \uac83\uc774\ub77c\uace0 \uc751\ub2f5\ud588\ub2e4. \uc774\ub294 \uc804\uccb4 \ub4dd\ud45c\uc728\uc758 7.12%p\uc5d0 \ud574\ub2f9\ub41c\ub2e4. \uc5ec\ub860\uc870\uc0ac \uc5c5\uccb4 \ub9ac\uc5bc\ubbf8\ud130\uc758 \uc774\ud0dd\uc218\ub300\ud45c\ub294 \uae40\uc6a9\ud310\uc758 \ud5c8\uc704\uc218\uc0ac\ubc1c\ud45c\uc9c1\uc804 \ubb38\uc7ac\uc778 \ud6c4\ubcf4\uac00 \uc5ec\ub860\uc870\uc0ac\uc5d0\uc11c \ubc15\uadfc\ud61c \ud6c4\ubcf4\uc758 \uc9c0\uc9c0\uc728\uc744 \ucd94\uc6d4\ud558\uc600\uc9c0\ub9cc, \uacbd\ucc30\uc758 \ubc1c\ud45c \uc774\ud6c4 \ub2e4\uc2dc \uc544\ub798\ub85c \ub0b4\ub824\uac14\ub2e4\uace0 \ubc1d\ud614\ub2e4. \ub9ac\uc11c\uce58\ubdf0\uac00 2013\ub144 10\uc6d4 27\uc77c \uc804\uad6d \ub9cc 19\uc138 \uc774\uc0c1 \uc720\uad8c\uc790 1000\uba85\uc744 \ub300\uc0c1\uc73c\ub85c \uc5ec\ub860\uc870\uc0ac\ub97c \ubc8c\uc778 \uacb0\uacfc\ub3c4, \uacbd\ucc30\uc774 \uc81c\ub300\ub85c \ubc1c\ud45c\ud588\uc5c8\ub2e4\uba74 \ub300\uc120 \uacb0\uacfc\uac00 \ubc14\ub00c\uc5c8\uc744 \uac83\uc774\ub77c\ub294 \ucd94\uc815\uc744 \uac00\ub2a5\ud558\uac8c \ud55c\ub2e4. \ubc15\uadfc\ud61c \ud6c4\ubcf4\ub97c \ucc0d\uc5c8\ub358 \ud22c\ud45c\uce35 \uc911 8.3%\ub294 \uacbd\ucc30\uc774 \uc0ac\uc2e4\ub300\ub85c \ubc1c\ud45c\ud588\uc744 \uacbd\uc6b0 \ubb38\uc7ac\uc778 \ud6c4\ubcf4\ub97c \ucc0d\uc5c8\uc744 \uac83\uc774\ub77c\uace0 \uc751\ub2f5\ud588\ub2e4\ub294 \uac83\uc774\ub2e4. \ub9ac\uc11c\uce58\ubdf0 \uce21\uc740 \"\uc774\ub7ec\ud55c \uacb0\uacfc\ub97c \ub450 \ud6c4\ubcf4\uc758 \ucd5c\uc885\ub4dd\ud45c\uc728\uc5d0 \ubc18\uc601\ud560 \uacbd\uc6b0 \ubc15\uadfc\ud61c \ud6c4\ubcf4\ub294 51.55\u219247.27%, \ubb38\uc7ac\uc778 \ud6c4\ubcf4\ub294 48.02%\u219252.3%\ub85c \ub098\ud0c0\ub0ac\ub2e4\"\ub77c\uace0 \ubc1d\ud614\ub2e4."} {"question": "\uc911\uad6d\uc778\ub4e4\uc774 \uce90\ub098\ub2e4\ub85c \uc774\ubbfc\uc624\ub294 \ub300\ubd80\ubd84\uc758 \uc720\ud615\uc744 \uae08\uc9c0\ud55c \uc5f0\ub3c4\ub294?", "paragraph": "\uc911\uad6d\uc778 \ubc30\ucc99\ubc95\uc740 \ub610\ud55c \uc774 \uae30\uac04\ub3d9\uc548 \ubbf8\uad6d-\uce90\ub098\ub2e4 \uad6d\uacbd\uc5d0\uc11c\uc758 \uacbd\uacc4 \ubcf4\uac15\uc758 \ud544\uc694\uc131\uc73c\ub85c \uc778\ud574 \uc911\uad6d\uc778 \ubc30\ucc99\uc5d0 \ub300\ud55c \uce90\ub098\ub2e4 \uc815\ucc45\uc758 \uc601\ud5a5\ub825\uc744 \ud1b5\ud574 \ubbf8\uad6d \uc774\ubbfc\ubc95\uc758 \ud30c\uc6cc \ud655\uc7a5\uc73c\ub85c \uc774\uc5b4\uc84c\ub2e4. \ubbf8\uad6d\uc758 \uc911\uad6d\uc778 \ubc30\ucc99\ubc95 \uc9c1\ud6c4, \uce90\ub098\ub2e4\ub294 \uc790\uad6d\uc5d0 \uc785\uad6d\ud558\ub294 \uc911\uad6d\uc778 \uc774\ubbfc\uc790\ub4e4\uc5d0\uac8c \uc778\ub450\uc138\ub97c \ubd80\uacfc\ud558\ub294 1885\ub144 \uc911\uad6d\uc778 \uc774\ubbfc\ubc95\uc744 \ud1b5\uacfc\uc2dc\ucf30\ub2e4. \ubbf8\uad6d \uc815\ubd80\uc758 \uc99d\uc9c4\ub418\ub294 \uc555\ub825 \uc774\ud6c4, \uce90\ub098\ub2e4\ub294 \ub9c8\uce68\ub0b4 \uc911\uad6d\uc778 \uc774\ubbfc\ubc95\uc744 \ub9cc\ub4e4\uc5c8\ub2e4. 1923\ub144 \uc911\uad6d\uc778\ub4e4\uc774 \uce90\ub098\ub2e4\ub85c \uc774\ubbfc\uc624\ub294 \ub300\ubd80\ubd84\uc758 \uc720\ud615\uc744 \uae08\uc9c0\uc2dc\ucf30\ub2e4. \ub610\ud55c \ubbf8\uad6d-\uba55\uc2dc\ucf54 \uad6d\uacbd\uc744 \ub530\ub77c, \uc774\ub7ec\ud55c \uc885\ub958\uc758 \uad6d\uacbd \ud1b5\uc81c\uc758 \ud544\uc694\uc131\uc774 \ub300\ub450\ub418\uc5c8\uc9c0\ub9cc, \uad6d\uacbd\uc744 \ud1b5\uc81c\ud558\ub824\ub294 \ub178\ub825\uc740 \ubbf8\uad6d\uc758 \uc81c\uad6d\uc8fc\uc758\ub97c \uacbd\uacc4\ud558\uace0 \uc788\uc5c8\uae30 \ub54c\ubb38\uc5d0 \uc5c7\ub098\uac14\uc73c\uba70, \uba55\uc2dc\ucf54 \ub0b4\uc758 \ubbf8\uad6d\uc758 \uac04\uc12d\uc744 \uc6d0\ud558\uc9c0 \uc54a\uc558\ub2e4. \uc774\uac83 \ubfd0\ub9cc \uc544\ub2c8\ub77c \uba55\uc2dc\ucf54\ub85c \uc774\ubbfc\uc744 \uac00\ub294 \uc911\uad6d\uc778\ub4e4\uc740 \uba55\uc2dc\ucf54\uc758 \ub178\ub3d9\ub825 \ubd80\uc871\uc744 \ucc44\uc6cc\uc8fc\uc5c8\uae30 \ub54c\ubb38\uc5d0 \ud658\uc601\uc744 \ubc1b\uc558\ub2e4. \uc911\uad6d\uc778 \ubc30\ucc99\ubc95\uc740 \uc2e4\uc81c\ub85c \uba55\uc2dc\ucf54 \uc774\ubbfc\uc790\ub4e4\uc758 \uc218\ub97c \ub192\uc600\ub2e4. \ub530\ub77c\uc11c, \ubbf8\uad6d\uc740 \uba55\uc2dc\ucf54 \uad6d\uacbd\uc758 \uce58\uc548\uc5d0 \uc804\uc801\uc73c\ub85c \uc758\uc9c0\ud558\uace0 \uc788\uc5c8\ub2e4.", "answer": "1923\ub144", "sentence": "1923\ub144 \uc911\uad6d\uc778\ub4e4\uc774 \uce90\ub098\ub2e4\ub85c \uc774\ubbfc\uc624\ub294 \ub300\ubd80\ubd84\uc758 \uc720\ud615\uc744 \uae08\uc9c0\uc2dc\ucf30\ub2e4.", "paragraph_sentence": "\uc911\uad6d\uc778 \ubc30\ucc99\ubc95\uc740 \ub610\ud55c \uc774 \uae30\uac04\ub3d9\uc548 \ubbf8\uad6d-\uce90\ub098\ub2e4 \uad6d\uacbd\uc5d0\uc11c\uc758 \uacbd\uacc4 \ubcf4\uac15\uc758 \ud544\uc694\uc131\uc73c\ub85c \uc778\ud574 \uc911\uad6d\uc778 \ubc30\ucc99\uc5d0 \ub300\ud55c \uce90\ub098\ub2e4 \uc815\ucc45\uc758 \uc601\ud5a5\ub825\uc744 \ud1b5\ud574 \ubbf8\uad6d \uc774\ubbfc\ubc95\uc758 \ud30c\uc6cc \ud655\uc7a5\uc73c\ub85c \uc774\uc5b4\uc84c\ub2e4. \ubbf8\uad6d\uc758 \uc911\uad6d\uc778 \ubc30\ucc99\ubc95 \uc9c1\ud6c4, \uce90\ub098\ub2e4\ub294 \uc790\uad6d\uc5d0 \uc785\uad6d\ud558\ub294 \uc911\uad6d\uc778 \uc774\ubbfc\uc790\ub4e4\uc5d0\uac8c \uc778\ub450\uc138\ub97c \ubd80\uacfc\ud558\ub294 1885\ub144 \uc911\uad6d\uc778 \uc774\ubbfc\ubc95\uc744 \ud1b5\uacfc\uc2dc\ucf30\ub2e4. \ubbf8\uad6d \uc815\ubd80\uc758 \uc99d\uc9c4\ub418\ub294 \uc555\ub825 \uc774\ud6c4, \uce90\ub098\ub2e4\ub294 \ub9c8\uce68\ub0b4 \uc911\uad6d\uc778 \uc774\ubbfc\ubc95\uc744 \ub9cc\ub4e4\uc5c8\ub2e4. 1923\ub144 \uc911\uad6d\uc778\ub4e4\uc774 \uce90\ub098\ub2e4\ub85c \uc774\ubbfc\uc624\ub294 \ub300\ubd80\ubd84\uc758 \uc720\ud615\uc744 \uae08\uc9c0\uc2dc\ucf30\ub2e4. \ub610\ud55c \ubbf8\uad6d-\uba55\uc2dc\ucf54 \uad6d\uacbd\uc744 \ub530\ub77c, \uc774\ub7ec\ud55c \uc885\ub958\uc758 \uad6d\uacbd \ud1b5\uc81c\uc758 \ud544\uc694\uc131\uc774 \ub300\ub450\ub418\uc5c8\uc9c0\ub9cc, \uad6d\uacbd\uc744 \ud1b5\uc81c\ud558\ub824\ub294 \ub178\ub825\uc740 \ubbf8\uad6d\uc758 \uc81c\uad6d\uc8fc\uc758\ub97c \uacbd\uacc4\ud558\uace0 \uc788\uc5c8\uae30 \ub54c\ubb38\uc5d0 \uc5c7\ub098\uac14\uc73c\uba70, \uba55\uc2dc\ucf54 \ub0b4\uc758 \ubbf8\uad6d\uc758 \uac04\uc12d\uc744 \uc6d0\ud558\uc9c0 \uc54a\uc558\ub2e4. \uc774\uac83 \ubfd0\ub9cc \uc544\ub2c8\ub77c \uba55\uc2dc\ucf54\ub85c \uc774\ubbfc\uc744 \uac00\ub294 \uc911\uad6d\uc778\ub4e4\uc740 \uba55\uc2dc\ucf54\uc758 \ub178\ub3d9\ub825 \ubd80\uc871\uc744 \ucc44\uc6cc\uc8fc\uc5c8\uae30 \ub54c\ubb38\uc5d0 \ud658\uc601\uc744 \ubc1b\uc558\ub2e4. \uc911\uad6d\uc778 \ubc30\ucc99\ubc95\uc740 \uc2e4\uc81c\ub85c \uba55\uc2dc\ucf54 \uc774\ubbfc\uc790\ub4e4\uc758 \uc218\ub97c \ub192\uc600\ub2e4. \ub530\ub77c\uc11c, \ubbf8\uad6d\uc740 \uba55\uc2dc\ucf54 \uad6d\uacbd\uc758 \uce58\uc548\uc5d0 \uc804\uc801\uc73c\ub85c \uc758\uc9c0\ud558\uace0 \uc788\uc5c8\ub2e4.", "paragraph_answer": "\uc911\uad6d\uc778 \ubc30\ucc99\ubc95\uc740 \ub610\ud55c \uc774 \uae30\uac04\ub3d9\uc548 \ubbf8\uad6d-\uce90\ub098\ub2e4 \uad6d\uacbd\uc5d0\uc11c\uc758 \uacbd\uacc4 \ubcf4\uac15\uc758 \ud544\uc694\uc131\uc73c\ub85c \uc778\ud574 \uc911\uad6d\uc778 \ubc30\ucc99\uc5d0 \ub300\ud55c \uce90\ub098\ub2e4 \uc815\ucc45\uc758 \uc601\ud5a5\ub825\uc744 \ud1b5\ud574 \ubbf8\uad6d \uc774\ubbfc\ubc95\uc758 \ud30c\uc6cc \ud655\uc7a5\uc73c\ub85c \uc774\uc5b4\uc84c\ub2e4. \ubbf8\uad6d\uc758 \uc911\uad6d\uc778 \ubc30\ucc99\ubc95 \uc9c1\ud6c4, \uce90\ub098\ub2e4\ub294 \uc790\uad6d\uc5d0 \uc785\uad6d\ud558\ub294 \uc911\uad6d\uc778 \uc774\ubbfc\uc790\ub4e4\uc5d0\uac8c \uc778\ub450\uc138\ub97c \ubd80\uacfc\ud558\ub294 1885\ub144 \uc911\uad6d\uc778 \uc774\ubbfc\ubc95\uc744 \ud1b5\uacfc\uc2dc\ucf30\ub2e4. \ubbf8\uad6d \uc815\ubd80\uc758 \uc99d\uc9c4\ub418\ub294 \uc555\ub825 \uc774\ud6c4, \uce90\ub098\ub2e4\ub294 \ub9c8\uce68\ub0b4 \uc911\uad6d\uc778 \uc774\ubbfc\ubc95\uc744 \ub9cc\ub4e4\uc5c8\ub2e4. 1923\ub144 \uc911\uad6d\uc778\ub4e4\uc774 \uce90\ub098\ub2e4\ub85c \uc774\ubbfc\uc624\ub294 \ub300\ubd80\ubd84\uc758 \uc720\ud615\uc744 \uae08\uc9c0\uc2dc\ucf30\ub2e4. \ub610\ud55c \ubbf8\uad6d-\uba55\uc2dc\ucf54 \uad6d\uacbd\uc744 \ub530\ub77c, \uc774\ub7ec\ud55c \uc885\ub958\uc758 \uad6d\uacbd \ud1b5\uc81c\uc758 \ud544\uc694\uc131\uc774 \ub300\ub450\ub418\uc5c8\uc9c0\ub9cc, \uad6d\uacbd\uc744 \ud1b5\uc81c\ud558\ub824\ub294 \ub178\ub825\uc740 \ubbf8\uad6d\uc758 \uc81c\uad6d\uc8fc\uc758\ub97c \uacbd\uacc4\ud558\uace0 \uc788\uc5c8\uae30 \ub54c\ubb38\uc5d0 \uc5c7\ub098\uac14\uc73c\uba70, \uba55\uc2dc\ucf54 \ub0b4\uc758 \ubbf8\uad6d\uc758 \uac04\uc12d\uc744 \uc6d0\ud558\uc9c0 \uc54a\uc558\ub2e4. \uc774\uac83 \ubfd0\ub9cc \uc544\ub2c8\ub77c \uba55\uc2dc\ucf54\ub85c \uc774\ubbfc\uc744 \uac00\ub294 \uc911\uad6d\uc778\ub4e4\uc740 \uba55\uc2dc\ucf54\uc758 \ub178\ub3d9\ub825 \ubd80\uc871\uc744 \ucc44\uc6cc\uc8fc\uc5c8\uae30 \ub54c\ubb38\uc5d0 \ud658\uc601\uc744 \ubc1b\uc558\ub2e4. \uc911\uad6d\uc778 \ubc30\ucc99\ubc95\uc740 \uc2e4\uc81c\ub85c \uba55\uc2dc\ucf54 \uc774\ubbfc\uc790\ub4e4\uc758 \uc218\ub97c \ub192\uc600\ub2e4. \ub530\ub77c\uc11c, \ubbf8\uad6d\uc740 \uba55\uc2dc\ucf54 \uad6d\uacbd\uc758 \uce58\uc548\uc5d0 \uc804\uc801\uc73c\ub85c \uc758\uc9c0\ud558\uace0 \uc788\uc5c8\ub2e4.", "sentence_answer": " 1923\ub144 \uc911\uad6d\uc778\ub4e4\uc774 \uce90\ub098\ub2e4\ub85c \uc774\ubbfc\uc624\ub294 \ub300\ubd80\ubd84\uc758 \uc720\ud615\uc744 \uae08\uc9c0\uc2dc\ucf30\ub2e4.", "paragraph_id": "6596153-2-2", "paragraph_question": "question: \uc911\uad6d\uc778\ub4e4\uc774 \uce90\ub098\ub2e4\ub85c \uc774\ubbfc\uc624\ub294 \ub300\ubd80\ubd84\uc758 \uc720\ud615\uc744 \uae08\uc9c0\ud55c \uc5f0\ub3c4\ub294?, context: \uc911\uad6d\uc778 \ubc30\ucc99\ubc95\uc740 \ub610\ud55c \uc774 \uae30\uac04\ub3d9\uc548 \ubbf8\uad6d-\uce90\ub098\ub2e4 \uad6d\uacbd\uc5d0\uc11c\uc758 \uacbd\uacc4 \ubcf4\uac15\uc758 \ud544\uc694\uc131\uc73c\ub85c \uc778\ud574 \uc911\uad6d\uc778 \ubc30\ucc99\uc5d0 \ub300\ud55c \uce90\ub098\ub2e4 \uc815\ucc45\uc758 \uc601\ud5a5\ub825\uc744 \ud1b5\ud574 \ubbf8\uad6d \uc774\ubbfc\ubc95\uc758 \ud30c\uc6cc \ud655\uc7a5\uc73c\ub85c \uc774\uc5b4\uc84c\ub2e4. \ubbf8\uad6d\uc758 \uc911\uad6d\uc778 \ubc30\ucc99\ubc95 \uc9c1\ud6c4, \uce90\ub098\ub2e4\ub294 \uc790\uad6d\uc5d0 \uc785\uad6d\ud558\ub294 \uc911\uad6d\uc778 \uc774\ubbfc\uc790\ub4e4\uc5d0\uac8c \uc778\ub450\uc138\ub97c \ubd80\uacfc\ud558\ub294 1885\ub144 \uc911\uad6d\uc778 \uc774\ubbfc\ubc95\uc744 \ud1b5\uacfc\uc2dc\ucf30\ub2e4. \ubbf8\uad6d \uc815\ubd80\uc758 \uc99d\uc9c4\ub418\ub294 \uc555\ub825 \uc774\ud6c4, \uce90\ub098\ub2e4\ub294 \ub9c8\uce68\ub0b4 \uc911\uad6d\uc778 \uc774\ubbfc\ubc95\uc744 \ub9cc\ub4e4\uc5c8\ub2e4. 1923\ub144 \uc911\uad6d\uc778\ub4e4\uc774 \uce90\ub098\ub2e4\ub85c \uc774\ubbfc\uc624\ub294 \ub300\ubd80\ubd84\uc758 \uc720\ud615\uc744 \uae08\uc9c0\uc2dc\ucf30\ub2e4. \ub610\ud55c \ubbf8\uad6d-\uba55\uc2dc\ucf54 \uad6d\uacbd\uc744 \ub530\ub77c, \uc774\ub7ec\ud55c \uc885\ub958\uc758 \uad6d\uacbd \ud1b5\uc81c\uc758 \ud544\uc694\uc131\uc774 \ub300\ub450\ub418\uc5c8\uc9c0\ub9cc, \uad6d\uacbd\uc744 \ud1b5\uc81c\ud558\ub824\ub294 \ub178\ub825\uc740 \ubbf8\uad6d\uc758 \uc81c\uad6d\uc8fc\uc758\ub97c \uacbd\uacc4\ud558\uace0 \uc788\uc5c8\uae30 \ub54c\ubb38\uc5d0 \uc5c7\ub098\uac14\uc73c\uba70, \uba55\uc2dc\ucf54 \ub0b4\uc758 \ubbf8\uad6d\uc758 \uac04\uc12d\uc744 \uc6d0\ud558\uc9c0 \uc54a\uc558\ub2e4. \uc774\uac83 \ubfd0\ub9cc \uc544\ub2c8\ub77c \uba55\uc2dc\ucf54\ub85c \uc774\ubbfc\uc744 \uac00\ub294 \uc911\uad6d\uc778\ub4e4\uc740 \uba55\uc2dc\ucf54\uc758 \ub178\ub3d9\ub825 \ubd80\uc871\uc744 \ucc44\uc6cc\uc8fc\uc5c8\uae30 \ub54c\ubb38\uc5d0 \ud658\uc601\uc744 \ubc1b\uc558\ub2e4. \uc911\uad6d\uc778 \ubc30\ucc99\ubc95\uc740 \uc2e4\uc81c\ub85c \uba55\uc2dc\ucf54 \uc774\ubbfc\uc790\ub4e4\uc758 \uc218\ub97c \ub192\uc600\ub2e4. \ub530\ub77c\uc11c, \ubbf8\uad6d\uc740 \uba55\uc2dc\ucf54 \uad6d\uacbd\uc758 \uce58\uc548\uc5d0 \uc804\uc801\uc73c\ub85c \uc758\uc9c0\ud558\uace0 \uc788\uc5c8\ub2e4."} {"question": "\ubc18\ubd09\uac74\uc801\uc774\uac70\ub098 \ud601\uba85\uc801\uc778 \uc131\uaca9\uc744 \ub744\uc9c0 \uc54a\uc558\uc73c\uba70 \uccad\ub098\ub77c \ub18d\ubbfc\ub4e4\uc5d0 \uc758\ud574 \uc790\ubc1c\uc801\uc73c\ub85c \ubc8c\uc5b4\uc9c4 \ubc18\uc81c\uad6d\uc8fc\uc758\uc6b4\ub3d9\uc740?", "paragraph": "\uc758\ud654\ub2e8 \uc6b4\ub3d9\uc740 \uac00\ub09c\ud55c \ub18d\ubbfc\uc774 \uc8fc\uccb4\uac00 \ub418\uc5b4 \uc790\ubc1c\uc801\uc73c\ub85c \uc77c\uc5b4\ub09c \ubc18\uc81c\uad6d\uc8fc\uc758 \uc560\uad6d\uc6b4\ub3d9\uc774\uc5c8\uc73c\ub098 \ubc18\ubd09\uac74\uc801\uc774\uac70\ub098 \ud601\uba85\uc801\uc778 \uc131\uaca9\uc740 \uc5c6\uc5c8\ub2e4. \ube44\ub85d \uc2e4\ud328\ub85c \ub05d\ub0ac\uc9c0\ub9cc \uadf8 \uc758\uc758\ub294 \uc0c1\ub2f9\ud788 \ucef8\ub2e4. \uccab\uc9f8, \uc758\ud654\ub2e8 \uc6b4\ub3d9\uc758 \uacb0\uacfc \uc911\uad6d\uc758 \ubbfc\uc871\uc8fc\uc758\uac00 \ud0c4\uc0dd\ud558\uc600\ub2e4. \uc774\ub4e4\uc774 \ub3c4\ucc98\uc5d0 \ubd99\uc778 \uac8c\ucca9(\u63ed\u5e16)\uc5d0\ub294 '\uc911\uad6d\uc774 \ub2f9\uba74\ud55c \ubb38\uc81c'\ub97c \uc9c0\uc801\ud558\uace0 \ubc18\uc81c\uad6d\uc8fc\uc758\uc801\uc778 \ub0b4\uc6a9\uc774 \ucda9\ub9cc\ud788 \uc788\uc5c8\uc73c\uba70, \ud589\ub3d9\uc73c\ub85c \ud22c\uc7c1\ud558\uc790\ub294 \uac83\uc73c\ub85c \uac00\ub4dd\ucc3c\ub2e4. \ub458\uc9f8, \uc81c\uad6d\uc8fc\uc758\uc5d0 \ub300\ud558\uc5ec \ud0c0\uaca9\uc744 \uc8fc\uc5c8\uc744 \ubfd0\ub9cc \uc544\ub2c8\ub77c \uc81c\uad6d\uc8fc\uc758 \uad6d\uac00\ub4e4 \ub0b4\ubd80\uc758 \ub300\ub9bd\uc73c\ub85c \uc911\uad6d\uc744 \ub118\ubcf4\uc9c0 \ubabb\ud558\ub3c4\ub85d \ub9cc\ub4e4\uc5c8\ub2e4. \ube44\ub85d \ubb34\uae30\ub294 \uc5f4\uc545\ud558\uc600\uc73c\ub098 \uc758\ud654\ub2e8\uc758 \uc800\ud56d\uc740 8\uac1c\uad6d \uc5f0\ud569\uad70 \uc0ac\ub839\uad00\uc774\uc5c8\ub358 \ub3c5\uc77c\uc758 \uc0ac\ub839\uad00 \uc54c\ud504\ub808\ud2b8 \ud3f0 \ubc1c\ub354\uc81c\uac00 \ud669\uc81c \ube4c\ud5ec\ub984 2\uc138\uc5d0\uac8c \uc911\uad6d\uc744 \ub118\uaca8\uc900\ub2e4\ub294 \uac83\uc740 '\uc808\ub300 \uc2e4\ud604\ud560 \uc218 \uc5c6\ub2e4'\uace0 \ud558\uc600\uc73c\uba70, \uc911\uad6d\uc778\ub4e4\uc740 '\ubb34\ud55c\ud55c \uc0dd\uae30\ub97c \uac16\uace0 \uc788\ub2e4'\uace0 \ud558\uc600\ub358 \uac83\uc73c\ub85c \ubcf4\uc544 \uc5f4\uac15\ub4e4\uc5d0\uac8c \uc911\uad6d\uc744 \ub118\uaca8\uc904 \uc218 \uc5c6\ub2e4\uace0 \uc778\uc2dd\uc2dc\ucf30\ub2e4.", "answer": "\uc758\ud654\ub2e8 \uc6b4\ub3d9", "sentence": "\uc758\ud654\ub2e8 \uc6b4\ub3d9 \uc740 \uac00\ub09c\ud55c \ub18d\ubbfc\uc774 \uc8fc\uccb4\uac00 \ub418\uc5b4 \uc790\ubc1c\uc801\uc73c\ub85c \uc77c\uc5b4\ub09c \ubc18\uc81c\uad6d\uc8fc\uc758 \uc560\uad6d\uc6b4\ub3d9\uc774\uc5c8\uc73c\ub098 \ubc18\ubd09\uac74\uc801\uc774\uac70\ub098 \ud601\uba85\uc801\uc778 \uc131\uaca9\uc740 \uc5c6\uc5c8\ub2e4.", "paragraph_sentence": " \uc758\ud654\ub2e8 \uc6b4\ub3d9 \uc740 \uac00\ub09c\ud55c \ub18d\ubbfc\uc774 \uc8fc\uccb4\uac00 \ub418\uc5b4 \uc790\ubc1c\uc801\uc73c\ub85c \uc77c\uc5b4\ub09c \ubc18\uc81c\uad6d\uc8fc\uc758 \uc560\uad6d\uc6b4\ub3d9\uc774\uc5c8\uc73c\ub098 \ubc18\ubd09\uac74\uc801\uc774\uac70\ub098 \ud601\uba85\uc801\uc778 \uc131\uaca9\uc740 \uc5c6\uc5c8\ub2e4. \ube44\ub85d \uc2e4\ud328\ub85c \ub05d\ub0ac\uc9c0\ub9cc \uadf8 \uc758\uc758\ub294 \uc0c1\ub2f9\ud788 \ucef8\ub2e4. \uccab\uc9f8, \uc758\ud654\ub2e8 \uc6b4\ub3d9\uc758 \uacb0\uacfc \uc911\uad6d\uc758 \ubbfc\uc871\uc8fc\uc758\uac00 \ud0c4\uc0dd\ud558\uc600\ub2e4. \uc774\ub4e4\uc774 \ub3c4\ucc98\uc5d0 \ubd99\uc778 \uac8c\ucca9(\u63ed\u5e16)\uc5d0\ub294 '\uc911\uad6d\uc774 \ub2f9\uba74\ud55c \ubb38\uc81c'\ub97c \uc9c0\uc801\ud558\uace0 \ubc18\uc81c\uad6d\uc8fc\uc758\uc801\uc778 \ub0b4\uc6a9\uc774 \ucda9\ub9cc\ud788 \uc788\uc5c8\uc73c\uba70, \ud589\ub3d9\uc73c\ub85c \ud22c\uc7c1\ud558\uc790\ub294 \uac83\uc73c\ub85c \uac00\ub4dd\ucc3c\ub2e4. \ub458\uc9f8, \uc81c\uad6d\uc8fc\uc758\uc5d0 \ub300\ud558\uc5ec \ud0c0\uaca9\uc744 \uc8fc\uc5c8\uc744 \ubfd0\ub9cc \uc544\ub2c8\ub77c \uc81c\uad6d\uc8fc\uc758 \uad6d\uac00\ub4e4 \ub0b4\ubd80\uc758 \ub300\ub9bd\uc73c\ub85c \uc911\uad6d\uc744 \ub118\ubcf4\uc9c0 \ubabb\ud558\ub3c4\ub85d \ub9cc\ub4e4\uc5c8\ub2e4. \ube44\ub85d \ubb34\uae30\ub294 \uc5f4\uc545\ud558\uc600\uc73c\ub098 \uc758\ud654\ub2e8\uc758 \uc800\ud56d\uc740 8\uac1c\uad6d \uc5f0\ud569\uad70 \uc0ac\ub839\uad00\uc774\uc5c8\ub358 \ub3c5\uc77c\uc758 \uc0ac\ub839\uad00 \uc54c\ud504\ub808\ud2b8 \ud3f0 \ubc1c\ub354\uc81c\uac00 \ud669\uc81c \ube4c\ud5ec\ub984 2\uc138\uc5d0\uac8c \uc911\uad6d\uc744 \ub118\uaca8\uc900\ub2e4\ub294 \uac83\uc740 '\uc808\ub300 \uc2e4\ud604\ud560 \uc218 \uc5c6\ub2e4'\uace0 \ud558\uc600\uc73c\uba70, \uc911\uad6d\uc778\ub4e4\uc740 '\ubb34\ud55c\ud55c \uc0dd\uae30\ub97c \uac16\uace0 \uc788\ub2e4'\uace0 \ud558\uc600\ub358 \uac83\uc73c\ub85c \ubcf4\uc544 \uc5f4\uac15\ub4e4\uc5d0\uac8c \uc911\uad6d\uc744 \ub118\uaca8\uc904 \uc218 \uc5c6\ub2e4\uace0 \uc778\uc2dd\uc2dc\ucf30\ub2e4.", "paragraph_answer": " \uc758\ud654\ub2e8 \uc6b4\ub3d9 \uc740 \uac00\ub09c\ud55c \ub18d\ubbfc\uc774 \uc8fc\uccb4\uac00 \ub418\uc5b4 \uc790\ubc1c\uc801\uc73c\ub85c \uc77c\uc5b4\ub09c \ubc18\uc81c\uad6d\uc8fc\uc758 \uc560\uad6d\uc6b4\ub3d9\uc774\uc5c8\uc73c\ub098 \ubc18\ubd09\uac74\uc801\uc774\uac70\ub098 \ud601\uba85\uc801\uc778 \uc131\uaca9\uc740 \uc5c6\uc5c8\ub2e4. \ube44\ub85d \uc2e4\ud328\ub85c \ub05d\ub0ac\uc9c0\ub9cc \uadf8 \uc758\uc758\ub294 \uc0c1\ub2f9\ud788 \ucef8\ub2e4. \uccab\uc9f8, \uc758\ud654\ub2e8 \uc6b4\ub3d9\uc758 \uacb0\uacfc \uc911\uad6d\uc758 \ubbfc\uc871\uc8fc\uc758\uac00 \ud0c4\uc0dd\ud558\uc600\ub2e4. \uc774\ub4e4\uc774 \ub3c4\ucc98\uc5d0 \ubd99\uc778 \uac8c\ucca9(\u63ed\u5e16)\uc5d0\ub294 '\uc911\uad6d\uc774 \ub2f9\uba74\ud55c \ubb38\uc81c'\ub97c \uc9c0\uc801\ud558\uace0 \ubc18\uc81c\uad6d\uc8fc\uc758\uc801\uc778 \ub0b4\uc6a9\uc774 \ucda9\ub9cc\ud788 \uc788\uc5c8\uc73c\uba70, \ud589\ub3d9\uc73c\ub85c \ud22c\uc7c1\ud558\uc790\ub294 \uac83\uc73c\ub85c \uac00\ub4dd\ucc3c\ub2e4. \ub458\uc9f8, \uc81c\uad6d\uc8fc\uc758\uc5d0 \ub300\ud558\uc5ec \ud0c0\uaca9\uc744 \uc8fc\uc5c8\uc744 \ubfd0\ub9cc \uc544\ub2c8\ub77c \uc81c\uad6d\uc8fc\uc758 \uad6d\uac00\ub4e4 \ub0b4\ubd80\uc758 \ub300\ub9bd\uc73c\ub85c \uc911\uad6d\uc744 \ub118\ubcf4\uc9c0 \ubabb\ud558\ub3c4\ub85d \ub9cc\ub4e4\uc5c8\ub2e4. \ube44\ub85d \ubb34\uae30\ub294 \uc5f4\uc545\ud558\uc600\uc73c\ub098 \uc758\ud654\ub2e8\uc758 \uc800\ud56d\uc740 8\uac1c\uad6d \uc5f0\ud569\uad70 \uc0ac\ub839\uad00\uc774\uc5c8\ub358 \ub3c5\uc77c\uc758 \uc0ac\ub839\uad00 \uc54c\ud504\ub808\ud2b8 \ud3f0 \ubc1c\ub354\uc81c\uac00 \ud669\uc81c \ube4c\ud5ec\ub984 2\uc138\uc5d0\uac8c \uc911\uad6d\uc744 \ub118\uaca8\uc900\ub2e4\ub294 \uac83\uc740 '\uc808\ub300 \uc2e4\ud604\ud560 \uc218 \uc5c6\ub2e4'\uace0 \ud558\uc600\uc73c\uba70, \uc911\uad6d\uc778\ub4e4\uc740 '\ubb34\ud55c\ud55c \uc0dd\uae30\ub97c \uac16\uace0 \uc788\ub2e4'\uace0 \ud558\uc600\ub358 \uac83\uc73c\ub85c \ubcf4\uc544 \uc5f4\uac15\ub4e4\uc5d0\uac8c \uc911\uad6d\uc744 \ub118\uaca8\uc904 \uc218 \uc5c6\ub2e4\uace0 \uc778\uc2dd\uc2dc\ucf30\ub2e4.", "sentence_answer": " \uc758\ud654\ub2e8 \uc6b4\ub3d9 \uc740 \uac00\ub09c\ud55c \ub18d\ubbfc\uc774 \uc8fc\uccb4\uac00 \ub418\uc5b4 \uc790\ubc1c\uc801\uc73c\ub85c \uc77c\uc5b4\ub09c \ubc18\uc81c\uad6d\uc8fc\uc758 \uc560\uad6d\uc6b4\ub3d9\uc774\uc5c8\uc73c\ub098 \ubc18\ubd09\uac74\uc801\uc774\uac70\ub098 \ud601\uba85\uc801\uc778 \uc131\uaca9\uc740 \uc5c6\uc5c8\ub2e4.", "paragraph_id": "6544410-26-0", "paragraph_question": "question: \ubc18\ubd09\uac74\uc801\uc774\uac70\ub098 \ud601\uba85\uc801\uc778 \uc131\uaca9\uc744 \ub744\uc9c0 \uc54a\uc558\uc73c\uba70 \uccad\ub098\ub77c \ub18d\ubbfc\ub4e4\uc5d0 \uc758\ud574 \uc790\ubc1c\uc801\uc73c\ub85c \ubc8c\uc5b4\uc9c4 \ubc18\uc81c\uad6d\uc8fc\uc758\uc6b4\ub3d9\uc740?, context: \uc758\ud654\ub2e8 \uc6b4\ub3d9\uc740 \uac00\ub09c\ud55c \ub18d\ubbfc\uc774 \uc8fc\uccb4\uac00 \ub418\uc5b4 \uc790\ubc1c\uc801\uc73c\ub85c \uc77c\uc5b4\ub09c \ubc18\uc81c\uad6d\uc8fc\uc758 \uc560\uad6d\uc6b4\ub3d9\uc774\uc5c8\uc73c\ub098 \ubc18\ubd09\uac74\uc801\uc774\uac70\ub098 \ud601\uba85\uc801\uc778 \uc131\uaca9\uc740 \uc5c6\uc5c8\ub2e4. \ube44\ub85d \uc2e4\ud328\ub85c \ub05d\ub0ac\uc9c0\ub9cc \uadf8 \uc758\uc758\ub294 \uc0c1\ub2f9\ud788 \ucef8\ub2e4. \uccab\uc9f8, \uc758\ud654\ub2e8 \uc6b4\ub3d9\uc758 \uacb0\uacfc \uc911\uad6d\uc758 \ubbfc\uc871\uc8fc\uc758\uac00 \ud0c4\uc0dd\ud558\uc600\ub2e4. \uc774\ub4e4\uc774 \ub3c4\ucc98\uc5d0 \ubd99\uc778 \uac8c\ucca9(\u63ed\u5e16)\uc5d0\ub294 '\uc911\uad6d\uc774 \ub2f9\uba74\ud55c \ubb38\uc81c'\ub97c \uc9c0\uc801\ud558\uace0 \ubc18\uc81c\uad6d\uc8fc\uc758\uc801\uc778 \ub0b4\uc6a9\uc774 \ucda9\ub9cc\ud788 \uc788\uc5c8\uc73c\uba70, \ud589\ub3d9\uc73c\ub85c \ud22c\uc7c1\ud558\uc790\ub294 \uac83\uc73c\ub85c \uac00\ub4dd\ucc3c\ub2e4. \ub458\uc9f8, \uc81c\uad6d\uc8fc\uc758\uc5d0 \ub300\ud558\uc5ec \ud0c0\uaca9\uc744 \uc8fc\uc5c8\uc744 \ubfd0\ub9cc \uc544\ub2c8\ub77c \uc81c\uad6d\uc8fc\uc758 \uad6d\uac00\ub4e4 \ub0b4\ubd80\uc758 \ub300\ub9bd\uc73c\ub85c \uc911\uad6d\uc744 \ub118\ubcf4\uc9c0 \ubabb\ud558\ub3c4\ub85d \ub9cc\ub4e4\uc5c8\ub2e4. \ube44\ub85d \ubb34\uae30\ub294 \uc5f4\uc545\ud558\uc600\uc73c\ub098 \uc758\ud654\ub2e8\uc758 \uc800\ud56d\uc740 8\uac1c\uad6d \uc5f0\ud569\uad70 \uc0ac\ub839\uad00\uc774\uc5c8\ub358 \ub3c5\uc77c\uc758 \uc0ac\ub839\uad00 \uc54c\ud504\ub808\ud2b8 \ud3f0 \ubc1c\ub354\uc81c\uac00 \ud669\uc81c \ube4c\ud5ec\ub984 2\uc138\uc5d0\uac8c \uc911\uad6d\uc744 \ub118\uaca8\uc900\ub2e4\ub294 \uac83\uc740 '\uc808\ub300 \uc2e4\ud604\ud560 \uc218 \uc5c6\ub2e4'\uace0 \ud558\uc600\uc73c\uba70, \uc911\uad6d\uc778\ub4e4\uc740 '\ubb34\ud55c\ud55c \uc0dd\uae30\ub97c \uac16\uace0 \uc788\ub2e4'\uace0 \ud558\uc600\ub358 \uac83\uc73c\ub85c \ubcf4\uc544 \uc5f4\uac15\ub4e4\uc5d0\uac8c \uc911\uad6d\uc744 \ub118\uaca8\uc904 \uc218 \uc5c6\ub2e4\uace0 \uc778\uc2dd\uc2dc\ucf30\ub2e4."} {"question": "\ucd5c\ucd08\uc758 \ud604\ub300 \ubcf4\ubcd1\uc804\ud22c\ucc28\ub294?", "paragraph": "\ud604\ub300 \ubcf4\ubcd1\uc804\ud22c\ucc28\ub294 \uc18c\ub828\uc5d0\uc11c \ucc98\uc74c \uac1c\ubc1c\ud55c BMP-1\uc774\uba70, \uadf8 \ub2e4\uc74c\uc73c\ub85c \ub3c5\uc77c\uc758 \ub9c8\ub354 \ubcf4\ubcd1\uc804\ud22c\ucc28\uac00 \uac1c\ubc1c\ub418\uc5c8\ub2e4. \ub3c5\uc77c\uacfc \uc18c\ub828\uc740 \ub3c5\uc18c\uc804\uc7c1 \uc911\uc5d0 \uaca9\ub82c\ud55c \uae30\uac11\uc804\uc744 \uce58\ub974\uba74\uc11c \uc801\uc808\ud55c \ubc29\ud638\ub97c \uc81c\uacf5\ubc1b\uc9c0 \ubabb\ud55c \ubcf4\ubcd1\uc758 \uc5c4\uccad\ub09c \uc190\uc2e4\uc744 \uacaa\uc73c\uba74\uc11c \uae30\uacc4\ud654\ubcf4\ubcd1\uc744 \uc548\uc804\ud558\uac8c \ubcf4\ud638\ud558\uba74\uc11c\ub3c4 \uc804\ud22c\uac00 \uac00\ub2a5\ud55c \ucc28\ub7c9\uc744 \ucc3e\uc558\ub358 \uac83\uc774\ub2e4. \ubc18\uba74\uc5d0 \uc81c2\ucc28 \uc138\uacc4 \ub300\uc804 \uc911\uc5d0 \ub3d9\ubd80 \uc804\uc120 \ub9cc\ud07c\uc758 \uaca9\ub82c\ud55c \uae30\uac11\uc804\ud22c\ub294 \uce58\ub974\uc9c0 \uc54a\uc558\ub358 \uc601\uad6d\uacfc \ubbf8\uad6d\uc740 APC \uc815\ub3c4\uba74 \ucda9\ubd84\ud558\ub2e4\uace0 \uc5ec\uacbc\ub2e4. \ubcf4\ubcd1\uc804\ud22c\ucc28\ub3c4 \uc804\ucc28\uc640 \uae30\ubcf8\uc801\uc73c\ub85c \uac19\uc774 \uc784\ubb34\ub97c \uc218\ud589\ud574\uc57c \ud558\ubbc0\ub85c \uada4\ub3c4\uc2dd\uc774 \uc8fc\ub958\ub97c \uc774\ub8e8\uace0 \uc788\ub2e4. \ub0c9\uc804 \uc885\uc2dd \uc774\ud6c4\uc5d0\ub294 \uc138\uacc4\uc801\uc73c\ub85c\ub294 \uc218\uc694\uac00 \uac10\uc18c\ud55c \uc0c1\ud0dc\uc9c0\ub9cc, \ubbf8\uad6d-\uc774\ub77c\ud06c \uc804\uc7c1\uc744 \ud1b5\ud574 \uada4\ub3c4\uc2dd \ubcf4\ubcd1\uc804\ud22c\ucc28\uc758 \uac00\uce58\uac00 \ub2e4\uc2dc \uc8fc\ubaa9\ubc1b\uc558\ub2e4.", "answer": "BMP-1", "sentence": "\ud604\ub300 \ubcf4\ubcd1\uc804\ud22c\ucc28\ub294 \uc18c\ub828\uc5d0\uc11c \ucc98\uc74c \uac1c\ubc1c\ud55c BMP-1 \uc774\uba70, \uadf8 \ub2e4\uc74c\uc73c\ub85c \ub3c5\uc77c\uc758 \ub9c8\ub354 \ubcf4\ubcd1\uc804\ud22c\ucc28\uac00 \uac1c\ubc1c\ub418\uc5c8\ub2e4.", "paragraph_sentence": " \ud604\ub300 \ubcf4\ubcd1\uc804\ud22c\ucc28\ub294 \uc18c\ub828\uc5d0\uc11c \ucc98\uc74c \uac1c\ubc1c\ud55c BMP-1 \uc774\uba70, \uadf8 \ub2e4\uc74c\uc73c\ub85c \ub3c5\uc77c\uc758 \ub9c8\ub354 \ubcf4\ubcd1\uc804\ud22c\ucc28\uac00 \uac1c\ubc1c\ub418\uc5c8\ub2e4. \ub3c5\uc77c\uacfc \uc18c\ub828\uc740 \ub3c5\uc18c\uc804\uc7c1 \uc911\uc5d0 \uaca9\ub82c\ud55c \uae30\uac11\uc804\uc744 \uce58\ub974\uba74\uc11c \uc801\uc808\ud55c \ubc29\ud638\ub97c \uc81c\uacf5\ubc1b\uc9c0 \ubabb\ud55c \ubcf4\ubcd1\uc758 \uc5c4\uccad\ub09c \uc190\uc2e4\uc744 \uacaa\uc73c\uba74\uc11c \uae30\uacc4\ud654\ubcf4\ubcd1\uc744 \uc548\uc804\ud558\uac8c \ubcf4\ud638\ud558\uba74\uc11c\ub3c4 \uc804\ud22c\uac00 \uac00\ub2a5\ud55c \ucc28\ub7c9\uc744 \ucc3e\uc558\ub358 \uac83\uc774\ub2e4. \ubc18\uba74\uc5d0 \uc81c2\ucc28 \uc138\uacc4 \ub300\uc804 \uc911\uc5d0 \ub3d9\ubd80 \uc804\uc120 \ub9cc\ud07c\uc758 \uaca9\ub82c\ud55c \uae30\uac11\uc804\ud22c\ub294 \uce58\ub974\uc9c0 \uc54a\uc558\ub358 \uc601\uad6d\uacfc \ubbf8\uad6d\uc740 APC \uc815\ub3c4\uba74 \ucda9\ubd84\ud558\ub2e4\uace0 \uc5ec\uacbc\ub2e4. \ubcf4\ubcd1\uc804\ud22c\ucc28\ub3c4 \uc804\ucc28\uc640 \uae30\ubcf8\uc801\uc73c\ub85c \uac19\uc774 \uc784\ubb34\ub97c \uc218\ud589\ud574\uc57c \ud558\ubbc0\ub85c \uada4\ub3c4\uc2dd\uc774 \uc8fc\ub958\ub97c \uc774\ub8e8\uace0 \uc788\ub2e4. \ub0c9\uc804 \uc885\uc2dd \uc774\ud6c4\uc5d0\ub294 \uc138\uacc4\uc801\uc73c\ub85c\ub294 \uc218\uc694\uac00 \uac10\uc18c\ud55c \uc0c1\ud0dc\uc9c0\ub9cc, \ubbf8\uad6d-\uc774\ub77c\ud06c \uc804\uc7c1\uc744 \ud1b5\ud574 \uada4\ub3c4\uc2dd \ubcf4\ubcd1\uc804\ud22c\ucc28\uc758 \uac00\uce58\uac00 \ub2e4\uc2dc \uc8fc\ubaa9\ubc1b\uc558\ub2e4.", "paragraph_answer": "\ud604\ub300 \ubcf4\ubcd1\uc804\ud22c\ucc28\ub294 \uc18c\ub828\uc5d0\uc11c \ucc98\uc74c \uac1c\ubc1c\ud55c BMP-1 \uc774\uba70, \uadf8 \ub2e4\uc74c\uc73c\ub85c \ub3c5\uc77c\uc758 \ub9c8\ub354 \ubcf4\ubcd1\uc804\ud22c\ucc28\uac00 \uac1c\ubc1c\ub418\uc5c8\ub2e4. \ub3c5\uc77c\uacfc \uc18c\ub828\uc740 \ub3c5\uc18c\uc804\uc7c1 \uc911\uc5d0 \uaca9\ub82c\ud55c \uae30\uac11\uc804\uc744 \uce58\ub974\uba74\uc11c \uc801\uc808\ud55c \ubc29\ud638\ub97c \uc81c\uacf5\ubc1b\uc9c0 \ubabb\ud55c \ubcf4\ubcd1\uc758 \uc5c4\uccad\ub09c \uc190\uc2e4\uc744 \uacaa\uc73c\uba74\uc11c \uae30\uacc4\ud654\ubcf4\ubcd1\uc744 \uc548\uc804\ud558\uac8c \ubcf4\ud638\ud558\uba74\uc11c\ub3c4 \uc804\ud22c\uac00 \uac00\ub2a5\ud55c \ucc28\ub7c9\uc744 \ucc3e\uc558\ub358 \uac83\uc774\ub2e4. \ubc18\uba74\uc5d0 \uc81c2\ucc28 \uc138\uacc4 \ub300\uc804 \uc911\uc5d0 \ub3d9\ubd80 \uc804\uc120 \ub9cc\ud07c\uc758 \uaca9\ub82c\ud55c \uae30\uac11\uc804\ud22c\ub294 \uce58\ub974\uc9c0 \uc54a\uc558\ub358 \uc601\uad6d\uacfc \ubbf8\uad6d\uc740 APC \uc815\ub3c4\uba74 \ucda9\ubd84\ud558\ub2e4\uace0 \uc5ec\uacbc\ub2e4. \ubcf4\ubcd1\uc804\ud22c\ucc28\ub3c4 \uc804\ucc28\uc640 \uae30\ubcf8\uc801\uc73c\ub85c \uac19\uc774 \uc784\ubb34\ub97c \uc218\ud589\ud574\uc57c \ud558\ubbc0\ub85c \uada4\ub3c4\uc2dd\uc774 \uc8fc\ub958\ub97c \uc774\ub8e8\uace0 \uc788\ub2e4. \ub0c9\uc804 \uc885\uc2dd \uc774\ud6c4\uc5d0\ub294 \uc138\uacc4\uc801\uc73c\ub85c\ub294 \uc218\uc694\uac00 \uac10\uc18c\ud55c \uc0c1\ud0dc\uc9c0\ub9cc, \ubbf8\uad6d-\uc774\ub77c\ud06c \uc804\uc7c1\uc744 \ud1b5\ud574 \uada4\ub3c4\uc2dd \ubcf4\ubcd1\uc804\ud22c\ucc28\uc758 \uac00\uce58\uac00 \ub2e4\uc2dc \uc8fc\ubaa9\ubc1b\uc558\ub2e4.", "sentence_answer": "\ud604\ub300 \ubcf4\ubcd1\uc804\ud22c\ucc28\ub294 \uc18c\ub828\uc5d0\uc11c \ucc98\uc74c \uac1c\ubc1c\ud55c BMP-1 \uc774\uba70, \uadf8 \ub2e4\uc74c\uc73c\ub85c \ub3c5\uc77c\uc758 \ub9c8\ub354 \ubcf4\ubcd1\uc804\ud22c\ucc28\uac00 \uac1c\ubc1c\ub418\uc5c8\ub2e4.", "paragraph_id": "6377400-2-0", "paragraph_question": "question: \ucd5c\ucd08\uc758 \ud604\ub300 \ubcf4\ubcd1\uc804\ud22c\ucc28\ub294?, context: \ud604\ub300 \ubcf4\ubcd1\uc804\ud22c\ucc28\ub294 \uc18c\ub828\uc5d0\uc11c \ucc98\uc74c \uac1c\ubc1c\ud55c BMP-1\uc774\uba70, \uadf8 \ub2e4\uc74c\uc73c\ub85c \ub3c5\uc77c\uc758 \ub9c8\ub354 \ubcf4\ubcd1\uc804\ud22c\ucc28\uac00 \uac1c\ubc1c\ub418\uc5c8\ub2e4. \ub3c5\uc77c\uacfc \uc18c\ub828\uc740 \ub3c5\uc18c\uc804\uc7c1 \uc911\uc5d0 \uaca9\ub82c\ud55c \uae30\uac11\uc804\uc744 \uce58\ub974\uba74\uc11c \uc801\uc808\ud55c \ubc29\ud638\ub97c \uc81c\uacf5\ubc1b\uc9c0 \ubabb\ud55c \ubcf4\ubcd1\uc758 \uc5c4\uccad\ub09c \uc190\uc2e4\uc744 \uacaa\uc73c\uba74\uc11c \uae30\uacc4\ud654\ubcf4\ubcd1\uc744 \uc548\uc804\ud558\uac8c \ubcf4\ud638\ud558\uba74\uc11c\ub3c4 \uc804\ud22c\uac00 \uac00\ub2a5\ud55c \ucc28\ub7c9\uc744 \ucc3e\uc558\ub358 \uac83\uc774\ub2e4. \ubc18\uba74\uc5d0 \uc81c2\ucc28 \uc138\uacc4 \ub300\uc804 \uc911\uc5d0 \ub3d9\ubd80 \uc804\uc120 \ub9cc\ud07c\uc758 \uaca9\ub82c\ud55c \uae30\uac11\uc804\ud22c\ub294 \uce58\ub974\uc9c0 \uc54a\uc558\ub358 \uc601\uad6d\uacfc \ubbf8\uad6d\uc740 APC \uc815\ub3c4\uba74 \ucda9\ubd84\ud558\ub2e4\uace0 \uc5ec\uacbc\ub2e4. \ubcf4\ubcd1\uc804\ud22c\ucc28\ub3c4 \uc804\ucc28\uc640 \uae30\ubcf8\uc801\uc73c\ub85c \uac19\uc774 \uc784\ubb34\ub97c \uc218\ud589\ud574\uc57c \ud558\ubbc0\ub85c \uada4\ub3c4\uc2dd\uc774 \uc8fc\ub958\ub97c \uc774\ub8e8\uace0 \uc788\ub2e4. \ub0c9\uc804 \uc885\uc2dd \uc774\ud6c4\uc5d0\ub294 \uc138\uacc4\uc801\uc73c\ub85c\ub294 \uc218\uc694\uac00 \uac10\uc18c\ud55c \uc0c1\ud0dc\uc9c0\ub9cc, \ubbf8\uad6d-\uc774\ub77c\ud06c \uc804\uc7c1\uc744 \ud1b5\ud574 \uada4\ub3c4\uc2dd \ubcf4\ubcd1\uc804\ud22c\ucc28\uc758 \uac00\uce58\uac00 \ub2e4\uc2dc \uc8fc\ubaa9\ubc1b\uc558\ub2e4."} {"question": "\uae40\ub300\uc911\uc758 \ubaa9\ud3ec\ud574\uc0c1\ubc29\uc704\ub300 \uacbd\ub825\uc744 \uc99d\uba85\ud574\uc900 \uc7ac\ubbf8 \ud574\ubcd1\ub300 \uc608\ube44\uc5ed \uc900\uc7a5\uc740?", "paragraph": "\uc774\uc5b4 \ud574\uad70 \uacbd\ube44\ub300 \uc18c\uc18d \ubaa9\ud3ec\ud574\uc0c1\ubc29\uc704\ub300 \uc804\ub0a8\uc9c0\uad6c \ubd80\ub300\uc7a5\uc744 \ub9e1\uc544 \ub2f9\uc2dc \uc790\uc2e0\uc758 \ud574\uc6b4\ud68c\uc0ac\uc5d0\uc11c \ubcf4\uc720\ud558\uace0 \uc788\ub358 \uc120\ubc15\uc744 \ub3d9\uc6d0\ud574 \ucc38\uc804\ud588\ub2e4\uace0 \uc99d\uc5b8\ud588\ub2e4. \uadf8\ub7ec\ub098 \uc2e0\ud55c\uad6d\ub2f9\uc740 \ubaa9\ud3ec\ud574\uc0c1\ubc29\uc704\ub300\uac00 \uc720\ub839 \uad70\ubd80\ub300\ub77c\uace0 \uc8fc\uc7a5\ud588\uace0, \uad6d\ubbfc\ud68c\uc758 \uce21\uc5d0\uc120 \ud574\uc0c1\ubc29\uc704\ub300\ub294 1950\ub144 \uc721\uad70\ubcf8\ubd80 \uc791\uc804\uba85\ub839 \uc81c38\ud638\uc640 \uad6d\ubc29\uad70\uc0ac\uc5f0\uad6c\uc18c\uc5d0\uc11c \ubc1c\ud589\ud55c \uad6d\ubc29\uc0ac\uc5f0\ud45c\uc5d0 \ubd84\uba85\ud788 \uadfc\uac70\uac00 \uc788\ub2e4\uace0 \ubc18\ubc15\ud558\ub294 \ub4f1 \uc591\uce21\uc758 \uacf5\ubc29\uc774 \uc774\uc5b4\uc84c\ub2e4. \uc774\ud6c4 \uad6d\ubbfc\ud68c\uc758 \ucc9c\uc6a9\ud0dd \uc758\uc6d0\uc774 \ubbf8\uad6d\uc5d0 \uac70\uc8fc\uc911\uc778 \ud574\ubcd1\ub300 \uc608\ube44\uc5ed \uc900\uc7a5 \uc1a1\uc778\uba85\uc744 \ub9cc\ub098 \uadf8\uc758 \uc99d\uc5b8\ub85d\uacfc \uc9c1\uc811 \uc4f4 \uc99d\uba85\uc11c \ub4f1\uc744 \uac00\uc838\uc640 \uacf5\uac1c\ud558\uace0, \uc1a1\uc778\uba85\uc740 \uc804 \ubaa9\ud3ec\uacbd\ube44\ubd80\uc0ac\ub839\uad00\uc73c\ub85c \ubd80\uc784\ud588\uc744 \ub54c \uad70\uacfc \uacbd\ucc30\ub9cc\uc73c\ub85c \uacf5\uc0b0\uc138\ub825\uc758 \uc794\ub2f9\uc18c\ud0d5\uacfc \uc548\ub155\uc9c8\uc11c \uc720\uc9c0\uc5d0 \uc5b4\ub824\uc6c0\uc774 \ub9ce\uc558\uae30\ub54c\ubb38\uc5d0 \ubaa9\ud3ec\uc2dc\ub0b4 \uccad\ub144\ub2e8\uacfc \uc5ec\uc131\uccad\ub144\ub2e8 \ub4f1 \uc560\uad6d\ub2e8\uccb4\ub4e4\uc744 \ubaa8\ub450 \ub3d9\uc6d0\ud574 \uc791\uc804\uc744 \ubcf4\uc870\ud558\uac8c \ud558\uc600\uc73c\uba70, \ud2b9\ud788 \ud574\uc0c1\uc791\uc804\uc774 \uc911\uc694\ud588\ub2e4\uace0 \ud310\ub2e8\ud588\uae30 \ub54c\ubb38\uc5d0 \ub2f9\uc2dc \uc9c0\uc5ed\uc720\uc9c0 \uc624\uc7ac\uade0\uc5d0\uac8c \uc9c0\uc2dc, \ud574\uc0c1\ubc29\uc704\ub300\ub97c \ucc3d\uc124\ud574 \uae40\ub300\uc911\uc5d0\uac8c \ubd80\ub300\uc7a5\uc744 \ub9e1\uacbc\ub2e4\uace0 \ud574\uba85\ud588\ub2e4.", "answer": "\uc1a1\uc778\uba85", "sentence": "\uc774\ud6c4 \uad6d\ubbfc\ud68c\uc758 \ucc9c\uc6a9\ud0dd \uc758\uc6d0\uc774 \ubbf8\uad6d\uc5d0 \uac70\uc8fc\uc911\uc778 \ud574\ubcd1\ub300 \uc608\ube44\uc5ed \uc900\uc7a5 \uc1a1\uc778\uba85 \uc744 \ub9cc\ub098 \uadf8\uc758 \uc99d\uc5b8\ub85d\uacfc \uc9c1\uc811 \uc4f4 \uc99d\uba85\uc11c \ub4f1\uc744 \uac00\uc838\uc640 \uacf5\uac1c\ud558\uace0, \uc1a1\uc778\uba85\uc740 \uc804 \ubaa9\ud3ec\uacbd\ube44\ubd80\uc0ac\ub839\uad00\uc73c\ub85c \ubd80\uc784\ud588\uc744 \ub54c \uad70\uacfc \uacbd\ucc30\ub9cc\uc73c\ub85c \uacf5\uc0b0\uc138\ub825\uc758 \uc794\ub2f9\uc18c\ud0d5\uacfc \uc548\ub155\uc9c8\uc11c \uc720\uc9c0\uc5d0 \uc5b4\ub824\uc6c0\uc774 \ub9ce\uc558\uae30\ub54c\ubb38\uc5d0 \ubaa9\ud3ec\uc2dc\ub0b4 \uccad\ub144\ub2e8\uacfc \uc5ec\uc131\uccad\ub144\ub2e8 \ub4f1 \uc560\uad6d\ub2e8\uccb4\ub4e4\uc744 \ubaa8\ub450 \ub3d9\uc6d0\ud574 \uc791\uc804\uc744 \ubcf4\uc870\ud558\uac8c \ud558\uc600\uc73c\uba70, \ud2b9\ud788 \ud574\uc0c1\uc791\uc804\uc774 \uc911\uc694\ud588\ub2e4\uace0 \ud310\ub2e8\ud588\uae30 \ub54c\ubb38\uc5d0 \ub2f9\uc2dc \uc9c0\uc5ed\uc720\uc9c0 \uc624\uc7ac\uade0\uc5d0\uac8c \uc9c0\uc2dc, \ud574\uc0c1\ubc29\uc704\ub300\ub97c \ucc3d\uc124\ud574 \uae40\ub300\uc911\uc5d0\uac8c \ubd80\ub300\uc7a5\uc744 \ub9e1\uacbc\ub2e4\uace0 \ud574\uba85\ud588\ub2e4.", "paragraph_sentence": "\uc774\uc5b4 \ud574\uad70 \uacbd\ube44\ub300 \uc18c\uc18d \ubaa9\ud3ec\ud574\uc0c1\ubc29\uc704\ub300 \uc804\ub0a8\uc9c0\uad6c \ubd80\ub300\uc7a5\uc744 \ub9e1\uc544 \ub2f9\uc2dc \uc790\uc2e0\uc758 \ud574\uc6b4\ud68c\uc0ac\uc5d0\uc11c \ubcf4\uc720\ud558\uace0 \uc788\ub358 \uc120\ubc15\uc744 \ub3d9\uc6d0\ud574 \ucc38\uc804\ud588\ub2e4\uace0 \uc99d\uc5b8\ud588\ub2e4. \uadf8\ub7ec\ub098 \uc2e0\ud55c\uad6d\ub2f9\uc740 \ubaa9\ud3ec\ud574\uc0c1\ubc29\uc704\ub300\uac00 \uc720\ub839 \uad70\ubd80\ub300\ub77c\uace0 \uc8fc\uc7a5\ud588\uace0, \uad6d\ubbfc\ud68c\uc758 \uce21\uc5d0\uc120 \ud574\uc0c1\ubc29\uc704\ub300\ub294 1950\ub144 \uc721\uad70\ubcf8\ubd80 \uc791\uc804\uba85\ub839 \uc81c38\ud638\uc640 \uad6d\ubc29\uad70\uc0ac\uc5f0\uad6c\uc18c\uc5d0\uc11c \ubc1c\ud589\ud55c \uad6d\ubc29\uc0ac\uc5f0\ud45c\uc5d0 \ubd84\uba85\ud788 \uadfc\uac70\uac00 \uc788\ub2e4\uace0 \ubc18\ubc15\ud558\ub294 \ub4f1 \uc591\uce21\uc758 \uacf5\ubc29\uc774 \uc774\uc5b4\uc84c\ub2e4. \uc774\ud6c4 \uad6d\ubbfc\ud68c\uc758 \ucc9c\uc6a9\ud0dd \uc758\uc6d0\uc774 \ubbf8\uad6d\uc5d0 \uac70\uc8fc\uc911\uc778 \ud574\ubcd1\ub300 \uc608\ube44\uc5ed \uc900\uc7a5 \uc1a1\uc778\uba85 \uc744 \ub9cc\ub098 \uadf8\uc758 \uc99d\uc5b8\ub85d\uacfc \uc9c1\uc811 \uc4f4 \uc99d\uba85\uc11c \ub4f1\uc744 \uac00\uc838\uc640 \uacf5\uac1c\ud558\uace0, \uc1a1\uc778\uba85\uc740 \uc804 \ubaa9\ud3ec\uacbd\ube44\ubd80\uc0ac\ub839\uad00\uc73c\ub85c \ubd80\uc784\ud588\uc744 \ub54c \uad70\uacfc \uacbd\ucc30\ub9cc\uc73c\ub85c \uacf5\uc0b0\uc138\ub825\uc758 \uc794\ub2f9\uc18c\ud0d5\uacfc \uc548\ub155\uc9c8\uc11c \uc720\uc9c0\uc5d0 \uc5b4\ub824\uc6c0\uc774 \ub9ce\uc558\uae30\ub54c\ubb38\uc5d0 \ubaa9\ud3ec\uc2dc\ub0b4 \uccad\ub144\ub2e8\uacfc \uc5ec\uc131\uccad\ub144\ub2e8 \ub4f1 \uc560\uad6d\ub2e8\uccb4\ub4e4\uc744 \ubaa8\ub450 \ub3d9\uc6d0\ud574 \uc791\uc804\uc744 \ubcf4\uc870\ud558\uac8c \ud558\uc600\uc73c\uba70, \ud2b9\ud788 \ud574\uc0c1\uc791\uc804\uc774 \uc911\uc694\ud588\ub2e4\uace0 \ud310\ub2e8\ud588\uae30 \ub54c\ubb38\uc5d0 \ub2f9\uc2dc \uc9c0\uc5ed\uc720\uc9c0 \uc624\uc7ac\uade0\uc5d0\uac8c \uc9c0\uc2dc, \ud574\uc0c1\ubc29\uc704\ub300\ub97c \ucc3d\uc124\ud574 \uae40\ub300\uc911\uc5d0\uac8c \ubd80\ub300\uc7a5\uc744 \ub9e1\uacbc\ub2e4\uace0 \ud574\uba85\ud588\ub2e4. ", "paragraph_answer": "\uc774\uc5b4 \ud574\uad70 \uacbd\ube44\ub300 \uc18c\uc18d \ubaa9\ud3ec\ud574\uc0c1\ubc29\uc704\ub300 \uc804\ub0a8\uc9c0\uad6c \ubd80\ub300\uc7a5\uc744 \ub9e1\uc544 \ub2f9\uc2dc \uc790\uc2e0\uc758 \ud574\uc6b4\ud68c\uc0ac\uc5d0\uc11c \ubcf4\uc720\ud558\uace0 \uc788\ub358 \uc120\ubc15\uc744 \ub3d9\uc6d0\ud574 \ucc38\uc804\ud588\ub2e4\uace0 \uc99d\uc5b8\ud588\ub2e4. \uadf8\ub7ec\ub098 \uc2e0\ud55c\uad6d\ub2f9\uc740 \ubaa9\ud3ec\ud574\uc0c1\ubc29\uc704\ub300\uac00 \uc720\ub839 \uad70\ubd80\ub300\ub77c\uace0 \uc8fc\uc7a5\ud588\uace0, \uad6d\ubbfc\ud68c\uc758 \uce21\uc5d0\uc120 \ud574\uc0c1\ubc29\uc704\ub300\ub294 1950\ub144 \uc721\uad70\ubcf8\ubd80 \uc791\uc804\uba85\ub839 \uc81c38\ud638\uc640 \uad6d\ubc29\uad70\uc0ac\uc5f0\uad6c\uc18c\uc5d0\uc11c \ubc1c\ud589\ud55c \uad6d\ubc29\uc0ac\uc5f0\ud45c\uc5d0 \ubd84\uba85\ud788 \uadfc\uac70\uac00 \uc788\ub2e4\uace0 \ubc18\ubc15\ud558\ub294 \ub4f1 \uc591\uce21\uc758 \uacf5\ubc29\uc774 \uc774\uc5b4\uc84c\ub2e4. \uc774\ud6c4 \uad6d\ubbfc\ud68c\uc758 \ucc9c\uc6a9\ud0dd \uc758\uc6d0\uc774 \ubbf8\uad6d\uc5d0 \uac70\uc8fc\uc911\uc778 \ud574\ubcd1\ub300 \uc608\ube44\uc5ed \uc900\uc7a5 \uc1a1\uc778\uba85 \uc744 \ub9cc\ub098 \uadf8\uc758 \uc99d\uc5b8\ub85d\uacfc \uc9c1\uc811 \uc4f4 \uc99d\uba85\uc11c \ub4f1\uc744 \uac00\uc838\uc640 \uacf5\uac1c\ud558\uace0, \uc1a1\uc778\uba85\uc740 \uc804 \ubaa9\ud3ec\uacbd\ube44\ubd80\uc0ac\ub839\uad00\uc73c\ub85c \ubd80\uc784\ud588\uc744 \ub54c \uad70\uacfc \uacbd\ucc30\ub9cc\uc73c\ub85c \uacf5\uc0b0\uc138\ub825\uc758 \uc794\ub2f9\uc18c\ud0d5\uacfc \uc548\ub155\uc9c8\uc11c \uc720\uc9c0\uc5d0 \uc5b4\ub824\uc6c0\uc774 \ub9ce\uc558\uae30\ub54c\ubb38\uc5d0 \ubaa9\ud3ec\uc2dc\ub0b4 \uccad\ub144\ub2e8\uacfc \uc5ec\uc131\uccad\ub144\ub2e8 \ub4f1 \uc560\uad6d\ub2e8\uccb4\ub4e4\uc744 \ubaa8\ub450 \ub3d9\uc6d0\ud574 \uc791\uc804\uc744 \ubcf4\uc870\ud558\uac8c \ud558\uc600\uc73c\uba70, \ud2b9\ud788 \ud574\uc0c1\uc791\uc804\uc774 \uc911\uc694\ud588\ub2e4\uace0 \ud310\ub2e8\ud588\uae30 \ub54c\ubb38\uc5d0 \ub2f9\uc2dc \uc9c0\uc5ed\uc720\uc9c0 \uc624\uc7ac\uade0\uc5d0\uac8c \uc9c0\uc2dc, \ud574\uc0c1\ubc29\uc704\ub300\ub97c \ucc3d\uc124\ud574 \uae40\ub300\uc911\uc5d0\uac8c \ubd80\ub300\uc7a5\uc744 \ub9e1\uacbc\ub2e4\uace0 \ud574\uba85\ud588\ub2e4.", "sentence_answer": "\uc774\ud6c4 \uad6d\ubbfc\ud68c\uc758 \ucc9c\uc6a9\ud0dd \uc758\uc6d0\uc774 \ubbf8\uad6d\uc5d0 \uac70\uc8fc\uc911\uc778 \ud574\ubcd1\ub300 \uc608\ube44\uc5ed \uc900\uc7a5 \uc1a1\uc778\uba85 \uc744 \ub9cc\ub098 \uadf8\uc758 \uc99d\uc5b8\ub85d\uacfc \uc9c1\uc811 \uc4f4 \uc99d\uba85\uc11c \ub4f1\uc744 \uac00\uc838\uc640 \uacf5\uac1c\ud558\uace0, \uc1a1\uc778\uba85\uc740 \uc804 \ubaa9\ud3ec\uacbd\ube44\ubd80\uc0ac\ub839\uad00\uc73c\ub85c \ubd80\uc784\ud588\uc744 \ub54c \uad70\uacfc \uacbd\ucc30\ub9cc\uc73c\ub85c \uacf5\uc0b0\uc138\ub825\uc758 \uc794\ub2f9\uc18c\ud0d5\uacfc \uc548\ub155\uc9c8\uc11c \uc720\uc9c0\uc5d0 \uc5b4\ub824\uc6c0\uc774 \ub9ce\uc558\uae30\ub54c\ubb38\uc5d0 \ubaa9\ud3ec\uc2dc\ub0b4 \uccad\ub144\ub2e8\uacfc \uc5ec\uc131\uccad\ub144\ub2e8 \ub4f1 \uc560\uad6d\ub2e8\uccb4\ub4e4\uc744 \ubaa8\ub450 \ub3d9\uc6d0\ud574 \uc791\uc804\uc744 \ubcf4\uc870\ud558\uac8c \ud558\uc600\uc73c\uba70, \ud2b9\ud788 \ud574\uc0c1\uc791\uc804\uc774 \uc911\uc694\ud588\ub2e4\uace0 \ud310\ub2e8\ud588\uae30 \ub54c\ubb38\uc5d0 \ub2f9\uc2dc \uc9c0\uc5ed\uc720\uc9c0 \uc624\uc7ac\uade0\uc5d0\uac8c \uc9c0\uc2dc, \ud574\uc0c1\ubc29\uc704\ub300\ub97c \ucc3d\uc124\ud574 \uae40\ub300\uc911\uc5d0\uac8c \ubd80\ub300\uc7a5\uc744 \ub9e1\uacbc\ub2e4\uace0 \ud574\uba85\ud588\ub2e4.", "paragraph_id": "6488203-85-1", "paragraph_question": "question: \uae40\ub300\uc911\uc758 \ubaa9\ud3ec\ud574\uc0c1\ubc29\uc704\ub300 \uacbd\ub825\uc744 \uc99d\uba85\ud574\uc900 \uc7ac\ubbf8 \ud574\ubcd1\ub300 \uc608\ube44\uc5ed \uc900\uc7a5\uc740?, context: \uc774\uc5b4 \ud574\uad70 \uacbd\ube44\ub300 \uc18c\uc18d \ubaa9\ud3ec\ud574\uc0c1\ubc29\uc704\ub300 \uc804\ub0a8\uc9c0\uad6c \ubd80\ub300\uc7a5\uc744 \ub9e1\uc544 \ub2f9\uc2dc \uc790\uc2e0\uc758 \ud574\uc6b4\ud68c\uc0ac\uc5d0\uc11c \ubcf4\uc720\ud558\uace0 \uc788\ub358 \uc120\ubc15\uc744 \ub3d9\uc6d0\ud574 \ucc38\uc804\ud588\ub2e4\uace0 \uc99d\uc5b8\ud588\ub2e4. \uadf8\ub7ec\ub098 \uc2e0\ud55c\uad6d\ub2f9\uc740 \ubaa9\ud3ec\ud574\uc0c1\ubc29\uc704\ub300\uac00 \uc720\ub839 \uad70\ubd80\ub300\ub77c\uace0 \uc8fc\uc7a5\ud588\uace0, \uad6d\ubbfc\ud68c\uc758 \uce21\uc5d0\uc120 \ud574\uc0c1\ubc29\uc704\ub300\ub294 1950\ub144 \uc721\uad70\ubcf8\ubd80 \uc791\uc804\uba85\ub839 \uc81c38\ud638\uc640 \uad6d\ubc29\uad70\uc0ac\uc5f0\uad6c\uc18c\uc5d0\uc11c \ubc1c\ud589\ud55c \uad6d\ubc29\uc0ac\uc5f0\ud45c\uc5d0 \ubd84\uba85\ud788 \uadfc\uac70\uac00 \uc788\ub2e4\uace0 \ubc18\ubc15\ud558\ub294 \ub4f1 \uc591\uce21\uc758 \uacf5\ubc29\uc774 \uc774\uc5b4\uc84c\ub2e4. \uc774\ud6c4 \uad6d\ubbfc\ud68c\uc758 \ucc9c\uc6a9\ud0dd \uc758\uc6d0\uc774 \ubbf8\uad6d\uc5d0 \uac70\uc8fc\uc911\uc778 \ud574\ubcd1\ub300 \uc608\ube44\uc5ed \uc900\uc7a5 \uc1a1\uc778\uba85\uc744 \ub9cc\ub098 \uadf8\uc758 \uc99d\uc5b8\ub85d\uacfc \uc9c1\uc811 \uc4f4 \uc99d\uba85\uc11c \ub4f1\uc744 \uac00\uc838\uc640 \uacf5\uac1c\ud558\uace0, \uc1a1\uc778\uba85\uc740 \uc804 \ubaa9\ud3ec\uacbd\ube44\ubd80\uc0ac\ub839\uad00\uc73c\ub85c \ubd80\uc784\ud588\uc744 \ub54c \uad70\uacfc \uacbd\ucc30\ub9cc\uc73c\ub85c \uacf5\uc0b0\uc138\ub825\uc758 \uc794\ub2f9\uc18c\ud0d5\uacfc \uc548\ub155\uc9c8\uc11c \uc720\uc9c0\uc5d0 \uc5b4\ub824\uc6c0\uc774 \ub9ce\uc558\uae30\ub54c\ubb38\uc5d0 \ubaa9\ud3ec\uc2dc\ub0b4 \uccad\ub144\ub2e8\uacfc \uc5ec\uc131\uccad\ub144\ub2e8 \ub4f1 \uc560\uad6d\ub2e8\uccb4\ub4e4\uc744 \ubaa8\ub450 \ub3d9\uc6d0\ud574 \uc791\uc804\uc744 \ubcf4\uc870\ud558\uac8c \ud558\uc600\uc73c\uba70, \ud2b9\ud788 \ud574\uc0c1\uc791\uc804\uc774 \uc911\uc694\ud588\ub2e4\uace0 \ud310\ub2e8\ud588\uae30 \ub54c\ubb38\uc5d0 \ub2f9\uc2dc \uc9c0\uc5ed\uc720\uc9c0 \uc624\uc7ac\uade0\uc5d0\uac8c \uc9c0\uc2dc, \ud574\uc0c1\ubc29\uc704\ub300\ub97c \ucc3d\uc124\ud574 \uae40\ub300\uc911\uc5d0\uac8c \ubd80\ub300\uc7a5\uc744 \ub9e1\uacbc\ub2e4\uace0 \ud574\uba85\ud588\ub2e4."} {"question": "JAXA\uac00 2007\ub144 9\uc6d4 14\uc77c \ubc1c\uc0ac\ud55c \ub2ec \ud0d0\uc0ac \uc704\uc131\uc740?", "paragraph": "\uc77c\ubcf8 \uc6b0\uc8fc\ud56d\uacf5\uc5f0\uad6c\uac1c\ubc1c\uae30\uad6c(JAXA)\ub294 \uc77c\ubcf8\uc758 \uc6b0\uc8fc\uae30\uad00\uc73c\ub85c \uc6b0\uc8fc\uc640 \ud589\uc131 \uc5f0\uad6c\ub97c \uac10\ub3c5\ud558\uace0, \ud56d\uacf5\uae30 \uc0b0\uc5c5\uacfc \ub85c\ucf13\uacfc \uc704\uc131 \uac1c\ubc1c\uc744 \uc9c0\ud718\ud558\uace0 \uc788\ub2e4. \uc774 \uae30\uad6c\ub294 \uad6d\uc81c \uc6b0\uc8fc\uc815\uac70\uc7a5\uc5d0 \ucc38\uc5ec\ud558\uace0 \uc788\uc73c\uba70 \uc77c\ubcf8 \uc2e4\ud5d8 \ubaa8\ub4c8\uc744 \uad6d\uc81c\uc6b0\uc8fc\uc815\uac70\uc7a5\uc5d0 \ucd94\uac00\ud558\uc600\ub2e4. JAXA\ub294 \uae08\uc131 \uae30\ud6c4 \ud0d0\uc0ac \uc778\uacf5\uc704\uc131(PLANET-C)\uc744 2010\ub144\uc5d0 \uc3d8\uc544\uc62c\ub9ac\ub294 \uac83\uacfc \uac19\uc740 \uc6b0\uc8fc \ud0d0\ud5d8 \uacc4\ud68d\uc744 \uac16\uace0 \uc788\ub2e4. \ub610\ud55c JAXA\ub294 2013\ub144\uc5d0 \uc218\uc131\ud0d0\uc0ac\uc704\uc131\uc744 \uc3d8\uc544\uc62c\ub9ac\ub824\uace0 \uacc4\ud68d\ud558\uace0 \uc788\uc73c\uba70 2030\ub144\uc5d0\ub294 \ub2ec\uc5d0 \uc2dd\ubbfc\uc9c0\ub97c \uac74\uc124\ud558\ub824\uace0 \ud558\uace0 \uc788\ub2e4. JAXA\ub294 2007\ub144 9\uc6d4 14\uc77c\uc5d0 \ub2ec \ud0d0\uc0ac \uc704\uc131\uc778 \uc140\ub808\ub124(SELENE, \uc77c\ubcf8\uc5b4: \u304b\u3050\u3084 \uac00\uad6c\uc57c/\uce74\uad6c\uc57c)\ub97c \ubc1c\uc0ac\ud588\ub2e4. \uc140\ub808\ub124\ub294 \ubbf8\uad6d\uc758 \uc544\ud3f4\ub85c \uacc4\ud68d \uc774\ud6c4\uc5d0 \uac00\uc7a5 \ud070 \uaddc\ubaa8\uc758 \ub2ec \ud0d0\uc0ac \uacc4\ud68d\uc774\ub2e4. \uc774 \uacc4\ud68d\uc758 \ubaa9\ud45c\ub294 \ub2ec\uc758 \uae30\uc6d0\uacfc \ubc1c\ub2ec\uc5d0 \ub300\ud55c \uc790\ub8cc\ub97c \ubaa8\uc73c\ub294 \uac83\uc774\ub2e4. \uc704\uc131\uc740 2007\ub144 10\uc6d4 4\uc77c\uc5d0 \ub2ec \uacf5\uc804 \uada4\ub3c4\uc5d0 \uc9c4\uc785\ud558\uc600\ub2e4.", "answer": "\uc140\ub808\ub124", "sentence": "JAXA\ub294 2007\ub144 9\uc6d4 14\uc77c\uc5d0 \ub2ec \ud0d0\uc0ac \uc704\uc131\uc778 \uc140\ub808\ub124 (SELENE, \uc77c\ubcf8\uc5b4: \u304b\u3050\u3084 \uac00\uad6c\uc57c/\uce74\uad6c\uc57c)\ub97c \ubc1c\uc0ac\ud588\ub2e4.", "paragraph_sentence": "\uc77c\ubcf8 \uc6b0\uc8fc\ud56d\uacf5\uc5f0\uad6c\uac1c\ubc1c\uae30\uad6c(JAXA)\ub294 \uc77c\ubcf8\uc758 \uc6b0\uc8fc\uae30\uad00\uc73c\ub85c \uc6b0\uc8fc\uc640 \ud589\uc131 \uc5f0\uad6c\ub97c \uac10\ub3c5\ud558\uace0, \ud56d\uacf5\uae30 \uc0b0\uc5c5\uacfc \ub85c\ucf13\uacfc \uc704\uc131 \uac1c\ubc1c\uc744 \uc9c0\ud718\ud558\uace0 \uc788\ub2e4. \uc774 \uae30\uad6c\ub294 \uad6d\uc81c \uc6b0\uc8fc\uc815\uac70\uc7a5\uc5d0 \ucc38\uc5ec\ud558\uace0 \uc788\uc73c\uba70 \uc77c\ubcf8 \uc2e4\ud5d8 \ubaa8\ub4c8\uc744 \uad6d\uc81c\uc6b0\uc8fc\uc815\uac70\uc7a5\uc5d0 \ucd94\uac00\ud558\uc600\ub2e4. JAXA\ub294 \uae08\uc131 \uae30\ud6c4 \ud0d0\uc0ac \uc778\uacf5\uc704\uc131(PLANET-C)\uc744 2010\ub144\uc5d0 \uc3d8\uc544\uc62c\ub9ac\ub294 \uac83\uacfc \uac19\uc740 \uc6b0\uc8fc \ud0d0\ud5d8 \uacc4\ud68d\uc744 \uac16\uace0 \uc788\ub2e4. \ub610\ud55c JAXA\ub294 2013\ub144\uc5d0 \uc218\uc131\ud0d0\uc0ac\uc704\uc131\uc744 \uc3d8\uc544\uc62c\ub9ac\ub824\uace0 \uacc4\ud68d\ud558\uace0 \uc788\uc73c\uba70 2030\ub144\uc5d0\ub294 \ub2ec\uc5d0 \uc2dd\ubbfc\uc9c0\ub97c \uac74\uc124\ud558\ub824\uace0 \ud558\uace0 \uc788\ub2e4. JAXA\ub294 2007\ub144 9\uc6d4 14\uc77c\uc5d0 \ub2ec \ud0d0\uc0ac \uc704\uc131\uc778 \uc140\ub808\ub124 (SELENE, \uc77c\ubcf8\uc5b4: \u304b\u3050\u3084 \uac00\uad6c\uc57c/\uce74\uad6c\uc57c)\ub97c \ubc1c\uc0ac\ud588\ub2e4. \uc140\ub808\ub124\ub294 \ubbf8\uad6d\uc758 \uc544\ud3f4\ub85c \uacc4\ud68d \uc774\ud6c4\uc5d0 \uac00\uc7a5 \ud070 \uaddc\ubaa8\uc758 \ub2ec \ud0d0\uc0ac \uacc4\ud68d\uc774\ub2e4. \uc774 \uacc4\ud68d\uc758 \ubaa9\ud45c\ub294 \ub2ec\uc758 \uae30\uc6d0\uacfc \ubc1c\ub2ec\uc5d0 \ub300\ud55c \uc790\ub8cc\ub97c \ubaa8\uc73c\ub294 \uac83\uc774\ub2e4. \uc704\uc131\uc740 2007\ub144 10\uc6d4 4\uc77c\uc5d0 \ub2ec \uacf5\uc804 \uada4\ub3c4\uc5d0 \uc9c4\uc785\ud558\uc600\ub2e4.", "paragraph_answer": "\uc77c\ubcf8 \uc6b0\uc8fc\ud56d\uacf5\uc5f0\uad6c\uac1c\ubc1c\uae30\uad6c(JAXA)\ub294 \uc77c\ubcf8\uc758 \uc6b0\uc8fc\uae30\uad00\uc73c\ub85c \uc6b0\uc8fc\uc640 \ud589\uc131 \uc5f0\uad6c\ub97c \uac10\ub3c5\ud558\uace0, \ud56d\uacf5\uae30 \uc0b0\uc5c5\uacfc \ub85c\ucf13\uacfc \uc704\uc131 \uac1c\ubc1c\uc744 \uc9c0\ud718\ud558\uace0 \uc788\ub2e4. \uc774 \uae30\uad6c\ub294 \uad6d\uc81c \uc6b0\uc8fc\uc815\uac70\uc7a5\uc5d0 \ucc38\uc5ec\ud558\uace0 \uc788\uc73c\uba70 \uc77c\ubcf8 \uc2e4\ud5d8 \ubaa8\ub4c8\uc744 \uad6d\uc81c\uc6b0\uc8fc\uc815\uac70\uc7a5\uc5d0 \ucd94\uac00\ud558\uc600\ub2e4. JAXA\ub294 \uae08\uc131 \uae30\ud6c4 \ud0d0\uc0ac \uc778\uacf5\uc704\uc131(PLANET-C)\uc744 2010\ub144\uc5d0 \uc3d8\uc544\uc62c\ub9ac\ub294 \uac83\uacfc \uac19\uc740 \uc6b0\uc8fc \ud0d0\ud5d8 \uacc4\ud68d\uc744 \uac16\uace0 \uc788\ub2e4. \ub610\ud55c JAXA\ub294 2013\ub144\uc5d0 \uc218\uc131\ud0d0\uc0ac\uc704\uc131\uc744 \uc3d8\uc544\uc62c\ub9ac\ub824\uace0 \uacc4\ud68d\ud558\uace0 \uc788\uc73c\uba70 2030\ub144\uc5d0\ub294 \ub2ec\uc5d0 \uc2dd\ubbfc\uc9c0\ub97c \uac74\uc124\ud558\ub824\uace0 \ud558\uace0 \uc788\ub2e4. JAXA\ub294 2007\ub144 9\uc6d4 14\uc77c\uc5d0 \ub2ec \ud0d0\uc0ac \uc704\uc131\uc778 \uc140\ub808\ub124 (SELENE, \uc77c\ubcf8\uc5b4: \u304b\u3050\u3084 \uac00\uad6c\uc57c/\uce74\uad6c\uc57c)\ub97c \ubc1c\uc0ac\ud588\ub2e4. \uc140\ub808\ub124\ub294 \ubbf8\uad6d\uc758 \uc544\ud3f4\ub85c \uacc4\ud68d \uc774\ud6c4\uc5d0 \uac00\uc7a5 \ud070 \uaddc\ubaa8\uc758 \ub2ec \ud0d0\uc0ac \uacc4\ud68d\uc774\ub2e4. \uc774 \uacc4\ud68d\uc758 \ubaa9\ud45c\ub294 \ub2ec\uc758 \uae30\uc6d0\uacfc \ubc1c\ub2ec\uc5d0 \ub300\ud55c \uc790\ub8cc\ub97c \ubaa8\uc73c\ub294 \uac83\uc774\ub2e4. \uc704\uc131\uc740 2007\ub144 10\uc6d4 4\uc77c\uc5d0 \ub2ec \uacf5\uc804 \uada4\ub3c4\uc5d0 \uc9c4\uc785\ud558\uc600\ub2e4.", "sentence_answer": "JAXA\ub294 2007\ub144 9\uc6d4 14\uc77c\uc5d0 \ub2ec \ud0d0\uc0ac \uc704\uc131\uc778 \uc140\ub808\ub124 (SELENE, \uc77c\ubcf8\uc5b4: \u304b\u3050\u3084 \uac00\uad6c\uc57c/\uce74\uad6c\uc57c)\ub97c \ubc1c\uc0ac\ud588\ub2e4.", "paragraph_id": "6657580-26-0", "paragraph_question": "question: JAXA\uac00 2007\ub144 9\uc6d4 14\uc77c \ubc1c\uc0ac\ud55c \ub2ec \ud0d0\uc0ac \uc704\uc131\uc740?, context: \uc77c\ubcf8 \uc6b0\uc8fc\ud56d\uacf5\uc5f0\uad6c\uac1c\ubc1c\uae30\uad6c(JAXA)\ub294 \uc77c\ubcf8\uc758 \uc6b0\uc8fc\uae30\uad00\uc73c\ub85c \uc6b0\uc8fc\uc640 \ud589\uc131 \uc5f0\uad6c\ub97c \uac10\ub3c5\ud558\uace0, \ud56d\uacf5\uae30 \uc0b0\uc5c5\uacfc \ub85c\ucf13\uacfc \uc704\uc131 \uac1c\ubc1c\uc744 \uc9c0\ud718\ud558\uace0 \uc788\ub2e4. \uc774 \uae30\uad6c\ub294 \uad6d\uc81c \uc6b0\uc8fc\uc815\uac70\uc7a5\uc5d0 \ucc38\uc5ec\ud558\uace0 \uc788\uc73c\uba70 \uc77c\ubcf8 \uc2e4\ud5d8 \ubaa8\ub4c8\uc744 \uad6d\uc81c\uc6b0\uc8fc\uc815\uac70\uc7a5\uc5d0 \ucd94\uac00\ud558\uc600\ub2e4. JAXA\ub294 \uae08\uc131 \uae30\ud6c4 \ud0d0\uc0ac \uc778\uacf5\uc704\uc131(PLANET-C)\uc744 2010\ub144\uc5d0 \uc3d8\uc544\uc62c\ub9ac\ub294 \uac83\uacfc \uac19\uc740 \uc6b0\uc8fc \ud0d0\ud5d8 \uacc4\ud68d\uc744 \uac16\uace0 \uc788\ub2e4. \ub610\ud55c JAXA\ub294 2013\ub144\uc5d0 \uc218\uc131\ud0d0\uc0ac\uc704\uc131\uc744 \uc3d8\uc544\uc62c\ub9ac\ub824\uace0 \uacc4\ud68d\ud558\uace0 \uc788\uc73c\uba70 2030\ub144\uc5d0\ub294 \ub2ec\uc5d0 \uc2dd\ubbfc\uc9c0\ub97c \uac74\uc124\ud558\ub824\uace0 \ud558\uace0 \uc788\ub2e4. JAXA\ub294 2007\ub144 9\uc6d4 14\uc77c\uc5d0 \ub2ec \ud0d0\uc0ac \uc704\uc131\uc778 \uc140\ub808\ub124(SELENE, \uc77c\ubcf8\uc5b4: \u304b\u3050\u3084 \uac00\uad6c\uc57c/\uce74\uad6c\uc57c)\ub97c \ubc1c\uc0ac\ud588\ub2e4. \uc140\ub808\ub124\ub294 \ubbf8\uad6d\uc758 \uc544\ud3f4\ub85c \uacc4\ud68d \uc774\ud6c4\uc5d0 \uac00\uc7a5 \ud070 \uaddc\ubaa8\uc758 \ub2ec \ud0d0\uc0ac \uacc4\ud68d\uc774\ub2e4. \uc774 \uacc4\ud68d\uc758 \ubaa9\ud45c\ub294 \ub2ec\uc758 \uae30\uc6d0\uacfc \ubc1c\ub2ec\uc5d0 \ub300\ud55c \uc790\ub8cc\ub97c \ubaa8\uc73c\ub294 \uac83\uc774\ub2e4. \uc704\uc131\uc740 2007\ub144 10\uc6d4 4\uc77c\uc5d0 \ub2ec \uacf5\uc804 \uada4\ub3c4\uc5d0 \uc9c4\uc785\ud558\uc600\ub2e4."} {"question": "\uba54\ub4dc\uba58\ud5f4\uc758 \uc218\ub3c4\uc0ac\ub97c \ub530\ub77c\ud588\ub358 18\uc138\uae30\uc640 19\uc138\uae30 \uc874\uc7ac\ud55c \ub2e8\uccb4\uc758 \uc774\ub984\uc740?", "paragraph": "\ubc18\ub3c4\ub355\uc8fc\uc758\uc640 \uad00\ub828\ub41c \uc9d1\ub2e8\uc740 \ub2e4\uc591\ud588\ub294\ub370, \u201c\uba54\ub4dc\uba58\ud584(Medmenham)\uc758 \uc218\ub3c4\uc0ac\ub4e4\u201d\uc774\ub780 \uc774\ub984\uc744 \uac00\uc9c4 17\uc138\uae30\uc758 \uc720\ub7fd \uc9c0\uc2dd\uc778 \ud611\ud68c\ub294 \ub3c4\ubc15 \ubc0f \ud5a5\uc5f0, \uc74c\uc8fc, \uace0\uae09 \ub9e4\ucd98\ubd80\uc640 \uc131\uc801\uc778 \uac8c\uc784\uc744 \ud588\ub2e4. 18\uc138\uae30\uc640 19\uc138\uae30\uc5d0 \uc874\uc7ac\ud588\ub358 \u201c\uc9c0\uc625\ubd88 \ud611\ud68c\u201d(Hell-Fire Club) \ud68c\uc6d0\ub4e4\uc740 \ud154\ub818\uacfc \uba54\ub4dc\uba58\ud5f4\uc758 \uc218\ub3c4\uc0ac\ub97c \ub530\ub790\uc73c\uba70, \u201c\ud751\uc0c9\uc758 \ubbf8\uc0ac\u201d(Black Mass)\uc640 \uac19\uc740 \uc885\uad50\uc801\uc778 \uc758\uc2dd\uc774\ub098 \uc74c\uc8fc\uc640 \ub09c\uad50\ub97c \uc990\uacbc\ub2e4. \ub2e4\ub978 \uc9d1\ub2e8\uc5d0\ub294 \uc5d0\ub4dc\uba3c\ub4dc \uceec(Edmund Curll), \ud504\ub80c\uc2dc\uc2a4 \ub370\uc2dc\uc6b0\ub4dc\uacbd(Sir Francis Dashwood), \uadf8\ub9ac\uace0 \uc874 \ubaac\ud14c\uadf8(John Montague), \uc0cc\ub4dc\uc704\uce58 \ubc31\uc791(The Earl of Sandwich) \uac19\uc740 \uc9c0\uc2dd\uc778\uc774 \ucc38\uc5ec\ud588\ub2e4. \uc774\ub4e4\uc740 \uacc4\ubabd\uc801\uc774\uace0 \ubb34\uc2e0\ub860\uc801\uc778 \uc9c0\uc2dd\uc778 \uc9d1\ub2e8\uc774\uc5c8\ub2e4.", "answer": "\uc9c0\uc625\ubd88 \ud611\ud68c", "sentence": "18\uc138\uae30\uc640 19\uc138\uae30\uc5d0 \uc874\uc7ac\ud588\ub358 \u201c \uc9c0\uc625\ubd88 \ud611\ud68c \u201d(Hell-Fire Club) \ud68c\uc6d0\ub4e4\uc740 \ud154\ub818\uacfc \uba54\ub4dc\uba58\ud5f4\uc758 \uc218\ub3c4\uc0ac\ub97c \ub530\ub790\uc73c\uba70, \u201c\ud751\uc0c9\uc758 \ubbf8\uc0ac\u201d(Black Mass)\uc640 \uac19\uc740 \uc885\uad50\uc801\uc778 \uc758\uc2dd\uc774\ub098 \uc74c\uc8fc\uc640 \ub09c\uad50\ub97c \uc990\uacbc\ub2e4.", "paragraph_sentence": "\ubc18\ub3c4\ub355\uc8fc\uc758\uc640 \uad00\ub828\ub41c \uc9d1\ub2e8\uc740 \ub2e4\uc591\ud588\ub294\ub370, \u201c\uba54\ub4dc\uba58\ud584(Medmenham)\uc758 \uc218\ub3c4\uc0ac\ub4e4\u201d\uc774\ub780 \uc774\ub984\uc744 \uac00\uc9c4 17\uc138\uae30\uc758 \uc720\ub7fd \uc9c0\uc2dd\uc778 \ud611\ud68c\ub294 \ub3c4\ubc15 \ubc0f \ud5a5\uc5f0, \uc74c\uc8fc, \uace0\uae09 \ub9e4\ucd98\ubd80\uc640 \uc131\uc801\uc778 \uac8c\uc784\uc744 \ud588\ub2e4. 18\uc138\uae30\uc640 19\uc138\uae30\uc5d0 \uc874\uc7ac\ud588\ub358 \u201c \uc9c0\uc625\ubd88 \ud611\ud68c \u201d(Hell-Fire Club) \ud68c\uc6d0\ub4e4\uc740 \ud154\ub818\uacfc \uba54\ub4dc\uba58\ud5f4\uc758 \uc218\ub3c4\uc0ac\ub97c \ub530\ub790\uc73c\uba70, \u201c\ud751\uc0c9\uc758 \ubbf8\uc0ac\u201d(Black Mass)\uc640 \uac19\uc740 \uc885\uad50\uc801\uc778 \uc758\uc2dd\uc774\ub098 \uc74c\uc8fc\uc640 \ub09c\uad50\ub97c \uc990\uacbc\ub2e4. \ub2e4\ub978 \uc9d1\ub2e8\uc5d0\ub294 \uc5d0\ub4dc\uba3c\ub4dc \uceec(Edmund Curll), \ud504\ub80c\uc2dc\uc2a4 \ub370\uc2dc\uc6b0\ub4dc\uacbd(Sir Francis Dashwood), \uadf8\ub9ac\uace0 \uc874 \ubaac\ud14c\uadf8(John Montague), \uc0cc\ub4dc\uc704\uce58 \ubc31\uc791(The Earl of Sandwich) \uac19\uc740 \uc9c0\uc2dd\uc778\uc774 \ucc38\uc5ec\ud588\ub2e4. \uc774\ub4e4\uc740 \uacc4\ubabd\uc801\uc774\uace0 \ubb34\uc2e0\ub860\uc801\uc778 \uc9c0\uc2dd\uc778 \uc9d1\ub2e8\uc774\uc5c8\ub2e4.", "paragraph_answer": "\ubc18\ub3c4\ub355\uc8fc\uc758\uc640 \uad00\ub828\ub41c \uc9d1\ub2e8\uc740 \ub2e4\uc591\ud588\ub294\ub370, \u201c\uba54\ub4dc\uba58\ud584(Medmenham)\uc758 \uc218\ub3c4\uc0ac\ub4e4\u201d\uc774\ub780 \uc774\ub984\uc744 \uac00\uc9c4 17\uc138\uae30\uc758 \uc720\ub7fd \uc9c0\uc2dd\uc778 \ud611\ud68c\ub294 \ub3c4\ubc15 \ubc0f \ud5a5\uc5f0, \uc74c\uc8fc, \uace0\uae09 \ub9e4\ucd98\ubd80\uc640 \uc131\uc801\uc778 \uac8c\uc784\uc744 \ud588\ub2e4. 18\uc138\uae30\uc640 19\uc138\uae30\uc5d0 \uc874\uc7ac\ud588\ub358 \u201c \uc9c0\uc625\ubd88 \ud611\ud68c \u201d(Hell-Fire Club) \ud68c\uc6d0\ub4e4\uc740 \ud154\ub818\uacfc \uba54\ub4dc\uba58\ud5f4\uc758 \uc218\ub3c4\uc0ac\ub97c \ub530\ub790\uc73c\uba70, \u201c\ud751\uc0c9\uc758 \ubbf8\uc0ac\u201d(Black Mass)\uc640 \uac19\uc740 \uc885\uad50\uc801\uc778 \uc758\uc2dd\uc774\ub098 \uc74c\uc8fc\uc640 \ub09c\uad50\ub97c \uc990\uacbc\ub2e4. \ub2e4\ub978 \uc9d1\ub2e8\uc5d0\ub294 \uc5d0\ub4dc\uba3c\ub4dc \uceec(Edmund Curll), \ud504\ub80c\uc2dc\uc2a4 \ub370\uc2dc\uc6b0\ub4dc\uacbd(Sir Francis Dashwood), \uadf8\ub9ac\uace0 \uc874 \ubaac\ud14c\uadf8(John Montague), \uc0cc\ub4dc\uc704\uce58 \ubc31\uc791(The Earl of Sandwich) \uac19\uc740 \uc9c0\uc2dd\uc778\uc774 \ucc38\uc5ec\ud588\ub2e4. \uc774\ub4e4\uc740 \uacc4\ubabd\uc801\uc774\uace0 \ubb34\uc2e0\ub860\uc801\uc778 \uc9c0\uc2dd\uc778 \uc9d1\ub2e8\uc774\uc5c8\ub2e4.", "sentence_answer": "18\uc138\uae30\uc640 19\uc138\uae30\uc5d0 \uc874\uc7ac\ud588\ub358 \u201c \uc9c0\uc625\ubd88 \ud611\ud68c \u201d(Hell-Fire Club) \ud68c\uc6d0\ub4e4\uc740 \ud154\ub818\uacfc \uba54\ub4dc\uba58\ud5f4\uc758 \uc218\ub3c4\uc0ac\ub97c \ub530\ub790\uc73c\uba70, \u201c\ud751\uc0c9\uc758 \ubbf8\uc0ac\u201d(Black Mass)\uc640 \uac19\uc740 \uc885\uad50\uc801\uc778 \uc758\uc2dd\uc774\ub098 \uc74c\uc8fc\uc640 \ub09c\uad50\ub97c \uc990\uacbc\ub2e4.", "paragraph_id": "6510985-2-0", "paragraph_question": "question: \uba54\ub4dc\uba58\ud5f4\uc758 \uc218\ub3c4\uc0ac\ub97c \ub530\ub77c\ud588\ub358 18\uc138\uae30\uc640 19\uc138\uae30 \uc874\uc7ac\ud55c \ub2e8\uccb4\uc758 \uc774\ub984\uc740?, context: \ubc18\ub3c4\ub355\uc8fc\uc758\uc640 \uad00\ub828\ub41c \uc9d1\ub2e8\uc740 \ub2e4\uc591\ud588\ub294\ub370, \u201c\uba54\ub4dc\uba58\ud584(Medmenham)\uc758 \uc218\ub3c4\uc0ac\ub4e4\u201d\uc774\ub780 \uc774\ub984\uc744 \uac00\uc9c4 17\uc138\uae30\uc758 \uc720\ub7fd \uc9c0\uc2dd\uc778 \ud611\ud68c\ub294 \ub3c4\ubc15 \ubc0f \ud5a5\uc5f0, \uc74c\uc8fc, \uace0\uae09 \ub9e4\ucd98\ubd80\uc640 \uc131\uc801\uc778 \uac8c\uc784\uc744 \ud588\ub2e4. 18\uc138\uae30\uc640 19\uc138\uae30\uc5d0 \uc874\uc7ac\ud588\ub358 \u201c\uc9c0\uc625\ubd88 \ud611\ud68c\u201d(Hell-Fire Club) \ud68c\uc6d0\ub4e4\uc740 \ud154\ub818\uacfc \uba54\ub4dc\uba58\ud5f4\uc758 \uc218\ub3c4\uc0ac\ub97c \ub530\ub790\uc73c\uba70, \u201c\ud751\uc0c9\uc758 \ubbf8\uc0ac\u201d(Black Mass)\uc640 \uac19\uc740 \uc885\uad50\uc801\uc778 \uc758\uc2dd\uc774\ub098 \uc74c\uc8fc\uc640 \ub09c\uad50\ub97c \uc990\uacbc\ub2e4. \ub2e4\ub978 \uc9d1\ub2e8\uc5d0\ub294 \uc5d0\ub4dc\uba3c\ub4dc \uceec(Edmund Curll), \ud504\ub80c\uc2dc\uc2a4 \ub370\uc2dc\uc6b0\ub4dc\uacbd(Sir Francis Dashwood), \uadf8\ub9ac\uace0 \uc874 \ubaac\ud14c\uadf8(John Montague), \uc0cc\ub4dc\uc704\uce58 \ubc31\uc791(The Earl of Sandwich) \uac19\uc740 \uc9c0\uc2dd\uc778\uc774 \ucc38\uc5ec\ud588\ub2e4. \uc774\ub4e4\uc740 \uacc4\ubabd\uc801\uc774\uace0 \ubb34\uc2e0\ub860\uc801\uc778 \uc9c0\uc2dd\uc778 \uc9d1\ub2e8\uc774\uc5c8\ub2e4."} {"question": "\uc804\ud6c4 \uc778\ub825 \ubd80\uc871\uc73c\ub85c \uacf5\uc0ac\uac00 \uc911\ub2e8\ub418\uc5c8\ub2e4\uac00 \uc7ac\uac1c\ub41c \uac83\uc740 \uc5b8\uc81c\uc778\uac00?", "paragraph": "\ub2f9\uc2dc \uadf8 \uc548\uc5d0\ub294 \uc5f0\ubc29 \uc815\ubd80\uac00 20%\uc758 \ubcf4\uc870\uae08\uc744 \uc8fc\ub294 \uac83\uc774 \ud3ec\ud568\ub418\uc5b4 \uc788\uc5c8\ub2e4. C. D. \ud558\uc6b0 \uc5f0\ubc29 \uc7ac\uac74\ubd80 \uc7a5\uad00\uc740 1945\ub144 10\uc6d4 3\uc77c, \uc5f0\ubc29 \uc815\ubd80\uc758 \uc9c0\uc6d0\uc744 \uc57d\uc18d\ud558\uc600\ub2e4. \uadf8\ub7ec\ub098, \uc815\ubd80 \uc9c0\uc6d0\uc740 \uace0\uc6a9 \ubc29\uc548\uc5d0 \ub300\ud55c \uac70\uc808\ub85c \uc778\ud574 \uc774\ub8e8\uc5b4\uc9c0\uc9c0 \uc54a\uc558\ub2e4. \uacb0\uad6d \uacc4\ud68d \uaddc\ubaa8\ub97c 20% \uc815\ub3c4 \uc904\uc5ec \ub2e4\uc2dc \uacf5\uc0ac\uc5d0 \ub4e4\uc5b4\uac08 \uc218 \uc788\uc5c8\ub2e4. \ud038 \uc2a4\ud2b8\ub9ac\ud2b8 \uacf5\uc0ac\ub294 \uc77c\uc2dc\uc801\uc73c\ub85c \uc911\ub2e8\ub418\uc5c8\uace0, \uae30\uc874 4230\ub9cc \ub2ec\ub7ec\uc758 \uc608\uc0b0\uc740 2890\ub9cc \ub2ec\ub7ec\uc5d0 \uac1d\ucc28 \ube44\uc6a9 350\ub9cc \ub2ec\ub7ec\ub85c \uc904\uc5b4\ub4e4\uc5c8\ub2e4. \uc804\ud6c4 \uc778\ub825 \ubd80\uc871\uc73c\ub85c \uc778\ud574 \uacf5\uc0ac\uac00 2\ub144\uac04 \uc9c4\ud589\ub418\uc9c0 \uc54a\ub2e4\uac00 \uacb0\uad6d 1949\ub144 9\uc6d4 8\uc77c \uacf5\uc0ac\uac00 \uc7ac\uac1c\ub418\uc5c8\ub2e4. \uacf5\uc0ac\uac00 \uc7ac\uac1c\ub420 \ub2f9\uc2dc \uc628\ud0c0\ub9ac\uc624 \uc8fc\uc758 \uc218\uc0c1 \ub808\uc774 \ub85c\uc2a8\uc740 \uc5f0\uc124\uc744 \ud1b5\ud574 \"\uc774 \ub0a0\uc740 \ud1a0\ub860\ud1a0 \uc2dc\ubbfc\ub4e4\uc774 \uc218 \ub144\ub3d9\uc548 \ubc14\ub77c\uc654\ub358 \ubc14\ub85c \uadf8 \ub0a0\"\uc774\ub77c\uace0 \ubc1d\ud614\ub2e4. \uc774 \uacf5\uc0ac\ub85c 130\ub9ccm\uc758 \ud1a0\uc9c0\uac00 \uc5c6\uc5b4\uc84c\uace0 1\ub9cc 2700\ud1a4\uc758 \ucca0\uacfc 140\ub9cc \ud1b5\uc758 \uc2dc\uba58\ud2b8\uac00 \ud604\uc7a5\uc5d0 \ud22c\uc785\ub418\uc5c8\ub2e4.", "answer": "1949\ub144 9\uc6d4 8\uc77c", "sentence": "\uacb0\uad6d 1949\ub144 9\uc6d4 8\uc77c \uacf5\uc0ac\uac00 \uc7ac\uac1c\ub418\uc5c8\ub2e4.", "paragraph_sentence": "\ub2f9\uc2dc \uadf8 \uc548\uc5d0\ub294 \uc5f0\ubc29 \uc815\ubd80\uac00 20%\uc758 \ubcf4\uc870\uae08\uc744 \uc8fc\ub294 \uac83\uc774 \ud3ec\ud568\ub418\uc5b4 \uc788\uc5c8\ub2e4. C. D. \ud558\uc6b0 \uc5f0\ubc29 \uc7ac\uac74\ubd80 \uc7a5\uad00\uc740 1945\ub144 10\uc6d4 3\uc77c, \uc5f0\ubc29 \uc815\ubd80\uc758 \uc9c0\uc6d0\uc744 \uc57d\uc18d\ud558\uc600\ub2e4. \uadf8\ub7ec\ub098, \uc815\ubd80 \uc9c0\uc6d0\uc740 \uace0\uc6a9 \ubc29\uc548\uc5d0 \ub300\ud55c \uac70\uc808\ub85c \uc778\ud574 \uc774\ub8e8\uc5b4\uc9c0\uc9c0 \uc54a\uc558\ub2e4. \uacb0\uad6d \uacc4\ud68d \uaddc\ubaa8\ub97c 20% \uc815\ub3c4 \uc904\uc5ec \ub2e4\uc2dc \uacf5\uc0ac\uc5d0 \ub4e4\uc5b4\uac08 \uc218 \uc788\uc5c8\ub2e4. \ud038 \uc2a4\ud2b8\ub9ac\ud2b8 \uacf5\uc0ac\ub294 \uc77c\uc2dc\uc801\uc73c\ub85c \uc911\ub2e8\ub418\uc5c8\uace0, \uae30\uc874 4230\ub9cc \ub2ec\ub7ec\uc758 \uc608\uc0b0\uc740 2890\ub9cc \ub2ec\ub7ec\uc5d0 \uac1d\ucc28 \ube44\uc6a9 350\ub9cc \ub2ec\ub7ec\ub85c \uc904\uc5b4\ub4e4\uc5c8\ub2e4. \uc804\ud6c4 \uc778\ub825 \ubd80\uc871\uc73c\ub85c \uc778\ud574 \uacf5\uc0ac\uac00 2\ub144\uac04 \uc9c4\ud589\ub418\uc9c0 \uc54a\ub2e4\uac00 \uacb0\uad6d 1949\ub144 9\uc6d4 8\uc77c \uacf5\uc0ac\uac00 \uc7ac\uac1c\ub418\uc5c8\ub2e4. \uacf5\uc0ac\uac00 \uc7ac\uac1c\ub420 \ub2f9\uc2dc \uc628\ud0c0\ub9ac\uc624 \uc8fc\uc758 \uc218\uc0c1 \ub808\uc774 \ub85c\uc2a8\uc740 \uc5f0\uc124\uc744 \ud1b5\ud574 \"\uc774 \ub0a0\uc740 \ud1a0\ub860\ud1a0 \uc2dc\ubbfc\ub4e4\uc774 \uc218 \ub144\ub3d9\uc548 \ubc14\ub77c\uc654\ub358 \ubc14\ub85c \uadf8 \ub0a0\"\uc774\ub77c\uace0 \ubc1d\ud614\ub2e4. \uc774 \uacf5\uc0ac\ub85c 130\ub9ccm\uc758 \ud1a0\uc9c0\uac00 \uc5c6\uc5b4\uc84c\uace0 1\ub9cc 2700\ud1a4\uc758 \ucca0\uacfc 140\ub9cc \ud1b5\uc758 \uc2dc\uba58\ud2b8\uac00 \ud604\uc7a5\uc5d0 \ud22c\uc785\ub418\uc5c8\ub2e4.", "paragraph_answer": "\ub2f9\uc2dc \uadf8 \uc548\uc5d0\ub294 \uc5f0\ubc29 \uc815\ubd80\uac00 20%\uc758 \ubcf4\uc870\uae08\uc744 \uc8fc\ub294 \uac83\uc774 \ud3ec\ud568\ub418\uc5b4 \uc788\uc5c8\ub2e4. C. D. \ud558\uc6b0 \uc5f0\ubc29 \uc7ac\uac74\ubd80 \uc7a5\uad00\uc740 1945\ub144 10\uc6d4 3\uc77c, \uc5f0\ubc29 \uc815\ubd80\uc758 \uc9c0\uc6d0\uc744 \uc57d\uc18d\ud558\uc600\ub2e4. \uadf8\ub7ec\ub098, \uc815\ubd80 \uc9c0\uc6d0\uc740 \uace0\uc6a9 \ubc29\uc548\uc5d0 \ub300\ud55c \uac70\uc808\ub85c \uc778\ud574 \uc774\ub8e8\uc5b4\uc9c0\uc9c0 \uc54a\uc558\ub2e4. \uacb0\uad6d \uacc4\ud68d \uaddc\ubaa8\ub97c 20% \uc815\ub3c4 \uc904\uc5ec \ub2e4\uc2dc \uacf5\uc0ac\uc5d0 \ub4e4\uc5b4\uac08 \uc218 \uc788\uc5c8\ub2e4. \ud038 \uc2a4\ud2b8\ub9ac\ud2b8 \uacf5\uc0ac\ub294 \uc77c\uc2dc\uc801\uc73c\ub85c \uc911\ub2e8\ub418\uc5c8\uace0, \uae30\uc874 4230\ub9cc \ub2ec\ub7ec\uc758 \uc608\uc0b0\uc740 2890\ub9cc \ub2ec\ub7ec\uc5d0 \uac1d\ucc28 \ube44\uc6a9 350\ub9cc \ub2ec\ub7ec\ub85c \uc904\uc5b4\ub4e4\uc5c8\ub2e4. \uc804\ud6c4 \uc778\ub825 \ubd80\uc871\uc73c\ub85c \uc778\ud574 \uacf5\uc0ac\uac00 2\ub144\uac04 \uc9c4\ud589\ub418\uc9c0 \uc54a\ub2e4\uac00 \uacb0\uad6d 1949\ub144 9\uc6d4 8\uc77c \uacf5\uc0ac\uac00 \uc7ac\uac1c\ub418\uc5c8\ub2e4. \uacf5\uc0ac\uac00 \uc7ac\uac1c\ub420 \ub2f9\uc2dc \uc628\ud0c0\ub9ac\uc624 \uc8fc\uc758 \uc218\uc0c1 \ub808\uc774 \ub85c\uc2a8\uc740 \uc5f0\uc124\uc744 \ud1b5\ud574 \"\uc774 \ub0a0\uc740 \ud1a0\ub860\ud1a0 \uc2dc\ubbfc\ub4e4\uc774 \uc218 \ub144\ub3d9\uc548 \ubc14\ub77c\uc654\ub358 \ubc14\ub85c \uadf8 \ub0a0\"\uc774\ub77c\uace0 \ubc1d\ud614\ub2e4. \uc774 \uacf5\uc0ac\ub85c 130\ub9ccm\uc758 \ud1a0\uc9c0\uac00 \uc5c6\uc5b4\uc84c\uace0 1\ub9cc 2700\ud1a4\uc758 \ucca0\uacfc 140\ub9cc \ud1b5\uc758 \uc2dc\uba58\ud2b8\uac00 \ud604\uc7a5\uc5d0 \ud22c\uc785\ub418\uc5c8\ub2e4.", "sentence_answer": "\uacb0\uad6d 1949\ub144 9\uc6d4 8\uc77c \uacf5\uc0ac\uac00 \uc7ac\uac1c\ub418\uc5c8\ub2e4.", "paragraph_id": "6566081-2-1", "paragraph_question": "question: \uc804\ud6c4 \uc778\ub825 \ubd80\uc871\uc73c\ub85c \uacf5\uc0ac\uac00 \uc911\ub2e8\ub418\uc5c8\ub2e4\uac00 \uc7ac\uac1c\ub41c \uac83\uc740 \uc5b8\uc81c\uc778\uac00?, context: \ub2f9\uc2dc \uadf8 \uc548\uc5d0\ub294 \uc5f0\ubc29 \uc815\ubd80\uac00 20%\uc758 \ubcf4\uc870\uae08\uc744 \uc8fc\ub294 \uac83\uc774 \ud3ec\ud568\ub418\uc5b4 \uc788\uc5c8\ub2e4. C. D. \ud558\uc6b0 \uc5f0\ubc29 \uc7ac\uac74\ubd80 \uc7a5\uad00\uc740 1945\ub144 10\uc6d4 3\uc77c, \uc5f0\ubc29 \uc815\ubd80\uc758 \uc9c0\uc6d0\uc744 \uc57d\uc18d\ud558\uc600\ub2e4. \uadf8\ub7ec\ub098, \uc815\ubd80 \uc9c0\uc6d0\uc740 \uace0\uc6a9 \ubc29\uc548\uc5d0 \ub300\ud55c \uac70\uc808\ub85c \uc778\ud574 \uc774\ub8e8\uc5b4\uc9c0\uc9c0 \uc54a\uc558\ub2e4. \uacb0\uad6d \uacc4\ud68d \uaddc\ubaa8\ub97c 20% \uc815\ub3c4 \uc904\uc5ec \ub2e4\uc2dc \uacf5\uc0ac\uc5d0 \ub4e4\uc5b4\uac08 \uc218 \uc788\uc5c8\ub2e4. \ud038 \uc2a4\ud2b8\ub9ac\ud2b8 \uacf5\uc0ac\ub294 \uc77c\uc2dc\uc801\uc73c\ub85c \uc911\ub2e8\ub418\uc5c8\uace0, \uae30\uc874 4230\ub9cc \ub2ec\ub7ec\uc758 \uc608\uc0b0\uc740 2890\ub9cc \ub2ec\ub7ec\uc5d0 \uac1d\ucc28 \ube44\uc6a9 350\ub9cc \ub2ec\ub7ec\ub85c \uc904\uc5b4\ub4e4\uc5c8\ub2e4. \uc804\ud6c4 \uc778\ub825 \ubd80\uc871\uc73c\ub85c \uc778\ud574 \uacf5\uc0ac\uac00 2\ub144\uac04 \uc9c4\ud589\ub418\uc9c0 \uc54a\ub2e4\uac00 \uacb0\uad6d 1949\ub144 9\uc6d4 8\uc77c \uacf5\uc0ac\uac00 \uc7ac\uac1c\ub418\uc5c8\ub2e4. \uacf5\uc0ac\uac00 \uc7ac\uac1c\ub420 \ub2f9\uc2dc \uc628\ud0c0\ub9ac\uc624 \uc8fc\uc758 \uc218\uc0c1 \ub808\uc774 \ub85c\uc2a8\uc740 \uc5f0\uc124\uc744 \ud1b5\ud574 \"\uc774 \ub0a0\uc740 \ud1a0\ub860\ud1a0 \uc2dc\ubbfc\ub4e4\uc774 \uc218 \ub144\ub3d9\uc548 \ubc14\ub77c\uc654\ub358 \ubc14\ub85c \uadf8 \ub0a0\"\uc774\ub77c\uace0 \ubc1d\ud614\ub2e4. \uc774 \uacf5\uc0ac\ub85c 130\ub9ccm\uc758 \ud1a0\uc9c0\uac00 \uc5c6\uc5b4\uc84c\uace0 1\ub9cc 2700\ud1a4\uc758 \ucca0\uacfc 140\ub9cc \ud1b5\uc758 \uc2dc\uba58\ud2b8\uac00 \ud604\uc7a5\uc5d0 \ud22c\uc785\ub418\uc5c8\ub2e4."} {"question": "\uc774\uc0c1\ud55c \ub098\ub77c\uc758 \uc568\ub9ac\uc2a4\uc758 \uc624\ub9ac\uc9c0\ub110\ud310\uc744 \ucd9c\ud310\ud55c \ucd9c\ud310\uc0ac\ub294?", "paragraph": "\ub2e4\ucf00\ubaa8\ud1a0 \ub178\ubc14\ub77c\ub294 \uac00\uce58\uad00\uc774 \ub2e4\ub978 \ub85c\ub9ac\ud0c0 \uc0ac\uc774\uc5d0\uc11c\ub3c4, \uc660\uc9c0 \ubc29\uc2dd\uc740 \ub2e4\ub974\uc9c0\ub9cc \ubaa8\ub4e0 \ub85c\ub9ac\ud0c0\uac00 \uac01\ubcc4\ud788 \uc0ac\ub791\ud558\ub294 \uac83\uc774 \uae30\uc801\uc801\uc73c\ub85c \uc874\uc7ac\ud558\ub294\ub370, \uadf8\uac83\uc774 \ub8e8\uc774\uc2a4 \uce90\ub7f4 \uc800 \u300a\uc774\uc0c1\ud55c \ub098\ub77c\uc758 \uc568\ub9ac\uc2a4\u300b\ub77c\uace0 \ub9d0\ud558\uace0 \uc788\ub2e4. \ub610\ud55c, \ub2e4\ucf00\ubaa8\ud1a0 \ub178\ubc14\ub77c\ub294 \u201c\uc568\ub9ac\uc2a4\ub294 \ub85c\ub9ac\ud0c0\uc758 \ubc14\uc774\ube14\uc774 \uc544\ub2c8\ub2e4. \ub85c\ub9ac\ud0c0\uc758 \uad00\uc2ec\uc744 \ub04c\uace0 \uc788\ub294 \uac74 \ucc45 \ub0b4\uc6a9\ubcf4\ub2e4\ub294, \ub9e5\ubc00\ub780 \ucd9c\ud310\uc0ac\uac00 \ucd9c\ud310\ud55c \uc624\ub9ac\uc9c0\ub110\ud310 \u300a\uc774\uc0c1\ud55c \ub098\ub77c\uc758 \uc568\ub9ac\uc2a4\u300b\uc5d0 \ucd94\uac00\ub41c \uc874 \ud14c\ub2c8\uc5bc\uc774 \uadf8\ub9b0 \uc0bd\ud654\uc774\uba70, \uc874 \ud14c\ub2c8\uc5bc\uc774 \uadf8\ub9b0 \uc568\ub9ac\uc2a4\ub294 \ub3c5\ubcf4\ud558\uc5ec \ub2e4\uc591\ud55c \uad7f\uc988\uac00 \ub418\uc5b4 \uc138\uc0c1\uc744 \ub098\ub3cc\uace0\uc788\uace0, \uc568\ub9ac\uc2a4\uc758 \uc5f4\ub82c\ud55c \uceec\ub809\ud130\uac00 \ub418\ub294 \ub85c\ub9ac\ud0c0\ub294 \ub9ce\ub2e4\uace0 \ud55c\ub2e4. \ub85c\ub9ac\ud0c0\uac00 \uc9c0\uc9c0\ud558\ub294 \ud14c\ub2c8\uc5bc, \ub514\uc988\ub2c8, \uac00\ub124\ucf54 \uad6c\ub2c8\uc694\uc2dc\uac00 \uadf8\ub9b0 \uac10\uc131\uc774 \ub2e4\ub978 \uac01\uac01\uc758 \uc568\ub9ac\uc2a4\ub97c, \uc0bc\ub300 \uc568\ub9ac\uc2a4(\u4e09\u5927\u30a2\u30ea\u30b9)\ub77c \uc774\ub984\uc9c0\uc5b4, \uc790\uc2e0\ub3c4 \ubaa8\uc73c\uace0 \uc788\ub2e4.\u201d\ub77c\uace0 \ud558\uc600\uace0,\uc568\ub9ac\uc2a4\uac00 \ub85c\ub9ac\ud0c0\uc5d0\uac8c \uc120\ud638\ub418\ub294 \uc774\uc720\uc5d0 \ub300\ud558\uc5ec \u201c\ub098\uce74\ud558\ub77c \uc900\uc774\uce58(\u4e2d\u539f\u6df3\u4e00), \uce74\ub124\ucf54 \ucfe0\ub2c8\uc694\uc2dc(\u91d1\u5b50\u570b\u7fa9), \ub8e8\uc774\uc2a4 \uce90\ub7f4 (\ud14c\ub2c8\uc5bc\uc740 \uc544\ub2cc?) \ub4f1\uc774 \uadf8\ub9b0 \uc18c\ub140\uc0c1\uc740 \uba85\ubc31\ud558\uac8c \uc5ec\uc131 \uc778\uad8c\uc744 \ubb34\uc2dc\ud558\uace0 \uc788\uc73c\uba70, \uadf8\ub7f0 \ud658\uc0c1\uc744 \uc138\uc0c1\uc758 \uc18c\ub140\ub4e4\uc740 \ubc14\ubcf4 \ucde8\uae09\ud558\uace0 \uc788\uc9c0\ub9cc, \ub0a8\uc131\uc774 \ud558\ub294 \ub108\ubb34\ub098\ub3c4 \uc644\ubcbd\ud55c \uc694\uad6c\ub294, \ub514\uc2a4 \ucee4\ubba4\ub2c8\ucf00\uc774\uc158(\ub300\uc778 \ucee4\ubba4\ub2c8\ucf00\uc774\uc158 \uc7a5\uc560)\uc774 \ud604\uc838\ud558\uba74 \ud604\uc800\ud560 \uc218\ub85d, \uc18c\ub140\ub4e4\uc740 \uac70\uae30\uc5d0 \uc2ac\ub808\uc774\ube0c\uac00 \ub418\uc5b4 \ub0b4 \uc544\uc774\ub374\ud2f0\ud2f0\ub97c \ud655\ubcf4\ud558\ub294 \uac83\uc774 \uac00\ub2a5\ud574\uc9c0\uace0, \uac70\uafb8\ub85c \uc5b4\ub5a4 \ub9dd\uc124\uc784\ub3c4 \uc5c6\uc774 \uc791\ud488\uc5d0 \uacf5\uac10\ud560 \uc218 \uc788\ub2e4.\u201d \ub77c\uace0 \ub9d0\ud588\ub2e4.", "answer": "\ub9e5\ubc00\ub780", "sentence": "\ub85c\ub9ac\ud0c0\uc758 \uad00\uc2ec\uc744 \ub04c\uace0 \uc788\ub294 \uac74 \ucc45 \ub0b4\uc6a9\ubcf4\ub2e4\ub294, \ub9e5\ubc00\ub780 \ucd9c\ud310\uc0ac\uac00 \ucd9c\ud310\ud55c \uc624\ub9ac\uc9c0\ub110\ud310 \u300a\uc774\uc0c1\ud55c \ub098\ub77c\uc758 \uc568\ub9ac\uc2a4\u300b\uc5d0 \ucd94\uac00\ub41c \uc874 \ud14c\ub2c8\uc5bc\uc774 \uadf8\ub9b0 \uc0bd\ud654\uc774\uba70, \uc874 \ud14c\ub2c8\uc5bc\uc774 \uadf8\ub9b0 \uc568\ub9ac\uc2a4\ub294 \ub3c5\ubcf4\ud558\uc5ec \ub2e4\uc591\ud55c \uad7f\uc988\uac00 \ub418\uc5b4 \uc138\uc0c1\uc744 \ub098\ub3cc\uace0\uc788\uace0, \uc568\ub9ac\uc2a4\uc758 \uc5f4\ub82c\ud55c \uceec\ub809\ud130\uac00 \ub418\ub294 \ub85c\ub9ac\ud0c0\ub294 \ub9ce\ub2e4\uace0 \ud55c\ub2e4.", "paragraph_sentence": "\ub2e4\ucf00\ubaa8\ud1a0 \ub178\ubc14\ub77c\ub294 \uac00\uce58\uad00\uc774 \ub2e4\ub978 \ub85c\ub9ac\ud0c0 \uc0ac\uc774\uc5d0\uc11c\ub3c4, \uc660\uc9c0 \ubc29\uc2dd\uc740 \ub2e4\ub974\uc9c0\ub9cc \ubaa8\ub4e0 \ub85c\ub9ac\ud0c0\uac00 \uac01\ubcc4\ud788 \uc0ac\ub791\ud558\ub294 \uac83\uc774 \uae30\uc801\uc801\uc73c\ub85c \uc874\uc7ac\ud558\ub294\ub370, \uadf8\uac83\uc774 \ub8e8\uc774\uc2a4 \uce90\ub7f4 \uc800 \u300a\uc774\uc0c1\ud55c \ub098\ub77c\uc758 \uc568\ub9ac\uc2a4\u300b\ub77c\uace0 \ub9d0\ud558\uace0 \uc788\ub2e4. \ub610\ud55c, \ub2e4\ucf00\ubaa8\ud1a0 \ub178\ubc14\ub77c\ub294 \u201c\uc568\ub9ac\uc2a4\ub294 \ub85c\ub9ac\ud0c0\uc758 \ubc14\uc774\ube14\uc774 \uc544\ub2c8\ub2e4. \ub85c\ub9ac\ud0c0\uc758 \uad00\uc2ec\uc744 \ub04c\uace0 \uc788\ub294 \uac74 \ucc45 \ub0b4\uc6a9\ubcf4\ub2e4\ub294, \ub9e5\ubc00\ub780 \ucd9c\ud310\uc0ac\uac00 \ucd9c\ud310\ud55c \uc624\ub9ac\uc9c0\ub110\ud310 \u300a\uc774\uc0c1\ud55c \ub098\ub77c\uc758 \uc568\ub9ac\uc2a4\u300b\uc5d0 \ucd94\uac00\ub41c \uc874 \ud14c\ub2c8\uc5bc\uc774 \uadf8\ub9b0 \uc0bd\ud654\uc774\uba70, \uc874 \ud14c\ub2c8\uc5bc\uc774 \uadf8\ub9b0 \uc568\ub9ac\uc2a4\ub294 \ub3c5\ubcf4\ud558\uc5ec \ub2e4\uc591\ud55c \uad7f\uc988\uac00 \ub418\uc5b4 \uc138\uc0c1\uc744 \ub098\ub3cc\uace0\uc788\uace0, \uc568\ub9ac\uc2a4\uc758 \uc5f4\ub82c\ud55c \uceec\ub809\ud130\uac00 \ub418\ub294 \ub85c\ub9ac\ud0c0\ub294 \ub9ce\ub2e4\uace0 \ud55c\ub2e4. \ub85c\ub9ac\ud0c0\uac00 \uc9c0\uc9c0\ud558\ub294 \ud14c\ub2c8\uc5bc, \ub514\uc988\ub2c8, \uac00\ub124\ucf54 \uad6c\ub2c8\uc694\uc2dc\uac00 \uadf8\ub9b0 \uac10\uc131\uc774 \ub2e4\ub978 \uac01\uac01\uc758 \uc568\ub9ac\uc2a4\ub97c, \uc0bc\ub300 \uc568\ub9ac\uc2a4(\u4e09\u5927\u30a2\u30ea\u30b9)\ub77c \uc774\ub984\uc9c0\uc5b4, \uc790\uc2e0\ub3c4 \ubaa8\uc73c\uace0 \uc788\ub2e4. \u201d\ub77c\uace0 \ud558\uc600\uace0,\uc568\ub9ac\uc2a4\uac00 \ub85c\ub9ac\ud0c0\uc5d0\uac8c \uc120\ud638\ub418\ub294 \uc774\uc720\uc5d0 \ub300\ud558\uc5ec \u201c\ub098\uce74\ud558\ub77c \uc900\uc774\uce58(\u4e2d\u539f\u6df3\u4e00), \uce74\ub124\ucf54 \ucfe0\ub2c8\uc694\uc2dc(\u91d1\u5b50\u570b\u7fa9), \ub8e8\uc774\uc2a4 \uce90\ub7f4 (\ud14c\ub2c8\uc5bc\uc740 \uc544\ub2cc?) \ub4f1\uc774 \uadf8\ub9b0 \uc18c\ub140\uc0c1\uc740 \uba85\ubc31\ud558\uac8c \uc5ec\uc131 \uc778\uad8c\uc744 \ubb34\uc2dc\ud558\uace0 \uc788\uc73c\uba70, \uadf8\ub7f0 \ud658\uc0c1\uc744 \uc138\uc0c1\uc758 \uc18c\ub140\ub4e4\uc740 \ubc14\ubcf4 \ucde8\uae09\ud558\uace0 \uc788\uc9c0\ub9cc, \ub0a8\uc131\uc774 \ud558\ub294 \ub108\ubb34\ub098\ub3c4 \uc644\ubcbd\ud55c \uc694\uad6c\ub294, \ub514\uc2a4 \ucee4\ubba4\ub2c8\ucf00\uc774\uc158(\ub300\uc778 \ucee4\ubba4\ub2c8\ucf00\uc774\uc158 \uc7a5\uc560)\uc774 \ud604\uc838\ud558\uba74 \ud604\uc800\ud560 \uc218\ub85d, \uc18c\ub140\ub4e4\uc740 \uac70\uae30\uc5d0 \uc2ac\ub808\uc774\ube0c\uac00 \ub418\uc5b4 \ub0b4 \uc544\uc774\ub374\ud2f0\ud2f0\ub97c \ud655\ubcf4\ud558\ub294 \uac83\uc774 \uac00\ub2a5\ud574\uc9c0\uace0, \uac70\uafb8\ub85c \uc5b4\ub5a4 \ub9dd\uc124\uc784\ub3c4 \uc5c6\uc774 \uc791\ud488\uc5d0 \uacf5\uac10\ud560 \uc218 \uc788\ub2e4. \u201d \ub77c\uace0 \ub9d0\ud588\ub2e4.", "paragraph_answer": "\ub2e4\ucf00\ubaa8\ud1a0 \ub178\ubc14\ub77c\ub294 \uac00\uce58\uad00\uc774 \ub2e4\ub978 \ub85c\ub9ac\ud0c0 \uc0ac\uc774\uc5d0\uc11c\ub3c4, \uc660\uc9c0 \ubc29\uc2dd\uc740 \ub2e4\ub974\uc9c0\ub9cc \ubaa8\ub4e0 \ub85c\ub9ac\ud0c0\uac00 \uac01\ubcc4\ud788 \uc0ac\ub791\ud558\ub294 \uac83\uc774 \uae30\uc801\uc801\uc73c\ub85c \uc874\uc7ac\ud558\ub294\ub370, \uadf8\uac83\uc774 \ub8e8\uc774\uc2a4 \uce90\ub7f4 \uc800 \u300a\uc774\uc0c1\ud55c \ub098\ub77c\uc758 \uc568\ub9ac\uc2a4\u300b\ub77c\uace0 \ub9d0\ud558\uace0 \uc788\ub2e4. \ub610\ud55c, \ub2e4\ucf00\ubaa8\ud1a0 \ub178\ubc14\ub77c\ub294 \u201c\uc568\ub9ac\uc2a4\ub294 \ub85c\ub9ac\ud0c0\uc758 \ubc14\uc774\ube14\uc774 \uc544\ub2c8\ub2e4. \ub85c\ub9ac\ud0c0\uc758 \uad00\uc2ec\uc744 \ub04c\uace0 \uc788\ub294 \uac74 \ucc45 \ub0b4\uc6a9\ubcf4\ub2e4\ub294, \ub9e5\ubc00\ub780 \ucd9c\ud310\uc0ac\uac00 \ucd9c\ud310\ud55c \uc624\ub9ac\uc9c0\ub110\ud310 \u300a\uc774\uc0c1\ud55c \ub098\ub77c\uc758 \uc568\ub9ac\uc2a4\u300b\uc5d0 \ucd94\uac00\ub41c \uc874 \ud14c\ub2c8\uc5bc\uc774 \uadf8\ub9b0 \uc0bd\ud654\uc774\uba70, \uc874 \ud14c\ub2c8\uc5bc\uc774 \uadf8\ub9b0 \uc568\ub9ac\uc2a4\ub294 \ub3c5\ubcf4\ud558\uc5ec \ub2e4\uc591\ud55c \uad7f\uc988\uac00 \ub418\uc5b4 \uc138\uc0c1\uc744 \ub098\ub3cc\uace0\uc788\uace0, \uc568\ub9ac\uc2a4\uc758 \uc5f4\ub82c\ud55c \uceec\ub809\ud130\uac00 \ub418\ub294 \ub85c\ub9ac\ud0c0\ub294 \ub9ce\ub2e4\uace0 \ud55c\ub2e4. \ub85c\ub9ac\ud0c0\uac00 \uc9c0\uc9c0\ud558\ub294 \ud14c\ub2c8\uc5bc, \ub514\uc988\ub2c8, \uac00\ub124\ucf54 \uad6c\ub2c8\uc694\uc2dc\uac00 \uadf8\ub9b0 \uac10\uc131\uc774 \ub2e4\ub978 \uac01\uac01\uc758 \uc568\ub9ac\uc2a4\ub97c, \uc0bc\ub300 \uc568\ub9ac\uc2a4(\u4e09\u5927\u30a2\u30ea\u30b9)\ub77c \uc774\ub984\uc9c0\uc5b4, \uc790\uc2e0\ub3c4 \ubaa8\uc73c\uace0 \uc788\ub2e4.\u201d\ub77c\uace0 \ud558\uc600\uace0,\uc568\ub9ac\uc2a4\uac00 \ub85c\ub9ac\ud0c0\uc5d0\uac8c \uc120\ud638\ub418\ub294 \uc774\uc720\uc5d0 \ub300\ud558\uc5ec \u201c\ub098\uce74\ud558\ub77c \uc900\uc774\uce58(\u4e2d\u539f\u6df3\u4e00), \uce74\ub124\ucf54 \ucfe0\ub2c8\uc694\uc2dc(\u91d1\u5b50\u570b\u7fa9), \ub8e8\uc774\uc2a4 \uce90\ub7f4 (\ud14c\ub2c8\uc5bc\uc740 \uc544\ub2cc?) \ub4f1\uc774 \uadf8\ub9b0 \uc18c\ub140\uc0c1\uc740 \uba85\ubc31\ud558\uac8c \uc5ec\uc131 \uc778\uad8c\uc744 \ubb34\uc2dc\ud558\uace0 \uc788\uc73c\uba70, \uadf8\ub7f0 \ud658\uc0c1\uc744 \uc138\uc0c1\uc758 \uc18c\ub140\ub4e4\uc740 \ubc14\ubcf4 \ucde8\uae09\ud558\uace0 \uc788\uc9c0\ub9cc, \ub0a8\uc131\uc774 \ud558\ub294 \ub108\ubb34\ub098\ub3c4 \uc644\ubcbd\ud55c \uc694\uad6c\ub294, \ub514\uc2a4 \ucee4\ubba4\ub2c8\ucf00\uc774\uc158(\ub300\uc778 \ucee4\ubba4\ub2c8\ucf00\uc774\uc158 \uc7a5\uc560)\uc774 \ud604\uc838\ud558\uba74 \ud604\uc800\ud560 \uc218\ub85d, \uc18c\ub140\ub4e4\uc740 \uac70\uae30\uc5d0 \uc2ac\ub808\uc774\ube0c\uac00 \ub418\uc5b4 \ub0b4 \uc544\uc774\ub374\ud2f0\ud2f0\ub97c \ud655\ubcf4\ud558\ub294 \uac83\uc774 \uac00\ub2a5\ud574\uc9c0\uace0, \uac70\uafb8\ub85c \uc5b4\ub5a4 \ub9dd\uc124\uc784\ub3c4 \uc5c6\uc774 \uc791\ud488\uc5d0 \uacf5\uac10\ud560 \uc218 \uc788\ub2e4.\u201d \ub77c\uace0 \ub9d0\ud588\ub2e4.", "sentence_answer": "\ub85c\ub9ac\ud0c0\uc758 \uad00\uc2ec\uc744 \ub04c\uace0 \uc788\ub294 \uac74 \ucc45 \ub0b4\uc6a9\ubcf4\ub2e4\ub294, \ub9e5\ubc00\ub780 \ucd9c\ud310\uc0ac\uac00 \ucd9c\ud310\ud55c \uc624\ub9ac\uc9c0\ub110\ud310 \u300a\uc774\uc0c1\ud55c \ub098\ub77c\uc758 \uc568\ub9ac\uc2a4\u300b\uc5d0 \ucd94\uac00\ub41c \uc874 \ud14c\ub2c8\uc5bc\uc774 \uadf8\ub9b0 \uc0bd\ud654\uc774\uba70, \uc874 \ud14c\ub2c8\uc5bc\uc774 \uadf8\ub9b0 \uc568\ub9ac\uc2a4\ub294 \ub3c5\ubcf4\ud558\uc5ec \ub2e4\uc591\ud55c \uad7f\uc988\uac00 \ub418\uc5b4 \uc138\uc0c1\uc744 \ub098\ub3cc\uace0\uc788\uace0, \uc568\ub9ac\uc2a4\uc758 \uc5f4\ub82c\ud55c \uceec\ub809\ud130\uac00 \ub418\ub294 \ub85c\ub9ac\ud0c0\ub294 \ub9ce\ub2e4\uace0 \ud55c\ub2e4.", "paragraph_id": "6054480-19-1", "paragraph_question": "question: \uc774\uc0c1\ud55c \ub098\ub77c\uc758 \uc568\ub9ac\uc2a4\uc758 \uc624\ub9ac\uc9c0\ub110\ud310\uc744 \ucd9c\ud310\ud55c \ucd9c\ud310\uc0ac\ub294?, context: \ub2e4\ucf00\ubaa8\ud1a0 \ub178\ubc14\ub77c\ub294 \uac00\uce58\uad00\uc774 \ub2e4\ub978 \ub85c\ub9ac\ud0c0 \uc0ac\uc774\uc5d0\uc11c\ub3c4, \uc660\uc9c0 \ubc29\uc2dd\uc740 \ub2e4\ub974\uc9c0\ub9cc \ubaa8\ub4e0 \ub85c\ub9ac\ud0c0\uac00 \uac01\ubcc4\ud788 \uc0ac\ub791\ud558\ub294 \uac83\uc774 \uae30\uc801\uc801\uc73c\ub85c \uc874\uc7ac\ud558\ub294\ub370, \uadf8\uac83\uc774 \ub8e8\uc774\uc2a4 \uce90\ub7f4 \uc800 \u300a\uc774\uc0c1\ud55c \ub098\ub77c\uc758 \uc568\ub9ac\uc2a4\u300b\ub77c\uace0 \ub9d0\ud558\uace0 \uc788\ub2e4. \ub610\ud55c, \ub2e4\ucf00\ubaa8\ud1a0 \ub178\ubc14\ub77c\ub294 \u201c\uc568\ub9ac\uc2a4\ub294 \ub85c\ub9ac\ud0c0\uc758 \ubc14\uc774\ube14\uc774 \uc544\ub2c8\ub2e4. \ub85c\ub9ac\ud0c0\uc758 \uad00\uc2ec\uc744 \ub04c\uace0 \uc788\ub294 \uac74 \ucc45 \ub0b4\uc6a9\ubcf4\ub2e4\ub294, \ub9e5\ubc00\ub780 \ucd9c\ud310\uc0ac\uac00 \ucd9c\ud310\ud55c \uc624\ub9ac\uc9c0\ub110\ud310 \u300a\uc774\uc0c1\ud55c \ub098\ub77c\uc758 \uc568\ub9ac\uc2a4\u300b\uc5d0 \ucd94\uac00\ub41c \uc874 \ud14c\ub2c8\uc5bc\uc774 \uadf8\ub9b0 \uc0bd\ud654\uc774\uba70, \uc874 \ud14c\ub2c8\uc5bc\uc774 \uadf8\ub9b0 \uc568\ub9ac\uc2a4\ub294 \ub3c5\ubcf4\ud558\uc5ec \ub2e4\uc591\ud55c \uad7f\uc988\uac00 \ub418\uc5b4 \uc138\uc0c1\uc744 \ub098\ub3cc\uace0\uc788\uace0, \uc568\ub9ac\uc2a4\uc758 \uc5f4\ub82c\ud55c \uceec\ub809\ud130\uac00 \ub418\ub294 \ub85c\ub9ac\ud0c0\ub294 \ub9ce\ub2e4\uace0 \ud55c\ub2e4. \ub85c\ub9ac\ud0c0\uac00 \uc9c0\uc9c0\ud558\ub294 \ud14c\ub2c8\uc5bc, \ub514\uc988\ub2c8, \uac00\ub124\ucf54 \uad6c\ub2c8\uc694\uc2dc\uac00 \uadf8\ub9b0 \uac10\uc131\uc774 \ub2e4\ub978 \uac01\uac01\uc758 \uc568\ub9ac\uc2a4\ub97c, \uc0bc\ub300 \uc568\ub9ac\uc2a4(\u4e09\u5927\u30a2\u30ea\u30b9)\ub77c \uc774\ub984\uc9c0\uc5b4, \uc790\uc2e0\ub3c4 \ubaa8\uc73c\uace0 \uc788\ub2e4.\u201d\ub77c\uace0 \ud558\uc600\uace0,\uc568\ub9ac\uc2a4\uac00 \ub85c\ub9ac\ud0c0\uc5d0\uac8c \uc120\ud638\ub418\ub294 \uc774\uc720\uc5d0 \ub300\ud558\uc5ec \u201c\ub098\uce74\ud558\ub77c \uc900\uc774\uce58(\u4e2d\u539f\u6df3\u4e00), \uce74\ub124\ucf54 \ucfe0\ub2c8\uc694\uc2dc(\u91d1\u5b50\u570b\u7fa9), \ub8e8\uc774\uc2a4 \uce90\ub7f4 (\ud14c\ub2c8\uc5bc\uc740 \uc544\ub2cc?) \ub4f1\uc774 \uadf8\ub9b0 \uc18c\ub140\uc0c1\uc740 \uba85\ubc31\ud558\uac8c \uc5ec\uc131 \uc778\uad8c\uc744 \ubb34\uc2dc\ud558\uace0 \uc788\uc73c\uba70, \uadf8\ub7f0 \ud658\uc0c1\uc744 \uc138\uc0c1\uc758 \uc18c\ub140\ub4e4\uc740 \ubc14\ubcf4 \ucde8\uae09\ud558\uace0 \uc788\uc9c0\ub9cc, \ub0a8\uc131\uc774 \ud558\ub294 \ub108\ubb34\ub098\ub3c4 \uc644\ubcbd\ud55c \uc694\uad6c\ub294, \ub514\uc2a4 \ucee4\ubba4\ub2c8\ucf00\uc774\uc158(\ub300\uc778 \ucee4\ubba4\ub2c8\ucf00\uc774\uc158 \uc7a5\uc560)\uc774 \ud604\uc838\ud558\uba74 \ud604\uc800\ud560 \uc218\ub85d, \uc18c\ub140\ub4e4\uc740 \uac70\uae30\uc5d0 \uc2ac\ub808\uc774\ube0c\uac00 \ub418\uc5b4 \ub0b4 \uc544\uc774\ub374\ud2f0\ud2f0\ub97c \ud655\ubcf4\ud558\ub294 \uac83\uc774 \uac00\ub2a5\ud574\uc9c0\uace0, \uac70\uafb8\ub85c \uc5b4\ub5a4 \ub9dd\uc124\uc784\ub3c4 \uc5c6\uc774 \uc791\ud488\uc5d0 \uacf5\uac10\ud560 \uc218 \uc788\ub2e4.\u201d \ub77c\uace0 \ub9d0\ud588\ub2e4."} {"question": "\ub274\uce90\uc2ac\uc758 \uc5b4\ub5a4 \uc720\ub2c8\ud3fc\uc740 \uc77c\uad00\uc131\uc774 \uc5c6\uc774 \uc790\uc8fc \ubc14\ub00c\ub294\uac00?", "paragraph": "\ubc18\uba74 \uc5b4\uc6e8\uc774 \uc720\ub2c8\ud3fc\uc740 \uc77c\uad00\uc131\uc774 \ubcc4\ub85c \uc5c6\uc5c8\ub294\ub370 \uc77c\uc815\ud55c \uae30\uc900 \uc5c6\uc774 \uc5b4\uc6e8\uc774 \uc720\ub2c8\ud3fc\uc740 \uc790\uc8fc \ubc14\ub00c\uc5c8\ub2e4. 1914\ub144\ubd80\ud130 1961\ub144\uae4c\uc9c0 \ud770\uc0c9 \uc0c1\uc758\uc5d0 \uac80\uc740\uc0c9 \ud558\uc758\ub97c \ucc29\uc6a9\ud558\uc600\uc73c\uba70, 1966\ub144\uae4c\uc9c0\ub294 \ud770\uc0c9 \ud558\uc758\ub97c \ucc29\uc6a9\ud558\uc600\ub2e4. 1967-68 \uc2dc\uc98c \ub178\ub780\uc0c9 \uc0c1\uc758\uc5d0 \ud30c\ub780\uc0c9 \ud558\uc758\ub97c \uc785\uae30\ub3c4 \ud558\uc600\uc73c\uba70, 1969\ub144\ubd80\ud130 1974\ub144\uae4c\uc9c0 \uc0c1\ud558\uc758 \ubaa8\ub450 \ube68\uac04\uc0c9\uc774\uac70\ub098 \ud30c\ub780\uc0c9\uc778 \uc368\ub4dc \uc720\ub2c8\ud3fc\uc744 \ucc29\uc6a9\ud558\uc600\ub2e4. 1983\ub144\ubd80\ud130 1988\ub144\uae4c\uc9c0\ub294 \uc0c1\ud558\uc758 \ubaa8\ub450 \ud68c\uc0c9\uc744 \uc785\uc5c8\ub2e4. 1993\ub144 \uc774\ud6c4\ubd80\ud130 \uc5b4\uc6e8\uc774 \uc720\ub2c8\ud3fc\uc740 1\ub144\uc5d0 \ud55c\ubc88\uc529 \ub9e4\ub144 \ubcc0\uacbd\ub418\uc5c8\ub2e4. \uac00\uc7a5 \ud3c9\ubc94\ud558\uc9c0 \uc54a\uc558\ub358 \uc720\ub2c8\ud3fc\uc774 1995-96 \uc2dc\uc98c\uacfc, 2006-07 \uc2dc\uc98c\uc5d0 \uc4f0\uc600\ub358 \uac08\uc0c9\uacfc \ud30c\ub780\uc0c9\uc758 \uac00\ub85c\uc904\ubb34\ub2ac \uc720\ub2c8\ud3fc\uc774\uc5c8\ub294\ub370, \uc774\uac83\uc740 \uc6e8\uc2a4\ud2b8 \uc5d4\ub4dc\ub97c \ucd94\uc5b5\ud558\uae30 \uc704\ud55c \uac83\uc774\ub2e4.", "answer": "\uc5b4\uc6e8\uc774 \uc720\ub2c8\ud3fc", "sentence": "\ubc18\uba74 \uc5b4\uc6e8\uc774 \uc720\ub2c8\ud3fc \uc740 \uc77c\uad00\uc131\uc774 \ubcc4\ub85c \uc5c6\uc5c8\ub294\ub370 \uc77c\uc815\ud55c \uae30\uc900 \uc5c6\uc774 \uc5b4\uc6e8\uc774 \uc720\ub2c8\ud3fc\uc740 \uc790\uc8fc \ubc14\ub00c\uc5c8\ub2e4.", "paragraph_sentence": " \ubc18\uba74 \uc5b4\uc6e8\uc774 \uc720\ub2c8\ud3fc \uc740 \uc77c\uad00\uc131\uc774 \ubcc4\ub85c \uc5c6\uc5c8\ub294\ub370 \uc77c\uc815\ud55c \uae30\uc900 \uc5c6\uc774 \uc5b4\uc6e8\uc774 \uc720\ub2c8\ud3fc\uc740 \uc790\uc8fc \ubc14\ub00c\uc5c8\ub2e4. 1914\ub144\ubd80\ud130 1961\ub144\uae4c\uc9c0 \ud770\uc0c9 \uc0c1\uc758\uc5d0 \uac80\uc740\uc0c9 \ud558\uc758\ub97c \ucc29\uc6a9\ud558\uc600\uc73c\uba70, 1966\ub144\uae4c\uc9c0\ub294 \ud770\uc0c9 \ud558\uc758\ub97c \ucc29\uc6a9\ud558\uc600\ub2e4. 1967-68 \uc2dc\uc98c \ub178\ub780\uc0c9 \uc0c1\uc758\uc5d0 \ud30c\ub780\uc0c9 \ud558\uc758\ub97c \uc785\uae30\ub3c4 \ud558\uc600\uc73c\uba70, 1969\ub144\ubd80\ud130 1974\ub144\uae4c\uc9c0 \uc0c1\ud558\uc758 \ubaa8\ub450 \ube68\uac04\uc0c9\uc774\uac70\ub098 \ud30c\ub780\uc0c9\uc778 \uc368\ub4dc \uc720\ub2c8\ud3fc\uc744 \ucc29\uc6a9\ud558\uc600\ub2e4. 1983\ub144\ubd80\ud130 1988\ub144\uae4c\uc9c0\ub294 \uc0c1\ud558\uc758 \ubaa8\ub450 \ud68c\uc0c9\uc744 \uc785\uc5c8\ub2e4. 1993\ub144 \uc774\ud6c4\ubd80\ud130 \uc5b4\uc6e8\uc774 \uc720\ub2c8\ud3fc\uc740 1\ub144\uc5d0 \ud55c\ubc88\uc529 \ub9e4\ub144 \ubcc0\uacbd\ub418\uc5c8\ub2e4. \uac00\uc7a5 \ud3c9\ubc94\ud558\uc9c0 \uc54a\uc558\ub358 \uc720\ub2c8\ud3fc\uc774 1995-96 \uc2dc\uc98c\uacfc, 2006-07 \uc2dc\uc98c\uc5d0 \uc4f0\uc600\ub358 \uac08\uc0c9\uacfc \ud30c\ub780\uc0c9\uc758 \uac00\ub85c\uc904\ubb34\ub2ac \uc720\ub2c8\ud3fc\uc774\uc5c8\ub294\ub370, \uc774\uac83\uc740 \uc6e8\uc2a4\ud2b8 \uc5d4\ub4dc\ub97c \ucd94\uc5b5\ud558\uae30 \uc704\ud55c \uac83\uc774\ub2e4.", "paragraph_answer": "\ubc18\uba74 \uc5b4\uc6e8\uc774 \uc720\ub2c8\ud3fc \uc740 \uc77c\uad00\uc131\uc774 \ubcc4\ub85c \uc5c6\uc5c8\ub294\ub370 \uc77c\uc815\ud55c \uae30\uc900 \uc5c6\uc774 \uc5b4\uc6e8\uc774 \uc720\ub2c8\ud3fc\uc740 \uc790\uc8fc \ubc14\ub00c\uc5c8\ub2e4. 1914\ub144\ubd80\ud130 1961\ub144\uae4c\uc9c0 \ud770\uc0c9 \uc0c1\uc758\uc5d0 \uac80\uc740\uc0c9 \ud558\uc758\ub97c \ucc29\uc6a9\ud558\uc600\uc73c\uba70, 1966\ub144\uae4c\uc9c0\ub294 \ud770\uc0c9 \ud558\uc758\ub97c \ucc29\uc6a9\ud558\uc600\ub2e4. 1967-68 \uc2dc\uc98c \ub178\ub780\uc0c9 \uc0c1\uc758\uc5d0 \ud30c\ub780\uc0c9 \ud558\uc758\ub97c \uc785\uae30\ub3c4 \ud558\uc600\uc73c\uba70, 1969\ub144\ubd80\ud130 1974\ub144\uae4c\uc9c0 \uc0c1\ud558\uc758 \ubaa8\ub450 \ube68\uac04\uc0c9\uc774\uac70\ub098 \ud30c\ub780\uc0c9\uc778 \uc368\ub4dc \uc720\ub2c8\ud3fc\uc744 \ucc29\uc6a9\ud558\uc600\ub2e4. 1983\ub144\ubd80\ud130 1988\ub144\uae4c\uc9c0\ub294 \uc0c1\ud558\uc758 \ubaa8\ub450 \ud68c\uc0c9\uc744 \uc785\uc5c8\ub2e4. 1993\ub144 \uc774\ud6c4\ubd80\ud130 \uc5b4\uc6e8\uc774 \uc720\ub2c8\ud3fc\uc740 1\ub144\uc5d0 \ud55c\ubc88\uc529 \ub9e4\ub144 \ubcc0\uacbd\ub418\uc5c8\ub2e4. \uac00\uc7a5 \ud3c9\ubc94\ud558\uc9c0 \uc54a\uc558\ub358 \uc720\ub2c8\ud3fc\uc774 1995-96 \uc2dc\uc98c\uacfc, 2006-07 \uc2dc\uc98c\uc5d0 \uc4f0\uc600\ub358 \uac08\uc0c9\uacfc \ud30c\ub780\uc0c9\uc758 \uac00\ub85c\uc904\ubb34\ub2ac \uc720\ub2c8\ud3fc\uc774\uc5c8\ub294\ub370, \uc774\uac83\uc740 \uc6e8\uc2a4\ud2b8 \uc5d4\ub4dc\ub97c \ucd94\uc5b5\ud558\uae30 \uc704\ud55c \uac83\uc774\ub2e4.", "sentence_answer": "\ubc18\uba74 \uc5b4\uc6e8\uc774 \uc720\ub2c8\ud3fc \uc740 \uc77c\uad00\uc131\uc774 \ubcc4\ub85c \uc5c6\uc5c8\ub294\ub370 \uc77c\uc815\ud55c \uae30\uc900 \uc5c6\uc774 \uc5b4\uc6e8\uc774 \uc720\ub2c8\ud3fc\uc740 \uc790\uc8fc \ubc14\ub00c\uc5c8\ub2e4.", "paragraph_id": "6489372-5-0", "paragraph_question": "question: \ub274\uce90\uc2ac\uc758 \uc5b4\ub5a4 \uc720\ub2c8\ud3fc\uc740 \uc77c\uad00\uc131\uc774 \uc5c6\uc774 \uc790\uc8fc \ubc14\ub00c\ub294\uac00?, context: \ubc18\uba74 \uc5b4\uc6e8\uc774 \uc720\ub2c8\ud3fc\uc740 \uc77c\uad00\uc131\uc774 \ubcc4\ub85c \uc5c6\uc5c8\ub294\ub370 \uc77c\uc815\ud55c \uae30\uc900 \uc5c6\uc774 \uc5b4\uc6e8\uc774 \uc720\ub2c8\ud3fc\uc740 \uc790\uc8fc \ubc14\ub00c\uc5c8\ub2e4. 1914\ub144\ubd80\ud130 1961\ub144\uae4c\uc9c0 \ud770\uc0c9 \uc0c1\uc758\uc5d0 \uac80\uc740\uc0c9 \ud558\uc758\ub97c \ucc29\uc6a9\ud558\uc600\uc73c\uba70, 1966\ub144\uae4c\uc9c0\ub294 \ud770\uc0c9 \ud558\uc758\ub97c \ucc29\uc6a9\ud558\uc600\ub2e4. 1967-68 \uc2dc\uc98c \ub178\ub780\uc0c9 \uc0c1\uc758\uc5d0 \ud30c\ub780\uc0c9 \ud558\uc758\ub97c \uc785\uae30\ub3c4 \ud558\uc600\uc73c\uba70, 1969\ub144\ubd80\ud130 1974\ub144\uae4c\uc9c0 \uc0c1\ud558\uc758 \ubaa8\ub450 \ube68\uac04\uc0c9\uc774\uac70\ub098 \ud30c\ub780\uc0c9\uc778 \uc368\ub4dc \uc720\ub2c8\ud3fc\uc744 \ucc29\uc6a9\ud558\uc600\ub2e4. 1983\ub144\ubd80\ud130 1988\ub144\uae4c\uc9c0\ub294 \uc0c1\ud558\uc758 \ubaa8\ub450 \ud68c\uc0c9\uc744 \uc785\uc5c8\ub2e4. 1993\ub144 \uc774\ud6c4\ubd80\ud130 \uc5b4\uc6e8\uc774 \uc720\ub2c8\ud3fc\uc740 1\ub144\uc5d0 \ud55c\ubc88\uc529 \ub9e4\ub144 \ubcc0\uacbd\ub418\uc5c8\ub2e4. \uac00\uc7a5 \ud3c9\ubc94\ud558\uc9c0 \uc54a\uc558\ub358 \uc720\ub2c8\ud3fc\uc774 1995-96 \uc2dc\uc98c\uacfc, 2006-07 \uc2dc\uc98c\uc5d0 \uc4f0\uc600\ub358 \uac08\uc0c9\uacfc \ud30c\ub780\uc0c9\uc758 \uac00\ub85c\uc904\ubb34\ub2ac \uc720\ub2c8\ud3fc\uc774\uc5c8\ub294\ub370, \uc774\uac83\uc740 \uc6e8\uc2a4\ud2b8 \uc5d4\ub4dc\ub97c \ucd94\uc5b5\ud558\uae30 \uc704\ud55c \uac83\uc774\ub2e4."} {"question": "\ubcf4\uc2a4\ud1a4 \ub300\ud559\uad50 \uc18c\uc18d\uc774\uba70 \ud53c\uc5b4\uc120\uc5d0 \uad00\ud55c \uc120\uad6c\uc801\uc778 \uc5f0\uad6c\uc790\uc758 \uc774\ub984\uc740?", "paragraph": "\ud53c\uc5b4\uc120\uc5d0 \uad00\ud55c \uc120\uad6c\uc801\uc778 \uc5f0\uad6c\uac00\ub85c\ub294 \ubbf8\uad6d \ubcf4\uc2a4\ud1a4\ub300\ud559\uad50\uc758 \ub370\uc774\ub098 L. \ub85c\ubc84\ud2b8(Dana L. Robert)\uad50\uc218\uac00 \uc788\uc73c\uba70 \ud3c9\ud0dd\ub300\ud559\uad50 \uad50\uc218\uc778 \uae40\ubb38\uae30, \ub958\uc6d0\ub82c, \uc2e0\ud604\uc218, \uc548\uba85\uc900, \uc720\uc724\uc885, \uc774\uad11\ud76c, \uc870\uc0c1\uc5f4, \uc591\uc720\uc131, \ud55c\ub3d9\uad6c, \ub958\uc6d0\ub82c, \uae40\ud0dc\uc12d \ub4f1\uc774 \ud53c\uc5b4\uc120\uc5d0 \uad00\ud55c \uc5f0\uad6c\ub17c\ubb38\ub4e4\uc744 \ud65c\ubc1c\ud558\uac8c \ubc1c\ud45c\ud558\uace0 \uc788\ub2e4. 2009\ub144\uc5d0\ub294 \uace0\uad11\uad6d\ubc15\uc0ac\uac00 \"\ud53c\uc5b4\uc120\uc758 \uc120\uad50\uc804\ub7b5\uacfc \uc2e4\uc81c\uc5d0 \uad00\ud55c \uc5f0\uad6c\"\ub85c, 2011\ub144\uc5d0\ub294 \uc804\ub300\uacbd\ubc15\uc0ac\uac00 \"\uc544\ub354 \ud53c\uc5b4\uc120\uc758 \ubcc0\uc99d\uc801 \ubc29\ubc95\ub860\uc73c\ub85c\uc11c\uc758 \uc131\uc11c \ud574\uc11d \uc6d0\ub9ac\"\ub85c \uac01\uac01 \ud3c9\ud0dd\ub300\ud559\uad50 \ud53c\uc5b4\uc120\uc2e0\ud559\uc804\ubb38\ub300\ud559\uc6d0\uc5d0\uc11c \ubc15\uc0ac\ud559\uc704\ub97c \ubc1b\uc558\ub2e4. 2010\ub144\uc5d0\ub294 \uc870\uc0c1\uc5f4\ubc15\uc0ac\uac00 \u300c\ud53c\uc5b4\uc120\uae30\ub150\uc131\uacbd\ud559\uc6d0\u300d(\uac1c\uc815\ud310, 2011)\uc744 \ucd9c\ud310\ud558\uc600\ub2e4. \ud53c\uc5b4\uc120 \uad00\ub828 \uc5f0\uad6c \ubc1c\ud45c\ub294 \ub9e4\ub144 \ud53c\uc5b4\uc120\uae30\ub150\uc131\uacbd\uc5f0\uad6c\uc6d0\uc5d0\uc11c 2 \ucc28\ub840\uac00 \uc788\uc73c\uba70, \ud53c\uc5b4\uc120\uc2e0\ud559\ub17c\ub2e8\uc744 \ud1b5\ud574 \ucd9c\ud310\ub418\uace0 \uc788\ub2e4.", "answer": "\ub370\uc774\ub098 L. \ub85c\ubc84\ud2b8", "sentence": "\ubbf8\uad6d \ubcf4\uc2a4\ud1a4\ub300\ud559\uad50\uc758 \ub370\uc774\ub098 L. \ub85c\ubc84\ud2b8 (Dana L. Robert)\uad50\uc218\uac00 \uc788\uc73c\uba70 \ud3c9\ud0dd\ub300\ud559\uad50 \uad50\uc218\uc778 \uae40\ubb38\uae30, \ub958\uc6d0\ub82c, \uc2e0\ud604\uc218, \uc548\uba85\uc900, \uc720\uc724\uc885, \uc774\uad11\ud76c, \uc870\uc0c1\uc5f4, \uc591\uc720\uc131, \ud55c\ub3d9\uad6c, \ub958\uc6d0\ub82c, \uae40\ud0dc\uc12d \ub4f1\uc774 \ud53c\uc5b4\uc120\uc5d0 \uad00\ud55c \uc5f0\uad6c\ub17c\ubb38\ub4e4\uc744 \ud65c\ubc1c\ud558\uac8c \ubc1c\ud45c\ud558\uace0 \uc788\ub2e4.", "paragraph_sentence": "\ud53c\uc5b4\uc120\uc5d0 \uad00\ud55c \uc120\uad6c\uc801\uc778 \uc5f0\uad6c\uac00\ub85c\ub294 \ubbf8\uad6d \ubcf4\uc2a4\ud1a4\ub300\ud559\uad50\uc758 \ub370\uc774\ub098 L. \ub85c\ubc84\ud2b8 (Dana L. Robert)\uad50\uc218\uac00 \uc788\uc73c\uba70 \ud3c9\ud0dd\ub300\ud559\uad50 \uad50\uc218\uc778 \uae40\ubb38\uae30, \ub958\uc6d0\ub82c, \uc2e0\ud604\uc218, \uc548\uba85\uc900, \uc720\uc724\uc885, \uc774\uad11\ud76c, \uc870\uc0c1\uc5f4, \uc591\uc720\uc131, \ud55c\ub3d9\uad6c, \ub958\uc6d0\ub82c, \uae40\ud0dc\uc12d \ub4f1\uc774 \ud53c\uc5b4\uc120\uc5d0 \uad00\ud55c \uc5f0\uad6c\ub17c\ubb38\ub4e4\uc744 \ud65c\ubc1c\ud558\uac8c \ubc1c\ud45c\ud558\uace0 \uc788\ub2e4. 2009\ub144\uc5d0\ub294 \uace0\uad11\uad6d\ubc15\uc0ac\uac00 \"\ud53c\uc5b4\uc120\uc758 \uc120\uad50\uc804\ub7b5\uacfc \uc2e4\uc81c\uc5d0 \uad00\ud55c \uc5f0\uad6c\"\ub85c, 2011\ub144\uc5d0\ub294 \uc804\ub300\uacbd\ubc15\uc0ac\uac00 \"\uc544\ub354 \ud53c\uc5b4\uc120\uc758 \ubcc0\uc99d\uc801 \ubc29\ubc95\ub860\uc73c\ub85c\uc11c\uc758 \uc131\uc11c \ud574\uc11d \uc6d0\ub9ac\"\ub85c \uac01\uac01 \ud3c9\ud0dd\ub300\ud559\uad50 \ud53c\uc5b4\uc120\uc2e0\ud559\uc804\ubb38\ub300\ud559\uc6d0\uc5d0\uc11c \ubc15\uc0ac\ud559\uc704\ub97c \ubc1b\uc558\ub2e4. 2010\ub144\uc5d0\ub294 \uc870\uc0c1\uc5f4\ubc15\uc0ac\uac00 \u300c\ud53c\uc5b4\uc120\uae30\ub150\uc131\uacbd\ud559\uc6d0\u300d(\uac1c\uc815\ud310, 2011)\uc744 \ucd9c\ud310\ud558\uc600\ub2e4. \ud53c\uc5b4\uc120 \uad00\ub828 \uc5f0\uad6c \ubc1c\ud45c\ub294 \ub9e4\ub144 \ud53c\uc5b4\uc120\uae30\ub150\uc131\uacbd\uc5f0\uad6c\uc6d0\uc5d0\uc11c 2 \ucc28\ub840\uac00 \uc788\uc73c\uba70, \ud53c\uc5b4\uc120\uc2e0\ud559\ub17c\ub2e8\uc744 \ud1b5\ud574 \ucd9c\ud310\ub418\uace0 \uc788\ub2e4.", "paragraph_answer": "\ud53c\uc5b4\uc120\uc5d0 \uad00\ud55c \uc120\uad6c\uc801\uc778 \uc5f0\uad6c\uac00\ub85c\ub294 \ubbf8\uad6d \ubcf4\uc2a4\ud1a4\ub300\ud559\uad50\uc758 \ub370\uc774\ub098 L. \ub85c\ubc84\ud2b8 (Dana L. Robert)\uad50\uc218\uac00 \uc788\uc73c\uba70 \ud3c9\ud0dd\ub300\ud559\uad50 \uad50\uc218\uc778 \uae40\ubb38\uae30, \ub958\uc6d0\ub82c, \uc2e0\ud604\uc218, \uc548\uba85\uc900, \uc720\uc724\uc885, \uc774\uad11\ud76c, \uc870\uc0c1\uc5f4, \uc591\uc720\uc131, \ud55c\ub3d9\uad6c, \ub958\uc6d0\ub82c, \uae40\ud0dc\uc12d \ub4f1\uc774 \ud53c\uc5b4\uc120\uc5d0 \uad00\ud55c \uc5f0\uad6c\ub17c\ubb38\ub4e4\uc744 \ud65c\ubc1c\ud558\uac8c \ubc1c\ud45c\ud558\uace0 \uc788\ub2e4. 2009\ub144\uc5d0\ub294 \uace0\uad11\uad6d\ubc15\uc0ac\uac00 \"\ud53c\uc5b4\uc120\uc758 \uc120\uad50\uc804\ub7b5\uacfc \uc2e4\uc81c\uc5d0 \uad00\ud55c \uc5f0\uad6c\"\ub85c, 2011\ub144\uc5d0\ub294 \uc804\ub300\uacbd\ubc15\uc0ac\uac00 \"\uc544\ub354 \ud53c\uc5b4\uc120\uc758 \ubcc0\uc99d\uc801 \ubc29\ubc95\ub860\uc73c\ub85c\uc11c\uc758 \uc131\uc11c \ud574\uc11d \uc6d0\ub9ac\"\ub85c \uac01\uac01 \ud3c9\ud0dd\ub300\ud559\uad50 \ud53c\uc5b4\uc120\uc2e0\ud559\uc804\ubb38\ub300\ud559\uc6d0\uc5d0\uc11c \ubc15\uc0ac\ud559\uc704\ub97c \ubc1b\uc558\ub2e4. 2010\ub144\uc5d0\ub294 \uc870\uc0c1\uc5f4\ubc15\uc0ac\uac00 \u300c\ud53c\uc5b4\uc120\uae30\ub150\uc131\uacbd\ud559\uc6d0\u300d(\uac1c\uc815\ud310, 2011)\uc744 \ucd9c\ud310\ud558\uc600\ub2e4. \ud53c\uc5b4\uc120 \uad00\ub828 \uc5f0\uad6c \ubc1c\ud45c\ub294 \ub9e4\ub144 \ud53c\uc5b4\uc120\uae30\ub150\uc131\uacbd\uc5f0\uad6c\uc6d0\uc5d0\uc11c 2 \ucc28\ub840\uac00 \uc788\uc73c\uba70, \ud53c\uc5b4\uc120\uc2e0\ud559\ub17c\ub2e8\uc744 \ud1b5\ud574 \ucd9c\ud310\ub418\uace0 \uc788\ub2e4.", "sentence_answer": "\ubbf8\uad6d \ubcf4\uc2a4\ud1a4\ub300\ud559\uad50\uc758 \ub370\uc774\ub098 L. \ub85c\ubc84\ud2b8 (Dana L. Robert)\uad50\uc218\uac00 \uc788\uc73c\uba70 \ud3c9\ud0dd\ub300\ud559\uad50 \uad50\uc218\uc778 \uae40\ubb38\uae30, \ub958\uc6d0\ub82c, \uc2e0\ud604\uc218, \uc548\uba85\uc900, \uc720\uc724\uc885, \uc774\uad11\ud76c, \uc870\uc0c1\uc5f4, \uc591\uc720\uc131, \ud55c\ub3d9\uad6c, \ub958\uc6d0\ub82c, \uae40\ud0dc\uc12d \ub4f1\uc774 \ud53c\uc5b4\uc120\uc5d0 \uad00\ud55c \uc5f0\uad6c\ub17c\ubb38\ub4e4\uc744 \ud65c\ubc1c\ud558\uac8c \ubc1c\ud45c\ud558\uace0 \uc788\ub2e4.", "paragraph_id": "6463995-1-0", "paragraph_question": "question: \ubcf4\uc2a4\ud1a4 \ub300\ud559\uad50 \uc18c\uc18d\uc774\uba70 \ud53c\uc5b4\uc120\uc5d0 \uad00\ud55c \uc120\uad6c\uc801\uc778 \uc5f0\uad6c\uc790\uc758 \uc774\ub984\uc740?, context: \ud53c\uc5b4\uc120\uc5d0 \uad00\ud55c \uc120\uad6c\uc801\uc778 \uc5f0\uad6c\uac00\ub85c\ub294 \ubbf8\uad6d \ubcf4\uc2a4\ud1a4\ub300\ud559\uad50\uc758 \ub370\uc774\ub098 L. \ub85c\ubc84\ud2b8(Dana L. Robert)\uad50\uc218\uac00 \uc788\uc73c\uba70 \ud3c9\ud0dd\ub300\ud559\uad50 \uad50\uc218\uc778 \uae40\ubb38\uae30, \ub958\uc6d0\ub82c, \uc2e0\ud604\uc218, \uc548\uba85\uc900, \uc720\uc724\uc885, \uc774\uad11\ud76c, \uc870\uc0c1\uc5f4, \uc591\uc720\uc131, \ud55c\ub3d9\uad6c, \ub958\uc6d0\ub82c, \uae40\ud0dc\uc12d \ub4f1\uc774 \ud53c\uc5b4\uc120\uc5d0 \uad00\ud55c \uc5f0\uad6c\ub17c\ubb38\ub4e4\uc744 \ud65c\ubc1c\ud558\uac8c \ubc1c\ud45c\ud558\uace0 \uc788\ub2e4. 2009\ub144\uc5d0\ub294 \uace0\uad11\uad6d\ubc15\uc0ac\uac00 \"\ud53c\uc5b4\uc120\uc758 \uc120\uad50\uc804\ub7b5\uacfc \uc2e4\uc81c\uc5d0 \uad00\ud55c \uc5f0\uad6c\"\ub85c, 2011\ub144\uc5d0\ub294 \uc804\ub300\uacbd\ubc15\uc0ac\uac00 \"\uc544\ub354 \ud53c\uc5b4\uc120\uc758 \ubcc0\uc99d\uc801 \ubc29\ubc95\ub860\uc73c\ub85c\uc11c\uc758 \uc131\uc11c \ud574\uc11d \uc6d0\ub9ac\"\ub85c \uac01\uac01 \ud3c9\ud0dd\ub300\ud559\uad50 \ud53c\uc5b4\uc120\uc2e0\ud559\uc804\ubb38\ub300\ud559\uc6d0\uc5d0\uc11c \ubc15\uc0ac\ud559\uc704\ub97c \ubc1b\uc558\ub2e4. 2010\ub144\uc5d0\ub294 \uc870\uc0c1\uc5f4\ubc15\uc0ac\uac00 \u300c\ud53c\uc5b4\uc120\uae30\ub150\uc131\uacbd\ud559\uc6d0\u300d(\uac1c\uc815\ud310, 2011)\uc744 \ucd9c\ud310\ud558\uc600\ub2e4. \ud53c\uc5b4\uc120 \uad00\ub828 \uc5f0\uad6c \ubc1c\ud45c\ub294 \ub9e4\ub144 \ud53c\uc5b4\uc120\uae30\ub150\uc131\uacbd\uc5f0\uad6c\uc6d0\uc5d0\uc11c 2 \ucc28\ub840\uac00 \uc788\uc73c\uba70, \ud53c\uc5b4\uc120\uc2e0\ud559\ub17c\ub2e8\uc744 \ud1b5\ud574 \ucd9c\ud310\ub418\uace0 \uc788\ub2e4."} {"question": "Ms.\uc758 \uc6a9\uc5b4\ub97c \ub110\ub9ac \uc54c\ub9b0 \uacc4\uae30\uac00 \ub41c \uc7a1\uc9c0 \uc774\ub984\uc740?", "paragraph": "1961\ub144, \uc274\ub77c \ub9c8\uc774\ud074\uc988\ub294 \uadf8\ub140\uc758 \ub8f8\uba54\uc774\ud2b8\uc5d0\uac8c \ubc30\ub2ec\ub41c \ud55c \ubd80\uc758 \u300a\ub274\uc2a4 & \ub808\ud130\uc2a4\u300b(News & Letters)\uc758 \uc8fc\uc18c\ub780\uc5d0 \uc778\uc1c4 \uc624\ub958\ub77c\uace0 \uadf8\ub140\uac00 \uc0dd\uac01\ud588\ub358 \uac83\uc744 \ubcf4\uac8c \ub418\uc5c8\uc744 \ub54c, \uc774 \uc6a9\uc5b4\ub97c \uc0ac\uc6a9\ud558\ub824\uace0 \uc2dc\ub3c4\ub97c \ud558\uac8c \ub41c\ub2e4. \ub9c8\uc774\ud074\uc988\ub294 \ub0a8\uc131\uc5d0\uac8c \uadc0\uc18d\ub418\uc9c0 \uc54a\uc740 \uc5ec\uc131\uc5d0 \ub300\ud55c \ud638\uce6d\uc744 \ucc3e\uace0 \uc788\uc5c8\ub2e4. \uadf8\ub140\ub294 \ud604\uc7ac\ub294 \ud754\ud55c \uc6a9\uc5b4\uc778 Miss\uc640 Mrs.\uc758 \uad6c\ubd84\uc774 Mistress\uc5d0\uc11c \ud30c\uc0dd\ub418\uc5b4 \ub098\uc628 \uac83\uc744 \uc54c\uc558\uc9c0\ub9cc, \uc774\uc81c\ub294 \ud3c9\ud310\uc774 \ub098\uc05c \ub73b\uc744 \ub0b4\ud3ec\ud55c \uc6d0\ub798\uc758 \ud638\uce6d\uc744 \uc5ec\uc131\ub4e4\uc774 \uc0ac\uc6a9\ud558\ub294 \uac83\uc744 \ub204\uad6c\ub3c4 \uc81c\uc548\ud558\uc9c0 \ubabb\ud588\ub2e4. \uc0c8\ub85c\uc6b4 \ud638\uce6d \uc0ac\uc6a9\uc744 \uc7a5\ub824\ud558\ub824\ub294 \uadf8\ub140\uc758 \ub178\ub825\uc740 \ucc98\uc74c\uc5d0\ub294 \ubb34\uc2dc\ub2f9\ud588\ub2e4. 1971\ub144\uacbd, \uc5ec\uc131\uc6b4\ub3d9\uac00 \uc9d1\ub2e8\uacfc WBAI \ub77c\ub514\uc624 \uc778\ud130\ubdf0\ub97c \ud558\ub294 \ub3d9\uc548 \ub9c8\uc774\ud074\uc988\ub294 Ms.\uc758 \uc0ac\uc6a9\uc744 \uc81c\uc548\ud588\ub2e4. \uae00\ub85c\ub9ac\uc544 \uc2a4\ud0c0\uc774\ub118\uc758 \ud55c \uce5c\uad6c\uac00 \uadf8 \uc778\ud130\ubdf0\ub97c \ub4e3\uace0\uc11c, \uadf8\ub140\uc758 \uc0c8\ub85c\uc6b4 \uc7a1\uc9c0\uba85\uc73c\ub85c \uadf8 \uc774\ub984\uc744 \uc81c\uc548\ud588\ub2e4. \u300a\ubbf8\uc988 \ub9e4\uac70\uc9c4\u300b\uc758 \uc778\uae30\ub294 \ub9c8\uce68\ub0b4 \uadf8 \uc6a9\uc5b4\ub97c \ub110\ub9ac \uc0ac\uc6a9\ud558\uac8c \ub9cc\ub4e4\uc5c8\ub2e4. 1972\ub144 2\uc6d4, \ubbf8\uad6d \uc815\ubd80 \uc778\uc1c4\uad6d\uc740 \uc815\ubd80 \uacf5\ubb38\uc11c\uc5d0\uc11c Ms.\uc758 \uc0ac\uc6a9\uc744 \uc2b9\uc778\ud558\uc600\ub2e4.", "answer": "\ubbf8\uc988 \ub9e4\uac70\uc9c4", "sentence": "\u300a \ubbf8\uc988 \ub9e4\uac70\uc9c4 \u300b\uc758 \uc778\uae30\ub294 \ub9c8\uce68\ub0b4 \uadf8 \uc6a9\uc5b4\ub97c \ub110\ub9ac \uc0ac\uc6a9\ud558\uac8c \ub9cc\ub4e4\uc5c8\ub2e4.", "paragraph_sentence": "1961\ub144, \uc274\ub77c \ub9c8\uc774\ud074\uc988\ub294 \uadf8\ub140\uc758 \ub8f8\uba54\uc774\ud2b8\uc5d0\uac8c \ubc30\ub2ec\ub41c \ud55c \ubd80\uc758 \u300a\ub274\uc2a4 & \ub808\ud130\uc2a4\u300b(News & Letters)\uc758 \uc8fc\uc18c\ub780\uc5d0 \uc778\uc1c4 \uc624\ub958\ub77c\uace0 \uadf8\ub140\uac00 \uc0dd\uac01\ud588\ub358 \uac83\uc744 \ubcf4\uac8c \ub418\uc5c8\uc744 \ub54c, \uc774 \uc6a9\uc5b4\ub97c \uc0ac\uc6a9\ud558\ub824\uace0 \uc2dc\ub3c4\ub97c \ud558\uac8c \ub41c\ub2e4. \ub9c8\uc774\ud074\uc988\ub294 \ub0a8\uc131\uc5d0\uac8c \uadc0\uc18d\ub418\uc9c0 \uc54a\uc740 \uc5ec\uc131\uc5d0 \ub300\ud55c \ud638\uce6d\uc744 \ucc3e\uace0 \uc788\uc5c8\ub2e4. \uadf8\ub140\ub294 \ud604\uc7ac\ub294 \ud754\ud55c \uc6a9\uc5b4\uc778 Miss\uc640 Mrs. \uc758 \uad6c\ubd84\uc774 Mistress\uc5d0\uc11c \ud30c\uc0dd\ub418\uc5b4 \ub098\uc628 \uac83\uc744 \uc54c\uc558\uc9c0\ub9cc, \uc774\uc81c\ub294 \ud3c9\ud310\uc774 \ub098\uc05c \ub73b\uc744 \ub0b4\ud3ec\ud55c \uc6d0\ub798\uc758 \ud638\uce6d\uc744 \uc5ec\uc131\ub4e4\uc774 \uc0ac\uc6a9\ud558\ub294 \uac83\uc744 \ub204\uad6c\ub3c4 \uc81c\uc548\ud558\uc9c0 \ubabb\ud588\ub2e4. \uc0c8\ub85c\uc6b4 \ud638\uce6d \uc0ac\uc6a9\uc744 \uc7a5\ub824\ud558\ub824\ub294 \uadf8\ub140\uc758 \ub178\ub825\uc740 \ucc98\uc74c\uc5d0\ub294 \ubb34\uc2dc\ub2f9\ud588\ub2e4. 1971\ub144\uacbd, \uc5ec\uc131\uc6b4\ub3d9\uac00 \uc9d1\ub2e8\uacfc WBAI \ub77c\ub514\uc624 \uc778\ud130\ubdf0\ub97c \ud558\ub294 \ub3d9\uc548 \ub9c8\uc774\ud074\uc988\ub294 Ms. \uc758 \uc0ac\uc6a9\uc744 \uc81c\uc548\ud588\ub2e4. \uae00\ub85c\ub9ac\uc544 \uc2a4\ud0c0\uc774\ub118\uc758 \ud55c \uce5c\uad6c\uac00 \uadf8 \uc778\ud130\ubdf0\ub97c \ub4e3\uace0\uc11c, \uadf8\ub140\uc758 \uc0c8\ub85c\uc6b4 \uc7a1\uc9c0\uba85\uc73c\ub85c \uadf8 \uc774\ub984\uc744 \uc81c\uc548\ud588\ub2e4. \u300a \ubbf8\uc988 \ub9e4\uac70\uc9c4 \u300b\uc758 \uc778\uae30\ub294 \ub9c8\uce68\ub0b4 \uadf8 \uc6a9\uc5b4\ub97c \ub110\ub9ac \uc0ac\uc6a9\ud558\uac8c \ub9cc\ub4e4\uc5c8\ub2e4. 1972\ub144 2\uc6d4, \ubbf8\uad6d \uc815\ubd80 \uc778\uc1c4\uad6d\uc740 \uc815\ubd80 \uacf5\ubb38\uc11c\uc5d0\uc11c Ms. \uc758 \uc0ac\uc6a9\uc744 \uc2b9\uc778\ud558\uc600\ub2e4.", "paragraph_answer": "1961\ub144, \uc274\ub77c \ub9c8\uc774\ud074\uc988\ub294 \uadf8\ub140\uc758 \ub8f8\uba54\uc774\ud2b8\uc5d0\uac8c \ubc30\ub2ec\ub41c \ud55c \ubd80\uc758 \u300a\ub274\uc2a4 & \ub808\ud130\uc2a4\u300b(News & Letters)\uc758 \uc8fc\uc18c\ub780\uc5d0 \uc778\uc1c4 \uc624\ub958\ub77c\uace0 \uadf8\ub140\uac00 \uc0dd\uac01\ud588\ub358 \uac83\uc744 \ubcf4\uac8c \ub418\uc5c8\uc744 \ub54c, \uc774 \uc6a9\uc5b4\ub97c \uc0ac\uc6a9\ud558\ub824\uace0 \uc2dc\ub3c4\ub97c \ud558\uac8c \ub41c\ub2e4. \ub9c8\uc774\ud074\uc988\ub294 \ub0a8\uc131\uc5d0\uac8c \uadc0\uc18d\ub418\uc9c0 \uc54a\uc740 \uc5ec\uc131\uc5d0 \ub300\ud55c \ud638\uce6d\uc744 \ucc3e\uace0 \uc788\uc5c8\ub2e4. \uadf8\ub140\ub294 \ud604\uc7ac\ub294 \ud754\ud55c \uc6a9\uc5b4\uc778 Miss\uc640 Mrs.\uc758 \uad6c\ubd84\uc774 Mistress\uc5d0\uc11c \ud30c\uc0dd\ub418\uc5b4 \ub098\uc628 \uac83\uc744 \uc54c\uc558\uc9c0\ub9cc, \uc774\uc81c\ub294 \ud3c9\ud310\uc774 \ub098\uc05c \ub73b\uc744 \ub0b4\ud3ec\ud55c \uc6d0\ub798\uc758 \ud638\uce6d\uc744 \uc5ec\uc131\ub4e4\uc774 \uc0ac\uc6a9\ud558\ub294 \uac83\uc744 \ub204\uad6c\ub3c4 \uc81c\uc548\ud558\uc9c0 \ubabb\ud588\ub2e4. \uc0c8\ub85c\uc6b4 \ud638\uce6d \uc0ac\uc6a9\uc744 \uc7a5\ub824\ud558\ub824\ub294 \uadf8\ub140\uc758 \ub178\ub825\uc740 \ucc98\uc74c\uc5d0\ub294 \ubb34\uc2dc\ub2f9\ud588\ub2e4. 1971\ub144\uacbd, \uc5ec\uc131\uc6b4\ub3d9\uac00 \uc9d1\ub2e8\uacfc WBAI \ub77c\ub514\uc624 \uc778\ud130\ubdf0\ub97c \ud558\ub294 \ub3d9\uc548 \ub9c8\uc774\ud074\uc988\ub294 Ms.\uc758 \uc0ac\uc6a9\uc744 \uc81c\uc548\ud588\ub2e4. \uae00\ub85c\ub9ac\uc544 \uc2a4\ud0c0\uc774\ub118\uc758 \ud55c \uce5c\uad6c\uac00 \uadf8 \uc778\ud130\ubdf0\ub97c \ub4e3\uace0\uc11c, \uadf8\ub140\uc758 \uc0c8\ub85c\uc6b4 \uc7a1\uc9c0\uba85\uc73c\ub85c \uadf8 \uc774\ub984\uc744 \uc81c\uc548\ud588\ub2e4. \u300a \ubbf8\uc988 \ub9e4\uac70\uc9c4 \u300b\uc758 \uc778\uae30\ub294 \ub9c8\uce68\ub0b4 \uadf8 \uc6a9\uc5b4\ub97c \ub110\ub9ac \uc0ac\uc6a9\ud558\uac8c \ub9cc\ub4e4\uc5c8\ub2e4. 1972\ub144 2\uc6d4, \ubbf8\uad6d \uc815\ubd80 \uc778\uc1c4\uad6d\uc740 \uc815\ubd80 \uacf5\ubb38\uc11c\uc5d0\uc11c Ms.\uc758 \uc0ac\uc6a9\uc744 \uc2b9\uc778\ud558\uc600\ub2e4.", "sentence_answer": "\u300a \ubbf8\uc988 \ub9e4\uac70\uc9c4 \u300b\uc758 \uc778\uae30\ub294 \ub9c8\uce68\ub0b4 \uadf8 \uc6a9\uc5b4\ub97c \ub110\ub9ac \uc0ac\uc6a9\ud558\uac8c \ub9cc\ub4e4\uc5c8\ub2e4.", "paragraph_id": "5782854-1-0", "paragraph_question": "question: Ms.\uc758 \uc6a9\uc5b4\ub97c \ub110\ub9ac \uc54c\ub9b0 \uacc4\uae30\uac00 \ub41c \uc7a1\uc9c0 \uc774\ub984\uc740?, context: 1961\ub144, \uc274\ub77c \ub9c8\uc774\ud074\uc988\ub294 \uadf8\ub140\uc758 \ub8f8\uba54\uc774\ud2b8\uc5d0\uac8c \ubc30\ub2ec\ub41c \ud55c \ubd80\uc758 \u300a\ub274\uc2a4 & \ub808\ud130\uc2a4\u300b(News & Letters)\uc758 \uc8fc\uc18c\ub780\uc5d0 \uc778\uc1c4 \uc624\ub958\ub77c\uace0 \uadf8\ub140\uac00 \uc0dd\uac01\ud588\ub358 \uac83\uc744 \ubcf4\uac8c \ub418\uc5c8\uc744 \ub54c, \uc774 \uc6a9\uc5b4\ub97c \uc0ac\uc6a9\ud558\ub824\uace0 \uc2dc\ub3c4\ub97c \ud558\uac8c \ub41c\ub2e4. \ub9c8\uc774\ud074\uc988\ub294 \ub0a8\uc131\uc5d0\uac8c \uadc0\uc18d\ub418\uc9c0 \uc54a\uc740 \uc5ec\uc131\uc5d0 \ub300\ud55c \ud638\uce6d\uc744 \ucc3e\uace0 \uc788\uc5c8\ub2e4. \uadf8\ub140\ub294 \ud604\uc7ac\ub294 \ud754\ud55c \uc6a9\uc5b4\uc778 Miss\uc640 Mrs.\uc758 \uad6c\ubd84\uc774 Mistress\uc5d0\uc11c \ud30c\uc0dd\ub418\uc5b4 \ub098\uc628 \uac83\uc744 \uc54c\uc558\uc9c0\ub9cc, \uc774\uc81c\ub294 \ud3c9\ud310\uc774 \ub098\uc05c \ub73b\uc744 \ub0b4\ud3ec\ud55c \uc6d0\ub798\uc758 \ud638\uce6d\uc744 \uc5ec\uc131\ub4e4\uc774 \uc0ac\uc6a9\ud558\ub294 \uac83\uc744 \ub204\uad6c\ub3c4 \uc81c\uc548\ud558\uc9c0 \ubabb\ud588\ub2e4. \uc0c8\ub85c\uc6b4 \ud638\uce6d \uc0ac\uc6a9\uc744 \uc7a5\ub824\ud558\ub824\ub294 \uadf8\ub140\uc758 \ub178\ub825\uc740 \ucc98\uc74c\uc5d0\ub294 \ubb34\uc2dc\ub2f9\ud588\ub2e4. 1971\ub144\uacbd, \uc5ec\uc131\uc6b4\ub3d9\uac00 \uc9d1\ub2e8\uacfc WBAI \ub77c\ub514\uc624 \uc778\ud130\ubdf0\ub97c \ud558\ub294 \ub3d9\uc548 \ub9c8\uc774\ud074\uc988\ub294 Ms.\uc758 \uc0ac\uc6a9\uc744 \uc81c\uc548\ud588\ub2e4. \uae00\ub85c\ub9ac\uc544 \uc2a4\ud0c0\uc774\ub118\uc758 \ud55c \uce5c\uad6c\uac00 \uadf8 \uc778\ud130\ubdf0\ub97c \ub4e3\uace0\uc11c, \uadf8\ub140\uc758 \uc0c8\ub85c\uc6b4 \uc7a1\uc9c0\uba85\uc73c\ub85c \uadf8 \uc774\ub984\uc744 \uc81c\uc548\ud588\ub2e4. \u300a\ubbf8\uc988 \ub9e4\uac70\uc9c4\u300b\uc758 \uc778\uae30\ub294 \ub9c8\uce68\ub0b4 \uadf8 \uc6a9\uc5b4\ub97c \ub110\ub9ac \uc0ac\uc6a9\ud558\uac8c \ub9cc\ub4e4\uc5c8\ub2e4. 1972\ub144 2\uc6d4, \ubbf8\uad6d \uc815\ubd80 \uc778\uc1c4\uad6d\uc740 \uc815\ubd80 \uacf5\ubb38\uc11c\uc5d0\uc11c Ms.\uc758 \uc0ac\uc6a9\uc744 \uc2b9\uc778\ud558\uc600\ub2e4."} {"question": "\ube44\ub458\uae30 \uc54c \ub9cc\ud55c \uc6b0\ubc15\uc774 \ub0b4\ub838\ub2e4\uace0 \uc54c\ub824\uc9c4 \uacf3\uc740 \uc5b4\ub514\uc778\uac00?", "paragraph": "\uc724\ub2ec 2\uc6d4 26\uc77c, \uc11c\uc6b8\uc5d0 \ub54c\ub2a6\uc740 \ub208\uacfc \uc6b0\ubc15\uc774 \ub0b4\ub838\ub2e4. \uc724\ub2ec 2\uc6d4 28\uc77c, \uacbd\uc0c1\ub3c4\uc5d0 \ud06c\uae30\uac00 \uc0c8\uc54c\ub9cc\ud55c \uc6b0\ubc15\uc774 \ub0b4\ub838\uc74c\uc774 3\uc6d4 8\uc77c\uc5d0 \ubcf4\uace0\ub418\uc5c8\ub2e4. 3\uc6d4 25\uc77c, \ud3c9\uc548\ub3c4\uc5d0 \uc11c\ub9ac\uac00 \ub0b4\ub838\uc74c\uc774 4\uc6d4 7\uc77c\uc5d0 \ubcf4\uace0\ub418\uc5c8\ub2e4. \ud3c9\uc548\ub3c4\ub294 \uc774\uc5b4 3\uc6d4 27\uc77c, 4\uc6d4 6\uc77c, 4\uc6d4 7\uc77c\uc5d0 \uc5f0\uc774\uc5b4 \uc11c\ub9ac \uc6b0\ubc15\uc774 \ub0b4\ub824 \uace1\uc2dd\uc758 \uc2f9\uc774 \uc8fd\uace0 \ubaa9\ud654\u2022\uc0bc\ubca0\uac00 \ubaa8\ub450 \ud53c\ud574\ub97c \uc785\uc5c8\uc73c\uba70, \uc2ec\uc9c0\uc5b4\ub294 3\uc6d4\uc5d0 \ub208\uae4c\uc9c0 \ub0b4\ub9ac\ub294 \uae30\ud604\uc0c1\uc744 \ubcf4\uc778\ub2e4. 4\uc6d4 6\uc77c\uc5d0\ub294 \uacbd\uae30\ub3c4\uc5d0 \uc6b0\ubc15\uc774 \ub0b4\ub824 \ubc00\uacfc \ubcf4\ub9ac\uac00 \ud53c\ud574\ub97c \uc785\uc5c8\ub2e4. \uc804\ub77c\ub3c4\uc640 \uacbd\uc0c1\ub3c4\ub294 4\uc6d4 8\uc77c\uc5d0 \uc11c\ub9ac\uac00, 4\uc6d4 9\uc77c\uc5d0 \uc6b0\ubc15\uc774 \ub0b4\ub838\uc73c\uba70, \ub3d9\uc2dc\uc5d0 \ub9e4\uc6b0 \uac00\ubb3c\uc5c8\ub2e4. 4\uc6d4 16\uc77c\uc5d0 \ud568\uacbd\ub3c4\uc5d0 \uc6b0\ubc15\uc774 \ub0b4\ub838\ub294\ub370, 5\uc6d4 5\uc77c\uc5d0 \ubcf4\uace0\ub418\uc5c8\ub2e4. 4\uc6d4 23\uc77c\uc5d0\ub294 \uc804\ub77c\ub3c4\uc5d0\uc11c \ubc24\ub9c8\ub2e4 \uc11c\ub9ac\uac00 \ub0b4\ub824 \ub18d\uc791\ubb3c\uc774 \ud53c\ud574\ub97c \uc785\ub294\ub2e4\ub294 \ubcf4\uace0\uac00 \uc62c\ub77c\uc654\ub2e4. 4\uc6d4 28\uc77c\uc5d0\ub294 \uc6d0\uc591\ub3c4(\ud604\ub300\uc758 \uac15\uc6d0\ub3c4)\uc5d0 \uc6b0\ubc15\uc774 \ub0b4\ub838\ub2e4. 5\uc6d4 7\uc77c, \ud3c9\uc548 \uac10\uc0ac\uc758 \ubcf4\uace0\uac00 \ub3c4\ucc29\ud588\ub2e4. \ud3c9\uc548\ub3c4\uc5d0 \uc5f0\uc77c \uc6b0\ubc15\uc774 \ub0b4\ub824 \ub545\uc5d0 \ubc18 \uc790(\uc57d 15 cm)\ub098 \uc313\uc600\uc73c\uba70, \ub3d9\uc2dc\uc5d0 \uac00\ubb44\uc774 \ub108\ubb34 \uc2ec\uac01\ud558\uc5ec \ub18d\uc0ac\uac00 \uac00\ub9dd\uc774 \uc5c6\ub2e4\ub294 \ub0b4\uc6a9\uc774\uc5c8\ub2e4. 5\uc6d4 9\uc77c\uc5d0\ub294 \uacbd\uae30\ub3c4\uc5d0 \uc6b0\ubc15\uc774 \ub0b4\ub824 5\uc6d4 12\uc77c\uc5d0 \ubcf4\uace0\ub418\uc5c8\ub2e4. 5\uc6d4 16\uc77c\uc5d0\ub294 \uc6d0\uc591\ub3c4\uc640 \ud669\ud574\ub3c4\uc5d0 \uc6b0\ubc15\uc774 \ub0b4\ub838\uc74c\uc774 \ubcf4\uace0\ub418\uc5c8\ub2e4. 5\uc6d4 17\uc77c\uc5d0 \ud3c9\uc548 \uac10\uc0ac\uc758 \ubcf4\uace0\ub97c \uc62c\ub838\ub294\ub370, \uadf8 \ub0b4\uc6a9\uc774 \uae30\uac00 \ub9c9\ud614\ub2e4. \uc6b0\ubc15\uc73c\ub85c \uc778\ud574 \ubcbc\uac00 \uc0c1\ud55c \uac83\uc740 \ubb3c\ub860, \uc624\ub9ac\uc54c \ub9cc\ud55c \ud06c\uae30\uc758 \uc6b0\ubc15\uc774 \uc3df\uc544\uc838 \ubc18 \uc790\ub098 \uc313\uc774\uace0, \ub124 \uc0b4\uc9dc\ub9ac \uc544\uc774\uac00 \uadf8 \uc6b0\ubc15\uc5d0 \ub9de\uc544 \uc8fd\uc5c8\uc73c\uba70, \uafe9\u2022\ud1a0\ub07c\u2022\uae4c\ub9c8\uadc0\u2022\uae4c\uce58 \ub4f1 \uc9d0\uc2b9\ub4e4\uc774 \uc231\ud558\uac8c \uc8fd\uc5c8\ub2e4. 5\uc6d4 22\uc77c\uc5d0\ub294 \uc9c0\ub09c 5\uc6d4 5\uc77c\uc5d0 \ud3c9\uc548\ub3c4\uc5d0\uc11c \uc6b0\ubc15\uc774 \ub0b4\ub838\uc74c\uc774 \ubcf4\uace0\ub418\uc5c8\ub2e4. \uac00\ubb44\uc774 \ubb3c\ub7ec\uac00\uace0 \ud64d\uc218\uac00 \uc2dc\uc791\ub41c 6\uc6d4\uc5d0\ub294 \ub2e4\uc2dc \ud3c9\uc548\ub3c4\uc5d0\uc11c \uc6b0\ub808\uc640 \ubc14\ub78c\uc744 \ub3d9\ubc18\ud55c \uc6b0\ubc15\uc774 \ub0b4\ub838\ub2e4. 6\uc6d4 16\uc77c\uc5d0 \ub610\ub2e4\uc2dc \ud3c9\uc548\ub3c4\uc5d0 \uc6b0\ubc15\uc774 \ub0b4\ub824 \uace1\uc2dd\uc774 \uc0c1\ud588\ub294\ub370, 7\uc6d4 11\uc77c\uc5d0 \ubcf4\uace0\ub418\uc5c8\ub2e4. 7\uc6d4 11\uc77c\uc5d0\ub294 \ub610 \uc9c0\ub09c 6\uc6d4 5\uc77c\uc5d0 \ud568\uacbd\ub3c4\uc5d0 \ube44\ub458\uae30 \uc54c \ub9cc\ud55c \uc6b0\ubc15\uc774 \ub0b4\ub838\uc74c\uc774 \ubcf4\uace0\ub418\uc5c8\ub2e4.", "answer": "\ud568\uacbd\ub3c4", "sentence": "4\uc6d4 16\uc77c\uc5d0 \ud568\uacbd\ub3c4 \uc5d0 \uc6b0\ubc15\uc774 \ub0b4\ub838\ub294\ub370, 5\uc6d4 5\uc77c\uc5d0 \ubcf4\uace0\ub418\uc5c8\ub2e4.", "paragraph_sentence": "\uc724\ub2ec 2\uc6d4 26\uc77c, \uc11c\uc6b8\uc5d0 \ub54c\ub2a6\uc740 \ub208\uacfc \uc6b0\ubc15\uc774 \ub0b4\ub838\ub2e4. \uc724\ub2ec 2\uc6d4 28\uc77c, \uacbd\uc0c1\ub3c4\uc5d0 \ud06c\uae30\uac00 \uc0c8\uc54c\ub9cc\ud55c \uc6b0\ubc15\uc774 \ub0b4\ub838\uc74c\uc774 3\uc6d4 8\uc77c\uc5d0 \ubcf4\uace0\ub418\uc5c8\ub2e4. 3\uc6d4 25\uc77c, \ud3c9\uc548\ub3c4\uc5d0 \uc11c\ub9ac\uac00 \ub0b4\ub838\uc74c\uc774 4\uc6d4 7\uc77c\uc5d0 \ubcf4\uace0\ub418\uc5c8\ub2e4. \ud3c9\uc548\ub3c4\ub294 \uc774\uc5b4 3\uc6d4 27\uc77c, 4\uc6d4 6\uc77c, 4\uc6d4 7\uc77c\uc5d0 \uc5f0\uc774\uc5b4 \uc11c\ub9ac \uc6b0\ubc15\uc774 \ub0b4\ub824 \uace1\uc2dd\uc758 \uc2f9\uc774 \uc8fd\uace0 \ubaa9\ud654\u2022\uc0bc\ubca0\uac00 \ubaa8\ub450 \ud53c\ud574\ub97c \uc785\uc5c8\uc73c\uba70, \uc2ec\uc9c0\uc5b4\ub294 3\uc6d4\uc5d0 \ub208\uae4c\uc9c0 \ub0b4\ub9ac\ub294 \uae30\ud604\uc0c1\uc744 \ubcf4\uc778\ub2e4. 4\uc6d4 6\uc77c\uc5d0\ub294 \uacbd\uae30\ub3c4\uc5d0 \uc6b0\ubc15\uc774 \ub0b4\ub824 \ubc00\uacfc \ubcf4\ub9ac\uac00 \ud53c\ud574\ub97c \uc785\uc5c8\ub2e4. \uc804\ub77c\ub3c4\uc640 \uacbd\uc0c1\ub3c4\ub294 4\uc6d4 8\uc77c\uc5d0 \uc11c\ub9ac\uac00, 4\uc6d4 9\uc77c\uc5d0 \uc6b0\ubc15\uc774 \ub0b4\ub838\uc73c\uba70, \ub3d9\uc2dc\uc5d0 \ub9e4\uc6b0 \uac00\ubb3c\uc5c8\ub2e4. 4\uc6d4 16\uc77c\uc5d0 \ud568\uacbd\ub3c4 \uc5d0 \uc6b0\ubc15\uc774 \ub0b4\ub838\ub294\ub370, 5\uc6d4 5\uc77c\uc5d0 \ubcf4\uace0\ub418\uc5c8\ub2e4. 4\uc6d4 23\uc77c\uc5d0\ub294 \uc804\ub77c\ub3c4\uc5d0\uc11c \ubc24\ub9c8\ub2e4 \uc11c\ub9ac\uac00 \ub0b4\ub824 \ub18d\uc791\ubb3c\uc774 \ud53c\ud574\ub97c \uc785\ub294\ub2e4\ub294 \ubcf4\uace0\uac00 \uc62c\ub77c\uc654\ub2e4. 4\uc6d4 28\uc77c\uc5d0\ub294 \uc6d0\uc591\ub3c4(\ud604\ub300\uc758 \uac15\uc6d0\ub3c4)\uc5d0 \uc6b0\ubc15\uc774 \ub0b4\ub838\ub2e4. 5\uc6d4 7\uc77c, \ud3c9\uc548 \uac10\uc0ac\uc758 \ubcf4\uace0\uac00 \ub3c4\ucc29\ud588\ub2e4. \ud3c9\uc548\ub3c4\uc5d0 \uc5f0\uc77c \uc6b0\ubc15\uc774 \ub0b4\ub824 \ub545\uc5d0 \ubc18 \uc790(\uc57d 15 cm)\ub098 \uc313\uc600\uc73c\uba70, \ub3d9\uc2dc\uc5d0 \uac00\ubb44\uc774 \ub108\ubb34 \uc2ec\uac01\ud558\uc5ec \ub18d\uc0ac\uac00 \uac00\ub9dd\uc774 \uc5c6\ub2e4\ub294 \ub0b4\uc6a9\uc774\uc5c8\ub2e4. 5\uc6d4 9\uc77c\uc5d0\ub294 \uacbd\uae30\ub3c4\uc5d0 \uc6b0\ubc15\uc774 \ub0b4\ub824 5\uc6d4 12\uc77c\uc5d0 \ubcf4\uace0\ub418\uc5c8\ub2e4. 5\uc6d4 16\uc77c\uc5d0\ub294 \uc6d0\uc591\ub3c4\uc640 \ud669\ud574\ub3c4\uc5d0 \uc6b0\ubc15\uc774 \ub0b4\ub838\uc74c\uc774 \ubcf4\uace0\ub418\uc5c8\ub2e4. 5\uc6d4 17\uc77c\uc5d0 \ud3c9\uc548 \uac10\uc0ac\uc758 \ubcf4\uace0\ub97c \uc62c\ub838\ub294\ub370, \uadf8 \ub0b4\uc6a9\uc774 \uae30\uac00 \ub9c9\ud614\ub2e4. \uc6b0\ubc15\uc73c\ub85c \uc778\ud574 \ubcbc\uac00 \uc0c1\ud55c \uac83\uc740 \ubb3c\ub860, \uc624\ub9ac\uc54c \ub9cc\ud55c \ud06c\uae30\uc758 \uc6b0\ubc15\uc774 \uc3df\uc544\uc838 \ubc18 \uc790\ub098 \uc313\uc774\uace0, \ub124 \uc0b4\uc9dc\ub9ac \uc544\uc774\uac00 \uadf8 \uc6b0\ubc15\uc5d0 \ub9de\uc544 \uc8fd\uc5c8\uc73c\uba70, \uafe9\u2022\ud1a0\ub07c\u2022\uae4c\ub9c8\uadc0\u2022\uae4c\uce58 \ub4f1 \uc9d0\uc2b9\ub4e4\uc774 \uc231\ud558\uac8c \uc8fd\uc5c8\ub2e4. 5\uc6d4 22\uc77c\uc5d0\ub294 \uc9c0\ub09c 5\uc6d4 5\uc77c\uc5d0 \ud3c9\uc548\ub3c4\uc5d0\uc11c \uc6b0\ubc15\uc774 \ub0b4\ub838\uc74c\uc774 \ubcf4\uace0\ub418\uc5c8\ub2e4. \uac00\ubb44\uc774 \ubb3c\ub7ec\uac00\uace0 \ud64d\uc218\uac00 \uc2dc\uc791\ub41c 6\uc6d4\uc5d0\ub294 \ub2e4\uc2dc \ud3c9\uc548\ub3c4\uc5d0\uc11c \uc6b0\ub808\uc640 \ubc14\ub78c\uc744 \ub3d9\ubc18\ud55c \uc6b0\ubc15\uc774 \ub0b4\ub838\ub2e4. 6\uc6d4 16\uc77c\uc5d0 \ub610\ub2e4\uc2dc \ud3c9\uc548\ub3c4\uc5d0 \uc6b0\ubc15\uc774 \ub0b4\ub824 \uace1\uc2dd\uc774 \uc0c1\ud588\ub294\ub370, 7\uc6d4 11\uc77c\uc5d0 \ubcf4\uace0\ub418\uc5c8\ub2e4. 7\uc6d4 11\uc77c\uc5d0\ub294 \ub610 \uc9c0\ub09c 6\uc6d4 5\uc77c\uc5d0 \ud568\uacbd\ub3c4\uc5d0 \ube44\ub458\uae30 \uc54c \ub9cc\ud55c \uc6b0\ubc15\uc774 \ub0b4\ub838\uc74c\uc774 \ubcf4\uace0\ub418\uc5c8\ub2e4.", "paragraph_answer": "\uc724\ub2ec 2\uc6d4 26\uc77c, \uc11c\uc6b8\uc5d0 \ub54c\ub2a6\uc740 \ub208\uacfc \uc6b0\ubc15\uc774 \ub0b4\ub838\ub2e4. \uc724\ub2ec 2\uc6d4 28\uc77c, \uacbd\uc0c1\ub3c4\uc5d0 \ud06c\uae30\uac00 \uc0c8\uc54c\ub9cc\ud55c \uc6b0\ubc15\uc774 \ub0b4\ub838\uc74c\uc774 3\uc6d4 8\uc77c\uc5d0 \ubcf4\uace0\ub418\uc5c8\ub2e4. 3\uc6d4 25\uc77c, \ud3c9\uc548\ub3c4\uc5d0 \uc11c\ub9ac\uac00 \ub0b4\ub838\uc74c\uc774 4\uc6d4 7\uc77c\uc5d0 \ubcf4\uace0\ub418\uc5c8\ub2e4. \ud3c9\uc548\ub3c4\ub294 \uc774\uc5b4 3\uc6d4 27\uc77c, 4\uc6d4 6\uc77c, 4\uc6d4 7\uc77c\uc5d0 \uc5f0\uc774\uc5b4 \uc11c\ub9ac \uc6b0\ubc15\uc774 \ub0b4\ub824 \uace1\uc2dd\uc758 \uc2f9\uc774 \uc8fd\uace0 \ubaa9\ud654\u2022\uc0bc\ubca0\uac00 \ubaa8\ub450 \ud53c\ud574\ub97c \uc785\uc5c8\uc73c\uba70, \uc2ec\uc9c0\uc5b4\ub294 3\uc6d4\uc5d0 \ub208\uae4c\uc9c0 \ub0b4\ub9ac\ub294 \uae30\ud604\uc0c1\uc744 \ubcf4\uc778\ub2e4. 4\uc6d4 6\uc77c\uc5d0\ub294 \uacbd\uae30\ub3c4\uc5d0 \uc6b0\ubc15\uc774 \ub0b4\ub824 \ubc00\uacfc \ubcf4\ub9ac\uac00 \ud53c\ud574\ub97c \uc785\uc5c8\ub2e4. \uc804\ub77c\ub3c4\uc640 \uacbd\uc0c1\ub3c4\ub294 4\uc6d4 8\uc77c\uc5d0 \uc11c\ub9ac\uac00, 4\uc6d4 9\uc77c\uc5d0 \uc6b0\ubc15\uc774 \ub0b4\ub838\uc73c\uba70, \ub3d9\uc2dc\uc5d0 \ub9e4\uc6b0 \uac00\ubb3c\uc5c8\ub2e4. 4\uc6d4 16\uc77c\uc5d0 \ud568\uacbd\ub3c4 \uc5d0 \uc6b0\ubc15\uc774 \ub0b4\ub838\ub294\ub370, 5\uc6d4 5\uc77c\uc5d0 \ubcf4\uace0\ub418\uc5c8\ub2e4. 4\uc6d4 23\uc77c\uc5d0\ub294 \uc804\ub77c\ub3c4\uc5d0\uc11c \ubc24\ub9c8\ub2e4 \uc11c\ub9ac\uac00 \ub0b4\ub824 \ub18d\uc791\ubb3c\uc774 \ud53c\ud574\ub97c \uc785\ub294\ub2e4\ub294 \ubcf4\uace0\uac00 \uc62c\ub77c\uc654\ub2e4. 4\uc6d4 28\uc77c\uc5d0\ub294 \uc6d0\uc591\ub3c4(\ud604\ub300\uc758 \uac15\uc6d0\ub3c4)\uc5d0 \uc6b0\ubc15\uc774 \ub0b4\ub838\ub2e4. 5\uc6d4 7\uc77c, \ud3c9\uc548 \uac10\uc0ac\uc758 \ubcf4\uace0\uac00 \ub3c4\ucc29\ud588\ub2e4. \ud3c9\uc548\ub3c4\uc5d0 \uc5f0\uc77c \uc6b0\ubc15\uc774 \ub0b4\ub824 \ub545\uc5d0 \ubc18 \uc790(\uc57d 15 cm)\ub098 \uc313\uc600\uc73c\uba70, \ub3d9\uc2dc\uc5d0 \uac00\ubb44\uc774 \ub108\ubb34 \uc2ec\uac01\ud558\uc5ec \ub18d\uc0ac\uac00 \uac00\ub9dd\uc774 \uc5c6\ub2e4\ub294 \ub0b4\uc6a9\uc774\uc5c8\ub2e4. 5\uc6d4 9\uc77c\uc5d0\ub294 \uacbd\uae30\ub3c4\uc5d0 \uc6b0\ubc15\uc774 \ub0b4\ub824 5\uc6d4 12\uc77c\uc5d0 \ubcf4\uace0\ub418\uc5c8\ub2e4. 5\uc6d4 16\uc77c\uc5d0\ub294 \uc6d0\uc591\ub3c4\uc640 \ud669\ud574\ub3c4\uc5d0 \uc6b0\ubc15\uc774 \ub0b4\ub838\uc74c\uc774 \ubcf4\uace0\ub418\uc5c8\ub2e4. 5\uc6d4 17\uc77c\uc5d0 \ud3c9\uc548 \uac10\uc0ac\uc758 \ubcf4\uace0\ub97c \uc62c\ub838\ub294\ub370, \uadf8 \ub0b4\uc6a9\uc774 \uae30\uac00 \ub9c9\ud614\ub2e4. \uc6b0\ubc15\uc73c\ub85c \uc778\ud574 \ubcbc\uac00 \uc0c1\ud55c \uac83\uc740 \ubb3c\ub860, \uc624\ub9ac\uc54c \ub9cc\ud55c \ud06c\uae30\uc758 \uc6b0\ubc15\uc774 \uc3df\uc544\uc838 \ubc18 \uc790\ub098 \uc313\uc774\uace0, \ub124 \uc0b4\uc9dc\ub9ac \uc544\uc774\uac00 \uadf8 \uc6b0\ubc15\uc5d0 \ub9de\uc544 \uc8fd\uc5c8\uc73c\uba70, \uafe9\u2022\ud1a0\ub07c\u2022\uae4c\ub9c8\uadc0\u2022\uae4c\uce58 \ub4f1 \uc9d0\uc2b9\ub4e4\uc774 \uc231\ud558\uac8c \uc8fd\uc5c8\ub2e4. 5\uc6d4 22\uc77c\uc5d0\ub294 \uc9c0\ub09c 5\uc6d4 5\uc77c\uc5d0 \ud3c9\uc548\ub3c4\uc5d0\uc11c \uc6b0\ubc15\uc774 \ub0b4\ub838\uc74c\uc774 \ubcf4\uace0\ub418\uc5c8\ub2e4. \uac00\ubb44\uc774 \ubb3c\ub7ec\uac00\uace0 \ud64d\uc218\uac00 \uc2dc\uc791\ub41c 6\uc6d4\uc5d0\ub294 \ub2e4\uc2dc \ud3c9\uc548\ub3c4\uc5d0\uc11c \uc6b0\ub808\uc640 \ubc14\ub78c\uc744 \ub3d9\ubc18\ud55c \uc6b0\ubc15\uc774 \ub0b4\ub838\ub2e4. 6\uc6d4 16\uc77c\uc5d0 \ub610\ub2e4\uc2dc \ud3c9\uc548\ub3c4\uc5d0 \uc6b0\ubc15\uc774 \ub0b4\ub824 \uace1\uc2dd\uc774 \uc0c1\ud588\ub294\ub370, 7\uc6d4 11\uc77c\uc5d0 \ubcf4\uace0\ub418\uc5c8\ub2e4. 7\uc6d4 11\uc77c\uc5d0\ub294 \ub610 \uc9c0\ub09c 6\uc6d4 5\uc77c\uc5d0 \ud568\uacbd\ub3c4\uc5d0 \ube44\ub458\uae30 \uc54c \ub9cc\ud55c \uc6b0\ubc15\uc774 \ub0b4\ub838\uc74c\uc774 \ubcf4\uace0\ub418\uc5c8\ub2e4.", "sentence_answer": "4\uc6d4 16\uc77c\uc5d0 \ud568\uacbd\ub3c4 \uc5d0 \uc6b0\ubc15\uc774 \ub0b4\ub838\ub294\ub370, 5\uc6d4 5\uc77c\uc5d0 \ubcf4\uace0\ub418\uc5c8\ub2e4.", "paragraph_id": "6657773-1-1", "paragraph_question": "question: \ube44\ub458\uae30 \uc54c \ub9cc\ud55c \uc6b0\ubc15\uc774 \ub0b4\ub838\ub2e4\uace0 \uc54c\ub824\uc9c4 \uacf3\uc740 \uc5b4\ub514\uc778\uac00?, context: \uc724\ub2ec 2\uc6d4 26\uc77c, \uc11c\uc6b8\uc5d0 \ub54c\ub2a6\uc740 \ub208\uacfc \uc6b0\ubc15\uc774 \ub0b4\ub838\ub2e4. \uc724\ub2ec 2\uc6d4 28\uc77c, \uacbd\uc0c1\ub3c4\uc5d0 \ud06c\uae30\uac00 \uc0c8\uc54c\ub9cc\ud55c \uc6b0\ubc15\uc774 \ub0b4\ub838\uc74c\uc774 3\uc6d4 8\uc77c\uc5d0 \ubcf4\uace0\ub418\uc5c8\ub2e4. 3\uc6d4 25\uc77c, \ud3c9\uc548\ub3c4\uc5d0 \uc11c\ub9ac\uac00 \ub0b4\ub838\uc74c\uc774 4\uc6d4 7\uc77c\uc5d0 \ubcf4\uace0\ub418\uc5c8\ub2e4. \ud3c9\uc548\ub3c4\ub294 \uc774\uc5b4 3\uc6d4 27\uc77c, 4\uc6d4 6\uc77c, 4\uc6d4 7\uc77c\uc5d0 \uc5f0\uc774\uc5b4 \uc11c\ub9ac \uc6b0\ubc15\uc774 \ub0b4\ub824 \uace1\uc2dd\uc758 \uc2f9\uc774 \uc8fd\uace0 \ubaa9\ud654\u2022\uc0bc\ubca0\uac00 \ubaa8\ub450 \ud53c\ud574\ub97c \uc785\uc5c8\uc73c\uba70, \uc2ec\uc9c0\uc5b4\ub294 3\uc6d4\uc5d0 \ub208\uae4c\uc9c0 \ub0b4\ub9ac\ub294 \uae30\ud604\uc0c1\uc744 \ubcf4\uc778\ub2e4. 4\uc6d4 6\uc77c\uc5d0\ub294 \uacbd\uae30\ub3c4\uc5d0 \uc6b0\ubc15\uc774 \ub0b4\ub824 \ubc00\uacfc \ubcf4\ub9ac\uac00 \ud53c\ud574\ub97c \uc785\uc5c8\ub2e4. \uc804\ub77c\ub3c4\uc640 \uacbd\uc0c1\ub3c4\ub294 4\uc6d4 8\uc77c\uc5d0 \uc11c\ub9ac\uac00, 4\uc6d4 9\uc77c\uc5d0 \uc6b0\ubc15\uc774 \ub0b4\ub838\uc73c\uba70, \ub3d9\uc2dc\uc5d0 \ub9e4\uc6b0 \uac00\ubb3c\uc5c8\ub2e4. 4\uc6d4 16\uc77c\uc5d0 \ud568\uacbd\ub3c4\uc5d0 \uc6b0\ubc15\uc774 \ub0b4\ub838\ub294\ub370, 5\uc6d4 5\uc77c\uc5d0 \ubcf4\uace0\ub418\uc5c8\ub2e4. 4\uc6d4 23\uc77c\uc5d0\ub294 \uc804\ub77c\ub3c4\uc5d0\uc11c \ubc24\ub9c8\ub2e4 \uc11c\ub9ac\uac00 \ub0b4\ub824 \ub18d\uc791\ubb3c\uc774 \ud53c\ud574\ub97c \uc785\ub294\ub2e4\ub294 \ubcf4\uace0\uac00 \uc62c\ub77c\uc654\ub2e4. 4\uc6d4 28\uc77c\uc5d0\ub294 \uc6d0\uc591\ub3c4(\ud604\ub300\uc758 \uac15\uc6d0\ub3c4)\uc5d0 \uc6b0\ubc15\uc774 \ub0b4\ub838\ub2e4. 5\uc6d4 7\uc77c, \ud3c9\uc548 \uac10\uc0ac\uc758 \ubcf4\uace0\uac00 \ub3c4\ucc29\ud588\ub2e4. \ud3c9\uc548\ub3c4\uc5d0 \uc5f0\uc77c \uc6b0\ubc15\uc774 \ub0b4\ub824 \ub545\uc5d0 \ubc18 \uc790(\uc57d 15 cm)\ub098 \uc313\uc600\uc73c\uba70, \ub3d9\uc2dc\uc5d0 \uac00\ubb44\uc774 \ub108\ubb34 \uc2ec\uac01\ud558\uc5ec \ub18d\uc0ac\uac00 \uac00\ub9dd\uc774 \uc5c6\ub2e4\ub294 \ub0b4\uc6a9\uc774\uc5c8\ub2e4. 5\uc6d4 9\uc77c\uc5d0\ub294 \uacbd\uae30\ub3c4\uc5d0 \uc6b0\ubc15\uc774 \ub0b4\ub824 5\uc6d4 12\uc77c\uc5d0 \ubcf4\uace0\ub418\uc5c8\ub2e4. 5\uc6d4 16\uc77c\uc5d0\ub294 \uc6d0\uc591\ub3c4\uc640 \ud669\ud574\ub3c4\uc5d0 \uc6b0\ubc15\uc774 \ub0b4\ub838\uc74c\uc774 \ubcf4\uace0\ub418\uc5c8\ub2e4. 5\uc6d4 17\uc77c\uc5d0 \ud3c9\uc548 \uac10\uc0ac\uc758 \ubcf4\uace0\ub97c \uc62c\ub838\ub294\ub370, \uadf8 \ub0b4\uc6a9\uc774 \uae30\uac00 \ub9c9\ud614\ub2e4. \uc6b0\ubc15\uc73c\ub85c \uc778\ud574 \ubcbc\uac00 \uc0c1\ud55c \uac83\uc740 \ubb3c\ub860, \uc624\ub9ac\uc54c \ub9cc\ud55c \ud06c\uae30\uc758 \uc6b0\ubc15\uc774 \uc3df\uc544\uc838 \ubc18 \uc790\ub098 \uc313\uc774\uace0, \ub124 \uc0b4\uc9dc\ub9ac \uc544\uc774\uac00 \uadf8 \uc6b0\ubc15\uc5d0 \ub9de\uc544 \uc8fd\uc5c8\uc73c\uba70, \uafe9\u2022\ud1a0\ub07c\u2022\uae4c\ub9c8\uadc0\u2022\uae4c\uce58 \ub4f1 \uc9d0\uc2b9\ub4e4\uc774 \uc231\ud558\uac8c \uc8fd\uc5c8\ub2e4. 5\uc6d4 22\uc77c\uc5d0\ub294 \uc9c0\ub09c 5\uc6d4 5\uc77c\uc5d0 \ud3c9\uc548\ub3c4\uc5d0\uc11c \uc6b0\ubc15\uc774 \ub0b4\ub838\uc74c\uc774 \ubcf4\uace0\ub418\uc5c8\ub2e4. \uac00\ubb44\uc774 \ubb3c\ub7ec\uac00\uace0 \ud64d\uc218\uac00 \uc2dc\uc791\ub41c 6\uc6d4\uc5d0\ub294 \ub2e4\uc2dc \ud3c9\uc548\ub3c4\uc5d0\uc11c \uc6b0\ub808\uc640 \ubc14\ub78c\uc744 \ub3d9\ubc18\ud55c \uc6b0\ubc15\uc774 \ub0b4\ub838\ub2e4. 6\uc6d4 16\uc77c\uc5d0 \ub610\ub2e4\uc2dc \ud3c9\uc548\ub3c4\uc5d0 \uc6b0\ubc15\uc774 \ub0b4\ub824 \uace1\uc2dd\uc774 \uc0c1\ud588\ub294\ub370, 7\uc6d4 11\uc77c\uc5d0 \ubcf4\uace0\ub418\uc5c8\ub2e4. 7\uc6d4 11\uc77c\uc5d0\ub294 \ub610 \uc9c0\ub09c 6\uc6d4 5\uc77c\uc5d0 \ud568\uacbd\ub3c4\uc5d0 \ube44\ub458\uae30 \uc54c \ub9cc\ud55c \uc6b0\ubc15\uc774 \ub0b4\ub838\uc74c\uc774 \ubcf4\uace0\ub418\uc5c8\ub2e4."} {"question": "\ub8e8\ub9c8\ub2c8\uc544 \uc774\uc544\uc2dc\uc758 \ud559\uc0b4\uc774 \uc77c\uc5b4\ub09c \uac83\uc740 \uc5b8\uc81c\uc778\uac00?", "paragraph": "\uc9c0\uc5ed\uc758 \uc778\uad6c\uc5d0 \uc758\ud55c \uc218\ub9ce\uc740 \uce58\uba85\uc801\uc778 \ud559\uc0b4\uc740 \uc81c 2\ucc28 \uc138\uacc4\ub300\uc804 \ub098\uce58\uc758 \uaca9\ub824\uc5d0 \uc758\ud574 \uc77c\ubd80\ub294 \uc790\ubc1c\uc801\uc73c\ub85c \ubc1c\uc0dd\ud588\ub2e4. \uc774\uac83\uc740 1941\ub144 6\uc6d4 30\uc77c \ub8e8\ub9c8\ub2c8\uc544 \uc774\uc544\uc2dc\uc758 \ud559\uc0b4\uc744 \ud3ec\ud568\ud558\ub294\ub370 \uadf8 \ud559\uc0b4\uc740 14,000 \uc720\ub300\uc778\uc774 \ub8e8\ub9c8\ub2c8\uc544 \uc8fc\ubbfc\uacfc \uacbd\ucc30\uc5d0 \uc758\ud574 \uc0ac\ub9dd\ud558\ub294 \uc0ac\uac74\uc774\ub2e4. \ub610\ud55c 1941\ub144 7\uc6d4\uc758 \uc2e4\ud06c \ud504\ub85c\uadf8\ub7a8(Jedwabne pogrom)\uc744 \ud3ec\ud568\ud558\ub294\ub370 \uc774\ub294 \ub098\uce58 \uc9c8\uc11c\uacbd\ucc30(Nazi Ordnungspolizei)\uac00 \uc788\ub294 \ub370\uc11c 300\uba85 \uc720\ub300\uc778\uc774 \ubd88 \ub09c \ud5db\uac04\uc5d0 \uc7a0\uae30\uac8c \ub418\ub294 \uc0ac\uac74\uc774\ub2e4. \ub3c5\uc77c\uc778\uc5d0 \uc758\ud574 \uac19\uc740 \uacf3\uc5d0\uc11c 40\uba85\uc758 \uc720\ub300\uc778 \ub0a8\uc131\uc774 \ucc98\ud615\ub418\uc5c8\ub2e4. \uc774\uac83\uc740 \ubc1c\uad74\uc5d0 \ucc38\uc5ec\ud55c \uace0\uace0\ud559 \ubc0f \uc778\ub958\ud559 \ud300\uc5d0 \uc758\ud574 \uc870\uc0ac\ub41c \ub450 \uac1c\uc758 \ubb34\ub364\uc5d0 \uc788\ub294 \ud53c\ud574\uc790\uc758 \uc22b\uc790\ub85c \ud655\uc778\ub418\uc5c8\uace0 \uad6d\ub9bd\ucd94\ubaa8\uc5f0\uad6c\uc18c\uc5d0 \uc758\ud574 2000\ub144\uc5d0\uc11c 2003\ub144 \uc0ac\uc774\uc5d0 \uc2e4\uc2dc\ud55c \uacf5\uc2dd\uc870\uc0ac\ub85c \ucd5c\uc885 \ubc1c\uacac\ub418\uc5c8\ub2e4. \uc18c\ubb38\ubcf4\ub2e4 \ub354 \ub9ce\uc740 \uc0ac\ub78c\ub4e4\uc774 \ud76c\uc0dd\ub418\uc5c8\uc74c\uc744 \ubc18\uc99d\ud55c \uac83\uc774\ub2e4.", "answer": "1941\ub144 6\uc6d4 30\uc77c", "sentence": "\uc774\uac83\uc740 1941\ub144 6\uc6d4 30\uc77c \ub8e8\ub9c8\ub2c8\uc544 \uc774\uc544\uc2dc\uc758 \ud559\uc0b4\uc744 \ud3ec\ud568\ud558\ub294\ub370 \uadf8 \ud559\uc0b4\uc740 14,000 \uc720\ub300\uc778\uc774 \ub8e8\ub9c8\ub2c8\uc544 \uc8fc\ubbfc\uacfc \uacbd\ucc30\uc5d0 \uc758\ud574 \uc0ac\ub9dd\ud558\ub294 \uc0ac\uac74\uc774\ub2e4.", "paragraph_sentence": "\uc9c0\uc5ed\uc758 \uc778\uad6c\uc5d0 \uc758\ud55c \uc218\ub9ce\uc740 \uce58\uba85\uc801\uc778 \ud559\uc0b4\uc740 \uc81c 2\ucc28 \uc138\uacc4\ub300\uc804 \ub098\uce58\uc758 \uaca9\ub824\uc5d0 \uc758\ud574 \uc77c\ubd80\ub294 \uc790\ubc1c\uc801\uc73c\ub85c \ubc1c\uc0dd\ud588\ub2e4. \uc774\uac83\uc740 1941\ub144 6\uc6d4 30\uc77c \ub8e8\ub9c8\ub2c8\uc544 \uc774\uc544\uc2dc\uc758 \ud559\uc0b4\uc744 \ud3ec\ud568\ud558\ub294\ub370 \uadf8 \ud559\uc0b4\uc740 14,000 \uc720\ub300\uc778\uc774 \ub8e8\ub9c8\ub2c8\uc544 \uc8fc\ubbfc\uacfc \uacbd\ucc30\uc5d0 \uc758\ud574 \uc0ac\ub9dd\ud558\ub294 \uc0ac\uac74\uc774\ub2e4. \ub610\ud55c 1941\ub144 7\uc6d4\uc758 \uc2e4\ud06c \ud504\ub85c\uadf8\ub7a8(Jedwabne pogrom)\uc744 \ud3ec\ud568\ud558\ub294\ub370 \uc774\ub294 \ub098\uce58 \uc9c8\uc11c\uacbd\ucc30(Nazi Ordnungspolizei)\uac00 \uc788\ub294 \ub370\uc11c 300\uba85 \uc720\ub300\uc778\uc774 \ubd88 \ub09c \ud5db\uac04\uc5d0 \uc7a0\uae30\uac8c \ub418\ub294 \uc0ac\uac74\uc774\ub2e4. \ub3c5\uc77c\uc778\uc5d0 \uc758\ud574 \uac19\uc740 \uacf3\uc5d0\uc11c 40\uba85\uc758 \uc720\ub300\uc778 \ub0a8\uc131\uc774 \ucc98\ud615\ub418\uc5c8\ub2e4. \uc774\uac83\uc740 \ubc1c\uad74\uc5d0 \ucc38\uc5ec\ud55c \uace0\uace0\ud559 \ubc0f \uc778\ub958\ud559 \ud300\uc5d0 \uc758\ud574 \uc870\uc0ac\ub41c \ub450 \uac1c\uc758 \ubb34\ub364\uc5d0 \uc788\ub294 \ud53c\ud574\uc790\uc758 \uc22b\uc790\ub85c \ud655\uc778\ub418\uc5c8\uace0 \uad6d\ub9bd\ucd94\ubaa8\uc5f0\uad6c\uc18c\uc5d0 \uc758\ud574 2000\ub144\uc5d0\uc11c 2003\ub144 \uc0ac\uc774\uc5d0 \uc2e4\uc2dc\ud55c \uacf5\uc2dd\uc870\uc0ac\ub85c \ucd5c\uc885 \ubc1c\uacac\ub418\uc5c8\ub2e4. \uc18c\ubb38\ubcf4\ub2e4 \ub354 \ub9ce\uc740 \uc0ac\ub78c\ub4e4\uc774 \ud76c\uc0dd\ub418\uc5c8\uc74c\uc744 \ubc18\uc99d\ud55c \uac83\uc774\ub2e4.", "paragraph_answer": "\uc9c0\uc5ed\uc758 \uc778\uad6c\uc5d0 \uc758\ud55c \uc218\ub9ce\uc740 \uce58\uba85\uc801\uc778 \ud559\uc0b4\uc740 \uc81c 2\ucc28 \uc138\uacc4\ub300\uc804 \ub098\uce58\uc758 \uaca9\ub824\uc5d0 \uc758\ud574 \uc77c\ubd80\ub294 \uc790\ubc1c\uc801\uc73c\ub85c \ubc1c\uc0dd\ud588\ub2e4. \uc774\uac83\uc740 1941\ub144 6\uc6d4 30\uc77c \ub8e8\ub9c8\ub2c8\uc544 \uc774\uc544\uc2dc\uc758 \ud559\uc0b4\uc744 \ud3ec\ud568\ud558\ub294\ub370 \uadf8 \ud559\uc0b4\uc740 14,000 \uc720\ub300\uc778\uc774 \ub8e8\ub9c8\ub2c8\uc544 \uc8fc\ubbfc\uacfc \uacbd\ucc30\uc5d0 \uc758\ud574 \uc0ac\ub9dd\ud558\ub294 \uc0ac\uac74\uc774\ub2e4. \ub610\ud55c 1941\ub144 7\uc6d4\uc758 \uc2e4\ud06c \ud504\ub85c\uadf8\ub7a8(Jedwabne pogrom)\uc744 \ud3ec\ud568\ud558\ub294\ub370 \uc774\ub294 \ub098\uce58 \uc9c8\uc11c\uacbd\ucc30(Nazi Ordnungspolizei)\uac00 \uc788\ub294 \ub370\uc11c 300\uba85 \uc720\ub300\uc778\uc774 \ubd88 \ub09c \ud5db\uac04\uc5d0 \uc7a0\uae30\uac8c \ub418\ub294 \uc0ac\uac74\uc774\ub2e4. \ub3c5\uc77c\uc778\uc5d0 \uc758\ud574 \uac19\uc740 \uacf3\uc5d0\uc11c 40\uba85\uc758 \uc720\ub300\uc778 \ub0a8\uc131\uc774 \ucc98\ud615\ub418\uc5c8\ub2e4. \uc774\uac83\uc740 \ubc1c\uad74\uc5d0 \ucc38\uc5ec\ud55c \uace0\uace0\ud559 \ubc0f \uc778\ub958\ud559 \ud300\uc5d0 \uc758\ud574 \uc870\uc0ac\ub41c \ub450 \uac1c\uc758 \ubb34\ub364\uc5d0 \uc788\ub294 \ud53c\ud574\uc790\uc758 \uc22b\uc790\ub85c \ud655\uc778\ub418\uc5c8\uace0 \uad6d\ub9bd\ucd94\ubaa8\uc5f0\uad6c\uc18c\uc5d0 \uc758\ud574 2000\ub144\uc5d0\uc11c 2003\ub144 \uc0ac\uc774\uc5d0 \uc2e4\uc2dc\ud55c \uacf5\uc2dd\uc870\uc0ac\ub85c \ucd5c\uc885 \ubc1c\uacac\ub418\uc5c8\ub2e4. \uc18c\ubb38\ubcf4\ub2e4 \ub354 \ub9ce\uc740 \uc0ac\ub78c\ub4e4\uc774 \ud76c\uc0dd\ub418\uc5c8\uc74c\uc744 \ubc18\uc99d\ud55c \uac83\uc774\ub2e4.", "sentence_answer": "\uc774\uac83\uc740 1941\ub144 6\uc6d4 30\uc77c \ub8e8\ub9c8\ub2c8\uc544 \uc774\uc544\uc2dc\uc758 \ud559\uc0b4\uc744 \ud3ec\ud568\ud558\ub294\ub370 \uadf8 \ud559\uc0b4\uc740 14,000 \uc720\ub300\uc778\uc774 \ub8e8\ub9c8\ub2c8\uc544 \uc8fc\ubbfc\uacfc \uacbd\ucc30\uc5d0 \uc758\ud574 \uc0ac\ub9dd\ud558\ub294 \uc0ac\uac74\uc774\ub2e4.", "paragraph_id": "6571961-12-0", "paragraph_question": "question: \ub8e8\ub9c8\ub2c8\uc544 \uc774\uc544\uc2dc\uc758 \ud559\uc0b4\uc774 \uc77c\uc5b4\ub09c \uac83\uc740 \uc5b8\uc81c\uc778\uac00?, context: \uc9c0\uc5ed\uc758 \uc778\uad6c\uc5d0 \uc758\ud55c \uc218\ub9ce\uc740 \uce58\uba85\uc801\uc778 \ud559\uc0b4\uc740 \uc81c 2\ucc28 \uc138\uacc4\ub300\uc804 \ub098\uce58\uc758 \uaca9\ub824\uc5d0 \uc758\ud574 \uc77c\ubd80\ub294 \uc790\ubc1c\uc801\uc73c\ub85c \ubc1c\uc0dd\ud588\ub2e4. \uc774\uac83\uc740 1941\ub144 6\uc6d4 30\uc77c \ub8e8\ub9c8\ub2c8\uc544 \uc774\uc544\uc2dc\uc758 \ud559\uc0b4\uc744 \ud3ec\ud568\ud558\ub294\ub370 \uadf8 \ud559\uc0b4\uc740 14,000 \uc720\ub300\uc778\uc774 \ub8e8\ub9c8\ub2c8\uc544 \uc8fc\ubbfc\uacfc \uacbd\ucc30\uc5d0 \uc758\ud574 \uc0ac\ub9dd\ud558\ub294 \uc0ac\uac74\uc774\ub2e4. \ub610\ud55c 1941\ub144 7\uc6d4\uc758 \uc2e4\ud06c \ud504\ub85c\uadf8\ub7a8(Jedwabne pogrom)\uc744 \ud3ec\ud568\ud558\ub294\ub370 \uc774\ub294 \ub098\uce58 \uc9c8\uc11c\uacbd\ucc30(Nazi Ordnungspolizei)\uac00 \uc788\ub294 \ub370\uc11c 300\uba85 \uc720\ub300\uc778\uc774 \ubd88 \ub09c \ud5db\uac04\uc5d0 \uc7a0\uae30\uac8c \ub418\ub294 \uc0ac\uac74\uc774\ub2e4. \ub3c5\uc77c\uc778\uc5d0 \uc758\ud574 \uac19\uc740 \uacf3\uc5d0\uc11c 40\uba85\uc758 \uc720\ub300\uc778 \ub0a8\uc131\uc774 \ucc98\ud615\ub418\uc5c8\ub2e4. \uc774\uac83\uc740 \ubc1c\uad74\uc5d0 \ucc38\uc5ec\ud55c \uace0\uace0\ud559 \ubc0f \uc778\ub958\ud559 \ud300\uc5d0 \uc758\ud574 \uc870\uc0ac\ub41c \ub450 \uac1c\uc758 \ubb34\ub364\uc5d0 \uc788\ub294 \ud53c\ud574\uc790\uc758 \uc22b\uc790\ub85c \ud655\uc778\ub418\uc5c8\uace0 \uad6d\ub9bd\ucd94\ubaa8\uc5f0\uad6c\uc18c\uc5d0 \uc758\ud574 2000\ub144\uc5d0\uc11c 2003\ub144 \uc0ac\uc774\uc5d0 \uc2e4\uc2dc\ud55c \uacf5\uc2dd\uc870\uc0ac\ub85c \ucd5c\uc885 \ubc1c\uacac\ub418\uc5c8\ub2e4. \uc18c\ubb38\ubcf4\ub2e4 \ub354 \ub9ce\uc740 \uc0ac\ub78c\ub4e4\uc774 \ud76c\uc0dd\ub418\uc5c8\uc74c\uc744 \ubc18\uc99d\ud55c \uac83\uc774\ub2e4."} {"question": "\ub204\ucf08\ub77c\ube44\uc5d0\uac8c \ucad3\uae38\ub54c \uc0b4\uc544\ub0a8\ub294 \ubc29\ubc95\uc740?", "paragraph": "\ub204\ucf08\ub77c\ube44\ub294 \uc2a4\ucf54\ud2c0\ub79c\ub4dc \ubc0f \uadf8 \ubd80\uc18d \ub3c4\uc11c\uc5d0\uc11c \uc804\uc2b9\ub418\ub294 \uc545\uadc0\ub4e4 \uc911\uc5d0\uc11c\ub3c4 \uac00\uc7a5 \uc0ac\uc545\ud55c \uac83\uc73c\ub85c, \uc5b4\ub5a0\ud55c \uc57d\uc810\ub3c4 \uc804\uc2b9\ub418\uc9c0 \uc54a\ub294\ub2e4. \ub204\ucf08\ub77c\ube44\ub97c \uc81c\uc5b4\ud560 \uc218 \uc788\ub294 \uac83\uc740 \uc624\ud06c\ub2c8 \uc804\uc124\uc5d0 \ub098\uc624\ub294 \uc5ec\ub984\uc758 \uc5ec\uc2e0 \ubc14\ub2e4\uc5b4\ubbf8\ub85c, \uadf8\ub140\ub294 \uc5ec\ub984 \ub3d9\uc548 \ub204\ucf08\ub77c\ube44\uac00 \ub0a0\ub6f0\uc9c0 \ubabb\ud558\ub3c4\ub85d \uac00\ub450\uc5b4 \ub193\ub294\ub2e4. \ubc14\ub2e4 \uad34\ubb3c\ub4e4\uc774 \ud754\ud788 \uadf8\ub807\ub4ef(\ucf08\ud53c \uac19\uc740 \uc608\uc678\ub294 \uc788\uc9c0\ub9cc), \ub204\ucf08\ub77c\ube44\ub294 \ud750\ub974\ub294 \ubbfc\ubb3c\uc744 \uac74\ub110 \uc218 \uc5c6\ub2e4. \uadf8\ub807\uae30\uc5d0 \ub204\ucf08\ub77c\ube44\uc5d0\uac8c \ucad3\uae38 \ub54c \uc0b4 \uc218 \uc788\ub294 \ubc29\ubc95\uc740 \uac15\uc744 \uac74\ub108\ub294 \uac83\ubc16\uc5d0 \uc5c6\ub2e4. \uc55e\uc11c \uc5b8\uae09\ub41c \ud0c0\ub9c8\uc2a4\ub294 \uc606\uc5d0 \uc788\ub358 \ud638\uc18c\uc758 \ubb3c\uc744 \uc6b0\uc5f0\ud788 \ub204\ucf08\ub77c\ube44\uc5d0\uac8c \ud280\uacbc\uae30 \ub54c\ubb38\uc5d0 \uac04\uc2e0\ud788 \uc0b4\uc544\ub0a0 \uc218 \uc788\uc5c8\ub2e4. \ubbfc\ubb3c\ub85c \uc778\ud574 \uad34\ubb3c\uc774 \uc2e0\uacbd\uc774 \ud750\ud2b8\ub7ec\uc9c4 \ud2c8\uc744 \ud0c0 \ud0c0\ub9c8\uc2a4\ub294 \uc606\uc758 \ubbfc\ubb3c \uc218\ub85c\ub97c \uac74\ub108\ub6f0\uc5c8\uace0 \uc548\uc804\ud558\uac8c \ubc18\ub300\ud3b8 \uac15\ub451\uc5d0 \ub3c4\ucc29\ud588\ub2e4.", "answer": "\uac15\uc744 \uac74\ub108\ub294 \uac83", "sentence": "\uadf8\ub807\uae30\uc5d0 \ub204\ucf08\ub77c\ube44\uc5d0\uac8c \ucad3\uae38 \ub54c \uc0b4 \uc218 \uc788\ub294 \ubc29\ubc95\uc740 \uac15\uc744 \uac74\ub108\ub294 \uac83 \ubc16\uc5d0 \uc5c6\ub2e4.", "paragraph_sentence": "\ub204\ucf08\ub77c\ube44\ub294 \uc2a4\ucf54\ud2c0\ub79c\ub4dc \ubc0f \uadf8 \ubd80\uc18d \ub3c4\uc11c\uc5d0\uc11c \uc804\uc2b9\ub418\ub294 \uc545\uadc0\ub4e4 \uc911\uc5d0\uc11c\ub3c4 \uac00\uc7a5 \uc0ac\uc545\ud55c \uac83\uc73c\ub85c, \uc5b4\ub5a0\ud55c \uc57d\uc810\ub3c4 \uc804\uc2b9\ub418\uc9c0 \uc54a\ub294\ub2e4. \ub204\ucf08\ub77c\ube44\ub97c \uc81c\uc5b4\ud560 \uc218 \uc788\ub294 \uac83\uc740 \uc624\ud06c\ub2c8 \uc804\uc124\uc5d0 \ub098\uc624\ub294 \uc5ec\ub984\uc758 \uc5ec\uc2e0 \ubc14\ub2e4\uc5b4\ubbf8\ub85c, \uadf8\ub140\ub294 \uc5ec\ub984 \ub3d9\uc548 \ub204\ucf08\ub77c\ube44\uac00 \ub0a0\ub6f0\uc9c0 \ubabb\ud558\ub3c4\ub85d \uac00\ub450\uc5b4 \ub193\ub294\ub2e4. \ubc14\ub2e4 \uad34\ubb3c\ub4e4\uc774 \ud754\ud788 \uadf8\ub807\ub4ef(\ucf08\ud53c \uac19\uc740 \uc608\uc678\ub294 \uc788\uc9c0\ub9cc), \ub204\ucf08\ub77c\ube44\ub294 \ud750\ub974\ub294 \ubbfc\ubb3c\uc744 \uac74\ub110 \uc218 \uc5c6\ub2e4. \uadf8\ub807\uae30\uc5d0 \ub204\ucf08\ub77c\ube44\uc5d0\uac8c \ucad3\uae38 \ub54c \uc0b4 \uc218 \uc788\ub294 \ubc29\ubc95\uc740 \uac15\uc744 \uac74\ub108\ub294 \uac83 \ubc16\uc5d0 \uc5c6\ub2e4. \uc55e\uc11c \uc5b8\uae09\ub41c \ud0c0\ub9c8\uc2a4\ub294 \uc606\uc5d0 \uc788\ub358 \ud638\uc18c\uc758 \ubb3c\uc744 \uc6b0\uc5f0\ud788 \ub204\ucf08\ub77c\ube44\uc5d0\uac8c \ud280\uacbc\uae30 \ub54c\ubb38\uc5d0 \uac04\uc2e0\ud788 \uc0b4\uc544\ub0a0 \uc218 \uc788\uc5c8\ub2e4. \ubbfc\ubb3c\ub85c \uc778\ud574 \uad34\ubb3c\uc774 \uc2e0\uacbd\uc774 \ud750\ud2b8\ub7ec\uc9c4 \ud2c8\uc744 \ud0c0 \ud0c0\ub9c8\uc2a4\ub294 \uc606\uc758 \ubbfc\ubb3c \uc218\ub85c\ub97c \uac74\ub108\ub6f0\uc5c8\uace0 \uc548\uc804\ud558\uac8c \ubc18\ub300\ud3b8 \uac15\ub451\uc5d0 \ub3c4\ucc29\ud588\ub2e4.", "paragraph_answer": "\ub204\ucf08\ub77c\ube44\ub294 \uc2a4\ucf54\ud2c0\ub79c\ub4dc \ubc0f \uadf8 \ubd80\uc18d \ub3c4\uc11c\uc5d0\uc11c \uc804\uc2b9\ub418\ub294 \uc545\uadc0\ub4e4 \uc911\uc5d0\uc11c\ub3c4 \uac00\uc7a5 \uc0ac\uc545\ud55c \uac83\uc73c\ub85c, \uc5b4\ub5a0\ud55c \uc57d\uc810\ub3c4 \uc804\uc2b9\ub418\uc9c0 \uc54a\ub294\ub2e4. \ub204\ucf08\ub77c\ube44\ub97c \uc81c\uc5b4\ud560 \uc218 \uc788\ub294 \uac83\uc740 \uc624\ud06c\ub2c8 \uc804\uc124\uc5d0 \ub098\uc624\ub294 \uc5ec\ub984\uc758 \uc5ec\uc2e0 \ubc14\ub2e4\uc5b4\ubbf8\ub85c, \uadf8\ub140\ub294 \uc5ec\ub984 \ub3d9\uc548 \ub204\ucf08\ub77c\ube44\uac00 \ub0a0\ub6f0\uc9c0 \ubabb\ud558\ub3c4\ub85d \uac00\ub450\uc5b4 \ub193\ub294\ub2e4. \ubc14\ub2e4 \uad34\ubb3c\ub4e4\uc774 \ud754\ud788 \uadf8\ub807\ub4ef(\ucf08\ud53c \uac19\uc740 \uc608\uc678\ub294 \uc788\uc9c0\ub9cc), \ub204\ucf08\ub77c\ube44\ub294 \ud750\ub974\ub294 \ubbfc\ubb3c\uc744 \uac74\ub110 \uc218 \uc5c6\ub2e4. \uadf8\ub807\uae30\uc5d0 \ub204\ucf08\ub77c\ube44\uc5d0\uac8c \ucad3\uae38 \ub54c \uc0b4 \uc218 \uc788\ub294 \ubc29\ubc95\uc740 \uac15\uc744 \uac74\ub108\ub294 \uac83 \ubc16\uc5d0 \uc5c6\ub2e4. \uc55e\uc11c \uc5b8\uae09\ub41c \ud0c0\ub9c8\uc2a4\ub294 \uc606\uc5d0 \uc788\ub358 \ud638\uc18c\uc758 \ubb3c\uc744 \uc6b0\uc5f0\ud788 \ub204\ucf08\ub77c\ube44\uc5d0\uac8c \ud280\uacbc\uae30 \ub54c\ubb38\uc5d0 \uac04\uc2e0\ud788 \uc0b4\uc544\ub0a0 \uc218 \uc788\uc5c8\ub2e4. \ubbfc\ubb3c\ub85c \uc778\ud574 \uad34\ubb3c\uc774 \uc2e0\uacbd\uc774 \ud750\ud2b8\ub7ec\uc9c4 \ud2c8\uc744 \ud0c0 \ud0c0\ub9c8\uc2a4\ub294 \uc606\uc758 \ubbfc\ubb3c \uc218\ub85c\ub97c \uac74\ub108\ub6f0\uc5c8\uace0 \uc548\uc804\ud558\uac8c \ubc18\ub300\ud3b8 \uac15\ub451\uc5d0 \ub3c4\ucc29\ud588\ub2e4.", "sentence_answer": "\uadf8\ub807\uae30\uc5d0 \ub204\ucf08\ub77c\ube44\uc5d0\uac8c \ucad3\uae38 \ub54c \uc0b4 \uc218 \uc788\ub294 \ubc29\ubc95\uc740 \uac15\uc744 \uac74\ub108\ub294 \uac83 \ubc16\uc5d0 \uc5c6\ub2e4.", "paragraph_id": "6028245-3-1", "paragraph_question": "question: \ub204\ucf08\ub77c\ube44\uc5d0\uac8c \ucad3\uae38\ub54c \uc0b4\uc544\ub0a8\ub294 \ubc29\ubc95\uc740?, context: \ub204\ucf08\ub77c\ube44\ub294 \uc2a4\ucf54\ud2c0\ub79c\ub4dc \ubc0f \uadf8 \ubd80\uc18d \ub3c4\uc11c\uc5d0\uc11c \uc804\uc2b9\ub418\ub294 \uc545\uadc0\ub4e4 \uc911\uc5d0\uc11c\ub3c4 \uac00\uc7a5 \uc0ac\uc545\ud55c \uac83\uc73c\ub85c, \uc5b4\ub5a0\ud55c \uc57d\uc810\ub3c4 \uc804\uc2b9\ub418\uc9c0 \uc54a\ub294\ub2e4. \ub204\ucf08\ub77c\ube44\ub97c \uc81c\uc5b4\ud560 \uc218 \uc788\ub294 \uac83\uc740 \uc624\ud06c\ub2c8 \uc804\uc124\uc5d0 \ub098\uc624\ub294 \uc5ec\ub984\uc758 \uc5ec\uc2e0 \ubc14\ub2e4\uc5b4\ubbf8\ub85c, \uadf8\ub140\ub294 \uc5ec\ub984 \ub3d9\uc548 \ub204\ucf08\ub77c\ube44\uac00 \ub0a0\ub6f0\uc9c0 \ubabb\ud558\ub3c4\ub85d \uac00\ub450\uc5b4 \ub193\ub294\ub2e4. \ubc14\ub2e4 \uad34\ubb3c\ub4e4\uc774 \ud754\ud788 \uadf8\ub807\ub4ef(\ucf08\ud53c \uac19\uc740 \uc608\uc678\ub294 \uc788\uc9c0\ub9cc), \ub204\ucf08\ub77c\ube44\ub294 \ud750\ub974\ub294 \ubbfc\ubb3c\uc744 \uac74\ub110 \uc218 \uc5c6\ub2e4. \uadf8\ub807\uae30\uc5d0 \ub204\ucf08\ub77c\ube44\uc5d0\uac8c \ucad3\uae38 \ub54c \uc0b4 \uc218 \uc788\ub294 \ubc29\ubc95\uc740 \uac15\uc744 \uac74\ub108\ub294 \uac83\ubc16\uc5d0 \uc5c6\ub2e4. \uc55e\uc11c \uc5b8\uae09\ub41c \ud0c0\ub9c8\uc2a4\ub294 \uc606\uc5d0 \uc788\ub358 \ud638\uc18c\uc758 \ubb3c\uc744 \uc6b0\uc5f0\ud788 \ub204\ucf08\ub77c\ube44\uc5d0\uac8c \ud280\uacbc\uae30 \ub54c\ubb38\uc5d0 \uac04\uc2e0\ud788 \uc0b4\uc544\ub0a0 \uc218 \uc788\uc5c8\ub2e4. \ubbfc\ubb3c\ub85c \uc778\ud574 \uad34\ubb3c\uc774 \uc2e0\uacbd\uc774 \ud750\ud2b8\ub7ec\uc9c4 \ud2c8\uc744 \ud0c0 \ud0c0\ub9c8\uc2a4\ub294 \uc606\uc758 \ubbfc\ubb3c \uc218\ub85c\ub97c \uac74\ub108\ub6f0\uc5c8\uace0 \uc548\uc804\ud558\uac8c \ubc18\ub300\ud3b8 \uac15\ub451\uc5d0 \ub3c4\ucc29\ud588\ub2e4."} {"question": "1998\ub144 \uc720\ud1a0\ud53c\uc544 \ub514\uc2a4\ud1a0\ud53c\uc544\ub85c \uae00\uc4f0\uae30\uc2dc\uc791\ud55c \uacf3\uc740 \uc5b4\ub514\uc778\uac00?", "paragraph": "1962\ub144 \uc804\ub77c\ubd81\ub3c4\uc5d0\uc11c \ud0dc\uc5b4\ub0ac\ub2e4. \uc9c1\uc5c5\uad70\uc778\uc778 \uc544\ubc84\uc9c0\ub97c \ub530\ub77c \uc790\uc8fc \uc774\uc0ac\ub97c \ub2e4\ub2c8\uba70 \uc5ec\ub7ec \uac00\uc9c0 \uc0ac\ud68c\uc801 \ubb38\uc81c\uc5d0 \ub300\ud574 \ub9ce\uc740 \uc815\uc11c\uc801 \uc790\uadf9\uc744 \ubc1b\uc558\ub2e4\uace0 \ud55c\ub2e4. 1980\ub144\ub300 \ucd08 \ud55c\uc2e0\ub300\ud559\uad50\uc5d0 \uc7ac\ud559\ud558\uba70 \ub098\ub984\uc758 \uc0ac\ud68c\uc758\uc2dd\uc744 \uac16\uac8c \ub418\uc5c8\uace0, \uae30\ub3c5\uad50\ub97c \ubbff\uac8c\ub418\uc5c8\ub2e4. 1990\ub144\ub300 \ucd08\uae4c\uc9c0 \uc11c\uc6b8\uc601\uc0c1\uc9d1\ub2e8\uacfc \ubbfc\uc911\ubb38\ud654\uc6b4\ub3d9\uc5f0\ud569\uc5d0\uc11c \ud65c\ub3d9\ud588\ub2e4. 1998\ub144 \u300a\uc528\ub12421\u300b\uc758 \u2018\uc720\ud1a0\ud53c\uc544 \ub514\uc2a4\ud1a0\ud53c\uc544\u2019\ub85c \uae00\uc4f0\uae30\ub97c \uc2dc\uc791\ud588\ub2e4. 2000\ub144 \ud64d\uc138\ud654, \uc9c4\uc911\uad8c \ub4f1\uacfc \ud568\uaed8 \uc0ac\ud68c\ubb38\ud654 \ube44\ud3c9\uc9c0 \u300a\uc544\uc6c3\uc0ac\uc774\ub354\u300b\ub97c \ub9cc\ub4e4\uc5b4 \ud3b8\uc9d1\uc8fc\uac04\uc744 \uc9c0\ub0c8\ub2e4. 2003\ub144 10\uc6d4\ubd80\ud130 \uc5b4\ub9b0\uc774 \uad50\uc591\uc9c0 \u300a\uace0\ub798\uac00 \uadf8\ub7ac\uc5b4\u300b\ub97c \ub9cc\ub4e4\uc5b4 \ubc1c\ud589\uc778\uc744 \ub9e1\uace0 \uc788\ub2e4. \uc800\uc11c\ub85c\ub294 \u300aB\uae09 \uc88c\ud30c\u300b(2001)\uc640 \u300a\ub098\ub294 \uc65c \ubd88\uc628\ud55c\uac00\u300b(2005), \u300a\uc608\uc218\uc804\u300b(2009)\uc774 \uc788\ub2e4.", "answer": "\uc528\ub12421", "sentence": "1998\ub144 \u300a \uc528\ub12421 \u300b\uc758 \u2018\uc720\ud1a0\ud53c\uc544 \ub514\uc2a4\ud1a0\ud53c\uc544\u2019\ub85c \uae00\uc4f0\uae30\ub97c \uc2dc\uc791\ud588\ub2e4.", "paragraph_sentence": "1962\ub144 \uc804\ub77c\ubd81\ub3c4\uc5d0\uc11c \ud0dc\uc5b4\ub0ac\ub2e4. \uc9c1\uc5c5\uad70\uc778\uc778 \uc544\ubc84\uc9c0\ub97c \ub530\ub77c \uc790\uc8fc \uc774\uc0ac\ub97c \ub2e4\ub2c8\uba70 \uc5ec\ub7ec \uac00\uc9c0 \uc0ac\ud68c\uc801 \ubb38\uc81c\uc5d0 \ub300\ud574 \ub9ce\uc740 \uc815\uc11c\uc801 \uc790\uadf9\uc744 \ubc1b\uc558\ub2e4\uace0 \ud55c\ub2e4. 1980\ub144\ub300 \ucd08 \ud55c\uc2e0\ub300\ud559\uad50\uc5d0 \uc7ac\ud559\ud558\uba70 \ub098\ub984\uc758 \uc0ac\ud68c\uc758\uc2dd\uc744 \uac16\uac8c \ub418\uc5c8\uace0, \uae30\ub3c5\uad50\ub97c \ubbff\uac8c\ub418\uc5c8\ub2e4. 1990\ub144\ub300 \ucd08\uae4c\uc9c0 \uc11c\uc6b8\uc601\uc0c1\uc9d1\ub2e8\uacfc \ubbfc\uc911\ubb38\ud654\uc6b4\ub3d9\uc5f0\ud569\uc5d0\uc11c \ud65c\ub3d9\ud588\ub2e4. 1998\ub144 \u300a \uc528\ub12421 \u300b\uc758 \u2018\uc720\ud1a0\ud53c\uc544 \ub514\uc2a4\ud1a0\ud53c\uc544\u2019\ub85c \uae00\uc4f0\uae30\ub97c \uc2dc\uc791\ud588\ub2e4. 2000\ub144 \ud64d\uc138\ud654, \uc9c4\uc911\uad8c \ub4f1\uacfc \ud568\uaed8 \uc0ac\ud68c\ubb38\ud654 \ube44\ud3c9\uc9c0 \u300a\uc544\uc6c3\uc0ac\uc774\ub354\u300b\ub97c \ub9cc\ub4e4\uc5b4 \ud3b8\uc9d1\uc8fc\uac04\uc744 \uc9c0\ub0c8\ub2e4. 2003\ub144 10\uc6d4\ubd80\ud130 \uc5b4\ub9b0\uc774 \uad50\uc591\uc9c0 \u300a\uace0\ub798\uac00 \uadf8\ub7ac\uc5b4\u300b\ub97c \ub9cc\ub4e4\uc5b4 \ubc1c\ud589\uc778\uc744 \ub9e1\uace0 \uc788\ub2e4. \uc800\uc11c\ub85c\ub294 \u300aB\uae09 \uc88c\ud30c\u300b(2001)\uc640 \u300a\ub098\ub294 \uc65c \ubd88\uc628\ud55c\uac00\u300b(2005), \u300a\uc608\uc218\uc804\u300b(2009)\uc774 \uc788\ub2e4.", "paragraph_answer": "1962\ub144 \uc804\ub77c\ubd81\ub3c4\uc5d0\uc11c \ud0dc\uc5b4\ub0ac\ub2e4. \uc9c1\uc5c5\uad70\uc778\uc778 \uc544\ubc84\uc9c0\ub97c \ub530\ub77c \uc790\uc8fc \uc774\uc0ac\ub97c \ub2e4\ub2c8\uba70 \uc5ec\ub7ec \uac00\uc9c0 \uc0ac\ud68c\uc801 \ubb38\uc81c\uc5d0 \ub300\ud574 \ub9ce\uc740 \uc815\uc11c\uc801 \uc790\uadf9\uc744 \ubc1b\uc558\ub2e4\uace0 \ud55c\ub2e4. 1980\ub144\ub300 \ucd08 \ud55c\uc2e0\ub300\ud559\uad50\uc5d0 \uc7ac\ud559\ud558\uba70 \ub098\ub984\uc758 \uc0ac\ud68c\uc758\uc2dd\uc744 \uac16\uac8c \ub418\uc5c8\uace0, \uae30\ub3c5\uad50\ub97c \ubbff\uac8c\ub418\uc5c8\ub2e4. 1990\ub144\ub300 \ucd08\uae4c\uc9c0 \uc11c\uc6b8\uc601\uc0c1\uc9d1\ub2e8\uacfc \ubbfc\uc911\ubb38\ud654\uc6b4\ub3d9\uc5f0\ud569\uc5d0\uc11c \ud65c\ub3d9\ud588\ub2e4. 1998\ub144 \u300a \uc528\ub12421 \u300b\uc758 \u2018\uc720\ud1a0\ud53c\uc544 \ub514\uc2a4\ud1a0\ud53c\uc544\u2019\ub85c \uae00\uc4f0\uae30\ub97c \uc2dc\uc791\ud588\ub2e4. 2000\ub144 \ud64d\uc138\ud654, \uc9c4\uc911\uad8c \ub4f1\uacfc \ud568\uaed8 \uc0ac\ud68c\ubb38\ud654 \ube44\ud3c9\uc9c0 \u300a\uc544\uc6c3\uc0ac\uc774\ub354\u300b\ub97c \ub9cc\ub4e4\uc5b4 \ud3b8\uc9d1\uc8fc\uac04\uc744 \uc9c0\ub0c8\ub2e4. 2003\ub144 10\uc6d4\ubd80\ud130 \uc5b4\ub9b0\uc774 \uad50\uc591\uc9c0 \u300a\uace0\ub798\uac00 \uadf8\ub7ac\uc5b4\u300b\ub97c \ub9cc\ub4e4\uc5b4 \ubc1c\ud589\uc778\uc744 \ub9e1\uace0 \uc788\ub2e4. \uc800\uc11c\ub85c\ub294 \u300aB\uae09 \uc88c\ud30c\u300b(2001)\uc640 \u300a\ub098\ub294 \uc65c \ubd88\uc628\ud55c\uac00\u300b(2005), \u300a\uc608\uc218\uc804\u300b(2009)\uc774 \uc788\ub2e4.", "sentence_answer": "1998\ub144 \u300a \uc528\ub12421 \u300b\uc758 \u2018\uc720\ud1a0\ud53c\uc544 \ub514\uc2a4\ud1a0\ud53c\uc544\u2019\ub85c \uae00\uc4f0\uae30\ub97c \uc2dc\uc791\ud588\ub2e4.", "paragraph_id": "6576785-0-2", "paragraph_question": "question: 1998\ub144 \uc720\ud1a0\ud53c\uc544 \ub514\uc2a4\ud1a0\ud53c\uc544\ub85c \uae00\uc4f0\uae30\uc2dc\uc791\ud55c \uacf3\uc740 \uc5b4\ub514\uc778\uac00?, context: 1962\ub144 \uc804\ub77c\ubd81\ub3c4\uc5d0\uc11c \ud0dc\uc5b4\ub0ac\ub2e4. \uc9c1\uc5c5\uad70\uc778\uc778 \uc544\ubc84\uc9c0\ub97c \ub530\ub77c \uc790\uc8fc \uc774\uc0ac\ub97c \ub2e4\ub2c8\uba70 \uc5ec\ub7ec \uac00\uc9c0 \uc0ac\ud68c\uc801 \ubb38\uc81c\uc5d0 \ub300\ud574 \ub9ce\uc740 \uc815\uc11c\uc801 \uc790\uadf9\uc744 \ubc1b\uc558\ub2e4\uace0 \ud55c\ub2e4. 1980\ub144\ub300 \ucd08 \ud55c\uc2e0\ub300\ud559\uad50\uc5d0 \uc7ac\ud559\ud558\uba70 \ub098\ub984\uc758 \uc0ac\ud68c\uc758\uc2dd\uc744 \uac16\uac8c \ub418\uc5c8\uace0, \uae30\ub3c5\uad50\ub97c \ubbff\uac8c\ub418\uc5c8\ub2e4. 1990\ub144\ub300 \ucd08\uae4c\uc9c0 \uc11c\uc6b8\uc601\uc0c1\uc9d1\ub2e8\uacfc \ubbfc\uc911\ubb38\ud654\uc6b4\ub3d9\uc5f0\ud569\uc5d0\uc11c \ud65c\ub3d9\ud588\ub2e4. 1998\ub144 \u300a\uc528\ub12421\u300b\uc758 \u2018\uc720\ud1a0\ud53c\uc544 \ub514\uc2a4\ud1a0\ud53c\uc544\u2019\ub85c \uae00\uc4f0\uae30\ub97c \uc2dc\uc791\ud588\ub2e4. 2000\ub144 \ud64d\uc138\ud654, \uc9c4\uc911\uad8c \ub4f1\uacfc \ud568\uaed8 \uc0ac\ud68c\ubb38\ud654 \ube44\ud3c9\uc9c0 \u300a\uc544\uc6c3\uc0ac\uc774\ub354\u300b\ub97c \ub9cc\ub4e4\uc5b4 \ud3b8\uc9d1\uc8fc\uac04\uc744 \uc9c0\ub0c8\ub2e4. 2003\ub144 10\uc6d4\ubd80\ud130 \uc5b4\ub9b0\uc774 \uad50\uc591\uc9c0 \u300a\uace0\ub798\uac00 \uadf8\ub7ac\uc5b4\u300b\ub97c \ub9cc\ub4e4\uc5b4 \ubc1c\ud589\uc778\uc744 \ub9e1\uace0 \uc788\ub2e4. \uc800\uc11c\ub85c\ub294 \u300aB\uae09 \uc88c\ud30c\u300b(2001)\uc640 \u300a\ub098\ub294 \uc65c \ubd88\uc628\ud55c\uac00\u300b(2005), \u300a\uc608\uc218\uc804\u300b(2009)\uc774 \uc788\ub2e4."} {"question": "\ube4c\ub9ac\uac00 \uc790\uc2e0\ub4e4\uacfc\ub294 \ub2e4\ub978 \uc0b6\uc744 \uc0b4 \uc218 \uc788\ub2e4\uace0 \uc548\ub3c4\ud55c \uc0ac\ub78c\ub4e4\uc740 \ub204\uad6c\uc778\uac00\uc694?", "paragraph": "\ub354\ub7fc\uc73c\ub85c \ub3cc\uc544\uc640\uc11c, \ube4c\ub9ac\uc640 \ube4c\ub9ac\uc758 \uc544\ube60\ub294 \ud3c9\uc18c\uc640 \uac19\uc740 \uc0dd\ud65c\uc744 \uc774\uc5b4\uac04\ub2e4. \uadf8\ub7ec\ub358 \uc5b4\ub290 \ub0a0 \ud559\uad50\uc5d0\uc11c \ud3b8\uc9c0\uac00 \ub3c4\ucc29\ud558\uace0, \ube4c\ub9ac\ub294 \ubaa8\ub450\uc5d0\uac8c \ub5a8\uc5b4\uc9c4 \uac83 \ucc98\ub7fc \uc774\uc57c\uae30\ud55c\ub2e4. \ud1a0\ub2c8\ub294 \uc4f0\ub808\uae30\ud1b5\uc5d0\uc11c \uad6c\uaca8\uc9c4 \ud3b8\uc9c0\ub97c \uc8fc\uc6e0\uace0, \ube4c\ub9ac\uac00 \uc7a5\ub09c\uc744 \ucce4\uc73c\uba70 \ud569\uaca9\ud588\ub2e4\ub294 \uc0ac\uc2e4\uc744 \uc54c\uac8c\ub41c\ub2e4. \ub3d9\uc2dc\uc5d0, \uad11\ubd80\ub4e4\uc740 \ud30c\uc5c5\uc744 \uadf8\ub9cc\ub450\uace0 \ub3c4\ub85c \uc77c\uc744 \ud560 \uac83\uc744 \uacb0\uc2ec\ud55c\ub2e4. \uadf8\ub7ec\ub098 \uad11\ubd80\ub4e4\uc740 \uc801\uc5b4\ub3c4 \ube4c\ub9ac\uac00 \uc7bf\ub354\ubbf8 \uc18d\uc5d0\uc11c \uc790\ub77c\uc9c0 \uc54a\ub294\ub2e4\ub294 \uc0ac\uc2e4\uc5d0 \uc548\ub3c4\ud55c\ub2e4. \ube4c\ub9ac\ub294 \ubaa8\ub4e0 \uad11\ubd80\ub4e4\uacfc (*Once We Were Kings) \uc70c\ud0a8\uc2a8 \ubd80\uc778, \uadf8\ub9ac\uace0 \ube4c\ub9ac\ub97c \uadf8\ub9ac\uc6cc\ud558\ub294 \ub370\ube44 (*The Letter Reprise (Billy's Reply))\uc5d0\uac8c \uc778\uc0ac\ud55c\ub2e4. \ub9c8\uc774\ud074 \uc5ed\uc2dc \ube4c\ub9ac\uc5d0\uac8c \uc791\ubcc4\uc744 \uace0\ud558\uace0, \ube4c\ub9ac\ub294 \ub9c8\uc774\ud074\uc758 \ubea8\uc5d0 \uce5c\uadfc\ud55c \ud0a4\uc2a4\ub97c \uac74\ub128\ub2e4 (*Finale). \uac77\ub294 \ube4c\ub9ac\uc758 \ub4a4\ub85c \ud6c4\uad11\uc774 \ube44\uce5c\ub2e4.", "answer": "\ubaa8\ub4e0 \uad11\ubd80\ub4e4", "sentence": "\ube4c\ub9ac\ub294 \ubaa8\ub4e0 \uad11\ubd80\ub4e4 \uacfc (*Once We Were Kings) \uc70c\ud0a8\uc2a8 \ubd80\uc778, \uadf8\ub9ac\uace0 \ube4c\ub9ac\ub97c \uadf8\ub9ac\uc6cc\ud558\ub294 \ub370\ube44 (*The Letter Reprise (Billy's Reply))\uc5d0\uac8c \uc778\uc0ac\ud55c\ub2e4.", "paragraph_sentence": "\ub354\ub7fc\uc73c\ub85c \ub3cc\uc544\uc640\uc11c, \ube4c\ub9ac\uc640 \ube4c\ub9ac\uc758 \uc544\ube60\ub294 \ud3c9\uc18c\uc640 \uac19\uc740 \uc0dd\ud65c\uc744 \uc774\uc5b4\uac04\ub2e4. \uadf8\ub7ec\ub358 \uc5b4\ub290 \ub0a0 \ud559\uad50\uc5d0\uc11c \ud3b8\uc9c0\uac00 \ub3c4\ucc29\ud558\uace0, \ube4c\ub9ac\ub294 \ubaa8\ub450\uc5d0\uac8c \ub5a8\uc5b4\uc9c4 \uac83 \ucc98\ub7fc \uc774\uc57c\uae30\ud55c\ub2e4. \ud1a0\ub2c8\ub294 \uc4f0\ub808\uae30\ud1b5\uc5d0\uc11c \uad6c\uaca8\uc9c4 \ud3b8\uc9c0\ub97c \uc8fc\uc6e0\uace0, \ube4c\ub9ac\uac00 \uc7a5\ub09c\uc744 \ucce4\uc73c\uba70 \ud569\uaca9\ud588\ub2e4\ub294 \uc0ac\uc2e4\uc744 \uc54c\uac8c\ub41c\ub2e4. \ub3d9\uc2dc\uc5d0, \uad11\ubd80\ub4e4\uc740 \ud30c\uc5c5\uc744 \uadf8\ub9cc\ub450\uace0 \ub3c4\ub85c \uc77c\uc744 \ud560 \uac83\uc744 \uacb0\uc2ec\ud55c\ub2e4. \uadf8\ub7ec\ub098 \uad11\ubd80\ub4e4\uc740 \uc801\uc5b4\ub3c4 \ube4c\ub9ac\uac00 \uc7bf\ub354\ubbf8 \uc18d\uc5d0\uc11c \uc790\ub77c\uc9c0 \uc54a\ub294\ub2e4\ub294 \uc0ac\uc2e4\uc5d0 \uc548\ub3c4\ud55c\ub2e4. \ube4c\ub9ac\ub294 \ubaa8\ub4e0 \uad11\ubd80\ub4e4 \uacfc (*Once We Were Kings) \uc70c\ud0a8\uc2a8 \ubd80\uc778, \uadf8\ub9ac\uace0 \ube4c\ub9ac\ub97c \uadf8\ub9ac\uc6cc\ud558\ub294 \ub370\ube44 (*The Letter Reprise (Billy's Reply))\uc5d0\uac8c \uc778\uc0ac\ud55c\ub2e4. \ub9c8\uc774\ud074 \uc5ed\uc2dc \ube4c\ub9ac\uc5d0\uac8c \uc791\ubcc4\uc744 \uace0\ud558\uace0, \ube4c\ub9ac\ub294 \ub9c8\uc774\ud074\uc758 \ubea8\uc5d0 \uce5c\uadfc\ud55c \ud0a4\uc2a4\ub97c \uac74\ub128\ub2e4 (*Finale). \uac77\ub294 \ube4c\ub9ac\uc758 \ub4a4\ub85c \ud6c4\uad11\uc774 \ube44\uce5c\ub2e4.", "paragraph_answer": "\ub354\ub7fc\uc73c\ub85c \ub3cc\uc544\uc640\uc11c, \ube4c\ub9ac\uc640 \ube4c\ub9ac\uc758 \uc544\ube60\ub294 \ud3c9\uc18c\uc640 \uac19\uc740 \uc0dd\ud65c\uc744 \uc774\uc5b4\uac04\ub2e4. \uadf8\ub7ec\ub358 \uc5b4\ub290 \ub0a0 \ud559\uad50\uc5d0\uc11c \ud3b8\uc9c0\uac00 \ub3c4\ucc29\ud558\uace0, \ube4c\ub9ac\ub294 \ubaa8\ub450\uc5d0\uac8c \ub5a8\uc5b4\uc9c4 \uac83 \ucc98\ub7fc \uc774\uc57c\uae30\ud55c\ub2e4. \ud1a0\ub2c8\ub294 \uc4f0\ub808\uae30\ud1b5\uc5d0\uc11c \uad6c\uaca8\uc9c4 \ud3b8\uc9c0\ub97c \uc8fc\uc6e0\uace0, \ube4c\ub9ac\uac00 \uc7a5\ub09c\uc744 \ucce4\uc73c\uba70 \ud569\uaca9\ud588\ub2e4\ub294 \uc0ac\uc2e4\uc744 \uc54c\uac8c\ub41c\ub2e4. \ub3d9\uc2dc\uc5d0, \uad11\ubd80\ub4e4\uc740 \ud30c\uc5c5\uc744 \uadf8\ub9cc\ub450\uace0 \ub3c4\ub85c \uc77c\uc744 \ud560 \uac83\uc744 \uacb0\uc2ec\ud55c\ub2e4. \uadf8\ub7ec\ub098 \uad11\ubd80\ub4e4\uc740 \uc801\uc5b4\ub3c4 \ube4c\ub9ac\uac00 \uc7bf\ub354\ubbf8 \uc18d\uc5d0\uc11c \uc790\ub77c\uc9c0 \uc54a\ub294\ub2e4\ub294 \uc0ac\uc2e4\uc5d0 \uc548\ub3c4\ud55c\ub2e4. \ube4c\ub9ac\ub294 \ubaa8\ub4e0 \uad11\ubd80\ub4e4 \uacfc (*Once We Were Kings) \uc70c\ud0a8\uc2a8 \ubd80\uc778, \uadf8\ub9ac\uace0 \ube4c\ub9ac\ub97c \uadf8\ub9ac\uc6cc\ud558\ub294 \ub370\ube44 (*The Letter Reprise (Billy's Reply))\uc5d0\uac8c \uc778\uc0ac\ud55c\ub2e4. \ub9c8\uc774\ud074 \uc5ed\uc2dc \ube4c\ub9ac\uc5d0\uac8c \uc791\ubcc4\uc744 \uace0\ud558\uace0, \ube4c\ub9ac\ub294 \ub9c8\uc774\ud074\uc758 \ubea8\uc5d0 \uce5c\uadfc\ud55c \ud0a4\uc2a4\ub97c \uac74\ub128\ub2e4 (*Finale). \uac77\ub294 \ube4c\ub9ac\uc758 \ub4a4\ub85c \ud6c4\uad11\uc774 \ube44\uce5c\ub2e4.", "sentence_answer": "\ube4c\ub9ac\ub294 \ubaa8\ub4e0 \uad11\ubd80\ub4e4 \uacfc (*Once We Were Kings) \uc70c\ud0a8\uc2a8 \ubd80\uc778, \uadf8\ub9ac\uace0 \ube4c\ub9ac\ub97c \uadf8\ub9ac\uc6cc\ud558\ub294 \ub370\ube44 (*The Letter Reprise (Billy's Reply))\uc5d0\uac8c \uc778\uc0ac\ud55c\ub2e4.", "paragraph_id": "6129078-9-2", "paragraph_question": "question: \ube4c\ub9ac\uac00 \uc790\uc2e0\ub4e4\uacfc\ub294 \ub2e4\ub978 \uc0b6\uc744 \uc0b4 \uc218 \uc788\ub2e4\uace0 \uc548\ub3c4\ud55c \uc0ac\ub78c\ub4e4\uc740 \ub204\uad6c\uc778\uac00\uc694?, context: \ub354\ub7fc\uc73c\ub85c \ub3cc\uc544\uc640\uc11c, \ube4c\ub9ac\uc640 \ube4c\ub9ac\uc758 \uc544\ube60\ub294 \ud3c9\uc18c\uc640 \uac19\uc740 \uc0dd\ud65c\uc744 \uc774\uc5b4\uac04\ub2e4. \uadf8\ub7ec\ub358 \uc5b4\ub290 \ub0a0 \ud559\uad50\uc5d0\uc11c \ud3b8\uc9c0\uac00 \ub3c4\ucc29\ud558\uace0, \ube4c\ub9ac\ub294 \ubaa8\ub450\uc5d0\uac8c \ub5a8\uc5b4\uc9c4 \uac83 \ucc98\ub7fc \uc774\uc57c\uae30\ud55c\ub2e4. \ud1a0\ub2c8\ub294 \uc4f0\ub808\uae30\ud1b5\uc5d0\uc11c \uad6c\uaca8\uc9c4 \ud3b8\uc9c0\ub97c \uc8fc\uc6e0\uace0, \ube4c\ub9ac\uac00 \uc7a5\ub09c\uc744 \ucce4\uc73c\uba70 \ud569\uaca9\ud588\ub2e4\ub294 \uc0ac\uc2e4\uc744 \uc54c\uac8c\ub41c\ub2e4. \ub3d9\uc2dc\uc5d0, \uad11\ubd80\ub4e4\uc740 \ud30c\uc5c5\uc744 \uadf8\ub9cc\ub450\uace0 \ub3c4\ub85c \uc77c\uc744 \ud560 \uac83\uc744 \uacb0\uc2ec\ud55c\ub2e4. \uadf8\ub7ec\ub098 \uad11\ubd80\ub4e4\uc740 \uc801\uc5b4\ub3c4 \ube4c\ub9ac\uac00 \uc7bf\ub354\ubbf8 \uc18d\uc5d0\uc11c \uc790\ub77c\uc9c0 \uc54a\ub294\ub2e4\ub294 \uc0ac\uc2e4\uc5d0 \uc548\ub3c4\ud55c\ub2e4. \ube4c\ub9ac\ub294 \ubaa8\ub4e0 \uad11\ubd80\ub4e4\uacfc (*Once We Were Kings) \uc70c\ud0a8\uc2a8 \ubd80\uc778, \uadf8\ub9ac\uace0 \ube4c\ub9ac\ub97c \uadf8\ub9ac\uc6cc\ud558\ub294 \ub370\ube44 (*The Letter Reprise (Billy's Reply))\uc5d0\uac8c \uc778\uc0ac\ud55c\ub2e4. \ub9c8\uc774\ud074 \uc5ed\uc2dc \ube4c\ub9ac\uc5d0\uac8c \uc791\ubcc4\uc744 \uace0\ud558\uace0, \ube4c\ub9ac\ub294 \ub9c8\uc774\ud074\uc758 \ubea8\uc5d0 \uce5c\uadfc\ud55c \ud0a4\uc2a4\ub97c \uac74\ub128\ub2e4 (*Finale). \uac77\ub294 \ube4c\ub9ac\uc758 \ub4a4\ub85c \ud6c4\uad11\uc774 \ube44\uce5c\ub2e4."} {"question": "\ub178\uc544\uc758 \ubc29\uc8fc\uc5d0\uc11c \uc81c\uc77c \ucc98\uc74c \ub0a0\ub824\ubcf4\ub0b8 \ub3d9\ubb3c\uc740 \ubb34\uc5c7\uc778\uac00?", "paragraph": "\uc720\ub300\uad50, \uae30\ub3c5\uad50, \uc774\uc2ac\ub78c\uad50\uc5d0\uc11c\ub294 \ud070\uae4c\ub9c8\uadc0\uac00 \ub178\uc544\uc758 \ubc29\uc8fc\uc5d0\uc11c \ub9e8 \ucc98\uc74c \ub0a0\ub824\ubcf4\ub0b8 \ub3d9\ubb3c\ub85c \ub098\uc628\ub2e4(\ucc3d\uc138\uae30 8\uc7a5 6~8\uc808). \u201c\uc0ac\uc2ed \uc77c\uc744 \uc9c0\ub098\uc11c \ub178\uc544\uac00 \uadf8 \ubc29\uc8fc\uc5d0 \uc9c0\uc740 \ucc3d\uc744 \uc5f4\uace0 \uae4c\ub9c8\uadc0\ub97c \ub0b4\uc5b4 \ub193\uc73c\ub9e4 \uae4c\ub9c8\uadc0\uac00 \ubb3c\uc774 \ub545\uc5d0\uc11c \ub9c8\ub974\uae30\uae4c\uc9c0 \ub0a0\uc544 \uc655\ub798\ud558\uc600\ub354\ub77c \uadf8\uac00 \ub610 \ube44\ub458\uae30\ub97c \ub0b4\uc5b4 \ub193\uc544 \uc9c0\uba74\uc5d0 \ubb3c\uc774 \uac10\ud55c \uc5ec\ubd80\ub97c \uc54c\uace0\uc790 \ud558\ub9e4 \uc628 \uc9c0\uba74\uc5d0 \ubb3c\uc774 \uc788\uc73c\ubbc0\ub85c \ube44\ub458\uae30\uac00 \uc811\uc871\ud560 \uacf3\uc744 \ucc3e\uc9c0 \ubabb\ud558\uace0 \ubc29\uc8fc\ub85c \ub3cc\uc544\uc640 \uadf8\uc5d0\uac8c\ub85c \uc624\ub294\uc9c0\ub77c.\u201d \uc774 \uc678\uc5d0\ub3c4 \ud070\uae4c\ub9c8\uadc0\ub294 \uc131\uacbd\uc5d0\uc11c \uc5ec\ub7ec \ucc28\ub840 \uc5b8\uae09\ub41c\ub2e4. \uc2e0\uc57d\uc5d0\uc11c \uc608\uc218\uac00 \uac00\ub85c\ub418 \ud070\uae4c\ub9c8\uadc0\ub97c \ube44\uc720\ub85c \ub4e4\uba74\uc11c \ubd80\uadc0\uc601\ud654\ub97c \ubc14\ub77c\uace0 \uc2e0\uc744 \ubbff\uc9c0 \ub9d0\ub77c\uace0 \ud55c\ub2e4(\ub204\uac00\ubcf5\uc74c 12\uc7a5 24\uc808). \u201c\uae4c\ub9c8\uadc0\ub97c \uc0dd\uac01\ud558\ub77c \uc2ec\uc9c0\ub3c4 \uc544\ub2c8\ud558\uace0 \uac70\ub450\uc9c0\ub3c4 \uc544\ub2c8\ud558\uba70 \uace8\ubc29\ub3c4 \uc5c6\uace0 \ucc3d\uace0\ub3c4 \uc5c6\uc73c\ub418 \ud558\ub098\ub2d8\uc774 \uae30\ub974\uc2dc\ub098\ub2c8 \ub108\ud76c\ub294 \uc0c8\ubcf4\ub2e4 \uc5bc\ub9c8\ub098 \ub354 \uadc0\ud558\ub0d0?\u201d \ucfe0\ub780\uc5d0\uc11c\ub294 \uce74\uc778\uacfc \uc544\ubca8 \uc774\uc57c\uae30\uc5d0\uc11c \ud070\uae4c\ub9c8\uadc0\uac00 \ub4f1\uc7a5\ud55c\ub2e4(\ucfe0\ub780 5\uc7a5 31\uc808). \uc544\ub2f4\uc758 \uc7a5\ub0a8 \uce74\uc778\uc774 \uc544\uc6b0 \uc544\ubca8\uc744 \uccd0 \uc8fd\uc600\ub294\ub370 \uadf8 \uc2dc\uccb4\ub97c \uc5b4\ucc0c \ucc98\ub9ac\ud574\uc57c \ud560\uc9c0 \ubab0\ub790\ub2e4. \uc774\ub54c \u201c\ud558\ub098\ub2d8\uc774 \ud070\uae4c\ub9c8\uadc0 \ud55c \ub9c8\ub9ac\ub97c \ubcf4\ub0b4\uc2dc\uc5b4 \ub545\ubc14\ub2e5\uc744 \uae01\uae30\ub85c \uadf8 \ud615\uc81c\uc758 \ubc8c\uac70\uc22d\uc774 \uc1a1\uc7a5\uc744 \uc228\uae38 \ubc95\ub3c4\ub97c \ubcf4\uc5ec\uc8fc\uc2dc\ub2c8 \uadf8\uac00 \uac00\ub85c\ub418, \ube44\ud1b5\uc774\ub85c\ub2e4! \ub0b4\uac00 \uc774 \uae4c\ub9c8\uadc0\ucc98\ub7fc \ud560 \uc904 \ubab0\ub77c \ud615\uc81c\uc758 \ubc8c\uac70\uc22d\uc774 \uc1a1\uc7a5\uc744 \uc228\uae30\uc9c0 \ubabb\ud558\uaca0\ub294\uac00? \uadf8\ub9ac\uace0 \uadf8\ub294 \ud68c\uac1c\ud558\uc600\ub2e4.\u201d", "answer": "\ud070\uae4c\ub9c8\uadc0", "sentence": "\uc720\ub300\uad50, \uae30\ub3c5\uad50, \uc774\uc2ac\ub78c\uad50\uc5d0\uc11c\ub294 \ud070\uae4c\ub9c8\uadc0 \uac00 \ub178\uc544\uc758 \ubc29\uc8fc\uc5d0\uc11c \ub9e8 \ucc98\uc74c \ub0a0\ub824\ubcf4\ub0b8 \ub3d9\ubb3c\ub85c \ub098\uc628\ub2e4(\ucc3d\uc138\uae30 8\uc7a5 6~8\uc808)", "paragraph_sentence": " \uc720\ub300\uad50, \uae30\ub3c5\uad50, \uc774\uc2ac\ub78c\uad50\uc5d0\uc11c\ub294 \ud070\uae4c\ub9c8\uadc0 \uac00 \ub178\uc544\uc758 \ubc29\uc8fc\uc5d0\uc11c \ub9e8 \ucc98\uc74c \ub0a0\ub824\ubcf4\ub0b8 \ub3d9\ubb3c\ub85c \ub098\uc628\ub2e4(\ucc3d\uc138\uae30 8\uc7a5 6~8\uc808) . \u201c\uc0ac\uc2ed \uc77c\uc744 \uc9c0\ub098\uc11c \ub178\uc544\uac00 \uadf8 \ubc29\uc8fc\uc5d0 \uc9c0\uc740 \ucc3d\uc744 \uc5f4\uace0 \uae4c\ub9c8\uadc0\ub97c \ub0b4\uc5b4 \ub193\uc73c\ub9e4 \uae4c\ub9c8\uadc0\uac00 \ubb3c\uc774 \ub545\uc5d0\uc11c \ub9c8\ub974\uae30\uae4c\uc9c0 \ub0a0\uc544 \uc655\ub798\ud558\uc600\ub354\ub77c \uadf8\uac00 \ub610 \ube44\ub458\uae30\ub97c \ub0b4\uc5b4 \ub193\uc544 \uc9c0\uba74\uc5d0 \ubb3c\uc774 \uac10\ud55c \uc5ec\ubd80\ub97c \uc54c\uace0\uc790 \ud558\ub9e4 \uc628 \uc9c0\uba74\uc5d0 \ubb3c\uc774 \uc788\uc73c\ubbc0\ub85c \ube44\ub458\uae30\uac00 \uc811\uc871\ud560 \uacf3\uc744 \ucc3e\uc9c0 \ubabb\ud558\uace0 \ubc29\uc8fc\ub85c \ub3cc\uc544\uc640 \uadf8\uc5d0\uac8c\ub85c \uc624\ub294\uc9c0\ub77c. \u201d \uc774 \uc678\uc5d0\ub3c4 \ud070\uae4c\ub9c8\uadc0\ub294 \uc131\uacbd\uc5d0\uc11c \uc5ec\ub7ec \ucc28\ub840 \uc5b8\uae09\ub41c\ub2e4. \uc2e0\uc57d\uc5d0\uc11c \uc608\uc218\uac00 \uac00\ub85c\ub418 \ud070\uae4c\ub9c8\uadc0\ub97c \ube44\uc720\ub85c \ub4e4\uba74\uc11c \ubd80\uadc0\uc601\ud654\ub97c \ubc14\ub77c\uace0 \uc2e0\uc744 \ubbff\uc9c0 \ub9d0\ub77c\uace0 \ud55c\ub2e4(\ub204\uac00\ubcf5\uc74c 12\uc7a5 24\uc808). \u201c\uae4c\ub9c8\uadc0\ub97c \uc0dd\uac01\ud558\ub77c \uc2ec\uc9c0\ub3c4 \uc544\ub2c8\ud558\uace0 \uac70\ub450\uc9c0\ub3c4 \uc544\ub2c8\ud558\uba70 \uace8\ubc29\ub3c4 \uc5c6\uace0 \ucc3d\uace0\ub3c4 \uc5c6\uc73c\ub418 \ud558\ub098\ub2d8\uc774 \uae30\ub974\uc2dc\ub098\ub2c8 \ub108\ud76c\ub294 \uc0c8\ubcf4\ub2e4 \uc5bc\ub9c8\ub098 \ub354 \uadc0\ud558\ub0d0? \u201d \ucfe0\ub780\uc5d0\uc11c\ub294 \uce74\uc778\uacfc \uc544\ubca8 \uc774\uc57c\uae30\uc5d0\uc11c \ud070\uae4c\ub9c8\uadc0\uac00 \ub4f1\uc7a5\ud55c\ub2e4(\ucfe0\ub780 5\uc7a5 31\uc808). \uc544\ub2f4\uc758 \uc7a5\ub0a8 \uce74\uc778\uc774 \uc544\uc6b0 \uc544\ubca8\uc744 \uccd0 \uc8fd\uc600\ub294\ub370 \uadf8 \uc2dc\uccb4\ub97c \uc5b4\ucc0c \ucc98\ub9ac\ud574\uc57c \ud560\uc9c0 \ubab0\ub790\ub2e4. \uc774\ub54c \u201c\ud558\ub098\ub2d8\uc774 \ud070\uae4c\ub9c8\uadc0 \ud55c \ub9c8\ub9ac\ub97c \ubcf4\ub0b4\uc2dc\uc5b4 \ub545\ubc14\ub2e5\uc744 \uae01\uae30\ub85c \uadf8 \ud615\uc81c\uc758 \ubc8c\uac70\uc22d\uc774 \uc1a1\uc7a5\uc744 \uc228\uae38 \ubc95\ub3c4\ub97c \ubcf4\uc5ec\uc8fc\uc2dc\ub2c8 \uadf8\uac00 \uac00\ub85c\ub418, \ube44\ud1b5\uc774\ub85c\ub2e4! \ub0b4\uac00 \uc774 \uae4c\ub9c8\uadc0\ucc98\ub7fc \ud560 \uc904 \ubab0\ub77c \ud615\uc81c\uc758 \ubc8c\uac70\uc22d\uc774 \uc1a1\uc7a5\uc744 \uc228\uae30\uc9c0 \ubabb\ud558\uaca0\ub294\uac00? \uadf8\ub9ac\uace0 \uadf8\ub294 \ud68c\uac1c\ud558\uc600\ub2e4. \u201d", "paragraph_answer": "\uc720\ub300\uad50, \uae30\ub3c5\uad50, \uc774\uc2ac\ub78c\uad50\uc5d0\uc11c\ub294 \ud070\uae4c\ub9c8\uadc0 \uac00 \ub178\uc544\uc758 \ubc29\uc8fc\uc5d0\uc11c \ub9e8 \ucc98\uc74c \ub0a0\ub824\ubcf4\ub0b8 \ub3d9\ubb3c\ub85c \ub098\uc628\ub2e4(\ucc3d\uc138\uae30 8\uc7a5 6~8\uc808). \u201c\uc0ac\uc2ed \uc77c\uc744 \uc9c0\ub098\uc11c \ub178\uc544\uac00 \uadf8 \ubc29\uc8fc\uc5d0 \uc9c0\uc740 \ucc3d\uc744 \uc5f4\uace0 \uae4c\ub9c8\uadc0\ub97c \ub0b4\uc5b4 \ub193\uc73c\ub9e4 \uae4c\ub9c8\uadc0\uac00 \ubb3c\uc774 \ub545\uc5d0\uc11c \ub9c8\ub974\uae30\uae4c\uc9c0 \ub0a0\uc544 \uc655\ub798\ud558\uc600\ub354\ub77c \uadf8\uac00 \ub610 \ube44\ub458\uae30\ub97c \ub0b4\uc5b4 \ub193\uc544 \uc9c0\uba74\uc5d0 \ubb3c\uc774 \uac10\ud55c \uc5ec\ubd80\ub97c \uc54c\uace0\uc790 \ud558\ub9e4 \uc628 \uc9c0\uba74\uc5d0 \ubb3c\uc774 \uc788\uc73c\ubbc0\ub85c \ube44\ub458\uae30\uac00 \uc811\uc871\ud560 \uacf3\uc744 \ucc3e\uc9c0 \ubabb\ud558\uace0 \ubc29\uc8fc\ub85c \ub3cc\uc544\uc640 \uadf8\uc5d0\uac8c\ub85c \uc624\ub294\uc9c0\ub77c.\u201d \uc774 \uc678\uc5d0\ub3c4 \ud070\uae4c\ub9c8\uadc0\ub294 \uc131\uacbd\uc5d0\uc11c \uc5ec\ub7ec \ucc28\ub840 \uc5b8\uae09\ub41c\ub2e4. \uc2e0\uc57d\uc5d0\uc11c \uc608\uc218\uac00 \uac00\ub85c\ub418 \ud070\uae4c\ub9c8\uadc0\ub97c \ube44\uc720\ub85c \ub4e4\uba74\uc11c \ubd80\uadc0\uc601\ud654\ub97c \ubc14\ub77c\uace0 \uc2e0\uc744 \ubbff\uc9c0 \ub9d0\ub77c\uace0 \ud55c\ub2e4(\ub204\uac00\ubcf5\uc74c 12\uc7a5 24\uc808). \u201c\uae4c\ub9c8\uadc0\ub97c \uc0dd\uac01\ud558\ub77c \uc2ec\uc9c0\ub3c4 \uc544\ub2c8\ud558\uace0 \uac70\ub450\uc9c0\ub3c4 \uc544\ub2c8\ud558\uba70 \uace8\ubc29\ub3c4 \uc5c6\uace0 \ucc3d\uace0\ub3c4 \uc5c6\uc73c\ub418 \ud558\ub098\ub2d8\uc774 \uae30\ub974\uc2dc\ub098\ub2c8 \ub108\ud76c\ub294 \uc0c8\ubcf4\ub2e4 \uc5bc\ub9c8\ub098 \ub354 \uadc0\ud558\ub0d0?\u201d \ucfe0\ub780\uc5d0\uc11c\ub294 \uce74\uc778\uacfc \uc544\ubca8 \uc774\uc57c\uae30\uc5d0\uc11c \ud070\uae4c\ub9c8\uadc0\uac00 \ub4f1\uc7a5\ud55c\ub2e4(\ucfe0\ub780 5\uc7a5 31\uc808). \uc544\ub2f4\uc758 \uc7a5\ub0a8 \uce74\uc778\uc774 \uc544\uc6b0 \uc544\ubca8\uc744 \uccd0 \uc8fd\uc600\ub294\ub370 \uadf8 \uc2dc\uccb4\ub97c \uc5b4\ucc0c \ucc98\ub9ac\ud574\uc57c \ud560\uc9c0 \ubab0\ub790\ub2e4. \uc774\ub54c \u201c\ud558\ub098\ub2d8\uc774 \ud070\uae4c\ub9c8\uadc0 \ud55c \ub9c8\ub9ac\ub97c \ubcf4\ub0b4\uc2dc\uc5b4 \ub545\ubc14\ub2e5\uc744 \uae01\uae30\ub85c \uadf8 \ud615\uc81c\uc758 \ubc8c\uac70\uc22d\uc774 \uc1a1\uc7a5\uc744 \uc228\uae38 \ubc95\ub3c4\ub97c \ubcf4\uc5ec\uc8fc\uc2dc\ub2c8 \uadf8\uac00 \uac00\ub85c\ub418, \ube44\ud1b5\uc774\ub85c\ub2e4! \ub0b4\uac00 \uc774 \uae4c\ub9c8\uadc0\ucc98\ub7fc \ud560 \uc904 \ubab0\ub77c \ud615\uc81c\uc758 \ubc8c\uac70\uc22d\uc774 \uc1a1\uc7a5\uc744 \uc228\uae30\uc9c0 \ubabb\ud558\uaca0\ub294\uac00? \uadf8\ub9ac\uace0 \uadf8\ub294 \ud68c\uac1c\ud558\uc600\ub2e4.\u201d", "sentence_answer": "\uc720\ub300\uad50, \uae30\ub3c5\uad50, \uc774\uc2ac\ub78c\uad50\uc5d0\uc11c\ub294 \ud070\uae4c\ub9c8\uadc0 \uac00 \ub178\uc544\uc758 \ubc29\uc8fc\uc5d0\uc11c \ub9e8 \ucc98\uc74c \ub0a0\ub824\ubcf4\ub0b8 \ub3d9\ubb3c\ub85c \ub098\uc628\ub2e4(\ucc3d\uc138\uae30 8\uc7a5 6~8\uc808)", "paragraph_id": "6564339-13-0", "paragraph_question": "question: \ub178\uc544\uc758 \ubc29\uc8fc\uc5d0\uc11c \uc81c\uc77c \ucc98\uc74c \ub0a0\ub824\ubcf4\ub0b8 \ub3d9\ubb3c\uc740 \ubb34\uc5c7\uc778\uac00?, context: \uc720\ub300\uad50, \uae30\ub3c5\uad50, \uc774\uc2ac\ub78c\uad50\uc5d0\uc11c\ub294 \ud070\uae4c\ub9c8\uadc0\uac00 \ub178\uc544\uc758 \ubc29\uc8fc\uc5d0\uc11c \ub9e8 \ucc98\uc74c \ub0a0\ub824\ubcf4\ub0b8 \ub3d9\ubb3c\ub85c \ub098\uc628\ub2e4(\ucc3d\uc138\uae30 8\uc7a5 6~8\uc808). \u201c\uc0ac\uc2ed \uc77c\uc744 \uc9c0\ub098\uc11c \ub178\uc544\uac00 \uadf8 \ubc29\uc8fc\uc5d0 \uc9c0\uc740 \ucc3d\uc744 \uc5f4\uace0 \uae4c\ub9c8\uadc0\ub97c \ub0b4\uc5b4 \ub193\uc73c\ub9e4 \uae4c\ub9c8\uadc0\uac00 \ubb3c\uc774 \ub545\uc5d0\uc11c \ub9c8\ub974\uae30\uae4c\uc9c0 \ub0a0\uc544 \uc655\ub798\ud558\uc600\ub354\ub77c \uadf8\uac00 \ub610 \ube44\ub458\uae30\ub97c \ub0b4\uc5b4 \ub193\uc544 \uc9c0\uba74\uc5d0 \ubb3c\uc774 \uac10\ud55c \uc5ec\ubd80\ub97c \uc54c\uace0\uc790 \ud558\ub9e4 \uc628 \uc9c0\uba74\uc5d0 \ubb3c\uc774 \uc788\uc73c\ubbc0\ub85c \ube44\ub458\uae30\uac00 \uc811\uc871\ud560 \uacf3\uc744 \ucc3e\uc9c0 \ubabb\ud558\uace0 \ubc29\uc8fc\ub85c \ub3cc\uc544\uc640 \uadf8\uc5d0\uac8c\ub85c \uc624\ub294\uc9c0\ub77c.\u201d \uc774 \uc678\uc5d0\ub3c4 \ud070\uae4c\ub9c8\uadc0\ub294 \uc131\uacbd\uc5d0\uc11c \uc5ec\ub7ec \ucc28\ub840 \uc5b8\uae09\ub41c\ub2e4. \uc2e0\uc57d\uc5d0\uc11c \uc608\uc218\uac00 \uac00\ub85c\ub418 \ud070\uae4c\ub9c8\uadc0\ub97c \ube44\uc720\ub85c \ub4e4\uba74\uc11c \ubd80\uadc0\uc601\ud654\ub97c \ubc14\ub77c\uace0 \uc2e0\uc744 \ubbff\uc9c0 \ub9d0\ub77c\uace0 \ud55c\ub2e4(\ub204\uac00\ubcf5\uc74c 12\uc7a5 24\uc808). \u201c\uae4c\ub9c8\uadc0\ub97c \uc0dd\uac01\ud558\ub77c \uc2ec\uc9c0\ub3c4 \uc544\ub2c8\ud558\uace0 \uac70\ub450\uc9c0\ub3c4 \uc544\ub2c8\ud558\uba70 \uace8\ubc29\ub3c4 \uc5c6\uace0 \ucc3d\uace0\ub3c4 \uc5c6\uc73c\ub418 \ud558\ub098\ub2d8\uc774 \uae30\ub974\uc2dc\ub098\ub2c8 \ub108\ud76c\ub294 \uc0c8\ubcf4\ub2e4 \uc5bc\ub9c8\ub098 \ub354 \uadc0\ud558\ub0d0?\u201d \ucfe0\ub780\uc5d0\uc11c\ub294 \uce74\uc778\uacfc \uc544\ubca8 \uc774\uc57c\uae30\uc5d0\uc11c \ud070\uae4c\ub9c8\uadc0\uac00 \ub4f1\uc7a5\ud55c\ub2e4(\ucfe0\ub780 5\uc7a5 31\uc808). \uc544\ub2f4\uc758 \uc7a5\ub0a8 \uce74\uc778\uc774 \uc544\uc6b0 \uc544\ubca8\uc744 \uccd0 \uc8fd\uc600\ub294\ub370 \uadf8 \uc2dc\uccb4\ub97c \uc5b4\ucc0c \ucc98\ub9ac\ud574\uc57c \ud560\uc9c0 \ubab0\ub790\ub2e4. \uc774\ub54c \u201c\ud558\ub098\ub2d8\uc774 \ud070\uae4c\ub9c8\uadc0 \ud55c \ub9c8\ub9ac\ub97c \ubcf4\ub0b4\uc2dc\uc5b4 \ub545\ubc14\ub2e5\uc744 \uae01\uae30\ub85c \uadf8 \ud615\uc81c\uc758 \ubc8c\uac70\uc22d\uc774 \uc1a1\uc7a5\uc744 \uc228\uae38 \ubc95\ub3c4\ub97c \ubcf4\uc5ec\uc8fc\uc2dc\ub2c8 \uadf8\uac00 \uac00\ub85c\ub418, \ube44\ud1b5\uc774\ub85c\ub2e4! \ub0b4\uac00 \uc774 \uae4c\ub9c8\uadc0\ucc98\ub7fc \ud560 \uc904 \ubab0\ub77c \ud615\uc81c\uc758 \ubc8c\uac70\uc22d\uc774 \uc1a1\uc7a5\uc744 \uc228\uae30\uc9c0 \ubabb\ud558\uaca0\ub294\uac00? \uadf8\ub9ac\uace0 \uadf8\ub294 \ud68c\uac1c\ud558\uc600\ub2e4.\u201d"} {"question": "\ucd98\ud5a5\uc804\uc758 \uc5b4\uc0ac \ucd9c\ub3c4 \uc7a5\uba74\uc5d0\uc11c \uc11c\uc0ac\uc801\uc778 \uc7a5\uba74\ubb18\uc0ac\ub97c \ube80 \ud6c4\uc5d0 \uc8fc\ub85c \ubb34\uc5c7\uc73c\ub85c\ub9cc \uad6c\uc131 \ub418\ub294\uac00?", "paragraph": "\uc6b0\uc120 \ucc3d\uadf9\uc740 \uadf9\uc911\uc778\ubb3c\uc758 \ucc3d\uacfc \ub300\uc0ac, \uadf8\ub9ac\uace0 \ubb34\ub300 \ubc16\uc5d0\uc11c\uc758 \ub3c4\ucc3d(\u5c0e\u5531)\uc73c\ub85c \uc5ee\uc5b4\uac00\ubbc0\ub85c \uc774 \uc138 \uac00\uc9c0\ub97c \uc801\ub2f9\ud788 \ubc30\ud569\ud558\uc5ec\uc57c \ud55c\ub2e4. <\ucd98\ud5a5\uc804>\uc758 \uacbd\uc6b0, \ud754\ud788 \uad11\ud55c\ub8e8 \uc7a5\uba74\uacfc \uc5b4\uc0ac \ucd9c\ub3c4 \uc7a5\uba74\uc740 \ub9ce\uc740 \ub300\uc870\uac00 \ub41c\ub2e4. \uc804\uc790\uc5d0\uc11c\ub294 \ubcf8\ub798 \ud310\uc18c\ub9ac\uc758 \uc774 \ub300\ubaa9\uc740 \ubc29\uc790\uc640 \uc774\ub3c4\ub839\uc758 \ub300\ud654\uac00 \ucc3d\uc73c\ub85c \uc544\uae30\uc790\uae30\ud558\uac8c \uc5ee\uc5b4\uc838 \uc788\uc73c\ubbc0\ub85c \uadf8\uac83\uc744 \uadf8\ub300\ub85c \uc62e\uaca8 \uc500\uc73c\ub85c\uc368 \ucc3d\uc774 \uc8fc\uac00 \ub418\uac8c \uad6c\uc131\ud558\ub294 \uc77c\uc774 \ub9ce\uc740 \ubc18\uba74, \uc5b4\uc0ac \ucd9c\ub3c4 \uc7a5\uba74\uc5d0\uc11c\ub294 \ubcf8\ub798 \ud310\uc18c\ub9ac\uc758 \uc774 \ub300\ubaa9\uc5d0\uc11c \uc790\uc9c4\ubaa8\ub9ac \uc7a5\ub2e8\uc758 \uc11c\uc0ac\uc801\uc778 \uc7a5\uba74\ubb18\uc0ac\ub97c \ube7c\uba74 \uc544\ub2c8\ub9ac\ub85c\ub9cc \uad6c\uc131\ub418\ubbc0\ub85c, \uc774\uac83\uc744 \ucc3d\uadf9\uc73c\ub85c \ubb34\ub300\ud654\ud588\uc744 \uacbd\uc6b0 \uc790\uc9c4\ubaa8\ub9ac\ub85c \ubd80\ub974\ub294 \ub300\ubaa9\uc740 \uadf9\uc911 \uc778\ubb3c\uc758 \uc5f0\uae30\ub85c \ub300\uccb4\ub418\uace0 \uc544\ub2c8\ub9ac \ubd80\ubd84\uc740 \ubaa8\ub450 \ub300\uc0ac\ub85c \uc5ee\uc5b4\uac00\ubbc0\ub85c \ucc3d\uc774 \uacb0\ud54d\ub41c\ub2e4. \ub530\ub77c\uc11c \uad11\ud55c\ub8e8 \uc7a5\uba74\uacfc \uc5b4\uc0ac\ucd9c\ub3c4 \uc7a5\uba74\uc740 \uc74c\uc545\uc801 \uade0\ud615\uc774 \uae68\uc9c0\uace0 \uc788\ub2e4.", "answer": "\uc544\ub2c8\ub9ac", "sentence": "\uc804\uc790\uc5d0\uc11c\ub294 \ubcf8\ub798 \ud310\uc18c\ub9ac\uc758 \uc774 \ub300\ubaa9\uc740 \ubc29\uc790\uc640 \uc774\ub3c4\ub839\uc758 \ub300\ud654\uac00 \ucc3d\uc73c\ub85c \uc544\uae30\uc790\uae30\ud558\uac8c \uc5ee\uc5b4\uc838 \uc788\uc73c\ubbc0\ub85c \uadf8\uac83\uc744 \uadf8\ub300\ub85c \uc62e\uaca8 \uc500\uc73c\ub85c\uc368 \ucc3d\uc774 \uc8fc\uac00 \ub418\uac8c \uad6c\uc131\ud558\ub294 \uc77c\uc774 \ub9ce\uc740 \ubc18\uba74, \uc5b4\uc0ac \ucd9c\ub3c4 \uc7a5\uba74\uc5d0\uc11c\ub294 \ubcf8\ub798 \ud310\uc18c\ub9ac\uc758 \uc774 \ub300\ubaa9\uc5d0\uc11c \uc790\uc9c4\ubaa8\ub9ac \uc7a5\ub2e8\uc758 \uc11c\uc0ac\uc801\uc778 \uc7a5\uba74\ubb18\uc0ac\ub97c \ube7c\uba74 \uc544\ub2c8\ub9ac \ub85c\ub9cc \uad6c\uc131\ub418\ubbc0\ub85c, \uc774\uac83\uc744 \ucc3d\uadf9\uc73c\ub85c \ubb34\ub300\ud654\ud588\uc744 \uacbd\uc6b0 \uc790\uc9c4\ubaa8\ub9ac\ub85c \ubd80\ub974\ub294 \ub300\ubaa9\uc740 \uadf9\uc911 \uc778\ubb3c\uc758 \uc5f0\uae30\ub85c \ub300\uccb4\ub418\uace0 \uc544\ub2c8\ub9ac \ubd80\ubd84\uc740 \ubaa8\ub450 \ub300\uc0ac\ub85c \uc5ee\uc5b4\uac00\ubbc0\ub85c \ucc3d\uc774 \uacb0\ud54d\ub41c\ub2e4.", "paragraph_sentence": "\uc6b0\uc120 \ucc3d\uadf9\uc740 \uadf9\uc911\uc778\ubb3c\uc758 \ucc3d\uacfc \ub300\uc0ac, \uadf8\ub9ac\uace0 \ubb34\ub300 \ubc16\uc5d0\uc11c\uc758 \ub3c4\ucc3d(\u5c0e\u5531)\uc73c\ub85c \uc5ee\uc5b4\uac00\ubbc0\ub85c \uc774 \uc138 \uac00\uc9c0\ub97c \uc801\ub2f9\ud788 \ubc30\ud569\ud558\uc5ec\uc57c \ud55c\ub2e4. <\ucd98\ud5a5\uc804>\uc758 \uacbd\uc6b0, \ud754\ud788 \uad11\ud55c\ub8e8 \uc7a5\uba74\uacfc \uc5b4\uc0ac \ucd9c\ub3c4 \uc7a5\uba74\uc740 \ub9ce\uc740 \ub300\uc870\uac00 \ub41c\ub2e4. \uc804\uc790\uc5d0\uc11c\ub294 \ubcf8\ub798 \ud310\uc18c\ub9ac\uc758 \uc774 \ub300\ubaa9\uc740 \ubc29\uc790\uc640 \uc774\ub3c4\ub839\uc758 \ub300\ud654\uac00 \ucc3d\uc73c\ub85c \uc544\uae30\uc790\uae30\ud558\uac8c \uc5ee\uc5b4\uc838 \uc788\uc73c\ubbc0\ub85c \uadf8\uac83\uc744 \uadf8\ub300\ub85c \uc62e\uaca8 \uc500\uc73c\ub85c\uc368 \ucc3d\uc774 \uc8fc\uac00 \ub418\uac8c \uad6c\uc131\ud558\ub294 \uc77c\uc774 \ub9ce\uc740 \ubc18\uba74, \uc5b4\uc0ac \ucd9c\ub3c4 \uc7a5\uba74\uc5d0\uc11c\ub294 \ubcf8\ub798 \ud310\uc18c\ub9ac\uc758 \uc774 \ub300\ubaa9\uc5d0\uc11c \uc790\uc9c4\ubaa8\ub9ac \uc7a5\ub2e8\uc758 \uc11c\uc0ac\uc801\uc778 \uc7a5\uba74\ubb18\uc0ac\ub97c \ube7c\uba74 \uc544\ub2c8\ub9ac \ub85c\ub9cc \uad6c\uc131\ub418\ubbc0\ub85c, \uc774\uac83\uc744 \ucc3d\uadf9\uc73c\ub85c \ubb34\ub300\ud654\ud588\uc744 \uacbd\uc6b0 \uc790\uc9c4\ubaa8\ub9ac\ub85c \ubd80\ub974\ub294 \ub300\ubaa9\uc740 \uadf9\uc911 \uc778\ubb3c\uc758 \uc5f0\uae30\ub85c \ub300\uccb4\ub418\uace0 \uc544\ub2c8\ub9ac \ubd80\ubd84\uc740 \ubaa8\ub450 \ub300\uc0ac\ub85c \uc5ee\uc5b4\uac00\ubbc0\ub85c \ucc3d\uc774 \uacb0\ud54d\ub41c\ub2e4. \ub530\ub77c\uc11c \uad11\ud55c\ub8e8 \uc7a5\uba74\uacfc \uc5b4\uc0ac\ucd9c\ub3c4 \uc7a5\uba74\uc740 \uc74c\uc545\uc801 \uade0\ud615\uc774 \uae68\uc9c0\uace0 \uc788\ub2e4.", "paragraph_answer": "\uc6b0\uc120 \ucc3d\uadf9\uc740 \uadf9\uc911\uc778\ubb3c\uc758 \ucc3d\uacfc \ub300\uc0ac, \uadf8\ub9ac\uace0 \ubb34\ub300 \ubc16\uc5d0\uc11c\uc758 \ub3c4\ucc3d(\u5c0e\u5531)\uc73c\ub85c \uc5ee\uc5b4\uac00\ubbc0\ub85c \uc774 \uc138 \uac00\uc9c0\ub97c \uc801\ub2f9\ud788 \ubc30\ud569\ud558\uc5ec\uc57c \ud55c\ub2e4. <\ucd98\ud5a5\uc804>\uc758 \uacbd\uc6b0, \ud754\ud788 \uad11\ud55c\ub8e8 \uc7a5\uba74\uacfc \uc5b4\uc0ac \ucd9c\ub3c4 \uc7a5\uba74\uc740 \ub9ce\uc740 \ub300\uc870\uac00 \ub41c\ub2e4. \uc804\uc790\uc5d0\uc11c\ub294 \ubcf8\ub798 \ud310\uc18c\ub9ac\uc758 \uc774 \ub300\ubaa9\uc740 \ubc29\uc790\uc640 \uc774\ub3c4\ub839\uc758 \ub300\ud654\uac00 \ucc3d\uc73c\ub85c \uc544\uae30\uc790\uae30\ud558\uac8c \uc5ee\uc5b4\uc838 \uc788\uc73c\ubbc0\ub85c \uadf8\uac83\uc744 \uadf8\ub300\ub85c \uc62e\uaca8 \uc500\uc73c\ub85c\uc368 \ucc3d\uc774 \uc8fc\uac00 \ub418\uac8c \uad6c\uc131\ud558\ub294 \uc77c\uc774 \ub9ce\uc740 \ubc18\uba74, \uc5b4\uc0ac \ucd9c\ub3c4 \uc7a5\uba74\uc5d0\uc11c\ub294 \ubcf8\ub798 \ud310\uc18c\ub9ac\uc758 \uc774 \ub300\ubaa9\uc5d0\uc11c \uc790\uc9c4\ubaa8\ub9ac \uc7a5\ub2e8\uc758 \uc11c\uc0ac\uc801\uc778 \uc7a5\uba74\ubb18\uc0ac\ub97c \ube7c\uba74 \uc544\ub2c8\ub9ac \ub85c\ub9cc \uad6c\uc131\ub418\ubbc0\ub85c, \uc774\uac83\uc744 \ucc3d\uadf9\uc73c\ub85c \ubb34\ub300\ud654\ud588\uc744 \uacbd\uc6b0 \uc790\uc9c4\ubaa8\ub9ac\ub85c \ubd80\ub974\ub294 \ub300\ubaa9\uc740 \uadf9\uc911 \uc778\ubb3c\uc758 \uc5f0\uae30\ub85c \ub300\uccb4\ub418\uace0 \uc544\ub2c8\ub9ac \ubd80\ubd84\uc740 \ubaa8\ub450 \ub300\uc0ac\ub85c \uc5ee\uc5b4\uac00\ubbc0\ub85c \ucc3d\uc774 \uacb0\ud54d\ub41c\ub2e4. \ub530\ub77c\uc11c \uad11\ud55c\ub8e8 \uc7a5\uba74\uacfc \uc5b4\uc0ac\ucd9c\ub3c4 \uc7a5\uba74\uc740 \uc74c\uc545\uc801 \uade0\ud615\uc774 \uae68\uc9c0\uace0 \uc788\ub2e4.", "sentence_answer": "\uc804\uc790\uc5d0\uc11c\ub294 \ubcf8\ub798 \ud310\uc18c\ub9ac\uc758 \uc774 \ub300\ubaa9\uc740 \ubc29\uc790\uc640 \uc774\ub3c4\ub839\uc758 \ub300\ud654\uac00 \ucc3d\uc73c\ub85c \uc544\uae30\uc790\uae30\ud558\uac8c \uc5ee\uc5b4\uc838 \uc788\uc73c\ubbc0\ub85c \uadf8\uac83\uc744 \uadf8\ub300\ub85c \uc62e\uaca8 \uc500\uc73c\ub85c\uc368 \ucc3d\uc774 \uc8fc\uac00 \ub418\uac8c \uad6c\uc131\ud558\ub294 \uc77c\uc774 \ub9ce\uc740 \ubc18\uba74, \uc5b4\uc0ac \ucd9c\ub3c4 \uc7a5\uba74\uc5d0\uc11c\ub294 \ubcf8\ub798 \ud310\uc18c\ub9ac\uc758 \uc774 \ub300\ubaa9\uc5d0\uc11c \uc790\uc9c4\ubaa8\ub9ac \uc7a5\ub2e8\uc758 \uc11c\uc0ac\uc801\uc778 \uc7a5\uba74\ubb18\uc0ac\ub97c \ube7c\uba74 \uc544\ub2c8\ub9ac \ub85c\ub9cc \uad6c\uc131\ub418\ubbc0\ub85c, \uc774\uac83\uc744 \ucc3d\uadf9\uc73c\ub85c \ubb34\ub300\ud654\ud588\uc744 \uacbd\uc6b0 \uc790\uc9c4\ubaa8\ub9ac\ub85c \ubd80\ub974\ub294 \ub300\ubaa9\uc740 \uadf9\uc911 \uc778\ubb3c\uc758 \uc5f0\uae30\ub85c \ub300\uccb4\ub418\uace0 \uc544\ub2c8\ub9ac \ubd80\ubd84\uc740 \ubaa8\ub450 \ub300\uc0ac\ub85c \uc5ee\uc5b4\uac00\ubbc0\ub85c \ucc3d\uc774 \uacb0\ud54d\ub41c\ub2e4.", "paragraph_id": "6557859-5-2", "paragraph_question": "question: \ucd98\ud5a5\uc804\uc758 \uc5b4\uc0ac \ucd9c\ub3c4 \uc7a5\uba74\uc5d0\uc11c \uc11c\uc0ac\uc801\uc778 \uc7a5\uba74\ubb18\uc0ac\ub97c \ube80 \ud6c4\uc5d0 \uc8fc\ub85c \ubb34\uc5c7\uc73c\ub85c\ub9cc \uad6c\uc131 \ub418\ub294\uac00?, context: \uc6b0\uc120 \ucc3d\uadf9\uc740 \uadf9\uc911\uc778\ubb3c\uc758 \ucc3d\uacfc \ub300\uc0ac, \uadf8\ub9ac\uace0 \ubb34\ub300 \ubc16\uc5d0\uc11c\uc758 \ub3c4\ucc3d(\u5c0e\u5531)\uc73c\ub85c \uc5ee\uc5b4\uac00\ubbc0\ub85c \uc774 \uc138 \uac00\uc9c0\ub97c \uc801\ub2f9\ud788 \ubc30\ud569\ud558\uc5ec\uc57c \ud55c\ub2e4. <\ucd98\ud5a5\uc804>\uc758 \uacbd\uc6b0, \ud754\ud788 \uad11\ud55c\ub8e8 \uc7a5\uba74\uacfc \uc5b4\uc0ac \ucd9c\ub3c4 \uc7a5\uba74\uc740 \ub9ce\uc740 \ub300\uc870\uac00 \ub41c\ub2e4. \uc804\uc790\uc5d0\uc11c\ub294 \ubcf8\ub798 \ud310\uc18c\ub9ac\uc758 \uc774 \ub300\ubaa9\uc740 \ubc29\uc790\uc640 \uc774\ub3c4\ub839\uc758 \ub300\ud654\uac00 \ucc3d\uc73c\ub85c \uc544\uae30\uc790\uae30\ud558\uac8c \uc5ee\uc5b4\uc838 \uc788\uc73c\ubbc0\ub85c \uadf8\uac83\uc744 \uadf8\ub300\ub85c \uc62e\uaca8 \uc500\uc73c\ub85c\uc368 \ucc3d\uc774 \uc8fc\uac00 \ub418\uac8c \uad6c\uc131\ud558\ub294 \uc77c\uc774 \ub9ce\uc740 \ubc18\uba74, \uc5b4\uc0ac \ucd9c\ub3c4 \uc7a5\uba74\uc5d0\uc11c\ub294 \ubcf8\ub798 \ud310\uc18c\ub9ac\uc758 \uc774 \ub300\ubaa9\uc5d0\uc11c \uc790\uc9c4\ubaa8\ub9ac \uc7a5\ub2e8\uc758 \uc11c\uc0ac\uc801\uc778 \uc7a5\uba74\ubb18\uc0ac\ub97c \ube7c\uba74 \uc544\ub2c8\ub9ac\ub85c\ub9cc \uad6c\uc131\ub418\ubbc0\ub85c, \uc774\uac83\uc744 \ucc3d\uadf9\uc73c\ub85c \ubb34\ub300\ud654\ud588\uc744 \uacbd\uc6b0 \uc790\uc9c4\ubaa8\ub9ac\ub85c \ubd80\ub974\ub294 \ub300\ubaa9\uc740 \uadf9\uc911 \uc778\ubb3c\uc758 \uc5f0\uae30\ub85c \ub300\uccb4\ub418\uace0 \uc544\ub2c8\ub9ac \ubd80\ubd84\uc740 \ubaa8\ub450 \ub300\uc0ac\ub85c \uc5ee\uc5b4\uac00\ubbc0\ub85c \ucc3d\uc774 \uacb0\ud54d\ub41c\ub2e4. \ub530\ub77c\uc11c \uad11\ud55c\ub8e8 \uc7a5\uba74\uacfc \uc5b4\uc0ac\ucd9c\ub3c4 \uc7a5\uba74\uc740 \uc74c\uc545\uc801 \uade0\ud615\uc774 \uae68\uc9c0\uace0 \uc788\ub2e4."} {"question": "HP\ub294 \ubb34\uc5c7\uc744 \ub73b\ud569\ub2c8\uae4c?", "paragraph": "\ud50c\ub808\uc774\uc5b4\uac00 \uc57c\uc0dd \ud3ec\ucf13\ubaac\uacfc \uc870\uc6b0\ud558\uac70\ub098 \ub2e4\ub978 \ud2b8\ub808\uc774\ub108\uc5d0\uac8c \ub3c4\uc804\uc744 \ubc1b\uc73c\uba74 \ud654\uba74\uc774 \ud3ec\ucf13\ubaac \ub300\uc804\uc744 \ud558\ub294 \ud134\uc81c \ubc30\ud2c0 \ud654\uba74\uc73c\ub85c \uc804\ud658\ub41c\ub2e4. \ud50c\ub808\uc774\uc5b4\ub294 \ubc30\ud2c0\uc744 \ud558\ub294 \ub3d9\uc548\uc5d0 \uc2f8\uc6b0\uac70\ub098, \uc544\uc774\ud15c\uc744 \uc0ac\uc6a9\ud558\uac70\ub098, \uc2f8\uc6b0\ub294 \ud3ec\ucf13\ubaac\uc744 \ubc14\uafb8\uac70\ub098, \ub3c4\ub9dd\uc744 \uce60 \uc218 \uc788\ub2e4. (\ub2e4\ub978 \ud2b8\ub808\uc774\ub108\uc640\uc758 \ubc30\ud2c0\uc5d0\uc11c\ub294 \ub3c4\ub9dd\uce60 \uc218 \uc5c6\ub2e4.) \ud3ec\ucf13\ubaac\uc740 \uc77c\uc815\ud55c \uccb4\ub825 (HP)\uc774 \uc788\uc73c\uba70, \uc804\ud22c\uac00 \uc9c4\ud589\ub418\ub294 \ub3d9\uc548 \ud654\uba74\uc5d0 \ud45c\uc2dc\ub41c\ub2e4. HP\uac00 0\uc774 \ub418\uba74 \ud3ec\ucf13\ubaac\uc740 \uc4f0\ub7ec\uc9c0\uba70, \ud3ec\ucf13\ubaac \uc13c\ud130\uc5d0 \uac00\uac70\ub098, \ud3ec\ucf13\ubaac\uc758 \uae30\uc220\uc744 \uc4f0\uac70\ub098, \uc544\uc774\ud15c\uc744 \uc0ac\uc6a9\ud558\uc9c0 \uc54a\ub294 \uc774\uc0c1 \ubc30\ud2c0\uc744 \ud560 \uc218 \uc5c6\uac8c \ub41c\ub2e4. \ud50c\ub808\uc774\uc5b4\uc758 \ud3ec\ucf13\ubaac\uc774 \uc0c1\ub300 \ud3ec\ucf13\ubaac\uc744 \uc4f0\ub7ec\ub728\ub824\uc11c \uc774\uae30\uba74 \uacbd\ud5d8\uce58\ub97c \uc5bb\uac8c \ub41c\ub2e4. \uacbd\ud5d8\uce58\ub97c \ucda9\ubd84\ud788 \ubaa8\uc73c\uba74 \ub808\ubca8\uc774 \uc624\ub974\uba70, \ub300\ubd80\ubd84\uc758 \ud3ec\ucf13\ubaac\uc740 \uc77c\uc815\ud55c \ub808\ubca8\uc5d0 \ub2e4\ub2e4\ub974\uac70\ub098 \uc77c\uc815\ud55c \uc870\uac74 (\ud2b8\ub808\uc774\ub108\uc640\uc758 \uce5c\ubc00\ub3c4, \uc18c\uc9c0 \uc544\uc774\ud15c \ub4f1)\uc744 \ub9cc\uc871\ud558\uba74 \uc0c8\ub85c\uc6b4 \uc885\uc73c\ub85c \uc9c4\ud654\ud558\uac8c \ub41c\ub2e4.", "answer": "\uccb4\ub825", "sentence": "\ud3ec\ucf13\ubaac\uc740 \uc77c\uc815\ud55c \uccb4\ub825 (HP)\uc774 \uc788\uc73c\uba70, \uc804\ud22c\uac00 \uc9c4\ud589\ub418\ub294 \ub3d9\uc548 \ud654\uba74\uc5d0 \ud45c\uc2dc\ub41c\ub2e4.", "paragraph_sentence": "\ud50c\ub808\uc774\uc5b4\uac00 \uc57c\uc0dd \ud3ec\ucf13\ubaac\uacfc \uc870\uc6b0\ud558\uac70\ub098 \ub2e4\ub978 \ud2b8\ub808\uc774\ub108\uc5d0\uac8c \ub3c4\uc804\uc744 \ubc1b\uc73c\uba74 \ud654\uba74\uc774 \ud3ec\ucf13\ubaac \ub300\uc804\uc744 \ud558\ub294 \ud134\uc81c \ubc30\ud2c0 \ud654\uba74\uc73c\ub85c \uc804\ud658\ub41c\ub2e4. \ud50c\ub808\uc774\uc5b4\ub294 \ubc30\ud2c0\uc744 \ud558\ub294 \ub3d9\uc548\uc5d0 \uc2f8\uc6b0\uac70\ub098, \uc544\uc774\ud15c\uc744 \uc0ac\uc6a9\ud558\uac70\ub098, \uc2f8\uc6b0\ub294 \ud3ec\ucf13\ubaac\uc744 \ubc14\uafb8\uac70\ub098, \ub3c4\ub9dd\uc744 \uce60 \uc218 \uc788\ub2e4. (\ub2e4\ub978 \ud2b8\ub808\uc774\ub108\uc640\uc758 \ubc30\ud2c0\uc5d0\uc11c\ub294 \ub3c4\ub9dd\uce60 \uc218 \uc5c6\ub2e4. ) \ud3ec\ucf13\ubaac\uc740 \uc77c\uc815\ud55c \uccb4\ub825 (HP)\uc774 \uc788\uc73c\uba70, \uc804\ud22c\uac00 \uc9c4\ud589\ub418\ub294 \ub3d9\uc548 \ud654\uba74\uc5d0 \ud45c\uc2dc\ub41c\ub2e4. HP\uac00 0\uc774 \ub418\uba74 \ud3ec\ucf13\ubaac\uc740 \uc4f0\ub7ec\uc9c0\uba70, \ud3ec\ucf13\ubaac \uc13c\ud130\uc5d0 \uac00\uac70\ub098, \ud3ec\ucf13\ubaac\uc758 \uae30\uc220\uc744 \uc4f0\uac70\ub098, \uc544\uc774\ud15c\uc744 \uc0ac\uc6a9\ud558\uc9c0 \uc54a\ub294 \uc774\uc0c1 \ubc30\ud2c0\uc744 \ud560 \uc218 \uc5c6\uac8c \ub41c\ub2e4. \ud50c\ub808\uc774\uc5b4\uc758 \ud3ec\ucf13\ubaac\uc774 \uc0c1\ub300 \ud3ec\ucf13\ubaac\uc744 \uc4f0\ub7ec\ub728\ub824\uc11c \uc774\uae30\uba74 \uacbd\ud5d8\uce58\ub97c \uc5bb\uac8c \ub41c\ub2e4. \uacbd\ud5d8\uce58\ub97c \ucda9\ubd84\ud788 \ubaa8\uc73c\uba74 \ub808\ubca8\uc774 \uc624\ub974\uba70, \ub300\ubd80\ubd84\uc758 \ud3ec\ucf13\ubaac\uc740 \uc77c\uc815\ud55c \ub808\ubca8\uc5d0 \ub2e4\ub2e4\ub974\uac70\ub098 \uc77c\uc815\ud55c \uc870\uac74 (\ud2b8\ub808\uc774\ub108\uc640\uc758 \uce5c\ubc00\ub3c4, \uc18c\uc9c0 \uc544\uc774\ud15c \ub4f1)\uc744 \ub9cc\uc871\ud558\uba74 \uc0c8\ub85c\uc6b4 \uc885\uc73c\ub85c \uc9c4\ud654\ud558\uac8c \ub41c\ub2e4.", "paragraph_answer": "\ud50c\ub808\uc774\uc5b4\uac00 \uc57c\uc0dd \ud3ec\ucf13\ubaac\uacfc \uc870\uc6b0\ud558\uac70\ub098 \ub2e4\ub978 \ud2b8\ub808\uc774\ub108\uc5d0\uac8c \ub3c4\uc804\uc744 \ubc1b\uc73c\uba74 \ud654\uba74\uc774 \ud3ec\ucf13\ubaac \ub300\uc804\uc744 \ud558\ub294 \ud134\uc81c \ubc30\ud2c0 \ud654\uba74\uc73c\ub85c \uc804\ud658\ub41c\ub2e4. \ud50c\ub808\uc774\uc5b4\ub294 \ubc30\ud2c0\uc744 \ud558\ub294 \ub3d9\uc548\uc5d0 \uc2f8\uc6b0\uac70\ub098, \uc544\uc774\ud15c\uc744 \uc0ac\uc6a9\ud558\uac70\ub098, \uc2f8\uc6b0\ub294 \ud3ec\ucf13\ubaac\uc744 \ubc14\uafb8\uac70\ub098, \ub3c4\ub9dd\uc744 \uce60 \uc218 \uc788\ub2e4. (\ub2e4\ub978 \ud2b8\ub808\uc774\ub108\uc640\uc758 \ubc30\ud2c0\uc5d0\uc11c\ub294 \ub3c4\ub9dd\uce60 \uc218 \uc5c6\ub2e4.) \ud3ec\ucf13\ubaac\uc740 \uc77c\uc815\ud55c \uccb4\ub825 (HP)\uc774 \uc788\uc73c\uba70, \uc804\ud22c\uac00 \uc9c4\ud589\ub418\ub294 \ub3d9\uc548 \ud654\uba74\uc5d0 \ud45c\uc2dc\ub41c\ub2e4. HP\uac00 0\uc774 \ub418\uba74 \ud3ec\ucf13\ubaac\uc740 \uc4f0\ub7ec\uc9c0\uba70, \ud3ec\ucf13\ubaac \uc13c\ud130\uc5d0 \uac00\uac70\ub098, \ud3ec\ucf13\ubaac\uc758 \uae30\uc220\uc744 \uc4f0\uac70\ub098, \uc544\uc774\ud15c\uc744 \uc0ac\uc6a9\ud558\uc9c0 \uc54a\ub294 \uc774\uc0c1 \ubc30\ud2c0\uc744 \ud560 \uc218 \uc5c6\uac8c \ub41c\ub2e4. \ud50c\ub808\uc774\uc5b4\uc758 \ud3ec\ucf13\ubaac\uc774 \uc0c1\ub300 \ud3ec\ucf13\ubaac\uc744 \uc4f0\ub7ec\ub728\ub824\uc11c \uc774\uae30\uba74 \uacbd\ud5d8\uce58\ub97c \uc5bb\uac8c \ub41c\ub2e4. \uacbd\ud5d8\uce58\ub97c \ucda9\ubd84\ud788 \ubaa8\uc73c\uba74 \ub808\ubca8\uc774 \uc624\ub974\uba70, \ub300\ubd80\ubd84\uc758 \ud3ec\ucf13\ubaac\uc740 \uc77c\uc815\ud55c \ub808\ubca8\uc5d0 \ub2e4\ub2e4\ub974\uac70\ub098 \uc77c\uc815\ud55c \uc870\uac74 (\ud2b8\ub808\uc774\ub108\uc640\uc758 \uce5c\ubc00\ub3c4, \uc18c\uc9c0 \uc544\uc774\ud15c \ub4f1)\uc744 \ub9cc\uc871\ud558\uba74 \uc0c8\ub85c\uc6b4 \uc885\uc73c\ub85c \uc9c4\ud654\ud558\uac8c \ub41c\ub2e4.", "sentence_answer": "\ud3ec\ucf13\ubaac\uc740 \uc77c\uc815\ud55c \uccb4\ub825 (HP)\uc774 \uc788\uc73c\uba70, \uc804\ud22c\uac00 \uc9c4\ud589\ub418\ub294 \ub3d9\uc548 \ud654\uba74\uc5d0 \ud45c\uc2dc\ub41c\ub2e4.", "paragraph_id": "6220514-0-0", "paragraph_question": "question: HP\ub294 \ubb34\uc5c7\uc744 \ub73b\ud569\ub2c8\uae4c?, context: \ud50c\ub808\uc774\uc5b4\uac00 \uc57c\uc0dd \ud3ec\ucf13\ubaac\uacfc \uc870\uc6b0\ud558\uac70\ub098 \ub2e4\ub978 \ud2b8\ub808\uc774\ub108\uc5d0\uac8c \ub3c4\uc804\uc744 \ubc1b\uc73c\uba74 \ud654\uba74\uc774 \ud3ec\ucf13\ubaac \ub300\uc804\uc744 \ud558\ub294 \ud134\uc81c \ubc30\ud2c0 \ud654\uba74\uc73c\ub85c \uc804\ud658\ub41c\ub2e4. \ud50c\ub808\uc774\uc5b4\ub294 \ubc30\ud2c0\uc744 \ud558\ub294 \ub3d9\uc548\uc5d0 \uc2f8\uc6b0\uac70\ub098, \uc544\uc774\ud15c\uc744 \uc0ac\uc6a9\ud558\uac70\ub098, \uc2f8\uc6b0\ub294 \ud3ec\ucf13\ubaac\uc744 \ubc14\uafb8\uac70\ub098, \ub3c4\ub9dd\uc744 \uce60 \uc218 \uc788\ub2e4. (\ub2e4\ub978 \ud2b8\ub808\uc774\ub108\uc640\uc758 \ubc30\ud2c0\uc5d0\uc11c\ub294 \ub3c4\ub9dd\uce60 \uc218 \uc5c6\ub2e4.) \ud3ec\ucf13\ubaac\uc740 \uc77c\uc815\ud55c \uccb4\ub825 (HP)\uc774 \uc788\uc73c\uba70, \uc804\ud22c\uac00 \uc9c4\ud589\ub418\ub294 \ub3d9\uc548 \ud654\uba74\uc5d0 \ud45c\uc2dc\ub41c\ub2e4. HP\uac00 0\uc774 \ub418\uba74 \ud3ec\ucf13\ubaac\uc740 \uc4f0\ub7ec\uc9c0\uba70, \ud3ec\ucf13\ubaac \uc13c\ud130\uc5d0 \uac00\uac70\ub098, \ud3ec\ucf13\ubaac\uc758 \uae30\uc220\uc744 \uc4f0\uac70\ub098, \uc544\uc774\ud15c\uc744 \uc0ac\uc6a9\ud558\uc9c0 \uc54a\ub294 \uc774\uc0c1 \ubc30\ud2c0\uc744 \ud560 \uc218 \uc5c6\uac8c \ub41c\ub2e4. \ud50c\ub808\uc774\uc5b4\uc758 \ud3ec\ucf13\ubaac\uc774 \uc0c1\ub300 \ud3ec\ucf13\ubaac\uc744 \uc4f0\ub7ec\ub728\ub824\uc11c \uc774\uae30\uba74 \uacbd\ud5d8\uce58\ub97c \uc5bb\uac8c \ub41c\ub2e4. \uacbd\ud5d8\uce58\ub97c \ucda9\ubd84\ud788 \ubaa8\uc73c\uba74 \ub808\ubca8\uc774 \uc624\ub974\uba70, \ub300\ubd80\ubd84\uc758 \ud3ec\ucf13\ubaac\uc740 \uc77c\uc815\ud55c \ub808\ubca8\uc5d0 \ub2e4\ub2e4\ub974\uac70\ub098 \uc77c\uc815\ud55c \uc870\uac74 (\ud2b8\ub808\uc774\ub108\uc640\uc758 \uce5c\ubc00\ub3c4, \uc18c\uc9c0 \uc544\uc774\ud15c \ub4f1)\uc744 \ub9cc\uc871\ud558\uba74 \uc0c8\ub85c\uc6b4 \uc885\uc73c\ub85c \uc9c4\ud654\ud558\uac8c \ub41c\ub2e4."} {"question": "\uac1c\uc2e0\uad50 \uc9c4\uc601\uc758 \ucd1d\uc0ac\ub839\uad00 \uc774\ub984\uc740?", "paragraph": "\uac1c\uc2e0\uad50 \uc9c4\uc601\uc740 \ubcf4\ubcd1\uc744 2\uc5f4\ub85c \ubc30\uce58\ud558\uace0, \uadf8 \uc591\uc775\uc5d0 \uae30\ubcd1\uc744 \ubc30\uce58\ud588\ub2e4. \ubcf4\ubcd1\ub300\uc5f4\uc758 \uc804\ubc29\uc5d0\ub294 \uc911\ud3ec\ub97c \ubc30\uce58\ud558\uace0 \ub0a8\uc740 \uacbd\ud3ec\ub294 \ubd80\ub300 \uc0ac\uc774\uc5d0 \ucd1d\ubcd1\uc758 \ubd84\uacac\ub300\uc640 \ub354\ubd88\uc5b4 \ubc30\uce58\ub418\uc5c8\ub2e4. \ucd1d\uc0ac\ub839\uad00 \uad6c\uc2a4\ud0c0\ube0c 2\uc138 \uc544\ub3cc\ud504\ub294 \uc6b0\uc775\uc5d0 \ubc30\uce58\ud55c \uae30\ubcd1\ub300 \uc9c0\ud718\ub97c \ub9e1\uc558\uace0, \uc88c\uc775 \uae30\ubcd1\ub300\ub294 \uc791\uc13c \ubc14\uc774\ub9c8\ub974\uacf5(\u516c) \ubca0\ub978\ud558\ub974\ud2b8, \uc911\uc559 \ubcf4\ubcd1\ub300 \uc9c0\ud718\ub294 \uc804\uc5f4\uc5d4 \ub2d0\uc2a4 \ube0c\ub77c\ud5e4, \ud6c4\uc5f4\uc5d4 \ub3c4\ub3c4 \ud3f0 \ud06c\ub2c8\ud504\ud558\uc6b0\uc820(Dodo Knyphausen)\uc774 \ub9e1\uc558\ub2e4. \uc2a4\uc6e8\ub374\uad70\uc758 \uc804\ud22c\ub300\ud615\uc740 \uad6c\uc2dd\uc758 \ud14c\ub974\uc2dc\uc624 \uc9c4\ud615\uc774 \uc544\ub2c8\uace0, \uc5ec\ub7ec\uac1c\uc758 \uc911\ub300(\u4e2d\u968a)\uac00 \ud615\uc131\ub41c \ub300\ub300(\u5927\u968a)\uc640 \uc774 \ub300\ub300\ub97c \ubaa8\uc544 \ub9cc\ub4e0 \uc5ec\ub2e8(\u65c5\u5718)\uc73c\ub85c \uad6c\uc131\ub418\uc5c8\uae30 \ub54c\ubb38\uc5d0 \ud6a1\uc5f4\uc774 \ub113\uc740 \uac83\uc5d0 \ube44\ud574 \uc885\uc5f4\uc758 \ub450\uaed8\ub294 \uc587\uc544\uc11c \uadf8 \ubd84\ud654\ub825(\u5206\u706b\u529b)\uacfc \uc720\uc5f0\uc131\uc774 \ub6f0\uc5b4\ub0ac\ub2e4.", "answer": "\uad6c\uc2a4\ud0c0\ube0c 2\uc138 \uc544\ub3cc\ud504", "sentence": "\ucd1d\uc0ac\ub839\uad00 \uad6c\uc2a4\ud0c0\ube0c 2\uc138 \uc544\ub3cc\ud504 \ub294 \uc6b0\uc775\uc5d0 \ubc30\uce58\ud55c \uae30\ubcd1\ub300 \uc9c0\ud718\ub97c \ub9e1\uc558\uace0, \uc88c\uc775 \uae30\ubcd1\ub300\ub294 \uc791\uc13c \ubc14\uc774\ub9c8\ub974\uacf5(\u516c) \ubca0\ub978\ud558\ub974\ud2b8, \uc911\uc559 \ubcf4\ubcd1\ub300 \uc9c0\ud718\ub294 \uc804\uc5f4\uc5d4 \ub2d0\uc2a4 \ube0c\ub77c\ud5e4, \ud6c4\uc5f4\uc5d4 \ub3c4\ub3c4 \ud3f0 \ud06c\ub2c8\ud504\ud558\uc6b0\uc820(Dodo Knyphausen)\uc774 \ub9e1\uc558\ub2e4.", "paragraph_sentence": "\uac1c\uc2e0\uad50 \uc9c4\uc601\uc740 \ubcf4\ubcd1\uc744 2\uc5f4\ub85c \ubc30\uce58\ud558\uace0, \uadf8 \uc591\uc775\uc5d0 \uae30\ubcd1\uc744 \ubc30\uce58\ud588\ub2e4. \ubcf4\ubcd1\ub300\uc5f4\uc758 \uc804\ubc29\uc5d0\ub294 \uc911\ud3ec\ub97c \ubc30\uce58\ud558\uace0 \ub0a8\uc740 \uacbd\ud3ec\ub294 \ubd80\ub300 \uc0ac\uc774\uc5d0 \ucd1d\ubcd1\uc758 \ubd84\uacac\ub300\uc640 \ub354\ubd88\uc5b4 \ubc30\uce58\ub418\uc5c8\ub2e4. \ucd1d\uc0ac\ub839\uad00 \uad6c\uc2a4\ud0c0\ube0c 2\uc138 \uc544\ub3cc\ud504 \ub294 \uc6b0\uc775\uc5d0 \ubc30\uce58\ud55c \uae30\ubcd1\ub300 \uc9c0\ud718\ub97c \ub9e1\uc558\uace0, \uc88c\uc775 \uae30\ubcd1\ub300\ub294 \uc791\uc13c \ubc14\uc774\ub9c8\ub974\uacf5(\u516c) \ubca0\ub978\ud558\ub974\ud2b8, \uc911\uc559 \ubcf4\ubcd1\ub300 \uc9c0\ud718\ub294 \uc804\uc5f4\uc5d4 \ub2d0\uc2a4 \ube0c\ub77c\ud5e4, \ud6c4\uc5f4\uc5d4 \ub3c4\ub3c4 \ud3f0 \ud06c\ub2c8\ud504\ud558\uc6b0\uc820(Dodo Knyphausen)\uc774 \ub9e1\uc558\ub2e4. \uc2a4\uc6e8\ub374\uad70\uc758 \uc804\ud22c\ub300\ud615\uc740 \uad6c\uc2dd\uc758 \ud14c\ub974\uc2dc\uc624 \uc9c4\ud615\uc774 \uc544\ub2c8\uace0, \uc5ec\ub7ec\uac1c\uc758 \uc911\ub300(\u4e2d\u968a)\uac00 \ud615\uc131\ub41c \ub300\ub300(\u5927\u968a)\uc640 \uc774 \ub300\ub300\ub97c \ubaa8\uc544 \ub9cc\ub4e0 \uc5ec\ub2e8(\u65c5\u5718)\uc73c\ub85c \uad6c\uc131\ub418\uc5c8\uae30 \ub54c\ubb38\uc5d0 \ud6a1\uc5f4\uc774 \ub113\uc740 \uac83\uc5d0 \ube44\ud574 \uc885\uc5f4\uc758 \ub450\uaed8\ub294 \uc587\uc544\uc11c \uadf8 \ubd84\ud654\ub825(\u5206\u706b\u529b)\uacfc \uc720\uc5f0\uc131\uc774 \ub6f0\uc5b4\ub0ac\ub2e4.", "paragraph_answer": "\uac1c\uc2e0\uad50 \uc9c4\uc601\uc740 \ubcf4\ubcd1\uc744 2\uc5f4\ub85c \ubc30\uce58\ud558\uace0, \uadf8 \uc591\uc775\uc5d0 \uae30\ubcd1\uc744 \ubc30\uce58\ud588\ub2e4. \ubcf4\ubcd1\ub300\uc5f4\uc758 \uc804\ubc29\uc5d0\ub294 \uc911\ud3ec\ub97c \ubc30\uce58\ud558\uace0 \ub0a8\uc740 \uacbd\ud3ec\ub294 \ubd80\ub300 \uc0ac\uc774\uc5d0 \ucd1d\ubcd1\uc758 \ubd84\uacac\ub300\uc640 \ub354\ubd88\uc5b4 \ubc30\uce58\ub418\uc5c8\ub2e4. \ucd1d\uc0ac\ub839\uad00 \uad6c\uc2a4\ud0c0\ube0c 2\uc138 \uc544\ub3cc\ud504 \ub294 \uc6b0\uc775\uc5d0 \ubc30\uce58\ud55c \uae30\ubcd1\ub300 \uc9c0\ud718\ub97c \ub9e1\uc558\uace0, \uc88c\uc775 \uae30\ubcd1\ub300\ub294 \uc791\uc13c \ubc14\uc774\ub9c8\ub974\uacf5(\u516c) \ubca0\ub978\ud558\ub974\ud2b8, \uc911\uc559 \ubcf4\ubcd1\ub300 \uc9c0\ud718\ub294 \uc804\uc5f4\uc5d4 \ub2d0\uc2a4 \ube0c\ub77c\ud5e4, \ud6c4\uc5f4\uc5d4 \ub3c4\ub3c4 \ud3f0 \ud06c\ub2c8\ud504\ud558\uc6b0\uc820(Dodo Knyphausen)\uc774 \ub9e1\uc558\ub2e4. \uc2a4\uc6e8\ub374\uad70\uc758 \uc804\ud22c\ub300\ud615\uc740 \uad6c\uc2dd\uc758 \ud14c\ub974\uc2dc\uc624 \uc9c4\ud615\uc774 \uc544\ub2c8\uace0, \uc5ec\ub7ec\uac1c\uc758 \uc911\ub300(\u4e2d\u968a)\uac00 \ud615\uc131\ub41c \ub300\ub300(\u5927\u968a)\uc640 \uc774 \ub300\ub300\ub97c \ubaa8\uc544 \ub9cc\ub4e0 \uc5ec\ub2e8(\u65c5\u5718)\uc73c\ub85c \uad6c\uc131\ub418\uc5c8\uae30 \ub54c\ubb38\uc5d0 \ud6a1\uc5f4\uc774 \ub113\uc740 \uac83\uc5d0 \ube44\ud574 \uc885\uc5f4\uc758 \ub450\uaed8\ub294 \uc587\uc544\uc11c \uadf8 \ubd84\ud654\ub825(\u5206\u706b\u529b)\uacfc \uc720\uc5f0\uc131\uc774 \ub6f0\uc5b4\ub0ac\ub2e4.", "sentence_answer": "\ucd1d\uc0ac\ub839\uad00 \uad6c\uc2a4\ud0c0\ube0c 2\uc138 \uc544\ub3cc\ud504 \ub294 \uc6b0\uc775\uc5d0 \ubc30\uce58\ud55c \uae30\ubcd1\ub300 \uc9c0\ud718\ub97c \ub9e1\uc558\uace0, \uc88c\uc775 \uae30\ubcd1\ub300\ub294 \uc791\uc13c \ubc14\uc774\ub9c8\ub974\uacf5(\u516c) \ubca0\ub978\ud558\ub974\ud2b8, \uc911\uc559 \ubcf4\ubcd1\ub300 \uc9c0\ud718\ub294 \uc804\uc5f4\uc5d4 \ub2d0\uc2a4 \ube0c\ub77c\ud5e4, \ud6c4\uc5f4\uc5d4 \ub3c4\ub3c4 \ud3f0 \ud06c\ub2c8\ud504\ud558\uc6b0\uc820(Dodo Knyphausen)\uc774 \ub9e1\uc558\ub2e4.", "paragraph_id": "6497975-1-0", "paragraph_question": "question: \uac1c\uc2e0\uad50 \uc9c4\uc601\uc758 \ucd1d\uc0ac\ub839\uad00 \uc774\ub984\uc740?, context: \uac1c\uc2e0\uad50 \uc9c4\uc601\uc740 \ubcf4\ubcd1\uc744 2\uc5f4\ub85c \ubc30\uce58\ud558\uace0, \uadf8 \uc591\uc775\uc5d0 \uae30\ubcd1\uc744 \ubc30\uce58\ud588\ub2e4. \ubcf4\ubcd1\ub300\uc5f4\uc758 \uc804\ubc29\uc5d0\ub294 \uc911\ud3ec\ub97c \ubc30\uce58\ud558\uace0 \ub0a8\uc740 \uacbd\ud3ec\ub294 \ubd80\ub300 \uc0ac\uc774\uc5d0 \ucd1d\ubcd1\uc758 \ubd84\uacac\ub300\uc640 \ub354\ubd88\uc5b4 \ubc30\uce58\ub418\uc5c8\ub2e4. \ucd1d\uc0ac\ub839\uad00 \uad6c\uc2a4\ud0c0\ube0c 2\uc138 \uc544\ub3cc\ud504\ub294 \uc6b0\uc775\uc5d0 \ubc30\uce58\ud55c \uae30\ubcd1\ub300 \uc9c0\ud718\ub97c \ub9e1\uc558\uace0, \uc88c\uc775 \uae30\ubcd1\ub300\ub294 \uc791\uc13c \ubc14\uc774\ub9c8\ub974\uacf5(\u516c) \ubca0\ub978\ud558\ub974\ud2b8, \uc911\uc559 \ubcf4\ubcd1\ub300 \uc9c0\ud718\ub294 \uc804\uc5f4\uc5d4 \ub2d0\uc2a4 \ube0c\ub77c\ud5e4, \ud6c4\uc5f4\uc5d4 \ub3c4\ub3c4 \ud3f0 \ud06c\ub2c8\ud504\ud558\uc6b0\uc820(Dodo Knyphausen)\uc774 \ub9e1\uc558\ub2e4. \uc2a4\uc6e8\ub374\uad70\uc758 \uc804\ud22c\ub300\ud615\uc740 \uad6c\uc2dd\uc758 \ud14c\ub974\uc2dc\uc624 \uc9c4\ud615\uc774 \uc544\ub2c8\uace0, \uc5ec\ub7ec\uac1c\uc758 \uc911\ub300(\u4e2d\u968a)\uac00 \ud615\uc131\ub41c \ub300\ub300(\u5927\u968a)\uc640 \uc774 \ub300\ub300\ub97c \ubaa8\uc544 \ub9cc\ub4e0 \uc5ec\ub2e8(\u65c5\u5718)\uc73c\ub85c \uad6c\uc131\ub418\uc5c8\uae30 \ub54c\ubb38\uc5d0 \ud6a1\uc5f4\uc774 \ub113\uc740 \uac83\uc5d0 \ube44\ud574 \uc885\uc5f4\uc758 \ub450\uaed8\ub294 \uc587\uc544\uc11c \uadf8 \ubd84\ud654\ub825(\u5206\u706b\u529b)\uacfc \uc720\uc5f0\uc131\uc774 \ub6f0\uc5b4\ub0ac\ub2e4."} {"question": "\ucc29\ud55c \uac74 \ub098\uc05c\uac8c \uc544\ub2c8\uc57c pt.2\uc5d0\uc11c \ubcf4\uceec\uc744 \ub9e1\uc740 \uc0ac\ub78c\uc740?", "paragraph": "\uc774\uc804 \ubc31\uc9c0\uc218\ud45c \ud504\ub85c\uc81d\ud2b8\ub97c \ud1b5\ud574 \uacf5\uac1c\ud55c \u3008\uc88b\ub2e4 \ub9d0\uc558\ub124\u3009\ub294 \ub2e4\uc2dc \ub179\uc74c\ud588\uc9c0\ub9cc \uacb0\uad6d\uc740 \uc6d0\ub798 \uac83\uc774 \ub0ab\ub2e4\uace0 \ud310\ub2e8\ud558\uc5ec \uc791\ub144 \ubc84\uc804 \uadf8\ub300\ub85c \uc2e4\ub838\ub2e4. \u3008\uc62c \uc0dd\uac01\uc744 \uc54a\ub124\u3009\uc758 \"\ud2b8\ub7fc\ubcf8 \uc5f0\uc8fc\ub294 \ud0a4\ubcf4\ub514\uc2a4\ud2b8 \uc774\uc885\ubbfc\uc774 \uad70\uc545\ub300 \uc2dc\uc808 \ubc30\uc6b4 \uc2e4\ub825\uc744 \ubc1c\ud718\ud55c \uac83\"\uc774\ub77c\uace0 \ud588\ub2e4. \u3008\uae30\uc5b5 \uc548 \ub098\u3009\uc5d0\uc11c\ub294 \uc5ec\uc790\uac00\uc218\uc758 \ud53c\uccd0\ub9c1\uc744 \uc5fc\ub450\uc5d0 \ub450\uace0 \uc7a5\uae30\ud558\uac00 \uac00\uc131\uc73c\ub85c \uac00\ub179\uc74c\uc744 \ud588\ub294\ub370 \uacb0\uad6d \uadf8 \uac00\ub179\uc74c\uc774 \ucc44\ud0dd\ub418\uace0 \ud53c\uccd0\ub9c1\uc740 \uc5c6\uc5c8\ub358 \uc77c\uc774 \ub418\uc5c8\ub2e4. \uc7a5\uae30\ud558\ub294 \"\uc801\ub2f9\ud55c \uc5ec\uc790 \uac00\uc218\uac00 \uc548 \ub5a0\uc62c\ub77c \uace0\uc2ec\ud558\ub358 \uc911 \uc74c\ubc18\uc0ac \ub300\ud45c\uac00 '\uadf8\ub0e5 \uac00\ub179\uc74c \uc4f0\uba74 \uc548\ub3fc?\u2019 \ud558\uace0 \ud22d \ub358\uc9c0\ub354\ub77c\uace0\uc694. '\uc544! \uadf8\ub7f4 \uc218\ub3c4 \uc788\uad6c\ub098' \ud588\uc5b4\uc694. \ubc1c\uc0c1\uc758 \uc804\ud658\uc774\uc5c8\uc8e0. \uac00\ub179\uc74c \ub54c \ub300\ucda9 \ubd88\ub7ec \uc74c\uc774\ud0c8\ub3c4 \uc788\uc9c0\ub9cc, \uacb0\uacfc\uc801\uc73c\ub85c \uc568\ubc94\uc5d0\uc11c \uac00\uc7a5 \uc7ac\ubbf8\uc788\ub294 \ub178\ub798\uac00 \ub410\uc5b4\uc694.\"\ub77c\uace0 \ud68c\uc0c1\ud588\ub2e4. \u3008\uc78a\ud600\uc9c0\uc9c0 \uc54a\ub124\u3009\uc5d0 \ub300\ud574\uc11c \uc7a5\uae30\ud558\ub294 \"\uc544\uc8fc \ub2e8\uc21c\ud55c \ube44\ud2b8\uc600\uc73c\uba74 \uc88b\uaca0\ub2e4\ub294 \uc0dd\uac01\uc73c\ub85c 80\ub144\ub300\uc758 \ub9ac\ub4ec\uba38\uc2e0\uc744 \uc0ac\uc6a9\ud588\ub2e4. \ub4dc\ub7ec\uba38 \uc804\uc77c\uc900\uc774 \uc9c1\uc811 \ubc84\ud2bc\uc744 \ub20c\ub7ec\uac00\uba70 \uc5f0\uc8fc\ud588\ub2e4.\"\uace0 \uc37c\ub2e4. \u3008\ucc29\ud55c \uac74 \ub098\uc05c \uac8c \uc544\ub2c8\uc57c pt. 2\u3009\ub294 \uc81c\uc791 \ub3c4\uc911 \"\ud55c \ud0a4\uac00 \uc62c\ub77c\uac00\uace0 \uc9c0\ub974\ub294 \ubd80\ubd84\uc774 \ub4e4\uc5c8\uc744 \ub54c, \ucc98\uc74c\ubd80\ud130 \uc804\uc778\uad8c \uc120\ubc30\ub2d8\uc774 \ubd88\ub7ec\uc8fc\uc168\uc73c\uba74 \ud588\ub2e4.\"\uba70 \ud754\ucf8c\ud788 \uc804\uc778\uad8c \uc120\ubc30\ub2d8\uc774 \ubcf4\uceec\uc744 \ub9e1\uc544\uc8fc\uc5c8\ub2e4\uace0 \ud588\ub2e4. \uc7a5\uae30\ud558\uac00 2005\ub144 \uccad\ub144\uc2e4\uc5c5 \uc2dc\uc808 \ubc1c\ud45c\ud558\uace0 \ub77c\uc774\ube0c \ub808\ud37c\ud1a0\ub9ac\ub85c \uc0ac\uc6a9\ub41c \u3008\uae30\uc0c1\uc2dc\uac04\uc740 \uc815\ud574\uc838\uc788\ub2e4\u3009\ub294 \ucc98\uc74c\uc73c\ub85c \ubc34\ub4dc\uc758 \ubc84\uc804\uc744 \ub179\uc74c\ud574 \uc2e4\uc5c8\ub2e4.", "answer": "\uc804\uc778\uad8c", "sentence": "2\u3009\ub294 \uc81c\uc791 \ub3c4\uc911 \"\ud55c \ud0a4\uac00 \uc62c\ub77c\uac00\uace0 \uc9c0\ub974\ub294 \ubd80\ubd84\uc774 \ub4e4\uc5c8\uc744 \ub54c, \ucc98\uc74c\ubd80\ud130 \uc804\uc778\uad8c \uc120\ubc30\ub2d8\uc774 \ubd88\ub7ec\uc8fc\uc168\uc73c\uba74 \ud588\ub2e4.", "paragraph_sentence": "\uc774\uc804 \ubc31\uc9c0\uc218\ud45c \ud504\ub85c\uc81d\ud2b8\ub97c \ud1b5\ud574 \uacf5\uac1c\ud55c \u3008\uc88b\ub2e4 \ub9d0\uc558\ub124\u3009\ub294 \ub2e4\uc2dc \ub179\uc74c\ud588\uc9c0\ub9cc \uacb0\uad6d\uc740 \uc6d0\ub798 \uac83\uc774 \ub0ab\ub2e4\uace0 \ud310\ub2e8\ud558\uc5ec \uc791\ub144 \ubc84\uc804 \uadf8\ub300\ub85c \uc2e4\ub838\ub2e4. \u3008\uc62c \uc0dd\uac01\uc744 \uc54a\ub124\u3009\uc758 \"\ud2b8\ub7fc\ubcf8 \uc5f0\uc8fc\ub294 \ud0a4\ubcf4\ub514\uc2a4\ud2b8 \uc774\uc885\ubbfc\uc774 \uad70\uc545\ub300 \uc2dc\uc808 \ubc30\uc6b4 \uc2e4\ub825\uc744 \ubc1c\ud718\ud55c \uac83\"\uc774\ub77c\uace0 \ud588\ub2e4. \u3008\uae30\uc5b5 \uc548 \ub098\u3009\uc5d0\uc11c\ub294 \uc5ec\uc790\uac00\uc218\uc758 \ud53c\uccd0\ub9c1\uc744 \uc5fc\ub450\uc5d0 \ub450\uace0 \uc7a5\uae30\ud558\uac00 \uac00\uc131\uc73c\ub85c \uac00\ub179\uc74c\uc744 \ud588\ub294\ub370 \uacb0\uad6d \uadf8 \uac00\ub179\uc74c\uc774 \ucc44\ud0dd\ub418\uace0 \ud53c\uccd0\ub9c1\uc740 \uc5c6\uc5c8\ub358 \uc77c\uc774 \ub418\uc5c8\ub2e4. \uc7a5\uae30\ud558\ub294 \"\uc801\ub2f9\ud55c \uc5ec\uc790 \uac00\uc218\uac00 \uc548 \ub5a0\uc62c\ub77c \uace0\uc2ec\ud558\ub358 \uc911 \uc74c\ubc18\uc0ac \ub300\ud45c\uac00 '\uadf8\ub0e5 \uac00\ub179\uc74c \uc4f0\uba74 \uc548\ub3fc? \u2019 \ud558\uace0 \ud22d \ub358\uc9c0\ub354\ub77c\uace0\uc694. '\uc544! \uadf8\ub7f4 \uc218\ub3c4 \uc788\uad6c\ub098' \ud588\uc5b4\uc694. \ubc1c\uc0c1\uc758 \uc804\ud658\uc774\uc5c8\uc8e0. \uac00\ub179\uc74c \ub54c \ub300\ucda9 \ubd88\ub7ec \uc74c\uc774\ud0c8\ub3c4 \uc788\uc9c0\ub9cc, \uacb0\uacfc\uc801\uc73c\ub85c \uc568\ubc94\uc5d0\uc11c \uac00\uc7a5 \uc7ac\ubbf8\uc788\ub294 \ub178\ub798\uac00 \ub410\uc5b4\uc694. \"\ub77c\uace0 \ud68c\uc0c1\ud588\ub2e4. \u3008\uc78a\ud600\uc9c0\uc9c0 \uc54a\ub124\u3009\uc5d0 \ub300\ud574\uc11c \uc7a5\uae30\ud558\ub294 \"\uc544\uc8fc \ub2e8\uc21c\ud55c \ube44\ud2b8\uc600\uc73c\uba74 \uc88b\uaca0\ub2e4\ub294 \uc0dd\uac01\uc73c\ub85c 80\ub144\ub300\uc758 \ub9ac\ub4ec\uba38\uc2e0\uc744 \uc0ac\uc6a9\ud588\ub2e4. \ub4dc\ub7ec\uba38 \uc804\uc77c\uc900\uc774 \uc9c1\uc811 \ubc84\ud2bc\uc744 \ub20c\ub7ec\uac00\uba70 \uc5f0\uc8fc\ud588\ub2e4. \"\uace0 \uc37c\ub2e4. \u3008\ucc29\ud55c \uac74 \ub098\uc05c \uac8c \uc544\ub2c8\uc57c pt. 2\u3009\ub294 \uc81c\uc791 \ub3c4\uc911 \"\ud55c \ud0a4\uac00 \uc62c\ub77c\uac00\uace0 \uc9c0\ub974\ub294 \ubd80\ubd84\uc774 \ub4e4\uc5c8\uc744 \ub54c, \ucc98\uc74c\ubd80\ud130 \uc804\uc778\uad8c \uc120\ubc30\ub2d8\uc774 \ubd88\ub7ec\uc8fc\uc168\uc73c\uba74 \ud588\ub2e4. \"\uba70 \ud754\ucf8c\ud788 \uc804\uc778\uad8c \uc120\ubc30\ub2d8\uc774 \ubcf4\uceec\uc744 \ub9e1\uc544\uc8fc\uc5c8\ub2e4\uace0 \ud588\ub2e4. \uc7a5\uae30\ud558\uac00 2005\ub144 \uccad\ub144\uc2e4\uc5c5 \uc2dc\uc808 \ubc1c\ud45c\ud558\uace0 \ub77c\uc774\ube0c \ub808\ud37c\ud1a0\ub9ac\ub85c \uc0ac\uc6a9\ub41c \u3008\uae30\uc0c1\uc2dc\uac04\uc740 \uc815\ud574\uc838\uc788\ub2e4\u3009\ub294 \ucc98\uc74c\uc73c\ub85c \ubc34\ub4dc\uc758 \ubc84\uc804\uc744 \ub179\uc74c\ud574 \uc2e4\uc5c8\ub2e4.", "paragraph_answer": "\uc774\uc804 \ubc31\uc9c0\uc218\ud45c \ud504\ub85c\uc81d\ud2b8\ub97c \ud1b5\ud574 \uacf5\uac1c\ud55c \u3008\uc88b\ub2e4 \ub9d0\uc558\ub124\u3009\ub294 \ub2e4\uc2dc \ub179\uc74c\ud588\uc9c0\ub9cc \uacb0\uad6d\uc740 \uc6d0\ub798 \uac83\uc774 \ub0ab\ub2e4\uace0 \ud310\ub2e8\ud558\uc5ec \uc791\ub144 \ubc84\uc804 \uadf8\ub300\ub85c \uc2e4\ub838\ub2e4. \u3008\uc62c \uc0dd\uac01\uc744 \uc54a\ub124\u3009\uc758 \"\ud2b8\ub7fc\ubcf8 \uc5f0\uc8fc\ub294 \ud0a4\ubcf4\ub514\uc2a4\ud2b8 \uc774\uc885\ubbfc\uc774 \uad70\uc545\ub300 \uc2dc\uc808 \ubc30\uc6b4 \uc2e4\ub825\uc744 \ubc1c\ud718\ud55c \uac83\"\uc774\ub77c\uace0 \ud588\ub2e4. \u3008\uae30\uc5b5 \uc548 \ub098\u3009\uc5d0\uc11c\ub294 \uc5ec\uc790\uac00\uc218\uc758 \ud53c\uccd0\ub9c1\uc744 \uc5fc\ub450\uc5d0 \ub450\uace0 \uc7a5\uae30\ud558\uac00 \uac00\uc131\uc73c\ub85c \uac00\ub179\uc74c\uc744 \ud588\ub294\ub370 \uacb0\uad6d \uadf8 \uac00\ub179\uc74c\uc774 \ucc44\ud0dd\ub418\uace0 \ud53c\uccd0\ub9c1\uc740 \uc5c6\uc5c8\ub358 \uc77c\uc774 \ub418\uc5c8\ub2e4. \uc7a5\uae30\ud558\ub294 \"\uc801\ub2f9\ud55c \uc5ec\uc790 \uac00\uc218\uac00 \uc548 \ub5a0\uc62c\ub77c \uace0\uc2ec\ud558\ub358 \uc911 \uc74c\ubc18\uc0ac \ub300\ud45c\uac00 '\uadf8\ub0e5 \uac00\ub179\uc74c \uc4f0\uba74 \uc548\ub3fc?\u2019 \ud558\uace0 \ud22d \ub358\uc9c0\ub354\ub77c\uace0\uc694. '\uc544! \uadf8\ub7f4 \uc218\ub3c4 \uc788\uad6c\ub098' \ud588\uc5b4\uc694. \ubc1c\uc0c1\uc758 \uc804\ud658\uc774\uc5c8\uc8e0. \uac00\ub179\uc74c \ub54c \ub300\ucda9 \ubd88\ub7ec \uc74c\uc774\ud0c8\ub3c4 \uc788\uc9c0\ub9cc, \uacb0\uacfc\uc801\uc73c\ub85c \uc568\ubc94\uc5d0\uc11c \uac00\uc7a5 \uc7ac\ubbf8\uc788\ub294 \ub178\ub798\uac00 \ub410\uc5b4\uc694.\"\ub77c\uace0 \ud68c\uc0c1\ud588\ub2e4. \u3008\uc78a\ud600\uc9c0\uc9c0 \uc54a\ub124\u3009\uc5d0 \ub300\ud574\uc11c \uc7a5\uae30\ud558\ub294 \"\uc544\uc8fc \ub2e8\uc21c\ud55c \ube44\ud2b8\uc600\uc73c\uba74 \uc88b\uaca0\ub2e4\ub294 \uc0dd\uac01\uc73c\ub85c 80\ub144\ub300\uc758 \ub9ac\ub4ec\uba38\uc2e0\uc744 \uc0ac\uc6a9\ud588\ub2e4. \ub4dc\ub7ec\uba38 \uc804\uc77c\uc900\uc774 \uc9c1\uc811 \ubc84\ud2bc\uc744 \ub20c\ub7ec\uac00\uba70 \uc5f0\uc8fc\ud588\ub2e4.\"\uace0 \uc37c\ub2e4. \u3008\ucc29\ud55c \uac74 \ub098\uc05c \uac8c \uc544\ub2c8\uc57c pt. 2\u3009\ub294 \uc81c\uc791 \ub3c4\uc911 \"\ud55c \ud0a4\uac00 \uc62c\ub77c\uac00\uace0 \uc9c0\ub974\ub294 \ubd80\ubd84\uc774 \ub4e4\uc5c8\uc744 \ub54c, \ucc98\uc74c\ubd80\ud130 \uc804\uc778\uad8c \uc120\ubc30\ub2d8\uc774 \ubd88\ub7ec\uc8fc\uc168\uc73c\uba74 \ud588\ub2e4.\"\uba70 \ud754\ucf8c\ud788 \uc804\uc778\uad8c \uc120\ubc30\ub2d8\uc774 \ubcf4\uceec\uc744 \ub9e1\uc544\uc8fc\uc5c8\ub2e4\uace0 \ud588\ub2e4. \uc7a5\uae30\ud558\uac00 2005\ub144 \uccad\ub144\uc2e4\uc5c5 \uc2dc\uc808 \ubc1c\ud45c\ud558\uace0 \ub77c\uc774\ube0c \ub808\ud37c\ud1a0\ub9ac\ub85c \uc0ac\uc6a9\ub41c \u3008\uae30\uc0c1\uc2dc\uac04\uc740 \uc815\ud574\uc838\uc788\ub2e4\u3009\ub294 \ucc98\uc74c\uc73c\ub85c \ubc34\ub4dc\uc758 \ubc84\uc804\uc744 \ub179\uc74c\ud574 \uc2e4\uc5c8\ub2e4.", "sentence_answer": "2\u3009\ub294 \uc81c\uc791 \ub3c4\uc911 \"\ud55c \ud0a4\uac00 \uc62c\ub77c\uac00\uace0 \uc9c0\ub974\ub294 \ubd80\ubd84\uc774 \ub4e4\uc5c8\uc744 \ub54c, \ucc98\uc74c\ubd80\ud130 \uc804\uc778\uad8c \uc120\ubc30\ub2d8\uc774 \ubd88\ub7ec\uc8fc\uc168\uc73c\uba74 \ud588\ub2e4.", "paragraph_id": "6535321-5-2", "paragraph_question": "question: \ucc29\ud55c \uac74 \ub098\uc05c\uac8c \uc544\ub2c8\uc57c pt.2\uc5d0\uc11c \ubcf4\uceec\uc744 \ub9e1\uc740 \uc0ac\ub78c\uc740?, context: \uc774\uc804 \ubc31\uc9c0\uc218\ud45c \ud504\ub85c\uc81d\ud2b8\ub97c \ud1b5\ud574 \uacf5\uac1c\ud55c \u3008\uc88b\ub2e4 \ub9d0\uc558\ub124\u3009\ub294 \ub2e4\uc2dc \ub179\uc74c\ud588\uc9c0\ub9cc \uacb0\uad6d\uc740 \uc6d0\ub798 \uac83\uc774 \ub0ab\ub2e4\uace0 \ud310\ub2e8\ud558\uc5ec \uc791\ub144 \ubc84\uc804 \uadf8\ub300\ub85c \uc2e4\ub838\ub2e4. \u3008\uc62c \uc0dd\uac01\uc744 \uc54a\ub124\u3009\uc758 \"\ud2b8\ub7fc\ubcf8 \uc5f0\uc8fc\ub294 \ud0a4\ubcf4\ub514\uc2a4\ud2b8 \uc774\uc885\ubbfc\uc774 \uad70\uc545\ub300 \uc2dc\uc808 \ubc30\uc6b4 \uc2e4\ub825\uc744 \ubc1c\ud718\ud55c \uac83\"\uc774\ub77c\uace0 \ud588\ub2e4. \u3008\uae30\uc5b5 \uc548 \ub098\u3009\uc5d0\uc11c\ub294 \uc5ec\uc790\uac00\uc218\uc758 \ud53c\uccd0\ub9c1\uc744 \uc5fc\ub450\uc5d0 \ub450\uace0 \uc7a5\uae30\ud558\uac00 \uac00\uc131\uc73c\ub85c \uac00\ub179\uc74c\uc744 \ud588\ub294\ub370 \uacb0\uad6d \uadf8 \uac00\ub179\uc74c\uc774 \ucc44\ud0dd\ub418\uace0 \ud53c\uccd0\ub9c1\uc740 \uc5c6\uc5c8\ub358 \uc77c\uc774 \ub418\uc5c8\ub2e4. \uc7a5\uae30\ud558\ub294 \"\uc801\ub2f9\ud55c \uc5ec\uc790 \uac00\uc218\uac00 \uc548 \ub5a0\uc62c\ub77c \uace0\uc2ec\ud558\ub358 \uc911 \uc74c\ubc18\uc0ac \ub300\ud45c\uac00 '\uadf8\ub0e5 \uac00\ub179\uc74c \uc4f0\uba74 \uc548\ub3fc?\u2019 \ud558\uace0 \ud22d \ub358\uc9c0\ub354\ub77c\uace0\uc694. '\uc544! \uadf8\ub7f4 \uc218\ub3c4 \uc788\uad6c\ub098' \ud588\uc5b4\uc694. \ubc1c\uc0c1\uc758 \uc804\ud658\uc774\uc5c8\uc8e0. \uac00\ub179\uc74c \ub54c \ub300\ucda9 \ubd88\ub7ec \uc74c\uc774\ud0c8\ub3c4 \uc788\uc9c0\ub9cc, \uacb0\uacfc\uc801\uc73c\ub85c \uc568\ubc94\uc5d0\uc11c \uac00\uc7a5 \uc7ac\ubbf8\uc788\ub294 \ub178\ub798\uac00 \ub410\uc5b4\uc694.\"\ub77c\uace0 \ud68c\uc0c1\ud588\ub2e4. \u3008\uc78a\ud600\uc9c0\uc9c0 \uc54a\ub124\u3009\uc5d0 \ub300\ud574\uc11c \uc7a5\uae30\ud558\ub294 \"\uc544\uc8fc \ub2e8\uc21c\ud55c \ube44\ud2b8\uc600\uc73c\uba74 \uc88b\uaca0\ub2e4\ub294 \uc0dd\uac01\uc73c\ub85c 80\ub144\ub300\uc758 \ub9ac\ub4ec\uba38\uc2e0\uc744 \uc0ac\uc6a9\ud588\ub2e4. \ub4dc\ub7ec\uba38 \uc804\uc77c\uc900\uc774 \uc9c1\uc811 \ubc84\ud2bc\uc744 \ub20c\ub7ec\uac00\uba70 \uc5f0\uc8fc\ud588\ub2e4.\"\uace0 \uc37c\ub2e4. \u3008\ucc29\ud55c \uac74 \ub098\uc05c \uac8c \uc544\ub2c8\uc57c pt. 2\u3009\ub294 \uc81c\uc791 \ub3c4\uc911 \"\ud55c \ud0a4\uac00 \uc62c\ub77c\uac00\uace0 \uc9c0\ub974\ub294 \ubd80\ubd84\uc774 \ub4e4\uc5c8\uc744 \ub54c, \ucc98\uc74c\ubd80\ud130 \uc804\uc778\uad8c \uc120\ubc30\ub2d8\uc774 \ubd88\ub7ec\uc8fc\uc168\uc73c\uba74 \ud588\ub2e4.\"\uba70 \ud754\ucf8c\ud788 \uc804\uc778\uad8c \uc120\ubc30\ub2d8\uc774 \ubcf4\uceec\uc744 \ub9e1\uc544\uc8fc\uc5c8\ub2e4\uace0 \ud588\ub2e4. \uc7a5\uae30\ud558\uac00 2005\ub144 \uccad\ub144\uc2e4\uc5c5 \uc2dc\uc808 \ubc1c\ud45c\ud558\uace0 \ub77c\uc774\ube0c \ub808\ud37c\ud1a0\ub9ac\ub85c \uc0ac\uc6a9\ub41c \u3008\uae30\uc0c1\uc2dc\uac04\uc740 \uc815\ud574\uc838\uc788\ub2e4\u3009\ub294 \ucc98\uc74c\uc73c\ub85c \ubc34\ub4dc\uc758 \ubc84\uc804\uc744 \ub179\uc74c\ud574 \uc2e4\uc5c8\ub2e4."} {"question": "\uac04\ub514\uac00 \uc81c1\ucc28 \uc138\uacc4\ub300\uc804\uc5d0 \ucc38\uc804\ud558\uc9c0 \ubabb\ud558\uac8c \ub41c \uac83\uc740 \ubb34\uc5c7\uc5d0 \uc758\ud55c \uc5fc\uc99d\uc5d0 \uac78\ub838\uae30 \ub54c\ubb38\uc778\uac00?", "paragraph": "7\uc0b4 \ub54c \uc774\ubbf8 \uadf8\ub294 \uc57d\ud63c\uc744 \ud588\uc5c8\uc9c0\ub9cc \uc57d\ud63c\ub140\ub4e4\uc774 \ubaa8\ub450 \ucc28\ub840\ub85c \uc8fd\uc5b4, \uc138 \ubc88\uc9f8 \uc57d\ud63c\ub140\uc778 \uc0c1\uc778 \uc9d1\uc548\uc758 \ub538\uc778 \uce74\uc2a4\ud22c\ub974\ubc14\uc774 \ub9c8\uce78\uc9c0\uc640 13\uc138\uc5d0 \uacb0\ud63c\ud55c\ub2e4. \uac04\ub514\ub294 \uc544\ub0b4\uc640 \uc7a0\uc790\ub9ac\ub97c \uac00\uc9c0\ub290\ub77c \uc544\ubc84\uc9c0\uc758 \uc784\uc885\uc744 \uc9c0\ud0a4\uc9c0 \ubabb\ud588\ub2e4\uace0 \ud558\ub294\ub370, \uc774 \uc0ac\uac74 \ub54c\ubb38\uc5d0 \uadf8\ub294 \uae4a\uc774 \ubc18\uc131\ud558\uace0 \uae08\uc695\uc5d0 \ub300\ud55c \uc0dd\uac01\uc744 \uac16\uac8c \ub41c\ub2e4. \uadf8 \ub4a4 \ubc29\ud0d5\ud55c \uc0dd\ud65c\uc744 \ubc84\ub9ac\uace0 \ud78c\ub450\uad50\uc758 \uad50\ub9ac\ub97c \ucda9\uc2e4\ud788 \uc9c0\ud0a4\uba70 \uc131\uc695\uc744 \uc790\uc81c\ud558\uae30\ub3c4 \ud588\ub2e4. \uadf8\ub294 \uc815\uc561\uc740 \uc0dd\uba85\ub825\uc758 \uadfc\uc6d0\uc774\uace0 \uc815\uc561\uc758 \ubc30\ucd9c\uc740 \ubab8\uacfc \uba38\ub9ac\ub97c \uc57d\ud558\uac8c \ub9cc\ub4e0\ub2e4\uace0 \ubbff\uc5c8\ub2e4. \uadf8\ub4a4\ub85c \uc77c\uc0dd \ub3d9\uc548 \uc218\ub9ce\uc740 \uc5ec\uc131\uc758 \uc0ac\ub791\uc744 \ubc1b\uace0 \uadf8\ub4e4\uc744 \uac00\uae4c\uc774 \ub450\uc5c8\uc74c\uc5d0\ub3c4 \uc9c1\uc811\uc801\uc778 \uc131\ud589\uc704\ub098 \uc0ac\uc815\uc740 \uc790\uc81c\ud558\uc600\ub2e4 \ud55c\ub2e4. \uadf8\ub294 \ubcf4\uc5b4 \uc804\uc7c1\uacfc \uc544\ud504\ub9ac\uce74 \uc804\uc7c1 \uadf8\ub9ac\uace0 \uc81c1\ucc28 \uc138\uacc4\ub300\uc804\uc5d0 \ucc38\uc804\ud558\ub824\uace0 \uc601\uad6d\uad70\uc5d0 \uc9c0\uc6d0\ud588\uc73c\ub098 \ucc38\uc804\ud558\uae30 \uba70\uce60 \uc804 \uacb0\ud575\uc5d0 \uc758\ud55c \uc5fc\uc99d\uc5d0 \uac78\ub824\uc11c \ud3ec\uae30\ud574\uc57c\ub9cc \ud588\ub2e4.", "answer": "\uacb0\ud575", "sentence": "\uadf8\ub294 \ubcf4\uc5b4 \uc804\uc7c1\uacfc \uc544\ud504\ub9ac\uce74 \uc804\uc7c1 \uadf8\ub9ac\uace0 \uc81c1\ucc28 \uc138\uacc4\ub300\uc804\uc5d0 \ucc38\uc804\ud558\ub824\uace0 \uc601\uad6d\uad70\uc5d0 \uc9c0\uc6d0\ud588\uc73c\ub098 \ucc38\uc804\ud558\uae30 \uba70\uce60 \uc804 \uacb0\ud575 \uc5d0 \uc758\ud55c \uc5fc\uc99d\uc5d0 \uac78\ub824\uc11c \ud3ec\uae30\ud574\uc57c\ub9cc \ud588\ub2e4.", "paragraph_sentence": "7\uc0b4 \ub54c \uc774\ubbf8 \uadf8\ub294 \uc57d\ud63c\uc744 \ud588\uc5c8\uc9c0\ub9cc \uc57d\ud63c\ub140\ub4e4\uc774 \ubaa8\ub450 \ucc28\ub840\ub85c \uc8fd\uc5b4, \uc138 \ubc88\uc9f8 \uc57d\ud63c\ub140\uc778 \uc0c1\uc778 \uc9d1\uc548\uc758 \ub538\uc778 \uce74\uc2a4\ud22c\ub974\ubc14\uc774 \ub9c8\uce78\uc9c0\uc640 13\uc138\uc5d0 \uacb0\ud63c\ud55c\ub2e4. \uac04\ub514\ub294 \uc544\ub0b4\uc640 \uc7a0\uc790\ub9ac\ub97c \uac00\uc9c0\ub290\ub77c \uc544\ubc84\uc9c0\uc758 \uc784\uc885\uc744 \uc9c0\ud0a4\uc9c0 \ubabb\ud588\ub2e4\uace0 \ud558\ub294\ub370, \uc774 \uc0ac\uac74 \ub54c\ubb38\uc5d0 \uadf8\ub294 \uae4a\uc774 \ubc18\uc131\ud558\uace0 \uae08\uc695\uc5d0 \ub300\ud55c \uc0dd\uac01\uc744 \uac16\uac8c \ub41c\ub2e4. \uadf8 \ub4a4 \ubc29\ud0d5\ud55c \uc0dd\ud65c\uc744 \ubc84\ub9ac\uace0 \ud78c\ub450\uad50\uc758 \uad50\ub9ac\ub97c \ucda9\uc2e4\ud788 \uc9c0\ud0a4\uba70 \uc131\uc695\uc744 \uc790\uc81c\ud558\uae30\ub3c4 \ud588\ub2e4. \uadf8\ub294 \uc815\uc561\uc740 \uc0dd\uba85\ub825\uc758 \uadfc\uc6d0\uc774\uace0 \uc815\uc561\uc758 \ubc30\ucd9c\uc740 \ubab8\uacfc \uba38\ub9ac\ub97c \uc57d\ud558\uac8c \ub9cc\ub4e0\ub2e4\uace0 \ubbff\uc5c8\ub2e4. \uadf8\ub4a4\ub85c \uc77c\uc0dd \ub3d9\uc548 \uc218\ub9ce\uc740 \uc5ec\uc131\uc758 \uc0ac\ub791\uc744 \ubc1b\uace0 \uadf8\ub4e4\uc744 \uac00\uae4c\uc774 \ub450\uc5c8\uc74c\uc5d0\ub3c4 \uc9c1\uc811\uc801\uc778 \uc131\ud589\uc704\ub098 \uc0ac\uc815\uc740 \uc790\uc81c\ud558\uc600\ub2e4 \ud55c\ub2e4. \uadf8\ub294 \ubcf4\uc5b4 \uc804\uc7c1\uacfc \uc544\ud504\ub9ac\uce74 \uc804\uc7c1 \uadf8\ub9ac\uace0 \uc81c1\ucc28 \uc138\uacc4\ub300\uc804\uc5d0 \ucc38\uc804\ud558\ub824\uace0 \uc601\uad6d\uad70\uc5d0 \uc9c0\uc6d0\ud588\uc73c\ub098 \ucc38\uc804\ud558\uae30 \uba70\uce60 \uc804 \uacb0\ud575 \uc5d0 \uc758\ud55c \uc5fc\uc99d\uc5d0 \uac78\ub824\uc11c \ud3ec\uae30\ud574\uc57c\ub9cc \ud588\ub2e4. ", "paragraph_answer": "7\uc0b4 \ub54c \uc774\ubbf8 \uadf8\ub294 \uc57d\ud63c\uc744 \ud588\uc5c8\uc9c0\ub9cc \uc57d\ud63c\ub140\ub4e4\uc774 \ubaa8\ub450 \ucc28\ub840\ub85c \uc8fd\uc5b4, \uc138 \ubc88\uc9f8 \uc57d\ud63c\ub140\uc778 \uc0c1\uc778 \uc9d1\uc548\uc758 \ub538\uc778 \uce74\uc2a4\ud22c\ub974\ubc14\uc774 \ub9c8\uce78\uc9c0\uc640 13\uc138\uc5d0 \uacb0\ud63c\ud55c\ub2e4. \uac04\ub514\ub294 \uc544\ub0b4\uc640 \uc7a0\uc790\ub9ac\ub97c \uac00\uc9c0\ub290\ub77c \uc544\ubc84\uc9c0\uc758 \uc784\uc885\uc744 \uc9c0\ud0a4\uc9c0 \ubabb\ud588\ub2e4\uace0 \ud558\ub294\ub370, \uc774 \uc0ac\uac74 \ub54c\ubb38\uc5d0 \uadf8\ub294 \uae4a\uc774 \ubc18\uc131\ud558\uace0 \uae08\uc695\uc5d0 \ub300\ud55c \uc0dd\uac01\uc744 \uac16\uac8c \ub41c\ub2e4. \uadf8 \ub4a4 \ubc29\ud0d5\ud55c \uc0dd\ud65c\uc744 \ubc84\ub9ac\uace0 \ud78c\ub450\uad50\uc758 \uad50\ub9ac\ub97c \ucda9\uc2e4\ud788 \uc9c0\ud0a4\uba70 \uc131\uc695\uc744 \uc790\uc81c\ud558\uae30\ub3c4 \ud588\ub2e4. \uadf8\ub294 \uc815\uc561\uc740 \uc0dd\uba85\ub825\uc758 \uadfc\uc6d0\uc774\uace0 \uc815\uc561\uc758 \ubc30\ucd9c\uc740 \ubab8\uacfc \uba38\ub9ac\ub97c \uc57d\ud558\uac8c \ub9cc\ub4e0\ub2e4\uace0 \ubbff\uc5c8\ub2e4. \uadf8\ub4a4\ub85c \uc77c\uc0dd \ub3d9\uc548 \uc218\ub9ce\uc740 \uc5ec\uc131\uc758 \uc0ac\ub791\uc744 \ubc1b\uace0 \uadf8\ub4e4\uc744 \uac00\uae4c\uc774 \ub450\uc5c8\uc74c\uc5d0\ub3c4 \uc9c1\uc811\uc801\uc778 \uc131\ud589\uc704\ub098 \uc0ac\uc815\uc740 \uc790\uc81c\ud558\uc600\ub2e4 \ud55c\ub2e4. \uadf8\ub294 \ubcf4\uc5b4 \uc804\uc7c1\uacfc \uc544\ud504\ub9ac\uce74 \uc804\uc7c1 \uadf8\ub9ac\uace0 \uc81c1\ucc28 \uc138\uacc4\ub300\uc804\uc5d0 \ucc38\uc804\ud558\ub824\uace0 \uc601\uad6d\uad70\uc5d0 \uc9c0\uc6d0\ud588\uc73c\ub098 \ucc38\uc804\ud558\uae30 \uba70\uce60 \uc804 \uacb0\ud575 \uc5d0 \uc758\ud55c \uc5fc\uc99d\uc5d0 \uac78\ub824\uc11c \ud3ec\uae30\ud574\uc57c\ub9cc \ud588\ub2e4.", "sentence_answer": "\uadf8\ub294 \ubcf4\uc5b4 \uc804\uc7c1\uacfc \uc544\ud504\ub9ac\uce74 \uc804\uc7c1 \uadf8\ub9ac\uace0 \uc81c1\ucc28 \uc138\uacc4\ub300\uc804\uc5d0 \ucc38\uc804\ud558\ub824\uace0 \uc601\uad6d\uad70\uc5d0 \uc9c0\uc6d0\ud588\uc73c\ub098 \ucc38\uc804\ud558\uae30 \uba70\uce60 \uc804 \uacb0\ud575 \uc5d0 \uc758\ud55c \uc5fc\uc99d\uc5d0 \uac78\ub824\uc11c \ud3ec\uae30\ud574\uc57c\ub9cc \ud588\ub2e4.", "paragraph_id": "6657017-0-2", "paragraph_question": "question: \uac04\ub514\uac00 \uc81c1\ucc28 \uc138\uacc4\ub300\uc804\uc5d0 \ucc38\uc804\ud558\uc9c0 \ubabb\ud558\uac8c \ub41c \uac83\uc740 \ubb34\uc5c7\uc5d0 \uc758\ud55c \uc5fc\uc99d\uc5d0 \uac78\ub838\uae30 \ub54c\ubb38\uc778\uac00?, context: 7\uc0b4 \ub54c \uc774\ubbf8 \uadf8\ub294 \uc57d\ud63c\uc744 \ud588\uc5c8\uc9c0\ub9cc \uc57d\ud63c\ub140\ub4e4\uc774 \ubaa8\ub450 \ucc28\ub840\ub85c \uc8fd\uc5b4, \uc138 \ubc88\uc9f8 \uc57d\ud63c\ub140\uc778 \uc0c1\uc778 \uc9d1\uc548\uc758 \ub538\uc778 \uce74\uc2a4\ud22c\ub974\ubc14\uc774 \ub9c8\uce78\uc9c0\uc640 13\uc138\uc5d0 \uacb0\ud63c\ud55c\ub2e4. \uac04\ub514\ub294 \uc544\ub0b4\uc640 \uc7a0\uc790\ub9ac\ub97c \uac00\uc9c0\ub290\ub77c \uc544\ubc84\uc9c0\uc758 \uc784\uc885\uc744 \uc9c0\ud0a4\uc9c0 \ubabb\ud588\ub2e4\uace0 \ud558\ub294\ub370, \uc774 \uc0ac\uac74 \ub54c\ubb38\uc5d0 \uadf8\ub294 \uae4a\uc774 \ubc18\uc131\ud558\uace0 \uae08\uc695\uc5d0 \ub300\ud55c \uc0dd\uac01\uc744 \uac16\uac8c \ub41c\ub2e4. \uadf8 \ub4a4 \ubc29\ud0d5\ud55c \uc0dd\ud65c\uc744 \ubc84\ub9ac\uace0 \ud78c\ub450\uad50\uc758 \uad50\ub9ac\ub97c \ucda9\uc2e4\ud788 \uc9c0\ud0a4\uba70 \uc131\uc695\uc744 \uc790\uc81c\ud558\uae30\ub3c4 \ud588\ub2e4. \uadf8\ub294 \uc815\uc561\uc740 \uc0dd\uba85\ub825\uc758 \uadfc\uc6d0\uc774\uace0 \uc815\uc561\uc758 \ubc30\ucd9c\uc740 \ubab8\uacfc \uba38\ub9ac\ub97c \uc57d\ud558\uac8c \ub9cc\ub4e0\ub2e4\uace0 \ubbff\uc5c8\ub2e4. \uadf8\ub4a4\ub85c \uc77c\uc0dd \ub3d9\uc548 \uc218\ub9ce\uc740 \uc5ec\uc131\uc758 \uc0ac\ub791\uc744 \ubc1b\uace0 \uadf8\ub4e4\uc744 \uac00\uae4c\uc774 \ub450\uc5c8\uc74c\uc5d0\ub3c4 \uc9c1\uc811\uc801\uc778 \uc131\ud589\uc704\ub098 \uc0ac\uc815\uc740 \uc790\uc81c\ud558\uc600\ub2e4 \ud55c\ub2e4. \uadf8\ub294 \ubcf4\uc5b4 \uc804\uc7c1\uacfc \uc544\ud504\ub9ac\uce74 \uc804\uc7c1 \uadf8\ub9ac\uace0 \uc81c1\ucc28 \uc138\uacc4\ub300\uc804\uc5d0 \ucc38\uc804\ud558\ub824\uace0 \uc601\uad6d\uad70\uc5d0 \uc9c0\uc6d0\ud588\uc73c\ub098 \ucc38\uc804\ud558\uae30 \uba70\uce60 \uc804 \uacb0\ud575\uc5d0 \uc758\ud55c \uc5fc\uc99d\uc5d0 \uac78\ub824\uc11c \ud3ec\uae30\ud574\uc57c\ub9cc \ud588\ub2e4."} {"question": "1720\ub144 \uc0ac\ubcf4\uc774 \uacf5\uad6d\uc774 \uc0ac\ub974\ub370\ub0d0 \uc12c\uc744 \uc5bb\uc740 \ud6c4 \uc815\ud55c \uad6d\uba85\uc740?", "paragraph": "1720\ub144 \uc0ac\ubcf4\uc774 \uacf5\uad6d\uc758 \uc655\uac00\uac00 \uc5d0\uc2a4\ud30c\ub0d0 \uc655\uc704 \uacc4\uc2b9 \uc804\uc7c1 \uacb0\uacfc \uc2b9\ub9ac\ud558\uc5ec \uc0ac\ub974\ub370\ub0d0 \uc12c\uc744 \uc5bb\uc740 \ud6c4 \uad6d\uba85\uc744 \uc0ac\ub974\ub370\ub0d0 \uc655\uad6d\uc73c\ub85c \uc815\ud588\ub2e4. \ud504\ub791\uc2a4 \ud601\uba85(1789)\uc774 \uc77c\uc5b4\ub098\uace0, 1793\ub144 1\uc6d4 \ub8e8\uc774 16\uc138\uac00 \ucc98\ud615\ub2f9\ud55c \ub4a4\uc758 \ud504\ub791\uc2a4\uc758 \ub300\uc678\ud33d\ucc3d\uc758 \uae38\uc744 \uac77\uac8c \ub418\uc5b4, \ud504\ub791\uc2a4\ub294 \ud604\uc7ac\uc758 \uc0ac\ubd80\uc544 \uc8fc\ub97c \ud569\ubcd1\ud558\uc600\ub2e4. \uc0ac\ub974\ub370\ub0d0 \uc655\uad6d\uc740 \uc789\uae00\ub79c\ub4dc\uac00 \uc8fc\ub3c4\ud558\ub294 \uc81c1\ucc28 \ub300\ud504\ub791\uc2a4 \ub3d9\ub9f9(1793)\uc5d0 \uac00\ub2f4\ud558\uc5ec, \uac19\uc740 \ud574 \uad6d\uacbd\uc744 \ub118\uc5b4 \ud504\ub791\uc2a4\ub85c \uc9c4\uaca9\ud558\uae30\ub3c4 \ud558\uc600\ub2e4. \uadf8\ub7ec\ub098, \ub098\ud3f4\ub808\uc639 \ubcf4\ub098\ud30c\ub974\ud2b8\uac00 \uc774\ud0c8\ub9ac\uc544 \uc6d0\uc815\uc744 \uac10\ud589(1796)\ud560 \ub2f9\uc2dc, \uadf8\ub54c\uae4c\uc9c0 \ucd5c\uc804\uc120\uc5d0\uc11c \ud504\ub791\uc2a4\uad70\uacfc \ub300\uce58 \ud574\uc654\ub358 \uc0ac\ub974\ub370\ub0d0 \uc655\uad6d\uc740 \uc57d 1\uac1c\uc6d4 \ub9cc\uc5d0 \ud56d\ubcf5\ud558\uc600\ub2e4. \uadf8 \ud6c4 \ub098\ud3f4\ub808\uc639 \uc804\uc7c1\ub54c \uad6d\ud1a0\uac00 \uc810\ub839\ub2f9\ud558\uc5ec \uc0ac\ub974\ub370\ub0d0\uc12c\uc758 \uce7c\ub9ac\uc544\ub9ac\ub85c \uc218\ub3c4\ub97c \uc62e\uacbc\uc5c8\ub2e4. \uadf8 \ud6c4 \ube48\uc870\uc57d \uc73c\ub85c \ud504\ub791\uc2a4\uc758 \uc810\ub839\uc774 \ub05d\ub098\uace0 \uad6d\ud1a0\uac00 \ud68c\ubcf5\ub418\uc790 \ub2e4\uc2dc \uc218\ub3c4\ub97c \ud1a0\ub9ac\ub178\ub85c \uc62e\uacbc\ub2e4.", "answer": "\uc0ac\ub974\ub370\ub0d0 \uc655\uad6d", "sentence": "1720\ub144 \uc0ac\ubcf4\uc774 \uacf5\uad6d\uc758 \uc655\uac00\uac00 \uc5d0\uc2a4\ud30c\ub0d0 \uc655\uc704 \uacc4\uc2b9 \uc804\uc7c1 \uacb0\uacfc \uc2b9\ub9ac\ud558\uc5ec \uc0ac\ub974\ub370\ub0d0 \uc12c\uc744 \uc5bb\uc740 \ud6c4 \uad6d\uba85\uc744 \uc0ac\ub974\ub370\ub0d0 \uc655\uad6d \uc73c\ub85c \uc815\ud588\ub2e4.", "paragraph_sentence": " 1720\ub144 \uc0ac\ubcf4\uc774 \uacf5\uad6d\uc758 \uc655\uac00\uac00 \uc5d0\uc2a4\ud30c\ub0d0 \uc655\uc704 \uacc4\uc2b9 \uc804\uc7c1 \uacb0\uacfc \uc2b9\ub9ac\ud558\uc5ec \uc0ac\ub974\ub370\ub0d0 \uc12c\uc744 \uc5bb\uc740 \ud6c4 \uad6d\uba85\uc744 \uc0ac\ub974\ub370\ub0d0 \uc655\uad6d \uc73c\ub85c \uc815\ud588\ub2e4. \ud504\ub791\uc2a4 \ud601\uba85(1789)\uc774 \uc77c\uc5b4\ub098\uace0, 1793\ub144 1\uc6d4 \ub8e8\uc774 16\uc138\uac00 \ucc98\ud615\ub2f9\ud55c \ub4a4\uc758 \ud504\ub791\uc2a4\uc758 \ub300\uc678\ud33d\ucc3d\uc758 \uae38\uc744 \uac77\uac8c \ub418\uc5b4, \ud504\ub791\uc2a4\ub294 \ud604\uc7ac\uc758 \uc0ac\ubd80\uc544 \uc8fc\ub97c \ud569\ubcd1\ud558\uc600\ub2e4. \uc0ac\ub974\ub370\ub0d0 \uc655\uad6d\uc740 \uc789\uae00\ub79c\ub4dc\uac00 \uc8fc\ub3c4\ud558\ub294 \uc81c1\ucc28 \ub300\ud504\ub791\uc2a4 \ub3d9\ub9f9(1793)\uc5d0 \uac00\ub2f4\ud558\uc5ec, \uac19\uc740 \ud574 \uad6d\uacbd\uc744 \ub118\uc5b4 \ud504\ub791\uc2a4\ub85c \uc9c4\uaca9\ud558\uae30\ub3c4 \ud558\uc600\ub2e4. \uadf8\ub7ec\ub098, \ub098\ud3f4\ub808\uc639 \ubcf4\ub098\ud30c\ub974\ud2b8\uac00 \uc774\ud0c8\ub9ac\uc544 \uc6d0\uc815\uc744 \uac10\ud589(1796)\ud560 \ub2f9\uc2dc, \uadf8\ub54c\uae4c\uc9c0 \ucd5c\uc804\uc120\uc5d0\uc11c \ud504\ub791\uc2a4\uad70\uacfc \ub300\uce58 \ud574\uc654\ub358 \uc0ac\ub974\ub370\ub0d0 \uc655\uad6d\uc740 \uc57d 1\uac1c\uc6d4 \ub9cc\uc5d0 \ud56d\ubcf5\ud558\uc600\ub2e4. \uadf8 \ud6c4 \ub098\ud3f4\ub808\uc639 \uc804\uc7c1\ub54c \uad6d\ud1a0\uac00 \uc810\ub839\ub2f9\ud558\uc5ec \uc0ac\ub974\ub370\ub0d0\uc12c\uc758 \uce7c\ub9ac\uc544\ub9ac\ub85c \uc218\ub3c4\ub97c \uc62e\uacbc\uc5c8\ub2e4. \uadf8 \ud6c4 \ube48\uc870\uc57d \uc73c\ub85c \ud504\ub791\uc2a4\uc758 \uc810\ub839\uc774 \ub05d\ub098\uace0 \uad6d\ud1a0\uac00 \ud68c\ubcf5\ub418\uc790 \ub2e4\uc2dc \uc218\ub3c4\ub97c \ud1a0\ub9ac\ub178\ub85c \uc62e\uacbc\ub2e4.", "paragraph_answer": "1720\ub144 \uc0ac\ubcf4\uc774 \uacf5\uad6d\uc758 \uc655\uac00\uac00 \uc5d0\uc2a4\ud30c\ub0d0 \uc655\uc704 \uacc4\uc2b9 \uc804\uc7c1 \uacb0\uacfc \uc2b9\ub9ac\ud558\uc5ec \uc0ac\ub974\ub370\ub0d0 \uc12c\uc744 \uc5bb\uc740 \ud6c4 \uad6d\uba85\uc744 \uc0ac\ub974\ub370\ub0d0 \uc655\uad6d \uc73c\ub85c \uc815\ud588\ub2e4. \ud504\ub791\uc2a4 \ud601\uba85(1789)\uc774 \uc77c\uc5b4\ub098\uace0, 1793\ub144 1\uc6d4 \ub8e8\uc774 16\uc138\uac00 \ucc98\ud615\ub2f9\ud55c \ub4a4\uc758 \ud504\ub791\uc2a4\uc758 \ub300\uc678\ud33d\ucc3d\uc758 \uae38\uc744 \uac77\uac8c \ub418\uc5b4, \ud504\ub791\uc2a4\ub294 \ud604\uc7ac\uc758 \uc0ac\ubd80\uc544 \uc8fc\ub97c \ud569\ubcd1\ud558\uc600\ub2e4. \uc0ac\ub974\ub370\ub0d0 \uc655\uad6d\uc740 \uc789\uae00\ub79c\ub4dc\uac00 \uc8fc\ub3c4\ud558\ub294 \uc81c1\ucc28 \ub300\ud504\ub791\uc2a4 \ub3d9\ub9f9(1793)\uc5d0 \uac00\ub2f4\ud558\uc5ec, \uac19\uc740 \ud574 \uad6d\uacbd\uc744 \ub118\uc5b4 \ud504\ub791\uc2a4\ub85c \uc9c4\uaca9\ud558\uae30\ub3c4 \ud558\uc600\ub2e4. \uadf8\ub7ec\ub098, \ub098\ud3f4\ub808\uc639 \ubcf4\ub098\ud30c\ub974\ud2b8\uac00 \uc774\ud0c8\ub9ac\uc544 \uc6d0\uc815\uc744 \uac10\ud589(1796)\ud560 \ub2f9\uc2dc, \uadf8\ub54c\uae4c\uc9c0 \ucd5c\uc804\uc120\uc5d0\uc11c \ud504\ub791\uc2a4\uad70\uacfc \ub300\uce58 \ud574\uc654\ub358 \uc0ac\ub974\ub370\ub0d0 \uc655\uad6d\uc740 \uc57d 1\uac1c\uc6d4 \ub9cc\uc5d0 \ud56d\ubcf5\ud558\uc600\ub2e4. \uadf8 \ud6c4 \ub098\ud3f4\ub808\uc639 \uc804\uc7c1\ub54c \uad6d\ud1a0\uac00 \uc810\ub839\ub2f9\ud558\uc5ec \uc0ac\ub974\ub370\ub0d0\uc12c\uc758 \uce7c\ub9ac\uc544\ub9ac\ub85c \uc218\ub3c4\ub97c \uc62e\uacbc\uc5c8\ub2e4. \uadf8 \ud6c4 \ube48\uc870\uc57d \uc73c\ub85c \ud504\ub791\uc2a4\uc758 \uc810\ub839\uc774 \ub05d\ub098\uace0 \uad6d\ud1a0\uac00 \ud68c\ubcf5\ub418\uc790 \ub2e4\uc2dc \uc218\ub3c4\ub97c \ud1a0\ub9ac\ub178\ub85c \uc62e\uacbc\ub2e4.", "sentence_answer": "1720\ub144 \uc0ac\ubcf4\uc774 \uacf5\uad6d\uc758 \uc655\uac00\uac00 \uc5d0\uc2a4\ud30c\ub0d0 \uc655\uc704 \uacc4\uc2b9 \uc804\uc7c1 \uacb0\uacfc \uc2b9\ub9ac\ud558\uc5ec \uc0ac\ub974\ub370\ub0d0 \uc12c\uc744 \uc5bb\uc740 \ud6c4 \uad6d\uba85\uc744 \uc0ac\ub974\ub370\ub0d0 \uc655\uad6d \uc73c\ub85c \uc815\ud588\ub2e4.", "paragraph_id": "6568624-6-1", "paragraph_question": "question: 1720\ub144 \uc0ac\ubcf4\uc774 \uacf5\uad6d\uc774 \uc0ac\ub974\ub370\ub0d0 \uc12c\uc744 \uc5bb\uc740 \ud6c4 \uc815\ud55c \uad6d\uba85\uc740?, context: 1720\ub144 \uc0ac\ubcf4\uc774 \uacf5\uad6d\uc758 \uc655\uac00\uac00 \uc5d0\uc2a4\ud30c\ub0d0 \uc655\uc704 \uacc4\uc2b9 \uc804\uc7c1 \uacb0\uacfc \uc2b9\ub9ac\ud558\uc5ec \uc0ac\ub974\ub370\ub0d0 \uc12c\uc744 \uc5bb\uc740 \ud6c4 \uad6d\uba85\uc744 \uc0ac\ub974\ub370\ub0d0 \uc655\uad6d\uc73c\ub85c \uc815\ud588\ub2e4. \ud504\ub791\uc2a4 \ud601\uba85(1789)\uc774 \uc77c\uc5b4\ub098\uace0, 1793\ub144 1\uc6d4 \ub8e8\uc774 16\uc138\uac00 \ucc98\ud615\ub2f9\ud55c \ub4a4\uc758 \ud504\ub791\uc2a4\uc758 \ub300\uc678\ud33d\ucc3d\uc758 \uae38\uc744 \uac77\uac8c \ub418\uc5b4, \ud504\ub791\uc2a4\ub294 \ud604\uc7ac\uc758 \uc0ac\ubd80\uc544 \uc8fc\ub97c \ud569\ubcd1\ud558\uc600\ub2e4. \uc0ac\ub974\ub370\ub0d0 \uc655\uad6d\uc740 \uc789\uae00\ub79c\ub4dc\uac00 \uc8fc\ub3c4\ud558\ub294 \uc81c1\ucc28 \ub300\ud504\ub791\uc2a4 \ub3d9\ub9f9(1793)\uc5d0 \uac00\ub2f4\ud558\uc5ec, \uac19\uc740 \ud574 \uad6d\uacbd\uc744 \ub118\uc5b4 \ud504\ub791\uc2a4\ub85c \uc9c4\uaca9\ud558\uae30\ub3c4 \ud558\uc600\ub2e4. \uadf8\ub7ec\ub098, \ub098\ud3f4\ub808\uc639 \ubcf4\ub098\ud30c\ub974\ud2b8\uac00 \uc774\ud0c8\ub9ac\uc544 \uc6d0\uc815\uc744 \uac10\ud589(1796)\ud560 \ub2f9\uc2dc, \uadf8\ub54c\uae4c\uc9c0 \ucd5c\uc804\uc120\uc5d0\uc11c \ud504\ub791\uc2a4\uad70\uacfc \ub300\uce58 \ud574\uc654\ub358 \uc0ac\ub974\ub370\ub0d0 \uc655\uad6d\uc740 \uc57d 1\uac1c\uc6d4 \ub9cc\uc5d0 \ud56d\ubcf5\ud558\uc600\ub2e4. \uadf8 \ud6c4 \ub098\ud3f4\ub808\uc639 \uc804\uc7c1\ub54c \uad6d\ud1a0\uac00 \uc810\ub839\ub2f9\ud558\uc5ec \uc0ac\ub974\ub370\ub0d0\uc12c\uc758 \uce7c\ub9ac\uc544\ub9ac\ub85c \uc218\ub3c4\ub97c \uc62e\uacbc\uc5c8\ub2e4. \uadf8 \ud6c4 \ube48\uc870\uc57d \uc73c\ub85c \ud504\ub791\uc2a4\uc758 \uc810\ub839\uc774 \ub05d\ub098\uace0 \uad6d\ud1a0\uac00 \ud68c\ubcf5\ub418\uc790 \ub2e4\uc2dc \uc218\ub3c4\ub97c \ud1a0\ub9ac\ub178\ub85c \uc62e\uacbc\ub2e4."} {"question": "\uc704\ub839\ube44\uac00 \ud788\ub85c\uc2dc\ub9c8 \ud3c9\ud654\uae30\ub150\uacf5\uc6d0\uc73c\ub85c \uc774\uc804\ub41c \uc5f0\ub3c4\ub294?", "paragraph": "1970\ub144 4\uc6d4 10\uc77c\uc5d0 \uc7ac\uc77c\ubcf8\ub300\ud55c\ubbfc\uad6d\uac70\ub958\ubbfc\ub2e8 \ud788\ub85c\uc2dc\ub9c8 \ud604 \ubcf8\ubd80\uc5d0 \uc758\ud574 \u2018\ud55c\uad6d\uc778 \uc6d0\ud3ed \ud76c\uc0dd\uc790 \uc704\ub839\ube44\u2019\uac00 \uac74\ub9bd\ub418\uc5c8\ub2e4. \ud788\ub85c\uc2dc\ub9c8 \uc2dc\uc758 \ubc18\ub300\ub85c \uc704\ub839\ube44\uc758 \ud788\ub85c\uc2dc\ub9c8 \ud3c9\ud654\uae30\ub150\uacf5\uc6d0 \ub0b4 \uac74\ub9bd\uc774 \uc5b4\ub824\uc6cc\uc9c0\uc790 \uc774\uc6b0\uac00 \ud53c\ud3ed\uc744 \ub2f9\ud55c \ud6c4 \ubc1c\uacac\ub41c \uc7a5\uc18c\uc600\ub358 \ud788\ub85c\uc2dc\ub9c8 \ud63c\uac00\uc640\uc758 \uc544\uc774\uc624\uc774 \uad50 \uadfc\ucc98\uc5d0 \uc138\uc6cc\uc84c\ub2e4. \uc704\ub839\ube44\uc758 \uc804\uba74\uc5d0 \u2018\ud55c\uad6d\uc778 \uc6d0\ud3ed \ud76c\uc0dd\uc790 \uc704\ub839\ube44\u2019\uc640 \ud568\uaed8 \u2018\uc774\uc6b0\uacf5 \uc804\ud558 \uc678 2\ub9cc\uc5ec \uc601\uc704\u2019\ub77c\ub294 \ubb38\uad6c\uac00 \uc801\ud600\uc788\uace0, \ub4b7\uba74\uc758 \ube44\ubb38\uc5d0\ub294 \u2018\ub098\ub77c \uc783\uc740 \uc655\uc190\uc774\uae30\uc5d0 \ub0a8\ubaa8\ub97c \uc124\uc6c0\uacfc \uace0\ub09c\uc774 \ud55c\uce35 \ub354 \ud588\ub358 \uc774\uc6b0\uacf5 \uc804\ud558\ub97c \ube44\ub86f\ud558\uc5ec \u2026 \uc774\uc6b0\uacf5 \uc804\ud558 \uc678 \ubb34\uace0\ud55c \ub3d9\ud3ec 2\ub9cc\uc5ec \uc704 \u2026\u2019\ub77c\ub294 \ubb38\uad6c\uac00 \uc0c8\uaca8\uc84c\ub2e4. 1999\ub144 7\uc6d4 21\uc77c\uc5d0 \uc704\ub839\ube44\ub294 \ud788\ub85c\uc2dc\ub9c8 \ud3c9\ud654\uae30\ub150\uacf5\uc6d0 \uc548\uc73c\ub85c \uc774\uc804\ub418\uc5c8\ub2e4.", "answer": "1999\ub144", "sentence": "1999\ub144 7\uc6d4 21\uc77c\uc5d0 \uc704\ub839\ube44\ub294 \ud788\ub85c\uc2dc\ub9c8 \ud3c9\ud654\uae30\ub150\uacf5\uc6d0 \uc548\uc73c\ub85c \uc774\uc804\ub418\uc5c8\ub2e4.", "paragraph_sentence": "1970\ub144 4\uc6d4 10\uc77c\uc5d0 \uc7ac\uc77c\ubcf8\ub300\ud55c\ubbfc\uad6d\uac70\ub958\ubbfc\ub2e8 \ud788\ub85c\uc2dc\ub9c8 \ud604 \ubcf8\ubd80\uc5d0 \uc758\ud574 \u2018\ud55c\uad6d\uc778 \uc6d0\ud3ed \ud76c\uc0dd\uc790 \uc704\ub839\ube44\u2019\uac00 \uac74\ub9bd\ub418\uc5c8\ub2e4. \ud788\ub85c\uc2dc\ub9c8 \uc2dc\uc758 \ubc18\ub300\ub85c \uc704\ub839\ube44\uc758 \ud788\ub85c\uc2dc\ub9c8 \ud3c9\ud654\uae30\ub150\uacf5\uc6d0 \ub0b4 \uac74\ub9bd\uc774 \uc5b4\ub824\uc6cc\uc9c0\uc790 \uc774\uc6b0\uac00 \ud53c\ud3ed\uc744 \ub2f9\ud55c \ud6c4 \ubc1c\uacac\ub41c \uc7a5\uc18c\uc600\ub358 \ud788\ub85c\uc2dc\ub9c8 \ud63c\uac00\uc640\uc758 \uc544\uc774\uc624\uc774 \uad50 \uadfc\ucc98\uc5d0 \uc138\uc6cc\uc84c\ub2e4. \uc704\ub839\ube44\uc758 \uc804\uba74\uc5d0 \u2018\ud55c\uad6d\uc778 \uc6d0\ud3ed \ud76c\uc0dd\uc790 \uc704\ub839\ube44\u2019\uc640 \ud568\uaed8 \u2018\uc774\uc6b0\uacf5 \uc804\ud558 \uc678 2\ub9cc\uc5ec \uc601\uc704\u2019\ub77c\ub294 \ubb38\uad6c\uac00 \uc801\ud600\uc788\uace0, \ub4b7\uba74\uc758 \ube44\ubb38\uc5d0\ub294 \u2018\ub098\ub77c \uc783\uc740 \uc655\uc190\uc774\uae30\uc5d0 \ub0a8\ubaa8\ub97c \uc124\uc6c0\uacfc \uace0\ub09c\uc774 \ud55c\uce35 \ub354 \ud588\ub358 \uc774\uc6b0\uacf5 \uc804\ud558\ub97c \ube44\ub86f\ud558\uc5ec \u2026 \uc774\uc6b0\uacf5 \uc804\ud558 \uc678 \ubb34\uace0\ud55c \ub3d9\ud3ec 2\ub9cc\uc5ec \uc704 \u2026\u2019\ub77c\ub294 \ubb38\uad6c\uac00 \uc0c8\uaca8\uc84c\ub2e4. 1999\ub144 7\uc6d4 21\uc77c\uc5d0 \uc704\ub839\ube44\ub294 \ud788\ub85c\uc2dc\ub9c8 \ud3c9\ud654\uae30\ub150\uacf5\uc6d0 \uc548\uc73c\ub85c \uc774\uc804\ub418\uc5c8\ub2e4. ", "paragraph_answer": "1970\ub144 4\uc6d4 10\uc77c\uc5d0 \uc7ac\uc77c\ubcf8\ub300\ud55c\ubbfc\uad6d\uac70\ub958\ubbfc\ub2e8 \ud788\ub85c\uc2dc\ub9c8 \ud604 \ubcf8\ubd80\uc5d0 \uc758\ud574 \u2018\ud55c\uad6d\uc778 \uc6d0\ud3ed \ud76c\uc0dd\uc790 \uc704\ub839\ube44\u2019\uac00 \uac74\ub9bd\ub418\uc5c8\ub2e4. \ud788\ub85c\uc2dc\ub9c8 \uc2dc\uc758 \ubc18\ub300\ub85c \uc704\ub839\ube44\uc758 \ud788\ub85c\uc2dc\ub9c8 \ud3c9\ud654\uae30\ub150\uacf5\uc6d0 \ub0b4 \uac74\ub9bd\uc774 \uc5b4\ub824\uc6cc\uc9c0\uc790 \uc774\uc6b0\uac00 \ud53c\ud3ed\uc744 \ub2f9\ud55c \ud6c4 \ubc1c\uacac\ub41c \uc7a5\uc18c\uc600\ub358 \ud788\ub85c\uc2dc\ub9c8 \ud63c\uac00\uc640\uc758 \uc544\uc774\uc624\uc774 \uad50 \uadfc\ucc98\uc5d0 \uc138\uc6cc\uc84c\ub2e4. \uc704\ub839\ube44\uc758 \uc804\uba74\uc5d0 \u2018\ud55c\uad6d\uc778 \uc6d0\ud3ed \ud76c\uc0dd\uc790 \uc704\ub839\ube44\u2019\uc640 \ud568\uaed8 \u2018\uc774\uc6b0\uacf5 \uc804\ud558 \uc678 2\ub9cc\uc5ec \uc601\uc704\u2019\ub77c\ub294 \ubb38\uad6c\uac00 \uc801\ud600\uc788\uace0, \ub4b7\uba74\uc758 \ube44\ubb38\uc5d0\ub294 \u2018\ub098\ub77c \uc783\uc740 \uc655\uc190\uc774\uae30\uc5d0 \ub0a8\ubaa8\ub97c \uc124\uc6c0\uacfc \uace0\ub09c\uc774 \ud55c\uce35 \ub354 \ud588\ub358 \uc774\uc6b0\uacf5 \uc804\ud558\ub97c \ube44\ub86f\ud558\uc5ec \u2026 \uc774\uc6b0\uacf5 \uc804\ud558 \uc678 \ubb34\uace0\ud55c \ub3d9\ud3ec 2\ub9cc\uc5ec \uc704 \u2026\u2019\ub77c\ub294 \ubb38\uad6c\uac00 \uc0c8\uaca8\uc84c\ub2e4. 1999\ub144 7\uc6d4 21\uc77c\uc5d0 \uc704\ub839\ube44\ub294 \ud788\ub85c\uc2dc\ub9c8 \ud3c9\ud654\uae30\ub150\uacf5\uc6d0 \uc548\uc73c\ub85c \uc774\uc804\ub418\uc5c8\ub2e4.", "sentence_answer": " 1999\ub144 7\uc6d4 21\uc77c\uc5d0 \uc704\ub839\ube44\ub294 \ud788\ub85c\uc2dc\ub9c8 \ud3c9\ud654\uae30\ub150\uacf5\uc6d0 \uc548\uc73c\ub85c \uc774\uc804\ub418\uc5c8\ub2e4.", "paragraph_id": "6545847-12-2", "paragraph_question": "question: \uc704\ub839\ube44\uac00 \ud788\ub85c\uc2dc\ub9c8 \ud3c9\ud654\uae30\ub150\uacf5\uc6d0\uc73c\ub85c \uc774\uc804\ub41c \uc5f0\ub3c4\ub294?, context: 1970\ub144 4\uc6d4 10\uc77c\uc5d0 \uc7ac\uc77c\ubcf8\ub300\ud55c\ubbfc\uad6d\uac70\ub958\ubbfc\ub2e8 \ud788\ub85c\uc2dc\ub9c8 \ud604 \ubcf8\ubd80\uc5d0 \uc758\ud574 \u2018\ud55c\uad6d\uc778 \uc6d0\ud3ed \ud76c\uc0dd\uc790 \uc704\ub839\ube44\u2019\uac00 \uac74\ub9bd\ub418\uc5c8\ub2e4. \ud788\ub85c\uc2dc\ub9c8 \uc2dc\uc758 \ubc18\ub300\ub85c \uc704\ub839\ube44\uc758 \ud788\ub85c\uc2dc\ub9c8 \ud3c9\ud654\uae30\ub150\uacf5\uc6d0 \ub0b4 \uac74\ub9bd\uc774 \uc5b4\ub824\uc6cc\uc9c0\uc790 \uc774\uc6b0\uac00 \ud53c\ud3ed\uc744 \ub2f9\ud55c \ud6c4 \ubc1c\uacac\ub41c \uc7a5\uc18c\uc600\ub358 \ud788\ub85c\uc2dc\ub9c8 \ud63c\uac00\uc640\uc758 \uc544\uc774\uc624\uc774 \uad50 \uadfc\ucc98\uc5d0 \uc138\uc6cc\uc84c\ub2e4. \uc704\ub839\ube44\uc758 \uc804\uba74\uc5d0 \u2018\ud55c\uad6d\uc778 \uc6d0\ud3ed \ud76c\uc0dd\uc790 \uc704\ub839\ube44\u2019\uc640 \ud568\uaed8 \u2018\uc774\uc6b0\uacf5 \uc804\ud558 \uc678 2\ub9cc\uc5ec \uc601\uc704\u2019\ub77c\ub294 \ubb38\uad6c\uac00 \uc801\ud600\uc788\uace0, \ub4b7\uba74\uc758 \ube44\ubb38\uc5d0\ub294 \u2018\ub098\ub77c \uc783\uc740 \uc655\uc190\uc774\uae30\uc5d0 \ub0a8\ubaa8\ub97c \uc124\uc6c0\uacfc \uace0\ub09c\uc774 \ud55c\uce35 \ub354 \ud588\ub358 \uc774\uc6b0\uacf5 \uc804\ud558\ub97c \ube44\ub86f\ud558\uc5ec \u2026 \uc774\uc6b0\uacf5 \uc804\ud558 \uc678 \ubb34\uace0\ud55c \ub3d9\ud3ec 2\ub9cc\uc5ec \uc704 \u2026\u2019\ub77c\ub294 \ubb38\uad6c\uac00 \uc0c8\uaca8\uc84c\ub2e4. 1999\ub144 7\uc6d4 21\uc77c\uc5d0 \uc704\ub839\ube44\ub294 \ud788\ub85c\uc2dc\ub9c8 \ud3c9\ud654\uae30\ub150\uacf5\uc6d0 \uc548\uc73c\ub85c \uc774\uc804\ub418\uc5c8\ub2e4."} {"question": "\uc5ec\uc131\uc778\ub825\uac1c\ubc1c\uc5f0\uad6c\uc6d0\uc774 \ubc1c\uc871\ub41c \uc2dc\uae30\ub294 \uc5b8\uc81c\uc778\uac00?", "paragraph": "\ucd1d\uc5ec\ud559\uc0dd\ud68c\ub294 \uc5ec\uc131 \ubb38\uc81c\ub97c \uace0\ubbfc\ud558\uace0 \ub0a8\uc131\uc911\uc2ec\uc801 \ub300\ud559 \ubb38\ud654\ub97c \uc5ec\uc131\uc8fc\uc758\uc801\uc73c\ub85c \uc7ac\uad6c\uc131\ud558\uae30 \uc704\ud574\uc11c \ud65c\ub3d9\ud558\uace0 \uc788\ub2e4. \ucd1d\uc5ec\ud559\uc0dd\ud68c\ub294 1988\ub144 3\uc6d4\uc5d0 \uae30\uc874 \ucd1d\ud559\uc0dd\ud68c \uc0b0\ud558 \uc5ec\ud559\uc0dd\ubd80\uc758 \ud55c\uacc4\uc640 \ud559\ub0b4\uc5d0\uc11c \uc18c\uc218\uc758 \uc704\uce58\uc5d0 \uc788\ub358 \uc5ec\ud559\uc0dd \ubb38\uc81c\ub97c \uadf9\ubcf5\ud558\uace0\uc790 \ubc1c\uc871\ud558\uc600\ub2e4. \ub9e4\ub144 \uc120\uac70\ub97c \ud1b5\ud574 \ucd1d\uc5ec\ud559\uc0dd\ud68c\uac00 \uc120\ucd9c\ub418\uace0 \uc788\uc73c\uba70 2016\ub144 \uae30\uc900\uc73c\ub85c \uc81c27\ub300 \ucd1d\uc5ec\ud559\uc0dd\ud68c \u2018\uc787\ub2e4\u2019\uac00 \ud65c\ub3d9\ud558\uace0 \uc788\ub2e4. \ucd1d\uc5ec\ud559\uc0dd\ud68c\ub294 \ud0c4\uc0dd \uc774\ub798 \uc704\uc0c1\uc774 \ubaa8\ud638\ud558\ub2e4\ub294 \ube44\ud310\uc744 \ubc1b\uc544\uc654\uc9c0\ub9cc \ub9e4\ub144 \ub0a8\uc131 \uc911\uc2ec\uc758 \ucea0\ud37c\uc2a4 \ubb38\ud654\ub97c \uadf9\ubcf5\ud558\uae30 \uc704\ud55c \ud589\uc0ac\ub97c \uc5f4\uc5b4\uc654\uace0 2000\ub144 2\uc6d4\ubd80\ud130\ub294 \u2018\uc0c8\ub0b4\uae30\ubc30\uc6c0\ud130 \ubc18\uc131\ud3ed\ub825 \ub0b4\uaddc\u2019\ub97c \uc81c\uc815\ud574 \uc2dc\ud589\ud558\uae30\ub3c4 \ud558\uc600\ub2e4. \uc774\uc640 \ub354\ubd88\uc5b4 \uc5ec\ud559\uc0dd\ucc98\ub294 1956\ub144\uc5d0 \uc124\ub9bd\ub41c \uc774\ub798 \uc131\ud3ed\ub825\uc0c1\ub2f4\uc2e4 \uac1c\uc124, \ubc18\uc131\ud3ed\ub825\ud559\uce59 \uc81c\uc815 \ub4f1 \ub0a8\ub140\ud3c9\ub4f1\uc801 \uad50\uc721 \ud658\uacbd\uc744 \ub9c8\ub828\ud558\uae30 \uc704\ud574 \ub178\ub825\ud558\uc600\ub2e4. 2002\ub144\uc5d0\ub294 \uc5ec\uc131\uc778\ub825\uac1c\ubc1c\uc5f0\uad6c\uc6d0\uc774 \ubc1c\uc871\ub418\uc5b4 \uc5ec\ud559\uc0dd\ub4e4\uc758 \uc0ac\ud68c \uc9c4\ucd9c \ub4f1\uc744 \uc704\ud55c \ud559\ub0b4\uc678\uc801\uc778 \ud65c\ub3d9\uc744 \uc218\ud589\ud558\uace0 \uc788\ub2e4.", "answer": "2002\ub144", "sentence": "2002\ub144 \uc5d0\ub294 \uc5ec\uc131\uc778\ub825\uac1c\ubc1c\uc5f0\uad6c\uc6d0\uc774 \ubc1c\uc871\ub418\uc5b4 \uc5ec\ud559\uc0dd\ub4e4\uc758 \uc0ac\ud68c \uc9c4\ucd9c \ub4f1\uc744 \uc704\ud55c \ud559\ub0b4\uc678\uc801\uc778 \ud65c\ub3d9\uc744 \uc218\ud589\ud558\uace0 \uc788\ub2e4.", "paragraph_sentence": "\ucd1d\uc5ec\ud559\uc0dd\ud68c\ub294 \uc5ec\uc131 \ubb38\uc81c\ub97c \uace0\ubbfc\ud558\uace0 \ub0a8\uc131\uc911\uc2ec\uc801 \ub300\ud559 \ubb38\ud654\ub97c \uc5ec\uc131\uc8fc\uc758\uc801\uc73c\ub85c \uc7ac\uad6c\uc131\ud558\uae30 \uc704\ud574\uc11c \ud65c\ub3d9\ud558\uace0 \uc788\ub2e4. \ucd1d\uc5ec\ud559\uc0dd\ud68c\ub294 1988\ub144 3\uc6d4\uc5d0 \uae30\uc874 \ucd1d\ud559\uc0dd\ud68c \uc0b0\ud558 \uc5ec\ud559\uc0dd\ubd80\uc758 \ud55c\uacc4\uc640 \ud559\ub0b4\uc5d0\uc11c \uc18c\uc218\uc758 \uc704\uce58\uc5d0 \uc788\ub358 \uc5ec\ud559\uc0dd \ubb38\uc81c\ub97c \uadf9\ubcf5\ud558\uace0\uc790 \ubc1c\uc871\ud558\uc600\ub2e4. \ub9e4\ub144 \uc120\uac70\ub97c \ud1b5\ud574 \ucd1d\uc5ec\ud559\uc0dd\ud68c\uac00 \uc120\ucd9c\ub418\uace0 \uc788\uc73c\uba70 2016\ub144 \uae30\uc900\uc73c\ub85c \uc81c27\ub300 \ucd1d\uc5ec\ud559\uc0dd\ud68c \u2018\uc787\ub2e4\u2019\uac00 \ud65c\ub3d9\ud558\uace0 \uc788\ub2e4. \ucd1d\uc5ec\ud559\uc0dd\ud68c\ub294 \ud0c4\uc0dd \uc774\ub798 \uc704\uc0c1\uc774 \ubaa8\ud638\ud558\ub2e4\ub294 \ube44\ud310\uc744 \ubc1b\uc544\uc654\uc9c0\ub9cc \ub9e4\ub144 \ub0a8\uc131 \uc911\uc2ec\uc758 \ucea0\ud37c\uc2a4 \ubb38\ud654\ub97c \uadf9\ubcf5\ud558\uae30 \uc704\ud55c \ud589\uc0ac\ub97c \uc5f4\uc5b4\uc654\uace0 2000\ub144 2\uc6d4\ubd80\ud130\ub294 \u2018\uc0c8\ub0b4\uae30\ubc30\uc6c0\ud130 \ubc18\uc131\ud3ed\ub825 \ub0b4\uaddc\u2019\ub97c \uc81c\uc815\ud574 \uc2dc\ud589\ud558\uae30\ub3c4 \ud558\uc600\ub2e4. \uc774\uc640 \ub354\ubd88\uc5b4 \uc5ec\ud559\uc0dd\ucc98\ub294 1956\ub144\uc5d0 \uc124\ub9bd\ub41c \uc774\ub798 \uc131\ud3ed\ub825\uc0c1\ub2f4\uc2e4 \uac1c\uc124, \ubc18\uc131\ud3ed\ub825\ud559\uce59 \uc81c\uc815 \ub4f1 \ub0a8\ub140\ud3c9\ub4f1\uc801 \uad50\uc721 \ud658\uacbd\uc744 \ub9c8\ub828\ud558\uae30 \uc704\ud574 \ub178\ub825\ud558\uc600\ub2e4. 2002\ub144 \uc5d0\ub294 \uc5ec\uc131\uc778\ub825\uac1c\ubc1c\uc5f0\uad6c\uc6d0\uc774 \ubc1c\uc871\ub418\uc5b4 \uc5ec\ud559\uc0dd\ub4e4\uc758 \uc0ac\ud68c \uc9c4\ucd9c \ub4f1\uc744 \uc704\ud55c \ud559\ub0b4\uc678\uc801\uc778 \ud65c\ub3d9\uc744 \uc218\ud589\ud558\uace0 \uc788\ub2e4. ", "paragraph_answer": "\ucd1d\uc5ec\ud559\uc0dd\ud68c\ub294 \uc5ec\uc131 \ubb38\uc81c\ub97c \uace0\ubbfc\ud558\uace0 \ub0a8\uc131\uc911\uc2ec\uc801 \ub300\ud559 \ubb38\ud654\ub97c \uc5ec\uc131\uc8fc\uc758\uc801\uc73c\ub85c \uc7ac\uad6c\uc131\ud558\uae30 \uc704\ud574\uc11c \ud65c\ub3d9\ud558\uace0 \uc788\ub2e4. \ucd1d\uc5ec\ud559\uc0dd\ud68c\ub294 1988\ub144 3\uc6d4\uc5d0 \uae30\uc874 \ucd1d\ud559\uc0dd\ud68c \uc0b0\ud558 \uc5ec\ud559\uc0dd\ubd80\uc758 \ud55c\uacc4\uc640 \ud559\ub0b4\uc5d0\uc11c \uc18c\uc218\uc758 \uc704\uce58\uc5d0 \uc788\ub358 \uc5ec\ud559\uc0dd \ubb38\uc81c\ub97c \uadf9\ubcf5\ud558\uace0\uc790 \ubc1c\uc871\ud558\uc600\ub2e4. \ub9e4\ub144 \uc120\uac70\ub97c \ud1b5\ud574 \ucd1d\uc5ec\ud559\uc0dd\ud68c\uac00 \uc120\ucd9c\ub418\uace0 \uc788\uc73c\uba70 2016\ub144 \uae30\uc900\uc73c\ub85c \uc81c27\ub300 \ucd1d\uc5ec\ud559\uc0dd\ud68c \u2018\uc787\ub2e4\u2019\uac00 \ud65c\ub3d9\ud558\uace0 \uc788\ub2e4. \ucd1d\uc5ec\ud559\uc0dd\ud68c\ub294 \ud0c4\uc0dd \uc774\ub798 \uc704\uc0c1\uc774 \ubaa8\ud638\ud558\ub2e4\ub294 \ube44\ud310\uc744 \ubc1b\uc544\uc654\uc9c0\ub9cc \ub9e4\ub144 \ub0a8\uc131 \uc911\uc2ec\uc758 \ucea0\ud37c\uc2a4 \ubb38\ud654\ub97c \uadf9\ubcf5\ud558\uae30 \uc704\ud55c \ud589\uc0ac\ub97c \uc5f4\uc5b4\uc654\uace0 2000\ub144 2\uc6d4\ubd80\ud130\ub294 \u2018\uc0c8\ub0b4\uae30\ubc30\uc6c0\ud130 \ubc18\uc131\ud3ed\ub825 \ub0b4\uaddc\u2019\ub97c \uc81c\uc815\ud574 \uc2dc\ud589\ud558\uae30\ub3c4 \ud558\uc600\ub2e4. \uc774\uc640 \ub354\ubd88\uc5b4 \uc5ec\ud559\uc0dd\ucc98\ub294 1956\ub144\uc5d0 \uc124\ub9bd\ub41c \uc774\ub798 \uc131\ud3ed\ub825\uc0c1\ub2f4\uc2e4 \uac1c\uc124, \ubc18\uc131\ud3ed\ub825\ud559\uce59 \uc81c\uc815 \ub4f1 \ub0a8\ub140\ud3c9\ub4f1\uc801 \uad50\uc721 \ud658\uacbd\uc744 \ub9c8\ub828\ud558\uae30 \uc704\ud574 \ub178\ub825\ud558\uc600\ub2e4. 2002\ub144 \uc5d0\ub294 \uc5ec\uc131\uc778\ub825\uac1c\ubc1c\uc5f0\uad6c\uc6d0\uc774 \ubc1c\uc871\ub418\uc5b4 \uc5ec\ud559\uc0dd\ub4e4\uc758 \uc0ac\ud68c \uc9c4\ucd9c \ub4f1\uc744 \uc704\ud55c \ud559\ub0b4\uc678\uc801\uc778 \ud65c\ub3d9\uc744 \uc218\ud589\ud558\uace0 \uc788\ub2e4.", "sentence_answer": " 2002\ub144 \uc5d0\ub294 \uc5ec\uc131\uc778\ub825\uac1c\ubc1c\uc5f0\uad6c\uc6d0\uc774 \ubc1c\uc871\ub418\uc5b4 \uc5ec\ud559\uc0dd\ub4e4\uc758 \uc0ac\ud68c \uc9c4\ucd9c \ub4f1\uc744 \uc704\ud55c \ud559\ub0b4\uc678\uc801\uc778 \ud65c\ub3d9\uc744 \uc218\ud589\ud558\uace0 \uc788\ub2e4.", "paragraph_id": "6524049-12-1", "paragraph_question": "question: \uc5ec\uc131\uc778\ub825\uac1c\ubc1c\uc5f0\uad6c\uc6d0\uc774 \ubc1c\uc871\ub41c \uc2dc\uae30\ub294 \uc5b8\uc81c\uc778\uac00?, context: \ucd1d\uc5ec\ud559\uc0dd\ud68c\ub294 \uc5ec\uc131 \ubb38\uc81c\ub97c \uace0\ubbfc\ud558\uace0 \ub0a8\uc131\uc911\uc2ec\uc801 \ub300\ud559 \ubb38\ud654\ub97c \uc5ec\uc131\uc8fc\uc758\uc801\uc73c\ub85c \uc7ac\uad6c\uc131\ud558\uae30 \uc704\ud574\uc11c \ud65c\ub3d9\ud558\uace0 \uc788\ub2e4. \ucd1d\uc5ec\ud559\uc0dd\ud68c\ub294 1988\ub144 3\uc6d4\uc5d0 \uae30\uc874 \ucd1d\ud559\uc0dd\ud68c \uc0b0\ud558 \uc5ec\ud559\uc0dd\ubd80\uc758 \ud55c\uacc4\uc640 \ud559\ub0b4\uc5d0\uc11c \uc18c\uc218\uc758 \uc704\uce58\uc5d0 \uc788\ub358 \uc5ec\ud559\uc0dd \ubb38\uc81c\ub97c \uadf9\ubcf5\ud558\uace0\uc790 \ubc1c\uc871\ud558\uc600\ub2e4. \ub9e4\ub144 \uc120\uac70\ub97c \ud1b5\ud574 \ucd1d\uc5ec\ud559\uc0dd\ud68c\uac00 \uc120\ucd9c\ub418\uace0 \uc788\uc73c\uba70 2016\ub144 \uae30\uc900\uc73c\ub85c \uc81c27\ub300 \ucd1d\uc5ec\ud559\uc0dd\ud68c \u2018\uc787\ub2e4\u2019\uac00 \ud65c\ub3d9\ud558\uace0 \uc788\ub2e4. \ucd1d\uc5ec\ud559\uc0dd\ud68c\ub294 \ud0c4\uc0dd \uc774\ub798 \uc704\uc0c1\uc774 \ubaa8\ud638\ud558\ub2e4\ub294 \ube44\ud310\uc744 \ubc1b\uc544\uc654\uc9c0\ub9cc \ub9e4\ub144 \ub0a8\uc131 \uc911\uc2ec\uc758 \ucea0\ud37c\uc2a4 \ubb38\ud654\ub97c \uadf9\ubcf5\ud558\uae30 \uc704\ud55c \ud589\uc0ac\ub97c \uc5f4\uc5b4\uc654\uace0 2000\ub144 2\uc6d4\ubd80\ud130\ub294 \u2018\uc0c8\ub0b4\uae30\ubc30\uc6c0\ud130 \ubc18\uc131\ud3ed\ub825 \ub0b4\uaddc\u2019\ub97c \uc81c\uc815\ud574 \uc2dc\ud589\ud558\uae30\ub3c4 \ud558\uc600\ub2e4. \uc774\uc640 \ub354\ubd88\uc5b4 \uc5ec\ud559\uc0dd\ucc98\ub294 1956\ub144\uc5d0 \uc124\ub9bd\ub41c \uc774\ub798 \uc131\ud3ed\ub825\uc0c1\ub2f4\uc2e4 \uac1c\uc124, \ubc18\uc131\ud3ed\ub825\ud559\uce59 \uc81c\uc815 \ub4f1 \ub0a8\ub140\ud3c9\ub4f1\uc801 \uad50\uc721 \ud658\uacbd\uc744 \ub9c8\ub828\ud558\uae30 \uc704\ud574 \ub178\ub825\ud558\uc600\ub2e4. 2002\ub144\uc5d0\ub294 \uc5ec\uc131\uc778\ub825\uac1c\ubc1c\uc5f0\uad6c\uc6d0\uc774 \ubc1c\uc871\ub418\uc5b4 \uc5ec\ud559\uc0dd\ub4e4\uc758 \uc0ac\ud68c \uc9c4\ucd9c \ub4f1\uc744 \uc704\ud55c \ud559\ub0b4\uc678\uc801\uc778 \ud65c\ub3d9\uc744 \uc218\ud589\ud558\uace0 \uc788\ub2e4."} {"question": "\uc0bc\ucd0c \uc555\ub458 \ub77c\ud798 \ubc14\uce58\ud06c\uac00 \uacb0\uc131\ud558\uace0 \ubcf4\uceec\ub85c \ud560\ub3d9\ud55c \ubc34\ub4dc\uc758 \uc774\ub984\uc740?", "paragraph": "\uc0bc\ucd0c \uc555\ub458 \ub77c\ud798 \ubc14\uce58\ud06c\uac00 \uacb0\uc131\ud55c \ubc34\ub4dc '\ud328\ubc00\ub9ac \uadf8\ub8f9(Family Group)'\uc758 \ubcf4\uceec\ub85c\ub3c4 \ud65c\ub3d9\ud588\uc5c8\ub2e4. 16\uc138\uac00 \ub418\uae30 \uc804\uae4c\uc9c0 \uace0\ud5a5\uc5d0\uc11c \uc5f4\ub9ac\ub294 \uc5ec\ub7ec \ucd95\uc81c\ub9c8\ub2e4 \ucc38\uc5ec\ud574 \ub178\ub798 \uc2e4\ub825\uc744 \ud0a4\uc6b0\uace4 \ud588\ub2e4. \ub9c8\uce68\ub0b4 1995\ub144, \uc2dc\ud2f0\ub294 RTM Bintang HMI \ub300\ud68c \uc624\ub514\uc158\uc5d0 \ucc38\uc5ec\ud574 \uc778\ub3c4\ub124\uc2dc\uc544 \uac00\uc218 \ub8e8\uc2a4 \uc0ac\ud558\ub098\uc57c(Ruth Sahanaya)\uc758 \ub178\ub798 \ub97c \uc120\ubcf4\uc600\ub2e4. \uc0ac\uc2e4, \uc774\ubbf8 14\uc138 \ub54c\uc778 1993\ub144 \uc544\uc2dc\uc544 \ubc14\uad6c\uc2a4(Asia Bagus) \uc624\ub514\uc158\uc5d0 \ucd9c\uc5f0\ud588\uc73c\ub098 \ud0c8\ub77d\ud588\ub2e4. Bintang HMI \ub300\ud68c \uc911 \uc791\uace1\uac00 \uc544\ub4dc\ub09c \uc544\ubd80 \ud558\uc0b0(Adnan Abu Hassan)\uc744 \ub9cc\ub0ac\uc73c\uba70, \ud558\uc0b0\uc740 \uc774\ud6c4 \uc2dc\ud2f0\uc758 \ubcf4\uceec \ud37c\ud3ec\uba3c\uc2a4\ub97c \uac00\ub974\uce58\uace0 \ub3c4\uc640\uc92c\ub2e4. \uadf8 \ud6c4 \uc544\uc774\uc0e4(Aishah)\uc758 \ub178\ub798 \ub97c \ubd80\ub974\uace0 \uc6b0\uc2b9\ud588\ub2e4. \uc774\ud6c4 \uc218\ub9ac\uc544 \ub808\ucf54\ub4dc(Suria Records)\uc640 \uacc4\uc57d\uc744 \ub9fa\uc5c8\uc73c\uba70, \ud6c4\uc5d0 Sony Music, BMG Music, Warner Music\uacfc \ucd94\uac00\uc801\uc73c\ub85c \uacc4\uc57d\uc744 \ub9fa\uc5c8\ub2e4. \uc774\ub4ec\ud574\uc778 1996\ub144 \uc218\ub9ac\uc544 \ub808\ucf54\ub4dc\ub97c \ud1b5\ud574 \uccab \ubc88\uc9f8 \uc2f1\uae00 \ubc0f \uccab \ubc88\uc9f8 \uc815\uaddc\uc74c\ubc18 \uc744 \ubc1c\ub9e4\ud588\uc73c\uba70, \uc81c11\ud68c \uc544\ub204\uac8c\ub77c \uc8fc\uc544\ub77c \ub77c\uad6c(Anugerah Juara Lagu) \ub300\ud68c\uc5d0\uc11c \ucd5c\uace0\uc758 \ud37c\ud3ec\uba3c\uc2a4 \ubc0f \ucd5c\uace0\uc758 \ubc1c\ub77c\ub4dc \uc0c1\uc744 \ud0d4\ub2e4. \uc774\ud6c4 \uc544\ub204\uac8c\ub77c \ube48\ud0d5 \ud3ec\ud480\ub77c\ub974(Anugerah Bintang Popular)\uc5d0\uc11c \uac00\uc7a5 \uc778\uae30 \uc788\ub294 \uc5f0\uc608\uc778, \uac00\uc7a5 \uc778\uae30 \uc788\ub294 \uc5ec\uac00\uc218, \uac00\uc7a5 \uc778\uae30 \uc788\ub294 10\ub300 \uc5f0\uc608\uc778\uc73c\ub85c \uc2b9\ub9ac\ud588\uc73c\uba70, \uc804\uccb4\uc801\uc73c\ub85c\ub294 \uac00\uc7a5 \uc778\uae30 \uc788\ub294 \uc2a4\ud0c0\ub85c \uc2b9\ub9ac\ud588\ub2e4.", "answer": "\ud328\ubc00\ub9ac \uadf8\ub8f9", "sentence": "\uc0bc\ucd0c \uc555\ub458 \ub77c\ud798 \ubc14\uce58\ud06c\uac00 \uacb0\uc131\ud55c \ubc34\ub4dc ' \ud328\ubc00\ub9ac \uadf8\ub8f9 (Family Group)", "paragraph_sentence": " \uc0bc\ucd0c \uc555\ub458 \ub77c\ud798 \ubc14\uce58\ud06c\uac00 \uacb0\uc131\ud55c \ubc34\ub4dc ' \ud328\ubc00\ub9ac \uadf8\ub8f9 (Family Group) '\uc758 \ubcf4\uceec\ub85c\ub3c4 \ud65c\ub3d9\ud588\uc5c8\ub2e4. 16\uc138\uac00 \ub418\uae30 \uc804\uae4c\uc9c0 \uace0\ud5a5\uc5d0\uc11c \uc5f4\ub9ac\ub294 \uc5ec\ub7ec \ucd95\uc81c\ub9c8\ub2e4 \ucc38\uc5ec\ud574 \ub178\ub798 \uc2e4\ub825\uc744 \ud0a4\uc6b0\uace4 \ud588\ub2e4. \ub9c8\uce68\ub0b4 1995\ub144, \uc2dc\ud2f0\ub294 RTM Bintang HMI \ub300\ud68c \uc624\ub514\uc158\uc5d0 \ucc38\uc5ec\ud574 \uc778\ub3c4\ub124\uc2dc\uc544 \uac00\uc218 \ub8e8\uc2a4 \uc0ac\ud558\ub098\uc57c(Ruth Sahanaya)\uc758 \ub178\ub798 \ub97c \uc120\ubcf4\uc600\ub2e4. \uc0ac\uc2e4, \uc774\ubbf8 14\uc138 \ub54c\uc778 1993\ub144 \uc544\uc2dc\uc544 \ubc14\uad6c\uc2a4(Asia Bagus) \uc624\ub514\uc158\uc5d0 \ucd9c\uc5f0\ud588\uc73c\ub098 \ud0c8\ub77d\ud588\ub2e4. Bintang HMI \ub300\ud68c \uc911 \uc791\uace1\uac00 \uc544\ub4dc\ub09c \uc544\ubd80 \ud558\uc0b0(Adnan Abu Hassan)\uc744 \ub9cc\ub0ac\uc73c\uba70, \ud558\uc0b0\uc740 \uc774\ud6c4 \uc2dc\ud2f0\uc758 \ubcf4\uceec \ud37c\ud3ec\uba3c\uc2a4\ub97c \uac00\ub974\uce58\uace0 \ub3c4\uc640\uc92c\ub2e4. \uadf8 \ud6c4 \uc544\uc774\uc0e4(Aishah)\uc758 \ub178\ub798 \ub97c \ubd80\ub974\uace0 \uc6b0\uc2b9\ud588\ub2e4. \uc774\ud6c4 \uc218\ub9ac\uc544 \ub808\ucf54\ub4dc(Suria Records)\uc640 \uacc4\uc57d\uc744 \ub9fa\uc5c8\uc73c\uba70, \ud6c4\uc5d0 Sony Music, BMG Music, Warner Music\uacfc \ucd94\uac00\uc801\uc73c\ub85c \uacc4\uc57d\uc744 \ub9fa\uc5c8\ub2e4. \uc774\ub4ec\ud574\uc778 1996\ub144 \uc218\ub9ac\uc544 \ub808\ucf54\ub4dc\ub97c \ud1b5\ud574 \uccab \ubc88\uc9f8 \uc2f1\uae00 \ubc0f \uccab \ubc88\uc9f8 \uc815\uaddc\uc74c\ubc18 \uc744 \ubc1c\ub9e4\ud588\uc73c\uba70, \uc81c11\ud68c \uc544\ub204\uac8c\ub77c \uc8fc\uc544\ub77c \ub77c\uad6c(Anugerah Juara Lagu) \ub300\ud68c\uc5d0\uc11c \ucd5c\uace0\uc758 \ud37c\ud3ec\uba3c\uc2a4 \ubc0f \ucd5c\uace0\uc758 \ubc1c\ub77c\ub4dc \uc0c1\uc744 \ud0d4\ub2e4. \uc774\ud6c4 \uc544\ub204\uac8c\ub77c \ube48\ud0d5 \ud3ec\ud480\ub77c\ub974(Anugerah Bintang Popular)\uc5d0\uc11c \uac00\uc7a5 \uc778\uae30 \uc788\ub294 \uc5f0\uc608\uc778, \uac00\uc7a5 \uc778\uae30 \uc788\ub294 \uc5ec\uac00\uc218, \uac00\uc7a5 \uc778\uae30 \uc788\ub294 10\ub300 \uc5f0\uc608\uc778\uc73c\ub85c \uc2b9\ub9ac\ud588\uc73c\uba70, \uc804\uccb4\uc801\uc73c\ub85c\ub294 \uac00\uc7a5 \uc778\uae30 \uc788\ub294 \uc2a4\ud0c0\ub85c \uc2b9\ub9ac\ud588\ub2e4.", "paragraph_answer": "\uc0bc\ucd0c \uc555\ub458 \ub77c\ud798 \ubc14\uce58\ud06c\uac00 \uacb0\uc131\ud55c \ubc34\ub4dc ' \ud328\ubc00\ub9ac \uadf8\ub8f9 (Family Group)'\uc758 \ubcf4\uceec\ub85c\ub3c4 \ud65c\ub3d9\ud588\uc5c8\ub2e4. 16\uc138\uac00 \ub418\uae30 \uc804\uae4c\uc9c0 \uace0\ud5a5\uc5d0\uc11c \uc5f4\ub9ac\ub294 \uc5ec\ub7ec \ucd95\uc81c\ub9c8\ub2e4 \ucc38\uc5ec\ud574 \ub178\ub798 \uc2e4\ub825\uc744 \ud0a4\uc6b0\uace4 \ud588\ub2e4. \ub9c8\uce68\ub0b4 1995\ub144, \uc2dc\ud2f0\ub294 RTM Bintang HMI \ub300\ud68c \uc624\ub514\uc158\uc5d0 \ucc38\uc5ec\ud574 \uc778\ub3c4\ub124\uc2dc\uc544 \uac00\uc218 \ub8e8\uc2a4 \uc0ac\ud558\ub098\uc57c(Ruth Sahanaya)\uc758 \ub178\ub798 \ub97c \uc120\ubcf4\uc600\ub2e4. \uc0ac\uc2e4, \uc774\ubbf8 14\uc138 \ub54c\uc778 1993\ub144 \uc544\uc2dc\uc544 \ubc14\uad6c\uc2a4(Asia Bagus) \uc624\ub514\uc158\uc5d0 \ucd9c\uc5f0\ud588\uc73c\ub098 \ud0c8\ub77d\ud588\ub2e4. Bintang HMI \ub300\ud68c \uc911 \uc791\uace1\uac00 \uc544\ub4dc\ub09c \uc544\ubd80 \ud558\uc0b0(Adnan Abu Hassan)\uc744 \ub9cc\ub0ac\uc73c\uba70, \ud558\uc0b0\uc740 \uc774\ud6c4 \uc2dc\ud2f0\uc758 \ubcf4\uceec \ud37c\ud3ec\uba3c\uc2a4\ub97c \uac00\ub974\uce58\uace0 \ub3c4\uc640\uc92c\ub2e4. \uadf8 \ud6c4 \uc544\uc774\uc0e4(Aishah)\uc758 \ub178\ub798 \ub97c \ubd80\ub974\uace0 \uc6b0\uc2b9\ud588\ub2e4. \uc774\ud6c4 \uc218\ub9ac\uc544 \ub808\ucf54\ub4dc(Suria Records)\uc640 \uacc4\uc57d\uc744 \ub9fa\uc5c8\uc73c\uba70, \ud6c4\uc5d0 Sony Music, BMG Music, Warner Music\uacfc \ucd94\uac00\uc801\uc73c\ub85c \uacc4\uc57d\uc744 \ub9fa\uc5c8\ub2e4. \uc774\ub4ec\ud574\uc778 1996\ub144 \uc218\ub9ac\uc544 \ub808\ucf54\ub4dc\ub97c \ud1b5\ud574 \uccab \ubc88\uc9f8 \uc2f1\uae00 \ubc0f \uccab \ubc88\uc9f8 \uc815\uaddc\uc74c\ubc18 \uc744 \ubc1c\ub9e4\ud588\uc73c\uba70, \uc81c11\ud68c \uc544\ub204\uac8c\ub77c \uc8fc\uc544\ub77c \ub77c\uad6c(Anugerah Juara Lagu) \ub300\ud68c\uc5d0\uc11c \ucd5c\uace0\uc758 \ud37c\ud3ec\uba3c\uc2a4 \ubc0f \ucd5c\uace0\uc758 \ubc1c\ub77c\ub4dc \uc0c1\uc744 \ud0d4\ub2e4. \uc774\ud6c4 \uc544\ub204\uac8c\ub77c \ube48\ud0d5 \ud3ec\ud480\ub77c\ub974(Anugerah Bintang Popular)\uc5d0\uc11c \uac00\uc7a5 \uc778\uae30 \uc788\ub294 \uc5f0\uc608\uc778, \uac00\uc7a5 \uc778\uae30 \uc788\ub294 \uc5ec\uac00\uc218, \uac00\uc7a5 \uc778\uae30 \uc788\ub294 10\ub300 \uc5f0\uc608\uc778\uc73c\ub85c \uc2b9\ub9ac\ud588\uc73c\uba70, \uc804\uccb4\uc801\uc73c\ub85c\ub294 \uac00\uc7a5 \uc778\uae30 \uc788\ub294 \uc2a4\ud0c0\ub85c \uc2b9\ub9ac\ud588\ub2e4.", "sentence_answer": "\uc0bc\ucd0c \uc555\ub458 \ub77c\ud798 \ubc14\uce58\ud06c\uac00 \uacb0\uc131\ud55c \ubc34\ub4dc ' \ud328\ubc00\ub9ac \uadf8\ub8f9 (Family Group)", "paragraph_id": "6457397-1-0", "paragraph_question": "question: \uc0bc\ucd0c \uc555\ub458 \ub77c\ud798 \ubc14\uce58\ud06c\uac00 \uacb0\uc131\ud558\uace0 \ubcf4\uceec\ub85c \ud560\ub3d9\ud55c \ubc34\ub4dc\uc758 \uc774\ub984\uc740?, context: \uc0bc\ucd0c \uc555\ub458 \ub77c\ud798 \ubc14\uce58\ud06c\uac00 \uacb0\uc131\ud55c \ubc34\ub4dc '\ud328\ubc00\ub9ac \uadf8\ub8f9(Family Group)'\uc758 \ubcf4\uceec\ub85c\ub3c4 \ud65c\ub3d9\ud588\uc5c8\ub2e4. 16\uc138\uac00 \ub418\uae30 \uc804\uae4c\uc9c0 \uace0\ud5a5\uc5d0\uc11c \uc5f4\ub9ac\ub294 \uc5ec\ub7ec \ucd95\uc81c\ub9c8\ub2e4 \ucc38\uc5ec\ud574 \ub178\ub798 \uc2e4\ub825\uc744 \ud0a4\uc6b0\uace4 \ud588\ub2e4. \ub9c8\uce68\ub0b4 1995\ub144, \uc2dc\ud2f0\ub294 RTM Bintang HMI \ub300\ud68c \uc624\ub514\uc158\uc5d0 \ucc38\uc5ec\ud574 \uc778\ub3c4\ub124\uc2dc\uc544 \uac00\uc218 \ub8e8\uc2a4 \uc0ac\ud558\ub098\uc57c(Ruth Sahanaya)\uc758 \ub178\ub798 \ub97c \uc120\ubcf4\uc600\ub2e4. \uc0ac\uc2e4, \uc774\ubbf8 14\uc138 \ub54c\uc778 1993\ub144 \uc544\uc2dc\uc544 \ubc14\uad6c\uc2a4(Asia Bagus) \uc624\ub514\uc158\uc5d0 \ucd9c\uc5f0\ud588\uc73c\ub098 \ud0c8\ub77d\ud588\ub2e4. Bintang HMI \ub300\ud68c \uc911 \uc791\uace1\uac00 \uc544\ub4dc\ub09c \uc544\ubd80 \ud558\uc0b0(Adnan Abu Hassan)\uc744 \ub9cc\ub0ac\uc73c\uba70, \ud558\uc0b0\uc740 \uc774\ud6c4 \uc2dc\ud2f0\uc758 \ubcf4\uceec \ud37c\ud3ec\uba3c\uc2a4\ub97c \uac00\ub974\uce58\uace0 \ub3c4\uc640\uc92c\ub2e4. \uadf8 \ud6c4 \uc544\uc774\uc0e4(Aishah)\uc758 \ub178\ub798 \ub97c \ubd80\ub974\uace0 \uc6b0\uc2b9\ud588\ub2e4. \uc774\ud6c4 \uc218\ub9ac\uc544 \ub808\ucf54\ub4dc(Suria Records)\uc640 \uacc4\uc57d\uc744 \ub9fa\uc5c8\uc73c\uba70, \ud6c4\uc5d0 Sony Music, BMG Music, Warner Music\uacfc \ucd94\uac00\uc801\uc73c\ub85c \uacc4\uc57d\uc744 \ub9fa\uc5c8\ub2e4. \uc774\ub4ec\ud574\uc778 1996\ub144 \uc218\ub9ac\uc544 \ub808\ucf54\ub4dc\ub97c \ud1b5\ud574 \uccab \ubc88\uc9f8 \uc2f1\uae00 \ubc0f \uccab \ubc88\uc9f8 \uc815\uaddc\uc74c\ubc18 \uc744 \ubc1c\ub9e4\ud588\uc73c\uba70, \uc81c11\ud68c \uc544\ub204\uac8c\ub77c \uc8fc\uc544\ub77c \ub77c\uad6c(Anugerah Juara Lagu) \ub300\ud68c\uc5d0\uc11c \ucd5c\uace0\uc758 \ud37c\ud3ec\uba3c\uc2a4 \ubc0f \ucd5c\uace0\uc758 \ubc1c\ub77c\ub4dc \uc0c1\uc744 \ud0d4\ub2e4. \uc774\ud6c4 \uc544\ub204\uac8c\ub77c \ube48\ud0d5 \ud3ec\ud480\ub77c\ub974(Anugerah Bintang Popular)\uc5d0\uc11c \uac00\uc7a5 \uc778\uae30 \uc788\ub294 \uc5f0\uc608\uc778, \uac00\uc7a5 \uc778\uae30 \uc788\ub294 \uc5ec\uac00\uc218, \uac00\uc7a5 \uc778\uae30 \uc788\ub294 10\ub300 \uc5f0\uc608\uc778\uc73c\ub85c \uc2b9\ub9ac\ud588\uc73c\uba70, \uc804\uccb4\uc801\uc73c\ub85c\ub294 \uac00\uc7a5 \uc778\uae30 \uc788\ub294 \uc2a4\ud0c0\ub85c \uc2b9\ub9ac\ud588\ub2e4."} {"question": "\ube14\ub799\ubc88 \ub85c\ubc84\uc2a4\ub97c \uc0c1\ub300\ub85c \ub9ac\uadf8 \ub370\ubdd4\ub97c \uae30\ub85d\ud55c \ub54c\ub294?", "paragraph": "2008\ub144 6\uc6d4 10\uc77c \uc544\uc2a4\ub110, \ub9e8\uccb4\uc2a4\ud130 \uc720\ub098\uc774\ud2f0\ub4dc, \uc5d0\ubc84\ud2bc\uc758 \uacbd\uc7c1 \ub05d\uc5d0, \uc544\uc2a4\ub110\uc5d0 \ud569\ub958\ud558\uae30\ub85c \uacb0\uc815\ud588\ub2e4. \uc774 \uc774\uc801\uc740 \uce74\ub514\ud504 \uc2dc\ud2f0\uc5d0\uac8c 480\ub9cc \ud30c\uc6b4\ub4dc\uc758 \uc774\uc801\ub8cc\ub97c \uc548\uacbc\ub2e4. \uc774\uc801\uc740 6\uc6d4 13\uc77c \uc644\ub8cc\ub418\uc5c8\uc73c\uba70 \uc7a5\uae30\uacc4\uc57d\uc744 \uccb4\uacb0\ud55c \uac83\uc73c\ub85c \uc54c\ub824\uc84c\ub2e4. \ub2e4\ub978 \ud300\ub4e4 \ub300\uc2e0 \uc544\uc2a4\ub110\uc744 \ud0dd\ud55c \uc774\uc720\ub85c \uc2a4\uc704\uc2a4\uae4c\uc9c0 \ucc3e\uc544\uc640 \ub7a8\uc9c0\uc640 \uac00\uc871\ub4e4\uc5d0\uac8c \uc544\uc2a4\ub110\uc774 \uac00\uc9c0\uace0\uc788\ub294 \uacc4\ud68d\uacfc \ub7a8\uc9c0\uc5d0 \ub300\ud55c \uae30\ub300\ub97c \ubc1d\ud78c \uc544\ub974\uc13c \ubcb5\uac70 \uac10\ub3c5\uc758 \ub178\ub825\uc5d0 \uac10\uba85\ubc1b\uc558\ub2e4\uace0 \ud55c\ub2e4. \ubcb5\uac70\ub294 \ub7a8\uc9c0\ub97c \ud6cc\ub96d\ud55c \uccb4\ub825\uacfc \uc88b\uc740 \ud328\uc2a4\uc640 \uae30\uc220\uacfc \uc2dc\uc57c\ub97c \uc9c0\ub2cc \uc120\uc218\ub77c \ud3c9\uac00\ud588\ub2e4. 2008\ub144 8\uc6d4 13\uc77c, \ucc54\ud53c\uc5b8\uc2a4\ub9ac\uadf8 \uc608\uc120 3\ub77c\uc6b4\ub4dc FC \ud2b8\ubca4\ud14c\ub97c \uc0c1\ub300\ub85c \uc778\uc0c1\uc801\uc778 \ub370\ubdd4\uc804\uc744 \uce58\ub918\uc73c\uba70 \ud55c \ub2ec \ub4a4\uc778 9\uc6d4 13\uc77c \ube14\ub799\ubc88 \ub85c\ubc84\uc2a4\ub97c \uc0c1\ub300\ub85c \ub9ac\uadf8 \ub370\ubdd4\ub97c \uae30\ub85d\ud55c\ub2e4. \uc774 \ub9ac\uadf8 \ub370\ubdd4\uc804\uc5d0\uc11c \ub7a8\uc9c0\uc758 \uc5b4\uc2dc\uc2a4\ud2b8\ub85c \uc544\ub370\ubc14\uc694\ub974\uac00 \ud300\uc758 4\ubc88\uc9f8 \uace8\uc744 \uae30\ub85d\ud558\uba70 \ud574\ud2b8\ud2b8\ub9ad\uc744 \uc644\uc131\ud588\ub2e4. 9\uc6d4 23\uc77c \ub9ac\uadf8\ucef5 3\ub77c\uc6b4\ub4dc 6:0\uc73c\ub85c \ub300\uc2b9\uc744 \uac70\ub454 \uc170\ud544\ub4dc \uc720\ub098\uc774\ud2f0\ub4dc\uc640\uc758 \uacbd\uae30\uc5d0\uc11c \ub2c8\ud074\ub77c\uc2a4 \ubca4\ud2b8\ub108\uc640 \uce74\ub97c\ub85c\uc2a4 \ubca8\ub77c\uc758 \uace8\uc5d0 \uac01\uac01 \uc5b4\uc2dc\uc2a4\ud2b8\ub97c \uae30\ub85d\ud558\uba70 \uc5b4\uc2dc\uc2a4\ud2b8 2\uac1c\ub97c \ucd94\uac00\ud55c\ub2e4. 5:2\ub85c \uc2b9\ub9ac\ud55c \ud398\ub124\ub974\ubc14\ud750\uccb4\uc640\uc758 \ucc54\ud53c\uc5b8\uc2a4\ub9ac\uadf8 \uc870\ubcc4\ub9ac\uadf8 \uacbd\uae30\uc5d0\uc11c \uc67c\ucabd \uace8\ub300 \ub9de\uace0 \ub4e4\uc5b4\uac04 \uc7a5\uac70\ub9ac \uace8\ub85c \uccab \uace8\uc744 \uae30\ub85d\ud588\ub2e4. \uc774 \uace8\ub85c \ucc54\ud53c\uc5b8\uc2a4\ub9ac\uadf8 \uc5ed\uc0ac\uc0c1 5\ubc88\uc9f8\ub85c \uc5b4\ub9b0 \ub4dd\uc810\uc744 \uae30\ub85d\ud55c \uc120\uc218\uac00 \ub418\uc5c8\uc73c\uba70, 90\ub144\ub300 \ud0dc\uc5b4\ub09c \uc120\uc218 \uc911 \ucc54\ud53c\uc5b8\uc2a4\ub9ac\uadf8 \uace8\uc744 \uae30\ub85d\ud55c 2\ubc88\uc9f8 \uc120\uc218\uac00 \ub418\uc5c8\ub2e4. 2009\ub144 \ud53c\ud30c\uc5d0\uc11c \uc120\uc815\ud55c \uc8fc\ubaa9\ud574\uc57c \ud560 \uc720\ub9dd\uc8fc\uc5d0 \uc774\ub984\uc744 \uc62c\ub9ac\uae30\ub3c4 \ud588\ub2e4.", "answer": "9\uc6d4 13\uc77c", "sentence": "2008\ub144 8\uc6d4 13\uc77c, \ucc54\ud53c\uc5b8\uc2a4\ub9ac\uadf8 \uc608\uc120 3\ub77c\uc6b4\ub4dc FC \ud2b8\ubca4\ud14c\ub97c \uc0c1\ub300\ub85c \uc778\uc0c1\uc801\uc778 \ub370\ubdd4\uc804\uc744 \uce58\ub918\uc73c\uba70 \ud55c \ub2ec \ub4a4\uc778 9\uc6d4 13\uc77c \ube14\ub799\ubc88 \ub85c\ubc84\uc2a4\ub97c \uc0c1\ub300\ub85c \ub9ac\uadf8 \ub370\ubdd4\ub97c \uae30\ub85d\ud55c\ub2e4.", "paragraph_sentence": "2008\ub144 6\uc6d4 10\uc77c \uc544\uc2a4\ub110, \ub9e8\uccb4\uc2a4\ud130 \uc720\ub098\uc774\ud2f0\ub4dc, \uc5d0\ubc84\ud2bc\uc758 \uacbd\uc7c1 \ub05d\uc5d0, \uc544\uc2a4\ub110\uc5d0 \ud569\ub958\ud558\uae30\ub85c \uacb0\uc815\ud588\ub2e4. \uc774 \uc774\uc801\uc740 \uce74\ub514\ud504 \uc2dc\ud2f0\uc5d0\uac8c 480\ub9cc \ud30c\uc6b4\ub4dc\uc758 \uc774\uc801\ub8cc\ub97c \uc548\uacbc\ub2e4. \uc774\uc801\uc740 6\uc6d4 13\uc77c \uc644\ub8cc\ub418\uc5c8\uc73c\uba70 \uc7a5\uae30\uacc4\uc57d\uc744 \uccb4\uacb0\ud55c \uac83\uc73c\ub85c \uc54c\ub824\uc84c\ub2e4. \ub2e4\ub978 \ud300\ub4e4 \ub300\uc2e0 \uc544\uc2a4\ub110\uc744 \ud0dd\ud55c \uc774\uc720\ub85c \uc2a4\uc704\uc2a4\uae4c\uc9c0 \ucc3e\uc544\uc640 \ub7a8\uc9c0\uc640 \uac00\uc871\ub4e4\uc5d0\uac8c \uc544\uc2a4\ub110\uc774 \uac00\uc9c0\uace0\uc788\ub294 \uacc4\ud68d\uacfc \ub7a8\uc9c0\uc5d0 \ub300\ud55c \uae30\ub300\ub97c \ubc1d\ud78c \uc544\ub974\uc13c \ubcb5\uac70 \uac10\ub3c5\uc758 \ub178\ub825\uc5d0 \uac10\uba85\ubc1b\uc558\ub2e4\uace0 \ud55c\ub2e4. \ubcb5\uac70\ub294 \ub7a8\uc9c0\ub97c \ud6cc\ub96d\ud55c \uccb4\ub825\uacfc \uc88b\uc740 \ud328\uc2a4\uc640 \uae30\uc220\uacfc \uc2dc\uc57c\ub97c \uc9c0\ub2cc \uc120\uc218\ub77c \ud3c9\uac00\ud588\ub2e4. 2008\ub144 8\uc6d4 13\uc77c, \ucc54\ud53c\uc5b8\uc2a4\ub9ac\uadf8 \uc608\uc120 3\ub77c\uc6b4\ub4dc FC \ud2b8\ubca4\ud14c\ub97c \uc0c1\ub300\ub85c \uc778\uc0c1\uc801\uc778 \ub370\ubdd4\uc804\uc744 \uce58\ub918\uc73c\uba70 \ud55c \ub2ec \ub4a4\uc778 9\uc6d4 13\uc77c \ube14\ub799\ubc88 \ub85c\ubc84\uc2a4\ub97c \uc0c1\ub300\ub85c \ub9ac\uadf8 \ub370\ubdd4\ub97c \uae30\ub85d\ud55c\ub2e4. \uc774 \ub9ac\uadf8 \ub370\ubdd4\uc804\uc5d0\uc11c \ub7a8\uc9c0\uc758 \uc5b4\uc2dc\uc2a4\ud2b8\ub85c \uc544\ub370\ubc14\uc694\ub974\uac00 \ud300\uc758 4\ubc88\uc9f8 \uace8\uc744 \uae30\ub85d\ud558\uba70 \ud574\ud2b8\ud2b8\ub9ad\uc744 \uc644\uc131\ud588\ub2e4. 9\uc6d4 23\uc77c \ub9ac\uadf8\ucef5 3\ub77c\uc6b4\ub4dc 6:0\uc73c\ub85c \ub300\uc2b9\uc744 \uac70\ub454 \uc170\ud544\ub4dc \uc720\ub098\uc774\ud2f0\ub4dc\uc640\uc758 \uacbd\uae30\uc5d0\uc11c \ub2c8\ud074\ub77c\uc2a4 \ubca4\ud2b8\ub108\uc640 \uce74\ub97c\ub85c\uc2a4 \ubca8\ub77c\uc758 \uace8\uc5d0 \uac01\uac01 \uc5b4\uc2dc\uc2a4\ud2b8\ub97c \uae30\ub85d\ud558\uba70 \uc5b4\uc2dc\uc2a4\ud2b8 2\uac1c\ub97c \ucd94\uac00\ud55c\ub2e4. 5:2\ub85c \uc2b9\ub9ac\ud55c \ud398\ub124\ub974\ubc14\ud750\uccb4\uc640\uc758 \ucc54\ud53c\uc5b8\uc2a4\ub9ac\uadf8 \uc870\ubcc4\ub9ac\uadf8 \uacbd\uae30\uc5d0\uc11c \uc67c\ucabd \uace8\ub300 \ub9de\uace0 \ub4e4\uc5b4\uac04 \uc7a5\uac70\ub9ac \uace8\ub85c \uccab \uace8\uc744 \uae30\ub85d\ud588\ub2e4. \uc774 \uace8\ub85c \ucc54\ud53c\uc5b8\uc2a4\ub9ac\uadf8 \uc5ed\uc0ac\uc0c1 5\ubc88\uc9f8\ub85c \uc5b4\ub9b0 \ub4dd\uc810\uc744 \uae30\ub85d\ud55c \uc120\uc218\uac00 \ub418\uc5c8\uc73c\uba70, 90\ub144\ub300 \ud0dc\uc5b4\ub09c \uc120\uc218 \uc911 \ucc54\ud53c\uc5b8\uc2a4\ub9ac\uadf8 \uace8\uc744 \uae30\ub85d\ud55c 2\ubc88\uc9f8 \uc120\uc218\uac00 \ub418\uc5c8\ub2e4. 2009\ub144 \ud53c\ud30c\uc5d0\uc11c \uc120\uc815\ud55c \uc8fc\ubaa9\ud574\uc57c \ud560 \uc720\ub9dd\uc8fc\uc5d0 \uc774\ub984\uc744 \uc62c\ub9ac\uae30\ub3c4 \ud588\ub2e4.", "paragraph_answer": "2008\ub144 6\uc6d4 10\uc77c \uc544\uc2a4\ub110, \ub9e8\uccb4\uc2a4\ud130 \uc720\ub098\uc774\ud2f0\ub4dc, \uc5d0\ubc84\ud2bc\uc758 \uacbd\uc7c1 \ub05d\uc5d0, \uc544\uc2a4\ub110\uc5d0 \ud569\ub958\ud558\uae30\ub85c \uacb0\uc815\ud588\ub2e4. \uc774 \uc774\uc801\uc740 \uce74\ub514\ud504 \uc2dc\ud2f0\uc5d0\uac8c 480\ub9cc \ud30c\uc6b4\ub4dc\uc758 \uc774\uc801\ub8cc\ub97c \uc548\uacbc\ub2e4. \uc774\uc801\uc740 6\uc6d4 13\uc77c \uc644\ub8cc\ub418\uc5c8\uc73c\uba70 \uc7a5\uae30\uacc4\uc57d\uc744 \uccb4\uacb0\ud55c \uac83\uc73c\ub85c \uc54c\ub824\uc84c\ub2e4. \ub2e4\ub978 \ud300\ub4e4 \ub300\uc2e0 \uc544\uc2a4\ub110\uc744 \ud0dd\ud55c \uc774\uc720\ub85c \uc2a4\uc704\uc2a4\uae4c\uc9c0 \ucc3e\uc544\uc640 \ub7a8\uc9c0\uc640 \uac00\uc871\ub4e4\uc5d0\uac8c \uc544\uc2a4\ub110\uc774 \uac00\uc9c0\uace0\uc788\ub294 \uacc4\ud68d\uacfc \ub7a8\uc9c0\uc5d0 \ub300\ud55c \uae30\ub300\ub97c \ubc1d\ud78c \uc544\ub974\uc13c \ubcb5\uac70 \uac10\ub3c5\uc758 \ub178\ub825\uc5d0 \uac10\uba85\ubc1b\uc558\ub2e4\uace0 \ud55c\ub2e4. \ubcb5\uac70\ub294 \ub7a8\uc9c0\ub97c \ud6cc\ub96d\ud55c \uccb4\ub825\uacfc \uc88b\uc740 \ud328\uc2a4\uc640 \uae30\uc220\uacfc \uc2dc\uc57c\ub97c \uc9c0\ub2cc \uc120\uc218\ub77c \ud3c9\uac00\ud588\ub2e4. 2008\ub144 8\uc6d4 13\uc77c, \ucc54\ud53c\uc5b8\uc2a4\ub9ac\uadf8 \uc608\uc120 3\ub77c\uc6b4\ub4dc FC \ud2b8\ubca4\ud14c\ub97c \uc0c1\ub300\ub85c \uc778\uc0c1\uc801\uc778 \ub370\ubdd4\uc804\uc744 \uce58\ub918\uc73c\uba70 \ud55c \ub2ec \ub4a4\uc778 9\uc6d4 13\uc77c \ube14\ub799\ubc88 \ub85c\ubc84\uc2a4\ub97c \uc0c1\ub300\ub85c \ub9ac\uadf8 \ub370\ubdd4\ub97c \uae30\ub85d\ud55c\ub2e4. \uc774 \ub9ac\uadf8 \ub370\ubdd4\uc804\uc5d0\uc11c \ub7a8\uc9c0\uc758 \uc5b4\uc2dc\uc2a4\ud2b8\ub85c \uc544\ub370\ubc14\uc694\ub974\uac00 \ud300\uc758 4\ubc88\uc9f8 \uace8\uc744 \uae30\ub85d\ud558\uba70 \ud574\ud2b8\ud2b8\ub9ad\uc744 \uc644\uc131\ud588\ub2e4. 9\uc6d4 23\uc77c \ub9ac\uadf8\ucef5 3\ub77c\uc6b4\ub4dc 6:0\uc73c\ub85c \ub300\uc2b9\uc744 \uac70\ub454 \uc170\ud544\ub4dc \uc720\ub098\uc774\ud2f0\ub4dc\uc640\uc758 \uacbd\uae30\uc5d0\uc11c \ub2c8\ud074\ub77c\uc2a4 \ubca4\ud2b8\ub108\uc640 \uce74\ub97c\ub85c\uc2a4 \ubca8\ub77c\uc758 \uace8\uc5d0 \uac01\uac01 \uc5b4\uc2dc\uc2a4\ud2b8\ub97c \uae30\ub85d\ud558\uba70 \uc5b4\uc2dc\uc2a4\ud2b8 2\uac1c\ub97c \ucd94\uac00\ud55c\ub2e4. 5:2\ub85c \uc2b9\ub9ac\ud55c \ud398\ub124\ub974\ubc14\ud750\uccb4\uc640\uc758 \ucc54\ud53c\uc5b8\uc2a4\ub9ac\uadf8 \uc870\ubcc4\ub9ac\uadf8 \uacbd\uae30\uc5d0\uc11c \uc67c\ucabd \uace8\ub300 \ub9de\uace0 \ub4e4\uc5b4\uac04 \uc7a5\uac70\ub9ac \uace8\ub85c \uccab \uace8\uc744 \uae30\ub85d\ud588\ub2e4. \uc774 \uace8\ub85c \ucc54\ud53c\uc5b8\uc2a4\ub9ac\uadf8 \uc5ed\uc0ac\uc0c1 5\ubc88\uc9f8\ub85c \uc5b4\ub9b0 \ub4dd\uc810\uc744 \uae30\ub85d\ud55c \uc120\uc218\uac00 \ub418\uc5c8\uc73c\uba70, 90\ub144\ub300 \ud0dc\uc5b4\ub09c \uc120\uc218 \uc911 \ucc54\ud53c\uc5b8\uc2a4\ub9ac\uadf8 \uace8\uc744 \uae30\ub85d\ud55c 2\ubc88\uc9f8 \uc120\uc218\uac00 \ub418\uc5c8\ub2e4. 2009\ub144 \ud53c\ud30c\uc5d0\uc11c \uc120\uc815\ud55c \uc8fc\ubaa9\ud574\uc57c \ud560 \uc720\ub9dd\uc8fc\uc5d0 \uc774\ub984\uc744 \uc62c\ub9ac\uae30\ub3c4 \ud588\ub2e4.", "sentence_answer": "2008\ub144 8\uc6d4 13\uc77c, \ucc54\ud53c\uc5b8\uc2a4\ub9ac\uadf8 \uc608\uc120 3\ub77c\uc6b4\ub4dc FC \ud2b8\ubca4\ud14c\ub97c \uc0c1\ub300\ub85c \uc778\uc0c1\uc801\uc778 \ub370\ubdd4\uc804\uc744 \uce58\ub918\uc73c\uba70 \ud55c \ub2ec \ub4a4\uc778 9\uc6d4 13\uc77c \ube14\ub799\ubc88 \ub85c\ubc84\uc2a4\ub97c \uc0c1\ub300\ub85c \ub9ac\uadf8 \ub370\ubdd4\ub97c \uae30\ub85d\ud55c\ub2e4.", "paragraph_id": "6586552-1-2", "paragraph_question": "question: \ube14\ub799\ubc88 \ub85c\ubc84\uc2a4\ub97c \uc0c1\ub300\ub85c \ub9ac\uadf8 \ub370\ubdd4\ub97c \uae30\ub85d\ud55c \ub54c\ub294?, context: 2008\ub144 6\uc6d4 10\uc77c \uc544\uc2a4\ub110, \ub9e8\uccb4\uc2a4\ud130 \uc720\ub098\uc774\ud2f0\ub4dc, \uc5d0\ubc84\ud2bc\uc758 \uacbd\uc7c1 \ub05d\uc5d0, \uc544\uc2a4\ub110\uc5d0 \ud569\ub958\ud558\uae30\ub85c \uacb0\uc815\ud588\ub2e4. \uc774 \uc774\uc801\uc740 \uce74\ub514\ud504 \uc2dc\ud2f0\uc5d0\uac8c 480\ub9cc \ud30c\uc6b4\ub4dc\uc758 \uc774\uc801\ub8cc\ub97c \uc548\uacbc\ub2e4. \uc774\uc801\uc740 6\uc6d4 13\uc77c \uc644\ub8cc\ub418\uc5c8\uc73c\uba70 \uc7a5\uae30\uacc4\uc57d\uc744 \uccb4\uacb0\ud55c \uac83\uc73c\ub85c \uc54c\ub824\uc84c\ub2e4. \ub2e4\ub978 \ud300\ub4e4 \ub300\uc2e0 \uc544\uc2a4\ub110\uc744 \ud0dd\ud55c \uc774\uc720\ub85c \uc2a4\uc704\uc2a4\uae4c\uc9c0 \ucc3e\uc544\uc640 \ub7a8\uc9c0\uc640 \uac00\uc871\ub4e4\uc5d0\uac8c \uc544\uc2a4\ub110\uc774 \uac00\uc9c0\uace0\uc788\ub294 \uacc4\ud68d\uacfc \ub7a8\uc9c0\uc5d0 \ub300\ud55c \uae30\ub300\ub97c \ubc1d\ud78c \uc544\ub974\uc13c \ubcb5\uac70 \uac10\ub3c5\uc758 \ub178\ub825\uc5d0 \uac10\uba85\ubc1b\uc558\ub2e4\uace0 \ud55c\ub2e4. \ubcb5\uac70\ub294 \ub7a8\uc9c0\ub97c \ud6cc\ub96d\ud55c \uccb4\ub825\uacfc \uc88b\uc740 \ud328\uc2a4\uc640 \uae30\uc220\uacfc \uc2dc\uc57c\ub97c \uc9c0\ub2cc \uc120\uc218\ub77c \ud3c9\uac00\ud588\ub2e4. 2008\ub144 8\uc6d4 13\uc77c, \ucc54\ud53c\uc5b8\uc2a4\ub9ac\uadf8 \uc608\uc120 3\ub77c\uc6b4\ub4dc FC \ud2b8\ubca4\ud14c\ub97c \uc0c1\ub300\ub85c \uc778\uc0c1\uc801\uc778 \ub370\ubdd4\uc804\uc744 \uce58\ub918\uc73c\uba70 \ud55c \ub2ec \ub4a4\uc778 9\uc6d4 13\uc77c \ube14\ub799\ubc88 \ub85c\ubc84\uc2a4\ub97c \uc0c1\ub300\ub85c \ub9ac\uadf8 \ub370\ubdd4\ub97c \uae30\ub85d\ud55c\ub2e4. \uc774 \ub9ac\uadf8 \ub370\ubdd4\uc804\uc5d0\uc11c \ub7a8\uc9c0\uc758 \uc5b4\uc2dc\uc2a4\ud2b8\ub85c \uc544\ub370\ubc14\uc694\ub974\uac00 \ud300\uc758 4\ubc88\uc9f8 \uace8\uc744 \uae30\ub85d\ud558\uba70 \ud574\ud2b8\ud2b8\ub9ad\uc744 \uc644\uc131\ud588\ub2e4. 9\uc6d4 23\uc77c \ub9ac\uadf8\ucef5 3\ub77c\uc6b4\ub4dc 6:0\uc73c\ub85c \ub300\uc2b9\uc744 \uac70\ub454 \uc170\ud544\ub4dc \uc720\ub098\uc774\ud2f0\ub4dc\uc640\uc758 \uacbd\uae30\uc5d0\uc11c \ub2c8\ud074\ub77c\uc2a4 \ubca4\ud2b8\ub108\uc640 \uce74\ub97c\ub85c\uc2a4 \ubca8\ub77c\uc758 \uace8\uc5d0 \uac01\uac01 \uc5b4\uc2dc\uc2a4\ud2b8\ub97c \uae30\ub85d\ud558\uba70 \uc5b4\uc2dc\uc2a4\ud2b8 2\uac1c\ub97c \ucd94\uac00\ud55c\ub2e4. 5:2\ub85c \uc2b9\ub9ac\ud55c \ud398\ub124\ub974\ubc14\ud750\uccb4\uc640\uc758 \ucc54\ud53c\uc5b8\uc2a4\ub9ac\uadf8 \uc870\ubcc4\ub9ac\uadf8 \uacbd\uae30\uc5d0\uc11c \uc67c\ucabd \uace8\ub300 \ub9de\uace0 \ub4e4\uc5b4\uac04 \uc7a5\uac70\ub9ac \uace8\ub85c \uccab \uace8\uc744 \uae30\ub85d\ud588\ub2e4. \uc774 \uace8\ub85c \ucc54\ud53c\uc5b8\uc2a4\ub9ac\uadf8 \uc5ed\uc0ac\uc0c1 5\ubc88\uc9f8\ub85c \uc5b4\ub9b0 \ub4dd\uc810\uc744 \uae30\ub85d\ud55c \uc120\uc218\uac00 \ub418\uc5c8\uc73c\uba70, 90\ub144\ub300 \ud0dc\uc5b4\ub09c \uc120\uc218 \uc911 \ucc54\ud53c\uc5b8\uc2a4\ub9ac\uadf8 \uace8\uc744 \uae30\ub85d\ud55c 2\ubc88\uc9f8 \uc120\uc218\uac00 \ub418\uc5c8\ub2e4. 2009\ub144 \ud53c\ud30c\uc5d0\uc11c \uc120\uc815\ud55c \uc8fc\ubaa9\ud574\uc57c \ud560 \uc720\ub9dd\uc8fc\uc5d0 \uc774\ub984\uc744 \uc62c\ub9ac\uae30\ub3c4 \ud588\ub2e4."} {"question": "\uc721\uc601\uc218\uac00 1969\ub144 \uc124\ub9bd\ud55c \uc5b4\ub9b0\uc774 \ubcf5\uc9c0\uc7ac\ub2e8\uc758 \uc774\ub984\uc740?", "paragraph": "\uc721\uc601\uc218\ub294 \uc804\uad6d \ud55c\uc13c\ubcd1 \ud658\uc790(\ub098\ud658\uc790)\ucd0c\uc744 \uc9c1\uc811 \ubc29\ubb38\ud558\uc5ec \uc790\ud65c\uc0ac\uc5c5\uc744 \uc9c0\uc6d0\ud558\uc600\uc73c\uba70, 1964\ub144 9\uc6d4\ubd80\ud130 \ubca0\ud2b8\ub0a8\uc5d0 \ud30c\ubcd1\ub41c \ud30c\uc6d4 \uc7a5\ubcd1 \uac00\uc871\ub4e4\uc744 \ucc3e\uc544 \uc704\ub85c, \uc704\ubb38\ud558\uc600\ub2e4. \ub610\ud55c \ud48d\uc218\ud574 \ud604\uc7a5\uc5d0 \ube44\ub97c \ub9de\uc73c\uba70 \ub2ec\ub824\uac00 \uc7ac\ub09c\ub2f9\ud55c \uc0ac\ub78c\ub4e4\uc744 \uc704\ubb38\ud558\uc600\ub2e4. 1968\ub144 \uc11c\uc6b8\ub300\ud559\uad50\uc5d0 \uae30\uc219\uc0ac \uc815\uc601\uc0ac\uc758 \uc124\uce58\ub97c \uc8fc\uad00\ud588\uace0 1969\ub144 4\uc6d4 \uc5b4\ub9b0\uc774 \ubcf5\uc9c0\uc7ac\ub2e8\uc778 \uc721\uc601\uc7ac\ub2e8\uc744 \uc124\ub9bd\ud558\uc600\uc73c\uba70 \uc5b4\ub9b0\uc774\ub0a0\uc5d0 \ub9de\ucdb0 \uc5b4\ub9b0\uc774\ub300\uacf5\uc6d0, \uc5b4\ub9b0\uc774\ud68c\uad00\uc758 \uac74\ub9bd\uc744 \uc8fc\ub3c4\ud558\uc600\ub2e4. 1972\ub144\uc5d0\ub294 \ubd80\uc0b0 \uc5b4\ub9b0\uc774\ud68c\uad00\uc744 \uae30\uacf5\ud558\uc600\ub2e4. \uc721\uc601\uc218\ub294 \uc9c1\uc811 \uc18c\ub144\uc18c\ub140 \uc7a1\uc9c0 \u2018\uc5b4\uae68\ub3d9\ubb34\u2019\ub97c \ubc1c\uac04\ud558\uc5ec \ub18d\uc5b4\ucd0c \uc5b4\ub9b0\uc774\uc5d0\uac8c\uae4c\uc9c0 \ubc30\ud3ec\ud558\uace0, 1973\ub144 \ubd88\uc6b0 \uccad\uc18c\ub144\uacfc \ube48\uc790\ub4e4\uc744 \ub300\uc0c1\uc73c\ub85c \ud55c \uc9c1\uc5c5\ud6c8\ub828 \uae30\uad00 \uc815\uc218\uc9c1\uc5c5\ud6c8\ub828\uc6d0\uc744 \uc124\uce58\ud558\uc600\ub2e4. \uc721\uc601\uc218\uac00 \uc774\ub807\ub4ef \ud65c\ubc1c\ud55c \uc0ac\ud68c\ubd09\uc0ac\ub97c \ud55c \uc774\uc720\ub294 \uc9c1\uc5c5\uc744 \uac16\ub3c4\ub85d \ud558\ub294 \uc9c1\uc5c5\ud6c8\ub828 \uc704\uc8fc\uc758 \uc0ac\ud68c\ubcf5\uc9c0\uac00 \ube48\uace4\ud1f4\uce58\uc758 \uae38\uc784\uc744 \uc54c\uace0 \uc788\uc5c8\uae30 \ub54c\ubb38\uc774\ub2e4.", "answer": "\uc721\uc601\uc7ac\ub2e8", "sentence": "1969\ub144 4\uc6d4 \uc5b4\ub9b0\uc774 \ubcf5\uc9c0\uc7ac\ub2e8\uc778 \uc721\uc601\uc7ac\ub2e8 \uc744 \uc124\ub9bd\ud558\uc600\uc73c\uba70 \uc5b4\ub9b0\uc774\ub0a0\uc5d0 \ub9de\ucdb0 \uc5b4\ub9b0\uc774\ub300\uacf5\uc6d0, \uc5b4\ub9b0\uc774\ud68c\uad00\uc758 \uac74\ub9bd\uc744 \uc8fc\ub3c4\ud558\uc600\ub2e4.", "paragraph_sentence": "\uc721\uc601\uc218\ub294 \uc804\uad6d \ud55c\uc13c\ubcd1 \ud658\uc790(\ub098\ud658\uc790)\ucd0c\uc744 \uc9c1\uc811 \ubc29\ubb38\ud558\uc5ec \uc790\ud65c\uc0ac\uc5c5\uc744 \uc9c0\uc6d0\ud558\uc600\uc73c\uba70, 1964\ub144 9\uc6d4\ubd80\ud130 \ubca0\ud2b8\ub0a8\uc5d0 \ud30c\ubcd1\ub41c \ud30c\uc6d4 \uc7a5\ubcd1 \uac00\uc871\ub4e4\uc744 \ucc3e\uc544 \uc704\ub85c, \uc704\ubb38\ud558\uc600\ub2e4. \ub610\ud55c \ud48d\uc218\ud574 \ud604\uc7a5\uc5d0 \ube44\ub97c \ub9de\uc73c\uba70 \ub2ec\ub824\uac00 \uc7ac\ub09c\ub2f9\ud55c \uc0ac\ub78c\ub4e4\uc744 \uc704\ubb38\ud558\uc600\ub2e4. 1968\ub144 \uc11c\uc6b8\ub300\ud559\uad50\uc5d0 \uae30\uc219\uc0ac \uc815\uc601\uc0ac\uc758 \uc124\uce58\ub97c \uc8fc\uad00\ud588\uace0 1969\ub144 4\uc6d4 \uc5b4\ub9b0\uc774 \ubcf5\uc9c0\uc7ac\ub2e8\uc778 \uc721\uc601\uc7ac\ub2e8 \uc744 \uc124\ub9bd\ud558\uc600\uc73c\uba70 \uc5b4\ub9b0\uc774\ub0a0\uc5d0 \ub9de\ucdb0 \uc5b4\ub9b0\uc774\ub300\uacf5\uc6d0, \uc5b4\ub9b0\uc774\ud68c\uad00\uc758 \uac74\ub9bd\uc744 \uc8fc\ub3c4\ud558\uc600\ub2e4. 1972\ub144\uc5d0\ub294 \ubd80\uc0b0 \uc5b4\ub9b0\uc774\ud68c\uad00\uc744 \uae30\uacf5\ud558\uc600\ub2e4. \uc721\uc601\uc218\ub294 \uc9c1\uc811 \uc18c\ub144\uc18c\ub140 \uc7a1\uc9c0 \u2018\uc5b4\uae68\ub3d9\ubb34\u2019\ub97c \ubc1c\uac04\ud558\uc5ec \ub18d\uc5b4\ucd0c \uc5b4\ub9b0\uc774\uc5d0\uac8c\uae4c\uc9c0 \ubc30\ud3ec\ud558\uace0, 1973\ub144 \ubd88\uc6b0 \uccad\uc18c\ub144\uacfc \ube48\uc790\ub4e4\uc744 \ub300\uc0c1\uc73c\ub85c \ud55c \uc9c1\uc5c5\ud6c8\ub828 \uae30\uad00 \uc815\uc218\uc9c1\uc5c5\ud6c8\ub828\uc6d0\uc744 \uc124\uce58\ud558\uc600\ub2e4. \uc721\uc601\uc218\uac00 \uc774\ub807\ub4ef \ud65c\ubc1c\ud55c \uc0ac\ud68c\ubd09\uc0ac\ub97c \ud55c \uc774\uc720\ub294 \uc9c1\uc5c5\uc744 \uac16\ub3c4\ub85d \ud558\ub294 \uc9c1\uc5c5\ud6c8\ub828 \uc704\uc8fc\uc758 \uc0ac\ud68c\ubcf5\uc9c0\uac00 \ube48\uace4\ud1f4\uce58\uc758 \uae38\uc784\uc744 \uc54c\uace0 \uc788\uc5c8\uae30 \ub54c\ubb38\uc774\ub2e4.", "paragraph_answer": "\uc721\uc601\uc218\ub294 \uc804\uad6d \ud55c\uc13c\ubcd1 \ud658\uc790(\ub098\ud658\uc790)\ucd0c\uc744 \uc9c1\uc811 \ubc29\ubb38\ud558\uc5ec \uc790\ud65c\uc0ac\uc5c5\uc744 \uc9c0\uc6d0\ud558\uc600\uc73c\uba70, 1964\ub144 9\uc6d4\ubd80\ud130 \ubca0\ud2b8\ub0a8\uc5d0 \ud30c\ubcd1\ub41c \ud30c\uc6d4 \uc7a5\ubcd1 \uac00\uc871\ub4e4\uc744 \ucc3e\uc544 \uc704\ub85c, \uc704\ubb38\ud558\uc600\ub2e4. \ub610\ud55c \ud48d\uc218\ud574 \ud604\uc7a5\uc5d0 \ube44\ub97c \ub9de\uc73c\uba70 \ub2ec\ub824\uac00 \uc7ac\ub09c\ub2f9\ud55c \uc0ac\ub78c\ub4e4\uc744 \uc704\ubb38\ud558\uc600\ub2e4. 1968\ub144 \uc11c\uc6b8\ub300\ud559\uad50\uc5d0 \uae30\uc219\uc0ac \uc815\uc601\uc0ac\uc758 \uc124\uce58\ub97c \uc8fc\uad00\ud588\uace0 1969\ub144 4\uc6d4 \uc5b4\ub9b0\uc774 \ubcf5\uc9c0\uc7ac\ub2e8\uc778 \uc721\uc601\uc7ac\ub2e8 \uc744 \uc124\ub9bd\ud558\uc600\uc73c\uba70 \uc5b4\ub9b0\uc774\ub0a0\uc5d0 \ub9de\ucdb0 \uc5b4\ub9b0\uc774\ub300\uacf5\uc6d0, \uc5b4\ub9b0\uc774\ud68c\uad00\uc758 \uac74\ub9bd\uc744 \uc8fc\ub3c4\ud558\uc600\ub2e4. 1972\ub144\uc5d0\ub294 \ubd80\uc0b0 \uc5b4\ub9b0\uc774\ud68c\uad00\uc744 \uae30\uacf5\ud558\uc600\ub2e4. \uc721\uc601\uc218\ub294 \uc9c1\uc811 \uc18c\ub144\uc18c\ub140 \uc7a1\uc9c0 \u2018\uc5b4\uae68\ub3d9\ubb34\u2019\ub97c \ubc1c\uac04\ud558\uc5ec \ub18d\uc5b4\ucd0c \uc5b4\ub9b0\uc774\uc5d0\uac8c\uae4c\uc9c0 \ubc30\ud3ec\ud558\uace0, 1973\ub144 \ubd88\uc6b0 \uccad\uc18c\ub144\uacfc \ube48\uc790\ub4e4\uc744 \ub300\uc0c1\uc73c\ub85c \ud55c \uc9c1\uc5c5\ud6c8\ub828 \uae30\uad00 \uc815\uc218\uc9c1\uc5c5\ud6c8\ub828\uc6d0\uc744 \uc124\uce58\ud558\uc600\ub2e4. \uc721\uc601\uc218\uac00 \uc774\ub807\ub4ef \ud65c\ubc1c\ud55c \uc0ac\ud68c\ubd09\uc0ac\ub97c \ud55c \uc774\uc720\ub294 \uc9c1\uc5c5\uc744 \uac16\ub3c4\ub85d \ud558\ub294 \uc9c1\uc5c5\ud6c8\ub828 \uc704\uc8fc\uc758 \uc0ac\ud68c\ubcf5\uc9c0\uac00 \ube48\uace4\ud1f4\uce58\uc758 \uae38\uc784\uc744 \uc54c\uace0 \uc788\uc5c8\uae30 \ub54c\ubb38\uc774\ub2e4.", "sentence_answer": "1969\ub144 4\uc6d4 \uc5b4\ub9b0\uc774 \ubcf5\uc9c0\uc7ac\ub2e8\uc778 \uc721\uc601\uc7ac\ub2e8 \uc744 \uc124\ub9bd\ud558\uc600\uc73c\uba70 \uc5b4\ub9b0\uc774\ub0a0\uc5d0 \ub9de\ucdb0 \uc5b4\ub9b0\uc774\ub300\uacf5\uc6d0, \uc5b4\ub9b0\uc774\ud68c\uad00\uc758 \uac74\ub9bd\uc744 \uc8fc\ub3c4\ud558\uc600\ub2e4.", "paragraph_id": "6543705-0-2", "paragraph_question": "question: \uc721\uc601\uc218\uac00 1969\ub144 \uc124\ub9bd\ud55c \uc5b4\ub9b0\uc774 \ubcf5\uc9c0\uc7ac\ub2e8\uc758 \uc774\ub984\uc740?, context: \uc721\uc601\uc218\ub294 \uc804\uad6d \ud55c\uc13c\ubcd1 \ud658\uc790(\ub098\ud658\uc790)\ucd0c\uc744 \uc9c1\uc811 \ubc29\ubb38\ud558\uc5ec \uc790\ud65c\uc0ac\uc5c5\uc744 \uc9c0\uc6d0\ud558\uc600\uc73c\uba70, 1964\ub144 9\uc6d4\ubd80\ud130 \ubca0\ud2b8\ub0a8\uc5d0 \ud30c\ubcd1\ub41c \ud30c\uc6d4 \uc7a5\ubcd1 \uac00\uc871\ub4e4\uc744 \ucc3e\uc544 \uc704\ub85c, \uc704\ubb38\ud558\uc600\ub2e4. \ub610\ud55c \ud48d\uc218\ud574 \ud604\uc7a5\uc5d0 \ube44\ub97c \ub9de\uc73c\uba70 \ub2ec\ub824\uac00 \uc7ac\ub09c\ub2f9\ud55c \uc0ac\ub78c\ub4e4\uc744 \uc704\ubb38\ud558\uc600\ub2e4. 1968\ub144 \uc11c\uc6b8\ub300\ud559\uad50\uc5d0 \uae30\uc219\uc0ac \uc815\uc601\uc0ac\uc758 \uc124\uce58\ub97c \uc8fc\uad00\ud588\uace0 1969\ub144 4\uc6d4 \uc5b4\ub9b0\uc774 \ubcf5\uc9c0\uc7ac\ub2e8\uc778 \uc721\uc601\uc7ac\ub2e8\uc744 \uc124\ub9bd\ud558\uc600\uc73c\uba70 \uc5b4\ub9b0\uc774\ub0a0\uc5d0 \ub9de\ucdb0 \uc5b4\ub9b0\uc774\ub300\uacf5\uc6d0, \uc5b4\ub9b0\uc774\ud68c\uad00\uc758 \uac74\ub9bd\uc744 \uc8fc\ub3c4\ud558\uc600\ub2e4. 1972\ub144\uc5d0\ub294 \ubd80\uc0b0 \uc5b4\ub9b0\uc774\ud68c\uad00\uc744 \uae30\uacf5\ud558\uc600\ub2e4. \uc721\uc601\uc218\ub294 \uc9c1\uc811 \uc18c\ub144\uc18c\ub140 \uc7a1\uc9c0 \u2018\uc5b4\uae68\ub3d9\ubb34\u2019\ub97c \ubc1c\uac04\ud558\uc5ec \ub18d\uc5b4\ucd0c \uc5b4\ub9b0\uc774\uc5d0\uac8c\uae4c\uc9c0 \ubc30\ud3ec\ud558\uace0, 1973\ub144 \ubd88\uc6b0 \uccad\uc18c\ub144\uacfc \ube48\uc790\ub4e4\uc744 \ub300\uc0c1\uc73c\ub85c \ud55c \uc9c1\uc5c5\ud6c8\ub828 \uae30\uad00 \uc815\uc218\uc9c1\uc5c5\ud6c8\ub828\uc6d0\uc744 \uc124\uce58\ud558\uc600\ub2e4. \uc721\uc601\uc218\uac00 \uc774\ub807\ub4ef \ud65c\ubc1c\ud55c \uc0ac\ud68c\ubd09\uc0ac\ub97c \ud55c \uc774\uc720\ub294 \uc9c1\uc5c5\uc744 \uac16\ub3c4\ub85d \ud558\ub294 \uc9c1\uc5c5\ud6c8\ub828 \uc704\uc8fc\uc758 \uc0ac\ud68c\ubcf5\uc9c0\uac00 \ube48\uace4\ud1f4\uce58\uc758 \uae38\uc784\uc744 \uc54c\uace0 \uc788\uc5c8\uae30 \ub54c\ubb38\uc774\ub2e4."} {"question": "TSPM\uc774 \uc7ac\uc870\uc9c1\ub41c \ud574\ub294?", "paragraph": "TSPM\uc740 \ubb38\ud654\ud601\uba85\uc774 \uc2dc\uc791\ub418\ub358 \ud574\uc778 1966\ub144\uc5d0 \ud574\uccb4\ub418\uc5c8\ub2e4. TSPM\uc774 \uc7ac\uc870\uc9c1\ub418\ub358 \ud574\uc778 1979\ub144 \ucd08\uc5d0\ub294 \uadf8 \uc601\ud5a5\ub825\uc774 \ubca0\uc774\uc9d5\uc774\ub098 \uc0c1\ud558\uc774 \ub4f1\uc758 \ub300\ub3c4\uc2dc \uc9c0\uc5ed\uc73c\ub85c \uc81c\ud55c\ub418\uc5b4 \uc788\uc5c8\ub358 \ubc18\uba74\uc5d0 \uae30\ub3c5\uad50 \uc2e0\uc559\uc740 \ud5c8\ub09c, \uc800\uc7a5, \ud478\uc820, \uc548\ud6c4\uc774\uc640 \uac19\uc740 \uc9c0\uc5ed\uc5d0\uc11c \uae09\uc18d\ud788 \ud655\uc0b0\ub418\uc5c8\ub2e4. \uc911\uad6d \ub0b4\uc758 \ub9ce\uc740 \uadf8\ub9ac\uc2a4\ub3c4\uc778\ub4e4\uc774 TSPM\uc744 \uc758\uc2ec\ud588\ub294\ub370, \uadf8 \uc774\uc720\ub294 TSPM\uc774 \ud615\uc131\ub420 \ucd08\uae30\uc778 1950\ub144\ub300\uc5d0 \ud604\ub300\uc8fc\uc758 \uc2e0\ud559\uc744 \uac15\uc870\ud588\uc744 \ubfd0 \uc544\ub2c8\ub77c \uc911\uad6d \uc815\ubd80\uac00 \ub9ce\uc740 \uc131\uacbd \uad50\uc0ac\ub4e4\uc744 \uc5b5\uc555\ud558\uace0 \ubc15\ud574\ud560 \ub54c \uadf8\ub4e4\uc774 \ud611\ub825\ud588\uae30 \ub54c\ubb38\uc774\ub2e4. \uc774\ub7ec\ud55c \uadf8\ub9ac\uc2a4\ub3c4\uc778\ub4e4\uc740 1979\ub144\uc5d0 TSPM\uc774 \uc7ac\uc124\ub9bd\ub41c \uac83\uc740 \uc815\ubd80\uac00 \uc790\uc2e0\ub4e4\uc5d0 \ub300\ud55c \ud1b5\uc81c\ub97c \uac15\ud654\ud558\uace0 \uae30\ub3c5\uad50 \ubcf5\uc74c\uc774 \ud655\uc0b0\ub418\ub294 \uac83\uc744 \ub2a6\ucd94\ub824\ub294 \uc2dc\ub3c4\ub85c \ubcf4\uc558\ub2e4.", "answer": "1979\ub144", "sentence": "TSPM\uc774 \uc7ac\uc870\uc9c1\ub418\ub358 \ud574\uc778 1979\ub144 \ucd08\uc5d0\ub294 \uadf8 \uc601\ud5a5\ub825\uc774 \ubca0\uc774\uc9d5\uc774\ub098 \uc0c1\ud558\uc774 \ub4f1\uc758 \ub300\ub3c4\uc2dc \uc9c0\uc5ed\uc73c\ub85c \uc81c\ud55c\ub418\uc5b4 \uc788\uc5c8\ub358 \ubc18\uba74\uc5d0 \uae30\ub3c5\uad50 \uc2e0\uc559\uc740 \ud5c8\ub09c, \uc800\uc7a5, \ud478\uc820, \uc548\ud6c4\uc774\uc640 \uac19\uc740 \uc9c0\uc5ed\uc5d0\uc11c \uae09\uc18d\ud788 \ud655\uc0b0\ub418\uc5c8\ub2e4.", "paragraph_sentence": "TSPM\uc740 \ubb38\ud654\ud601\uba85\uc774 \uc2dc\uc791\ub418\ub358 \ud574\uc778 1966\ub144\uc5d0 \ud574\uccb4\ub418\uc5c8\ub2e4. TSPM\uc774 \uc7ac\uc870\uc9c1\ub418\ub358 \ud574\uc778 1979\ub144 \ucd08\uc5d0\ub294 \uadf8 \uc601\ud5a5\ub825\uc774 \ubca0\uc774\uc9d5\uc774\ub098 \uc0c1\ud558\uc774 \ub4f1\uc758 \ub300\ub3c4\uc2dc \uc9c0\uc5ed\uc73c\ub85c \uc81c\ud55c\ub418\uc5b4 \uc788\uc5c8\ub358 \ubc18\uba74\uc5d0 \uae30\ub3c5\uad50 \uc2e0\uc559\uc740 \ud5c8\ub09c, \uc800\uc7a5, \ud478\uc820, \uc548\ud6c4\uc774\uc640 \uac19\uc740 \uc9c0\uc5ed\uc5d0\uc11c \uae09\uc18d\ud788 \ud655\uc0b0\ub418\uc5c8\ub2e4. \uc911\uad6d \ub0b4\uc758 \ub9ce\uc740 \uadf8\ub9ac\uc2a4\ub3c4\uc778\ub4e4\uc774 TSPM\uc744 \uc758\uc2ec\ud588\ub294\ub370, \uadf8 \uc774\uc720\ub294 TSPM\uc774 \ud615\uc131\ub420 \ucd08\uae30\uc778 1950\ub144\ub300\uc5d0 \ud604\ub300\uc8fc\uc758 \uc2e0\ud559\uc744 \uac15\uc870\ud588\uc744 \ubfd0 \uc544\ub2c8\ub77c \uc911\uad6d \uc815\ubd80\uac00 \ub9ce\uc740 \uc131\uacbd \uad50\uc0ac\ub4e4\uc744 \uc5b5\uc555\ud558\uace0 \ubc15\ud574\ud560 \ub54c \uadf8\ub4e4\uc774 \ud611\ub825\ud588\uae30 \ub54c\ubb38\uc774\ub2e4. \uc774\ub7ec\ud55c \uadf8\ub9ac\uc2a4\ub3c4\uc778\ub4e4\uc740 1979\ub144\uc5d0 TSPM\uc774 \uc7ac\uc124\ub9bd\ub41c \uac83\uc740 \uc815\ubd80\uac00 \uc790\uc2e0\ub4e4\uc5d0 \ub300\ud55c \ud1b5\uc81c\ub97c \uac15\ud654\ud558\uace0 \uae30\ub3c5\uad50 \ubcf5\uc74c\uc774 \ud655\uc0b0\ub418\ub294 \uac83\uc744 \ub2a6\ucd94\ub824\ub294 \uc2dc\ub3c4\ub85c \ubcf4\uc558\ub2e4.", "paragraph_answer": "TSPM\uc740 \ubb38\ud654\ud601\uba85\uc774 \uc2dc\uc791\ub418\ub358 \ud574\uc778 1966\ub144\uc5d0 \ud574\uccb4\ub418\uc5c8\ub2e4. TSPM\uc774 \uc7ac\uc870\uc9c1\ub418\ub358 \ud574\uc778 1979\ub144 \ucd08\uc5d0\ub294 \uadf8 \uc601\ud5a5\ub825\uc774 \ubca0\uc774\uc9d5\uc774\ub098 \uc0c1\ud558\uc774 \ub4f1\uc758 \ub300\ub3c4\uc2dc \uc9c0\uc5ed\uc73c\ub85c \uc81c\ud55c\ub418\uc5b4 \uc788\uc5c8\ub358 \ubc18\uba74\uc5d0 \uae30\ub3c5\uad50 \uc2e0\uc559\uc740 \ud5c8\ub09c, \uc800\uc7a5, \ud478\uc820, \uc548\ud6c4\uc774\uc640 \uac19\uc740 \uc9c0\uc5ed\uc5d0\uc11c \uae09\uc18d\ud788 \ud655\uc0b0\ub418\uc5c8\ub2e4. \uc911\uad6d \ub0b4\uc758 \ub9ce\uc740 \uadf8\ub9ac\uc2a4\ub3c4\uc778\ub4e4\uc774 TSPM\uc744 \uc758\uc2ec\ud588\ub294\ub370, \uadf8 \uc774\uc720\ub294 TSPM\uc774 \ud615\uc131\ub420 \ucd08\uae30\uc778 1950\ub144\ub300\uc5d0 \ud604\ub300\uc8fc\uc758 \uc2e0\ud559\uc744 \uac15\uc870\ud588\uc744 \ubfd0 \uc544\ub2c8\ub77c \uc911\uad6d \uc815\ubd80\uac00 \ub9ce\uc740 \uc131\uacbd \uad50\uc0ac\ub4e4\uc744 \uc5b5\uc555\ud558\uace0 \ubc15\ud574\ud560 \ub54c \uadf8\ub4e4\uc774 \ud611\ub825\ud588\uae30 \ub54c\ubb38\uc774\ub2e4. \uc774\ub7ec\ud55c \uadf8\ub9ac\uc2a4\ub3c4\uc778\ub4e4\uc740 1979\ub144\uc5d0 TSPM\uc774 \uc7ac\uc124\ub9bd\ub41c \uac83\uc740 \uc815\ubd80\uac00 \uc790\uc2e0\ub4e4\uc5d0 \ub300\ud55c \ud1b5\uc81c\ub97c \uac15\ud654\ud558\uace0 \uae30\ub3c5\uad50 \ubcf5\uc74c\uc774 \ud655\uc0b0\ub418\ub294 \uac83\uc744 \ub2a6\ucd94\ub824\ub294 \uc2dc\ub3c4\ub85c \ubcf4\uc558\ub2e4.", "sentence_answer": "TSPM\uc774 \uc7ac\uc870\uc9c1\ub418\ub358 \ud574\uc778 1979\ub144 \ucd08\uc5d0\ub294 \uadf8 \uc601\ud5a5\ub825\uc774 \ubca0\uc774\uc9d5\uc774\ub098 \uc0c1\ud558\uc774 \ub4f1\uc758 \ub300\ub3c4\uc2dc \uc9c0\uc5ed\uc73c\ub85c \uc81c\ud55c\ub418\uc5b4 \uc788\uc5c8\ub358 \ubc18\uba74\uc5d0 \uae30\ub3c5\uad50 \uc2e0\uc559\uc740 \ud5c8\ub09c, \uc800\uc7a5, \ud478\uc820, \uc548\ud6c4\uc774\uc640 \uac19\uc740 \uc9c0\uc5ed\uc5d0\uc11c \uae09\uc18d\ud788 \ud655\uc0b0\ub418\uc5c8\ub2e4.", "paragraph_id": "6571934-4-0", "paragraph_question": "question: TSPM\uc774 \uc7ac\uc870\uc9c1\ub41c \ud574\ub294?, context: TSPM\uc740 \ubb38\ud654\ud601\uba85\uc774 \uc2dc\uc791\ub418\ub358 \ud574\uc778 1966\ub144\uc5d0 \ud574\uccb4\ub418\uc5c8\ub2e4. TSPM\uc774 \uc7ac\uc870\uc9c1\ub418\ub358 \ud574\uc778 1979\ub144 \ucd08\uc5d0\ub294 \uadf8 \uc601\ud5a5\ub825\uc774 \ubca0\uc774\uc9d5\uc774\ub098 \uc0c1\ud558\uc774 \ub4f1\uc758 \ub300\ub3c4\uc2dc \uc9c0\uc5ed\uc73c\ub85c \uc81c\ud55c\ub418\uc5b4 \uc788\uc5c8\ub358 \ubc18\uba74\uc5d0 \uae30\ub3c5\uad50 \uc2e0\uc559\uc740 \ud5c8\ub09c, \uc800\uc7a5, \ud478\uc820, \uc548\ud6c4\uc774\uc640 \uac19\uc740 \uc9c0\uc5ed\uc5d0\uc11c \uae09\uc18d\ud788 \ud655\uc0b0\ub418\uc5c8\ub2e4. \uc911\uad6d \ub0b4\uc758 \ub9ce\uc740 \uadf8\ub9ac\uc2a4\ub3c4\uc778\ub4e4\uc774 TSPM\uc744 \uc758\uc2ec\ud588\ub294\ub370, \uadf8 \uc774\uc720\ub294 TSPM\uc774 \ud615\uc131\ub420 \ucd08\uae30\uc778 1950\ub144\ub300\uc5d0 \ud604\ub300\uc8fc\uc758 \uc2e0\ud559\uc744 \uac15\uc870\ud588\uc744 \ubfd0 \uc544\ub2c8\ub77c \uc911\uad6d \uc815\ubd80\uac00 \ub9ce\uc740 \uc131\uacbd \uad50\uc0ac\ub4e4\uc744 \uc5b5\uc555\ud558\uace0 \ubc15\ud574\ud560 \ub54c \uadf8\ub4e4\uc774 \ud611\ub825\ud588\uae30 \ub54c\ubb38\uc774\ub2e4. \uc774\ub7ec\ud55c \uadf8\ub9ac\uc2a4\ub3c4\uc778\ub4e4\uc740 1979\ub144\uc5d0 TSPM\uc774 \uc7ac\uc124\ub9bd\ub41c \uac83\uc740 \uc815\ubd80\uac00 \uc790\uc2e0\ub4e4\uc5d0 \ub300\ud55c \ud1b5\uc81c\ub97c \uac15\ud654\ud558\uace0 \uae30\ub3c5\uad50 \ubcf5\uc74c\uc774 \ud655\uc0b0\ub418\ub294 \uac83\uc744 \ub2a6\ucd94\ub824\ub294 \uc2dc\ub3c4\ub85c \ubcf4\uc558\ub2e4."} {"question": "\ud615\uc81c\ubcf5\uc9c0\uc6d0 \uc0ac\uac74\uc758 \ub2f4\ub2f9 \uac80\uc0ac\ub294 \ub204\uad6c\uc600\ub294\uac00?", "paragraph": "\uc774 \uc0ac\uac74 \ub2f4\ub2f9 \uac80\uc0ac\ub294 \uae40\uc6a9\uc6d0\uc774\uc5c8\uace0, \uc774 \uc0ac\uac74\uc73c\ub85c \uc778\ud574 <\ubd80\uc0b0 \ud615\uc81c\ubcf5\uc9c0\uc6d0> \uc6d0\uc7a5 \ubc15\uc778\uadfc\uacfc \uc9c1\uc6d0 \uc8fc\uc601\uc740(\ub2f9\uc2dc 48\uc138) \ub4f1 5\uba85\uc774 \uad6c\uc18d\ub418\uc5c8\ub2e4. \uc774 \uc0ac\uac74\uc740 \ub2f9\uc2dc \ubbfc\uc8fc\ud654 \uc9c4\ud589 \uacfc\uc815\uc5d0\uc11c \ub099\ud6c4\ub41c \ub300\ud55c\ubbfc\uad6d \ub0b4 \ubcf5\uc9c0\uc2dc\uc124\uc758 \uc218\uc900\uacfc \ucc38\uc0c1\uc744 \ubcf4\uc5ec\uc92c\ub2e4. \uc774\ud6c4 <\ubd80\uc0b0 \ud615\uc81c\ubcf5\uc9c0\uc6d0>\uc740 \uc5c6\uc5b4\uc9c0\uace0 \uadf8 \uc9c0\uc5ed\uc5d0 \uc544\ud30c\ud2b8 \ub2e8\uc9c0\uac00 \uc0dd\uacbc\ub2e4. \uc0ac\uac74\uc744 \uc218\uc0ac\ud558\ub358 \uae40\uc6a9\uc6d0 \uac80\uc0ac\ub294 \ub2f9\uc2dc \ubd80\uc0b0\uc9c0\ubc29\uac80\ucc30\uccad \uac80\uc0ac\uc7a5 \uc774\uc5c8\ub358 \ubc15\ud76c\ud0dc\uc5d0\uac8c \uc0ac\uac74 \ucd95\uc18c \ubc0f \uc678\uc555 \ub4f1\uc758 \uc9c0\uc2dc\ub97c \ubc1b\uc558\ub2e4. \uc6d0\uc7a5 \ubc15\uc778\uadfc\uc740 \ud6a1\ub839\uc8c4 \ub4f1\uc73c\ub85c \uac00\ubcbc\uc6b4 \ucc98\ubc8c\uc744 \ubc1b\uc558\uc744 \ubfd0 \ubd88\ubc95\uad6c\uae08, \ud3ed\ud589, \uc0b4\uc778 \ub4f1\uc5d0 \ub300\ud574\uc11c\ub294 \uc7ac\ud310\uc870\ucc28 \ubc1b\uc9c0 \uc54a\uc558\ub2e4. \uc6d0\uc7a5\uc774 \ud6a1\ub839\ud55c \uad6d\uace0\ubcf4\uc870\uae08\uc774 12\uc5b5 \uc6d0\uc5d0 \uc774\ub974\uc9c0\ub9cc \uac80\ucc30\uc740 7\uc5b5\uc5d0 \ub300\ud574\uc11c\ub9cc \uae30\uc18c\ud558\uc600\ub2e4. \uadf8\ub294 \ud56d\uc18c\uc2ec\uc744 \uac70\uccd0 \uc9d5\uc5ed 2\ub144 6\uac1c\uc6d4\ud615\uc744 \uc120\uace0\ubc1b\uc558\ub2e4.", "answer": "\uae40\uc6a9\uc6d0", "sentence": "\uc774 \uc0ac\uac74 \ub2f4\ub2f9 \uac80\uc0ac\ub294 \uae40\uc6a9\uc6d0 \uc774\uc5c8\uace0, \uc774 \uc0ac\uac74\uc73c\ub85c \uc778\ud574 <\ubd80\uc0b0 \ud615\uc81c\ubcf5\uc9c0\uc6d0> \uc6d0\uc7a5 \ubc15\uc778\uadfc\uacfc \uc9c1\uc6d0 \uc8fc\uc601\uc740(\ub2f9\uc2dc 48\uc138) \ub4f1 5\uba85\uc774 \uad6c\uc18d\ub418\uc5c8\ub2e4.", "paragraph_sentence": " \uc774 \uc0ac\uac74 \ub2f4\ub2f9 \uac80\uc0ac\ub294 \uae40\uc6a9\uc6d0 \uc774\uc5c8\uace0, \uc774 \uc0ac\uac74\uc73c\ub85c \uc778\ud574 <\ubd80\uc0b0 \ud615\uc81c\ubcf5\uc9c0\uc6d0> \uc6d0\uc7a5 \ubc15\uc778\uadfc\uacfc \uc9c1\uc6d0 \uc8fc\uc601\uc740(\ub2f9\uc2dc 48\uc138) \ub4f1 5\uba85\uc774 \uad6c\uc18d\ub418\uc5c8\ub2e4. \uc774 \uc0ac\uac74\uc740 \ub2f9\uc2dc \ubbfc\uc8fc\ud654 \uc9c4\ud589 \uacfc\uc815\uc5d0\uc11c \ub099\ud6c4\ub41c \ub300\ud55c\ubbfc\uad6d \ub0b4 \ubcf5\uc9c0\uc2dc\uc124\uc758 \uc218\uc900\uacfc \ucc38\uc0c1\uc744 \ubcf4\uc5ec\uc92c\ub2e4. \uc774\ud6c4 <\ubd80\uc0b0 \ud615\uc81c\ubcf5\uc9c0\uc6d0>\uc740 \uc5c6\uc5b4\uc9c0\uace0 \uadf8 \uc9c0\uc5ed\uc5d0 \uc544\ud30c\ud2b8 \ub2e8\uc9c0\uac00 \uc0dd\uacbc\ub2e4. \uc0ac\uac74\uc744 \uc218\uc0ac\ud558\ub358 \uae40\uc6a9\uc6d0 \uac80\uc0ac\ub294 \ub2f9\uc2dc \ubd80\uc0b0\uc9c0\ubc29\uac80\ucc30\uccad \uac80\uc0ac\uc7a5 \uc774\uc5c8\ub358 \ubc15\ud76c\ud0dc\uc5d0\uac8c \uc0ac\uac74 \ucd95\uc18c \ubc0f \uc678\uc555 \ub4f1\uc758 \uc9c0\uc2dc\ub97c \ubc1b\uc558\ub2e4. \uc6d0\uc7a5 \ubc15\uc778\uadfc\uc740 \ud6a1\ub839\uc8c4 \ub4f1\uc73c\ub85c \uac00\ubcbc\uc6b4 \ucc98\ubc8c\uc744 \ubc1b\uc558\uc744 \ubfd0 \ubd88\ubc95\uad6c\uae08, \ud3ed\ud589, \uc0b4\uc778 \ub4f1\uc5d0 \ub300\ud574\uc11c\ub294 \uc7ac\ud310\uc870\ucc28 \ubc1b\uc9c0 \uc54a\uc558\ub2e4. \uc6d0\uc7a5\uc774 \ud6a1\ub839\ud55c \uad6d\uace0\ubcf4\uc870\uae08\uc774 12\uc5b5 \uc6d0\uc5d0 \uc774\ub974\uc9c0\ub9cc \uac80\ucc30\uc740 7\uc5b5\uc5d0 \ub300\ud574\uc11c\ub9cc \uae30\uc18c\ud558\uc600\ub2e4. \uadf8\ub294 \ud56d\uc18c\uc2ec\uc744 \uac70\uccd0 \uc9d5\uc5ed 2\ub144 6\uac1c\uc6d4\ud615\uc744 \uc120\uace0\ubc1b\uc558\ub2e4.", "paragraph_answer": "\uc774 \uc0ac\uac74 \ub2f4\ub2f9 \uac80\uc0ac\ub294 \uae40\uc6a9\uc6d0 \uc774\uc5c8\uace0, \uc774 \uc0ac\uac74\uc73c\ub85c \uc778\ud574 <\ubd80\uc0b0 \ud615\uc81c\ubcf5\uc9c0\uc6d0> \uc6d0\uc7a5 \ubc15\uc778\uadfc\uacfc \uc9c1\uc6d0 \uc8fc\uc601\uc740(\ub2f9\uc2dc 48\uc138) \ub4f1 5\uba85\uc774 \uad6c\uc18d\ub418\uc5c8\ub2e4. \uc774 \uc0ac\uac74\uc740 \ub2f9\uc2dc \ubbfc\uc8fc\ud654 \uc9c4\ud589 \uacfc\uc815\uc5d0\uc11c \ub099\ud6c4\ub41c \ub300\ud55c\ubbfc\uad6d \ub0b4 \ubcf5\uc9c0\uc2dc\uc124\uc758 \uc218\uc900\uacfc \ucc38\uc0c1\uc744 \ubcf4\uc5ec\uc92c\ub2e4. \uc774\ud6c4 <\ubd80\uc0b0 \ud615\uc81c\ubcf5\uc9c0\uc6d0>\uc740 \uc5c6\uc5b4\uc9c0\uace0 \uadf8 \uc9c0\uc5ed\uc5d0 \uc544\ud30c\ud2b8 \ub2e8\uc9c0\uac00 \uc0dd\uacbc\ub2e4. \uc0ac\uac74\uc744 \uc218\uc0ac\ud558\ub358 \uae40\uc6a9\uc6d0 \uac80\uc0ac\ub294 \ub2f9\uc2dc \ubd80\uc0b0\uc9c0\ubc29\uac80\ucc30\uccad \uac80\uc0ac\uc7a5 \uc774\uc5c8\ub358 \ubc15\ud76c\ud0dc\uc5d0\uac8c \uc0ac\uac74 \ucd95\uc18c \ubc0f \uc678\uc555 \ub4f1\uc758 \uc9c0\uc2dc\ub97c \ubc1b\uc558\ub2e4. \uc6d0\uc7a5 \ubc15\uc778\uadfc\uc740 \ud6a1\ub839\uc8c4 \ub4f1\uc73c\ub85c \uac00\ubcbc\uc6b4 \ucc98\ubc8c\uc744 \ubc1b\uc558\uc744 \ubfd0 \ubd88\ubc95\uad6c\uae08, \ud3ed\ud589, \uc0b4\uc778 \ub4f1\uc5d0 \ub300\ud574\uc11c\ub294 \uc7ac\ud310\uc870\ucc28 \ubc1b\uc9c0 \uc54a\uc558\ub2e4. \uc6d0\uc7a5\uc774 \ud6a1\ub839\ud55c \uad6d\uace0\ubcf4\uc870\uae08\uc774 12\uc5b5 \uc6d0\uc5d0 \uc774\ub974\uc9c0\ub9cc \uac80\ucc30\uc740 7\uc5b5\uc5d0 \ub300\ud574\uc11c\ub9cc \uae30\uc18c\ud558\uc600\ub2e4. \uadf8\ub294 \ud56d\uc18c\uc2ec\uc744 \uac70\uccd0 \uc9d5\uc5ed 2\ub144 6\uac1c\uc6d4\ud615\uc744 \uc120\uace0\ubc1b\uc558\ub2e4.", "sentence_answer": "\uc774 \uc0ac\uac74 \ub2f4\ub2f9 \uac80\uc0ac\ub294 \uae40\uc6a9\uc6d0 \uc774\uc5c8\uace0, \uc774 \uc0ac\uac74\uc73c\ub85c \uc778\ud574 <\ubd80\uc0b0 \ud615\uc81c\ubcf5\uc9c0\uc6d0> \uc6d0\uc7a5 \ubc15\uc778\uadfc\uacfc \uc9c1\uc6d0 \uc8fc\uc601\uc740(\ub2f9\uc2dc 48\uc138) \ub4f1 5\uba85\uc774 \uad6c\uc18d\ub418\uc5c8\ub2e4.", "paragraph_id": "6571760-2-0", "paragraph_question": "question: \ud615\uc81c\ubcf5\uc9c0\uc6d0 \uc0ac\uac74\uc758 \ub2f4\ub2f9 \uac80\uc0ac\ub294 \ub204\uad6c\uc600\ub294\uac00?, context: \uc774 \uc0ac\uac74 \ub2f4\ub2f9 \uac80\uc0ac\ub294 \uae40\uc6a9\uc6d0\uc774\uc5c8\uace0, \uc774 \uc0ac\uac74\uc73c\ub85c \uc778\ud574 <\ubd80\uc0b0 \ud615\uc81c\ubcf5\uc9c0\uc6d0> \uc6d0\uc7a5 \ubc15\uc778\uadfc\uacfc \uc9c1\uc6d0 \uc8fc\uc601\uc740(\ub2f9\uc2dc 48\uc138) \ub4f1 5\uba85\uc774 \uad6c\uc18d\ub418\uc5c8\ub2e4. \uc774 \uc0ac\uac74\uc740 \ub2f9\uc2dc \ubbfc\uc8fc\ud654 \uc9c4\ud589 \uacfc\uc815\uc5d0\uc11c \ub099\ud6c4\ub41c \ub300\ud55c\ubbfc\uad6d \ub0b4 \ubcf5\uc9c0\uc2dc\uc124\uc758 \uc218\uc900\uacfc \ucc38\uc0c1\uc744 \ubcf4\uc5ec\uc92c\ub2e4. \uc774\ud6c4 <\ubd80\uc0b0 \ud615\uc81c\ubcf5\uc9c0\uc6d0>\uc740 \uc5c6\uc5b4\uc9c0\uace0 \uadf8 \uc9c0\uc5ed\uc5d0 \uc544\ud30c\ud2b8 \ub2e8\uc9c0\uac00 \uc0dd\uacbc\ub2e4. \uc0ac\uac74\uc744 \uc218\uc0ac\ud558\ub358 \uae40\uc6a9\uc6d0 \uac80\uc0ac\ub294 \ub2f9\uc2dc \ubd80\uc0b0\uc9c0\ubc29\uac80\ucc30\uccad \uac80\uc0ac\uc7a5 \uc774\uc5c8\ub358 \ubc15\ud76c\ud0dc\uc5d0\uac8c \uc0ac\uac74 \ucd95\uc18c \ubc0f \uc678\uc555 \ub4f1\uc758 \uc9c0\uc2dc\ub97c \ubc1b\uc558\ub2e4. \uc6d0\uc7a5 \ubc15\uc778\uadfc\uc740 \ud6a1\ub839\uc8c4 \ub4f1\uc73c\ub85c \uac00\ubcbc\uc6b4 \ucc98\ubc8c\uc744 \ubc1b\uc558\uc744 \ubfd0 \ubd88\ubc95\uad6c\uae08, \ud3ed\ud589, \uc0b4\uc778 \ub4f1\uc5d0 \ub300\ud574\uc11c\ub294 \uc7ac\ud310\uc870\ucc28 \ubc1b\uc9c0 \uc54a\uc558\ub2e4. \uc6d0\uc7a5\uc774 \ud6a1\ub839\ud55c \uad6d\uace0\ubcf4\uc870\uae08\uc774 12\uc5b5 \uc6d0\uc5d0 \uc774\ub974\uc9c0\ub9cc \uac80\ucc30\uc740 7\uc5b5\uc5d0 \ub300\ud574\uc11c\ub9cc \uae30\uc18c\ud558\uc600\ub2e4. \uadf8\ub294 \ud56d\uc18c\uc2ec\uc744 \uac70\uccd0 \uc9d5\uc5ed 2\ub144 6\uac1c\uc6d4\ud615\uc744 \uc120\uace0\ubc1b\uc558\ub2e4."} {"question": "\uc544\uc6b0\uad6c\uc2a4\ud2f0\ub178\ud68c\uc758 \uc218\ub3c4\uc0ac \ub9c8\ub974\ud2f4 \ub8e8\ud130\uac00 \uc885\uad50 \uac1c\ud601\uc744 \ub2e8\ud589\ud55c \ud574\ub294?", "paragraph": "130\ub144\uac04 \uc9c0\uc18d\ub418\ub358 \ub974\ub124\uc0c1\uc2a4\ub294 1530\ub144\uacbd \ub05d\uc774 \ub0ac\ub2e4. \uadf8 \uc774\uc720\ub294 \uc6b0\uc120 1492\ub144 \ud06c\ub9ac\uc2a4\ud1a0\ubc1c \ucf54\ub860\uc774 \ud3ec\ub974\ud22c\uac08\uc778\ub4e4\uacfc \ud568\uaed8 \uc778\ub3c4\ub85c \uac00\ub294 \ud574\ub85c\ub97c \ubc1c\uacac\ud588\uae30 \ub54c\ubb38\uc774\ub2e4.(\uc0ac\uc2e4\uc740 \uc544\uba54\ub9ac\uce74\ub85c \uac00\ub294 \ud574\ub85c\uc600\ub2e4.) \uadf8 \ud6c4\uc5d0\ub294 \ubd81\uc11c\ubd80 \uc720\ub7fd\uc758 \uc0c1\uc778\ub4e4\uc774 \ubb34\uc5ed\uc0c1\ud488\uc744 \ub9ac\uc2a4\ubcf8\uacfc \uc548\ud2b8\uc6e8\ub974\ud39c\uc744 \ud1b5\ud574 \uac70\ub798\ud558\ub294 \uac83\uc744 \uc120\ud638\ud558\uac8c \ub418\uc5c8\ub2e4. \ub610\ud55c 1517\ub144 \uc544\uc6b0\uad6c\uc2a4\ud2f0\ub178\ud68c\uc758 \uc218\ub3c4\uc0ac\uc778 \ub9c8\ub974\ud2f4 \ub8e8\ud130\uac00 \uc885\uad50 \uac1c\ud601\uc744 \ub2e8\ud589\ud588\uae30 \ub54c\ubb38\uc774\ub2e4. \uc774\ub85c \uc778\ud574 \uc11c\uc720\ub7fd \uad50\ud68c\ub294 \ub85c\ub9c8 \uac00\ud1a8\ub9ad\uad50\ud68c\uc640 \uac1c\uc2e0\uad50 \uad50\ud68c\ub85c \ubd84\uc5f4\ub418\uc5c8\ub2e4. \uad50\ud68c\uc758 \ubd84\uc5f4\uc740 \uadf8\ub3d9\uc548 \ub0a9\ubd80\uae08\uacfc \uc138\uae08\uc758 \ud615\ud0dc\ub85c \uc774\ud0c8\ub9ac\uc544\ub97c \ud48d\uc694\ub86d\uac8c \ud588\ub358 \ub3c8\uc904\uc758 \uace0\uac08\uc744 \uc758\ubbf8\ud588\ub2e4. \uc720\ub7fd\uc778\uc758 \uc544\uba54\ub9ac\uce74 \ub300\ub959 \uc0c1\ub959\uacfc \uc885\uad50\uac1c\ud601\uc73c\ub85c \uacb0\uad6d \uc774\ud0c8\ub9ac\uc544\ub294 \uc0c1\uc5c5\uc801 \ubb34\uc5ed\uc790\ubcf8\uacfc \uc790\ubcf8, \ub450\uac00\uc9c0\ub97c \ub3d9\uc2dc\uc5d0 \uc783\uc5c8\ub2e4 \ub610 \uc774\ud0c8\ub9ac\uc544\ub294 \uadf8 \ub2f9\uc2dc \uc815\uce58\uc801\uc73c\ub85c \uad49\uc7a5\ud788 \ud63c\ub780\uc2a4\ub7ec\uc6b4 \uc0c1\ud0dc\uc774\uae30\ub3c4 \ud558\uc600\ub2e4. \uc774\ub85c \uc778\ud574 \ub974\ub124\uc0c1\uc2a4\ub294 \uc1e0\ud1f4\ud560 \uc218\ubc16\uc5d0 \uc5c6\uc5c8\ub2e4.", "answer": "1517\ub144", "sentence": "\ub610\ud55c 1517\ub144 \uc544\uc6b0\uad6c\uc2a4\ud2f0\ub178\ud68c\uc758 \uc218\ub3c4\uc0ac\uc778 \ub9c8\ub974\ud2f4 \ub8e8\ud130\uac00 \uc885\uad50 \uac1c\ud601\uc744 \ub2e8\ud589\ud588\uae30 \ub54c\ubb38\uc774\ub2e4.", "paragraph_sentence": "130\ub144\uac04 \uc9c0\uc18d\ub418\ub358 \ub974\ub124\uc0c1\uc2a4\ub294 1530\ub144\uacbd \ub05d\uc774 \ub0ac\ub2e4. \uadf8 \uc774\uc720\ub294 \uc6b0\uc120 1492\ub144 \ud06c\ub9ac\uc2a4\ud1a0\ubc1c \ucf54\ub860\uc774 \ud3ec\ub974\ud22c\uac08\uc778\ub4e4\uacfc \ud568\uaed8 \uc778\ub3c4\ub85c \uac00\ub294 \ud574\ub85c\ub97c \ubc1c\uacac\ud588\uae30 \ub54c\ubb38\uc774\ub2e4. (\uc0ac\uc2e4\uc740 \uc544\uba54\ub9ac\uce74\ub85c \uac00\ub294 \ud574\ub85c\uc600\ub2e4.) \uadf8 \ud6c4\uc5d0\ub294 \ubd81\uc11c\ubd80 \uc720\ub7fd\uc758 \uc0c1\uc778\ub4e4\uc774 \ubb34\uc5ed\uc0c1\ud488\uc744 \ub9ac\uc2a4\ubcf8\uacfc \uc548\ud2b8\uc6e8\ub974\ud39c\uc744 \ud1b5\ud574 \uac70\ub798\ud558\ub294 \uac83\uc744 \uc120\ud638\ud558\uac8c \ub418\uc5c8\ub2e4. \ub610\ud55c 1517\ub144 \uc544\uc6b0\uad6c\uc2a4\ud2f0\ub178\ud68c\uc758 \uc218\ub3c4\uc0ac\uc778 \ub9c8\ub974\ud2f4 \ub8e8\ud130\uac00 \uc885\uad50 \uac1c\ud601\uc744 \ub2e8\ud589\ud588\uae30 \ub54c\ubb38\uc774\ub2e4. \uc774\ub85c \uc778\ud574 \uc11c\uc720\ub7fd \uad50\ud68c\ub294 \ub85c\ub9c8 \uac00\ud1a8\ub9ad\uad50\ud68c\uc640 \uac1c\uc2e0\uad50 \uad50\ud68c\ub85c \ubd84\uc5f4\ub418\uc5c8\ub2e4. \uad50\ud68c\uc758 \ubd84\uc5f4\uc740 \uadf8\ub3d9\uc548 \ub0a9\ubd80\uae08\uacfc \uc138\uae08\uc758 \ud615\ud0dc\ub85c \uc774\ud0c8\ub9ac\uc544\ub97c \ud48d\uc694\ub86d\uac8c \ud588\ub358 \ub3c8\uc904\uc758 \uace0\uac08\uc744 \uc758\ubbf8\ud588\ub2e4. \uc720\ub7fd\uc778\uc758 \uc544\uba54\ub9ac\uce74 \ub300\ub959 \uc0c1\ub959\uacfc \uc885\uad50\uac1c\ud601\uc73c\ub85c \uacb0\uad6d \uc774\ud0c8\ub9ac\uc544\ub294 \uc0c1\uc5c5\uc801 \ubb34\uc5ed\uc790\ubcf8\uacfc \uc790\ubcf8, \ub450\uac00\uc9c0\ub97c \ub3d9\uc2dc\uc5d0 \uc783\uc5c8\ub2e4 \ub610 \uc774\ud0c8\ub9ac\uc544\ub294 \uadf8 \ub2f9\uc2dc \uc815\uce58\uc801\uc73c\ub85c \uad49\uc7a5\ud788 \ud63c\ub780\uc2a4\ub7ec\uc6b4 \uc0c1\ud0dc\uc774\uae30\ub3c4 \ud558\uc600\ub2e4. \uc774\ub85c \uc778\ud574 \ub974\ub124\uc0c1\uc2a4\ub294 \uc1e0\ud1f4\ud560 \uc218\ubc16\uc5d0 \uc5c6\uc5c8\ub2e4.", "paragraph_answer": "130\ub144\uac04 \uc9c0\uc18d\ub418\ub358 \ub974\ub124\uc0c1\uc2a4\ub294 1530\ub144\uacbd \ub05d\uc774 \ub0ac\ub2e4. \uadf8 \uc774\uc720\ub294 \uc6b0\uc120 1492\ub144 \ud06c\ub9ac\uc2a4\ud1a0\ubc1c \ucf54\ub860\uc774 \ud3ec\ub974\ud22c\uac08\uc778\ub4e4\uacfc \ud568\uaed8 \uc778\ub3c4\ub85c \uac00\ub294 \ud574\ub85c\ub97c \ubc1c\uacac\ud588\uae30 \ub54c\ubb38\uc774\ub2e4.(\uc0ac\uc2e4\uc740 \uc544\uba54\ub9ac\uce74\ub85c \uac00\ub294 \ud574\ub85c\uc600\ub2e4.) \uadf8 \ud6c4\uc5d0\ub294 \ubd81\uc11c\ubd80 \uc720\ub7fd\uc758 \uc0c1\uc778\ub4e4\uc774 \ubb34\uc5ed\uc0c1\ud488\uc744 \ub9ac\uc2a4\ubcf8\uacfc \uc548\ud2b8\uc6e8\ub974\ud39c\uc744 \ud1b5\ud574 \uac70\ub798\ud558\ub294 \uac83\uc744 \uc120\ud638\ud558\uac8c \ub418\uc5c8\ub2e4. \ub610\ud55c 1517\ub144 \uc544\uc6b0\uad6c\uc2a4\ud2f0\ub178\ud68c\uc758 \uc218\ub3c4\uc0ac\uc778 \ub9c8\ub974\ud2f4 \ub8e8\ud130\uac00 \uc885\uad50 \uac1c\ud601\uc744 \ub2e8\ud589\ud588\uae30 \ub54c\ubb38\uc774\ub2e4. \uc774\ub85c \uc778\ud574 \uc11c\uc720\ub7fd \uad50\ud68c\ub294 \ub85c\ub9c8 \uac00\ud1a8\ub9ad\uad50\ud68c\uc640 \uac1c\uc2e0\uad50 \uad50\ud68c\ub85c \ubd84\uc5f4\ub418\uc5c8\ub2e4. \uad50\ud68c\uc758 \ubd84\uc5f4\uc740 \uadf8\ub3d9\uc548 \ub0a9\ubd80\uae08\uacfc \uc138\uae08\uc758 \ud615\ud0dc\ub85c \uc774\ud0c8\ub9ac\uc544\ub97c \ud48d\uc694\ub86d\uac8c \ud588\ub358 \ub3c8\uc904\uc758 \uace0\uac08\uc744 \uc758\ubbf8\ud588\ub2e4. \uc720\ub7fd\uc778\uc758 \uc544\uba54\ub9ac\uce74 \ub300\ub959 \uc0c1\ub959\uacfc \uc885\uad50\uac1c\ud601\uc73c\ub85c \uacb0\uad6d \uc774\ud0c8\ub9ac\uc544\ub294 \uc0c1\uc5c5\uc801 \ubb34\uc5ed\uc790\ubcf8\uacfc \uc790\ubcf8, \ub450\uac00\uc9c0\ub97c \ub3d9\uc2dc\uc5d0 \uc783\uc5c8\ub2e4 \ub610 \uc774\ud0c8\ub9ac\uc544\ub294 \uadf8 \ub2f9\uc2dc \uc815\uce58\uc801\uc73c\ub85c \uad49\uc7a5\ud788 \ud63c\ub780\uc2a4\ub7ec\uc6b4 \uc0c1\ud0dc\uc774\uae30\ub3c4 \ud558\uc600\ub2e4. \uc774\ub85c \uc778\ud574 \ub974\ub124\uc0c1\uc2a4\ub294 \uc1e0\ud1f4\ud560 \uc218\ubc16\uc5d0 \uc5c6\uc5c8\ub2e4.", "sentence_answer": "\ub610\ud55c 1517\ub144 \uc544\uc6b0\uad6c\uc2a4\ud2f0\ub178\ud68c\uc758 \uc218\ub3c4\uc0ac\uc778 \ub9c8\ub974\ud2f4 \ub8e8\ud130\uac00 \uc885\uad50 \uac1c\ud601\uc744 \ub2e8\ud589\ud588\uae30 \ub54c\ubb38\uc774\ub2e4.", "paragraph_id": "6656981-5-2", "paragraph_question": "question: \uc544\uc6b0\uad6c\uc2a4\ud2f0\ub178\ud68c\uc758 \uc218\ub3c4\uc0ac \ub9c8\ub974\ud2f4 \ub8e8\ud130\uac00 \uc885\uad50 \uac1c\ud601\uc744 \ub2e8\ud589\ud55c \ud574\ub294?, context: 130\ub144\uac04 \uc9c0\uc18d\ub418\ub358 \ub974\ub124\uc0c1\uc2a4\ub294 1530\ub144\uacbd \ub05d\uc774 \ub0ac\ub2e4. \uadf8 \uc774\uc720\ub294 \uc6b0\uc120 1492\ub144 \ud06c\ub9ac\uc2a4\ud1a0\ubc1c \ucf54\ub860\uc774 \ud3ec\ub974\ud22c\uac08\uc778\ub4e4\uacfc \ud568\uaed8 \uc778\ub3c4\ub85c \uac00\ub294 \ud574\ub85c\ub97c \ubc1c\uacac\ud588\uae30 \ub54c\ubb38\uc774\ub2e4.(\uc0ac\uc2e4\uc740 \uc544\uba54\ub9ac\uce74\ub85c \uac00\ub294 \ud574\ub85c\uc600\ub2e4.) \uadf8 \ud6c4\uc5d0\ub294 \ubd81\uc11c\ubd80 \uc720\ub7fd\uc758 \uc0c1\uc778\ub4e4\uc774 \ubb34\uc5ed\uc0c1\ud488\uc744 \ub9ac\uc2a4\ubcf8\uacfc \uc548\ud2b8\uc6e8\ub974\ud39c\uc744 \ud1b5\ud574 \uac70\ub798\ud558\ub294 \uac83\uc744 \uc120\ud638\ud558\uac8c \ub418\uc5c8\ub2e4. \ub610\ud55c 1517\ub144 \uc544\uc6b0\uad6c\uc2a4\ud2f0\ub178\ud68c\uc758 \uc218\ub3c4\uc0ac\uc778 \ub9c8\ub974\ud2f4 \ub8e8\ud130\uac00 \uc885\uad50 \uac1c\ud601\uc744 \ub2e8\ud589\ud588\uae30 \ub54c\ubb38\uc774\ub2e4. \uc774\ub85c \uc778\ud574 \uc11c\uc720\ub7fd \uad50\ud68c\ub294 \ub85c\ub9c8 \uac00\ud1a8\ub9ad\uad50\ud68c\uc640 \uac1c\uc2e0\uad50 \uad50\ud68c\ub85c \ubd84\uc5f4\ub418\uc5c8\ub2e4. \uad50\ud68c\uc758 \ubd84\uc5f4\uc740 \uadf8\ub3d9\uc548 \ub0a9\ubd80\uae08\uacfc \uc138\uae08\uc758 \ud615\ud0dc\ub85c \uc774\ud0c8\ub9ac\uc544\ub97c \ud48d\uc694\ub86d\uac8c \ud588\ub358 \ub3c8\uc904\uc758 \uace0\uac08\uc744 \uc758\ubbf8\ud588\ub2e4. \uc720\ub7fd\uc778\uc758 \uc544\uba54\ub9ac\uce74 \ub300\ub959 \uc0c1\ub959\uacfc \uc885\uad50\uac1c\ud601\uc73c\ub85c \uacb0\uad6d \uc774\ud0c8\ub9ac\uc544\ub294 \uc0c1\uc5c5\uc801 \ubb34\uc5ed\uc790\ubcf8\uacfc \uc790\ubcf8, \ub450\uac00\uc9c0\ub97c \ub3d9\uc2dc\uc5d0 \uc783\uc5c8\ub2e4 \ub610 \uc774\ud0c8\ub9ac\uc544\ub294 \uadf8 \ub2f9\uc2dc \uc815\uce58\uc801\uc73c\ub85c \uad49\uc7a5\ud788 \ud63c\ub780\uc2a4\ub7ec\uc6b4 \uc0c1\ud0dc\uc774\uae30\ub3c4 \ud558\uc600\ub2e4. \uc774\ub85c \uc778\ud574 \ub974\ub124\uc0c1\uc2a4\ub294 \uc1e0\ud1f4\ud560 \uc218\ubc16\uc5d0 \uc5c6\uc5c8\ub2e4."} {"question": "\ub2c8\ubca8\ub8fd\uc758 \ubc18\uc9c0\uc5d0\uc11c \ub9ac\ud558\ub974\ud2b8 \ubc14\uadf8\ub108\ub294 \ubb34\uc5c7\uc744 \ubb18\uc0ac\ud588\ub294\uac00?", "paragraph": "\ubc1c\ud640\uc5d0 \uad00\ud55c \uc5b8\uae09\uc740 \ubb38\ud559, \uc608\uc220, \uadf8 \uc678 \ub9e4\uccb4\uc5d0\uc11c \uc218 \ucc28\ub840 \uc774\ub8e8\uc5b4\uc9c4\ub2e4. K. Ehrenberg\uc758 1880\ub144 \ubaa9\ud0c4\ud654 \uc791\ud488 Gastmahl in Walhalla (mit einziehenden Einheriern)\uac00 \uc788\uace0, \ub9ac\ud558\ub974\ud2b8 \ubc14\uadf8\ub108\ub294 \uc624\ud398\ub77c \ub300\uacc4 \u300a\ub2c8\ubca8\ub8fd\uc758 \ubc18\uc9c0\u300b\uc5d0\uc11c \ubc1c\ud560\ub77c\ub97c \ubb18\uc0ac\ud588\ub2e4. \ubb8c\ud5e8\uc5d0\uc11c 1905\ub144\ubd80\ud130 1913\ub144\uae4c\uc9c0 \ubc1c\ud589\ud55c \uac8c\ub974\ub9cc \uc2e0\uc774\uad50\uc8fc\uc758 \uc7a1\uc9c0 \uc81c\ubaa9\ub3c4 \u300a\ubc1c\ud560\ub77c\u300b(Walhalla)\uc600\ub2e4. \ub374\ub9c8\ud06c\uc5d0\uc11c\ub294 1978\ub144\ubd80\ud130 2009\ub144\uae4c\uc9c0 \ud398\ud14c\ub974 \ub9cc\ub4dc\uc13c\uc774 \ub9cc\ud654 \u300a\ubc1c\ud560\ub77c\u300b\ub97c \uc5f0\uc7ac\ud588\uace0, 1986\ub144\uc5d0\ub294 \uadf8\uac83\uc744 \uc6d0\uc791\uc73c\ub85c \ud55c \uc560\ub2c8\uba54\uc774\uc158 \uc601\ud654\ub3c4 \uac1c\ubd09\ud558\uc600\ub2e4. \ubbf8\uad6d \ubc29\uc1a1\uad6d \uc1fc\ud0c0\uc784\uc758 \ud3ec\uc2a4\ud2b8 \uc544\ud3ec\uce7c\ub9bd\uc2a4 \ub4dc\ub77c\ub9c8 \u300a\uc608\ub808\ubbf8\uc57c\u300b(2002\ub144 ~ 2004\ub144)\ub294 \uc81c\ubaa9\uacfc \ub3d9\uc77c\ud55c \uc774\ub984\uc758 \uc8fc\uc778\uacf5 \uc608\ub808\ubbf8\uc57c\uac00 \"\ubc1c\ud560\ub77c \uc139\ud130\"\ub97c \ucc3e\uc544\uac00\ub294 \uc5ec\uc815\uc744 \ub2e4\ub8e8\uace0 \uc788\ub2e4. \ub808\ub4dc \uc81c\ud50c\ub9b0\uc758 \ud788\ud2b8\uace1 \"Immigrant Song\"\uc740 \ubc14\uc774\ud0b9\uc758 \ud574\uc801\uc9c8\uc744 \ubb18\uc0ac\ud55c \ub178\ub798\ub85c, \u201c\ubc1c\ud560\ub77c\uc5ec \ub0b4\uac00 \uac04\ub2e4(Valhalla, I am coming)\u201d\ub294 \uad6c\uc808\uc774 \uc788\ub2e4. 1975\ub144\uc5d0 \uc81c\uc4f0\ub85c \ud234\uc774 \ubc1c\ub9e4\ud55c \uc74c\ubc18 Minstrel In The Gallery\uc5d0\ub3c4 \"Cold Wind To Valhalla\"\ub77c\ub294 \ub178\ub798\uac00 \uc218\ub85d\ub418\uc5b4 \uc788\uace0. \ube14\ub77c\uc778\ub4dc \uac00\ub514\uc5b8\uc774\ub098 \ubc2d\uc18c\ub9ac\ub3c4 \"Valhalla\"\ub77c\ub294 \uc81c\ubaa9\uc758 \ub178\ub798\ub4e4\uc744 \ub9cc\ub4e4\uc5c8\ub2e4.", "answer": "\ubc1c\ud560\ub77c", "sentence": "\uc5d0\uc11c \ubc1c\ud560\ub77c \ub97c \ubb18\uc0ac\ud588\ub2e4.", "paragraph_sentence": "\ubc1c\ud640\uc5d0 \uad00\ud55c \uc5b8\uae09\uc740 \ubb38\ud559, \uc608\uc220, \uadf8 \uc678 \ub9e4\uccb4\uc5d0\uc11c \uc218 \ucc28\ub840 \uc774\ub8e8\uc5b4\uc9c4\ub2e4. K. Ehrenberg\uc758 1880\ub144 \ubaa9\ud0c4\ud654 \uc791\ud488 Gastmahl in Walhalla (mit einziehenden Einheriern)\uac00 \uc788\uace0, \ub9ac\ud558\ub974\ud2b8 \ubc14\uadf8\ub108\ub294 \uc624\ud398\ub77c \ub300\uacc4 \u300a\ub2c8\ubca8\ub8fd\uc758 \ubc18\uc9c0\u300b \uc5d0\uc11c \ubc1c\ud560\ub77c \ub97c \ubb18\uc0ac\ud588\ub2e4. \ubb8c\ud5e8\uc5d0\uc11c 1905\ub144\ubd80\ud130 1913\ub144\uae4c\uc9c0 \ubc1c\ud589\ud55c \uac8c\ub974\ub9cc \uc2e0\uc774\uad50\uc8fc\uc758 \uc7a1\uc9c0 \uc81c\ubaa9\ub3c4 \u300a\ubc1c\ud560\ub77c\u300b(Walhalla)\uc600\ub2e4. \ub374\ub9c8\ud06c\uc5d0\uc11c\ub294 1978\ub144\ubd80\ud130 2009\ub144\uae4c\uc9c0 \ud398\ud14c\ub974 \ub9cc\ub4dc\uc13c\uc774 \ub9cc\ud654 \u300a\ubc1c\ud560\ub77c\u300b\ub97c \uc5f0\uc7ac\ud588\uace0, 1986\ub144\uc5d0\ub294 \uadf8\uac83\uc744 \uc6d0\uc791\uc73c\ub85c \ud55c \uc560\ub2c8\uba54\uc774\uc158 \uc601\ud654\ub3c4 \uac1c\ubd09\ud558\uc600\ub2e4. \ubbf8\uad6d \ubc29\uc1a1\uad6d \uc1fc\ud0c0\uc784\uc758 \ud3ec\uc2a4\ud2b8 \uc544\ud3ec\uce7c\ub9bd\uc2a4 \ub4dc\ub77c\ub9c8 \u300a\uc608\ub808\ubbf8\uc57c\u300b(2002\ub144 ~ 2004\ub144)\ub294 \uc81c\ubaa9\uacfc \ub3d9\uc77c\ud55c \uc774\ub984\uc758 \uc8fc\uc778\uacf5 \uc608\ub808\ubbf8\uc57c\uac00 \"\ubc1c\ud560\ub77c \uc139\ud130\"\ub97c \ucc3e\uc544\uac00\ub294 \uc5ec\uc815\uc744 \ub2e4\ub8e8\uace0 \uc788\ub2e4. \ub808\ub4dc \uc81c\ud50c\ub9b0\uc758 \ud788\ud2b8\uace1 \"Immigrant Song\"\uc740 \ubc14\uc774\ud0b9\uc758 \ud574\uc801\uc9c8\uc744 \ubb18\uc0ac\ud55c \ub178\ub798\ub85c, \u201c\ubc1c\ud560\ub77c\uc5ec \ub0b4\uac00 \uac04\ub2e4(Valhalla, I am coming)\u201d\ub294 \uad6c\uc808\uc774 \uc788\ub2e4. 1975\ub144\uc5d0 \uc81c\uc4f0\ub85c \ud234\uc774 \ubc1c\ub9e4\ud55c \uc74c\ubc18 Minstrel In The Gallery\uc5d0\ub3c4 \"Cold Wind To Valhalla\"\ub77c\ub294 \ub178\ub798\uac00 \uc218\ub85d\ub418\uc5b4 \uc788\uace0. \ube14\ub77c\uc778\ub4dc \uac00\ub514\uc5b8\uc774\ub098 \ubc2d\uc18c\ub9ac\ub3c4 \"Valhalla\"\ub77c\ub294 \uc81c\ubaa9\uc758 \ub178\ub798\ub4e4\uc744 \ub9cc\ub4e4\uc5c8\ub2e4.", "paragraph_answer": "\ubc1c\ud640\uc5d0 \uad00\ud55c \uc5b8\uae09\uc740 \ubb38\ud559, \uc608\uc220, \uadf8 \uc678 \ub9e4\uccb4\uc5d0\uc11c \uc218 \ucc28\ub840 \uc774\ub8e8\uc5b4\uc9c4\ub2e4. K. Ehrenberg\uc758 1880\ub144 \ubaa9\ud0c4\ud654 \uc791\ud488 Gastmahl in Walhalla (mit einziehenden Einheriern)\uac00 \uc788\uace0, \ub9ac\ud558\ub974\ud2b8 \ubc14\uadf8\ub108\ub294 \uc624\ud398\ub77c \ub300\uacc4 \u300a\ub2c8\ubca8\ub8fd\uc758 \ubc18\uc9c0\u300b\uc5d0\uc11c \ubc1c\ud560\ub77c \ub97c \ubb18\uc0ac\ud588\ub2e4. \ubb8c\ud5e8\uc5d0\uc11c 1905\ub144\ubd80\ud130 1913\ub144\uae4c\uc9c0 \ubc1c\ud589\ud55c \uac8c\ub974\ub9cc \uc2e0\uc774\uad50\uc8fc\uc758 \uc7a1\uc9c0 \uc81c\ubaa9\ub3c4 \u300a\ubc1c\ud560\ub77c\u300b(Walhalla)\uc600\ub2e4. \ub374\ub9c8\ud06c\uc5d0\uc11c\ub294 1978\ub144\ubd80\ud130 2009\ub144\uae4c\uc9c0 \ud398\ud14c\ub974 \ub9cc\ub4dc\uc13c\uc774 \ub9cc\ud654 \u300a\ubc1c\ud560\ub77c\u300b\ub97c \uc5f0\uc7ac\ud588\uace0, 1986\ub144\uc5d0\ub294 \uadf8\uac83\uc744 \uc6d0\uc791\uc73c\ub85c \ud55c \uc560\ub2c8\uba54\uc774\uc158 \uc601\ud654\ub3c4 \uac1c\ubd09\ud558\uc600\ub2e4. \ubbf8\uad6d \ubc29\uc1a1\uad6d \uc1fc\ud0c0\uc784\uc758 \ud3ec\uc2a4\ud2b8 \uc544\ud3ec\uce7c\ub9bd\uc2a4 \ub4dc\ub77c\ub9c8 \u300a\uc608\ub808\ubbf8\uc57c\u300b(2002\ub144 ~ 2004\ub144)\ub294 \uc81c\ubaa9\uacfc \ub3d9\uc77c\ud55c \uc774\ub984\uc758 \uc8fc\uc778\uacf5 \uc608\ub808\ubbf8\uc57c\uac00 \"\ubc1c\ud560\ub77c \uc139\ud130\"\ub97c \ucc3e\uc544\uac00\ub294 \uc5ec\uc815\uc744 \ub2e4\ub8e8\uace0 \uc788\ub2e4. \ub808\ub4dc \uc81c\ud50c\ub9b0\uc758 \ud788\ud2b8\uace1 \"Immigrant Song\"\uc740 \ubc14\uc774\ud0b9\uc758 \ud574\uc801\uc9c8\uc744 \ubb18\uc0ac\ud55c \ub178\ub798\ub85c, \u201c\ubc1c\ud560\ub77c\uc5ec \ub0b4\uac00 \uac04\ub2e4(Valhalla, I am coming)\u201d\ub294 \uad6c\uc808\uc774 \uc788\ub2e4. 1975\ub144\uc5d0 \uc81c\uc4f0\ub85c \ud234\uc774 \ubc1c\ub9e4\ud55c \uc74c\ubc18 Minstrel In The Gallery\uc5d0\ub3c4 \"Cold Wind To Valhalla\"\ub77c\ub294 \ub178\ub798\uac00 \uc218\ub85d\ub418\uc5b4 \uc788\uace0. \ube14\ub77c\uc778\ub4dc \uac00\ub514\uc5b8\uc774\ub098 \ubc2d\uc18c\ub9ac\ub3c4 \"Valhalla\"\ub77c\ub294 \uc81c\ubaa9\uc758 \ub178\ub798\ub4e4\uc744 \ub9cc\ub4e4\uc5c8\ub2e4.", "sentence_answer": "\uc5d0\uc11c \ubc1c\ud560\ub77c \ub97c \ubb18\uc0ac\ud588\ub2e4.", "paragraph_id": "6507845-8-0", "paragraph_question": "question: \ub2c8\ubca8\ub8fd\uc758 \ubc18\uc9c0\uc5d0\uc11c \ub9ac\ud558\ub974\ud2b8 \ubc14\uadf8\ub108\ub294 \ubb34\uc5c7\uc744 \ubb18\uc0ac\ud588\ub294\uac00?, context: \ubc1c\ud640\uc5d0 \uad00\ud55c \uc5b8\uae09\uc740 \ubb38\ud559, \uc608\uc220, \uadf8 \uc678 \ub9e4\uccb4\uc5d0\uc11c \uc218 \ucc28\ub840 \uc774\ub8e8\uc5b4\uc9c4\ub2e4. K. Ehrenberg\uc758 1880\ub144 \ubaa9\ud0c4\ud654 \uc791\ud488 Gastmahl in Walhalla (mit einziehenden Einheriern)\uac00 \uc788\uace0, \ub9ac\ud558\ub974\ud2b8 \ubc14\uadf8\ub108\ub294 \uc624\ud398\ub77c \ub300\uacc4 \u300a\ub2c8\ubca8\ub8fd\uc758 \ubc18\uc9c0\u300b\uc5d0\uc11c \ubc1c\ud560\ub77c\ub97c \ubb18\uc0ac\ud588\ub2e4. \ubb8c\ud5e8\uc5d0\uc11c 1905\ub144\ubd80\ud130 1913\ub144\uae4c\uc9c0 \ubc1c\ud589\ud55c \uac8c\ub974\ub9cc \uc2e0\uc774\uad50\uc8fc\uc758 \uc7a1\uc9c0 \uc81c\ubaa9\ub3c4 \u300a\ubc1c\ud560\ub77c\u300b(Walhalla)\uc600\ub2e4. \ub374\ub9c8\ud06c\uc5d0\uc11c\ub294 1978\ub144\ubd80\ud130 2009\ub144\uae4c\uc9c0 \ud398\ud14c\ub974 \ub9cc\ub4dc\uc13c\uc774 \ub9cc\ud654 \u300a\ubc1c\ud560\ub77c\u300b\ub97c \uc5f0\uc7ac\ud588\uace0, 1986\ub144\uc5d0\ub294 \uadf8\uac83\uc744 \uc6d0\uc791\uc73c\ub85c \ud55c \uc560\ub2c8\uba54\uc774\uc158 \uc601\ud654\ub3c4 \uac1c\ubd09\ud558\uc600\ub2e4. \ubbf8\uad6d \ubc29\uc1a1\uad6d \uc1fc\ud0c0\uc784\uc758 \ud3ec\uc2a4\ud2b8 \uc544\ud3ec\uce7c\ub9bd\uc2a4 \ub4dc\ub77c\ub9c8 \u300a\uc608\ub808\ubbf8\uc57c\u300b(2002\ub144 ~ 2004\ub144)\ub294 \uc81c\ubaa9\uacfc \ub3d9\uc77c\ud55c \uc774\ub984\uc758 \uc8fc\uc778\uacf5 \uc608\ub808\ubbf8\uc57c\uac00 \"\ubc1c\ud560\ub77c \uc139\ud130\"\ub97c \ucc3e\uc544\uac00\ub294 \uc5ec\uc815\uc744 \ub2e4\ub8e8\uace0 \uc788\ub2e4. \ub808\ub4dc \uc81c\ud50c\ub9b0\uc758 \ud788\ud2b8\uace1 \"Immigrant Song\"\uc740 \ubc14\uc774\ud0b9\uc758 \ud574\uc801\uc9c8\uc744 \ubb18\uc0ac\ud55c \ub178\ub798\ub85c, \u201c\ubc1c\ud560\ub77c\uc5ec \ub0b4\uac00 \uac04\ub2e4(Valhalla, I am coming)\u201d\ub294 \uad6c\uc808\uc774 \uc788\ub2e4. 1975\ub144\uc5d0 \uc81c\uc4f0\ub85c \ud234\uc774 \ubc1c\ub9e4\ud55c \uc74c\ubc18 Minstrel In The Gallery\uc5d0\ub3c4 \"Cold Wind To Valhalla\"\ub77c\ub294 \ub178\ub798\uac00 \uc218\ub85d\ub418\uc5b4 \uc788\uace0. \ube14\ub77c\uc778\ub4dc \uac00\ub514\uc5b8\uc774\ub098 \ubc2d\uc18c\ub9ac\ub3c4 \"Valhalla\"\ub77c\ub294 \uc81c\ubaa9\uc758 \ub178\ub798\ub4e4\uc744 \ub9cc\ub4e4\uc5c8\ub2e4."} {"question": "\ud55c\ud6a8\uc8fc\uac00 2009\ub144\uc5d0 \ucd9c\uc5f0\ud55c \ud55c\uc77c \ud569\uc791\uc758 \uc601\ud654\ub294?", "paragraph": "\ud55c\ud6a8\uc8fc\ub294 2009\ub144, \ub4dc\ub77c\ub9c8 \u300a\ucc2c\ub780\ud55c \uc720\uc0b0\u300b\uc5d0\uc11c \ud654\ub824\ud558\uac8c \uc544\ub984\ub2f5\uc9c4 \uc54a\uc9c0\ub9cc \ub4e4\uaf43 \uac19\uc740 \ud574\ub9d1\uc740 \ub9e4\ub825\uc744 \uac00\uc9c4 \uace0\uc740\uc131 \uc5ed\uc744 \ub9e1\uc544 1\ub144\uc5ec\ub9cc\uc5d0 \ube0c\ub77c\uc6b4\uad00\uc5d0 \ubcf5\uadc0\ud558\uc600\ub2e4. \uc774 \uc791\ud488\uc740 \ucd5c\uace0 \uc2dc\uccad\ub960 45.2%\ub97c \uae30\ub85d\ud558\uba70 \ud070 \uc778\uae30\ub97c \ub204\ub838\uace0, \ud55c\ud6a8\uc8fc\ub294 \uadf8\ub3d9\uc548 \ubd80\uc871\ud55c \uc810\uc73c\ub85c \uc9c0\uc801\ub410\ub358 \ub0b4\uba74\uc5f0\uae30\ub3c4 \ubcf4\uc644\ud558\uba70 \uc5f0\uae30\uc790\ub85c\uc11c \ucc2c\uc0ac\ub97c \ubc1b\uc558\ub2e4. \uc5f0\uae30\uc804\ubb38\ud559\uc6d0\uc778 \ubcf8\uc2a4\ud0c0\ud2b8\ub808\uc774\ub2dd\uc13c\ud130\uc758 \ud64d\uae30\uc120 \ubd80\uc6d0\uc7a5\uc740 \"\ub4dc\ub77c\ub9c8\uc758 \ubc30\uc5ed\ud558\uace0 \ud55c\ud6a8\uc8fc\uc758 \uc774\ubbf8\uc9c0\uac00 \uc798 \ub9e4\uce6d\uc774 \ub41c\ub2e4. \uac70\uae30\uc5d0\ub2e4\uac00 \ub300\uc0ac\ub294 \ubb3c\ub860\uc774\uace0 \ud45c\uc815 \uc5f0\uae30\ub3c4 \ub6f0\uc5b4\ub0ac\ub2e4\"\uace0 \ud638\ud3c9\ud588\ub2e4. \ud55c\ud6a8\uc8fc\ub294 SBS \uc5f0\uae30\ub300\uc0c1 \ud2b9\ubcc4\uae30\ud68d\ubd80\ubb38 \uc5ec\uc790\uc6b0\uc218\uc5f0\uae30\uc0c1, \ubca0\uc2a4\ud2b8\ucee4\ud50c\uc0c1 \ub4f1 3\uad00\uc655\uc744 \uc218\uc0c1\ud588\ub2e4. \uac19\uc740 \ud574, 11\uc6d4\uc5d0\ub294 \ud55c\uc77c \ud569\uc791\uc758 \ub4dc\ub77c\ub9c8 \ud615\uc2dd\uc758 \uc601\ud654 \u300a\ucc9c\uad6d\uc758 \uc6b0\ud3b8\ubc30\ub2ec\ubd80\u300b\uc5d0 \ucd9c\uc5f0\ud588\ub2e4.", "answer": "\ucc9c\uad6d\uc758 \uc6b0\ud3b8\ubc30\ub2ec\ubd80", "sentence": "\uac19\uc740 \ud574, 11\uc6d4\uc5d0\ub294 \ud55c\uc77c \ud569\uc791\uc758 \ub4dc\ub77c\ub9c8 \ud615\uc2dd\uc758 \uc601\ud654 \u300a \ucc9c\uad6d\uc758 \uc6b0\ud3b8\ubc30\ub2ec\ubd80 \u300b\uc5d0 \ucd9c\uc5f0\ud588\ub2e4.", "paragraph_sentence": "\ud55c\ud6a8\uc8fc\ub294 2009\ub144, \ub4dc\ub77c\ub9c8 \u300a\ucc2c\ub780\ud55c \uc720\uc0b0\u300b\uc5d0\uc11c \ud654\ub824\ud558\uac8c \uc544\ub984\ub2f5\uc9c4 \uc54a\uc9c0\ub9cc \ub4e4\uaf43 \uac19\uc740 \ud574\ub9d1\uc740 \ub9e4\ub825\uc744 \uac00\uc9c4 \uace0\uc740\uc131 \uc5ed\uc744 \ub9e1\uc544 1\ub144\uc5ec\ub9cc\uc5d0 \ube0c\ub77c\uc6b4\uad00\uc5d0 \ubcf5\uadc0\ud558\uc600\ub2e4. \uc774 \uc791\ud488\uc740 \ucd5c\uace0 \uc2dc\uccad\ub960 45.2%\ub97c \uae30\ub85d\ud558\uba70 \ud070 \uc778\uae30\ub97c \ub204\ub838\uace0, \ud55c\ud6a8\uc8fc\ub294 \uadf8\ub3d9\uc548 \ubd80\uc871\ud55c \uc810\uc73c\ub85c \uc9c0\uc801\ub410\ub358 \ub0b4\uba74\uc5f0\uae30\ub3c4 \ubcf4\uc644\ud558\uba70 \uc5f0\uae30\uc790\ub85c\uc11c \ucc2c\uc0ac\ub97c \ubc1b\uc558\ub2e4. \uc5f0\uae30\uc804\ubb38\ud559\uc6d0\uc778 \ubcf8\uc2a4\ud0c0\ud2b8\ub808\uc774\ub2dd\uc13c\ud130\uc758 \ud64d\uae30\uc120 \ubd80\uc6d0\uc7a5\uc740 \"\ub4dc\ub77c\ub9c8\uc758 \ubc30\uc5ed\ud558\uace0 \ud55c\ud6a8\uc8fc\uc758 \uc774\ubbf8\uc9c0\uac00 \uc798 \ub9e4\uce6d\uc774 \ub41c\ub2e4. \uac70\uae30\uc5d0\ub2e4\uac00 \ub300\uc0ac\ub294 \ubb3c\ub860\uc774\uace0 \ud45c\uc815 \uc5f0\uae30\ub3c4 \ub6f0\uc5b4\ub0ac\ub2e4\"\uace0 \ud638\ud3c9\ud588\ub2e4. \ud55c\ud6a8\uc8fc\ub294 SBS \uc5f0\uae30\ub300\uc0c1 \ud2b9\ubcc4\uae30\ud68d\ubd80\ubb38 \uc5ec\uc790\uc6b0\uc218\uc5f0\uae30\uc0c1, \ubca0\uc2a4\ud2b8\ucee4\ud50c\uc0c1 \ub4f1 3\uad00\uc655\uc744 \uc218\uc0c1\ud588\ub2e4. \uac19\uc740 \ud574, 11\uc6d4\uc5d0\ub294 \ud55c\uc77c \ud569\uc791\uc758 \ub4dc\ub77c\ub9c8 \ud615\uc2dd\uc758 \uc601\ud654 \u300a \ucc9c\uad6d\uc758 \uc6b0\ud3b8\ubc30\ub2ec\ubd80 \u300b\uc5d0 \ucd9c\uc5f0\ud588\ub2e4. ", "paragraph_answer": "\ud55c\ud6a8\uc8fc\ub294 2009\ub144, \ub4dc\ub77c\ub9c8 \u300a\ucc2c\ub780\ud55c \uc720\uc0b0\u300b\uc5d0\uc11c \ud654\ub824\ud558\uac8c \uc544\ub984\ub2f5\uc9c4 \uc54a\uc9c0\ub9cc \ub4e4\uaf43 \uac19\uc740 \ud574\ub9d1\uc740 \ub9e4\ub825\uc744 \uac00\uc9c4 \uace0\uc740\uc131 \uc5ed\uc744 \ub9e1\uc544 1\ub144\uc5ec\ub9cc\uc5d0 \ube0c\ub77c\uc6b4\uad00\uc5d0 \ubcf5\uadc0\ud558\uc600\ub2e4. \uc774 \uc791\ud488\uc740 \ucd5c\uace0 \uc2dc\uccad\ub960 45.2%\ub97c \uae30\ub85d\ud558\uba70 \ud070 \uc778\uae30\ub97c \ub204\ub838\uace0, \ud55c\ud6a8\uc8fc\ub294 \uadf8\ub3d9\uc548 \ubd80\uc871\ud55c \uc810\uc73c\ub85c \uc9c0\uc801\ub410\ub358 \ub0b4\uba74\uc5f0\uae30\ub3c4 \ubcf4\uc644\ud558\uba70 \uc5f0\uae30\uc790\ub85c\uc11c \ucc2c\uc0ac\ub97c \ubc1b\uc558\ub2e4. \uc5f0\uae30\uc804\ubb38\ud559\uc6d0\uc778 \ubcf8\uc2a4\ud0c0\ud2b8\ub808\uc774\ub2dd\uc13c\ud130\uc758 \ud64d\uae30\uc120 \ubd80\uc6d0\uc7a5\uc740 \"\ub4dc\ub77c\ub9c8\uc758 \ubc30\uc5ed\ud558\uace0 \ud55c\ud6a8\uc8fc\uc758 \uc774\ubbf8\uc9c0\uac00 \uc798 \ub9e4\uce6d\uc774 \ub41c\ub2e4. \uac70\uae30\uc5d0\ub2e4\uac00 \ub300\uc0ac\ub294 \ubb3c\ub860\uc774\uace0 \ud45c\uc815 \uc5f0\uae30\ub3c4 \ub6f0\uc5b4\ub0ac\ub2e4\"\uace0 \ud638\ud3c9\ud588\ub2e4. \ud55c\ud6a8\uc8fc\ub294 SBS \uc5f0\uae30\ub300\uc0c1 \ud2b9\ubcc4\uae30\ud68d\ubd80\ubb38 \uc5ec\uc790\uc6b0\uc218\uc5f0\uae30\uc0c1, \ubca0\uc2a4\ud2b8\ucee4\ud50c\uc0c1 \ub4f1 3\uad00\uc655\uc744 \uc218\uc0c1\ud588\ub2e4. \uac19\uc740 \ud574, 11\uc6d4\uc5d0\ub294 \ud55c\uc77c \ud569\uc791\uc758 \ub4dc\ub77c\ub9c8 \ud615\uc2dd\uc758 \uc601\ud654 \u300a \ucc9c\uad6d\uc758 \uc6b0\ud3b8\ubc30\ub2ec\ubd80 \u300b\uc5d0 \ucd9c\uc5f0\ud588\ub2e4.", "sentence_answer": "\uac19\uc740 \ud574, 11\uc6d4\uc5d0\ub294 \ud55c\uc77c \ud569\uc791\uc758 \ub4dc\ub77c\ub9c8 \ud615\uc2dd\uc758 \uc601\ud654 \u300a \ucc9c\uad6d\uc758 \uc6b0\ud3b8\ubc30\ub2ec\ubd80 \u300b\uc5d0 \ucd9c\uc5f0\ud588\ub2e4.", "paragraph_id": "6570194-4-1", "paragraph_question": "question: \ud55c\ud6a8\uc8fc\uac00 2009\ub144\uc5d0 \ucd9c\uc5f0\ud55c \ud55c\uc77c \ud569\uc791\uc758 \uc601\ud654\ub294?, context: \ud55c\ud6a8\uc8fc\ub294 2009\ub144, \ub4dc\ub77c\ub9c8 \u300a\ucc2c\ub780\ud55c \uc720\uc0b0\u300b\uc5d0\uc11c \ud654\ub824\ud558\uac8c \uc544\ub984\ub2f5\uc9c4 \uc54a\uc9c0\ub9cc \ub4e4\uaf43 \uac19\uc740 \ud574\ub9d1\uc740 \ub9e4\ub825\uc744 \uac00\uc9c4 \uace0\uc740\uc131 \uc5ed\uc744 \ub9e1\uc544 1\ub144\uc5ec\ub9cc\uc5d0 \ube0c\ub77c\uc6b4\uad00\uc5d0 \ubcf5\uadc0\ud558\uc600\ub2e4. \uc774 \uc791\ud488\uc740 \ucd5c\uace0 \uc2dc\uccad\ub960 45.2%\ub97c \uae30\ub85d\ud558\uba70 \ud070 \uc778\uae30\ub97c \ub204\ub838\uace0, \ud55c\ud6a8\uc8fc\ub294 \uadf8\ub3d9\uc548 \ubd80\uc871\ud55c \uc810\uc73c\ub85c \uc9c0\uc801\ub410\ub358 \ub0b4\uba74\uc5f0\uae30\ub3c4 \ubcf4\uc644\ud558\uba70 \uc5f0\uae30\uc790\ub85c\uc11c \ucc2c\uc0ac\ub97c \ubc1b\uc558\ub2e4. \uc5f0\uae30\uc804\ubb38\ud559\uc6d0\uc778 \ubcf8\uc2a4\ud0c0\ud2b8\ub808\uc774\ub2dd\uc13c\ud130\uc758 \ud64d\uae30\uc120 \ubd80\uc6d0\uc7a5\uc740 \"\ub4dc\ub77c\ub9c8\uc758 \ubc30\uc5ed\ud558\uace0 \ud55c\ud6a8\uc8fc\uc758 \uc774\ubbf8\uc9c0\uac00 \uc798 \ub9e4\uce6d\uc774 \ub41c\ub2e4. \uac70\uae30\uc5d0\ub2e4\uac00 \ub300\uc0ac\ub294 \ubb3c\ub860\uc774\uace0 \ud45c\uc815 \uc5f0\uae30\ub3c4 \ub6f0\uc5b4\ub0ac\ub2e4\"\uace0 \ud638\ud3c9\ud588\ub2e4. \ud55c\ud6a8\uc8fc\ub294 SBS \uc5f0\uae30\ub300\uc0c1 \ud2b9\ubcc4\uae30\ud68d\ubd80\ubb38 \uc5ec\uc790\uc6b0\uc218\uc5f0\uae30\uc0c1, \ubca0\uc2a4\ud2b8\ucee4\ud50c\uc0c1 \ub4f1 3\uad00\uc655\uc744 \uc218\uc0c1\ud588\ub2e4. \uac19\uc740 \ud574, 11\uc6d4\uc5d0\ub294 \ud55c\uc77c \ud569\uc791\uc758 \ub4dc\ub77c\ub9c8 \ud615\uc2dd\uc758 \uc601\ud654 \u300a\ucc9c\uad6d\uc758 \uc6b0\ud3b8\ubc30\ub2ec\ubd80\u300b\uc5d0 \ucd9c\uc5f0\ud588\ub2e4."} {"question": "\uc1fc\uad70\ub9cc\uc774 \uc0ac\uc6a9\ud560 \uc218 \uc788\uc5c8\ub358 \uc0c9\uc0c1\uc740 \ubb34\uc5c7\uc778\uac00?", "paragraph": "\ud788\ud0c0\ud0c0\ub808\ub098 \ub2e4\uc774\ubaac, \uc2a4\uc624\ub294 \uace0\uc704 \ubb34\uac00\uc5d0\uc11c\ub9cc \ucc29\uc6a9\ud558\ub294 \uc758\ubcf5\uc774 \ub418\uc5c8\ub2e4. \ud788\ud0c0\ud0c0\ub808\ub97c \uc785\ub294 \uac83\uc774 \ud5c8\ub77d\ub41c \uac83\uc740 4\uc704 \uc774\uc0c1\uc758 \ubb34\uc0ac\ub4e4 \ubfd0\uc774\uc5c8\uace0, \uc774\ub4e4 \ubaa8\ub450\uac00 \ub9c9\uac1d(\u5e55\u95a3)\uc744 \ubc30\ucd9c\ud55c \uc720\ub825 \ud6c4\ub2e4\uc774 \ub2e4\uc774\ubb18(\u8b5c\u4ee3\u5927\u540d)\uac70\ub098 \uad6c\ub2c8\ubaa8\uce58 \ub2e4\uc774\ubb18(\u56fd\u6301\u5927\u540d), \uace0\uc0b0\ucf00(\u5fa1\u4e09\u5bb6) \uadf8\ub9ac\uace0 \ub9c9\ubd80\uc758 \uc1fc\uad70\uc5d0 \ud574\ub2f9\ud558\ub294 \uc790\ub4e4\uc774\uc5c8\ub2e4. \ub610\ud55c \uc637\uc758 \uc0c9\uae54\ub3c4 \ud3ec\ub3c4\uc0c9(\u8461\u8404\u8272)\ub294 \uc1fc\uad70\ub9cc\uc774 \uc0ac\uc6a9\ud560 \uc218 \uc788\ub3c4\ub85d \ud588\uace0, \ube44\uc0c9(\u7dcb\u8272)\uc740 \ub2e4\uc774\ub098\uace4(\u5927\u7d0d\u8a00) \uc774\uc0c1\ub9cc\uc774, \ub2f4\ud669\uc0c9\uacfc \uc5f0\ub450\uc0c9(\u840c\u9ec4\u8272)\uc740 \uc544\uc608 \uc0ac\uc6a9\uc744 \uae08\uc9c0\uc2dc\ucf1c \ubc84\ub838\ub2e4. \uc0ac\uc6a9\ud560 \uc218 \uc788\ub294 \uc2dc\uae30\ub3c4 \uad00\ub840 \ub54c\ub098 \uc1fc\uad70\uc5d0\uac8c \uc0c8\ud574 \uc778\uc0ac\ub97c \ub4dc\ub9b4 \ub54c \ubfd0\uc73c\ub85c \uadf8 \uc0ac\uc6a9\uc774 \ubab9\uc2dc \uc81c\ud55c\ub418\uc5c8\ub2e4. \ud558\uce74\ub9c8\uc758 \uae38\uc774\ub3c4 \ubab9\uc2dc \uae38\uc5b4\uc838 \ub545\uc5d0 \uc9c8\uc9c8 \ub04c\ub9b4 \uc815\ub3c4\ub85c \ud65c\ub3d9\uc131\uc774 \ub5a8\uc5b4\uc9c0\uace0, \ud788\ud0c0\ud0c0\ub808\uc5d0 \uac16\ucdb0\uc11c \ucc28\ub294 \uce7c\ub3c4 \uc9e7\uc544\uc84c\ub2e4.", "answer": "\ud3ec\ub3c4\uc0c9", "sentence": "\ub610\ud55c \uc637\uc758 \uc0c9\uae54\ub3c4 \ud3ec\ub3c4\uc0c9 (\u8461\u8404\u8272)", "paragraph_sentence": "\ud788\ud0c0\ud0c0\ub808\ub098 \ub2e4\uc774\ubaac, \uc2a4\uc624\ub294 \uace0\uc704 \ubb34\uac00\uc5d0\uc11c\ub9cc \ucc29\uc6a9\ud558\ub294 \uc758\ubcf5\uc774 \ub418\uc5c8\ub2e4. \ud788\ud0c0\ud0c0\ub808\ub97c \uc785\ub294 \uac83\uc774 \ud5c8\ub77d\ub41c \uac83\uc740 4\uc704 \uc774\uc0c1\uc758 \ubb34\uc0ac\ub4e4 \ubfd0\uc774\uc5c8\uace0, \uc774\ub4e4 \ubaa8\ub450\uac00 \ub9c9\uac1d(\u5e55\u95a3)\uc744 \ubc30\ucd9c\ud55c \uc720\ub825 \ud6c4\ub2e4\uc774 \ub2e4\uc774\ubb18(\u8b5c\u4ee3\u5927\u540d)\uac70\ub098 \uad6c\ub2c8\ubaa8\uce58 \ub2e4\uc774\ubb18(\u56fd\u6301\u5927\u540d), \uace0\uc0b0\ucf00(\u5fa1\u4e09\u5bb6) \uadf8\ub9ac\uace0 \ub9c9\ubd80\uc758 \uc1fc\uad70\uc5d0 \ud574\ub2f9\ud558\ub294 \uc790\ub4e4\uc774\uc5c8\ub2e4. \ub610\ud55c \uc637\uc758 \uc0c9\uae54\ub3c4 \ud3ec\ub3c4\uc0c9 (\u8461\u8404\u8272) \ub294 \uc1fc\uad70\ub9cc\uc774 \uc0ac\uc6a9\ud560 \uc218 \uc788\ub3c4\ub85d \ud588\uace0, \ube44\uc0c9(\u7dcb\u8272)\uc740 \ub2e4\uc774\ub098\uace4(\u5927\u7d0d\u8a00) \uc774\uc0c1\ub9cc\uc774, \ub2f4\ud669\uc0c9\uacfc \uc5f0\ub450\uc0c9(\u840c\u9ec4\u8272)\uc740 \uc544\uc608 \uc0ac\uc6a9\uc744 \uae08\uc9c0\uc2dc\ucf1c \ubc84\ub838\ub2e4. \uc0ac\uc6a9\ud560 \uc218 \uc788\ub294 \uc2dc\uae30\ub3c4 \uad00\ub840 \ub54c\ub098 \uc1fc\uad70\uc5d0\uac8c \uc0c8\ud574 \uc778\uc0ac\ub97c \ub4dc\ub9b4 \ub54c \ubfd0\uc73c\ub85c \uadf8 \uc0ac\uc6a9\uc774 \ubab9\uc2dc \uc81c\ud55c\ub418\uc5c8\ub2e4. \ud558\uce74\ub9c8\uc758 \uae38\uc774\ub3c4 \ubab9\uc2dc \uae38\uc5b4\uc838 \ub545\uc5d0 \uc9c8\uc9c8 \ub04c\ub9b4 \uc815\ub3c4\ub85c \ud65c\ub3d9\uc131\uc774 \ub5a8\uc5b4\uc9c0\uace0, \ud788\ud0c0\ud0c0\ub808\uc5d0 \uac16\ucdb0\uc11c \ucc28\ub294 \uce7c\ub3c4 \uc9e7\uc544\uc84c\ub2e4.", "paragraph_answer": "\ud788\ud0c0\ud0c0\ub808\ub098 \ub2e4\uc774\ubaac, \uc2a4\uc624\ub294 \uace0\uc704 \ubb34\uac00\uc5d0\uc11c\ub9cc \ucc29\uc6a9\ud558\ub294 \uc758\ubcf5\uc774 \ub418\uc5c8\ub2e4. \ud788\ud0c0\ud0c0\ub808\ub97c \uc785\ub294 \uac83\uc774 \ud5c8\ub77d\ub41c \uac83\uc740 4\uc704 \uc774\uc0c1\uc758 \ubb34\uc0ac\ub4e4 \ubfd0\uc774\uc5c8\uace0, \uc774\ub4e4 \ubaa8\ub450\uac00 \ub9c9\uac1d(\u5e55\u95a3)\uc744 \ubc30\ucd9c\ud55c \uc720\ub825 \ud6c4\ub2e4\uc774 \ub2e4\uc774\ubb18(\u8b5c\u4ee3\u5927\u540d)\uac70\ub098 \uad6c\ub2c8\ubaa8\uce58 \ub2e4\uc774\ubb18(\u56fd\u6301\u5927\u540d), \uace0\uc0b0\ucf00(\u5fa1\u4e09\u5bb6) \uadf8\ub9ac\uace0 \ub9c9\ubd80\uc758 \uc1fc\uad70\uc5d0 \ud574\ub2f9\ud558\ub294 \uc790\ub4e4\uc774\uc5c8\ub2e4. \ub610\ud55c \uc637\uc758 \uc0c9\uae54\ub3c4 \ud3ec\ub3c4\uc0c9 (\u8461\u8404\u8272)\ub294 \uc1fc\uad70\ub9cc\uc774 \uc0ac\uc6a9\ud560 \uc218 \uc788\ub3c4\ub85d \ud588\uace0, \ube44\uc0c9(\u7dcb\u8272)\uc740 \ub2e4\uc774\ub098\uace4(\u5927\u7d0d\u8a00) \uc774\uc0c1\ub9cc\uc774, \ub2f4\ud669\uc0c9\uacfc \uc5f0\ub450\uc0c9(\u840c\u9ec4\u8272)\uc740 \uc544\uc608 \uc0ac\uc6a9\uc744 \uae08\uc9c0\uc2dc\ucf1c \ubc84\ub838\ub2e4. \uc0ac\uc6a9\ud560 \uc218 \uc788\ub294 \uc2dc\uae30\ub3c4 \uad00\ub840 \ub54c\ub098 \uc1fc\uad70\uc5d0\uac8c \uc0c8\ud574 \uc778\uc0ac\ub97c \ub4dc\ub9b4 \ub54c \ubfd0\uc73c\ub85c \uadf8 \uc0ac\uc6a9\uc774 \ubab9\uc2dc \uc81c\ud55c\ub418\uc5c8\ub2e4. \ud558\uce74\ub9c8\uc758 \uae38\uc774\ub3c4 \ubab9\uc2dc \uae38\uc5b4\uc838 \ub545\uc5d0 \uc9c8\uc9c8 \ub04c\ub9b4 \uc815\ub3c4\ub85c \ud65c\ub3d9\uc131\uc774 \ub5a8\uc5b4\uc9c0\uace0, \ud788\ud0c0\ud0c0\ub808\uc5d0 \uac16\ucdb0\uc11c \ucc28\ub294 \uce7c\ub3c4 \uc9e7\uc544\uc84c\ub2e4.", "sentence_answer": "\ub610\ud55c \uc637\uc758 \uc0c9\uae54\ub3c4 \ud3ec\ub3c4\uc0c9 (\u8461\u8404\u8272)", "paragraph_id": "6574284-2-1", "paragraph_question": "question: \uc1fc\uad70\ub9cc\uc774 \uc0ac\uc6a9\ud560 \uc218 \uc788\uc5c8\ub358 \uc0c9\uc0c1\uc740 \ubb34\uc5c7\uc778\uac00?, context: \ud788\ud0c0\ud0c0\ub808\ub098 \ub2e4\uc774\ubaac, \uc2a4\uc624\ub294 \uace0\uc704 \ubb34\uac00\uc5d0\uc11c\ub9cc \ucc29\uc6a9\ud558\ub294 \uc758\ubcf5\uc774 \ub418\uc5c8\ub2e4. \ud788\ud0c0\ud0c0\ub808\ub97c \uc785\ub294 \uac83\uc774 \ud5c8\ub77d\ub41c \uac83\uc740 4\uc704 \uc774\uc0c1\uc758 \ubb34\uc0ac\ub4e4 \ubfd0\uc774\uc5c8\uace0, \uc774\ub4e4 \ubaa8\ub450\uac00 \ub9c9\uac1d(\u5e55\u95a3)\uc744 \ubc30\ucd9c\ud55c \uc720\ub825 \ud6c4\ub2e4\uc774 \ub2e4\uc774\ubb18(\u8b5c\u4ee3\u5927\u540d)\uac70\ub098 \uad6c\ub2c8\ubaa8\uce58 \ub2e4\uc774\ubb18(\u56fd\u6301\u5927\u540d), \uace0\uc0b0\ucf00(\u5fa1\u4e09\u5bb6) \uadf8\ub9ac\uace0 \ub9c9\ubd80\uc758 \uc1fc\uad70\uc5d0 \ud574\ub2f9\ud558\ub294 \uc790\ub4e4\uc774\uc5c8\ub2e4. \ub610\ud55c \uc637\uc758 \uc0c9\uae54\ub3c4 \ud3ec\ub3c4\uc0c9(\u8461\u8404\u8272)\ub294 \uc1fc\uad70\ub9cc\uc774 \uc0ac\uc6a9\ud560 \uc218 \uc788\ub3c4\ub85d \ud588\uace0, \ube44\uc0c9(\u7dcb\u8272)\uc740 \ub2e4\uc774\ub098\uace4(\u5927\u7d0d\u8a00) \uc774\uc0c1\ub9cc\uc774, \ub2f4\ud669\uc0c9\uacfc \uc5f0\ub450\uc0c9(\u840c\u9ec4\u8272)\uc740 \uc544\uc608 \uc0ac\uc6a9\uc744 \uae08\uc9c0\uc2dc\ucf1c \ubc84\ub838\ub2e4. \uc0ac\uc6a9\ud560 \uc218 \uc788\ub294 \uc2dc\uae30\ub3c4 \uad00\ub840 \ub54c\ub098 \uc1fc\uad70\uc5d0\uac8c \uc0c8\ud574 \uc778\uc0ac\ub97c \ub4dc\ub9b4 \ub54c \ubfd0\uc73c\ub85c \uadf8 \uc0ac\uc6a9\uc774 \ubab9\uc2dc \uc81c\ud55c\ub418\uc5c8\ub2e4. \ud558\uce74\ub9c8\uc758 \uae38\uc774\ub3c4 \ubab9\uc2dc \uae38\uc5b4\uc838 \ub545\uc5d0 \uc9c8\uc9c8 \ub04c\ub9b4 \uc815\ub3c4\ub85c \ud65c\ub3d9\uc131\uc774 \ub5a8\uc5b4\uc9c0\uace0, \ud788\ud0c0\ud0c0\ub808\uc5d0 \uac16\ucdb0\uc11c \ucc28\ub294 \uce7c\ub3c4 \uc9e7\uc544\uc84c\ub2e4."} {"question": "\uc57c\ub178\uac00 \ub450 \ubc88\uc9f8 \ub178\ud788\ud2b8 \ub178\ub7f0\uc744 \ud568\uaed8 \uae30\ub85d\ud55c \uc0ac\ub78c\uc740 \ub204\uad6c\uc778\uac00\uc694?", "paragraph": "\ud55c\uc2e0 \ud0c0\uc774\uac70\uc2a4\uc5d0\uc11c\ub294 \ub2f9\uc2dc \ud55c\uc2e0\uc758 \uac10\ub3c5\uc778 \uc694\uc2dc\ub2e4 \uc694\uc2dc\uc624\uc5d0\uac8c \ub192\uc740 \ud3c9\uac00\ub97c \ubc1b\uc73c\uba70, \uc57c\ub9c8\ub2e4 \uac00\uc4f0\ud788\ucf54\ub97c \ubc00\uc5b4\ub0b4\uace0 \uc8fc\uc804 \ud3ec\uc218 \uc790\ub9ac\ub97c \ucc28\uc9c0\ud558\uc600\ub2e4. 1998\ub144 5\uc6d4 26\uc77c\uc5d0 \uc5f4\ub9b0 \uc774\uc804 \uc18c\uc18d\ud300\uc778 \uc8fc\ub2c8\uce58\uc640\uc758 \uacbd\uae30\uc5d0\uc11c \uac00\uc640\uc9c0\ub9ac \ub370\uc4f0\ub85c\uc640 \ud568\uaed8 \ub450 \ubc88\uc9f8 \ub178\ud788\ud2b8 \ub178\ub7f0\uc744 \uae30\ub85d\ud588\uace0, 1999\ub144\uc5d0\ub294 \ucc98\uc74c\uc73c\ub85c 3\ud560 \ub300\uc758 \ud0c0\uc728\uc744 \uae30\ub85d\ud588\ub2e4. 2001\ub144\uc5d0 \uac10\ub3c5\uc9c1\uc744 \uc0ac\uc784\ud55c \ub178\ubb34\ub77c \uac00\uc4f0\uc57c\uc758 \ud6c4\uc784\uc73c\ub85c \uc8fc\ub2c8\uce58 \uc18c\uc18d \uc2dc\uc808\uc5d0 \uc790\uc2e0\uc744 \ud2b8\ub808\uc774\ub4dc\ub85c \ubc29\ucd9c\ud55c \ud638\uc2dc\ub178 \uc13c\uc774\uce58\uac00 \ud55c\uc2e0\uc758 \ucc28\uae30 \uac10\ub3c5\uc73c\ub85c \ucde8\uc784\ud558\uc790 \ub2e4\uc2dc \uc911\uc6a9\ub418\uc9c0 \uc54a\uace0 \ubc29\ucd9c\ub420\uae4c\ubd10 \ubd88\uc548\ud574 \ud558\uba70 \uc2e4\ub9dd\uac10\uc744 \ub4dc\ub7ec\ub0b4\uae30\ub3c4 \ud588\ub2e4. 2002\ub144\uc5d0\ub294 \ub450 \ucc28\ub840\uc758 \uace8\uc808\uc0c1\uc744 \uc785\ub294 \ub4f1 \ubd80\uc0c1\uc774 \uc7a6\uc544\uc9c0\uba74\uc11c \ud300\ub3c4 \uc57c\ub178\uc758 \uc804\ub825 \uc774\ud0c8\uacfc \ud568\uaed8 \ucd5c\ud558\uc704(4\uc704)\ub97c \uae30\ub85d\ud588\ub2e4.", "answer": "\uac00\uc640\uc9c0\ub9ac \ub370\uc4f0\ub85c", "sentence": "1998\ub144 5\uc6d4 26\uc77c\uc5d0 \uc5f4\ub9b0 \uc774\uc804 \uc18c\uc18d\ud300\uc778 \uc8fc\ub2c8\uce58\uc640\uc758 \uacbd\uae30\uc5d0\uc11c \uac00\uc640\uc9c0\ub9ac \ub370\uc4f0\ub85c \uc640 \ud568\uaed8 \ub450 \ubc88\uc9f8 \ub178\ud788\ud2b8 \ub178\ub7f0\uc744 \uae30\ub85d\ud588\uace0, 1999\ub144\uc5d0\ub294 \ucc98\uc74c\uc73c\ub85c 3\ud560 \ub300\uc758 \ud0c0\uc728\uc744 \uae30\ub85d\ud588\ub2e4.", "paragraph_sentence": "\ud55c\uc2e0 \ud0c0\uc774\uac70\uc2a4\uc5d0\uc11c\ub294 \ub2f9\uc2dc \ud55c\uc2e0\uc758 \uac10\ub3c5\uc778 \uc694\uc2dc\ub2e4 \uc694\uc2dc\uc624\uc5d0\uac8c \ub192\uc740 \ud3c9\uac00\ub97c \ubc1b\uc73c\uba70, \uc57c\ub9c8\ub2e4 \uac00\uc4f0\ud788\ucf54\ub97c \ubc00\uc5b4\ub0b4\uace0 \uc8fc\uc804 \ud3ec\uc218 \uc790\ub9ac\ub97c \ucc28\uc9c0\ud558\uc600\ub2e4. 1998\ub144 5\uc6d4 26\uc77c\uc5d0 \uc5f4\ub9b0 \uc774\uc804 \uc18c\uc18d\ud300\uc778 \uc8fc\ub2c8\uce58\uc640\uc758 \uacbd\uae30\uc5d0\uc11c \uac00\uc640\uc9c0\ub9ac \ub370\uc4f0\ub85c \uc640 \ud568\uaed8 \ub450 \ubc88\uc9f8 \ub178\ud788\ud2b8 \ub178\ub7f0\uc744 \uae30\ub85d\ud588\uace0, 1999\ub144\uc5d0\ub294 \ucc98\uc74c\uc73c\ub85c 3\ud560 \ub300\uc758 \ud0c0\uc728\uc744 \uae30\ub85d\ud588\ub2e4. 2001\ub144\uc5d0 \uac10\ub3c5\uc9c1\uc744 \uc0ac\uc784\ud55c \ub178\ubb34\ub77c \uac00\uc4f0\uc57c\uc758 \ud6c4\uc784\uc73c\ub85c \uc8fc\ub2c8\uce58 \uc18c\uc18d \uc2dc\uc808\uc5d0 \uc790\uc2e0\uc744 \ud2b8\ub808\uc774\ub4dc\ub85c \ubc29\ucd9c\ud55c \ud638\uc2dc\ub178 \uc13c\uc774\uce58\uac00 \ud55c\uc2e0\uc758 \ucc28\uae30 \uac10\ub3c5\uc73c\ub85c \ucde8\uc784\ud558\uc790 \ub2e4\uc2dc \uc911\uc6a9\ub418\uc9c0 \uc54a\uace0 \ubc29\ucd9c\ub420\uae4c\ubd10 \ubd88\uc548\ud574 \ud558\uba70 \uc2e4\ub9dd\uac10\uc744 \ub4dc\ub7ec\ub0b4\uae30\ub3c4 \ud588\ub2e4. 2002\ub144\uc5d0\ub294 \ub450 \ucc28\ub840\uc758 \uace8\uc808\uc0c1\uc744 \uc785\ub294 \ub4f1 \ubd80\uc0c1\uc774 \uc7a6\uc544\uc9c0\uba74\uc11c \ud300\ub3c4 \uc57c\ub178\uc758 \uc804\ub825 \uc774\ud0c8\uacfc \ud568\uaed8 \ucd5c\ud558\uc704(4\uc704)\ub97c \uae30\ub85d\ud588\ub2e4.", "paragraph_answer": "\ud55c\uc2e0 \ud0c0\uc774\uac70\uc2a4\uc5d0\uc11c\ub294 \ub2f9\uc2dc \ud55c\uc2e0\uc758 \uac10\ub3c5\uc778 \uc694\uc2dc\ub2e4 \uc694\uc2dc\uc624\uc5d0\uac8c \ub192\uc740 \ud3c9\uac00\ub97c \ubc1b\uc73c\uba70, \uc57c\ub9c8\ub2e4 \uac00\uc4f0\ud788\ucf54\ub97c \ubc00\uc5b4\ub0b4\uace0 \uc8fc\uc804 \ud3ec\uc218 \uc790\ub9ac\ub97c \ucc28\uc9c0\ud558\uc600\ub2e4. 1998\ub144 5\uc6d4 26\uc77c\uc5d0 \uc5f4\ub9b0 \uc774\uc804 \uc18c\uc18d\ud300\uc778 \uc8fc\ub2c8\uce58\uc640\uc758 \uacbd\uae30\uc5d0\uc11c \uac00\uc640\uc9c0\ub9ac \ub370\uc4f0\ub85c \uc640 \ud568\uaed8 \ub450 \ubc88\uc9f8 \ub178\ud788\ud2b8 \ub178\ub7f0\uc744 \uae30\ub85d\ud588\uace0, 1999\ub144\uc5d0\ub294 \ucc98\uc74c\uc73c\ub85c 3\ud560 \ub300\uc758 \ud0c0\uc728\uc744 \uae30\ub85d\ud588\ub2e4. 2001\ub144\uc5d0 \uac10\ub3c5\uc9c1\uc744 \uc0ac\uc784\ud55c \ub178\ubb34\ub77c \uac00\uc4f0\uc57c\uc758 \ud6c4\uc784\uc73c\ub85c \uc8fc\ub2c8\uce58 \uc18c\uc18d \uc2dc\uc808\uc5d0 \uc790\uc2e0\uc744 \ud2b8\ub808\uc774\ub4dc\ub85c \ubc29\ucd9c\ud55c \ud638\uc2dc\ub178 \uc13c\uc774\uce58\uac00 \ud55c\uc2e0\uc758 \ucc28\uae30 \uac10\ub3c5\uc73c\ub85c \ucde8\uc784\ud558\uc790 \ub2e4\uc2dc \uc911\uc6a9\ub418\uc9c0 \uc54a\uace0 \ubc29\ucd9c\ub420\uae4c\ubd10 \ubd88\uc548\ud574 \ud558\uba70 \uc2e4\ub9dd\uac10\uc744 \ub4dc\ub7ec\ub0b4\uae30\ub3c4 \ud588\ub2e4. 2002\ub144\uc5d0\ub294 \ub450 \ucc28\ub840\uc758 \uace8\uc808\uc0c1\uc744 \uc785\ub294 \ub4f1 \ubd80\uc0c1\uc774 \uc7a6\uc544\uc9c0\uba74\uc11c \ud300\ub3c4 \uc57c\ub178\uc758 \uc804\ub825 \uc774\ud0c8\uacfc \ud568\uaed8 \ucd5c\ud558\uc704(4\uc704)\ub97c \uae30\ub85d\ud588\ub2e4.", "sentence_answer": "1998\ub144 5\uc6d4 26\uc77c\uc5d0 \uc5f4\ub9b0 \uc774\uc804 \uc18c\uc18d\ud300\uc778 \uc8fc\ub2c8\uce58\uc640\uc758 \uacbd\uae30\uc5d0\uc11c \uac00\uc640\uc9c0\ub9ac \ub370\uc4f0\ub85c \uc640 \ud568\uaed8 \ub450 \ubc88\uc9f8 \ub178\ud788\ud2b8 \ub178\ub7f0\uc744 \uae30\ub85d\ud588\uace0, 1999\ub144\uc5d0\ub294 \ucc98\uc74c\uc73c\ub85c 3\ud560 \ub300\uc758 \ud0c0\uc728\uc744 \uae30\ub85d\ud588\ub2e4.", "paragraph_id": "6522582-0-1", "paragraph_question": "question: \uc57c\ub178\uac00 \ub450 \ubc88\uc9f8 \ub178\ud788\ud2b8 \ub178\ub7f0\uc744 \ud568\uaed8 \uae30\ub85d\ud55c \uc0ac\ub78c\uc740 \ub204\uad6c\uc778\uac00\uc694?, context: \ud55c\uc2e0 \ud0c0\uc774\uac70\uc2a4\uc5d0\uc11c\ub294 \ub2f9\uc2dc \ud55c\uc2e0\uc758 \uac10\ub3c5\uc778 \uc694\uc2dc\ub2e4 \uc694\uc2dc\uc624\uc5d0\uac8c \ub192\uc740 \ud3c9\uac00\ub97c \ubc1b\uc73c\uba70, \uc57c\ub9c8\ub2e4 \uac00\uc4f0\ud788\ucf54\ub97c \ubc00\uc5b4\ub0b4\uace0 \uc8fc\uc804 \ud3ec\uc218 \uc790\ub9ac\ub97c \ucc28\uc9c0\ud558\uc600\ub2e4. 1998\ub144 5\uc6d4 26\uc77c\uc5d0 \uc5f4\ub9b0 \uc774\uc804 \uc18c\uc18d\ud300\uc778 \uc8fc\ub2c8\uce58\uc640\uc758 \uacbd\uae30\uc5d0\uc11c \uac00\uc640\uc9c0\ub9ac \ub370\uc4f0\ub85c\uc640 \ud568\uaed8 \ub450 \ubc88\uc9f8 \ub178\ud788\ud2b8 \ub178\ub7f0\uc744 \uae30\ub85d\ud588\uace0, 1999\ub144\uc5d0\ub294 \ucc98\uc74c\uc73c\ub85c 3\ud560 \ub300\uc758 \ud0c0\uc728\uc744 \uae30\ub85d\ud588\ub2e4. 2001\ub144\uc5d0 \uac10\ub3c5\uc9c1\uc744 \uc0ac\uc784\ud55c \ub178\ubb34\ub77c \uac00\uc4f0\uc57c\uc758 \ud6c4\uc784\uc73c\ub85c \uc8fc\ub2c8\uce58 \uc18c\uc18d \uc2dc\uc808\uc5d0 \uc790\uc2e0\uc744 \ud2b8\ub808\uc774\ub4dc\ub85c \ubc29\ucd9c\ud55c \ud638\uc2dc\ub178 \uc13c\uc774\uce58\uac00 \ud55c\uc2e0\uc758 \ucc28\uae30 \uac10\ub3c5\uc73c\ub85c \ucde8\uc784\ud558\uc790 \ub2e4\uc2dc \uc911\uc6a9\ub418\uc9c0 \uc54a\uace0 \ubc29\ucd9c\ub420\uae4c\ubd10 \ubd88\uc548\ud574 \ud558\uba70 \uc2e4\ub9dd\uac10\uc744 \ub4dc\ub7ec\ub0b4\uae30\ub3c4 \ud588\ub2e4. 2002\ub144\uc5d0\ub294 \ub450 \ucc28\ub840\uc758 \uace8\uc808\uc0c1\uc744 \uc785\ub294 \ub4f1 \ubd80\uc0c1\uc774 \uc7a6\uc544\uc9c0\uba74\uc11c \ud300\ub3c4 \uc57c\ub178\uc758 \uc804\ub825 \uc774\ud0c8\uacfc \ud568\uaed8 \ucd5c\ud558\uc704(4\uc704)\ub97c \uae30\ub85d\ud588\ub2e4."} {"question": "\uc774\uc2a4\ud2b8\ub85c\uc758 \ud574\uccb4\uac00 \ucd5c\uc885 \uacb0\uc815\ub41c \uc2dc\uae30\ub294?", "paragraph": "2009\ub144 3\uc6d4 29\uc77c \uc2a4\ud398\uc15c\ud3ec\uc2a4 \ud504\ub85c\uac8c\uc784\ub2e8 \ub4dc\ub798\ud504\ud2b8\uc5d0\uc11c e.sports-united\ub97c \uc9c0\uba85\ud558\uc600\ub2e4. 2009\ub144 3\uc6d4 KeSPA \ub7ad\ud0b9 1\uc704\uc774\uc790 \ucd5c\ub300\uc5b4\ub85c \ud3c9\uac00\ubc1b\uace0 \uc788\ub294 e.sports-united\ub294 \uc774\uc2a4\ud2b8\ub85c\uc758 \ud300 \ub2e8\uc704 \uc9c0\uba85\uc744 \ubc1b\uc558\ub2e4. \ucd94\ucca8\uc744 \ud1b5\ud574 1\uc21c\uc704 \uc9c0\uba85\uad8c\uc744 \uc5bb\uc740 \uc774\uc2a4\ud2b8\ub85c\uc758 \uae40\ud604\uc9c4 \uac10\ub3c5\uc740 \uc120\ubc1c\uc804 2\uc704\ub97c \ucc28\uc9c0\ud55c e.sports-united\ub97c \uc120\ubc1c\ud588\ub2e4. \uadf8\ub9ac\ud558\uc5ec, 2009\ub144 8\uc6d4 6\uc77c \uc2a4\ud398\uc15c\ud3ec\uc2a4 \uc774\uc2a4\ud2b8\ub85c\uac00 \uad11\uc548\ub9ac \ubb34\ub300\ub97c \uc815\ubcf5\ud588\ub2e4. \uc774\uc2a4\ud2b8\ub85c\uac00 KT \ud551\uac70\ubd90\uc744 \uaebe\uace0 \ud504\ub85c\ub9ac\uadf8 \uc815\uc0c1\uc5d0 \uc62c\ub790\ub2e4. \uc774\ub85c\uc368 \uc774\uc2a4\ud2b8\ub85c\ub294 \uc2a4\ud398\uc15c\ud3ec\uc2a4 \ud504\ub85c\ub9ac\uadf8 \uc6d0\ub144\ub9ac\uadf8\uc5d0\uc11c \ucd08\ub300 \ucc54\ud53c\uc5b8\uc5d0 \uc62c\ub790\ub2e4. \uc774\ub85c\uc368 \uc774\uc2a4\ud2b8\ub85c\uc758 \ucc98\uc74c\uc774\uc790 \ub9c8\uc9c0\ub9c9 \uc6b0\uc2b9\uc744 \ucc28\uc9c0\ud588\ub2e4. 2010\ub144 9\uc6d4 IEG\uac00 \uc774\uc2a4\ud2b8\ub85c\ub97c \ud55c\uad6de\uc2a4\ud3ec\uce20\ud611\ud68c\uc5d0 \ub9e4\uac01 \uc704\ud0c1\ud558\uba70 \uad6c\ub2e8\uc6b4\uc601\uc744 \ud3ec\uae30\ud558\uac8c \ub41c\ub2e4. \uc774\uc2a4\ud2b8\ub85c\uc758 \ub9e4\uac01 \uc704\ud0c1\uc5d0 \ub300\ud574 \ud55c\uad6de\uc2a4\ud3ec\uce20\ud611\ud68c \uad00\uacc4\uc790\ub294 \"\ud604\uc7ac \uc0c1\ud669\uc5d0\uc11c\ub294 \ucd5c\ub300\ud55c \uc6d0\ud65c\ud558\uac8c \ub9e4\uac01 \ud611\uc0c1\uc744 \uc9c4\ud589\ud574\uc11c \uac8c\uc784\ub2e8\uc744 \uc720\uc9c0\ud558\uace0 \uc120\uc218\ub4e4\uc758 \uace0\uc6a9 \uc2b9\uacc4\ub97c \uc704\ud574 \ucd5c\uc120\uc758 \ub178\ub825\uc744 \ub2e4\ud558\uace0 \uc788\ub2e4\" \uace0 \uc804\ud588\ub2e4. \uadf8\ub7ec\ub098, 2010\ub144 10\uc6d4 13\uc77c \ud55c\uad6de\uc2a4\ud3ec\uce20\ud611\ud68c\ub294 \uc774\uc2a4\ud2b8\ub85c\uc758 \ud574\uccb4\ub97c \ucd5c\uc885 \uacb0\uc815\ud558\uc600\ub2e4. \uae30\uc874\uc758 \uc120\uc218\ub4e4\uc740 \uc774\uc2a4\ud2b8\ub85c \uc120\uc218\ub2e8 \uacf5\uac1c \ub4dc\ub798\ud504\ud2b8\ub97c \ud1b5\ud574 \ub2e4\ub978 \ud300\uc5d0 \uac01\uac01 \uc601\uc785\ub418\uc5b4, \ub4dc\ub798\ud504\ud2b8\ub97c \uc644\ub8cc\ud558\uc600\ub2e4. \uc774\ubc88 \uacf5\uac1c \ub4dc\ub798\ud504\ud2b8\ub294 \uc785\ucc30 \uacfc\uc815\uc744 \ud1b5\ud574 \ud574\ub2f9 \uc120\uc218\ub97c \uc601\uc785\ud558\ub294 \ud3ec\uc2a4\ud305 \uc2dc\uc2a4\ud15c(Posting System)\uc744 \ud1b5\ud574 \u2018\ubc15\uc0c1\uc6b0, \uae40\uc131\ub300, \uc2e0\ub300\uadfc, \uc2e0\uc7ac\uc6b1, \uae40\ub3c4\uc6b0\u2019 \uac00 \uac8c\uc784\ub2e8\uc758 \uc9c0\uba85\uc744 \ubc1b\uc558\uc73c\uba70, \uc774\ud6c4 \uacf5\uad70\uc744 \uc81c\uc678\ud55c \ud3ec\uc2a4\ud305\uc5d0 \ucc38\uc5ec\ud558\uc9c0 \uc54a\uc740 \uac8c\uc784\ub2e8 \uc9c0\uba85\uc5d0\uc11c \u2018\uc720\ubcd1\uc900, \ubc31\ub3d9\uc900, \uc784\ud64d\uaddc, \ub178\uc7ac\uc0c1, \uc8fc\uc131\uc6b1, \uae40\uae30\ud6c8\u2019\uc774 \uc9c0\uba85\uc744 \ubc1b\uc544 \uc120\uc218 \uc804\uc6d0\uc774 \uc0c8\ub85c\uc6b4 \uac8c\uc784\ub2e8\uc5d0 \uc785\ub2e8\ud558\uac8c \ub418\uc5c8\ub2e4. 2010\ub144 11\uc6d4 4\uc77c \uc2a4\ud398\uc15c\ud3ec\uc2a4 \ud504\ub85c\uac8c\uc784\ub2e8\ub3c4 \ub4dc\ub798\ud504\ud2b8\ub97c \ub9c8\ucce4\ub2e4. \uc774\ubc88 \ub4dc\ub798\ud504\ud2b8\ub294 \uc9c0\ub09c \uc2dc\uc98c \ud300 \uc131\uc801\uc758 \uc5ed\uc21c\uc73c\ub85c \uc758\ubb34\uc9c0\uba85\uc73c\ub85c \uc2e4\uc2dc\ud588\ub2e4.", "answer": "2010\ub144 10\uc6d4 13\uc77c", "sentence": "\uadf8\ub7ec\ub098, 2010\ub144 10\uc6d4 13\uc77c \ud55c\uad6de\uc2a4\ud3ec\uce20\ud611\ud68c\ub294 \uc774\uc2a4\ud2b8\ub85c\uc758 \ud574\uccb4\ub97c \ucd5c\uc885 \uacb0\uc815\ud558\uc600\ub2e4.", "paragraph_sentence": "2009\ub144 3\uc6d4 29\uc77c \uc2a4\ud398\uc15c\ud3ec\uc2a4 \ud504\ub85c\uac8c\uc784\ub2e8 \ub4dc\ub798\ud504\ud2b8\uc5d0\uc11c e.sports-united\ub97c \uc9c0\uba85\ud558\uc600\ub2e4. 2009\ub144 3\uc6d4 KeSPA \ub7ad\ud0b9 1\uc704\uc774\uc790 \ucd5c\ub300\uc5b4\ub85c \ud3c9\uac00\ubc1b\uace0 \uc788\ub294 e.sports-united\ub294 \uc774\uc2a4\ud2b8\ub85c\uc758 \ud300 \ub2e8\uc704 \uc9c0\uba85\uc744 \ubc1b\uc558\ub2e4. \ucd94\ucca8\uc744 \ud1b5\ud574 1\uc21c\uc704 \uc9c0\uba85\uad8c\uc744 \uc5bb\uc740 \uc774\uc2a4\ud2b8\ub85c\uc758 \uae40\ud604\uc9c4 \uac10\ub3c5\uc740 \uc120\ubc1c\uc804 2\uc704\ub97c \ucc28\uc9c0\ud55c e.sports-united\ub97c \uc120\ubc1c\ud588\ub2e4. \uadf8\ub9ac\ud558\uc5ec, 2009\ub144 8\uc6d4 6\uc77c \uc2a4\ud398\uc15c\ud3ec\uc2a4 \uc774\uc2a4\ud2b8\ub85c\uac00 \uad11\uc548\ub9ac \ubb34\ub300\ub97c \uc815\ubcf5\ud588\ub2e4. \uc774\uc2a4\ud2b8\ub85c\uac00 KT \ud551\uac70\ubd90\uc744 \uaebe\uace0 \ud504\ub85c\ub9ac\uadf8 \uc815\uc0c1\uc5d0 \uc62c\ub790\ub2e4. \uc774\ub85c\uc368 \uc774\uc2a4\ud2b8\ub85c\ub294 \uc2a4\ud398\uc15c\ud3ec\uc2a4 \ud504\ub85c\ub9ac\uadf8 \uc6d0\ub144\ub9ac\uadf8\uc5d0\uc11c \ucd08\ub300 \ucc54\ud53c\uc5b8\uc5d0 \uc62c\ub790\ub2e4. \uc774\ub85c\uc368 \uc774\uc2a4\ud2b8\ub85c\uc758 \ucc98\uc74c\uc774\uc790 \ub9c8\uc9c0\ub9c9 \uc6b0\uc2b9\uc744 \ucc28\uc9c0\ud588\ub2e4. 2010\ub144 9\uc6d4 IEG\uac00 \uc774\uc2a4\ud2b8\ub85c\ub97c \ud55c\uad6de\uc2a4\ud3ec\uce20\ud611\ud68c\uc5d0 \ub9e4\uac01 \uc704\ud0c1\ud558\uba70 \uad6c\ub2e8\uc6b4\uc601\uc744 \ud3ec\uae30\ud558\uac8c \ub41c\ub2e4. \uc774\uc2a4\ud2b8\ub85c\uc758 \ub9e4\uac01 \uc704\ud0c1\uc5d0 \ub300\ud574 \ud55c\uad6de\uc2a4\ud3ec\uce20\ud611\ud68c \uad00\uacc4\uc790\ub294 \"\ud604\uc7ac \uc0c1\ud669\uc5d0\uc11c\ub294 \ucd5c\ub300\ud55c \uc6d0\ud65c\ud558\uac8c \ub9e4\uac01 \ud611\uc0c1\uc744 \uc9c4\ud589\ud574\uc11c \uac8c\uc784\ub2e8\uc744 \uc720\uc9c0\ud558\uace0 \uc120\uc218\ub4e4\uc758 \uace0\uc6a9 \uc2b9\uacc4\ub97c \uc704\ud574 \ucd5c\uc120\uc758 \ub178\ub825\uc744 \ub2e4\ud558\uace0 \uc788\ub2e4\" \uace0 \uc804\ud588\ub2e4. \uadf8\ub7ec\ub098, 2010\ub144 10\uc6d4 13\uc77c \ud55c\uad6de\uc2a4\ud3ec\uce20\ud611\ud68c\ub294 \uc774\uc2a4\ud2b8\ub85c\uc758 \ud574\uccb4\ub97c \ucd5c\uc885 \uacb0\uc815\ud558\uc600\ub2e4. \uae30\uc874\uc758 \uc120\uc218\ub4e4\uc740 \uc774\uc2a4\ud2b8\ub85c \uc120\uc218\ub2e8 \uacf5\uac1c \ub4dc\ub798\ud504\ud2b8\ub97c \ud1b5\ud574 \ub2e4\ub978 \ud300\uc5d0 \uac01\uac01 \uc601\uc785\ub418\uc5b4, \ub4dc\ub798\ud504\ud2b8\ub97c \uc644\ub8cc\ud558\uc600\ub2e4. \uc774\ubc88 \uacf5\uac1c \ub4dc\ub798\ud504\ud2b8\ub294 \uc785\ucc30 \uacfc\uc815\uc744 \ud1b5\ud574 \ud574\ub2f9 \uc120\uc218\ub97c \uc601\uc785\ud558\ub294 \ud3ec\uc2a4\ud305 \uc2dc\uc2a4\ud15c(Posting System)\uc744 \ud1b5\ud574 \u2018\ubc15\uc0c1\uc6b0, \uae40\uc131\ub300, \uc2e0\ub300\uadfc, \uc2e0\uc7ac\uc6b1, \uae40\ub3c4\uc6b0\u2019 \uac00 \uac8c\uc784\ub2e8\uc758 \uc9c0\uba85\uc744 \ubc1b\uc558\uc73c\uba70, \uc774\ud6c4 \uacf5\uad70\uc744 \uc81c\uc678\ud55c \ud3ec\uc2a4\ud305\uc5d0 \ucc38\uc5ec\ud558\uc9c0 \uc54a\uc740 \uac8c\uc784\ub2e8 \uc9c0\uba85\uc5d0\uc11c \u2018\uc720\ubcd1\uc900, \ubc31\ub3d9\uc900, \uc784\ud64d\uaddc, \ub178\uc7ac\uc0c1, \uc8fc\uc131\uc6b1, \uae40\uae30\ud6c8\u2019\uc774 \uc9c0\uba85\uc744 \ubc1b\uc544 \uc120\uc218 \uc804\uc6d0\uc774 \uc0c8\ub85c\uc6b4 \uac8c\uc784\ub2e8\uc5d0 \uc785\ub2e8\ud558\uac8c \ub418\uc5c8\ub2e4. 2010\ub144 11\uc6d4 4\uc77c \uc2a4\ud398\uc15c\ud3ec\uc2a4 \ud504\ub85c\uac8c\uc784\ub2e8\ub3c4 \ub4dc\ub798\ud504\ud2b8\ub97c \ub9c8\ucce4\ub2e4. \uc774\ubc88 \ub4dc\ub798\ud504\ud2b8\ub294 \uc9c0\ub09c \uc2dc\uc98c \ud300 \uc131\uc801\uc758 \uc5ed\uc21c\uc73c\ub85c \uc758\ubb34\uc9c0\uba85\uc73c\ub85c \uc2e4\uc2dc\ud588\ub2e4.", "paragraph_answer": "2009\ub144 3\uc6d4 29\uc77c \uc2a4\ud398\uc15c\ud3ec\uc2a4 \ud504\ub85c\uac8c\uc784\ub2e8 \ub4dc\ub798\ud504\ud2b8\uc5d0\uc11c e.sports-united\ub97c \uc9c0\uba85\ud558\uc600\ub2e4. 2009\ub144 3\uc6d4 KeSPA \ub7ad\ud0b9 1\uc704\uc774\uc790 \ucd5c\ub300\uc5b4\ub85c \ud3c9\uac00\ubc1b\uace0 \uc788\ub294 e.sports-united\ub294 \uc774\uc2a4\ud2b8\ub85c\uc758 \ud300 \ub2e8\uc704 \uc9c0\uba85\uc744 \ubc1b\uc558\ub2e4. \ucd94\ucca8\uc744 \ud1b5\ud574 1\uc21c\uc704 \uc9c0\uba85\uad8c\uc744 \uc5bb\uc740 \uc774\uc2a4\ud2b8\ub85c\uc758 \uae40\ud604\uc9c4 \uac10\ub3c5\uc740 \uc120\ubc1c\uc804 2\uc704\ub97c \ucc28\uc9c0\ud55c e.sports-united\ub97c \uc120\ubc1c\ud588\ub2e4. \uadf8\ub9ac\ud558\uc5ec, 2009\ub144 8\uc6d4 6\uc77c \uc2a4\ud398\uc15c\ud3ec\uc2a4 \uc774\uc2a4\ud2b8\ub85c\uac00 \uad11\uc548\ub9ac \ubb34\ub300\ub97c \uc815\ubcf5\ud588\ub2e4. \uc774\uc2a4\ud2b8\ub85c\uac00 KT \ud551\uac70\ubd90\uc744 \uaebe\uace0 \ud504\ub85c\ub9ac\uadf8 \uc815\uc0c1\uc5d0 \uc62c\ub790\ub2e4. \uc774\ub85c\uc368 \uc774\uc2a4\ud2b8\ub85c\ub294 \uc2a4\ud398\uc15c\ud3ec\uc2a4 \ud504\ub85c\ub9ac\uadf8 \uc6d0\ub144\ub9ac\uadf8\uc5d0\uc11c \ucd08\ub300 \ucc54\ud53c\uc5b8\uc5d0 \uc62c\ub790\ub2e4. \uc774\ub85c\uc368 \uc774\uc2a4\ud2b8\ub85c\uc758 \ucc98\uc74c\uc774\uc790 \ub9c8\uc9c0\ub9c9 \uc6b0\uc2b9\uc744 \ucc28\uc9c0\ud588\ub2e4. 2010\ub144 9\uc6d4 IEG\uac00 \uc774\uc2a4\ud2b8\ub85c\ub97c \ud55c\uad6de\uc2a4\ud3ec\uce20\ud611\ud68c\uc5d0 \ub9e4\uac01 \uc704\ud0c1\ud558\uba70 \uad6c\ub2e8\uc6b4\uc601\uc744 \ud3ec\uae30\ud558\uac8c \ub41c\ub2e4. \uc774\uc2a4\ud2b8\ub85c\uc758 \ub9e4\uac01 \uc704\ud0c1\uc5d0 \ub300\ud574 \ud55c\uad6de\uc2a4\ud3ec\uce20\ud611\ud68c \uad00\uacc4\uc790\ub294 \"\ud604\uc7ac \uc0c1\ud669\uc5d0\uc11c\ub294 \ucd5c\ub300\ud55c \uc6d0\ud65c\ud558\uac8c \ub9e4\uac01 \ud611\uc0c1\uc744 \uc9c4\ud589\ud574\uc11c \uac8c\uc784\ub2e8\uc744 \uc720\uc9c0\ud558\uace0 \uc120\uc218\ub4e4\uc758 \uace0\uc6a9 \uc2b9\uacc4\ub97c \uc704\ud574 \ucd5c\uc120\uc758 \ub178\ub825\uc744 \ub2e4\ud558\uace0 \uc788\ub2e4\" \uace0 \uc804\ud588\ub2e4. \uadf8\ub7ec\ub098, 2010\ub144 10\uc6d4 13\uc77c \ud55c\uad6de\uc2a4\ud3ec\uce20\ud611\ud68c\ub294 \uc774\uc2a4\ud2b8\ub85c\uc758 \ud574\uccb4\ub97c \ucd5c\uc885 \uacb0\uc815\ud558\uc600\ub2e4. \uae30\uc874\uc758 \uc120\uc218\ub4e4\uc740 \uc774\uc2a4\ud2b8\ub85c \uc120\uc218\ub2e8 \uacf5\uac1c \ub4dc\ub798\ud504\ud2b8\ub97c \ud1b5\ud574 \ub2e4\ub978 \ud300\uc5d0 \uac01\uac01 \uc601\uc785\ub418\uc5b4, \ub4dc\ub798\ud504\ud2b8\ub97c \uc644\ub8cc\ud558\uc600\ub2e4. \uc774\ubc88 \uacf5\uac1c \ub4dc\ub798\ud504\ud2b8\ub294 \uc785\ucc30 \uacfc\uc815\uc744 \ud1b5\ud574 \ud574\ub2f9 \uc120\uc218\ub97c \uc601\uc785\ud558\ub294 \ud3ec\uc2a4\ud305 \uc2dc\uc2a4\ud15c(Posting System)\uc744 \ud1b5\ud574 \u2018\ubc15\uc0c1\uc6b0, \uae40\uc131\ub300, \uc2e0\ub300\uadfc, \uc2e0\uc7ac\uc6b1, \uae40\ub3c4\uc6b0\u2019 \uac00 \uac8c\uc784\ub2e8\uc758 \uc9c0\uba85\uc744 \ubc1b\uc558\uc73c\uba70, \uc774\ud6c4 \uacf5\uad70\uc744 \uc81c\uc678\ud55c \ud3ec\uc2a4\ud305\uc5d0 \ucc38\uc5ec\ud558\uc9c0 \uc54a\uc740 \uac8c\uc784\ub2e8 \uc9c0\uba85\uc5d0\uc11c \u2018\uc720\ubcd1\uc900, \ubc31\ub3d9\uc900, \uc784\ud64d\uaddc, \ub178\uc7ac\uc0c1, \uc8fc\uc131\uc6b1, \uae40\uae30\ud6c8\u2019\uc774 \uc9c0\uba85\uc744 \ubc1b\uc544 \uc120\uc218 \uc804\uc6d0\uc774 \uc0c8\ub85c\uc6b4 \uac8c\uc784\ub2e8\uc5d0 \uc785\ub2e8\ud558\uac8c \ub418\uc5c8\ub2e4. 2010\ub144 11\uc6d4 4\uc77c \uc2a4\ud398\uc15c\ud3ec\uc2a4 \ud504\ub85c\uac8c\uc784\ub2e8\ub3c4 \ub4dc\ub798\ud504\ud2b8\ub97c \ub9c8\ucce4\ub2e4. \uc774\ubc88 \ub4dc\ub798\ud504\ud2b8\ub294 \uc9c0\ub09c \uc2dc\uc98c \ud300 \uc131\uc801\uc758 \uc5ed\uc21c\uc73c\ub85c \uc758\ubb34\uc9c0\uba85\uc73c\ub85c \uc2e4\uc2dc\ud588\ub2e4.", "sentence_answer": "\uadf8\ub7ec\ub098, 2010\ub144 10\uc6d4 13\uc77c \ud55c\uad6de\uc2a4\ud3ec\uce20\ud611\ud68c\ub294 \uc774\uc2a4\ud2b8\ub85c\uc758 \ud574\uccb4\ub97c \ucd5c\uc885 \uacb0\uc815\ud558\uc600\ub2e4.", "paragraph_id": "6545655-4-2", "paragraph_question": "question: \uc774\uc2a4\ud2b8\ub85c\uc758 \ud574\uccb4\uac00 \ucd5c\uc885 \uacb0\uc815\ub41c \uc2dc\uae30\ub294?, context: 2009\ub144 3\uc6d4 29\uc77c \uc2a4\ud398\uc15c\ud3ec\uc2a4 \ud504\ub85c\uac8c\uc784\ub2e8 \ub4dc\ub798\ud504\ud2b8\uc5d0\uc11c e.sports-united\ub97c \uc9c0\uba85\ud558\uc600\ub2e4. 2009\ub144 3\uc6d4 KeSPA \ub7ad\ud0b9 1\uc704\uc774\uc790 \ucd5c\ub300\uc5b4\ub85c \ud3c9\uac00\ubc1b\uace0 \uc788\ub294 e.sports-united\ub294 \uc774\uc2a4\ud2b8\ub85c\uc758 \ud300 \ub2e8\uc704 \uc9c0\uba85\uc744 \ubc1b\uc558\ub2e4. \ucd94\ucca8\uc744 \ud1b5\ud574 1\uc21c\uc704 \uc9c0\uba85\uad8c\uc744 \uc5bb\uc740 \uc774\uc2a4\ud2b8\ub85c\uc758 \uae40\ud604\uc9c4 \uac10\ub3c5\uc740 \uc120\ubc1c\uc804 2\uc704\ub97c \ucc28\uc9c0\ud55c e.sports-united\ub97c \uc120\ubc1c\ud588\ub2e4. \uadf8\ub9ac\ud558\uc5ec, 2009\ub144 8\uc6d4 6\uc77c \uc2a4\ud398\uc15c\ud3ec\uc2a4 \uc774\uc2a4\ud2b8\ub85c\uac00 \uad11\uc548\ub9ac \ubb34\ub300\ub97c \uc815\ubcf5\ud588\ub2e4. \uc774\uc2a4\ud2b8\ub85c\uac00 KT \ud551\uac70\ubd90\uc744 \uaebe\uace0 \ud504\ub85c\ub9ac\uadf8 \uc815\uc0c1\uc5d0 \uc62c\ub790\ub2e4. \uc774\ub85c\uc368 \uc774\uc2a4\ud2b8\ub85c\ub294 \uc2a4\ud398\uc15c\ud3ec\uc2a4 \ud504\ub85c\ub9ac\uadf8 \uc6d0\ub144\ub9ac\uadf8\uc5d0\uc11c \ucd08\ub300 \ucc54\ud53c\uc5b8\uc5d0 \uc62c\ub790\ub2e4. \uc774\ub85c\uc368 \uc774\uc2a4\ud2b8\ub85c\uc758 \ucc98\uc74c\uc774\uc790 \ub9c8\uc9c0\ub9c9 \uc6b0\uc2b9\uc744 \ucc28\uc9c0\ud588\ub2e4. 2010\ub144 9\uc6d4 IEG\uac00 \uc774\uc2a4\ud2b8\ub85c\ub97c \ud55c\uad6de\uc2a4\ud3ec\uce20\ud611\ud68c\uc5d0 \ub9e4\uac01 \uc704\ud0c1\ud558\uba70 \uad6c\ub2e8\uc6b4\uc601\uc744 \ud3ec\uae30\ud558\uac8c \ub41c\ub2e4. \uc774\uc2a4\ud2b8\ub85c\uc758 \ub9e4\uac01 \uc704\ud0c1\uc5d0 \ub300\ud574 \ud55c\uad6de\uc2a4\ud3ec\uce20\ud611\ud68c \uad00\uacc4\uc790\ub294 \"\ud604\uc7ac \uc0c1\ud669\uc5d0\uc11c\ub294 \ucd5c\ub300\ud55c \uc6d0\ud65c\ud558\uac8c \ub9e4\uac01 \ud611\uc0c1\uc744 \uc9c4\ud589\ud574\uc11c \uac8c\uc784\ub2e8\uc744 \uc720\uc9c0\ud558\uace0 \uc120\uc218\ub4e4\uc758 \uace0\uc6a9 \uc2b9\uacc4\ub97c \uc704\ud574 \ucd5c\uc120\uc758 \ub178\ub825\uc744 \ub2e4\ud558\uace0 \uc788\ub2e4\" \uace0 \uc804\ud588\ub2e4. \uadf8\ub7ec\ub098, 2010\ub144 10\uc6d4 13\uc77c \ud55c\uad6de\uc2a4\ud3ec\uce20\ud611\ud68c\ub294 \uc774\uc2a4\ud2b8\ub85c\uc758 \ud574\uccb4\ub97c \ucd5c\uc885 \uacb0\uc815\ud558\uc600\ub2e4. \uae30\uc874\uc758 \uc120\uc218\ub4e4\uc740 \uc774\uc2a4\ud2b8\ub85c \uc120\uc218\ub2e8 \uacf5\uac1c \ub4dc\ub798\ud504\ud2b8\ub97c \ud1b5\ud574 \ub2e4\ub978 \ud300\uc5d0 \uac01\uac01 \uc601\uc785\ub418\uc5b4, \ub4dc\ub798\ud504\ud2b8\ub97c \uc644\ub8cc\ud558\uc600\ub2e4. \uc774\ubc88 \uacf5\uac1c \ub4dc\ub798\ud504\ud2b8\ub294 \uc785\ucc30 \uacfc\uc815\uc744 \ud1b5\ud574 \ud574\ub2f9 \uc120\uc218\ub97c \uc601\uc785\ud558\ub294 \ud3ec\uc2a4\ud305 \uc2dc\uc2a4\ud15c(Posting System)\uc744 \ud1b5\ud574 \u2018\ubc15\uc0c1\uc6b0, \uae40\uc131\ub300, \uc2e0\ub300\uadfc, \uc2e0\uc7ac\uc6b1, \uae40\ub3c4\uc6b0\u2019 \uac00 \uac8c\uc784\ub2e8\uc758 \uc9c0\uba85\uc744 \ubc1b\uc558\uc73c\uba70, \uc774\ud6c4 \uacf5\uad70\uc744 \uc81c\uc678\ud55c \ud3ec\uc2a4\ud305\uc5d0 \ucc38\uc5ec\ud558\uc9c0 \uc54a\uc740 \uac8c\uc784\ub2e8 \uc9c0\uba85\uc5d0\uc11c \u2018\uc720\ubcd1\uc900, \ubc31\ub3d9\uc900, \uc784\ud64d\uaddc, \ub178\uc7ac\uc0c1, \uc8fc\uc131\uc6b1, \uae40\uae30\ud6c8\u2019\uc774 \uc9c0\uba85\uc744 \ubc1b\uc544 \uc120\uc218 \uc804\uc6d0\uc774 \uc0c8\ub85c\uc6b4 \uac8c\uc784\ub2e8\uc5d0 \uc785\ub2e8\ud558\uac8c \ub418\uc5c8\ub2e4. 2010\ub144 11\uc6d4 4\uc77c \uc2a4\ud398\uc15c\ud3ec\uc2a4 \ud504\ub85c\uac8c\uc784\ub2e8\ub3c4 \ub4dc\ub798\ud504\ud2b8\ub97c \ub9c8\ucce4\ub2e4. \uc774\ubc88 \ub4dc\ub798\ud504\ud2b8\ub294 \uc9c0\ub09c \uc2dc\uc98c \ud300 \uc131\uc801\uc758 \uc5ed\uc21c\uc73c\ub85c \uc758\ubb34\uc9c0\uba85\uc73c\ub85c \uc2e4\uc2dc\ud588\ub2e4."} {"question": "\uc218\ucc28\ub97c \uad6c\ubcc4\ud558\ub294 \uae30\uc900\uc740?", "paragraph": "\uc218\ucc28\ub294 17\uc138\uae30 \ub9d0\uacbd\uc5d0 \uc77c\ub2e8\uc740 \uc99d\uae30\ub3d9\ub825\uc5d0 \uc758\ud558\uc5ec \uadf8 \uc790\ub9ac\ub97c \uc783\uc740 \uac83 \uac19\uc558\uc73c\ub098, 19\uc138\uae30 \uc911\uc5fd\uc5d0 \uc774\ub974\ub7ec\uc11c \uc0c8\ub85c\uc6b4 \uc124\uacc4\uc640 \uae30\uc220\uc5d0 \uc758\ud558\uc5ec \ud130\ube48\uc774\ub77c\ub294 \uc774\ub984\uc73c\ub85c \ub2e4\uc2dc\uae08 \ub4f1\uc7a5\ud558\uac8c \ub418\uc5c8\ub2e4. \uadf8\ub798\uc11c \uc99d\uae30\ub3d9\ub825 \uc774\uc804 \ud615\uc2dd\uc758 \uc218\ucc28\ub294 \uad6c\uc2dd \uc218\ucc28, \uadf8 \uc774\ud6c4\uc5d0 \uac1c\ubc1c\ub41c \uc218\ucc28\ub294 \uc2e0\uc2dd \uc218\ucc28\ub77c\uace0 \ud3b8\uc758\uc0c1 \ub098\ub204\uae30\ub85c \ud55c\ub2e4. \uc218\ucc28\ub294 \uadf8 \uc2e0\uad6c\ud615\uc5d0 \uad00\uacc4\uc5c6\uc774 \ubb3c\uc758 \uc791\uc6a9\uc5d0 \ub530\ub77c \uc911\ub825\uc218\ucc28(\u91cd\u529b\u6c34\u8eca)\u00b7\ucda9\ub3d9\uc218\ucc28(\u885d\u52d5\u6c34\u8eca)\u00b7\ubc18\ub3d9\uc218\ucc28(\u53cd\u52d5\u6c34\u8eca)\uc758 3\uc885\uc73c\ub85c \uad6c\ubcc4\ud560 \uc218\uac00 \uc788\ub2e4. \uc911\ub825\uc218\ucc28\ub294 \uc8fc\ub85c \uc911\ub825\uacfc \uad00\uacc4\ub41c \ub099\ucc28\uc5d0 \uc758\ud574\uc11c \uc6c0\uc9c1\uc5ec\uc9c4\ub2e4. \ucda9\ub3d9\uc218\ucc28\ub294 \ubb3c \uc5d0\ub108\uc9c0\uc758 \uac70\uc758 \ubaa8\ub4e0 \uac83\uc744 \uc6b4\ub3d9\uc5d0\ub108\uc9c0\ub85c \ub9cc\ub4e4\uace0, \uadf8 \ucda9\uaca9\uc744 \uc774\uc6a9\ud558\uace0 \uc788\ub2e4. \ubc18\ub3d9\uc218\ucc28\ub294 \uc6a9\uae30\ub85c\ubd80\ud130 \ubb3c\uc744 \ubd84\ucd9c\uc2dc\ucf30\uc744 \ub54c\uc758 \ubc18\uc791\uc6a9\uc744, \uc5ed\ud559\uc801\uc73c\ub85c \ub9d0\ud558\uba74 \uc6b4\ub3d9\ub7c9 \ubcf4\uc874 \ubc95\uce59\uc744 \uc774\uc6a9\ud558\uace0 \uc788\ub2e4. \uc2e4\uc81c\uc801\uc778 \uc218\ucc28\ub294, \uc774 3\uc885\ub958\uc758 \ud6a8\uacfc\ub97c \uc54c\ub9de\uc740 \ube44\uc728\ub85c \uc120\ud0dd\ud55c \uad6c\uc870\ub85c \ub418\uc5b4 \uc788\ub2e4.", "answer": "\ubb3c\uc758 \uc791\uc6a9", "sentence": "\uc218\ucc28\ub294 \uadf8 \uc2e0\uad6c\ud615\uc5d0 \uad00\uacc4\uc5c6\uc774 \ubb3c\uc758 \uc791\uc6a9 \uc5d0", "paragraph_sentence": "\uc218\ucc28\ub294 17\uc138\uae30 \ub9d0\uacbd\uc5d0 \uc77c\ub2e8\uc740 \uc99d\uae30\ub3d9\ub825\uc5d0 \uc758\ud558\uc5ec \uadf8 \uc790\ub9ac\ub97c \uc783\uc740 \uac83 \uac19\uc558\uc73c\ub098, 19\uc138\uae30 \uc911\uc5fd\uc5d0 \uc774\ub974\ub7ec\uc11c \uc0c8\ub85c\uc6b4 \uc124\uacc4\uc640 \uae30\uc220\uc5d0 \uc758\ud558\uc5ec \ud130\ube48\uc774\ub77c\ub294 \uc774\ub984\uc73c\ub85c \ub2e4\uc2dc\uae08 \ub4f1\uc7a5\ud558\uac8c \ub418\uc5c8\ub2e4. \uadf8\ub798\uc11c \uc99d\uae30\ub3d9\ub825 \uc774\uc804 \ud615\uc2dd\uc758 \uc218\ucc28\ub294 \uad6c\uc2dd \uc218\ucc28, \uadf8 \uc774\ud6c4\uc5d0 \uac1c\ubc1c\ub41c \uc218\ucc28\ub294 \uc2e0\uc2dd \uc218\ucc28\ub77c\uace0 \ud3b8\uc758\uc0c1 \ub098\ub204\uae30\ub85c \ud55c\ub2e4. \uc218\ucc28\ub294 \uadf8 \uc2e0\uad6c\ud615\uc5d0 \uad00\uacc4\uc5c6\uc774 \ubb3c\uc758 \uc791\uc6a9 \uc5d0 \ub530\ub77c \uc911\ub825\uc218\ucc28(\u91cd\u529b\u6c34\u8eca)\u00b7\ucda9\ub3d9\uc218\ucc28(\u885d\u52d5\u6c34\u8eca)\u00b7\ubc18\ub3d9\uc218\ucc28(\u53cd\u52d5\u6c34\u8eca)\uc758 3\uc885\uc73c\ub85c \uad6c\ubcc4\ud560 \uc218\uac00 \uc788\ub2e4. \uc911\ub825\uc218\ucc28\ub294 \uc8fc\ub85c \uc911\ub825\uacfc \uad00\uacc4\ub41c \ub099\ucc28\uc5d0 \uc758\ud574\uc11c \uc6c0\uc9c1\uc5ec\uc9c4\ub2e4. \ucda9\ub3d9\uc218\ucc28\ub294 \ubb3c \uc5d0\ub108\uc9c0\uc758 \uac70\uc758 \ubaa8\ub4e0 \uac83\uc744 \uc6b4\ub3d9\uc5d0\ub108\uc9c0\ub85c \ub9cc\ub4e4\uace0, \uadf8 \ucda9\uaca9\uc744 \uc774\uc6a9\ud558\uace0 \uc788\ub2e4. \ubc18\ub3d9\uc218\ucc28\ub294 \uc6a9\uae30\ub85c\ubd80\ud130 \ubb3c\uc744 \ubd84\ucd9c\uc2dc\ucf30\uc744 \ub54c\uc758 \ubc18\uc791\uc6a9\uc744, \uc5ed\ud559\uc801\uc73c\ub85c \ub9d0\ud558\uba74 \uc6b4\ub3d9\ub7c9 \ubcf4\uc874 \ubc95\uce59\uc744 \uc774\uc6a9\ud558\uace0 \uc788\ub2e4. \uc2e4\uc81c\uc801\uc778 \uc218\ucc28\ub294, \uc774 3\uc885\ub958\uc758 \ud6a8\uacfc\ub97c \uc54c\ub9de\uc740 \ube44\uc728\ub85c \uc120\ud0dd\ud55c \uad6c\uc870\ub85c \ub418\uc5b4 \uc788\ub2e4.", "paragraph_answer": "\uc218\ucc28\ub294 17\uc138\uae30 \ub9d0\uacbd\uc5d0 \uc77c\ub2e8\uc740 \uc99d\uae30\ub3d9\ub825\uc5d0 \uc758\ud558\uc5ec \uadf8 \uc790\ub9ac\ub97c \uc783\uc740 \uac83 \uac19\uc558\uc73c\ub098, 19\uc138\uae30 \uc911\uc5fd\uc5d0 \uc774\ub974\ub7ec\uc11c \uc0c8\ub85c\uc6b4 \uc124\uacc4\uc640 \uae30\uc220\uc5d0 \uc758\ud558\uc5ec \ud130\ube48\uc774\ub77c\ub294 \uc774\ub984\uc73c\ub85c \ub2e4\uc2dc\uae08 \ub4f1\uc7a5\ud558\uac8c \ub418\uc5c8\ub2e4. \uadf8\ub798\uc11c \uc99d\uae30\ub3d9\ub825 \uc774\uc804 \ud615\uc2dd\uc758 \uc218\ucc28\ub294 \uad6c\uc2dd \uc218\ucc28, \uadf8 \uc774\ud6c4\uc5d0 \uac1c\ubc1c\ub41c \uc218\ucc28\ub294 \uc2e0\uc2dd \uc218\ucc28\ub77c\uace0 \ud3b8\uc758\uc0c1 \ub098\ub204\uae30\ub85c \ud55c\ub2e4. \uc218\ucc28\ub294 \uadf8 \uc2e0\uad6c\ud615\uc5d0 \uad00\uacc4\uc5c6\uc774 \ubb3c\uc758 \uc791\uc6a9 \uc5d0 \ub530\ub77c \uc911\ub825\uc218\ucc28(\u91cd\u529b\u6c34\u8eca)\u00b7\ucda9\ub3d9\uc218\ucc28(\u885d\u52d5\u6c34\u8eca)\u00b7\ubc18\ub3d9\uc218\ucc28(\u53cd\u52d5\u6c34\u8eca)\uc758 3\uc885\uc73c\ub85c \uad6c\ubcc4\ud560 \uc218\uac00 \uc788\ub2e4. \uc911\ub825\uc218\ucc28\ub294 \uc8fc\ub85c \uc911\ub825\uacfc \uad00\uacc4\ub41c \ub099\ucc28\uc5d0 \uc758\ud574\uc11c \uc6c0\uc9c1\uc5ec\uc9c4\ub2e4. \ucda9\ub3d9\uc218\ucc28\ub294 \ubb3c \uc5d0\ub108\uc9c0\uc758 \uac70\uc758 \ubaa8\ub4e0 \uac83\uc744 \uc6b4\ub3d9\uc5d0\ub108\uc9c0\ub85c \ub9cc\ub4e4\uace0, \uadf8 \ucda9\uaca9\uc744 \uc774\uc6a9\ud558\uace0 \uc788\ub2e4. \ubc18\ub3d9\uc218\ucc28\ub294 \uc6a9\uae30\ub85c\ubd80\ud130 \ubb3c\uc744 \ubd84\ucd9c\uc2dc\ucf30\uc744 \ub54c\uc758 \ubc18\uc791\uc6a9\uc744, \uc5ed\ud559\uc801\uc73c\ub85c \ub9d0\ud558\uba74 \uc6b4\ub3d9\ub7c9 \ubcf4\uc874 \ubc95\uce59\uc744 \uc774\uc6a9\ud558\uace0 \uc788\ub2e4. \uc2e4\uc81c\uc801\uc778 \uc218\ucc28\ub294, \uc774 3\uc885\ub958\uc758 \ud6a8\uacfc\ub97c \uc54c\ub9de\uc740 \ube44\uc728\ub85c \uc120\ud0dd\ud55c \uad6c\uc870\ub85c \ub418\uc5b4 \uc788\ub2e4.", "sentence_answer": "\uc218\ucc28\ub294 \uadf8 \uc2e0\uad6c\ud615\uc5d0 \uad00\uacc4\uc5c6\uc774 \ubb3c\uc758 \uc791\uc6a9 \uc5d0", "paragraph_id": "6518784-0-0", "paragraph_question": "question: \uc218\ucc28\ub97c \uad6c\ubcc4\ud558\ub294 \uae30\uc900\uc740?, context: \uc218\ucc28\ub294 17\uc138\uae30 \ub9d0\uacbd\uc5d0 \uc77c\ub2e8\uc740 \uc99d\uae30\ub3d9\ub825\uc5d0 \uc758\ud558\uc5ec \uadf8 \uc790\ub9ac\ub97c \uc783\uc740 \uac83 \uac19\uc558\uc73c\ub098, 19\uc138\uae30 \uc911\uc5fd\uc5d0 \uc774\ub974\ub7ec\uc11c \uc0c8\ub85c\uc6b4 \uc124\uacc4\uc640 \uae30\uc220\uc5d0 \uc758\ud558\uc5ec \ud130\ube48\uc774\ub77c\ub294 \uc774\ub984\uc73c\ub85c \ub2e4\uc2dc\uae08 \ub4f1\uc7a5\ud558\uac8c \ub418\uc5c8\ub2e4. \uadf8\ub798\uc11c \uc99d\uae30\ub3d9\ub825 \uc774\uc804 \ud615\uc2dd\uc758 \uc218\ucc28\ub294 \uad6c\uc2dd \uc218\ucc28, \uadf8 \uc774\ud6c4\uc5d0 \uac1c\ubc1c\ub41c \uc218\ucc28\ub294 \uc2e0\uc2dd \uc218\ucc28\ub77c\uace0 \ud3b8\uc758\uc0c1 \ub098\ub204\uae30\ub85c \ud55c\ub2e4. \uc218\ucc28\ub294 \uadf8 \uc2e0\uad6c\ud615\uc5d0 \uad00\uacc4\uc5c6\uc774 \ubb3c\uc758 \uc791\uc6a9\uc5d0 \ub530\ub77c \uc911\ub825\uc218\ucc28(\u91cd\u529b\u6c34\u8eca)\u00b7\ucda9\ub3d9\uc218\ucc28(\u885d\u52d5\u6c34\u8eca)\u00b7\ubc18\ub3d9\uc218\ucc28(\u53cd\u52d5\u6c34\u8eca)\uc758 3\uc885\uc73c\ub85c \uad6c\ubcc4\ud560 \uc218\uac00 \uc788\ub2e4. \uc911\ub825\uc218\ucc28\ub294 \uc8fc\ub85c \uc911\ub825\uacfc \uad00\uacc4\ub41c \ub099\ucc28\uc5d0 \uc758\ud574\uc11c \uc6c0\uc9c1\uc5ec\uc9c4\ub2e4. \ucda9\ub3d9\uc218\ucc28\ub294 \ubb3c \uc5d0\ub108\uc9c0\uc758 \uac70\uc758 \ubaa8\ub4e0 \uac83\uc744 \uc6b4\ub3d9\uc5d0\ub108\uc9c0\ub85c \ub9cc\ub4e4\uace0, \uadf8 \ucda9\uaca9\uc744 \uc774\uc6a9\ud558\uace0 \uc788\ub2e4. \ubc18\ub3d9\uc218\ucc28\ub294 \uc6a9\uae30\ub85c\ubd80\ud130 \ubb3c\uc744 \ubd84\ucd9c\uc2dc\ucf30\uc744 \ub54c\uc758 \ubc18\uc791\uc6a9\uc744, \uc5ed\ud559\uc801\uc73c\ub85c \ub9d0\ud558\uba74 \uc6b4\ub3d9\ub7c9 \ubcf4\uc874 \ubc95\uce59\uc744 \uc774\uc6a9\ud558\uace0 \uc788\ub2e4. \uc2e4\uc81c\uc801\uc778 \uc218\ucc28\ub294, \uc774 3\uc885\ub958\uc758 \ud6a8\uacfc\ub97c \uc54c\ub9de\uc740 \ube44\uc728\ub85c \uc120\ud0dd\ud55c \uad6c\uc870\ub85c \ub418\uc5b4 \uc788\ub2e4."} {"question": "\uc0ac\ub294 \uc124\uc77c\uccb4\uc720\ubd80\uc758 5\uc704 75\ubc95\uc5d0\uc11c \uc2ec\uc18c\ubc95 \uc911 \ubb34\uc2a8\ubc95 \uac00\uc6b4\ub370 \ud558\ub098\uc778\uac00?", "paragraph": "\uc0ac(\u6368, \ub0b4\ub824\ub193\uc74c, \ubc84\ub9bc, \uace0\uc694, \ud3c9\uc815, \ud3c9\uc815\uc2ec, \ud3c9\uc628, \uade0\ud615, \ud3c9\ud615, \uc0b0\uc2a4\ud06c\ub9ac\ud2b8\uc5b4: upeks\u0101, \ud314\ub9ac\uc5b4: upekkh\u0101 \ub610\ub294 upekh\u0101, \uc601\uc5b4: serenity, equilibrium, equanimity, stability, composure)\ub294 \uc124\uc77c\uccb4\uc720\ubd80\uc758 5\uc704 75\ubc95\uc5d0\uc11c \uc2ec\uc18c\ubc95(\u5fc3\u6240\u6cd5: 46\uac00\uc9c0) \uc911 \ub300\uc120\uc9c0\ubc95(\u5927\u5584\u5730\u6cd5: 10\uac00\uc9c0) \uac00\uc6b4\ub370 \ud558\ub098\uc774\uba70, \uc720\uc2dd\uc720\uac00\ud589\ud30c\uc640 \ubc95\uc0c1\uc885\uc758 5\uc704 100\ubc95\uc5d0\uc11c \uc2ec\uc18c\ubc95(\u5fc3\u6240\u6cd5: 51\uac00\uc9c0) \uc911 \uc120\uc2ec\uc18c(\u5584\u5fc3\u6240: 11\uac00\uc9c0) \uac00\uc6b4\ub370 \ud558\ub098\uc778 \ud589\uc0ac(\u884c\u6368)\uc774\ub2e4. \uc0ac\uc218(\u6368\u53d7: \uad34\ub86d\uc9c0\ub3c4 \uc990\uac81\uc9c0\ub3c4 \uc54a\uc740 \uac10\uac01)\uc640 \uad6c\ubcc4\ud558\uae30 \uc704\ud558\uc5ec \ud589\uc0ac(\u884c\u6368)\ub77c \ud55c\ub2e4. \uc124\uc77c\uccb4\uc720\ubd80\uc5d0 \ub530\ub974\uba74, \uc0ac(\u6368)\ub294 \ud63c\uce68(\u60db\u6c88: \ubb34\uae30\ub825, \uce68\uccb4)\uacfc \ub3c4\uac70(\u6389\u64e7: \ud750\ud2b8\ub7ec\uc9d0, \ub4e4\ub738, \ub534\ub370\ub85c \ub2ec\uc544\ub0a8)\ub97c \ub5a0\ub09c \ub9c8\uc74c(6\uc2dd \ub610\ub294 8\uc2dd, \uc989 \uc2ec\uc655, \uc989 \uc2ec\ubc95)\uc758 \ud3c9\ub4f1\uc131(\u5e73\u7b49\u6027: \ud3c9\uc815)\uacfc \ubb34\uacbd\uac01\uc131(\u7121\u8b66\u89ba\u6027: \ub3d9\uc694\ub428\uc774 \uc5c6\ub294 \uac83)\uc744 \ub9d0\ud55c\ub2e4.", "answer": "\ub300\uc120\uc9c0\ubc95", "sentence": "\uc0ac(\u6368, \ub0b4\ub824\ub193\uc74c, \ubc84\ub9bc, \uace0\uc694, \ud3c9\uc815, \ud3c9\uc815\uc2ec, \ud3c9\uc628, \uade0\ud615, \ud3c9\ud615, \uc0b0\uc2a4\ud06c\ub9ac\ud2b8\uc5b4: upeks\u0101, \ud314\ub9ac\uc5b4: upekkh\u0101 \ub610\ub294 upekh\u0101, \uc601\uc5b4: serenity, equilibrium, equanimity, stability, composure)\ub294 \uc124\uc77c\uccb4\uc720\ubd80\uc758 5\uc704 75\ubc95\uc5d0\uc11c \uc2ec\uc18c\ubc95(\u5fc3\u6240\u6cd5: 46\uac00\uc9c0) \uc911 \ub300\uc120\uc9c0\ubc95 (\u5927\u5584\u5730\u6cd5: 10\uac00\uc9c0) \uac00\uc6b4\ub370 \ud558\ub098\uc774\uba70, \uc720\uc2dd\uc720\uac00\ud589\ud30c\uc640 \ubc95\uc0c1\uc885\uc758 5\uc704 100\ubc95\uc5d0\uc11c \uc2ec\uc18c\ubc95(\u5fc3\u6240\u6cd5: 51\uac00\uc9c0) \uc911 \uc120\uc2ec\uc18c(\u5584\u5fc3\u6240: 11\uac00\uc9c0) \uac00\uc6b4\ub370 \ud558\ub098\uc778 \ud589\uc0ac(\u884c\u6368)\uc774\ub2e4.", "paragraph_sentence": " \uc0ac(\u6368, \ub0b4\ub824\ub193\uc74c, \ubc84\ub9bc, \uace0\uc694, \ud3c9\uc815, \ud3c9\uc815\uc2ec, \ud3c9\uc628, \uade0\ud615, \ud3c9\ud615, \uc0b0\uc2a4\ud06c\ub9ac\ud2b8\uc5b4: upeks\u0101, \ud314\ub9ac\uc5b4: upekkh\u0101 \ub610\ub294 upekh\u0101, \uc601\uc5b4: serenity, equilibrium, equanimity, stability, composure)\ub294 \uc124\uc77c\uccb4\uc720\ubd80\uc758 5\uc704 75\ubc95\uc5d0\uc11c \uc2ec\uc18c\ubc95(\u5fc3\u6240\u6cd5: 46\uac00\uc9c0) \uc911 \ub300\uc120\uc9c0\ubc95 (\u5927\u5584\u5730\u6cd5: 10\uac00\uc9c0) \uac00\uc6b4\ub370 \ud558\ub098\uc774\uba70, \uc720\uc2dd\uc720\uac00\ud589\ud30c\uc640 \ubc95\uc0c1\uc885\uc758 5\uc704 100\ubc95\uc5d0\uc11c \uc2ec\uc18c\ubc95(\u5fc3\u6240\u6cd5: 51\uac00\uc9c0) \uc911 \uc120\uc2ec\uc18c(\u5584\u5fc3\u6240: 11\uac00\uc9c0) \uac00\uc6b4\ub370 \ud558\ub098\uc778 \ud589\uc0ac(\u884c\u6368)\uc774\ub2e4. \uc0ac\uc218(\u6368\u53d7: \uad34\ub86d\uc9c0\ub3c4 \uc990\uac81\uc9c0\ub3c4 \uc54a\uc740 \uac10\uac01)\uc640 \uad6c\ubcc4\ud558\uae30 \uc704\ud558\uc5ec \ud589\uc0ac(\u884c\u6368)\ub77c \ud55c\ub2e4. \uc124\uc77c\uccb4\uc720\ubd80\uc5d0 \ub530\ub974\uba74, \uc0ac(\u6368)\ub294 \ud63c\uce68(\u60db\u6c88: \ubb34\uae30\ub825, \uce68\uccb4)\uacfc \ub3c4\uac70(\u6389\u64e7: \ud750\ud2b8\ub7ec\uc9d0, \ub4e4\ub738, \ub534\ub370\ub85c \ub2ec\uc544\ub0a8)\ub97c \ub5a0\ub09c \ub9c8\uc74c(6\uc2dd \ub610\ub294 8\uc2dd, \uc989 \uc2ec\uc655, \uc989 \uc2ec\ubc95)\uc758 \ud3c9\ub4f1\uc131(\u5e73\u7b49\u6027: \ud3c9\uc815)\uacfc \ubb34\uacbd\uac01\uc131(\u7121\u8b66\u89ba\u6027: \ub3d9\uc694\ub428\uc774 \uc5c6\ub294 \uac83)\uc744 \ub9d0\ud55c\ub2e4.", "paragraph_answer": "\uc0ac(\u6368, \ub0b4\ub824\ub193\uc74c, \ubc84\ub9bc, \uace0\uc694, \ud3c9\uc815, \ud3c9\uc815\uc2ec, \ud3c9\uc628, \uade0\ud615, \ud3c9\ud615, \uc0b0\uc2a4\ud06c\ub9ac\ud2b8\uc5b4: upeks\u0101, \ud314\ub9ac\uc5b4: upekkh\u0101 \ub610\ub294 upekh\u0101, \uc601\uc5b4: serenity, equilibrium, equanimity, stability, composure)\ub294 \uc124\uc77c\uccb4\uc720\ubd80\uc758 5\uc704 75\ubc95\uc5d0\uc11c \uc2ec\uc18c\ubc95(\u5fc3\u6240\u6cd5: 46\uac00\uc9c0) \uc911 \ub300\uc120\uc9c0\ubc95 (\u5927\u5584\u5730\u6cd5: 10\uac00\uc9c0) \uac00\uc6b4\ub370 \ud558\ub098\uc774\uba70, \uc720\uc2dd\uc720\uac00\ud589\ud30c\uc640 \ubc95\uc0c1\uc885\uc758 5\uc704 100\ubc95\uc5d0\uc11c \uc2ec\uc18c\ubc95(\u5fc3\u6240\u6cd5: 51\uac00\uc9c0) \uc911 \uc120\uc2ec\uc18c(\u5584\u5fc3\u6240: 11\uac00\uc9c0) \uac00\uc6b4\ub370 \ud558\ub098\uc778 \ud589\uc0ac(\u884c\u6368)\uc774\ub2e4. \uc0ac\uc218(\u6368\u53d7: \uad34\ub86d\uc9c0\ub3c4 \uc990\uac81\uc9c0\ub3c4 \uc54a\uc740 \uac10\uac01)\uc640 \uad6c\ubcc4\ud558\uae30 \uc704\ud558\uc5ec \ud589\uc0ac(\u884c\u6368)\ub77c \ud55c\ub2e4. \uc124\uc77c\uccb4\uc720\ubd80\uc5d0 \ub530\ub974\uba74, \uc0ac(\u6368)\ub294 \ud63c\uce68(\u60db\u6c88: \ubb34\uae30\ub825, \uce68\uccb4)\uacfc \ub3c4\uac70(\u6389\u64e7: \ud750\ud2b8\ub7ec\uc9d0, \ub4e4\ub738, \ub534\ub370\ub85c \ub2ec\uc544\ub0a8)\ub97c \ub5a0\ub09c \ub9c8\uc74c(6\uc2dd \ub610\ub294 8\uc2dd, \uc989 \uc2ec\uc655, \uc989 \uc2ec\ubc95)\uc758 \ud3c9\ub4f1\uc131(\u5e73\u7b49\u6027: \ud3c9\uc815)\uacfc \ubb34\uacbd\uac01\uc131(\u7121\u8b66\u89ba\u6027: \ub3d9\uc694\ub428\uc774 \uc5c6\ub294 \uac83)\uc744 \ub9d0\ud55c\ub2e4.", "sentence_answer": "\uc0ac(\u6368, \ub0b4\ub824\ub193\uc74c, \ubc84\ub9bc, \uace0\uc694, \ud3c9\uc815, \ud3c9\uc815\uc2ec, \ud3c9\uc628, \uade0\ud615, \ud3c9\ud615, \uc0b0\uc2a4\ud06c\ub9ac\ud2b8\uc5b4: upeks\u0101, \ud314\ub9ac\uc5b4: upekkh\u0101 \ub610\ub294 upekh\u0101, \uc601\uc5b4: serenity, equilibrium, equanimity, stability, composure)\ub294 \uc124\uc77c\uccb4\uc720\ubd80\uc758 5\uc704 75\ubc95\uc5d0\uc11c \uc2ec\uc18c\ubc95(\u5fc3\u6240\u6cd5: 46\uac00\uc9c0) \uc911 \ub300\uc120\uc9c0\ubc95 (\u5927\u5584\u5730\u6cd5: 10\uac00\uc9c0) \uac00\uc6b4\ub370 \ud558\ub098\uc774\uba70, \uc720\uc2dd\uc720\uac00\ud589\ud30c\uc640 \ubc95\uc0c1\uc885\uc758 5\uc704 100\ubc95\uc5d0\uc11c \uc2ec\uc18c\ubc95(\u5fc3\u6240\u6cd5: 51\uac00\uc9c0) \uc911 \uc120\uc2ec\uc18c(\u5584\u5fc3\u6240: 11\uac00\uc9c0) \uac00\uc6b4\ub370 \ud558\ub098\uc778 \ud589\uc0ac(\u884c\u6368)\uc774\ub2e4.", "paragraph_id": "6431675-0-0", "paragraph_question": "question: \uc0ac\ub294 \uc124\uc77c\uccb4\uc720\ubd80\uc758 5\uc704 75\ubc95\uc5d0\uc11c \uc2ec\uc18c\ubc95 \uc911 \ubb34\uc2a8\ubc95 \uac00\uc6b4\ub370 \ud558\ub098\uc778\uac00?, context: \uc0ac(\u6368, \ub0b4\ub824\ub193\uc74c, \ubc84\ub9bc, \uace0\uc694, \ud3c9\uc815, \ud3c9\uc815\uc2ec, \ud3c9\uc628, \uade0\ud615, \ud3c9\ud615, \uc0b0\uc2a4\ud06c\ub9ac\ud2b8\uc5b4: upeks\u0101, \ud314\ub9ac\uc5b4: upekkh\u0101 \ub610\ub294 upekh\u0101, \uc601\uc5b4: serenity, equilibrium, equanimity, stability, composure)\ub294 \uc124\uc77c\uccb4\uc720\ubd80\uc758 5\uc704 75\ubc95\uc5d0\uc11c \uc2ec\uc18c\ubc95(\u5fc3\u6240\u6cd5: 46\uac00\uc9c0) \uc911 \ub300\uc120\uc9c0\ubc95(\u5927\u5584\u5730\u6cd5: 10\uac00\uc9c0) \uac00\uc6b4\ub370 \ud558\ub098\uc774\uba70, \uc720\uc2dd\uc720\uac00\ud589\ud30c\uc640 \ubc95\uc0c1\uc885\uc758 5\uc704 100\ubc95\uc5d0\uc11c \uc2ec\uc18c\ubc95(\u5fc3\u6240\u6cd5: 51\uac00\uc9c0) \uc911 \uc120\uc2ec\uc18c(\u5584\u5fc3\u6240: 11\uac00\uc9c0) \uac00\uc6b4\ub370 \ud558\ub098\uc778 \ud589\uc0ac(\u884c\u6368)\uc774\ub2e4. \uc0ac\uc218(\u6368\u53d7: \uad34\ub86d\uc9c0\ub3c4 \uc990\uac81\uc9c0\ub3c4 \uc54a\uc740 \uac10\uac01)\uc640 \uad6c\ubcc4\ud558\uae30 \uc704\ud558\uc5ec \ud589\uc0ac(\u884c\u6368)\ub77c \ud55c\ub2e4. \uc124\uc77c\uccb4\uc720\ubd80\uc5d0 \ub530\ub974\uba74, \uc0ac(\u6368)\ub294 \ud63c\uce68(\u60db\u6c88: \ubb34\uae30\ub825, \uce68\uccb4)\uacfc \ub3c4\uac70(\u6389\u64e7: \ud750\ud2b8\ub7ec\uc9d0, \ub4e4\ub738, \ub534\ub370\ub85c \ub2ec\uc544\ub0a8)\ub97c \ub5a0\ub09c \ub9c8\uc74c(6\uc2dd \ub610\ub294 8\uc2dd, \uc989 \uc2ec\uc655, \uc989 \uc2ec\ubc95)\uc758 \ud3c9\ub4f1\uc131(\u5e73\u7b49\u6027: \ud3c9\uc815)\uacfc \ubb34\uacbd\uac01\uc131(\u7121\u8b66\u89ba\u6027: \ub3d9\uc694\ub428\uc774 \uc5c6\ub294 \uac83)\uc744 \ub9d0\ud55c\ub2e4."} {"question": "\uc625\ud0c0\ube44\uc544\ub204\uc2a4\ub294 \uc5b4\ub514\uc5d0 \uc8fc\ub454\ud588\ub358 \uce74\uc774\uc0ac\ub974\ub97c \uc22d\uc0c1\ud55c \ub178\ubcd1\uc744 \uc5bb\uc744 \uc218 \uc788\uc5c8\ub098?", "paragraph": "\uc625\ud0c0\ube44\uc544\ub204\uc2a4\uac00 \ub85c\ub9c8 \uc815\uad6d\uc5d0 \uc131\uacf5\uc801\uc73c\ub85c \uc785\ubb38\ud558\ub824\uba74 \uc790\uc2e0\uc758 \ube48\uc57d\ud55c \uc7ac\uc0b0\uc5d0\ub9cc \uc758\uc874\ud560 \uc218 \uc5c6\uc5c8\ub2e4. \ube0c\ub8ec\ub514\uc2dc\uc6c0\uc5d0\uc11c \uce74\uc774\uc0ac\ub974\uc758 \ubcd1\uc0ac\ub4e4\ub85c\ubd80\ud130 \uc5f4\uad11\uc801\uc778 \ud658\uc601\uc744 \ubc1b\uc740 \ud6c4, \uc625\ud0c0\ube44\uc544\ub204\uc2a4\ub294 \ud30c\ub974\ud2f0\uc544\uc640 \uc2f8\uc6b0\ub824\uace0 \uce74\uc774\uc0ac\ub974\uac00 \ucda9\ub2f9\ud588\ub358 \uc77c\ubd80 \uc790\uae08\uc744 \uc694\uad6c\ud558\uc600\ub2e4. \uc774 \ub3c8\uc740 \ube0c\ub8ec\ub514\uc2dc\uc6c0\uc5d0 \ubcf4\uad00\ub418\uc5b4 \uc788\uc5c8\ub294\ub370 \ubb34\ub824 7\uc5b5 \uc138\uc2a4\ud14c\ub974\ud2f0\uc6b0\uc2a4\uc5d0 \ud574\ub2f9\ud558\ub294 \uac70\uc561\uc774\uc5c8\ub2e4. \uc625\ud0c0\ube44\uc544\ub204\uc2a4\uac00 \uadf8 \ub3c8\uc73c\ub85c \uc6d0\ub85c\uc6d0\uc758 \uac00\uc7a5 \ud070 \uc801\uc778 \ub9c8\ub974\ucfe0\uc2a4 \uc548\ud1a0\ub2c8\uc6b0\uc2a4\ub97c \uce58\ub824\uace0 \uc4f0\uace0 \uc788\ub2e4\ub294 \uac83\uc774 \uc54c\ub824\uc9c0\uc790, \uc0ac\ub77c\uc9c4 \uacf5\uae08\uc758 \ud589\ubc29\uc744 \ucc3e\uace0 \uc788\uc5c8\ub358 \uc6d0\ub85c\uc6d0\uc740 \uc774\ub0b4 \uc218\uc0ac\ub97c \uc911\ub2e8\ud558\uc600\ub2e4. \uc625\ud0c0\ube44\uc544\ub204\uc2a4\ub294 \uadf8\ud574\uc5d0 \ub2e4\uc2dc \ud070\uc77c\uc744 \ubc8c\uc600\ub294\ub370, \ub3d9\ubc29 \uc18d\uc8fc\uc5d0\uc11c \uc774\ud0c8\ub9ac\uc544\ub85c \ubcf4\ub0b8 \uc870\uacf5\uc744 \uacf5\uc2dd\uc801\uc778 \ud5c8\uac00\ub3c4 \ubc1b\uc9c0 \uc54a\uace0 \uac00\ub85c\ucc58\ub2e4. \uc625\ud0c0\ube44\uc544\ub204\uc2a4\ub294 \uacfc\uac70 \uce74\uc774\uc0ac\ub974\uac00 \uc774\ub04c\ub358 \uc815\uc608\ubcd1\uacfc \ud30c\ub974\ud2f0\uc544\ub97c \uce58\ub824\uace0 \ubaa8\uc740 \uad70\uc0ac\ub4e4\uc744 \ud1b5\ud574\uc11c \uc790\uc2e0\uc758 \uc138\ub825\uc744 \ub113\ud614\uace0, \ud2b9\ud788 \uc790\uc2e0\uc774 \uce74\uc774\uc0ac\ub974\uc758 \ud6c4\uacc4\uc790\uc784\uc744 \uac15\uc870\ud558\uba70 \ub9ce\uc740 \uc9c0\uc9c0\ub97c \uc5bb\uc5b4\ub0c8\ub2e4. \uadf8\uac00 \uc774\ud0c8\ub9ac\uc544 \ubc18\ub3c4\ub97c \uac00\ub85c\uc9c8\ub7ec \ub85c\ub9c8\ub85c \uc774\ub3d9\ud560 \ub54c, \uc18c\ubb38\ub09c \uc778\ud488\uacfc \uadf8\uac00 \uc0c8\ub85c \uc5bb\uc740 \uc790\uae08 \ub355\ubd84\uc5d0 \uce84\ud30c\ub2c8\uc544\uc5d0 \uc8fc\ub454\ud558\uace0 \uc788\ub358 \uce74\uc774\uc0ac\ub974\ub97c \uc22d\uc0c1\ud55c \uc61b \ub178\ubcd1\ub4e4\uc758 \ub9c8\uc74c\uc744 \uc5bb\uc744 \uc218 \uc788\uc5c8\ub2e4. \uae30\uc6d0\uc804 44\ub144 6\uc6d4\uae4c\uc9c0 \uadf8\ub294 3\ucc9c \uba85\uc758 \ucda9\uc131\uc2a4\ub7ec\uc6b4 \ub178\ub828\ud55c \ubcd1\uc0ac\ub4e4\uc744 \uc5bb\uc5c8\uace0 \ubd09\uae09\uc73c\ub85c 5\ubc31 \ub370\ub098\ub9ac\uc6b0\uc2a4\ub97c \uc8fc\uc5c8\ub2e4.", "answer": "\uce84\ud30c\ub2c8\uc544", "sentence": "\uadf8\uac00 \uc0c8\ub85c \uc5bb\uc740 \uc790\uae08 \ub355\ubd84\uc5d0 \uce84\ud30c\ub2c8\uc544 \uc5d0 \uc8fc\ub454\ud558\uace0 \uc788\ub358 \uce74\uc774\uc0ac\ub974\ub97c \uc22d\uc0c1\ud55c \uc61b \ub178\ubcd1\ub4e4\uc758 \ub9c8\uc74c\uc744 \uc5bb\uc744 \uc218 \uc788\uc5c8\ub2e4.", "paragraph_sentence": "\uc625\ud0c0\ube44\uc544\ub204\uc2a4\uac00 \ub85c\ub9c8 \uc815\uad6d\uc5d0 \uc131\uacf5\uc801\uc73c\ub85c \uc785\ubb38\ud558\ub824\uba74 \uc790\uc2e0\uc758 \ube48\uc57d\ud55c \uc7ac\uc0b0\uc5d0\ub9cc \uc758\uc874\ud560 \uc218 \uc5c6\uc5c8\ub2e4. \ube0c\ub8ec\ub514\uc2dc\uc6c0\uc5d0\uc11c \uce74\uc774\uc0ac\ub974\uc758 \ubcd1\uc0ac\ub4e4\ub85c\ubd80\ud130 \uc5f4\uad11\uc801\uc778 \ud658\uc601\uc744 \ubc1b\uc740 \ud6c4, \uc625\ud0c0\ube44\uc544\ub204\uc2a4\ub294 \ud30c\ub974\ud2f0\uc544\uc640 \uc2f8\uc6b0\ub824\uace0 \uce74\uc774\uc0ac\ub974\uac00 \ucda9\ub2f9\ud588\ub358 \uc77c\ubd80 \uc790\uae08\uc744 \uc694\uad6c\ud558\uc600\ub2e4. \uc774 \ub3c8\uc740 \ube0c\ub8ec\ub514\uc2dc\uc6c0\uc5d0 \ubcf4\uad00\ub418\uc5b4 \uc788\uc5c8\ub294\ub370 \ubb34\ub824 7\uc5b5 \uc138\uc2a4\ud14c\ub974\ud2f0\uc6b0\uc2a4\uc5d0 \ud574\ub2f9\ud558\ub294 \uac70\uc561\uc774\uc5c8\ub2e4. \uc625\ud0c0\ube44\uc544\ub204\uc2a4\uac00 \uadf8 \ub3c8\uc73c\ub85c \uc6d0\ub85c\uc6d0\uc758 \uac00\uc7a5 \ud070 \uc801\uc778 \ub9c8\ub974\ucfe0\uc2a4 \uc548\ud1a0\ub2c8\uc6b0\uc2a4\ub97c \uce58\ub824\uace0 \uc4f0\uace0 \uc788\ub2e4\ub294 \uac83\uc774 \uc54c\ub824\uc9c0\uc790, \uc0ac\ub77c\uc9c4 \uacf5\uae08\uc758 \ud589\ubc29\uc744 \ucc3e\uace0 \uc788\uc5c8\ub358 \uc6d0\ub85c\uc6d0\uc740 \uc774\ub0b4 \uc218\uc0ac\ub97c \uc911\ub2e8\ud558\uc600\ub2e4. \uc625\ud0c0\ube44\uc544\ub204\uc2a4\ub294 \uadf8\ud574\uc5d0 \ub2e4\uc2dc \ud070\uc77c\uc744 \ubc8c\uc600\ub294\ub370, \ub3d9\ubc29 \uc18d\uc8fc\uc5d0\uc11c \uc774\ud0c8\ub9ac\uc544\ub85c \ubcf4\ub0b8 \uc870\uacf5\uc744 \uacf5\uc2dd\uc801\uc778 \ud5c8\uac00\ub3c4 \ubc1b\uc9c0 \uc54a\uace0 \uac00\ub85c\ucc58\ub2e4. \uc625\ud0c0\ube44\uc544\ub204\uc2a4\ub294 \uacfc\uac70 \uce74\uc774\uc0ac\ub974\uac00 \uc774\ub04c\ub358 \uc815\uc608\ubcd1\uacfc \ud30c\ub974\ud2f0\uc544\ub97c \uce58\ub824\uace0 \ubaa8\uc740 \uad70\uc0ac\ub4e4\uc744 \ud1b5\ud574\uc11c \uc790\uc2e0\uc758 \uc138\ub825\uc744 \ub113\ud614\uace0, \ud2b9\ud788 \uc790\uc2e0\uc774 \uce74\uc774\uc0ac\ub974\uc758 \ud6c4\uacc4\uc790\uc784\uc744 \uac15\uc870\ud558\uba70 \ub9ce\uc740 \uc9c0\uc9c0\ub97c \uc5bb\uc5b4\ub0c8\ub2e4. \uadf8\uac00 \uc774\ud0c8\ub9ac\uc544 \ubc18\ub3c4\ub97c \uac00\ub85c\uc9c8\ub7ec \ub85c\ub9c8\ub85c \uc774\ub3d9\ud560 \ub54c, \uc18c\ubb38\ub09c \uc778\ud488\uacfc \uadf8\uac00 \uc0c8\ub85c \uc5bb\uc740 \uc790\uae08 \ub355\ubd84\uc5d0 \uce84\ud30c\ub2c8\uc544 \uc5d0 \uc8fc\ub454\ud558\uace0 \uc788\ub358 \uce74\uc774\uc0ac\ub974\ub97c \uc22d\uc0c1\ud55c \uc61b \ub178\ubcd1\ub4e4\uc758 \ub9c8\uc74c\uc744 \uc5bb\uc744 \uc218 \uc788\uc5c8\ub2e4. \uae30\uc6d0\uc804 44\ub144 6\uc6d4\uae4c\uc9c0 \uadf8\ub294 3\ucc9c \uba85\uc758 \ucda9\uc131\uc2a4\ub7ec\uc6b4 \ub178\ub828\ud55c \ubcd1\uc0ac\ub4e4\uc744 \uc5bb\uc5c8\uace0 \ubd09\uae09\uc73c\ub85c 5\ubc31 \ub370\ub098\ub9ac\uc6b0\uc2a4\ub97c \uc8fc\uc5c8\ub2e4.", "paragraph_answer": "\uc625\ud0c0\ube44\uc544\ub204\uc2a4\uac00 \ub85c\ub9c8 \uc815\uad6d\uc5d0 \uc131\uacf5\uc801\uc73c\ub85c \uc785\ubb38\ud558\ub824\uba74 \uc790\uc2e0\uc758 \ube48\uc57d\ud55c \uc7ac\uc0b0\uc5d0\ub9cc \uc758\uc874\ud560 \uc218 \uc5c6\uc5c8\ub2e4. \ube0c\ub8ec\ub514\uc2dc\uc6c0\uc5d0\uc11c \uce74\uc774\uc0ac\ub974\uc758 \ubcd1\uc0ac\ub4e4\ub85c\ubd80\ud130 \uc5f4\uad11\uc801\uc778 \ud658\uc601\uc744 \ubc1b\uc740 \ud6c4, \uc625\ud0c0\ube44\uc544\ub204\uc2a4\ub294 \ud30c\ub974\ud2f0\uc544\uc640 \uc2f8\uc6b0\ub824\uace0 \uce74\uc774\uc0ac\ub974\uac00 \ucda9\ub2f9\ud588\ub358 \uc77c\ubd80 \uc790\uae08\uc744 \uc694\uad6c\ud558\uc600\ub2e4. \uc774 \ub3c8\uc740 \ube0c\ub8ec\ub514\uc2dc\uc6c0\uc5d0 \ubcf4\uad00\ub418\uc5b4 \uc788\uc5c8\ub294\ub370 \ubb34\ub824 7\uc5b5 \uc138\uc2a4\ud14c\ub974\ud2f0\uc6b0\uc2a4\uc5d0 \ud574\ub2f9\ud558\ub294 \uac70\uc561\uc774\uc5c8\ub2e4. \uc625\ud0c0\ube44\uc544\ub204\uc2a4\uac00 \uadf8 \ub3c8\uc73c\ub85c \uc6d0\ub85c\uc6d0\uc758 \uac00\uc7a5 \ud070 \uc801\uc778 \ub9c8\ub974\ucfe0\uc2a4 \uc548\ud1a0\ub2c8\uc6b0\uc2a4\ub97c \uce58\ub824\uace0 \uc4f0\uace0 \uc788\ub2e4\ub294 \uac83\uc774 \uc54c\ub824\uc9c0\uc790, \uc0ac\ub77c\uc9c4 \uacf5\uae08\uc758 \ud589\ubc29\uc744 \ucc3e\uace0 \uc788\uc5c8\ub358 \uc6d0\ub85c\uc6d0\uc740 \uc774\ub0b4 \uc218\uc0ac\ub97c \uc911\ub2e8\ud558\uc600\ub2e4. \uc625\ud0c0\ube44\uc544\ub204\uc2a4\ub294 \uadf8\ud574\uc5d0 \ub2e4\uc2dc \ud070\uc77c\uc744 \ubc8c\uc600\ub294\ub370, \ub3d9\ubc29 \uc18d\uc8fc\uc5d0\uc11c \uc774\ud0c8\ub9ac\uc544\ub85c \ubcf4\ub0b8 \uc870\uacf5\uc744 \uacf5\uc2dd\uc801\uc778 \ud5c8\uac00\ub3c4 \ubc1b\uc9c0 \uc54a\uace0 \uac00\ub85c\ucc58\ub2e4. \uc625\ud0c0\ube44\uc544\ub204\uc2a4\ub294 \uacfc\uac70 \uce74\uc774\uc0ac\ub974\uac00 \uc774\ub04c\ub358 \uc815\uc608\ubcd1\uacfc \ud30c\ub974\ud2f0\uc544\ub97c \uce58\ub824\uace0 \ubaa8\uc740 \uad70\uc0ac\ub4e4\uc744 \ud1b5\ud574\uc11c \uc790\uc2e0\uc758 \uc138\ub825\uc744 \ub113\ud614\uace0, \ud2b9\ud788 \uc790\uc2e0\uc774 \uce74\uc774\uc0ac\ub974\uc758 \ud6c4\uacc4\uc790\uc784\uc744 \uac15\uc870\ud558\uba70 \ub9ce\uc740 \uc9c0\uc9c0\ub97c \uc5bb\uc5b4\ub0c8\ub2e4. \uadf8\uac00 \uc774\ud0c8\ub9ac\uc544 \ubc18\ub3c4\ub97c \uac00\ub85c\uc9c8\ub7ec \ub85c\ub9c8\ub85c \uc774\ub3d9\ud560 \ub54c, \uc18c\ubb38\ub09c \uc778\ud488\uacfc \uadf8\uac00 \uc0c8\ub85c \uc5bb\uc740 \uc790\uae08 \ub355\ubd84\uc5d0 \uce84\ud30c\ub2c8\uc544 \uc5d0 \uc8fc\ub454\ud558\uace0 \uc788\ub358 \uce74\uc774\uc0ac\ub974\ub97c \uc22d\uc0c1\ud55c \uc61b \ub178\ubcd1\ub4e4\uc758 \ub9c8\uc74c\uc744 \uc5bb\uc744 \uc218 \uc788\uc5c8\ub2e4. \uae30\uc6d0\uc804 44\ub144 6\uc6d4\uae4c\uc9c0 \uadf8\ub294 3\ucc9c \uba85\uc758 \ucda9\uc131\uc2a4\ub7ec\uc6b4 \ub178\ub828\ud55c \ubcd1\uc0ac\ub4e4\uc744 \uc5bb\uc5c8\uace0 \ubd09\uae09\uc73c\ub85c 5\ubc31 \ub370\ub098\ub9ac\uc6b0\uc2a4\ub97c \uc8fc\uc5c8\ub2e4.", "sentence_answer": "\uadf8\uac00 \uc0c8\ub85c \uc5bb\uc740 \uc790\uae08 \ub355\ubd84\uc5d0 \uce84\ud30c\ub2c8\uc544 \uc5d0 \uc8fc\ub454\ud558\uace0 \uc788\ub358 \uce74\uc774\uc0ac\ub974\ub97c \uc22d\uc0c1\ud55c \uc61b \ub178\ubcd1\ub4e4\uc758 \ub9c8\uc74c\uc744 \uc5bb\uc744 \uc218 \uc788\uc5c8\ub2e4.", "paragraph_id": "6585254-5-2", "paragraph_question": "question: \uc625\ud0c0\ube44\uc544\ub204\uc2a4\ub294 \uc5b4\ub514\uc5d0 \uc8fc\ub454\ud588\ub358 \uce74\uc774\uc0ac\ub974\ub97c \uc22d\uc0c1\ud55c \ub178\ubcd1\uc744 \uc5bb\uc744 \uc218 \uc788\uc5c8\ub098?, context: \uc625\ud0c0\ube44\uc544\ub204\uc2a4\uac00 \ub85c\ub9c8 \uc815\uad6d\uc5d0 \uc131\uacf5\uc801\uc73c\ub85c \uc785\ubb38\ud558\ub824\uba74 \uc790\uc2e0\uc758 \ube48\uc57d\ud55c \uc7ac\uc0b0\uc5d0\ub9cc \uc758\uc874\ud560 \uc218 \uc5c6\uc5c8\ub2e4. \ube0c\ub8ec\ub514\uc2dc\uc6c0\uc5d0\uc11c \uce74\uc774\uc0ac\ub974\uc758 \ubcd1\uc0ac\ub4e4\ub85c\ubd80\ud130 \uc5f4\uad11\uc801\uc778 \ud658\uc601\uc744 \ubc1b\uc740 \ud6c4, \uc625\ud0c0\ube44\uc544\ub204\uc2a4\ub294 \ud30c\ub974\ud2f0\uc544\uc640 \uc2f8\uc6b0\ub824\uace0 \uce74\uc774\uc0ac\ub974\uac00 \ucda9\ub2f9\ud588\ub358 \uc77c\ubd80 \uc790\uae08\uc744 \uc694\uad6c\ud558\uc600\ub2e4. \uc774 \ub3c8\uc740 \ube0c\ub8ec\ub514\uc2dc\uc6c0\uc5d0 \ubcf4\uad00\ub418\uc5b4 \uc788\uc5c8\ub294\ub370 \ubb34\ub824 7\uc5b5 \uc138\uc2a4\ud14c\ub974\ud2f0\uc6b0\uc2a4\uc5d0 \ud574\ub2f9\ud558\ub294 \uac70\uc561\uc774\uc5c8\ub2e4. \uc625\ud0c0\ube44\uc544\ub204\uc2a4\uac00 \uadf8 \ub3c8\uc73c\ub85c \uc6d0\ub85c\uc6d0\uc758 \uac00\uc7a5 \ud070 \uc801\uc778 \ub9c8\ub974\ucfe0\uc2a4 \uc548\ud1a0\ub2c8\uc6b0\uc2a4\ub97c \uce58\ub824\uace0 \uc4f0\uace0 \uc788\ub2e4\ub294 \uac83\uc774 \uc54c\ub824\uc9c0\uc790, \uc0ac\ub77c\uc9c4 \uacf5\uae08\uc758 \ud589\ubc29\uc744 \ucc3e\uace0 \uc788\uc5c8\ub358 \uc6d0\ub85c\uc6d0\uc740 \uc774\ub0b4 \uc218\uc0ac\ub97c \uc911\ub2e8\ud558\uc600\ub2e4. \uc625\ud0c0\ube44\uc544\ub204\uc2a4\ub294 \uadf8\ud574\uc5d0 \ub2e4\uc2dc \ud070\uc77c\uc744 \ubc8c\uc600\ub294\ub370, \ub3d9\ubc29 \uc18d\uc8fc\uc5d0\uc11c \uc774\ud0c8\ub9ac\uc544\ub85c \ubcf4\ub0b8 \uc870\uacf5\uc744 \uacf5\uc2dd\uc801\uc778 \ud5c8\uac00\ub3c4 \ubc1b\uc9c0 \uc54a\uace0 \uac00\ub85c\ucc58\ub2e4. \uc625\ud0c0\ube44\uc544\ub204\uc2a4\ub294 \uacfc\uac70 \uce74\uc774\uc0ac\ub974\uac00 \uc774\ub04c\ub358 \uc815\uc608\ubcd1\uacfc \ud30c\ub974\ud2f0\uc544\ub97c \uce58\ub824\uace0 \ubaa8\uc740 \uad70\uc0ac\ub4e4\uc744 \ud1b5\ud574\uc11c \uc790\uc2e0\uc758 \uc138\ub825\uc744 \ub113\ud614\uace0, \ud2b9\ud788 \uc790\uc2e0\uc774 \uce74\uc774\uc0ac\ub974\uc758 \ud6c4\uacc4\uc790\uc784\uc744 \uac15\uc870\ud558\uba70 \ub9ce\uc740 \uc9c0\uc9c0\ub97c \uc5bb\uc5b4\ub0c8\ub2e4. \uadf8\uac00 \uc774\ud0c8\ub9ac\uc544 \ubc18\ub3c4\ub97c \uac00\ub85c\uc9c8\ub7ec \ub85c\ub9c8\ub85c \uc774\ub3d9\ud560 \ub54c, \uc18c\ubb38\ub09c \uc778\ud488\uacfc \uadf8\uac00 \uc0c8\ub85c \uc5bb\uc740 \uc790\uae08 \ub355\ubd84\uc5d0 \uce84\ud30c\ub2c8\uc544\uc5d0 \uc8fc\ub454\ud558\uace0 \uc788\ub358 \uce74\uc774\uc0ac\ub974\ub97c \uc22d\uc0c1\ud55c \uc61b \ub178\ubcd1\ub4e4\uc758 \ub9c8\uc74c\uc744 \uc5bb\uc744 \uc218 \uc788\uc5c8\ub2e4. \uae30\uc6d0\uc804 44\ub144 6\uc6d4\uae4c\uc9c0 \uadf8\ub294 3\ucc9c \uba85\uc758 \ucda9\uc131\uc2a4\ub7ec\uc6b4 \ub178\ub828\ud55c \ubcd1\uc0ac\ub4e4\uc744 \uc5bb\uc5c8\uace0 \ubd09\uae09\uc73c\ub85c 5\ubc31 \ub370\ub098\ub9ac\uc6b0\uc2a4\ub97c \uc8fc\uc5c8\ub2e4."} {"question": "\ucd08\ub300 \uc54c\ub809\uc0b0\ub4dc\ub9ac\uc544 \ucd1d\ub300\uc8fc\uad50\ub85c \uc5ec\uaca8\uc9c0\ub294 \uc778\ubb3c\uc740 \ub204\uad6c\uc778\uac00\uc694?", "paragraph": "\ub9c8\ub974\ucf54(\ud788\ube0c\ub9ac\uc5b4: \u05de\u05e8\u05e7\u05d5\u05e1, \uadf8\ub9ac\uc2a4\uc5b4: \u039c\u1fb6\u03c1\u03ba\u03bf\u03c2, \ub77c\ud2f4\uc5b4: M\u0101rcus, \uac1c\uc5ed\uc131\uacbd: \ub9c8\uac00)\ub294 \uae30\ub3c5\uad50 \ucd5c\ucd08\uc758 \ubcf5\uc74c\uc11c\ub85c \uc54c\ub824\uc9c4 '\ub9c8\ub974\ucf54\uc2a4\uc758 \uae30\uc05c \uc18c\uc2dd'\uc758 \uc800\uc790\uc774\uc790 \ucd08\ub300 \uc54c\ub809\uc0b0\ub4dc\ub9ac\uc544 \ucd1d\ub300\uc8fc\uad50\ub85c \uc5ec\uaca8\uc9c0\ub294 \uc778\ubb3c\ub85c, \ub85c\ub9c8 \uac00\ud1a8\ub9ad\uad50\ud68c\uc640 \ub3d9\ubc29 \uc815\uad50\ud68c\uc5d0\uc11c \uacf5\uacbd\ud558\ub294 \uc131\uc778\uc774\ub2e4. \uc9c0\uc815\ub41c \ucd95\uc77c\uc740 \uac01\uac01 4\uc6d4 25\uc77c\uacfc 5\uc6d4 2\uc77c\ub85c \ucc28\uc774\uac00 \uc788\ub2e4. \uc131\uacf5\ud68c\uc5d0\uc11c\ub3c4 \uc885\uad50\uac1c\ud601 \uc774\uc804\uc758 \uc2e0\uc559\uc804\ud1b5\uc744 \uc874\uc911\ud558\uc5ec \ub9c8\ub974\ucf54\uc2a4\ub97c \uc131\uc778\uc73c\ub85c \uae30\ub150\ud55c\ub2e4. \ub9c8\ub974\ucf54\uc2a4\ub77c\ub294 \uc774\ub984\uc740 \u2018\ube44\ucd94\uc774\ub2e4\u2019\ub77c\ub294 \ub73b\uc758 \ub77c\ud2f4\uc5b4 \ub9c8\ub974\ud2f0\uc6b0\uc2a4(Martius)\uc5d0\uc11c \ud30c\uc0dd\ub418\uc5c8\uace0 \uadf8\ub9ac\uc2a4 \uc2e0\ud654\uc758 \ub9c8\ub974\uc2a4\uc640\ub3c4 \uad00\uacc4\ub41c \uc774\ub984\uc774\ub2e4. \uacf5\uc99d\uc778\u00b7\uc548\uacbd\uc0ac\u00b7\uc720\ub9ac \uc81c\uc791\uc790\u00b7\ub3d9\ubb3c \uc0ac\uc721\uc790\u00b7\ubc88\uc5ed\uac00\u00b7\ubca0\ub124\uce58\uc544\uc758 \uc218\ud638 \uc131\uc778\uc73c\ub85c, \ud754\ud788 \uc0ac\uc790\ub97c \uc606\uc5d0 \ub450\uace0 \ubcf5\uc74c\uc11c\ub97c \uc800\uc220\ud558\ub294 \ubaa8\uc2b5\uc73c\ub85c \uadf8\ub824\uc9c4\ub2e4.", "answer": "\ub9c8\ub974\ucf54", "sentence": "\ub9c8\ub974\ucf54 (\ud788\ube0c\ub9ac\uc5b4: \u05de\u05e8\u05e7\u05d5\u05e1, \uadf8\ub9ac\uc2a4\uc5b4: \u039c\u1fb6\u03c1\u03ba\u03bf\u03c2, \ub77c\ud2f4\uc5b4: M\u0101rcus, \uac1c\uc5ed\uc131\uacbd: \ub9c8\uac00)\ub294 \uae30\ub3c5\uad50 \ucd5c\ucd08\uc758 \ubcf5\uc74c\uc11c\ub85c \uc54c\ub824\uc9c4 '\ub9c8\ub974\ucf54\uc2a4\uc758 \uae30\uc05c \uc18c\uc2dd'\uc758 \uc800\uc790\uc774\uc790 \ucd08\ub300 \uc54c\ub809\uc0b0\ub4dc\ub9ac\uc544 \ucd1d\ub300\uc8fc\uad50\ub85c \uc5ec\uaca8\uc9c0\ub294 \uc778\ubb3c\ub85c, \ub85c\ub9c8 \uac00\ud1a8\ub9ad\uad50\ud68c\uc640 \ub3d9\ubc29 \uc815\uad50\ud68c\uc5d0\uc11c \uacf5\uacbd\ud558\ub294 \uc131\uc778\uc774\ub2e4.", "paragraph_sentence": " \ub9c8\ub974\ucf54 (\ud788\ube0c\ub9ac\uc5b4: \u05de\u05e8\u05e7\u05d5\u05e1, \uadf8\ub9ac\uc2a4\uc5b4: \u039c\u1fb6\u03c1\u03ba\u03bf\u03c2, \ub77c\ud2f4\uc5b4: M\u0101rcus, \uac1c\uc5ed\uc131\uacbd: \ub9c8\uac00)\ub294 \uae30\ub3c5\uad50 \ucd5c\ucd08\uc758 \ubcf5\uc74c\uc11c\ub85c \uc54c\ub824\uc9c4 '\ub9c8\ub974\ucf54\uc2a4\uc758 \uae30\uc05c \uc18c\uc2dd'\uc758 \uc800\uc790\uc774\uc790 \ucd08\ub300 \uc54c\ub809\uc0b0\ub4dc\ub9ac\uc544 \ucd1d\ub300\uc8fc\uad50\ub85c \uc5ec\uaca8\uc9c0\ub294 \uc778\ubb3c\ub85c, \ub85c\ub9c8 \uac00\ud1a8\ub9ad\uad50\ud68c\uc640 \ub3d9\ubc29 \uc815\uad50\ud68c\uc5d0\uc11c \uacf5\uacbd\ud558\ub294 \uc131\uc778\uc774\ub2e4. \uc9c0\uc815\ub41c \ucd95\uc77c\uc740 \uac01\uac01 4\uc6d4 25\uc77c\uacfc 5\uc6d4 2\uc77c\ub85c \ucc28\uc774\uac00 \uc788\ub2e4. \uc131\uacf5\ud68c\uc5d0\uc11c\ub3c4 \uc885\uad50\uac1c\ud601 \uc774\uc804\uc758 \uc2e0\uc559\uc804\ud1b5\uc744 \uc874\uc911\ud558\uc5ec \ub9c8\ub974\ucf54\uc2a4\ub97c \uc131\uc778\uc73c\ub85c \uae30\ub150\ud55c\ub2e4. \ub9c8\ub974\ucf54\uc2a4\ub77c\ub294 \uc774\ub984\uc740 \u2018\ube44\ucd94\uc774\ub2e4\u2019\ub77c\ub294 \ub73b\uc758 \ub77c\ud2f4\uc5b4 \ub9c8\ub974\ud2f0\uc6b0\uc2a4(Martius)\uc5d0\uc11c \ud30c\uc0dd\ub418\uc5c8\uace0 \uadf8\ub9ac\uc2a4 \uc2e0\ud654\uc758 \ub9c8\ub974\uc2a4\uc640\ub3c4 \uad00\uacc4\ub41c \uc774\ub984\uc774\ub2e4. \uacf5\uc99d\uc778\u00b7\uc548\uacbd\uc0ac\u00b7\uc720\ub9ac \uc81c\uc791\uc790\u00b7\ub3d9\ubb3c \uc0ac\uc721\uc790\u00b7\ubc88\uc5ed\uac00\u00b7\ubca0\ub124\uce58\uc544\uc758 \uc218\ud638 \uc131\uc778\uc73c\ub85c, \ud754\ud788 \uc0ac\uc790\ub97c \uc606\uc5d0 \ub450\uace0 \ubcf5\uc74c\uc11c\ub97c \uc800\uc220\ud558\ub294 \ubaa8\uc2b5\uc73c\ub85c \uadf8\ub824\uc9c4\ub2e4.", "paragraph_answer": " \ub9c8\ub974\ucf54 (\ud788\ube0c\ub9ac\uc5b4: \u05de\u05e8\u05e7\u05d5\u05e1, \uadf8\ub9ac\uc2a4\uc5b4: \u039c\u1fb6\u03c1\u03ba\u03bf\u03c2, \ub77c\ud2f4\uc5b4: M\u0101rcus, \uac1c\uc5ed\uc131\uacbd: \ub9c8\uac00)\ub294 \uae30\ub3c5\uad50 \ucd5c\ucd08\uc758 \ubcf5\uc74c\uc11c\ub85c \uc54c\ub824\uc9c4 '\ub9c8\ub974\ucf54\uc2a4\uc758 \uae30\uc05c \uc18c\uc2dd'\uc758 \uc800\uc790\uc774\uc790 \ucd08\ub300 \uc54c\ub809\uc0b0\ub4dc\ub9ac\uc544 \ucd1d\ub300\uc8fc\uad50\ub85c \uc5ec\uaca8\uc9c0\ub294 \uc778\ubb3c\ub85c, \ub85c\ub9c8 \uac00\ud1a8\ub9ad\uad50\ud68c\uc640 \ub3d9\ubc29 \uc815\uad50\ud68c\uc5d0\uc11c \uacf5\uacbd\ud558\ub294 \uc131\uc778\uc774\ub2e4. \uc9c0\uc815\ub41c \ucd95\uc77c\uc740 \uac01\uac01 4\uc6d4 25\uc77c\uacfc 5\uc6d4 2\uc77c\ub85c \ucc28\uc774\uac00 \uc788\ub2e4. \uc131\uacf5\ud68c\uc5d0\uc11c\ub3c4 \uc885\uad50\uac1c\ud601 \uc774\uc804\uc758 \uc2e0\uc559\uc804\ud1b5\uc744 \uc874\uc911\ud558\uc5ec \ub9c8\ub974\ucf54\uc2a4\ub97c \uc131\uc778\uc73c\ub85c \uae30\ub150\ud55c\ub2e4. \ub9c8\ub974\ucf54\uc2a4\ub77c\ub294 \uc774\ub984\uc740 \u2018\ube44\ucd94\uc774\ub2e4\u2019\ub77c\ub294 \ub73b\uc758 \ub77c\ud2f4\uc5b4 \ub9c8\ub974\ud2f0\uc6b0\uc2a4(Martius)\uc5d0\uc11c \ud30c\uc0dd\ub418\uc5c8\uace0 \uadf8\ub9ac\uc2a4 \uc2e0\ud654\uc758 \ub9c8\ub974\uc2a4\uc640\ub3c4 \uad00\uacc4\ub41c \uc774\ub984\uc774\ub2e4. \uacf5\uc99d\uc778\u00b7\uc548\uacbd\uc0ac\u00b7\uc720\ub9ac \uc81c\uc791\uc790\u00b7\ub3d9\ubb3c \uc0ac\uc721\uc790\u00b7\ubc88\uc5ed\uac00\u00b7\ubca0\ub124\uce58\uc544\uc758 \uc218\ud638 \uc131\uc778\uc73c\ub85c, \ud754\ud788 \uc0ac\uc790\ub97c \uc606\uc5d0 \ub450\uace0 \ubcf5\uc74c\uc11c\ub97c \uc800\uc220\ud558\ub294 \ubaa8\uc2b5\uc73c\ub85c \uadf8\ub824\uc9c4\ub2e4.", "sentence_answer": " \ub9c8\ub974\ucf54 (\ud788\ube0c\ub9ac\uc5b4: \u05de\u05e8\u05e7\u05d5\u05e1, \uadf8\ub9ac\uc2a4\uc5b4: \u039c\u1fb6\u03c1\u03ba\u03bf\u03c2, \ub77c\ud2f4\uc5b4: M\u0101rcus, \uac1c\uc5ed\uc131\uacbd: \ub9c8\uac00)\ub294 \uae30\ub3c5\uad50 \ucd5c\ucd08\uc758 \ubcf5\uc74c\uc11c\ub85c \uc54c\ub824\uc9c4 '\ub9c8\ub974\ucf54\uc2a4\uc758 \uae30\uc05c \uc18c\uc2dd'\uc758 \uc800\uc790\uc774\uc790 \ucd08\ub300 \uc54c\ub809\uc0b0\ub4dc\ub9ac\uc544 \ucd1d\ub300\uc8fc\uad50\ub85c \uc5ec\uaca8\uc9c0\ub294 \uc778\ubb3c\ub85c, \ub85c\ub9c8 \uac00\ud1a8\ub9ad\uad50\ud68c\uc640 \ub3d9\ubc29 \uc815\uad50\ud68c\uc5d0\uc11c \uacf5\uacbd\ud558\ub294 \uc131\uc778\uc774\ub2e4.", "paragraph_id": "6508932-0-0", "paragraph_question": "question: \ucd08\ub300 \uc54c\ub809\uc0b0\ub4dc\ub9ac\uc544 \ucd1d\ub300\uc8fc\uad50\ub85c \uc5ec\uaca8\uc9c0\ub294 \uc778\ubb3c\uc740 \ub204\uad6c\uc778\uac00\uc694?, context: \ub9c8\ub974\ucf54(\ud788\ube0c\ub9ac\uc5b4: \u05de\u05e8\u05e7\u05d5\u05e1, \uadf8\ub9ac\uc2a4\uc5b4: \u039c\u1fb6\u03c1\u03ba\u03bf\u03c2, \ub77c\ud2f4\uc5b4: M\u0101rcus, \uac1c\uc5ed\uc131\uacbd: \ub9c8\uac00)\ub294 \uae30\ub3c5\uad50 \ucd5c\ucd08\uc758 \ubcf5\uc74c\uc11c\ub85c \uc54c\ub824\uc9c4 '\ub9c8\ub974\ucf54\uc2a4\uc758 \uae30\uc05c \uc18c\uc2dd'\uc758 \uc800\uc790\uc774\uc790 \ucd08\ub300 \uc54c\ub809\uc0b0\ub4dc\ub9ac\uc544 \ucd1d\ub300\uc8fc\uad50\ub85c \uc5ec\uaca8\uc9c0\ub294 \uc778\ubb3c\ub85c, \ub85c\ub9c8 \uac00\ud1a8\ub9ad\uad50\ud68c\uc640 \ub3d9\ubc29 \uc815\uad50\ud68c\uc5d0\uc11c \uacf5\uacbd\ud558\ub294 \uc131\uc778\uc774\ub2e4. \uc9c0\uc815\ub41c \ucd95\uc77c\uc740 \uac01\uac01 4\uc6d4 25\uc77c\uacfc 5\uc6d4 2\uc77c\ub85c \ucc28\uc774\uac00 \uc788\ub2e4. \uc131\uacf5\ud68c\uc5d0\uc11c\ub3c4 \uc885\uad50\uac1c\ud601 \uc774\uc804\uc758 \uc2e0\uc559\uc804\ud1b5\uc744 \uc874\uc911\ud558\uc5ec \ub9c8\ub974\ucf54\uc2a4\ub97c \uc131\uc778\uc73c\ub85c \uae30\ub150\ud55c\ub2e4. \ub9c8\ub974\ucf54\uc2a4\ub77c\ub294 \uc774\ub984\uc740 \u2018\ube44\ucd94\uc774\ub2e4\u2019\ub77c\ub294 \ub73b\uc758 \ub77c\ud2f4\uc5b4 \ub9c8\ub974\ud2f0\uc6b0\uc2a4(Martius)\uc5d0\uc11c \ud30c\uc0dd\ub418\uc5c8\uace0 \uadf8\ub9ac\uc2a4 \uc2e0\ud654\uc758 \ub9c8\ub974\uc2a4\uc640\ub3c4 \uad00\uacc4\ub41c \uc774\ub984\uc774\ub2e4. \uacf5\uc99d\uc778\u00b7\uc548\uacbd\uc0ac\u00b7\uc720\ub9ac \uc81c\uc791\uc790\u00b7\ub3d9\ubb3c \uc0ac\uc721\uc790\u00b7\ubc88\uc5ed\uac00\u00b7\ubca0\ub124\uce58\uc544\uc758 \uc218\ud638 \uc131\uc778\uc73c\ub85c, \ud754\ud788 \uc0ac\uc790\ub97c \uc606\uc5d0 \ub450\uace0 \ubcf5\uc74c\uc11c\ub97c \uc800\uc220\ud558\ub294 \ubaa8\uc2b5\uc73c\ub85c \uadf8\ub824\uc9c4\ub2e4."} {"question": "\uad50\ud1a0 \uc758\uc815\uc11c\uac00 \ubc1c\ud6a8\ub41c \ub144\ub3c4\ub294?", "paragraph": "\uc370\ubb3c \ub54c \ubb3c\uc744 \ube7c\ub294 \uc218\ubb38 8\uac1c\ub97c \uc9d3\ub294 \ud55c\ud3b8, \uadf8 \uc606\uc5d0\ub294 \uc9c0\ub984 7.5m, \ubb34\uac8c 800t\uc758 \uc218\ucc28 10\uae30\ub97c \uc9c0\ub2c8\ub294 15\uce35 \ub192\uc774\uc758 \ubc1c\uc804\uc18c\ub97c \uc9c0\uc5c8\ub2e4. \uc218\ucc28\uc758 \uc8fc\uc694\uc124\ube44\uc124\uacc4\uc640 \uc81c\uc791 \uc77c\ubd80\ub294 \uc624\uc2a4\ud2b8\ub9ac\uc544\uc758 \uc548\ub4dc\ub9ac\uce20 AG\uc0ac\uac00 \ub9e1\uc558\ub2e4. \ubaa8\ub4e0 \uc124\ube44\ub294 \uc678\ubd80\uc5d0\uc11c \uc81c\uc791\u00b7\uc870\ub9bd\ud558\uc5ec \ub300\ud615 \ud06c\ub808\uc778\uc744 \ud1b5\ud574 \uc6b4\ubc18\u00b7\uc124\uce58\ud558\uc600\ub2e4. \ubc1c\uc804 \uc124\ube44\uc758 \ud558\uc911\uc744 \uc9c0\uc9c0\ud558\ub294 \uc218\ucc28\ubc1c\uc804\uae30 \uc9c0\uc9c0\ub300 \uc548\uc5d0, \uc218\ucc28 \ub0a0\uac1c\uc758 \ud68c\uc804\ub825\uc744 \ud68c\uc804\uc790\ub85c \uc804\ub2ec\ud558\ub294 \uc218\ucc28\ucd95\uc744 \ub123\ub294\ub2e4. \uadf8\ub2e4\uc74c \uc218\ucc28\ucd95\uacfc \uc5f0\uacb0\ub3fc \ub3cc\uc544\uac00\ub294 \ud68c\uc804\uc790\ub97c \uc5f0\uacb0\ud558\uace0 \uc8fc\ubcc0\uc5d0 \uace0\uc815\uc790\ub97c \ub450\uc5b4, \ud68c\uc804 \uc2dc \ud68c\uc804\uc790\uc758 N\uadf9\u00b7S\uadf9\uc5d0\uc11c \uc790\ub825\uc774 \ubc1c\uc0dd\ud558\ub3c4\ub85d \ub9cc\ub4e0\ub2e4. \ubb3c\uc758 \uc591\uc744 \uc870\uc808\ud558\uc5ec \uc218\ucc28\uc758 \ud68c\uc804\ub7c9\uc744 \uc77c\uc815\ud558\uac8c \ud558\ub294 \uc720\ub7c9\uc870\uc808\uc7a5\uce58\ub97c \uc218\ucc28 \ub0a0\uac1c \ubc14\ub85c \uc55e\uc5d0 \uc124\uce58\ud55c\ub2e4. \uc218\ucc28\ub294 \uc9c0\ub984 14m\uc758 \uac00\ub85c\ucd95 \uce74\ud50c\ub780\uc218\ucc28\ub85c, \ubc14\ub78c\uac1c\ube44 \ubaa8\uc591 \ud68c\uc804\uccb4 \ub0a0\uac1c\uc758 \uae38\uc774\ub294 7.5m\uc774\ub2e4. \uae30\uc874\uc758 \uc218\ucc28\ubc1c\uc804\uae30\ub4e4\uc774 \uc77c\ubc18 \ud0c4\uc18c\uac15 \uc7ac\uc9c8\ub9cc\uc744 \uc0ac\uc6a9\ud574 \ubb3c\uc774 \ub2ff\uc73c\uba74 \ub179\uc2a4\ub294 \ucde8\uc57d\uc810\uc774 \uc788\uc5b4, \uc774\ub97c \ubc29\uc9c0\ud558\uace0\uc790 \uc218\ucc28\uc758 \ud574\uc218\uc811\ucd09 \ubd80\ubd84\uc740 \ub179\uc774 \uc2ac\uc9c0 \uc54a\ub294 \uc2a4\ud14c\uc778\ub9ac\uc2a4\uac15 \ucc98\ub9ac\ud558\uc600\uace0, \uc774\uc678\uc758 \ud0c4\uc18c\uac15 \ubd80\ubd84\uc740 \uc804\uc120\uc73c\ub85c \uc5f0\uacb0\ub41c \ub9c8\uadf8\ub124\uc298\uc774 \ub300\uc2e0 \ubd80\uc2dd\ub418\uac8c \ud558\ub294 \uc804\uae30 \ubc29\uc2dd\ubc95\uc744 \uc801\uc6a9\ud558\uc600\ub2e4. \uce5c\ud658\uacbd \uacf5\ubc95\ub3c4 \uc801\uc6a9\ud558\uc600\ub2e4. \ubc14\ub2e4 \uc0dd\ubb3c\uc758 \uc218\ucc28 \uc774\ub3d9 \uc2dc \ud53c\ud574 \ucd5c\uc18c\ud654\ub97c \uc704\ud574 3\uac1c\ub85c \ub0a0\uac1c\ub97c \ucd5c\uc18c\ud654\ud588\ub2e4. \uc0ac\uc6a9\ub41c \ub0c9\uac01\uc218\ub97c \uc7ac\ud65c\uc6a9\ud574 \ubc29\ub958\ub97c \uc904\uc774\ub294 \ud3d0\uc1c4\ud68c\ub85c \ubc29\uc2dd\uc744 \uc801\uc6a9\ud558\uc600\ub2e4. \uc0dd\ud654\ud559\uc801 \ubd84\ud574 \uac00\ub2a5 \uae30\ub984\uc744 \ubca0\uc5b4\ub9c1 \uc724\ud65c\uc720\uc640 \uc720\uc555 \uc791\ub3d9\uc720\uc5d0 \uc801\uc6a9\ud558\uc600\ub2e4. 2005\ub144 \ubc1c\ud6a8\ub41c \uad50\ud1a0 \uc758\uc815\uc11c\uc5d0 \ubd80\ud569\ud558\uae30 \uc704\ud574 2006\ub144\ubd80\ud130 \uc720\uc5d4\uae30\ud6c4\ubcc0\ud654\ud611\uc57d\uc5d0 \uccad\uc815\uac1c\ubc1c\uccb4\uc81c\ub85c \ub4f1\ub85d\ud558\uc5ec \uc2dc\uacf5\ud574 \ud0c4\uc18c\ubc30\ucd9c\uad8c\uc744 \ud68d\ub4dd\ud558\uac8c \ub418\uc5c8\ub2e4. \uc774\ub85c\uc368 \uc5bb\uac8c \ub41c \ubd80\uc218\uc785\uc740 \uc5f0 70~80\uc5b5 \uc6d0\uac00\ub7c9\uc774\ub2e4. 2011\ub144 8\uc6d4 3\uc77c \uc644\uacf5\ud558\uc5ec \uac00\ub3d9\uc744 \uc2dc\uc791\ud558\uc600\ub2e4.", "answer": "2005\ub144", "sentence": "2005\ub144 \ubc1c\ud6a8\ub41c \uad50\ud1a0 \uc758\uc815\uc11c\uc5d0 \ubd80\ud569\ud558\uae30 \uc704\ud574 2006\ub144\ubd80\ud130 \uc720\uc5d4\uae30\ud6c4\ubcc0\ud654\ud611\uc57d\uc5d0 \uccad\uc815\uac1c\ubc1c\uccb4\uc81c\ub85c \ub4f1\ub85d\ud558\uc5ec \uc2dc\uacf5\ud574 \ud0c4\uc18c\ubc30\ucd9c\uad8c\uc744 \ud68d\ub4dd\ud558\uac8c \ub418\uc5c8\ub2e4.", "paragraph_sentence": "\uc370\ubb3c \ub54c \ubb3c\uc744 \ube7c\ub294 \uc218\ubb38 8\uac1c\ub97c \uc9d3\ub294 \ud55c\ud3b8, \uadf8 \uc606\uc5d0\ub294 \uc9c0\ub984 7.5m, \ubb34\uac8c 800t\uc758 \uc218\ucc28 10\uae30\ub97c \uc9c0\ub2c8\ub294 15\uce35 \ub192\uc774\uc758 \ubc1c\uc804\uc18c\ub97c \uc9c0\uc5c8\ub2e4. \uc218\ucc28\uc758 \uc8fc\uc694\uc124\ube44\uc124\uacc4\uc640 \uc81c\uc791 \uc77c\ubd80\ub294 \uc624\uc2a4\ud2b8\ub9ac\uc544\uc758 \uc548\ub4dc\ub9ac\uce20 AG\uc0ac\uac00 \ub9e1\uc558\ub2e4. \ubaa8\ub4e0 \uc124\ube44\ub294 \uc678\ubd80\uc5d0\uc11c \uc81c\uc791\u00b7\uc870\ub9bd\ud558\uc5ec \ub300\ud615 \ud06c\ub808\uc778\uc744 \ud1b5\ud574 \uc6b4\ubc18\u00b7\uc124\uce58\ud558\uc600\ub2e4. \ubc1c\uc804 \uc124\ube44\uc758 \ud558\uc911\uc744 \uc9c0\uc9c0\ud558\ub294 \uc218\ucc28\ubc1c\uc804\uae30 \uc9c0\uc9c0\ub300 \uc548\uc5d0, \uc218\ucc28 \ub0a0\uac1c\uc758 \ud68c\uc804\ub825\uc744 \ud68c\uc804\uc790\ub85c \uc804\ub2ec\ud558\ub294 \uc218\ucc28\ucd95\uc744 \ub123\ub294\ub2e4. \uadf8\ub2e4\uc74c \uc218\ucc28\ucd95\uacfc \uc5f0\uacb0\ub3fc \ub3cc\uc544\uac00\ub294 \ud68c\uc804\uc790\ub97c \uc5f0\uacb0\ud558\uace0 \uc8fc\ubcc0\uc5d0 \uace0\uc815\uc790\ub97c \ub450\uc5b4, \ud68c\uc804 \uc2dc \ud68c\uc804\uc790\uc758 N\uadf9\u00b7S\uadf9\uc5d0\uc11c \uc790\ub825\uc774 \ubc1c\uc0dd\ud558\ub3c4\ub85d \ub9cc\ub4e0\ub2e4. \ubb3c\uc758 \uc591\uc744 \uc870\uc808\ud558\uc5ec \uc218\ucc28\uc758 \ud68c\uc804\ub7c9\uc744 \uc77c\uc815\ud558\uac8c \ud558\ub294 \uc720\ub7c9\uc870\uc808\uc7a5\uce58\ub97c \uc218\ucc28 \ub0a0\uac1c \ubc14\ub85c \uc55e\uc5d0 \uc124\uce58\ud55c\ub2e4. \uc218\ucc28\ub294 \uc9c0\ub984 14m\uc758 \uac00\ub85c\ucd95 \uce74\ud50c\ub780\uc218\ucc28\ub85c, \ubc14\ub78c\uac1c\ube44 \ubaa8\uc591 \ud68c\uc804\uccb4 \ub0a0\uac1c\uc758 \uae38\uc774\ub294 7.5m\uc774\ub2e4. \uae30\uc874\uc758 \uc218\ucc28\ubc1c\uc804\uae30\ub4e4\uc774 \uc77c\ubc18 \ud0c4\uc18c\uac15 \uc7ac\uc9c8\ub9cc\uc744 \uc0ac\uc6a9\ud574 \ubb3c\uc774 \ub2ff\uc73c\uba74 \ub179\uc2a4\ub294 \ucde8\uc57d\uc810\uc774 \uc788\uc5b4, \uc774\ub97c \ubc29\uc9c0\ud558\uace0\uc790 \uc218\ucc28\uc758 \ud574\uc218\uc811\ucd09 \ubd80\ubd84\uc740 \ub179\uc774 \uc2ac\uc9c0 \uc54a\ub294 \uc2a4\ud14c\uc778\ub9ac\uc2a4\uac15 \ucc98\ub9ac\ud558\uc600\uace0, \uc774\uc678\uc758 \ud0c4\uc18c\uac15 \ubd80\ubd84\uc740 \uc804\uc120\uc73c\ub85c \uc5f0\uacb0\ub41c \ub9c8\uadf8\ub124\uc298\uc774 \ub300\uc2e0 \ubd80\uc2dd\ub418\uac8c \ud558\ub294 \uc804\uae30 \ubc29\uc2dd\ubc95\uc744 \uc801\uc6a9\ud558\uc600\ub2e4. \uce5c\ud658\uacbd \uacf5\ubc95\ub3c4 \uc801\uc6a9\ud558\uc600\ub2e4. \ubc14\ub2e4 \uc0dd\ubb3c\uc758 \uc218\ucc28 \uc774\ub3d9 \uc2dc \ud53c\ud574 \ucd5c\uc18c\ud654\ub97c \uc704\ud574 3\uac1c\ub85c \ub0a0\uac1c\ub97c \ucd5c\uc18c\ud654\ud588\ub2e4. \uc0ac\uc6a9\ub41c \ub0c9\uac01\uc218\ub97c \uc7ac\ud65c\uc6a9\ud574 \ubc29\ub958\ub97c \uc904\uc774\ub294 \ud3d0\uc1c4\ud68c\ub85c \ubc29\uc2dd\uc744 \uc801\uc6a9\ud558\uc600\ub2e4. \uc0dd\ud654\ud559\uc801 \ubd84\ud574 \uac00\ub2a5 \uae30\ub984\uc744 \ubca0\uc5b4\ub9c1 \uc724\ud65c\uc720\uc640 \uc720\uc555 \uc791\ub3d9\uc720\uc5d0 \uc801\uc6a9\ud558\uc600\ub2e4. 2005\ub144 \ubc1c\ud6a8\ub41c \uad50\ud1a0 \uc758\uc815\uc11c\uc5d0 \ubd80\ud569\ud558\uae30 \uc704\ud574 2006\ub144\ubd80\ud130 \uc720\uc5d4\uae30\ud6c4\ubcc0\ud654\ud611\uc57d\uc5d0 \uccad\uc815\uac1c\ubc1c\uccb4\uc81c\ub85c \ub4f1\ub85d\ud558\uc5ec \uc2dc\uacf5\ud574 \ud0c4\uc18c\ubc30\ucd9c\uad8c\uc744 \ud68d\ub4dd\ud558\uac8c \ub418\uc5c8\ub2e4. \uc774\ub85c\uc368 \uc5bb\uac8c \ub41c \ubd80\uc218\uc785\uc740 \uc5f0 70~80\uc5b5 \uc6d0\uac00\ub7c9\uc774\ub2e4. 2011\ub144 8\uc6d4 3\uc77c \uc644\uacf5\ud558\uc5ec \uac00\ub3d9\uc744 \uc2dc\uc791\ud558\uc600\ub2e4.", "paragraph_answer": "\uc370\ubb3c \ub54c \ubb3c\uc744 \ube7c\ub294 \uc218\ubb38 8\uac1c\ub97c \uc9d3\ub294 \ud55c\ud3b8, \uadf8 \uc606\uc5d0\ub294 \uc9c0\ub984 7.5m, \ubb34\uac8c 800t\uc758 \uc218\ucc28 10\uae30\ub97c \uc9c0\ub2c8\ub294 15\uce35 \ub192\uc774\uc758 \ubc1c\uc804\uc18c\ub97c \uc9c0\uc5c8\ub2e4. \uc218\ucc28\uc758 \uc8fc\uc694\uc124\ube44\uc124\uacc4\uc640 \uc81c\uc791 \uc77c\ubd80\ub294 \uc624\uc2a4\ud2b8\ub9ac\uc544\uc758 \uc548\ub4dc\ub9ac\uce20 AG\uc0ac\uac00 \ub9e1\uc558\ub2e4. \ubaa8\ub4e0 \uc124\ube44\ub294 \uc678\ubd80\uc5d0\uc11c \uc81c\uc791\u00b7\uc870\ub9bd\ud558\uc5ec \ub300\ud615 \ud06c\ub808\uc778\uc744 \ud1b5\ud574 \uc6b4\ubc18\u00b7\uc124\uce58\ud558\uc600\ub2e4. \ubc1c\uc804 \uc124\ube44\uc758 \ud558\uc911\uc744 \uc9c0\uc9c0\ud558\ub294 \uc218\ucc28\ubc1c\uc804\uae30 \uc9c0\uc9c0\ub300 \uc548\uc5d0, \uc218\ucc28 \ub0a0\uac1c\uc758 \ud68c\uc804\ub825\uc744 \ud68c\uc804\uc790\ub85c \uc804\ub2ec\ud558\ub294 \uc218\ucc28\ucd95\uc744 \ub123\ub294\ub2e4. \uadf8\ub2e4\uc74c \uc218\ucc28\ucd95\uacfc \uc5f0\uacb0\ub3fc \ub3cc\uc544\uac00\ub294 \ud68c\uc804\uc790\ub97c \uc5f0\uacb0\ud558\uace0 \uc8fc\ubcc0\uc5d0 \uace0\uc815\uc790\ub97c \ub450\uc5b4, \ud68c\uc804 \uc2dc \ud68c\uc804\uc790\uc758 N\uadf9\u00b7S\uadf9\uc5d0\uc11c \uc790\ub825\uc774 \ubc1c\uc0dd\ud558\ub3c4\ub85d \ub9cc\ub4e0\ub2e4. \ubb3c\uc758 \uc591\uc744 \uc870\uc808\ud558\uc5ec \uc218\ucc28\uc758 \ud68c\uc804\ub7c9\uc744 \uc77c\uc815\ud558\uac8c \ud558\ub294 \uc720\ub7c9\uc870\uc808\uc7a5\uce58\ub97c \uc218\ucc28 \ub0a0\uac1c \ubc14\ub85c \uc55e\uc5d0 \uc124\uce58\ud55c\ub2e4. \uc218\ucc28\ub294 \uc9c0\ub984 14m\uc758 \uac00\ub85c\ucd95 \uce74\ud50c\ub780\uc218\ucc28\ub85c, \ubc14\ub78c\uac1c\ube44 \ubaa8\uc591 \ud68c\uc804\uccb4 \ub0a0\uac1c\uc758 \uae38\uc774\ub294 7.5m\uc774\ub2e4. \uae30\uc874\uc758 \uc218\ucc28\ubc1c\uc804\uae30\ub4e4\uc774 \uc77c\ubc18 \ud0c4\uc18c\uac15 \uc7ac\uc9c8\ub9cc\uc744 \uc0ac\uc6a9\ud574 \ubb3c\uc774 \ub2ff\uc73c\uba74 \ub179\uc2a4\ub294 \ucde8\uc57d\uc810\uc774 \uc788\uc5b4, \uc774\ub97c \ubc29\uc9c0\ud558\uace0\uc790 \uc218\ucc28\uc758 \ud574\uc218\uc811\ucd09 \ubd80\ubd84\uc740 \ub179\uc774 \uc2ac\uc9c0 \uc54a\ub294 \uc2a4\ud14c\uc778\ub9ac\uc2a4\uac15 \ucc98\ub9ac\ud558\uc600\uace0, \uc774\uc678\uc758 \ud0c4\uc18c\uac15 \ubd80\ubd84\uc740 \uc804\uc120\uc73c\ub85c \uc5f0\uacb0\ub41c \ub9c8\uadf8\ub124\uc298\uc774 \ub300\uc2e0 \ubd80\uc2dd\ub418\uac8c \ud558\ub294 \uc804\uae30 \ubc29\uc2dd\ubc95\uc744 \uc801\uc6a9\ud558\uc600\ub2e4. \uce5c\ud658\uacbd \uacf5\ubc95\ub3c4 \uc801\uc6a9\ud558\uc600\ub2e4. \ubc14\ub2e4 \uc0dd\ubb3c\uc758 \uc218\ucc28 \uc774\ub3d9 \uc2dc \ud53c\ud574 \ucd5c\uc18c\ud654\ub97c \uc704\ud574 3\uac1c\ub85c \ub0a0\uac1c\ub97c \ucd5c\uc18c\ud654\ud588\ub2e4. \uc0ac\uc6a9\ub41c \ub0c9\uac01\uc218\ub97c \uc7ac\ud65c\uc6a9\ud574 \ubc29\ub958\ub97c \uc904\uc774\ub294 \ud3d0\uc1c4\ud68c\ub85c \ubc29\uc2dd\uc744 \uc801\uc6a9\ud558\uc600\ub2e4. \uc0dd\ud654\ud559\uc801 \ubd84\ud574 \uac00\ub2a5 \uae30\ub984\uc744 \ubca0\uc5b4\ub9c1 \uc724\ud65c\uc720\uc640 \uc720\uc555 \uc791\ub3d9\uc720\uc5d0 \uc801\uc6a9\ud558\uc600\ub2e4. 2005\ub144 \ubc1c\ud6a8\ub41c \uad50\ud1a0 \uc758\uc815\uc11c\uc5d0 \ubd80\ud569\ud558\uae30 \uc704\ud574 2006\ub144\ubd80\ud130 \uc720\uc5d4\uae30\ud6c4\ubcc0\ud654\ud611\uc57d\uc5d0 \uccad\uc815\uac1c\ubc1c\uccb4\uc81c\ub85c \ub4f1\ub85d\ud558\uc5ec \uc2dc\uacf5\ud574 \ud0c4\uc18c\ubc30\ucd9c\uad8c\uc744 \ud68d\ub4dd\ud558\uac8c \ub418\uc5c8\ub2e4. \uc774\ub85c\uc368 \uc5bb\uac8c \ub41c \ubd80\uc218\uc785\uc740 \uc5f0 70~80\uc5b5 \uc6d0\uac00\ub7c9\uc774\ub2e4. 2011\ub144 8\uc6d4 3\uc77c \uc644\uacf5\ud558\uc5ec \uac00\ub3d9\uc744 \uc2dc\uc791\ud558\uc600\ub2e4.", "sentence_answer": " 2005\ub144 \ubc1c\ud6a8\ub41c \uad50\ud1a0 \uc758\uc815\uc11c\uc5d0 \ubd80\ud569\ud558\uae30 \uc704\ud574 2006\ub144\ubd80\ud130 \uc720\uc5d4\uae30\ud6c4\ubcc0\ud654\ud611\uc57d\uc5d0 \uccad\uc815\uac1c\ubc1c\uccb4\uc81c\ub85c \ub4f1\ub85d\ud558\uc5ec \uc2dc\uacf5\ud574 \ud0c4\uc18c\ubc30\ucd9c\uad8c\uc744 \ud68d\ub4dd\ud558\uac8c \ub418\uc5c8\ub2e4.", "paragraph_id": "6463858-4-1", "paragraph_question": "question: \uad50\ud1a0 \uc758\uc815\uc11c\uac00 \ubc1c\ud6a8\ub41c \ub144\ub3c4\ub294?, context: \uc370\ubb3c \ub54c \ubb3c\uc744 \ube7c\ub294 \uc218\ubb38 8\uac1c\ub97c \uc9d3\ub294 \ud55c\ud3b8, \uadf8 \uc606\uc5d0\ub294 \uc9c0\ub984 7.5m, \ubb34\uac8c 800t\uc758 \uc218\ucc28 10\uae30\ub97c \uc9c0\ub2c8\ub294 15\uce35 \ub192\uc774\uc758 \ubc1c\uc804\uc18c\ub97c \uc9c0\uc5c8\ub2e4. \uc218\ucc28\uc758 \uc8fc\uc694\uc124\ube44\uc124\uacc4\uc640 \uc81c\uc791 \uc77c\ubd80\ub294 \uc624\uc2a4\ud2b8\ub9ac\uc544\uc758 \uc548\ub4dc\ub9ac\uce20 AG\uc0ac\uac00 \ub9e1\uc558\ub2e4. \ubaa8\ub4e0 \uc124\ube44\ub294 \uc678\ubd80\uc5d0\uc11c \uc81c\uc791\u00b7\uc870\ub9bd\ud558\uc5ec \ub300\ud615 \ud06c\ub808\uc778\uc744 \ud1b5\ud574 \uc6b4\ubc18\u00b7\uc124\uce58\ud558\uc600\ub2e4. \ubc1c\uc804 \uc124\ube44\uc758 \ud558\uc911\uc744 \uc9c0\uc9c0\ud558\ub294 \uc218\ucc28\ubc1c\uc804\uae30 \uc9c0\uc9c0\ub300 \uc548\uc5d0, \uc218\ucc28 \ub0a0\uac1c\uc758 \ud68c\uc804\ub825\uc744 \ud68c\uc804\uc790\ub85c \uc804\ub2ec\ud558\ub294 \uc218\ucc28\ucd95\uc744 \ub123\ub294\ub2e4. \uadf8\ub2e4\uc74c \uc218\ucc28\ucd95\uacfc \uc5f0\uacb0\ub3fc \ub3cc\uc544\uac00\ub294 \ud68c\uc804\uc790\ub97c \uc5f0\uacb0\ud558\uace0 \uc8fc\ubcc0\uc5d0 \uace0\uc815\uc790\ub97c \ub450\uc5b4, \ud68c\uc804 \uc2dc \ud68c\uc804\uc790\uc758 N\uadf9\u00b7S\uadf9\uc5d0\uc11c \uc790\ub825\uc774 \ubc1c\uc0dd\ud558\ub3c4\ub85d \ub9cc\ub4e0\ub2e4. \ubb3c\uc758 \uc591\uc744 \uc870\uc808\ud558\uc5ec \uc218\ucc28\uc758 \ud68c\uc804\ub7c9\uc744 \uc77c\uc815\ud558\uac8c \ud558\ub294 \uc720\ub7c9\uc870\uc808\uc7a5\uce58\ub97c \uc218\ucc28 \ub0a0\uac1c \ubc14\ub85c \uc55e\uc5d0 \uc124\uce58\ud55c\ub2e4. \uc218\ucc28\ub294 \uc9c0\ub984 14m\uc758 \uac00\ub85c\ucd95 \uce74\ud50c\ub780\uc218\ucc28\ub85c, \ubc14\ub78c\uac1c\ube44 \ubaa8\uc591 \ud68c\uc804\uccb4 \ub0a0\uac1c\uc758 \uae38\uc774\ub294 7.5m\uc774\ub2e4. \uae30\uc874\uc758 \uc218\ucc28\ubc1c\uc804\uae30\ub4e4\uc774 \uc77c\ubc18 \ud0c4\uc18c\uac15 \uc7ac\uc9c8\ub9cc\uc744 \uc0ac\uc6a9\ud574 \ubb3c\uc774 \ub2ff\uc73c\uba74 \ub179\uc2a4\ub294 \ucde8\uc57d\uc810\uc774 \uc788\uc5b4, \uc774\ub97c \ubc29\uc9c0\ud558\uace0\uc790 \uc218\ucc28\uc758 \ud574\uc218\uc811\ucd09 \ubd80\ubd84\uc740 \ub179\uc774 \uc2ac\uc9c0 \uc54a\ub294 \uc2a4\ud14c\uc778\ub9ac\uc2a4\uac15 \ucc98\ub9ac\ud558\uc600\uace0, \uc774\uc678\uc758 \ud0c4\uc18c\uac15 \ubd80\ubd84\uc740 \uc804\uc120\uc73c\ub85c \uc5f0\uacb0\ub41c \ub9c8\uadf8\ub124\uc298\uc774 \ub300\uc2e0 \ubd80\uc2dd\ub418\uac8c \ud558\ub294 \uc804\uae30 \ubc29\uc2dd\ubc95\uc744 \uc801\uc6a9\ud558\uc600\ub2e4. \uce5c\ud658\uacbd \uacf5\ubc95\ub3c4 \uc801\uc6a9\ud558\uc600\ub2e4. \ubc14\ub2e4 \uc0dd\ubb3c\uc758 \uc218\ucc28 \uc774\ub3d9 \uc2dc \ud53c\ud574 \ucd5c\uc18c\ud654\ub97c \uc704\ud574 3\uac1c\ub85c \ub0a0\uac1c\ub97c \ucd5c\uc18c\ud654\ud588\ub2e4. \uc0ac\uc6a9\ub41c \ub0c9\uac01\uc218\ub97c \uc7ac\ud65c\uc6a9\ud574 \ubc29\ub958\ub97c \uc904\uc774\ub294 \ud3d0\uc1c4\ud68c\ub85c \ubc29\uc2dd\uc744 \uc801\uc6a9\ud558\uc600\ub2e4. \uc0dd\ud654\ud559\uc801 \ubd84\ud574 \uac00\ub2a5 \uae30\ub984\uc744 \ubca0\uc5b4\ub9c1 \uc724\ud65c\uc720\uc640 \uc720\uc555 \uc791\ub3d9\uc720\uc5d0 \uc801\uc6a9\ud558\uc600\ub2e4. 2005\ub144 \ubc1c\ud6a8\ub41c \uad50\ud1a0 \uc758\uc815\uc11c\uc5d0 \ubd80\ud569\ud558\uae30 \uc704\ud574 2006\ub144\ubd80\ud130 \uc720\uc5d4\uae30\ud6c4\ubcc0\ud654\ud611\uc57d\uc5d0 \uccad\uc815\uac1c\ubc1c\uccb4\uc81c\ub85c \ub4f1\ub85d\ud558\uc5ec \uc2dc\uacf5\ud574 \ud0c4\uc18c\ubc30\ucd9c\uad8c\uc744 \ud68d\ub4dd\ud558\uac8c \ub418\uc5c8\ub2e4. \uc774\ub85c\uc368 \uc5bb\uac8c \ub41c \ubd80\uc218\uc785\uc740 \uc5f0 70~80\uc5b5 \uc6d0\uac00\ub7c9\uc774\ub2e4. 2011\ub144 8\uc6d4 3\uc77c \uc644\uacf5\ud558\uc5ec \uac00\ub3d9\uc744 \uc2dc\uc791\ud558\uc600\ub2e4."} {"question": "2008\ub144 \ubca0\ucef4\uc774 \ubc00\ub780\uc5d0 \uc785\ub2e8\ud558\uba74\uc11c \uc0c8\ub85c \uc120\ud0dd\ud55c \ub4f1\ubc88\ud638\ub294 \uba87\ubc88\uc778\uac00?", "paragraph": "2008\ub144 12\uc6d4 20\uc77c, \uc544\ub4dc\ub9ac\uc544\ub178 \uac08\ub9ac\uc544\ub2c8 \uad6c\ub2e8\uc7a5\uc740 \ubc00\ub780\uc758 \ud6c8\ub828\uc7a5\uc5d0\uc11c \ubca0\ucef4\uc744 \uacf5\uac1c\ud588\ub2e4. \uc120\uc218\ub294 \uc804\uc5d0 \ud06c\ub9ac\uc2a4\ud2f0\uc548 \ube44\uc5d0\ub9ac\uac00 \ucc29\uc6a9\ud55c \ub4f1\ubc88\ud638 32\ubc88\uc744 \uc120\ud0dd\ud588\ub294\ub370, \ub2f9\uc2dc \ub4f1\ubc88\ud638 7\ubc88\uacfc 23\ubc88\uc774 \uc54c\ub808\uc0e8\ub4dc\ub9ac \ud30c\ud22c\uc640 \ubc00\ub780\uc758 \ubd80\uc8fc\uc7a5 \ub9c8\uc2dc\ubaa8 \uc554\ube0c\ub85c\uc2dc\ub2c8\uc758 \uac83\uc73c\ub85c \uac01\uac01 \ub4f1\ub85d\ub418\uc5c8\uae30 \ub54c\ubb38\uc774\uc5c8\ub2e4. \uc758\ub8cc \uac80\uc9c4 \ud6c4, \ubca0\ucef4\uc740 \uad6c\ub2e8 \uc758\ub8cc\uc9c4\uc73c\ub85c\ubd80\ud130 38\uc138\uac00 \ub418\ub294 5\ub144 \ud6c4\uae4c\uc9c0\ub294 \ub354 \ucd95\uad6c\ub97c \ud560 \uc218 \uc788\uc744 \uac83\uc774\ub77c\ub294 \ud3c9\uc744 \ub4e4\uc5c8\ub2e4. \uc785\ub2e8\uc2dd \uc774\ud2bf\ub0a0, \ubca0\ucef4\uc740 \uc0b0 \uc2dc\ub85c\ub85c \uac00 \uc6b0\ub514\ub124\uc138\uc640\uc758 \ub9ac\uadf8 \uacbd\uae30\ub97c \uc55e\ub450\uace0 \uacbd\uae30\uc7a5 \uc548\uc73c\ub85c \uac78\uc5b4 \ub4e4\uc5b4\uac00 \uc548\ubc29\uc758 \uc9c0\uc9c0\uc790\ub4e4\uc758 \ud658\uc601\uc744 \ubc1b\uc558\uace0, \ubc29\uc1a1 \uc124\ube44\ub97c \ud1b5\ud574 \"\uac00\uc790 \ubc00\ub780\"\uc744 \uc678\ucce4\ub2e4. \uadf8 \ud6c4, \uadf8\uc640 \ubc30\uc6b0\uc790 \ube45\ud1a0\ub9ac\uc544\ub294 \uc694\uc778 \uad00\uc911\uc11d\uc5d0\uc11c \ubc00\ub780\uc758 5-1 \ub300\uc2b9\uc744 \uad00\uc804\ud588\ub2e4.", "answer": "32\ubc88", "sentence": "\uc120\uc218\ub294 \uc804\uc5d0 \ud06c\ub9ac\uc2a4\ud2f0\uc548 \ube44\uc5d0\ub9ac\uac00 \ucc29\uc6a9\ud55c \ub4f1\ubc88\ud638 32\ubc88 \uc744 \uc120\ud0dd\ud588\ub294\ub370, \ub2f9\uc2dc \ub4f1\ubc88\ud638 7\ubc88\uacfc 23\ubc88\uc774 \uc54c\ub808\uc0e8\ub4dc\ub9ac \ud30c\ud22c\uc640 \ubc00\ub780\uc758 \ubd80\uc8fc\uc7a5 \ub9c8\uc2dc\ubaa8 \uc554\ube0c\ub85c\uc2dc\ub2c8\uc758 \uac83\uc73c\ub85c \uac01\uac01 \ub4f1\ub85d\ub418\uc5c8\uae30 \ub54c\ubb38\uc774\uc5c8\ub2e4.", "paragraph_sentence": "2008\ub144 12\uc6d4 20\uc77c, \uc544\ub4dc\ub9ac\uc544\ub178 \uac08\ub9ac\uc544\ub2c8 \uad6c\ub2e8\uc7a5\uc740 \ubc00\ub780\uc758 \ud6c8\ub828\uc7a5\uc5d0\uc11c \ubca0\ucef4\uc744 \uacf5\uac1c\ud588\ub2e4. \uc120\uc218\ub294 \uc804\uc5d0 \ud06c\ub9ac\uc2a4\ud2f0\uc548 \ube44\uc5d0\ub9ac\uac00 \ucc29\uc6a9\ud55c \ub4f1\ubc88\ud638 32\ubc88 \uc744 \uc120\ud0dd\ud588\ub294\ub370, \ub2f9\uc2dc \ub4f1\ubc88\ud638 7\ubc88\uacfc 23\ubc88\uc774 \uc54c\ub808\uc0e8\ub4dc\ub9ac \ud30c\ud22c\uc640 \ubc00\ub780\uc758 \ubd80\uc8fc\uc7a5 \ub9c8\uc2dc\ubaa8 \uc554\ube0c\ub85c\uc2dc\ub2c8\uc758 \uac83\uc73c\ub85c \uac01\uac01 \ub4f1\ub85d\ub418\uc5c8\uae30 \ub54c\ubb38\uc774\uc5c8\ub2e4. \uc758\ub8cc \uac80\uc9c4 \ud6c4, \ubca0\ucef4\uc740 \uad6c\ub2e8 \uc758\ub8cc\uc9c4\uc73c\ub85c\ubd80\ud130 38\uc138\uac00 \ub418\ub294 5\ub144 \ud6c4\uae4c\uc9c0\ub294 \ub354 \ucd95\uad6c\ub97c \ud560 \uc218 \uc788\uc744 \uac83\uc774\ub77c\ub294 \ud3c9\uc744 \ub4e4\uc5c8\ub2e4. \uc785\ub2e8\uc2dd \uc774\ud2bf\ub0a0, \ubca0\ucef4\uc740 \uc0b0 \uc2dc\ub85c\ub85c \uac00 \uc6b0\ub514\ub124\uc138\uc640\uc758 \ub9ac\uadf8 \uacbd\uae30\ub97c \uc55e\ub450\uace0 \uacbd\uae30\uc7a5 \uc548\uc73c\ub85c \uac78\uc5b4 \ub4e4\uc5b4\uac00 \uc548\ubc29\uc758 \uc9c0\uc9c0\uc790\ub4e4\uc758 \ud658\uc601\uc744 \ubc1b\uc558\uace0, \ubc29\uc1a1 \uc124\ube44\ub97c \ud1b5\ud574 \"\uac00\uc790 \ubc00\ub780\"\uc744 \uc678\ucce4\ub2e4. \uadf8 \ud6c4, \uadf8\uc640 \ubc30\uc6b0\uc790 \ube45\ud1a0\ub9ac\uc544\ub294 \uc694\uc778 \uad00\uc911\uc11d\uc5d0\uc11c \ubc00\ub780\uc758 5-1 \ub300\uc2b9\uc744 \uad00\uc804\ud588\ub2e4.", "paragraph_answer": "2008\ub144 12\uc6d4 20\uc77c, \uc544\ub4dc\ub9ac\uc544\ub178 \uac08\ub9ac\uc544\ub2c8 \uad6c\ub2e8\uc7a5\uc740 \ubc00\ub780\uc758 \ud6c8\ub828\uc7a5\uc5d0\uc11c \ubca0\ucef4\uc744 \uacf5\uac1c\ud588\ub2e4. \uc120\uc218\ub294 \uc804\uc5d0 \ud06c\ub9ac\uc2a4\ud2f0\uc548 \ube44\uc5d0\ub9ac\uac00 \ucc29\uc6a9\ud55c \ub4f1\ubc88\ud638 32\ubc88 \uc744 \uc120\ud0dd\ud588\ub294\ub370, \ub2f9\uc2dc \ub4f1\ubc88\ud638 7\ubc88\uacfc 23\ubc88\uc774 \uc54c\ub808\uc0e8\ub4dc\ub9ac \ud30c\ud22c\uc640 \ubc00\ub780\uc758 \ubd80\uc8fc\uc7a5 \ub9c8\uc2dc\ubaa8 \uc554\ube0c\ub85c\uc2dc\ub2c8\uc758 \uac83\uc73c\ub85c \uac01\uac01 \ub4f1\ub85d\ub418\uc5c8\uae30 \ub54c\ubb38\uc774\uc5c8\ub2e4. \uc758\ub8cc \uac80\uc9c4 \ud6c4, \ubca0\ucef4\uc740 \uad6c\ub2e8 \uc758\ub8cc\uc9c4\uc73c\ub85c\ubd80\ud130 38\uc138\uac00 \ub418\ub294 5\ub144 \ud6c4\uae4c\uc9c0\ub294 \ub354 \ucd95\uad6c\ub97c \ud560 \uc218 \uc788\uc744 \uac83\uc774\ub77c\ub294 \ud3c9\uc744 \ub4e4\uc5c8\ub2e4. \uc785\ub2e8\uc2dd \uc774\ud2bf\ub0a0, \ubca0\ucef4\uc740 \uc0b0 \uc2dc\ub85c\ub85c \uac00 \uc6b0\ub514\ub124\uc138\uc640\uc758 \ub9ac\uadf8 \uacbd\uae30\ub97c \uc55e\ub450\uace0 \uacbd\uae30\uc7a5 \uc548\uc73c\ub85c \uac78\uc5b4 \ub4e4\uc5b4\uac00 \uc548\ubc29\uc758 \uc9c0\uc9c0\uc790\ub4e4\uc758 \ud658\uc601\uc744 \ubc1b\uc558\uace0, \ubc29\uc1a1 \uc124\ube44\ub97c \ud1b5\ud574 \"\uac00\uc790 \ubc00\ub780\"\uc744 \uc678\ucce4\ub2e4. \uadf8 \ud6c4, \uadf8\uc640 \ubc30\uc6b0\uc790 \ube45\ud1a0\ub9ac\uc544\ub294 \uc694\uc778 \uad00\uc911\uc11d\uc5d0\uc11c \ubc00\ub780\uc758 5-1 \ub300\uc2b9\uc744 \uad00\uc804\ud588\ub2e4.", "sentence_answer": "\uc120\uc218\ub294 \uc804\uc5d0 \ud06c\ub9ac\uc2a4\ud2f0\uc548 \ube44\uc5d0\ub9ac\uac00 \ucc29\uc6a9\ud55c \ub4f1\ubc88\ud638 32\ubc88 \uc744 \uc120\ud0dd\ud588\ub294\ub370, \ub2f9\uc2dc \ub4f1\ubc88\ud638 7\ubc88\uacfc 23\ubc88\uc774 \uc54c\ub808\uc0e8\ub4dc\ub9ac \ud30c\ud22c\uc640 \ubc00\ub780\uc758 \ubd80\uc8fc\uc7a5 \ub9c8\uc2dc\ubaa8 \uc554\ube0c\ub85c\uc2dc\ub2c8\uc758 \uac83\uc73c\ub85c \uac01\uac01 \ub4f1\ub85d\ub418\uc5c8\uae30 \ub54c\ubb38\uc774\uc5c8\ub2e4.", "paragraph_id": "6602120-24-0", "paragraph_question": "question: 2008\ub144 \ubca0\ucef4\uc774 \ubc00\ub780\uc5d0 \uc785\ub2e8\ud558\uba74\uc11c \uc0c8\ub85c \uc120\ud0dd\ud55c \ub4f1\ubc88\ud638\ub294 \uba87\ubc88\uc778\uac00?, context: 2008\ub144 12\uc6d4 20\uc77c, \uc544\ub4dc\ub9ac\uc544\ub178 \uac08\ub9ac\uc544\ub2c8 \uad6c\ub2e8\uc7a5\uc740 \ubc00\ub780\uc758 \ud6c8\ub828\uc7a5\uc5d0\uc11c \ubca0\ucef4\uc744 \uacf5\uac1c\ud588\ub2e4. \uc120\uc218\ub294 \uc804\uc5d0 \ud06c\ub9ac\uc2a4\ud2f0\uc548 \ube44\uc5d0\ub9ac\uac00 \ucc29\uc6a9\ud55c \ub4f1\ubc88\ud638 32\ubc88\uc744 \uc120\ud0dd\ud588\ub294\ub370, \ub2f9\uc2dc \ub4f1\ubc88\ud638 7\ubc88\uacfc 23\ubc88\uc774 \uc54c\ub808\uc0e8\ub4dc\ub9ac \ud30c\ud22c\uc640 \ubc00\ub780\uc758 \ubd80\uc8fc\uc7a5 \ub9c8\uc2dc\ubaa8 \uc554\ube0c\ub85c\uc2dc\ub2c8\uc758 \uac83\uc73c\ub85c \uac01\uac01 \ub4f1\ub85d\ub418\uc5c8\uae30 \ub54c\ubb38\uc774\uc5c8\ub2e4. \uc758\ub8cc \uac80\uc9c4 \ud6c4, \ubca0\ucef4\uc740 \uad6c\ub2e8 \uc758\ub8cc\uc9c4\uc73c\ub85c\ubd80\ud130 38\uc138\uac00 \ub418\ub294 5\ub144 \ud6c4\uae4c\uc9c0\ub294 \ub354 \ucd95\uad6c\ub97c \ud560 \uc218 \uc788\uc744 \uac83\uc774\ub77c\ub294 \ud3c9\uc744 \ub4e4\uc5c8\ub2e4. \uc785\ub2e8\uc2dd \uc774\ud2bf\ub0a0, \ubca0\ucef4\uc740 \uc0b0 \uc2dc\ub85c\ub85c \uac00 \uc6b0\ub514\ub124\uc138\uc640\uc758 \ub9ac\uadf8 \uacbd\uae30\ub97c \uc55e\ub450\uace0 \uacbd\uae30\uc7a5 \uc548\uc73c\ub85c \uac78\uc5b4 \ub4e4\uc5b4\uac00 \uc548\ubc29\uc758 \uc9c0\uc9c0\uc790\ub4e4\uc758 \ud658\uc601\uc744 \ubc1b\uc558\uace0, \ubc29\uc1a1 \uc124\ube44\ub97c \ud1b5\ud574 \"\uac00\uc790 \ubc00\ub780\"\uc744 \uc678\ucce4\ub2e4. \uadf8 \ud6c4, \uadf8\uc640 \ubc30\uc6b0\uc790 \ube45\ud1a0\ub9ac\uc544\ub294 \uc694\uc778 \uad00\uc911\uc11d\uc5d0\uc11c \ubc00\ub780\uc758 5-1 \ub300\uc2b9\uc744 \uad00\uc804\ud588\ub2e4."} {"question": "\uc2e0\uce78\uc13c E6\uacc4\uc5d0\uc11c \uc0ac\uc6a9\ud55c \uc18c\uc74c\ud310\uc758 \uc7a5\ucc29 \ubc29\uc2dd\uc740?", "paragraph": "\ud3b8\uc131 \ub0b4\uc5d0\uc11c \uc0ac\uc6a9\ud558\ub294 \ud32c\ud130\uadf8\ub798\ud504\ub294 1\uae30 \ubfd0(12, 16\ud638\ucc28\uc5d0 \ud0d1\uc7ac\ub41c \uac83 \uc911 \uc9c4\ud589\ubc29\ud5a5 \ub4b7\ud3b8\uc744 \uc0ac\uc6a9)\uc774\ubbc0\ub85c, \uc774\uc120\uc774 \uc798 \ubc1c\uc0dd\ud558\uc9c0 \uc54a\ub294 \ub2e4\ubd84\ud560 \uc2b5\ud310\uc2dd \uc800\uc18c\uc74c \ud32c\ud130\uadf8\ub798\ud504 \"PS209\"\ub97c \uc7a5\ube44\ud55c\ub2e4. \ud32c\ud130\uadf8\ub798\ud504 \uce21\uba74\uc5d0\ub294 \uc18c\uc74c \ubc29\uc9c0\ub97c \uc704\ud55c \ubc29\uc74c\ud310\uc774 \uc7a5\ucc29\ub418\uc5b4 \uc788\ub294\ub370, E955\ud615\uc5d0\uc11c \uc0ac\uc6a9\ud55c \uc774\ub3d9\uc2dd\uc774 \uc544\ub2cc \uace0\uc815\uc2dd\uc774\ub2e4. \uc774\uac83\uc740 E5\uacc4\uc640 \uac19\uc740 \uc0ac\uc774\uc988\uc758 \ubc29\uc74c\ud310\uc740 \uc7ac\ub798\uc120 \uad6c\uac04\uc5d0\uc11c \ucc28\ub7c9\ud55c\uacc4\ub97c \ucd08\uacfc\ud558\uc9c0\ub9cc, \uc774\ub3d9\uc2dd\uc73c\ub85c \ud55c \uacbd\uc6b0 \uace0\uc7a5\uc73c\ub85c \uac00\ub3d9\ub418\uc9c0 \uc54a\uc558\uc744 \uacbd\uc6b0\uc758 \ucc28\uc9c8\uc774 \ub108\ubb34 \ucee4(\ubc29\uc74c\ud310\uc774 \uc791\ub3d9\ud558\uc9c0 \uc54a\uc744 \uacbd\uc6b0\uc5d0\ub294 \uc2e0\uce78\uc13c \uad6c\uac04\uc5d0\uc11c \uace0\uc18d\uc8fc\ud589\uc2dc \uc18c\uc74c \uae30\uc900\uc744 \ucda9\uc871\ud558\uc9c0 \ubabb\ud558\uace0, \uc7ac\ub798\uc120 \uad6c\uac04\uc5d0\uc11c\ub294 \ucc28\ub7c9\ud55c\uacc4\ub97c \ucd08\uacfc\ud558\uc5ec \uc8fc\ud589 \uc790\uccb4\uac00 \ubd88\uac00\ub2a5) \uc2e0\ub8b0\uc131\uc758 \uad00\uc810\uc5d0\uc11c \uace0\uc815\uc2dd\uc73c\ub85c \ud558\uace0 \ubc29\uc74c\ud310\uc744 \uc7ac\ub798\uc120 \ucc28\ub7c9\ud55c\uacc4 \uc548\uc5d0 \ub123\uae30 \uc704\ud574 E5\uacc4\ubcf4\ub2e4 \uc0ac\uc774\uc988\uac00 \uc791\uac8c \ud558\uc600\ub2e4.", "answer": "\uace0\uc815\uc2dd", "sentence": "\ud32c\ud130\uadf8\ub798\ud504 \uce21\uba74\uc5d0\ub294 \uc18c\uc74c \ubc29\uc9c0\ub97c \uc704\ud55c \ubc29\uc74c\ud310\uc774 \uc7a5\ucc29\ub418\uc5b4 \uc788\ub294\ub370, E955\ud615\uc5d0\uc11c \uc0ac\uc6a9\ud55c \uc774\ub3d9\uc2dd\uc774 \uc544\ub2cc \uace0\uc815\uc2dd \uc774\ub2e4.", "paragraph_sentence": "\ud3b8\uc131 \ub0b4\uc5d0\uc11c \uc0ac\uc6a9\ud558\ub294 \ud32c\ud130\uadf8\ub798\ud504\ub294 1\uae30 \ubfd0(12, 16\ud638\ucc28\uc5d0 \ud0d1\uc7ac\ub41c \uac83 \uc911 \uc9c4\ud589\ubc29\ud5a5 \ub4b7\ud3b8\uc744 \uc0ac\uc6a9)\uc774\ubbc0\ub85c, \uc774\uc120\uc774 \uc798 \ubc1c\uc0dd\ud558\uc9c0 \uc54a\ub294 \ub2e4\ubd84\ud560 \uc2b5\ud310\uc2dd \uc800\uc18c\uc74c \ud32c\ud130\uadf8\ub798\ud504 \"PS209\"\ub97c \uc7a5\ube44\ud55c\ub2e4. \ud32c\ud130\uadf8\ub798\ud504 \uce21\uba74\uc5d0\ub294 \uc18c\uc74c \ubc29\uc9c0\ub97c \uc704\ud55c \ubc29\uc74c\ud310\uc774 \uc7a5\ucc29\ub418\uc5b4 \uc788\ub294\ub370, E955\ud615\uc5d0\uc11c \uc0ac\uc6a9\ud55c \uc774\ub3d9\uc2dd\uc774 \uc544\ub2cc \uace0\uc815\uc2dd \uc774\ub2e4. \uc774\uac83\uc740 E5\uacc4\uc640 \uac19\uc740 \uc0ac\uc774\uc988\uc758 \ubc29\uc74c\ud310\uc740 \uc7ac\ub798\uc120 \uad6c\uac04\uc5d0\uc11c \ucc28\ub7c9\ud55c\uacc4\ub97c \ucd08\uacfc\ud558\uc9c0\ub9cc, \uc774\ub3d9\uc2dd\uc73c\ub85c \ud55c \uacbd\uc6b0 \uace0\uc7a5\uc73c\ub85c \uac00\ub3d9\ub418\uc9c0 \uc54a\uc558\uc744 \uacbd\uc6b0\uc758 \ucc28\uc9c8\uc774 \ub108\ubb34 \ucee4(\ubc29\uc74c\ud310\uc774 \uc791\ub3d9\ud558\uc9c0 \uc54a\uc744 \uacbd\uc6b0\uc5d0\ub294 \uc2e0\uce78\uc13c \uad6c\uac04\uc5d0\uc11c \uace0\uc18d\uc8fc\ud589\uc2dc \uc18c\uc74c \uae30\uc900\uc744 \ucda9\uc871\ud558\uc9c0 \ubabb\ud558\uace0, \uc7ac\ub798\uc120 \uad6c\uac04\uc5d0\uc11c\ub294 \ucc28\ub7c9\ud55c\uacc4\ub97c \ucd08\uacfc\ud558\uc5ec \uc8fc\ud589 \uc790\uccb4\uac00 \ubd88\uac00\ub2a5) \uc2e0\ub8b0\uc131\uc758 \uad00\uc810\uc5d0\uc11c \uace0\uc815\uc2dd\uc73c\ub85c \ud558\uace0 \ubc29\uc74c\ud310\uc744 \uc7ac\ub798\uc120 \ucc28\ub7c9\ud55c\uacc4 \uc548\uc5d0 \ub123\uae30 \uc704\ud574 E5\uacc4\ubcf4\ub2e4 \uc0ac\uc774\uc988\uac00 \uc791\uac8c \ud558\uc600\ub2e4.", "paragraph_answer": "\ud3b8\uc131 \ub0b4\uc5d0\uc11c \uc0ac\uc6a9\ud558\ub294 \ud32c\ud130\uadf8\ub798\ud504\ub294 1\uae30 \ubfd0(12, 16\ud638\ucc28\uc5d0 \ud0d1\uc7ac\ub41c \uac83 \uc911 \uc9c4\ud589\ubc29\ud5a5 \ub4b7\ud3b8\uc744 \uc0ac\uc6a9)\uc774\ubbc0\ub85c, \uc774\uc120\uc774 \uc798 \ubc1c\uc0dd\ud558\uc9c0 \uc54a\ub294 \ub2e4\ubd84\ud560 \uc2b5\ud310\uc2dd \uc800\uc18c\uc74c \ud32c\ud130\uadf8\ub798\ud504 \"PS209\"\ub97c \uc7a5\ube44\ud55c\ub2e4. \ud32c\ud130\uadf8\ub798\ud504 \uce21\uba74\uc5d0\ub294 \uc18c\uc74c \ubc29\uc9c0\ub97c \uc704\ud55c \ubc29\uc74c\ud310\uc774 \uc7a5\ucc29\ub418\uc5b4 \uc788\ub294\ub370, E955\ud615\uc5d0\uc11c \uc0ac\uc6a9\ud55c \uc774\ub3d9\uc2dd\uc774 \uc544\ub2cc \uace0\uc815\uc2dd \uc774\ub2e4. \uc774\uac83\uc740 E5\uacc4\uc640 \uac19\uc740 \uc0ac\uc774\uc988\uc758 \ubc29\uc74c\ud310\uc740 \uc7ac\ub798\uc120 \uad6c\uac04\uc5d0\uc11c \ucc28\ub7c9\ud55c\uacc4\ub97c \ucd08\uacfc\ud558\uc9c0\ub9cc, \uc774\ub3d9\uc2dd\uc73c\ub85c \ud55c \uacbd\uc6b0 \uace0\uc7a5\uc73c\ub85c \uac00\ub3d9\ub418\uc9c0 \uc54a\uc558\uc744 \uacbd\uc6b0\uc758 \ucc28\uc9c8\uc774 \ub108\ubb34 \ucee4(\ubc29\uc74c\ud310\uc774 \uc791\ub3d9\ud558\uc9c0 \uc54a\uc744 \uacbd\uc6b0\uc5d0\ub294 \uc2e0\uce78\uc13c \uad6c\uac04\uc5d0\uc11c \uace0\uc18d\uc8fc\ud589\uc2dc \uc18c\uc74c \uae30\uc900\uc744 \ucda9\uc871\ud558\uc9c0 \ubabb\ud558\uace0, \uc7ac\ub798\uc120 \uad6c\uac04\uc5d0\uc11c\ub294 \ucc28\ub7c9\ud55c\uacc4\ub97c \ucd08\uacfc\ud558\uc5ec \uc8fc\ud589 \uc790\uccb4\uac00 \ubd88\uac00\ub2a5) \uc2e0\ub8b0\uc131\uc758 \uad00\uc810\uc5d0\uc11c \uace0\uc815\uc2dd\uc73c\ub85c \ud558\uace0 \ubc29\uc74c\ud310\uc744 \uc7ac\ub798\uc120 \ucc28\ub7c9\ud55c\uacc4 \uc548\uc5d0 \ub123\uae30 \uc704\ud574 E5\uacc4\ubcf4\ub2e4 \uc0ac\uc774\uc988\uac00 \uc791\uac8c \ud558\uc600\ub2e4.", "sentence_answer": "\ud32c\ud130\uadf8\ub798\ud504 \uce21\uba74\uc5d0\ub294 \uc18c\uc74c \ubc29\uc9c0\ub97c \uc704\ud55c \ubc29\uc74c\ud310\uc774 \uc7a5\ucc29\ub418\uc5b4 \uc788\ub294\ub370, E955\ud615\uc5d0\uc11c \uc0ac\uc6a9\ud55c \uc774\ub3d9\uc2dd\uc774 \uc544\ub2cc \uace0\uc815\uc2dd \uc774\ub2e4.", "paragraph_id": "6533353-2-2", "paragraph_question": "question: \uc2e0\uce78\uc13c E6\uacc4\uc5d0\uc11c \uc0ac\uc6a9\ud55c \uc18c\uc74c\ud310\uc758 \uc7a5\ucc29 \ubc29\uc2dd\uc740?, context: \ud3b8\uc131 \ub0b4\uc5d0\uc11c \uc0ac\uc6a9\ud558\ub294 \ud32c\ud130\uadf8\ub798\ud504\ub294 1\uae30 \ubfd0(12, 16\ud638\ucc28\uc5d0 \ud0d1\uc7ac\ub41c \uac83 \uc911 \uc9c4\ud589\ubc29\ud5a5 \ub4b7\ud3b8\uc744 \uc0ac\uc6a9)\uc774\ubbc0\ub85c, \uc774\uc120\uc774 \uc798 \ubc1c\uc0dd\ud558\uc9c0 \uc54a\ub294 \ub2e4\ubd84\ud560 \uc2b5\ud310\uc2dd \uc800\uc18c\uc74c \ud32c\ud130\uadf8\ub798\ud504 \"PS209\"\ub97c \uc7a5\ube44\ud55c\ub2e4. \ud32c\ud130\uadf8\ub798\ud504 \uce21\uba74\uc5d0\ub294 \uc18c\uc74c \ubc29\uc9c0\ub97c \uc704\ud55c \ubc29\uc74c\ud310\uc774 \uc7a5\ucc29\ub418\uc5b4 \uc788\ub294\ub370, E955\ud615\uc5d0\uc11c \uc0ac\uc6a9\ud55c \uc774\ub3d9\uc2dd\uc774 \uc544\ub2cc \uace0\uc815\uc2dd\uc774\ub2e4. \uc774\uac83\uc740 E5\uacc4\uc640 \uac19\uc740 \uc0ac\uc774\uc988\uc758 \ubc29\uc74c\ud310\uc740 \uc7ac\ub798\uc120 \uad6c\uac04\uc5d0\uc11c \ucc28\ub7c9\ud55c\uacc4\ub97c \ucd08\uacfc\ud558\uc9c0\ub9cc, \uc774\ub3d9\uc2dd\uc73c\ub85c \ud55c \uacbd\uc6b0 \uace0\uc7a5\uc73c\ub85c \uac00\ub3d9\ub418\uc9c0 \uc54a\uc558\uc744 \uacbd\uc6b0\uc758 \ucc28\uc9c8\uc774 \ub108\ubb34 \ucee4(\ubc29\uc74c\ud310\uc774 \uc791\ub3d9\ud558\uc9c0 \uc54a\uc744 \uacbd\uc6b0\uc5d0\ub294 \uc2e0\uce78\uc13c \uad6c\uac04\uc5d0\uc11c \uace0\uc18d\uc8fc\ud589\uc2dc \uc18c\uc74c \uae30\uc900\uc744 \ucda9\uc871\ud558\uc9c0 \ubabb\ud558\uace0, \uc7ac\ub798\uc120 \uad6c\uac04\uc5d0\uc11c\ub294 \ucc28\ub7c9\ud55c\uacc4\ub97c \ucd08\uacfc\ud558\uc5ec \uc8fc\ud589 \uc790\uccb4\uac00 \ubd88\uac00\ub2a5) \uc2e0\ub8b0\uc131\uc758 \uad00\uc810\uc5d0\uc11c \uace0\uc815\uc2dd\uc73c\ub85c \ud558\uace0 \ubc29\uc74c\ud310\uc744 \uc7ac\ub798\uc120 \ucc28\ub7c9\ud55c\uacc4 \uc548\uc5d0 \ub123\uae30 \uc704\ud574 E5\uacc4\ubcf4\ub2e4 \uc0ac\uc774\uc988\uac00 \uc791\uac8c \ud558\uc600\ub2e4."} {"question": "\uc5f4\ud3ed\uc8fc\ub85c \uc778\ud558\uc5ec \ubd80\ucc29\ub41c \uae30\uccb4\uac00 \ub2ec\uc544\ub098\ub824\uba74 \uc5b4\ub5a4 \ud658\uacbd\uc5d0 \ub3c4\ub2ec\ud574\uc57c \ud558\ub294\uac00?", "paragraph": "\ubc31\uc0c9 \uc65c\uc131 B\uc758 \ud45c\uba74\uc5d0\uc11c\ub294 \ubd80\ucc29\ub41c \uae30\uccb4\uac00 \uc555\ucd95\ub418\uace0 \ub728\uac70\uc6cc\uc9c8 \uac83\uc774\ub2e4. \ubd80\ucc29\ub41c \uae30\uccb4\uac00 \uc218\uc18c \ud575\uc735\ud569\uc774 \uc77c\uc5b4\ub0a0 \uc218 \uc788\ub294 \ud658\uacbd\uc5d0 \ub2e4\ub2e4\ub974\uba74, \uc5f4\ud3ed\uc8fc\uac00 \uc77c\uc5b4\ub098 \uae30\uccb4 \uc77c\ubd80\uac00 \ud45c\uba74\uc5d0\uc11c \ub2ec\uc544\ub0a0 \uac83\uc774\ub2e4. \uc774\ub807\uac8c \ub418\uba74 \uc815\uae30\uc801\uc73c\ub85c \uc77c\uc5b4\ub098\ub294 \uc2e0\uc131 \ud3ed\ubc1c, \uc989 \uaca9\ubcc0 \ubcc0\uad11\uc131\uc774 \ub418\uace0, \ubc31\uc0c9 \uc65c\uc131 B\uc758 \uad11\ub3c4\ub294 \uae09\uaca9\ud788 \uc0c1\uc2b9\ud558\uc5ec \uc218\uc77c \ub610\ub294 \uc218 \uac1c\uc6d4\uc758 \uc8fc\uae30\ub97c \uac00\uc9c0\uace0 \uc2e4\uc2dc\ub4f1\uae09\uc758 \ubcc0\ud654\uac00 \uc788\uc744 \uac83\uc774\ub2e4. \uc774\ub807\uac8c \uc801\uc0c9 \uac70\uc131\uacfc \ubc31\uc0c9 \uc65c\uc131\uc774 \uc30d\uc131\uacc4\ub97c \uc774\ub8e8\ub294 \ud56d\uc131\uacc4\uc758 \ud615\ud0dc\ub97c \ubc40\uc8fc\uc778\uc790\ub9ac RS \uc30d\uc131\uacc4\ub77c\uace0 \ubd80\ub978\ub2e4. \ubc40\uc8fc\uc778\uc790\ub9ac RS\ub294 \uc815\uae30\uc801\uc778 \uc2e0\uc131\uc774 \ub418\uc5b4 \ucd5c\uc18c 6\ubc88 \ud3ed\ubc1c\ud588\uc73c\uba70, \uadf8\ub54c\ub9c8\ub2e4 \ud3ed\uc8fc \ud3ed\ubc1c\uc744 \uc77c\uc73c\ud0a4\uae30 \uc704\ud574 \ud544\uc6a9\ud55c \ub9cc\ud07c\uc758 \uc784\uacc4\ub7c9\uc758 \uc218\uc18c\uac00 \ubd80\ucc29\ub418\uc5c8\uc744 \uac83\uc774\ub2e4. \ud398\uac00\uc218\uc2a4\uc790\ub9ac IK B\ub3c4 \ube44\uc2b7\ud55c \ud328\ud134\uc744 \uacaa\uc744 \uac83\uc774\ub2e4. \ud558\uc9c0\ub9cc \uc9c8\ub7c9\uc744 \ucd95\uc801\uc2dc\ud0a4\uae30 \uc704\ud574\uc11c \ubd80\ucc29\ub41c \uae30\uccb4\uc758 \uc77c\ubd80\ub9cc \ubd84\ucd9c\ub420 \uac83\uc774\uace0 \ub9e4 \uc8fc\uae30\ub9c8\ub2e4 \ubc31\uc0c9 \uc65c\uc131 B\uc758 \uc9c8\ub7c9\uc740 \uafb8\uc900\ud788 \uc99d\uac00\ud560 \uac83\uc774\ub2e4. \uadf8\ub7ec\ubbc0\ub85c \ub418\ud480\uc774\ud558\uba70 \uc2e0\uc131\uc73c\ub85c\uc11c \ud3ed\ubc1c\ud574\ub3c4 \ud398\uac00\uc218\uc2a4\uc790\ub9ac IK B\uc758 \uc678\ud53c\ub294 \uacc4\uc18d\ud574\uc11c \uc131\uc7a5\ud560 \uac83\uc774\ub2e4.", "answer": "\uc218\uc18c \ud575\uc735\ud569\uc774 \uc77c\uc5b4\ub0a0 \uc218 \uc788\ub294 \ud658\uacbd", "sentence": "\ubd80\ucc29\ub41c \uae30\uccb4\uac00 \uc218\uc18c \ud575\uc735\ud569\uc774 \uc77c\uc5b4\ub0a0 \uc218 \uc788\ub294 \ud658\uacbd \uc5d0 \ub2e4\ub2e4\ub974\uba74, \uc5f4\ud3ed\uc8fc\uac00 \uc77c\uc5b4\ub098 \uae30\uccb4 \uc77c\ubd80\uac00 \ud45c\uba74\uc5d0\uc11c \ub2ec\uc544\ub0a0 \uac83\uc774\ub2e4.", "paragraph_sentence": "\ubc31\uc0c9 \uc65c\uc131 B\uc758 \ud45c\uba74\uc5d0\uc11c\ub294 \ubd80\ucc29\ub41c \uae30\uccb4\uac00 \uc555\ucd95\ub418\uace0 \ub728\uac70\uc6cc\uc9c8 \uac83\uc774\ub2e4. \ubd80\ucc29\ub41c \uae30\uccb4\uac00 \uc218\uc18c \ud575\uc735\ud569\uc774 \uc77c\uc5b4\ub0a0 \uc218 \uc788\ub294 \ud658\uacbd \uc5d0 \ub2e4\ub2e4\ub974\uba74, \uc5f4\ud3ed\uc8fc\uac00 \uc77c\uc5b4\ub098 \uae30\uccb4 \uc77c\ubd80\uac00 \ud45c\uba74\uc5d0\uc11c \ub2ec\uc544\ub0a0 \uac83\uc774\ub2e4. \uc774\ub807\uac8c \ub418\uba74 \uc815\uae30\uc801\uc73c\ub85c \uc77c\uc5b4\ub098\ub294 \uc2e0\uc131 \ud3ed\ubc1c, \uc989 \uaca9\ubcc0 \ubcc0\uad11\uc131\uc774 \ub418\uace0, \ubc31\uc0c9 \uc65c\uc131 B\uc758 \uad11\ub3c4\ub294 \uae09\uaca9\ud788 \uc0c1\uc2b9\ud558\uc5ec \uc218\uc77c \ub610\ub294 \uc218 \uac1c\uc6d4\uc758 \uc8fc\uae30\ub97c \uac00\uc9c0\uace0 \uc2e4\uc2dc\ub4f1\uae09\uc758 \ubcc0\ud654\uac00 \uc788\uc744 \uac83\uc774\ub2e4. \uc774\ub807\uac8c \uc801\uc0c9 \uac70\uc131\uacfc \ubc31\uc0c9 \uc65c\uc131\uc774 \uc30d\uc131\uacc4\ub97c \uc774\ub8e8\ub294 \ud56d\uc131\uacc4\uc758 \ud615\ud0dc\ub97c \ubc40\uc8fc\uc778\uc790\ub9ac RS \uc30d\uc131\uacc4\ub77c\uace0 \ubd80\ub978\ub2e4. \ubc40\uc8fc\uc778\uc790\ub9ac RS\ub294 \uc815\uae30\uc801\uc778 \uc2e0\uc131\uc774 \ub418\uc5b4 \ucd5c\uc18c 6\ubc88 \ud3ed\ubc1c\ud588\uc73c\uba70, \uadf8\ub54c\ub9c8\ub2e4 \ud3ed\uc8fc \ud3ed\ubc1c\uc744 \uc77c\uc73c\ud0a4\uae30 \uc704\ud574 \ud544\uc6a9\ud55c \ub9cc\ud07c\uc758 \uc784\uacc4\ub7c9\uc758 \uc218\uc18c\uac00 \ubd80\ucc29\ub418\uc5c8\uc744 \uac83\uc774\ub2e4. \ud398\uac00\uc218\uc2a4\uc790\ub9ac IK B\ub3c4 \ube44\uc2b7\ud55c \ud328\ud134\uc744 \uacaa\uc744 \uac83\uc774\ub2e4. \ud558\uc9c0\ub9cc \uc9c8\ub7c9\uc744 \ucd95\uc801\uc2dc\ud0a4\uae30 \uc704\ud574\uc11c \ubd80\ucc29\ub41c \uae30\uccb4\uc758 \uc77c\ubd80\ub9cc \ubd84\ucd9c\ub420 \uac83\uc774\uace0 \ub9e4 \uc8fc\uae30\ub9c8\ub2e4 \ubc31\uc0c9 \uc65c\uc131 B\uc758 \uc9c8\ub7c9\uc740 \uafb8\uc900\ud788 \uc99d\uac00\ud560 \uac83\uc774\ub2e4. \uadf8\ub7ec\ubbc0\ub85c \ub418\ud480\uc774\ud558\uba70 \uc2e0\uc131\uc73c\ub85c\uc11c \ud3ed\ubc1c\ud574\ub3c4 \ud398\uac00\uc218\uc2a4\uc790\ub9ac IK B\uc758 \uc678\ud53c\ub294 \uacc4\uc18d\ud574\uc11c \uc131\uc7a5\ud560 \uac83\uc774\ub2e4.", "paragraph_answer": "\ubc31\uc0c9 \uc65c\uc131 B\uc758 \ud45c\uba74\uc5d0\uc11c\ub294 \ubd80\ucc29\ub41c \uae30\uccb4\uac00 \uc555\ucd95\ub418\uace0 \ub728\uac70\uc6cc\uc9c8 \uac83\uc774\ub2e4. \ubd80\ucc29\ub41c \uae30\uccb4\uac00 \uc218\uc18c \ud575\uc735\ud569\uc774 \uc77c\uc5b4\ub0a0 \uc218 \uc788\ub294 \ud658\uacbd \uc5d0 \ub2e4\ub2e4\ub974\uba74, \uc5f4\ud3ed\uc8fc\uac00 \uc77c\uc5b4\ub098 \uae30\uccb4 \uc77c\ubd80\uac00 \ud45c\uba74\uc5d0\uc11c \ub2ec\uc544\ub0a0 \uac83\uc774\ub2e4. \uc774\ub807\uac8c \ub418\uba74 \uc815\uae30\uc801\uc73c\ub85c \uc77c\uc5b4\ub098\ub294 \uc2e0\uc131 \ud3ed\ubc1c, \uc989 \uaca9\ubcc0 \ubcc0\uad11\uc131\uc774 \ub418\uace0, \ubc31\uc0c9 \uc65c\uc131 B\uc758 \uad11\ub3c4\ub294 \uae09\uaca9\ud788 \uc0c1\uc2b9\ud558\uc5ec \uc218\uc77c \ub610\ub294 \uc218 \uac1c\uc6d4\uc758 \uc8fc\uae30\ub97c \uac00\uc9c0\uace0 \uc2e4\uc2dc\ub4f1\uae09\uc758 \ubcc0\ud654\uac00 \uc788\uc744 \uac83\uc774\ub2e4. \uc774\ub807\uac8c \uc801\uc0c9 \uac70\uc131\uacfc \ubc31\uc0c9 \uc65c\uc131\uc774 \uc30d\uc131\uacc4\ub97c \uc774\ub8e8\ub294 \ud56d\uc131\uacc4\uc758 \ud615\ud0dc\ub97c \ubc40\uc8fc\uc778\uc790\ub9ac RS \uc30d\uc131\uacc4\ub77c\uace0 \ubd80\ub978\ub2e4. \ubc40\uc8fc\uc778\uc790\ub9ac RS\ub294 \uc815\uae30\uc801\uc778 \uc2e0\uc131\uc774 \ub418\uc5b4 \ucd5c\uc18c 6\ubc88 \ud3ed\ubc1c\ud588\uc73c\uba70, \uadf8\ub54c\ub9c8\ub2e4 \ud3ed\uc8fc \ud3ed\ubc1c\uc744 \uc77c\uc73c\ud0a4\uae30 \uc704\ud574 \ud544\uc6a9\ud55c \ub9cc\ud07c\uc758 \uc784\uacc4\ub7c9\uc758 \uc218\uc18c\uac00 \ubd80\ucc29\ub418\uc5c8\uc744 \uac83\uc774\ub2e4. \ud398\uac00\uc218\uc2a4\uc790\ub9ac IK B\ub3c4 \ube44\uc2b7\ud55c \ud328\ud134\uc744 \uacaa\uc744 \uac83\uc774\ub2e4. \ud558\uc9c0\ub9cc \uc9c8\ub7c9\uc744 \ucd95\uc801\uc2dc\ud0a4\uae30 \uc704\ud574\uc11c \ubd80\ucc29\ub41c \uae30\uccb4\uc758 \uc77c\ubd80\ub9cc \ubd84\ucd9c\ub420 \uac83\uc774\uace0 \ub9e4 \uc8fc\uae30\ub9c8\ub2e4 \ubc31\uc0c9 \uc65c\uc131 B\uc758 \uc9c8\ub7c9\uc740 \uafb8\uc900\ud788 \uc99d\uac00\ud560 \uac83\uc774\ub2e4. \uadf8\ub7ec\ubbc0\ub85c \ub418\ud480\uc774\ud558\uba70 \uc2e0\uc131\uc73c\ub85c\uc11c \ud3ed\ubc1c\ud574\ub3c4 \ud398\uac00\uc218\uc2a4\uc790\ub9ac IK B\uc758 \uc678\ud53c\ub294 \uacc4\uc18d\ud574\uc11c \uc131\uc7a5\ud560 \uac83\uc774\ub2e4.", "sentence_answer": "\ubd80\ucc29\ub41c \uae30\uccb4\uac00 \uc218\uc18c \ud575\uc735\ud569\uc774 \uc77c\uc5b4\ub0a0 \uc218 \uc788\ub294 \ud658\uacbd \uc5d0 \ub2e4\ub2e4\ub974\uba74, \uc5f4\ud3ed\uc8fc\uac00 \uc77c\uc5b4\ub098 \uae30\uccb4 \uc77c\ubd80\uac00 \ud45c\uba74\uc5d0\uc11c \ub2ec\uc544\ub0a0 \uac83\uc774\ub2e4.", "paragraph_id": "6566884-4-0", "paragraph_question": "question: \uc5f4\ud3ed\uc8fc\ub85c \uc778\ud558\uc5ec \ubd80\ucc29\ub41c \uae30\uccb4\uac00 \ub2ec\uc544\ub098\ub824\uba74 \uc5b4\ub5a4 \ud658\uacbd\uc5d0 \ub3c4\ub2ec\ud574\uc57c \ud558\ub294\uac00?, context: \ubc31\uc0c9 \uc65c\uc131 B\uc758 \ud45c\uba74\uc5d0\uc11c\ub294 \ubd80\ucc29\ub41c \uae30\uccb4\uac00 \uc555\ucd95\ub418\uace0 \ub728\uac70\uc6cc\uc9c8 \uac83\uc774\ub2e4. \ubd80\ucc29\ub41c \uae30\uccb4\uac00 \uc218\uc18c \ud575\uc735\ud569\uc774 \uc77c\uc5b4\ub0a0 \uc218 \uc788\ub294 \ud658\uacbd\uc5d0 \ub2e4\ub2e4\ub974\uba74, \uc5f4\ud3ed\uc8fc\uac00 \uc77c\uc5b4\ub098 \uae30\uccb4 \uc77c\ubd80\uac00 \ud45c\uba74\uc5d0\uc11c \ub2ec\uc544\ub0a0 \uac83\uc774\ub2e4. \uc774\ub807\uac8c \ub418\uba74 \uc815\uae30\uc801\uc73c\ub85c \uc77c\uc5b4\ub098\ub294 \uc2e0\uc131 \ud3ed\ubc1c, \uc989 \uaca9\ubcc0 \ubcc0\uad11\uc131\uc774 \ub418\uace0, \ubc31\uc0c9 \uc65c\uc131 B\uc758 \uad11\ub3c4\ub294 \uae09\uaca9\ud788 \uc0c1\uc2b9\ud558\uc5ec \uc218\uc77c \ub610\ub294 \uc218 \uac1c\uc6d4\uc758 \uc8fc\uae30\ub97c \uac00\uc9c0\uace0 \uc2e4\uc2dc\ub4f1\uae09\uc758 \ubcc0\ud654\uac00 \uc788\uc744 \uac83\uc774\ub2e4. \uc774\ub807\uac8c \uc801\uc0c9 \uac70\uc131\uacfc \ubc31\uc0c9 \uc65c\uc131\uc774 \uc30d\uc131\uacc4\ub97c \uc774\ub8e8\ub294 \ud56d\uc131\uacc4\uc758 \ud615\ud0dc\ub97c \ubc40\uc8fc\uc778\uc790\ub9ac RS \uc30d\uc131\uacc4\ub77c\uace0 \ubd80\ub978\ub2e4. \ubc40\uc8fc\uc778\uc790\ub9ac RS\ub294 \uc815\uae30\uc801\uc778 \uc2e0\uc131\uc774 \ub418\uc5b4 \ucd5c\uc18c 6\ubc88 \ud3ed\ubc1c\ud588\uc73c\uba70, \uadf8\ub54c\ub9c8\ub2e4 \ud3ed\uc8fc \ud3ed\ubc1c\uc744 \uc77c\uc73c\ud0a4\uae30 \uc704\ud574 \ud544\uc6a9\ud55c \ub9cc\ud07c\uc758 \uc784\uacc4\ub7c9\uc758 \uc218\uc18c\uac00 \ubd80\ucc29\ub418\uc5c8\uc744 \uac83\uc774\ub2e4. \ud398\uac00\uc218\uc2a4\uc790\ub9ac IK B\ub3c4 \ube44\uc2b7\ud55c \ud328\ud134\uc744 \uacaa\uc744 \uac83\uc774\ub2e4. \ud558\uc9c0\ub9cc \uc9c8\ub7c9\uc744 \ucd95\uc801\uc2dc\ud0a4\uae30 \uc704\ud574\uc11c \ubd80\ucc29\ub41c \uae30\uccb4\uc758 \uc77c\ubd80\ub9cc \ubd84\ucd9c\ub420 \uac83\uc774\uace0 \ub9e4 \uc8fc\uae30\ub9c8\ub2e4 \ubc31\uc0c9 \uc65c\uc131 B\uc758 \uc9c8\ub7c9\uc740 \uafb8\uc900\ud788 \uc99d\uac00\ud560 \uac83\uc774\ub2e4. \uadf8\ub7ec\ubbc0\ub85c \ub418\ud480\uc774\ud558\uba70 \uc2e0\uc131\uc73c\ub85c\uc11c \ud3ed\ubc1c\ud574\ub3c4 \ud398\uac00\uc218\uc2a4\uc790\ub9ac IK B\uc758 \uc678\ud53c\ub294 \uacc4\uc18d\ud574\uc11c \uc131\uc7a5\ud560 \uac83\uc774\ub2e4."} {"question": "\ucd1d\ud559\uc0dd\ud68c\ub97c \ub300\ud45c\ub85c \uc6b4\uc601\uc704\uc6d0\ud68c\uc640 \uc9d1\ud589\ubd80\uc758 \uc7a5\uc758 \uc5ed\ud560\uc744 \ub9e1\ub294 \uc9c1\ucc45\uc740?", "paragraph": "\ucd1d\ud559\uc0dd\ud68c\ub294 \uc219\uba85\uc5ec\ub300\uc758 \uc7ac\ud559\uc0dd \ubaa8\uc784\uc774\uba70 \ud559\uc0dd\ucd1d\ud68c, \ub300\uc758\uc6d0\ud68c, \ucd1d\ud559\uc0dd\ud68c, \uc6b4\uc601\uc704\uc6d0\ud68c, \ub2e8\uacfc \ub300\ud559 \ud559\uc0dd\ud68c, \ud559\uacfc\u2219\ud559\ubd80\u2219\uc804\uacf5 \ud559\uc0dd\ud68c, \uae30\ud0c0 \ud559\uc0dd \ubb38\ud654 \ud65c\ub3d9 \ub2e8\uccb4\ub85c \uad6c\uc131\ub41c\ub2e4. \ud559\uc0dd\ucd1d\ud68c\ub294 \ucd1d\ud559\uc0dd\ud68c\uc5d0 \uad00\ub828\ub41c \uc911\ub300\ud55c \uc0ac\ud56d\uc744 \uc758\uacb0\ud558\ub294 \uae30\uad6c\ub85c\uc11c \ub300\uc758\uc6d0\ud68c\uc758 \uc694\uad6c \ub610\ub294 \ucd1d\ud559\uc0dd\ud68c \ud68c\uc6d0 10\ubd84\uc758 1 \uc774\uc0c1\uc758 \uc694\uad6c\uac00 \uc788\uc744 \ub54c \ucd1d\ud559\uc0dd\ud68c\uc7a5\uc774 \uc18c\uc9d1\ud55c\ub2e4. \ub300\uc758\uc6d0\ud68c\ub294 \uc219\uba85\uc5ec\ub300\uc758 \ud559\uacfc\u2219\ud559\ubd80\u2219\uc804\uacf5 \ud559\ud68c\uc7a5\uacfc \ubd80\ud559\ud68c\uc7a5\ub85c \uad6c\uc131\ub418\ub294 \uae30\uad6c\ub85c\uc11c \ud559\uc0dd\ucd1d\ud68c\uc758 \uc18c\uc9d1\uc744 \uc694\uad6c\ud560 \uad8c\ud55c \ubfd0\ub9cc \uc544\ub098\ub77c \ucd1d\ud559\uc0dd\ud68c \ud68c\uce59\uc758 \uac1c\uc815\uacfc \ubc1c\uc758\ub97c \ud655\uc815\ud560 \uad8c\ud55c, \ucd1d\ud559\uc0dd\ud68c\uc758 \uc5c5\ubb34\ub97c \uac10\uc0ac\ud560 \uad8c\ud55c, \ucd1d\ud559\uc0dd\ud68c\uc7a5\ub2e8\uc5d0 \ub300\ud55c \ubd88\uc2e0\uc784\uc744 \uacb0\uc758\ud560 \uad8c\ud55c \ub4f1\uc744 \uac00\uc9c4\ub2e4. \ucd1d\ud559\uc0dd\ud68c\uc7a5\uc740 \ucd1d\ud559\uc0dd\ud68c\ub97c \ub300\ud45c\ud558\uba70 \uc6b4\uc601\uc704\uc6d0\ud68c\uc640 \uc9d1\ud589\ubd80\uc758 \uc7a5\uc774 \ub41c\ub2e4. \ucd1d\ud559\uc0dd\ud68c\uc7a5\uc740 \ucd1d\ud559\uc0dd\ud68c\uc758 \uc0ac\uc5c5 \uacc4\ud68d\uc744 \uc218\ub9bd\ud558\uace0 \ud3b8\uc131\ub41c \uc608\uc0b0\uacfc \uacb0\uc0b0\uc744 \ub300\uc758\uc6d0\ud68c\uc5d0 \uc81c\ucd9c\ud55c\ub2e4. \ub610\ud55c \ucd1d\ud559\uc0dd\ud68c\uc5d0 \uc911\ub300\ud55c \uc601\ud5a5\uc744 \ubbf8\uce58\ub294 \uc0ac\ud56d\uc5d0 \uad00\ud558\uc5ec \ud559\uad50\uc640 \uad00\uacc4 \uae30\uad00\uc5d0 \uc758\uacac\uc11c\ub97c \uc81c\ucd9c\ud560 \uc218 \uc788\ub2e4. \uc6b4\uc601 \uc704\uc6d0\ud68c\ub294 \ucd1d\ud559\uc0dd\ud68c\uc758 \uc2ec\uc758, \uc790\ubb38 \uae30\uad6c\ub85c\uc11c \ucd1d\ud559\uc0dd\ud68c\uc7a5, \ubd80\ud559\uc0dd\ud68c\uc7a5, \ub2e8\uacfc \ub300\ud559 \ud559\uc0dd\ud68c\uc7a5, \ucd1d\ud559\uc0dd\ud68c \uc9d1\ud589\ubd80\uc758 \ubd80\uc7a5\uc73c\ub85c \uad6c\uc131\ub418\uba70, \ucd1d\ud559\uc0dd\ud68c\uac00 \uc6b4\uc601\ud558\ub294 \uc81c\ubc18 \uc0ac\uc5c5, \ucd1d\ud559\uc0dd\ud68c\uc7a5\uc774 \ub300\uc758\uc6d0\ud68c\uc5d0 \uc81c\ucd9c\ud560 \uc608\uc0b0\uacfc \uacb0\uc0b0, \ucd1d\ud559\uc0dd\ud68c\uc7a5\uc758 \ub300\uc758\uc6d0\ud68c\uc5d0 \ub300\ud55c \uc18c\uc9d1 \uc694\uad6c \ub4f1\uc744 \uc2ec\uc758, \uc790\ubb38\ud55c\ub2e4.", "answer": "\ucd1d\ud559\uc0dd\ud68c\uc7a5", "sentence": "\ud559\uc0dd\ucd1d\ud68c\ub294 \ucd1d\ud559\uc0dd\ud68c\uc5d0 \uad00\ub828\ub41c \uc911\ub300\ud55c \uc0ac\ud56d\uc744 \uc758\uacb0\ud558\ub294 \uae30\uad6c\ub85c\uc11c \ub300\uc758\uc6d0\ud68c\uc758 \uc694\uad6c \ub610\ub294 \ucd1d\ud559\uc0dd\ud68c \ud68c\uc6d0 10\ubd84\uc758 1 \uc774\uc0c1\uc758 \uc694\uad6c\uac00 \uc788\uc744 \ub54c \ucd1d\ud559\uc0dd\ud68c\uc7a5 \uc774 \uc18c\uc9d1\ud55c\ub2e4.", "paragraph_sentence": "\ucd1d\ud559\uc0dd\ud68c\ub294 \uc219\uba85\uc5ec\ub300\uc758 \uc7ac\ud559\uc0dd \ubaa8\uc784\uc774\uba70 \ud559\uc0dd\ucd1d\ud68c, \ub300\uc758\uc6d0\ud68c, \ucd1d\ud559\uc0dd\ud68c, \uc6b4\uc601\uc704\uc6d0\ud68c, \ub2e8\uacfc \ub300\ud559 \ud559\uc0dd\ud68c, \ud559\uacfc\u2219\ud559\ubd80\u2219\uc804\uacf5 \ud559\uc0dd\ud68c, \uae30\ud0c0 \ud559\uc0dd \ubb38\ud654 \ud65c\ub3d9 \ub2e8\uccb4\ub85c \uad6c\uc131\ub41c\ub2e4. \ud559\uc0dd\ucd1d\ud68c\ub294 \ucd1d\ud559\uc0dd\ud68c\uc5d0 \uad00\ub828\ub41c \uc911\ub300\ud55c \uc0ac\ud56d\uc744 \uc758\uacb0\ud558\ub294 \uae30\uad6c\ub85c\uc11c \ub300\uc758\uc6d0\ud68c\uc758 \uc694\uad6c \ub610\ub294 \ucd1d\ud559\uc0dd\ud68c \ud68c\uc6d0 10\ubd84\uc758 1 \uc774\uc0c1\uc758 \uc694\uad6c\uac00 \uc788\uc744 \ub54c \ucd1d\ud559\uc0dd\ud68c\uc7a5 \uc774 \uc18c\uc9d1\ud55c\ub2e4. \ub300\uc758\uc6d0\ud68c\ub294 \uc219\uba85\uc5ec\ub300\uc758 \ud559\uacfc\u2219\ud559\ubd80\u2219\uc804\uacf5 \ud559\ud68c\uc7a5\uacfc \ubd80\ud559\ud68c\uc7a5\ub85c \uad6c\uc131\ub418\ub294 \uae30\uad6c\ub85c\uc11c \ud559\uc0dd\ucd1d\ud68c\uc758 \uc18c\uc9d1\uc744 \uc694\uad6c\ud560 \uad8c\ud55c \ubfd0\ub9cc \uc544\ub098\ub77c \ucd1d\ud559\uc0dd\ud68c \ud68c\uce59\uc758 \uac1c\uc815\uacfc \ubc1c\uc758\ub97c \ud655\uc815\ud560 \uad8c\ud55c, \ucd1d\ud559\uc0dd\ud68c\uc758 \uc5c5\ubb34\ub97c \uac10\uc0ac\ud560 \uad8c\ud55c, \ucd1d\ud559\uc0dd\ud68c\uc7a5\ub2e8\uc5d0 \ub300\ud55c \ubd88\uc2e0\uc784\uc744 \uacb0\uc758\ud560 \uad8c\ud55c \ub4f1\uc744 \uac00\uc9c4\ub2e4. \ucd1d\ud559\uc0dd\ud68c\uc7a5\uc740 \ucd1d\ud559\uc0dd\ud68c\ub97c \ub300\ud45c\ud558\uba70 \uc6b4\uc601\uc704\uc6d0\ud68c\uc640 \uc9d1\ud589\ubd80\uc758 \uc7a5\uc774 \ub41c\ub2e4. \ucd1d\ud559\uc0dd\ud68c\uc7a5\uc740 \ucd1d\ud559\uc0dd\ud68c\uc758 \uc0ac\uc5c5 \uacc4\ud68d\uc744 \uc218\ub9bd\ud558\uace0 \ud3b8\uc131\ub41c \uc608\uc0b0\uacfc \uacb0\uc0b0\uc744 \ub300\uc758\uc6d0\ud68c\uc5d0 \uc81c\ucd9c\ud55c\ub2e4. \ub610\ud55c \ucd1d\ud559\uc0dd\ud68c\uc5d0 \uc911\ub300\ud55c \uc601\ud5a5\uc744 \ubbf8\uce58\ub294 \uc0ac\ud56d\uc5d0 \uad00\ud558\uc5ec \ud559\uad50\uc640 \uad00\uacc4 \uae30\uad00\uc5d0 \uc758\uacac\uc11c\ub97c \uc81c\ucd9c\ud560 \uc218 \uc788\ub2e4. \uc6b4\uc601 \uc704\uc6d0\ud68c\ub294 \ucd1d\ud559\uc0dd\ud68c\uc758 \uc2ec\uc758, \uc790\ubb38 \uae30\uad6c\ub85c\uc11c \ucd1d\ud559\uc0dd\ud68c\uc7a5, \ubd80\ud559\uc0dd\ud68c\uc7a5, \ub2e8\uacfc \ub300\ud559 \ud559\uc0dd\ud68c\uc7a5, \ucd1d\ud559\uc0dd\ud68c \uc9d1\ud589\ubd80\uc758 \ubd80\uc7a5\uc73c\ub85c \uad6c\uc131\ub418\uba70, \ucd1d\ud559\uc0dd\ud68c\uac00 \uc6b4\uc601\ud558\ub294 \uc81c\ubc18 \uc0ac\uc5c5, \ucd1d\ud559\uc0dd\ud68c\uc7a5\uc774 \ub300\uc758\uc6d0\ud68c\uc5d0 \uc81c\ucd9c\ud560 \uc608\uc0b0\uacfc \uacb0\uc0b0, \ucd1d\ud559\uc0dd\ud68c\uc7a5\uc758 \ub300\uc758\uc6d0\ud68c\uc5d0 \ub300\ud55c \uc18c\uc9d1 \uc694\uad6c \ub4f1\uc744 \uc2ec\uc758, \uc790\ubb38\ud55c\ub2e4.", "paragraph_answer": "\ucd1d\ud559\uc0dd\ud68c\ub294 \uc219\uba85\uc5ec\ub300\uc758 \uc7ac\ud559\uc0dd \ubaa8\uc784\uc774\uba70 \ud559\uc0dd\ucd1d\ud68c, \ub300\uc758\uc6d0\ud68c, \ucd1d\ud559\uc0dd\ud68c, \uc6b4\uc601\uc704\uc6d0\ud68c, \ub2e8\uacfc \ub300\ud559 \ud559\uc0dd\ud68c, \ud559\uacfc\u2219\ud559\ubd80\u2219\uc804\uacf5 \ud559\uc0dd\ud68c, \uae30\ud0c0 \ud559\uc0dd \ubb38\ud654 \ud65c\ub3d9 \ub2e8\uccb4\ub85c \uad6c\uc131\ub41c\ub2e4. \ud559\uc0dd\ucd1d\ud68c\ub294 \ucd1d\ud559\uc0dd\ud68c\uc5d0 \uad00\ub828\ub41c \uc911\ub300\ud55c \uc0ac\ud56d\uc744 \uc758\uacb0\ud558\ub294 \uae30\uad6c\ub85c\uc11c \ub300\uc758\uc6d0\ud68c\uc758 \uc694\uad6c \ub610\ub294 \ucd1d\ud559\uc0dd\ud68c \ud68c\uc6d0 10\ubd84\uc758 1 \uc774\uc0c1\uc758 \uc694\uad6c\uac00 \uc788\uc744 \ub54c \ucd1d\ud559\uc0dd\ud68c\uc7a5 \uc774 \uc18c\uc9d1\ud55c\ub2e4. \ub300\uc758\uc6d0\ud68c\ub294 \uc219\uba85\uc5ec\ub300\uc758 \ud559\uacfc\u2219\ud559\ubd80\u2219\uc804\uacf5 \ud559\ud68c\uc7a5\uacfc \ubd80\ud559\ud68c\uc7a5\ub85c \uad6c\uc131\ub418\ub294 \uae30\uad6c\ub85c\uc11c \ud559\uc0dd\ucd1d\ud68c\uc758 \uc18c\uc9d1\uc744 \uc694\uad6c\ud560 \uad8c\ud55c \ubfd0\ub9cc \uc544\ub098\ub77c \ucd1d\ud559\uc0dd\ud68c \ud68c\uce59\uc758 \uac1c\uc815\uacfc \ubc1c\uc758\ub97c \ud655\uc815\ud560 \uad8c\ud55c, \ucd1d\ud559\uc0dd\ud68c\uc758 \uc5c5\ubb34\ub97c \uac10\uc0ac\ud560 \uad8c\ud55c, \ucd1d\ud559\uc0dd\ud68c\uc7a5\ub2e8\uc5d0 \ub300\ud55c \ubd88\uc2e0\uc784\uc744 \uacb0\uc758\ud560 \uad8c\ud55c \ub4f1\uc744 \uac00\uc9c4\ub2e4. \ucd1d\ud559\uc0dd\ud68c\uc7a5\uc740 \ucd1d\ud559\uc0dd\ud68c\ub97c \ub300\ud45c\ud558\uba70 \uc6b4\uc601\uc704\uc6d0\ud68c\uc640 \uc9d1\ud589\ubd80\uc758 \uc7a5\uc774 \ub41c\ub2e4. \ucd1d\ud559\uc0dd\ud68c\uc7a5\uc740 \ucd1d\ud559\uc0dd\ud68c\uc758 \uc0ac\uc5c5 \uacc4\ud68d\uc744 \uc218\ub9bd\ud558\uace0 \ud3b8\uc131\ub41c \uc608\uc0b0\uacfc \uacb0\uc0b0\uc744 \ub300\uc758\uc6d0\ud68c\uc5d0 \uc81c\ucd9c\ud55c\ub2e4. \ub610\ud55c \ucd1d\ud559\uc0dd\ud68c\uc5d0 \uc911\ub300\ud55c \uc601\ud5a5\uc744 \ubbf8\uce58\ub294 \uc0ac\ud56d\uc5d0 \uad00\ud558\uc5ec \ud559\uad50\uc640 \uad00\uacc4 \uae30\uad00\uc5d0 \uc758\uacac\uc11c\ub97c \uc81c\ucd9c\ud560 \uc218 \uc788\ub2e4. \uc6b4\uc601 \uc704\uc6d0\ud68c\ub294 \ucd1d\ud559\uc0dd\ud68c\uc758 \uc2ec\uc758, \uc790\ubb38 \uae30\uad6c\ub85c\uc11c \ucd1d\ud559\uc0dd\ud68c\uc7a5, \ubd80\ud559\uc0dd\ud68c\uc7a5, \ub2e8\uacfc \ub300\ud559 \ud559\uc0dd\ud68c\uc7a5, \ucd1d\ud559\uc0dd\ud68c \uc9d1\ud589\ubd80\uc758 \ubd80\uc7a5\uc73c\ub85c \uad6c\uc131\ub418\uba70, \ucd1d\ud559\uc0dd\ud68c\uac00 \uc6b4\uc601\ud558\ub294 \uc81c\ubc18 \uc0ac\uc5c5, \ucd1d\ud559\uc0dd\ud68c\uc7a5\uc774 \ub300\uc758\uc6d0\ud68c\uc5d0 \uc81c\ucd9c\ud560 \uc608\uc0b0\uacfc \uacb0\uc0b0, \ucd1d\ud559\uc0dd\ud68c\uc7a5\uc758 \ub300\uc758\uc6d0\ud68c\uc5d0 \ub300\ud55c \uc18c\uc9d1 \uc694\uad6c \ub4f1\uc744 \uc2ec\uc758, \uc790\ubb38\ud55c\ub2e4.", "sentence_answer": "\ud559\uc0dd\ucd1d\ud68c\ub294 \ucd1d\ud559\uc0dd\ud68c\uc5d0 \uad00\ub828\ub41c \uc911\ub300\ud55c \uc0ac\ud56d\uc744 \uc758\uacb0\ud558\ub294 \uae30\uad6c\ub85c\uc11c \ub300\uc758\uc6d0\ud68c\uc758 \uc694\uad6c \ub610\ub294 \ucd1d\ud559\uc0dd\ud68c \ud68c\uc6d0 10\ubd84\uc758 1 \uc774\uc0c1\uc758 \uc694\uad6c\uac00 \uc788\uc744 \ub54c \ucd1d\ud559\uc0dd\ud68c\uc7a5 \uc774 \uc18c\uc9d1\ud55c\ub2e4.", "paragraph_id": "6532598-5-2", "paragraph_question": "question: \ucd1d\ud559\uc0dd\ud68c\ub97c \ub300\ud45c\ub85c \uc6b4\uc601\uc704\uc6d0\ud68c\uc640 \uc9d1\ud589\ubd80\uc758 \uc7a5\uc758 \uc5ed\ud560\uc744 \ub9e1\ub294 \uc9c1\ucc45\uc740?, context: \ucd1d\ud559\uc0dd\ud68c\ub294 \uc219\uba85\uc5ec\ub300\uc758 \uc7ac\ud559\uc0dd \ubaa8\uc784\uc774\uba70 \ud559\uc0dd\ucd1d\ud68c, \ub300\uc758\uc6d0\ud68c, \ucd1d\ud559\uc0dd\ud68c, \uc6b4\uc601\uc704\uc6d0\ud68c, \ub2e8\uacfc \ub300\ud559 \ud559\uc0dd\ud68c, \ud559\uacfc\u2219\ud559\ubd80\u2219\uc804\uacf5 \ud559\uc0dd\ud68c, \uae30\ud0c0 \ud559\uc0dd \ubb38\ud654 \ud65c\ub3d9 \ub2e8\uccb4\ub85c \uad6c\uc131\ub41c\ub2e4. \ud559\uc0dd\ucd1d\ud68c\ub294 \ucd1d\ud559\uc0dd\ud68c\uc5d0 \uad00\ub828\ub41c \uc911\ub300\ud55c \uc0ac\ud56d\uc744 \uc758\uacb0\ud558\ub294 \uae30\uad6c\ub85c\uc11c \ub300\uc758\uc6d0\ud68c\uc758 \uc694\uad6c \ub610\ub294 \ucd1d\ud559\uc0dd\ud68c \ud68c\uc6d0 10\ubd84\uc758 1 \uc774\uc0c1\uc758 \uc694\uad6c\uac00 \uc788\uc744 \ub54c \ucd1d\ud559\uc0dd\ud68c\uc7a5\uc774 \uc18c\uc9d1\ud55c\ub2e4. \ub300\uc758\uc6d0\ud68c\ub294 \uc219\uba85\uc5ec\ub300\uc758 \ud559\uacfc\u2219\ud559\ubd80\u2219\uc804\uacf5 \ud559\ud68c\uc7a5\uacfc \ubd80\ud559\ud68c\uc7a5\ub85c \uad6c\uc131\ub418\ub294 \uae30\uad6c\ub85c\uc11c \ud559\uc0dd\ucd1d\ud68c\uc758 \uc18c\uc9d1\uc744 \uc694\uad6c\ud560 \uad8c\ud55c \ubfd0\ub9cc \uc544\ub098\ub77c \ucd1d\ud559\uc0dd\ud68c \ud68c\uce59\uc758 \uac1c\uc815\uacfc \ubc1c\uc758\ub97c \ud655\uc815\ud560 \uad8c\ud55c, \ucd1d\ud559\uc0dd\ud68c\uc758 \uc5c5\ubb34\ub97c \uac10\uc0ac\ud560 \uad8c\ud55c, \ucd1d\ud559\uc0dd\ud68c\uc7a5\ub2e8\uc5d0 \ub300\ud55c \ubd88\uc2e0\uc784\uc744 \uacb0\uc758\ud560 \uad8c\ud55c \ub4f1\uc744 \uac00\uc9c4\ub2e4. \ucd1d\ud559\uc0dd\ud68c\uc7a5\uc740 \ucd1d\ud559\uc0dd\ud68c\ub97c \ub300\ud45c\ud558\uba70 \uc6b4\uc601\uc704\uc6d0\ud68c\uc640 \uc9d1\ud589\ubd80\uc758 \uc7a5\uc774 \ub41c\ub2e4. \ucd1d\ud559\uc0dd\ud68c\uc7a5\uc740 \ucd1d\ud559\uc0dd\ud68c\uc758 \uc0ac\uc5c5 \uacc4\ud68d\uc744 \uc218\ub9bd\ud558\uace0 \ud3b8\uc131\ub41c \uc608\uc0b0\uacfc \uacb0\uc0b0\uc744 \ub300\uc758\uc6d0\ud68c\uc5d0 \uc81c\ucd9c\ud55c\ub2e4. \ub610\ud55c \ucd1d\ud559\uc0dd\ud68c\uc5d0 \uc911\ub300\ud55c \uc601\ud5a5\uc744 \ubbf8\uce58\ub294 \uc0ac\ud56d\uc5d0 \uad00\ud558\uc5ec \ud559\uad50\uc640 \uad00\uacc4 \uae30\uad00\uc5d0 \uc758\uacac\uc11c\ub97c \uc81c\ucd9c\ud560 \uc218 \uc788\ub2e4. \uc6b4\uc601 \uc704\uc6d0\ud68c\ub294 \ucd1d\ud559\uc0dd\ud68c\uc758 \uc2ec\uc758, \uc790\ubb38 \uae30\uad6c\ub85c\uc11c \ucd1d\ud559\uc0dd\ud68c\uc7a5, \ubd80\ud559\uc0dd\ud68c\uc7a5, \ub2e8\uacfc \ub300\ud559 \ud559\uc0dd\ud68c\uc7a5, \ucd1d\ud559\uc0dd\ud68c \uc9d1\ud589\ubd80\uc758 \ubd80\uc7a5\uc73c\ub85c \uad6c\uc131\ub418\uba70, \ucd1d\ud559\uc0dd\ud68c\uac00 \uc6b4\uc601\ud558\ub294 \uc81c\ubc18 \uc0ac\uc5c5, \ucd1d\ud559\uc0dd\ud68c\uc7a5\uc774 \ub300\uc758\uc6d0\ud68c\uc5d0 \uc81c\ucd9c\ud560 \uc608\uc0b0\uacfc \uacb0\uc0b0, \ucd1d\ud559\uc0dd\ud68c\uc7a5\uc758 \ub300\uc758\uc6d0\ud68c\uc5d0 \ub300\ud55c \uc18c\uc9d1 \uc694\uad6c \ub4f1\uc744 \uc2ec\uc758, \uc790\ubb38\ud55c\ub2e4."} {"question": "\ubbf8\ucf08 \uc544\ub974\ud14c\ud0c0\uc758 \ud3c9\uade0 \ud328\uc2a4 \uc131\uacf5\ub960\uc740?", "paragraph": "\ud504\ub9ac\uc2dc\uc98c \uc88b\uc740 \uacbd\uae30\ub825\uc744 \ud3bc\uce58\uba74\uc11c \uc544\uc2a4\ub0a0 \ud32c\ub4e4\uc5d0\uac8c \uae30\ub300\uac10\uc744 \uc548\uacbc\ub2e4. \ube44\ub85d \uc2dc\uc98c \ucd08\ubc18\uc774\uae34 \ud558\uc9c0\ub9cc \ub9ac\uadf8 4\uacbd\uae30 2\uace8, \ucc54\ud53c\uc5b8\uc2a4\ub9ac\uadf8 3\uacbd\uae30 3\uace8\uc744 \uae30\ub85d\ud558\uba70 \uc62c\ub9ac\ube44\uc5d0 \uc9c0\ub8e8\uc640 \ud568\uaed8 \ud300 \ub0b4 \ub4dd\uc810 1\uc704\ub97c \ub2ec\ub9ac\uace0 \uc788\ub2e4. \ub9c8\ud2f0\uc678 \ud50c\ub77c\ubbf8\ub2c8\uac00 \uc544\uc2a4\ub0a0\uc5d0 \uc601\uc785\ub418\uba74\uc11c \uc5c4\uccad\ub09c \ud65c\ub3d9\ub7c9\uc744 \uac16\ucd98 \ub450 \ubbf8\ub4dc\ud544\ub354\uac00 \uc544\uc2a4\ub0a0\uc758 \uc0c1\uc2b9\uc138\ub97c \uc774\ub04c\uace0 \uc788\ub2e4. 11-12 \uc2dc\uc98c\uc5d0 \uc9c0\uc801\ub418\uc5c8\ub358 \ub2e8\uc810\ub4e4\uc744 \uac1c\uc120\ud588\ub294\ub370 \uc774\ub294 \ubd80\uc815\ud655\ud55c \ud328\uc2a4\uc640 \uacbd\uae30 \ud15c\ud3ec \uc870\uc808 \uc2e4\ud328, \uadf8\ub9ac\uace0 \ub4dc\ub9ac\ube14\uc5d0 \uc790\uc2e0\uac10\uc774 \uc5c6\ub358 \ubd80\ubd84\uc774\uc5c8\ub2e4. 12-13 \uc2dc\uc98c \ubbf8\ucf08 \uc544\ub974\ud14c\ud0c0\uac00 \uc218\ud589\ud558\ub358 \ub525-\ub77c\uc789 \ud50c\ub808\uc774\uba54\uc774\ucee4 \uc5ed\ud560\uc744 \ub9e1\uc73c\uba74\uc11c 87.3%\uc758 \uc815\ud655\ub3c4\uc600\ub358 \ud328\uc2a4 \uc815\ud655\ub3c4\ub97c 88.2%\uae4c\uc9c0 \ub04c\uc5b4\uc62c\ub838\uc73c\uba70, \ubbf8\ucf08 \uc544\ub974\ud14c\ud0c0\uac00 \ubd80\uc7ac\ud560 \ub54c\uc5d4 \uadf8\uac00 \ud3c9\uade0\uc801\uc73c\ub85c \uae30\ub85d\ud558\ub358 91% \uc815\ub3c4\uc758 \ud328\uc2a4 \uc131\uacf5\ub960\uc744 \uae30\ub85d\ud558\uc600\ub2e4. \ubd80\uc0c1 \uc774\ud6c4 \uacbd\uae30 \ud15c\ud3ec \uc870\uc808\uacfc \ubbf8\uc219\ud55c \ub4dc\ub9ac\ube14 \uc2e4\ub825\uc740 \uc544\ub974\uc13c \ubcb5\uac70 \uac10\ub3c5\uc774 \uc88c\uc6b0 \uc719\uc5b4\ub85c \uae30\uc6a9\ud558\uba74\uc11c \uc544\uc2a4\ub0a0\uc758 \ud15c\ud3ec\uc5d0 \uc801\uc751 \uc2dc\ud0a4\ub294 \ub3d9\uc2dc\uc5d0 \ub4dc\ub9ac\ube14\uc5d0 \uc790\uc2e0\uac10\uc744 \uac00\uc838\uc624\uac8c \ud558\uc600\ub2e4. \ud1a0\ub9c8\uc2dc \ub85c\uc2dc\uce20\ud0a4\ub54c\uc640 \ub9c8\ucc2c\uac00\uc9c0\ub85c \uc719\uc5b4\ub85c \ubcc0\uc2e0\ud558\ub294\ub370\ub294 \uc2e4\ud328\ud558\uc600\uc9c0\ub9cc, \ubbf8\ub4dc\ud544\ub354\uc5d0\uc11c\uc758 \uc790\uc2e0\uac10\uc744 \ud5a5\uc0c1\uc2dc\ud0a8 \ub3d9\uc2dc\uc5d0, \ud0c8\uc555\ubc15 \ub2a5\ub825\uc744 \uae30\ub974\ub294\ub370\ub294 \ud070 \ub3c4\uc6c0\uc774 \ub418\uc5c8\ub2e4. \ub9ac\uadf8\uc5d0\uc11c\uc758 \uc120\ub35c\ub79c\ub4dc AFC\uc640\uc758 \uacbd\uae30\uc5d0\uc11c\ub294 111\ud68c\uc758 \ud130\uce58\ub97c \uae30\ub85d\ud558\uc600\uace0, \uacbd\uae30 \ub0b4 \uc591\ud300 \ud1b5\ud2c0\uc5b4 \ucd5c\ub2e4 \ud328\uc2a4\uc778 88\uac1c\uc758 \ud328\uc2a4\ub97c \ud558\uc600\uc73c\uba70 \uadf8 \uc911 81\uac1c\ub97c \uc131\uacf5\uc2dc\ud0a4\uba74\uc11c 92%\uc758 \uc131\uacf5\ub960\uc744 \ubcf4\uc600\ub2e4. \ub610\ud55c 2014\ub144 5\uc6d4 18\uc77c \uc601\uad6d \uc6f8\ube14\ub9ac \uc2a4\ud0c0\ub514\uc6c0\uc5d0\uc11c \uc5f4\ub9b0 \uc6e8\uc2a4\ud2b8\ud584 \uc720\ub098\uc774\ud2f0\ub4dc\uc640\uc758 2013-14 \uc789\uae00\ub79c\ub4dc FA\ucef5 \uacb0\uc2b9\uc804\uc5d0\uc11c \uc5f0\uc7a5 \ud6c4\ubc18 4\ubd84 \uc2a4\ucf54\uc5b4 2:2\uc0c1\ud669\uc5d0\uc11c \uc5ed\uc804 \uacb0\uc2b9\uace8\uc744 \ubf51\uc544\ub0b4\uba70 \ud300\uc758 9\ub144 \ubb34\uad00\uc744 \ub04a\ub294\ub370 \ud070 \uacf5\ud5cc\uc744 \ud558\uc600\ub2e4.", "answer": "91", "sentence": "12-13 \uc2dc\uc98c \ubbf8\ucf08 \uc544\ub974\ud14c\ud0c0\uac00 \uc218\ud589\ud558\ub358 \ub525-\ub77c\uc789 \ud50c\ub808\uc774\uba54\uc774\ucee4 \uc5ed\ud560\uc744 \ub9e1\uc73c\uba74\uc11c 87.3%\uc758 \uc815\ud655\ub3c4\uc600\ub358 \ud328\uc2a4 \uc815\ud655\ub3c4\ub97c 88.2%\uae4c\uc9c0 \ub04c\uc5b4\uc62c\ub838\uc73c\uba70, \ubbf8\ucf08 \uc544\ub974\ud14c\ud0c0\uac00 \ubd80\uc7ac\ud560 \ub54c\uc5d4 \uadf8\uac00 \ud3c9\uade0\uc801\uc73c\ub85c \uae30\ub85d\ud558\ub358 91 %", "paragraph_sentence": "\ud504\ub9ac\uc2dc\uc98c \uc88b\uc740 \uacbd\uae30\ub825\uc744 \ud3bc\uce58\uba74\uc11c \uc544\uc2a4\ub0a0 \ud32c\ub4e4\uc5d0\uac8c \uae30\ub300\uac10\uc744 \uc548\uacbc\ub2e4. \ube44\ub85d \uc2dc\uc98c \ucd08\ubc18\uc774\uae34 \ud558\uc9c0\ub9cc \ub9ac\uadf8 4\uacbd\uae30 2\uace8, \ucc54\ud53c\uc5b8\uc2a4\ub9ac\uadf8 3\uacbd\uae30 3\uace8\uc744 \uae30\ub85d\ud558\uba70 \uc62c\ub9ac\ube44\uc5d0 \uc9c0\ub8e8\uc640 \ud568\uaed8 \ud300 \ub0b4 \ub4dd\uc810 1\uc704\ub97c \ub2ec\ub9ac\uace0 \uc788\ub2e4. \ub9c8\ud2f0\uc678 \ud50c\ub77c\ubbf8\ub2c8\uac00 \uc544\uc2a4\ub0a0\uc5d0 \uc601\uc785\ub418\uba74\uc11c \uc5c4\uccad\ub09c \ud65c\ub3d9\ub7c9\uc744 \uac16\ucd98 \ub450 \ubbf8\ub4dc\ud544\ub354\uac00 \uc544\uc2a4\ub0a0\uc758 \uc0c1\uc2b9\uc138\ub97c \uc774\ub04c\uace0 \uc788\ub2e4. 11-12 \uc2dc\uc98c\uc5d0 \uc9c0\uc801\ub418\uc5c8\ub358 \ub2e8\uc810\ub4e4\uc744 \uac1c\uc120\ud588\ub294\ub370 \uc774\ub294 \ubd80\uc815\ud655\ud55c \ud328\uc2a4\uc640 \uacbd\uae30 \ud15c\ud3ec \uc870\uc808 \uc2e4\ud328, \uadf8\ub9ac\uace0 \ub4dc\ub9ac\ube14\uc5d0 \uc790\uc2e0\uac10\uc774 \uc5c6\ub358 \ubd80\ubd84\uc774\uc5c8\ub2e4. 12-13 \uc2dc\uc98c \ubbf8\ucf08 \uc544\ub974\ud14c\ud0c0\uac00 \uc218\ud589\ud558\ub358 \ub525-\ub77c\uc789 \ud50c\ub808\uc774\uba54\uc774\ucee4 \uc5ed\ud560\uc744 \ub9e1\uc73c\uba74\uc11c 87.3%\uc758 \uc815\ud655\ub3c4\uc600\ub358 \ud328\uc2a4 \uc815\ud655\ub3c4\ub97c 88.2%\uae4c\uc9c0 \ub04c\uc5b4\uc62c\ub838\uc73c\uba70, \ubbf8\ucf08 \uc544\ub974\ud14c\ud0c0\uac00 \ubd80\uc7ac\ud560 \ub54c\uc5d4 \uadf8\uac00 \ud3c9\uade0\uc801\uc73c\ub85c \uae30\ub85d\ud558\ub358 91 % \uc815\ub3c4\uc758 \ud328\uc2a4 \uc131\uacf5\ub960\uc744 \uae30\ub85d\ud558\uc600\ub2e4. \ubd80\uc0c1 \uc774\ud6c4 \uacbd\uae30 \ud15c\ud3ec \uc870\uc808\uacfc \ubbf8\uc219\ud55c \ub4dc\ub9ac\ube14 \uc2e4\ub825\uc740 \uc544\ub974\uc13c \ubcb5\uac70 \uac10\ub3c5\uc774 \uc88c\uc6b0 \uc719\uc5b4\ub85c \uae30\uc6a9\ud558\uba74\uc11c \uc544\uc2a4\ub0a0\uc758 \ud15c\ud3ec\uc5d0 \uc801\uc751 \uc2dc\ud0a4\ub294 \ub3d9\uc2dc\uc5d0 \ub4dc\ub9ac\ube14\uc5d0 \uc790\uc2e0\uac10\uc744 \uac00\uc838\uc624\uac8c \ud558\uc600\ub2e4. \ud1a0\ub9c8\uc2dc \ub85c\uc2dc\uce20\ud0a4\ub54c\uc640 \ub9c8\ucc2c\uac00\uc9c0\ub85c \uc719\uc5b4\ub85c \ubcc0\uc2e0\ud558\ub294\ub370\ub294 \uc2e4\ud328\ud558\uc600\uc9c0\ub9cc, \ubbf8\ub4dc\ud544\ub354\uc5d0\uc11c\uc758 \uc790\uc2e0\uac10\uc744 \ud5a5\uc0c1\uc2dc\ud0a8 \ub3d9\uc2dc\uc5d0, \ud0c8\uc555\ubc15 \ub2a5\ub825\uc744 \uae30\ub974\ub294\ub370\ub294 \ud070 \ub3c4\uc6c0\uc774 \ub418\uc5c8\ub2e4. \ub9ac\uadf8\uc5d0\uc11c\uc758 \uc120\ub35c\ub79c\ub4dc AFC\uc640\uc758 \uacbd\uae30\uc5d0\uc11c\ub294 111\ud68c\uc758 \ud130\uce58\ub97c \uae30\ub85d\ud558\uc600\uace0, \uacbd\uae30 \ub0b4 \uc591\ud300 \ud1b5\ud2c0\uc5b4 \ucd5c\ub2e4 \ud328\uc2a4\uc778 88\uac1c\uc758 \ud328\uc2a4\ub97c \ud558\uc600\uc73c\uba70 \uadf8 \uc911 81\uac1c\ub97c \uc131\uacf5\uc2dc\ud0a4\uba74\uc11c 92%\uc758 \uc131\uacf5\ub960\uc744 \ubcf4\uc600\ub2e4. \ub610\ud55c 2014\ub144 5\uc6d4 18\uc77c \uc601\uad6d \uc6f8\ube14\ub9ac \uc2a4\ud0c0\ub514\uc6c0\uc5d0\uc11c \uc5f4\ub9b0 \uc6e8\uc2a4\ud2b8\ud584 \uc720\ub098\uc774\ud2f0\ub4dc\uc640\uc758 2013-14 \uc789\uae00\ub79c\ub4dc FA\ucef5 \uacb0\uc2b9\uc804\uc5d0\uc11c \uc5f0\uc7a5 \ud6c4\ubc18 4\ubd84 \uc2a4\ucf54\uc5b4 2:2\uc0c1\ud669\uc5d0\uc11c \uc5ed\uc804 \uacb0\uc2b9\uace8\uc744 \ubf51\uc544\ub0b4\uba70 \ud300\uc758 9\ub144 \ubb34\uad00\uc744 \ub04a\ub294\ub370 \ud070 \uacf5\ud5cc\uc744 \ud558\uc600\ub2e4.", "paragraph_answer": "\ud504\ub9ac\uc2dc\uc98c \uc88b\uc740 \uacbd\uae30\ub825\uc744 \ud3bc\uce58\uba74\uc11c \uc544\uc2a4\ub0a0 \ud32c\ub4e4\uc5d0\uac8c \uae30\ub300\uac10\uc744 \uc548\uacbc\ub2e4. \ube44\ub85d \uc2dc\uc98c \ucd08\ubc18\uc774\uae34 \ud558\uc9c0\ub9cc \ub9ac\uadf8 4\uacbd\uae30 2\uace8, \ucc54\ud53c\uc5b8\uc2a4\ub9ac\uadf8 3\uacbd\uae30 3\uace8\uc744 \uae30\ub85d\ud558\uba70 \uc62c\ub9ac\ube44\uc5d0 \uc9c0\ub8e8\uc640 \ud568\uaed8 \ud300 \ub0b4 \ub4dd\uc810 1\uc704\ub97c \ub2ec\ub9ac\uace0 \uc788\ub2e4. \ub9c8\ud2f0\uc678 \ud50c\ub77c\ubbf8\ub2c8\uac00 \uc544\uc2a4\ub0a0\uc5d0 \uc601\uc785\ub418\uba74\uc11c \uc5c4\uccad\ub09c \ud65c\ub3d9\ub7c9\uc744 \uac16\ucd98 \ub450 \ubbf8\ub4dc\ud544\ub354\uac00 \uc544\uc2a4\ub0a0\uc758 \uc0c1\uc2b9\uc138\ub97c \uc774\ub04c\uace0 \uc788\ub2e4. 11-12 \uc2dc\uc98c\uc5d0 \uc9c0\uc801\ub418\uc5c8\ub358 \ub2e8\uc810\ub4e4\uc744 \uac1c\uc120\ud588\ub294\ub370 \uc774\ub294 \ubd80\uc815\ud655\ud55c \ud328\uc2a4\uc640 \uacbd\uae30 \ud15c\ud3ec \uc870\uc808 \uc2e4\ud328, \uadf8\ub9ac\uace0 \ub4dc\ub9ac\ube14\uc5d0 \uc790\uc2e0\uac10\uc774 \uc5c6\ub358 \ubd80\ubd84\uc774\uc5c8\ub2e4. 12-13 \uc2dc\uc98c \ubbf8\ucf08 \uc544\ub974\ud14c\ud0c0\uac00 \uc218\ud589\ud558\ub358 \ub525-\ub77c\uc789 \ud50c\ub808\uc774\uba54\uc774\ucee4 \uc5ed\ud560\uc744 \ub9e1\uc73c\uba74\uc11c 87.3%\uc758 \uc815\ud655\ub3c4\uc600\ub358 \ud328\uc2a4 \uc815\ud655\ub3c4\ub97c 88.2%\uae4c\uc9c0 \ub04c\uc5b4\uc62c\ub838\uc73c\uba70, \ubbf8\ucf08 \uc544\ub974\ud14c\ud0c0\uac00 \ubd80\uc7ac\ud560 \ub54c\uc5d4 \uadf8\uac00 \ud3c9\uade0\uc801\uc73c\ub85c \uae30\ub85d\ud558\ub358 91 % \uc815\ub3c4\uc758 \ud328\uc2a4 \uc131\uacf5\ub960\uc744 \uae30\ub85d\ud558\uc600\ub2e4. \ubd80\uc0c1 \uc774\ud6c4 \uacbd\uae30 \ud15c\ud3ec \uc870\uc808\uacfc \ubbf8\uc219\ud55c \ub4dc\ub9ac\ube14 \uc2e4\ub825\uc740 \uc544\ub974\uc13c \ubcb5\uac70 \uac10\ub3c5\uc774 \uc88c\uc6b0 \uc719\uc5b4\ub85c \uae30\uc6a9\ud558\uba74\uc11c \uc544\uc2a4\ub0a0\uc758 \ud15c\ud3ec\uc5d0 \uc801\uc751 \uc2dc\ud0a4\ub294 \ub3d9\uc2dc\uc5d0 \ub4dc\ub9ac\ube14\uc5d0 \uc790\uc2e0\uac10\uc744 \uac00\uc838\uc624\uac8c \ud558\uc600\ub2e4. \ud1a0\ub9c8\uc2dc \ub85c\uc2dc\uce20\ud0a4\ub54c\uc640 \ub9c8\ucc2c\uac00\uc9c0\ub85c \uc719\uc5b4\ub85c \ubcc0\uc2e0\ud558\ub294\ub370\ub294 \uc2e4\ud328\ud558\uc600\uc9c0\ub9cc, \ubbf8\ub4dc\ud544\ub354\uc5d0\uc11c\uc758 \uc790\uc2e0\uac10\uc744 \ud5a5\uc0c1\uc2dc\ud0a8 \ub3d9\uc2dc\uc5d0, \ud0c8\uc555\ubc15 \ub2a5\ub825\uc744 \uae30\ub974\ub294\ub370\ub294 \ud070 \ub3c4\uc6c0\uc774 \ub418\uc5c8\ub2e4. \ub9ac\uadf8\uc5d0\uc11c\uc758 \uc120\ub35c\ub79c\ub4dc AFC\uc640\uc758 \uacbd\uae30\uc5d0\uc11c\ub294 111\ud68c\uc758 \ud130\uce58\ub97c \uae30\ub85d\ud558\uc600\uace0, \uacbd\uae30 \ub0b4 \uc591\ud300 \ud1b5\ud2c0\uc5b4 \ucd5c\ub2e4 \ud328\uc2a4\uc778 88\uac1c\uc758 \ud328\uc2a4\ub97c \ud558\uc600\uc73c\uba70 \uadf8 \uc911 81\uac1c\ub97c \uc131\uacf5\uc2dc\ud0a4\uba74\uc11c 92%\uc758 \uc131\uacf5\ub960\uc744 \ubcf4\uc600\ub2e4. \ub610\ud55c 2014\ub144 5\uc6d4 18\uc77c \uc601\uad6d \uc6f8\ube14\ub9ac \uc2a4\ud0c0\ub514\uc6c0\uc5d0\uc11c \uc5f4\ub9b0 \uc6e8\uc2a4\ud2b8\ud584 \uc720\ub098\uc774\ud2f0\ub4dc\uc640\uc758 2013-14 \uc789\uae00\ub79c\ub4dc FA\ucef5 \uacb0\uc2b9\uc804\uc5d0\uc11c \uc5f0\uc7a5 \ud6c4\ubc18 4\ubd84 \uc2a4\ucf54\uc5b4 2:2\uc0c1\ud669\uc5d0\uc11c \uc5ed\uc804 \uacb0\uc2b9\uace8\uc744 \ubf51\uc544\ub0b4\uba70 \ud300\uc758 9\ub144 \ubb34\uad00\uc744 \ub04a\ub294\ub370 \ud070 \uacf5\ud5cc\uc744 \ud558\uc600\ub2e4.", "sentence_answer": "12-13 \uc2dc\uc98c \ubbf8\ucf08 \uc544\ub974\ud14c\ud0c0\uac00 \uc218\ud589\ud558\ub358 \ub525-\ub77c\uc789 \ud50c\ub808\uc774\uba54\uc774\ucee4 \uc5ed\ud560\uc744 \ub9e1\uc73c\uba74\uc11c 87.3%\uc758 \uc815\ud655\ub3c4\uc600\ub358 \ud328\uc2a4 \uc815\ud655\ub3c4\ub97c 88.2%\uae4c\uc9c0 \ub04c\uc5b4\uc62c\ub838\uc73c\uba70, \ubbf8\ucf08 \uc544\ub974\ud14c\ud0c0\uac00 \ubd80\uc7ac\ud560 \ub54c\uc5d4 \uadf8\uac00 \ud3c9\uade0\uc801\uc73c\ub85c \uae30\ub85d\ud558\ub358 91 %", "paragraph_id": "6586552-7-2", "paragraph_question": "question: \ubbf8\ucf08 \uc544\ub974\ud14c\ud0c0\uc758 \ud3c9\uade0 \ud328\uc2a4 \uc131\uacf5\ub960\uc740?, context: \ud504\ub9ac\uc2dc\uc98c \uc88b\uc740 \uacbd\uae30\ub825\uc744 \ud3bc\uce58\uba74\uc11c \uc544\uc2a4\ub0a0 \ud32c\ub4e4\uc5d0\uac8c \uae30\ub300\uac10\uc744 \uc548\uacbc\ub2e4. \ube44\ub85d \uc2dc\uc98c \ucd08\ubc18\uc774\uae34 \ud558\uc9c0\ub9cc \ub9ac\uadf8 4\uacbd\uae30 2\uace8, \ucc54\ud53c\uc5b8\uc2a4\ub9ac\uadf8 3\uacbd\uae30 3\uace8\uc744 \uae30\ub85d\ud558\uba70 \uc62c\ub9ac\ube44\uc5d0 \uc9c0\ub8e8\uc640 \ud568\uaed8 \ud300 \ub0b4 \ub4dd\uc810 1\uc704\ub97c \ub2ec\ub9ac\uace0 \uc788\ub2e4. \ub9c8\ud2f0\uc678 \ud50c\ub77c\ubbf8\ub2c8\uac00 \uc544\uc2a4\ub0a0\uc5d0 \uc601\uc785\ub418\uba74\uc11c \uc5c4\uccad\ub09c \ud65c\ub3d9\ub7c9\uc744 \uac16\ucd98 \ub450 \ubbf8\ub4dc\ud544\ub354\uac00 \uc544\uc2a4\ub0a0\uc758 \uc0c1\uc2b9\uc138\ub97c \uc774\ub04c\uace0 \uc788\ub2e4. 11-12 \uc2dc\uc98c\uc5d0 \uc9c0\uc801\ub418\uc5c8\ub358 \ub2e8\uc810\ub4e4\uc744 \uac1c\uc120\ud588\ub294\ub370 \uc774\ub294 \ubd80\uc815\ud655\ud55c \ud328\uc2a4\uc640 \uacbd\uae30 \ud15c\ud3ec \uc870\uc808 \uc2e4\ud328, \uadf8\ub9ac\uace0 \ub4dc\ub9ac\ube14\uc5d0 \uc790\uc2e0\uac10\uc774 \uc5c6\ub358 \ubd80\ubd84\uc774\uc5c8\ub2e4. 12-13 \uc2dc\uc98c \ubbf8\ucf08 \uc544\ub974\ud14c\ud0c0\uac00 \uc218\ud589\ud558\ub358 \ub525-\ub77c\uc789 \ud50c\ub808\uc774\uba54\uc774\ucee4 \uc5ed\ud560\uc744 \ub9e1\uc73c\uba74\uc11c 87.3%\uc758 \uc815\ud655\ub3c4\uc600\ub358 \ud328\uc2a4 \uc815\ud655\ub3c4\ub97c 88.2%\uae4c\uc9c0 \ub04c\uc5b4\uc62c\ub838\uc73c\uba70, \ubbf8\ucf08 \uc544\ub974\ud14c\ud0c0\uac00 \ubd80\uc7ac\ud560 \ub54c\uc5d4 \uadf8\uac00 \ud3c9\uade0\uc801\uc73c\ub85c \uae30\ub85d\ud558\ub358 91% \uc815\ub3c4\uc758 \ud328\uc2a4 \uc131\uacf5\ub960\uc744 \uae30\ub85d\ud558\uc600\ub2e4. \ubd80\uc0c1 \uc774\ud6c4 \uacbd\uae30 \ud15c\ud3ec \uc870\uc808\uacfc \ubbf8\uc219\ud55c \ub4dc\ub9ac\ube14 \uc2e4\ub825\uc740 \uc544\ub974\uc13c \ubcb5\uac70 \uac10\ub3c5\uc774 \uc88c\uc6b0 \uc719\uc5b4\ub85c \uae30\uc6a9\ud558\uba74\uc11c \uc544\uc2a4\ub0a0\uc758 \ud15c\ud3ec\uc5d0 \uc801\uc751 \uc2dc\ud0a4\ub294 \ub3d9\uc2dc\uc5d0 \ub4dc\ub9ac\ube14\uc5d0 \uc790\uc2e0\uac10\uc744 \uac00\uc838\uc624\uac8c \ud558\uc600\ub2e4. \ud1a0\ub9c8\uc2dc \ub85c\uc2dc\uce20\ud0a4\ub54c\uc640 \ub9c8\ucc2c\uac00\uc9c0\ub85c \uc719\uc5b4\ub85c \ubcc0\uc2e0\ud558\ub294\ub370\ub294 \uc2e4\ud328\ud558\uc600\uc9c0\ub9cc, \ubbf8\ub4dc\ud544\ub354\uc5d0\uc11c\uc758 \uc790\uc2e0\uac10\uc744 \ud5a5\uc0c1\uc2dc\ud0a8 \ub3d9\uc2dc\uc5d0, \ud0c8\uc555\ubc15 \ub2a5\ub825\uc744 \uae30\ub974\ub294\ub370\ub294 \ud070 \ub3c4\uc6c0\uc774 \ub418\uc5c8\ub2e4. \ub9ac\uadf8\uc5d0\uc11c\uc758 \uc120\ub35c\ub79c\ub4dc AFC\uc640\uc758 \uacbd\uae30\uc5d0\uc11c\ub294 111\ud68c\uc758 \ud130\uce58\ub97c \uae30\ub85d\ud558\uc600\uace0, \uacbd\uae30 \ub0b4 \uc591\ud300 \ud1b5\ud2c0\uc5b4 \ucd5c\ub2e4 \ud328\uc2a4\uc778 88\uac1c\uc758 \ud328\uc2a4\ub97c \ud558\uc600\uc73c\uba70 \uadf8 \uc911 81\uac1c\ub97c \uc131\uacf5\uc2dc\ud0a4\uba74\uc11c 92%\uc758 \uc131\uacf5\ub960\uc744 \ubcf4\uc600\ub2e4. \ub610\ud55c 2014\ub144 5\uc6d4 18\uc77c \uc601\uad6d \uc6f8\ube14\ub9ac \uc2a4\ud0c0\ub514\uc6c0\uc5d0\uc11c \uc5f4\ub9b0 \uc6e8\uc2a4\ud2b8\ud584 \uc720\ub098\uc774\ud2f0\ub4dc\uc640\uc758 2013-14 \uc789\uae00\ub79c\ub4dc FA\ucef5 \uacb0\uc2b9\uc804\uc5d0\uc11c \uc5f0\uc7a5 \ud6c4\ubc18 4\ubd84 \uc2a4\ucf54\uc5b4 2:2\uc0c1\ud669\uc5d0\uc11c \uc5ed\uc804 \uacb0\uc2b9\uace8\uc744 \ubf51\uc544\ub0b4\uba70 \ud300\uc758 9\ub144 \ubb34\uad00\uc744 \ub04a\ub294\ub370 \ud070 \uacf5\ud5cc\uc744 \ud558\uc600\ub2e4."} {"question": "\ub308\ub7ec\uc2a4 \ubaa8\ub2dd \ub274\uc2a4\uc758 \ud06c\ub9ac\uc2a4 \ubd09\uac70\ub294 \uc5b4\ub5a4 \ud3c9\uac00\ub97c \ub0b4\ub838\ub098?", "paragraph": "\ud0c0\uc784\uc758 \ub9ac\ucc98\ub4dc \ucf5c\ub9ac\uc2a4\ub294 \"\uc601\ud654 \uc18d \ud5e4\uc774\uc990\uacfc \uc5b4\uac70\uc2a4\ud130\uc2a4\ub294 \ub208 \uc55e\uc5d0 \ubcf4\uc774\ub294 \uac83\ub4e4\uc744 \uc0dd\uac01\ud558\uba70, \uc0dd\uae30\uac00 \uc788\ub2e4. \uc774 \uc810\uc740 \ub450 \uc0ac\ub78c\uc744 \uc5f0\uae30\ud558\uace0 \uc788\ub294 \ubc30\uc6b0\uac00 \uc80a\uae30 \ub54c\ubb38\uc77c \uac83\uc774\ub2e4.\"\uace0 \ub9d0\ud588\ub2e4. \ub308\ub7ec\uc2a4 \ubaa8\ub2dd \ub274\uc2a4\uc758 \ud06c\ub9ac\uc2a4 \ubd09\uac70\ub294 B+\ub77c\ub294 \ud3c9\uac00\ub97c \ub0b4\ub838\ub2e4 \"\u300a\uc548\ub155, \ud5e4\uc774\uc990\u300b\uc740 \uc7ac\uce58\uc788\uace0 \uc0dd\uae30 \uc788\ub2e4. \ub2e8\uc9c0 \uc870\uae08 \uac10\uc0c1\uc801\uc774\ub2e4.\"\uace0 \ud588\ub2e4. USA \ud22c\ub370\uc774\uc758 \ud074\ub77c\uc6b0\ub514\uc544 \ud478\uc774\uadf8\ub294 4\uc810 \uc911 3.5\uc810\uc744 \uc8fc\uace0, \"\uc88b\uc740 \uac01\ubcf8\uc774\uba70 \uc5f0\uae30\ub3c4 \uc88b\ub2e4. \uc7ac\ubbf8\ub3c4 \uc788\uace0 \ud55c\ud3b8\uc73c\ub85c\ub294 \ud1b5\ucc30\ub825\uc774 \uc788\ub2e4.\"\uace0 \ud3c9\ud558\uc600\ub2e4. \uc5d4\ud130\ud14c\uc778\uba3c\ud2b8 \uc704\ud074\ub9ac\uc758 \ud06c\ub9ac\uc2a4 \ub0b4\uc154\uc6e8\uc774\ud2f0\ub294 \"\uc7ac\ubbf8\uc788\uace0 \ub2ec\ucf64\ud55c \ub9e4\uc6b0 \uac10\ub3d9\uc2a4\ub7ec\uc6b4 \uc601\ud654\"\ub77c\uba70 B \ud3c9\uac00\ub97c \ub0b4\ub838\ub2e4. \uae00\ub85c\ube0c \uc564 \uba54\uc77c\uc758 \uc81c\ud504 \ud398\ubca0\ub808\ub294 \"\uc774 \uc791\ud488\uc740 \uad00\uac1d\uc774 \uc2e4\uc81c \uc0dd\ud65c\uc5d0\uc11c \uac00\uc9c0\ub294 \uae30\ub300\ub97c \ubc30\ubc18\ud560\uc9c0\ub3c4 \ubaa8\ub978\ub2e4. \ud558\uc9c0\ub9cc \uccad\uc18c\ub144\uc744 \uc704\ud55c \uc601\ud654\uc758 \uad00\uc2b5\uc744 \uc2f9\ub451 \uc790\ub974\uace0 \uc788\ub2e4.\"\uace0 \ud558\uc600\ub2e4. \ub9c8\uc774\uc560\ubbf8 \ud5e4\ub7f4\ub4dc\uc758 \ucf54\ub2c8 \uc624\uae00\uc740 \"\uac11\uc790\uae30 \ub530\ub73b\ud574\uc9c0\uac70\ub098 \uc720\uba38\uac00 \uc788\ub294 \ub2ec\ucf64\ud55c \ub85c\ub9e8\uc2a4 \uc601\ud654\ub2e4.\"\ub77c\uace0 \ud558\uc600\ub2e4. \ub514 A.V. \ud074\ub7fd\uc758 A. A. \ub2e4\uc6b0\ub4dc\ub294 B \ud3c9\uac00\ub97c \ub0b4\ub9ac\uba70, \"\ubc88\uac1c \uac19\uc740 \uc704\ud2b8\uac00 \uc788\uc73c\uba70, \uac1c\uc131 \ud48d\ubd80\ud558\uace0 \uc7ac\ub2a5\uc788\ub294 \ubc30\uc6b0\ub4e4\uc5d0 \uc758\ud574 \ub9e4\ub825\uc774 \uc99d\uac00\ud55c\ub2e4.\"\uace0 \ub9d0\ud588\ub2e4. \ub864\ub9c1 \uc2a4\ud1a4\uc758 \ud53c\ud130 \ud2b8\ub798\ubc84\uc2a4\ub294 \"\uc2e0\uc120\ud558\uace0 \uc0dd\uc0dd\ud55c \ub85c\ub9e8\uc2a4 \uc601\ud654\uc774\ub2e4. \uac10\ub3d9\uacfc \uc720\uba38\uac00 \ub118\uce58\uace0 \uc170\uc77c\ub9b0 \uc6b0\ub4e4\ub9ac\uac00 \uc5f0\uae30\ub85c \uc9c8\uc744 \ub192\uc774\uace0 \uc788\ub2e4. \uc170\uc77c\ub9b0 \uc6b0\ub4e4\ub9ac\ub294 \uce74\uba54\ub77c \uc55e\uc5d0\uc11c \uc2e4\uc218\ub97c \ud560 \uc218 \uc5c6\ub2e4\uace0 \ud560 \uc815\ub3c4\ub85c \uc88b\uc740 \ubc30\uc6b0\uc774\ub2e4.\"\ub77c\uba70 \uc170\uc77c\ub9b0 \uc6b0\ub4e4\ub9ac\ub97c \uce6d\ucc2c\ud558\uace0 4\uc810 \uc911 3\uc810\uc744 \uc8fc\uc5c8\ub2e4.", "answer": "B+", "sentence": "\ub308\ub7ec\uc2a4 \ubaa8\ub2dd \ub274\uc2a4\uc758 \ud06c\ub9ac\uc2a4 \ubd09\uac70\ub294 B+ \ub77c\ub294 \ud3c9\uac00\ub97c \ub0b4\ub838\ub2e4 \"\u300a\uc548\ub155, \ud5e4\uc774\uc990\u300b\uc740 \uc7ac\uce58\uc788\uace0 \uc0dd\uae30 \uc788\ub2e4.", "paragraph_sentence": "\ud0c0\uc784\uc758 \ub9ac\ucc98\ub4dc \ucf5c\ub9ac\uc2a4\ub294 \"\uc601\ud654 \uc18d \ud5e4\uc774\uc990\uacfc \uc5b4\uac70\uc2a4\ud130\uc2a4\ub294 \ub208 \uc55e\uc5d0 \ubcf4\uc774\ub294 \uac83\ub4e4\uc744 \uc0dd\uac01\ud558\uba70, \uc0dd\uae30\uac00 \uc788\ub2e4. \uc774 \uc810\uc740 \ub450 \uc0ac\ub78c\uc744 \uc5f0\uae30\ud558\uace0 \uc788\ub294 \ubc30\uc6b0\uac00 \uc80a\uae30 \ub54c\ubb38\uc77c \uac83\uc774\ub2e4. \"\uace0 \ub9d0\ud588\ub2e4. \ub308\ub7ec\uc2a4 \ubaa8\ub2dd \ub274\uc2a4\uc758 \ud06c\ub9ac\uc2a4 \ubd09\uac70\ub294 B+ \ub77c\ub294 \ud3c9\uac00\ub97c \ub0b4\ub838\ub2e4 \"\u300a\uc548\ub155, \ud5e4\uc774\uc990\u300b\uc740 \uc7ac\uce58\uc788\uace0 \uc0dd\uae30 \uc788\ub2e4. \ub2e8\uc9c0 \uc870\uae08 \uac10\uc0c1\uc801\uc774\ub2e4. \"\uace0 \ud588\ub2e4. USA \ud22c\ub370\uc774\uc758 \ud074\ub77c\uc6b0\ub514\uc544 \ud478\uc774\uadf8\ub294 4\uc810 \uc911 3.5\uc810\uc744 \uc8fc\uace0, \"\uc88b\uc740 \uac01\ubcf8\uc774\uba70 \uc5f0\uae30\ub3c4 \uc88b\ub2e4. \uc7ac\ubbf8\ub3c4 \uc788\uace0 \ud55c\ud3b8\uc73c\ub85c\ub294 \ud1b5\ucc30\ub825\uc774 \uc788\ub2e4. \"\uace0 \ud3c9\ud558\uc600\ub2e4. \uc5d4\ud130\ud14c\uc778\uba3c\ud2b8 \uc704\ud074\ub9ac\uc758 \ud06c\ub9ac\uc2a4 \ub0b4\uc154\uc6e8\uc774\ud2f0\ub294 \"\uc7ac\ubbf8\uc788\uace0 \ub2ec\ucf64\ud55c \ub9e4\uc6b0 \uac10\ub3d9\uc2a4\ub7ec\uc6b4 \uc601\ud654\"\ub77c\uba70 B \ud3c9\uac00\ub97c \ub0b4\ub838\ub2e4. \uae00\ub85c\ube0c \uc564 \uba54\uc77c\uc758 \uc81c\ud504 \ud398\ubca0\ub808\ub294 \"\uc774 \uc791\ud488\uc740 \uad00\uac1d\uc774 \uc2e4\uc81c \uc0dd\ud65c\uc5d0\uc11c \uac00\uc9c0\ub294 \uae30\ub300\ub97c \ubc30\ubc18\ud560\uc9c0\ub3c4 \ubaa8\ub978\ub2e4. \ud558\uc9c0\ub9cc \uccad\uc18c\ub144\uc744 \uc704\ud55c \uc601\ud654\uc758 \uad00\uc2b5\uc744 \uc2f9\ub451 \uc790\ub974\uace0 \uc788\ub2e4. \"\uace0 \ud558\uc600\ub2e4. \ub9c8\uc774\uc560\ubbf8 \ud5e4\ub7f4\ub4dc\uc758 \ucf54\ub2c8 \uc624\uae00\uc740 \"\uac11\uc790\uae30 \ub530\ub73b\ud574\uc9c0\uac70\ub098 \uc720\uba38\uac00 \uc788\ub294 \ub2ec\ucf64\ud55c \ub85c\ub9e8\uc2a4 \uc601\ud654\ub2e4. \"\ub77c\uace0 \ud558\uc600\ub2e4. \ub514 A.V. \ud074\ub7fd\uc758 A. A. \ub2e4\uc6b0\ub4dc\ub294 B \ud3c9\uac00\ub97c \ub0b4\ub9ac\uba70, \"\ubc88\uac1c \uac19\uc740 \uc704\ud2b8\uac00 \uc788\uc73c\uba70, \uac1c\uc131 \ud48d\ubd80\ud558\uace0 \uc7ac\ub2a5\uc788\ub294 \ubc30\uc6b0\ub4e4\uc5d0 \uc758\ud574 \ub9e4\ub825\uc774 \uc99d\uac00\ud55c\ub2e4. \"\uace0 \ub9d0\ud588\ub2e4. \ub864\ub9c1 \uc2a4\ud1a4\uc758 \ud53c\ud130 \ud2b8\ub798\ubc84\uc2a4\ub294 \"\uc2e0\uc120\ud558\uace0 \uc0dd\uc0dd\ud55c \ub85c\ub9e8\uc2a4 \uc601\ud654\uc774\ub2e4. \uac10\ub3d9\uacfc \uc720\uba38\uac00 \ub118\uce58\uace0 \uc170\uc77c\ub9b0 \uc6b0\ub4e4\ub9ac\uac00 \uc5f0\uae30\ub85c \uc9c8\uc744 \ub192\uc774\uace0 \uc788\ub2e4. \uc170\uc77c\ub9b0 \uc6b0\ub4e4\ub9ac\ub294 \uce74\uba54\ub77c \uc55e\uc5d0\uc11c \uc2e4\uc218\ub97c \ud560 \uc218 \uc5c6\ub2e4\uace0 \ud560 \uc815\ub3c4\ub85c \uc88b\uc740 \ubc30\uc6b0\uc774\ub2e4. \"\ub77c\uba70 \uc170\uc77c\ub9b0 \uc6b0\ub4e4\ub9ac\ub97c \uce6d\ucc2c\ud558\uace0 4\uc810 \uc911 3\uc810\uc744 \uc8fc\uc5c8\ub2e4.", "paragraph_answer": "\ud0c0\uc784\uc758 \ub9ac\ucc98\ub4dc \ucf5c\ub9ac\uc2a4\ub294 \"\uc601\ud654 \uc18d \ud5e4\uc774\uc990\uacfc \uc5b4\uac70\uc2a4\ud130\uc2a4\ub294 \ub208 \uc55e\uc5d0 \ubcf4\uc774\ub294 \uac83\ub4e4\uc744 \uc0dd\uac01\ud558\uba70, \uc0dd\uae30\uac00 \uc788\ub2e4. \uc774 \uc810\uc740 \ub450 \uc0ac\ub78c\uc744 \uc5f0\uae30\ud558\uace0 \uc788\ub294 \ubc30\uc6b0\uac00 \uc80a\uae30 \ub54c\ubb38\uc77c \uac83\uc774\ub2e4.\"\uace0 \ub9d0\ud588\ub2e4. \ub308\ub7ec\uc2a4 \ubaa8\ub2dd \ub274\uc2a4\uc758 \ud06c\ub9ac\uc2a4 \ubd09\uac70\ub294 B+ \ub77c\ub294 \ud3c9\uac00\ub97c \ub0b4\ub838\ub2e4 \"\u300a\uc548\ub155, \ud5e4\uc774\uc990\u300b\uc740 \uc7ac\uce58\uc788\uace0 \uc0dd\uae30 \uc788\ub2e4. \ub2e8\uc9c0 \uc870\uae08 \uac10\uc0c1\uc801\uc774\ub2e4.\"\uace0 \ud588\ub2e4. USA \ud22c\ub370\uc774\uc758 \ud074\ub77c\uc6b0\ub514\uc544 \ud478\uc774\uadf8\ub294 4\uc810 \uc911 3.5\uc810\uc744 \uc8fc\uace0, \"\uc88b\uc740 \uac01\ubcf8\uc774\uba70 \uc5f0\uae30\ub3c4 \uc88b\ub2e4. \uc7ac\ubbf8\ub3c4 \uc788\uace0 \ud55c\ud3b8\uc73c\ub85c\ub294 \ud1b5\ucc30\ub825\uc774 \uc788\ub2e4.\"\uace0 \ud3c9\ud558\uc600\ub2e4. \uc5d4\ud130\ud14c\uc778\uba3c\ud2b8 \uc704\ud074\ub9ac\uc758 \ud06c\ub9ac\uc2a4 \ub0b4\uc154\uc6e8\uc774\ud2f0\ub294 \"\uc7ac\ubbf8\uc788\uace0 \ub2ec\ucf64\ud55c \ub9e4\uc6b0 \uac10\ub3d9\uc2a4\ub7ec\uc6b4 \uc601\ud654\"\ub77c\uba70 B \ud3c9\uac00\ub97c \ub0b4\ub838\ub2e4. \uae00\ub85c\ube0c \uc564 \uba54\uc77c\uc758 \uc81c\ud504 \ud398\ubca0\ub808\ub294 \"\uc774 \uc791\ud488\uc740 \uad00\uac1d\uc774 \uc2e4\uc81c \uc0dd\ud65c\uc5d0\uc11c \uac00\uc9c0\ub294 \uae30\ub300\ub97c \ubc30\ubc18\ud560\uc9c0\ub3c4 \ubaa8\ub978\ub2e4. \ud558\uc9c0\ub9cc \uccad\uc18c\ub144\uc744 \uc704\ud55c \uc601\ud654\uc758 \uad00\uc2b5\uc744 \uc2f9\ub451 \uc790\ub974\uace0 \uc788\ub2e4.\"\uace0 \ud558\uc600\ub2e4. \ub9c8\uc774\uc560\ubbf8 \ud5e4\ub7f4\ub4dc\uc758 \ucf54\ub2c8 \uc624\uae00\uc740 \"\uac11\uc790\uae30 \ub530\ub73b\ud574\uc9c0\uac70\ub098 \uc720\uba38\uac00 \uc788\ub294 \ub2ec\ucf64\ud55c \ub85c\ub9e8\uc2a4 \uc601\ud654\ub2e4.\"\ub77c\uace0 \ud558\uc600\ub2e4. \ub514 A.V. \ud074\ub7fd\uc758 A. A. \ub2e4\uc6b0\ub4dc\ub294 B \ud3c9\uac00\ub97c \ub0b4\ub9ac\uba70, \"\ubc88\uac1c \uac19\uc740 \uc704\ud2b8\uac00 \uc788\uc73c\uba70, \uac1c\uc131 \ud48d\ubd80\ud558\uace0 \uc7ac\ub2a5\uc788\ub294 \ubc30\uc6b0\ub4e4\uc5d0 \uc758\ud574 \ub9e4\ub825\uc774 \uc99d\uac00\ud55c\ub2e4.\"\uace0 \ub9d0\ud588\ub2e4. \ub864\ub9c1 \uc2a4\ud1a4\uc758 \ud53c\ud130 \ud2b8\ub798\ubc84\uc2a4\ub294 \"\uc2e0\uc120\ud558\uace0 \uc0dd\uc0dd\ud55c \ub85c\ub9e8\uc2a4 \uc601\ud654\uc774\ub2e4. \uac10\ub3d9\uacfc \uc720\uba38\uac00 \ub118\uce58\uace0 \uc170\uc77c\ub9b0 \uc6b0\ub4e4\ub9ac\uac00 \uc5f0\uae30\ub85c \uc9c8\uc744 \ub192\uc774\uace0 \uc788\ub2e4. \uc170\uc77c\ub9b0 \uc6b0\ub4e4\ub9ac\ub294 \uce74\uba54\ub77c \uc55e\uc5d0\uc11c \uc2e4\uc218\ub97c \ud560 \uc218 \uc5c6\ub2e4\uace0 \ud560 \uc815\ub3c4\ub85c \uc88b\uc740 \ubc30\uc6b0\uc774\ub2e4.\"\ub77c\uba70 \uc170\uc77c\ub9b0 \uc6b0\ub4e4\ub9ac\ub97c \uce6d\ucc2c\ud558\uace0 4\uc810 \uc911 3\uc810\uc744 \uc8fc\uc5c8\ub2e4.", "sentence_answer": "\ub308\ub7ec\uc2a4 \ubaa8\ub2dd \ub274\uc2a4\uc758 \ud06c\ub9ac\uc2a4 \ubd09\uac70\ub294 B+ \ub77c\ub294 \ud3c9\uac00\ub97c \ub0b4\ub838\ub2e4 \"\u300a\uc548\ub155, \ud5e4\uc774\uc990\u300b\uc740 \uc7ac\uce58\uc788\uace0 \uc0dd\uae30 \uc788\ub2e4.", "paragraph_id": "6657271-9-0", "paragraph_question": "question: \ub308\ub7ec\uc2a4 \ubaa8\ub2dd \ub274\uc2a4\uc758 \ud06c\ub9ac\uc2a4 \ubd09\uac70\ub294 \uc5b4\ub5a4 \ud3c9\uac00\ub97c \ub0b4\ub838\ub098?, context: \ud0c0\uc784\uc758 \ub9ac\ucc98\ub4dc \ucf5c\ub9ac\uc2a4\ub294 \"\uc601\ud654 \uc18d \ud5e4\uc774\uc990\uacfc \uc5b4\uac70\uc2a4\ud130\uc2a4\ub294 \ub208 \uc55e\uc5d0 \ubcf4\uc774\ub294 \uac83\ub4e4\uc744 \uc0dd\uac01\ud558\uba70, \uc0dd\uae30\uac00 \uc788\ub2e4. \uc774 \uc810\uc740 \ub450 \uc0ac\ub78c\uc744 \uc5f0\uae30\ud558\uace0 \uc788\ub294 \ubc30\uc6b0\uac00 \uc80a\uae30 \ub54c\ubb38\uc77c \uac83\uc774\ub2e4.\"\uace0 \ub9d0\ud588\ub2e4. \ub308\ub7ec\uc2a4 \ubaa8\ub2dd \ub274\uc2a4\uc758 \ud06c\ub9ac\uc2a4 \ubd09\uac70\ub294 B+\ub77c\ub294 \ud3c9\uac00\ub97c \ub0b4\ub838\ub2e4 \"\u300a\uc548\ub155, \ud5e4\uc774\uc990\u300b\uc740 \uc7ac\uce58\uc788\uace0 \uc0dd\uae30 \uc788\ub2e4. \ub2e8\uc9c0 \uc870\uae08 \uac10\uc0c1\uc801\uc774\ub2e4.\"\uace0 \ud588\ub2e4. USA \ud22c\ub370\uc774\uc758 \ud074\ub77c\uc6b0\ub514\uc544 \ud478\uc774\uadf8\ub294 4\uc810 \uc911 3.5\uc810\uc744 \uc8fc\uace0, \"\uc88b\uc740 \uac01\ubcf8\uc774\uba70 \uc5f0\uae30\ub3c4 \uc88b\ub2e4. \uc7ac\ubbf8\ub3c4 \uc788\uace0 \ud55c\ud3b8\uc73c\ub85c\ub294 \ud1b5\ucc30\ub825\uc774 \uc788\ub2e4.\"\uace0 \ud3c9\ud558\uc600\ub2e4. \uc5d4\ud130\ud14c\uc778\uba3c\ud2b8 \uc704\ud074\ub9ac\uc758 \ud06c\ub9ac\uc2a4 \ub0b4\uc154\uc6e8\uc774\ud2f0\ub294 \"\uc7ac\ubbf8\uc788\uace0 \ub2ec\ucf64\ud55c \ub9e4\uc6b0 \uac10\ub3d9\uc2a4\ub7ec\uc6b4 \uc601\ud654\"\ub77c\uba70 B \ud3c9\uac00\ub97c \ub0b4\ub838\ub2e4. \uae00\ub85c\ube0c \uc564 \uba54\uc77c\uc758 \uc81c\ud504 \ud398\ubca0\ub808\ub294 \"\uc774 \uc791\ud488\uc740 \uad00\uac1d\uc774 \uc2e4\uc81c \uc0dd\ud65c\uc5d0\uc11c \uac00\uc9c0\ub294 \uae30\ub300\ub97c \ubc30\ubc18\ud560\uc9c0\ub3c4 \ubaa8\ub978\ub2e4. \ud558\uc9c0\ub9cc \uccad\uc18c\ub144\uc744 \uc704\ud55c \uc601\ud654\uc758 \uad00\uc2b5\uc744 \uc2f9\ub451 \uc790\ub974\uace0 \uc788\ub2e4.\"\uace0 \ud558\uc600\ub2e4. \ub9c8\uc774\uc560\ubbf8 \ud5e4\ub7f4\ub4dc\uc758 \ucf54\ub2c8 \uc624\uae00\uc740 \"\uac11\uc790\uae30 \ub530\ub73b\ud574\uc9c0\uac70\ub098 \uc720\uba38\uac00 \uc788\ub294 \ub2ec\ucf64\ud55c \ub85c\ub9e8\uc2a4 \uc601\ud654\ub2e4.\"\ub77c\uace0 \ud558\uc600\ub2e4. \ub514 A.V. \ud074\ub7fd\uc758 A. A. \ub2e4\uc6b0\ub4dc\ub294 B \ud3c9\uac00\ub97c \ub0b4\ub9ac\uba70, \"\ubc88\uac1c \uac19\uc740 \uc704\ud2b8\uac00 \uc788\uc73c\uba70, \uac1c\uc131 \ud48d\ubd80\ud558\uace0 \uc7ac\ub2a5\uc788\ub294 \ubc30\uc6b0\ub4e4\uc5d0 \uc758\ud574 \ub9e4\ub825\uc774 \uc99d\uac00\ud55c\ub2e4.\"\uace0 \ub9d0\ud588\ub2e4. \ub864\ub9c1 \uc2a4\ud1a4\uc758 \ud53c\ud130 \ud2b8\ub798\ubc84\uc2a4\ub294 \"\uc2e0\uc120\ud558\uace0 \uc0dd\uc0dd\ud55c \ub85c\ub9e8\uc2a4 \uc601\ud654\uc774\ub2e4. \uac10\ub3d9\uacfc \uc720\uba38\uac00 \ub118\uce58\uace0 \uc170\uc77c\ub9b0 \uc6b0\ub4e4\ub9ac\uac00 \uc5f0\uae30\ub85c \uc9c8\uc744 \ub192\uc774\uace0 \uc788\ub2e4. \uc170\uc77c\ub9b0 \uc6b0\ub4e4\ub9ac\ub294 \uce74\uba54\ub77c \uc55e\uc5d0\uc11c \uc2e4\uc218\ub97c \ud560 \uc218 \uc5c6\ub2e4\uace0 \ud560 \uc815\ub3c4\ub85c \uc88b\uc740 \ubc30\uc6b0\uc774\ub2e4.\"\ub77c\uba70 \uc170\uc77c\ub9b0 \uc6b0\ub4e4\ub9ac\ub97c \uce6d\ucc2c\ud558\uace0 4\uc810 \uc911 3\uc810\uc744 \uc8fc\uc5c8\ub2e4."} {"question": "\uac15\uc81c\ub178\ub3d9 \uc218\uc6a9\uc18c\ub85c \ubcf4\ub0bc \uc778\ubd80\ub97c \uc120\ubcc4\ud55c \uc0ac\ub78c\uc740 \ub204\uad6c\uc778\uac00?", "paragraph": "\uac00\uc2a4\uc2e4\uc744 \uad6c\ube44\ud55c \ubaa8\ub4e0 \uc9d1\ub2e8\ud559\uc0b4 \uc218\uc6a9\uc18c\ub294 \uae30\ucc28\ub97c \ud1b5\ud574 \ubaa8\ub4e0 \ud3ec\ub85c\ub4e4\uc744 \ubc1b\uc558\ub2e4. \ub54c\ub85c\ub294 \uc2e4\ub824\uc628 \uadf8\ub300\ub85c \uac00\uc2a4\uc2e4\ub85c \ubcf4\ub0b4\uc9c0\uae30\ub3c4 \ud588\uc9c0\ub9cc, \ub300\uac1c \uc218\uc6a9\uc18c \uad70\uc758\uad00\uc774 \uac80\uc0ac\ud558\uc5ec \uac15\uc81c\ub178\ub3d9 \uc218\uc6a9\uc18c\ub85c \ubcf4\ub0bc \uadf9\uc18c\uc218\uc758 \uc778\ubd80\ub97c \uc120\ubcc4\ud558\uace4 \ud588\ub2e4. \uadf8\ub9ac\uace0 \ub098\uba38\uc9c0\ub294 \uc811\uc218 \ud50c\ub7ab\ud3fc\uc73c\ub85c \ubcf4\ub0b4\uc9c0\uace0 \uc5ec\uae30\uc11c \ubaa8\ub4e0 \uc18c\uc720\ud488\ub4e4\uc740 \ub098\uce58\uc758 \uc804\uc7c1\uc790\uae08 \ub9c8\ub828\uc744 \uc704\ud574 \uc555\uc218\ub418\uc5c8\ub2e4. \uadf8\ub9ac\uace0 \uc54c\ubab8\uc73c\ub85c \uac00\uc2a4\uc2e4\ub85c \ubab0\uc544\ub123\uc5b4\uc84c\ub2e4. \uac04\uc218\ub4e4\uc740 \ub300\uac1c \ubc29\uc5ed\uc744 \uc704\ud574 \uc0e4\uc6cc\ub97c \uc2dc\ud0a4\ub294 \uac83\uc774\ub77c\uace0 \uc54c\ub9ac\uace0\ub294 \uc785\uc2e4\uc774 \uc644\ub8cc\ub418\uba74 \uc678\ubd80\uc5d0\uc11c \u2018\uc785\uc695\u2019 \ub4f1\uacfc \uac19\uc740 \uc2e0\ud638\ub97c \uc8fc\uace0\ubc1b\ub294\ub2e4. \uac00\ub054\uc529 \uc785\uc2e4 \uc804\uc5d0 \ube44\ub204\ub098 \uc218\uac74\uc744 \uc950\uc5b4\uc8fc\uc5b4 \ud639\uc2dc \ubaa8\ub97c \uc720\ub300\uc778\ub4e4\uc758 \uacf5\ud669\uc0c1\ud0dc\ub97c \uc608\ubc29\ud558\uae30\ub3c4 \ud588\uc73c\uba70 \uc2ec\uc9c0\uc5b4 \uae34 \uc5ec\uc815\uc73c\ub85c \uc778\ud574 \uac08\uc99d\uc744 \ud638\uc18c\ud558\ub294 \uc778\uc6d0\ub4e4\uc5d0\uac8c\ub294 \uc0e4\uc6cc \ud6c4\uc5d0 \uc9c0\uae09\ub420 \ucee4\ud53c\uac00 \uc2dd\uace0 \uc788\uc73c\ub2c8 \ube60\ub974\uac8c \uc53b\uc744 \uc900\ube44\ub098 \ud558\ub77c\uace0 \uc774\uc57c\uae30\ud558\uae30\ub3c4 \ud588\ub2e4.", "answer": "\uc218\uc6a9\uc18c \uad70\uc758\uad00", "sentence": "\ub300\uac1c \uc218\uc6a9\uc18c \uad70\uc758\uad00 \uc774 \uac80\uc0ac\ud558\uc5ec \uac15\uc81c\ub178\ub3d9 \uc218\uc6a9\uc18c\ub85c \ubcf4\ub0bc \uadf9\uc18c\uc218\uc758 \uc778\ubd80\ub97c \uc120\ubcc4\ud558\uace4 \ud588\ub2e4.", "paragraph_sentence": "\uac00\uc2a4\uc2e4\uc744 \uad6c\ube44\ud55c \ubaa8\ub4e0 \uc9d1\ub2e8\ud559\uc0b4 \uc218\uc6a9\uc18c\ub294 \uae30\ucc28\ub97c \ud1b5\ud574 \ubaa8\ub4e0 \ud3ec\ub85c\ub4e4\uc744 \ubc1b\uc558\ub2e4. \ub54c\ub85c\ub294 \uc2e4\ub824\uc628 \uadf8\ub300\ub85c \uac00\uc2a4\uc2e4\ub85c \ubcf4\ub0b4\uc9c0\uae30\ub3c4 \ud588\uc9c0\ub9cc, \ub300\uac1c \uc218\uc6a9\uc18c \uad70\uc758\uad00 \uc774 \uac80\uc0ac\ud558\uc5ec \uac15\uc81c\ub178\ub3d9 \uc218\uc6a9\uc18c\ub85c \ubcf4\ub0bc \uadf9\uc18c\uc218\uc758 \uc778\ubd80\ub97c \uc120\ubcc4\ud558\uace4 \ud588\ub2e4. \uadf8\ub9ac\uace0 \ub098\uba38\uc9c0\ub294 \uc811\uc218 \ud50c\ub7ab\ud3fc\uc73c\ub85c \ubcf4\ub0b4\uc9c0\uace0 \uc5ec\uae30\uc11c \ubaa8\ub4e0 \uc18c\uc720\ud488\ub4e4\uc740 \ub098\uce58\uc758 \uc804\uc7c1\uc790\uae08 \ub9c8\ub828\uc744 \uc704\ud574 \uc555\uc218\ub418\uc5c8\ub2e4. \uadf8\ub9ac\uace0 \uc54c\ubab8\uc73c\ub85c \uac00\uc2a4\uc2e4\ub85c \ubab0\uc544\ub123\uc5b4\uc84c\ub2e4. \uac04\uc218\ub4e4\uc740 \ub300\uac1c \ubc29\uc5ed\uc744 \uc704\ud574 \uc0e4\uc6cc\ub97c \uc2dc\ud0a4\ub294 \uac83\uc774\ub77c\uace0 \uc54c\ub9ac\uace0\ub294 \uc785\uc2e4\uc774 \uc644\ub8cc\ub418\uba74 \uc678\ubd80\uc5d0\uc11c \u2018\uc785\uc695\u2019 \ub4f1\uacfc \uac19\uc740 \uc2e0\ud638\ub97c \uc8fc\uace0\ubc1b\ub294\ub2e4. \uac00\ub054\uc529 \uc785\uc2e4 \uc804\uc5d0 \ube44\ub204\ub098 \uc218\uac74\uc744 \uc950\uc5b4\uc8fc\uc5b4 \ud639\uc2dc \ubaa8\ub97c \uc720\ub300\uc778\ub4e4\uc758 \uacf5\ud669\uc0c1\ud0dc\ub97c \uc608\ubc29\ud558\uae30\ub3c4 \ud588\uc73c\uba70 \uc2ec\uc9c0\uc5b4 \uae34 \uc5ec\uc815\uc73c\ub85c \uc778\ud574 \uac08\uc99d\uc744 \ud638\uc18c\ud558\ub294 \uc778\uc6d0\ub4e4\uc5d0\uac8c\ub294 \uc0e4\uc6cc \ud6c4\uc5d0 \uc9c0\uae09\ub420 \ucee4\ud53c\uac00 \uc2dd\uace0 \uc788\uc73c\ub2c8 \ube60\ub974\uac8c \uc53b\uc744 \uc900\ube44\ub098 \ud558\ub77c\uace0 \uc774\uc57c\uae30\ud558\uae30\ub3c4 \ud588\ub2e4.", "paragraph_answer": "\uac00\uc2a4\uc2e4\uc744 \uad6c\ube44\ud55c \ubaa8\ub4e0 \uc9d1\ub2e8\ud559\uc0b4 \uc218\uc6a9\uc18c\ub294 \uae30\ucc28\ub97c \ud1b5\ud574 \ubaa8\ub4e0 \ud3ec\ub85c\ub4e4\uc744 \ubc1b\uc558\ub2e4. \ub54c\ub85c\ub294 \uc2e4\ub824\uc628 \uadf8\ub300\ub85c \uac00\uc2a4\uc2e4\ub85c \ubcf4\ub0b4\uc9c0\uae30\ub3c4 \ud588\uc9c0\ub9cc, \ub300\uac1c \uc218\uc6a9\uc18c \uad70\uc758\uad00 \uc774 \uac80\uc0ac\ud558\uc5ec \uac15\uc81c\ub178\ub3d9 \uc218\uc6a9\uc18c\ub85c \ubcf4\ub0bc \uadf9\uc18c\uc218\uc758 \uc778\ubd80\ub97c \uc120\ubcc4\ud558\uace4 \ud588\ub2e4. \uadf8\ub9ac\uace0 \ub098\uba38\uc9c0\ub294 \uc811\uc218 \ud50c\ub7ab\ud3fc\uc73c\ub85c \ubcf4\ub0b4\uc9c0\uace0 \uc5ec\uae30\uc11c \ubaa8\ub4e0 \uc18c\uc720\ud488\ub4e4\uc740 \ub098\uce58\uc758 \uc804\uc7c1\uc790\uae08 \ub9c8\ub828\uc744 \uc704\ud574 \uc555\uc218\ub418\uc5c8\ub2e4. \uadf8\ub9ac\uace0 \uc54c\ubab8\uc73c\ub85c \uac00\uc2a4\uc2e4\ub85c \ubab0\uc544\ub123\uc5b4\uc84c\ub2e4. \uac04\uc218\ub4e4\uc740 \ub300\uac1c \ubc29\uc5ed\uc744 \uc704\ud574 \uc0e4\uc6cc\ub97c \uc2dc\ud0a4\ub294 \uac83\uc774\ub77c\uace0 \uc54c\ub9ac\uace0\ub294 \uc785\uc2e4\uc774 \uc644\ub8cc\ub418\uba74 \uc678\ubd80\uc5d0\uc11c \u2018\uc785\uc695\u2019 \ub4f1\uacfc \uac19\uc740 \uc2e0\ud638\ub97c \uc8fc\uace0\ubc1b\ub294\ub2e4. \uac00\ub054\uc529 \uc785\uc2e4 \uc804\uc5d0 \ube44\ub204\ub098 \uc218\uac74\uc744 \uc950\uc5b4\uc8fc\uc5b4 \ud639\uc2dc \ubaa8\ub97c \uc720\ub300\uc778\ub4e4\uc758 \uacf5\ud669\uc0c1\ud0dc\ub97c \uc608\ubc29\ud558\uae30\ub3c4 \ud588\uc73c\uba70 \uc2ec\uc9c0\uc5b4 \uae34 \uc5ec\uc815\uc73c\ub85c \uc778\ud574 \uac08\uc99d\uc744 \ud638\uc18c\ud558\ub294 \uc778\uc6d0\ub4e4\uc5d0\uac8c\ub294 \uc0e4\uc6cc \ud6c4\uc5d0 \uc9c0\uae09\ub420 \ucee4\ud53c\uac00 \uc2dd\uace0 \uc788\uc73c\ub2c8 \ube60\ub974\uac8c \uc53b\uc744 \uc900\ube44\ub098 \ud558\ub77c\uace0 \uc774\uc57c\uae30\ud558\uae30\ub3c4 \ud588\ub2e4.", "sentence_answer": "\ub300\uac1c \uc218\uc6a9\uc18c \uad70\uc758\uad00 \uc774 \uac80\uc0ac\ud558\uc5ec \uac15\uc81c\ub178\ub3d9 \uc218\uc6a9\uc18c\ub85c \ubcf4\ub0bc \uadf9\uc18c\uc218\uc758 \uc778\ubd80\ub97c \uc120\ubcc4\ud558\uace4 \ud588\ub2e4.", "paragraph_id": "6571960-20-1", "paragraph_question": "question: \uac15\uc81c\ub178\ub3d9 \uc218\uc6a9\uc18c\ub85c \ubcf4\ub0bc \uc778\ubd80\ub97c \uc120\ubcc4\ud55c \uc0ac\ub78c\uc740 \ub204\uad6c\uc778\uac00?, context: \uac00\uc2a4\uc2e4\uc744 \uad6c\ube44\ud55c \ubaa8\ub4e0 \uc9d1\ub2e8\ud559\uc0b4 \uc218\uc6a9\uc18c\ub294 \uae30\ucc28\ub97c \ud1b5\ud574 \ubaa8\ub4e0 \ud3ec\ub85c\ub4e4\uc744 \ubc1b\uc558\ub2e4. \ub54c\ub85c\ub294 \uc2e4\ub824\uc628 \uadf8\ub300\ub85c \uac00\uc2a4\uc2e4\ub85c \ubcf4\ub0b4\uc9c0\uae30\ub3c4 \ud588\uc9c0\ub9cc, \ub300\uac1c \uc218\uc6a9\uc18c \uad70\uc758\uad00\uc774 \uac80\uc0ac\ud558\uc5ec \uac15\uc81c\ub178\ub3d9 \uc218\uc6a9\uc18c\ub85c \ubcf4\ub0bc \uadf9\uc18c\uc218\uc758 \uc778\ubd80\ub97c \uc120\ubcc4\ud558\uace4 \ud588\ub2e4. \uadf8\ub9ac\uace0 \ub098\uba38\uc9c0\ub294 \uc811\uc218 \ud50c\ub7ab\ud3fc\uc73c\ub85c \ubcf4\ub0b4\uc9c0\uace0 \uc5ec\uae30\uc11c \ubaa8\ub4e0 \uc18c\uc720\ud488\ub4e4\uc740 \ub098\uce58\uc758 \uc804\uc7c1\uc790\uae08 \ub9c8\ub828\uc744 \uc704\ud574 \uc555\uc218\ub418\uc5c8\ub2e4. \uadf8\ub9ac\uace0 \uc54c\ubab8\uc73c\ub85c \uac00\uc2a4\uc2e4\ub85c \ubab0\uc544\ub123\uc5b4\uc84c\ub2e4. \uac04\uc218\ub4e4\uc740 \ub300\uac1c \ubc29\uc5ed\uc744 \uc704\ud574 \uc0e4\uc6cc\ub97c \uc2dc\ud0a4\ub294 \uac83\uc774\ub77c\uace0 \uc54c\ub9ac\uace0\ub294 \uc785\uc2e4\uc774 \uc644\ub8cc\ub418\uba74 \uc678\ubd80\uc5d0\uc11c \u2018\uc785\uc695\u2019 \ub4f1\uacfc \uac19\uc740 \uc2e0\ud638\ub97c \uc8fc\uace0\ubc1b\ub294\ub2e4. \uac00\ub054\uc529 \uc785\uc2e4 \uc804\uc5d0 \ube44\ub204\ub098 \uc218\uac74\uc744 \uc950\uc5b4\uc8fc\uc5b4 \ud639\uc2dc \ubaa8\ub97c \uc720\ub300\uc778\ub4e4\uc758 \uacf5\ud669\uc0c1\ud0dc\ub97c \uc608\ubc29\ud558\uae30\ub3c4 \ud588\uc73c\uba70 \uc2ec\uc9c0\uc5b4 \uae34 \uc5ec\uc815\uc73c\ub85c \uc778\ud574 \uac08\uc99d\uc744 \ud638\uc18c\ud558\ub294 \uc778\uc6d0\ub4e4\uc5d0\uac8c\ub294 \uc0e4\uc6cc \ud6c4\uc5d0 \uc9c0\uae09\ub420 \ucee4\ud53c\uac00 \uc2dd\uace0 \uc788\uc73c\ub2c8 \ube60\ub974\uac8c \uc53b\uc744 \uc900\ube44\ub098 \ud558\ub77c\uace0 \uc774\uc57c\uae30\ud558\uae30\ub3c4 \ud588\ub2e4."} {"question": "\uad6c\uc81c\uc5ed \uc6d0\uc778\uc744 \ubc1c\uacac\ud55c \uc0ac\ub78c\uc740?", "paragraph": "\uad6c\uc81c\uc5ed(\u53e3\u8e44\u75ab, \uc601\uc5b4: foot-and-mouth disease, hoof-and-mouth disease, \ud559\uba85: Aphtae epizoot) \ub610\ub294 \uc785\ubc1c\uad7d\ubcd1 \uc740 \uc18c\uc640 \ub3fc\uc9c0 \ub4f1 \uac00\ucd95\uc5d0 \ub300\ud55c \uc804\uc5fc\uc131\uc774 \ub192\uc740 \uae09\uc131 \ubc14\uc774\ub7ec\uc2a4\uc131 \uc804\uc5fc\ubcd1\uc758 \ud558\ub098\uc774\ub2e4. \uc0ac\uc2b4\uc774\ub098 \uc5fc\uc18c, \uc591\uacfc \uae30\ud0c0 \uc18c\uacfc \uc6b0\uc81c\ub958 \uac00\ucd95\ub4e4, \uadf8\ub9ac\uace0 \ucf54\ub07c\ub9ac, \uc950, \uace0\uc2b4\ub3c4\uce58 \ub4f1 \ubc1c\uad7d\uc774 \ub450\uac1c\ub85c \uac08\ub77c\uc9c4 \uac00\ucd95\ub4e4\uc5d0\uac8c \uac10\uc5fc\ub41c\ub2e4. \ub77c\ub9c8, \uc54c\ud30c\uce74\ub3c4 \uac00\ubcbc\uc6b4 \uc99d\uc0c1\uc744 \ubcf4\uc77c \uc218 \uc788\uc73c\ub098 \uc800\ud56d\ub825\uc744 \uac00\uc9c0\uace0 \uc788\uace0 \uac19\uc740 \uc885\uc758 \ub2e4\ub978 \ub3d9\ubb3c\uc5d0 \uc804\uc5fc\uc2dc\ud0a4\uc9c0 \uc54a\ub294\ub2e4. \uc2e4\ud5d8\uc2e4 \uc2e4\ud5d8\uc5d0\uc11c \uc950, \ub2ed\uc5d0 \uc778\uacf5\uc801\uc73c\ub85c \uac10\uc5fc\uc2dc\ud0a4\ub294\ub370 \uc131\uacf5\ud558\uc600\ub2e4 \ud558\ub098 \uc790\uc5f0\uc0c1\ud0dc\uc5d0\uc11c\ub3c4 \uac10\uc5fc\ub418\ub294\uc9c0\ub294 \ud655\uc2e4\ud558\uc9c0 \uc54a\ub2e4. \uc0ac\ub78c\ub3c4 \uade0\uc744 \uc637\uc774\ub098 \ubab8\uc5d0 \ubb3b\ud788\uace0 \ub2e4\ub2d8\uc73c\ub85c\uc368 \ubcd1\uc744 \ud37c\ub728\ub9b4 \uc218 \uc788\uc73c\ub098, \uc774 \ubcd1\uc5d0 \uac78\ub9ac\uc9c0 \uc54a\ub294 \ub3d9\ubb3c\uc740 \uc5b4\ub5a4 \ub3c4\uc6c0\uc774 \uc788\uc5b4\uc57c \uc804\ud30c\uc2dc\ud0a8\ub2e4. 1952\ub144 \uce90\ub098\ub2e4\uc5d0\uc11c\ub294 \uac1c\ub4e4\uc774 \uc8fd\uc740 \ub3d9\ubb3c\ub4e4\uc758 \ubf08\ub97c \uc62e\uae34 \ud6c4\uc5d0 \ud655\uc0b0\uc5d0 \ub2e4\uc2dc \ubd88\uae38\uc774 \ubd99\uc5c8\uace0, \uc61b \uc18c\ub828\uc5d0\uc11c\ub294 \ub291\ub300\uac00 \ube44\uc2b7\ud55c \uc5ed\ud560\uc744 \ud588\ub358 \uac83\uc73c\ub85c \uc0dd\uac01\ub418\uace0 \uc788\ub2e4. \uc0ac\ub78c\uc5d0 \uc601\ud5a5\uc744 \ubbf8\uce58\ub294 \uacbd\uc6b0\ub294 \ub9e4\uc6b0 \ub4dc\ubb3c\uba70, \uc77c\ubc18\uc801\uc73c\ub85c \ubcc4\ub2e4\ub978 \uce58\ub8cc\ub97c \ud558\uc9c0 \uc54a\ub294\ub2e4. \ub300\ud55c\ubbfc\uad6d\uc5d0\uc11c\ub294 \uc81c1\uc885\uac00\ucd95\uc804\uc5fc\ubcd1\uc73c\ub85c \uc9c0\uc815\ub418\uc5b4 \uc788\ub2e4. 1897\ub144\uc5d0 \ud504\ub9ac\ub4dc\ub9ac\ud788 \ub8b0\ud50c\ub7ec\uac00 \uad6c\uc81c\uc5ed\uc758 \uc6d0\uc778\uc740 \ubc14\uc774\ub7ec\uc2a4\ub77c\ub294 \uac83\uc744 \ubc1c\uacac\ud558\uc600\ub2e4. \uadf8\ub294 \uac10\uc5fc\ub41c \ub3d9\ubb3c\uc758 \ud608\uc561\uc744 \ud3ec\uc140\ub9b0 \ud544\ud130\uc5d0 \ud1b5\uacfc\uc2dc\ucf1c \uac78\ub7ec\ub3c4 \uc5ec\uc804\ud788 \ub2e4\ub978 \ub3d9\ubb3c\uc744 \uac10\uc5fc\uc2dc\ud0ac \uc218 \uc788\uc74c\uc744 \ud655\uc778\ud588\ub2e4. \uad6c\uc81c\uc5ed\uc740 \uc138\uacc4 \ub300\ubd80\ubd84\uc758 \uc9c0\uc5ed\uc5d0\uc11c \ubc1c\uc0dd\ud558\uba70, \uc219\uc8fc\uac00 \ub418\ub294 \ub3d9\ubb3c\uc758 \uc885\ub958\uc640 \uac1c\uccb4\uc218\uac00 \ub9ce\uace0 \uc804\uc5fc\uc131\uc774 \ub192\uae30 \ub54c\ubb38\uc5d0, \ud55c\ub3d9\uc548 \uad6c\uc81c\uc5ed\uc774 \ubc1c\uc0dd\ud558\uc9c0 \uc54a\ub358 \uacf3\uc5d0\uc11c\ub3c4 \ubc1c\uc0dd\ud560 \uc218 \uc788\ub2e4.", "answer": "\ud504\ub9ac\ub4dc\ub9ac\ud788 \ub8b0\ud50c\ub7ec", "sentence": "1897\ub144\uc5d0 \ud504\ub9ac\ub4dc\ub9ac\ud788 \ub8b0\ud50c\ub7ec \uac00 \uad6c\uc81c\uc5ed\uc758 \uc6d0\uc778\uc740 \ubc14\uc774\ub7ec\uc2a4\ub77c\ub294 \uac83\uc744 \ubc1c\uacac\ud558\uc600\ub2e4.", "paragraph_sentence": "\uad6c\uc81c\uc5ed(\u53e3\u8e44\u75ab, \uc601\uc5b4: foot-and-mouth disease, hoof-and-mouth disease, \ud559\uba85: Aphtae epizoot) \ub610\ub294 \uc785\ubc1c\uad7d\ubcd1 \uc740 \uc18c\uc640 \ub3fc\uc9c0 \ub4f1 \uac00\ucd95\uc5d0 \ub300\ud55c \uc804\uc5fc\uc131\uc774 \ub192\uc740 \uae09\uc131 \ubc14\uc774\ub7ec\uc2a4\uc131 \uc804\uc5fc\ubcd1\uc758 \ud558\ub098\uc774\ub2e4. \uc0ac\uc2b4\uc774\ub098 \uc5fc\uc18c, \uc591\uacfc \uae30\ud0c0 \uc18c\uacfc \uc6b0\uc81c\ub958 \uac00\ucd95\ub4e4, \uadf8\ub9ac\uace0 \ucf54\ub07c\ub9ac, \uc950, \uace0\uc2b4\ub3c4\uce58 \ub4f1 \ubc1c\uad7d\uc774 \ub450\uac1c\ub85c \uac08\ub77c\uc9c4 \uac00\ucd95\ub4e4\uc5d0\uac8c \uac10\uc5fc\ub41c\ub2e4. \ub77c\ub9c8, \uc54c\ud30c\uce74\ub3c4 \uac00\ubcbc\uc6b4 \uc99d\uc0c1\uc744 \ubcf4\uc77c \uc218 \uc788\uc73c\ub098 \uc800\ud56d\ub825\uc744 \uac00\uc9c0\uace0 \uc788\uace0 \uac19\uc740 \uc885\uc758 \ub2e4\ub978 \ub3d9\ubb3c\uc5d0 \uc804\uc5fc\uc2dc\ud0a4\uc9c0 \uc54a\ub294\ub2e4. \uc2e4\ud5d8\uc2e4 \uc2e4\ud5d8\uc5d0\uc11c \uc950, \ub2ed\uc5d0 \uc778\uacf5\uc801\uc73c\ub85c \uac10\uc5fc\uc2dc\ud0a4\ub294\ub370 \uc131\uacf5\ud558\uc600\ub2e4 \ud558\ub098 \uc790\uc5f0\uc0c1\ud0dc\uc5d0\uc11c\ub3c4 \uac10\uc5fc\ub418\ub294\uc9c0\ub294 \ud655\uc2e4\ud558\uc9c0 \uc54a\ub2e4. \uc0ac\ub78c\ub3c4 \uade0\uc744 \uc637\uc774\ub098 \ubab8\uc5d0 \ubb3b\ud788\uace0 \ub2e4\ub2d8\uc73c\ub85c\uc368 \ubcd1\uc744 \ud37c\ub728\ub9b4 \uc218 \uc788\uc73c\ub098, \uc774 \ubcd1\uc5d0 \uac78\ub9ac\uc9c0 \uc54a\ub294 \ub3d9\ubb3c\uc740 \uc5b4\ub5a4 \ub3c4\uc6c0\uc774 \uc788\uc5b4\uc57c \uc804\ud30c\uc2dc\ud0a8\ub2e4. 1952\ub144 \uce90\ub098\ub2e4\uc5d0\uc11c\ub294 \uac1c\ub4e4\uc774 \uc8fd\uc740 \ub3d9\ubb3c\ub4e4\uc758 \ubf08\ub97c \uc62e\uae34 \ud6c4\uc5d0 \ud655\uc0b0\uc5d0 \ub2e4\uc2dc \ubd88\uae38\uc774 \ubd99\uc5c8\uace0, \uc61b \uc18c\ub828\uc5d0\uc11c\ub294 \ub291\ub300\uac00 \ube44\uc2b7\ud55c \uc5ed\ud560\uc744 \ud588\ub358 \uac83\uc73c\ub85c \uc0dd\uac01\ub418\uace0 \uc788\ub2e4. \uc0ac\ub78c\uc5d0 \uc601\ud5a5\uc744 \ubbf8\uce58\ub294 \uacbd\uc6b0\ub294 \ub9e4\uc6b0 \ub4dc\ubb3c\uba70, \uc77c\ubc18\uc801\uc73c\ub85c \ubcc4\ub2e4\ub978 \uce58\ub8cc\ub97c \ud558\uc9c0 \uc54a\ub294\ub2e4. \ub300\ud55c\ubbfc\uad6d\uc5d0\uc11c\ub294 \uc81c1\uc885\uac00\ucd95\uc804\uc5fc\ubcd1\uc73c\ub85c \uc9c0\uc815\ub418\uc5b4 \uc788\ub2e4. 1897\ub144\uc5d0 \ud504\ub9ac\ub4dc\ub9ac\ud788 \ub8b0\ud50c\ub7ec \uac00 \uad6c\uc81c\uc5ed\uc758 \uc6d0\uc778\uc740 \ubc14\uc774\ub7ec\uc2a4\ub77c\ub294 \uac83\uc744 \ubc1c\uacac\ud558\uc600\ub2e4. \uadf8\ub294 \uac10\uc5fc\ub41c \ub3d9\ubb3c\uc758 \ud608\uc561\uc744 \ud3ec\uc140\ub9b0 \ud544\ud130\uc5d0 \ud1b5\uacfc\uc2dc\ucf1c \uac78\ub7ec\ub3c4 \uc5ec\uc804\ud788 \ub2e4\ub978 \ub3d9\ubb3c\uc744 \uac10\uc5fc\uc2dc\ud0ac \uc218 \uc788\uc74c\uc744 \ud655\uc778\ud588\ub2e4. \uad6c\uc81c\uc5ed\uc740 \uc138\uacc4 \ub300\ubd80\ubd84\uc758 \uc9c0\uc5ed\uc5d0\uc11c \ubc1c\uc0dd\ud558\uba70, \uc219\uc8fc\uac00 \ub418\ub294 \ub3d9\ubb3c\uc758 \uc885\ub958\uc640 \uac1c\uccb4\uc218\uac00 \ub9ce\uace0 \uc804\uc5fc\uc131\uc774 \ub192\uae30 \ub54c\ubb38\uc5d0, \ud55c\ub3d9\uc548 \uad6c\uc81c\uc5ed\uc774 \ubc1c\uc0dd\ud558\uc9c0 \uc54a\ub358 \uacf3\uc5d0\uc11c\ub3c4 \ubc1c\uc0dd\ud560 \uc218 \uc788\ub2e4.", "paragraph_answer": "\uad6c\uc81c\uc5ed(\u53e3\u8e44\u75ab, \uc601\uc5b4: foot-and-mouth disease, hoof-and-mouth disease, \ud559\uba85: Aphtae epizoot) \ub610\ub294 \uc785\ubc1c\uad7d\ubcd1 \uc740 \uc18c\uc640 \ub3fc\uc9c0 \ub4f1 \uac00\ucd95\uc5d0 \ub300\ud55c \uc804\uc5fc\uc131\uc774 \ub192\uc740 \uae09\uc131 \ubc14\uc774\ub7ec\uc2a4\uc131 \uc804\uc5fc\ubcd1\uc758 \ud558\ub098\uc774\ub2e4. \uc0ac\uc2b4\uc774\ub098 \uc5fc\uc18c, \uc591\uacfc \uae30\ud0c0 \uc18c\uacfc \uc6b0\uc81c\ub958 \uac00\ucd95\ub4e4, \uadf8\ub9ac\uace0 \ucf54\ub07c\ub9ac, \uc950, \uace0\uc2b4\ub3c4\uce58 \ub4f1 \ubc1c\uad7d\uc774 \ub450\uac1c\ub85c \uac08\ub77c\uc9c4 \uac00\ucd95\ub4e4\uc5d0\uac8c \uac10\uc5fc\ub41c\ub2e4. \ub77c\ub9c8, \uc54c\ud30c\uce74\ub3c4 \uac00\ubcbc\uc6b4 \uc99d\uc0c1\uc744 \ubcf4\uc77c \uc218 \uc788\uc73c\ub098 \uc800\ud56d\ub825\uc744 \uac00\uc9c0\uace0 \uc788\uace0 \uac19\uc740 \uc885\uc758 \ub2e4\ub978 \ub3d9\ubb3c\uc5d0 \uc804\uc5fc\uc2dc\ud0a4\uc9c0 \uc54a\ub294\ub2e4. \uc2e4\ud5d8\uc2e4 \uc2e4\ud5d8\uc5d0\uc11c \uc950, \ub2ed\uc5d0 \uc778\uacf5\uc801\uc73c\ub85c \uac10\uc5fc\uc2dc\ud0a4\ub294\ub370 \uc131\uacf5\ud558\uc600\ub2e4 \ud558\ub098 \uc790\uc5f0\uc0c1\ud0dc\uc5d0\uc11c\ub3c4 \uac10\uc5fc\ub418\ub294\uc9c0\ub294 \ud655\uc2e4\ud558\uc9c0 \uc54a\ub2e4. \uc0ac\ub78c\ub3c4 \uade0\uc744 \uc637\uc774\ub098 \ubab8\uc5d0 \ubb3b\ud788\uace0 \ub2e4\ub2d8\uc73c\ub85c\uc368 \ubcd1\uc744 \ud37c\ub728\ub9b4 \uc218 \uc788\uc73c\ub098, \uc774 \ubcd1\uc5d0 \uac78\ub9ac\uc9c0 \uc54a\ub294 \ub3d9\ubb3c\uc740 \uc5b4\ub5a4 \ub3c4\uc6c0\uc774 \uc788\uc5b4\uc57c \uc804\ud30c\uc2dc\ud0a8\ub2e4. 1952\ub144 \uce90\ub098\ub2e4\uc5d0\uc11c\ub294 \uac1c\ub4e4\uc774 \uc8fd\uc740 \ub3d9\ubb3c\ub4e4\uc758 \ubf08\ub97c \uc62e\uae34 \ud6c4\uc5d0 \ud655\uc0b0\uc5d0 \ub2e4\uc2dc \ubd88\uae38\uc774 \ubd99\uc5c8\uace0, \uc61b \uc18c\ub828\uc5d0\uc11c\ub294 \ub291\ub300\uac00 \ube44\uc2b7\ud55c \uc5ed\ud560\uc744 \ud588\ub358 \uac83\uc73c\ub85c \uc0dd\uac01\ub418\uace0 \uc788\ub2e4. \uc0ac\ub78c\uc5d0 \uc601\ud5a5\uc744 \ubbf8\uce58\ub294 \uacbd\uc6b0\ub294 \ub9e4\uc6b0 \ub4dc\ubb3c\uba70, \uc77c\ubc18\uc801\uc73c\ub85c \ubcc4\ub2e4\ub978 \uce58\ub8cc\ub97c \ud558\uc9c0 \uc54a\ub294\ub2e4. \ub300\ud55c\ubbfc\uad6d\uc5d0\uc11c\ub294 \uc81c1\uc885\uac00\ucd95\uc804\uc5fc\ubcd1\uc73c\ub85c \uc9c0\uc815\ub418\uc5b4 \uc788\ub2e4. 1897\ub144\uc5d0 \ud504\ub9ac\ub4dc\ub9ac\ud788 \ub8b0\ud50c\ub7ec \uac00 \uad6c\uc81c\uc5ed\uc758 \uc6d0\uc778\uc740 \ubc14\uc774\ub7ec\uc2a4\ub77c\ub294 \uac83\uc744 \ubc1c\uacac\ud558\uc600\ub2e4. \uadf8\ub294 \uac10\uc5fc\ub41c \ub3d9\ubb3c\uc758 \ud608\uc561\uc744 \ud3ec\uc140\ub9b0 \ud544\ud130\uc5d0 \ud1b5\uacfc\uc2dc\ucf1c \uac78\ub7ec\ub3c4 \uc5ec\uc804\ud788 \ub2e4\ub978 \ub3d9\ubb3c\uc744 \uac10\uc5fc\uc2dc\ud0ac \uc218 \uc788\uc74c\uc744 \ud655\uc778\ud588\ub2e4. \uad6c\uc81c\uc5ed\uc740 \uc138\uacc4 \ub300\ubd80\ubd84\uc758 \uc9c0\uc5ed\uc5d0\uc11c \ubc1c\uc0dd\ud558\uba70, \uc219\uc8fc\uac00 \ub418\ub294 \ub3d9\ubb3c\uc758 \uc885\ub958\uc640 \uac1c\uccb4\uc218\uac00 \ub9ce\uace0 \uc804\uc5fc\uc131\uc774 \ub192\uae30 \ub54c\ubb38\uc5d0, \ud55c\ub3d9\uc548 \uad6c\uc81c\uc5ed\uc774 \ubc1c\uc0dd\ud558\uc9c0 \uc54a\ub358 \uacf3\uc5d0\uc11c\ub3c4 \ubc1c\uc0dd\ud560 \uc218 \uc788\ub2e4.", "sentence_answer": "1897\ub144\uc5d0 \ud504\ub9ac\ub4dc\ub9ac\ud788 \ub8b0\ud50c\ub7ec \uac00 \uad6c\uc81c\uc5ed\uc758 \uc6d0\uc778\uc740 \ubc14\uc774\ub7ec\uc2a4\ub77c\ub294 \uac83\uc744 \ubc1c\uacac\ud558\uc600\ub2e4.", "paragraph_id": "5972391-0-2", "paragraph_question": "question: \uad6c\uc81c\uc5ed \uc6d0\uc778\uc744 \ubc1c\uacac\ud55c \uc0ac\ub78c\uc740?, context: \uad6c\uc81c\uc5ed(\u53e3\u8e44\u75ab, \uc601\uc5b4: foot-and-mouth disease, hoof-and-mouth disease, \ud559\uba85: Aphtae epizoot) \ub610\ub294 \uc785\ubc1c\uad7d\ubcd1 \uc740 \uc18c\uc640 \ub3fc\uc9c0 \ub4f1 \uac00\ucd95\uc5d0 \ub300\ud55c \uc804\uc5fc\uc131\uc774 \ub192\uc740 \uae09\uc131 \ubc14\uc774\ub7ec\uc2a4\uc131 \uc804\uc5fc\ubcd1\uc758 \ud558\ub098\uc774\ub2e4. \uc0ac\uc2b4\uc774\ub098 \uc5fc\uc18c, \uc591\uacfc \uae30\ud0c0 \uc18c\uacfc \uc6b0\uc81c\ub958 \uac00\ucd95\ub4e4, \uadf8\ub9ac\uace0 \ucf54\ub07c\ub9ac, \uc950, \uace0\uc2b4\ub3c4\uce58 \ub4f1 \ubc1c\uad7d\uc774 \ub450\uac1c\ub85c \uac08\ub77c\uc9c4 \uac00\ucd95\ub4e4\uc5d0\uac8c \uac10\uc5fc\ub41c\ub2e4. \ub77c\ub9c8, \uc54c\ud30c\uce74\ub3c4 \uac00\ubcbc\uc6b4 \uc99d\uc0c1\uc744 \ubcf4\uc77c \uc218 \uc788\uc73c\ub098 \uc800\ud56d\ub825\uc744 \uac00\uc9c0\uace0 \uc788\uace0 \uac19\uc740 \uc885\uc758 \ub2e4\ub978 \ub3d9\ubb3c\uc5d0 \uc804\uc5fc\uc2dc\ud0a4\uc9c0 \uc54a\ub294\ub2e4. \uc2e4\ud5d8\uc2e4 \uc2e4\ud5d8\uc5d0\uc11c \uc950, \ub2ed\uc5d0 \uc778\uacf5\uc801\uc73c\ub85c \uac10\uc5fc\uc2dc\ud0a4\ub294\ub370 \uc131\uacf5\ud558\uc600\ub2e4 \ud558\ub098 \uc790\uc5f0\uc0c1\ud0dc\uc5d0\uc11c\ub3c4 \uac10\uc5fc\ub418\ub294\uc9c0\ub294 \ud655\uc2e4\ud558\uc9c0 \uc54a\ub2e4. \uc0ac\ub78c\ub3c4 \uade0\uc744 \uc637\uc774\ub098 \ubab8\uc5d0 \ubb3b\ud788\uace0 \ub2e4\ub2d8\uc73c\ub85c\uc368 \ubcd1\uc744 \ud37c\ub728\ub9b4 \uc218 \uc788\uc73c\ub098, \uc774 \ubcd1\uc5d0 \uac78\ub9ac\uc9c0 \uc54a\ub294 \ub3d9\ubb3c\uc740 \uc5b4\ub5a4 \ub3c4\uc6c0\uc774 \uc788\uc5b4\uc57c \uc804\ud30c\uc2dc\ud0a8\ub2e4. 1952\ub144 \uce90\ub098\ub2e4\uc5d0\uc11c\ub294 \uac1c\ub4e4\uc774 \uc8fd\uc740 \ub3d9\ubb3c\ub4e4\uc758 \ubf08\ub97c \uc62e\uae34 \ud6c4\uc5d0 \ud655\uc0b0\uc5d0 \ub2e4\uc2dc \ubd88\uae38\uc774 \ubd99\uc5c8\uace0, \uc61b \uc18c\ub828\uc5d0\uc11c\ub294 \ub291\ub300\uac00 \ube44\uc2b7\ud55c \uc5ed\ud560\uc744 \ud588\ub358 \uac83\uc73c\ub85c \uc0dd\uac01\ub418\uace0 \uc788\ub2e4. \uc0ac\ub78c\uc5d0 \uc601\ud5a5\uc744 \ubbf8\uce58\ub294 \uacbd\uc6b0\ub294 \ub9e4\uc6b0 \ub4dc\ubb3c\uba70, \uc77c\ubc18\uc801\uc73c\ub85c \ubcc4\ub2e4\ub978 \uce58\ub8cc\ub97c \ud558\uc9c0 \uc54a\ub294\ub2e4. \ub300\ud55c\ubbfc\uad6d\uc5d0\uc11c\ub294 \uc81c1\uc885\uac00\ucd95\uc804\uc5fc\ubcd1\uc73c\ub85c \uc9c0\uc815\ub418\uc5b4 \uc788\ub2e4. 1897\ub144\uc5d0 \ud504\ub9ac\ub4dc\ub9ac\ud788 \ub8b0\ud50c\ub7ec\uac00 \uad6c\uc81c\uc5ed\uc758 \uc6d0\uc778\uc740 \ubc14\uc774\ub7ec\uc2a4\ub77c\ub294 \uac83\uc744 \ubc1c\uacac\ud558\uc600\ub2e4. \uadf8\ub294 \uac10\uc5fc\ub41c \ub3d9\ubb3c\uc758 \ud608\uc561\uc744 \ud3ec\uc140\ub9b0 \ud544\ud130\uc5d0 \ud1b5\uacfc\uc2dc\ucf1c \uac78\ub7ec\ub3c4 \uc5ec\uc804\ud788 \ub2e4\ub978 \ub3d9\ubb3c\uc744 \uac10\uc5fc\uc2dc\ud0ac \uc218 \uc788\uc74c\uc744 \ud655\uc778\ud588\ub2e4. \uad6c\uc81c\uc5ed\uc740 \uc138\uacc4 \ub300\ubd80\ubd84\uc758 \uc9c0\uc5ed\uc5d0\uc11c \ubc1c\uc0dd\ud558\uba70, \uc219\uc8fc\uac00 \ub418\ub294 \ub3d9\ubb3c\uc758 \uc885\ub958\uc640 \uac1c\uccb4\uc218\uac00 \ub9ce\uace0 \uc804\uc5fc\uc131\uc774 \ub192\uae30 \ub54c\ubb38\uc5d0, \ud55c\ub3d9\uc548 \uad6c\uc81c\uc5ed\uc774 \ubc1c\uc0dd\ud558\uc9c0 \uc54a\ub358 \uacf3\uc5d0\uc11c\ub3c4 \ubc1c\uc0dd\ud560 \uc218 \uc788\ub2e4."} {"question": "\ubca0\ub9ac\uac00 90\ud68c \uc0dd\uc77c\uc744 \ub9de\uc544 \ucd9c\uc2dc\ud558\uae30\ub85c \ud55c, 38\ub144 \ub9cc\uc758 \uc568\ubc94 \uc81c\ubaa9\uc740 \ubb34\uc5c7\uc778\uac00?", "paragraph": "2000\ub144 11\uc6d4 \ubca0\ub9ac\ub294 \ub3d9\ub8cc \ud53c\uc544\ub2c8\uc2a4\ud2b8 \uc870\ub2c8 \uc874\uc2a8\uc5d0 \uc758\ud574 \ubc95\uc801 \ubb38\uc81c\uc5d0 \ud718\ub9d0\ub9b0\ub2e4. \uc870\ub2c8\ub294 \ubca0\ub9ac\uc640 \uc790\uc2e0\uc774 \u3008No Particular Place to Go\u3009, \u3008Sweet Little Sixteen\u3009, \u3008Roll Over Beethoven\u3009\ub97c \ube44\ub86f\ud55c 50\uace1\uc744 \uacf5\ub3d9 \uc791\uace1\ud588\uc73c\ub098 \ud06c\ub808\ub514\ud2b8\uc5d0\ub294 \ub2ec\ub791 \ubca0\ub9ac \ud63c\uc790 \ubfd0\uc774\ub77c\uace0 \uc8fc\uc7a5\ud588\ub2e4. \uc774 \uc18c\uc1a1\uc740 \ud310\uc0ac\uac00 \uc791\uace1\ud55c \uc2dc\uc810\uc5d0\uc11c \ub108\ubb34 \uc624\ub79c \uc2dc\uac04\uc774 \uc9c0\ub0ac\ub2e4\uace0 \ud310\uacb0\ud558\uba74\uc11c \uae30\uac01\ub418\uc5c8\ub2e4. 2008\ub144 \ubca0\ub9ac\ub294 \uc2a4\uc6e8\ub374, \ub178\ub974\uc6e8\uc774, \ud540\ub780\ub4dc, \uc601\uad6d, \ub124\ub35c\ub780\ub4dc, \uc544\uc77c\ub79c\ub4dc, \uc2a4\uc704\uc2a4, \ud3f4\ub780\ub4dc, \uc2a4\ud398\uc778 \ub4f1\uc758 \uc720\ub7fd \uac01\uad6d\uc5d0\uc11c \ud22c\uc5b4\ub97c \ub3cc\uc558\ub2e4. 2008\ub144 \uc911\ubc18\uc5d0\ub294 \uba54\ub9b4\ub79c\ub4dc \uc8fc \ubcfc\ud2f0\ubaa8\uc5b4\uc758 \ubc84\uc9c4 \ud398\uc2a4\ud2f0\ubc8c\uc5d0\uc11c \uacf5\uc5f0\ud588\ub2e4. 2011\ub144 \uc0c8\ud574 \ucf58\uc11c\ud2b8 \ub3c4\uc911 \ud0c8\uc9c4\ud558\uc5ec \uc4f0\ub7ec\uc9c0\ub294 \uc0c1\ud669\uc774 \ubc1c\uc0dd, \ub3c4\uc6c0\uc744 \ubc1b\uc544 \ubb34\ub300 \ubc16\uc73c\ub85c \uc774\uc1a1\ub418\uc5c8\ub2e4. 2016\ub144 10\uc6d4 18\uc77c, \u300a\ud0c0\uc784\u300b\uc740 90\ud68c \uc0dd\uc77c\uc744 \ub9de\uc544 \uc6f9\uc0ac\uc774\ud2b8\uc5d0 \uac8c\uc7ac\ud55c \uc131\uba85\uc744 \ud1b5\ud574 38\ub144 \ub9cc\uc5d0 \uc568\ubc94 \u300aChuck\u300b\uc744 \ucd9c\uc2dc\ud560 \uac83\uc774\ub77c\uace0 \uacf5\uc5b8\ud588\ub2e4. \ubca0\ub9ac\ub294 \ucd9c\uc2dc \uc2dc\uae30\uc5d0 \ub300\ud574\uc11c\ub294 \uc5ec\uc720\ub97c \ub450\uba74\uc11c \"\uc774\ubc88 \uc74c\ubc18\uc740 \ub0b4 \uc0ac\ub791\ud558\ub294 \ud1a0\ub514\uc5d0\uac8c \ud5cc\uc815\ub41c\ub2e4\"\uace0 68\ub144 \ub3d9\uc548 \ud574\ub85c\ud55c \ud14c\uba54\ud0c0\ub97c \uc5b8\uae09\ud588\ub2e4. \uadf8\ub9ac\uace0 \"\ub0b4 \uc0ac\ub791. \ub098\ub3c4 \ub098\uc774\uac00 \uba39\uc5b4\uac00\ub124! \uc774 \uc74c\ubc18 \uc791\uc5c5\uc744 \uc624\ub7ab\ub3d9\uc548 \ud574\uc654\uc5b4\uc694. \uc774\uc81c \uc77c\uc744 \uadf8\ub9cc \ub458 \uc218 \uc788\uc5b4\uc694!\"\ub77c\uace0 \ub367\ubd99\uc600\ub2e4.", "answer": "Chuck", "sentence": "2016\ub144 10\uc6d4 18\uc77c, \u300a\ud0c0\uc784\u300b\uc740 90\ud68c \uc0dd\uc77c\uc744 \ub9de\uc544 \uc6f9\uc0ac\uc774\ud2b8\uc5d0 \uac8c\uc7ac\ud55c \uc131\uba85\uc744 \ud1b5\ud574 38\ub144 \ub9cc\uc5d0 \uc568\ubc94 \u300a Chuck \u300b\uc744 \ucd9c\uc2dc\ud560 \uac83\uc774\ub77c\uace0 \uacf5\uc5b8\ud588\ub2e4.", "paragraph_sentence": "2000\ub144 11\uc6d4 \ubca0\ub9ac\ub294 \ub3d9\ub8cc \ud53c\uc544\ub2c8\uc2a4\ud2b8 \uc870\ub2c8 \uc874\uc2a8\uc5d0 \uc758\ud574 \ubc95\uc801 \ubb38\uc81c\uc5d0 \ud718\ub9d0\ub9b0\ub2e4. \uc870\ub2c8\ub294 \ubca0\ub9ac\uc640 \uc790\uc2e0\uc774 \u3008No Particular Place to Go\u3009, \u3008Sweet Little Sixteen\u3009, \u3008Roll Over Beethoven\u3009\ub97c \ube44\ub86f\ud55c 50\uace1\uc744 \uacf5\ub3d9 \uc791\uace1\ud588\uc73c\ub098 \ud06c\ub808\ub514\ud2b8\uc5d0\ub294 \ub2ec\ub791 \ubca0\ub9ac \ud63c\uc790 \ubfd0\uc774\ub77c\uace0 \uc8fc\uc7a5\ud588\ub2e4. \uc774 \uc18c\uc1a1\uc740 \ud310\uc0ac\uac00 \uc791\uace1\ud55c \uc2dc\uc810\uc5d0\uc11c \ub108\ubb34 \uc624\ub79c \uc2dc\uac04\uc774 \uc9c0\ub0ac\ub2e4\uace0 \ud310\uacb0\ud558\uba74\uc11c \uae30\uac01\ub418\uc5c8\ub2e4. 2008\ub144 \ubca0\ub9ac\ub294 \uc2a4\uc6e8\ub374, \ub178\ub974\uc6e8\uc774, \ud540\ub780\ub4dc, \uc601\uad6d, \ub124\ub35c\ub780\ub4dc, \uc544\uc77c\ub79c\ub4dc, \uc2a4\uc704\uc2a4, \ud3f4\ub780\ub4dc, \uc2a4\ud398\uc778 \ub4f1\uc758 \uc720\ub7fd \uac01\uad6d\uc5d0\uc11c \ud22c\uc5b4\ub97c \ub3cc\uc558\ub2e4. 2008\ub144 \uc911\ubc18\uc5d0\ub294 \uba54\ub9b4\ub79c\ub4dc \uc8fc \ubcfc\ud2f0\ubaa8\uc5b4\uc758 \ubc84\uc9c4 \ud398\uc2a4\ud2f0\ubc8c\uc5d0\uc11c \uacf5\uc5f0\ud588\ub2e4. 2011\ub144 \uc0c8\ud574 \ucf58\uc11c\ud2b8 \ub3c4\uc911 \ud0c8\uc9c4\ud558\uc5ec \uc4f0\ub7ec\uc9c0\ub294 \uc0c1\ud669\uc774 \ubc1c\uc0dd, \ub3c4\uc6c0\uc744 \ubc1b\uc544 \ubb34\ub300 \ubc16\uc73c\ub85c \uc774\uc1a1\ub418\uc5c8\ub2e4. 2016\ub144 10\uc6d4 18\uc77c, \u300a\ud0c0\uc784\u300b\uc740 90\ud68c \uc0dd\uc77c\uc744 \ub9de\uc544 \uc6f9\uc0ac\uc774\ud2b8\uc5d0 \uac8c\uc7ac\ud55c \uc131\uba85\uc744 \ud1b5\ud574 38\ub144 \ub9cc\uc5d0 \uc568\ubc94 \u300a Chuck \u300b\uc744 \ucd9c\uc2dc\ud560 \uac83\uc774\ub77c\uace0 \uacf5\uc5b8\ud588\ub2e4. \ubca0\ub9ac\ub294 \ucd9c\uc2dc \uc2dc\uae30\uc5d0 \ub300\ud574\uc11c\ub294 \uc5ec\uc720\ub97c \ub450\uba74\uc11c \"\uc774\ubc88 \uc74c\ubc18\uc740 \ub0b4 \uc0ac\ub791\ud558\ub294 \ud1a0\ub514\uc5d0\uac8c \ud5cc\uc815\ub41c\ub2e4\"\uace0 68\ub144 \ub3d9\uc548 \ud574\ub85c\ud55c \ud14c\uba54\ud0c0\ub97c \uc5b8\uae09\ud588\ub2e4. \uadf8\ub9ac\uace0 \"\ub0b4 \uc0ac\ub791. \ub098\ub3c4 \ub098\uc774\uac00 \uba39\uc5b4\uac00\ub124! \uc774 \uc74c\ubc18 \uc791\uc5c5\uc744 \uc624\ub7ab\ub3d9\uc548 \ud574\uc654\uc5b4\uc694. \uc774\uc81c \uc77c\uc744 \uadf8\ub9cc \ub458 \uc218 \uc788\uc5b4\uc694!\"\ub77c\uace0 \ub367\ubd99\uc600\ub2e4.", "paragraph_answer": "2000\ub144 11\uc6d4 \ubca0\ub9ac\ub294 \ub3d9\ub8cc \ud53c\uc544\ub2c8\uc2a4\ud2b8 \uc870\ub2c8 \uc874\uc2a8\uc5d0 \uc758\ud574 \ubc95\uc801 \ubb38\uc81c\uc5d0 \ud718\ub9d0\ub9b0\ub2e4. \uc870\ub2c8\ub294 \ubca0\ub9ac\uc640 \uc790\uc2e0\uc774 \u3008No Particular Place to Go\u3009, \u3008Sweet Little Sixteen\u3009, \u3008Roll Over Beethoven\u3009\ub97c \ube44\ub86f\ud55c 50\uace1\uc744 \uacf5\ub3d9 \uc791\uace1\ud588\uc73c\ub098 \ud06c\ub808\ub514\ud2b8\uc5d0\ub294 \ub2ec\ub791 \ubca0\ub9ac \ud63c\uc790 \ubfd0\uc774\ub77c\uace0 \uc8fc\uc7a5\ud588\ub2e4. \uc774 \uc18c\uc1a1\uc740 \ud310\uc0ac\uac00 \uc791\uace1\ud55c \uc2dc\uc810\uc5d0\uc11c \ub108\ubb34 \uc624\ub79c \uc2dc\uac04\uc774 \uc9c0\ub0ac\ub2e4\uace0 \ud310\uacb0\ud558\uba74\uc11c \uae30\uac01\ub418\uc5c8\ub2e4. 2008\ub144 \ubca0\ub9ac\ub294 \uc2a4\uc6e8\ub374, \ub178\ub974\uc6e8\uc774, \ud540\ub780\ub4dc, \uc601\uad6d, \ub124\ub35c\ub780\ub4dc, \uc544\uc77c\ub79c\ub4dc, \uc2a4\uc704\uc2a4, \ud3f4\ub780\ub4dc, \uc2a4\ud398\uc778 \ub4f1\uc758 \uc720\ub7fd \uac01\uad6d\uc5d0\uc11c \ud22c\uc5b4\ub97c \ub3cc\uc558\ub2e4. 2008\ub144 \uc911\ubc18\uc5d0\ub294 \uba54\ub9b4\ub79c\ub4dc \uc8fc \ubcfc\ud2f0\ubaa8\uc5b4\uc758 \ubc84\uc9c4 \ud398\uc2a4\ud2f0\ubc8c\uc5d0\uc11c \uacf5\uc5f0\ud588\ub2e4. 2011\ub144 \uc0c8\ud574 \ucf58\uc11c\ud2b8 \ub3c4\uc911 \ud0c8\uc9c4\ud558\uc5ec \uc4f0\ub7ec\uc9c0\ub294 \uc0c1\ud669\uc774 \ubc1c\uc0dd, \ub3c4\uc6c0\uc744 \ubc1b\uc544 \ubb34\ub300 \ubc16\uc73c\ub85c \uc774\uc1a1\ub418\uc5c8\ub2e4. 2016\ub144 10\uc6d4 18\uc77c, \u300a\ud0c0\uc784\u300b\uc740 90\ud68c \uc0dd\uc77c\uc744 \ub9de\uc544 \uc6f9\uc0ac\uc774\ud2b8\uc5d0 \uac8c\uc7ac\ud55c \uc131\uba85\uc744 \ud1b5\ud574 38\ub144 \ub9cc\uc5d0 \uc568\ubc94 \u300a Chuck \u300b\uc744 \ucd9c\uc2dc\ud560 \uac83\uc774\ub77c\uace0 \uacf5\uc5b8\ud588\ub2e4. \ubca0\ub9ac\ub294 \ucd9c\uc2dc \uc2dc\uae30\uc5d0 \ub300\ud574\uc11c\ub294 \uc5ec\uc720\ub97c \ub450\uba74\uc11c \"\uc774\ubc88 \uc74c\ubc18\uc740 \ub0b4 \uc0ac\ub791\ud558\ub294 \ud1a0\ub514\uc5d0\uac8c \ud5cc\uc815\ub41c\ub2e4\"\uace0 68\ub144 \ub3d9\uc548 \ud574\ub85c\ud55c \ud14c\uba54\ud0c0\ub97c \uc5b8\uae09\ud588\ub2e4. \uadf8\ub9ac\uace0 \"\ub0b4 \uc0ac\ub791. \ub098\ub3c4 \ub098\uc774\uac00 \uba39\uc5b4\uac00\ub124! \uc774 \uc74c\ubc18 \uc791\uc5c5\uc744 \uc624\ub7ab\ub3d9\uc548 \ud574\uc654\uc5b4\uc694. \uc774\uc81c \uc77c\uc744 \uadf8\ub9cc \ub458 \uc218 \uc788\uc5b4\uc694!\"\ub77c\uace0 \ub367\ubd99\uc600\ub2e4.", "sentence_answer": "2016\ub144 10\uc6d4 18\uc77c, \u300a\ud0c0\uc784\u300b\uc740 90\ud68c \uc0dd\uc77c\uc744 \ub9de\uc544 \uc6f9\uc0ac\uc774\ud2b8\uc5d0 \uac8c\uc7ac\ud55c \uc131\uba85\uc744 \ud1b5\ud574 38\ub144 \ub9cc\uc5d0 \uc568\ubc94 \u300a Chuck \u300b\uc744 \ucd9c\uc2dc\ud560 \uac83\uc774\ub77c\uace0 \uacf5\uc5b8\ud588\ub2e4.", "paragraph_id": "6596458-14-2", "paragraph_question": "question: \ubca0\ub9ac\uac00 90\ud68c \uc0dd\uc77c\uc744 \ub9de\uc544 \ucd9c\uc2dc\ud558\uae30\ub85c \ud55c, 38\ub144 \ub9cc\uc758 \uc568\ubc94 \uc81c\ubaa9\uc740 \ubb34\uc5c7\uc778\uac00?, context: 2000\ub144 11\uc6d4 \ubca0\ub9ac\ub294 \ub3d9\ub8cc \ud53c\uc544\ub2c8\uc2a4\ud2b8 \uc870\ub2c8 \uc874\uc2a8\uc5d0 \uc758\ud574 \ubc95\uc801 \ubb38\uc81c\uc5d0 \ud718\ub9d0\ub9b0\ub2e4. \uc870\ub2c8\ub294 \ubca0\ub9ac\uc640 \uc790\uc2e0\uc774 \u3008No Particular Place to Go\u3009, \u3008Sweet Little Sixteen\u3009, \u3008Roll Over Beethoven\u3009\ub97c \ube44\ub86f\ud55c 50\uace1\uc744 \uacf5\ub3d9 \uc791\uace1\ud588\uc73c\ub098 \ud06c\ub808\ub514\ud2b8\uc5d0\ub294 \ub2ec\ub791 \ubca0\ub9ac \ud63c\uc790 \ubfd0\uc774\ub77c\uace0 \uc8fc\uc7a5\ud588\ub2e4. \uc774 \uc18c\uc1a1\uc740 \ud310\uc0ac\uac00 \uc791\uace1\ud55c \uc2dc\uc810\uc5d0\uc11c \ub108\ubb34 \uc624\ub79c \uc2dc\uac04\uc774 \uc9c0\ub0ac\ub2e4\uace0 \ud310\uacb0\ud558\uba74\uc11c \uae30\uac01\ub418\uc5c8\ub2e4. 2008\ub144 \ubca0\ub9ac\ub294 \uc2a4\uc6e8\ub374, \ub178\ub974\uc6e8\uc774, \ud540\ub780\ub4dc, \uc601\uad6d, \ub124\ub35c\ub780\ub4dc, \uc544\uc77c\ub79c\ub4dc, \uc2a4\uc704\uc2a4, \ud3f4\ub780\ub4dc, \uc2a4\ud398\uc778 \ub4f1\uc758 \uc720\ub7fd \uac01\uad6d\uc5d0\uc11c \ud22c\uc5b4\ub97c \ub3cc\uc558\ub2e4. 2008\ub144 \uc911\ubc18\uc5d0\ub294 \uba54\ub9b4\ub79c\ub4dc \uc8fc \ubcfc\ud2f0\ubaa8\uc5b4\uc758 \ubc84\uc9c4 \ud398\uc2a4\ud2f0\ubc8c\uc5d0\uc11c \uacf5\uc5f0\ud588\ub2e4. 2011\ub144 \uc0c8\ud574 \ucf58\uc11c\ud2b8 \ub3c4\uc911 \ud0c8\uc9c4\ud558\uc5ec \uc4f0\ub7ec\uc9c0\ub294 \uc0c1\ud669\uc774 \ubc1c\uc0dd, \ub3c4\uc6c0\uc744 \ubc1b\uc544 \ubb34\ub300 \ubc16\uc73c\ub85c \uc774\uc1a1\ub418\uc5c8\ub2e4. 2016\ub144 10\uc6d4 18\uc77c, \u300a\ud0c0\uc784\u300b\uc740 90\ud68c \uc0dd\uc77c\uc744 \ub9de\uc544 \uc6f9\uc0ac\uc774\ud2b8\uc5d0 \uac8c\uc7ac\ud55c \uc131\uba85\uc744 \ud1b5\ud574 38\ub144 \ub9cc\uc5d0 \uc568\ubc94 \u300aChuck\u300b\uc744 \ucd9c\uc2dc\ud560 \uac83\uc774\ub77c\uace0 \uacf5\uc5b8\ud588\ub2e4. \ubca0\ub9ac\ub294 \ucd9c\uc2dc \uc2dc\uae30\uc5d0 \ub300\ud574\uc11c\ub294 \uc5ec\uc720\ub97c \ub450\uba74\uc11c \"\uc774\ubc88 \uc74c\ubc18\uc740 \ub0b4 \uc0ac\ub791\ud558\ub294 \ud1a0\ub514\uc5d0\uac8c \ud5cc\uc815\ub41c\ub2e4\"\uace0 68\ub144 \ub3d9\uc548 \ud574\ub85c\ud55c \ud14c\uba54\ud0c0\ub97c \uc5b8\uae09\ud588\ub2e4. \uadf8\ub9ac\uace0 \"\ub0b4 \uc0ac\ub791. \ub098\ub3c4 \ub098\uc774\uac00 \uba39\uc5b4\uac00\ub124! \uc774 \uc74c\ubc18 \uc791\uc5c5\uc744 \uc624\ub7ab\ub3d9\uc548 \ud574\uc654\uc5b4\uc694. \uc774\uc81c \uc77c\uc744 \uadf8\ub9cc \ub458 \uc218 \uc788\uc5b4\uc694!\"\ub77c\uace0 \ub367\ubd99\uc600\ub2e4."} {"question": "\uae40\ud0dd\uc6a9\uc774 2012\ub144 9\uc6d4 MvP\uc5d0\uc11c Huk\uc5d0\uac8c \ud328\ud588\ub358 \uacbd\uae30\uc758 \uc2a4\ucf54\uc5b4\ub294?", "paragraph": "\ud3ec\uc2a4\ud2b8\uc2dc\uc98c \uc804\uc801\uc744 \ud3ec\ud568\ud574 3\uc2b96\ud328\uc758 \uc800\uc870\ud55c \uae30\ub85d\uc73c\ub85c \uc2dc\uc98c\uc744 \ub9c8\ubb34\ub9ac\ud588\ub2e4. 2012\ub144 9\uc6d4 MLG vs Proleague \ub300\ud68c(\uc774\ud558MvP)\uac00 \uc5f4\ub838\ub2e4. \uae40\ud0dd\uc6a9\uc740 \ucc98\uc74c \ubc29\uc1a1\ub41c \uacbd\uae30\uc5d0\uc11c Huk\uc5d0\uac8c 2:0 \uc644\ud328\ub97c \ub2f9\ud588\uc9c0\ub9cc \uc774\ud6c4 \ub2e4\ub978 \uc120\uc218\ub4e4\uc744 \uc774\uae30\uba70 4\uc5f0\uc2b9\uc744 \ud558\ub294\ub370 \uc131\uacf5\ud588\ub2e4. \uc774\ud6c4\uc5d0 \uc804\uc2b9\uc744 \ud558\uba70 10\uc2b9 2\ud328\ub77c\ub294 \uc131\uc801\uc73c\ub85c \ud611\ud68c\uc18c\uc18d \uacf5\ub3d93\uc704\uc5d0 \uc62c\ub790\ub2e4. \ucd08\ubc18\uc5d0 \ud65c\uc57d\uc774 \ub300\ub2e8\ud574\uc11c \uc2a4\ud0c02\uc5d0\uc11c \uae40\ud0dd\uc6a9\uc758 \uc2e4\ub825\uc774 \ub6f0\uc5b4\ub098\ub294\uac00 \uc2f6\uc5c8\uc73c\ub098, \ub9ac\uadf8\uac00 \uc9c4\ud589\ub420\uc218\ub85d \uc810\uc810 \ud328\uac00 \ub298\uc5b4\uc11c \uc544\uc9c1\uae4c\uc9c0\ub294 \ub354 \uc9c0\ucf1c\ubd10\uc57c \ub420\ub4ef\ud558\ub2e4. \ud558\uc9c0\ub9cc \uac1c\uc778\uc801\uc778 \uc0ac\uc815\uc73c\ub85c \uae30\uad8c\ud558\uc5ec \uae30\ub85d\uc740 \ubb34\uc758\ubbf8 \ud574\uc84c\ub2e4. \ub300\ud68c\ub97c \uae30\uad8c\ud558\uac8c \ub41c \uc774\ud6c4\uc5d0 \uc18c\uc591\uad50\uc721\uc5d0\uc11c\ub3c4 \ubaa8\uc2b5\uc744 \ube44\ucd94\uc9c0 \uc54a\uace0 SK\ud154\ub808\ucf64 T1\uacfc \uc5f0\ubd09\ud611\uc0c1 \uc7ac\uacc4\uc57d\uc774 \ub418\uc9c0 \uc54a\uc558\ub2e4\ub294 \uc18c\ubb38\uc774 \ub5a0\ub3cc\uba74\uc11c \uac01\uc885 \ucee4\ubba4\ub2c8\ud2f0\uc5d0\uc11c \uc740\ud1f4\uc124\uc774 \ub098\ub3cc\uc558\uc73c\ub098 SK\uac00 \ubaa8\ub4e0 \uc120\uc218\ub4e4\uacfc \uc7ac\uacc4\uc57d\uc744 \ud588\ub2e4\ub294 \uc18c\uc2dd\uc774 \ub4e4\ub9ac\uba74\uc11c \uae40\ud0dd\uc6a9\uc740 \uc120\uc218 \uc0dd\ud65c\uc744 \uc774\uc5b4\ub098\uac08\uc218 \uc788\uc5c8\ub2e4.", "answer": "2:0", "sentence": "\uae40\ud0dd\uc6a9\uc740 \ucc98\uc74c \ubc29\uc1a1\ub41c \uacbd\uae30\uc5d0\uc11c Huk\uc5d0\uac8c 2:0 \uc644\ud328\ub97c \ub2f9\ud588\uc9c0\ub9cc \uc774\ud6c4 \ub2e4\ub978 \uc120\uc218\ub4e4\uc744 \uc774\uae30\uba70 4\uc5f0\uc2b9\uc744 \ud558\ub294\ub370 \uc131\uacf5\ud588\ub2e4.", "paragraph_sentence": "\ud3ec\uc2a4\ud2b8\uc2dc\uc98c \uc804\uc801\uc744 \ud3ec\ud568\ud574 3\uc2b96\ud328\uc758 \uc800\uc870\ud55c \uae30\ub85d\uc73c\ub85c \uc2dc\uc98c\uc744 \ub9c8\ubb34\ub9ac\ud588\ub2e4. 2012\ub144 9\uc6d4 MLG vs Proleague \ub300\ud68c(\uc774\ud558MvP)\uac00 \uc5f4\ub838\ub2e4. \uae40\ud0dd\uc6a9\uc740 \ucc98\uc74c \ubc29\uc1a1\ub41c \uacbd\uae30\uc5d0\uc11c Huk\uc5d0\uac8c 2:0 \uc644\ud328\ub97c \ub2f9\ud588\uc9c0\ub9cc \uc774\ud6c4 \ub2e4\ub978 \uc120\uc218\ub4e4\uc744 \uc774\uae30\uba70 4\uc5f0\uc2b9\uc744 \ud558\ub294\ub370 \uc131\uacf5\ud588\ub2e4. \uc774\ud6c4\uc5d0 \uc804\uc2b9\uc744 \ud558\uba70 10\uc2b9 2\ud328\ub77c\ub294 \uc131\uc801\uc73c\ub85c \ud611\ud68c\uc18c\uc18d \uacf5\ub3d93\uc704\uc5d0 \uc62c\ub790\ub2e4. \ucd08\ubc18\uc5d0 \ud65c\uc57d\uc774 \ub300\ub2e8\ud574\uc11c \uc2a4\ud0c02\uc5d0\uc11c \uae40\ud0dd\uc6a9\uc758 \uc2e4\ub825\uc774 \ub6f0\uc5b4\ub098\ub294\uac00 \uc2f6\uc5c8\uc73c\ub098, \ub9ac\uadf8\uac00 \uc9c4\ud589\ub420\uc218\ub85d \uc810\uc810 \ud328\uac00 \ub298\uc5b4\uc11c \uc544\uc9c1\uae4c\uc9c0\ub294 \ub354 \uc9c0\ucf1c\ubd10\uc57c \ub420\ub4ef\ud558\ub2e4. \ud558\uc9c0\ub9cc \uac1c\uc778\uc801\uc778 \uc0ac\uc815\uc73c\ub85c \uae30\uad8c\ud558\uc5ec \uae30\ub85d\uc740 \ubb34\uc758\ubbf8 \ud574\uc84c\ub2e4. \ub300\ud68c\ub97c \uae30\uad8c\ud558\uac8c \ub41c \uc774\ud6c4\uc5d0 \uc18c\uc591\uad50\uc721\uc5d0\uc11c\ub3c4 \ubaa8\uc2b5\uc744 \ube44\ucd94\uc9c0 \uc54a\uace0 SK\ud154\ub808\ucf64 T1\uacfc \uc5f0\ubd09\ud611\uc0c1 \uc7ac\uacc4\uc57d\uc774 \ub418\uc9c0 \uc54a\uc558\ub2e4\ub294 \uc18c\ubb38\uc774 \ub5a0\ub3cc\uba74\uc11c \uac01\uc885 \ucee4\ubba4\ub2c8\ud2f0\uc5d0\uc11c \uc740\ud1f4\uc124\uc774 \ub098\ub3cc\uc558\uc73c\ub098 SK\uac00 \ubaa8\ub4e0 \uc120\uc218\ub4e4\uacfc \uc7ac\uacc4\uc57d\uc744 \ud588\ub2e4\ub294 \uc18c\uc2dd\uc774 \ub4e4\ub9ac\uba74\uc11c \uae40\ud0dd\uc6a9\uc740 \uc120\uc218 \uc0dd\ud65c\uc744 \uc774\uc5b4\ub098\uac08\uc218 \uc788\uc5c8\ub2e4.", "paragraph_answer": "\ud3ec\uc2a4\ud2b8\uc2dc\uc98c \uc804\uc801\uc744 \ud3ec\ud568\ud574 3\uc2b96\ud328\uc758 \uc800\uc870\ud55c \uae30\ub85d\uc73c\ub85c \uc2dc\uc98c\uc744 \ub9c8\ubb34\ub9ac\ud588\ub2e4. 2012\ub144 9\uc6d4 MLG vs Proleague \ub300\ud68c(\uc774\ud558MvP)\uac00 \uc5f4\ub838\ub2e4. \uae40\ud0dd\uc6a9\uc740 \ucc98\uc74c \ubc29\uc1a1\ub41c \uacbd\uae30\uc5d0\uc11c Huk\uc5d0\uac8c 2:0 \uc644\ud328\ub97c \ub2f9\ud588\uc9c0\ub9cc \uc774\ud6c4 \ub2e4\ub978 \uc120\uc218\ub4e4\uc744 \uc774\uae30\uba70 4\uc5f0\uc2b9\uc744 \ud558\ub294\ub370 \uc131\uacf5\ud588\ub2e4. \uc774\ud6c4\uc5d0 \uc804\uc2b9\uc744 \ud558\uba70 10\uc2b9 2\ud328\ub77c\ub294 \uc131\uc801\uc73c\ub85c \ud611\ud68c\uc18c\uc18d \uacf5\ub3d93\uc704\uc5d0 \uc62c\ub790\ub2e4. \ucd08\ubc18\uc5d0 \ud65c\uc57d\uc774 \ub300\ub2e8\ud574\uc11c \uc2a4\ud0c02\uc5d0\uc11c \uae40\ud0dd\uc6a9\uc758 \uc2e4\ub825\uc774 \ub6f0\uc5b4\ub098\ub294\uac00 \uc2f6\uc5c8\uc73c\ub098, \ub9ac\uadf8\uac00 \uc9c4\ud589\ub420\uc218\ub85d \uc810\uc810 \ud328\uac00 \ub298\uc5b4\uc11c \uc544\uc9c1\uae4c\uc9c0\ub294 \ub354 \uc9c0\ucf1c\ubd10\uc57c \ub420\ub4ef\ud558\ub2e4. \ud558\uc9c0\ub9cc \uac1c\uc778\uc801\uc778 \uc0ac\uc815\uc73c\ub85c \uae30\uad8c\ud558\uc5ec \uae30\ub85d\uc740 \ubb34\uc758\ubbf8 \ud574\uc84c\ub2e4. \ub300\ud68c\ub97c \uae30\uad8c\ud558\uac8c \ub41c \uc774\ud6c4\uc5d0 \uc18c\uc591\uad50\uc721\uc5d0\uc11c\ub3c4 \ubaa8\uc2b5\uc744 \ube44\ucd94\uc9c0 \uc54a\uace0 SK\ud154\ub808\ucf64 T1\uacfc \uc5f0\ubd09\ud611\uc0c1 \uc7ac\uacc4\uc57d\uc774 \ub418\uc9c0 \uc54a\uc558\ub2e4\ub294 \uc18c\ubb38\uc774 \ub5a0\ub3cc\uba74\uc11c \uac01\uc885 \ucee4\ubba4\ub2c8\ud2f0\uc5d0\uc11c \uc740\ud1f4\uc124\uc774 \ub098\ub3cc\uc558\uc73c\ub098 SK\uac00 \ubaa8\ub4e0 \uc120\uc218\ub4e4\uacfc \uc7ac\uacc4\uc57d\uc744 \ud588\ub2e4\ub294 \uc18c\uc2dd\uc774 \ub4e4\ub9ac\uba74\uc11c \uae40\ud0dd\uc6a9\uc740 \uc120\uc218 \uc0dd\ud65c\uc744 \uc774\uc5b4\ub098\uac08\uc218 \uc788\uc5c8\ub2e4.", "sentence_answer": "\uae40\ud0dd\uc6a9\uc740 \ucc98\uc74c \ubc29\uc1a1\ub41c \uacbd\uae30\uc5d0\uc11c Huk\uc5d0\uac8c 2:0 \uc644\ud328\ub97c \ub2f9\ud588\uc9c0\ub9cc \uc774\ud6c4 \ub2e4\ub978 \uc120\uc218\ub4e4\uc744 \uc774\uae30\uba70 4\uc5f0\uc2b9\uc744 \ud558\ub294\ub370 \uc131\uacf5\ud588\ub2e4.", "paragraph_id": "6482812-2-0", "paragraph_question": "question: \uae40\ud0dd\uc6a9\uc774 2012\ub144 9\uc6d4 MvP\uc5d0\uc11c Huk\uc5d0\uac8c \ud328\ud588\ub358 \uacbd\uae30\uc758 \uc2a4\ucf54\uc5b4\ub294?, context: \ud3ec\uc2a4\ud2b8\uc2dc\uc98c \uc804\uc801\uc744 \ud3ec\ud568\ud574 3\uc2b96\ud328\uc758 \uc800\uc870\ud55c \uae30\ub85d\uc73c\ub85c \uc2dc\uc98c\uc744 \ub9c8\ubb34\ub9ac\ud588\ub2e4. 2012\ub144 9\uc6d4 MLG vs Proleague \ub300\ud68c(\uc774\ud558MvP)\uac00 \uc5f4\ub838\ub2e4. \uae40\ud0dd\uc6a9\uc740 \ucc98\uc74c \ubc29\uc1a1\ub41c \uacbd\uae30\uc5d0\uc11c Huk\uc5d0\uac8c 2:0 \uc644\ud328\ub97c \ub2f9\ud588\uc9c0\ub9cc \uc774\ud6c4 \ub2e4\ub978 \uc120\uc218\ub4e4\uc744 \uc774\uae30\uba70 4\uc5f0\uc2b9\uc744 \ud558\ub294\ub370 \uc131\uacf5\ud588\ub2e4. \uc774\ud6c4\uc5d0 \uc804\uc2b9\uc744 \ud558\uba70 10\uc2b9 2\ud328\ub77c\ub294 \uc131\uc801\uc73c\ub85c \ud611\ud68c\uc18c\uc18d \uacf5\ub3d93\uc704\uc5d0 \uc62c\ub790\ub2e4. \ucd08\ubc18\uc5d0 \ud65c\uc57d\uc774 \ub300\ub2e8\ud574\uc11c \uc2a4\ud0c02\uc5d0\uc11c \uae40\ud0dd\uc6a9\uc758 \uc2e4\ub825\uc774 \ub6f0\uc5b4\ub098\ub294\uac00 \uc2f6\uc5c8\uc73c\ub098, \ub9ac\uadf8\uac00 \uc9c4\ud589\ub420\uc218\ub85d \uc810\uc810 \ud328\uac00 \ub298\uc5b4\uc11c \uc544\uc9c1\uae4c\uc9c0\ub294 \ub354 \uc9c0\ucf1c\ubd10\uc57c \ub420\ub4ef\ud558\ub2e4. \ud558\uc9c0\ub9cc \uac1c\uc778\uc801\uc778 \uc0ac\uc815\uc73c\ub85c \uae30\uad8c\ud558\uc5ec \uae30\ub85d\uc740 \ubb34\uc758\ubbf8 \ud574\uc84c\ub2e4. \ub300\ud68c\ub97c \uae30\uad8c\ud558\uac8c \ub41c \uc774\ud6c4\uc5d0 \uc18c\uc591\uad50\uc721\uc5d0\uc11c\ub3c4 \ubaa8\uc2b5\uc744 \ube44\ucd94\uc9c0 \uc54a\uace0 SK\ud154\ub808\ucf64 T1\uacfc \uc5f0\ubd09\ud611\uc0c1 \uc7ac\uacc4\uc57d\uc774 \ub418\uc9c0 \uc54a\uc558\ub2e4\ub294 \uc18c\ubb38\uc774 \ub5a0\ub3cc\uba74\uc11c \uac01\uc885 \ucee4\ubba4\ub2c8\ud2f0\uc5d0\uc11c \uc740\ud1f4\uc124\uc774 \ub098\ub3cc\uc558\uc73c\ub098 SK\uac00 \ubaa8\ub4e0 \uc120\uc218\ub4e4\uacfc \uc7ac\uacc4\uc57d\uc744 \ud588\ub2e4\ub294 \uc18c\uc2dd\uc774 \ub4e4\ub9ac\uba74\uc11c \uae40\ud0dd\uc6a9\uc740 \uc120\uc218 \uc0dd\ud65c\uc744 \uc774\uc5b4\ub098\uac08\uc218 \uc788\uc5c8\ub2e4."} {"question": "\ub370\uc639\uc740 \ub204\uad6c\uc5d0\uac8c \uc11c\uc2e0\uc73c\ub85c \uc790\uc2e0\uc758 \uc554\uc0b4\uc744 \uc8fc\uc7a5\ud588\ub294\uac00?", "paragraph": "\uc0c8 \ud504\ub791\uc2a4 \ub300\uc0ac\ub85c \uac8c\ub974\uc26c \ubc31\uc791\uc774 \ubd80\uc784\ud558\uba74\uc11c \ub370\uc639\uc740 \ub2e4\uc2dc \ucc28\uad00\uc73c\ub85c \uc9c0\uc704\uac00 \ud558\ub77d\ud558\uc600\uc73c\uba70, \ubc31\uc791\uc5d0\uac8c \ubaa8\uc695\uae4c\uc9c0 \ub2f9\ud558\uc600\ub2e4. \uc774\uc5d0 \ub370\uc639\uc740 \ubd88\ub9cc\uc744 \ud488\uace0 \ud504\ub791\uc2a4\ub85c \uadc0\uad6d\ud558\ub77c\ub294 \uc815\ubd80\uc758 \uba85\ub839\uc744 \ubb34\uc2dc\ud558\uae30\ub85c \ud558\uc600\ub2e4. \ub370\uc639\uc740 \ub8e8\uc774 15\uc138\uc5d0\uac8c \ubcf4\ub0b8 \uc11c\uc2e0\uc5d0\uac8c \uc0c8\ub85c \ubd80\uc784\ud55c \ub300\uc0ac\uac00 \uc790\uae30\uc5d0\uac8c \uc57d\ubb3c\uc744 \ud22c\uc5ec\ud574 \uc554\uc0b4\ud558\ub824\uace0 \ud588\ub2e4\uace0 \uc8fc\uc7a5\ud558\uc600\ub2e4. \uadf8\ub294 \ub7f0\ub358\uc5d0\uc11c\uc758 \uc790\uae30 \uc704\uce58\ub97c \uc720\uc9c0\ud558\uae30 \uc704\ud574 1764\ub144\uc5d0 \uc790\uc2e0\uc774 \ubc1b\uc740 \ube44\ubc00\uc678\uad50\uc11c\uc2e0\uc758 \ub300\ubd80\ubd84\uc744 \ud55c \uad8c\uc758 \ucc45\uc73c\ub85c \ubb36\uc5b4 \ucd9c\ud310\ud558\uc600\ub2e4. \ub610\ud55c \uc774 \ucc45\uc5d0\uc11c \uadf8\ub294 \uac8c\ub974\uc26c \ubc31\uc791\uc774 \ud504\ub791\uc2a4 \ub300\uc0ac\ub85c \ubd80\uc801\uc808\ud55c \uc778\ubb3c\uc774\ub77c\uace0 \uc2e0\ub784\ud558\uac8c \ube44\ud310\ud558\uc600\ub2e4. \uc774\ucc98\ub7fc \uc678\uad50\uc801 \ube44\ubc00\uc744 \uacf5\uac1c\uc801\uc73c\ub85c \ub204\uc124\ud55c \uac83\uc740 \uc804\ub840 \uc5c6\ub294 \uc0ac\uac74\uc774\uc5c8\uc9c0\ub9cc, \uadf8\ub807\ub2e4\uace0 \ub370\uc639\uc774 \ubaa8\ub4e0 \ub0b4\uc6a9\uc744 \ub2e4 \uacf5\uac1c\ud55c \uac83\uc740 \uc544\ub2c8\uc5c8\ub2e4. \ub370\uc639\uc740 \uc655\uc758 \uc804\uc7c1 \uacc4\ud68d \ub4f1 \ub354 \uc911\uc694\ud558\uace0 \ub9ce\uc740 \ubb38\uc11c\ub294 \uc790\uc2e0\uc758 \uc2e0\ubcc0\ubcf4\ud638\ucc28\uc6d0\uc5d0\uc11c \uacc4\uc18d \uc228\uaca8\uac00\uc9c0\uace0 \uc788\uc5c8\ub2e4. \uc774\ud6c4 \ud504\ub791\uc2a4 \uc815\ubd80\ub294 \ub370\uc639\uacfc \uc811\ucd09\ud560 \ub54c \uad49\uc7a5\ud788 \uc2e0\uc911\uc744 \uae30\ud558\uac8c \ub418\uc5c8\ub2e4.", "answer": "\ub8e8\uc774 15\uc138", "sentence": "\ub370\uc639\uc740 \ub8e8\uc774 15\uc138 \uc5d0\uac8c \ubcf4\ub0b8 \uc11c\uc2e0\uc5d0\uac8c \uc0c8\ub85c \ubd80\uc784\ud55c \ub300\uc0ac\uac00 \uc790\uae30\uc5d0\uac8c \uc57d\ubb3c\uc744 \ud22c\uc5ec\ud574 \uc554\uc0b4\ud558\ub824\uace0 \ud588\ub2e4\uace0 \uc8fc\uc7a5\ud558\uc600\ub2e4.", "paragraph_sentence": "\uc0c8 \ud504\ub791\uc2a4 \ub300\uc0ac\ub85c \uac8c\ub974\uc26c \ubc31\uc791\uc774 \ubd80\uc784\ud558\uba74\uc11c \ub370\uc639\uc740 \ub2e4\uc2dc \ucc28\uad00\uc73c\ub85c \uc9c0\uc704\uac00 \ud558\ub77d\ud558\uc600\uc73c\uba70, \ubc31\uc791\uc5d0\uac8c \ubaa8\uc695\uae4c\uc9c0 \ub2f9\ud558\uc600\ub2e4. \uc774\uc5d0 \ub370\uc639\uc740 \ubd88\ub9cc\uc744 \ud488\uace0 \ud504\ub791\uc2a4\ub85c \uadc0\uad6d\ud558\ub77c\ub294 \uc815\ubd80\uc758 \uba85\ub839\uc744 \ubb34\uc2dc\ud558\uae30\ub85c \ud558\uc600\ub2e4. \ub370\uc639\uc740 \ub8e8\uc774 15\uc138 \uc5d0\uac8c \ubcf4\ub0b8 \uc11c\uc2e0\uc5d0\uac8c \uc0c8\ub85c \ubd80\uc784\ud55c \ub300\uc0ac\uac00 \uc790\uae30\uc5d0\uac8c \uc57d\ubb3c\uc744 \ud22c\uc5ec\ud574 \uc554\uc0b4\ud558\ub824\uace0 \ud588\ub2e4\uace0 \uc8fc\uc7a5\ud558\uc600\ub2e4. \uadf8\ub294 \ub7f0\ub358\uc5d0\uc11c\uc758 \uc790\uae30 \uc704\uce58\ub97c \uc720\uc9c0\ud558\uae30 \uc704\ud574 1764\ub144\uc5d0 \uc790\uc2e0\uc774 \ubc1b\uc740 \ube44\ubc00\uc678\uad50\uc11c\uc2e0\uc758 \ub300\ubd80\ubd84\uc744 \ud55c \uad8c\uc758 \ucc45\uc73c\ub85c \ubb36\uc5b4 \ucd9c\ud310\ud558\uc600\ub2e4. \ub610\ud55c \uc774 \ucc45\uc5d0\uc11c \uadf8\ub294 \uac8c\ub974\uc26c \ubc31\uc791\uc774 \ud504\ub791\uc2a4 \ub300\uc0ac\ub85c \ubd80\uc801\uc808\ud55c \uc778\ubb3c\uc774\ub77c\uace0 \uc2e0\ub784\ud558\uac8c \ube44\ud310\ud558\uc600\ub2e4. \uc774\ucc98\ub7fc \uc678\uad50\uc801 \ube44\ubc00\uc744 \uacf5\uac1c\uc801\uc73c\ub85c \ub204\uc124\ud55c \uac83\uc740 \uc804\ub840 \uc5c6\ub294 \uc0ac\uac74\uc774\uc5c8\uc9c0\ub9cc, \uadf8\ub807\ub2e4\uace0 \ub370\uc639\uc774 \ubaa8\ub4e0 \ub0b4\uc6a9\uc744 \ub2e4 \uacf5\uac1c\ud55c \uac83\uc740 \uc544\ub2c8\uc5c8\ub2e4. \ub370\uc639\uc740 \uc655\uc758 \uc804\uc7c1 \uacc4\ud68d \ub4f1 \ub354 \uc911\uc694\ud558\uace0 \ub9ce\uc740 \ubb38\uc11c\ub294 \uc790\uc2e0\uc758 \uc2e0\ubcc0\ubcf4\ud638\ucc28\uc6d0\uc5d0\uc11c \uacc4\uc18d \uc228\uaca8\uac00\uc9c0\uace0 \uc788\uc5c8\ub2e4. \uc774\ud6c4 \ud504\ub791\uc2a4 \uc815\ubd80\ub294 \ub370\uc639\uacfc \uc811\ucd09\ud560 \ub54c \uad49\uc7a5\ud788 \uc2e0\uc911\uc744 \uae30\ud558\uac8c \ub418\uc5c8\ub2e4.", "paragraph_answer": "\uc0c8 \ud504\ub791\uc2a4 \ub300\uc0ac\ub85c \uac8c\ub974\uc26c \ubc31\uc791\uc774 \ubd80\uc784\ud558\uba74\uc11c \ub370\uc639\uc740 \ub2e4\uc2dc \ucc28\uad00\uc73c\ub85c \uc9c0\uc704\uac00 \ud558\ub77d\ud558\uc600\uc73c\uba70, \ubc31\uc791\uc5d0\uac8c \ubaa8\uc695\uae4c\uc9c0 \ub2f9\ud558\uc600\ub2e4. \uc774\uc5d0 \ub370\uc639\uc740 \ubd88\ub9cc\uc744 \ud488\uace0 \ud504\ub791\uc2a4\ub85c \uadc0\uad6d\ud558\ub77c\ub294 \uc815\ubd80\uc758 \uba85\ub839\uc744 \ubb34\uc2dc\ud558\uae30\ub85c \ud558\uc600\ub2e4. \ub370\uc639\uc740 \ub8e8\uc774 15\uc138 \uc5d0\uac8c \ubcf4\ub0b8 \uc11c\uc2e0\uc5d0\uac8c \uc0c8\ub85c \ubd80\uc784\ud55c \ub300\uc0ac\uac00 \uc790\uae30\uc5d0\uac8c \uc57d\ubb3c\uc744 \ud22c\uc5ec\ud574 \uc554\uc0b4\ud558\ub824\uace0 \ud588\ub2e4\uace0 \uc8fc\uc7a5\ud558\uc600\ub2e4. \uadf8\ub294 \ub7f0\ub358\uc5d0\uc11c\uc758 \uc790\uae30 \uc704\uce58\ub97c \uc720\uc9c0\ud558\uae30 \uc704\ud574 1764\ub144\uc5d0 \uc790\uc2e0\uc774 \ubc1b\uc740 \ube44\ubc00\uc678\uad50\uc11c\uc2e0\uc758 \ub300\ubd80\ubd84\uc744 \ud55c \uad8c\uc758 \ucc45\uc73c\ub85c \ubb36\uc5b4 \ucd9c\ud310\ud558\uc600\ub2e4. \ub610\ud55c \uc774 \ucc45\uc5d0\uc11c \uadf8\ub294 \uac8c\ub974\uc26c \ubc31\uc791\uc774 \ud504\ub791\uc2a4 \ub300\uc0ac\ub85c \ubd80\uc801\uc808\ud55c \uc778\ubb3c\uc774\ub77c\uace0 \uc2e0\ub784\ud558\uac8c \ube44\ud310\ud558\uc600\ub2e4. \uc774\ucc98\ub7fc \uc678\uad50\uc801 \ube44\ubc00\uc744 \uacf5\uac1c\uc801\uc73c\ub85c \ub204\uc124\ud55c \uac83\uc740 \uc804\ub840 \uc5c6\ub294 \uc0ac\uac74\uc774\uc5c8\uc9c0\ub9cc, \uadf8\ub807\ub2e4\uace0 \ub370\uc639\uc774 \ubaa8\ub4e0 \ub0b4\uc6a9\uc744 \ub2e4 \uacf5\uac1c\ud55c \uac83\uc740 \uc544\ub2c8\uc5c8\ub2e4. \ub370\uc639\uc740 \uc655\uc758 \uc804\uc7c1 \uacc4\ud68d \ub4f1 \ub354 \uc911\uc694\ud558\uace0 \ub9ce\uc740 \ubb38\uc11c\ub294 \uc790\uc2e0\uc758 \uc2e0\ubcc0\ubcf4\ud638\ucc28\uc6d0\uc5d0\uc11c \uacc4\uc18d \uc228\uaca8\uac00\uc9c0\uace0 \uc788\uc5c8\ub2e4. \uc774\ud6c4 \ud504\ub791\uc2a4 \uc815\ubd80\ub294 \ub370\uc639\uacfc \uc811\ucd09\ud560 \ub54c \uad49\uc7a5\ud788 \uc2e0\uc911\uc744 \uae30\ud558\uac8c \ub418\uc5c8\ub2e4.", "sentence_answer": "\ub370\uc639\uc740 \ub8e8\uc774 15\uc138 \uc5d0\uac8c \ubcf4\ub0b8 \uc11c\uc2e0\uc5d0\uac8c \uc0c8\ub85c \ubd80\uc784\ud55c \ub300\uc0ac\uac00 \uc790\uae30\uc5d0\uac8c \uc57d\ubb3c\uc744 \ud22c\uc5ec\ud574 \uc554\uc0b4\ud558\ub824\uace0 \ud588\ub2e4\uace0 \uc8fc\uc7a5\ud558\uc600\ub2e4.", "paragraph_id": "6531685-1-0", "paragraph_question": "question: \ub370\uc639\uc740 \ub204\uad6c\uc5d0\uac8c \uc11c\uc2e0\uc73c\ub85c \uc790\uc2e0\uc758 \uc554\uc0b4\uc744 \uc8fc\uc7a5\ud588\ub294\uac00?, context: \uc0c8 \ud504\ub791\uc2a4 \ub300\uc0ac\ub85c \uac8c\ub974\uc26c \ubc31\uc791\uc774 \ubd80\uc784\ud558\uba74\uc11c \ub370\uc639\uc740 \ub2e4\uc2dc \ucc28\uad00\uc73c\ub85c \uc9c0\uc704\uac00 \ud558\ub77d\ud558\uc600\uc73c\uba70, \ubc31\uc791\uc5d0\uac8c \ubaa8\uc695\uae4c\uc9c0 \ub2f9\ud558\uc600\ub2e4. \uc774\uc5d0 \ub370\uc639\uc740 \ubd88\ub9cc\uc744 \ud488\uace0 \ud504\ub791\uc2a4\ub85c \uadc0\uad6d\ud558\ub77c\ub294 \uc815\ubd80\uc758 \uba85\ub839\uc744 \ubb34\uc2dc\ud558\uae30\ub85c \ud558\uc600\ub2e4. \ub370\uc639\uc740 \ub8e8\uc774 15\uc138\uc5d0\uac8c \ubcf4\ub0b8 \uc11c\uc2e0\uc5d0\uac8c \uc0c8\ub85c \ubd80\uc784\ud55c \ub300\uc0ac\uac00 \uc790\uae30\uc5d0\uac8c \uc57d\ubb3c\uc744 \ud22c\uc5ec\ud574 \uc554\uc0b4\ud558\ub824\uace0 \ud588\ub2e4\uace0 \uc8fc\uc7a5\ud558\uc600\ub2e4. \uadf8\ub294 \ub7f0\ub358\uc5d0\uc11c\uc758 \uc790\uae30 \uc704\uce58\ub97c \uc720\uc9c0\ud558\uae30 \uc704\ud574 1764\ub144\uc5d0 \uc790\uc2e0\uc774 \ubc1b\uc740 \ube44\ubc00\uc678\uad50\uc11c\uc2e0\uc758 \ub300\ubd80\ubd84\uc744 \ud55c \uad8c\uc758 \ucc45\uc73c\ub85c \ubb36\uc5b4 \ucd9c\ud310\ud558\uc600\ub2e4. \ub610\ud55c \uc774 \ucc45\uc5d0\uc11c \uadf8\ub294 \uac8c\ub974\uc26c \ubc31\uc791\uc774 \ud504\ub791\uc2a4 \ub300\uc0ac\ub85c \ubd80\uc801\uc808\ud55c \uc778\ubb3c\uc774\ub77c\uace0 \uc2e0\ub784\ud558\uac8c \ube44\ud310\ud558\uc600\ub2e4. \uc774\ucc98\ub7fc \uc678\uad50\uc801 \ube44\ubc00\uc744 \uacf5\uac1c\uc801\uc73c\ub85c \ub204\uc124\ud55c \uac83\uc740 \uc804\ub840 \uc5c6\ub294 \uc0ac\uac74\uc774\uc5c8\uc9c0\ub9cc, \uadf8\ub807\ub2e4\uace0 \ub370\uc639\uc774 \ubaa8\ub4e0 \ub0b4\uc6a9\uc744 \ub2e4 \uacf5\uac1c\ud55c \uac83\uc740 \uc544\ub2c8\uc5c8\ub2e4. \ub370\uc639\uc740 \uc655\uc758 \uc804\uc7c1 \uacc4\ud68d \ub4f1 \ub354 \uc911\uc694\ud558\uace0 \ub9ce\uc740 \ubb38\uc11c\ub294 \uc790\uc2e0\uc758 \uc2e0\ubcc0\ubcf4\ud638\ucc28\uc6d0\uc5d0\uc11c \uacc4\uc18d \uc228\uaca8\uac00\uc9c0\uace0 \uc788\uc5c8\ub2e4. \uc774\ud6c4 \ud504\ub791\uc2a4 \uc815\ubd80\ub294 \ub370\uc639\uacfc \uc811\ucd09\ud560 \ub54c \uad49\uc7a5\ud788 \uc2e0\uc911\uc744 \uae30\ud558\uac8c \ub418\uc5c8\ub2e4."} {"question": "\uc870\uc81c\ud504 \ube4c\ub810\ub818\uc73c\ub85c\ubd80\ud130 \ubc1b\uc740 \uc138\ub840\uba85\uc740 \ubb34\uc5c7\uc778\uac00\uc694?", "paragraph": "1897\ub144 \uc544\ubc84\uc9c0\ub97c \ub530\ub77c \ucc9c\uc8fc\uad50\uc5d0 \uc785\uad50\ud558\uc5ec \ud504\ub791\uc2a4 \ucd9c\uc2e0\uc758 \ucc9c\uc8fc\uad50 \uc2e0\ubd80\uc600\ub358 \uc870\uc81c\ud504 \ube4c\ub810\ub818(Joseph Wilhelem, \ud55c\uad6d\uba85 \ud64d\uc11d\uad6c(\u6d2a\u932b\u4e5d))\uc73c\ub85c\ubd80\ud130 \ud1a0\ub9c8\uc2a4(\ub3c4\ub9c8, \ub2e4\ubb35(\u591a\u9ed8))\ub77c\ub294 \uc138\ub840\uba85\uc744 \ubc1b\uc558\ub2e4. \uc548\uc911\uadfc\uc774 \ud64d\uc11d\uad6c \uc2e0\ubd80\uc5d0\uac8c \ud504\ub791\uc2a4\uc5b4\ub97c \ubc30\uc6b0\uae30\ub3c4 \ud588\ub2e4\ub294 \uc124\uba85\ub3c4 \uc788\uc9c0\ub9cc, \uc548\uc911\uadfc\uc740 \ud64d\uc11d\uad6c \uc2e0\ubd80\uc640 \ud568\uaed8 \ubb88\ud154\uc8fc\uad50\uc640 \ub9cc\ub098 \uc774\uc57c\uae30\ub97c \ud55c \ud6c4\uc5d0 \ud504\ub791\uc2a4\uc5b4 \ubc30\uc6b0\uae30\ub97c \uadf8\ub9cc\ub450\uc5c8\ub2e4. \u300a\uc548\uc911\uadfc\u300b(\uc870\uc815\ub798 \uc9c0\uc74c, \ubb38\ud559\ub3d9\ub124)\uc5d0 \uc758\ud558\uba74, \ubb88\ud154 \uc8fc\uad50(\ud55c\uad6d\uc774\ub984 \ubbfc\uc8fc\uad50)\uac00 \uc870\uc120\uc0ac\ub78c\ub4e4\uc774 \ud559\ubb38\uc744 \ud558\ub294 \uac83\uc744 \ubc14\ub77c\uc9c0 \uc54a\uc558\uae30 \ub54c\ubb38\uc774\uc5c8\ub2e4. \ud559\ubb38\uc744 \ud558\uba74 \uba38\ub9ac\uac00 \uae68\uc77c \uac83\uc774\uace0, \uba38\ub9ac\uac00 \uae68\uc774\uba74 \ucc9c\uc8fc\uad50 \uad50\ub9ac\uc640 \uc2e0\uc559\uc5d0 \ub300\ud574 \ub098\ub984\ub300\ub85c\uc758 \ub17c\ub9ac\uc640 \ube44\ud310\uc73c\ub85c\uc368 \uc758\ubb38\uc744 \uac00\uc9c8 \uac83\uc774\uae30 \ub54c\ubb38\uc774\uc5c8\ub2e4. \uc774\ub97c \ubcf4\uba74\uc11c \uc2e4\ub9dd\ud55c \uc548\uc911\uadfc\uc740 \ud504\ub791\uc2a4\uc5b4 \ubc30\uc6b0\uae30\ub97c \uadf8\ub9cc\ub450\uc5c8\ub2e4. \uadf8 \ub4a4 \uc7a0\uc2dc \uad50\ud68c\uc758 \ucd1d\ub300(\u7e3d\u4ee3)\ub97c \ub9e1\uc558\uace0 \ub4a4\uc5d0 \ub9cc\uc778\uacc4(\u842c\u4eba\u5951\uff1a1,000\uba85 \uc774\uc0c1\uc758 \uacc4\uc6d0\uc744 \ubaa8\uc544 \ub3c8\uc744 \ucd9c\uc790\ud55c \ub4a4 \ucd94\ucca8\uc774\ub098 \uc785\ucc30\ub85c \ub3c8\uc744 \uc735\ud1b5\ud574 \uc8fc\ub294 \ubaa8\uc784)\uc758 \ucc44\ud45c\ud68c\uc0ac(\u5f69\u7968\u6703\u793e\uff1a\ub9cc\uc778\uacc4\uc758 \ub3c8\uc744 \uad00\ub9ac\ud558\uace0 \ucd94\ucca8\uc744 \ud558\ub294 \ud68c\uc0ac)\ub97c \uc124\ub9bd\ud558\uace0 \uc0ac\uc7a5\uc774 \ub418\uc5c8\ub2e4.", "answer": "\ud1a0\ub9c8\uc2a4", "sentence": "1897\ub144 \uc544\ubc84\uc9c0\ub97c \ub530\ub77c \ucc9c\uc8fc\uad50\uc5d0 \uc785\uad50\ud558\uc5ec \ud504\ub791\uc2a4 \ucd9c\uc2e0\uc758 \ucc9c\uc8fc\uad50 \uc2e0\ubd80\uc600\ub358 \uc870\uc81c\ud504 \ube4c\ub810\ub818(Joseph Wilhelem, \ud55c\uad6d\uba85 \ud64d\uc11d\uad6c(\u6d2a\u932b\u4e5d))\uc73c\ub85c\ubd80\ud130 \ud1a0\ub9c8\uc2a4 (\ub3c4\ub9c8, \ub2e4\ubb35(\u591a\u9ed8))\ub77c\ub294 \uc138\ub840\uba85\uc744 \ubc1b\uc558\ub2e4.", "paragraph_sentence": " 1897\ub144 \uc544\ubc84\uc9c0\ub97c \ub530\ub77c \ucc9c\uc8fc\uad50\uc5d0 \uc785\uad50\ud558\uc5ec \ud504\ub791\uc2a4 \ucd9c\uc2e0\uc758 \ucc9c\uc8fc\uad50 \uc2e0\ubd80\uc600\ub358 \uc870\uc81c\ud504 \ube4c\ub810\ub818(Joseph Wilhelem, \ud55c\uad6d\uba85 \ud64d\uc11d\uad6c(\u6d2a\u932b\u4e5d))\uc73c\ub85c\ubd80\ud130 \ud1a0\ub9c8\uc2a4 (\ub3c4\ub9c8, \ub2e4\ubb35(\u591a\u9ed8))\ub77c\ub294 \uc138\ub840\uba85\uc744 \ubc1b\uc558\ub2e4. \uc548\uc911\uadfc\uc774 \ud64d\uc11d\uad6c \uc2e0\ubd80\uc5d0\uac8c \ud504\ub791\uc2a4\uc5b4\ub97c \ubc30\uc6b0\uae30\ub3c4 \ud588\ub2e4\ub294 \uc124\uba85\ub3c4 \uc788\uc9c0\ub9cc, \uc548\uc911\uadfc\uc740 \ud64d\uc11d\uad6c \uc2e0\ubd80\uc640 \ud568\uaed8 \ubb88\ud154\uc8fc\uad50\uc640 \ub9cc\ub098 \uc774\uc57c\uae30\ub97c \ud55c \ud6c4\uc5d0 \ud504\ub791\uc2a4\uc5b4 \ubc30\uc6b0\uae30\ub97c \uadf8\ub9cc\ub450\uc5c8\ub2e4. \u300a\uc548\uc911\uadfc\u300b(\uc870\uc815\ub798 \uc9c0\uc74c, \ubb38\ud559\ub3d9\ub124)\uc5d0 \uc758\ud558\uba74, \ubb88\ud154 \uc8fc\uad50(\ud55c\uad6d\uc774\ub984 \ubbfc\uc8fc\uad50)\uac00 \uc870\uc120\uc0ac\ub78c\ub4e4\uc774 \ud559\ubb38\uc744 \ud558\ub294 \uac83\uc744 \ubc14\ub77c\uc9c0 \uc54a\uc558\uae30 \ub54c\ubb38\uc774\uc5c8\ub2e4. \ud559\ubb38\uc744 \ud558\uba74 \uba38\ub9ac\uac00 \uae68\uc77c \uac83\uc774\uace0, \uba38\ub9ac\uac00 \uae68\uc774\uba74 \ucc9c\uc8fc\uad50 \uad50\ub9ac\uc640 \uc2e0\uc559\uc5d0 \ub300\ud574 \ub098\ub984\ub300\ub85c\uc758 \ub17c\ub9ac\uc640 \ube44\ud310\uc73c\ub85c\uc368 \uc758\ubb38\uc744 \uac00\uc9c8 \uac83\uc774\uae30 \ub54c\ubb38\uc774\uc5c8\ub2e4. \uc774\ub97c \ubcf4\uba74\uc11c \uc2e4\ub9dd\ud55c \uc548\uc911\uadfc\uc740 \ud504\ub791\uc2a4\uc5b4 \ubc30\uc6b0\uae30\ub97c \uadf8\ub9cc\ub450\uc5c8\ub2e4. \uadf8 \ub4a4 \uc7a0\uc2dc \uad50\ud68c\uc758 \ucd1d\ub300(\u7e3d\u4ee3)\ub97c \ub9e1\uc558\uace0 \ub4a4\uc5d0 \ub9cc\uc778\uacc4(\u842c\u4eba\u5951\uff1a1,000\uba85 \uc774\uc0c1\uc758 \uacc4\uc6d0\uc744 \ubaa8\uc544 \ub3c8\uc744 \ucd9c\uc790\ud55c \ub4a4 \ucd94\ucca8\uc774\ub098 \uc785\ucc30\ub85c \ub3c8\uc744 \uc735\ud1b5\ud574 \uc8fc\ub294 \ubaa8\uc784)\uc758 \ucc44\ud45c\ud68c\uc0ac(\u5f69\u7968\u6703\u793e\uff1a\ub9cc\uc778\uacc4\uc758 \ub3c8\uc744 \uad00\ub9ac\ud558\uace0 \ucd94\ucca8\uc744 \ud558\ub294 \ud68c\uc0ac)\ub97c \uc124\ub9bd\ud558\uace0 \uc0ac\uc7a5\uc774 \ub418\uc5c8\ub2e4.", "paragraph_answer": "1897\ub144 \uc544\ubc84\uc9c0\ub97c \ub530\ub77c \ucc9c\uc8fc\uad50\uc5d0 \uc785\uad50\ud558\uc5ec \ud504\ub791\uc2a4 \ucd9c\uc2e0\uc758 \ucc9c\uc8fc\uad50 \uc2e0\ubd80\uc600\ub358 \uc870\uc81c\ud504 \ube4c\ub810\ub818(Joseph Wilhelem, \ud55c\uad6d\uba85 \ud64d\uc11d\uad6c(\u6d2a\u932b\u4e5d))\uc73c\ub85c\ubd80\ud130 \ud1a0\ub9c8\uc2a4 (\ub3c4\ub9c8, \ub2e4\ubb35(\u591a\u9ed8))\ub77c\ub294 \uc138\ub840\uba85\uc744 \ubc1b\uc558\ub2e4. \uc548\uc911\uadfc\uc774 \ud64d\uc11d\uad6c \uc2e0\ubd80\uc5d0\uac8c \ud504\ub791\uc2a4\uc5b4\ub97c \ubc30\uc6b0\uae30\ub3c4 \ud588\ub2e4\ub294 \uc124\uba85\ub3c4 \uc788\uc9c0\ub9cc, \uc548\uc911\uadfc\uc740 \ud64d\uc11d\uad6c \uc2e0\ubd80\uc640 \ud568\uaed8 \ubb88\ud154\uc8fc\uad50\uc640 \ub9cc\ub098 \uc774\uc57c\uae30\ub97c \ud55c \ud6c4\uc5d0 \ud504\ub791\uc2a4\uc5b4 \ubc30\uc6b0\uae30\ub97c \uadf8\ub9cc\ub450\uc5c8\ub2e4. \u300a\uc548\uc911\uadfc\u300b(\uc870\uc815\ub798 \uc9c0\uc74c, \ubb38\ud559\ub3d9\ub124)\uc5d0 \uc758\ud558\uba74, \ubb88\ud154 \uc8fc\uad50(\ud55c\uad6d\uc774\ub984 \ubbfc\uc8fc\uad50)\uac00 \uc870\uc120\uc0ac\ub78c\ub4e4\uc774 \ud559\ubb38\uc744 \ud558\ub294 \uac83\uc744 \ubc14\ub77c\uc9c0 \uc54a\uc558\uae30 \ub54c\ubb38\uc774\uc5c8\ub2e4. \ud559\ubb38\uc744 \ud558\uba74 \uba38\ub9ac\uac00 \uae68\uc77c \uac83\uc774\uace0, \uba38\ub9ac\uac00 \uae68\uc774\uba74 \ucc9c\uc8fc\uad50 \uad50\ub9ac\uc640 \uc2e0\uc559\uc5d0 \ub300\ud574 \ub098\ub984\ub300\ub85c\uc758 \ub17c\ub9ac\uc640 \ube44\ud310\uc73c\ub85c\uc368 \uc758\ubb38\uc744 \uac00\uc9c8 \uac83\uc774\uae30 \ub54c\ubb38\uc774\uc5c8\ub2e4. \uc774\ub97c \ubcf4\uba74\uc11c \uc2e4\ub9dd\ud55c \uc548\uc911\uadfc\uc740 \ud504\ub791\uc2a4\uc5b4 \ubc30\uc6b0\uae30\ub97c \uadf8\ub9cc\ub450\uc5c8\ub2e4. \uadf8 \ub4a4 \uc7a0\uc2dc \uad50\ud68c\uc758 \ucd1d\ub300(\u7e3d\u4ee3)\ub97c \ub9e1\uc558\uace0 \ub4a4\uc5d0 \ub9cc\uc778\uacc4(\u842c\u4eba\u5951\uff1a1,000\uba85 \uc774\uc0c1\uc758 \uacc4\uc6d0\uc744 \ubaa8\uc544 \ub3c8\uc744 \ucd9c\uc790\ud55c \ub4a4 \ucd94\ucca8\uc774\ub098 \uc785\ucc30\ub85c \ub3c8\uc744 \uc735\ud1b5\ud574 \uc8fc\ub294 \ubaa8\uc784)\uc758 \ucc44\ud45c\ud68c\uc0ac(\u5f69\u7968\u6703\u793e\uff1a\ub9cc\uc778\uacc4\uc758 \ub3c8\uc744 \uad00\ub9ac\ud558\uace0 \ucd94\ucca8\uc744 \ud558\ub294 \ud68c\uc0ac)\ub97c \uc124\ub9bd\ud558\uace0 \uc0ac\uc7a5\uc774 \ub418\uc5c8\ub2e4.", "sentence_answer": "1897\ub144 \uc544\ubc84\uc9c0\ub97c \ub530\ub77c \ucc9c\uc8fc\uad50\uc5d0 \uc785\uad50\ud558\uc5ec \ud504\ub791\uc2a4 \ucd9c\uc2e0\uc758 \ucc9c\uc8fc\uad50 \uc2e0\ubd80\uc600\ub358 \uc870\uc81c\ud504 \ube4c\ub810\ub818(Joseph Wilhelem, \ud55c\uad6d\uba85 \ud64d\uc11d\uad6c(\u6d2a\u932b\u4e5d))\uc73c\ub85c\ubd80\ud130 \ud1a0\ub9c8\uc2a4 (\ub3c4\ub9c8, \ub2e4\ubb35(\u591a\u9ed8))\ub77c\ub294 \uc138\ub840\uba85\uc744 \ubc1b\uc558\ub2e4.", "paragraph_id": "6521585-2-1", "paragraph_question": "question: \uc870\uc81c\ud504 \ube4c\ub810\ub818\uc73c\ub85c\ubd80\ud130 \ubc1b\uc740 \uc138\ub840\uba85\uc740 \ubb34\uc5c7\uc778\uac00\uc694?, context: 1897\ub144 \uc544\ubc84\uc9c0\ub97c \ub530\ub77c \ucc9c\uc8fc\uad50\uc5d0 \uc785\uad50\ud558\uc5ec \ud504\ub791\uc2a4 \ucd9c\uc2e0\uc758 \ucc9c\uc8fc\uad50 \uc2e0\ubd80\uc600\ub358 \uc870\uc81c\ud504 \ube4c\ub810\ub818(Joseph Wilhelem, \ud55c\uad6d\uba85 \ud64d\uc11d\uad6c(\u6d2a\u932b\u4e5d))\uc73c\ub85c\ubd80\ud130 \ud1a0\ub9c8\uc2a4(\ub3c4\ub9c8, \ub2e4\ubb35(\u591a\u9ed8))\ub77c\ub294 \uc138\ub840\uba85\uc744 \ubc1b\uc558\ub2e4. \uc548\uc911\uadfc\uc774 \ud64d\uc11d\uad6c \uc2e0\ubd80\uc5d0\uac8c \ud504\ub791\uc2a4\uc5b4\ub97c \ubc30\uc6b0\uae30\ub3c4 \ud588\ub2e4\ub294 \uc124\uba85\ub3c4 \uc788\uc9c0\ub9cc, \uc548\uc911\uadfc\uc740 \ud64d\uc11d\uad6c \uc2e0\ubd80\uc640 \ud568\uaed8 \ubb88\ud154\uc8fc\uad50\uc640 \ub9cc\ub098 \uc774\uc57c\uae30\ub97c \ud55c \ud6c4\uc5d0 \ud504\ub791\uc2a4\uc5b4 \ubc30\uc6b0\uae30\ub97c \uadf8\ub9cc\ub450\uc5c8\ub2e4. \u300a\uc548\uc911\uadfc\u300b(\uc870\uc815\ub798 \uc9c0\uc74c, \ubb38\ud559\ub3d9\ub124)\uc5d0 \uc758\ud558\uba74, \ubb88\ud154 \uc8fc\uad50(\ud55c\uad6d\uc774\ub984 \ubbfc\uc8fc\uad50)\uac00 \uc870\uc120\uc0ac\ub78c\ub4e4\uc774 \ud559\ubb38\uc744 \ud558\ub294 \uac83\uc744 \ubc14\ub77c\uc9c0 \uc54a\uc558\uae30 \ub54c\ubb38\uc774\uc5c8\ub2e4. \ud559\ubb38\uc744 \ud558\uba74 \uba38\ub9ac\uac00 \uae68\uc77c \uac83\uc774\uace0, \uba38\ub9ac\uac00 \uae68\uc774\uba74 \ucc9c\uc8fc\uad50 \uad50\ub9ac\uc640 \uc2e0\uc559\uc5d0 \ub300\ud574 \ub098\ub984\ub300\ub85c\uc758 \ub17c\ub9ac\uc640 \ube44\ud310\uc73c\ub85c\uc368 \uc758\ubb38\uc744 \uac00\uc9c8 \uac83\uc774\uae30 \ub54c\ubb38\uc774\uc5c8\ub2e4. \uc774\ub97c \ubcf4\uba74\uc11c \uc2e4\ub9dd\ud55c \uc548\uc911\uadfc\uc740 \ud504\ub791\uc2a4\uc5b4 \ubc30\uc6b0\uae30\ub97c \uadf8\ub9cc\ub450\uc5c8\ub2e4. \uadf8 \ub4a4 \uc7a0\uc2dc \uad50\ud68c\uc758 \ucd1d\ub300(\u7e3d\u4ee3)\ub97c \ub9e1\uc558\uace0 \ub4a4\uc5d0 \ub9cc\uc778\uacc4(\u842c\u4eba\u5951\uff1a1,000\uba85 \uc774\uc0c1\uc758 \uacc4\uc6d0\uc744 \ubaa8\uc544 \ub3c8\uc744 \ucd9c\uc790\ud55c \ub4a4 \ucd94\ucca8\uc774\ub098 \uc785\ucc30\ub85c \ub3c8\uc744 \uc735\ud1b5\ud574 \uc8fc\ub294 \ubaa8\uc784)\uc758 \ucc44\ud45c\ud68c\uc0ac(\u5f69\u7968\u6703\u793e\uff1a\ub9cc\uc778\uacc4\uc758 \ub3c8\uc744 \uad00\ub9ac\ud558\uace0 \ucd94\ucca8\uc744 \ud558\ub294 \ud68c\uc0ac)\ub97c \uc124\ub9bd\ud558\uace0 \uc0ac\uc7a5\uc774 \ub418\uc5c8\ub2e4."} {"question": "\uc138\uc6b4\uc0c1\uac00\ub294 \ubb34\uc5c7\uc744 \uc8fc\ub85c \ud30c\ub294 \uc0c1\uac00\uc600\ub294\uac00?", "paragraph": "\uc138\uc6b4\uc0c1\uac00\ub294 \uc11c\uc6b8\uc758 \uc720\uc77c\ubb34\uc774\ud55c \uc885\ud569 \uac00\uc804\uc81c\ud488 \uc0c1\uac00\ub85c\ub3c4 \ubc1c\uc804\ud588\ub2e4. \ud2b9\ud788 80\ub144\ub300 \ub9d0 \uac1c\uc778\uc6a9 \ucef4\ud4e8\ud130\uc758 \ubc1c\uc804\uc73c\ub85c \uc804\uc131\uae30\ub97c \uad6c\uac00\ud558\uac8c \ub418\uc5c8\ub2e4. \ud55c\uad6d\uc758 8\ube44\ud2b8\uc640 \uc774\ud6c4 16\ube44\ud2b8 \ucef4\ud4e8\ud130, \uadf8\ub9ac\uace0 \uc18c\ud504\ud2b8\uc6e8\uc5b4 \ub300\ubd80\ubd84\uc774 \uc138\uc6b4\uc0c1\uac00\uc5d0\uc11c \uac70\ub798\ub418\uc5c8\ub2e4. 1987\ub144 \uc800\uc791\uad8c\ubc95\uc774 \ub3c4\uc785\ub418\uae30 \uc804 \ud55c\ub3d9\uc548 \uc18c\ud504\ud2b8\uc6e8\uc5b4\ub97c \uce74\ud53c\ud558\ub294 \uce74\ud53c\uc810\uc774 \uc131\ud589\ud588\ub2e4. \uc138\uc6b4\uc0c1\uac00 \uc0ac\ub78c\ub4e4\uc774 \ubaa8\uc774\uba74 \ubbf8\uc0ac\uc77c\uc774\ub098 \uc7a0\uc218\ud568\ub3c4 \ub9cc\ub4e4 \uc218 \uc788\ub2e4\ub294 \uc6b0\uc2a4\uac2f\uc18c\ub9ac\ub3c4 \uc788\uc5c8\ub2e4. \uc774\ud6c4 1987\ub144 \uc815\ubd80\uac00 \uc6a9\uc0b0\uc5ed \uc11c\ubd80\uccad\uacfc\ubb3c\uc2dc\uc7a5\uc744 \uac00\ub77d\ub3d9 \ub18d\uc218\uc0b0\ubb3c\ub3c4\ub9e4\uc2dc\uc7a5\uc73c\ub85c \uc774\uc804\ud558\uace0, \uadf8 \uc790\ub9ac\uc5d0 \uc6a9\uc0b0\uc804\uc790\uc0c1\uac00\ub97c \uc870\uc131\ud574 \uc138\uc6b4\uc0c1\uac00\uc758 \uc804\uc790\uc0c1\uc744 \uc774\uc804\ud55c\ub2e4\ub294 \uacc4\ud68d\uc744 \uc138\uc6e0\ub2e4. \uc2e4\uc81c\ub85c\ub3c4 \ub9ce\uc740 \uc218\uac00 \uc774\uc804\ud558\uc600\uace0, \uc77c\ubd80\ub294 \uc774\uc5d0 \ubc18\ubc1c\ud558\uace0 \uadf8 \uc790\ub9ac\ub97c \uc9c0\ucf30\uc9c0\ub9cc \uc138\uc6b4\uc0c1\uac00\ub97c \ucc3e\ub294 \uc0ac\ub78c\uc740 \ub354\uc6b1 \uc904\uc5b4\ub4e4\uc5c8\ub2e4.", "answer": "\uac00\uc804\uc81c\ud488", "sentence": "\uc138\uc6b4\uc0c1\uac00\ub294 \uc11c\uc6b8\uc758 \uc720\uc77c\ubb34\uc774\ud55c \uc885\ud569 \uac00\uc804\uc81c\ud488 \uc0c1\uac00\ub85c\ub3c4 \ubc1c\uc804\ud588\ub2e4.", "paragraph_sentence": " \uc138\uc6b4\uc0c1\uac00\ub294 \uc11c\uc6b8\uc758 \uc720\uc77c\ubb34\uc774\ud55c \uc885\ud569 \uac00\uc804\uc81c\ud488 \uc0c1\uac00\ub85c\ub3c4 \ubc1c\uc804\ud588\ub2e4. \ud2b9\ud788 80\ub144\ub300 \ub9d0 \uac1c\uc778\uc6a9 \ucef4\ud4e8\ud130\uc758 \ubc1c\uc804\uc73c\ub85c \uc804\uc131\uae30\ub97c \uad6c\uac00\ud558\uac8c \ub418\uc5c8\ub2e4. \ud55c\uad6d\uc758 8\ube44\ud2b8\uc640 \uc774\ud6c4 16\ube44\ud2b8 \ucef4\ud4e8\ud130, \uadf8\ub9ac\uace0 \uc18c\ud504\ud2b8\uc6e8\uc5b4 \ub300\ubd80\ubd84\uc774 \uc138\uc6b4\uc0c1\uac00\uc5d0\uc11c \uac70\ub798\ub418\uc5c8\ub2e4. 1987\ub144 \uc800\uc791\uad8c\ubc95\uc774 \ub3c4\uc785\ub418\uae30 \uc804 \ud55c\ub3d9\uc548 \uc18c\ud504\ud2b8\uc6e8\uc5b4\ub97c \uce74\ud53c\ud558\ub294 \uce74\ud53c\uc810\uc774 \uc131\ud589\ud588\ub2e4. \uc138\uc6b4\uc0c1\uac00 \uc0ac\ub78c\ub4e4\uc774 \ubaa8\uc774\uba74 \ubbf8\uc0ac\uc77c\uc774\ub098 \uc7a0\uc218\ud568\ub3c4 \ub9cc\ub4e4 \uc218 \uc788\ub2e4\ub294 \uc6b0\uc2a4\uac2f\uc18c\ub9ac\ub3c4 \uc788\uc5c8\ub2e4. \uc774\ud6c4 1987\ub144 \uc815\ubd80\uac00 \uc6a9\uc0b0\uc5ed \uc11c\ubd80\uccad\uacfc\ubb3c\uc2dc\uc7a5\uc744 \uac00\ub77d\ub3d9 \ub18d\uc218\uc0b0\ubb3c\ub3c4\ub9e4\uc2dc\uc7a5\uc73c\ub85c \uc774\uc804\ud558\uace0, \uadf8 \uc790\ub9ac\uc5d0 \uc6a9\uc0b0\uc804\uc790\uc0c1\uac00\ub97c \uc870\uc131\ud574 \uc138\uc6b4\uc0c1\uac00\uc758 \uc804\uc790\uc0c1\uc744 \uc774\uc804\ud55c\ub2e4\ub294 \uacc4\ud68d\uc744 \uc138\uc6e0\ub2e4. \uc2e4\uc81c\ub85c\ub3c4 \ub9ce\uc740 \uc218\uac00 \uc774\uc804\ud558\uc600\uace0, \uc77c\ubd80\ub294 \uc774\uc5d0 \ubc18\ubc1c\ud558\uace0 \uadf8 \uc790\ub9ac\ub97c \uc9c0\ucf30\uc9c0\ub9cc \uc138\uc6b4\uc0c1\uac00\ub97c \ucc3e\ub294 \uc0ac\ub78c\uc740 \ub354\uc6b1 \uc904\uc5b4\ub4e4\uc5c8\ub2e4.", "paragraph_answer": "\uc138\uc6b4\uc0c1\uac00\ub294 \uc11c\uc6b8\uc758 \uc720\uc77c\ubb34\uc774\ud55c \uc885\ud569 \uac00\uc804\uc81c\ud488 \uc0c1\uac00\ub85c\ub3c4 \ubc1c\uc804\ud588\ub2e4. \ud2b9\ud788 80\ub144\ub300 \ub9d0 \uac1c\uc778\uc6a9 \ucef4\ud4e8\ud130\uc758 \ubc1c\uc804\uc73c\ub85c \uc804\uc131\uae30\ub97c \uad6c\uac00\ud558\uac8c \ub418\uc5c8\ub2e4. \ud55c\uad6d\uc758 8\ube44\ud2b8\uc640 \uc774\ud6c4 16\ube44\ud2b8 \ucef4\ud4e8\ud130, \uadf8\ub9ac\uace0 \uc18c\ud504\ud2b8\uc6e8\uc5b4 \ub300\ubd80\ubd84\uc774 \uc138\uc6b4\uc0c1\uac00\uc5d0\uc11c \uac70\ub798\ub418\uc5c8\ub2e4. 1987\ub144 \uc800\uc791\uad8c\ubc95\uc774 \ub3c4\uc785\ub418\uae30 \uc804 \ud55c\ub3d9\uc548 \uc18c\ud504\ud2b8\uc6e8\uc5b4\ub97c \uce74\ud53c\ud558\ub294 \uce74\ud53c\uc810\uc774 \uc131\ud589\ud588\ub2e4. \uc138\uc6b4\uc0c1\uac00 \uc0ac\ub78c\ub4e4\uc774 \ubaa8\uc774\uba74 \ubbf8\uc0ac\uc77c\uc774\ub098 \uc7a0\uc218\ud568\ub3c4 \ub9cc\ub4e4 \uc218 \uc788\ub2e4\ub294 \uc6b0\uc2a4\uac2f\uc18c\ub9ac\ub3c4 \uc788\uc5c8\ub2e4. \uc774\ud6c4 1987\ub144 \uc815\ubd80\uac00 \uc6a9\uc0b0\uc5ed \uc11c\ubd80\uccad\uacfc\ubb3c\uc2dc\uc7a5\uc744 \uac00\ub77d\ub3d9 \ub18d\uc218\uc0b0\ubb3c\ub3c4\ub9e4\uc2dc\uc7a5\uc73c\ub85c \uc774\uc804\ud558\uace0, \uadf8 \uc790\ub9ac\uc5d0 \uc6a9\uc0b0\uc804\uc790\uc0c1\uac00\ub97c \uc870\uc131\ud574 \uc138\uc6b4\uc0c1\uac00\uc758 \uc804\uc790\uc0c1\uc744 \uc774\uc804\ud55c\ub2e4\ub294 \uacc4\ud68d\uc744 \uc138\uc6e0\ub2e4. \uc2e4\uc81c\ub85c\ub3c4 \ub9ce\uc740 \uc218\uac00 \uc774\uc804\ud558\uc600\uace0, \uc77c\ubd80\ub294 \uc774\uc5d0 \ubc18\ubc1c\ud558\uace0 \uadf8 \uc790\ub9ac\ub97c \uc9c0\ucf30\uc9c0\ub9cc \uc138\uc6b4\uc0c1\uac00\ub97c \ucc3e\ub294 \uc0ac\ub78c\uc740 \ub354\uc6b1 \uc904\uc5b4\ub4e4\uc5c8\ub2e4.", "sentence_answer": "\uc138\uc6b4\uc0c1\uac00\ub294 \uc11c\uc6b8\uc758 \uc720\uc77c\ubb34\uc774\ud55c \uc885\ud569 \uac00\uc804\uc81c\ud488 \uc0c1\uac00\ub85c\ub3c4 \ubc1c\uc804\ud588\ub2e4.", "paragraph_id": "6444039-3-1", "paragraph_question": "question: \uc138\uc6b4\uc0c1\uac00\ub294 \ubb34\uc5c7\uc744 \uc8fc\ub85c \ud30c\ub294 \uc0c1\uac00\uc600\ub294\uac00?, context: \uc138\uc6b4\uc0c1\uac00\ub294 \uc11c\uc6b8\uc758 \uc720\uc77c\ubb34\uc774\ud55c \uc885\ud569 \uac00\uc804\uc81c\ud488 \uc0c1\uac00\ub85c\ub3c4 \ubc1c\uc804\ud588\ub2e4. \ud2b9\ud788 80\ub144\ub300 \ub9d0 \uac1c\uc778\uc6a9 \ucef4\ud4e8\ud130\uc758 \ubc1c\uc804\uc73c\ub85c \uc804\uc131\uae30\ub97c \uad6c\uac00\ud558\uac8c \ub418\uc5c8\ub2e4. \ud55c\uad6d\uc758 8\ube44\ud2b8\uc640 \uc774\ud6c4 16\ube44\ud2b8 \ucef4\ud4e8\ud130, \uadf8\ub9ac\uace0 \uc18c\ud504\ud2b8\uc6e8\uc5b4 \ub300\ubd80\ubd84\uc774 \uc138\uc6b4\uc0c1\uac00\uc5d0\uc11c \uac70\ub798\ub418\uc5c8\ub2e4. 1987\ub144 \uc800\uc791\uad8c\ubc95\uc774 \ub3c4\uc785\ub418\uae30 \uc804 \ud55c\ub3d9\uc548 \uc18c\ud504\ud2b8\uc6e8\uc5b4\ub97c \uce74\ud53c\ud558\ub294 \uce74\ud53c\uc810\uc774 \uc131\ud589\ud588\ub2e4. \uc138\uc6b4\uc0c1\uac00 \uc0ac\ub78c\ub4e4\uc774 \ubaa8\uc774\uba74 \ubbf8\uc0ac\uc77c\uc774\ub098 \uc7a0\uc218\ud568\ub3c4 \ub9cc\ub4e4 \uc218 \uc788\ub2e4\ub294 \uc6b0\uc2a4\uac2f\uc18c\ub9ac\ub3c4 \uc788\uc5c8\ub2e4. \uc774\ud6c4 1987\ub144 \uc815\ubd80\uac00 \uc6a9\uc0b0\uc5ed \uc11c\ubd80\uccad\uacfc\ubb3c\uc2dc\uc7a5\uc744 \uac00\ub77d\ub3d9 \ub18d\uc218\uc0b0\ubb3c\ub3c4\ub9e4\uc2dc\uc7a5\uc73c\ub85c \uc774\uc804\ud558\uace0, \uadf8 \uc790\ub9ac\uc5d0 \uc6a9\uc0b0\uc804\uc790\uc0c1\uac00\ub97c \uc870\uc131\ud574 \uc138\uc6b4\uc0c1\uac00\uc758 \uc804\uc790\uc0c1\uc744 \uc774\uc804\ud55c\ub2e4\ub294 \uacc4\ud68d\uc744 \uc138\uc6e0\ub2e4. \uc2e4\uc81c\ub85c\ub3c4 \ub9ce\uc740 \uc218\uac00 \uc774\uc804\ud558\uc600\uace0, \uc77c\ubd80\ub294 \uc774\uc5d0 \ubc18\ubc1c\ud558\uace0 \uadf8 \uc790\ub9ac\ub97c \uc9c0\ucf30\uc9c0\ub9cc \uc138\uc6b4\uc0c1\uac00\ub97c \ucc3e\ub294 \uc0ac\ub78c\uc740 \ub354\uc6b1 \uc904\uc5b4\ub4e4\uc5c8\ub2e4."} {"question": "2010\ub144 FIFA \uc6d4\ub4dc\ucef5 \uacb0\uc2b9\uc804\uc5d0 \ucd9c\uc804\ud55c \uc2a4\ud398\uc778\uc758 \uace8\ud0a4\ud37c\ub294 \ub204\uad6c\uc778\uac00?", "paragraph": "\ub124\ub35c\ub780\ub4dc\ub294 \uba87 \ucc28\ub840 \ub4dd\uc810 \uae30\ud68c\ub97c \uc7a1\uc558\ub294\ub370, \ud2b9\ud788 62\ubd84 \ubca0\uc2ac\ub808\uc774 \uc2a4\ub124\uc774\ub354\ub974\uc758 \uce68\ud22c \ud328\uc2a4\ub97c \uc544\ub974\uc5f0 \ub85c\ubc88\uc774 \ubc1b\uc544 \uc2a4\ud398\uc778 \uc218\ubb38\uc7a5 \uc774\ucf00\ub974 \uce74\uc2dc\uc57c\uc2a4\uc640 1\ub3001 \uc0c1\ud669\uc744 \ucc3d\ucd9c\ud588\uc9c0\ub9cc, \uce74\uc2dc\uc57c\uc2a4\uac00 \ub2e4\ub9ac\ub97c \ucb49 \ubed7\uc5b4 \uc29b\uc744 \ubb34\ud6a8\ud654\uc2dc\ucf30\ub2e4. \uc2a4\ud398\uc778\uc758 \uacbd\uc6b0\uc5d0\ub294 \uacac\uc81c\ubc1b\uc9c0 \uc54a\ub358 \uc138\ub974\ud788\uc624 \ub77c\ubaa8\uc2a4\uac00 \ucf54\ub108\ud0a5\uc744 \uba38\ub9ac\ub85c \uc5f0\uacb0\ud558\ub824 \ud588\uc9c0\ub9cc \ube57\ub098\uac14\ub2e4. \ub124\ub35c\ub780\ub4dc \uc8fc\uc7a5 \ud788\uc624\ubc14\ub2c8 \ud310 \ube0c\ub860\ud06c\ud638\ub974\uc2a4\ud2b8\uac00 105\ubd84\uc5d0 \uc5e3\uc190 \ube0c\ub77c\ud504\ud5e4\uc774\ud2b8\ub85c \uad50\uccb4 \ud22c\uc785\ub418\uc5c8\ub2e4. \ub808\uc54c \ub9c8\ub4dc\ub9ac\ub4dc \ubbf8\ub4dc\ud544\ub354 \ub77c\ud30c\uc5d8 \ud310 \ub370\ub974 \ud30c\ub974\ud2b8\uac00 99\ubd84\uc5d0 \ub098\uc774\uc808 \ub354 \uc6a9\uacfc \uad50\uccb4\ub85c \ub4e4\uc5b4\uac00, \ub0a8\uc740 15\ubd84\ub3d9\uc548 \uc8fc\uc7a5 \uc644\uc7a5\uc744 \ucc3c\ub2e4. 109\ubd84\uc744 \uae30\uc810\uc73c\ub85c \ub124\ub35c\ub780\ub4dc\ub294 \ud5e4\uc774\ud305\uc544\uac00 \ub450\ubc88\uc9f8 \uc610\ub85c\uce74\ub4dc\ub97c \ubc1b\uc73c\uba74\uc11c 10\uba85\uc73c\ub85c \uc904\uc5b4\ub4e4\uc5c8\ub2e4. \uc2b9\ubd80\ucc28\uae30\uac00 \ub2e4\uac00\uc62c \ubb34\ub835, \ud5e4\uc218\uc2a4 \ub098\ubc14\uc2a4\uac00 \ub124\ub35c\ub780\ub4dc \ubb38\uc804\uc73c\ub85c \uc1c4\ub3c4\ud558\uc5ec \ud328\uc2a4\uc758 \uc2dc\ubc1c\uc810\uc774 \ub418\uc5c8\uace0, \uc774\ub2c8\uc5d0\uc2a4\ud0c0\uac00 \uc138\uc2a4\ud06c \ud30c\ube0c\ub808\uac00\uc2a4\uc758 \ud398\ub110\ud2f0 \uad6c\uc5ed \uc6b0\uce21\uc73c\ub85c\ubd80\ud130\uc758 \ud328\uc2a4\ub97c \ubc1b\uc544 \uc5f0\uc7a5\uc804 \uc885\ub8cc 4\ubd84\uc744 \ub0a8\uaca8\ub193\uace0 \uc624\ub978\ubc1c\ub85c \ubc1c\ub9ac\uc29b\uc744 \uc3d8\uc544 \uacf5\uc774 \uace8\ud0a4\ud37c \uc6b0\uce21\uc744 \ub118\uae30\uba74\uc11c \ubb34\ub4dd\uc810 \uce68\ubb35\uc744 \uae7c\ub2e4.", "answer": "\uc774\ucf00\ub974 \uce74\uc2dc\uc57c\uc2a4", "sentence": "\ub124\ub35c\ub780\ub4dc\ub294 \uba87 \ucc28\ub840 \ub4dd\uc810 \uae30\ud68c\ub97c \uc7a1\uc558\ub294\ub370, \ud2b9\ud788 62\ubd84 \ubca0\uc2ac\ub808\uc774 \uc2a4\ub124\uc774\ub354\ub974\uc758 \uce68\ud22c \ud328\uc2a4\ub97c \uc544\ub974\uc5f0 \ub85c\ubc88\uc774 \ubc1b\uc544 \uc2a4\ud398\uc778 \uc218\ubb38\uc7a5 \uc774\ucf00\ub974 \uce74\uc2dc\uc57c\uc2a4 \uc640 1\ub3001 \uc0c1\ud669\uc744 \ucc3d\ucd9c\ud588\uc9c0\ub9cc, \uce74\uc2dc\uc57c\uc2a4\uac00 \ub2e4\ub9ac\ub97c \ucb49 \ubed7\uc5b4 \uc29b\uc744 \ubb34\ud6a8\ud654\uc2dc\ucf30\ub2e4.", "paragraph_sentence": " \ub124\ub35c\ub780\ub4dc\ub294 \uba87 \ucc28\ub840 \ub4dd\uc810 \uae30\ud68c\ub97c \uc7a1\uc558\ub294\ub370, \ud2b9\ud788 62\ubd84 \ubca0\uc2ac\ub808\uc774 \uc2a4\ub124\uc774\ub354\ub974\uc758 \uce68\ud22c \ud328\uc2a4\ub97c \uc544\ub974\uc5f0 \ub85c\ubc88\uc774 \ubc1b\uc544 \uc2a4\ud398\uc778 \uc218\ubb38\uc7a5 \uc774\ucf00\ub974 \uce74\uc2dc\uc57c\uc2a4 \uc640 1\ub3001 \uc0c1\ud669\uc744 \ucc3d\ucd9c\ud588\uc9c0\ub9cc, \uce74\uc2dc\uc57c\uc2a4\uac00 \ub2e4\ub9ac\ub97c \ucb49 \ubed7\uc5b4 \uc29b\uc744 \ubb34\ud6a8\ud654\uc2dc\ucf30\ub2e4. \uc2a4\ud398\uc778\uc758 \uacbd\uc6b0\uc5d0\ub294 \uacac\uc81c\ubc1b\uc9c0 \uc54a\ub358 \uc138\ub974\ud788\uc624 \ub77c\ubaa8\uc2a4\uac00 \ucf54\ub108\ud0a5\uc744 \uba38\ub9ac\ub85c \uc5f0\uacb0\ud558\ub824 \ud588\uc9c0\ub9cc \ube57\ub098\uac14\ub2e4. \ub124\ub35c\ub780\ub4dc \uc8fc\uc7a5 \ud788\uc624\ubc14\ub2c8 \ud310 \ube0c\ub860\ud06c\ud638\ub974\uc2a4\ud2b8\uac00 105\ubd84\uc5d0 \uc5e3\uc190 \ube0c\ub77c\ud504\ud5e4\uc774\ud2b8\ub85c \uad50\uccb4 \ud22c\uc785\ub418\uc5c8\ub2e4. \ub808\uc54c \ub9c8\ub4dc\ub9ac\ub4dc \ubbf8\ub4dc\ud544\ub354 \ub77c\ud30c\uc5d8 \ud310 \ub370\ub974 \ud30c\ub974\ud2b8\uac00 99\ubd84\uc5d0 \ub098\uc774\uc808 \ub354 \uc6a9\uacfc \uad50\uccb4\ub85c \ub4e4\uc5b4\uac00, \ub0a8\uc740 15\ubd84\ub3d9\uc548 \uc8fc\uc7a5 \uc644\uc7a5\uc744 \ucc3c\ub2e4. 109\ubd84\uc744 \uae30\uc810\uc73c\ub85c \ub124\ub35c\ub780\ub4dc\ub294 \ud5e4\uc774\ud305\uc544\uac00 \ub450\ubc88\uc9f8 \uc610\ub85c\uce74\ub4dc\ub97c \ubc1b\uc73c\uba74\uc11c 10\uba85\uc73c\ub85c \uc904\uc5b4\ub4e4\uc5c8\ub2e4. \uc2b9\ubd80\ucc28\uae30\uac00 \ub2e4\uac00\uc62c \ubb34\ub835, \ud5e4\uc218\uc2a4 \ub098\ubc14\uc2a4\uac00 \ub124\ub35c\ub780\ub4dc \ubb38\uc804\uc73c\ub85c \uc1c4\ub3c4\ud558\uc5ec \ud328\uc2a4\uc758 \uc2dc\ubc1c\uc810\uc774 \ub418\uc5c8\uace0, \uc774\ub2c8\uc5d0\uc2a4\ud0c0\uac00 \uc138\uc2a4\ud06c \ud30c\ube0c\ub808\uac00\uc2a4\uc758 \ud398\ub110\ud2f0 \uad6c\uc5ed \uc6b0\uce21\uc73c\ub85c\ubd80\ud130\uc758 \ud328\uc2a4\ub97c \ubc1b\uc544 \uc5f0\uc7a5\uc804 \uc885\ub8cc 4\ubd84\uc744 \ub0a8\uaca8\ub193\uace0 \uc624\ub978\ubc1c\ub85c \ubc1c\ub9ac\uc29b\uc744 \uc3d8\uc544 \uacf5\uc774 \uace8\ud0a4\ud37c \uc6b0\uce21\uc744 \ub118\uae30\uba74\uc11c \ubb34\ub4dd\uc810 \uce68\ubb35\uc744 \uae7c\ub2e4.", "paragraph_answer": "\ub124\ub35c\ub780\ub4dc\ub294 \uba87 \ucc28\ub840 \ub4dd\uc810 \uae30\ud68c\ub97c \uc7a1\uc558\ub294\ub370, \ud2b9\ud788 62\ubd84 \ubca0\uc2ac\ub808\uc774 \uc2a4\ub124\uc774\ub354\ub974\uc758 \uce68\ud22c \ud328\uc2a4\ub97c \uc544\ub974\uc5f0 \ub85c\ubc88\uc774 \ubc1b\uc544 \uc2a4\ud398\uc778 \uc218\ubb38\uc7a5 \uc774\ucf00\ub974 \uce74\uc2dc\uc57c\uc2a4 \uc640 1\ub3001 \uc0c1\ud669\uc744 \ucc3d\ucd9c\ud588\uc9c0\ub9cc, \uce74\uc2dc\uc57c\uc2a4\uac00 \ub2e4\ub9ac\ub97c \ucb49 \ubed7\uc5b4 \uc29b\uc744 \ubb34\ud6a8\ud654\uc2dc\ucf30\ub2e4. \uc2a4\ud398\uc778\uc758 \uacbd\uc6b0\uc5d0\ub294 \uacac\uc81c\ubc1b\uc9c0 \uc54a\ub358 \uc138\ub974\ud788\uc624 \ub77c\ubaa8\uc2a4\uac00 \ucf54\ub108\ud0a5\uc744 \uba38\ub9ac\ub85c \uc5f0\uacb0\ud558\ub824 \ud588\uc9c0\ub9cc \ube57\ub098\uac14\ub2e4. \ub124\ub35c\ub780\ub4dc \uc8fc\uc7a5 \ud788\uc624\ubc14\ub2c8 \ud310 \ube0c\ub860\ud06c\ud638\ub974\uc2a4\ud2b8\uac00 105\ubd84\uc5d0 \uc5e3\uc190 \ube0c\ub77c\ud504\ud5e4\uc774\ud2b8\ub85c \uad50\uccb4 \ud22c\uc785\ub418\uc5c8\ub2e4. \ub808\uc54c \ub9c8\ub4dc\ub9ac\ub4dc \ubbf8\ub4dc\ud544\ub354 \ub77c\ud30c\uc5d8 \ud310 \ub370\ub974 \ud30c\ub974\ud2b8\uac00 99\ubd84\uc5d0 \ub098\uc774\uc808 \ub354 \uc6a9\uacfc \uad50\uccb4\ub85c \ub4e4\uc5b4\uac00, \ub0a8\uc740 15\ubd84\ub3d9\uc548 \uc8fc\uc7a5 \uc644\uc7a5\uc744 \ucc3c\ub2e4. 109\ubd84\uc744 \uae30\uc810\uc73c\ub85c \ub124\ub35c\ub780\ub4dc\ub294 \ud5e4\uc774\ud305\uc544\uac00 \ub450\ubc88\uc9f8 \uc610\ub85c\uce74\ub4dc\ub97c \ubc1b\uc73c\uba74\uc11c 10\uba85\uc73c\ub85c \uc904\uc5b4\ub4e4\uc5c8\ub2e4. \uc2b9\ubd80\ucc28\uae30\uac00 \ub2e4\uac00\uc62c \ubb34\ub835, \ud5e4\uc218\uc2a4 \ub098\ubc14\uc2a4\uac00 \ub124\ub35c\ub780\ub4dc \ubb38\uc804\uc73c\ub85c \uc1c4\ub3c4\ud558\uc5ec \ud328\uc2a4\uc758 \uc2dc\ubc1c\uc810\uc774 \ub418\uc5c8\uace0, \uc774\ub2c8\uc5d0\uc2a4\ud0c0\uac00 \uc138\uc2a4\ud06c \ud30c\ube0c\ub808\uac00\uc2a4\uc758 \ud398\ub110\ud2f0 \uad6c\uc5ed \uc6b0\uce21\uc73c\ub85c\ubd80\ud130\uc758 \ud328\uc2a4\ub97c \ubc1b\uc544 \uc5f0\uc7a5\uc804 \uc885\ub8cc 4\ubd84\uc744 \ub0a8\uaca8\ub193\uace0 \uc624\ub978\ubc1c\ub85c \ubc1c\ub9ac\uc29b\uc744 \uc3d8\uc544 \uacf5\uc774 \uace8\ud0a4\ud37c \uc6b0\uce21\uc744 \ub118\uae30\uba74\uc11c \ubb34\ub4dd\uc810 \uce68\ubb35\uc744 \uae7c\ub2e4.", "sentence_answer": "\ub124\ub35c\ub780\ub4dc\ub294 \uba87 \ucc28\ub840 \ub4dd\uc810 \uae30\ud68c\ub97c \uc7a1\uc558\ub294\ub370, \ud2b9\ud788 62\ubd84 \ubca0\uc2ac\ub808\uc774 \uc2a4\ub124\uc774\ub354\ub974\uc758 \uce68\ud22c \ud328\uc2a4\ub97c \uc544\ub974\uc5f0 \ub85c\ubc88\uc774 \ubc1b\uc544 \uc2a4\ud398\uc778 \uc218\ubb38\uc7a5 \uc774\ucf00\ub974 \uce74\uc2dc\uc57c\uc2a4 \uc640 1\ub3001 \uc0c1\ud669\uc744 \ucc3d\ucd9c\ud588\uc9c0\ub9cc, \uce74\uc2dc\uc57c\uc2a4\uac00 \ub2e4\ub9ac\ub97c \ucb49 \ubed7\uc5b4 \uc29b\uc744 \ubb34\ud6a8\ud654\uc2dc\ucf30\ub2e4.", "paragraph_id": "6533357-3-0", "paragraph_question": "question: 2010\ub144 FIFA \uc6d4\ub4dc\ucef5 \uacb0\uc2b9\uc804\uc5d0 \ucd9c\uc804\ud55c \uc2a4\ud398\uc778\uc758 \uace8\ud0a4\ud37c\ub294 \ub204\uad6c\uc778\uac00?, context: \ub124\ub35c\ub780\ub4dc\ub294 \uba87 \ucc28\ub840 \ub4dd\uc810 \uae30\ud68c\ub97c \uc7a1\uc558\ub294\ub370, \ud2b9\ud788 62\ubd84 \ubca0\uc2ac\ub808\uc774 \uc2a4\ub124\uc774\ub354\ub974\uc758 \uce68\ud22c \ud328\uc2a4\ub97c \uc544\ub974\uc5f0 \ub85c\ubc88\uc774 \ubc1b\uc544 \uc2a4\ud398\uc778 \uc218\ubb38\uc7a5 \uc774\ucf00\ub974 \uce74\uc2dc\uc57c\uc2a4\uc640 1\ub3001 \uc0c1\ud669\uc744 \ucc3d\ucd9c\ud588\uc9c0\ub9cc, \uce74\uc2dc\uc57c\uc2a4\uac00 \ub2e4\ub9ac\ub97c \ucb49 \ubed7\uc5b4 \uc29b\uc744 \ubb34\ud6a8\ud654\uc2dc\ucf30\ub2e4. \uc2a4\ud398\uc778\uc758 \uacbd\uc6b0\uc5d0\ub294 \uacac\uc81c\ubc1b\uc9c0 \uc54a\ub358 \uc138\ub974\ud788\uc624 \ub77c\ubaa8\uc2a4\uac00 \ucf54\ub108\ud0a5\uc744 \uba38\ub9ac\ub85c \uc5f0\uacb0\ud558\ub824 \ud588\uc9c0\ub9cc \ube57\ub098\uac14\ub2e4. \ub124\ub35c\ub780\ub4dc \uc8fc\uc7a5 \ud788\uc624\ubc14\ub2c8 \ud310 \ube0c\ub860\ud06c\ud638\ub974\uc2a4\ud2b8\uac00 105\ubd84\uc5d0 \uc5e3\uc190 \ube0c\ub77c\ud504\ud5e4\uc774\ud2b8\ub85c \uad50\uccb4 \ud22c\uc785\ub418\uc5c8\ub2e4. \ub808\uc54c \ub9c8\ub4dc\ub9ac\ub4dc \ubbf8\ub4dc\ud544\ub354 \ub77c\ud30c\uc5d8 \ud310 \ub370\ub974 \ud30c\ub974\ud2b8\uac00 99\ubd84\uc5d0 \ub098\uc774\uc808 \ub354 \uc6a9\uacfc \uad50\uccb4\ub85c \ub4e4\uc5b4\uac00, \ub0a8\uc740 15\ubd84\ub3d9\uc548 \uc8fc\uc7a5 \uc644\uc7a5\uc744 \ucc3c\ub2e4. 109\ubd84\uc744 \uae30\uc810\uc73c\ub85c \ub124\ub35c\ub780\ub4dc\ub294 \ud5e4\uc774\ud305\uc544\uac00 \ub450\ubc88\uc9f8 \uc610\ub85c\uce74\ub4dc\ub97c \ubc1b\uc73c\uba74\uc11c 10\uba85\uc73c\ub85c \uc904\uc5b4\ub4e4\uc5c8\ub2e4. \uc2b9\ubd80\ucc28\uae30\uac00 \ub2e4\uac00\uc62c \ubb34\ub835, \ud5e4\uc218\uc2a4 \ub098\ubc14\uc2a4\uac00 \ub124\ub35c\ub780\ub4dc \ubb38\uc804\uc73c\ub85c \uc1c4\ub3c4\ud558\uc5ec \ud328\uc2a4\uc758 \uc2dc\ubc1c\uc810\uc774 \ub418\uc5c8\uace0, \uc774\ub2c8\uc5d0\uc2a4\ud0c0\uac00 \uc138\uc2a4\ud06c \ud30c\ube0c\ub808\uac00\uc2a4\uc758 \ud398\ub110\ud2f0 \uad6c\uc5ed \uc6b0\uce21\uc73c\ub85c\ubd80\ud130\uc758 \ud328\uc2a4\ub97c \ubc1b\uc544 \uc5f0\uc7a5\uc804 \uc885\ub8cc 4\ubd84\uc744 \ub0a8\uaca8\ub193\uace0 \uc624\ub978\ubc1c\ub85c \ubc1c\ub9ac\uc29b\uc744 \uc3d8\uc544 \uacf5\uc774 \uace8\ud0a4\ud37c \uc6b0\uce21\uc744 \ub118\uae30\uba74\uc11c \ubb34\ub4dd\uc810 \uce68\ubb35\uc744 \uae7c\ub2e4."} {"question": "2013\ub144 \uc18c\ub140\uc2dc\ub300\uac00 \ud55c\uad6d\uc5d0\uc11c \ubc1c\ub9e4\ud55c \ub124\ubc88\uc9f8 \uc815\uaddc\uc568\ubc94 \uc81c\ubaa9\uc740?", "paragraph": "2012\ub144 6\uc6d4 27\uc77c\uc5d0\ub294 \uc77c\ubcf8\uc5d0\uc11c \uc0c8 \uc2f1\uae00 \u3008PAPARAZZI\u3009\ub97c \ubc1c\ub9e4\ud588\ub2e4. \uc774 \uc2f1\uae00\uc740 10\ub9cc \uc7a5 \uc774\uc0c1\uc744 \ud310\ub9e4\ud558\uba70 \uace8\ub4dc \uc778\uc99d\uc744 \ubc1b\uc558\ub2e4. 2012\ub144 11\uc6d4 28\uc77c\uc5d0\ub294 \ub450 \ubc88\uc9f8 \uc77c\ubcf8 \uc815\uaddc \uc568\ubc94 \u300aGirls' Generation II: Girls & Peace\u300b\ub97c \ubc1c\ub9e4\ud588\uace0, \uccab \uc8fc 116,963\ub9cc \uc7a5\uc744 \ud314\uc544 \uc624\ub9ac\ucf58 \uc8fc\uac04 \uc568\ubc94 \ucc28\ud2b8 3\uc704\uc5d0 \uc62c\ub790\ub2e4. SM \uc5d4\ud130\ud14c\uc778\uba3c\ud2b8\ub294 2012\ub144 12\uc6d4 21\uc77c, \uc218\ub85d\uace1 \u3008Dancing Queen\u3009\uc758 \ubba4\uc9c1\ube44\ub514\uc624\uc640 \uc74c\uc6d0\uc744 \uc120\uacf5\uac1c\ud588\ub2e4. 2013\ub144 1\uc6d4 1\uc77c \ub124 \ubc88\uc9f8 \ud55c\uad6d \uc815\uaddc \uc568\ubc94 \u300aI Got a Boy\u300b\ub97c \ubc1c\ub9e4\ub97c \ud558\uc600\ub2e4. \ubc1c\ub9e4 \ud6c4 \uccab \ubc18\uc751\uc740 \ud638\ubd88\ud638\ub85c \uac08\ub838\ub2e4. \uadf8\ub7ec\ub098, \ud3c9\ub860\uac00\ub4e4\uc5d0\uac8c \ud601\uc2e0\uc801\uc774\ub77c\ub294 \ud638\ud3c9\uc744 \ubc1b\uc558\ub2e4. \ub4a4\uc774\uc5b4, \uc720\ud29c\ube0c \uacf5\uc2dd \ucc44\ub110\uc5d0 \ub4f1\ub85d\ub41c \ubba4\ube44 \u3008I Got a Boy\u3009\uac00 \uc5ed\ub300 K-POP \uc0ac\uc0c1 \ucd5c\ub2e8 2,000\ub9cc \ubdf0\ub97c \uae30\ub85d\ud588\ub2e4. \uc774\uc804 \ucd5c\ub2e8 \uae30\ub85d\uc778 \ud604\uc544\uc758 \u3008Ice cream\u3009 \ubba4\uc9c1\ube44\ub514\uc624\uc758 10\uc77c \uae30\ub85d\uc758 50%\ub97c \ub2e8\ucd95\ud55c 5\uc77c\ub9cc\uc5d0 \uc774\ub8e9\ud55c \uacb0\uacfc\uc774\ub2e4. 2013\ub144 6\uc6d4 8\uc77c \uc11c\uc6b8 \uc62c\ub9bc\ud53d\uacf5\uc6d0 \uccb4\uc870\uacbd\uae30\uc7a5\uc5d0\uc11c \uccab \uc6d4\ub4dc \ud22c\uc5b4\uc778 Girls' Generation World Tour - Girls & Peace\ub97c \uc2dc\uc791\ud588\ub2e4. \uac19\uc740 \ub2ec 19\uc77c\uc5d0\ub294 \uc77c\ubcf8\uc5d0\uc11c \uc2f1\uae00 \u3008LOVE&GIRLS\u3009\ub97c \ubc1c\ub9e4\ud588\ub2e4. \ubc1c\ub9e4 \ub2f9\uc77c\uc5d0 \uc720\ub2c8\ubc84\uc124 \uc2a4\ud29c\ub514\uc624 \uc7ac\ud32c\uc5d0\uc11c \uc77c\ubcf8 \ud32c 4\ucc9c\uc5ec\uba85\uacfc \ud568\uaed8 \ud50c\ub798\uc2dc\ubab9\uc744 \uc9c4\ud589\ud574 \ud654\uc81c\uac00 \ub418\uae30\ub3c4 \ud588\ub2e4.", "answer": "I Got a Boy", "sentence": "2013\ub144 1\uc6d4 1\uc77c \ub124 \ubc88\uc9f8 \ud55c\uad6d \uc815\uaddc \uc568\ubc94 \u300a I Got a Boy \u300b\ub97c \ubc1c\ub9e4\ub97c \ud558\uc600\ub2e4.", "paragraph_sentence": "2012\ub144 6\uc6d4 27\uc77c\uc5d0\ub294 \uc77c\ubcf8\uc5d0\uc11c \uc0c8 \uc2f1\uae00 \u3008PAPARAZZI\u3009\ub97c \ubc1c\ub9e4\ud588\ub2e4. \uc774 \uc2f1\uae00\uc740 10\ub9cc \uc7a5 \uc774\uc0c1\uc744 \ud310\ub9e4\ud558\uba70 \uace8\ub4dc \uc778\uc99d\uc744 \ubc1b\uc558\ub2e4. 2012\ub144 11\uc6d4 28\uc77c\uc5d0\ub294 \ub450 \ubc88\uc9f8 \uc77c\ubcf8 \uc815\uaddc \uc568\ubc94 \u300aGirls' Generation II: Girls & Peace\u300b\ub97c \ubc1c\ub9e4\ud588\uace0, \uccab \uc8fc 116,963\ub9cc \uc7a5\uc744 \ud314\uc544 \uc624\ub9ac\ucf58 \uc8fc\uac04 \uc568\ubc94 \ucc28\ud2b8 3\uc704\uc5d0 \uc62c\ub790\ub2e4. SM \uc5d4\ud130\ud14c\uc778\uba3c\ud2b8\ub294 2012\ub144 12\uc6d4 21\uc77c, \uc218\ub85d\uace1 \u3008Dancing Queen\u3009\uc758 \ubba4\uc9c1\ube44\ub514\uc624\uc640 \uc74c\uc6d0\uc744 \uc120\uacf5\uac1c\ud588\ub2e4. 2013\ub144 1\uc6d4 1\uc77c \ub124 \ubc88\uc9f8 \ud55c\uad6d \uc815\uaddc \uc568\ubc94 \u300a I Got a Boy \u300b\ub97c \ubc1c\ub9e4\ub97c \ud558\uc600\ub2e4. \ubc1c\ub9e4 \ud6c4 \uccab \ubc18\uc751\uc740 \ud638\ubd88\ud638\ub85c \uac08\ub838\ub2e4. \uadf8\ub7ec\ub098, \ud3c9\ub860\uac00\ub4e4\uc5d0\uac8c \ud601\uc2e0\uc801\uc774\ub77c\ub294 \ud638\ud3c9\uc744 \ubc1b\uc558\ub2e4. \ub4a4\uc774\uc5b4, \uc720\ud29c\ube0c \uacf5\uc2dd \ucc44\ub110\uc5d0 \ub4f1\ub85d\ub41c \ubba4\ube44 \u3008I Got a Boy\u3009\uac00 \uc5ed\ub300 K-POP \uc0ac\uc0c1 \ucd5c\ub2e8 2,000\ub9cc \ubdf0\ub97c \uae30\ub85d\ud588\ub2e4. \uc774\uc804 \ucd5c\ub2e8 \uae30\ub85d\uc778 \ud604\uc544\uc758 \u3008Ice cream\u3009 \ubba4\uc9c1\ube44\ub514\uc624\uc758 10\uc77c \uae30\ub85d\uc758 50%\ub97c \ub2e8\ucd95\ud55c 5\uc77c\ub9cc\uc5d0 \uc774\ub8e9\ud55c \uacb0\uacfc\uc774\ub2e4. 2013\ub144 6\uc6d4 8\uc77c \uc11c\uc6b8 \uc62c\ub9bc\ud53d\uacf5\uc6d0 \uccb4\uc870\uacbd\uae30\uc7a5\uc5d0\uc11c \uccab \uc6d4\ub4dc \ud22c\uc5b4\uc778 Girls' Generation World Tour - Girls & Peace\ub97c \uc2dc\uc791\ud588\ub2e4. \uac19\uc740 \ub2ec 19\uc77c\uc5d0\ub294 \uc77c\ubcf8\uc5d0\uc11c \uc2f1\uae00 \u3008LOVE&GIRLS\u3009\ub97c \ubc1c\ub9e4\ud588\ub2e4. \ubc1c\ub9e4 \ub2f9\uc77c\uc5d0 \uc720\ub2c8\ubc84\uc124 \uc2a4\ud29c\ub514\uc624 \uc7ac\ud32c\uc5d0\uc11c \uc77c\ubcf8 \ud32c 4\ucc9c\uc5ec\uba85\uacfc \ud568\uaed8 \ud50c\ub798\uc2dc\ubab9\uc744 \uc9c4\ud589\ud574 \ud654\uc81c\uac00 \ub418\uae30\ub3c4 \ud588\ub2e4.", "paragraph_answer": "2012\ub144 6\uc6d4 27\uc77c\uc5d0\ub294 \uc77c\ubcf8\uc5d0\uc11c \uc0c8 \uc2f1\uae00 \u3008PAPARAZZI\u3009\ub97c \ubc1c\ub9e4\ud588\ub2e4. \uc774 \uc2f1\uae00\uc740 10\ub9cc \uc7a5 \uc774\uc0c1\uc744 \ud310\ub9e4\ud558\uba70 \uace8\ub4dc \uc778\uc99d\uc744 \ubc1b\uc558\ub2e4. 2012\ub144 11\uc6d4 28\uc77c\uc5d0\ub294 \ub450 \ubc88\uc9f8 \uc77c\ubcf8 \uc815\uaddc \uc568\ubc94 \u300aGirls' Generation II: Girls & Peace\u300b\ub97c \ubc1c\ub9e4\ud588\uace0, \uccab \uc8fc 116,963\ub9cc \uc7a5\uc744 \ud314\uc544 \uc624\ub9ac\ucf58 \uc8fc\uac04 \uc568\ubc94 \ucc28\ud2b8 3\uc704\uc5d0 \uc62c\ub790\ub2e4. SM \uc5d4\ud130\ud14c\uc778\uba3c\ud2b8\ub294 2012\ub144 12\uc6d4 21\uc77c, \uc218\ub85d\uace1 \u3008Dancing Queen\u3009\uc758 \ubba4\uc9c1\ube44\ub514\uc624\uc640 \uc74c\uc6d0\uc744 \uc120\uacf5\uac1c\ud588\ub2e4. 2013\ub144 1\uc6d4 1\uc77c \ub124 \ubc88\uc9f8 \ud55c\uad6d \uc815\uaddc \uc568\ubc94 \u300a I Got a Boy \u300b\ub97c \ubc1c\ub9e4\ub97c \ud558\uc600\ub2e4. \ubc1c\ub9e4 \ud6c4 \uccab \ubc18\uc751\uc740 \ud638\ubd88\ud638\ub85c \uac08\ub838\ub2e4. \uadf8\ub7ec\ub098, \ud3c9\ub860\uac00\ub4e4\uc5d0\uac8c \ud601\uc2e0\uc801\uc774\ub77c\ub294 \ud638\ud3c9\uc744 \ubc1b\uc558\ub2e4. \ub4a4\uc774\uc5b4, \uc720\ud29c\ube0c \uacf5\uc2dd \ucc44\ub110\uc5d0 \ub4f1\ub85d\ub41c \ubba4\ube44 \u3008I Got a Boy\u3009\uac00 \uc5ed\ub300 K-POP \uc0ac\uc0c1 \ucd5c\ub2e8 2,000\ub9cc \ubdf0\ub97c \uae30\ub85d\ud588\ub2e4. \uc774\uc804 \ucd5c\ub2e8 \uae30\ub85d\uc778 \ud604\uc544\uc758 \u3008Ice cream\u3009 \ubba4\uc9c1\ube44\ub514\uc624\uc758 10\uc77c \uae30\ub85d\uc758 50%\ub97c \ub2e8\ucd95\ud55c 5\uc77c\ub9cc\uc5d0 \uc774\ub8e9\ud55c \uacb0\uacfc\uc774\ub2e4. 2013\ub144 6\uc6d4 8\uc77c \uc11c\uc6b8 \uc62c\ub9bc\ud53d\uacf5\uc6d0 \uccb4\uc870\uacbd\uae30\uc7a5\uc5d0\uc11c \uccab \uc6d4\ub4dc \ud22c\uc5b4\uc778 Girls' Generation World Tour - Girls & Peace\ub97c \uc2dc\uc791\ud588\ub2e4. \uac19\uc740 \ub2ec 19\uc77c\uc5d0\ub294 \uc77c\ubcf8\uc5d0\uc11c \uc2f1\uae00 \u3008LOVE&GIRLS\u3009\ub97c \ubc1c\ub9e4\ud588\ub2e4. \ubc1c\ub9e4 \ub2f9\uc77c\uc5d0 \uc720\ub2c8\ubc84\uc124 \uc2a4\ud29c\ub514\uc624 \uc7ac\ud32c\uc5d0\uc11c \uc77c\ubcf8 \ud32c 4\ucc9c\uc5ec\uba85\uacfc \ud568\uaed8 \ud50c\ub798\uc2dc\ubab9\uc744 \uc9c4\ud589\ud574 \ud654\uc81c\uac00 \ub418\uae30\ub3c4 \ud588\ub2e4.", "sentence_answer": "2013\ub144 1\uc6d4 1\uc77c \ub124 \ubc88\uc9f8 \ud55c\uad6d \uc815\uaddc \uc568\ubc94 \u300a I Got a Boy \u300b\ub97c \ubc1c\ub9e4\ub97c \ud558\uc600\ub2e4.", "paragraph_id": "6532311-11-1", "paragraph_question": "question: 2013\ub144 \uc18c\ub140\uc2dc\ub300\uac00 \ud55c\uad6d\uc5d0\uc11c \ubc1c\ub9e4\ud55c \ub124\ubc88\uc9f8 \uc815\uaddc\uc568\ubc94 \uc81c\ubaa9\uc740?, context: 2012\ub144 6\uc6d4 27\uc77c\uc5d0\ub294 \uc77c\ubcf8\uc5d0\uc11c \uc0c8 \uc2f1\uae00 \u3008PAPARAZZI\u3009\ub97c \ubc1c\ub9e4\ud588\ub2e4. \uc774 \uc2f1\uae00\uc740 10\ub9cc \uc7a5 \uc774\uc0c1\uc744 \ud310\ub9e4\ud558\uba70 \uace8\ub4dc \uc778\uc99d\uc744 \ubc1b\uc558\ub2e4. 2012\ub144 11\uc6d4 28\uc77c\uc5d0\ub294 \ub450 \ubc88\uc9f8 \uc77c\ubcf8 \uc815\uaddc \uc568\ubc94 \u300aGirls' Generation II: Girls & Peace\u300b\ub97c \ubc1c\ub9e4\ud588\uace0, \uccab \uc8fc 116,963\ub9cc \uc7a5\uc744 \ud314\uc544 \uc624\ub9ac\ucf58 \uc8fc\uac04 \uc568\ubc94 \ucc28\ud2b8 3\uc704\uc5d0 \uc62c\ub790\ub2e4. SM \uc5d4\ud130\ud14c\uc778\uba3c\ud2b8\ub294 2012\ub144 12\uc6d4 21\uc77c, \uc218\ub85d\uace1 \u3008Dancing Queen\u3009\uc758 \ubba4\uc9c1\ube44\ub514\uc624\uc640 \uc74c\uc6d0\uc744 \uc120\uacf5\uac1c\ud588\ub2e4. 2013\ub144 1\uc6d4 1\uc77c \ub124 \ubc88\uc9f8 \ud55c\uad6d \uc815\uaddc \uc568\ubc94 \u300aI Got a Boy\u300b\ub97c \ubc1c\ub9e4\ub97c \ud558\uc600\ub2e4. \ubc1c\ub9e4 \ud6c4 \uccab \ubc18\uc751\uc740 \ud638\ubd88\ud638\ub85c \uac08\ub838\ub2e4. \uadf8\ub7ec\ub098, \ud3c9\ub860\uac00\ub4e4\uc5d0\uac8c \ud601\uc2e0\uc801\uc774\ub77c\ub294 \ud638\ud3c9\uc744 \ubc1b\uc558\ub2e4. \ub4a4\uc774\uc5b4, \uc720\ud29c\ube0c \uacf5\uc2dd \ucc44\ub110\uc5d0 \ub4f1\ub85d\ub41c \ubba4\ube44 \u3008I Got a Boy\u3009\uac00 \uc5ed\ub300 K-POP \uc0ac\uc0c1 \ucd5c\ub2e8 2,000\ub9cc \ubdf0\ub97c \uae30\ub85d\ud588\ub2e4. \uc774\uc804 \ucd5c\ub2e8 \uae30\ub85d\uc778 \ud604\uc544\uc758 \u3008Ice cream\u3009 \ubba4\uc9c1\ube44\ub514\uc624\uc758 10\uc77c \uae30\ub85d\uc758 50%\ub97c \ub2e8\ucd95\ud55c 5\uc77c\ub9cc\uc5d0 \uc774\ub8e9\ud55c \uacb0\uacfc\uc774\ub2e4. 2013\ub144 6\uc6d4 8\uc77c \uc11c\uc6b8 \uc62c\ub9bc\ud53d\uacf5\uc6d0 \uccb4\uc870\uacbd\uae30\uc7a5\uc5d0\uc11c \uccab \uc6d4\ub4dc \ud22c\uc5b4\uc778 Girls' Generation World Tour - Girls & Peace\ub97c \uc2dc\uc791\ud588\ub2e4. \uac19\uc740 \ub2ec 19\uc77c\uc5d0\ub294 \uc77c\ubcf8\uc5d0\uc11c \uc2f1\uae00 \u3008LOVE&GIRLS\u3009\ub97c \ubc1c\ub9e4\ud588\ub2e4. \ubc1c\ub9e4 \ub2f9\uc77c\uc5d0 \uc720\ub2c8\ubc84\uc124 \uc2a4\ud29c\ub514\uc624 \uc7ac\ud32c\uc5d0\uc11c \uc77c\ubcf8 \ud32c 4\ucc9c\uc5ec\uba85\uacfc \ud568\uaed8 \ud50c\ub798\uc2dc\ubab9\uc744 \uc9c4\ud589\ud574 \ud654\uc81c\uac00 \ub418\uae30\ub3c4 \ud588\ub2e4."} {"question": "\uc5d0\ub7f0\ub7a8\uc9c0\ub294 \uc120\ub35c\ub79c\ub4dc AFC\uc640\uc758 \uacbd\uae30\uc5d0\uc11c \ucd1d \uba87 \ud68c\uc758 \ud130\uce58\ub97c \uae30\ub85d\ud558\uc600\ub098?", "paragraph": "\ud504\ub9ac\uc2dc\uc98c \uc88b\uc740 \uacbd\uae30\ub825\uc744 \ud3bc\uce58\uba74\uc11c \uc544\uc2a4\ub0a0 \ud32c\ub4e4\uc5d0\uac8c \uae30\ub300\uac10\uc744 \uc548\uacbc\ub2e4. \ube44\ub85d \uc2dc\uc98c \ucd08\ubc18\uc774\uae34 \ud558\uc9c0\ub9cc \ub9ac\uadf8 4\uacbd\uae30 2\uace8, \ucc54\ud53c\uc5b8\uc2a4\ub9ac\uadf8 3\uacbd\uae30 3\uace8\uc744 \uae30\ub85d\ud558\uba70 \uc62c\ub9ac\ube44\uc5d0 \uc9c0\ub8e8\uc640 \ud568\uaed8 \ud300 \ub0b4 \ub4dd\uc810 1\uc704\ub97c \ub2ec\ub9ac\uace0 \uc788\ub2e4. \ub9c8\ud2f0\uc678 \ud50c\ub77c\ubbf8\ub2c8\uac00 \uc544\uc2a4\ub0a0\uc5d0 \uc601\uc785\ub418\uba74\uc11c \uc5c4\uccad\ub09c \ud65c\ub3d9\ub7c9\uc744 \uac16\ucd98 \ub450 \ubbf8\ub4dc\ud544\ub354\uac00 \uc544\uc2a4\ub0a0\uc758 \uc0c1\uc2b9\uc138\ub97c \uc774\ub04c\uace0 \uc788\ub2e4. 11-12 \uc2dc\uc98c\uc5d0 \uc9c0\uc801\ub418\uc5c8\ub358 \ub2e8\uc810\ub4e4\uc744 \uac1c\uc120\ud588\ub294\ub370 \uc774\ub294 \ubd80\uc815\ud655\ud55c \ud328\uc2a4\uc640 \uacbd\uae30 \ud15c\ud3ec \uc870\uc808 \uc2e4\ud328, \uadf8\ub9ac\uace0 \ub4dc\ub9ac\ube14\uc5d0 \uc790\uc2e0\uac10\uc774 \uc5c6\ub358 \ubd80\ubd84\uc774\uc5c8\ub2e4. 12-13 \uc2dc\uc98c \ubbf8\ucf08 \uc544\ub974\ud14c\ud0c0\uac00 \uc218\ud589\ud558\ub358 \ub525-\ub77c\uc789 \ud50c\ub808\uc774\uba54\uc774\ucee4 \uc5ed\ud560\uc744 \ub9e1\uc73c\uba74\uc11c 87.3%\uc758 \uc815\ud655\ub3c4\uc600\ub358 \ud328\uc2a4 \uc815\ud655\ub3c4\ub97c 88.2%\uae4c\uc9c0 \ub04c\uc5b4\uc62c\ub838\uc73c\uba70, \ubbf8\ucf08 \uc544\ub974\ud14c\ud0c0\uac00 \ubd80\uc7ac\ud560 \ub54c\uc5d4 \uadf8\uac00 \ud3c9\uade0\uc801\uc73c\ub85c \uae30\ub85d\ud558\ub358 91% \uc815\ub3c4\uc758 \ud328\uc2a4 \uc131\uacf5\ub960\uc744 \uae30\ub85d\ud558\uc600\ub2e4. \ubd80\uc0c1 \uc774\ud6c4 \uacbd\uae30 \ud15c\ud3ec \uc870\uc808\uacfc \ubbf8\uc219\ud55c \ub4dc\ub9ac\ube14 \uc2e4\ub825\uc740 \uc544\ub974\uc13c \ubcb5\uac70 \uac10\ub3c5\uc774 \uc88c\uc6b0 \uc719\uc5b4\ub85c \uae30\uc6a9\ud558\uba74\uc11c \uc544\uc2a4\ub0a0\uc758 \ud15c\ud3ec\uc5d0 \uc801\uc751 \uc2dc\ud0a4\ub294 \ub3d9\uc2dc\uc5d0 \ub4dc\ub9ac\ube14\uc5d0 \uc790\uc2e0\uac10\uc744 \uac00\uc838\uc624\uac8c \ud558\uc600\ub2e4. \ud1a0\ub9c8\uc2dc \ub85c\uc2dc\uce20\ud0a4\ub54c\uc640 \ub9c8\ucc2c\uac00\uc9c0\ub85c \uc719\uc5b4\ub85c \ubcc0\uc2e0\ud558\ub294\ub370\ub294 \uc2e4\ud328\ud558\uc600\uc9c0\ub9cc, \ubbf8\ub4dc\ud544\ub354\uc5d0\uc11c\uc758 \uc790\uc2e0\uac10\uc744 \ud5a5\uc0c1\uc2dc\ud0a8 \ub3d9\uc2dc\uc5d0, \ud0c8\uc555\ubc15 \ub2a5\ub825\uc744 \uae30\ub974\ub294\ub370\ub294 \ud070 \ub3c4\uc6c0\uc774 \ub418\uc5c8\ub2e4. \ub9ac\uadf8\uc5d0\uc11c\uc758 \uc120\ub35c\ub79c\ub4dc AFC\uc640\uc758 \uacbd\uae30\uc5d0\uc11c\ub294 111\ud68c\uc758 \ud130\uce58\ub97c \uae30\ub85d\ud558\uc600\uace0, \uacbd\uae30 \ub0b4 \uc591\ud300 \ud1b5\ud2c0\uc5b4 \ucd5c\ub2e4 \ud328\uc2a4\uc778 88\uac1c\uc758 \ud328\uc2a4\ub97c \ud558\uc600\uc73c\uba70 \uadf8 \uc911 81\uac1c\ub97c \uc131\uacf5\uc2dc\ud0a4\uba74\uc11c 92%\uc758 \uc131\uacf5\ub960\uc744 \ubcf4\uc600\ub2e4. \ub610\ud55c 2014\ub144 5\uc6d4 18\uc77c \uc601\uad6d \uc6f8\ube14\ub9ac \uc2a4\ud0c0\ub514\uc6c0\uc5d0\uc11c \uc5f4\ub9b0 \uc6e8\uc2a4\ud2b8\ud584 \uc720\ub098\uc774\ud2f0\ub4dc\uc640\uc758 2013-14 \uc789\uae00\ub79c\ub4dc FA\ucef5 \uacb0\uc2b9\uc804\uc5d0\uc11c \uc5f0\uc7a5 \ud6c4\ubc18 4\ubd84 \uc2a4\ucf54\uc5b4 2:2\uc0c1\ud669\uc5d0\uc11c \uc5ed\uc804 \uacb0\uc2b9\uace8\uc744 \ubf51\uc544\ub0b4\uba70 \ud300\uc758 9\ub144 \ubb34\uad00\uc744 \ub04a\ub294\ub370 \ud070 \uacf5\ud5cc\uc744 \ud558\uc600\ub2e4.", "answer": "111\ud68c", "sentence": "\ub9ac\uadf8\uc5d0\uc11c\uc758 \uc120\ub35c\ub79c\ub4dc AFC\uc640\uc758 \uacbd\uae30\uc5d0\uc11c\ub294 111\ud68c \uc758 \ud130\uce58\ub97c \uae30\ub85d\ud558\uc600\uace0, \uacbd\uae30 \ub0b4 \uc591\ud300 \ud1b5\ud2c0\uc5b4 \ucd5c\ub2e4 \ud328\uc2a4\uc778 88\uac1c\uc758 \ud328\uc2a4\ub97c \ud558\uc600\uc73c\uba70 \uadf8 \uc911 81\uac1c\ub97c \uc131\uacf5\uc2dc\ud0a4\uba74\uc11c 92%\uc758 \uc131\uacf5\ub960\uc744 \ubcf4\uc600\ub2e4.", "paragraph_sentence": "\ud504\ub9ac\uc2dc\uc98c \uc88b\uc740 \uacbd\uae30\ub825\uc744 \ud3bc\uce58\uba74\uc11c \uc544\uc2a4\ub0a0 \ud32c\ub4e4\uc5d0\uac8c \uae30\ub300\uac10\uc744 \uc548\uacbc\ub2e4. \ube44\ub85d \uc2dc\uc98c \ucd08\ubc18\uc774\uae34 \ud558\uc9c0\ub9cc \ub9ac\uadf8 4\uacbd\uae30 2\uace8, \ucc54\ud53c\uc5b8\uc2a4\ub9ac\uadf8 3\uacbd\uae30 3\uace8\uc744 \uae30\ub85d\ud558\uba70 \uc62c\ub9ac\ube44\uc5d0 \uc9c0\ub8e8\uc640 \ud568\uaed8 \ud300 \ub0b4 \ub4dd\uc810 1\uc704\ub97c \ub2ec\ub9ac\uace0 \uc788\ub2e4. \ub9c8\ud2f0\uc678 \ud50c\ub77c\ubbf8\ub2c8\uac00 \uc544\uc2a4\ub0a0\uc5d0 \uc601\uc785\ub418\uba74\uc11c \uc5c4\uccad\ub09c \ud65c\ub3d9\ub7c9\uc744 \uac16\ucd98 \ub450 \ubbf8\ub4dc\ud544\ub354\uac00 \uc544\uc2a4\ub0a0\uc758 \uc0c1\uc2b9\uc138\ub97c \uc774\ub04c\uace0 \uc788\ub2e4. 11-12 \uc2dc\uc98c\uc5d0 \uc9c0\uc801\ub418\uc5c8\ub358 \ub2e8\uc810\ub4e4\uc744 \uac1c\uc120\ud588\ub294\ub370 \uc774\ub294 \ubd80\uc815\ud655\ud55c \ud328\uc2a4\uc640 \uacbd\uae30 \ud15c\ud3ec \uc870\uc808 \uc2e4\ud328, \uadf8\ub9ac\uace0 \ub4dc\ub9ac\ube14\uc5d0 \uc790\uc2e0\uac10\uc774 \uc5c6\ub358 \ubd80\ubd84\uc774\uc5c8\ub2e4. 12-13 \uc2dc\uc98c \ubbf8\ucf08 \uc544\ub974\ud14c\ud0c0\uac00 \uc218\ud589\ud558\ub358 \ub525-\ub77c\uc789 \ud50c\ub808\uc774\uba54\uc774\ucee4 \uc5ed\ud560\uc744 \ub9e1\uc73c\uba74\uc11c 87.3%\uc758 \uc815\ud655\ub3c4\uc600\ub358 \ud328\uc2a4 \uc815\ud655\ub3c4\ub97c 88.2%\uae4c\uc9c0 \ub04c\uc5b4\uc62c\ub838\uc73c\uba70, \ubbf8\ucf08 \uc544\ub974\ud14c\ud0c0\uac00 \ubd80\uc7ac\ud560 \ub54c\uc5d4 \uadf8\uac00 \ud3c9\uade0\uc801\uc73c\ub85c \uae30\ub85d\ud558\ub358 91% \uc815\ub3c4\uc758 \ud328\uc2a4 \uc131\uacf5\ub960\uc744 \uae30\ub85d\ud558\uc600\ub2e4. \ubd80\uc0c1 \uc774\ud6c4 \uacbd\uae30 \ud15c\ud3ec \uc870\uc808\uacfc \ubbf8\uc219\ud55c \ub4dc\ub9ac\ube14 \uc2e4\ub825\uc740 \uc544\ub974\uc13c \ubcb5\uac70 \uac10\ub3c5\uc774 \uc88c\uc6b0 \uc719\uc5b4\ub85c \uae30\uc6a9\ud558\uba74\uc11c \uc544\uc2a4\ub0a0\uc758 \ud15c\ud3ec\uc5d0 \uc801\uc751 \uc2dc\ud0a4\ub294 \ub3d9\uc2dc\uc5d0 \ub4dc\ub9ac\ube14\uc5d0 \uc790\uc2e0\uac10\uc744 \uac00\uc838\uc624\uac8c \ud558\uc600\ub2e4. \ud1a0\ub9c8\uc2dc \ub85c\uc2dc\uce20\ud0a4\ub54c\uc640 \ub9c8\ucc2c\uac00\uc9c0\ub85c \uc719\uc5b4\ub85c \ubcc0\uc2e0\ud558\ub294\ub370\ub294 \uc2e4\ud328\ud558\uc600\uc9c0\ub9cc, \ubbf8\ub4dc\ud544\ub354\uc5d0\uc11c\uc758 \uc790\uc2e0\uac10\uc744 \ud5a5\uc0c1\uc2dc\ud0a8 \ub3d9\uc2dc\uc5d0, \ud0c8\uc555\ubc15 \ub2a5\ub825\uc744 \uae30\ub974\ub294\ub370\ub294 \ud070 \ub3c4\uc6c0\uc774 \ub418\uc5c8\ub2e4. \ub9ac\uadf8\uc5d0\uc11c\uc758 \uc120\ub35c\ub79c\ub4dc AFC\uc640\uc758 \uacbd\uae30\uc5d0\uc11c\ub294 111\ud68c \uc758 \ud130\uce58\ub97c \uae30\ub85d\ud558\uc600\uace0, \uacbd\uae30 \ub0b4 \uc591\ud300 \ud1b5\ud2c0\uc5b4 \ucd5c\ub2e4 \ud328\uc2a4\uc778 88\uac1c\uc758 \ud328\uc2a4\ub97c \ud558\uc600\uc73c\uba70 \uadf8 \uc911 81\uac1c\ub97c \uc131\uacf5\uc2dc\ud0a4\uba74\uc11c 92%\uc758 \uc131\uacf5\ub960\uc744 \ubcf4\uc600\ub2e4. \ub610\ud55c 2014\ub144 5\uc6d4 18\uc77c \uc601\uad6d \uc6f8\ube14\ub9ac \uc2a4\ud0c0\ub514\uc6c0\uc5d0\uc11c \uc5f4\ub9b0 \uc6e8\uc2a4\ud2b8\ud584 \uc720\ub098\uc774\ud2f0\ub4dc\uc640\uc758 2013-14 \uc789\uae00\ub79c\ub4dc FA\ucef5 \uacb0\uc2b9\uc804\uc5d0\uc11c \uc5f0\uc7a5 \ud6c4\ubc18 4\ubd84 \uc2a4\ucf54\uc5b4 2:2\uc0c1\ud669\uc5d0\uc11c \uc5ed\uc804 \uacb0\uc2b9\uace8\uc744 \ubf51\uc544\ub0b4\uba70 \ud300\uc758 9\ub144 \ubb34\uad00\uc744 \ub04a\ub294\ub370 \ud070 \uacf5\ud5cc\uc744 \ud558\uc600\ub2e4.", "paragraph_answer": "\ud504\ub9ac\uc2dc\uc98c \uc88b\uc740 \uacbd\uae30\ub825\uc744 \ud3bc\uce58\uba74\uc11c \uc544\uc2a4\ub0a0 \ud32c\ub4e4\uc5d0\uac8c \uae30\ub300\uac10\uc744 \uc548\uacbc\ub2e4. \ube44\ub85d \uc2dc\uc98c \ucd08\ubc18\uc774\uae34 \ud558\uc9c0\ub9cc \ub9ac\uadf8 4\uacbd\uae30 2\uace8, \ucc54\ud53c\uc5b8\uc2a4\ub9ac\uadf8 3\uacbd\uae30 3\uace8\uc744 \uae30\ub85d\ud558\uba70 \uc62c\ub9ac\ube44\uc5d0 \uc9c0\ub8e8\uc640 \ud568\uaed8 \ud300 \ub0b4 \ub4dd\uc810 1\uc704\ub97c \ub2ec\ub9ac\uace0 \uc788\ub2e4. \ub9c8\ud2f0\uc678 \ud50c\ub77c\ubbf8\ub2c8\uac00 \uc544\uc2a4\ub0a0\uc5d0 \uc601\uc785\ub418\uba74\uc11c \uc5c4\uccad\ub09c \ud65c\ub3d9\ub7c9\uc744 \uac16\ucd98 \ub450 \ubbf8\ub4dc\ud544\ub354\uac00 \uc544\uc2a4\ub0a0\uc758 \uc0c1\uc2b9\uc138\ub97c \uc774\ub04c\uace0 \uc788\ub2e4. 11-12 \uc2dc\uc98c\uc5d0 \uc9c0\uc801\ub418\uc5c8\ub358 \ub2e8\uc810\ub4e4\uc744 \uac1c\uc120\ud588\ub294\ub370 \uc774\ub294 \ubd80\uc815\ud655\ud55c \ud328\uc2a4\uc640 \uacbd\uae30 \ud15c\ud3ec \uc870\uc808 \uc2e4\ud328, \uadf8\ub9ac\uace0 \ub4dc\ub9ac\ube14\uc5d0 \uc790\uc2e0\uac10\uc774 \uc5c6\ub358 \ubd80\ubd84\uc774\uc5c8\ub2e4. 12-13 \uc2dc\uc98c \ubbf8\ucf08 \uc544\ub974\ud14c\ud0c0\uac00 \uc218\ud589\ud558\ub358 \ub525-\ub77c\uc789 \ud50c\ub808\uc774\uba54\uc774\ucee4 \uc5ed\ud560\uc744 \ub9e1\uc73c\uba74\uc11c 87.3%\uc758 \uc815\ud655\ub3c4\uc600\ub358 \ud328\uc2a4 \uc815\ud655\ub3c4\ub97c 88.2%\uae4c\uc9c0 \ub04c\uc5b4\uc62c\ub838\uc73c\uba70, \ubbf8\ucf08 \uc544\ub974\ud14c\ud0c0\uac00 \ubd80\uc7ac\ud560 \ub54c\uc5d4 \uadf8\uac00 \ud3c9\uade0\uc801\uc73c\ub85c \uae30\ub85d\ud558\ub358 91% \uc815\ub3c4\uc758 \ud328\uc2a4 \uc131\uacf5\ub960\uc744 \uae30\ub85d\ud558\uc600\ub2e4. \ubd80\uc0c1 \uc774\ud6c4 \uacbd\uae30 \ud15c\ud3ec \uc870\uc808\uacfc \ubbf8\uc219\ud55c \ub4dc\ub9ac\ube14 \uc2e4\ub825\uc740 \uc544\ub974\uc13c \ubcb5\uac70 \uac10\ub3c5\uc774 \uc88c\uc6b0 \uc719\uc5b4\ub85c \uae30\uc6a9\ud558\uba74\uc11c \uc544\uc2a4\ub0a0\uc758 \ud15c\ud3ec\uc5d0 \uc801\uc751 \uc2dc\ud0a4\ub294 \ub3d9\uc2dc\uc5d0 \ub4dc\ub9ac\ube14\uc5d0 \uc790\uc2e0\uac10\uc744 \uac00\uc838\uc624\uac8c \ud558\uc600\ub2e4. \ud1a0\ub9c8\uc2dc \ub85c\uc2dc\uce20\ud0a4\ub54c\uc640 \ub9c8\ucc2c\uac00\uc9c0\ub85c \uc719\uc5b4\ub85c \ubcc0\uc2e0\ud558\ub294\ub370\ub294 \uc2e4\ud328\ud558\uc600\uc9c0\ub9cc, \ubbf8\ub4dc\ud544\ub354\uc5d0\uc11c\uc758 \uc790\uc2e0\uac10\uc744 \ud5a5\uc0c1\uc2dc\ud0a8 \ub3d9\uc2dc\uc5d0, \ud0c8\uc555\ubc15 \ub2a5\ub825\uc744 \uae30\ub974\ub294\ub370\ub294 \ud070 \ub3c4\uc6c0\uc774 \ub418\uc5c8\ub2e4. \ub9ac\uadf8\uc5d0\uc11c\uc758 \uc120\ub35c\ub79c\ub4dc AFC\uc640\uc758 \uacbd\uae30\uc5d0\uc11c\ub294 111\ud68c \uc758 \ud130\uce58\ub97c \uae30\ub85d\ud558\uc600\uace0, \uacbd\uae30 \ub0b4 \uc591\ud300 \ud1b5\ud2c0\uc5b4 \ucd5c\ub2e4 \ud328\uc2a4\uc778 88\uac1c\uc758 \ud328\uc2a4\ub97c \ud558\uc600\uc73c\uba70 \uadf8 \uc911 81\uac1c\ub97c \uc131\uacf5\uc2dc\ud0a4\uba74\uc11c 92%\uc758 \uc131\uacf5\ub960\uc744 \ubcf4\uc600\ub2e4. \ub610\ud55c 2014\ub144 5\uc6d4 18\uc77c \uc601\uad6d \uc6f8\ube14\ub9ac \uc2a4\ud0c0\ub514\uc6c0\uc5d0\uc11c \uc5f4\ub9b0 \uc6e8\uc2a4\ud2b8\ud584 \uc720\ub098\uc774\ud2f0\ub4dc\uc640\uc758 2013-14 \uc789\uae00\ub79c\ub4dc FA\ucef5 \uacb0\uc2b9\uc804\uc5d0\uc11c \uc5f0\uc7a5 \ud6c4\ubc18 4\ubd84 \uc2a4\ucf54\uc5b4 2:2\uc0c1\ud669\uc5d0\uc11c \uc5ed\uc804 \uacb0\uc2b9\uace8\uc744 \ubf51\uc544\ub0b4\uba70 \ud300\uc758 9\ub144 \ubb34\uad00\uc744 \ub04a\ub294\ub370 \ud070 \uacf5\ud5cc\uc744 \ud558\uc600\ub2e4.", "sentence_answer": "\ub9ac\uadf8\uc5d0\uc11c\uc758 \uc120\ub35c\ub79c\ub4dc AFC\uc640\uc758 \uacbd\uae30\uc5d0\uc11c\ub294 111\ud68c \uc758 \ud130\uce58\ub97c \uae30\ub85d\ud558\uc600\uace0, \uacbd\uae30 \ub0b4 \uc591\ud300 \ud1b5\ud2c0\uc5b4 \ucd5c\ub2e4 \ud328\uc2a4\uc778 88\uac1c\uc758 \ud328\uc2a4\ub97c \ud558\uc600\uc73c\uba70 \uadf8 \uc911 81\uac1c\ub97c \uc131\uacf5\uc2dc\ud0a4\uba74\uc11c 92%\uc758 \uc131\uacf5\ub960\uc744 \ubcf4\uc600\ub2e4.", "paragraph_id": "6540618-7-0", "paragraph_question": "question: \uc5d0\ub7f0\ub7a8\uc9c0\ub294 \uc120\ub35c\ub79c\ub4dc AFC\uc640\uc758 \uacbd\uae30\uc5d0\uc11c \ucd1d \uba87 \ud68c\uc758 \ud130\uce58\ub97c \uae30\ub85d\ud558\uc600\ub098?, context: \ud504\ub9ac\uc2dc\uc98c \uc88b\uc740 \uacbd\uae30\ub825\uc744 \ud3bc\uce58\uba74\uc11c \uc544\uc2a4\ub0a0 \ud32c\ub4e4\uc5d0\uac8c \uae30\ub300\uac10\uc744 \uc548\uacbc\ub2e4. \ube44\ub85d \uc2dc\uc98c \ucd08\ubc18\uc774\uae34 \ud558\uc9c0\ub9cc \ub9ac\uadf8 4\uacbd\uae30 2\uace8, \ucc54\ud53c\uc5b8\uc2a4\ub9ac\uadf8 3\uacbd\uae30 3\uace8\uc744 \uae30\ub85d\ud558\uba70 \uc62c\ub9ac\ube44\uc5d0 \uc9c0\ub8e8\uc640 \ud568\uaed8 \ud300 \ub0b4 \ub4dd\uc810 1\uc704\ub97c \ub2ec\ub9ac\uace0 \uc788\ub2e4. \ub9c8\ud2f0\uc678 \ud50c\ub77c\ubbf8\ub2c8\uac00 \uc544\uc2a4\ub0a0\uc5d0 \uc601\uc785\ub418\uba74\uc11c \uc5c4\uccad\ub09c \ud65c\ub3d9\ub7c9\uc744 \uac16\ucd98 \ub450 \ubbf8\ub4dc\ud544\ub354\uac00 \uc544\uc2a4\ub0a0\uc758 \uc0c1\uc2b9\uc138\ub97c \uc774\ub04c\uace0 \uc788\ub2e4. 11-12 \uc2dc\uc98c\uc5d0 \uc9c0\uc801\ub418\uc5c8\ub358 \ub2e8\uc810\ub4e4\uc744 \uac1c\uc120\ud588\ub294\ub370 \uc774\ub294 \ubd80\uc815\ud655\ud55c \ud328\uc2a4\uc640 \uacbd\uae30 \ud15c\ud3ec \uc870\uc808 \uc2e4\ud328, \uadf8\ub9ac\uace0 \ub4dc\ub9ac\ube14\uc5d0 \uc790\uc2e0\uac10\uc774 \uc5c6\ub358 \ubd80\ubd84\uc774\uc5c8\ub2e4. 12-13 \uc2dc\uc98c \ubbf8\ucf08 \uc544\ub974\ud14c\ud0c0\uac00 \uc218\ud589\ud558\ub358 \ub525-\ub77c\uc789 \ud50c\ub808\uc774\uba54\uc774\ucee4 \uc5ed\ud560\uc744 \ub9e1\uc73c\uba74\uc11c 87.3%\uc758 \uc815\ud655\ub3c4\uc600\ub358 \ud328\uc2a4 \uc815\ud655\ub3c4\ub97c 88.2%\uae4c\uc9c0 \ub04c\uc5b4\uc62c\ub838\uc73c\uba70, \ubbf8\ucf08 \uc544\ub974\ud14c\ud0c0\uac00 \ubd80\uc7ac\ud560 \ub54c\uc5d4 \uadf8\uac00 \ud3c9\uade0\uc801\uc73c\ub85c \uae30\ub85d\ud558\ub358 91% \uc815\ub3c4\uc758 \ud328\uc2a4 \uc131\uacf5\ub960\uc744 \uae30\ub85d\ud558\uc600\ub2e4. \ubd80\uc0c1 \uc774\ud6c4 \uacbd\uae30 \ud15c\ud3ec \uc870\uc808\uacfc \ubbf8\uc219\ud55c \ub4dc\ub9ac\ube14 \uc2e4\ub825\uc740 \uc544\ub974\uc13c \ubcb5\uac70 \uac10\ub3c5\uc774 \uc88c\uc6b0 \uc719\uc5b4\ub85c \uae30\uc6a9\ud558\uba74\uc11c \uc544\uc2a4\ub0a0\uc758 \ud15c\ud3ec\uc5d0 \uc801\uc751 \uc2dc\ud0a4\ub294 \ub3d9\uc2dc\uc5d0 \ub4dc\ub9ac\ube14\uc5d0 \uc790\uc2e0\uac10\uc744 \uac00\uc838\uc624\uac8c \ud558\uc600\ub2e4. \ud1a0\ub9c8\uc2dc \ub85c\uc2dc\uce20\ud0a4\ub54c\uc640 \ub9c8\ucc2c\uac00\uc9c0\ub85c \uc719\uc5b4\ub85c \ubcc0\uc2e0\ud558\ub294\ub370\ub294 \uc2e4\ud328\ud558\uc600\uc9c0\ub9cc, \ubbf8\ub4dc\ud544\ub354\uc5d0\uc11c\uc758 \uc790\uc2e0\uac10\uc744 \ud5a5\uc0c1\uc2dc\ud0a8 \ub3d9\uc2dc\uc5d0, \ud0c8\uc555\ubc15 \ub2a5\ub825\uc744 \uae30\ub974\ub294\ub370\ub294 \ud070 \ub3c4\uc6c0\uc774 \ub418\uc5c8\ub2e4. \ub9ac\uadf8\uc5d0\uc11c\uc758 \uc120\ub35c\ub79c\ub4dc AFC\uc640\uc758 \uacbd\uae30\uc5d0\uc11c\ub294 111\ud68c\uc758 \ud130\uce58\ub97c \uae30\ub85d\ud558\uc600\uace0, \uacbd\uae30 \ub0b4 \uc591\ud300 \ud1b5\ud2c0\uc5b4 \ucd5c\ub2e4 \ud328\uc2a4\uc778 88\uac1c\uc758 \ud328\uc2a4\ub97c \ud558\uc600\uc73c\uba70 \uadf8 \uc911 81\uac1c\ub97c \uc131\uacf5\uc2dc\ud0a4\uba74\uc11c 92%\uc758 \uc131\uacf5\ub960\uc744 \ubcf4\uc600\ub2e4. \ub610\ud55c 2014\ub144 5\uc6d4 18\uc77c \uc601\uad6d \uc6f8\ube14\ub9ac \uc2a4\ud0c0\ub514\uc6c0\uc5d0\uc11c \uc5f4\ub9b0 \uc6e8\uc2a4\ud2b8\ud584 \uc720\ub098\uc774\ud2f0\ub4dc\uc640\uc758 2013-14 \uc789\uae00\ub79c\ub4dc FA\ucef5 \uacb0\uc2b9\uc804\uc5d0\uc11c \uc5f0\uc7a5 \ud6c4\ubc18 4\ubd84 \uc2a4\ucf54\uc5b4 2:2\uc0c1\ud669\uc5d0\uc11c \uc5ed\uc804 \uacb0\uc2b9\uace8\uc744 \ubf51\uc544\ub0b4\uba70 \ud300\uc758 9\ub144 \ubb34\uad00\uc744 \ub04a\ub294\ub370 \ud070 \uacf5\ud5cc\uc744 \ud558\uc600\ub2e4."} {"question": "1996\ub144 \uc77c\ubcf8\uc774 \ud574\uc11d\ud55c \ub85c\ub9ac\ud0c0\uc801 \uc774\ubbf8\uc9c0\ub85c \ud65c\ub3d9\ud55c \uc601\uad6d\uc758 2\uc778 \uc544\uc774\ub3cc \uadf8\ub8f9\uc740?", "paragraph": "\uc2e4\uc81c\ub85c \uc791\uc911\uc5d0\uc11c \ud5d8\ubc84\ud2b8\uac00 15\uc138\uac00 \ub41c \ub864\ub9ac\ud0c0\ub97c \uaebc\ub824\ud558\ub294 \uc7a5\uba74\uc774 \uc788\ub2e4.\uac70\uae30\uc5d0, \uc791\uc911\uc5d0\uc11c\ub294 \ub864\ub9ac\ud0c0\ub294 12~16\uc138 \ubb34\ub835\uae4c\uc9c0 \uadf8\ub824\uc838\uc788\uc73c\ubbc0\ub85c, \uc18c\uc124 \uc790\uccb4\ub85c \ubcf4\uba74 \ub2f9\uc2dc 15\uc138\uc778 \uc218 \ub77c\uc774\uc628\ub3c4 \ubc30\uc5ed\uc73c\ub85c\ub294 \uc544\ubb34\ud2bc \uc801\ud569\ud558\ub2e4. \ub610\ud55c, 1995\ub144 3\uc6d4 16\uc77c \ub2c8\ud63c\ucf00\uc774\uc790\uc774 \uc2e0\ubb38\uc740 \ub2f9\uc2dc \uc77c\ubcf8\uc5d0\uc11c \uc720\ud589\ud558\ub358 \uc6d0\uc0c9\uacc4 \ub85c\ub9ac\ud0c0 \uc2a4\ud0c0\uc77c\uacfc \ud050\ube0c\ub9ad\ud310 \ub85c\ub9ac\ud0c0\uac00 \uacf5\ud1b5\ub41c\ub2e4\uace0 \uc9c0\uc801\ud588\ub2e4.\uadf8\ub9ac\uace0 1995\ub144 \ub2f9\uc2dc Shampoo\ub77c\ub294 \uc601\uad6d 2\uc778 \ud3b8\uc131 \uc544\uc774\ub3cc \uadf8\ub8f9\uc774 \uc774 \ud050\ube0c\ub9ad\ud310 \ub864\ub9ac\ud0c0\ub97c \ubc29\ubd88\ucf00\ud558\ub294 \uc18c\ub140\uc801\uc778 \uceec\ub7ec\ud480&\ud0a4\uce58\ud55c \ud328\uc158\uc744 \uc785\uace0, \ubc18\uc0ac\ud68c\uc801\uc778 \ud391\ud06c \ub85d\uc744 \uc544\uae30\uc790\uae30\ud558\uace0 \ub2ec\ucf64\ud55c \ubcf4\uc774\uc2a4\ub85c \ub178\ub798\ud558\uc5ec \uc8fc\ubaa9\uc744 \ub04c\uc5c8\uace0, \uc77c\ubcf8 \ub3c5\uc790 \ud574\uc11d\uc758 \ub85c\ub9ac\ud0c0\uc5d0 \uac00\uae5d\uac8c \ud65c\ub3d9\ud588\uc5c8\ub2e4. \ubb34\uc5c7\ubcf4\ub2e4\ub3c4, \ub2f9\uc2dc HMV \uc2e0\uc0ac\uc774\ubc14\uc2dc\uc810\uc740 \uc544\uae30\uc790\uae30\ud558\uace0 \ub2ec\ucf64\ud55c \ubaa9\uc18c\ub9ac\uc640 \uc2a4\ud0c0\uc77c\uc740 \uacf5\ud1b5\ub418\uc9c0\ub9cc \uc74c\uc545\uc801\uc73c\ub85c \ubd84\ub958\ud560 \uc218\ub294 \uc5c6\ub2e4\uace0 \ud558\uae30\ub3c4 \ud588\ub2e4.", "answer": "Shampoo", "sentence": "\ub610\ud55c, 1995\ub144 3\uc6d4 16\uc77c \ub2c8\ud63c\ucf00\uc774\uc790\uc774 \uc2e0\ubb38\uc740 \ub2f9\uc2dc \uc77c\ubcf8\uc5d0\uc11c \uc720\ud589\ud558\ub358 \uc6d0\uc0c9\uacc4 \ub85c\ub9ac\ud0c0 \uc2a4\ud0c0\uc77c\uacfc \ud050\ube0c\ub9ad\ud310 \ub85c\ub9ac\ud0c0\uac00 \uacf5\ud1b5\ub41c\ub2e4\uace0 \uc9c0\uc801\ud588\ub2e4.\uadf8\ub9ac\uace0 1995\ub144 \ub2f9\uc2dc Shampoo \ub77c\ub294 \uc601\uad6d 2\uc778 \ud3b8\uc131 \uc544\uc774\ub3cc \uadf8\ub8f9\uc774 \uc774 \ud050\ube0c\ub9ad\ud310 \ub864\ub9ac\ud0c0\ub97c \ubc29\ubd88\ucf00\ud558\ub294 \uc18c\ub140\uc801\uc778 \uceec\ub7ec\ud480&\ud0a4\uce58\ud55c \ud328\uc158\uc744 \uc785\uace0, \ubc18\uc0ac\ud68c\uc801\uc778 \ud391\ud06c \ub85d\uc744 \uc544\uae30\uc790\uae30\ud558\uace0 \ub2ec\ucf64\ud55c \ubcf4\uc774\uc2a4\ub85c \ub178\ub798\ud558\uc5ec \uc8fc\ubaa9\uc744 \ub04c\uc5c8\uace0, \uc77c\ubcf8 \ub3c5\uc790 \ud574\uc11d\uc758 \ub85c\ub9ac\ud0c0\uc5d0 \uac00\uae5d\uac8c \ud65c\ub3d9\ud588\uc5c8\ub2e4.", "paragraph_sentence": "\uc2e4\uc81c\ub85c \uc791\uc911\uc5d0\uc11c \ud5d8\ubc84\ud2b8\uac00 15\uc138\uac00 \ub41c \ub864\ub9ac\ud0c0\ub97c \uaebc\ub824\ud558\ub294 \uc7a5\uba74\uc774 \uc788\ub2e4.\uac70\uae30\uc5d0, \uc791\uc911\uc5d0\uc11c\ub294 \ub864\ub9ac\ud0c0\ub294 12~16\uc138 \ubb34\ub835\uae4c\uc9c0 \uadf8\ub824\uc838\uc788\uc73c\ubbc0\ub85c, \uc18c\uc124 \uc790\uccb4\ub85c \ubcf4\uba74 \ub2f9\uc2dc 15\uc138\uc778 \uc218 \ub77c\uc774\uc628\ub3c4 \ubc30\uc5ed\uc73c\ub85c\ub294 \uc544\ubb34\ud2bc \uc801\ud569\ud558\ub2e4. \ub610\ud55c, 1995\ub144 3\uc6d4 16\uc77c \ub2c8\ud63c\ucf00\uc774\uc790\uc774 \uc2e0\ubb38\uc740 \ub2f9\uc2dc \uc77c\ubcf8\uc5d0\uc11c \uc720\ud589\ud558\ub358 \uc6d0\uc0c9\uacc4 \ub85c\ub9ac\ud0c0 \uc2a4\ud0c0\uc77c\uacfc \ud050\ube0c\ub9ad\ud310 \ub85c\ub9ac\ud0c0\uac00 \uacf5\ud1b5\ub41c\ub2e4\uace0 \uc9c0\uc801\ud588\ub2e4.\uadf8\ub9ac\uace0 1995\ub144 \ub2f9\uc2dc Shampoo \ub77c\ub294 \uc601\uad6d 2\uc778 \ud3b8\uc131 \uc544\uc774\ub3cc \uadf8\ub8f9\uc774 \uc774 \ud050\ube0c\ub9ad\ud310 \ub864\ub9ac\ud0c0\ub97c \ubc29\ubd88\ucf00\ud558\ub294 \uc18c\ub140\uc801\uc778 \uceec\ub7ec\ud480&\ud0a4\uce58\ud55c \ud328\uc158\uc744 \uc785\uace0, \ubc18\uc0ac\ud68c\uc801\uc778 \ud391\ud06c \ub85d\uc744 \uc544\uae30\uc790\uae30\ud558\uace0 \ub2ec\ucf64\ud55c \ubcf4\uc774\uc2a4\ub85c \ub178\ub798\ud558\uc5ec \uc8fc\ubaa9\uc744 \ub04c\uc5c8\uace0, \uc77c\ubcf8 \ub3c5\uc790 \ud574\uc11d\uc758 \ub85c\ub9ac\ud0c0\uc5d0 \uac00\uae5d\uac8c \ud65c\ub3d9\ud588\uc5c8\ub2e4. \ubb34\uc5c7\ubcf4\ub2e4\ub3c4, \ub2f9\uc2dc HMV \uc2e0\uc0ac\uc774\ubc14\uc2dc\uc810\uc740 \uc544\uae30\uc790\uae30\ud558\uace0 \ub2ec\ucf64\ud55c \ubaa9\uc18c\ub9ac\uc640 \uc2a4\ud0c0\uc77c\uc740 \uacf5\ud1b5\ub418\uc9c0\ub9cc \uc74c\uc545\uc801\uc73c\ub85c \ubd84\ub958\ud560 \uc218\ub294 \uc5c6\ub2e4\uace0 \ud558\uae30\ub3c4 \ud588\ub2e4.", "paragraph_answer": "\uc2e4\uc81c\ub85c \uc791\uc911\uc5d0\uc11c \ud5d8\ubc84\ud2b8\uac00 15\uc138\uac00 \ub41c \ub864\ub9ac\ud0c0\ub97c \uaebc\ub824\ud558\ub294 \uc7a5\uba74\uc774 \uc788\ub2e4.\uac70\uae30\uc5d0, \uc791\uc911\uc5d0\uc11c\ub294 \ub864\ub9ac\ud0c0\ub294 12~16\uc138 \ubb34\ub835\uae4c\uc9c0 \uadf8\ub824\uc838\uc788\uc73c\ubbc0\ub85c, \uc18c\uc124 \uc790\uccb4\ub85c \ubcf4\uba74 \ub2f9\uc2dc 15\uc138\uc778 \uc218 \ub77c\uc774\uc628\ub3c4 \ubc30\uc5ed\uc73c\ub85c\ub294 \uc544\ubb34\ud2bc \uc801\ud569\ud558\ub2e4. \ub610\ud55c, 1995\ub144 3\uc6d4 16\uc77c \ub2c8\ud63c\ucf00\uc774\uc790\uc774 \uc2e0\ubb38\uc740 \ub2f9\uc2dc \uc77c\ubcf8\uc5d0\uc11c \uc720\ud589\ud558\ub358 \uc6d0\uc0c9\uacc4 \ub85c\ub9ac\ud0c0 \uc2a4\ud0c0\uc77c\uacfc \ud050\ube0c\ub9ad\ud310 \ub85c\ub9ac\ud0c0\uac00 \uacf5\ud1b5\ub41c\ub2e4\uace0 \uc9c0\uc801\ud588\ub2e4.\uadf8\ub9ac\uace0 1995\ub144 \ub2f9\uc2dc Shampoo \ub77c\ub294 \uc601\uad6d 2\uc778 \ud3b8\uc131 \uc544\uc774\ub3cc \uadf8\ub8f9\uc774 \uc774 \ud050\ube0c\ub9ad\ud310 \ub864\ub9ac\ud0c0\ub97c \ubc29\ubd88\ucf00\ud558\ub294 \uc18c\ub140\uc801\uc778 \uceec\ub7ec\ud480&\ud0a4\uce58\ud55c \ud328\uc158\uc744 \uc785\uace0, \ubc18\uc0ac\ud68c\uc801\uc778 \ud391\ud06c \ub85d\uc744 \uc544\uae30\uc790\uae30\ud558\uace0 \ub2ec\ucf64\ud55c \ubcf4\uc774\uc2a4\ub85c \ub178\ub798\ud558\uc5ec \uc8fc\ubaa9\uc744 \ub04c\uc5c8\uace0, \uc77c\ubcf8 \ub3c5\uc790 \ud574\uc11d\uc758 \ub85c\ub9ac\ud0c0\uc5d0 \uac00\uae5d\uac8c \ud65c\ub3d9\ud588\uc5c8\ub2e4. \ubb34\uc5c7\ubcf4\ub2e4\ub3c4, \ub2f9\uc2dc HMV \uc2e0\uc0ac\uc774\ubc14\uc2dc\uc810\uc740 \uc544\uae30\uc790\uae30\ud558\uace0 \ub2ec\ucf64\ud55c \ubaa9\uc18c\ub9ac\uc640 \uc2a4\ud0c0\uc77c\uc740 \uacf5\ud1b5\ub418\uc9c0\ub9cc \uc74c\uc545\uc801\uc73c\ub85c \ubd84\ub958\ud560 \uc218\ub294 \uc5c6\ub2e4\uace0 \ud558\uae30\ub3c4 \ud588\ub2e4.", "sentence_answer": "\ub610\ud55c, 1995\ub144 3\uc6d4 16\uc77c \ub2c8\ud63c\ucf00\uc774\uc790\uc774 \uc2e0\ubb38\uc740 \ub2f9\uc2dc \uc77c\ubcf8\uc5d0\uc11c \uc720\ud589\ud558\ub358 \uc6d0\uc0c9\uacc4 \ub85c\ub9ac\ud0c0 \uc2a4\ud0c0\uc77c\uacfc \ud050\ube0c\ub9ad\ud310 \ub85c\ub9ac\ud0c0\uac00 \uacf5\ud1b5\ub41c\ub2e4\uace0 \uc9c0\uc801\ud588\ub2e4.\uadf8\ub9ac\uace0 1995\ub144 \ub2f9\uc2dc Shampoo \ub77c\ub294 \uc601\uad6d 2\uc778 \ud3b8\uc131 \uc544\uc774\ub3cc \uadf8\ub8f9\uc774 \uc774 \ud050\ube0c\ub9ad\ud310 \ub864\ub9ac\ud0c0\ub97c \ubc29\ubd88\ucf00\ud558\ub294 \uc18c\ub140\uc801\uc778 \uceec\ub7ec\ud480&\ud0a4\uce58\ud55c \ud328\uc158\uc744 \uc785\uace0, \ubc18\uc0ac\ud68c\uc801\uc778 \ud391\ud06c \ub85d\uc744 \uc544\uae30\uc790\uae30\ud558\uace0 \ub2ec\ucf64\ud55c \ubcf4\uc774\uc2a4\ub85c \ub178\ub798\ud558\uc5ec \uc8fc\ubaa9\uc744 \ub04c\uc5c8\uace0, \uc77c\ubcf8 \ub3c5\uc790 \ud574\uc11d\uc758 \ub85c\ub9ac\ud0c0\uc5d0 \uac00\uae5d\uac8c \ud65c\ub3d9\ud588\uc5c8\ub2e4.", "paragraph_id": "6513684-2-1", "paragraph_question": "question: 1996\ub144 \uc77c\ubcf8\uc774 \ud574\uc11d\ud55c \ub85c\ub9ac\ud0c0\uc801 \uc774\ubbf8\uc9c0\ub85c \ud65c\ub3d9\ud55c \uc601\uad6d\uc758 2\uc778 \uc544\uc774\ub3cc \uadf8\ub8f9\uc740?, context: \uc2e4\uc81c\ub85c \uc791\uc911\uc5d0\uc11c \ud5d8\ubc84\ud2b8\uac00 15\uc138\uac00 \ub41c \ub864\ub9ac\ud0c0\ub97c \uaebc\ub824\ud558\ub294 \uc7a5\uba74\uc774 \uc788\ub2e4.\uac70\uae30\uc5d0, \uc791\uc911\uc5d0\uc11c\ub294 \ub864\ub9ac\ud0c0\ub294 12~16\uc138 \ubb34\ub835\uae4c\uc9c0 \uadf8\ub824\uc838\uc788\uc73c\ubbc0\ub85c, \uc18c\uc124 \uc790\uccb4\ub85c \ubcf4\uba74 \ub2f9\uc2dc 15\uc138\uc778 \uc218 \ub77c\uc774\uc628\ub3c4 \ubc30\uc5ed\uc73c\ub85c\ub294 \uc544\ubb34\ud2bc \uc801\ud569\ud558\ub2e4. \ub610\ud55c, 1995\ub144 3\uc6d4 16\uc77c \ub2c8\ud63c\ucf00\uc774\uc790\uc774 \uc2e0\ubb38\uc740 \ub2f9\uc2dc \uc77c\ubcf8\uc5d0\uc11c \uc720\ud589\ud558\ub358 \uc6d0\uc0c9\uacc4 \ub85c\ub9ac\ud0c0 \uc2a4\ud0c0\uc77c\uacfc \ud050\ube0c\ub9ad\ud310 \ub85c\ub9ac\ud0c0\uac00 \uacf5\ud1b5\ub41c\ub2e4\uace0 \uc9c0\uc801\ud588\ub2e4.\uadf8\ub9ac\uace0 1995\ub144 \ub2f9\uc2dc Shampoo\ub77c\ub294 \uc601\uad6d 2\uc778 \ud3b8\uc131 \uc544\uc774\ub3cc \uadf8\ub8f9\uc774 \uc774 \ud050\ube0c\ub9ad\ud310 \ub864\ub9ac\ud0c0\ub97c \ubc29\ubd88\ucf00\ud558\ub294 \uc18c\ub140\uc801\uc778 \uceec\ub7ec\ud480&\ud0a4\uce58\ud55c \ud328\uc158\uc744 \uc785\uace0, \ubc18\uc0ac\ud68c\uc801\uc778 \ud391\ud06c \ub85d\uc744 \uc544\uae30\uc790\uae30\ud558\uace0 \ub2ec\ucf64\ud55c \ubcf4\uc774\uc2a4\ub85c \ub178\ub798\ud558\uc5ec \uc8fc\ubaa9\uc744 \ub04c\uc5c8\uace0, \uc77c\ubcf8 \ub3c5\uc790 \ud574\uc11d\uc758 \ub85c\ub9ac\ud0c0\uc5d0 \uac00\uae5d\uac8c \ud65c\ub3d9\ud588\uc5c8\ub2e4. \ubb34\uc5c7\ubcf4\ub2e4\ub3c4, \ub2f9\uc2dc HMV \uc2e0\uc0ac\uc774\ubc14\uc2dc\uc810\uc740 \uc544\uae30\uc790\uae30\ud558\uace0 \ub2ec\ucf64\ud55c \ubaa9\uc18c\ub9ac\uc640 \uc2a4\ud0c0\uc77c\uc740 \uacf5\ud1b5\ub418\uc9c0\ub9cc \uc74c\uc545\uc801\uc73c\ub85c \ubd84\ub958\ud560 \uc218\ub294 \uc5c6\ub2e4\uace0 \ud558\uae30\ub3c4 \ud588\ub2e4."} {"question": "\ub178\ubb34\ud604\uc758 \uc8fd\uc74c\uc5d0 \ub300\ud574 \ub2f9\uc2dc \uc870\ubb38\uc744 \ubc1c\ud45c\ud55c \uc911\ud654\uc778\ubbfc\uacf5\ud654\uad6d \uc8fc\uc11d\uc740?", "paragraph": "\ud6c4\uc9c4\ud0c0\uc624 \uc911\ud654\uc778\ubbfc\uacf5\ud654\uad6d \uc8fc\uc11d\uc740 \uc870\ubb38\uc5d0\uc11c \"\ub178 \uc804 \ub300\ud1b5\ub839\uc740 \ub098\uc758 \uc624\ub798\ub41c \uce5c\uad6c\"\ub77c\uba70 \"\uc7ac\uc784 \uae30\uac04\uc5d0 \uc911\uad6d\uacfc \ud55c\uad6d\uc758 \uc804\uba74\uc801 \ud611\ub825 \ub3d9\ubc18\uc790 \uad00\uacc4 \uc218\ub9bd \ubc0f \ubc1c\uc804\uc744 \uc704\ud574 \uc911\uc694\ud55c \uae30\uc5ec\ub97c \ud588\ub2e4\"\ub77c\uace0 \ubc1d\ud614\ub2e4. \uc6d0\uc790\ubc14\uc624 \ucd1d\ub9ac\ub294 \uc560\ub3c4\ud558\uba74\uc11c \ub178\ubb34\ud604\uc758 \ub300(\u5c0d)\uc911\uad6d \uad00\uacc4\uc758 \uc911\uc2dc, \ub178\ubb34\ud604\uc758 \uc194\uc9c1\ud568\uacfc \uc131\uc2e4\ud568\uc774 \uae4a\uc740 \uc778\uc0c1\uc744 \ubc1b\uc558\ub2e4\uba70 \ud55c\uc911 \uad00\uacc4\uc758 \uc804\uba74\uc801 \ubc1c\uc804 \ucd94\uc9c4\uc744 \uc704\ud574 \uae30\uc6b8\uc778 \uacf5\ud5cc\uc744 \uae30\uc5b5\ud558\uaca0\ub2e4\ub294 \uc18c\ud68c\ub97c \ub367\ubd99\uc600\ub2e4. \ubbf8\uad6d\uc758 \ubc84\ub77d \uc624\ubc14\ub9c8 \ub300\ud1b5\ub839\uc740 \uae34\uae09 \uc560\ub3c4 \uc131\uba85\uc744 \ubc1c\ud45c\ud558\uc600\ub2e4. \uc774 \uc131\uba85\uc5d0\uc11c \ub178\ubb34\ud604 \uc7ac\uc784 \uae30\uac04\uc5d0 \ud55c\uad6d\uacfc \ubbf8\uad6d \uac04\uc758 '\uac15\ub825\ud558\uace0 \ud65c\uae30\ucc2c'(strong and vital) \uad00\uacc4\ub97c \ud615\uc131\ud558\ub294 \ub370 \uae30\uc5ec\ud588\ub2e4\uba70 \uc560\ub3c4\uc758 \ub73b\uc744 \uc804\ud588\ub2e4. \uc544\uc18c \ub2e4\ub85c \uc77c\ubcf8 \ucd1d\ub9ac\ub294 \uc678\uc0c1 \uc2dc\uc808 \ub178 \uc804 \ub300\ud1b5\ub839\uc744 \ub9cc\ub0ac\ub358 \uae30\uc5b5\uc744 \ub5a0\uc62c\ub9ac\uba70 \uc560\ub3c4\ud588\ub2e4. \uc601\uad6d\uc758 \uc5d8\ub9ac\uc790\ubca0\uc2a4 2\uc138 \uc5ec\uc655\ub3c4 \uccad\uc640\ub300\uc5d0 \uc560\ub3c4 \uc870\ubb38\uc744 \ubcf4\ub0b4\uc5b4, \"\uc9c0\ub09c 2004\ub144 \ub178 \uc804 \ub300\ud1b5\ub839\uc758 \uc601\uad6d \uacf5\uc2dd \ubc29\ubb38\uc740 \ud55c\u00b7\uc601 \uc591\uad6d \uad00\uacc4 \uc99d\uc9c4\uc5d0 \uc911\uc694\ud55c \uc774\uc815\ud45c\uc600\ub2e4\"\ub77c\uace0 \uc804\ud588\ub2e4. \ubc18\uae30\ubb38 \uad6d\uc81c \uc5f0\ud569 \uc0ac\ubb34\ucd1d\uc7a5\uc740 \uc0ac\ub9dd \uad00\ub828 \uc131\uba85\uc744 \ubc1c\ud45c\ud558\uace0 \uc560\ub3c4\uc758 \ub73b\uc744 \ud45c\uba85\ud558\uba74\uc11c, \"\ub178 \uc804 \ub300\ud1b5\ub839\uc740 \ubbfc\uc8fc\uc8fc\uc758 \ucd09\uc9c4\uc744 \uc704\ud574 \uc5ec\ub7ec \uac00\uc9c0 \ub178\ub825\uc744 \uae30\uc6b8\uc600\ub2e4\"\ub77c\uace0 \ub178 \uc804 \ub300\ud1b5\ub839\uc744 \uce6d\uc1a1\ud588\ub2e4. \uace0\ub4e0 \ube0c\ub77c\uc6b4 \uc601\uad6d \ucd1d\ub9ac \uc5ed\uc2dc \uc560\ub3c4\uc758 \ub73b\uc744 \uc804\ud588\ub2e4.", "answer": "\ud6c4\uc9c4\ud0c0\uc624", "sentence": "\ud6c4\uc9c4\ud0c0\uc624 \uc911\ud654\uc778\ubbfc\uacf5\ud654\uad6d \uc8fc\uc11d\uc740 \uc870\ubb38\uc5d0\uc11c \"\ub178 \uc804 \ub300\ud1b5\ub839\uc740 \ub098\uc758 \uc624\ub798\ub41c \uce5c\uad6c\"\ub77c\uba70 \"\uc7ac\uc784 \uae30\uac04\uc5d0 \uc911\uad6d\uacfc \ud55c\uad6d\uc758 \uc804\uba74\uc801 \ud611\ub825 \ub3d9\ubc18\uc790 \uad00\uacc4 \uc218\ub9bd \ubc0f \ubc1c\uc804\uc744 \uc704\ud574 \uc911\uc694\ud55c \uae30\uc5ec\ub97c \ud588\ub2e4\"\ub77c\uace0 \ubc1d\ud614\ub2e4.", "paragraph_sentence": " \ud6c4\uc9c4\ud0c0\uc624 \uc911\ud654\uc778\ubbfc\uacf5\ud654\uad6d \uc8fc\uc11d\uc740 \uc870\ubb38\uc5d0\uc11c \"\ub178 \uc804 \ub300\ud1b5\ub839\uc740 \ub098\uc758 \uc624\ub798\ub41c \uce5c\uad6c\"\ub77c\uba70 \"\uc7ac\uc784 \uae30\uac04\uc5d0 \uc911\uad6d\uacfc \ud55c\uad6d\uc758 \uc804\uba74\uc801 \ud611\ub825 \ub3d9\ubc18\uc790 \uad00\uacc4 \uc218\ub9bd \ubc0f \ubc1c\uc804\uc744 \uc704\ud574 \uc911\uc694\ud55c \uae30\uc5ec\ub97c \ud588\ub2e4\"\ub77c\uace0 \ubc1d\ud614\ub2e4. \uc6d0\uc790\ubc14\uc624 \ucd1d\ub9ac\ub294 \uc560\ub3c4\ud558\uba74\uc11c \ub178\ubb34\ud604\uc758 \ub300(\u5c0d)\uc911\uad6d \uad00\uacc4\uc758 \uc911\uc2dc, \ub178\ubb34\ud604\uc758 \uc194\uc9c1\ud568\uacfc \uc131\uc2e4\ud568\uc774 \uae4a\uc740 \uc778\uc0c1\uc744 \ubc1b\uc558\ub2e4\uba70 \ud55c\uc911 \uad00\uacc4\uc758 \uc804\uba74\uc801 \ubc1c\uc804 \ucd94\uc9c4\uc744 \uc704\ud574 \uae30\uc6b8\uc778 \uacf5\ud5cc\uc744 \uae30\uc5b5\ud558\uaca0\ub2e4\ub294 \uc18c\ud68c\ub97c \ub367\ubd99\uc600\ub2e4. \ubbf8\uad6d\uc758 \ubc84\ub77d \uc624\ubc14\ub9c8 \ub300\ud1b5\ub839\uc740 \uae34\uae09 \uc560\ub3c4 \uc131\uba85\uc744 \ubc1c\ud45c\ud558\uc600\ub2e4. \uc774 \uc131\uba85\uc5d0\uc11c \ub178\ubb34\ud604 \uc7ac\uc784 \uae30\uac04\uc5d0 \ud55c\uad6d\uacfc \ubbf8\uad6d \uac04\uc758 '\uac15\ub825\ud558\uace0 \ud65c\uae30\ucc2c'(strong and vital) \uad00\uacc4\ub97c \ud615\uc131\ud558\ub294 \ub370 \uae30\uc5ec\ud588\ub2e4\uba70 \uc560\ub3c4\uc758 \ub73b\uc744 \uc804\ud588\ub2e4. \uc544\uc18c \ub2e4\ub85c \uc77c\ubcf8 \ucd1d\ub9ac\ub294 \uc678\uc0c1 \uc2dc\uc808 \ub178 \uc804 \ub300\ud1b5\ub839\uc744 \ub9cc\ub0ac\ub358 \uae30\uc5b5\uc744 \ub5a0\uc62c\ub9ac\uba70 \uc560\ub3c4\ud588\ub2e4. \uc601\uad6d\uc758 \uc5d8\ub9ac\uc790\ubca0\uc2a4 2\uc138 \uc5ec\uc655\ub3c4 \uccad\uc640\ub300\uc5d0 \uc560\ub3c4 \uc870\ubb38\uc744 \ubcf4\ub0b4\uc5b4, \"\uc9c0\ub09c 2004\ub144 \ub178 \uc804 \ub300\ud1b5\ub839\uc758 \uc601\uad6d \uacf5\uc2dd \ubc29\ubb38\uc740 \ud55c\u00b7\uc601 \uc591\uad6d \uad00\uacc4 \uc99d\uc9c4\uc5d0 \uc911\uc694\ud55c \uc774\uc815\ud45c\uc600\ub2e4\"\ub77c\uace0 \uc804\ud588\ub2e4. \ubc18\uae30\ubb38 \uad6d\uc81c \uc5f0\ud569 \uc0ac\ubb34\ucd1d\uc7a5\uc740 \uc0ac\ub9dd \uad00\ub828 \uc131\uba85\uc744 \ubc1c\ud45c\ud558\uace0 \uc560\ub3c4\uc758 \ub73b\uc744 \ud45c\uba85\ud558\uba74\uc11c, \"\ub178 \uc804 \ub300\ud1b5\ub839\uc740 \ubbfc\uc8fc\uc8fc\uc758 \ucd09\uc9c4\uc744 \uc704\ud574 \uc5ec\ub7ec \uac00\uc9c0 \ub178\ub825\uc744 \uae30\uc6b8\uc600\ub2e4\"\ub77c\uace0 \ub178 \uc804 \ub300\ud1b5\ub839\uc744 \uce6d\uc1a1\ud588\ub2e4. \uace0\ub4e0 \ube0c\ub77c\uc6b4 \uc601\uad6d \ucd1d\ub9ac \uc5ed\uc2dc \uc560\ub3c4\uc758 \ub73b\uc744 \uc804\ud588\ub2e4.", "paragraph_answer": " \ud6c4\uc9c4\ud0c0\uc624 \uc911\ud654\uc778\ubbfc\uacf5\ud654\uad6d \uc8fc\uc11d\uc740 \uc870\ubb38\uc5d0\uc11c \"\ub178 \uc804 \ub300\ud1b5\ub839\uc740 \ub098\uc758 \uc624\ub798\ub41c \uce5c\uad6c\"\ub77c\uba70 \"\uc7ac\uc784 \uae30\uac04\uc5d0 \uc911\uad6d\uacfc \ud55c\uad6d\uc758 \uc804\uba74\uc801 \ud611\ub825 \ub3d9\ubc18\uc790 \uad00\uacc4 \uc218\ub9bd \ubc0f \ubc1c\uc804\uc744 \uc704\ud574 \uc911\uc694\ud55c \uae30\uc5ec\ub97c \ud588\ub2e4\"\ub77c\uace0 \ubc1d\ud614\ub2e4. \uc6d0\uc790\ubc14\uc624 \ucd1d\ub9ac\ub294 \uc560\ub3c4\ud558\uba74\uc11c \ub178\ubb34\ud604\uc758 \ub300(\u5c0d)\uc911\uad6d \uad00\uacc4\uc758 \uc911\uc2dc, \ub178\ubb34\ud604\uc758 \uc194\uc9c1\ud568\uacfc \uc131\uc2e4\ud568\uc774 \uae4a\uc740 \uc778\uc0c1\uc744 \ubc1b\uc558\ub2e4\uba70 \ud55c\uc911 \uad00\uacc4\uc758 \uc804\uba74\uc801 \ubc1c\uc804 \ucd94\uc9c4\uc744 \uc704\ud574 \uae30\uc6b8\uc778 \uacf5\ud5cc\uc744 \uae30\uc5b5\ud558\uaca0\ub2e4\ub294 \uc18c\ud68c\ub97c \ub367\ubd99\uc600\ub2e4. \ubbf8\uad6d\uc758 \ubc84\ub77d \uc624\ubc14\ub9c8 \ub300\ud1b5\ub839\uc740 \uae34\uae09 \uc560\ub3c4 \uc131\uba85\uc744 \ubc1c\ud45c\ud558\uc600\ub2e4. \uc774 \uc131\uba85\uc5d0\uc11c \ub178\ubb34\ud604 \uc7ac\uc784 \uae30\uac04\uc5d0 \ud55c\uad6d\uacfc \ubbf8\uad6d \uac04\uc758 '\uac15\ub825\ud558\uace0 \ud65c\uae30\ucc2c'(strong and vital) \uad00\uacc4\ub97c \ud615\uc131\ud558\ub294 \ub370 \uae30\uc5ec\ud588\ub2e4\uba70 \uc560\ub3c4\uc758 \ub73b\uc744 \uc804\ud588\ub2e4. \uc544\uc18c \ub2e4\ub85c \uc77c\ubcf8 \ucd1d\ub9ac\ub294 \uc678\uc0c1 \uc2dc\uc808 \ub178 \uc804 \ub300\ud1b5\ub839\uc744 \ub9cc\ub0ac\ub358 \uae30\uc5b5\uc744 \ub5a0\uc62c\ub9ac\uba70 \uc560\ub3c4\ud588\ub2e4. \uc601\uad6d\uc758 \uc5d8\ub9ac\uc790\ubca0\uc2a4 2\uc138 \uc5ec\uc655\ub3c4 \uccad\uc640\ub300\uc5d0 \uc560\ub3c4 \uc870\ubb38\uc744 \ubcf4\ub0b4\uc5b4, \"\uc9c0\ub09c 2004\ub144 \ub178 \uc804 \ub300\ud1b5\ub839\uc758 \uc601\uad6d \uacf5\uc2dd \ubc29\ubb38\uc740 \ud55c\u00b7\uc601 \uc591\uad6d \uad00\uacc4 \uc99d\uc9c4\uc5d0 \uc911\uc694\ud55c \uc774\uc815\ud45c\uc600\ub2e4\"\ub77c\uace0 \uc804\ud588\ub2e4. \ubc18\uae30\ubb38 \uad6d\uc81c \uc5f0\ud569 \uc0ac\ubb34\ucd1d\uc7a5\uc740 \uc0ac\ub9dd \uad00\ub828 \uc131\uba85\uc744 \ubc1c\ud45c\ud558\uace0 \uc560\ub3c4\uc758 \ub73b\uc744 \ud45c\uba85\ud558\uba74\uc11c, \"\ub178 \uc804 \ub300\ud1b5\ub839\uc740 \ubbfc\uc8fc\uc8fc\uc758 \ucd09\uc9c4\uc744 \uc704\ud574 \uc5ec\ub7ec \uac00\uc9c0 \ub178\ub825\uc744 \uae30\uc6b8\uc600\ub2e4\"\ub77c\uace0 \ub178 \uc804 \ub300\ud1b5\ub839\uc744 \uce6d\uc1a1\ud588\ub2e4. \uace0\ub4e0 \ube0c\ub77c\uc6b4 \uc601\uad6d \ucd1d\ub9ac \uc5ed\uc2dc \uc560\ub3c4\uc758 \ub73b\uc744 \uc804\ud588\ub2e4.", "sentence_answer": " \ud6c4\uc9c4\ud0c0\uc624 \uc911\ud654\uc778\ubbfc\uacf5\ud654\uad6d \uc8fc\uc11d\uc740 \uc870\ubb38\uc5d0\uc11c \"\ub178 \uc804 \ub300\ud1b5\ub839\uc740 \ub098\uc758 \uc624\ub798\ub41c \uce5c\uad6c\"\ub77c\uba70 \"\uc7ac\uc784 \uae30\uac04\uc5d0 \uc911\uad6d\uacfc \ud55c\uad6d\uc758 \uc804\uba74\uc801 \ud611\ub825 \ub3d9\ubc18\uc790 \uad00\uacc4 \uc218\ub9bd \ubc0f \ubc1c\uc804\uc744 \uc704\ud574 \uc911\uc694\ud55c \uae30\uc5ec\ub97c \ud588\ub2e4\"\ub77c\uace0 \ubc1d\ud614\ub2e4.", "paragraph_id": "6512090-26-0", "paragraph_question": "question: \ub178\ubb34\ud604\uc758 \uc8fd\uc74c\uc5d0 \ub300\ud574 \ub2f9\uc2dc \uc870\ubb38\uc744 \ubc1c\ud45c\ud55c \uc911\ud654\uc778\ubbfc\uacf5\ud654\uad6d \uc8fc\uc11d\uc740?, context: \ud6c4\uc9c4\ud0c0\uc624 \uc911\ud654\uc778\ubbfc\uacf5\ud654\uad6d \uc8fc\uc11d\uc740 \uc870\ubb38\uc5d0\uc11c \"\ub178 \uc804 \ub300\ud1b5\ub839\uc740 \ub098\uc758 \uc624\ub798\ub41c \uce5c\uad6c\"\ub77c\uba70 \"\uc7ac\uc784 \uae30\uac04\uc5d0 \uc911\uad6d\uacfc \ud55c\uad6d\uc758 \uc804\uba74\uc801 \ud611\ub825 \ub3d9\ubc18\uc790 \uad00\uacc4 \uc218\ub9bd \ubc0f \ubc1c\uc804\uc744 \uc704\ud574 \uc911\uc694\ud55c \uae30\uc5ec\ub97c \ud588\ub2e4\"\ub77c\uace0 \ubc1d\ud614\ub2e4. \uc6d0\uc790\ubc14\uc624 \ucd1d\ub9ac\ub294 \uc560\ub3c4\ud558\uba74\uc11c \ub178\ubb34\ud604\uc758 \ub300(\u5c0d)\uc911\uad6d \uad00\uacc4\uc758 \uc911\uc2dc, \ub178\ubb34\ud604\uc758 \uc194\uc9c1\ud568\uacfc \uc131\uc2e4\ud568\uc774 \uae4a\uc740 \uc778\uc0c1\uc744 \ubc1b\uc558\ub2e4\uba70 \ud55c\uc911 \uad00\uacc4\uc758 \uc804\uba74\uc801 \ubc1c\uc804 \ucd94\uc9c4\uc744 \uc704\ud574 \uae30\uc6b8\uc778 \uacf5\ud5cc\uc744 \uae30\uc5b5\ud558\uaca0\ub2e4\ub294 \uc18c\ud68c\ub97c \ub367\ubd99\uc600\ub2e4. \ubbf8\uad6d\uc758 \ubc84\ub77d \uc624\ubc14\ub9c8 \ub300\ud1b5\ub839\uc740 \uae34\uae09 \uc560\ub3c4 \uc131\uba85\uc744 \ubc1c\ud45c\ud558\uc600\ub2e4. \uc774 \uc131\uba85\uc5d0\uc11c \ub178\ubb34\ud604 \uc7ac\uc784 \uae30\uac04\uc5d0 \ud55c\uad6d\uacfc \ubbf8\uad6d \uac04\uc758 '\uac15\ub825\ud558\uace0 \ud65c\uae30\ucc2c'(strong and vital) \uad00\uacc4\ub97c \ud615\uc131\ud558\ub294 \ub370 \uae30\uc5ec\ud588\ub2e4\uba70 \uc560\ub3c4\uc758 \ub73b\uc744 \uc804\ud588\ub2e4. \uc544\uc18c \ub2e4\ub85c \uc77c\ubcf8 \ucd1d\ub9ac\ub294 \uc678\uc0c1 \uc2dc\uc808 \ub178 \uc804 \ub300\ud1b5\ub839\uc744 \ub9cc\ub0ac\ub358 \uae30\uc5b5\uc744 \ub5a0\uc62c\ub9ac\uba70 \uc560\ub3c4\ud588\ub2e4. \uc601\uad6d\uc758 \uc5d8\ub9ac\uc790\ubca0\uc2a4 2\uc138 \uc5ec\uc655\ub3c4 \uccad\uc640\ub300\uc5d0 \uc560\ub3c4 \uc870\ubb38\uc744 \ubcf4\ub0b4\uc5b4, \"\uc9c0\ub09c 2004\ub144 \ub178 \uc804 \ub300\ud1b5\ub839\uc758 \uc601\uad6d \uacf5\uc2dd \ubc29\ubb38\uc740 \ud55c\u00b7\uc601 \uc591\uad6d \uad00\uacc4 \uc99d\uc9c4\uc5d0 \uc911\uc694\ud55c \uc774\uc815\ud45c\uc600\ub2e4\"\ub77c\uace0 \uc804\ud588\ub2e4. \ubc18\uae30\ubb38 \uad6d\uc81c \uc5f0\ud569 \uc0ac\ubb34\ucd1d\uc7a5\uc740 \uc0ac\ub9dd \uad00\ub828 \uc131\uba85\uc744 \ubc1c\ud45c\ud558\uace0 \uc560\ub3c4\uc758 \ub73b\uc744 \ud45c\uba85\ud558\uba74\uc11c, \"\ub178 \uc804 \ub300\ud1b5\ub839\uc740 \ubbfc\uc8fc\uc8fc\uc758 \ucd09\uc9c4\uc744 \uc704\ud574 \uc5ec\ub7ec \uac00\uc9c0 \ub178\ub825\uc744 \uae30\uc6b8\uc600\ub2e4\"\ub77c\uace0 \ub178 \uc804 \ub300\ud1b5\ub839\uc744 \uce6d\uc1a1\ud588\ub2e4. \uace0\ub4e0 \ube0c\ub77c\uc6b4 \uc601\uad6d \ucd1d\ub9ac \uc5ed\uc2dc \uc560\ub3c4\uc758 \ub73b\uc744 \uc804\ud588\ub2e4."} {"question": "526\ub144\uc5d0 \uc9c4\uacbd\uc9c0\uac00 \ubc29\uc5b4\ub300\uc2e0 \uc801\uadf9\uc801\uc778 \uacf5\uc138\ub85c 2\ub2ec \ub9cc\uc5d0 \uc810\ub839\ud55c \uc131\uc740 \uba87 \uac1c\uc778\uac00?", "paragraph": "526\ub144, \ubd81\uc704\uc5d0\uc11c \uc2b9\uc138\ub97c \ubab0\uc544 11\ub9cc 5\ucc9c \uba85\uc758 \uad70\ub300\ub97c \ud30c\uacac\ud574 \uc591\uc744 \uacf5\uaca9\ud55c\ub2e4. \uc804\uc120\uc740 \uc21c\uc2dd\uac04\uc5d0 \ubb34\ub108\uc84c\uace0 \uad6d\uacbd\uc758 \ucc45\uc784\uc790\ub9c8\uc800 \ubd81\uc704\uc5d0\uac8c \ud56d\ubcf5\ud588\uae30\uc5d0 \ub2e8\uc228\uc5d0\ub77c\ub3c4 \uc801\uad70\uc774 \uc218\ub3c4\uc5d0\uae4c\uc9c0 \ubc00\uace0 \ub4e4\uc5b4\uc62c \uc218 \uc788\ub294 \uc0c1\ud669\uc774\uc5c8\ub2e4. \ub300\uc7a5\uad70\uc774 \ub41c \uc9c4\uacbd\uc9c0\ub294 \ubc29\uc5b4 \ub300\uc2e0 \uc801\uadf9\uc801\uc778 \uacf5\uc138\ub85c \ub098\uc11c \uc801\uc758 \uae30\uc810 \uc5ed\ud560\uc744 \ud558\ub358 \uc218\ucd98\uc73c\ub85c \uc9c4\uc785\ud574 2\ub2ec \ub9cc\uc5d0 53\uac1c\uc758 \uc131\uc744 \uc810\ub839\ud558\uace0 9\ub9cc \uba85\uc758 \uc801\uc744 \uc8fd\uc600\ub2e4. \uc55e\ub4a4\ub85c \ud3ec\uc704\ub420 \uac83\uc744 \ub450\ub824\uc6cc\ud574 \uc218\ucd98\uc73c\ub85c \uacf5\uaca9\ud574 \uc628 \ubd81\uc704\uc758 \uc8fc\ub825\uad70\uc740 \uc640\uc591 \ud3c9\uc6d0\uc5d0\uc11c \uc9c4\uacbd\uc9c0\uc640 \ub9c8\uc8fc\ubcf4\uace0 \ud3ec\uc9c4\ud558\uc600\ub2e4. \ubd80\uc0ac\ub839\uad00\uc774\ub358 \uc704\ubc29\uc740 \"\uc9c0\uae08\uc740 \uc218\ube44\ub97c \uad73\uac74\ud788 \ud558\uba74\uc11c \uc801\uc774 \uc9c0\uce58\uae38 \uae30\ub2e4\ub824\uc57c \ud558\uba70, \uc591\uc758 \uc218\ub3c4\uc5d0\uc11c \uc6d0\uad70\uc774 \uc624\uba74 \uc801\uc744 \ud3ec\uc704\ud574\uc11c \ubb34\ucc0c\ub97c \uc218 \uc788\ub2e4\" \uace0 \uc8fc\uc7a5\ud588\ub2e4. \ud558\uc9c0\ub9cc \uc9c4\uacbd\uc9c0\ub294 \uccab\uc9f8, \uc591\uc758 \ubcf8\uad6d\uc5d0 \uc788\ub294 \uc804\ub825\uc740 \ubbff\uc744 \uc218 \uc5c6\uc73c\uba70, \ubd81\uc704\uad70\uc5d0\uac8c \uac01\uac1c\uaca9\ud30c\ub2f9\ud560 \uc704\ud5d8\uc774 \ud06c\ub2e4. \ub458\uc9f8, \uc9c0\uae08 \uc810\ub839\ud55c \uc218\ucd98\uc740 \ubbff\uc744 \uc218 \uc5c6\uace0 \uc6b0\ub9ac \uad70\ub300\ub294 \uc801\uc73c\ub2c8 \uc5b8\uc81c \uc801\uc744 \ub3c4\uc640 \ubc30\ubc18\ud560\uc9c0 \ubaa8\ub978\ub2e4. \uadf8\ub807\uac8c \ub418\uba74 \uc544\uad70\uc740 \ub3c5\uc548\uc5d0 \ub4e0 \uc950 \uc2e0\uc138\uac00 \ub420 \uac83\u2018 \uc774\ub77c\uba70 \uc801\uadf9\uc801\uc778 \uacf5\uc138\ub97c \ud3bc\ucce4\ub2e4. \uc801\uc758 \uc218\uac00 \ub9ce\uc740 \uac83\uc744 \ubcf4\uace0 \ubcd1\uc0ac\ub4e4\uc774 \ub450\ub824\uc6cc\ud558\uba70 \ub098\uc544\uac00\uc9c0 \uc54a\uc790 \uc9c4\uacbd\uc9c0\ub294 \uc9c1\uc811 \ubcd1\uc0ac\ub4e4\uc744 \ud798\uc788\uac8c \ub3c5\ub824\ud558\uba70 \u2019\uc6b0\ub9ac\ub294 \uc774\uc81c\uaecf \uc2b9\ub9ac\ub97c \uac70\ub4ed\ud588\ub2e4. \uc774\ubc88 \uc804\ud22c\ub294 \uc6b0\ub9ac\uc758 \uba85\uc608\ub97c \uc601\uc6d0\ud55c \uac83\uc73c\ub85c \ub9cc\ub4dc\ub294 \ub9c8\uc9c0\ub9c9 \uc804\ud22c\uac00 \ub420 \uac83\uc774\ub2e4\u2018\ub77c\uba70 \uc2a4\uc2a4\ub85c \uc120\ub450\uc5d0 \uc11c\uc11c \uc804\uc7c1\uc744 \uc9c0\ud718\ud588\ub2e4. \uc804\ud22c\ub294 \uc9c4\uacbd\uc9c0\uc758 \uc2b9\ub9ac\ub85c \ub05d\ub0ac\uace0, \uc591\uc740 \uc591\uc790\uac15 \uc774\ubd81\uc5d0 \uc601\ud1a0\ub97c \uac00\uc9c0\uba74\uc11c \uacbd\uc81c\uc801, \uadf8\ub9ac\uace0 \uad70\uc0ac\uc801\uc73c\ub85c \ub300\ub2e8\ud55c \uc6b0\uc704\uc5d0 \uc124 \uc218 \uc788\uac8c \ub41c\ub2e4.", "answer": "53\uac1c", "sentence": "\ub300\uc7a5\uad70\uc774 \ub41c \uc9c4\uacbd\uc9c0\ub294 \ubc29\uc5b4 \ub300\uc2e0 \uc801\uadf9\uc801\uc778 \uacf5\uc138\ub85c \ub098\uc11c \uc801\uc758 \uae30\uc810 \uc5ed\ud560\uc744 \ud558\ub358 \uc218\ucd98\uc73c\ub85c \uc9c4\uc785\ud574 2\ub2ec \ub9cc\uc5d0 53\uac1c \uc758 \uc131\uc744 \uc810\ub839\ud558\uace0 9\ub9cc \uba85\uc758 \uc801\uc744 \uc8fd\uc600\ub2e4.", "paragraph_sentence": "526\ub144, \ubd81\uc704\uc5d0\uc11c \uc2b9\uc138\ub97c \ubab0\uc544 11\ub9cc 5\ucc9c \uba85\uc758 \uad70\ub300\ub97c \ud30c\uacac\ud574 \uc591\uc744 \uacf5\uaca9\ud55c\ub2e4. \uc804\uc120\uc740 \uc21c\uc2dd\uac04\uc5d0 \ubb34\ub108\uc84c\uace0 \uad6d\uacbd\uc758 \ucc45\uc784\uc790\ub9c8\uc800 \ubd81\uc704\uc5d0\uac8c \ud56d\ubcf5\ud588\uae30\uc5d0 \ub2e8\uc228\uc5d0\ub77c\ub3c4 \uc801\uad70\uc774 \uc218\ub3c4\uc5d0\uae4c\uc9c0 \ubc00\uace0 \ub4e4\uc5b4\uc62c \uc218 \uc788\ub294 \uc0c1\ud669\uc774\uc5c8\ub2e4. \ub300\uc7a5\uad70\uc774 \ub41c \uc9c4\uacbd\uc9c0\ub294 \ubc29\uc5b4 \ub300\uc2e0 \uc801\uadf9\uc801\uc778 \uacf5\uc138\ub85c \ub098\uc11c \uc801\uc758 \uae30\uc810 \uc5ed\ud560\uc744 \ud558\ub358 \uc218\ucd98\uc73c\ub85c \uc9c4\uc785\ud574 2\ub2ec \ub9cc\uc5d0 53\uac1c \uc758 \uc131\uc744 \uc810\ub839\ud558\uace0 9\ub9cc \uba85\uc758 \uc801\uc744 \uc8fd\uc600\ub2e4. \uc55e\ub4a4\ub85c \ud3ec\uc704\ub420 \uac83\uc744 \ub450\ub824\uc6cc\ud574 \uc218\ucd98\uc73c\ub85c \uacf5\uaca9\ud574 \uc628 \ubd81\uc704\uc758 \uc8fc\ub825\uad70\uc740 \uc640\uc591 \ud3c9\uc6d0\uc5d0\uc11c \uc9c4\uacbd\uc9c0\uc640 \ub9c8\uc8fc\ubcf4\uace0 \ud3ec\uc9c4\ud558\uc600\ub2e4. \ubd80\uc0ac\ub839\uad00\uc774\ub358 \uc704\ubc29\uc740 \"\uc9c0\uae08\uc740 \uc218\ube44\ub97c \uad73\uac74\ud788 \ud558\uba74\uc11c \uc801\uc774 \uc9c0\uce58\uae38 \uae30\ub2e4\ub824\uc57c \ud558\uba70, \uc591\uc758 \uc218\ub3c4\uc5d0\uc11c \uc6d0\uad70\uc774 \uc624\uba74 \uc801\uc744 \ud3ec\uc704\ud574\uc11c \ubb34\ucc0c\ub97c \uc218 \uc788\ub2e4\" \uace0 \uc8fc\uc7a5\ud588\ub2e4. \ud558\uc9c0\ub9cc \uc9c4\uacbd\uc9c0\ub294 \uccab\uc9f8, \uc591\uc758 \ubcf8\uad6d\uc5d0 \uc788\ub294 \uc804\ub825\uc740 \ubbff\uc744 \uc218 \uc5c6\uc73c\uba70, \ubd81\uc704\uad70\uc5d0\uac8c \uac01\uac1c\uaca9\ud30c\ub2f9\ud560 \uc704\ud5d8\uc774 \ud06c\ub2e4. \ub458\uc9f8, \uc9c0\uae08 \uc810\ub839\ud55c \uc218\ucd98\uc740 \ubbff\uc744 \uc218 \uc5c6\uace0 \uc6b0\ub9ac \uad70\ub300\ub294 \uc801\uc73c\ub2c8 \uc5b8\uc81c \uc801\uc744 \ub3c4\uc640 \ubc30\ubc18\ud560\uc9c0 \ubaa8\ub978\ub2e4. \uadf8\ub807\uac8c \ub418\uba74 \uc544\uad70\uc740 \ub3c5\uc548\uc5d0 \ub4e0 \uc950 \uc2e0\uc138\uac00 \ub420 \uac83\u2018 \uc774\ub77c\uba70 \uc801\uadf9\uc801\uc778 \uacf5\uc138\ub97c \ud3bc\ucce4\ub2e4. \uc801\uc758 \uc218\uac00 \ub9ce\uc740 \uac83\uc744 \ubcf4\uace0 \ubcd1\uc0ac\ub4e4\uc774 \ub450\ub824\uc6cc\ud558\uba70 \ub098\uc544\uac00\uc9c0 \uc54a\uc790 \uc9c4\uacbd\uc9c0\ub294 \uc9c1\uc811 \ubcd1\uc0ac\ub4e4\uc744 \ud798\uc788\uac8c \ub3c5\ub824\ud558\uba70 \u2019\uc6b0\ub9ac\ub294 \uc774\uc81c\uaecf \uc2b9\ub9ac\ub97c \uac70\ub4ed\ud588\ub2e4. \uc774\ubc88 \uc804\ud22c\ub294 \uc6b0\ub9ac\uc758 \uba85\uc608\ub97c \uc601\uc6d0\ud55c \uac83\uc73c\ub85c \ub9cc\ub4dc\ub294 \ub9c8\uc9c0\ub9c9 \uc804\ud22c\uac00 \ub420 \uac83\uc774\ub2e4\u2018\ub77c\uba70 \uc2a4\uc2a4\ub85c \uc120\ub450\uc5d0 \uc11c\uc11c \uc804\uc7c1\uc744 \uc9c0\ud718\ud588\ub2e4. \uc804\ud22c\ub294 \uc9c4\uacbd\uc9c0\uc758 \uc2b9\ub9ac\ub85c \ub05d\ub0ac\uace0, \uc591\uc740 \uc591\uc790\uac15 \uc774\ubd81\uc5d0 \uc601\ud1a0\ub97c \uac00\uc9c0\uba74\uc11c \uacbd\uc81c\uc801, \uadf8\ub9ac\uace0 \uad70\uc0ac\uc801\uc73c\ub85c \ub300\ub2e8\ud55c \uc6b0\uc704\uc5d0 \uc124 \uc218 \uc788\uac8c \ub41c\ub2e4.", "paragraph_answer": "526\ub144, \ubd81\uc704\uc5d0\uc11c \uc2b9\uc138\ub97c \ubab0\uc544 11\ub9cc 5\ucc9c \uba85\uc758 \uad70\ub300\ub97c \ud30c\uacac\ud574 \uc591\uc744 \uacf5\uaca9\ud55c\ub2e4. \uc804\uc120\uc740 \uc21c\uc2dd\uac04\uc5d0 \ubb34\ub108\uc84c\uace0 \uad6d\uacbd\uc758 \ucc45\uc784\uc790\ub9c8\uc800 \ubd81\uc704\uc5d0\uac8c \ud56d\ubcf5\ud588\uae30\uc5d0 \ub2e8\uc228\uc5d0\ub77c\ub3c4 \uc801\uad70\uc774 \uc218\ub3c4\uc5d0\uae4c\uc9c0 \ubc00\uace0 \ub4e4\uc5b4\uc62c \uc218 \uc788\ub294 \uc0c1\ud669\uc774\uc5c8\ub2e4. \ub300\uc7a5\uad70\uc774 \ub41c \uc9c4\uacbd\uc9c0\ub294 \ubc29\uc5b4 \ub300\uc2e0 \uc801\uadf9\uc801\uc778 \uacf5\uc138\ub85c \ub098\uc11c \uc801\uc758 \uae30\uc810 \uc5ed\ud560\uc744 \ud558\ub358 \uc218\ucd98\uc73c\ub85c \uc9c4\uc785\ud574 2\ub2ec \ub9cc\uc5d0 53\uac1c \uc758 \uc131\uc744 \uc810\ub839\ud558\uace0 9\ub9cc \uba85\uc758 \uc801\uc744 \uc8fd\uc600\ub2e4. \uc55e\ub4a4\ub85c \ud3ec\uc704\ub420 \uac83\uc744 \ub450\ub824\uc6cc\ud574 \uc218\ucd98\uc73c\ub85c \uacf5\uaca9\ud574 \uc628 \ubd81\uc704\uc758 \uc8fc\ub825\uad70\uc740 \uc640\uc591 \ud3c9\uc6d0\uc5d0\uc11c \uc9c4\uacbd\uc9c0\uc640 \ub9c8\uc8fc\ubcf4\uace0 \ud3ec\uc9c4\ud558\uc600\ub2e4. \ubd80\uc0ac\ub839\uad00\uc774\ub358 \uc704\ubc29\uc740 \"\uc9c0\uae08\uc740 \uc218\ube44\ub97c \uad73\uac74\ud788 \ud558\uba74\uc11c \uc801\uc774 \uc9c0\uce58\uae38 \uae30\ub2e4\ub824\uc57c \ud558\uba70, \uc591\uc758 \uc218\ub3c4\uc5d0\uc11c \uc6d0\uad70\uc774 \uc624\uba74 \uc801\uc744 \ud3ec\uc704\ud574\uc11c \ubb34\ucc0c\ub97c \uc218 \uc788\ub2e4\" \uace0 \uc8fc\uc7a5\ud588\ub2e4. \ud558\uc9c0\ub9cc \uc9c4\uacbd\uc9c0\ub294 \uccab\uc9f8, \uc591\uc758 \ubcf8\uad6d\uc5d0 \uc788\ub294 \uc804\ub825\uc740 \ubbff\uc744 \uc218 \uc5c6\uc73c\uba70, \ubd81\uc704\uad70\uc5d0\uac8c \uac01\uac1c\uaca9\ud30c\ub2f9\ud560 \uc704\ud5d8\uc774 \ud06c\ub2e4. \ub458\uc9f8, \uc9c0\uae08 \uc810\ub839\ud55c \uc218\ucd98\uc740 \ubbff\uc744 \uc218 \uc5c6\uace0 \uc6b0\ub9ac \uad70\ub300\ub294 \uc801\uc73c\ub2c8 \uc5b8\uc81c \uc801\uc744 \ub3c4\uc640 \ubc30\ubc18\ud560\uc9c0 \ubaa8\ub978\ub2e4. \uadf8\ub807\uac8c \ub418\uba74 \uc544\uad70\uc740 \ub3c5\uc548\uc5d0 \ub4e0 \uc950 \uc2e0\uc138\uac00 \ub420 \uac83\u2018 \uc774\ub77c\uba70 \uc801\uadf9\uc801\uc778 \uacf5\uc138\ub97c \ud3bc\ucce4\ub2e4. \uc801\uc758 \uc218\uac00 \ub9ce\uc740 \uac83\uc744 \ubcf4\uace0 \ubcd1\uc0ac\ub4e4\uc774 \ub450\ub824\uc6cc\ud558\uba70 \ub098\uc544\uac00\uc9c0 \uc54a\uc790 \uc9c4\uacbd\uc9c0\ub294 \uc9c1\uc811 \ubcd1\uc0ac\ub4e4\uc744 \ud798\uc788\uac8c \ub3c5\ub824\ud558\uba70 \u2019\uc6b0\ub9ac\ub294 \uc774\uc81c\uaecf \uc2b9\ub9ac\ub97c \uac70\ub4ed\ud588\ub2e4. \uc774\ubc88 \uc804\ud22c\ub294 \uc6b0\ub9ac\uc758 \uba85\uc608\ub97c \uc601\uc6d0\ud55c \uac83\uc73c\ub85c \ub9cc\ub4dc\ub294 \ub9c8\uc9c0\ub9c9 \uc804\ud22c\uac00 \ub420 \uac83\uc774\ub2e4\u2018\ub77c\uba70 \uc2a4\uc2a4\ub85c \uc120\ub450\uc5d0 \uc11c\uc11c \uc804\uc7c1\uc744 \uc9c0\ud718\ud588\ub2e4. \uc804\ud22c\ub294 \uc9c4\uacbd\uc9c0\uc758 \uc2b9\ub9ac\ub85c \ub05d\ub0ac\uace0, \uc591\uc740 \uc591\uc790\uac15 \uc774\ubd81\uc5d0 \uc601\ud1a0\ub97c \uac00\uc9c0\uba74\uc11c \uacbd\uc81c\uc801, \uadf8\ub9ac\uace0 \uad70\uc0ac\uc801\uc73c\ub85c \ub300\ub2e8\ud55c \uc6b0\uc704\uc5d0 \uc124 \uc218 \uc788\uac8c \ub41c\ub2e4.", "sentence_answer": "\ub300\uc7a5\uad70\uc774 \ub41c \uc9c4\uacbd\uc9c0\ub294 \ubc29\uc5b4 \ub300\uc2e0 \uc801\uadf9\uc801\uc778 \uacf5\uc138\ub85c \ub098\uc11c \uc801\uc758 \uae30\uc810 \uc5ed\ud560\uc744 \ud558\ub358 \uc218\ucd98\uc73c\ub85c \uc9c4\uc785\ud574 2\ub2ec \ub9cc\uc5d0 53\uac1c \uc758 \uc131\uc744 \uc810\ub839\ud558\uace0 9\ub9cc \uba85\uc758 \uc801\uc744 \uc8fd\uc600\ub2e4.", "paragraph_id": "6557350-1-1", "paragraph_question": "question: 526\ub144\uc5d0 \uc9c4\uacbd\uc9c0\uac00 \ubc29\uc5b4\ub300\uc2e0 \uc801\uadf9\uc801\uc778 \uacf5\uc138\ub85c 2\ub2ec \ub9cc\uc5d0 \uc810\ub839\ud55c \uc131\uc740 \uba87 \uac1c\uc778\uac00?, context: 526\ub144, \ubd81\uc704\uc5d0\uc11c \uc2b9\uc138\ub97c \ubab0\uc544 11\ub9cc 5\ucc9c \uba85\uc758 \uad70\ub300\ub97c \ud30c\uacac\ud574 \uc591\uc744 \uacf5\uaca9\ud55c\ub2e4. \uc804\uc120\uc740 \uc21c\uc2dd\uac04\uc5d0 \ubb34\ub108\uc84c\uace0 \uad6d\uacbd\uc758 \ucc45\uc784\uc790\ub9c8\uc800 \ubd81\uc704\uc5d0\uac8c \ud56d\ubcf5\ud588\uae30\uc5d0 \ub2e8\uc228\uc5d0\ub77c\ub3c4 \uc801\uad70\uc774 \uc218\ub3c4\uc5d0\uae4c\uc9c0 \ubc00\uace0 \ub4e4\uc5b4\uc62c \uc218 \uc788\ub294 \uc0c1\ud669\uc774\uc5c8\ub2e4. \ub300\uc7a5\uad70\uc774 \ub41c \uc9c4\uacbd\uc9c0\ub294 \ubc29\uc5b4 \ub300\uc2e0 \uc801\uadf9\uc801\uc778 \uacf5\uc138\ub85c \ub098\uc11c \uc801\uc758 \uae30\uc810 \uc5ed\ud560\uc744 \ud558\ub358 \uc218\ucd98\uc73c\ub85c \uc9c4\uc785\ud574 2\ub2ec \ub9cc\uc5d0 53\uac1c\uc758 \uc131\uc744 \uc810\ub839\ud558\uace0 9\ub9cc \uba85\uc758 \uc801\uc744 \uc8fd\uc600\ub2e4. \uc55e\ub4a4\ub85c \ud3ec\uc704\ub420 \uac83\uc744 \ub450\ub824\uc6cc\ud574 \uc218\ucd98\uc73c\ub85c \uacf5\uaca9\ud574 \uc628 \ubd81\uc704\uc758 \uc8fc\ub825\uad70\uc740 \uc640\uc591 \ud3c9\uc6d0\uc5d0\uc11c \uc9c4\uacbd\uc9c0\uc640 \ub9c8\uc8fc\ubcf4\uace0 \ud3ec\uc9c4\ud558\uc600\ub2e4. \ubd80\uc0ac\ub839\uad00\uc774\ub358 \uc704\ubc29\uc740 \"\uc9c0\uae08\uc740 \uc218\ube44\ub97c \uad73\uac74\ud788 \ud558\uba74\uc11c \uc801\uc774 \uc9c0\uce58\uae38 \uae30\ub2e4\ub824\uc57c \ud558\uba70, \uc591\uc758 \uc218\ub3c4\uc5d0\uc11c \uc6d0\uad70\uc774 \uc624\uba74 \uc801\uc744 \ud3ec\uc704\ud574\uc11c \ubb34\ucc0c\ub97c \uc218 \uc788\ub2e4\" \uace0 \uc8fc\uc7a5\ud588\ub2e4. \ud558\uc9c0\ub9cc \uc9c4\uacbd\uc9c0\ub294 \uccab\uc9f8, \uc591\uc758 \ubcf8\uad6d\uc5d0 \uc788\ub294 \uc804\ub825\uc740 \ubbff\uc744 \uc218 \uc5c6\uc73c\uba70, \ubd81\uc704\uad70\uc5d0\uac8c \uac01\uac1c\uaca9\ud30c\ub2f9\ud560 \uc704\ud5d8\uc774 \ud06c\ub2e4. \ub458\uc9f8, \uc9c0\uae08 \uc810\ub839\ud55c \uc218\ucd98\uc740 \ubbff\uc744 \uc218 \uc5c6\uace0 \uc6b0\ub9ac \uad70\ub300\ub294 \uc801\uc73c\ub2c8 \uc5b8\uc81c \uc801\uc744 \ub3c4\uc640 \ubc30\ubc18\ud560\uc9c0 \ubaa8\ub978\ub2e4. \uadf8\ub807\uac8c \ub418\uba74 \uc544\uad70\uc740 \ub3c5\uc548\uc5d0 \ub4e0 \uc950 \uc2e0\uc138\uac00 \ub420 \uac83\u2018 \uc774\ub77c\uba70 \uc801\uadf9\uc801\uc778 \uacf5\uc138\ub97c \ud3bc\ucce4\ub2e4. \uc801\uc758 \uc218\uac00 \ub9ce\uc740 \uac83\uc744 \ubcf4\uace0 \ubcd1\uc0ac\ub4e4\uc774 \ub450\ub824\uc6cc\ud558\uba70 \ub098\uc544\uac00\uc9c0 \uc54a\uc790 \uc9c4\uacbd\uc9c0\ub294 \uc9c1\uc811 \ubcd1\uc0ac\ub4e4\uc744 \ud798\uc788\uac8c \ub3c5\ub824\ud558\uba70 \u2019\uc6b0\ub9ac\ub294 \uc774\uc81c\uaecf \uc2b9\ub9ac\ub97c \uac70\ub4ed\ud588\ub2e4. \uc774\ubc88 \uc804\ud22c\ub294 \uc6b0\ub9ac\uc758 \uba85\uc608\ub97c \uc601\uc6d0\ud55c \uac83\uc73c\ub85c \ub9cc\ub4dc\ub294 \ub9c8\uc9c0\ub9c9 \uc804\ud22c\uac00 \ub420 \uac83\uc774\ub2e4\u2018\ub77c\uba70 \uc2a4\uc2a4\ub85c \uc120\ub450\uc5d0 \uc11c\uc11c \uc804\uc7c1\uc744 \uc9c0\ud718\ud588\ub2e4. \uc804\ud22c\ub294 \uc9c4\uacbd\uc9c0\uc758 \uc2b9\ub9ac\ub85c \ub05d\ub0ac\uace0, \uc591\uc740 \uc591\uc790\uac15 \uc774\ubd81\uc5d0 \uc601\ud1a0\ub97c \uac00\uc9c0\uba74\uc11c \uacbd\uc81c\uc801, \uadf8\ub9ac\uace0 \uad70\uc0ac\uc801\uc73c\ub85c \ub300\ub2e8\ud55c \uc6b0\uc704\uc5d0 \uc124 \uc218 \uc788\uac8c \ub41c\ub2e4."} {"question": "4\uc6d428\uc77c \uc5f0\ud569\uad70 53\uc0ac\ub2e8\uc774 \uc232\uc5d0 \ub0a8\uc544 \ub3c5\uc77c\uad70\uc744 \uacf5\uaca9\ud558\uc5ec \uc7a1\uc740 \ud3ec\ub85c\uc758 \uc218\ub294 \uc5bc\ub9c8\ub098 \ub418\ub294\uac00?", "paragraph": "4\uc6d4 28\uc77c, \ub3c5\uc77c\uad70\uc740 \ub3c4\uc2dc \ub0b4\ub85c \uacf5\uc138\ub97c \ud3bc\uce58\uae30 \uc2dc\uc791\ud588\ub2e4. 5 \uc601\uad6d \uc804\ucc28 \uc5f0\ub300, 9 \ub354\ub7fc \uacbd\ubcf4\ubcd1 \uc5f0\ub300\uc640 1 \uc18c\ucd1d\ubcd1 \uc5ec\ub2e8\uc774 \uc544\uc6b0\ud1a0\ubc18\uc774 \uc788\ub294 \uc81c\uc2a4\ud14c\ubd80\ub974\ud06c\uc640 \ud788\ud2b8\ubca8\ub4dc\ub97c \uc810\ub839\ud588\ub2e4. \uadf8\ub7fc\uc5d0\ub3c4 \ubd88\uad6c\ud558\uace0, \ub3c5\uc77c\uad70\uc774 \uc544\uc6b0\ud1a0\ubc18\uc758 \uc77c\ubd80\ub864 \ud3ed\ud30c\uc2dc\ud0a4\uba74\uc11c \uc601\uad6d\uad70\uc758 \uc9c4\uaca9\uc774 \ub454\ud654\ub418\uc5c8\ub2e4. \uc77c\ub2e8 \uc601\uad6d\uad70\uc774 \ud558\ub974\ubd80\ub974\ud06c\ub85c \ub4e4\uc5b4\uac00\uc790 \ub3c5\uc77c\uad70\uc740 \uce58\uc5f4\ud558\uac8c \uc800\ud56d\ud588\uace0 \ub3c5\uc77c\uad70\uc774 \ud56d\ubcf5\ud558\uc9c0 \uc54a\uc544 \uac74\ubb3c \ud558\ub098\ud558\ub098\ub97c \uc810\ub839\ud574\uc57c \ud588\ub2e4. \uc774 \ub54c \ub3c5\uc77c 1 \uacf5\uc218\uad70\uc740 SS, \ub099\ud558\uc0b0\ubcd1, \uad6d\ubbfc\ub3cc\uaca9\ub300, \uc815\uaddc \ub3c5\uc77c \uad6d\ubc29\uad70 \uad70\uc778, \uc120\uc6d0, \uacbd\ucc30, \uc18c\ubc29\uad00, \ud788\ud2c0\ub7ec \uc720\uac90\ud2b8 \ub4f1\uc774 \uc11e\uc5ec\uc788\uc5c8\ub2e4. \ub3c4\uc2dc \uc911\uc2ec\uc73c\ub85c \uc9c4\uaca9\ud560\uc218\ub85d \ub3c5\uc77c\uad70\uc758 \uc800\ud56d\uc740 \uc810\uc810 \uc2ec\ud574\uc84c\ub2e4. \uc774\ub4e4\uc740 \ud568\ubd80\ub974\ud06c\uae4c\uc9c0 \uac00\ub294 \uc544\uc6b0\ud1a0\ubc18\uc5d0\uc11c \ud544\uc0ac\uc801\uc73c\ub85c \uc2f8\uc6b0\uace0 \uc788\uc5c8\ub2e4. \uadf8\ub0a0 \uc800\ub141, \uccab \ubc88\uc9f8 \uc601\uad6d\uad70\uc774 \uc5d8\ubca0 \uac15\uc744 \uac74\ub108 \ud568\ubd80\ub974\ud06c\ub85c \ub4e4\uc5b4\uac14\ub2e4. 88mm \ud3ec\ub294 \ub3c4\uc2dc \uacf3\uacf3\uc5d0 \ubd84\uc0b0\ub418\uc5b4 \uc788\uc5c8\uc9c0\ub9cc \uc804\ud22c\uac00 \uc9c4\ud589\ub428\uc5d0 \ub530\ub77c \ud6a8\uacfc\uac00 \ub098\ud0c0\ub0ac\ub2e4. \ub300\uc804\ucc28 \ub300\ub300\ub97c \ud3ec\ud568\ud55c \ub9ce\uc740 \ub3c5\uc77c\uad70, \ud5dd\uac00\ub9ac SS \ubd80\ub300, \"\ud310\uc800\ud30c\uc6b0\uc2a4\ud2b8\" \ubd80\ub300\uac00 \uc5ec\uc804\ud788 \ud558\ub974\ubd80\ub974\ud06c \ub0a8\ucabd \uc232\uc5d0 \ub0a8\uc544\uc788\uc5c8\uae30 \ub54c\ubb38\uc5d0 \uc601\uad6d\uad70\uc740 \uc774 \uc9c0\uc5ed\uc744 \uc6b0\ud68c\ud588\uace0, \ub098\uc911\uc5d0 \uc815\ub9ac\ub418\uc5c8\ub2e4. 1 \uc601\uad6d \uc804\ucc28 \ub300\ub300\uc758 \uc9c0\uc6d0\uc744 \ubc1b\uc740 53 \uc0ac\ub2e8\uc774 \uc232\uc5d0 \ub0a8\uc544 \uc788\ub358 \ub3c5\uc77c\uad70\uc744 \uacf5\uaca9\ud558\uc5ec 2,000\uba85\uc744 \ud3ec\ub85c\ub85c \uc7a1\uc558\ub2e4. \uc804\ud22c\uac00 \uacc4\uc18d\ub418\ub294 \ub3d9\uc548, \ub3c5\uc77c\uad70\uc758 \ubc29\uc5b4\uc120\uc740 \ubb34\ub108\uc9c0\uae30 \uc2dc\uc791\ud588\ub2e4. 4\uc6d4 30\uc77c, \ud788\ud2c0\ub7ec\uac00 \ubca0\ub97c\ub9b0\uc5d0\uc11c \uc790\uc0b4\ud558\uace0 \ubd81\ucabd\uc5d0\uc11c \uad70\ub300\ub97c \uc9c0\ud718\ud558\ub358 \ub300\uc81c\ub3c5 \uce74\ub97c \ub418\ub2c8\uce20\ub294 \uc5d0\ub974\uc708 \ubcfc\uce20\uc5d0\uac8c \uc601\uad6d\uad70\uc5d0\uac8c \ub3c4\uc2dc\ub97c \ud3ec\uae30\ud558\ub77c\uace0 \ub9d0\ud588\ub2e4. \ubcfc\uce20\ub294 \ub3c5\uc77c \ub300\ud45c\ub2e8\uacfc \ud568\uaed8 5\uc6d4 2\uc77c \uc0ac\ub2e8 \ubcf8\ubd80\uc5d0 \ub3c4\ucc29\ud588\uc73c\uba70 \uacf5\uc2dd\uc801\uc73c\ub85c 5\uc6d4 3\uc77c \ud568\ubd80\ub974\ud06c\uac00 \ud56d\ubcf5\ud588\ub2e4. \uac19\uc740 \ub0a0 7 \uc0ac\ub2e8\uc774 \ud3d0\ud5c8\uac00 \ub41c \ub3c4\uc2dc\ub85c \uc9c4\uc785\ud588\ub2e4.", "answer": "2,000\uba85", "sentence": "1 \uc601\uad6d \uc804\ucc28 \ub300\ub300\uc758 \uc9c0\uc6d0\uc744 \ubc1b\uc740 53 \uc0ac\ub2e8\uc774 \uc232\uc5d0 \ub0a8\uc544 \uc788\ub358 \ub3c5\uc77c\uad70\uc744 \uacf5\uaca9\ud558\uc5ec 2,000\uba85 \uc744 \ud3ec\ub85c\ub85c \uc7a1\uc558\ub2e4.", "paragraph_sentence": "4\uc6d4 28\uc77c, \ub3c5\uc77c\uad70\uc740 \ub3c4\uc2dc \ub0b4\ub85c \uacf5\uc138\ub97c \ud3bc\uce58\uae30 \uc2dc\uc791\ud588\ub2e4. 5 \uc601\uad6d \uc804\ucc28 \uc5f0\ub300, 9 \ub354\ub7fc \uacbd\ubcf4\ubcd1 \uc5f0\ub300\uc640 1 \uc18c\ucd1d\ubcd1 \uc5ec\ub2e8\uc774 \uc544\uc6b0\ud1a0\ubc18\uc774 \uc788\ub294 \uc81c\uc2a4\ud14c\ubd80\ub974\ud06c\uc640 \ud788\ud2b8\ubca8\ub4dc\ub97c \uc810\ub839\ud588\ub2e4. \uadf8\ub7fc\uc5d0\ub3c4 \ubd88\uad6c\ud558\uace0, \ub3c5\uc77c\uad70\uc774 \uc544\uc6b0\ud1a0\ubc18\uc758 \uc77c\ubd80\ub864 \ud3ed\ud30c\uc2dc\ud0a4\uba74\uc11c \uc601\uad6d\uad70\uc758 \uc9c4\uaca9\uc774 \ub454\ud654\ub418\uc5c8\ub2e4. \uc77c\ub2e8 \uc601\uad6d\uad70\uc774 \ud558\ub974\ubd80\ub974\ud06c\ub85c \ub4e4\uc5b4\uac00\uc790 \ub3c5\uc77c\uad70\uc740 \uce58\uc5f4\ud558\uac8c \uc800\ud56d\ud588\uace0 \ub3c5\uc77c\uad70\uc774 \ud56d\ubcf5\ud558\uc9c0 \uc54a\uc544 \uac74\ubb3c \ud558\ub098\ud558\ub098\ub97c \uc810\ub839\ud574\uc57c \ud588\ub2e4. \uc774 \ub54c \ub3c5\uc77c 1 \uacf5\uc218\uad70\uc740 SS, \ub099\ud558\uc0b0\ubcd1, \uad6d\ubbfc\ub3cc\uaca9\ub300, \uc815\uaddc \ub3c5\uc77c \uad6d\ubc29\uad70 \uad70\uc778, \uc120\uc6d0, \uacbd\ucc30, \uc18c\ubc29\uad00, \ud788\ud2c0\ub7ec \uc720\uac90\ud2b8 \ub4f1\uc774 \uc11e\uc5ec\uc788\uc5c8\ub2e4. \ub3c4\uc2dc \uc911\uc2ec\uc73c\ub85c \uc9c4\uaca9\ud560\uc218\ub85d \ub3c5\uc77c\uad70\uc758 \uc800\ud56d\uc740 \uc810\uc810 \uc2ec\ud574\uc84c\ub2e4. \uc774\ub4e4\uc740 \ud568\ubd80\ub974\ud06c\uae4c\uc9c0 \uac00\ub294 \uc544\uc6b0\ud1a0\ubc18\uc5d0\uc11c \ud544\uc0ac\uc801\uc73c\ub85c \uc2f8\uc6b0\uace0 \uc788\uc5c8\ub2e4. \uadf8\ub0a0 \uc800\ub141, \uccab \ubc88\uc9f8 \uc601\uad6d\uad70\uc774 \uc5d8\ubca0 \uac15\uc744 \uac74\ub108 \ud568\ubd80\ub974\ud06c\ub85c \ub4e4\uc5b4\uac14\ub2e4. 88mm \ud3ec\ub294 \ub3c4\uc2dc \uacf3\uacf3\uc5d0 \ubd84\uc0b0\ub418\uc5b4 \uc788\uc5c8\uc9c0\ub9cc \uc804\ud22c\uac00 \uc9c4\ud589\ub428\uc5d0 \ub530\ub77c \ud6a8\uacfc\uac00 \ub098\ud0c0\ub0ac\ub2e4. \ub300\uc804\ucc28 \ub300\ub300\ub97c \ud3ec\ud568\ud55c \ub9ce\uc740 \ub3c5\uc77c\uad70, \ud5dd\uac00\ub9ac SS \ubd80\ub300, \"\ud310\uc800\ud30c\uc6b0\uc2a4\ud2b8\" \ubd80\ub300\uac00 \uc5ec\uc804\ud788 \ud558\ub974\ubd80\ub974\ud06c \ub0a8\ucabd \uc232\uc5d0 \ub0a8\uc544\uc788\uc5c8\uae30 \ub54c\ubb38\uc5d0 \uc601\uad6d\uad70\uc740 \uc774 \uc9c0\uc5ed\uc744 \uc6b0\ud68c\ud588\uace0, \ub098\uc911\uc5d0 \uc815\ub9ac\ub418\uc5c8\ub2e4. 1 \uc601\uad6d \uc804\ucc28 \ub300\ub300\uc758 \uc9c0\uc6d0\uc744 \ubc1b\uc740 53 \uc0ac\ub2e8\uc774 \uc232\uc5d0 \ub0a8\uc544 \uc788\ub358 \ub3c5\uc77c\uad70\uc744 \uacf5\uaca9\ud558\uc5ec 2,000\uba85 \uc744 \ud3ec\ub85c\ub85c \uc7a1\uc558\ub2e4. \uc804\ud22c\uac00 \uacc4\uc18d\ub418\ub294 \ub3d9\uc548, \ub3c5\uc77c\uad70\uc758 \ubc29\uc5b4\uc120\uc740 \ubb34\ub108\uc9c0\uae30 \uc2dc\uc791\ud588\ub2e4. 4\uc6d4 30\uc77c, \ud788\ud2c0\ub7ec\uac00 \ubca0\ub97c\ub9b0\uc5d0\uc11c \uc790\uc0b4\ud558\uace0 \ubd81\ucabd\uc5d0\uc11c \uad70\ub300\ub97c \uc9c0\ud718\ud558\ub358 \ub300\uc81c\ub3c5 \uce74\ub97c \ub418\ub2c8\uce20\ub294 \uc5d0\ub974\uc708 \ubcfc\uce20\uc5d0\uac8c \uc601\uad6d\uad70\uc5d0\uac8c \ub3c4\uc2dc\ub97c \ud3ec\uae30\ud558\ub77c\uace0 \ub9d0\ud588\ub2e4. \ubcfc\uce20\ub294 \ub3c5\uc77c \ub300\ud45c\ub2e8\uacfc \ud568\uaed8 5\uc6d4 2\uc77c \uc0ac\ub2e8 \ubcf8\ubd80\uc5d0 \ub3c4\ucc29\ud588\uc73c\uba70 \uacf5\uc2dd\uc801\uc73c\ub85c 5\uc6d4 3\uc77c \ud568\ubd80\ub974\ud06c\uac00 \ud56d\ubcf5\ud588\ub2e4. \uac19\uc740 \ub0a0 7 \uc0ac\ub2e8\uc774 \ud3d0\ud5c8\uac00 \ub41c \ub3c4\uc2dc\ub85c \uc9c4\uc785\ud588\ub2e4.", "paragraph_answer": "4\uc6d4 28\uc77c, \ub3c5\uc77c\uad70\uc740 \ub3c4\uc2dc \ub0b4\ub85c \uacf5\uc138\ub97c \ud3bc\uce58\uae30 \uc2dc\uc791\ud588\ub2e4. 5 \uc601\uad6d \uc804\ucc28 \uc5f0\ub300, 9 \ub354\ub7fc \uacbd\ubcf4\ubcd1 \uc5f0\ub300\uc640 1 \uc18c\ucd1d\ubcd1 \uc5ec\ub2e8\uc774 \uc544\uc6b0\ud1a0\ubc18\uc774 \uc788\ub294 \uc81c\uc2a4\ud14c\ubd80\ub974\ud06c\uc640 \ud788\ud2b8\ubca8\ub4dc\ub97c \uc810\ub839\ud588\ub2e4. \uadf8\ub7fc\uc5d0\ub3c4 \ubd88\uad6c\ud558\uace0, \ub3c5\uc77c\uad70\uc774 \uc544\uc6b0\ud1a0\ubc18\uc758 \uc77c\ubd80\ub864 \ud3ed\ud30c\uc2dc\ud0a4\uba74\uc11c \uc601\uad6d\uad70\uc758 \uc9c4\uaca9\uc774 \ub454\ud654\ub418\uc5c8\ub2e4. \uc77c\ub2e8 \uc601\uad6d\uad70\uc774 \ud558\ub974\ubd80\ub974\ud06c\ub85c \ub4e4\uc5b4\uac00\uc790 \ub3c5\uc77c\uad70\uc740 \uce58\uc5f4\ud558\uac8c \uc800\ud56d\ud588\uace0 \ub3c5\uc77c\uad70\uc774 \ud56d\ubcf5\ud558\uc9c0 \uc54a\uc544 \uac74\ubb3c \ud558\ub098\ud558\ub098\ub97c \uc810\ub839\ud574\uc57c \ud588\ub2e4. \uc774 \ub54c \ub3c5\uc77c 1 \uacf5\uc218\uad70\uc740 SS, \ub099\ud558\uc0b0\ubcd1, \uad6d\ubbfc\ub3cc\uaca9\ub300, \uc815\uaddc \ub3c5\uc77c \uad6d\ubc29\uad70 \uad70\uc778, \uc120\uc6d0, \uacbd\ucc30, \uc18c\ubc29\uad00, \ud788\ud2c0\ub7ec \uc720\uac90\ud2b8 \ub4f1\uc774 \uc11e\uc5ec\uc788\uc5c8\ub2e4. \ub3c4\uc2dc \uc911\uc2ec\uc73c\ub85c \uc9c4\uaca9\ud560\uc218\ub85d \ub3c5\uc77c\uad70\uc758 \uc800\ud56d\uc740 \uc810\uc810 \uc2ec\ud574\uc84c\ub2e4. \uc774\ub4e4\uc740 \ud568\ubd80\ub974\ud06c\uae4c\uc9c0 \uac00\ub294 \uc544\uc6b0\ud1a0\ubc18\uc5d0\uc11c \ud544\uc0ac\uc801\uc73c\ub85c \uc2f8\uc6b0\uace0 \uc788\uc5c8\ub2e4. \uadf8\ub0a0 \uc800\ub141, \uccab \ubc88\uc9f8 \uc601\uad6d\uad70\uc774 \uc5d8\ubca0 \uac15\uc744 \uac74\ub108 \ud568\ubd80\ub974\ud06c\ub85c \ub4e4\uc5b4\uac14\ub2e4. 88mm \ud3ec\ub294 \ub3c4\uc2dc \uacf3\uacf3\uc5d0 \ubd84\uc0b0\ub418\uc5b4 \uc788\uc5c8\uc9c0\ub9cc \uc804\ud22c\uac00 \uc9c4\ud589\ub428\uc5d0 \ub530\ub77c \ud6a8\uacfc\uac00 \ub098\ud0c0\ub0ac\ub2e4. \ub300\uc804\ucc28 \ub300\ub300\ub97c \ud3ec\ud568\ud55c \ub9ce\uc740 \ub3c5\uc77c\uad70, \ud5dd\uac00\ub9ac SS \ubd80\ub300, \"\ud310\uc800\ud30c\uc6b0\uc2a4\ud2b8\" \ubd80\ub300\uac00 \uc5ec\uc804\ud788 \ud558\ub974\ubd80\ub974\ud06c \ub0a8\ucabd \uc232\uc5d0 \ub0a8\uc544\uc788\uc5c8\uae30 \ub54c\ubb38\uc5d0 \uc601\uad6d\uad70\uc740 \uc774 \uc9c0\uc5ed\uc744 \uc6b0\ud68c\ud588\uace0, \ub098\uc911\uc5d0 \uc815\ub9ac\ub418\uc5c8\ub2e4. 1 \uc601\uad6d \uc804\ucc28 \ub300\ub300\uc758 \uc9c0\uc6d0\uc744 \ubc1b\uc740 53 \uc0ac\ub2e8\uc774 \uc232\uc5d0 \ub0a8\uc544 \uc788\ub358 \ub3c5\uc77c\uad70\uc744 \uacf5\uaca9\ud558\uc5ec 2,000\uba85 \uc744 \ud3ec\ub85c\ub85c \uc7a1\uc558\ub2e4. \uc804\ud22c\uac00 \uacc4\uc18d\ub418\ub294 \ub3d9\uc548, \ub3c5\uc77c\uad70\uc758 \ubc29\uc5b4\uc120\uc740 \ubb34\ub108\uc9c0\uae30 \uc2dc\uc791\ud588\ub2e4. 4\uc6d4 30\uc77c, \ud788\ud2c0\ub7ec\uac00 \ubca0\ub97c\ub9b0\uc5d0\uc11c \uc790\uc0b4\ud558\uace0 \ubd81\ucabd\uc5d0\uc11c \uad70\ub300\ub97c \uc9c0\ud718\ud558\ub358 \ub300\uc81c\ub3c5 \uce74\ub97c \ub418\ub2c8\uce20\ub294 \uc5d0\ub974\uc708 \ubcfc\uce20\uc5d0\uac8c \uc601\uad6d\uad70\uc5d0\uac8c \ub3c4\uc2dc\ub97c \ud3ec\uae30\ud558\ub77c\uace0 \ub9d0\ud588\ub2e4. \ubcfc\uce20\ub294 \ub3c5\uc77c \ub300\ud45c\ub2e8\uacfc \ud568\uaed8 5\uc6d4 2\uc77c \uc0ac\ub2e8 \ubcf8\ubd80\uc5d0 \ub3c4\ucc29\ud588\uc73c\uba70 \uacf5\uc2dd\uc801\uc73c\ub85c 5\uc6d4 3\uc77c \ud568\ubd80\ub974\ud06c\uac00 \ud56d\ubcf5\ud588\ub2e4. \uac19\uc740 \ub0a0 7 \uc0ac\ub2e8\uc774 \ud3d0\ud5c8\uac00 \ub41c \ub3c4\uc2dc\ub85c \uc9c4\uc785\ud588\ub2e4.", "sentence_answer": "1 \uc601\uad6d \uc804\ucc28 \ub300\ub300\uc758 \uc9c0\uc6d0\uc744 \ubc1b\uc740 53 \uc0ac\ub2e8\uc774 \uc232\uc5d0 \ub0a8\uc544 \uc788\ub358 \ub3c5\uc77c\uad70\uc744 \uacf5\uaca9\ud558\uc5ec 2,000\uba85 \uc744 \ud3ec\ub85c\ub85c \uc7a1\uc558\ub2e4.", "paragraph_id": "6466133-2-0", "paragraph_question": "question: 4\uc6d428\uc77c \uc5f0\ud569\uad70 53\uc0ac\ub2e8\uc774 \uc232\uc5d0 \ub0a8\uc544 \ub3c5\uc77c\uad70\uc744 \uacf5\uaca9\ud558\uc5ec \uc7a1\uc740 \ud3ec\ub85c\uc758 \uc218\ub294 \uc5bc\ub9c8\ub098 \ub418\ub294\uac00?, context: 4\uc6d4 28\uc77c, \ub3c5\uc77c\uad70\uc740 \ub3c4\uc2dc \ub0b4\ub85c \uacf5\uc138\ub97c \ud3bc\uce58\uae30 \uc2dc\uc791\ud588\ub2e4. 5 \uc601\uad6d \uc804\ucc28 \uc5f0\ub300, 9 \ub354\ub7fc \uacbd\ubcf4\ubcd1 \uc5f0\ub300\uc640 1 \uc18c\ucd1d\ubcd1 \uc5ec\ub2e8\uc774 \uc544\uc6b0\ud1a0\ubc18\uc774 \uc788\ub294 \uc81c\uc2a4\ud14c\ubd80\ub974\ud06c\uc640 \ud788\ud2b8\ubca8\ub4dc\ub97c \uc810\ub839\ud588\ub2e4. \uadf8\ub7fc\uc5d0\ub3c4 \ubd88\uad6c\ud558\uace0, \ub3c5\uc77c\uad70\uc774 \uc544\uc6b0\ud1a0\ubc18\uc758 \uc77c\ubd80\ub864 \ud3ed\ud30c\uc2dc\ud0a4\uba74\uc11c \uc601\uad6d\uad70\uc758 \uc9c4\uaca9\uc774 \ub454\ud654\ub418\uc5c8\ub2e4. \uc77c\ub2e8 \uc601\uad6d\uad70\uc774 \ud558\ub974\ubd80\ub974\ud06c\ub85c \ub4e4\uc5b4\uac00\uc790 \ub3c5\uc77c\uad70\uc740 \uce58\uc5f4\ud558\uac8c \uc800\ud56d\ud588\uace0 \ub3c5\uc77c\uad70\uc774 \ud56d\ubcf5\ud558\uc9c0 \uc54a\uc544 \uac74\ubb3c \ud558\ub098\ud558\ub098\ub97c \uc810\ub839\ud574\uc57c \ud588\ub2e4. \uc774 \ub54c \ub3c5\uc77c 1 \uacf5\uc218\uad70\uc740 SS, \ub099\ud558\uc0b0\ubcd1, \uad6d\ubbfc\ub3cc\uaca9\ub300, \uc815\uaddc \ub3c5\uc77c \uad6d\ubc29\uad70 \uad70\uc778, \uc120\uc6d0, \uacbd\ucc30, \uc18c\ubc29\uad00, \ud788\ud2c0\ub7ec \uc720\uac90\ud2b8 \ub4f1\uc774 \uc11e\uc5ec\uc788\uc5c8\ub2e4. \ub3c4\uc2dc \uc911\uc2ec\uc73c\ub85c \uc9c4\uaca9\ud560\uc218\ub85d \ub3c5\uc77c\uad70\uc758 \uc800\ud56d\uc740 \uc810\uc810 \uc2ec\ud574\uc84c\ub2e4. \uc774\ub4e4\uc740 \ud568\ubd80\ub974\ud06c\uae4c\uc9c0 \uac00\ub294 \uc544\uc6b0\ud1a0\ubc18\uc5d0\uc11c \ud544\uc0ac\uc801\uc73c\ub85c \uc2f8\uc6b0\uace0 \uc788\uc5c8\ub2e4. \uadf8\ub0a0 \uc800\ub141, \uccab \ubc88\uc9f8 \uc601\uad6d\uad70\uc774 \uc5d8\ubca0 \uac15\uc744 \uac74\ub108 \ud568\ubd80\ub974\ud06c\ub85c \ub4e4\uc5b4\uac14\ub2e4. 88mm \ud3ec\ub294 \ub3c4\uc2dc \uacf3\uacf3\uc5d0 \ubd84\uc0b0\ub418\uc5b4 \uc788\uc5c8\uc9c0\ub9cc \uc804\ud22c\uac00 \uc9c4\ud589\ub428\uc5d0 \ub530\ub77c \ud6a8\uacfc\uac00 \ub098\ud0c0\ub0ac\ub2e4. \ub300\uc804\ucc28 \ub300\ub300\ub97c \ud3ec\ud568\ud55c \ub9ce\uc740 \ub3c5\uc77c\uad70, \ud5dd\uac00\ub9ac SS \ubd80\ub300, \"\ud310\uc800\ud30c\uc6b0\uc2a4\ud2b8\" \ubd80\ub300\uac00 \uc5ec\uc804\ud788 \ud558\ub974\ubd80\ub974\ud06c \ub0a8\ucabd \uc232\uc5d0 \ub0a8\uc544\uc788\uc5c8\uae30 \ub54c\ubb38\uc5d0 \uc601\uad6d\uad70\uc740 \uc774 \uc9c0\uc5ed\uc744 \uc6b0\ud68c\ud588\uace0, \ub098\uc911\uc5d0 \uc815\ub9ac\ub418\uc5c8\ub2e4. 1 \uc601\uad6d \uc804\ucc28 \ub300\ub300\uc758 \uc9c0\uc6d0\uc744 \ubc1b\uc740 53 \uc0ac\ub2e8\uc774 \uc232\uc5d0 \ub0a8\uc544 \uc788\ub358 \ub3c5\uc77c\uad70\uc744 \uacf5\uaca9\ud558\uc5ec 2,000\uba85\uc744 \ud3ec\ub85c\ub85c \uc7a1\uc558\ub2e4. \uc804\ud22c\uac00 \uacc4\uc18d\ub418\ub294 \ub3d9\uc548, \ub3c5\uc77c\uad70\uc758 \ubc29\uc5b4\uc120\uc740 \ubb34\ub108\uc9c0\uae30 \uc2dc\uc791\ud588\ub2e4. 4\uc6d4 30\uc77c, \ud788\ud2c0\ub7ec\uac00 \ubca0\ub97c\ub9b0\uc5d0\uc11c \uc790\uc0b4\ud558\uace0 \ubd81\ucabd\uc5d0\uc11c \uad70\ub300\ub97c \uc9c0\ud718\ud558\ub358 \ub300\uc81c\ub3c5 \uce74\ub97c \ub418\ub2c8\uce20\ub294 \uc5d0\ub974\uc708 \ubcfc\uce20\uc5d0\uac8c \uc601\uad6d\uad70\uc5d0\uac8c \ub3c4\uc2dc\ub97c \ud3ec\uae30\ud558\ub77c\uace0 \ub9d0\ud588\ub2e4. \ubcfc\uce20\ub294 \ub3c5\uc77c \ub300\ud45c\ub2e8\uacfc \ud568\uaed8 5\uc6d4 2\uc77c \uc0ac\ub2e8 \ubcf8\ubd80\uc5d0 \ub3c4\ucc29\ud588\uc73c\uba70 \uacf5\uc2dd\uc801\uc73c\ub85c 5\uc6d4 3\uc77c \ud568\ubd80\ub974\ud06c\uac00 \ud56d\ubcf5\ud588\ub2e4. \uac19\uc740 \ub0a0 7 \uc0ac\ub2e8\uc774 \ud3d0\ud5c8\uac00 \ub41c \ub3c4\uc2dc\ub85c \uc9c4\uc785\ud588\ub2e4."} {"question": "\ubbf8\ub9ac\uac00 \ubc14\ub78c\uc9c0\uc5ed\uc758 \uc911\uc2ec\uad00\ud560\uad6c\uac00 \ub41c \ud574\ub294?", "paragraph": "\ub9c8\uce68\ub0b4, 1910\ub144 9\uc6d4 10\uc77c \uccab \ubc88\uc9f8 \uc11d\uc720\uc2dc\ucd94\uc791\uc5c5\uc774 \uc2dc\uc791\ub418\uc5c8\ub2e4. \ub098\ubb34 \uae30\uc911\uae30\ub4e4\uacfc \uc2dc\ucd94\uc6a9 \ucf00\uc774\ube14 \ud234\ub85c \ub9cc\ub4e4\uc5b4\uc9c4 A\ud615 30\ubbf8\ud130\ub192\uc774\uc758 \ub9ac\uadf8(\uadf8\ub79c\ub4dc\uc62c\ub4dc\ub808\uc774\ub514\ub780 \ubcc4\uba85\uc73c\ub85c \ubd88\ub9ac\ub294)\ub294 \ub85c\uc584 \ub354\uce58 \uc258\uc5d0 \uc758\ud574 \uc6b4\uc601\ub418\uc5c8\ub2e4. 1910\ub144 11\uc6d4 22\uc77c 130\ubbf8\ud130\ub97c \ud30c\uace0\ub09c \ud6c4 \uc11d\uc720\uac00 \ubc1c\uacac\ub418\uc5c8\ub2e4. \ub85c\uc584 \ub354\uce58 \uc258\uc740 \ub610\ud55c \uc790\ud68c\uc0ac\ub97c \uc124\ub9bd, \uc0ac\ub77c\uc641 \uc624\uc77c \ud544\ub4dc \uc720\ud55c\ud68c\uc0ac(Sarawak Oil Field Ltd, \ud604\uc7ac\uc758 Sarawak Shell Berhad)\ub85c \uc774\ub984\ubd99\uc600\ub2e4. \uadf8 \uc774\ub798\ub85c 1972\ub144\uae4c\uc9c0 \ubbf8\ub9ac \uc8fc\ubcc0\uc5d0\uc11c \ub610\ub2e4\ub978 624\uac1c \uc721\uc0c1 \uc720\uc815\uc774 \uc2dc\ucd94\ub418\uc5c8\uc73c\uba70 \ubbf8\ub9ac\ud544\ub4dc\ub85c \uc774\ub984\ub0a0\ub838\ub2e4. \ubbf8\ub9ac\ud544\ub4dc\ub294 \uc0ac\ub77c\uc641\uc5d0\uc11c \uc720\uc77c\ud55c \uc721\uc0c1 \uc11d\uc720\ub9e4\uc7a5\uc9c0\uc774\ub2e4. \uc65c\ub0d0\ud558\uba74 1950\ub144\ub300 \ud6c4\ubc18\ubd80\ud130 \uc11d\uc720\uae30\uc5c5\ub4e4\uc774 \ud574\uc591\uc73c\ub85c \ub208\uc744 \ub3cc\ub838\uae30 \ub54c\ubb38\uc774\ub2e4. \uccab \ubc88\uc9f8 \uc720\uc815\uc778 \uadf8\ub79c\ub4dc\uc62c\ub4dc\ub808\uc774\ub514(Grand Old Lady)\ub294 1972\ub144 10\uc6d4 31\uc77c\uae4c\uc9c0 \ucd1d 650,000\ubc30\ub7f4\uc758 \uc11d\uc720\ub97c \uc0dd\uc0b0\ud574\ub0c8\ub2e4. \uadf8\ub9ac\uace0 \uccab\ubc88\uc9f8 \uc815\uc720\uc2dc\uc124\uacfc \ud574\uc800\ud30c\uc774\ud504\ub77c\uc778\uc740 1914\ub144\uc5d0 \uac74\uc124\ub418\uc5c8\ub2e4. \uc815\uc720\uc2dc\uc124\uc740 1916\ub144 \ub8e8\ud1b5(Lutong, \ubbf8\ub9ac\uc5d0\uc11c \ubd81\ucabd\uc73c\ub85c 11km \ub5a8\uc5b4\uc838\uc788\ub2e4.)\uc73c\ub85c \uc7ac\ubc30\uce58\ub418\uc5c8\ub2e4. \ucd1d\ub3c5\ub300\ub9ac \uc9d1\ubb34\uc2e4\uc740 1912\ub144 \ub9c8\ub8e8\ub514\uc5d0\uc11c \ubbf8\ub9ac\ub85c \uc774\uc804\ub418\uc5c8\ub2e4. \ubbf8\ub9ac\ub294 \uc258\uc5d0 \uc758\ud55c \uc11d\uc720\uc0dd\uc0b0\ube44\uc728\ub9cc\ud07c \ube60\ub974\uac8c \uc131\uc7a5\ud588\ub2e4. 1920\ub144, \ub3c4\ub85c\uac00 \uac74\uc124\ub418\uc5c8\uace0 \uc790\uc804\uac70\uc640 \uc774\ub95c\ucc28\uac00 \uad6c\ub9e4\ub418\uc5c8\uace0, 1921\ub144\uc5d0\ub294 40\uac1c\uc758 \uc0c1\uac00\ub4e4\uacfc \uc601\uad6d\uc778 \ud559\uad50\uc640 \uc911\uad6d\uc778 \ud559\uad50\uac00 \uc0dd\uacbc\uc73c\uba70 1924\ub144, \ubbf8\ub9ac\uc640 \ub8e8\ud1b5\uc744 \uc787\ub294 \ub300\ub85c\uac00 \uac74\uc124\ub418\uc5c8\ub2e4. 1925\ub144\uae4c\uc9c0 \ud68c\uc804 \uad74\ucc29\uae30\uc5d0 \uc758\ud55c \uc720\uc815 \uad74\ucc29 \ubc29\ubc95\uc774 \uc18c\uac1c\ub418\uc5c8\uc73c\uba70, \uc11d\uc720\uc0dd\uc0b0\ub7c9\uc740 \uacc4\uc18d \uc99d\uac00\ud558\uc5ec 1929\ub144 \ud558\ub8e8\ub2f9 15,211\ubc30\ub7f4\uc5d0 \ub2ec\ud558\ub294 \ucd5c\uace0\uc810\uc744 \uae30\ub85d\ud588\ub2e4. \uc2dd\uc218\uacf5\uae09\ub3c4 \uac1c\uc120\ub418\uc5c8\ub2e4. \uc815\uae00\uc774 \uc81c\uac70 \ub418\uace0, \ub354 \ub9ce\uc740 \ub3c4\ub85c\uac00 \uac74\uc124\ub418\uc5c8\ub2e4. 1929\ub144 \ubbf8\ub9ac\ub294 \ubc14\ub78c\uc9c0\uc5ed\uc758 \uc911\uc2ec\uad00\ud560\uad6c\uac00 \ub418\uc5c8\ub2e4.", "answer": "1929\ub144", "sentence": "1925\ub144\uae4c\uc9c0 \ud68c\uc804 \uad74\ucc29\uae30\uc5d0 \uc758\ud55c \uc720\uc815 \uad74\ucc29 \ubc29\ubc95\uc774 \uc18c\uac1c\ub418\uc5c8\uc73c\uba70, \uc11d\uc720\uc0dd\uc0b0\ub7c9\uc740 \uacc4\uc18d \uc99d\uac00\ud558\uc5ec 1929\ub144 \ud558\ub8e8\ub2f9 15,211\ubc30\ub7f4\uc5d0 \ub2ec\ud558\ub294 \ucd5c\uace0\uc810\uc744 \uae30\ub85d\ud588\ub2e4.", "paragraph_sentence": "\ub9c8\uce68\ub0b4, 1910\ub144 9\uc6d4 10\uc77c \uccab \ubc88\uc9f8 \uc11d\uc720\uc2dc\ucd94\uc791\uc5c5\uc774 \uc2dc\uc791\ub418\uc5c8\ub2e4. \ub098\ubb34 \uae30\uc911\uae30\ub4e4\uacfc \uc2dc\ucd94\uc6a9 \ucf00\uc774\ube14 \ud234\ub85c \ub9cc\ub4e4\uc5b4\uc9c4 A\ud615 30\ubbf8\ud130\ub192\uc774\uc758 \ub9ac\uadf8(\uadf8\ub79c\ub4dc\uc62c\ub4dc\ub808\uc774\ub514\ub780 \ubcc4\uba85\uc73c\ub85c \ubd88\ub9ac\ub294)\ub294 \ub85c\uc584 \ub354\uce58 \uc258\uc5d0 \uc758\ud574 \uc6b4\uc601\ub418\uc5c8\ub2e4. 1910\ub144 11\uc6d4 22\uc77c 130\ubbf8\ud130\ub97c \ud30c\uace0\ub09c \ud6c4 \uc11d\uc720\uac00 \ubc1c\uacac\ub418\uc5c8\ub2e4. \ub85c\uc584 \ub354\uce58 \uc258\uc740 \ub610\ud55c \uc790\ud68c\uc0ac\ub97c \uc124\ub9bd, \uc0ac\ub77c\uc641 \uc624\uc77c \ud544\ub4dc \uc720\ud55c\ud68c\uc0ac(Sarawak Oil Field Ltd, \ud604\uc7ac\uc758 Sarawak Shell Berhad)\ub85c \uc774\ub984\ubd99\uc600\ub2e4. \uadf8 \uc774\ub798\ub85c 1972\ub144\uae4c\uc9c0 \ubbf8\ub9ac \uc8fc\ubcc0\uc5d0\uc11c \ub610\ub2e4\ub978 624\uac1c \uc721\uc0c1 \uc720\uc815\uc774 \uc2dc\ucd94\ub418\uc5c8\uc73c\uba70 \ubbf8\ub9ac\ud544\ub4dc\ub85c \uc774\ub984\ub0a0\ub838\ub2e4. \ubbf8\ub9ac\ud544\ub4dc\ub294 \uc0ac\ub77c\uc641\uc5d0\uc11c \uc720\uc77c\ud55c \uc721\uc0c1 \uc11d\uc720\ub9e4\uc7a5\uc9c0\uc774\ub2e4. \uc65c\ub0d0\ud558\uba74 1950\ub144\ub300 \ud6c4\ubc18\ubd80\ud130 \uc11d\uc720\uae30\uc5c5\ub4e4\uc774 \ud574\uc591\uc73c\ub85c \ub208\uc744 \ub3cc\ub838\uae30 \ub54c\ubb38\uc774\ub2e4. \uccab \ubc88\uc9f8 \uc720\uc815\uc778 \uadf8\ub79c\ub4dc\uc62c\ub4dc\ub808\uc774\ub514(Grand Old Lady)\ub294 1972\ub144 10\uc6d4 31\uc77c\uae4c\uc9c0 \ucd1d 650,000\ubc30\ub7f4\uc758 \uc11d\uc720\ub97c \uc0dd\uc0b0\ud574\ub0c8\ub2e4. \uadf8\ub9ac\uace0 \uccab\ubc88\uc9f8 \uc815\uc720\uc2dc\uc124\uacfc \ud574\uc800\ud30c\uc774\ud504\ub77c\uc778\uc740 1914\ub144\uc5d0 \uac74\uc124\ub418\uc5c8\ub2e4. \uc815\uc720\uc2dc\uc124\uc740 1916\ub144 \ub8e8\ud1b5(Lutong, \ubbf8\ub9ac\uc5d0\uc11c \ubd81\ucabd\uc73c\ub85c 11km \ub5a8\uc5b4\uc838\uc788\ub2e4.)\uc73c\ub85c \uc7ac\ubc30\uce58\ub418\uc5c8\ub2e4. \ucd1d\ub3c5\ub300\ub9ac \uc9d1\ubb34\uc2e4\uc740 1912\ub144 \ub9c8\ub8e8\ub514\uc5d0\uc11c \ubbf8\ub9ac\ub85c \uc774\uc804\ub418\uc5c8\ub2e4. \ubbf8\ub9ac\ub294 \uc258\uc5d0 \uc758\ud55c \uc11d\uc720\uc0dd\uc0b0\ube44\uc728\ub9cc\ud07c \ube60\ub974\uac8c \uc131\uc7a5\ud588\ub2e4. 1920\ub144, \ub3c4\ub85c\uac00 \uac74\uc124\ub418\uc5c8\uace0 \uc790\uc804\uac70\uc640 \uc774\ub95c\ucc28\uac00 \uad6c\ub9e4\ub418\uc5c8\uace0, 1921\ub144\uc5d0\ub294 40\uac1c\uc758 \uc0c1\uac00\ub4e4\uacfc \uc601\uad6d\uc778 \ud559\uad50\uc640 \uc911\uad6d\uc778 \ud559\uad50\uac00 \uc0dd\uacbc\uc73c\uba70 1924\ub144, \ubbf8\ub9ac\uc640 \ub8e8\ud1b5\uc744 \uc787\ub294 \ub300\ub85c\uac00 \uac74\uc124\ub418\uc5c8\ub2e4. 1925\ub144\uae4c\uc9c0 \ud68c\uc804 \uad74\ucc29\uae30\uc5d0 \uc758\ud55c \uc720\uc815 \uad74\ucc29 \ubc29\ubc95\uc774 \uc18c\uac1c\ub418\uc5c8\uc73c\uba70, \uc11d\uc720\uc0dd\uc0b0\ub7c9\uc740 \uacc4\uc18d \uc99d\uac00\ud558\uc5ec 1929\ub144 \ud558\ub8e8\ub2f9 15,211\ubc30\ub7f4\uc5d0 \ub2ec\ud558\ub294 \ucd5c\uace0\uc810\uc744 \uae30\ub85d\ud588\ub2e4. \uc2dd\uc218\uacf5\uae09\ub3c4 \uac1c\uc120\ub418\uc5c8\ub2e4. \uc815\uae00\uc774 \uc81c\uac70 \ub418\uace0, \ub354 \ub9ce\uc740 \ub3c4\ub85c\uac00 \uac74\uc124\ub418\uc5c8\ub2e4. 1929\ub144 \ubbf8\ub9ac\ub294 \ubc14\ub78c\uc9c0\uc5ed\uc758 \uc911\uc2ec\uad00\ud560\uad6c\uac00 \ub418\uc5c8\ub2e4.", "paragraph_answer": "\ub9c8\uce68\ub0b4, 1910\ub144 9\uc6d4 10\uc77c \uccab \ubc88\uc9f8 \uc11d\uc720\uc2dc\ucd94\uc791\uc5c5\uc774 \uc2dc\uc791\ub418\uc5c8\ub2e4. \ub098\ubb34 \uae30\uc911\uae30\ub4e4\uacfc \uc2dc\ucd94\uc6a9 \ucf00\uc774\ube14 \ud234\ub85c \ub9cc\ub4e4\uc5b4\uc9c4 A\ud615 30\ubbf8\ud130\ub192\uc774\uc758 \ub9ac\uadf8(\uadf8\ub79c\ub4dc\uc62c\ub4dc\ub808\uc774\ub514\ub780 \ubcc4\uba85\uc73c\ub85c \ubd88\ub9ac\ub294)\ub294 \ub85c\uc584 \ub354\uce58 \uc258\uc5d0 \uc758\ud574 \uc6b4\uc601\ub418\uc5c8\ub2e4. 1910\ub144 11\uc6d4 22\uc77c 130\ubbf8\ud130\ub97c \ud30c\uace0\ub09c \ud6c4 \uc11d\uc720\uac00 \ubc1c\uacac\ub418\uc5c8\ub2e4. \ub85c\uc584 \ub354\uce58 \uc258\uc740 \ub610\ud55c \uc790\ud68c\uc0ac\ub97c \uc124\ub9bd, \uc0ac\ub77c\uc641 \uc624\uc77c \ud544\ub4dc \uc720\ud55c\ud68c\uc0ac(Sarawak Oil Field Ltd, \ud604\uc7ac\uc758 Sarawak Shell Berhad)\ub85c \uc774\ub984\ubd99\uc600\ub2e4. \uadf8 \uc774\ub798\ub85c 1972\ub144\uae4c\uc9c0 \ubbf8\ub9ac \uc8fc\ubcc0\uc5d0\uc11c \ub610\ub2e4\ub978 624\uac1c \uc721\uc0c1 \uc720\uc815\uc774 \uc2dc\ucd94\ub418\uc5c8\uc73c\uba70 \ubbf8\ub9ac\ud544\ub4dc\ub85c \uc774\ub984\ub0a0\ub838\ub2e4. \ubbf8\ub9ac\ud544\ub4dc\ub294 \uc0ac\ub77c\uc641\uc5d0\uc11c \uc720\uc77c\ud55c \uc721\uc0c1 \uc11d\uc720\ub9e4\uc7a5\uc9c0\uc774\ub2e4. \uc65c\ub0d0\ud558\uba74 1950\ub144\ub300 \ud6c4\ubc18\ubd80\ud130 \uc11d\uc720\uae30\uc5c5\ub4e4\uc774 \ud574\uc591\uc73c\ub85c \ub208\uc744 \ub3cc\ub838\uae30 \ub54c\ubb38\uc774\ub2e4. \uccab \ubc88\uc9f8 \uc720\uc815\uc778 \uadf8\ub79c\ub4dc\uc62c\ub4dc\ub808\uc774\ub514(Grand Old Lady)\ub294 1972\ub144 10\uc6d4 31\uc77c\uae4c\uc9c0 \ucd1d 650,000\ubc30\ub7f4\uc758 \uc11d\uc720\ub97c \uc0dd\uc0b0\ud574\ub0c8\ub2e4. \uadf8\ub9ac\uace0 \uccab\ubc88\uc9f8 \uc815\uc720\uc2dc\uc124\uacfc \ud574\uc800\ud30c\uc774\ud504\ub77c\uc778\uc740 1914\ub144\uc5d0 \uac74\uc124\ub418\uc5c8\ub2e4. \uc815\uc720\uc2dc\uc124\uc740 1916\ub144 \ub8e8\ud1b5(Lutong, \ubbf8\ub9ac\uc5d0\uc11c \ubd81\ucabd\uc73c\ub85c 11km \ub5a8\uc5b4\uc838\uc788\ub2e4.)\uc73c\ub85c \uc7ac\ubc30\uce58\ub418\uc5c8\ub2e4. \ucd1d\ub3c5\ub300\ub9ac \uc9d1\ubb34\uc2e4\uc740 1912\ub144 \ub9c8\ub8e8\ub514\uc5d0\uc11c \ubbf8\ub9ac\ub85c \uc774\uc804\ub418\uc5c8\ub2e4. \ubbf8\ub9ac\ub294 \uc258\uc5d0 \uc758\ud55c \uc11d\uc720\uc0dd\uc0b0\ube44\uc728\ub9cc\ud07c \ube60\ub974\uac8c \uc131\uc7a5\ud588\ub2e4. 1920\ub144, \ub3c4\ub85c\uac00 \uac74\uc124\ub418\uc5c8\uace0 \uc790\uc804\uac70\uc640 \uc774\ub95c\ucc28\uac00 \uad6c\ub9e4\ub418\uc5c8\uace0, 1921\ub144\uc5d0\ub294 40\uac1c\uc758 \uc0c1\uac00\ub4e4\uacfc \uc601\uad6d\uc778 \ud559\uad50\uc640 \uc911\uad6d\uc778 \ud559\uad50\uac00 \uc0dd\uacbc\uc73c\uba70 1924\ub144, \ubbf8\ub9ac\uc640 \ub8e8\ud1b5\uc744 \uc787\ub294 \ub300\ub85c\uac00 \uac74\uc124\ub418\uc5c8\ub2e4. 1925\ub144\uae4c\uc9c0 \ud68c\uc804 \uad74\ucc29\uae30\uc5d0 \uc758\ud55c \uc720\uc815 \uad74\ucc29 \ubc29\ubc95\uc774 \uc18c\uac1c\ub418\uc5c8\uc73c\uba70, \uc11d\uc720\uc0dd\uc0b0\ub7c9\uc740 \uacc4\uc18d \uc99d\uac00\ud558\uc5ec 1929\ub144 \ud558\ub8e8\ub2f9 15,211\ubc30\ub7f4\uc5d0 \ub2ec\ud558\ub294 \ucd5c\uace0\uc810\uc744 \uae30\ub85d\ud588\ub2e4. \uc2dd\uc218\uacf5\uae09\ub3c4 \uac1c\uc120\ub418\uc5c8\ub2e4. \uc815\uae00\uc774 \uc81c\uac70 \ub418\uace0, \ub354 \ub9ce\uc740 \ub3c4\ub85c\uac00 \uac74\uc124\ub418\uc5c8\ub2e4. 1929\ub144 \ubbf8\ub9ac\ub294 \ubc14\ub78c\uc9c0\uc5ed\uc758 \uc911\uc2ec\uad00\ud560\uad6c\uac00 \ub418\uc5c8\ub2e4.", "sentence_answer": "1925\ub144\uae4c\uc9c0 \ud68c\uc804 \uad74\ucc29\uae30\uc5d0 \uc758\ud55c \uc720\uc815 \uad74\ucc29 \ubc29\ubc95\uc774 \uc18c\uac1c\ub418\uc5c8\uc73c\uba70, \uc11d\uc720\uc0dd\uc0b0\ub7c9\uc740 \uacc4\uc18d \uc99d\uac00\ud558\uc5ec 1929\ub144 \ud558\ub8e8\ub2f9 15,211\ubc30\ub7f4\uc5d0 \ub2ec\ud558\ub294 \ucd5c\uace0\uc810\uc744 \uae30\ub85d\ud588\ub2e4.", "paragraph_id": "6505561-3-1", "paragraph_question": "question: \ubbf8\ub9ac\uac00 \ubc14\ub78c\uc9c0\uc5ed\uc758 \uc911\uc2ec\uad00\ud560\uad6c\uac00 \ub41c \ud574\ub294?, context: \ub9c8\uce68\ub0b4, 1910\ub144 9\uc6d4 10\uc77c \uccab \ubc88\uc9f8 \uc11d\uc720\uc2dc\ucd94\uc791\uc5c5\uc774 \uc2dc\uc791\ub418\uc5c8\ub2e4. \ub098\ubb34 \uae30\uc911\uae30\ub4e4\uacfc \uc2dc\ucd94\uc6a9 \ucf00\uc774\ube14 \ud234\ub85c \ub9cc\ub4e4\uc5b4\uc9c4 A\ud615 30\ubbf8\ud130\ub192\uc774\uc758 \ub9ac\uadf8(\uadf8\ub79c\ub4dc\uc62c\ub4dc\ub808\uc774\ub514\ub780 \ubcc4\uba85\uc73c\ub85c \ubd88\ub9ac\ub294)\ub294 \ub85c\uc584 \ub354\uce58 \uc258\uc5d0 \uc758\ud574 \uc6b4\uc601\ub418\uc5c8\ub2e4. 1910\ub144 11\uc6d4 22\uc77c 130\ubbf8\ud130\ub97c \ud30c\uace0\ub09c \ud6c4 \uc11d\uc720\uac00 \ubc1c\uacac\ub418\uc5c8\ub2e4. \ub85c\uc584 \ub354\uce58 \uc258\uc740 \ub610\ud55c \uc790\ud68c\uc0ac\ub97c \uc124\ub9bd, \uc0ac\ub77c\uc641 \uc624\uc77c \ud544\ub4dc \uc720\ud55c\ud68c\uc0ac(Sarawak Oil Field Ltd, \ud604\uc7ac\uc758 Sarawak Shell Berhad)\ub85c \uc774\ub984\ubd99\uc600\ub2e4. \uadf8 \uc774\ub798\ub85c 1972\ub144\uae4c\uc9c0 \ubbf8\ub9ac \uc8fc\ubcc0\uc5d0\uc11c \ub610\ub2e4\ub978 624\uac1c \uc721\uc0c1 \uc720\uc815\uc774 \uc2dc\ucd94\ub418\uc5c8\uc73c\uba70 \ubbf8\ub9ac\ud544\ub4dc\ub85c \uc774\ub984\ub0a0\ub838\ub2e4. \ubbf8\ub9ac\ud544\ub4dc\ub294 \uc0ac\ub77c\uc641\uc5d0\uc11c \uc720\uc77c\ud55c \uc721\uc0c1 \uc11d\uc720\ub9e4\uc7a5\uc9c0\uc774\ub2e4. \uc65c\ub0d0\ud558\uba74 1950\ub144\ub300 \ud6c4\ubc18\ubd80\ud130 \uc11d\uc720\uae30\uc5c5\ub4e4\uc774 \ud574\uc591\uc73c\ub85c \ub208\uc744 \ub3cc\ub838\uae30 \ub54c\ubb38\uc774\ub2e4. \uccab \ubc88\uc9f8 \uc720\uc815\uc778 \uadf8\ub79c\ub4dc\uc62c\ub4dc\ub808\uc774\ub514(Grand Old Lady)\ub294 1972\ub144 10\uc6d4 31\uc77c\uae4c\uc9c0 \ucd1d 650,000\ubc30\ub7f4\uc758 \uc11d\uc720\ub97c \uc0dd\uc0b0\ud574\ub0c8\ub2e4. \uadf8\ub9ac\uace0 \uccab\ubc88\uc9f8 \uc815\uc720\uc2dc\uc124\uacfc \ud574\uc800\ud30c\uc774\ud504\ub77c\uc778\uc740 1914\ub144\uc5d0 \uac74\uc124\ub418\uc5c8\ub2e4. \uc815\uc720\uc2dc\uc124\uc740 1916\ub144 \ub8e8\ud1b5(Lutong, \ubbf8\ub9ac\uc5d0\uc11c \ubd81\ucabd\uc73c\ub85c 11km \ub5a8\uc5b4\uc838\uc788\ub2e4.)\uc73c\ub85c \uc7ac\ubc30\uce58\ub418\uc5c8\ub2e4. \ucd1d\ub3c5\ub300\ub9ac \uc9d1\ubb34\uc2e4\uc740 1912\ub144 \ub9c8\ub8e8\ub514\uc5d0\uc11c \ubbf8\ub9ac\ub85c \uc774\uc804\ub418\uc5c8\ub2e4. \ubbf8\ub9ac\ub294 \uc258\uc5d0 \uc758\ud55c \uc11d\uc720\uc0dd\uc0b0\ube44\uc728\ub9cc\ud07c \ube60\ub974\uac8c \uc131\uc7a5\ud588\ub2e4. 1920\ub144, \ub3c4\ub85c\uac00 \uac74\uc124\ub418\uc5c8\uace0 \uc790\uc804\uac70\uc640 \uc774\ub95c\ucc28\uac00 \uad6c\ub9e4\ub418\uc5c8\uace0, 1921\ub144\uc5d0\ub294 40\uac1c\uc758 \uc0c1\uac00\ub4e4\uacfc \uc601\uad6d\uc778 \ud559\uad50\uc640 \uc911\uad6d\uc778 \ud559\uad50\uac00 \uc0dd\uacbc\uc73c\uba70 1924\ub144, \ubbf8\ub9ac\uc640 \ub8e8\ud1b5\uc744 \uc787\ub294 \ub300\ub85c\uac00 \uac74\uc124\ub418\uc5c8\ub2e4. 1925\ub144\uae4c\uc9c0 \ud68c\uc804 \uad74\ucc29\uae30\uc5d0 \uc758\ud55c \uc720\uc815 \uad74\ucc29 \ubc29\ubc95\uc774 \uc18c\uac1c\ub418\uc5c8\uc73c\uba70, \uc11d\uc720\uc0dd\uc0b0\ub7c9\uc740 \uacc4\uc18d \uc99d\uac00\ud558\uc5ec 1929\ub144 \ud558\ub8e8\ub2f9 15,211\ubc30\ub7f4\uc5d0 \ub2ec\ud558\ub294 \ucd5c\uace0\uc810\uc744 \uae30\ub85d\ud588\ub2e4. \uc2dd\uc218\uacf5\uae09\ub3c4 \uac1c\uc120\ub418\uc5c8\ub2e4. \uc815\uae00\uc774 \uc81c\uac70 \ub418\uace0, \ub354 \ub9ce\uc740 \ub3c4\ub85c\uac00 \uac74\uc124\ub418\uc5c8\ub2e4. 1929\ub144 \ubbf8\ub9ac\ub294 \ubc14\ub78c\uc9c0\uc5ed\uc758 \uc911\uc2ec\uad00\ud560\uad6c\uac00 \ub418\uc5c8\ub2e4."} {"question": "\ubbfc\uc8fc\ud654 \uc6b4\ub3d9\uc774 \uc804\ub450\ud658 \uc815\uad8c\uc758 \ube44\ubbfc\uc8fc\uc131\uc5d0 \ub9de\uc120 \uc6b4\ub3d9\uc73c\ub85c \uc7ac\ud3c9\uac00 \ubc1b\uc740 \ub144\ub3c4\ub294?", "paragraph": "\uc804\ub450\ud658 \uc815\uad8c\uc740 \uad11\uc8fc \ubbfc\uc8fc\ud654 \uc6b4\ub3d9\uc744 \uae40\ub300\uc911\uc758 \uc0ac\uc8fc\uc5d0 \uc758\ud574 \ubc1c\uc0dd\ud55c \uc18c\uc694\uc0ac\ud0dc\ub85c \uc870\uc791\ud588\ub2e4. \ud558\uc9c0\ub9cc 1988\ub144 5\uacf5 \uccad\ubb38\ud68c\ub97c \uac70\uce58\uace0 1995\ub144 12\uc6d4 21\uc77c \uad6d\ud68c\uc5d0\uc11c \uad11\uc8fc \ubbfc\uc8fc\ud654 \uc6b4\ub3d9\uc73c\ub85c \uaddc\uc815\ud574, \uacc4\uc5c4\uad70\uc758 \uc9c4\uc555 \uacfc\uc815\uc5d0\uc11c \uc8fd\uac70\ub098 \ubd80\uc0c1\ub2f9\ud55c \uad11\uc8fc \ubbfc\uc8fc\ud654 \uc6b4\ub3d9 \uad00\ub828\uc790\ub4e4\uc5d0 \ub300\ud55c \uba85\uc608\ud68c\ubcf5 \ubc0f \ud53c\ud574 \ubc30\uc0c1\uc744 \uc704\ud55c 5\u00b718\ubbfc\uc8fc\ud654\uc6b4\ub3d9\ub4f1\uc5d0\uad00\ud55c\ud2b9\ubcc4\ubc95(1995. 12. 21.)\uacfc 5\u00b718 \uad11\uc8fc \ubbfc\uc8fc\ud654\uc6b4\ub3d9 \uad00\ub828\uc790 \ubcf4\uc0c1 \ub4f1\uc5d0 \uad00\ud55c \ubc95\ub960(1997. 12. 17.)\uc774 \uc81c\uc815 \ub418\uba74\uc11c \uc804\ub450\ud658 \uc815\uad8c\uc758 \ube44(\u975e)\ubbfc\uc8fc\uc131\uacfc \ud3ed\ub825\uc5d0 \ub9de\uc11c \uc2f8\uc6b4 \ubbfc\uc8fc\ud654 \uc6b4\ub3d9\uc73c\ub85c \ub2e4\uc2dc \ud3c9\uac00\ubc1b\uc558\ub2e4. \uad11\uc8fc \uc2dc\ubbfc\ub4e4\uc744 \ud559\uc0b4\ud55c \uad11\uc8fc\ud559\uc0b4 \ucc45\uc784\uc790\ub4e4\uc740 \uc11c\ud6c8\uc774 \ucde8\uc18c\ub410\uc73c\uba70 \uadf8 \uc790\uaca9\ub3c4 \ubc15\ud0c8\ub410\ub2e4. \uc774 \uc0ac\uac74\uc758 \ud575\uc2ec \uad00\ub828\uc790\uc778 \uc804\ub450\ud658, \ub178\ud0dc\uc6b0\ub294 1997\ub144 \ub300\ubc95\uc6d0\uc73c\ub85c\ubd80\ud130 \uc9d5\uc5ed\ud615\uacfc 2\ucc9c\uc5b5 \uc6d0\uc774 \ub118\ub294 \ucd94\uc9d5\uae08\uc744 \uc120\uace0\ubc1b\uc558\ub2e4.", "answer": "1995\ub144", "sentence": "\ud558\uc9c0\ub9cc 1988\ub144 5\uacf5 \uccad\ubb38\ud68c\ub97c \uac70\uce58\uace0 1995\ub144 12\uc6d4 21\uc77c \uad6d\ud68c\uc5d0\uc11c \uad11\uc8fc \ubbfc\uc8fc\ud654 \uc6b4\ub3d9\uc73c\ub85c \uaddc\uc815\ud574, \uacc4\uc5c4\uad70\uc758 \uc9c4\uc555 \uacfc\uc815\uc5d0\uc11c \uc8fd\uac70\ub098 \ubd80\uc0c1\ub2f9\ud55c \uad11\uc8fc \ubbfc\uc8fc\ud654 \uc6b4\ub3d9 \uad00\ub828\uc790\ub4e4\uc5d0 \ub300\ud55c \uba85\uc608\ud68c\ubcf5 \ubc0f \ud53c\ud574 \ubc30\uc0c1\uc744 \uc704\ud55c 5\u00b718\ubbfc\uc8fc\ud654\uc6b4\ub3d9\ub4f1\uc5d0\uad00\ud55c\ud2b9\ubcc4\ubc95(1995.", "paragraph_sentence": "\uc804\ub450\ud658 \uc815\uad8c\uc740 \uad11\uc8fc \ubbfc\uc8fc\ud654 \uc6b4\ub3d9\uc744 \uae40\ub300\uc911\uc758 \uc0ac\uc8fc\uc5d0 \uc758\ud574 \ubc1c\uc0dd\ud55c \uc18c\uc694\uc0ac\ud0dc\ub85c \uc870\uc791\ud588\ub2e4. \ud558\uc9c0\ub9cc 1988\ub144 5\uacf5 \uccad\ubb38\ud68c\ub97c \uac70\uce58\uace0 1995\ub144 12\uc6d4 21\uc77c \uad6d\ud68c\uc5d0\uc11c \uad11\uc8fc \ubbfc\uc8fc\ud654 \uc6b4\ub3d9\uc73c\ub85c \uaddc\uc815\ud574, \uacc4\uc5c4\uad70\uc758 \uc9c4\uc555 \uacfc\uc815\uc5d0\uc11c \uc8fd\uac70\ub098 \ubd80\uc0c1\ub2f9\ud55c \uad11\uc8fc \ubbfc\uc8fc\ud654 \uc6b4\ub3d9 \uad00\ub828\uc790\ub4e4\uc5d0 \ub300\ud55c \uba85\uc608\ud68c\ubcf5 \ubc0f \ud53c\ud574 \ubc30\uc0c1\uc744 \uc704\ud55c 5\u00b718\ubbfc\uc8fc\ud654\uc6b4\ub3d9\ub4f1\uc5d0\uad00\ud55c\ud2b9\ubcc4\ubc95(1995. 12. 21.)\uacfc 5\u00b718 \uad11\uc8fc \ubbfc\uc8fc\ud654\uc6b4\ub3d9 \uad00\ub828\uc790 \ubcf4\uc0c1 \ub4f1\uc5d0 \uad00\ud55c \ubc95\ub960(1997. 12. 17.) \uc774 \uc81c\uc815 \ub418\uba74\uc11c \uc804\ub450\ud658 \uc815\uad8c\uc758 \ube44(\u975e)\ubbfc\uc8fc\uc131\uacfc \ud3ed\ub825\uc5d0 \ub9de\uc11c \uc2f8\uc6b4 \ubbfc\uc8fc\ud654 \uc6b4\ub3d9\uc73c\ub85c \ub2e4\uc2dc \ud3c9\uac00\ubc1b\uc558\ub2e4. \uad11\uc8fc \uc2dc\ubbfc\ub4e4\uc744 \ud559\uc0b4\ud55c \uad11\uc8fc\ud559\uc0b4 \ucc45\uc784\uc790\ub4e4\uc740 \uc11c\ud6c8\uc774 \ucde8\uc18c\ub410\uc73c\uba70 \uadf8 \uc790\uaca9\ub3c4 \ubc15\ud0c8\ub410\ub2e4. \uc774 \uc0ac\uac74\uc758 \ud575\uc2ec \uad00\ub828\uc790\uc778 \uc804\ub450\ud658, \ub178\ud0dc\uc6b0\ub294 1997\ub144 \ub300\ubc95\uc6d0\uc73c\ub85c\ubd80\ud130 \uc9d5\uc5ed\ud615\uacfc 2\ucc9c\uc5b5 \uc6d0\uc774 \ub118\ub294 \ucd94\uc9d5\uae08\uc744 \uc120\uace0\ubc1b\uc558\ub2e4.", "paragraph_answer": "\uc804\ub450\ud658 \uc815\uad8c\uc740 \uad11\uc8fc \ubbfc\uc8fc\ud654 \uc6b4\ub3d9\uc744 \uae40\ub300\uc911\uc758 \uc0ac\uc8fc\uc5d0 \uc758\ud574 \ubc1c\uc0dd\ud55c \uc18c\uc694\uc0ac\ud0dc\ub85c \uc870\uc791\ud588\ub2e4. \ud558\uc9c0\ub9cc 1988\ub144 5\uacf5 \uccad\ubb38\ud68c\ub97c \uac70\uce58\uace0 1995\ub144 12\uc6d4 21\uc77c \uad6d\ud68c\uc5d0\uc11c \uad11\uc8fc \ubbfc\uc8fc\ud654 \uc6b4\ub3d9\uc73c\ub85c \uaddc\uc815\ud574, \uacc4\uc5c4\uad70\uc758 \uc9c4\uc555 \uacfc\uc815\uc5d0\uc11c \uc8fd\uac70\ub098 \ubd80\uc0c1\ub2f9\ud55c \uad11\uc8fc \ubbfc\uc8fc\ud654 \uc6b4\ub3d9 \uad00\ub828\uc790\ub4e4\uc5d0 \ub300\ud55c \uba85\uc608\ud68c\ubcf5 \ubc0f \ud53c\ud574 \ubc30\uc0c1\uc744 \uc704\ud55c 5\u00b718\ubbfc\uc8fc\ud654\uc6b4\ub3d9\ub4f1\uc5d0\uad00\ud55c\ud2b9\ubcc4\ubc95(1995. 12. 21.)\uacfc 5\u00b718 \uad11\uc8fc \ubbfc\uc8fc\ud654\uc6b4\ub3d9 \uad00\ub828\uc790 \ubcf4\uc0c1 \ub4f1\uc5d0 \uad00\ud55c \ubc95\ub960(1997. 12. 17.)\uc774 \uc81c\uc815 \ub418\uba74\uc11c \uc804\ub450\ud658 \uc815\uad8c\uc758 \ube44(\u975e)\ubbfc\uc8fc\uc131\uacfc \ud3ed\ub825\uc5d0 \ub9de\uc11c \uc2f8\uc6b4 \ubbfc\uc8fc\ud654 \uc6b4\ub3d9\uc73c\ub85c \ub2e4\uc2dc \ud3c9\uac00\ubc1b\uc558\ub2e4. \uad11\uc8fc \uc2dc\ubbfc\ub4e4\uc744 \ud559\uc0b4\ud55c \uad11\uc8fc\ud559\uc0b4 \ucc45\uc784\uc790\ub4e4\uc740 \uc11c\ud6c8\uc774 \ucde8\uc18c\ub410\uc73c\uba70 \uadf8 \uc790\uaca9\ub3c4 \ubc15\ud0c8\ub410\ub2e4. \uc774 \uc0ac\uac74\uc758 \ud575\uc2ec \uad00\ub828\uc790\uc778 \uc804\ub450\ud658, \ub178\ud0dc\uc6b0\ub294 1997\ub144 \ub300\ubc95\uc6d0\uc73c\ub85c\ubd80\ud130 \uc9d5\uc5ed\ud615\uacfc 2\ucc9c\uc5b5 \uc6d0\uc774 \ub118\ub294 \ucd94\uc9d5\uae08\uc744 \uc120\uace0\ubc1b\uc558\ub2e4.", "sentence_answer": "\ud558\uc9c0\ub9cc 1988\ub144 5\uacf5 \uccad\ubb38\ud68c\ub97c \uac70\uce58\uace0 1995\ub144 12\uc6d4 21\uc77c \uad6d\ud68c\uc5d0\uc11c \uad11\uc8fc \ubbfc\uc8fc\ud654 \uc6b4\ub3d9\uc73c\ub85c \uaddc\uc815\ud574, \uacc4\uc5c4\uad70\uc758 \uc9c4\uc555 \uacfc\uc815\uc5d0\uc11c \uc8fd\uac70\ub098 \ubd80\uc0c1\ub2f9\ud55c \uad11\uc8fc \ubbfc\uc8fc\ud654 \uc6b4\ub3d9 \uad00\ub828\uc790\ub4e4\uc5d0 \ub300\ud55c \uba85\uc608\ud68c\ubcf5 \ubc0f \ud53c\ud574 \ubc30\uc0c1\uc744 \uc704\ud55c 5\u00b718\ubbfc\uc8fc\ud654\uc6b4\ub3d9\ub4f1\uc5d0\uad00\ud55c\ud2b9\ubcc4\ubc95(1995.", "paragraph_id": "6481190-19-1", "paragraph_question": "question: \ubbfc\uc8fc\ud654 \uc6b4\ub3d9\uc774 \uc804\ub450\ud658 \uc815\uad8c\uc758 \ube44\ubbfc\uc8fc\uc131\uc5d0 \ub9de\uc120 \uc6b4\ub3d9\uc73c\ub85c \uc7ac\ud3c9\uac00 \ubc1b\uc740 \ub144\ub3c4\ub294?, context: \uc804\ub450\ud658 \uc815\uad8c\uc740 \uad11\uc8fc \ubbfc\uc8fc\ud654 \uc6b4\ub3d9\uc744 \uae40\ub300\uc911\uc758 \uc0ac\uc8fc\uc5d0 \uc758\ud574 \ubc1c\uc0dd\ud55c \uc18c\uc694\uc0ac\ud0dc\ub85c \uc870\uc791\ud588\ub2e4. \ud558\uc9c0\ub9cc 1988\ub144 5\uacf5 \uccad\ubb38\ud68c\ub97c \uac70\uce58\uace0 1995\ub144 12\uc6d4 21\uc77c \uad6d\ud68c\uc5d0\uc11c \uad11\uc8fc \ubbfc\uc8fc\ud654 \uc6b4\ub3d9\uc73c\ub85c \uaddc\uc815\ud574, \uacc4\uc5c4\uad70\uc758 \uc9c4\uc555 \uacfc\uc815\uc5d0\uc11c \uc8fd\uac70\ub098 \ubd80\uc0c1\ub2f9\ud55c \uad11\uc8fc \ubbfc\uc8fc\ud654 \uc6b4\ub3d9 \uad00\ub828\uc790\ub4e4\uc5d0 \ub300\ud55c \uba85\uc608\ud68c\ubcf5 \ubc0f \ud53c\ud574 \ubc30\uc0c1\uc744 \uc704\ud55c 5\u00b718\ubbfc\uc8fc\ud654\uc6b4\ub3d9\ub4f1\uc5d0\uad00\ud55c\ud2b9\ubcc4\ubc95(1995. 12. 21.)\uacfc 5\u00b718 \uad11\uc8fc \ubbfc\uc8fc\ud654\uc6b4\ub3d9 \uad00\ub828\uc790 \ubcf4\uc0c1 \ub4f1\uc5d0 \uad00\ud55c \ubc95\ub960(1997. 12. 17.)\uc774 \uc81c\uc815 \ub418\uba74\uc11c \uc804\ub450\ud658 \uc815\uad8c\uc758 \ube44(\u975e)\ubbfc\uc8fc\uc131\uacfc \ud3ed\ub825\uc5d0 \ub9de\uc11c \uc2f8\uc6b4 \ubbfc\uc8fc\ud654 \uc6b4\ub3d9\uc73c\ub85c \ub2e4\uc2dc \ud3c9\uac00\ubc1b\uc558\ub2e4. \uad11\uc8fc \uc2dc\ubbfc\ub4e4\uc744 \ud559\uc0b4\ud55c \uad11\uc8fc\ud559\uc0b4 \ucc45\uc784\uc790\ub4e4\uc740 \uc11c\ud6c8\uc774 \ucde8\uc18c\ub410\uc73c\uba70 \uadf8 \uc790\uaca9\ub3c4 \ubc15\ud0c8\ub410\ub2e4. \uc774 \uc0ac\uac74\uc758 \ud575\uc2ec \uad00\ub828\uc790\uc778 \uc804\ub450\ud658, \ub178\ud0dc\uc6b0\ub294 1997\ub144 \ub300\ubc95\uc6d0\uc73c\ub85c\ubd80\ud130 \uc9d5\uc5ed\ud615\uacfc 2\ucc9c\uc5b5 \uc6d0\uc774 \ub118\ub294 \ucd94\uc9d5\uae08\uc744 \uc120\uace0\ubc1b\uc558\ub2e4."} {"question": "\uc138\uacc4\uc758\uc870\ud654 \ucd9c\ud310 \uc9c1\ud6c4 \uce90\ud50c\ub7ec\ub208 \ub204\uad6c\uc640 \uc120\ucde8\uad8c \ub17c\uc7c1\uc5d0 \ud729\uc2f8\uc600\ub294\uac00?", "paragraph": "\ucf00\ud50c\ub7ec\ub294 \uc5b4\uba38\ub2c8\uac00 \ub9c8\ub140\ub85c \ubab0\ub9b0 \uc5b4\ucc98\uad6c\ub2c8\uc5c6\ub294 \uc0ac\uac74\uc744 \ud574\uacb0\ud558\uae30 \uc704\ud574 \uc774\ub9ac\uc800\ub9ac \ub2e4\ub2c8\ub358 \ub3c4\uc911 \uae30\ubd84 \uc804\ud658\uc6a9\uc73c\ub85c \ube48\uc13c\uc870 \uac08\ub9b4\ub808\uc774 \uc758 \u300a\uace0\ub300\uc640 \ud604\ub300 \uc74c\uc545\uacfc\uc758 \ub300\ud654\u300b\ub77c\ub294 \ucc45\uc744 \ub4e4\uace0 \ub2e4\ub154\ub2e4. \uc5ec\uae30\uc11c \ube48\uc13c\uc870\ub294 \ud53c\ud0c0\uace0\ub77c\uc2a4\uc758 \uc870\ud654\uc5d0 \ub300\ud55c \uc774\ub860\uc744 \uc639\ud638\ud588\ub2e4. \ucf00\ud50c\ub7ec\ub294 \u2018\uc870\ud654\u2019\ub77c\uace0 \ud558\ub294 \uc138\uacc4\uc758 \uc218\ud559\uc801 \uaddc\uce59\uc744 \ucc3e\uc73c\ub824 \ud588\uace0, \uae30\ud558\ud559\uc5d0 \uac10\uba85\uc744 \ubc1b\uc544 \u201c\uae30\ud558\ud559\uc801 \ubb3c\uccb4\uac00 \uc804 \uc6b0\uc8fc\uc758 \ubaa8\ud615\uc744 \ub5a0\uc624\ub974\uac8c \ud558\uc600\ub2e4\u201d\uace0 \ud588\ub2e4. \u300a\uc138\uacc4\uc758 \uc870\ud654\u300b\uc5d0\uc11c \ucf00\ud50c\ub7ec\ub294 \uc790\uc5f0\uacc4, \ud2b9\ud788 \ucc9c\ubb38\ud559\uc801, \uc810\uc131\ud559\uc801\uc778 \uba74\uc5d0\uc11c\uc758 \uc790\uc5f0\uacc4\uc758 \uc870\ud654\ub97c \uc74c\uc545\uc801 \uad00\uc810\uc73c\ub85c \uc124\uba85\ud558\ub824\uace0 \uc2dc\ub3c4\ud588\ub2e4. \u300a\uc138\uacc4\uc758 \uc870\ud654\u300b\uc758 \uc911\uc2ec \uc124\uc815\uc740 \ud53c\ud0c0\uace0\ub77c\uc2a4, \ud504\ud1a8\ub808\ubbf8 \ub4f1 \ub9ce\uc740 \ucf00\ud50c\ub7ec \uc774\uc804\uc758 \ucc9c\ubb38\ud559\uc790\ub4e4\uc774 \uc5f0\uad6c\ud588\ub358 \u3008\ucc9c\uccb4\uc758 \uc74c\uc545\u3009\uc774\ub098 \u3008\ucc9c\uc0c1\uc758 \uc74c\uc545\u3009 \uc774\uc5c8\ub294\ub370, \u300a\uc138\uacc4\uc758 \uc870\ud654\u300b\uc758 \ucd9c\ud310 \uc9c1\ud6c4\uc5d0 \ucf00\ud50c\ub7ec\ub294 \ube44\uc2b7\ud55c \ub54c\uc5d0 \ub3c5\uc790\uc801\uc73c\ub85c \uc870\ud654 \uc774\ub860\uc744 \ucd9c\ud310\ud588\ub358 \ub85c\ubc84\ud2b8 \ud50c\ub8e8\ub4dc\uc640 \uc120\ucde8\uad8c \ub17c\uc7c1\uc5d0 \ud729\uc2f8\uc600\ub2e4.", "answer": "\ub85c\ubc84\ud2b8 \ud50c\ub8e8\ub4dc", "sentence": "\u300a\uc138\uacc4\uc758 \uc870\ud654\u300b\uc758 \uc911\uc2ec \uc124\uc815\uc740 \ud53c\ud0c0\uace0\ub77c\uc2a4, \ud504\ud1a8\ub808\ubbf8 \ub4f1 \ub9ce\uc740 \ucf00\ud50c\ub7ec \uc774\uc804\uc758 \ucc9c\ubb38\ud559\uc790\ub4e4\uc774 \uc5f0\uad6c\ud588\ub358 \u3008\ucc9c\uccb4\uc758 \uc74c\uc545\u3009\uc774\ub098 \u3008\ucc9c\uc0c1\uc758 \uc74c\uc545\u3009 \uc774\uc5c8\ub294\ub370, \u300a\uc138\uacc4\uc758 \uc870\ud654\u300b\uc758 \ucd9c\ud310 \uc9c1\ud6c4\uc5d0 \ucf00\ud50c\ub7ec\ub294 \ube44\uc2b7\ud55c \ub54c\uc5d0 \ub3c5\uc790\uc801\uc73c\ub85c \uc870\ud654 \uc774\ub860\uc744 \ucd9c\ud310\ud588\ub358 \ub85c\ubc84\ud2b8 \ud50c\ub8e8\ub4dc \uc640 \uc120\ucde8\uad8c \ub17c\uc7c1\uc5d0 \ud729\uc2f8\uc600\ub2e4.", "paragraph_sentence": "\ucf00\ud50c\ub7ec\ub294 \uc5b4\uba38\ub2c8\uac00 \ub9c8\ub140\ub85c \ubab0\ub9b0 \uc5b4\ucc98\uad6c\ub2c8\uc5c6\ub294 \uc0ac\uac74\uc744 \ud574\uacb0\ud558\uae30 \uc704\ud574 \uc774\ub9ac\uc800\ub9ac \ub2e4\ub2c8\ub358 \ub3c4\uc911 \uae30\ubd84 \uc804\ud658\uc6a9\uc73c\ub85c \ube48\uc13c\uc870 \uac08\ub9b4\ub808\uc774 \uc758 \u300a\uace0\ub300\uc640 \ud604\ub300 \uc74c\uc545\uacfc\uc758 \ub300\ud654\u300b\ub77c\ub294 \ucc45\uc744 \ub4e4\uace0 \ub2e4\ub154\ub2e4. \uc5ec\uae30\uc11c \ube48\uc13c\uc870\ub294 \ud53c\ud0c0\uace0\ub77c\uc2a4\uc758 \uc870\ud654\uc5d0 \ub300\ud55c \uc774\ub860\uc744 \uc639\ud638\ud588\ub2e4. \ucf00\ud50c\ub7ec\ub294 \u2018\uc870\ud654\u2019\ub77c\uace0 \ud558\ub294 \uc138\uacc4\uc758 \uc218\ud559\uc801 \uaddc\uce59\uc744 \ucc3e\uc73c\ub824 \ud588\uace0, \uae30\ud558\ud559\uc5d0 \uac10\uba85\uc744 \ubc1b\uc544 \u201c\uae30\ud558\ud559\uc801 \ubb3c\uccb4\uac00 \uc804 \uc6b0\uc8fc\uc758 \ubaa8\ud615\uc744 \ub5a0\uc624\ub974\uac8c \ud558\uc600\ub2e4\u201d\uace0 \ud588\ub2e4. \u300a\uc138\uacc4\uc758 \uc870\ud654\u300b\uc5d0\uc11c \ucf00\ud50c\ub7ec\ub294 \uc790\uc5f0\uacc4, \ud2b9\ud788 \ucc9c\ubb38\ud559\uc801, \uc810\uc131\ud559\uc801\uc778 \uba74\uc5d0\uc11c\uc758 \uc790\uc5f0\uacc4\uc758 \uc870\ud654\ub97c \uc74c\uc545\uc801 \uad00\uc810\uc73c\ub85c \uc124\uba85\ud558\ub824\uace0 \uc2dc\ub3c4\ud588\ub2e4. \u300a\uc138\uacc4\uc758 \uc870\ud654\u300b\uc758 \uc911\uc2ec \uc124\uc815\uc740 \ud53c\ud0c0\uace0\ub77c\uc2a4, \ud504\ud1a8\ub808\ubbf8 \ub4f1 \ub9ce\uc740 \ucf00\ud50c\ub7ec \uc774\uc804\uc758 \ucc9c\ubb38\ud559\uc790\ub4e4\uc774 \uc5f0\uad6c\ud588\ub358 \u3008\ucc9c\uccb4\uc758 \uc74c\uc545\u3009\uc774\ub098 \u3008\ucc9c\uc0c1\uc758 \uc74c\uc545\u3009 \uc774\uc5c8\ub294\ub370, \u300a\uc138\uacc4\uc758 \uc870\ud654\u300b\uc758 \ucd9c\ud310 \uc9c1\ud6c4\uc5d0 \ucf00\ud50c\ub7ec\ub294 \ube44\uc2b7\ud55c \ub54c\uc5d0 \ub3c5\uc790\uc801\uc73c\ub85c \uc870\ud654 \uc774\ub860\uc744 \ucd9c\ud310\ud588\ub358 \ub85c\ubc84\ud2b8 \ud50c\ub8e8\ub4dc \uc640 \uc120\ucde8\uad8c \ub17c\uc7c1\uc5d0 \ud729\uc2f8\uc600\ub2e4. ", "paragraph_answer": "\ucf00\ud50c\ub7ec\ub294 \uc5b4\uba38\ub2c8\uac00 \ub9c8\ub140\ub85c \ubab0\ub9b0 \uc5b4\ucc98\uad6c\ub2c8\uc5c6\ub294 \uc0ac\uac74\uc744 \ud574\uacb0\ud558\uae30 \uc704\ud574 \uc774\ub9ac\uc800\ub9ac \ub2e4\ub2c8\ub358 \ub3c4\uc911 \uae30\ubd84 \uc804\ud658\uc6a9\uc73c\ub85c \ube48\uc13c\uc870 \uac08\ub9b4\ub808\uc774 \uc758 \u300a\uace0\ub300\uc640 \ud604\ub300 \uc74c\uc545\uacfc\uc758 \ub300\ud654\u300b\ub77c\ub294 \ucc45\uc744 \ub4e4\uace0 \ub2e4\ub154\ub2e4. \uc5ec\uae30\uc11c \ube48\uc13c\uc870\ub294 \ud53c\ud0c0\uace0\ub77c\uc2a4\uc758 \uc870\ud654\uc5d0 \ub300\ud55c \uc774\ub860\uc744 \uc639\ud638\ud588\ub2e4. \ucf00\ud50c\ub7ec\ub294 \u2018\uc870\ud654\u2019\ub77c\uace0 \ud558\ub294 \uc138\uacc4\uc758 \uc218\ud559\uc801 \uaddc\uce59\uc744 \ucc3e\uc73c\ub824 \ud588\uace0, \uae30\ud558\ud559\uc5d0 \uac10\uba85\uc744 \ubc1b\uc544 \u201c\uae30\ud558\ud559\uc801 \ubb3c\uccb4\uac00 \uc804 \uc6b0\uc8fc\uc758 \ubaa8\ud615\uc744 \ub5a0\uc624\ub974\uac8c \ud558\uc600\ub2e4\u201d\uace0 \ud588\ub2e4. \u300a\uc138\uacc4\uc758 \uc870\ud654\u300b\uc5d0\uc11c \ucf00\ud50c\ub7ec\ub294 \uc790\uc5f0\uacc4, \ud2b9\ud788 \ucc9c\ubb38\ud559\uc801, \uc810\uc131\ud559\uc801\uc778 \uba74\uc5d0\uc11c\uc758 \uc790\uc5f0\uacc4\uc758 \uc870\ud654\ub97c \uc74c\uc545\uc801 \uad00\uc810\uc73c\ub85c \uc124\uba85\ud558\ub824\uace0 \uc2dc\ub3c4\ud588\ub2e4. \u300a\uc138\uacc4\uc758 \uc870\ud654\u300b\uc758 \uc911\uc2ec \uc124\uc815\uc740 \ud53c\ud0c0\uace0\ub77c\uc2a4, \ud504\ud1a8\ub808\ubbf8 \ub4f1 \ub9ce\uc740 \ucf00\ud50c\ub7ec \uc774\uc804\uc758 \ucc9c\ubb38\ud559\uc790\ub4e4\uc774 \uc5f0\uad6c\ud588\ub358 \u3008\ucc9c\uccb4\uc758 \uc74c\uc545\u3009\uc774\ub098 \u3008\ucc9c\uc0c1\uc758 \uc74c\uc545\u3009 \uc774\uc5c8\ub294\ub370, \u300a\uc138\uacc4\uc758 \uc870\ud654\u300b\uc758 \ucd9c\ud310 \uc9c1\ud6c4\uc5d0 \ucf00\ud50c\ub7ec\ub294 \ube44\uc2b7\ud55c \ub54c\uc5d0 \ub3c5\uc790\uc801\uc73c\ub85c \uc870\ud654 \uc774\ub860\uc744 \ucd9c\ud310\ud588\ub358 \ub85c\ubc84\ud2b8 \ud50c\ub8e8\ub4dc \uc640 \uc120\ucde8\uad8c \ub17c\uc7c1\uc5d0 \ud729\uc2f8\uc600\ub2e4.", "sentence_answer": "\u300a\uc138\uacc4\uc758 \uc870\ud654\u300b\uc758 \uc911\uc2ec \uc124\uc815\uc740 \ud53c\ud0c0\uace0\ub77c\uc2a4, \ud504\ud1a8\ub808\ubbf8 \ub4f1 \ub9ce\uc740 \ucf00\ud50c\ub7ec \uc774\uc804\uc758 \ucc9c\ubb38\ud559\uc790\ub4e4\uc774 \uc5f0\uad6c\ud588\ub358 \u3008\ucc9c\uccb4\uc758 \uc74c\uc545\u3009\uc774\ub098 \u3008\ucc9c\uc0c1\uc758 \uc74c\uc545\u3009 \uc774\uc5c8\ub294\ub370, \u300a\uc138\uacc4\uc758 \uc870\ud654\u300b\uc758 \ucd9c\ud310 \uc9c1\ud6c4\uc5d0 \ucf00\ud50c\ub7ec\ub294 \ube44\uc2b7\ud55c \ub54c\uc5d0 \ub3c5\uc790\uc801\uc73c\ub85c \uc870\ud654 \uc774\ub860\uc744 \ucd9c\ud310\ud588\ub358 \ub85c\ubc84\ud2b8 \ud50c\ub8e8\ub4dc \uc640 \uc120\ucde8\uad8c \ub17c\uc7c1\uc5d0 \ud729\uc2f8\uc600\ub2e4.", "paragraph_id": "6384301-29-1", "paragraph_question": "question: \uc138\uacc4\uc758\uc870\ud654 \ucd9c\ud310 \uc9c1\ud6c4 \uce90\ud50c\ub7ec\ub208 \ub204\uad6c\uc640 \uc120\ucde8\uad8c \ub17c\uc7c1\uc5d0 \ud729\uc2f8\uc600\ub294\uac00?, context: \ucf00\ud50c\ub7ec\ub294 \uc5b4\uba38\ub2c8\uac00 \ub9c8\ub140\ub85c \ubab0\ub9b0 \uc5b4\ucc98\uad6c\ub2c8\uc5c6\ub294 \uc0ac\uac74\uc744 \ud574\uacb0\ud558\uae30 \uc704\ud574 \uc774\ub9ac\uc800\ub9ac \ub2e4\ub2c8\ub358 \ub3c4\uc911 \uae30\ubd84 \uc804\ud658\uc6a9\uc73c\ub85c \ube48\uc13c\uc870 \uac08\ub9b4\ub808\uc774 \uc758 \u300a\uace0\ub300\uc640 \ud604\ub300 \uc74c\uc545\uacfc\uc758 \ub300\ud654\u300b\ub77c\ub294 \ucc45\uc744 \ub4e4\uace0 \ub2e4\ub154\ub2e4. \uc5ec\uae30\uc11c \ube48\uc13c\uc870\ub294 \ud53c\ud0c0\uace0\ub77c\uc2a4\uc758 \uc870\ud654\uc5d0 \ub300\ud55c \uc774\ub860\uc744 \uc639\ud638\ud588\ub2e4. \ucf00\ud50c\ub7ec\ub294 \u2018\uc870\ud654\u2019\ub77c\uace0 \ud558\ub294 \uc138\uacc4\uc758 \uc218\ud559\uc801 \uaddc\uce59\uc744 \ucc3e\uc73c\ub824 \ud588\uace0, \uae30\ud558\ud559\uc5d0 \uac10\uba85\uc744 \ubc1b\uc544 \u201c\uae30\ud558\ud559\uc801 \ubb3c\uccb4\uac00 \uc804 \uc6b0\uc8fc\uc758 \ubaa8\ud615\uc744 \ub5a0\uc624\ub974\uac8c \ud558\uc600\ub2e4\u201d\uace0 \ud588\ub2e4. \u300a\uc138\uacc4\uc758 \uc870\ud654\u300b\uc5d0\uc11c \ucf00\ud50c\ub7ec\ub294 \uc790\uc5f0\uacc4, \ud2b9\ud788 \ucc9c\ubb38\ud559\uc801, \uc810\uc131\ud559\uc801\uc778 \uba74\uc5d0\uc11c\uc758 \uc790\uc5f0\uacc4\uc758 \uc870\ud654\ub97c \uc74c\uc545\uc801 \uad00\uc810\uc73c\ub85c \uc124\uba85\ud558\ub824\uace0 \uc2dc\ub3c4\ud588\ub2e4. \u300a\uc138\uacc4\uc758 \uc870\ud654\u300b\uc758 \uc911\uc2ec \uc124\uc815\uc740 \ud53c\ud0c0\uace0\ub77c\uc2a4, \ud504\ud1a8\ub808\ubbf8 \ub4f1 \ub9ce\uc740 \ucf00\ud50c\ub7ec \uc774\uc804\uc758 \ucc9c\ubb38\ud559\uc790\ub4e4\uc774 \uc5f0\uad6c\ud588\ub358 \u3008\ucc9c\uccb4\uc758 \uc74c\uc545\u3009\uc774\ub098 \u3008\ucc9c\uc0c1\uc758 \uc74c\uc545\u3009 \uc774\uc5c8\ub294\ub370, \u300a\uc138\uacc4\uc758 \uc870\ud654\u300b\uc758 \ucd9c\ud310 \uc9c1\ud6c4\uc5d0 \ucf00\ud50c\ub7ec\ub294 \ube44\uc2b7\ud55c \ub54c\uc5d0 \ub3c5\uc790\uc801\uc73c\ub85c \uc870\ud654 \uc774\ub860\uc744 \ucd9c\ud310\ud588\ub358 \ub85c\ubc84\ud2b8 \ud50c\ub8e8\ub4dc\uc640 \uc120\ucde8\uad8c \ub17c\uc7c1\uc5d0 \ud729\uc2f8\uc600\ub2e4."} {"question": "\uc568\ub7f0 \ubb34\uc5b4\uc758 \uc6d0\ub798 \uc774\uc57c\uae30\uc5d0\uc11c \ube44\ud310\ud55c \uac83\uc740 \ubb34\uc5c7\uc778\uac00?", "paragraph": "\uc601\ud654\uc640 \uc6d0\uc791 \uc0ac\uc774\uc5d0\ub294 \uc5ec\ub7ec\uac00\uc9c0 \uc911\uc694\ud55c \ucc28\uc774\uac00 \uc788\ub2e4. \uc608\ub97c \ub4e4\uc5b4, \uc6d0\uc791 \uadf8\ub798\ud53d \ub178\ube14\uc758 \ubc30\uacbd\uc740 90\ub144\ub300\uc774\uc9c0\ub9cc \uc601\ud654\uc758 \ubc30\uacbd\uc740 \uac00\uae4c\uc6b4 \ubbf8\ub798(\uc544\ub9c8\ub3c4 2028\ub144\uacfc 2038\ub144 \uc0ac\uc774)\uc774\ub2e4. \uc568\ub7f0 \ubb34\uc5b4\uc758 \uc6d0\ub798 \uc774\uc57c\uae30\ub294 80\ub144\ub300\uc758 \uc601\uad6d\uc758 \ub300\ucc98\ub9ac\uc998\uc5d0 \ub300\ud55c \ud1b5\ub82c\ud55c \ube44\ud310\uc774\uc5c8\uace0, \ud30c\uc2dc\uc998\uacfc \ubb34\uc815\ubd80\uc8fc\uc758 \uac04\uc758 \ud22c\uc7c1\uc774 \uc8fc\uc694\ud55c \ud14c\ub9c8\uc600\ub2e4. \uadf8\ub7ec\ub098 \uc601\ud654\uc5d0\uc11c\uc758 \uac08\ub4f1\uc740 \uc6cc\uc1fc\uc2a4\ud0a4 \ud615\uc81c\uc5d0 \uc758\ud574 \ud604\ub300\uc758 \uc815\uce58 \uc0c1\ud669\uc5d0 \uc5b4\uc6b8\ub9ac\ub3c4\ub85d \uc218\uc815\ub418\uc5c8\uace0, \uadf8 \ubc14\ub78c\uc5d0 \uc6d0\uc791\uc758 \ubb34\uc815\ubd80\uc8fc\uc758-\ud30c\uc2dc\uc998 \ub300\uacb0 \uad6c\ub3c4\ub97c \ubc84\ub9ac\uace0 \uc790\uc720\uc8fc\uc758\uc640 \uc2e0\ubcf4\uc218\uc8fc\uc758\uac04\uc758, \ubbf8\uad6d\uc801\uc778 \ucda9\ub3cc\ub85c \uc774\uc57c\uae30\uac00 \ubc14\ub00c\uc5b4 \ubc84\ub838\ub294\ub370, \uc568\ub7f0 \ubb34\uc5b4\ub294 \uc774\ub97c \ube44\ud310\ud588\ub2e4. \ubb34\uc5b4\ub294 \u201c\ub0b4\uac00 \ubd88 \uc218 \uc788\ub294 \ud55c, \uc601\ud654 \uc548\uc5d0\ub294 \ubb34\uc815\ubd80\uc5d0 \ub300\ud55c \uc5b8\uae09\uc774 \uc5c6\ub2e4. \ud30c\uc2dc\uc998\uc740 \uc644\ubcbd\ud558\uac8c \uac70\uc138\ub418\uc5c8\ub2e4. \uadf8\ub7ec\ub2c8\uae4c \ub0b4 \ub9d0\uc740, \uc778\uc885\uc801 \uc21c\uc218\uc131\uc5d0 \ub300\ud55c \uac83\ub4e4\uc774 \ubaa8\ub450 \uc0ad\uc81c\ub418\uc5c8\ub2e4. \uadf8\ub7ec\ub098 \ud30c\uc2dc\uc2a4\ud2b8\uc5d0\uac8c \uc778\uc885\uc801 \uc21c\uc218\uc131\uc740 \uc911\ub300\ud55c \uc77c\uc774\ub2e4.\u201d \ub77c\uace0 \ub9d0\ud588\ub2e4. \ub354\uad70\ub2e4\ub098, \uc6d0\uc791\uc5d0\uc11c \ubb34\uc5b4\uac00 \ub3c4\ub355\uc801 \ubaa8\ud638\uc131\uc744 \uc720\uc9c0\uc2dc\ud0a4\uace0 \ud30c\uc2dc\uc2a4\ud2b8\ub4e4\uc744 \ud48d\uc790\ud654\ud558\uc9c0 \uc54a\uc740 \uac83\uacfc \ub2ec\ub9ac \uc2e4\uc0ac \uc601\ud654\uc5d0\uc11c\uc758 \ud30c\uc2dc\uc2a4\ud2b8\ub4e4\uc740 \ud3c9\uba74\uc801\uc778 \uce90\ub9ad\ud130\ub4e4\uc774\ub2e4. \uc601\ud654\uc758 \uc2dc\uac04 \uc81c\ud55c\uc740 \uc774\uc57c\uae30\ub97c \uc0dd\ub7b5\ud558\uac70\ub098 \uc77c\ubd80 \uce90\ub9ad\ud130, \ub514\ud14c\uc77c, \uc6d0\uc791\uc758 \ud50c\ub86f\ub4e4\uc744 \ub2a5\ub960\uc801\uc73c\ub85c \ud574\uc57c \ud55c\ub2e4\ub294 \uac83\uc744 \uc758\ubbf8\ud55c\ub2e4. \ud2b9\ud788, \uc6d0\uc791 \ub9cc\ud654\uc5d0\uc11c \ub178\uc2a4\ud30c\uc774\uc5b4\uc758 \uc9d1\uad8c\uc740 \ub300\uc911\ub4e4\uc758 \ubb34\uad00\uc2ec\uc73c\ub85c \uc778\ud55c \ud569\ubc95\uc801 \uc808\ucc28\uc600\uc9c0\ub9cc, \uc601\ud654\uc5d0\uc11c\ub294 \ub178\uc2a4\ud30c\uc774\uc5b4\uac00 \uc0dd\ud654\ud559 \ubb34\uae30 \u2018\uc138\uc778\ud2b8 \uba54\ub9ac \ubc14\uc774\ub7ec\uc2a4\u2019\ub97c \ub0a8\ubab0\ub798 \ub9cc\ub4e4\uace0 \uc0b4\ud3ec\ud574 \uad8c\ub825\uc744 \uc5bb\uac8c \ub41c \uac83\uc73c\ub85c \ubb18\uc0ac\ud588\ub2e4.", "answer": "\uc601\uad6d\uc758 \ub300\ucc98\ub9ac\uc998", "sentence": "\uc568\ub7f0 \ubb34\uc5b4\uc758 \uc6d0\ub798 \uc774\uc57c\uae30\ub294 80\ub144\ub300\uc758 \uc601\uad6d\uc758 \ub300\ucc98\ub9ac\uc998 \uc5d0 \ub300\ud55c \ud1b5\ub82c\ud55c \ube44\ud310\uc774\uc5c8\uace0, \ud30c\uc2dc\uc998\uacfc \ubb34\uc815\ubd80\uc8fc\uc758 \uac04\uc758 \ud22c\uc7c1\uc774 \uc8fc\uc694\ud55c \ud14c\ub9c8\uc600\ub2e4.", "paragraph_sentence": "\uc601\ud654\uc640 \uc6d0\uc791 \uc0ac\uc774\uc5d0\ub294 \uc5ec\ub7ec\uac00\uc9c0 \uc911\uc694\ud55c \ucc28\uc774\uac00 \uc788\ub2e4. \uc608\ub97c \ub4e4\uc5b4, \uc6d0\uc791 \uadf8\ub798\ud53d \ub178\ube14\uc758 \ubc30\uacbd\uc740 90\ub144\ub300\uc774\uc9c0\ub9cc \uc601\ud654\uc758 \ubc30\uacbd\uc740 \uac00\uae4c\uc6b4 \ubbf8\ub798(\uc544\ub9c8\ub3c4 2028\ub144\uacfc 2038\ub144 \uc0ac\uc774)\uc774\ub2e4. \uc568\ub7f0 \ubb34\uc5b4\uc758 \uc6d0\ub798 \uc774\uc57c\uae30\ub294 80\ub144\ub300\uc758 \uc601\uad6d\uc758 \ub300\ucc98\ub9ac\uc998 \uc5d0 \ub300\ud55c \ud1b5\ub82c\ud55c \ube44\ud310\uc774\uc5c8\uace0, \ud30c\uc2dc\uc998\uacfc \ubb34\uc815\ubd80\uc8fc\uc758 \uac04\uc758 \ud22c\uc7c1\uc774 \uc8fc\uc694\ud55c \ud14c\ub9c8\uc600\ub2e4. \uadf8\ub7ec\ub098 \uc601\ud654\uc5d0\uc11c\uc758 \uac08\ub4f1\uc740 \uc6cc\uc1fc\uc2a4\ud0a4 \ud615\uc81c\uc5d0 \uc758\ud574 \ud604\ub300\uc758 \uc815\uce58 \uc0c1\ud669\uc5d0 \uc5b4\uc6b8\ub9ac\ub3c4\ub85d \uc218\uc815\ub418\uc5c8\uace0, \uadf8 \ubc14\ub78c\uc5d0 \uc6d0\uc791\uc758 \ubb34\uc815\ubd80\uc8fc\uc758-\ud30c\uc2dc\uc998 \ub300\uacb0 \uad6c\ub3c4\ub97c \ubc84\ub9ac\uace0 \uc790\uc720\uc8fc\uc758\uc640 \uc2e0\ubcf4\uc218\uc8fc\uc758\uac04\uc758, \ubbf8\uad6d\uc801\uc778 \ucda9\ub3cc\ub85c \uc774\uc57c\uae30\uac00 \ubc14\ub00c\uc5b4 \ubc84\ub838\ub294\ub370, \uc568\ub7f0 \ubb34\uc5b4\ub294 \uc774\ub97c \ube44\ud310\ud588\ub2e4. \ubb34\uc5b4\ub294 \u201c\ub0b4\uac00 \ubd88 \uc218 \uc788\ub294 \ud55c, \uc601\ud654 \uc548\uc5d0\ub294 \ubb34\uc815\ubd80\uc5d0 \ub300\ud55c \uc5b8\uae09\uc774 \uc5c6\ub2e4. \ud30c\uc2dc\uc998\uc740 \uc644\ubcbd\ud558\uac8c \uac70\uc138\ub418\uc5c8\ub2e4. \uadf8\ub7ec\ub2c8\uae4c \ub0b4 \ub9d0\uc740, \uc778\uc885\uc801 \uc21c\uc218\uc131\uc5d0 \ub300\ud55c \uac83\ub4e4\uc774 \ubaa8\ub450 \uc0ad\uc81c\ub418\uc5c8\ub2e4. \uadf8\ub7ec\ub098 \ud30c\uc2dc\uc2a4\ud2b8\uc5d0\uac8c \uc778\uc885\uc801 \uc21c\uc218\uc131\uc740 \uc911\ub300\ud55c \uc77c\uc774\ub2e4. \u201d \ub77c\uace0 \ub9d0\ud588\ub2e4. \ub354\uad70\ub2e4\ub098, \uc6d0\uc791\uc5d0\uc11c \ubb34\uc5b4\uac00 \ub3c4\ub355\uc801 \ubaa8\ud638\uc131\uc744 \uc720\uc9c0\uc2dc\ud0a4\uace0 \ud30c\uc2dc\uc2a4\ud2b8\ub4e4\uc744 \ud48d\uc790\ud654\ud558\uc9c0 \uc54a\uc740 \uac83\uacfc \ub2ec\ub9ac \uc2e4\uc0ac \uc601\ud654\uc5d0\uc11c\uc758 \ud30c\uc2dc\uc2a4\ud2b8\ub4e4\uc740 \ud3c9\uba74\uc801\uc778 \uce90\ub9ad\ud130\ub4e4\uc774\ub2e4. \uc601\ud654\uc758 \uc2dc\uac04 \uc81c\ud55c\uc740 \uc774\uc57c\uae30\ub97c \uc0dd\ub7b5\ud558\uac70\ub098 \uc77c\ubd80 \uce90\ub9ad\ud130, \ub514\ud14c\uc77c, \uc6d0\uc791\uc758 \ud50c\ub86f\ub4e4\uc744 \ub2a5\ub960\uc801\uc73c\ub85c \ud574\uc57c \ud55c\ub2e4\ub294 \uac83\uc744 \uc758\ubbf8\ud55c\ub2e4. \ud2b9\ud788, \uc6d0\uc791 \ub9cc\ud654\uc5d0\uc11c \ub178\uc2a4\ud30c\uc774\uc5b4\uc758 \uc9d1\uad8c\uc740 \ub300\uc911\ub4e4\uc758 \ubb34\uad00\uc2ec\uc73c\ub85c \uc778\ud55c \ud569\ubc95\uc801 \uc808\ucc28\uc600\uc9c0\ub9cc, \uc601\ud654\uc5d0\uc11c\ub294 \ub178\uc2a4\ud30c\uc774\uc5b4\uac00 \uc0dd\ud654\ud559 \ubb34\uae30 \u2018\uc138\uc778\ud2b8 \uba54\ub9ac \ubc14\uc774\ub7ec\uc2a4\u2019\ub97c \ub0a8\ubab0\ub798 \ub9cc\ub4e4\uace0 \uc0b4\ud3ec\ud574 \uad8c\ub825\uc744 \uc5bb\uac8c \ub41c \uac83\uc73c\ub85c \ubb18\uc0ac\ud588\ub2e4.", "paragraph_answer": "\uc601\ud654\uc640 \uc6d0\uc791 \uc0ac\uc774\uc5d0\ub294 \uc5ec\ub7ec\uac00\uc9c0 \uc911\uc694\ud55c \ucc28\uc774\uac00 \uc788\ub2e4. \uc608\ub97c \ub4e4\uc5b4, \uc6d0\uc791 \uadf8\ub798\ud53d \ub178\ube14\uc758 \ubc30\uacbd\uc740 90\ub144\ub300\uc774\uc9c0\ub9cc \uc601\ud654\uc758 \ubc30\uacbd\uc740 \uac00\uae4c\uc6b4 \ubbf8\ub798(\uc544\ub9c8\ub3c4 2028\ub144\uacfc 2038\ub144 \uc0ac\uc774)\uc774\ub2e4. \uc568\ub7f0 \ubb34\uc5b4\uc758 \uc6d0\ub798 \uc774\uc57c\uae30\ub294 80\ub144\ub300\uc758 \uc601\uad6d\uc758 \ub300\ucc98\ub9ac\uc998 \uc5d0 \ub300\ud55c \ud1b5\ub82c\ud55c \ube44\ud310\uc774\uc5c8\uace0, \ud30c\uc2dc\uc998\uacfc \ubb34\uc815\ubd80\uc8fc\uc758 \uac04\uc758 \ud22c\uc7c1\uc774 \uc8fc\uc694\ud55c \ud14c\ub9c8\uc600\ub2e4. \uadf8\ub7ec\ub098 \uc601\ud654\uc5d0\uc11c\uc758 \uac08\ub4f1\uc740 \uc6cc\uc1fc\uc2a4\ud0a4 \ud615\uc81c\uc5d0 \uc758\ud574 \ud604\ub300\uc758 \uc815\uce58 \uc0c1\ud669\uc5d0 \uc5b4\uc6b8\ub9ac\ub3c4\ub85d \uc218\uc815\ub418\uc5c8\uace0, \uadf8 \ubc14\ub78c\uc5d0 \uc6d0\uc791\uc758 \ubb34\uc815\ubd80\uc8fc\uc758-\ud30c\uc2dc\uc998 \ub300\uacb0 \uad6c\ub3c4\ub97c \ubc84\ub9ac\uace0 \uc790\uc720\uc8fc\uc758\uc640 \uc2e0\ubcf4\uc218\uc8fc\uc758\uac04\uc758, \ubbf8\uad6d\uc801\uc778 \ucda9\ub3cc\ub85c \uc774\uc57c\uae30\uac00 \ubc14\ub00c\uc5b4 \ubc84\ub838\ub294\ub370, \uc568\ub7f0 \ubb34\uc5b4\ub294 \uc774\ub97c \ube44\ud310\ud588\ub2e4. \ubb34\uc5b4\ub294 \u201c\ub0b4\uac00 \ubd88 \uc218 \uc788\ub294 \ud55c, \uc601\ud654 \uc548\uc5d0\ub294 \ubb34\uc815\ubd80\uc5d0 \ub300\ud55c \uc5b8\uae09\uc774 \uc5c6\ub2e4. \ud30c\uc2dc\uc998\uc740 \uc644\ubcbd\ud558\uac8c \uac70\uc138\ub418\uc5c8\ub2e4. \uadf8\ub7ec\ub2c8\uae4c \ub0b4 \ub9d0\uc740, \uc778\uc885\uc801 \uc21c\uc218\uc131\uc5d0 \ub300\ud55c \uac83\ub4e4\uc774 \ubaa8\ub450 \uc0ad\uc81c\ub418\uc5c8\ub2e4. \uadf8\ub7ec\ub098 \ud30c\uc2dc\uc2a4\ud2b8\uc5d0\uac8c \uc778\uc885\uc801 \uc21c\uc218\uc131\uc740 \uc911\ub300\ud55c \uc77c\uc774\ub2e4.\u201d \ub77c\uace0 \ub9d0\ud588\ub2e4. \ub354\uad70\ub2e4\ub098, \uc6d0\uc791\uc5d0\uc11c \ubb34\uc5b4\uac00 \ub3c4\ub355\uc801 \ubaa8\ud638\uc131\uc744 \uc720\uc9c0\uc2dc\ud0a4\uace0 \ud30c\uc2dc\uc2a4\ud2b8\ub4e4\uc744 \ud48d\uc790\ud654\ud558\uc9c0 \uc54a\uc740 \uac83\uacfc \ub2ec\ub9ac \uc2e4\uc0ac \uc601\ud654\uc5d0\uc11c\uc758 \ud30c\uc2dc\uc2a4\ud2b8\ub4e4\uc740 \ud3c9\uba74\uc801\uc778 \uce90\ub9ad\ud130\ub4e4\uc774\ub2e4. \uc601\ud654\uc758 \uc2dc\uac04 \uc81c\ud55c\uc740 \uc774\uc57c\uae30\ub97c \uc0dd\ub7b5\ud558\uac70\ub098 \uc77c\ubd80 \uce90\ub9ad\ud130, \ub514\ud14c\uc77c, \uc6d0\uc791\uc758 \ud50c\ub86f\ub4e4\uc744 \ub2a5\ub960\uc801\uc73c\ub85c \ud574\uc57c \ud55c\ub2e4\ub294 \uac83\uc744 \uc758\ubbf8\ud55c\ub2e4. \ud2b9\ud788, \uc6d0\uc791 \ub9cc\ud654\uc5d0\uc11c \ub178\uc2a4\ud30c\uc774\uc5b4\uc758 \uc9d1\uad8c\uc740 \ub300\uc911\ub4e4\uc758 \ubb34\uad00\uc2ec\uc73c\ub85c \uc778\ud55c \ud569\ubc95\uc801 \uc808\ucc28\uc600\uc9c0\ub9cc, \uc601\ud654\uc5d0\uc11c\ub294 \ub178\uc2a4\ud30c\uc774\uc5b4\uac00 \uc0dd\ud654\ud559 \ubb34\uae30 \u2018\uc138\uc778\ud2b8 \uba54\ub9ac \ubc14\uc774\ub7ec\uc2a4\u2019\ub97c \ub0a8\ubab0\ub798 \ub9cc\ub4e4\uace0 \uc0b4\ud3ec\ud574 \uad8c\ub825\uc744 \uc5bb\uac8c \ub41c \uac83\uc73c\ub85c \ubb18\uc0ac\ud588\ub2e4.", "sentence_answer": "\uc568\ub7f0 \ubb34\uc5b4\uc758 \uc6d0\ub798 \uc774\uc57c\uae30\ub294 80\ub144\ub300\uc758 \uc601\uad6d\uc758 \ub300\ucc98\ub9ac\uc998 \uc5d0 \ub300\ud55c \ud1b5\ub82c\ud55c \ube44\ud310\uc774\uc5c8\uace0, \ud30c\uc2dc\uc998\uacfc \ubb34\uc815\ubd80\uc8fc\uc758 \uac04\uc758 \ud22c\uc7c1\uc774 \uc8fc\uc694\ud55c \ud14c\ub9c8\uc600\ub2e4.", "paragraph_id": "6516998-19-0", "paragraph_question": "question: \uc568\ub7f0 \ubb34\uc5b4\uc758 \uc6d0\ub798 \uc774\uc57c\uae30\uc5d0\uc11c \ube44\ud310\ud55c \uac83\uc740 \ubb34\uc5c7\uc778\uac00?, context: \uc601\ud654\uc640 \uc6d0\uc791 \uc0ac\uc774\uc5d0\ub294 \uc5ec\ub7ec\uac00\uc9c0 \uc911\uc694\ud55c \ucc28\uc774\uac00 \uc788\ub2e4. \uc608\ub97c \ub4e4\uc5b4, \uc6d0\uc791 \uadf8\ub798\ud53d \ub178\ube14\uc758 \ubc30\uacbd\uc740 90\ub144\ub300\uc774\uc9c0\ub9cc \uc601\ud654\uc758 \ubc30\uacbd\uc740 \uac00\uae4c\uc6b4 \ubbf8\ub798(\uc544\ub9c8\ub3c4 2028\ub144\uacfc 2038\ub144 \uc0ac\uc774)\uc774\ub2e4. \uc568\ub7f0 \ubb34\uc5b4\uc758 \uc6d0\ub798 \uc774\uc57c\uae30\ub294 80\ub144\ub300\uc758 \uc601\uad6d\uc758 \ub300\ucc98\ub9ac\uc998\uc5d0 \ub300\ud55c \ud1b5\ub82c\ud55c \ube44\ud310\uc774\uc5c8\uace0, \ud30c\uc2dc\uc998\uacfc \ubb34\uc815\ubd80\uc8fc\uc758 \uac04\uc758 \ud22c\uc7c1\uc774 \uc8fc\uc694\ud55c \ud14c\ub9c8\uc600\ub2e4. \uadf8\ub7ec\ub098 \uc601\ud654\uc5d0\uc11c\uc758 \uac08\ub4f1\uc740 \uc6cc\uc1fc\uc2a4\ud0a4 \ud615\uc81c\uc5d0 \uc758\ud574 \ud604\ub300\uc758 \uc815\uce58 \uc0c1\ud669\uc5d0 \uc5b4\uc6b8\ub9ac\ub3c4\ub85d \uc218\uc815\ub418\uc5c8\uace0, \uadf8 \ubc14\ub78c\uc5d0 \uc6d0\uc791\uc758 \ubb34\uc815\ubd80\uc8fc\uc758-\ud30c\uc2dc\uc998 \ub300\uacb0 \uad6c\ub3c4\ub97c \ubc84\ub9ac\uace0 \uc790\uc720\uc8fc\uc758\uc640 \uc2e0\ubcf4\uc218\uc8fc\uc758\uac04\uc758, \ubbf8\uad6d\uc801\uc778 \ucda9\ub3cc\ub85c \uc774\uc57c\uae30\uac00 \ubc14\ub00c\uc5b4 \ubc84\ub838\ub294\ub370, \uc568\ub7f0 \ubb34\uc5b4\ub294 \uc774\ub97c \ube44\ud310\ud588\ub2e4. \ubb34\uc5b4\ub294 \u201c\ub0b4\uac00 \ubd88 \uc218 \uc788\ub294 \ud55c, \uc601\ud654 \uc548\uc5d0\ub294 \ubb34\uc815\ubd80\uc5d0 \ub300\ud55c \uc5b8\uae09\uc774 \uc5c6\ub2e4. \ud30c\uc2dc\uc998\uc740 \uc644\ubcbd\ud558\uac8c \uac70\uc138\ub418\uc5c8\ub2e4. \uadf8\ub7ec\ub2c8\uae4c \ub0b4 \ub9d0\uc740, \uc778\uc885\uc801 \uc21c\uc218\uc131\uc5d0 \ub300\ud55c \uac83\ub4e4\uc774 \ubaa8\ub450 \uc0ad\uc81c\ub418\uc5c8\ub2e4. \uadf8\ub7ec\ub098 \ud30c\uc2dc\uc2a4\ud2b8\uc5d0\uac8c \uc778\uc885\uc801 \uc21c\uc218\uc131\uc740 \uc911\ub300\ud55c \uc77c\uc774\ub2e4.\u201d \ub77c\uace0 \ub9d0\ud588\ub2e4. \ub354\uad70\ub2e4\ub098, \uc6d0\uc791\uc5d0\uc11c \ubb34\uc5b4\uac00 \ub3c4\ub355\uc801 \ubaa8\ud638\uc131\uc744 \uc720\uc9c0\uc2dc\ud0a4\uace0 \ud30c\uc2dc\uc2a4\ud2b8\ub4e4\uc744 \ud48d\uc790\ud654\ud558\uc9c0 \uc54a\uc740 \uac83\uacfc \ub2ec\ub9ac \uc2e4\uc0ac \uc601\ud654\uc5d0\uc11c\uc758 \ud30c\uc2dc\uc2a4\ud2b8\ub4e4\uc740 \ud3c9\uba74\uc801\uc778 \uce90\ub9ad\ud130\ub4e4\uc774\ub2e4. \uc601\ud654\uc758 \uc2dc\uac04 \uc81c\ud55c\uc740 \uc774\uc57c\uae30\ub97c \uc0dd\ub7b5\ud558\uac70\ub098 \uc77c\ubd80 \uce90\ub9ad\ud130, \ub514\ud14c\uc77c, \uc6d0\uc791\uc758 \ud50c\ub86f\ub4e4\uc744 \ub2a5\ub960\uc801\uc73c\ub85c \ud574\uc57c \ud55c\ub2e4\ub294 \uac83\uc744 \uc758\ubbf8\ud55c\ub2e4. \ud2b9\ud788, \uc6d0\uc791 \ub9cc\ud654\uc5d0\uc11c \ub178\uc2a4\ud30c\uc774\uc5b4\uc758 \uc9d1\uad8c\uc740 \ub300\uc911\ub4e4\uc758 \ubb34\uad00\uc2ec\uc73c\ub85c \uc778\ud55c \ud569\ubc95\uc801 \uc808\ucc28\uc600\uc9c0\ub9cc, \uc601\ud654\uc5d0\uc11c\ub294 \ub178\uc2a4\ud30c\uc774\uc5b4\uac00 \uc0dd\ud654\ud559 \ubb34\uae30 \u2018\uc138\uc778\ud2b8 \uba54\ub9ac \ubc14\uc774\ub7ec\uc2a4\u2019\ub97c \ub0a8\ubab0\ub798 \ub9cc\ub4e4\uace0 \uc0b4\ud3ec\ud574 \uad8c\ub825\uc744 \uc5bb\uac8c \ub41c \uac83\uc73c\ub85c \ubb18\uc0ac\ud588\ub2e4."} {"question": "\ubd81\ud55c\uc774 \uc18c\ube44\uc5d0\ud2b8\uc2dd \uacc4\ud68d \uacbd\uc81c\ub97c \ud3ec\uae30\ud558\uace0 \ub098\uc11c\uc758 \uacbd\uc81c\ub294?", "paragraph": "\uc18c\ube44\uc5d0\ud2b8\uc2dd \uacc4\ud68d \uacbd\uc81c \uc0ac\ud68c\uc5d0\uc11c\ub294 \uc911\uc559 \uc815\ubd80\uc640 \uc9c0\uc5ed \uacf5\ub3d9\uccb4\uac00 \ubaa8\ub4e0 \uacbd\uc81c\uad8c\uc744 \uc950\uba70, \ubd84\ubc30, \uacf5\uae09, \uc218\uc694, \uc0c1\ud488 \uac00\uaca9 \ub4f1 \uc218\ub9ce\uc740 \uacbd\uc81c \uc694\uc18c\ub4e4\uc744 \uacc4\ud68d \ubc0f \uad00\ub9ac\ud55c\ub2e4. \uadf8\ub9ac\uace0 \uc774\ub7ec\ud55c \ud2b9\uc9d5 \ub54c\ubb38\uc5d0 \ud589\uc815\ud559\uc801, \uc0ac\ud68c\ud559\uc801 \uce21\uba74\ub3c4 \uace0\ub824\ud574\uc57c \ud558\uba70, \uc815\uce58\uacbd\uc81c\ud559\uacfc \uacb9\uccd0\uc9c8 \uc218 \uc788\ub2e4. \uc18c\uaddc\ubaa8 \uacbd\uc81c\uc5d0\uc11c \uc18c\ube44\uc5d0\ud2b8\uc2dd \uacc4\ud68d \uacbd\uc81c\ub294 \uacbd\uc81c \uc131\uc7a5 \ub610\ub294 \ubc1c\uc804\uc5d0\ub294 \ubd88\ud544\uc694\ud55c \ubbfc\uac04 \uac04\uc758 \uc774\ud574\uad00\uacc4\ub97c \uc81c\uc57d\ud558\uace0, \uacbd\uc81c \ubc1c\uc804\uc5d0 \ubab0\ub450\ud560 \uc218 \uc788\ub2e4\ub294 \uc810\uc5d0\uc11c \uc720\uc6a9\ud55c \uc0ac\ud68c\uc8fc\uc758\uc801 \uacbd\uc81c \uc774\ub860\uc774\uc9c0\ub9cc, \uacbd\uc81c \uaddc\ubaa8\uac00 \ucee4\uc9c8 \uc218\ub85d \uacbd\uc81c \uc131\uc7a5 \uce21\uba74\uc5d0\uc11c \ud55c\uacc4\uac00 \uc788\ub2e4\uace0 \uc804\ud574\uc9c0\uba70, \uc911\uc559 \ub2f9\uad6d\uc758 \ud310\ub2e8\ucc29\uc624\uac00 \uacb9\uccd0\uc9c0\uba74 \uadf8 \ud53c\ud574\ub294 \ub9c9\ub300\ud558\ub2e4. \ub3d9\uc2dc\uc5d0 \uc911\uc559 \uc815\ubd80 \uacbd\uc81c \uad00\ub8cc\ub4e4\uc5d0\uac8c \uacfc\ud55c \uc5c5\ubb34\ub7c9\uc744 \ubd80\ub2f4\ud558\uae30\ub3c4 \ud558\ub294 \uccb4\uc81c\uc774\ub2e4. \ubd81\ud55c\uc758 \uacbd\uc6b0\ub294 \uc18c\ube44\uc5d0\ud2b8\uc2dd \uacc4\ud68d \uacbd\uc81c\ub97c 90\ub144\ub300 \uc911\ubc18\uc5d0 \ud3ec\uae30\ud588\uc73c\uba70, \uacc4\ud68d-\uc9c0\ub839 \uacbd\uc81c \uc131\uaca9\uc744 \uac00\uc9c0\uac8c \ub418\uc5c8\ub2e4. \uc624\ub298\ub0a0 \ub9c8\ub974\ud06c\uc2a4-\ub808\ub2cc\uc8fc\uc758\uc790\ub4e4\uc740 \uc778\ud130\ub137 \uc2dc\uc2a4\ud15c\uc774 \ubc1c\ub2ec\ud558\uba74\uc11c \uc18c\ube44\uc5d0\ud2b8\uc2dd \uacc4\ud68d \uacbd\uc81c\uc758 21\uc138\uae30\ud654\ub97c \uc9c4\ud589\uc911\uc774\ub2e4. \ub300\ud55c\ubbfc\uad6d\uc5d0\uc11c\ub294 \ubc15\uc815\ud76c\uac00 \uc18c\ube44\uc5d0\ud2b8\uc2dd \uacc4\ud68d \uacbd\uc81c\ub97c \ud588\ub2e4\ub294 \uc8fc\uc7a5\ub3c4 \uc788\uc9c0\ub9cc, \uc18c\ube44\uc5d0\ud2b8\uc2dd \uacc4\ud68d \uacbd\uc81c\ub294 \uc784\uae08\uc744 '\uc9c1\uc811 \uc784\uae08'\uc758 \ud615\ud0dc\ubcf4\ub2e8, '\uc0ac\ud68c\uc801 \uc784\uae08' \uc758 \ud615\ud0dc\ub85c \uc9c0\uae09\ud558\uba70, \uac1c\ubcc4 \uc18c\ube44\uc758 \uc790\uc720\ub97c \uc77c\uc815 \ubd80\ubd84 \uc81c\ud55c\ud558\uace0, \uc0dd\uc0b0\uc218\ub2e8\uc744 \ubaa8\ub450 \uad6d\uc720\ud654\ud55c\ub2e4\ub294 \uc810\uc5d0\uc11c \ubc15\uc815\ud76c\uc758 \uc790\ubcf8\uc8fc\uc758\uc801 \uc9c0\ub839 \uacbd\uc81c\ub791\uc740 \ub2e4\ub978 \uac1c\ub150\uc774\ub2e4. \ud574\ub2f9 \uacbd\uc81c \uc815\ucc45\uc744 \uc2dc\ud589\ud558\ub294 \uad6d\uac00\ub294 \ucfe0\ubc14\uac00 \uc788\uc73c\uba70, \uacfc\uac70\uc5d0\ub294 \uc18c\ub828, \ubaa8\ud0dd\ub3d9 \uce58\ud558\uc758 \uc911\uad6d, \ubd81\ud55c \ub4f1 \uc218\ub9ce\uc740 \uacf5\uc0b0\uad8c\uc774 \uc2e4\ud589\ud588\uc5c8\ub2e4.", "answer": "\uacc4\ud68d-\uc9c0\ub839 \uacbd\uc81c", "sentence": "\ubd81\ud55c\uc758 \uacbd\uc6b0\ub294 \uc18c\ube44\uc5d0\ud2b8\uc2dd \uacc4\ud68d \uacbd\uc81c\ub97c 90\ub144\ub300 \uc911\ubc18\uc5d0 \ud3ec\uae30\ud588\uc73c\uba70, \uacc4\ud68d-\uc9c0\ub839 \uacbd\uc81c \uc131\uaca9\uc744 \uac00\uc9c0\uac8c \ub418\uc5c8\ub2e4.", "paragraph_sentence": "\uc18c\ube44\uc5d0\ud2b8\uc2dd \uacc4\ud68d \uacbd\uc81c \uc0ac\ud68c\uc5d0\uc11c\ub294 \uc911\uc559 \uc815\ubd80\uc640 \uc9c0\uc5ed \uacf5\ub3d9\uccb4\uac00 \ubaa8\ub4e0 \uacbd\uc81c\uad8c\uc744 \uc950\uba70, \ubd84\ubc30, \uacf5\uae09, \uc218\uc694, \uc0c1\ud488 \uac00\uaca9 \ub4f1 \uc218\ub9ce\uc740 \uacbd\uc81c \uc694\uc18c\ub4e4\uc744 \uacc4\ud68d \ubc0f \uad00\ub9ac\ud55c\ub2e4. \uadf8\ub9ac\uace0 \uc774\ub7ec\ud55c \ud2b9\uc9d5 \ub54c\ubb38\uc5d0 \ud589\uc815\ud559\uc801, \uc0ac\ud68c\ud559\uc801 \uce21\uba74\ub3c4 \uace0\ub824\ud574\uc57c \ud558\uba70, \uc815\uce58\uacbd\uc81c\ud559\uacfc \uacb9\uccd0\uc9c8 \uc218 \uc788\ub2e4. \uc18c\uaddc\ubaa8 \uacbd\uc81c\uc5d0\uc11c \uc18c\ube44\uc5d0\ud2b8\uc2dd \uacc4\ud68d \uacbd\uc81c\ub294 \uacbd\uc81c \uc131\uc7a5 \ub610\ub294 \ubc1c\uc804\uc5d0\ub294 \ubd88\ud544\uc694\ud55c \ubbfc\uac04 \uac04\uc758 \uc774\ud574\uad00\uacc4\ub97c \uc81c\uc57d\ud558\uace0, \uacbd\uc81c \ubc1c\uc804\uc5d0 \ubab0\ub450\ud560 \uc218 \uc788\ub2e4\ub294 \uc810\uc5d0\uc11c \uc720\uc6a9\ud55c \uc0ac\ud68c\uc8fc\uc758\uc801 \uacbd\uc81c \uc774\ub860\uc774\uc9c0\ub9cc, \uacbd\uc81c \uaddc\ubaa8\uac00 \ucee4\uc9c8 \uc218\ub85d \uacbd\uc81c \uc131\uc7a5 \uce21\uba74\uc5d0\uc11c \ud55c\uacc4\uac00 \uc788\ub2e4\uace0 \uc804\ud574\uc9c0\uba70, \uc911\uc559 \ub2f9\uad6d\uc758 \ud310\ub2e8\ucc29\uc624\uac00 \uacb9\uccd0\uc9c0\uba74 \uadf8 \ud53c\ud574\ub294 \ub9c9\ub300\ud558\ub2e4. \ub3d9\uc2dc\uc5d0 \uc911\uc559 \uc815\ubd80 \uacbd\uc81c \uad00\ub8cc\ub4e4\uc5d0\uac8c \uacfc\ud55c \uc5c5\ubb34\ub7c9\uc744 \ubd80\ub2f4\ud558\uae30\ub3c4 \ud558\ub294 \uccb4\uc81c\uc774\ub2e4. \ubd81\ud55c\uc758 \uacbd\uc6b0\ub294 \uc18c\ube44\uc5d0\ud2b8\uc2dd \uacc4\ud68d \uacbd\uc81c\ub97c 90\ub144\ub300 \uc911\ubc18\uc5d0 \ud3ec\uae30\ud588\uc73c\uba70, \uacc4\ud68d-\uc9c0\ub839 \uacbd\uc81c \uc131\uaca9\uc744 \uac00\uc9c0\uac8c \ub418\uc5c8\ub2e4. \uc624\ub298\ub0a0 \ub9c8\ub974\ud06c\uc2a4-\ub808\ub2cc\uc8fc\uc758\uc790\ub4e4\uc740 \uc778\ud130\ub137 \uc2dc\uc2a4\ud15c\uc774 \ubc1c\ub2ec\ud558\uba74\uc11c \uc18c\ube44\uc5d0\ud2b8\uc2dd \uacc4\ud68d \uacbd\uc81c\uc758 21\uc138\uae30\ud654\ub97c \uc9c4\ud589\uc911\uc774\ub2e4. \ub300\ud55c\ubbfc\uad6d\uc5d0\uc11c\ub294 \ubc15\uc815\ud76c\uac00 \uc18c\ube44\uc5d0\ud2b8\uc2dd \uacc4\ud68d \uacbd\uc81c\ub97c \ud588\ub2e4\ub294 \uc8fc\uc7a5\ub3c4 \uc788\uc9c0\ub9cc, \uc18c\ube44\uc5d0\ud2b8\uc2dd \uacc4\ud68d \uacbd\uc81c\ub294 \uc784\uae08\uc744 '\uc9c1\uc811 \uc784\uae08'\uc758 \ud615\ud0dc\ubcf4\ub2e8, '\uc0ac\ud68c\uc801 \uc784\uae08' \uc758 \ud615\ud0dc\ub85c \uc9c0\uae09\ud558\uba70, \uac1c\ubcc4 \uc18c\ube44\uc758 \uc790\uc720\ub97c \uc77c\uc815 \ubd80\ubd84 \uc81c\ud55c\ud558\uace0, \uc0dd\uc0b0\uc218\ub2e8\uc744 \ubaa8\ub450 \uad6d\uc720\ud654\ud55c\ub2e4\ub294 \uc810\uc5d0\uc11c \ubc15\uc815\ud76c\uc758 \uc790\ubcf8\uc8fc\uc758\uc801 \uc9c0\ub839 \uacbd\uc81c\ub791\uc740 \ub2e4\ub978 \uac1c\ub150\uc774\ub2e4. \ud574\ub2f9 \uacbd\uc81c \uc815\ucc45\uc744 \uc2dc\ud589\ud558\ub294 \uad6d\uac00\ub294 \ucfe0\ubc14\uac00 \uc788\uc73c\uba70, \uacfc\uac70\uc5d0\ub294 \uc18c\ub828, \ubaa8\ud0dd\ub3d9 \uce58\ud558\uc758 \uc911\uad6d, \ubd81\ud55c \ub4f1 \uc218\ub9ce\uc740 \uacf5\uc0b0\uad8c\uc774 \uc2e4\ud589\ud588\uc5c8\ub2e4.", "paragraph_answer": "\uc18c\ube44\uc5d0\ud2b8\uc2dd \uacc4\ud68d \uacbd\uc81c \uc0ac\ud68c\uc5d0\uc11c\ub294 \uc911\uc559 \uc815\ubd80\uc640 \uc9c0\uc5ed \uacf5\ub3d9\uccb4\uac00 \ubaa8\ub4e0 \uacbd\uc81c\uad8c\uc744 \uc950\uba70, \ubd84\ubc30, \uacf5\uae09, \uc218\uc694, \uc0c1\ud488 \uac00\uaca9 \ub4f1 \uc218\ub9ce\uc740 \uacbd\uc81c \uc694\uc18c\ub4e4\uc744 \uacc4\ud68d \ubc0f \uad00\ub9ac\ud55c\ub2e4. \uadf8\ub9ac\uace0 \uc774\ub7ec\ud55c \ud2b9\uc9d5 \ub54c\ubb38\uc5d0 \ud589\uc815\ud559\uc801, \uc0ac\ud68c\ud559\uc801 \uce21\uba74\ub3c4 \uace0\ub824\ud574\uc57c \ud558\uba70, \uc815\uce58\uacbd\uc81c\ud559\uacfc \uacb9\uccd0\uc9c8 \uc218 \uc788\ub2e4. \uc18c\uaddc\ubaa8 \uacbd\uc81c\uc5d0\uc11c \uc18c\ube44\uc5d0\ud2b8\uc2dd \uacc4\ud68d \uacbd\uc81c\ub294 \uacbd\uc81c \uc131\uc7a5 \ub610\ub294 \ubc1c\uc804\uc5d0\ub294 \ubd88\ud544\uc694\ud55c \ubbfc\uac04 \uac04\uc758 \uc774\ud574\uad00\uacc4\ub97c \uc81c\uc57d\ud558\uace0, \uacbd\uc81c \ubc1c\uc804\uc5d0 \ubab0\ub450\ud560 \uc218 \uc788\ub2e4\ub294 \uc810\uc5d0\uc11c \uc720\uc6a9\ud55c \uc0ac\ud68c\uc8fc\uc758\uc801 \uacbd\uc81c \uc774\ub860\uc774\uc9c0\ub9cc, \uacbd\uc81c \uaddc\ubaa8\uac00 \ucee4\uc9c8 \uc218\ub85d \uacbd\uc81c \uc131\uc7a5 \uce21\uba74\uc5d0\uc11c \ud55c\uacc4\uac00 \uc788\ub2e4\uace0 \uc804\ud574\uc9c0\uba70, \uc911\uc559 \ub2f9\uad6d\uc758 \ud310\ub2e8\ucc29\uc624\uac00 \uacb9\uccd0\uc9c0\uba74 \uadf8 \ud53c\ud574\ub294 \ub9c9\ub300\ud558\ub2e4. \ub3d9\uc2dc\uc5d0 \uc911\uc559 \uc815\ubd80 \uacbd\uc81c \uad00\ub8cc\ub4e4\uc5d0\uac8c \uacfc\ud55c \uc5c5\ubb34\ub7c9\uc744 \ubd80\ub2f4\ud558\uae30\ub3c4 \ud558\ub294 \uccb4\uc81c\uc774\ub2e4. \ubd81\ud55c\uc758 \uacbd\uc6b0\ub294 \uc18c\ube44\uc5d0\ud2b8\uc2dd \uacc4\ud68d \uacbd\uc81c\ub97c 90\ub144\ub300 \uc911\ubc18\uc5d0 \ud3ec\uae30\ud588\uc73c\uba70, \uacc4\ud68d-\uc9c0\ub839 \uacbd\uc81c \uc131\uaca9\uc744 \uac00\uc9c0\uac8c \ub418\uc5c8\ub2e4. \uc624\ub298\ub0a0 \ub9c8\ub974\ud06c\uc2a4-\ub808\ub2cc\uc8fc\uc758\uc790\ub4e4\uc740 \uc778\ud130\ub137 \uc2dc\uc2a4\ud15c\uc774 \ubc1c\ub2ec\ud558\uba74\uc11c \uc18c\ube44\uc5d0\ud2b8\uc2dd \uacc4\ud68d \uacbd\uc81c\uc758 21\uc138\uae30\ud654\ub97c \uc9c4\ud589\uc911\uc774\ub2e4. \ub300\ud55c\ubbfc\uad6d\uc5d0\uc11c\ub294 \ubc15\uc815\ud76c\uac00 \uc18c\ube44\uc5d0\ud2b8\uc2dd \uacc4\ud68d \uacbd\uc81c\ub97c \ud588\ub2e4\ub294 \uc8fc\uc7a5\ub3c4 \uc788\uc9c0\ub9cc, \uc18c\ube44\uc5d0\ud2b8\uc2dd \uacc4\ud68d \uacbd\uc81c\ub294 \uc784\uae08\uc744 '\uc9c1\uc811 \uc784\uae08'\uc758 \ud615\ud0dc\ubcf4\ub2e8, '\uc0ac\ud68c\uc801 \uc784\uae08' \uc758 \ud615\ud0dc\ub85c \uc9c0\uae09\ud558\uba70, \uac1c\ubcc4 \uc18c\ube44\uc758 \uc790\uc720\ub97c \uc77c\uc815 \ubd80\ubd84 \uc81c\ud55c\ud558\uace0, \uc0dd\uc0b0\uc218\ub2e8\uc744 \ubaa8\ub450 \uad6d\uc720\ud654\ud55c\ub2e4\ub294 \uc810\uc5d0\uc11c \ubc15\uc815\ud76c\uc758 \uc790\ubcf8\uc8fc\uc758\uc801 \uc9c0\ub839 \uacbd\uc81c\ub791\uc740 \ub2e4\ub978 \uac1c\ub150\uc774\ub2e4. \ud574\ub2f9 \uacbd\uc81c \uc815\ucc45\uc744 \uc2dc\ud589\ud558\ub294 \uad6d\uac00\ub294 \ucfe0\ubc14\uac00 \uc788\uc73c\uba70, \uacfc\uac70\uc5d0\ub294 \uc18c\ub828, \ubaa8\ud0dd\ub3d9 \uce58\ud558\uc758 \uc911\uad6d, \ubd81\ud55c \ub4f1 \uc218\ub9ce\uc740 \uacf5\uc0b0\uad8c\uc774 \uc2e4\ud589\ud588\uc5c8\ub2e4.", "sentence_answer": "\ubd81\ud55c\uc758 \uacbd\uc6b0\ub294 \uc18c\ube44\uc5d0\ud2b8\uc2dd \uacc4\ud68d \uacbd\uc81c\ub97c 90\ub144\ub300 \uc911\ubc18\uc5d0 \ud3ec\uae30\ud588\uc73c\uba70, \uacc4\ud68d-\uc9c0\ub839 \uacbd\uc81c \uc131\uaca9\uc744 \uac00\uc9c0\uac8c \ub418\uc5c8\ub2e4.", "paragraph_id": "6511026-10-0", "paragraph_question": "question: \ubd81\ud55c\uc774 \uc18c\ube44\uc5d0\ud2b8\uc2dd \uacc4\ud68d \uacbd\uc81c\ub97c \ud3ec\uae30\ud558\uace0 \ub098\uc11c\uc758 \uacbd\uc81c\ub294?, context: \uc18c\ube44\uc5d0\ud2b8\uc2dd \uacc4\ud68d \uacbd\uc81c \uc0ac\ud68c\uc5d0\uc11c\ub294 \uc911\uc559 \uc815\ubd80\uc640 \uc9c0\uc5ed \uacf5\ub3d9\uccb4\uac00 \ubaa8\ub4e0 \uacbd\uc81c\uad8c\uc744 \uc950\uba70, \ubd84\ubc30, \uacf5\uae09, \uc218\uc694, \uc0c1\ud488 \uac00\uaca9 \ub4f1 \uc218\ub9ce\uc740 \uacbd\uc81c \uc694\uc18c\ub4e4\uc744 \uacc4\ud68d \ubc0f \uad00\ub9ac\ud55c\ub2e4. \uadf8\ub9ac\uace0 \uc774\ub7ec\ud55c \ud2b9\uc9d5 \ub54c\ubb38\uc5d0 \ud589\uc815\ud559\uc801, \uc0ac\ud68c\ud559\uc801 \uce21\uba74\ub3c4 \uace0\ub824\ud574\uc57c \ud558\uba70, \uc815\uce58\uacbd\uc81c\ud559\uacfc \uacb9\uccd0\uc9c8 \uc218 \uc788\ub2e4. \uc18c\uaddc\ubaa8 \uacbd\uc81c\uc5d0\uc11c \uc18c\ube44\uc5d0\ud2b8\uc2dd \uacc4\ud68d \uacbd\uc81c\ub294 \uacbd\uc81c \uc131\uc7a5 \ub610\ub294 \ubc1c\uc804\uc5d0\ub294 \ubd88\ud544\uc694\ud55c \ubbfc\uac04 \uac04\uc758 \uc774\ud574\uad00\uacc4\ub97c \uc81c\uc57d\ud558\uace0, \uacbd\uc81c \ubc1c\uc804\uc5d0 \ubab0\ub450\ud560 \uc218 \uc788\ub2e4\ub294 \uc810\uc5d0\uc11c \uc720\uc6a9\ud55c \uc0ac\ud68c\uc8fc\uc758\uc801 \uacbd\uc81c \uc774\ub860\uc774\uc9c0\ub9cc, \uacbd\uc81c \uaddc\ubaa8\uac00 \ucee4\uc9c8 \uc218\ub85d \uacbd\uc81c \uc131\uc7a5 \uce21\uba74\uc5d0\uc11c \ud55c\uacc4\uac00 \uc788\ub2e4\uace0 \uc804\ud574\uc9c0\uba70, \uc911\uc559 \ub2f9\uad6d\uc758 \ud310\ub2e8\ucc29\uc624\uac00 \uacb9\uccd0\uc9c0\uba74 \uadf8 \ud53c\ud574\ub294 \ub9c9\ub300\ud558\ub2e4. \ub3d9\uc2dc\uc5d0 \uc911\uc559 \uc815\ubd80 \uacbd\uc81c \uad00\ub8cc\ub4e4\uc5d0\uac8c \uacfc\ud55c \uc5c5\ubb34\ub7c9\uc744 \ubd80\ub2f4\ud558\uae30\ub3c4 \ud558\ub294 \uccb4\uc81c\uc774\ub2e4. \ubd81\ud55c\uc758 \uacbd\uc6b0\ub294 \uc18c\ube44\uc5d0\ud2b8\uc2dd \uacc4\ud68d \uacbd\uc81c\ub97c 90\ub144\ub300 \uc911\ubc18\uc5d0 \ud3ec\uae30\ud588\uc73c\uba70, \uacc4\ud68d-\uc9c0\ub839 \uacbd\uc81c \uc131\uaca9\uc744 \uac00\uc9c0\uac8c \ub418\uc5c8\ub2e4. \uc624\ub298\ub0a0 \ub9c8\ub974\ud06c\uc2a4-\ub808\ub2cc\uc8fc\uc758\uc790\ub4e4\uc740 \uc778\ud130\ub137 \uc2dc\uc2a4\ud15c\uc774 \ubc1c\ub2ec\ud558\uba74\uc11c \uc18c\ube44\uc5d0\ud2b8\uc2dd \uacc4\ud68d \uacbd\uc81c\uc758 21\uc138\uae30\ud654\ub97c \uc9c4\ud589\uc911\uc774\ub2e4. \ub300\ud55c\ubbfc\uad6d\uc5d0\uc11c\ub294 \ubc15\uc815\ud76c\uac00 \uc18c\ube44\uc5d0\ud2b8\uc2dd \uacc4\ud68d \uacbd\uc81c\ub97c \ud588\ub2e4\ub294 \uc8fc\uc7a5\ub3c4 \uc788\uc9c0\ub9cc, \uc18c\ube44\uc5d0\ud2b8\uc2dd \uacc4\ud68d \uacbd\uc81c\ub294 \uc784\uae08\uc744 '\uc9c1\uc811 \uc784\uae08'\uc758 \ud615\ud0dc\ubcf4\ub2e8, '\uc0ac\ud68c\uc801 \uc784\uae08' \uc758 \ud615\ud0dc\ub85c \uc9c0\uae09\ud558\uba70, \uac1c\ubcc4 \uc18c\ube44\uc758 \uc790\uc720\ub97c \uc77c\uc815 \ubd80\ubd84 \uc81c\ud55c\ud558\uace0, \uc0dd\uc0b0\uc218\ub2e8\uc744 \ubaa8\ub450 \uad6d\uc720\ud654\ud55c\ub2e4\ub294 \uc810\uc5d0\uc11c \ubc15\uc815\ud76c\uc758 \uc790\ubcf8\uc8fc\uc758\uc801 \uc9c0\ub839 \uacbd\uc81c\ub791\uc740 \ub2e4\ub978 \uac1c\ub150\uc774\ub2e4. \ud574\ub2f9 \uacbd\uc81c \uc815\ucc45\uc744 \uc2dc\ud589\ud558\ub294 \uad6d\uac00\ub294 \ucfe0\ubc14\uac00 \uc788\uc73c\uba70, \uacfc\uac70\uc5d0\ub294 \uc18c\ub828, \ubaa8\ud0dd\ub3d9 \uce58\ud558\uc758 \uc911\uad6d, \ubd81\ud55c \ub4f1 \uc218\ub9ce\uc740 \uacf5\uc0b0\uad8c\uc774 \uc2e4\ud589\ud588\uc5c8\ub2e4."} {"question": "\ud64d\ucf69 \uc778\uad6c\uc5d0\uc11c \uac00\uc7a5 \ud070 \ube44\uc911\uc744 \ucc28\uc9c0\ud558\ub294 \ubbfc\uc871\uc740 \ubb34\uc5c7\uc778\uac00?", "paragraph": "2008\ub144 \ud64d\ucf69\uc758 \uc778\uad6c\ub294 \uc57d 6,985,200 \uba85\uc774\ub2e4. 1841\ub144\uc5d0\ub294 7,500 \uba85, 1931\ub144\uc5d0\ub294 849,800 \uba85, 1945\ub144\uc5d0\ub294 750,000 \uba85\uc774\uc5c8\ub2e4. \uc5f0\ub839\ubcc4\ub85c \uc870\uc0ac\ud558\uba74 0~14\uc138\uac00 18%, 15~64\uc138\uac00 71%, 65\uc138 \uc774\uc0c1\uc774 11%\uc774\uace0, \ud3c9\uade0 \uc5f0\ub839\uc740 34\uc138\uc774\ub2e4. \ud64d\ucf69\uc758 \uc778\uad6c \uac00\uc7a5 \ub9ce\uc740 \uc218\ub97c \ucc28\uc9c0\ud558\ub294 \ubbfc\uc871\uc740 \ud55c\uc871\uc73c\ub85c, \ud64d\ucf69 \uc778\uad6c\uc758 95%\ub97c \ucc28\uc9c0\ud55c\ub2e4. \ud55c\uc871 \uc774\uc678\uc5d0 \uac00\uc815\ubd80 \ub4f1\uc758 \uc77c\uc5d0 \uc885\uc0ac\ud558\uace0 \uc788\ub294 \uc678\uad6d\uc778 \ub178\ub3d9\uc790\ub85c\uc11c \ud544\ub9ac\ud540\uc778\uacfc \uc778\ub3c4\ub124\uc2dc\uc544\uc778\uc774 \uc788\uace0, \ub2e4\uc74c\uc774 \ubbf8\uad6d\uc778, \uadf8 \ub2e4\uc74c\uc73c\ub85c \uc2dd\ubbfc\uc9c0 \uc2dc\ub300\uc5d0 \uc774\uc8fc\ud55c \uc601\uad6d\uc778 \uc21c\uc774\ub2e4. \uc77c\ubcf8\uc778\ub3c4 \uc57d 2\ub9cc \uba85 \uac70\uc8fc\ud558\uace0 \uc788\ub2e4. \uadf8 \uc678\uc758 \ubbfc\uc871\ub3c4 \uac70\uc8fc\ud558\uba70, \ud55c\uad6d\uc778\uc758 \uc218\ub294 2011\ub144 \uae30\uc900\uc73c\ub85c \uad50\ubbfc\uacfc \uccb4\ub958\uc790 \ud3ec\ud568 \ucd1d 1\ub9cc 3\ucc9c \uba85\uc774\ub2e4.", "answer": "\ud55c\uc871", "sentence": "\ud64d\ucf69\uc758 \uc778\uad6c \uac00\uc7a5 \ub9ce\uc740 \uc218\ub97c \ucc28\uc9c0\ud558\ub294 \ubbfc\uc871\uc740 \ud55c\uc871 \uc73c\ub85c, \ud64d\ucf69 \uc778\uad6c\uc758 95%\ub97c \ucc28\uc9c0\ud55c\ub2e4.", "paragraph_sentence": "2008\ub144 \ud64d\ucf69\uc758 \uc778\uad6c\ub294 \uc57d 6,985,200 \uba85\uc774\ub2e4. 1841\ub144\uc5d0\ub294 7,500 \uba85, 1931\ub144\uc5d0\ub294 849,800 \uba85, 1945\ub144\uc5d0\ub294 750,000 \uba85\uc774\uc5c8\ub2e4. \uc5f0\ub839\ubcc4\ub85c \uc870\uc0ac\ud558\uba74 0~14\uc138\uac00 18%, 15~64\uc138\uac00 71%, 65\uc138 \uc774\uc0c1\uc774 11%\uc774\uace0, \ud3c9\uade0 \uc5f0\ub839\uc740 34\uc138\uc774\ub2e4. \ud64d\ucf69\uc758 \uc778\uad6c \uac00\uc7a5 \ub9ce\uc740 \uc218\ub97c \ucc28\uc9c0\ud558\ub294 \ubbfc\uc871\uc740 \ud55c\uc871 \uc73c\ub85c, \ud64d\ucf69 \uc778\uad6c\uc758 95%\ub97c \ucc28\uc9c0\ud55c\ub2e4. \ud55c\uc871 \uc774\uc678\uc5d0 \uac00\uc815\ubd80 \ub4f1\uc758 \uc77c\uc5d0 \uc885\uc0ac\ud558\uace0 \uc788\ub294 \uc678\uad6d\uc778 \ub178\ub3d9\uc790\ub85c\uc11c \ud544\ub9ac\ud540\uc778\uacfc \uc778\ub3c4\ub124\uc2dc\uc544\uc778\uc774 \uc788\uace0, \ub2e4\uc74c\uc774 \ubbf8\uad6d\uc778, \uadf8 \ub2e4\uc74c\uc73c\ub85c \uc2dd\ubbfc\uc9c0 \uc2dc\ub300\uc5d0 \uc774\uc8fc\ud55c \uc601\uad6d\uc778 \uc21c\uc774\ub2e4. \uc77c\ubcf8\uc778\ub3c4 \uc57d 2\ub9cc \uba85 \uac70\uc8fc\ud558\uace0 \uc788\ub2e4. \uadf8 \uc678\uc758 \ubbfc\uc871\ub3c4 \uac70\uc8fc\ud558\uba70, \ud55c\uad6d\uc778\uc758 \uc218\ub294 2011\ub144 \uae30\uc900\uc73c\ub85c \uad50\ubbfc\uacfc \uccb4\ub958\uc790 \ud3ec\ud568 \ucd1d 1\ub9cc 3\ucc9c \uba85\uc774\ub2e4.", "paragraph_answer": "2008\ub144 \ud64d\ucf69\uc758 \uc778\uad6c\ub294 \uc57d 6,985,200 \uba85\uc774\ub2e4. 1841\ub144\uc5d0\ub294 7,500 \uba85, 1931\ub144\uc5d0\ub294 849,800 \uba85, 1945\ub144\uc5d0\ub294 750,000 \uba85\uc774\uc5c8\ub2e4. \uc5f0\ub839\ubcc4\ub85c \uc870\uc0ac\ud558\uba74 0~14\uc138\uac00 18%, 15~64\uc138\uac00 71%, 65\uc138 \uc774\uc0c1\uc774 11%\uc774\uace0, \ud3c9\uade0 \uc5f0\ub839\uc740 34\uc138\uc774\ub2e4. \ud64d\ucf69\uc758 \uc778\uad6c \uac00\uc7a5 \ub9ce\uc740 \uc218\ub97c \ucc28\uc9c0\ud558\ub294 \ubbfc\uc871\uc740 \ud55c\uc871 \uc73c\ub85c, \ud64d\ucf69 \uc778\uad6c\uc758 95%\ub97c \ucc28\uc9c0\ud55c\ub2e4. \ud55c\uc871 \uc774\uc678\uc5d0 \uac00\uc815\ubd80 \ub4f1\uc758 \uc77c\uc5d0 \uc885\uc0ac\ud558\uace0 \uc788\ub294 \uc678\uad6d\uc778 \ub178\ub3d9\uc790\ub85c\uc11c \ud544\ub9ac\ud540\uc778\uacfc \uc778\ub3c4\ub124\uc2dc\uc544\uc778\uc774 \uc788\uace0, \ub2e4\uc74c\uc774 \ubbf8\uad6d\uc778, \uadf8 \ub2e4\uc74c\uc73c\ub85c \uc2dd\ubbfc\uc9c0 \uc2dc\ub300\uc5d0 \uc774\uc8fc\ud55c \uc601\uad6d\uc778 \uc21c\uc774\ub2e4. \uc77c\ubcf8\uc778\ub3c4 \uc57d 2\ub9cc \uba85 \uac70\uc8fc\ud558\uace0 \uc788\ub2e4. \uadf8 \uc678\uc758 \ubbfc\uc871\ub3c4 \uac70\uc8fc\ud558\uba70, \ud55c\uad6d\uc778\uc758 \uc218\ub294 2011\ub144 \uae30\uc900\uc73c\ub85c \uad50\ubbfc\uacfc \uccb4\ub958\uc790 \ud3ec\ud568 \ucd1d 1\ub9cc 3\ucc9c \uba85\uc774\ub2e4.", "sentence_answer": "\ud64d\ucf69\uc758 \uc778\uad6c \uac00\uc7a5 \ub9ce\uc740 \uc218\ub97c \ucc28\uc9c0\ud558\ub294 \ubbfc\uc871\uc740 \ud55c\uc871 \uc73c\ub85c, \ud64d\ucf69 \uc778\uad6c\uc758 95%\ub97c \ucc28\uc9c0\ud55c\ub2e4.", "paragraph_id": "6366535-15-1", "paragraph_question": "question: \ud64d\ucf69 \uc778\uad6c\uc5d0\uc11c \uac00\uc7a5 \ud070 \ube44\uc911\uc744 \ucc28\uc9c0\ud558\ub294 \ubbfc\uc871\uc740 \ubb34\uc5c7\uc778\uac00?, context: 2008\ub144 \ud64d\ucf69\uc758 \uc778\uad6c\ub294 \uc57d 6,985,200 \uba85\uc774\ub2e4. 1841\ub144\uc5d0\ub294 7,500 \uba85, 1931\ub144\uc5d0\ub294 849,800 \uba85, 1945\ub144\uc5d0\ub294 750,000 \uba85\uc774\uc5c8\ub2e4. \uc5f0\ub839\ubcc4\ub85c \uc870\uc0ac\ud558\uba74 0~14\uc138\uac00 18%, 15~64\uc138\uac00 71%, 65\uc138 \uc774\uc0c1\uc774 11%\uc774\uace0, \ud3c9\uade0 \uc5f0\ub839\uc740 34\uc138\uc774\ub2e4. \ud64d\ucf69\uc758 \uc778\uad6c \uac00\uc7a5 \ub9ce\uc740 \uc218\ub97c \ucc28\uc9c0\ud558\ub294 \ubbfc\uc871\uc740 \ud55c\uc871\uc73c\ub85c, \ud64d\ucf69 \uc778\uad6c\uc758 95%\ub97c \ucc28\uc9c0\ud55c\ub2e4. \ud55c\uc871 \uc774\uc678\uc5d0 \uac00\uc815\ubd80 \ub4f1\uc758 \uc77c\uc5d0 \uc885\uc0ac\ud558\uace0 \uc788\ub294 \uc678\uad6d\uc778 \ub178\ub3d9\uc790\ub85c\uc11c \ud544\ub9ac\ud540\uc778\uacfc \uc778\ub3c4\ub124\uc2dc\uc544\uc778\uc774 \uc788\uace0, \ub2e4\uc74c\uc774 \ubbf8\uad6d\uc778, \uadf8 \ub2e4\uc74c\uc73c\ub85c \uc2dd\ubbfc\uc9c0 \uc2dc\ub300\uc5d0 \uc774\uc8fc\ud55c \uc601\uad6d\uc778 \uc21c\uc774\ub2e4. \uc77c\ubcf8\uc778\ub3c4 \uc57d 2\ub9cc \uba85 \uac70\uc8fc\ud558\uace0 \uc788\ub2e4. \uadf8 \uc678\uc758 \ubbfc\uc871\ub3c4 \uac70\uc8fc\ud558\uba70, \ud55c\uad6d\uc778\uc758 \uc218\ub294 2011\ub144 \uae30\uc900\uc73c\ub85c \uad50\ubbfc\uacfc \uccb4\ub958\uc790 \ud3ec\ud568 \ucd1d 1\ub9cc 3\ucc9c \uba85\uc774\ub2e4."} {"question": "\uce7c \ud14c\uc624\ub3c4\ub974 \ubca8\ucf00\ub294 \uba87 \ub144\ub3c4\uc5d0 \ud504\ub9ac\ub4dc\ub9ac\ud788 \ub2e4\ub2c8\uc5d8 \ubc14\uc11c\ub9cc\uacfc \ube44\uc2b7\ud55c \uc548\uac74\uc744 \uc81c\ucd9c\ud588\ub294\uac00?", "paragraph": "\ub3c5\uc77c \uacf5\ub3d9 \uad6d\ubbfc\uc758\ud68c \uc120\ucd9c\uc744 \ud5a5\ud55c \ub9c8\uc9c0\ub9c9 \ucd09\ub9e4\uc81c\ub294 \uc790\uc720\uc8fc\uc758\uc790\uc600\ub358 \uc758\uc6d0 \ud504\ub9ac\ub4dc\ub9ac\ud788 \ub2e4\ub2c8\uc5d8 \ubc14\uc11c\ub9cc(Friedrich Daniel Bassermann)\uc774 \uc81c\uacf5\ud588\ub2e4. \uce7c \ud14c\uc624\ub3c4\ub974 \ubca8\ucee4(Carl Theodor Welcker)\uc758 1831\ub144 \uc81c\uc548\uacfc \uc720\uc0ac\ud55c \uc548\uac74(\u2018\ubc14\uc11c\ub9cc \ubc95\uc548\u2019, Motion Bassermann)\uc744 1844\ub144 \uc81c\ucd9c\ud588\ub358 \uadf8\ub294 1848\ub144 2\uc6d4 12\uc77c \ubc14\ub374 \ub300\uacf5\uad6d \uc0c1\uc6d0\uc5d0\uc11c \uc774\ub97c \ub2e4\uc2dc \ub17c\uc758\ud558\uba74\uc11c \uad6d\ubbfc\uc774 \ubf51\uc740 \ub300\ud45c\ub97c \ud504\ub791\ud06c\ud478\ub974\ud2b8\uc554\ub9c8\uc778\uc758 \uc5f0\ubc29\uc758\ud68c\uc5d0 \ub450\uace0, \ub3c5\uc77c \uc81c\ud6c4\ub4e4\uc774 \ubf51\uc740 \uc758\uc6d0\ub4e4\uacfc \ud568\uaed8 \uc804\uccb4 \ub3c5\uc77c\uc744 \ub300\ud45c\ud558\ub294 \uc720\uc77c\ud55c \uae30\uad00\uc744 \uc124\ub9bd\ud560 \uac83\uc744 \uc694\uad6c\ud588\ub2e4. \uc774 \uc8fc \ud6c4 \ud504\ub791\uc2a4\uc5d0\uc11c \uc131\uacf5\uc801 \uc804\ubcf5 \uc18c\uc2dd\uc774 \uc804\ud574\uc9c0\uc790 \ud601\uba85\uc801 \ubd84\uc704\uae30\ub294 \ub354\uc6b1 \ubb34\ub974\uc775\uc5c8\ub2e4. \ub3c5\uc77c \ub545\uc5d0\uc11c\uc758 \ud601\uba85\uc740 \ub9e8 \ucc98\uc74c \ubc14\ub374\uc5d0\uc11c \uce74\ub97c\uc2a4\ub8e8\uc5d0\uc5d0 \uc788\ub294 \uc758\uc0ac\ub2f9\uc744 \uc810\ub839\ud558\uba74\uc11c \uc2dc\uc791\ub418\uc5c8\ub2e4. 4\uc6d4\uc5d0\ub294 \ubc14\ub374 \ub300\uacf5\uad6d\uc758 3\uac1c\uc758 \ud601\uba85\uc801 \ubd09\uae30 \uc911 \uccab \ubd09\uae30\uc600\ub358 \ud5e4\ucee4\uc758 \ub09c(Heckerzug)\uc774 \uc774\uc5b4\uc84c\ub2e4. \uadf8 \ud6c4 \uba70\uce60, \uba87 \uc8fc \uc548\uc5d0 \ub3c5\uc77c\uc758 \ub098\uba38\uc9c0 \uc81c\ud6c4\uad6d\ub4e4\ub85c\ub3c4 \ubd09\uae30\uac00 \ud37c\uc84c\ub2e4.", "answer": "1831\ub144", "sentence": "\uce7c \ud14c\uc624\ub3c4\ub974 \ubca8\ucee4(Carl Theodor Welcker)\uc758 1831\ub144 \uc81c\uc548\uacfc \uc720\uc0ac\ud55c \uc548\uac74(\u2018\ubc14\uc11c\ub9cc \ubc95\uc548\u2019, Motion Bassermann)\uc744 1844\ub144 \uc81c\ucd9c\ud588\ub358 \uadf8\ub294 1848\ub144 2\uc6d4 12\uc77c \ubc14\ub374 \ub300\uacf5\uad6d \uc0c1\uc6d0\uc5d0\uc11c \uc774\ub97c \ub2e4\uc2dc \ub17c\uc758\ud558\uba74\uc11c \uad6d\ubbfc\uc774 \ubf51\uc740 \ub300\ud45c\ub97c \ud504\ub791\ud06c\ud478\ub974\ud2b8\uc554\ub9c8\uc778\uc758 \uc5f0\ubc29\uc758\ud68c\uc5d0 \ub450\uace0, \ub3c5\uc77c \uc81c\ud6c4\ub4e4\uc774 \ubf51\uc740 \uc758\uc6d0\ub4e4\uacfc \ud568\uaed8 \uc804\uccb4 \ub3c5\uc77c\uc744 \ub300\ud45c\ud558\ub294 \uc720\uc77c\ud55c \uae30\uad00\uc744 \uc124\ub9bd\ud560 \uac83\uc744 \uc694\uad6c\ud588\ub2e4.", "paragraph_sentence": "\ub3c5\uc77c \uacf5\ub3d9 \uad6d\ubbfc\uc758\ud68c \uc120\ucd9c\uc744 \ud5a5\ud55c \ub9c8\uc9c0\ub9c9 \ucd09\ub9e4\uc81c\ub294 \uc790\uc720\uc8fc\uc758\uc790\uc600\ub358 \uc758\uc6d0 \ud504\ub9ac\ub4dc\ub9ac\ud788 \ub2e4\ub2c8\uc5d8 \ubc14\uc11c\ub9cc(Friedrich Daniel Bassermann)\uc774 \uc81c\uacf5\ud588\ub2e4. \uce7c \ud14c\uc624\ub3c4\ub974 \ubca8\ucee4(Carl Theodor Welcker)\uc758 1831\ub144 \uc81c\uc548\uacfc \uc720\uc0ac\ud55c \uc548\uac74(\u2018\ubc14\uc11c\ub9cc \ubc95\uc548\u2019, Motion Bassermann)\uc744 1844\ub144 \uc81c\ucd9c\ud588\ub358 \uadf8\ub294 1848\ub144 2\uc6d4 12\uc77c \ubc14\ub374 \ub300\uacf5\uad6d \uc0c1\uc6d0\uc5d0\uc11c \uc774\ub97c \ub2e4\uc2dc \ub17c\uc758\ud558\uba74\uc11c \uad6d\ubbfc\uc774 \ubf51\uc740 \ub300\ud45c\ub97c \ud504\ub791\ud06c\ud478\ub974\ud2b8\uc554\ub9c8\uc778\uc758 \uc5f0\ubc29\uc758\ud68c\uc5d0 \ub450\uace0, \ub3c5\uc77c \uc81c\ud6c4\ub4e4\uc774 \ubf51\uc740 \uc758\uc6d0\ub4e4\uacfc \ud568\uaed8 \uc804\uccb4 \ub3c5\uc77c\uc744 \ub300\ud45c\ud558\ub294 \uc720\uc77c\ud55c \uae30\uad00\uc744 \uc124\ub9bd\ud560 \uac83\uc744 \uc694\uad6c\ud588\ub2e4. \uc774 \uc8fc \ud6c4 \ud504\ub791\uc2a4\uc5d0\uc11c \uc131\uacf5\uc801 \uc804\ubcf5 \uc18c\uc2dd\uc774 \uc804\ud574\uc9c0\uc790 \ud601\uba85\uc801 \ubd84\uc704\uae30\ub294 \ub354\uc6b1 \ubb34\ub974\uc775\uc5c8\ub2e4. \ub3c5\uc77c \ub545\uc5d0\uc11c\uc758 \ud601\uba85\uc740 \ub9e8 \ucc98\uc74c \ubc14\ub374\uc5d0\uc11c \uce74\ub97c\uc2a4\ub8e8\uc5d0\uc5d0 \uc788\ub294 \uc758\uc0ac\ub2f9\uc744 \uc810\ub839\ud558\uba74\uc11c \uc2dc\uc791\ub418\uc5c8\ub2e4. 4\uc6d4\uc5d0\ub294 \ubc14\ub374 \ub300\uacf5\uad6d\uc758 3\uac1c\uc758 \ud601\uba85\uc801 \ubd09\uae30 \uc911 \uccab \ubd09\uae30\uc600\ub358 \ud5e4\ucee4\uc758 \ub09c(Heckerzug)\uc774 \uc774\uc5b4\uc84c\ub2e4. \uadf8 \ud6c4 \uba70\uce60, \uba87 \uc8fc \uc548\uc5d0 \ub3c5\uc77c\uc758 \ub098\uba38\uc9c0 \uc81c\ud6c4\uad6d\ub4e4\ub85c\ub3c4 \ubd09\uae30\uac00 \ud37c\uc84c\ub2e4.", "paragraph_answer": "\ub3c5\uc77c \uacf5\ub3d9 \uad6d\ubbfc\uc758\ud68c \uc120\ucd9c\uc744 \ud5a5\ud55c \ub9c8\uc9c0\ub9c9 \ucd09\ub9e4\uc81c\ub294 \uc790\uc720\uc8fc\uc758\uc790\uc600\ub358 \uc758\uc6d0 \ud504\ub9ac\ub4dc\ub9ac\ud788 \ub2e4\ub2c8\uc5d8 \ubc14\uc11c\ub9cc(Friedrich Daniel Bassermann)\uc774 \uc81c\uacf5\ud588\ub2e4. \uce7c \ud14c\uc624\ub3c4\ub974 \ubca8\ucee4(Carl Theodor Welcker)\uc758 1831\ub144 \uc81c\uc548\uacfc \uc720\uc0ac\ud55c \uc548\uac74(\u2018\ubc14\uc11c\ub9cc \ubc95\uc548\u2019, Motion Bassermann)\uc744 1844\ub144 \uc81c\ucd9c\ud588\ub358 \uadf8\ub294 1848\ub144 2\uc6d4 12\uc77c \ubc14\ub374 \ub300\uacf5\uad6d \uc0c1\uc6d0\uc5d0\uc11c \uc774\ub97c \ub2e4\uc2dc \ub17c\uc758\ud558\uba74\uc11c \uad6d\ubbfc\uc774 \ubf51\uc740 \ub300\ud45c\ub97c \ud504\ub791\ud06c\ud478\ub974\ud2b8\uc554\ub9c8\uc778\uc758 \uc5f0\ubc29\uc758\ud68c\uc5d0 \ub450\uace0, \ub3c5\uc77c \uc81c\ud6c4\ub4e4\uc774 \ubf51\uc740 \uc758\uc6d0\ub4e4\uacfc \ud568\uaed8 \uc804\uccb4 \ub3c5\uc77c\uc744 \ub300\ud45c\ud558\ub294 \uc720\uc77c\ud55c \uae30\uad00\uc744 \uc124\ub9bd\ud560 \uac83\uc744 \uc694\uad6c\ud588\ub2e4. \uc774 \uc8fc \ud6c4 \ud504\ub791\uc2a4\uc5d0\uc11c \uc131\uacf5\uc801 \uc804\ubcf5 \uc18c\uc2dd\uc774 \uc804\ud574\uc9c0\uc790 \ud601\uba85\uc801 \ubd84\uc704\uae30\ub294 \ub354\uc6b1 \ubb34\ub974\uc775\uc5c8\ub2e4. \ub3c5\uc77c \ub545\uc5d0\uc11c\uc758 \ud601\uba85\uc740 \ub9e8 \ucc98\uc74c \ubc14\ub374\uc5d0\uc11c \uce74\ub97c\uc2a4\ub8e8\uc5d0\uc5d0 \uc788\ub294 \uc758\uc0ac\ub2f9\uc744 \uc810\ub839\ud558\uba74\uc11c \uc2dc\uc791\ub418\uc5c8\ub2e4. 4\uc6d4\uc5d0\ub294 \ubc14\ub374 \ub300\uacf5\uad6d\uc758 3\uac1c\uc758 \ud601\uba85\uc801 \ubd09\uae30 \uc911 \uccab \ubd09\uae30\uc600\ub358 \ud5e4\ucee4\uc758 \ub09c(Heckerzug)\uc774 \uc774\uc5b4\uc84c\ub2e4. \uadf8 \ud6c4 \uba70\uce60, \uba87 \uc8fc \uc548\uc5d0 \ub3c5\uc77c\uc758 \ub098\uba38\uc9c0 \uc81c\ud6c4\uad6d\ub4e4\ub85c\ub3c4 \ubd09\uae30\uac00 \ud37c\uc84c\ub2e4.", "sentence_answer": "\uce7c \ud14c\uc624\ub3c4\ub974 \ubca8\ucee4(Carl Theodor Welcker)\uc758 1831\ub144 \uc81c\uc548\uacfc \uc720\uc0ac\ud55c \uc548\uac74(\u2018\ubc14\uc11c\ub9cc \ubc95\uc548\u2019, Motion Bassermann)\uc744 1844\ub144 \uc81c\ucd9c\ud588\ub358 \uadf8\ub294 1848\ub144 2\uc6d4 12\uc77c \ubc14\ub374 \ub300\uacf5\uad6d \uc0c1\uc6d0\uc5d0\uc11c \uc774\ub97c \ub2e4\uc2dc \ub17c\uc758\ud558\uba74\uc11c \uad6d\ubbfc\uc774 \ubf51\uc740 \ub300\ud45c\ub97c \ud504\ub791\ud06c\ud478\ub974\ud2b8\uc554\ub9c8\uc778\uc758 \uc5f0\ubc29\uc758\ud68c\uc5d0 \ub450\uace0, \ub3c5\uc77c \uc81c\ud6c4\ub4e4\uc774 \ubf51\uc740 \uc758\uc6d0\ub4e4\uacfc \ud568\uaed8 \uc804\uccb4 \ub3c5\uc77c\uc744 \ub300\ud45c\ud558\ub294 \uc720\uc77c\ud55c \uae30\uad00\uc744 \uc124\ub9bd\ud560 \uac83\uc744 \uc694\uad6c\ud588\ub2e4.", "paragraph_id": "6568666-6-0", "paragraph_question": "question: \uce7c \ud14c\uc624\ub3c4\ub974 \ubca8\ucf00\ub294 \uba87 \ub144\ub3c4\uc5d0 \ud504\ub9ac\ub4dc\ub9ac\ud788 \ub2e4\ub2c8\uc5d8 \ubc14\uc11c\ub9cc\uacfc \ube44\uc2b7\ud55c \uc548\uac74\uc744 \uc81c\ucd9c\ud588\ub294\uac00?, context: \ub3c5\uc77c \uacf5\ub3d9 \uad6d\ubbfc\uc758\ud68c \uc120\ucd9c\uc744 \ud5a5\ud55c \ub9c8\uc9c0\ub9c9 \ucd09\ub9e4\uc81c\ub294 \uc790\uc720\uc8fc\uc758\uc790\uc600\ub358 \uc758\uc6d0 \ud504\ub9ac\ub4dc\ub9ac\ud788 \ub2e4\ub2c8\uc5d8 \ubc14\uc11c\ub9cc(Friedrich Daniel Bassermann)\uc774 \uc81c\uacf5\ud588\ub2e4. \uce7c \ud14c\uc624\ub3c4\ub974 \ubca8\ucee4(Carl Theodor Welcker)\uc758 1831\ub144 \uc81c\uc548\uacfc \uc720\uc0ac\ud55c \uc548\uac74(\u2018\ubc14\uc11c\ub9cc \ubc95\uc548\u2019, Motion Bassermann)\uc744 1844\ub144 \uc81c\ucd9c\ud588\ub358 \uadf8\ub294 1848\ub144 2\uc6d4 12\uc77c \ubc14\ub374 \ub300\uacf5\uad6d \uc0c1\uc6d0\uc5d0\uc11c \uc774\ub97c \ub2e4\uc2dc \ub17c\uc758\ud558\uba74\uc11c \uad6d\ubbfc\uc774 \ubf51\uc740 \ub300\ud45c\ub97c \ud504\ub791\ud06c\ud478\ub974\ud2b8\uc554\ub9c8\uc778\uc758 \uc5f0\ubc29\uc758\ud68c\uc5d0 \ub450\uace0, \ub3c5\uc77c \uc81c\ud6c4\ub4e4\uc774 \ubf51\uc740 \uc758\uc6d0\ub4e4\uacfc \ud568\uaed8 \uc804\uccb4 \ub3c5\uc77c\uc744 \ub300\ud45c\ud558\ub294 \uc720\uc77c\ud55c \uae30\uad00\uc744 \uc124\ub9bd\ud560 \uac83\uc744 \uc694\uad6c\ud588\ub2e4. \uc774 \uc8fc \ud6c4 \ud504\ub791\uc2a4\uc5d0\uc11c \uc131\uacf5\uc801 \uc804\ubcf5 \uc18c\uc2dd\uc774 \uc804\ud574\uc9c0\uc790 \ud601\uba85\uc801 \ubd84\uc704\uae30\ub294 \ub354\uc6b1 \ubb34\ub974\uc775\uc5c8\ub2e4. \ub3c5\uc77c \ub545\uc5d0\uc11c\uc758 \ud601\uba85\uc740 \ub9e8 \ucc98\uc74c \ubc14\ub374\uc5d0\uc11c \uce74\ub97c\uc2a4\ub8e8\uc5d0\uc5d0 \uc788\ub294 \uc758\uc0ac\ub2f9\uc744 \uc810\ub839\ud558\uba74\uc11c \uc2dc\uc791\ub418\uc5c8\ub2e4. 4\uc6d4\uc5d0\ub294 \ubc14\ub374 \ub300\uacf5\uad6d\uc758 3\uac1c\uc758 \ud601\uba85\uc801 \ubd09\uae30 \uc911 \uccab \ubd09\uae30\uc600\ub358 \ud5e4\ucee4\uc758 \ub09c(Heckerzug)\uc774 \uc774\uc5b4\uc84c\ub2e4. \uadf8 \ud6c4 \uba70\uce60, \uba87 \uc8fc \uc548\uc5d0 \ub3c5\uc77c\uc758 \ub098\uba38\uc9c0 \uc81c\ud6c4\uad6d\ub4e4\ub85c\ub3c4 \ubd09\uae30\uac00 \ud37c\uc84c\ub2e4."} {"question": "\ubbf8\uad6d \ub808\uc774\ud06c\ud50c\ub798\uc2dc\ub4dc \ub300\ud68c\ub294 \uba87\ub144\ub3c4\uc5d0 \uc5f4\ub838\ub294\uac00?", "paragraph": "1932\ub144 \ubbf8\uad6d \ub808\uc774\ud06c\ud50c\ub798\uc2dc\ub4dc \ub300\ud68c\ub294 \ucc98\uc74c\uc73c\ub85c \uc720\ub7fd \ubc16\uc758 \ub300\ub959\uc5d0\uc11c \uac1c\ucd5c\ud55c \ub300\ud68c\uc600\ub2e4. 17\uac1c\uad6d\uc5d0\uc11c \ucd1d 252\uba85\uc758 \uc120\uc218\uac00 \ucc38\uac00\ud588\ub294\ub370, \uc774\ub294 \ub2f9\uc2dc \ub300\uacf5\ud669\uc73c\ub85c \uc904\uc5b4\ub4e0 \uc2a4\ud3ec\uce20 \ud22c\uc790\uc640 \uac1c\ucd5c\uc9c0\uae4c\uc9c0\uc758 \uae34 \uc5ec\uc815\uc73c\ub85c \uc778\ud574 \uc9c0\ub09c \ub300\ud68c\ubcf4\ub2e4 \uc904\uc5b4\ub4e0 \uc22b\uc790\uc774\ub2e4. \uc120\uc218\ub4e4\uc740 \ucd1d 14\uac1c\uc758 \uc885\ubaa9\uc5d0 \ucc38\uac00\ud574 \uacbd\uc7c1\uc744 \ubc8c\uc600\ub2e4. 1932\ub144 \ub300\ud68c \ub610\ud55c \uc628\ud654\ud55c \ub0a0\uc528 \ub54c\ubb38\uc5d0 \ub300\ud68c\uac00 \uc2dc\uc791\ub418\uae30 2\uac1c\uc6d4 \uc804\uae4c\uc9c0 \uac70\uc758 \ub208\uc774 \ub0b4\ub9ac\uc9c0 \uc54a\ub2e4\uac00 1\uc6d4 \uc911\uc21c\uc774 \ub418\uc5b4\uc11c\uc57c \ub300\ud68c\ub97c \uac1c\ucd5c\ud560 \uc218 \uc788\uc744 \ub9cc\ud07c\uc758 \ub208\uc774 \uc313\uc600\ub2e4. \uc774\uc804 \ub300\ud68c\uc758 \ud53c\uaca8\uc2a4\ucf00\uc774\ud305 \ucc54\ud53c\uc5b8\uc778 \uc18c\ub0d0 \ud5e4\ub2c8\ub294 \uc62c\ub9bc\ud53d \ud0c0\uc774\ud2c0\uc744 \uc9c0\ucf1c\ub0b4\uc5c8\uace0, 1920\ub144 \uc62c\ub9bc\ud53d \ubcf5\uc2f1 \ucc54\ud53c\uc5b8\uc774\uc5c8\ub358 \uc5d0\ub514 \uc774\uac74\uc774 \ub0a8\uc790 4\uc778\uc2b9 \ubd05\uc2ac\ub808\uc774 \uacbd\uae30\uc5d0\uc11c \uae08\uba54\ub2ec\uc744 \ub530\ub0b4\uba74\uc11c, \uc62c\ub9bc\ud53d \uc5ed\uc0ac\ub97c \ud1b5\ud2c0\uc5b4 \uc720\uc77c\ud558\uac8c \ud558\uacc4\uc640 \ub3d9\uacc4 \uc62c\ub9bc\ud53d \uacbd\uae30 \ubaa8\ub450\uc5d0\uc11c \uae08\uba54\ub2ec\uc744 \ud68d\ub4dd\ud55c \uc120\uc218\uac00 \ub418\uc5c8\ub2e4.", "answer": "1932\ub144", "sentence": "1932\ub144 \ubbf8\uad6d \ub808\uc774\ud06c\ud50c\ub798\uc2dc\ub4dc \ub300\ud68c\ub294 \ucc98\uc74c\uc73c\ub85c \uc720\ub7fd \ubc16\uc758 \ub300\ub959\uc5d0\uc11c \uac1c\ucd5c\ud55c \ub300\ud68c\uc600\ub2e4.", "paragraph_sentence": " 1932\ub144 \ubbf8\uad6d \ub808\uc774\ud06c\ud50c\ub798\uc2dc\ub4dc \ub300\ud68c\ub294 \ucc98\uc74c\uc73c\ub85c \uc720\ub7fd \ubc16\uc758 \ub300\ub959\uc5d0\uc11c \uac1c\ucd5c\ud55c \ub300\ud68c\uc600\ub2e4. 17\uac1c\uad6d\uc5d0\uc11c \ucd1d 252\uba85\uc758 \uc120\uc218\uac00 \ucc38\uac00\ud588\ub294\ub370, \uc774\ub294 \ub2f9\uc2dc \ub300\uacf5\ud669\uc73c\ub85c \uc904\uc5b4\ub4e0 \uc2a4\ud3ec\uce20 \ud22c\uc790\uc640 \uac1c\ucd5c\uc9c0\uae4c\uc9c0\uc758 \uae34 \uc5ec\uc815\uc73c\ub85c \uc778\ud574 \uc9c0\ub09c \ub300\ud68c\ubcf4\ub2e4 \uc904\uc5b4\ub4e0 \uc22b\uc790\uc774\ub2e4. \uc120\uc218\ub4e4\uc740 \ucd1d 14\uac1c\uc758 \uc885\ubaa9\uc5d0 \ucc38\uac00\ud574 \uacbd\uc7c1\uc744 \ubc8c\uc600\ub2e4. 1932\ub144 \ub300\ud68c \ub610\ud55c \uc628\ud654\ud55c \ub0a0\uc528 \ub54c\ubb38\uc5d0 \ub300\ud68c\uac00 \uc2dc\uc791\ub418\uae30 2\uac1c\uc6d4 \uc804\uae4c\uc9c0 \uac70\uc758 \ub208\uc774 \ub0b4\ub9ac\uc9c0 \uc54a\ub2e4\uac00 1\uc6d4 \uc911\uc21c\uc774 \ub418\uc5b4\uc11c\uc57c \ub300\ud68c\ub97c \uac1c\ucd5c\ud560 \uc218 \uc788\uc744 \ub9cc\ud07c\uc758 \ub208\uc774 \uc313\uc600\ub2e4. \uc774\uc804 \ub300\ud68c\uc758 \ud53c\uaca8\uc2a4\ucf00\uc774\ud305 \ucc54\ud53c\uc5b8\uc778 \uc18c\ub0d0 \ud5e4\ub2c8\ub294 \uc62c\ub9bc\ud53d \ud0c0\uc774\ud2c0\uc744 \uc9c0\ucf1c\ub0b4\uc5c8\uace0, 1920\ub144 \uc62c\ub9bc\ud53d \ubcf5\uc2f1 \ucc54\ud53c\uc5b8\uc774\uc5c8\ub358 \uc5d0\ub514 \uc774\uac74\uc774 \ub0a8\uc790 4\uc778\uc2b9 \ubd05\uc2ac\ub808\uc774 \uacbd\uae30\uc5d0\uc11c \uae08\uba54\ub2ec\uc744 \ub530\ub0b4\uba74\uc11c, \uc62c\ub9bc\ud53d \uc5ed\uc0ac\ub97c \ud1b5\ud2c0\uc5b4 \uc720\uc77c\ud558\uac8c \ud558\uacc4\uc640 \ub3d9\uacc4 \uc62c\ub9bc\ud53d \uacbd\uae30 \ubaa8\ub450\uc5d0\uc11c \uae08\uba54\ub2ec\uc744 \ud68d\ub4dd\ud55c \uc120\uc218\uac00 \ub418\uc5c8\ub2e4.", "paragraph_answer": " 1932\ub144 \ubbf8\uad6d \ub808\uc774\ud06c\ud50c\ub798\uc2dc\ub4dc \ub300\ud68c\ub294 \ucc98\uc74c\uc73c\ub85c \uc720\ub7fd \ubc16\uc758 \ub300\ub959\uc5d0\uc11c \uac1c\ucd5c\ud55c \ub300\ud68c\uc600\ub2e4. 17\uac1c\uad6d\uc5d0\uc11c \ucd1d 252\uba85\uc758 \uc120\uc218\uac00 \ucc38\uac00\ud588\ub294\ub370, \uc774\ub294 \ub2f9\uc2dc \ub300\uacf5\ud669\uc73c\ub85c \uc904\uc5b4\ub4e0 \uc2a4\ud3ec\uce20 \ud22c\uc790\uc640 \uac1c\ucd5c\uc9c0\uae4c\uc9c0\uc758 \uae34 \uc5ec\uc815\uc73c\ub85c \uc778\ud574 \uc9c0\ub09c \ub300\ud68c\ubcf4\ub2e4 \uc904\uc5b4\ub4e0 \uc22b\uc790\uc774\ub2e4. \uc120\uc218\ub4e4\uc740 \ucd1d 14\uac1c\uc758 \uc885\ubaa9\uc5d0 \ucc38\uac00\ud574 \uacbd\uc7c1\uc744 \ubc8c\uc600\ub2e4. 1932\ub144 \ub300\ud68c \ub610\ud55c \uc628\ud654\ud55c \ub0a0\uc528 \ub54c\ubb38\uc5d0 \ub300\ud68c\uac00 \uc2dc\uc791\ub418\uae30 2\uac1c\uc6d4 \uc804\uae4c\uc9c0 \uac70\uc758 \ub208\uc774 \ub0b4\ub9ac\uc9c0 \uc54a\ub2e4\uac00 1\uc6d4 \uc911\uc21c\uc774 \ub418\uc5b4\uc11c\uc57c \ub300\ud68c\ub97c \uac1c\ucd5c\ud560 \uc218 \uc788\uc744 \ub9cc\ud07c\uc758 \ub208\uc774 \uc313\uc600\ub2e4. \uc774\uc804 \ub300\ud68c\uc758 \ud53c\uaca8\uc2a4\ucf00\uc774\ud305 \ucc54\ud53c\uc5b8\uc778 \uc18c\ub0d0 \ud5e4\ub2c8\ub294 \uc62c\ub9bc\ud53d \ud0c0\uc774\ud2c0\uc744 \uc9c0\ucf1c\ub0b4\uc5c8\uace0, 1920\ub144 \uc62c\ub9bc\ud53d \ubcf5\uc2f1 \ucc54\ud53c\uc5b8\uc774\uc5c8\ub358 \uc5d0\ub514 \uc774\uac74\uc774 \ub0a8\uc790 4\uc778\uc2b9 \ubd05\uc2ac\ub808\uc774 \uacbd\uae30\uc5d0\uc11c \uae08\uba54\ub2ec\uc744 \ub530\ub0b4\uba74\uc11c, \uc62c\ub9bc\ud53d \uc5ed\uc0ac\ub97c \ud1b5\ud2c0\uc5b4 \uc720\uc77c\ud558\uac8c \ud558\uacc4\uc640 \ub3d9\uacc4 \uc62c\ub9bc\ud53d \uacbd\uae30 \ubaa8\ub450\uc5d0\uc11c \uae08\uba54\ub2ec\uc744 \ud68d\ub4dd\ud55c \uc120\uc218\uac00 \ub418\uc5c8\ub2e4.", "sentence_answer": " 1932\ub144 \ubbf8\uad6d \ub808\uc774\ud06c\ud50c\ub798\uc2dc\ub4dc \ub300\ud68c\ub294 \ucc98\uc74c\uc73c\ub85c \uc720\ub7fd \ubc16\uc758 \ub300\ub959\uc5d0\uc11c \uac1c\ucd5c\ud55c \ub300\ud68c\uc600\ub2e4.", "paragraph_id": "6534018-2-1", "paragraph_question": "question: \ubbf8\uad6d \ub808\uc774\ud06c\ud50c\ub798\uc2dc\ub4dc \ub300\ud68c\ub294 \uba87\ub144\ub3c4\uc5d0 \uc5f4\ub838\ub294\uac00?, context: 1932\ub144 \ubbf8\uad6d \ub808\uc774\ud06c\ud50c\ub798\uc2dc\ub4dc \ub300\ud68c\ub294 \ucc98\uc74c\uc73c\ub85c \uc720\ub7fd \ubc16\uc758 \ub300\ub959\uc5d0\uc11c \uac1c\ucd5c\ud55c \ub300\ud68c\uc600\ub2e4. 17\uac1c\uad6d\uc5d0\uc11c \ucd1d 252\uba85\uc758 \uc120\uc218\uac00 \ucc38\uac00\ud588\ub294\ub370, \uc774\ub294 \ub2f9\uc2dc \ub300\uacf5\ud669\uc73c\ub85c \uc904\uc5b4\ub4e0 \uc2a4\ud3ec\uce20 \ud22c\uc790\uc640 \uac1c\ucd5c\uc9c0\uae4c\uc9c0\uc758 \uae34 \uc5ec\uc815\uc73c\ub85c \uc778\ud574 \uc9c0\ub09c \ub300\ud68c\ubcf4\ub2e4 \uc904\uc5b4\ub4e0 \uc22b\uc790\uc774\ub2e4. \uc120\uc218\ub4e4\uc740 \ucd1d 14\uac1c\uc758 \uc885\ubaa9\uc5d0 \ucc38\uac00\ud574 \uacbd\uc7c1\uc744 \ubc8c\uc600\ub2e4. 1932\ub144 \ub300\ud68c \ub610\ud55c \uc628\ud654\ud55c \ub0a0\uc528 \ub54c\ubb38\uc5d0 \ub300\ud68c\uac00 \uc2dc\uc791\ub418\uae30 2\uac1c\uc6d4 \uc804\uae4c\uc9c0 \uac70\uc758 \ub208\uc774 \ub0b4\ub9ac\uc9c0 \uc54a\ub2e4\uac00 1\uc6d4 \uc911\uc21c\uc774 \ub418\uc5b4\uc11c\uc57c \ub300\ud68c\ub97c \uac1c\ucd5c\ud560 \uc218 \uc788\uc744 \ub9cc\ud07c\uc758 \ub208\uc774 \uc313\uc600\ub2e4. \uc774\uc804 \ub300\ud68c\uc758 \ud53c\uaca8\uc2a4\ucf00\uc774\ud305 \ucc54\ud53c\uc5b8\uc778 \uc18c\ub0d0 \ud5e4\ub2c8\ub294 \uc62c\ub9bc\ud53d \ud0c0\uc774\ud2c0\uc744 \uc9c0\ucf1c\ub0b4\uc5c8\uace0, 1920\ub144 \uc62c\ub9bc\ud53d \ubcf5\uc2f1 \ucc54\ud53c\uc5b8\uc774\uc5c8\ub358 \uc5d0\ub514 \uc774\uac74\uc774 \ub0a8\uc790 4\uc778\uc2b9 \ubd05\uc2ac\ub808\uc774 \uacbd\uae30\uc5d0\uc11c \uae08\uba54\ub2ec\uc744 \ub530\ub0b4\uba74\uc11c, \uc62c\ub9bc\ud53d \uc5ed\uc0ac\ub97c \ud1b5\ud2c0\uc5b4 \uc720\uc77c\ud558\uac8c \ud558\uacc4\uc640 \ub3d9\uacc4 \uc62c\ub9bc\ud53d \uacbd\uae30 \ubaa8\ub450\uc5d0\uc11c \uae08\uba54\ub2ec\uc744 \ud68d\ub4dd\ud55c \uc120\uc218\uac00 \ub418\uc5c8\ub2e4."} {"question": "\ucee4\ub125\uc190\uc744 \uad6c\uc131\ud558\ub294 \ub2e8\ubc31\uc9c8 \ub2e8\uc704\uccb4\ub294 \ubb34\uc5c7\uc778\uac00?", "paragraph": "\uac01 \uac04\uadf9\uc5f0\uc811(\uacb0\ud569\uc5f0\uc811\uc774\ub77c\uace0\ub3c4 \ud568)\uc740 \ub450 \uc138\ud3ec\uc758 \ub9c9\uc744 \uac00\ub85c\uc9c0\ub974\ub294 \ub9ce\uc740 \uac04\uadf9\uc5f0\uc811 \ucc44\ub110\uc744 \uac00\uc9c0\uace0 \uc788\ub2e4. \ucc44\ub110\uc758 \uad6c\uba4d \ud06c\uae30\uac00 1.2~2.0nm\uc774\ubbc0\ub85c, \uac04\uadf9\uc5f0\uc811\uc758 \uad6c\uba4d\uc740 \uc774\uc628\ub4e4\uc774\ub098 \uc2e0\ud638 \ubd84\uc790\uac19\uc774 \uc911\uac04 \ud06c\uae30\uc758 \ubd84\uc790\ub3c4 \ud55c \uc138\ud3ec\uc5d0\uc11c \ub2e4\ub978 \uc138\ud3ec\ub85c \ud1b5\uacfc\ud560 \uc815\ub3c4\ub85c \ud06c\ub2e4. \uc989, \ub450 \uc138\ud3ec\uc758 \uc138\ud3ec\uc9c8\uc744 \uc5f0\uacb0\uc2dc\ucf1c\uc900\ub2e4. \ub530\ub77c\uc11c \ud55c \uc138\ud3ec\uc758 \uc804\uc704\uac00 \ubc14\ub00c\uba74, \uc774\uc628\ub4e4\uc740 \ud55c \uc138\ud3ec\uc5d0\uc11c \ub2e4\ub978 \uc138\ud3ec\ub85c \uc774\ub3d9\ud558\uba74\uc11c \uc2dc\ub0c5\uc2a4 \ud6c4 \uc138\ud3ec\ub97c \ud0c8\ubd84\uadf9 \uc2dc\ud0a4\uac8c \ub41c\ub2e4. \uac04\uadf9\uc5f0\uc811\uc740 \ucc99\ucd94\ub3d9\ubb3c\uc758 \uacbd\uc6b0 \ucee4\ub125\uc190(connexon)\uc774\ub77c \ubd88\ub9ac\ub294 2\uac1c\uc758 \uc808\ubc18\ucc44\ub110\ub85c \uc774\ub8e8\uc5b4\uc838 \uc788\uc73c\uba70 \uc2dc\ub0c5\uc2a4\uc5d0\uc11c \uac01 \uc138\ud3ec\uc5d0 \ud558\ub098\uc529 \uc788\ub2e4. \ucee4\ub125\uc190(connexon)\uc740 6\uac1c\uc758 7.5nm \uae38\uc774\uc758 \ub9c9\uc744 4\ubc88 \ud1b5\uacfc\ud558\ub294 \ucee4\ub125\uc2e0(connexin)\uc774\ub77c \ubd88\ub9ac\ub294 \ub2e8\ubc31\uc9c8 \ub2e8\uc704\uccb4\ub85c \uad6c\uc131\ub418\uba70, \uac01 \ub2e8\uc704\uccb4\ub4e4\uc740 \uac70\uc758 \ub611\uac19\uac70\ub098 \uc57d\uac04 \ub2e4\ub974\ub2e4 .", "answer": "\ucee4\ub125\uc2e0", "sentence": "\ucee4\ub125\uc190(connexon)\uc740 6\uac1c\uc758 7.5nm \uae38\uc774\uc758 \ub9c9\uc744 4\ubc88 \ud1b5\uacfc\ud558\ub294 \ucee4\ub125\uc2e0 (connexin)\uc774\ub77c \ubd88\ub9ac\ub294 \ub2e8\ubc31\uc9c8 \ub2e8\uc704\uccb4\ub85c \uad6c\uc131\ub418\uba70, \uac01 \ub2e8\uc704\uccb4\ub4e4\uc740 \uac70\uc758 \ub611\uac19\uac70\ub098 \uc57d\uac04 \ub2e4\ub974\ub2e4 .", "paragraph_sentence": "\uac01 \uac04\uadf9\uc5f0\uc811(\uacb0\ud569\uc5f0\uc811\uc774\ub77c\uace0\ub3c4 \ud568)\uc740 \ub450 \uc138\ud3ec\uc758 \ub9c9\uc744 \uac00\ub85c\uc9c0\ub974\ub294 \ub9ce\uc740 \uac04\uadf9\uc5f0\uc811 \ucc44\ub110\uc744 \uac00\uc9c0\uace0 \uc788\ub2e4. \ucc44\ub110\uc758 \uad6c\uba4d \ud06c\uae30\uac00 1.2~2.0nm\uc774\ubbc0\ub85c, \uac04\uadf9\uc5f0\uc811\uc758 \uad6c\uba4d\uc740 \uc774\uc628\ub4e4\uc774\ub098 \uc2e0\ud638 \ubd84\uc790\uac19\uc774 \uc911\uac04 \ud06c\uae30\uc758 \ubd84\uc790\ub3c4 \ud55c \uc138\ud3ec\uc5d0\uc11c \ub2e4\ub978 \uc138\ud3ec\ub85c \ud1b5\uacfc\ud560 \uc815\ub3c4\ub85c \ud06c\ub2e4. \uc989, \ub450 \uc138\ud3ec\uc758 \uc138\ud3ec\uc9c8\uc744 \uc5f0\uacb0\uc2dc\ucf1c\uc900\ub2e4. \ub530\ub77c\uc11c \ud55c \uc138\ud3ec\uc758 \uc804\uc704\uac00 \ubc14\ub00c\uba74, \uc774\uc628\ub4e4\uc740 \ud55c \uc138\ud3ec\uc5d0\uc11c \ub2e4\ub978 \uc138\ud3ec\ub85c \uc774\ub3d9\ud558\uba74\uc11c \uc2dc\ub0c5\uc2a4 \ud6c4 \uc138\ud3ec\ub97c \ud0c8\ubd84\uadf9 \uc2dc\ud0a4\uac8c \ub41c\ub2e4. \uac04\uadf9\uc5f0\uc811\uc740 \ucc99\ucd94\ub3d9\ubb3c\uc758 \uacbd\uc6b0 \ucee4\ub125\uc190(connexon)\uc774\ub77c \ubd88\ub9ac\ub294 2\uac1c\uc758 \uc808\ubc18\ucc44\ub110\ub85c \uc774\ub8e8\uc5b4\uc838 \uc788\uc73c\uba70 \uc2dc\ub0c5\uc2a4\uc5d0\uc11c \uac01 \uc138\ud3ec\uc5d0 \ud558\ub098\uc529 \uc788\ub2e4. \ucee4\ub125\uc190(connexon)\uc740 6\uac1c\uc758 7.5nm \uae38\uc774\uc758 \ub9c9\uc744 4\ubc88 \ud1b5\uacfc\ud558\ub294 \ucee4\ub125\uc2e0 (connexin)\uc774\ub77c \ubd88\ub9ac\ub294 \ub2e8\ubc31\uc9c8 \ub2e8\uc704\uccb4\ub85c \uad6c\uc131\ub418\uba70, \uac01 \ub2e8\uc704\uccb4\ub4e4\uc740 \uac70\uc758 \ub611\uac19\uac70\ub098 \uc57d\uac04 \ub2e4\ub974\ub2e4 . ", "paragraph_answer": "\uac01 \uac04\uadf9\uc5f0\uc811(\uacb0\ud569\uc5f0\uc811\uc774\ub77c\uace0\ub3c4 \ud568)\uc740 \ub450 \uc138\ud3ec\uc758 \ub9c9\uc744 \uac00\ub85c\uc9c0\ub974\ub294 \ub9ce\uc740 \uac04\uadf9\uc5f0\uc811 \ucc44\ub110\uc744 \uac00\uc9c0\uace0 \uc788\ub2e4. \ucc44\ub110\uc758 \uad6c\uba4d \ud06c\uae30\uac00 1.2~2.0nm\uc774\ubbc0\ub85c, \uac04\uadf9\uc5f0\uc811\uc758 \uad6c\uba4d\uc740 \uc774\uc628\ub4e4\uc774\ub098 \uc2e0\ud638 \ubd84\uc790\uac19\uc774 \uc911\uac04 \ud06c\uae30\uc758 \ubd84\uc790\ub3c4 \ud55c \uc138\ud3ec\uc5d0\uc11c \ub2e4\ub978 \uc138\ud3ec\ub85c \ud1b5\uacfc\ud560 \uc815\ub3c4\ub85c \ud06c\ub2e4. \uc989, \ub450 \uc138\ud3ec\uc758 \uc138\ud3ec\uc9c8\uc744 \uc5f0\uacb0\uc2dc\ucf1c\uc900\ub2e4. \ub530\ub77c\uc11c \ud55c \uc138\ud3ec\uc758 \uc804\uc704\uac00 \ubc14\ub00c\uba74, \uc774\uc628\ub4e4\uc740 \ud55c \uc138\ud3ec\uc5d0\uc11c \ub2e4\ub978 \uc138\ud3ec\ub85c \uc774\ub3d9\ud558\uba74\uc11c \uc2dc\ub0c5\uc2a4 \ud6c4 \uc138\ud3ec\ub97c \ud0c8\ubd84\uadf9 \uc2dc\ud0a4\uac8c \ub41c\ub2e4. \uac04\uadf9\uc5f0\uc811\uc740 \ucc99\ucd94\ub3d9\ubb3c\uc758 \uacbd\uc6b0 \ucee4\ub125\uc190(connexon)\uc774\ub77c \ubd88\ub9ac\ub294 2\uac1c\uc758 \uc808\ubc18\ucc44\ub110\ub85c \uc774\ub8e8\uc5b4\uc838 \uc788\uc73c\uba70 \uc2dc\ub0c5\uc2a4\uc5d0\uc11c \uac01 \uc138\ud3ec\uc5d0 \ud558\ub098\uc529 \uc788\ub2e4. \ucee4\ub125\uc190(connexon)\uc740 6\uac1c\uc758 7.5nm \uae38\uc774\uc758 \ub9c9\uc744 4\ubc88 \ud1b5\uacfc\ud558\ub294 \ucee4\ub125\uc2e0 (connexin)\uc774\ub77c \ubd88\ub9ac\ub294 \ub2e8\ubc31\uc9c8 \ub2e8\uc704\uccb4\ub85c \uad6c\uc131\ub418\uba70, \uac01 \ub2e8\uc704\uccb4\ub4e4\uc740 \uac70\uc758 \ub611\uac19\uac70\ub098 \uc57d\uac04 \ub2e4\ub974\ub2e4 .", "sentence_answer": "\ucee4\ub125\uc190(connexon)\uc740 6\uac1c\uc758 7.5nm \uae38\uc774\uc758 \ub9c9\uc744 4\ubc88 \ud1b5\uacfc\ud558\ub294 \ucee4\ub125\uc2e0 (connexin)\uc774\ub77c \ubd88\ub9ac\ub294 \ub2e8\ubc31\uc9c8 \ub2e8\uc704\uccb4\ub85c \uad6c\uc131\ub418\uba70, \uac01 \ub2e8\uc704\uccb4\ub4e4\uc740 \uac70\uc758 \ub611\uac19\uac70\ub098 \uc57d\uac04 \ub2e4\ub974\ub2e4 .", "paragraph_id": "6549493-1-1", "paragraph_question": "question: \ucee4\ub125\uc190\uc744 \uad6c\uc131\ud558\ub294 \ub2e8\ubc31\uc9c8 \ub2e8\uc704\uccb4\ub294 \ubb34\uc5c7\uc778\uac00?, context: \uac01 \uac04\uadf9\uc5f0\uc811(\uacb0\ud569\uc5f0\uc811\uc774\ub77c\uace0\ub3c4 \ud568)\uc740 \ub450 \uc138\ud3ec\uc758 \ub9c9\uc744 \uac00\ub85c\uc9c0\ub974\ub294 \ub9ce\uc740 \uac04\uadf9\uc5f0\uc811 \ucc44\ub110\uc744 \uac00\uc9c0\uace0 \uc788\ub2e4. \ucc44\ub110\uc758 \uad6c\uba4d \ud06c\uae30\uac00 1.2~2.0nm\uc774\ubbc0\ub85c, \uac04\uadf9\uc5f0\uc811\uc758 \uad6c\uba4d\uc740 \uc774\uc628\ub4e4\uc774\ub098 \uc2e0\ud638 \ubd84\uc790\uac19\uc774 \uc911\uac04 \ud06c\uae30\uc758 \ubd84\uc790\ub3c4 \ud55c \uc138\ud3ec\uc5d0\uc11c \ub2e4\ub978 \uc138\ud3ec\ub85c \ud1b5\uacfc\ud560 \uc815\ub3c4\ub85c \ud06c\ub2e4. \uc989, \ub450 \uc138\ud3ec\uc758 \uc138\ud3ec\uc9c8\uc744 \uc5f0\uacb0\uc2dc\ucf1c\uc900\ub2e4. \ub530\ub77c\uc11c \ud55c \uc138\ud3ec\uc758 \uc804\uc704\uac00 \ubc14\ub00c\uba74, \uc774\uc628\ub4e4\uc740 \ud55c \uc138\ud3ec\uc5d0\uc11c \ub2e4\ub978 \uc138\ud3ec\ub85c \uc774\ub3d9\ud558\uba74\uc11c \uc2dc\ub0c5\uc2a4 \ud6c4 \uc138\ud3ec\ub97c \ud0c8\ubd84\uadf9 \uc2dc\ud0a4\uac8c \ub41c\ub2e4. \uac04\uadf9\uc5f0\uc811\uc740 \ucc99\ucd94\ub3d9\ubb3c\uc758 \uacbd\uc6b0 \ucee4\ub125\uc190(connexon)\uc774\ub77c \ubd88\ub9ac\ub294 2\uac1c\uc758 \uc808\ubc18\ucc44\ub110\ub85c \uc774\ub8e8\uc5b4\uc838 \uc788\uc73c\uba70 \uc2dc\ub0c5\uc2a4\uc5d0\uc11c \uac01 \uc138\ud3ec\uc5d0 \ud558\ub098\uc529 \uc788\ub2e4. \ucee4\ub125\uc190(connexon)\uc740 6\uac1c\uc758 7.5nm \uae38\uc774\uc758 \ub9c9\uc744 4\ubc88 \ud1b5\uacfc\ud558\ub294 \ucee4\ub125\uc2e0(connexin)\uc774\ub77c \ubd88\ub9ac\ub294 \ub2e8\ubc31\uc9c8 \ub2e8\uc704\uccb4\ub85c \uad6c\uc131\ub418\uba70, \uac01 \ub2e8\uc704\uccb4\ub4e4\uc740 \uac70\uc758 \ub611\uac19\uac70\ub098 \uc57d\uac04 \ub2e4\ub974\ub2e4 ."} {"question": "\ucf00\uc774\ud2f0 \ud398\ub9ac\uac00 \uce90\ud53c\ud1a8 \ub808\ucf54\ub4dc\uc640 \uacc4\uc57d\uc744 \ub9fa\uc740 \ub144\ub3c4\ub294?", "paragraph": "\uceec\ub7fc\ube44\uc544\uc640 \ud5e4\uc5b4\uc9c4 \ud6c4 \ud398\ub9ac\ub294 \ud64d\ubcf4\ubd80 \uac04\ubd80\ub85c \uc788\ub358 \uc548\uc824\ub9ac\uce74 \ucf54\ube0c\ubc1c\ub7ec\ub97c \ud1b5\ud574 \ubc84\uc9c4 \ub808\ucf54\ub4dc\uc758 \uc0ac\uc7a5 \uc81c\uc774\uc2a8 \ud50c\ub86c\uc5d0\uac8c \ub370\ubaa8\ub97c \uc804\ub2ec\ud588\ub2e4. \ud50c\ub86c\uc740 \ud398\ub9ac\uac00 \ud68d\uae30\uc801\uc778 \uc2a4\ud0c0\uac00 \ub420 \uac83\uc774\ub77c\uace0 \ud655\uc2e0\ud588\uace0 2007\ub144 4\uc6d4 \uc778\uc218\ud588\ub358 \ub808\uc774\ube14\uc0ac \uce90\ud53c\ud1a8 \ub808\ucf54\ub4dc\uc640 \uacc4\uc57d\uc744 \ub9fa\uac8c \ud574\uc92c\ub2e4. \ub808\uc774\ube14\uc0ac\uc5d0\uc11c\ub294 \ud398\ub9ac\uac00 \uae30\uc874\uc5d0 \ub9cc\ub4e4\uc5b4\ub1a8\ub358 \ub178\ub798\uc5d0 \"\uac15\ub825\ud55c \ud55c\ubc29\"\uc774 \ud544\uc694\ud558\ub2e4 \uc0dd\uac01\ud574 \ub2e5\ud130 \ub8e8\ud06c\uc640 \ud568\uaed8 \uc791\uc5c5\uc744 \uc9c4\ud589\ud558\uac8c \ud588\ub2e4. \ud398\ub9ac\uc640 \ub2e5\ud130 \ub8e8\ud06c\ub294 \uacf5\ub3d9 \uc791\uace1 \ub05d\uc5d0 \u3008I Kissed a Girl\u3009\uacfc \u3008Hot n Cold\u3009\ub97c \ub9cc\ub4e4\uc5b4\ub0c8\ub2e4. 2007\ub144 11\uc6d4 \uc74c\uc545 \uc2dc\uc7a5\uc5d0 \ud398\ub9ac\ub97c \uc18c\uac1c\ud558\ub294 \uac83\uc744 \ubaa9\ud45c\ub85c \u3008Ur So Gay\u3009\uc758 \ubba4\uc9c1\ube44\ub514\uc624\ub97c \uc120\ubcf4\uc600\ub2e4. \uc0ac\ub78c\ub4e4\uc758 \uad00\uc2ec\uc744 \uc774\ub04c\uc5b4\ub0b4\uba74\uc11c \uc774\ud6c4 \u3008Ur So Gay\u3009\uc758 \ub514\uc9c0\ud138 EP\uac00 \ubc1c\ub9e4\ub418\uc5c8\ub2e4. \ub9c8\ub3c8\ub098\ub294 2008\ub144 4\uc6d4 \u300aJohnJay & Rich\u300b\ub77c\ub294 \ub77c\ub514\uc624\uc1fc\uc5d0 \ucd9c\uc5f0\ud574 \"\uc88b\uc544\ud558\ub294 \ub178\ub798\"\ub77c\uace0 \uc5b8\uae09\ud558\uba74\uc11c \ub178\ub798\uac00 \uc54c\ub824\uc9c0\ub294\ub370 \uc77c\uc870\ud588\ub2e4. 2008\ub144 3\uc6d4\uc5d0\ub294 \u300a\uc640\uc77c\ub4dc\ud30c\uc774\uc5b4\u300b\uc758 \"Life's Too Short\"\ud3b8\uc5d0 \ud074\ub7fd \uac00\uc218\ub85c \uce74\uba54\uc624 \ucd9c\uc5f0\ud588\ub2e4. 6\uc6d4\uc5d0\ub294 \u300a\ub354 \uc601 \uc564 \ub354 \ub808\uc2a4\ud2b8\ub9ac\uc2a4\u300b\uc758 \ub4dc\ub77c\ub9c8 \uc18d \uc7a1\uc9c0 \u300aRestless Style\u300b\uc5d0 \ud45c\uc9c0 \ubaa8\ub378\ub85c \ub4f1\uc7a5\ud588\ub2e4.", "answer": "2007\ub144", "sentence": "\ud50c\ub86c\uc740 \ud398\ub9ac\uac00 \ud68d\uae30\uc801\uc778 \uc2a4\ud0c0\uac00 \ub420 \uac83\uc774\ub77c\uace0 \ud655\uc2e0\ud588\uace0 2007\ub144 4\uc6d4 \uc778\uc218\ud588\ub358 \ub808\uc774\ube14\uc0ac \uce90\ud53c\ud1a8 \ub808\ucf54\ub4dc\uc640 \uacc4\uc57d\uc744 \ub9fa\uac8c \ud574\uc92c\ub2e4.", "paragraph_sentence": "\uceec\ub7fc\ube44\uc544\uc640 \ud5e4\uc5b4\uc9c4 \ud6c4 \ud398\ub9ac\ub294 \ud64d\ubcf4\ubd80 \uac04\ubd80\ub85c \uc788\ub358 \uc548\uc824\ub9ac\uce74 \ucf54\ube0c\ubc1c\ub7ec\ub97c \ud1b5\ud574 \ubc84\uc9c4 \ub808\ucf54\ub4dc\uc758 \uc0ac\uc7a5 \uc81c\uc774\uc2a8 \ud50c\ub86c\uc5d0\uac8c \ub370\ubaa8\ub97c \uc804\ub2ec\ud588\ub2e4. \ud50c\ub86c\uc740 \ud398\ub9ac\uac00 \ud68d\uae30\uc801\uc778 \uc2a4\ud0c0\uac00 \ub420 \uac83\uc774\ub77c\uace0 \ud655\uc2e0\ud588\uace0 2007\ub144 4\uc6d4 \uc778\uc218\ud588\ub358 \ub808\uc774\ube14\uc0ac \uce90\ud53c\ud1a8 \ub808\ucf54\ub4dc\uc640 \uacc4\uc57d\uc744 \ub9fa\uac8c \ud574\uc92c\ub2e4. \ub808\uc774\ube14\uc0ac\uc5d0\uc11c\ub294 \ud398\ub9ac\uac00 \uae30\uc874\uc5d0 \ub9cc\ub4e4\uc5b4\ub1a8\ub358 \ub178\ub798\uc5d0 \"\uac15\ub825\ud55c \ud55c\ubc29\"\uc774 \ud544\uc694\ud558\ub2e4 \uc0dd\uac01\ud574 \ub2e5\ud130 \ub8e8\ud06c\uc640 \ud568\uaed8 \uc791\uc5c5\uc744 \uc9c4\ud589\ud558\uac8c \ud588\ub2e4. \ud398\ub9ac\uc640 \ub2e5\ud130 \ub8e8\ud06c\ub294 \uacf5\ub3d9 \uc791\uace1 \ub05d\uc5d0 \u3008I Kissed a Girl\u3009\uacfc \u3008Hot n Cold\u3009\ub97c \ub9cc\ub4e4\uc5b4\ub0c8\ub2e4. 2007\ub144 11\uc6d4 \uc74c\uc545 \uc2dc\uc7a5\uc5d0 \ud398\ub9ac\ub97c \uc18c\uac1c\ud558\ub294 \uac83\uc744 \ubaa9\ud45c\ub85c \u3008Ur So Gay\u3009\uc758 \ubba4\uc9c1\ube44\ub514\uc624\ub97c \uc120\ubcf4\uc600\ub2e4. \uc0ac\ub78c\ub4e4\uc758 \uad00\uc2ec\uc744 \uc774\ub04c\uc5b4\ub0b4\uba74\uc11c \uc774\ud6c4 \u3008Ur So Gay\u3009\uc758 \ub514\uc9c0\ud138 EP\uac00 \ubc1c\ub9e4\ub418\uc5c8\ub2e4. \ub9c8\ub3c8\ub098\ub294 2008\ub144 4\uc6d4 \u300aJohnJay & Rich\u300b\ub77c\ub294 \ub77c\ub514\uc624\uc1fc\uc5d0 \ucd9c\uc5f0\ud574 \"\uc88b\uc544\ud558\ub294 \ub178\ub798\"\ub77c\uace0 \uc5b8\uae09\ud558\uba74\uc11c \ub178\ub798\uac00 \uc54c\ub824\uc9c0\ub294\ub370 \uc77c\uc870\ud588\ub2e4. 2008\ub144 3\uc6d4\uc5d0\ub294 \u300a\uc640\uc77c\ub4dc\ud30c\uc774\uc5b4\u300b\uc758 \"Life's Too Short\"\ud3b8\uc5d0 \ud074\ub7fd \uac00\uc218\ub85c \uce74\uba54\uc624 \ucd9c\uc5f0\ud588\ub2e4. 6\uc6d4\uc5d0\ub294 \u300a\ub354 \uc601 \uc564 \ub354 \ub808\uc2a4\ud2b8\ub9ac\uc2a4\u300b\uc758 \ub4dc\ub77c\ub9c8 \uc18d \uc7a1\uc9c0 \u300aRestless Style\u300b\uc5d0 \ud45c\uc9c0 \ubaa8\ub378\ub85c \ub4f1\uc7a5\ud588\ub2e4.", "paragraph_answer": "\uceec\ub7fc\ube44\uc544\uc640 \ud5e4\uc5b4\uc9c4 \ud6c4 \ud398\ub9ac\ub294 \ud64d\ubcf4\ubd80 \uac04\ubd80\ub85c \uc788\ub358 \uc548\uc824\ub9ac\uce74 \ucf54\ube0c\ubc1c\ub7ec\ub97c \ud1b5\ud574 \ubc84\uc9c4 \ub808\ucf54\ub4dc\uc758 \uc0ac\uc7a5 \uc81c\uc774\uc2a8 \ud50c\ub86c\uc5d0\uac8c \ub370\ubaa8\ub97c \uc804\ub2ec\ud588\ub2e4. \ud50c\ub86c\uc740 \ud398\ub9ac\uac00 \ud68d\uae30\uc801\uc778 \uc2a4\ud0c0\uac00 \ub420 \uac83\uc774\ub77c\uace0 \ud655\uc2e0\ud588\uace0 2007\ub144 4\uc6d4 \uc778\uc218\ud588\ub358 \ub808\uc774\ube14\uc0ac \uce90\ud53c\ud1a8 \ub808\ucf54\ub4dc\uc640 \uacc4\uc57d\uc744 \ub9fa\uac8c \ud574\uc92c\ub2e4. \ub808\uc774\ube14\uc0ac\uc5d0\uc11c\ub294 \ud398\ub9ac\uac00 \uae30\uc874\uc5d0 \ub9cc\ub4e4\uc5b4\ub1a8\ub358 \ub178\ub798\uc5d0 \"\uac15\ub825\ud55c \ud55c\ubc29\"\uc774 \ud544\uc694\ud558\ub2e4 \uc0dd\uac01\ud574 \ub2e5\ud130 \ub8e8\ud06c\uc640 \ud568\uaed8 \uc791\uc5c5\uc744 \uc9c4\ud589\ud558\uac8c \ud588\ub2e4. \ud398\ub9ac\uc640 \ub2e5\ud130 \ub8e8\ud06c\ub294 \uacf5\ub3d9 \uc791\uace1 \ub05d\uc5d0 \u3008I Kissed a Girl\u3009\uacfc \u3008Hot n Cold\u3009\ub97c \ub9cc\ub4e4\uc5b4\ub0c8\ub2e4. 2007\ub144 11\uc6d4 \uc74c\uc545 \uc2dc\uc7a5\uc5d0 \ud398\ub9ac\ub97c \uc18c\uac1c\ud558\ub294 \uac83\uc744 \ubaa9\ud45c\ub85c \u3008Ur So Gay\u3009\uc758 \ubba4\uc9c1\ube44\ub514\uc624\ub97c \uc120\ubcf4\uc600\ub2e4. \uc0ac\ub78c\ub4e4\uc758 \uad00\uc2ec\uc744 \uc774\ub04c\uc5b4\ub0b4\uba74\uc11c \uc774\ud6c4 \u3008Ur So Gay\u3009\uc758 \ub514\uc9c0\ud138 EP\uac00 \ubc1c\ub9e4\ub418\uc5c8\ub2e4. \ub9c8\ub3c8\ub098\ub294 2008\ub144 4\uc6d4 \u300aJohnJay & Rich\u300b\ub77c\ub294 \ub77c\ub514\uc624\uc1fc\uc5d0 \ucd9c\uc5f0\ud574 \"\uc88b\uc544\ud558\ub294 \ub178\ub798\"\ub77c\uace0 \uc5b8\uae09\ud558\uba74\uc11c \ub178\ub798\uac00 \uc54c\ub824\uc9c0\ub294\ub370 \uc77c\uc870\ud588\ub2e4. 2008\ub144 3\uc6d4\uc5d0\ub294 \u300a\uc640\uc77c\ub4dc\ud30c\uc774\uc5b4\u300b\uc758 \"Life's Too Short\"\ud3b8\uc5d0 \ud074\ub7fd \uac00\uc218\ub85c \uce74\uba54\uc624 \ucd9c\uc5f0\ud588\ub2e4. 6\uc6d4\uc5d0\ub294 \u300a\ub354 \uc601 \uc564 \ub354 \ub808\uc2a4\ud2b8\ub9ac\uc2a4\u300b\uc758 \ub4dc\ub77c\ub9c8 \uc18d \uc7a1\uc9c0 \u300aRestless Style\u300b\uc5d0 \ud45c\uc9c0 \ubaa8\ub378\ub85c \ub4f1\uc7a5\ud588\ub2e4.", "sentence_answer": "\ud50c\ub86c\uc740 \ud398\ub9ac\uac00 \ud68d\uae30\uc801\uc778 \uc2a4\ud0c0\uac00 \ub420 \uac83\uc774\ub77c\uace0 \ud655\uc2e0\ud588\uace0 2007\ub144 4\uc6d4 \uc778\uc218\ud588\ub358 \ub808\uc774\ube14\uc0ac \uce90\ud53c\ud1a8 \ub808\ucf54\ub4dc\uc640 \uacc4\uc57d\uc744 \ub9fa\uac8c \ud574\uc92c\ub2e4.", "paragraph_id": "6209726-7-0", "paragraph_question": "question: \ucf00\uc774\ud2f0 \ud398\ub9ac\uac00 \uce90\ud53c\ud1a8 \ub808\ucf54\ub4dc\uc640 \uacc4\uc57d\uc744 \ub9fa\uc740 \ub144\ub3c4\ub294?, context: \uceec\ub7fc\ube44\uc544\uc640 \ud5e4\uc5b4\uc9c4 \ud6c4 \ud398\ub9ac\ub294 \ud64d\ubcf4\ubd80 \uac04\ubd80\ub85c \uc788\ub358 \uc548\uc824\ub9ac\uce74 \ucf54\ube0c\ubc1c\ub7ec\ub97c \ud1b5\ud574 \ubc84\uc9c4 \ub808\ucf54\ub4dc\uc758 \uc0ac\uc7a5 \uc81c\uc774\uc2a8 \ud50c\ub86c\uc5d0\uac8c \ub370\ubaa8\ub97c \uc804\ub2ec\ud588\ub2e4. \ud50c\ub86c\uc740 \ud398\ub9ac\uac00 \ud68d\uae30\uc801\uc778 \uc2a4\ud0c0\uac00 \ub420 \uac83\uc774\ub77c\uace0 \ud655\uc2e0\ud588\uace0 2007\ub144 4\uc6d4 \uc778\uc218\ud588\ub358 \ub808\uc774\ube14\uc0ac \uce90\ud53c\ud1a8 \ub808\ucf54\ub4dc\uc640 \uacc4\uc57d\uc744 \ub9fa\uac8c \ud574\uc92c\ub2e4. \ub808\uc774\ube14\uc0ac\uc5d0\uc11c\ub294 \ud398\ub9ac\uac00 \uae30\uc874\uc5d0 \ub9cc\ub4e4\uc5b4\ub1a8\ub358 \ub178\ub798\uc5d0 \"\uac15\ub825\ud55c \ud55c\ubc29\"\uc774 \ud544\uc694\ud558\ub2e4 \uc0dd\uac01\ud574 \ub2e5\ud130 \ub8e8\ud06c\uc640 \ud568\uaed8 \uc791\uc5c5\uc744 \uc9c4\ud589\ud558\uac8c \ud588\ub2e4. \ud398\ub9ac\uc640 \ub2e5\ud130 \ub8e8\ud06c\ub294 \uacf5\ub3d9 \uc791\uace1 \ub05d\uc5d0 \u3008I Kissed a Girl\u3009\uacfc \u3008Hot n Cold\u3009\ub97c \ub9cc\ub4e4\uc5b4\ub0c8\ub2e4. 2007\ub144 11\uc6d4 \uc74c\uc545 \uc2dc\uc7a5\uc5d0 \ud398\ub9ac\ub97c \uc18c\uac1c\ud558\ub294 \uac83\uc744 \ubaa9\ud45c\ub85c \u3008Ur So Gay\u3009\uc758 \ubba4\uc9c1\ube44\ub514\uc624\ub97c \uc120\ubcf4\uc600\ub2e4. \uc0ac\ub78c\ub4e4\uc758 \uad00\uc2ec\uc744 \uc774\ub04c\uc5b4\ub0b4\uba74\uc11c \uc774\ud6c4 \u3008Ur So Gay\u3009\uc758 \ub514\uc9c0\ud138 EP\uac00 \ubc1c\ub9e4\ub418\uc5c8\ub2e4. \ub9c8\ub3c8\ub098\ub294 2008\ub144 4\uc6d4 \u300aJohnJay & Rich\u300b\ub77c\ub294 \ub77c\ub514\uc624\uc1fc\uc5d0 \ucd9c\uc5f0\ud574 \"\uc88b\uc544\ud558\ub294 \ub178\ub798\"\ub77c\uace0 \uc5b8\uae09\ud558\uba74\uc11c \ub178\ub798\uac00 \uc54c\ub824\uc9c0\ub294\ub370 \uc77c\uc870\ud588\ub2e4. 2008\ub144 3\uc6d4\uc5d0\ub294 \u300a\uc640\uc77c\ub4dc\ud30c\uc774\uc5b4\u300b\uc758 \"Life's Too Short\"\ud3b8\uc5d0 \ud074\ub7fd \uac00\uc218\ub85c \uce74\uba54\uc624 \ucd9c\uc5f0\ud588\ub2e4. 6\uc6d4\uc5d0\ub294 \u300a\ub354 \uc601 \uc564 \ub354 \ub808\uc2a4\ud2b8\ub9ac\uc2a4\u300b\uc758 \ub4dc\ub77c\ub9c8 \uc18d \uc7a1\uc9c0 \u300aRestless Style\u300b\uc5d0 \ud45c\uc9c0 \ubaa8\ub378\ub85c \ub4f1\uc7a5\ud588\ub2e4."} {"question": "\uac8c\ubc1c\uc120\uc778\uc7a5\uc774\ub098 \uacf5\uc791\uc120\uc778\uc7a5\uc774 \ub9ce\uc774 \uc7ac\ubc30\ub418\ub294 \uc774\uc720\ub294?", "paragraph": "\uc120\uc778\uc7a5 \ubd84\ud3ec\uc9c0\uc5ed\uc740 \uc8fc\ub85c \uac74\uc870\ud55c \ubc18\uac74\uc870\uc9c0\ub300\ub098 \uc0ac\ub9c9\uc9c0\uc5ed, \uace0\uc0b0\uc9c0\uc5ed\uc73c\ub85c \uc5f0 \uac15\uc6b0\ub7c9\uc774 20mm \uc774\uc0c1\uc774\uac70\ub098 \uc548\uac1c \ub4f1\uc73c\ub85c \uacc4\uc18d\ud574\uc11c \uc218\ubd84\uc774 \uacf5\uae09\ub418\ub294 \uc9c0\uc5ed\uc774\ub2e4. \uc5f0 \uac15\uc6b0\ub7c9\uc774 20mm \ubbf8\ub9cc\uc774\uace0, \uc218\ubd84\uacf5\uae09\uc774 \uc6d0\ud65c\ud558\uc9c0 \uc54a\uc740 \uacf3\uc5d0\uc11c\ub294 \uc120\uc778\uc7a5\uc774 \uc0dd\uc721\ud560 \uc218 \uc5c6\ub2e4. \ubc18\ub300\ub85c \ub2e4\uc2dc \ub2e4\uc2b5\ud55c \uc5f4\ub300\uc6b0\ub9bc \ub4f1\uc5d0 \uc801\uc751\ud55c \uc120\uc778\uc7a5\ub4e4\ub3c4 \uc874\uc7ac\ud558\ub294\ub370, \uc6d0\uc608\uc6a9\uc73c\ub85c \ub9ce\uc774 \uc7ac\ubc30\ub418\uace0 \uc788\ub294 \uac8c\ubc1c\uc120\uc778\uc7a5\uc774\ub098 \uacf5\uc791\uc120\uc778\uc7a5\uacfc \uac19\uc740 \uc885\uc740 \ub0a8\uc544\uba54\ub9ac\uce74\uc758 \uc5ec\ub7ec \uc5f4\ub300\uc6b0\ub9bc\uc758 \ub098\ubb47\uac00\uc9c0\uc5d0 \uc874\uc7ac\ud558\ub294 \uc774\ub07c\uc5d0 \ubfcc\ub9ac\ub97c \ub0b4\ub9ac\uace0 \uc11c\uc2dd\ud560 \uc218 \uc788\ub3c4\ub85d \uc9c4\ud654\ud588\ub2e4. (\uc774\ub4e4 \uc120\uc778\uc7a5\ub4e4\uc744 \ud759\uc5d0 \uc2ec\uc9c0 \uc54a\uace0 \uc774\ub07c\uc5d0 \uc2ec\ub294 \uc774\uc720\ub294 \uc774\ub7ec\ud558\uae30 \ub54c\ubb38\uc774\ub2e4.) \ub300\ud55c\ubbfc\uad6d\uc5d0 \uad70\ub77d\uc744 \ud615\uc131\ud55c Opuntia ficus-indica,Opuntia humifusa \uacc4\uc5f4\uc758 \uc120\uc778\uc7a5\ub4e4\ub3c4 \uc81c\uc8fc\ub3c4 \uc7a5\ub9c8 \ub4f1\uc5d0 \uacac\ub514\uba70 \uc131\uc7a5\ud558\ub294 \uc2b5\ud55c \ud658\uacbd\uc5d0 \uac15\ud55c \ubaa8\uc2b5\uc744 \ubcf4\uc5ec\uc8fc\uace0 \uc788\ub2e4.", "answer": "\uc6d0\uc608\uc6a9", "sentence": "\ubc18\ub300\ub85c \ub2e4\uc2dc \ub2e4\uc2b5\ud55c \uc5f4\ub300\uc6b0\ub9bc \ub4f1\uc5d0 \uc801\uc751\ud55c \uc120\uc778\uc7a5\ub4e4\ub3c4 \uc874\uc7ac\ud558\ub294\ub370, \uc6d0\uc608\uc6a9 \uc73c\ub85c \ub9ce\uc774 \uc7ac\ubc30\ub418\uace0 \uc788\ub294 \uac8c\ubc1c\uc120\uc778\uc7a5\uc774\ub098 \uacf5\uc791\uc120\uc778\uc7a5\uacfc \uac19\uc740 \uc885\uc740 \ub0a8\uc544\uba54\ub9ac\uce74\uc758 \uc5ec\ub7ec \uc5f4\ub300\uc6b0\ub9bc\uc758 \ub098\ubb47\uac00\uc9c0\uc5d0 \uc874\uc7ac\ud558\ub294 \uc774\ub07c\uc5d0 \ubfcc\ub9ac\ub97c \ub0b4\ub9ac\uace0 \uc11c\uc2dd\ud560 \uc218 \uc788\ub3c4\ub85d \uc9c4\ud654\ud588\ub2e4.", "paragraph_sentence": "\uc120\uc778\uc7a5 \ubd84\ud3ec\uc9c0\uc5ed\uc740 \uc8fc\ub85c \uac74\uc870\ud55c \ubc18\uac74\uc870\uc9c0\ub300\ub098 \uc0ac\ub9c9\uc9c0\uc5ed, \uace0\uc0b0\uc9c0\uc5ed\uc73c\ub85c \uc5f0 \uac15\uc6b0\ub7c9\uc774 20mm \uc774\uc0c1\uc774\uac70\ub098 \uc548\uac1c \ub4f1\uc73c\ub85c \uacc4\uc18d\ud574\uc11c \uc218\ubd84\uc774 \uacf5\uae09\ub418\ub294 \uc9c0\uc5ed\uc774\ub2e4. \uc5f0 \uac15\uc6b0\ub7c9\uc774 20mm \ubbf8\ub9cc\uc774\uace0, \uc218\ubd84\uacf5\uae09\uc774 \uc6d0\ud65c\ud558\uc9c0 \uc54a\uc740 \uacf3\uc5d0\uc11c\ub294 \uc120\uc778\uc7a5\uc774 \uc0dd\uc721\ud560 \uc218 \uc5c6\ub2e4. \ubc18\ub300\ub85c \ub2e4\uc2dc \ub2e4\uc2b5\ud55c \uc5f4\ub300\uc6b0\ub9bc \ub4f1\uc5d0 \uc801\uc751\ud55c \uc120\uc778\uc7a5\ub4e4\ub3c4 \uc874\uc7ac\ud558\ub294\ub370, \uc6d0\uc608\uc6a9 \uc73c\ub85c \ub9ce\uc774 \uc7ac\ubc30\ub418\uace0 \uc788\ub294 \uac8c\ubc1c\uc120\uc778\uc7a5\uc774\ub098 \uacf5\uc791\uc120\uc778\uc7a5\uacfc \uac19\uc740 \uc885\uc740 \ub0a8\uc544\uba54\ub9ac\uce74\uc758 \uc5ec\ub7ec \uc5f4\ub300\uc6b0\ub9bc\uc758 \ub098\ubb47\uac00\uc9c0\uc5d0 \uc874\uc7ac\ud558\ub294 \uc774\ub07c\uc5d0 \ubfcc\ub9ac\ub97c \ub0b4\ub9ac\uace0 \uc11c\uc2dd\ud560 \uc218 \uc788\ub3c4\ub85d \uc9c4\ud654\ud588\ub2e4. (\uc774\ub4e4 \uc120\uc778\uc7a5\ub4e4\uc744 \ud759\uc5d0 \uc2ec\uc9c0 \uc54a\uace0 \uc774\ub07c\uc5d0 \uc2ec\ub294 \uc774\uc720\ub294 \uc774\ub7ec\ud558\uae30 \ub54c\ubb38\uc774\ub2e4. ) \ub300\ud55c\ubbfc\uad6d\uc5d0 \uad70\ub77d\uc744 \ud615\uc131\ud55c Opuntia ficus-indica,Opuntia humifusa \uacc4\uc5f4\uc758 \uc120\uc778\uc7a5\ub4e4\ub3c4 \uc81c\uc8fc\ub3c4 \uc7a5\ub9c8 \ub4f1\uc5d0 \uacac\ub514\uba70 \uc131\uc7a5\ud558\ub294 \uc2b5\ud55c \ud658\uacbd\uc5d0 \uac15\ud55c \ubaa8\uc2b5\uc744 \ubcf4\uc5ec\uc8fc\uace0 \uc788\ub2e4.", "paragraph_answer": "\uc120\uc778\uc7a5 \ubd84\ud3ec\uc9c0\uc5ed\uc740 \uc8fc\ub85c \uac74\uc870\ud55c \ubc18\uac74\uc870\uc9c0\ub300\ub098 \uc0ac\ub9c9\uc9c0\uc5ed, \uace0\uc0b0\uc9c0\uc5ed\uc73c\ub85c \uc5f0 \uac15\uc6b0\ub7c9\uc774 20mm \uc774\uc0c1\uc774\uac70\ub098 \uc548\uac1c \ub4f1\uc73c\ub85c \uacc4\uc18d\ud574\uc11c \uc218\ubd84\uc774 \uacf5\uae09\ub418\ub294 \uc9c0\uc5ed\uc774\ub2e4. \uc5f0 \uac15\uc6b0\ub7c9\uc774 20mm \ubbf8\ub9cc\uc774\uace0, \uc218\ubd84\uacf5\uae09\uc774 \uc6d0\ud65c\ud558\uc9c0 \uc54a\uc740 \uacf3\uc5d0\uc11c\ub294 \uc120\uc778\uc7a5\uc774 \uc0dd\uc721\ud560 \uc218 \uc5c6\ub2e4. \ubc18\ub300\ub85c \ub2e4\uc2dc \ub2e4\uc2b5\ud55c \uc5f4\ub300\uc6b0\ub9bc \ub4f1\uc5d0 \uc801\uc751\ud55c \uc120\uc778\uc7a5\ub4e4\ub3c4 \uc874\uc7ac\ud558\ub294\ub370, \uc6d0\uc608\uc6a9 \uc73c\ub85c \ub9ce\uc774 \uc7ac\ubc30\ub418\uace0 \uc788\ub294 \uac8c\ubc1c\uc120\uc778\uc7a5\uc774\ub098 \uacf5\uc791\uc120\uc778\uc7a5\uacfc \uac19\uc740 \uc885\uc740 \ub0a8\uc544\uba54\ub9ac\uce74\uc758 \uc5ec\ub7ec \uc5f4\ub300\uc6b0\ub9bc\uc758 \ub098\ubb47\uac00\uc9c0\uc5d0 \uc874\uc7ac\ud558\ub294 \uc774\ub07c\uc5d0 \ubfcc\ub9ac\ub97c \ub0b4\ub9ac\uace0 \uc11c\uc2dd\ud560 \uc218 \uc788\ub3c4\ub85d \uc9c4\ud654\ud588\ub2e4. (\uc774\ub4e4 \uc120\uc778\uc7a5\ub4e4\uc744 \ud759\uc5d0 \uc2ec\uc9c0 \uc54a\uace0 \uc774\ub07c\uc5d0 \uc2ec\ub294 \uc774\uc720\ub294 \uc774\ub7ec\ud558\uae30 \ub54c\ubb38\uc774\ub2e4.) \ub300\ud55c\ubbfc\uad6d\uc5d0 \uad70\ub77d\uc744 \ud615\uc131\ud55c Opuntia ficus-indica,Opuntia humifusa \uacc4\uc5f4\uc758 \uc120\uc778\uc7a5\ub4e4\ub3c4 \uc81c\uc8fc\ub3c4 \uc7a5\ub9c8 \ub4f1\uc5d0 \uacac\ub514\uba70 \uc131\uc7a5\ud558\ub294 \uc2b5\ud55c \ud658\uacbd\uc5d0 \uac15\ud55c \ubaa8\uc2b5\uc744 \ubcf4\uc5ec\uc8fc\uace0 \uc788\ub2e4.", "sentence_answer": "\ubc18\ub300\ub85c \ub2e4\uc2dc \ub2e4\uc2b5\ud55c \uc5f4\ub300\uc6b0\ub9bc \ub4f1\uc5d0 \uc801\uc751\ud55c \uc120\uc778\uc7a5\ub4e4\ub3c4 \uc874\uc7ac\ud558\ub294\ub370, \uc6d0\uc608\uc6a9 \uc73c\ub85c \ub9ce\uc774 \uc7ac\ubc30\ub418\uace0 \uc788\ub294 \uac8c\ubc1c\uc120\uc778\uc7a5\uc774\ub098 \uacf5\uc791\uc120\uc778\uc7a5\uacfc \uac19\uc740 \uc885\uc740 \ub0a8\uc544\uba54\ub9ac\uce74\uc758 \uc5ec\ub7ec \uc5f4\ub300\uc6b0\ub9bc\uc758 \ub098\ubb47\uac00\uc9c0\uc5d0 \uc874\uc7ac\ud558\ub294 \uc774\ub07c\uc5d0 \ubfcc\ub9ac\ub97c \ub0b4\ub9ac\uace0 \uc11c\uc2dd\ud560 \uc218 \uc788\ub3c4\ub85d \uc9c4\ud654\ud588\ub2e4.", "paragraph_id": "6531921-0-1", "paragraph_question": "question: \uac8c\ubc1c\uc120\uc778\uc7a5\uc774\ub098 \uacf5\uc791\uc120\uc778\uc7a5\uc774 \ub9ce\uc774 \uc7ac\ubc30\ub418\ub294 \uc774\uc720\ub294?, context: \uc120\uc778\uc7a5 \ubd84\ud3ec\uc9c0\uc5ed\uc740 \uc8fc\ub85c \uac74\uc870\ud55c \ubc18\uac74\uc870\uc9c0\ub300\ub098 \uc0ac\ub9c9\uc9c0\uc5ed, \uace0\uc0b0\uc9c0\uc5ed\uc73c\ub85c \uc5f0 \uac15\uc6b0\ub7c9\uc774 20mm \uc774\uc0c1\uc774\uac70\ub098 \uc548\uac1c \ub4f1\uc73c\ub85c \uacc4\uc18d\ud574\uc11c \uc218\ubd84\uc774 \uacf5\uae09\ub418\ub294 \uc9c0\uc5ed\uc774\ub2e4. \uc5f0 \uac15\uc6b0\ub7c9\uc774 20mm \ubbf8\ub9cc\uc774\uace0, \uc218\ubd84\uacf5\uae09\uc774 \uc6d0\ud65c\ud558\uc9c0 \uc54a\uc740 \uacf3\uc5d0\uc11c\ub294 \uc120\uc778\uc7a5\uc774 \uc0dd\uc721\ud560 \uc218 \uc5c6\ub2e4. \ubc18\ub300\ub85c \ub2e4\uc2dc \ub2e4\uc2b5\ud55c \uc5f4\ub300\uc6b0\ub9bc \ub4f1\uc5d0 \uc801\uc751\ud55c \uc120\uc778\uc7a5\ub4e4\ub3c4 \uc874\uc7ac\ud558\ub294\ub370, \uc6d0\uc608\uc6a9\uc73c\ub85c \ub9ce\uc774 \uc7ac\ubc30\ub418\uace0 \uc788\ub294 \uac8c\ubc1c\uc120\uc778\uc7a5\uc774\ub098 \uacf5\uc791\uc120\uc778\uc7a5\uacfc \uac19\uc740 \uc885\uc740 \ub0a8\uc544\uba54\ub9ac\uce74\uc758 \uc5ec\ub7ec \uc5f4\ub300\uc6b0\ub9bc\uc758 \ub098\ubb47\uac00\uc9c0\uc5d0 \uc874\uc7ac\ud558\ub294 \uc774\ub07c\uc5d0 \ubfcc\ub9ac\ub97c \ub0b4\ub9ac\uace0 \uc11c\uc2dd\ud560 \uc218 \uc788\ub3c4\ub85d \uc9c4\ud654\ud588\ub2e4. (\uc774\ub4e4 \uc120\uc778\uc7a5\ub4e4\uc744 \ud759\uc5d0 \uc2ec\uc9c0 \uc54a\uace0 \uc774\ub07c\uc5d0 \uc2ec\ub294 \uc774\uc720\ub294 \uc774\ub7ec\ud558\uae30 \ub54c\ubb38\uc774\ub2e4.) \ub300\ud55c\ubbfc\uad6d\uc5d0 \uad70\ub77d\uc744 \ud615\uc131\ud55c Opuntia ficus-indica,Opuntia humifusa \uacc4\uc5f4\uc758 \uc120\uc778\uc7a5\ub4e4\ub3c4 \uc81c\uc8fc\ub3c4 \uc7a5\ub9c8 \ub4f1\uc5d0 \uacac\ub514\uba70 \uc131\uc7a5\ud558\ub294 \uc2b5\ud55c \ud658\uacbd\uc5d0 \uac15\ud55c \ubaa8\uc2b5\uc744 \ubcf4\uc5ec\uc8fc\uace0 \uc788\ub2e4."} {"question": "\uc804\ub77c\ub3c4 \ub3c4\uc11c\uc9c0\uc5ed\uacfc \ube44\uc2b7\ud55c \uc678\uad6d\uc740?", "paragraph": "\uc804\ub77c\ub3c4 \ub3c4\uc11c \uc9c0\uc5ed\uc740 2000\uc5ec\uac1c\uac00 \ub118\ub294 \uc12c\ub4e4\ub85c \uad6c\uc131\ub418\uc5b4 \uc788\uc5b4, \ubb34\uc218\ud788 \ub9ce\uc740 \ud3d0\uc1c4\uc801 \uc9c0\uc5ed\uc0ac\ud68c\uc5d0 \uc911\uc559\uc758 \uacf5\uad8c\ub825\uc774 \uae4a\uc219\ud558\uac8c \uad00\uc5ec\ud558\uae30 \ud798\ub4e0 \ud2b9\uc218\ud55c \uc0ac\ud68c \uad6c\uc870\ub97c \uc774\ub8e8\uace0 \uc788\ub2e4. \uc774\uc640 \ube44\uc2b7\ud55c \uacbd\uc6b0\ub85c\ub294 \ubc18\uad70\uacfc \ubb34\ubc95\uc790\ub4e4\uc774 \ud65c\uac1c\uce58\ub294 \ub9ce\uc740 \uc12c\ub4e4\uc5d0 \uce58\uc548 \ub2f9\uad6d\uc758 \ud798\uc774 \uc81c\ub300\ub85c \ubbf8\uce58\uc9c0 \uc54a\ub294 \ud544\ub9ac\ud540\uc774 \uc788\ub2e4. \ud55c\ubc88 \uc678\uc9c0\uc778\ub4e4\uc774 \ub0a9\uce58\ub97c \ub2f9\ud574\uc11c \uc774\uacf3 \uc12c \uc9c0\uc5ed\uc5d0 \ubc1c\uc744 \ub4e4\uc774\uba74, \ubc14\ub2e4\uc5d0 \ub458\ub7ec\uc2f8\uc778 \uc218\ub9ce\uc740 \uc12c\ub4e4 \uc911\uc5d0 \ud558\ub098\ub77c\ub294 \uad6c\uc870\uc801 \uc7a5\ubcbd\uacfc \ud3d0\uc1c4\uc801\uc778 \uc9c0\uc5ed \uce74\ub974\ud154\ub85c \uc778\ud574 \ud0c8\ucd9c\ud558\uae30\uac00 \uc27d\uc9c0 \uc54a\ub2e4. \uc774\ub4e4\uc740 \uc678\ubd80\ub85c\ubd80\ud130 \uace0\ub9bd\ub418\uc5b4 \uc9c0\uc5ed\uc0ac\ud68c\uc758 \ubb35\uc778 \ud558\uc5d0 \uac15\uc81c\uc801\uc73c\ub85c \ub178\ub3d9\uc774\ub098 \uc131\ub9e4\ub9e4\ub97c \ud558\uac8c \ub41c\ub2e4. \uc804\ubb38\uac00\ub4e4\uc740 \uc12c\uc774 \ub9ce\uc740 \uc804\ub77c\ub3c4\uc758 \ub3c4\uc11c \uc9c0\uc5ed\uc5d0\uc11c\ub294 \ud3d0\uc1c4\uc131\uc73c\ub85c \uc778\ud574 \uc678\uc9c0\uc778\uc5d0 \ub300\ud55c \ubc94\uc8c4\ub294 \uc740\ud3d0\uac00 \uc6a9\uc774\ud560 \uac83\uc774\ub77c\ub294 \uacf5\ub3d9\uccb4 \uad6c\uc131\uc6d0\ub4e4\uc758 \ubc94\uc8c4\uc2ec\ub9ac\uac00 \uc791\uc6a9\ud560 \uc218 \uc788\ub2e4\uace0 \uc9c0\uc801\ud55c\ub2e4.", "answer": "\ud544\ub9ac\ud540", "sentence": "\uc774\uc640 \ube44\uc2b7\ud55c \uacbd\uc6b0\ub85c\ub294 \ubc18\uad70\uacfc \ubb34\ubc95\uc790\ub4e4\uc774 \ud65c\uac1c\uce58\ub294 \ub9ce\uc740 \uc12c\ub4e4\uc5d0 \uce58\uc548 \ub2f9\uad6d\uc758 \ud798\uc774 \uc81c\ub300\ub85c \ubbf8\uce58\uc9c0 \uc54a\ub294 \ud544\ub9ac\ud540 \uc774 \uc788\ub2e4.", "paragraph_sentence": "\uc804\ub77c\ub3c4 \ub3c4\uc11c \uc9c0\uc5ed\uc740 2000\uc5ec\uac1c\uac00 \ub118\ub294 \uc12c\ub4e4\ub85c \uad6c\uc131\ub418\uc5b4 \uc788\uc5b4, \ubb34\uc218\ud788 \ub9ce\uc740 \ud3d0\uc1c4\uc801 \uc9c0\uc5ed\uc0ac\ud68c\uc5d0 \uc911\uc559\uc758 \uacf5\uad8c\ub825\uc774 \uae4a\uc219\ud558\uac8c \uad00\uc5ec\ud558\uae30 \ud798\ub4e0 \ud2b9\uc218\ud55c \uc0ac\ud68c \uad6c\uc870\ub97c \uc774\ub8e8\uace0 \uc788\ub2e4. \uc774\uc640 \ube44\uc2b7\ud55c \uacbd\uc6b0\ub85c\ub294 \ubc18\uad70\uacfc \ubb34\ubc95\uc790\ub4e4\uc774 \ud65c\uac1c\uce58\ub294 \ub9ce\uc740 \uc12c\ub4e4\uc5d0 \uce58\uc548 \ub2f9\uad6d\uc758 \ud798\uc774 \uc81c\ub300\ub85c \ubbf8\uce58\uc9c0 \uc54a\ub294 \ud544\ub9ac\ud540 \uc774 \uc788\ub2e4. \ud55c\ubc88 \uc678\uc9c0\uc778\ub4e4\uc774 \ub0a9\uce58\ub97c \ub2f9\ud574\uc11c \uc774\uacf3 \uc12c \uc9c0\uc5ed\uc5d0 \ubc1c\uc744 \ub4e4\uc774\uba74, \ubc14\ub2e4\uc5d0 \ub458\ub7ec\uc2f8\uc778 \uc218\ub9ce\uc740 \uc12c\ub4e4 \uc911\uc5d0 \ud558\ub098\ub77c\ub294 \uad6c\uc870\uc801 \uc7a5\ubcbd\uacfc \ud3d0\uc1c4\uc801\uc778 \uc9c0\uc5ed \uce74\ub974\ud154\ub85c \uc778\ud574 \ud0c8\ucd9c\ud558\uae30\uac00 \uc27d\uc9c0 \uc54a\ub2e4. \uc774\ub4e4\uc740 \uc678\ubd80\ub85c\ubd80\ud130 \uace0\ub9bd\ub418\uc5b4 \uc9c0\uc5ed\uc0ac\ud68c\uc758 \ubb35\uc778 \ud558\uc5d0 \uac15\uc81c\uc801\uc73c\ub85c \ub178\ub3d9\uc774\ub098 \uc131\ub9e4\ub9e4\ub97c \ud558\uac8c \ub41c\ub2e4. \uc804\ubb38\uac00\ub4e4\uc740 \uc12c\uc774 \ub9ce\uc740 \uc804\ub77c\ub3c4\uc758 \ub3c4\uc11c \uc9c0\uc5ed\uc5d0\uc11c\ub294 \ud3d0\uc1c4\uc131\uc73c\ub85c \uc778\ud574 \uc678\uc9c0\uc778\uc5d0 \ub300\ud55c \ubc94\uc8c4\ub294 \uc740\ud3d0\uac00 \uc6a9\uc774\ud560 \uac83\uc774\ub77c\ub294 \uacf5\ub3d9\uccb4 \uad6c\uc131\uc6d0\ub4e4\uc758 \ubc94\uc8c4\uc2ec\ub9ac\uac00 \uc791\uc6a9\ud560 \uc218 \uc788\ub2e4\uace0 \uc9c0\uc801\ud55c\ub2e4.", "paragraph_answer": "\uc804\ub77c\ub3c4 \ub3c4\uc11c \uc9c0\uc5ed\uc740 2000\uc5ec\uac1c\uac00 \ub118\ub294 \uc12c\ub4e4\ub85c \uad6c\uc131\ub418\uc5b4 \uc788\uc5b4, \ubb34\uc218\ud788 \ub9ce\uc740 \ud3d0\uc1c4\uc801 \uc9c0\uc5ed\uc0ac\ud68c\uc5d0 \uc911\uc559\uc758 \uacf5\uad8c\ub825\uc774 \uae4a\uc219\ud558\uac8c \uad00\uc5ec\ud558\uae30 \ud798\ub4e0 \ud2b9\uc218\ud55c \uc0ac\ud68c \uad6c\uc870\ub97c \uc774\ub8e8\uace0 \uc788\ub2e4. \uc774\uc640 \ube44\uc2b7\ud55c \uacbd\uc6b0\ub85c\ub294 \ubc18\uad70\uacfc \ubb34\ubc95\uc790\ub4e4\uc774 \ud65c\uac1c\uce58\ub294 \ub9ce\uc740 \uc12c\ub4e4\uc5d0 \uce58\uc548 \ub2f9\uad6d\uc758 \ud798\uc774 \uc81c\ub300\ub85c \ubbf8\uce58\uc9c0 \uc54a\ub294 \ud544\ub9ac\ud540 \uc774 \uc788\ub2e4. \ud55c\ubc88 \uc678\uc9c0\uc778\ub4e4\uc774 \ub0a9\uce58\ub97c \ub2f9\ud574\uc11c \uc774\uacf3 \uc12c \uc9c0\uc5ed\uc5d0 \ubc1c\uc744 \ub4e4\uc774\uba74, \ubc14\ub2e4\uc5d0 \ub458\ub7ec\uc2f8\uc778 \uc218\ub9ce\uc740 \uc12c\ub4e4 \uc911\uc5d0 \ud558\ub098\ub77c\ub294 \uad6c\uc870\uc801 \uc7a5\ubcbd\uacfc \ud3d0\uc1c4\uc801\uc778 \uc9c0\uc5ed \uce74\ub974\ud154\ub85c \uc778\ud574 \ud0c8\ucd9c\ud558\uae30\uac00 \uc27d\uc9c0 \uc54a\ub2e4. \uc774\ub4e4\uc740 \uc678\ubd80\ub85c\ubd80\ud130 \uace0\ub9bd\ub418\uc5b4 \uc9c0\uc5ed\uc0ac\ud68c\uc758 \ubb35\uc778 \ud558\uc5d0 \uac15\uc81c\uc801\uc73c\ub85c \ub178\ub3d9\uc774\ub098 \uc131\ub9e4\ub9e4\ub97c \ud558\uac8c \ub41c\ub2e4. \uc804\ubb38\uac00\ub4e4\uc740 \uc12c\uc774 \ub9ce\uc740 \uc804\ub77c\ub3c4\uc758 \ub3c4\uc11c \uc9c0\uc5ed\uc5d0\uc11c\ub294 \ud3d0\uc1c4\uc131\uc73c\ub85c \uc778\ud574 \uc678\uc9c0\uc778\uc5d0 \ub300\ud55c \ubc94\uc8c4\ub294 \uc740\ud3d0\uac00 \uc6a9\uc774\ud560 \uac83\uc774\ub77c\ub294 \uacf5\ub3d9\uccb4 \uad6c\uc131\uc6d0\ub4e4\uc758 \ubc94\uc8c4\uc2ec\ub9ac\uac00 \uc791\uc6a9\ud560 \uc218 \uc788\ub2e4\uace0 \uc9c0\uc801\ud55c\ub2e4.", "sentence_answer": "\uc774\uc640 \ube44\uc2b7\ud55c \uacbd\uc6b0\ub85c\ub294 \ubc18\uad70\uacfc \ubb34\ubc95\uc790\ub4e4\uc774 \ud65c\uac1c\uce58\ub294 \ub9ce\uc740 \uc12c\ub4e4\uc5d0 \uce58\uc548 \ub2f9\uad6d\uc758 \ud798\uc774 \uc81c\ub300\ub85c \ubbf8\uce58\uc9c0 \uc54a\ub294 \ud544\ub9ac\ud540 \uc774 \uc788\ub2e4.", "paragraph_id": "5862007-0-1", "paragraph_question": "question: \uc804\ub77c\ub3c4 \ub3c4\uc11c\uc9c0\uc5ed\uacfc \ube44\uc2b7\ud55c \uc678\uad6d\uc740?, context: \uc804\ub77c\ub3c4 \ub3c4\uc11c \uc9c0\uc5ed\uc740 2000\uc5ec\uac1c\uac00 \ub118\ub294 \uc12c\ub4e4\ub85c \uad6c\uc131\ub418\uc5b4 \uc788\uc5b4, \ubb34\uc218\ud788 \ub9ce\uc740 \ud3d0\uc1c4\uc801 \uc9c0\uc5ed\uc0ac\ud68c\uc5d0 \uc911\uc559\uc758 \uacf5\uad8c\ub825\uc774 \uae4a\uc219\ud558\uac8c \uad00\uc5ec\ud558\uae30 \ud798\ub4e0 \ud2b9\uc218\ud55c \uc0ac\ud68c \uad6c\uc870\ub97c \uc774\ub8e8\uace0 \uc788\ub2e4. \uc774\uc640 \ube44\uc2b7\ud55c \uacbd\uc6b0\ub85c\ub294 \ubc18\uad70\uacfc \ubb34\ubc95\uc790\ub4e4\uc774 \ud65c\uac1c\uce58\ub294 \ub9ce\uc740 \uc12c\ub4e4\uc5d0 \uce58\uc548 \ub2f9\uad6d\uc758 \ud798\uc774 \uc81c\ub300\ub85c \ubbf8\uce58\uc9c0 \uc54a\ub294 \ud544\ub9ac\ud540\uc774 \uc788\ub2e4. \ud55c\ubc88 \uc678\uc9c0\uc778\ub4e4\uc774 \ub0a9\uce58\ub97c \ub2f9\ud574\uc11c \uc774\uacf3 \uc12c \uc9c0\uc5ed\uc5d0 \ubc1c\uc744 \ub4e4\uc774\uba74, \ubc14\ub2e4\uc5d0 \ub458\ub7ec\uc2f8\uc778 \uc218\ub9ce\uc740 \uc12c\ub4e4 \uc911\uc5d0 \ud558\ub098\ub77c\ub294 \uad6c\uc870\uc801 \uc7a5\ubcbd\uacfc \ud3d0\uc1c4\uc801\uc778 \uc9c0\uc5ed \uce74\ub974\ud154\ub85c \uc778\ud574 \ud0c8\ucd9c\ud558\uae30\uac00 \uc27d\uc9c0 \uc54a\ub2e4. \uc774\ub4e4\uc740 \uc678\ubd80\ub85c\ubd80\ud130 \uace0\ub9bd\ub418\uc5b4 \uc9c0\uc5ed\uc0ac\ud68c\uc758 \ubb35\uc778 \ud558\uc5d0 \uac15\uc81c\uc801\uc73c\ub85c \ub178\ub3d9\uc774\ub098 \uc131\ub9e4\ub9e4\ub97c \ud558\uac8c \ub41c\ub2e4. \uc804\ubb38\uac00\ub4e4\uc740 \uc12c\uc774 \ub9ce\uc740 \uc804\ub77c\ub3c4\uc758 \ub3c4\uc11c \uc9c0\uc5ed\uc5d0\uc11c\ub294 \ud3d0\uc1c4\uc131\uc73c\ub85c \uc778\ud574 \uc678\uc9c0\uc778\uc5d0 \ub300\ud55c \ubc94\uc8c4\ub294 \uc740\ud3d0\uac00 \uc6a9\uc774\ud560 \uac83\uc774\ub77c\ub294 \uacf5\ub3d9\uccb4 \uad6c\uc131\uc6d0\ub4e4\uc758 \ubc94\uc8c4\uc2ec\ub9ac\uac00 \uc791\uc6a9\ud560 \uc218 \uc788\ub2e4\uace0 \uc9c0\uc801\ud55c\ub2e4."} {"question": "\ubc15\uadfc\ud61c\uac00 \uc804\ud0dc\uc77c\uc7ac\ub2e8\uc744 \ubc29\ubb38\ud558\ub824\uace0 \ud588\uc73c\ub098 \ubb34\uc0b0\ub41c \uac83\uc740 \uc5b8\uc81c\uc778\uac00?", "paragraph": "\ud55c\ud3b8 \uacbd\uc120 8\uc77c \ub4a4\uc778 2012\ub144 8\uc6d4 28\uc77c \ubc15\uadfc\ud61c\ub294 \uc804\ud0dc\uc77c\uc7ac\ub2e8\uc744 \ubc29\ubb38\ud558\ub824\uace0 \ud588\uc73c\ub098, \uc720\uc871\uacfc \ub178\ub3d9\uc790\ub4e4\uc758 \uac70\ubd80\uc640 \ubc18\ub300\ub85c \ubb34\uc0b0\ub418\uc5c8\ub2e4. \uc804\ud0dc\uc77c\uc758 \uc5ec\ub3d9\uc0dd\uc774\uc790 \uad6c \ubbfc\uc8fc\ud1b5\ud569\ub2f9 \uad6d\ud68c\uc758\uc6d0\uc778 \uc804\uc21c\uc625\uc740 \uc9c4\uc815\uc73c\ub85c \uc804\ud0dc\uc77c \uc815\uc2e0\uc744 \uc0dd\uac01\ud55c\ub2e4\uba74 \uc30d\uc6a9\ucc28\ub098 \uc6a9\uc0b0\ucc38\uc0ac \ud76c\uc0dd\uc790 \ub4f1 \uace0\ud1b5 \ub2f9\ud558\ub294 \uc0ac\ub78c\ub4e4\uc744 \uba3c\uc800 \ucc3e\uc558\uc5b4\uc57c \ud55c\ub2e4\uace0 \ub9d0\ud558\uace0, \uc774\ub978\ubc14 \uad6d\ubbfc\ub300\ud1b5\ud569\uc5d0 \ub300\ud574\uc11c\ub3c4, \uc815\ucc45\uc801\uc73c\ub85c \uc811\uadfc\uc744 \ud558\uac70\ub098 \ud604\uc548\uc774 \uc77c\uc5b4\ub098\ub294 \ud604\uc7a5\uc744 \uba3c\uc800 \ucc3e\uc544\uac00 \ubb38\uc81c\ub97c \uc5b4\ub5bb\uac8c \ud574\uacb0\ud560 \uac83\uc778\uac00\ub97c \uc774\uc57c\uae30\ud558\uba70 \ud76c\ub9dd\uc744 \uc918\uc57c \ud55c\ub2e4\uace0 \ub9d0\ud588\ub2e4. \uc804 \uc758\uc6d0\uc740 \ud604\uc7ac\uc758 \uc9c4\uc2e4\uc740 \ubbf8\ub798\uc5d0 \ub300\ud55c \uc9c0\ud5a5\uacfc \uacfc\uac70\uc758 \uc0b6\uc774 \uc77c\uce58\ud560 \ub54c \ube5b\uc744 \ubc1c\ud558\ub294 \uac83\uc774\ub77c\uba70 5\u00b716 \ucfe0\ub370\ud0c0\uc640 \uc720\uc2e0, \uad70\uc0ac \ub3c5\uc7ac\uc5d0\uc11c \uc9c0\uae08\uc758 \uc815\uc218\uc7a5\ud559\ud68c\uae4c\uc9c0 \uacfc\uac70\uc5d0 \ub300\ud55c \ucca0\uc800\ud55c \ubc18\uc131\uc774 \uc5c6\ub2e4\uba74 \uc9c0\uae08\uc758 \ub9d0\uacfc \ud589\ub3d9\uc740 \uadf8 \uc9c4\uc2e4\uc744 \uc758\uc2ec\ubc1b\uc744 \uc218 \ubc16\uc5d0 \uc5c6\ub2e4\uace0 \uc9c0\uc801\ud558\uc600\ub2e4. \uadf8\ub7ec\ub098 \uc774\uc0c1\uc77c \ub300\ubcc0\uc778\uc740 \"\ubc15 \ud6c4\ubcf4\ub97c \uc798 \uc774\ud574\ud558\uc9c0 \ubabb\ud558\ub294 \uc774\ub4e4\uc774 \uc544\ubb34\ub9ac \ubc29\ud574\ub97c \ud558\uace0 \uc7a5\ub9c9\uc744 \uce5c\ub2e4 \ud574\ub3c4 \uad6d\ubbfc\uc744 \ud1b5\ud569\ud558\uaca0\ub2e4\ub294 \ubc15 \ud6c4\ubcf4\uc758 \ud589\ubcf4\ub97c \ub9c9\uc9c0 \ubabb\ud560 \uac83\"\uc774\ub77c\uba70 \"\uad6d\ubbfc\uc744 \ubd84\uc5f4\uc2dc\ucf1c \uacc4\uce35\uac04, \uc138\ub300\uac04, \uc9c0\uc5ed\uac04 \uac08\ub4f1\uc744 \uc870\uc7a5\ud558\ub294 \uc138\ub825\uc744 \ubc18\ub4dc\uc2dc \ubb3c\ub9ac\uce58\uace0 \uad6d\ubbfc\ub300\ud1b5\ud569\uc758 '100% \ub300\ud55c\ubbfc\uad6d'\uc744 \uac74\uc124\ud560 \uac83\"\uc774\ub77c\uace0 \ub9d0\ud588\ub2e4.", "answer": "2012\ub144 8\uc6d4 28\uc77c", "sentence": "\ud55c\ud3b8 \uacbd\uc120 8\uc77c \ub4a4\uc778 2012\ub144 8\uc6d4 28\uc77c \ubc15\uadfc\ud61c\ub294 \uc804\ud0dc\uc77c\uc7ac\ub2e8\uc744 \ubc29\ubb38\ud558\ub824\uace0 \ud588\uc73c\ub098, \uc720\uc871\uacfc \ub178\ub3d9\uc790\ub4e4\uc758 \uac70\ubd80\uc640 \ubc18\ub300\ub85c \ubb34\uc0b0\ub418\uc5c8\ub2e4.", "paragraph_sentence": " \ud55c\ud3b8 \uacbd\uc120 8\uc77c \ub4a4\uc778 2012\ub144 8\uc6d4 28\uc77c \ubc15\uadfc\ud61c\ub294 \uc804\ud0dc\uc77c\uc7ac\ub2e8\uc744 \ubc29\ubb38\ud558\ub824\uace0 \ud588\uc73c\ub098, \uc720\uc871\uacfc \ub178\ub3d9\uc790\ub4e4\uc758 \uac70\ubd80\uc640 \ubc18\ub300\ub85c \ubb34\uc0b0\ub418\uc5c8\ub2e4. \uc804\ud0dc\uc77c\uc758 \uc5ec\ub3d9\uc0dd\uc774\uc790 \uad6c \ubbfc\uc8fc\ud1b5\ud569\ub2f9 \uad6d\ud68c\uc758\uc6d0\uc778 \uc804\uc21c\uc625\uc740 \uc9c4\uc815\uc73c\ub85c \uc804\ud0dc\uc77c \uc815\uc2e0\uc744 \uc0dd\uac01\ud55c\ub2e4\uba74 \uc30d\uc6a9\ucc28\ub098 \uc6a9\uc0b0\ucc38\uc0ac \ud76c\uc0dd\uc790 \ub4f1 \uace0\ud1b5 \ub2f9\ud558\ub294 \uc0ac\ub78c\ub4e4\uc744 \uba3c\uc800 \ucc3e\uc558\uc5b4\uc57c \ud55c\ub2e4\uace0 \ub9d0\ud558\uace0, \uc774\ub978\ubc14 \uad6d\ubbfc\ub300\ud1b5\ud569\uc5d0 \ub300\ud574\uc11c\ub3c4, \uc815\ucc45\uc801\uc73c\ub85c \uc811\uadfc\uc744 \ud558\uac70\ub098 \ud604\uc548\uc774 \uc77c\uc5b4\ub098\ub294 \ud604\uc7a5\uc744 \uba3c\uc800 \ucc3e\uc544\uac00 \ubb38\uc81c\ub97c \uc5b4\ub5bb\uac8c \ud574\uacb0\ud560 \uac83\uc778\uac00\ub97c \uc774\uc57c\uae30\ud558\uba70 \ud76c\ub9dd\uc744 \uc918\uc57c \ud55c\ub2e4\uace0 \ub9d0\ud588\ub2e4. \uc804 \uc758\uc6d0\uc740 \ud604\uc7ac\uc758 \uc9c4\uc2e4\uc740 \ubbf8\ub798\uc5d0 \ub300\ud55c \uc9c0\ud5a5\uacfc \uacfc\uac70\uc758 \uc0b6\uc774 \uc77c\uce58\ud560 \ub54c \ube5b\uc744 \ubc1c\ud558\ub294 \uac83\uc774\ub77c\uba70 5\u00b716 \ucfe0\ub370\ud0c0\uc640 \uc720\uc2e0, \uad70\uc0ac \ub3c5\uc7ac\uc5d0\uc11c \uc9c0\uae08\uc758 \uc815\uc218\uc7a5\ud559\ud68c\uae4c\uc9c0 \uacfc\uac70\uc5d0 \ub300\ud55c \ucca0\uc800\ud55c \ubc18\uc131\uc774 \uc5c6\ub2e4\uba74 \uc9c0\uae08\uc758 \ub9d0\uacfc \ud589\ub3d9\uc740 \uadf8 \uc9c4\uc2e4\uc744 \uc758\uc2ec\ubc1b\uc744 \uc218 \ubc16\uc5d0 \uc5c6\ub2e4\uace0 \uc9c0\uc801\ud558\uc600\ub2e4. \uadf8\ub7ec\ub098 \uc774\uc0c1\uc77c \ub300\ubcc0\uc778\uc740 \"\ubc15 \ud6c4\ubcf4\ub97c \uc798 \uc774\ud574\ud558\uc9c0 \ubabb\ud558\ub294 \uc774\ub4e4\uc774 \uc544\ubb34\ub9ac \ubc29\ud574\ub97c \ud558\uace0 \uc7a5\ub9c9\uc744 \uce5c\ub2e4 \ud574\ub3c4 \uad6d\ubbfc\uc744 \ud1b5\ud569\ud558\uaca0\ub2e4\ub294 \ubc15 \ud6c4\ubcf4\uc758 \ud589\ubcf4\ub97c \ub9c9\uc9c0 \ubabb\ud560 \uac83\"\uc774\ub77c\uba70 \"\uad6d\ubbfc\uc744 \ubd84\uc5f4\uc2dc\ucf1c \uacc4\uce35\uac04, \uc138\ub300\uac04, \uc9c0\uc5ed\uac04 \uac08\ub4f1\uc744 \uc870\uc7a5\ud558\ub294 \uc138\ub825\uc744 \ubc18\ub4dc\uc2dc \ubb3c\ub9ac\uce58\uace0 \uad6d\ubbfc\ub300\ud1b5\ud569\uc758 '100% \ub300\ud55c\ubbfc\uad6d'\uc744 \uac74\uc124\ud560 \uac83\"\uc774\ub77c\uace0 \ub9d0\ud588\ub2e4.", "paragraph_answer": "\ud55c\ud3b8 \uacbd\uc120 8\uc77c \ub4a4\uc778 2012\ub144 8\uc6d4 28\uc77c \ubc15\uadfc\ud61c\ub294 \uc804\ud0dc\uc77c\uc7ac\ub2e8\uc744 \ubc29\ubb38\ud558\ub824\uace0 \ud588\uc73c\ub098, \uc720\uc871\uacfc \ub178\ub3d9\uc790\ub4e4\uc758 \uac70\ubd80\uc640 \ubc18\ub300\ub85c \ubb34\uc0b0\ub418\uc5c8\ub2e4. \uc804\ud0dc\uc77c\uc758 \uc5ec\ub3d9\uc0dd\uc774\uc790 \uad6c \ubbfc\uc8fc\ud1b5\ud569\ub2f9 \uad6d\ud68c\uc758\uc6d0\uc778 \uc804\uc21c\uc625\uc740 \uc9c4\uc815\uc73c\ub85c \uc804\ud0dc\uc77c \uc815\uc2e0\uc744 \uc0dd\uac01\ud55c\ub2e4\uba74 \uc30d\uc6a9\ucc28\ub098 \uc6a9\uc0b0\ucc38\uc0ac \ud76c\uc0dd\uc790 \ub4f1 \uace0\ud1b5 \ub2f9\ud558\ub294 \uc0ac\ub78c\ub4e4\uc744 \uba3c\uc800 \ucc3e\uc558\uc5b4\uc57c \ud55c\ub2e4\uace0 \ub9d0\ud558\uace0, \uc774\ub978\ubc14 \uad6d\ubbfc\ub300\ud1b5\ud569\uc5d0 \ub300\ud574\uc11c\ub3c4, \uc815\ucc45\uc801\uc73c\ub85c \uc811\uadfc\uc744 \ud558\uac70\ub098 \ud604\uc548\uc774 \uc77c\uc5b4\ub098\ub294 \ud604\uc7a5\uc744 \uba3c\uc800 \ucc3e\uc544\uac00 \ubb38\uc81c\ub97c \uc5b4\ub5bb\uac8c \ud574\uacb0\ud560 \uac83\uc778\uac00\ub97c \uc774\uc57c\uae30\ud558\uba70 \ud76c\ub9dd\uc744 \uc918\uc57c \ud55c\ub2e4\uace0 \ub9d0\ud588\ub2e4. \uc804 \uc758\uc6d0\uc740 \ud604\uc7ac\uc758 \uc9c4\uc2e4\uc740 \ubbf8\ub798\uc5d0 \ub300\ud55c \uc9c0\ud5a5\uacfc \uacfc\uac70\uc758 \uc0b6\uc774 \uc77c\uce58\ud560 \ub54c \ube5b\uc744 \ubc1c\ud558\ub294 \uac83\uc774\ub77c\uba70 5\u00b716 \ucfe0\ub370\ud0c0\uc640 \uc720\uc2e0, \uad70\uc0ac \ub3c5\uc7ac\uc5d0\uc11c \uc9c0\uae08\uc758 \uc815\uc218\uc7a5\ud559\ud68c\uae4c\uc9c0 \uacfc\uac70\uc5d0 \ub300\ud55c \ucca0\uc800\ud55c \ubc18\uc131\uc774 \uc5c6\ub2e4\uba74 \uc9c0\uae08\uc758 \ub9d0\uacfc \ud589\ub3d9\uc740 \uadf8 \uc9c4\uc2e4\uc744 \uc758\uc2ec\ubc1b\uc744 \uc218 \ubc16\uc5d0 \uc5c6\ub2e4\uace0 \uc9c0\uc801\ud558\uc600\ub2e4. \uadf8\ub7ec\ub098 \uc774\uc0c1\uc77c \ub300\ubcc0\uc778\uc740 \"\ubc15 \ud6c4\ubcf4\ub97c \uc798 \uc774\ud574\ud558\uc9c0 \ubabb\ud558\ub294 \uc774\ub4e4\uc774 \uc544\ubb34\ub9ac \ubc29\ud574\ub97c \ud558\uace0 \uc7a5\ub9c9\uc744 \uce5c\ub2e4 \ud574\ub3c4 \uad6d\ubbfc\uc744 \ud1b5\ud569\ud558\uaca0\ub2e4\ub294 \ubc15 \ud6c4\ubcf4\uc758 \ud589\ubcf4\ub97c \ub9c9\uc9c0 \ubabb\ud560 \uac83\"\uc774\ub77c\uba70 \"\uad6d\ubbfc\uc744 \ubd84\uc5f4\uc2dc\ucf1c \uacc4\uce35\uac04, \uc138\ub300\uac04, \uc9c0\uc5ed\uac04 \uac08\ub4f1\uc744 \uc870\uc7a5\ud558\ub294 \uc138\ub825\uc744 \ubc18\ub4dc\uc2dc \ubb3c\ub9ac\uce58\uace0 \uad6d\ubbfc\ub300\ud1b5\ud569\uc758 '100% \ub300\ud55c\ubbfc\uad6d'\uc744 \uac74\uc124\ud560 \uac83\"\uc774\ub77c\uace0 \ub9d0\ud588\ub2e4.", "sentence_answer": "\ud55c\ud3b8 \uacbd\uc120 8\uc77c \ub4a4\uc778 2012\ub144 8\uc6d4 28\uc77c \ubc15\uadfc\ud61c\ub294 \uc804\ud0dc\uc77c\uc7ac\ub2e8\uc744 \ubc29\ubb38\ud558\ub824\uace0 \ud588\uc73c\ub098, \uc720\uc871\uacfc \ub178\ub3d9\uc790\ub4e4\uc758 \uac70\ubd80\uc640 \ubc18\ub300\ub85c \ubb34\uc0b0\ub418\uc5c8\ub2e4.", "paragraph_id": "6581158-13-0", "paragraph_question": "question: \ubc15\uadfc\ud61c\uac00 \uc804\ud0dc\uc77c\uc7ac\ub2e8\uc744 \ubc29\ubb38\ud558\ub824\uace0 \ud588\uc73c\ub098 \ubb34\uc0b0\ub41c \uac83\uc740 \uc5b8\uc81c\uc778\uac00?, context: \ud55c\ud3b8 \uacbd\uc120 8\uc77c \ub4a4\uc778 2012\ub144 8\uc6d4 28\uc77c \ubc15\uadfc\ud61c\ub294 \uc804\ud0dc\uc77c\uc7ac\ub2e8\uc744 \ubc29\ubb38\ud558\ub824\uace0 \ud588\uc73c\ub098, \uc720\uc871\uacfc \ub178\ub3d9\uc790\ub4e4\uc758 \uac70\ubd80\uc640 \ubc18\ub300\ub85c \ubb34\uc0b0\ub418\uc5c8\ub2e4. \uc804\ud0dc\uc77c\uc758 \uc5ec\ub3d9\uc0dd\uc774\uc790 \uad6c \ubbfc\uc8fc\ud1b5\ud569\ub2f9 \uad6d\ud68c\uc758\uc6d0\uc778 \uc804\uc21c\uc625\uc740 \uc9c4\uc815\uc73c\ub85c \uc804\ud0dc\uc77c \uc815\uc2e0\uc744 \uc0dd\uac01\ud55c\ub2e4\uba74 \uc30d\uc6a9\ucc28\ub098 \uc6a9\uc0b0\ucc38\uc0ac \ud76c\uc0dd\uc790 \ub4f1 \uace0\ud1b5 \ub2f9\ud558\ub294 \uc0ac\ub78c\ub4e4\uc744 \uba3c\uc800 \ucc3e\uc558\uc5b4\uc57c \ud55c\ub2e4\uace0 \ub9d0\ud558\uace0, \uc774\ub978\ubc14 \uad6d\ubbfc\ub300\ud1b5\ud569\uc5d0 \ub300\ud574\uc11c\ub3c4, \uc815\ucc45\uc801\uc73c\ub85c \uc811\uadfc\uc744 \ud558\uac70\ub098 \ud604\uc548\uc774 \uc77c\uc5b4\ub098\ub294 \ud604\uc7a5\uc744 \uba3c\uc800 \ucc3e\uc544\uac00 \ubb38\uc81c\ub97c \uc5b4\ub5bb\uac8c \ud574\uacb0\ud560 \uac83\uc778\uac00\ub97c \uc774\uc57c\uae30\ud558\uba70 \ud76c\ub9dd\uc744 \uc918\uc57c \ud55c\ub2e4\uace0 \ub9d0\ud588\ub2e4. \uc804 \uc758\uc6d0\uc740 \ud604\uc7ac\uc758 \uc9c4\uc2e4\uc740 \ubbf8\ub798\uc5d0 \ub300\ud55c \uc9c0\ud5a5\uacfc \uacfc\uac70\uc758 \uc0b6\uc774 \uc77c\uce58\ud560 \ub54c \ube5b\uc744 \ubc1c\ud558\ub294 \uac83\uc774\ub77c\uba70 5\u00b716 \ucfe0\ub370\ud0c0\uc640 \uc720\uc2e0, \uad70\uc0ac \ub3c5\uc7ac\uc5d0\uc11c \uc9c0\uae08\uc758 \uc815\uc218\uc7a5\ud559\ud68c\uae4c\uc9c0 \uacfc\uac70\uc5d0 \ub300\ud55c \ucca0\uc800\ud55c \ubc18\uc131\uc774 \uc5c6\ub2e4\uba74 \uc9c0\uae08\uc758 \ub9d0\uacfc \ud589\ub3d9\uc740 \uadf8 \uc9c4\uc2e4\uc744 \uc758\uc2ec\ubc1b\uc744 \uc218 \ubc16\uc5d0 \uc5c6\ub2e4\uace0 \uc9c0\uc801\ud558\uc600\ub2e4. \uadf8\ub7ec\ub098 \uc774\uc0c1\uc77c \ub300\ubcc0\uc778\uc740 \"\ubc15 \ud6c4\ubcf4\ub97c \uc798 \uc774\ud574\ud558\uc9c0 \ubabb\ud558\ub294 \uc774\ub4e4\uc774 \uc544\ubb34\ub9ac \ubc29\ud574\ub97c \ud558\uace0 \uc7a5\ub9c9\uc744 \uce5c\ub2e4 \ud574\ub3c4 \uad6d\ubbfc\uc744 \ud1b5\ud569\ud558\uaca0\ub2e4\ub294 \ubc15 \ud6c4\ubcf4\uc758 \ud589\ubcf4\ub97c \ub9c9\uc9c0 \ubabb\ud560 \uac83\"\uc774\ub77c\uba70 \"\uad6d\ubbfc\uc744 \ubd84\uc5f4\uc2dc\ucf1c \uacc4\uce35\uac04, \uc138\ub300\uac04, \uc9c0\uc5ed\uac04 \uac08\ub4f1\uc744 \uc870\uc7a5\ud558\ub294 \uc138\ub825\uc744 \ubc18\ub4dc\uc2dc \ubb3c\ub9ac\uce58\uace0 \uad6d\ubbfc\ub300\ud1b5\ud569\uc758 '100% \ub300\ud55c\ubbfc\uad6d'\uc744 \uac74\uc124\ud560 \uac83\"\uc774\ub77c\uace0 \ub9d0\ud588\ub2e4."} {"question": "\ucc3d\ub2e8 \ucd08\uae30\uc5d0 \uc120\uc218 \ud310\ub9e4\ub85c \uc5f0\uba85\ud558\uc9c0 \uc54a\uaca0\ub2e4\uace0 \uc8fc\uc7a5\ud55c \uac83\uc740?", "paragraph": "2008\ub144 11\uc6d4 14\uc77c, \uc7a5\uc6d0\uc0bc\uc774 \uc0bc\uc131 \ub77c\uc774\uc628\uc988\ub85c \ud2b8\ub808\uc774\ub4dc\uac00 \uc131\uc0ac\ub418\uc5c8\ub2e4\ub294 \uae30\uc0ac\uac00 \ub098\uc624\uc790 \uc77c\uac01\uc5d0\uc11c\ub294 \uc8fc\ucd95 \uc120\uc218\ub97c \ud314\uc544\uc11c \uad6c\ub2e8\uc744 \uc6b4\uc601\ud558\ub294 \ud788\uc5b4\ub85c\uc988\uc758 \ud589\ud0dc\uc640 \uadf8\ub7f0 \ud300\uc758 \uae30\ub465\uc744 \ube7c\uc624\ub294 \uc0bc\uc131 \ub77c\uc774\uc628\uc988\uc5d0 \uc5c4\uccad\ub09c \ube44\ub09c\uc744 \uc3df\uc544\ub0c8\ub2e4. \uadf8\ub4e4\uc740 \uc608\uc804\uc5d0 \uc8fc\ucd95 \uc120\uc218\ub97c \ud314\uc544 \uc790\uae08\uc744 \ubaa8\uc558\ub358 \uc30d\ubc29\uc6b8 \ub808\uc774\ub354\uc2a4\uc758 \uc608\ub97c \ub4e4\uba70, \ud788\uc5b4\ub85c\uc988\ub3c4 \uc870\ub9cc\uac04 \ud574\uccb4\ud560 \uc218\ub3c4 \uc788\ub2e4\uace0 \ud070 \uc6b0\ub824\ub97c \ud588\ub2e4. \uc774\uc640 \uac19\uc740 \uc0ac\ud0dc\ub97c \uc811\ud55c \ub204\ub9ac\uafbc\ub4e4\uc740 \ud788\uc5b4\ub85c\uc988\uc640 \uc0bc\uc131\uc758 \ud589\ud0dc\uc5d0 \uc5c4\uccad\ub09c \ube44\ub09c\uc744 \uc3df\uc544\ub0c8\ub2e4. \ub204\ub9ac\uafbc\ub4e4\uc740 \ud788\uc5b4\ub85c\uc988\uc758 \ubaa8\uae30\uc5c5 \uaca9\uc774\uc790 \ucc3d\ub2e8 \ucd08\uae30\uc5d0 \uc120\uc218 \ud310\ub9e4\ub85c \uc5f0\uba85\ud558\uc9c0 \uc54a\uaca0\ub2e4\uace0 \uc57d\uc18d\ud588\ub358 \uc13c\ud14c\ub2c8\uc5bc \uc778\ubca0\uc2a4\ud2b8\uba3c\ud2b8\uc5d0 \ube44\ub09c\uc744 \uc3df\uc544\ub0b4\uc5c8\uace0, \uc0bc\uc131 \ub77c\uc774\uc628\uc988 \ub610\ud55c \uc7a5\uc6d0\uc0bc\uc758 \uc601\uc785\uc744 \uc704\ud574 \uafb8\uc900\ud788 \ucd94\ud30c\ub97c \ub358\uc838\uc654\ub358 \uac83\uc73c\ub85c \uc54c\ub824\uc838 \ub204\ub9ac\uafbc\ub4e4\uc758 \ube44\ub09c\uc758 \ud654\uc0b4\uc744 \ud53c\ud558\uc9c0 \ubabb\ud558\uc600\ub2e4. \uc774 \ud2b8\ub808\uc774\ub4dc\ub294 \uace7\ubc14\ub85c \ud574\ub2f9 \ub450 \uad6c\ub2e8\uc744 \uc81c\uc678\ud55c \ub098\uba38\uc9c0 6\uac1c \uad6c\ub2e8\uc758 \uc5c4\uccad\ub09c \ubc18\ubc1c\uc744 \ubd88\ub7ec\uc654\ub2e4. \uadf8\ub4e4\uc740 \ud55c \ubaa9\uc18c\ub9ac\ub85c \ud2b8\ub808\uc774\ub4dc\ub97c \ubcf4\ub958\ud574\uc57c \ud55c\ub2e4\uace0 \uc785\uc744 \ubaa8\uc558\ub2e4. \uc544\uc2dc\uc544 \uc2dc\ub9ac\uc988\ub97c \uce58\ub974\uace0 \uc788\uc5c8\ub358 SK \uc640\uc774\ubc88\uc2a4\uc758 \uac10\ub3c5\uc778 \uae40\uc131\uadfc \uac10\ub3c5\uc740 \uc77c\ubcf8 \ud604\uc9c0\uc5d0\uc11c \uadf8 \uc18c\uc2dd\uc744 \uc811\ud558\uace0\ub294 \ud070 \uc6b0\ub824\ub97c \ud558\uc600\ub2e4. \uae40\uc131\uadfc \uac10\ub3c5\uc740 \uc120\uc218\ub97c \ud314\uc544\uc11c \uad6c\ub2e8\uc744 \uc6b4\uc601\ud558\ub2e4 \uacf5\uba78\ud558\uc600\ub358 \uc30d\ubc29\uc6b8 \ub808\uc774\ub354\uc2a4\uc758 \uac10\ub3c5\uc774\uc600\ub2e4.", "answer": "\uc13c\ud14c\ub2c8\uc5bc \uc778\ubca0\uc2a4\ud2b8\uba3c\ud2b8", "sentence": "\ub204\ub9ac\uafbc\ub4e4\uc740 \ud788\uc5b4\ub85c\uc988\uc758 \ubaa8\uae30\uc5c5 \uaca9\uc774\uc790 \ucc3d\ub2e8 \ucd08\uae30\uc5d0 \uc120\uc218 \ud310\ub9e4\ub85c \uc5f0\uba85\ud558\uc9c0 \uc54a\uaca0\ub2e4\uace0 \uc57d\uc18d\ud588\ub358 \uc13c\ud14c\ub2c8\uc5bc \uc778\ubca0\uc2a4\ud2b8\uba3c\ud2b8 \uc5d0 \ube44\ub09c\uc744 \uc3df\uc544\ub0b4\uc5c8\uace0, \uc0bc\uc131 \ub77c\uc774\uc628\uc988 \ub610\ud55c \uc7a5\uc6d0\uc0bc\uc758 \uc601\uc785\uc744 \uc704\ud574 \uafb8\uc900\ud788 \ucd94\ud30c\ub97c \ub358\uc838\uc654\ub358 \uac83\uc73c\ub85c \uc54c\ub824\uc838 \ub204\ub9ac\uafbc\ub4e4\uc758 \ube44\ub09c\uc758 \ud654\uc0b4\uc744 \ud53c\ud558\uc9c0 \ubabb\ud558\uc600\ub2e4.", "paragraph_sentence": "2008\ub144 11\uc6d4 14\uc77c, \uc7a5\uc6d0\uc0bc\uc774 \uc0bc\uc131 \ub77c\uc774\uc628\uc988\ub85c \ud2b8\ub808\uc774\ub4dc\uac00 \uc131\uc0ac\ub418\uc5c8\ub2e4\ub294 \uae30\uc0ac\uac00 \ub098\uc624\uc790 \uc77c\uac01\uc5d0\uc11c\ub294 \uc8fc\ucd95 \uc120\uc218\ub97c \ud314\uc544\uc11c \uad6c\ub2e8\uc744 \uc6b4\uc601\ud558\ub294 \ud788\uc5b4\ub85c\uc988\uc758 \ud589\ud0dc\uc640 \uadf8\ub7f0 \ud300\uc758 \uae30\ub465\uc744 \ube7c\uc624\ub294 \uc0bc\uc131 \ub77c\uc774\uc628\uc988\uc5d0 \uc5c4\uccad\ub09c \ube44\ub09c\uc744 \uc3df\uc544\ub0c8\ub2e4. \uadf8\ub4e4\uc740 \uc608\uc804\uc5d0 \uc8fc\ucd95 \uc120\uc218\ub97c \ud314\uc544 \uc790\uae08\uc744 \ubaa8\uc558\ub358 \uc30d\ubc29\uc6b8 \ub808\uc774\ub354\uc2a4\uc758 \uc608\ub97c \ub4e4\uba70, \ud788\uc5b4\ub85c\uc988\ub3c4 \uc870\ub9cc\uac04 \ud574\uccb4\ud560 \uc218\ub3c4 \uc788\ub2e4\uace0 \ud070 \uc6b0\ub824\ub97c \ud588\ub2e4. \uc774\uc640 \uac19\uc740 \uc0ac\ud0dc\ub97c \uc811\ud55c \ub204\ub9ac\uafbc\ub4e4\uc740 \ud788\uc5b4\ub85c\uc988\uc640 \uc0bc\uc131\uc758 \ud589\ud0dc\uc5d0 \uc5c4\uccad\ub09c \ube44\ub09c\uc744 \uc3df\uc544\ub0c8\ub2e4. \ub204\ub9ac\uafbc\ub4e4\uc740 \ud788\uc5b4\ub85c\uc988\uc758 \ubaa8\uae30\uc5c5 \uaca9\uc774\uc790 \ucc3d\ub2e8 \ucd08\uae30\uc5d0 \uc120\uc218 \ud310\ub9e4\ub85c \uc5f0\uba85\ud558\uc9c0 \uc54a\uaca0\ub2e4\uace0 \uc57d\uc18d\ud588\ub358 \uc13c\ud14c\ub2c8\uc5bc \uc778\ubca0\uc2a4\ud2b8\uba3c\ud2b8 \uc5d0 \ube44\ub09c\uc744 \uc3df\uc544\ub0b4\uc5c8\uace0, \uc0bc\uc131 \ub77c\uc774\uc628\uc988 \ub610\ud55c \uc7a5\uc6d0\uc0bc\uc758 \uc601\uc785\uc744 \uc704\ud574 \uafb8\uc900\ud788 \ucd94\ud30c\ub97c \ub358\uc838\uc654\ub358 \uac83\uc73c\ub85c \uc54c\ub824\uc838 \ub204\ub9ac\uafbc\ub4e4\uc758 \ube44\ub09c\uc758 \ud654\uc0b4\uc744 \ud53c\ud558\uc9c0 \ubabb\ud558\uc600\ub2e4. \uc774 \ud2b8\ub808\uc774\ub4dc\ub294 \uace7\ubc14\ub85c \ud574\ub2f9 \ub450 \uad6c\ub2e8\uc744 \uc81c\uc678\ud55c \ub098\uba38\uc9c0 6\uac1c \uad6c\ub2e8\uc758 \uc5c4\uccad\ub09c \ubc18\ubc1c\uc744 \ubd88\ub7ec\uc654\ub2e4. \uadf8\ub4e4\uc740 \ud55c \ubaa9\uc18c\ub9ac\ub85c \ud2b8\ub808\uc774\ub4dc\ub97c \ubcf4\ub958\ud574\uc57c \ud55c\ub2e4\uace0 \uc785\uc744 \ubaa8\uc558\ub2e4. \uc544\uc2dc\uc544 \uc2dc\ub9ac\uc988\ub97c \uce58\ub974\uace0 \uc788\uc5c8\ub358 SK \uc640\uc774\ubc88\uc2a4\uc758 \uac10\ub3c5\uc778 \uae40\uc131\uadfc \uac10\ub3c5\uc740 \uc77c\ubcf8 \ud604\uc9c0\uc5d0\uc11c \uadf8 \uc18c\uc2dd\uc744 \uc811\ud558\uace0\ub294 \ud070 \uc6b0\ub824\ub97c \ud558\uc600\ub2e4. \uae40\uc131\uadfc \uac10\ub3c5\uc740 \uc120\uc218\ub97c \ud314\uc544\uc11c \uad6c\ub2e8\uc744 \uc6b4\uc601\ud558\ub2e4 \uacf5\uba78\ud558\uc600\ub358 \uc30d\ubc29\uc6b8 \ub808\uc774\ub354\uc2a4\uc758 \uac10\ub3c5\uc774\uc600\ub2e4.", "paragraph_answer": "2008\ub144 11\uc6d4 14\uc77c, \uc7a5\uc6d0\uc0bc\uc774 \uc0bc\uc131 \ub77c\uc774\uc628\uc988\ub85c \ud2b8\ub808\uc774\ub4dc\uac00 \uc131\uc0ac\ub418\uc5c8\ub2e4\ub294 \uae30\uc0ac\uac00 \ub098\uc624\uc790 \uc77c\uac01\uc5d0\uc11c\ub294 \uc8fc\ucd95 \uc120\uc218\ub97c \ud314\uc544\uc11c \uad6c\ub2e8\uc744 \uc6b4\uc601\ud558\ub294 \ud788\uc5b4\ub85c\uc988\uc758 \ud589\ud0dc\uc640 \uadf8\ub7f0 \ud300\uc758 \uae30\ub465\uc744 \ube7c\uc624\ub294 \uc0bc\uc131 \ub77c\uc774\uc628\uc988\uc5d0 \uc5c4\uccad\ub09c \ube44\ub09c\uc744 \uc3df\uc544\ub0c8\ub2e4. \uadf8\ub4e4\uc740 \uc608\uc804\uc5d0 \uc8fc\ucd95 \uc120\uc218\ub97c \ud314\uc544 \uc790\uae08\uc744 \ubaa8\uc558\ub358 \uc30d\ubc29\uc6b8 \ub808\uc774\ub354\uc2a4\uc758 \uc608\ub97c \ub4e4\uba70, \ud788\uc5b4\ub85c\uc988\ub3c4 \uc870\ub9cc\uac04 \ud574\uccb4\ud560 \uc218\ub3c4 \uc788\ub2e4\uace0 \ud070 \uc6b0\ub824\ub97c \ud588\ub2e4. \uc774\uc640 \uac19\uc740 \uc0ac\ud0dc\ub97c \uc811\ud55c \ub204\ub9ac\uafbc\ub4e4\uc740 \ud788\uc5b4\ub85c\uc988\uc640 \uc0bc\uc131\uc758 \ud589\ud0dc\uc5d0 \uc5c4\uccad\ub09c \ube44\ub09c\uc744 \uc3df\uc544\ub0c8\ub2e4. \ub204\ub9ac\uafbc\ub4e4\uc740 \ud788\uc5b4\ub85c\uc988\uc758 \ubaa8\uae30\uc5c5 \uaca9\uc774\uc790 \ucc3d\ub2e8 \ucd08\uae30\uc5d0 \uc120\uc218 \ud310\ub9e4\ub85c \uc5f0\uba85\ud558\uc9c0 \uc54a\uaca0\ub2e4\uace0 \uc57d\uc18d\ud588\ub358 \uc13c\ud14c\ub2c8\uc5bc \uc778\ubca0\uc2a4\ud2b8\uba3c\ud2b8 \uc5d0 \ube44\ub09c\uc744 \uc3df\uc544\ub0b4\uc5c8\uace0, \uc0bc\uc131 \ub77c\uc774\uc628\uc988 \ub610\ud55c \uc7a5\uc6d0\uc0bc\uc758 \uc601\uc785\uc744 \uc704\ud574 \uafb8\uc900\ud788 \ucd94\ud30c\ub97c \ub358\uc838\uc654\ub358 \uac83\uc73c\ub85c \uc54c\ub824\uc838 \ub204\ub9ac\uafbc\ub4e4\uc758 \ube44\ub09c\uc758 \ud654\uc0b4\uc744 \ud53c\ud558\uc9c0 \ubabb\ud558\uc600\ub2e4. \uc774 \ud2b8\ub808\uc774\ub4dc\ub294 \uace7\ubc14\ub85c \ud574\ub2f9 \ub450 \uad6c\ub2e8\uc744 \uc81c\uc678\ud55c \ub098\uba38\uc9c0 6\uac1c \uad6c\ub2e8\uc758 \uc5c4\uccad\ub09c \ubc18\ubc1c\uc744 \ubd88\ub7ec\uc654\ub2e4. \uadf8\ub4e4\uc740 \ud55c \ubaa9\uc18c\ub9ac\ub85c \ud2b8\ub808\uc774\ub4dc\ub97c \ubcf4\ub958\ud574\uc57c \ud55c\ub2e4\uace0 \uc785\uc744 \ubaa8\uc558\ub2e4. \uc544\uc2dc\uc544 \uc2dc\ub9ac\uc988\ub97c \uce58\ub974\uace0 \uc788\uc5c8\ub358 SK \uc640\uc774\ubc88\uc2a4\uc758 \uac10\ub3c5\uc778 \uae40\uc131\uadfc \uac10\ub3c5\uc740 \uc77c\ubcf8 \ud604\uc9c0\uc5d0\uc11c \uadf8 \uc18c\uc2dd\uc744 \uc811\ud558\uace0\ub294 \ud070 \uc6b0\ub824\ub97c \ud558\uc600\ub2e4. \uae40\uc131\uadfc \uac10\ub3c5\uc740 \uc120\uc218\ub97c \ud314\uc544\uc11c \uad6c\ub2e8\uc744 \uc6b4\uc601\ud558\ub2e4 \uacf5\uba78\ud558\uc600\ub358 \uc30d\ubc29\uc6b8 \ub808\uc774\ub354\uc2a4\uc758 \uac10\ub3c5\uc774\uc600\ub2e4.", "sentence_answer": "\ub204\ub9ac\uafbc\ub4e4\uc740 \ud788\uc5b4\ub85c\uc988\uc758 \ubaa8\uae30\uc5c5 \uaca9\uc774\uc790 \ucc3d\ub2e8 \ucd08\uae30\uc5d0 \uc120\uc218 \ud310\ub9e4\ub85c \uc5f0\uba85\ud558\uc9c0 \uc54a\uaca0\ub2e4\uace0 \uc57d\uc18d\ud588\ub358 \uc13c\ud14c\ub2c8\uc5bc \uc778\ubca0\uc2a4\ud2b8\uba3c\ud2b8 \uc5d0 \ube44\ub09c\uc744 \uc3df\uc544\ub0b4\uc5c8\uace0, \uc0bc\uc131 \ub77c\uc774\uc628\uc988 \ub610\ud55c \uc7a5\uc6d0\uc0bc\uc758 \uc601\uc785\uc744 \uc704\ud574 \uafb8\uc900\ud788 \ucd94\ud30c\ub97c \ub358\uc838\uc654\ub358 \uac83\uc73c\ub85c \uc54c\ub824\uc838 \ub204\ub9ac\uafbc\ub4e4\uc758 \ube44\ub09c\uc758 \ud654\uc0b4\uc744 \ud53c\ud558\uc9c0 \ubabb\ud558\uc600\ub2e4.", "paragraph_id": "6533929-26-1", "paragraph_question": "question: \ucc3d\ub2e8 \ucd08\uae30\uc5d0 \uc120\uc218 \ud310\ub9e4\ub85c \uc5f0\uba85\ud558\uc9c0 \uc54a\uaca0\ub2e4\uace0 \uc8fc\uc7a5\ud55c \uac83\uc740?, context: 2008\ub144 11\uc6d4 14\uc77c, \uc7a5\uc6d0\uc0bc\uc774 \uc0bc\uc131 \ub77c\uc774\uc628\uc988\ub85c \ud2b8\ub808\uc774\ub4dc\uac00 \uc131\uc0ac\ub418\uc5c8\ub2e4\ub294 \uae30\uc0ac\uac00 \ub098\uc624\uc790 \uc77c\uac01\uc5d0\uc11c\ub294 \uc8fc\ucd95 \uc120\uc218\ub97c \ud314\uc544\uc11c \uad6c\ub2e8\uc744 \uc6b4\uc601\ud558\ub294 \ud788\uc5b4\ub85c\uc988\uc758 \ud589\ud0dc\uc640 \uadf8\ub7f0 \ud300\uc758 \uae30\ub465\uc744 \ube7c\uc624\ub294 \uc0bc\uc131 \ub77c\uc774\uc628\uc988\uc5d0 \uc5c4\uccad\ub09c \ube44\ub09c\uc744 \uc3df\uc544\ub0c8\ub2e4. \uadf8\ub4e4\uc740 \uc608\uc804\uc5d0 \uc8fc\ucd95 \uc120\uc218\ub97c \ud314\uc544 \uc790\uae08\uc744 \ubaa8\uc558\ub358 \uc30d\ubc29\uc6b8 \ub808\uc774\ub354\uc2a4\uc758 \uc608\ub97c \ub4e4\uba70, \ud788\uc5b4\ub85c\uc988\ub3c4 \uc870\ub9cc\uac04 \ud574\uccb4\ud560 \uc218\ub3c4 \uc788\ub2e4\uace0 \ud070 \uc6b0\ub824\ub97c \ud588\ub2e4. \uc774\uc640 \uac19\uc740 \uc0ac\ud0dc\ub97c \uc811\ud55c \ub204\ub9ac\uafbc\ub4e4\uc740 \ud788\uc5b4\ub85c\uc988\uc640 \uc0bc\uc131\uc758 \ud589\ud0dc\uc5d0 \uc5c4\uccad\ub09c \ube44\ub09c\uc744 \uc3df\uc544\ub0c8\ub2e4. \ub204\ub9ac\uafbc\ub4e4\uc740 \ud788\uc5b4\ub85c\uc988\uc758 \ubaa8\uae30\uc5c5 \uaca9\uc774\uc790 \ucc3d\ub2e8 \ucd08\uae30\uc5d0 \uc120\uc218 \ud310\ub9e4\ub85c \uc5f0\uba85\ud558\uc9c0 \uc54a\uaca0\ub2e4\uace0 \uc57d\uc18d\ud588\ub358 \uc13c\ud14c\ub2c8\uc5bc \uc778\ubca0\uc2a4\ud2b8\uba3c\ud2b8\uc5d0 \ube44\ub09c\uc744 \uc3df\uc544\ub0b4\uc5c8\uace0, \uc0bc\uc131 \ub77c\uc774\uc628\uc988 \ub610\ud55c \uc7a5\uc6d0\uc0bc\uc758 \uc601\uc785\uc744 \uc704\ud574 \uafb8\uc900\ud788 \ucd94\ud30c\ub97c \ub358\uc838\uc654\ub358 \uac83\uc73c\ub85c \uc54c\ub824\uc838 \ub204\ub9ac\uafbc\ub4e4\uc758 \ube44\ub09c\uc758 \ud654\uc0b4\uc744 \ud53c\ud558\uc9c0 \ubabb\ud558\uc600\ub2e4. \uc774 \ud2b8\ub808\uc774\ub4dc\ub294 \uace7\ubc14\ub85c \ud574\ub2f9 \ub450 \uad6c\ub2e8\uc744 \uc81c\uc678\ud55c \ub098\uba38\uc9c0 6\uac1c \uad6c\ub2e8\uc758 \uc5c4\uccad\ub09c \ubc18\ubc1c\uc744 \ubd88\ub7ec\uc654\ub2e4. \uadf8\ub4e4\uc740 \ud55c \ubaa9\uc18c\ub9ac\ub85c \ud2b8\ub808\uc774\ub4dc\ub97c \ubcf4\ub958\ud574\uc57c \ud55c\ub2e4\uace0 \uc785\uc744 \ubaa8\uc558\ub2e4. \uc544\uc2dc\uc544 \uc2dc\ub9ac\uc988\ub97c \uce58\ub974\uace0 \uc788\uc5c8\ub358 SK \uc640\uc774\ubc88\uc2a4\uc758 \uac10\ub3c5\uc778 \uae40\uc131\uadfc \uac10\ub3c5\uc740 \uc77c\ubcf8 \ud604\uc9c0\uc5d0\uc11c \uadf8 \uc18c\uc2dd\uc744 \uc811\ud558\uace0\ub294 \ud070 \uc6b0\ub824\ub97c \ud558\uc600\ub2e4. \uae40\uc131\uadfc \uac10\ub3c5\uc740 \uc120\uc218\ub97c \ud314\uc544\uc11c \uad6c\ub2e8\uc744 \uc6b4\uc601\ud558\ub2e4 \uacf5\uba78\ud558\uc600\ub358 \uc30d\ubc29\uc6b8 \ub808\uc774\ub354\uc2a4\uc758 \uac10\ub3c5\uc774\uc600\ub2e4."} {"question": "2017\ub144 \uae40\uc720\uc815\uacfc \uc7ac\uacc4\uc57d\uc744 \ub9fa\uc740 \uc18c\uc18d\uc0ac\ub294?", "paragraph": "2017\ub144, \uae40\uc720\uc815\uc740 \ub85c\ub9e8\uc2a4 \ucf54\ubbf8\ub514 \uc601\ud654 \u300a\uc0ac\ub791\ud558\uae30 \ub54c\ubb38\uc5d0\u300b\uc5d0\uc11c \ucc28\ud0dc\ud604, \uc11c\ud604\uc9c4\uacfc \uacf5\ub3d9\uc73c\ub85c \uc8fc\uc5f0\uc744 \ub9e1\uc544 \ud65c\uc57d\ud558\uc600\ub2e4. \uc774 \uc601\ud654\ub294 \uc0ac\ub791\uc5d0 \uc11c\ud230 \uc0ac\ub78c\ub4e4\uc758 \ub9c8\uc74c\uc744 \ud2b9\ubcc4\ud55c \ubc29\ubc95\uc73c\ub85c \uc774\uc5b4\uc8fc\ub294 \ud790\ub9c1 \ucf54\ubbf8\ub514\ub85c \uae40\uc720\uc815\uc740 4\ucc28\uc6d0\uc758 \uc5c9\ub6b1 \uc18c\ub140 \uc2a4\uceec\ub9ac \uc5ed\ud560\uc744 \uc5f0\uae30\ud558\uc600\ub2e4. \uc9c0\ub09c 2015\ub144 7\uc6d4\ubd80\ud130 10\uc6d4\uc5d0 \uac78\uccd0 \ucd2c\uc601\uc744 \ub9c8\uccd0, 2017\ub144 1\uc6d4 4\uc77c \uac1c\ubd09 \ub418\uc5c8\ub2e4. \uc601\ud654\ub294 \uc804\uad6d \uad00\uac1d\uc218 34\ub9cc\uba85\uc744 \ub3d9\uc6d0\ud558\uba70 \ud765\ud589 \ucc38\ud328\ud588\ub2e4. \uae40\uc720\uc815\uc740 2017\ub144 2\uc6d4 4\uc77c \ud0c0\uc774\uc644\uc758 \ud0c0\uc774\ubca0\uc774\uc5d0\uc11c \uadf8\ub140\uc758 \uccab \ud32c\ubbf8\ud305\uc774\uc790 \ud574\uc678 \ud32c\ubbf8\ud305\uc744 \uac1c\ucd5c\ud558\uc600\ub2e4. 5\uc6d4 20\uc77c\uc5d0\ub294 \uc2f1\uac00\ud3ec\ub974\uc5d0\uc11c \ub450 \ubc88\uc9f8 \ud32c\ubbf8\ud305\uc744 \uac1c\ucd5c\ud55c\ub370 \uc774\uc5b4 9\uc6d4 23\uc77c \uc11c\uc6b8 \uc0c1\uba85\uc544\ud2b8\uc13c\ud130\uc5d0\uc11c \uccab \ubc88\uc9f8 \uad6d\ub0b4 \ud32c\ubbf8\ud305 'you_r_love'\ub97c \uac1c\ucd5c\ud588\ub2e4. \uae40\uc720\uc815\uc740 2017\ub144 9\uc6d4 15\uc77c, \uc9c0\ub09c 2010\ub144 8\uc6d4\ubd80\ud130 7\ub144\uac04 \ud568\uaed8\ud574\uc628 \uc18c\uc18d\uc0ac \uc2f8\uc774\ub354\uc2a4HQ\uc640 \uc7ac\uacc4\uc57d\ud588\ub2e4\uace0 \ubc1c\ud45c\ud588\ub2e4.", "answer": "\uc2f8\uc774\ub354\uc2a4HQ", "sentence": "\uae40\uc720\uc815\uc740 2017\ub144 9\uc6d4 15\uc77c, \uc9c0\ub09c 2010\ub144 8\uc6d4\ubd80\ud130 7\ub144\uac04 \ud568\uaed8\ud574\uc628 \uc18c\uc18d\uc0ac \uc2f8\uc774\ub354\uc2a4HQ \uc640 \uc7ac\uacc4\uc57d\ud588\ub2e4\uace0 \ubc1c\ud45c\ud588\ub2e4.", "paragraph_sentence": "2017\ub144, \uae40\uc720\uc815\uc740 \ub85c\ub9e8\uc2a4 \ucf54\ubbf8\ub514 \uc601\ud654 \u300a\uc0ac\ub791\ud558\uae30 \ub54c\ubb38\uc5d0\u300b\uc5d0\uc11c \ucc28\ud0dc\ud604, \uc11c\ud604\uc9c4\uacfc \uacf5\ub3d9\uc73c\ub85c \uc8fc\uc5f0\uc744 \ub9e1\uc544 \ud65c\uc57d\ud558\uc600\ub2e4. \uc774 \uc601\ud654\ub294 \uc0ac\ub791\uc5d0 \uc11c\ud230 \uc0ac\ub78c\ub4e4\uc758 \ub9c8\uc74c\uc744 \ud2b9\ubcc4\ud55c \ubc29\ubc95\uc73c\ub85c \uc774\uc5b4\uc8fc\ub294 \ud790\ub9c1 \ucf54\ubbf8\ub514\ub85c \uae40\uc720\uc815\uc740 4\ucc28\uc6d0\uc758 \uc5c9\ub6b1 \uc18c\ub140 \uc2a4\uceec\ub9ac \uc5ed\ud560\uc744 \uc5f0\uae30\ud558\uc600\ub2e4. \uc9c0\ub09c 2015\ub144 7\uc6d4\ubd80\ud130 10\uc6d4\uc5d0 \uac78\uccd0 \ucd2c\uc601\uc744 \ub9c8\uccd0, 2017\ub144 1\uc6d4 4\uc77c \uac1c\ubd09 \ub418\uc5c8\ub2e4. \uc601\ud654\ub294 \uc804\uad6d \uad00\uac1d\uc218 34\ub9cc\uba85\uc744 \ub3d9\uc6d0\ud558\uba70 \ud765\ud589 \ucc38\ud328\ud588\ub2e4. \uae40\uc720\uc815\uc740 2017\ub144 2\uc6d4 4\uc77c \ud0c0\uc774\uc644\uc758 \ud0c0\uc774\ubca0\uc774\uc5d0\uc11c \uadf8\ub140\uc758 \uccab \ud32c\ubbf8\ud305\uc774\uc790 \ud574\uc678 \ud32c\ubbf8\ud305\uc744 \uac1c\ucd5c\ud558\uc600\ub2e4. 5\uc6d4 20\uc77c\uc5d0\ub294 \uc2f1\uac00\ud3ec\ub974\uc5d0\uc11c \ub450 \ubc88\uc9f8 \ud32c\ubbf8\ud305\uc744 \uac1c\ucd5c\ud55c\ub370 \uc774\uc5b4 9\uc6d4 23\uc77c \uc11c\uc6b8 \uc0c1\uba85\uc544\ud2b8\uc13c\ud130\uc5d0\uc11c \uccab \ubc88\uc9f8 \uad6d\ub0b4 \ud32c\ubbf8\ud305 'you_r_love'\ub97c \uac1c\ucd5c\ud588\ub2e4. \uae40\uc720\uc815\uc740 2017\ub144 9\uc6d4 15\uc77c, \uc9c0\ub09c 2010\ub144 8\uc6d4\ubd80\ud130 7\ub144\uac04 \ud568\uaed8\ud574\uc628 \uc18c\uc18d\uc0ac \uc2f8\uc774\ub354\uc2a4HQ \uc640 \uc7ac\uacc4\uc57d\ud588\ub2e4\uace0 \ubc1c\ud45c\ud588\ub2e4. ", "paragraph_answer": "2017\ub144, \uae40\uc720\uc815\uc740 \ub85c\ub9e8\uc2a4 \ucf54\ubbf8\ub514 \uc601\ud654 \u300a\uc0ac\ub791\ud558\uae30 \ub54c\ubb38\uc5d0\u300b\uc5d0\uc11c \ucc28\ud0dc\ud604, \uc11c\ud604\uc9c4\uacfc \uacf5\ub3d9\uc73c\ub85c \uc8fc\uc5f0\uc744 \ub9e1\uc544 \ud65c\uc57d\ud558\uc600\ub2e4. \uc774 \uc601\ud654\ub294 \uc0ac\ub791\uc5d0 \uc11c\ud230 \uc0ac\ub78c\ub4e4\uc758 \ub9c8\uc74c\uc744 \ud2b9\ubcc4\ud55c \ubc29\ubc95\uc73c\ub85c \uc774\uc5b4\uc8fc\ub294 \ud790\ub9c1 \ucf54\ubbf8\ub514\ub85c \uae40\uc720\uc815\uc740 4\ucc28\uc6d0\uc758 \uc5c9\ub6b1 \uc18c\ub140 \uc2a4\uceec\ub9ac \uc5ed\ud560\uc744 \uc5f0\uae30\ud558\uc600\ub2e4. \uc9c0\ub09c 2015\ub144 7\uc6d4\ubd80\ud130 10\uc6d4\uc5d0 \uac78\uccd0 \ucd2c\uc601\uc744 \ub9c8\uccd0, 2017\ub144 1\uc6d4 4\uc77c \uac1c\ubd09 \ub418\uc5c8\ub2e4. \uc601\ud654\ub294 \uc804\uad6d \uad00\uac1d\uc218 34\ub9cc\uba85\uc744 \ub3d9\uc6d0\ud558\uba70 \ud765\ud589 \ucc38\ud328\ud588\ub2e4. \uae40\uc720\uc815\uc740 2017\ub144 2\uc6d4 4\uc77c \ud0c0\uc774\uc644\uc758 \ud0c0\uc774\ubca0\uc774\uc5d0\uc11c \uadf8\ub140\uc758 \uccab \ud32c\ubbf8\ud305\uc774\uc790 \ud574\uc678 \ud32c\ubbf8\ud305\uc744 \uac1c\ucd5c\ud558\uc600\ub2e4. 5\uc6d4 20\uc77c\uc5d0\ub294 \uc2f1\uac00\ud3ec\ub974\uc5d0\uc11c \ub450 \ubc88\uc9f8 \ud32c\ubbf8\ud305\uc744 \uac1c\ucd5c\ud55c\ub370 \uc774\uc5b4 9\uc6d4 23\uc77c \uc11c\uc6b8 \uc0c1\uba85\uc544\ud2b8\uc13c\ud130\uc5d0\uc11c \uccab \ubc88\uc9f8 \uad6d\ub0b4 \ud32c\ubbf8\ud305 'you_r_love'\ub97c \uac1c\ucd5c\ud588\ub2e4. \uae40\uc720\uc815\uc740 2017\ub144 9\uc6d4 15\uc77c, \uc9c0\ub09c 2010\ub144 8\uc6d4\ubd80\ud130 7\ub144\uac04 \ud568\uaed8\ud574\uc628 \uc18c\uc18d\uc0ac \uc2f8\uc774\ub354\uc2a4HQ \uc640 \uc7ac\uacc4\uc57d\ud588\ub2e4\uace0 \ubc1c\ud45c\ud588\ub2e4.", "sentence_answer": "\uae40\uc720\uc815\uc740 2017\ub144 9\uc6d4 15\uc77c, \uc9c0\ub09c 2010\ub144 8\uc6d4\ubd80\ud130 7\ub144\uac04 \ud568\uaed8\ud574\uc628 \uc18c\uc18d\uc0ac \uc2f8\uc774\ub354\uc2a4HQ \uc640 \uc7ac\uacc4\uc57d\ud588\ub2e4\uace0 \ubc1c\ud45c\ud588\ub2e4.", "paragraph_id": "6488457-4-2", "paragraph_question": "question: 2017\ub144 \uae40\uc720\uc815\uacfc \uc7ac\uacc4\uc57d\uc744 \ub9fa\uc740 \uc18c\uc18d\uc0ac\ub294?, context: 2017\ub144, \uae40\uc720\uc815\uc740 \ub85c\ub9e8\uc2a4 \ucf54\ubbf8\ub514 \uc601\ud654 \u300a\uc0ac\ub791\ud558\uae30 \ub54c\ubb38\uc5d0\u300b\uc5d0\uc11c \ucc28\ud0dc\ud604, \uc11c\ud604\uc9c4\uacfc \uacf5\ub3d9\uc73c\ub85c \uc8fc\uc5f0\uc744 \ub9e1\uc544 \ud65c\uc57d\ud558\uc600\ub2e4. \uc774 \uc601\ud654\ub294 \uc0ac\ub791\uc5d0 \uc11c\ud230 \uc0ac\ub78c\ub4e4\uc758 \ub9c8\uc74c\uc744 \ud2b9\ubcc4\ud55c \ubc29\ubc95\uc73c\ub85c \uc774\uc5b4\uc8fc\ub294 \ud790\ub9c1 \ucf54\ubbf8\ub514\ub85c \uae40\uc720\uc815\uc740 4\ucc28\uc6d0\uc758 \uc5c9\ub6b1 \uc18c\ub140 \uc2a4\uceec\ub9ac \uc5ed\ud560\uc744 \uc5f0\uae30\ud558\uc600\ub2e4. \uc9c0\ub09c 2015\ub144 7\uc6d4\ubd80\ud130 10\uc6d4\uc5d0 \uac78\uccd0 \ucd2c\uc601\uc744 \ub9c8\uccd0, 2017\ub144 1\uc6d4 4\uc77c \uac1c\ubd09 \ub418\uc5c8\ub2e4. \uc601\ud654\ub294 \uc804\uad6d \uad00\uac1d\uc218 34\ub9cc\uba85\uc744 \ub3d9\uc6d0\ud558\uba70 \ud765\ud589 \ucc38\ud328\ud588\ub2e4. \uae40\uc720\uc815\uc740 2017\ub144 2\uc6d4 4\uc77c \ud0c0\uc774\uc644\uc758 \ud0c0\uc774\ubca0\uc774\uc5d0\uc11c \uadf8\ub140\uc758 \uccab \ud32c\ubbf8\ud305\uc774\uc790 \ud574\uc678 \ud32c\ubbf8\ud305\uc744 \uac1c\ucd5c\ud558\uc600\ub2e4. 5\uc6d4 20\uc77c\uc5d0\ub294 \uc2f1\uac00\ud3ec\ub974\uc5d0\uc11c \ub450 \ubc88\uc9f8 \ud32c\ubbf8\ud305\uc744 \uac1c\ucd5c\ud55c\ub370 \uc774\uc5b4 9\uc6d4 23\uc77c \uc11c\uc6b8 \uc0c1\uba85\uc544\ud2b8\uc13c\ud130\uc5d0\uc11c \uccab \ubc88\uc9f8 \uad6d\ub0b4 \ud32c\ubbf8\ud305 'you_r_love'\ub97c \uac1c\ucd5c\ud588\ub2e4. \uae40\uc720\uc815\uc740 2017\ub144 9\uc6d4 15\uc77c, \uc9c0\ub09c 2010\ub144 8\uc6d4\ubd80\ud130 7\ub144\uac04 \ud568\uaed8\ud574\uc628 \uc18c\uc18d\uc0ac \uc2f8\uc774\ub354\uc2a4HQ\uc640 \uc7ac\uacc4\uc57d\ud588\ub2e4\uace0 \ubc1c\ud45c\ud588\ub2e4."} {"question": "\ub514\uc2a4\uc6cc\uc624\ube0c\ub9c8\uc778\uc5d0\uc11c \uc5c4\ub9c8\uac00 \uc544\ud504\ub2e4\uba70 \ub3c4\uc6c0\uc744 \uccad\ud558\ub7ec \uc624\ub294 \uc544\uc774\uc758 \uc774\ub984\uc740?", "paragraph": "\u2018\ub2c8\ub098\ub4dc\u2019\ub77c\ub294 \uc544\uc774\uc640 \ub2e4\ub978 \uc544\uc774\uac00 \uc640\uc11c \ub3c4\uc6c0\uc744 \uccad\ud55c\ub2e4. \ucc98\uc74c\uc5d0\ub294 \uc5c4\ub9c8\uac00 \uc544\ud504\ub2e4\uba70 \uc57d\ud488 1, 2\uac1c\ub97c \ub2ec\ub77c\uace0 \uc694\uccad\ud55c\ub2e4. \uac70\uc808\ud558\uba74 \ub098\uc911\uc5d0 \ub2e4\uc2dc \uc640\uc11c \uc5c4\ub9c8\ub97c \ubcd1\uc6d0\uae4c\uc9c0 \ub370\ub824\uac00\uc904 \uc0ac\ub78c \ud55c \uba85\uc744 \uc694\uccad\ud55c\ub2e4. \ub2e4\uc74c\uc5d0 \ubc30\uac00 \uace0\ud504\ub2e4\uba74\uc11c \uce94 \uc74c\uc2dd 2\uac1c\ub97c \ub2ec\ub77c\uace0 \ucc3e\uc544\uc624\uae30\ub3c4 \ud55c\ub2e4. \uc544\uc774\ub4e4\uc5d0\uac8c \uc57d, \uc74c\uc2dd\uc744 \ubaa8\ub450 \uc92c\uac70\ub098 \uc57d\uc744 \uc8fc\uc9c0 \ubabb\ud574\ub3c4 \uc5c4\ub9c8\ub97c \ubcd1\uc6d0\uc5d0 \ub370\ub824\ub2e4 \uc8fc\uc5c8\uc744 \uacbd\uc6b0 \uba70\uce60 \ub4a4 \uc5c4\ub9c8\uac00 \ucc3e\uc544\uc640 \ucee4\ud53c\ub97c \uc120\ubb3c\ud55c\ub2e4. \ub9cc\uc57d \uccab \ubc29\ubb38\uc5d0 \ub098\ub220\uc904 \ub9cc\ud07c\uc758 \uc57d\ud488\uc774 \uc5c6\ub2e4\uba74 \ub2e4\uc74c\ub0a0 \uc7ac\ucc28 \ubc29\ubb38\ud558\ub294\ub370, \uadf8\ub54c\uc5d0\ub3c4 \uc218\ub7c9\uc774 \ubd80\uc871\ud558\uba74 \"Sorry can't help you\"\ub77c\ub294 \uc120\ud0dd\uc9c0\uac00 \ub728\uba74\uc11c \uc0ac\uae30\uac00 \uc57d\uac04 \ud558\ub77d\ud55c\ub2e4. \ucc3e\uc544\uc62c \ub54c 2\uac1c\ub97c \uc694\uccad\ud588\ub294\ub370 \ub9c8\uce68 2\uac1c\uac00 \uc788\uc5b4\uc11c \ud558\ub098\ub97c \uc544\ud508 \uce90\ub9ad\ud130\uc5d0\uac8c \uc0ac\uc6a9\ud558\uace0 \uc800 \uc120\ud0dd\uc9c0\ub97c \uace0\ub974\uba74 \uc57d\ud488 1\uac1c\ub3c4 \uc544\ub07c\uace0 \uc0ac\uae30\ub3c4 \ubcf4\uc804\ud558\ub294 \ud50c\ub808\uc774\uac00 \uac00\ub2a5\ud558\ub2e4. \uc774\ubca4\ud2b8\ub97c \uc9c4\ud589\ud558\uba74\uc11c \ubcf4\ub78c\uc744 \ub290\ub084 \uc218\ub294 \uc788\uc73c\ub098, \ucee4\ud53c \uc2dc\uc138\uac00 \uc624\ub974\uc9c0 \uc54a\ub294 \ud55c \ud06c\uac8c \uc190\ud574 \ubcf4\ub294 \uc77c\uc774\ub2e4. \ubb3c\ub860 \uac70\uc808\ud558\uac70\ub098 \ubb34\uc2dc\ud558\uba74 \uc5d4\ub529\uc5d0 \ubc18\uc601\ub418\ubbc0\ub85c \uae30\ubd84\uc774 \uc88b\uc9c0 \uc54a\uc744 \uc218 \uc788\ub2e4. \ub354\uad70\ub2e4\ub098 \uba58\ud0c8\uc774 \uc57d\ud558\uac70\ub098 \ub3d9\uc815\uc2ec \ub9ce\uc740 \uce90\ub9ad\ud130\uac00 \ub9ce\ub2e4\uba74 \uac70\uc808\ud558\ub294 \uac83\uc774 \ub2a5\ub825\uce58\uc0c1\uc758 \ud070 \uc190\ud574\ub85c \uc774\uc5b4\uc9c8 \uc218 \uc788\ub2e4. \uc608\ub97c \ub4e4\uc5b4 \ud06c\ubca0\ud0c0\ub294 \uc544\uc774\ub4e4\uc758 \ub3c4\uc6c0\uc744 \uac70\uc808\ud558\uba74 \uba58\ud0c8\uc774 \ud06c\uac8c \ud558\ub77d\ud55c\ub2e4. \ubc18\ub300\ub85c \ub0c9\uc815\ud55c \uce90\ub9ad\ud130\uac00 \ub9ce\uc740 \ud30c\ud2f0\ub85c \ud50c\ub808\uc774\ud560 \uacbd\uc6b0 \uac70\uc808\ud558\ub294 \uac8c \ub354 \uc88b\uc744 \uc218\ub3c4 \uc788\ub2e4. \uadf8\ub7ec\ub098 \uc6ec\ub9cc\ud558\uba74 \uc2b9\ub099\ud558\ub294 \uac83\uc774 \uc88b\ub2e4. \ub3c4\uc640\uc8fc\uace0 \ud55c\ucc38 \ub4a4\uc5d0 \uc5c4\ub9c8\uac00 \ucc3e\uc544\uc624\ub294\ub370, \uc774\ub7f0 \uc2a4\ud1a0\ub9ac\uac00 \uaf64 \uac10\ub3d9\uc801\uc774\ub2e4.", "answer": "\ub2c8\ub098\ub4dc", "sentence": "\u2018 \ub2c8\ub098\ub4dc \u2019\ub77c\ub294 \uc544\uc774\uc640 \ub2e4\ub978 \uc544\uc774\uac00 \uc640\uc11c \ub3c4\uc6c0\uc744 \uccad\ud55c\ub2e4.", "paragraph_sentence": " \u2018 \ub2c8\ub098\ub4dc \u2019\ub77c\ub294 \uc544\uc774\uc640 \ub2e4\ub978 \uc544\uc774\uac00 \uc640\uc11c \ub3c4\uc6c0\uc744 \uccad\ud55c\ub2e4. \ucc98\uc74c\uc5d0\ub294 \uc5c4\ub9c8\uac00 \uc544\ud504\ub2e4\uba70 \uc57d\ud488 1, 2\uac1c\ub97c \ub2ec\ub77c\uace0 \uc694\uccad\ud55c\ub2e4. \uac70\uc808\ud558\uba74 \ub098\uc911\uc5d0 \ub2e4\uc2dc \uc640\uc11c \uc5c4\ub9c8\ub97c \ubcd1\uc6d0\uae4c\uc9c0 \ub370\ub824\uac00\uc904 \uc0ac\ub78c \ud55c \uba85\uc744 \uc694\uccad\ud55c\ub2e4. \ub2e4\uc74c\uc5d0 \ubc30\uac00 \uace0\ud504\ub2e4\uba74\uc11c \uce94 \uc74c\uc2dd 2\uac1c\ub97c \ub2ec\ub77c\uace0 \ucc3e\uc544\uc624\uae30\ub3c4 \ud55c\ub2e4. \uc544\uc774\ub4e4\uc5d0\uac8c \uc57d, \uc74c\uc2dd\uc744 \ubaa8\ub450 \uc92c\uac70\ub098 \uc57d\uc744 \uc8fc\uc9c0 \ubabb\ud574\ub3c4 \uc5c4\ub9c8\ub97c \ubcd1\uc6d0\uc5d0 \ub370\ub824\ub2e4 \uc8fc\uc5c8\uc744 \uacbd\uc6b0 \uba70\uce60 \ub4a4 \uc5c4\ub9c8\uac00 \ucc3e\uc544\uc640 \ucee4\ud53c\ub97c \uc120\ubb3c\ud55c\ub2e4. \ub9cc\uc57d \uccab \ubc29\ubb38\uc5d0 \ub098\ub220\uc904 \ub9cc\ud07c\uc758 \uc57d\ud488\uc774 \uc5c6\ub2e4\uba74 \ub2e4\uc74c\ub0a0 \uc7ac\ucc28 \ubc29\ubb38\ud558\ub294\ub370, \uadf8\ub54c\uc5d0\ub3c4 \uc218\ub7c9\uc774 \ubd80\uc871\ud558\uba74 \"Sorry can't help you\"\ub77c\ub294 \uc120\ud0dd\uc9c0\uac00 \ub728\uba74\uc11c \uc0ac\uae30\uac00 \uc57d\uac04 \ud558\ub77d\ud55c\ub2e4. \ucc3e\uc544\uc62c \ub54c 2\uac1c\ub97c \uc694\uccad\ud588\ub294\ub370 \ub9c8\uce68 2\uac1c\uac00 \uc788\uc5b4\uc11c \ud558\ub098\ub97c \uc544\ud508 \uce90\ub9ad\ud130\uc5d0\uac8c \uc0ac\uc6a9\ud558\uace0 \uc800 \uc120\ud0dd\uc9c0\ub97c \uace0\ub974\uba74 \uc57d\ud488 1\uac1c\ub3c4 \uc544\ub07c\uace0 \uc0ac\uae30\ub3c4 \ubcf4\uc804\ud558\ub294 \ud50c\ub808\uc774\uac00 \uac00\ub2a5\ud558\ub2e4. \uc774\ubca4\ud2b8\ub97c \uc9c4\ud589\ud558\uba74\uc11c \ubcf4\ub78c\uc744 \ub290\ub084 \uc218\ub294 \uc788\uc73c\ub098, \ucee4\ud53c \uc2dc\uc138\uac00 \uc624\ub974\uc9c0 \uc54a\ub294 \ud55c \ud06c\uac8c \uc190\ud574 \ubcf4\ub294 \uc77c\uc774\ub2e4. \ubb3c\ub860 \uac70\uc808\ud558\uac70\ub098 \ubb34\uc2dc\ud558\uba74 \uc5d4\ub529\uc5d0 \ubc18\uc601\ub418\ubbc0\ub85c \uae30\ubd84\uc774 \uc88b\uc9c0 \uc54a\uc744 \uc218 \uc788\ub2e4. \ub354\uad70\ub2e4\ub098 \uba58\ud0c8\uc774 \uc57d\ud558\uac70\ub098 \ub3d9\uc815\uc2ec \ub9ce\uc740 \uce90\ub9ad\ud130\uac00 \ub9ce\ub2e4\uba74 \uac70\uc808\ud558\ub294 \uac83\uc774 \ub2a5\ub825\uce58\uc0c1\uc758 \ud070 \uc190\ud574\ub85c \uc774\uc5b4\uc9c8 \uc218 \uc788\ub2e4. \uc608\ub97c \ub4e4\uc5b4 \ud06c\ubca0\ud0c0\ub294 \uc544\uc774\ub4e4\uc758 \ub3c4\uc6c0\uc744 \uac70\uc808\ud558\uba74 \uba58\ud0c8\uc774 \ud06c\uac8c \ud558\ub77d\ud55c\ub2e4. \ubc18\ub300\ub85c \ub0c9\uc815\ud55c \uce90\ub9ad\ud130\uac00 \ub9ce\uc740 \ud30c\ud2f0\ub85c \ud50c\ub808\uc774\ud560 \uacbd\uc6b0 \uac70\uc808\ud558\ub294 \uac8c \ub354 \uc88b\uc744 \uc218\ub3c4 \uc788\ub2e4. \uadf8\ub7ec\ub098 \uc6ec\ub9cc\ud558\uba74 \uc2b9\ub099\ud558\ub294 \uac83\uc774 \uc88b\ub2e4. \ub3c4\uc640\uc8fc\uace0 \ud55c\ucc38 \ub4a4\uc5d0 \uc5c4\ub9c8\uac00 \ucc3e\uc544\uc624\ub294\ub370, \uc774\ub7f0 \uc2a4\ud1a0\ub9ac\uac00 \uaf64 \uac10\ub3d9\uc801\uc774\ub2e4.", "paragraph_answer": "\u2018 \ub2c8\ub098\ub4dc \u2019\ub77c\ub294 \uc544\uc774\uc640 \ub2e4\ub978 \uc544\uc774\uac00 \uc640\uc11c \ub3c4\uc6c0\uc744 \uccad\ud55c\ub2e4. \ucc98\uc74c\uc5d0\ub294 \uc5c4\ub9c8\uac00 \uc544\ud504\ub2e4\uba70 \uc57d\ud488 1, 2\uac1c\ub97c \ub2ec\ub77c\uace0 \uc694\uccad\ud55c\ub2e4. \uac70\uc808\ud558\uba74 \ub098\uc911\uc5d0 \ub2e4\uc2dc \uc640\uc11c \uc5c4\ub9c8\ub97c \ubcd1\uc6d0\uae4c\uc9c0 \ub370\ub824\uac00\uc904 \uc0ac\ub78c \ud55c \uba85\uc744 \uc694\uccad\ud55c\ub2e4. \ub2e4\uc74c\uc5d0 \ubc30\uac00 \uace0\ud504\ub2e4\uba74\uc11c \uce94 \uc74c\uc2dd 2\uac1c\ub97c \ub2ec\ub77c\uace0 \ucc3e\uc544\uc624\uae30\ub3c4 \ud55c\ub2e4. \uc544\uc774\ub4e4\uc5d0\uac8c \uc57d, \uc74c\uc2dd\uc744 \ubaa8\ub450 \uc92c\uac70\ub098 \uc57d\uc744 \uc8fc\uc9c0 \ubabb\ud574\ub3c4 \uc5c4\ub9c8\ub97c \ubcd1\uc6d0\uc5d0 \ub370\ub824\ub2e4 \uc8fc\uc5c8\uc744 \uacbd\uc6b0 \uba70\uce60 \ub4a4 \uc5c4\ub9c8\uac00 \ucc3e\uc544\uc640 \ucee4\ud53c\ub97c \uc120\ubb3c\ud55c\ub2e4. \ub9cc\uc57d \uccab \ubc29\ubb38\uc5d0 \ub098\ub220\uc904 \ub9cc\ud07c\uc758 \uc57d\ud488\uc774 \uc5c6\ub2e4\uba74 \ub2e4\uc74c\ub0a0 \uc7ac\ucc28 \ubc29\ubb38\ud558\ub294\ub370, \uadf8\ub54c\uc5d0\ub3c4 \uc218\ub7c9\uc774 \ubd80\uc871\ud558\uba74 \"Sorry can't help you\"\ub77c\ub294 \uc120\ud0dd\uc9c0\uac00 \ub728\uba74\uc11c \uc0ac\uae30\uac00 \uc57d\uac04 \ud558\ub77d\ud55c\ub2e4. \ucc3e\uc544\uc62c \ub54c 2\uac1c\ub97c \uc694\uccad\ud588\ub294\ub370 \ub9c8\uce68 2\uac1c\uac00 \uc788\uc5b4\uc11c \ud558\ub098\ub97c \uc544\ud508 \uce90\ub9ad\ud130\uc5d0\uac8c \uc0ac\uc6a9\ud558\uace0 \uc800 \uc120\ud0dd\uc9c0\ub97c \uace0\ub974\uba74 \uc57d\ud488 1\uac1c\ub3c4 \uc544\ub07c\uace0 \uc0ac\uae30\ub3c4 \ubcf4\uc804\ud558\ub294 \ud50c\ub808\uc774\uac00 \uac00\ub2a5\ud558\ub2e4. \uc774\ubca4\ud2b8\ub97c \uc9c4\ud589\ud558\uba74\uc11c \ubcf4\ub78c\uc744 \ub290\ub084 \uc218\ub294 \uc788\uc73c\ub098, \ucee4\ud53c \uc2dc\uc138\uac00 \uc624\ub974\uc9c0 \uc54a\ub294 \ud55c \ud06c\uac8c \uc190\ud574 \ubcf4\ub294 \uc77c\uc774\ub2e4. \ubb3c\ub860 \uac70\uc808\ud558\uac70\ub098 \ubb34\uc2dc\ud558\uba74 \uc5d4\ub529\uc5d0 \ubc18\uc601\ub418\ubbc0\ub85c \uae30\ubd84\uc774 \uc88b\uc9c0 \uc54a\uc744 \uc218 \uc788\ub2e4. \ub354\uad70\ub2e4\ub098 \uba58\ud0c8\uc774 \uc57d\ud558\uac70\ub098 \ub3d9\uc815\uc2ec \ub9ce\uc740 \uce90\ub9ad\ud130\uac00 \ub9ce\ub2e4\uba74 \uac70\uc808\ud558\ub294 \uac83\uc774 \ub2a5\ub825\uce58\uc0c1\uc758 \ud070 \uc190\ud574\ub85c \uc774\uc5b4\uc9c8 \uc218 \uc788\ub2e4. \uc608\ub97c \ub4e4\uc5b4 \ud06c\ubca0\ud0c0\ub294 \uc544\uc774\ub4e4\uc758 \ub3c4\uc6c0\uc744 \uac70\uc808\ud558\uba74 \uba58\ud0c8\uc774 \ud06c\uac8c \ud558\ub77d\ud55c\ub2e4. \ubc18\ub300\ub85c \ub0c9\uc815\ud55c \uce90\ub9ad\ud130\uac00 \ub9ce\uc740 \ud30c\ud2f0\ub85c \ud50c\ub808\uc774\ud560 \uacbd\uc6b0 \uac70\uc808\ud558\ub294 \uac8c \ub354 \uc88b\uc744 \uc218\ub3c4 \uc788\ub2e4. \uadf8\ub7ec\ub098 \uc6ec\ub9cc\ud558\uba74 \uc2b9\ub099\ud558\ub294 \uac83\uc774 \uc88b\ub2e4. \ub3c4\uc640\uc8fc\uace0 \ud55c\ucc38 \ub4a4\uc5d0 \uc5c4\ub9c8\uac00 \ucc3e\uc544\uc624\ub294\ub370, \uc774\ub7f0 \uc2a4\ud1a0\ub9ac\uac00 \uaf64 \uac10\ub3d9\uc801\uc774\ub2e4.", "sentence_answer": "\u2018 \ub2c8\ub098\ub4dc \u2019\ub77c\ub294 \uc544\uc774\uc640 \ub2e4\ub978 \uc544\uc774\uac00 \uc640\uc11c \ub3c4\uc6c0\uc744 \uccad\ud55c\ub2e4.", "paragraph_id": "6492782-6-0", "paragraph_question": "question: \ub514\uc2a4\uc6cc\uc624\ube0c\ub9c8\uc778\uc5d0\uc11c \uc5c4\ub9c8\uac00 \uc544\ud504\ub2e4\uba70 \ub3c4\uc6c0\uc744 \uccad\ud558\ub7ec \uc624\ub294 \uc544\uc774\uc758 \uc774\ub984\uc740?, context: \u2018\ub2c8\ub098\ub4dc\u2019\ub77c\ub294 \uc544\uc774\uc640 \ub2e4\ub978 \uc544\uc774\uac00 \uc640\uc11c \ub3c4\uc6c0\uc744 \uccad\ud55c\ub2e4. \ucc98\uc74c\uc5d0\ub294 \uc5c4\ub9c8\uac00 \uc544\ud504\ub2e4\uba70 \uc57d\ud488 1, 2\uac1c\ub97c \ub2ec\ub77c\uace0 \uc694\uccad\ud55c\ub2e4. \uac70\uc808\ud558\uba74 \ub098\uc911\uc5d0 \ub2e4\uc2dc \uc640\uc11c \uc5c4\ub9c8\ub97c \ubcd1\uc6d0\uae4c\uc9c0 \ub370\ub824\uac00\uc904 \uc0ac\ub78c \ud55c \uba85\uc744 \uc694\uccad\ud55c\ub2e4. \ub2e4\uc74c\uc5d0 \ubc30\uac00 \uace0\ud504\ub2e4\uba74\uc11c \uce94 \uc74c\uc2dd 2\uac1c\ub97c \ub2ec\ub77c\uace0 \ucc3e\uc544\uc624\uae30\ub3c4 \ud55c\ub2e4. \uc544\uc774\ub4e4\uc5d0\uac8c \uc57d, \uc74c\uc2dd\uc744 \ubaa8\ub450 \uc92c\uac70\ub098 \uc57d\uc744 \uc8fc\uc9c0 \ubabb\ud574\ub3c4 \uc5c4\ub9c8\ub97c \ubcd1\uc6d0\uc5d0 \ub370\ub824\ub2e4 \uc8fc\uc5c8\uc744 \uacbd\uc6b0 \uba70\uce60 \ub4a4 \uc5c4\ub9c8\uac00 \ucc3e\uc544\uc640 \ucee4\ud53c\ub97c \uc120\ubb3c\ud55c\ub2e4. \ub9cc\uc57d \uccab \ubc29\ubb38\uc5d0 \ub098\ub220\uc904 \ub9cc\ud07c\uc758 \uc57d\ud488\uc774 \uc5c6\ub2e4\uba74 \ub2e4\uc74c\ub0a0 \uc7ac\ucc28 \ubc29\ubb38\ud558\ub294\ub370, \uadf8\ub54c\uc5d0\ub3c4 \uc218\ub7c9\uc774 \ubd80\uc871\ud558\uba74 \"Sorry can't help you\"\ub77c\ub294 \uc120\ud0dd\uc9c0\uac00 \ub728\uba74\uc11c \uc0ac\uae30\uac00 \uc57d\uac04 \ud558\ub77d\ud55c\ub2e4. \ucc3e\uc544\uc62c \ub54c 2\uac1c\ub97c \uc694\uccad\ud588\ub294\ub370 \ub9c8\uce68 2\uac1c\uac00 \uc788\uc5b4\uc11c \ud558\ub098\ub97c \uc544\ud508 \uce90\ub9ad\ud130\uc5d0\uac8c \uc0ac\uc6a9\ud558\uace0 \uc800 \uc120\ud0dd\uc9c0\ub97c \uace0\ub974\uba74 \uc57d\ud488 1\uac1c\ub3c4 \uc544\ub07c\uace0 \uc0ac\uae30\ub3c4 \ubcf4\uc804\ud558\ub294 \ud50c\ub808\uc774\uac00 \uac00\ub2a5\ud558\ub2e4. \uc774\ubca4\ud2b8\ub97c \uc9c4\ud589\ud558\uba74\uc11c \ubcf4\ub78c\uc744 \ub290\ub084 \uc218\ub294 \uc788\uc73c\ub098, \ucee4\ud53c \uc2dc\uc138\uac00 \uc624\ub974\uc9c0 \uc54a\ub294 \ud55c \ud06c\uac8c \uc190\ud574 \ubcf4\ub294 \uc77c\uc774\ub2e4. \ubb3c\ub860 \uac70\uc808\ud558\uac70\ub098 \ubb34\uc2dc\ud558\uba74 \uc5d4\ub529\uc5d0 \ubc18\uc601\ub418\ubbc0\ub85c \uae30\ubd84\uc774 \uc88b\uc9c0 \uc54a\uc744 \uc218 \uc788\ub2e4. \ub354\uad70\ub2e4\ub098 \uba58\ud0c8\uc774 \uc57d\ud558\uac70\ub098 \ub3d9\uc815\uc2ec \ub9ce\uc740 \uce90\ub9ad\ud130\uac00 \ub9ce\ub2e4\uba74 \uac70\uc808\ud558\ub294 \uac83\uc774 \ub2a5\ub825\uce58\uc0c1\uc758 \ud070 \uc190\ud574\ub85c \uc774\uc5b4\uc9c8 \uc218 \uc788\ub2e4. \uc608\ub97c \ub4e4\uc5b4 \ud06c\ubca0\ud0c0\ub294 \uc544\uc774\ub4e4\uc758 \ub3c4\uc6c0\uc744 \uac70\uc808\ud558\uba74 \uba58\ud0c8\uc774 \ud06c\uac8c \ud558\ub77d\ud55c\ub2e4. \ubc18\ub300\ub85c \ub0c9\uc815\ud55c \uce90\ub9ad\ud130\uac00 \ub9ce\uc740 \ud30c\ud2f0\ub85c \ud50c\ub808\uc774\ud560 \uacbd\uc6b0 \uac70\uc808\ud558\ub294 \uac8c \ub354 \uc88b\uc744 \uc218\ub3c4 \uc788\ub2e4. \uadf8\ub7ec\ub098 \uc6ec\ub9cc\ud558\uba74 \uc2b9\ub099\ud558\ub294 \uac83\uc774 \uc88b\ub2e4. \ub3c4\uc640\uc8fc\uace0 \ud55c\ucc38 \ub4a4\uc5d0 \uc5c4\ub9c8\uac00 \ucc3e\uc544\uc624\ub294\ub370, \uc774\ub7f0 \uc2a4\ud1a0\ub9ac\uac00 \uaf64 \uac10\ub3d9\uc801\uc774\ub2e4."} {"question": "\uc911\uad6d\uc774 \uc81c1\ub3c4\ub828\uc120\uc744 \uc810\ub839\ud558\uae30\ub85c \ud55c \uae30\ud55c\uc740 \uba87 \ub144\uae4c\uc9c0\uc778\uac00?", "paragraph": "\uc870\uc9c0\ud0c0\uc6b4 \ub300\ud559\uad50 \ube45\ud130 \ucc28 \uad50\uc218\ub294 \u201c\ud55c\uad6d\uc740 \ubbf8\uad6d\uacfc \uc804\ub7b5\uc801\uc73c\ub85c \ubc00\uc811\ud55c \uad00\uacc4\ub97c \ub9fa\uc5b4\uc57c \uc911\uad6d\uc758 \uc874\uc911\uc744 \ubc1b\uace0 \ub354 \ub098\uc740 \ub300\uc6b0\ub97c \ubc1b\uc744 \uc218 \uc788\ub2e4\u201d\uba70 \ud55c\ubbf8\ub3d9\ub9f9\uc5d0 \ubb38\uc81c\uac00 \uc0dd\uacbc\uc744 \uacbd\uc6b0 \ud55c\uad6d\uc774 \uc911\uad6d\uc73c\ub85c\ubd80\ud130 \uc874\uc911\ubc1b\uc9c0 \ubabb\ud558\uace0 \ub300\uc6b0\ubc1b\uc9c0 \ubabb\ud560 \uc218 \uc788\ub2e4\uace0 \ud588\ub2e4. 2017\ub144 \uae30\uc900\uc73c\ub85c 200\uac1c\uc758 \ubd81\ud55c \ubbf8\uc0ac\uc77c\uacfc 600\uac1c \uc774\uc0c1\uc758 \uc911\uad6d \ubbf8\uc0ac\uc77c\uc774 \ud55c\uad6d\uad70\uacfc \uc8fc\ud55c\ubbf8\uad70 \uae30\uc9c0\ub97c \ud5a5\ud574 \uc870\uc900\ub418\uc5b4 \uc788\ub2e4. \uc8fc\ud55c\ubbf8\uad70\uc774 \ud55c\ubc18\ub3c4\uc5d0 \uc8fc\ub454\ud568\uc73c\ub85c\uc11c \ubbf8\uad6d\uc774 \uc5bb\uace0 \uc788\ub294 \uac00\uc7a5 \uc8fc\ub41c \uc804\ub7b5\uc801 \uac00\uce58\ub294 \uc911\uad6d\uc744 \uacac\uc81c\ud560 \uc218 \uc788\ub2e4\ub294 \uac83\uc774\uace0 \uc0ac\ub4dc \ubc30\uce58\uac00 \ucca0\ud68c\ub418\uba74 \uc624\ud788\ub824 \ubc18\ub300\ub85c \uc8fc\ud55c\ubbf8\uad70\uc774 600\uac1c \uc774\uc0c1\uc758 \uc911\uad6d \ubbf8\uc0ac\uc77c\ub85c \uc778\ud574 \uc704\ud611\uc744 \ubc1b\uac8c \ub418\uae30 \ub54c\ubb38\uc5d0 \ubbf8\uad6d\uc740 \ud55c\ubc18\ub3c4\ub97c \ub354 \uc774\uc0c1 \uc804\ub7b5\uc801 \uc694\ucda9\uc9c0\ub85c \ud65c\uc6a9\ud560 \uc218 \uc5c6\uac8c \ub41c\ub2e4. \uc0ac\ub4dc\ub294 \uc8fc\ud55c\ubbf8\uad70\uc744 \ubcf4\ud638\ud558\ub294 \uac83\uc774 \uc77c\ucc28\uc801 \ubaa9\ud45c\uc774\uace0 \ud55c\ubc18\ub3c4\uc5d0 \uc8fc\ub454\ud558\uace0 \uc788\ub294 \uc8fc\ud55c\ubbf8\uad70\uc774 \ubd81\ud55c\uacfc \uc911\uad6d\uc758 \ubbf8\uc0ac\uc77c\ub85c \uc778\ud574 \uc704\ud611\uc744 \ubc1b\uace0 \uc788\ub294 \uc0c1\ud669\uc5d0\uc11c\ub294 \ud55c\ubc18\ub3c4\ub97c \ub354 \uc774\uc0c1 \uc804\ub7b5\uc801 \uc694\ucda9\uc9c0\ub85c \ud65c\uc6a9\ud560 \uc218 \uc5c6\uc744 \ubfd0\ub9cc \uc544\ub2c8\ub77c \uc624\ud788\ub824 \ud55c\ubc18\ub3c4\uc5d0 \uc8fc\ub454\ud558\uace0 \uc788\ub294 \uc8fc\ud55c\ubbf8\uad70\uc774 \uc911\uad6d \ubbf8\uc0ac\uc77c\uc758 \uc778\uc9c8\ub85c \uc7a1\ud600 \uc788\ub294 \ud615\uad6d\uc774 \ub418\uc5b4 \ubc84\ub9b0\ub2e4. \ud55c\ubc18\ub3c4\uc5d0 \uc8fc\ub454\ud558\uace0 \uc788\ub294 \uc8fc\ud55c\ubbf8\uad70\uc744 \ubcf4\ud638\ud558\uae30 \uc704\ud574 \ubc30\uce58\ub41c \uc0ac\ub4dc\uac00 \ucca0\ud68c\ub41c\ub2e4\uba74 \uc8fc\ud55c\ubbf8\uad70\uc740 \uc911\uad6d \ubbf8\uc0ac\uc77c\uc5d0 \ubb34\ubc29\ube44 \uc0c1\ud0dc\ub85c \ub178\ucd9c\ub418\uc5b4 \uc704\ud611\uc744 \ubc1b\uac8c \ub418\uace0 \uc774\ub85c \uc778\ud574 \uc8fc\ud55c\ubbf8\uad70\uc774 \ud55c\ubc18\ub3c4\uc5d0 \uc8fc\ub454\ud568\uc73c\ub85c\uc11c \ubbf8\uad6d\uc774 \uc5bb\uc744 \uc218 \uc788\ub294 \uc804\ub7b5\uc801 \uac00\uce58\uac00 \uc18c\uba78\ub418\uae30 \ub54c\ubb38\uc5d0 \ubbf8\uad6d\uc815\ubd80\uac00 \uc8fc\ud55c\ubbf8\uad70\uc744 \ucca0\uc218\uc2dc\ud0ac \uc218\ubc16\uc5d0 \uc5c6\uac8c \ub41c\ub2e4. \uc911\uad6d\uc740 \ubbfc\uc8fc\ud601\uba85\uc744 \uac70\uce58\uc9c0 \uc54a\uc740 \uacf5\uc0b0\ub2f9 1\ub2f9\uccb4\uc81c \uacf5\uc0b0\uc8fc\uc758 \uad6d\uac00\uc774\uba70 \uc911\uad6d\uc740 \"\ubbf8\uad6d\ub9cc \uc5c6\uc5c8\uc73c\uba74 \ud55c\uad6d\uc740 \uc9c4\uc791\uc5d0 \uc190\ubd24\uc744 \ub098\ub77c\"\ub77c\uace0 \ud558\uba70 \ud55c\uad6d\uc744 \uad70\uc0ac\uc801\uc73c\ub85c \uc704\ud611\ud558\ub294 \ubc1c\uc5b8\uc744 \ud558\uc600\uace0 \ub9cc\uc57d\uc5d0 \uc8fc\ud55c\ubbf8\uad70\uc774 \ud55c\ubc18\ub3c4\uc5d0\uc11c \ucca0\uc218\ud55c\ub2e4\uba74 \uc911\uad6d\uc774 \uc774 \ubc1c\uc5b8\uc744 \uc2e4\ud589\uc73c\ub85c \uc62e\uae38 \uac00\ub2a5\uc131\uc774 \uc788\ub2e4. \uc911\uad6d\uc740 2025\ub144\uae4c\uc9c0 \uc81c1\ub3c4\ub828\uc120\uc744 \uc810\ub839\ud558\uaca0\ub2e4\ub294 \uacc4\ud68d\uc744 \uac16\uace0 \uc788\ub294\ub370, \uc81c1\ub3c4\ub828\uc120\uc5d0\ub294 \ud55c\ubc18\ub3c4\uac00 \ud3ec\ud568\ub41c\ub2e4.", "answer": "2025\ub144", "sentence": "\uc911\uad6d\uc740 2025\ub144 \uae4c\uc9c0 \uc81c1\ub3c4\ub828\uc120\uc744 \uc810\ub839\ud558\uaca0\ub2e4\ub294 \uacc4\ud68d\uc744 \uac16\uace0 \uc788\ub294\ub370, \uc81c1\ub3c4\ub828\uc120\uc5d0\ub294 \ud55c\ubc18\ub3c4\uac00 \ud3ec\ud568\ub41c\ub2e4.", "paragraph_sentence": "\uc870\uc9c0\ud0c0\uc6b4 \ub300\ud559\uad50 \ube45\ud130 \ucc28 \uad50\uc218\ub294 \u201c\ud55c\uad6d\uc740 \ubbf8\uad6d\uacfc \uc804\ub7b5\uc801\uc73c\ub85c \ubc00\uc811\ud55c \uad00\uacc4\ub97c \ub9fa\uc5b4\uc57c \uc911\uad6d\uc758 \uc874\uc911\uc744 \ubc1b\uace0 \ub354 \ub098\uc740 \ub300\uc6b0\ub97c \ubc1b\uc744 \uc218 \uc788\ub2e4\u201d\uba70 \ud55c\ubbf8\ub3d9\ub9f9\uc5d0 \ubb38\uc81c\uac00 \uc0dd\uacbc\uc744 \uacbd\uc6b0 \ud55c\uad6d\uc774 \uc911\uad6d\uc73c\ub85c\ubd80\ud130 \uc874\uc911\ubc1b\uc9c0 \ubabb\ud558\uace0 \ub300\uc6b0\ubc1b\uc9c0 \ubabb\ud560 \uc218 \uc788\ub2e4\uace0 \ud588\ub2e4. 2017\ub144 \uae30\uc900\uc73c\ub85c 200\uac1c\uc758 \ubd81\ud55c \ubbf8\uc0ac\uc77c\uacfc 600\uac1c \uc774\uc0c1\uc758 \uc911\uad6d \ubbf8\uc0ac\uc77c\uc774 \ud55c\uad6d\uad70\uacfc \uc8fc\ud55c\ubbf8\uad70 \uae30\uc9c0\ub97c \ud5a5\ud574 \uc870\uc900\ub418\uc5b4 \uc788\ub2e4. \uc8fc\ud55c\ubbf8\uad70\uc774 \ud55c\ubc18\ub3c4\uc5d0 \uc8fc\ub454\ud568\uc73c\ub85c\uc11c \ubbf8\uad6d\uc774 \uc5bb\uace0 \uc788\ub294 \uac00\uc7a5 \uc8fc\ub41c \uc804\ub7b5\uc801 \uac00\uce58\ub294 \uc911\uad6d\uc744 \uacac\uc81c\ud560 \uc218 \uc788\ub2e4\ub294 \uac83\uc774\uace0 \uc0ac\ub4dc \ubc30\uce58\uac00 \ucca0\ud68c\ub418\uba74 \uc624\ud788\ub824 \ubc18\ub300\ub85c \uc8fc\ud55c\ubbf8\uad70\uc774 600\uac1c \uc774\uc0c1\uc758 \uc911\uad6d \ubbf8\uc0ac\uc77c\ub85c \uc778\ud574 \uc704\ud611\uc744 \ubc1b\uac8c \ub418\uae30 \ub54c\ubb38\uc5d0 \ubbf8\uad6d\uc740 \ud55c\ubc18\ub3c4\ub97c \ub354 \uc774\uc0c1 \uc804\ub7b5\uc801 \uc694\ucda9\uc9c0\ub85c \ud65c\uc6a9\ud560 \uc218 \uc5c6\uac8c \ub41c\ub2e4. \uc0ac\ub4dc\ub294 \uc8fc\ud55c\ubbf8\uad70\uc744 \ubcf4\ud638\ud558\ub294 \uac83\uc774 \uc77c\ucc28\uc801 \ubaa9\ud45c\uc774\uace0 \ud55c\ubc18\ub3c4\uc5d0 \uc8fc\ub454\ud558\uace0 \uc788\ub294 \uc8fc\ud55c\ubbf8\uad70\uc774 \ubd81\ud55c\uacfc \uc911\uad6d\uc758 \ubbf8\uc0ac\uc77c\ub85c \uc778\ud574 \uc704\ud611\uc744 \ubc1b\uace0 \uc788\ub294 \uc0c1\ud669\uc5d0\uc11c\ub294 \ud55c\ubc18\ub3c4\ub97c \ub354 \uc774\uc0c1 \uc804\ub7b5\uc801 \uc694\ucda9\uc9c0\ub85c \ud65c\uc6a9\ud560 \uc218 \uc5c6\uc744 \ubfd0\ub9cc \uc544\ub2c8\ub77c \uc624\ud788\ub824 \ud55c\ubc18\ub3c4\uc5d0 \uc8fc\ub454\ud558\uace0 \uc788\ub294 \uc8fc\ud55c\ubbf8\uad70\uc774 \uc911\uad6d \ubbf8\uc0ac\uc77c\uc758 \uc778\uc9c8\ub85c \uc7a1\ud600 \uc788\ub294 \ud615\uad6d\uc774 \ub418\uc5b4 \ubc84\ub9b0\ub2e4. \ud55c\ubc18\ub3c4\uc5d0 \uc8fc\ub454\ud558\uace0 \uc788\ub294 \uc8fc\ud55c\ubbf8\uad70\uc744 \ubcf4\ud638\ud558\uae30 \uc704\ud574 \ubc30\uce58\ub41c \uc0ac\ub4dc\uac00 \ucca0\ud68c\ub41c\ub2e4\uba74 \uc8fc\ud55c\ubbf8\uad70\uc740 \uc911\uad6d \ubbf8\uc0ac\uc77c\uc5d0 \ubb34\ubc29\ube44 \uc0c1\ud0dc\ub85c \ub178\ucd9c\ub418\uc5b4 \uc704\ud611\uc744 \ubc1b\uac8c \ub418\uace0 \uc774\ub85c \uc778\ud574 \uc8fc\ud55c\ubbf8\uad70\uc774 \ud55c\ubc18\ub3c4\uc5d0 \uc8fc\ub454\ud568\uc73c\ub85c\uc11c \ubbf8\uad6d\uc774 \uc5bb\uc744 \uc218 \uc788\ub294 \uc804\ub7b5\uc801 \uac00\uce58\uac00 \uc18c\uba78\ub418\uae30 \ub54c\ubb38\uc5d0 \ubbf8\uad6d\uc815\ubd80\uac00 \uc8fc\ud55c\ubbf8\uad70\uc744 \ucca0\uc218\uc2dc\ud0ac \uc218\ubc16\uc5d0 \uc5c6\uac8c \ub41c\ub2e4. \uc911\uad6d\uc740 \ubbfc\uc8fc\ud601\uba85\uc744 \uac70\uce58\uc9c0 \uc54a\uc740 \uacf5\uc0b0\ub2f9 1\ub2f9\uccb4\uc81c \uacf5\uc0b0\uc8fc\uc758 \uad6d\uac00\uc774\uba70 \uc911\uad6d\uc740 \"\ubbf8\uad6d\ub9cc \uc5c6\uc5c8\uc73c\uba74 \ud55c\uad6d\uc740 \uc9c4\uc791\uc5d0 \uc190\ubd24\uc744 \ub098\ub77c\"\ub77c\uace0 \ud558\uba70 \ud55c\uad6d\uc744 \uad70\uc0ac\uc801\uc73c\ub85c \uc704\ud611\ud558\ub294 \ubc1c\uc5b8\uc744 \ud558\uc600\uace0 \ub9cc\uc57d\uc5d0 \uc8fc\ud55c\ubbf8\uad70\uc774 \ud55c\ubc18\ub3c4\uc5d0\uc11c \ucca0\uc218\ud55c\ub2e4\uba74 \uc911\uad6d\uc774 \uc774 \ubc1c\uc5b8\uc744 \uc2e4\ud589\uc73c\ub85c \uc62e\uae38 \uac00\ub2a5\uc131\uc774 \uc788\ub2e4. \uc911\uad6d\uc740 2025\ub144 \uae4c\uc9c0 \uc81c1\ub3c4\ub828\uc120\uc744 \uc810\ub839\ud558\uaca0\ub2e4\ub294 \uacc4\ud68d\uc744 \uac16\uace0 \uc788\ub294\ub370, \uc81c1\ub3c4\ub828\uc120\uc5d0\ub294 \ud55c\ubc18\ub3c4\uac00 \ud3ec\ud568\ub41c\ub2e4. ", "paragraph_answer": "\uc870\uc9c0\ud0c0\uc6b4 \ub300\ud559\uad50 \ube45\ud130 \ucc28 \uad50\uc218\ub294 \u201c\ud55c\uad6d\uc740 \ubbf8\uad6d\uacfc \uc804\ub7b5\uc801\uc73c\ub85c \ubc00\uc811\ud55c \uad00\uacc4\ub97c \ub9fa\uc5b4\uc57c \uc911\uad6d\uc758 \uc874\uc911\uc744 \ubc1b\uace0 \ub354 \ub098\uc740 \ub300\uc6b0\ub97c \ubc1b\uc744 \uc218 \uc788\ub2e4\u201d\uba70 \ud55c\ubbf8\ub3d9\ub9f9\uc5d0 \ubb38\uc81c\uac00 \uc0dd\uacbc\uc744 \uacbd\uc6b0 \ud55c\uad6d\uc774 \uc911\uad6d\uc73c\ub85c\ubd80\ud130 \uc874\uc911\ubc1b\uc9c0 \ubabb\ud558\uace0 \ub300\uc6b0\ubc1b\uc9c0 \ubabb\ud560 \uc218 \uc788\ub2e4\uace0 \ud588\ub2e4. 2017\ub144 \uae30\uc900\uc73c\ub85c 200\uac1c\uc758 \ubd81\ud55c \ubbf8\uc0ac\uc77c\uacfc 600\uac1c \uc774\uc0c1\uc758 \uc911\uad6d \ubbf8\uc0ac\uc77c\uc774 \ud55c\uad6d\uad70\uacfc \uc8fc\ud55c\ubbf8\uad70 \uae30\uc9c0\ub97c \ud5a5\ud574 \uc870\uc900\ub418\uc5b4 \uc788\ub2e4. \uc8fc\ud55c\ubbf8\uad70\uc774 \ud55c\ubc18\ub3c4\uc5d0 \uc8fc\ub454\ud568\uc73c\ub85c\uc11c \ubbf8\uad6d\uc774 \uc5bb\uace0 \uc788\ub294 \uac00\uc7a5 \uc8fc\ub41c \uc804\ub7b5\uc801 \uac00\uce58\ub294 \uc911\uad6d\uc744 \uacac\uc81c\ud560 \uc218 \uc788\ub2e4\ub294 \uac83\uc774\uace0 \uc0ac\ub4dc \ubc30\uce58\uac00 \ucca0\ud68c\ub418\uba74 \uc624\ud788\ub824 \ubc18\ub300\ub85c \uc8fc\ud55c\ubbf8\uad70\uc774 600\uac1c \uc774\uc0c1\uc758 \uc911\uad6d \ubbf8\uc0ac\uc77c\ub85c \uc778\ud574 \uc704\ud611\uc744 \ubc1b\uac8c \ub418\uae30 \ub54c\ubb38\uc5d0 \ubbf8\uad6d\uc740 \ud55c\ubc18\ub3c4\ub97c \ub354 \uc774\uc0c1 \uc804\ub7b5\uc801 \uc694\ucda9\uc9c0\ub85c \ud65c\uc6a9\ud560 \uc218 \uc5c6\uac8c \ub41c\ub2e4. \uc0ac\ub4dc\ub294 \uc8fc\ud55c\ubbf8\uad70\uc744 \ubcf4\ud638\ud558\ub294 \uac83\uc774 \uc77c\ucc28\uc801 \ubaa9\ud45c\uc774\uace0 \ud55c\ubc18\ub3c4\uc5d0 \uc8fc\ub454\ud558\uace0 \uc788\ub294 \uc8fc\ud55c\ubbf8\uad70\uc774 \ubd81\ud55c\uacfc \uc911\uad6d\uc758 \ubbf8\uc0ac\uc77c\ub85c \uc778\ud574 \uc704\ud611\uc744 \ubc1b\uace0 \uc788\ub294 \uc0c1\ud669\uc5d0\uc11c\ub294 \ud55c\ubc18\ub3c4\ub97c \ub354 \uc774\uc0c1 \uc804\ub7b5\uc801 \uc694\ucda9\uc9c0\ub85c \ud65c\uc6a9\ud560 \uc218 \uc5c6\uc744 \ubfd0\ub9cc \uc544\ub2c8\ub77c \uc624\ud788\ub824 \ud55c\ubc18\ub3c4\uc5d0 \uc8fc\ub454\ud558\uace0 \uc788\ub294 \uc8fc\ud55c\ubbf8\uad70\uc774 \uc911\uad6d \ubbf8\uc0ac\uc77c\uc758 \uc778\uc9c8\ub85c \uc7a1\ud600 \uc788\ub294 \ud615\uad6d\uc774 \ub418\uc5b4 \ubc84\ub9b0\ub2e4. \ud55c\ubc18\ub3c4\uc5d0 \uc8fc\ub454\ud558\uace0 \uc788\ub294 \uc8fc\ud55c\ubbf8\uad70\uc744 \ubcf4\ud638\ud558\uae30 \uc704\ud574 \ubc30\uce58\ub41c \uc0ac\ub4dc\uac00 \ucca0\ud68c\ub41c\ub2e4\uba74 \uc8fc\ud55c\ubbf8\uad70\uc740 \uc911\uad6d \ubbf8\uc0ac\uc77c\uc5d0 \ubb34\ubc29\ube44 \uc0c1\ud0dc\ub85c \ub178\ucd9c\ub418\uc5b4 \uc704\ud611\uc744 \ubc1b\uac8c \ub418\uace0 \uc774\ub85c \uc778\ud574 \uc8fc\ud55c\ubbf8\uad70\uc774 \ud55c\ubc18\ub3c4\uc5d0 \uc8fc\ub454\ud568\uc73c\ub85c\uc11c \ubbf8\uad6d\uc774 \uc5bb\uc744 \uc218 \uc788\ub294 \uc804\ub7b5\uc801 \uac00\uce58\uac00 \uc18c\uba78\ub418\uae30 \ub54c\ubb38\uc5d0 \ubbf8\uad6d\uc815\ubd80\uac00 \uc8fc\ud55c\ubbf8\uad70\uc744 \ucca0\uc218\uc2dc\ud0ac \uc218\ubc16\uc5d0 \uc5c6\uac8c \ub41c\ub2e4. \uc911\uad6d\uc740 \ubbfc\uc8fc\ud601\uba85\uc744 \uac70\uce58\uc9c0 \uc54a\uc740 \uacf5\uc0b0\ub2f9 1\ub2f9\uccb4\uc81c \uacf5\uc0b0\uc8fc\uc758 \uad6d\uac00\uc774\uba70 \uc911\uad6d\uc740 \"\ubbf8\uad6d\ub9cc \uc5c6\uc5c8\uc73c\uba74 \ud55c\uad6d\uc740 \uc9c4\uc791\uc5d0 \uc190\ubd24\uc744 \ub098\ub77c\"\ub77c\uace0 \ud558\uba70 \ud55c\uad6d\uc744 \uad70\uc0ac\uc801\uc73c\ub85c \uc704\ud611\ud558\ub294 \ubc1c\uc5b8\uc744 \ud558\uc600\uace0 \ub9cc\uc57d\uc5d0 \uc8fc\ud55c\ubbf8\uad70\uc774 \ud55c\ubc18\ub3c4\uc5d0\uc11c \ucca0\uc218\ud55c\ub2e4\uba74 \uc911\uad6d\uc774 \uc774 \ubc1c\uc5b8\uc744 \uc2e4\ud589\uc73c\ub85c \uc62e\uae38 \uac00\ub2a5\uc131\uc774 \uc788\ub2e4. \uc911\uad6d\uc740 2025\ub144 \uae4c\uc9c0 \uc81c1\ub3c4\ub828\uc120\uc744 \uc810\ub839\ud558\uaca0\ub2e4\ub294 \uacc4\ud68d\uc744 \uac16\uace0 \uc788\ub294\ub370, \uc81c1\ub3c4\ub828\uc120\uc5d0\ub294 \ud55c\ubc18\ub3c4\uac00 \ud3ec\ud568\ub41c\ub2e4.", "sentence_answer": "\uc911\uad6d\uc740 2025\ub144 \uae4c\uc9c0 \uc81c1\ub3c4\ub828\uc120\uc744 \uc810\ub839\ud558\uaca0\ub2e4\ub294 \uacc4\ud68d\uc744 \uac16\uace0 \uc788\ub294\ub370, \uc81c1\ub3c4\ub828\uc120\uc5d0\ub294 \ud55c\ubc18\ub3c4\uac00 \ud3ec\ud568\ub41c\ub2e4.", "paragraph_id": "6512357-24-1", "paragraph_question": "question: \uc911\uad6d\uc774 \uc81c1\ub3c4\ub828\uc120\uc744 \uc810\ub839\ud558\uae30\ub85c \ud55c \uae30\ud55c\uc740 \uba87 \ub144\uae4c\uc9c0\uc778\uac00?, context: \uc870\uc9c0\ud0c0\uc6b4 \ub300\ud559\uad50 \ube45\ud130 \ucc28 \uad50\uc218\ub294 \u201c\ud55c\uad6d\uc740 \ubbf8\uad6d\uacfc \uc804\ub7b5\uc801\uc73c\ub85c \ubc00\uc811\ud55c \uad00\uacc4\ub97c \ub9fa\uc5b4\uc57c \uc911\uad6d\uc758 \uc874\uc911\uc744 \ubc1b\uace0 \ub354 \ub098\uc740 \ub300\uc6b0\ub97c \ubc1b\uc744 \uc218 \uc788\ub2e4\u201d\uba70 \ud55c\ubbf8\ub3d9\ub9f9\uc5d0 \ubb38\uc81c\uac00 \uc0dd\uacbc\uc744 \uacbd\uc6b0 \ud55c\uad6d\uc774 \uc911\uad6d\uc73c\ub85c\ubd80\ud130 \uc874\uc911\ubc1b\uc9c0 \ubabb\ud558\uace0 \ub300\uc6b0\ubc1b\uc9c0 \ubabb\ud560 \uc218 \uc788\ub2e4\uace0 \ud588\ub2e4. 2017\ub144 \uae30\uc900\uc73c\ub85c 200\uac1c\uc758 \ubd81\ud55c \ubbf8\uc0ac\uc77c\uacfc 600\uac1c \uc774\uc0c1\uc758 \uc911\uad6d \ubbf8\uc0ac\uc77c\uc774 \ud55c\uad6d\uad70\uacfc \uc8fc\ud55c\ubbf8\uad70 \uae30\uc9c0\ub97c \ud5a5\ud574 \uc870\uc900\ub418\uc5b4 \uc788\ub2e4. \uc8fc\ud55c\ubbf8\uad70\uc774 \ud55c\ubc18\ub3c4\uc5d0 \uc8fc\ub454\ud568\uc73c\ub85c\uc11c \ubbf8\uad6d\uc774 \uc5bb\uace0 \uc788\ub294 \uac00\uc7a5 \uc8fc\ub41c \uc804\ub7b5\uc801 \uac00\uce58\ub294 \uc911\uad6d\uc744 \uacac\uc81c\ud560 \uc218 \uc788\ub2e4\ub294 \uac83\uc774\uace0 \uc0ac\ub4dc \ubc30\uce58\uac00 \ucca0\ud68c\ub418\uba74 \uc624\ud788\ub824 \ubc18\ub300\ub85c \uc8fc\ud55c\ubbf8\uad70\uc774 600\uac1c \uc774\uc0c1\uc758 \uc911\uad6d \ubbf8\uc0ac\uc77c\ub85c \uc778\ud574 \uc704\ud611\uc744 \ubc1b\uac8c \ub418\uae30 \ub54c\ubb38\uc5d0 \ubbf8\uad6d\uc740 \ud55c\ubc18\ub3c4\ub97c \ub354 \uc774\uc0c1 \uc804\ub7b5\uc801 \uc694\ucda9\uc9c0\ub85c \ud65c\uc6a9\ud560 \uc218 \uc5c6\uac8c \ub41c\ub2e4. \uc0ac\ub4dc\ub294 \uc8fc\ud55c\ubbf8\uad70\uc744 \ubcf4\ud638\ud558\ub294 \uac83\uc774 \uc77c\ucc28\uc801 \ubaa9\ud45c\uc774\uace0 \ud55c\ubc18\ub3c4\uc5d0 \uc8fc\ub454\ud558\uace0 \uc788\ub294 \uc8fc\ud55c\ubbf8\uad70\uc774 \ubd81\ud55c\uacfc \uc911\uad6d\uc758 \ubbf8\uc0ac\uc77c\ub85c \uc778\ud574 \uc704\ud611\uc744 \ubc1b\uace0 \uc788\ub294 \uc0c1\ud669\uc5d0\uc11c\ub294 \ud55c\ubc18\ub3c4\ub97c \ub354 \uc774\uc0c1 \uc804\ub7b5\uc801 \uc694\ucda9\uc9c0\ub85c \ud65c\uc6a9\ud560 \uc218 \uc5c6\uc744 \ubfd0\ub9cc \uc544\ub2c8\ub77c \uc624\ud788\ub824 \ud55c\ubc18\ub3c4\uc5d0 \uc8fc\ub454\ud558\uace0 \uc788\ub294 \uc8fc\ud55c\ubbf8\uad70\uc774 \uc911\uad6d \ubbf8\uc0ac\uc77c\uc758 \uc778\uc9c8\ub85c \uc7a1\ud600 \uc788\ub294 \ud615\uad6d\uc774 \ub418\uc5b4 \ubc84\ub9b0\ub2e4. \ud55c\ubc18\ub3c4\uc5d0 \uc8fc\ub454\ud558\uace0 \uc788\ub294 \uc8fc\ud55c\ubbf8\uad70\uc744 \ubcf4\ud638\ud558\uae30 \uc704\ud574 \ubc30\uce58\ub41c \uc0ac\ub4dc\uac00 \ucca0\ud68c\ub41c\ub2e4\uba74 \uc8fc\ud55c\ubbf8\uad70\uc740 \uc911\uad6d \ubbf8\uc0ac\uc77c\uc5d0 \ubb34\ubc29\ube44 \uc0c1\ud0dc\ub85c \ub178\ucd9c\ub418\uc5b4 \uc704\ud611\uc744 \ubc1b\uac8c \ub418\uace0 \uc774\ub85c \uc778\ud574 \uc8fc\ud55c\ubbf8\uad70\uc774 \ud55c\ubc18\ub3c4\uc5d0 \uc8fc\ub454\ud568\uc73c\ub85c\uc11c \ubbf8\uad6d\uc774 \uc5bb\uc744 \uc218 \uc788\ub294 \uc804\ub7b5\uc801 \uac00\uce58\uac00 \uc18c\uba78\ub418\uae30 \ub54c\ubb38\uc5d0 \ubbf8\uad6d\uc815\ubd80\uac00 \uc8fc\ud55c\ubbf8\uad70\uc744 \ucca0\uc218\uc2dc\ud0ac \uc218\ubc16\uc5d0 \uc5c6\uac8c \ub41c\ub2e4. \uc911\uad6d\uc740 \ubbfc\uc8fc\ud601\uba85\uc744 \uac70\uce58\uc9c0 \uc54a\uc740 \uacf5\uc0b0\ub2f9 1\ub2f9\uccb4\uc81c \uacf5\uc0b0\uc8fc\uc758 \uad6d\uac00\uc774\uba70 \uc911\uad6d\uc740 \"\ubbf8\uad6d\ub9cc \uc5c6\uc5c8\uc73c\uba74 \ud55c\uad6d\uc740 \uc9c4\uc791\uc5d0 \uc190\ubd24\uc744 \ub098\ub77c\"\ub77c\uace0 \ud558\uba70 \ud55c\uad6d\uc744 \uad70\uc0ac\uc801\uc73c\ub85c \uc704\ud611\ud558\ub294 \ubc1c\uc5b8\uc744 \ud558\uc600\uace0 \ub9cc\uc57d\uc5d0 \uc8fc\ud55c\ubbf8\uad70\uc774 \ud55c\ubc18\ub3c4\uc5d0\uc11c \ucca0\uc218\ud55c\ub2e4\uba74 \uc911\uad6d\uc774 \uc774 \ubc1c\uc5b8\uc744 \uc2e4\ud589\uc73c\ub85c \uc62e\uae38 \uac00\ub2a5\uc131\uc774 \uc788\ub2e4. \uc911\uad6d\uc740 2025\ub144\uae4c\uc9c0 \uc81c1\ub3c4\ub828\uc120\uc744 \uc810\ub839\ud558\uaca0\ub2e4\ub294 \uacc4\ud68d\uc744 \uac16\uace0 \uc788\ub294\ub370, \uc81c1\ub3c4\ub828\uc120\uc5d0\ub294 \ud55c\ubc18\ub3c4\uac00 \ud3ec\ud568\ub41c\ub2e4."} {"question": "\ub3c4\ub9dd\uce5c \ub178\uc608\ub4e4\uc744 \uce90\ub098\uae4c\uc9c0 \uac08\uc218 \uc788\uc73c\uba70, \uc9c1\uc5c5\uae4c\uc9c0 \ucc3e\uc544\uc900 \uc2dc\uae30\ub294 \ubb34\uc5c7\uc774 \uc81c\uc815\ub420 \ub54c\uc778\uac00\uc694?", "paragraph": "1849\ub144, \ud130\ube0c\uba3c\uc740 \ud544\ub77c\ub378\ud53c\uc544\ub85c \ub3c4\ub9dd\ucce4\ub2e4\uac00, \ubc14\ub85c \uba54\ub9b4\ub79c\ub4dc\ub85c \uadf8\uc758 \uac00\uc871\uc744 \uad6c\ud574\ub0b4\uae30 \uc704\ud574 \ub3cc\uc544\uc654\ub2e4. \ucc9c\ucc9c\ud788, \ud55c \ubc88\uc5d0 \ud55c \uadf8\ub8f9\uc529, \uadf8\ub294 \uc790\uc2e0\uc758 \uce5c\ucc99\ub4e4\uc744 \uc8fc \ubc16\uc73c\ub85c \ub370\ub9ac\uace0 \ub098\uac14\uc73c\uba70, \uacb0\uad6d \uc218\uc2ed \uba85\uc758 \ub178\uc608\uc5d0\uac8c \uc790\uc720\ub97c \ucc3e\uc544\uc8fc\uc5c8\ub2e4. \uc544\uc8fc \uc740\ubc00\ud558\uac8c \ubc24\uc5d0\ub9cc \uc6c0\uc9c1\uc600\ub358 \ud130\ube0c\uba3c(\uc774\ub54c\ubd80\ud130 \ud558\ube44\ub8e8 \ub97c \uad6c\uc57d\uc131\uc11c\uc758 \ucd9c\uc560\uad7d\uae30\uc5d0 \ub530\ub974\uba74 \uc720\ub300\ubbfc\uc871\uc744 \uc774\uc9d1\ud2b8\uc758 \ucc29\ucde8\uc640 \uc5b5\uc555\uc5d0\uc11c \uad6c\uc6d0\ud55c\u201c\ubaa8\uc138\u201d\ub77c\uace0 \ubd88\ub838\ub2e4)\uc740 \u201c\ud55c \ubc88\ub3c4 \uc190\ub2d8\uc744 \ub193\uce5c \uc801\uc774 \uc5c6\uc5c8\ub2e4.\u201d \uadf8 \ub355\ubd84\uc5d0 \ud0c8\ucd9c\ud55c \uc0ac\ub78c\ub4e4\uc5d0\uac90 \uac70\uc561\uc758 \ud604\uc0c1\uae08\uc774 \uac78\ub9ac\uace4 \ud558\uc600\uc9c0\ub9cc, \uadf8\ub4e4\uc744 \ub3d5\ub294 \uac8c \ud574\ub9ac\uc5c7 \ud130\ube0c\uba3c\uc778 \uac83\uc740 \uc544\ubb34\ub3c4 \ubab0\ub790\ub2e4. \ub3c4\ub9dd\ub178\uc608\ubc95\uc774 \uc81c\uc815\ub41c \ub54c\uc5d0\ub294, \uadf8\ub294 \ub3c4\ub9dd\uce5c \ub178\uc608\ub4e4\uc744 \uce90\ub098\ub2e4\uae4c\uc9c0 \uac08 \uc218 \uc788\uac8c \ub3c4\uc640\uc8fc\uc5c8\uc73c\uba70, \ub3d9\uc2dc\uc5d0 \uadf8\ub4e4\uc5d0\uac8c \uc9c1\uc5c5\uc744 \ucc3e\uc544\uc8fc\uae30\ub3c4 \ud558\uc600\ub2e4.", "answer": "\ub3c4\ub9dd\ub178\uc608\ubc95", "sentence": "\ub3c4\ub9dd\ub178\uc608\ubc95 \uc774 \uc81c\uc815\ub41c \ub54c\uc5d0\ub294, \uadf8\ub294 \ub3c4\ub9dd\uce5c \ub178\uc608\ub4e4\uc744 \uce90\ub098\ub2e4\uae4c\uc9c0 \uac08 \uc218 \uc788\uac8c \ub3c4\uc640\uc8fc\uc5c8\uc73c\uba70, \ub3d9\uc2dc\uc5d0 \uadf8\ub4e4\uc5d0\uac8c \uc9c1\uc5c5\uc744 \ucc3e\uc544\uc8fc\uae30\ub3c4 \ud558\uc600\ub2e4.", "paragraph_sentence": "1849\ub144, \ud130\ube0c\uba3c\uc740 \ud544\ub77c\ub378\ud53c\uc544\ub85c \ub3c4\ub9dd\ucce4\ub2e4\uac00, \ubc14\ub85c \uba54\ub9b4\ub79c\ub4dc\ub85c \uadf8\uc758 \uac00\uc871\uc744 \uad6c\ud574\ub0b4\uae30 \uc704\ud574 \ub3cc\uc544\uc654\ub2e4. \ucc9c\ucc9c\ud788, \ud55c \ubc88\uc5d0 \ud55c \uadf8\ub8f9\uc529, \uadf8\ub294 \uc790\uc2e0\uc758 \uce5c\ucc99\ub4e4\uc744 \uc8fc \ubc16\uc73c\ub85c \ub370\ub9ac\uace0 \ub098\uac14\uc73c\uba70, \uacb0\uad6d \uc218\uc2ed \uba85\uc758 \ub178\uc608\uc5d0\uac8c \uc790\uc720\ub97c \ucc3e\uc544\uc8fc\uc5c8\ub2e4. \uc544\uc8fc \uc740\ubc00\ud558\uac8c \ubc24\uc5d0\ub9cc \uc6c0\uc9c1\uc600\ub358 \ud130\ube0c\uba3c(\uc774\ub54c\ubd80\ud130 \ud558\ube44\ub8e8 \ub97c \uad6c\uc57d\uc131\uc11c\uc758 \ucd9c\uc560\uad7d\uae30\uc5d0 \ub530\ub974\uba74 \uc720\ub300\ubbfc\uc871\uc744 \uc774\uc9d1\ud2b8\uc758 \ucc29\ucde8\uc640 \uc5b5\uc555\uc5d0\uc11c \uad6c\uc6d0\ud55c\u201c\ubaa8\uc138\u201d\ub77c\uace0 \ubd88\ub838\ub2e4)\uc740 \u201c\ud55c \ubc88\ub3c4 \uc190\ub2d8\uc744 \ub193\uce5c \uc801\uc774 \uc5c6\uc5c8\ub2e4. \u201d \uadf8 \ub355\ubd84\uc5d0 \ud0c8\ucd9c\ud55c \uc0ac\ub78c\ub4e4\uc5d0\uac90 \uac70\uc561\uc758 \ud604\uc0c1\uae08\uc774 \uac78\ub9ac\uace4 \ud558\uc600\uc9c0\ub9cc, \uadf8\ub4e4\uc744 \ub3d5\ub294 \uac8c \ud574\ub9ac\uc5c7 \ud130\ube0c\uba3c\uc778 \uac83\uc740 \uc544\ubb34\ub3c4 \ubab0\ub790\ub2e4. \ub3c4\ub9dd\ub178\uc608\ubc95 \uc774 \uc81c\uc815\ub41c \ub54c\uc5d0\ub294, \uadf8\ub294 \ub3c4\ub9dd\uce5c \ub178\uc608\ub4e4\uc744 \uce90\ub098\ub2e4\uae4c\uc9c0 \uac08 \uc218 \uc788\uac8c \ub3c4\uc640\uc8fc\uc5c8\uc73c\uba70, \ub3d9\uc2dc\uc5d0 \uadf8\ub4e4\uc5d0\uac8c \uc9c1\uc5c5\uc744 \ucc3e\uc544\uc8fc\uae30\ub3c4 \ud558\uc600\ub2e4. ", "paragraph_answer": "1849\ub144, \ud130\ube0c\uba3c\uc740 \ud544\ub77c\ub378\ud53c\uc544\ub85c \ub3c4\ub9dd\ucce4\ub2e4\uac00, \ubc14\ub85c \uba54\ub9b4\ub79c\ub4dc\ub85c \uadf8\uc758 \uac00\uc871\uc744 \uad6c\ud574\ub0b4\uae30 \uc704\ud574 \ub3cc\uc544\uc654\ub2e4. \ucc9c\ucc9c\ud788, \ud55c \ubc88\uc5d0 \ud55c \uadf8\ub8f9\uc529, \uadf8\ub294 \uc790\uc2e0\uc758 \uce5c\ucc99\ub4e4\uc744 \uc8fc \ubc16\uc73c\ub85c \ub370\ub9ac\uace0 \ub098\uac14\uc73c\uba70, \uacb0\uad6d \uc218\uc2ed \uba85\uc758 \ub178\uc608\uc5d0\uac8c \uc790\uc720\ub97c \ucc3e\uc544\uc8fc\uc5c8\ub2e4. \uc544\uc8fc \uc740\ubc00\ud558\uac8c \ubc24\uc5d0\ub9cc \uc6c0\uc9c1\uc600\ub358 \ud130\ube0c\uba3c(\uc774\ub54c\ubd80\ud130 \ud558\ube44\ub8e8 \ub97c \uad6c\uc57d\uc131\uc11c\uc758 \ucd9c\uc560\uad7d\uae30\uc5d0 \ub530\ub974\uba74 \uc720\ub300\ubbfc\uc871\uc744 \uc774\uc9d1\ud2b8\uc758 \ucc29\ucde8\uc640 \uc5b5\uc555\uc5d0\uc11c \uad6c\uc6d0\ud55c\u201c\ubaa8\uc138\u201d\ub77c\uace0 \ubd88\ub838\ub2e4)\uc740 \u201c\ud55c \ubc88\ub3c4 \uc190\ub2d8\uc744 \ub193\uce5c \uc801\uc774 \uc5c6\uc5c8\ub2e4.\u201d \uadf8 \ub355\ubd84\uc5d0 \ud0c8\ucd9c\ud55c \uc0ac\ub78c\ub4e4\uc5d0\uac90 \uac70\uc561\uc758 \ud604\uc0c1\uae08\uc774 \uac78\ub9ac\uace4 \ud558\uc600\uc9c0\ub9cc, \uadf8\ub4e4\uc744 \ub3d5\ub294 \uac8c \ud574\ub9ac\uc5c7 \ud130\ube0c\uba3c\uc778 \uac83\uc740 \uc544\ubb34\ub3c4 \ubab0\ub790\ub2e4. \ub3c4\ub9dd\ub178\uc608\ubc95 \uc774 \uc81c\uc815\ub41c \ub54c\uc5d0\ub294, \uadf8\ub294 \ub3c4\ub9dd\uce5c \ub178\uc608\ub4e4\uc744 \uce90\ub098\ub2e4\uae4c\uc9c0 \uac08 \uc218 \uc788\uac8c \ub3c4\uc640\uc8fc\uc5c8\uc73c\uba70, \ub3d9\uc2dc\uc5d0 \uadf8\ub4e4\uc5d0\uac8c \uc9c1\uc5c5\uc744 \ucc3e\uc544\uc8fc\uae30\ub3c4 \ud558\uc600\ub2e4.", "sentence_answer": " \ub3c4\ub9dd\ub178\uc608\ubc95 \uc774 \uc81c\uc815\ub41c \ub54c\uc5d0\ub294, \uadf8\ub294 \ub3c4\ub9dd\uce5c \ub178\uc608\ub4e4\uc744 \uce90\ub098\ub2e4\uae4c\uc9c0 \uac08 \uc218 \uc788\uac8c \ub3c4\uc640\uc8fc\uc5c8\uc73c\uba70, \ub3d9\uc2dc\uc5d0 \uadf8\ub4e4\uc5d0\uac8c \uc9c1\uc5c5\uc744 \ucc3e\uc544\uc8fc\uae30\ub3c4 \ud558\uc600\ub2e4.", "paragraph_id": "6570493-1-1", "paragraph_question": "question: \ub3c4\ub9dd\uce5c \ub178\uc608\ub4e4\uc744 \uce90\ub098\uae4c\uc9c0 \uac08\uc218 \uc788\uc73c\uba70, \uc9c1\uc5c5\uae4c\uc9c0 \ucc3e\uc544\uc900 \uc2dc\uae30\ub294 \ubb34\uc5c7\uc774 \uc81c\uc815\ub420 \ub54c\uc778\uac00\uc694?, context: 1849\ub144, \ud130\ube0c\uba3c\uc740 \ud544\ub77c\ub378\ud53c\uc544\ub85c \ub3c4\ub9dd\ucce4\ub2e4\uac00, \ubc14\ub85c \uba54\ub9b4\ub79c\ub4dc\ub85c \uadf8\uc758 \uac00\uc871\uc744 \uad6c\ud574\ub0b4\uae30 \uc704\ud574 \ub3cc\uc544\uc654\ub2e4. \ucc9c\ucc9c\ud788, \ud55c \ubc88\uc5d0 \ud55c \uadf8\ub8f9\uc529, \uadf8\ub294 \uc790\uc2e0\uc758 \uce5c\ucc99\ub4e4\uc744 \uc8fc \ubc16\uc73c\ub85c \ub370\ub9ac\uace0 \ub098\uac14\uc73c\uba70, \uacb0\uad6d \uc218\uc2ed \uba85\uc758 \ub178\uc608\uc5d0\uac8c \uc790\uc720\ub97c \ucc3e\uc544\uc8fc\uc5c8\ub2e4. \uc544\uc8fc \uc740\ubc00\ud558\uac8c \ubc24\uc5d0\ub9cc \uc6c0\uc9c1\uc600\ub358 \ud130\ube0c\uba3c(\uc774\ub54c\ubd80\ud130 \ud558\ube44\ub8e8 \ub97c \uad6c\uc57d\uc131\uc11c\uc758 \ucd9c\uc560\uad7d\uae30\uc5d0 \ub530\ub974\uba74 \uc720\ub300\ubbfc\uc871\uc744 \uc774\uc9d1\ud2b8\uc758 \ucc29\ucde8\uc640 \uc5b5\uc555\uc5d0\uc11c \uad6c\uc6d0\ud55c\u201c\ubaa8\uc138\u201d\ub77c\uace0 \ubd88\ub838\ub2e4)\uc740 \u201c\ud55c \ubc88\ub3c4 \uc190\ub2d8\uc744 \ub193\uce5c \uc801\uc774 \uc5c6\uc5c8\ub2e4.\u201d \uadf8 \ub355\ubd84\uc5d0 \ud0c8\ucd9c\ud55c \uc0ac\ub78c\ub4e4\uc5d0\uac90 \uac70\uc561\uc758 \ud604\uc0c1\uae08\uc774 \uac78\ub9ac\uace4 \ud558\uc600\uc9c0\ub9cc, \uadf8\ub4e4\uc744 \ub3d5\ub294 \uac8c \ud574\ub9ac\uc5c7 \ud130\ube0c\uba3c\uc778 \uac83\uc740 \uc544\ubb34\ub3c4 \ubab0\ub790\ub2e4. \ub3c4\ub9dd\ub178\uc608\ubc95\uc774 \uc81c\uc815\ub41c \ub54c\uc5d0\ub294, \uadf8\ub294 \ub3c4\ub9dd\uce5c \ub178\uc608\ub4e4\uc744 \uce90\ub098\ub2e4\uae4c\uc9c0 \uac08 \uc218 \uc788\uac8c \ub3c4\uc640\uc8fc\uc5c8\uc73c\uba70, \ub3d9\uc2dc\uc5d0 \uadf8\ub4e4\uc5d0\uac8c \uc9c1\uc5c5\uc744 \ucc3e\uc544\uc8fc\uae30\ub3c4 \ud558\uc600\ub2e4."} {"question": "2008\ub144 \uc2e4\uc2dc\ub41c \ube44\ub514\uc624\uac8c\uc784 \uad00\ub828 \uc804\ud654\uc870\uc0ac\ub294 \uc5b4\ub290 \ub098\ub77c\uc5d0\uc11c \uc2e4\uc2dc\ub418\uc5c8\ub294\uac00?", "paragraph": "\uac8c\uc774\uba38 \uc778\uad6c\uac00 \uc99d\uac00\ud558\uace0 \uc788\ub294 \ud604\uc0c1\uc5d0 \ub300\ud55c \ud55c \uac00\uc9c0 \uc774\uc720\ub85c\ub294 \uc810\uc810 \ub354 \ub9ce\uc740 \uac8c\uc784 \uc7a5\ub974\uac00 \ub2e4\uc591\ud55c \ub300\uc911\uc744 \ub300\uc0c1\uc73c\ub85c \uc0bc\uace0 \uc788\ub2e4\ub294 \uac83\uc744 \ub4e4 \uc218 \uc788\ub2e4. \uc608\ub97c \ub4e4\uc5b4, Wii \ucf58\uc194\uc740 Wii \uc2a4\ud3ec\uce20\uc640 Wii Fit \ub4f1\uc758 \uac8c\uc784\uc744 \ud1b5\ud574\uc11c \uc720\uc800 \uce35\uc744 \ub113\ud600\uc654\ub2e4. \ub450 \uac00\uc9c0 \uac8c\uc784 \ubaa8\ub450 \uc0ac\uc6a9\uc790\uc758 \ud65c\ub3d9\uc801\uc778 \uc6c0\uc9c1\uc784\uc744 \uc774\ub04c\uc5b4\ub0b4\uba70 \uac00\uc871 \uac04\uc758 \uaca8\ub8e8\uae30\ub098 \uc6b4\ub3d9 \ub4f1\uc758 \uc990\uae38\ub9cc\ud55c \uc774\uc720\ub97c \uc81c\uacf5\ud558\uae30 \ub54c\ubb38\uc774\ub2e4. \ubfd0\ub9cc \uc544\ub2c8\ub77c, \uc5b4\ub9b0 \uc2dc\uc808\ubd80\ud130 \ube44\ub514\uc624 \uac8c\uc784\uc744 \uc990\uaca8\uc628 \uc0ac\ub78c\ub4e4\uc774 \ub098\uc774\ub97c \uba39\uace0 \ub098\uc11c\ub3c4 \ube44\ub514\uc624 \uac8c\uc784\uc5d0 \ud765\ubbf8\ub97c \uac00\uc9c0\uace0 \uc788\uae30 \ub54c\ubb38\uc774\ub77c\ub294 \uc774\uc720\ub3c4 \uc788\ub2e4. \ud604\uc7ac, \uc5b4\ub9b0\uc774\ub4e4\uc744 \uc704\ud55c \uac00\uc7a5 \ud070 \uc624\ub77d \uc0b0\uc5c5\uc774 \ubc14\ub85c \uac8c\uc784\uc774\uae30\ub3c4 \ud558\ub2e4. 2008\ub144\uc5d0 \uc2e4\uc2dc\ub418\uc5b4 1,102\uba85\uc758 \uc751\ub2f5\uc790\ub97c \ud45c\ubcf8\uc73c\ub85c \uac00\uc9c0\ub294 \uc804\ud654\uc870\uc0ac\uc5d0\uc11c\ub294 \ubbf8\uad6d\uc5d0 \uc0ac\ub294 12\uc138\uc5d0\uc11c 17\uc138 \uc0ac\uc774\uc758 \uc5b4\ub9b0\uc774 \uc911 97%\uac00 \ube44\ub514\uc624 \uac8c\uc784\uc744 \uc990\uae34\ub2e4\uace0 \uc751\ub2f5\ud558\uc600\ub2e4.", "answer": "\ubbf8\uad6d", "sentence": "2008\ub144\uc5d0 \uc2e4\uc2dc\ub418\uc5b4 1,102\uba85\uc758 \uc751\ub2f5\uc790\ub97c \ud45c\ubcf8\uc73c\ub85c \uac00\uc9c0\ub294 \uc804\ud654\uc870\uc0ac\uc5d0\uc11c\ub294 \ubbf8\uad6d \uc5d0 \uc0ac\ub294 12\uc138\uc5d0\uc11c 17\uc138 \uc0ac\uc774\uc758 \uc5b4\ub9b0\uc774 \uc911 97%\uac00 \ube44\ub514\uc624 \uac8c\uc784\uc744 \uc990\uae34\ub2e4\uace0 \uc751\ub2f5\ud558\uc600\ub2e4.", "paragraph_sentence": "\uac8c\uc774\uba38 \uc778\uad6c\uac00 \uc99d\uac00\ud558\uace0 \uc788\ub294 \ud604\uc0c1\uc5d0 \ub300\ud55c \ud55c \uac00\uc9c0 \uc774\uc720\ub85c\ub294 \uc810\uc810 \ub354 \ub9ce\uc740 \uac8c\uc784 \uc7a5\ub974\uac00 \ub2e4\uc591\ud55c \ub300\uc911\uc744 \ub300\uc0c1\uc73c\ub85c \uc0bc\uace0 \uc788\ub2e4\ub294 \uac83\uc744 \ub4e4 \uc218 \uc788\ub2e4. \uc608\ub97c \ub4e4\uc5b4, Wii \ucf58\uc194\uc740 Wii \uc2a4\ud3ec\uce20\uc640 Wii Fit \ub4f1\uc758 \uac8c\uc784\uc744 \ud1b5\ud574\uc11c \uc720\uc800 \uce35\uc744 \ub113\ud600\uc654\ub2e4. \ub450 \uac00\uc9c0 \uac8c\uc784 \ubaa8\ub450 \uc0ac\uc6a9\uc790\uc758 \ud65c\ub3d9\uc801\uc778 \uc6c0\uc9c1\uc784\uc744 \uc774\ub04c\uc5b4\ub0b4\uba70 \uac00\uc871 \uac04\uc758 \uaca8\ub8e8\uae30\ub098 \uc6b4\ub3d9 \ub4f1\uc758 \uc990\uae38\ub9cc\ud55c \uc774\uc720\ub97c \uc81c\uacf5\ud558\uae30 \ub54c\ubb38\uc774\ub2e4. \ubfd0\ub9cc \uc544\ub2c8\ub77c, \uc5b4\ub9b0 \uc2dc\uc808\ubd80\ud130 \ube44\ub514\uc624 \uac8c\uc784\uc744 \uc990\uaca8\uc628 \uc0ac\ub78c\ub4e4\uc774 \ub098\uc774\ub97c \uba39\uace0 \ub098\uc11c\ub3c4 \ube44\ub514\uc624 \uac8c\uc784\uc5d0 \ud765\ubbf8\ub97c \uac00\uc9c0\uace0 \uc788\uae30 \ub54c\ubb38\uc774\ub77c\ub294 \uc774\uc720\ub3c4 \uc788\ub2e4. \ud604\uc7ac, \uc5b4\ub9b0\uc774\ub4e4\uc744 \uc704\ud55c \uac00\uc7a5 \ud070 \uc624\ub77d \uc0b0\uc5c5\uc774 \ubc14\ub85c \uac8c\uc784\uc774\uae30\ub3c4 \ud558\ub2e4. 2008\ub144\uc5d0 \uc2e4\uc2dc\ub418\uc5b4 1,102\uba85\uc758 \uc751\ub2f5\uc790\ub97c \ud45c\ubcf8\uc73c\ub85c \uac00\uc9c0\ub294 \uc804\ud654\uc870\uc0ac\uc5d0\uc11c\ub294 \ubbf8\uad6d \uc5d0 \uc0ac\ub294 12\uc138\uc5d0\uc11c 17\uc138 \uc0ac\uc774\uc758 \uc5b4\ub9b0\uc774 \uc911 97%\uac00 \ube44\ub514\uc624 \uac8c\uc784\uc744 \uc990\uae34\ub2e4\uace0 \uc751\ub2f5\ud558\uc600\ub2e4. ", "paragraph_answer": "\uac8c\uc774\uba38 \uc778\uad6c\uac00 \uc99d\uac00\ud558\uace0 \uc788\ub294 \ud604\uc0c1\uc5d0 \ub300\ud55c \ud55c \uac00\uc9c0 \uc774\uc720\ub85c\ub294 \uc810\uc810 \ub354 \ub9ce\uc740 \uac8c\uc784 \uc7a5\ub974\uac00 \ub2e4\uc591\ud55c \ub300\uc911\uc744 \ub300\uc0c1\uc73c\ub85c \uc0bc\uace0 \uc788\ub2e4\ub294 \uac83\uc744 \ub4e4 \uc218 \uc788\ub2e4. \uc608\ub97c \ub4e4\uc5b4, Wii \ucf58\uc194\uc740 Wii \uc2a4\ud3ec\uce20\uc640 Wii Fit \ub4f1\uc758 \uac8c\uc784\uc744 \ud1b5\ud574\uc11c \uc720\uc800 \uce35\uc744 \ub113\ud600\uc654\ub2e4. \ub450 \uac00\uc9c0 \uac8c\uc784 \ubaa8\ub450 \uc0ac\uc6a9\uc790\uc758 \ud65c\ub3d9\uc801\uc778 \uc6c0\uc9c1\uc784\uc744 \uc774\ub04c\uc5b4\ub0b4\uba70 \uac00\uc871 \uac04\uc758 \uaca8\ub8e8\uae30\ub098 \uc6b4\ub3d9 \ub4f1\uc758 \uc990\uae38\ub9cc\ud55c \uc774\uc720\ub97c \uc81c\uacf5\ud558\uae30 \ub54c\ubb38\uc774\ub2e4. \ubfd0\ub9cc \uc544\ub2c8\ub77c, \uc5b4\ub9b0 \uc2dc\uc808\ubd80\ud130 \ube44\ub514\uc624 \uac8c\uc784\uc744 \uc990\uaca8\uc628 \uc0ac\ub78c\ub4e4\uc774 \ub098\uc774\ub97c \uba39\uace0 \ub098\uc11c\ub3c4 \ube44\ub514\uc624 \uac8c\uc784\uc5d0 \ud765\ubbf8\ub97c \uac00\uc9c0\uace0 \uc788\uae30 \ub54c\ubb38\uc774\ub77c\ub294 \uc774\uc720\ub3c4 \uc788\ub2e4. \ud604\uc7ac, \uc5b4\ub9b0\uc774\ub4e4\uc744 \uc704\ud55c \uac00\uc7a5 \ud070 \uc624\ub77d \uc0b0\uc5c5\uc774 \ubc14\ub85c \uac8c\uc784\uc774\uae30\ub3c4 \ud558\ub2e4. 2008\ub144\uc5d0 \uc2e4\uc2dc\ub418\uc5b4 1,102\uba85\uc758 \uc751\ub2f5\uc790\ub97c \ud45c\ubcf8\uc73c\ub85c \uac00\uc9c0\ub294 \uc804\ud654\uc870\uc0ac\uc5d0\uc11c\ub294 \ubbf8\uad6d \uc5d0 \uc0ac\ub294 12\uc138\uc5d0\uc11c 17\uc138 \uc0ac\uc774\uc758 \uc5b4\ub9b0\uc774 \uc911 97%\uac00 \ube44\ub514\uc624 \uac8c\uc784\uc744 \uc990\uae34\ub2e4\uace0 \uc751\ub2f5\ud558\uc600\ub2e4.", "sentence_answer": "2008\ub144\uc5d0 \uc2e4\uc2dc\ub418\uc5b4 1,102\uba85\uc758 \uc751\ub2f5\uc790\ub97c \ud45c\ubcf8\uc73c\ub85c \uac00\uc9c0\ub294 \uc804\ud654\uc870\uc0ac\uc5d0\uc11c\ub294 \ubbf8\uad6d \uc5d0 \uc0ac\ub294 12\uc138\uc5d0\uc11c 17\uc138 \uc0ac\uc774\uc758 \uc5b4\ub9b0\uc774 \uc911 97%\uac00 \ube44\ub514\uc624 \uac8c\uc784\uc744 \uc990\uae34\ub2e4\uace0 \uc751\ub2f5\ud558\uc600\ub2e4.", "paragraph_id": "6535282-1-1", "paragraph_question": "question: 2008\ub144 \uc2e4\uc2dc\ub41c \ube44\ub514\uc624\uac8c\uc784 \uad00\ub828 \uc804\ud654\uc870\uc0ac\ub294 \uc5b4\ub290 \ub098\ub77c\uc5d0\uc11c \uc2e4\uc2dc\ub418\uc5c8\ub294\uac00?, context: \uac8c\uc774\uba38 \uc778\uad6c\uac00 \uc99d\uac00\ud558\uace0 \uc788\ub294 \ud604\uc0c1\uc5d0 \ub300\ud55c \ud55c \uac00\uc9c0 \uc774\uc720\ub85c\ub294 \uc810\uc810 \ub354 \ub9ce\uc740 \uac8c\uc784 \uc7a5\ub974\uac00 \ub2e4\uc591\ud55c \ub300\uc911\uc744 \ub300\uc0c1\uc73c\ub85c \uc0bc\uace0 \uc788\ub2e4\ub294 \uac83\uc744 \ub4e4 \uc218 \uc788\ub2e4. \uc608\ub97c \ub4e4\uc5b4, Wii \ucf58\uc194\uc740 Wii \uc2a4\ud3ec\uce20\uc640 Wii Fit \ub4f1\uc758 \uac8c\uc784\uc744 \ud1b5\ud574\uc11c \uc720\uc800 \uce35\uc744 \ub113\ud600\uc654\ub2e4. \ub450 \uac00\uc9c0 \uac8c\uc784 \ubaa8\ub450 \uc0ac\uc6a9\uc790\uc758 \ud65c\ub3d9\uc801\uc778 \uc6c0\uc9c1\uc784\uc744 \uc774\ub04c\uc5b4\ub0b4\uba70 \uac00\uc871 \uac04\uc758 \uaca8\ub8e8\uae30\ub098 \uc6b4\ub3d9 \ub4f1\uc758 \uc990\uae38\ub9cc\ud55c \uc774\uc720\ub97c \uc81c\uacf5\ud558\uae30 \ub54c\ubb38\uc774\ub2e4. \ubfd0\ub9cc \uc544\ub2c8\ub77c, \uc5b4\ub9b0 \uc2dc\uc808\ubd80\ud130 \ube44\ub514\uc624 \uac8c\uc784\uc744 \uc990\uaca8\uc628 \uc0ac\ub78c\ub4e4\uc774 \ub098\uc774\ub97c \uba39\uace0 \ub098\uc11c\ub3c4 \ube44\ub514\uc624 \uac8c\uc784\uc5d0 \ud765\ubbf8\ub97c \uac00\uc9c0\uace0 \uc788\uae30 \ub54c\ubb38\uc774\ub77c\ub294 \uc774\uc720\ub3c4 \uc788\ub2e4. \ud604\uc7ac, \uc5b4\ub9b0\uc774\ub4e4\uc744 \uc704\ud55c \uac00\uc7a5 \ud070 \uc624\ub77d \uc0b0\uc5c5\uc774 \ubc14\ub85c \uac8c\uc784\uc774\uae30\ub3c4 \ud558\ub2e4. 2008\ub144\uc5d0 \uc2e4\uc2dc\ub418\uc5b4 1,102\uba85\uc758 \uc751\ub2f5\uc790\ub97c \ud45c\ubcf8\uc73c\ub85c \uac00\uc9c0\ub294 \uc804\ud654\uc870\uc0ac\uc5d0\uc11c\ub294 \ubbf8\uad6d\uc5d0 \uc0ac\ub294 12\uc138\uc5d0\uc11c 17\uc138 \uc0ac\uc774\uc758 \uc5b4\ub9b0\uc774 \uc911 97%\uac00 \ube44\ub514\uc624 \uac8c\uc784\uc744 \uc990\uae34\ub2e4\uace0 \uc751\ub2f5\ud558\uc600\ub2e4."} {"question": "\ub300\uc6d0\ub4e4\uc5d0\uac8c \ubaa8\ubc94\uc801\uc774\uace0 \uc608\uc758\ubc14\ub978 \ubaa8\uc2b5\uc744 \uac16\ucd94\uae38 \uac15\uc870\ud55c \uac83\uc740 \ub204\uad6c\uc778\uac00?", "paragraph": "\ub3c5\uc77c\uc758 \uc5ed\uc0ac\ud559\uc790 Hans Buchheim\uc774 \ud504\ub791\ud06c\ud478\ub974\ud2b8\uc758 \ub258\ub978\ubca0\ub974\ud06c \uc7ac\ud310\uc5d0\uc11c \uae30\uc18c \uce21 \uc804\ubb38 \uac10\uc815\uc778\uc73c\ub85c\uc11c \ud65c\ub3d9\ud588\ub358 \uacbd\ud5d8\uc744 \ub2f4\uc544 1965\ub144\uc5d0 \ubc1c\uac04\ud55c \uc5d0\uc138\uc774 \u201cCommand and Compliance\u201d\uc5d0 \ub530\ub974\uba74, \uc544\uc6b0\uc288\ube44\uce20 \ub0b4\uc5d0\uc11c \uc720\ub300\uc778\uc744 \ube44\ub86f\ud55c \uc218\uac10\uc790\ub4e4\uc744 \uc0b4\ud574\ud558\ub294\ub370 \uc788\uc5b4 \uac00\ud574\uc790\ub4e4\uc758 \uc790\uc720\uc758\uc9c0\ub97c \uce68\ubc94\ud558\ub294 \uc5b4\ub5a0\ud55c \uac15\uc555\uc801\uc778 \uba85\ub839\ub3c4 \uc5c6\uc5c8\ub2e4\uace0 \ud55c\ub2e4. Buchheim\uc740 \ubc94\uc8c4\ub098 \ub2e4\ub984\uc5c6\ub294 \uadf8 \uba85\ub839\ub4e4\uc740 \ucda9\ubd84\ud788 \ud68c\ud53c\ud560 \uae30\ud68c\uac00 \uc788\uc5c8\ub2e4\uace0 \uc800\uc220\ud558\uba70, \uc774\ub7ec\ud55c \uba85\ub839\uc744 \uc774\ud589\ud558\uc9c0 \uc54a\uc740 SS\uce5c\uc704\ub300\uc6d0\ub4e4\uc774 \uac15\uc81c\uc218\uc6a9\uc18c\uc5d0 \uc218\uac10\ub418\uac70\ub098 \ucc98\ud615\ub2f9\ud588\ub2e4\ub294 \uc5b4\ub5a0\ud55c \uc99d\uac70\ub3c4 \ucc3e\uc744 \uc218 \uc5c6\uc5c8\ub2e4\uace0 \ud55c\ub2e4. \uac8c\ub2e4\uac00 Heinrich Himmler\uac00 \uadf8\uc758 \ub300\uc6d0\ub4e4\uc5d0\uac8c \u2018\ubaa8\ubc94\uc801\uc774\uace0 \uc608\uc758\ubc14\ub978\u2019 \ubaa8\uc2b5\uc744 \uac16\ucd94\uae38 \uac15\uc870\ud55c \ub9cc\ud07c SS\uce5c\uc704\ub300\ub294 \ubd88\ud544\uc694\ud55c \uac00\ud559\ud589\uc704\ub294 \uc624\ud788\ub824 \uc9c0\uc591\ud558\uace0 \uc788\uc5c8\uc73c\uba70, \uac00\ud559\ud589\uc704\ub294 \uadf8\ub4e4 \uc911 \ud2b9\ubcc4\ud788 \uc794\uc778\ud55c \uc131\ud5a5\uc744 \uc9c0\ub154\uac70\ub098 \uad6d\uac00 \uc0ac\ud68c\uc8fc\uc758\uc5d0 \uacfc\ub3c4\ud55c \ucda9\uc131\uc744 \uc99d\uba85\ud574\ubcf4\uc774\ub824\ub294 \uac1c\uc778\ub4e4\uc758 \ub3cc\ucd9c\ud589\ub3d9 \uc815\ub3c4\ub85c \uc5ec\uacbc\ub2e4. \ub9c8\uc9c0\ub9c9\uc73c\ub85c Buchheim\uc740 \uacb0\ubc31\ud55c \uc0ac\ub78c\ub4e4 \uc911 \uadf8\ub7f0 \uae38\ub85c \ube60\uc838\ubc84\ub9b0 \uc774\ub4e4 \uc5ed\uc2dc \uc18c\uc18d\uac10\uc744 \uc720\uc9c0\ud558\uae30\uc704\ud574, \ub610\ub294 \uba85\ub839\uc744 \uac70\ubd80\ud560 \uacbd\uc6b0 \ub3d9\ub8cc\ub85c\ubd80\ud130 \u2018\uc57d\uc790\u2019\ub85c \ucde8\uae09\ubc1b\uc9c0 \uc54a\uae30 \uc704\ud574 \uadf8\ub7ec\ud55c \uc120\ud0dd\uc744 \ud55c \uac83\uc77c \ubfd0 \ud53c\ud560 \uc218 \uc5c6\ub294 \uba85\ub839\uc774 \uc544\ub2c8\uc5c8\ub2e4\uace0 \uc8fc\uc7a5\ud55c\ub2e4.", "answer": "Heinrich Himmler", "sentence": "\uac8c\ub2e4\uac00 Heinrich Himmler \uac00 \uadf8\uc758 \ub300\uc6d0\ub4e4\uc5d0\uac8c \u2018\ubaa8\ubc94\uc801\uc774\uace0 \uc608\uc758\ubc14\ub978\u2019 \ubaa8\uc2b5\uc744 \uac16\ucd94\uae38 \uac15\uc870\ud55c \ub9cc\ud07c SS\uce5c\uc704\ub300\ub294 \ubd88\ud544\uc694\ud55c \uac00\ud559\ud589\uc704\ub294 \uc624\ud788\ub824 \uc9c0\uc591\ud558\uace0 \uc788\uc5c8\uc73c\uba70, \uac00\ud559\ud589\uc704\ub294 \uadf8\ub4e4 \uc911 \ud2b9\ubcc4\ud788 \uc794\uc778\ud55c \uc131\ud5a5\uc744 \uc9c0\ub154\uac70\ub098 \uad6d\uac00 \uc0ac\ud68c\uc8fc\uc758\uc5d0 \uacfc\ub3c4\ud55c \ucda9\uc131\uc744 \uc99d\uba85\ud574\ubcf4\uc774\ub824\ub294 \uac1c\uc778\ub4e4\uc758 \ub3cc\ucd9c\ud589\ub3d9 \uc815\ub3c4\ub85c \uc5ec\uacbc\ub2e4.", "paragraph_sentence": "\ub3c5\uc77c\uc758 \uc5ed\uc0ac\ud559\uc790 Hans Buchheim\uc774 \ud504\ub791\ud06c\ud478\ub974\ud2b8\uc758 \ub258\ub978\ubca0\ub974\ud06c \uc7ac\ud310\uc5d0\uc11c \uae30\uc18c \uce21 \uc804\ubb38 \uac10\uc815\uc778\uc73c\ub85c\uc11c \ud65c\ub3d9\ud588\ub358 \uacbd\ud5d8\uc744 \ub2f4\uc544 1965\ub144\uc5d0 \ubc1c\uac04\ud55c \uc5d0\uc138\uc774 \u201cCommand and Compliance\u201d\uc5d0 \ub530\ub974\uba74, \uc544\uc6b0\uc288\ube44\uce20 \ub0b4\uc5d0\uc11c \uc720\ub300\uc778\uc744 \ube44\ub86f\ud55c \uc218\uac10\uc790\ub4e4\uc744 \uc0b4\ud574\ud558\ub294\ub370 \uc788\uc5b4 \uac00\ud574\uc790\ub4e4\uc758 \uc790\uc720\uc758\uc9c0\ub97c \uce68\ubc94\ud558\ub294 \uc5b4\ub5a0\ud55c \uac15\uc555\uc801\uc778 \uba85\ub839\ub3c4 \uc5c6\uc5c8\ub2e4\uace0 \ud55c\ub2e4. Buchheim\uc740 \ubc94\uc8c4\ub098 \ub2e4\ub984\uc5c6\ub294 \uadf8 \uba85\ub839\ub4e4\uc740 \ucda9\ubd84\ud788 \ud68c\ud53c\ud560 \uae30\ud68c\uac00 \uc788\uc5c8\ub2e4\uace0 \uc800\uc220\ud558\uba70, \uc774\ub7ec\ud55c \uba85\ub839\uc744 \uc774\ud589\ud558\uc9c0 \uc54a\uc740 SS\uce5c\uc704\ub300\uc6d0\ub4e4\uc774 \uac15\uc81c\uc218\uc6a9\uc18c\uc5d0 \uc218\uac10\ub418\uac70\ub098 \ucc98\ud615\ub2f9\ud588\ub2e4\ub294 \uc5b4\ub5a0\ud55c \uc99d\uac70\ub3c4 \ucc3e\uc744 \uc218 \uc5c6\uc5c8\ub2e4\uace0 \ud55c\ub2e4. \uac8c\ub2e4\uac00 Heinrich Himmler \uac00 \uadf8\uc758 \ub300\uc6d0\ub4e4\uc5d0\uac8c \u2018\ubaa8\ubc94\uc801\uc774\uace0 \uc608\uc758\ubc14\ub978\u2019 \ubaa8\uc2b5\uc744 \uac16\ucd94\uae38 \uac15\uc870\ud55c \ub9cc\ud07c SS\uce5c\uc704\ub300\ub294 \ubd88\ud544\uc694\ud55c \uac00\ud559\ud589\uc704\ub294 \uc624\ud788\ub824 \uc9c0\uc591\ud558\uace0 \uc788\uc5c8\uc73c\uba70, \uac00\ud559\ud589\uc704\ub294 \uadf8\ub4e4 \uc911 \ud2b9\ubcc4\ud788 \uc794\uc778\ud55c \uc131\ud5a5\uc744 \uc9c0\ub154\uac70\ub098 \uad6d\uac00 \uc0ac\ud68c\uc8fc\uc758\uc5d0 \uacfc\ub3c4\ud55c \ucda9\uc131\uc744 \uc99d\uba85\ud574\ubcf4\uc774\ub824\ub294 \uac1c\uc778\ub4e4\uc758 \ub3cc\ucd9c\ud589\ub3d9 \uc815\ub3c4\ub85c \uc5ec\uacbc\ub2e4. \ub9c8\uc9c0\ub9c9\uc73c\ub85c Buchheim\uc740 \uacb0\ubc31\ud55c \uc0ac\ub78c\ub4e4 \uc911 \uadf8\ub7f0 \uae38\ub85c \ube60\uc838\ubc84\ub9b0 \uc774\ub4e4 \uc5ed\uc2dc \uc18c\uc18d\uac10\uc744 \uc720\uc9c0\ud558\uae30\uc704\ud574, \ub610\ub294 \uba85\ub839\uc744 \uac70\ubd80\ud560 \uacbd\uc6b0 \ub3d9\ub8cc\ub85c\ubd80\ud130 \u2018\uc57d\uc790\u2019\ub85c \ucde8\uae09\ubc1b\uc9c0 \uc54a\uae30 \uc704\ud574 \uadf8\ub7ec\ud55c \uc120\ud0dd\uc744 \ud55c \uac83\uc77c \ubfd0 \ud53c\ud560 \uc218 \uc5c6\ub294 \uba85\ub839\uc774 \uc544\ub2c8\uc5c8\ub2e4\uace0 \uc8fc\uc7a5\ud55c\ub2e4.", "paragraph_answer": "\ub3c5\uc77c\uc758 \uc5ed\uc0ac\ud559\uc790 Hans Buchheim\uc774 \ud504\ub791\ud06c\ud478\ub974\ud2b8\uc758 \ub258\ub978\ubca0\ub974\ud06c \uc7ac\ud310\uc5d0\uc11c \uae30\uc18c \uce21 \uc804\ubb38 \uac10\uc815\uc778\uc73c\ub85c\uc11c \ud65c\ub3d9\ud588\ub358 \uacbd\ud5d8\uc744 \ub2f4\uc544 1965\ub144\uc5d0 \ubc1c\uac04\ud55c \uc5d0\uc138\uc774 \u201cCommand and Compliance\u201d\uc5d0 \ub530\ub974\uba74, \uc544\uc6b0\uc288\ube44\uce20 \ub0b4\uc5d0\uc11c \uc720\ub300\uc778\uc744 \ube44\ub86f\ud55c \uc218\uac10\uc790\ub4e4\uc744 \uc0b4\ud574\ud558\ub294\ub370 \uc788\uc5b4 \uac00\ud574\uc790\ub4e4\uc758 \uc790\uc720\uc758\uc9c0\ub97c \uce68\ubc94\ud558\ub294 \uc5b4\ub5a0\ud55c \uac15\uc555\uc801\uc778 \uba85\ub839\ub3c4 \uc5c6\uc5c8\ub2e4\uace0 \ud55c\ub2e4. Buchheim\uc740 \ubc94\uc8c4\ub098 \ub2e4\ub984\uc5c6\ub294 \uadf8 \uba85\ub839\ub4e4\uc740 \ucda9\ubd84\ud788 \ud68c\ud53c\ud560 \uae30\ud68c\uac00 \uc788\uc5c8\ub2e4\uace0 \uc800\uc220\ud558\uba70, \uc774\ub7ec\ud55c \uba85\ub839\uc744 \uc774\ud589\ud558\uc9c0 \uc54a\uc740 SS\uce5c\uc704\ub300\uc6d0\ub4e4\uc774 \uac15\uc81c\uc218\uc6a9\uc18c\uc5d0 \uc218\uac10\ub418\uac70\ub098 \ucc98\ud615\ub2f9\ud588\ub2e4\ub294 \uc5b4\ub5a0\ud55c \uc99d\uac70\ub3c4 \ucc3e\uc744 \uc218 \uc5c6\uc5c8\ub2e4\uace0 \ud55c\ub2e4. \uac8c\ub2e4\uac00 Heinrich Himmler \uac00 \uadf8\uc758 \ub300\uc6d0\ub4e4\uc5d0\uac8c \u2018\ubaa8\ubc94\uc801\uc774\uace0 \uc608\uc758\ubc14\ub978\u2019 \ubaa8\uc2b5\uc744 \uac16\ucd94\uae38 \uac15\uc870\ud55c \ub9cc\ud07c SS\uce5c\uc704\ub300\ub294 \ubd88\ud544\uc694\ud55c \uac00\ud559\ud589\uc704\ub294 \uc624\ud788\ub824 \uc9c0\uc591\ud558\uace0 \uc788\uc5c8\uc73c\uba70, \uac00\ud559\ud589\uc704\ub294 \uadf8\ub4e4 \uc911 \ud2b9\ubcc4\ud788 \uc794\uc778\ud55c \uc131\ud5a5\uc744 \uc9c0\ub154\uac70\ub098 \uad6d\uac00 \uc0ac\ud68c\uc8fc\uc758\uc5d0 \uacfc\ub3c4\ud55c \ucda9\uc131\uc744 \uc99d\uba85\ud574\ubcf4\uc774\ub824\ub294 \uac1c\uc778\ub4e4\uc758 \ub3cc\ucd9c\ud589\ub3d9 \uc815\ub3c4\ub85c \uc5ec\uacbc\ub2e4. \ub9c8\uc9c0\ub9c9\uc73c\ub85c Buchheim\uc740 \uacb0\ubc31\ud55c \uc0ac\ub78c\ub4e4 \uc911 \uadf8\ub7f0 \uae38\ub85c \ube60\uc838\ubc84\ub9b0 \uc774\ub4e4 \uc5ed\uc2dc \uc18c\uc18d\uac10\uc744 \uc720\uc9c0\ud558\uae30\uc704\ud574, \ub610\ub294 \uba85\ub839\uc744 \uac70\ubd80\ud560 \uacbd\uc6b0 \ub3d9\ub8cc\ub85c\ubd80\ud130 \u2018\uc57d\uc790\u2019\ub85c \ucde8\uae09\ubc1b\uc9c0 \uc54a\uae30 \uc704\ud574 \uadf8\ub7ec\ud55c \uc120\ud0dd\uc744 \ud55c \uac83\uc77c \ubfd0 \ud53c\ud560 \uc218 \uc5c6\ub294 \uba85\ub839\uc774 \uc544\ub2c8\uc5c8\ub2e4\uace0 \uc8fc\uc7a5\ud55c\ub2e4.", "sentence_answer": "\uac8c\ub2e4\uac00 Heinrich Himmler \uac00 \uadf8\uc758 \ub300\uc6d0\ub4e4\uc5d0\uac8c \u2018\ubaa8\ubc94\uc801\uc774\uace0 \uc608\uc758\ubc14\ub978\u2019 \ubaa8\uc2b5\uc744 \uac16\ucd94\uae38 \uac15\uc870\ud55c \ub9cc\ud07c SS\uce5c\uc704\ub300\ub294 \ubd88\ud544\uc694\ud55c \uac00\ud559\ud589\uc704\ub294 \uc624\ud788\ub824 \uc9c0\uc591\ud558\uace0 \uc788\uc5c8\uc73c\uba70, \uac00\ud559\ud589\uc704\ub294 \uadf8\ub4e4 \uc911 \ud2b9\ubcc4\ud788 \uc794\uc778\ud55c \uc131\ud5a5\uc744 \uc9c0\ub154\uac70\ub098 \uad6d\uac00 \uc0ac\ud68c\uc8fc\uc758\uc5d0 \uacfc\ub3c4\ud55c \ucda9\uc131\uc744 \uc99d\uba85\ud574\ubcf4\uc774\ub824\ub294 \uac1c\uc778\ub4e4\uc758 \ub3cc\ucd9c\ud589\ub3d9 \uc815\ub3c4\ub85c \uc5ec\uacbc\ub2e4.", "paragraph_id": "6571961-18-2", "paragraph_question": "question: \ub300\uc6d0\ub4e4\uc5d0\uac8c \ubaa8\ubc94\uc801\uc774\uace0 \uc608\uc758\ubc14\ub978 \ubaa8\uc2b5\uc744 \uac16\ucd94\uae38 \uac15\uc870\ud55c \uac83\uc740 \ub204\uad6c\uc778\uac00?, context: \ub3c5\uc77c\uc758 \uc5ed\uc0ac\ud559\uc790 Hans Buchheim\uc774 \ud504\ub791\ud06c\ud478\ub974\ud2b8\uc758 \ub258\ub978\ubca0\ub974\ud06c \uc7ac\ud310\uc5d0\uc11c \uae30\uc18c \uce21 \uc804\ubb38 \uac10\uc815\uc778\uc73c\ub85c\uc11c \ud65c\ub3d9\ud588\ub358 \uacbd\ud5d8\uc744 \ub2f4\uc544 1965\ub144\uc5d0 \ubc1c\uac04\ud55c \uc5d0\uc138\uc774 \u201cCommand and Compliance\u201d\uc5d0 \ub530\ub974\uba74, \uc544\uc6b0\uc288\ube44\uce20 \ub0b4\uc5d0\uc11c \uc720\ub300\uc778\uc744 \ube44\ub86f\ud55c \uc218\uac10\uc790\ub4e4\uc744 \uc0b4\ud574\ud558\ub294\ub370 \uc788\uc5b4 \uac00\ud574\uc790\ub4e4\uc758 \uc790\uc720\uc758\uc9c0\ub97c \uce68\ubc94\ud558\ub294 \uc5b4\ub5a0\ud55c \uac15\uc555\uc801\uc778 \uba85\ub839\ub3c4 \uc5c6\uc5c8\ub2e4\uace0 \ud55c\ub2e4. Buchheim\uc740 \ubc94\uc8c4\ub098 \ub2e4\ub984\uc5c6\ub294 \uadf8 \uba85\ub839\ub4e4\uc740 \ucda9\ubd84\ud788 \ud68c\ud53c\ud560 \uae30\ud68c\uac00 \uc788\uc5c8\ub2e4\uace0 \uc800\uc220\ud558\uba70, \uc774\ub7ec\ud55c \uba85\ub839\uc744 \uc774\ud589\ud558\uc9c0 \uc54a\uc740 SS\uce5c\uc704\ub300\uc6d0\ub4e4\uc774 \uac15\uc81c\uc218\uc6a9\uc18c\uc5d0 \uc218\uac10\ub418\uac70\ub098 \ucc98\ud615\ub2f9\ud588\ub2e4\ub294 \uc5b4\ub5a0\ud55c \uc99d\uac70\ub3c4 \ucc3e\uc744 \uc218 \uc5c6\uc5c8\ub2e4\uace0 \ud55c\ub2e4. \uac8c\ub2e4\uac00 Heinrich Himmler\uac00 \uadf8\uc758 \ub300\uc6d0\ub4e4\uc5d0\uac8c \u2018\ubaa8\ubc94\uc801\uc774\uace0 \uc608\uc758\ubc14\ub978\u2019 \ubaa8\uc2b5\uc744 \uac16\ucd94\uae38 \uac15\uc870\ud55c \ub9cc\ud07c SS\uce5c\uc704\ub300\ub294 \ubd88\ud544\uc694\ud55c \uac00\ud559\ud589\uc704\ub294 \uc624\ud788\ub824 \uc9c0\uc591\ud558\uace0 \uc788\uc5c8\uc73c\uba70, \uac00\ud559\ud589\uc704\ub294 \uadf8\ub4e4 \uc911 \ud2b9\ubcc4\ud788 \uc794\uc778\ud55c \uc131\ud5a5\uc744 \uc9c0\ub154\uac70\ub098 \uad6d\uac00 \uc0ac\ud68c\uc8fc\uc758\uc5d0 \uacfc\ub3c4\ud55c \ucda9\uc131\uc744 \uc99d\uba85\ud574\ubcf4\uc774\ub824\ub294 \uac1c\uc778\ub4e4\uc758 \ub3cc\ucd9c\ud589\ub3d9 \uc815\ub3c4\ub85c \uc5ec\uacbc\ub2e4. \ub9c8\uc9c0\ub9c9\uc73c\ub85c Buchheim\uc740 \uacb0\ubc31\ud55c \uc0ac\ub78c\ub4e4 \uc911 \uadf8\ub7f0 \uae38\ub85c \ube60\uc838\ubc84\ub9b0 \uc774\ub4e4 \uc5ed\uc2dc \uc18c\uc18d\uac10\uc744 \uc720\uc9c0\ud558\uae30\uc704\ud574, \ub610\ub294 \uba85\ub839\uc744 \uac70\ubd80\ud560 \uacbd\uc6b0 \ub3d9\ub8cc\ub85c\ubd80\ud130 \u2018\uc57d\uc790\u2019\ub85c \ucde8\uae09\ubc1b\uc9c0 \uc54a\uae30 \uc704\ud574 \uadf8\ub7ec\ud55c \uc120\ud0dd\uc744 \ud55c \uac83\uc77c \ubfd0 \ud53c\ud560 \uc218 \uc5c6\ub294 \uba85\ub839\uc774 \uc544\ub2c8\uc5c8\ub2e4\uace0 \uc8fc\uc7a5\ud55c\ub2e4."} {"question": "\ube0c\ub9ac\ud2b8\ub2c8 \uc2a4\ud53c\uc5b4\uc2a4\uc758 \uc5ec\uc12f \ubc88\uc9f8 \uc815\uaddc \uc568\ubc94\uc758 \uc81c\ubaa9\uc740 \ubb34\uc5c7\uc778\uac00\uc694?", "paragraph": "2008\ub144 12\uc6d4 \uc2a4\ud53c\uc5b4\uc2a4\uc758 \uc5ec\uc12f \ubc88\uc9f8 \uc815\uaddc \uc568\ubc94 Circus\uac00 \ubc1c\ub9e4\ub418\uc5c8\ub2e4. \uc568\ubc94\uc740 \ud3c9\ub860\uac00\ub4e4\ub85c\ubd80\ud130 \ub300\ubd80\ubd84 \uc88b\uc740 \ud3c9\uac00\ub97c \ubc1b\uc558\ub294\ub370, \uba54\ud0c0\ud06c\ub9ac\ud2f1\uc5d0 \ub530\ub974\uba74 \ud3c9\uade0 100\uc810 \ub9cc\uc810 \uc911 64\uc810\uc744 \ubc1b\uc558\ub2e4\uace0 \ud55c\ub2e4. Circus\ub294 \ubbf8\uad6d, \uce90\ub098\ub2e4, \uccb4\ucf54\uc5d0\uc11c 1\uc704\ub85c \ub370\ubdd4\ud588\uc73c\uba70, \uc774 \uc678\uc5d0\ub3c4 \ub9ce\uc740 \uc720\ub7fd \uad6d\uac00\uc5d0\uc11c 10\uc704\uad8c \uc548\uc73c\ub85c \ub370\ubdd4\ud588\ub2e4. \ubbf8\uad6d\uc5d0\uc11c\ub294 \ub370\ubdd4 \uc568\ubc94\ubd80\ud130 \ub2e4\uc12f \ubc88\uc9f8 \uc568\ubc94\uae4c\uc9c0 1\uc704\ub85c \ub370\ubdd4\uc2dc\ud0a8 \uac00\uc7a5 \uc5b4\ub9b0 \uc5ec\uc131 \uc74c\uc545\uac00\ub77c\ub294 \uae30\ub85d\uc744 \uc138\uc6b0\uba70 \uae30\ub124\uc2a4 \uc138\uacc4 \uae30\ub85d\uc5d0 \uc62c\ub790\ub2e4. \ub610\ud55c \uc74c\ubc18 \ud310\ub9e4\ub7c9\uc744 \uc9d1\uacc4\ud558\ub294 \ub2d0\uc2a8 \uc0ac\uc6b4\ub4dc\uc2a4\uce94\uc5d0\uc11c \uc720\uc77c\ud558\uac8c \ub124 \uc7a5 \uc5f0\uc18d\uc73c\ub85c 50\ub9cc \uc7a5 \uc774\uc0c1\uc758 \ud310\ub9e4\ub7c9\uc744 \uae30\ub85d\ud558\uba70 \ub370\ubdd4\uc2dc\ucf30\ub2e4. Circus\ub294 \uadf8 \ud574 \uac00\uc7a5 \ube68\ub9ac \ud314\ub9b0 \uc74c\ubc18 \uc911 \ud558\ub098\uc600\uc73c\uba70, \uc138\uacc4\uc801\uc73c\ub85c 400\ub9cc \uc7a5 \uc774\uc0c1\uc774 \ud314\ub838\ub2e4. \uc74c\ubc18\uc758 \uccab \uc2f1\uae00 \"Womanizer\"\ub294 \"...Baby One More Time\" \uc774\ud6c4 9\ub144 \ub9cc\uc5d0 \ube4c\ubcf4\ub4dc \ud56b 100 1\uc704\ub97c \ucc28\uc9c0\ud588\ub2e4. \uc774\uc678\uc5d0 \ubca8\uae30\uc5d0, \uce90\ub098\ub2e4, \ub374\ub9c8\ud06c, \ud540\ub780\ub4dc, \ud504\ub791\uc2a4, \ub178\ub974\uc6e8\uc774\uc640 \uc2a4\uc6e8\ub374 \ub4f1 \ub9ce\uc740 \uad6d\uac00\uc5d0\uc11c \uc815\uc0c1\uc744 \ucc28\uc9c0\ud588\ub2e4. \uc774 \ub178\ub798\ub85c \uc81c52\ud68c \uadf8\ub798\ubbf8\uc0c1 \ucd5c\uc6b0\uc218 \ub304\uc2a4 \ub808\ucf54\ub529 \ubd80\ubb38\uc5d0 \ud6c4\ubcf4\ub85c \uc624\ub974\uae30\ub3c4 \ud588\ub2e4. \uc568\ubc94\uacfc \ub3d9\uba85\uc758 \ub450 \ubc88\uc9f8 \uc2f1\uae00 \"Circus\"\ub294 \ube4c\ubcf4\ub4dc \ud56b 100 \ucd5c\uace0 3\uc704\uae4c\uc9c0 \uc62c\ub790\ub2e4. 2009\ub144 1\uc6d4 \uc2a4\ud53c\uc5b4\uc2a4\uc640 \uc81c\uc784\uc2a4 \uc2a4\ud53c\uc5b4\uc2a4\ub294 \ub178\ub825\ub05d\uc5d0 \uc774\uc804 \ub9e4\ub2c8\uc800 \uc0d8 \ub8e8\ud2b8\ud53c, \uc804 \ub0a8\uc790 \uce5c\uad6c \uc544\ub4dc\ub09c \ucc30\ub9bd, \ubcc0\ud638\uc0ac \uc874 \uc774\uc5b4\ub4e4\ub9ac\ub85c\ubd80\ud130 \uc811\uadfc \uae08\uc9c0\ub77c\ub294 \ubc95\uc6d0\uc758 \ud310\uacb0\uc744 \uc5bb\uc5b4\ub0c8\ub2e4. 2009\ub144 3\uc6d4\ubd80\ud130\ub294 The Circus Starring Britney Spears\ub77c\ub294 \uc81c\ubaa9\uc758 \ud22c\uc5b4\ub97c \uc2dc\uc791\ud588\ub2e4. \ubbf8\uad6d\uc5d0\uc11c\ub9cc 1\uc5b5 3,800\ub9cc \ub2ec\ub7ec\ub97c \ubc8c\uc5b4\ub4e4\uc600\ub294\ub370, \uadf8 \ud574 \ubbf8\uad6d\uc5d0\uc11c \ub2e4\uc12f \ubc88\uc9f8\ub85c \ub192\uc740 \uc218\uc775\ub960\uc744 \uc62c\ub9b0 \ud22c\uc5b4\uc600\ub2e4. \ud638\uc8fc\uc5d0\uc11c\ub294 4\uc77c\uac04 \ucd1d 6\ub9cc \uc7a5\uc758 \ud2f0\ucf13\uc744 \ud314\uc544 \uc2e0\uae30\ub85d\uc744 \uc218\ub9bd\ud588\ub2e4.", "answer": "Circus", "sentence": "2008\ub144 12\uc6d4 \uc2a4\ud53c\uc5b4\uc2a4\uc758 \uc5ec\uc12f \ubc88\uc9f8 \uc815\uaddc \uc568\ubc94 Circus \uac00 \ubc1c\ub9e4\ub418\uc5c8\ub2e4.", "paragraph_sentence": " 2008\ub144 12\uc6d4 \uc2a4\ud53c\uc5b4\uc2a4\uc758 \uc5ec\uc12f \ubc88\uc9f8 \uc815\uaddc \uc568\ubc94 Circus \uac00 \ubc1c\ub9e4\ub418\uc5c8\ub2e4. \uc568\ubc94\uc740 \ud3c9\ub860\uac00\ub4e4\ub85c\ubd80\ud130 \ub300\ubd80\ubd84 \uc88b\uc740 \ud3c9\uac00\ub97c \ubc1b\uc558\ub294\ub370, \uba54\ud0c0\ud06c\ub9ac\ud2f1\uc5d0 \ub530\ub974\uba74 \ud3c9\uade0 100\uc810 \ub9cc\uc810 \uc911 64\uc810\uc744 \ubc1b\uc558\ub2e4\uace0 \ud55c\ub2e4. Circus\ub294 \ubbf8\uad6d, \uce90\ub098\ub2e4, \uccb4\ucf54\uc5d0\uc11c 1\uc704\ub85c \ub370\ubdd4\ud588\uc73c\uba70, \uc774 \uc678\uc5d0\ub3c4 \ub9ce\uc740 \uc720\ub7fd \uad6d\uac00\uc5d0\uc11c 10\uc704\uad8c \uc548\uc73c\ub85c \ub370\ubdd4\ud588\ub2e4. \ubbf8\uad6d\uc5d0\uc11c\ub294 \ub370\ubdd4 \uc568\ubc94\ubd80\ud130 \ub2e4\uc12f \ubc88\uc9f8 \uc568\ubc94\uae4c\uc9c0 1\uc704\ub85c \ub370\ubdd4\uc2dc\ud0a8 \uac00\uc7a5 \uc5b4\ub9b0 \uc5ec\uc131 \uc74c\uc545\uac00\ub77c\ub294 \uae30\ub85d\uc744 \uc138\uc6b0\uba70 \uae30\ub124\uc2a4 \uc138\uacc4 \uae30\ub85d\uc5d0 \uc62c\ub790\ub2e4. \ub610\ud55c \uc74c\ubc18 \ud310\ub9e4\ub7c9\uc744 \uc9d1\uacc4\ud558\ub294 \ub2d0\uc2a8 \uc0ac\uc6b4\ub4dc\uc2a4\uce94\uc5d0\uc11c \uc720\uc77c\ud558\uac8c \ub124 \uc7a5 \uc5f0\uc18d\uc73c\ub85c 50\ub9cc \uc7a5 \uc774\uc0c1\uc758 \ud310\ub9e4\ub7c9\uc744 \uae30\ub85d\ud558\uba70 \ub370\ubdd4\uc2dc\ucf30\ub2e4. Circus\ub294 \uadf8 \ud574 \uac00\uc7a5 \ube68\ub9ac \ud314\ub9b0 \uc74c\ubc18 \uc911 \ud558\ub098\uc600\uc73c\uba70, \uc138\uacc4\uc801\uc73c\ub85c 400\ub9cc \uc7a5 \uc774\uc0c1\uc774 \ud314\ub838\ub2e4. \uc74c\ubc18\uc758 \uccab \uc2f1\uae00 \"Womanizer\"\ub294 \"...Baby One More Time\" \uc774\ud6c4 9\ub144 \ub9cc\uc5d0 \ube4c\ubcf4\ub4dc \ud56b 100 1\uc704\ub97c \ucc28\uc9c0\ud588\ub2e4. \uc774\uc678\uc5d0 \ubca8\uae30\uc5d0, \uce90\ub098\ub2e4, \ub374\ub9c8\ud06c, \ud540\ub780\ub4dc, \ud504\ub791\uc2a4, \ub178\ub974\uc6e8\uc774\uc640 \uc2a4\uc6e8\ub374 \ub4f1 \ub9ce\uc740 \uad6d\uac00\uc5d0\uc11c \uc815\uc0c1\uc744 \ucc28\uc9c0\ud588\ub2e4. \uc774 \ub178\ub798\ub85c \uc81c52\ud68c \uadf8\ub798\ubbf8\uc0c1 \ucd5c\uc6b0\uc218 \ub304\uc2a4 \ub808\ucf54\ub529 \ubd80\ubb38\uc5d0 \ud6c4\ubcf4\ub85c \uc624\ub974\uae30\ub3c4 \ud588\ub2e4. \uc568\ubc94\uacfc \ub3d9\uba85\uc758 \ub450 \ubc88\uc9f8 \uc2f1\uae00 \"Circus\"\ub294 \ube4c\ubcf4\ub4dc \ud56b 100 \ucd5c\uace0 3\uc704\uae4c\uc9c0 \uc62c\ub790\ub2e4. 2009\ub144 1\uc6d4 \uc2a4\ud53c\uc5b4\uc2a4\uc640 \uc81c\uc784\uc2a4 \uc2a4\ud53c\uc5b4\uc2a4\ub294 \ub178\ub825\ub05d\uc5d0 \uc774\uc804 \ub9e4\ub2c8\uc800 \uc0d8 \ub8e8\ud2b8\ud53c, \uc804 \ub0a8\uc790 \uce5c\uad6c \uc544\ub4dc\ub09c \ucc30\ub9bd, \ubcc0\ud638\uc0ac \uc874 \uc774\uc5b4\ub4e4\ub9ac\ub85c\ubd80\ud130 \uc811\uadfc \uae08\uc9c0\ub77c\ub294 \ubc95\uc6d0\uc758 \ud310\uacb0\uc744 \uc5bb\uc5b4\ub0c8\ub2e4. 2009\ub144 3\uc6d4\ubd80\ud130\ub294 The Circus Starring Britney Spears\ub77c\ub294 \uc81c\ubaa9\uc758 \ud22c\uc5b4\ub97c \uc2dc\uc791\ud588\ub2e4. \ubbf8\uad6d\uc5d0\uc11c\ub9cc 1\uc5b5 3,800\ub9cc \ub2ec\ub7ec\ub97c \ubc8c\uc5b4\ub4e4\uc600\ub294\ub370, \uadf8 \ud574 \ubbf8\uad6d\uc5d0\uc11c \ub2e4\uc12f \ubc88\uc9f8\ub85c \ub192\uc740 \uc218\uc775\ub960\uc744 \uc62c\ub9b0 \ud22c\uc5b4\uc600\ub2e4. \ud638\uc8fc\uc5d0\uc11c\ub294 4\uc77c\uac04 \ucd1d 6\ub9cc \uc7a5\uc758 \ud2f0\ucf13\uc744 \ud314\uc544 \uc2e0\uae30\ub85d\uc744 \uc218\ub9bd\ud588\ub2e4.", "paragraph_answer": "2008\ub144 12\uc6d4 \uc2a4\ud53c\uc5b4\uc2a4\uc758 \uc5ec\uc12f \ubc88\uc9f8 \uc815\uaddc \uc568\ubc94 Circus \uac00 \ubc1c\ub9e4\ub418\uc5c8\ub2e4. \uc568\ubc94\uc740 \ud3c9\ub860\uac00\ub4e4\ub85c\ubd80\ud130 \ub300\ubd80\ubd84 \uc88b\uc740 \ud3c9\uac00\ub97c \ubc1b\uc558\ub294\ub370, \uba54\ud0c0\ud06c\ub9ac\ud2f1\uc5d0 \ub530\ub974\uba74 \ud3c9\uade0 100\uc810 \ub9cc\uc810 \uc911 64\uc810\uc744 \ubc1b\uc558\ub2e4\uace0 \ud55c\ub2e4. Circus\ub294 \ubbf8\uad6d, \uce90\ub098\ub2e4, \uccb4\ucf54\uc5d0\uc11c 1\uc704\ub85c \ub370\ubdd4\ud588\uc73c\uba70, \uc774 \uc678\uc5d0\ub3c4 \ub9ce\uc740 \uc720\ub7fd \uad6d\uac00\uc5d0\uc11c 10\uc704\uad8c \uc548\uc73c\ub85c \ub370\ubdd4\ud588\ub2e4. \ubbf8\uad6d\uc5d0\uc11c\ub294 \ub370\ubdd4 \uc568\ubc94\ubd80\ud130 \ub2e4\uc12f \ubc88\uc9f8 \uc568\ubc94\uae4c\uc9c0 1\uc704\ub85c \ub370\ubdd4\uc2dc\ud0a8 \uac00\uc7a5 \uc5b4\ub9b0 \uc5ec\uc131 \uc74c\uc545\uac00\ub77c\ub294 \uae30\ub85d\uc744 \uc138\uc6b0\uba70 \uae30\ub124\uc2a4 \uc138\uacc4 \uae30\ub85d\uc5d0 \uc62c\ub790\ub2e4. \ub610\ud55c \uc74c\ubc18 \ud310\ub9e4\ub7c9\uc744 \uc9d1\uacc4\ud558\ub294 \ub2d0\uc2a8 \uc0ac\uc6b4\ub4dc\uc2a4\uce94\uc5d0\uc11c \uc720\uc77c\ud558\uac8c \ub124 \uc7a5 \uc5f0\uc18d\uc73c\ub85c 50\ub9cc \uc7a5 \uc774\uc0c1\uc758 \ud310\ub9e4\ub7c9\uc744 \uae30\ub85d\ud558\uba70 \ub370\ubdd4\uc2dc\ucf30\ub2e4. Circus\ub294 \uadf8 \ud574 \uac00\uc7a5 \ube68\ub9ac \ud314\ub9b0 \uc74c\ubc18 \uc911 \ud558\ub098\uc600\uc73c\uba70, \uc138\uacc4\uc801\uc73c\ub85c 400\ub9cc \uc7a5 \uc774\uc0c1\uc774 \ud314\ub838\ub2e4. \uc74c\ubc18\uc758 \uccab \uc2f1\uae00 \"Womanizer\"\ub294 \"...Baby One More Time\" \uc774\ud6c4 9\ub144 \ub9cc\uc5d0 \ube4c\ubcf4\ub4dc \ud56b 100 1\uc704\ub97c \ucc28\uc9c0\ud588\ub2e4. \uc774\uc678\uc5d0 \ubca8\uae30\uc5d0, \uce90\ub098\ub2e4, \ub374\ub9c8\ud06c, \ud540\ub780\ub4dc, \ud504\ub791\uc2a4, \ub178\ub974\uc6e8\uc774\uc640 \uc2a4\uc6e8\ub374 \ub4f1 \ub9ce\uc740 \uad6d\uac00\uc5d0\uc11c \uc815\uc0c1\uc744 \ucc28\uc9c0\ud588\ub2e4. \uc774 \ub178\ub798\ub85c \uc81c52\ud68c \uadf8\ub798\ubbf8\uc0c1 \ucd5c\uc6b0\uc218 \ub304\uc2a4 \ub808\ucf54\ub529 \ubd80\ubb38\uc5d0 \ud6c4\ubcf4\ub85c \uc624\ub974\uae30\ub3c4 \ud588\ub2e4. \uc568\ubc94\uacfc \ub3d9\uba85\uc758 \ub450 \ubc88\uc9f8 \uc2f1\uae00 \"Circus\"\ub294 \ube4c\ubcf4\ub4dc \ud56b 100 \ucd5c\uace0 3\uc704\uae4c\uc9c0 \uc62c\ub790\ub2e4. 2009\ub144 1\uc6d4 \uc2a4\ud53c\uc5b4\uc2a4\uc640 \uc81c\uc784\uc2a4 \uc2a4\ud53c\uc5b4\uc2a4\ub294 \ub178\ub825\ub05d\uc5d0 \uc774\uc804 \ub9e4\ub2c8\uc800 \uc0d8 \ub8e8\ud2b8\ud53c, \uc804 \ub0a8\uc790 \uce5c\uad6c \uc544\ub4dc\ub09c \ucc30\ub9bd, \ubcc0\ud638\uc0ac \uc874 \uc774\uc5b4\ub4e4\ub9ac\ub85c\ubd80\ud130 \uc811\uadfc \uae08\uc9c0\ub77c\ub294 \ubc95\uc6d0\uc758 \ud310\uacb0\uc744 \uc5bb\uc5b4\ub0c8\ub2e4. 2009\ub144 3\uc6d4\ubd80\ud130\ub294 The Circus Starring Britney Spears\ub77c\ub294 \uc81c\ubaa9\uc758 \ud22c\uc5b4\ub97c \uc2dc\uc791\ud588\ub2e4. \ubbf8\uad6d\uc5d0\uc11c\ub9cc 1\uc5b5 3,800\ub9cc \ub2ec\ub7ec\ub97c \ubc8c\uc5b4\ub4e4\uc600\ub294\ub370, \uadf8 \ud574 \ubbf8\uad6d\uc5d0\uc11c \ub2e4\uc12f \ubc88\uc9f8\ub85c \ub192\uc740 \uc218\uc775\ub960\uc744 \uc62c\ub9b0 \ud22c\uc5b4\uc600\ub2e4. \ud638\uc8fc\uc5d0\uc11c\ub294 4\uc77c\uac04 \ucd1d 6\ub9cc \uc7a5\uc758 \ud2f0\ucf13\uc744 \ud314\uc544 \uc2e0\uae30\ub85d\uc744 \uc218\ub9bd\ud588\ub2e4.", "sentence_answer": "2008\ub144 12\uc6d4 \uc2a4\ud53c\uc5b4\uc2a4\uc758 \uc5ec\uc12f \ubc88\uc9f8 \uc815\uaddc \uc568\ubc94 Circus \uac00 \ubc1c\ub9e4\ub418\uc5c8\ub2e4.", "paragraph_id": "6509455-15-0", "paragraph_question": "question: \ube0c\ub9ac\ud2b8\ub2c8 \uc2a4\ud53c\uc5b4\uc2a4\uc758 \uc5ec\uc12f \ubc88\uc9f8 \uc815\uaddc \uc568\ubc94\uc758 \uc81c\ubaa9\uc740 \ubb34\uc5c7\uc778\uac00\uc694?, context: 2008\ub144 12\uc6d4 \uc2a4\ud53c\uc5b4\uc2a4\uc758 \uc5ec\uc12f \ubc88\uc9f8 \uc815\uaddc \uc568\ubc94 Circus\uac00 \ubc1c\ub9e4\ub418\uc5c8\ub2e4. \uc568\ubc94\uc740 \ud3c9\ub860\uac00\ub4e4\ub85c\ubd80\ud130 \ub300\ubd80\ubd84 \uc88b\uc740 \ud3c9\uac00\ub97c \ubc1b\uc558\ub294\ub370, \uba54\ud0c0\ud06c\ub9ac\ud2f1\uc5d0 \ub530\ub974\uba74 \ud3c9\uade0 100\uc810 \ub9cc\uc810 \uc911 64\uc810\uc744 \ubc1b\uc558\ub2e4\uace0 \ud55c\ub2e4. Circus\ub294 \ubbf8\uad6d, \uce90\ub098\ub2e4, \uccb4\ucf54\uc5d0\uc11c 1\uc704\ub85c \ub370\ubdd4\ud588\uc73c\uba70, \uc774 \uc678\uc5d0\ub3c4 \ub9ce\uc740 \uc720\ub7fd \uad6d\uac00\uc5d0\uc11c 10\uc704\uad8c \uc548\uc73c\ub85c \ub370\ubdd4\ud588\ub2e4. \ubbf8\uad6d\uc5d0\uc11c\ub294 \ub370\ubdd4 \uc568\ubc94\ubd80\ud130 \ub2e4\uc12f \ubc88\uc9f8 \uc568\ubc94\uae4c\uc9c0 1\uc704\ub85c \ub370\ubdd4\uc2dc\ud0a8 \uac00\uc7a5 \uc5b4\ub9b0 \uc5ec\uc131 \uc74c\uc545\uac00\ub77c\ub294 \uae30\ub85d\uc744 \uc138\uc6b0\uba70 \uae30\ub124\uc2a4 \uc138\uacc4 \uae30\ub85d\uc5d0 \uc62c\ub790\ub2e4. \ub610\ud55c \uc74c\ubc18 \ud310\ub9e4\ub7c9\uc744 \uc9d1\uacc4\ud558\ub294 \ub2d0\uc2a8 \uc0ac\uc6b4\ub4dc\uc2a4\uce94\uc5d0\uc11c \uc720\uc77c\ud558\uac8c \ub124 \uc7a5 \uc5f0\uc18d\uc73c\ub85c 50\ub9cc \uc7a5 \uc774\uc0c1\uc758 \ud310\ub9e4\ub7c9\uc744 \uae30\ub85d\ud558\uba70 \ub370\ubdd4\uc2dc\ucf30\ub2e4. Circus\ub294 \uadf8 \ud574 \uac00\uc7a5 \ube68\ub9ac \ud314\ub9b0 \uc74c\ubc18 \uc911 \ud558\ub098\uc600\uc73c\uba70, \uc138\uacc4\uc801\uc73c\ub85c 400\ub9cc \uc7a5 \uc774\uc0c1\uc774 \ud314\ub838\ub2e4. \uc74c\ubc18\uc758 \uccab \uc2f1\uae00 \"Womanizer\"\ub294 \"...Baby One More Time\" \uc774\ud6c4 9\ub144 \ub9cc\uc5d0 \ube4c\ubcf4\ub4dc \ud56b 100 1\uc704\ub97c \ucc28\uc9c0\ud588\ub2e4. \uc774\uc678\uc5d0 \ubca8\uae30\uc5d0, \uce90\ub098\ub2e4, \ub374\ub9c8\ud06c, \ud540\ub780\ub4dc, \ud504\ub791\uc2a4, \ub178\ub974\uc6e8\uc774\uc640 \uc2a4\uc6e8\ub374 \ub4f1 \ub9ce\uc740 \uad6d\uac00\uc5d0\uc11c \uc815\uc0c1\uc744 \ucc28\uc9c0\ud588\ub2e4. \uc774 \ub178\ub798\ub85c \uc81c52\ud68c \uadf8\ub798\ubbf8\uc0c1 \ucd5c\uc6b0\uc218 \ub304\uc2a4 \ub808\ucf54\ub529 \ubd80\ubb38\uc5d0 \ud6c4\ubcf4\ub85c \uc624\ub974\uae30\ub3c4 \ud588\ub2e4. \uc568\ubc94\uacfc \ub3d9\uba85\uc758 \ub450 \ubc88\uc9f8 \uc2f1\uae00 \"Circus\"\ub294 \ube4c\ubcf4\ub4dc \ud56b 100 \ucd5c\uace0 3\uc704\uae4c\uc9c0 \uc62c\ub790\ub2e4. 2009\ub144 1\uc6d4 \uc2a4\ud53c\uc5b4\uc2a4\uc640 \uc81c\uc784\uc2a4 \uc2a4\ud53c\uc5b4\uc2a4\ub294 \ub178\ub825\ub05d\uc5d0 \uc774\uc804 \ub9e4\ub2c8\uc800 \uc0d8 \ub8e8\ud2b8\ud53c, \uc804 \ub0a8\uc790 \uce5c\uad6c \uc544\ub4dc\ub09c \ucc30\ub9bd, \ubcc0\ud638\uc0ac \uc874 \uc774\uc5b4\ub4e4\ub9ac\ub85c\ubd80\ud130 \uc811\uadfc \uae08\uc9c0\ub77c\ub294 \ubc95\uc6d0\uc758 \ud310\uacb0\uc744 \uc5bb\uc5b4\ub0c8\ub2e4. 2009\ub144 3\uc6d4\ubd80\ud130\ub294 The Circus Starring Britney Spears\ub77c\ub294 \uc81c\ubaa9\uc758 \ud22c\uc5b4\ub97c \uc2dc\uc791\ud588\ub2e4. \ubbf8\uad6d\uc5d0\uc11c\ub9cc 1\uc5b5 3,800\ub9cc \ub2ec\ub7ec\ub97c \ubc8c\uc5b4\ub4e4\uc600\ub294\ub370, \uadf8 \ud574 \ubbf8\uad6d\uc5d0\uc11c \ub2e4\uc12f \ubc88\uc9f8\ub85c \ub192\uc740 \uc218\uc775\ub960\uc744 \uc62c\ub9b0 \ud22c\uc5b4\uc600\ub2e4. \ud638\uc8fc\uc5d0\uc11c\ub294 4\uc77c\uac04 \ucd1d 6\ub9cc \uc7a5\uc758 \ud2f0\ucf13\uc744 \ud314\uc544 \uc2e0\uae30\ub85d\uc744 \uc218\ub9bd\ud588\ub2e4."} {"question": "\uc624\ubc14\ub9c8\ub294 \uc77c\ub9ac\ub178\uc774 \uc0c1\uc6d0\uc758 \uc5b4\ub5a4 \uc704\uc6d0\ud68c \uc758\uc7a5\uc774 \ub418\uc5c8\ub098?", "paragraph": "2003\ub144 1\uc6d4, \uc2ed\uc5ec \ub144\uac04 \uc18c\uc218\ub2f9\uc774\uc5c8\ub358 \ubbfc\uc8fc\ub2f9\uc774 \ub2e4\uc2dc \ub2e4\uc218\ub2f9\uc758 \uc9c0\uc704\ub97c \ud68c\ubcf5\ud558\uba74\uc11c, \uc624\ubc14\ub9c8\ub294 \uc77c\ub9ac\ub178\uc774 \uc0c1\uc6d0\uc758 \uc758\ub8cc \ubc0f \uc778\uac04 \uc11c\ube44\uc2a4 \uc704\uc6d0\ud68c \uc758\uc7a5\uc774 \ub418\uc5c8\ub2e4. \uadf8\ub294 \uacbd\ucc30\uc774 \uad6c\uae08\ud55c \uc6b4\uc804\uc790\uc758 \uc778\uc885\uc744 \uae30\ub85d\ud558\ub3c4\ub85d \uaddc\uc815\ud558\uc5ec \uc778\uc885\ucc28\ubcc4\uc801 \uac80\ubb38 \uad00\ud589(racial profiling)\uc744 \uac10\uc2dc\ud558\ub294 \ubc95\uc548\uacfc \ub354\ubd88\uc5b4 (\ubbf8\uad6d\uc5d0\uc11c \ucd5c\ucd08\ub85c) \uc0b4\uc778 \uc0ac\uac74 \uc2ec\ubb38\uc744 \ubc18\ub4dc\uc2dc \ube44\ub514\uc624\ub85c \uae30\ub85d\ud558\ub3c4\ub85d \uaddc\uc815\ud558\ub294 \ubc95\uc548\uc744 \ubc1c\uc758\ud558\uc5ec \ub9cc\uc7a5\uc77c\uce58\uc758 \ucd08\ub2f9\uc801\uc778 \uc9c0\uc9c0\ub85c \ud1b5\uacfc\uc2dc\ucf30\ub2e4. 2004\ub144\uc5d0 \uc624\ubc14\ub9c8\uac00 \ubbf8\uad6d \uc0c1\uc6d0 \ucd1d\uc120\uc5d0 \ucd9c\ub9c8\ud574 \uc720\uc138\ud560 \ub2f9\uc2dc, \uacbd\ucc30 \ub300\ud45c\uc790\ub4e4\uc740 \uc624\ubc14\ub9c8\uac00 \uc0ac\ud615\uc81c \uac1c\ud601 \ubc95\ub839\ud654\ub97c \uc704\ud55c \uacbd\ucc30 \ub2e8\uccb4\uc5d0\uc11c \uadf8\uac00 \ud65c\ubc1c\ud558\uac8c \ud65c\ub3d9\ud588\ub2e4\uace0 \ucc2c\uc0ac\ub97c \ubcf4\ub0c8\ub2e4. 2004\ub144 11\uc6d4\uc5d0 \uc624\ubc14\ub9c8\ub294 \uc77c\ub9ac\ub178\uc774 \uc0c1\uc6d0 \uc758\uc6d0\uc9c1\uc5d0\uc11c \uc0ac\uc784\ud558\uace0, \ub2e4\uac00\uc624\ub294 \ubbf8\uad6d \uc0c1\uc6d0 \uc120\uac70\uc5d0 \ub3c4\uc804\ud558\uac8c \ub41c\ub2e4.", "answer": "\uc758\ub8cc \ubc0f \uc778\uac04 \uc11c\ube44\uc2a4 \uc704\uc6d0\ud68c", "sentence": "\uc624\ubc14\ub9c8\ub294 \uc77c\ub9ac\ub178\uc774 \uc0c1\uc6d0\uc758 \uc758\ub8cc \ubc0f \uc778\uac04 \uc11c\ube44\uc2a4 \uc704\uc6d0\ud68c \uc758\uc7a5\uc774 \ub418\uc5c8\ub2e4.", "paragraph_sentence": "2003\ub144 1\uc6d4, \uc2ed\uc5ec \ub144\uac04 \uc18c\uc218\ub2f9\uc774\uc5c8\ub358 \ubbfc\uc8fc\ub2f9\uc774 \ub2e4\uc2dc \ub2e4\uc218\ub2f9\uc758 \uc9c0\uc704\ub97c \ud68c\ubcf5\ud558\uba74\uc11c, \uc624\ubc14\ub9c8\ub294 \uc77c\ub9ac\ub178\uc774 \uc0c1\uc6d0\uc758 \uc758\ub8cc \ubc0f \uc778\uac04 \uc11c\ube44\uc2a4 \uc704\uc6d0\ud68c \uc758\uc7a5\uc774 \ub418\uc5c8\ub2e4. \uadf8\ub294 \uacbd\ucc30\uc774 \uad6c\uae08\ud55c \uc6b4\uc804\uc790\uc758 \uc778\uc885\uc744 \uae30\ub85d\ud558\ub3c4\ub85d \uaddc\uc815\ud558\uc5ec \uc778\uc885\ucc28\ubcc4\uc801 \uac80\ubb38 \uad00\ud589(racial profiling)\uc744 \uac10\uc2dc\ud558\ub294 \ubc95\uc548\uacfc \ub354\ubd88\uc5b4 (\ubbf8\uad6d\uc5d0\uc11c \ucd5c\ucd08\ub85c) \uc0b4\uc778 \uc0ac\uac74 \uc2ec\ubb38\uc744 \ubc18\ub4dc\uc2dc \ube44\ub514\uc624\ub85c \uae30\ub85d\ud558\ub3c4\ub85d \uaddc\uc815\ud558\ub294 \ubc95\uc548\uc744 \ubc1c\uc758\ud558\uc5ec \ub9cc\uc7a5\uc77c\uce58\uc758 \ucd08\ub2f9\uc801\uc778 \uc9c0\uc9c0\ub85c \ud1b5\uacfc\uc2dc\ucf30\ub2e4. 2004\ub144\uc5d0 \uc624\ubc14\ub9c8\uac00 \ubbf8\uad6d \uc0c1\uc6d0 \ucd1d\uc120\uc5d0 \ucd9c\ub9c8\ud574 \uc720\uc138\ud560 \ub2f9\uc2dc, \uacbd\ucc30 \ub300\ud45c\uc790\ub4e4\uc740 \uc624\ubc14\ub9c8\uac00 \uc0ac\ud615\uc81c \uac1c\ud601 \ubc95\ub839\ud654\ub97c \uc704\ud55c \uacbd\ucc30 \ub2e8\uccb4\uc5d0\uc11c \uadf8\uac00 \ud65c\ubc1c\ud558\uac8c \ud65c\ub3d9\ud588\ub2e4\uace0 \ucc2c\uc0ac\ub97c \ubcf4\ub0c8\ub2e4. 2004\ub144 11\uc6d4\uc5d0 \uc624\ubc14\ub9c8\ub294 \uc77c\ub9ac\ub178\uc774 \uc0c1\uc6d0 \uc758\uc6d0\uc9c1\uc5d0\uc11c \uc0ac\uc784\ud558\uace0, \ub2e4\uac00\uc624\ub294 \ubbf8\uad6d \uc0c1\uc6d0 \uc120\uac70\uc5d0 \ub3c4\uc804\ud558\uac8c \ub41c\ub2e4.", "paragraph_answer": "2003\ub144 1\uc6d4, \uc2ed\uc5ec \ub144\uac04 \uc18c\uc218\ub2f9\uc774\uc5c8\ub358 \ubbfc\uc8fc\ub2f9\uc774 \ub2e4\uc2dc \ub2e4\uc218\ub2f9\uc758 \uc9c0\uc704\ub97c \ud68c\ubcf5\ud558\uba74\uc11c, \uc624\ubc14\ub9c8\ub294 \uc77c\ub9ac\ub178\uc774 \uc0c1\uc6d0\uc758 \uc758\ub8cc \ubc0f \uc778\uac04 \uc11c\ube44\uc2a4 \uc704\uc6d0\ud68c \uc758\uc7a5\uc774 \ub418\uc5c8\ub2e4. \uadf8\ub294 \uacbd\ucc30\uc774 \uad6c\uae08\ud55c \uc6b4\uc804\uc790\uc758 \uc778\uc885\uc744 \uae30\ub85d\ud558\ub3c4\ub85d \uaddc\uc815\ud558\uc5ec \uc778\uc885\ucc28\ubcc4\uc801 \uac80\ubb38 \uad00\ud589(racial profiling)\uc744 \uac10\uc2dc\ud558\ub294 \ubc95\uc548\uacfc \ub354\ubd88\uc5b4 (\ubbf8\uad6d\uc5d0\uc11c \ucd5c\ucd08\ub85c) \uc0b4\uc778 \uc0ac\uac74 \uc2ec\ubb38\uc744 \ubc18\ub4dc\uc2dc \ube44\ub514\uc624\ub85c \uae30\ub85d\ud558\ub3c4\ub85d \uaddc\uc815\ud558\ub294 \ubc95\uc548\uc744 \ubc1c\uc758\ud558\uc5ec \ub9cc\uc7a5\uc77c\uce58\uc758 \ucd08\ub2f9\uc801\uc778 \uc9c0\uc9c0\ub85c \ud1b5\uacfc\uc2dc\ucf30\ub2e4. 2004\ub144\uc5d0 \uc624\ubc14\ub9c8\uac00 \ubbf8\uad6d \uc0c1\uc6d0 \ucd1d\uc120\uc5d0 \ucd9c\ub9c8\ud574 \uc720\uc138\ud560 \ub2f9\uc2dc, \uacbd\ucc30 \ub300\ud45c\uc790\ub4e4\uc740 \uc624\ubc14\ub9c8\uac00 \uc0ac\ud615\uc81c \uac1c\ud601 \ubc95\ub839\ud654\ub97c \uc704\ud55c \uacbd\ucc30 \ub2e8\uccb4\uc5d0\uc11c \uadf8\uac00 \ud65c\ubc1c\ud558\uac8c \ud65c\ub3d9\ud588\ub2e4\uace0 \ucc2c\uc0ac\ub97c \ubcf4\ub0c8\ub2e4. 2004\ub144 11\uc6d4\uc5d0 \uc624\ubc14\ub9c8\ub294 \uc77c\ub9ac\ub178\uc774 \uc0c1\uc6d0 \uc758\uc6d0\uc9c1\uc5d0\uc11c \uc0ac\uc784\ud558\uace0, \ub2e4\uac00\uc624\ub294 \ubbf8\uad6d \uc0c1\uc6d0 \uc120\uac70\uc5d0 \ub3c4\uc804\ud558\uac8c \ub41c\ub2e4.", "sentence_answer": "\uc624\ubc14\ub9c8\ub294 \uc77c\ub9ac\ub178\uc774 \uc0c1\uc6d0\uc758 \uc758\ub8cc \ubc0f \uc778\uac04 \uc11c\ube44\uc2a4 \uc704\uc6d0\ud68c \uc758\uc7a5\uc774 \ub418\uc5c8\ub2e4.", "paragraph_id": "6581697-7-1", "paragraph_question": "question: \uc624\ubc14\ub9c8\ub294 \uc77c\ub9ac\ub178\uc774 \uc0c1\uc6d0\uc758 \uc5b4\ub5a4 \uc704\uc6d0\ud68c \uc758\uc7a5\uc774 \ub418\uc5c8\ub098?, context: 2003\ub144 1\uc6d4, \uc2ed\uc5ec \ub144\uac04 \uc18c\uc218\ub2f9\uc774\uc5c8\ub358 \ubbfc\uc8fc\ub2f9\uc774 \ub2e4\uc2dc \ub2e4\uc218\ub2f9\uc758 \uc9c0\uc704\ub97c \ud68c\ubcf5\ud558\uba74\uc11c, \uc624\ubc14\ub9c8\ub294 \uc77c\ub9ac\ub178\uc774 \uc0c1\uc6d0\uc758 \uc758\ub8cc \ubc0f \uc778\uac04 \uc11c\ube44\uc2a4 \uc704\uc6d0\ud68c \uc758\uc7a5\uc774 \ub418\uc5c8\ub2e4. \uadf8\ub294 \uacbd\ucc30\uc774 \uad6c\uae08\ud55c \uc6b4\uc804\uc790\uc758 \uc778\uc885\uc744 \uae30\ub85d\ud558\ub3c4\ub85d \uaddc\uc815\ud558\uc5ec \uc778\uc885\ucc28\ubcc4\uc801 \uac80\ubb38 \uad00\ud589(racial profiling)\uc744 \uac10\uc2dc\ud558\ub294 \ubc95\uc548\uacfc \ub354\ubd88\uc5b4 (\ubbf8\uad6d\uc5d0\uc11c \ucd5c\ucd08\ub85c) \uc0b4\uc778 \uc0ac\uac74 \uc2ec\ubb38\uc744 \ubc18\ub4dc\uc2dc \ube44\ub514\uc624\ub85c \uae30\ub85d\ud558\ub3c4\ub85d \uaddc\uc815\ud558\ub294 \ubc95\uc548\uc744 \ubc1c\uc758\ud558\uc5ec \ub9cc\uc7a5\uc77c\uce58\uc758 \ucd08\ub2f9\uc801\uc778 \uc9c0\uc9c0\ub85c \ud1b5\uacfc\uc2dc\ucf30\ub2e4. 2004\ub144\uc5d0 \uc624\ubc14\ub9c8\uac00 \ubbf8\uad6d \uc0c1\uc6d0 \ucd1d\uc120\uc5d0 \ucd9c\ub9c8\ud574 \uc720\uc138\ud560 \ub2f9\uc2dc, \uacbd\ucc30 \ub300\ud45c\uc790\ub4e4\uc740 \uc624\ubc14\ub9c8\uac00 \uc0ac\ud615\uc81c \uac1c\ud601 \ubc95\ub839\ud654\ub97c \uc704\ud55c \uacbd\ucc30 \ub2e8\uccb4\uc5d0\uc11c \uadf8\uac00 \ud65c\ubc1c\ud558\uac8c \ud65c\ub3d9\ud588\ub2e4\uace0 \ucc2c\uc0ac\ub97c \ubcf4\ub0c8\ub2e4. 2004\ub144 11\uc6d4\uc5d0 \uc624\ubc14\ub9c8\ub294 \uc77c\ub9ac\ub178\uc774 \uc0c1\uc6d0 \uc758\uc6d0\uc9c1\uc5d0\uc11c \uc0ac\uc784\ud558\uace0, \ub2e4\uac00\uc624\ub294 \ubbf8\uad6d \uc0c1\uc6d0 \uc120\uac70\uc5d0 \ub3c4\uc804\ud558\uac8c \ub41c\ub2e4."} {"question": "\ub9c8\uc758 \ud0dc\uc790\uac00 \uae08\uac15\uc0b0\uc73c\ub85c \uac00\ub294 \uae38 \uc808\uc5d0 \ub4e4\ub7ec \uc9c0\ud321\uc774\ub97c \uaf42\ub294\ub370\uc11c \uc720\ub798\ud588\ub2e4\ub294 \uc740\ud589\ub098\ubb34\uac00 \uc788\ub294 \uacf3\uc740 \uc5b4\ub514\uc778\uac00?", "paragraph": "\uac15\uc6d0\ub3c4 \uc778\uc81c\uc640 \ub354\ubd88\uc5b4 \ub9c8\uc758 \ud0dc\uc790(\u9ebb\u8863 \u592a\u5b50)\uc640 \uad00\ub828\ub41c \uc804\uc124\uc774 \ub9ce\uc774 \ub0a8\uc544\uc788\ub294 \uacf3\uc740 \ucda9\uc8fc\uc2dc\ub85c, \ub9c8\uc758 \ud0dc\uc790(\u9ebb\u8863 \u592a\u5b50)\uc640 \uadf8 \ub204\uc774\ub3d9\uc0dd \ub355\uc8fc \uacf5\uc8fc(\u5fb7\u4e3b \u516c\u4e3b)\uac00 \uac01\uac01 \uc870\uc131\ud588\ub2e4\ub294 \uc6d4\uc545\uc0b0 \uc790\ub77d\uc758 \ubbf8\ub975\ub300\uc6d0 \ud130\uc640 \ub355\uc8fc\uc0ac(\u5fb7\u4f4f\u5bfa)\uac00 \uc788\ub2e4. \uc591\ud3c9\uad70\uc758 \uc6a9\ubb38\uc0ac\uc5d0\ub294 \ub9c8\uc758\ud0dc\uc790\uac00 \uae08\uac15\uc0b0\uc73c\ub85c \uac00\ub294 \uae38\uc5d0 \uc808\uc5d0 \ub4e4\ub7ec\uc11c \uc9da\uace0 \uc788\ub358 \uc9c0\ud321\uc774\ub97c \ub545\uc5d0 \uaf42\uc740 \ub370\uc11c \uc720\ub798\ud588\ub2e4\ub294 \uc740\ud589\ub098\ubb34\uac00 \uc788\uc73c\uba70, \ud64d\ucc9c\uc5d0\uc11c \ub9c8\uc758 \ud0dc\uc790 \uc804\uc2b9\uc774 \ub0a8\uc544\uc788\ub294 \uc778\uc81c\ub85c \ub118\uc5b4\uac00\ub294 \uae38\ubaa9\uc5d0 \uc9c0\uc655\ub3d9(\uc655\uc774 \uc9c0\ub098\uac04 \ub9c8\uc744)\uacfc \uc655\ud130(\uc655\uc774 \ub118\uc5b4\uac04 \uc790\ub9ac)\ub77c\ub294 \uc9c0\uba85\uc774 \ub0a8\uc544\uc788\ub2e4. \uc774\ub4e4 \ub9c8\uc758 \ud0dc\uc790 \uad00\ub828 \uc804\uc2b9 \uc9c0\uc5ed\uc744 \uc9c0\ub09c 2000\ub144 \ub2f5\uc0ac \ucde8\uc7ac\ud558\uace0 \ub9c8\uc758 \ud0dc\uc790\uc758 \ubd81\ud589(\u5317\u884c) \ub8e8\ud2b8\ub97c \ucd94\uc815\ud55c KBS \uc5ed\uc0ac\uc2a4\ud398\uc15c \uc81c\uc791\ud300\uc5d0\uc11c\ub294 2000\ub144 4\uc6d4 15\uc77c\uc5d0 \u300c\uc2e0\ub77c \ucd5c\ud6c4\uc758 \ubbf8\uc2a4\ud14c\ub9ac\u2015\ub9c8\uc758\ud0dc\uc790\u300d\ub77c\ub294 \ubd80\uc81c\ub85c \ubc29\uc601\ub418\uc5c8\ub358 \u300e\uc5ed\uc0ac\uc2a4\ud398\uc15c\u300f\ubc29\uc1a1\ubd84\uc5d0\uc11c \uae08\uac15\uc0b0\uc73c\ub85c \uac14\ub2e4\ub294 \ub9c8\uc758 \ud0dc\uc790\uc758 \uc804\uc2b9\uc774 \ub0a8\uc544\uc788\ub294 \uc9c0\uc5ed\uc774 \uad73\uc774 \uacbd\uc8fc\uc5d0\uc11c \uae08\uac15\uc0b0\uc73c\ub85c \uac00\uc7a5 \ube68\ub9ac \uac08 \uc218 \uc788\ub294 \ub3d9\ud574\uc548 \uad50\ud1b5\ub85c\uac00 \uc544\ub2cc \ub0b4\ub959\uc5d0 \ub0a8\uc544\uc788\ub294 \uac83\uc5d0\uc11c, \uacbd\uc8fc\uc5d0\uc11c \uacc4\ub9bd\ub839\uc73c\ub85c \uac00\uc11c \ucda9\uc8fc\ub97c \uc9c0\ub098 \ubb3c\uae38\ub85c \uc591\ud3c9\uc73c\ub85c, \ud64d\ucc9c\uc744 \uac70\uccd0 \uc778\uc81c\uc640 \ud55c\uacc4\ub839\uc744 \uc9c0\ub098\ub294 \ub9c8\uc758 \ud0dc\uc790\uc758 \ubd81\ud589 \ub8e8\ud2b8\ub97c \uc0c1\uc815\ud55c \ub4a4, \ud574\ub2f9 \ub3c4\uc2dc\ub4e4\uc774 \uc2e0\ub77c \uc2dc\ub300\uc5d0\ub294 \uc81c2\uc758 \uc218\ub3c4\ub85c \ubd88\ub838\ub2e4\ub294 \uc810(\ucda9\uc8fc\uc758 \uacbd\uc6b0), \uadf8\ub9ac\uace0 \uc678\ubd80 \uc138\ub825\uacfc \ucca0\uc800\ud558\uac8c \ucc28\ub2e8\ub41c \uacf3(\ubbf8\ub975\ub300\uc6d0 \ud130\ub098 \ub355\uc8fc\uc0ac\uc758 \uacbd\uc6b0)\uc774\ub77c\ub294 \uc810\uc744 \ub4e4\uc5b4 \ub9c8\uc758 \ud0dc\uc790\uac00 \uc2e0\ub77c\uc758 \uc8fc\uc694 \uac70\uc810 \uac00\uc6b4\ub370\uc11c\ub3c4 \ucc9c\ud61c\uc758 \uc694\uc0c8\ub4e4\ub9cc\uc744 \ud0dd\ud574 \uac70\uce58\uba74\uc11c \uc790\uc2e0\uacfc \ub73b\uc744 \ud568\uaed8\ud560 \uc0ac\ub78c\uc744 \uaddc\ud569\ud558\ub824 \ud588\ub358 \uac83\uc774\ub77c\ub294 \ucd94\ub860\uc744 \uc81c\uae30\ud588\ub2e4.", "answer": "\uc6a9\ubb38\uc0ac", "sentence": "\uc591\ud3c9\uad70\uc758 \uc6a9\ubb38\uc0ac \uc5d0\ub294 \ub9c8\uc758\ud0dc\uc790\uac00 \uae08\uac15\uc0b0\uc73c\ub85c \uac00\ub294 \uae38\uc5d0 \uc808\uc5d0 \ub4e4\ub7ec\uc11c \uc9da\uace0 \uc788\ub358 \uc9c0\ud321\uc774\ub97c \ub545\uc5d0 \uaf42\uc740 \ub370\uc11c \uc720\ub798\ud588\ub2e4\ub294 \uc740\ud589\ub098\ubb34\uac00 \uc788\uc73c\uba70, \ud64d\ucc9c\uc5d0\uc11c \ub9c8\uc758 \ud0dc\uc790 \uc804\uc2b9\uc774 \ub0a8\uc544\uc788\ub294 \uc778\uc81c\ub85c \ub118\uc5b4\uac00\ub294 \uae38\ubaa9\uc5d0 \uc9c0\uc655\ub3d9(\uc655\uc774 \uc9c0\ub098\uac04 \ub9c8\uc744)\uacfc \uc655\ud130(\uc655\uc774 \ub118\uc5b4\uac04 \uc790\ub9ac)\ub77c\ub294 \uc9c0\uba85\uc774 \ub0a8\uc544\uc788\ub2e4.", "paragraph_sentence": "\uac15\uc6d0\ub3c4 \uc778\uc81c\uc640 \ub354\ubd88\uc5b4 \ub9c8\uc758 \ud0dc\uc790(\u9ebb\u8863 \u592a\u5b50)\uc640 \uad00\ub828\ub41c \uc804\uc124\uc774 \ub9ce\uc774 \ub0a8\uc544\uc788\ub294 \uacf3\uc740 \ucda9\uc8fc\uc2dc\ub85c, \ub9c8\uc758 \ud0dc\uc790(\u9ebb\u8863 \u592a\u5b50)\uc640 \uadf8 \ub204\uc774\ub3d9\uc0dd \ub355\uc8fc \uacf5\uc8fc(\u5fb7\u4e3b \u516c\u4e3b)\uac00 \uac01\uac01 \uc870\uc131\ud588\ub2e4\ub294 \uc6d4\uc545\uc0b0 \uc790\ub77d\uc758 \ubbf8\ub975\ub300\uc6d0 \ud130\uc640 \ub355\uc8fc\uc0ac(\u5fb7\u4f4f\u5bfa)\uac00 \uc788\ub2e4. \uc591\ud3c9\uad70\uc758 \uc6a9\ubb38\uc0ac \uc5d0\ub294 \ub9c8\uc758\ud0dc\uc790\uac00 \uae08\uac15\uc0b0\uc73c\ub85c \uac00\ub294 \uae38\uc5d0 \uc808\uc5d0 \ub4e4\ub7ec\uc11c \uc9da\uace0 \uc788\ub358 \uc9c0\ud321\uc774\ub97c \ub545\uc5d0 \uaf42\uc740 \ub370\uc11c \uc720\ub798\ud588\ub2e4\ub294 \uc740\ud589\ub098\ubb34\uac00 \uc788\uc73c\uba70, \ud64d\ucc9c\uc5d0\uc11c \ub9c8\uc758 \ud0dc\uc790 \uc804\uc2b9\uc774 \ub0a8\uc544\uc788\ub294 \uc778\uc81c\ub85c \ub118\uc5b4\uac00\ub294 \uae38\ubaa9\uc5d0 \uc9c0\uc655\ub3d9(\uc655\uc774 \uc9c0\ub098\uac04 \ub9c8\uc744)\uacfc \uc655\ud130(\uc655\uc774 \ub118\uc5b4\uac04 \uc790\ub9ac)\ub77c\ub294 \uc9c0\uba85\uc774 \ub0a8\uc544\uc788\ub2e4. \uc774\ub4e4 \ub9c8\uc758 \ud0dc\uc790 \uad00\ub828 \uc804\uc2b9 \uc9c0\uc5ed\uc744 \uc9c0\ub09c 2000\ub144 \ub2f5\uc0ac \ucde8\uc7ac\ud558\uace0 \ub9c8\uc758 \ud0dc\uc790\uc758 \ubd81\ud589(\u5317\u884c) \ub8e8\ud2b8\ub97c \ucd94\uc815\ud55c KBS \uc5ed\uc0ac\uc2a4\ud398\uc15c \uc81c\uc791\ud300\uc5d0\uc11c\ub294 2000\ub144 4\uc6d4 15\uc77c\uc5d0 \u300c\uc2e0\ub77c \ucd5c\ud6c4\uc758 \ubbf8\uc2a4\ud14c\ub9ac\u2015\ub9c8\uc758\ud0dc\uc790\u300d\ub77c\ub294 \ubd80\uc81c\ub85c \ubc29\uc601\ub418\uc5c8\ub358 \u300e\uc5ed\uc0ac\uc2a4\ud398\uc15c\u300f\ubc29\uc1a1\ubd84\uc5d0\uc11c \uae08\uac15\uc0b0\uc73c\ub85c \uac14\ub2e4\ub294 \ub9c8\uc758 \ud0dc\uc790\uc758 \uc804\uc2b9\uc774 \ub0a8\uc544\uc788\ub294 \uc9c0\uc5ed\uc774 \uad73\uc774 \uacbd\uc8fc\uc5d0\uc11c \uae08\uac15\uc0b0\uc73c\ub85c \uac00\uc7a5 \ube68\ub9ac \uac08 \uc218 \uc788\ub294 \ub3d9\ud574\uc548 \uad50\ud1b5\ub85c\uac00 \uc544\ub2cc \ub0b4\ub959\uc5d0 \ub0a8\uc544\uc788\ub294 \uac83\uc5d0\uc11c, \uacbd\uc8fc\uc5d0\uc11c \uacc4\ub9bd\ub839\uc73c\ub85c \uac00\uc11c \ucda9\uc8fc\ub97c \uc9c0\ub098 \ubb3c\uae38\ub85c \uc591\ud3c9\uc73c\ub85c, \ud64d\ucc9c\uc744 \uac70\uccd0 \uc778\uc81c\uc640 \ud55c\uacc4\ub839\uc744 \uc9c0\ub098\ub294 \ub9c8\uc758 \ud0dc\uc790\uc758 \ubd81\ud589 \ub8e8\ud2b8\ub97c \uc0c1\uc815\ud55c \ub4a4, \ud574\ub2f9 \ub3c4\uc2dc\ub4e4\uc774 \uc2e0\ub77c \uc2dc\ub300\uc5d0\ub294 \uc81c2\uc758 \uc218\ub3c4\ub85c \ubd88\ub838\ub2e4\ub294 \uc810(\ucda9\uc8fc\uc758 \uacbd\uc6b0), \uadf8\ub9ac\uace0 \uc678\ubd80 \uc138\ub825\uacfc \ucca0\uc800\ud558\uac8c \ucc28\ub2e8\ub41c \uacf3(\ubbf8\ub975\ub300\uc6d0 \ud130\ub098 \ub355\uc8fc\uc0ac\uc758 \uacbd\uc6b0)\uc774\ub77c\ub294 \uc810\uc744 \ub4e4\uc5b4 \ub9c8\uc758 \ud0dc\uc790\uac00 \uc2e0\ub77c\uc758 \uc8fc\uc694 \uac70\uc810 \uac00\uc6b4\ub370\uc11c\ub3c4 \ucc9c\ud61c\uc758 \uc694\uc0c8\ub4e4\ub9cc\uc744 \ud0dd\ud574 \uac70\uce58\uba74\uc11c \uc790\uc2e0\uacfc \ub73b\uc744 \ud568\uaed8\ud560 \uc0ac\ub78c\uc744 \uaddc\ud569\ud558\ub824 \ud588\ub358 \uac83\uc774\ub77c\ub294 \ucd94\ub860\uc744 \uc81c\uae30\ud588\ub2e4.", "paragraph_answer": "\uac15\uc6d0\ub3c4 \uc778\uc81c\uc640 \ub354\ubd88\uc5b4 \ub9c8\uc758 \ud0dc\uc790(\u9ebb\u8863 \u592a\u5b50)\uc640 \uad00\ub828\ub41c \uc804\uc124\uc774 \ub9ce\uc774 \ub0a8\uc544\uc788\ub294 \uacf3\uc740 \ucda9\uc8fc\uc2dc\ub85c, \ub9c8\uc758 \ud0dc\uc790(\u9ebb\u8863 \u592a\u5b50)\uc640 \uadf8 \ub204\uc774\ub3d9\uc0dd \ub355\uc8fc \uacf5\uc8fc(\u5fb7\u4e3b \u516c\u4e3b)\uac00 \uac01\uac01 \uc870\uc131\ud588\ub2e4\ub294 \uc6d4\uc545\uc0b0 \uc790\ub77d\uc758 \ubbf8\ub975\ub300\uc6d0 \ud130\uc640 \ub355\uc8fc\uc0ac(\u5fb7\u4f4f\u5bfa)\uac00 \uc788\ub2e4. \uc591\ud3c9\uad70\uc758 \uc6a9\ubb38\uc0ac \uc5d0\ub294 \ub9c8\uc758\ud0dc\uc790\uac00 \uae08\uac15\uc0b0\uc73c\ub85c \uac00\ub294 \uae38\uc5d0 \uc808\uc5d0 \ub4e4\ub7ec\uc11c \uc9da\uace0 \uc788\ub358 \uc9c0\ud321\uc774\ub97c \ub545\uc5d0 \uaf42\uc740 \ub370\uc11c \uc720\ub798\ud588\ub2e4\ub294 \uc740\ud589\ub098\ubb34\uac00 \uc788\uc73c\uba70, \ud64d\ucc9c\uc5d0\uc11c \ub9c8\uc758 \ud0dc\uc790 \uc804\uc2b9\uc774 \ub0a8\uc544\uc788\ub294 \uc778\uc81c\ub85c \ub118\uc5b4\uac00\ub294 \uae38\ubaa9\uc5d0 \uc9c0\uc655\ub3d9(\uc655\uc774 \uc9c0\ub098\uac04 \ub9c8\uc744)\uacfc \uc655\ud130(\uc655\uc774 \ub118\uc5b4\uac04 \uc790\ub9ac)\ub77c\ub294 \uc9c0\uba85\uc774 \ub0a8\uc544\uc788\ub2e4. \uc774\ub4e4 \ub9c8\uc758 \ud0dc\uc790 \uad00\ub828 \uc804\uc2b9 \uc9c0\uc5ed\uc744 \uc9c0\ub09c 2000\ub144 \ub2f5\uc0ac \ucde8\uc7ac\ud558\uace0 \ub9c8\uc758 \ud0dc\uc790\uc758 \ubd81\ud589(\u5317\u884c) \ub8e8\ud2b8\ub97c \ucd94\uc815\ud55c KBS \uc5ed\uc0ac\uc2a4\ud398\uc15c \uc81c\uc791\ud300\uc5d0\uc11c\ub294 2000\ub144 4\uc6d4 15\uc77c\uc5d0 \u300c\uc2e0\ub77c \ucd5c\ud6c4\uc758 \ubbf8\uc2a4\ud14c\ub9ac\u2015\ub9c8\uc758\ud0dc\uc790\u300d\ub77c\ub294 \ubd80\uc81c\ub85c \ubc29\uc601\ub418\uc5c8\ub358 \u300e\uc5ed\uc0ac\uc2a4\ud398\uc15c\u300f\ubc29\uc1a1\ubd84\uc5d0\uc11c \uae08\uac15\uc0b0\uc73c\ub85c \uac14\ub2e4\ub294 \ub9c8\uc758 \ud0dc\uc790\uc758 \uc804\uc2b9\uc774 \ub0a8\uc544\uc788\ub294 \uc9c0\uc5ed\uc774 \uad73\uc774 \uacbd\uc8fc\uc5d0\uc11c \uae08\uac15\uc0b0\uc73c\ub85c \uac00\uc7a5 \ube68\ub9ac \uac08 \uc218 \uc788\ub294 \ub3d9\ud574\uc548 \uad50\ud1b5\ub85c\uac00 \uc544\ub2cc \ub0b4\ub959\uc5d0 \ub0a8\uc544\uc788\ub294 \uac83\uc5d0\uc11c, \uacbd\uc8fc\uc5d0\uc11c \uacc4\ub9bd\ub839\uc73c\ub85c \uac00\uc11c \ucda9\uc8fc\ub97c \uc9c0\ub098 \ubb3c\uae38\ub85c \uc591\ud3c9\uc73c\ub85c, \ud64d\ucc9c\uc744 \uac70\uccd0 \uc778\uc81c\uc640 \ud55c\uacc4\ub839\uc744 \uc9c0\ub098\ub294 \ub9c8\uc758 \ud0dc\uc790\uc758 \ubd81\ud589 \ub8e8\ud2b8\ub97c \uc0c1\uc815\ud55c \ub4a4, \ud574\ub2f9 \ub3c4\uc2dc\ub4e4\uc774 \uc2e0\ub77c \uc2dc\ub300\uc5d0\ub294 \uc81c2\uc758 \uc218\ub3c4\ub85c \ubd88\ub838\ub2e4\ub294 \uc810(\ucda9\uc8fc\uc758 \uacbd\uc6b0), \uadf8\ub9ac\uace0 \uc678\ubd80 \uc138\ub825\uacfc \ucca0\uc800\ud558\uac8c \ucc28\ub2e8\ub41c \uacf3(\ubbf8\ub975\ub300\uc6d0 \ud130\ub098 \ub355\uc8fc\uc0ac\uc758 \uacbd\uc6b0)\uc774\ub77c\ub294 \uc810\uc744 \ub4e4\uc5b4 \ub9c8\uc758 \ud0dc\uc790\uac00 \uc2e0\ub77c\uc758 \uc8fc\uc694 \uac70\uc810 \uac00\uc6b4\ub370\uc11c\ub3c4 \ucc9c\ud61c\uc758 \uc694\uc0c8\ub4e4\ub9cc\uc744 \ud0dd\ud574 \uac70\uce58\uba74\uc11c \uc790\uc2e0\uacfc \ub73b\uc744 \ud568\uaed8\ud560 \uc0ac\ub78c\uc744 \uaddc\ud569\ud558\ub824 \ud588\ub358 \uac83\uc774\ub77c\ub294 \ucd94\ub860\uc744 \uc81c\uae30\ud588\ub2e4.", "sentence_answer": "\uc591\ud3c9\uad70\uc758 \uc6a9\ubb38\uc0ac \uc5d0\ub294 \ub9c8\uc758\ud0dc\uc790\uac00 \uae08\uac15\uc0b0\uc73c\ub85c \uac00\ub294 \uae38\uc5d0 \uc808\uc5d0 \ub4e4\ub7ec\uc11c \uc9da\uace0 \uc788\ub358 \uc9c0\ud321\uc774\ub97c \ub545\uc5d0 \uaf42\uc740 \ub370\uc11c \uc720\ub798\ud588\ub2e4\ub294 \uc740\ud589\ub098\ubb34\uac00 \uc788\uc73c\uba70, \ud64d\ucc9c\uc5d0\uc11c \ub9c8\uc758 \ud0dc\uc790 \uc804\uc2b9\uc774 \ub0a8\uc544\uc788\ub294 \uc778\uc81c\ub85c \ub118\uc5b4\uac00\ub294 \uae38\ubaa9\uc5d0 \uc9c0\uc655\ub3d9(\uc655\uc774 \uc9c0\ub098\uac04 \ub9c8\uc744)\uacfc \uc655\ud130(\uc655\uc774 \ub118\uc5b4\uac04 \uc790\ub9ac)\ub77c\ub294 \uc9c0\uba85\uc774 \ub0a8\uc544\uc788\ub2e4.", "paragraph_id": "6500216-2-0", "paragraph_question": "question: \ub9c8\uc758 \ud0dc\uc790\uac00 \uae08\uac15\uc0b0\uc73c\ub85c \uac00\ub294 \uae38 \uc808\uc5d0 \ub4e4\ub7ec \uc9c0\ud321\uc774\ub97c \uaf42\ub294\ub370\uc11c \uc720\ub798\ud588\ub2e4\ub294 \uc740\ud589\ub098\ubb34\uac00 \uc788\ub294 \uacf3\uc740 \uc5b4\ub514\uc778\uac00?, context: \uac15\uc6d0\ub3c4 \uc778\uc81c\uc640 \ub354\ubd88\uc5b4 \ub9c8\uc758 \ud0dc\uc790(\u9ebb\u8863 \u592a\u5b50)\uc640 \uad00\ub828\ub41c \uc804\uc124\uc774 \ub9ce\uc774 \ub0a8\uc544\uc788\ub294 \uacf3\uc740 \ucda9\uc8fc\uc2dc\ub85c, \ub9c8\uc758 \ud0dc\uc790(\u9ebb\u8863 \u592a\u5b50)\uc640 \uadf8 \ub204\uc774\ub3d9\uc0dd \ub355\uc8fc \uacf5\uc8fc(\u5fb7\u4e3b \u516c\u4e3b)\uac00 \uac01\uac01 \uc870\uc131\ud588\ub2e4\ub294 \uc6d4\uc545\uc0b0 \uc790\ub77d\uc758 \ubbf8\ub975\ub300\uc6d0 \ud130\uc640 \ub355\uc8fc\uc0ac(\u5fb7\u4f4f\u5bfa)\uac00 \uc788\ub2e4. \uc591\ud3c9\uad70\uc758 \uc6a9\ubb38\uc0ac\uc5d0\ub294 \ub9c8\uc758\ud0dc\uc790\uac00 \uae08\uac15\uc0b0\uc73c\ub85c \uac00\ub294 \uae38\uc5d0 \uc808\uc5d0 \ub4e4\ub7ec\uc11c \uc9da\uace0 \uc788\ub358 \uc9c0\ud321\uc774\ub97c \ub545\uc5d0 \uaf42\uc740 \ub370\uc11c \uc720\ub798\ud588\ub2e4\ub294 \uc740\ud589\ub098\ubb34\uac00 \uc788\uc73c\uba70, \ud64d\ucc9c\uc5d0\uc11c \ub9c8\uc758 \ud0dc\uc790 \uc804\uc2b9\uc774 \ub0a8\uc544\uc788\ub294 \uc778\uc81c\ub85c \ub118\uc5b4\uac00\ub294 \uae38\ubaa9\uc5d0 \uc9c0\uc655\ub3d9(\uc655\uc774 \uc9c0\ub098\uac04 \ub9c8\uc744)\uacfc \uc655\ud130(\uc655\uc774 \ub118\uc5b4\uac04 \uc790\ub9ac)\ub77c\ub294 \uc9c0\uba85\uc774 \ub0a8\uc544\uc788\ub2e4. \uc774\ub4e4 \ub9c8\uc758 \ud0dc\uc790 \uad00\ub828 \uc804\uc2b9 \uc9c0\uc5ed\uc744 \uc9c0\ub09c 2000\ub144 \ub2f5\uc0ac \ucde8\uc7ac\ud558\uace0 \ub9c8\uc758 \ud0dc\uc790\uc758 \ubd81\ud589(\u5317\u884c) \ub8e8\ud2b8\ub97c \ucd94\uc815\ud55c KBS \uc5ed\uc0ac\uc2a4\ud398\uc15c \uc81c\uc791\ud300\uc5d0\uc11c\ub294 2000\ub144 4\uc6d4 15\uc77c\uc5d0 \u300c\uc2e0\ub77c \ucd5c\ud6c4\uc758 \ubbf8\uc2a4\ud14c\ub9ac\u2015\ub9c8\uc758\ud0dc\uc790\u300d\ub77c\ub294 \ubd80\uc81c\ub85c \ubc29\uc601\ub418\uc5c8\ub358 \u300e\uc5ed\uc0ac\uc2a4\ud398\uc15c\u300f\ubc29\uc1a1\ubd84\uc5d0\uc11c \uae08\uac15\uc0b0\uc73c\ub85c \uac14\ub2e4\ub294 \ub9c8\uc758 \ud0dc\uc790\uc758 \uc804\uc2b9\uc774 \ub0a8\uc544\uc788\ub294 \uc9c0\uc5ed\uc774 \uad73\uc774 \uacbd\uc8fc\uc5d0\uc11c \uae08\uac15\uc0b0\uc73c\ub85c \uac00\uc7a5 \ube68\ub9ac \uac08 \uc218 \uc788\ub294 \ub3d9\ud574\uc548 \uad50\ud1b5\ub85c\uac00 \uc544\ub2cc \ub0b4\ub959\uc5d0 \ub0a8\uc544\uc788\ub294 \uac83\uc5d0\uc11c, \uacbd\uc8fc\uc5d0\uc11c \uacc4\ub9bd\ub839\uc73c\ub85c \uac00\uc11c \ucda9\uc8fc\ub97c \uc9c0\ub098 \ubb3c\uae38\ub85c \uc591\ud3c9\uc73c\ub85c, \ud64d\ucc9c\uc744 \uac70\uccd0 \uc778\uc81c\uc640 \ud55c\uacc4\ub839\uc744 \uc9c0\ub098\ub294 \ub9c8\uc758 \ud0dc\uc790\uc758 \ubd81\ud589 \ub8e8\ud2b8\ub97c \uc0c1\uc815\ud55c \ub4a4, \ud574\ub2f9 \ub3c4\uc2dc\ub4e4\uc774 \uc2e0\ub77c \uc2dc\ub300\uc5d0\ub294 \uc81c2\uc758 \uc218\ub3c4\ub85c \ubd88\ub838\ub2e4\ub294 \uc810(\ucda9\uc8fc\uc758 \uacbd\uc6b0), \uadf8\ub9ac\uace0 \uc678\ubd80 \uc138\ub825\uacfc \ucca0\uc800\ud558\uac8c \ucc28\ub2e8\ub41c \uacf3(\ubbf8\ub975\ub300\uc6d0 \ud130\ub098 \ub355\uc8fc\uc0ac\uc758 \uacbd\uc6b0)\uc774\ub77c\ub294 \uc810\uc744 \ub4e4\uc5b4 \ub9c8\uc758 \ud0dc\uc790\uac00 \uc2e0\ub77c\uc758 \uc8fc\uc694 \uac70\uc810 \uac00\uc6b4\ub370\uc11c\ub3c4 \ucc9c\ud61c\uc758 \uc694\uc0c8\ub4e4\ub9cc\uc744 \ud0dd\ud574 \uac70\uce58\uba74\uc11c \uc790\uc2e0\uacfc \ub73b\uc744 \ud568\uaed8\ud560 \uc0ac\ub78c\uc744 \uaddc\ud569\ud558\ub824 \ud588\ub358 \uac83\uc774\ub77c\ub294 \ucd94\ub860\uc744 \uc81c\uae30\ud588\ub2e4."} {"question": "\uc2e0\uce78\uc13c E6\uacc4 \uc804\ub3d9\ucc28\ub294 \uba87 \uc720\ub2db\uc73c\ub85c \uad6c\uc131\ub418\uc5b4 \uc788\ub294\uac00?", "paragraph": "\ud3b8\uc131 \uc804\uccb4\ub294 11\ud638\ucc28\uc5d0\uc11c 14\ud638\ucc28\uae4c\uc9c0 4\ub7c9(M1sc+Tk+M1+M1)\uacfc 15\ud638\ucc28\uc5d0\uc11c 17\ud638\ucc28\uae4c\uc9c0 3\ub7c9(M1+T+M1c)\uc758 2\uc720\ub2db\uc73c\ub85c \uad6c\uc131\ub418\uc5b4 \uc788\uc73c\uba70, \uc804\ub3d9\ucc28\uc5d0\ub294 E5\uacc4\uc6a9\uc73c\ub85c \uacf5\ud1b5\ud654 \uc124\uacc4\ub41c \uc8fc\ubcc0\ud658\uc7a5\uce58\ub97c, \ubd80\uc218\ucc28\uc5d0 \uc8fc\ubcc0\uc555\uae30\uc640 \uc9d1\uc804\uc7a5\uce58\ub97c \ud0d1\uc7ac\ud55c\ub2e4. \uc0c1\ud558(\u5e8a\u4e0b) \uae30\uae30\ub294 \uc0b0(\u5c71) \ucabd\uc5d0 \uc810\uac80\ube48\ub3c4\uac00 \ub9ce\uc740 \uc8fc\uc694 \uae30\uae30\ub97c \uc9d1\uc911 \ubc30\uce58\uc2dc\ucf1c \uad00\ub9ac\ub97c \uc6a9\uc774\ud558\uac8c \ud558\uace0, \ubc18\ub300\ub85c \uc810\uac80 \ube48\ub3c4\uac00 \uc801\uc740 \uae30\uae30\ub294 \uc8fc\ubcc0\ud658\uc7a5\uce58 \uc911\uc559\uc758 \ube48 \uacf5\uac04\uc5d0 \ubc30\uce58\ub418\uc5b4 \uae30\uae30\uc758 \uc18c\ud615\ud654\ub97c \uaf80\ud558\uace0 \uc788\ub2e4. \ubcf4\uc870 \uc804\uc6d0 \uc7a5\uce58\ub294 11\u00b717\ud638\ucc28, \uc804\ub3d9 \uacf5\uae30\uc555\ucd95\uae30\ub294 11\u00b713\u00b715\u00b717\ud638\ucc28\uc5d0 \ud0d1\uc7ac\ub418\uace0 \uc788\uc73c\uba70, 11\ud638\ucc28\uc5d0\ub294 \ubcf4\uc870\uc6a9 \uc804\ub3d9 \uacf5\uae30 \uc555\ucd95\uae30\ub97c \ud0d1\uc7ac\ud558\uace0 \uc788\ub2e4. \uc774\uac83\uc740 \uc774 \ucc28\ub7c9\uc758 \uacf5\uae30 \uc2a4\ud504\ub9c1\uc5d0 \uc758\ud55c \ud2f8\ud305 \uc2dc\uc2a4\ud15c \ub4f1\uc5d0 \uc0ac\uc6a9\ub418\ub294 \uc555\ucd95 \uacf5\uae30 \uc18c\ube44\uac00 \ub9ce\uc740 \uac83\uc744 \uace0\ub824\ud574\uc11c\uc774\ub2e4.", "answer": "2\uc720\ub2db", "sentence": "\ud3b8\uc131 \uc804\uccb4\ub294 11\ud638\ucc28\uc5d0\uc11c 14\ud638\ucc28\uae4c\uc9c0 4\ub7c9(M1sc+Tk+M1+M1)\uacfc 15\ud638\ucc28\uc5d0\uc11c 17\ud638\ucc28\uae4c\uc9c0 3\ub7c9(M1+T+M1c)\uc758 2\uc720\ub2db \uc73c\ub85c \uad6c\uc131\ub418\uc5b4 \uc788\uc73c\uba70, \uc804\ub3d9\ucc28\uc5d0\ub294 E5\uacc4\uc6a9\uc73c\ub85c \uacf5\ud1b5\ud654 \uc124\uacc4\ub41c \uc8fc\ubcc0\ud658\uc7a5\uce58\ub97c, \ubd80\uc218\ucc28\uc5d0 \uc8fc\ubcc0\uc555\uae30\uc640 \uc9d1\uc804\uc7a5\uce58\ub97c \ud0d1\uc7ac\ud55c\ub2e4.", "paragraph_sentence": " \ud3b8\uc131 \uc804\uccb4\ub294 11\ud638\ucc28\uc5d0\uc11c 14\ud638\ucc28\uae4c\uc9c0 4\ub7c9(M1sc+Tk+M1+M1)\uacfc 15\ud638\ucc28\uc5d0\uc11c 17\ud638\ucc28\uae4c\uc9c0 3\ub7c9(M1+T+M1c)\uc758 2\uc720\ub2db \uc73c\ub85c \uad6c\uc131\ub418\uc5b4 \uc788\uc73c\uba70, \uc804\ub3d9\ucc28\uc5d0\ub294 E5\uacc4\uc6a9\uc73c\ub85c \uacf5\ud1b5\ud654 \uc124\uacc4\ub41c \uc8fc\ubcc0\ud658\uc7a5\uce58\ub97c, \ubd80\uc218\ucc28\uc5d0 \uc8fc\ubcc0\uc555\uae30\uc640 \uc9d1\uc804\uc7a5\uce58\ub97c \ud0d1\uc7ac\ud55c\ub2e4. \uc0c1\ud558(\u5e8a\u4e0b) \uae30\uae30\ub294 \uc0b0(\u5c71) \ucabd\uc5d0 \uc810\uac80\ube48\ub3c4\uac00 \ub9ce\uc740 \uc8fc\uc694 \uae30\uae30\ub97c \uc9d1\uc911 \ubc30\uce58\uc2dc\ucf1c \uad00\ub9ac\ub97c \uc6a9\uc774\ud558\uac8c \ud558\uace0, \ubc18\ub300\ub85c \uc810\uac80 \ube48\ub3c4\uac00 \uc801\uc740 \uae30\uae30\ub294 \uc8fc\ubcc0\ud658\uc7a5\uce58 \uc911\uc559\uc758 \ube48 \uacf5\uac04\uc5d0 \ubc30\uce58\ub418\uc5b4 \uae30\uae30\uc758 \uc18c\ud615\ud654\ub97c \uaf80\ud558\uace0 \uc788\ub2e4. \ubcf4\uc870 \uc804\uc6d0 \uc7a5\uce58\ub294 11\u00b717\ud638\ucc28, \uc804\ub3d9 \uacf5\uae30\uc555\ucd95\uae30\ub294 11\u00b713\u00b715\u00b717\ud638\ucc28\uc5d0 \ud0d1\uc7ac\ub418\uace0 \uc788\uc73c\uba70, 11\ud638\ucc28\uc5d0\ub294 \ubcf4\uc870\uc6a9 \uc804\ub3d9 \uacf5\uae30 \uc555\ucd95\uae30\ub97c \ud0d1\uc7ac\ud558\uace0 \uc788\ub2e4. \uc774\uac83\uc740 \uc774 \ucc28\ub7c9\uc758 \uacf5\uae30 \uc2a4\ud504\ub9c1\uc5d0 \uc758\ud55c \ud2f8\ud305 \uc2dc\uc2a4\ud15c \ub4f1\uc5d0 \uc0ac\uc6a9\ub418\ub294 \uc555\ucd95 \uacf5\uae30 \uc18c\ube44\uac00 \ub9ce\uc740 \uac83\uc744 \uace0\ub824\ud574\uc11c\uc774\ub2e4.", "paragraph_answer": "\ud3b8\uc131 \uc804\uccb4\ub294 11\ud638\ucc28\uc5d0\uc11c 14\ud638\ucc28\uae4c\uc9c0 4\ub7c9(M1sc+Tk+M1+M1)\uacfc 15\ud638\ucc28\uc5d0\uc11c 17\ud638\ucc28\uae4c\uc9c0 3\ub7c9(M1+T+M1c)\uc758 2\uc720\ub2db \uc73c\ub85c \uad6c\uc131\ub418\uc5b4 \uc788\uc73c\uba70, \uc804\ub3d9\ucc28\uc5d0\ub294 E5\uacc4\uc6a9\uc73c\ub85c \uacf5\ud1b5\ud654 \uc124\uacc4\ub41c \uc8fc\ubcc0\ud658\uc7a5\uce58\ub97c, \ubd80\uc218\ucc28\uc5d0 \uc8fc\ubcc0\uc555\uae30\uc640 \uc9d1\uc804\uc7a5\uce58\ub97c \ud0d1\uc7ac\ud55c\ub2e4. \uc0c1\ud558(\u5e8a\u4e0b) \uae30\uae30\ub294 \uc0b0(\u5c71) \ucabd\uc5d0 \uc810\uac80\ube48\ub3c4\uac00 \ub9ce\uc740 \uc8fc\uc694 \uae30\uae30\ub97c \uc9d1\uc911 \ubc30\uce58\uc2dc\ucf1c \uad00\ub9ac\ub97c \uc6a9\uc774\ud558\uac8c \ud558\uace0, \ubc18\ub300\ub85c \uc810\uac80 \ube48\ub3c4\uac00 \uc801\uc740 \uae30\uae30\ub294 \uc8fc\ubcc0\ud658\uc7a5\uce58 \uc911\uc559\uc758 \ube48 \uacf5\uac04\uc5d0 \ubc30\uce58\ub418\uc5b4 \uae30\uae30\uc758 \uc18c\ud615\ud654\ub97c \uaf80\ud558\uace0 \uc788\ub2e4. \ubcf4\uc870 \uc804\uc6d0 \uc7a5\uce58\ub294 11\u00b717\ud638\ucc28, \uc804\ub3d9 \uacf5\uae30\uc555\ucd95\uae30\ub294 11\u00b713\u00b715\u00b717\ud638\ucc28\uc5d0 \ud0d1\uc7ac\ub418\uace0 \uc788\uc73c\uba70, 11\ud638\ucc28\uc5d0\ub294 \ubcf4\uc870\uc6a9 \uc804\ub3d9 \uacf5\uae30 \uc555\ucd95\uae30\ub97c \ud0d1\uc7ac\ud558\uace0 \uc788\ub2e4. \uc774\uac83\uc740 \uc774 \ucc28\ub7c9\uc758 \uacf5\uae30 \uc2a4\ud504\ub9c1\uc5d0 \uc758\ud55c \ud2f8\ud305 \uc2dc\uc2a4\ud15c \ub4f1\uc5d0 \uc0ac\uc6a9\ub418\ub294 \uc555\ucd95 \uacf5\uae30 \uc18c\ube44\uac00 \ub9ce\uc740 \uac83\uc744 \uace0\ub824\ud574\uc11c\uc774\ub2e4.", "sentence_answer": "\ud3b8\uc131 \uc804\uccb4\ub294 11\ud638\ucc28\uc5d0\uc11c 14\ud638\ucc28\uae4c\uc9c0 4\ub7c9(M1sc+Tk+M1+M1)\uacfc 15\ud638\ucc28\uc5d0\uc11c 17\ud638\ucc28\uae4c\uc9c0 3\ub7c9(M1+T+M1c)\uc758 2\uc720\ub2db \uc73c\ub85c \uad6c\uc131\ub418\uc5b4 \uc788\uc73c\uba70, \uc804\ub3d9\ucc28\uc5d0\ub294 E5\uacc4\uc6a9\uc73c\ub85c \uacf5\ud1b5\ud654 \uc124\uacc4\ub41c \uc8fc\ubcc0\ud658\uc7a5\uce58\ub97c, \ubd80\uc218\ucc28\uc5d0 \uc8fc\ubcc0\uc555\uae30\uc640 \uc9d1\uc804\uc7a5\uce58\ub97c \ud0d1\uc7ac\ud55c\ub2e4.", "paragraph_id": "6519826-0-0", "paragraph_question": "question: \uc2e0\uce78\uc13c E6\uacc4 \uc804\ub3d9\ucc28\ub294 \uba87 \uc720\ub2db\uc73c\ub85c \uad6c\uc131\ub418\uc5b4 \uc788\ub294\uac00?, context: \ud3b8\uc131 \uc804\uccb4\ub294 11\ud638\ucc28\uc5d0\uc11c 14\ud638\ucc28\uae4c\uc9c0 4\ub7c9(M1sc+Tk+M1+M1)\uacfc 15\ud638\ucc28\uc5d0\uc11c 17\ud638\ucc28\uae4c\uc9c0 3\ub7c9(M1+T+M1c)\uc758 2\uc720\ub2db\uc73c\ub85c \uad6c\uc131\ub418\uc5b4 \uc788\uc73c\uba70, \uc804\ub3d9\ucc28\uc5d0\ub294 E5\uacc4\uc6a9\uc73c\ub85c \uacf5\ud1b5\ud654 \uc124\uacc4\ub41c \uc8fc\ubcc0\ud658\uc7a5\uce58\ub97c, \ubd80\uc218\ucc28\uc5d0 \uc8fc\ubcc0\uc555\uae30\uc640 \uc9d1\uc804\uc7a5\uce58\ub97c \ud0d1\uc7ac\ud55c\ub2e4. \uc0c1\ud558(\u5e8a\u4e0b) \uae30\uae30\ub294 \uc0b0(\u5c71) \ucabd\uc5d0 \uc810\uac80\ube48\ub3c4\uac00 \ub9ce\uc740 \uc8fc\uc694 \uae30\uae30\ub97c \uc9d1\uc911 \ubc30\uce58\uc2dc\ucf1c \uad00\ub9ac\ub97c \uc6a9\uc774\ud558\uac8c \ud558\uace0, \ubc18\ub300\ub85c \uc810\uac80 \ube48\ub3c4\uac00 \uc801\uc740 \uae30\uae30\ub294 \uc8fc\ubcc0\ud658\uc7a5\uce58 \uc911\uc559\uc758 \ube48 \uacf5\uac04\uc5d0 \ubc30\uce58\ub418\uc5b4 \uae30\uae30\uc758 \uc18c\ud615\ud654\ub97c \uaf80\ud558\uace0 \uc788\ub2e4. \ubcf4\uc870 \uc804\uc6d0 \uc7a5\uce58\ub294 11\u00b717\ud638\ucc28, \uc804\ub3d9 \uacf5\uae30\uc555\ucd95\uae30\ub294 11\u00b713\u00b715\u00b717\ud638\ucc28\uc5d0 \ud0d1\uc7ac\ub418\uace0 \uc788\uc73c\uba70, 11\ud638\ucc28\uc5d0\ub294 \ubcf4\uc870\uc6a9 \uc804\ub3d9 \uacf5\uae30 \uc555\ucd95\uae30\ub97c \ud0d1\uc7ac\ud558\uace0 \uc788\ub2e4. \uc774\uac83\uc740 \uc774 \ucc28\ub7c9\uc758 \uacf5\uae30 \uc2a4\ud504\ub9c1\uc5d0 \uc758\ud55c \ud2f8\ud305 \uc2dc\uc2a4\ud15c \ub4f1\uc5d0 \uc0ac\uc6a9\ub418\ub294 \uc555\ucd95 \uacf5\uae30 \uc18c\ube44\uac00 \ub9ce\uc740 \uac83\uc744 \uace0\ub824\ud574\uc11c\uc774\ub2e4."} {"question": "\uc138\uc6d4\ud638 \uc0ac\uace0\ub85c \ud76c\uc0dd\ub41c \uae30\uac04\uc81c \uad50\uc0ac\ub294 \uacf5\ubb34\uc6d0\uc774 \uc544\ub2c8\uae30 \ub54c\ubb38\uc5d0 \ubb34\uc5c7\uc744 \uc778\uc815 \ubc1b\uc744 \uc218 \uc5c6\ub294\uac00?", "paragraph": "\uc0ac\uace0 \ucd08\uae30\ubd80\ud130 \uc774\ub4e4 '\uc0b4\uc2e0\uc131\uc778' \uad50\uc0ac\ub4e4\uc744 \uc758\uc0ac\uc790\ub85c \uc9c0\uc815\ud574\ub2ec\ub77c\ub294 \ub204\ub9ac\uafbc\ub4e4\uc758 \uccad\uc6d0\uc774 \uc774\uc5b4\uc9c0\uace0 \uc788\uc9c0\ub9cc '\uc758\uc0ac\uc790' \uc2e0\uccad\uc740 \uc720\ubcf4\ub41c \uac83\uc73c\ub85c \ud655\uc778\ub410\ub2e4. \ud558\uc9c0\ub9cc \uad50\uc0ac \uc720\uac00\uc871\ub4e4\uc740 '\uc21c\uc9c1' \ucc98\ub9ac\ub97c \ud1b5\ud574 \uad6d\uac00 \uc720\uacf5\uc790\ub85c \ub4f1\ub85d\ub418\ub294 \uac83\uc744 \uc6b0\uc120\uc801\uc73c\ub85c \ucd94\uc9c4\ud558\ub294 \uac83\uc73c\ub85c \uc54c\ub824\uc84c\ub2e4. '\ud0c0\uc778\uc758 \uc0dd\uba85\uc744 \uad6c\ud558\uae30 \uc704\ud574 \uc560\uc4f0\ub2e4 \uc228\uc84c\ub2e4'\ub294 \uc810\uc744 \uc785\uc99d\ud574 \uc758\uc0ac\uc790\ub85c \ub4f1\ub85d\ud558\uae30 \ubcf4\ub2e4\ub294 \uad6d\uac00\uacf5\ubb34\uc6d0\uc73c\ub85c\uc11c \uacf5\ubb34\uc218\ud589 \uc911 \ud76c\uc0dd\ub2f9\ud588\ub2e4\ub294 \uc810\uc744 \uc778\uc815\ubc1b\ub294 \uac83\uc774 \uc6a9\uc774\ud574 \uc774 \uac19\uc740 \ubc29\ud5a5\uc73c\ub85c \ucd94\uc9c4 \uc911\uc73c\ub85c \uc804\ud574\uc84c\ub2e4. \ubaa8\ub4e0 \uad50\uc0ac\uac00 \uacf5\ubb34\uc0c1 \uc0ac\ub9dd\uc744 \uc778\uc815\ubc1b\uc744 \uc218 \uc788\ub294 \uac74 \uc544\ub2c8\ub2e4. \uadf8\ub7ec\ub098 \uc774\ubc88 \uc0ac\uace0\ub85c \ud76c\uc0dd\ub418\uac70\ub098 \uc2e4\uc885\ub41c \uad50\uc6d0 12\uba85 \uc911 \uae30\uac04\uc81c \uad50\uc0ac\uc778 2\uba85\uc740 \uacf5\ubb34\uc6d0\uc774 \uc544\ub2c8\uba70, \uc774\ub4e4\uc740 \uacf5\ubb34\uc6d0\uc5f0\uae08\uad00\ub9ac\uacf5\ub2e8\uc758 '\uacf5\ubb34\uc0c1 \uc0ac\ub9dd' \uc778\uc815\uc744 \ubc1b\uc744 \uc218 \uc788\ub294 \uc790\uaca9\uc774 \ubabb \ub41c\ub2e4. \ud55c\ud3b8 \uc720\uac00\uc871 \ub300\ucc45\uc704\ub294 '\uc138\uc6d4\ud638 \ud2b9\ubcc4\ubc95'\uc5d0 \ub2e8\uc6d0\uace0 \uad50\uc0ac\uc640 \ud559\uc0dd\ub4e4\uc744 \uc804\uc6d0 \uc758\uc0ac\uc790\ub85c \uc9c0\uc815\ud558\ub294 \ubc29\uc548\uc744 \ucd94\uc9c4\ud588\ub358 \uac83\uc73c\ub85c \uc54c\ub824\uc84c\uc73c\ub098 \uc774 \ub610\ud55c \uc758\uacac\uc774 \ub098\ub258\uc5b4 \uc544\uc9c1 \ud655\uc815\ub418\uc9c0 \uc54a\uc740 \uac83\uc73c\ub85c \ud655\uc778\ub410\ub2e4.", "answer": "\uacf5\ubb34\uc0c1 \uc0ac\ub9dd", "sentence": "\ubaa8\ub4e0 \uad50\uc0ac\uac00 \uacf5\ubb34\uc0c1 \uc0ac\ub9dd \uc744 \uc778\uc815\ubc1b\uc744 \uc218 \uc788\ub294 \uac74 \uc544\ub2c8\ub2e4.", "paragraph_sentence": "\uc0ac\uace0 \ucd08\uae30\ubd80\ud130 \uc774\ub4e4 '\uc0b4\uc2e0\uc131\uc778' \uad50\uc0ac\ub4e4\uc744 \uc758\uc0ac\uc790\ub85c \uc9c0\uc815\ud574\ub2ec\ub77c\ub294 \ub204\ub9ac\uafbc\ub4e4\uc758 \uccad\uc6d0\uc774 \uc774\uc5b4\uc9c0\uace0 \uc788\uc9c0\ub9cc '\uc758\uc0ac\uc790' \uc2e0\uccad\uc740 \uc720\ubcf4\ub41c \uac83\uc73c\ub85c \ud655\uc778\ub410\ub2e4. \ud558\uc9c0\ub9cc \uad50\uc0ac \uc720\uac00\uc871\ub4e4\uc740 '\uc21c\uc9c1' \ucc98\ub9ac\ub97c \ud1b5\ud574 \uad6d\uac00 \uc720\uacf5\uc790\ub85c \ub4f1\ub85d\ub418\ub294 \uac83\uc744 \uc6b0\uc120\uc801\uc73c\ub85c \ucd94\uc9c4\ud558\ub294 \uac83\uc73c\ub85c \uc54c\ub824\uc84c\ub2e4. '\ud0c0\uc778\uc758 \uc0dd\uba85\uc744 \uad6c\ud558\uae30 \uc704\ud574 \uc560\uc4f0\ub2e4 \uc228\uc84c\ub2e4'\ub294 \uc810\uc744 \uc785\uc99d\ud574 \uc758\uc0ac\uc790\ub85c \ub4f1\ub85d\ud558\uae30 \ubcf4\ub2e4\ub294 \uad6d\uac00\uacf5\ubb34\uc6d0\uc73c\ub85c\uc11c \uacf5\ubb34\uc218\ud589 \uc911 \ud76c\uc0dd\ub2f9\ud588\ub2e4\ub294 \uc810\uc744 \uc778\uc815\ubc1b\ub294 \uac83\uc774 \uc6a9\uc774\ud574 \uc774 \uac19\uc740 \ubc29\ud5a5\uc73c\ub85c \ucd94\uc9c4 \uc911\uc73c\ub85c \uc804\ud574\uc84c\ub2e4. \ubaa8\ub4e0 \uad50\uc0ac\uac00 \uacf5\ubb34\uc0c1 \uc0ac\ub9dd \uc744 \uc778\uc815\ubc1b\uc744 \uc218 \uc788\ub294 \uac74 \uc544\ub2c8\ub2e4. \uadf8\ub7ec\ub098 \uc774\ubc88 \uc0ac\uace0\ub85c \ud76c\uc0dd\ub418\uac70\ub098 \uc2e4\uc885\ub41c \uad50\uc6d0 12\uba85 \uc911 \uae30\uac04\uc81c \uad50\uc0ac\uc778 2\uba85\uc740 \uacf5\ubb34\uc6d0\uc774 \uc544\ub2c8\uba70, \uc774\ub4e4\uc740 \uacf5\ubb34\uc6d0\uc5f0\uae08\uad00\ub9ac\uacf5\ub2e8\uc758 '\uacf5\ubb34\uc0c1 \uc0ac\ub9dd' \uc778\uc815\uc744 \ubc1b\uc744 \uc218 \uc788\ub294 \uc790\uaca9\uc774 \ubabb \ub41c\ub2e4. \ud55c\ud3b8 \uc720\uac00\uc871 \ub300\ucc45\uc704\ub294 '\uc138\uc6d4\ud638 \ud2b9\ubcc4\ubc95'\uc5d0 \ub2e8\uc6d0\uace0 \uad50\uc0ac\uc640 \ud559\uc0dd\ub4e4\uc744 \uc804\uc6d0 \uc758\uc0ac\uc790\ub85c \uc9c0\uc815\ud558\ub294 \ubc29\uc548\uc744 \ucd94\uc9c4\ud588\ub358 \uac83\uc73c\ub85c \uc54c\ub824\uc84c\uc73c\ub098 \uc774 \ub610\ud55c \uc758\uacac\uc774 \ub098\ub258\uc5b4 \uc544\uc9c1 \ud655\uc815\ub418\uc9c0 \uc54a\uc740 \uac83\uc73c\ub85c \ud655\uc778\ub410\ub2e4.", "paragraph_answer": "\uc0ac\uace0 \ucd08\uae30\ubd80\ud130 \uc774\ub4e4 '\uc0b4\uc2e0\uc131\uc778' \uad50\uc0ac\ub4e4\uc744 \uc758\uc0ac\uc790\ub85c \uc9c0\uc815\ud574\ub2ec\ub77c\ub294 \ub204\ub9ac\uafbc\ub4e4\uc758 \uccad\uc6d0\uc774 \uc774\uc5b4\uc9c0\uace0 \uc788\uc9c0\ub9cc '\uc758\uc0ac\uc790' \uc2e0\uccad\uc740 \uc720\ubcf4\ub41c \uac83\uc73c\ub85c \ud655\uc778\ub410\ub2e4. \ud558\uc9c0\ub9cc \uad50\uc0ac \uc720\uac00\uc871\ub4e4\uc740 '\uc21c\uc9c1' \ucc98\ub9ac\ub97c \ud1b5\ud574 \uad6d\uac00 \uc720\uacf5\uc790\ub85c \ub4f1\ub85d\ub418\ub294 \uac83\uc744 \uc6b0\uc120\uc801\uc73c\ub85c \ucd94\uc9c4\ud558\ub294 \uac83\uc73c\ub85c \uc54c\ub824\uc84c\ub2e4. '\ud0c0\uc778\uc758 \uc0dd\uba85\uc744 \uad6c\ud558\uae30 \uc704\ud574 \uc560\uc4f0\ub2e4 \uc228\uc84c\ub2e4'\ub294 \uc810\uc744 \uc785\uc99d\ud574 \uc758\uc0ac\uc790\ub85c \ub4f1\ub85d\ud558\uae30 \ubcf4\ub2e4\ub294 \uad6d\uac00\uacf5\ubb34\uc6d0\uc73c\ub85c\uc11c \uacf5\ubb34\uc218\ud589 \uc911 \ud76c\uc0dd\ub2f9\ud588\ub2e4\ub294 \uc810\uc744 \uc778\uc815\ubc1b\ub294 \uac83\uc774 \uc6a9\uc774\ud574 \uc774 \uac19\uc740 \ubc29\ud5a5\uc73c\ub85c \ucd94\uc9c4 \uc911\uc73c\ub85c \uc804\ud574\uc84c\ub2e4. \ubaa8\ub4e0 \uad50\uc0ac\uac00 \uacf5\ubb34\uc0c1 \uc0ac\ub9dd \uc744 \uc778\uc815\ubc1b\uc744 \uc218 \uc788\ub294 \uac74 \uc544\ub2c8\ub2e4. \uadf8\ub7ec\ub098 \uc774\ubc88 \uc0ac\uace0\ub85c \ud76c\uc0dd\ub418\uac70\ub098 \uc2e4\uc885\ub41c \uad50\uc6d0 12\uba85 \uc911 \uae30\uac04\uc81c \uad50\uc0ac\uc778 2\uba85\uc740 \uacf5\ubb34\uc6d0\uc774 \uc544\ub2c8\uba70, \uc774\ub4e4\uc740 \uacf5\ubb34\uc6d0\uc5f0\uae08\uad00\ub9ac\uacf5\ub2e8\uc758 '\uacf5\ubb34\uc0c1 \uc0ac\ub9dd' \uc778\uc815\uc744 \ubc1b\uc744 \uc218 \uc788\ub294 \uc790\uaca9\uc774 \ubabb \ub41c\ub2e4. \ud55c\ud3b8 \uc720\uac00\uc871 \ub300\ucc45\uc704\ub294 '\uc138\uc6d4\ud638 \ud2b9\ubcc4\ubc95'\uc5d0 \ub2e8\uc6d0\uace0 \uad50\uc0ac\uc640 \ud559\uc0dd\ub4e4\uc744 \uc804\uc6d0 \uc758\uc0ac\uc790\ub85c \uc9c0\uc815\ud558\ub294 \ubc29\uc548\uc744 \ucd94\uc9c4\ud588\ub358 \uac83\uc73c\ub85c \uc54c\ub824\uc84c\uc73c\ub098 \uc774 \ub610\ud55c \uc758\uacac\uc774 \ub098\ub258\uc5b4 \uc544\uc9c1 \ud655\uc815\ub418\uc9c0 \uc54a\uc740 \uac83\uc73c\ub85c \ud655\uc778\ub410\ub2e4.", "sentence_answer": "\ubaa8\ub4e0 \uad50\uc0ac\uac00 \uacf5\ubb34\uc0c1 \uc0ac\ub9dd \uc744 \uc778\uc815\ubc1b\uc744 \uc218 \uc788\ub294 \uac74 \uc544\ub2c8\ub2e4.", "paragraph_id": "6532130-25-0", "paragraph_question": "question: \uc138\uc6d4\ud638 \uc0ac\uace0\ub85c \ud76c\uc0dd\ub41c \uae30\uac04\uc81c \uad50\uc0ac\ub294 \uacf5\ubb34\uc6d0\uc774 \uc544\ub2c8\uae30 \ub54c\ubb38\uc5d0 \ubb34\uc5c7\uc744 \uc778\uc815 \ubc1b\uc744 \uc218 \uc5c6\ub294\uac00?, context: \uc0ac\uace0 \ucd08\uae30\ubd80\ud130 \uc774\ub4e4 '\uc0b4\uc2e0\uc131\uc778' \uad50\uc0ac\ub4e4\uc744 \uc758\uc0ac\uc790\ub85c \uc9c0\uc815\ud574\ub2ec\ub77c\ub294 \ub204\ub9ac\uafbc\ub4e4\uc758 \uccad\uc6d0\uc774 \uc774\uc5b4\uc9c0\uace0 \uc788\uc9c0\ub9cc '\uc758\uc0ac\uc790' \uc2e0\uccad\uc740 \uc720\ubcf4\ub41c \uac83\uc73c\ub85c \ud655\uc778\ub410\ub2e4. \ud558\uc9c0\ub9cc \uad50\uc0ac \uc720\uac00\uc871\ub4e4\uc740 '\uc21c\uc9c1' \ucc98\ub9ac\ub97c \ud1b5\ud574 \uad6d\uac00 \uc720\uacf5\uc790\ub85c \ub4f1\ub85d\ub418\ub294 \uac83\uc744 \uc6b0\uc120\uc801\uc73c\ub85c \ucd94\uc9c4\ud558\ub294 \uac83\uc73c\ub85c \uc54c\ub824\uc84c\ub2e4. '\ud0c0\uc778\uc758 \uc0dd\uba85\uc744 \uad6c\ud558\uae30 \uc704\ud574 \uc560\uc4f0\ub2e4 \uc228\uc84c\ub2e4'\ub294 \uc810\uc744 \uc785\uc99d\ud574 \uc758\uc0ac\uc790\ub85c \ub4f1\ub85d\ud558\uae30 \ubcf4\ub2e4\ub294 \uad6d\uac00\uacf5\ubb34\uc6d0\uc73c\ub85c\uc11c \uacf5\ubb34\uc218\ud589 \uc911 \ud76c\uc0dd\ub2f9\ud588\ub2e4\ub294 \uc810\uc744 \uc778\uc815\ubc1b\ub294 \uac83\uc774 \uc6a9\uc774\ud574 \uc774 \uac19\uc740 \ubc29\ud5a5\uc73c\ub85c \ucd94\uc9c4 \uc911\uc73c\ub85c \uc804\ud574\uc84c\ub2e4. \ubaa8\ub4e0 \uad50\uc0ac\uac00 \uacf5\ubb34\uc0c1 \uc0ac\ub9dd\uc744 \uc778\uc815\ubc1b\uc744 \uc218 \uc788\ub294 \uac74 \uc544\ub2c8\ub2e4. \uadf8\ub7ec\ub098 \uc774\ubc88 \uc0ac\uace0\ub85c \ud76c\uc0dd\ub418\uac70\ub098 \uc2e4\uc885\ub41c \uad50\uc6d0 12\uba85 \uc911 \uae30\uac04\uc81c \uad50\uc0ac\uc778 2\uba85\uc740 \uacf5\ubb34\uc6d0\uc774 \uc544\ub2c8\uba70, \uc774\ub4e4\uc740 \uacf5\ubb34\uc6d0\uc5f0\uae08\uad00\ub9ac\uacf5\ub2e8\uc758 '\uacf5\ubb34\uc0c1 \uc0ac\ub9dd' \uc778\uc815\uc744 \ubc1b\uc744 \uc218 \uc788\ub294 \uc790\uaca9\uc774 \ubabb \ub41c\ub2e4. \ud55c\ud3b8 \uc720\uac00\uc871 \ub300\ucc45\uc704\ub294 '\uc138\uc6d4\ud638 \ud2b9\ubcc4\ubc95'\uc5d0 \ub2e8\uc6d0\uace0 \uad50\uc0ac\uc640 \ud559\uc0dd\ub4e4\uc744 \uc804\uc6d0 \uc758\uc0ac\uc790\ub85c \uc9c0\uc815\ud558\ub294 \ubc29\uc548\uc744 \ucd94\uc9c4\ud588\ub358 \uac83\uc73c\ub85c \uc54c\ub824\uc84c\uc73c\ub098 \uc774 \ub610\ud55c \uc758\uacac\uc774 \ub098\ub258\uc5b4 \uc544\uc9c1 \ud655\uc815\ub418\uc9c0 \uc54a\uc740 \uac83\uc73c\ub85c \ud655\uc778\ub410\ub2e4."} {"question": "\uc655\uac74\uc740 \ucca0\uc6d0\uc5d0\uc11c \uc989\uc704\ud55c \ub4a4 \uc218\ub3c4\ub97c \uc5b4\ub514\ub85c \uc62e\uacbc\ub294\uac00?", "paragraph": "\ucca0\uc6d0\uc5d0\uc11c \uc989\uc704\ud55c \ub4a4 \ub2e4\uc2dc \uc218\ub3c4\ub97c \uc1a1\uc545(\u677e\u5dbd)\uc73c\ub85c \uc62e\uae30\uace0 \uc735\ud654\uc815\ucc45\u00b7\ubd81\uc9c4\uc815\ucc45\u00b7\uc22d\ubd88\uc815\ucc45\uc744 \ucd94\uc9c4\ud558\uc600\ub2e4. \ucca0\uc6d0 \uc9c0\uc5ed\uc5d0\uc11c\ub294 \uadf8\uc5d0 \ub300\ud574 \uac70\ubd80\uac10\uc744 \uac16\uace0 \uc788\uc5c8\uace0, \uadf8\ub294 \ubd88\uc548\uac10 \ub54c\ubb38\uc5d0 \uc1a1\ub3c4\ub85c \uc62e\uaca8\uac00\uac8c \ub41c\ub2e4. \ucca0\uc6d0\uc740 \uad81\uc608\uc758 \ud130\uc804\uc774\uc5c8\uae30\uc5d0 \ub300\ub2e4\uc218\uc758 \ucca0\uc6d0 \uc8fc\ubbfc\ub4e4\uc740 \uc655\uac74\uc5d0 \ub300\ud55c \ubc18\uac10\uc744 \uac00\uc9c0\uace0 \uc788\uc5c8\uace0 \uc774\ub7ec\ud55c \ubc18\uac10\uc740 \uc655\uad8c\uc744 \uc704\ud611\ud558\ub294 \uc694\uc18c\uc600\uae30\uc5d0 \uc655\uac74\uc740 \uc790\uc2e0\uc758 \uc9c0\uc9c0 \uae30\ubc18\uc774 \uc788\ub294 \uc1a1\uc545\uc73c\ub85c \ub3c4\uc131\uc744 \uc62e\uaca8 \uc655\uac74\uc744 \uc548\uc815\uc2dc\ud0a4\uace0 \ubbfc\uc2ec\uc744 \uc218\uc2b5\ud560 \uc694\ub7c9\uc774\uc5c8\ub2e4. \uadf8\ub9ac\uace0 \uace0\ud1a0 \ud68c\ubcf5\uc758 \uae30\uce58\ub97c \ub0b4\uac78\uace0 \uc11c\uacbd(\u897f\u4eac)\uc744 \uac1c\ucc99\ud558\uace0 \uc5ec\uc9c4\uc744 \uacf5\ub7b5\ud588\uc73c\uba70 \ubd88\uad50\ub97c \ud638\uad6d\uc2e0\uc559\uc73c\ub85c \uc0bc\uc544 \uac01\ucc98\uc5d0 \uc808\uc744 \uc138\uc6e0\ub2e4. \uc989\uc704 \ucd08\ubc18 \uc9c0\uc5ed \ud638\uc871\ub4e4\uc744 \ud68c\uc720\u00b7\ubb34\ub9c8\ud558\ub294 \ud55c\ud3b8, \ud638\uc871\ub4e4\uc758 \uc138\ub825\uc744 \uacac\uc81c\ud558\uae30 \uc704\ud574 \uadf8\ub294 \uac01\uc9c0 \ud638\uc871\uc758 \ub538\ub4e4\uc744 \ud6c4\uad81\ub85c \ub9de\uc774\ud558\uc600\uace0, \uc544\ub4e4\ub4e4\uc744 \ud2b9\ubcc4\ud788 \uc1a1\ub3c4\uc5d0\uc11c \uc218\ud559\ud558\uac8c \ud558\uc600\ub294\ub370 \uc774\ub294 \ud63c\uc778\uad00\uacc4\ub97c \ud1b5\ud574 \ud638\uc871\uc138\ub825\uc744 \ud1b5\ud569\ud558\ub294 \uac83\uacfc \uc790\uc81c\ub4e4\uc744 \ubcfc\ubaa8\ub85c \uc0bc\uc544 \uac01\uc9c0\uc758 \ud638\uc871\uc744 \uacac\uc81c\ud560 \ubaa9\uc801\uc774\uc5c8\ub2e4.", "answer": "\uc1a1\uc545", "sentence": "\ucca0\uc6d0\uc5d0\uc11c \uc989\uc704\ud55c \ub4a4 \ub2e4\uc2dc \uc218\ub3c4\ub97c \uc1a1\uc545 (\u677e\u5dbd)", "paragraph_sentence": " \ucca0\uc6d0\uc5d0\uc11c \uc989\uc704\ud55c \ub4a4 \ub2e4\uc2dc \uc218\ub3c4\ub97c \uc1a1\uc545 (\u677e\u5dbd) \uc73c\ub85c \uc62e\uae30\uace0 \uc735\ud654\uc815\ucc45\u00b7\ubd81\uc9c4\uc815\ucc45\u00b7\uc22d\ubd88\uc815\ucc45\uc744 \ucd94\uc9c4\ud558\uc600\ub2e4. \ucca0\uc6d0 \uc9c0\uc5ed\uc5d0\uc11c\ub294 \uadf8\uc5d0 \ub300\ud574 \uac70\ubd80\uac10\uc744 \uac16\uace0 \uc788\uc5c8\uace0, \uadf8\ub294 \ubd88\uc548\uac10 \ub54c\ubb38\uc5d0 \uc1a1\ub3c4\ub85c \uc62e\uaca8\uac00\uac8c \ub41c\ub2e4. \ucca0\uc6d0\uc740 \uad81\uc608\uc758 \ud130\uc804\uc774\uc5c8\uae30\uc5d0 \ub300\ub2e4\uc218\uc758 \ucca0\uc6d0 \uc8fc\ubbfc\ub4e4\uc740 \uc655\uac74\uc5d0 \ub300\ud55c \ubc18\uac10\uc744 \uac00\uc9c0\uace0 \uc788\uc5c8\uace0 \uc774\ub7ec\ud55c \ubc18\uac10\uc740 \uc655\uad8c\uc744 \uc704\ud611\ud558\ub294 \uc694\uc18c\uc600\uae30\uc5d0 \uc655\uac74\uc740 \uc790\uc2e0\uc758 \uc9c0\uc9c0 \uae30\ubc18\uc774 \uc788\ub294 \uc1a1\uc545\uc73c\ub85c \ub3c4\uc131\uc744 \uc62e\uaca8 \uc655\uac74\uc744 \uc548\uc815\uc2dc\ud0a4\uace0 \ubbfc\uc2ec\uc744 \uc218\uc2b5\ud560 \uc694\ub7c9\uc774\uc5c8\ub2e4. \uadf8\ub9ac\uace0 \uace0\ud1a0 \ud68c\ubcf5\uc758 \uae30\uce58\ub97c \ub0b4\uac78\uace0 \uc11c\uacbd(\u897f\u4eac)\uc744 \uac1c\ucc99\ud558\uace0 \uc5ec\uc9c4\uc744 \uacf5\ub7b5\ud588\uc73c\uba70 \ubd88\uad50\ub97c \ud638\uad6d\uc2e0\uc559\uc73c\ub85c \uc0bc\uc544 \uac01\ucc98\uc5d0 \uc808\uc744 \uc138\uc6e0\ub2e4. \uc989\uc704 \ucd08\ubc18 \uc9c0\uc5ed \ud638\uc871\ub4e4\uc744 \ud68c\uc720\u00b7\ubb34\ub9c8\ud558\ub294 \ud55c\ud3b8, \ud638\uc871\ub4e4\uc758 \uc138\ub825\uc744 \uacac\uc81c\ud558\uae30 \uc704\ud574 \uadf8\ub294 \uac01\uc9c0 \ud638\uc871\uc758 \ub538\ub4e4\uc744 \ud6c4\uad81\ub85c \ub9de\uc774\ud558\uc600\uace0, \uc544\ub4e4\ub4e4\uc744 \ud2b9\ubcc4\ud788 \uc1a1\ub3c4\uc5d0\uc11c \uc218\ud559\ud558\uac8c \ud558\uc600\ub294\ub370 \uc774\ub294 \ud63c\uc778\uad00\uacc4\ub97c \ud1b5\ud574 \ud638\uc871\uc138\ub825\uc744 \ud1b5\ud569\ud558\ub294 \uac83\uacfc \uc790\uc81c\ub4e4\uc744 \ubcfc\ubaa8\ub85c \uc0bc\uc544 \uac01\uc9c0\uc758 \ud638\uc871\uc744 \uacac\uc81c\ud560 \ubaa9\uc801\uc774\uc5c8\ub2e4.", "paragraph_answer": "\ucca0\uc6d0\uc5d0\uc11c \uc989\uc704\ud55c \ub4a4 \ub2e4\uc2dc \uc218\ub3c4\ub97c \uc1a1\uc545 (\u677e\u5dbd)\uc73c\ub85c \uc62e\uae30\uace0 \uc735\ud654\uc815\ucc45\u00b7\ubd81\uc9c4\uc815\ucc45\u00b7\uc22d\ubd88\uc815\ucc45\uc744 \ucd94\uc9c4\ud558\uc600\ub2e4. \ucca0\uc6d0 \uc9c0\uc5ed\uc5d0\uc11c\ub294 \uadf8\uc5d0 \ub300\ud574 \uac70\ubd80\uac10\uc744 \uac16\uace0 \uc788\uc5c8\uace0, \uadf8\ub294 \ubd88\uc548\uac10 \ub54c\ubb38\uc5d0 \uc1a1\ub3c4\ub85c \uc62e\uaca8\uac00\uac8c \ub41c\ub2e4. \ucca0\uc6d0\uc740 \uad81\uc608\uc758 \ud130\uc804\uc774\uc5c8\uae30\uc5d0 \ub300\ub2e4\uc218\uc758 \ucca0\uc6d0 \uc8fc\ubbfc\ub4e4\uc740 \uc655\uac74\uc5d0 \ub300\ud55c \ubc18\uac10\uc744 \uac00\uc9c0\uace0 \uc788\uc5c8\uace0 \uc774\ub7ec\ud55c \ubc18\uac10\uc740 \uc655\uad8c\uc744 \uc704\ud611\ud558\ub294 \uc694\uc18c\uc600\uae30\uc5d0 \uc655\uac74\uc740 \uc790\uc2e0\uc758 \uc9c0\uc9c0 \uae30\ubc18\uc774 \uc788\ub294 \uc1a1\uc545\uc73c\ub85c \ub3c4\uc131\uc744 \uc62e\uaca8 \uc655\uac74\uc744 \uc548\uc815\uc2dc\ud0a4\uace0 \ubbfc\uc2ec\uc744 \uc218\uc2b5\ud560 \uc694\ub7c9\uc774\uc5c8\ub2e4. \uadf8\ub9ac\uace0 \uace0\ud1a0 \ud68c\ubcf5\uc758 \uae30\uce58\ub97c \ub0b4\uac78\uace0 \uc11c\uacbd(\u897f\u4eac)\uc744 \uac1c\ucc99\ud558\uace0 \uc5ec\uc9c4\uc744 \uacf5\ub7b5\ud588\uc73c\uba70 \ubd88\uad50\ub97c \ud638\uad6d\uc2e0\uc559\uc73c\ub85c \uc0bc\uc544 \uac01\ucc98\uc5d0 \uc808\uc744 \uc138\uc6e0\ub2e4. \uc989\uc704 \ucd08\ubc18 \uc9c0\uc5ed \ud638\uc871\ub4e4\uc744 \ud68c\uc720\u00b7\ubb34\ub9c8\ud558\ub294 \ud55c\ud3b8, \ud638\uc871\ub4e4\uc758 \uc138\ub825\uc744 \uacac\uc81c\ud558\uae30 \uc704\ud574 \uadf8\ub294 \uac01\uc9c0 \ud638\uc871\uc758 \ub538\ub4e4\uc744 \ud6c4\uad81\ub85c \ub9de\uc774\ud558\uc600\uace0, \uc544\ub4e4\ub4e4\uc744 \ud2b9\ubcc4\ud788 \uc1a1\ub3c4\uc5d0\uc11c \uc218\ud559\ud558\uac8c \ud558\uc600\ub294\ub370 \uc774\ub294 \ud63c\uc778\uad00\uacc4\ub97c \ud1b5\ud574 \ud638\uc871\uc138\ub825\uc744 \ud1b5\ud569\ud558\ub294 \uac83\uacfc \uc790\uc81c\ub4e4\uc744 \ubcfc\ubaa8\ub85c \uc0bc\uc544 \uac01\uc9c0\uc758 \ud638\uc871\uc744 \uacac\uc81c\ud560 \ubaa9\uc801\uc774\uc5c8\ub2e4.", "sentence_answer": "\ucca0\uc6d0\uc5d0\uc11c \uc989\uc704\ud55c \ub4a4 \ub2e4\uc2dc \uc218\ub3c4\ub97c \uc1a1\uc545 (\u677e\u5dbd)", "paragraph_id": "6471291-7-0", "paragraph_question": "question: \uc655\uac74\uc740 \ucca0\uc6d0\uc5d0\uc11c \uc989\uc704\ud55c \ub4a4 \uc218\ub3c4\ub97c \uc5b4\ub514\ub85c \uc62e\uacbc\ub294\uac00?, context: \ucca0\uc6d0\uc5d0\uc11c \uc989\uc704\ud55c \ub4a4 \ub2e4\uc2dc \uc218\ub3c4\ub97c \uc1a1\uc545(\u677e\u5dbd)\uc73c\ub85c \uc62e\uae30\uace0 \uc735\ud654\uc815\ucc45\u00b7\ubd81\uc9c4\uc815\ucc45\u00b7\uc22d\ubd88\uc815\ucc45\uc744 \ucd94\uc9c4\ud558\uc600\ub2e4. \ucca0\uc6d0 \uc9c0\uc5ed\uc5d0\uc11c\ub294 \uadf8\uc5d0 \ub300\ud574 \uac70\ubd80\uac10\uc744 \uac16\uace0 \uc788\uc5c8\uace0, \uadf8\ub294 \ubd88\uc548\uac10 \ub54c\ubb38\uc5d0 \uc1a1\ub3c4\ub85c \uc62e\uaca8\uac00\uac8c \ub41c\ub2e4. \ucca0\uc6d0\uc740 \uad81\uc608\uc758 \ud130\uc804\uc774\uc5c8\uae30\uc5d0 \ub300\ub2e4\uc218\uc758 \ucca0\uc6d0 \uc8fc\ubbfc\ub4e4\uc740 \uc655\uac74\uc5d0 \ub300\ud55c \ubc18\uac10\uc744 \uac00\uc9c0\uace0 \uc788\uc5c8\uace0 \uc774\ub7ec\ud55c \ubc18\uac10\uc740 \uc655\uad8c\uc744 \uc704\ud611\ud558\ub294 \uc694\uc18c\uc600\uae30\uc5d0 \uc655\uac74\uc740 \uc790\uc2e0\uc758 \uc9c0\uc9c0 \uae30\ubc18\uc774 \uc788\ub294 \uc1a1\uc545\uc73c\ub85c \ub3c4\uc131\uc744 \uc62e\uaca8 \uc655\uac74\uc744 \uc548\uc815\uc2dc\ud0a4\uace0 \ubbfc\uc2ec\uc744 \uc218\uc2b5\ud560 \uc694\ub7c9\uc774\uc5c8\ub2e4. \uadf8\ub9ac\uace0 \uace0\ud1a0 \ud68c\ubcf5\uc758 \uae30\uce58\ub97c \ub0b4\uac78\uace0 \uc11c\uacbd(\u897f\u4eac)\uc744 \uac1c\ucc99\ud558\uace0 \uc5ec\uc9c4\uc744 \uacf5\ub7b5\ud588\uc73c\uba70 \ubd88\uad50\ub97c \ud638\uad6d\uc2e0\uc559\uc73c\ub85c \uc0bc\uc544 \uac01\ucc98\uc5d0 \uc808\uc744 \uc138\uc6e0\ub2e4. \uc989\uc704 \ucd08\ubc18 \uc9c0\uc5ed \ud638\uc871\ub4e4\uc744 \ud68c\uc720\u00b7\ubb34\ub9c8\ud558\ub294 \ud55c\ud3b8, \ud638\uc871\ub4e4\uc758 \uc138\ub825\uc744 \uacac\uc81c\ud558\uae30 \uc704\ud574 \uadf8\ub294 \uac01\uc9c0 \ud638\uc871\uc758 \ub538\ub4e4\uc744 \ud6c4\uad81\ub85c \ub9de\uc774\ud558\uc600\uace0, \uc544\ub4e4\ub4e4\uc744 \ud2b9\ubcc4\ud788 \uc1a1\ub3c4\uc5d0\uc11c \uc218\ud559\ud558\uac8c \ud558\uc600\ub294\ub370 \uc774\ub294 \ud63c\uc778\uad00\uacc4\ub97c \ud1b5\ud574 \ud638\uc871\uc138\ub825\uc744 \ud1b5\ud569\ud558\ub294 \uac83\uacfc \uc790\uc81c\ub4e4\uc744 \ubcfc\ubaa8\ub85c \uc0bc\uc544 \uac01\uc9c0\uc758 \ud638\uc871\uc744 \uacac\uc81c\ud560 \ubaa9\uc801\uc774\uc5c8\ub2e4."} {"question": "\uc5d0\ub77c\uc2a4\ubb88\uc2a4\uac00 \uc9c0\uc9c0\ud55c \ub85c\ub9c8 \uac00\ud1a8\ub9ad \uad50\ud68c\uc758 \uad00\ub150\uc740?", "paragraph": "\uc5d0\ub77c\uc2a4\ubb34\uc2a4\uac00 \uc0b4\uc558\ub358 \uc2dc\ub300\ub294 \ub9ce\uc740 \uc9c0\uc2dd\uc778\ub4e4\uc774 \uad50\ud68c \uc131\uc9c1\uc790\uc758 \uad8c\ud55c \ub0a8\uc6a9\uc744 \ube44\ud310\ud558\ub358 \ub54c\uc600\ub2e4. \uc5b4\ub5a4 \ube44\ud3c9\uac00\ub4e4\uc740 \uad50\ud669\uc758 \uad8c\ub2a5\uc744 \ubd80\uc815\ud588\uace0 \uc0c8\ub85c\uc6b4 \uc2e0\ud559 \uccb4\uacc4\ub97c \uc138\uc6e0\ub2e4. \uc5d0\ub77c\uc2a4\ubb88\uc2a4\ub3c4 \ub3d9\uc2dc\ub300 \uc8fc\ub958 \uae30\ub3c5\uad50\uc758 \uc2e0\uc559\uacfc \ubd80\uc815, \uad00\ud589\uc744 \ube44\ud310\ud558\ub358 \uad50\ud68c \uac1c\ud601\uac00 \uc911 \ud55c \uc0ac\ub78c\uc774\ub2e4. \ud55c\ud3b8 \uadf8\ub294 \ub85c\ub9c8 \uac00\ud1a8\ub9ad \uad50\ud68c\uc758 \uc790\uc720 \uc758\uc9c0(free will) \uad00\ub150\uc744 \uc9c0\uc9c0\ud588\ub2e4. \uadf8\ub798\uc11c \uadf8\ub294 \uc608\uc815\ub860\uc744 \uc9c0\uc9c0\ud558\ub294 \ub9ce\uc740 \uc885\uad50\uac1c\ud601\uac00\ub4e4\uacfc \ub9c8\ucc30\uc774 \uc788\uc5c8\ub2e4. \uadf8\uc758 \uc774\ub7ec\ud55c \uc911\ub9bd \ub178\uc120\uc740 \ub9c8\ub974\ud2f4 \ub8e8\ud130 \uac19\uc740 \uac1c\uc2e0\uad50 \uc885\uad50\uac1c\ud601\uc790\ub4e4\uacfc \uae09\uc9c4\uc801\uc778 \uad50\ud669\uc8fc\uc758\uc790 \uc591\ucabd\uc758 \uc2e4\ub9dd\uacfc \ubd84\ub178\ub97c \uc0c0\ub2e4. \ub85c\ub9c8 \uad50\ud669\uccad\uc758 \ubd80\uc815\ubd80\ud328\uc640 \uc131\uc9c1\uc790\uc758 \uad8c\ud55c \ub0a8\uc6a9\uc744 \ube44\ud310\ud588\uc9c0\ub9cc, \uc5d0\ub77c\uc2a4\ubb88\uc2a4\ub294 \uac1c\uc2e0\uad50\uc640 \ub85c\ub9c8 \uac00\ud1a8\ub9ad \uad50\ud68c \uc5b4\ub290 \ud55c \ucabd\uc73c\ub85c \uae30\uc6b8\uc9c0 \uc54a\uc558\ub2e4. \uc885\uad50\uac1c\ud601\uac00 \ub9c8\ub974\ud2f4 \ub8e8\ud130\ub294 \uadf8\ub97c \uacbd\uba78\uc870\ub85c \uc870\ub871\ud588\uace0, \ud2b8\ub9ac\uc5d4\ud2b8 \uacf5\uc758\ud68c\ub294 \uadf8\ub97c \u2018\ubd88\uacbd\uc2a4\ub7f0 \uc774\uad50\ub3c4\u2019\ub85c \uc815\uc8c4\ud588\ub2e4. \uc774\ucc98\ub7fc \uac1c\uc2e0\uad50\uc640 \ub85c\ub9c8 \uac00\ud1a8\ub9ad \uc591\ucabd \uad50\ud68c\ub4e4\ub85c\ubd80\ud130 \ubc30\ucc99\uc744 \ub2f9\ud588\uc9c0\ub9cc \uc5d0\ub77c\uc2a4\ubb88\uc2a4\ub294 \ub9ce\uc740 \ub2f9\ub300 \uc2e0\ud559\uc790\ub4e4\uc758 \uc9c0\uc9c0\ub97c \ubc1b\uc73c\uba70 \uc804 \uc720\ub7fd\uc5d0\uc11c \ub300\ub2e8\ud55c \uc778\uae30\ub97c \ub204\ub838\ub2e4. \ubfd0\ub9cc \uc544\ub2c8\ub77c \uc11c\uad6c \ubb38\ud559\uc0ac\uc5d0\uc11c \uadfc\ub300\uc131\uc758 \ud6a8\uc2dc\ub85c \uc5ec\uaca8\uc9c0\ub294 \ud504\ub791\uc2a4\uc758 \ub77c\ube14\ub808, \uc2a4\ud398\uc778\uc758 \uc138\ub974\ubc18\ud14c\uc2a4, \uc601\uad6d\uc758 \uc170\uc775\uc2a4\ud53c\uc5b4\uc5d0\uac8c\ub3c4 \uc9c0\ub300\ud55c \uc601\ud5a5\uc744 \ubbf8\ucce4\ub2e4. \uadf8\ub294 1536\ub144 \ubc14\uc824\uc5d0\uc11c \uc8fd\uc5c8\ub2e4.", "answer": "\uc790\uc720 \uc758\uc9c0", "sentence": "\uadf8\ub294 \ub85c\ub9c8 \uac00\ud1a8\ub9ad \uad50\ud68c\uc758 \uc790\uc720 \uc758\uc9c0 (free will) \uad00\ub150\uc744 \uc9c0\uc9c0\ud588\ub2e4.", "paragraph_sentence": "\uc5d0\ub77c\uc2a4\ubb34\uc2a4\uac00 \uc0b4\uc558\ub358 \uc2dc\ub300\ub294 \ub9ce\uc740 \uc9c0\uc2dd\uc778\ub4e4\uc774 \uad50\ud68c \uc131\uc9c1\uc790\uc758 \uad8c\ud55c \ub0a8\uc6a9\uc744 \ube44\ud310\ud558\ub358 \ub54c\uc600\ub2e4. \uc5b4\ub5a4 \ube44\ud3c9\uac00\ub4e4\uc740 \uad50\ud669\uc758 \uad8c\ub2a5\uc744 \ubd80\uc815\ud588\uace0 \uc0c8\ub85c\uc6b4 \uc2e0\ud559 \uccb4\uacc4\ub97c \uc138\uc6e0\ub2e4. \uc5d0\ub77c\uc2a4\ubb88\uc2a4\ub3c4 \ub3d9\uc2dc\ub300 \uc8fc\ub958 \uae30\ub3c5\uad50\uc758 \uc2e0\uc559\uacfc \ubd80\uc815, \uad00\ud589\uc744 \ube44\ud310\ud558\ub358 \uad50\ud68c \uac1c\ud601\uac00 \uc911 \ud55c \uc0ac\ub78c\uc774\ub2e4. \ud55c\ud3b8 \uadf8\ub294 \ub85c\ub9c8 \uac00\ud1a8\ub9ad \uad50\ud68c\uc758 \uc790\uc720 \uc758\uc9c0 (free will) \uad00\ub150\uc744 \uc9c0\uc9c0\ud588\ub2e4. \uadf8\ub798\uc11c \uadf8\ub294 \uc608\uc815\ub860\uc744 \uc9c0\uc9c0\ud558\ub294 \ub9ce\uc740 \uc885\uad50\uac1c\ud601\uac00\ub4e4\uacfc \ub9c8\ucc30\uc774 \uc788\uc5c8\ub2e4. \uadf8\uc758 \uc774\ub7ec\ud55c \uc911\ub9bd \ub178\uc120\uc740 \ub9c8\ub974\ud2f4 \ub8e8\ud130 \uac19\uc740 \uac1c\uc2e0\uad50 \uc885\uad50\uac1c\ud601\uc790\ub4e4\uacfc \uae09\uc9c4\uc801\uc778 \uad50\ud669\uc8fc\uc758\uc790 \uc591\ucabd\uc758 \uc2e4\ub9dd\uacfc \ubd84\ub178\ub97c \uc0c0\ub2e4. \ub85c\ub9c8 \uad50\ud669\uccad\uc758 \ubd80\uc815\ubd80\ud328\uc640 \uc131\uc9c1\uc790\uc758 \uad8c\ud55c \ub0a8\uc6a9\uc744 \ube44\ud310\ud588\uc9c0\ub9cc, \uc5d0\ub77c\uc2a4\ubb88\uc2a4\ub294 \uac1c\uc2e0\uad50\uc640 \ub85c\ub9c8 \uac00\ud1a8\ub9ad \uad50\ud68c \uc5b4\ub290 \ud55c \ucabd\uc73c\ub85c \uae30\uc6b8\uc9c0 \uc54a\uc558\ub2e4. \uc885\uad50\uac1c\ud601\uac00 \ub9c8\ub974\ud2f4 \ub8e8\ud130\ub294 \uadf8\ub97c \uacbd\uba78\uc870\ub85c \uc870\ub871\ud588\uace0, \ud2b8\ub9ac\uc5d4\ud2b8 \uacf5\uc758\ud68c\ub294 \uadf8\ub97c \u2018\ubd88\uacbd\uc2a4\ub7f0 \uc774\uad50\ub3c4\u2019\ub85c \uc815\uc8c4\ud588\ub2e4. \uc774\ucc98\ub7fc \uac1c\uc2e0\uad50\uc640 \ub85c\ub9c8 \uac00\ud1a8\ub9ad \uc591\ucabd \uad50\ud68c\ub4e4\ub85c\ubd80\ud130 \ubc30\ucc99\uc744 \ub2f9\ud588\uc9c0\ub9cc \uc5d0\ub77c\uc2a4\ubb88\uc2a4\ub294 \ub9ce\uc740 \ub2f9\ub300 \uc2e0\ud559\uc790\ub4e4\uc758 \uc9c0\uc9c0\ub97c \ubc1b\uc73c\uba70 \uc804 \uc720\ub7fd\uc5d0\uc11c \ub300\ub2e8\ud55c \uc778\uae30\ub97c \ub204\ub838\ub2e4. \ubfd0\ub9cc \uc544\ub2c8\ub77c \uc11c\uad6c \ubb38\ud559\uc0ac\uc5d0\uc11c \uadfc\ub300\uc131\uc758 \ud6a8\uc2dc\ub85c \uc5ec\uaca8\uc9c0\ub294 \ud504\ub791\uc2a4\uc758 \ub77c\ube14\ub808, \uc2a4\ud398\uc778\uc758 \uc138\ub974\ubc18\ud14c\uc2a4, \uc601\uad6d\uc758 \uc170\uc775\uc2a4\ud53c\uc5b4\uc5d0\uac8c\ub3c4 \uc9c0\ub300\ud55c \uc601\ud5a5\uc744 \ubbf8\ucce4\ub2e4. \uadf8\ub294 1536\ub144 \ubc14\uc824\uc5d0\uc11c \uc8fd\uc5c8\ub2e4.", "paragraph_answer": "\uc5d0\ub77c\uc2a4\ubb34\uc2a4\uac00 \uc0b4\uc558\ub358 \uc2dc\ub300\ub294 \ub9ce\uc740 \uc9c0\uc2dd\uc778\ub4e4\uc774 \uad50\ud68c \uc131\uc9c1\uc790\uc758 \uad8c\ud55c \ub0a8\uc6a9\uc744 \ube44\ud310\ud558\ub358 \ub54c\uc600\ub2e4. \uc5b4\ub5a4 \ube44\ud3c9\uac00\ub4e4\uc740 \uad50\ud669\uc758 \uad8c\ub2a5\uc744 \ubd80\uc815\ud588\uace0 \uc0c8\ub85c\uc6b4 \uc2e0\ud559 \uccb4\uacc4\ub97c \uc138\uc6e0\ub2e4. \uc5d0\ub77c\uc2a4\ubb88\uc2a4\ub3c4 \ub3d9\uc2dc\ub300 \uc8fc\ub958 \uae30\ub3c5\uad50\uc758 \uc2e0\uc559\uacfc \ubd80\uc815, \uad00\ud589\uc744 \ube44\ud310\ud558\ub358 \uad50\ud68c \uac1c\ud601\uac00 \uc911 \ud55c \uc0ac\ub78c\uc774\ub2e4. \ud55c\ud3b8 \uadf8\ub294 \ub85c\ub9c8 \uac00\ud1a8\ub9ad \uad50\ud68c\uc758 \uc790\uc720 \uc758\uc9c0 (free will) \uad00\ub150\uc744 \uc9c0\uc9c0\ud588\ub2e4. \uadf8\ub798\uc11c \uadf8\ub294 \uc608\uc815\ub860\uc744 \uc9c0\uc9c0\ud558\ub294 \ub9ce\uc740 \uc885\uad50\uac1c\ud601\uac00\ub4e4\uacfc \ub9c8\ucc30\uc774 \uc788\uc5c8\ub2e4. \uadf8\uc758 \uc774\ub7ec\ud55c \uc911\ub9bd \ub178\uc120\uc740 \ub9c8\ub974\ud2f4 \ub8e8\ud130 \uac19\uc740 \uac1c\uc2e0\uad50 \uc885\uad50\uac1c\ud601\uc790\ub4e4\uacfc \uae09\uc9c4\uc801\uc778 \uad50\ud669\uc8fc\uc758\uc790 \uc591\ucabd\uc758 \uc2e4\ub9dd\uacfc \ubd84\ub178\ub97c \uc0c0\ub2e4. \ub85c\ub9c8 \uad50\ud669\uccad\uc758 \ubd80\uc815\ubd80\ud328\uc640 \uc131\uc9c1\uc790\uc758 \uad8c\ud55c \ub0a8\uc6a9\uc744 \ube44\ud310\ud588\uc9c0\ub9cc, \uc5d0\ub77c\uc2a4\ubb88\uc2a4\ub294 \uac1c\uc2e0\uad50\uc640 \ub85c\ub9c8 \uac00\ud1a8\ub9ad \uad50\ud68c \uc5b4\ub290 \ud55c \ucabd\uc73c\ub85c \uae30\uc6b8\uc9c0 \uc54a\uc558\ub2e4. \uc885\uad50\uac1c\ud601\uac00 \ub9c8\ub974\ud2f4 \ub8e8\ud130\ub294 \uadf8\ub97c \uacbd\uba78\uc870\ub85c \uc870\ub871\ud588\uace0, \ud2b8\ub9ac\uc5d4\ud2b8 \uacf5\uc758\ud68c\ub294 \uadf8\ub97c \u2018\ubd88\uacbd\uc2a4\ub7f0 \uc774\uad50\ub3c4\u2019\ub85c \uc815\uc8c4\ud588\ub2e4. \uc774\ucc98\ub7fc \uac1c\uc2e0\uad50\uc640 \ub85c\ub9c8 \uac00\ud1a8\ub9ad \uc591\ucabd \uad50\ud68c\ub4e4\ub85c\ubd80\ud130 \ubc30\ucc99\uc744 \ub2f9\ud588\uc9c0\ub9cc \uc5d0\ub77c\uc2a4\ubb88\uc2a4\ub294 \ub9ce\uc740 \ub2f9\ub300 \uc2e0\ud559\uc790\ub4e4\uc758 \uc9c0\uc9c0\ub97c \ubc1b\uc73c\uba70 \uc804 \uc720\ub7fd\uc5d0\uc11c \ub300\ub2e8\ud55c \uc778\uae30\ub97c \ub204\ub838\ub2e4. \ubfd0\ub9cc \uc544\ub2c8\ub77c \uc11c\uad6c \ubb38\ud559\uc0ac\uc5d0\uc11c \uadfc\ub300\uc131\uc758 \ud6a8\uc2dc\ub85c \uc5ec\uaca8\uc9c0\ub294 \ud504\ub791\uc2a4\uc758 \ub77c\ube14\ub808, \uc2a4\ud398\uc778\uc758 \uc138\ub974\ubc18\ud14c\uc2a4, \uc601\uad6d\uc758 \uc170\uc775\uc2a4\ud53c\uc5b4\uc5d0\uac8c\ub3c4 \uc9c0\ub300\ud55c \uc601\ud5a5\uc744 \ubbf8\ucce4\ub2e4. \uadf8\ub294 1536\ub144 \ubc14\uc824\uc5d0\uc11c \uc8fd\uc5c8\ub2e4.", "sentence_answer": "\uadf8\ub294 \ub85c\ub9c8 \uac00\ud1a8\ub9ad \uad50\ud68c\uc758 \uc790\uc720 \uc758\uc9c0 (free will) \uad00\ub150\uc744 \uc9c0\uc9c0\ud588\ub2e4.", "paragraph_id": "6404155-0-0", "paragraph_question": "question: \uc5d0\ub77c\uc2a4\ubb88\uc2a4\uac00 \uc9c0\uc9c0\ud55c \ub85c\ub9c8 \uac00\ud1a8\ub9ad \uad50\ud68c\uc758 \uad00\ub150\uc740?, context: \uc5d0\ub77c\uc2a4\ubb34\uc2a4\uac00 \uc0b4\uc558\ub358 \uc2dc\ub300\ub294 \ub9ce\uc740 \uc9c0\uc2dd\uc778\ub4e4\uc774 \uad50\ud68c \uc131\uc9c1\uc790\uc758 \uad8c\ud55c \ub0a8\uc6a9\uc744 \ube44\ud310\ud558\ub358 \ub54c\uc600\ub2e4. \uc5b4\ub5a4 \ube44\ud3c9\uac00\ub4e4\uc740 \uad50\ud669\uc758 \uad8c\ub2a5\uc744 \ubd80\uc815\ud588\uace0 \uc0c8\ub85c\uc6b4 \uc2e0\ud559 \uccb4\uacc4\ub97c \uc138\uc6e0\ub2e4. \uc5d0\ub77c\uc2a4\ubb88\uc2a4\ub3c4 \ub3d9\uc2dc\ub300 \uc8fc\ub958 \uae30\ub3c5\uad50\uc758 \uc2e0\uc559\uacfc \ubd80\uc815, \uad00\ud589\uc744 \ube44\ud310\ud558\ub358 \uad50\ud68c \uac1c\ud601\uac00 \uc911 \ud55c \uc0ac\ub78c\uc774\ub2e4. \ud55c\ud3b8 \uadf8\ub294 \ub85c\ub9c8 \uac00\ud1a8\ub9ad \uad50\ud68c\uc758 \uc790\uc720 \uc758\uc9c0(free will) \uad00\ub150\uc744 \uc9c0\uc9c0\ud588\ub2e4. \uadf8\ub798\uc11c \uadf8\ub294 \uc608\uc815\ub860\uc744 \uc9c0\uc9c0\ud558\ub294 \ub9ce\uc740 \uc885\uad50\uac1c\ud601\uac00\ub4e4\uacfc \ub9c8\ucc30\uc774 \uc788\uc5c8\ub2e4. \uadf8\uc758 \uc774\ub7ec\ud55c \uc911\ub9bd \ub178\uc120\uc740 \ub9c8\ub974\ud2f4 \ub8e8\ud130 \uac19\uc740 \uac1c\uc2e0\uad50 \uc885\uad50\uac1c\ud601\uc790\ub4e4\uacfc \uae09\uc9c4\uc801\uc778 \uad50\ud669\uc8fc\uc758\uc790 \uc591\ucabd\uc758 \uc2e4\ub9dd\uacfc \ubd84\ub178\ub97c \uc0c0\ub2e4. \ub85c\ub9c8 \uad50\ud669\uccad\uc758 \ubd80\uc815\ubd80\ud328\uc640 \uc131\uc9c1\uc790\uc758 \uad8c\ud55c \ub0a8\uc6a9\uc744 \ube44\ud310\ud588\uc9c0\ub9cc, \uc5d0\ub77c\uc2a4\ubb88\uc2a4\ub294 \uac1c\uc2e0\uad50\uc640 \ub85c\ub9c8 \uac00\ud1a8\ub9ad \uad50\ud68c \uc5b4\ub290 \ud55c \ucabd\uc73c\ub85c \uae30\uc6b8\uc9c0 \uc54a\uc558\ub2e4. \uc885\uad50\uac1c\ud601\uac00 \ub9c8\ub974\ud2f4 \ub8e8\ud130\ub294 \uadf8\ub97c \uacbd\uba78\uc870\ub85c \uc870\ub871\ud588\uace0, \ud2b8\ub9ac\uc5d4\ud2b8 \uacf5\uc758\ud68c\ub294 \uadf8\ub97c \u2018\ubd88\uacbd\uc2a4\ub7f0 \uc774\uad50\ub3c4\u2019\ub85c \uc815\uc8c4\ud588\ub2e4. \uc774\ucc98\ub7fc \uac1c\uc2e0\uad50\uc640 \ub85c\ub9c8 \uac00\ud1a8\ub9ad \uc591\ucabd \uad50\ud68c\ub4e4\ub85c\ubd80\ud130 \ubc30\ucc99\uc744 \ub2f9\ud588\uc9c0\ub9cc \uc5d0\ub77c\uc2a4\ubb88\uc2a4\ub294 \ub9ce\uc740 \ub2f9\ub300 \uc2e0\ud559\uc790\ub4e4\uc758 \uc9c0\uc9c0\ub97c \ubc1b\uc73c\uba70 \uc804 \uc720\ub7fd\uc5d0\uc11c \ub300\ub2e8\ud55c \uc778\uae30\ub97c \ub204\ub838\ub2e4. \ubfd0\ub9cc \uc544\ub2c8\ub77c \uc11c\uad6c \ubb38\ud559\uc0ac\uc5d0\uc11c \uadfc\ub300\uc131\uc758 \ud6a8\uc2dc\ub85c \uc5ec\uaca8\uc9c0\ub294 \ud504\ub791\uc2a4\uc758 \ub77c\ube14\ub808, \uc2a4\ud398\uc778\uc758 \uc138\ub974\ubc18\ud14c\uc2a4, \uc601\uad6d\uc758 \uc170\uc775\uc2a4\ud53c\uc5b4\uc5d0\uac8c\ub3c4 \uc9c0\ub300\ud55c \uc601\ud5a5\uc744 \ubbf8\ucce4\ub2e4. \uadf8\ub294 1536\ub144 \ubc14\uc824\uc5d0\uc11c \uc8fd\uc5c8\ub2e4."} {"question": "\ub4dc\ub808\ud4cc\uc2a4\uc758 \ubb34\uc8c4\ub294 \uc5b8\uc81c \uc120\uace0\ub418\uc5c8\ub098?", "paragraph": "\ub2e8, \uc120\uace0 \ud6c4 \ub300\ud1b5\ub839\uc774 \uc0ac\uba74\ud574\uc11c \uac10\uc625\uc5d0\uc11c\ub294 \ud480\ub824\ub098\uc654\uc9c0\ub9cc \ubcf5\uad8c\uc740 \ub418\uc9c0 \uc54a\uc558\ub2e4.\uadf8\ub7ec\ub098 \uc120\uac70\uc5d0\uc11c \uad70\uc0ac\uce21\uc774\uc600\ub358 \uc6b0\ud30c\uac00 \uc9c0\uc2dd\uc778 \uce21\uc778 \uc88c\ud30c\uc5d0 \ud328\ubc30, \uc989\uc2dc 1904\ub144\uc5d0 \uc7ac\uc2ec\uc774 \uccad\uad6c\ub418\uc5c8\uace0 1906\ub144\uc5d0 \ub4dc\ub808\ud4cc\uc2a4\uc758 \ubb34\uc8c4\uac00 \uc120\uace0\ub418\uc5b4 \ubaa8\ub4e0 \ud610\uc758\ub97c \ubc97\uace0, \ubcf5\uad8c\ub3c4 \ub418\uc5b4 \uc721\uad70\uc5d0 \ubcf5\uc9c1\ud588\ub2e4. \uadf8 \uc774\ud6c4 \uc18c\ub839\uc73c\ub85c \uc2b9\uc9c4\ud568\uc740 \ubb3c\ub860, \ub808\uc9c0\uc639 \ub3c4\ub1cc\ub974 \ud6c8\uc7a5\uae4c\uc9c0 \ubc1b\uac8c \ub41c\ub2e4. \uc81c1\ucc28 \uc138\uacc4 \ub300\uc804\uc5d0\ub3c4 \ucc38\uc804\ud558\uace0 \uc9c4\uae09\ub3c4 \ud558\uace0 \uc77c\ubc18 \uad70\uc778\ub4e4\ucc98\ub7fc \uc0dd\ud65c\ud558\ub2e4\uac00 1935\ub144\uc5d0 \uc9c0\ubcd1\uc73c\ub85c \ubcc4\uc138\ud588\ub2e4.\uadf8\ub7ec\ub098 \uc774 \uc0ac\uac74\uc740 \ub4dc\ub808\ud4cc\uc2a4 \uc790\uc2e0\uc5d0 \uc0b6\uc5d0 \ub108\ubb34\ub098\ub3c4 \ud070 \uc0c1\ucc98\ub97c \uc785\uc5c8\uace0, \uadf8 \ud6c4\uc5d0\ub3c4 \uc6b0\ud30c\ub294 \ub05d\uae4c\uc9c0 \ub4dc\ub808\ud4cc\uc2a4\uc758 \uc720\uc8c4\ub97c \uc678\ucce4\ub2e4. \uadf8\ub7ec\uba74\uc11c \ub4dc\ub808\ud4cc\uc2a4\uc758 \uc0c1\ucc98\ub294 \ub354 \uc2ec\ud574\uc9c0\ub294 \uadf8 \ub2f9\uc2dc \ubc18\uc720\ub300\uc8fc\uc758\uc640 \ubc18\ub3c4\uc774\uce60\ub780\ub4dc \uc8fc\uc758\uac00 \ub0b3\uc740 \ucd5c\uc545\uc758 \uacb0\uacfc\uc600\ub2e4.", "answer": "1906\ub144", "sentence": "\ub2e8, \uc120\uace0 \ud6c4 \ub300\ud1b5\ub839\uc774 \uc0ac\uba74\ud574\uc11c \uac10\uc625\uc5d0\uc11c\ub294 \ud480\ub824\ub098\uc654\uc9c0\ub9cc \ubcf5\uad8c\uc740 \ub418\uc9c0 \uc54a\uc558\ub2e4.\uadf8\ub7ec\ub098 \uc120\uac70\uc5d0\uc11c \uad70\uc0ac\uce21\uc774\uc600\ub358 \uc6b0\ud30c\uac00 \uc9c0\uc2dd\uc778 \uce21\uc778 \uc88c\ud30c\uc5d0 \ud328\ubc30, \uc989\uc2dc 1904\ub144\uc5d0 \uc7ac\uc2ec\uc774 \uccad\uad6c\ub418\uc5c8\uace0 1906\ub144 \uc5d0 \ub4dc\ub808\ud4cc\uc2a4\uc758 \ubb34\uc8c4\uac00 \uc120\uace0\ub418\uc5b4 \ubaa8\ub4e0 \ud610\uc758\ub97c \ubc97\uace0, \ubcf5\uad8c\ub3c4 \ub418\uc5b4 \uc721\uad70\uc5d0 \ubcf5\uc9c1\ud588\ub2e4.", "paragraph_sentence": " \ub2e8, \uc120\uace0 \ud6c4 \ub300\ud1b5\ub839\uc774 \uc0ac\uba74\ud574\uc11c \uac10\uc625\uc5d0\uc11c\ub294 \ud480\ub824\ub098\uc654\uc9c0\ub9cc \ubcf5\uad8c\uc740 \ub418\uc9c0 \uc54a\uc558\ub2e4.\uadf8\ub7ec\ub098 \uc120\uac70\uc5d0\uc11c \uad70\uc0ac\uce21\uc774\uc600\ub358 \uc6b0\ud30c\uac00 \uc9c0\uc2dd\uc778 \uce21\uc778 \uc88c\ud30c\uc5d0 \ud328\ubc30, \uc989\uc2dc 1904\ub144\uc5d0 \uc7ac\uc2ec\uc774 \uccad\uad6c\ub418\uc5c8\uace0 1906\ub144 \uc5d0 \ub4dc\ub808\ud4cc\uc2a4\uc758 \ubb34\uc8c4\uac00 \uc120\uace0\ub418\uc5b4 \ubaa8\ub4e0 \ud610\uc758\ub97c \ubc97\uace0, \ubcf5\uad8c\ub3c4 \ub418\uc5b4 \uc721\uad70\uc5d0 \ubcf5\uc9c1\ud588\ub2e4. \uadf8 \uc774\ud6c4 \uc18c\ub839\uc73c\ub85c \uc2b9\uc9c4\ud568\uc740 \ubb3c\ub860, \ub808\uc9c0\uc639 \ub3c4\ub1cc\ub974 \ud6c8\uc7a5\uae4c\uc9c0 \ubc1b\uac8c \ub41c\ub2e4. \uc81c1\ucc28 \uc138\uacc4 \ub300\uc804\uc5d0\ub3c4 \ucc38\uc804\ud558\uace0 \uc9c4\uae09\ub3c4 \ud558\uace0 \uc77c\ubc18 \uad70\uc778\ub4e4\ucc98\ub7fc \uc0dd\ud65c\ud558\ub2e4\uac00 1935\ub144\uc5d0 \uc9c0\ubcd1\uc73c\ub85c \ubcc4\uc138\ud588\ub2e4.\uadf8\ub7ec\ub098 \uc774 \uc0ac\uac74\uc740 \ub4dc\ub808\ud4cc\uc2a4 \uc790\uc2e0\uc5d0 \uc0b6\uc5d0 \ub108\ubb34\ub098\ub3c4 \ud070 \uc0c1\ucc98\ub97c \uc785\uc5c8\uace0, \uadf8 \ud6c4\uc5d0\ub3c4 \uc6b0\ud30c\ub294 \ub05d\uae4c\uc9c0 \ub4dc\ub808\ud4cc\uc2a4\uc758 \uc720\uc8c4\ub97c \uc678\ucce4\ub2e4. \uadf8\ub7ec\uba74\uc11c \ub4dc\ub808\ud4cc\uc2a4\uc758 \uc0c1\ucc98\ub294 \ub354 \uc2ec\ud574\uc9c0\ub294 \uadf8 \ub2f9\uc2dc \ubc18\uc720\ub300\uc8fc\uc758\uc640 \ubc18\ub3c4\uc774\uce60\ub780\ub4dc \uc8fc\uc758\uac00 \ub0b3\uc740 \ucd5c\uc545\uc758 \uacb0\uacfc\uc600\ub2e4.", "paragraph_answer": "\ub2e8, \uc120\uace0 \ud6c4 \ub300\ud1b5\ub839\uc774 \uc0ac\uba74\ud574\uc11c \uac10\uc625\uc5d0\uc11c\ub294 \ud480\ub824\ub098\uc654\uc9c0\ub9cc \ubcf5\uad8c\uc740 \ub418\uc9c0 \uc54a\uc558\ub2e4.\uadf8\ub7ec\ub098 \uc120\uac70\uc5d0\uc11c \uad70\uc0ac\uce21\uc774\uc600\ub358 \uc6b0\ud30c\uac00 \uc9c0\uc2dd\uc778 \uce21\uc778 \uc88c\ud30c\uc5d0 \ud328\ubc30, \uc989\uc2dc 1904\ub144\uc5d0 \uc7ac\uc2ec\uc774 \uccad\uad6c\ub418\uc5c8\uace0 1906\ub144 \uc5d0 \ub4dc\ub808\ud4cc\uc2a4\uc758 \ubb34\uc8c4\uac00 \uc120\uace0\ub418\uc5b4 \ubaa8\ub4e0 \ud610\uc758\ub97c \ubc97\uace0, \ubcf5\uad8c\ub3c4 \ub418\uc5b4 \uc721\uad70\uc5d0 \ubcf5\uc9c1\ud588\ub2e4. \uadf8 \uc774\ud6c4 \uc18c\ub839\uc73c\ub85c \uc2b9\uc9c4\ud568\uc740 \ubb3c\ub860, \ub808\uc9c0\uc639 \ub3c4\ub1cc\ub974 \ud6c8\uc7a5\uae4c\uc9c0 \ubc1b\uac8c \ub41c\ub2e4. \uc81c1\ucc28 \uc138\uacc4 \ub300\uc804\uc5d0\ub3c4 \ucc38\uc804\ud558\uace0 \uc9c4\uae09\ub3c4 \ud558\uace0 \uc77c\ubc18 \uad70\uc778\ub4e4\ucc98\ub7fc \uc0dd\ud65c\ud558\ub2e4\uac00 1935\ub144\uc5d0 \uc9c0\ubcd1\uc73c\ub85c \ubcc4\uc138\ud588\ub2e4.\uadf8\ub7ec\ub098 \uc774 \uc0ac\uac74\uc740 \ub4dc\ub808\ud4cc\uc2a4 \uc790\uc2e0\uc5d0 \uc0b6\uc5d0 \ub108\ubb34\ub098\ub3c4 \ud070 \uc0c1\ucc98\ub97c \uc785\uc5c8\uace0, \uadf8 \ud6c4\uc5d0\ub3c4 \uc6b0\ud30c\ub294 \ub05d\uae4c\uc9c0 \ub4dc\ub808\ud4cc\uc2a4\uc758 \uc720\uc8c4\ub97c \uc678\ucce4\ub2e4. \uadf8\ub7ec\uba74\uc11c \ub4dc\ub808\ud4cc\uc2a4\uc758 \uc0c1\ucc98\ub294 \ub354 \uc2ec\ud574\uc9c0\ub294 \uadf8 \ub2f9\uc2dc \ubc18\uc720\ub300\uc8fc\uc758\uc640 \ubc18\ub3c4\uc774\uce60\ub780\ub4dc \uc8fc\uc758\uac00 \ub0b3\uc740 \ucd5c\uc545\uc758 \uacb0\uacfc\uc600\ub2e4.", "sentence_answer": "\ub2e8, \uc120\uace0 \ud6c4 \ub300\ud1b5\ub839\uc774 \uc0ac\uba74\ud574\uc11c \uac10\uc625\uc5d0\uc11c\ub294 \ud480\ub824\ub098\uc654\uc9c0\ub9cc \ubcf5\uad8c\uc740 \ub418\uc9c0 \uc54a\uc558\ub2e4.\uadf8\ub7ec\ub098 \uc120\uac70\uc5d0\uc11c \uad70\uc0ac\uce21\uc774\uc600\ub358 \uc6b0\ud30c\uac00 \uc9c0\uc2dd\uc778 \uce21\uc778 \uc88c\ud30c\uc5d0 \ud328\ubc30, \uc989\uc2dc 1904\ub144\uc5d0 \uc7ac\uc2ec\uc774 \uccad\uad6c\ub418\uc5c8\uace0 1906\ub144 \uc5d0 \ub4dc\ub808\ud4cc\uc2a4\uc758 \ubb34\uc8c4\uac00 \uc120\uace0\ub418\uc5b4 \ubaa8\ub4e0 \ud610\uc758\ub97c \ubc97\uace0, \ubcf5\uad8c\ub3c4 \ub418\uc5b4 \uc721\uad70\uc5d0 \ubcf5\uc9c1\ud588\ub2e4.", "paragraph_id": "6579200-3-2", "paragraph_question": "question: \ub4dc\ub808\ud4cc\uc2a4\uc758 \ubb34\uc8c4\ub294 \uc5b8\uc81c \uc120\uace0\ub418\uc5c8\ub098?, context: \ub2e8, \uc120\uace0 \ud6c4 \ub300\ud1b5\ub839\uc774 \uc0ac\uba74\ud574\uc11c \uac10\uc625\uc5d0\uc11c\ub294 \ud480\ub824\ub098\uc654\uc9c0\ub9cc \ubcf5\uad8c\uc740 \ub418\uc9c0 \uc54a\uc558\ub2e4.\uadf8\ub7ec\ub098 \uc120\uac70\uc5d0\uc11c \uad70\uc0ac\uce21\uc774\uc600\ub358 \uc6b0\ud30c\uac00 \uc9c0\uc2dd\uc778 \uce21\uc778 \uc88c\ud30c\uc5d0 \ud328\ubc30, \uc989\uc2dc 1904\ub144\uc5d0 \uc7ac\uc2ec\uc774 \uccad\uad6c\ub418\uc5c8\uace0 1906\ub144\uc5d0 \ub4dc\ub808\ud4cc\uc2a4\uc758 \ubb34\uc8c4\uac00 \uc120\uace0\ub418\uc5b4 \ubaa8\ub4e0 \ud610\uc758\ub97c \ubc97\uace0, \ubcf5\uad8c\ub3c4 \ub418\uc5b4 \uc721\uad70\uc5d0 \ubcf5\uc9c1\ud588\ub2e4. \uadf8 \uc774\ud6c4 \uc18c\ub839\uc73c\ub85c \uc2b9\uc9c4\ud568\uc740 \ubb3c\ub860, \ub808\uc9c0\uc639 \ub3c4\ub1cc\ub974 \ud6c8\uc7a5\uae4c\uc9c0 \ubc1b\uac8c \ub41c\ub2e4. \uc81c1\ucc28 \uc138\uacc4 \ub300\uc804\uc5d0\ub3c4 \ucc38\uc804\ud558\uace0 \uc9c4\uae09\ub3c4 \ud558\uace0 \uc77c\ubc18 \uad70\uc778\ub4e4\ucc98\ub7fc \uc0dd\ud65c\ud558\ub2e4\uac00 1935\ub144\uc5d0 \uc9c0\ubcd1\uc73c\ub85c \ubcc4\uc138\ud588\ub2e4.\uadf8\ub7ec\ub098 \uc774 \uc0ac\uac74\uc740 \ub4dc\ub808\ud4cc\uc2a4 \uc790\uc2e0\uc5d0 \uc0b6\uc5d0 \ub108\ubb34\ub098\ub3c4 \ud070 \uc0c1\ucc98\ub97c \uc785\uc5c8\uace0, \uadf8 \ud6c4\uc5d0\ub3c4 \uc6b0\ud30c\ub294 \ub05d\uae4c\uc9c0 \ub4dc\ub808\ud4cc\uc2a4\uc758 \uc720\uc8c4\ub97c \uc678\ucce4\ub2e4. \uadf8\ub7ec\uba74\uc11c \ub4dc\ub808\ud4cc\uc2a4\uc758 \uc0c1\ucc98\ub294 \ub354 \uc2ec\ud574\uc9c0\ub294 \uadf8 \ub2f9\uc2dc \ubc18\uc720\ub300\uc8fc\uc758\uc640 \ubc18\ub3c4\uc774\uce60\ub780\ub4dc \uc8fc\uc758\uac00 \ub0b3\uc740 \ucd5c\uc545\uc758 \uacb0\uacfc\uc600\ub2e4."} {"question": "\uc6e8\uc77c\uc2a4\uc758 \ubd81\ubd80 \ubc0f \uc911\ubd80 \uc9c0\uc5ed\uc758 \ucd5c\uace0\ubd09\uc740 \ubb34\uc5c7\uc77c\uae4c?", "paragraph": "\uc6e8\uc77c\uc2a4(\uc601\uc5b4: Wales /\u02c8we\u026alz/, \uc6e8\uc77c\uc2a4\uc5b4: Cymru \ucef4\ub9ac [\u02c8k\u0259mri] ( \ub4e3\uae30))\ub294 \uc601\uad6d\uc758 \uad6c\uc131\uad6d \uc911 \ud558\ub098\uc774\uba70, \uadf8\ub808\uc774\ud2b8\ube0c\ub9ac\ud2bc \uc12c \uc11c\ubd80\uc5d0 \uc704\uce58\ud55c\ub2e4. \ub3d9\ucabd\uc73c\ub85c\ub294 \uc789\uae00\ub79c\ub4dc\uc640 \uc811\ud558\uace0 \uc11c\ucabd\uc73c\ub85c\ub294 \uc544\uc77c\ub79c\ub4dc \ud574\uc5d0 \uc811\ud558\uba70, \ub0a8\ucabd\uc73c\ub85c\ub294 \ube0c\ub9ac\uc2a4\ud1a8 \ucc44\ub110\uacfc \uc811\ud55c\ub2e4. 2011\ub144 \uae30\uc900 \uc778\uad6c\ub294 3,043,456\uba85\uc774\uace0, \ucd1d \uba74\uc801\uc740 20,779 km(8,023 sq mi)\uc774\ub2e4. \uc6e8\uc77c\uc2a4\ub294 2,700 km(1,680 \ub9c8\uc77c)\uc758 \ud574\uc548\uc120\uc744 \ubcf4\uc720\ud558\uace0 \uc788\uc73c\uba70 \ub300\ubd80\ubd84 \uc0b0\uc545 \uc9c0\ud615\uc774\ub2e4. \ubd81\ubd80 \ubc0f \uc911\ubd80 \uc9c0\uc5ed\uc758 \ub192\uc740 \ubd09\uc6b0\ub9ac\ub294 \uc2a4\ub178\ub4e0 \uc0b0(Snowdon, Yr Wyddfa)\uc774 \ucd5c\uace0\ubd09\uc774\ub2e4. \uc6e8\uc77c\uc2a4\ub294 \ubd81\ucabd \uc628\ub300 \uc9c0\uc5ed\uc5d0 \uc18d\ud574 \uc788\uc73c\uba70 \ubcc0\ud654\ud558\ub294 \ud574\uc591 \ud658\uacbd\uc744 \uac00\uc9c0\uace0 \uc788\ub2e4.", "answer": "\uc2a4\ub178\ub4e0 \uc0b0", "sentence": "\ubd81\ubd80 \ubc0f \uc911\ubd80 \uc9c0\uc5ed\uc758 \ub192\uc740 \ubd09\uc6b0\ub9ac\ub294 \uc2a4\ub178\ub4e0 \uc0b0 (Snowdon, Yr Wyddfa)", "paragraph_sentence": "\uc6e8\uc77c\uc2a4(\uc601\uc5b4: Wales /\u02c8we\u026alz/, \uc6e8\uc77c\uc2a4\uc5b4: Cymru \ucef4\ub9ac [\u02c8k\u0259mri] ( \ub4e3\uae30))\ub294 \uc601\uad6d\uc758 \uad6c\uc131\uad6d \uc911 \ud558\ub098\uc774\uba70, \uadf8\ub808\uc774\ud2b8\ube0c\ub9ac\ud2bc \uc12c \uc11c\ubd80\uc5d0 \uc704\uce58\ud55c\ub2e4. \ub3d9\ucabd\uc73c\ub85c\ub294 \uc789\uae00\ub79c\ub4dc\uc640 \uc811\ud558\uace0 \uc11c\ucabd\uc73c\ub85c\ub294 \uc544\uc77c\ub79c\ub4dc \ud574\uc5d0 \uc811\ud558\uba70, \ub0a8\ucabd\uc73c\ub85c\ub294 \ube0c\ub9ac\uc2a4\ud1a8 \ucc44\ub110\uacfc \uc811\ud55c\ub2e4. 2011\ub144 \uae30\uc900 \uc778\uad6c\ub294 3,043,456\uba85\uc774\uace0, \ucd1d \uba74\uc801\uc740 20,779 km(8,023 sq mi)\uc774\ub2e4. \uc6e8\uc77c\uc2a4\ub294 2,700 km(1,680 \ub9c8\uc77c)\uc758 \ud574\uc548\uc120\uc744 \ubcf4\uc720\ud558\uace0 \uc788\uc73c\uba70 \ub300\ubd80\ubd84 \uc0b0\uc545 \uc9c0\ud615\uc774\ub2e4. \ubd81\ubd80 \ubc0f \uc911\ubd80 \uc9c0\uc5ed\uc758 \ub192\uc740 \ubd09\uc6b0\ub9ac\ub294 \uc2a4\ub178\ub4e0 \uc0b0 (Snowdon, Yr Wyddfa) \uc774 \ucd5c\uace0\ubd09\uc774\ub2e4. \uc6e8\uc77c\uc2a4\ub294 \ubd81\ucabd \uc628\ub300 \uc9c0\uc5ed\uc5d0 \uc18d\ud574 \uc788\uc73c\uba70 \ubcc0\ud654\ud558\ub294 \ud574\uc591 \ud658\uacbd\uc744 \uac00\uc9c0\uace0 \uc788\ub2e4.", "paragraph_answer": "\uc6e8\uc77c\uc2a4(\uc601\uc5b4: Wales /\u02c8we\u026alz/, \uc6e8\uc77c\uc2a4\uc5b4: Cymru \ucef4\ub9ac [\u02c8k\u0259mri] ( \ub4e3\uae30))\ub294 \uc601\uad6d\uc758 \uad6c\uc131\uad6d \uc911 \ud558\ub098\uc774\uba70, \uadf8\ub808\uc774\ud2b8\ube0c\ub9ac\ud2bc \uc12c \uc11c\ubd80\uc5d0 \uc704\uce58\ud55c\ub2e4. \ub3d9\ucabd\uc73c\ub85c\ub294 \uc789\uae00\ub79c\ub4dc\uc640 \uc811\ud558\uace0 \uc11c\ucabd\uc73c\ub85c\ub294 \uc544\uc77c\ub79c\ub4dc \ud574\uc5d0 \uc811\ud558\uba70, \ub0a8\ucabd\uc73c\ub85c\ub294 \ube0c\ub9ac\uc2a4\ud1a8 \ucc44\ub110\uacfc \uc811\ud55c\ub2e4. 2011\ub144 \uae30\uc900 \uc778\uad6c\ub294 3,043,456\uba85\uc774\uace0, \ucd1d \uba74\uc801\uc740 20,779 km(8,023 sq mi)\uc774\ub2e4. \uc6e8\uc77c\uc2a4\ub294 2,700 km(1,680 \ub9c8\uc77c)\uc758 \ud574\uc548\uc120\uc744 \ubcf4\uc720\ud558\uace0 \uc788\uc73c\uba70 \ub300\ubd80\ubd84 \uc0b0\uc545 \uc9c0\ud615\uc774\ub2e4. \ubd81\ubd80 \ubc0f \uc911\ubd80 \uc9c0\uc5ed\uc758 \ub192\uc740 \ubd09\uc6b0\ub9ac\ub294 \uc2a4\ub178\ub4e0 \uc0b0 (Snowdon, Yr Wyddfa)\uc774 \ucd5c\uace0\ubd09\uc774\ub2e4. \uc6e8\uc77c\uc2a4\ub294 \ubd81\ucabd \uc628\ub300 \uc9c0\uc5ed\uc5d0 \uc18d\ud574 \uc788\uc73c\uba70 \ubcc0\ud654\ud558\ub294 \ud574\uc591 \ud658\uacbd\uc744 \uac00\uc9c0\uace0 \uc788\ub2e4.", "sentence_answer": "\ubd81\ubd80 \ubc0f \uc911\ubd80 \uc9c0\uc5ed\uc758 \ub192\uc740 \ubd09\uc6b0\ub9ac\ub294 \uc2a4\ub178\ub4e0 \uc0b0 (Snowdon, Yr Wyddfa)", "paragraph_id": "6526868-0-1", "paragraph_question": "question: \uc6e8\uc77c\uc2a4\uc758 \ubd81\ubd80 \ubc0f \uc911\ubd80 \uc9c0\uc5ed\uc758 \ucd5c\uace0\ubd09\uc740 \ubb34\uc5c7\uc77c\uae4c?, context: \uc6e8\uc77c\uc2a4(\uc601\uc5b4: Wales /\u02c8we\u026alz/, \uc6e8\uc77c\uc2a4\uc5b4: Cymru \ucef4\ub9ac [\u02c8k\u0259mri] ( \ub4e3\uae30))\ub294 \uc601\uad6d\uc758 \uad6c\uc131\uad6d \uc911 \ud558\ub098\uc774\uba70, \uadf8\ub808\uc774\ud2b8\ube0c\ub9ac\ud2bc \uc12c \uc11c\ubd80\uc5d0 \uc704\uce58\ud55c\ub2e4. \ub3d9\ucabd\uc73c\ub85c\ub294 \uc789\uae00\ub79c\ub4dc\uc640 \uc811\ud558\uace0 \uc11c\ucabd\uc73c\ub85c\ub294 \uc544\uc77c\ub79c\ub4dc \ud574\uc5d0 \uc811\ud558\uba70, \ub0a8\ucabd\uc73c\ub85c\ub294 \ube0c\ub9ac\uc2a4\ud1a8 \ucc44\ub110\uacfc \uc811\ud55c\ub2e4. 2011\ub144 \uae30\uc900 \uc778\uad6c\ub294 3,043,456\uba85\uc774\uace0, \ucd1d \uba74\uc801\uc740 20,779 km(8,023 sq mi)\uc774\ub2e4. \uc6e8\uc77c\uc2a4\ub294 2,700 km(1,680 \ub9c8\uc77c)\uc758 \ud574\uc548\uc120\uc744 \ubcf4\uc720\ud558\uace0 \uc788\uc73c\uba70 \ub300\ubd80\ubd84 \uc0b0\uc545 \uc9c0\ud615\uc774\ub2e4. \ubd81\ubd80 \ubc0f \uc911\ubd80 \uc9c0\uc5ed\uc758 \ub192\uc740 \ubd09\uc6b0\ub9ac\ub294 \uc2a4\ub178\ub4e0 \uc0b0(Snowdon, Yr Wyddfa)\uc774 \ucd5c\uace0\ubd09\uc774\ub2e4. \uc6e8\uc77c\uc2a4\ub294 \ubd81\ucabd \uc628\ub300 \uc9c0\uc5ed\uc5d0 \uc18d\ud574 \uc788\uc73c\uba70 \ubcc0\ud654\ud558\ub294 \ud574\uc591 \ud658\uacbd\uc744 \uac00\uc9c0\uace0 \uc788\ub2e4."} {"question": "\ub77c\uc988\ubca0\ub9ac \ud30c\uc774 \ud504\ub85c\uc138\uc11c \uc624\ubc84\ud074\ub7ed \uc2dc \ubc1c\ud718\ud560 \uc218 \uc788\ub294 \ucd5c\ub300 \uc131\ub2a5\uc740?", "paragraph": "\ub77c\uc988\ubca0\ub9ac \ud30c\uc774 2\ub97c \uc81c\uc678\ud55c \ub77c\uc988\ubca0\ub9ac \ud30c\uc774 \ubaa8\ub378\ub4e4\uc740 \ube0c\ub85c\ub4dc\ucef4\uc758 BCM2835 \ub2e8\uc77c \uce69 \uc2dc\uc2a4\ud15c\uc744 \uc0ac\uc6a9\ud558\uba70, \uc774 \uce69\uc5d0\ub294 ARM1176JZF-S 700 MHz \uc2f1\uae00\ucf54\uc5b4\ud504\ub85c\uc138\uc11c(\uc77c\ubc18 \ub370\uc2a4\ud06c\ud0d1\uc740 \ubcf4\ud1b5 2500 MHz~3500 MHz), \ube44\ub514\uc624\ucf54\uc5b4 IV VGA \uc640 512 \uba54\uac00\ubc14\uc774\ud2b8 RAM\uc774 \ub4e4\uc5b4 \uc788\ub2e4. \uadf8\ub9ac\uace0 \ub77c\uc988\ubca0\ub9ac \ud30c\uc774\uc758 \ud504\ub85c\uc138\uc11c\ub294 \uc624\ubc84\ud074\ub7ed\uc2dc \ucd5c\ub300 1000 MHz\uae4c\uc9c0\uc758 \uc131\ub2a5\uc744 \ubc1c\ud718 \ud560 \uc218 \uc788\uc73c\uba70, \ud558\ub4dc \ub514\uc2a4\ud06c \ub4dc\ub77c\uc774\ube0c\ub098 \uc194\ub9ac\ub4dc \uc2a4\ud14c\uc774\ud2b8 \ub4dc\ub77c\uc774\ube0c\ub97c \ub0b4\uc7a5\ud558\uace0 \uc788\uc9c0 \uc54a\uc73c\uba70, SD \uce74\ub4dc(B+,2 B+ \ubaa8\ub378\uc740 Micro SD Card\ub97c \uc0ac\uc6a9)\ub97c \uc678\ubd80 \uae30\uc5b5\uc7a5\uce58\ub85c \uc0ac\uc6a9\ud55c\ub2e4. \uc0c8\ub85c \ucd9c\uc2dc\ud55c 2 \ubaa8\ub378 B\ub294 ARM Cortex-A7 0.9GHz\ud504\ub85c\uc138\uc11c\uc640 \ub7a8\uc6a9\ub7c9\uc774 1GB\ub85c \uc131\ub2a5\uc774 \uc5c5\uadf8\ub808\uc774\ub4dc\ub418\uc5b4 \ucd9c\uc2dc \ub418\uc5c8\ub2e4. \ub77c\uc988\ubca0\ub9ac \uc7ac\ub2e8\uc740 \ucef4\ud4e8\ud130 \uad50\uc721 \uc99d\uc9c4\uc744 \uc704\ud574 2\uac00\uc9c0 \ubaa8\ub378\uc744 \ub0b4\ub193\uc558\uc73c\uba70, \uac01\uac01 25\ub2ec\ub7ec\uc640 35\ub2ec\ub7ec\ub85c \ucc45\uc815\ub418\uc5c8\ub2e4. 2012\ub144 2\uc6d4 29\uc77c \uc7ac\ub2e8\uc5d0\uc11c 35\ub2ec\ub7ec\uc9dc\ub9ac \ubaa8\ub378\uc758 \uc8fc\ubb38\uc744 \ubc1b\uae30 \uc2dc\uc791\ud558\uc600\ub2e4.", "answer": "1000 MHz", "sentence": "\ucd5c\ub300 1000 MHz \uae4c\uc9c0\uc758 \uc131\ub2a5\uc744 \ubc1c\ud718 \ud560 \uc218 \uc788\uc73c\uba70, \ud558\ub4dc \ub514\uc2a4\ud06c \ub4dc\ub77c\uc774\ube0c\ub098 \uc194\ub9ac\ub4dc \uc2a4\ud14c\uc774\ud2b8 \ub4dc\ub77c\uc774\ube0c\ub97c \ub0b4\uc7a5\ud558\uace0 \uc788\uc9c0 \uc54a\uc73c\uba70, SD \uce74\ub4dc(B+,2 B+ \ubaa8\ub378\uc740 Micro SD Card\ub97c \uc0ac\uc6a9)\ub97c \uc678\ubd80 \uae30\uc5b5\uc7a5\uce58\ub85c \uc0ac\uc6a9\ud55c\ub2e4.", "paragraph_sentence": "\ub77c\uc988\ubca0\ub9ac \ud30c\uc774 2\ub97c \uc81c\uc678\ud55c \ub77c\uc988\ubca0\ub9ac \ud30c\uc774 \ubaa8\ub378\ub4e4\uc740 \ube0c\ub85c\ub4dc\ucef4\uc758 BCM2835 \ub2e8\uc77c \uce69 \uc2dc\uc2a4\ud15c\uc744 \uc0ac\uc6a9\ud558\uba70, \uc774 \uce69\uc5d0\ub294 ARM1176JZF-S 700 MHz \uc2f1\uae00\ucf54\uc5b4\ud504\ub85c\uc138\uc11c(\uc77c\ubc18 \ub370\uc2a4\ud06c\ud0d1\uc740 \ubcf4\ud1b5 2500 MHz~3500 MHz), \ube44\ub514\uc624\ucf54\uc5b4 IV VGA \uc640 512 \uba54\uac00\ubc14\uc774\ud2b8 RAM\uc774 \ub4e4\uc5b4 \uc788\ub2e4. \uadf8\ub9ac\uace0 \ub77c\uc988\ubca0\ub9ac \ud30c\uc774\uc758 \ud504\ub85c\uc138\uc11c\ub294 \uc624\ubc84\ud074\ub7ed\uc2dc \ucd5c\ub300 1000 MHz \uae4c\uc9c0\uc758 \uc131\ub2a5\uc744 \ubc1c\ud718 \ud560 \uc218 \uc788\uc73c\uba70, \ud558\ub4dc \ub514\uc2a4\ud06c \ub4dc\ub77c\uc774\ube0c\ub098 \uc194\ub9ac\ub4dc \uc2a4\ud14c\uc774\ud2b8 \ub4dc\ub77c\uc774\ube0c\ub97c \ub0b4\uc7a5\ud558\uace0 \uc788\uc9c0 \uc54a\uc73c\uba70, SD \uce74\ub4dc(B+,2 B+ \ubaa8\ub378\uc740 Micro SD Card\ub97c \uc0ac\uc6a9)\ub97c \uc678\ubd80 \uae30\uc5b5\uc7a5\uce58\ub85c \uc0ac\uc6a9\ud55c\ub2e4. \uc0c8\ub85c \ucd9c\uc2dc\ud55c 2 \ubaa8\ub378 B\ub294 ARM Cortex-A7 0.9GHz\ud504\ub85c\uc138\uc11c\uc640 \ub7a8\uc6a9\ub7c9\uc774 1GB\ub85c \uc131\ub2a5\uc774 \uc5c5\uadf8\ub808\uc774\ub4dc\ub418\uc5b4 \ucd9c\uc2dc \ub418\uc5c8\ub2e4. \ub77c\uc988\ubca0\ub9ac \uc7ac\ub2e8\uc740 \ucef4\ud4e8\ud130 \uad50\uc721 \uc99d\uc9c4\uc744 \uc704\ud574 2\uac00\uc9c0 \ubaa8\ub378\uc744 \ub0b4\ub193\uc558\uc73c\uba70, \uac01\uac01 25\ub2ec\ub7ec\uc640 35\ub2ec\ub7ec\ub85c \ucc45\uc815\ub418\uc5c8\ub2e4. 2012\ub144 2\uc6d4 29\uc77c \uc7ac\ub2e8\uc5d0\uc11c 35\ub2ec\ub7ec\uc9dc\ub9ac \ubaa8\ub378\uc758 \uc8fc\ubb38\uc744 \ubc1b\uae30 \uc2dc\uc791\ud558\uc600\ub2e4.", "paragraph_answer": "\ub77c\uc988\ubca0\ub9ac \ud30c\uc774 2\ub97c \uc81c\uc678\ud55c \ub77c\uc988\ubca0\ub9ac \ud30c\uc774 \ubaa8\ub378\ub4e4\uc740 \ube0c\ub85c\ub4dc\ucef4\uc758 BCM2835 \ub2e8\uc77c \uce69 \uc2dc\uc2a4\ud15c\uc744 \uc0ac\uc6a9\ud558\uba70, \uc774 \uce69\uc5d0\ub294 ARM1176JZF-S 700 MHz \uc2f1\uae00\ucf54\uc5b4\ud504\ub85c\uc138\uc11c(\uc77c\ubc18 \ub370\uc2a4\ud06c\ud0d1\uc740 \ubcf4\ud1b5 2500 MHz~3500 MHz), \ube44\ub514\uc624\ucf54\uc5b4 IV VGA \uc640 512 \uba54\uac00\ubc14\uc774\ud2b8 RAM\uc774 \ub4e4\uc5b4 \uc788\ub2e4. \uadf8\ub9ac\uace0 \ub77c\uc988\ubca0\ub9ac \ud30c\uc774\uc758 \ud504\ub85c\uc138\uc11c\ub294 \uc624\ubc84\ud074\ub7ed\uc2dc \ucd5c\ub300 1000 MHz \uae4c\uc9c0\uc758 \uc131\ub2a5\uc744 \ubc1c\ud718 \ud560 \uc218 \uc788\uc73c\uba70, \ud558\ub4dc \ub514\uc2a4\ud06c \ub4dc\ub77c\uc774\ube0c\ub098 \uc194\ub9ac\ub4dc \uc2a4\ud14c\uc774\ud2b8 \ub4dc\ub77c\uc774\ube0c\ub97c \ub0b4\uc7a5\ud558\uace0 \uc788\uc9c0 \uc54a\uc73c\uba70, SD \uce74\ub4dc(B+,2 B+ \ubaa8\ub378\uc740 Micro SD Card\ub97c \uc0ac\uc6a9)\ub97c \uc678\ubd80 \uae30\uc5b5\uc7a5\uce58\ub85c \uc0ac\uc6a9\ud55c\ub2e4. \uc0c8\ub85c \ucd9c\uc2dc\ud55c 2 \ubaa8\ub378 B\ub294 ARM Cortex-A7 0.9GHz\ud504\ub85c\uc138\uc11c\uc640 \ub7a8\uc6a9\ub7c9\uc774 1GB\ub85c \uc131\ub2a5\uc774 \uc5c5\uadf8\ub808\uc774\ub4dc\ub418\uc5b4 \ucd9c\uc2dc \ub418\uc5c8\ub2e4. \ub77c\uc988\ubca0\ub9ac \uc7ac\ub2e8\uc740 \ucef4\ud4e8\ud130 \uad50\uc721 \uc99d\uc9c4\uc744 \uc704\ud574 2\uac00\uc9c0 \ubaa8\ub378\uc744 \ub0b4\ub193\uc558\uc73c\uba70, \uac01\uac01 25\ub2ec\ub7ec\uc640 35\ub2ec\ub7ec\ub85c \ucc45\uc815\ub418\uc5c8\ub2e4. 2012\ub144 2\uc6d4 29\uc77c \uc7ac\ub2e8\uc5d0\uc11c 35\ub2ec\ub7ec\uc9dc\ub9ac \ubaa8\ub378\uc758 \uc8fc\ubb38\uc744 \ubc1b\uae30 \uc2dc\uc791\ud558\uc600\ub2e4.", "sentence_answer": "\ucd5c\ub300 1000 MHz \uae4c\uc9c0\uc758 \uc131\ub2a5\uc744 \ubc1c\ud718 \ud560 \uc218 \uc788\uc73c\uba70, \ud558\ub4dc \ub514\uc2a4\ud06c \ub4dc\ub77c\uc774\ube0c\ub098 \uc194\ub9ac\ub4dc \uc2a4\ud14c\uc774\ud2b8 \ub4dc\ub77c\uc774\ube0c\ub97c \ub0b4\uc7a5\ud558\uace0 \uc788\uc9c0 \uc54a\uc73c\uba70, SD \uce74\ub4dc(B+,2 B+ \ubaa8\ub378\uc740 Micro SD Card\ub97c \uc0ac\uc6a9)\ub97c \uc678\ubd80 \uae30\uc5b5\uc7a5\uce58\ub85c \uc0ac\uc6a9\ud55c\ub2e4.", "paragraph_id": "6302462-0-0", "paragraph_question": "question: \ub77c\uc988\ubca0\ub9ac \ud30c\uc774 \ud504\ub85c\uc138\uc11c \uc624\ubc84\ud074\ub7ed \uc2dc \ubc1c\ud718\ud560 \uc218 \uc788\ub294 \ucd5c\ub300 \uc131\ub2a5\uc740?, context: \ub77c\uc988\ubca0\ub9ac \ud30c\uc774 2\ub97c \uc81c\uc678\ud55c \ub77c\uc988\ubca0\ub9ac \ud30c\uc774 \ubaa8\ub378\ub4e4\uc740 \ube0c\ub85c\ub4dc\ucef4\uc758 BCM2835 \ub2e8\uc77c \uce69 \uc2dc\uc2a4\ud15c\uc744 \uc0ac\uc6a9\ud558\uba70, \uc774 \uce69\uc5d0\ub294 ARM1176JZF-S 700 MHz \uc2f1\uae00\ucf54\uc5b4\ud504\ub85c\uc138\uc11c(\uc77c\ubc18 \ub370\uc2a4\ud06c\ud0d1\uc740 \ubcf4\ud1b5 2500 MHz~3500 MHz), \ube44\ub514\uc624\ucf54\uc5b4 IV VGA \uc640 512 \uba54\uac00\ubc14\uc774\ud2b8 RAM\uc774 \ub4e4\uc5b4 \uc788\ub2e4. \uadf8\ub9ac\uace0 \ub77c\uc988\ubca0\ub9ac \ud30c\uc774\uc758 \ud504\ub85c\uc138\uc11c\ub294 \uc624\ubc84\ud074\ub7ed\uc2dc \ucd5c\ub300 1000 MHz\uae4c\uc9c0\uc758 \uc131\ub2a5\uc744 \ubc1c\ud718 \ud560 \uc218 \uc788\uc73c\uba70, \ud558\ub4dc \ub514\uc2a4\ud06c \ub4dc\ub77c\uc774\ube0c\ub098 \uc194\ub9ac\ub4dc \uc2a4\ud14c\uc774\ud2b8 \ub4dc\ub77c\uc774\ube0c\ub97c \ub0b4\uc7a5\ud558\uace0 \uc788\uc9c0 \uc54a\uc73c\uba70, SD \uce74\ub4dc(B+,2 B+ \ubaa8\ub378\uc740 Micro SD Card\ub97c \uc0ac\uc6a9)\ub97c \uc678\ubd80 \uae30\uc5b5\uc7a5\uce58\ub85c \uc0ac\uc6a9\ud55c\ub2e4. \uc0c8\ub85c \ucd9c\uc2dc\ud55c 2 \ubaa8\ub378 B\ub294 ARM Cortex-A7 0.9GHz\ud504\ub85c\uc138\uc11c\uc640 \ub7a8\uc6a9\ub7c9\uc774 1GB\ub85c \uc131\ub2a5\uc774 \uc5c5\uadf8\ub808\uc774\ub4dc\ub418\uc5b4 \ucd9c\uc2dc \ub418\uc5c8\ub2e4. \ub77c\uc988\ubca0\ub9ac \uc7ac\ub2e8\uc740 \ucef4\ud4e8\ud130 \uad50\uc721 \uc99d\uc9c4\uc744 \uc704\ud574 2\uac00\uc9c0 \ubaa8\ub378\uc744 \ub0b4\ub193\uc558\uc73c\uba70, \uac01\uac01 25\ub2ec\ub7ec\uc640 35\ub2ec\ub7ec\ub85c \ucc45\uc815\ub418\uc5c8\ub2e4. 2012\ub144 2\uc6d4 29\uc77c \uc7ac\ub2e8\uc5d0\uc11c 35\ub2ec\ub7ec\uc9dc\ub9ac \ubaa8\ub378\uc758 \uc8fc\ubb38\uc744 \ubc1b\uae30 \uc2dc\uc791\ud558\uc600\ub2e4."} {"question": "\uc815\ud655\ud55c \uc2e4\uc9c8\uc784\uae08\uc758 \ucd94\uc815\uc744 \uc704\ud558\uc5ec \uc81c\ub300\ub85c \ubc18\uc601\ud574\uc57c \ud558\ub294 \uac83\uc740?", "paragraph": "20\uc138\uae30 \ub4e4\uc5b4 \uc815\ub7c9\uc801\uc778 \uc9c0\ud45c\ub97c \uc0ac\uc6a9\ud558\uae30 \uc2dc\uc791\ud588\ub294\ub370 \uadf8\uac83\uc740 \ubc14\ub85c \uc2e4\uc9c8 \uc784\uae08\uc774\uc5c8\ub2e4. \ud2b9\uc815 \uc9c1\uc5c5\uad70\uc758 \uc784\uae08 \uc790\ub8cc\ub97c \ud3c9\uade0\ud558\uace0 \uc774\ub97c \ud574\ub2f9 \uc5f0\ub3c4\uc758 \ubb3c\uac00\uc218\uc900\uc73c\ub85c \ubcf4\uc815\ud558\uc5ec \uc18c\ube44\uc218\uc900\uc758 \ud55c \ucc99\ub3c4\ub85c \uc0ac\uc6a9\ud558\uc600\ub2e4. \uc774\ub294 \uc0b0\uc5c5\ud654 \uc2dc\uae30 \uc784\uae08 \ud1b5\uacc4\uac00 \ub2e4\uc591\ud55c \ud615\ud0dc\ub85c \uc874\uc7ac\ud558\uc5ec \uc790\ub8cc \uc811\uadfc\ub3c4\uac00 \ub192\uc558\uace0, \uc2dc\uacc4\uc5f4 \uc784\uae08 \uc790\ub8cc \uad6c\ucd95\uc774 \uac00\ub2a5\ud558\uc5ec \uc0dd\ud65c\uc218\uc900 \uc7a5\uae30 \ucd94\uc774 \ubd84\uc11d\uc774 \uc720\uc6a9\ud558\ub2e4\ub294 \uc7a5\uc810\uc774 \uc788\uc5c8\ub2e4. \ud558\uc9c0\ub9cc \uc2e4\uc9c8\uc784\uae08\uc758 \uc7a5\uae30 \ucd94\uc774 \ubd84\uc11d\uc5d0\ub294 \uc0b0\uc5c5\ud654\ub97c \uac70\uce58\ub294 \ub3d9\uc548 \uc9c1\uc5c5\uad70\uc758 \ubd84\ud3ec\uac00 \ubcc0\ud654\ud558\uc5ec \uc7a5\uae30\uac04\uc5d0 \uac78\uce5c \uc77c\uad00\ub418\uace0 \ube44\uad50 \uac00\ub2a5\ud55c \uc2e4\uc9c8\uc784\uae08\uc758 \ucd94\uc815\uc774 \uc5b4\ub835\ub2e4\ub294 \uc810\uacfc \uc2e4\uc9c8\uc784\uae08\uc774 \uc18c\ube44\uc758 \uc9c8\uc801 \uc218\uc900\uc744 \ubc18\uc601\ud558\uc9c0 \ubabb\ud55c\ub2e4\ub294 \uc810, \uadf8\ub9ac\uace0 \uc815\ud655\ud55c \uc2e4\uc9c8\uc784\uae08\uc758 \ucd94\uc815\uc5d0\ub294 \ubb3c\uac00\uc218\uc900\uc744 \uc81c\ub300\ub85c \ubc18\uc601\ud574\uc57c \ud558\uc9c0\ub9cc \uc9c0\ucd9c \ud56d\ubaa9\uc758 \uc7a6\uc740 \ubcc0\ud654\ub85c \uc27d\uc9c0 \uc54a\uc544 \ubb3c\uac00\ub97c \ubc18\uc601\ud558\uc9c0 \ubabb\ud55c\ub2e4\ub294 \ub2e8\uc810\uc774 \uc874\uc7ac\ud55c\ub2e4. \uc774\ub7ec\ud55c \ub2e8\uc810\uc73c\ub85c \uc2e4\uc9c8\uc784\uae08\uc744 \uad6d\uac00 \uac04 \ube44\uad50\uc5d0 \uc0ac\uc6a9\ud558\ub294 \ub370 \ud070 \uc5b4\ub824\uc6c0\uc774 \uc788\uc5c8\ub2e4\uace0 \ud55c\ub2e4.", "answer": "\ubb3c\uac00\uc218\uc900", "sentence": "\ud2b9\uc815 \uc9c1\uc5c5\uad70\uc758 \uc784\uae08 \uc790\ub8cc\ub97c \ud3c9\uade0\ud558\uace0 \uc774\ub97c \ud574\ub2f9 \uc5f0\ub3c4\uc758 \ubb3c\uac00\uc218\uc900 \uc73c\ub85c \ubcf4\uc815\ud558\uc5ec \uc18c\ube44\uc218\uc900\uc758 \ud55c \ucc99\ub3c4\ub85c \uc0ac\uc6a9\ud558\uc600\ub2e4.", "paragraph_sentence": "20\uc138\uae30 \ub4e4\uc5b4 \uc815\ub7c9\uc801\uc778 \uc9c0\ud45c\ub97c \uc0ac\uc6a9\ud558\uae30 \uc2dc\uc791\ud588\ub294\ub370 \uadf8\uac83\uc740 \ubc14\ub85c \uc2e4\uc9c8 \uc784\uae08\uc774\uc5c8\ub2e4. \ud2b9\uc815 \uc9c1\uc5c5\uad70\uc758 \uc784\uae08 \uc790\ub8cc\ub97c \ud3c9\uade0\ud558\uace0 \uc774\ub97c \ud574\ub2f9 \uc5f0\ub3c4\uc758 \ubb3c\uac00\uc218\uc900 \uc73c\ub85c \ubcf4\uc815\ud558\uc5ec \uc18c\ube44\uc218\uc900\uc758 \ud55c \ucc99\ub3c4\ub85c \uc0ac\uc6a9\ud558\uc600\ub2e4. \uc774\ub294 \uc0b0\uc5c5\ud654 \uc2dc\uae30 \uc784\uae08 \ud1b5\uacc4\uac00 \ub2e4\uc591\ud55c \ud615\ud0dc\ub85c \uc874\uc7ac\ud558\uc5ec \uc790\ub8cc \uc811\uadfc\ub3c4\uac00 \ub192\uc558\uace0, \uc2dc\uacc4\uc5f4 \uc784\uae08 \uc790\ub8cc \uad6c\ucd95\uc774 \uac00\ub2a5\ud558\uc5ec \uc0dd\ud65c\uc218\uc900 \uc7a5\uae30 \ucd94\uc774 \ubd84\uc11d\uc774 \uc720\uc6a9\ud558\ub2e4\ub294 \uc7a5\uc810\uc774 \uc788\uc5c8\ub2e4. \ud558\uc9c0\ub9cc \uc2e4\uc9c8\uc784\uae08\uc758 \uc7a5\uae30 \ucd94\uc774 \ubd84\uc11d\uc5d0\ub294 \uc0b0\uc5c5\ud654\ub97c \uac70\uce58\ub294 \ub3d9\uc548 \uc9c1\uc5c5\uad70\uc758 \ubd84\ud3ec\uac00 \ubcc0\ud654\ud558\uc5ec \uc7a5\uae30\uac04\uc5d0 \uac78\uce5c \uc77c\uad00\ub418\uace0 \ube44\uad50 \uac00\ub2a5\ud55c \uc2e4\uc9c8\uc784\uae08\uc758 \ucd94\uc815\uc774 \uc5b4\ub835\ub2e4\ub294 \uc810\uacfc \uc2e4\uc9c8\uc784\uae08\uc774 \uc18c\ube44\uc758 \uc9c8\uc801 \uc218\uc900\uc744 \ubc18\uc601\ud558\uc9c0 \ubabb\ud55c\ub2e4\ub294 \uc810, \uadf8\ub9ac\uace0 \uc815\ud655\ud55c \uc2e4\uc9c8\uc784\uae08\uc758 \ucd94\uc815\uc5d0\ub294 \ubb3c\uac00\uc218\uc900\uc744 \uc81c\ub300\ub85c \ubc18\uc601\ud574\uc57c \ud558\uc9c0\ub9cc \uc9c0\ucd9c \ud56d\ubaa9\uc758 \uc7a6\uc740 \ubcc0\ud654\ub85c \uc27d\uc9c0 \uc54a\uc544 \ubb3c\uac00\ub97c \ubc18\uc601\ud558\uc9c0 \ubabb\ud55c\ub2e4\ub294 \ub2e8\uc810\uc774 \uc874\uc7ac\ud55c\ub2e4. \uc774\ub7ec\ud55c \ub2e8\uc810\uc73c\ub85c \uc2e4\uc9c8\uc784\uae08\uc744 \uad6d\uac00 \uac04 \ube44\uad50\uc5d0 \uc0ac\uc6a9\ud558\ub294 \ub370 \ud070 \uc5b4\ub824\uc6c0\uc774 \uc788\uc5c8\ub2e4\uace0 \ud55c\ub2e4.", "paragraph_answer": "20\uc138\uae30 \ub4e4\uc5b4 \uc815\ub7c9\uc801\uc778 \uc9c0\ud45c\ub97c \uc0ac\uc6a9\ud558\uae30 \uc2dc\uc791\ud588\ub294\ub370 \uadf8\uac83\uc740 \ubc14\ub85c \uc2e4\uc9c8 \uc784\uae08\uc774\uc5c8\ub2e4. \ud2b9\uc815 \uc9c1\uc5c5\uad70\uc758 \uc784\uae08 \uc790\ub8cc\ub97c \ud3c9\uade0\ud558\uace0 \uc774\ub97c \ud574\ub2f9 \uc5f0\ub3c4\uc758 \ubb3c\uac00\uc218\uc900 \uc73c\ub85c \ubcf4\uc815\ud558\uc5ec \uc18c\ube44\uc218\uc900\uc758 \ud55c \ucc99\ub3c4\ub85c \uc0ac\uc6a9\ud558\uc600\ub2e4. \uc774\ub294 \uc0b0\uc5c5\ud654 \uc2dc\uae30 \uc784\uae08 \ud1b5\uacc4\uac00 \ub2e4\uc591\ud55c \ud615\ud0dc\ub85c \uc874\uc7ac\ud558\uc5ec \uc790\ub8cc \uc811\uadfc\ub3c4\uac00 \ub192\uc558\uace0, \uc2dc\uacc4\uc5f4 \uc784\uae08 \uc790\ub8cc \uad6c\ucd95\uc774 \uac00\ub2a5\ud558\uc5ec \uc0dd\ud65c\uc218\uc900 \uc7a5\uae30 \ucd94\uc774 \ubd84\uc11d\uc774 \uc720\uc6a9\ud558\ub2e4\ub294 \uc7a5\uc810\uc774 \uc788\uc5c8\ub2e4. \ud558\uc9c0\ub9cc \uc2e4\uc9c8\uc784\uae08\uc758 \uc7a5\uae30 \ucd94\uc774 \ubd84\uc11d\uc5d0\ub294 \uc0b0\uc5c5\ud654\ub97c \uac70\uce58\ub294 \ub3d9\uc548 \uc9c1\uc5c5\uad70\uc758 \ubd84\ud3ec\uac00 \ubcc0\ud654\ud558\uc5ec \uc7a5\uae30\uac04\uc5d0 \uac78\uce5c \uc77c\uad00\ub418\uace0 \ube44\uad50 \uac00\ub2a5\ud55c \uc2e4\uc9c8\uc784\uae08\uc758 \ucd94\uc815\uc774 \uc5b4\ub835\ub2e4\ub294 \uc810\uacfc \uc2e4\uc9c8\uc784\uae08\uc774 \uc18c\ube44\uc758 \uc9c8\uc801 \uc218\uc900\uc744 \ubc18\uc601\ud558\uc9c0 \ubabb\ud55c\ub2e4\ub294 \uc810, \uadf8\ub9ac\uace0 \uc815\ud655\ud55c \uc2e4\uc9c8\uc784\uae08\uc758 \ucd94\uc815\uc5d0\ub294 \ubb3c\uac00\uc218\uc900\uc744 \uc81c\ub300\ub85c \ubc18\uc601\ud574\uc57c \ud558\uc9c0\ub9cc \uc9c0\ucd9c \ud56d\ubaa9\uc758 \uc7a6\uc740 \ubcc0\ud654\ub85c \uc27d\uc9c0 \uc54a\uc544 \ubb3c\uac00\ub97c \ubc18\uc601\ud558\uc9c0 \ubabb\ud55c\ub2e4\ub294 \ub2e8\uc810\uc774 \uc874\uc7ac\ud55c\ub2e4. \uc774\ub7ec\ud55c \ub2e8\uc810\uc73c\ub85c \uc2e4\uc9c8\uc784\uae08\uc744 \uad6d\uac00 \uac04 \ube44\uad50\uc5d0 \uc0ac\uc6a9\ud558\ub294 \ub370 \ud070 \uc5b4\ub824\uc6c0\uc774 \uc788\uc5c8\ub2e4\uace0 \ud55c\ub2e4.", "sentence_answer": "\ud2b9\uc815 \uc9c1\uc5c5\uad70\uc758 \uc784\uae08 \uc790\ub8cc\ub97c \ud3c9\uade0\ud558\uace0 \uc774\ub97c \ud574\ub2f9 \uc5f0\ub3c4\uc758 \ubb3c\uac00\uc218\uc900 \uc73c\ub85c \ubcf4\uc815\ud558\uc5ec \uc18c\ube44\uc218\uc900\uc758 \ud55c \ucc99\ub3c4\ub85c \uc0ac\uc6a9\ud558\uc600\ub2e4.", "paragraph_id": "6511379-1-1", "paragraph_question": "question: \uc815\ud655\ud55c \uc2e4\uc9c8\uc784\uae08\uc758 \ucd94\uc815\uc744 \uc704\ud558\uc5ec \uc81c\ub300\ub85c \ubc18\uc601\ud574\uc57c \ud558\ub294 \uac83\uc740?, context: 20\uc138\uae30 \ub4e4\uc5b4 \uc815\ub7c9\uc801\uc778 \uc9c0\ud45c\ub97c \uc0ac\uc6a9\ud558\uae30 \uc2dc\uc791\ud588\ub294\ub370 \uadf8\uac83\uc740 \ubc14\ub85c \uc2e4\uc9c8 \uc784\uae08\uc774\uc5c8\ub2e4. \ud2b9\uc815 \uc9c1\uc5c5\uad70\uc758 \uc784\uae08 \uc790\ub8cc\ub97c \ud3c9\uade0\ud558\uace0 \uc774\ub97c \ud574\ub2f9 \uc5f0\ub3c4\uc758 \ubb3c\uac00\uc218\uc900\uc73c\ub85c \ubcf4\uc815\ud558\uc5ec \uc18c\ube44\uc218\uc900\uc758 \ud55c \ucc99\ub3c4\ub85c \uc0ac\uc6a9\ud558\uc600\ub2e4. \uc774\ub294 \uc0b0\uc5c5\ud654 \uc2dc\uae30 \uc784\uae08 \ud1b5\uacc4\uac00 \ub2e4\uc591\ud55c \ud615\ud0dc\ub85c \uc874\uc7ac\ud558\uc5ec \uc790\ub8cc \uc811\uadfc\ub3c4\uac00 \ub192\uc558\uace0, \uc2dc\uacc4\uc5f4 \uc784\uae08 \uc790\ub8cc \uad6c\ucd95\uc774 \uac00\ub2a5\ud558\uc5ec \uc0dd\ud65c\uc218\uc900 \uc7a5\uae30 \ucd94\uc774 \ubd84\uc11d\uc774 \uc720\uc6a9\ud558\ub2e4\ub294 \uc7a5\uc810\uc774 \uc788\uc5c8\ub2e4. \ud558\uc9c0\ub9cc \uc2e4\uc9c8\uc784\uae08\uc758 \uc7a5\uae30 \ucd94\uc774 \ubd84\uc11d\uc5d0\ub294 \uc0b0\uc5c5\ud654\ub97c \uac70\uce58\ub294 \ub3d9\uc548 \uc9c1\uc5c5\uad70\uc758 \ubd84\ud3ec\uac00 \ubcc0\ud654\ud558\uc5ec \uc7a5\uae30\uac04\uc5d0 \uac78\uce5c \uc77c\uad00\ub418\uace0 \ube44\uad50 \uac00\ub2a5\ud55c \uc2e4\uc9c8\uc784\uae08\uc758 \ucd94\uc815\uc774 \uc5b4\ub835\ub2e4\ub294 \uc810\uacfc \uc2e4\uc9c8\uc784\uae08\uc774 \uc18c\ube44\uc758 \uc9c8\uc801 \uc218\uc900\uc744 \ubc18\uc601\ud558\uc9c0 \ubabb\ud55c\ub2e4\ub294 \uc810, \uadf8\ub9ac\uace0 \uc815\ud655\ud55c \uc2e4\uc9c8\uc784\uae08\uc758 \ucd94\uc815\uc5d0\ub294 \ubb3c\uac00\uc218\uc900\uc744 \uc81c\ub300\ub85c \ubc18\uc601\ud574\uc57c \ud558\uc9c0\ub9cc \uc9c0\ucd9c \ud56d\ubaa9\uc758 \uc7a6\uc740 \ubcc0\ud654\ub85c \uc27d\uc9c0 \uc54a\uc544 \ubb3c\uac00\ub97c \ubc18\uc601\ud558\uc9c0 \ubabb\ud55c\ub2e4\ub294 \ub2e8\uc810\uc774 \uc874\uc7ac\ud55c\ub2e4. \uc774\ub7ec\ud55c \ub2e8\uc810\uc73c\ub85c \uc2e4\uc9c8\uc784\uae08\uc744 \uad6d\uac00 \uac04 \ube44\uad50\uc5d0 \uc0ac\uc6a9\ud558\ub294 \ub370 \ud070 \uc5b4\ub824\uc6c0\uc774 \uc788\uc5c8\ub2e4\uace0 \ud55c\ub2e4."} {"question": "\uace0\ub300 \uc774\uc9d1\ud2b8\uc758 \uc0ac\ud6c4 \uc138\uacc4 \uc2e0\uc559\uc744 \uc5ff\ubcfc \uc218 \uc788\ub294 \ub300\ud45c\uc801\uc778 \uac83\uc73c\ub85c \ubbf8\ub77c\uc640 \ud568\uaed8 \ub3c5\ud2b9\ud55c \ub9e4\uc7a5\ubc29\uc2dd\uc758 \ud0c4\uc0dd\uc73c\ub85c \ubcfc \uc218 \uc788\ub294 \uac83\uc740?", "paragraph": "\uace0\ub300 \uc774\uc9d1\ud2b8 \uc885\uad50(\u53e4\u4ee3-\u5b97\u654e)\ub294 \uace0\ub300 \uc774\uc9d1\ud2b8 \uc0ac\ub78c\ub4e4\uc774 \uccab \uc655\uc870\ubd80\ud130 \uae30\ub3c5\uad50 \uc720\uc785\uae30\uae4c\uc9c0 \uc57d 3\ucc9c \ub144 \ub118\uac8c \uc720\uc9c0\ud55c \ub2e4\uc591\ud55c \uc2e0\uc559\uacfc \uc7a5\ub840 \uc758\uc2dd\uc744 \ud3ec\ud568\ud558\ub294 \uc885\uad50\uc774\ub2e4. \uc774 \ubbff\uc74c\uc758 \uc911\uc2ec\uc5d0\ub294 \uc628\uac16 \uc790\uc5f0\uc758 \ud798\uc744 \uc0c1\uc9d5\ud558\ub294 \ub2e4\uc591\ud55c \uc2e0\ub4e4\uc774 \uc788\ub2e4. \uc774 \uc2e0\ub4e4\uc740 \uac01\uac01 \ub2e4\ub978 \uac1c\uc131\uc744 \uac00\uc9c0\uace0 \uc788\uc73c\ub098, \uc774\uc9d1\ud2b8 \uc81c18\uc655\uc870\uc5d0 \ub4e4\uc5b4\uc11c\uc11c\ub294 \uc544\ubb38\uacfc \uac19\uc740 \ud558\ub098\uc758 \uc2e0\uc774 \ub2e4\uc591\ud55c \uc778\uaca9\uccb4\uc640 \ubaa8\ub4e0 \uc2e0\uc801 \uad8c\ub2a5\uc744 \ud3ec\ud568\ud558\ub294 \uac83\uc73c\ub85c \uace0\ub300 \uc774\uc9d1\ud2b8 \uc885\uad50\uc758 \uc2e0\ud559\uc774 \ubcc0\ud574\uac10\uc5d0 \ub530\ub77c \ud604\uc7ac \uae30\ub3c5\uad50\uc5d0\uc11c \ucc3e\uc744 \uc218 \uc788\ub294 \uc0bc\uc704\uc77c\uccb4\uc758 \uae30\uc6d0\uc774 \ub418\ub294 \ubaa8\uc2b5\ub3c4 \ubcf4\uc774\uace0 \uc788\ub2e4. \uc774\uc9d1\ud2b8\uc758 \uc2e0\ub4e4\uc740 \uac01 \uac00\uc815\uc9d1\uc774\ub098 \uc9c0\uc5ed\uc758 \uc911\uc2ec \uc2e0\uc804\uc5d0\uc11c \uc7ac\ubb3c \ubd09\ub0a9\uacfc \uae30\ub3c4\ub85c \uc22d\ubc30\ub418\uc5c8\ub294\ub370, \uac01 \uc774\uc9d1\ud2b8\uc758 \uc655\uc870\ub9c8\ub2e4 \uba87\uba87 \uc2e0\ub4e4\uc774 \ub450\ub4dc\ub7ec\uc9c0\uac8c \uc22d\ubc30\ub418\uac70\ub098 \uc2e0\ub4e4\uc774 \uc5f0\ud569\ub418\uae30\ub3c4 \ud558\uc600\uc73c\uba70 \uc774\ub294 \uac01 \uc655\uc870\uc758 \ud30c\ub77c\uc624 \uc774\ub984\uc5d0\uc11c\ub3c4 \ucc3e\uc544\ubcfc \uc218 \uc788\ub2e4. \uc774 \uc885\uad50\ub294 \ub2e4\uc591\ud55c \uc2e0\ub150\uc744 \ud3ec\ud568\ud558\uace0 \uc788\ub294\ub370, \uac00\ub839 \ud30c\ub77c\uc624\uc758 \uc2e0\uc131\uacfc \uac19\uc740 \uad50\ub9ac\ub294 \uc774\uc9d1\ud2b8\uc758 \uc815\uce58\uc801 \uc548\uc815\uc5d0 \uc774\ubc14\uc9c0\ud558\uc600\ub2e4. \ub610\ud55c \uc0ac\ud6c4 \uc138\uacc4\uc640 \uac19\uc740 \uc2e0\uc559\uc740 \ubbf8\ub77c\ub098 \ud53c\ub77c\ubbf8\ub4dc\uc640 \uac19\uc740 \ub3c5\ud2b9\ud55c \ub9e4\uc7a5\ubc29\uc2dd\uc744 \ud0c4\uc0dd\ud558\uac8c \ud558\uc600\ub2e4.", "answer": "\ud53c\ub77c\ubbf8\ub4dc", "sentence": "\ub610\ud55c \uc0ac\ud6c4 \uc138\uacc4\uc640 \uac19\uc740 \uc2e0\uc559\uc740 \ubbf8\ub77c\ub098 \ud53c\ub77c\ubbf8\ub4dc \uc640 \uac19\uc740 \ub3c5\ud2b9\ud55c \ub9e4\uc7a5\ubc29\uc2dd\uc744 \ud0c4\uc0dd\ud558\uac8c \ud558\uc600\ub2e4.", "paragraph_sentence": "\uace0\ub300 \uc774\uc9d1\ud2b8 \uc885\uad50(\u53e4\u4ee3-\u5b97\u654e)\ub294 \uace0\ub300 \uc774\uc9d1\ud2b8 \uc0ac\ub78c\ub4e4\uc774 \uccab \uc655\uc870\ubd80\ud130 \uae30\ub3c5\uad50 \uc720\uc785\uae30\uae4c\uc9c0 \uc57d 3\ucc9c \ub144 \ub118\uac8c \uc720\uc9c0\ud55c \ub2e4\uc591\ud55c \uc2e0\uc559\uacfc \uc7a5\ub840 \uc758\uc2dd\uc744 \ud3ec\ud568\ud558\ub294 \uc885\uad50\uc774\ub2e4. \uc774 \ubbff\uc74c\uc758 \uc911\uc2ec\uc5d0\ub294 \uc628\uac16 \uc790\uc5f0\uc758 \ud798\uc744 \uc0c1\uc9d5\ud558\ub294 \ub2e4\uc591\ud55c \uc2e0\ub4e4\uc774 \uc788\ub2e4. \uc774 \uc2e0\ub4e4\uc740 \uac01\uac01 \ub2e4\ub978 \uac1c\uc131\uc744 \uac00\uc9c0\uace0 \uc788\uc73c\ub098, \uc774\uc9d1\ud2b8 \uc81c18\uc655\uc870\uc5d0 \ub4e4\uc5b4\uc11c\uc11c\ub294 \uc544\ubb38\uacfc \uac19\uc740 \ud558\ub098\uc758 \uc2e0\uc774 \ub2e4\uc591\ud55c \uc778\uaca9\uccb4\uc640 \ubaa8\ub4e0 \uc2e0\uc801 \uad8c\ub2a5\uc744 \ud3ec\ud568\ud558\ub294 \uac83\uc73c\ub85c \uace0\ub300 \uc774\uc9d1\ud2b8 \uc885\uad50\uc758 \uc2e0\ud559\uc774 \ubcc0\ud574\uac10\uc5d0 \ub530\ub77c \ud604\uc7ac \uae30\ub3c5\uad50\uc5d0\uc11c \ucc3e\uc744 \uc218 \uc788\ub294 \uc0bc\uc704\uc77c\uccb4\uc758 \uae30\uc6d0\uc774 \ub418\ub294 \ubaa8\uc2b5\ub3c4 \ubcf4\uc774\uace0 \uc788\ub2e4. \uc774\uc9d1\ud2b8\uc758 \uc2e0\ub4e4\uc740 \uac01 \uac00\uc815\uc9d1\uc774\ub098 \uc9c0\uc5ed\uc758 \uc911\uc2ec \uc2e0\uc804\uc5d0\uc11c \uc7ac\ubb3c \ubd09\ub0a9\uacfc \uae30\ub3c4\ub85c \uc22d\ubc30\ub418\uc5c8\ub294\ub370, \uac01 \uc774\uc9d1\ud2b8\uc758 \uc655\uc870\ub9c8\ub2e4 \uba87\uba87 \uc2e0\ub4e4\uc774 \ub450\ub4dc\ub7ec\uc9c0\uac8c \uc22d\ubc30\ub418\uac70\ub098 \uc2e0\ub4e4\uc774 \uc5f0\ud569\ub418\uae30\ub3c4 \ud558\uc600\uc73c\uba70 \uc774\ub294 \uac01 \uc655\uc870\uc758 \ud30c\ub77c\uc624 \uc774\ub984\uc5d0\uc11c\ub3c4 \ucc3e\uc544\ubcfc \uc218 \uc788\ub2e4. \uc774 \uc885\uad50\ub294 \ub2e4\uc591\ud55c \uc2e0\ub150\uc744 \ud3ec\ud568\ud558\uace0 \uc788\ub294\ub370, \uac00\ub839 \ud30c\ub77c\uc624\uc758 \uc2e0\uc131\uacfc \uac19\uc740 \uad50\ub9ac\ub294 \uc774\uc9d1\ud2b8\uc758 \uc815\uce58\uc801 \uc548\uc815\uc5d0 \uc774\ubc14\uc9c0\ud558\uc600\ub2e4. \ub610\ud55c \uc0ac\ud6c4 \uc138\uacc4\uc640 \uac19\uc740 \uc2e0\uc559\uc740 \ubbf8\ub77c\ub098 \ud53c\ub77c\ubbf8\ub4dc \uc640 \uac19\uc740 \ub3c5\ud2b9\ud55c \ub9e4\uc7a5\ubc29\uc2dd\uc744 \ud0c4\uc0dd\ud558\uac8c \ud558\uc600\ub2e4. ", "paragraph_answer": "\uace0\ub300 \uc774\uc9d1\ud2b8 \uc885\uad50(\u53e4\u4ee3-\u5b97\u654e)\ub294 \uace0\ub300 \uc774\uc9d1\ud2b8 \uc0ac\ub78c\ub4e4\uc774 \uccab \uc655\uc870\ubd80\ud130 \uae30\ub3c5\uad50 \uc720\uc785\uae30\uae4c\uc9c0 \uc57d 3\ucc9c \ub144 \ub118\uac8c \uc720\uc9c0\ud55c \ub2e4\uc591\ud55c \uc2e0\uc559\uacfc \uc7a5\ub840 \uc758\uc2dd\uc744 \ud3ec\ud568\ud558\ub294 \uc885\uad50\uc774\ub2e4. \uc774 \ubbff\uc74c\uc758 \uc911\uc2ec\uc5d0\ub294 \uc628\uac16 \uc790\uc5f0\uc758 \ud798\uc744 \uc0c1\uc9d5\ud558\ub294 \ub2e4\uc591\ud55c \uc2e0\ub4e4\uc774 \uc788\ub2e4. \uc774 \uc2e0\ub4e4\uc740 \uac01\uac01 \ub2e4\ub978 \uac1c\uc131\uc744 \uac00\uc9c0\uace0 \uc788\uc73c\ub098, \uc774\uc9d1\ud2b8 \uc81c18\uc655\uc870\uc5d0 \ub4e4\uc5b4\uc11c\uc11c\ub294 \uc544\ubb38\uacfc \uac19\uc740 \ud558\ub098\uc758 \uc2e0\uc774 \ub2e4\uc591\ud55c \uc778\uaca9\uccb4\uc640 \ubaa8\ub4e0 \uc2e0\uc801 \uad8c\ub2a5\uc744 \ud3ec\ud568\ud558\ub294 \uac83\uc73c\ub85c \uace0\ub300 \uc774\uc9d1\ud2b8 \uc885\uad50\uc758 \uc2e0\ud559\uc774 \ubcc0\ud574\uac10\uc5d0 \ub530\ub77c \ud604\uc7ac \uae30\ub3c5\uad50\uc5d0\uc11c \ucc3e\uc744 \uc218 \uc788\ub294 \uc0bc\uc704\uc77c\uccb4\uc758 \uae30\uc6d0\uc774 \ub418\ub294 \ubaa8\uc2b5\ub3c4 \ubcf4\uc774\uace0 \uc788\ub2e4. \uc774\uc9d1\ud2b8\uc758 \uc2e0\ub4e4\uc740 \uac01 \uac00\uc815\uc9d1\uc774\ub098 \uc9c0\uc5ed\uc758 \uc911\uc2ec \uc2e0\uc804\uc5d0\uc11c \uc7ac\ubb3c \ubd09\ub0a9\uacfc \uae30\ub3c4\ub85c \uc22d\ubc30\ub418\uc5c8\ub294\ub370, \uac01 \uc774\uc9d1\ud2b8\uc758 \uc655\uc870\ub9c8\ub2e4 \uba87\uba87 \uc2e0\ub4e4\uc774 \ub450\ub4dc\ub7ec\uc9c0\uac8c \uc22d\ubc30\ub418\uac70\ub098 \uc2e0\ub4e4\uc774 \uc5f0\ud569\ub418\uae30\ub3c4 \ud558\uc600\uc73c\uba70 \uc774\ub294 \uac01 \uc655\uc870\uc758 \ud30c\ub77c\uc624 \uc774\ub984\uc5d0\uc11c\ub3c4 \ucc3e\uc544\ubcfc \uc218 \uc788\ub2e4. \uc774 \uc885\uad50\ub294 \ub2e4\uc591\ud55c \uc2e0\ub150\uc744 \ud3ec\ud568\ud558\uace0 \uc788\ub294\ub370, \uac00\ub839 \ud30c\ub77c\uc624\uc758 \uc2e0\uc131\uacfc \uac19\uc740 \uad50\ub9ac\ub294 \uc774\uc9d1\ud2b8\uc758 \uc815\uce58\uc801 \uc548\uc815\uc5d0 \uc774\ubc14\uc9c0\ud558\uc600\ub2e4. \ub610\ud55c \uc0ac\ud6c4 \uc138\uacc4\uc640 \uac19\uc740 \uc2e0\uc559\uc740 \ubbf8\ub77c\ub098 \ud53c\ub77c\ubbf8\ub4dc \uc640 \uac19\uc740 \ub3c5\ud2b9\ud55c \ub9e4\uc7a5\ubc29\uc2dd\uc744 \ud0c4\uc0dd\ud558\uac8c \ud558\uc600\ub2e4.", "sentence_answer": "\ub610\ud55c \uc0ac\ud6c4 \uc138\uacc4\uc640 \uac19\uc740 \uc2e0\uc559\uc740 \ubbf8\ub77c\ub098 \ud53c\ub77c\ubbf8\ub4dc \uc640 \uac19\uc740 \ub3c5\ud2b9\ud55c \ub9e4\uc7a5\ubc29\uc2dd\uc744 \ud0c4\uc0dd\ud558\uac8c \ud558\uc600\ub2e4.", "paragraph_id": "6511617-0-2", "paragraph_question": "question: \uace0\ub300 \uc774\uc9d1\ud2b8\uc758 \uc0ac\ud6c4 \uc138\uacc4 \uc2e0\uc559\uc744 \uc5ff\ubcfc \uc218 \uc788\ub294 \ub300\ud45c\uc801\uc778 \uac83\uc73c\ub85c \ubbf8\ub77c\uc640 \ud568\uaed8 \ub3c5\ud2b9\ud55c \ub9e4\uc7a5\ubc29\uc2dd\uc758 \ud0c4\uc0dd\uc73c\ub85c \ubcfc \uc218 \uc788\ub294 \uac83\uc740?, context: \uace0\ub300 \uc774\uc9d1\ud2b8 \uc885\uad50(\u53e4\u4ee3-\u5b97\u654e)\ub294 \uace0\ub300 \uc774\uc9d1\ud2b8 \uc0ac\ub78c\ub4e4\uc774 \uccab \uc655\uc870\ubd80\ud130 \uae30\ub3c5\uad50 \uc720\uc785\uae30\uae4c\uc9c0 \uc57d 3\ucc9c \ub144 \ub118\uac8c \uc720\uc9c0\ud55c \ub2e4\uc591\ud55c \uc2e0\uc559\uacfc \uc7a5\ub840 \uc758\uc2dd\uc744 \ud3ec\ud568\ud558\ub294 \uc885\uad50\uc774\ub2e4. \uc774 \ubbff\uc74c\uc758 \uc911\uc2ec\uc5d0\ub294 \uc628\uac16 \uc790\uc5f0\uc758 \ud798\uc744 \uc0c1\uc9d5\ud558\ub294 \ub2e4\uc591\ud55c \uc2e0\ub4e4\uc774 \uc788\ub2e4. \uc774 \uc2e0\ub4e4\uc740 \uac01\uac01 \ub2e4\ub978 \uac1c\uc131\uc744 \uac00\uc9c0\uace0 \uc788\uc73c\ub098, \uc774\uc9d1\ud2b8 \uc81c18\uc655\uc870\uc5d0 \ub4e4\uc5b4\uc11c\uc11c\ub294 \uc544\ubb38\uacfc \uac19\uc740 \ud558\ub098\uc758 \uc2e0\uc774 \ub2e4\uc591\ud55c \uc778\uaca9\uccb4\uc640 \ubaa8\ub4e0 \uc2e0\uc801 \uad8c\ub2a5\uc744 \ud3ec\ud568\ud558\ub294 \uac83\uc73c\ub85c \uace0\ub300 \uc774\uc9d1\ud2b8 \uc885\uad50\uc758 \uc2e0\ud559\uc774 \ubcc0\ud574\uac10\uc5d0 \ub530\ub77c \ud604\uc7ac \uae30\ub3c5\uad50\uc5d0\uc11c \ucc3e\uc744 \uc218 \uc788\ub294 \uc0bc\uc704\uc77c\uccb4\uc758 \uae30\uc6d0\uc774 \ub418\ub294 \ubaa8\uc2b5\ub3c4 \ubcf4\uc774\uace0 \uc788\ub2e4. \uc774\uc9d1\ud2b8\uc758 \uc2e0\ub4e4\uc740 \uac01 \uac00\uc815\uc9d1\uc774\ub098 \uc9c0\uc5ed\uc758 \uc911\uc2ec \uc2e0\uc804\uc5d0\uc11c \uc7ac\ubb3c \ubd09\ub0a9\uacfc \uae30\ub3c4\ub85c \uc22d\ubc30\ub418\uc5c8\ub294\ub370, \uac01 \uc774\uc9d1\ud2b8\uc758 \uc655\uc870\ub9c8\ub2e4 \uba87\uba87 \uc2e0\ub4e4\uc774 \ub450\ub4dc\ub7ec\uc9c0\uac8c \uc22d\ubc30\ub418\uac70\ub098 \uc2e0\ub4e4\uc774 \uc5f0\ud569\ub418\uae30\ub3c4 \ud558\uc600\uc73c\uba70 \uc774\ub294 \uac01 \uc655\uc870\uc758 \ud30c\ub77c\uc624 \uc774\ub984\uc5d0\uc11c\ub3c4 \ucc3e\uc544\ubcfc \uc218 \uc788\ub2e4. \uc774 \uc885\uad50\ub294 \ub2e4\uc591\ud55c \uc2e0\ub150\uc744 \ud3ec\ud568\ud558\uace0 \uc788\ub294\ub370, \uac00\ub839 \ud30c\ub77c\uc624\uc758 \uc2e0\uc131\uacfc \uac19\uc740 \uad50\ub9ac\ub294 \uc774\uc9d1\ud2b8\uc758 \uc815\uce58\uc801 \uc548\uc815\uc5d0 \uc774\ubc14\uc9c0\ud558\uc600\ub2e4. \ub610\ud55c \uc0ac\ud6c4 \uc138\uacc4\uc640 \uac19\uc740 \uc2e0\uc559\uc740 \ubbf8\ub77c\ub098 \ud53c\ub77c\ubbf8\ub4dc\uc640 \uac19\uc740 \ub3c5\ud2b9\ud55c \ub9e4\uc7a5\ubc29\uc2dd\uc744 \ud0c4\uc0dd\ud558\uac8c \ud558\uc600\ub2e4."} {"question": "1921\ub144 \ud130\ucee4\uac00 \uc0c8\ub85c\uc6b4 \uc18c\uc18d\ud300 \ud654\uc774\ud2b8\uc0ad\uc2a4\uc5d0\uc11c \ub9c8\uc9c0\ub9c9 \uc678\uc57c \uc790\ub9ac\ub97c \ub193\uace0 \uacbd\uc7c1\ud55c \uc0ac\ub78c\uc740?", "paragraph": "\ud074\ub77d\uc2a4\ub370\uc77c 2\ub144\ucc28\uc774\ub358 1940\ub144\uc740 \ud130\ucee4\uc5d0\uac8c \uc788\uc5b4\uc11c \uc0c8\ub85c\uc6b4 \ubcc0\ud601\uc758 \uc2dc\uc98c\uc774\uc5c8\ub2e4. 6\uc6d4\ub9d0\uae4c\uc9c0 .374\uc758 \ud0c0\uc728\ub85c \ucf54\ud2bc \uc2a4\ud14c\uc774\ud2b8 \ub9ac\uadf8 \ud0c0\uaca9 1\uc704\ub97c \ub2ec\ub838\ub2e4. \ubaa8\ub450 97\uacbd\uae30\uc5d0 \ucd9c\uc804\ud574 .390\uc758 \uace0\uac10\ub3c4\uc758 \ud0c0\uaca9\uc744 \uc120\ubcf4\uc600\uc73c\uba70, \uc624\ud074\ub77c\ud638\ub9c8\uc2dc\ud2f0 \uc778\ub514\uc5b8\uc2a4\ub85c \uc2b9\uaca9\ub418\uc5b4 \uadf8\uacf3\uc5d0\uc11c 40\uacbd\uae30\ub97c \ub6f0\uc5c8\ub2e4. \uadf8\ub7f0 \uadf8\uc758 \ud65c\uc57d\uc744 \ub208\uc5ec\uaca8 \ubcf4\uace0 \uc788\ub358 \uc2dc\uce74\uace0 \ud654\uc774\ud2b8\uc0ad\uc2a4\uc758 \ud31c \ub9e4\ub2c8\uc800 \ube4c\ub9ac \uc6f9\uc740 \uc2dc\uc98c \ud6c4 \uc624\ud074\ub77c\ud638\ub9c8\uc2dc\ud2f0\uc5d0\uac8c\uc11c \uadf8\uc758 \uacc4\uc57d\uc744 \uc0ac\ub4e4\uc600\ub2e4. 1941\ub144 \uc2dc\uc98c \uc2dc\uc791\uacfc \ud568\uaed8 \ud130\ucee4\ub294 \uc0c8\ub85c\uc6b4 \uc18c\uc18d\ud300 \ud654\uc774\ud2b8\uc0ad\uc2a4\uc5d0\uc11c \ub370\uc774\ube0c \uc1fc\ud2b8 \ub4f1\uacfc \ub9c8\uc9c0\ub9c9 \uc678\uc57c \uc790\ub9ac\ub97c \ub193\uace0 \uacbd\uc7c1\uc744 \ubc8c\uc600\uc9c0\ub9cc \ubc00\ub824\ub0ac\uace0, \uadf8\ud574 \uc2dc\uc98c\uc744 \ub85c\uc800\uc2a4 \ud63c\uc988\ube44\uac00 \uc774\ub044\ub294 \uc624\ud074\ub77c\ud638\ub9c8\uc2dc\ud2f0 \uc778\ub514\uc5b8\uc2a4\uc5d0\uc11c \ubcf4\ub0c8\ub2e4. \uc778\ub514\uc5b8\uc2a4\uc5d0\uc11c\uc758 \uc2dc\uc98c \uc131\uc801\uc740 141\uacbd\uae30\uc5d0\uc11c \ud0c0\uc728 .246, 3\ub8e8\ud0c0 12\uac1c\ub97c \uae30\ub85d\ud588\ub2e4.", "answer": "\ub370\uc774\ube0c \uc1fc\ud2b8", "sentence": "1941\ub144 \uc2dc\uc98c \uc2dc\uc791\uacfc \ud568\uaed8 \ud130\ucee4\ub294 \uc0c8\ub85c\uc6b4 \uc18c\uc18d\ud300 \ud654\uc774\ud2b8\uc0ad\uc2a4\uc5d0\uc11c \ub370\uc774\ube0c \uc1fc\ud2b8 \ub4f1\uacfc \ub9c8\uc9c0\ub9c9 \uc678\uc57c \uc790\ub9ac\ub97c \ub193\uace0 \uacbd\uc7c1\uc744 \ubc8c\uc600\uc9c0\ub9cc \ubc00\ub824\ub0ac\uace0, \uadf8\ud574 \uc2dc\uc98c\uc744 \ub85c\uc800\uc2a4 \ud63c\uc988\ube44\uac00 \uc774\ub044\ub294 \uc624\ud074\ub77c\ud638\ub9c8\uc2dc\ud2f0 \uc778\ub514\uc5b8\uc2a4\uc5d0\uc11c \ubcf4\ub0c8\ub2e4.", "paragraph_sentence": "\ud074\ub77d\uc2a4\ub370\uc77c 2\ub144\ucc28\uc774\ub358 1940\ub144\uc740 \ud130\ucee4\uc5d0\uac8c \uc788\uc5b4\uc11c \uc0c8\ub85c\uc6b4 \ubcc0\ud601\uc758 \uc2dc\uc98c\uc774\uc5c8\ub2e4. 6\uc6d4\ub9d0\uae4c\uc9c0 .374\uc758 \ud0c0\uc728\ub85c \ucf54\ud2bc \uc2a4\ud14c\uc774\ud2b8 \ub9ac\uadf8 \ud0c0\uaca9 1\uc704\ub97c \ub2ec\ub838\ub2e4. \ubaa8\ub450 97\uacbd\uae30\uc5d0 \ucd9c\uc804\ud574 .390\uc758 \uace0\uac10\ub3c4\uc758 \ud0c0\uaca9\uc744 \uc120\ubcf4\uc600\uc73c\uba70, \uc624\ud074\ub77c\ud638\ub9c8\uc2dc\ud2f0 \uc778\ub514\uc5b8\uc2a4\ub85c \uc2b9\uaca9\ub418\uc5b4 \uadf8\uacf3\uc5d0\uc11c 40\uacbd\uae30\ub97c \ub6f0\uc5c8\ub2e4. \uadf8\ub7f0 \uadf8\uc758 \ud65c\uc57d\uc744 \ub208\uc5ec\uaca8 \ubcf4\uace0 \uc788\ub358 \uc2dc\uce74\uace0 \ud654\uc774\ud2b8\uc0ad\uc2a4\uc758 \ud31c \ub9e4\ub2c8\uc800 \ube4c\ub9ac \uc6f9\uc740 \uc2dc\uc98c \ud6c4 \uc624\ud074\ub77c\ud638\ub9c8\uc2dc\ud2f0\uc5d0\uac8c\uc11c \uadf8\uc758 \uacc4\uc57d\uc744 \uc0ac\ub4e4\uc600\ub2e4. 1941\ub144 \uc2dc\uc98c \uc2dc\uc791\uacfc \ud568\uaed8 \ud130\ucee4\ub294 \uc0c8\ub85c\uc6b4 \uc18c\uc18d\ud300 \ud654\uc774\ud2b8\uc0ad\uc2a4\uc5d0\uc11c \ub370\uc774\ube0c \uc1fc\ud2b8 \ub4f1\uacfc \ub9c8\uc9c0\ub9c9 \uc678\uc57c \uc790\ub9ac\ub97c \ub193\uace0 \uacbd\uc7c1\uc744 \ubc8c\uc600\uc9c0\ub9cc \ubc00\ub824\ub0ac\uace0, \uadf8\ud574 \uc2dc\uc98c\uc744 \ub85c\uc800\uc2a4 \ud63c\uc988\ube44\uac00 \uc774\ub044\ub294 \uc624\ud074\ub77c\ud638\ub9c8\uc2dc\ud2f0 \uc778\ub514\uc5b8\uc2a4\uc5d0\uc11c \ubcf4\ub0c8\ub2e4. \uc778\ub514\uc5b8\uc2a4\uc5d0\uc11c\uc758 \uc2dc\uc98c \uc131\uc801\uc740 141\uacbd\uae30\uc5d0\uc11c \ud0c0\uc728 .246, 3\ub8e8\ud0c0 12\uac1c\ub97c \uae30\ub85d\ud588\ub2e4.", "paragraph_answer": "\ud074\ub77d\uc2a4\ub370\uc77c 2\ub144\ucc28\uc774\ub358 1940\ub144\uc740 \ud130\ucee4\uc5d0\uac8c \uc788\uc5b4\uc11c \uc0c8\ub85c\uc6b4 \ubcc0\ud601\uc758 \uc2dc\uc98c\uc774\uc5c8\ub2e4. 6\uc6d4\ub9d0\uae4c\uc9c0 .374\uc758 \ud0c0\uc728\ub85c \ucf54\ud2bc \uc2a4\ud14c\uc774\ud2b8 \ub9ac\uadf8 \ud0c0\uaca9 1\uc704\ub97c \ub2ec\ub838\ub2e4. \ubaa8\ub450 97\uacbd\uae30\uc5d0 \ucd9c\uc804\ud574 .390\uc758 \uace0\uac10\ub3c4\uc758 \ud0c0\uaca9\uc744 \uc120\ubcf4\uc600\uc73c\uba70, \uc624\ud074\ub77c\ud638\ub9c8\uc2dc\ud2f0 \uc778\ub514\uc5b8\uc2a4\ub85c \uc2b9\uaca9\ub418\uc5b4 \uadf8\uacf3\uc5d0\uc11c 40\uacbd\uae30\ub97c \ub6f0\uc5c8\ub2e4. \uadf8\ub7f0 \uadf8\uc758 \ud65c\uc57d\uc744 \ub208\uc5ec\uaca8 \ubcf4\uace0 \uc788\ub358 \uc2dc\uce74\uace0 \ud654\uc774\ud2b8\uc0ad\uc2a4\uc758 \ud31c \ub9e4\ub2c8\uc800 \ube4c\ub9ac \uc6f9\uc740 \uc2dc\uc98c \ud6c4 \uc624\ud074\ub77c\ud638\ub9c8\uc2dc\ud2f0\uc5d0\uac8c\uc11c \uadf8\uc758 \uacc4\uc57d\uc744 \uc0ac\ub4e4\uc600\ub2e4. 1941\ub144 \uc2dc\uc98c \uc2dc\uc791\uacfc \ud568\uaed8 \ud130\ucee4\ub294 \uc0c8\ub85c\uc6b4 \uc18c\uc18d\ud300 \ud654\uc774\ud2b8\uc0ad\uc2a4\uc5d0\uc11c \ub370\uc774\ube0c \uc1fc\ud2b8 \ub4f1\uacfc \ub9c8\uc9c0\ub9c9 \uc678\uc57c \uc790\ub9ac\ub97c \ub193\uace0 \uacbd\uc7c1\uc744 \ubc8c\uc600\uc9c0\ub9cc \ubc00\ub824\ub0ac\uace0, \uadf8\ud574 \uc2dc\uc98c\uc744 \ub85c\uc800\uc2a4 \ud63c\uc988\ube44\uac00 \uc774\ub044\ub294 \uc624\ud074\ub77c\ud638\ub9c8\uc2dc\ud2f0 \uc778\ub514\uc5b8\uc2a4\uc5d0\uc11c \ubcf4\ub0c8\ub2e4. \uc778\ub514\uc5b8\uc2a4\uc5d0\uc11c\uc758 \uc2dc\uc98c \uc131\uc801\uc740 141\uacbd\uae30\uc5d0\uc11c \ud0c0\uc728 .246, 3\ub8e8\ud0c0 12\uac1c\ub97c \uae30\ub85d\ud588\ub2e4.", "sentence_answer": "1941\ub144 \uc2dc\uc98c \uc2dc\uc791\uacfc \ud568\uaed8 \ud130\ucee4\ub294 \uc0c8\ub85c\uc6b4 \uc18c\uc18d\ud300 \ud654\uc774\ud2b8\uc0ad\uc2a4\uc5d0\uc11c \ub370\uc774\ube0c \uc1fc\ud2b8 \ub4f1\uacfc \ub9c8\uc9c0\ub9c9 \uc678\uc57c \uc790\ub9ac\ub97c \ub193\uace0 \uacbd\uc7c1\uc744 \ubc8c\uc600\uc9c0\ub9cc \ubc00\ub824\ub0ac\uace0, \uadf8\ud574 \uc2dc\uc98c\uc744 \ub85c\uc800\uc2a4 \ud63c\uc988\ube44\uac00 \uc774\ub044\ub294 \uc624\ud074\ub77c\ud638\ub9c8\uc2dc\ud2f0 \uc778\ub514\uc5b8\uc2a4\uc5d0\uc11c \ubcf4\ub0c8\ub2e4.", "paragraph_id": "6511496-1-2", "paragraph_question": "question: 1921\ub144 \ud130\ucee4\uac00 \uc0c8\ub85c\uc6b4 \uc18c\uc18d\ud300 \ud654\uc774\ud2b8\uc0ad\uc2a4\uc5d0\uc11c \ub9c8\uc9c0\ub9c9 \uc678\uc57c \uc790\ub9ac\ub97c \ub193\uace0 \uacbd\uc7c1\ud55c \uc0ac\ub78c\uc740?, context: \ud074\ub77d\uc2a4\ub370\uc77c 2\ub144\ucc28\uc774\ub358 1940\ub144\uc740 \ud130\ucee4\uc5d0\uac8c \uc788\uc5b4\uc11c \uc0c8\ub85c\uc6b4 \ubcc0\ud601\uc758 \uc2dc\uc98c\uc774\uc5c8\ub2e4. 6\uc6d4\ub9d0\uae4c\uc9c0 .374\uc758 \ud0c0\uc728\ub85c \ucf54\ud2bc \uc2a4\ud14c\uc774\ud2b8 \ub9ac\uadf8 \ud0c0\uaca9 1\uc704\ub97c \ub2ec\ub838\ub2e4. \ubaa8\ub450 97\uacbd\uae30\uc5d0 \ucd9c\uc804\ud574 .390\uc758 \uace0\uac10\ub3c4\uc758 \ud0c0\uaca9\uc744 \uc120\ubcf4\uc600\uc73c\uba70, \uc624\ud074\ub77c\ud638\ub9c8\uc2dc\ud2f0 \uc778\ub514\uc5b8\uc2a4\ub85c \uc2b9\uaca9\ub418\uc5b4 \uadf8\uacf3\uc5d0\uc11c 40\uacbd\uae30\ub97c \ub6f0\uc5c8\ub2e4. \uadf8\ub7f0 \uadf8\uc758 \ud65c\uc57d\uc744 \ub208\uc5ec\uaca8 \ubcf4\uace0 \uc788\ub358 \uc2dc\uce74\uace0 \ud654\uc774\ud2b8\uc0ad\uc2a4\uc758 \ud31c \ub9e4\ub2c8\uc800 \ube4c\ub9ac \uc6f9\uc740 \uc2dc\uc98c \ud6c4 \uc624\ud074\ub77c\ud638\ub9c8\uc2dc\ud2f0\uc5d0\uac8c\uc11c \uadf8\uc758 \uacc4\uc57d\uc744 \uc0ac\ub4e4\uc600\ub2e4. 1941\ub144 \uc2dc\uc98c \uc2dc\uc791\uacfc \ud568\uaed8 \ud130\ucee4\ub294 \uc0c8\ub85c\uc6b4 \uc18c\uc18d\ud300 \ud654\uc774\ud2b8\uc0ad\uc2a4\uc5d0\uc11c \ub370\uc774\ube0c \uc1fc\ud2b8 \ub4f1\uacfc \ub9c8\uc9c0\ub9c9 \uc678\uc57c \uc790\ub9ac\ub97c \ub193\uace0 \uacbd\uc7c1\uc744 \ubc8c\uc600\uc9c0\ub9cc \ubc00\ub824\ub0ac\uace0, \uadf8\ud574 \uc2dc\uc98c\uc744 \ub85c\uc800\uc2a4 \ud63c\uc988\ube44\uac00 \uc774\ub044\ub294 \uc624\ud074\ub77c\ud638\ub9c8\uc2dc\ud2f0 \uc778\ub514\uc5b8\uc2a4\uc5d0\uc11c \ubcf4\ub0c8\ub2e4. \uc778\ub514\uc5b8\uc2a4\uc5d0\uc11c\uc758 \uc2dc\uc98c \uc131\uc801\uc740 141\uacbd\uae30\uc5d0\uc11c \ud0c0\uc728 .246, 3\ub8e8\ud0c0 12\uac1c\ub97c \uae30\ub85d\ud588\ub2e4."} {"question": "\ubc15\uc21c\uc774 \ud568\ud765\uae4c\uc9c0 \ub04c\uace0 \uac14\ub358 \uac83\uc740?", "paragraph": "\ubc15\uc21c\uc740 \uc0c8\ub07c \ub538\ub9b0 \uc5b4\ubbf8 \ub9d0\uc744 \ud568\ud765\uae4c\uc9c0 \ub04c\uace0 \uac00\uc11c \u201c\uc5b4\ubbf8\ub97c \ub530\ub974\ub294 \ub9d0\ub3c4 \uc800\ub7ec\ud55c\ub370 \ud558\ubb3c\uba70 \uc778\uac04\uc758 \ubd80\ubaa8\uc5d0 \ub300\ud55c \uc815\uc774\uc57c \uc5bc\ub9c8\ub098 \uae4a\uaca0\uc2b5\ub2c8\uae4c?\u201d\ub77c\uba70 \ud0dc\uc870\uc758 \uadc0\uacbd\uc744 \uc124\ub4dd\ud588\ub2e4. \uba70\uce60\uc744 \ubb35\uc73c\uba70, \ud558\ub8e8\ub294 \ud0dc\uc870\uc640 \ubc15\uc21c\uc774 \uc7a5\uae30\ub97c \ub450\ub294\ub370 \ucc9c\uc7a5\uc5d0\uc11c \uc950\uac00 \uc0c8\ub07c\ub97c \uc548\uace0 \ub5a8\uc5b4\uc838 \uc8fd\uc744 \uc9c0\uacbd\uc774 \ub418\uc5c8\uc73c\ub098 \uc11c\ub85c \ub5a8\uc5b4\uc9c0\uc9c0 \uc54a\uc558\ub2e4. \uc774\uc5d0 \ubc15\uc21c\uc774 \uc7a5\uae30\ud310\uc744 \uc81c\uccd0\ub193\uace0 \ub208\ubb3c\uc744 \ud758\ub9ac\uba70 \ub3cc\uc544\uac00\uc790\uace0 \uccad\ud558\ub2c8 \ud0dc\uc870\uac00 \ub3cc\uc544\uac00\uae30\ub85c \uacb0\uc2ec\ud588\ub2e4\uace0 \ud55c\ub2e4. \uadf8\ub7ec\ub098 \uc8fc\ubcc0 \uc0ac\ub78c\uc774 \ubc15\uc21c\uc744 \uc8fd\uc5ec\uc57c \ud55c\ub2e4\uace0 \ud0dc\uc870\uc5d0\uac8c \uace0\ud558\uc790, \ud0dc\uc870\ub294 \uc774\ubbf8 \ubc15\uc21c\uc774 \uc6a9\ud765\uac15\uc744 \uac74\ub11c\uc73c\ub9ac\ub77c \uc5ec\uae30\uc5b4 \u201c\uc6a9\ud765\uac15\uc744 \uac74\ub11c\ub2e4\uba74 \ucad3\uc9c0 \ub9d0\ub77c\u201d\ub77c\uace0 \uba85\ub839\ud588\ub294\ub370, \uc548\ud0c0\uae5d\uac8c\ub3c4 \uadf8\ub294 \ubcd1\uc774 \ub098\uc11c \uc26c\ub2e4\uac00 \uc6a9\ud765\uac15\uc744 \uac74\ub110 \ub54c, \ucad3\uc544 \uc628 \ubcd1\uc0ac\uc5d0 \uc758\ud574 \uc8fd\uc74c\uc744 \ub9de\uc558\ub2e4. \uc774 \uc774\uc57c\uae30\ub294 \uc5b4\ub514\uae4c\uc9c0\ub098 \uc57c\uc0ac\uc77c \ubfd0 \uc0ac\uc2e4\uc740 \uc544\ub2c8\uba70, \uc2e4\uc81c\ub85c \ubc15\uc21c\uc740 \uc870\uc0ac\uc758\uc758 \ub09c \ub54c \ud568\ud765\uc5d0\uc11c \ub3c4\uc21c\ubb38\uc0ac \ubc15\ub9cc(\u6734\u8513)\uc744 \uc124\ub4dd\ud558\ub2e4\uac00 \uc8fd\uc784\uc744 \ub2f9\ud55c\ub2e4.", "answer": "\uc0c8\ub07c \ub538\ub9b0 \uc5b4\ubbf8 \ub9d0", "sentence": "\ubc15\uc21c\uc740 \uc0c8\ub07c \ub538\ub9b0 \uc5b4\ubbf8 \ub9d0 \uc744 \ud568\ud765\uae4c\uc9c0 \ub04c\uace0 \uac00\uc11c \u201c\uc5b4\ubbf8\ub97c \ub530\ub974\ub294 \ub9d0\ub3c4 \uc800\ub7ec\ud55c\ub370 \ud558\ubb3c\uba70 \uc778\uac04\uc758 \ubd80\ubaa8\uc5d0 \ub300\ud55c \uc815\uc774\uc57c \uc5bc\ub9c8\ub098 \uae4a\uaca0\uc2b5\ub2c8\uae4c?\u201d\ub77c\uba70 \ud0dc\uc870\uc758 \uadc0\uacbd\uc744 \uc124\ub4dd\ud588\ub2e4.", "paragraph_sentence": " \ubc15\uc21c\uc740 \uc0c8\ub07c \ub538\ub9b0 \uc5b4\ubbf8 \ub9d0 \uc744 \ud568\ud765\uae4c\uc9c0 \ub04c\uace0 \uac00\uc11c \u201c\uc5b4\ubbf8\ub97c \ub530\ub974\ub294 \ub9d0\ub3c4 \uc800\ub7ec\ud55c\ub370 \ud558\ubb3c\uba70 \uc778\uac04\uc758 \ubd80\ubaa8\uc5d0 \ub300\ud55c \uc815\uc774\uc57c \uc5bc\ub9c8\ub098 \uae4a\uaca0\uc2b5\ub2c8\uae4c?\u201d\ub77c\uba70 \ud0dc\uc870\uc758 \uadc0\uacbd\uc744 \uc124\ub4dd\ud588\ub2e4. \uba70\uce60\uc744 \ubb35\uc73c\uba70, \ud558\ub8e8\ub294 \ud0dc\uc870\uc640 \ubc15\uc21c\uc774 \uc7a5\uae30\ub97c \ub450\ub294\ub370 \ucc9c\uc7a5\uc5d0\uc11c \uc950\uac00 \uc0c8\ub07c\ub97c \uc548\uace0 \ub5a8\uc5b4\uc838 \uc8fd\uc744 \uc9c0\uacbd\uc774 \ub418\uc5c8\uc73c\ub098 \uc11c\ub85c \ub5a8\uc5b4\uc9c0\uc9c0 \uc54a\uc558\ub2e4. \uc774\uc5d0 \ubc15\uc21c\uc774 \uc7a5\uae30\ud310\uc744 \uc81c\uccd0\ub193\uace0 \ub208\ubb3c\uc744 \ud758\ub9ac\uba70 \ub3cc\uc544\uac00\uc790\uace0 \uccad\ud558\ub2c8 \ud0dc\uc870\uac00 \ub3cc\uc544\uac00\uae30\ub85c \uacb0\uc2ec\ud588\ub2e4\uace0 \ud55c\ub2e4. \uadf8\ub7ec\ub098 \uc8fc\ubcc0 \uc0ac\ub78c\uc774 \ubc15\uc21c\uc744 \uc8fd\uc5ec\uc57c \ud55c\ub2e4\uace0 \ud0dc\uc870\uc5d0\uac8c \uace0\ud558\uc790, \ud0dc\uc870\ub294 \uc774\ubbf8 \ubc15\uc21c\uc774 \uc6a9\ud765\uac15\uc744 \uac74\ub11c\uc73c\ub9ac\ub77c \uc5ec\uae30\uc5b4 \u201c\uc6a9\ud765\uac15\uc744 \uac74\ub11c\ub2e4\uba74 \ucad3\uc9c0 \ub9d0\ub77c\u201d\ub77c\uace0 \uba85\ub839\ud588\ub294\ub370, \uc548\ud0c0\uae5d\uac8c\ub3c4 \uadf8\ub294 \ubcd1\uc774 \ub098\uc11c \uc26c\ub2e4\uac00 \uc6a9\ud765\uac15\uc744 \uac74\ub110 \ub54c, \ucad3\uc544 \uc628 \ubcd1\uc0ac\uc5d0 \uc758\ud574 \uc8fd\uc74c\uc744 \ub9de\uc558\ub2e4. \uc774 \uc774\uc57c\uae30\ub294 \uc5b4\ub514\uae4c\uc9c0\ub098 \uc57c\uc0ac\uc77c \ubfd0 \uc0ac\uc2e4\uc740 \uc544\ub2c8\uba70, \uc2e4\uc81c\ub85c \ubc15\uc21c\uc740 \uc870\uc0ac\uc758\uc758 \ub09c \ub54c \ud568\ud765\uc5d0\uc11c \ub3c4\uc21c\ubb38\uc0ac \ubc15\ub9cc(\u6734\u8513)\uc744 \uc124\ub4dd\ud558\ub2e4\uac00 \uc8fd\uc784\uc744 \ub2f9\ud55c\ub2e4.", "paragraph_answer": "\ubc15\uc21c\uc740 \uc0c8\ub07c \ub538\ub9b0 \uc5b4\ubbf8 \ub9d0 \uc744 \ud568\ud765\uae4c\uc9c0 \ub04c\uace0 \uac00\uc11c \u201c\uc5b4\ubbf8\ub97c \ub530\ub974\ub294 \ub9d0\ub3c4 \uc800\ub7ec\ud55c\ub370 \ud558\ubb3c\uba70 \uc778\uac04\uc758 \ubd80\ubaa8\uc5d0 \ub300\ud55c \uc815\uc774\uc57c \uc5bc\ub9c8\ub098 \uae4a\uaca0\uc2b5\ub2c8\uae4c?\u201d\ub77c\uba70 \ud0dc\uc870\uc758 \uadc0\uacbd\uc744 \uc124\ub4dd\ud588\ub2e4. \uba70\uce60\uc744 \ubb35\uc73c\uba70, \ud558\ub8e8\ub294 \ud0dc\uc870\uc640 \ubc15\uc21c\uc774 \uc7a5\uae30\ub97c \ub450\ub294\ub370 \ucc9c\uc7a5\uc5d0\uc11c \uc950\uac00 \uc0c8\ub07c\ub97c \uc548\uace0 \ub5a8\uc5b4\uc838 \uc8fd\uc744 \uc9c0\uacbd\uc774 \ub418\uc5c8\uc73c\ub098 \uc11c\ub85c \ub5a8\uc5b4\uc9c0\uc9c0 \uc54a\uc558\ub2e4. \uc774\uc5d0 \ubc15\uc21c\uc774 \uc7a5\uae30\ud310\uc744 \uc81c\uccd0\ub193\uace0 \ub208\ubb3c\uc744 \ud758\ub9ac\uba70 \ub3cc\uc544\uac00\uc790\uace0 \uccad\ud558\ub2c8 \ud0dc\uc870\uac00 \ub3cc\uc544\uac00\uae30\ub85c \uacb0\uc2ec\ud588\ub2e4\uace0 \ud55c\ub2e4. \uadf8\ub7ec\ub098 \uc8fc\ubcc0 \uc0ac\ub78c\uc774 \ubc15\uc21c\uc744 \uc8fd\uc5ec\uc57c \ud55c\ub2e4\uace0 \ud0dc\uc870\uc5d0\uac8c \uace0\ud558\uc790, \ud0dc\uc870\ub294 \uc774\ubbf8 \ubc15\uc21c\uc774 \uc6a9\ud765\uac15\uc744 \uac74\ub11c\uc73c\ub9ac\ub77c \uc5ec\uae30\uc5b4 \u201c\uc6a9\ud765\uac15\uc744 \uac74\ub11c\ub2e4\uba74 \ucad3\uc9c0 \ub9d0\ub77c\u201d\ub77c\uace0 \uba85\ub839\ud588\ub294\ub370, \uc548\ud0c0\uae5d\uac8c\ub3c4 \uadf8\ub294 \ubcd1\uc774 \ub098\uc11c \uc26c\ub2e4\uac00 \uc6a9\ud765\uac15\uc744 \uac74\ub110 \ub54c, \ucad3\uc544 \uc628 \ubcd1\uc0ac\uc5d0 \uc758\ud574 \uc8fd\uc74c\uc744 \ub9de\uc558\ub2e4. \uc774 \uc774\uc57c\uae30\ub294 \uc5b4\ub514\uae4c\uc9c0\ub098 \uc57c\uc0ac\uc77c \ubfd0 \uc0ac\uc2e4\uc740 \uc544\ub2c8\uba70, \uc2e4\uc81c\ub85c \ubc15\uc21c\uc740 \uc870\uc0ac\uc758\uc758 \ub09c \ub54c \ud568\ud765\uc5d0\uc11c \ub3c4\uc21c\ubb38\uc0ac \ubc15\ub9cc(\u6734\u8513)\uc744 \uc124\ub4dd\ud558\ub2e4\uac00 \uc8fd\uc784\uc744 \ub2f9\ud55c\ub2e4.", "sentence_answer": "\ubc15\uc21c\uc740 \uc0c8\ub07c \ub538\ub9b0 \uc5b4\ubbf8 \ub9d0 \uc744 \ud568\ud765\uae4c\uc9c0 \ub04c\uace0 \uac00\uc11c \u201c\uc5b4\ubbf8\ub97c \ub530\ub974\ub294 \ub9d0\ub3c4 \uc800\ub7ec\ud55c\ub370 \ud558\ubb3c\uba70 \uc778\uac04\uc758 \ubd80\ubaa8\uc5d0 \ub300\ud55c \uc815\uc774\uc57c \uc5bc\ub9c8\ub098 \uae4a\uaca0\uc2b5\ub2c8\uae4c?\u201d\ub77c\uba70 \ud0dc\uc870\uc758 \uadc0\uacbd\uc744 \uc124\ub4dd\ud588\ub2e4.", "paragraph_id": "6570245-1-1", "paragraph_question": "question: \ubc15\uc21c\uc774 \ud568\ud765\uae4c\uc9c0 \ub04c\uace0 \uac14\ub358 \uac83\uc740?, context: \ubc15\uc21c\uc740 \uc0c8\ub07c \ub538\ub9b0 \uc5b4\ubbf8 \ub9d0\uc744 \ud568\ud765\uae4c\uc9c0 \ub04c\uace0 \uac00\uc11c \u201c\uc5b4\ubbf8\ub97c \ub530\ub974\ub294 \ub9d0\ub3c4 \uc800\ub7ec\ud55c\ub370 \ud558\ubb3c\uba70 \uc778\uac04\uc758 \ubd80\ubaa8\uc5d0 \ub300\ud55c \uc815\uc774\uc57c \uc5bc\ub9c8\ub098 \uae4a\uaca0\uc2b5\ub2c8\uae4c?\u201d\ub77c\uba70 \ud0dc\uc870\uc758 \uadc0\uacbd\uc744 \uc124\ub4dd\ud588\ub2e4. \uba70\uce60\uc744 \ubb35\uc73c\uba70, \ud558\ub8e8\ub294 \ud0dc\uc870\uc640 \ubc15\uc21c\uc774 \uc7a5\uae30\ub97c \ub450\ub294\ub370 \ucc9c\uc7a5\uc5d0\uc11c \uc950\uac00 \uc0c8\ub07c\ub97c \uc548\uace0 \ub5a8\uc5b4\uc838 \uc8fd\uc744 \uc9c0\uacbd\uc774 \ub418\uc5c8\uc73c\ub098 \uc11c\ub85c \ub5a8\uc5b4\uc9c0\uc9c0 \uc54a\uc558\ub2e4. \uc774\uc5d0 \ubc15\uc21c\uc774 \uc7a5\uae30\ud310\uc744 \uc81c\uccd0\ub193\uace0 \ub208\ubb3c\uc744 \ud758\ub9ac\uba70 \ub3cc\uc544\uac00\uc790\uace0 \uccad\ud558\ub2c8 \ud0dc\uc870\uac00 \ub3cc\uc544\uac00\uae30\ub85c \uacb0\uc2ec\ud588\ub2e4\uace0 \ud55c\ub2e4. \uadf8\ub7ec\ub098 \uc8fc\ubcc0 \uc0ac\ub78c\uc774 \ubc15\uc21c\uc744 \uc8fd\uc5ec\uc57c \ud55c\ub2e4\uace0 \ud0dc\uc870\uc5d0\uac8c \uace0\ud558\uc790, \ud0dc\uc870\ub294 \uc774\ubbf8 \ubc15\uc21c\uc774 \uc6a9\ud765\uac15\uc744 \uac74\ub11c\uc73c\ub9ac\ub77c \uc5ec\uae30\uc5b4 \u201c\uc6a9\ud765\uac15\uc744 \uac74\ub11c\ub2e4\uba74 \ucad3\uc9c0 \ub9d0\ub77c\u201d\ub77c\uace0 \uba85\ub839\ud588\ub294\ub370, \uc548\ud0c0\uae5d\uac8c\ub3c4 \uadf8\ub294 \ubcd1\uc774 \ub098\uc11c \uc26c\ub2e4\uac00 \uc6a9\ud765\uac15\uc744 \uac74\ub110 \ub54c, \ucad3\uc544 \uc628 \ubcd1\uc0ac\uc5d0 \uc758\ud574 \uc8fd\uc74c\uc744 \ub9de\uc558\ub2e4. \uc774 \uc774\uc57c\uae30\ub294 \uc5b4\ub514\uae4c\uc9c0\ub098 \uc57c\uc0ac\uc77c \ubfd0 \uc0ac\uc2e4\uc740 \uc544\ub2c8\uba70, \uc2e4\uc81c\ub85c \ubc15\uc21c\uc740 \uc870\uc0ac\uc758\uc758 \ub09c \ub54c \ud568\ud765\uc5d0\uc11c \ub3c4\uc21c\ubb38\uc0ac \ubc15\ub9cc(\u6734\u8513)\uc744 \uc124\ub4dd\ud558\ub2e4\uac00 \uc8fd\uc784\uc744 \ub2f9\ud55c\ub2e4."} {"question": "\uae40\uc740\uc131\uc740 \uad6d\uc815\uc6d0\uc7a5\uc774 \ubd88\ubc95\uac10\uccad\uc5d0 \ub300\ud574 \uc778\uc9c0\ud558\uace0 \uc788\uc5c8\ub2e4\uace0 \uc0dd\uac01\ub41c\ub2e4\ub294 \ub2f5\ubcc0\uc744 \ud588\uace0 \uadf8\uc5d0 \ub300\ud55c \uadfc\uac70\ub85c \uba87 \uac00\uc9c0\ub97c \uc5b8\uae09\ud558\uc600\ub098?", "paragraph": "\uae40\uc740\uc131 \ucc28\uc7a5\uc740 \ub610 2000\ub144 4\uc6d4 \uad6d\uc815\uc6d0 2\ucc28\uc7a5\uc73c\ub85c \ubd80\uc784\ud560 \ubb34\ub835\ubd80\ud130 8\uad6d(\uacfc\ud559\ubcf4\uc548\uad6d)\uc774 \uc624\ub798\uc804\ubd80\ud130 \uad00\ud589\uc801\uc73c\ub85c \ubd88\ubc95\uac10\uccad\uc744 \ud55c \uc0ac\uc2e4\uc744 \uc54c\uace0 \uc788\uc5c8\ub2e4\uace0 \ud138\uc5b4\ub1a8\ub2e4. \uc774\uc5b4 \uc784\ub3d9\uc6d0 \uc804 \uc6d0\uc7a5\uc774 \ubd88\ubc95\uac10\uccad \uc0ac\uc2e4\uc744 \uc54c\uace0 \uc788\uc5c8\ub294\uc9c0\uc5d0 \ub300\ud574 \"\ub2e8\uc815\uc801\uc73c\ub85c \uc54c\uace0 \uc788\uc5c8\ub294\uc9c0\ub294 \ubaa8\ub974\uaca0\ub294\ub370 \uc81c \uc0dd\uac01\uc73c\ub85c \ubcfc \ub54c\ub294 \uc54c\uace0 \uc788\uc5c8\ub2e4\uace0 \uc0dd\uac01\ud55c\ub2e4\"\uba70 \ub9e4\uc77c \ubcf4\uace0\uc11c\uac00 \uc62c\ub77c\uac00\ub294 \ub370 \ud2b9\ubcc4\ud55c \uc870\uce58\uac00 \uc5c6\uc5c8\ub358 \uc810 \ub4f1 4\uac00\uc9c0\ub97c \uadf8 \uadfc\uac70\ub85c \ub4e4\uc5c8\ub2e4. \uadf8\ub294 \"\uad6d\uc815\uc6d0\uc7a5\uc5d0\uac8c \ub9e4\uc77c \ubcf4\uace0\ub418\ub294 \ud1b5\uc2e0\ucca9\ubcf4 \ubcf4\uace0\uc11c\uc5d0 \ub300\ud574 \ub450 \uc6d0\uc7a5\uc740 `\uc774\ub7f0 \uac83\uc774 \ubc16\uc5d0 \ub098\uac00\uc9c0 \uc54a\uac8c \uc870\uc2ec\ud574\uc57c \ud55c\ub2e4'\uace0 \ub9d0\ud55c \uc801\uc774 \uc788\ub2e4\"\ub294 \ub9d0\ub3c4 \ub367\ubd99\uc600\ub2e4. \uae40\uc740\uc131\uc758 \uc9c4\uc220 \uc774\ud6c4 \uad6d\uac00\uc815\ubcf4\uc6d0\uacfc \uc804\ud604\uc9c1 \uc2e4\uad6d\uc7a5, \ub2e8\uc7a5\uae09 \uace0\uc704 \uac04\ubd80\ub4e4\uc5d0 \ub300\ud55c \uc11c\uc6b8\uc911\uc559\uc9c0\uac80\uacfc \ub300\uac80 \uc911\uc218\ubd80\uc758 \uc218\uc0ac\uc640 \uac00\ud0dd \uc870\uc0ac\ub294 \uacc4\uc18d\ub418\uc5c8\ub2e4.", "answer": "4\uac00\uc9c0", "sentence": "\"\uba70 \ub9e4\uc77c \ubcf4\uace0\uc11c\uac00 \uc62c\ub77c\uac00\ub294 \ub370 \ud2b9\ubcc4\ud55c \uc870\uce58\uac00 \uc5c6\uc5c8\ub358 \uc810 \ub4f1 4\uac00\uc9c0 \ub97c \uadf8 \uadfc\uac70\ub85c \ub4e4\uc5c8\ub2e4.", "paragraph_sentence": "\uae40\uc740\uc131 \ucc28\uc7a5\uc740 \ub610 2000\ub144 4\uc6d4 \uad6d\uc815\uc6d0 2\ucc28\uc7a5\uc73c\ub85c \ubd80\uc784\ud560 \ubb34\ub835\ubd80\ud130 8\uad6d(\uacfc\ud559\ubcf4\uc548\uad6d)\uc774 \uc624\ub798\uc804\ubd80\ud130 \uad00\ud589\uc801\uc73c\ub85c \ubd88\ubc95\uac10\uccad\uc744 \ud55c \uc0ac\uc2e4\uc744 \uc54c\uace0 \uc788\uc5c8\ub2e4\uace0 \ud138\uc5b4\ub1a8\ub2e4. \uc774\uc5b4 \uc784\ub3d9\uc6d0 \uc804 \uc6d0\uc7a5\uc774 \ubd88\ubc95\uac10\uccad \uc0ac\uc2e4\uc744 \uc54c\uace0 \uc788\uc5c8\ub294\uc9c0\uc5d0 \ub300\ud574 \"\ub2e8\uc815\uc801\uc73c\ub85c \uc54c\uace0 \uc788\uc5c8\ub294\uc9c0\ub294 \ubaa8\ub974\uaca0\ub294\ub370 \uc81c \uc0dd\uac01\uc73c\ub85c \ubcfc \ub54c\ub294 \uc54c\uace0 \uc788\uc5c8\ub2e4\uace0 \uc0dd\uac01\ud55c\ub2e4 \"\uba70 \ub9e4\uc77c \ubcf4\uace0\uc11c\uac00 \uc62c\ub77c\uac00\ub294 \ub370 \ud2b9\ubcc4\ud55c \uc870\uce58\uac00 \uc5c6\uc5c8\ub358 \uc810 \ub4f1 4\uac00\uc9c0 \ub97c \uadf8 \uadfc\uac70\ub85c \ub4e4\uc5c8\ub2e4. \uadf8\ub294 \"\uad6d\uc815\uc6d0\uc7a5\uc5d0\uac8c \ub9e4\uc77c \ubcf4\uace0\ub418\ub294 \ud1b5\uc2e0\ucca9\ubcf4 \ubcf4\uace0\uc11c\uc5d0 \ub300\ud574 \ub450 \uc6d0\uc7a5\uc740 `\uc774\ub7f0 \uac83\uc774 \ubc16\uc5d0 \ub098\uac00\uc9c0 \uc54a\uac8c \uc870\uc2ec\ud574\uc57c \ud55c\ub2e4'\uace0 \ub9d0\ud55c \uc801\uc774 \uc788\ub2e4\"\ub294 \ub9d0\ub3c4 \ub367\ubd99\uc600\ub2e4. \uae40\uc740\uc131\uc758 \uc9c4\uc220 \uc774\ud6c4 \uad6d\uac00\uc815\ubcf4\uc6d0\uacfc \uc804\ud604\uc9c1 \uc2e4\uad6d\uc7a5, \ub2e8\uc7a5\uae09 \uace0\uc704 \uac04\ubd80\ub4e4\uc5d0 \ub300\ud55c \uc11c\uc6b8\uc911\uc559\uc9c0\uac80\uacfc \ub300\uac80 \uc911\uc218\ubd80\uc758 \uc218\uc0ac\uc640 \uac00\ud0dd \uc870\uc0ac\ub294 \uacc4\uc18d\ub418\uc5c8\ub2e4.", "paragraph_answer": "\uae40\uc740\uc131 \ucc28\uc7a5\uc740 \ub610 2000\ub144 4\uc6d4 \uad6d\uc815\uc6d0 2\ucc28\uc7a5\uc73c\ub85c \ubd80\uc784\ud560 \ubb34\ub835\ubd80\ud130 8\uad6d(\uacfc\ud559\ubcf4\uc548\uad6d)\uc774 \uc624\ub798\uc804\ubd80\ud130 \uad00\ud589\uc801\uc73c\ub85c \ubd88\ubc95\uac10\uccad\uc744 \ud55c \uc0ac\uc2e4\uc744 \uc54c\uace0 \uc788\uc5c8\ub2e4\uace0 \ud138\uc5b4\ub1a8\ub2e4. \uc774\uc5b4 \uc784\ub3d9\uc6d0 \uc804 \uc6d0\uc7a5\uc774 \ubd88\ubc95\uac10\uccad \uc0ac\uc2e4\uc744 \uc54c\uace0 \uc788\uc5c8\ub294\uc9c0\uc5d0 \ub300\ud574 \"\ub2e8\uc815\uc801\uc73c\ub85c \uc54c\uace0 \uc788\uc5c8\ub294\uc9c0\ub294 \ubaa8\ub974\uaca0\ub294\ub370 \uc81c \uc0dd\uac01\uc73c\ub85c \ubcfc \ub54c\ub294 \uc54c\uace0 \uc788\uc5c8\ub2e4\uace0 \uc0dd\uac01\ud55c\ub2e4\"\uba70 \ub9e4\uc77c \ubcf4\uace0\uc11c\uac00 \uc62c\ub77c\uac00\ub294 \ub370 \ud2b9\ubcc4\ud55c \uc870\uce58\uac00 \uc5c6\uc5c8\ub358 \uc810 \ub4f1 4\uac00\uc9c0 \ub97c \uadf8 \uadfc\uac70\ub85c \ub4e4\uc5c8\ub2e4. \uadf8\ub294 \"\uad6d\uc815\uc6d0\uc7a5\uc5d0\uac8c \ub9e4\uc77c \ubcf4\uace0\ub418\ub294 \ud1b5\uc2e0\ucca9\ubcf4 \ubcf4\uace0\uc11c\uc5d0 \ub300\ud574 \ub450 \uc6d0\uc7a5\uc740 `\uc774\ub7f0 \uac83\uc774 \ubc16\uc5d0 \ub098\uac00\uc9c0 \uc54a\uac8c \uc870\uc2ec\ud574\uc57c \ud55c\ub2e4'\uace0 \ub9d0\ud55c \uc801\uc774 \uc788\ub2e4\"\ub294 \ub9d0\ub3c4 \ub367\ubd99\uc600\ub2e4. \uae40\uc740\uc131\uc758 \uc9c4\uc220 \uc774\ud6c4 \uad6d\uac00\uc815\ubcf4\uc6d0\uacfc \uc804\ud604\uc9c1 \uc2e4\uad6d\uc7a5, \ub2e8\uc7a5\uae09 \uace0\uc704 \uac04\ubd80\ub4e4\uc5d0 \ub300\ud55c \uc11c\uc6b8\uc911\uc559\uc9c0\uac80\uacfc \ub300\uac80 \uc911\uc218\ubd80\uc758 \uc218\uc0ac\uc640 \uac00\ud0dd \uc870\uc0ac\ub294 \uacc4\uc18d\ub418\uc5c8\ub2e4.", "sentence_answer": "\"\uba70 \ub9e4\uc77c \ubcf4\uace0\uc11c\uac00 \uc62c\ub77c\uac00\ub294 \ub370 \ud2b9\ubcc4\ud55c \uc870\uce58\uac00 \uc5c6\uc5c8\ub358 \uc810 \ub4f1 4\uac00\uc9c0 \ub97c \uadf8 \uadfc\uac70\ub85c \ub4e4\uc5c8\ub2e4.", "paragraph_id": "6577692-6-1", "paragraph_question": "question: \uae40\uc740\uc131\uc740 \uad6d\uc815\uc6d0\uc7a5\uc774 \ubd88\ubc95\uac10\uccad\uc5d0 \ub300\ud574 \uc778\uc9c0\ud558\uace0 \uc788\uc5c8\ub2e4\uace0 \uc0dd\uac01\ub41c\ub2e4\ub294 \ub2f5\ubcc0\uc744 \ud588\uace0 \uadf8\uc5d0 \ub300\ud55c \uadfc\uac70\ub85c \uba87 \uac00\uc9c0\ub97c \uc5b8\uae09\ud558\uc600\ub098?, context: \uae40\uc740\uc131 \ucc28\uc7a5\uc740 \ub610 2000\ub144 4\uc6d4 \uad6d\uc815\uc6d0 2\ucc28\uc7a5\uc73c\ub85c \ubd80\uc784\ud560 \ubb34\ub835\ubd80\ud130 8\uad6d(\uacfc\ud559\ubcf4\uc548\uad6d)\uc774 \uc624\ub798\uc804\ubd80\ud130 \uad00\ud589\uc801\uc73c\ub85c \ubd88\ubc95\uac10\uccad\uc744 \ud55c \uc0ac\uc2e4\uc744 \uc54c\uace0 \uc788\uc5c8\ub2e4\uace0 \ud138\uc5b4\ub1a8\ub2e4. \uc774\uc5b4 \uc784\ub3d9\uc6d0 \uc804 \uc6d0\uc7a5\uc774 \ubd88\ubc95\uac10\uccad \uc0ac\uc2e4\uc744 \uc54c\uace0 \uc788\uc5c8\ub294\uc9c0\uc5d0 \ub300\ud574 \"\ub2e8\uc815\uc801\uc73c\ub85c \uc54c\uace0 \uc788\uc5c8\ub294\uc9c0\ub294 \ubaa8\ub974\uaca0\ub294\ub370 \uc81c \uc0dd\uac01\uc73c\ub85c \ubcfc \ub54c\ub294 \uc54c\uace0 \uc788\uc5c8\ub2e4\uace0 \uc0dd\uac01\ud55c\ub2e4\"\uba70 \ub9e4\uc77c \ubcf4\uace0\uc11c\uac00 \uc62c\ub77c\uac00\ub294 \ub370 \ud2b9\ubcc4\ud55c \uc870\uce58\uac00 \uc5c6\uc5c8\ub358 \uc810 \ub4f1 4\uac00\uc9c0\ub97c \uadf8 \uadfc\uac70\ub85c \ub4e4\uc5c8\ub2e4. \uadf8\ub294 \"\uad6d\uc815\uc6d0\uc7a5\uc5d0\uac8c \ub9e4\uc77c \ubcf4\uace0\ub418\ub294 \ud1b5\uc2e0\ucca9\ubcf4 \ubcf4\uace0\uc11c\uc5d0 \ub300\ud574 \ub450 \uc6d0\uc7a5\uc740 `\uc774\ub7f0 \uac83\uc774 \ubc16\uc5d0 \ub098\uac00\uc9c0 \uc54a\uac8c \uc870\uc2ec\ud574\uc57c \ud55c\ub2e4'\uace0 \ub9d0\ud55c \uc801\uc774 \uc788\ub2e4\"\ub294 \ub9d0\ub3c4 \ub367\ubd99\uc600\ub2e4. \uae40\uc740\uc131\uc758 \uc9c4\uc220 \uc774\ud6c4 \uad6d\uac00\uc815\ubcf4\uc6d0\uacfc \uc804\ud604\uc9c1 \uc2e4\uad6d\uc7a5, \ub2e8\uc7a5\uae09 \uace0\uc704 \uac04\ubd80\ub4e4\uc5d0 \ub300\ud55c \uc11c\uc6b8\uc911\uc559\uc9c0\uac80\uacfc \ub300\uac80 \uc911\uc218\ubd80\uc758 \uc218\uc0ac\uc640 \uac00\ud0dd \uc870\uc0ac\ub294 \uacc4\uc18d\ub418\uc5c8\ub2e4."} {"question": "\uc218\uc900\ubcc4 \uc218\ub2a5\uc774 \ud3d0\uc9c0\ub41c \ud559\ub144\ub3c4\ub294 \uba87\ud559\ub144\ub3c4 \uc778\uac00?", "paragraph": "2014\ud559\ub144\ub3c4 \uc218\ub2a5\ubd80\ud130 \uad6d\uc5b4, \uc218\ud559, \uc601\uc5b4 \uc601\uc5ed\uc5d0\uc11c\ub294 \uc218\uc900\ubcc4 \ubb38\uc81c\uc9c0\ub97c \uc81c\uacf5\ud558\ub294\ub370, \uc218\ud5d8\uc0dd\uc740 \uc26c\uc6b4 \ub09c\uc774\ub3c4\uc758 A\ud615\uacfc \uc5b4\ub824\uc6b4 \ub09c\uc774\ub3c4\uc758 B\ud615\uc744 \uc120\ud0dd\ud560 \uc218 \uc788\ub2e4. \uc774\ub54c, B\ud615\uc740 \ucd5c\ub300 2\uac1c \uc601\uc5ed\uae4c\uc9c0\ub9cc \uc120\ud0dd\ud560 \uc218 \uc788\uc73c\uba70, \uad6d\uc5b4 B\ud615\uacfc \uc218\ud559 B\ud615\uc740 \ub3d9\uc2dc\uc5d0 \uc120\ud0dd\ud560 \uc218 \uc5c6\ub2e4. \ub300\ubd80\ubd84 \ub300\ud559\uc5d0\uc11c\ub294 \uc778\ubb38\uacc4\uc5f4\uc740 \uad6d\uc5b4 B\ud615, \uc218\ud559 A\ud615, \uc601\uc5b4 B\ud615\uc744, \uc790\uc5f0\uacc4\uc5f4\uc740 \uad6d\uc5b4 A\ud615, \uc218\ud559 B\ud615, \uc601\uc5b4 B\ud615\uc744, \uadf8\ub9ac\uace0 \uc608\u00b7\uccb4\ub2a5\uacc4\uc5f4\uc740 \uad6d\uc5b4 A\ud615, \uc218\ud559 A\ud615, \uc601\uc5b4 A\ud615\uc744 \uae30\ubcf8 \ubc29\uce68\uc73c\ub85c \ucc44\ud0dd\ud558\uc600\ub2e4. 2015\ud559\ub144\ub3c4\ubd80\ud130 \uc601\uc5b4 \uc601\uc5ed\uc5d0\uc11c \uc218\uc900\ubcc4 \uc2dc\ud5d8\uc774 \ud3d0\uc9c0\ub418\uc5c8\uc73c\ub098, \uad6d\uc5b4 \uc601\uc5ed\uacfc \uc218\ud559 \uc601\uc5ed\uc740 \ub3d9\uc2dc\uc5d0 B\ud615\uc744 \uc120\ud0dd\ud560 \uc218 \uc5c6\ub2e4. \uc218\uc900\ubcc4 \uc218\ub2a5\uc740 2016\ud559\ub144\ub3c4\ub97c \ub05d\uc73c\ub85c \ud3d0\uc9c0\ub418\uc5c8\ub2e4.", "answer": "2016\ud559\ub144\ub3c4", "sentence": "\uc218\uc900\ubcc4 \uc218\ub2a5\uc740 2016\ud559\ub144\ub3c4 \ub97c \ub05d\uc73c\ub85c \ud3d0\uc9c0\ub418\uc5c8\ub2e4.", "paragraph_sentence": "2014\ud559\ub144\ub3c4 \uc218\ub2a5\ubd80\ud130 \uad6d\uc5b4, \uc218\ud559, \uc601\uc5b4 \uc601\uc5ed\uc5d0\uc11c\ub294 \uc218\uc900\ubcc4 \ubb38\uc81c\uc9c0\ub97c \uc81c\uacf5\ud558\ub294\ub370, \uc218\ud5d8\uc0dd\uc740 \uc26c\uc6b4 \ub09c\uc774\ub3c4\uc758 A\ud615\uacfc \uc5b4\ub824\uc6b4 \ub09c\uc774\ub3c4\uc758 B\ud615\uc744 \uc120\ud0dd\ud560 \uc218 \uc788\ub2e4. \uc774\ub54c, B\ud615\uc740 \ucd5c\ub300 2\uac1c \uc601\uc5ed\uae4c\uc9c0\ub9cc \uc120\ud0dd\ud560 \uc218 \uc788\uc73c\uba70, \uad6d\uc5b4 B\ud615\uacfc \uc218\ud559 B\ud615\uc740 \ub3d9\uc2dc\uc5d0 \uc120\ud0dd\ud560 \uc218 \uc5c6\ub2e4. \ub300\ubd80\ubd84 \ub300\ud559\uc5d0\uc11c\ub294 \uc778\ubb38\uacc4\uc5f4\uc740 \uad6d\uc5b4 B\ud615, \uc218\ud559 A\ud615, \uc601\uc5b4 B\ud615\uc744, \uc790\uc5f0\uacc4\uc5f4\uc740 \uad6d\uc5b4 A\ud615, \uc218\ud559 B\ud615, \uc601\uc5b4 B\ud615\uc744, \uadf8\ub9ac\uace0 \uc608\u00b7\uccb4\ub2a5\uacc4\uc5f4\uc740 \uad6d\uc5b4 A\ud615, \uc218\ud559 A\ud615, \uc601\uc5b4 A\ud615\uc744 \uae30\ubcf8 \ubc29\uce68\uc73c\ub85c \ucc44\ud0dd\ud558\uc600\ub2e4. 2015\ud559\ub144\ub3c4\ubd80\ud130 \uc601\uc5b4 \uc601\uc5ed\uc5d0\uc11c \uc218\uc900\ubcc4 \uc2dc\ud5d8\uc774 \ud3d0\uc9c0\ub418\uc5c8\uc73c\ub098, \uad6d\uc5b4 \uc601\uc5ed\uacfc \uc218\ud559 \uc601\uc5ed\uc740 \ub3d9\uc2dc\uc5d0 B\ud615\uc744 \uc120\ud0dd\ud560 \uc218 \uc5c6\ub2e4. \uc218\uc900\ubcc4 \uc218\ub2a5\uc740 2016\ud559\ub144\ub3c4 \ub97c \ub05d\uc73c\ub85c \ud3d0\uc9c0\ub418\uc5c8\ub2e4. ", "paragraph_answer": "2014\ud559\ub144\ub3c4 \uc218\ub2a5\ubd80\ud130 \uad6d\uc5b4, \uc218\ud559, \uc601\uc5b4 \uc601\uc5ed\uc5d0\uc11c\ub294 \uc218\uc900\ubcc4 \ubb38\uc81c\uc9c0\ub97c \uc81c\uacf5\ud558\ub294\ub370, \uc218\ud5d8\uc0dd\uc740 \uc26c\uc6b4 \ub09c\uc774\ub3c4\uc758 A\ud615\uacfc \uc5b4\ub824\uc6b4 \ub09c\uc774\ub3c4\uc758 B\ud615\uc744 \uc120\ud0dd\ud560 \uc218 \uc788\ub2e4. \uc774\ub54c, B\ud615\uc740 \ucd5c\ub300 2\uac1c \uc601\uc5ed\uae4c\uc9c0\ub9cc \uc120\ud0dd\ud560 \uc218 \uc788\uc73c\uba70, \uad6d\uc5b4 B\ud615\uacfc \uc218\ud559 B\ud615\uc740 \ub3d9\uc2dc\uc5d0 \uc120\ud0dd\ud560 \uc218 \uc5c6\ub2e4. \ub300\ubd80\ubd84 \ub300\ud559\uc5d0\uc11c\ub294 \uc778\ubb38\uacc4\uc5f4\uc740 \uad6d\uc5b4 B\ud615, \uc218\ud559 A\ud615, \uc601\uc5b4 B\ud615\uc744, \uc790\uc5f0\uacc4\uc5f4\uc740 \uad6d\uc5b4 A\ud615, \uc218\ud559 B\ud615, \uc601\uc5b4 B\ud615\uc744, \uadf8\ub9ac\uace0 \uc608\u00b7\uccb4\ub2a5\uacc4\uc5f4\uc740 \uad6d\uc5b4 A\ud615, \uc218\ud559 A\ud615, \uc601\uc5b4 A\ud615\uc744 \uae30\ubcf8 \ubc29\uce68\uc73c\ub85c \ucc44\ud0dd\ud558\uc600\ub2e4. 2015\ud559\ub144\ub3c4\ubd80\ud130 \uc601\uc5b4 \uc601\uc5ed\uc5d0\uc11c \uc218\uc900\ubcc4 \uc2dc\ud5d8\uc774 \ud3d0\uc9c0\ub418\uc5c8\uc73c\ub098, \uad6d\uc5b4 \uc601\uc5ed\uacfc \uc218\ud559 \uc601\uc5ed\uc740 \ub3d9\uc2dc\uc5d0 B\ud615\uc744 \uc120\ud0dd\ud560 \uc218 \uc5c6\ub2e4. \uc218\uc900\ubcc4 \uc218\ub2a5\uc740 2016\ud559\ub144\ub3c4 \ub97c \ub05d\uc73c\ub85c \ud3d0\uc9c0\ub418\uc5c8\ub2e4.", "sentence_answer": "\uc218\uc900\ubcc4 \uc218\ub2a5\uc740 2016\ud559\ub144\ub3c4 \ub97c \ub05d\uc73c\ub85c \ud3d0\uc9c0\ub418\uc5c8\ub2e4.", "paragraph_id": "6031731-4-0", "paragraph_question": "question: \uc218\uc900\ubcc4 \uc218\ub2a5\uc774 \ud3d0\uc9c0\ub41c \ud559\ub144\ub3c4\ub294 \uba87\ud559\ub144\ub3c4 \uc778\uac00?, context: 2014\ud559\ub144\ub3c4 \uc218\ub2a5\ubd80\ud130 \uad6d\uc5b4, \uc218\ud559, \uc601\uc5b4 \uc601\uc5ed\uc5d0\uc11c\ub294 \uc218\uc900\ubcc4 \ubb38\uc81c\uc9c0\ub97c \uc81c\uacf5\ud558\ub294\ub370, \uc218\ud5d8\uc0dd\uc740 \uc26c\uc6b4 \ub09c\uc774\ub3c4\uc758 A\ud615\uacfc \uc5b4\ub824\uc6b4 \ub09c\uc774\ub3c4\uc758 B\ud615\uc744 \uc120\ud0dd\ud560 \uc218 \uc788\ub2e4. \uc774\ub54c, B\ud615\uc740 \ucd5c\ub300 2\uac1c \uc601\uc5ed\uae4c\uc9c0\ub9cc \uc120\ud0dd\ud560 \uc218 \uc788\uc73c\uba70, \uad6d\uc5b4 B\ud615\uacfc \uc218\ud559 B\ud615\uc740 \ub3d9\uc2dc\uc5d0 \uc120\ud0dd\ud560 \uc218 \uc5c6\ub2e4. \ub300\ubd80\ubd84 \ub300\ud559\uc5d0\uc11c\ub294 \uc778\ubb38\uacc4\uc5f4\uc740 \uad6d\uc5b4 B\ud615, \uc218\ud559 A\ud615, \uc601\uc5b4 B\ud615\uc744, \uc790\uc5f0\uacc4\uc5f4\uc740 \uad6d\uc5b4 A\ud615, \uc218\ud559 B\ud615, \uc601\uc5b4 B\ud615\uc744, \uadf8\ub9ac\uace0 \uc608\u00b7\uccb4\ub2a5\uacc4\uc5f4\uc740 \uad6d\uc5b4 A\ud615, \uc218\ud559 A\ud615, \uc601\uc5b4 A\ud615\uc744 \uae30\ubcf8 \ubc29\uce68\uc73c\ub85c \ucc44\ud0dd\ud558\uc600\ub2e4. 2015\ud559\ub144\ub3c4\ubd80\ud130 \uc601\uc5b4 \uc601\uc5ed\uc5d0\uc11c \uc218\uc900\ubcc4 \uc2dc\ud5d8\uc774 \ud3d0\uc9c0\ub418\uc5c8\uc73c\ub098, \uad6d\uc5b4 \uc601\uc5ed\uacfc \uc218\ud559 \uc601\uc5ed\uc740 \ub3d9\uc2dc\uc5d0 B\ud615\uc744 \uc120\ud0dd\ud560 \uc218 \uc5c6\ub2e4. \uc218\uc900\ubcc4 \uc218\ub2a5\uc740 2016\ud559\ub144\ub3c4\ub97c \ub05d\uc73c\ub85c \ud3d0\uc9c0\ub418\uc5c8\ub2e4."} {"question": "\ub3d9\uc77c\ubcf8 \ub300\uc9c0\uc9c4\uc774 \uc77c\uc5b4\ub098\uae30 \uc804\uae4c\uc9c0 \uc77c\ubcf8\uc5d0\uc11c \ubc1c\uc0dd\ud55c \uac00\uc7a5 \ud070 \uaddc\ubaa8\uc758 \uc9c0\uc9c4\uc740?", "paragraph": "\uc77c\ubcf8 \uae30\uc0c1\uccad\uc740 \uc9c0\uc9c4 \ubcf4\ub3c4 \ucd08\uc5d0 \uaddc\ubaa8 7.9\uc758 \uc9c0\uc9c4\uc73c\ub85c \ucd94\uc815\ud558\uace0 \uc18d\ubcf4\ub97c \ub0b4\ubcf4\ub0c8\uc9c0\ub9cc, 16\uc2dc \uc815\uac01\uc5d0\ub294 \uaddc\ubaa8 8.4\ub85c \uc0c1\ud5a5 \ubc1c\ud45c\ud588\ub2e4. \uc774\ud6c4 \ucd94\uc815\uce58\ub97c \uacc4\uc18d \uac31\uc2e0\ud558\uba74\uc11c 17\uc2dc 30\ubd84\uc5d0\ub294 \uaddc\ubaa8 8.8\ub85c \uc0c1\ud5a5 \ubc1c\ud45c\ud558\uba74\uc11c 1900\ub144 \uc774\ud6c4 \uc77c\ubcf8\uc5d0\uc11c \uc788\uc5c8\ub358 \uc9c0\uc9c4 \uc911 \uac00\uc7a5 \ucef8\ub358 \uc9c0\uc9c4\uc778 1933\ub144 \uc1fc\uc640 \uc0b0\ub9ac\ucfe0 \ud574\uc5ed \uc9c0\uc9c4(Mw8.4) \uc758 \uaddc\ubaa8\ub97c \ub118\uc73c\uba74\uc11c \uc77c\ubcf8 \uadfc\ub300 \uc9c0\uc9c4 \uad00\uce21 \uc0ac\uc0c1 \ucd5c\ub300\uc758 \uc9c0\uc9c4\uc744 \uae30\ub85d\ud588\ub2e4. \uacb0\uad6d 3\uc6d4 13\uc77c\uc5d0\ub294 \uc77c\ubcf8 \ubc16\uc5d0\uc11c \uad00\uce21\ud55c \uc548\uc815\ub41c \uc9c0\uc9c4 \ud30c\ud615 \ub370\uc774\ud130\uc640 50\ubd84\uac04\uc758 \uc7a5\uae30\uac04 \uad00\uce21 \ub370\uc774\ud130\ub97c \uac00\uc9c0\uace0 \uaddc\ubaa8\ub97c 9.0\uc73c\ub85c \ucd5c\uc885 \uc0c1\ud5a5 \ubc1c\ud45c\ud558\uc600\ub2e4. \ubcf4\ud1b5 \uc77c\ubcf8 \ub0b4\uc5d0\uc11c \uc0ac\uc6a9\ud558\ub294 \uc77c\ubcf8 \uae30\uc0c1\uccad \uaddc\ubaa8(Mj)\ub294 M7.9, M8.4\uc600\uc73c\ub098 \ubaa8\uba58\ud2b8 \uaddc\ubaa8(Mw)\ub85c\ub294 M8.9, M9.0\uc73c\ub85c \ub098\uc654\ub2e4. M9.0\uc774\ub77c\ub294 \uaddc\ubaa8\ub294 1923\ub144\uc758 \uad00\ub3d9\ub300\uc9c0\uc9c4(Mw 8.2)\uc758 \uc57d 16\ubc30\uc774\uba70, 1995\ub144 \ud6a8\uace0 \ud604 \ub0a8\ubd80 \uc9c0\uc9c4(Mw6.9)\uc758 1450\ubc30\uc758 \uc5d0\ub108\uc9c0\uc5d0 \ud574\ub2f9\ud55c\ub2e4.", "answer": "\uc1fc\uc640 \uc0b0\ub9ac\ucfe0 \ud574\uc5ed \uc9c0\uc9c4", "sentence": "1933\ub144 \uc1fc\uc640 \uc0b0\ub9ac\ucfe0 \ud574\uc5ed \uc9c0\uc9c4 (Mw8.4)", "paragraph_sentence": "\uc77c\ubcf8 \uae30\uc0c1\uccad\uc740 \uc9c0\uc9c4 \ubcf4\ub3c4 \ucd08\uc5d0 \uaddc\ubaa8 7.9\uc758 \uc9c0\uc9c4\uc73c\ub85c \ucd94\uc815\ud558\uace0 \uc18d\ubcf4\ub97c \ub0b4\ubcf4\ub0c8\uc9c0\ub9cc, 16\uc2dc \uc815\uac01\uc5d0\ub294 \uaddc\ubaa8 8.4\ub85c \uc0c1\ud5a5 \ubc1c\ud45c\ud588\ub2e4. \uc774\ud6c4 \ucd94\uc815\uce58\ub97c \uacc4\uc18d \uac31\uc2e0\ud558\uba74\uc11c 17\uc2dc 30\ubd84\uc5d0\ub294 \uaddc\ubaa8 8.8\ub85c \uc0c1\ud5a5 \ubc1c\ud45c\ud558\uba74\uc11c 1900\ub144 \uc774\ud6c4 \uc77c\ubcf8\uc5d0\uc11c \uc788\uc5c8\ub358 \uc9c0\uc9c4 \uc911 \uac00\uc7a5 \ucef8\ub358 \uc9c0\uc9c4\uc778 1933\ub144 \uc1fc\uc640 \uc0b0\ub9ac\ucfe0 \ud574\uc5ed \uc9c0\uc9c4 (Mw8.4) \uc758 \uaddc\ubaa8\ub97c \ub118\uc73c\uba74\uc11c \uc77c\ubcf8 \uadfc\ub300 \uc9c0\uc9c4 \uad00\uce21 \uc0ac\uc0c1 \ucd5c\ub300\uc758 \uc9c0\uc9c4\uc744 \uae30\ub85d\ud588\ub2e4. \uacb0\uad6d 3\uc6d4 13\uc77c\uc5d0\ub294 \uc77c\ubcf8 \ubc16\uc5d0\uc11c \uad00\uce21\ud55c \uc548\uc815\ub41c \uc9c0\uc9c4 \ud30c\ud615 \ub370\uc774\ud130\uc640 50\ubd84\uac04\uc758 \uc7a5\uae30\uac04 \uad00\uce21 \ub370\uc774\ud130\ub97c \uac00\uc9c0\uace0 \uaddc\ubaa8\ub97c 9.0\uc73c\ub85c \ucd5c\uc885 \uc0c1\ud5a5 \ubc1c\ud45c\ud558\uc600\ub2e4. \ubcf4\ud1b5 \uc77c\ubcf8 \ub0b4\uc5d0\uc11c \uc0ac\uc6a9\ud558\ub294 \uc77c\ubcf8 \uae30\uc0c1\uccad \uaddc\ubaa8(Mj)\ub294 M7.9, M8.4\uc600\uc73c\ub098 \ubaa8\uba58\ud2b8 \uaddc\ubaa8(Mw)\ub85c\ub294 M8.9, M9.0\uc73c\ub85c \ub098\uc654\ub2e4. M9.0\uc774\ub77c\ub294 \uaddc\ubaa8\ub294 1923\ub144\uc758 \uad00\ub3d9\ub300\uc9c0\uc9c4(Mw 8.2)\uc758 \uc57d 16\ubc30\uc774\uba70, 1995\ub144 \ud6a8\uace0 \ud604 \ub0a8\ubd80 \uc9c0\uc9c4(Mw6.9)\uc758 1450\ubc30\uc758 \uc5d0\ub108\uc9c0\uc5d0 \ud574\ub2f9\ud55c\ub2e4.", "paragraph_answer": "\uc77c\ubcf8 \uae30\uc0c1\uccad\uc740 \uc9c0\uc9c4 \ubcf4\ub3c4 \ucd08\uc5d0 \uaddc\ubaa8 7.9\uc758 \uc9c0\uc9c4\uc73c\ub85c \ucd94\uc815\ud558\uace0 \uc18d\ubcf4\ub97c \ub0b4\ubcf4\ub0c8\uc9c0\ub9cc, 16\uc2dc \uc815\uac01\uc5d0\ub294 \uaddc\ubaa8 8.4\ub85c \uc0c1\ud5a5 \ubc1c\ud45c\ud588\ub2e4. \uc774\ud6c4 \ucd94\uc815\uce58\ub97c \uacc4\uc18d \uac31\uc2e0\ud558\uba74\uc11c 17\uc2dc 30\ubd84\uc5d0\ub294 \uaddc\ubaa8 8.8\ub85c \uc0c1\ud5a5 \ubc1c\ud45c\ud558\uba74\uc11c 1900\ub144 \uc774\ud6c4 \uc77c\ubcf8\uc5d0\uc11c \uc788\uc5c8\ub358 \uc9c0\uc9c4 \uc911 \uac00\uc7a5 \ucef8\ub358 \uc9c0\uc9c4\uc778 1933\ub144 \uc1fc\uc640 \uc0b0\ub9ac\ucfe0 \ud574\uc5ed \uc9c0\uc9c4 (Mw8.4) \uc758 \uaddc\ubaa8\ub97c \ub118\uc73c\uba74\uc11c \uc77c\ubcf8 \uadfc\ub300 \uc9c0\uc9c4 \uad00\uce21 \uc0ac\uc0c1 \ucd5c\ub300\uc758 \uc9c0\uc9c4\uc744 \uae30\ub85d\ud588\ub2e4. \uacb0\uad6d 3\uc6d4 13\uc77c\uc5d0\ub294 \uc77c\ubcf8 \ubc16\uc5d0\uc11c \uad00\uce21\ud55c \uc548\uc815\ub41c \uc9c0\uc9c4 \ud30c\ud615 \ub370\uc774\ud130\uc640 50\ubd84\uac04\uc758 \uc7a5\uae30\uac04 \uad00\uce21 \ub370\uc774\ud130\ub97c \uac00\uc9c0\uace0 \uaddc\ubaa8\ub97c 9.0\uc73c\ub85c \ucd5c\uc885 \uc0c1\ud5a5 \ubc1c\ud45c\ud558\uc600\ub2e4. \ubcf4\ud1b5 \uc77c\ubcf8 \ub0b4\uc5d0\uc11c \uc0ac\uc6a9\ud558\ub294 \uc77c\ubcf8 \uae30\uc0c1\uccad \uaddc\ubaa8(Mj)\ub294 M7.9, M8.4\uc600\uc73c\ub098 \ubaa8\uba58\ud2b8 \uaddc\ubaa8(Mw)\ub85c\ub294 M8.9, M9.0\uc73c\ub85c \ub098\uc654\ub2e4. M9.0\uc774\ub77c\ub294 \uaddc\ubaa8\ub294 1923\ub144\uc758 \uad00\ub3d9\ub300\uc9c0\uc9c4(Mw 8.2)\uc758 \uc57d 16\ubc30\uc774\uba70, 1995\ub144 \ud6a8\uace0 \ud604 \ub0a8\ubd80 \uc9c0\uc9c4(Mw6.9)\uc758 1450\ubc30\uc758 \uc5d0\ub108\uc9c0\uc5d0 \ud574\ub2f9\ud55c\ub2e4.", "sentence_answer": "1933\ub144 \uc1fc\uc640 \uc0b0\ub9ac\ucfe0 \ud574\uc5ed \uc9c0\uc9c4 (Mw8.4)", "paragraph_id": "6528699-4-0", "paragraph_question": "question: \ub3d9\uc77c\ubcf8 \ub300\uc9c0\uc9c4\uc774 \uc77c\uc5b4\ub098\uae30 \uc804\uae4c\uc9c0 \uc77c\ubcf8\uc5d0\uc11c \ubc1c\uc0dd\ud55c \uac00\uc7a5 \ud070 \uaddc\ubaa8\uc758 \uc9c0\uc9c4\uc740?, context: \uc77c\ubcf8 \uae30\uc0c1\uccad\uc740 \uc9c0\uc9c4 \ubcf4\ub3c4 \ucd08\uc5d0 \uaddc\ubaa8 7.9\uc758 \uc9c0\uc9c4\uc73c\ub85c \ucd94\uc815\ud558\uace0 \uc18d\ubcf4\ub97c \ub0b4\ubcf4\ub0c8\uc9c0\ub9cc, 16\uc2dc \uc815\uac01\uc5d0\ub294 \uaddc\ubaa8 8.4\ub85c \uc0c1\ud5a5 \ubc1c\ud45c\ud588\ub2e4. \uc774\ud6c4 \ucd94\uc815\uce58\ub97c \uacc4\uc18d \uac31\uc2e0\ud558\uba74\uc11c 17\uc2dc 30\ubd84\uc5d0\ub294 \uaddc\ubaa8 8.8\ub85c \uc0c1\ud5a5 \ubc1c\ud45c\ud558\uba74\uc11c 1900\ub144 \uc774\ud6c4 \uc77c\ubcf8\uc5d0\uc11c \uc788\uc5c8\ub358 \uc9c0\uc9c4 \uc911 \uac00\uc7a5 \ucef8\ub358 \uc9c0\uc9c4\uc778 1933\ub144 \uc1fc\uc640 \uc0b0\ub9ac\ucfe0 \ud574\uc5ed \uc9c0\uc9c4(Mw8.4) \uc758 \uaddc\ubaa8\ub97c \ub118\uc73c\uba74\uc11c \uc77c\ubcf8 \uadfc\ub300 \uc9c0\uc9c4 \uad00\uce21 \uc0ac\uc0c1 \ucd5c\ub300\uc758 \uc9c0\uc9c4\uc744 \uae30\ub85d\ud588\ub2e4. \uacb0\uad6d 3\uc6d4 13\uc77c\uc5d0\ub294 \uc77c\ubcf8 \ubc16\uc5d0\uc11c \uad00\uce21\ud55c \uc548\uc815\ub41c \uc9c0\uc9c4 \ud30c\ud615 \ub370\uc774\ud130\uc640 50\ubd84\uac04\uc758 \uc7a5\uae30\uac04 \uad00\uce21 \ub370\uc774\ud130\ub97c \uac00\uc9c0\uace0 \uaddc\ubaa8\ub97c 9.0\uc73c\ub85c \ucd5c\uc885 \uc0c1\ud5a5 \ubc1c\ud45c\ud558\uc600\ub2e4. \ubcf4\ud1b5 \uc77c\ubcf8 \ub0b4\uc5d0\uc11c \uc0ac\uc6a9\ud558\ub294 \uc77c\ubcf8 \uae30\uc0c1\uccad \uaddc\ubaa8(Mj)\ub294 M7.9, M8.4\uc600\uc73c\ub098 \ubaa8\uba58\ud2b8 \uaddc\ubaa8(Mw)\ub85c\ub294 M8.9, M9.0\uc73c\ub85c \ub098\uc654\ub2e4. M9.0\uc774\ub77c\ub294 \uaddc\ubaa8\ub294 1923\ub144\uc758 \uad00\ub3d9\ub300\uc9c0\uc9c4(Mw 8.2)\uc758 \uc57d 16\ubc30\uc774\uba70, 1995\ub144 \ud6a8\uace0 \ud604 \ub0a8\ubd80 \uc9c0\uc9c4(Mw6.9)\uc758 1450\ubc30\uc758 \uc5d0\ub108\uc9c0\uc5d0 \ud574\ub2f9\ud55c\ub2e4."} {"question": "\uc6d0\uc220\uc740 \ubb34\uc5c7\uc744 \ud551\uacc4\ub85c \uc720\uc6b0\uac00 \ub4f1\uadf9\ud558\ub294 \uac83\uc744 \ubc18\ub300\ud588\ub294\uac00?", "paragraph": "191\ub144\uc5d0 \ubc1c\ud574\ud0dc\uc218 \uc6d0\uc18c\uc640 \uae30\uc8fc\ubaa9 \ud55c\ubcf5\uc744 \ube44\ub86f\ud55c \uc0b0\ub3d9\uc758 \uc5ec\ub7ec \uc81c\uc7a5\ub4e4\uc740 \ud5cc\uc81c\uac00 \ub3d9\ud0c1\uc774 \uc804 \ud669\uc81c\ub97c \uc2dc\ud574\ud558\uace0 \uc639\ub9bd\ud55c \uad34\ub8b0\uad70\uc8fc\uc774\uba70 \uba40\ub9ac \ubcc0\ubc29\uc5d0 \uc720\ud3d0\ub418\uc5b4 \uc874\uc7ac\ud558\ub294 \uc758\uc758\uc870\ucc28 \uc0c1\uc2e4\ub418\uc5c8\ub2e4\uace0 \ubcf4\uc558\ub2e4. \uc774\uc5d0 \ud669\uc2e4\uc758 \uc6c3\uc5b4\ub978\uc774\uba70 \ud604\uba85\ud558\uace0 \uc778\ub9dd \ub192\uae30\ub85c \uc774\ub984\ub09c \uc720\uc6b0\ub97c \ud669\uc81c\ub85c \ucd94\ub300\ud588\uc73c\ub098, \uc720\uc6b0\ub294 \uc774\ub97c \ucc38\ub78c\ud55c \uc9d3\uc73c\ub85c \uc5ec\uaca8 \uc989\uc704\ud558\uae30\ub97c \uac70\ubd80\ud558\uc600\ub2e4. \uc6d0\uc18c\ub294 \uac70\ub4ed \uc720\uc6b0\uc5d0\uac8c \uc874\ud638\ub97c \ubc14\ucce4\uc73c\ub098 \uc720\uc6b0\ub294 \uc774\ub97c \ubc1b\uc544\ub4e4\uc774\ub290\ub2c8 \ud749\ub178\uc871\ub4e4\uc758 \ub545\uc73c\ub85c \ub2ec\uc544\ub098 \ubaa8\ub4e0 \uad00\uacc4\ub97c \ub04a\uace0\uc790 \ud588\uc73c\ubbc0\ub85c \uacb0\uad6d \uc6d0\uc18c\ub294 \uc774\ub97c \uadf8\ub9cc\ub450\uc5c8\ub2e4. \ub610\ud55c \ud55c\ubcf5 \ub4f1\uc740 \uc720\uc6b0\uac00 \uc601\uc0c1\uc11c\uc0ac(\u9818\u5c19\u66f8\u4e8b)\uac00 \ub418\uc5b4 \ucc9c\uc790 \ub300\uc2e0 \ubcbc\uc2ac\uc744 \ub0b4\ub9ac\ub3c4\ub85d \uad8c\uc720\ud588\uc73c\ub098, \uc720\uc6b0\ub294 \uc774 \uc5ed\uc2dc \ubc1b\uc544\ub4e4\uc774\uc9c0 \uc54a\uc558\ub2e4. \ub610\ud55c \uc5f0\ud569\uad70 \ub0b4\uc5d0\uc11c\ub3c4 \uc6d0\uc220\uc740 \uc7a5\ucc28 \uc790\uc2e0\uc774 \ud669\uc81c\uac00 \ub418\uc5b4 \uc655\uc870\ub97c \uac74\uad6d\ud560 \uc57c\uc2ec\uc744 \ud488\uace0 \uc788\uc5c8\ub294\ub370, \ud604\uba85\ud55c \uc720\uc6b0\uac00 \ub4f1\uadf9\ud558\uba74 \uc790\uc2e0\uc758 \uc57c\ub9dd\uc5d0 \uc7a5\uc560\uac00 \ub420 \uac83\uc73c\ub85c \uc0dd\uac01\ud558\uc600\uc73c\ubbc0\ub85c \uac89\uc73c\ub860 \uacf5\uc758\ub97c \ud551\uacc4\ub300\uba70 \uc774\ub97c \ubc18\ub300\ud588\ub2e4.", "answer": "\uacf5\uc758", "sentence": "\uac89\uc73c\ub860 \uacf5\uc758 \ub97c \ud551\uacc4\ub300\uba70 \uc774\ub97c \ubc18\ub300\ud588\ub2e4.", "paragraph_sentence": "191\ub144\uc5d0 \ubc1c\ud574\ud0dc\uc218 \uc6d0\uc18c\uc640 \uae30\uc8fc\ubaa9 \ud55c\ubcf5\uc744 \ube44\ub86f\ud55c \uc0b0\ub3d9\uc758 \uc5ec\ub7ec \uc81c\uc7a5\ub4e4\uc740 \ud5cc\uc81c\uac00 \ub3d9\ud0c1\uc774 \uc804 \ud669\uc81c\ub97c \uc2dc\ud574\ud558\uace0 \uc639\ub9bd\ud55c \uad34\ub8b0\uad70\uc8fc\uc774\uba70 \uba40\ub9ac \ubcc0\ubc29\uc5d0 \uc720\ud3d0\ub418\uc5b4 \uc874\uc7ac\ud558\ub294 \uc758\uc758\uc870\ucc28 \uc0c1\uc2e4\ub418\uc5c8\ub2e4\uace0 \ubcf4\uc558\ub2e4. \uc774\uc5d0 \ud669\uc2e4\uc758 \uc6c3\uc5b4\ub978\uc774\uba70 \ud604\uba85\ud558\uace0 \uc778\ub9dd \ub192\uae30\ub85c \uc774\ub984\ub09c \uc720\uc6b0\ub97c \ud669\uc81c\ub85c \ucd94\ub300\ud588\uc73c\ub098, \uc720\uc6b0\ub294 \uc774\ub97c \ucc38\ub78c\ud55c \uc9d3\uc73c\ub85c \uc5ec\uaca8 \uc989\uc704\ud558\uae30\ub97c \uac70\ubd80\ud558\uc600\ub2e4. \uc6d0\uc18c\ub294 \uac70\ub4ed \uc720\uc6b0\uc5d0\uac8c \uc874\ud638\ub97c \ubc14\ucce4\uc73c\ub098 \uc720\uc6b0\ub294 \uc774\ub97c \ubc1b\uc544\ub4e4\uc774\ub290\ub2c8 \ud749\ub178\uc871\ub4e4\uc758 \ub545\uc73c\ub85c \ub2ec\uc544\ub098 \ubaa8\ub4e0 \uad00\uacc4\ub97c \ub04a\uace0\uc790 \ud588\uc73c\ubbc0\ub85c \uacb0\uad6d \uc6d0\uc18c\ub294 \uc774\ub97c \uadf8\ub9cc\ub450\uc5c8\ub2e4. \ub610\ud55c \ud55c\ubcf5 \ub4f1\uc740 \uc720\uc6b0\uac00 \uc601\uc0c1\uc11c\uc0ac(\u9818\u5c19\u66f8\u4e8b)\uac00 \ub418\uc5b4 \ucc9c\uc790 \ub300\uc2e0 \ubcbc\uc2ac\uc744 \ub0b4\ub9ac\ub3c4\ub85d \uad8c\uc720\ud588\uc73c\ub098, \uc720\uc6b0\ub294 \uc774 \uc5ed\uc2dc \ubc1b\uc544\ub4e4\uc774\uc9c0 \uc54a\uc558\ub2e4. \ub610\ud55c \uc5f0\ud569\uad70 \ub0b4\uc5d0\uc11c\ub3c4 \uc6d0\uc220\uc740 \uc7a5\ucc28 \uc790\uc2e0\uc774 \ud669\uc81c\uac00 \ub418\uc5b4 \uc655\uc870\ub97c \uac74\uad6d\ud560 \uc57c\uc2ec\uc744 \ud488\uace0 \uc788\uc5c8\ub294\ub370, \ud604\uba85\ud55c \uc720\uc6b0\uac00 \ub4f1\uadf9\ud558\uba74 \uc790\uc2e0\uc758 \uc57c\ub9dd\uc5d0 \uc7a5\uc560\uac00 \ub420 \uac83\uc73c\ub85c \uc0dd\uac01\ud558\uc600\uc73c\ubbc0\ub85c \uac89\uc73c\ub860 \uacf5\uc758 \ub97c \ud551\uacc4\ub300\uba70 \uc774\ub97c \ubc18\ub300\ud588\ub2e4. ", "paragraph_answer": "191\ub144\uc5d0 \ubc1c\ud574\ud0dc\uc218 \uc6d0\uc18c\uc640 \uae30\uc8fc\ubaa9 \ud55c\ubcf5\uc744 \ube44\ub86f\ud55c \uc0b0\ub3d9\uc758 \uc5ec\ub7ec \uc81c\uc7a5\ub4e4\uc740 \ud5cc\uc81c\uac00 \ub3d9\ud0c1\uc774 \uc804 \ud669\uc81c\ub97c \uc2dc\ud574\ud558\uace0 \uc639\ub9bd\ud55c \uad34\ub8b0\uad70\uc8fc\uc774\uba70 \uba40\ub9ac \ubcc0\ubc29\uc5d0 \uc720\ud3d0\ub418\uc5b4 \uc874\uc7ac\ud558\ub294 \uc758\uc758\uc870\ucc28 \uc0c1\uc2e4\ub418\uc5c8\ub2e4\uace0 \ubcf4\uc558\ub2e4. \uc774\uc5d0 \ud669\uc2e4\uc758 \uc6c3\uc5b4\ub978\uc774\uba70 \ud604\uba85\ud558\uace0 \uc778\ub9dd \ub192\uae30\ub85c \uc774\ub984\ub09c \uc720\uc6b0\ub97c \ud669\uc81c\ub85c \ucd94\ub300\ud588\uc73c\ub098, \uc720\uc6b0\ub294 \uc774\ub97c \ucc38\ub78c\ud55c \uc9d3\uc73c\ub85c \uc5ec\uaca8 \uc989\uc704\ud558\uae30\ub97c \uac70\ubd80\ud558\uc600\ub2e4. \uc6d0\uc18c\ub294 \uac70\ub4ed \uc720\uc6b0\uc5d0\uac8c \uc874\ud638\ub97c \ubc14\ucce4\uc73c\ub098 \uc720\uc6b0\ub294 \uc774\ub97c \ubc1b\uc544\ub4e4\uc774\ub290\ub2c8 \ud749\ub178\uc871\ub4e4\uc758 \ub545\uc73c\ub85c \ub2ec\uc544\ub098 \ubaa8\ub4e0 \uad00\uacc4\ub97c \ub04a\uace0\uc790 \ud588\uc73c\ubbc0\ub85c \uacb0\uad6d \uc6d0\uc18c\ub294 \uc774\ub97c \uadf8\ub9cc\ub450\uc5c8\ub2e4. \ub610\ud55c \ud55c\ubcf5 \ub4f1\uc740 \uc720\uc6b0\uac00 \uc601\uc0c1\uc11c\uc0ac(\u9818\u5c19\u66f8\u4e8b)\uac00 \ub418\uc5b4 \ucc9c\uc790 \ub300\uc2e0 \ubcbc\uc2ac\uc744 \ub0b4\ub9ac\ub3c4\ub85d \uad8c\uc720\ud588\uc73c\ub098, \uc720\uc6b0\ub294 \uc774 \uc5ed\uc2dc \ubc1b\uc544\ub4e4\uc774\uc9c0 \uc54a\uc558\ub2e4. \ub610\ud55c \uc5f0\ud569\uad70 \ub0b4\uc5d0\uc11c\ub3c4 \uc6d0\uc220\uc740 \uc7a5\ucc28 \uc790\uc2e0\uc774 \ud669\uc81c\uac00 \ub418\uc5b4 \uc655\uc870\ub97c \uac74\uad6d\ud560 \uc57c\uc2ec\uc744 \ud488\uace0 \uc788\uc5c8\ub294\ub370, \ud604\uba85\ud55c \uc720\uc6b0\uac00 \ub4f1\uadf9\ud558\uba74 \uc790\uc2e0\uc758 \uc57c\ub9dd\uc5d0 \uc7a5\uc560\uac00 \ub420 \uac83\uc73c\ub85c \uc0dd\uac01\ud558\uc600\uc73c\ubbc0\ub85c \uac89\uc73c\ub860 \uacf5\uc758 \ub97c \ud551\uacc4\ub300\uba70 \uc774\ub97c \ubc18\ub300\ud588\ub2e4.", "sentence_answer": "\uac89\uc73c\ub860 \uacf5\uc758 \ub97c \ud551\uacc4\ub300\uba70 \uc774\ub97c \ubc18\ub300\ud588\ub2e4.", "paragraph_id": "6543340-1-1", "paragraph_question": "question: \uc6d0\uc220\uc740 \ubb34\uc5c7\uc744 \ud551\uacc4\ub85c \uc720\uc6b0\uac00 \ub4f1\uadf9\ud558\ub294 \uac83\uc744 \ubc18\ub300\ud588\ub294\uac00?, context: 191\ub144\uc5d0 \ubc1c\ud574\ud0dc\uc218 \uc6d0\uc18c\uc640 \uae30\uc8fc\ubaa9 \ud55c\ubcf5\uc744 \ube44\ub86f\ud55c \uc0b0\ub3d9\uc758 \uc5ec\ub7ec \uc81c\uc7a5\ub4e4\uc740 \ud5cc\uc81c\uac00 \ub3d9\ud0c1\uc774 \uc804 \ud669\uc81c\ub97c \uc2dc\ud574\ud558\uace0 \uc639\ub9bd\ud55c \uad34\ub8b0\uad70\uc8fc\uc774\uba70 \uba40\ub9ac \ubcc0\ubc29\uc5d0 \uc720\ud3d0\ub418\uc5b4 \uc874\uc7ac\ud558\ub294 \uc758\uc758\uc870\ucc28 \uc0c1\uc2e4\ub418\uc5c8\ub2e4\uace0 \ubcf4\uc558\ub2e4. \uc774\uc5d0 \ud669\uc2e4\uc758 \uc6c3\uc5b4\ub978\uc774\uba70 \ud604\uba85\ud558\uace0 \uc778\ub9dd \ub192\uae30\ub85c \uc774\ub984\ub09c \uc720\uc6b0\ub97c \ud669\uc81c\ub85c \ucd94\ub300\ud588\uc73c\ub098, \uc720\uc6b0\ub294 \uc774\ub97c \ucc38\ub78c\ud55c \uc9d3\uc73c\ub85c \uc5ec\uaca8 \uc989\uc704\ud558\uae30\ub97c \uac70\ubd80\ud558\uc600\ub2e4. \uc6d0\uc18c\ub294 \uac70\ub4ed \uc720\uc6b0\uc5d0\uac8c \uc874\ud638\ub97c \ubc14\ucce4\uc73c\ub098 \uc720\uc6b0\ub294 \uc774\ub97c \ubc1b\uc544\ub4e4\uc774\ub290\ub2c8 \ud749\ub178\uc871\ub4e4\uc758 \ub545\uc73c\ub85c \ub2ec\uc544\ub098 \ubaa8\ub4e0 \uad00\uacc4\ub97c \ub04a\uace0\uc790 \ud588\uc73c\ubbc0\ub85c \uacb0\uad6d \uc6d0\uc18c\ub294 \uc774\ub97c \uadf8\ub9cc\ub450\uc5c8\ub2e4. \ub610\ud55c \ud55c\ubcf5 \ub4f1\uc740 \uc720\uc6b0\uac00 \uc601\uc0c1\uc11c\uc0ac(\u9818\u5c19\u66f8\u4e8b)\uac00 \ub418\uc5b4 \ucc9c\uc790 \ub300\uc2e0 \ubcbc\uc2ac\uc744 \ub0b4\ub9ac\ub3c4\ub85d \uad8c\uc720\ud588\uc73c\ub098, \uc720\uc6b0\ub294 \uc774 \uc5ed\uc2dc \ubc1b\uc544\ub4e4\uc774\uc9c0 \uc54a\uc558\ub2e4. \ub610\ud55c \uc5f0\ud569\uad70 \ub0b4\uc5d0\uc11c\ub3c4 \uc6d0\uc220\uc740 \uc7a5\ucc28 \uc790\uc2e0\uc774 \ud669\uc81c\uac00 \ub418\uc5b4 \uc655\uc870\ub97c \uac74\uad6d\ud560 \uc57c\uc2ec\uc744 \ud488\uace0 \uc788\uc5c8\ub294\ub370, \ud604\uba85\ud55c \uc720\uc6b0\uac00 \ub4f1\uadf9\ud558\uba74 \uc790\uc2e0\uc758 \uc57c\ub9dd\uc5d0 \uc7a5\uc560\uac00 \ub420 \uac83\uc73c\ub85c \uc0dd\uac01\ud558\uc600\uc73c\ubbc0\ub85c \uac89\uc73c\ub860 \uacf5\uc758\ub97c \ud551\uacc4\ub300\uba70 \uc774\ub97c \ubc18\ub300\ud588\ub2e4."} {"question": "\uc7a0\uc218\ubd80\ub4e4\uc774 \ubc30 \uc548\uc73c\ub85c \ub4e4\uc5b4\uac00\uba74 \ucd5c\ub300 \uc5bc\ub9c8\ub098 \uc218\uc0c9\uc791\uc5c5\uc744 \ud560 \uc218 \uc788\ub294\uac00?", "paragraph": "\uc7a0\uc218\uc694\uc6d0\ub4e4\uc740 \uc6b0\uc120 \uc218\uba74\uc5d0\uc11c \uc120\uccb4\uae4c\uc9c0 \uac00\uc774\ub4dc\ub77c\uc778(\uc720\ub3c4\uc120)\uc744 \uc5f0\uacb0\ud55c\ub2e4. \uc774 \uc120\uc740 \ube60\ub978 \uc720\uc18d\uacfc \uac00\uc2dc\uac70\ub9ac\uac00 10~20cm\uc5d0 \ubd88\uacfc\ud55c \uc0ac\uace0 \ud604\uc7a5\uc5d0\uc11c \uc7a0\uc218\uc694\uc6d0\ub4e4\uc774 \uc548\uc804\ud558\uac8c \uc120\uccb4\uae4c\uc9c0 \ub0b4\ub824\uac08 \uc218 \uc788\ub3c4\ub85d \ud574\uc8fc\ub294 \uc77c\uc885\uc758 \uae38\uc774\ub2e4. \ubb3c\uc18d\uc774 \uc798 \ubcf4\uc774\uc9c0 \uc54a\uae30 \ub54c\ubb38\uc5d0, \uc120\uc744 \uc5f0\uacb0\ud558\uae30 \uc704\ud574\uc11c\ub294 \uc190\uc73c\ub85c \uc9da\uc5b4\uac00\uba70 \uc6c0\uc9c1\uc5ec\uc57c \ud55c\ub2e4. \ub610\ud55c, \ube60\ub978 \uc720\uc18d \ub54c\ubb38\uc5d0 \uc720\ub3c4\uc120\uc744 \uc5f0\uacb0\ud558\ub354\ub77c\ub3c4 \uc120\ub07c\ub9ac \uaf2c\uc77c \uc218 \uc788\uace0(\uaf2c\uc774\uba74 \ub2e4\uc2dc \uc124\uce58\ud558\ub294 \uc791\uc5c5\uc744 \ud574\uc57c \ud55c\ub2e4) \uc7a0\uc218\uc694\uc6d0\uc758 \ub9c8\uc2a4\ud06c\uae4c\uc9c0 \ubc97\uaca8\uc9c8 \uc218 \uc788\ub2e4. \uaf2c\uc778 \uc120\uc744 \uc815\ub9ac\ud574\uac00\uba70 \uc120\uccb4\ub85c \uc9c4\uc785\uc774 \uac00\ub2a5\ud55c \uc120\uc2e4\uacfc \ud654\ubb3c\uce78 \uc785\uad6c, \uc870\ud0c0\uc2e4 \ub4f1\uc5d0 \ub3c4\ub2ec\ud55c\ub2e4. \uadf8\ub9ac\uace0 \uc120\uc744 \uadf8\uacf3\uc5d0 \ub9e4\uc5b4 \ub193\uc73c\uba74, \uadf8\ub54c\ubd80\ud130 \uc120\uccb4\uc9c4\uc785\uc744 \ubcf8\uaca9\uc801\uc73c\ub85c \uc2dc\ub3c4\ud55c\ub2e4. \uc9c4\uc785 \uc785\uad6c\ub97c \ud655\ubcf4\ud558\uace0, \ub2e4\uc2dc \uc790\uc2e0\uc758 \ubab8\uc758 \uc0dd\uba85\uc904\uc744 \uc774\uc740 \ub4a4 \uc120\uccb4 \ub0b4\ubd80\ub85c \uc9c4\uc785\uc744 \uc2dc\ub3c4\ud55c\ub2e4. \uc120\uccb4 \uc548\uc740 \uce60\ud751 \uac19\uc740 \uc5b4\ub460 \uc18d\uc774\ubbc0\ub85c \ubc29\ud5a5\uc744 \uc783\uac8c \ub418\uba74 \uc120\uccb4 \ub0b4\uc5d0 \uace0\ub9bd\ub418\uc5b4 \uc0dd\uba85\uc774 \uc704\ud5d8\ud558\uae30 \ub54c\ubb38\uc774\ub2e4. \uc120\uccb4\ub85c \uc9c4\uc785 \ud6c4\uc5d0\ub294 \ud734\ub300 \uc870\uba85 \uc7a5\ube44\ub85c \ub9e4\uc6b0 \ud63c\ub780\uc2a4\ub7ec\uc6b4 \uc120\uccb4 \ub0b4\ubd80\ub97c \uc77c\uc77c\uc774 \ub208\uc73c\ub85c \uc218\uc0c9\ud574\uc57c \ud55c\ub2e4. \uadf8\ub7ec\ub098 \uc218\uc2ec\uc5d0\uc11c\uc758 \uc791\uc5c5\uc2dc\uac04\uc740 \ucd5c\ub300 30\ubd84\uc5d0 \ubd88\uacfc\ud558\uc5ec, \uc7a0\uc218\uc694\uc6d0\uc5d0\uac8c \uc8fc\uc5b4\uc9c4 \uc2dc\uac04\uc740 \uc9e7\ub2e4. \ub530\ub77c\uc11c \uace0\ub3c4\ub85c \ud6c8\ub828\ub41c \uc77c\ubd80 \uc7a0\uc218\uc694\uc6d0\ub9cc\uc774 \uc791\uc5c5\uc5d0 \ucc38\uc5ec\ud560 \uc218 \uc788\ub294 \ud55c\uacc4\uac00 \uc788\ub2e4.", "answer": "30\ubd84", "sentence": "\uc218\uc2ec\uc5d0\uc11c\uc758 \uc791\uc5c5\uc2dc\uac04\uc740 \ucd5c\ub300 30\ubd84 \uc5d0", "paragraph_sentence": "\uc7a0\uc218\uc694\uc6d0\ub4e4\uc740 \uc6b0\uc120 \uc218\uba74\uc5d0\uc11c \uc120\uccb4\uae4c\uc9c0 \uac00\uc774\ub4dc\ub77c\uc778(\uc720\ub3c4\uc120)\uc744 \uc5f0\uacb0\ud55c\ub2e4. \uc774 \uc120\uc740 \ube60\ub978 \uc720\uc18d\uacfc \uac00\uc2dc\uac70\ub9ac\uac00 10~20cm\uc5d0 \ubd88\uacfc\ud55c \uc0ac\uace0 \ud604\uc7a5\uc5d0\uc11c \uc7a0\uc218\uc694\uc6d0\ub4e4\uc774 \uc548\uc804\ud558\uac8c \uc120\uccb4\uae4c\uc9c0 \ub0b4\ub824\uac08 \uc218 \uc788\ub3c4\ub85d \ud574\uc8fc\ub294 \uc77c\uc885\uc758 \uae38\uc774\ub2e4. \ubb3c\uc18d\uc774 \uc798 \ubcf4\uc774\uc9c0 \uc54a\uae30 \ub54c\ubb38\uc5d0, \uc120\uc744 \uc5f0\uacb0\ud558\uae30 \uc704\ud574\uc11c\ub294 \uc190\uc73c\ub85c \uc9da\uc5b4\uac00\uba70 \uc6c0\uc9c1\uc5ec\uc57c \ud55c\ub2e4. \ub610\ud55c, \ube60\ub978 \uc720\uc18d \ub54c\ubb38\uc5d0 \uc720\ub3c4\uc120\uc744 \uc5f0\uacb0\ud558\ub354\ub77c\ub3c4 \uc120\ub07c\ub9ac \uaf2c\uc77c \uc218 \uc788\uace0(\uaf2c\uc774\uba74 \ub2e4\uc2dc \uc124\uce58\ud558\ub294 \uc791\uc5c5\uc744 \ud574\uc57c \ud55c\ub2e4) \uc7a0\uc218\uc694\uc6d0\uc758 \ub9c8\uc2a4\ud06c\uae4c\uc9c0 \ubc97\uaca8\uc9c8 \uc218 \uc788\ub2e4. \uaf2c\uc778 \uc120\uc744 \uc815\ub9ac\ud574\uac00\uba70 \uc120\uccb4\ub85c \uc9c4\uc785\uc774 \uac00\ub2a5\ud55c \uc120\uc2e4\uacfc \ud654\ubb3c\uce78 \uc785\uad6c, \uc870\ud0c0\uc2e4 \ub4f1\uc5d0 \ub3c4\ub2ec\ud55c\ub2e4. \uadf8\ub9ac\uace0 \uc120\uc744 \uadf8\uacf3\uc5d0 \ub9e4\uc5b4 \ub193\uc73c\uba74, \uadf8\ub54c\ubd80\ud130 \uc120\uccb4\uc9c4\uc785\uc744 \ubcf8\uaca9\uc801\uc73c\ub85c \uc2dc\ub3c4\ud55c\ub2e4. \uc9c4\uc785 \uc785\uad6c\ub97c \ud655\ubcf4\ud558\uace0, \ub2e4\uc2dc \uc790\uc2e0\uc758 \ubab8\uc758 \uc0dd\uba85\uc904\uc744 \uc774\uc740 \ub4a4 \uc120\uccb4 \ub0b4\ubd80\ub85c \uc9c4\uc785\uc744 \uc2dc\ub3c4\ud55c\ub2e4. \uc120\uccb4 \uc548\uc740 \uce60\ud751 \uac19\uc740 \uc5b4\ub460 \uc18d\uc774\ubbc0\ub85c \ubc29\ud5a5\uc744 \uc783\uac8c \ub418\uba74 \uc120\uccb4 \ub0b4\uc5d0 \uace0\ub9bd\ub418\uc5b4 \uc0dd\uba85\uc774 \uc704\ud5d8\ud558\uae30 \ub54c\ubb38\uc774\ub2e4. \uc120\uccb4\ub85c \uc9c4\uc785 \ud6c4\uc5d0\ub294 \ud734\ub300 \uc870\uba85 \uc7a5\ube44\ub85c \ub9e4\uc6b0 \ud63c\ub780\uc2a4\ub7ec\uc6b4 \uc120\uccb4 \ub0b4\ubd80\ub97c \uc77c\uc77c\uc774 \ub208\uc73c\ub85c \uc218\uc0c9\ud574\uc57c \ud55c\ub2e4. \uadf8\ub7ec\ub098 \uc218\uc2ec\uc5d0\uc11c\uc758 \uc791\uc5c5\uc2dc\uac04\uc740 \ucd5c\ub300 30\ubd84 \uc5d0 \ubd88\uacfc\ud558\uc5ec, \uc7a0\uc218\uc694\uc6d0\uc5d0\uac8c \uc8fc\uc5b4\uc9c4 \uc2dc\uac04\uc740 \uc9e7\ub2e4. \ub530\ub77c\uc11c \uace0\ub3c4\ub85c \ud6c8\ub828\ub41c \uc77c\ubd80 \uc7a0\uc218\uc694\uc6d0\ub9cc\uc774 \uc791\uc5c5\uc5d0 \ucc38\uc5ec\ud560 \uc218 \uc788\ub294 \ud55c\uacc4\uac00 \uc788\ub2e4.", "paragraph_answer": "\uc7a0\uc218\uc694\uc6d0\ub4e4\uc740 \uc6b0\uc120 \uc218\uba74\uc5d0\uc11c \uc120\uccb4\uae4c\uc9c0 \uac00\uc774\ub4dc\ub77c\uc778(\uc720\ub3c4\uc120)\uc744 \uc5f0\uacb0\ud55c\ub2e4. \uc774 \uc120\uc740 \ube60\ub978 \uc720\uc18d\uacfc \uac00\uc2dc\uac70\ub9ac\uac00 10~20cm\uc5d0 \ubd88\uacfc\ud55c \uc0ac\uace0 \ud604\uc7a5\uc5d0\uc11c \uc7a0\uc218\uc694\uc6d0\ub4e4\uc774 \uc548\uc804\ud558\uac8c \uc120\uccb4\uae4c\uc9c0 \ub0b4\ub824\uac08 \uc218 \uc788\ub3c4\ub85d \ud574\uc8fc\ub294 \uc77c\uc885\uc758 \uae38\uc774\ub2e4. \ubb3c\uc18d\uc774 \uc798 \ubcf4\uc774\uc9c0 \uc54a\uae30 \ub54c\ubb38\uc5d0, \uc120\uc744 \uc5f0\uacb0\ud558\uae30 \uc704\ud574\uc11c\ub294 \uc190\uc73c\ub85c \uc9da\uc5b4\uac00\uba70 \uc6c0\uc9c1\uc5ec\uc57c \ud55c\ub2e4. \ub610\ud55c, \ube60\ub978 \uc720\uc18d \ub54c\ubb38\uc5d0 \uc720\ub3c4\uc120\uc744 \uc5f0\uacb0\ud558\ub354\ub77c\ub3c4 \uc120\ub07c\ub9ac \uaf2c\uc77c \uc218 \uc788\uace0(\uaf2c\uc774\uba74 \ub2e4\uc2dc \uc124\uce58\ud558\ub294 \uc791\uc5c5\uc744 \ud574\uc57c \ud55c\ub2e4) \uc7a0\uc218\uc694\uc6d0\uc758 \ub9c8\uc2a4\ud06c\uae4c\uc9c0 \ubc97\uaca8\uc9c8 \uc218 \uc788\ub2e4. \uaf2c\uc778 \uc120\uc744 \uc815\ub9ac\ud574\uac00\uba70 \uc120\uccb4\ub85c \uc9c4\uc785\uc774 \uac00\ub2a5\ud55c \uc120\uc2e4\uacfc \ud654\ubb3c\uce78 \uc785\uad6c, \uc870\ud0c0\uc2e4 \ub4f1\uc5d0 \ub3c4\ub2ec\ud55c\ub2e4. \uadf8\ub9ac\uace0 \uc120\uc744 \uadf8\uacf3\uc5d0 \ub9e4\uc5b4 \ub193\uc73c\uba74, \uadf8\ub54c\ubd80\ud130 \uc120\uccb4\uc9c4\uc785\uc744 \ubcf8\uaca9\uc801\uc73c\ub85c \uc2dc\ub3c4\ud55c\ub2e4. \uc9c4\uc785 \uc785\uad6c\ub97c \ud655\ubcf4\ud558\uace0, \ub2e4\uc2dc \uc790\uc2e0\uc758 \ubab8\uc758 \uc0dd\uba85\uc904\uc744 \uc774\uc740 \ub4a4 \uc120\uccb4 \ub0b4\ubd80\ub85c \uc9c4\uc785\uc744 \uc2dc\ub3c4\ud55c\ub2e4. \uc120\uccb4 \uc548\uc740 \uce60\ud751 \uac19\uc740 \uc5b4\ub460 \uc18d\uc774\ubbc0\ub85c \ubc29\ud5a5\uc744 \uc783\uac8c \ub418\uba74 \uc120\uccb4 \ub0b4\uc5d0 \uace0\ub9bd\ub418\uc5b4 \uc0dd\uba85\uc774 \uc704\ud5d8\ud558\uae30 \ub54c\ubb38\uc774\ub2e4. \uc120\uccb4\ub85c \uc9c4\uc785 \ud6c4\uc5d0\ub294 \ud734\ub300 \uc870\uba85 \uc7a5\ube44\ub85c \ub9e4\uc6b0 \ud63c\ub780\uc2a4\ub7ec\uc6b4 \uc120\uccb4 \ub0b4\ubd80\ub97c \uc77c\uc77c\uc774 \ub208\uc73c\ub85c \uc218\uc0c9\ud574\uc57c \ud55c\ub2e4. \uadf8\ub7ec\ub098 \uc218\uc2ec\uc5d0\uc11c\uc758 \uc791\uc5c5\uc2dc\uac04\uc740 \ucd5c\ub300 30\ubd84 \uc5d0 \ubd88\uacfc\ud558\uc5ec, \uc7a0\uc218\uc694\uc6d0\uc5d0\uac8c \uc8fc\uc5b4\uc9c4 \uc2dc\uac04\uc740 \uc9e7\ub2e4. \ub530\ub77c\uc11c \uace0\ub3c4\ub85c \ud6c8\ub828\ub41c \uc77c\ubd80 \uc7a0\uc218\uc694\uc6d0\ub9cc\uc774 \uc791\uc5c5\uc5d0 \ucc38\uc5ec\ud560 \uc218 \uc788\ub294 \ud55c\uacc4\uac00 \uc788\ub2e4.", "sentence_answer": "\uc218\uc2ec\uc5d0\uc11c\uc758 \uc791\uc5c5\uc2dc\uac04\uc740 \ucd5c\ub300 30\ubd84 \uc5d0", "paragraph_id": "6603760-2-0", "paragraph_question": "question: \uc7a0\uc218\ubd80\ub4e4\uc774 \ubc30 \uc548\uc73c\ub85c \ub4e4\uc5b4\uac00\uba74 \ucd5c\ub300 \uc5bc\ub9c8\ub098 \uc218\uc0c9\uc791\uc5c5\uc744 \ud560 \uc218 \uc788\ub294\uac00?, context: \uc7a0\uc218\uc694\uc6d0\ub4e4\uc740 \uc6b0\uc120 \uc218\uba74\uc5d0\uc11c \uc120\uccb4\uae4c\uc9c0 \uac00\uc774\ub4dc\ub77c\uc778(\uc720\ub3c4\uc120)\uc744 \uc5f0\uacb0\ud55c\ub2e4. \uc774 \uc120\uc740 \ube60\ub978 \uc720\uc18d\uacfc \uac00\uc2dc\uac70\ub9ac\uac00 10~20cm\uc5d0 \ubd88\uacfc\ud55c \uc0ac\uace0 \ud604\uc7a5\uc5d0\uc11c \uc7a0\uc218\uc694\uc6d0\ub4e4\uc774 \uc548\uc804\ud558\uac8c \uc120\uccb4\uae4c\uc9c0 \ub0b4\ub824\uac08 \uc218 \uc788\ub3c4\ub85d \ud574\uc8fc\ub294 \uc77c\uc885\uc758 \uae38\uc774\ub2e4. \ubb3c\uc18d\uc774 \uc798 \ubcf4\uc774\uc9c0 \uc54a\uae30 \ub54c\ubb38\uc5d0, \uc120\uc744 \uc5f0\uacb0\ud558\uae30 \uc704\ud574\uc11c\ub294 \uc190\uc73c\ub85c \uc9da\uc5b4\uac00\uba70 \uc6c0\uc9c1\uc5ec\uc57c \ud55c\ub2e4. \ub610\ud55c, \ube60\ub978 \uc720\uc18d \ub54c\ubb38\uc5d0 \uc720\ub3c4\uc120\uc744 \uc5f0\uacb0\ud558\ub354\ub77c\ub3c4 \uc120\ub07c\ub9ac \uaf2c\uc77c \uc218 \uc788\uace0(\uaf2c\uc774\uba74 \ub2e4\uc2dc \uc124\uce58\ud558\ub294 \uc791\uc5c5\uc744 \ud574\uc57c \ud55c\ub2e4) \uc7a0\uc218\uc694\uc6d0\uc758 \ub9c8\uc2a4\ud06c\uae4c\uc9c0 \ubc97\uaca8\uc9c8 \uc218 \uc788\ub2e4. \uaf2c\uc778 \uc120\uc744 \uc815\ub9ac\ud574\uac00\uba70 \uc120\uccb4\ub85c \uc9c4\uc785\uc774 \uac00\ub2a5\ud55c \uc120\uc2e4\uacfc \ud654\ubb3c\uce78 \uc785\uad6c, \uc870\ud0c0\uc2e4 \ub4f1\uc5d0 \ub3c4\ub2ec\ud55c\ub2e4. \uadf8\ub9ac\uace0 \uc120\uc744 \uadf8\uacf3\uc5d0 \ub9e4\uc5b4 \ub193\uc73c\uba74, \uadf8\ub54c\ubd80\ud130 \uc120\uccb4\uc9c4\uc785\uc744 \ubcf8\uaca9\uc801\uc73c\ub85c \uc2dc\ub3c4\ud55c\ub2e4. \uc9c4\uc785 \uc785\uad6c\ub97c \ud655\ubcf4\ud558\uace0, \ub2e4\uc2dc \uc790\uc2e0\uc758 \ubab8\uc758 \uc0dd\uba85\uc904\uc744 \uc774\uc740 \ub4a4 \uc120\uccb4 \ub0b4\ubd80\ub85c \uc9c4\uc785\uc744 \uc2dc\ub3c4\ud55c\ub2e4. \uc120\uccb4 \uc548\uc740 \uce60\ud751 \uac19\uc740 \uc5b4\ub460 \uc18d\uc774\ubbc0\ub85c \ubc29\ud5a5\uc744 \uc783\uac8c \ub418\uba74 \uc120\uccb4 \ub0b4\uc5d0 \uace0\ub9bd\ub418\uc5b4 \uc0dd\uba85\uc774 \uc704\ud5d8\ud558\uae30 \ub54c\ubb38\uc774\ub2e4. \uc120\uccb4\ub85c \uc9c4\uc785 \ud6c4\uc5d0\ub294 \ud734\ub300 \uc870\uba85 \uc7a5\ube44\ub85c \ub9e4\uc6b0 \ud63c\ub780\uc2a4\ub7ec\uc6b4 \uc120\uccb4 \ub0b4\ubd80\ub97c \uc77c\uc77c\uc774 \ub208\uc73c\ub85c \uc218\uc0c9\ud574\uc57c \ud55c\ub2e4. \uadf8\ub7ec\ub098 \uc218\uc2ec\uc5d0\uc11c\uc758 \uc791\uc5c5\uc2dc\uac04\uc740 \ucd5c\ub300 30\ubd84\uc5d0 \ubd88\uacfc\ud558\uc5ec, \uc7a0\uc218\uc694\uc6d0\uc5d0\uac8c \uc8fc\uc5b4\uc9c4 \uc2dc\uac04\uc740 \uc9e7\ub2e4. \ub530\ub77c\uc11c \uace0\ub3c4\ub85c \ud6c8\ub828\ub41c \uc77c\ubd80 \uc7a0\uc218\uc694\uc6d0\ub9cc\uc774 \uc791\uc5c5\uc5d0 \ucc38\uc5ec\ud560 \uc218 \uc788\ub294 \ud55c\uacc4\uac00 \uc788\ub2e4."} {"question": "\ubcc4\uc758 \ucee4\ube44\uc758 \uc2a4\ud14c\uc774\uc9c0 \uc911\uac04\uc5d0 \uc911\uac04\ubcf4\uc2a4\ub294 \ubb34\uc5c7\uacfc \ud568\uaed8 \ub098\uc635\ub2c8\uae4c?", "paragraph": "\uae08\uc791\uc758 \uc8fc\uc694 \ub0b4\uc6a9\uc73c\ub85c \ub2e4\ub8e8\uc5b4\uc9c0\ub2e4\uc2dc\ud53c \ud558\ub294, \uc624\ub9dd\uc131\uc778 \ud31d\uc2a4\ud0c0\ub97c \ub3cc\uc544 \ucd1d \ub2e4\uc12f \uc9c0\uc5ed\uc5d0\uc11c \ubcf4\uc2a4\uc5d0\uac8c\uc11c \ud769\uc5b4\uc9c4 \ubc30\uc758 \ubd80\ud488\uc744 \ucc3e\uc544\ub2e4\uc8fc\ub294 \ub0b4\uc6a9\uc774\ub2e4. \ub3d9\uc2dc\uc5d0, \ud30c\uace0\ub4e4\uae30 \uc694\uc18c\ub85c \ud568\uaed8 \uc783\uc5b4\ubc84\ub9b0 \ucd1d 120\uac1c\uc758 \uc5d0\ub108\uc9c0 \uc2a4\ud53c\uc5b4 \ub418\ucc3e\uae30 \ubbf8\uc158\ub3c4 \uc2dc\uc791. \uc2a4\ud14c\uc774\uc9c0\ub97c \uc9c4\ud589\ud558\ub2e4\ubcf4\uba74 \ube5b\uacfc \ud568\uaed8 \uc911\uac04\ubcf4\uc2a4\uac00 \ud280\uc5b4\ub098\uc628\ub2e4. \uc2a4\ud53c\uc5b4 \ub85c\ud37c\ub97c \uc81c\uc678\ud55c \ubaa8\ub4e0 \uc911\uac04\ubcf4\uc2a4\ub294 \uc4f0\ub7ec\ud2b8\ub9ac\uba74 \uc7a0\uc2dc\ub3d9\uc548 \ub0a8\ub294\ub370, \uadf8\uac78 \ud761\uc218\ud558\uba74 \ud574\ub2f9 \uce74\ud53c\ub2a5\ub825\uc744 \uc5bb\uc744 \uc218 \uc788\ub2e4. 100% \ud074\ub9ac\uc5b4\ub97c \ub178\ub9b0\ub2e4\uba74 \uc774\uac78 \uadf8\ub0e5 \uc9c0\ub098\uce58\uba74 \uc548\ub418\ub294\ub370, \uadf8 \uc774\uc720\ub294 \uc911\uac04\ubcf4\uc2a4\uac00 \ub098\uc628\ud6c4 \uc9c4\ud589\ud558\ub2e4\ubcf4\uba74 \uadf8 \uce74\ud53c\ub2a5\ub825\uc73c\ub85c \uc2a4\ud53c\uc5b4\ub97c \ud68d\ub4dd\ud574\uc57c\ud558\ub294 \uc0c1\ud669\uc774 \ubb34\uc870\uac74 \uc788\uae30\ub54c\ubb38\uc774\ub2e4.(\uc2a4\ud53c\uc5b4\ub97c \ubaa8\ub450 \ud68d\ub4dd\ud574\ub3c4 \ud30c\uc77c\uc758 \uc9c4\ud589\ub3c4\ub294 94%\ubc16\uc5d0 \ub418\uc9c0 \uc54a\ub294\ub2e4.) EX\ubaa8\ub4dc\uc5d0\uc11c\ub294 \uc678\ud615\uc774 \ubcc0\uacbd\ub418\uba70, \ud328\ud134\uc774 \ub2ec\ub77c\uc9c0\ubbc0\ub85c \uc8fc\uc758. \ud558\uc9c0\ub9cc \ubc29\uc5b4\ub825\uc774\ub098 HP\uc740 \uadf8\ub300\ub85c\ub2e4. \uc2a4\ud14c\uc774\uc9c0 \uc218\ub294 \ubcf4\uc2a4 \uc2a4\ud14c\uc774\uc9c0\uae4c\uc9c0 \ud569\ud558\uc5ec \ud45c\uae30. \ucc38\uace0\ub85c 1~7\uc2a4\ud14c\uc774\uc9c0\uc758 \uccab\ubc88\uc9f8 \uc54c\ud30c\ubcb3\uc744 \ud569\ud558\uba74 \ub098\uc624\ub294 \ub2e8\uc5b4\ub294 crowned. \uc655\uad00\uc744 \uc4f4\uc774\ub77c\ub294 \ub73b\uc774\uc9c0\ub9cc \ud6c4\ubc18\ubd80 \uac00\uba74 \uc774\ub2e8\uc5b4\uac00 \uc774 \uc791\ud488\uc758 \uc2a4\ud3ec\uc77c\ub7ec\uc5d0 \ud574\ub2f9\ub418\ub294 \ub0b4\uc6a9\uc774\ub77c\ub294\uac78 \uc9d0\uc791\ud560 \uc218\uc788\ub2e4. \ucc38\uace0\ub85c \ub808\ubca8 \uc774\ub984\uc774 \ub2e4 \uba39\uc744 \uac83\uc774\ub2e4.", "answer": "\ube5b", "sentence": "\uc2a4\ud14c\uc774\uc9c0\ub97c \uc9c4\ud589\ud558\ub2e4\ubcf4\uba74 \ube5b \uacfc \ud568\uaed8 \uc911\uac04\ubcf4\uc2a4\uac00 \ud280\uc5b4\ub098\uc628\ub2e4.", "paragraph_sentence": "\uae08\uc791\uc758 \uc8fc\uc694 \ub0b4\uc6a9\uc73c\ub85c \ub2e4\ub8e8\uc5b4\uc9c0\ub2e4\uc2dc\ud53c \ud558\ub294, \uc624\ub9dd\uc131\uc778 \ud31d\uc2a4\ud0c0\ub97c \ub3cc\uc544 \ucd1d \ub2e4\uc12f \uc9c0\uc5ed\uc5d0\uc11c \ubcf4\uc2a4\uc5d0\uac8c\uc11c \ud769\uc5b4\uc9c4 \ubc30\uc758 \ubd80\ud488\uc744 \ucc3e\uc544\ub2e4\uc8fc\ub294 \ub0b4\uc6a9\uc774\ub2e4. \ub3d9\uc2dc\uc5d0, \ud30c\uace0\ub4e4\uae30 \uc694\uc18c\ub85c \ud568\uaed8 \uc783\uc5b4\ubc84\ub9b0 \ucd1d 120\uac1c\uc758 \uc5d0\ub108\uc9c0 \uc2a4\ud53c\uc5b4 \ub418\ucc3e\uae30 \ubbf8\uc158\ub3c4 \uc2dc\uc791. \uc2a4\ud14c\uc774\uc9c0\ub97c \uc9c4\ud589\ud558\ub2e4\ubcf4\uba74 \ube5b \uacfc \ud568\uaed8 \uc911\uac04\ubcf4\uc2a4\uac00 \ud280\uc5b4\ub098\uc628\ub2e4. \uc2a4\ud53c\uc5b4 \ub85c\ud37c\ub97c \uc81c\uc678\ud55c \ubaa8\ub4e0 \uc911\uac04\ubcf4\uc2a4\ub294 \uc4f0\ub7ec\ud2b8\ub9ac\uba74 \uc7a0\uc2dc\ub3d9\uc548 \ub0a8\ub294\ub370, \uadf8\uac78 \ud761\uc218\ud558\uba74 \ud574\ub2f9 \uce74\ud53c\ub2a5\ub825\uc744 \uc5bb\uc744 \uc218 \uc788\ub2e4. 100% \ud074\ub9ac\uc5b4\ub97c \ub178\ub9b0\ub2e4\uba74 \uc774\uac78 \uadf8\ub0e5 \uc9c0\ub098\uce58\uba74 \uc548\ub418\ub294\ub370, \uadf8 \uc774\uc720\ub294 \uc911\uac04\ubcf4\uc2a4\uac00 \ub098\uc628\ud6c4 \uc9c4\ud589\ud558\ub2e4\ubcf4\uba74 \uadf8 \uce74\ud53c\ub2a5\ub825\uc73c\ub85c \uc2a4\ud53c\uc5b4\ub97c \ud68d\ub4dd\ud574\uc57c\ud558\ub294 \uc0c1\ud669\uc774 \ubb34\uc870\uac74 \uc788\uae30\ub54c\ubb38\uc774\ub2e4.(\uc2a4\ud53c\uc5b4\ub97c \ubaa8\ub450 \ud68d\ub4dd\ud574\ub3c4 \ud30c\uc77c\uc758 \uc9c4\ud589\ub3c4\ub294 94%\ubc16\uc5d0 \ub418\uc9c0 \uc54a\ub294\ub2e4. ) EX\ubaa8\ub4dc\uc5d0\uc11c\ub294 \uc678\ud615\uc774 \ubcc0\uacbd\ub418\uba70, \ud328\ud134\uc774 \ub2ec\ub77c\uc9c0\ubbc0\ub85c \uc8fc\uc758. \ud558\uc9c0\ub9cc \ubc29\uc5b4\ub825\uc774\ub098 HP\uc740 \uadf8\ub300\ub85c\ub2e4. \uc2a4\ud14c\uc774\uc9c0 \uc218\ub294 \ubcf4\uc2a4 \uc2a4\ud14c\uc774\uc9c0\uae4c\uc9c0 \ud569\ud558\uc5ec \ud45c\uae30. \ucc38\uace0\ub85c 1~7\uc2a4\ud14c\uc774\uc9c0\uc758 \uccab\ubc88\uc9f8 \uc54c\ud30c\ubcb3\uc744 \ud569\ud558\uba74 \ub098\uc624\ub294 \ub2e8\uc5b4\ub294 crowned. \uc655\uad00\uc744 \uc4f4\uc774\ub77c\ub294 \ub73b\uc774\uc9c0\ub9cc \ud6c4\ubc18\ubd80 \uac00\uba74 \uc774\ub2e8\uc5b4\uac00 \uc774 \uc791\ud488\uc758 \uc2a4\ud3ec\uc77c\ub7ec\uc5d0 \ud574\ub2f9\ub418\ub294 \ub0b4\uc6a9\uc774\ub77c\ub294\uac78 \uc9d0\uc791\ud560 \uc218\uc788\ub2e4. \ucc38\uace0\ub85c \ub808\ubca8 \uc774\ub984\uc774 \ub2e4 \uba39\uc744 \uac83\uc774\ub2e4.", "paragraph_answer": "\uae08\uc791\uc758 \uc8fc\uc694 \ub0b4\uc6a9\uc73c\ub85c \ub2e4\ub8e8\uc5b4\uc9c0\ub2e4\uc2dc\ud53c \ud558\ub294, \uc624\ub9dd\uc131\uc778 \ud31d\uc2a4\ud0c0\ub97c \ub3cc\uc544 \ucd1d \ub2e4\uc12f \uc9c0\uc5ed\uc5d0\uc11c \ubcf4\uc2a4\uc5d0\uac8c\uc11c \ud769\uc5b4\uc9c4 \ubc30\uc758 \ubd80\ud488\uc744 \ucc3e\uc544\ub2e4\uc8fc\ub294 \ub0b4\uc6a9\uc774\ub2e4. \ub3d9\uc2dc\uc5d0, \ud30c\uace0\ub4e4\uae30 \uc694\uc18c\ub85c \ud568\uaed8 \uc783\uc5b4\ubc84\ub9b0 \ucd1d 120\uac1c\uc758 \uc5d0\ub108\uc9c0 \uc2a4\ud53c\uc5b4 \ub418\ucc3e\uae30 \ubbf8\uc158\ub3c4 \uc2dc\uc791. \uc2a4\ud14c\uc774\uc9c0\ub97c \uc9c4\ud589\ud558\ub2e4\ubcf4\uba74 \ube5b \uacfc \ud568\uaed8 \uc911\uac04\ubcf4\uc2a4\uac00 \ud280\uc5b4\ub098\uc628\ub2e4. \uc2a4\ud53c\uc5b4 \ub85c\ud37c\ub97c \uc81c\uc678\ud55c \ubaa8\ub4e0 \uc911\uac04\ubcf4\uc2a4\ub294 \uc4f0\ub7ec\ud2b8\ub9ac\uba74 \uc7a0\uc2dc\ub3d9\uc548 \ub0a8\ub294\ub370, \uadf8\uac78 \ud761\uc218\ud558\uba74 \ud574\ub2f9 \uce74\ud53c\ub2a5\ub825\uc744 \uc5bb\uc744 \uc218 \uc788\ub2e4. 100% \ud074\ub9ac\uc5b4\ub97c \ub178\ub9b0\ub2e4\uba74 \uc774\uac78 \uadf8\ub0e5 \uc9c0\ub098\uce58\uba74 \uc548\ub418\ub294\ub370, \uadf8 \uc774\uc720\ub294 \uc911\uac04\ubcf4\uc2a4\uac00 \ub098\uc628\ud6c4 \uc9c4\ud589\ud558\ub2e4\ubcf4\uba74 \uadf8 \uce74\ud53c\ub2a5\ub825\uc73c\ub85c \uc2a4\ud53c\uc5b4\ub97c \ud68d\ub4dd\ud574\uc57c\ud558\ub294 \uc0c1\ud669\uc774 \ubb34\uc870\uac74 \uc788\uae30\ub54c\ubb38\uc774\ub2e4.(\uc2a4\ud53c\uc5b4\ub97c \ubaa8\ub450 \ud68d\ub4dd\ud574\ub3c4 \ud30c\uc77c\uc758 \uc9c4\ud589\ub3c4\ub294 94%\ubc16\uc5d0 \ub418\uc9c0 \uc54a\ub294\ub2e4.) EX\ubaa8\ub4dc\uc5d0\uc11c\ub294 \uc678\ud615\uc774 \ubcc0\uacbd\ub418\uba70, \ud328\ud134\uc774 \ub2ec\ub77c\uc9c0\ubbc0\ub85c \uc8fc\uc758. \ud558\uc9c0\ub9cc \ubc29\uc5b4\ub825\uc774\ub098 HP\uc740 \uadf8\ub300\ub85c\ub2e4. \uc2a4\ud14c\uc774\uc9c0 \uc218\ub294 \ubcf4\uc2a4 \uc2a4\ud14c\uc774\uc9c0\uae4c\uc9c0 \ud569\ud558\uc5ec \ud45c\uae30. \ucc38\uace0\ub85c 1~7\uc2a4\ud14c\uc774\uc9c0\uc758 \uccab\ubc88\uc9f8 \uc54c\ud30c\ubcb3\uc744 \ud569\ud558\uba74 \ub098\uc624\ub294 \ub2e8\uc5b4\ub294 crowned. \uc655\uad00\uc744 \uc4f4\uc774\ub77c\ub294 \ub73b\uc774\uc9c0\ub9cc \ud6c4\ubc18\ubd80 \uac00\uba74 \uc774\ub2e8\uc5b4\uac00 \uc774 \uc791\ud488\uc758 \uc2a4\ud3ec\uc77c\ub7ec\uc5d0 \ud574\ub2f9\ub418\ub294 \ub0b4\uc6a9\uc774\ub77c\ub294\uac78 \uc9d0\uc791\ud560 \uc218\uc788\ub2e4. \ucc38\uace0\ub85c \ub808\ubca8 \uc774\ub984\uc774 \ub2e4 \uba39\uc744 \uac83\uc774\ub2e4.", "sentence_answer": "\uc2a4\ud14c\uc774\uc9c0\ub97c \uc9c4\ud589\ud558\ub2e4\ubcf4\uba74 \ube5b \uacfc \ud568\uaed8 \uc911\uac04\ubcf4\uc2a4\uac00 \ud280\uc5b4\ub098\uc628\ub2e4.", "paragraph_id": "6508600-0-1", "paragraph_question": "question: \ubcc4\uc758 \ucee4\ube44\uc758 \uc2a4\ud14c\uc774\uc9c0 \uc911\uac04\uc5d0 \uc911\uac04\ubcf4\uc2a4\ub294 \ubb34\uc5c7\uacfc \ud568\uaed8 \ub098\uc635\ub2c8\uae4c?, context: \uae08\uc791\uc758 \uc8fc\uc694 \ub0b4\uc6a9\uc73c\ub85c \ub2e4\ub8e8\uc5b4\uc9c0\ub2e4\uc2dc\ud53c \ud558\ub294, \uc624\ub9dd\uc131\uc778 \ud31d\uc2a4\ud0c0\ub97c \ub3cc\uc544 \ucd1d \ub2e4\uc12f \uc9c0\uc5ed\uc5d0\uc11c \ubcf4\uc2a4\uc5d0\uac8c\uc11c \ud769\uc5b4\uc9c4 \ubc30\uc758 \ubd80\ud488\uc744 \ucc3e\uc544\ub2e4\uc8fc\ub294 \ub0b4\uc6a9\uc774\ub2e4. \ub3d9\uc2dc\uc5d0, \ud30c\uace0\ub4e4\uae30 \uc694\uc18c\ub85c \ud568\uaed8 \uc783\uc5b4\ubc84\ub9b0 \ucd1d 120\uac1c\uc758 \uc5d0\ub108\uc9c0 \uc2a4\ud53c\uc5b4 \ub418\ucc3e\uae30 \ubbf8\uc158\ub3c4 \uc2dc\uc791. \uc2a4\ud14c\uc774\uc9c0\ub97c \uc9c4\ud589\ud558\ub2e4\ubcf4\uba74 \ube5b\uacfc \ud568\uaed8 \uc911\uac04\ubcf4\uc2a4\uac00 \ud280\uc5b4\ub098\uc628\ub2e4. \uc2a4\ud53c\uc5b4 \ub85c\ud37c\ub97c \uc81c\uc678\ud55c \ubaa8\ub4e0 \uc911\uac04\ubcf4\uc2a4\ub294 \uc4f0\ub7ec\ud2b8\ub9ac\uba74 \uc7a0\uc2dc\ub3d9\uc548 \ub0a8\ub294\ub370, \uadf8\uac78 \ud761\uc218\ud558\uba74 \ud574\ub2f9 \uce74\ud53c\ub2a5\ub825\uc744 \uc5bb\uc744 \uc218 \uc788\ub2e4. 100% \ud074\ub9ac\uc5b4\ub97c \ub178\ub9b0\ub2e4\uba74 \uc774\uac78 \uadf8\ub0e5 \uc9c0\ub098\uce58\uba74 \uc548\ub418\ub294\ub370, \uadf8 \uc774\uc720\ub294 \uc911\uac04\ubcf4\uc2a4\uac00 \ub098\uc628\ud6c4 \uc9c4\ud589\ud558\ub2e4\ubcf4\uba74 \uadf8 \uce74\ud53c\ub2a5\ub825\uc73c\ub85c \uc2a4\ud53c\uc5b4\ub97c \ud68d\ub4dd\ud574\uc57c\ud558\ub294 \uc0c1\ud669\uc774 \ubb34\uc870\uac74 \uc788\uae30\ub54c\ubb38\uc774\ub2e4.(\uc2a4\ud53c\uc5b4\ub97c \ubaa8\ub450 \ud68d\ub4dd\ud574\ub3c4 \ud30c\uc77c\uc758 \uc9c4\ud589\ub3c4\ub294 94%\ubc16\uc5d0 \ub418\uc9c0 \uc54a\ub294\ub2e4.) EX\ubaa8\ub4dc\uc5d0\uc11c\ub294 \uc678\ud615\uc774 \ubcc0\uacbd\ub418\uba70, \ud328\ud134\uc774 \ub2ec\ub77c\uc9c0\ubbc0\ub85c \uc8fc\uc758. \ud558\uc9c0\ub9cc \ubc29\uc5b4\ub825\uc774\ub098 HP\uc740 \uadf8\ub300\ub85c\ub2e4. \uc2a4\ud14c\uc774\uc9c0 \uc218\ub294 \ubcf4\uc2a4 \uc2a4\ud14c\uc774\uc9c0\uae4c\uc9c0 \ud569\ud558\uc5ec \ud45c\uae30. \ucc38\uace0\ub85c 1~7\uc2a4\ud14c\uc774\uc9c0\uc758 \uccab\ubc88\uc9f8 \uc54c\ud30c\ubcb3\uc744 \ud569\ud558\uba74 \ub098\uc624\ub294 \ub2e8\uc5b4\ub294 crowned. \uc655\uad00\uc744 \uc4f4\uc774\ub77c\ub294 \ub73b\uc774\uc9c0\ub9cc \ud6c4\ubc18\ubd80 \uac00\uba74 \uc774\ub2e8\uc5b4\uac00 \uc774 \uc791\ud488\uc758 \uc2a4\ud3ec\uc77c\ub7ec\uc5d0 \ud574\ub2f9\ub418\ub294 \ub0b4\uc6a9\uc774\ub77c\ub294\uac78 \uc9d0\uc791\ud560 \uc218\uc788\ub2e4. \ucc38\uace0\ub85c \ub808\ubca8 \uc774\ub984\uc774 \ub2e4 \uba39\uc744 \uac83\uc774\ub2e4."} {"question": "\uae40\ub300\uc911 \uc9d1\uad8c \uc2dc\uae30\uc5d0 \ub298\uc5b4\ub09c \uacbd\uc0c1\uc218\uc9c0 \ud751\uc790\uc758 \uc561\uc218\ub294?", "paragraph": "\ud55c\uad6d\uc740\ud589\uacfc \ud1b5\uacc4\uccad \ub4f1\uc5d0 \ub530\ub974\uba74 \uae40\ub300\uc911\uc758 \uc9d1\uad8c \uc2dc\uae30\uc778 1998\ub144 ~ 2002\ub144 \uacbd\uc0c1\uc218\uc9c0 \ud751\uc790\ub294 906\uc5b5\ub2ec\ub7ec \uc99d\uac00\ud588\ub2e4. \uc5f0\ud3c9\uade0 \uc99d\uac00\uc561\uc740 181\uc5b5 1\ucc9c 400\ub9cc\ub2ec\ub7ec\ub85c \uac00\uc7a5 \ub9ce\uc558\ub2e4. \uae40\ub300\uc911 \uc815\ubd80 \uc2dc\uc808\uc5d0\ub294 \uacbd\uc0c1\ud751\uc790\uc5d0 \ub530\ub978 \uc678\ud654 \uc720\uc785 \uc99d\uac00\ub85c \uc678\ud658\ubcf4\uc720\uc561\ub3c4 \ub9ce\uc774 \ub298\uc5b4\ub0ac\ub2e4. \uae40\uc601\uc0bc \uc815\ubd80 \ub9d0\uae30\uc778 1997\ub144\ub9d0 204\uc5b5 600\ub9cc\ub2ec\ub7ec\uc5d0 \ubd88\uacfc\ud558\ub358 \uc678\ud658\ubcf4\uc720\uc561\uc740 \uae40\ub300\uc911 \uc815\ubd80 \ub9d0\uae30\uc778 2002\ub144\ub9d0\uc5d0\ub294 1\ucc9c 214\uc5b5 1\ucc9c 300\ub9cc\ub2ec\ub7ec\ub85c \ub298\uc5b4\ub098\uba74\uc11c 5\ub144\uac04 1\ucc9c 10\uc5b5 700\ub9cc\ub2ec\ub7ec \uc99d\uac00\ud588\ub2e4. \uc774\ub85c \uc778\ud574 \uc138\uacc4 4\ub300 \uc678\ud658\ubcf4\uc720\uad6d\uc774 \ub418\uc5c8\ub2e4. \uc5f0\ud3c9\uade0 \uc99d\uac00\uc561\uc740 202\uc5b5 100\ub9cc\ub2ec\ub7ec\ub85c \ub178\ubb34\ud604 \uc815\ubd80\uc758 281\uc5b5 6\ucc9c 200\ub9cc\ub2ec\ub7ec\uc5d0 \uc774\uc5b4 \ub450 \ubc88\uc9f8\ub85c \ud070 \uaddc\ubaa8\ub2e4. \uae40\ub300\uc911 \uc815\ubd80 \ub54c\ub294 \ubb3c\uac00\ub3c4 \ube44\uad50\uc801 \uc548\uc815\uc801\uc778 \ubaa8\uc2b5\uc744 \ubcf4\uc600\ub2e4. \uae40\ub300\uc911 \uc815\ubd80 \uc2dc\uc808 \uc5f0\ud3c9\uade0 \uc18c\ube44\uc790\ubb3c\uac00 \uc0c1\uc2b9\ub960\uc740 \uc5f0\ud3c9\uade0 3.5%\ub85c \ub178\ubb34\ud604 \uc815\ubd80 \ub54c\uc758 3.0%\uc5d0 \uc774\uc5b4 \ub450 \ubc88\uc9f8\ub85c \ub0ae\uc558\ub2e4. \ubb3c\uac00 \uc0c1\uc2b9\ub960\uc740 \ub178\ud0dc\uc6b0 \uc815\ubd80 \ub54c 7.4%\ub85c \uac00\uc7a5 \ub192\uc558\uace0 \uc804\ub450\ud658 \uc815\ubd80 6.1%, \uae40\uc601\uc0bc \uc815\ubd80 5.0% \ub4f1\uc774\uc5c8\ub2e4. \uc678\ud658\uc704\uae30\uc5d0 \ub530\ub978 \uae30\uc5c5 \ubd80\ub3c4 \ub4f1\uc758 \uc5ec\ud30c\ub85c \uacbd\uc81c\uc131\uc7a5\ub960\uacfc \uace0\uc6a9\ub960\uc740 \uc774\uc804 \uc815\uad8c\ub4e4\ubcf4\ub2e4 \ud06c\uac8c \ub0ae\uc558\ub2e4.", "answer": "906\uc5b5\ub2ec\ub7ec", "sentence": "\ud55c\uad6d\uc740\ud589\uacfc \ud1b5\uacc4\uccad \ub4f1\uc5d0 \ub530\ub974\uba74 \uae40\ub300\uc911\uc758 \uc9d1\uad8c \uc2dc\uae30\uc778 1998\ub144 ~ 2002\ub144 \uacbd\uc0c1\uc218\uc9c0 \ud751\uc790\ub294 906\uc5b5\ub2ec\ub7ec \uc99d\uac00\ud588\ub2e4.", "paragraph_sentence": " \ud55c\uad6d\uc740\ud589\uacfc \ud1b5\uacc4\uccad \ub4f1\uc5d0 \ub530\ub974\uba74 \uae40\ub300\uc911\uc758 \uc9d1\uad8c \uc2dc\uae30\uc778 1998\ub144 ~ 2002\ub144 \uacbd\uc0c1\uc218\uc9c0 \ud751\uc790\ub294 906\uc5b5\ub2ec\ub7ec \uc99d\uac00\ud588\ub2e4. \uc5f0\ud3c9\uade0 \uc99d\uac00\uc561\uc740 181\uc5b5 1\ucc9c 400\ub9cc\ub2ec\ub7ec\ub85c \uac00\uc7a5 \ub9ce\uc558\ub2e4. \uae40\ub300\uc911 \uc815\ubd80 \uc2dc\uc808\uc5d0\ub294 \uacbd\uc0c1\ud751\uc790\uc5d0 \ub530\ub978 \uc678\ud654 \uc720\uc785 \uc99d\uac00\ub85c \uc678\ud658\ubcf4\uc720\uc561\ub3c4 \ub9ce\uc774 \ub298\uc5b4\ub0ac\ub2e4. \uae40\uc601\uc0bc \uc815\ubd80 \ub9d0\uae30\uc778 1997\ub144\ub9d0 204\uc5b5 600\ub9cc\ub2ec\ub7ec\uc5d0 \ubd88\uacfc\ud558\ub358 \uc678\ud658\ubcf4\uc720\uc561\uc740 \uae40\ub300\uc911 \uc815\ubd80 \ub9d0\uae30\uc778 2002\ub144\ub9d0\uc5d0\ub294 1\ucc9c 214\uc5b5 1\ucc9c 300\ub9cc\ub2ec\ub7ec\ub85c \ub298\uc5b4\ub098\uba74\uc11c 5\ub144\uac04 1\ucc9c 10\uc5b5 700\ub9cc\ub2ec\ub7ec \uc99d\uac00\ud588\ub2e4. \uc774\ub85c \uc778\ud574 \uc138\uacc4 4\ub300 \uc678\ud658\ubcf4\uc720\uad6d\uc774 \ub418\uc5c8\ub2e4. \uc5f0\ud3c9\uade0 \uc99d\uac00\uc561\uc740 202\uc5b5 100\ub9cc\ub2ec\ub7ec\ub85c \ub178\ubb34\ud604 \uc815\ubd80\uc758 281\uc5b5 6\ucc9c 200\ub9cc\ub2ec\ub7ec\uc5d0 \uc774\uc5b4 \ub450 \ubc88\uc9f8\ub85c \ud070 \uaddc\ubaa8\ub2e4. \uae40\ub300\uc911 \uc815\ubd80 \ub54c\ub294 \ubb3c\uac00\ub3c4 \ube44\uad50\uc801 \uc548\uc815\uc801\uc778 \ubaa8\uc2b5\uc744 \ubcf4\uc600\ub2e4. \uae40\ub300\uc911 \uc815\ubd80 \uc2dc\uc808 \uc5f0\ud3c9\uade0 \uc18c\ube44\uc790\ubb3c\uac00 \uc0c1\uc2b9\ub960\uc740 \uc5f0\ud3c9\uade0 3.5%\ub85c \ub178\ubb34\ud604 \uc815\ubd80 \ub54c\uc758 3.0%\uc5d0 \uc774\uc5b4 \ub450 \ubc88\uc9f8\ub85c \ub0ae\uc558\ub2e4. \ubb3c\uac00 \uc0c1\uc2b9\ub960\uc740 \ub178\ud0dc\uc6b0 \uc815\ubd80 \ub54c 7.4%\ub85c \uac00\uc7a5 \ub192\uc558\uace0 \uc804\ub450\ud658 \uc815\ubd80 6.1%, \uae40\uc601\uc0bc \uc815\ubd80 5.0% \ub4f1\uc774\uc5c8\ub2e4. \uc678\ud658\uc704\uae30\uc5d0 \ub530\ub978 \uae30\uc5c5 \ubd80\ub3c4 \ub4f1\uc758 \uc5ec\ud30c\ub85c \uacbd\uc81c\uc131\uc7a5\ub960\uacfc \uace0\uc6a9\ub960\uc740 \uc774\uc804 \uc815\uad8c\ub4e4\ubcf4\ub2e4 \ud06c\uac8c \ub0ae\uc558\ub2e4.", "paragraph_answer": "\ud55c\uad6d\uc740\ud589\uacfc \ud1b5\uacc4\uccad \ub4f1\uc5d0 \ub530\ub974\uba74 \uae40\ub300\uc911\uc758 \uc9d1\uad8c \uc2dc\uae30\uc778 1998\ub144 ~ 2002\ub144 \uacbd\uc0c1\uc218\uc9c0 \ud751\uc790\ub294 906\uc5b5\ub2ec\ub7ec \uc99d\uac00\ud588\ub2e4. \uc5f0\ud3c9\uade0 \uc99d\uac00\uc561\uc740 181\uc5b5 1\ucc9c 400\ub9cc\ub2ec\ub7ec\ub85c \uac00\uc7a5 \ub9ce\uc558\ub2e4. \uae40\ub300\uc911 \uc815\ubd80 \uc2dc\uc808\uc5d0\ub294 \uacbd\uc0c1\ud751\uc790\uc5d0 \ub530\ub978 \uc678\ud654 \uc720\uc785 \uc99d\uac00\ub85c \uc678\ud658\ubcf4\uc720\uc561\ub3c4 \ub9ce\uc774 \ub298\uc5b4\ub0ac\ub2e4. \uae40\uc601\uc0bc \uc815\ubd80 \ub9d0\uae30\uc778 1997\ub144\ub9d0 204\uc5b5 600\ub9cc\ub2ec\ub7ec\uc5d0 \ubd88\uacfc\ud558\ub358 \uc678\ud658\ubcf4\uc720\uc561\uc740 \uae40\ub300\uc911 \uc815\ubd80 \ub9d0\uae30\uc778 2002\ub144\ub9d0\uc5d0\ub294 1\ucc9c 214\uc5b5 1\ucc9c 300\ub9cc\ub2ec\ub7ec\ub85c \ub298\uc5b4\ub098\uba74\uc11c 5\ub144\uac04 1\ucc9c 10\uc5b5 700\ub9cc\ub2ec\ub7ec \uc99d\uac00\ud588\ub2e4. \uc774\ub85c \uc778\ud574 \uc138\uacc4 4\ub300 \uc678\ud658\ubcf4\uc720\uad6d\uc774 \ub418\uc5c8\ub2e4. \uc5f0\ud3c9\uade0 \uc99d\uac00\uc561\uc740 202\uc5b5 100\ub9cc\ub2ec\ub7ec\ub85c \ub178\ubb34\ud604 \uc815\ubd80\uc758 281\uc5b5 6\ucc9c 200\ub9cc\ub2ec\ub7ec\uc5d0 \uc774\uc5b4 \ub450 \ubc88\uc9f8\ub85c \ud070 \uaddc\ubaa8\ub2e4. \uae40\ub300\uc911 \uc815\ubd80 \ub54c\ub294 \ubb3c\uac00\ub3c4 \ube44\uad50\uc801 \uc548\uc815\uc801\uc778 \ubaa8\uc2b5\uc744 \ubcf4\uc600\ub2e4. \uae40\ub300\uc911 \uc815\ubd80 \uc2dc\uc808 \uc5f0\ud3c9\uade0 \uc18c\ube44\uc790\ubb3c\uac00 \uc0c1\uc2b9\ub960\uc740 \uc5f0\ud3c9\uade0 3.5%\ub85c \ub178\ubb34\ud604 \uc815\ubd80 \ub54c\uc758 3.0%\uc5d0 \uc774\uc5b4 \ub450 \ubc88\uc9f8\ub85c \ub0ae\uc558\ub2e4. \ubb3c\uac00 \uc0c1\uc2b9\ub960\uc740 \ub178\ud0dc\uc6b0 \uc815\ubd80 \ub54c 7.4%\ub85c \uac00\uc7a5 \ub192\uc558\uace0 \uc804\ub450\ud658 \uc815\ubd80 6.1%, \uae40\uc601\uc0bc \uc815\ubd80 5.0% \ub4f1\uc774\uc5c8\ub2e4. \uc678\ud658\uc704\uae30\uc5d0 \ub530\ub978 \uae30\uc5c5 \ubd80\ub3c4 \ub4f1\uc758 \uc5ec\ud30c\ub85c \uacbd\uc81c\uc131\uc7a5\ub960\uacfc \uace0\uc6a9\ub960\uc740 \uc774\uc804 \uc815\uad8c\ub4e4\ubcf4\ub2e4 \ud06c\uac8c \ub0ae\uc558\ub2e4.", "sentence_answer": "\ud55c\uad6d\uc740\ud589\uacfc \ud1b5\uacc4\uccad \ub4f1\uc5d0 \ub530\ub974\uba74 \uae40\ub300\uc911\uc758 \uc9d1\uad8c \uc2dc\uae30\uc778 1998\ub144 ~ 2002\ub144 \uacbd\uc0c1\uc218\uc9c0 \ud751\uc790\ub294 906\uc5b5\ub2ec\ub7ec \uc99d\uac00\ud588\ub2e4.", "paragraph_id": "6576830-52-0", "paragraph_question": "question: \uae40\ub300\uc911 \uc9d1\uad8c \uc2dc\uae30\uc5d0 \ub298\uc5b4\ub09c \uacbd\uc0c1\uc218\uc9c0 \ud751\uc790\uc758 \uc561\uc218\ub294?, context: \ud55c\uad6d\uc740\ud589\uacfc \ud1b5\uacc4\uccad \ub4f1\uc5d0 \ub530\ub974\uba74 \uae40\ub300\uc911\uc758 \uc9d1\uad8c \uc2dc\uae30\uc778 1998\ub144 ~ 2002\ub144 \uacbd\uc0c1\uc218\uc9c0 \ud751\uc790\ub294 906\uc5b5\ub2ec\ub7ec \uc99d\uac00\ud588\ub2e4. \uc5f0\ud3c9\uade0 \uc99d\uac00\uc561\uc740 181\uc5b5 1\ucc9c 400\ub9cc\ub2ec\ub7ec\ub85c \uac00\uc7a5 \ub9ce\uc558\ub2e4. \uae40\ub300\uc911 \uc815\ubd80 \uc2dc\uc808\uc5d0\ub294 \uacbd\uc0c1\ud751\uc790\uc5d0 \ub530\ub978 \uc678\ud654 \uc720\uc785 \uc99d\uac00\ub85c \uc678\ud658\ubcf4\uc720\uc561\ub3c4 \ub9ce\uc774 \ub298\uc5b4\ub0ac\ub2e4. \uae40\uc601\uc0bc \uc815\ubd80 \ub9d0\uae30\uc778 1997\ub144\ub9d0 204\uc5b5 600\ub9cc\ub2ec\ub7ec\uc5d0 \ubd88\uacfc\ud558\ub358 \uc678\ud658\ubcf4\uc720\uc561\uc740 \uae40\ub300\uc911 \uc815\ubd80 \ub9d0\uae30\uc778 2002\ub144\ub9d0\uc5d0\ub294 1\ucc9c 214\uc5b5 1\ucc9c 300\ub9cc\ub2ec\ub7ec\ub85c \ub298\uc5b4\ub098\uba74\uc11c 5\ub144\uac04 1\ucc9c 10\uc5b5 700\ub9cc\ub2ec\ub7ec \uc99d\uac00\ud588\ub2e4. \uc774\ub85c \uc778\ud574 \uc138\uacc4 4\ub300 \uc678\ud658\ubcf4\uc720\uad6d\uc774 \ub418\uc5c8\ub2e4. \uc5f0\ud3c9\uade0 \uc99d\uac00\uc561\uc740 202\uc5b5 100\ub9cc\ub2ec\ub7ec\ub85c \ub178\ubb34\ud604 \uc815\ubd80\uc758 281\uc5b5 6\ucc9c 200\ub9cc\ub2ec\ub7ec\uc5d0 \uc774\uc5b4 \ub450 \ubc88\uc9f8\ub85c \ud070 \uaddc\ubaa8\ub2e4. \uae40\ub300\uc911 \uc815\ubd80 \ub54c\ub294 \ubb3c\uac00\ub3c4 \ube44\uad50\uc801 \uc548\uc815\uc801\uc778 \ubaa8\uc2b5\uc744 \ubcf4\uc600\ub2e4. \uae40\ub300\uc911 \uc815\ubd80 \uc2dc\uc808 \uc5f0\ud3c9\uade0 \uc18c\ube44\uc790\ubb3c\uac00 \uc0c1\uc2b9\ub960\uc740 \uc5f0\ud3c9\uade0 3.5%\ub85c \ub178\ubb34\ud604 \uc815\ubd80 \ub54c\uc758 3.0%\uc5d0 \uc774\uc5b4 \ub450 \ubc88\uc9f8\ub85c \ub0ae\uc558\ub2e4. \ubb3c\uac00 \uc0c1\uc2b9\ub960\uc740 \ub178\ud0dc\uc6b0 \uc815\ubd80 \ub54c 7.4%\ub85c \uac00\uc7a5 \ub192\uc558\uace0 \uc804\ub450\ud658 \uc815\ubd80 6.1%, \uae40\uc601\uc0bc \uc815\ubd80 5.0% \ub4f1\uc774\uc5c8\ub2e4. \uc678\ud658\uc704\uae30\uc5d0 \ub530\ub978 \uae30\uc5c5 \ubd80\ub3c4 \ub4f1\uc758 \uc5ec\ud30c\ub85c \uacbd\uc81c\uc131\uc7a5\ub960\uacfc \uace0\uc6a9\ub960\uc740 \uc774\uc804 \uc815\uad8c\ub4e4\ubcf4\ub2e4 \ud06c\uac8c \ub0ae\uc558\ub2e4."} {"question": "\ube44\ud2b8\ucf54\uc778\uc740 \uc624\ud508 \uc18c\uc2a4 \uc18c\ud504\ud2b8\uc6e8\uc5b4 \uc0c1\uc5d0 \uad6c\ud604\ub41c \ubb34\uc5c7\uc778\uac00?", "paragraph": "2016\ub144 \uc774\uc804\uae4c\uc9c0\ub9cc \ud574\ub3c4 \uc0ac\ud1a0\uc2dc \ub098\uce74\ubaa8\ud1a0\uc758 \uc815\uccb4\uac00 \ubc1d\ud600\uc9c4 \uc77c\uc740 \uc5c6\uc5c8\ub2e4. \uc774\ub984\uc740 \uc804\ud615\uc801\uc778 \uc77c\ubcf8\uc2dd \uc774\ub984\uc774\uc9c0\ub9cc, \ube44\ud2b8\ucf54\uc778\uc758 \uac1c\ubc1c\uc744 \ubc1c\ud45c\ud560 \ub2f9\uc2dc\uc5d0\ub294 \uc0ac\ud1a0\uc2dc\uc758 \uc774\ub984\uc774 \uc601\uc5b4\ub85c\ub9cc \uc4f0\uc5ec\uc788\uc5c8\uace0, \uc77c\ubcf8\uacc4 \uc774\ub984\uc5d0 \uac70\uc758 \ubc18\ub4dc\uc2dc \ubd99\uc5b4\uc788\ub294 \ud55c\uc790 \ud45c\uae30\ub97c \ubc1d\ud788\uc9c0 \uc54a\uc558\ub358 \uad00\uacc4\ub85c, \uc77c\ubcf8\uc778\uc73c\ub85c \uc704\uc7a5\ud55c \uc0ac\ub78c\uc77c \uac00\ub2a5\uc131\ub3c4 \uc81c\uae30\ub418\uc5c8\ub2e4. \uc2e4\uc81c\ub85c '\uc0ac\ud1a0\uc2dc \ub098\uce74\ubaa8\ud1a0'\ub77c\ub294 \uc774\ub984\uc744 \ubcf8\uba85\uc73c\ub85c \uc0ac\uc6a9\ud558\ub294 \ud55c \uc77c\ubcf8\uacc4 \ubbf8\uad6d\uc778\uc774 \uc7a0\uc2dc \uc0ac\ud1a0\uc2dc\uc758 \uc815\uccb4\ub77c\uace0 \ucd94\uc815\ub418\uc5c8\uc73c\ub098, \uc0ac\ud1a0\uc2dc\ub77c\uace0 \uc9c0\ubaa9\ub41c \ub2f9\uc0ac\uc790\ub294 \uc2a4\uc2a4\ub85c '\ube44\ud2b8\ucf54\uc778\uc758 \uac1c\ubc1c\uc790 \uc0ac\ud1a0\uc2dc'\uac00 \uc544\ub2c8\uba70, \ub3d9\uba85\uc774\uc778\uc784\uc744 \uc8fc\uc7a5\ud558\uc600\ub2e4. \ub610\ud55c \ud55c\ub54c\ub294 \uc77c\ubcf8\uc758 \uc218\ud559\uc790\ub85c \uad50\ud1a0 \ub300\ud559\uc758 \uad50\uc218\uc778 \ubaa8\uce58\uc988\ud0a4 \uc2e0\uc774\uce58\uac00 \uc0ac\ud1a0\uc2dc\ub77c\uace0 \uc9c0\ubaa9\ub418\uae30\ub3c4 \ud588\uc73c\ub098, \ubaa8\uce58\uc988\ud0a4 \uc2e0\uc774\uce58\ub294 \uc790\uc2e0\uc740 \uc0ac\ud1a0\uc2dc\uac00 \uc544\ub2c8\ub77c\uace0 \ud588\ub2e4. \uc2e4\uc81c\ub85c \uc0ac\ud1a0\uc2dc\ub294 \uc77c\ubcf8\uc5b4\ub97c \uc4f4 \uc801\uc774 \ub2e8 \ud55c\ubc88\ub3c4 \uc5c6\uc5c8\uace0, \ube44\ud2b8\ucf54\uc778 \uc790\uccb4\uac00 \uc624\ud508 \uc18c\uc2a4 \uc18c\ud504\ud2b8\uc6e8\uc5b4 \uc0c1\uc5d0 \uad6c\ud604\ub41c \uc804\uc790\ud654\ud3d0\ub77c\uc11c \uc2e4\uc81c \uac1c\ubc1c\uc790\uac00 \ub204\uad6c\uc778\uc9c0 \uc54c\uc544\ub0b4\uae30\ub294 \ub9e4\uc6b0 \ud798\ub4dc\ubbc0\ub85c, \uc77c\ubcf8\uc778\uc774 \uc544\ub2d0 \uac00\ub2a5\uc131\ub3c4 \uc81c\uae30\ub418\uc5c8\ub2e4. \ub610\ub294 '\uc0ac\ud1a0\uc2dc \ub098\uce74\ubaa8\ud1a0'\ub77c\ub294 \uc774\ub984 \uc790\uccb4\uac00 \ub3d9\ubd81\uc544\uc2dc\uc544\uc640 \ubd81\uc544\uba54\ub9ac\uce74 \uc9c0\uc5ed\uc758 \ub300\ud45c\uc801\uc778 \ub300\uae30\uc5c5\ub4e4\uc778 \ub098\uce74\ubbf8\uce58, \ubaa8\ud1a0\ub85c\ub77c, \uc0bc\uc131, \ub3c4\uc2dc\ubc14\uc758 4\uac1c \uae30\uc5c5\uc774 \uacf5\ub3d9\uc73c\ub85c \uc4f0\ub294 \uac00\uba85\uc774\ub77c\ub294 \uc124\ub3c4 \uc81c\uae30\ub41c \ubc14 \uc788\ub2e4.", "answer": "\uc804\uc790\ud654\ud3d0", "sentence": "\ube44\ud2b8\ucf54\uc778 \uc790\uccb4\uac00 \uc624\ud508 \uc18c\uc2a4 \uc18c\ud504\ud2b8\uc6e8\uc5b4 \uc0c1\uc5d0 \uad6c\ud604\ub41c \uc804\uc790\ud654\ud3d0 \ub77c\uc11c \uc2e4\uc81c \uac1c\ubc1c\uc790\uac00 \ub204\uad6c\uc778\uc9c0 \uc54c\uc544\ub0b4\uae30\ub294 \ub9e4\uc6b0 \ud798\ub4dc\ubbc0\ub85c, \uc77c\ubcf8\uc778\uc774 \uc544\ub2d0 \uac00\ub2a5\uc131\ub3c4 \uc81c\uae30\ub418\uc5c8\ub2e4.", "paragraph_sentence": "2016\ub144 \uc774\uc804\uae4c\uc9c0\ub9cc \ud574\ub3c4 \uc0ac\ud1a0\uc2dc \ub098\uce74\ubaa8\ud1a0\uc758 \uc815\uccb4\uac00 \ubc1d\ud600\uc9c4 \uc77c\uc740 \uc5c6\uc5c8\ub2e4. \uc774\ub984\uc740 \uc804\ud615\uc801\uc778 \uc77c\ubcf8\uc2dd \uc774\ub984\uc774\uc9c0\ub9cc, \ube44\ud2b8\ucf54\uc778\uc758 \uac1c\ubc1c\uc744 \ubc1c\ud45c\ud560 \ub2f9\uc2dc\uc5d0\ub294 \uc0ac\ud1a0\uc2dc\uc758 \uc774\ub984\uc774 \uc601\uc5b4\ub85c\ub9cc \uc4f0\uc5ec\uc788\uc5c8\uace0, \uc77c\ubcf8\uacc4 \uc774\ub984\uc5d0 \uac70\uc758 \ubc18\ub4dc\uc2dc \ubd99\uc5b4\uc788\ub294 \ud55c\uc790 \ud45c\uae30\ub97c \ubc1d\ud788\uc9c0 \uc54a\uc558\ub358 \uad00\uacc4\ub85c, \uc77c\ubcf8\uc778\uc73c\ub85c \uc704\uc7a5\ud55c \uc0ac\ub78c\uc77c \uac00\ub2a5\uc131\ub3c4 \uc81c\uae30\ub418\uc5c8\ub2e4. \uc2e4\uc81c\ub85c '\uc0ac\ud1a0\uc2dc \ub098\uce74\ubaa8\ud1a0'\ub77c\ub294 \uc774\ub984\uc744 \ubcf8\uba85\uc73c\ub85c \uc0ac\uc6a9\ud558\ub294 \ud55c \uc77c\ubcf8\uacc4 \ubbf8\uad6d\uc778\uc774 \uc7a0\uc2dc \uc0ac\ud1a0\uc2dc\uc758 \uc815\uccb4\ub77c\uace0 \ucd94\uc815\ub418\uc5c8\uc73c\ub098, \uc0ac\ud1a0\uc2dc\ub77c\uace0 \uc9c0\ubaa9\ub41c \ub2f9\uc0ac\uc790\ub294 \uc2a4\uc2a4\ub85c '\ube44\ud2b8\ucf54\uc778\uc758 \uac1c\ubc1c\uc790 \uc0ac\ud1a0\uc2dc'\uac00 \uc544\ub2c8\uba70, \ub3d9\uba85\uc774\uc778\uc784\uc744 \uc8fc\uc7a5\ud558\uc600\ub2e4. \ub610\ud55c \ud55c\ub54c\ub294 \uc77c\ubcf8\uc758 \uc218\ud559\uc790\ub85c \uad50\ud1a0 \ub300\ud559\uc758 \uad50\uc218\uc778 \ubaa8\uce58\uc988\ud0a4 \uc2e0\uc774\uce58\uac00 \uc0ac\ud1a0\uc2dc\ub77c\uace0 \uc9c0\ubaa9\ub418\uae30\ub3c4 \ud588\uc73c\ub098, \ubaa8\uce58\uc988\ud0a4 \uc2e0\uc774\uce58\ub294 \uc790\uc2e0\uc740 \uc0ac\ud1a0\uc2dc\uac00 \uc544\ub2c8\ub77c\uace0 \ud588\ub2e4. \uc2e4\uc81c\ub85c \uc0ac\ud1a0\uc2dc\ub294 \uc77c\ubcf8\uc5b4\ub97c \uc4f4 \uc801\uc774 \ub2e8 \ud55c\ubc88\ub3c4 \uc5c6\uc5c8\uace0, \ube44\ud2b8\ucf54\uc778 \uc790\uccb4\uac00 \uc624\ud508 \uc18c\uc2a4 \uc18c\ud504\ud2b8\uc6e8\uc5b4 \uc0c1\uc5d0 \uad6c\ud604\ub41c \uc804\uc790\ud654\ud3d0 \ub77c\uc11c \uc2e4\uc81c \uac1c\ubc1c\uc790\uac00 \ub204\uad6c\uc778\uc9c0 \uc54c\uc544\ub0b4\uae30\ub294 \ub9e4\uc6b0 \ud798\ub4dc\ubbc0\ub85c, \uc77c\ubcf8\uc778\uc774 \uc544\ub2d0 \uac00\ub2a5\uc131\ub3c4 \uc81c\uae30\ub418\uc5c8\ub2e4. \ub610\ub294 '\uc0ac\ud1a0\uc2dc \ub098\uce74\ubaa8\ud1a0'\ub77c\ub294 \uc774\ub984 \uc790\uccb4\uac00 \ub3d9\ubd81\uc544\uc2dc\uc544\uc640 \ubd81\uc544\uba54\ub9ac\uce74 \uc9c0\uc5ed\uc758 \ub300\ud45c\uc801\uc778 \ub300\uae30\uc5c5\ub4e4\uc778 \ub098\uce74\ubbf8\uce58, \ubaa8\ud1a0\ub85c\ub77c, \uc0bc\uc131, \ub3c4\uc2dc\ubc14\uc758 4\uac1c \uae30\uc5c5\uc774 \uacf5\ub3d9\uc73c\ub85c \uc4f0\ub294 \uac00\uba85\uc774\ub77c\ub294 \uc124\ub3c4 \uc81c\uae30\ub41c \ubc14 \uc788\ub2e4.", "paragraph_answer": "2016\ub144 \uc774\uc804\uae4c\uc9c0\ub9cc \ud574\ub3c4 \uc0ac\ud1a0\uc2dc \ub098\uce74\ubaa8\ud1a0\uc758 \uc815\uccb4\uac00 \ubc1d\ud600\uc9c4 \uc77c\uc740 \uc5c6\uc5c8\ub2e4. \uc774\ub984\uc740 \uc804\ud615\uc801\uc778 \uc77c\ubcf8\uc2dd \uc774\ub984\uc774\uc9c0\ub9cc, \ube44\ud2b8\ucf54\uc778\uc758 \uac1c\ubc1c\uc744 \ubc1c\ud45c\ud560 \ub2f9\uc2dc\uc5d0\ub294 \uc0ac\ud1a0\uc2dc\uc758 \uc774\ub984\uc774 \uc601\uc5b4\ub85c\ub9cc \uc4f0\uc5ec\uc788\uc5c8\uace0, \uc77c\ubcf8\uacc4 \uc774\ub984\uc5d0 \uac70\uc758 \ubc18\ub4dc\uc2dc \ubd99\uc5b4\uc788\ub294 \ud55c\uc790 \ud45c\uae30\ub97c \ubc1d\ud788\uc9c0 \uc54a\uc558\ub358 \uad00\uacc4\ub85c, \uc77c\ubcf8\uc778\uc73c\ub85c \uc704\uc7a5\ud55c \uc0ac\ub78c\uc77c \uac00\ub2a5\uc131\ub3c4 \uc81c\uae30\ub418\uc5c8\ub2e4. \uc2e4\uc81c\ub85c '\uc0ac\ud1a0\uc2dc \ub098\uce74\ubaa8\ud1a0'\ub77c\ub294 \uc774\ub984\uc744 \ubcf8\uba85\uc73c\ub85c \uc0ac\uc6a9\ud558\ub294 \ud55c \uc77c\ubcf8\uacc4 \ubbf8\uad6d\uc778\uc774 \uc7a0\uc2dc \uc0ac\ud1a0\uc2dc\uc758 \uc815\uccb4\ub77c\uace0 \ucd94\uc815\ub418\uc5c8\uc73c\ub098, \uc0ac\ud1a0\uc2dc\ub77c\uace0 \uc9c0\ubaa9\ub41c \ub2f9\uc0ac\uc790\ub294 \uc2a4\uc2a4\ub85c '\ube44\ud2b8\ucf54\uc778\uc758 \uac1c\ubc1c\uc790 \uc0ac\ud1a0\uc2dc'\uac00 \uc544\ub2c8\uba70, \ub3d9\uba85\uc774\uc778\uc784\uc744 \uc8fc\uc7a5\ud558\uc600\ub2e4. \ub610\ud55c \ud55c\ub54c\ub294 \uc77c\ubcf8\uc758 \uc218\ud559\uc790\ub85c \uad50\ud1a0 \ub300\ud559\uc758 \uad50\uc218\uc778 \ubaa8\uce58\uc988\ud0a4 \uc2e0\uc774\uce58\uac00 \uc0ac\ud1a0\uc2dc\ub77c\uace0 \uc9c0\ubaa9\ub418\uae30\ub3c4 \ud588\uc73c\ub098, \ubaa8\uce58\uc988\ud0a4 \uc2e0\uc774\uce58\ub294 \uc790\uc2e0\uc740 \uc0ac\ud1a0\uc2dc\uac00 \uc544\ub2c8\ub77c\uace0 \ud588\ub2e4. \uc2e4\uc81c\ub85c \uc0ac\ud1a0\uc2dc\ub294 \uc77c\ubcf8\uc5b4\ub97c \uc4f4 \uc801\uc774 \ub2e8 \ud55c\ubc88\ub3c4 \uc5c6\uc5c8\uace0, \ube44\ud2b8\ucf54\uc778 \uc790\uccb4\uac00 \uc624\ud508 \uc18c\uc2a4 \uc18c\ud504\ud2b8\uc6e8\uc5b4 \uc0c1\uc5d0 \uad6c\ud604\ub41c \uc804\uc790\ud654\ud3d0 \ub77c\uc11c \uc2e4\uc81c \uac1c\ubc1c\uc790\uac00 \ub204\uad6c\uc778\uc9c0 \uc54c\uc544\ub0b4\uae30\ub294 \ub9e4\uc6b0 \ud798\ub4dc\ubbc0\ub85c, \uc77c\ubcf8\uc778\uc774 \uc544\ub2d0 \uac00\ub2a5\uc131\ub3c4 \uc81c\uae30\ub418\uc5c8\ub2e4. \ub610\ub294 '\uc0ac\ud1a0\uc2dc \ub098\uce74\ubaa8\ud1a0'\ub77c\ub294 \uc774\ub984 \uc790\uccb4\uac00 \ub3d9\ubd81\uc544\uc2dc\uc544\uc640 \ubd81\uc544\uba54\ub9ac\uce74 \uc9c0\uc5ed\uc758 \ub300\ud45c\uc801\uc778 \ub300\uae30\uc5c5\ub4e4\uc778 \ub098\uce74\ubbf8\uce58, \ubaa8\ud1a0\ub85c\ub77c, \uc0bc\uc131, \ub3c4\uc2dc\ubc14\uc758 4\uac1c \uae30\uc5c5\uc774 \uacf5\ub3d9\uc73c\ub85c \uc4f0\ub294 \uac00\uba85\uc774\ub77c\ub294 \uc124\ub3c4 \uc81c\uae30\ub41c \ubc14 \uc788\ub2e4.", "sentence_answer": "\ube44\ud2b8\ucf54\uc778 \uc790\uccb4\uac00 \uc624\ud508 \uc18c\uc2a4 \uc18c\ud504\ud2b8\uc6e8\uc5b4 \uc0c1\uc5d0 \uad6c\ud604\ub41c \uc804\uc790\ud654\ud3d0 \ub77c\uc11c \uc2e4\uc81c \uac1c\ubc1c\uc790\uac00 \ub204\uad6c\uc778\uc9c0 \uc54c\uc544\ub0b4\uae30\ub294 \ub9e4\uc6b0 \ud798\ub4dc\ubbc0\ub85c, \uc77c\ubcf8\uc778\uc774 \uc544\ub2d0 \uac00\ub2a5\uc131\ub3c4 \uc81c\uae30\ub418\uc5c8\ub2e4.", "paragraph_id": "6435663-0-1", "paragraph_question": "question: \ube44\ud2b8\ucf54\uc778\uc740 \uc624\ud508 \uc18c\uc2a4 \uc18c\ud504\ud2b8\uc6e8\uc5b4 \uc0c1\uc5d0 \uad6c\ud604\ub41c \ubb34\uc5c7\uc778\uac00?, context: 2016\ub144 \uc774\uc804\uae4c\uc9c0\ub9cc \ud574\ub3c4 \uc0ac\ud1a0\uc2dc \ub098\uce74\ubaa8\ud1a0\uc758 \uc815\uccb4\uac00 \ubc1d\ud600\uc9c4 \uc77c\uc740 \uc5c6\uc5c8\ub2e4. \uc774\ub984\uc740 \uc804\ud615\uc801\uc778 \uc77c\ubcf8\uc2dd \uc774\ub984\uc774\uc9c0\ub9cc, \ube44\ud2b8\ucf54\uc778\uc758 \uac1c\ubc1c\uc744 \ubc1c\ud45c\ud560 \ub2f9\uc2dc\uc5d0\ub294 \uc0ac\ud1a0\uc2dc\uc758 \uc774\ub984\uc774 \uc601\uc5b4\ub85c\ub9cc \uc4f0\uc5ec\uc788\uc5c8\uace0, \uc77c\ubcf8\uacc4 \uc774\ub984\uc5d0 \uac70\uc758 \ubc18\ub4dc\uc2dc \ubd99\uc5b4\uc788\ub294 \ud55c\uc790 \ud45c\uae30\ub97c \ubc1d\ud788\uc9c0 \uc54a\uc558\ub358 \uad00\uacc4\ub85c, \uc77c\ubcf8\uc778\uc73c\ub85c \uc704\uc7a5\ud55c \uc0ac\ub78c\uc77c \uac00\ub2a5\uc131\ub3c4 \uc81c\uae30\ub418\uc5c8\ub2e4. \uc2e4\uc81c\ub85c '\uc0ac\ud1a0\uc2dc \ub098\uce74\ubaa8\ud1a0'\ub77c\ub294 \uc774\ub984\uc744 \ubcf8\uba85\uc73c\ub85c \uc0ac\uc6a9\ud558\ub294 \ud55c \uc77c\ubcf8\uacc4 \ubbf8\uad6d\uc778\uc774 \uc7a0\uc2dc \uc0ac\ud1a0\uc2dc\uc758 \uc815\uccb4\ub77c\uace0 \ucd94\uc815\ub418\uc5c8\uc73c\ub098, \uc0ac\ud1a0\uc2dc\ub77c\uace0 \uc9c0\ubaa9\ub41c \ub2f9\uc0ac\uc790\ub294 \uc2a4\uc2a4\ub85c '\ube44\ud2b8\ucf54\uc778\uc758 \uac1c\ubc1c\uc790 \uc0ac\ud1a0\uc2dc'\uac00 \uc544\ub2c8\uba70, \ub3d9\uba85\uc774\uc778\uc784\uc744 \uc8fc\uc7a5\ud558\uc600\ub2e4. \ub610\ud55c \ud55c\ub54c\ub294 \uc77c\ubcf8\uc758 \uc218\ud559\uc790\ub85c \uad50\ud1a0 \ub300\ud559\uc758 \uad50\uc218\uc778 \ubaa8\uce58\uc988\ud0a4 \uc2e0\uc774\uce58\uac00 \uc0ac\ud1a0\uc2dc\ub77c\uace0 \uc9c0\ubaa9\ub418\uae30\ub3c4 \ud588\uc73c\ub098, \ubaa8\uce58\uc988\ud0a4 \uc2e0\uc774\uce58\ub294 \uc790\uc2e0\uc740 \uc0ac\ud1a0\uc2dc\uac00 \uc544\ub2c8\ub77c\uace0 \ud588\ub2e4. \uc2e4\uc81c\ub85c \uc0ac\ud1a0\uc2dc\ub294 \uc77c\ubcf8\uc5b4\ub97c \uc4f4 \uc801\uc774 \ub2e8 \ud55c\ubc88\ub3c4 \uc5c6\uc5c8\uace0, \ube44\ud2b8\ucf54\uc778 \uc790\uccb4\uac00 \uc624\ud508 \uc18c\uc2a4 \uc18c\ud504\ud2b8\uc6e8\uc5b4 \uc0c1\uc5d0 \uad6c\ud604\ub41c \uc804\uc790\ud654\ud3d0\ub77c\uc11c \uc2e4\uc81c \uac1c\ubc1c\uc790\uac00 \ub204\uad6c\uc778\uc9c0 \uc54c\uc544\ub0b4\uae30\ub294 \ub9e4\uc6b0 \ud798\ub4dc\ubbc0\ub85c, \uc77c\ubcf8\uc778\uc774 \uc544\ub2d0 \uac00\ub2a5\uc131\ub3c4 \uc81c\uae30\ub418\uc5c8\ub2e4. \ub610\ub294 '\uc0ac\ud1a0\uc2dc \ub098\uce74\ubaa8\ud1a0'\ub77c\ub294 \uc774\ub984 \uc790\uccb4\uac00 \ub3d9\ubd81\uc544\uc2dc\uc544\uc640 \ubd81\uc544\uba54\ub9ac\uce74 \uc9c0\uc5ed\uc758 \ub300\ud45c\uc801\uc778 \ub300\uae30\uc5c5\ub4e4\uc778 \ub098\uce74\ubbf8\uce58, \ubaa8\ud1a0\ub85c\ub77c, \uc0bc\uc131, \ub3c4\uc2dc\ubc14\uc758 4\uac1c \uae30\uc5c5\uc774 \uacf5\ub3d9\uc73c\ub85c \uc4f0\ub294 \uac00\uba85\uc774\ub77c\ub294 \uc124\ub3c4 \uc81c\uae30\ub41c \ubc14 \uc788\ub2e4."} {"question": "2007\ub144 \uacbd\uc120 \ub2f9\uc2dc \ubc15\uadfc\ud61c \uc9c4\uc601\uc5d0\uc11c \uc774\uba85\ubc15\uc774 \uc804\uacfc \uba87\ubc94\uc774\ub77c\uace0 \uc8fc\uc7a5\ud558\uc600\ub098\uc694?", "paragraph": "2007\ub144 \ub300\uc120 \uc774\uc804\uc5d0 \uc788\uc5c8\ub358 \uacbd\uc120\uc5d0\uc11c \ubc15\uadfc\ud61c\ub294 \uc774\uba85\ubc15\uacfc \uacbd\uc7c1\ud558\uba70 \uac08\ub4f1\uc744 \uacaa\uc5b4\uc654\ub2e4. \uacbd\uc120 \ub2f9\uc2dc \ubc15\uadfc\ud61c \uc9c4\uc601\uc740 \uc774\uba85\ubc15\uc774 \uc804\uacfc 14\ubc94\uc774\ub77c\ub294 \uc8fc\uc7a5\uc744 \ud558\uac70\ub098, \uc704\uc7a5\uc804\uc785 \ubb38\uc81c\ub3c4 \ud30c\ud5e4\uccd0 \uc774\uba85\ubc15\uc740 \uc774 \ub54c\ubb38\uc5d0 \uad6d\ubbfc\ub4e4\uc5d0\uac8c \uc0ac\uacfc\ub97c \ud574\uc57c \ud588\ub2e4. \uc774\ub7ec\ud55c \uac08\ub4f1\uc740 \uc774\uba85\ubc15\uc774 \ub300\ud1b5\ub839\uc5d0 \ub2f9\uc120\ub41c \uc774\ud6c4\uc5d0\ub3c4 \uc9c0\uc18d\ub418\uc5b4 \uc654\ub2e4. \uce5c\ubc15\uacc4 \uc758\uc6d0\ub4e4\uc758 \ud55c\ub098\ub77c\ub2f9 \uacf5\ucc9c \ud0c8\ub77d\uc5d0 \ub300\ud558\uc5ec \ubc15\uadfc\ud61c\ub294 \"\uad6d\ubbfc\ub3c4 \uc18d\uc558\uace0 \ub098\ub3c4 \uc18d\uc558\ub2e4\"\ub77c\ub294 \ub9d0\uc744 \ud558\uc5ec \ud30c\uc7a5\uc744 \uc77c\uc73c\ucf30\uc73c\uba70, 2008\ub144 \ubbf8\uad6d\uc0b0 \uc1e0\uace0\uae30 \uc218\uc785 \ud611\uc0c1\uacfc \uad00\ub828\ud55c \ucd1b\ubd88\ud30c\ub3d9\ub54c\ub294 \uc7ac\ud611\uc0c1\uc744 \uc694\uad6c\ud588\uace0, 2009\ub144 \uc6a9\uc0b0 \ucc38\uc0ac \ub54c\ub294 \uacbd\ucc30\uc758 \uac15\uacbd\uc9c4\uc555\uc744 \ube44\ud310\ud558\uc600\uc73c\uba70, \ud55c\ub098\ub77c\ub2f9\uc5d0 \uc758\ud574 \ubbf8\ub514\uc5b4\ubc95 \uac1c\uc815 \ucd94\uc9c4 \ub2f9\uc2dc\uc5d0\ub3c4 \ubc15\uadfc\ud61c\uac00 \"\uac1c\uc815\uc548\uc5d0 \ubc18\ub300\ud45c\ub97c \ub358\uc9c8 \uac83\"\uc774\ub77c\ub294 \ub9d0\uc744 \ub358\uc9c0\ub294 \ub4f1 \uc815\ucc45 \ubb38\uc81c\uc5d0 \ub300\ud55c \uc2dc\uac01\ucc28\uc774\uc640 \uac1c\uc778\uc801\uc778 \uac08\ub4f1\ub4e4\uc774 \ubcf5\ud569\uc801\uc73c\ub85c \ub0a8\uc544\uc788\uc5c8\ub2e4.", "answer": "\uc804\uacfc 14\ubc94", "sentence": "\uacbd\uc120 \ub2f9\uc2dc \ubc15\uadfc\ud61c \uc9c4\uc601\uc740 \uc774\uba85\ubc15\uc774 \uc804\uacfc 14\ubc94 \uc774\ub77c\ub294 \uc8fc\uc7a5\uc744 \ud558\uac70\ub098, \uc704\uc7a5\uc804\uc785 \ubb38\uc81c\ub3c4 \ud30c\ud5e4\uccd0 \uc774\uba85\ubc15\uc740 \uc774 \ub54c\ubb38\uc5d0 \uad6d\ubbfc\ub4e4\uc5d0\uac8c \uc0ac\uacfc\ub97c \ud574\uc57c \ud588\ub2e4.", "paragraph_sentence": "2007\ub144 \ub300\uc120 \uc774\uc804\uc5d0 \uc788\uc5c8\ub358 \uacbd\uc120\uc5d0\uc11c \ubc15\uadfc\ud61c\ub294 \uc774\uba85\ubc15\uacfc \uacbd\uc7c1\ud558\uba70 \uac08\ub4f1\uc744 \uacaa\uc5b4\uc654\ub2e4. \uacbd\uc120 \ub2f9\uc2dc \ubc15\uadfc\ud61c \uc9c4\uc601\uc740 \uc774\uba85\ubc15\uc774 \uc804\uacfc 14\ubc94 \uc774\ub77c\ub294 \uc8fc\uc7a5\uc744 \ud558\uac70\ub098, \uc704\uc7a5\uc804\uc785 \ubb38\uc81c\ub3c4 \ud30c\ud5e4\uccd0 \uc774\uba85\ubc15\uc740 \uc774 \ub54c\ubb38\uc5d0 \uad6d\ubbfc\ub4e4\uc5d0\uac8c \uc0ac\uacfc\ub97c \ud574\uc57c \ud588\ub2e4. \uc774\ub7ec\ud55c \uac08\ub4f1\uc740 \uc774\uba85\ubc15\uc774 \ub300\ud1b5\ub839\uc5d0 \ub2f9\uc120\ub41c \uc774\ud6c4\uc5d0\ub3c4 \uc9c0\uc18d\ub418\uc5b4 \uc654\ub2e4. \uce5c\ubc15\uacc4 \uc758\uc6d0\ub4e4\uc758 \ud55c\ub098\ub77c\ub2f9 \uacf5\ucc9c \ud0c8\ub77d\uc5d0 \ub300\ud558\uc5ec \ubc15\uadfc\ud61c\ub294 \"\uad6d\ubbfc\ub3c4 \uc18d\uc558\uace0 \ub098\ub3c4 \uc18d\uc558\ub2e4\"\ub77c\ub294 \ub9d0\uc744 \ud558\uc5ec \ud30c\uc7a5\uc744 \uc77c\uc73c\ucf30\uc73c\uba70, 2008\ub144 \ubbf8\uad6d\uc0b0 \uc1e0\uace0\uae30 \uc218\uc785 \ud611\uc0c1\uacfc \uad00\ub828\ud55c \ucd1b\ubd88\ud30c\ub3d9\ub54c\ub294 \uc7ac\ud611\uc0c1\uc744 \uc694\uad6c\ud588\uace0, 2009\ub144 \uc6a9\uc0b0 \ucc38\uc0ac \ub54c\ub294 \uacbd\ucc30\uc758 \uac15\uacbd\uc9c4\uc555\uc744 \ube44\ud310\ud558\uc600\uc73c\uba70, \ud55c\ub098\ub77c\ub2f9\uc5d0 \uc758\ud574 \ubbf8\ub514\uc5b4\ubc95 \uac1c\uc815 \ucd94\uc9c4 \ub2f9\uc2dc\uc5d0\ub3c4 \ubc15\uadfc\ud61c\uac00 \"\uac1c\uc815\uc548\uc5d0 \ubc18\ub300\ud45c\ub97c \ub358\uc9c8 \uac83\"\uc774\ub77c\ub294 \ub9d0\uc744 \ub358\uc9c0\ub294 \ub4f1 \uc815\ucc45 \ubb38\uc81c\uc5d0 \ub300\ud55c \uc2dc\uac01\ucc28\uc774\uc640 \uac1c\uc778\uc801\uc778 \uac08\ub4f1\ub4e4\uc774 \ubcf5\ud569\uc801\uc73c\ub85c \ub0a8\uc544\uc788\uc5c8\ub2e4.", "paragraph_answer": "2007\ub144 \ub300\uc120 \uc774\uc804\uc5d0 \uc788\uc5c8\ub358 \uacbd\uc120\uc5d0\uc11c \ubc15\uadfc\ud61c\ub294 \uc774\uba85\ubc15\uacfc \uacbd\uc7c1\ud558\uba70 \uac08\ub4f1\uc744 \uacaa\uc5b4\uc654\ub2e4. \uacbd\uc120 \ub2f9\uc2dc \ubc15\uadfc\ud61c \uc9c4\uc601\uc740 \uc774\uba85\ubc15\uc774 \uc804\uacfc 14\ubc94 \uc774\ub77c\ub294 \uc8fc\uc7a5\uc744 \ud558\uac70\ub098, \uc704\uc7a5\uc804\uc785 \ubb38\uc81c\ub3c4 \ud30c\ud5e4\uccd0 \uc774\uba85\ubc15\uc740 \uc774 \ub54c\ubb38\uc5d0 \uad6d\ubbfc\ub4e4\uc5d0\uac8c \uc0ac\uacfc\ub97c \ud574\uc57c \ud588\ub2e4. \uc774\ub7ec\ud55c \uac08\ub4f1\uc740 \uc774\uba85\ubc15\uc774 \ub300\ud1b5\ub839\uc5d0 \ub2f9\uc120\ub41c \uc774\ud6c4\uc5d0\ub3c4 \uc9c0\uc18d\ub418\uc5b4 \uc654\ub2e4. \uce5c\ubc15\uacc4 \uc758\uc6d0\ub4e4\uc758 \ud55c\ub098\ub77c\ub2f9 \uacf5\ucc9c \ud0c8\ub77d\uc5d0 \ub300\ud558\uc5ec \ubc15\uadfc\ud61c\ub294 \"\uad6d\ubbfc\ub3c4 \uc18d\uc558\uace0 \ub098\ub3c4 \uc18d\uc558\ub2e4\"\ub77c\ub294 \ub9d0\uc744 \ud558\uc5ec \ud30c\uc7a5\uc744 \uc77c\uc73c\ucf30\uc73c\uba70, 2008\ub144 \ubbf8\uad6d\uc0b0 \uc1e0\uace0\uae30 \uc218\uc785 \ud611\uc0c1\uacfc \uad00\ub828\ud55c \ucd1b\ubd88\ud30c\ub3d9\ub54c\ub294 \uc7ac\ud611\uc0c1\uc744 \uc694\uad6c\ud588\uace0, 2009\ub144 \uc6a9\uc0b0 \ucc38\uc0ac \ub54c\ub294 \uacbd\ucc30\uc758 \uac15\uacbd\uc9c4\uc555\uc744 \ube44\ud310\ud558\uc600\uc73c\uba70, \ud55c\ub098\ub77c\ub2f9\uc5d0 \uc758\ud574 \ubbf8\ub514\uc5b4\ubc95 \uac1c\uc815 \ucd94\uc9c4 \ub2f9\uc2dc\uc5d0\ub3c4 \ubc15\uadfc\ud61c\uac00 \"\uac1c\uc815\uc548\uc5d0 \ubc18\ub300\ud45c\ub97c \ub358\uc9c8 \uac83\"\uc774\ub77c\ub294 \ub9d0\uc744 \ub358\uc9c0\ub294 \ub4f1 \uc815\ucc45 \ubb38\uc81c\uc5d0 \ub300\ud55c \uc2dc\uac01\ucc28\uc774\uc640 \uac1c\uc778\uc801\uc778 \uac08\ub4f1\ub4e4\uc774 \ubcf5\ud569\uc801\uc73c\ub85c \ub0a8\uc544\uc788\uc5c8\ub2e4.", "sentence_answer": "\uacbd\uc120 \ub2f9\uc2dc \ubc15\uadfc\ud61c \uc9c4\uc601\uc740 \uc774\uba85\ubc15\uc774 \uc804\uacfc 14\ubc94 \uc774\ub77c\ub294 \uc8fc\uc7a5\uc744 \ud558\uac70\ub098, \uc704\uc7a5\uc804\uc785 \ubb38\uc81c\ub3c4 \ud30c\ud5e4\uccd0 \uc774\uba85\ubc15\uc740 \uc774 \ub54c\ubb38\uc5d0 \uad6d\ubbfc\ub4e4\uc5d0\uac8c \uc0ac\uacfc\ub97c \ud574\uc57c \ud588\ub2e4.", "paragraph_id": "6506520-38-0", "paragraph_question": "question: 2007\ub144 \uacbd\uc120 \ub2f9\uc2dc \ubc15\uadfc\ud61c \uc9c4\uc601\uc5d0\uc11c \uc774\uba85\ubc15\uc774 \uc804\uacfc \uba87\ubc94\uc774\ub77c\uace0 \uc8fc\uc7a5\ud558\uc600\ub098\uc694?, context: 2007\ub144 \ub300\uc120 \uc774\uc804\uc5d0 \uc788\uc5c8\ub358 \uacbd\uc120\uc5d0\uc11c \ubc15\uadfc\ud61c\ub294 \uc774\uba85\ubc15\uacfc \uacbd\uc7c1\ud558\uba70 \uac08\ub4f1\uc744 \uacaa\uc5b4\uc654\ub2e4. \uacbd\uc120 \ub2f9\uc2dc \ubc15\uadfc\ud61c \uc9c4\uc601\uc740 \uc774\uba85\ubc15\uc774 \uc804\uacfc 14\ubc94\uc774\ub77c\ub294 \uc8fc\uc7a5\uc744 \ud558\uac70\ub098, \uc704\uc7a5\uc804\uc785 \ubb38\uc81c\ub3c4 \ud30c\ud5e4\uccd0 \uc774\uba85\ubc15\uc740 \uc774 \ub54c\ubb38\uc5d0 \uad6d\ubbfc\ub4e4\uc5d0\uac8c \uc0ac\uacfc\ub97c \ud574\uc57c \ud588\ub2e4. \uc774\ub7ec\ud55c \uac08\ub4f1\uc740 \uc774\uba85\ubc15\uc774 \ub300\ud1b5\ub839\uc5d0 \ub2f9\uc120\ub41c \uc774\ud6c4\uc5d0\ub3c4 \uc9c0\uc18d\ub418\uc5b4 \uc654\ub2e4. \uce5c\ubc15\uacc4 \uc758\uc6d0\ub4e4\uc758 \ud55c\ub098\ub77c\ub2f9 \uacf5\ucc9c \ud0c8\ub77d\uc5d0 \ub300\ud558\uc5ec \ubc15\uadfc\ud61c\ub294 \"\uad6d\ubbfc\ub3c4 \uc18d\uc558\uace0 \ub098\ub3c4 \uc18d\uc558\ub2e4\"\ub77c\ub294 \ub9d0\uc744 \ud558\uc5ec \ud30c\uc7a5\uc744 \uc77c\uc73c\ucf30\uc73c\uba70, 2008\ub144 \ubbf8\uad6d\uc0b0 \uc1e0\uace0\uae30 \uc218\uc785 \ud611\uc0c1\uacfc \uad00\ub828\ud55c \ucd1b\ubd88\ud30c\ub3d9\ub54c\ub294 \uc7ac\ud611\uc0c1\uc744 \uc694\uad6c\ud588\uace0, 2009\ub144 \uc6a9\uc0b0 \ucc38\uc0ac \ub54c\ub294 \uacbd\ucc30\uc758 \uac15\uacbd\uc9c4\uc555\uc744 \ube44\ud310\ud558\uc600\uc73c\uba70, \ud55c\ub098\ub77c\ub2f9\uc5d0 \uc758\ud574 \ubbf8\ub514\uc5b4\ubc95 \uac1c\uc815 \ucd94\uc9c4 \ub2f9\uc2dc\uc5d0\ub3c4 \ubc15\uadfc\ud61c\uac00 \"\uac1c\uc815\uc548\uc5d0 \ubc18\ub300\ud45c\ub97c \ub358\uc9c8 \uac83\"\uc774\ub77c\ub294 \ub9d0\uc744 \ub358\uc9c0\ub294 \ub4f1 \uc815\ucc45 \ubb38\uc81c\uc5d0 \ub300\ud55c \uc2dc\uac01\ucc28\uc774\uc640 \uac1c\uc778\uc801\uc778 \uac08\ub4f1\ub4e4\uc774 \ubcf5\ud569\uc801\uc73c\ub85c \ub0a8\uc544\uc788\uc5c8\ub2e4."} {"question": "\uce6b\uc18c \uacf5\uc7a5\uc740 \uc5b4\ub514\uc5d0 \ub354\ub7ec\uc6b4 \ud3d0\uc218\ub97c \ud758\ub824\ubcf4\ub0c8\ub098\uc694?", "paragraph": "\uce6b\uc18c \uacf5\uc7a5\uc774 1908\ub144\uc5d0 \uac1c\uc7a5\ud55c \uc774\ud6c4\ub85c \ub354\ub7ec\uc6b4 \ud3d0\uc218\ub294 \ubbf8\ub098\ub9c8\ud0c0 \uc8fc\uc704\uc758 \uc5b4\uc5c5\uc5d0 \uc190\uc2e4\uc744 \uc785\ud600\uc654\uc5c8\ub2e4. '\ubbf8\ub098\ub9c8\ud0c0 \uc5b4\uc5c5 \uacf5\ub3d9\uccb4'\ub294 \ud68c\uc0ac\ub85c\ubd80\ud130 1926\ub144\uacfc 1943\ub144\uc5d0 \uc791\uc740 \ubcf4\uc0c1\uc744 \ubc1b\uc558\uc5c8\ub2e4. \ud558\uc9c0\ub9cc \ubbf8\ub098\ub9c8\ud0c0 \ubcd1 \ubc1c\uc0dd \ud6c4\uc5d0 \uc5b4\ud669\uc740 \ub208\uc5d0 \ub744\uac8c \ubcc0\ud588\ub2e4. \uc5b4\ud68d\ub7c9\uc740 1953\ub144\uacfc 1957\ub144 \uc0ac\uc774 91%\uac00 \uac10\uc18c\ud588\ub2e4. \uad6c\ub9c8\ubaa8\ud1a0 \uc9c0\ubc29 \ub2f9\uad6d\uc740 \uc624\uc5fc\ub41c \ubbf8\ub098\ub9c8\ud0c0 \ub9cc\uc5d0\uc11c \uc7a1\ud78c \ubb3c\uace0\uae30\uc758 \ud310\ub9e4\ub97c \ubd80\ubd84\uc801\uc73c\ub85c \uae08\uc9c0\ud558\uc600\ub2e4. \uc5b4\uc5c5 \uacf5\ub3d9\uccb4\ub294 \uce6b\uc18c\uc5d0 \ub300\ud574 \ud56d\uc758\ud588\uace0 8\uc6d4 6\uc77c\uacfc 8\uc6d4 12\uc77c\uc5d0 \ubcf4\uc0c1\uc744 \uac15\ud558\uac8c \uc694\uad6c\ud558\uc600\ub2e4. \ub450 \uc785\uc7a5\uc744 \uc911\uc7ac\ud558\uae30 \uc704\ud558\uc5ec \uc704\uc6d0\ud68c\uac00 \ubbf8\ub098\ub9c8\ud0c0 \uc2dc\uc7a5\uc5d0 \uc758\ud574 \uc18c\uc9d1\ub418\uc5c8\uc9c0\ub9cc \uc704\uc6d0\ud68c\ub294 \ud68c\uc0ac\uc758 \uc785\uc7a5\uc5d0\uc11c \ubd80\ub2f9\ud558\uac8c \ud3b8\uc131\ub418\uc5c8\ub2e4. 8\uc6d4 29\uc77c, \uc5b4\uc5c5 \uacf5\ub3d9\uccb4\ub294 \uc704\uc6d0\ud68c\uc758 \uc911\uc7ac \uc81c\uc548\uc5d0 \ub3d9\uc758\ud558\uc600\uace0 \"\uc2dc\ubbfc\ub4e4\uc758 \uace0\ud1b5\uc744 \ub05d\ub0b4\uae30 \uc704\ud558\uc5ec, \uc6b0\ub9ac\uac00 \ub208\ubb3c\uc744 \ucc38\uace0 \ubc1b\uc544\ub4e4\uc774\uaca0\ub2e4\"\ub77c\uace0 \ub9d0\ud588\ub2e4. \ud68c\uc0ac\ub294 2\ucc9c\ub9cc \uc5d4\uc744 \uacf5\ub3d9\uccb4\uc5d0 \uc9c0\ubd88\ud558\uc600\uace0 \uc5b4\uc5c5\uc758 \ud68c\ubcf5\uc744 \uc99d\uc9c4\ud558\uae30 \uc704\ud558\uc5ec 1,500\ub9cc \uc5d4\uc758 \uae30\uae08\uc744 \ub9c8\ub828\ud588\ub2e4.", "answer": "\ubbf8\ub098\ub9c8\ud0c0", "sentence": "\uce6b\uc18c \uacf5\uc7a5\uc774 1908\ub144\uc5d0 \uac1c\uc7a5\ud55c \uc774\ud6c4\ub85c \ub354\ub7ec\uc6b4 \ud3d0\uc218\ub294 \ubbf8\ub098\ub9c8\ud0c0 \uc8fc\uc704\uc758 \uc5b4\uc5c5\uc5d0 \uc190\uc2e4\uc744 \uc785\ud600\uc654\uc5c8\ub2e4.", "paragraph_sentence": " \uce6b\uc18c \uacf5\uc7a5\uc774 1908\ub144\uc5d0 \uac1c\uc7a5\ud55c \uc774\ud6c4\ub85c \ub354\ub7ec\uc6b4 \ud3d0\uc218\ub294 \ubbf8\ub098\ub9c8\ud0c0 \uc8fc\uc704\uc758 \uc5b4\uc5c5\uc5d0 \uc190\uc2e4\uc744 \uc785\ud600\uc654\uc5c8\ub2e4. '\ubbf8\ub098\ub9c8\ud0c0 \uc5b4\uc5c5 \uacf5\ub3d9\uccb4'\ub294 \ud68c\uc0ac\ub85c\ubd80\ud130 1926\ub144\uacfc 1943\ub144\uc5d0 \uc791\uc740 \ubcf4\uc0c1\uc744 \ubc1b\uc558\uc5c8\ub2e4. \ud558\uc9c0\ub9cc \ubbf8\ub098\ub9c8\ud0c0 \ubcd1 \ubc1c\uc0dd \ud6c4\uc5d0 \uc5b4\ud669\uc740 \ub208\uc5d0 \ub744\uac8c \ubcc0\ud588\ub2e4. \uc5b4\ud68d\ub7c9\uc740 1953\ub144\uacfc 1957\ub144 \uc0ac\uc774 91%\uac00 \uac10\uc18c\ud588\ub2e4. \uad6c\ub9c8\ubaa8\ud1a0 \uc9c0\ubc29 \ub2f9\uad6d\uc740 \uc624\uc5fc\ub41c \ubbf8\ub098\ub9c8\ud0c0 \ub9cc\uc5d0\uc11c \uc7a1\ud78c \ubb3c\uace0\uae30\uc758 \ud310\ub9e4\ub97c \ubd80\ubd84\uc801\uc73c\ub85c \uae08\uc9c0\ud558\uc600\ub2e4. \uc5b4\uc5c5 \uacf5\ub3d9\uccb4\ub294 \uce6b\uc18c\uc5d0 \ub300\ud574 \ud56d\uc758\ud588\uace0 8\uc6d4 6\uc77c\uacfc 8\uc6d4 12\uc77c\uc5d0 \ubcf4\uc0c1\uc744 \uac15\ud558\uac8c \uc694\uad6c\ud558\uc600\ub2e4. \ub450 \uc785\uc7a5\uc744 \uc911\uc7ac\ud558\uae30 \uc704\ud558\uc5ec \uc704\uc6d0\ud68c\uac00 \ubbf8\ub098\ub9c8\ud0c0 \uc2dc\uc7a5\uc5d0 \uc758\ud574 \uc18c\uc9d1\ub418\uc5c8\uc9c0\ub9cc \uc704\uc6d0\ud68c\ub294 \ud68c\uc0ac\uc758 \uc785\uc7a5\uc5d0\uc11c \ubd80\ub2f9\ud558\uac8c \ud3b8\uc131\ub418\uc5c8\ub2e4. 8\uc6d4 29\uc77c, \uc5b4\uc5c5 \uacf5\ub3d9\uccb4\ub294 \uc704\uc6d0\ud68c\uc758 \uc911\uc7ac \uc81c\uc548\uc5d0 \ub3d9\uc758\ud558\uc600\uace0 \"\uc2dc\ubbfc\ub4e4\uc758 \uace0\ud1b5\uc744 \ub05d\ub0b4\uae30 \uc704\ud558\uc5ec, \uc6b0\ub9ac\uac00 \ub208\ubb3c\uc744 \ucc38\uace0 \ubc1b\uc544\ub4e4\uc774\uaca0\ub2e4\"\ub77c\uace0 \ub9d0\ud588\ub2e4. \ud68c\uc0ac\ub294 2\ucc9c\ub9cc \uc5d4\uc744 \uacf5\ub3d9\uccb4\uc5d0 \uc9c0\ubd88\ud558\uc600\uace0 \uc5b4\uc5c5\uc758 \ud68c\ubcf5\uc744 \uc99d\uc9c4\ud558\uae30 \uc704\ud558\uc5ec 1,500\ub9cc \uc5d4\uc758 \uae30\uae08\uc744 \ub9c8\ub828\ud588\ub2e4.", "paragraph_answer": "\uce6b\uc18c \uacf5\uc7a5\uc774 1908\ub144\uc5d0 \uac1c\uc7a5\ud55c \uc774\ud6c4\ub85c \ub354\ub7ec\uc6b4 \ud3d0\uc218\ub294 \ubbf8\ub098\ub9c8\ud0c0 \uc8fc\uc704\uc758 \uc5b4\uc5c5\uc5d0 \uc190\uc2e4\uc744 \uc785\ud600\uc654\uc5c8\ub2e4. '\ubbf8\ub098\ub9c8\ud0c0 \uc5b4\uc5c5 \uacf5\ub3d9\uccb4'\ub294 \ud68c\uc0ac\ub85c\ubd80\ud130 1926\ub144\uacfc 1943\ub144\uc5d0 \uc791\uc740 \ubcf4\uc0c1\uc744 \ubc1b\uc558\uc5c8\ub2e4. \ud558\uc9c0\ub9cc \ubbf8\ub098\ub9c8\ud0c0 \ubcd1 \ubc1c\uc0dd \ud6c4\uc5d0 \uc5b4\ud669\uc740 \ub208\uc5d0 \ub744\uac8c \ubcc0\ud588\ub2e4. \uc5b4\ud68d\ub7c9\uc740 1953\ub144\uacfc 1957\ub144 \uc0ac\uc774 91%\uac00 \uac10\uc18c\ud588\ub2e4. \uad6c\ub9c8\ubaa8\ud1a0 \uc9c0\ubc29 \ub2f9\uad6d\uc740 \uc624\uc5fc\ub41c \ubbf8\ub098\ub9c8\ud0c0 \ub9cc\uc5d0\uc11c \uc7a1\ud78c \ubb3c\uace0\uae30\uc758 \ud310\ub9e4\ub97c \ubd80\ubd84\uc801\uc73c\ub85c \uae08\uc9c0\ud558\uc600\ub2e4. \uc5b4\uc5c5 \uacf5\ub3d9\uccb4\ub294 \uce6b\uc18c\uc5d0 \ub300\ud574 \ud56d\uc758\ud588\uace0 8\uc6d4 6\uc77c\uacfc 8\uc6d4 12\uc77c\uc5d0 \ubcf4\uc0c1\uc744 \uac15\ud558\uac8c \uc694\uad6c\ud558\uc600\ub2e4. \ub450 \uc785\uc7a5\uc744 \uc911\uc7ac\ud558\uae30 \uc704\ud558\uc5ec \uc704\uc6d0\ud68c\uac00 \ubbf8\ub098\ub9c8\ud0c0 \uc2dc\uc7a5\uc5d0 \uc758\ud574 \uc18c\uc9d1\ub418\uc5c8\uc9c0\ub9cc \uc704\uc6d0\ud68c\ub294 \ud68c\uc0ac\uc758 \uc785\uc7a5\uc5d0\uc11c \ubd80\ub2f9\ud558\uac8c \ud3b8\uc131\ub418\uc5c8\ub2e4. 8\uc6d4 29\uc77c, \uc5b4\uc5c5 \uacf5\ub3d9\uccb4\ub294 \uc704\uc6d0\ud68c\uc758 \uc911\uc7ac \uc81c\uc548\uc5d0 \ub3d9\uc758\ud558\uc600\uace0 \"\uc2dc\ubbfc\ub4e4\uc758 \uace0\ud1b5\uc744 \ub05d\ub0b4\uae30 \uc704\ud558\uc5ec, \uc6b0\ub9ac\uac00 \ub208\ubb3c\uc744 \ucc38\uace0 \ubc1b\uc544\ub4e4\uc774\uaca0\ub2e4\"\ub77c\uace0 \ub9d0\ud588\ub2e4. \ud68c\uc0ac\ub294 2\ucc9c\ub9cc \uc5d4\uc744 \uacf5\ub3d9\uccb4\uc5d0 \uc9c0\ubd88\ud558\uc600\uace0 \uc5b4\uc5c5\uc758 \ud68c\ubcf5\uc744 \uc99d\uc9c4\ud558\uae30 \uc704\ud558\uc5ec 1,500\ub9cc \uc5d4\uc758 \uae30\uae08\uc744 \ub9c8\ub828\ud588\ub2e4.", "sentence_answer": "\uce6b\uc18c \uacf5\uc7a5\uc774 1908\ub144\uc5d0 \uac1c\uc7a5\ud55c \uc774\ud6c4\ub85c \ub354\ub7ec\uc6b4 \ud3d0\uc218\ub294 \ubbf8\ub098\ub9c8\ud0c0 \uc8fc\uc704\uc758 \uc5b4\uc5c5\uc5d0 \uc190\uc2e4\uc744 \uc785\ud600\uc654\uc5c8\ub2e4.", "paragraph_id": "6093147-8-1", "paragraph_question": "question: \uce6b\uc18c \uacf5\uc7a5\uc740 \uc5b4\ub514\uc5d0 \ub354\ub7ec\uc6b4 \ud3d0\uc218\ub97c \ud758\ub824\ubcf4\ub0c8\ub098\uc694?, context: \uce6b\uc18c \uacf5\uc7a5\uc774 1908\ub144\uc5d0 \uac1c\uc7a5\ud55c \uc774\ud6c4\ub85c \ub354\ub7ec\uc6b4 \ud3d0\uc218\ub294 \ubbf8\ub098\ub9c8\ud0c0 \uc8fc\uc704\uc758 \uc5b4\uc5c5\uc5d0 \uc190\uc2e4\uc744 \uc785\ud600\uc654\uc5c8\ub2e4. '\ubbf8\ub098\ub9c8\ud0c0 \uc5b4\uc5c5 \uacf5\ub3d9\uccb4'\ub294 \ud68c\uc0ac\ub85c\ubd80\ud130 1926\ub144\uacfc 1943\ub144\uc5d0 \uc791\uc740 \ubcf4\uc0c1\uc744 \ubc1b\uc558\uc5c8\ub2e4. \ud558\uc9c0\ub9cc \ubbf8\ub098\ub9c8\ud0c0 \ubcd1 \ubc1c\uc0dd \ud6c4\uc5d0 \uc5b4\ud669\uc740 \ub208\uc5d0 \ub744\uac8c \ubcc0\ud588\ub2e4. \uc5b4\ud68d\ub7c9\uc740 1953\ub144\uacfc 1957\ub144 \uc0ac\uc774 91%\uac00 \uac10\uc18c\ud588\ub2e4. \uad6c\ub9c8\ubaa8\ud1a0 \uc9c0\ubc29 \ub2f9\uad6d\uc740 \uc624\uc5fc\ub41c \ubbf8\ub098\ub9c8\ud0c0 \ub9cc\uc5d0\uc11c \uc7a1\ud78c \ubb3c\uace0\uae30\uc758 \ud310\ub9e4\ub97c \ubd80\ubd84\uc801\uc73c\ub85c \uae08\uc9c0\ud558\uc600\ub2e4. \uc5b4\uc5c5 \uacf5\ub3d9\uccb4\ub294 \uce6b\uc18c\uc5d0 \ub300\ud574 \ud56d\uc758\ud588\uace0 8\uc6d4 6\uc77c\uacfc 8\uc6d4 12\uc77c\uc5d0 \ubcf4\uc0c1\uc744 \uac15\ud558\uac8c \uc694\uad6c\ud558\uc600\ub2e4. \ub450 \uc785\uc7a5\uc744 \uc911\uc7ac\ud558\uae30 \uc704\ud558\uc5ec \uc704\uc6d0\ud68c\uac00 \ubbf8\ub098\ub9c8\ud0c0 \uc2dc\uc7a5\uc5d0 \uc758\ud574 \uc18c\uc9d1\ub418\uc5c8\uc9c0\ub9cc \uc704\uc6d0\ud68c\ub294 \ud68c\uc0ac\uc758 \uc785\uc7a5\uc5d0\uc11c \ubd80\ub2f9\ud558\uac8c \ud3b8\uc131\ub418\uc5c8\ub2e4. 8\uc6d4 29\uc77c, \uc5b4\uc5c5 \uacf5\ub3d9\uccb4\ub294 \uc704\uc6d0\ud68c\uc758 \uc911\uc7ac \uc81c\uc548\uc5d0 \ub3d9\uc758\ud558\uc600\uace0 \"\uc2dc\ubbfc\ub4e4\uc758 \uace0\ud1b5\uc744 \ub05d\ub0b4\uae30 \uc704\ud558\uc5ec, \uc6b0\ub9ac\uac00 \ub208\ubb3c\uc744 \ucc38\uace0 \ubc1b\uc544\ub4e4\uc774\uaca0\ub2e4\"\ub77c\uace0 \ub9d0\ud588\ub2e4. \ud68c\uc0ac\ub294 2\ucc9c\ub9cc \uc5d4\uc744 \uacf5\ub3d9\uccb4\uc5d0 \uc9c0\ubd88\ud558\uc600\uace0 \uc5b4\uc5c5\uc758 \ud68c\ubcf5\uc744 \uc99d\uc9c4\ud558\uae30 \uc704\ud558\uc5ec 1,500\ub9cc \uc5d4\uc758 \uae30\uae08\uc744 \ub9c8\ub828\ud588\ub2e4."} {"question": "\uc870\uc57d\uc744 \ubb34\uc2dc\ud558\uace0 \uc6b0\ud06c\ub77c\uc774\ub098\uc5d0 \ub300\ud55c \uad8c\ub9ac\ub97c \uc8fc\uc7a5\ud558\ub294 \uacf3\uc740 \uc5b4\ub514\uc778\uac00?", "paragraph": "1658\ub144 9\uc6d4 16\uc77c \ud558\ub514\uc544\uce58 \uc870\uc57d(Treaty of Hadiach)\uc5d0 \uc758\ud574 \uc5f0\ubc29\uc758 \ub0a8\ub3d9\ubd80\uc5d0 \uc788\ub294 \ub8e8\ud14c\ub2c8\uc544\uc640 \uc6b0\ud06c\ub77c\uc774\ub098 \ucf54\uc0ac\ud06c\ub294 \ud3f4\ub780\ub4dc\uc640 \ub9ac\ud22c\uc544\ub2c8\uc544\uc758 \uc591\uad6d\uacfc \ub3d9\uaca9\uc758 \uc9c0\uc704\ub85c \uc62c\ub824\uc838 \ud3f4\ub780\ub4dc-\ub9ac\ud22c\uc544\ub2c8\uc544 \uc5f0\ubc29\uc740 \ud3f4\ub780\ub4dc-\ub9ac\ud22c\uc544\ub2c8\uc544-\ub8e8\ud14c\ub2c8\uc544 \uc5f0\ubc29( Polish-Lithuanian-Ruthenian Commonwealth, \ud3f4\ub780\ub4dc\uc5b4:Rzeczpospolita Trojga Narod\u00f3w, \"3\uac1c \ubbfc\uc871 \uc5f0\ud569\"(Commonwealth of Three Nations)\ub85c \ubcc0\ud558\uac8c \ub418\uc5c8\ub2e4. \uc6b0\ud06c\ub77c\uc774\ub098 \ud5e4\ud2b8\ub9cc \uc774\ubc18 \ube44\ud638\ud504\uc2a4\ud0a4(Ivan Vyhovsky)\uc640 \ucf54\uc0ac\ud06c\uc758 \uc9c0\ub3c4\uce35(\uc2a4\ud0c0\ub974\uc2dc\ub098;starshyna)\uc5d0 \uc758\ud574 \uc9c0\uc9c0 \ubc1b\uc558\ub358 \uc774 \uc870\uc57d\uc740 \ub3d9\uc720\ub7fd\uc758 \uad6d\uc81c\uad00\uacc4\ub97c \ud06c\uac8c \ubcc0\ud654\uc2dc\ud0a4\ub294 \uc77c\uc774 \ub418\uc5c8\ub2e4. \uadf8\ub7f0\ub370 \uc774 \uc870\uc57d\uc774 \ubc1c\ud6a8\ub418\ub294 \uc77c\uc740 \uc5c6\uc5c8\ub2e4. \uc774 \uc870\uc57d\uc744 \ubb34\uc2dc\ud558\uace0 \ub7ec\uc2dc\uc544 \ucc28\ub974\uad6d\uc740 \uc6b0\ud06c\ub77c\uc774\ub098\uc5d0 \ub300\ud55c \uad8c\ub9ac\ub97c \uc8fc\uc7a5\ud558\uace0 \uc788\uc5c8\ub2e4.", "answer": "\ub7ec\uc2dc\uc544 \ucc28\ub974\uad6d", "sentence": "\uc774 \uc870\uc57d\uc744 \ubb34\uc2dc\ud558\uace0 \ub7ec\uc2dc\uc544 \ucc28\ub974\uad6d \uc740 \uc6b0\ud06c\ub77c\uc774\ub098\uc5d0 \ub300\ud55c \uad8c\ub9ac\ub97c \uc8fc\uc7a5\ud558\uace0 \uc788\uc5c8\ub2e4.", "paragraph_sentence": "1658\ub144 9\uc6d4 16\uc77c \ud558\ub514\uc544\uce58 \uc870\uc57d(Treaty of Hadiach)\uc5d0 \uc758\ud574 \uc5f0\ubc29\uc758 \ub0a8\ub3d9\ubd80\uc5d0 \uc788\ub294 \ub8e8\ud14c\ub2c8\uc544\uc640 \uc6b0\ud06c\ub77c\uc774\ub098 \ucf54\uc0ac\ud06c\ub294 \ud3f4\ub780\ub4dc\uc640 \ub9ac\ud22c\uc544\ub2c8\uc544\uc758 \uc591\uad6d\uacfc \ub3d9\uaca9\uc758 \uc9c0\uc704\ub85c \uc62c\ub824\uc838 \ud3f4\ub780\ub4dc-\ub9ac\ud22c\uc544\ub2c8\uc544 \uc5f0\ubc29\uc740 \ud3f4\ub780\ub4dc-\ub9ac\ud22c\uc544\ub2c8\uc544-\ub8e8\ud14c\ub2c8\uc544 \uc5f0\ubc29( Polish-Lithuanian-Ruthenian Commonwealth, \ud3f4\ub780\ub4dc\uc5b4:Rzeczpospolita Trojga Narod\u00f3w, \"3\uac1c \ubbfc\uc871 \uc5f0\ud569\"(Commonwealth of Three Nations)\ub85c \ubcc0\ud558\uac8c \ub418\uc5c8\ub2e4. \uc6b0\ud06c\ub77c\uc774\ub098 \ud5e4\ud2b8\ub9cc \uc774\ubc18 \ube44\ud638\ud504\uc2a4\ud0a4(Ivan Vyhovsky)\uc640 \ucf54\uc0ac\ud06c\uc758 \uc9c0\ub3c4\uce35(\uc2a4\ud0c0\ub974\uc2dc\ub098;starshyna)\uc5d0 \uc758\ud574 \uc9c0\uc9c0 \ubc1b\uc558\ub358 \uc774 \uc870\uc57d\uc740 \ub3d9\uc720\ub7fd\uc758 \uad6d\uc81c\uad00\uacc4\ub97c \ud06c\uac8c \ubcc0\ud654\uc2dc\ud0a4\ub294 \uc77c\uc774 \ub418\uc5c8\ub2e4. \uadf8\ub7f0\ub370 \uc774 \uc870\uc57d\uc774 \ubc1c\ud6a8\ub418\ub294 \uc77c\uc740 \uc5c6\uc5c8\ub2e4. \uc774 \uc870\uc57d\uc744 \ubb34\uc2dc\ud558\uace0 \ub7ec\uc2dc\uc544 \ucc28\ub974\uad6d \uc740 \uc6b0\ud06c\ub77c\uc774\ub098\uc5d0 \ub300\ud55c \uad8c\ub9ac\ub97c \uc8fc\uc7a5\ud558\uace0 \uc788\uc5c8\ub2e4. ", "paragraph_answer": "1658\ub144 9\uc6d4 16\uc77c \ud558\ub514\uc544\uce58 \uc870\uc57d(Treaty of Hadiach)\uc5d0 \uc758\ud574 \uc5f0\ubc29\uc758 \ub0a8\ub3d9\ubd80\uc5d0 \uc788\ub294 \ub8e8\ud14c\ub2c8\uc544\uc640 \uc6b0\ud06c\ub77c\uc774\ub098 \ucf54\uc0ac\ud06c\ub294 \ud3f4\ub780\ub4dc\uc640 \ub9ac\ud22c\uc544\ub2c8\uc544\uc758 \uc591\uad6d\uacfc \ub3d9\uaca9\uc758 \uc9c0\uc704\ub85c \uc62c\ub824\uc838 \ud3f4\ub780\ub4dc-\ub9ac\ud22c\uc544\ub2c8\uc544 \uc5f0\ubc29\uc740 \ud3f4\ub780\ub4dc-\ub9ac\ud22c\uc544\ub2c8\uc544-\ub8e8\ud14c\ub2c8\uc544 \uc5f0\ubc29( Polish-Lithuanian-Ruthenian Commonwealth, \ud3f4\ub780\ub4dc\uc5b4:Rzeczpospolita Trojga Narod\u00f3w, \"3\uac1c \ubbfc\uc871 \uc5f0\ud569\"(Commonwealth of Three Nations)\ub85c \ubcc0\ud558\uac8c \ub418\uc5c8\ub2e4. \uc6b0\ud06c\ub77c\uc774\ub098 \ud5e4\ud2b8\ub9cc \uc774\ubc18 \ube44\ud638\ud504\uc2a4\ud0a4(Ivan Vyhovsky)\uc640 \ucf54\uc0ac\ud06c\uc758 \uc9c0\ub3c4\uce35(\uc2a4\ud0c0\ub974\uc2dc\ub098;starshyna)\uc5d0 \uc758\ud574 \uc9c0\uc9c0 \ubc1b\uc558\ub358 \uc774 \uc870\uc57d\uc740 \ub3d9\uc720\ub7fd\uc758 \uad6d\uc81c\uad00\uacc4\ub97c \ud06c\uac8c \ubcc0\ud654\uc2dc\ud0a4\ub294 \uc77c\uc774 \ub418\uc5c8\ub2e4. \uadf8\ub7f0\ub370 \uc774 \uc870\uc57d\uc774 \ubc1c\ud6a8\ub418\ub294 \uc77c\uc740 \uc5c6\uc5c8\ub2e4. \uc774 \uc870\uc57d\uc744 \ubb34\uc2dc\ud558\uace0 \ub7ec\uc2dc\uc544 \ucc28\ub974\uad6d \uc740 \uc6b0\ud06c\ub77c\uc774\ub098\uc5d0 \ub300\ud55c \uad8c\ub9ac\ub97c \uc8fc\uc7a5\ud558\uace0 \uc788\uc5c8\ub2e4.", "sentence_answer": "\uc774 \uc870\uc57d\uc744 \ubb34\uc2dc\ud558\uace0 \ub7ec\uc2dc\uc544 \ucc28\ub974\uad6d \uc740 \uc6b0\ud06c\ub77c\uc774\ub098\uc5d0 \ub300\ud55c \uad8c\ub9ac\ub97c \uc8fc\uc7a5\ud558\uace0 \uc788\uc5c8\ub2e4.", "paragraph_id": "6602057-5-0", "paragraph_question": "question: \uc870\uc57d\uc744 \ubb34\uc2dc\ud558\uace0 \uc6b0\ud06c\ub77c\uc774\ub098\uc5d0 \ub300\ud55c \uad8c\ub9ac\ub97c \uc8fc\uc7a5\ud558\ub294 \uacf3\uc740 \uc5b4\ub514\uc778\uac00?, context: 1658\ub144 9\uc6d4 16\uc77c \ud558\ub514\uc544\uce58 \uc870\uc57d(Treaty of Hadiach)\uc5d0 \uc758\ud574 \uc5f0\ubc29\uc758 \ub0a8\ub3d9\ubd80\uc5d0 \uc788\ub294 \ub8e8\ud14c\ub2c8\uc544\uc640 \uc6b0\ud06c\ub77c\uc774\ub098 \ucf54\uc0ac\ud06c\ub294 \ud3f4\ub780\ub4dc\uc640 \ub9ac\ud22c\uc544\ub2c8\uc544\uc758 \uc591\uad6d\uacfc \ub3d9\uaca9\uc758 \uc9c0\uc704\ub85c \uc62c\ub824\uc838 \ud3f4\ub780\ub4dc-\ub9ac\ud22c\uc544\ub2c8\uc544 \uc5f0\ubc29\uc740 \ud3f4\ub780\ub4dc-\ub9ac\ud22c\uc544\ub2c8\uc544-\ub8e8\ud14c\ub2c8\uc544 \uc5f0\ubc29( Polish-Lithuanian-Ruthenian Commonwealth, \ud3f4\ub780\ub4dc\uc5b4:Rzeczpospolita Trojga Narod\u00f3w, \"3\uac1c \ubbfc\uc871 \uc5f0\ud569\"(Commonwealth of Three Nations)\ub85c \ubcc0\ud558\uac8c \ub418\uc5c8\ub2e4. \uc6b0\ud06c\ub77c\uc774\ub098 \ud5e4\ud2b8\ub9cc \uc774\ubc18 \ube44\ud638\ud504\uc2a4\ud0a4(Ivan Vyhovsky)\uc640 \ucf54\uc0ac\ud06c\uc758 \uc9c0\ub3c4\uce35(\uc2a4\ud0c0\ub974\uc2dc\ub098;starshyna)\uc5d0 \uc758\ud574 \uc9c0\uc9c0 \ubc1b\uc558\ub358 \uc774 \uc870\uc57d\uc740 \ub3d9\uc720\ub7fd\uc758 \uad6d\uc81c\uad00\uacc4\ub97c \ud06c\uac8c \ubcc0\ud654\uc2dc\ud0a4\ub294 \uc77c\uc774 \ub418\uc5c8\ub2e4. \uadf8\ub7f0\ub370 \uc774 \uc870\uc57d\uc774 \ubc1c\ud6a8\ub418\ub294 \uc77c\uc740 \uc5c6\uc5c8\ub2e4. \uc774 \uc870\uc57d\uc744 \ubb34\uc2dc\ud558\uace0 \ub7ec\uc2dc\uc544 \ucc28\ub974\uad6d\uc740 \uc6b0\ud06c\ub77c\uc774\ub098\uc5d0 \ub300\ud55c \uad8c\ub9ac\ub97c \uc8fc\uc7a5\ud558\uace0 \uc788\uc5c8\ub2e4."} {"question": "\uac00\uc18d\ub3c4\uacc4\ub97c \ud65c\uc6a9\ud558\uc5ec \uce58\ub294 \uacf5\uaca9\uc744 \uac10\uc9c0\ud558\uae30 \uc704\ud574 \uc0ac\uc6a9\ub41c \uc18c\ucf13\uc740?", "paragraph": "\ub2e4\ub978 \ub450 \uc885\ub958\uc758 \uac80\uacfc\ub294 \ub2e4\ub974\uac8c, \uc804\uc790\uc2dd \uc0ac\ube0c\ub974\uc640 \ube44\uc804\uc790\uc2dd (\uac74\uc2dd / \uc2b5\uc2dd) \uc0ac\ube0c\ub974 \uc0ac\uc774\uc5d0\ub294 \ubcc4\ub2e4\ub978 \ucc28\uc774\uac00 \uc5c6\ub2e4. \uce7c\ub0a0\uc740 \uce7c\ub05d\uae4c\uc9c0 \uc5f0\uacb0\ud560 \uac80 \uc804\uc120\uacfc \uc555\ub825\uc5d0 \ubbfc\uac10\ud55c \uce7c\ub05d\uc774 \ud544\uc694\ud558\uc9c0 \uc54a\uc73c\ubbc0\ub85c, \ube44\uc804\uc790\uc2dd\uacfc \uc804\uc790\uc2dd \ubaa8\ub450 \uad6c\uc870\uac00 \ub3d9\uc77c\ud558\ub2e4. \uc804\uc790 \uc0ac\ube0c\ub974\ub294 \uc18c\ucf13\uc774 \ub2ec\ub824 \uc788\ub294\ub370, \uc774 \uc18c\ucf13 \ubaa8\uc591\uc740 \uc77c\ubc18\uc801\uc73c\ub85c \ud50c\ub8b0\ub808\uc758 \uc18c\ucf13\uacfc \ube44\uc2b7\ud55c 2\uac1c\uc758 \uc18c\ucf13\uc73c\ub85c \ub41c \ubc14\uc694\ub137\uc2dd\uc73c\ub85c \ub418\uc5b4 \uc788\ub2e4. \ucd08\uae30\uc758 \uc804\uc790 \uc0ac\ube0c\ub974\ub294 \uc9d1\uc801\ud615 \uc18c\ucf13\uc73c\ub85c \ub418\uc5b4 \uc788\uc5c8\ub2e4. \uc9d1\uc801\ud615 \uc18c\ucf13\uc740 \uac00\uc18d\ub3c4\uacc4\ub97c \ud65c\uc6a9\ud558\uc5ec \uce58\ub294 \uacf5\uaca9\uc744 \uac10\uc9c0\ud558\uae30 \uc704\ud558\uc5ec \uc0ac\uc6a9\ub418\uc5c8\ub2e4. \uce58\ub294 \uacf5\uaca9\uc774 \uac10\uc9c0\ub418\uc5c8\uc744\ub54c \uc804\uc790 \uc7a5\ube44\ub294 \uc774\ud6c4\uc758 \ub3d9\uc791\uc744 \uac10\uc9c0\ud558\uc9c0 \ubabb\ud558\ub3c4\ub85d \uc810\uc218 \ud68c\ub85c\ub97c \ub2eb\uc544 \uc720\uc5f0\ud55c \uac80\uc73c\ub85c \uc778\ud55c \uc9c4\ub3d9\uc744 \ubb34\uc2dc\ud558\uc5ec\uc57c \ud588\ub2e4. \uc774 \uc7a5\ube44\ub294 \ubcf8 \uc758\ub3c4\ub300\ub85c \uc791\ub3d9\ub418\uc9c0 \uc54a\uc558\uace0, \uae08\uc138 \uc0ac\uc6a9\uc774 \uc911\uc9c0\ub418\uc5c8\uc73c\uba70, \uad6d\uc81c \ud39c\uc2f1 \uc5f0\ub9f9 (F\u00e9d\u00e9ration Internationale d'Escrime, FIE) \uc5d0\uc11c \ub35c \uad6c\ubd80\ub7ec\uc9c0\ub294 S2000\uc2dd \uc0ac\ube0c\ub974 \uce7c\ub0a0 \uc0ac\uc6a9\uc744 \uc758\ubb34\ud654\ud558\uba74\uc11c \uce7c\ub0a0\uc758 \uc9c4\ub3d9 \ud6a8\uacfc\ub294 \uc644\ud654\ub418\uc5c8\ub2e4. \uc804\uc790 \uc0ac\ube0c\ub974\ub294 \ud3ec\uba5c\uacfc \ucef5\uac00\ub4dc\uac00 \uc808\uc5f0\uc2dd\uc73c\ub85c \ub418\uc5b4 \uc788\uc5b4 \uc0ac\ube0c\ub974\uc640 \ub77c\uba54 \uc0ac\uc774\uc758 \uc804\ub958\ub85c \uc778\ud55c \uae30\uae30 \uc624\uc791\ub3d9\uc744 \ubc29\uc9c0\ud55c\ub2e4. \ub9cc\uc57d, \ud3ec\uba5c\uc774 \uc808\uc5f0\uccb4\uac00 \uc544\ub2cc \uacbd\uc6b0 \uc0ac\ube0c\ub974\uac00 \ud3ec\uba5c\uc744 \ub77c\uba54\ub85c \uc778\uc2dd\ud558\uac8c \ub418\uace0, \uae30\uae30\uac00 \uc720\ud6a8\ud0c0\ub85c \uc778\uc2dd\ud560 \uc218 \uc788\ub2e4.", "answer": "\uc9d1\uc801\ud615 \uc18c\ucf13", "sentence": "\ucd08\uae30\uc758 \uc804\uc790 \uc0ac\ube0c\ub974\ub294 \uc9d1\uc801\ud615 \uc18c\ucf13 \uc73c\ub85c \ub418\uc5b4 \uc788\uc5c8\ub2e4.", "paragraph_sentence": "\ub2e4\ub978 \ub450 \uc885\ub958\uc758 \uac80\uacfc\ub294 \ub2e4\ub974\uac8c, \uc804\uc790\uc2dd \uc0ac\ube0c\ub974\uc640 \ube44\uc804\uc790\uc2dd (\uac74\uc2dd / \uc2b5\uc2dd) \uc0ac\ube0c\ub974 \uc0ac\uc774\uc5d0\ub294 \ubcc4\ub2e4\ub978 \ucc28\uc774\uac00 \uc5c6\ub2e4. \uce7c\ub0a0\uc740 \uce7c\ub05d\uae4c\uc9c0 \uc5f0\uacb0\ud560 \uac80 \uc804\uc120\uacfc \uc555\ub825\uc5d0 \ubbfc\uac10\ud55c \uce7c\ub05d\uc774 \ud544\uc694\ud558\uc9c0 \uc54a\uc73c\ubbc0\ub85c, \ube44\uc804\uc790\uc2dd\uacfc \uc804\uc790\uc2dd \ubaa8\ub450 \uad6c\uc870\uac00 \ub3d9\uc77c\ud558\ub2e4. \uc804\uc790 \uc0ac\ube0c\ub974\ub294 \uc18c\ucf13\uc774 \ub2ec\ub824 \uc788\ub294\ub370, \uc774 \uc18c\ucf13 \ubaa8\uc591\uc740 \uc77c\ubc18\uc801\uc73c\ub85c \ud50c\ub8b0\ub808\uc758 \uc18c\ucf13\uacfc \ube44\uc2b7\ud55c 2\uac1c\uc758 \uc18c\ucf13\uc73c\ub85c \ub41c \ubc14\uc694\ub137\uc2dd\uc73c\ub85c \ub418\uc5b4 \uc788\ub2e4. \ucd08\uae30\uc758 \uc804\uc790 \uc0ac\ube0c\ub974\ub294 \uc9d1\uc801\ud615 \uc18c\ucf13 \uc73c\ub85c \ub418\uc5b4 \uc788\uc5c8\ub2e4. \uc9d1\uc801\ud615 \uc18c\ucf13\uc740 \uac00\uc18d\ub3c4\uacc4\ub97c \ud65c\uc6a9\ud558\uc5ec \uce58\ub294 \uacf5\uaca9\uc744 \uac10\uc9c0\ud558\uae30 \uc704\ud558\uc5ec \uc0ac\uc6a9\ub418\uc5c8\ub2e4. \uce58\ub294 \uacf5\uaca9\uc774 \uac10\uc9c0\ub418\uc5c8\uc744\ub54c \uc804\uc790 \uc7a5\ube44\ub294 \uc774\ud6c4\uc758 \ub3d9\uc791\uc744 \uac10\uc9c0\ud558\uc9c0 \ubabb\ud558\ub3c4\ub85d \uc810\uc218 \ud68c\ub85c\ub97c \ub2eb\uc544 \uc720\uc5f0\ud55c \uac80\uc73c\ub85c \uc778\ud55c \uc9c4\ub3d9\uc744 \ubb34\uc2dc\ud558\uc5ec\uc57c \ud588\ub2e4. \uc774 \uc7a5\ube44\ub294 \ubcf8 \uc758\ub3c4\ub300\ub85c \uc791\ub3d9\ub418\uc9c0 \uc54a\uc558\uace0, \uae08\uc138 \uc0ac\uc6a9\uc774 \uc911\uc9c0\ub418\uc5c8\uc73c\uba70, \uad6d\uc81c \ud39c\uc2f1 \uc5f0\ub9f9 (F\u00e9d\u00e9ration Internationale d'Escrime, FIE) \uc5d0\uc11c \ub35c \uad6c\ubd80\ub7ec\uc9c0\ub294 S2000\uc2dd \uc0ac\ube0c\ub974 \uce7c\ub0a0 \uc0ac\uc6a9\uc744 \uc758\ubb34\ud654\ud558\uba74\uc11c \uce7c\ub0a0\uc758 \uc9c4\ub3d9 \ud6a8\uacfc\ub294 \uc644\ud654\ub418\uc5c8\ub2e4. \uc804\uc790 \uc0ac\ube0c\ub974\ub294 \ud3ec\uba5c\uacfc \ucef5\uac00\ub4dc\uac00 \uc808\uc5f0\uc2dd\uc73c\ub85c \ub418\uc5b4 \uc788\uc5b4 \uc0ac\ube0c\ub974\uc640 \ub77c\uba54 \uc0ac\uc774\uc758 \uc804\ub958\ub85c \uc778\ud55c \uae30\uae30 \uc624\uc791\ub3d9\uc744 \ubc29\uc9c0\ud55c\ub2e4. \ub9cc\uc57d, \ud3ec\uba5c\uc774 \uc808\uc5f0\uccb4\uac00 \uc544\ub2cc \uacbd\uc6b0 \uc0ac\ube0c\ub974\uac00 \ud3ec\uba5c\uc744 \ub77c\uba54\ub85c \uc778\uc2dd\ud558\uac8c \ub418\uace0, \uae30\uae30\uac00 \uc720\ud6a8\ud0c0\ub85c \uc778\uc2dd\ud560 \uc218 \uc788\ub2e4.", "paragraph_answer": "\ub2e4\ub978 \ub450 \uc885\ub958\uc758 \uac80\uacfc\ub294 \ub2e4\ub974\uac8c, \uc804\uc790\uc2dd \uc0ac\ube0c\ub974\uc640 \ube44\uc804\uc790\uc2dd (\uac74\uc2dd / \uc2b5\uc2dd) \uc0ac\ube0c\ub974 \uc0ac\uc774\uc5d0\ub294 \ubcc4\ub2e4\ub978 \ucc28\uc774\uac00 \uc5c6\ub2e4. \uce7c\ub0a0\uc740 \uce7c\ub05d\uae4c\uc9c0 \uc5f0\uacb0\ud560 \uac80 \uc804\uc120\uacfc \uc555\ub825\uc5d0 \ubbfc\uac10\ud55c \uce7c\ub05d\uc774 \ud544\uc694\ud558\uc9c0 \uc54a\uc73c\ubbc0\ub85c, \ube44\uc804\uc790\uc2dd\uacfc \uc804\uc790\uc2dd \ubaa8\ub450 \uad6c\uc870\uac00 \ub3d9\uc77c\ud558\ub2e4. \uc804\uc790 \uc0ac\ube0c\ub974\ub294 \uc18c\ucf13\uc774 \ub2ec\ub824 \uc788\ub294\ub370, \uc774 \uc18c\ucf13 \ubaa8\uc591\uc740 \uc77c\ubc18\uc801\uc73c\ub85c \ud50c\ub8b0\ub808\uc758 \uc18c\ucf13\uacfc \ube44\uc2b7\ud55c 2\uac1c\uc758 \uc18c\ucf13\uc73c\ub85c \ub41c \ubc14\uc694\ub137\uc2dd\uc73c\ub85c \ub418\uc5b4 \uc788\ub2e4. \ucd08\uae30\uc758 \uc804\uc790 \uc0ac\ube0c\ub974\ub294 \uc9d1\uc801\ud615 \uc18c\ucf13 \uc73c\ub85c \ub418\uc5b4 \uc788\uc5c8\ub2e4. \uc9d1\uc801\ud615 \uc18c\ucf13\uc740 \uac00\uc18d\ub3c4\uacc4\ub97c \ud65c\uc6a9\ud558\uc5ec \uce58\ub294 \uacf5\uaca9\uc744 \uac10\uc9c0\ud558\uae30 \uc704\ud558\uc5ec \uc0ac\uc6a9\ub418\uc5c8\ub2e4. \uce58\ub294 \uacf5\uaca9\uc774 \uac10\uc9c0\ub418\uc5c8\uc744\ub54c \uc804\uc790 \uc7a5\ube44\ub294 \uc774\ud6c4\uc758 \ub3d9\uc791\uc744 \uac10\uc9c0\ud558\uc9c0 \ubabb\ud558\ub3c4\ub85d \uc810\uc218 \ud68c\ub85c\ub97c \ub2eb\uc544 \uc720\uc5f0\ud55c \uac80\uc73c\ub85c \uc778\ud55c \uc9c4\ub3d9\uc744 \ubb34\uc2dc\ud558\uc5ec\uc57c \ud588\ub2e4. \uc774 \uc7a5\ube44\ub294 \ubcf8 \uc758\ub3c4\ub300\ub85c \uc791\ub3d9\ub418\uc9c0 \uc54a\uc558\uace0, \uae08\uc138 \uc0ac\uc6a9\uc774 \uc911\uc9c0\ub418\uc5c8\uc73c\uba70, \uad6d\uc81c \ud39c\uc2f1 \uc5f0\ub9f9 (F\u00e9d\u00e9ration Internationale d'Escrime, FIE) \uc5d0\uc11c \ub35c \uad6c\ubd80\ub7ec\uc9c0\ub294 S2000\uc2dd \uc0ac\ube0c\ub974 \uce7c\ub0a0 \uc0ac\uc6a9\uc744 \uc758\ubb34\ud654\ud558\uba74\uc11c \uce7c\ub0a0\uc758 \uc9c4\ub3d9 \ud6a8\uacfc\ub294 \uc644\ud654\ub418\uc5c8\ub2e4. \uc804\uc790 \uc0ac\ube0c\ub974\ub294 \ud3ec\uba5c\uacfc \ucef5\uac00\ub4dc\uac00 \uc808\uc5f0\uc2dd\uc73c\ub85c \ub418\uc5b4 \uc788\uc5b4 \uc0ac\ube0c\ub974\uc640 \ub77c\uba54 \uc0ac\uc774\uc758 \uc804\ub958\ub85c \uc778\ud55c \uae30\uae30 \uc624\uc791\ub3d9\uc744 \ubc29\uc9c0\ud55c\ub2e4. \ub9cc\uc57d, \ud3ec\uba5c\uc774 \uc808\uc5f0\uccb4\uac00 \uc544\ub2cc \uacbd\uc6b0 \uc0ac\ube0c\ub974\uac00 \ud3ec\uba5c\uc744 \ub77c\uba54\ub85c \uc778\uc2dd\ud558\uac8c \ub418\uace0, \uae30\uae30\uac00 \uc720\ud6a8\ud0c0\ub85c \uc778\uc2dd\ud560 \uc218 \uc788\ub2e4.", "sentence_answer": "\ucd08\uae30\uc758 \uc804\uc790 \uc0ac\ube0c\ub974\ub294 \uc9d1\uc801\ud615 \uc18c\ucf13 \uc73c\ub85c \ub418\uc5b4 \uc788\uc5c8\ub2e4.", "paragraph_id": "6510838-1-0", "paragraph_question": "question: \uac00\uc18d\ub3c4\uacc4\ub97c \ud65c\uc6a9\ud558\uc5ec \uce58\ub294 \uacf5\uaca9\uc744 \uac10\uc9c0\ud558\uae30 \uc704\ud574 \uc0ac\uc6a9\ub41c \uc18c\ucf13\uc740?, context: \ub2e4\ub978 \ub450 \uc885\ub958\uc758 \uac80\uacfc\ub294 \ub2e4\ub974\uac8c, \uc804\uc790\uc2dd \uc0ac\ube0c\ub974\uc640 \ube44\uc804\uc790\uc2dd (\uac74\uc2dd / \uc2b5\uc2dd) \uc0ac\ube0c\ub974 \uc0ac\uc774\uc5d0\ub294 \ubcc4\ub2e4\ub978 \ucc28\uc774\uac00 \uc5c6\ub2e4. \uce7c\ub0a0\uc740 \uce7c\ub05d\uae4c\uc9c0 \uc5f0\uacb0\ud560 \uac80 \uc804\uc120\uacfc \uc555\ub825\uc5d0 \ubbfc\uac10\ud55c \uce7c\ub05d\uc774 \ud544\uc694\ud558\uc9c0 \uc54a\uc73c\ubbc0\ub85c, \ube44\uc804\uc790\uc2dd\uacfc \uc804\uc790\uc2dd \ubaa8\ub450 \uad6c\uc870\uac00 \ub3d9\uc77c\ud558\ub2e4. \uc804\uc790 \uc0ac\ube0c\ub974\ub294 \uc18c\ucf13\uc774 \ub2ec\ub824 \uc788\ub294\ub370, \uc774 \uc18c\ucf13 \ubaa8\uc591\uc740 \uc77c\ubc18\uc801\uc73c\ub85c \ud50c\ub8b0\ub808\uc758 \uc18c\ucf13\uacfc \ube44\uc2b7\ud55c 2\uac1c\uc758 \uc18c\ucf13\uc73c\ub85c \ub41c \ubc14\uc694\ub137\uc2dd\uc73c\ub85c \ub418\uc5b4 \uc788\ub2e4. \ucd08\uae30\uc758 \uc804\uc790 \uc0ac\ube0c\ub974\ub294 \uc9d1\uc801\ud615 \uc18c\ucf13\uc73c\ub85c \ub418\uc5b4 \uc788\uc5c8\ub2e4. \uc9d1\uc801\ud615 \uc18c\ucf13\uc740 \uac00\uc18d\ub3c4\uacc4\ub97c \ud65c\uc6a9\ud558\uc5ec \uce58\ub294 \uacf5\uaca9\uc744 \uac10\uc9c0\ud558\uae30 \uc704\ud558\uc5ec \uc0ac\uc6a9\ub418\uc5c8\ub2e4. \uce58\ub294 \uacf5\uaca9\uc774 \uac10\uc9c0\ub418\uc5c8\uc744\ub54c \uc804\uc790 \uc7a5\ube44\ub294 \uc774\ud6c4\uc758 \ub3d9\uc791\uc744 \uac10\uc9c0\ud558\uc9c0 \ubabb\ud558\ub3c4\ub85d \uc810\uc218 \ud68c\ub85c\ub97c \ub2eb\uc544 \uc720\uc5f0\ud55c \uac80\uc73c\ub85c \uc778\ud55c \uc9c4\ub3d9\uc744 \ubb34\uc2dc\ud558\uc5ec\uc57c \ud588\ub2e4. \uc774 \uc7a5\ube44\ub294 \ubcf8 \uc758\ub3c4\ub300\ub85c \uc791\ub3d9\ub418\uc9c0 \uc54a\uc558\uace0, \uae08\uc138 \uc0ac\uc6a9\uc774 \uc911\uc9c0\ub418\uc5c8\uc73c\uba70, \uad6d\uc81c \ud39c\uc2f1 \uc5f0\ub9f9 (F\u00e9d\u00e9ration Internationale d'Escrime, FIE) \uc5d0\uc11c \ub35c \uad6c\ubd80\ub7ec\uc9c0\ub294 S2000\uc2dd \uc0ac\ube0c\ub974 \uce7c\ub0a0 \uc0ac\uc6a9\uc744 \uc758\ubb34\ud654\ud558\uba74\uc11c \uce7c\ub0a0\uc758 \uc9c4\ub3d9 \ud6a8\uacfc\ub294 \uc644\ud654\ub418\uc5c8\ub2e4. \uc804\uc790 \uc0ac\ube0c\ub974\ub294 \ud3ec\uba5c\uacfc \ucef5\uac00\ub4dc\uac00 \uc808\uc5f0\uc2dd\uc73c\ub85c \ub418\uc5b4 \uc788\uc5b4 \uc0ac\ube0c\ub974\uc640 \ub77c\uba54 \uc0ac\uc774\uc758 \uc804\ub958\ub85c \uc778\ud55c \uae30\uae30 \uc624\uc791\ub3d9\uc744 \ubc29\uc9c0\ud55c\ub2e4. \ub9cc\uc57d, \ud3ec\uba5c\uc774 \uc808\uc5f0\uccb4\uac00 \uc544\ub2cc \uacbd\uc6b0 \uc0ac\ube0c\ub974\uac00 \ud3ec\uba5c\uc744 \ub77c\uba54\ub85c \uc778\uc2dd\ud558\uac8c \ub418\uace0, \uae30\uae30\uac00 \uc720\ud6a8\ud0c0\ub85c \uc778\uc2dd\ud560 \uc218 \uc788\ub2e4."} {"question": "\ub85c\uc800 \ud398\ub354\ub7ec\uac00 \ubca0\uc774\uc9d5 \uc62c\ub9bc\ud53d \ubcf5\uc2dd\uc5d0\uc11c \ub534 \uba54\ub2ec\uc740?", "paragraph": "2008\ub144\uc5d0\ub294 \uadf8\ub79c\ub4dc \uc2ac\ub7a8 \ub300\ud68c \uc911 US \uc624\ud508\uc5d0\uc11c \uc6b0\uc2b9\ud588\ub2e4. \ud504\ub791\uc2a4 \uc624\ud508\uacfc \uc714\ube14\ub358\uc5d0\uc11c\ub294 \uacb0\uc2b9\uc5d0\uc11c \ub450 \ubc88 \ub098\ub2ec\uc744 \ub9cc\ub098 \ud328\ud558\uba74\uc11c \uc900\uc6b0\uc2b9\uc5d0 \uadf8\ucce4\ub2e4. \ud638\uc8fc \uc624\ud508\uc5d0\uc11c\ub294 \uc900\uacb0\uc2b9\uc5d0\uc11c \ub178\ubc15 \uc870\ucf54\ube44\uce58\uc5d0\uac8c \ud328\ud588\uc73c\uba70, \uc774\ub85c\uc368 \uadf8\uc758 \uadf8\ub79c\ub4dc \uc2ac\ub7a8 \ub300\ud68c 10\ud68c \uc5f0\uc18d \uacb0\uc2b9 \uc9c4\ucd9c \uae30\ub85d\ub3c4 \uba48\ucd94\uac8c \ub418\uc5c8\ub2e4. \ud074\ub808\uc774 \ucf54\ud2b8 \ub9c8\uc2a4\ud130\uc2a4 \uc2dc\ub9ac\uc988 \ub300\ud68c\uc778 \ubaac\ud14c\uce74\ub97c\ub85c \ub9c8\uc2a4\ud130\uc2a4\uc640 \ud568\ubd80\ub974\ud06c \ub9c8\uc2a4\ud130\uc2a4 \uacb0\uc2b9\uc5d0\uc11c\ub3c4 \ub098\ub2ec\uc5d0\uac8c \ud328\ud574 \uc900\uc6b0\uc2b9\uc5d0 \uba38\ubb3c\ub800\ub2e4. ATP 250 \uc2dc\ub9ac\uc988 \ub300\ud68c\uc778 \uac8c\ub9ac \uc6e8\ubc84 \uc624\ud508\uacfc ATP 500 \ub300\ud68c\uc778 \ubc14\uc824 \uc624\ud508\uc5d0\uc11c \uc6b0\uc2b9\ud588\ub2e4. \uadf8\ub294 2008\ub144 \ubca0\uc774\uc9d5 \uc62c\ub9bc\ud53d\uc5d0\ub3c4 \ucc38\uac00\ud588\uc73c\uba70, \ub2e8\uc2dd\uc5d0\uc11c\ub294 8\uac15\uc5d0\uc11c \ubbf8\uad6d\uc758 \uc81c\uc784\uc2a4 \ube14\ub808\uc774\ud06c\uc5d0\uac8c \ud328\ud588\uace0 \ubcf5\uc2dd\uc5d0\uc11c\ub294 \uc2a4\uc704\uc2a4\uc758 \ub3d9\ub8cc \uc120\uc218\uc778 \uc2a4\ud0c0\ub2c8\uc2ac\ub77c\uc2a4 \ubc14\ube0c\ub9b0\uce74\uc640 \ud568\uaed8 \ucd9c\uc804\ud558\uc5ec \uc6b0\uc2b9, \uae08\uba54\ub2ec\uc744 \ud68d\ub4dd\ud588\ub2e4.", "answer": "\uae08\uba54\ub2ec", "sentence": "\uadf8\ub294 2008\ub144 \ubca0\uc774\uc9d5 \uc62c\ub9bc\ud53d\uc5d0\ub3c4 \ucc38\uac00\ud588\uc73c\uba70, \ub2e8\uc2dd\uc5d0\uc11c\ub294 8\uac15\uc5d0\uc11c \ubbf8\uad6d\uc758 \uc81c\uc784\uc2a4 \ube14\ub808\uc774\ud06c\uc5d0\uac8c \ud328\ud588\uace0 \ubcf5\uc2dd\uc5d0\uc11c\ub294 \uc2a4\uc704\uc2a4\uc758 \ub3d9\ub8cc \uc120\uc218\uc778 \uc2a4\ud0c0\ub2c8\uc2ac\ub77c\uc2a4 \ubc14\ube0c\ub9b0\uce74\uc640 \ud568\uaed8 \ucd9c\uc804\ud558\uc5ec \uc6b0\uc2b9, \uae08\uba54\ub2ec \uc744 \ud68d\ub4dd\ud588\ub2e4.", "paragraph_sentence": "2008\ub144\uc5d0\ub294 \uadf8\ub79c\ub4dc \uc2ac\ub7a8 \ub300\ud68c \uc911 US \uc624\ud508\uc5d0\uc11c \uc6b0\uc2b9\ud588\ub2e4. \ud504\ub791\uc2a4 \uc624\ud508\uacfc \uc714\ube14\ub358\uc5d0\uc11c\ub294 \uacb0\uc2b9\uc5d0\uc11c \ub450 \ubc88 \ub098\ub2ec\uc744 \ub9cc\ub098 \ud328\ud558\uba74\uc11c \uc900\uc6b0\uc2b9\uc5d0 \uadf8\ucce4\ub2e4. \ud638\uc8fc \uc624\ud508\uc5d0\uc11c\ub294 \uc900\uacb0\uc2b9\uc5d0\uc11c \ub178\ubc15 \uc870\ucf54\ube44\uce58\uc5d0\uac8c \ud328\ud588\uc73c\uba70, \uc774\ub85c\uc368 \uadf8\uc758 \uadf8\ub79c\ub4dc \uc2ac\ub7a8 \ub300\ud68c 10\ud68c \uc5f0\uc18d \uacb0\uc2b9 \uc9c4\ucd9c \uae30\ub85d\ub3c4 \uba48\ucd94\uac8c \ub418\uc5c8\ub2e4. \ud074\ub808\uc774 \ucf54\ud2b8 \ub9c8\uc2a4\ud130\uc2a4 \uc2dc\ub9ac\uc988 \ub300\ud68c\uc778 \ubaac\ud14c\uce74\ub97c\ub85c \ub9c8\uc2a4\ud130\uc2a4\uc640 \ud568\ubd80\ub974\ud06c \ub9c8\uc2a4\ud130\uc2a4 \uacb0\uc2b9\uc5d0\uc11c\ub3c4 \ub098\ub2ec\uc5d0\uac8c \ud328\ud574 \uc900\uc6b0\uc2b9\uc5d0 \uba38\ubb3c\ub800\ub2e4. ATP 250 \uc2dc\ub9ac\uc988 \ub300\ud68c\uc778 \uac8c\ub9ac \uc6e8\ubc84 \uc624\ud508\uacfc ATP 500 \ub300\ud68c\uc778 \ubc14\uc824 \uc624\ud508\uc5d0\uc11c \uc6b0\uc2b9\ud588\ub2e4. \uadf8\ub294 2008\ub144 \ubca0\uc774\uc9d5 \uc62c\ub9bc\ud53d\uc5d0\ub3c4 \ucc38\uac00\ud588\uc73c\uba70, \ub2e8\uc2dd\uc5d0\uc11c\ub294 8\uac15\uc5d0\uc11c \ubbf8\uad6d\uc758 \uc81c\uc784\uc2a4 \ube14\ub808\uc774\ud06c\uc5d0\uac8c \ud328\ud588\uace0 \ubcf5\uc2dd\uc5d0\uc11c\ub294 \uc2a4\uc704\uc2a4\uc758 \ub3d9\ub8cc \uc120\uc218\uc778 \uc2a4\ud0c0\ub2c8\uc2ac\ub77c\uc2a4 \ubc14\ube0c\ub9b0\uce74\uc640 \ud568\uaed8 \ucd9c\uc804\ud558\uc5ec \uc6b0\uc2b9, \uae08\uba54\ub2ec \uc744 \ud68d\ub4dd\ud588\ub2e4. ", "paragraph_answer": "2008\ub144\uc5d0\ub294 \uadf8\ub79c\ub4dc \uc2ac\ub7a8 \ub300\ud68c \uc911 US \uc624\ud508\uc5d0\uc11c \uc6b0\uc2b9\ud588\ub2e4. \ud504\ub791\uc2a4 \uc624\ud508\uacfc \uc714\ube14\ub358\uc5d0\uc11c\ub294 \uacb0\uc2b9\uc5d0\uc11c \ub450 \ubc88 \ub098\ub2ec\uc744 \ub9cc\ub098 \ud328\ud558\uba74\uc11c \uc900\uc6b0\uc2b9\uc5d0 \uadf8\ucce4\ub2e4. \ud638\uc8fc \uc624\ud508\uc5d0\uc11c\ub294 \uc900\uacb0\uc2b9\uc5d0\uc11c \ub178\ubc15 \uc870\ucf54\ube44\uce58\uc5d0\uac8c \ud328\ud588\uc73c\uba70, \uc774\ub85c\uc368 \uadf8\uc758 \uadf8\ub79c\ub4dc \uc2ac\ub7a8 \ub300\ud68c 10\ud68c \uc5f0\uc18d \uacb0\uc2b9 \uc9c4\ucd9c \uae30\ub85d\ub3c4 \uba48\ucd94\uac8c \ub418\uc5c8\ub2e4. \ud074\ub808\uc774 \ucf54\ud2b8 \ub9c8\uc2a4\ud130\uc2a4 \uc2dc\ub9ac\uc988 \ub300\ud68c\uc778 \ubaac\ud14c\uce74\ub97c\ub85c \ub9c8\uc2a4\ud130\uc2a4\uc640 \ud568\ubd80\ub974\ud06c \ub9c8\uc2a4\ud130\uc2a4 \uacb0\uc2b9\uc5d0\uc11c\ub3c4 \ub098\ub2ec\uc5d0\uac8c \ud328\ud574 \uc900\uc6b0\uc2b9\uc5d0 \uba38\ubb3c\ub800\ub2e4. ATP 250 \uc2dc\ub9ac\uc988 \ub300\ud68c\uc778 \uac8c\ub9ac \uc6e8\ubc84 \uc624\ud508\uacfc ATP 500 \ub300\ud68c\uc778 \ubc14\uc824 \uc624\ud508\uc5d0\uc11c \uc6b0\uc2b9\ud588\ub2e4. \uadf8\ub294 2008\ub144 \ubca0\uc774\uc9d5 \uc62c\ub9bc\ud53d\uc5d0\ub3c4 \ucc38\uac00\ud588\uc73c\uba70, \ub2e8\uc2dd\uc5d0\uc11c\ub294 8\uac15\uc5d0\uc11c \ubbf8\uad6d\uc758 \uc81c\uc784\uc2a4 \ube14\ub808\uc774\ud06c\uc5d0\uac8c \ud328\ud588\uace0 \ubcf5\uc2dd\uc5d0\uc11c\ub294 \uc2a4\uc704\uc2a4\uc758 \ub3d9\ub8cc \uc120\uc218\uc778 \uc2a4\ud0c0\ub2c8\uc2ac\ub77c\uc2a4 \ubc14\ube0c\ub9b0\uce74\uc640 \ud568\uaed8 \ucd9c\uc804\ud558\uc5ec \uc6b0\uc2b9, \uae08\uba54\ub2ec \uc744 \ud68d\ub4dd\ud588\ub2e4.", "sentence_answer": "\uadf8\ub294 2008\ub144 \ubca0\uc774\uc9d5 \uc62c\ub9bc\ud53d\uc5d0\ub3c4 \ucc38\uac00\ud588\uc73c\uba70, \ub2e8\uc2dd\uc5d0\uc11c\ub294 8\uac15\uc5d0\uc11c \ubbf8\uad6d\uc758 \uc81c\uc784\uc2a4 \ube14\ub808\uc774\ud06c\uc5d0\uac8c \ud328\ud588\uace0 \ubcf5\uc2dd\uc5d0\uc11c\ub294 \uc2a4\uc704\uc2a4\uc758 \ub3d9\ub8cc \uc120\uc218\uc778 \uc2a4\ud0c0\ub2c8\uc2ac\ub77c\uc2a4 \ubc14\ube0c\ub9b0\uce74\uc640 \ud568\uaed8 \ucd9c\uc804\ud558\uc5ec \uc6b0\uc2b9, \uae08\uba54\ub2ec \uc744 \ud68d\ub4dd\ud588\ub2e4.", "paragraph_id": "6496547-9-2", "paragraph_question": "question: \ub85c\uc800 \ud398\ub354\ub7ec\uac00 \ubca0\uc774\uc9d5 \uc62c\ub9bc\ud53d \ubcf5\uc2dd\uc5d0\uc11c \ub534 \uba54\ub2ec\uc740?, context: 2008\ub144\uc5d0\ub294 \uadf8\ub79c\ub4dc \uc2ac\ub7a8 \ub300\ud68c \uc911 US \uc624\ud508\uc5d0\uc11c \uc6b0\uc2b9\ud588\ub2e4. \ud504\ub791\uc2a4 \uc624\ud508\uacfc \uc714\ube14\ub358\uc5d0\uc11c\ub294 \uacb0\uc2b9\uc5d0\uc11c \ub450 \ubc88 \ub098\ub2ec\uc744 \ub9cc\ub098 \ud328\ud558\uba74\uc11c \uc900\uc6b0\uc2b9\uc5d0 \uadf8\ucce4\ub2e4. \ud638\uc8fc \uc624\ud508\uc5d0\uc11c\ub294 \uc900\uacb0\uc2b9\uc5d0\uc11c \ub178\ubc15 \uc870\ucf54\ube44\uce58\uc5d0\uac8c \ud328\ud588\uc73c\uba70, \uc774\ub85c\uc368 \uadf8\uc758 \uadf8\ub79c\ub4dc \uc2ac\ub7a8 \ub300\ud68c 10\ud68c \uc5f0\uc18d \uacb0\uc2b9 \uc9c4\ucd9c \uae30\ub85d\ub3c4 \uba48\ucd94\uac8c \ub418\uc5c8\ub2e4. \ud074\ub808\uc774 \ucf54\ud2b8 \ub9c8\uc2a4\ud130\uc2a4 \uc2dc\ub9ac\uc988 \ub300\ud68c\uc778 \ubaac\ud14c\uce74\ub97c\ub85c \ub9c8\uc2a4\ud130\uc2a4\uc640 \ud568\ubd80\ub974\ud06c \ub9c8\uc2a4\ud130\uc2a4 \uacb0\uc2b9\uc5d0\uc11c\ub3c4 \ub098\ub2ec\uc5d0\uac8c \ud328\ud574 \uc900\uc6b0\uc2b9\uc5d0 \uba38\ubb3c\ub800\ub2e4. ATP 250 \uc2dc\ub9ac\uc988 \ub300\ud68c\uc778 \uac8c\ub9ac \uc6e8\ubc84 \uc624\ud508\uacfc ATP 500 \ub300\ud68c\uc778 \ubc14\uc824 \uc624\ud508\uc5d0\uc11c \uc6b0\uc2b9\ud588\ub2e4. \uadf8\ub294 2008\ub144 \ubca0\uc774\uc9d5 \uc62c\ub9bc\ud53d\uc5d0\ub3c4 \ucc38\uac00\ud588\uc73c\uba70, \ub2e8\uc2dd\uc5d0\uc11c\ub294 8\uac15\uc5d0\uc11c \ubbf8\uad6d\uc758 \uc81c\uc784\uc2a4 \ube14\ub808\uc774\ud06c\uc5d0\uac8c \ud328\ud588\uace0 \ubcf5\uc2dd\uc5d0\uc11c\ub294 \uc2a4\uc704\uc2a4\uc758 \ub3d9\ub8cc \uc120\uc218\uc778 \uc2a4\ud0c0\ub2c8\uc2ac\ub77c\uc2a4 \ubc14\ube0c\ub9b0\uce74\uc640 \ud568\uaed8 \ucd9c\uc804\ud558\uc5ec \uc6b0\uc2b9, \uae08\uba54\ub2ec\uc744 \ud68d\ub4dd\ud588\ub2e4."} {"question": "\ud64d\ucf69\uc774 \uc720\uc5d4\uc758 \uc218\ucd9c\uc785 \uae08\uc9c0 \uc870\uce58\uc5d0 \uc9c1\uba74\ud558\uac8c \ub41c \ub54c\ub294?", "paragraph": "\ud64d\ucf69\uc740 \ube44\uc881\uc740 \ub545\uc73c\ub85c \ucc9c\uc5f0\uc790\uc6d0\uc774 \ubd80\uc871\ud574, \ubb34\uc5ed\uc744 \uc18c\ud640\ud788 \ud560 \uc218 \uc5c6\uc5c8\ub2e4. \uc601\uad6d\uc740 \ud64d\ucf69\uc744 \ud560\uc591 \ubc1b\uc740 \ub4a4 1841\ub144 6\uc6d4 7\uc77c\uc5d0 \uc790\uc720\ud56d\uc774 \ub418\uc5c8\uc74c\uc744 \uc120\ud3ec\ud558\uace0, \uc0c1\uc120 \uc790\uc720 \uc9c4\ucd9c\uc744 \ud5c8\uac00\ud588\ub2e4. \ud64d\ucf69\uc740 \ud3b8\ub9ac\ud55c \uc9c0\ub9ac \uc704\uce58 \ubc0f \uae4a\uc740 \uc218\uc2ec\uacfc \ub113\uc740 \ud56d\uad6c\uc758 \ud574\ud56d\uc744 \uc774\uc6a9\ud574\uc11c, \uc911\uacc4 \ubb34\uc5ed \ubb3c\ub958\uc758 \uba54\uce74\ub85c \ubc1c\uc804\ud558\uc600\ub2e4. 1949\ub144 \uad6d\uacf5 \ub0b4\uc804\uc744 \ud53c\ud574 \uae30\uc220\uacfc \uc790\uae08\uc744 \uac00\uc9c4 \uc911\uad6d\uc778\ub4e4\uc774 \ud64d\ucf69\uc73c\ub85c \uc774\uc8fc\ud558\uc600\ub2e4. \ub300\ud45c\uc801\uc73c\ub85c \ud64d\ucf69 \uc0c1\ud558\uc774 \uc740\ud589\uacfc \uc601\uad6d\uacc4 \uc2a4\ud0e0\ub2e4\ub4dc\ucc28\ud0c0\ub4dc\uc740\ud589\uc774 \uc788\ub2e4. \uace7 \uc774\uc5b4 \ud55c\ubc18\ub3c4\uc5d0\uc11c \ud130\uc9c4 \ud55c\uad6d \uc804\uc7c1\uc5d0 \uc911\ud654\uc778\ubbfc\uacf5\ud654\uad6d\uc774 \uc720\uc5d4\uc5d0 \ub300\uc801\ud558\uc5ec \uc870\uc120\ubbfc\uc8fc\uc8fc\uc758\uc778\ubbfc\uacf5\ud654\uad6d \ud3b8\uc5d0 \uc11c\uc11c \ucc38\uc804\ud588\uae30 \ub54c\ubb38\uc5d0 1950\ub144\ub300 \ucd08 \uc720\uc5d4\uc774 \uc2e4\uc2dc\ud55c \uc218\ucd9c\uc785 \uae08\uc9c0 \uc870\uce58\uc5d0 \uc9c1\uba74\ud558\uac8c \ub418\uc5c8\uace0, \ud64d\ucf69\uc740 \uc911\uacc4\ud56d\uc73c\ub85c \ud070 \ud0c0\uaca9\uc744 \ubc1b\uac8c \ub418\uc5c8\ub2e4. \uc5d4\uc9c0\ub2c8\uc5b4\uc758 \ucc3d\uc758\ub825 \ubc0f \uac70\ub300\ud558\uace0 \uccad\ub834\ud55c \ub178\ub3d9\ub825\uc5d0 \uc758\ud574\uc11c, \ud64d\ucf69\uc740 \uacbd\uacf5\uc5c5\ud615 \uacbd\uc81c\ub85c\uc758 \ubcc0\ud654\uac00 \uc2dc\uc791\ub418\uc5c8\uc73c\uba70 \uacf5\ub2e8\uc774 \uac74\uc124\ub418\uae30 \uc2dc\uc791\ud588\ub2e4. \uac00\uacf5 \uc218\ucd9c \uc81c\ud488\uc73c\ub85c\ub294 \ubc29\uc9c1\ubb3c, \ud50c\ub77c\uc2a4\ud2f1 ,\uac00\ubc1c, \uc2dc\uacc4, \uc804\uc790 \ubd80\uc18d \ub4f1\uc774 \uc788\uc5c8\ub2e4. 1970\ub144\ub300 \ub9d0\uc5d0 \uc774\ub974\ub7ec, \uc911\ud654\uc778\ubbfc\uacf5\ud654\uad6d\uc740 \ub369\uc0e4\uc624\ud551\uc758 \uc9c0\ub3c4 \uc544\ub798 \uac1c\ud601\uac1c\ubc29\uc744 \uc2e4\uc2dc\ud558\uace0, \uc81c\uc870\uc5c5\uc740 \uc911\uad6d \ub300\ub959\uc73c\ub85c \uc774\ub3d9\ud558\uace0, \ud64d\ucf69\uc740 \ub2e4\uc2dc \uc911\uacc4\ud56d\uc73c\ub85c\uc11c\uc758 \uc704\uce58\ub97c \ud68c\ubcf5\ud558\uc600\ub2e4. \uae08\uc735\uc5c5, \uc11c\ube44\uc2a4\uc5c5\uc758 \ube44\uc911\uc774 \uac08\uc218\ub85d \ucee4\uc838 \uac14\uace0, \uad6d\uc81c \uae08\uc735\uc758 \uc911\uc2ec\uc73c\ub85c \ubcc0\ud654\uc5d0 \uc131\uacf5\ud588\ub2e4. \ud64d\ucf69 \uacbd\uc81c\ub294 \ube44\uc57d\uc801\uc73c\ub85c \ubc1c\uc804\ud588\uace0, \ub300\ud55c\ubbfc\uad6d, \uc911\ud654\ubbfc\uad6d, \uc2f1\uac00\ud3ec\ub974\uc640 \ud568\uaed8 \uc544\uc2dc\uc544\uc758 4\ub9c8\ub9ac\uc758 \uc6a9 \uc911 \ud558\ub098\ub85c \uba85\uc131\uacfc \uba85\uc608\ub97c \uc5bb\uc5c8\ub2e4.", "answer": "1950\ub144\ub300 \ucd08", "sentence": "\uace7 \uc774\uc5b4 \ud55c\ubc18\ub3c4\uc5d0\uc11c \ud130\uc9c4 \ud55c\uad6d \uc804\uc7c1\uc5d0 \uc911\ud654\uc778\ubbfc\uacf5\ud654\uad6d\uc774 \uc720\uc5d4\uc5d0 \ub300\uc801\ud558\uc5ec \uc870\uc120\ubbfc\uc8fc\uc8fc\uc758\uc778\ubbfc\uacf5\ud654\uad6d \ud3b8\uc5d0 \uc11c\uc11c \ucc38\uc804\ud588\uae30 \ub54c\ubb38\uc5d0 1950\ub144\ub300 \ucd08 \uc720\uc5d4\uc774 \uc2e4\uc2dc\ud55c \uc218\ucd9c\uc785 \uae08\uc9c0 \uc870\uce58\uc5d0 \uc9c1\uba74\ud558\uac8c \ub418\uc5c8\uace0, \ud64d\ucf69\uc740 \uc911\uacc4\ud56d\uc73c\ub85c \ud070 \ud0c0\uaca9\uc744 \ubc1b\uac8c \ub418\uc5c8\ub2e4.", "paragraph_sentence": "\ud64d\ucf69\uc740 \ube44\uc881\uc740 \ub545\uc73c\ub85c \ucc9c\uc5f0\uc790\uc6d0\uc774 \ubd80\uc871\ud574, \ubb34\uc5ed\uc744 \uc18c\ud640\ud788 \ud560 \uc218 \uc5c6\uc5c8\ub2e4. \uc601\uad6d\uc740 \ud64d\ucf69\uc744 \ud560\uc591 \ubc1b\uc740 \ub4a4 1841\ub144 6\uc6d4 7\uc77c\uc5d0 \uc790\uc720\ud56d\uc774 \ub418\uc5c8\uc74c\uc744 \uc120\ud3ec\ud558\uace0, \uc0c1\uc120 \uc790\uc720 \uc9c4\ucd9c\uc744 \ud5c8\uac00\ud588\ub2e4. \ud64d\ucf69\uc740 \ud3b8\ub9ac\ud55c \uc9c0\ub9ac \uc704\uce58 \ubc0f \uae4a\uc740 \uc218\uc2ec\uacfc \ub113\uc740 \ud56d\uad6c\uc758 \ud574\ud56d\uc744 \uc774\uc6a9\ud574\uc11c, \uc911\uacc4 \ubb34\uc5ed \ubb3c\ub958\uc758 \uba54\uce74\ub85c \ubc1c\uc804\ud558\uc600\ub2e4. 1949\ub144 \uad6d\uacf5 \ub0b4\uc804\uc744 \ud53c\ud574 \uae30\uc220\uacfc \uc790\uae08\uc744 \uac00\uc9c4 \uc911\uad6d\uc778\ub4e4\uc774 \ud64d\ucf69\uc73c\ub85c \uc774\uc8fc\ud558\uc600\ub2e4. \ub300\ud45c\uc801\uc73c\ub85c \ud64d\ucf69 \uc0c1\ud558\uc774 \uc740\ud589\uacfc \uc601\uad6d\uacc4 \uc2a4\ud0e0\ub2e4\ub4dc\ucc28\ud0c0\ub4dc\uc740\ud589\uc774 \uc788\ub2e4. \uace7 \uc774\uc5b4 \ud55c\ubc18\ub3c4\uc5d0\uc11c \ud130\uc9c4 \ud55c\uad6d \uc804\uc7c1\uc5d0 \uc911\ud654\uc778\ubbfc\uacf5\ud654\uad6d\uc774 \uc720\uc5d4\uc5d0 \ub300\uc801\ud558\uc5ec \uc870\uc120\ubbfc\uc8fc\uc8fc\uc758\uc778\ubbfc\uacf5\ud654\uad6d \ud3b8\uc5d0 \uc11c\uc11c \ucc38\uc804\ud588\uae30 \ub54c\ubb38\uc5d0 1950\ub144\ub300 \ucd08 \uc720\uc5d4\uc774 \uc2e4\uc2dc\ud55c \uc218\ucd9c\uc785 \uae08\uc9c0 \uc870\uce58\uc5d0 \uc9c1\uba74\ud558\uac8c \ub418\uc5c8\uace0, \ud64d\ucf69\uc740 \uc911\uacc4\ud56d\uc73c\ub85c \ud070 \ud0c0\uaca9\uc744 \ubc1b\uac8c \ub418\uc5c8\ub2e4. \uc5d4\uc9c0\ub2c8\uc5b4\uc758 \ucc3d\uc758\ub825 \ubc0f \uac70\ub300\ud558\uace0 \uccad\ub834\ud55c \ub178\ub3d9\ub825\uc5d0 \uc758\ud574\uc11c, \ud64d\ucf69\uc740 \uacbd\uacf5\uc5c5\ud615 \uacbd\uc81c\ub85c\uc758 \ubcc0\ud654\uac00 \uc2dc\uc791\ub418\uc5c8\uc73c\uba70 \uacf5\ub2e8\uc774 \uac74\uc124\ub418\uae30 \uc2dc\uc791\ud588\ub2e4. \uac00\uacf5 \uc218\ucd9c \uc81c\ud488\uc73c\ub85c\ub294 \ubc29\uc9c1\ubb3c, \ud50c\ub77c\uc2a4\ud2f1 ,\uac00\ubc1c, \uc2dc\uacc4, \uc804\uc790 \ubd80\uc18d \ub4f1\uc774 \uc788\uc5c8\ub2e4. 1970\ub144\ub300 \ub9d0\uc5d0 \uc774\ub974\ub7ec, \uc911\ud654\uc778\ubbfc\uacf5\ud654\uad6d\uc740 \ub369\uc0e4\uc624\ud551\uc758 \uc9c0\ub3c4 \uc544\ub798 \uac1c\ud601\uac1c\ubc29\uc744 \uc2e4\uc2dc\ud558\uace0, \uc81c\uc870\uc5c5\uc740 \uc911\uad6d \ub300\ub959\uc73c\ub85c \uc774\ub3d9\ud558\uace0, \ud64d\ucf69\uc740 \ub2e4\uc2dc \uc911\uacc4\ud56d\uc73c\ub85c\uc11c\uc758 \uc704\uce58\ub97c \ud68c\ubcf5\ud558\uc600\ub2e4. \uae08\uc735\uc5c5, \uc11c\ube44\uc2a4\uc5c5\uc758 \ube44\uc911\uc774 \uac08\uc218\ub85d \ucee4\uc838 \uac14\uace0, \uad6d\uc81c \uae08\uc735\uc758 \uc911\uc2ec\uc73c\ub85c \ubcc0\ud654\uc5d0 \uc131\uacf5\ud588\ub2e4. \ud64d\ucf69 \uacbd\uc81c\ub294 \ube44\uc57d\uc801\uc73c\ub85c \ubc1c\uc804\ud588\uace0, \ub300\ud55c\ubbfc\uad6d, \uc911\ud654\ubbfc\uad6d, \uc2f1\uac00\ud3ec\ub974\uc640 \ud568\uaed8 \uc544\uc2dc\uc544\uc758 4\ub9c8\ub9ac\uc758 \uc6a9 \uc911 \ud558\ub098\ub85c \uba85\uc131\uacfc \uba85\uc608\ub97c \uc5bb\uc5c8\ub2e4.", "paragraph_answer": "\ud64d\ucf69\uc740 \ube44\uc881\uc740 \ub545\uc73c\ub85c \ucc9c\uc5f0\uc790\uc6d0\uc774 \ubd80\uc871\ud574, \ubb34\uc5ed\uc744 \uc18c\ud640\ud788 \ud560 \uc218 \uc5c6\uc5c8\ub2e4. \uc601\uad6d\uc740 \ud64d\ucf69\uc744 \ud560\uc591 \ubc1b\uc740 \ub4a4 1841\ub144 6\uc6d4 7\uc77c\uc5d0 \uc790\uc720\ud56d\uc774 \ub418\uc5c8\uc74c\uc744 \uc120\ud3ec\ud558\uace0, \uc0c1\uc120 \uc790\uc720 \uc9c4\ucd9c\uc744 \ud5c8\uac00\ud588\ub2e4. \ud64d\ucf69\uc740 \ud3b8\ub9ac\ud55c \uc9c0\ub9ac \uc704\uce58 \ubc0f \uae4a\uc740 \uc218\uc2ec\uacfc \ub113\uc740 \ud56d\uad6c\uc758 \ud574\ud56d\uc744 \uc774\uc6a9\ud574\uc11c, \uc911\uacc4 \ubb34\uc5ed \ubb3c\ub958\uc758 \uba54\uce74\ub85c \ubc1c\uc804\ud558\uc600\ub2e4. 1949\ub144 \uad6d\uacf5 \ub0b4\uc804\uc744 \ud53c\ud574 \uae30\uc220\uacfc \uc790\uae08\uc744 \uac00\uc9c4 \uc911\uad6d\uc778\ub4e4\uc774 \ud64d\ucf69\uc73c\ub85c \uc774\uc8fc\ud558\uc600\ub2e4. \ub300\ud45c\uc801\uc73c\ub85c \ud64d\ucf69 \uc0c1\ud558\uc774 \uc740\ud589\uacfc \uc601\uad6d\uacc4 \uc2a4\ud0e0\ub2e4\ub4dc\ucc28\ud0c0\ub4dc\uc740\ud589\uc774 \uc788\ub2e4. \uace7 \uc774\uc5b4 \ud55c\ubc18\ub3c4\uc5d0\uc11c \ud130\uc9c4 \ud55c\uad6d \uc804\uc7c1\uc5d0 \uc911\ud654\uc778\ubbfc\uacf5\ud654\uad6d\uc774 \uc720\uc5d4\uc5d0 \ub300\uc801\ud558\uc5ec \uc870\uc120\ubbfc\uc8fc\uc8fc\uc758\uc778\ubbfc\uacf5\ud654\uad6d \ud3b8\uc5d0 \uc11c\uc11c \ucc38\uc804\ud588\uae30 \ub54c\ubb38\uc5d0 1950\ub144\ub300 \ucd08 \uc720\uc5d4\uc774 \uc2e4\uc2dc\ud55c \uc218\ucd9c\uc785 \uae08\uc9c0 \uc870\uce58\uc5d0 \uc9c1\uba74\ud558\uac8c \ub418\uc5c8\uace0, \ud64d\ucf69\uc740 \uc911\uacc4\ud56d\uc73c\ub85c \ud070 \ud0c0\uaca9\uc744 \ubc1b\uac8c \ub418\uc5c8\ub2e4. \uc5d4\uc9c0\ub2c8\uc5b4\uc758 \ucc3d\uc758\ub825 \ubc0f \uac70\ub300\ud558\uace0 \uccad\ub834\ud55c \ub178\ub3d9\ub825\uc5d0 \uc758\ud574\uc11c, \ud64d\ucf69\uc740 \uacbd\uacf5\uc5c5\ud615 \uacbd\uc81c\ub85c\uc758 \ubcc0\ud654\uac00 \uc2dc\uc791\ub418\uc5c8\uc73c\uba70 \uacf5\ub2e8\uc774 \uac74\uc124\ub418\uae30 \uc2dc\uc791\ud588\ub2e4. \uac00\uacf5 \uc218\ucd9c \uc81c\ud488\uc73c\ub85c\ub294 \ubc29\uc9c1\ubb3c, \ud50c\ub77c\uc2a4\ud2f1 ,\uac00\ubc1c, \uc2dc\uacc4, \uc804\uc790 \ubd80\uc18d \ub4f1\uc774 \uc788\uc5c8\ub2e4. 1970\ub144\ub300 \ub9d0\uc5d0 \uc774\ub974\ub7ec, \uc911\ud654\uc778\ubbfc\uacf5\ud654\uad6d\uc740 \ub369\uc0e4\uc624\ud551\uc758 \uc9c0\ub3c4 \uc544\ub798 \uac1c\ud601\uac1c\ubc29\uc744 \uc2e4\uc2dc\ud558\uace0, \uc81c\uc870\uc5c5\uc740 \uc911\uad6d \ub300\ub959\uc73c\ub85c \uc774\ub3d9\ud558\uace0, \ud64d\ucf69\uc740 \ub2e4\uc2dc \uc911\uacc4\ud56d\uc73c\ub85c\uc11c\uc758 \uc704\uce58\ub97c \ud68c\ubcf5\ud558\uc600\ub2e4. \uae08\uc735\uc5c5, \uc11c\ube44\uc2a4\uc5c5\uc758 \ube44\uc911\uc774 \uac08\uc218\ub85d \ucee4\uc838 \uac14\uace0, \uad6d\uc81c \uae08\uc735\uc758 \uc911\uc2ec\uc73c\ub85c \ubcc0\ud654\uc5d0 \uc131\uacf5\ud588\ub2e4. \ud64d\ucf69 \uacbd\uc81c\ub294 \ube44\uc57d\uc801\uc73c\ub85c \ubc1c\uc804\ud588\uace0, \ub300\ud55c\ubbfc\uad6d, \uc911\ud654\ubbfc\uad6d, \uc2f1\uac00\ud3ec\ub974\uc640 \ud568\uaed8 \uc544\uc2dc\uc544\uc758 4\ub9c8\ub9ac\uc758 \uc6a9 \uc911 \ud558\ub098\ub85c \uba85\uc131\uacfc \uba85\uc608\ub97c \uc5bb\uc5c8\ub2e4.", "sentence_answer": "\uace7 \uc774\uc5b4 \ud55c\ubc18\ub3c4\uc5d0\uc11c \ud130\uc9c4 \ud55c\uad6d \uc804\uc7c1\uc5d0 \uc911\ud654\uc778\ubbfc\uacf5\ud654\uad6d\uc774 \uc720\uc5d4\uc5d0 \ub300\uc801\ud558\uc5ec \uc870\uc120\ubbfc\uc8fc\uc8fc\uc758\uc778\ubbfc\uacf5\ud654\uad6d \ud3b8\uc5d0 \uc11c\uc11c \ucc38\uc804\ud588\uae30 \ub54c\ubb38\uc5d0 1950\ub144\ub300 \ucd08 \uc720\uc5d4\uc774 \uc2e4\uc2dc\ud55c \uc218\ucd9c\uc785 \uae08\uc9c0 \uc870\uce58\uc5d0 \uc9c1\uba74\ud558\uac8c \ub418\uc5c8\uace0, \ud64d\ucf69\uc740 \uc911\uacc4\ud56d\uc73c\ub85c \ud070 \ud0c0\uaca9\uc744 \ubc1b\uac8c \ub418\uc5c8\ub2e4.", "paragraph_id": "6572248-8-2", "paragraph_question": "question: \ud64d\ucf69\uc774 \uc720\uc5d4\uc758 \uc218\ucd9c\uc785 \uae08\uc9c0 \uc870\uce58\uc5d0 \uc9c1\uba74\ud558\uac8c \ub41c \ub54c\ub294?, context: \ud64d\ucf69\uc740 \ube44\uc881\uc740 \ub545\uc73c\ub85c \ucc9c\uc5f0\uc790\uc6d0\uc774 \ubd80\uc871\ud574, \ubb34\uc5ed\uc744 \uc18c\ud640\ud788 \ud560 \uc218 \uc5c6\uc5c8\ub2e4. \uc601\uad6d\uc740 \ud64d\ucf69\uc744 \ud560\uc591 \ubc1b\uc740 \ub4a4 1841\ub144 6\uc6d4 7\uc77c\uc5d0 \uc790\uc720\ud56d\uc774 \ub418\uc5c8\uc74c\uc744 \uc120\ud3ec\ud558\uace0, \uc0c1\uc120 \uc790\uc720 \uc9c4\ucd9c\uc744 \ud5c8\uac00\ud588\ub2e4. \ud64d\ucf69\uc740 \ud3b8\ub9ac\ud55c \uc9c0\ub9ac \uc704\uce58 \ubc0f \uae4a\uc740 \uc218\uc2ec\uacfc \ub113\uc740 \ud56d\uad6c\uc758 \ud574\ud56d\uc744 \uc774\uc6a9\ud574\uc11c, \uc911\uacc4 \ubb34\uc5ed \ubb3c\ub958\uc758 \uba54\uce74\ub85c \ubc1c\uc804\ud558\uc600\ub2e4. 1949\ub144 \uad6d\uacf5 \ub0b4\uc804\uc744 \ud53c\ud574 \uae30\uc220\uacfc \uc790\uae08\uc744 \uac00\uc9c4 \uc911\uad6d\uc778\ub4e4\uc774 \ud64d\ucf69\uc73c\ub85c \uc774\uc8fc\ud558\uc600\ub2e4. \ub300\ud45c\uc801\uc73c\ub85c \ud64d\ucf69 \uc0c1\ud558\uc774 \uc740\ud589\uacfc \uc601\uad6d\uacc4 \uc2a4\ud0e0\ub2e4\ub4dc\ucc28\ud0c0\ub4dc\uc740\ud589\uc774 \uc788\ub2e4. \uace7 \uc774\uc5b4 \ud55c\ubc18\ub3c4\uc5d0\uc11c \ud130\uc9c4 \ud55c\uad6d \uc804\uc7c1\uc5d0 \uc911\ud654\uc778\ubbfc\uacf5\ud654\uad6d\uc774 \uc720\uc5d4\uc5d0 \ub300\uc801\ud558\uc5ec \uc870\uc120\ubbfc\uc8fc\uc8fc\uc758\uc778\ubbfc\uacf5\ud654\uad6d \ud3b8\uc5d0 \uc11c\uc11c \ucc38\uc804\ud588\uae30 \ub54c\ubb38\uc5d0 1950\ub144\ub300 \ucd08 \uc720\uc5d4\uc774 \uc2e4\uc2dc\ud55c \uc218\ucd9c\uc785 \uae08\uc9c0 \uc870\uce58\uc5d0 \uc9c1\uba74\ud558\uac8c \ub418\uc5c8\uace0, \ud64d\ucf69\uc740 \uc911\uacc4\ud56d\uc73c\ub85c \ud070 \ud0c0\uaca9\uc744 \ubc1b\uac8c \ub418\uc5c8\ub2e4. \uc5d4\uc9c0\ub2c8\uc5b4\uc758 \ucc3d\uc758\ub825 \ubc0f \uac70\ub300\ud558\uace0 \uccad\ub834\ud55c \ub178\ub3d9\ub825\uc5d0 \uc758\ud574\uc11c, \ud64d\ucf69\uc740 \uacbd\uacf5\uc5c5\ud615 \uacbd\uc81c\ub85c\uc758 \ubcc0\ud654\uac00 \uc2dc\uc791\ub418\uc5c8\uc73c\uba70 \uacf5\ub2e8\uc774 \uac74\uc124\ub418\uae30 \uc2dc\uc791\ud588\ub2e4. \uac00\uacf5 \uc218\ucd9c \uc81c\ud488\uc73c\ub85c\ub294 \ubc29\uc9c1\ubb3c, \ud50c\ub77c\uc2a4\ud2f1 ,\uac00\ubc1c, \uc2dc\uacc4, \uc804\uc790 \ubd80\uc18d \ub4f1\uc774 \uc788\uc5c8\ub2e4. 1970\ub144\ub300 \ub9d0\uc5d0 \uc774\ub974\ub7ec, \uc911\ud654\uc778\ubbfc\uacf5\ud654\uad6d\uc740 \ub369\uc0e4\uc624\ud551\uc758 \uc9c0\ub3c4 \uc544\ub798 \uac1c\ud601\uac1c\ubc29\uc744 \uc2e4\uc2dc\ud558\uace0, \uc81c\uc870\uc5c5\uc740 \uc911\uad6d \ub300\ub959\uc73c\ub85c \uc774\ub3d9\ud558\uace0, \ud64d\ucf69\uc740 \ub2e4\uc2dc \uc911\uacc4\ud56d\uc73c\ub85c\uc11c\uc758 \uc704\uce58\ub97c \ud68c\ubcf5\ud558\uc600\ub2e4. \uae08\uc735\uc5c5, \uc11c\ube44\uc2a4\uc5c5\uc758 \ube44\uc911\uc774 \uac08\uc218\ub85d \ucee4\uc838 \uac14\uace0, \uad6d\uc81c \uae08\uc735\uc758 \uc911\uc2ec\uc73c\ub85c \ubcc0\ud654\uc5d0 \uc131\uacf5\ud588\ub2e4. \ud64d\ucf69 \uacbd\uc81c\ub294 \ube44\uc57d\uc801\uc73c\ub85c \ubc1c\uc804\ud588\uace0, \ub300\ud55c\ubbfc\uad6d, \uc911\ud654\ubbfc\uad6d, \uc2f1\uac00\ud3ec\ub974\uc640 \ud568\uaed8 \uc544\uc2dc\uc544\uc758 4\ub9c8\ub9ac\uc758 \uc6a9 \uc911 \ud558\ub098\ub85c \uba85\uc131\uacfc \uba85\uc608\ub97c \uc5bb\uc5c8\ub2e4."} {"question": "1897\ub144 \ud669\uc81c\uc5d0 \uc624\ub978 \uc0ac\ub78c\uc740?", "paragraph": "\uc774\uc5b4 1897\ub144(\uad11\ubb34 \uc6d0\ub144), \uad6d\ud638\ub97c \ub300\ud55c\uc81c\uad6d\uc73c\ub85c \uc120\ud3ec\ud558\uace0 \ud669\uc81c\uc5d0 \uc624\ub978 \uace0\uc885\ud669\uc81c\ub294 \ud669\uac00\uc758 \uc815\ud1b5\uc131\uc744 \uc704\ud574 \ud669\uac00\uc758 \uc9c1\uacc4 \uc870\uc0c1\uc778 \uc7a5\uc885, \uc815\uc885, \uc21c\uc870, \uc775\uc885\uc744 \ubaa8\ub450 \ud669\uc81c\ub85c \ub192\uc774\uace0, \uc7a5\uc885, \uc815\uc885, \uc775\uc885\uc758 \ubb18\ud638 \ub610\ud55c \ub192\uc5ec \uac01\uac01 \uc7a5\uc870, \uc815\uc870, \ubb38\uc870\ub85c \uace0\ucce4\ub2e4. \ub610\ud55c 1908\ub144(\uc735\ud76c 2\ub144)\uc5d0 \uc21c\uc885\ud669\uc81c\ub294 \ud669\uc81c\uad6d\uc758 \uc885\ubb18 \uc81c\ub3c4\uc778 \uce60\ubb18\uc81c\uc5d0 \ub530\ub77c \uc9c4\uc885, \ud5cc\uc885, \ucca0\uc885\ub3c4 \ubaa8\ub450 \ud669\uc81c\ub85c \ucd94\uc22d\ud558\uc600\ub2e4. 1910\ub144(\uc735\ud76c 4\ub144), \ud55c\uc77c\ud569\ubcd1\uc870\uc57d\uc73c\ub85c \uc778\ud574 \ub300\ud55c\uc81c\uad6d\uc774 \uba78\ub9dd\ud55c \ud6c4 \uc774\ubbf8 \ubb3c\ub7ec\ub098\uc788\ub358 \uace0\uc885\uacfc \uc21c\uc885\uc740 \uac01\uac01 \ub355\uc218\uad81 \uc774\ud0dc\uc655, \ucc3d\ub355\uad81 \uc774\uc655 \ub4f1\uc73c\ub85c \uaca9\ud558\ub418\uc5c8\ub2e4. 1919\ub144\uc5d0 \uace0\uc885\uc774, 1926\ub144\uc5d0 \uc21c\uc885\uc774 \uc138\uc0c1\uc744 \ub5a0\ub0a0 \ub54c, \ubcf8\ub798 \ub9dd\uad6d\uc758 \uad70\uc8fc\uc5d0\uac8c \ubb18\ud638\ub97c \ubd99\uc774\ub294 \uc77c\uc740 \ub4dc\ubb3c\uc5c8\uc73c\ub098 \uc774\uc655\uc9c1\uacfc \uc870\uc120\ucd1d\ub3c5\ubd80\uc5d0\uc11c \uc870\uc120 \uc655\uc2e4\uc758 \uc608\uc640 \ud615\uc2dd\uc5d0 \ub530\ub77c \ubb18\ud638\ub97c \uc62c\ub9ac\uae30\ub3c4 \ud588\ub2e4.", "answer": "\uace0\uc885\ud669\uc81c", "sentence": "\uc774\uc5b4 1897\ub144(\uad11\ubb34 \uc6d0\ub144), \uad6d\ud638\ub97c \ub300\ud55c\uc81c\uad6d\uc73c\ub85c \uc120\ud3ec\ud558\uace0 \ud669\uc81c\uc5d0 \uc624\ub978 \uace0\uc885\ud669\uc81c \ub294 \ud669\uac00\uc758 \uc815\ud1b5\uc131\uc744 \uc704\ud574 \ud669\uac00\uc758 \uc9c1\uacc4 \uc870\uc0c1\uc778 \uc7a5\uc885, \uc815\uc885, \uc21c\uc870, \uc775\uc885\uc744 \ubaa8\ub450 \ud669\uc81c\ub85c \ub192\uc774\uace0, \uc7a5\uc885, \uc815\uc885, \uc775\uc885\uc758 \ubb18\ud638 \ub610\ud55c \ub192\uc5ec \uac01\uac01 \uc7a5\uc870, \uc815\uc870, \ubb38\uc870\ub85c \uace0\ucce4\ub2e4.", "paragraph_sentence": " \uc774\uc5b4 1897\ub144(\uad11\ubb34 \uc6d0\ub144), \uad6d\ud638\ub97c \ub300\ud55c\uc81c\uad6d\uc73c\ub85c \uc120\ud3ec\ud558\uace0 \ud669\uc81c\uc5d0 \uc624\ub978 \uace0\uc885\ud669\uc81c \ub294 \ud669\uac00\uc758 \uc815\ud1b5\uc131\uc744 \uc704\ud574 \ud669\uac00\uc758 \uc9c1\uacc4 \uc870\uc0c1\uc778 \uc7a5\uc885, \uc815\uc885, \uc21c\uc870, \uc775\uc885\uc744 \ubaa8\ub450 \ud669\uc81c\ub85c \ub192\uc774\uace0, \uc7a5\uc885, \uc815\uc885, \uc775\uc885\uc758 \ubb18\ud638 \ub610\ud55c \ub192\uc5ec \uac01\uac01 \uc7a5\uc870, \uc815\uc870, \ubb38\uc870\ub85c \uace0\ucce4\ub2e4. \ub610\ud55c 1908\ub144(\uc735\ud76c 2\ub144)\uc5d0 \uc21c\uc885\ud669\uc81c\ub294 \ud669\uc81c\uad6d\uc758 \uc885\ubb18 \uc81c\ub3c4\uc778 \uce60\ubb18\uc81c\uc5d0 \ub530\ub77c \uc9c4\uc885, \ud5cc\uc885, \ucca0\uc885\ub3c4 \ubaa8\ub450 \ud669\uc81c\ub85c \ucd94\uc22d\ud558\uc600\ub2e4. 1910\ub144(\uc735\ud76c 4\ub144), \ud55c\uc77c\ud569\ubcd1\uc870\uc57d\uc73c\ub85c \uc778\ud574 \ub300\ud55c\uc81c\uad6d\uc774 \uba78\ub9dd\ud55c \ud6c4 \uc774\ubbf8 \ubb3c\ub7ec\ub098\uc788\ub358 \uace0\uc885\uacfc \uc21c\uc885\uc740 \uac01\uac01 \ub355\uc218\uad81 \uc774\ud0dc\uc655, \ucc3d\ub355\uad81 \uc774\uc655 \ub4f1\uc73c\ub85c \uaca9\ud558\ub418\uc5c8\ub2e4. 1919\ub144\uc5d0 \uace0\uc885\uc774, 1926\ub144\uc5d0 \uc21c\uc885\uc774 \uc138\uc0c1\uc744 \ub5a0\ub0a0 \ub54c, \ubcf8\ub798 \ub9dd\uad6d\uc758 \uad70\uc8fc\uc5d0\uac8c \ubb18\ud638\ub97c \ubd99\uc774\ub294 \uc77c\uc740 \ub4dc\ubb3c\uc5c8\uc73c\ub098 \uc774\uc655\uc9c1\uacfc \uc870\uc120\ucd1d\ub3c5\ubd80\uc5d0\uc11c \uc870\uc120 \uc655\uc2e4\uc758 \uc608\uc640 \ud615\uc2dd\uc5d0 \ub530\ub77c \ubb18\ud638\ub97c \uc62c\ub9ac\uae30\ub3c4 \ud588\ub2e4.", "paragraph_answer": "\uc774\uc5b4 1897\ub144(\uad11\ubb34 \uc6d0\ub144), \uad6d\ud638\ub97c \ub300\ud55c\uc81c\uad6d\uc73c\ub85c \uc120\ud3ec\ud558\uace0 \ud669\uc81c\uc5d0 \uc624\ub978 \uace0\uc885\ud669\uc81c \ub294 \ud669\uac00\uc758 \uc815\ud1b5\uc131\uc744 \uc704\ud574 \ud669\uac00\uc758 \uc9c1\uacc4 \uc870\uc0c1\uc778 \uc7a5\uc885, \uc815\uc885, \uc21c\uc870, \uc775\uc885\uc744 \ubaa8\ub450 \ud669\uc81c\ub85c \ub192\uc774\uace0, \uc7a5\uc885, \uc815\uc885, \uc775\uc885\uc758 \ubb18\ud638 \ub610\ud55c \ub192\uc5ec \uac01\uac01 \uc7a5\uc870, \uc815\uc870, \ubb38\uc870\ub85c \uace0\ucce4\ub2e4. \ub610\ud55c 1908\ub144(\uc735\ud76c 2\ub144)\uc5d0 \uc21c\uc885\ud669\uc81c\ub294 \ud669\uc81c\uad6d\uc758 \uc885\ubb18 \uc81c\ub3c4\uc778 \uce60\ubb18\uc81c\uc5d0 \ub530\ub77c \uc9c4\uc885, \ud5cc\uc885, \ucca0\uc885\ub3c4 \ubaa8\ub450 \ud669\uc81c\ub85c \ucd94\uc22d\ud558\uc600\ub2e4. 1910\ub144(\uc735\ud76c 4\ub144), \ud55c\uc77c\ud569\ubcd1\uc870\uc57d\uc73c\ub85c \uc778\ud574 \ub300\ud55c\uc81c\uad6d\uc774 \uba78\ub9dd\ud55c \ud6c4 \uc774\ubbf8 \ubb3c\ub7ec\ub098\uc788\ub358 \uace0\uc885\uacfc \uc21c\uc885\uc740 \uac01\uac01 \ub355\uc218\uad81 \uc774\ud0dc\uc655, \ucc3d\ub355\uad81 \uc774\uc655 \ub4f1\uc73c\ub85c \uaca9\ud558\ub418\uc5c8\ub2e4. 1919\ub144\uc5d0 \uace0\uc885\uc774, 1926\ub144\uc5d0 \uc21c\uc885\uc774 \uc138\uc0c1\uc744 \ub5a0\ub0a0 \ub54c, \ubcf8\ub798 \ub9dd\uad6d\uc758 \uad70\uc8fc\uc5d0\uac8c \ubb18\ud638\ub97c \ubd99\uc774\ub294 \uc77c\uc740 \ub4dc\ubb3c\uc5c8\uc73c\ub098 \uc774\uc655\uc9c1\uacfc \uc870\uc120\ucd1d\ub3c5\ubd80\uc5d0\uc11c \uc870\uc120 \uc655\uc2e4\uc758 \uc608\uc640 \ud615\uc2dd\uc5d0 \ub530\ub77c \ubb18\ud638\ub97c \uc62c\ub9ac\uae30\ub3c4 \ud588\ub2e4.", "sentence_answer": "\uc774\uc5b4 1897\ub144(\uad11\ubb34 \uc6d0\ub144), \uad6d\ud638\ub97c \ub300\ud55c\uc81c\uad6d\uc73c\ub85c \uc120\ud3ec\ud558\uace0 \ud669\uc81c\uc5d0 \uc624\ub978 \uace0\uc885\ud669\uc81c \ub294 \ud669\uac00\uc758 \uc815\ud1b5\uc131\uc744 \uc704\ud574 \ud669\uac00\uc758 \uc9c1\uacc4 \uc870\uc0c1\uc778 \uc7a5\uc885, \uc815\uc885, \uc21c\uc870, \uc775\uc885\uc744 \ubaa8\ub450 \ud669\uc81c\ub85c \ub192\uc774\uace0, \uc7a5\uc885, \uc815\uc885, \uc775\uc885\uc758 \ubb18\ud638 \ub610\ud55c \ub192\uc5ec \uac01\uac01 \uc7a5\uc870, \uc815\uc870, \ubb38\uc870\ub85c \uace0\ucce4\ub2e4.", "paragraph_id": "6657037-11-1", "paragraph_question": "question: 1897\ub144 \ud669\uc81c\uc5d0 \uc624\ub978 \uc0ac\ub78c\uc740?, context: \uc774\uc5b4 1897\ub144(\uad11\ubb34 \uc6d0\ub144), \uad6d\ud638\ub97c \ub300\ud55c\uc81c\uad6d\uc73c\ub85c \uc120\ud3ec\ud558\uace0 \ud669\uc81c\uc5d0 \uc624\ub978 \uace0\uc885\ud669\uc81c\ub294 \ud669\uac00\uc758 \uc815\ud1b5\uc131\uc744 \uc704\ud574 \ud669\uac00\uc758 \uc9c1\uacc4 \uc870\uc0c1\uc778 \uc7a5\uc885, \uc815\uc885, \uc21c\uc870, \uc775\uc885\uc744 \ubaa8\ub450 \ud669\uc81c\ub85c \ub192\uc774\uace0, \uc7a5\uc885, \uc815\uc885, \uc775\uc885\uc758 \ubb18\ud638 \ub610\ud55c \ub192\uc5ec \uac01\uac01 \uc7a5\uc870, \uc815\uc870, \ubb38\uc870\ub85c \uace0\ucce4\ub2e4. \ub610\ud55c 1908\ub144(\uc735\ud76c 2\ub144)\uc5d0 \uc21c\uc885\ud669\uc81c\ub294 \ud669\uc81c\uad6d\uc758 \uc885\ubb18 \uc81c\ub3c4\uc778 \uce60\ubb18\uc81c\uc5d0 \ub530\ub77c \uc9c4\uc885, \ud5cc\uc885, \ucca0\uc885\ub3c4 \ubaa8\ub450 \ud669\uc81c\ub85c \ucd94\uc22d\ud558\uc600\ub2e4. 1910\ub144(\uc735\ud76c 4\ub144), \ud55c\uc77c\ud569\ubcd1\uc870\uc57d\uc73c\ub85c \uc778\ud574 \ub300\ud55c\uc81c\uad6d\uc774 \uba78\ub9dd\ud55c \ud6c4 \uc774\ubbf8 \ubb3c\ub7ec\ub098\uc788\ub358 \uace0\uc885\uacfc \uc21c\uc885\uc740 \uac01\uac01 \ub355\uc218\uad81 \uc774\ud0dc\uc655, \ucc3d\ub355\uad81 \uc774\uc655 \ub4f1\uc73c\ub85c \uaca9\ud558\ub418\uc5c8\ub2e4. 1919\ub144\uc5d0 \uace0\uc885\uc774, 1926\ub144\uc5d0 \uc21c\uc885\uc774 \uc138\uc0c1\uc744 \ub5a0\ub0a0 \ub54c, \ubcf8\ub798 \ub9dd\uad6d\uc758 \uad70\uc8fc\uc5d0\uac8c \ubb18\ud638\ub97c \ubd99\uc774\ub294 \uc77c\uc740 \ub4dc\ubb3c\uc5c8\uc73c\ub098 \uc774\uc655\uc9c1\uacfc \uc870\uc120\ucd1d\ub3c5\ubd80\uc5d0\uc11c \uc870\uc120 \uc655\uc2e4\uc758 \uc608\uc640 \ud615\uc2dd\uc5d0 \ub530\ub77c \ubb18\ud638\ub97c \uc62c\ub9ac\uae30\ub3c4 \ud588\ub2e4."} {"question": "\ubaa8\ub4e0 \uad6c\uba4d \ubc30\uc5f4\uc774 \uc720,\ubb34\ud55c \ubd80\ubd84\uc5d0\uc11c \uaf2d \ud544\uc694\ub85c \ud558\uc9c0 \uc54a\ub294 \uac83\uc740?", "paragraph": "\ubaa8\ub4e0 \uad6c\uba4d \ubc30\uc5f4\uc774 \ubc18\ub4dc\uc2dc \uc720\ud55c\ud55c \ubd80\ubd84(s=0\uc77c \ub54c \uadf9\uc810\uc758 \uacb0\uc5ec)\uc774\ub098 \ubb34\ud55c\ud55c \ubd80\ubd84\uc758 \ubaa8\uc591\ub3c5\ub9bd\uc131\uc744 \ud544\uc694\ub85c \ud558\uc9c0 \uc54a\ub294\ub2e4. \uc774 \uacbd\uc6b0 \ucd94\uac00\uc801\uc778 \ubb3c\ub9ac\uc801 \uc0ac\ud56d\uc774 \uace0\ub824\ub418\uc5b4 \uc774\ud574\ub420 \uc218 \uc788\uc5b4\uc57c\ud55c\ub2e4. \ud2b9\ud788, \uadf9\ub3c4\ub85c \ud070 \uc9c4\ub3d9\uc218\uc5d0\uc11c(\ud50c\ub77c\uc988\ub9c8 \uc9c4\ub3d9\ubcf4\ub2e4 \ub354 \ud07c.) \uae08\uc18d\uc740 \ud22c\uba85\ud55c \uad11\uc790\uac00 \ub418\uba70(\uc608\ub97c \ub4e4\uc5b4 X\uc120), \uc720\uc804\uccb4 \ub610\ud55c \uc9c4\ub3d9\uc218 \uc758\uc874\uc131\uc744 \ucc28\ub2e8\ud55c\ub2e4. \uc774 \uc8fc\ud30c\uc218 \uc758\uc874\uc131\uc740 \uc790\uc5f0\uc801\uc778 \uc870\uc808\uc790\ub85c \ud589\ub3d9\ud55c\ub2e4. \uace0\uccb4\ubb3c\ub9ac\ud559\uc758 \ub2e4\uc591\ud55c \uc885\ub958\uc758 \ubd80\ud53c \ud6a8\uacfc\uac00 \ucc28\ub2e8 \uc9c4\ub3d9\uc218\uac00 \uba85\ubc31\ud558\uac8c \uc774 \ud45c\ud604\uc744 \uc720\ud55c\ud558\uac8c \ud558\ub294 \uac83\ucc98\ub7fc \uc791\uc6a9\ud558\ub294 \uc810\uc5d0\uc11c \uc218\ud559\uc801\uc73c\ub85c \uce74\uc2dc\ubbf8\ub974 \ud6a8\uacfc\uc640 \ub9e4\uc6b0 \ube44\uc2b7\ud558\ub2e4.(\uc5ec\uae30\uc5d0 \ub300\ud55c \uc790\uc138\ud55c \ub17c\uc758\ub294 \ub780\ub2e4\uc6b0\uc640 \ub9bd\uc2dc\uce20\uc758 \"Theory of Continuous Media\" \ucc38\uc870.)", "answer": "\ubaa8\uc591\ub3c5\ub9bd\uc131", "sentence": "\ubaa8\ub4e0 \uad6c\uba4d \ubc30\uc5f4\uc774 \ubc18\ub4dc\uc2dc \uc720\ud55c\ud55c \ubd80\ubd84(s=0\uc77c \ub54c \uadf9\uc810\uc758 \uacb0\uc5ec)\uc774\ub098 \ubb34\ud55c\ud55c \ubd80\ubd84\uc758 \ubaa8\uc591\ub3c5\ub9bd\uc131 \uc744 \ud544\uc694\ub85c \ud558\uc9c0 \uc54a\ub294\ub2e4.", "paragraph_sentence": " \ubaa8\ub4e0 \uad6c\uba4d \ubc30\uc5f4\uc774 \ubc18\ub4dc\uc2dc \uc720\ud55c\ud55c \ubd80\ubd84(s=0\uc77c \ub54c \uadf9\uc810\uc758 \uacb0\uc5ec)\uc774\ub098 \ubb34\ud55c\ud55c \ubd80\ubd84\uc758 \ubaa8\uc591\ub3c5\ub9bd\uc131 \uc744 \ud544\uc694\ub85c \ud558\uc9c0 \uc54a\ub294\ub2e4. \uc774 \uacbd\uc6b0 \ucd94\uac00\uc801\uc778 \ubb3c\ub9ac\uc801 \uc0ac\ud56d\uc774 \uace0\ub824\ub418\uc5b4 \uc774\ud574\ub420 \uc218 \uc788\uc5b4\uc57c\ud55c\ub2e4. \ud2b9\ud788, \uadf9\ub3c4\ub85c \ud070 \uc9c4\ub3d9\uc218\uc5d0\uc11c(\ud50c\ub77c\uc988\ub9c8 \uc9c4\ub3d9\ubcf4\ub2e4 \ub354 \ud07c. ) \uae08\uc18d\uc740 \ud22c\uba85\ud55c \uad11\uc790\uac00 \ub418\uba70(\uc608\ub97c \ub4e4\uc5b4 X\uc120), \uc720\uc804\uccb4 \ub610\ud55c \uc9c4\ub3d9\uc218 \uc758\uc874\uc131\uc744 \ucc28\ub2e8\ud55c\ub2e4. \uc774 \uc8fc\ud30c\uc218 \uc758\uc874\uc131\uc740 \uc790\uc5f0\uc801\uc778 \uc870\uc808\uc790\ub85c \ud589\ub3d9\ud55c\ub2e4. \uace0\uccb4\ubb3c\ub9ac\ud559\uc758 \ub2e4\uc591\ud55c \uc885\ub958\uc758 \ubd80\ud53c \ud6a8\uacfc\uac00 \ucc28\ub2e8 \uc9c4\ub3d9\uc218\uac00 \uba85\ubc31\ud558\uac8c \uc774 \ud45c\ud604\uc744 \uc720\ud55c\ud558\uac8c \ud558\ub294 \uac83\ucc98\ub7fc \uc791\uc6a9\ud558\ub294 \uc810\uc5d0\uc11c \uc218\ud559\uc801\uc73c\ub85c \uce74\uc2dc\ubbf8\ub974 \ud6a8\uacfc\uc640 \ub9e4\uc6b0 \ube44\uc2b7\ud558\ub2e4.(\uc5ec\uae30\uc5d0 \ub300\ud55c \uc790\uc138\ud55c \ub17c\uc758\ub294 \ub780\ub2e4\uc6b0\uc640 \ub9bd\uc2dc\uce20\uc758 \"Theory of Continuous Media\" \ucc38\uc870.)", "paragraph_answer": "\ubaa8\ub4e0 \uad6c\uba4d \ubc30\uc5f4\uc774 \ubc18\ub4dc\uc2dc \uc720\ud55c\ud55c \ubd80\ubd84(s=0\uc77c \ub54c \uadf9\uc810\uc758 \uacb0\uc5ec)\uc774\ub098 \ubb34\ud55c\ud55c \ubd80\ubd84\uc758 \ubaa8\uc591\ub3c5\ub9bd\uc131 \uc744 \ud544\uc694\ub85c \ud558\uc9c0 \uc54a\ub294\ub2e4. \uc774 \uacbd\uc6b0 \ucd94\uac00\uc801\uc778 \ubb3c\ub9ac\uc801 \uc0ac\ud56d\uc774 \uace0\ub824\ub418\uc5b4 \uc774\ud574\ub420 \uc218 \uc788\uc5b4\uc57c\ud55c\ub2e4. \ud2b9\ud788, \uadf9\ub3c4\ub85c \ud070 \uc9c4\ub3d9\uc218\uc5d0\uc11c(\ud50c\ub77c\uc988\ub9c8 \uc9c4\ub3d9\ubcf4\ub2e4 \ub354 \ud07c.) \uae08\uc18d\uc740 \ud22c\uba85\ud55c \uad11\uc790\uac00 \ub418\uba70(\uc608\ub97c \ub4e4\uc5b4 X\uc120), \uc720\uc804\uccb4 \ub610\ud55c \uc9c4\ub3d9\uc218 \uc758\uc874\uc131\uc744 \ucc28\ub2e8\ud55c\ub2e4. \uc774 \uc8fc\ud30c\uc218 \uc758\uc874\uc131\uc740 \uc790\uc5f0\uc801\uc778 \uc870\uc808\uc790\ub85c \ud589\ub3d9\ud55c\ub2e4. \uace0\uccb4\ubb3c\ub9ac\ud559\uc758 \ub2e4\uc591\ud55c \uc885\ub958\uc758 \ubd80\ud53c \ud6a8\uacfc\uac00 \ucc28\ub2e8 \uc9c4\ub3d9\uc218\uac00 \uba85\ubc31\ud558\uac8c \uc774 \ud45c\ud604\uc744 \uc720\ud55c\ud558\uac8c \ud558\ub294 \uac83\ucc98\ub7fc \uc791\uc6a9\ud558\ub294 \uc810\uc5d0\uc11c \uc218\ud559\uc801\uc73c\ub85c \uce74\uc2dc\ubbf8\ub974 \ud6a8\uacfc\uc640 \ub9e4\uc6b0 \ube44\uc2b7\ud558\ub2e4.(\uc5ec\uae30\uc5d0 \ub300\ud55c \uc790\uc138\ud55c \ub17c\uc758\ub294 \ub780\ub2e4\uc6b0\uc640 \ub9bd\uc2dc\uce20\uc758 \"Theory of Continuous Media\" \ucc38\uc870.)", "sentence_answer": "\ubaa8\ub4e0 \uad6c\uba4d \ubc30\uc5f4\uc774 \ubc18\ub4dc\uc2dc \uc720\ud55c\ud55c \ubd80\ubd84(s=0\uc77c \ub54c \uadf9\uc810\uc758 \uacb0\uc5ec)\uc774\ub098 \ubb34\ud55c\ud55c \ubd80\ubd84\uc758 \ubaa8\uc591\ub3c5\ub9bd\uc131 \uc744 \ud544\uc694\ub85c \ud558\uc9c0 \uc54a\ub294\ub2e4.", "paragraph_id": "6561276-3-0", "paragraph_question": "question: \ubaa8\ub4e0 \uad6c\uba4d \ubc30\uc5f4\uc774 \uc720,\ubb34\ud55c \ubd80\ubd84\uc5d0\uc11c \uaf2d \ud544\uc694\ub85c \ud558\uc9c0 \uc54a\ub294 \uac83\uc740?, context: \ubaa8\ub4e0 \uad6c\uba4d \ubc30\uc5f4\uc774 \ubc18\ub4dc\uc2dc \uc720\ud55c\ud55c \ubd80\ubd84(s=0\uc77c \ub54c \uadf9\uc810\uc758 \uacb0\uc5ec)\uc774\ub098 \ubb34\ud55c\ud55c \ubd80\ubd84\uc758 \ubaa8\uc591\ub3c5\ub9bd\uc131\uc744 \ud544\uc694\ub85c \ud558\uc9c0 \uc54a\ub294\ub2e4. \uc774 \uacbd\uc6b0 \ucd94\uac00\uc801\uc778 \ubb3c\ub9ac\uc801 \uc0ac\ud56d\uc774 \uace0\ub824\ub418\uc5b4 \uc774\ud574\ub420 \uc218 \uc788\uc5b4\uc57c\ud55c\ub2e4. \ud2b9\ud788, \uadf9\ub3c4\ub85c \ud070 \uc9c4\ub3d9\uc218\uc5d0\uc11c(\ud50c\ub77c\uc988\ub9c8 \uc9c4\ub3d9\ubcf4\ub2e4 \ub354 \ud07c.) \uae08\uc18d\uc740 \ud22c\uba85\ud55c \uad11\uc790\uac00 \ub418\uba70(\uc608\ub97c \ub4e4\uc5b4 X\uc120), \uc720\uc804\uccb4 \ub610\ud55c \uc9c4\ub3d9\uc218 \uc758\uc874\uc131\uc744 \ucc28\ub2e8\ud55c\ub2e4. \uc774 \uc8fc\ud30c\uc218 \uc758\uc874\uc131\uc740 \uc790\uc5f0\uc801\uc778 \uc870\uc808\uc790\ub85c \ud589\ub3d9\ud55c\ub2e4. \uace0\uccb4\ubb3c\ub9ac\ud559\uc758 \ub2e4\uc591\ud55c \uc885\ub958\uc758 \ubd80\ud53c \ud6a8\uacfc\uac00 \ucc28\ub2e8 \uc9c4\ub3d9\uc218\uac00 \uba85\ubc31\ud558\uac8c \uc774 \ud45c\ud604\uc744 \uc720\ud55c\ud558\uac8c \ud558\ub294 \uac83\ucc98\ub7fc \uc791\uc6a9\ud558\ub294 \uc810\uc5d0\uc11c \uc218\ud559\uc801\uc73c\ub85c \uce74\uc2dc\ubbf8\ub974 \ud6a8\uacfc\uc640 \ub9e4\uc6b0 \ube44\uc2b7\ud558\ub2e4.(\uc5ec\uae30\uc5d0 \ub300\ud55c \uc790\uc138\ud55c \ub17c\uc758\ub294 \ub780\ub2e4\uc6b0\uc640 \ub9bd\uc2dc\uce20\uc758 \"Theory of Continuous Media\" \ucc38\uc870.)"} {"question": "\uc9c4\ubcf4\uc815\ub2f9\uc774 \ucc3d\ub2f9\ub41c \ub54c\ub294 \uc5b8\uc81c\uc778\uac00?", "paragraph": "2017\ub144 10\uc6d4 15\uc77c\uc5d0 \ucc3d\ub2f9\ub41c \ub300\ud55c\ubbfc\uad6d\uc758 \uc9c4\ubcf4\uc815\ub2f9\uc774\ub2e4. 2017\ub144 10\uc6d4 15\uc77c \uc624\uc804 10\uc2dc \uad6d\ud68c\uc758\uc6d0\ud68c\uad00 \uc18c\ud68c\uc758\uc2e4\uc5d0\uc11c \uc0c8\ubbfc\uc911\uc815\ub2f9\uacfc \ubbfc\uc911\uc5f0\ud569\ub2f9 \uc591\ub2f9\uc758 \uc218\uc784\uae30\uad00 \ud569\ub3d9\ud68c\uc758\ub97c \ud1b5\ud574 \ud569\ub2f9\uc744 \uacf5\uc2dd \uc644\ub8cc\ud558\uace0, \uc624\ud6c4 2\uc2dc \uc11c\uc6b8\uc2dc\uccad \uc55e \uad11\uc7a5\uc5d0\uc11c '\ube44\uc815\uaddc\uc9c1 \uc5c6\ub294 \uc138\uc0c1, \uc804\uc7c1 \uc5c6\ub294 \ub098\ub77c', '\ubbfc\uc911\uc5d0\uac8c \uad8c\ub825\uc744, \ubbfc\uc911\uc5d0\uac8c \ud589\ubcf5'\uc744 \ub300\ud68c \uc2ac\ub85c\uac74\uc73c\ub85c \ub0b4\uac78\uace0 1\ub9cc\uc5ec \uba85\uc758 \uc804\uad6d \ub2f9\uc6d0\uacfc \ud568\uaed8 \ubbfc\uc911\ub2f9 \ucc3d\ub2f9\uc744 \uc120\uc5b8\ud558\uba70 \"\ucd1b\ubd88\ud601\uba85\uc758 \uacc4\uc2b9, \uc790\uc8fc\uc640 \ud3c9\ud654, \ube44\uc815\uaddc\uc9c1 \ucca0\ud3d0\ub97c \ud5a5\ud55c \uc9c4\ubcf4 \ub2e8\uacb0\uc758 \ub300\uc7a5\uc815\uc744 \uc2dc\uc791\ud55c\ub2e4.\"\uace0 \ud558\uba74\uc11c \"\ub178\ub3d9\uc790, \ub18d\ubbfc, \ube48\ubbfc, \uccad\ub144 \uc5ec\uc131\ub4e4\uc758 \uc9c1\uc811 \uc815\uce58 \uc6b4\ub3d9\uc758 \ud655\ub300\ub85c \uc0c8\ub85c\uc6b4 \uc815\uce58 \ubc14\ub78c\uc744 \ubd88\ub7ec\uc77c\uc73c\ud0ac \uac83\"\uc774\ub77c\uace0 \ubc1d\ud788\ub294 \uad11\uc7a5 \ucd9c\ubc94\uc2dd\uc744 \uac1c\ucd5c\ud558\uc600\ub2e4.", "answer": "2017\ub144 10\uc6d4 15\uc77c", "sentence": "2017\ub144 10\uc6d4 15\uc77c \uc5d0", "paragraph_sentence": " 2017\ub144 10\uc6d4 15\uc77c \uc5d0 \ucc3d\ub2f9\ub41c \ub300\ud55c\ubbfc\uad6d\uc758 \uc9c4\ubcf4\uc815\ub2f9\uc774\ub2e4. 2017\ub144 10\uc6d4 15\uc77c \uc624\uc804 10\uc2dc \uad6d\ud68c\uc758\uc6d0\ud68c\uad00 \uc18c\ud68c\uc758\uc2e4\uc5d0\uc11c \uc0c8\ubbfc\uc911\uc815\ub2f9\uacfc \ubbfc\uc911\uc5f0\ud569\ub2f9 \uc591\ub2f9\uc758 \uc218\uc784\uae30\uad00 \ud569\ub3d9\ud68c\uc758\ub97c \ud1b5\ud574 \ud569\ub2f9\uc744 \uacf5\uc2dd \uc644\ub8cc\ud558\uace0, \uc624\ud6c4 2\uc2dc \uc11c\uc6b8\uc2dc\uccad \uc55e \uad11\uc7a5\uc5d0\uc11c '\ube44\uc815\uaddc\uc9c1 \uc5c6\ub294 \uc138\uc0c1, \uc804\uc7c1 \uc5c6\ub294 \ub098\ub77c', '\ubbfc\uc911\uc5d0\uac8c \uad8c\ub825\uc744, \ubbfc\uc911\uc5d0\uac8c \ud589\ubcf5'\uc744 \ub300\ud68c \uc2ac\ub85c\uac74\uc73c\ub85c \ub0b4\uac78\uace0 1\ub9cc\uc5ec \uba85\uc758 \uc804\uad6d \ub2f9\uc6d0\uacfc \ud568\uaed8 \ubbfc\uc911\ub2f9 \ucc3d\ub2f9\uc744 \uc120\uc5b8\ud558\uba70 \"\ucd1b\ubd88\ud601\uba85\uc758 \uacc4\uc2b9, \uc790\uc8fc\uc640 \ud3c9\ud654, \ube44\uc815\uaddc\uc9c1 \ucca0\ud3d0\ub97c \ud5a5\ud55c \uc9c4\ubcf4 \ub2e8\uacb0\uc758 \ub300\uc7a5\uc815\uc744 \uc2dc\uc791\ud55c\ub2e4. \"\uace0 \ud558\uba74\uc11c \"\ub178\ub3d9\uc790, \ub18d\ubbfc, \ube48\ubbfc, \uccad\ub144 \uc5ec\uc131\ub4e4\uc758 \uc9c1\uc811 \uc815\uce58 \uc6b4\ub3d9\uc758 \ud655\ub300\ub85c \uc0c8\ub85c\uc6b4 \uc815\uce58 \ubc14\ub78c\uc744 \ubd88\ub7ec\uc77c\uc73c\ud0ac \uac83\"\uc774\ub77c\uace0 \ubc1d\ud788\ub294 \uad11\uc7a5 \ucd9c\ubc94\uc2dd\uc744 \uac1c\ucd5c\ud558\uc600\ub2e4.", "paragraph_answer": " 2017\ub144 10\uc6d4 15\uc77c \uc5d0 \ucc3d\ub2f9\ub41c \ub300\ud55c\ubbfc\uad6d\uc758 \uc9c4\ubcf4\uc815\ub2f9\uc774\ub2e4. 2017\ub144 10\uc6d4 15\uc77c \uc624\uc804 10\uc2dc \uad6d\ud68c\uc758\uc6d0\ud68c\uad00 \uc18c\ud68c\uc758\uc2e4\uc5d0\uc11c \uc0c8\ubbfc\uc911\uc815\ub2f9\uacfc \ubbfc\uc911\uc5f0\ud569\ub2f9 \uc591\ub2f9\uc758 \uc218\uc784\uae30\uad00 \ud569\ub3d9\ud68c\uc758\ub97c \ud1b5\ud574 \ud569\ub2f9\uc744 \uacf5\uc2dd \uc644\ub8cc\ud558\uace0, \uc624\ud6c4 2\uc2dc \uc11c\uc6b8\uc2dc\uccad \uc55e \uad11\uc7a5\uc5d0\uc11c '\ube44\uc815\uaddc\uc9c1 \uc5c6\ub294 \uc138\uc0c1, \uc804\uc7c1 \uc5c6\ub294 \ub098\ub77c', '\ubbfc\uc911\uc5d0\uac8c \uad8c\ub825\uc744, \ubbfc\uc911\uc5d0\uac8c \ud589\ubcf5'\uc744 \ub300\ud68c \uc2ac\ub85c\uac74\uc73c\ub85c \ub0b4\uac78\uace0 1\ub9cc\uc5ec \uba85\uc758 \uc804\uad6d \ub2f9\uc6d0\uacfc \ud568\uaed8 \ubbfc\uc911\ub2f9 \ucc3d\ub2f9\uc744 \uc120\uc5b8\ud558\uba70 \"\ucd1b\ubd88\ud601\uba85\uc758 \uacc4\uc2b9, \uc790\uc8fc\uc640 \ud3c9\ud654, \ube44\uc815\uaddc\uc9c1 \ucca0\ud3d0\ub97c \ud5a5\ud55c \uc9c4\ubcf4 \ub2e8\uacb0\uc758 \ub300\uc7a5\uc815\uc744 \uc2dc\uc791\ud55c\ub2e4.\"\uace0 \ud558\uba74\uc11c \"\ub178\ub3d9\uc790, \ub18d\ubbfc, \ube48\ubbfc, \uccad\ub144 \uc5ec\uc131\ub4e4\uc758 \uc9c1\uc811 \uc815\uce58 \uc6b4\ub3d9\uc758 \ud655\ub300\ub85c \uc0c8\ub85c\uc6b4 \uc815\uce58 \ubc14\ub78c\uc744 \ubd88\ub7ec\uc77c\uc73c\ud0ac \uac83\"\uc774\ub77c\uace0 \ubc1d\ud788\ub294 \uad11\uc7a5 \ucd9c\ubc94\uc2dd\uc744 \uac1c\ucd5c\ud558\uc600\ub2e4.", "sentence_answer": " 2017\ub144 10\uc6d4 15\uc77c \uc5d0", "paragraph_id": "6462144-4-0", "paragraph_question": "question: \uc9c4\ubcf4\uc815\ub2f9\uc774 \ucc3d\ub2f9\ub41c \ub54c\ub294 \uc5b8\uc81c\uc778\uac00?, context: 2017\ub144 10\uc6d4 15\uc77c\uc5d0 \ucc3d\ub2f9\ub41c \ub300\ud55c\ubbfc\uad6d\uc758 \uc9c4\ubcf4\uc815\ub2f9\uc774\ub2e4. 2017\ub144 10\uc6d4 15\uc77c \uc624\uc804 10\uc2dc \uad6d\ud68c\uc758\uc6d0\ud68c\uad00 \uc18c\ud68c\uc758\uc2e4\uc5d0\uc11c \uc0c8\ubbfc\uc911\uc815\ub2f9\uacfc \ubbfc\uc911\uc5f0\ud569\ub2f9 \uc591\ub2f9\uc758 \uc218\uc784\uae30\uad00 \ud569\ub3d9\ud68c\uc758\ub97c \ud1b5\ud574 \ud569\ub2f9\uc744 \uacf5\uc2dd \uc644\ub8cc\ud558\uace0, \uc624\ud6c4 2\uc2dc \uc11c\uc6b8\uc2dc\uccad \uc55e \uad11\uc7a5\uc5d0\uc11c '\ube44\uc815\uaddc\uc9c1 \uc5c6\ub294 \uc138\uc0c1, \uc804\uc7c1 \uc5c6\ub294 \ub098\ub77c', '\ubbfc\uc911\uc5d0\uac8c \uad8c\ub825\uc744, \ubbfc\uc911\uc5d0\uac8c \ud589\ubcf5'\uc744 \ub300\ud68c \uc2ac\ub85c\uac74\uc73c\ub85c \ub0b4\uac78\uace0 1\ub9cc\uc5ec \uba85\uc758 \uc804\uad6d \ub2f9\uc6d0\uacfc \ud568\uaed8 \ubbfc\uc911\ub2f9 \ucc3d\ub2f9\uc744 \uc120\uc5b8\ud558\uba70 \"\ucd1b\ubd88\ud601\uba85\uc758 \uacc4\uc2b9, \uc790\uc8fc\uc640 \ud3c9\ud654, \ube44\uc815\uaddc\uc9c1 \ucca0\ud3d0\ub97c \ud5a5\ud55c \uc9c4\ubcf4 \ub2e8\uacb0\uc758 \ub300\uc7a5\uc815\uc744 \uc2dc\uc791\ud55c\ub2e4.\"\uace0 \ud558\uba74\uc11c \"\ub178\ub3d9\uc790, \ub18d\ubbfc, \ube48\ubbfc, \uccad\ub144 \uc5ec\uc131\ub4e4\uc758 \uc9c1\uc811 \uc815\uce58 \uc6b4\ub3d9\uc758 \ud655\ub300\ub85c \uc0c8\ub85c\uc6b4 \uc815\uce58 \ubc14\ub78c\uc744 \ubd88\ub7ec\uc77c\uc73c\ud0ac \uac83\"\uc774\ub77c\uace0 \ubc1d\ud788\ub294 \uad11\uc7a5 \ucd9c\ubc94\uc2dd\uc744 \uac1c\ucd5c\ud558\uc600\ub2e4."} {"question": "\uc0c1\uc11c\ubc29\uc5d0\uc11c \uacf5\ubd80\ud560 \uc218 \uc788\uc5c8\ub358 \uacc4\uce35\uc740?", "paragraph": "\ud669\uc190\uc778 \ud64d\ub825\uc740 \ud669\uc790\ub4e4\ub9cc \uad50\uc721\ubc1b\uc744 \uc218 \uc788\ub358 \uc0c1\uc11c\ubc29(\u4e0a\u66f8\u623f)\uc5d0\uc11c \uacf5\ubd80\ud558\uc600\ub294\ub370 \uc774\uacf3\uc5d0\uc120 \ud64d\ub825 \uc774\uc678\uc5d0\ub3c4 \uc790\uc9c8\uc774 \ub0a8\ub2e4\ub978 \uc5ec\ub7ec \ud669\uc190\ub4e4\uacfc \ud64d\ub825\uacfc \ub3d9\uac11\uc778 \uc219\ubd80 \uc724\ud76c(\u80e4\u79a7)\ub3c4 \uac19\uc774 \uacf5\ubd80\ud558\uace0 \uc788\uc5c8\ub2e4. \ub2f9\uc2dc \uac15\ud76c\uc81c\uc5d0\uac8c\ub294 20\uba85\uc758 \ud669\uc790\uc640 100\uba85 \uc548\ud30e\uc758 \uc190\uc790\uac00 \uc788\uc5c8\ub294\ub370 \uac15\ud76c\uc81c\ub294 \uc190\uc790\ub4e4\uc758 \uc0c1\ub2f9\uc218\ub97c \uc54c\uc9c0\ub3c4, \uc2ec\uc9c0\uc5b4\ub294 \uc5bc\uad74 \ud55c \ubc88 \ubcf8 \uc801\ub3c4 \uc5c6\ub294 \uacbd\uc6b0\ub3c4 \uc788\uc5c8\ub2e4. \uadf8\ub7ec\ub098 \uadf8 \ub9ce\uc740 \ud669\uc190\ub4e4 \uc911 \uad81\uc815\uc5d0\uc11c \uc0dd\ud65c\ud560 \uc218 \uc788\ub3c4\ub85d \ud2b9\ubcc4\ud788 \ubf51\ud78c \ud64d\ub825\uc774 \ubb38\uc7ac\uc5d0\ub3c4 \ud0c1\uc6d4\ud558\uc600\uace0 \uc0ac\ub0e5\uc5d0\uc11c\ub294 \uc5b4\ub9b0 \ub098\uc774\uc5d0 \uacf0\uc744 \ub450\ub824\uc6cc\ud558\uc9c0 \uc54a\uace0 \ud654\uc0b4\ub85c \ud798\ub4e4\uc774\uc9c0 \uc54a\uace0 \uc7a1\uc790 \uc774\ub97c \ub9e4\uc6b0 \uae30\ud2b9\ud558\uac8c \uc5ec\uae34 \uac15\ud76c\uc81c\ub294 \uc5b4\ub9b4 \ub54c\uc758 \uc790\uc2e0\uc744 \ub2ee\uc558\ub2e4\uba70 \ud64d\ub825\uc5d0\uac8c \uae30\ubcf8\uc801\uc778 \uc81c\uc655\ud559\uc744 \uac00\ub974\uce58\uae30 \uc2dc\uc791\ud558\uc600\uace0 \ud2b9\ubcc4\ud788 \uc790\uae08\uc131\uc744 \uc9c0\ud0a4\ub294 \uae08\uad70 \ubb34\uad00\uc5d0\uac8c \uba85\ud574 \ud64d\ub825\uc5d0\uac8c \ubb34\uc608\ub97c \uc804\uc218\ud574\uc904 \uac83\uc744 \uba85\ud558\uc600\ub2e4. \uc5b4\ub5a4 \ub54c\uc5d0\ub294 \ub178\uad6c\uc758 \uac15\ud76c\uc81c\uac00 \uc9c1\uc811 \ud64d\ub825\uc758 \ubb34\uc220\uc744 \ubcf4\uace0 \uac00\ub974\uce58\uace0 \uac19\uc774 \uc218\ub828\ud558\uc600\ub2e4. \uad81\uc815\uc5d0\uc11c \uc0b4\uba74\uc11c \ud560\uc544\ubc84\uc9c0\uc758 \ucd1d\uc560\ub97c \ubc1b\uc558\uc73c\ub098, \ud55c\ud3b8\uc73c\ub85c\ub294 \uc790\uc8fc \uc790\uc2e0\uc774 \uc0b4\ub358 \uc639\uce5c\uc655\ubd80\ub85c \uac00 \ubd80\uc655\uc778 \uc639\uce5c\uc655 \uc724\uc9c4\uacfc \uc801\ubaa8\uc778 \uc801\ubcf5\uc9c4 \uc624\ub77c\ub098\ub78d\uc528, \uadf8\ub9ac\uace0 \uc5b4\uba38\ub2c8\uc5d0\uac8c \ubb38\uc548\ub4dc\ub838\uace0 \uc774\ub7ec\ud55c \uc131\uc2e4\ud55c \ubaa8\uc2b5\uc744 \ubcf8 \uac15\ud76c\uc81c\uac00 \ub354\uc6b1\ub354 \ucd1d\uc560\ud558\uc600\ub2e4.", "answer": "\ud669\uc790", "sentence": "\ud669\uc190\uc778 \ud64d\ub825\uc740 \ud669\uc790 \ub4e4\ub9cc \uad50\uc721\ubc1b\uc744 \uc218 \uc788\ub358 \uc0c1\uc11c\ubc29(\u4e0a\u66f8\u623f)\uc5d0\uc11c \uacf5\ubd80\ud558\uc600\ub294\ub370 \uc774\uacf3\uc5d0\uc120 \ud64d\ub825 \uc774\uc678\uc5d0\ub3c4 \uc790\uc9c8\uc774 \ub0a8\ub2e4\ub978 \uc5ec\ub7ec \ud669\uc190\ub4e4\uacfc \ud64d\ub825\uacfc \ub3d9\uac11\uc778 \uc219\ubd80 \uc724\ud76c(\u80e4\u79a7)\ub3c4 \uac19\uc774 \uacf5\ubd80\ud558\uace0 \uc788\uc5c8\ub2e4.", "paragraph_sentence": " \ud669\uc190\uc778 \ud64d\ub825\uc740 \ud669\uc790 \ub4e4\ub9cc \uad50\uc721\ubc1b\uc744 \uc218 \uc788\ub358 \uc0c1\uc11c\ubc29(\u4e0a\u66f8\u623f)\uc5d0\uc11c \uacf5\ubd80\ud558\uc600\ub294\ub370 \uc774\uacf3\uc5d0\uc120 \ud64d\ub825 \uc774\uc678\uc5d0\ub3c4 \uc790\uc9c8\uc774 \ub0a8\ub2e4\ub978 \uc5ec\ub7ec \ud669\uc190\ub4e4\uacfc \ud64d\ub825\uacfc \ub3d9\uac11\uc778 \uc219\ubd80 \uc724\ud76c(\u80e4\u79a7)\ub3c4 \uac19\uc774 \uacf5\ubd80\ud558\uace0 \uc788\uc5c8\ub2e4. \ub2f9\uc2dc \uac15\ud76c\uc81c\uc5d0\uac8c\ub294 20\uba85\uc758 \ud669\uc790\uc640 100\uba85 \uc548\ud30e\uc758 \uc190\uc790\uac00 \uc788\uc5c8\ub294\ub370 \uac15\ud76c\uc81c\ub294 \uc190\uc790\ub4e4\uc758 \uc0c1\ub2f9\uc218\ub97c \uc54c\uc9c0\ub3c4, \uc2ec\uc9c0\uc5b4\ub294 \uc5bc\uad74 \ud55c \ubc88 \ubcf8 \uc801\ub3c4 \uc5c6\ub294 \uacbd\uc6b0\ub3c4 \uc788\uc5c8\ub2e4. \uadf8\ub7ec\ub098 \uadf8 \ub9ce\uc740 \ud669\uc190\ub4e4 \uc911 \uad81\uc815\uc5d0\uc11c \uc0dd\ud65c\ud560 \uc218 \uc788\ub3c4\ub85d \ud2b9\ubcc4\ud788 \ubf51\ud78c \ud64d\ub825\uc774 \ubb38\uc7ac\uc5d0\ub3c4 \ud0c1\uc6d4\ud558\uc600\uace0 \uc0ac\ub0e5\uc5d0\uc11c\ub294 \uc5b4\ub9b0 \ub098\uc774\uc5d0 \uacf0\uc744 \ub450\ub824\uc6cc\ud558\uc9c0 \uc54a\uace0 \ud654\uc0b4\ub85c \ud798\ub4e4\uc774\uc9c0 \uc54a\uace0 \uc7a1\uc790 \uc774\ub97c \ub9e4\uc6b0 \uae30\ud2b9\ud558\uac8c \uc5ec\uae34 \uac15\ud76c\uc81c\ub294 \uc5b4\ub9b4 \ub54c\uc758 \uc790\uc2e0\uc744 \ub2ee\uc558\ub2e4\uba70 \ud64d\ub825\uc5d0\uac8c \uae30\ubcf8\uc801\uc778 \uc81c\uc655\ud559\uc744 \uac00\ub974\uce58\uae30 \uc2dc\uc791\ud558\uc600\uace0 \ud2b9\ubcc4\ud788 \uc790\uae08\uc131\uc744 \uc9c0\ud0a4\ub294 \uae08\uad70 \ubb34\uad00\uc5d0\uac8c \uba85\ud574 \ud64d\ub825\uc5d0\uac8c \ubb34\uc608\ub97c \uc804\uc218\ud574\uc904 \uac83\uc744 \uba85\ud558\uc600\ub2e4. \uc5b4\ub5a4 \ub54c\uc5d0\ub294 \ub178\uad6c\uc758 \uac15\ud76c\uc81c\uac00 \uc9c1\uc811 \ud64d\ub825\uc758 \ubb34\uc220\uc744 \ubcf4\uace0 \uac00\ub974\uce58\uace0 \uac19\uc774 \uc218\ub828\ud558\uc600\ub2e4. \uad81\uc815\uc5d0\uc11c \uc0b4\uba74\uc11c \ud560\uc544\ubc84\uc9c0\uc758 \ucd1d\uc560\ub97c \ubc1b\uc558\uc73c\ub098, \ud55c\ud3b8\uc73c\ub85c\ub294 \uc790\uc8fc \uc790\uc2e0\uc774 \uc0b4\ub358 \uc639\uce5c\uc655\ubd80\ub85c \uac00 \ubd80\uc655\uc778 \uc639\uce5c\uc655 \uc724\uc9c4\uacfc \uc801\ubaa8\uc778 \uc801\ubcf5\uc9c4 \uc624\ub77c\ub098\ub78d\uc528, \uadf8\ub9ac\uace0 \uc5b4\uba38\ub2c8\uc5d0\uac8c \ubb38\uc548\ub4dc\ub838\uace0 \uc774\ub7ec\ud55c \uc131\uc2e4\ud55c \ubaa8\uc2b5\uc744 \ubcf8 \uac15\ud76c\uc81c\uac00 \ub354\uc6b1\ub354 \ucd1d\uc560\ud558\uc600\ub2e4.", "paragraph_answer": "\ud669\uc190\uc778 \ud64d\ub825\uc740 \ud669\uc790 \ub4e4\ub9cc \uad50\uc721\ubc1b\uc744 \uc218 \uc788\ub358 \uc0c1\uc11c\ubc29(\u4e0a\u66f8\u623f)\uc5d0\uc11c \uacf5\ubd80\ud558\uc600\ub294\ub370 \uc774\uacf3\uc5d0\uc120 \ud64d\ub825 \uc774\uc678\uc5d0\ub3c4 \uc790\uc9c8\uc774 \ub0a8\ub2e4\ub978 \uc5ec\ub7ec \ud669\uc190\ub4e4\uacfc \ud64d\ub825\uacfc \ub3d9\uac11\uc778 \uc219\ubd80 \uc724\ud76c(\u80e4\u79a7)\ub3c4 \uac19\uc774 \uacf5\ubd80\ud558\uace0 \uc788\uc5c8\ub2e4. \ub2f9\uc2dc \uac15\ud76c\uc81c\uc5d0\uac8c\ub294 20\uba85\uc758 \ud669\uc790\uc640 100\uba85 \uc548\ud30e\uc758 \uc190\uc790\uac00 \uc788\uc5c8\ub294\ub370 \uac15\ud76c\uc81c\ub294 \uc190\uc790\ub4e4\uc758 \uc0c1\ub2f9\uc218\ub97c \uc54c\uc9c0\ub3c4, \uc2ec\uc9c0\uc5b4\ub294 \uc5bc\uad74 \ud55c \ubc88 \ubcf8 \uc801\ub3c4 \uc5c6\ub294 \uacbd\uc6b0\ub3c4 \uc788\uc5c8\ub2e4. \uadf8\ub7ec\ub098 \uadf8 \ub9ce\uc740 \ud669\uc190\ub4e4 \uc911 \uad81\uc815\uc5d0\uc11c \uc0dd\ud65c\ud560 \uc218 \uc788\ub3c4\ub85d \ud2b9\ubcc4\ud788 \ubf51\ud78c \ud64d\ub825\uc774 \ubb38\uc7ac\uc5d0\ub3c4 \ud0c1\uc6d4\ud558\uc600\uace0 \uc0ac\ub0e5\uc5d0\uc11c\ub294 \uc5b4\ub9b0 \ub098\uc774\uc5d0 \uacf0\uc744 \ub450\ub824\uc6cc\ud558\uc9c0 \uc54a\uace0 \ud654\uc0b4\ub85c \ud798\ub4e4\uc774\uc9c0 \uc54a\uace0 \uc7a1\uc790 \uc774\ub97c \ub9e4\uc6b0 \uae30\ud2b9\ud558\uac8c \uc5ec\uae34 \uac15\ud76c\uc81c\ub294 \uc5b4\ub9b4 \ub54c\uc758 \uc790\uc2e0\uc744 \ub2ee\uc558\ub2e4\uba70 \ud64d\ub825\uc5d0\uac8c \uae30\ubcf8\uc801\uc778 \uc81c\uc655\ud559\uc744 \uac00\ub974\uce58\uae30 \uc2dc\uc791\ud558\uc600\uace0 \ud2b9\ubcc4\ud788 \uc790\uae08\uc131\uc744 \uc9c0\ud0a4\ub294 \uae08\uad70 \ubb34\uad00\uc5d0\uac8c \uba85\ud574 \ud64d\ub825\uc5d0\uac8c \ubb34\uc608\ub97c \uc804\uc218\ud574\uc904 \uac83\uc744 \uba85\ud558\uc600\ub2e4. \uc5b4\ub5a4 \ub54c\uc5d0\ub294 \ub178\uad6c\uc758 \uac15\ud76c\uc81c\uac00 \uc9c1\uc811 \ud64d\ub825\uc758 \ubb34\uc220\uc744 \ubcf4\uace0 \uac00\ub974\uce58\uace0 \uac19\uc774 \uc218\ub828\ud558\uc600\ub2e4. \uad81\uc815\uc5d0\uc11c \uc0b4\uba74\uc11c \ud560\uc544\ubc84\uc9c0\uc758 \ucd1d\uc560\ub97c \ubc1b\uc558\uc73c\ub098, \ud55c\ud3b8\uc73c\ub85c\ub294 \uc790\uc8fc \uc790\uc2e0\uc774 \uc0b4\ub358 \uc639\uce5c\uc655\ubd80\ub85c \uac00 \ubd80\uc655\uc778 \uc639\uce5c\uc655 \uc724\uc9c4\uacfc \uc801\ubaa8\uc778 \uc801\ubcf5\uc9c4 \uc624\ub77c\ub098\ub78d\uc528, \uadf8\ub9ac\uace0 \uc5b4\uba38\ub2c8\uc5d0\uac8c \ubb38\uc548\ub4dc\ub838\uace0 \uc774\ub7ec\ud55c \uc131\uc2e4\ud55c \ubaa8\uc2b5\uc744 \ubcf8 \uac15\ud76c\uc81c\uac00 \ub354\uc6b1\ub354 \ucd1d\uc560\ud558\uc600\ub2e4.", "sentence_answer": "\ud669\uc190\uc778 \ud64d\ub825\uc740 \ud669\uc790 \ub4e4\ub9cc \uad50\uc721\ubc1b\uc744 \uc218 \uc788\ub358 \uc0c1\uc11c\ubc29(\u4e0a\u66f8\u623f)\uc5d0\uc11c \uacf5\ubd80\ud558\uc600\ub294\ub370 \uc774\uacf3\uc5d0\uc120 \ud64d\ub825 \uc774\uc678\uc5d0\ub3c4 \uc790\uc9c8\uc774 \ub0a8\ub2e4\ub978 \uc5ec\ub7ec \ud669\uc190\ub4e4\uacfc \ud64d\ub825\uacfc \ub3d9\uac11\uc778 \uc219\ubd80 \uc724\ud76c(\u80e4\u79a7)\ub3c4 \uac19\uc774 \uacf5\ubd80\ud558\uace0 \uc788\uc5c8\ub2e4.", "paragraph_id": "6391190-3-1", "paragraph_question": "question: \uc0c1\uc11c\ubc29\uc5d0\uc11c \uacf5\ubd80\ud560 \uc218 \uc788\uc5c8\ub358 \uacc4\uce35\uc740?, context: \ud669\uc190\uc778 \ud64d\ub825\uc740 \ud669\uc790\ub4e4\ub9cc \uad50\uc721\ubc1b\uc744 \uc218 \uc788\ub358 \uc0c1\uc11c\ubc29(\u4e0a\u66f8\u623f)\uc5d0\uc11c \uacf5\ubd80\ud558\uc600\ub294\ub370 \uc774\uacf3\uc5d0\uc120 \ud64d\ub825 \uc774\uc678\uc5d0\ub3c4 \uc790\uc9c8\uc774 \ub0a8\ub2e4\ub978 \uc5ec\ub7ec \ud669\uc190\ub4e4\uacfc \ud64d\ub825\uacfc \ub3d9\uac11\uc778 \uc219\ubd80 \uc724\ud76c(\u80e4\u79a7)\ub3c4 \uac19\uc774 \uacf5\ubd80\ud558\uace0 \uc788\uc5c8\ub2e4. \ub2f9\uc2dc \uac15\ud76c\uc81c\uc5d0\uac8c\ub294 20\uba85\uc758 \ud669\uc790\uc640 100\uba85 \uc548\ud30e\uc758 \uc190\uc790\uac00 \uc788\uc5c8\ub294\ub370 \uac15\ud76c\uc81c\ub294 \uc190\uc790\ub4e4\uc758 \uc0c1\ub2f9\uc218\ub97c \uc54c\uc9c0\ub3c4, \uc2ec\uc9c0\uc5b4\ub294 \uc5bc\uad74 \ud55c \ubc88 \ubcf8 \uc801\ub3c4 \uc5c6\ub294 \uacbd\uc6b0\ub3c4 \uc788\uc5c8\ub2e4. \uadf8\ub7ec\ub098 \uadf8 \ub9ce\uc740 \ud669\uc190\ub4e4 \uc911 \uad81\uc815\uc5d0\uc11c \uc0dd\ud65c\ud560 \uc218 \uc788\ub3c4\ub85d \ud2b9\ubcc4\ud788 \ubf51\ud78c \ud64d\ub825\uc774 \ubb38\uc7ac\uc5d0\ub3c4 \ud0c1\uc6d4\ud558\uc600\uace0 \uc0ac\ub0e5\uc5d0\uc11c\ub294 \uc5b4\ub9b0 \ub098\uc774\uc5d0 \uacf0\uc744 \ub450\ub824\uc6cc\ud558\uc9c0 \uc54a\uace0 \ud654\uc0b4\ub85c \ud798\ub4e4\uc774\uc9c0 \uc54a\uace0 \uc7a1\uc790 \uc774\ub97c \ub9e4\uc6b0 \uae30\ud2b9\ud558\uac8c \uc5ec\uae34 \uac15\ud76c\uc81c\ub294 \uc5b4\ub9b4 \ub54c\uc758 \uc790\uc2e0\uc744 \ub2ee\uc558\ub2e4\uba70 \ud64d\ub825\uc5d0\uac8c \uae30\ubcf8\uc801\uc778 \uc81c\uc655\ud559\uc744 \uac00\ub974\uce58\uae30 \uc2dc\uc791\ud558\uc600\uace0 \ud2b9\ubcc4\ud788 \uc790\uae08\uc131\uc744 \uc9c0\ud0a4\ub294 \uae08\uad70 \ubb34\uad00\uc5d0\uac8c \uba85\ud574 \ud64d\ub825\uc5d0\uac8c \ubb34\uc608\ub97c \uc804\uc218\ud574\uc904 \uac83\uc744 \uba85\ud558\uc600\ub2e4. \uc5b4\ub5a4 \ub54c\uc5d0\ub294 \ub178\uad6c\uc758 \uac15\ud76c\uc81c\uac00 \uc9c1\uc811 \ud64d\ub825\uc758 \ubb34\uc220\uc744 \ubcf4\uace0 \uac00\ub974\uce58\uace0 \uac19\uc774 \uc218\ub828\ud558\uc600\ub2e4. \uad81\uc815\uc5d0\uc11c \uc0b4\uba74\uc11c \ud560\uc544\ubc84\uc9c0\uc758 \ucd1d\uc560\ub97c \ubc1b\uc558\uc73c\ub098, \ud55c\ud3b8\uc73c\ub85c\ub294 \uc790\uc8fc \uc790\uc2e0\uc774 \uc0b4\ub358 \uc639\uce5c\uc655\ubd80\ub85c \uac00 \ubd80\uc655\uc778 \uc639\uce5c\uc655 \uc724\uc9c4\uacfc \uc801\ubaa8\uc778 \uc801\ubcf5\uc9c4 \uc624\ub77c\ub098\ub78d\uc528, \uadf8\ub9ac\uace0 \uc5b4\uba38\ub2c8\uc5d0\uac8c \ubb38\uc548\ub4dc\ub838\uace0 \uc774\ub7ec\ud55c \uc131\uc2e4\ud55c \ubaa8\uc2b5\uc744 \ubcf8 \uac15\ud76c\uc81c\uac00 \ub354\uc6b1\ub354 \ucd1d\uc560\ud558\uc600\ub2e4."} {"question": "2012\ub144 4\uc6d4 27\uc77c \ubc1c\ub9e4\ud55c \uc544\uc774\ube44\uc758 \uccab \ubbf8\ub2c8 \uc74c\ubc18\uc758 \ud0c0\uc774\ud2c0 \uace1\uc740 \ubb34\uc5c7\uc778\uac00?", "paragraph": "\ud55c\ud3b8, \uc544\uc774\ube44\ub294 2009\ub144 \ucd08 \ud3f4\ub77c\ub9ac\uc2a4 \uc5d4\ud130\ud14c\uc778\uba3c\ud2b8\ub85c \uc18c\uc18d\uc0ac\ub97c \uc774\uc801\ud560 \uc608\uc815\uc774\uc5c8\uc9c0\ub9cc, \uc804\uc18d\uad8c\uc740 \uc2a4\ud1b0\uc774\uc564\uc5d0\ud504\uc5d0\uac8c \uc788\ub2e4\ub294 \uc774\uc720\ub85c \ubc95\uc815 \ubd84\uc7c1\uc744 \uc77c\uc73c\ucf30\uace0, \uadf8 \uacb0\uacfc 2011\ub144 9\uc6d4 \uc2b9\uc18c\ud558\uba70 \uc18c\uc18d\uc0ac\ub97c \uc62e\uacbc\ub2e4. 2012\ub144 2\uc6d4, \uc544\uc774\ube44\ub294 2008\ub144 3\uc6d4\ubd80\ud130 \ub9cc\ub0ac\ub358 \uc791\uace1\uac00 \uae40\ud0dc\uc131\uacfc 2011\ub144 \ub9d0\uc5d0 \uacb0\ubcc4\ud588\ub2e4\uace0 \ubc1d\ud614\ub2e4. \uc18c\uc18d\uc0ac\uc640 \ubd84\uc7c1\uc774 \ub05d\ub09c \ud6c4, 2012\ub144 4\uc6d4 27\uc77c \uc544\uc774\ube44\uc758 \uccab \ubbf8\ub2c8 \uc74c\ubc18 INTERVIEW\ub97c \ubc1c\ub9e4\ud588\ub2e4. \uc774\ubc88 \uc568\ubc94\uc740 \uc774\uc804\uc758 \uc568\ubc94\ub4e4\uacfc \ub2ec\ub9ac \ud0c0\uc774\ud2c0\uace1 \u3008\ucc22\uae34 \uac00\uc2b4\u3009\uc744 \ud3ec\ud568\ud574 \ub2e4\uc12f \uace1 \ubaa8\ub450 \ubc1c\ub77c\ub4dc\uc640 \ubbf8\ub514\uc5c4 \ud15c\ud3ec \ub178\ub798\ub4e4\ub85c \uc774\ub8e8\uc5b4\uc84c\ub2e4. 2012\ub144 \uc5ec\ub984\uc5d0\ub294 \ubba4\uc9c0\uceec \uc2dc\uce74\uace0\uc5d0 \uc8fc\uc5f0 \ub85d\uc2dc \ud558\ud2b8\ub85c \ucd9c\uc5f0\ud588\ub2e4. 6\uc6d4 19\uc77c\uc5d0\ub294 \ucd94\uc801\uc790 SBS \ub4dc\ub77c\ub9c8 \u300a\ucd94\uc801\uc790 THE CHASER\u300b\uc758 OST \u3008\uc88b\uc740 \uc0ac\ub78c\u3009\uc744 \ubc1c\ub9e4\ud588\ub2e4. 9\uc6d4\ubd80\ud130\ub294 SBS\uc758 \uc624\ub514\uc158 \ud504\ub85c\uadf8\ub7a8 \u300a\ub0b4 \uc0dd\uc560 \ub9c8\uc9c0\ub9c9 \uc624\ub514\uc158\u300b\uc758 \uc2ec\uc0ac\uc704\uc6d0\uc73c\ub85c \ubc1c\ud0c1 \ub3fc \ucd9c\uc5f0\ud588\ub2e4. \ub610\ud55c KBS \uc608\ub2a5 \u300a\ub0a8\uc790\uc758 \uc790\uaca9\u300b \ud328\ubc00\ub9ac \ud569\ucc3d\ub2e8 \ud3b8\uc5d0 \uc5b4\uba38\ub2c8\uc640 \ud568\uaed8 \uc131\uc545\ub2e8\uc73c\ub85c \ucd9c\uc5f0\ud588\ub2e4.", "answer": "\ucc22\uae34 \uac00\uc2b4", "sentence": "\uc774\ubc88 \uc568\ubc94\uc740 \uc774\uc804\uc758 \uc568\ubc94\ub4e4\uacfc \ub2ec\ub9ac \ud0c0\uc774\ud2c0\uace1 \u3008 \ucc22\uae34 \uac00\uc2b4 \u3009\uc744 \ud3ec\ud568\ud574 \ub2e4\uc12f \uace1 \ubaa8\ub450 \ubc1c\ub77c\ub4dc\uc640 \ubbf8\ub514\uc5c4 \ud15c\ud3ec \ub178\ub798\ub4e4\ub85c \uc774\ub8e8\uc5b4\uc84c\ub2e4.", "paragraph_sentence": "\ud55c\ud3b8, \uc544\uc774\ube44\ub294 2009\ub144 \ucd08 \ud3f4\ub77c\ub9ac\uc2a4 \uc5d4\ud130\ud14c\uc778\uba3c\ud2b8\ub85c \uc18c\uc18d\uc0ac\ub97c \uc774\uc801\ud560 \uc608\uc815\uc774\uc5c8\uc9c0\ub9cc, \uc804\uc18d\uad8c\uc740 \uc2a4\ud1b0\uc774\uc564\uc5d0\ud504\uc5d0\uac8c \uc788\ub2e4\ub294 \uc774\uc720\ub85c \ubc95\uc815 \ubd84\uc7c1\uc744 \uc77c\uc73c\ucf30\uace0, \uadf8 \uacb0\uacfc 2011\ub144 9\uc6d4 \uc2b9\uc18c\ud558\uba70 \uc18c\uc18d\uc0ac\ub97c \uc62e\uacbc\ub2e4. 2012\ub144 2\uc6d4, \uc544\uc774\ube44\ub294 2008\ub144 3\uc6d4\ubd80\ud130 \ub9cc\ub0ac\ub358 \uc791\uace1\uac00 \uae40\ud0dc\uc131\uacfc 2011\ub144 \ub9d0\uc5d0 \uacb0\ubcc4\ud588\ub2e4\uace0 \ubc1d\ud614\ub2e4. \uc18c\uc18d\uc0ac\uc640 \ubd84\uc7c1\uc774 \ub05d\ub09c \ud6c4, 2012\ub144 4\uc6d4 27\uc77c \uc544\uc774\ube44\uc758 \uccab \ubbf8\ub2c8 \uc74c\ubc18 INTERVIEW\ub97c \ubc1c\ub9e4\ud588\ub2e4. \uc774\ubc88 \uc568\ubc94\uc740 \uc774\uc804\uc758 \uc568\ubc94\ub4e4\uacfc \ub2ec\ub9ac \ud0c0\uc774\ud2c0\uace1 \u3008 \ucc22\uae34 \uac00\uc2b4 \u3009\uc744 \ud3ec\ud568\ud574 \ub2e4\uc12f \uace1 \ubaa8\ub450 \ubc1c\ub77c\ub4dc\uc640 \ubbf8\ub514\uc5c4 \ud15c\ud3ec \ub178\ub798\ub4e4\ub85c \uc774\ub8e8\uc5b4\uc84c\ub2e4. 2012\ub144 \uc5ec\ub984\uc5d0\ub294 \ubba4\uc9c0\uceec \uc2dc\uce74\uace0\uc5d0 \uc8fc\uc5f0 \ub85d\uc2dc \ud558\ud2b8\ub85c \ucd9c\uc5f0\ud588\ub2e4. 6\uc6d4 19\uc77c\uc5d0\ub294 \ucd94\uc801\uc790 SBS \ub4dc\ub77c\ub9c8 \u300a\ucd94\uc801\uc790 THE CHASER\u300b\uc758 OST \u3008\uc88b\uc740 \uc0ac\ub78c\u3009\uc744 \ubc1c\ub9e4\ud588\ub2e4. 9\uc6d4\ubd80\ud130\ub294 SBS\uc758 \uc624\ub514\uc158 \ud504\ub85c\uadf8\ub7a8 \u300a\ub0b4 \uc0dd\uc560 \ub9c8\uc9c0\ub9c9 \uc624\ub514\uc158\u300b\uc758 \uc2ec\uc0ac\uc704\uc6d0\uc73c\ub85c \ubc1c\ud0c1 \ub3fc \ucd9c\uc5f0\ud588\ub2e4. \ub610\ud55c KBS \uc608\ub2a5 \u300a\ub0a8\uc790\uc758 \uc790\uaca9\u300b \ud328\ubc00\ub9ac \ud569\ucc3d\ub2e8 \ud3b8\uc5d0 \uc5b4\uba38\ub2c8\uc640 \ud568\uaed8 \uc131\uc545\ub2e8\uc73c\ub85c \ucd9c\uc5f0\ud588\ub2e4.", "paragraph_answer": "\ud55c\ud3b8, \uc544\uc774\ube44\ub294 2009\ub144 \ucd08 \ud3f4\ub77c\ub9ac\uc2a4 \uc5d4\ud130\ud14c\uc778\uba3c\ud2b8\ub85c \uc18c\uc18d\uc0ac\ub97c \uc774\uc801\ud560 \uc608\uc815\uc774\uc5c8\uc9c0\ub9cc, \uc804\uc18d\uad8c\uc740 \uc2a4\ud1b0\uc774\uc564\uc5d0\ud504\uc5d0\uac8c \uc788\ub2e4\ub294 \uc774\uc720\ub85c \ubc95\uc815 \ubd84\uc7c1\uc744 \uc77c\uc73c\ucf30\uace0, \uadf8 \uacb0\uacfc 2011\ub144 9\uc6d4 \uc2b9\uc18c\ud558\uba70 \uc18c\uc18d\uc0ac\ub97c \uc62e\uacbc\ub2e4. 2012\ub144 2\uc6d4, \uc544\uc774\ube44\ub294 2008\ub144 3\uc6d4\ubd80\ud130 \ub9cc\ub0ac\ub358 \uc791\uace1\uac00 \uae40\ud0dc\uc131\uacfc 2011\ub144 \ub9d0\uc5d0 \uacb0\ubcc4\ud588\ub2e4\uace0 \ubc1d\ud614\ub2e4. \uc18c\uc18d\uc0ac\uc640 \ubd84\uc7c1\uc774 \ub05d\ub09c \ud6c4, 2012\ub144 4\uc6d4 27\uc77c \uc544\uc774\ube44\uc758 \uccab \ubbf8\ub2c8 \uc74c\ubc18 INTERVIEW\ub97c \ubc1c\ub9e4\ud588\ub2e4. \uc774\ubc88 \uc568\ubc94\uc740 \uc774\uc804\uc758 \uc568\ubc94\ub4e4\uacfc \ub2ec\ub9ac \ud0c0\uc774\ud2c0\uace1 \u3008 \ucc22\uae34 \uac00\uc2b4 \u3009\uc744 \ud3ec\ud568\ud574 \ub2e4\uc12f \uace1 \ubaa8\ub450 \ubc1c\ub77c\ub4dc\uc640 \ubbf8\ub514\uc5c4 \ud15c\ud3ec \ub178\ub798\ub4e4\ub85c \uc774\ub8e8\uc5b4\uc84c\ub2e4. 2012\ub144 \uc5ec\ub984\uc5d0\ub294 \ubba4\uc9c0\uceec \uc2dc\uce74\uace0\uc5d0 \uc8fc\uc5f0 \ub85d\uc2dc \ud558\ud2b8\ub85c \ucd9c\uc5f0\ud588\ub2e4. 6\uc6d4 19\uc77c\uc5d0\ub294 \ucd94\uc801\uc790 SBS \ub4dc\ub77c\ub9c8 \u300a\ucd94\uc801\uc790 THE CHASER\u300b\uc758 OST \u3008\uc88b\uc740 \uc0ac\ub78c\u3009\uc744 \ubc1c\ub9e4\ud588\ub2e4. 9\uc6d4\ubd80\ud130\ub294 SBS\uc758 \uc624\ub514\uc158 \ud504\ub85c\uadf8\ub7a8 \u300a\ub0b4 \uc0dd\uc560 \ub9c8\uc9c0\ub9c9 \uc624\ub514\uc158\u300b\uc758 \uc2ec\uc0ac\uc704\uc6d0\uc73c\ub85c \ubc1c\ud0c1 \ub3fc \ucd9c\uc5f0\ud588\ub2e4. \ub610\ud55c KBS \uc608\ub2a5 \u300a\ub0a8\uc790\uc758 \uc790\uaca9\u300b \ud328\ubc00\ub9ac \ud569\ucc3d\ub2e8 \ud3b8\uc5d0 \uc5b4\uba38\ub2c8\uc640 \ud568\uaed8 \uc131\uc545\ub2e8\uc73c\ub85c \ucd9c\uc5f0\ud588\ub2e4.", "sentence_answer": "\uc774\ubc88 \uc568\ubc94\uc740 \uc774\uc804\uc758 \uc568\ubc94\ub4e4\uacfc \ub2ec\ub9ac \ud0c0\uc774\ud2c0\uace1 \u3008 \ucc22\uae34 \uac00\uc2b4 \u3009\uc744 \ud3ec\ud568\ud574 \ub2e4\uc12f \uace1 \ubaa8\ub450 \ubc1c\ub77c\ub4dc\uc640 \ubbf8\ub514\uc5c4 \ud15c\ud3ec \ub178\ub798\ub4e4\ub85c \uc774\ub8e8\uc5b4\uc84c\ub2e4.", "paragraph_id": "6547446-5-1", "paragraph_question": "question: 2012\ub144 4\uc6d4 27\uc77c \ubc1c\ub9e4\ud55c \uc544\uc774\ube44\uc758 \uccab \ubbf8\ub2c8 \uc74c\ubc18\uc758 \ud0c0\uc774\ud2c0 \uace1\uc740 \ubb34\uc5c7\uc778\uac00?, context: \ud55c\ud3b8, \uc544\uc774\ube44\ub294 2009\ub144 \ucd08 \ud3f4\ub77c\ub9ac\uc2a4 \uc5d4\ud130\ud14c\uc778\uba3c\ud2b8\ub85c \uc18c\uc18d\uc0ac\ub97c \uc774\uc801\ud560 \uc608\uc815\uc774\uc5c8\uc9c0\ub9cc, \uc804\uc18d\uad8c\uc740 \uc2a4\ud1b0\uc774\uc564\uc5d0\ud504\uc5d0\uac8c \uc788\ub2e4\ub294 \uc774\uc720\ub85c \ubc95\uc815 \ubd84\uc7c1\uc744 \uc77c\uc73c\ucf30\uace0, \uadf8 \uacb0\uacfc 2011\ub144 9\uc6d4 \uc2b9\uc18c\ud558\uba70 \uc18c\uc18d\uc0ac\ub97c \uc62e\uacbc\ub2e4. 2012\ub144 2\uc6d4, \uc544\uc774\ube44\ub294 2008\ub144 3\uc6d4\ubd80\ud130 \ub9cc\ub0ac\ub358 \uc791\uace1\uac00 \uae40\ud0dc\uc131\uacfc 2011\ub144 \ub9d0\uc5d0 \uacb0\ubcc4\ud588\ub2e4\uace0 \ubc1d\ud614\ub2e4. \uc18c\uc18d\uc0ac\uc640 \ubd84\uc7c1\uc774 \ub05d\ub09c \ud6c4, 2012\ub144 4\uc6d4 27\uc77c \uc544\uc774\ube44\uc758 \uccab \ubbf8\ub2c8 \uc74c\ubc18 INTERVIEW\ub97c \ubc1c\ub9e4\ud588\ub2e4. \uc774\ubc88 \uc568\ubc94\uc740 \uc774\uc804\uc758 \uc568\ubc94\ub4e4\uacfc \ub2ec\ub9ac \ud0c0\uc774\ud2c0\uace1 \u3008\ucc22\uae34 \uac00\uc2b4\u3009\uc744 \ud3ec\ud568\ud574 \ub2e4\uc12f \uace1 \ubaa8\ub450 \ubc1c\ub77c\ub4dc\uc640 \ubbf8\ub514\uc5c4 \ud15c\ud3ec \ub178\ub798\ub4e4\ub85c \uc774\ub8e8\uc5b4\uc84c\ub2e4. 2012\ub144 \uc5ec\ub984\uc5d0\ub294 \ubba4\uc9c0\uceec \uc2dc\uce74\uace0\uc5d0 \uc8fc\uc5f0 \ub85d\uc2dc \ud558\ud2b8\ub85c \ucd9c\uc5f0\ud588\ub2e4. 6\uc6d4 19\uc77c\uc5d0\ub294 \ucd94\uc801\uc790 SBS \ub4dc\ub77c\ub9c8 \u300a\ucd94\uc801\uc790 THE CHASER\u300b\uc758 OST \u3008\uc88b\uc740 \uc0ac\ub78c\u3009\uc744 \ubc1c\ub9e4\ud588\ub2e4. 9\uc6d4\ubd80\ud130\ub294 SBS\uc758 \uc624\ub514\uc158 \ud504\ub85c\uadf8\ub7a8 \u300a\ub0b4 \uc0dd\uc560 \ub9c8\uc9c0\ub9c9 \uc624\ub514\uc158\u300b\uc758 \uc2ec\uc0ac\uc704\uc6d0\uc73c\ub85c \ubc1c\ud0c1 \ub3fc \ucd9c\uc5f0\ud588\ub2e4. \ub610\ud55c KBS \uc608\ub2a5 \u300a\ub0a8\uc790\uc758 \uc790\uaca9\u300b \ud328\ubc00\ub9ac \ud569\ucc3d\ub2e8 \ud3b8\uc5d0 \uc5b4\uba38\ub2c8\uc640 \ud568\uaed8 \uc131\uc545\ub2e8\uc73c\ub85c \ucd9c\uc5f0\ud588\ub2e4."} {"question": "\uc624\ub78d\ub098\ub78d\uc528\uac00 \ud669\ud6c4\ub85c\uc11c\uc758 \uad8c\uc704\ub97c \ubc15\ud0c8\ub2f9\ud55c \ucc44 \uc720\ud3d0\ub418\uc5b4 \uc0ac\ub9dd\ud55c \ub54c\ub294 \uc5b8\uc81c\uc778\uac00?", "paragraph": "\uadf8\ub7ec\ub098 \uc21c\ud589\uc744 \ud558\uba74\uc11c \uadf8\ub294 \uac00\uc815\uc801\uc73c\ub85c \ud070 \uc77c\uc744 \ub450 \ubc88 \uacaa\uc5c8\ub2e4. 1748\ub144(\uac74\ub96d 13\ub144) \uac74\ub96d\uc81c\ub294 \uc790\uc2e0\uc758 \uc815\uc2e4 \ubd80\uc778\uc778 \ud6a8\ud604\uc21c\ud669\ud6c4 \ubd80\ucc30\uc528\uc640 \ub354\ubd88\uc5b4 \ub3d9\uc21c\uc744 \ub098\uac00 \uc81c\ub0a8\uc744 \uac70\uccd0 \uacf5\uc790\uc758 \uace0\ud5a5 \uace1\ubd80\uae4c\uc9c0 \ub0b4\ub824\uac00 \uacf5\ub9bc\uc744 \ucc38\ubc30\ud558\uc600\uc73c\ub098 \ud6a8\ud604\uc21c\ud669\ud6c4\ub294 \ubc30 \uc704\uc5d0\uc11c \uc5f0\ud68c\ub97c \uc990\uae30\ub294 \ub3c4\uc911\uc5d0 \uac15\ubb3c\uc5d0 \ube60\uc838 \ubd81\uacbd\uc73c\ub85c \uae09\ud788 \uac00\ub2e4\uac00 \ub355\uc8fc\uc5d0\uc11c \uc624\ud55c\uc5d0 \uac78\ub824 \uc0ac\ub9dd\ud558\uc600\ub2e4. \uadf8\ub140\ub97c \ub9e4\uc6b0 \uc0ac\ub791\ud558\uace0 \uc544\uaf08\ub358 \uac74\ub96d\uc81c\ub294 \ud06c\uac8c \uc0c1\uc2ec\ud558\uc5ec \uc21c\ud589\uc744 \ub5a0\ub098\ub294 \uac83\uc744 \uba87 \ub144\uac04 \uc911\uc9c0\ud558\uc600\uace0 \ub3d9\uc21c\uc744 \uac08 \ub54c\uc5d0\ub3c4 \ub355\uc8fc\ub97c \uacbd\uc720\ud558\uc9c0 \uc54a\uace0 \uc6b0\ud68c\ud558\uc5ec \uac14\ub2e4. \uadf8 \ud6c4 \uac74\ub96d\uc81c\ub294 \ud55c\ud669\uadc0\ube44 \uc624\ub78d\ub098\ub78d\uc528\ub97c \ud669\ud6c4\ub85c \uc0c8\ub85c \ub9de\uc544\ub4e4\uc600\uc73c\ub098 1765\ub144(\uac74\ub96d 30\ub144) \ub124 \ubc88\uc9f8 \ub0a8\uc21c \ub54c, \uc5f0\ud68c \ud6c4 \ud568\ubd80\ub85c \uba38\ub9ac\ub97c \uc790\ub974\uace0 \uc5ec\uc2b9\uc774 \ub418\ub824 \ud558\uc600\ub2e4\ub294 \uc774\uc720\ub85c \uac74\ub96d\uc81c\uc758 \ub178\uc5ec\uc6c0\uc744 \uc0ac\uc11c \uac15\uc81c\ub85c \ubd81\uacbd\uc73c\ub85c \ubcf4\ub0b4\uc84c\ub2e4. \uadf8 \ud6c4 \uc624\ub78d\ub098\ub78d\uc528\ub294 \ud669\ud6c4\ub85c\uc11c\uc758 \uad8c\uc704\ub97c \ubc15\ud0c8\ub2f9\ud55c \ucc44 \uc720\ud3d0\ub418\uc5b4 1768\ub144(\uac74\ub96d 33\ub144)\uc5d0 \uc0ac\ub9dd\ud558\uc600\uace0 \ud669\ud6c4\uac00 \uc544\ub2cc \uadc0\ube44\ub85c\uc11c\uc758 \uc7a5\ub840\ub85c \uc9c0\ub0b4\uc9c0\uace0 \ud669\ud6c4\ub85c\uc11c\uc758 \uc2dc\ud638\ub3c4 \ubc1b\uc9c0 \ubabb\ud558\ub294 \ub4f1 \uac74\ub96d\uc81c\ub85c\ubd80\ud130 \ud640\ub300\ub97c \ubc1b\uc558\ub2e4.", "answer": "1768\ub144", "sentence": "\uadf8 \ud6c4 \uc624\ub78d\ub098\ub78d\uc528\ub294 \ud669\ud6c4\ub85c\uc11c\uc758 \uad8c\uc704\ub97c \ubc15\ud0c8\ub2f9\ud55c \ucc44 \uc720\ud3d0\ub418\uc5b4 1768\ub144 (\uac74\ub96d 33\ub144)", "paragraph_sentence": "\uadf8\ub7ec\ub098 \uc21c\ud589\uc744 \ud558\uba74\uc11c \uadf8\ub294 \uac00\uc815\uc801\uc73c\ub85c \ud070 \uc77c\uc744 \ub450 \ubc88 \uacaa\uc5c8\ub2e4. 1748\ub144(\uac74\ub96d 13\ub144) \uac74\ub96d\uc81c\ub294 \uc790\uc2e0\uc758 \uc815\uc2e4 \ubd80\uc778\uc778 \ud6a8\ud604\uc21c\ud669\ud6c4 \ubd80\ucc30\uc528\uc640 \ub354\ubd88\uc5b4 \ub3d9\uc21c\uc744 \ub098\uac00 \uc81c\ub0a8\uc744 \uac70\uccd0 \uacf5\uc790\uc758 \uace0\ud5a5 \uace1\ubd80\uae4c\uc9c0 \ub0b4\ub824\uac00 \uacf5\ub9bc\uc744 \ucc38\ubc30\ud558\uc600\uc73c\ub098 \ud6a8\ud604\uc21c\ud669\ud6c4\ub294 \ubc30 \uc704\uc5d0\uc11c \uc5f0\ud68c\ub97c \uc990\uae30\ub294 \ub3c4\uc911\uc5d0 \uac15\ubb3c\uc5d0 \ube60\uc838 \ubd81\uacbd\uc73c\ub85c \uae09\ud788 \uac00\ub2e4\uac00 \ub355\uc8fc\uc5d0\uc11c \uc624\ud55c\uc5d0 \uac78\ub824 \uc0ac\ub9dd\ud558\uc600\ub2e4. \uadf8\ub140\ub97c \ub9e4\uc6b0 \uc0ac\ub791\ud558\uace0 \uc544\uaf08\ub358 \uac74\ub96d\uc81c\ub294 \ud06c\uac8c \uc0c1\uc2ec\ud558\uc5ec \uc21c\ud589\uc744 \ub5a0\ub098\ub294 \uac83\uc744 \uba87 \ub144\uac04 \uc911\uc9c0\ud558\uc600\uace0 \ub3d9\uc21c\uc744 \uac08 \ub54c\uc5d0\ub3c4 \ub355\uc8fc\ub97c \uacbd\uc720\ud558\uc9c0 \uc54a\uace0 \uc6b0\ud68c\ud558\uc5ec \uac14\ub2e4. \uadf8 \ud6c4 \uac74\ub96d\uc81c\ub294 \ud55c\ud669\uadc0\ube44 \uc624\ub78d\ub098\ub78d\uc528\ub97c \ud669\ud6c4\ub85c \uc0c8\ub85c \ub9de\uc544\ub4e4\uc600\uc73c\ub098 1765\ub144(\uac74\ub96d 30\ub144) \ub124 \ubc88\uc9f8 \ub0a8\uc21c \ub54c, \uc5f0\ud68c \ud6c4 \ud568\ubd80\ub85c \uba38\ub9ac\ub97c \uc790\ub974\uace0 \uc5ec\uc2b9\uc774 \ub418\ub824 \ud558\uc600\ub2e4\ub294 \uc774\uc720\ub85c \uac74\ub96d\uc81c\uc758 \ub178\uc5ec\uc6c0\uc744 \uc0ac\uc11c \uac15\uc81c\ub85c \ubd81\uacbd\uc73c\ub85c \ubcf4\ub0b4\uc84c\ub2e4. \uadf8 \ud6c4 \uc624\ub78d\ub098\ub78d\uc528\ub294 \ud669\ud6c4\ub85c\uc11c\uc758 \uad8c\uc704\ub97c \ubc15\ud0c8\ub2f9\ud55c \ucc44 \uc720\ud3d0\ub418\uc5b4 1768\ub144 (\uac74\ub96d 33\ub144) \uc5d0 \uc0ac\ub9dd\ud558\uc600\uace0 \ud669\ud6c4\uac00 \uc544\ub2cc \uadc0\ube44\ub85c\uc11c\uc758 \uc7a5\ub840\ub85c \uc9c0\ub0b4\uc9c0\uace0 \ud669\ud6c4\ub85c\uc11c\uc758 \uc2dc\ud638\ub3c4 \ubc1b\uc9c0 \ubabb\ud558\ub294 \ub4f1 \uac74\ub96d\uc81c\ub85c\ubd80\ud130 \ud640\ub300\ub97c \ubc1b\uc558\ub2e4.", "paragraph_answer": "\uadf8\ub7ec\ub098 \uc21c\ud589\uc744 \ud558\uba74\uc11c \uadf8\ub294 \uac00\uc815\uc801\uc73c\ub85c \ud070 \uc77c\uc744 \ub450 \ubc88 \uacaa\uc5c8\ub2e4. 1748\ub144(\uac74\ub96d 13\ub144) \uac74\ub96d\uc81c\ub294 \uc790\uc2e0\uc758 \uc815\uc2e4 \ubd80\uc778\uc778 \ud6a8\ud604\uc21c\ud669\ud6c4 \ubd80\ucc30\uc528\uc640 \ub354\ubd88\uc5b4 \ub3d9\uc21c\uc744 \ub098\uac00 \uc81c\ub0a8\uc744 \uac70\uccd0 \uacf5\uc790\uc758 \uace0\ud5a5 \uace1\ubd80\uae4c\uc9c0 \ub0b4\ub824\uac00 \uacf5\ub9bc\uc744 \ucc38\ubc30\ud558\uc600\uc73c\ub098 \ud6a8\ud604\uc21c\ud669\ud6c4\ub294 \ubc30 \uc704\uc5d0\uc11c \uc5f0\ud68c\ub97c \uc990\uae30\ub294 \ub3c4\uc911\uc5d0 \uac15\ubb3c\uc5d0 \ube60\uc838 \ubd81\uacbd\uc73c\ub85c \uae09\ud788 \uac00\ub2e4\uac00 \ub355\uc8fc\uc5d0\uc11c \uc624\ud55c\uc5d0 \uac78\ub824 \uc0ac\ub9dd\ud558\uc600\ub2e4. \uadf8\ub140\ub97c \ub9e4\uc6b0 \uc0ac\ub791\ud558\uace0 \uc544\uaf08\ub358 \uac74\ub96d\uc81c\ub294 \ud06c\uac8c \uc0c1\uc2ec\ud558\uc5ec \uc21c\ud589\uc744 \ub5a0\ub098\ub294 \uac83\uc744 \uba87 \ub144\uac04 \uc911\uc9c0\ud558\uc600\uace0 \ub3d9\uc21c\uc744 \uac08 \ub54c\uc5d0\ub3c4 \ub355\uc8fc\ub97c \uacbd\uc720\ud558\uc9c0 \uc54a\uace0 \uc6b0\ud68c\ud558\uc5ec \uac14\ub2e4. \uadf8 \ud6c4 \uac74\ub96d\uc81c\ub294 \ud55c\ud669\uadc0\ube44 \uc624\ub78d\ub098\ub78d\uc528\ub97c \ud669\ud6c4\ub85c \uc0c8\ub85c \ub9de\uc544\ub4e4\uc600\uc73c\ub098 1765\ub144(\uac74\ub96d 30\ub144) \ub124 \ubc88\uc9f8 \ub0a8\uc21c \ub54c, \uc5f0\ud68c \ud6c4 \ud568\ubd80\ub85c \uba38\ub9ac\ub97c \uc790\ub974\uace0 \uc5ec\uc2b9\uc774 \ub418\ub824 \ud558\uc600\ub2e4\ub294 \uc774\uc720\ub85c \uac74\ub96d\uc81c\uc758 \ub178\uc5ec\uc6c0\uc744 \uc0ac\uc11c \uac15\uc81c\ub85c \ubd81\uacbd\uc73c\ub85c \ubcf4\ub0b4\uc84c\ub2e4. \uadf8 \ud6c4 \uc624\ub78d\ub098\ub78d\uc528\ub294 \ud669\ud6c4\ub85c\uc11c\uc758 \uad8c\uc704\ub97c \ubc15\ud0c8\ub2f9\ud55c \ucc44 \uc720\ud3d0\ub418\uc5b4 1768\ub144 (\uac74\ub96d 33\ub144)\uc5d0 \uc0ac\ub9dd\ud558\uc600\uace0 \ud669\ud6c4\uac00 \uc544\ub2cc \uadc0\ube44\ub85c\uc11c\uc758 \uc7a5\ub840\ub85c \uc9c0\ub0b4\uc9c0\uace0 \ud669\ud6c4\ub85c\uc11c\uc758 \uc2dc\ud638\ub3c4 \ubc1b\uc9c0 \ubabb\ud558\ub294 \ub4f1 \uac74\ub96d\uc81c\ub85c\ubd80\ud130 \ud640\ub300\ub97c \ubc1b\uc558\ub2e4.", "sentence_answer": "\uadf8 \ud6c4 \uc624\ub78d\ub098\ub78d\uc528\ub294 \ud669\ud6c4\ub85c\uc11c\uc758 \uad8c\uc704\ub97c \ubc15\ud0c8\ub2f9\ud55c \ucc44 \uc720\ud3d0\ub418\uc5b4 1768\ub144 (\uac74\ub96d 33\ub144)", "paragraph_id": "6472331-15-1", "paragraph_question": "question: \uc624\ub78d\ub098\ub78d\uc528\uac00 \ud669\ud6c4\ub85c\uc11c\uc758 \uad8c\uc704\ub97c \ubc15\ud0c8\ub2f9\ud55c \ucc44 \uc720\ud3d0\ub418\uc5b4 \uc0ac\ub9dd\ud55c \ub54c\ub294 \uc5b8\uc81c\uc778\uac00?, context: \uadf8\ub7ec\ub098 \uc21c\ud589\uc744 \ud558\uba74\uc11c \uadf8\ub294 \uac00\uc815\uc801\uc73c\ub85c \ud070 \uc77c\uc744 \ub450 \ubc88 \uacaa\uc5c8\ub2e4. 1748\ub144(\uac74\ub96d 13\ub144) \uac74\ub96d\uc81c\ub294 \uc790\uc2e0\uc758 \uc815\uc2e4 \ubd80\uc778\uc778 \ud6a8\ud604\uc21c\ud669\ud6c4 \ubd80\ucc30\uc528\uc640 \ub354\ubd88\uc5b4 \ub3d9\uc21c\uc744 \ub098\uac00 \uc81c\ub0a8\uc744 \uac70\uccd0 \uacf5\uc790\uc758 \uace0\ud5a5 \uace1\ubd80\uae4c\uc9c0 \ub0b4\ub824\uac00 \uacf5\ub9bc\uc744 \ucc38\ubc30\ud558\uc600\uc73c\ub098 \ud6a8\ud604\uc21c\ud669\ud6c4\ub294 \ubc30 \uc704\uc5d0\uc11c \uc5f0\ud68c\ub97c \uc990\uae30\ub294 \ub3c4\uc911\uc5d0 \uac15\ubb3c\uc5d0 \ube60\uc838 \ubd81\uacbd\uc73c\ub85c \uae09\ud788 \uac00\ub2e4\uac00 \ub355\uc8fc\uc5d0\uc11c \uc624\ud55c\uc5d0 \uac78\ub824 \uc0ac\ub9dd\ud558\uc600\ub2e4. \uadf8\ub140\ub97c \ub9e4\uc6b0 \uc0ac\ub791\ud558\uace0 \uc544\uaf08\ub358 \uac74\ub96d\uc81c\ub294 \ud06c\uac8c \uc0c1\uc2ec\ud558\uc5ec \uc21c\ud589\uc744 \ub5a0\ub098\ub294 \uac83\uc744 \uba87 \ub144\uac04 \uc911\uc9c0\ud558\uc600\uace0 \ub3d9\uc21c\uc744 \uac08 \ub54c\uc5d0\ub3c4 \ub355\uc8fc\ub97c \uacbd\uc720\ud558\uc9c0 \uc54a\uace0 \uc6b0\ud68c\ud558\uc5ec \uac14\ub2e4. \uadf8 \ud6c4 \uac74\ub96d\uc81c\ub294 \ud55c\ud669\uadc0\ube44 \uc624\ub78d\ub098\ub78d\uc528\ub97c \ud669\ud6c4\ub85c \uc0c8\ub85c \ub9de\uc544\ub4e4\uc600\uc73c\ub098 1765\ub144(\uac74\ub96d 30\ub144) \ub124 \ubc88\uc9f8 \ub0a8\uc21c \ub54c, \uc5f0\ud68c \ud6c4 \ud568\ubd80\ub85c \uba38\ub9ac\ub97c \uc790\ub974\uace0 \uc5ec\uc2b9\uc774 \ub418\ub824 \ud558\uc600\ub2e4\ub294 \uc774\uc720\ub85c \uac74\ub96d\uc81c\uc758 \ub178\uc5ec\uc6c0\uc744 \uc0ac\uc11c \uac15\uc81c\ub85c \ubd81\uacbd\uc73c\ub85c \ubcf4\ub0b4\uc84c\ub2e4. \uadf8 \ud6c4 \uc624\ub78d\ub098\ub78d\uc528\ub294 \ud669\ud6c4\ub85c\uc11c\uc758 \uad8c\uc704\ub97c \ubc15\ud0c8\ub2f9\ud55c \ucc44 \uc720\ud3d0\ub418\uc5b4 1768\ub144(\uac74\ub96d 33\ub144)\uc5d0 \uc0ac\ub9dd\ud558\uc600\uace0 \ud669\ud6c4\uac00 \uc544\ub2cc \uadc0\ube44\ub85c\uc11c\uc758 \uc7a5\ub840\ub85c \uc9c0\ub0b4\uc9c0\uace0 \ud669\ud6c4\ub85c\uc11c\uc758 \uc2dc\ud638\ub3c4 \ubc1b\uc9c0 \ubabb\ud558\ub294 \ub4f1 \uac74\ub96d\uc81c\ub85c\ubd80\ud130 \ud640\ub300\ub97c \ubc1b\uc558\ub2e4."} {"question": "\uc870\uc601\ub0a8\uc758 \uacb0\ud63c \uc0c1\ub300\uc790 \uc774\ub984\uc740?", "paragraph": "1982\ub144\uc5d0 \ub300\ud55c\ubbfc\uad6d\uc73c\ub85c \uadc0\uad6d\ud558\uc600\uace0 \uac00\uc218 \uae40\ub3c4\ud5a5\uacfc \ud568\uaed8 \uc74c\uc545\uc791\uc5c5\uc744 \ud558\uba74\uc11c \uadf8\uc640 \ud568\uaed8 \uc74c\ubc18\uc744 \ubc1c\ub9e4\ud558\uba70 \ud65c\ub3d9\uc744 \ud574\uc654\ub2e4. 1987\ub144\uc5d0\ub294 \uc8fc\uc2dc\uacbd \uc120\uc0dd\uc744 \uc874\uacbd\ud55c\ub2e4\ub294 \uc2e0\ub150\uc5d0 \ub530\ub77c \uc0ac\uadf9 MBC \ub4dc\ub77c\ub9c8 \u300a\ud55c\ud78c\uc0d8\u300b(MBC \ud154\ub808\ube44\uc804 \ub4dc\ub77c\ub9c8)\uc5d0\ub3c4 \ucd9c\uc5f0\ud558\uc600\ub2e4. \ud55c\ud3b8 \uc870\uc601\ub0a8\uc740 \uacb0\ud63c \ud6c4 \uc724\uc5ec\uc815 \uc0ac\uc774\uc5d0 2\ub0a8\uc744 \ub450\uc5c8\uc73c\ub098 \uc131\uaca9 \ucc28\uc774\uc640 \ubcf8\uc778\uc774 \uac00\uc815\uc5d0 \ucda9\uc2e4\ud558\uc9c0 \ubabb\ud55c \uc810\uc73c\ub85c \uacb0\uad6d 1987\ub144\uc5d0 \uc774\ud63c\uc744 \ud558\uace0 \ud55c\uad6d\uc73c\ub85c \ub3cc\uc544\uc628 \ud6c4 \uacf5\ubc31\uae30\ub97c \uac00\uc84c\ub2e4. \uacf5\ubc31 \uae30\uac04 \ub3d9\uc548 \uc870\uc601\ub0a8\uacfc \uac19\uc774 \uc9c0\ub0b4\uba70 \uc758\uc9c0\uac00 \ub418\uc5b4\uc8fc\uc5c8\ub358 \uae40\ud55c\uae38\uacfc \ud568\uaed8 \uc9c0\ub0c8\uc744 \ubb34\ub835, \uc5b4\ub290 \ub0a0 \uc2e0\ubb38\uc5d0 \ud654\uac1c\uc7a5\ud130 \uae30\uc0ac\ub97c \ubcf4\uace0 \uc18c\uc124\uac00\ub85c \ud65c\ub3d9\ud558\ub358 \uae40\ud55c\uae38\uc774 \uac00\uc0ac\ub97c \uc4f0\uac8c \ub418\uc5c8\ub2e4. \uadf8 \uac00\uc0ac\uc5d0 \uc870\uc601\ub0a8\uc774 \uc9c1\uc811 \uc791\uace1\ud55c \uba5c\ub85c\ub514\ub85c \ub178\ub798\ub97c \ub9cc\ub4e4\uc5c8\ub294\ub370 \uace1\uba85\uc744 \u3008\ud654\uac1c\uc7a5\ud130\u3009\ub85c \ud558\uace0 \uc568\ubc94\uc744 \ubc1c\ud45c\ud558\uc600\ub2e4. \ub370\ubdd4 \uc774\ud6c4 \ubc88\uc548\uace1\uc774\ub098 \uc131\uac00\ub97c \uc8fc\ub85c \ubd88\ub7ec \uad6d\ubbfc\ub4e4\uc5d0\uac8c \ub110\ub9ac \ubd88\ub9ac\ub294 \ud788\ud2b8\uace1\uc774 \uc5c6\uc5c8\ub294\ub370 \u3008\ud654\uac1c\uc7a5\ud130\u3009\uac00 \ud788\ud2b8\ud558\uba74\uc11c \uc870\uc601\ub0a8\uc744 TOP \uac00\uc218 \ubc18\uc5f4\uc5d0 \uc624\ub974\uac8c \ud558\uc5ec 80\ub144\ub300 \ucd5c\uace0\uc758 \uac00\uc694\ub85c \uc790\ub9ac\uc7a1\uc558\ub2e4. \ub610\ud55c \uc0ac\ub791\ub178\ub798 \u3008\uc0ac\ub791\uc5c6\uc778 \ub09c \ubabb\uc0b4\uc544\uc694\u3009\ub3c4 \uae40\ud55c\uae38\uc774 \uc791\uc0ac\ud558\uc600\uace0 \uc774 \uace1\ub3c4 \ud788\ud2b8\ud558\uc600\ub2e4. 1988\ub144\uc5d0 \ub450 \uace1\uc774\ub098 \ud788\ud2b8\ud558\uac8c \ub418\uc5b4 \uc120\ud48d\uc801\uc778 \uc778\uae30\ub97c \uc5bb\uace0 \uc815\uc0c1\uc5d0 \uc624\ub978 \uc870\uc601\ub0a8\uc740 \uc5ec\ub7ec \uacf5\uc5f0\uacfc \ubb34\ub300\uc5d0 \ucd9c\uc5f0\ud558\uc5ec \uc778\uae30\uac00\ub3c4\ub97c \ub2ec\ub9ac\uac8c \ub418\uc5c8\ub2e4.", "answer": "\uc724\uc5ec\uc815", "sentence": "\uc870\uc601\ub0a8\uc740 \uacb0\ud63c \ud6c4 \uc724\uc5ec\uc815 \uc0ac\uc774\uc5d0 2\ub0a8\uc744 \ub450\uc5c8\uc73c\ub098 \uc131\uaca9 \ucc28\uc774\uc640 \ubcf8\uc778\uc774 \uac00\uc815\uc5d0 \ucda9\uc2e4\ud558\uc9c0 \ubabb\ud55c \uc810\uc73c\ub85c \uacb0\uad6d 1987\ub144\uc5d0 \uc774\ud63c\uc744 \ud558\uace0 \ud55c\uad6d\uc73c\ub85c \ub3cc\uc544\uc628 \ud6c4 \uacf5\ubc31\uae30\ub97c \uac00\uc84c\ub2e4.", "paragraph_sentence": "1982\ub144\uc5d0 \ub300\ud55c\ubbfc\uad6d\uc73c\ub85c \uadc0\uad6d\ud558\uc600\uace0 \uac00\uc218 \uae40\ub3c4\ud5a5\uacfc \ud568\uaed8 \uc74c\uc545\uc791\uc5c5\uc744 \ud558\uba74\uc11c \uadf8\uc640 \ud568\uaed8 \uc74c\ubc18\uc744 \ubc1c\ub9e4\ud558\uba70 \ud65c\ub3d9\uc744 \ud574\uc654\ub2e4. 1987\ub144\uc5d0\ub294 \uc8fc\uc2dc\uacbd \uc120\uc0dd\uc744 \uc874\uacbd\ud55c\ub2e4\ub294 \uc2e0\ub150\uc5d0 \ub530\ub77c \uc0ac\uadf9 MBC \ub4dc\ub77c\ub9c8 \u300a\ud55c\ud78c\uc0d8\u300b(MBC \ud154\ub808\ube44\uc804 \ub4dc\ub77c\ub9c8)\uc5d0\ub3c4 \ucd9c\uc5f0\ud558\uc600\ub2e4. \ud55c\ud3b8 \uc870\uc601\ub0a8\uc740 \uacb0\ud63c \ud6c4 \uc724\uc5ec\uc815 \uc0ac\uc774\uc5d0 2\ub0a8\uc744 \ub450\uc5c8\uc73c\ub098 \uc131\uaca9 \ucc28\uc774\uc640 \ubcf8\uc778\uc774 \uac00\uc815\uc5d0 \ucda9\uc2e4\ud558\uc9c0 \ubabb\ud55c \uc810\uc73c\ub85c \uacb0\uad6d 1987\ub144\uc5d0 \uc774\ud63c\uc744 \ud558\uace0 \ud55c\uad6d\uc73c\ub85c \ub3cc\uc544\uc628 \ud6c4 \uacf5\ubc31\uae30\ub97c \uac00\uc84c\ub2e4. \uacf5\ubc31 \uae30\uac04 \ub3d9\uc548 \uc870\uc601\ub0a8\uacfc \uac19\uc774 \uc9c0\ub0b4\uba70 \uc758\uc9c0\uac00 \ub418\uc5b4\uc8fc\uc5c8\ub358 \uae40\ud55c\uae38\uacfc \ud568\uaed8 \uc9c0\ub0c8\uc744 \ubb34\ub835, \uc5b4\ub290 \ub0a0 \uc2e0\ubb38\uc5d0 \ud654\uac1c\uc7a5\ud130 \uae30\uc0ac\ub97c \ubcf4\uace0 \uc18c\uc124\uac00\ub85c \ud65c\ub3d9\ud558\ub358 \uae40\ud55c\uae38\uc774 \uac00\uc0ac\ub97c \uc4f0\uac8c \ub418\uc5c8\ub2e4. \uadf8 \uac00\uc0ac\uc5d0 \uc870\uc601\ub0a8\uc774 \uc9c1\uc811 \uc791\uace1\ud55c \uba5c\ub85c\ub514\ub85c \ub178\ub798\ub97c \ub9cc\ub4e4\uc5c8\ub294\ub370 \uace1\uba85\uc744 \u3008\ud654\uac1c\uc7a5\ud130\u3009\ub85c \ud558\uace0 \uc568\ubc94\uc744 \ubc1c\ud45c\ud558\uc600\ub2e4. \ub370\ubdd4 \uc774\ud6c4 \ubc88\uc548\uace1\uc774\ub098 \uc131\uac00\ub97c \uc8fc\ub85c \ubd88\ub7ec \uad6d\ubbfc\ub4e4\uc5d0\uac8c \ub110\ub9ac \ubd88\ub9ac\ub294 \ud788\ud2b8\uace1\uc774 \uc5c6\uc5c8\ub294\ub370 \u3008\ud654\uac1c\uc7a5\ud130\u3009\uac00 \ud788\ud2b8\ud558\uba74\uc11c \uc870\uc601\ub0a8\uc744 TOP \uac00\uc218 \ubc18\uc5f4\uc5d0 \uc624\ub974\uac8c \ud558\uc5ec 80\ub144\ub300 \ucd5c\uace0\uc758 \uac00\uc694\ub85c \uc790\ub9ac\uc7a1\uc558\ub2e4. \ub610\ud55c \uc0ac\ub791\ub178\ub798 \u3008\uc0ac\ub791\uc5c6\uc778 \ub09c \ubabb\uc0b4\uc544\uc694\u3009\ub3c4 \uae40\ud55c\uae38\uc774 \uc791\uc0ac\ud558\uc600\uace0 \uc774 \uace1\ub3c4 \ud788\ud2b8\ud558\uc600\ub2e4. 1988\ub144\uc5d0 \ub450 \uace1\uc774\ub098 \ud788\ud2b8\ud558\uac8c \ub418\uc5b4 \uc120\ud48d\uc801\uc778 \uc778\uae30\ub97c \uc5bb\uace0 \uc815\uc0c1\uc5d0 \uc624\ub978 \uc870\uc601\ub0a8\uc740 \uc5ec\ub7ec \uacf5\uc5f0\uacfc \ubb34\ub300\uc5d0 \ucd9c\uc5f0\ud558\uc5ec \uc778\uae30\uac00\ub3c4\ub97c \ub2ec\ub9ac\uac8c \ub418\uc5c8\ub2e4.", "paragraph_answer": "1982\ub144\uc5d0 \ub300\ud55c\ubbfc\uad6d\uc73c\ub85c \uadc0\uad6d\ud558\uc600\uace0 \uac00\uc218 \uae40\ub3c4\ud5a5\uacfc \ud568\uaed8 \uc74c\uc545\uc791\uc5c5\uc744 \ud558\uba74\uc11c \uadf8\uc640 \ud568\uaed8 \uc74c\ubc18\uc744 \ubc1c\ub9e4\ud558\uba70 \ud65c\ub3d9\uc744 \ud574\uc654\ub2e4. 1987\ub144\uc5d0\ub294 \uc8fc\uc2dc\uacbd \uc120\uc0dd\uc744 \uc874\uacbd\ud55c\ub2e4\ub294 \uc2e0\ub150\uc5d0 \ub530\ub77c \uc0ac\uadf9 MBC \ub4dc\ub77c\ub9c8 \u300a\ud55c\ud78c\uc0d8\u300b(MBC \ud154\ub808\ube44\uc804 \ub4dc\ub77c\ub9c8)\uc5d0\ub3c4 \ucd9c\uc5f0\ud558\uc600\ub2e4. \ud55c\ud3b8 \uc870\uc601\ub0a8\uc740 \uacb0\ud63c \ud6c4 \uc724\uc5ec\uc815 \uc0ac\uc774\uc5d0 2\ub0a8\uc744 \ub450\uc5c8\uc73c\ub098 \uc131\uaca9 \ucc28\uc774\uc640 \ubcf8\uc778\uc774 \uac00\uc815\uc5d0 \ucda9\uc2e4\ud558\uc9c0 \ubabb\ud55c \uc810\uc73c\ub85c \uacb0\uad6d 1987\ub144\uc5d0 \uc774\ud63c\uc744 \ud558\uace0 \ud55c\uad6d\uc73c\ub85c \ub3cc\uc544\uc628 \ud6c4 \uacf5\ubc31\uae30\ub97c \uac00\uc84c\ub2e4. \uacf5\ubc31 \uae30\uac04 \ub3d9\uc548 \uc870\uc601\ub0a8\uacfc \uac19\uc774 \uc9c0\ub0b4\uba70 \uc758\uc9c0\uac00 \ub418\uc5b4\uc8fc\uc5c8\ub358 \uae40\ud55c\uae38\uacfc \ud568\uaed8 \uc9c0\ub0c8\uc744 \ubb34\ub835, \uc5b4\ub290 \ub0a0 \uc2e0\ubb38\uc5d0 \ud654\uac1c\uc7a5\ud130 \uae30\uc0ac\ub97c \ubcf4\uace0 \uc18c\uc124\uac00\ub85c \ud65c\ub3d9\ud558\ub358 \uae40\ud55c\uae38\uc774 \uac00\uc0ac\ub97c \uc4f0\uac8c \ub418\uc5c8\ub2e4. \uadf8 \uac00\uc0ac\uc5d0 \uc870\uc601\ub0a8\uc774 \uc9c1\uc811 \uc791\uace1\ud55c \uba5c\ub85c\ub514\ub85c \ub178\ub798\ub97c \ub9cc\ub4e4\uc5c8\ub294\ub370 \uace1\uba85\uc744 \u3008\ud654\uac1c\uc7a5\ud130\u3009\ub85c \ud558\uace0 \uc568\ubc94\uc744 \ubc1c\ud45c\ud558\uc600\ub2e4. \ub370\ubdd4 \uc774\ud6c4 \ubc88\uc548\uace1\uc774\ub098 \uc131\uac00\ub97c \uc8fc\ub85c \ubd88\ub7ec \uad6d\ubbfc\ub4e4\uc5d0\uac8c \ub110\ub9ac \ubd88\ub9ac\ub294 \ud788\ud2b8\uace1\uc774 \uc5c6\uc5c8\ub294\ub370 \u3008\ud654\uac1c\uc7a5\ud130\u3009\uac00 \ud788\ud2b8\ud558\uba74\uc11c \uc870\uc601\ub0a8\uc744 TOP \uac00\uc218 \ubc18\uc5f4\uc5d0 \uc624\ub974\uac8c \ud558\uc5ec 80\ub144\ub300 \ucd5c\uace0\uc758 \uac00\uc694\ub85c \uc790\ub9ac\uc7a1\uc558\ub2e4. \ub610\ud55c \uc0ac\ub791\ub178\ub798 \u3008\uc0ac\ub791\uc5c6\uc778 \ub09c \ubabb\uc0b4\uc544\uc694\u3009\ub3c4 \uae40\ud55c\uae38\uc774 \uc791\uc0ac\ud558\uc600\uace0 \uc774 \uace1\ub3c4 \ud788\ud2b8\ud558\uc600\ub2e4. 1988\ub144\uc5d0 \ub450 \uace1\uc774\ub098 \ud788\ud2b8\ud558\uac8c \ub418\uc5b4 \uc120\ud48d\uc801\uc778 \uc778\uae30\ub97c \uc5bb\uace0 \uc815\uc0c1\uc5d0 \uc624\ub978 \uc870\uc601\ub0a8\uc740 \uc5ec\ub7ec \uacf5\uc5f0\uacfc \ubb34\ub300\uc5d0 \ucd9c\uc5f0\ud558\uc5ec \uc778\uae30\uac00\ub3c4\ub97c \ub2ec\ub9ac\uac8c \ub418\uc5c8\ub2e4.", "sentence_answer": "\uc870\uc601\ub0a8\uc740 \uacb0\ud63c \ud6c4 \uc724\uc5ec\uc815 \uc0ac\uc774\uc5d0 2\ub0a8\uc744 \ub450\uc5c8\uc73c\ub098 \uc131\uaca9 \ucc28\uc774\uc640 \ubcf8\uc778\uc774 \uac00\uc815\uc5d0 \ucda9\uc2e4\ud558\uc9c0 \ubabb\ud55c \uc810\uc73c\ub85c \uacb0\uad6d 1987\ub144\uc5d0 \uc774\ud63c\uc744 \ud558\uace0 \ud55c\uad6d\uc73c\ub85c \ub3cc\uc544\uc628 \ud6c4 \uacf5\ubc31\uae30\ub97c \uac00\uc84c\ub2e4.", "paragraph_id": "6553415-0-1", "paragraph_question": "question: \uc870\uc601\ub0a8\uc758 \uacb0\ud63c \uc0c1\ub300\uc790 \uc774\ub984\uc740?, context: 1982\ub144\uc5d0 \ub300\ud55c\ubbfc\uad6d\uc73c\ub85c \uadc0\uad6d\ud558\uc600\uace0 \uac00\uc218 \uae40\ub3c4\ud5a5\uacfc \ud568\uaed8 \uc74c\uc545\uc791\uc5c5\uc744 \ud558\uba74\uc11c \uadf8\uc640 \ud568\uaed8 \uc74c\ubc18\uc744 \ubc1c\ub9e4\ud558\uba70 \ud65c\ub3d9\uc744 \ud574\uc654\ub2e4. 1987\ub144\uc5d0\ub294 \uc8fc\uc2dc\uacbd \uc120\uc0dd\uc744 \uc874\uacbd\ud55c\ub2e4\ub294 \uc2e0\ub150\uc5d0 \ub530\ub77c \uc0ac\uadf9 MBC \ub4dc\ub77c\ub9c8 \u300a\ud55c\ud78c\uc0d8\u300b(MBC \ud154\ub808\ube44\uc804 \ub4dc\ub77c\ub9c8)\uc5d0\ub3c4 \ucd9c\uc5f0\ud558\uc600\ub2e4. \ud55c\ud3b8 \uc870\uc601\ub0a8\uc740 \uacb0\ud63c \ud6c4 \uc724\uc5ec\uc815 \uc0ac\uc774\uc5d0 2\ub0a8\uc744 \ub450\uc5c8\uc73c\ub098 \uc131\uaca9 \ucc28\uc774\uc640 \ubcf8\uc778\uc774 \uac00\uc815\uc5d0 \ucda9\uc2e4\ud558\uc9c0 \ubabb\ud55c \uc810\uc73c\ub85c \uacb0\uad6d 1987\ub144\uc5d0 \uc774\ud63c\uc744 \ud558\uace0 \ud55c\uad6d\uc73c\ub85c \ub3cc\uc544\uc628 \ud6c4 \uacf5\ubc31\uae30\ub97c \uac00\uc84c\ub2e4. \uacf5\ubc31 \uae30\uac04 \ub3d9\uc548 \uc870\uc601\ub0a8\uacfc \uac19\uc774 \uc9c0\ub0b4\uba70 \uc758\uc9c0\uac00 \ub418\uc5b4\uc8fc\uc5c8\ub358 \uae40\ud55c\uae38\uacfc \ud568\uaed8 \uc9c0\ub0c8\uc744 \ubb34\ub835, \uc5b4\ub290 \ub0a0 \uc2e0\ubb38\uc5d0 \ud654\uac1c\uc7a5\ud130 \uae30\uc0ac\ub97c \ubcf4\uace0 \uc18c\uc124\uac00\ub85c \ud65c\ub3d9\ud558\ub358 \uae40\ud55c\uae38\uc774 \uac00\uc0ac\ub97c \uc4f0\uac8c \ub418\uc5c8\ub2e4. \uadf8 \uac00\uc0ac\uc5d0 \uc870\uc601\ub0a8\uc774 \uc9c1\uc811 \uc791\uace1\ud55c \uba5c\ub85c\ub514\ub85c \ub178\ub798\ub97c \ub9cc\ub4e4\uc5c8\ub294\ub370 \uace1\uba85\uc744 \u3008\ud654\uac1c\uc7a5\ud130\u3009\ub85c \ud558\uace0 \uc568\ubc94\uc744 \ubc1c\ud45c\ud558\uc600\ub2e4. \ub370\ubdd4 \uc774\ud6c4 \ubc88\uc548\uace1\uc774\ub098 \uc131\uac00\ub97c \uc8fc\ub85c \ubd88\ub7ec \uad6d\ubbfc\ub4e4\uc5d0\uac8c \ub110\ub9ac \ubd88\ub9ac\ub294 \ud788\ud2b8\uace1\uc774 \uc5c6\uc5c8\ub294\ub370 \u3008\ud654\uac1c\uc7a5\ud130\u3009\uac00 \ud788\ud2b8\ud558\uba74\uc11c \uc870\uc601\ub0a8\uc744 TOP \uac00\uc218 \ubc18\uc5f4\uc5d0 \uc624\ub974\uac8c \ud558\uc5ec 80\ub144\ub300 \ucd5c\uace0\uc758 \uac00\uc694\ub85c \uc790\ub9ac\uc7a1\uc558\ub2e4. \ub610\ud55c \uc0ac\ub791\ub178\ub798 \u3008\uc0ac\ub791\uc5c6\uc778 \ub09c \ubabb\uc0b4\uc544\uc694\u3009\ub3c4 \uae40\ud55c\uae38\uc774 \uc791\uc0ac\ud558\uc600\uace0 \uc774 \uace1\ub3c4 \ud788\ud2b8\ud558\uc600\ub2e4. 1988\ub144\uc5d0 \ub450 \uace1\uc774\ub098 \ud788\ud2b8\ud558\uac8c \ub418\uc5b4 \uc120\ud48d\uc801\uc778 \uc778\uae30\ub97c \uc5bb\uace0 \uc815\uc0c1\uc5d0 \uc624\ub978 \uc870\uc601\ub0a8\uc740 \uc5ec\ub7ec \uacf5\uc5f0\uacfc \ubb34\ub300\uc5d0 \ucd9c\uc5f0\ud558\uc5ec \uc778\uae30\uac00\ub3c4\ub97c \ub2ec\ub9ac\uac8c \ub418\uc5c8\ub2e4."} {"question": "\ud3c9\uc591\uac10\uc625 \ud22c\uc625 \ub2f9\uc2dc \uc7a5\uc77c\ud658\uc758 \uc720\uc5b8\uc744 \ub4e3\uace0 \uadf8\uc758 \uc21c\uad6d\uacfc\uc815\uc744 \uae30\uc220\ud55c \uc778\ubb3c\uc740?", "paragraph": "1\uc8fc\uc77c \uc774\uc0c1\uc774\ub098 \uc7a5\uc77c\ud658 \ub4f1 \ud68c\uc6d0\ub4e4\uc744 \uace0\ubb38\ud558\uc600\ub294\ub370, \ub9cc\uc8fc\uc5d0\uc11c \uc628 \ubc31\uc138\ube48\uc774 \uace0\ubb38\uc5d0 \uad74\ubcf5\ud558\uc5ec \uc870\uc9c1\uc5d0 \ub300\ud574 \uc2e4\ud1a0\ud558\uac8c \ub418\uc5c8\ub2e4. \uc774\ub9ac\ud558\uc5ec \ud68c\uc758\uc5d0 \ucc38\uc11d\ud558\uc9c0 \uc54a\uc558\ub358 \ud68c\uc6d0\uc744 \ube44\ub86f\ud574\uc11c \ud68c\uc6d0\ub4e4\uc758 \uc9c0\uc778\ub4e4\uae4c\uc9c0 \ubaa8\ub450 \uccb4\ud3ec\ub418\uc5c8\uc73c\ub098 \uc785\uac74\uc740 30\uba85 \uc815\ub3c4\uac00 \ub418\uc5c8\ub2e4. \ud68c\uc7a5 \uc7a5\uc77c\ud658\uc744 \ube44\ub86f\ud574\uc154 \ubc30\ubbfc\uc218, \uae40\ud615\uc9c1, \uac15\uc11d\ubd09 \ub4f1\uc740 \ud3c9\uc591\uac10\uc625\uc5d0\uc11c \uc625\uace0\ub97c \uce58\ub800\ub2e4. \ub3c5\uc2e4\ud55c \uae30\ub3c5\uad50 \uc2e0\uc790\ub85c\uc11c \ub3c5\ub9bd\uc744 \uc5fc\uc6d0\ud558\uc600\ub358 \uc7a5\uc77c\ud658\uc740 \ubc30\ubbfc\uc218\uc5d0\uac8c \uc720\uc5b8\uc744 \ub0a8\uae30\uace0 \uace0\ubb38\uc73c\ub85c \uc0ac\ub9dd\ud558\uc600\ub2e4. \ubc30\ubbfc\uc218\ub294 \ud68c\uc7a5 \uc7a5\uc77c\ud658\uc758 \uc21c\uad6d\uacfc\uc815\uc5d0 \ub300\ud574 \uae30\uc220\ud558\uc600\ub2e4. \"\uc77c\uc8fc\uc77c \ud6c4\uc5d0, \uadf8\ub4e4\uc740 \uc6b0\ub9ac\uc758 \ubc29\uc744 \ubc14\uafb8\uc5b4 \ub098\ub97c \uc7a5\uc77c\ud658\uc774 \uc788\ub294 \ubc29\uc5d0. \ub123\uc5c8\ub2e4. \uc870\uc9c1\uc758 \ud68c\uc7a5\uc774\uc5c8\ub358 \uc77c\ud658\uc740 \ub204\uad6c\ubcf4\ub2e4\ub3c4 \uace0\ubb38\uc744 \uc2ec\ud558\uac8c \ubc1b\uc558\ub2e4. \ud558\ub8e8\ub294 \ud615\uc0ac\uc2e4\uc5d0 \ub04c\ub824 \uac00 \ub3cc\uc544\uc62c \uae30\ubbf8\uac00 \uc548 \ubcf4\uc774\ub354\ub2c8 \ubc18\uc8fd\uc74c\uc774 \ub418\uc5b4\uc11c \ub3cc\uc544\uc654\ub2e4. \ub098\ub294 \uadf8\ub97c \uc548\uace0 \ubcf4\uc0b4\ud3b4\uc8fc\uc5c8\ub2e4. \uadf8\ub294 \uc6b8\uba70 \ub098\uc5d0\uac8c \ub9d0\ud588\ub2e4.\u201c\ubbfc\uc218\uc57c, \uc774\ub7f0 \ub9d0\ud574\uc11c \ubbf8\uc548\ud558\uc9c0\ub9cc, \ub098\ub294 \ub354 \uc774\uc0c1 \uc0b4 \uc218 \uc788\uc744 \uac83 \uac19\uc9c0 \uc54a\uc544. \ub098\ub294 \uace7 \uc8fd\uac8c \ub420 \uac70\uc57c. \uc8fc\ub2d8\uacfc \ud3c9\ud654\ub85c\uc6b4 \ucc9c\uad6d\uc5d0 \uac19\uc774 \uc788\uc744 \uc218 \uc788\ub2e4\uba74 \uadf8\uac8c \ub354 \ub098\uc744 \uac70\uc57c, \ud558\uc9c0\ub9cc \uc5b4\ub5bb\uac8c \ub108\ub97c \ud63c\uc790 \uace0\uc0dd\ud558\ub3c4\ub85d \ub450\uace0 \uac00\uc9c0? \ub108\ubb34\ub098\ub3c4 \uc548\ud0c0\uae5d\uad6c\ub098. \ub0b4\uac00 \uc5c6\uc5b4\ub3c4 \uc798 \ud560 \uc904 \ubbff\uc5b4. \uc8fc\ub2d8\uc758 \uacc1\uc5d0\uc11c \ud56d\uc0c1 \ub110 \uc704\ud574 \uae30\ub3c4\ud574 \uc904\uac8c. \uc6b0\ub9ac\uc758 \ub3c5\ub9bd\uc744 \uc7c1\ucde8\ud558\ub294 \uadf8\ub0a0\uae4c\uc9c0 \uc6a9\uae30\ub97c \uc783\uc9c0 \ub9c8, \ubbfc\uc218\uc57c. \uc5b8\uc81c \uc8fd\ub290\ub0d0\uac00 \ubb38\uc81c\uc9c0 \uc5b8\uc820\uac00 \uc6b0\ub9ac\ub294 \ub2e4 \uac19\uc740 \uacf3\uc5d0\uc11c \ub9cc\ub0a0 \uac70\uc57c. \uac12\uc9c0\uac8c \uc8fd\ub294 \uac83\uc774 \ub73b \uc5c6\uc774 \uc0ac\ub294 \uac83\ubcf4\ub2e4 \uac00\uce58\uac00 \uc788\uc744 \uac70\uc57c. \uadf8\ub7ec\ub2c8\uae4c, \uac12\uc9c4 \ub124 \uc778\uc0dd\uc744 \uc18c\uc911\ud788 \ud574\ub77c. \uac00\uce58 \uc5c6\ub294 \uac83\uc5d0 \ubaa9\uc228\uc744 \uac78\uc9c0 \ub9d0\uc544\ub77c. \ub124\uac00 \ubaa9\uc228\uc744 \uc783\uc9c0 \uc54a\uace0\ub3c4 \uc790\uc720\ub97c \ucc3e\uc744 \uc218 \uc788\ub3c4\ub85d \uc8fc\ub2d8\uaed8 \uae30\ub3c4\ud558\ub9c8. \uc5b4\uba38\ub2c8\uaed8 \ub124 \uc544\ubc84\uc9c0\uc758 \ub73b\uc744 \uc774\ub8e8\ub294 \ubaa8\uc2b5\uc744 \ubcf4\uc5ec\ub4dc\ub824\ub77c. \ubbfc\uc218\uc57c \uc5b4\ub5bb\uac8c \ub110 \ub5a0\ub098\uc9c0, \uc5b4\ub5bb\uac8c \ub110 \ub5a0\ub098\uc9c0?\u201d\uadf8\ub294 \uc5b4\ub9b0\uc544\uc774\ucc98\ub7fc \uc6b8\uace0 \uc788\uc5c8\uace0 \ub098\ub294 \uac10\uc815\uc5d0 \uce58\uc6b0\uce58\uc9c0 \ub9d0\ub77c\uace0 \ud588\ub2e4. \u201c\uc9c0\uae08\uc740 \uc6b8 \ub54c\uac00 \uc544\ub2c8\ub77c \uc2f8\uc6b8 \ub54c\uc774\ub2e4. \ub124\uac00 \uc8fd\ub354\ub77c\ub3c4 \ub0b4\uac00 \ubaa8\ub4e0 \uac78 \ucc45\uc784\uc9c0\ub9c8. \ud558\uc9c0\ub9cc \ub098\ub294 \ub124\uac00 \uc8fd\uc9c0 \uc54a\uc744 \uac70\ub77c\uace0 \ubbff\ub294\ub2e4. \ub108\uc640 \ub098\ub294 \uac19\uc774 \uc77c\ud558\ub2e4\uac00 \uac19\uc774 \uc8fd\uc5b4\uc57c \ud574. \uc8fc\ub2d8\uc740 \uc6b0\ub9ac\uac00 \ubb34\uc5c7\uc744 \ud574\uc57c \ud558\ub294\uc9c0 \uc54c\uace0 \uacc4\uc2dc\uc7ce\uc544. \uc77c\ud658\uc544, \ub204\uc6cc\uc11c \uc880 \uc26c\ub3c4\ub85d \ud574. \ub10c \ub108\ubb34 \uc9c0\uccd0 \uc788\uc5b4.\u201d", "answer": "\ubc30\ubbfc\uc218", "sentence": "\ud68c\uc7a5 \uc7a5\uc77c\ud658\uc744 \ube44\ub86f\ud574\uc154 \ubc30\ubbfc\uc218 ,", "paragraph_sentence": "1\uc8fc\uc77c \uc774\uc0c1\uc774\ub098 \uc7a5\uc77c\ud658 \ub4f1 \ud68c\uc6d0\ub4e4\uc744 \uace0\ubb38\ud558\uc600\ub294\ub370, \ub9cc\uc8fc\uc5d0\uc11c \uc628 \ubc31\uc138\ube48\uc774 \uace0\ubb38\uc5d0 \uad74\ubcf5\ud558\uc5ec \uc870\uc9c1\uc5d0 \ub300\ud574 \uc2e4\ud1a0\ud558\uac8c \ub418\uc5c8\ub2e4. \uc774\ub9ac\ud558\uc5ec \ud68c\uc758\uc5d0 \ucc38\uc11d\ud558\uc9c0 \uc54a\uc558\ub358 \ud68c\uc6d0\uc744 \ube44\ub86f\ud574\uc11c \ud68c\uc6d0\ub4e4\uc758 \uc9c0\uc778\ub4e4\uae4c\uc9c0 \ubaa8\ub450 \uccb4\ud3ec\ub418\uc5c8\uc73c\ub098 \uc785\uac74\uc740 30\uba85 \uc815\ub3c4\uac00 \ub418\uc5c8\ub2e4. \ud68c\uc7a5 \uc7a5\uc77c\ud658\uc744 \ube44\ub86f\ud574\uc154 \ubc30\ubbfc\uc218 , \uae40\ud615\uc9c1, \uac15\uc11d\ubd09 \ub4f1\uc740 \ud3c9\uc591\uac10\uc625\uc5d0\uc11c \uc625\uace0\ub97c \uce58\ub800\ub2e4. \ub3c5\uc2e4\ud55c \uae30\ub3c5\uad50 \uc2e0\uc790\ub85c\uc11c \ub3c5\ub9bd\uc744 \uc5fc\uc6d0\ud558\uc600\ub358 \uc7a5\uc77c\ud658\uc740 \ubc30\ubbfc\uc218\uc5d0\uac8c \uc720\uc5b8\uc744 \ub0a8\uae30\uace0 \uace0\ubb38\uc73c\ub85c \uc0ac\ub9dd\ud558\uc600\ub2e4. \ubc30\ubbfc\uc218\ub294 \ud68c\uc7a5 \uc7a5\uc77c\ud658\uc758 \uc21c\uad6d\uacfc\uc815\uc5d0 \ub300\ud574 \uae30\uc220\ud558\uc600\ub2e4. \"\uc77c\uc8fc\uc77c \ud6c4\uc5d0, \uadf8\ub4e4\uc740 \uc6b0\ub9ac\uc758 \ubc29\uc744 \ubc14\uafb8\uc5b4 \ub098\ub97c \uc7a5\uc77c\ud658\uc774 \uc788\ub294 \ubc29\uc5d0. \ub123\uc5c8\ub2e4. \uc870\uc9c1\uc758 \ud68c\uc7a5\uc774\uc5c8\ub358 \uc77c\ud658\uc740 \ub204\uad6c\ubcf4\ub2e4\ub3c4 \uace0\ubb38\uc744 \uc2ec\ud558\uac8c \ubc1b\uc558\ub2e4. \ud558\ub8e8\ub294 \ud615\uc0ac\uc2e4\uc5d0 \ub04c\ub824 \uac00 \ub3cc\uc544\uc62c \uae30\ubbf8\uac00 \uc548 \ubcf4\uc774\ub354\ub2c8 \ubc18\uc8fd\uc74c\uc774 \ub418\uc5b4\uc11c \ub3cc\uc544\uc654\ub2e4. \ub098\ub294 \uadf8\ub97c \uc548\uace0 \ubcf4\uc0b4\ud3b4\uc8fc\uc5c8\ub2e4. \uadf8\ub294 \uc6b8\uba70 \ub098\uc5d0\uac8c \ub9d0\ud588\ub2e4. \u201c\ubbfc\uc218\uc57c, \uc774\ub7f0 \ub9d0\ud574\uc11c \ubbf8\uc548\ud558\uc9c0\ub9cc, \ub098\ub294 \ub354 \uc774\uc0c1 \uc0b4 \uc218 \uc788\uc744 \uac83 \uac19\uc9c0 \uc54a\uc544. \ub098\ub294 \uace7 \uc8fd\uac8c \ub420 \uac70\uc57c. \uc8fc\ub2d8\uacfc \ud3c9\ud654\ub85c\uc6b4 \ucc9c\uad6d\uc5d0 \uac19\uc774 \uc788\uc744 \uc218 \uc788\ub2e4\uba74 \uadf8\uac8c \ub354 \ub098\uc744 \uac70\uc57c, \ud558\uc9c0\ub9cc \uc5b4\ub5bb\uac8c \ub108\ub97c \ud63c\uc790 \uace0\uc0dd\ud558\ub3c4\ub85d \ub450\uace0 \uac00\uc9c0? \ub108\ubb34\ub098\ub3c4 \uc548\ud0c0\uae5d\uad6c\ub098. \ub0b4\uac00 \uc5c6\uc5b4\ub3c4 \uc798 \ud560 \uc904 \ubbff\uc5b4. \uc8fc\ub2d8\uc758 \uacc1\uc5d0\uc11c \ud56d\uc0c1 \ub110 \uc704\ud574 \uae30\ub3c4\ud574 \uc904\uac8c. \uc6b0\ub9ac\uc758 \ub3c5\ub9bd\uc744 \uc7c1\ucde8\ud558\ub294 \uadf8\ub0a0\uae4c\uc9c0 \uc6a9\uae30\ub97c \uc783\uc9c0 \ub9c8, \ubbfc\uc218\uc57c. \uc5b8\uc81c \uc8fd\ub290\ub0d0\uac00 \ubb38\uc81c\uc9c0 \uc5b8\uc820\uac00 \uc6b0\ub9ac\ub294 \ub2e4 \uac19\uc740 \uacf3\uc5d0\uc11c \ub9cc\ub0a0 \uac70\uc57c. \uac12\uc9c0\uac8c \uc8fd\ub294 \uac83\uc774 \ub73b \uc5c6\uc774 \uc0ac\ub294 \uac83\ubcf4\ub2e4 \uac00\uce58\uac00 \uc788\uc744 \uac70\uc57c. \uadf8\ub7ec\ub2c8\uae4c, \uac12\uc9c4 \ub124 \uc778\uc0dd\uc744 \uc18c\uc911\ud788 \ud574\ub77c. \uac00\uce58 \uc5c6\ub294 \uac83\uc5d0 \ubaa9\uc228\uc744 \uac78\uc9c0 \ub9d0\uc544\ub77c. \ub124\uac00 \ubaa9\uc228\uc744 \uc783\uc9c0 \uc54a\uace0\ub3c4 \uc790\uc720\ub97c \ucc3e\uc744 \uc218 \uc788\ub3c4\ub85d \uc8fc\ub2d8\uaed8 \uae30\ub3c4\ud558\ub9c8. \uc5b4\uba38\ub2c8\uaed8 \ub124 \uc544\ubc84\uc9c0\uc758 \ub73b\uc744 \uc774\ub8e8\ub294 \ubaa8\uc2b5\uc744 \ubcf4\uc5ec\ub4dc\ub824\ub77c. \ubbfc\uc218\uc57c \uc5b4\ub5bb\uac8c \ub110 \ub5a0\ub098\uc9c0, \uc5b4\ub5bb\uac8c \ub110 \ub5a0\ub098\uc9c0?\u201d\uadf8\ub294 \uc5b4\ub9b0\uc544\uc774\ucc98\ub7fc \uc6b8\uace0 \uc788\uc5c8\uace0 \ub098\ub294 \uac10\uc815\uc5d0 \uce58\uc6b0\uce58\uc9c0 \ub9d0\ub77c\uace0 \ud588\ub2e4. \u201c\uc9c0\uae08\uc740 \uc6b8 \ub54c\uac00 \uc544\ub2c8\ub77c \uc2f8\uc6b8 \ub54c\uc774\ub2e4. \ub124\uac00 \uc8fd\ub354\ub77c\ub3c4 \ub0b4\uac00 \ubaa8\ub4e0 \uac78 \ucc45\uc784\uc9c0\ub9c8. \ud558\uc9c0\ub9cc \ub098\ub294 \ub124\uac00 \uc8fd\uc9c0 \uc54a\uc744 \uac70\ub77c\uace0 \ubbff\ub294\ub2e4. \ub108\uc640 \ub098\ub294 \uac19\uc774 \uc77c\ud558\ub2e4\uac00 \uac19\uc774 \uc8fd\uc5b4\uc57c \ud574. \uc8fc\ub2d8\uc740 \uc6b0\ub9ac\uac00 \ubb34\uc5c7\uc744 \ud574\uc57c \ud558\ub294\uc9c0 \uc54c\uace0 \uacc4\uc2dc\uc7ce\uc544. \uc77c\ud658\uc544, \ub204\uc6cc\uc11c \uc880 \uc26c\ub3c4\ub85d \ud574. \ub10c \ub108\ubb34 \uc9c0\uccd0 \uc788\uc5b4. \u201d", "paragraph_answer": "1\uc8fc\uc77c \uc774\uc0c1\uc774\ub098 \uc7a5\uc77c\ud658 \ub4f1 \ud68c\uc6d0\ub4e4\uc744 \uace0\ubb38\ud558\uc600\ub294\ub370, \ub9cc\uc8fc\uc5d0\uc11c \uc628 \ubc31\uc138\ube48\uc774 \uace0\ubb38\uc5d0 \uad74\ubcf5\ud558\uc5ec \uc870\uc9c1\uc5d0 \ub300\ud574 \uc2e4\ud1a0\ud558\uac8c \ub418\uc5c8\ub2e4. \uc774\ub9ac\ud558\uc5ec \ud68c\uc758\uc5d0 \ucc38\uc11d\ud558\uc9c0 \uc54a\uc558\ub358 \ud68c\uc6d0\uc744 \ube44\ub86f\ud574\uc11c \ud68c\uc6d0\ub4e4\uc758 \uc9c0\uc778\ub4e4\uae4c\uc9c0 \ubaa8\ub450 \uccb4\ud3ec\ub418\uc5c8\uc73c\ub098 \uc785\uac74\uc740 30\uba85 \uc815\ub3c4\uac00 \ub418\uc5c8\ub2e4. \ud68c\uc7a5 \uc7a5\uc77c\ud658\uc744 \ube44\ub86f\ud574\uc154 \ubc30\ubbfc\uc218 , \uae40\ud615\uc9c1, \uac15\uc11d\ubd09 \ub4f1\uc740 \ud3c9\uc591\uac10\uc625\uc5d0\uc11c \uc625\uace0\ub97c \uce58\ub800\ub2e4. \ub3c5\uc2e4\ud55c \uae30\ub3c5\uad50 \uc2e0\uc790\ub85c\uc11c \ub3c5\ub9bd\uc744 \uc5fc\uc6d0\ud558\uc600\ub358 \uc7a5\uc77c\ud658\uc740 \ubc30\ubbfc\uc218\uc5d0\uac8c \uc720\uc5b8\uc744 \ub0a8\uae30\uace0 \uace0\ubb38\uc73c\ub85c \uc0ac\ub9dd\ud558\uc600\ub2e4. \ubc30\ubbfc\uc218\ub294 \ud68c\uc7a5 \uc7a5\uc77c\ud658\uc758 \uc21c\uad6d\uacfc\uc815\uc5d0 \ub300\ud574 \uae30\uc220\ud558\uc600\ub2e4. \"\uc77c\uc8fc\uc77c \ud6c4\uc5d0, \uadf8\ub4e4\uc740 \uc6b0\ub9ac\uc758 \ubc29\uc744 \ubc14\uafb8\uc5b4 \ub098\ub97c \uc7a5\uc77c\ud658\uc774 \uc788\ub294 \ubc29\uc5d0. \ub123\uc5c8\ub2e4. \uc870\uc9c1\uc758 \ud68c\uc7a5\uc774\uc5c8\ub358 \uc77c\ud658\uc740 \ub204\uad6c\ubcf4\ub2e4\ub3c4 \uace0\ubb38\uc744 \uc2ec\ud558\uac8c \ubc1b\uc558\ub2e4. \ud558\ub8e8\ub294 \ud615\uc0ac\uc2e4\uc5d0 \ub04c\ub824 \uac00 \ub3cc\uc544\uc62c \uae30\ubbf8\uac00 \uc548 \ubcf4\uc774\ub354\ub2c8 \ubc18\uc8fd\uc74c\uc774 \ub418\uc5b4\uc11c \ub3cc\uc544\uc654\ub2e4. \ub098\ub294 \uadf8\ub97c \uc548\uace0 \ubcf4\uc0b4\ud3b4\uc8fc\uc5c8\ub2e4. \uadf8\ub294 \uc6b8\uba70 \ub098\uc5d0\uac8c \ub9d0\ud588\ub2e4.\u201c\ubbfc\uc218\uc57c, \uc774\ub7f0 \ub9d0\ud574\uc11c \ubbf8\uc548\ud558\uc9c0\ub9cc, \ub098\ub294 \ub354 \uc774\uc0c1 \uc0b4 \uc218 \uc788\uc744 \uac83 \uac19\uc9c0 \uc54a\uc544. \ub098\ub294 \uace7 \uc8fd\uac8c \ub420 \uac70\uc57c. \uc8fc\ub2d8\uacfc \ud3c9\ud654\ub85c\uc6b4 \ucc9c\uad6d\uc5d0 \uac19\uc774 \uc788\uc744 \uc218 \uc788\ub2e4\uba74 \uadf8\uac8c \ub354 \ub098\uc744 \uac70\uc57c, \ud558\uc9c0\ub9cc \uc5b4\ub5bb\uac8c \ub108\ub97c \ud63c\uc790 \uace0\uc0dd\ud558\ub3c4\ub85d \ub450\uace0 \uac00\uc9c0? \ub108\ubb34\ub098\ub3c4 \uc548\ud0c0\uae5d\uad6c\ub098. \ub0b4\uac00 \uc5c6\uc5b4\ub3c4 \uc798 \ud560 \uc904 \ubbff\uc5b4. \uc8fc\ub2d8\uc758 \uacc1\uc5d0\uc11c \ud56d\uc0c1 \ub110 \uc704\ud574 \uae30\ub3c4\ud574 \uc904\uac8c. \uc6b0\ub9ac\uc758 \ub3c5\ub9bd\uc744 \uc7c1\ucde8\ud558\ub294 \uadf8\ub0a0\uae4c\uc9c0 \uc6a9\uae30\ub97c \uc783\uc9c0 \ub9c8, \ubbfc\uc218\uc57c. \uc5b8\uc81c \uc8fd\ub290\ub0d0\uac00 \ubb38\uc81c\uc9c0 \uc5b8\uc820\uac00 \uc6b0\ub9ac\ub294 \ub2e4 \uac19\uc740 \uacf3\uc5d0\uc11c \ub9cc\ub0a0 \uac70\uc57c. \uac12\uc9c0\uac8c \uc8fd\ub294 \uac83\uc774 \ub73b \uc5c6\uc774 \uc0ac\ub294 \uac83\ubcf4\ub2e4 \uac00\uce58\uac00 \uc788\uc744 \uac70\uc57c. \uadf8\ub7ec\ub2c8\uae4c, \uac12\uc9c4 \ub124 \uc778\uc0dd\uc744 \uc18c\uc911\ud788 \ud574\ub77c. \uac00\uce58 \uc5c6\ub294 \uac83\uc5d0 \ubaa9\uc228\uc744 \uac78\uc9c0 \ub9d0\uc544\ub77c. \ub124\uac00 \ubaa9\uc228\uc744 \uc783\uc9c0 \uc54a\uace0\ub3c4 \uc790\uc720\ub97c \ucc3e\uc744 \uc218 \uc788\ub3c4\ub85d \uc8fc\ub2d8\uaed8 \uae30\ub3c4\ud558\ub9c8. \uc5b4\uba38\ub2c8\uaed8 \ub124 \uc544\ubc84\uc9c0\uc758 \ub73b\uc744 \uc774\ub8e8\ub294 \ubaa8\uc2b5\uc744 \ubcf4\uc5ec\ub4dc\ub824\ub77c. \ubbfc\uc218\uc57c \uc5b4\ub5bb\uac8c \ub110 \ub5a0\ub098\uc9c0, \uc5b4\ub5bb\uac8c \ub110 \ub5a0\ub098\uc9c0?\u201d\uadf8\ub294 \uc5b4\ub9b0\uc544\uc774\ucc98\ub7fc \uc6b8\uace0 \uc788\uc5c8\uace0 \ub098\ub294 \uac10\uc815\uc5d0 \uce58\uc6b0\uce58\uc9c0 \ub9d0\ub77c\uace0 \ud588\ub2e4. \u201c\uc9c0\uae08\uc740 \uc6b8 \ub54c\uac00 \uc544\ub2c8\ub77c \uc2f8\uc6b8 \ub54c\uc774\ub2e4. \ub124\uac00 \uc8fd\ub354\ub77c\ub3c4 \ub0b4\uac00 \ubaa8\ub4e0 \uac78 \ucc45\uc784\uc9c0\ub9c8. \ud558\uc9c0\ub9cc \ub098\ub294 \ub124\uac00 \uc8fd\uc9c0 \uc54a\uc744 \uac70\ub77c\uace0 \ubbff\ub294\ub2e4. \ub108\uc640 \ub098\ub294 \uac19\uc774 \uc77c\ud558\ub2e4\uac00 \uac19\uc774 \uc8fd\uc5b4\uc57c \ud574. \uc8fc\ub2d8\uc740 \uc6b0\ub9ac\uac00 \ubb34\uc5c7\uc744 \ud574\uc57c \ud558\ub294\uc9c0 \uc54c\uace0 \uacc4\uc2dc\uc7ce\uc544. \uc77c\ud658\uc544, \ub204\uc6cc\uc11c \uc880 \uc26c\ub3c4\ub85d \ud574. \ub10c \ub108\ubb34 \uc9c0\uccd0 \uc788\uc5b4.\u201d", "sentence_answer": "\ud68c\uc7a5 \uc7a5\uc77c\ud658\uc744 \ube44\ub86f\ud574\uc154 \ubc30\ubbfc\uc218 ,", "paragraph_id": "6592603-1-1", "paragraph_question": "question: \ud3c9\uc591\uac10\uc625 \ud22c\uc625 \ub2f9\uc2dc \uc7a5\uc77c\ud658\uc758 \uc720\uc5b8\uc744 \ub4e3\uace0 \uadf8\uc758 \uc21c\uad6d\uacfc\uc815\uc744 \uae30\uc220\ud55c \uc778\ubb3c\uc740?, context: 1\uc8fc\uc77c \uc774\uc0c1\uc774\ub098 \uc7a5\uc77c\ud658 \ub4f1 \ud68c\uc6d0\ub4e4\uc744 \uace0\ubb38\ud558\uc600\ub294\ub370, \ub9cc\uc8fc\uc5d0\uc11c \uc628 \ubc31\uc138\ube48\uc774 \uace0\ubb38\uc5d0 \uad74\ubcf5\ud558\uc5ec \uc870\uc9c1\uc5d0 \ub300\ud574 \uc2e4\ud1a0\ud558\uac8c \ub418\uc5c8\ub2e4. \uc774\ub9ac\ud558\uc5ec \ud68c\uc758\uc5d0 \ucc38\uc11d\ud558\uc9c0 \uc54a\uc558\ub358 \ud68c\uc6d0\uc744 \ube44\ub86f\ud574\uc11c \ud68c\uc6d0\ub4e4\uc758 \uc9c0\uc778\ub4e4\uae4c\uc9c0 \ubaa8\ub450 \uccb4\ud3ec\ub418\uc5c8\uc73c\ub098 \uc785\uac74\uc740 30\uba85 \uc815\ub3c4\uac00 \ub418\uc5c8\ub2e4. \ud68c\uc7a5 \uc7a5\uc77c\ud658\uc744 \ube44\ub86f\ud574\uc154 \ubc30\ubbfc\uc218, \uae40\ud615\uc9c1, \uac15\uc11d\ubd09 \ub4f1\uc740 \ud3c9\uc591\uac10\uc625\uc5d0\uc11c \uc625\uace0\ub97c \uce58\ub800\ub2e4. \ub3c5\uc2e4\ud55c \uae30\ub3c5\uad50 \uc2e0\uc790\ub85c\uc11c \ub3c5\ub9bd\uc744 \uc5fc\uc6d0\ud558\uc600\ub358 \uc7a5\uc77c\ud658\uc740 \ubc30\ubbfc\uc218\uc5d0\uac8c \uc720\uc5b8\uc744 \ub0a8\uae30\uace0 \uace0\ubb38\uc73c\ub85c \uc0ac\ub9dd\ud558\uc600\ub2e4. \ubc30\ubbfc\uc218\ub294 \ud68c\uc7a5 \uc7a5\uc77c\ud658\uc758 \uc21c\uad6d\uacfc\uc815\uc5d0 \ub300\ud574 \uae30\uc220\ud558\uc600\ub2e4. \"\uc77c\uc8fc\uc77c \ud6c4\uc5d0, \uadf8\ub4e4\uc740 \uc6b0\ub9ac\uc758 \ubc29\uc744 \ubc14\uafb8\uc5b4 \ub098\ub97c \uc7a5\uc77c\ud658\uc774 \uc788\ub294 \ubc29\uc5d0. \ub123\uc5c8\ub2e4. \uc870\uc9c1\uc758 \ud68c\uc7a5\uc774\uc5c8\ub358 \uc77c\ud658\uc740 \ub204\uad6c\ubcf4\ub2e4\ub3c4 \uace0\ubb38\uc744 \uc2ec\ud558\uac8c \ubc1b\uc558\ub2e4. \ud558\ub8e8\ub294 \ud615\uc0ac\uc2e4\uc5d0 \ub04c\ub824 \uac00 \ub3cc\uc544\uc62c \uae30\ubbf8\uac00 \uc548 \ubcf4\uc774\ub354\ub2c8 \ubc18\uc8fd\uc74c\uc774 \ub418\uc5b4\uc11c \ub3cc\uc544\uc654\ub2e4. \ub098\ub294 \uadf8\ub97c \uc548\uace0 \ubcf4\uc0b4\ud3b4\uc8fc\uc5c8\ub2e4. \uadf8\ub294 \uc6b8\uba70 \ub098\uc5d0\uac8c \ub9d0\ud588\ub2e4.\u201c\ubbfc\uc218\uc57c, \uc774\ub7f0 \ub9d0\ud574\uc11c \ubbf8\uc548\ud558\uc9c0\ub9cc, \ub098\ub294 \ub354 \uc774\uc0c1 \uc0b4 \uc218 \uc788\uc744 \uac83 \uac19\uc9c0 \uc54a\uc544. \ub098\ub294 \uace7 \uc8fd\uac8c \ub420 \uac70\uc57c. \uc8fc\ub2d8\uacfc \ud3c9\ud654\ub85c\uc6b4 \ucc9c\uad6d\uc5d0 \uac19\uc774 \uc788\uc744 \uc218 \uc788\ub2e4\uba74 \uadf8\uac8c \ub354 \ub098\uc744 \uac70\uc57c, \ud558\uc9c0\ub9cc \uc5b4\ub5bb\uac8c \ub108\ub97c \ud63c\uc790 \uace0\uc0dd\ud558\ub3c4\ub85d \ub450\uace0 \uac00\uc9c0? \ub108\ubb34\ub098\ub3c4 \uc548\ud0c0\uae5d\uad6c\ub098. \ub0b4\uac00 \uc5c6\uc5b4\ub3c4 \uc798 \ud560 \uc904 \ubbff\uc5b4. \uc8fc\ub2d8\uc758 \uacc1\uc5d0\uc11c \ud56d\uc0c1 \ub110 \uc704\ud574 \uae30\ub3c4\ud574 \uc904\uac8c. \uc6b0\ub9ac\uc758 \ub3c5\ub9bd\uc744 \uc7c1\ucde8\ud558\ub294 \uadf8\ub0a0\uae4c\uc9c0 \uc6a9\uae30\ub97c \uc783\uc9c0 \ub9c8, \ubbfc\uc218\uc57c. \uc5b8\uc81c \uc8fd\ub290\ub0d0\uac00 \ubb38\uc81c\uc9c0 \uc5b8\uc820\uac00 \uc6b0\ub9ac\ub294 \ub2e4 \uac19\uc740 \uacf3\uc5d0\uc11c \ub9cc\ub0a0 \uac70\uc57c. \uac12\uc9c0\uac8c \uc8fd\ub294 \uac83\uc774 \ub73b \uc5c6\uc774 \uc0ac\ub294 \uac83\ubcf4\ub2e4 \uac00\uce58\uac00 \uc788\uc744 \uac70\uc57c. \uadf8\ub7ec\ub2c8\uae4c, \uac12\uc9c4 \ub124 \uc778\uc0dd\uc744 \uc18c\uc911\ud788 \ud574\ub77c. \uac00\uce58 \uc5c6\ub294 \uac83\uc5d0 \ubaa9\uc228\uc744 \uac78\uc9c0 \ub9d0\uc544\ub77c. \ub124\uac00 \ubaa9\uc228\uc744 \uc783\uc9c0 \uc54a\uace0\ub3c4 \uc790\uc720\ub97c \ucc3e\uc744 \uc218 \uc788\ub3c4\ub85d \uc8fc\ub2d8\uaed8 \uae30\ub3c4\ud558\ub9c8. \uc5b4\uba38\ub2c8\uaed8 \ub124 \uc544\ubc84\uc9c0\uc758 \ub73b\uc744 \uc774\ub8e8\ub294 \ubaa8\uc2b5\uc744 \ubcf4\uc5ec\ub4dc\ub824\ub77c. \ubbfc\uc218\uc57c \uc5b4\ub5bb\uac8c \ub110 \ub5a0\ub098\uc9c0, \uc5b4\ub5bb\uac8c \ub110 \ub5a0\ub098\uc9c0?\u201d\uadf8\ub294 \uc5b4\ub9b0\uc544\uc774\ucc98\ub7fc \uc6b8\uace0 \uc788\uc5c8\uace0 \ub098\ub294 \uac10\uc815\uc5d0 \uce58\uc6b0\uce58\uc9c0 \ub9d0\ub77c\uace0 \ud588\ub2e4. \u201c\uc9c0\uae08\uc740 \uc6b8 \ub54c\uac00 \uc544\ub2c8\ub77c \uc2f8\uc6b8 \ub54c\uc774\ub2e4. \ub124\uac00 \uc8fd\ub354\ub77c\ub3c4 \ub0b4\uac00 \ubaa8\ub4e0 \uac78 \ucc45\uc784\uc9c0\ub9c8. \ud558\uc9c0\ub9cc \ub098\ub294 \ub124\uac00 \uc8fd\uc9c0 \uc54a\uc744 \uac70\ub77c\uace0 \ubbff\ub294\ub2e4. \ub108\uc640 \ub098\ub294 \uac19\uc774 \uc77c\ud558\ub2e4\uac00 \uac19\uc774 \uc8fd\uc5b4\uc57c \ud574. \uc8fc\ub2d8\uc740 \uc6b0\ub9ac\uac00 \ubb34\uc5c7\uc744 \ud574\uc57c \ud558\ub294\uc9c0 \uc54c\uace0 \uacc4\uc2dc\uc7ce\uc544. \uc77c\ud658\uc544, \ub204\uc6cc\uc11c \uc880 \uc26c\ub3c4\ub85d \ud574. \ub10c \ub108\ubb34 \uc9c0\uccd0 \uc788\uc5b4.\u201d"} {"question": "\ud0dc\uc591\uc758 \ud6c4\uc608\ub294 KBS \uacf5\uc0ac\ucc3d\ub9bd \uba87 \uc8fc\ub144 \ud2b9\ubcc4\uae30\ud68d \ub4dc\ub77c\ub9c8\uc778\uac00?", "paragraph": "\u300a\ud0dc\uc591\uc758 \ud6c4\uc608\u300b\ub294 2016\ub144 2\uc6d4 24\uc77c\ubd80\ud130 2016\ub144 4\uc6d4 14\uc77c\uae4c\uc9c0 \ubc29\uc601\ub41c KBS \uacf5\uc0ac\ucc3d\ub9bd 43\uc8fc\ub144 \ud2b9\ubcc4\uae30\ud68d\uc774\uc790 \ud55c\uad6d\ubc29\uc1a1 89\uc8fc\ub144 \ub4dc\ub77c\ub9c8\ub85c, \uae40\uc740\uc219\uacfc \uae40\uc6d0\uc11d\uc774 \uacf5\ub3d9 \uc9d1\ud544\ud558\uace0 KBS \ub4dc\ub77c\ub9c8 \uc81c\uc791\uad6d\uc758 \uc774\uc751\ubcf5\uacfc \ubc31\uc0c1\ud6c8\uc774 \uacf5\ub3d9 \uc5f0\ucd9c\uc744 \ub9e1\uc558\ub2e4. \uc601\ud654 \ud22c\uc790\ubc30\uae09\uc0ac \u321c\ub125\uc2a4\ud2b8\uc5d4\ud130\ud14c\uc778\uba3c\ud2b8\uc6d4\ub4dc\uc758 \uccab \ubc88\uc9f8 \ub4dc\ub77c\ub9c8 \ud22c\uc790\uc791\uc774\uba70, 2016\ub144 2\uc6d4 24\uc77c\ubd80\ud130 \ud55c\uc911 \ub3d9\uc2dc \ubc29\uc601\uc744 \uac1c\uc2dc\ud558\uc600\ub2e4. 30%\ub97c \uc6c3\ub3c4\ub294 \uc555\ub3c4\uc801\uc778 \uc2dc\uccad\ub960\uc744 \uae30\ub85d\ud558\uc600\uc73c\uba70, \ub9c8\uc9c0\ub9c9\ud68c\uc5d0\ub294, 38.8%\ub77c\ub294 \ucd5c\uace0\uc758 \uc2dc\uccad\ub960\uc744 \uae30\ub85d\ud558\uc600\ub2e4. \uc1a1\uc911\uae30-\uc1a1\ud61c\uad50 \ucee4\ud50c\uc744 '\uc1a1\uc1a1 \ucee4\ud50c' \ud639\uc740 '\uc2dc\uac15 \ucee4\ud50c'\uc774\ub77c \uc904\uc5ec \ubd80\ub974\uace0 \uc9c4\uad6c-\uae40\uc9c0\uc6d0 \ucee4\ud50c\uc744 \uc904\uc5ec '\uad6c\uc6d0 \ucee4\ud50c'\uc774\ub77c \ubd80\ub974\ub294 \uc2e0\uc870\uc5b4\uac00 \uc0dd\uae30\ub294 \ub4f1 \uc7a5\uc548\uc5d0 \ud654\uc81c\uac00 \ub420 \uc815\ub3c4\ub85c \ub9ce\uc740 \uad00\uc2ec\uacfc \ud638\ud3c9\uc744 \uc5bb\uc740 \uad50\ud6c8\uc801\uc778 \uc791\ud488\uc774\uc5c8\ub2e4.", "answer": "43\uc8fc\ub144", "sentence": "\u300a\ud0dc\uc591\uc758 \ud6c4\uc608\u300b\ub294 2016\ub144 2\uc6d4 24\uc77c\ubd80\ud130 2016\ub144 4\uc6d4 14\uc77c\uae4c\uc9c0 \ubc29\uc601\ub41c KBS \uacf5\uc0ac\ucc3d\ub9bd 43\uc8fc\ub144 \ud2b9\ubcc4\uae30\ud68d\uc774\uc790 \ud55c\uad6d\ubc29\uc1a1 89\uc8fc\ub144 \ub4dc\ub77c\ub9c8\ub85c, \uae40\uc740\uc219\uacfc \uae40\uc6d0\uc11d\uc774 \uacf5\ub3d9 \uc9d1\ud544\ud558\uace0 KBS \ub4dc\ub77c\ub9c8 \uc81c\uc791\uad6d\uc758 \uc774\uc751\ubcf5\uacfc \ubc31\uc0c1\ud6c8\uc774 \uacf5\ub3d9 \uc5f0\ucd9c\uc744 \ub9e1\uc558\ub2e4.", "paragraph_sentence": " \u300a\ud0dc\uc591\uc758 \ud6c4\uc608\u300b\ub294 2016\ub144 2\uc6d4 24\uc77c\ubd80\ud130 2016\ub144 4\uc6d4 14\uc77c\uae4c\uc9c0 \ubc29\uc601\ub41c KBS \uacf5\uc0ac\ucc3d\ub9bd 43\uc8fc\ub144 \ud2b9\ubcc4\uae30\ud68d\uc774\uc790 \ud55c\uad6d\ubc29\uc1a1 89\uc8fc\ub144 \ub4dc\ub77c\ub9c8\ub85c, \uae40\uc740\uc219\uacfc \uae40\uc6d0\uc11d\uc774 \uacf5\ub3d9 \uc9d1\ud544\ud558\uace0 KBS \ub4dc\ub77c\ub9c8 \uc81c\uc791\uad6d\uc758 \uc774\uc751\ubcf5\uacfc \ubc31\uc0c1\ud6c8\uc774 \uacf5\ub3d9 \uc5f0\ucd9c\uc744 \ub9e1\uc558\ub2e4. \uc601\ud654 \ud22c\uc790\ubc30\uae09\uc0ac \u321c\ub125\uc2a4\ud2b8\uc5d4\ud130\ud14c\uc778\uba3c\ud2b8\uc6d4\ub4dc\uc758 \uccab \ubc88\uc9f8 \ub4dc\ub77c\ub9c8 \ud22c\uc790\uc791\uc774\uba70, 2016\ub144 2\uc6d4 24\uc77c\ubd80\ud130 \ud55c\uc911 \ub3d9\uc2dc \ubc29\uc601\uc744 \uac1c\uc2dc\ud558\uc600\ub2e4. 30%\ub97c \uc6c3\ub3c4\ub294 \uc555\ub3c4\uc801\uc778 \uc2dc\uccad\ub960\uc744 \uae30\ub85d\ud558\uc600\uc73c\uba70, \ub9c8\uc9c0\ub9c9\ud68c\uc5d0\ub294, 38.8%\ub77c\ub294 \ucd5c\uace0\uc758 \uc2dc\uccad\ub960\uc744 \uae30\ub85d\ud558\uc600\ub2e4. \uc1a1\uc911\uae30-\uc1a1\ud61c\uad50 \ucee4\ud50c\uc744 '\uc1a1\uc1a1 \ucee4\ud50c' \ud639\uc740 '\uc2dc\uac15 \ucee4\ud50c'\uc774\ub77c \uc904\uc5ec \ubd80\ub974\uace0 \uc9c4\uad6c-\uae40\uc9c0\uc6d0 \ucee4\ud50c\uc744 \uc904\uc5ec '\uad6c\uc6d0 \ucee4\ud50c'\uc774\ub77c \ubd80\ub974\ub294 \uc2e0\uc870\uc5b4\uac00 \uc0dd\uae30\ub294 \ub4f1 \uc7a5\uc548\uc5d0 \ud654\uc81c\uac00 \ub420 \uc815\ub3c4\ub85c \ub9ce\uc740 \uad00\uc2ec\uacfc \ud638\ud3c9\uc744 \uc5bb\uc740 \uad50\ud6c8\uc801\uc778 \uc791\ud488\uc774\uc5c8\ub2e4.", "paragraph_answer": "\u300a\ud0dc\uc591\uc758 \ud6c4\uc608\u300b\ub294 2016\ub144 2\uc6d4 24\uc77c\ubd80\ud130 2016\ub144 4\uc6d4 14\uc77c\uae4c\uc9c0 \ubc29\uc601\ub41c KBS \uacf5\uc0ac\ucc3d\ub9bd 43\uc8fc\ub144 \ud2b9\ubcc4\uae30\ud68d\uc774\uc790 \ud55c\uad6d\ubc29\uc1a1 89\uc8fc\ub144 \ub4dc\ub77c\ub9c8\ub85c, \uae40\uc740\uc219\uacfc \uae40\uc6d0\uc11d\uc774 \uacf5\ub3d9 \uc9d1\ud544\ud558\uace0 KBS \ub4dc\ub77c\ub9c8 \uc81c\uc791\uad6d\uc758 \uc774\uc751\ubcf5\uacfc \ubc31\uc0c1\ud6c8\uc774 \uacf5\ub3d9 \uc5f0\ucd9c\uc744 \ub9e1\uc558\ub2e4. \uc601\ud654 \ud22c\uc790\ubc30\uae09\uc0ac \u321c\ub125\uc2a4\ud2b8\uc5d4\ud130\ud14c\uc778\uba3c\ud2b8\uc6d4\ub4dc\uc758 \uccab \ubc88\uc9f8 \ub4dc\ub77c\ub9c8 \ud22c\uc790\uc791\uc774\uba70, 2016\ub144 2\uc6d4 24\uc77c\ubd80\ud130 \ud55c\uc911 \ub3d9\uc2dc \ubc29\uc601\uc744 \uac1c\uc2dc\ud558\uc600\ub2e4. 30%\ub97c \uc6c3\ub3c4\ub294 \uc555\ub3c4\uc801\uc778 \uc2dc\uccad\ub960\uc744 \uae30\ub85d\ud558\uc600\uc73c\uba70, \ub9c8\uc9c0\ub9c9\ud68c\uc5d0\ub294, 38.8%\ub77c\ub294 \ucd5c\uace0\uc758 \uc2dc\uccad\ub960\uc744 \uae30\ub85d\ud558\uc600\ub2e4. \uc1a1\uc911\uae30-\uc1a1\ud61c\uad50 \ucee4\ud50c\uc744 '\uc1a1\uc1a1 \ucee4\ud50c' \ud639\uc740 '\uc2dc\uac15 \ucee4\ud50c'\uc774\ub77c \uc904\uc5ec \ubd80\ub974\uace0 \uc9c4\uad6c-\uae40\uc9c0\uc6d0 \ucee4\ud50c\uc744 \uc904\uc5ec '\uad6c\uc6d0 \ucee4\ud50c'\uc774\ub77c \ubd80\ub974\ub294 \uc2e0\uc870\uc5b4\uac00 \uc0dd\uae30\ub294 \ub4f1 \uc7a5\uc548\uc5d0 \ud654\uc81c\uac00 \ub420 \uc815\ub3c4\ub85c \ub9ce\uc740 \uad00\uc2ec\uacfc \ud638\ud3c9\uc744 \uc5bb\uc740 \uad50\ud6c8\uc801\uc778 \uc791\ud488\uc774\uc5c8\ub2e4.", "sentence_answer": "\u300a\ud0dc\uc591\uc758 \ud6c4\uc608\u300b\ub294 2016\ub144 2\uc6d4 24\uc77c\ubd80\ud130 2016\ub144 4\uc6d4 14\uc77c\uae4c\uc9c0 \ubc29\uc601\ub41c KBS \uacf5\uc0ac\ucc3d\ub9bd 43\uc8fc\ub144 \ud2b9\ubcc4\uae30\ud68d\uc774\uc790 \ud55c\uad6d\ubc29\uc1a1 89\uc8fc\ub144 \ub4dc\ub77c\ub9c8\ub85c, \uae40\uc740\uc219\uacfc \uae40\uc6d0\uc11d\uc774 \uacf5\ub3d9 \uc9d1\ud544\ud558\uace0 KBS \ub4dc\ub77c\ub9c8 \uc81c\uc791\uad6d\uc758 \uc774\uc751\ubcf5\uacfc \ubc31\uc0c1\ud6c8\uc774 \uacf5\ub3d9 \uc5f0\ucd9c\uc744 \ub9e1\uc558\ub2e4.", "paragraph_id": "6565257-0-0", "paragraph_question": "question: \ud0dc\uc591\uc758 \ud6c4\uc608\ub294 KBS \uacf5\uc0ac\ucc3d\ub9bd \uba87 \uc8fc\ub144 \ud2b9\ubcc4\uae30\ud68d \ub4dc\ub77c\ub9c8\uc778\uac00?, context: \u300a\ud0dc\uc591\uc758 \ud6c4\uc608\u300b\ub294 2016\ub144 2\uc6d4 24\uc77c\ubd80\ud130 2016\ub144 4\uc6d4 14\uc77c\uae4c\uc9c0 \ubc29\uc601\ub41c KBS \uacf5\uc0ac\ucc3d\ub9bd 43\uc8fc\ub144 \ud2b9\ubcc4\uae30\ud68d\uc774\uc790 \ud55c\uad6d\ubc29\uc1a1 89\uc8fc\ub144 \ub4dc\ub77c\ub9c8\ub85c, \uae40\uc740\uc219\uacfc \uae40\uc6d0\uc11d\uc774 \uacf5\ub3d9 \uc9d1\ud544\ud558\uace0 KBS \ub4dc\ub77c\ub9c8 \uc81c\uc791\uad6d\uc758 \uc774\uc751\ubcf5\uacfc \ubc31\uc0c1\ud6c8\uc774 \uacf5\ub3d9 \uc5f0\ucd9c\uc744 \ub9e1\uc558\ub2e4. \uc601\ud654 \ud22c\uc790\ubc30\uae09\uc0ac \u321c\ub125\uc2a4\ud2b8\uc5d4\ud130\ud14c\uc778\uba3c\ud2b8\uc6d4\ub4dc\uc758 \uccab \ubc88\uc9f8 \ub4dc\ub77c\ub9c8 \ud22c\uc790\uc791\uc774\uba70, 2016\ub144 2\uc6d4 24\uc77c\ubd80\ud130 \ud55c\uc911 \ub3d9\uc2dc \ubc29\uc601\uc744 \uac1c\uc2dc\ud558\uc600\ub2e4. 30%\ub97c \uc6c3\ub3c4\ub294 \uc555\ub3c4\uc801\uc778 \uc2dc\uccad\ub960\uc744 \uae30\ub85d\ud558\uc600\uc73c\uba70, \ub9c8\uc9c0\ub9c9\ud68c\uc5d0\ub294, 38.8%\ub77c\ub294 \ucd5c\uace0\uc758 \uc2dc\uccad\ub960\uc744 \uae30\ub85d\ud558\uc600\ub2e4. \uc1a1\uc911\uae30-\uc1a1\ud61c\uad50 \ucee4\ud50c\uc744 '\uc1a1\uc1a1 \ucee4\ud50c' \ud639\uc740 '\uc2dc\uac15 \ucee4\ud50c'\uc774\ub77c \uc904\uc5ec \ubd80\ub974\uace0 \uc9c4\uad6c-\uae40\uc9c0\uc6d0 \ucee4\ud50c\uc744 \uc904\uc5ec '\uad6c\uc6d0 \ucee4\ud50c'\uc774\ub77c \ubd80\ub974\ub294 \uc2e0\uc870\uc5b4\uac00 \uc0dd\uae30\ub294 \ub4f1 \uc7a5\uc548\uc5d0 \ud654\uc81c\uac00 \ub420 \uc815\ub3c4\ub85c \ub9ce\uc740 \uad00\uc2ec\uacfc \ud638\ud3c9\uc744 \uc5bb\uc740 \uad50\ud6c8\uc801\uc778 \uc791\ud488\uc774\uc5c8\ub2e4."} {"question": "\ud55c\uad6d\ud504\ub85c\uc57c\uad6c OB\ub2e8\uccb4\uc758 \uc774\ub984\uc740 \ubb34\uc5c7\uc778\uac00?", "paragraph": "\uadf8\ub9ac\uace0 \uc9c1\uc811\uc801\uc778 \uc9c0\uc6d0 \ud65c\ub3d9\uc73c\ub85c\uc11c 2012\ub144 5\uc6d4\ubd80\ud130 \ub3d9\uc77c\ubcf8 \ub300\uc9c0\uc9c4\uc758 \uc774\uc7ac\ubbfc\uc774 \uac70\uc8fc\ud558\uace0 \uc788\ub294 \uc784\uc2dc\uc8fc\ud0dd\uc744 \ud68c\uc6d0\ub4e4\uc774 \ubc29\ubb38\ud574 \uad50\ub958\ub97c \uc704\ud55c \u2018\ub9c8\uc74c\uc758 \uce58\uc720\u2019 \ud65c\ub3d9\uc744 \uad6d\uc81c \uc5f0\ud569\uc758 \ubc97 Asia-Pacific\uacfc \uacf5\ub3d9\uc73c\ub85c \uc9c4\ud589\ud558\uace0 \uc788\ub2e4. 2012\ub144 7\uc6d4\uc5d0\ub294 \uba85\uad6c\ud68c\uac00 \ub300\ud55c\ubbfc\uad6d\uacfc\uc758 \uad6d\uc81c \uce5c\uc120 \ucc28\uc6d0\uc5d0\uc11c \ud55c\uad6d\ud504\ub85c\uc57c\uad6c OB\ub2e8\uccb4\uc778 \uc77c\uad6c\ud68c\uc640 \uacf5\ub3d9\uc73c\ub85c \uccad\uac01\uc7a5\uc560\ub97c \uc553\ub294 \uc18c\ub144\ub4e4\uc5d0\uac8c \uc57c\uad6c \ud074\ub9ac\ub2c9\uacfc 7\uc6d4 20\uc77c \uc7a0\uc2e4\uc57c\uad6c\uc7a5\uc5d0\uc11c \ud55c\uc77c \ud504\ub85c \uc57c\uad6c \ub808\uc804\ub4dc\ub9e4\uce58\ub97c \uac1c\ucd5c\ud574 \ub300\ud55c\ubbfc\uad6d\uc758 \uc57c\uad6c \ud32c\ub4e4\uc744 \uc990\uac81\uac8c \ud588\ub2e4. \ub610\ud55c \uae30\ub150 \uc0ac\uc5c5\uc774\ub098 \uc5f0\uc218\ud68c\uc758 \ubc14\ub78c\ub300\ub85c \uc9c0\uc5ed\uc8fc\ubbfc, \uc790\uc704\ub300\uc6d0, \uae30\uc5c5\uc778\ub4e4, \uc120\uc218, \uac10\ub3c5\uc73c\ub85c\uc11c \uacbd\ud5d8\ud55c \uae30\ub85d\uc774\ub098 \uc2b9\ubd80, \uc9c0\ud718\ubd09\uc5d0 \uc5bd\ud600\uc788\ub294 \uc774\uc57c\uae30 \ub4f1 \ub9ce\uc740 \uac15\uc5f0\uc744 \uc5f4\uc5b4 \uad50\ud6c8\uc744 \uc804\ud558\uace0 \uc788\ub2e4.", "answer": "\uc77c\uad6c\ud68c", "sentence": "2012\ub144 7\uc6d4\uc5d0\ub294 \uba85\uad6c\ud68c\uac00 \ub300\ud55c\ubbfc\uad6d\uacfc\uc758 \uad6d\uc81c \uce5c\uc120 \ucc28\uc6d0\uc5d0\uc11c \ud55c\uad6d\ud504\ub85c\uc57c\uad6c OB\ub2e8\uccb4\uc778 \uc77c\uad6c\ud68c \uc640 \uacf5\ub3d9\uc73c\ub85c \uccad\uac01\uc7a5\uc560\ub97c \uc553\ub294 \uc18c\ub144\ub4e4\uc5d0\uac8c \uc57c\uad6c \ud074\ub9ac\ub2c9\uacfc 7\uc6d4 20\uc77c \uc7a0\uc2e4\uc57c\uad6c\uc7a5\uc5d0\uc11c \ud55c\uc77c \ud504\ub85c \uc57c\uad6c \ub808\uc804\ub4dc\ub9e4\uce58\ub97c \uac1c\ucd5c\ud574 \ub300\ud55c\ubbfc\uad6d\uc758 \uc57c\uad6c \ud32c\ub4e4\uc744 \uc990\uac81\uac8c \ud588\ub2e4.", "paragraph_sentence": "\uadf8\ub9ac\uace0 \uc9c1\uc811\uc801\uc778 \uc9c0\uc6d0 \ud65c\ub3d9\uc73c\ub85c\uc11c 2012\ub144 5\uc6d4\ubd80\ud130 \ub3d9\uc77c\ubcf8 \ub300\uc9c0\uc9c4\uc758 \uc774\uc7ac\ubbfc\uc774 \uac70\uc8fc\ud558\uace0 \uc788\ub294 \uc784\uc2dc\uc8fc\ud0dd\uc744 \ud68c\uc6d0\ub4e4\uc774 \ubc29\ubb38\ud574 \uad50\ub958\ub97c \uc704\ud55c \u2018\ub9c8\uc74c\uc758 \uce58\uc720\u2019 \ud65c\ub3d9\uc744 \uad6d\uc81c \uc5f0\ud569\uc758 \ubc97 Asia-Pacific\uacfc \uacf5\ub3d9\uc73c\ub85c \uc9c4\ud589\ud558\uace0 \uc788\ub2e4. 2012\ub144 7\uc6d4\uc5d0\ub294 \uba85\uad6c\ud68c\uac00 \ub300\ud55c\ubbfc\uad6d\uacfc\uc758 \uad6d\uc81c \uce5c\uc120 \ucc28\uc6d0\uc5d0\uc11c \ud55c\uad6d\ud504\ub85c\uc57c\uad6c OB\ub2e8\uccb4\uc778 \uc77c\uad6c\ud68c \uc640 \uacf5\ub3d9\uc73c\ub85c \uccad\uac01\uc7a5\uc560\ub97c \uc553\ub294 \uc18c\ub144\ub4e4\uc5d0\uac8c \uc57c\uad6c \ud074\ub9ac\ub2c9\uacfc 7\uc6d4 20\uc77c \uc7a0\uc2e4\uc57c\uad6c\uc7a5\uc5d0\uc11c \ud55c\uc77c \ud504\ub85c \uc57c\uad6c \ub808\uc804\ub4dc\ub9e4\uce58\ub97c \uac1c\ucd5c\ud574 \ub300\ud55c\ubbfc\uad6d\uc758 \uc57c\uad6c \ud32c\ub4e4\uc744 \uc990\uac81\uac8c \ud588\ub2e4. \ub610\ud55c \uae30\ub150 \uc0ac\uc5c5\uc774\ub098 \uc5f0\uc218\ud68c\uc758 \ubc14\ub78c\ub300\ub85c \uc9c0\uc5ed\uc8fc\ubbfc, \uc790\uc704\ub300\uc6d0, \uae30\uc5c5\uc778\ub4e4, \uc120\uc218, \uac10\ub3c5\uc73c\ub85c\uc11c \uacbd\ud5d8\ud55c \uae30\ub85d\uc774\ub098 \uc2b9\ubd80, \uc9c0\ud718\ubd09\uc5d0 \uc5bd\ud600\uc788\ub294 \uc774\uc57c\uae30 \ub4f1 \ub9ce\uc740 \uac15\uc5f0\uc744 \uc5f4\uc5b4 \uad50\ud6c8\uc744 \uc804\ud558\uace0 \uc788\ub2e4.", "paragraph_answer": "\uadf8\ub9ac\uace0 \uc9c1\uc811\uc801\uc778 \uc9c0\uc6d0 \ud65c\ub3d9\uc73c\ub85c\uc11c 2012\ub144 5\uc6d4\ubd80\ud130 \ub3d9\uc77c\ubcf8 \ub300\uc9c0\uc9c4\uc758 \uc774\uc7ac\ubbfc\uc774 \uac70\uc8fc\ud558\uace0 \uc788\ub294 \uc784\uc2dc\uc8fc\ud0dd\uc744 \ud68c\uc6d0\ub4e4\uc774 \ubc29\ubb38\ud574 \uad50\ub958\ub97c \uc704\ud55c \u2018\ub9c8\uc74c\uc758 \uce58\uc720\u2019 \ud65c\ub3d9\uc744 \uad6d\uc81c \uc5f0\ud569\uc758 \ubc97 Asia-Pacific\uacfc \uacf5\ub3d9\uc73c\ub85c \uc9c4\ud589\ud558\uace0 \uc788\ub2e4. 2012\ub144 7\uc6d4\uc5d0\ub294 \uba85\uad6c\ud68c\uac00 \ub300\ud55c\ubbfc\uad6d\uacfc\uc758 \uad6d\uc81c \uce5c\uc120 \ucc28\uc6d0\uc5d0\uc11c \ud55c\uad6d\ud504\ub85c\uc57c\uad6c OB\ub2e8\uccb4\uc778 \uc77c\uad6c\ud68c \uc640 \uacf5\ub3d9\uc73c\ub85c \uccad\uac01\uc7a5\uc560\ub97c \uc553\ub294 \uc18c\ub144\ub4e4\uc5d0\uac8c \uc57c\uad6c \ud074\ub9ac\ub2c9\uacfc 7\uc6d4 20\uc77c \uc7a0\uc2e4\uc57c\uad6c\uc7a5\uc5d0\uc11c \ud55c\uc77c \ud504\ub85c \uc57c\uad6c \ub808\uc804\ub4dc\ub9e4\uce58\ub97c \uac1c\ucd5c\ud574 \ub300\ud55c\ubbfc\uad6d\uc758 \uc57c\uad6c \ud32c\ub4e4\uc744 \uc990\uac81\uac8c \ud588\ub2e4. \ub610\ud55c \uae30\ub150 \uc0ac\uc5c5\uc774\ub098 \uc5f0\uc218\ud68c\uc758 \ubc14\ub78c\ub300\ub85c \uc9c0\uc5ed\uc8fc\ubbfc, \uc790\uc704\ub300\uc6d0, \uae30\uc5c5\uc778\ub4e4, \uc120\uc218, \uac10\ub3c5\uc73c\ub85c\uc11c \uacbd\ud5d8\ud55c \uae30\ub85d\uc774\ub098 \uc2b9\ubd80, \uc9c0\ud718\ubd09\uc5d0 \uc5bd\ud600\uc788\ub294 \uc774\uc57c\uae30 \ub4f1 \ub9ce\uc740 \uac15\uc5f0\uc744 \uc5f4\uc5b4 \uad50\ud6c8\uc744 \uc804\ud558\uace0 \uc788\ub2e4.", "sentence_answer": "2012\ub144 7\uc6d4\uc5d0\ub294 \uba85\uad6c\ud68c\uac00 \ub300\ud55c\ubbfc\uad6d\uacfc\uc758 \uad6d\uc81c \uce5c\uc120 \ucc28\uc6d0\uc5d0\uc11c \ud55c\uad6d\ud504\ub85c\uc57c\uad6c OB\ub2e8\uccb4\uc778 \uc77c\uad6c\ud68c \uc640 \uacf5\ub3d9\uc73c\ub85c \uccad\uac01\uc7a5\uc560\ub97c \uc553\ub294 \uc18c\ub144\ub4e4\uc5d0\uac8c \uc57c\uad6c \ud074\ub9ac\ub2c9\uacfc 7\uc6d4 20\uc77c \uc7a0\uc2e4\uc57c\uad6c\uc7a5\uc5d0\uc11c \ud55c\uc77c \ud504\ub85c \uc57c\uad6c \ub808\uc804\ub4dc\ub9e4\uce58\ub97c \uac1c\ucd5c\ud574 \ub300\ud55c\ubbfc\uad6d\uc758 \uc57c\uad6c \ud32c\ub4e4\uc744 \uc990\uac81\uac8c \ud588\ub2e4.", "paragraph_id": "6548427-2-2", "paragraph_question": "question: \ud55c\uad6d\ud504\ub85c\uc57c\uad6c OB\ub2e8\uccb4\uc758 \uc774\ub984\uc740 \ubb34\uc5c7\uc778\uac00?, context: \uadf8\ub9ac\uace0 \uc9c1\uc811\uc801\uc778 \uc9c0\uc6d0 \ud65c\ub3d9\uc73c\ub85c\uc11c 2012\ub144 5\uc6d4\ubd80\ud130 \ub3d9\uc77c\ubcf8 \ub300\uc9c0\uc9c4\uc758 \uc774\uc7ac\ubbfc\uc774 \uac70\uc8fc\ud558\uace0 \uc788\ub294 \uc784\uc2dc\uc8fc\ud0dd\uc744 \ud68c\uc6d0\ub4e4\uc774 \ubc29\ubb38\ud574 \uad50\ub958\ub97c \uc704\ud55c \u2018\ub9c8\uc74c\uc758 \uce58\uc720\u2019 \ud65c\ub3d9\uc744 \uad6d\uc81c \uc5f0\ud569\uc758 \ubc97 Asia-Pacific\uacfc \uacf5\ub3d9\uc73c\ub85c \uc9c4\ud589\ud558\uace0 \uc788\ub2e4. 2012\ub144 7\uc6d4\uc5d0\ub294 \uba85\uad6c\ud68c\uac00 \ub300\ud55c\ubbfc\uad6d\uacfc\uc758 \uad6d\uc81c \uce5c\uc120 \ucc28\uc6d0\uc5d0\uc11c \ud55c\uad6d\ud504\ub85c\uc57c\uad6c OB\ub2e8\uccb4\uc778 \uc77c\uad6c\ud68c\uc640 \uacf5\ub3d9\uc73c\ub85c \uccad\uac01\uc7a5\uc560\ub97c \uc553\ub294 \uc18c\ub144\ub4e4\uc5d0\uac8c \uc57c\uad6c \ud074\ub9ac\ub2c9\uacfc 7\uc6d4 20\uc77c \uc7a0\uc2e4\uc57c\uad6c\uc7a5\uc5d0\uc11c \ud55c\uc77c \ud504\ub85c \uc57c\uad6c \ub808\uc804\ub4dc\ub9e4\uce58\ub97c \uac1c\ucd5c\ud574 \ub300\ud55c\ubbfc\uad6d\uc758 \uc57c\uad6c \ud32c\ub4e4\uc744 \uc990\uac81\uac8c \ud588\ub2e4. \ub610\ud55c \uae30\ub150 \uc0ac\uc5c5\uc774\ub098 \uc5f0\uc218\ud68c\uc758 \ubc14\ub78c\ub300\ub85c \uc9c0\uc5ed\uc8fc\ubbfc, \uc790\uc704\ub300\uc6d0, \uae30\uc5c5\uc778\ub4e4, \uc120\uc218, \uac10\ub3c5\uc73c\ub85c\uc11c \uacbd\ud5d8\ud55c \uae30\ub85d\uc774\ub098 \uc2b9\ubd80, \uc9c0\ud718\ubd09\uc5d0 \uc5bd\ud600\uc788\ub294 \uc774\uc57c\uae30 \ub4f1 \ub9ce\uc740 \uac15\uc5f0\uc744 \uc5f4\uc5b4 \uad50\ud6c8\uc744 \uc804\ud558\uace0 \uc788\ub2e4."} {"question": "KT\uc5d0\uc11c \uc774\uac83\uc744 \uc0ac\uc6a9\ud558\uc9c0 \uc54a\uace0\ub3c4 \uae30\uc874 \uc0ac\uc6a9\uc790\ub4e4\uc758 \uc801\uadf9\uc801\uc778 \ud65c\uc6a9\uc73c\ub85c \uc785\uc18c\ubb38\uc774 \ud37c\uc84c\ub294\ub370 \uc774\uac83\uc744 \ubb34\uc5c7\uc77c\uae4c?", "paragraph": "2010\ub144 5\uc6d4, \uc544\uc774\ud3f0\uc740 \ud310\ub9e4\ub7c9 70\ub9cc \ub300\ub97c \ub3cc\ud30c\ud558\uc600\ub2e4. 50\ub9cc \ub300\uc5d0\uc11c 60\ub9cc \ub300\uae4c\uc9c0 10\ub9cc \ub300\ub97c \ud310\ub9e4\ud558\ub294\ub370 27\uc77c\uc774 \uac78\ub838\uc73c\ub098 60\ub9cc \ub300\uc5d0\uc11c 70\ub9cc \ub300\ub97c \ub3cc\ud30c\ud558\ub294\ub370\uc5d0\ub294 25\uc77c\uc774 \uac78\ub824 \ud310\ub9e4\ub7c9 \uc99d\uac00 \uc18d\ub3c4\ub294 \uc624\ud788\ub824 \ube68\ub77c\uc9c4 \uac83\uc73c\ub85c \ub4dc\ub7ec\ub0ac\ub2e4. KT\uce21\uc5d0 \uc758\ud574 \uacf5\uaca9\uc801\uc778 \ub9c8\ucf00\ud305\uc774 \uc5c6\uc5c8\uc74c\uc5d0\ub3c4 \ubd88\uad6c\ud558\uace0 \uc774\ub807\uac8c \ud310\ub9e4\ub7c9 \uc18d\ub3c4\uac00 \ub354 \uc99d\uac00\ud558\ub294 \uc774\uc720\ub294 \uae30\uc874 \uc0ac\uc6a9\uc790\ub4e4\uc758 \uc801\uadf9\uc801\uc778 \ud65c\uc6a9\uc73c\ub85c \uc778\ud574 \uc785\uc18c\ubb38\uc774 \ud37c\uc84c\uae30 \ub54c\ubb38\uc73c\ub85c \ubd84\uc11d\ub41c\ub2e4. 2010\ub144 10\uc6d4\uc5d0\ub294 100\ub9cc \ub300\ub97c \ub3cc\ud30c\ud588\ub2e4. 2010\ub144 11\uc6d4\uc5d0\ub294 \ucd1d \uac00\uc785\uc790 \uc218\uac00 162\ub9cc \uba85\uc774 \ub418\uc5c8\ub2e4. \uc774\ud6c4 \uc544\uc774\ud3f0 4\uac00 \ubc1c\ub9e4\ub418\uc5b4 \uc544\uc774\ud3f0\uc758 \ud310\ub9e4\ub7c9\uc740 \uc9c0\uc18d\uc801\uc73c\ub85c \ub298\uc5c8\uace0, 2011\ub144 1\uc6d4 23\uc77c \ub204\uc801 \ud310\ub9e4\ub7c9 200\ub9cc \ub300\ub97c \ub3cc\ud30c\ud588\ub2e4. \uc544\uc774\ud3f0\uc758 \uc778\uae30\uc5d0 \ud798\uc785\uc5b4 \uc560\ud50c\ucf54\ub9ac\uc544\uc758 \ub9e4\ucd9c\ub3c4 2009\ub144\uc5d0 \ube44\ud574 10\ubc30 \uc774\uc0c1 \uae09\uc99d\ud574 1\uc870 5000\uc5b5~2\uc870\uc6d0\uc758 \uc218\uc775\uc744 \uc62c\ub838\ub2e4.", "answer": "\uacf5\uaca9\uc801\uc778 \ub9c8\ucf00\ud305", "sentence": "KT\uce21\uc5d0 \uc758\ud574 \uacf5\uaca9\uc801\uc778 \ub9c8\ucf00\ud305 \uc774 \uc5c6\uc5c8\uc74c\uc5d0\ub3c4 \ubd88\uad6c\ud558\uace0 \uc774\ub807\uac8c \ud310\ub9e4\ub7c9 \uc18d\ub3c4\uac00 \ub354 \uc99d\uac00\ud558\ub294 \uc774\uc720\ub294 \uae30\uc874 \uc0ac\uc6a9\uc790\ub4e4\uc758 \uc801\uadf9\uc801\uc778 \ud65c\uc6a9\uc73c\ub85c \uc778\ud574 \uc785\uc18c\ubb38\uc774 \ud37c\uc84c\uae30 \ub54c\ubb38\uc73c\ub85c \ubd84\uc11d\ub41c\ub2e4.", "paragraph_sentence": "2010\ub144 5\uc6d4, \uc544\uc774\ud3f0\uc740 \ud310\ub9e4\ub7c9 70\ub9cc \ub300\ub97c \ub3cc\ud30c\ud558\uc600\ub2e4. 50\ub9cc \ub300\uc5d0\uc11c 60\ub9cc \ub300\uae4c\uc9c0 10\ub9cc \ub300\ub97c \ud310\ub9e4\ud558\ub294\ub370 27\uc77c\uc774 \uac78\ub838\uc73c\ub098 60\ub9cc \ub300\uc5d0\uc11c 70\ub9cc \ub300\ub97c \ub3cc\ud30c\ud558\ub294\ub370\uc5d0\ub294 25\uc77c\uc774 \uac78\ub824 \ud310\ub9e4\ub7c9 \uc99d\uac00 \uc18d\ub3c4\ub294 \uc624\ud788\ub824 \ube68\ub77c\uc9c4 \uac83\uc73c\ub85c \ub4dc\ub7ec\ub0ac\ub2e4. KT\uce21\uc5d0 \uc758\ud574 \uacf5\uaca9\uc801\uc778 \ub9c8\ucf00\ud305 \uc774 \uc5c6\uc5c8\uc74c\uc5d0\ub3c4 \ubd88\uad6c\ud558\uace0 \uc774\ub807\uac8c \ud310\ub9e4\ub7c9 \uc18d\ub3c4\uac00 \ub354 \uc99d\uac00\ud558\ub294 \uc774\uc720\ub294 \uae30\uc874 \uc0ac\uc6a9\uc790\ub4e4\uc758 \uc801\uadf9\uc801\uc778 \ud65c\uc6a9\uc73c\ub85c \uc778\ud574 \uc785\uc18c\ubb38\uc774 \ud37c\uc84c\uae30 \ub54c\ubb38\uc73c\ub85c \ubd84\uc11d\ub41c\ub2e4. 2010\ub144 10\uc6d4\uc5d0\ub294 100\ub9cc \ub300\ub97c \ub3cc\ud30c\ud588\ub2e4. 2010\ub144 11\uc6d4\uc5d0\ub294 \ucd1d \uac00\uc785\uc790 \uc218\uac00 162\ub9cc \uba85\uc774 \ub418\uc5c8\ub2e4. \uc774\ud6c4 \uc544\uc774\ud3f0 4\uac00 \ubc1c\ub9e4\ub418\uc5b4 \uc544\uc774\ud3f0\uc758 \ud310\ub9e4\ub7c9\uc740 \uc9c0\uc18d\uc801\uc73c\ub85c \ub298\uc5c8\uace0, 2011\ub144 1\uc6d4 23\uc77c \ub204\uc801 \ud310\ub9e4\ub7c9 200\ub9cc \ub300\ub97c \ub3cc\ud30c\ud588\ub2e4. \uc544\uc774\ud3f0\uc758 \uc778\uae30\uc5d0 \ud798\uc785\uc5b4 \uc560\ud50c\ucf54\ub9ac\uc544\uc758 \ub9e4\ucd9c\ub3c4 2009\ub144\uc5d0 \ube44\ud574 10\ubc30 \uc774\uc0c1 \uae09\uc99d\ud574 1\uc870 5000\uc5b5~2\uc870\uc6d0\uc758 \uc218\uc775\uc744 \uc62c\ub838\ub2e4.", "paragraph_answer": "2010\ub144 5\uc6d4, \uc544\uc774\ud3f0\uc740 \ud310\ub9e4\ub7c9 70\ub9cc \ub300\ub97c \ub3cc\ud30c\ud558\uc600\ub2e4. 50\ub9cc \ub300\uc5d0\uc11c 60\ub9cc \ub300\uae4c\uc9c0 10\ub9cc \ub300\ub97c \ud310\ub9e4\ud558\ub294\ub370 27\uc77c\uc774 \uac78\ub838\uc73c\ub098 60\ub9cc \ub300\uc5d0\uc11c 70\ub9cc \ub300\ub97c \ub3cc\ud30c\ud558\ub294\ub370\uc5d0\ub294 25\uc77c\uc774 \uac78\ub824 \ud310\ub9e4\ub7c9 \uc99d\uac00 \uc18d\ub3c4\ub294 \uc624\ud788\ub824 \ube68\ub77c\uc9c4 \uac83\uc73c\ub85c \ub4dc\ub7ec\ub0ac\ub2e4. KT\uce21\uc5d0 \uc758\ud574 \uacf5\uaca9\uc801\uc778 \ub9c8\ucf00\ud305 \uc774 \uc5c6\uc5c8\uc74c\uc5d0\ub3c4 \ubd88\uad6c\ud558\uace0 \uc774\ub807\uac8c \ud310\ub9e4\ub7c9 \uc18d\ub3c4\uac00 \ub354 \uc99d\uac00\ud558\ub294 \uc774\uc720\ub294 \uae30\uc874 \uc0ac\uc6a9\uc790\ub4e4\uc758 \uc801\uadf9\uc801\uc778 \ud65c\uc6a9\uc73c\ub85c \uc778\ud574 \uc785\uc18c\ubb38\uc774 \ud37c\uc84c\uae30 \ub54c\ubb38\uc73c\ub85c \ubd84\uc11d\ub41c\ub2e4. 2010\ub144 10\uc6d4\uc5d0\ub294 100\ub9cc \ub300\ub97c \ub3cc\ud30c\ud588\ub2e4. 2010\ub144 11\uc6d4\uc5d0\ub294 \ucd1d \uac00\uc785\uc790 \uc218\uac00 162\ub9cc \uba85\uc774 \ub418\uc5c8\ub2e4. \uc774\ud6c4 \uc544\uc774\ud3f0 4\uac00 \ubc1c\ub9e4\ub418\uc5b4 \uc544\uc774\ud3f0\uc758 \ud310\ub9e4\ub7c9\uc740 \uc9c0\uc18d\uc801\uc73c\ub85c \ub298\uc5c8\uace0, 2011\ub144 1\uc6d4 23\uc77c \ub204\uc801 \ud310\ub9e4\ub7c9 200\ub9cc \ub300\ub97c \ub3cc\ud30c\ud588\ub2e4. \uc544\uc774\ud3f0\uc758 \uc778\uae30\uc5d0 \ud798\uc785\uc5b4 \uc560\ud50c\ucf54\ub9ac\uc544\uc758 \ub9e4\ucd9c\ub3c4 2009\ub144\uc5d0 \ube44\ud574 10\ubc30 \uc774\uc0c1 \uae09\uc99d\ud574 1\uc870 5000\uc5b5~2\uc870\uc6d0\uc758 \uc218\uc775\uc744 \uc62c\ub838\ub2e4.", "sentence_answer": "KT\uce21\uc5d0 \uc758\ud574 \uacf5\uaca9\uc801\uc778 \ub9c8\ucf00\ud305 \uc774 \uc5c6\uc5c8\uc74c\uc5d0\ub3c4 \ubd88\uad6c\ud558\uace0 \uc774\ub807\uac8c \ud310\ub9e4\ub7c9 \uc18d\ub3c4\uac00 \ub354 \uc99d\uac00\ud558\ub294 \uc774\uc720\ub294 \uae30\uc874 \uc0ac\uc6a9\uc790\ub4e4\uc758 \uc801\uadf9\uc801\uc778 \ud65c\uc6a9\uc73c\ub85c \uc778\ud574 \uc785\uc18c\ubb38\uc774 \ud37c\uc84c\uae30 \ub54c\ubb38\uc73c\ub85c \ubd84\uc11d\ub41c\ub2e4.", "paragraph_id": "6512497-5-1", "paragraph_question": "question: KT\uc5d0\uc11c \uc774\uac83\uc744 \uc0ac\uc6a9\ud558\uc9c0 \uc54a\uace0\ub3c4 \uae30\uc874 \uc0ac\uc6a9\uc790\ub4e4\uc758 \uc801\uadf9\uc801\uc778 \ud65c\uc6a9\uc73c\ub85c \uc785\uc18c\ubb38\uc774 \ud37c\uc84c\ub294\ub370 \uc774\uac83\uc744 \ubb34\uc5c7\uc77c\uae4c?, context: 2010\ub144 5\uc6d4, \uc544\uc774\ud3f0\uc740 \ud310\ub9e4\ub7c9 70\ub9cc \ub300\ub97c \ub3cc\ud30c\ud558\uc600\ub2e4. 50\ub9cc \ub300\uc5d0\uc11c 60\ub9cc \ub300\uae4c\uc9c0 10\ub9cc \ub300\ub97c \ud310\ub9e4\ud558\ub294\ub370 27\uc77c\uc774 \uac78\ub838\uc73c\ub098 60\ub9cc \ub300\uc5d0\uc11c 70\ub9cc \ub300\ub97c \ub3cc\ud30c\ud558\ub294\ub370\uc5d0\ub294 25\uc77c\uc774 \uac78\ub824 \ud310\ub9e4\ub7c9 \uc99d\uac00 \uc18d\ub3c4\ub294 \uc624\ud788\ub824 \ube68\ub77c\uc9c4 \uac83\uc73c\ub85c \ub4dc\ub7ec\ub0ac\ub2e4. KT\uce21\uc5d0 \uc758\ud574 \uacf5\uaca9\uc801\uc778 \ub9c8\ucf00\ud305\uc774 \uc5c6\uc5c8\uc74c\uc5d0\ub3c4 \ubd88\uad6c\ud558\uace0 \uc774\ub807\uac8c \ud310\ub9e4\ub7c9 \uc18d\ub3c4\uac00 \ub354 \uc99d\uac00\ud558\ub294 \uc774\uc720\ub294 \uae30\uc874 \uc0ac\uc6a9\uc790\ub4e4\uc758 \uc801\uadf9\uc801\uc778 \ud65c\uc6a9\uc73c\ub85c \uc778\ud574 \uc785\uc18c\ubb38\uc774 \ud37c\uc84c\uae30 \ub54c\ubb38\uc73c\ub85c \ubd84\uc11d\ub41c\ub2e4. 2010\ub144 10\uc6d4\uc5d0\ub294 100\ub9cc \ub300\ub97c \ub3cc\ud30c\ud588\ub2e4. 2010\ub144 11\uc6d4\uc5d0\ub294 \ucd1d \uac00\uc785\uc790 \uc218\uac00 162\ub9cc \uba85\uc774 \ub418\uc5c8\ub2e4. \uc774\ud6c4 \uc544\uc774\ud3f0 4\uac00 \ubc1c\ub9e4\ub418\uc5b4 \uc544\uc774\ud3f0\uc758 \ud310\ub9e4\ub7c9\uc740 \uc9c0\uc18d\uc801\uc73c\ub85c \ub298\uc5c8\uace0, 2011\ub144 1\uc6d4 23\uc77c \ub204\uc801 \ud310\ub9e4\ub7c9 200\ub9cc \ub300\ub97c \ub3cc\ud30c\ud588\ub2e4. \uc544\uc774\ud3f0\uc758 \uc778\uae30\uc5d0 \ud798\uc785\uc5b4 \uc560\ud50c\ucf54\ub9ac\uc544\uc758 \ub9e4\ucd9c\ub3c4 2009\ub144\uc5d0 \ube44\ud574 10\ubc30 \uc774\uc0c1 \uae09\uc99d\ud574 1\uc870 5000\uc5b5~2\uc870\uc6d0\uc758 \uc218\uc775\uc744 \uc62c\ub838\ub2e4."} {"question": "\ud070 \ucda9\ub3cc\uad6c\ub4e4\uc774 \uc5c6\ub294 \uad6c\uc5ed\uc740?", "paragraph": "\uc774\ub2e4\ub294 \ud0dc\uc591\uacc4\uc5d0\uc11c \uac00\uc7a5 \uc2ec\ud558\uac8c \ucda9\ub3cc\uad6c\ub85c \ub36e\uc5ec \uc788\ub294 \ucc9c\uccb4 \uc911 \ud558\ub098\uc774\uba70, \uc774\ub7f0 \ud45c\uba74\uc744 \ud615\uc131\ud55c \uc77c\ucc28\uc801 \uacfc\uc815\uc740 \ucda9\ub3cc \uc0ac\uac74\uc774\ub2e4. \ucda9\ub3cc\uad6c\uc758 \uc591\uc774 \ud3ec\ud654 \uc0c1\ud0dc\uc5d0 \ub3c4\ub2ec\ud558\uc5ec, \uc0c8\ub85c \uc77c\uc5b4\ub09c \ucda9\ub3cc \uc0ac\uac74\uc774 \uc624\ub798\ub41c \ucda9\ub3cc\uad6c\ub97c \uc9c0\uc6cc \ubc84\ub9ac\uae30\uc5d0 \ucda9\ub3cc\uad6c\uc758 \ucd1d \uc218\ub294 \ub300\ucda9 \uc77c\uc815\ud558\ub2e4. \uc774\ub2e4\uc758 \ud45c\uba74\uc744 \ub4a4\ub36e\uc740 \ucda9\ub3cc\uad6c\ub4e4\uc740 \ud06c\uae30\uc640 \ub098\uc774\uac00 \uac01\uc591\uac01\uc0c9\uc73c\ub85c, \ucd5c\uadfc\uc5d0 \ud615\uc131\ub41c \uac83\ubd80\ud130 \uc774\ub2e4\uc758 \ub098\uc774\ub9cc\ud07c \uc624\ub798\ub41c \uac83\uae4c\uc9c0 \uc788\ub2e4. \uac00\uc7a5 \uc624\ub798\ub41c \uac83\uc740 \uc544\ub9c8 \ucf54\ub85c\ub2c8\uc2a4\uc871 \ubaa8\ucc9c\uccb4\uc758 \ub300\ubd95\uad34 \ub54c \ub9cc\ub4e4\uc5b4\uc84c\uc744 \uac83\uc774\ub2e4. \uac70\uc758 12 \ud0ac\ub85c\ubbf8\ud130\uc5d0 \ub2ec\ud558\ub294 \uac00\uc7a5 \ud070 \ucda9\ub3cc\uad6c\uc778 \ucda9\ub3cc\uad6c \ub77c\uc2a4\ucf54\ub97c \ube44\ub86f, \uad6c\uc5ed 2\uc5d0\ub294 \uac70\uc758 \ub300\ubd80\ubd84\uc758 \ucda9\ub3cc\uad6c\ub4e4\uc758 \uc9c1\uacbd\uc774 6 \ud0ac\ub85c\ubbf8\ud130\ub97c \ub118\uc9c0\ub9cc \uad6c\uc5ed 1\uc5d0\ub294 \ud070 \ucda9\ub3cc\uad6c\ub4e4\uc774 \uc804\ud600 \uc5c6\ub2e4. \uc0ac\uc2ac \ubaa8\uc591\uc73c\ub85c \ubc30\uc5f4\ub41c \ucda9\ub3cc\uad6c\ub4e4\ub3c4 \uc788\ub2e4.", "answer": "\uad6c\uc5ed 1", "sentence": "\uac70\uc758 12 \ud0ac\ub85c\ubbf8\ud130\uc5d0 \ub2ec\ud558\ub294 \uac00\uc7a5 \ud070 \ucda9\ub3cc\uad6c\uc778 \ucda9\ub3cc\uad6c \ub77c\uc2a4\ucf54\ub97c \ube44\ub86f, \uad6c\uc5ed 2\uc5d0\ub294 \uac70\uc758 \ub300\ubd80\ubd84\uc758 \ucda9\ub3cc\uad6c\ub4e4\uc758 \uc9c1\uacbd\uc774 6 \ud0ac\ub85c\ubbf8\ud130\ub97c \ub118\uc9c0\ub9cc \uad6c\uc5ed 1 \uc5d0\ub294 \ud070 \ucda9\ub3cc\uad6c\ub4e4\uc774 \uc804\ud600 \uc5c6\ub2e4.", "paragraph_sentence": "\uc774\ub2e4\ub294 \ud0dc\uc591\uacc4\uc5d0\uc11c \uac00\uc7a5 \uc2ec\ud558\uac8c \ucda9\ub3cc\uad6c\ub85c \ub36e\uc5ec \uc788\ub294 \ucc9c\uccb4 \uc911 \ud558\ub098\uc774\uba70, \uc774\ub7f0 \ud45c\uba74\uc744 \ud615\uc131\ud55c \uc77c\ucc28\uc801 \uacfc\uc815\uc740 \ucda9\ub3cc \uc0ac\uac74\uc774\ub2e4. \ucda9\ub3cc\uad6c\uc758 \uc591\uc774 \ud3ec\ud654 \uc0c1\ud0dc\uc5d0 \ub3c4\ub2ec\ud558\uc5ec, \uc0c8\ub85c \uc77c\uc5b4\ub09c \ucda9\ub3cc \uc0ac\uac74\uc774 \uc624\ub798\ub41c \ucda9\ub3cc\uad6c\ub97c \uc9c0\uc6cc \ubc84\ub9ac\uae30\uc5d0 \ucda9\ub3cc\uad6c\uc758 \ucd1d \uc218\ub294 \ub300\ucda9 \uc77c\uc815\ud558\ub2e4. \uc774\ub2e4\uc758 \ud45c\uba74\uc744 \ub4a4\ub36e\uc740 \ucda9\ub3cc\uad6c\ub4e4\uc740 \ud06c\uae30\uc640 \ub098\uc774\uac00 \uac01\uc591\uac01\uc0c9\uc73c\ub85c, \ucd5c\uadfc\uc5d0 \ud615\uc131\ub41c \uac83\ubd80\ud130 \uc774\ub2e4\uc758 \ub098\uc774\ub9cc\ud07c \uc624\ub798\ub41c \uac83\uae4c\uc9c0 \uc788\ub2e4. \uac00\uc7a5 \uc624\ub798\ub41c \uac83\uc740 \uc544\ub9c8 \ucf54\ub85c\ub2c8\uc2a4\uc871 \ubaa8\ucc9c\uccb4\uc758 \ub300\ubd95\uad34 \ub54c \ub9cc\ub4e4\uc5b4\uc84c\uc744 \uac83\uc774\ub2e4. \uac70\uc758 12 \ud0ac\ub85c\ubbf8\ud130\uc5d0 \ub2ec\ud558\ub294 \uac00\uc7a5 \ud070 \ucda9\ub3cc\uad6c\uc778 \ucda9\ub3cc\uad6c \ub77c\uc2a4\ucf54\ub97c \ube44\ub86f, \uad6c\uc5ed 2\uc5d0\ub294 \uac70\uc758 \ub300\ubd80\ubd84\uc758 \ucda9\ub3cc\uad6c\ub4e4\uc758 \uc9c1\uacbd\uc774 6 \ud0ac\ub85c\ubbf8\ud130\ub97c \ub118\uc9c0\ub9cc \uad6c\uc5ed 1 \uc5d0\ub294 \ud070 \ucda9\ub3cc\uad6c\ub4e4\uc774 \uc804\ud600 \uc5c6\ub2e4. \uc0ac\uc2ac \ubaa8\uc591\uc73c\ub85c \ubc30\uc5f4\ub41c \ucda9\ub3cc\uad6c\ub4e4\ub3c4 \uc788\ub2e4.", "paragraph_answer": "\uc774\ub2e4\ub294 \ud0dc\uc591\uacc4\uc5d0\uc11c \uac00\uc7a5 \uc2ec\ud558\uac8c \ucda9\ub3cc\uad6c\ub85c \ub36e\uc5ec \uc788\ub294 \ucc9c\uccb4 \uc911 \ud558\ub098\uc774\uba70, \uc774\ub7f0 \ud45c\uba74\uc744 \ud615\uc131\ud55c \uc77c\ucc28\uc801 \uacfc\uc815\uc740 \ucda9\ub3cc \uc0ac\uac74\uc774\ub2e4. \ucda9\ub3cc\uad6c\uc758 \uc591\uc774 \ud3ec\ud654 \uc0c1\ud0dc\uc5d0 \ub3c4\ub2ec\ud558\uc5ec, \uc0c8\ub85c \uc77c\uc5b4\ub09c \ucda9\ub3cc \uc0ac\uac74\uc774 \uc624\ub798\ub41c \ucda9\ub3cc\uad6c\ub97c \uc9c0\uc6cc \ubc84\ub9ac\uae30\uc5d0 \ucda9\ub3cc\uad6c\uc758 \ucd1d \uc218\ub294 \ub300\ucda9 \uc77c\uc815\ud558\ub2e4. \uc774\ub2e4\uc758 \ud45c\uba74\uc744 \ub4a4\ub36e\uc740 \ucda9\ub3cc\uad6c\ub4e4\uc740 \ud06c\uae30\uc640 \ub098\uc774\uac00 \uac01\uc591\uac01\uc0c9\uc73c\ub85c, \ucd5c\uadfc\uc5d0 \ud615\uc131\ub41c \uac83\ubd80\ud130 \uc774\ub2e4\uc758 \ub098\uc774\ub9cc\ud07c \uc624\ub798\ub41c \uac83\uae4c\uc9c0 \uc788\ub2e4. \uac00\uc7a5 \uc624\ub798\ub41c \uac83\uc740 \uc544\ub9c8 \ucf54\ub85c\ub2c8\uc2a4\uc871 \ubaa8\ucc9c\uccb4\uc758 \ub300\ubd95\uad34 \ub54c \ub9cc\ub4e4\uc5b4\uc84c\uc744 \uac83\uc774\ub2e4. \uac70\uc758 12 \ud0ac\ub85c\ubbf8\ud130\uc5d0 \ub2ec\ud558\ub294 \uac00\uc7a5 \ud070 \ucda9\ub3cc\uad6c\uc778 \ucda9\ub3cc\uad6c \ub77c\uc2a4\ucf54\ub97c \ube44\ub86f, \uad6c\uc5ed 2\uc5d0\ub294 \uac70\uc758 \ub300\ubd80\ubd84\uc758 \ucda9\ub3cc\uad6c\ub4e4\uc758 \uc9c1\uacbd\uc774 6 \ud0ac\ub85c\ubbf8\ud130\ub97c \ub118\uc9c0\ub9cc \uad6c\uc5ed 1 \uc5d0\ub294 \ud070 \ucda9\ub3cc\uad6c\ub4e4\uc774 \uc804\ud600 \uc5c6\ub2e4. \uc0ac\uc2ac \ubaa8\uc591\uc73c\ub85c \ubc30\uc5f4\ub41c \ucda9\ub3cc\uad6c\ub4e4\ub3c4 \uc788\ub2e4.", "sentence_answer": "\uac70\uc758 12 \ud0ac\ub85c\ubbf8\ud130\uc5d0 \ub2ec\ud558\ub294 \uac00\uc7a5 \ud070 \ucda9\ub3cc\uad6c\uc778 \ucda9\ub3cc\uad6c \ub77c\uc2a4\ucf54\ub97c \ube44\ub86f, \uad6c\uc5ed 2\uc5d0\ub294 \uac70\uc758 \ub300\ubd80\ubd84\uc758 \ucda9\ub3cc\uad6c\ub4e4\uc758 \uc9c1\uacbd\uc774 6 \ud0ac\ub85c\ubbf8\ud130\ub97c \ub118\uc9c0\ub9cc \uad6c\uc5ed 1 \uc5d0\ub294 \ud070 \ucda9\ub3cc\uad6c\ub4e4\uc774 \uc804\ud600 \uc5c6\ub2e4.", "paragraph_id": "6510299-1-2", "paragraph_question": "question: \ud070 \ucda9\ub3cc\uad6c\ub4e4\uc774 \uc5c6\ub294 \uad6c\uc5ed\uc740?, context: \uc774\ub2e4\ub294 \ud0dc\uc591\uacc4\uc5d0\uc11c \uac00\uc7a5 \uc2ec\ud558\uac8c \ucda9\ub3cc\uad6c\ub85c \ub36e\uc5ec \uc788\ub294 \ucc9c\uccb4 \uc911 \ud558\ub098\uc774\uba70, \uc774\ub7f0 \ud45c\uba74\uc744 \ud615\uc131\ud55c \uc77c\ucc28\uc801 \uacfc\uc815\uc740 \ucda9\ub3cc \uc0ac\uac74\uc774\ub2e4. \ucda9\ub3cc\uad6c\uc758 \uc591\uc774 \ud3ec\ud654 \uc0c1\ud0dc\uc5d0 \ub3c4\ub2ec\ud558\uc5ec, \uc0c8\ub85c \uc77c\uc5b4\ub09c \ucda9\ub3cc \uc0ac\uac74\uc774 \uc624\ub798\ub41c \ucda9\ub3cc\uad6c\ub97c \uc9c0\uc6cc \ubc84\ub9ac\uae30\uc5d0 \ucda9\ub3cc\uad6c\uc758 \ucd1d \uc218\ub294 \ub300\ucda9 \uc77c\uc815\ud558\ub2e4. \uc774\ub2e4\uc758 \ud45c\uba74\uc744 \ub4a4\ub36e\uc740 \ucda9\ub3cc\uad6c\ub4e4\uc740 \ud06c\uae30\uc640 \ub098\uc774\uac00 \uac01\uc591\uac01\uc0c9\uc73c\ub85c, \ucd5c\uadfc\uc5d0 \ud615\uc131\ub41c \uac83\ubd80\ud130 \uc774\ub2e4\uc758 \ub098\uc774\ub9cc\ud07c \uc624\ub798\ub41c \uac83\uae4c\uc9c0 \uc788\ub2e4. \uac00\uc7a5 \uc624\ub798\ub41c \uac83\uc740 \uc544\ub9c8 \ucf54\ub85c\ub2c8\uc2a4\uc871 \ubaa8\ucc9c\uccb4\uc758 \ub300\ubd95\uad34 \ub54c \ub9cc\ub4e4\uc5b4\uc84c\uc744 \uac83\uc774\ub2e4. \uac70\uc758 12 \ud0ac\ub85c\ubbf8\ud130\uc5d0 \ub2ec\ud558\ub294 \uac00\uc7a5 \ud070 \ucda9\ub3cc\uad6c\uc778 \ucda9\ub3cc\uad6c \ub77c\uc2a4\ucf54\ub97c \ube44\ub86f, \uad6c\uc5ed 2\uc5d0\ub294 \uac70\uc758 \ub300\ubd80\ubd84\uc758 \ucda9\ub3cc\uad6c\ub4e4\uc758 \uc9c1\uacbd\uc774 6 \ud0ac\ub85c\ubbf8\ud130\ub97c \ub118\uc9c0\ub9cc \uad6c\uc5ed 1\uc5d0\ub294 \ud070 \ucda9\ub3cc\uad6c\ub4e4\uc774 \uc804\ud600 \uc5c6\ub2e4. \uc0ac\uc2ac \ubaa8\uc591\uc73c\ub85c \ubc30\uc5f4\ub41c \ucda9\ub3cc\uad6c\ub4e4\ub3c4 \uc788\ub2e4."} {"question": "\uae09\uc131\uc911\uc774\uc5fc \ud658\uc790\uac00 \uace0\uc6a9\ub7c9 \ud56d\uc0dd\uc81c\ub97c \uac70\ub974\uc9c0 \uc54a\uace0 \uc758\uc0ac\uc758 \uad8c\uc720\uc5d0 \ub530\ub77c \ubcf5\uc6a9\ud558\uc600\uc744\ub54c \uc5c6\uc568 \uc218 \uc788\ub294 \uade0\uc740 \ubb34\uc5c7\uc778\uac00?", "paragraph": "\ud56d\uc0dd\uc81c \ub0b4\uc131\uade0\uc740 \uc5b4\ub9b0\uc774\uc9d1\uc5d0 \ub2e4\ub2c8\ub294 \uacbd\uc6b0, \ucd5c\uadfc 30\uc77c \uc774\ub0b4 \ud56d\uc0dd\uc81c\ub97c \uc0ac\uc6a9\ud55c \uacbd\uc6b0, 24\uac1c\uc6d4 \uc774\ud558\uc778 \uacbd\uc6b0 \ub0b4\uc131\uade0\uc758 \uac00\ub2a5\uc131\uc774 \ub192\uae30 \ub54c\ubb38\uc5d0 \uc774\ub7f0 \uacbd\uc6b0 \uace0\uc6a9\ub7c9 \uc544\ubaa9\uc2dc\uc2e4\ub9b0 \uc0ac\uc6a9\uc774 \uc88b\ub2e4. \ubd80\ube44\ub3d9\uc5fc \ub4f1\uc5d0\uc11c \ud1b5\uc0c1 \uc6a9\ub7c9\uc778 20~40 mg/kg \uacfc \ube44\uad50\ud558\uba74 \uae09\uc131 \uc911\uc774\uc5fc\uc758 \uace0\uc6a9\ub7c9 \ud56d\uc0dd\uc81c \ubcf5\uc6a9 \uc2dc \ucca0\uc800\ud55c \ubcf5\uc6a9\uc744 \ud574\uc57c \ub0b4\uc131\uade0\uc744 \uc5c6\uc568 \uc218 \uc788\uc74c\uc5d0\ub3c4 \ud56d\uc0dd\uc81c\ub97c \uac70\ub974\ub294 \uac78 \uc88b\uc544\ud558\ub294 \ud55c\uad6d\uc778\uc758 \uc2b5\uad00\uc740 \ud56d\uc0dd\uc81c \ub0b4\uc131\uc744 \ub354\uc6b1 \uc545\ud654\uc2dc\ud0b4\uc744 \uace0\ub824\ud574\uc57c \ud55c\ub2e4. \ud56d\uc0dd\uc81c \ubcf5\uc6a9 \uae30\uac04\uc740 \ud1b5\uc0c1 5\uc77c\uc774\ub098 \uc758\uc0ac\uc758 \ud310\ub2e8\uc5d0 \ub530\ub77c \uacb0\uc815\ud558\uba70, 24\uac1c\uc6d4 \uc774\ud558, \uace0\ub9c9 \ucc9c\uacf5\uc774 \uc788\ub294 \uacbd\uc6b0, \uc7ac\ubc1c\uc131 \uae09\uc131 \uc911\uc774\uc5fc\uc778 \uacbd\uc6b0 10\uc77c\uae4c\uc9c0 \uc5f0\uc7a5 \uc0ac\uc6a9\uc744 \uad8c\uc7a5\ud558\uace0 \uc788\ub2e4. \uc7a5\uc5fc, \uad6c\ud1a0 \ub4f1\uc73c\ub85c \uc778\ud574 \uacbd\uad6c \ud56d\uc0dd\uc81c \ud22c\uc5ec\uac00 \ubd88\uac00\ub2a5\ud560 \uacbd\uc6b0 Ceftriaxone (50 mg/kg) \uc8fc\uc0ac\ub97c 1\ud68c 1~3\uc77c \uc0ac\uc6a9 \uc2dc \uacbd\uad6c \ud56d\uc0dd\uc81c\uc640 \ub3d9\uc77c\ud55c \ud6a8\uacfc\ub97c \ubcfc \uc218 \uc788\ub2e4.", "answer": "\ub0b4\uc131\uade0", "sentence": "\ud56d\uc0dd\uc81c \ub0b4\uc131\uade0 \uc740 \uc5b4\ub9b0\uc774\uc9d1\uc5d0 \ub2e4\ub2c8\ub294 \uacbd\uc6b0, \ucd5c\uadfc 30\uc77c \uc774\ub0b4 \ud56d\uc0dd\uc81c\ub97c \uc0ac\uc6a9\ud55c \uacbd\uc6b0, 24\uac1c\uc6d4 \uc774\ud558\uc778 \uacbd\uc6b0 \ub0b4\uc131\uade0\uc758 \uac00\ub2a5\uc131\uc774 \ub192\uae30 \ub54c\ubb38\uc5d0 \uc774\ub7f0 \uacbd\uc6b0 \uace0\uc6a9\ub7c9 \uc544\ubaa9\uc2dc\uc2e4\ub9b0 \uc0ac\uc6a9\uc774 \uc88b\ub2e4.", "paragraph_sentence": " \ud56d\uc0dd\uc81c \ub0b4\uc131\uade0 \uc740 \uc5b4\ub9b0\uc774\uc9d1\uc5d0 \ub2e4\ub2c8\ub294 \uacbd\uc6b0, \ucd5c\uadfc 30\uc77c \uc774\ub0b4 \ud56d\uc0dd\uc81c\ub97c \uc0ac\uc6a9\ud55c \uacbd\uc6b0, 24\uac1c\uc6d4 \uc774\ud558\uc778 \uacbd\uc6b0 \ub0b4\uc131\uade0\uc758 \uac00\ub2a5\uc131\uc774 \ub192\uae30 \ub54c\ubb38\uc5d0 \uc774\ub7f0 \uacbd\uc6b0 \uace0\uc6a9\ub7c9 \uc544\ubaa9\uc2dc\uc2e4\ub9b0 \uc0ac\uc6a9\uc774 \uc88b\ub2e4. \ubd80\ube44\ub3d9\uc5fc \ub4f1\uc5d0\uc11c \ud1b5\uc0c1 \uc6a9\ub7c9\uc778 20~40 mg/kg \uacfc \ube44\uad50\ud558\uba74 \uae09\uc131 \uc911\uc774\uc5fc\uc758 \uace0\uc6a9\ub7c9 \ud56d\uc0dd\uc81c \ubcf5\uc6a9 \uc2dc \ucca0\uc800\ud55c \ubcf5\uc6a9\uc744 \ud574\uc57c \ub0b4\uc131\uade0\uc744 \uc5c6\uc568 \uc218 \uc788\uc74c\uc5d0\ub3c4 \ud56d\uc0dd\uc81c\ub97c \uac70\ub974\ub294 \uac78 \uc88b\uc544\ud558\ub294 \ud55c\uad6d\uc778\uc758 \uc2b5\uad00\uc740 \ud56d\uc0dd\uc81c \ub0b4\uc131\uc744 \ub354\uc6b1 \uc545\ud654\uc2dc\ud0b4\uc744 \uace0\ub824\ud574\uc57c \ud55c\ub2e4. \ud56d\uc0dd\uc81c \ubcf5\uc6a9 \uae30\uac04\uc740 \ud1b5\uc0c1 5\uc77c\uc774\ub098 \uc758\uc0ac\uc758 \ud310\ub2e8\uc5d0 \ub530\ub77c \uacb0\uc815\ud558\uba70, 24\uac1c\uc6d4 \uc774\ud558, \uace0\ub9c9 \ucc9c\uacf5\uc774 \uc788\ub294 \uacbd\uc6b0, \uc7ac\ubc1c\uc131 \uae09\uc131 \uc911\uc774\uc5fc\uc778 \uacbd\uc6b0 10\uc77c\uae4c\uc9c0 \uc5f0\uc7a5 \uc0ac\uc6a9\uc744 \uad8c\uc7a5\ud558\uace0 \uc788\ub2e4. \uc7a5\uc5fc, \uad6c\ud1a0 \ub4f1\uc73c\ub85c \uc778\ud574 \uacbd\uad6c \ud56d\uc0dd\uc81c \ud22c\uc5ec\uac00 \ubd88\uac00\ub2a5\ud560 \uacbd\uc6b0 Ceftriaxone (50 mg/kg) \uc8fc\uc0ac\ub97c 1\ud68c 1~3\uc77c \uc0ac\uc6a9 \uc2dc \uacbd\uad6c \ud56d\uc0dd\uc81c\uc640 \ub3d9\uc77c\ud55c \ud6a8\uacfc\ub97c \ubcfc \uc218 \uc788\ub2e4.", "paragraph_answer": "\ud56d\uc0dd\uc81c \ub0b4\uc131\uade0 \uc740 \uc5b4\ub9b0\uc774\uc9d1\uc5d0 \ub2e4\ub2c8\ub294 \uacbd\uc6b0, \ucd5c\uadfc 30\uc77c \uc774\ub0b4 \ud56d\uc0dd\uc81c\ub97c \uc0ac\uc6a9\ud55c \uacbd\uc6b0, 24\uac1c\uc6d4 \uc774\ud558\uc778 \uacbd\uc6b0 \ub0b4\uc131\uade0\uc758 \uac00\ub2a5\uc131\uc774 \ub192\uae30 \ub54c\ubb38\uc5d0 \uc774\ub7f0 \uacbd\uc6b0 \uace0\uc6a9\ub7c9 \uc544\ubaa9\uc2dc\uc2e4\ub9b0 \uc0ac\uc6a9\uc774 \uc88b\ub2e4. \ubd80\ube44\ub3d9\uc5fc \ub4f1\uc5d0\uc11c \ud1b5\uc0c1 \uc6a9\ub7c9\uc778 20~40 mg/kg \uacfc \ube44\uad50\ud558\uba74 \uae09\uc131 \uc911\uc774\uc5fc\uc758 \uace0\uc6a9\ub7c9 \ud56d\uc0dd\uc81c \ubcf5\uc6a9 \uc2dc \ucca0\uc800\ud55c \ubcf5\uc6a9\uc744 \ud574\uc57c \ub0b4\uc131\uade0\uc744 \uc5c6\uc568 \uc218 \uc788\uc74c\uc5d0\ub3c4 \ud56d\uc0dd\uc81c\ub97c \uac70\ub974\ub294 \uac78 \uc88b\uc544\ud558\ub294 \ud55c\uad6d\uc778\uc758 \uc2b5\uad00\uc740 \ud56d\uc0dd\uc81c \ub0b4\uc131\uc744 \ub354\uc6b1 \uc545\ud654\uc2dc\ud0b4\uc744 \uace0\ub824\ud574\uc57c \ud55c\ub2e4. \ud56d\uc0dd\uc81c \ubcf5\uc6a9 \uae30\uac04\uc740 \ud1b5\uc0c1 5\uc77c\uc774\ub098 \uc758\uc0ac\uc758 \ud310\ub2e8\uc5d0 \ub530\ub77c \uacb0\uc815\ud558\uba70, 24\uac1c\uc6d4 \uc774\ud558, \uace0\ub9c9 \ucc9c\uacf5\uc774 \uc788\ub294 \uacbd\uc6b0, \uc7ac\ubc1c\uc131 \uae09\uc131 \uc911\uc774\uc5fc\uc778 \uacbd\uc6b0 10\uc77c\uae4c\uc9c0 \uc5f0\uc7a5 \uc0ac\uc6a9\uc744 \uad8c\uc7a5\ud558\uace0 \uc788\ub2e4. \uc7a5\uc5fc, \uad6c\ud1a0 \ub4f1\uc73c\ub85c \uc778\ud574 \uacbd\uad6c \ud56d\uc0dd\uc81c \ud22c\uc5ec\uac00 \ubd88\uac00\ub2a5\ud560 \uacbd\uc6b0 Ceftriaxone (50 mg/kg) \uc8fc\uc0ac\ub97c 1\ud68c 1~3\uc77c \uc0ac\uc6a9 \uc2dc \uacbd\uad6c \ud56d\uc0dd\uc81c\uc640 \ub3d9\uc77c\ud55c \ud6a8\uacfc\ub97c \ubcfc \uc218 \uc788\ub2e4.", "sentence_answer": "\ud56d\uc0dd\uc81c \ub0b4\uc131\uade0 \uc740 \uc5b4\ub9b0\uc774\uc9d1\uc5d0 \ub2e4\ub2c8\ub294 \uacbd\uc6b0, \ucd5c\uadfc 30\uc77c \uc774\ub0b4 \ud56d\uc0dd\uc81c\ub97c \uc0ac\uc6a9\ud55c \uacbd\uc6b0, 24\uac1c\uc6d4 \uc774\ud558\uc778 \uacbd\uc6b0 \ub0b4\uc131\uade0\uc758 \uac00\ub2a5\uc131\uc774 \ub192\uae30 \ub54c\ubb38\uc5d0 \uc774\ub7f0 \uacbd\uc6b0 \uace0\uc6a9\ub7c9 \uc544\ubaa9\uc2dc\uc2e4\ub9b0 \uc0ac\uc6a9\uc774 \uc88b\ub2e4.", "paragraph_id": "6601704-1-0", "paragraph_question": "question: \uae09\uc131\uc911\uc774\uc5fc \ud658\uc790\uac00 \uace0\uc6a9\ub7c9 \ud56d\uc0dd\uc81c\ub97c \uac70\ub974\uc9c0 \uc54a\uace0 \uc758\uc0ac\uc758 \uad8c\uc720\uc5d0 \ub530\ub77c \ubcf5\uc6a9\ud558\uc600\uc744\ub54c \uc5c6\uc568 \uc218 \uc788\ub294 \uade0\uc740 \ubb34\uc5c7\uc778\uac00?, context: \ud56d\uc0dd\uc81c \ub0b4\uc131\uade0\uc740 \uc5b4\ub9b0\uc774\uc9d1\uc5d0 \ub2e4\ub2c8\ub294 \uacbd\uc6b0, \ucd5c\uadfc 30\uc77c \uc774\ub0b4 \ud56d\uc0dd\uc81c\ub97c \uc0ac\uc6a9\ud55c \uacbd\uc6b0, 24\uac1c\uc6d4 \uc774\ud558\uc778 \uacbd\uc6b0 \ub0b4\uc131\uade0\uc758 \uac00\ub2a5\uc131\uc774 \ub192\uae30 \ub54c\ubb38\uc5d0 \uc774\ub7f0 \uacbd\uc6b0 \uace0\uc6a9\ub7c9 \uc544\ubaa9\uc2dc\uc2e4\ub9b0 \uc0ac\uc6a9\uc774 \uc88b\ub2e4. \ubd80\ube44\ub3d9\uc5fc \ub4f1\uc5d0\uc11c \ud1b5\uc0c1 \uc6a9\ub7c9\uc778 20~40 mg/kg \uacfc \ube44\uad50\ud558\uba74 \uae09\uc131 \uc911\uc774\uc5fc\uc758 \uace0\uc6a9\ub7c9 \ud56d\uc0dd\uc81c \ubcf5\uc6a9 \uc2dc \ucca0\uc800\ud55c \ubcf5\uc6a9\uc744 \ud574\uc57c \ub0b4\uc131\uade0\uc744 \uc5c6\uc568 \uc218 \uc788\uc74c\uc5d0\ub3c4 \ud56d\uc0dd\uc81c\ub97c \uac70\ub974\ub294 \uac78 \uc88b\uc544\ud558\ub294 \ud55c\uad6d\uc778\uc758 \uc2b5\uad00\uc740 \ud56d\uc0dd\uc81c \ub0b4\uc131\uc744 \ub354\uc6b1 \uc545\ud654\uc2dc\ud0b4\uc744 \uace0\ub824\ud574\uc57c \ud55c\ub2e4. \ud56d\uc0dd\uc81c \ubcf5\uc6a9 \uae30\uac04\uc740 \ud1b5\uc0c1 5\uc77c\uc774\ub098 \uc758\uc0ac\uc758 \ud310\ub2e8\uc5d0 \ub530\ub77c \uacb0\uc815\ud558\uba70, 24\uac1c\uc6d4 \uc774\ud558, \uace0\ub9c9 \ucc9c\uacf5\uc774 \uc788\ub294 \uacbd\uc6b0, \uc7ac\ubc1c\uc131 \uae09\uc131 \uc911\uc774\uc5fc\uc778 \uacbd\uc6b0 10\uc77c\uae4c\uc9c0 \uc5f0\uc7a5 \uc0ac\uc6a9\uc744 \uad8c\uc7a5\ud558\uace0 \uc788\ub2e4. \uc7a5\uc5fc, \uad6c\ud1a0 \ub4f1\uc73c\ub85c \uc778\ud574 \uacbd\uad6c \ud56d\uc0dd\uc81c \ud22c\uc5ec\uac00 \ubd88\uac00\ub2a5\ud560 \uacbd\uc6b0 Ceftriaxone (50 mg/kg) \uc8fc\uc0ac\ub97c 1\ud68c 1~3\uc77c \uc0ac\uc6a9 \uc2dc \uacbd\uad6c \ud56d\uc0dd\uc81c\uc640 \ub3d9\uc77c\ud55c \ud6a8\uacfc\ub97c \ubcfc \uc218 \uc788\ub2e4."} {"question": "\ubb38\uc7ac\uc778\uc774 \uc815\uce58\ub85c\ubd80\ud130 \uc644\uc804\ud788 \ub3c5\ub9bd\uc2dc\ud0a4\uaca0\ub2e4\uace0 \ud55c \uac83\uc740?", "paragraph": "\ubb38\uc7ac\uc778\uc740 18\ub300 \ub300\uc120\uc5d0 \ucd9c\uc0ac\ud45c\ub97c \ub358\uc84c\uc744 \ub54c\ub3c4 \uace0\uc704\uacf5\uc9c1\uc790\ube44\ub9ac\uc218\uc0ac\ucc98\uc758 \uc124\uce58\ub97c \uacf5\uc57d\ud588\uc744 \ub9cc\ud07c \uac80\ucc30 \uac1c\ud601\uc5d0 \uc758\uc9c0\ub97c \ubcf4\uc600\ub2e4. 19\ub300 \ub300\uc120\uc5d0\uc11c\ub3c4 \ubb38\uc7ac\uc778\uc744 \ube44\ub86f\ud55c \uc8fc\uc694 \ud6c4\ubcf4\ub4e4\uc774 \uad8c\ub825\uae30\uad00 \uac1c\ud601 \ub300\uc0c1 1\uc21c\uc704\ub85c \uac80\ucc30\uc744 \uaf3d\uc558\uc73c\uba70, \ubb38\uc7ac\uc778\uc740 \uac80\uacbd \uc218\uc0ac\uad8c\uc758 \uc870\uc815\uacfc \uacf5\uc218\ucc98 \uc2e0\uc124\uc744 \uace8\uc790\ub85c \ud55c \uac80\ucc30 \uac1c\ud601 \ubc29\uc548\uc744 \uc81c\uc2dc\ud588\ub2e4. \uac80\ucc30 \uc678\uc5d0\ub3c4 \uad6d\uac00\uc815\ubcf4\uc6d0\uacfc \uacbd\ucc30\uccad \ub4f1 \uad8c\ub825\uae30\uad00\uc5d0 \ub300\ud55c \uac1c\ud601\ub3c4 \ucde8\uc784\uc0ac\ub97c \ud1b5\ud574 \uc608\uace0\ud588\ub2e4. \"\uad8c\ub825\uae30\uad00\uc740 \uc815\uce58\ub85c\ubd80\ud130 \uc644\uc804\ud788 \ub3c5\ub9bd\uc2dc\ud0a4\uaca0\ub2e4\"\uace0 \ud558\uba74\uc11c \ub610 \"\uc5b4\ub5a4 \uae30\uad00\ub3c4 \ubb34\uc18c\ubd88\uc704\uc758 \uad8c\ub825\uc744 \ud589\uc0ac\ud560 \uc218 \uc5c6\ub3c4\ub85d \uacac\uc81c\uc7a5\uce58\ub97c \ub9c8\ub828\ud558\uaca0\ub2e4\"\uace0 \uac15\uc870\ud55c \uac83\uc774\ub2e4. \uac80\ucc30\uc740 \uc778\uc0ac\uc758 \uc911\ub9bd\uc131\uacfc \ub3c5\ub9bd\uc131\uc744 \uac15\ud654\ud558\uace0 \uc678\ubd80\uae30\uac04 \ud30c\uacac\uc744 \uc5b5\uc81c\ud55c\ub2e4\ub294 \uacf5\uc57d\uc744 \ub0b4\ub193\uc558\uc73c\uba70, \uad6d\uc815\uc6d0\uc740 \uad6d\ub0b4 \uc815\ubcf4 \uc218\uc9d1 \uc5c5\ubb34\ub97c \uc804\uba74 \ud3d0\uc9c0\ud558\uc5ec \ub300\ubd81\u00b7\ud14c\ub7ec \uad00\ub828 \uc815\ubcf4 \uc218\uc9d1\uc744 \uc804\ub2f4\ud558\ub294 \uae30\uad00\uc73c\ub85c \ud0c8\ubc14\uafc8\uc2dc\ud0a4\uae30\ub85c \ud588\ub2e4. \ucde8\uc784 \ub2f9\uc77c \uad6d\uc815\uc6d0\uc7a5\uc73c\ub85c \uc9c0\uba85\ub41c \uc11c\ud6c8\ub3c4 \uad6d\uc815\uc6d0\uc758 \uc815\uce58\uac1c\uc785 \uadfc\uc808\uc774 \uc5b4\ub824\uc6b4 \ubb38\uc81c\uc774\uc9c0\ub9cc \ub9c8\uc9c0\ub9c9 \uae30\ud68c\ub77c\uace0 \uc0dd\uac01\ud55c\ub2e4\uba74\uc11c \"\uac74\uac15\ud55c \uad6d\uc815\uc6d0\uc744 \uc704\ud574 \uac00\uc7a5 \ube60\ub974\uace0 \ud6a8\uacfc\uc801\uc73c\ub85c \uc815\uce58\ub85c\ubd80\ud130 (\uad6d\uc815\uc6d0\uc744) \ub5bc\uc5b4\ub193\uc744 \ubc29\ubc95\uc744 \ucc3e\uaca0\ub2e4\"\uace0 \uc758\uc9c0\ub97c \ubc1d\ud614\ub2e4.", "answer": "\uad8c\ub825\uae30\uad00", "sentence": "19\ub300 \ub300\uc120\uc5d0\uc11c\ub3c4 \ubb38\uc7ac\uc778\uc744 \ube44\ub86f\ud55c \uc8fc\uc694 \ud6c4\ubcf4\ub4e4\uc774 \uad8c\ub825\uae30\uad00 \uac1c\ud601 \ub300\uc0c1 1\uc21c\uc704\ub85c \uac80\ucc30\uc744 \uaf3d\uc558\uc73c\uba70, \ubb38\uc7ac\uc778\uc740 \uac80\uacbd \uc218\uc0ac\uad8c\uc758 \uc870\uc815\uacfc \uacf5\uc218\ucc98 \uc2e0\uc124\uc744 \uace8\uc790\ub85c \ud55c \uac80\ucc30 \uac1c\ud601 \ubc29\uc548\uc744 \uc81c\uc2dc\ud588\ub2e4.", "paragraph_sentence": "\ubb38\uc7ac\uc778\uc740 18\ub300 \ub300\uc120\uc5d0 \ucd9c\uc0ac\ud45c\ub97c \ub358\uc84c\uc744 \ub54c\ub3c4 \uace0\uc704\uacf5\uc9c1\uc790\ube44\ub9ac\uc218\uc0ac\ucc98\uc758 \uc124\uce58\ub97c \uacf5\uc57d\ud588\uc744 \ub9cc\ud07c \uac80\ucc30 \uac1c\ud601\uc5d0 \uc758\uc9c0\ub97c \ubcf4\uc600\ub2e4. 19\ub300 \ub300\uc120\uc5d0\uc11c\ub3c4 \ubb38\uc7ac\uc778\uc744 \ube44\ub86f\ud55c \uc8fc\uc694 \ud6c4\ubcf4\ub4e4\uc774 \uad8c\ub825\uae30\uad00 \uac1c\ud601 \ub300\uc0c1 1\uc21c\uc704\ub85c \uac80\ucc30\uc744 \uaf3d\uc558\uc73c\uba70, \ubb38\uc7ac\uc778\uc740 \uac80\uacbd \uc218\uc0ac\uad8c\uc758 \uc870\uc815\uacfc \uacf5\uc218\ucc98 \uc2e0\uc124\uc744 \uace8\uc790\ub85c \ud55c \uac80\ucc30 \uac1c\ud601 \ubc29\uc548\uc744 \uc81c\uc2dc\ud588\ub2e4. \uac80\ucc30 \uc678\uc5d0\ub3c4 \uad6d\uac00\uc815\ubcf4\uc6d0\uacfc \uacbd\ucc30\uccad \ub4f1 \uad8c\ub825\uae30\uad00\uc5d0 \ub300\ud55c \uac1c\ud601\ub3c4 \ucde8\uc784\uc0ac\ub97c \ud1b5\ud574 \uc608\uace0\ud588\ub2e4. \"\uad8c\ub825\uae30\uad00\uc740 \uc815\uce58\ub85c\ubd80\ud130 \uc644\uc804\ud788 \ub3c5\ub9bd\uc2dc\ud0a4\uaca0\ub2e4\"\uace0 \ud558\uba74\uc11c \ub610 \"\uc5b4\ub5a4 \uae30\uad00\ub3c4 \ubb34\uc18c\ubd88\uc704\uc758 \uad8c\ub825\uc744 \ud589\uc0ac\ud560 \uc218 \uc5c6\ub3c4\ub85d \uacac\uc81c\uc7a5\uce58\ub97c \ub9c8\ub828\ud558\uaca0\ub2e4\"\uace0 \uac15\uc870\ud55c \uac83\uc774\ub2e4. \uac80\ucc30\uc740 \uc778\uc0ac\uc758 \uc911\ub9bd\uc131\uacfc \ub3c5\ub9bd\uc131\uc744 \uac15\ud654\ud558\uace0 \uc678\ubd80\uae30\uac04 \ud30c\uacac\uc744 \uc5b5\uc81c\ud55c\ub2e4\ub294 \uacf5\uc57d\uc744 \ub0b4\ub193\uc558\uc73c\uba70, \uad6d\uc815\uc6d0\uc740 \uad6d\ub0b4 \uc815\ubcf4 \uc218\uc9d1 \uc5c5\ubb34\ub97c \uc804\uba74 \ud3d0\uc9c0\ud558\uc5ec \ub300\ubd81\u00b7\ud14c\ub7ec \uad00\ub828 \uc815\ubcf4 \uc218\uc9d1\uc744 \uc804\ub2f4\ud558\ub294 \uae30\uad00\uc73c\ub85c \ud0c8\ubc14\uafc8\uc2dc\ud0a4\uae30\ub85c \ud588\ub2e4. \ucde8\uc784 \ub2f9\uc77c \uad6d\uc815\uc6d0\uc7a5\uc73c\ub85c \uc9c0\uba85\ub41c \uc11c\ud6c8\ub3c4 \uad6d\uc815\uc6d0\uc758 \uc815\uce58\uac1c\uc785 \uadfc\uc808\uc774 \uc5b4\ub824\uc6b4 \ubb38\uc81c\uc774\uc9c0\ub9cc \ub9c8\uc9c0\ub9c9 \uae30\ud68c\ub77c\uace0 \uc0dd\uac01\ud55c\ub2e4\uba74\uc11c \"\uac74\uac15\ud55c \uad6d\uc815\uc6d0\uc744 \uc704\ud574 \uac00\uc7a5 \ube60\ub974\uace0 \ud6a8\uacfc\uc801\uc73c\ub85c \uc815\uce58\ub85c\ubd80\ud130 (\uad6d\uc815\uc6d0\uc744) \ub5bc\uc5b4\ub193\uc744 \ubc29\ubc95\uc744 \ucc3e\uaca0\ub2e4\"\uace0 \uc758\uc9c0\ub97c \ubc1d\ud614\ub2e4.", "paragraph_answer": "\ubb38\uc7ac\uc778\uc740 18\ub300 \ub300\uc120\uc5d0 \ucd9c\uc0ac\ud45c\ub97c \ub358\uc84c\uc744 \ub54c\ub3c4 \uace0\uc704\uacf5\uc9c1\uc790\ube44\ub9ac\uc218\uc0ac\ucc98\uc758 \uc124\uce58\ub97c \uacf5\uc57d\ud588\uc744 \ub9cc\ud07c \uac80\ucc30 \uac1c\ud601\uc5d0 \uc758\uc9c0\ub97c \ubcf4\uc600\ub2e4. 19\ub300 \ub300\uc120\uc5d0\uc11c\ub3c4 \ubb38\uc7ac\uc778\uc744 \ube44\ub86f\ud55c \uc8fc\uc694 \ud6c4\ubcf4\ub4e4\uc774 \uad8c\ub825\uae30\uad00 \uac1c\ud601 \ub300\uc0c1 1\uc21c\uc704\ub85c \uac80\ucc30\uc744 \uaf3d\uc558\uc73c\uba70, \ubb38\uc7ac\uc778\uc740 \uac80\uacbd \uc218\uc0ac\uad8c\uc758 \uc870\uc815\uacfc \uacf5\uc218\ucc98 \uc2e0\uc124\uc744 \uace8\uc790\ub85c \ud55c \uac80\ucc30 \uac1c\ud601 \ubc29\uc548\uc744 \uc81c\uc2dc\ud588\ub2e4. \uac80\ucc30 \uc678\uc5d0\ub3c4 \uad6d\uac00\uc815\ubcf4\uc6d0\uacfc \uacbd\ucc30\uccad \ub4f1 \uad8c\ub825\uae30\uad00\uc5d0 \ub300\ud55c \uac1c\ud601\ub3c4 \ucde8\uc784\uc0ac\ub97c \ud1b5\ud574 \uc608\uace0\ud588\ub2e4. \"\uad8c\ub825\uae30\uad00\uc740 \uc815\uce58\ub85c\ubd80\ud130 \uc644\uc804\ud788 \ub3c5\ub9bd\uc2dc\ud0a4\uaca0\ub2e4\"\uace0 \ud558\uba74\uc11c \ub610 \"\uc5b4\ub5a4 \uae30\uad00\ub3c4 \ubb34\uc18c\ubd88\uc704\uc758 \uad8c\ub825\uc744 \ud589\uc0ac\ud560 \uc218 \uc5c6\ub3c4\ub85d \uacac\uc81c\uc7a5\uce58\ub97c \ub9c8\ub828\ud558\uaca0\ub2e4\"\uace0 \uac15\uc870\ud55c \uac83\uc774\ub2e4. \uac80\ucc30\uc740 \uc778\uc0ac\uc758 \uc911\ub9bd\uc131\uacfc \ub3c5\ub9bd\uc131\uc744 \uac15\ud654\ud558\uace0 \uc678\ubd80\uae30\uac04 \ud30c\uacac\uc744 \uc5b5\uc81c\ud55c\ub2e4\ub294 \uacf5\uc57d\uc744 \ub0b4\ub193\uc558\uc73c\uba70, \uad6d\uc815\uc6d0\uc740 \uad6d\ub0b4 \uc815\ubcf4 \uc218\uc9d1 \uc5c5\ubb34\ub97c \uc804\uba74 \ud3d0\uc9c0\ud558\uc5ec \ub300\ubd81\u00b7\ud14c\ub7ec \uad00\ub828 \uc815\ubcf4 \uc218\uc9d1\uc744 \uc804\ub2f4\ud558\ub294 \uae30\uad00\uc73c\ub85c \ud0c8\ubc14\uafc8\uc2dc\ud0a4\uae30\ub85c \ud588\ub2e4. \ucde8\uc784 \ub2f9\uc77c \uad6d\uc815\uc6d0\uc7a5\uc73c\ub85c \uc9c0\uba85\ub41c \uc11c\ud6c8\ub3c4 \uad6d\uc815\uc6d0\uc758 \uc815\uce58\uac1c\uc785 \uadfc\uc808\uc774 \uc5b4\ub824\uc6b4 \ubb38\uc81c\uc774\uc9c0\ub9cc \ub9c8\uc9c0\ub9c9 \uae30\ud68c\ub77c\uace0 \uc0dd\uac01\ud55c\ub2e4\uba74\uc11c \"\uac74\uac15\ud55c \uad6d\uc815\uc6d0\uc744 \uc704\ud574 \uac00\uc7a5 \ube60\ub974\uace0 \ud6a8\uacfc\uc801\uc73c\ub85c \uc815\uce58\ub85c\ubd80\ud130 (\uad6d\uc815\uc6d0\uc744) \ub5bc\uc5b4\ub193\uc744 \ubc29\ubc95\uc744 \ucc3e\uaca0\ub2e4\"\uace0 \uc758\uc9c0\ub97c \ubc1d\ud614\ub2e4.", "sentence_answer": "19\ub300 \ub300\uc120\uc5d0\uc11c\ub3c4 \ubb38\uc7ac\uc778\uc744 \ube44\ub86f\ud55c \uc8fc\uc694 \ud6c4\ubcf4\ub4e4\uc774 \uad8c\ub825\uae30\uad00 \uac1c\ud601 \ub300\uc0c1 1\uc21c\uc704\ub85c \uac80\ucc30\uc744 \uaf3d\uc558\uc73c\uba70, \ubb38\uc7ac\uc778\uc740 \uac80\uacbd \uc218\uc0ac\uad8c\uc758 \uc870\uc815\uacfc \uacf5\uc218\ucc98 \uc2e0\uc124\uc744 \uace8\uc790\ub85c \ud55c \uac80\ucc30 \uac1c\ud601 \ubc29\uc548\uc744 \uc81c\uc2dc\ud588\ub2e4.", "paragraph_id": "6580792-66-2", "paragraph_question": "question: \ubb38\uc7ac\uc778\uc774 \uc815\uce58\ub85c\ubd80\ud130 \uc644\uc804\ud788 \ub3c5\ub9bd\uc2dc\ud0a4\uaca0\ub2e4\uace0 \ud55c \uac83\uc740?, context: \ubb38\uc7ac\uc778\uc740 18\ub300 \ub300\uc120\uc5d0 \ucd9c\uc0ac\ud45c\ub97c \ub358\uc84c\uc744 \ub54c\ub3c4 \uace0\uc704\uacf5\uc9c1\uc790\ube44\ub9ac\uc218\uc0ac\ucc98\uc758 \uc124\uce58\ub97c \uacf5\uc57d\ud588\uc744 \ub9cc\ud07c \uac80\ucc30 \uac1c\ud601\uc5d0 \uc758\uc9c0\ub97c \ubcf4\uc600\ub2e4. 19\ub300 \ub300\uc120\uc5d0\uc11c\ub3c4 \ubb38\uc7ac\uc778\uc744 \ube44\ub86f\ud55c \uc8fc\uc694 \ud6c4\ubcf4\ub4e4\uc774 \uad8c\ub825\uae30\uad00 \uac1c\ud601 \ub300\uc0c1 1\uc21c\uc704\ub85c \uac80\ucc30\uc744 \uaf3d\uc558\uc73c\uba70, \ubb38\uc7ac\uc778\uc740 \uac80\uacbd \uc218\uc0ac\uad8c\uc758 \uc870\uc815\uacfc \uacf5\uc218\ucc98 \uc2e0\uc124\uc744 \uace8\uc790\ub85c \ud55c \uac80\ucc30 \uac1c\ud601 \ubc29\uc548\uc744 \uc81c\uc2dc\ud588\ub2e4. \uac80\ucc30 \uc678\uc5d0\ub3c4 \uad6d\uac00\uc815\ubcf4\uc6d0\uacfc \uacbd\ucc30\uccad \ub4f1 \uad8c\ub825\uae30\uad00\uc5d0 \ub300\ud55c \uac1c\ud601\ub3c4 \ucde8\uc784\uc0ac\ub97c \ud1b5\ud574 \uc608\uace0\ud588\ub2e4. \"\uad8c\ub825\uae30\uad00\uc740 \uc815\uce58\ub85c\ubd80\ud130 \uc644\uc804\ud788 \ub3c5\ub9bd\uc2dc\ud0a4\uaca0\ub2e4\"\uace0 \ud558\uba74\uc11c \ub610 \"\uc5b4\ub5a4 \uae30\uad00\ub3c4 \ubb34\uc18c\ubd88\uc704\uc758 \uad8c\ub825\uc744 \ud589\uc0ac\ud560 \uc218 \uc5c6\ub3c4\ub85d \uacac\uc81c\uc7a5\uce58\ub97c \ub9c8\ub828\ud558\uaca0\ub2e4\"\uace0 \uac15\uc870\ud55c \uac83\uc774\ub2e4. \uac80\ucc30\uc740 \uc778\uc0ac\uc758 \uc911\ub9bd\uc131\uacfc \ub3c5\ub9bd\uc131\uc744 \uac15\ud654\ud558\uace0 \uc678\ubd80\uae30\uac04 \ud30c\uacac\uc744 \uc5b5\uc81c\ud55c\ub2e4\ub294 \uacf5\uc57d\uc744 \ub0b4\ub193\uc558\uc73c\uba70, \uad6d\uc815\uc6d0\uc740 \uad6d\ub0b4 \uc815\ubcf4 \uc218\uc9d1 \uc5c5\ubb34\ub97c \uc804\uba74 \ud3d0\uc9c0\ud558\uc5ec \ub300\ubd81\u00b7\ud14c\ub7ec \uad00\ub828 \uc815\ubcf4 \uc218\uc9d1\uc744 \uc804\ub2f4\ud558\ub294 \uae30\uad00\uc73c\ub85c \ud0c8\ubc14\uafc8\uc2dc\ud0a4\uae30\ub85c \ud588\ub2e4. \ucde8\uc784 \ub2f9\uc77c \uad6d\uc815\uc6d0\uc7a5\uc73c\ub85c \uc9c0\uba85\ub41c \uc11c\ud6c8\ub3c4 \uad6d\uc815\uc6d0\uc758 \uc815\uce58\uac1c\uc785 \uadfc\uc808\uc774 \uc5b4\ub824\uc6b4 \ubb38\uc81c\uc774\uc9c0\ub9cc \ub9c8\uc9c0\ub9c9 \uae30\ud68c\ub77c\uace0 \uc0dd\uac01\ud55c\ub2e4\uba74\uc11c \"\uac74\uac15\ud55c \uad6d\uc815\uc6d0\uc744 \uc704\ud574 \uac00\uc7a5 \ube60\ub974\uace0 \ud6a8\uacfc\uc801\uc73c\ub85c \uc815\uce58\ub85c\ubd80\ud130 (\uad6d\uc815\uc6d0\uc744) \ub5bc\uc5b4\ub193\uc744 \ubc29\ubc95\uc744 \ucc3e\uaca0\ub2e4\"\uace0 \uc758\uc9c0\ub97c \ubc1d\ud614\ub2e4."} {"question": "\ud3f4\ub780\ub4dc-\ub9ac\ud22c\uc544\ub2c8\uc544 \uc5f0\ud569\uccb4\uc81c\uc640 \uc5f0\ubc29\uc815\ubd80\ub97c \ubd95\uad34\uc2dc\ud0a4\ub824\uace0 \uc2a4\uc6e8\ub374\uc758 \uc5b4\ub290 \uc655\uacfc \uad50\uc12d\uc744 \uac1c\uc2dc\ud558\uc600\ub098?", "paragraph": "\ub450\uba85\uc758 \ud3f4\ub780\ub4dc-\ub9ac\ud22c\uc544\ub2c8\uc544 \uadc0\uc871 \uc57c\ub204\uc2dc \ub77c\uc9c0\ube44\uc6b0(Janusz Radziwi\u0142\u0142) \uacf5\uacfc \ubcf4\uad6c\uc2a4\uc640\ud504 \ub77c\uc9c0\ube44\uc6b0(Bogus\u0142aw Radziwi\u0142\u0142) \uacf5\uc740 \ud63c\ub780\ud55c \uc5f0\ubc29\uc5d0 \ub0b4\ubd80\ubd84\uc7c1\uc744 \uc77c\uc73c\ucf30\uace0, \ud3f4\ub780\ub4dc-\ub9ac\ud22c\uc544\ub2c8\uc544 \uc5f0\ud569(Polish-Lithuanian union)\uccb4\uc81c\uc640 \uc5f0\ubc29\uc815\ubd80\ub97c \ubd95\uad34\uc2dc\ud0ac \ubaa9\uc801\uc73c\ub85c \uc2a4\uc6e8\ub374 \uc655 \uce7c 10\uc138 \uad6c\uc2a4\ud0c0\ube0c\uc640 \uad50\uc12d\uc744 \uac1c\uc2dc\ud588\ub2e4. \uadf8\ub4e4\uc740 \ucf00\ub2e4\uc774\ub124\uc774(K\u0117dainiai)\uc5d0\uc11c \ud569\ubcd1\uc5d0 \uad00\ud55c \uc870\uc57d\uc5d0 \uc870\uc778\ud588\uc73c\ub098, \uc774 \uc870\uc57d\uc5d0\ub294 \ub9ac\ud22c\uc544\ub2c8\uc544 \ub300\uacf5\uad6d\uc5d0\uc11c 2\uac1c\uc758 \uacf5\uad6d\uc744 \ubd84\ud560 \ub3c5\ub9bd\uc2dc\ucf1c \ub77c\uc9c0\ube44\uc6b0 \uac00\ubb38\uc758 \uacf5\ub4e4\uc5d0\uac8c \uc591\ub3c4\ud558\uace0, \ub300\uacf5\uad6d\ub839\uc758 \ub0a8\uc740 \uc9c0\uc5ed\uc740 \uc2a4\uc6e8\ub374\uc758 \uc18d\uad6d(vassal)\uc774 \ub41c\ub2e4\ub294 \uac83\uc774 \uc57d\uc18d\ub418\uc5c8\ub2e4(\ucf00\ub2e4\uc774\ub124\uc774 \ud569\ubcd1(Union of K\u0117dainiai).", "answer": "\uce7c 10\uc138 \uad6c\uc2a4\ud0c0\ube0c", "sentence": "\ub450\uba85\uc758 \ud3f4\ub780\ub4dc-\ub9ac\ud22c\uc544\ub2c8\uc544 \uadc0\uc871 \uc57c\ub204\uc2dc \ub77c\uc9c0\ube44\uc6b0(Janusz Radziwi\u0142\u0142) \uacf5\uacfc \ubcf4\uad6c\uc2a4\uc640\ud504 \ub77c\uc9c0\ube44\uc6b0(Bogus\u0142aw Radziwi\u0142\u0142) \uacf5\uc740 \ud63c\ub780\ud55c \uc5f0\ubc29\uc5d0 \ub0b4\ubd80\ubd84\uc7c1\uc744 \uc77c\uc73c\ucf30\uace0, \ud3f4\ub780\ub4dc-\ub9ac\ud22c\uc544\ub2c8\uc544 \uc5f0\ud569(Polish-Lithuanian union)\uccb4\uc81c\uc640 \uc5f0\ubc29\uc815\ubd80\ub97c \ubd95\uad34\uc2dc\ud0ac \ubaa9\uc801\uc73c\ub85c \uc2a4\uc6e8\ub374 \uc655 \uce7c 10\uc138 \uad6c\uc2a4\ud0c0\ube0c \uc640", "paragraph_sentence": " \ub450\uba85\uc758 \ud3f4\ub780\ub4dc-\ub9ac\ud22c\uc544\ub2c8\uc544 \uadc0\uc871 \uc57c\ub204\uc2dc \ub77c\uc9c0\ube44\uc6b0(Janusz Radziwi\u0142\u0142) \uacf5\uacfc \ubcf4\uad6c\uc2a4\uc640\ud504 \ub77c\uc9c0\ube44\uc6b0(Bogus\u0142aw Radziwi\u0142\u0142) \uacf5\uc740 \ud63c\ub780\ud55c \uc5f0\ubc29\uc5d0 \ub0b4\ubd80\ubd84\uc7c1\uc744 \uc77c\uc73c\ucf30\uace0, \ud3f4\ub780\ub4dc-\ub9ac\ud22c\uc544\ub2c8\uc544 \uc5f0\ud569(Polish-Lithuanian union)\uccb4\uc81c\uc640 \uc5f0\ubc29\uc815\ubd80\ub97c \ubd95\uad34\uc2dc\ud0ac \ubaa9\uc801\uc73c\ub85c \uc2a4\uc6e8\ub374 \uc655 \uce7c 10\uc138 \uad6c\uc2a4\ud0c0\ube0c \uc640 \uad50\uc12d\uc744 \uac1c\uc2dc\ud588\ub2e4. \uadf8\ub4e4\uc740 \ucf00\ub2e4\uc774\ub124\uc774(K\u0117dainiai)\uc5d0\uc11c \ud569\ubcd1\uc5d0 \uad00\ud55c \uc870\uc57d\uc5d0 \uc870\uc778\ud588\uc73c\ub098, \uc774 \uc870\uc57d\uc5d0\ub294 \ub9ac\ud22c\uc544\ub2c8\uc544 \ub300\uacf5\uad6d\uc5d0\uc11c 2\uac1c\uc758 \uacf5\uad6d\uc744 \ubd84\ud560 \ub3c5\ub9bd\uc2dc\ucf1c \ub77c\uc9c0\ube44\uc6b0 \uac00\ubb38\uc758 \uacf5\ub4e4\uc5d0\uac8c \uc591\ub3c4\ud558\uace0, \ub300\uacf5\uad6d\ub839\uc758 \ub0a8\uc740 \uc9c0\uc5ed\uc740 \uc2a4\uc6e8\ub374\uc758 \uc18d\uad6d(vassal)\uc774 \ub41c\ub2e4\ub294 \uac83\uc774 \uc57d\uc18d\ub418\uc5c8\ub2e4(\ucf00\ub2e4\uc774\ub124\uc774 \ud569\ubcd1(Union of K\u0117dainiai).", "paragraph_answer": "\ub450\uba85\uc758 \ud3f4\ub780\ub4dc-\ub9ac\ud22c\uc544\ub2c8\uc544 \uadc0\uc871 \uc57c\ub204\uc2dc \ub77c\uc9c0\ube44\uc6b0(Janusz Radziwi\u0142\u0142) \uacf5\uacfc \ubcf4\uad6c\uc2a4\uc640\ud504 \ub77c\uc9c0\ube44\uc6b0(Bogus\u0142aw Radziwi\u0142\u0142) \uacf5\uc740 \ud63c\ub780\ud55c \uc5f0\ubc29\uc5d0 \ub0b4\ubd80\ubd84\uc7c1\uc744 \uc77c\uc73c\ucf30\uace0, \ud3f4\ub780\ub4dc-\ub9ac\ud22c\uc544\ub2c8\uc544 \uc5f0\ud569(Polish-Lithuanian union)\uccb4\uc81c\uc640 \uc5f0\ubc29\uc815\ubd80\ub97c \ubd95\uad34\uc2dc\ud0ac \ubaa9\uc801\uc73c\ub85c \uc2a4\uc6e8\ub374 \uc655 \uce7c 10\uc138 \uad6c\uc2a4\ud0c0\ube0c \uc640 \uad50\uc12d\uc744 \uac1c\uc2dc\ud588\ub2e4. \uadf8\ub4e4\uc740 \ucf00\ub2e4\uc774\ub124\uc774(K\u0117dainiai)\uc5d0\uc11c \ud569\ubcd1\uc5d0 \uad00\ud55c \uc870\uc57d\uc5d0 \uc870\uc778\ud588\uc73c\ub098, \uc774 \uc870\uc57d\uc5d0\ub294 \ub9ac\ud22c\uc544\ub2c8\uc544 \ub300\uacf5\uad6d\uc5d0\uc11c 2\uac1c\uc758 \uacf5\uad6d\uc744 \ubd84\ud560 \ub3c5\ub9bd\uc2dc\ucf1c \ub77c\uc9c0\ube44\uc6b0 \uac00\ubb38\uc758 \uacf5\ub4e4\uc5d0\uac8c \uc591\ub3c4\ud558\uace0, \ub300\uacf5\uad6d\ub839\uc758 \ub0a8\uc740 \uc9c0\uc5ed\uc740 \uc2a4\uc6e8\ub374\uc758 \uc18d\uad6d(vassal)\uc774 \ub41c\ub2e4\ub294 \uac83\uc774 \uc57d\uc18d\ub418\uc5c8\ub2e4(\ucf00\ub2e4\uc774\ub124\uc774 \ud569\ubcd1(Union of K\u0117dainiai).", "sentence_answer": "\ub450\uba85\uc758 \ud3f4\ub780\ub4dc-\ub9ac\ud22c\uc544\ub2c8\uc544 \uadc0\uc871 \uc57c\ub204\uc2dc \ub77c\uc9c0\ube44\uc6b0(Janusz Radziwi\u0142\u0142) \uacf5\uacfc \ubcf4\uad6c\uc2a4\uc640\ud504 \ub77c\uc9c0\ube44\uc6b0(Bogus\u0142aw Radziwi\u0142\u0142) \uacf5\uc740 \ud63c\ub780\ud55c \uc5f0\ubc29\uc5d0 \ub0b4\ubd80\ubd84\uc7c1\uc744 \uc77c\uc73c\ucf30\uace0, \ud3f4\ub780\ub4dc-\ub9ac\ud22c\uc544\ub2c8\uc544 \uc5f0\ud569(Polish-Lithuanian union)\uccb4\uc81c\uc640 \uc5f0\ubc29\uc815\ubd80\ub97c \ubd95\uad34\uc2dc\ud0ac \ubaa9\uc801\uc73c\ub85c \uc2a4\uc6e8\ub374 \uc655 \uce7c 10\uc138 \uad6c\uc2a4\ud0c0\ube0c \uc640", "paragraph_id": "6577960-2-0", "paragraph_question": "question: \ud3f4\ub780\ub4dc-\ub9ac\ud22c\uc544\ub2c8\uc544 \uc5f0\ud569\uccb4\uc81c\uc640 \uc5f0\ubc29\uc815\ubd80\ub97c \ubd95\uad34\uc2dc\ud0a4\ub824\uace0 \uc2a4\uc6e8\ub374\uc758 \uc5b4\ub290 \uc655\uacfc \uad50\uc12d\uc744 \uac1c\uc2dc\ud558\uc600\ub098?, context: \ub450\uba85\uc758 \ud3f4\ub780\ub4dc-\ub9ac\ud22c\uc544\ub2c8\uc544 \uadc0\uc871 \uc57c\ub204\uc2dc \ub77c\uc9c0\ube44\uc6b0(Janusz Radziwi\u0142\u0142) \uacf5\uacfc \ubcf4\uad6c\uc2a4\uc640\ud504 \ub77c\uc9c0\ube44\uc6b0(Bogus\u0142aw Radziwi\u0142\u0142) \uacf5\uc740 \ud63c\ub780\ud55c \uc5f0\ubc29\uc5d0 \ub0b4\ubd80\ubd84\uc7c1\uc744 \uc77c\uc73c\ucf30\uace0, \ud3f4\ub780\ub4dc-\ub9ac\ud22c\uc544\ub2c8\uc544 \uc5f0\ud569(Polish-Lithuanian union)\uccb4\uc81c\uc640 \uc5f0\ubc29\uc815\ubd80\ub97c \ubd95\uad34\uc2dc\ud0ac \ubaa9\uc801\uc73c\ub85c \uc2a4\uc6e8\ub374 \uc655 \uce7c 10\uc138 \uad6c\uc2a4\ud0c0\ube0c\uc640 \uad50\uc12d\uc744 \uac1c\uc2dc\ud588\ub2e4. \uadf8\ub4e4\uc740 \ucf00\ub2e4\uc774\ub124\uc774(K\u0117dainiai)\uc5d0\uc11c \ud569\ubcd1\uc5d0 \uad00\ud55c \uc870\uc57d\uc5d0 \uc870\uc778\ud588\uc73c\ub098, \uc774 \uc870\uc57d\uc5d0\ub294 \ub9ac\ud22c\uc544\ub2c8\uc544 \ub300\uacf5\uad6d\uc5d0\uc11c 2\uac1c\uc758 \uacf5\uad6d\uc744 \ubd84\ud560 \ub3c5\ub9bd\uc2dc\ucf1c \ub77c\uc9c0\ube44\uc6b0 \uac00\ubb38\uc758 \uacf5\ub4e4\uc5d0\uac8c \uc591\ub3c4\ud558\uace0, \ub300\uacf5\uad6d\ub839\uc758 \ub0a8\uc740 \uc9c0\uc5ed\uc740 \uc2a4\uc6e8\ub374\uc758 \uc18d\uad6d(vassal)\uc774 \ub41c\ub2e4\ub294 \uac83\uc774 \uc57d\uc18d\ub418\uc5c8\ub2e4(\ucf00\ub2e4\uc774\ub124\uc774 \ud569\ubcd1(Union of K\u0117dainiai)."} {"question": "\ub85c\ud2b8\ub108\ub97c \uc0e4\ud06c\ubcf4\uc774\uc640 \ub77c\ubc14\uac78\uc758 \ubaa8\ud5d8\uc73c\ub85c \uc81c \uba87 \ud68c \uc80a\uc740\uc608\uc220\uac00\uc0c1 \ud6c4\ubcf4\uc5d0 \uc62c\ub790\ub294\uac00?", "paragraph": "\ub85c\ud2b8\ub108\ub294 \ub85c\uc2a4\uc564\uc824\ub808\uc2a4\ub85c \uc774\ub3d9\ud55c \ud6c4 \uba87\uac1c\uc6d4 \ub3d9\uc548 \ud154\ub808\ube44\uc804 \ubc0f \uc601\ud654\uc5d0\uc11c \uc791\uc740 \uc5ed\ud560\uc744 \ub9e1\uace0 \uad11\uace0\uc5d0\ub3c4 \ucd9c\uc5f0\ud558\uc600\ub2e4. 2001\ub144, \ub85c\ud2b8\ub108\ub294 \ud154\ub808\ube44\uc804 \uc601\ud654 \u300a\ud504\ub9ac\ub9e8\u300b\uc5d0 \ucd9c\uc5f0\ud558\uc600\uace0, \u300a\ub7ec\uadf8\ub798\uce20 3: \ubb34\uc778\ub3c4 \ub300\ubaa8\ud5d8\u300b\uc758 \uad11\uace0\uc5d0\uc11c \ubaa9\uc18c\ub9ac\ub85c \ucd9c\uc5f0\ud558\uc600\ub2e4. \ub610\ud55c \u300a\ubc84\ub2c8 \ub9e5 \uc1fc\u300b, \u300a\ub9c8\uc774 \uc640\uc774\ud504 \uc564 \ud0a4\uc988\u300b, \u300a\uc378\uba38\ub79c\ub4dc\u300b\uc5d0\uc11c \uc791\uc740 \uc5ed\ud560\ub85c \ucd9c\uc5f0\ud558\uc600\ub2e4. \u300aWhat's New, Scooby-Doo?\u300b\uc640 \u300a\ub300\ub2c8 \ud32c\ud140\u300b\uc5d0\uc11c\ub294 \ubaa9\uc18c\ub9ac\ub85c\ub3c4 \ucd9c\uc5f0\ud558\uc600\ub2e4. 2005\ub144\uc5d0\ub294 \ucf54\ubbf8\ub514 \uc601\ud654 \u300a\uc5f4\ub450\uba85\uc758 \uc6ec\uc218\ub4e4 2\u300b\uc5d0\ub3c4 \uc791\uc740 \uc5ed\ud560\ub85c \ucd9c\uc5f0\ud558\uc600\ub2e4. \uac19\uc740 \ud574\uc5d0 \uac19\uc740 \uace0\ub4f1\ud559\uad50\ub97c \uc7ac\ud559 \uc911\uc778 \ud14c\uc77c\ub7ec \ub458\ub9ac\uc640 \ud568\uaed8 \u300a\uc0e4\ud06c\ubcf4\uc774\uc640 \ub77c\ubc14\uac78\uc758 \ubaa8\ud5d8 3-D\u300b\uc5d0\uc11c \ucc98\uc74c\uc73c\ub85c \uc8fc\uc5f0\uc744 \ub9e1\uc558\ub2e4. \ub85c\ud2b8\ub108\ub294 \uc774 \uc601\ud654\uc5d0\uc11c \ubaa8\ub4e0 \ubb34\uc220 \uc7a5\uba74\uc744 \uc9c1\uc811 \uc5f0\uae30 \ud558\uc600\uc73c\uba70, \uc774 \uc601\ud654\ub97c \ucd2c\uc601\ud558\uae30 \uc704\ud574 \uc624\uc2a4\ud2f4\uc5d0 3\uac1c\uc6d4\ub3d9\uc548 \uba38\ubb3c\uc5c8\ub2e4. \uc601\ud654\ub294 \ubd80\uc815\uc801\uc778 \ud3c9\uc744 \ubc1b\uc558\uace0, \uad6d\uc81c\uc801\uc73c\ub85c\ub294 \ud06c\uac8c \ud765\ud589\ud558\uc9c0\ub294 \uc54a\uc558\ub2e4. \ub85c\ud2b8\ub108\ub294 \uc774 \uc601\ud654\ub85c 27\ud68c \uc80a\uc740 \uc608\uc220\uac00\uc0c1\uc5d0 \ud6c4\ubcf4\ub85c \uc62c\ub790\uc73c\ub098 \uc0c1\uc744 \ubc1b\uc9c0\ub294 \ubabb\ud588\ub2e4. \ub85c\ud2b8\ub108\ub294 \uc774 \uc601\ud654\uc5d0\uc11c \uc0ac\uc6b4\ub4dc\ud2b8\ub799 Dream Dream Dream Dream\uc744 \uc9c1\uc811 \ubd88\ub800\uc73c\uba70, \uc0ac\uc6b4\ub4dc\ud2b8\ub799 \uc74c\ubc18\uc5d0\ub3c4 \uc218\ub85d\uc774 \ub418\uc5c8\ub2e4. \uba87\uac1c\uc6d4 \ud6c4, \ub85c\ud2b8\ub108\ub294 \u300a\uc5f4\ub450\uba85\uc758 \uc6ec\uc218\ub4e4 2\u300b\uc5d0 \ucd9c\uc5f0\ud558\uc600\uace0, \uc601\ud654\ub294 \ub85c\ud2bc \ud1a0\ub9c8\ud1a0\uc5d0\uc11c \"2000\ub144\ub300 \ucd5c\uc545\uc758 \uc601\ud654\"\ub85c \ud3c9\uac00\ubc1b\uc558\ub2e4. \uce90\ub098\ub2e4\uc5d0\uc11c \ucd2c\uc601\uc744 \ub9c8\uce58\uace0 \ub3cc\uc544\uc628 \ud6c4\uc5d0\ub294 \uc0e4\ud06c\ubcf4\uc774\uc640 \ub77c\ubc14\uac78\uc5d0 \uc758\ud55c \uba85\uc131\uc744 \uc2e4\uac10\ud558\uc600\ub2e4. 2008\ub144 \ud154\ub808\ube44\uc804 \uc1fc \u300aLove, Inc.\u300b\uc640 \ud154\ub808\ube44\uc804 \uc2a4\ud398\uc15c \u300aHe's a Bully, Charlie Brown\u300b\uc5d0 \ucd9c\uc5f0\ud558\uc600\ub2e4. \uac19\uc740 \ud574 NBC\uc758 \ub4dc\ub77c\ub9c8 \u300a\ub9c8\uc774 \uc628 \uc6cc\uc2a4\ud2b8 \uc5d0\ub108\ubbf8\u300b\uc5d0 \ud06c\ub9ac\uc2a4\ucc9c \uc2ac\ub808\uc774\ud130\uac00 \ub9e1\uc740 \ud5e8\ub9ac \uc2a4\ud30c\uc774\ube44\uc758 \uc544\ub4e4 \uc7ad \uc2a4\ud30c\uc774\ube44 \uc5ed\ud560\ub85c \ucd9c\uc5f0\ud558\uc600\ub2e4. \ub864\ub9c1 \uc2a4\ud1a4\uc740 \ud14c\uc77c\ub7ec\uc758 \ucd08\uae30 \uc5ed\ud560\ub4e4\uc740 \"\ub85c\ud2b8\ub108\ub97c \uc778\uae30\uc788\ub294 \uc544\uc774, \uc6b4\ub3d9\uc744 \uc88b\uc544\ud558\ub294 \uc544\uc774 \ub610\ub294 \uace8\ubaa9 \ub300\uc7a5\"\uc73c\ub85c \ub9cc\ub4e4\uc5c8\ub2e4\uace0 \ud55c\ub2e4.", "answer": "27\ud68c", "sentence": "\ub85c\ud2b8\ub108\ub294 \uc774 \uc601\ud654\ub85c 27\ud68c \uc80a\uc740 \uc608\uc220\uac00\uc0c1\uc5d0 \ud6c4\ubcf4\ub85c \uc62c\ub790\uc73c\ub098 \uc0c1\uc744 \ubc1b\uc9c0\ub294 \ubabb\ud588\ub2e4.", "paragraph_sentence": "\ub85c\ud2b8\ub108\ub294 \ub85c\uc2a4\uc564\uc824\ub808\uc2a4\ub85c \uc774\ub3d9\ud55c \ud6c4 \uba87\uac1c\uc6d4 \ub3d9\uc548 \ud154\ub808\ube44\uc804 \ubc0f \uc601\ud654\uc5d0\uc11c \uc791\uc740 \uc5ed\ud560\uc744 \ub9e1\uace0 \uad11\uace0\uc5d0\ub3c4 \ucd9c\uc5f0\ud558\uc600\ub2e4. 2001\ub144, \ub85c\ud2b8\ub108\ub294 \ud154\ub808\ube44\uc804 \uc601\ud654 \u300a\ud504\ub9ac\ub9e8\u300b\uc5d0 \ucd9c\uc5f0\ud558\uc600\uace0, \u300a\ub7ec\uadf8\ub798\uce20 3: \ubb34\uc778\ub3c4 \ub300\ubaa8\ud5d8\u300b\uc758 \uad11\uace0\uc5d0\uc11c \ubaa9\uc18c\ub9ac\ub85c \ucd9c\uc5f0\ud558\uc600\ub2e4. \ub610\ud55c \u300a\ubc84\ub2c8 \ub9e5 \uc1fc\u300b, \u300a\ub9c8\uc774 \uc640\uc774\ud504 \uc564 \ud0a4\uc988\u300b, \u300a\uc378\uba38\ub79c\ub4dc\u300b\uc5d0\uc11c \uc791\uc740 \uc5ed\ud560\ub85c \ucd9c\uc5f0\ud558\uc600\ub2e4. \u300aWhat's New, Scooby-Doo?\u300b\uc640 \u300a\ub300\ub2c8 \ud32c\ud140\u300b\uc5d0\uc11c\ub294 \ubaa9\uc18c\ub9ac\ub85c\ub3c4 \ucd9c\uc5f0\ud558\uc600\ub2e4. 2005\ub144\uc5d0\ub294 \ucf54\ubbf8\ub514 \uc601\ud654 \u300a\uc5f4\ub450\uba85\uc758 \uc6ec\uc218\ub4e4 2\u300b\uc5d0\ub3c4 \uc791\uc740 \uc5ed\ud560\ub85c \ucd9c\uc5f0\ud558\uc600\ub2e4. \uac19\uc740 \ud574\uc5d0 \uac19\uc740 \uace0\ub4f1\ud559\uad50\ub97c \uc7ac\ud559 \uc911\uc778 \ud14c\uc77c\ub7ec \ub458\ub9ac\uc640 \ud568\uaed8 \u300a\uc0e4\ud06c\ubcf4\uc774\uc640 \ub77c\ubc14\uac78\uc758 \ubaa8\ud5d8 3-D\u300b\uc5d0\uc11c \ucc98\uc74c\uc73c\ub85c \uc8fc\uc5f0\uc744 \ub9e1\uc558\ub2e4. \ub85c\ud2b8\ub108\ub294 \uc774 \uc601\ud654\uc5d0\uc11c \ubaa8\ub4e0 \ubb34\uc220 \uc7a5\uba74\uc744 \uc9c1\uc811 \uc5f0\uae30 \ud558\uc600\uc73c\uba70, \uc774 \uc601\ud654\ub97c \ucd2c\uc601\ud558\uae30 \uc704\ud574 \uc624\uc2a4\ud2f4\uc5d0 3\uac1c\uc6d4\ub3d9\uc548 \uba38\ubb3c\uc5c8\ub2e4. \uc601\ud654\ub294 \ubd80\uc815\uc801\uc778 \ud3c9\uc744 \ubc1b\uc558\uace0, \uad6d\uc81c\uc801\uc73c\ub85c\ub294 \ud06c\uac8c \ud765\ud589\ud558\uc9c0\ub294 \uc54a\uc558\ub2e4. \ub85c\ud2b8\ub108\ub294 \uc774 \uc601\ud654\ub85c 27\ud68c \uc80a\uc740 \uc608\uc220\uac00\uc0c1\uc5d0 \ud6c4\ubcf4\ub85c \uc62c\ub790\uc73c\ub098 \uc0c1\uc744 \ubc1b\uc9c0\ub294 \ubabb\ud588\ub2e4. \ub85c\ud2b8\ub108\ub294 \uc774 \uc601\ud654\uc5d0\uc11c \uc0ac\uc6b4\ub4dc\ud2b8\ub799 Dream Dream Dream Dream\uc744 \uc9c1\uc811 \ubd88\ub800\uc73c\uba70, \uc0ac\uc6b4\ub4dc\ud2b8\ub799 \uc74c\ubc18\uc5d0\ub3c4 \uc218\ub85d\uc774 \ub418\uc5c8\ub2e4. \uba87\uac1c\uc6d4 \ud6c4, \ub85c\ud2b8\ub108\ub294 \u300a\uc5f4\ub450\uba85\uc758 \uc6ec\uc218\ub4e4 2\u300b\uc5d0 \ucd9c\uc5f0\ud558\uc600\uace0, \uc601\ud654\ub294 \ub85c\ud2bc \ud1a0\ub9c8\ud1a0\uc5d0\uc11c \"2000\ub144\ub300 \ucd5c\uc545\uc758 \uc601\ud654\"\ub85c \ud3c9\uac00\ubc1b\uc558\ub2e4. \uce90\ub098\ub2e4\uc5d0\uc11c \ucd2c\uc601\uc744 \ub9c8\uce58\uace0 \ub3cc\uc544\uc628 \ud6c4\uc5d0\ub294 \uc0e4\ud06c\ubcf4\uc774\uc640 \ub77c\ubc14\uac78\uc5d0 \uc758\ud55c \uba85\uc131\uc744 \uc2e4\uac10\ud558\uc600\ub2e4. 2008\ub144 \ud154\ub808\ube44\uc804 \uc1fc \u300aLove, Inc.\u300b\uc640 \ud154\ub808\ube44\uc804 \uc2a4\ud398\uc15c \u300aHe's a Bully, Charlie Brown\u300b\uc5d0 \ucd9c\uc5f0\ud558\uc600\ub2e4. \uac19\uc740 \ud574 NBC\uc758 \ub4dc\ub77c\ub9c8 \u300a\ub9c8\uc774 \uc628 \uc6cc\uc2a4\ud2b8 \uc5d0\ub108\ubbf8\u300b\uc5d0 \ud06c\ub9ac\uc2a4\ucc9c \uc2ac\ub808\uc774\ud130\uac00 \ub9e1\uc740 \ud5e8\ub9ac \uc2a4\ud30c\uc774\ube44\uc758 \uc544\ub4e4 \uc7ad \uc2a4\ud30c\uc774\ube44 \uc5ed\ud560\ub85c \ucd9c\uc5f0\ud558\uc600\ub2e4. \ub864\ub9c1 \uc2a4\ud1a4\uc740 \ud14c\uc77c\ub7ec\uc758 \ucd08\uae30 \uc5ed\ud560\ub4e4\uc740 \"\ub85c\ud2b8\ub108\ub97c \uc778\uae30\uc788\ub294 \uc544\uc774, \uc6b4\ub3d9\uc744 \uc88b\uc544\ud558\ub294 \uc544\uc774 \ub610\ub294 \uace8\ubaa9 \ub300\uc7a5\"\uc73c\ub85c \ub9cc\ub4e4\uc5c8\ub2e4\uace0 \ud55c\ub2e4.", "paragraph_answer": "\ub85c\ud2b8\ub108\ub294 \ub85c\uc2a4\uc564\uc824\ub808\uc2a4\ub85c \uc774\ub3d9\ud55c \ud6c4 \uba87\uac1c\uc6d4 \ub3d9\uc548 \ud154\ub808\ube44\uc804 \ubc0f \uc601\ud654\uc5d0\uc11c \uc791\uc740 \uc5ed\ud560\uc744 \ub9e1\uace0 \uad11\uace0\uc5d0\ub3c4 \ucd9c\uc5f0\ud558\uc600\ub2e4. 2001\ub144, \ub85c\ud2b8\ub108\ub294 \ud154\ub808\ube44\uc804 \uc601\ud654 \u300a\ud504\ub9ac\ub9e8\u300b\uc5d0 \ucd9c\uc5f0\ud558\uc600\uace0, \u300a\ub7ec\uadf8\ub798\uce20 3: \ubb34\uc778\ub3c4 \ub300\ubaa8\ud5d8\u300b\uc758 \uad11\uace0\uc5d0\uc11c \ubaa9\uc18c\ub9ac\ub85c \ucd9c\uc5f0\ud558\uc600\ub2e4. \ub610\ud55c \u300a\ubc84\ub2c8 \ub9e5 \uc1fc\u300b, \u300a\ub9c8\uc774 \uc640\uc774\ud504 \uc564 \ud0a4\uc988\u300b, \u300a\uc378\uba38\ub79c\ub4dc\u300b\uc5d0\uc11c \uc791\uc740 \uc5ed\ud560\ub85c \ucd9c\uc5f0\ud558\uc600\ub2e4. \u300aWhat's New, Scooby-Doo?\u300b\uc640 \u300a\ub300\ub2c8 \ud32c\ud140\u300b\uc5d0\uc11c\ub294 \ubaa9\uc18c\ub9ac\ub85c\ub3c4 \ucd9c\uc5f0\ud558\uc600\ub2e4. 2005\ub144\uc5d0\ub294 \ucf54\ubbf8\ub514 \uc601\ud654 \u300a\uc5f4\ub450\uba85\uc758 \uc6ec\uc218\ub4e4 2\u300b\uc5d0\ub3c4 \uc791\uc740 \uc5ed\ud560\ub85c \ucd9c\uc5f0\ud558\uc600\ub2e4. \uac19\uc740 \ud574\uc5d0 \uac19\uc740 \uace0\ub4f1\ud559\uad50\ub97c \uc7ac\ud559 \uc911\uc778 \ud14c\uc77c\ub7ec \ub458\ub9ac\uc640 \ud568\uaed8 \u300a\uc0e4\ud06c\ubcf4\uc774\uc640 \ub77c\ubc14\uac78\uc758 \ubaa8\ud5d8 3-D\u300b\uc5d0\uc11c \ucc98\uc74c\uc73c\ub85c \uc8fc\uc5f0\uc744 \ub9e1\uc558\ub2e4. \ub85c\ud2b8\ub108\ub294 \uc774 \uc601\ud654\uc5d0\uc11c \ubaa8\ub4e0 \ubb34\uc220 \uc7a5\uba74\uc744 \uc9c1\uc811 \uc5f0\uae30 \ud558\uc600\uc73c\uba70, \uc774 \uc601\ud654\ub97c \ucd2c\uc601\ud558\uae30 \uc704\ud574 \uc624\uc2a4\ud2f4\uc5d0 3\uac1c\uc6d4\ub3d9\uc548 \uba38\ubb3c\uc5c8\ub2e4. \uc601\ud654\ub294 \ubd80\uc815\uc801\uc778 \ud3c9\uc744 \ubc1b\uc558\uace0, \uad6d\uc81c\uc801\uc73c\ub85c\ub294 \ud06c\uac8c \ud765\ud589\ud558\uc9c0\ub294 \uc54a\uc558\ub2e4. \ub85c\ud2b8\ub108\ub294 \uc774 \uc601\ud654\ub85c 27\ud68c \uc80a\uc740 \uc608\uc220\uac00\uc0c1\uc5d0 \ud6c4\ubcf4\ub85c \uc62c\ub790\uc73c\ub098 \uc0c1\uc744 \ubc1b\uc9c0\ub294 \ubabb\ud588\ub2e4. \ub85c\ud2b8\ub108\ub294 \uc774 \uc601\ud654\uc5d0\uc11c \uc0ac\uc6b4\ub4dc\ud2b8\ub799 Dream Dream Dream Dream\uc744 \uc9c1\uc811 \ubd88\ub800\uc73c\uba70, \uc0ac\uc6b4\ub4dc\ud2b8\ub799 \uc74c\ubc18\uc5d0\ub3c4 \uc218\ub85d\uc774 \ub418\uc5c8\ub2e4. \uba87\uac1c\uc6d4 \ud6c4, \ub85c\ud2b8\ub108\ub294 \u300a\uc5f4\ub450\uba85\uc758 \uc6ec\uc218\ub4e4 2\u300b\uc5d0 \ucd9c\uc5f0\ud558\uc600\uace0, \uc601\ud654\ub294 \ub85c\ud2bc \ud1a0\ub9c8\ud1a0\uc5d0\uc11c \"2000\ub144\ub300 \ucd5c\uc545\uc758 \uc601\ud654\"\ub85c \ud3c9\uac00\ubc1b\uc558\ub2e4. \uce90\ub098\ub2e4\uc5d0\uc11c \ucd2c\uc601\uc744 \ub9c8\uce58\uace0 \ub3cc\uc544\uc628 \ud6c4\uc5d0\ub294 \uc0e4\ud06c\ubcf4\uc774\uc640 \ub77c\ubc14\uac78\uc5d0 \uc758\ud55c \uba85\uc131\uc744 \uc2e4\uac10\ud558\uc600\ub2e4. 2008\ub144 \ud154\ub808\ube44\uc804 \uc1fc \u300aLove, Inc.\u300b\uc640 \ud154\ub808\ube44\uc804 \uc2a4\ud398\uc15c \u300aHe's a Bully, Charlie Brown\u300b\uc5d0 \ucd9c\uc5f0\ud558\uc600\ub2e4. \uac19\uc740 \ud574 NBC\uc758 \ub4dc\ub77c\ub9c8 \u300a\ub9c8\uc774 \uc628 \uc6cc\uc2a4\ud2b8 \uc5d0\ub108\ubbf8\u300b\uc5d0 \ud06c\ub9ac\uc2a4\ucc9c \uc2ac\ub808\uc774\ud130\uac00 \ub9e1\uc740 \ud5e8\ub9ac \uc2a4\ud30c\uc774\ube44\uc758 \uc544\ub4e4 \uc7ad \uc2a4\ud30c\uc774\ube44 \uc5ed\ud560\ub85c \ucd9c\uc5f0\ud558\uc600\ub2e4. \ub864\ub9c1 \uc2a4\ud1a4\uc740 \ud14c\uc77c\ub7ec\uc758 \ucd08\uae30 \uc5ed\ud560\ub4e4\uc740 \"\ub85c\ud2b8\ub108\ub97c \uc778\uae30\uc788\ub294 \uc544\uc774, \uc6b4\ub3d9\uc744 \uc88b\uc544\ud558\ub294 \uc544\uc774 \ub610\ub294 \uace8\ubaa9 \ub300\uc7a5\"\uc73c\ub85c \ub9cc\ub4e4\uc5c8\ub2e4\uace0 \ud55c\ub2e4.", "sentence_answer": "\ub85c\ud2b8\ub108\ub294 \uc774 \uc601\ud654\ub85c 27\ud68c \uc80a\uc740 \uc608\uc220\uac00\uc0c1\uc5d0 \ud6c4\ubcf4\ub85c \uc62c\ub790\uc73c\ub098 \uc0c1\uc744 \ubc1b\uc9c0\ub294 \ubabb\ud588\ub2e4.", "paragraph_id": "5912999-0-0", "paragraph_question": "question: \ub85c\ud2b8\ub108\ub97c \uc0e4\ud06c\ubcf4\uc774\uc640 \ub77c\ubc14\uac78\uc758 \ubaa8\ud5d8\uc73c\ub85c \uc81c \uba87 \ud68c \uc80a\uc740\uc608\uc220\uac00\uc0c1 \ud6c4\ubcf4\uc5d0 \uc62c\ub790\ub294\uac00?, context: \ub85c\ud2b8\ub108\ub294 \ub85c\uc2a4\uc564\uc824\ub808\uc2a4\ub85c \uc774\ub3d9\ud55c \ud6c4 \uba87\uac1c\uc6d4 \ub3d9\uc548 \ud154\ub808\ube44\uc804 \ubc0f \uc601\ud654\uc5d0\uc11c \uc791\uc740 \uc5ed\ud560\uc744 \ub9e1\uace0 \uad11\uace0\uc5d0\ub3c4 \ucd9c\uc5f0\ud558\uc600\ub2e4. 2001\ub144, \ub85c\ud2b8\ub108\ub294 \ud154\ub808\ube44\uc804 \uc601\ud654 \u300a\ud504\ub9ac\ub9e8\u300b\uc5d0 \ucd9c\uc5f0\ud558\uc600\uace0, \u300a\ub7ec\uadf8\ub798\uce20 3: \ubb34\uc778\ub3c4 \ub300\ubaa8\ud5d8\u300b\uc758 \uad11\uace0\uc5d0\uc11c \ubaa9\uc18c\ub9ac\ub85c \ucd9c\uc5f0\ud558\uc600\ub2e4. \ub610\ud55c \u300a\ubc84\ub2c8 \ub9e5 \uc1fc\u300b, \u300a\ub9c8\uc774 \uc640\uc774\ud504 \uc564 \ud0a4\uc988\u300b, \u300a\uc378\uba38\ub79c\ub4dc\u300b\uc5d0\uc11c \uc791\uc740 \uc5ed\ud560\ub85c \ucd9c\uc5f0\ud558\uc600\ub2e4. \u300aWhat's New, Scooby-Doo?\u300b\uc640 \u300a\ub300\ub2c8 \ud32c\ud140\u300b\uc5d0\uc11c\ub294 \ubaa9\uc18c\ub9ac\ub85c\ub3c4 \ucd9c\uc5f0\ud558\uc600\ub2e4. 2005\ub144\uc5d0\ub294 \ucf54\ubbf8\ub514 \uc601\ud654 \u300a\uc5f4\ub450\uba85\uc758 \uc6ec\uc218\ub4e4 2\u300b\uc5d0\ub3c4 \uc791\uc740 \uc5ed\ud560\ub85c \ucd9c\uc5f0\ud558\uc600\ub2e4. \uac19\uc740 \ud574\uc5d0 \uac19\uc740 \uace0\ub4f1\ud559\uad50\ub97c \uc7ac\ud559 \uc911\uc778 \ud14c\uc77c\ub7ec \ub458\ub9ac\uc640 \ud568\uaed8 \u300a\uc0e4\ud06c\ubcf4\uc774\uc640 \ub77c\ubc14\uac78\uc758 \ubaa8\ud5d8 3-D\u300b\uc5d0\uc11c \ucc98\uc74c\uc73c\ub85c \uc8fc\uc5f0\uc744 \ub9e1\uc558\ub2e4. \ub85c\ud2b8\ub108\ub294 \uc774 \uc601\ud654\uc5d0\uc11c \ubaa8\ub4e0 \ubb34\uc220 \uc7a5\uba74\uc744 \uc9c1\uc811 \uc5f0\uae30 \ud558\uc600\uc73c\uba70, \uc774 \uc601\ud654\ub97c \ucd2c\uc601\ud558\uae30 \uc704\ud574 \uc624\uc2a4\ud2f4\uc5d0 3\uac1c\uc6d4\ub3d9\uc548 \uba38\ubb3c\uc5c8\ub2e4. \uc601\ud654\ub294 \ubd80\uc815\uc801\uc778 \ud3c9\uc744 \ubc1b\uc558\uace0, \uad6d\uc81c\uc801\uc73c\ub85c\ub294 \ud06c\uac8c \ud765\ud589\ud558\uc9c0\ub294 \uc54a\uc558\ub2e4. \ub85c\ud2b8\ub108\ub294 \uc774 \uc601\ud654\ub85c 27\ud68c \uc80a\uc740 \uc608\uc220\uac00\uc0c1\uc5d0 \ud6c4\ubcf4\ub85c \uc62c\ub790\uc73c\ub098 \uc0c1\uc744 \ubc1b\uc9c0\ub294 \ubabb\ud588\ub2e4. \ub85c\ud2b8\ub108\ub294 \uc774 \uc601\ud654\uc5d0\uc11c \uc0ac\uc6b4\ub4dc\ud2b8\ub799 Dream Dream Dream Dream\uc744 \uc9c1\uc811 \ubd88\ub800\uc73c\uba70, \uc0ac\uc6b4\ub4dc\ud2b8\ub799 \uc74c\ubc18\uc5d0\ub3c4 \uc218\ub85d\uc774 \ub418\uc5c8\ub2e4. \uba87\uac1c\uc6d4 \ud6c4, \ub85c\ud2b8\ub108\ub294 \u300a\uc5f4\ub450\uba85\uc758 \uc6ec\uc218\ub4e4 2\u300b\uc5d0 \ucd9c\uc5f0\ud558\uc600\uace0, \uc601\ud654\ub294 \ub85c\ud2bc \ud1a0\ub9c8\ud1a0\uc5d0\uc11c \"2000\ub144\ub300 \ucd5c\uc545\uc758 \uc601\ud654\"\ub85c \ud3c9\uac00\ubc1b\uc558\ub2e4. \uce90\ub098\ub2e4\uc5d0\uc11c \ucd2c\uc601\uc744 \ub9c8\uce58\uace0 \ub3cc\uc544\uc628 \ud6c4\uc5d0\ub294 \uc0e4\ud06c\ubcf4\uc774\uc640 \ub77c\ubc14\uac78\uc5d0 \uc758\ud55c \uba85\uc131\uc744 \uc2e4\uac10\ud558\uc600\ub2e4. 2008\ub144 \ud154\ub808\ube44\uc804 \uc1fc \u300aLove, Inc.\u300b\uc640 \ud154\ub808\ube44\uc804 \uc2a4\ud398\uc15c \u300aHe's a Bully, Charlie Brown\u300b\uc5d0 \ucd9c\uc5f0\ud558\uc600\ub2e4. \uac19\uc740 \ud574 NBC\uc758 \ub4dc\ub77c\ub9c8 \u300a\ub9c8\uc774 \uc628 \uc6cc\uc2a4\ud2b8 \uc5d0\ub108\ubbf8\u300b\uc5d0 \ud06c\ub9ac\uc2a4\ucc9c \uc2ac\ub808\uc774\ud130\uac00 \ub9e1\uc740 \ud5e8\ub9ac \uc2a4\ud30c\uc774\ube44\uc758 \uc544\ub4e4 \uc7ad \uc2a4\ud30c\uc774\ube44 \uc5ed\ud560\ub85c \ucd9c\uc5f0\ud558\uc600\ub2e4. \ub864\ub9c1 \uc2a4\ud1a4\uc740 \ud14c\uc77c\ub7ec\uc758 \ucd08\uae30 \uc5ed\ud560\ub4e4\uc740 \"\ub85c\ud2b8\ub108\ub97c \uc778\uae30\uc788\ub294 \uc544\uc774, \uc6b4\ub3d9\uc744 \uc88b\uc544\ud558\ub294 \uc544\uc774 \ub610\ub294 \uace8\ubaa9 \ub300\uc7a5\"\uc73c\ub85c \ub9cc\ub4e4\uc5c8\ub2e4\uace0 \ud55c\ub2e4."} {"question": "\uc624\ubc84\uc6cc\uce58\uc758 \uc218\uc7a5\uc774 \ub41c \uc0ac\ub78c\uc740?", "paragraph": "\ub450 \uba85\uc758 \ub178\ub828\ud55c \uc720\uc804\uc790 \uac15\ud654 \uad70\uc778\uc778 \uac00\ube0c\ub9ac\uc5d8 \ub808\uc608\uc2a4\uc640 \uc7ad \ubaa8\ub9ac\uc2a8\uc774 \uc624\ubc84\uc6cc\uce58\ub97c \ucc45\uc784\uc9c0\uac8c \ub418\uc5c8\ub2e4. \uc624\ubc84\uc6cc\uce58\ub294 \ub85c\ubd07\ub4e4\uc758 \ubc18\ub780\uc744 \uc131\uacf5\uc801\uc73c\ub85c \uc9c4\uc555\ud558\uace0 \uc7ac\ub2a5\uc774 \uc788\ub294 \ub9ce\uc740 \uc0ac\ub78c\ub4e4\uc744 \uc911\uc694\ud55c \uc704\uce58\ub85c \ub04c\uc5b4\uc62c\ub838\uc9c0\ub9cc, \ub808\uc608\uc2a4\uc640 \ubaa8\ub9ac\uc2a8 \uac04\uc758 \ubd88\ud654\uac00 \ubc1c\uc0dd\ud558\uac8c \ub418\uace0, \ubaa8\ub9ac\uc2a8\uc740 \uc0ac\uc2e4\uc0c1 \uc624\ubc84\uc6cc\uce58\uc758 \uc218\uc7a5\uc774 \ub41c \ubc18\uba74 \ub808\uc608\uc2a4\ub294 \uc624\ubc84\uc6cc\uce58\uac00 \uacf5\uac1c\uc801\uc73c\ub85c \uc218\ud589\ud560 \uc218 \uc5c6\uc744 \ub9cc\ud07c \ub354\ub7ec\uc6b4 \ube44\ubc00 \uc791\uc804\uc744 \uc218\ud589\ud558\ub294 \ube14\ub799\uc6cc\uce58\ub97c \ub5a0\ub9e1\uac8c \ub418\uc5c8\ub2e4. \uc624\ubc84\uc6cc\uce58\ub294 '\uc624\ubc84\uc6cc\uce58 \uc138\ub300'\ub77c\uace0 \ubd80\ub974\ub294 \uba87 \uc2ed\ub144\uac04 \uc138\uacc4\ub97c \uc885\ud6a1\ubb34\uc9c4\ud558\uba70 \ud3c9\ud654\ub97c \uc720\uc9c0\ud558\uc600\uc73c\ub098, \ubaa8\ub9ac\uc2a8\uacfc \ub808\uc608\uc2a4\uc758 \ubd88\ud654\ub294 \ub354\uc6b1 \uae4a\uc5b4\ub9cc \uac14\ub2e4. \ubc94\uc8c4\uc640 \uc2e4\ud328\uc5d0 \ub300\ud55c \ubd80\uc815\uc801 \uc5ec\ub860\uc774 \uc624\ubc84\uc6cc\uce58\ub85c \ud5a5\ud558\uace0, \uc624\ubc84\uc6cc\uce58\uc5d0 \ub300\ud55c \ub300\uc911\uc758 \uac15\ub825\ud55c \ud56d\uc758\uc640 \uc694\uc6d0\ub4e4 \uac04\uc758 \ub0b4\uc804\uc774 \ubc1c\uc0dd\ud568\uc5d0 \ub530\ub77c, \uc720\uc5d4\uc740 \uc989\uc2dc \uc0c1\ud669\uc5d0 \ub300\ud55c \uc870\uc0ac\uc5d0 \ucc29\uc218\ud558\uc600\ub2e4. \uc774\ub54c, \uc624\ubc84\uc6cc\uce58\uc758 \ubcf8\ubd80\uac00 \ud3ed\ubc1c\ub85c \ud30c\uad34\ub418\uace0, \uc774\ub85c \uc778\ud574 \ubaa8\ub9ac\uc2a8\uacfc \ub808\uc608\uc2a4\uac00 \uc0ac\ub9dd\ud55c \uac83\uc73c\ub85c \uc54c\ub824\uc9c0\uac8c \ub418\uc5c8\ub2e4. \uc720\uc5d4\uc740 \uc624\ubc84\uc6cc\uce58\ub97c \ud574\uccb4\ud558\uace0 \uadf8\ub4e4\uc758 \ud65c\ub3d9\uc744 \uae08\ud558\ub294 \ud398\ud2b8\ub77c\uc2a4 \ubc95\uc744 \ud1b5\uacfc\uc2dc\ud0a8\ub2e4.", "answer": "\ubaa8\ub9ac\uc2a8", "sentence": "\ub450 \uba85\uc758 \ub178\ub828\ud55c \uc720\uc804\uc790 \uac15\ud654 \uad70\uc778\uc778 \uac00\ube0c\ub9ac\uc5d8 \ub808\uc608\uc2a4\uc640 \uc7ad \ubaa8\ub9ac\uc2a8 \uc774 \uc624\ubc84\uc6cc\uce58\ub97c \ucc45\uc784\uc9c0\uac8c \ub418\uc5c8\ub2e4.", "paragraph_sentence": " \ub450 \uba85\uc758 \ub178\ub828\ud55c \uc720\uc804\uc790 \uac15\ud654 \uad70\uc778\uc778 \uac00\ube0c\ub9ac\uc5d8 \ub808\uc608\uc2a4\uc640 \uc7ad \ubaa8\ub9ac\uc2a8 \uc774 \uc624\ubc84\uc6cc\uce58\ub97c \ucc45\uc784\uc9c0\uac8c \ub418\uc5c8\ub2e4. \uc624\ubc84\uc6cc\uce58\ub294 \ub85c\ubd07\ub4e4\uc758 \ubc18\ub780\uc744 \uc131\uacf5\uc801\uc73c\ub85c \uc9c4\uc555\ud558\uace0 \uc7ac\ub2a5\uc774 \uc788\ub294 \ub9ce\uc740 \uc0ac\ub78c\ub4e4\uc744 \uc911\uc694\ud55c \uc704\uce58\ub85c \ub04c\uc5b4\uc62c\ub838\uc9c0\ub9cc, \ub808\uc608\uc2a4\uc640 \ubaa8\ub9ac\uc2a8 \uac04\uc758 \ubd88\ud654\uac00 \ubc1c\uc0dd\ud558\uac8c \ub418\uace0, \ubaa8\ub9ac\uc2a8\uc740 \uc0ac\uc2e4\uc0c1 \uc624\ubc84\uc6cc\uce58\uc758 \uc218\uc7a5\uc774 \ub41c \ubc18\uba74 \ub808\uc608\uc2a4\ub294 \uc624\ubc84\uc6cc\uce58\uac00 \uacf5\uac1c\uc801\uc73c\ub85c \uc218\ud589\ud560 \uc218 \uc5c6\uc744 \ub9cc\ud07c \ub354\ub7ec\uc6b4 \ube44\ubc00 \uc791\uc804\uc744 \uc218\ud589\ud558\ub294 \ube14\ub799\uc6cc\uce58\ub97c \ub5a0\ub9e1\uac8c \ub418\uc5c8\ub2e4. \uc624\ubc84\uc6cc\uce58\ub294 '\uc624\ubc84\uc6cc\uce58 \uc138\ub300'\ub77c\uace0 \ubd80\ub974\ub294 \uba87 \uc2ed\ub144\uac04 \uc138\uacc4\ub97c \uc885\ud6a1\ubb34\uc9c4\ud558\uba70 \ud3c9\ud654\ub97c \uc720\uc9c0\ud558\uc600\uc73c\ub098, \ubaa8\ub9ac\uc2a8\uacfc \ub808\uc608\uc2a4\uc758 \ubd88\ud654\ub294 \ub354\uc6b1 \uae4a\uc5b4\ub9cc \uac14\ub2e4. \ubc94\uc8c4\uc640 \uc2e4\ud328\uc5d0 \ub300\ud55c \ubd80\uc815\uc801 \uc5ec\ub860\uc774 \uc624\ubc84\uc6cc\uce58\ub85c \ud5a5\ud558\uace0, \uc624\ubc84\uc6cc\uce58\uc5d0 \ub300\ud55c \ub300\uc911\uc758 \uac15\ub825\ud55c \ud56d\uc758\uc640 \uc694\uc6d0\ub4e4 \uac04\uc758 \ub0b4\uc804\uc774 \ubc1c\uc0dd\ud568\uc5d0 \ub530\ub77c, \uc720\uc5d4\uc740 \uc989\uc2dc \uc0c1\ud669\uc5d0 \ub300\ud55c \uc870\uc0ac\uc5d0 \ucc29\uc218\ud558\uc600\ub2e4. \uc774\ub54c, \uc624\ubc84\uc6cc\uce58\uc758 \ubcf8\ubd80\uac00 \ud3ed\ubc1c\ub85c \ud30c\uad34\ub418\uace0, \uc774\ub85c \uc778\ud574 \ubaa8\ub9ac\uc2a8\uacfc \ub808\uc608\uc2a4\uac00 \uc0ac\ub9dd\ud55c \uac83\uc73c\ub85c \uc54c\ub824\uc9c0\uac8c \ub418\uc5c8\ub2e4. \uc720\uc5d4\uc740 \uc624\ubc84\uc6cc\uce58\ub97c \ud574\uccb4\ud558\uace0 \uadf8\ub4e4\uc758 \ud65c\ub3d9\uc744 \uae08\ud558\ub294 \ud398\ud2b8\ub77c\uc2a4 \ubc95\uc744 \ud1b5\uacfc\uc2dc\ud0a8\ub2e4.", "paragraph_answer": "\ub450 \uba85\uc758 \ub178\ub828\ud55c \uc720\uc804\uc790 \uac15\ud654 \uad70\uc778\uc778 \uac00\ube0c\ub9ac\uc5d8 \ub808\uc608\uc2a4\uc640 \uc7ad \ubaa8\ub9ac\uc2a8 \uc774 \uc624\ubc84\uc6cc\uce58\ub97c \ucc45\uc784\uc9c0\uac8c \ub418\uc5c8\ub2e4. \uc624\ubc84\uc6cc\uce58\ub294 \ub85c\ubd07\ub4e4\uc758 \ubc18\ub780\uc744 \uc131\uacf5\uc801\uc73c\ub85c \uc9c4\uc555\ud558\uace0 \uc7ac\ub2a5\uc774 \uc788\ub294 \ub9ce\uc740 \uc0ac\ub78c\ub4e4\uc744 \uc911\uc694\ud55c \uc704\uce58\ub85c \ub04c\uc5b4\uc62c\ub838\uc9c0\ub9cc, \ub808\uc608\uc2a4\uc640 \ubaa8\ub9ac\uc2a8 \uac04\uc758 \ubd88\ud654\uac00 \ubc1c\uc0dd\ud558\uac8c \ub418\uace0, \ubaa8\ub9ac\uc2a8\uc740 \uc0ac\uc2e4\uc0c1 \uc624\ubc84\uc6cc\uce58\uc758 \uc218\uc7a5\uc774 \ub41c \ubc18\uba74 \ub808\uc608\uc2a4\ub294 \uc624\ubc84\uc6cc\uce58\uac00 \uacf5\uac1c\uc801\uc73c\ub85c \uc218\ud589\ud560 \uc218 \uc5c6\uc744 \ub9cc\ud07c \ub354\ub7ec\uc6b4 \ube44\ubc00 \uc791\uc804\uc744 \uc218\ud589\ud558\ub294 \ube14\ub799\uc6cc\uce58\ub97c \ub5a0\ub9e1\uac8c \ub418\uc5c8\ub2e4. \uc624\ubc84\uc6cc\uce58\ub294 '\uc624\ubc84\uc6cc\uce58 \uc138\ub300'\ub77c\uace0 \ubd80\ub974\ub294 \uba87 \uc2ed\ub144\uac04 \uc138\uacc4\ub97c \uc885\ud6a1\ubb34\uc9c4\ud558\uba70 \ud3c9\ud654\ub97c \uc720\uc9c0\ud558\uc600\uc73c\ub098, \ubaa8\ub9ac\uc2a8\uacfc \ub808\uc608\uc2a4\uc758 \ubd88\ud654\ub294 \ub354\uc6b1 \uae4a\uc5b4\ub9cc \uac14\ub2e4. \ubc94\uc8c4\uc640 \uc2e4\ud328\uc5d0 \ub300\ud55c \ubd80\uc815\uc801 \uc5ec\ub860\uc774 \uc624\ubc84\uc6cc\uce58\ub85c \ud5a5\ud558\uace0, \uc624\ubc84\uc6cc\uce58\uc5d0 \ub300\ud55c \ub300\uc911\uc758 \uac15\ub825\ud55c \ud56d\uc758\uc640 \uc694\uc6d0\ub4e4 \uac04\uc758 \ub0b4\uc804\uc774 \ubc1c\uc0dd\ud568\uc5d0 \ub530\ub77c, \uc720\uc5d4\uc740 \uc989\uc2dc \uc0c1\ud669\uc5d0 \ub300\ud55c \uc870\uc0ac\uc5d0 \ucc29\uc218\ud558\uc600\ub2e4. \uc774\ub54c, \uc624\ubc84\uc6cc\uce58\uc758 \ubcf8\ubd80\uac00 \ud3ed\ubc1c\ub85c \ud30c\uad34\ub418\uace0, \uc774\ub85c \uc778\ud574 \ubaa8\ub9ac\uc2a8\uacfc \ub808\uc608\uc2a4\uac00 \uc0ac\ub9dd\ud55c \uac83\uc73c\ub85c \uc54c\ub824\uc9c0\uac8c \ub418\uc5c8\ub2e4. \uc720\uc5d4\uc740 \uc624\ubc84\uc6cc\uce58\ub97c \ud574\uccb4\ud558\uace0 \uadf8\ub4e4\uc758 \ud65c\ub3d9\uc744 \uae08\ud558\ub294 \ud398\ud2b8\ub77c\uc2a4 \ubc95\uc744 \ud1b5\uacfc\uc2dc\ud0a8\ub2e4.", "sentence_answer": "\ub450 \uba85\uc758 \ub178\ub828\ud55c \uc720\uc804\uc790 \uac15\ud654 \uad70\uc778\uc778 \uac00\ube0c\ub9ac\uc5d8 \ub808\uc608\uc2a4\uc640 \uc7ad \ubaa8\ub9ac\uc2a8 \uc774 \uc624\ubc84\uc6cc\uce58\ub97c \ucc45\uc784\uc9c0\uac8c \ub418\uc5c8\ub2e4.", "paragraph_id": "6524626-10-0", "paragraph_question": "question: \uc624\ubc84\uc6cc\uce58\uc758 \uc218\uc7a5\uc774 \ub41c \uc0ac\ub78c\uc740?, context: \ub450 \uba85\uc758 \ub178\ub828\ud55c \uc720\uc804\uc790 \uac15\ud654 \uad70\uc778\uc778 \uac00\ube0c\ub9ac\uc5d8 \ub808\uc608\uc2a4\uc640 \uc7ad \ubaa8\ub9ac\uc2a8\uc774 \uc624\ubc84\uc6cc\uce58\ub97c \ucc45\uc784\uc9c0\uac8c \ub418\uc5c8\ub2e4. \uc624\ubc84\uc6cc\uce58\ub294 \ub85c\ubd07\ub4e4\uc758 \ubc18\ub780\uc744 \uc131\uacf5\uc801\uc73c\ub85c \uc9c4\uc555\ud558\uace0 \uc7ac\ub2a5\uc774 \uc788\ub294 \ub9ce\uc740 \uc0ac\ub78c\ub4e4\uc744 \uc911\uc694\ud55c \uc704\uce58\ub85c \ub04c\uc5b4\uc62c\ub838\uc9c0\ub9cc, \ub808\uc608\uc2a4\uc640 \ubaa8\ub9ac\uc2a8 \uac04\uc758 \ubd88\ud654\uac00 \ubc1c\uc0dd\ud558\uac8c \ub418\uace0, \ubaa8\ub9ac\uc2a8\uc740 \uc0ac\uc2e4\uc0c1 \uc624\ubc84\uc6cc\uce58\uc758 \uc218\uc7a5\uc774 \ub41c \ubc18\uba74 \ub808\uc608\uc2a4\ub294 \uc624\ubc84\uc6cc\uce58\uac00 \uacf5\uac1c\uc801\uc73c\ub85c \uc218\ud589\ud560 \uc218 \uc5c6\uc744 \ub9cc\ud07c \ub354\ub7ec\uc6b4 \ube44\ubc00 \uc791\uc804\uc744 \uc218\ud589\ud558\ub294 \ube14\ub799\uc6cc\uce58\ub97c \ub5a0\ub9e1\uac8c \ub418\uc5c8\ub2e4. \uc624\ubc84\uc6cc\uce58\ub294 '\uc624\ubc84\uc6cc\uce58 \uc138\ub300'\ub77c\uace0 \ubd80\ub974\ub294 \uba87 \uc2ed\ub144\uac04 \uc138\uacc4\ub97c \uc885\ud6a1\ubb34\uc9c4\ud558\uba70 \ud3c9\ud654\ub97c \uc720\uc9c0\ud558\uc600\uc73c\ub098, \ubaa8\ub9ac\uc2a8\uacfc \ub808\uc608\uc2a4\uc758 \ubd88\ud654\ub294 \ub354\uc6b1 \uae4a\uc5b4\ub9cc \uac14\ub2e4. \ubc94\uc8c4\uc640 \uc2e4\ud328\uc5d0 \ub300\ud55c \ubd80\uc815\uc801 \uc5ec\ub860\uc774 \uc624\ubc84\uc6cc\uce58\ub85c \ud5a5\ud558\uace0, \uc624\ubc84\uc6cc\uce58\uc5d0 \ub300\ud55c \ub300\uc911\uc758 \uac15\ub825\ud55c \ud56d\uc758\uc640 \uc694\uc6d0\ub4e4 \uac04\uc758 \ub0b4\uc804\uc774 \ubc1c\uc0dd\ud568\uc5d0 \ub530\ub77c, \uc720\uc5d4\uc740 \uc989\uc2dc \uc0c1\ud669\uc5d0 \ub300\ud55c \uc870\uc0ac\uc5d0 \ucc29\uc218\ud558\uc600\ub2e4. \uc774\ub54c, \uc624\ubc84\uc6cc\uce58\uc758 \ubcf8\ubd80\uac00 \ud3ed\ubc1c\ub85c \ud30c\uad34\ub418\uace0, \uc774\ub85c \uc778\ud574 \ubaa8\ub9ac\uc2a8\uacfc \ub808\uc608\uc2a4\uac00 \uc0ac\ub9dd\ud55c \uac83\uc73c\ub85c \uc54c\ub824\uc9c0\uac8c \ub418\uc5c8\ub2e4. \uc720\uc5d4\uc740 \uc624\ubc84\uc6cc\uce58\ub97c \ud574\uccb4\ud558\uace0 \uadf8\ub4e4\uc758 \ud65c\ub3d9\uc744 \uae08\ud558\ub294 \ud398\ud2b8\ub77c\uc2a4 \ubc95\uc744 \ud1b5\uacfc\uc2dc\ud0a8\ub2e4."} {"question": "\ubd81\ud55c \uc608\uc220\ub2e8\uc774 \uac15\ub989\uc5d0\uc11c \uacf5\uc5f0\ud588\ub358 \uc7a5\uc18c\ub294 \uc5b4\ub514\uc778\uac00?", "paragraph": "2\uc6d4 6\uc77c \ud604\uc1a1\uc6d4 \ub2e8\uc7a5\uacfc \uad8c\ud601\ubd09 \ubb38\ud654\uc131 \uad6d\uc7a5\uc774 \uc774\ub044\ub294 \ubd81\ud55c \uc608\uc220\ub2e8 \ubcf8\uc9c4\uc774 \ub9cc\uacbd\ubd09\ud638\ub97c \ud0c0\uace0 \ubb35\ud638\ud56d\uc5d0 \uc785\ud56d\ud588\ub2e4. \uc608\uc220\ub2e8\uc740 \uacf5\uc5f0\uc744 \ub300\ube44\ud558\uba74\uc11c \uc721\uc9c0\uac00 \uc544\ub2c8\ub77c \ub9cc\uacbd\ubd09\ud638\uc5d0\uc11c \uba38\ubb34\ub974\ub294\ub370 \ud754\ub4e4\ub9ac\ub294 \ubc30 \uc704\uc5d0\uc11c \uc219\uc2dd\uc744 \ud574\uacb0\ud558\ub2e4\ubcf4\ub2c8 \uba40\ubbf8\uc640 \ud53c\ub85c\ub85c \uace0\uc0dd\ud558\uac8c \ub418\uc9c0\ub9cc \ub0a8\uce21 \ubb38\ud654\uc640 \ucd5c\ub300\ud55c \uba40\ub9ac \ub5bc\uc5b4\ub193\uc73c\ub824\ub294 \ubd81\ud55c\uc758 \uc758\ub3c4\ub85c \uc778\ud574 \uacb0\uc815\ub41c \uac83\uc774\ub77c \ud55c\ub2e4. \uc774\ub294 2002\ub144 \uc544\uc2dc\uc548 \uac8c\uc784 \ub54c\ub3c4 \ub9c8\ucc2c\uac00\uc9c0\uc778\ub370 \ud55c\uad6d \uc815\ubd80\ub294 \uad00\ub840\ub97c \uace0\ub824\ud574 \ub9cc\uacbd\ubd09\ud638\uc5d0 \uc74c\uc2dd\uacfc \uc5f0\ub8cc\ub97c \uc81c\uacf5\ud558\ub294 \uac83\ub3c4 \uace0\ub824\ud558\ub294 \ud55c\ud3b8, \ubb35\ud638\ud56d \uc77c\ub300\ub97c \ud5ec\uae30\ub098 \ub4dc\ub860 \ub4f1\uc744 \ub0a0\ub9ac\uc9c0 \ubabb\ud558\uac8c \ube44\ud589\uae08\uc9c0\uad6c\uc5ed\uc73c\ub85c \uc124\uc815\ud588\ub2e4. \ud55c\ud3b8, \ub9cc\uacbd\ubd09\ud638\ub294 5.24 \uc870\uce58 \uc81c\uc7ac\ub300\uc0c1\uc774\uc9c0\ub9cc \uc774\ubc88\uc5d0\ub294 \uc815\ubd80\uac00 \uc608\uc678\ub97c \uc778\uc815\ud588\uc73c\uba70, \ub0a8\ubd81 \uac04 \uc774\ub3d9\uc774\uae30\uc5d0 \ubbf8\uad6d \ub3c5\uc790 \uc81c\uc7ac\uc640\ub3c4 \ubcc4\uac1c\ub77c\uace0 \ud310\ub2e8\ud588\ub2e4. \uc608\uc220\ub2e8\uc740 8\uc77c\uc5d0 \uac15\ub989\uc5d0\uc11c, 11\uc77c\uc5d0\ub294 \uc11c\uc6b8\uc5d0\uc11c \uacf5\uc5f0\uc744 \ud3bc\ucce4\ub2e4. \uac15\ub989\uc544\ud2b8\uc13c\ud130\uc5d0\uc11c\ub294 \u3008\ubc18\uac11\uc2b5\ub2c8\ub2e4\u3009\ub97c \ube44\ub86f\ud574\uc11c \u3008\ud770\ub208\uc544 \ub0b4\ub824\ub77c\u3009 \ub4f1\uacfc \ud568\uaed8 \u3008J\uc5d0\uac8c\u3009, \u3008\uc5ec\uc815\u3009 \ub4f1 \ud55c\uad6d \uac00\uc694\ub3c4 \ubd88\ub800\ub2e4. \uad6d\ub9bd\uc911\uc559\uadf9\uc7a5\uc5d0\uc11c\ub3c4 \uac15\ub989\uc544\ud2b8\uc13c\ud130\uc640 \ube44\uc2b7\ud55c \ub178\ub798\ub4e4\uc744 \ubd88\ub800\ub294\ub370 \uc774\ubc88\uc5d0\ub294 \u3008Old Black Joe\u3009\uc640 \uac19\uc740 \ubbf8\uad6d \uc74c\uc545\ub3c4 \ud758\ub7ec\ub098\uc654\uc73c\uba70 \ud604\uc1a1\uc6d4\uc774 \uc9c1\uc811 \u3008\ubc31\ub450\uc640 \ud55c\ub098\ub294 \ub0b4 \uc870\uad6d\u3009\uc774\ub77c\ub294 \uace1\uc870\ub97c \ubd80\ub974\uae30\ub3c4 \ud588\ub2e4. \uacf5\uc5f0\uc5d0\ub294 \uc815\ubd80 \ucd08\uccad \uc778\uc0ac\uc640 \ucd94\ucca8\uc73c\ub85c \uc120\ubc1c\ub41c \uc77c\ubc18 \uc2dc\ubbfc\ub4e4\uc774 \uac10\uc0c1\ud588\uc73c\uba70 \ub178\ub798 \uac00\uc0ac\uac00 \ubb38\uc81c\uc2dc\ub418\uae30\ub3c4 \ud588\uc9c0\ub9cc \ub0a8\ubd81 \ud569\ub3d9\uacf5\uc5f0\ub3c4 \uc774\ub8e8\uc5b4\uc9c0\ub294 \ub4f1 \ud0c8\uc5c6\uc774 \ub9c8\ubb34\ub9ac\ub418\uc5c8\ub2e4. 7\uc77c\uc5d0\ub294 \uae40\uc77c\uad6d \uccb4\uc721\uc0c1\uc774 \uc774\ub044\ub294 \ud0dc\uad8c\ub3c4\uc2dc\ubc94\ub2e8\uacfc \uc751\uc6d0\ub2e8 \ub4f1 280\uc5ec \uba85\uc774 \uacbd\uc758\uc120\uc744 \ud1b5\ud574 \ubc29\ub0a8\ud588\ub2e4. \uc751\uc6d0\ub2e8\uc740 \uc5ec\uc790 \uc544\uc774\uc2a4\ud558\ud0a4 \ub2e8\uc77c\ud300\uc744 \ube44\ub86f\ud574 \ub0a8\ubd81 \uc120\uc218\ub4e4\uc758 \uacbd\uae30\uc5d0\uc11c \uc5f4\ub764 \uc751\uc6d0\uc804\uc744 \ud3bc\ucce4\uc73c\uba70, \ud0dc\uad8c\ub3c4\uc2dc\ubc94\ub2e8\uc740 9\uc77c \uc62c\ub9bc\ud53d \uac1c\ub9c9\uc2dd \uc2dd\uc804\uacf5\uc5f0\uc744 \ube44\ub86f\ud574 \ud3c9\ucc3d, \uc18d\ucd08, \uc11c\uc6b8\uc5d0\uc11c \ub124 \ucc28\ub840 \uacf5\uc5f0\uc744 \uac1c\ucd5c\ud588\ub2e4.", "answer": "\uac15\ub989\uc544\ud2b8\uc13c\ud130", "sentence": "\uac15\ub989\uc544\ud2b8\uc13c\ud130 \uc5d0\uc11c\ub294 \u3008\ubc18\uac11\uc2b5\ub2c8\ub2e4\u3009\ub97c \ube44\ub86f\ud574\uc11c \u3008\ud770\ub208\uc544 \ub0b4\ub824\ub77c\u3009 \ub4f1\uacfc \ud568\uaed8 \u3008J\uc5d0\uac8c\u3009, \u3008\uc5ec\uc815\u3009 \ub4f1 \ud55c\uad6d \uac00\uc694\ub3c4 \ubd88\ub800\ub2e4.", "paragraph_sentence": "2\uc6d4 6\uc77c \ud604\uc1a1\uc6d4 \ub2e8\uc7a5\uacfc \uad8c\ud601\ubd09 \ubb38\ud654\uc131 \uad6d\uc7a5\uc774 \uc774\ub044\ub294 \ubd81\ud55c \uc608\uc220\ub2e8 \ubcf8\uc9c4\uc774 \ub9cc\uacbd\ubd09\ud638\ub97c \ud0c0\uace0 \ubb35\ud638\ud56d\uc5d0 \uc785\ud56d\ud588\ub2e4. \uc608\uc220\ub2e8\uc740 \uacf5\uc5f0\uc744 \ub300\ube44\ud558\uba74\uc11c \uc721\uc9c0\uac00 \uc544\ub2c8\ub77c \ub9cc\uacbd\ubd09\ud638\uc5d0\uc11c \uba38\ubb34\ub974\ub294\ub370 \ud754\ub4e4\ub9ac\ub294 \ubc30 \uc704\uc5d0\uc11c \uc219\uc2dd\uc744 \ud574\uacb0\ud558\ub2e4\ubcf4\ub2c8 \uba40\ubbf8\uc640 \ud53c\ub85c\ub85c \uace0\uc0dd\ud558\uac8c \ub418\uc9c0\ub9cc \ub0a8\uce21 \ubb38\ud654\uc640 \ucd5c\ub300\ud55c \uba40\ub9ac \ub5bc\uc5b4\ub193\uc73c\ub824\ub294 \ubd81\ud55c\uc758 \uc758\ub3c4\ub85c \uc778\ud574 \uacb0\uc815\ub41c \uac83\uc774\ub77c \ud55c\ub2e4. \uc774\ub294 2002\ub144 \uc544\uc2dc\uc548 \uac8c\uc784 \ub54c\ub3c4 \ub9c8\ucc2c\uac00\uc9c0\uc778\ub370 \ud55c\uad6d \uc815\ubd80\ub294 \uad00\ub840\ub97c \uace0\ub824\ud574 \ub9cc\uacbd\ubd09\ud638\uc5d0 \uc74c\uc2dd\uacfc \uc5f0\ub8cc\ub97c \uc81c\uacf5\ud558\ub294 \uac83\ub3c4 \uace0\ub824\ud558\ub294 \ud55c\ud3b8, \ubb35\ud638\ud56d \uc77c\ub300\ub97c \ud5ec\uae30\ub098 \ub4dc\ub860 \ub4f1\uc744 \ub0a0\ub9ac\uc9c0 \ubabb\ud558\uac8c \ube44\ud589\uae08\uc9c0\uad6c\uc5ed\uc73c\ub85c \uc124\uc815\ud588\ub2e4. \ud55c\ud3b8, \ub9cc\uacbd\ubd09\ud638\ub294 5.24 \uc870\uce58 \uc81c\uc7ac\ub300\uc0c1\uc774\uc9c0\ub9cc \uc774\ubc88\uc5d0\ub294 \uc815\ubd80\uac00 \uc608\uc678\ub97c \uc778\uc815\ud588\uc73c\uba70, \ub0a8\ubd81 \uac04 \uc774\ub3d9\uc774\uae30\uc5d0 \ubbf8\uad6d \ub3c5\uc790 \uc81c\uc7ac\uc640\ub3c4 \ubcc4\uac1c\ub77c\uace0 \ud310\ub2e8\ud588\ub2e4. \uc608\uc220\ub2e8\uc740 8\uc77c\uc5d0 \uac15\ub989\uc5d0\uc11c, 11\uc77c\uc5d0\ub294 \uc11c\uc6b8\uc5d0\uc11c \uacf5\uc5f0\uc744 \ud3bc\ucce4\ub2e4. \uac15\ub989\uc544\ud2b8\uc13c\ud130 \uc5d0\uc11c\ub294 \u3008\ubc18\uac11\uc2b5\ub2c8\ub2e4\u3009\ub97c \ube44\ub86f\ud574\uc11c \u3008\ud770\ub208\uc544 \ub0b4\ub824\ub77c\u3009 \ub4f1\uacfc \ud568\uaed8 \u3008J\uc5d0\uac8c\u3009, \u3008\uc5ec\uc815\u3009 \ub4f1 \ud55c\uad6d \uac00\uc694\ub3c4 \ubd88\ub800\ub2e4. \uad6d\ub9bd\uc911\uc559\uadf9\uc7a5\uc5d0\uc11c\ub3c4 \uac15\ub989\uc544\ud2b8\uc13c\ud130\uc640 \ube44\uc2b7\ud55c \ub178\ub798\ub4e4\uc744 \ubd88\ub800\ub294\ub370 \uc774\ubc88\uc5d0\ub294 \u3008Old Black Joe\u3009\uc640 \uac19\uc740 \ubbf8\uad6d \uc74c\uc545\ub3c4 \ud758\ub7ec\ub098\uc654\uc73c\uba70 \ud604\uc1a1\uc6d4\uc774 \uc9c1\uc811 \u3008\ubc31\ub450\uc640 \ud55c\ub098\ub294 \ub0b4 \uc870\uad6d\u3009\uc774\ub77c\ub294 \uace1\uc870\ub97c \ubd80\ub974\uae30\ub3c4 \ud588\ub2e4. \uacf5\uc5f0\uc5d0\ub294 \uc815\ubd80 \ucd08\uccad \uc778\uc0ac\uc640 \ucd94\ucca8\uc73c\ub85c \uc120\ubc1c\ub41c \uc77c\ubc18 \uc2dc\ubbfc\ub4e4\uc774 \uac10\uc0c1\ud588\uc73c\uba70 \ub178\ub798 \uac00\uc0ac\uac00 \ubb38\uc81c\uc2dc\ub418\uae30\ub3c4 \ud588\uc9c0\ub9cc \ub0a8\ubd81 \ud569\ub3d9\uacf5\uc5f0\ub3c4 \uc774\ub8e8\uc5b4\uc9c0\ub294 \ub4f1 \ud0c8\uc5c6\uc774 \ub9c8\ubb34\ub9ac\ub418\uc5c8\ub2e4. 7\uc77c\uc5d0\ub294 \uae40\uc77c\uad6d \uccb4\uc721\uc0c1\uc774 \uc774\ub044\ub294 \ud0dc\uad8c\ub3c4\uc2dc\ubc94\ub2e8\uacfc \uc751\uc6d0\ub2e8 \ub4f1 280\uc5ec \uba85\uc774 \uacbd\uc758\uc120\uc744 \ud1b5\ud574 \ubc29\ub0a8\ud588\ub2e4. \uc751\uc6d0\ub2e8\uc740 \uc5ec\uc790 \uc544\uc774\uc2a4\ud558\ud0a4 \ub2e8\uc77c\ud300\uc744 \ube44\ub86f\ud574 \ub0a8\ubd81 \uc120\uc218\ub4e4\uc758 \uacbd\uae30\uc5d0\uc11c \uc5f4\ub764 \uc751\uc6d0\uc804\uc744 \ud3bc\ucce4\uc73c\uba70, \ud0dc\uad8c\ub3c4\uc2dc\ubc94\ub2e8\uc740 9\uc77c \uc62c\ub9bc\ud53d \uac1c\ub9c9\uc2dd \uc2dd\uc804\uacf5\uc5f0\uc744 \ube44\ub86f\ud574 \ud3c9\ucc3d, \uc18d\ucd08, \uc11c\uc6b8\uc5d0\uc11c \ub124 \ucc28\ub840 \uacf5\uc5f0\uc744 \uac1c\ucd5c\ud588\ub2e4.", "paragraph_answer": "2\uc6d4 6\uc77c \ud604\uc1a1\uc6d4 \ub2e8\uc7a5\uacfc \uad8c\ud601\ubd09 \ubb38\ud654\uc131 \uad6d\uc7a5\uc774 \uc774\ub044\ub294 \ubd81\ud55c \uc608\uc220\ub2e8 \ubcf8\uc9c4\uc774 \ub9cc\uacbd\ubd09\ud638\ub97c \ud0c0\uace0 \ubb35\ud638\ud56d\uc5d0 \uc785\ud56d\ud588\ub2e4. \uc608\uc220\ub2e8\uc740 \uacf5\uc5f0\uc744 \ub300\ube44\ud558\uba74\uc11c \uc721\uc9c0\uac00 \uc544\ub2c8\ub77c \ub9cc\uacbd\ubd09\ud638\uc5d0\uc11c \uba38\ubb34\ub974\ub294\ub370 \ud754\ub4e4\ub9ac\ub294 \ubc30 \uc704\uc5d0\uc11c \uc219\uc2dd\uc744 \ud574\uacb0\ud558\ub2e4\ubcf4\ub2c8 \uba40\ubbf8\uc640 \ud53c\ub85c\ub85c \uace0\uc0dd\ud558\uac8c \ub418\uc9c0\ub9cc \ub0a8\uce21 \ubb38\ud654\uc640 \ucd5c\ub300\ud55c \uba40\ub9ac \ub5bc\uc5b4\ub193\uc73c\ub824\ub294 \ubd81\ud55c\uc758 \uc758\ub3c4\ub85c \uc778\ud574 \uacb0\uc815\ub41c \uac83\uc774\ub77c \ud55c\ub2e4. \uc774\ub294 2002\ub144 \uc544\uc2dc\uc548 \uac8c\uc784 \ub54c\ub3c4 \ub9c8\ucc2c\uac00\uc9c0\uc778\ub370 \ud55c\uad6d \uc815\ubd80\ub294 \uad00\ub840\ub97c \uace0\ub824\ud574 \ub9cc\uacbd\ubd09\ud638\uc5d0 \uc74c\uc2dd\uacfc \uc5f0\ub8cc\ub97c \uc81c\uacf5\ud558\ub294 \uac83\ub3c4 \uace0\ub824\ud558\ub294 \ud55c\ud3b8, \ubb35\ud638\ud56d \uc77c\ub300\ub97c \ud5ec\uae30\ub098 \ub4dc\ub860 \ub4f1\uc744 \ub0a0\ub9ac\uc9c0 \ubabb\ud558\uac8c \ube44\ud589\uae08\uc9c0\uad6c\uc5ed\uc73c\ub85c \uc124\uc815\ud588\ub2e4. \ud55c\ud3b8, \ub9cc\uacbd\ubd09\ud638\ub294 5.24 \uc870\uce58 \uc81c\uc7ac\ub300\uc0c1\uc774\uc9c0\ub9cc \uc774\ubc88\uc5d0\ub294 \uc815\ubd80\uac00 \uc608\uc678\ub97c \uc778\uc815\ud588\uc73c\uba70, \ub0a8\ubd81 \uac04 \uc774\ub3d9\uc774\uae30\uc5d0 \ubbf8\uad6d \ub3c5\uc790 \uc81c\uc7ac\uc640\ub3c4 \ubcc4\uac1c\ub77c\uace0 \ud310\ub2e8\ud588\ub2e4. \uc608\uc220\ub2e8\uc740 8\uc77c\uc5d0 \uac15\ub989\uc5d0\uc11c, 11\uc77c\uc5d0\ub294 \uc11c\uc6b8\uc5d0\uc11c \uacf5\uc5f0\uc744 \ud3bc\ucce4\ub2e4. \uac15\ub989\uc544\ud2b8\uc13c\ud130 \uc5d0\uc11c\ub294 \u3008\ubc18\uac11\uc2b5\ub2c8\ub2e4\u3009\ub97c \ube44\ub86f\ud574\uc11c \u3008\ud770\ub208\uc544 \ub0b4\ub824\ub77c\u3009 \ub4f1\uacfc \ud568\uaed8 \u3008J\uc5d0\uac8c\u3009, \u3008\uc5ec\uc815\u3009 \ub4f1 \ud55c\uad6d \uac00\uc694\ub3c4 \ubd88\ub800\ub2e4. \uad6d\ub9bd\uc911\uc559\uadf9\uc7a5\uc5d0\uc11c\ub3c4 \uac15\ub989\uc544\ud2b8\uc13c\ud130\uc640 \ube44\uc2b7\ud55c \ub178\ub798\ub4e4\uc744 \ubd88\ub800\ub294\ub370 \uc774\ubc88\uc5d0\ub294 \u3008Old Black Joe\u3009\uc640 \uac19\uc740 \ubbf8\uad6d \uc74c\uc545\ub3c4 \ud758\ub7ec\ub098\uc654\uc73c\uba70 \ud604\uc1a1\uc6d4\uc774 \uc9c1\uc811 \u3008\ubc31\ub450\uc640 \ud55c\ub098\ub294 \ub0b4 \uc870\uad6d\u3009\uc774\ub77c\ub294 \uace1\uc870\ub97c \ubd80\ub974\uae30\ub3c4 \ud588\ub2e4. \uacf5\uc5f0\uc5d0\ub294 \uc815\ubd80 \ucd08\uccad \uc778\uc0ac\uc640 \ucd94\ucca8\uc73c\ub85c \uc120\ubc1c\ub41c \uc77c\ubc18 \uc2dc\ubbfc\ub4e4\uc774 \uac10\uc0c1\ud588\uc73c\uba70 \ub178\ub798 \uac00\uc0ac\uac00 \ubb38\uc81c\uc2dc\ub418\uae30\ub3c4 \ud588\uc9c0\ub9cc \ub0a8\ubd81 \ud569\ub3d9\uacf5\uc5f0\ub3c4 \uc774\ub8e8\uc5b4\uc9c0\ub294 \ub4f1 \ud0c8\uc5c6\uc774 \ub9c8\ubb34\ub9ac\ub418\uc5c8\ub2e4. 7\uc77c\uc5d0\ub294 \uae40\uc77c\uad6d \uccb4\uc721\uc0c1\uc774 \uc774\ub044\ub294 \ud0dc\uad8c\ub3c4\uc2dc\ubc94\ub2e8\uacfc \uc751\uc6d0\ub2e8 \ub4f1 280\uc5ec \uba85\uc774 \uacbd\uc758\uc120\uc744 \ud1b5\ud574 \ubc29\ub0a8\ud588\ub2e4. \uc751\uc6d0\ub2e8\uc740 \uc5ec\uc790 \uc544\uc774\uc2a4\ud558\ud0a4 \ub2e8\uc77c\ud300\uc744 \ube44\ub86f\ud574 \ub0a8\ubd81 \uc120\uc218\ub4e4\uc758 \uacbd\uae30\uc5d0\uc11c \uc5f4\ub764 \uc751\uc6d0\uc804\uc744 \ud3bc\ucce4\uc73c\uba70, \ud0dc\uad8c\ub3c4\uc2dc\ubc94\ub2e8\uc740 9\uc77c \uc62c\ub9bc\ud53d \uac1c\ub9c9\uc2dd \uc2dd\uc804\uacf5\uc5f0\uc744 \ube44\ub86f\ud574 \ud3c9\ucc3d, \uc18d\ucd08, \uc11c\uc6b8\uc5d0\uc11c \ub124 \ucc28\ub840 \uacf5\uc5f0\uc744 \uac1c\ucd5c\ud588\ub2e4.", "sentence_answer": " \uac15\ub989\uc544\ud2b8\uc13c\ud130 \uc5d0\uc11c\ub294 \u3008\ubc18\uac11\uc2b5\ub2c8\ub2e4\u3009\ub97c \ube44\ub86f\ud574\uc11c \u3008\ud770\ub208\uc544 \ub0b4\ub824\ub77c\u3009 \ub4f1\uacfc \ud568\uaed8 \u3008J\uc5d0\uac8c\u3009, \u3008\uc5ec\uc815\u3009 \ub4f1 \ud55c\uad6d \uac00\uc694\ub3c4 \ubd88\ub800\ub2e4.", "paragraph_id": "6580768-107-1", "paragraph_question": "question: \ubd81\ud55c \uc608\uc220\ub2e8\uc774 \uac15\ub989\uc5d0\uc11c \uacf5\uc5f0\ud588\ub358 \uc7a5\uc18c\ub294 \uc5b4\ub514\uc778\uac00?, context: 2\uc6d4 6\uc77c \ud604\uc1a1\uc6d4 \ub2e8\uc7a5\uacfc \uad8c\ud601\ubd09 \ubb38\ud654\uc131 \uad6d\uc7a5\uc774 \uc774\ub044\ub294 \ubd81\ud55c \uc608\uc220\ub2e8 \ubcf8\uc9c4\uc774 \ub9cc\uacbd\ubd09\ud638\ub97c \ud0c0\uace0 \ubb35\ud638\ud56d\uc5d0 \uc785\ud56d\ud588\ub2e4. \uc608\uc220\ub2e8\uc740 \uacf5\uc5f0\uc744 \ub300\ube44\ud558\uba74\uc11c \uc721\uc9c0\uac00 \uc544\ub2c8\ub77c \ub9cc\uacbd\ubd09\ud638\uc5d0\uc11c \uba38\ubb34\ub974\ub294\ub370 \ud754\ub4e4\ub9ac\ub294 \ubc30 \uc704\uc5d0\uc11c \uc219\uc2dd\uc744 \ud574\uacb0\ud558\ub2e4\ubcf4\ub2c8 \uba40\ubbf8\uc640 \ud53c\ub85c\ub85c \uace0\uc0dd\ud558\uac8c \ub418\uc9c0\ub9cc \ub0a8\uce21 \ubb38\ud654\uc640 \ucd5c\ub300\ud55c \uba40\ub9ac \ub5bc\uc5b4\ub193\uc73c\ub824\ub294 \ubd81\ud55c\uc758 \uc758\ub3c4\ub85c \uc778\ud574 \uacb0\uc815\ub41c \uac83\uc774\ub77c \ud55c\ub2e4. \uc774\ub294 2002\ub144 \uc544\uc2dc\uc548 \uac8c\uc784 \ub54c\ub3c4 \ub9c8\ucc2c\uac00\uc9c0\uc778\ub370 \ud55c\uad6d \uc815\ubd80\ub294 \uad00\ub840\ub97c \uace0\ub824\ud574 \ub9cc\uacbd\ubd09\ud638\uc5d0 \uc74c\uc2dd\uacfc \uc5f0\ub8cc\ub97c \uc81c\uacf5\ud558\ub294 \uac83\ub3c4 \uace0\ub824\ud558\ub294 \ud55c\ud3b8, \ubb35\ud638\ud56d \uc77c\ub300\ub97c \ud5ec\uae30\ub098 \ub4dc\ub860 \ub4f1\uc744 \ub0a0\ub9ac\uc9c0 \ubabb\ud558\uac8c \ube44\ud589\uae08\uc9c0\uad6c\uc5ed\uc73c\ub85c \uc124\uc815\ud588\ub2e4. \ud55c\ud3b8, \ub9cc\uacbd\ubd09\ud638\ub294 5.24 \uc870\uce58 \uc81c\uc7ac\ub300\uc0c1\uc774\uc9c0\ub9cc \uc774\ubc88\uc5d0\ub294 \uc815\ubd80\uac00 \uc608\uc678\ub97c \uc778\uc815\ud588\uc73c\uba70, \ub0a8\ubd81 \uac04 \uc774\ub3d9\uc774\uae30\uc5d0 \ubbf8\uad6d \ub3c5\uc790 \uc81c\uc7ac\uc640\ub3c4 \ubcc4\uac1c\ub77c\uace0 \ud310\ub2e8\ud588\ub2e4. \uc608\uc220\ub2e8\uc740 8\uc77c\uc5d0 \uac15\ub989\uc5d0\uc11c, 11\uc77c\uc5d0\ub294 \uc11c\uc6b8\uc5d0\uc11c \uacf5\uc5f0\uc744 \ud3bc\ucce4\ub2e4. \uac15\ub989\uc544\ud2b8\uc13c\ud130\uc5d0\uc11c\ub294 \u3008\ubc18\uac11\uc2b5\ub2c8\ub2e4\u3009\ub97c \ube44\ub86f\ud574\uc11c \u3008\ud770\ub208\uc544 \ub0b4\ub824\ub77c\u3009 \ub4f1\uacfc \ud568\uaed8 \u3008J\uc5d0\uac8c\u3009, \u3008\uc5ec\uc815\u3009 \ub4f1 \ud55c\uad6d \uac00\uc694\ub3c4 \ubd88\ub800\ub2e4. \uad6d\ub9bd\uc911\uc559\uadf9\uc7a5\uc5d0\uc11c\ub3c4 \uac15\ub989\uc544\ud2b8\uc13c\ud130\uc640 \ube44\uc2b7\ud55c \ub178\ub798\ub4e4\uc744 \ubd88\ub800\ub294\ub370 \uc774\ubc88\uc5d0\ub294 \u3008Old Black Joe\u3009\uc640 \uac19\uc740 \ubbf8\uad6d \uc74c\uc545\ub3c4 \ud758\ub7ec\ub098\uc654\uc73c\uba70 \ud604\uc1a1\uc6d4\uc774 \uc9c1\uc811 \u3008\ubc31\ub450\uc640 \ud55c\ub098\ub294 \ub0b4 \uc870\uad6d\u3009\uc774\ub77c\ub294 \uace1\uc870\ub97c \ubd80\ub974\uae30\ub3c4 \ud588\ub2e4. \uacf5\uc5f0\uc5d0\ub294 \uc815\ubd80 \ucd08\uccad \uc778\uc0ac\uc640 \ucd94\ucca8\uc73c\ub85c \uc120\ubc1c\ub41c \uc77c\ubc18 \uc2dc\ubbfc\ub4e4\uc774 \uac10\uc0c1\ud588\uc73c\uba70 \ub178\ub798 \uac00\uc0ac\uac00 \ubb38\uc81c\uc2dc\ub418\uae30\ub3c4 \ud588\uc9c0\ub9cc \ub0a8\ubd81 \ud569\ub3d9\uacf5\uc5f0\ub3c4 \uc774\ub8e8\uc5b4\uc9c0\ub294 \ub4f1 \ud0c8\uc5c6\uc774 \ub9c8\ubb34\ub9ac\ub418\uc5c8\ub2e4. 7\uc77c\uc5d0\ub294 \uae40\uc77c\uad6d \uccb4\uc721\uc0c1\uc774 \uc774\ub044\ub294 \ud0dc\uad8c\ub3c4\uc2dc\ubc94\ub2e8\uacfc \uc751\uc6d0\ub2e8 \ub4f1 280\uc5ec \uba85\uc774 \uacbd\uc758\uc120\uc744 \ud1b5\ud574 \ubc29\ub0a8\ud588\ub2e4. \uc751\uc6d0\ub2e8\uc740 \uc5ec\uc790 \uc544\uc774\uc2a4\ud558\ud0a4 \ub2e8\uc77c\ud300\uc744 \ube44\ub86f\ud574 \ub0a8\ubd81 \uc120\uc218\ub4e4\uc758 \uacbd\uae30\uc5d0\uc11c \uc5f4\ub764 \uc751\uc6d0\uc804\uc744 \ud3bc\ucce4\uc73c\uba70, \ud0dc\uad8c\ub3c4\uc2dc\ubc94\ub2e8\uc740 9\uc77c \uc62c\ub9bc\ud53d \uac1c\ub9c9\uc2dd \uc2dd\uc804\uacf5\uc5f0\uc744 \ube44\ub86f\ud574 \ud3c9\ucc3d, \uc18d\ucd08, \uc11c\uc6b8\uc5d0\uc11c \ub124 \ucc28\ub840 \uacf5\uc5f0\uc744 \uac1c\ucd5c\ud588\ub2e4."} {"question": "2015\ub144\uc5d0\uc11c 2016\ub144 \uc2dc\uc98c\uc5d0 \ucee4\ub9ac\ub294 \ud55c \uc2dc\uc98c 3\uc810 \uc131\uacf5 \uac2f\uc218 \uc2e0\uae30\ub85d\uc73c\ub85c \uba87 \uac1c\ub97c \ub2ec\uc131\ud558\ub294\uac00?", "paragraph": "\uadf8\ub9ac\uace0 2014-2015 \uc2dc\uc98c \ucee4\ub9ac\ub294 \ub9f9\ud65c\uc57d\ud558\uba74\uc11c \uc6cc\ub9ac\uc5b4\uc2a4\ub97c \uc815\uaddc\uc2dc\uc98c 1\uc704\ub85c \uc774\ub04c\uc5c8\uace0, \uc774\ub97c \uc778\uc815\ubc1b\uc544 \ucd5c\uc6b0\uc218\uc120\uc218\uc0c1(MVP)\uacfc \uc62c-NBA \ud37c\uc2a4\ud2b8\ud300\uc5d0 \uc120\uc815\ub418\ub294 \ub4f1 \ucd5c\uace0\uc758 \uc2dc\uc98c\uc744 \ubcf4\ub0c8\ub2e4. \ud50c\ub808\uc774\uc624\ud504\uc5d0\uc11c\ub294 \ub274\uc62c\ub9ac\uc5b8\uc2a4 \ud3a0\ub9ac\ucee8\uc2a4, \uba64\ud53c\uc2a4 \uadf8\ub9ac\uc990\ub9ac\uc2a4, \ud734\uc2a4\ud134 \ub85c\ud0a4\uce20\ub97c \uc81c\uc555\ud558\uba74\uc11c \ud30c\uc774\ub110\uc5d0 \uc9c4\ucd9c\ud558\uc600\uace0, \ub974\ube0c\ub860 \uc81c\uc784\uc2a4\uac00 \uc774\ub044\ub294 \ud074\ub9ac\ube14\ub79c\ub4dc \uce90\ubc8c\ub9ac\uc5b4\uc2a4\uc640\uc758 \ud30c\uc774\ub110 \ubb34\ub300\uc5d0\uc11c\ub3c4 \uc0c1\ub300\uc758 \uc801\uadf9\uc801\uc778 \uacac\uc81c\uc640 \uc555\ubc15\uc73c\ub85c \uc778\ud55c \ucd08\ubc18 \ubd80\uc9c4\uc744 \ub5a8\uccd0\ub0b4\uace0 \uacf5\uaca9\uc758 \uc911\uc2ec\uc73c\ub85c \ud65c\uc57d\ud558\uba74\uc11c \uacb0\uad6d \ud300\uc744 40\ub144 \ub9cc\uc758 \uc6b0\uc2b9\uc73c\ub85c \uc774\ub04c\uc5c8\ub2e4. 2015-2016 \uc2dc\uc98c\uc5d0\ub294 \uace8\ub4e0\uc2a4\ud14c\uc774\ud2b8 \uc6cc\ub9ac\uc5b4\uc2a4\uc758 \uc804\uc124\uc801\uc778 72\uc2b9 9\ud328\uc758 \uae30\ub85d\uc744 \uc774\ub04c\uba70 \uc870\ub358\uc758 \uc2dc\uce74\uace0 \ubd88\uc2a4\ub97c \ub6f0\uc5b4\ub118\uc5c8\uc9c0\ub9cc, \ud30c\uc774\ub110\uc5d0\uc11c \ub974\ube0c\ub860\uc758 \ud074\ub9ac\ube14\ub79c\ub4dc\uc5d0 4-3\uc73c\ub85c \ud328\ud558\uba70 \uc6b0\uc2b9\uc740 \uac00\uc838\uc624\uc9c0 \ubabb\ud55c\ub2e4. 3-1\ub85c \uc2dc\ub9ac\uc988 \uc6b0\uc138\ub97c \uac00\uc838\uac14\uc5c8\ub358 \uace8\ub4e0\uc2a4\ud14c\uc774\ud2b8 \uc6cc\ub9ac\uc5b4\uc2a4\uc600\uae30\uc5d0 \uc544\uc26c\uc6c0\uc740 \ubc30\uac00 \ub418\uc5c8\ub2e4. 7\ucc28\uc804\uc5d0\uc11c \ud328\ud55c \ud6c4 \ucee4\ub9ac\ub294 \ub77c\ucee4\ub8f8\uc5d0\uc11c 1\uc2dc\uac04\ub3d9\uc548 \ub208\ubb3c\uc744 \ubcf4\uc774\uba70 \uc544\uc26c\uc6c0\uc744 \uac10\ucd94\uc9c0 \ubabb\ud588\ub2e4. \uadf8\ub7ec\ub098 \uc774 \uc2dc\uc98c\uc5d0 \ucee4\ub9ac\ub294 \uc790\uc2e0\uc774 \ubcf4\uc720\ud558\uace0\uc788\ub358 \ud55c\uc2dc\uc98c 3\uc810 \uc131\uacf5 \uac2f\uc218 \ucd5c\uace0\uae30\ub85d\uc778 272\uac1c\ub97c \ud6e8\uc52c \ub6f0\uc5b4\ub118\ub294 402\uac1c\ub97c \uae30\ub85d\ud558\uba70 \ub2e4\uc2dc \ud55c\ubc88 \uc790\uc2e0\uc744 \ub6f0\uc5b4\ub118\ub294 \ubaa8\uc2b5\uc744 \ubcf4\uc778\ub2e4.", "answer": "402\uac1c", "sentence": "\uadf8\ub7ec\ub098 \uc774 \uc2dc\uc98c\uc5d0 \ucee4\ub9ac\ub294 \uc790\uc2e0\uc774 \ubcf4\uc720\ud558\uace0\uc788\ub358 \ud55c\uc2dc\uc98c 3\uc810 \uc131\uacf5 \uac2f\uc218 \ucd5c\uace0\uae30\ub85d\uc778 272\uac1c\ub97c \ud6e8\uc52c \ub6f0\uc5b4\ub118\ub294 402\uac1c \ub97c \uae30\ub85d\ud558\uba70 \ub2e4\uc2dc \ud55c\ubc88 \uc790\uc2e0\uc744 \ub6f0\uc5b4\ub118\ub294 \ubaa8\uc2b5\uc744 \ubcf4\uc778\ub2e4.", "paragraph_sentence": "\uadf8\ub9ac\uace0 2014-2015 \uc2dc\uc98c \ucee4\ub9ac\ub294 \ub9f9\ud65c\uc57d\ud558\uba74\uc11c \uc6cc\ub9ac\uc5b4\uc2a4\ub97c \uc815\uaddc\uc2dc\uc98c 1\uc704\ub85c \uc774\ub04c\uc5c8\uace0, \uc774\ub97c \uc778\uc815\ubc1b\uc544 \ucd5c\uc6b0\uc218\uc120\uc218\uc0c1(MVP)\uacfc \uc62c-NBA \ud37c\uc2a4\ud2b8\ud300\uc5d0 \uc120\uc815\ub418\ub294 \ub4f1 \ucd5c\uace0\uc758 \uc2dc\uc98c\uc744 \ubcf4\ub0c8\ub2e4. \ud50c\ub808\uc774\uc624\ud504\uc5d0\uc11c\ub294 \ub274\uc62c\ub9ac\uc5b8\uc2a4 \ud3a0\ub9ac\ucee8\uc2a4, \uba64\ud53c\uc2a4 \uadf8\ub9ac\uc990\ub9ac\uc2a4, \ud734\uc2a4\ud134 \ub85c\ud0a4\uce20\ub97c \uc81c\uc555\ud558\uba74\uc11c \ud30c\uc774\ub110\uc5d0 \uc9c4\ucd9c\ud558\uc600\uace0, \ub974\ube0c\ub860 \uc81c\uc784\uc2a4\uac00 \uc774\ub044\ub294 \ud074\ub9ac\ube14\ub79c\ub4dc \uce90\ubc8c\ub9ac\uc5b4\uc2a4\uc640\uc758 \ud30c\uc774\ub110 \ubb34\ub300\uc5d0\uc11c\ub3c4 \uc0c1\ub300\uc758 \uc801\uadf9\uc801\uc778 \uacac\uc81c\uc640 \uc555\ubc15\uc73c\ub85c \uc778\ud55c \ucd08\ubc18 \ubd80\uc9c4\uc744 \ub5a8\uccd0\ub0b4\uace0 \uacf5\uaca9\uc758 \uc911\uc2ec\uc73c\ub85c \ud65c\uc57d\ud558\uba74\uc11c \uacb0\uad6d \ud300\uc744 40\ub144 \ub9cc\uc758 \uc6b0\uc2b9\uc73c\ub85c \uc774\ub04c\uc5c8\ub2e4. 2015-2016 \uc2dc\uc98c\uc5d0\ub294 \uace8\ub4e0\uc2a4\ud14c\uc774\ud2b8 \uc6cc\ub9ac\uc5b4\uc2a4\uc758 \uc804\uc124\uc801\uc778 72\uc2b9 9\ud328\uc758 \uae30\ub85d\uc744 \uc774\ub04c\uba70 \uc870\ub358\uc758 \uc2dc\uce74\uace0 \ubd88\uc2a4\ub97c \ub6f0\uc5b4\ub118\uc5c8\uc9c0\ub9cc, \ud30c\uc774\ub110\uc5d0\uc11c \ub974\ube0c\ub860\uc758 \ud074\ub9ac\ube14\ub79c\ub4dc\uc5d0 4-3\uc73c\ub85c \ud328\ud558\uba70 \uc6b0\uc2b9\uc740 \uac00\uc838\uc624\uc9c0 \ubabb\ud55c\ub2e4. 3-1\ub85c \uc2dc\ub9ac\uc988 \uc6b0\uc138\ub97c \uac00\uc838\uac14\uc5c8\ub358 \uace8\ub4e0\uc2a4\ud14c\uc774\ud2b8 \uc6cc\ub9ac\uc5b4\uc2a4\uc600\uae30\uc5d0 \uc544\uc26c\uc6c0\uc740 \ubc30\uac00 \ub418\uc5c8\ub2e4. 7\ucc28\uc804\uc5d0\uc11c \ud328\ud55c \ud6c4 \ucee4\ub9ac\ub294 \ub77c\ucee4\ub8f8\uc5d0\uc11c 1\uc2dc\uac04\ub3d9\uc548 \ub208\ubb3c\uc744 \ubcf4\uc774\uba70 \uc544\uc26c\uc6c0\uc744 \uac10\ucd94\uc9c0 \ubabb\ud588\ub2e4. \uadf8\ub7ec\ub098 \uc774 \uc2dc\uc98c\uc5d0 \ucee4\ub9ac\ub294 \uc790\uc2e0\uc774 \ubcf4\uc720\ud558\uace0\uc788\ub358 \ud55c\uc2dc\uc98c 3\uc810 \uc131\uacf5 \uac2f\uc218 \ucd5c\uace0\uae30\ub85d\uc778 272\uac1c\ub97c \ud6e8\uc52c \ub6f0\uc5b4\ub118\ub294 402\uac1c \ub97c \uae30\ub85d\ud558\uba70 \ub2e4\uc2dc \ud55c\ubc88 \uc790\uc2e0\uc744 \ub6f0\uc5b4\ub118\ub294 \ubaa8\uc2b5\uc744 \ubcf4\uc778\ub2e4. ", "paragraph_answer": "\uadf8\ub9ac\uace0 2014-2015 \uc2dc\uc98c \ucee4\ub9ac\ub294 \ub9f9\ud65c\uc57d\ud558\uba74\uc11c \uc6cc\ub9ac\uc5b4\uc2a4\ub97c \uc815\uaddc\uc2dc\uc98c 1\uc704\ub85c \uc774\ub04c\uc5c8\uace0, \uc774\ub97c \uc778\uc815\ubc1b\uc544 \ucd5c\uc6b0\uc218\uc120\uc218\uc0c1(MVP)\uacfc \uc62c-NBA \ud37c\uc2a4\ud2b8\ud300\uc5d0 \uc120\uc815\ub418\ub294 \ub4f1 \ucd5c\uace0\uc758 \uc2dc\uc98c\uc744 \ubcf4\ub0c8\ub2e4. \ud50c\ub808\uc774\uc624\ud504\uc5d0\uc11c\ub294 \ub274\uc62c\ub9ac\uc5b8\uc2a4 \ud3a0\ub9ac\ucee8\uc2a4, \uba64\ud53c\uc2a4 \uadf8\ub9ac\uc990\ub9ac\uc2a4, \ud734\uc2a4\ud134 \ub85c\ud0a4\uce20\ub97c \uc81c\uc555\ud558\uba74\uc11c \ud30c\uc774\ub110\uc5d0 \uc9c4\ucd9c\ud558\uc600\uace0, \ub974\ube0c\ub860 \uc81c\uc784\uc2a4\uac00 \uc774\ub044\ub294 \ud074\ub9ac\ube14\ub79c\ub4dc \uce90\ubc8c\ub9ac\uc5b4\uc2a4\uc640\uc758 \ud30c\uc774\ub110 \ubb34\ub300\uc5d0\uc11c\ub3c4 \uc0c1\ub300\uc758 \uc801\uadf9\uc801\uc778 \uacac\uc81c\uc640 \uc555\ubc15\uc73c\ub85c \uc778\ud55c \ucd08\ubc18 \ubd80\uc9c4\uc744 \ub5a8\uccd0\ub0b4\uace0 \uacf5\uaca9\uc758 \uc911\uc2ec\uc73c\ub85c \ud65c\uc57d\ud558\uba74\uc11c \uacb0\uad6d \ud300\uc744 40\ub144 \ub9cc\uc758 \uc6b0\uc2b9\uc73c\ub85c \uc774\ub04c\uc5c8\ub2e4. 2015-2016 \uc2dc\uc98c\uc5d0\ub294 \uace8\ub4e0\uc2a4\ud14c\uc774\ud2b8 \uc6cc\ub9ac\uc5b4\uc2a4\uc758 \uc804\uc124\uc801\uc778 72\uc2b9 9\ud328\uc758 \uae30\ub85d\uc744 \uc774\ub04c\uba70 \uc870\ub358\uc758 \uc2dc\uce74\uace0 \ubd88\uc2a4\ub97c \ub6f0\uc5b4\ub118\uc5c8\uc9c0\ub9cc, \ud30c\uc774\ub110\uc5d0\uc11c \ub974\ube0c\ub860\uc758 \ud074\ub9ac\ube14\ub79c\ub4dc\uc5d0 4-3\uc73c\ub85c \ud328\ud558\uba70 \uc6b0\uc2b9\uc740 \uac00\uc838\uc624\uc9c0 \ubabb\ud55c\ub2e4. 3-1\ub85c \uc2dc\ub9ac\uc988 \uc6b0\uc138\ub97c \uac00\uc838\uac14\uc5c8\ub358 \uace8\ub4e0\uc2a4\ud14c\uc774\ud2b8 \uc6cc\ub9ac\uc5b4\uc2a4\uc600\uae30\uc5d0 \uc544\uc26c\uc6c0\uc740 \ubc30\uac00 \ub418\uc5c8\ub2e4. 7\ucc28\uc804\uc5d0\uc11c \ud328\ud55c \ud6c4 \ucee4\ub9ac\ub294 \ub77c\ucee4\ub8f8\uc5d0\uc11c 1\uc2dc\uac04\ub3d9\uc548 \ub208\ubb3c\uc744 \ubcf4\uc774\uba70 \uc544\uc26c\uc6c0\uc744 \uac10\ucd94\uc9c0 \ubabb\ud588\ub2e4. \uadf8\ub7ec\ub098 \uc774 \uc2dc\uc98c\uc5d0 \ucee4\ub9ac\ub294 \uc790\uc2e0\uc774 \ubcf4\uc720\ud558\uace0\uc788\ub358 \ud55c\uc2dc\uc98c 3\uc810 \uc131\uacf5 \uac2f\uc218 \ucd5c\uace0\uae30\ub85d\uc778 272\uac1c\ub97c \ud6e8\uc52c \ub6f0\uc5b4\ub118\ub294 402\uac1c \ub97c \uae30\ub85d\ud558\uba70 \ub2e4\uc2dc \ud55c\ubc88 \uc790\uc2e0\uc744 \ub6f0\uc5b4\ub118\ub294 \ubaa8\uc2b5\uc744 \ubcf4\uc778\ub2e4.", "sentence_answer": "\uadf8\ub7ec\ub098 \uc774 \uc2dc\uc98c\uc5d0 \ucee4\ub9ac\ub294 \uc790\uc2e0\uc774 \ubcf4\uc720\ud558\uace0\uc788\ub358 \ud55c\uc2dc\uc98c 3\uc810 \uc131\uacf5 \uac2f\uc218 \ucd5c\uace0\uae30\ub85d\uc778 272\uac1c\ub97c \ud6e8\uc52c \ub6f0\uc5b4\ub118\ub294 402\uac1c \ub97c \uae30\ub85d\ud558\uba70 \ub2e4\uc2dc \ud55c\ubc88 \uc790\uc2e0\uc744 \ub6f0\uc5b4\ub118\ub294 \ubaa8\uc2b5\uc744 \ubcf4\uc778\ub2e4.", "paragraph_id": "6532884-3-2", "paragraph_question": "question: 2015\ub144\uc5d0\uc11c 2016\ub144 \uc2dc\uc98c\uc5d0 \ucee4\ub9ac\ub294 \ud55c \uc2dc\uc98c 3\uc810 \uc131\uacf5 \uac2f\uc218 \uc2e0\uae30\ub85d\uc73c\ub85c \uba87 \uac1c\ub97c \ub2ec\uc131\ud558\ub294\uac00?, context: \uadf8\ub9ac\uace0 2014-2015 \uc2dc\uc98c \ucee4\ub9ac\ub294 \ub9f9\ud65c\uc57d\ud558\uba74\uc11c \uc6cc\ub9ac\uc5b4\uc2a4\ub97c \uc815\uaddc\uc2dc\uc98c 1\uc704\ub85c \uc774\ub04c\uc5c8\uace0, \uc774\ub97c \uc778\uc815\ubc1b\uc544 \ucd5c\uc6b0\uc218\uc120\uc218\uc0c1(MVP)\uacfc \uc62c-NBA \ud37c\uc2a4\ud2b8\ud300\uc5d0 \uc120\uc815\ub418\ub294 \ub4f1 \ucd5c\uace0\uc758 \uc2dc\uc98c\uc744 \ubcf4\ub0c8\ub2e4. \ud50c\ub808\uc774\uc624\ud504\uc5d0\uc11c\ub294 \ub274\uc62c\ub9ac\uc5b8\uc2a4 \ud3a0\ub9ac\ucee8\uc2a4, \uba64\ud53c\uc2a4 \uadf8\ub9ac\uc990\ub9ac\uc2a4, \ud734\uc2a4\ud134 \ub85c\ud0a4\uce20\ub97c \uc81c\uc555\ud558\uba74\uc11c \ud30c\uc774\ub110\uc5d0 \uc9c4\ucd9c\ud558\uc600\uace0, \ub974\ube0c\ub860 \uc81c\uc784\uc2a4\uac00 \uc774\ub044\ub294 \ud074\ub9ac\ube14\ub79c\ub4dc \uce90\ubc8c\ub9ac\uc5b4\uc2a4\uc640\uc758 \ud30c\uc774\ub110 \ubb34\ub300\uc5d0\uc11c\ub3c4 \uc0c1\ub300\uc758 \uc801\uadf9\uc801\uc778 \uacac\uc81c\uc640 \uc555\ubc15\uc73c\ub85c \uc778\ud55c \ucd08\ubc18 \ubd80\uc9c4\uc744 \ub5a8\uccd0\ub0b4\uace0 \uacf5\uaca9\uc758 \uc911\uc2ec\uc73c\ub85c \ud65c\uc57d\ud558\uba74\uc11c \uacb0\uad6d \ud300\uc744 40\ub144 \ub9cc\uc758 \uc6b0\uc2b9\uc73c\ub85c \uc774\ub04c\uc5c8\ub2e4. 2015-2016 \uc2dc\uc98c\uc5d0\ub294 \uace8\ub4e0\uc2a4\ud14c\uc774\ud2b8 \uc6cc\ub9ac\uc5b4\uc2a4\uc758 \uc804\uc124\uc801\uc778 72\uc2b9 9\ud328\uc758 \uae30\ub85d\uc744 \uc774\ub04c\uba70 \uc870\ub358\uc758 \uc2dc\uce74\uace0 \ubd88\uc2a4\ub97c \ub6f0\uc5b4\ub118\uc5c8\uc9c0\ub9cc, \ud30c\uc774\ub110\uc5d0\uc11c \ub974\ube0c\ub860\uc758 \ud074\ub9ac\ube14\ub79c\ub4dc\uc5d0 4-3\uc73c\ub85c \ud328\ud558\uba70 \uc6b0\uc2b9\uc740 \uac00\uc838\uc624\uc9c0 \ubabb\ud55c\ub2e4. 3-1\ub85c \uc2dc\ub9ac\uc988 \uc6b0\uc138\ub97c \uac00\uc838\uac14\uc5c8\ub358 \uace8\ub4e0\uc2a4\ud14c\uc774\ud2b8 \uc6cc\ub9ac\uc5b4\uc2a4\uc600\uae30\uc5d0 \uc544\uc26c\uc6c0\uc740 \ubc30\uac00 \ub418\uc5c8\ub2e4. 7\ucc28\uc804\uc5d0\uc11c \ud328\ud55c \ud6c4 \ucee4\ub9ac\ub294 \ub77c\ucee4\ub8f8\uc5d0\uc11c 1\uc2dc\uac04\ub3d9\uc548 \ub208\ubb3c\uc744 \ubcf4\uc774\uba70 \uc544\uc26c\uc6c0\uc744 \uac10\ucd94\uc9c0 \ubabb\ud588\ub2e4. \uadf8\ub7ec\ub098 \uc774 \uc2dc\uc98c\uc5d0 \ucee4\ub9ac\ub294 \uc790\uc2e0\uc774 \ubcf4\uc720\ud558\uace0\uc788\ub358 \ud55c\uc2dc\uc98c 3\uc810 \uc131\uacf5 \uac2f\uc218 \ucd5c\uace0\uae30\ub85d\uc778 272\uac1c\ub97c \ud6e8\uc52c \ub6f0\uc5b4\ub118\ub294 402\uac1c\ub97c \uae30\ub85d\ud558\uba70 \ub2e4\uc2dc \ud55c\ubc88 \uc790\uc2e0\uc744 \ub6f0\uc5b4\ub118\ub294 \ubaa8\uc2b5\uc744 \ubcf4\uc778\ub2e4."} {"question": "1992\ub144 \uad6d\uc81c\ud1b5\ud654\uae30\uae08\uc778 IMF\uac00 \ub0b4\ub193\uc740 \ubcf4\uace0\uc11c\uc758 \uc774\ub984\uc740?", "paragraph": "\uc624\ub298\ub0a0 \uc544\uc2dc\uc544\uc9c0\uc5ed\uc740 \uc138\uacc4\uc131\uc7a5\uc758 \uc911\uc2ec\uc9c0\ub85c \ubd80\uc0c1\ud558\uace0 \uc788\uc9c0\ub9cc, \uace0\ub3c4\uc131\uc7a5 \uc18d\uc5d0\uc11c\ub3c4 \uc778\ud50c\ub808\uc774\uc158, \uc810\uc810 \ub192\uc544\uc9c0\ub294 \ub300\ubbf8\u00b7\ub300\uc77c \ub9c8\ucc30\ubb38\uc81c, \ud658\uacbd\ubb38\uc81c, \uc2e0\uacbd\uc81c\uad8c \uc870\uc131 \ub4f1\uc758 \uc5ec\ub7ec \uac00\uc9c0 \ubb38\uc81c\ub97c \uc548\uace0 \uc788\ub2e4. 1990\ub144\ub300 \ucd08\uc758 \uc544\uc2dc\uc544\uacbd\uc81c \uc804\ubc18\uc5d0 \uac78\uce5c \ud2b9\uc9d5\uc740 \uc131\uc7a5\ub960\uc758 \uc800\ud558\uc640 \uc778\ud50c\ub808\uc774\uc158\uc774\ub2e4. \uc774\uac83\uc740 1980\ub144\ub300 \ud6c4\ubc18 \uc800(\u4f4e)\uc778\ud50c\ub808\uc774\uc158\ud558\uc5d0\uc11c \ub192\uc740 \uc131\uc7a5\uc744 \ub204\ub838\ub358 \uc2dc\uae30\uc5d0 \ube44\ud558\uba74 \ud06c\uac8c \uc545\ud654\ub41c \uc0c1\ud669\uc774\ub2e4. \uc9c0\ub09c 80\ub144\ub300 2\uc790\ub9bf\uc218\uc758 \uacbd\uc81c\uc131\uc7a5\uc744 \uae30\ub85d\ud558\uba74\uc11c \uc138\uacc4\uc758 \ubd80\ub7ec\uc6c0\uc744 \ubaa8\uc558\ub358 \ub3d9\uc544\uc2dc\uc544 \uc8fc\uc694 \uad6d\uac00\ub4e4\uc740 90\ub144\uc5d0 \ub4e4\uc5b4 \uc131\uc7a5\ub960\uc774 \ub454\ud654\ub41c \uac00\uc6b4\ub370 \uadf8\ub3d9\uc548 \ub178\ucd9c\ub41c \uacbd\uc81c\uc758 \uad6c\uc870\uc801 \ubb38\uc81c\uc810\uc744 \uc2dc\uc815\ud574 \ub098\uac00\uc57c \ud560 \uc0c1\ud669\uc5d0 \uc774\ub974\uace0 \uc788\ub2e4. 92\ub144 \uad6d\uc81c\ud1b5\ud654\uae30\uae08(IMF)\uc774 \ub0b4\ub193\uc740 '\uc138\uacc4\uacbd\uc81c\uc804\ub9dd' \ubcf4\uace0\uc11c\uc5d0\uc11c \uc608\uce21\ud55c \ub300\ub85c \ud55c\uad6d, \uc778\ub3c4\ub124\uc2dc\uc544 \ub4f1 \uc544\uc2dc\uc544 \uc2e0\ud765\uacf5\uc5c5\uad6d\ub4e4\uc740 90\ub144\ub300 \uc911\ubc18 \uc774\ud6c4 \uace0\ub3c4\uc131\uc7a5\ucc45 \ucd94\uc9c4 \uacfc\uc815\uc5d0\uc11c \ub204\uc801\ub41c \uc5ec\ub7ec \uac00\uc9c0 \ubd80\uc791\uc6a9\ub4e4\uc758 \ubd84\ucd9c\ub85c \uc778\ud558\uc5ec \uacbd\uc81c\uc801 \uc5b4\ub824\uc6c0\uc5d0 \ube60\uc9c0\uae30 \uc2dc\uc791\ud588\ub2e4. \uadf8\ub9ac\uace0 \ub9c8\uce68\ub0b4 1997\ub144 \ud558\ubc18\uae30 \uc774\ud6c4 \uc544\uc2dc\uc544 \uac01\uad6d\uc740 \ub2ec\ub7ec \ubd80\uc871\uc73c\ub85c \ucc44\ubb34\ubd88\uc774\ud589\uc758 \uc704\uae30\uc5d0 \ucc98\ud574 IMF \ub4f1 \uad6d\uc81c\uae08\uc735\uad00\ub828 \uae30\uad00\ub4e4\ub85c\ubd80\ud130 \uae34\uae09 \uc218\ud608\uc744 \ubc1b\uc544\uc57c\ub9cc \ud560 \uc0c1\ud669\uc5d0 \uc774\ub974\ub800\ub2e4. \uc774\ub7ec\ud55c \uc544\uc2dc\uc544\uc758 \uacbd\uc81c\uc704\uae30\uc5d0 \ub300\ud574\uc11c IMF\ub294 \uc815\ubd80\uc758 \uacfc\ub3c4\ud55c \uc2dc\uc7a5\uac1c\uc785, \ubd88\ud569\ub9ac\ud55c \ub178\ub3d9\uc815\ucc45, \uae08\uc735\uae30\uad00\uc758 \ube44\ud569\ub9ac\uc801 \uc6b4\uc601 \ub4f1\uc744 \uaf3d\uace0 \uc774\uc758 \uc2dc\uc815\uc744 \uc544\uc2dc\uc544 \uac01\uad6d\uc5d0 \uac15\ub825\ud558\uac8c \uc694\uad6c\ud588\ub2e4.\ucd5c\uadfc\uc758 \uc544\uc2dc\uc544\uc9c0\uc5ed \ubb34\uc5ed\ub3d9\ud5a5\uc744 \ubcf4\uba74, \ub300\ubbf8 \uc218\ucd9c\uc774 \uc904\uace0 \uc544\uc2dc\uc544 \uc5ed\ub0b4\ubb34\uc5ed\uc774 \ud655\ub300\ub418\ub294 \uacbd\ud5a5\uc774 \uc788\ub294\ub370, \uc774\uac83\uc740 \uc544\uc2dc\uc544 \uac01\uad6d\uc758 \uc218\ucd9c\uc2dc\uc7a5\uc744 \ub2e4\ubcc0\ud654\ud558\uace0 \uc678\uc218\uc8fc\ub3c4\ub85c\ubd80\ud130 \ub0b4\uc218\uc8fc\ub3c4\uc758 \uacbd\uc81c\uc131\uc7a5\uc73c\ub85c\uc758 \uc804\ud658\uc744 \uc2dc\ub3c4\ud588\uae30 \ub54c\ubb38\uc774\ub2e4. \uc5ed\ub0b4\ubb34\uc5ed\uc758 \ud655\ub300\uc640 \ud568\uaed8 \uc774 \uc9c0\uc5ed \ub0b4\uc758 \ubb34\uc5ed\ubd88\uade0\ud615\ub3c4 \ub450\ub4dc\ub7ec\uc9c4\ub2e4. \uc608\ub97c \ub4e4\uba74, \uc2e0\ud765\uacf5\uc5c5\uad6d\uac00\uad70(NICS)\uc758 \ub300\uc77c \ubb34\uc5ed\uc801\uc790\uac00 1985\ub144\uc758 130\uc5b5 \ub2ec\ub7ec\uc5d0\uc11c 91\ub144\uc5d0\ub294 315\uc5b5 \ub2ec\ub7ec\ub85c 2.3\ubc30 \ub298\uc5b4\ub0ac\ub2e4. \uc544\uc2dc\uc544\uc9c0\uc5ed\uc758 \uc9c1\uc811\ud22c\uc790\ubd80\ubb38\uc5d0\uc11c NICS \uae30\uc5c5\uc774 \uc77c\ubcf8 \uae30\uc5c5\ubcf4\ub2e4 \ud6e8\uc52c \uc9c4\ucd9c \uc18d\ub3c4\uac00 \ube60\ub974\ub2e4. 91\ub144\uc758 ASEAN\uc5d0 \ub300\ud55c NICS\uc758 \uc9c1\uc811 \ud22c\uc790\uc561\uc740 130\uc5b5 \ub2ec\ub7ec\ub85c 1989\ub144\uc758 43\uc5b5 \ub2ec\ub7ec\uc5d0 \ube44\ud574 \ubb34\ub824 3.1\ubc30\ub85c \uae09\uc99d\ud558\uc600\ub2e4. \uc774\uac83\uc740 89\ub144\uc758 50\uc5b5 \ub2ec\ub7ec\uc5d0\uc11c 90\ub144\uc758 61\uc5b5 \ub2ec\ub7ec\ub85c \ub298\uc5b4\ub09c \ubbf8\uad6d\uc758 \ud22c\uc790\uc561\uacfc \ube44\uad50\ud558\uba74 NICS\uc758 ASEAN\uc5d0 \ub300\ud55c \ud22c\uc790 \ubd90\uc774 \uc5b4\ub290 \uc815\ub3c4\uc778\uc9c0 \uac00\ub2a0\ud560 \uc218 \uc788\ub2e4. NICS\ub294 \uc544\uc2dc\uc544 \uad6d\uac00 \uc911\uc5d0\uc11c\ub3c4 \ud2b9\ud788 \ubca0\ud2b8\ub0a8\uc5d0 \uad00\uc2ec\uc774 \ub9ce\ub2e4. \ud2b9\ud788 \ud0c0\uc774\uc644\uacfc \ud64d\ucf69 \uc790\ubcf8\uc774 \uac74\uc218\ub85c 36.4%\ub97c \ucc28\uc9c0\ud558\uc5ec 2\uc704\uc778 \uc61b \uc885\uc8fc\uad6d \ud504\ub791\uc2a4\uc758 8.9%\ub97c \ud06c\uac8c \uc55e\uc9c0\ub974\uace0 \uc788\ub2e4. \ud0c0\uc774\uc644\uacfc \ud64d\ucf69 \uae30\uc5c5\uc758 \ud65c\ub3d9\uc774 \uc774\ub300\ub85c \uacc4\uc18d\ub41c\ub2e4\uba74 \ubca0\ud2b8\ub0a8\uc740 \ud654\uad50\uacbd\uc81c\uad8c\uc73c\ub85c \uacb0\uc18d\ub420 \uac00\ub2a5\uc131\uc774 \ud06c\ub2e4. \ucd5c\uadfc \ub4e4\uc5b4 ASEAN \uacbd\uc81c\uad8c\uc758 \uc131\uc7a5\uc774 \ub450\ub4dc\ub7ec\uc9c0\uace0 \uc788\ub2e4. 1980\ub144\ub300 \uc911\ubc18\uc5d0 \uadf9\uc2ec\ud55c \uacbd\uae30\uce68\uccb4\uc5d0 \ube60\uc84c\ub358 \uc774\ub4e4 ASEAN\uad6d\uac00\ub4e4\uc740 \uadf8 \ub4a4 3\uc800\ud604\uc0c1\uacfc \uc544\uc2dc\uc544\uc9c0\uc5ed\ub0b4 \uc0b0\uc5c5\uad6c\uc870 \uc7ac\ud3b8 \ub4f1\uacfc \uac19\uc740 \uad6d\ub0b4\uc678\uc758 \uacbd\uc81c\uc5ec\uac74 \ubcc0\ud654\uc5d0 \ud798\uc785\uc5b4 \uacbd\uc81c\uc131\uc7a5\uc138\ub97c \ub2e8\uae30\uac04 \ub0b4\uc5d0 \ud68c\ubcf5\ud568\uc73c\ub85c\uc368 \uc131\uc7a5\uc758 \ub300\uba85\uc0ac\uaca9\uc778 NICS\uc758 \uc131\uc7a5\ub960\uc744 \uc55e\uc9c0\ub974\ub294 \uc138\uacc4 \ucd5c\uace0\uc758 \uacbd\uc81c\uc131\uc7a5\uc9c0\uc5ed\uc73c\ub85c \ub4f1\uc7a5\ud558\uc600\ub2e4. \uc774\ub7ec\ud55c \ucd94\uc138\ub77c\uba74 90\ub144\ub300 \uc911\ubc18\uc5d0 \uac00\uc11c\ub294 \ud0c0\uc774\u00b7\ub9d0\ub808\uc774\uc2dc\uc544 \ub4f1\uc740 \uc2e0\ud765\uacf5\uc5c5\uad6d\uc73c\ub85c \uc9c4\uc785\ud560 \uac83\uc774 \ud655\uc2e4\uc2dc\ub418\uace0 \uc788\ub2e4. \uc774\uc5d0 \ub530\ub77c ASEAN \uad6d\uac00\uc5d0 \ub300\ud55c \uc138\uacc4 \uac01\uad6d, \ud2b9\ud788 \ud55c\uad6d\uacfc \uc77c\ubcf8 \ub4f1 \uc544\uc2dc\uc544 \uc120\u00b7\uc911\uc9c4\uad6d\uc758 \uad00\uc2ec\uc774 \ud06c\uac8c \ub192\uc544\uac00\uace0 \uc788\ub2e4. \uadf8\ubfd0\ub9cc \uc544\ub2c8\ub77c \ud55c\uad6d\uc740 \uacbd\uc81c\uc131\uc7a5\uc5d0 \uc804\ub825\uc744 \ub2e4\ud558\uace0 \uc788\ub294 \uc911\uad6d \ubc0f \ub0a8\u00b7\ubd81\ud55c \uad00\uacc4\uac1c\uc120\uc758 \uae30\uc6b4\uc5d0 \ud798\uc785\uc5b4 \uacbd\uc81c\uc758 \ud65c\uc131\ud654\uc5d0 \ud798\uc4f0\uace0 \uc788\ub294 \ubd81\ud55c\uc5d0 \ub300\ud55c \uc790\ubcf8\ud22c\uc790\uc640 \uad50\uc5ed\uc5d0\ub3c4 \uc801\uadf9\uc801\uc774\ub2e4.", "answer": "\uc138\uacc4\uacbd\uc81c\uc804\ub9dd", "sentence": "92\ub144 \uad6d\uc81c\ud1b5\ud654\uae30\uae08(IMF)\uc774 \ub0b4\ub193\uc740 ' \uc138\uacc4\uacbd\uc81c\uc804\ub9dd '", "paragraph_sentence": "\uc624\ub298\ub0a0 \uc544\uc2dc\uc544\uc9c0\uc5ed\uc740 \uc138\uacc4\uc131\uc7a5\uc758 \uc911\uc2ec\uc9c0\ub85c \ubd80\uc0c1\ud558\uace0 \uc788\uc9c0\ub9cc, \uace0\ub3c4\uc131\uc7a5 \uc18d\uc5d0\uc11c\ub3c4 \uc778\ud50c\ub808\uc774\uc158, \uc810\uc810 \ub192\uc544\uc9c0\ub294 \ub300\ubbf8\u00b7\ub300\uc77c \ub9c8\ucc30\ubb38\uc81c, \ud658\uacbd\ubb38\uc81c, \uc2e0\uacbd\uc81c\uad8c \uc870\uc131 \ub4f1\uc758 \uc5ec\ub7ec \uac00\uc9c0 \ubb38\uc81c\ub97c \uc548\uace0 \uc788\ub2e4. 1990\ub144\ub300 \ucd08\uc758 \uc544\uc2dc\uc544\uacbd\uc81c \uc804\ubc18\uc5d0 \uac78\uce5c \ud2b9\uc9d5\uc740 \uc131\uc7a5\ub960\uc758 \uc800\ud558\uc640 \uc778\ud50c\ub808\uc774\uc158\uc774\ub2e4. \uc774\uac83\uc740 1980\ub144\ub300 \ud6c4\ubc18 \uc800(\u4f4e)\uc778\ud50c\ub808\uc774\uc158\ud558\uc5d0\uc11c \ub192\uc740 \uc131\uc7a5\uc744 \ub204\ub838\ub358 \uc2dc\uae30\uc5d0 \ube44\ud558\uba74 \ud06c\uac8c \uc545\ud654\ub41c \uc0c1\ud669\uc774\ub2e4. \uc9c0\ub09c 80\ub144\ub300 2\uc790\ub9bf\uc218\uc758 \uacbd\uc81c\uc131\uc7a5\uc744 \uae30\ub85d\ud558\uba74\uc11c \uc138\uacc4\uc758 \ubd80\ub7ec\uc6c0\uc744 \ubaa8\uc558\ub358 \ub3d9\uc544\uc2dc\uc544 \uc8fc\uc694 \uad6d\uac00\ub4e4\uc740 90\ub144\uc5d0 \ub4e4\uc5b4 \uc131\uc7a5\ub960\uc774 \ub454\ud654\ub41c \uac00\uc6b4\ub370 \uadf8\ub3d9\uc548 \ub178\ucd9c\ub41c \uacbd\uc81c\uc758 \uad6c\uc870\uc801 \ubb38\uc81c\uc810\uc744 \uc2dc\uc815\ud574 \ub098\uac00\uc57c \ud560 \uc0c1\ud669\uc5d0 \uc774\ub974\uace0 \uc788\ub2e4. 92\ub144 \uad6d\uc81c\ud1b5\ud654\uae30\uae08(IMF)\uc774 \ub0b4\ub193\uc740 ' \uc138\uacc4\uacbd\uc81c\uc804\ub9dd ' \ubcf4\uace0\uc11c\uc5d0\uc11c \uc608\uce21\ud55c \ub300\ub85c \ud55c\uad6d, \uc778\ub3c4\ub124\uc2dc\uc544 \ub4f1 \uc544\uc2dc\uc544 \uc2e0\ud765\uacf5\uc5c5\uad6d\ub4e4\uc740 90\ub144\ub300 \uc911\ubc18 \uc774\ud6c4 \uace0\ub3c4\uc131\uc7a5\ucc45 \ucd94\uc9c4 \uacfc\uc815\uc5d0\uc11c \ub204\uc801\ub41c \uc5ec\ub7ec \uac00\uc9c0 \ubd80\uc791\uc6a9\ub4e4\uc758 \ubd84\ucd9c\ub85c \uc778\ud558\uc5ec \uacbd\uc81c\uc801 \uc5b4\ub824\uc6c0\uc5d0 \ube60\uc9c0\uae30 \uc2dc\uc791\ud588\ub2e4. \uadf8\ub9ac\uace0 \ub9c8\uce68\ub0b4 1997\ub144 \ud558\ubc18\uae30 \uc774\ud6c4 \uc544\uc2dc\uc544 \uac01\uad6d\uc740 \ub2ec\ub7ec \ubd80\uc871\uc73c\ub85c \ucc44\ubb34\ubd88\uc774\ud589\uc758 \uc704\uae30\uc5d0 \ucc98\ud574 IMF \ub4f1 \uad6d\uc81c\uae08\uc735\uad00\ub828 \uae30\uad00\ub4e4\ub85c\ubd80\ud130 \uae34\uae09 \uc218\ud608\uc744 \ubc1b\uc544\uc57c\ub9cc \ud560 \uc0c1\ud669\uc5d0 \uc774\ub974\ub800\ub2e4. \uc774\ub7ec\ud55c \uc544\uc2dc\uc544\uc758 \uacbd\uc81c\uc704\uae30\uc5d0 \ub300\ud574\uc11c IMF\ub294 \uc815\ubd80\uc758 \uacfc\ub3c4\ud55c \uc2dc\uc7a5\uac1c\uc785, \ubd88\ud569\ub9ac\ud55c \ub178\ub3d9\uc815\ucc45, \uae08\uc735\uae30\uad00\uc758 \ube44\ud569\ub9ac\uc801 \uc6b4\uc601 \ub4f1\uc744 \uaf3d\uace0 \uc774\uc758 \uc2dc\uc815\uc744 \uc544\uc2dc\uc544 \uac01\uad6d\uc5d0 \uac15\ub825\ud558\uac8c \uc694\uad6c\ud588\ub2e4.\ucd5c\uadfc\uc758 \uc544\uc2dc\uc544\uc9c0\uc5ed \ubb34\uc5ed\ub3d9\ud5a5\uc744 \ubcf4\uba74, \ub300\ubbf8 \uc218\ucd9c\uc774 \uc904\uace0 \uc544\uc2dc\uc544 \uc5ed\ub0b4\ubb34\uc5ed\uc774 \ud655\ub300\ub418\ub294 \uacbd\ud5a5\uc774 \uc788\ub294\ub370, \uc774\uac83\uc740 \uc544\uc2dc\uc544 \uac01\uad6d\uc758 \uc218\ucd9c\uc2dc\uc7a5\uc744 \ub2e4\ubcc0\ud654\ud558\uace0 \uc678\uc218\uc8fc\ub3c4\ub85c\ubd80\ud130 \ub0b4\uc218\uc8fc\ub3c4\uc758 \uacbd\uc81c\uc131\uc7a5\uc73c\ub85c\uc758 \uc804\ud658\uc744 \uc2dc\ub3c4\ud588\uae30 \ub54c\ubb38\uc774\ub2e4. \uc5ed\ub0b4\ubb34\uc5ed\uc758 \ud655\ub300\uc640 \ud568\uaed8 \uc774 \uc9c0\uc5ed \ub0b4\uc758 \ubb34\uc5ed\ubd88\uade0\ud615\ub3c4 \ub450\ub4dc\ub7ec\uc9c4\ub2e4. \uc608\ub97c \ub4e4\uba74, \uc2e0\ud765\uacf5\uc5c5\uad6d\uac00\uad70(NICS)\uc758 \ub300\uc77c \ubb34\uc5ed\uc801\uc790\uac00 1985\ub144\uc758 130\uc5b5 \ub2ec\ub7ec\uc5d0\uc11c 91\ub144\uc5d0\ub294 315\uc5b5 \ub2ec\ub7ec\ub85c 2.3\ubc30 \ub298\uc5b4\ub0ac\ub2e4. \uc544\uc2dc\uc544\uc9c0\uc5ed\uc758 \uc9c1\uc811\ud22c\uc790\ubd80\ubb38\uc5d0\uc11c NICS \uae30\uc5c5\uc774 \uc77c\ubcf8 \uae30\uc5c5\ubcf4\ub2e4 \ud6e8\uc52c \uc9c4\ucd9c \uc18d\ub3c4\uac00 \ube60\ub974\ub2e4. 91\ub144\uc758 ASEAN\uc5d0 \ub300\ud55c NICS\uc758 \uc9c1\uc811 \ud22c\uc790\uc561\uc740 130\uc5b5 \ub2ec\ub7ec\ub85c 1989\ub144\uc758 43\uc5b5 \ub2ec\ub7ec\uc5d0 \ube44\ud574 \ubb34\ub824 3.1\ubc30\ub85c \uae09\uc99d\ud558\uc600\ub2e4. \uc774\uac83\uc740 89\ub144\uc758 50\uc5b5 \ub2ec\ub7ec\uc5d0\uc11c 90\ub144\uc758 61\uc5b5 \ub2ec\ub7ec\ub85c \ub298\uc5b4\ub09c \ubbf8\uad6d\uc758 \ud22c\uc790\uc561\uacfc \ube44\uad50\ud558\uba74 NICS\uc758 ASEAN\uc5d0 \ub300\ud55c \ud22c\uc790 \ubd90\uc774 \uc5b4\ub290 \uc815\ub3c4\uc778\uc9c0 \uac00\ub2a0\ud560 \uc218 \uc788\ub2e4. NICS\ub294 \uc544\uc2dc\uc544 \uad6d\uac00 \uc911\uc5d0\uc11c\ub3c4 \ud2b9\ud788 \ubca0\ud2b8\ub0a8\uc5d0 \uad00\uc2ec\uc774 \ub9ce\ub2e4. \ud2b9\ud788 \ud0c0\uc774\uc644\uacfc \ud64d\ucf69 \uc790\ubcf8\uc774 \uac74\uc218\ub85c 36.4%\ub97c \ucc28\uc9c0\ud558\uc5ec 2\uc704\uc778 \uc61b \uc885\uc8fc\uad6d \ud504\ub791\uc2a4\uc758 8.9%\ub97c \ud06c\uac8c \uc55e\uc9c0\ub974\uace0 \uc788\ub2e4. \ud0c0\uc774\uc644\uacfc \ud64d\ucf69 \uae30\uc5c5\uc758 \ud65c\ub3d9\uc774 \uc774\ub300\ub85c \uacc4\uc18d\ub41c\ub2e4\uba74 \ubca0\ud2b8\ub0a8\uc740 \ud654\uad50\uacbd\uc81c\uad8c\uc73c\ub85c \uacb0\uc18d\ub420 \uac00\ub2a5\uc131\uc774 \ud06c\ub2e4. \ucd5c\uadfc \ub4e4\uc5b4 ASEAN \uacbd\uc81c\uad8c\uc758 \uc131\uc7a5\uc774 \ub450\ub4dc\ub7ec\uc9c0\uace0 \uc788\ub2e4. 1980\ub144\ub300 \uc911\ubc18\uc5d0 \uadf9\uc2ec\ud55c \uacbd\uae30\uce68\uccb4\uc5d0 \ube60\uc84c\ub358 \uc774\ub4e4 ASEAN\uad6d\uac00\ub4e4\uc740 \uadf8 \ub4a4 3\uc800\ud604\uc0c1\uacfc \uc544\uc2dc\uc544\uc9c0\uc5ed\ub0b4 \uc0b0\uc5c5\uad6c\uc870 \uc7ac\ud3b8 \ub4f1\uacfc \uac19\uc740 \uad6d\ub0b4\uc678\uc758 \uacbd\uc81c\uc5ec\uac74 \ubcc0\ud654\uc5d0 \ud798\uc785\uc5b4 \uacbd\uc81c\uc131\uc7a5\uc138\ub97c \ub2e8\uae30\uac04 \ub0b4\uc5d0 \ud68c\ubcf5\ud568\uc73c\ub85c\uc368 \uc131\uc7a5\uc758 \ub300\uba85\uc0ac\uaca9\uc778 NICS\uc758 \uc131\uc7a5\ub960\uc744 \uc55e\uc9c0\ub974\ub294 \uc138\uacc4 \ucd5c\uace0\uc758 \uacbd\uc81c\uc131\uc7a5\uc9c0\uc5ed\uc73c\ub85c \ub4f1\uc7a5\ud558\uc600\ub2e4. \uc774\ub7ec\ud55c \ucd94\uc138\ub77c\uba74 90\ub144\ub300 \uc911\ubc18\uc5d0 \uac00\uc11c\ub294 \ud0c0\uc774\u00b7\ub9d0\ub808\uc774\uc2dc\uc544 \ub4f1\uc740 \uc2e0\ud765\uacf5\uc5c5\uad6d\uc73c\ub85c \uc9c4\uc785\ud560 \uac83\uc774 \ud655\uc2e4\uc2dc\ub418\uace0 \uc788\ub2e4. \uc774\uc5d0 \ub530\ub77c ASEAN \uad6d\uac00\uc5d0 \ub300\ud55c \uc138\uacc4 \uac01\uad6d, \ud2b9\ud788 \ud55c\uad6d\uacfc \uc77c\ubcf8 \ub4f1 \uc544\uc2dc\uc544 \uc120\u00b7\uc911\uc9c4\uad6d\uc758 \uad00\uc2ec\uc774 \ud06c\uac8c \ub192\uc544\uac00\uace0 \uc788\ub2e4. \uadf8\ubfd0\ub9cc \uc544\ub2c8\ub77c \ud55c\uad6d\uc740 \uacbd\uc81c\uc131\uc7a5\uc5d0 \uc804\ub825\uc744 \ub2e4\ud558\uace0 \uc788\ub294 \uc911\uad6d \ubc0f \ub0a8\u00b7\ubd81\ud55c \uad00\uacc4\uac1c\uc120\uc758 \uae30\uc6b4\uc5d0 \ud798\uc785\uc5b4 \uacbd\uc81c\uc758 \ud65c\uc131\ud654\uc5d0 \ud798\uc4f0\uace0 \uc788\ub294 \ubd81\ud55c\uc5d0 \ub300\ud55c \uc790\ubcf8\ud22c\uc790\uc640 \uad50\uc5ed\uc5d0\ub3c4 \uc801\uadf9\uc801\uc774\ub2e4.", "paragraph_answer": "\uc624\ub298\ub0a0 \uc544\uc2dc\uc544\uc9c0\uc5ed\uc740 \uc138\uacc4\uc131\uc7a5\uc758 \uc911\uc2ec\uc9c0\ub85c \ubd80\uc0c1\ud558\uace0 \uc788\uc9c0\ub9cc, \uace0\ub3c4\uc131\uc7a5 \uc18d\uc5d0\uc11c\ub3c4 \uc778\ud50c\ub808\uc774\uc158, \uc810\uc810 \ub192\uc544\uc9c0\ub294 \ub300\ubbf8\u00b7\ub300\uc77c \ub9c8\ucc30\ubb38\uc81c, \ud658\uacbd\ubb38\uc81c, \uc2e0\uacbd\uc81c\uad8c \uc870\uc131 \ub4f1\uc758 \uc5ec\ub7ec \uac00\uc9c0 \ubb38\uc81c\ub97c \uc548\uace0 \uc788\ub2e4. 1990\ub144\ub300 \ucd08\uc758 \uc544\uc2dc\uc544\uacbd\uc81c \uc804\ubc18\uc5d0 \uac78\uce5c \ud2b9\uc9d5\uc740 \uc131\uc7a5\ub960\uc758 \uc800\ud558\uc640 \uc778\ud50c\ub808\uc774\uc158\uc774\ub2e4. \uc774\uac83\uc740 1980\ub144\ub300 \ud6c4\ubc18 \uc800(\u4f4e)\uc778\ud50c\ub808\uc774\uc158\ud558\uc5d0\uc11c \ub192\uc740 \uc131\uc7a5\uc744 \ub204\ub838\ub358 \uc2dc\uae30\uc5d0 \ube44\ud558\uba74 \ud06c\uac8c \uc545\ud654\ub41c \uc0c1\ud669\uc774\ub2e4. \uc9c0\ub09c 80\ub144\ub300 2\uc790\ub9bf\uc218\uc758 \uacbd\uc81c\uc131\uc7a5\uc744 \uae30\ub85d\ud558\uba74\uc11c \uc138\uacc4\uc758 \ubd80\ub7ec\uc6c0\uc744 \ubaa8\uc558\ub358 \ub3d9\uc544\uc2dc\uc544 \uc8fc\uc694 \uad6d\uac00\ub4e4\uc740 90\ub144\uc5d0 \ub4e4\uc5b4 \uc131\uc7a5\ub960\uc774 \ub454\ud654\ub41c \uac00\uc6b4\ub370 \uadf8\ub3d9\uc548 \ub178\ucd9c\ub41c \uacbd\uc81c\uc758 \uad6c\uc870\uc801 \ubb38\uc81c\uc810\uc744 \uc2dc\uc815\ud574 \ub098\uac00\uc57c \ud560 \uc0c1\ud669\uc5d0 \uc774\ub974\uace0 \uc788\ub2e4. 92\ub144 \uad6d\uc81c\ud1b5\ud654\uae30\uae08(IMF)\uc774 \ub0b4\ub193\uc740 ' \uc138\uacc4\uacbd\uc81c\uc804\ub9dd ' \ubcf4\uace0\uc11c\uc5d0\uc11c \uc608\uce21\ud55c \ub300\ub85c \ud55c\uad6d, \uc778\ub3c4\ub124\uc2dc\uc544 \ub4f1 \uc544\uc2dc\uc544 \uc2e0\ud765\uacf5\uc5c5\uad6d\ub4e4\uc740 90\ub144\ub300 \uc911\ubc18 \uc774\ud6c4 \uace0\ub3c4\uc131\uc7a5\ucc45 \ucd94\uc9c4 \uacfc\uc815\uc5d0\uc11c \ub204\uc801\ub41c \uc5ec\ub7ec \uac00\uc9c0 \ubd80\uc791\uc6a9\ub4e4\uc758 \ubd84\ucd9c\ub85c \uc778\ud558\uc5ec \uacbd\uc81c\uc801 \uc5b4\ub824\uc6c0\uc5d0 \ube60\uc9c0\uae30 \uc2dc\uc791\ud588\ub2e4. \uadf8\ub9ac\uace0 \ub9c8\uce68\ub0b4 1997\ub144 \ud558\ubc18\uae30 \uc774\ud6c4 \uc544\uc2dc\uc544 \uac01\uad6d\uc740 \ub2ec\ub7ec \ubd80\uc871\uc73c\ub85c \ucc44\ubb34\ubd88\uc774\ud589\uc758 \uc704\uae30\uc5d0 \ucc98\ud574 IMF \ub4f1 \uad6d\uc81c\uae08\uc735\uad00\ub828 \uae30\uad00\ub4e4\ub85c\ubd80\ud130 \uae34\uae09 \uc218\ud608\uc744 \ubc1b\uc544\uc57c\ub9cc \ud560 \uc0c1\ud669\uc5d0 \uc774\ub974\ub800\ub2e4. \uc774\ub7ec\ud55c \uc544\uc2dc\uc544\uc758 \uacbd\uc81c\uc704\uae30\uc5d0 \ub300\ud574\uc11c IMF\ub294 \uc815\ubd80\uc758 \uacfc\ub3c4\ud55c \uc2dc\uc7a5\uac1c\uc785, \ubd88\ud569\ub9ac\ud55c \ub178\ub3d9\uc815\ucc45, \uae08\uc735\uae30\uad00\uc758 \ube44\ud569\ub9ac\uc801 \uc6b4\uc601 \ub4f1\uc744 \uaf3d\uace0 \uc774\uc758 \uc2dc\uc815\uc744 \uc544\uc2dc\uc544 \uac01\uad6d\uc5d0 \uac15\ub825\ud558\uac8c \uc694\uad6c\ud588\ub2e4.\ucd5c\uadfc\uc758 \uc544\uc2dc\uc544\uc9c0\uc5ed \ubb34\uc5ed\ub3d9\ud5a5\uc744 \ubcf4\uba74, \ub300\ubbf8 \uc218\ucd9c\uc774 \uc904\uace0 \uc544\uc2dc\uc544 \uc5ed\ub0b4\ubb34\uc5ed\uc774 \ud655\ub300\ub418\ub294 \uacbd\ud5a5\uc774 \uc788\ub294\ub370, \uc774\uac83\uc740 \uc544\uc2dc\uc544 \uac01\uad6d\uc758 \uc218\ucd9c\uc2dc\uc7a5\uc744 \ub2e4\ubcc0\ud654\ud558\uace0 \uc678\uc218\uc8fc\ub3c4\ub85c\ubd80\ud130 \ub0b4\uc218\uc8fc\ub3c4\uc758 \uacbd\uc81c\uc131\uc7a5\uc73c\ub85c\uc758 \uc804\ud658\uc744 \uc2dc\ub3c4\ud588\uae30 \ub54c\ubb38\uc774\ub2e4. \uc5ed\ub0b4\ubb34\uc5ed\uc758 \ud655\ub300\uc640 \ud568\uaed8 \uc774 \uc9c0\uc5ed \ub0b4\uc758 \ubb34\uc5ed\ubd88\uade0\ud615\ub3c4 \ub450\ub4dc\ub7ec\uc9c4\ub2e4. \uc608\ub97c \ub4e4\uba74, \uc2e0\ud765\uacf5\uc5c5\uad6d\uac00\uad70(NICS)\uc758 \ub300\uc77c \ubb34\uc5ed\uc801\uc790\uac00 1985\ub144\uc758 130\uc5b5 \ub2ec\ub7ec\uc5d0\uc11c 91\ub144\uc5d0\ub294 315\uc5b5 \ub2ec\ub7ec\ub85c 2.3\ubc30 \ub298\uc5b4\ub0ac\ub2e4. \uc544\uc2dc\uc544\uc9c0\uc5ed\uc758 \uc9c1\uc811\ud22c\uc790\ubd80\ubb38\uc5d0\uc11c NICS \uae30\uc5c5\uc774 \uc77c\ubcf8 \uae30\uc5c5\ubcf4\ub2e4 \ud6e8\uc52c \uc9c4\ucd9c \uc18d\ub3c4\uac00 \ube60\ub974\ub2e4. 91\ub144\uc758 ASEAN\uc5d0 \ub300\ud55c NICS\uc758 \uc9c1\uc811 \ud22c\uc790\uc561\uc740 130\uc5b5 \ub2ec\ub7ec\ub85c 1989\ub144\uc758 43\uc5b5 \ub2ec\ub7ec\uc5d0 \ube44\ud574 \ubb34\ub824 3.1\ubc30\ub85c \uae09\uc99d\ud558\uc600\ub2e4. \uc774\uac83\uc740 89\ub144\uc758 50\uc5b5 \ub2ec\ub7ec\uc5d0\uc11c 90\ub144\uc758 61\uc5b5 \ub2ec\ub7ec\ub85c \ub298\uc5b4\ub09c \ubbf8\uad6d\uc758 \ud22c\uc790\uc561\uacfc \ube44\uad50\ud558\uba74 NICS\uc758 ASEAN\uc5d0 \ub300\ud55c \ud22c\uc790 \ubd90\uc774 \uc5b4\ub290 \uc815\ub3c4\uc778\uc9c0 \uac00\ub2a0\ud560 \uc218 \uc788\ub2e4. NICS\ub294 \uc544\uc2dc\uc544 \uad6d\uac00 \uc911\uc5d0\uc11c\ub3c4 \ud2b9\ud788 \ubca0\ud2b8\ub0a8\uc5d0 \uad00\uc2ec\uc774 \ub9ce\ub2e4. \ud2b9\ud788 \ud0c0\uc774\uc644\uacfc \ud64d\ucf69 \uc790\ubcf8\uc774 \uac74\uc218\ub85c 36.4%\ub97c \ucc28\uc9c0\ud558\uc5ec 2\uc704\uc778 \uc61b \uc885\uc8fc\uad6d \ud504\ub791\uc2a4\uc758 8.9%\ub97c \ud06c\uac8c \uc55e\uc9c0\ub974\uace0 \uc788\ub2e4. \ud0c0\uc774\uc644\uacfc \ud64d\ucf69 \uae30\uc5c5\uc758 \ud65c\ub3d9\uc774 \uc774\ub300\ub85c \uacc4\uc18d\ub41c\ub2e4\uba74 \ubca0\ud2b8\ub0a8\uc740 \ud654\uad50\uacbd\uc81c\uad8c\uc73c\ub85c \uacb0\uc18d\ub420 \uac00\ub2a5\uc131\uc774 \ud06c\ub2e4. \ucd5c\uadfc \ub4e4\uc5b4 ASEAN \uacbd\uc81c\uad8c\uc758 \uc131\uc7a5\uc774 \ub450\ub4dc\ub7ec\uc9c0\uace0 \uc788\ub2e4. 1980\ub144\ub300 \uc911\ubc18\uc5d0 \uadf9\uc2ec\ud55c \uacbd\uae30\uce68\uccb4\uc5d0 \ube60\uc84c\ub358 \uc774\ub4e4 ASEAN\uad6d\uac00\ub4e4\uc740 \uadf8 \ub4a4 3\uc800\ud604\uc0c1\uacfc \uc544\uc2dc\uc544\uc9c0\uc5ed\ub0b4 \uc0b0\uc5c5\uad6c\uc870 \uc7ac\ud3b8 \ub4f1\uacfc \uac19\uc740 \uad6d\ub0b4\uc678\uc758 \uacbd\uc81c\uc5ec\uac74 \ubcc0\ud654\uc5d0 \ud798\uc785\uc5b4 \uacbd\uc81c\uc131\uc7a5\uc138\ub97c \ub2e8\uae30\uac04 \ub0b4\uc5d0 \ud68c\ubcf5\ud568\uc73c\ub85c\uc368 \uc131\uc7a5\uc758 \ub300\uba85\uc0ac\uaca9\uc778 NICS\uc758 \uc131\uc7a5\ub960\uc744 \uc55e\uc9c0\ub974\ub294 \uc138\uacc4 \ucd5c\uace0\uc758 \uacbd\uc81c\uc131\uc7a5\uc9c0\uc5ed\uc73c\ub85c \ub4f1\uc7a5\ud558\uc600\ub2e4. \uc774\ub7ec\ud55c \ucd94\uc138\ub77c\uba74 90\ub144\ub300 \uc911\ubc18\uc5d0 \uac00\uc11c\ub294 \ud0c0\uc774\u00b7\ub9d0\ub808\uc774\uc2dc\uc544 \ub4f1\uc740 \uc2e0\ud765\uacf5\uc5c5\uad6d\uc73c\ub85c \uc9c4\uc785\ud560 \uac83\uc774 \ud655\uc2e4\uc2dc\ub418\uace0 \uc788\ub2e4. \uc774\uc5d0 \ub530\ub77c ASEAN \uad6d\uac00\uc5d0 \ub300\ud55c \uc138\uacc4 \uac01\uad6d, \ud2b9\ud788 \ud55c\uad6d\uacfc \uc77c\ubcf8 \ub4f1 \uc544\uc2dc\uc544 \uc120\u00b7\uc911\uc9c4\uad6d\uc758 \uad00\uc2ec\uc774 \ud06c\uac8c \ub192\uc544\uac00\uace0 \uc788\ub2e4. \uadf8\ubfd0\ub9cc \uc544\ub2c8\ub77c \ud55c\uad6d\uc740 \uacbd\uc81c\uc131\uc7a5\uc5d0 \uc804\ub825\uc744 \ub2e4\ud558\uace0 \uc788\ub294 \uc911\uad6d \ubc0f \ub0a8\u00b7\ubd81\ud55c \uad00\uacc4\uac1c\uc120\uc758 \uae30\uc6b4\uc5d0 \ud798\uc785\uc5b4 \uacbd\uc81c\uc758 \ud65c\uc131\ud654\uc5d0 \ud798\uc4f0\uace0 \uc788\ub294 \ubd81\ud55c\uc5d0 \ub300\ud55c \uc790\ubcf8\ud22c\uc790\uc640 \uad50\uc5ed\uc5d0\ub3c4 \uc801\uadf9\uc801\uc774\ub2e4.", "sentence_answer": "92\ub144 \uad6d\uc81c\ud1b5\ud654\uae30\uae08(IMF)\uc774 \ub0b4\ub193\uc740 ' \uc138\uacc4\uacbd\uc81c\uc804\ub9dd '", "paragraph_id": "6547341-1-0", "paragraph_question": "question: 1992\ub144 \uad6d\uc81c\ud1b5\ud654\uae30\uae08\uc778 IMF\uac00 \ub0b4\ub193\uc740 \ubcf4\uace0\uc11c\uc758 \uc774\ub984\uc740?, context: \uc624\ub298\ub0a0 \uc544\uc2dc\uc544\uc9c0\uc5ed\uc740 \uc138\uacc4\uc131\uc7a5\uc758 \uc911\uc2ec\uc9c0\ub85c \ubd80\uc0c1\ud558\uace0 \uc788\uc9c0\ub9cc, \uace0\ub3c4\uc131\uc7a5 \uc18d\uc5d0\uc11c\ub3c4 \uc778\ud50c\ub808\uc774\uc158, \uc810\uc810 \ub192\uc544\uc9c0\ub294 \ub300\ubbf8\u00b7\ub300\uc77c \ub9c8\ucc30\ubb38\uc81c, \ud658\uacbd\ubb38\uc81c, \uc2e0\uacbd\uc81c\uad8c \uc870\uc131 \ub4f1\uc758 \uc5ec\ub7ec \uac00\uc9c0 \ubb38\uc81c\ub97c \uc548\uace0 \uc788\ub2e4. 1990\ub144\ub300 \ucd08\uc758 \uc544\uc2dc\uc544\uacbd\uc81c \uc804\ubc18\uc5d0 \uac78\uce5c \ud2b9\uc9d5\uc740 \uc131\uc7a5\ub960\uc758 \uc800\ud558\uc640 \uc778\ud50c\ub808\uc774\uc158\uc774\ub2e4. \uc774\uac83\uc740 1980\ub144\ub300 \ud6c4\ubc18 \uc800(\u4f4e)\uc778\ud50c\ub808\uc774\uc158\ud558\uc5d0\uc11c \ub192\uc740 \uc131\uc7a5\uc744 \ub204\ub838\ub358 \uc2dc\uae30\uc5d0 \ube44\ud558\uba74 \ud06c\uac8c \uc545\ud654\ub41c \uc0c1\ud669\uc774\ub2e4. \uc9c0\ub09c 80\ub144\ub300 2\uc790\ub9bf\uc218\uc758 \uacbd\uc81c\uc131\uc7a5\uc744 \uae30\ub85d\ud558\uba74\uc11c \uc138\uacc4\uc758 \ubd80\ub7ec\uc6c0\uc744 \ubaa8\uc558\ub358 \ub3d9\uc544\uc2dc\uc544 \uc8fc\uc694 \uad6d\uac00\ub4e4\uc740 90\ub144\uc5d0 \ub4e4\uc5b4 \uc131\uc7a5\ub960\uc774 \ub454\ud654\ub41c \uac00\uc6b4\ub370 \uadf8\ub3d9\uc548 \ub178\ucd9c\ub41c \uacbd\uc81c\uc758 \uad6c\uc870\uc801 \ubb38\uc81c\uc810\uc744 \uc2dc\uc815\ud574 \ub098\uac00\uc57c \ud560 \uc0c1\ud669\uc5d0 \uc774\ub974\uace0 \uc788\ub2e4. 92\ub144 \uad6d\uc81c\ud1b5\ud654\uae30\uae08(IMF)\uc774 \ub0b4\ub193\uc740 '\uc138\uacc4\uacbd\uc81c\uc804\ub9dd' \ubcf4\uace0\uc11c\uc5d0\uc11c \uc608\uce21\ud55c \ub300\ub85c \ud55c\uad6d, \uc778\ub3c4\ub124\uc2dc\uc544 \ub4f1 \uc544\uc2dc\uc544 \uc2e0\ud765\uacf5\uc5c5\uad6d\ub4e4\uc740 90\ub144\ub300 \uc911\ubc18 \uc774\ud6c4 \uace0\ub3c4\uc131\uc7a5\ucc45 \ucd94\uc9c4 \uacfc\uc815\uc5d0\uc11c \ub204\uc801\ub41c \uc5ec\ub7ec \uac00\uc9c0 \ubd80\uc791\uc6a9\ub4e4\uc758 \ubd84\ucd9c\ub85c \uc778\ud558\uc5ec \uacbd\uc81c\uc801 \uc5b4\ub824\uc6c0\uc5d0 \ube60\uc9c0\uae30 \uc2dc\uc791\ud588\ub2e4. \uadf8\ub9ac\uace0 \ub9c8\uce68\ub0b4 1997\ub144 \ud558\ubc18\uae30 \uc774\ud6c4 \uc544\uc2dc\uc544 \uac01\uad6d\uc740 \ub2ec\ub7ec \ubd80\uc871\uc73c\ub85c \ucc44\ubb34\ubd88\uc774\ud589\uc758 \uc704\uae30\uc5d0 \ucc98\ud574 IMF \ub4f1 \uad6d\uc81c\uae08\uc735\uad00\ub828 \uae30\uad00\ub4e4\ub85c\ubd80\ud130 \uae34\uae09 \uc218\ud608\uc744 \ubc1b\uc544\uc57c\ub9cc \ud560 \uc0c1\ud669\uc5d0 \uc774\ub974\ub800\ub2e4. \uc774\ub7ec\ud55c \uc544\uc2dc\uc544\uc758 \uacbd\uc81c\uc704\uae30\uc5d0 \ub300\ud574\uc11c IMF\ub294 \uc815\ubd80\uc758 \uacfc\ub3c4\ud55c \uc2dc\uc7a5\uac1c\uc785, \ubd88\ud569\ub9ac\ud55c \ub178\ub3d9\uc815\ucc45, \uae08\uc735\uae30\uad00\uc758 \ube44\ud569\ub9ac\uc801 \uc6b4\uc601 \ub4f1\uc744 \uaf3d\uace0 \uc774\uc758 \uc2dc\uc815\uc744 \uc544\uc2dc\uc544 \uac01\uad6d\uc5d0 \uac15\ub825\ud558\uac8c \uc694\uad6c\ud588\ub2e4.\ucd5c\uadfc\uc758 \uc544\uc2dc\uc544\uc9c0\uc5ed \ubb34\uc5ed\ub3d9\ud5a5\uc744 \ubcf4\uba74, \ub300\ubbf8 \uc218\ucd9c\uc774 \uc904\uace0 \uc544\uc2dc\uc544 \uc5ed\ub0b4\ubb34\uc5ed\uc774 \ud655\ub300\ub418\ub294 \uacbd\ud5a5\uc774 \uc788\ub294\ub370, \uc774\uac83\uc740 \uc544\uc2dc\uc544 \uac01\uad6d\uc758 \uc218\ucd9c\uc2dc\uc7a5\uc744 \ub2e4\ubcc0\ud654\ud558\uace0 \uc678\uc218\uc8fc\ub3c4\ub85c\ubd80\ud130 \ub0b4\uc218\uc8fc\ub3c4\uc758 \uacbd\uc81c\uc131\uc7a5\uc73c\ub85c\uc758 \uc804\ud658\uc744 \uc2dc\ub3c4\ud588\uae30 \ub54c\ubb38\uc774\ub2e4. \uc5ed\ub0b4\ubb34\uc5ed\uc758 \ud655\ub300\uc640 \ud568\uaed8 \uc774 \uc9c0\uc5ed \ub0b4\uc758 \ubb34\uc5ed\ubd88\uade0\ud615\ub3c4 \ub450\ub4dc\ub7ec\uc9c4\ub2e4. \uc608\ub97c \ub4e4\uba74, \uc2e0\ud765\uacf5\uc5c5\uad6d\uac00\uad70(NICS)\uc758 \ub300\uc77c \ubb34\uc5ed\uc801\uc790\uac00 1985\ub144\uc758 130\uc5b5 \ub2ec\ub7ec\uc5d0\uc11c 91\ub144\uc5d0\ub294 315\uc5b5 \ub2ec\ub7ec\ub85c 2.3\ubc30 \ub298\uc5b4\ub0ac\ub2e4. \uc544\uc2dc\uc544\uc9c0\uc5ed\uc758 \uc9c1\uc811\ud22c\uc790\ubd80\ubb38\uc5d0\uc11c NICS \uae30\uc5c5\uc774 \uc77c\ubcf8 \uae30\uc5c5\ubcf4\ub2e4 \ud6e8\uc52c \uc9c4\ucd9c \uc18d\ub3c4\uac00 \ube60\ub974\ub2e4. 91\ub144\uc758 ASEAN\uc5d0 \ub300\ud55c NICS\uc758 \uc9c1\uc811 \ud22c\uc790\uc561\uc740 130\uc5b5 \ub2ec\ub7ec\ub85c 1989\ub144\uc758 43\uc5b5 \ub2ec\ub7ec\uc5d0 \ube44\ud574 \ubb34\ub824 3.1\ubc30\ub85c \uae09\uc99d\ud558\uc600\ub2e4. \uc774\uac83\uc740 89\ub144\uc758 50\uc5b5 \ub2ec\ub7ec\uc5d0\uc11c 90\ub144\uc758 61\uc5b5 \ub2ec\ub7ec\ub85c \ub298\uc5b4\ub09c \ubbf8\uad6d\uc758 \ud22c\uc790\uc561\uacfc \ube44\uad50\ud558\uba74 NICS\uc758 ASEAN\uc5d0 \ub300\ud55c \ud22c\uc790 \ubd90\uc774 \uc5b4\ub290 \uc815\ub3c4\uc778\uc9c0 \uac00\ub2a0\ud560 \uc218 \uc788\ub2e4. NICS\ub294 \uc544\uc2dc\uc544 \uad6d\uac00 \uc911\uc5d0\uc11c\ub3c4 \ud2b9\ud788 \ubca0\ud2b8\ub0a8\uc5d0 \uad00\uc2ec\uc774 \ub9ce\ub2e4. \ud2b9\ud788 \ud0c0\uc774\uc644\uacfc \ud64d\ucf69 \uc790\ubcf8\uc774 \uac74\uc218\ub85c 36.4%\ub97c \ucc28\uc9c0\ud558\uc5ec 2\uc704\uc778 \uc61b \uc885\uc8fc\uad6d \ud504\ub791\uc2a4\uc758 8.9%\ub97c \ud06c\uac8c \uc55e\uc9c0\ub974\uace0 \uc788\ub2e4. \ud0c0\uc774\uc644\uacfc \ud64d\ucf69 \uae30\uc5c5\uc758 \ud65c\ub3d9\uc774 \uc774\ub300\ub85c \uacc4\uc18d\ub41c\ub2e4\uba74 \ubca0\ud2b8\ub0a8\uc740 \ud654\uad50\uacbd\uc81c\uad8c\uc73c\ub85c \uacb0\uc18d\ub420 \uac00\ub2a5\uc131\uc774 \ud06c\ub2e4. \ucd5c\uadfc \ub4e4\uc5b4 ASEAN \uacbd\uc81c\uad8c\uc758 \uc131\uc7a5\uc774 \ub450\ub4dc\ub7ec\uc9c0\uace0 \uc788\ub2e4. 1980\ub144\ub300 \uc911\ubc18\uc5d0 \uadf9\uc2ec\ud55c \uacbd\uae30\uce68\uccb4\uc5d0 \ube60\uc84c\ub358 \uc774\ub4e4 ASEAN\uad6d\uac00\ub4e4\uc740 \uadf8 \ub4a4 3\uc800\ud604\uc0c1\uacfc \uc544\uc2dc\uc544\uc9c0\uc5ed\ub0b4 \uc0b0\uc5c5\uad6c\uc870 \uc7ac\ud3b8 \ub4f1\uacfc \uac19\uc740 \uad6d\ub0b4\uc678\uc758 \uacbd\uc81c\uc5ec\uac74 \ubcc0\ud654\uc5d0 \ud798\uc785\uc5b4 \uacbd\uc81c\uc131\uc7a5\uc138\ub97c \ub2e8\uae30\uac04 \ub0b4\uc5d0 \ud68c\ubcf5\ud568\uc73c\ub85c\uc368 \uc131\uc7a5\uc758 \ub300\uba85\uc0ac\uaca9\uc778 NICS\uc758 \uc131\uc7a5\ub960\uc744 \uc55e\uc9c0\ub974\ub294 \uc138\uacc4 \ucd5c\uace0\uc758 \uacbd\uc81c\uc131\uc7a5\uc9c0\uc5ed\uc73c\ub85c \ub4f1\uc7a5\ud558\uc600\ub2e4. \uc774\ub7ec\ud55c \ucd94\uc138\ub77c\uba74 90\ub144\ub300 \uc911\ubc18\uc5d0 \uac00\uc11c\ub294 \ud0c0\uc774\u00b7\ub9d0\ub808\uc774\uc2dc\uc544 \ub4f1\uc740 \uc2e0\ud765\uacf5\uc5c5\uad6d\uc73c\ub85c \uc9c4\uc785\ud560 \uac83\uc774 \ud655\uc2e4\uc2dc\ub418\uace0 \uc788\ub2e4. \uc774\uc5d0 \ub530\ub77c ASEAN \uad6d\uac00\uc5d0 \ub300\ud55c \uc138\uacc4 \uac01\uad6d, \ud2b9\ud788 \ud55c\uad6d\uacfc \uc77c\ubcf8 \ub4f1 \uc544\uc2dc\uc544 \uc120\u00b7\uc911\uc9c4\uad6d\uc758 \uad00\uc2ec\uc774 \ud06c\uac8c \ub192\uc544\uac00\uace0 \uc788\ub2e4. \uadf8\ubfd0\ub9cc \uc544\ub2c8\ub77c \ud55c\uad6d\uc740 \uacbd\uc81c\uc131\uc7a5\uc5d0 \uc804\ub825\uc744 \ub2e4\ud558\uace0 \uc788\ub294 \uc911\uad6d \ubc0f \ub0a8\u00b7\ubd81\ud55c \uad00\uacc4\uac1c\uc120\uc758 \uae30\uc6b4\uc5d0 \ud798\uc785\uc5b4 \uacbd\uc81c\uc758 \ud65c\uc131\ud654\uc5d0 \ud798\uc4f0\uace0 \uc788\ub294 \ubd81\ud55c\uc5d0 \ub300\ud55c \uc790\ubcf8\ud22c\uc790\uc640 \uad50\uc5ed\uc5d0\ub3c4 \uc801\uadf9\uc801\uc774\ub2e4."} {"question": "\ud398\ub974\uc2dc\uc544\uc640 \uba54\ub514\uc544\uc758 \uc624\ub79c \ud1b5\uce58\uae30\uac04 \ub3d9\uc548 \uc544\ub974\uba54\ub2c8\uc544\uc5d0 \uc815\ucc29\ud574 \uc788\uc5c8\ub358 \uc8fc\ubbfc\ub4e4\uc740?", "paragraph": "\uc544\ub974\uba54\ub2c8\uc544 \uc815\ubcf5 \uc774\uc804\uc758 \ucfe0\ub77c\uc758 \uc6b0\uce21 \uc81c\ubc29\uc5d0 \uc788\ub358 \uc601\ud1a0\ub4e4\uc758 \ud1a0\uc791 \uc778\uad6c\ub294 \ub2e4\uc591\ud55c \uc6d0\uc8fc\ubbfc\ub4e4\ub85c \uad6c\uc131\ub418\uc5c8\ub2e4. \uace0\ub300 \uc5f0\ubcf4\ub294 \uc54c\ud2b8\uc0ac\ud750, \uc6b0\ud2f0\ud06c\uc758 \uc9c0\uc5ed\ub4e4\uc744 \ud3ec\ud568\ud558\ub294 \uadf8 \uad6c\uc5ed\ub4e4\uc5d0 \uac70\uc8fc\ud588\ub358 \uc5ec\ub7ec \ubbfc\uc871\ub4e4\uc758 \uc774\ub984\uc744 \uc81c\uacf5\ud55c\ub2e4. \ubbfc\uc871\uba85\uc740 \uc6b0\ud2f0\uc871, \ubbf8\ud0a4\uc871, \uce74\uc2a4\ud53c\uc871, \uac00\ub974\uac00\ub9ac\uc544\uc871, \uc0ac\uce74\uc138\ub2c8\uc871, \uac94\ub9ac\uc871, \uc624\ub514\uc871, \ub8e8\ud398\ub2c8\uc871, \ubc1c\ub77c\uc2a4(\uc545)\uce74\ub2c8\uc871, \ud398\ub974\uc2dc\uc544\uc778 \uadf8\ub9ac\uace0 \ud30c\ub974\ub77c\uc2dc\uc544\uc778\uc774\ub2e4. \ub85c\ubca0\ub974\ud2b8 H. \ud734\uc13c\uc740 \uadf8 \ubd80\uc871\ub4e4\uc774 \"\uc544\ub974\uba54\ub2c8\uc544\uc758 \uae30\uc6d0\uc784\uc740 \ud655\uc2e4\uce58 \uc54a\ub2e4.\"\uace0 \ud588\uc73c\uba70, \"\ud655\uc2e4\ud788 \uc774\ub780\uc778\ub4e4\uc774 \ud398\ub974\uc2dc\uc544\uc640 \uba54\ub514\uc544\uc758 \uc624\ub79c \ud1b5\uce58\uae30\uac04 \ub3d9\uc548 \uadf8 \uc9c0\uc5ed\uc5d0 \uc815\ucc29\ud588\uc74c\uc774 \ubd84\uba85\ud558\uace0, \ub300\ubd80\ubd84\uc758 \ud1a0\ucc29\ubbfc\ub4e4\uc740 \uc778\ub3c4-\uc720\ub7fd\uc778 \uc870\ucc28\ub3c4 \uc544\ub2d8\uc774 \ud2c0\ub9bc\uc5c6\ub2e4.\" \uace0 \ud558\uc600\ub2e4. \uadf8\ub294 \ucfe0\ub77c\uc758 \uc6b0\uce21 \uc81c\ubc29\uc758 \ub2e4\uc591\ud55c \ubbfc\uc871\ub4e4\uc774 \"\ub9e4\uc6b0 \uc544\ub974\uba54\ub2c8\uc544\ud654 \ub418\uc5c8\uc73c\uba70 \uadf8\ub4e4\uc911 \ub9ce\uc740 \uc0ac\ub78c\ub4e4\uc774 \uc0ac\uc2e4\uc0c1 \uc544\ub974\uba54\ub2c8\uc544\uc778 \uc774\ub77c\ud568\uc740 \uadf8 \ucc28\uc81c\ub85c\ub3c4(per se) \uc758\uc2ec\ud560 \uc218\uc5c6\ub2e4.\"\uace0 \ud3c9\uac00\ud588\ub2e4. \ucfe0\ub77c\uc758 \uc6b0\uce21 \uc81c\ubc29\uc744 387\ub144\uc5d0 \uce85\uce74\uc2a4 \uc54c\ubc14\ub2c8\uc544\uac00 \ucc28\uc9c0\ud588\uc744 \ub2f9\uc2dc, \uadf8 \uc9c0\uc5ed\uc758 \ub9ce\uc740 \uc0ac\ub78c\ub4e4\uc740 \uc5ec\uc804\ud788 \ub3c5\ud2b9\ud55c \ubbfc\uc871\uc801 \ubcf8\uc9c8\uc758 \uadfc\uac70\uac00 \ub418\uace0 \uc788\ub2e4.", "answer": "\uc774\ub780\uc778", "sentence": "\"\ud655\uc2e4\ud788 \uc774\ub780\uc778 \ub4e4\uc774 \ud398\ub974\uc2dc\uc544\uc640 \uba54\ub514\uc544\uc758 \uc624\ub79c \ud1b5\uce58\uae30\uac04 \ub3d9\uc548 \uadf8 \uc9c0\uc5ed\uc5d0 \uc815\ucc29\ud588\uc74c\uc774 \ubd84\uba85\ud558\uace0, \ub300\ubd80\ubd84\uc758 \ud1a0\ucc29\ubbfc\ub4e4\uc740 \uc778\ub3c4-\uc720\ub7fd\uc778 \uc870\ucc28\ub3c4 \uc544\ub2d8\uc774 \ud2c0\ub9bc\uc5c6\ub2e4.", "paragraph_sentence": "\uc544\ub974\uba54\ub2c8\uc544 \uc815\ubcf5 \uc774\uc804\uc758 \ucfe0\ub77c\uc758 \uc6b0\uce21 \uc81c\ubc29\uc5d0 \uc788\ub358 \uc601\ud1a0\ub4e4\uc758 \ud1a0\uc791 \uc778\uad6c\ub294 \ub2e4\uc591\ud55c \uc6d0\uc8fc\ubbfc\ub4e4\ub85c \uad6c\uc131\ub418\uc5c8\ub2e4. \uace0\ub300 \uc5f0\ubcf4\ub294 \uc54c\ud2b8\uc0ac\ud750, \uc6b0\ud2f0\ud06c\uc758 \uc9c0\uc5ed\ub4e4\uc744 \ud3ec\ud568\ud558\ub294 \uadf8 \uad6c\uc5ed\ub4e4\uc5d0 \uac70\uc8fc\ud588\ub358 \uc5ec\ub7ec \ubbfc\uc871\ub4e4\uc758 \uc774\ub984\uc744 \uc81c\uacf5\ud55c\ub2e4. \ubbfc\uc871\uba85\uc740 \uc6b0\ud2f0\uc871, \ubbf8\ud0a4\uc871, \uce74\uc2a4\ud53c\uc871, \uac00\ub974\uac00\ub9ac\uc544\uc871, \uc0ac\uce74\uc138\ub2c8\uc871, \uac94\ub9ac\uc871, \uc624\ub514\uc871, \ub8e8\ud398\ub2c8\uc871, \ubc1c\ub77c\uc2a4(\uc545)\uce74\ub2c8\uc871, \ud398\ub974\uc2dc\uc544\uc778 \uadf8\ub9ac\uace0 \ud30c\ub974\ub77c\uc2dc\uc544\uc778\uc774\ub2e4. \ub85c\ubca0\ub974\ud2b8 H. \ud734\uc13c\uc740 \uadf8 \ubd80\uc871\ub4e4\uc774 \"\uc544\ub974\uba54\ub2c8\uc544\uc758 \uae30\uc6d0\uc784\uc740 \ud655\uc2e4\uce58 \uc54a\ub2e4. \"\uace0 \ud588\uc73c\uba70, \"\ud655\uc2e4\ud788 \uc774\ub780\uc778 \ub4e4\uc774 \ud398\ub974\uc2dc\uc544\uc640 \uba54\ub514\uc544\uc758 \uc624\ub79c \ud1b5\uce58\uae30\uac04 \ub3d9\uc548 \uadf8 \uc9c0\uc5ed\uc5d0 \uc815\ucc29\ud588\uc74c\uc774 \ubd84\uba85\ud558\uace0, \ub300\ubd80\ubd84\uc758 \ud1a0\ucc29\ubbfc\ub4e4\uc740 \uc778\ub3c4-\uc720\ub7fd\uc778 \uc870\ucc28\ub3c4 \uc544\ub2d8\uc774 \ud2c0\ub9bc\uc5c6\ub2e4. \" \uace0 \ud558\uc600\ub2e4. \uadf8\ub294 \ucfe0\ub77c\uc758 \uc6b0\uce21 \uc81c\ubc29\uc758 \ub2e4\uc591\ud55c \ubbfc\uc871\ub4e4\uc774 \"\ub9e4\uc6b0 \uc544\ub974\uba54\ub2c8\uc544\ud654 \ub418\uc5c8\uc73c\uba70 \uadf8\ub4e4\uc911 \ub9ce\uc740 \uc0ac\ub78c\ub4e4\uc774 \uc0ac\uc2e4\uc0c1 \uc544\ub974\uba54\ub2c8\uc544\uc778 \uc774\ub77c\ud568\uc740 \uadf8 \ucc28\uc81c\ub85c\ub3c4(per se) \uc758\uc2ec\ud560 \uc218\uc5c6\ub2e4. \"\uace0 \ud3c9\uac00\ud588\ub2e4. \ucfe0\ub77c\uc758 \uc6b0\uce21 \uc81c\ubc29\uc744 387\ub144\uc5d0 \uce85\uce74\uc2a4 \uc54c\ubc14\ub2c8\uc544\uac00 \ucc28\uc9c0\ud588\uc744 \ub2f9\uc2dc, \uadf8 \uc9c0\uc5ed\uc758 \ub9ce\uc740 \uc0ac\ub78c\ub4e4\uc740 \uc5ec\uc804\ud788 \ub3c5\ud2b9\ud55c \ubbfc\uc871\uc801 \ubcf8\uc9c8\uc758 \uadfc\uac70\uac00 \ub418\uace0 \uc788\ub2e4.", "paragraph_answer": "\uc544\ub974\uba54\ub2c8\uc544 \uc815\ubcf5 \uc774\uc804\uc758 \ucfe0\ub77c\uc758 \uc6b0\uce21 \uc81c\ubc29\uc5d0 \uc788\ub358 \uc601\ud1a0\ub4e4\uc758 \ud1a0\uc791 \uc778\uad6c\ub294 \ub2e4\uc591\ud55c \uc6d0\uc8fc\ubbfc\ub4e4\ub85c \uad6c\uc131\ub418\uc5c8\ub2e4. \uace0\ub300 \uc5f0\ubcf4\ub294 \uc54c\ud2b8\uc0ac\ud750, \uc6b0\ud2f0\ud06c\uc758 \uc9c0\uc5ed\ub4e4\uc744 \ud3ec\ud568\ud558\ub294 \uadf8 \uad6c\uc5ed\ub4e4\uc5d0 \uac70\uc8fc\ud588\ub358 \uc5ec\ub7ec \ubbfc\uc871\ub4e4\uc758 \uc774\ub984\uc744 \uc81c\uacf5\ud55c\ub2e4. \ubbfc\uc871\uba85\uc740 \uc6b0\ud2f0\uc871, \ubbf8\ud0a4\uc871, \uce74\uc2a4\ud53c\uc871, \uac00\ub974\uac00\ub9ac\uc544\uc871, \uc0ac\uce74\uc138\ub2c8\uc871, \uac94\ub9ac\uc871, \uc624\ub514\uc871, \ub8e8\ud398\ub2c8\uc871, \ubc1c\ub77c\uc2a4(\uc545)\uce74\ub2c8\uc871, \ud398\ub974\uc2dc\uc544\uc778 \uadf8\ub9ac\uace0 \ud30c\ub974\ub77c\uc2dc\uc544\uc778\uc774\ub2e4. \ub85c\ubca0\ub974\ud2b8 H. \ud734\uc13c\uc740 \uadf8 \ubd80\uc871\ub4e4\uc774 \"\uc544\ub974\uba54\ub2c8\uc544\uc758 \uae30\uc6d0\uc784\uc740 \ud655\uc2e4\uce58 \uc54a\ub2e4.\"\uace0 \ud588\uc73c\uba70, \"\ud655\uc2e4\ud788 \uc774\ub780\uc778 \ub4e4\uc774 \ud398\ub974\uc2dc\uc544\uc640 \uba54\ub514\uc544\uc758 \uc624\ub79c \ud1b5\uce58\uae30\uac04 \ub3d9\uc548 \uadf8 \uc9c0\uc5ed\uc5d0 \uc815\ucc29\ud588\uc74c\uc774 \ubd84\uba85\ud558\uace0, \ub300\ubd80\ubd84\uc758 \ud1a0\ucc29\ubbfc\ub4e4\uc740 \uc778\ub3c4-\uc720\ub7fd\uc778 \uc870\ucc28\ub3c4 \uc544\ub2d8\uc774 \ud2c0\ub9bc\uc5c6\ub2e4.\" \uace0 \ud558\uc600\ub2e4. \uadf8\ub294 \ucfe0\ub77c\uc758 \uc6b0\uce21 \uc81c\ubc29\uc758 \ub2e4\uc591\ud55c \ubbfc\uc871\ub4e4\uc774 \"\ub9e4\uc6b0 \uc544\ub974\uba54\ub2c8\uc544\ud654 \ub418\uc5c8\uc73c\uba70 \uadf8\ub4e4\uc911 \ub9ce\uc740 \uc0ac\ub78c\ub4e4\uc774 \uc0ac\uc2e4\uc0c1 \uc544\ub974\uba54\ub2c8\uc544\uc778 \uc774\ub77c\ud568\uc740 \uadf8 \ucc28\uc81c\ub85c\ub3c4(per se) \uc758\uc2ec\ud560 \uc218\uc5c6\ub2e4.\"\uace0 \ud3c9\uac00\ud588\ub2e4. \ucfe0\ub77c\uc758 \uc6b0\uce21 \uc81c\ubc29\uc744 387\ub144\uc5d0 \uce85\uce74\uc2a4 \uc54c\ubc14\ub2c8\uc544\uac00 \ucc28\uc9c0\ud588\uc744 \ub2f9\uc2dc, \uadf8 \uc9c0\uc5ed\uc758 \ub9ce\uc740 \uc0ac\ub78c\ub4e4\uc740 \uc5ec\uc804\ud788 \ub3c5\ud2b9\ud55c \ubbfc\uc871\uc801 \ubcf8\uc9c8\uc758 \uadfc\uac70\uac00 \ub418\uace0 \uc788\ub2e4.", "sentence_answer": "\"\ud655\uc2e4\ud788 \uc774\ub780\uc778 \ub4e4\uc774 \ud398\ub974\uc2dc\uc544\uc640 \uba54\ub514\uc544\uc758 \uc624\ub79c \ud1b5\uce58\uae30\uac04 \ub3d9\uc548 \uadf8 \uc9c0\uc5ed\uc5d0 \uc815\ucc29\ud588\uc74c\uc774 \ubd84\uba85\ud558\uace0, \ub300\ubd80\ubd84\uc758 \ud1a0\ucc29\ubbfc\ub4e4\uc740 \uc778\ub3c4-\uc720\ub7fd\uc778 \uc870\ucc28\ub3c4 \uc544\ub2d8\uc774 \ud2c0\ub9bc\uc5c6\ub2e4.", "paragraph_id": "6208903-1-2", "paragraph_question": "question: \ud398\ub974\uc2dc\uc544\uc640 \uba54\ub514\uc544\uc758 \uc624\ub79c \ud1b5\uce58\uae30\uac04 \ub3d9\uc548 \uc544\ub974\uba54\ub2c8\uc544\uc5d0 \uc815\ucc29\ud574 \uc788\uc5c8\ub358 \uc8fc\ubbfc\ub4e4\uc740?, context: \uc544\ub974\uba54\ub2c8\uc544 \uc815\ubcf5 \uc774\uc804\uc758 \ucfe0\ub77c\uc758 \uc6b0\uce21 \uc81c\ubc29\uc5d0 \uc788\ub358 \uc601\ud1a0\ub4e4\uc758 \ud1a0\uc791 \uc778\uad6c\ub294 \ub2e4\uc591\ud55c \uc6d0\uc8fc\ubbfc\ub4e4\ub85c \uad6c\uc131\ub418\uc5c8\ub2e4. \uace0\ub300 \uc5f0\ubcf4\ub294 \uc54c\ud2b8\uc0ac\ud750, \uc6b0\ud2f0\ud06c\uc758 \uc9c0\uc5ed\ub4e4\uc744 \ud3ec\ud568\ud558\ub294 \uadf8 \uad6c\uc5ed\ub4e4\uc5d0 \uac70\uc8fc\ud588\ub358 \uc5ec\ub7ec \ubbfc\uc871\ub4e4\uc758 \uc774\ub984\uc744 \uc81c\uacf5\ud55c\ub2e4. \ubbfc\uc871\uba85\uc740 \uc6b0\ud2f0\uc871, \ubbf8\ud0a4\uc871, \uce74\uc2a4\ud53c\uc871, \uac00\ub974\uac00\ub9ac\uc544\uc871, \uc0ac\uce74\uc138\ub2c8\uc871, \uac94\ub9ac\uc871, \uc624\ub514\uc871, \ub8e8\ud398\ub2c8\uc871, \ubc1c\ub77c\uc2a4(\uc545)\uce74\ub2c8\uc871, \ud398\ub974\uc2dc\uc544\uc778 \uadf8\ub9ac\uace0 \ud30c\ub974\ub77c\uc2dc\uc544\uc778\uc774\ub2e4. \ub85c\ubca0\ub974\ud2b8 H. \ud734\uc13c\uc740 \uadf8 \ubd80\uc871\ub4e4\uc774 \"\uc544\ub974\uba54\ub2c8\uc544\uc758 \uae30\uc6d0\uc784\uc740 \ud655\uc2e4\uce58 \uc54a\ub2e4.\"\uace0 \ud588\uc73c\uba70, \"\ud655\uc2e4\ud788 \uc774\ub780\uc778\ub4e4\uc774 \ud398\ub974\uc2dc\uc544\uc640 \uba54\ub514\uc544\uc758 \uc624\ub79c \ud1b5\uce58\uae30\uac04 \ub3d9\uc548 \uadf8 \uc9c0\uc5ed\uc5d0 \uc815\ucc29\ud588\uc74c\uc774 \ubd84\uba85\ud558\uace0, \ub300\ubd80\ubd84\uc758 \ud1a0\ucc29\ubbfc\ub4e4\uc740 \uc778\ub3c4-\uc720\ub7fd\uc778 \uc870\ucc28\ub3c4 \uc544\ub2d8\uc774 \ud2c0\ub9bc\uc5c6\ub2e4.\" \uace0 \ud558\uc600\ub2e4. \uadf8\ub294 \ucfe0\ub77c\uc758 \uc6b0\uce21 \uc81c\ubc29\uc758 \ub2e4\uc591\ud55c \ubbfc\uc871\ub4e4\uc774 \"\ub9e4\uc6b0 \uc544\ub974\uba54\ub2c8\uc544\ud654 \ub418\uc5c8\uc73c\uba70 \uadf8\ub4e4\uc911 \ub9ce\uc740 \uc0ac\ub78c\ub4e4\uc774 \uc0ac\uc2e4\uc0c1 \uc544\ub974\uba54\ub2c8\uc544\uc778 \uc774\ub77c\ud568\uc740 \uadf8 \ucc28\uc81c\ub85c\ub3c4(per se) \uc758\uc2ec\ud560 \uc218\uc5c6\ub2e4.\"\uace0 \ud3c9\uac00\ud588\ub2e4. \ucfe0\ub77c\uc758 \uc6b0\uce21 \uc81c\ubc29\uc744 387\ub144\uc5d0 \uce85\uce74\uc2a4 \uc54c\ubc14\ub2c8\uc544\uac00 \ucc28\uc9c0\ud588\uc744 \ub2f9\uc2dc, \uadf8 \uc9c0\uc5ed\uc758 \ub9ce\uc740 \uc0ac\ub78c\ub4e4\uc740 \uc5ec\uc804\ud788 \ub3c5\ud2b9\ud55c \ubbfc\uc871\uc801 \ubcf8\uc9c8\uc758 \uadfc\uac70\uac00 \ub418\uace0 \uc788\ub2e4."} {"question": "\uc77c\ubcf8\uc758 \ud0dc\ud3c9\uc591 \uc5f0\uc548 \uc9c0\ub300\uc5d0 \uc5ec\ub984\uc5d0 \ubd80\ub294 \ubc14\ub78c\uc740?", "paragraph": "\ub09c\uc138\uc774 \uc81c\ub3c4\ub97c \ud3ec\ud568\ud55c \uc77c\ubcf8 \uc5f4\ub3c4\ub294 \ub0a8\ubd81\uc73c\ub85c 5,000km\uc5d0 \uc774\ub974\uba70 \ub0c9\ub300 \uae30\ud6c4\uc5d0\uc11c \uc5f4\ub300 \uae30\ud6c4\uae4c\uc9c0 \uac78\uccd0\uc788\ub294 \ub370\ub2e4\uac00 \ubcf5\uc7a1\ud55c \uc9c0\ud615\uacfc \ud574\ub958\uc758 \uc601\ud5a5\uc774 \ud06c\uae30 \ub54c\ubb38\uc5d0 \uc9c0\uc5ed\ubcc4 \uae30\ud6c4\uc758 \ucc28\uc774\uac00 \ud604\uc800\ud558\ub2e4. \uc5f0\ud3c9\uade0 \uae30\uc628\uc740 \ub0a8\ucabd \uc624\ud0a4\ub098\uc640\ud604\uc758 \ub098\ud558 \uc2dc\uac00 \uc12d\uc528 22.7\ub3c4\ub97c \uae30\ub85d\ud558\uace0 \ub610 \uc57c\uc5d0\uc57c\ub9c8 \uc81c\ub3c4\uc758 \uc774\uc2dc\uac00\ud0a4 \uc12c\uc740 \uc12d\uc528 24\ub3c4\ub85c\uc11c \uc5f4\ub300 \uc6b0\ub9bc \uae30\ud6c4\uc5d0 \uc18d\ud55c\ub2e4, \uaddc\uc288 \uc9c0\ubc29\uc758 \uac00\uace0\uc2dc\ub9c8 \uc2dc\ub294 \uc12d\uc528 18.3\ub3c4, \uc624\uc0ac\uce74 \uc2dc\ub294 \uc12d\uc528 16.5\ub3c4, \ub3c4\ucfc4\ub3c4\uc758 \uad6c \uc9c0\uc5ed\uc740 \uc12d\uc528 15.9\ub3c4\uc774\ub2e4 \ubd81\ucabd \ud64b\uce74\uc774\ub3c4\uc758 \ub124\ubb34\ub85c \uc2dc\uac00 \uc12d\uc528 6.1\ub3c4\ub85c \ucd5c\uc800\ub97c \uae30\ub85d\ud55c\ub2e4. \uadf8\ub7ec\ub098 \ud64b\uce74\uc774\ub3c4\uc640 \uc624\ud0a4\ub098\uc640 \ud604\uc758 \uc57c\uc5d0\uc57c\ub9c8 \uc81c\ub3c4, \ub3c4\ucfc4\ub3c4\uc758 \uc624\ud0a4\ub178\ud1a0\ub9ac \uc12c, \ubbf8\ub098\ubbf8\ud1a0\ub9ac \uc12c\uc744 \uc81c\uc678\ud558\uba74 \ub300\ubd80\ubd84\uc758 \uc9c0\uc5ed\uc740 \uc628\ub300 \uae30\ud6c4\uc5d0 \uc18d\ud574 \uc628\ub09c\ud55c \uae30\ud6c4\uc774\uace0 \uc0ac\uacc4\uc808 \uad6c\ubd84\uc774 \ub69c\ub837\ud558\ub2e4. \ud0dc\ud3c9\uc591 \uc5f0\uc548 \uc9c0\ub300\ub294 \uc5ec\ub984\uc774\uba74 \ub0a8\ub3d9 \uacc4\uc808\ud48d\uc774 \ubd88\uc5b4 \ub9e4\uc6b0 \ubb34\ub365\uace0, \uaca8\uc6b8\uc5d0\ub294 \uac74\uc870\ud558\uace0 \ub9d1\uc740 \ub0a0\uc774 \ub9ce\uc544\uc9c4\ub2e4. \ud55c\ud3b8 \ub3d9\ud574 \ucabd\uc5d0\uc11c\ub294 \uaca8\uc6b8\uc5d0 \uc2dc\ubca0\ub9ac\uc544 \uacc4\uc808\ud48d\uc73c\ub85c \uc778\ud55c \uac15\uc124\uc774 \ub9ce\uc544 \ub2c8\uac00\ud0c0 \ud604\uc744 \ud3ec\ud568\ud55c \ud638\ucfe0\ub9ac\ucfe0 \uc9c0\ubc29\uc740 \uc138\uacc4\uc801\uc778 \ub2e4\uc124 \uc9c0\ub300\ub85c \uc190\uaf3d\ud600 4~5m\uc5d0 \uc774\ub974\ub294 \ub208\uc774 \uc313\uc774\ub294 \uacf3\ub3c4 \uc788\ub2e4. \ud64b\uce74\uc774\ub3c4\ub97c \uc81c\uc678\ud55c \uc9c0\uc5ed\uc5d0\uc11c\ub294 6\uc6d4 \ucd08\uc21c\ubd80\ud130 7\uc6d4 \uc911\uc21c\uc5d0 \uac78\uccd0 \uace0\uc628\ub2e4\uc2b5\ud55c \uc7a5\ub9c8\uac00 \ub0b4\ub9ac\uba70 8\uc6d4\ubd80\ud130 10\uc6d4\uc5d0 \uac78\uccd0 \uc77c\ubcf8 \uc5f4\ub3c4 \ub0a8\uc11c\ubd80 \uc9c0\uc5ed\uc740 \ud0dc\ud48d\uc758 \uc601\ud5a5\uc744 \uc2ec\ud558\uac8c \ubc1b\uc544 \ud48d\uc218\ud574\uc758 \ub300\ubd80\ubd84\uc744 \ucd08\ub798\ud55c\ub2e4.", "answer": "\ub0a8\ub3d9 \uacc4\uc808\ud48d", "sentence": "\ud0dc\ud3c9\uc591 \uc5f0\uc548 \uc9c0\ub300\ub294 \uc5ec\ub984\uc774\uba74 \ub0a8\ub3d9 \uacc4\uc808\ud48d \uc774 \ubd88\uc5b4 \ub9e4\uc6b0 \ubb34\ub365\uace0, \uaca8\uc6b8\uc5d0\ub294 \uac74\uc870\ud558\uace0 \ub9d1\uc740 \ub0a0\uc774 \ub9ce\uc544\uc9c4\ub2e4.", "paragraph_sentence": "\ub09c\uc138\uc774 \uc81c\ub3c4\ub97c \ud3ec\ud568\ud55c \uc77c\ubcf8 \uc5f4\ub3c4\ub294 \ub0a8\ubd81\uc73c\ub85c 5,000km\uc5d0 \uc774\ub974\uba70 \ub0c9\ub300 \uae30\ud6c4\uc5d0\uc11c \uc5f4\ub300 \uae30\ud6c4\uae4c\uc9c0 \uac78\uccd0\uc788\ub294 \ub370\ub2e4\uac00 \ubcf5\uc7a1\ud55c \uc9c0\ud615\uacfc \ud574\ub958\uc758 \uc601\ud5a5\uc774 \ud06c\uae30 \ub54c\ubb38\uc5d0 \uc9c0\uc5ed\ubcc4 \uae30\ud6c4\uc758 \ucc28\uc774\uac00 \ud604\uc800\ud558\ub2e4. \uc5f0\ud3c9\uade0 \uae30\uc628\uc740 \ub0a8\ucabd \uc624\ud0a4\ub098\uc640\ud604\uc758 \ub098\ud558 \uc2dc\uac00 \uc12d\uc528 22.7\ub3c4\ub97c \uae30\ub85d\ud558\uace0 \ub610 \uc57c\uc5d0\uc57c\ub9c8 \uc81c\ub3c4\uc758 \uc774\uc2dc\uac00\ud0a4 \uc12c\uc740 \uc12d\uc528 24\ub3c4\ub85c\uc11c \uc5f4\ub300 \uc6b0\ub9bc \uae30\ud6c4\uc5d0 \uc18d\ud55c\ub2e4, \uaddc\uc288 \uc9c0\ubc29\uc758 \uac00\uace0\uc2dc\ub9c8 \uc2dc\ub294 \uc12d\uc528 18.3\ub3c4, \uc624\uc0ac\uce74 \uc2dc\ub294 \uc12d\uc528 16.5\ub3c4, \ub3c4\ucfc4\ub3c4\uc758 \uad6c \uc9c0\uc5ed\uc740 \uc12d\uc528 15.9\ub3c4\uc774\ub2e4 \ubd81\ucabd \ud64b\uce74\uc774\ub3c4\uc758 \ub124\ubb34\ub85c \uc2dc\uac00 \uc12d\uc528 6.1\ub3c4\ub85c \ucd5c\uc800\ub97c \uae30\ub85d\ud55c\ub2e4. \uadf8\ub7ec\ub098 \ud64b\uce74\uc774\ub3c4\uc640 \uc624\ud0a4\ub098\uc640 \ud604\uc758 \uc57c\uc5d0\uc57c\ub9c8 \uc81c\ub3c4, \ub3c4\ucfc4\ub3c4\uc758 \uc624\ud0a4\ub178\ud1a0\ub9ac \uc12c, \ubbf8\ub098\ubbf8\ud1a0\ub9ac \uc12c\uc744 \uc81c\uc678\ud558\uba74 \ub300\ubd80\ubd84\uc758 \uc9c0\uc5ed\uc740 \uc628\ub300 \uae30\ud6c4\uc5d0 \uc18d\ud574 \uc628\ub09c\ud55c \uae30\ud6c4\uc774\uace0 \uc0ac\uacc4\uc808 \uad6c\ubd84\uc774 \ub69c\ub837\ud558\ub2e4. \ud0dc\ud3c9\uc591 \uc5f0\uc548 \uc9c0\ub300\ub294 \uc5ec\ub984\uc774\uba74 \ub0a8\ub3d9 \uacc4\uc808\ud48d \uc774 \ubd88\uc5b4 \ub9e4\uc6b0 \ubb34\ub365\uace0, \uaca8\uc6b8\uc5d0\ub294 \uac74\uc870\ud558\uace0 \ub9d1\uc740 \ub0a0\uc774 \ub9ce\uc544\uc9c4\ub2e4. \ud55c\ud3b8 \ub3d9\ud574 \ucabd\uc5d0\uc11c\ub294 \uaca8\uc6b8\uc5d0 \uc2dc\ubca0\ub9ac\uc544 \uacc4\uc808\ud48d\uc73c\ub85c \uc778\ud55c \uac15\uc124\uc774 \ub9ce\uc544 \ub2c8\uac00\ud0c0 \ud604\uc744 \ud3ec\ud568\ud55c \ud638\ucfe0\ub9ac\ucfe0 \uc9c0\ubc29\uc740 \uc138\uacc4\uc801\uc778 \ub2e4\uc124 \uc9c0\ub300\ub85c \uc190\uaf3d\ud600 4~5m\uc5d0 \uc774\ub974\ub294 \ub208\uc774 \uc313\uc774\ub294 \uacf3\ub3c4 \uc788\ub2e4. \ud64b\uce74\uc774\ub3c4\ub97c \uc81c\uc678\ud55c \uc9c0\uc5ed\uc5d0\uc11c\ub294 6\uc6d4 \ucd08\uc21c\ubd80\ud130 7\uc6d4 \uc911\uc21c\uc5d0 \uac78\uccd0 \uace0\uc628\ub2e4\uc2b5\ud55c \uc7a5\ub9c8\uac00 \ub0b4\ub9ac\uba70 8\uc6d4\ubd80\ud130 10\uc6d4\uc5d0 \uac78\uccd0 \uc77c\ubcf8 \uc5f4\ub3c4 \ub0a8\uc11c\ubd80 \uc9c0\uc5ed\uc740 \ud0dc\ud48d\uc758 \uc601\ud5a5\uc744 \uc2ec\ud558\uac8c \ubc1b\uc544 \ud48d\uc218\ud574\uc758 \ub300\ubd80\ubd84\uc744 \ucd08\ub798\ud55c\ub2e4.", "paragraph_answer": "\ub09c\uc138\uc774 \uc81c\ub3c4\ub97c \ud3ec\ud568\ud55c \uc77c\ubcf8 \uc5f4\ub3c4\ub294 \ub0a8\ubd81\uc73c\ub85c 5,000km\uc5d0 \uc774\ub974\uba70 \ub0c9\ub300 \uae30\ud6c4\uc5d0\uc11c \uc5f4\ub300 \uae30\ud6c4\uae4c\uc9c0 \uac78\uccd0\uc788\ub294 \ub370\ub2e4\uac00 \ubcf5\uc7a1\ud55c \uc9c0\ud615\uacfc \ud574\ub958\uc758 \uc601\ud5a5\uc774 \ud06c\uae30 \ub54c\ubb38\uc5d0 \uc9c0\uc5ed\ubcc4 \uae30\ud6c4\uc758 \ucc28\uc774\uac00 \ud604\uc800\ud558\ub2e4. \uc5f0\ud3c9\uade0 \uae30\uc628\uc740 \ub0a8\ucabd \uc624\ud0a4\ub098\uc640\ud604\uc758 \ub098\ud558 \uc2dc\uac00 \uc12d\uc528 22.7\ub3c4\ub97c \uae30\ub85d\ud558\uace0 \ub610 \uc57c\uc5d0\uc57c\ub9c8 \uc81c\ub3c4\uc758 \uc774\uc2dc\uac00\ud0a4 \uc12c\uc740 \uc12d\uc528 24\ub3c4\ub85c\uc11c \uc5f4\ub300 \uc6b0\ub9bc \uae30\ud6c4\uc5d0 \uc18d\ud55c\ub2e4, \uaddc\uc288 \uc9c0\ubc29\uc758 \uac00\uace0\uc2dc\ub9c8 \uc2dc\ub294 \uc12d\uc528 18.3\ub3c4, \uc624\uc0ac\uce74 \uc2dc\ub294 \uc12d\uc528 16.5\ub3c4, \ub3c4\ucfc4\ub3c4\uc758 \uad6c \uc9c0\uc5ed\uc740 \uc12d\uc528 15.9\ub3c4\uc774\ub2e4 \ubd81\ucabd \ud64b\uce74\uc774\ub3c4\uc758 \ub124\ubb34\ub85c \uc2dc\uac00 \uc12d\uc528 6.1\ub3c4\ub85c \ucd5c\uc800\ub97c \uae30\ub85d\ud55c\ub2e4. \uadf8\ub7ec\ub098 \ud64b\uce74\uc774\ub3c4\uc640 \uc624\ud0a4\ub098\uc640 \ud604\uc758 \uc57c\uc5d0\uc57c\ub9c8 \uc81c\ub3c4, \ub3c4\ucfc4\ub3c4\uc758 \uc624\ud0a4\ub178\ud1a0\ub9ac \uc12c, \ubbf8\ub098\ubbf8\ud1a0\ub9ac \uc12c\uc744 \uc81c\uc678\ud558\uba74 \ub300\ubd80\ubd84\uc758 \uc9c0\uc5ed\uc740 \uc628\ub300 \uae30\ud6c4\uc5d0 \uc18d\ud574 \uc628\ub09c\ud55c \uae30\ud6c4\uc774\uace0 \uc0ac\uacc4\uc808 \uad6c\ubd84\uc774 \ub69c\ub837\ud558\ub2e4. \ud0dc\ud3c9\uc591 \uc5f0\uc548 \uc9c0\ub300\ub294 \uc5ec\ub984\uc774\uba74 \ub0a8\ub3d9 \uacc4\uc808\ud48d \uc774 \ubd88\uc5b4 \ub9e4\uc6b0 \ubb34\ub365\uace0, \uaca8\uc6b8\uc5d0\ub294 \uac74\uc870\ud558\uace0 \ub9d1\uc740 \ub0a0\uc774 \ub9ce\uc544\uc9c4\ub2e4. \ud55c\ud3b8 \ub3d9\ud574 \ucabd\uc5d0\uc11c\ub294 \uaca8\uc6b8\uc5d0 \uc2dc\ubca0\ub9ac\uc544 \uacc4\uc808\ud48d\uc73c\ub85c \uc778\ud55c \uac15\uc124\uc774 \ub9ce\uc544 \ub2c8\uac00\ud0c0 \ud604\uc744 \ud3ec\ud568\ud55c \ud638\ucfe0\ub9ac\ucfe0 \uc9c0\ubc29\uc740 \uc138\uacc4\uc801\uc778 \ub2e4\uc124 \uc9c0\ub300\ub85c \uc190\uaf3d\ud600 4~5m\uc5d0 \uc774\ub974\ub294 \ub208\uc774 \uc313\uc774\ub294 \uacf3\ub3c4 \uc788\ub2e4. \ud64b\uce74\uc774\ub3c4\ub97c \uc81c\uc678\ud55c \uc9c0\uc5ed\uc5d0\uc11c\ub294 6\uc6d4 \ucd08\uc21c\ubd80\ud130 7\uc6d4 \uc911\uc21c\uc5d0 \uac78\uccd0 \uace0\uc628\ub2e4\uc2b5\ud55c \uc7a5\ub9c8\uac00 \ub0b4\ub9ac\uba70 8\uc6d4\ubd80\ud130 10\uc6d4\uc5d0 \uac78\uccd0 \uc77c\ubcf8 \uc5f4\ub3c4 \ub0a8\uc11c\ubd80 \uc9c0\uc5ed\uc740 \ud0dc\ud48d\uc758 \uc601\ud5a5\uc744 \uc2ec\ud558\uac8c \ubc1b\uc544 \ud48d\uc218\ud574\uc758 \ub300\ubd80\ubd84\uc744 \ucd08\ub798\ud55c\ub2e4.", "sentence_answer": "\ud0dc\ud3c9\uc591 \uc5f0\uc548 \uc9c0\ub300\ub294 \uc5ec\ub984\uc774\uba74 \ub0a8\ub3d9 \uacc4\uc808\ud48d \uc774 \ubd88\uc5b4 \ub9e4\uc6b0 \ubb34\ub365\uace0, \uaca8\uc6b8\uc5d0\ub294 \uac74\uc870\ud558\uace0 \ub9d1\uc740 \ub0a0\uc774 \ub9ce\uc544\uc9c4\ub2e4.", "paragraph_id": "6591477-12-1", "paragraph_question": "question: \uc77c\ubcf8\uc758 \ud0dc\ud3c9\uc591 \uc5f0\uc548 \uc9c0\ub300\uc5d0 \uc5ec\ub984\uc5d0 \ubd80\ub294 \ubc14\ub78c\uc740?, context: \ub09c\uc138\uc774 \uc81c\ub3c4\ub97c \ud3ec\ud568\ud55c \uc77c\ubcf8 \uc5f4\ub3c4\ub294 \ub0a8\ubd81\uc73c\ub85c 5,000km\uc5d0 \uc774\ub974\uba70 \ub0c9\ub300 \uae30\ud6c4\uc5d0\uc11c \uc5f4\ub300 \uae30\ud6c4\uae4c\uc9c0 \uac78\uccd0\uc788\ub294 \ub370\ub2e4\uac00 \ubcf5\uc7a1\ud55c \uc9c0\ud615\uacfc \ud574\ub958\uc758 \uc601\ud5a5\uc774 \ud06c\uae30 \ub54c\ubb38\uc5d0 \uc9c0\uc5ed\ubcc4 \uae30\ud6c4\uc758 \ucc28\uc774\uac00 \ud604\uc800\ud558\ub2e4. \uc5f0\ud3c9\uade0 \uae30\uc628\uc740 \ub0a8\ucabd \uc624\ud0a4\ub098\uc640\ud604\uc758 \ub098\ud558 \uc2dc\uac00 \uc12d\uc528 22.7\ub3c4\ub97c \uae30\ub85d\ud558\uace0 \ub610 \uc57c\uc5d0\uc57c\ub9c8 \uc81c\ub3c4\uc758 \uc774\uc2dc\uac00\ud0a4 \uc12c\uc740 \uc12d\uc528 24\ub3c4\ub85c\uc11c \uc5f4\ub300 \uc6b0\ub9bc \uae30\ud6c4\uc5d0 \uc18d\ud55c\ub2e4, \uaddc\uc288 \uc9c0\ubc29\uc758 \uac00\uace0\uc2dc\ub9c8 \uc2dc\ub294 \uc12d\uc528 18.3\ub3c4, \uc624\uc0ac\uce74 \uc2dc\ub294 \uc12d\uc528 16.5\ub3c4, \ub3c4\ucfc4\ub3c4\uc758 \uad6c \uc9c0\uc5ed\uc740 \uc12d\uc528 15.9\ub3c4\uc774\ub2e4 \ubd81\ucabd \ud64b\uce74\uc774\ub3c4\uc758 \ub124\ubb34\ub85c \uc2dc\uac00 \uc12d\uc528 6.1\ub3c4\ub85c \ucd5c\uc800\ub97c \uae30\ub85d\ud55c\ub2e4. \uadf8\ub7ec\ub098 \ud64b\uce74\uc774\ub3c4\uc640 \uc624\ud0a4\ub098\uc640 \ud604\uc758 \uc57c\uc5d0\uc57c\ub9c8 \uc81c\ub3c4, \ub3c4\ucfc4\ub3c4\uc758 \uc624\ud0a4\ub178\ud1a0\ub9ac \uc12c, \ubbf8\ub098\ubbf8\ud1a0\ub9ac \uc12c\uc744 \uc81c\uc678\ud558\uba74 \ub300\ubd80\ubd84\uc758 \uc9c0\uc5ed\uc740 \uc628\ub300 \uae30\ud6c4\uc5d0 \uc18d\ud574 \uc628\ub09c\ud55c \uae30\ud6c4\uc774\uace0 \uc0ac\uacc4\uc808 \uad6c\ubd84\uc774 \ub69c\ub837\ud558\ub2e4. \ud0dc\ud3c9\uc591 \uc5f0\uc548 \uc9c0\ub300\ub294 \uc5ec\ub984\uc774\uba74 \ub0a8\ub3d9 \uacc4\uc808\ud48d\uc774 \ubd88\uc5b4 \ub9e4\uc6b0 \ubb34\ub365\uace0, \uaca8\uc6b8\uc5d0\ub294 \uac74\uc870\ud558\uace0 \ub9d1\uc740 \ub0a0\uc774 \ub9ce\uc544\uc9c4\ub2e4. \ud55c\ud3b8 \ub3d9\ud574 \ucabd\uc5d0\uc11c\ub294 \uaca8\uc6b8\uc5d0 \uc2dc\ubca0\ub9ac\uc544 \uacc4\uc808\ud48d\uc73c\ub85c \uc778\ud55c \uac15\uc124\uc774 \ub9ce\uc544 \ub2c8\uac00\ud0c0 \ud604\uc744 \ud3ec\ud568\ud55c \ud638\ucfe0\ub9ac\ucfe0 \uc9c0\ubc29\uc740 \uc138\uacc4\uc801\uc778 \ub2e4\uc124 \uc9c0\ub300\ub85c \uc190\uaf3d\ud600 4~5m\uc5d0 \uc774\ub974\ub294 \ub208\uc774 \uc313\uc774\ub294 \uacf3\ub3c4 \uc788\ub2e4. \ud64b\uce74\uc774\ub3c4\ub97c \uc81c\uc678\ud55c \uc9c0\uc5ed\uc5d0\uc11c\ub294 6\uc6d4 \ucd08\uc21c\ubd80\ud130 7\uc6d4 \uc911\uc21c\uc5d0 \uac78\uccd0 \uace0\uc628\ub2e4\uc2b5\ud55c \uc7a5\ub9c8\uac00 \ub0b4\ub9ac\uba70 8\uc6d4\ubd80\ud130 10\uc6d4\uc5d0 \uac78\uccd0 \uc77c\ubcf8 \uc5f4\ub3c4 \ub0a8\uc11c\ubd80 \uc9c0\uc5ed\uc740 \ud0dc\ud48d\uc758 \uc601\ud5a5\uc744 \uc2ec\ud558\uac8c \ubc1b\uc544 \ud48d\uc218\ud574\uc758 \ub300\ubd80\ubd84\uc744 \ucd08\ub798\ud55c\ub2e4."} {"question": "\uc54c\ud30c \uce74\ud30c \uc54c\ud30c\uac00 \uc124\ub9bd\ub41c \ud574\ub294 \uc5b8\uc81c\uc778\uac00?", "paragraph": "\uc54c\ud30c \uce74\ud30c \uc54c\ud30c(\uadf8\ub9ac\uc2a4\uc5b4: \u0386\u039a\u0386, \uc601\uc5b4: Alpha Kappa Alpha, \uc774\ud558 AKA)\ub294 \uc544\ud504\ub9ac\uce74\uacc4 \ubbf8\uad6d\uc778 \uc5ec\ub300\uc0dd\ub4e4\uc774 \uc124\ub9bd\ud558\uace0 \ubc95\uc778\uc73c\ub85c \ub9cc\ub4e0 \uc5ec\uc131\ud68c\ub85c\uc11c, \ucd5c\ucd08\uc758 \uadf8\ub9ac\uc2a4 \ubb38\uc790\ub85c \uc774\ub984\uc9c0\uc5b4\uc9c4 \uc5ec\uc131\ud68c\uc774\ub2e4. \uc5d0\uc124 \ud5e4\uc9c0\uba3c \ub77c\uc77c\uc774 \uc8fc\ub3c4\ud55c 9\uba85\uc758 \ud559\uc0dd \ubaa8\uc784\uc5d0 \uc758\ud574 \uc6cc\uc2f1\ud134 D.C.\uc758 \ud558\uc6cc\ub4dc \ub300\ud559\uad50\uc5d0\uc11c 1908\ub144 1\uc6d4 15\uc77c \uc124\ub9bd\ub418\uc5c8\ub2e4. \uc5ec\uc131\ud68c\ub97c \uacb0\uc131\ud558\ub294 \uac83\uc740, 20\uc138\uae30 \ucd08 \uc18c\uc218\ubbfc\uc871 \ubc0f \uc5ec\uc131\ub4e4\uc774 \uae30\ud68c\uc758 \ubd88\ud3c9\ub4f1\uc73c\ub85c \uc778\ud558\uc5ec \ud798\uc774\ub098 \uad8c\uc704\uac00 \uac70\uc758 \uc5c6\ub358 \uc9c0\uc5ed\ub4e4\uc5d0\uc11c \uc544\ud504\ub9ac\uce74\uacc4 \ubbf8\uad6d\uc778 \uc5ec\uc131\uc5d0 \ub300\ud55c \uc7a5\ubcbd\uc744 \ubb34\ub108\ub728\ub9ac\ub294 \ub370 \uae30\uc5ec\ud588\ub2e4. AKA\ub294 1913\ub144 1\uc6d4 29\uc77c \ubc95\uc778\uc774 \ub418\uc5c8\ub2e4. \uc544\ud504\ub9ac\uce74\uacc4, \uce74\ud504\uce74\uc2a4\uacc4, \uc544\uc2dc\uc544\uacc4, \ud788\uc2a4\ud328\ub2c9\uc758 \uc5ec\ub300\uc0dd\ub4e4\ub85c \uad6c\uc131\ub41c \uc774 \uc5ec\uc131\ud68c\uc5d0\ub294 \ubbf8\uad6d \ubc0f \uc5ec\ub7ec \ub098\ub77c\uc5d0 975\uac1c \uc774\uc0c1\uc758 \uc9c0\ubd80\uac00 \uc788\uc73c\uba70, 20\ub9cc\uba85 \uc774\uc0c1\uc758 \ud68c\uc6d0\ub4e4\uc774 \uc788\ub2e4. \uc5ec\uc131\ub4e4\uc740 \ub300\ud559 \uc9c0\ubd80 \ub610\ub294 \ub300\ud559\uc6d0 \uc9c0\ubd80\ub97c \ud1b5\ud574 \uac00\uc785\ud560 \uc218 \uc788\ub2e4.", "answer": "1908\ub144", "sentence": "\uc5d0\uc124 \ud5e4\uc9c0\uba3c \ub77c\uc77c\uc774 \uc8fc\ub3c4\ud55c 9\uba85\uc758 \ud559\uc0dd \ubaa8\uc784\uc5d0 \uc758\ud574 \uc6cc\uc2f1\ud134 D.C.\uc758 \ud558\uc6cc\ub4dc \ub300\ud559\uad50\uc5d0\uc11c 1908\ub144 1\uc6d4 15\uc77c \uc124\ub9bd\ub418\uc5c8\ub2e4.", "paragraph_sentence": "\uc54c\ud30c \uce74\ud30c \uc54c\ud30c(\uadf8\ub9ac\uc2a4\uc5b4: \u0386\u039a\u0386, \uc601\uc5b4: Alpha Kappa Alpha, \uc774\ud558 AKA)\ub294 \uc544\ud504\ub9ac\uce74\uacc4 \ubbf8\uad6d\uc778 \uc5ec\ub300\uc0dd\ub4e4\uc774 \uc124\ub9bd\ud558\uace0 \ubc95\uc778\uc73c\ub85c \ub9cc\ub4e0 \uc5ec\uc131\ud68c\ub85c\uc11c, \ucd5c\ucd08\uc758 \uadf8\ub9ac\uc2a4 \ubb38\uc790\ub85c \uc774\ub984\uc9c0\uc5b4\uc9c4 \uc5ec\uc131\ud68c\uc774\ub2e4. \uc5d0\uc124 \ud5e4\uc9c0\uba3c \ub77c\uc77c\uc774 \uc8fc\ub3c4\ud55c 9\uba85\uc758 \ud559\uc0dd \ubaa8\uc784\uc5d0 \uc758\ud574 \uc6cc\uc2f1\ud134 D.C.\uc758 \ud558\uc6cc\ub4dc \ub300\ud559\uad50\uc5d0\uc11c 1908\ub144 1\uc6d4 15\uc77c \uc124\ub9bd\ub418\uc5c8\ub2e4. \uc5ec\uc131\ud68c\ub97c \uacb0\uc131\ud558\ub294 \uac83\uc740, 20\uc138\uae30 \ucd08 \uc18c\uc218\ubbfc\uc871 \ubc0f \uc5ec\uc131\ub4e4\uc774 \uae30\ud68c\uc758 \ubd88\ud3c9\ub4f1\uc73c\ub85c \uc778\ud558\uc5ec \ud798\uc774\ub098 \uad8c\uc704\uac00 \uac70\uc758 \uc5c6\ub358 \uc9c0\uc5ed\ub4e4\uc5d0\uc11c \uc544\ud504\ub9ac\uce74\uacc4 \ubbf8\uad6d\uc778 \uc5ec\uc131\uc5d0 \ub300\ud55c \uc7a5\ubcbd\uc744 \ubb34\ub108\ub728\ub9ac\ub294 \ub370 \uae30\uc5ec\ud588\ub2e4. AKA\ub294 1913\ub144 1\uc6d4 29\uc77c \ubc95\uc778\uc774 \ub418\uc5c8\ub2e4. \uc544\ud504\ub9ac\uce74\uacc4, \uce74\ud504\uce74\uc2a4\uacc4, \uc544\uc2dc\uc544\uacc4, \ud788\uc2a4\ud328\ub2c9\uc758 \uc5ec\ub300\uc0dd\ub4e4\ub85c \uad6c\uc131\ub41c \uc774 \uc5ec\uc131\ud68c\uc5d0\ub294 \ubbf8\uad6d \ubc0f \uc5ec\ub7ec \ub098\ub77c\uc5d0 975\uac1c \uc774\uc0c1\uc758 \uc9c0\ubd80\uac00 \uc788\uc73c\uba70, 20\ub9cc\uba85 \uc774\uc0c1\uc758 \ud68c\uc6d0\ub4e4\uc774 \uc788\ub2e4. \uc5ec\uc131\ub4e4\uc740 \ub300\ud559 \uc9c0\ubd80 \ub610\ub294 \ub300\ud559\uc6d0 \uc9c0\ubd80\ub97c \ud1b5\ud574 \uac00\uc785\ud560 \uc218 \uc788\ub2e4.", "paragraph_answer": "\uc54c\ud30c \uce74\ud30c \uc54c\ud30c(\uadf8\ub9ac\uc2a4\uc5b4: \u0386\u039a\u0386, \uc601\uc5b4: Alpha Kappa Alpha, \uc774\ud558 AKA)\ub294 \uc544\ud504\ub9ac\uce74\uacc4 \ubbf8\uad6d\uc778 \uc5ec\ub300\uc0dd\ub4e4\uc774 \uc124\ub9bd\ud558\uace0 \ubc95\uc778\uc73c\ub85c \ub9cc\ub4e0 \uc5ec\uc131\ud68c\ub85c\uc11c, \ucd5c\ucd08\uc758 \uadf8\ub9ac\uc2a4 \ubb38\uc790\ub85c \uc774\ub984\uc9c0\uc5b4\uc9c4 \uc5ec\uc131\ud68c\uc774\ub2e4. \uc5d0\uc124 \ud5e4\uc9c0\uba3c \ub77c\uc77c\uc774 \uc8fc\ub3c4\ud55c 9\uba85\uc758 \ud559\uc0dd \ubaa8\uc784\uc5d0 \uc758\ud574 \uc6cc\uc2f1\ud134 D.C.\uc758 \ud558\uc6cc\ub4dc \ub300\ud559\uad50\uc5d0\uc11c 1908\ub144 1\uc6d4 15\uc77c \uc124\ub9bd\ub418\uc5c8\ub2e4. \uc5ec\uc131\ud68c\ub97c \uacb0\uc131\ud558\ub294 \uac83\uc740, 20\uc138\uae30 \ucd08 \uc18c\uc218\ubbfc\uc871 \ubc0f \uc5ec\uc131\ub4e4\uc774 \uae30\ud68c\uc758 \ubd88\ud3c9\ub4f1\uc73c\ub85c \uc778\ud558\uc5ec \ud798\uc774\ub098 \uad8c\uc704\uac00 \uac70\uc758 \uc5c6\ub358 \uc9c0\uc5ed\ub4e4\uc5d0\uc11c \uc544\ud504\ub9ac\uce74\uacc4 \ubbf8\uad6d\uc778 \uc5ec\uc131\uc5d0 \ub300\ud55c \uc7a5\ubcbd\uc744 \ubb34\ub108\ub728\ub9ac\ub294 \ub370 \uae30\uc5ec\ud588\ub2e4. AKA\ub294 1913\ub144 1\uc6d4 29\uc77c \ubc95\uc778\uc774 \ub418\uc5c8\ub2e4. \uc544\ud504\ub9ac\uce74\uacc4, \uce74\ud504\uce74\uc2a4\uacc4, \uc544\uc2dc\uc544\uacc4, \ud788\uc2a4\ud328\ub2c9\uc758 \uc5ec\ub300\uc0dd\ub4e4\ub85c \uad6c\uc131\ub41c \uc774 \uc5ec\uc131\ud68c\uc5d0\ub294 \ubbf8\uad6d \ubc0f \uc5ec\ub7ec \ub098\ub77c\uc5d0 975\uac1c \uc774\uc0c1\uc758 \uc9c0\ubd80\uac00 \uc788\uc73c\uba70, 20\ub9cc\uba85 \uc774\uc0c1\uc758 \ud68c\uc6d0\ub4e4\uc774 \uc788\ub2e4. \uc5ec\uc131\ub4e4\uc740 \ub300\ud559 \uc9c0\ubd80 \ub610\ub294 \ub300\ud559\uc6d0 \uc9c0\ubd80\ub97c \ud1b5\ud574 \uac00\uc785\ud560 \uc218 \uc788\ub2e4.", "sentence_answer": "\uc5d0\uc124 \ud5e4\uc9c0\uba3c \ub77c\uc77c\uc774 \uc8fc\ub3c4\ud55c 9\uba85\uc758 \ud559\uc0dd \ubaa8\uc784\uc5d0 \uc758\ud574 \uc6cc\uc2f1\ud134 D.C.\uc758 \ud558\uc6cc\ub4dc \ub300\ud559\uad50\uc5d0\uc11c 1908\ub144 1\uc6d4 15\uc77c \uc124\ub9bd\ub418\uc5c8\ub2e4.", "paragraph_id": "6604746-0-1", "paragraph_question": "question: \uc54c\ud30c \uce74\ud30c \uc54c\ud30c\uac00 \uc124\ub9bd\ub41c \ud574\ub294 \uc5b8\uc81c\uc778\uac00?, context: \uc54c\ud30c \uce74\ud30c \uc54c\ud30c(\uadf8\ub9ac\uc2a4\uc5b4: \u0386\u039a\u0386, \uc601\uc5b4: Alpha Kappa Alpha, \uc774\ud558 AKA)\ub294 \uc544\ud504\ub9ac\uce74\uacc4 \ubbf8\uad6d\uc778 \uc5ec\ub300\uc0dd\ub4e4\uc774 \uc124\ub9bd\ud558\uace0 \ubc95\uc778\uc73c\ub85c \ub9cc\ub4e0 \uc5ec\uc131\ud68c\ub85c\uc11c, \ucd5c\ucd08\uc758 \uadf8\ub9ac\uc2a4 \ubb38\uc790\ub85c \uc774\ub984\uc9c0\uc5b4\uc9c4 \uc5ec\uc131\ud68c\uc774\ub2e4. \uc5d0\uc124 \ud5e4\uc9c0\uba3c \ub77c\uc77c\uc774 \uc8fc\ub3c4\ud55c 9\uba85\uc758 \ud559\uc0dd \ubaa8\uc784\uc5d0 \uc758\ud574 \uc6cc\uc2f1\ud134 D.C.\uc758 \ud558\uc6cc\ub4dc \ub300\ud559\uad50\uc5d0\uc11c 1908\ub144 1\uc6d4 15\uc77c \uc124\ub9bd\ub418\uc5c8\ub2e4. \uc5ec\uc131\ud68c\ub97c \uacb0\uc131\ud558\ub294 \uac83\uc740, 20\uc138\uae30 \ucd08 \uc18c\uc218\ubbfc\uc871 \ubc0f \uc5ec\uc131\ub4e4\uc774 \uae30\ud68c\uc758 \ubd88\ud3c9\ub4f1\uc73c\ub85c \uc778\ud558\uc5ec \ud798\uc774\ub098 \uad8c\uc704\uac00 \uac70\uc758 \uc5c6\ub358 \uc9c0\uc5ed\ub4e4\uc5d0\uc11c \uc544\ud504\ub9ac\uce74\uacc4 \ubbf8\uad6d\uc778 \uc5ec\uc131\uc5d0 \ub300\ud55c \uc7a5\ubcbd\uc744 \ubb34\ub108\ub728\ub9ac\ub294 \ub370 \uae30\uc5ec\ud588\ub2e4. AKA\ub294 1913\ub144 1\uc6d4 29\uc77c \ubc95\uc778\uc774 \ub418\uc5c8\ub2e4. \uc544\ud504\ub9ac\uce74\uacc4, \uce74\ud504\uce74\uc2a4\uacc4, \uc544\uc2dc\uc544\uacc4, \ud788\uc2a4\ud328\ub2c9\uc758 \uc5ec\ub300\uc0dd\ub4e4\ub85c \uad6c\uc131\ub41c \uc774 \uc5ec\uc131\ud68c\uc5d0\ub294 \ubbf8\uad6d \ubc0f \uc5ec\ub7ec \ub098\ub77c\uc5d0 975\uac1c \uc774\uc0c1\uc758 \uc9c0\ubd80\uac00 \uc788\uc73c\uba70, 20\ub9cc\uba85 \uc774\uc0c1\uc758 \ud68c\uc6d0\ub4e4\uc774 \uc788\ub2e4. \uc5ec\uc131\ub4e4\uc740 \ub300\ud559 \uc9c0\ubd80 \ub610\ub294 \ub300\ud559\uc6d0 \uc9c0\ubd80\ub97c \ud1b5\ud574 \uac00\uc785\ud560 \uc218 \uc788\ub2e4."} {"question": "1340\uc77c\uc758 \uc8fc\uae30\ub85c \ub3cc\uace0\uc788\ub294 \ud589\uc131\uc740 \ubaa9\uc131\uc9c8\ub7c9\uc758 \uba87 \ubc30\uc778\uac00?", "paragraph": "2009\ub144 11\uc6d4 6\uc77c \uc54c\uae30\uc5d0\ubc14\ub97c \ub3cc\uace0 \uc788\ub294 \uc678\uacc4 \ud589\uc131\uc758 \uc874\uc7ac\uac00 \uacf5\uc2dd\uc801\uc73c\ub85c \ubc1c\ud45c\ub418\uc5c8\ub2e4. \uc5b4\uba38\ub2c8 \ud56d\uc131\uacfc\uc758 \uac70\ub9ac\ub294 \uc9c0\uad6c~\ud0dc\uc591 \uc0ac\uc774 \uac04\uaca9\uacfc \ube44\uc2b7\ud558\ub098, \uc54c\uae30\uc5d0\ubc14\ub294 \ud0dc\uc591\ubcf4\ub2e4 \ud6e8\uc52c \ub354 \ub728\uac81\uae30 \ub54c\ubb38\uc5d0 \ud589\uc131\uc774 \ubc1b\ub294 \uc5f4\uae30\ub294 \uc9c0\uad6c\uac00 \ud0dc\uc591\uc5d0 0.066 \ucc9c\ubb38 \ub2e8\uc704\uae4c\uc9c0 \uc811\uadfc\ud588\uc744 \ub54c\uc640 \ube44\uc2b7\ud560 \uac83\uc774\ub2e4. \uc54c\uae30\uc5d0\ubc14\uc758 \uc2dc\uc120 \uc18d\ub3c4\uc5d0\uc11c\ub294 \ud06c\uac8c 8.5\uc77c\uacfc 1340\uc77c\uc758 \uac15\ud55c \uc8fc\uae30\uc131\uc774 \uad00\uce21\ub418\uace0 \uc788\ub2e4. \uc804\uc790\ub294 \ud56d\uc131\uc758 \ub300\uae30 \ud65c\ub3d9 \ub54c\ubb38\uc5d0 \uc0dd\uae34 \uac83\uc774\uba70, \ud6c4\uc790\ub294 \ubaa9\uc131\uc9c8\ub7c9 2.14\ubc30\uc758 \ub610\ub2e4\ub978 \ud589\uc131\uc774 \ud56d\uc131\uc73c\ub85c\ubd80\ud130 2.6 \ucc9c\ubb38\ub2e8\uc704 \ub5a8\uc5b4\uc838 \uc57d\uac04 \ucc0c\uadf8\ub7ec\uc9c4 \uada4\ub3c4(\uc774\uc2ec\ub960 0.13)\ub97c \uadf8\ub9ac\uba70 \ub3cc\uace0 \uc788\uc74c\uc744 \uc554\uc2dc\ud558\uace0 \uc788\ub2e4. \ub2e4\ub9cc \uc774\ub294 \uc5b4\ub514\uae4c\uc9c0\ub098 \ucd94\uce21\uc77c \ubfd0 \uc815\ud655\ud55c \uacb0\uacfc\uac00 \ub098\uc624\ub824\uba74 \uc2dc\uac04\uc774 \ub354 \ud544\uc694\ud558\ub2e4.", "answer": "2.14\ubc30", "sentence": "\ud6c4\uc790\ub294 \ubaa9\uc131\uc9c8\ub7c9 2.14\ubc30 \uc758 \ub610\ub2e4\ub978 \ud589\uc131\uc774 \ud56d\uc131\uc73c\ub85c\ubd80\ud130 2.6 \ucc9c\ubb38\ub2e8\uc704 \ub5a8\uc5b4\uc838 \uc57d\uac04 \ucc0c\uadf8\ub7ec\uc9c4 \uada4\ub3c4(\uc774\uc2ec\ub960 0.13)\ub97c \uadf8\ub9ac\uba70 \ub3cc\uace0 \uc788\uc74c\uc744 \uc554\uc2dc\ud558\uace0 \uc788\ub2e4.", "paragraph_sentence": "2009\ub144 11\uc6d4 6\uc77c \uc54c\uae30\uc5d0\ubc14\ub97c \ub3cc\uace0 \uc788\ub294 \uc678\uacc4 \ud589\uc131\uc758 \uc874\uc7ac\uac00 \uacf5\uc2dd\uc801\uc73c\ub85c \ubc1c\ud45c\ub418\uc5c8\ub2e4. \uc5b4\uba38\ub2c8 \ud56d\uc131\uacfc\uc758 \uac70\ub9ac\ub294 \uc9c0\uad6c~\ud0dc\uc591 \uc0ac\uc774 \uac04\uaca9\uacfc \ube44\uc2b7\ud558\ub098, \uc54c\uae30\uc5d0\ubc14\ub294 \ud0dc\uc591\ubcf4\ub2e4 \ud6e8\uc52c \ub354 \ub728\uac81\uae30 \ub54c\ubb38\uc5d0 \ud589\uc131\uc774 \ubc1b\ub294 \uc5f4\uae30\ub294 \uc9c0\uad6c\uac00 \ud0dc\uc591\uc5d0 0.066 \ucc9c\ubb38 \ub2e8\uc704\uae4c\uc9c0 \uc811\uadfc\ud588\uc744 \ub54c\uc640 \ube44\uc2b7\ud560 \uac83\uc774\ub2e4. \uc54c\uae30\uc5d0\ubc14\uc758 \uc2dc\uc120 \uc18d\ub3c4\uc5d0\uc11c\ub294 \ud06c\uac8c 8.5\uc77c\uacfc 1340\uc77c\uc758 \uac15\ud55c \uc8fc\uae30\uc131\uc774 \uad00\uce21\ub418\uace0 \uc788\ub2e4. \uc804\uc790\ub294 \ud56d\uc131\uc758 \ub300\uae30 \ud65c\ub3d9 \ub54c\ubb38\uc5d0 \uc0dd\uae34 \uac83\uc774\uba70, \ud6c4\uc790\ub294 \ubaa9\uc131\uc9c8\ub7c9 2.14\ubc30 \uc758 \ub610\ub2e4\ub978 \ud589\uc131\uc774 \ud56d\uc131\uc73c\ub85c\ubd80\ud130 2.6 \ucc9c\ubb38\ub2e8\uc704 \ub5a8\uc5b4\uc838 \uc57d\uac04 \ucc0c\uadf8\ub7ec\uc9c4 \uada4\ub3c4(\uc774\uc2ec\ub960 0.13)\ub97c \uadf8\ub9ac\uba70 \ub3cc\uace0 \uc788\uc74c\uc744 \uc554\uc2dc\ud558\uace0 \uc788\ub2e4. \ub2e4\ub9cc \uc774\ub294 \uc5b4\ub514\uae4c\uc9c0\ub098 \ucd94\uce21\uc77c \ubfd0 \uc815\ud655\ud55c \uacb0\uacfc\uac00 \ub098\uc624\ub824\uba74 \uc2dc\uac04\uc774 \ub354 \ud544\uc694\ud558\ub2e4.", "paragraph_answer": "2009\ub144 11\uc6d4 6\uc77c \uc54c\uae30\uc5d0\ubc14\ub97c \ub3cc\uace0 \uc788\ub294 \uc678\uacc4 \ud589\uc131\uc758 \uc874\uc7ac\uac00 \uacf5\uc2dd\uc801\uc73c\ub85c \ubc1c\ud45c\ub418\uc5c8\ub2e4. \uc5b4\uba38\ub2c8 \ud56d\uc131\uacfc\uc758 \uac70\ub9ac\ub294 \uc9c0\uad6c~\ud0dc\uc591 \uc0ac\uc774 \uac04\uaca9\uacfc \ube44\uc2b7\ud558\ub098, \uc54c\uae30\uc5d0\ubc14\ub294 \ud0dc\uc591\ubcf4\ub2e4 \ud6e8\uc52c \ub354 \ub728\uac81\uae30 \ub54c\ubb38\uc5d0 \ud589\uc131\uc774 \ubc1b\ub294 \uc5f4\uae30\ub294 \uc9c0\uad6c\uac00 \ud0dc\uc591\uc5d0 0.066 \ucc9c\ubb38 \ub2e8\uc704\uae4c\uc9c0 \uc811\uadfc\ud588\uc744 \ub54c\uc640 \ube44\uc2b7\ud560 \uac83\uc774\ub2e4. \uc54c\uae30\uc5d0\ubc14\uc758 \uc2dc\uc120 \uc18d\ub3c4\uc5d0\uc11c\ub294 \ud06c\uac8c 8.5\uc77c\uacfc 1340\uc77c\uc758 \uac15\ud55c \uc8fc\uae30\uc131\uc774 \uad00\uce21\ub418\uace0 \uc788\ub2e4. \uc804\uc790\ub294 \ud56d\uc131\uc758 \ub300\uae30 \ud65c\ub3d9 \ub54c\ubb38\uc5d0 \uc0dd\uae34 \uac83\uc774\uba70, \ud6c4\uc790\ub294 \ubaa9\uc131\uc9c8\ub7c9 2.14\ubc30 \uc758 \ub610\ub2e4\ub978 \ud589\uc131\uc774 \ud56d\uc131\uc73c\ub85c\ubd80\ud130 2.6 \ucc9c\ubb38\ub2e8\uc704 \ub5a8\uc5b4\uc838 \uc57d\uac04 \ucc0c\uadf8\ub7ec\uc9c4 \uada4\ub3c4(\uc774\uc2ec\ub960 0.13)\ub97c \uadf8\ub9ac\uba70 \ub3cc\uace0 \uc788\uc74c\uc744 \uc554\uc2dc\ud558\uace0 \uc788\ub2e4. \ub2e4\ub9cc \uc774\ub294 \uc5b4\ub514\uae4c\uc9c0\ub098 \ucd94\uce21\uc77c \ubfd0 \uc815\ud655\ud55c \uacb0\uacfc\uac00 \ub098\uc624\ub824\uba74 \uc2dc\uac04\uc774 \ub354 \ud544\uc694\ud558\ub2e4.", "sentence_answer": "\ud6c4\uc790\ub294 \ubaa9\uc131\uc9c8\ub7c9 2.14\ubc30 \uc758 \ub610\ub2e4\ub978 \ud589\uc131\uc774 \ud56d\uc131\uc73c\ub85c\ubd80\ud130 2.6 \ucc9c\ubb38\ub2e8\uc704 \ub5a8\uc5b4\uc838 \uc57d\uac04 \ucc0c\uadf8\ub7ec\uc9c4 \uada4\ub3c4(\uc774\uc2ec\ub960 0.13)\ub97c \uadf8\ub9ac\uba70 \ub3cc\uace0 \uc788\uc74c\uc744 \uc554\uc2dc\ud558\uace0 \uc788\ub2e4.", "paragraph_id": "6521724-2-1", "paragraph_question": "question: 1340\uc77c\uc758 \uc8fc\uae30\ub85c \ub3cc\uace0\uc788\ub294 \ud589\uc131\uc740 \ubaa9\uc131\uc9c8\ub7c9\uc758 \uba87 \ubc30\uc778\uac00?, context: 2009\ub144 11\uc6d4 6\uc77c \uc54c\uae30\uc5d0\ubc14\ub97c \ub3cc\uace0 \uc788\ub294 \uc678\uacc4 \ud589\uc131\uc758 \uc874\uc7ac\uac00 \uacf5\uc2dd\uc801\uc73c\ub85c \ubc1c\ud45c\ub418\uc5c8\ub2e4. \uc5b4\uba38\ub2c8 \ud56d\uc131\uacfc\uc758 \uac70\ub9ac\ub294 \uc9c0\uad6c~\ud0dc\uc591 \uc0ac\uc774 \uac04\uaca9\uacfc \ube44\uc2b7\ud558\ub098, \uc54c\uae30\uc5d0\ubc14\ub294 \ud0dc\uc591\ubcf4\ub2e4 \ud6e8\uc52c \ub354 \ub728\uac81\uae30 \ub54c\ubb38\uc5d0 \ud589\uc131\uc774 \ubc1b\ub294 \uc5f4\uae30\ub294 \uc9c0\uad6c\uac00 \ud0dc\uc591\uc5d0 0.066 \ucc9c\ubb38 \ub2e8\uc704\uae4c\uc9c0 \uc811\uadfc\ud588\uc744 \ub54c\uc640 \ube44\uc2b7\ud560 \uac83\uc774\ub2e4. \uc54c\uae30\uc5d0\ubc14\uc758 \uc2dc\uc120 \uc18d\ub3c4\uc5d0\uc11c\ub294 \ud06c\uac8c 8.5\uc77c\uacfc 1340\uc77c\uc758 \uac15\ud55c \uc8fc\uae30\uc131\uc774 \uad00\uce21\ub418\uace0 \uc788\ub2e4. \uc804\uc790\ub294 \ud56d\uc131\uc758 \ub300\uae30 \ud65c\ub3d9 \ub54c\ubb38\uc5d0 \uc0dd\uae34 \uac83\uc774\uba70, \ud6c4\uc790\ub294 \ubaa9\uc131\uc9c8\ub7c9 2.14\ubc30\uc758 \ub610\ub2e4\ub978 \ud589\uc131\uc774 \ud56d\uc131\uc73c\ub85c\ubd80\ud130 2.6 \ucc9c\ubb38\ub2e8\uc704 \ub5a8\uc5b4\uc838 \uc57d\uac04 \ucc0c\uadf8\ub7ec\uc9c4 \uada4\ub3c4(\uc774\uc2ec\ub960 0.13)\ub97c \uadf8\ub9ac\uba70 \ub3cc\uace0 \uc788\uc74c\uc744 \uc554\uc2dc\ud558\uace0 \uc788\ub2e4. \ub2e4\ub9cc \uc774\ub294 \uc5b4\ub514\uae4c\uc9c0\ub098 \ucd94\uce21\uc77c \ubfd0 \uc815\ud655\ud55c \uacb0\uacfc\uac00 \ub098\uc624\ub824\uba74 \uc2dc\uac04\uc774 \ub354 \ud544\uc694\ud558\ub2e4."} {"question": "\uc6cc\ub108\uc6d0\uc758 \ud32c\ubbf8\ud305 \ucf58\uc11c\ud2b8 \uc81c\ubaa9\uc740?", "paragraph": "2017\ub144 8\uc6d4 7\uc77c, \ub370\ubdd4 \uc568\ubc94\uc774\uc790 \uccab \ubc88\uc9f8 \ubbf8\ub2c8 \uc568\ubc94 \u300a1X1=1 (TO BE ONE)\u300b\uc744 \ubc1c\ub9e4\ud558\uace0 \uace0\ucc99\uc2a4\uce74\uc774\ub3d4\uc5d0\uc11c \ub370\ubdd4 \uc1fc\ucf00\uc774\uc2a4 \ucf58\uc11c\ud2b8 \u3008\uc6cc\ub108\uc6d0 \ud504\ub9ac\ubbf8\uc5b4 \uc1fc\ucf58\u3009\uc744 \uac1c\ucd5c\ud558\uba70 \uacf5\uc2dd\uc801\uc73c\ub85c \ub370\ubdd4\ud558\uc600\ub2e4. \ubc1c\ub9e4 \uc9c1\ud6c4, \ud0c0\uc774\ud2c0\uace1 '\uc5d0\ub108\uc81c\ud2f1 (Energetic)'\uc740 \ubaa8\ub4e0 \uc74c\uc6d0 \ucc28\ud2b8\uc5d0\uc11c 1\uc704\ub97c \uae30\ub85d\ud558\uace0, '\ud65c\ud65c (Burn It Up)'\uc744 \ube44\ub86f\ud55c \uc218\ub85d\uace1\ub4e4\ub3c4 10\uc704\uad8c \ub0b4\uc5d0 \uc9c4\uc785\ud558\uc600\ub2e4. \uc74c\ubc18\ub3c4 \ubaa8\ub4e0 \uc74c\ubc18 \ucc28\ud2b8\uc5d0\uc11c 1\uc704\ub97c \uae30\ub85d\ud558\uace0, \ubc1c\ub9e4 \uccab \uc8fc 41\ub9cc \uc7a5\uc744 \ud310\ub9e4\ud574 \uc5ed\ub300 \ub300\ud55c\ubbfc\uad6d \uc544\uc774\ub3cc \uadf8\ub8f9 \uc74c\ubc18 \ucd08\ub3d9 5\uc704 (\ubc1c\ub9e4 \ub2f9\uc2dc \ucd08\ub3d9 3\uc704)\ub97c \uae30\ub85d\ud588\uc73c\uba70, \ud604\uc7ac\uae4c\uc9c0 76\ub9cc \uc7a5\uc744 \ud310\ub9e4\ud558\uc600\ub2e4. \ub610\ud55c, \uc74c\uc545 \ubc29\uc1a1\uc5d0\uc11c 1\uc704\ub97c \ud729\uc4f8\uc5b4 15\uad00\uc655\uc744 \ub2ec\uc131\ud588\ub2e4. \uac19\uc740 \ud574 11\uc6d4 13\uc77c, \uccab \ubc88\uc9f8 \ub9ac\ud328\ud0a4\uc9c0 \uc568\ubc94 \u300a1-1=0 (NOTHING WITHOUT YOU)\u300b\ub97c \ubc1c\ub9e4\ud558\uc600\ub2e4. \ubc1c\ub9e4 \uc9c1\ud6c4, \ud0c0\uc774\ud2c0\uace1 'Beautiful'\uc740 \ubaa8\ub4e0 \uc74c\uc6d0 \ucc28\ud2b8\uc5d0\uc11c 1\uc704\ub97c \uae30\ub85d\ud558\uace0, '\uac16\uace0 \uc2f6\uc5b4'\ub97c \ube44\ub86f\ud55c \uc218\ub85d\uace1\ub4e4\ub3c4 10\uc704\uad8c \ub0b4\uc5d0 \uc9c4\uc785\ud558\uc600\ub2e4. \uc74c\ubc18\ub3c4 \ubaa8\ub4e0 \uc74c\ubc18 \ucc28\ud2b8\uc5d0\uc11c 1\uc704\ub97c \uae30\ub85d\ud558\uace0, \ubc1c\ub9e4 \uccab \uc8fc 41\ub9cc \uc7a5\uc744 \ud310\ub9e4\ud574 \uc5ed\ub300 \ub300\ud55c\ubbfc\uad6d \uc544\uc774\ub3cc \uadf8\ub8f9 \uc74c\ubc18 \ucd08\ub3d9 4\uc704\ub97c \uae30\ub85d\ud588\uc73c\uba70, \ud604\uc7ac\uae4c\uc9c0 64\ub9cc \uc7a5\uc744 \ud310\ub9e4\ud558\uc600\ub2e4. \ub610\ud55c, \uc74c\uc545 \ubc29\uc1a1\uc5d0\uc11c 1\uc704\ub97c \ud729\uc4f8\uc5b4 8\uad00\uc655\uc744 \ub2ec\uc131\ud588\ub2e4. \uac19\uc740 \ud574 12\uc6d4 9\uc77c, \uacf5\uc2dd \uc751\uc6d0\ubd09 \ub108\ube14\ubd09\uc744 \ube44\ub86f\ud55c \uacf5\uc2dd \uad7f\uc988\uac00 \uacf5\uac1c\ub418\uc5c8\uc73c\uba70, 12\uc6d4 15\uc77c\ubd80\ud130 17\uc77c, 23\uc77c\ubd80\ud130 24\uc77c\uae4c\uc9c0 \ud32c\ubbf8\ud305 \ucf58\uc11c\ud2b8 \u3008\uc6cc\ub108\uc6d0 \ud504\ub9ac\ubbf8\uc5b4 \ud32c\ucf58\u3009\uc744 \uac1c\ucd5c\ud558\uc600\ub2e4. 2017\ub144 \ud55c \ud574 \ub3d9\uc548 \uc6cc\ub108\uc6d0\uc740 \ubd80\uc0b0 \uc6d0\uc544\uc2dc\uc544\ud398\uc2a4\ud2f0\ubc8c \uc5b4\uc6cc\uc988\uc5d0\uc11c \ub77c\uc774\uc9d5 \uc2a4\ud0c0\uc0c1\uc744 \uc218\uc0c1\ud558\uace0, \uc8fc\uac04\uc544\uc774\ub3cc \uc5b4\uc6cc\uc988\uc5d0\uc11c \uc544\uc774\ub3cc \ucc54\ud504\uc0c1\uc744 \uc218\uc0c1\ud588\uc73c\uba70, \ub300\ud55c\ubbfc\uad6d \ud1b1\uc2a4\ud0c0\uc0c1 \uc2dc\uc0c1\uc2dd\uc5d0\uc11c \ud1b1\uac00\uc218\uc0c1\uc744 \uc218\uc0c1\ud588\ub2e4. \ub610\ud55c, \uc18c\ub9ac\ubc14\ub2e4 \ubca0\uc2a4\ud2b8 \ucf00\uc774\ubba4\uc9c1 \uc5b4\uc6cc\uc988\uc5d0\uc11c \uc2e0\ud55c\ub958 \uc2e0\uc778\uc0c1\uacfc \uc2e0\ud55c\ub958 \ub77c\uc774\uc9d5 \ud56b \uc2a4\ud0c0\uc0c1\uc744 \uc218\uc0c1\ud558\uace0, \uc544\uc2dc\uc544 \uc544\ud2f0\uc2a4\ud2b8 \uc5b4\uc6cc\uc988\uc5d0\uc11c\ub3c4 \uc2e0\uc778\uc0c1\uacfc \uc288\ud37c\ub8e8\ud0a4 \uc0bc\uc131\ud398\uc774\uc0c1\uc744 \uc218\uc0c1\ud574 \uac01\uac01 2\uad00\uc655\uc744 \ub2ec\uc131\ud588\ub2e4. \uc774\uc5b4 \uc5e0\ub137 \uc544\uc2dc\uc548 \ubba4\uc9c1 \uc5b4\uc6cc\ub4dc\uc5d0\uc11c \ub0a8\uc790 \uadf8\ub8f9\uc0c1, \ub0a8\uc790 \uc2e0\uc778\uc0c1, \ubca0\uc2a4\ud2b8 \uc624\ube0c \ub125\uc2a4\ud2b8\uc0c1\uc744 \uc218\uc0c1\ud558\uace0, \uba5c\ub860 \ubba4\uc9c1 \uc5b4\uc6cc\ub4dc\uc5d0\uc11c\ub3c4 Top 10, \uc2e0\uc778\uc0c1, \uce74\uce74\uc624 \ud56b \uc2a4\ud0c0\uc0c1\uc744 \uc218\uc0c1\ud574 \uac01\uac01 3\uad00\uc655\uc744 \ub2ec\uc131\ud588\ub2e4. \uc6cc\ub108\uc6d0\uc740 \ub370\ubdd4 \uc568\ubc94\uacfc \ub9ac\ud328\ud0a4\uc9c0 \uc568\ubc94 \ud310\ub9e4\ub7c9\uc774 100\ub9cc\uc7a5\uc744 \ub118\uc5b4 2000\ub144 \uc774\ud6c4 \ucd5c\ucd08\ub85c \ub300\ud55c\ubbfc\uad6d \uc544\uc774\ub3cc \uadf8\ub8f9 \ub370\ubdd4 \uc568\ubc94 \ubc00\ub9ac\uc5b8\uc140\ub7ec\uac00 \ub418\uc5c8\ub2e4.", "answer": "\uc6cc\ub108\uc6d0 \ud504\ub9ac\ubbf8\uc5b4 \ud32c\ucf58", "sentence": "\uac19\uc740 \ud574 12\uc6d4 9\uc77c, \uacf5\uc2dd \uc751\uc6d0\ubd09 \ub108\ube14\ubd09\uc744 \ube44\ub86f\ud55c \uacf5\uc2dd \uad7f\uc988\uac00 \uacf5\uac1c\ub418\uc5c8\uc73c\uba70, 12\uc6d4 15\uc77c\ubd80\ud130 17\uc77c, 23\uc77c\ubd80\ud130 24\uc77c\uae4c\uc9c0 \ud32c\ubbf8\ud305 \ucf58\uc11c\ud2b8 \u3008 \uc6cc\ub108\uc6d0 \ud504\ub9ac\ubbf8\uc5b4 \ud32c\ucf58 \u3009\uc744 \uac1c\ucd5c\ud558\uc600\ub2e4.", "paragraph_sentence": "2017\ub144 8\uc6d4 7\uc77c, \ub370\ubdd4 \uc568\ubc94\uc774\uc790 \uccab \ubc88\uc9f8 \ubbf8\ub2c8 \uc568\ubc94 \u300a1X1=1 (TO BE ONE)\u300b\uc744 \ubc1c\ub9e4\ud558\uace0 \uace0\ucc99\uc2a4\uce74\uc774\ub3d4\uc5d0\uc11c \ub370\ubdd4 \uc1fc\ucf00\uc774\uc2a4 \ucf58\uc11c\ud2b8 \u3008\uc6cc\ub108\uc6d0 \ud504\ub9ac\ubbf8\uc5b4 \uc1fc\ucf58\u3009\uc744 \uac1c\ucd5c\ud558\uba70 \uacf5\uc2dd\uc801\uc73c\ub85c \ub370\ubdd4\ud558\uc600\ub2e4. \ubc1c\ub9e4 \uc9c1\ud6c4, \ud0c0\uc774\ud2c0\uace1 '\uc5d0\ub108\uc81c\ud2f1 (Energetic)'\uc740 \ubaa8\ub4e0 \uc74c\uc6d0 \ucc28\ud2b8\uc5d0\uc11c 1\uc704\ub97c \uae30\ub85d\ud558\uace0, '\ud65c\ud65c (Burn It Up)'\uc744 \ube44\ub86f\ud55c \uc218\ub85d\uace1\ub4e4\ub3c4 10\uc704\uad8c \ub0b4\uc5d0 \uc9c4\uc785\ud558\uc600\ub2e4. \uc74c\ubc18\ub3c4 \ubaa8\ub4e0 \uc74c\ubc18 \ucc28\ud2b8\uc5d0\uc11c 1\uc704\ub97c \uae30\ub85d\ud558\uace0, \ubc1c\ub9e4 \uccab \uc8fc 41\ub9cc \uc7a5\uc744 \ud310\ub9e4\ud574 \uc5ed\ub300 \ub300\ud55c\ubbfc\uad6d \uc544\uc774\ub3cc \uadf8\ub8f9 \uc74c\ubc18 \ucd08\ub3d9 5\uc704 (\ubc1c\ub9e4 \ub2f9\uc2dc \ucd08\ub3d9 3\uc704)\ub97c \uae30\ub85d\ud588\uc73c\uba70, \ud604\uc7ac\uae4c\uc9c0 76\ub9cc \uc7a5\uc744 \ud310\ub9e4\ud558\uc600\ub2e4. \ub610\ud55c, \uc74c\uc545 \ubc29\uc1a1\uc5d0\uc11c 1\uc704\ub97c \ud729\uc4f8\uc5b4 15\uad00\uc655\uc744 \ub2ec\uc131\ud588\ub2e4. \uac19\uc740 \ud574 11\uc6d4 13\uc77c, \uccab \ubc88\uc9f8 \ub9ac\ud328\ud0a4\uc9c0 \uc568\ubc94 \u300a1-1=0 (NOTHING WITHOUT YOU)\u300b\ub97c \ubc1c\ub9e4\ud558\uc600\ub2e4. \ubc1c\ub9e4 \uc9c1\ud6c4, \ud0c0\uc774\ud2c0\uace1 'Beautiful'\uc740 \ubaa8\ub4e0 \uc74c\uc6d0 \ucc28\ud2b8\uc5d0\uc11c 1\uc704\ub97c \uae30\ub85d\ud558\uace0, '\uac16\uace0 \uc2f6\uc5b4'\ub97c \ube44\ub86f\ud55c \uc218\ub85d\uace1\ub4e4\ub3c4 10\uc704\uad8c \ub0b4\uc5d0 \uc9c4\uc785\ud558\uc600\ub2e4. \uc74c\ubc18\ub3c4 \ubaa8\ub4e0 \uc74c\ubc18 \ucc28\ud2b8\uc5d0\uc11c 1\uc704\ub97c \uae30\ub85d\ud558\uace0, \ubc1c\ub9e4 \uccab \uc8fc 41\ub9cc \uc7a5\uc744 \ud310\ub9e4\ud574 \uc5ed\ub300 \ub300\ud55c\ubbfc\uad6d \uc544\uc774\ub3cc \uadf8\ub8f9 \uc74c\ubc18 \ucd08\ub3d9 4\uc704\ub97c \uae30\ub85d\ud588\uc73c\uba70, \ud604\uc7ac\uae4c\uc9c0 64\ub9cc \uc7a5\uc744 \ud310\ub9e4\ud558\uc600\ub2e4. \ub610\ud55c, \uc74c\uc545 \ubc29\uc1a1\uc5d0\uc11c 1\uc704\ub97c \ud729\uc4f8\uc5b4 8\uad00\uc655\uc744 \ub2ec\uc131\ud588\ub2e4. \uac19\uc740 \ud574 12\uc6d4 9\uc77c, \uacf5\uc2dd \uc751\uc6d0\ubd09 \ub108\ube14\ubd09\uc744 \ube44\ub86f\ud55c \uacf5\uc2dd \uad7f\uc988\uac00 \uacf5\uac1c\ub418\uc5c8\uc73c\uba70, 12\uc6d4 15\uc77c\ubd80\ud130 17\uc77c, 23\uc77c\ubd80\ud130 24\uc77c\uae4c\uc9c0 \ud32c\ubbf8\ud305 \ucf58\uc11c\ud2b8 \u3008 \uc6cc\ub108\uc6d0 \ud504\ub9ac\ubbf8\uc5b4 \ud32c\ucf58 \u3009\uc744 \uac1c\ucd5c\ud558\uc600\ub2e4. 2017\ub144 \ud55c \ud574 \ub3d9\uc548 \uc6cc\ub108\uc6d0\uc740 \ubd80\uc0b0 \uc6d0\uc544\uc2dc\uc544\ud398\uc2a4\ud2f0\ubc8c \uc5b4\uc6cc\uc988\uc5d0\uc11c \ub77c\uc774\uc9d5 \uc2a4\ud0c0\uc0c1\uc744 \uc218\uc0c1\ud558\uace0, \uc8fc\uac04\uc544\uc774\ub3cc \uc5b4\uc6cc\uc988\uc5d0\uc11c \uc544\uc774\ub3cc \ucc54\ud504\uc0c1\uc744 \uc218\uc0c1\ud588\uc73c\uba70, \ub300\ud55c\ubbfc\uad6d \ud1b1\uc2a4\ud0c0\uc0c1 \uc2dc\uc0c1\uc2dd\uc5d0\uc11c \ud1b1\uac00\uc218\uc0c1\uc744 \uc218\uc0c1\ud588\ub2e4. \ub610\ud55c, \uc18c\ub9ac\ubc14\ub2e4 \ubca0\uc2a4\ud2b8 \ucf00\uc774\ubba4\uc9c1 \uc5b4\uc6cc\uc988\uc5d0\uc11c \uc2e0\ud55c\ub958 \uc2e0\uc778\uc0c1\uacfc \uc2e0\ud55c\ub958 \ub77c\uc774\uc9d5 \ud56b \uc2a4\ud0c0\uc0c1\uc744 \uc218\uc0c1\ud558\uace0, \uc544\uc2dc\uc544 \uc544\ud2f0\uc2a4\ud2b8 \uc5b4\uc6cc\uc988\uc5d0\uc11c\ub3c4 \uc2e0\uc778\uc0c1\uacfc \uc288\ud37c\ub8e8\ud0a4 \uc0bc\uc131\ud398\uc774\uc0c1\uc744 \uc218\uc0c1\ud574 \uac01\uac01 2\uad00\uc655\uc744 \ub2ec\uc131\ud588\ub2e4. \uc774\uc5b4 \uc5e0\ub137 \uc544\uc2dc\uc548 \ubba4\uc9c1 \uc5b4\uc6cc\ub4dc\uc5d0\uc11c \ub0a8\uc790 \uadf8\ub8f9\uc0c1, \ub0a8\uc790 \uc2e0\uc778\uc0c1, \ubca0\uc2a4\ud2b8 \uc624\ube0c \ub125\uc2a4\ud2b8\uc0c1\uc744 \uc218\uc0c1\ud558\uace0, \uba5c\ub860 \ubba4\uc9c1 \uc5b4\uc6cc\ub4dc\uc5d0\uc11c\ub3c4 Top 10, \uc2e0\uc778\uc0c1, \uce74\uce74\uc624 \ud56b \uc2a4\ud0c0\uc0c1\uc744 \uc218\uc0c1\ud574 \uac01\uac01 3\uad00\uc655\uc744 \ub2ec\uc131\ud588\ub2e4. \uc6cc\ub108\uc6d0\uc740 \ub370\ubdd4 \uc568\ubc94\uacfc \ub9ac\ud328\ud0a4\uc9c0 \uc568\ubc94 \ud310\ub9e4\ub7c9\uc774 100\ub9cc\uc7a5\uc744 \ub118\uc5b4 2000\ub144 \uc774\ud6c4 \ucd5c\ucd08\ub85c \ub300\ud55c\ubbfc\uad6d \uc544\uc774\ub3cc \uadf8\ub8f9 \ub370\ubdd4 \uc568\ubc94 \ubc00\ub9ac\uc5b8\uc140\ub7ec\uac00 \ub418\uc5c8\ub2e4.", "paragraph_answer": "2017\ub144 8\uc6d4 7\uc77c, \ub370\ubdd4 \uc568\ubc94\uc774\uc790 \uccab \ubc88\uc9f8 \ubbf8\ub2c8 \uc568\ubc94 \u300a1X1=1 (TO BE ONE)\u300b\uc744 \ubc1c\ub9e4\ud558\uace0 \uace0\ucc99\uc2a4\uce74\uc774\ub3d4\uc5d0\uc11c \ub370\ubdd4 \uc1fc\ucf00\uc774\uc2a4 \ucf58\uc11c\ud2b8 \u3008\uc6cc\ub108\uc6d0 \ud504\ub9ac\ubbf8\uc5b4 \uc1fc\ucf58\u3009\uc744 \uac1c\ucd5c\ud558\uba70 \uacf5\uc2dd\uc801\uc73c\ub85c \ub370\ubdd4\ud558\uc600\ub2e4. \ubc1c\ub9e4 \uc9c1\ud6c4, \ud0c0\uc774\ud2c0\uace1 '\uc5d0\ub108\uc81c\ud2f1 (Energetic)'\uc740 \ubaa8\ub4e0 \uc74c\uc6d0 \ucc28\ud2b8\uc5d0\uc11c 1\uc704\ub97c \uae30\ub85d\ud558\uace0, '\ud65c\ud65c (Burn It Up)'\uc744 \ube44\ub86f\ud55c \uc218\ub85d\uace1\ub4e4\ub3c4 10\uc704\uad8c \ub0b4\uc5d0 \uc9c4\uc785\ud558\uc600\ub2e4. \uc74c\ubc18\ub3c4 \ubaa8\ub4e0 \uc74c\ubc18 \ucc28\ud2b8\uc5d0\uc11c 1\uc704\ub97c \uae30\ub85d\ud558\uace0, \ubc1c\ub9e4 \uccab \uc8fc 41\ub9cc \uc7a5\uc744 \ud310\ub9e4\ud574 \uc5ed\ub300 \ub300\ud55c\ubbfc\uad6d \uc544\uc774\ub3cc \uadf8\ub8f9 \uc74c\ubc18 \ucd08\ub3d9 5\uc704 (\ubc1c\ub9e4 \ub2f9\uc2dc \ucd08\ub3d9 3\uc704)\ub97c \uae30\ub85d\ud588\uc73c\uba70, \ud604\uc7ac\uae4c\uc9c0 76\ub9cc \uc7a5\uc744 \ud310\ub9e4\ud558\uc600\ub2e4. \ub610\ud55c, \uc74c\uc545 \ubc29\uc1a1\uc5d0\uc11c 1\uc704\ub97c \ud729\uc4f8\uc5b4 15\uad00\uc655\uc744 \ub2ec\uc131\ud588\ub2e4. \uac19\uc740 \ud574 11\uc6d4 13\uc77c, \uccab \ubc88\uc9f8 \ub9ac\ud328\ud0a4\uc9c0 \uc568\ubc94 \u300a1-1=0 (NOTHING WITHOUT YOU)\u300b\ub97c \ubc1c\ub9e4\ud558\uc600\ub2e4. \ubc1c\ub9e4 \uc9c1\ud6c4, \ud0c0\uc774\ud2c0\uace1 'Beautiful'\uc740 \ubaa8\ub4e0 \uc74c\uc6d0 \ucc28\ud2b8\uc5d0\uc11c 1\uc704\ub97c \uae30\ub85d\ud558\uace0, '\uac16\uace0 \uc2f6\uc5b4'\ub97c \ube44\ub86f\ud55c \uc218\ub85d\uace1\ub4e4\ub3c4 10\uc704\uad8c \ub0b4\uc5d0 \uc9c4\uc785\ud558\uc600\ub2e4. \uc74c\ubc18\ub3c4 \ubaa8\ub4e0 \uc74c\ubc18 \ucc28\ud2b8\uc5d0\uc11c 1\uc704\ub97c \uae30\ub85d\ud558\uace0, \ubc1c\ub9e4 \uccab \uc8fc 41\ub9cc \uc7a5\uc744 \ud310\ub9e4\ud574 \uc5ed\ub300 \ub300\ud55c\ubbfc\uad6d \uc544\uc774\ub3cc \uadf8\ub8f9 \uc74c\ubc18 \ucd08\ub3d9 4\uc704\ub97c \uae30\ub85d\ud588\uc73c\uba70, \ud604\uc7ac\uae4c\uc9c0 64\ub9cc \uc7a5\uc744 \ud310\ub9e4\ud558\uc600\ub2e4. \ub610\ud55c, \uc74c\uc545 \ubc29\uc1a1\uc5d0\uc11c 1\uc704\ub97c \ud729\uc4f8\uc5b4 8\uad00\uc655\uc744 \ub2ec\uc131\ud588\ub2e4. \uac19\uc740 \ud574 12\uc6d4 9\uc77c, \uacf5\uc2dd \uc751\uc6d0\ubd09 \ub108\ube14\ubd09\uc744 \ube44\ub86f\ud55c \uacf5\uc2dd \uad7f\uc988\uac00 \uacf5\uac1c\ub418\uc5c8\uc73c\uba70, 12\uc6d4 15\uc77c\ubd80\ud130 17\uc77c, 23\uc77c\ubd80\ud130 24\uc77c\uae4c\uc9c0 \ud32c\ubbf8\ud305 \ucf58\uc11c\ud2b8 \u3008 \uc6cc\ub108\uc6d0 \ud504\ub9ac\ubbf8\uc5b4 \ud32c\ucf58 \u3009\uc744 \uac1c\ucd5c\ud558\uc600\ub2e4. 2017\ub144 \ud55c \ud574 \ub3d9\uc548 \uc6cc\ub108\uc6d0\uc740 \ubd80\uc0b0 \uc6d0\uc544\uc2dc\uc544\ud398\uc2a4\ud2f0\ubc8c \uc5b4\uc6cc\uc988\uc5d0\uc11c \ub77c\uc774\uc9d5 \uc2a4\ud0c0\uc0c1\uc744 \uc218\uc0c1\ud558\uace0, \uc8fc\uac04\uc544\uc774\ub3cc \uc5b4\uc6cc\uc988\uc5d0\uc11c \uc544\uc774\ub3cc \ucc54\ud504\uc0c1\uc744 \uc218\uc0c1\ud588\uc73c\uba70, \ub300\ud55c\ubbfc\uad6d \ud1b1\uc2a4\ud0c0\uc0c1 \uc2dc\uc0c1\uc2dd\uc5d0\uc11c \ud1b1\uac00\uc218\uc0c1\uc744 \uc218\uc0c1\ud588\ub2e4. \ub610\ud55c, \uc18c\ub9ac\ubc14\ub2e4 \ubca0\uc2a4\ud2b8 \ucf00\uc774\ubba4\uc9c1 \uc5b4\uc6cc\uc988\uc5d0\uc11c \uc2e0\ud55c\ub958 \uc2e0\uc778\uc0c1\uacfc \uc2e0\ud55c\ub958 \ub77c\uc774\uc9d5 \ud56b \uc2a4\ud0c0\uc0c1\uc744 \uc218\uc0c1\ud558\uace0, \uc544\uc2dc\uc544 \uc544\ud2f0\uc2a4\ud2b8 \uc5b4\uc6cc\uc988\uc5d0\uc11c\ub3c4 \uc2e0\uc778\uc0c1\uacfc \uc288\ud37c\ub8e8\ud0a4 \uc0bc\uc131\ud398\uc774\uc0c1\uc744 \uc218\uc0c1\ud574 \uac01\uac01 2\uad00\uc655\uc744 \ub2ec\uc131\ud588\ub2e4. \uc774\uc5b4 \uc5e0\ub137 \uc544\uc2dc\uc548 \ubba4\uc9c1 \uc5b4\uc6cc\ub4dc\uc5d0\uc11c \ub0a8\uc790 \uadf8\ub8f9\uc0c1, \ub0a8\uc790 \uc2e0\uc778\uc0c1, \ubca0\uc2a4\ud2b8 \uc624\ube0c \ub125\uc2a4\ud2b8\uc0c1\uc744 \uc218\uc0c1\ud558\uace0, \uba5c\ub860 \ubba4\uc9c1 \uc5b4\uc6cc\ub4dc\uc5d0\uc11c\ub3c4 Top 10, \uc2e0\uc778\uc0c1, \uce74\uce74\uc624 \ud56b \uc2a4\ud0c0\uc0c1\uc744 \uc218\uc0c1\ud574 \uac01\uac01 3\uad00\uc655\uc744 \ub2ec\uc131\ud588\ub2e4. \uc6cc\ub108\uc6d0\uc740 \ub370\ubdd4 \uc568\ubc94\uacfc \ub9ac\ud328\ud0a4\uc9c0 \uc568\ubc94 \ud310\ub9e4\ub7c9\uc774 100\ub9cc\uc7a5\uc744 \ub118\uc5b4 2000\ub144 \uc774\ud6c4 \ucd5c\ucd08\ub85c \ub300\ud55c\ubbfc\uad6d \uc544\uc774\ub3cc \uadf8\ub8f9 \ub370\ubdd4 \uc568\ubc94 \ubc00\ub9ac\uc5b8\uc140\ub7ec\uac00 \ub418\uc5c8\ub2e4.", "sentence_answer": "\uac19\uc740 \ud574 12\uc6d4 9\uc77c, \uacf5\uc2dd \uc751\uc6d0\ubd09 \ub108\ube14\ubd09\uc744 \ube44\ub86f\ud55c \uacf5\uc2dd \uad7f\uc988\uac00 \uacf5\uac1c\ub418\uc5c8\uc73c\uba70, 12\uc6d4 15\uc77c\ubd80\ud130 17\uc77c, 23\uc77c\ubd80\ud130 24\uc77c\uae4c\uc9c0 \ud32c\ubbf8\ud305 \ucf58\uc11c\ud2b8 \u3008 \uc6cc\ub108\uc6d0 \ud504\ub9ac\ubbf8\uc5b4 \ud32c\ucf58 \u3009\uc744 \uac1c\ucd5c\ud558\uc600\ub2e4.", "paragraph_id": "5824183-1-2", "paragraph_question": "question: \uc6cc\ub108\uc6d0\uc758 \ud32c\ubbf8\ud305 \ucf58\uc11c\ud2b8 \uc81c\ubaa9\uc740?, context: 2017\ub144 8\uc6d4 7\uc77c, \ub370\ubdd4 \uc568\ubc94\uc774\uc790 \uccab \ubc88\uc9f8 \ubbf8\ub2c8 \uc568\ubc94 \u300a1X1=1 (TO BE ONE)\u300b\uc744 \ubc1c\ub9e4\ud558\uace0 \uace0\ucc99\uc2a4\uce74\uc774\ub3d4\uc5d0\uc11c \ub370\ubdd4 \uc1fc\ucf00\uc774\uc2a4 \ucf58\uc11c\ud2b8 \u3008\uc6cc\ub108\uc6d0 \ud504\ub9ac\ubbf8\uc5b4 \uc1fc\ucf58\u3009\uc744 \uac1c\ucd5c\ud558\uba70 \uacf5\uc2dd\uc801\uc73c\ub85c \ub370\ubdd4\ud558\uc600\ub2e4. \ubc1c\ub9e4 \uc9c1\ud6c4, \ud0c0\uc774\ud2c0\uace1 '\uc5d0\ub108\uc81c\ud2f1 (Energetic)'\uc740 \ubaa8\ub4e0 \uc74c\uc6d0 \ucc28\ud2b8\uc5d0\uc11c 1\uc704\ub97c \uae30\ub85d\ud558\uace0, '\ud65c\ud65c (Burn It Up)'\uc744 \ube44\ub86f\ud55c \uc218\ub85d\uace1\ub4e4\ub3c4 10\uc704\uad8c \ub0b4\uc5d0 \uc9c4\uc785\ud558\uc600\ub2e4. \uc74c\ubc18\ub3c4 \ubaa8\ub4e0 \uc74c\ubc18 \ucc28\ud2b8\uc5d0\uc11c 1\uc704\ub97c \uae30\ub85d\ud558\uace0, \ubc1c\ub9e4 \uccab \uc8fc 41\ub9cc \uc7a5\uc744 \ud310\ub9e4\ud574 \uc5ed\ub300 \ub300\ud55c\ubbfc\uad6d \uc544\uc774\ub3cc \uadf8\ub8f9 \uc74c\ubc18 \ucd08\ub3d9 5\uc704 (\ubc1c\ub9e4 \ub2f9\uc2dc \ucd08\ub3d9 3\uc704)\ub97c \uae30\ub85d\ud588\uc73c\uba70, \ud604\uc7ac\uae4c\uc9c0 76\ub9cc \uc7a5\uc744 \ud310\ub9e4\ud558\uc600\ub2e4. \ub610\ud55c, \uc74c\uc545 \ubc29\uc1a1\uc5d0\uc11c 1\uc704\ub97c \ud729\uc4f8\uc5b4 15\uad00\uc655\uc744 \ub2ec\uc131\ud588\ub2e4. \uac19\uc740 \ud574 11\uc6d4 13\uc77c, \uccab \ubc88\uc9f8 \ub9ac\ud328\ud0a4\uc9c0 \uc568\ubc94 \u300a1-1=0 (NOTHING WITHOUT YOU)\u300b\ub97c \ubc1c\ub9e4\ud558\uc600\ub2e4. \ubc1c\ub9e4 \uc9c1\ud6c4, \ud0c0\uc774\ud2c0\uace1 'Beautiful'\uc740 \ubaa8\ub4e0 \uc74c\uc6d0 \ucc28\ud2b8\uc5d0\uc11c 1\uc704\ub97c \uae30\ub85d\ud558\uace0, '\uac16\uace0 \uc2f6\uc5b4'\ub97c \ube44\ub86f\ud55c \uc218\ub85d\uace1\ub4e4\ub3c4 10\uc704\uad8c \ub0b4\uc5d0 \uc9c4\uc785\ud558\uc600\ub2e4. \uc74c\ubc18\ub3c4 \ubaa8\ub4e0 \uc74c\ubc18 \ucc28\ud2b8\uc5d0\uc11c 1\uc704\ub97c \uae30\ub85d\ud558\uace0, \ubc1c\ub9e4 \uccab \uc8fc 41\ub9cc \uc7a5\uc744 \ud310\ub9e4\ud574 \uc5ed\ub300 \ub300\ud55c\ubbfc\uad6d \uc544\uc774\ub3cc \uadf8\ub8f9 \uc74c\ubc18 \ucd08\ub3d9 4\uc704\ub97c \uae30\ub85d\ud588\uc73c\uba70, \ud604\uc7ac\uae4c\uc9c0 64\ub9cc \uc7a5\uc744 \ud310\ub9e4\ud558\uc600\ub2e4. \ub610\ud55c, \uc74c\uc545 \ubc29\uc1a1\uc5d0\uc11c 1\uc704\ub97c \ud729\uc4f8\uc5b4 8\uad00\uc655\uc744 \ub2ec\uc131\ud588\ub2e4. \uac19\uc740 \ud574 12\uc6d4 9\uc77c, \uacf5\uc2dd \uc751\uc6d0\ubd09 \ub108\ube14\ubd09\uc744 \ube44\ub86f\ud55c \uacf5\uc2dd \uad7f\uc988\uac00 \uacf5\uac1c\ub418\uc5c8\uc73c\uba70, 12\uc6d4 15\uc77c\ubd80\ud130 17\uc77c, 23\uc77c\ubd80\ud130 24\uc77c\uae4c\uc9c0 \ud32c\ubbf8\ud305 \ucf58\uc11c\ud2b8 \u3008\uc6cc\ub108\uc6d0 \ud504\ub9ac\ubbf8\uc5b4 \ud32c\ucf58\u3009\uc744 \uac1c\ucd5c\ud558\uc600\ub2e4. 2017\ub144 \ud55c \ud574 \ub3d9\uc548 \uc6cc\ub108\uc6d0\uc740 \ubd80\uc0b0 \uc6d0\uc544\uc2dc\uc544\ud398\uc2a4\ud2f0\ubc8c \uc5b4\uc6cc\uc988\uc5d0\uc11c \ub77c\uc774\uc9d5 \uc2a4\ud0c0\uc0c1\uc744 \uc218\uc0c1\ud558\uace0, \uc8fc\uac04\uc544\uc774\ub3cc \uc5b4\uc6cc\uc988\uc5d0\uc11c \uc544\uc774\ub3cc \ucc54\ud504\uc0c1\uc744 \uc218\uc0c1\ud588\uc73c\uba70, \ub300\ud55c\ubbfc\uad6d \ud1b1\uc2a4\ud0c0\uc0c1 \uc2dc\uc0c1\uc2dd\uc5d0\uc11c \ud1b1\uac00\uc218\uc0c1\uc744 \uc218\uc0c1\ud588\ub2e4. \ub610\ud55c, \uc18c\ub9ac\ubc14\ub2e4 \ubca0\uc2a4\ud2b8 \ucf00\uc774\ubba4\uc9c1 \uc5b4\uc6cc\uc988\uc5d0\uc11c \uc2e0\ud55c\ub958 \uc2e0\uc778\uc0c1\uacfc \uc2e0\ud55c\ub958 \ub77c\uc774\uc9d5 \ud56b \uc2a4\ud0c0\uc0c1\uc744 \uc218\uc0c1\ud558\uace0, \uc544\uc2dc\uc544 \uc544\ud2f0\uc2a4\ud2b8 \uc5b4\uc6cc\uc988\uc5d0\uc11c\ub3c4 \uc2e0\uc778\uc0c1\uacfc \uc288\ud37c\ub8e8\ud0a4 \uc0bc\uc131\ud398\uc774\uc0c1\uc744 \uc218\uc0c1\ud574 \uac01\uac01 2\uad00\uc655\uc744 \ub2ec\uc131\ud588\ub2e4. \uc774\uc5b4 \uc5e0\ub137 \uc544\uc2dc\uc548 \ubba4\uc9c1 \uc5b4\uc6cc\ub4dc\uc5d0\uc11c \ub0a8\uc790 \uadf8\ub8f9\uc0c1, \ub0a8\uc790 \uc2e0\uc778\uc0c1, \ubca0\uc2a4\ud2b8 \uc624\ube0c \ub125\uc2a4\ud2b8\uc0c1\uc744 \uc218\uc0c1\ud558\uace0, \uba5c\ub860 \ubba4\uc9c1 \uc5b4\uc6cc\ub4dc\uc5d0\uc11c\ub3c4 Top 10, \uc2e0\uc778\uc0c1, \uce74\uce74\uc624 \ud56b \uc2a4\ud0c0\uc0c1\uc744 \uc218\uc0c1\ud574 \uac01\uac01 3\uad00\uc655\uc744 \ub2ec\uc131\ud588\ub2e4. \uc6cc\ub108\uc6d0\uc740 \ub370\ubdd4 \uc568\ubc94\uacfc \ub9ac\ud328\ud0a4\uc9c0 \uc568\ubc94 \ud310\ub9e4\ub7c9\uc774 100\ub9cc\uc7a5\uc744 \ub118\uc5b4 2000\ub144 \uc774\ud6c4 \ucd5c\ucd08\ub85c \ub300\ud55c\ubbfc\uad6d \uc544\uc774\ub3cc \uadf8\ub8f9 \ub370\ubdd4 \uc568\ubc94 \ubc00\ub9ac\uc5b8\uc140\ub7ec\uac00 \ub418\uc5c8\ub2e4."} {"question": "2009\ub144 \uae30\uc900, \uc77c\ubcf8\uc778\uc758 \uae30\ub300 \uc218\uba85\uc740 \uc57d \uba87 \uc138 \uc815\ub3c4\uc778\uac00?", "paragraph": "\uc77c\ubcf8\uc778\uc758 \ud3c9\uade0 \uc218\uba85\uc740 2009\ub144 \uae30\uc900 \ub0a8\uc131\uc774 79.29\uc138\uc774\uba70 \uc5ec\uc131\uc740 \uc138\uacc4\uc5d0\uc11c \uac00\uc7a5 \ub192\uc740 \uc218\uce58\uc778 86.05\uc138\ub97c \uae30\ub85d\ud558\uc600\ub2e4. \ud2b9\ud788 \uc5ec\uc131\uc758 \ud3c9\uade0 \uc218\uba85\uc740 1985\ub144 \uc774\ud6c4 \uacc4\uc18d \uc138\uacc4 1\uc704\ub97c \uc720\uc9c0\ud558\uace0 \uc788\ub2e4. \uc77c\ubcf8\uc778\uc758 \uc8fc\ub41c \uc0ac\uc778\uc740 1996\ub144 \uc870\uc0ac\uc5d0 \uc758\ud558\uba74 \uc554\uc774 \uac00\uc7a5 \ub9ce\uace0, \ub2e4\uc74c\uc73c\ub85c\ub294 \uc2ec\uc7a5\ubcd1, \ub1cc\uc878\uc911 \ub4f1\uc758 \uc131\uc778\ubcd1\uacfc \uc790\uc0b4, \uad50\ud1b5\uc0ac\uace0 \ub4f1\uc774 \uadf8 \ub4a4\ub97c \uc774\uc5c8\ub2e4. \ucd9c\uc0dd\ub960\uc774 \uacc4\uc18d \uc904\uc5b4\ub4e4\uace0 \uc788\uc9c0\ub9cc \uc81c2\ucc28 \uc138\uacc4 \ub300\uc804 \uc774\ud6c4 \uc758\ud559\uc774 \uae09\uc18d\ub3c4\ub85c \ubc1c\uc804\ud558\uc5ec \uc0ac\ub9dd\ub960 \ub610\ud55c \ud604\uc800\ud788 \uc904\uc5b4\ub4e4\uc5b4 \ub178\uc778 \uacc4\uce35\uc774 \uc99d\uac00\ud574 \uc774\ubbf8 \ucd08\uace0\ub839 \uc0ac\ud68c\uc5d0 \uc9c4\uc785\ud55c \uc0c1\ud0dc\uc774\ub2e4. \ubbf8\uad6d \uc9c8\ubcd1\ud1b5\uc81c\uc608\ubc29\uc13c\ud130\ub294 \uc77c\ubcf8\uc778\uc758 \uae30\ub300 \uc218\uba85\uc740 2009\ub144 \uae30\uc900 \uc2f1\uac00\ud3ec\ub974\uc640 \ub354\ubd88\uc5b4 \uc57d 82\uc138 \uc815\ub3c4\ub77c\uace0 \ubc1c\ud45c\ud558\uc600\ub2e4. \ud558\uc9c0\ub9cc \uc790\uc0b4\ub960\uc774 OECD \uad6d\uac00 \uc911 \ub300\ud55c\ubbfc\uad6d\uc5d0 \uc774\uc5b4 2\uc704\ub85c \ub192\uc544 \uc2ec\uac01\ud55c \uc0ac\ud68c \ubb38\uc81c\ub85c \ub300\ub450\ub418\uace0 \uc788\ub2e4.", "answer": "\uc57d 82\uc138", "sentence": "\ubbf8\uad6d \uc9c8\ubcd1\ud1b5\uc81c\uc608\ubc29\uc13c\ud130\ub294 \uc77c\ubcf8\uc778\uc758 \uae30\ub300 \uc218\uba85\uc740 2009\ub144 \uae30\uc900 \uc2f1\uac00\ud3ec\ub974\uc640 \ub354\ubd88\uc5b4 \uc57d 82\uc138 \uc815\ub3c4\ub77c\uace0 \ubc1c\ud45c\ud558\uc600\ub2e4.", "paragraph_sentence": "\uc77c\ubcf8\uc778\uc758 \ud3c9\uade0 \uc218\uba85\uc740 2009\ub144 \uae30\uc900 \ub0a8\uc131\uc774 79.29\uc138\uc774\uba70 \uc5ec\uc131\uc740 \uc138\uacc4\uc5d0\uc11c \uac00\uc7a5 \ub192\uc740 \uc218\uce58\uc778 86.05\uc138\ub97c \uae30\ub85d\ud558\uc600\ub2e4. \ud2b9\ud788 \uc5ec\uc131\uc758 \ud3c9\uade0 \uc218\uba85\uc740 1985\ub144 \uc774\ud6c4 \uacc4\uc18d \uc138\uacc4 1\uc704\ub97c \uc720\uc9c0\ud558\uace0 \uc788\ub2e4. \uc77c\ubcf8\uc778\uc758 \uc8fc\ub41c \uc0ac\uc778\uc740 1996\ub144 \uc870\uc0ac\uc5d0 \uc758\ud558\uba74 \uc554\uc774 \uac00\uc7a5 \ub9ce\uace0, \ub2e4\uc74c\uc73c\ub85c\ub294 \uc2ec\uc7a5\ubcd1, \ub1cc\uc878\uc911 \ub4f1\uc758 \uc131\uc778\ubcd1\uacfc \uc790\uc0b4, \uad50\ud1b5\uc0ac\uace0 \ub4f1\uc774 \uadf8 \ub4a4\ub97c \uc774\uc5c8\ub2e4. \ucd9c\uc0dd\ub960\uc774 \uacc4\uc18d \uc904\uc5b4\ub4e4\uace0 \uc788\uc9c0\ub9cc \uc81c2\ucc28 \uc138\uacc4 \ub300\uc804 \uc774\ud6c4 \uc758\ud559\uc774 \uae09\uc18d\ub3c4\ub85c \ubc1c\uc804\ud558\uc5ec \uc0ac\ub9dd\ub960 \ub610\ud55c \ud604\uc800\ud788 \uc904\uc5b4\ub4e4\uc5b4 \ub178\uc778 \uacc4\uce35\uc774 \uc99d\uac00\ud574 \uc774\ubbf8 \ucd08\uace0\ub839 \uc0ac\ud68c\uc5d0 \uc9c4\uc785\ud55c \uc0c1\ud0dc\uc774\ub2e4. \ubbf8\uad6d \uc9c8\ubcd1\ud1b5\uc81c\uc608\ubc29\uc13c\ud130\ub294 \uc77c\ubcf8\uc778\uc758 \uae30\ub300 \uc218\uba85\uc740 2009\ub144 \uae30\uc900 \uc2f1\uac00\ud3ec\ub974\uc640 \ub354\ubd88\uc5b4 \uc57d 82\uc138 \uc815\ub3c4\ub77c\uace0 \ubc1c\ud45c\ud558\uc600\ub2e4. \ud558\uc9c0\ub9cc \uc790\uc0b4\ub960\uc774 OECD \uad6d\uac00 \uc911 \ub300\ud55c\ubbfc\uad6d\uc5d0 \uc774\uc5b4 2\uc704\ub85c \ub192\uc544 \uc2ec\uac01\ud55c \uc0ac\ud68c \ubb38\uc81c\ub85c \ub300\ub450\ub418\uace0 \uc788\ub2e4.", "paragraph_answer": "\uc77c\ubcf8\uc778\uc758 \ud3c9\uade0 \uc218\uba85\uc740 2009\ub144 \uae30\uc900 \ub0a8\uc131\uc774 79.29\uc138\uc774\uba70 \uc5ec\uc131\uc740 \uc138\uacc4\uc5d0\uc11c \uac00\uc7a5 \ub192\uc740 \uc218\uce58\uc778 86.05\uc138\ub97c \uae30\ub85d\ud558\uc600\ub2e4. \ud2b9\ud788 \uc5ec\uc131\uc758 \ud3c9\uade0 \uc218\uba85\uc740 1985\ub144 \uc774\ud6c4 \uacc4\uc18d \uc138\uacc4 1\uc704\ub97c \uc720\uc9c0\ud558\uace0 \uc788\ub2e4. \uc77c\ubcf8\uc778\uc758 \uc8fc\ub41c \uc0ac\uc778\uc740 1996\ub144 \uc870\uc0ac\uc5d0 \uc758\ud558\uba74 \uc554\uc774 \uac00\uc7a5 \ub9ce\uace0, \ub2e4\uc74c\uc73c\ub85c\ub294 \uc2ec\uc7a5\ubcd1, \ub1cc\uc878\uc911 \ub4f1\uc758 \uc131\uc778\ubcd1\uacfc \uc790\uc0b4, \uad50\ud1b5\uc0ac\uace0 \ub4f1\uc774 \uadf8 \ub4a4\ub97c \uc774\uc5c8\ub2e4. \ucd9c\uc0dd\ub960\uc774 \uacc4\uc18d \uc904\uc5b4\ub4e4\uace0 \uc788\uc9c0\ub9cc \uc81c2\ucc28 \uc138\uacc4 \ub300\uc804 \uc774\ud6c4 \uc758\ud559\uc774 \uae09\uc18d\ub3c4\ub85c \ubc1c\uc804\ud558\uc5ec \uc0ac\ub9dd\ub960 \ub610\ud55c \ud604\uc800\ud788 \uc904\uc5b4\ub4e4\uc5b4 \ub178\uc778 \uacc4\uce35\uc774 \uc99d\uac00\ud574 \uc774\ubbf8 \ucd08\uace0\ub839 \uc0ac\ud68c\uc5d0 \uc9c4\uc785\ud55c \uc0c1\ud0dc\uc774\ub2e4. \ubbf8\uad6d \uc9c8\ubcd1\ud1b5\uc81c\uc608\ubc29\uc13c\ud130\ub294 \uc77c\ubcf8\uc778\uc758 \uae30\ub300 \uc218\uba85\uc740 2009\ub144 \uae30\uc900 \uc2f1\uac00\ud3ec\ub974\uc640 \ub354\ubd88\uc5b4 \uc57d 82\uc138 \uc815\ub3c4\ub77c\uace0 \ubc1c\ud45c\ud558\uc600\ub2e4. \ud558\uc9c0\ub9cc \uc790\uc0b4\ub960\uc774 OECD \uad6d\uac00 \uc911 \ub300\ud55c\ubbfc\uad6d\uc5d0 \uc774\uc5b4 2\uc704\ub85c \ub192\uc544 \uc2ec\uac01\ud55c \uc0ac\ud68c \ubb38\uc81c\ub85c \ub300\ub450\ub418\uace0 \uc788\ub2e4.", "sentence_answer": "\ubbf8\uad6d \uc9c8\ubcd1\ud1b5\uc81c\uc608\ubc29\uc13c\ud130\ub294 \uc77c\ubcf8\uc778\uc758 \uae30\ub300 \uc218\uba85\uc740 2009\ub144 \uae30\uc900 \uc2f1\uac00\ud3ec\ub974\uc640 \ub354\ubd88\uc5b4 \uc57d 82\uc138 \uc815\ub3c4\ub77c\uace0 \ubc1c\ud45c\ud558\uc600\ub2e4.", "paragraph_id": "6591477-27-2", "paragraph_question": "question: 2009\ub144 \uae30\uc900, \uc77c\ubcf8\uc778\uc758 \uae30\ub300 \uc218\uba85\uc740 \uc57d \uba87 \uc138 \uc815\ub3c4\uc778\uac00?, context: \uc77c\ubcf8\uc778\uc758 \ud3c9\uade0 \uc218\uba85\uc740 2009\ub144 \uae30\uc900 \ub0a8\uc131\uc774 79.29\uc138\uc774\uba70 \uc5ec\uc131\uc740 \uc138\uacc4\uc5d0\uc11c \uac00\uc7a5 \ub192\uc740 \uc218\uce58\uc778 86.05\uc138\ub97c \uae30\ub85d\ud558\uc600\ub2e4. \ud2b9\ud788 \uc5ec\uc131\uc758 \ud3c9\uade0 \uc218\uba85\uc740 1985\ub144 \uc774\ud6c4 \uacc4\uc18d \uc138\uacc4 1\uc704\ub97c \uc720\uc9c0\ud558\uace0 \uc788\ub2e4. \uc77c\ubcf8\uc778\uc758 \uc8fc\ub41c \uc0ac\uc778\uc740 1996\ub144 \uc870\uc0ac\uc5d0 \uc758\ud558\uba74 \uc554\uc774 \uac00\uc7a5 \ub9ce\uace0, \ub2e4\uc74c\uc73c\ub85c\ub294 \uc2ec\uc7a5\ubcd1, \ub1cc\uc878\uc911 \ub4f1\uc758 \uc131\uc778\ubcd1\uacfc \uc790\uc0b4, \uad50\ud1b5\uc0ac\uace0 \ub4f1\uc774 \uadf8 \ub4a4\ub97c \uc774\uc5c8\ub2e4. \ucd9c\uc0dd\ub960\uc774 \uacc4\uc18d \uc904\uc5b4\ub4e4\uace0 \uc788\uc9c0\ub9cc \uc81c2\ucc28 \uc138\uacc4 \ub300\uc804 \uc774\ud6c4 \uc758\ud559\uc774 \uae09\uc18d\ub3c4\ub85c \ubc1c\uc804\ud558\uc5ec \uc0ac\ub9dd\ub960 \ub610\ud55c \ud604\uc800\ud788 \uc904\uc5b4\ub4e4\uc5b4 \ub178\uc778 \uacc4\uce35\uc774 \uc99d\uac00\ud574 \uc774\ubbf8 \ucd08\uace0\ub839 \uc0ac\ud68c\uc5d0 \uc9c4\uc785\ud55c \uc0c1\ud0dc\uc774\ub2e4. \ubbf8\uad6d \uc9c8\ubcd1\ud1b5\uc81c\uc608\ubc29\uc13c\ud130\ub294 \uc77c\ubcf8\uc778\uc758 \uae30\ub300 \uc218\uba85\uc740 2009\ub144 \uae30\uc900 \uc2f1\uac00\ud3ec\ub974\uc640 \ub354\ubd88\uc5b4 \uc57d 82\uc138 \uc815\ub3c4\ub77c\uace0 \ubc1c\ud45c\ud558\uc600\ub2e4. \ud558\uc9c0\ub9cc \uc790\uc0b4\ub960\uc774 OECD \uad6d\uac00 \uc911 \ub300\ud55c\ubbfc\uad6d\uc5d0 \uc774\uc5b4 2\uc704\ub85c \ub192\uc544 \uc2ec\uac01\ud55c \uc0ac\ud68c \ubb38\uc81c\ub85c \ub300\ub450\ub418\uace0 \uc788\ub2e4."} {"question": "\uc5ec\uc57c\uac00 \ud55c\ub098\ub77c\ub2f9\uc758 \uc790\uc678 \ud22c\uc7c1 3\uac1c\uc6d4\ub9cc\uc5d0 \ud569\uc758\ud55c \uac83\uc740 \ubb34\uc5c7\uc778\uac00?", "paragraph": "2005\ub144 12\uc6d4 \uc5ec\ub2f9\uc778 \uc5f4\ub9b0\uc6b0\ub9ac\ub2f9\uc740 \uc0ac\ub9bd\ud559\uad50\ubc95 \uac1c\uc815\uc548\uc744 \uc9c1\uad8c\uc0c1\uc815\ud558\uc5ec \ucc98\ub9ac\ud558\uc600\ub2e4. \ub2f9\uc2dc \ub300\ud45c\uc778 \ubc15\uadfc\ud61c\ub294 \uace7\ubc14\ub85c \uc7a5\uc678 \ud22c\uc7c1\uc744 \uc120\uc5b8\ud558\uace0 \uac70\ub9ac\ub85c \ub098\uc130\ub2e4. \ub2f9\uc2dc \ud55c\ub098\ub77c\ub2f9 \ub0b4\ubd80\uc5d0\uc870\ucc28 \ud68c\uc758\uc801\uc778 \uc2dc\uc120\uc774 \ub9ce\uc558\ub2e4. \"\ud55c\ub098\ub77c\ub2f9\uc774 \ubb34\uc2a8 \uc7a5\uc678 \ud22c\uc7c1\uc774\ub0d0\"\ub77c\ub294 \ube44\uc544\ub0e5\ub3c4 \uc788\uc5c8\uace0, \uad6d\ud68c \ub4f1\uc6d0\uc5d0 \ub300\ud55c \uc5ec\ub860\uc758 \uc555\ubc15\ub3c4 \uc2ec\ud588\ub2e4. \ud558\uc9c0\ub9cc \ubc15\uadfc\ud61c \ub300\ud45c\ub294 \uc7a5\uc678 \ud22c\uc7c1\uc744 \uacc4\uc18d\ud588\uace0, \uacb0\uad6d \uc5ec\uc57c\ub294 \ud55c\ub098\ub77c\ub2f9\uc758 \uc7a5\uc678 \ud22c\uc7c1 3\uac1c\uc6d4 \ub9cc\uc5d0 \uc0ac\ud559\ubc95 \uc7ac\uac1c\uc815 \ub17c\uc758\uc5d0 \ud569\uc758\ud558\uac8c \ub41c\ub2e4. \ubc15\uadfc\ud61c \ub300\ud45c\ub294 \uc8fc\ubcc0\uc758 \ubc18\ub300\uc640 \uc6d0\uc678 \ud22c\uc7c1\uc5d0 \uc18c\uadf9\uc801\uc778 \ud55c\ub098\ub77c\ub2f9\uc758 \uccb4\uc9c8\uc774\ub77c\ub294 \ubb38\uc81c\uc5d0\ub3c4 \ubd88\uad6c\ud558\uace0, \uc0ac\ud559\ubc95 \uc7ac\uac1c\uc815\uc744 \uc774\ub04c\uc5b4\ub0b8 \uac83\uc73c\ub85c \ud3c9\uac00\ub418\uc5c8\ub2e4. 2004\ub144 17\ub300 \ucd1d\uc120\uc758 '\ud55c\ub098\ub77c\ub2f9 \ucc9c\ub9c9 \ub2f9\uc0ac' \uc5ed\uc2dc \"\uc1fc \uc544\ub2c8\ub0d0\"\ub77c\ub294 \ube44\ud310\uc5d0\ub3c4 \ubd88\uad6c\ud558\uace0 \ud55c\ub098\ub77c\ub2f9\uc758 \uc0c1\uc9d5\ubb3c\ub85c \uae30\uc5b5\ub410\ub2e4.", "answer": "\uc0ac\ud559\ubc95 \uc7ac\uac1c\uc815 \ub17c\uc758", "sentence": "\ud558\uc9c0\ub9cc \ubc15\uadfc\ud61c \ub300\ud45c\ub294 \uc7a5\uc678 \ud22c\uc7c1\uc744 \uacc4\uc18d\ud588\uace0, \uacb0\uad6d \uc5ec\uc57c\ub294 \ud55c\ub098\ub77c\ub2f9\uc758 \uc7a5\uc678 \ud22c\uc7c1 3\uac1c\uc6d4 \ub9cc\uc5d0 \uc0ac\ud559\ubc95 \uc7ac\uac1c\uc815 \ub17c\uc758 \uc5d0 \ud569\uc758\ud558\uac8c \ub41c\ub2e4.", "paragraph_sentence": "2005\ub144 12\uc6d4 \uc5ec\ub2f9\uc778 \uc5f4\ub9b0\uc6b0\ub9ac\ub2f9\uc740 \uc0ac\ub9bd\ud559\uad50\ubc95 \uac1c\uc815\uc548\uc744 \uc9c1\uad8c\uc0c1\uc815\ud558\uc5ec \ucc98\ub9ac\ud558\uc600\ub2e4. \ub2f9\uc2dc \ub300\ud45c\uc778 \ubc15\uadfc\ud61c\ub294 \uace7\ubc14\ub85c \uc7a5\uc678 \ud22c\uc7c1\uc744 \uc120\uc5b8\ud558\uace0 \uac70\ub9ac\ub85c \ub098\uc130\ub2e4. \ub2f9\uc2dc \ud55c\ub098\ub77c\ub2f9 \ub0b4\ubd80\uc5d0\uc870\ucc28 \ud68c\uc758\uc801\uc778 \uc2dc\uc120\uc774 \ub9ce\uc558\ub2e4. \"\ud55c\ub098\ub77c\ub2f9\uc774 \ubb34\uc2a8 \uc7a5\uc678 \ud22c\uc7c1\uc774\ub0d0\"\ub77c\ub294 \ube44\uc544\ub0e5\ub3c4 \uc788\uc5c8\uace0, \uad6d\ud68c \ub4f1\uc6d0\uc5d0 \ub300\ud55c \uc5ec\ub860\uc758 \uc555\ubc15\ub3c4 \uc2ec\ud588\ub2e4. \ud558\uc9c0\ub9cc \ubc15\uadfc\ud61c \ub300\ud45c\ub294 \uc7a5\uc678 \ud22c\uc7c1\uc744 \uacc4\uc18d\ud588\uace0, \uacb0\uad6d \uc5ec\uc57c\ub294 \ud55c\ub098\ub77c\ub2f9\uc758 \uc7a5\uc678 \ud22c\uc7c1 3\uac1c\uc6d4 \ub9cc\uc5d0 \uc0ac\ud559\ubc95 \uc7ac\uac1c\uc815 \ub17c\uc758 \uc5d0 \ud569\uc758\ud558\uac8c \ub41c\ub2e4. \ubc15\uadfc\ud61c \ub300\ud45c\ub294 \uc8fc\ubcc0\uc758 \ubc18\ub300\uc640 \uc6d0\uc678 \ud22c\uc7c1\uc5d0 \uc18c\uadf9\uc801\uc778 \ud55c\ub098\ub77c\ub2f9\uc758 \uccb4\uc9c8\uc774\ub77c\ub294 \ubb38\uc81c\uc5d0\ub3c4 \ubd88\uad6c\ud558\uace0, \uc0ac\ud559\ubc95 \uc7ac\uac1c\uc815\uc744 \uc774\ub04c\uc5b4\ub0b8 \uac83\uc73c\ub85c \ud3c9\uac00\ub418\uc5c8\ub2e4. 2004\ub144 17\ub300 \ucd1d\uc120\uc758 '\ud55c\ub098\ub77c\ub2f9 \ucc9c\ub9c9 \ub2f9\uc0ac' \uc5ed\uc2dc \"\uc1fc \uc544\ub2c8\ub0d0\"\ub77c\ub294 \ube44\ud310\uc5d0\ub3c4 \ubd88\uad6c\ud558\uace0 \ud55c\ub098\ub77c\ub2f9\uc758 \uc0c1\uc9d5\ubb3c\ub85c \uae30\uc5b5\ub410\ub2e4.", "paragraph_answer": "2005\ub144 12\uc6d4 \uc5ec\ub2f9\uc778 \uc5f4\ub9b0\uc6b0\ub9ac\ub2f9\uc740 \uc0ac\ub9bd\ud559\uad50\ubc95 \uac1c\uc815\uc548\uc744 \uc9c1\uad8c\uc0c1\uc815\ud558\uc5ec \ucc98\ub9ac\ud558\uc600\ub2e4. \ub2f9\uc2dc \ub300\ud45c\uc778 \ubc15\uadfc\ud61c\ub294 \uace7\ubc14\ub85c \uc7a5\uc678 \ud22c\uc7c1\uc744 \uc120\uc5b8\ud558\uace0 \uac70\ub9ac\ub85c \ub098\uc130\ub2e4. \ub2f9\uc2dc \ud55c\ub098\ub77c\ub2f9 \ub0b4\ubd80\uc5d0\uc870\ucc28 \ud68c\uc758\uc801\uc778 \uc2dc\uc120\uc774 \ub9ce\uc558\ub2e4. \"\ud55c\ub098\ub77c\ub2f9\uc774 \ubb34\uc2a8 \uc7a5\uc678 \ud22c\uc7c1\uc774\ub0d0\"\ub77c\ub294 \ube44\uc544\ub0e5\ub3c4 \uc788\uc5c8\uace0, \uad6d\ud68c \ub4f1\uc6d0\uc5d0 \ub300\ud55c \uc5ec\ub860\uc758 \uc555\ubc15\ub3c4 \uc2ec\ud588\ub2e4. \ud558\uc9c0\ub9cc \ubc15\uadfc\ud61c \ub300\ud45c\ub294 \uc7a5\uc678 \ud22c\uc7c1\uc744 \uacc4\uc18d\ud588\uace0, \uacb0\uad6d \uc5ec\uc57c\ub294 \ud55c\ub098\ub77c\ub2f9\uc758 \uc7a5\uc678 \ud22c\uc7c1 3\uac1c\uc6d4 \ub9cc\uc5d0 \uc0ac\ud559\ubc95 \uc7ac\uac1c\uc815 \ub17c\uc758 \uc5d0 \ud569\uc758\ud558\uac8c \ub41c\ub2e4. \ubc15\uadfc\ud61c \ub300\ud45c\ub294 \uc8fc\ubcc0\uc758 \ubc18\ub300\uc640 \uc6d0\uc678 \ud22c\uc7c1\uc5d0 \uc18c\uadf9\uc801\uc778 \ud55c\ub098\ub77c\ub2f9\uc758 \uccb4\uc9c8\uc774\ub77c\ub294 \ubb38\uc81c\uc5d0\ub3c4 \ubd88\uad6c\ud558\uace0, \uc0ac\ud559\ubc95 \uc7ac\uac1c\uc815\uc744 \uc774\ub04c\uc5b4\ub0b8 \uac83\uc73c\ub85c \ud3c9\uac00\ub418\uc5c8\ub2e4. 2004\ub144 17\ub300 \ucd1d\uc120\uc758 '\ud55c\ub098\ub77c\ub2f9 \ucc9c\ub9c9 \ub2f9\uc0ac' \uc5ed\uc2dc \"\uc1fc \uc544\ub2c8\ub0d0\"\ub77c\ub294 \ube44\ud310\uc5d0\ub3c4 \ubd88\uad6c\ud558\uace0 \ud55c\ub098\ub77c\ub2f9\uc758 \uc0c1\uc9d5\ubb3c\ub85c \uae30\uc5b5\ub410\ub2e4.", "sentence_answer": "\ud558\uc9c0\ub9cc \ubc15\uadfc\ud61c \ub300\ud45c\ub294 \uc7a5\uc678 \ud22c\uc7c1\uc744 \uacc4\uc18d\ud588\uace0, \uacb0\uad6d \uc5ec\uc57c\ub294 \ud55c\ub098\ub77c\ub2f9\uc758 \uc7a5\uc678 \ud22c\uc7c1 3\uac1c\uc6d4 \ub9cc\uc5d0 \uc0ac\ud559\ubc95 \uc7ac\uac1c\uc815 \ub17c\uc758 \uc5d0 \ud569\uc758\ud558\uac8c \ub41c\ub2e4.", "paragraph_id": "6581158-2-1", "paragraph_question": "question: \uc5ec\uc57c\uac00 \ud55c\ub098\ub77c\ub2f9\uc758 \uc790\uc678 \ud22c\uc7c1 3\uac1c\uc6d4\ub9cc\uc5d0 \ud569\uc758\ud55c \uac83\uc740 \ubb34\uc5c7\uc778\uac00?, context: 2005\ub144 12\uc6d4 \uc5ec\ub2f9\uc778 \uc5f4\ub9b0\uc6b0\ub9ac\ub2f9\uc740 \uc0ac\ub9bd\ud559\uad50\ubc95 \uac1c\uc815\uc548\uc744 \uc9c1\uad8c\uc0c1\uc815\ud558\uc5ec \ucc98\ub9ac\ud558\uc600\ub2e4. \ub2f9\uc2dc \ub300\ud45c\uc778 \ubc15\uadfc\ud61c\ub294 \uace7\ubc14\ub85c \uc7a5\uc678 \ud22c\uc7c1\uc744 \uc120\uc5b8\ud558\uace0 \uac70\ub9ac\ub85c \ub098\uc130\ub2e4. \ub2f9\uc2dc \ud55c\ub098\ub77c\ub2f9 \ub0b4\ubd80\uc5d0\uc870\ucc28 \ud68c\uc758\uc801\uc778 \uc2dc\uc120\uc774 \ub9ce\uc558\ub2e4. \"\ud55c\ub098\ub77c\ub2f9\uc774 \ubb34\uc2a8 \uc7a5\uc678 \ud22c\uc7c1\uc774\ub0d0\"\ub77c\ub294 \ube44\uc544\ub0e5\ub3c4 \uc788\uc5c8\uace0, \uad6d\ud68c \ub4f1\uc6d0\uc5d0 \ub300\ud55c \uc5ec\ub860\uc758 \uc555\ubc15\ub3c4 \uc2ec\ud588\ub2e4. \ud558\uc9c0\ub9cc \ubc15\uadfc\ud61c \ub300\ud45c\ub294 \uc7a5\uc678 \ud22c\uc7c1\uc744 \uacc4\uc18d\ud588\uace0, \uacb0\uad6d \uc5ec\uc57c\ub294 \ud55c\ub098\ub77c\ub2f9\uc758 \uc7a5\uc678 \ud22c\uc7c1 3\uac1c\uc6d4 \ub9cc\uc5d0 \uc0ac\ud559\ubc95 \uc7ac\uac1c\uc815 \ub17c\uc758\uc5d0 \ud569\uc758\ud558\uac8c \ub41c\ub2e4. \ubc15\uadfc\ud61c \ub300\ud45c\ub294 \uc8fc\ubcc0\uc758 \ubc18\ub300\uc640 \uc6d0\uc678 \ud22c\uc7c1\uc5d0 \uc18c\uadf9\uc801\uc778 \ud55c\ub098\ub77c\ub2f9\uc758 \uccb4\uc9c8\uc774\ub77c\ub294 \ubb38\uc81c\uc5d0\ub3c4 \ubd88\uad6c\ud558\uace0, \uc0ac\ud559\ubc95 \uc7ac\uac1c\uc815\uc744 \uc774\ub04c\uc5b4\ub0b8 \uac83\uc73c\ub85c \ud3c9\uac00\ub418\uc5c8\ub2e4. 2004\ub144 17\ub300 \ucd1d\uc120\uc758 '\ud55c\ub098\ub77c\ub2f9 \ucc9c\ub9c9 \ub2f9\uc0ac' \uc5ed\uc2dc \"\uc1fc \uc544\ub2c8\ub0d0\"\ub77c\ub294 \ube44\ud310\uc5d0\ub3c4 \ubd88\uad6c\ud558\uace0 \ud55c\ub098\ub77c\ub2f9\uc758 \uc0c1\uc9d5\ubb3c\ub85c \uae30\uc5b5\ub410\ub2e4."} {"question": "1989\ub144 \uc989\uc704\ud55c \uc77c\ubcf8\uc758 \ucc9c\uc655\uc740 \ub204\uad6c\uc778\uac00?", "paragraph": "\uc77c\ubcf8\uc758 \uc815\uce58 \uccb4\uc81c\ub294 \uad70\uc8fc\uc778 \ucc9c\ud669\uacfc \ud5cc\ubc95\uc774 \uc591\ub9bd\ud558\ub294 \uc785\ud5cc\uad70\uc8fc\uc81c\ub97c \ucc44\ud0dd\ud558\uace0 \uc788\ub2e4. \ud604\ud589 \uc77c\ubcf8\uad6d \ud5cc\ubc95\uc5d0\uc11c \ucc9c\ud669\uc740 \"\uc77c\ubcf8\uad6d\uc758 \uc0c1\uc9d5\uc774\uba70 \uadf8 \uc9c0\uc704\ub294 \uc77c\ubcf8 \uad6d\ubbfc\ub4e4\uc758 \ucd1d\uc758(\u7e3d\u610f)\uc5d0 \ubc14\ud0d5\uc744 \ub454\ub2e4\"\uace0 \uaddc\uc815\ub418\uc5b4 \uc788\uace0\ud5cc\ubc95\uc774 \uc815\ud558\ub294 \uad6d\uc0ac(\u570b\u4e8b)\uc5d0 \uad00\ud55c \ud589\uc704\ub9cc\uc744 \ub0b4\uac01\uc758 \uc870\uc5b8\uacfc \uc2b9\uc778, \ucc45\uc784\uc5d0 \ub530\ub77c \ud560 \uc218 \uc788\ub3c4\ub85d \ub418\uc5b4 \uc788\ub2e4. \uadf8\uc5d0 \ub530\ub77c \ub0b4\uac01\ucd1d\ub9ac\ub300\uc2e0\uacfc \ub0b4\uac01\uc758 \uc784\uc6d0\ub4e4\uc744 \uc784\uba85\ud558\uac70\ub098 \uadf8 \uc2e0\uc784\uc7a5\uc5d0 \uc778\uc99d\ud558\ub294 \uac83, \ucd5c\uace0\uc7ac\ud310\uc18c \uc7a5\uad00\uc744 \uc784\uba85\ud558\ub294 \uac83, \uad6d\ud68c\ub97c \uc18c\uc9d1\ud558\uace0 \ud5cc\ubc95 \uac1c\uc815\uacfc \ubc95\ub960 \ubc0f \uc815\ub839, \uc870\uc57d\uc744 \uacf5\ud3ec\ud558\ub294 \uac83 \ub4f1\uc73c\ub85c \ucc9c\ud669\uc774 \uad6d\uc0ac\uc5d0 \uad00\uc5ec\ud558\ub294 \uac83\uc740 \ud06c\uac8c \ud55c\uc815\ub418\uc5b4 \uc788\ub2e4. \ud558\uc9c0\ub9cc \ucc9c\ud669\uc740 \uc678\uad50\uc758\ub840 \uc0c1\uc5d0\uc11c \uc77c\ubcf8\uc758 \uad6d\uac00\uc6d0\uc218\ub85c \ub300\uc6b0\ubc1b\uace0 \uc788\ub2e4. \ud604\uc7ac \uc77c\ubcf8\uc758 \ucc9c\ud669\uc740 \uc544\ud0a4\ud788\ud1a0\uc774\uba70 1989\ub144 \uc989\uc704\ud55c \uc774\ud6c4 \ud604\uc7ac\uc5d0 \uc774\ub974\uace0 \uc788\ub2e4. \ucc9c\ud669 \uc2a4\uc2a4\ub85c \ubc31\uc81c\uc758 \uc655\uc190\uc774\ub77c\uace0 \ud55c \uac83\uc740 \ud070 \uc774\uc288\uac00 \ub41c \uc801\uc774 \uc788\ub2e4.", "answer": "\uc544\ud0a4\ud788\ud1a0", "sentence": "\ud604\uc7ac \uc77c\ubcf8\uc758 \ucc9c\ud669\uc740 \uc544\ud0a4\ud788\ud1a0 \uc774\uba70 1989\ub144 \uc989\uc704\ud55c \uc774\ud6c4 \ud604\uc7ac\uc5d0 \uc774\ub974\uace0 \uc788\ub2e4.", "paragraph_sentence": "\uc77c\ubcf8\uc758 \uc815\uce58 \uccb4\uc81c\ub294 \uad70\uc8fc\uc778 \ucc9c\ud669\uacfc \ud5cc\ubc95\uc774 \uc591\ub9bd\ud558\ub294 \uc785\ud5cc\uad70\uc8fc\uc81c\ub97c \ucc44\ud0dd\ud558\uace0 \uc788\ub2e4. \ud604\ud589 \uc77c\ubcf8\uad6d \ud5cc\ubc95\uc5d0\uc11c \ucc9c\ud669\uc740 \"\uc77c\ubcf8\uad6d\uc758 \uc0c1\uc9d5\uc774\uba70 \uadf8 \uc9c0\uc704\ub294 \uc77c\ubcf8 \uad6d\ubbfc\ub4e4\uc758 \ucd1d\uc758(\u7e3d\u610f)\uc5d0 \ubc14\ud0d5\uc744 \ub454\ub2e4\"\uace0 \uaddc\uc815\ub418\uc5b4 \uc788\uace0\ud5cc\ubc95\uc774 \uc815\ud558\ub294 \uad6d\uc0ac(\u570b\u4e8b)\uc5d0 \uad00\ud55c \ud589\uc704\ub9cc\uc744 \ub0b4\uac01\uc758 \uc870\uc5b8\uacfc \uc2b9\uc778, \ucc45\uc784\uc5d0 \ub530\ub77c \ud560 \uc218 \uc788\ub3c4\ub85d \ub418\uc5b4 \uc788\ub2e4. \uadf8\uc5d0 \ub530\ub77c \ub0b4\uac01\ucd1d\ub9ac\ub300\uc2e0\uacfc \ub0b4\uac01\uc758 \uc784\uc6d0\ub4e4\uc744 \uc784\uba85\ud558\uac70\ub098 \uadf8 \uc2e0\uc784\uc7a5\uc5d0 \uc778\uc99d\ud558\ub294 \uac83, \ucd5c\uace0\uc7ac\ud310\uc18c \uc7a5\uad00\uc744 \uc784\uba85\ud558\ub294 \uac83, \uad6d\ud68c\ub97c \uc18c\uc9d1\ud558\uace0 \ud5cc\ubc95 \uac1c\uc815\uacfc \ubc95\ub960 \ubc0f \uc815\ub839, \uc870\uc57d\uc744 \uacf5\ud3ec\ud558\ub294 \uac83 \ub4f1\uc73c\ub85c \ucc9c\ud669\uc774 \uad6d\uc0ac\uc5d0 \uad00\uc5ec\ud558\ub294 \uac83\uc740 \ud06c\uac8c \ud55c\uc815\ub418\uc5b4 \uc788\ub2e4. \ud558\uc9c0\ub9cc \ucc9c\ud669\uc740 \uc678\uad50\uc758\ub840 \uc0c1\uc5d0\uc11c \uc77c\ubcf8\uc758 \uad6d\uac00\uc6d0\uc218\ub85c \ub300\uc6b0\ubc1b\uace0 \uc788\ub2e4. \ud604\uc7ac \uc77c\ubcf8\uc758 \ucc9c\ud669\uc740 \uc544\ud0a4\ud788\ud1a0 \uc774\uba70 1989\ub144 \uc989\uc704\ud55c \uc774\ud6c4 \ud604\uc7ac\uc5d0 \uc774\ub974\uace0 \uc788\ub2e4. \ucc9c\ud669 \uc2a4\uc2a4\ub85c \ubc31\uc81c\uc758 \uc655\uc190\uc774\ub77c\uace0 \ud55c \uac83\uc740 \ud070 \uc774\uc288\uac00 \ub41c \uc801\uc774 \uc788\ub2e4.", "paragraph_answer": "\uc77c\ubcf8\uc758 \uc815\uce58 \uccb4\uc81c\ub294 \uad70\uc8fc\uc778 \ucc9c\ud669\uacfc \ud5cc\ubc95\uc774 \uc591\ub9bd\ud558\ub294 \uc785\ud5cc\uad70\uc8fc\uc81c\ub97c \ucc44\ud0dd\ud558\uace0 \uc788\ub2e4. \ud604\ud589 \uc77c\ubcf8\uad6d \ud5cc\ubc95\uc5d0\uc11c \ucc9c\ud669\uc740 \"\uc77c\ubcf8\uad6d\uc758 \uc0c1\uc9d5\uc774\uba70 \uadf8 \uc9c0\uc704\ub294 \uc77c\ubcf8 \uad6d\ubbfc\ub4e4\uc758 \ucd1d\uc758(\u7e3d\u610f)\uc5d0 \ubc14\ud0d5\uc744 \ub454\ub2e4\"\uace0 \uaddc\uc815\ub418\uc5b4 \uc788\uace0\ud5cc\ubc95\uc774 \uc815\ud558\ub294 \uad6d\uc0ac(\u570b\u4e8b)\uc5d0 \uad00\ud55c \ud589\uc704\ub9cc\uc744 \ub0b4\uac01\uc758 \uc870\uc5b8\uacfc \uc2b9\uc778, \ucc45\uc784\uc5d0 \ub530\ub77c \ud560 \uc218 \uc788\ub3c4\ub85d \ub418\uc5b4 \uc788\ub2e4. \uadf8\uc5d0 \ub530\ub77c \ub0b4\uac01\ucd1d\ub9ac\ub300\uc2e0\uacfc \ub0b4\uac01\uc758 \uc784\uc6d0\ub4e4\uc744 \uc784\uba85\ud558\uac70\ub098 \uadf8 \uc2e0\uc784\uc7a5\uc5d0 \uc778\uc99d\ud558\ub294 \uac83, \ucd5c\uace0\uc7ac\ud310\uc18c \uc7a5\uad00\uc744 \uc784\uba85\ud558\ub294 \uac83, \uad6d\ud68c\ub97c \uc18c\uc9d1\ud558\uace0 \ud5cc\ubc95 \uac1c\uc815\uacfc \ubc95\ub960 \ubc0f \uc815\ub839, \uc870\uc57d\uc744 \uacf5\ud3ec\ud558\ub294 \uac83 \ub4f1\uc73c\ub85c \ucc9c\ud669\uc774 \uad6d\uc0ac\uc5d0 \uad00\uc5ec\ud558\ub294 \uac83\uc740 \ud06c\uac8c \ud55c\uc815\ub418\uc5b4 \uc788\ub2e4. \ud558\uc9c0\ub9cc \ucc9c\ud669\uc740 \uc678\uad50\uc758\ub840 \uc0c1\uc5d0\uc11c \uc77c\ubcf8\uc758 \uad6d\uac00\uc6d0\uc218\ub85c \ub300\uc6b0\ubc1b\uace0 \uc788\ub2e4. \ud604\uc7ac \uc77c\ubcf8\uc758 \ucc9c\ud669\uc740 \uc544\ud0a4\ud788\ud1a0 \uc774\uba70 1989\ub144 \uc989\uc704\ud55c \uc774\ud6c4 \ud604\uc7ac\uc5d0 \uc774\ub974\uace0 \uc788\ub2e4. \ucc9c\ud669 \uc2a4\uc2a4\ub85c \ubc31\uc81c\uc758 \uc655\uc190\uc774\ub77c\uace0 \ud55c \uac83\uc740 \ud070 \uc774\uc288\uac00 \ub41c \uc801\uc774 \uc788\ub2e4.", "sentence_answer": "\ud604\uc7ac \uc77c\ubcf8\uc758 \ucc9c\ud669\uc740 \uc544\ud0a4\ud788\ud1a0 \uc774\uba70 1989\ub144 \uc989\uc704\ud55c \uc774\ud6c4 \ud604\uc7ac\uc5d0 \uc774\ub974\uace0 \uc788\ub2e4.", "paragraph_id": "6173085-6-1", "paragraph_question": "question: 1989\ub144 \uc989\uc704\ud55c \uc77c\ubcf8\uc758 \ucc9c\uc655\uc740 \ub204\uad6c\uc778\uac00?, context: \uc77c\ubcf8\uc758 \uc815\uce58 \uccb4\uc81c\ub294 \uad70\uc8fc\uc778 \ucc9c\ud669\uacfc \ud5cc\ubc95\uc774 \uc591\ub9bd\ud558\ub294 \uc785\ud5cc\uad70\uc8fc\uc81c\ub97c \ucc44\ud0dd\ud558\uace0 \uc788\ub2e4. \ud604\ud589 \uc77c\ubcf8\uad6d \ud5cc\ubc95\uc5d0\uc11c \ucc9c\ud669\uc740 \"\uc77c\ubcf8\uad6d\uc758 \uc0c1\uc9d5\uc774\uba70 \uadf8 \uc9c0\uc704\ub294 \uc77c\ubcf8 \uad6d\ubbfc\ub4e4\uc758 \ucd1d\uc758(\u7e3d\u610f)\uc5d0 \ubc14\ud0d5\uc744 \ub454\ub2e4\"\uace0 \uaddc\uc815\ub418\uc5b4 \uc788\uace0\ud5cc\ubc95\uc774 \uc815\ud558\ub294 \uad6d\uc0ac(\u570b\u4e8b)\uc5d0 \uad00\ud55c \ud589\uc704\ub9cc\uc744 \ub0b4\uac01\uc758 \uc870\uc5b8\uacfc \uc2b9\uc778, \ucc45\uc784\uc5d0 \ub530\ub77c \ud560 \uc218 \uc788\ub3c4\ub85d \ub418\uc5b4 \uc788\ub2e4. \uadf8\uc5d0 \ub530\ub77c \ub0b4\uac01\ucd1d\ub9ac\ub300\uc2e0\uacfc \ub0b4\uac01\uc758 \uc784\uc6d0\ub4e4\uc744 \uc784\uba85\ud558\uac70\ub098 \uadf8 \uc2e0\uc784\uc7a5\uc5d0 \uc778\uc99d\ud558\ub294 \uac83, \ucd5c\uace0\uc7ac\ud310\uc18c \uc7a5\uad00\uc744 \uc784\uba85\ud558\ub294 \uac83, \uad6d\ud68c\ub97c \uc18c\uc9d1\ud558\uace0 \ud5cc\ubc95 \uac1c\uc815\uacfc \ubc95\ub960 \ubc0f \uc815\ub839, \uc870\uc57d\uc744 \uacf5\ud3ec\ud558\ub294 \uac83 \ub4f1\uc73c\ub85c \ucc9c\ud669\uc774 \uad6d\uc0ac\uc5d0 \uad00\uc5ec\ud558\ub294 \uac83\uc740 \ud06c\uac8c \ud55c\uc815\ub418\uc5b4 \uc788\ub2e4. \ud558\uc9c0\ub9cc \ucc9c\ud669\uc740 \uc678\uad50\uc758\ub840 \uc0c1\uc5d0\uc11c \uc77c\ubcf8\uc758 \uad6d\uac00\uc6d0\uc218\ub85c \ub300\uc6b0\ubc1b\uace0 \uc788\ub2e4. \ud604\uc7ac \uc77c\ubcf8\uc758 \ucc9c\ud669\uc740 \uc544\ud0a4\ud788\ud1a0\uc774\uba70 1989\ub144 \uc989\uc704\ud55c \uc774\ud6c4 \ud604\uc7ac\uc5d0 \uc774\ub974\uace0 \uc788\ub2e4. \ucc9c\ud669 \uc2a4\uc2a4\ub85c \ubc31\uc81c\uc758 \uc655\uc190\uc774\ub77c\uace0 \ud55c \uac83\uc740 \ud070 \uc774\uc288\uac00 \ub41c \uc801\uc774 \uc788\ub2e4."} {"question": "\ucf54\ub77c\uc758 \uc790\uc2dd \uc911 \ub9b0\uc758 \uc9c1\uc5c5\uc740 \ubb34\uc5c7\uc778\uac00?", "paragraph": "\uc804\uc791 \uc544\uc559\uc758 \uc804\uc124\uc758 \uce90\ub9ad\ud130 \uc911 \ud55c\uba85\uc73c\ub85c \ucf54\ub77c\uc758 \uc804\uc124\uc5d0\uc11c\ub294 \uacf5\ud654\uad6d \ub3c4\uc2dc \uccab\ubc88\uc9f8 \uacbd\ucc30\uccad\uc7a5\uc774\uc5c8\ub2e4. \uc790\uc2dd\uc73c\ub85c\ub294 \ub9b0\uacfc \uc218\ub97c \ub370\ub9ac\uace0 \uc788\ub2e4. \uc5b4\ub9b4\uc801 \uc5c4\uaca9\ud55c \ubd80\ubaa8\ub2d8 \ubc11\uc5d0\uc11c \uc790\ub780\uac83\uc5d0 \ub300\ud55c \ud2b8\ub77c\uc6b0\ub9c8 \ub54c\ubb38\uc5d0 \uc790\uc2dd\ub4e4\uc5d0\uac8c\ub294 \ucd5c\ub300\ud55c\uc758 \uc790\uc720\ub97c \uc8fc\uc5c8\ub2e4. \uadf8\ub85c\uc778\ud574 \uacb0\uad6d \ub9b0\uacfc \uc218\uac00 \uc5b4\uba38\ub2c8\uc758 \uad00\uc2ec\uc744 \ubc1b\uae30 \uc704\ud574 \uacbd\uc7c1\ud588\uace0 \ub9b0\uc740 \uacbd\ucc30\uc774 \ub418\uc5c8\uc73c\uba70 \uc218\ub294 \uce5c\uad6c\ub4e4\uacfc \uc0ac\uace0\ub9cc \uce58\uace0 \ub2e4\ub154\ub2e4. \uacb0\uad6d \uc218\uac00 \ud06c\uac8c \uc0ac\uace0\ub97c \uce58\uc790 \uadf8 \uae30\ub85d\uc744 \uc740\ud3d0\ud558\uace0 \uacf5\ud654\uad6d \ub3c4\uc2dc\ub97c \ub5a0\ub098\uac8c \ud588\uace0 \uadf8\ub85c \uc778\ud55c \uc8c4\ucc45\uac10\uc73c\ub85c \uc5bc\ub9c8 \uc9c0\ub098\uc9c0 \uc54a\uc544 \uacbd\ucc30\uccad\uc7a5 \uc790\ub9ac\uc5d0\uc11c \uc740\ud1f4\ud55c\ub2e4. \uc740\ud1f4 \ud6c4\uc5d0\ub294 \uc218\uac00 \uc138\uc6b4 \uc790\uc624\ud478\ub97c \uba87\ubc88 \ubc29\ubb38\ud588\ub2e4\uac00 \ud589\ubc29\uc774 \ubb18\uc5f0\ud574\uc9c4\ub2e4. \uc2dc\uc98c 4\uc5d0\uc11c \ud759\uc758 \uc655\uad6d\uc758 \uc232\uc5d0 \uc0b4\uace0\uc788\ub2e4\ub294 \uac83\uc774 \ubc1d\ud600\uc84c\uace0 \ucf54\ub77c\uac00 \ud2b8\ub77c\uc6b0\ub9c8\ub97c \uadf9\ubcf5\ud558\ub294\ub370 \ub3c4\uc6c0\uc744 \uc900\ub2e4. \uc5b4\ub838\uc744\ub54c\ub098 \ub299\uc5c8\uc744\ub54c\ub098 \uad34\ud30d\ud558\uace0 \uc544\ubc14\ud0c0 \uad34\ub86d\ud788\uae30 \uc88b\uc544\ud558\ub294\uac83\uc740 \uc5ec\uc804\ud558\ub2e4.", "answer": "\uacbd\ucc30", "sentence": "\uc804\uc791 \uc544\uc559\uc758 \uc804\uc124\uc758 \uce90\ub9ad\ud130 \uc911 \ud55c\uba85\uc73c\ub85c \ucf54\ub77c\uc758 \uc804\uc124\uc5d0\uc11c\ub294 \uacf5\ud654\uad6d \ub3c4\uc2dc \uccab\ubc88\uc9f8 \uacbd\ucc30 \uccad\uc7a5\uc774\uc5c8\ub2e4.", "paragraph_sentence": " \uc804\uc791 \uc544\uc559\uc758 \uc804\uc124\uc758 \uce90\ub9ad\ud130 \uc911 \ud55c\uba85\uc73c\ub85c \ucf54\ub77c\uc758 \uc804\uc124\uc5d0\uc11c\ub294 \uacf5\ud654\uad6d \ub3c4\uc2dc \uccab\ubc88\uc9f8 \uacbd\ucc30 \uccad\uc7a5\uc774\uc5c8\ub2e4. \uc790\uc2dd\uc73c\ub85c\ub294 \ub9b0\uacfc \uc218\ub97c \ub370\ub9ac\uace0 \uc788\ub2e4. \uc5b4\ub9b4\uc801 \uc5c4\uaca9\ud55c \ubd80\ubaa8\ub2d8 \ubc11\uc5d0\uc11c \uc790\ub780\uac83\uc5d0 \ub300\ud55c \ud2b8\ub77c\uc6b0\ub9c8 \ub54c\ubb38\uc5d0 \uc790\uc2dd\ub4e4\uc5d0\uac8c\ub294 \ucd5c\ub300\ud55c\uc758 \uc790\uc720\ub97c \uc8fc\uc5c8\ub2e4. \uadf8\ub85c\uc778\ud574 \uacb0\uad6d \ub9b0\uacfc \uc218\uac00 \uc5b4\uba38\ub2c8\uc758 \uad00\uc2ec\uc744 \ubc1b\uae30 \uc704\ud574 \uacbd\uc7c1\ud588\uace0 \ub9b0\uc740 \uacbd\ucc30\uc774 \ub418\uc5c8\uc73c\uba70 \uc218\ub294 \uce5c\uad6c\ub4e4\uacfc \uc0ac\uace0\ub9cc \uce58\uace0 \ub2e4\ub154\ub2e4. \uacb0\uad6d \uc218\uac00 \ud06c\uac8c \uc0ac\uace0\ub97c \uce58\uc790 \uadf8 \uae30\ub85d\uc744 \uc740\ud3d0\ud558\uace0 \uacf5\ud654\uad6d \ub3c4\uc2dc\ub97c \ub5a0\ub098\uac8c \ud588\uace0 \uadf8\ub85c \uc778\ud55c \uc8c4\ucc45\uac10\uc73c\ub85c \uc5bc\ub9c8 \uc9c0\ub098\uc9c0 \uc54a\uc544 \uacbd\ucc30\uccad\uc7a5 \uc790\ub9ac\uc5d0\uc11c \uc740\ud1f4\ud55c\ub2e4. \uc740\ud1f4 \ud6c4\uc5d0\ub294 \uc218\uac00 \uc138\uc6b4 \uc790\uc624\ud478\ub97c \uba87\ubc88 \ubc29\ubb38\ud588\ub2e4\uac00 \ud589\ubc29\uc774 \ubb18\uc5f0\ud574\uc9c4\ub2e4. \uc2dc\uc98c 4\uc5d0\uc11c \ud759\uc758 \uc655\uad6d\uc758 \uc232\uc5d0 \uc0b4\uace0\uc788\ub2e4\ub294 \uac83\uc774 \ubc1d\ud600\uc84c\uace0 \ucf54\ub77c\uac00 \ud2b8\ub77c\uc6b0\ub9c8\ub97c \uadf9\ubcf5\ud558\ub294\ub370 \ub3c4\uc6c0\uc744 \uc900\ub2e4. \uc5b4\ub838\uc744\ub54c\ub098 \ub299\uc5c8\uc744\ub54c\ub098 \uad34\ud30d\ud558\uace0 \uc544\ubc14\ud0c0 \uad34\ub86d\ud788\uae30 \uc88b\uc544\ud558\ub294\uac83\uc740 \uc5ec\uc804\ud558\ub2e4.", "paragraph_answer": "\uc804\uc791 \uc544\uc559\uc758 \uc804\uc124\uc758 \uce90\ub9ad\ud130 \uc911 \ud55c\uba85\uc73c\ub85c \ucf54\ub77c\uc758 \uc804\uc124\uc5d0\uc11c\ub294 \uacf5\ud654\uad6d \ub3c4\uc2dc \uccab\ubc88\uc9f8 \uacbd\ucc30 \uccad\uc7a5\uc774\uc5c8\ub2e4. \uc790\uc2dd\uc73c\ub85c\ub294 \ub9b0\uacfc \uc218\ub97c \ub370\ub9ac\uace0 \uc788\ub2e4. \uc5b4\ub9b4\uc801 \uc5c4\uaca9\ud55c \ubd80\ubaa8\ub2d8 \ubc11\uc5d0\uc11c \uc790\ub780\uac83\uc5d0 \ub300\ud55c \ud2b8\ub77c\uc6b0\ub9c8 \ub54c\ubb38\uc5d0 \uc790\uc2dd\ub4e4\uc5d0\uac8c\ub294 \ucd5c\ub300\ud55c\uc758 \uc790\uc720\ub97c \uc8fc\uc5c8\ub2e4. \uadf8\ub85c\uc778\ud574 \uacb0\uad6d \ub9b0\uacfc \uc218\uac00 \uc5b4\uba38\ub2c8\uc758 \uad00\uc2ec\uc744 \ubc1b\uae30 \uc704\ud574 \uacbd\uc7c1\ud588\uace0 \ub9b0\uc740 \uacbd\ucc30\uc774 \ub418\uc5c8\uc73c\uba70 \uc218\ub294 \uce5c\uad6c\ub4e4\uacfc \uc0ac\uace0\ub9cc \uce58\uace0 \ub2e4\ub154\ub2e4. \uacb0\uad6d \uc218\uac00 \ud06c\uac8c \uc0ac\uace0\ub97c \uce58\uc790 \uadf8 \uae30\ub85d\uc744 \uc740\ud3d0\ud558\uace0 \uacf5\ud654\uad6d \ub3c4\uc2dc\ub97c \ub5a0\ub098\uac8c \ud588\uace0 \uadf8\ub85c \uc778\ud55c \uc8c4\ucc45\uac10\uc73c\ub85c \uc5bc\ub9c8 \uc9c0\ub098\uc9c0 \uc54a\uc544 \uacbd\ucc30\uccad\uc7a5 \uc790\ub9ac\uc5d0\uc11c \uc740\ud1f4\ud55c\ub2e4. \uc740\ud1f4 \ud6c4\uc5d0\ub294 \uc218\uac00 \uc138\uc6b4 \uc790\uc624\ud478\ub97c \uba87\ubc88 \ubc29\ubb38\ud588\ub2e4\uac00 \ud589\ubc29\uc774 \ubb18\uc5f0\ud574\uc9c4\ub2e4. \uc2dc\uc98c 4\uc5d0\uc11c \ud759\uc758 \uc655\uad6d\uc758 \uc232\uc5d0 \uc0b4\uace0\uc788\ub2e4\ub294 \uac83\uc774 \ubc1d\ud600\uc84c\uace0 \ucf54\ub77c\uac00 \ud2b8\ub77c\uc6b0\ub9c8\ub97c \uadf9\ubcf5\ud558\ub294\ub370 \ub3c4\uc6c0\uc744 \uc900\ub2e4. \uc5b4\ub838\uc744\ub54c\ub098 \ub299\uc5c8\uc744\ub54c\ub098 \uad34\ud30d\ud558\uace0 \uc544\ubc14\ud0c0 \uad34\ub86d\ud788\uae30 \uc88b\uc544\ud558\ub294\uac83\uc740 \uc5ec\uc804\ud558\ub2e4.", "sentence_answer": "\uc804\uc791 \uc544\uc559\uc758 \uc804\uc124\uc758 \uce90\ub9ad\ud130 \uc911 \ud55c\uba85\uc73c\ub85c \ucf54\ub77c\uc758 \uc804\uc124\uc5d0\uc11c\ub294 \uacf5\ud654\uad6d \ub3c4\uc2dc \uccab\ubc88\uc9f8 \uacbd\ucc30 \uccad\uc7a5\uc774\uc5c8\ub2e4.", "paragraph_id": "6562452-2-1", "paragraph_question": "question: \ucf54\ub77c\uc758 \uc790\uc2dd \uc911 \ub9b0\uc758 \uc9c1\uc5c5\uc740 \ubb34\uc5c7\uc778\uac00?, context: \uc804\uc791 \uc544\uc559\uc758 \uc804\uc124\uc758 \uce90\ub9ad\ud130 \uc911 \ud55c\uba85\uc73c\ub85c \ucf54\ub77c\uc758 \uc804\uc124\uc5d0\uc11c\ub294 \uacf5\ud654\uad6d \ub3c4\uc2dc \uccab\ubc88\uc9f8 \uacbd\ucc30\uccad\uc7a5\uc774\uc5c8\ub2e4. \uc790\uc2dd\uc73c\ub85c\ub294 \ub9b0\uacfc \uc218\ub97c \ub370\ub9ac\uace0 \uc788\ub2e4. \uc5b4\ub9b4\uc801 \uc5c4\uaca9\ud55c \ubd80\ubaa8\ub2d8 \ubc11\uc5d0\uc11c \uc790\ub780\uac83\uc5d0 \ub300\ud55c \ud2b8\ub77c\uc6b0\ub9c8 \ub54c\ubb38\uc5d0 \uc790\uc2dd\ub4e4\uc5d0\uac8c\ub294 \ucd5c\ub300\ud55c\uc758 \uc790\uc720\ub97c \uc8fc\uc5c8\ub2e4. \uadf8\ub85c\uc778\ud574 \uacb0\uad6d \ub9b0\uacfc \uc218\uac00 \uc5b4\uba38\ub2c8\uc758 \uad00\uc2ec\uc744 \ubc1b\uae30 \uc704\ud574 \uacbd\uc7c1\ud588\uace0 \ub9b0\uc740 \uacbd\ucc30\uc774 \ub418\uc5c8\uc73c\uba70 \uc218\ub294 \uce5c\uad6c\ub4e4\uacfc \uc0ac\uace0\ub9cc \uce58\uace0 \ub2e4\ub154\ub2e4. \uacb0\uad6d \uc218\uac00 \ud06c\uac8c \uc0ac\uace0\ub97c \uce58\uc790 \uadf8 \uae30\ub85d\uc744 \uc740\ud3d0\ud558\uace0 \uacf5\ud654\uad6d \ub3c4\uc2dc\ub97c \ub5a0\ub098\uac8c \ud588\uace0 \uadf8\ub85c \uc778\ud55c \uc8c4\ucc45\uac10\uc73c\ub85c \uc5bc\ub9c8 \uc9c0\ub098\uc9c0 \uc54a\uc544 \uacbd\ucc30\uccad\uc7a5 \uc790\ub9ac\uc5d0\uc11c \uc740\ud1f4\ud55c\ub2e4. \uc740\ud1f4 \ud6c4\uc5d0\ub294 \uc218\uac00 \uc138\uc6b4 \uc790\uc624\ud478\ub97c \uba87\ubc88 \ubc29\ubb38\ud588\ub2e4\uac00 \ud589\ubc29\uc774 \ubb18\uc5f0\ud574\uc9c4\ub2e4. \uc2dc\uc98c 4\uc5d0\uc11c \ud759\uc758 \uc655\uad6d\uc758 \uc232\uc5d0 \uc0b4\uace0\uc788\ub2e4\ub294 \uac83\uc774 \ubc1d\ud600\uc84c\uace0 \ucf54\ub77c\uac00 \ud2b8\ub77c\uc6b0\ub9c8\ub97c \uadf9\ubcf5\ud558\ub294\ub370 \ub3c4\uc6c0\uc744 \uc900\ub2e4. \uc5b4\ub838\uc744\ub54c\ub098 \ub299\uc5c8\uc744\ub54c\ub098 \uad34\ud30d\ud558\uace0 \uc544\ubc14\ud0c0 \uad34\ub86d\ud788\uae30 \uc88b\uc544\ud558\ub294\uac83\uc740 \uc5ec\uc804\ud558\ub2e4."} {"question": "\uc2dc\uc704\ub300\uc640 \uacbd\ucc30\uc774 \ub9c8\uc9c0\ub9c9 \ub300\uce58\ub97c \ud55c \uacf3\uc740 \uc5b4\ub514 \ubd80\uadfc\uc774\uc600\ub098?", "paragraph": "\uc885\uac01\uc5d0 \uc9c4\uc785\ud558\uc790 \uc120\ub450 \ucc38\uac00\uc790\ub4e4\uc774 \uc601\ud48d\ubb38\uace0 \uc885\uac01\ubcf8\uc810 \uc55e\uc5d0\uc11c \uc6b0\ud68c\uc804\uc73c\ub85c \uc608\uc815\ub41c \uacbd\ub85c\ub97c \uc774\ud0c8\ud574 \uc870\uacc4\uc0ac \ubc29\ud5a5\uc73c\ub85c \uc9c1\uc9c4\ud558\uae30 \uc2dc\uc791\ud588\ub2e4. \uacbd\ucc30\uc740 \uc885\uac01\uc5ed \ubc29\uba74 \uacf5\ud3c9\ub3d9 \uc77c\ub300\uc5d0\uc11c \uc2dc\uc704\ub300\uc758 \ud589\uc9c4\uc744 \ub9c9\uc558\uace0, \uc2dc\ubbfc\uacfc \uacbd\ucc30 \uac04\uc5d0 \ub300\uce58\uac00 \uc2dc\uc791\ub418\uc5c8\ub2e4. \uc774\uc5d0 \uc77c\ubd80 \uc2dc\ubbfc\ub4e4\uc740 \"\uccad\uc640\ub300\ub85c \uac00\uc790\"\ub294 \uad6c\ud638\ub97c \uc678\uce58\uba70 \uc88c\ud68c\uc804\uc73c\ub85c \uc804\ud658\ud574 \uad11\ud654\ubb38 \ubc29\ud5a5\uc73c\ub85c \ub098\uc11c\uace0\uc790 \ud588\uc73c\uba70, \uadf8 \uacfc\uc815\uc5d0\uc11c \uacbd\ucc30\uc758 1\ucc28 \uc800\uc9c0\uc120\uc774 \ub6ab\ub9ac\uc790 \uc2dc\ubbfc \ub300\uc5f4\uc740 \uc885\ub85c \ucc28\ub85c\ub85c \ub098\uc11c\uac8c \ub418\uc5c8\ub2e4. \uacbd\ucc30 \uce21\uc774 \uae09\ud788 \uad50\ud1b5\uc744 \ud1b5\uc81c\ud558\ub294 \uac00\uc6b4\ub370, 7\uc2dc 55\ubd84\uacbd \uc2dc\uc704\ub300\ub294 \uad50\ubcf4\ubb38\uace0 \uc55e\uc5d0 \uc124\uce58\ub41c 2\ucc28 \uc800\uc9c0\uc120\ub9c8\uc800 \ub6ab\uace0 \uad11\ud654\ubb38 \uad11\uc7a5\uc5d0 \uc9c4\uc785\ud558\uc600\uc73c\ub098, \uacbd\ucc30\uc774 \uc138\uc885\ub300\uc655\uc0c1 \ubd80\uadfc\uc5d0 3\ucc28 \uc800\uc9c0\uc120\uc744 \uc7ac\ube68\ub9ac \ub2e4\uc2dc \uc124\uce58\ud558\uba74\uc11c, \uc2dc\ubbfc\ub4e4\uc740 \uc774\uacf3\uc5d0\uc11c \ucd5c\uc885\uc801\uc778 \ub300\uce58\ub97c \uc774\uc5b4\uac14\ub2e4. \uacbd\ucc30\uc740 \uc2dc\ubbfc\ub4e4\uc774 \uac1c\ubcc4\uc801\uc73c\ub85c \uccad\uc640\ub300\ub85c \ud5a5\ud558\ub294 \uc0ac\ud0dc\uc5d0 \ub300\ube44\ud574 \uad11\ud654\ubb38\uc55e \uc0bc\uac70\ub9ac \uc591\uc606\uc5d0 \ucc28\ubcbd\uc744 \uc124\uce58\ud558\uae30\ub3c4 \ud588\ub2e4.", "answer": "\uc138\uc885\ub300\uc655\uc0c1", "sentence": "\uacbd\ucc30\uc774 \uc138\uc885\ub300\uc655\uc0c1 \ubd80\uadfc\uc5d0 3\ucc28 \uc800\uc9c0\uc120\uc744 \uc7ac\ube68\ub9ac \ub2e4\uc2dc \uc124\uce58\ud558\uba74\uc11c, \uc2dc\ubbfc\ub4e4\uc740 \uc774\uacf3\uc5d0\uc11c \ucd5c\uc885\uc801\uc778 \ub300\uce58\ub97c \uc774\uc5b4\uac14\ub2e4.", "paragraph_sentence": "\uc885\uac01\uc5d0 \uc9c4\uc785\ud558\uc790 \uc120\ub450 \ucc38\uac00\uc790\ub4e4\uc774 \uc601\ud48d\ubb38\uace0 \uc885\uac01\ubcf8\uc810 \uc55e\uc5d0\uc11c \uc6b0\ud68c\uc804\uc73c\ub85c \uc608\uc815\ub41c \uacbd\ub85c\ub97c \uc774\ud0c8\ud574 \uc870\uacc4\uc0ac \ubc29\ud5a5\uc73c\ub85c \uc9c1\uc9c4\ud558\uae30 \uc2dc\uc791\ud588\ub2e4. \uacbd\ucc30\uc740 \uc885\uac01\uc5ed \ubc29\uba74 \uacf5\ud3c9\ub3d9 \uc77c\ub300\uc5d0\uc11c \uc2dc\uc704\ub300\uc758 \ud589\uc9c4\uc744 \ub9c9\uc558\uace0, \uc2dc\ubbfc\uacfc \uacbd\ucc30 \uac04\uc5d0 \ub300\uce58\uac00 \uc2dc\uc791\ub418\uc5c8\ub2e4. \uc774\uc5d0 \uc77c\ubd80 \uc2dc\ubbfc\ub4e4\uc740 \"\uccad\uc640\ub300\ub85c \uac00\uc790\"\ub294 \uad6c\ud638\ub97c \uc678\uce58\uba70 \uc88c\ud68c\uc804\uc73c\ub85c \uc804\ud658\ud574 \uad11\ud654\ubb38 \ubc29\ud5a5\uc73c\ub85c \ub098\uc11c\uace0\uc790 \ud588\uc73c\uba70, \uadf8 \uacfc\uc815\uc5d0\uc11c \uacbd\ucc30\uc758 1\ucc28 \uc800\uc9c0\uc120\uc774 \ub6ab\ub9ac\uc790 \uc2dc\ubbfc \ub300\uc5f4\uc740 \uc885\ub85c \ucc28\ub85c\ub85c \ub098\uc11c\uac8c \ub418\uc5c8\ub2e4. \uacbd\ucc30 \uce21\uc774 \uae09\ud788 \uad50\ud1b5\uc744 \ud1b5\uc81c\ud558\ub294 \uac00\uc6b4\ub370, 7\uc2dc 55\ubd84\uacbd \uc2dc\uc704\ub300\ub294 \uad50\ubcf4\ubb38\uace0 \uc55e\uc5d0 \uc124\uce58\ub41c 2\ucc28 \uc800\uc9c0\uc120\ub9c8\uc800 \ub6ab\uace0 \uad11\ud654\ubb38 \uad11\uc7a5\uc5d0 \uc9c4\uc785\ud558\uc600\uc73c\ub098, \uacbd\ucc30\uc774 \uc138\uc885\ub300\uc655\uc0c1 \ubd80\uadfc\uc5d0 3\ucc28 \uc800\uc9c0\uc120\uc744 \uc7ac\ube68\ub9ac \ub2e4\uc2dc \uc124\uce58\ud558\uba74\uc11c, \uc2dc\ubbfc\ub4e4\uc740 \uc774\uacf3\uc5d0\uc11c \ucd5c\uc885\uc801\uc778 \ub300\uce58\ub97c \uc774\uc5b4\uac14\ub2e4. \uacbd\ucc30\uc740 \uc2dc\ubbfc\ub4e4\uc774 \uac1c\ubcc4\uc801\uc73c\ub85c \uccad\uc640\ub300\ub85c \ud5a5\ud558\ub294 \uc0ac\ud0dc\uc5d0 \ub300\ube44\ud574 \uad11\ud654\ubb38\uc55e \uc0bc\uac70\ub9ac \uc591\uc606\uc5d0 \ucc28\ubcbd\uc744 \uc124\uce58\ud558\uae30\ub3c4 \ud588\ub2e4.", "paragraph_answer": "\uc885\uac01\uc5d0 \uc9c4\uc785\ud558\uc790 \uc120\ub450 \ucc38\uac00\uc790\ub4e4\uc774 \uc601\ud48d\ubb38\uace0 \uc885\uac01\ubcf8\uc810 \uc55e\uc5d0\uc11c \uc6b0\ud68c\uc804\uc73c\ub85c \uc608\uc815\ub41c \uacbd\ub85c\ub97c \uc774\ud0c8\ud574 \uc870\uacc4\uc0ac \ubc29\ud5a5\uc73c\ub85c \uc9c1\uc9c4\ud558\uae30 \uc2dc\uc791\ud588\ub2e4. \uacbd\ucc30\uc740 \uc885\uac01\uc5ed \ubc29\uba74 \uacf5\ud3c9\ub3d9 \uc77c\ub300\uc5d0\uc11c \uc2dc\uc704\ub300\uc758 \ud589\uc9c4\uc744 \ub9c9\uc558\uace0, \uc2dc\ubbfc\uacfc \uacbd\ucc30 \uac04\uc5d0 \ub300\uce58\uac00 \uc2dc\uc791\ub418\uc5c8\ub2e4. \uc774\uc5d0 \uc77c\ubd80 \uc2dc\ubbfc\ub4e4\uc740 \"\uccad\uc640\ub300\ub85c \uac00\uc790\"\ub294 \uad6c\ud638\ub97c \uc678\uce58\uba70 \uc88c\ud68c\uc804\uc73c\ub85c \uc804\ud658\ud574 \uad11\ud654\ubb38 \ubc29\ud5a5\uc73c\ub85c \ub098\uc11c\uace0\uc790 \ud588\uc73c\uba70, \uadf8 \uacfc\uc815\uc5d0\uc11c \uacbd\ucc30\uc758 1\ucc28 \uc800\uc9c0\uc120\uc774 \ub6ab\ub9ac\uc790 \uc2dc\ubbfc \ub300\uc5f4\uc740 \uc885\ub85c \ucc28\ub85c\ub85c \ub098\uc11c\uac8c \ub418\uc5c8\ub2e4. \uacbd\ucc30 \uce21\uc774 \uae09\ud788 \uad50\ud1b5\uc744 \ud1b5\uc81c\ud558\ub294 \uac00\uc6b4\ub370, 7\uc2dc 55\ubd84\uacbd \uc2dc\uc704\ub300\ub294 \uad50\ubcf4\ubb38\uace0 \uc55e\uc5d0 \uc124\uce58\ub41c 2\ucc28 \uc800\uc9c0\uc120\ub9c8\uc800 \ub6ab\uace0 \uad11\ud654\ubb38 \uad11\uc7a5\uc5d0 \uc9c4\uc785\ud558\uc600\uc73c\ub098, \uacbd\ucc30\uc774 \uc138\uc885\ub300\uc655\uc0c1 \ubd80\uadfc\uc5d0 3\ucc28 \uc800\uc9c0\uc120\uc744 \uc7ac\ube68\ub9ac \ub2e4\uc2dc \uc124\uce58\ud558\uba74\uc11c, \uc2dc\ubbfc\ub4e4\uc740 \uc774\uacf3\uc5d0\uc11c \ucd5c\uc885\uc801\uc778 \ub300\uce58\ub97c \uc774\uc5b4\uac14\ub2e4. \uacbd\ucc30\uc740 \uc2dc\ubbfc\ub4e4\uc774 \uac1c\ubcc4\uc801\uc73c\ub85c \uccad\uc640\ub300\ub85c \ud5a5\ud558\ub294 \uc0ac\ud0dc\uc5d0 \ub300\ube44\ud574 \uad11\ud654\ubb38\uc55e \uc0bc\uac70\ub9ac \uc591\uc606\uc5d0 \ucc28\ubcbd\uc744 \uc124\uce58\ud558\uae30\ub3c4 \ud588\ub2e4.", "sentence_answer": "\uacbd\ucc30\uc774 \uc138\uc885\ub300\uc655\uc0c1 \ubd80\uadfc\uc5d0 3\ucc28 \uc800\uc9c0\uc120\uc744 \uc7ac\ube68\ub9ac \ub2e4\uc2dc \uc124\uce58\ud558\uba74\uc11c, \uc2dc\ubbfc\ub4e4\uc740 \uc774\uacf3\uc5d0\uc11c \ucd5c\uc885\uc801\uc778 \ub300\uce58\ub97c \uc774\uc5b4\uac14\ub2e4.", "paragraph_id": "6546893-12-2", "paragraph_question": "question: \uc2dc\uc704\ub300\uc640 \uacbd\ucc30\uc774 \ub9c8\uc9c0\ub9c9 \ub300\uce58\ub97c \ud55c \uacf3\uc740 \uc5b4\ub514 \ubd80\uadfc\uc774\uc600\ub098?, context: \uc885\uac01\uc5d0 \uc9c4\uc785\ud558\uc790 \uc120\ub450 \ucc38\uac00\uc790\ub4e4\uc774 \uc601\ud48d\ubb38\uace0 \uc885\uac01\ubcf8\uc810 \uc55e\uc5d0\uc11c \uc6b0\ud68c\uc804\uc73c\ub85c \uc608\uc815\ub41c \uacbd\ub85c\ub97c \uc774\ud0c8\ud574 \uc870\uacc4\uc0ac \ubc29\ud5a5\uc73c\ub85c \uc9c1\uc9c4\ud558\uae30 \uc2dc\uc791\ud588\ub2e4. \uacbd\ucc30\uc740 \uc885\uac01\uc5ed \ubc29\uba74 \uacf5\ud3c9\ub3d9 \uc77c\ub300\uc5d0\uc11c \uc2dc\uc704\ub300\uc758 \ud589\uc9c4\uc744 \ub9c9\uc558\uace0, \uc2dc\ubbfc\uacfc \uacbd\ucc30 \uac04\uc5d0 \ub300\uce58\uac00 \uc2dc\uc791\ub418\uc5c8\ub2e4. \uc774\uc5d0 \uc77c\ubd80 \uc2dc\ubbfc\ub4e4\uc740 \"\uccad\uc640\ub300\ub85c \uac00\uc790\"\ub294 \uad6c\ud638\ub97c \uc678\uce58\uba70 \uc88c\ud68c\uc804\uc73c\ub85c \uc804\ud658\ud574 \uad11\ud654\ubb38 \ubc29\ud5a5\uc73c\ub85c \ub098\uc11c\uace0\uc790 \ud588\uc73c\uba70, \uadf8 \uacfc\uc815\uc5d0\uc11c \uacbd\ucc30\uc758 1\ucc28 \uc800\uc9c0\uc120\uc774 \ub6ab\ub9ac\uc790 \uc2dc\ubbfc \ub300\uc5f4\uc740 \uc885\ub85c \ucc28\ub85c\ub85c \ub098\uc11c\uac8c \ub418\uc5c8\ub2e4. \uacbd\ucc30 \uce21\uc774 \uae09\ud788 \uad50\ud1b5\uc744 \ud1b5\uc81c\ud558\ub294 \uac00\uc6b4\ub370, 7\uc2dc 55\ubd84\uacbd \uc2dc\uc704\ub300\ub294 \uad50\ubcf4\ubb38\uace0 \uc55e\uc5d0 \uc124\uce58\ub41c 2\ucc28 \uc800\uc9c0\uc120\ub9c8\uc800 \ub6ab\uace0 \uad11\ud654\ubb38 \uad11\uc7a5\uc5d0 \uc9c4\uc785\ud558\uc600\uc73c\ub098, \uacbd\ucc30\uc774 \uc138\uc885\ub300\uc655\uc0c1 \ubd80\uadfc\uc5d0 3\ucc28 \uc800\uc9c0\uc120\uc744 \uc7ac\ube68\ub9ac \ub2e4\uc2dc \uc124\uce58\ud558\uba74\uc11c, \uc2dc\ubbfc\ub4e4\uc740 \uc774\uacf3\uc5d0\uc11c \ucd5c\uc885\uc801\uc778 \ub300\uce58\ub97c \uc774\uc5b4\uac14\ub2e4. \uacbd\ucc30\uc740 \uc2dc\ubbfc\ub4e4\uc774 \uac1c\ubcc4\uc801\uc73c\ub85c \uccad\uc640\ub300\ub85c \ud5a5\ud558\ub294 \uc0ac\ud0dc\uc5d0 \ub300\ube44\ud574 \uad11\ud654\ubb38\uc55e \uc0bc\uac70\ub9ac \uc591\uc606\uc5d0 \ucc28\ubcbd\uc744 \uc124\uce58\ud558\uae30\ub3c4 \ud588\ub2e4."} {"question": "\ub7ec\uc2dc\uc544 \uc18c\uce58 \ub3d9\uacc4 \uc62c\ub9bc\ud53d\uc5d0\uc11c \uc2ec\uc11d\ud76c\uac00 \uae08\uba54\ub2ec\uc744 \ub534 \uacbd\uae30\ub294 \uc5b4\ub290 \uacbd\uae30\uc778\uac00?", "paragraph": "\ucc98\uc74c \ucd9c\uc804\ud55c 2014\ub144 \ub7ec\uc2dc\uc544 \uc18c\uce58 \ub3d9\uacc4 \uc62c\ub9bc\ud53d\uc5d0\uc11c 2\uc6d4 15\uc77c \uc1fc\ud2b8\ud2b8\ub799 \uc5ec\uc790 1500m\uc5d0\uc11c \uc740\uba54\ub2ec\uc744 \ud68d\ub4dd\ud558\uc600\uace0, 3\uc77c \ud6c4\uc778 2\uc6d4 18\uc77c \uc1fc\ud2b8\ud2b8\ub799 \uc5ec\uc790 3000m \uacc4\uc8fc\uc5d0\uc11c \uae08\uba54\ub2ec\uc744 \ud68d\ub4dd\ud558\uc600\ub2e4. \ub300\ud55c\ubbfc\uad6d\uc758 \uc18c\uce58 \ub3d9\uacc4 \uc62c\ub9bc\ud53d \uccab \ubc88\uc9f8 \uc1fc\ud2b8\ud2b8\ub799 \uae08\uba54\ub2ec\uc774\uc5c8\ub2e4. \uc2ec\uc11d\ud76c\uac00 \ub9c9\ud310 \ubc18\ubc14\ud034\ub97c \ub0a8\uaca8\ub193\uace0 \uc911\uad6d \uc120\uc218\ub97c \uadf9\uc801\uc73c\ub85c \ucd94\uc6d4\ud558\uc5ec \uacb0\uc2b9\uc120\uc744 1\ub4f1\uc73c\ub85c \ud1b5\uacfc\ud558\uc600\ub2e4. \uc911\uad6d \ub300\ud45c\ud300\uc740 \ub9c8\uc9c0\ub9c9 2\ubc14\ud034\ub97c \ub0a8\uaca8\ub193\uace0 \uc2ec\uc11d\ud76c\uc5d0\uac8c \uc9c4\ub85c\ubc29\ud574\ud558\uc5ec \uc2e4\uaca9\ub2f9\ud558\uc600\ub2e4. 2010\ub144 \ubc34\ucfe0\ubc84 \ub3d9\uacc4 \uc62c\ub9bc\ud53d\uc5d0\uc11c \ub300\ud55c\ubbfc\uad6d \uc5ec\uc790 \uc1fc\ud2b8\ud2b8\ub799 \ub300\ud45c\ud300\uc774 1\ub4f1\uc73c\ub85c \uacb0\uc2b9\uc120\uc744 \ud1b5\uacfc\ud558\uace0\ub3c4 \uc2ec\ud310\uc758 \uc11d\uc5f0\uce58 \uc54a\uc740 \ud310\uc815\uc73c\ub85c \uc2e4\uaca9\ub2f9\ud558\uba70 \uc911\uad6d \ub300\ud45c\ud300\uc5d0\uac8c \uae08\uba54\ub2ec\uc744 \ube7c\uc557\uae34 \uc9c0 4\ub144\ub9cc\uc5d0 \uc911\uad6d \ub300\ud45c\ud300\uc5d0\uac8c \ub611\uac19\uc774 \uc124\uc695\ud55c \uac83\uc774\ub2e4. 4\uc77c \ud6c4\uc778 2\uc6d4 22\uc77c \uc1fc\ud2b8\ud2b8\ub799 \uc5ec\uc790 1000m\uc5d0\uc11c \ub3d9\uba54\ub2ec\uc744 \ud68d\ub4dd\ud558\uc600\ub2e4. \uc774\ub85c\uc368, \uc2ec\uc11d\ud76c\ub294 \uc18c\uce58 \ub3d9\uacc4 \uc62c\ub9bc\ud53d\uc5d0\uc11c \uae08\uba54\ub2ec 1\uac1c, \uc740\uba54\ub2ec 1\uac1c, \ub3d9\uba54\ub2ec 1\uac1c\ub85c, 3\uac00\uc9c0 \uba54\ub2ec\uc744 \ubaa8\ub450 \ud68d\ub4dd\ud55c \uc120\uc218\uac00 \ub418\uc5c8\ub2e4.", "answer": "\uc1fc\ud2b8\ud2b8\ub799 \uc5ec\uc790 3000m \uacc4\uc8fc", "sentence": "\ucc98\uc74c \ucd9c\uc804\ud55c 2014\ub144 \ub7ec\uc2dc\uc544 \uc18c\uce58 \ub3d9\uacc4 \uc62c\ub9bc\ud53d\uc5d0\uc11c 2\uc6d4 15\uc77c \uc1fc\ud2b8\ud2b8\ub799 \uc5ec\uc790 1500m\uc5d0\uc11c \uc740\uba54\ub2ec\uc744 \ud68d\ub4dd\ud558\uc600\uace0, 3\uc77c \ud6c4\uc778 2\uc6d4 18\uc77c \uc1fc\ud2b8\ud2b8\ub799 \uc5ec\uc790 3000m \uacc4\uc8fc \uc5d0\uc11c \uae08\uba54\ub2ec\uc744 \ud68d\ub4dd\ud558\uc600\ub2e4.", "paragraph_sentence": " \ucc98\uc74c \ucd9c\uc804\ud55c 2014\ub144 \ub7ec\uc2dc\uc544 \uc18c\uce58 \ub3d9\uacc4 \uc62c\ub9bc\ud53d\uc5d0\uc11c 2\uc6d4 15\uc77c \uc1fc\ud2b8\ud2b8\ub799 \uc5ec\uc790 1500m\uc5d0\uc11c \uc740\uba54\ub2ec\uc744 \ud68d\ub4dd\ud558\uc600\uace0, 3\uc77c \ud6c4\uc778 2\uc6d4 18\uc77c \uc1fc\ud2b8\ud2b8\ub799 \uc5ec\uc790 3000m \uacc4\uc8fc \uc5d0\uc11c \uae08\uba54\ub2ec\uc744 \ud68d\ub4dd\ud558\uc600\ub2e4. \ub300\ud55c\ubbfc\uad6d\uc758 \uc18c\uce58 \ub3d9\uacc4 \uc62c\ub9bc\ud53d \uccab \ubc88\uc9f8 \uc1fc\ud2b8\ud2b8\ub799 \uae08\uba54\ub2ec\uc774\uc5c8\ub2e4. \uc2ec\uc11d\ud76c\uac00 \ub9c9\ud310 \ubc18\ubc14\ud034\ub97c \ub0a8\uaca8\ub193\uace0 \uc911\uad6d \uc120\uc218\ub97c \uadf9\uc801\uc73c\ub85c \ucd94\uc6d4\ud558\uc5ec \uacb0\uc2b9\uc120\uc744 1\ub4f1\uc73c\ub85c \ud1b5\uacfc\ud558\uc600\ub2e4. \uc911\uad6d \ub300\ud45c\ud300\uc740 \ub9c8\uc9c0\ub9c9 2\ubc14\ud034\ub97c \ub0a8\uaca8\ub193\uace0 \uc2ec\uc11d\ud76c\uc5d0\uac8c \uc9c4\ub85c\ubc29\ud574\ud558\uc5ec \uc2e4\uaca9\ub2f9\ud558\uc600\ub2e4. 2010\ub144 \ubc34\ucfe0\ubc84 \ub3d9\uacc4 \uc62c\ub9bc\ud53d\uc5d0\uc11c \ub300\ud55c\ubbfc\uad6d \uc5ec\uc790 \uc1fc\ud2b8\ud2b8\ub799 \ub300\ud45c\ud300\uc774 1\ub4f1\uc73c\ub85c \uacb0\uc2b9\uc120\uc744 \ud1b5\uacfc\ud558\uace0\ub3c4 \uc2ec\ud310\uc758 \uc11d\uc5f0\uce58 \uc54a\uc740 \ud310\uc815\uc73c\ub85c \uc2e4\uaca9\ub2f9\ud558\uba70 \uc911\uad6d \ub300\ud45c\ud300\uc5d0\uac8c \uae08\uba54\ub2ec\uc744 \ube7c\uc557\uae34 \uc9c0 4\ub144\ub9cc\uc5d0 \uc911\uad6d \ub300\ud45c\ud300\uc5d0\uac8c \ub611\uac19\uc774 \uc124\uc695\ud55c \uac83\uc774\ub2e4. 4\uc77c \ud6c4\uc778 2\uc6d4 22\uc77c \uc1fc\ud2b8\ud2b8\ub799 \uc5ec\uc790 1000m\uc5d0\uc11c \ub3d9\uba54\ub2ec\uc744 \ud68d\ub4dd\ud558\uc600\ub2e4. \uc774\ub85c\uc368, \uc2ec\uc11d\ud76c\ub294 \uc18c\uce58 \ub3d9\uacc4 \uc62c\ub9bc\ud53d\uc5d0\uc11c \uae08\uba54\ub2ec 1\uac1c, \uc740\uba54\ub2ec 1\uac1c, \ub3d9\uba54\ub2ec 1\uac1c\ub85c, 3\uac00\uc9c0 \uba54\ub2ec\uc744 \ubaa8\ub450 \ud68d\ub4dd\ud55c \uc120\uc218\uac00 \ub418\uc5c8\ub2e4.", "paragraph_answer": "\ucc98\uc74c \ucd9c\uc804\ud55c 2014\ub144 \ub7ec\uc2dc\uc544 \uc18c\uce58 \ub3d9\uacc4 \uc62c\ub9bc\ud53d\uc5d0\uc11c 2\uc6d4 15\uc77c \uc1fc\ud2b8\ud2b8\ub799 \uc5ec\uc790 1500m\uc5d0\uc11c \uc740\uba54\ub2ec\uc744 \ud68d\ub4dd\ud558\uc600\uace0, 3\uc77c \ud6c4\uc778 2\uc6d4 18\uc77c \uc1fc\ud2b8\ud2b8\ub799 \uc5ec\uc790 3000m \uacc4\uc8fc \uc5d0\uc11c \uae08\uba54\ub2ec\uc744 \ud68d\ub4dd\ud558\uc600\ub2e4. \ub300\ud55c\ubbfc\uad6d\uc758 \uc18c\uce58 \ub3d9\uacc4 \uc62c\ub9bc\ud53d \uccab \ubc88\uc9f8 \uc1fc\ud2b8\ud2b8\ub799 \uae08\uba54\ub2ec\uc774\uc5c8\ub2e4. \uc2ec\uc11d\ud76c\uac00 \ub9c9\ud310 \ubc18\ubc14\ud034\ub97c \ub0a8\uaca8\ub193\uace0 \uc911\uad6d \uc120\uc218\ub97c \uadf9\uc801\uc73c\ub85c \ucd94\uc6d4\ud558\uc5ec \uacb0\uc2b9\uc120\uc744 1\ub4f1\uc73c\ub85c \ud1b5\uacfc\ud558\uc600\ub2e4. \uc911\uad6d \ub300\ud45c\ud300\uc740 \ub9c8\uc9c0\ub9c9 2\ubc14\ud034\ub97c \ub0a8\uaca8\ub193\uace0 \uc2ec\uc11d\ud76c\uc5d0\uac8c \uc9c4\ub85c\ubc29\ud574\ud558\uc5ec \uc2e4\uaca9\ub2f9\ud558\uc600\ub2e4. 2010\ub144 \ubc34\ucfe0\ubc84 \ub3d9\uacc4 \uc62c\ub9bc\ud53d\uc5d0\uc11c \ub300\ud55c\ubbfc\uad6d \uc5ec\uc790 \uc1fc\ud2b8\ud2b8\ub799 \ub300\ud45c\ud300\uc774 1\ub4f1\uc73c\ub85c \uacb0\uc2b9\uc120\uc744 \ud1b5\uacfc\ud558\uace0\ub3c4 \uc2ec\ud310\uc758 \uc11d\uc5f0\uce58 \uc54a\uc740 \ud310\uc815\uc73c\ub85c \uc2e4\uaca9\ub2f9\ud558\uba70 \uc911\uad6d \ub300\ud45c\ud300\uc5d0\uac8c \uae08\uba54\ub2ec\uc744 \ube7c\uc557\uae34 \uc9c0 4\ub144\ub9cc\uc5d0 \uc911\uad6d \ub300\ud45c\ud300\uc5d0\uac8c \ub611\uac19\uc774 \uc124\uc695\ud55c \uac83\uc774\ub2e4. 4\uc77c \ud6c4\uc778 2\uc6d4 22\uc77c \uc1fc\ud2b8\ud2b8\ub799 \uc5ec\uc790 1000m\uc5d0\uc11c \ub3d9\uba54\ub2ec\uc744 \ud68d\ub4dd\ud558\uc600\ub2e4. \uc774\ub85c\uc368, \uc2ec\uc11d\ud76c\ub294 \uc18c\uce58 \ub3d9\uacc4 \uc62c\ub9bc\ud53d\uc5d0\uc11c \uae08\uba54\ub2ec 1\uac1c, \uc740\uba54\ub2ec 1\uac1c, \ub3d9\uba54\ub2ec 1\uac1c\ub85c, 3\uac00\uc9c0 \uba54\ub2ec\uc744 \ubaa8\ub450 \ud68d\ub4dd\ud55c \uc120\uc218\uac00 \ub418\uc5c8\ub2e4.", "sentence_answer": "\ucc98\uc74c \ucd9c\uc804\ud55c 2014\ub144 \ub7ec\uc2dc\uc544 \uc18c\uce58 \ub3d9\uacc4 \uc62c\ub9bc\ud53d\uc5d0\uc11c 2\uc6d4 15\uc77c \uc1fc\ud2b8\ud2b8\ub799 \uc5ec\uc790 1500m\uc5d0\uc11c \uc740\uba54\ub2ec\uc744 \ud68d\ub4dd\ud558\uc600\uace0, 3\uc77c \ud6c4\uc778 2\uc6d4 18\uc77c \uc1fc\ud2b8\ud2b8\ub799 \uc5ec\uc790 3000m \uacc4\uc8fc \uc5d0\uc11c \uae08\uba54\ub2ec\uc744 \ud68d\ub4dd\ud558\uc600\ub2e4.", "paragraph_id": "6519943-1-1", "paragraph_question": "question: \ub7ec\uc2dc\uc544 \uc18c\uce58 \ub3d9\uacc4 \uc62c\ub9bc\ud53d\uc5d0\uc11c \uc2ec\uc11d\ud76c\uac00 \uae08\uba54\ub2ec\uc744 \ub534 \uacbd\uae30\ub294 \uc5b4\ub290 \uacbd\uae30\uc778\uac00?, context: \ucc98\uc74c \ucd9c\uc804\ud55c 2014\ub144 \ub7ec\uc2dc\uc544 \uc18c\uce58 \ub3d9\uacc4 \uc62c\ub9bc\ud53d\uc5d0\uc11c 2\uc6d4 15\uc77c \uc1fc\ud2b8\ud2b8\ub799 \uc5ec\uc790 1500m\uc5d0\uc11c \uc740\uba54\ub2ec\uc744 \ud68d\ub4dd\ud558\uc600\uace0, 3\uc77c \ud6c4\uc778 2\uc6d4 18\uc77c \uc1fc\ud2b8\ud2b8\ub799 \uc5ec\uc790 3000m \uacc4\uc8fc\uc5d0\uc11c \uae08\uba54\ub2ec\uc744 \ud68d\ub4dd\ud558\uc600\ub2e4. \ub300\ud55c\ubbfc\uad6d\uc758 \uc18c\uce58 \ub3d9\uacc4 \uc62c\ub9bc\ud53d \uccab \ubc88\uc9f8 \uc1fc\ud2b8\ud2b8\ub799 \uae08\uba54\ub2ec\uc774\uc5c8\ub2e4. \uc2ec\uc11d\ud76c\uac00 \ub9c9\ud310 \ubc18\ubc14\ud034\ub97c \ub0a8\uaca8\ub193\uace0 \uc911\uad6d \uc120\uc218\ub97c \uadf9\uc801\uc73c\ub85c \ucd94\uc6d4\ud558\uc5ec \uacb0\uc2b9\uc120\uc744 1\ub4f1\uc73c\ub85c \ud1b5\uacfc\ud558\uc600\ub2e4. \uc911\uad6d \ub300\ud45c\ud300\uc740 \ub9c8\uc9c0\ub9c9 2\ubc14\ud034\ub97c \ub0a8\uaca8\ub193\uace0 \uc2ec\uc11d\ud76c\uc5d0\uac8c \uc9c4\ub85c\ubc29\ud574\ud558\uc5ec \uc2e4\uaca9\ub2f9\ud558\uc600\ub2e4. 2010\ub144 \ubc34\ucfe0\ubc84 \ub3d9\uacc4 \uc62c\ub9bc\ud53d\uc5d0\uc11c \ub300\ud55c\ubbfc\uad6d \uc5ec\uc790 \uc1fc\ud2b8\ud2b8\ub799 \ub300\ud45c\ud300\uc774 1\ub4f1\uc73c\ub85c \uacb0\uc2b9\uc120\uc744 \ud1b5\uacfc\ud558\uace0\ub3c4 \uc2ec\ud310\uc758 \uc11d\uc5f0\uce58 \uc54a\uc740 \ud310\uc815\uc73c\ub85c \uc2e4\uaca9\ub2f9\ud558\uba70 \uc911\uad6d \ub300\ud45c\ud300\uc5d0\uac8c \uae08\uba54\ub2ec\uc744 \ube7c\uc557\uae34 \uc9c0 4\ub144\ub9cc\uc5d0 \uc911\uad6d \ub300\ud45c\ud300\uc5d0\uac8c \ub611\uac19\uc774 \uc124\uc695\ud55c \uac83\uc774\ub2e4. 4\uc77c \ud6c4\uc778 2\uc6d4 22\uc77c \uc1fc\ud2b8\ud2b8\ub799 \uc5ec\uc790 1000m\uc5d0\uc11c \ub3d9\uba54\ub2ec\uc744 \ud68d\ub4dd\ud558\uc600\ub2e4. \uc774\ub85c\uc368, \uc2ec\uc11d\ud76c\ub294 \uc18c\uce58 \ub3d9\uacc4 \uc62c\ub9bc\ud53d\uc5d0\uc11c \uae08\uba54\ub2ec 1\uac1c, \uc740\uba54\ub2ec 1\uac1c, \ub3d9\uba54\ub2ec 1\uac1c\ub85c, 3\uac00\uc9c0 \uba54\ub2ec\uc744 \ubaa8\ub450 \ud68d\ub4dd\ud55c \uc120\uc218\uac00 \ub418\uc5c8\ub2e4."} {"question": "\uc18c\ub828\uad70\uc774 \ud1a0\uc131 \uc791\uc804\uc744 \uc2e4\ud589\ud558\uc5ec \ud0c8\ud658\ud558\uace0\uc790 \ud588\ub358 \uc9c0\uc5ed\uc740 \uc5b4\ub514\uc778\uac00?", "paragraph": "\ud55c\ud3b8, \uc18c\ub828\uad70\uc740 \ub85c\uc2a4\ud1a0\ud504\ub97c \ud0c8\ud658\ud558\uace0 \ub3c5\uc77c A \uc9d1\ub2e8\uad70\uc744 \uace0\ub9bd\uc2dc\ud0a4\uae30 \uc704\ud55c \uc790\uae30\ub124\ub4e4\uc758 \uacf5\uc138\uc791\uc804\uc744 \uac1c\uc2dc\ud588\ub2e4(\ud1a0\uc131 \uc791\uc804). \uadf8\ub7ec\ub098 \uaca8\uc6b8\ud3ed\ud48d \uc791\uc804\uc774 \uac1c\uc2dc\ub418\uc790 \uc18c\ub828\uad70\uc740 \uc2a4\ud0c8\ub9b0\uadf8\ub77c\ub4dc \uad6c\uc6d0\uc744 \ub9c9\uae30 \uc704\ud558\uc5ec \ubcd1\ub825 \uc77c\ubd80\ub97c \uc7ac\ubc30\uce58\ud574\uc57c \ud588\ub2e4. \ub54c\ubb38\uc5d0 \uacf5\uc138\uc791\uc804\uc758 \uaddc\ubaa8\uac00 \uc904\uc5b4\ub4e4\uc5c8\uace0, \u201c\uc18c\ud1a0\uc131 \uc791\uc804\u201d\uc774\ub77c\ub294 \ubcc4\uba85\uc774 \ubd99\uc5c8\ub2e4. \uc544\ubb34\ud2bc \ud1a0\uc131 \uc791\uc804\uc774 \uc2dc\uc791\ub418\uc790 \ub9cc\uc288\ud0c0\uc778\uc740 \uc804\uc120 \uc804\uccb4\uc758 \ubd95\uad34\ub97c \ub9c9\uae30 \uc704\ud574 \ubcd1\ub825\uc758 \ubd84\uc0b0\uc744 \uac15\uc694\ub2f9\ud588\ub2e4. \ub610\ud55c \uc774 \uacf5\uaca9\uc73c\ub85c \uc778\ud574 \uc624\ud1a0 \ud3f0 \ud06c\ub178\ubca8\uc2a4\ub3c4\ub974\ud504 \uae30\uac11\ub300\uc7a5\uc758 \uc81c48\uae30\uac11\uad70\ub2e8(\uc608\ud558 \ubd80\ub300: \uc81c336\ubcf4\ubcd1\uc0ac\ub2e8, \uc81c3\uacf5\uad70\uc57c\uc804\uc0ac\ub2e8, \uc81c11\uae30\uac11\uc0ac\ub2e8)\uc774 \ud30c\uc6b8\ub8e8\uc2a4\uc758 \uc81c27\uae30\uac11\uad70\ub2e8\uacfc \uc811\uc120\ud558\ub824\ub358 \uacc4\ud68d \uc790\uccb4\uac00 \ud2c0\uc5b4\uc838 \ubc84\ub838\ub2e4. \uc81c48\uad70\ub2e8\uc740 \uad6c\ucd9c \uc784\ubb34\uc5d0 \ud22c\uc785\ub418\ub294 \ub300\uc2e0 \uce58\ub974 \uac15\uc744 \ub530\ub77c \ub3c4\uc5f4\ud558\uc5ec \uc18c\ub828\uad70\uc758 \uacf5\uaca9\uc744 \ub9c9\uc544\ub0c8\ub2e4. \ud5e4\ub974\ub9cc \ubc1c\ud06c \uae30\uac11\ub300\uc7a5\uc774 \uc81c11\uc0ac\ub2e8\uc73c\ub85c \uc5ed\uacf5\uc744 \uac00\ud55c \uac83\uc774 \uc8fc\ud6a8\ud588\ub2e4. \ub3c5\uc77c\uad70\uc740 \uc804\uc5f4\ubd95\uad34 \uc9c1\uc804\uc5d0 \uc804\uc5f4\uc744 \uc7ac\uc815\ube44\ud588\uc73c\ub098, \uce21\uba74\uc758 \uc774\ud0c8\ub9ac\uc544 \uc81c8\uad70\uc740 \uc18c\ub828\uad70\uc5d0\uac8c \uc555\ub3c4\ub418\uc5b4 \uada4\uba78\ub2f9\ud588\ub2e4.", "answer": "\ub85c\uc2a4\ud1a0\ud504", "sentence": "\uc18c\ub828\uad70\uc740 \ub85c\uc2a4\ud1a0\ud504 \ub97c \ud0c8\ud658\ud558\uace0 \ub3c5\uc77c A \uc9d1\ub2e8\uad70\uc744 \uace0\ub9bd\uc2dc\ud0a4\uae30 \uc704\ud55c \uc790\uae30\ub124\ub4e4\uc758 \uacf5\uc138\uc791\uc804\uc744 \uac1c\uc2dc\ud588\ub2e4(\ud1a0\uc131 \uc791\uc804).", "paragraph_sentence": "\ud55c\ud3b8, \uc18c\ub828\uad70\uc740 \ub85c\uc2a4\ud1a0\ud504 \ub97c \ud0c8\ud658\ud558\uace0 \ub3c5\uc77c A \uc9d1\ub2e8\uad70\uc744 \uace0\ub9bd\uc2dc\ud0a4\uae30 \uc704\ud55c \uc790\uae30\ub124\ub4e4\uc758 \uacf5\uc138\uc791\uc804\uc744 \uac1c\uc2dc\ud588\ub2e4(\ud1a0\uc131 \uc791\uc804). \uadf8\ub7ec\ub098 \uaca8\uc6b8\ud3ed\ud48d \uc791\uc804\uc774 \uac1c\uc2dc\ub418\uc790 \uc18c\ub828\uad70\uc740 \uc2a4\ud0c8\ub9b0\uadf8\ub77c\ub4dc \uad6c\uc6d0\uc744 \ub9c9\uae30 \uc704\ud558\uc5ec \ubcd1\ub825 \uc77c\ubd80\ub97c \uc7ac\ubc30\uce58\ud574\uc57c \ud588\ub2e4. \ub54c\ubb38\uc5d0 \uacf5\uc138\uc791\uc804\uc758 \uaddc\ubaa8\uac00 \uc904\uc5b4\ub4e4\uc5c8\uace0, \u201c\uc18c\ud1a0\uc131 \uc791\uc804\u201d\uc774\ub77c\ub294 \ubcc4\uba85\uc774 \ubd99\uc5c8\ub2e4. \uc544\ubb34\ud2bc \ud1a0\uc131 \uc791\uc804\uc774 \uc2dc\uc791\ub418\uc790 \ub9cc\uc288\ud0c0\uc778\uc740 \uc804\uc120 \uc804\uccb4\uc758 \ubd95\uad34\ub97c \ub9c9\uae30 \uc704\ud574 \ubcd1\ub825\uc758 \ubd84\uc0b0\uc744 \uac15\uc694\ub2f9\ud588\ub2e4. \ub610\ud55c \uc774 \uacf5\uaca9\uc73c\ub85c \uc778\ud574 \uc624\ud1a0 \ud3f0 \ud06c\ub178\ubca8\uc2a4\ub3c4\ub974\ud504 \uae30\uac11\ub300\uc7a5\uc758 \uc81c48\uae30\uac11\uad70\ub2e8(\uc608\ud558 \ubd80\ub300: \uc81c336\ubcf4\ubcd1\uc0ac\ub2e8, \uc81c3\uacf5\uad70\uc57c\uc804\uc0ac\ub2e8, \uc81c11\uae30\uac11\uc0ac\ub2e8)\uc774 \ud30c\uc6b8\ub8e8\uc2a4\uc758 \uc81c27\uae30\uac11\uad70\ub2e8\uacfc \uc811\uc120\ud558\ub824\ub358 \uacc4\ud68d \uc790\uccb4\uac00 \ud2c0\uc5b4\uc838 \ubc84\ub838\ub2e4. \uc81c48\uad70\ub2e8\uc740 \uad6c\ucd9c \uc784\ubb34\uc5d0 \ud22c\uc785\ub418\ub294 \ub300\uc2e0 \uce58\ub974 \uac15\uc744 \ub530\ub77c \ub3c4\uc5f4\ud558\uc5ec \uc18c\ub828\uad70\uc758 \uacf5\uaca9\uc744 \ub9c9\uc544\ub0c8\ub2e4. \ud5e4\ub974\ub9cc \ubc1c\ud06c \uae30\uac11\ub300\uc7a5\uc774 \uc81c11\uc0ac\ub2e8\uc73c\ub85c \uc5ed\uacf5\uc744 \uac00\ud55c \uac83\uc774 \uc8fc\ud6a8\ud588\ub2e4. \ub3c5\uc77c\uad70\uc740 \uc804\uc5f4\ubd95\uad34 \uc9c1\uc804\uc5d0 \uc804\uc5f4\uc744 \uc7ac\uc815\ube44\ud588\uc73c\ub098, \uce21\uba74\uc758 \uc774\ud0c8\ub9ac\uc544 \uc81c8\uad70\uc740 \uc18c\ub828\uad70\uc5d0\uac8c \uc555\ub3c4\ub418\uc5b4 \uada4\uba78\ub2f9\ud588\ub2e4.", "paragraph_answer": "\ud55c\ud3b8, \uc18c\ub828\uad70\uc740 \ub85c\uc2a4\ud1a0\ud504 \ub97c \ud0c8\ud658\ud558\uace0 \ub3c5\uc77c A \uc9d1\ub2e8\uad70\uc744 \uace0\ub9bd\uc2dc\ud0a4\uae30 \uc704\ud55c \uc790\uae30\ub124\ub4e4\uc758 \uacf5\uc138\uc791\uc804\uc744 \uac1c\uc2dc\ud588\ub2e4(\ud1a0\uc131 \uc791\uc804). \uadf8\ub7ec\ub098 \uaca8\uc6b8\ud3ed\ud48d \uc791\uc804\uc774 \uac1c\uc2dc\ub418\uc790 \uc18c\ub828\uad70\uc740 \uc2a4\ud0c8\ub9b0\uadf8\ub77c\ub4dc \uad6c\uc6d0\uc744 \ub9c9\uae30 \uc704\ud558\uc5ec \ubcd1\ub825 \uc77c\ubd80\ub97c \uc7ac\ubc30\uce58\ud574\uc57c \ud588\ub2e4. \ub54c\ubb38\uc5d0 \uacf5\uc138\uc791\uc804\uc758 \uaddc\ubaa8\uac00 \uc904\uc5b4\ub4e4\uc5c8\uace0, \u201c\uc18c\ud1a0\uc131 \uc791\uc804\u201d\uc774\ub77c\ub294 \ubcc4\uba85\uc774 \ubd99\uc5c8\ub2e4. \uc544\ubb34\ud2bc \ud1a0\uc131 \uc791\uc804\uc774 \uc2dc\uc791\ub418\uc790 \ub9cc\uc288\ud0c0\uc778\uc740 \uc804\uc120 \uc804\uccb4\uc758 \ubd95\uad34\ub97c \ub9c9\uae30 \uc704\ud574 \ubcd1\ub825\uc758 \ubd84\uc0b0\uc744 \uac15\uc694\ub2f9\ud588\ub2e4. \ub610\ud55c \uc774 \uacf5\uaca9\uc73c\ub85c \uc778\ud574 \uc624\ud1a0 \ud3f0 \ud06c\ub178\ubca8\uc2a4\ub3c4\ub974\ud504 \uae30\uac11\ub300\uc7a5\uc758 \uc81c48\uae30\uac11\uad70\ub2e8(\uc608\ud558 \ubd80\ub300: \uc81c336\ubcf4\ubcd1\uc0ac\ub2e8, \uc81c3\uacf5\uad70\uc57c\uc804\uc0ac\ub2e8, \uc81c11\uae30\uac11\uc0ac\ub2e8)\uc774 \ud30c\uc6b8\ub8e8\uc2a4\uc758 \uc81c27\uae30\uac11\uad70\ub2e8\uacfc \uc811\uc120\ud558\ub824\ub358 \uacc4\ud68d \uc790\uccb4\uac00 \ud2c0\uc5b4\uc838 \ubc84\ub838\ub2e4. \uc81c48\uad70\ub2e8\uc740 \uad6c\ucd9c \uc784\ubb34\uc5d0 \ud22c\uc785\ub418\ub294 \ub300\uc2e0 \uce58\ub974 \uac15\uc744 \ub530\ub77c \ub3c4\uc5f4\ud558\uc5ec \uc18c\ub828\uad70\uc758 \uacf5\uaca9\uc744 \ub9c9\uc544\ub0c8\ub2e4. \ud5e4\ub974\ub9cc \ubc1c\ud06c \uae30\uac11\ub300\uc7a5\uc774 \uc81c11\uc0ac\ub2e8\uc73c\ub85c \uc5ed\uacf5\uc744 \uac00\ud55c \uac83\uc774 \uc8fc\ud6a8\ud588\ub2e4. \ub3c5\uc77c\uad70\uc740 \uc804\uc5f4\ubd95\uad34 \uc9c1\uc804\uc5d0 \uc804\uc5f4\uc744 \uc7ac\uc815\ube44\ud588\uc73c\ub098, \uce21\uba74\uc758 \uc774\ud0c8\ub9ac\uc544 \uc81c8\uad70\uc740 \uc18c\ub828\uad70\uc5d0\uac8c \uc555\ub3c4\ub418\uc5b4 \uada4\uba78\ub2f9\ud588\ub2e4.", "sentence_answer": "\uc18c\ub828\uad70\uc740 \ub85c\uc2a4\ud1a0\ud504 \ub97c \ud0c8\ud658\ud558\uace0 \ub3c5\uc77c A \uc9d1\ub2e8\uad70\uc744 \uace0\ub9bd\uc2dc\ud0a4\uae30 \uc704\ud55c \uc790\uae30\ub124\ub4e4\uc758 \uacf5\uc138\uc791\uc804\uc744 \uac1c\uc2dc\ud588\ub2e4(\ud1a0\uc131 \uc791\uc804).", "paragraph_id": "6540675-22-0", "paragraph_question": "question: \uc18c\ub828\uad70\uc774 \ud1a0\uc131 \uc791\uc804\uc744 \uc2e4\ud589\ud558\uc5ec \ud0c8\ud658\ud558\uace0\uc790 \ud588\ub358 \uc9c0\uc5ed\uc740 \uc5b4\ub514\uc778\uac00?, context: \ud55c\ud3b8, \uc18c\ub828\uad70\uc740 \ub85c\uc2a4\ud1a0\ud504\ub97c \ud0c8\ud658\ud558\uace0 \ub3c5\uc77c A \uc9d1\ub2e8\uad70\uc744 \uace0\ub9bd\uc2dc\ud0a4\uae30 \uc704\ud55c \uc790\uae30\ub124\ub4e4\uc758 \uacf5\uc138\uc791\uc804\uc744 \uac1c\uc2dc\ud588\ub2e4(\ud1a0\uc131 \uc791\uc804). \uadf8\ub7ec\ub098 \uaca8\uc6b8\ud3ed\ud48d \uc791\uc804\uc774 \uac1c\uc2dc\ub418\uc790 \uc18c\ub828\uad70\uc740 \uc2a4\ud0c8\ub9b0\uadf8\ub77c\ub4dc \uad6c\uc6d0\uc744 \ub9c9\uae30 \uc704\ud558\uc5ec \ubcd1\ub825 \uc77c\ubd80\ub97c \uc7ac\ubc30\uce58\ud574\uc57c \ud588\ub2e4. \ub54c\ubb38\uc5d0 \uacf5\uc138\uc791\uc804\uc758 \uaddc\ubaa8\uac00 \uc904\uc5b4\ub4e4\uc5c8\uace0, \u201c\uc18c\ud1a0\uc131 \uc791\uc804\u201d\uc774\ub77c\ub294 \ubcc4\uba85\uc774 \ubd99\uc5c8\ub2e4. \uc544\ubb34\ud2bc \ud1a0\uc131 \uc791\uc804\uc774 \uc2dc\uc791\ub418\uc790 \ub9cc\uc288\ud0c0\uc778\uc740 \uc804\uc120 \uc804\uccb4\uc758 \ubd95\uad34\ub97c \ub9c9\uae30 \uc704\ud574 \ubcd1\ub825\uc758 \ubd84\uc0b0\uc744 \uac15\uc694\ub2f9\ud588\ub2e4. \ub610\ud55c \uc774 \uacf5\uaca9\uc73c\ub85c \uc778\ud574 \uc624\ud1a0 \ud3f0 \ud06c\ub178\ubca8\uc2a4\ub3c4\ub974\ud504 \uae30\uac11\ub300\uc7a5\uc758 \uc81c48\uae30\uac11\uad70\ub2e8(\uc608\ud558 \ubd80\ub300: \uc81c336\ubcf4\ubcd1\uc0ac\ub2e8, \uc81c3\uacf5\uad70\uc57c\uc804\uc0ac\ub2e8, \uc81c11\uae30\uac11\uc0ac\ub2e8)\uc774 \ud30c\uc6b8\ub8e8\uc2a4\uc758 \uc81c27\uae30\uac11\uad70\ub2e8\uacfc \uc811\uc120\ud558\ub824\ub358 \uacc4\ud68d \uc790\uccb4\uac00 \ud2c0\uc5b4\uc838 \ubc84\ub838\ub2e4. \uc81c48\uad70\ub2e8\uc740 \uad6c\ucd9c \uc784\ubb34\uc5d0 \ud22c\uc785\ub418\ub294 \ub300\uc2e0 \uce58\ub974 \uac15\uc744 \ub530\ub77c \ub3c4\uc5f4\ud558\uc5ec \uc18c\ub828\uad70\uc758 \uacf5\uaca9\uc744 \ub9c9\uc544\ub0c8\ub2e4. \ud5e4\ub974\ub9cc \ubc1c\ud06c \uae30\uac11\ub300\uc7a5\uc774 \uc81c11\uc0ac\ub2e8\uc73c\ub85c \uc5ed\uacf5\uc744 \uac00\ud55c \uac83\uc774 \uc8fc\ud6a8\ud588\ub2e4. \ub3c5\uc77c\uad70\uc740 \uc804\uc5f4\ubd95\uad34 \uc9c1\uc804\uc5d0 \uc804\uc5f4\uc744 \uc7ac\uc815\ube44\ud588\uc73c\ub098, \uce21\uba74\uc758 \uc774\ud0c8\ub9ac\uc544 \uc81c8\uad70\uc740 \uc18c\ub828\uad70\uc5d0\uac8c \uc555\ub3c4\ub418\uc5b4 \uada4\uba78\ub2f9\ud588\ub2e4."} {"question": "\ub274\uc695\uc758 \uc804 \uc2dc\uc7a5\uc73c\ub85c 3\ubc88 \ub2f9\uc120 \ub418\uc5c8\uace0, 2006\ub144\uc5d0 \ubcf4\uc2a4\ud134 \uc2dc\uc7a5\uacfc \ubd88\ubc95 \ucd1d\uae30\uc640 \uc2f8\uc6b0\ub294 \uc2dc\uc7a5 \ud611\ud68c\ub77c\ub294 \ub2e8\uccb4\ub97c \uacb0\uc131\ud55c \uc2dc\uc7a5\uc740?", "paragraph": "\ud604\uc7ac \uc2dc\uc7a5\uc740 \ubbfc\uc8fc\ub2f9 \uc18c\uc18d\uc758 \ube4c \ub354\ube14\ub77c\uc9c0\uc624\uc774\ub2e4. \uc804 \uc2dc\uc7a5\uc774\uc5c8\ub358 \ub9c8\uc774\ud074 \ube14\ub8f8\ubc84\uadf8\ub294 \ubb34\uc18c\uc18d\uc774\uc5c8\uc73c\ub098 \uacf5\ud654\ub2f9\uacfc \ub274\uc695 \ub3c5\ub9bd\ub2f9\uc758 \uc9c0\uc9c0\ub97c \ubc1b\uace0, 2001\ub144, 2005\ub144, 2009\ub144\uc5d0 3\ubc88 \ub2f9\uc120\ub418\uc5c8\ub2e4. \ube14\ub8f8\ubc84\uadf8\ub294 \ub274\uc695\uc758 \uad50\uc721 \uc2dc\uc2a4\ud15c\uc758 \uad8c\ud55c\uc744 \uad6d\uac00\uc5d0\uc11c \uc774\uad00\uc2dc\ud0a8 \uac83, \ub3c4\uc2dc \uacc4\ud68d, \uacbd\uc81c \uc9c4\ud765, \uc7ac\uc815 \uac74\uc804\ud654, \uacf5\uc911 \ubcf4\uac74 \uc815\ucc45 \ub4f1\uc73c\ub85c \uc720\uba85\ud558\ub2e4. 2\uae30 \ub54c\uc5d0\ub294 \ud559\uad50 \uac1c\ud601, \ube48\uace4 \uac10\uc18c, \uc5c4\uaca9\ud55c \ucd1d\uae30 \uaddc\uc81c \ub4f1\uc744 \ud589\uc815 \uc911\uc2ec \uacfc\uc81c\uc5d0 \ub450\uc5c8\ub2e4. \ubcf4\uc2a4\ud134 \uc2dc\uc7a5 \ud1a0\uba38\uc2a4 \uba54\ub2c8\ub178\ud558\uace0 2006\ub144, \ubd88\ubc95 \ucd1d\uae30\ub97c \ub3c4\uc2dc\uc5d0\uc11c \uc77c\uc18c\ud568\uc73c\ub85c\uc368 \uacf5\uacf5\uc758 \uc548\uc804\uc744 \ub3c4\ubaa8\ud55c\ub2e4\ub294 \ubaa9\ud45c\ub97c \ub0b4\uac78\uace0, \ubd88\ubc95 \ucd1d\uae30\uc640 \uc2f8\uc6b0\ub294 \uc2dc\uc7a5 \ud611\ud68c\ub77c\ub294 \ub2e8\uccb4\ub97c \uacb0\uc131\ud558\uc600\ub2e4. 1924\ub144 \uc774\ud6c4 \ub274\uc695 \ub0b4\uc5d0\uc11c\ub294 \uad6d\uac00 \uc218\uc900\uc758 \uc120\uac70\uc640 \ub300\ud1b5\ub839 \uc120\uac70\uc5d0\uc11c \uacf5\ud654\ub2f9\uc774 \uc774\uae34 \uc801\uc774 \uc5c6\ub2e4. \uc120\uac70 \uacf5\uc57d\uc758 \ud575\uc2ec \ub17c\uc810\uc740 \uc8fc\ud0dd \uac00\uaca9\uc758 \uc778\ud558, \uad50\uc721, \uacbd\uc81c \uc9c4\ud765 \ub4f1\uc774\uba70, \ub178\ub3d9 \uc815\ucc45\uc774 \uc911\uc694\uc131\uc744 \uac00\uc9c0\uace0 \uc788\ub2e4.", "answer": "\ub9c8\uc774\ud074 \ube14\ub8f8\ubc84\uadf8", "sentence": "\uc804 \uc2dc\uc7a5\uc774\uc5c8\ub358 \ub9c8\uc774\ud074 \ube14\ub8f8\ubc84\uadf8 \ub294 \ubb34\uc18c\uc18d\uc774\uc5c8\uc73c\ub098 \uacf5\ud654\ub2f9\uacfc \ub274\uc695 \ub3c5\ub9bd\ub2f9\uc758 \uc9c0\uc9c0\ub97c \ubc1b\uace0, 2001\ub144, 2005\ub144, 2009\ub144\uc5d0 3\ubc88 \ub2f9\uc120\ub418\uc5c8\ub2e4.", "paragraph_sentence": "\ud604\uc7ac \uc2dc\uc7a5\uc740 \ubbfc\uc8fc\ub2f9 \uc18c\uc18d\uc758 \ube4c \ub354\ube14\ub77c\uc9c0\uc624\uc774\ub2e4. \uc804 \uc2dc\uc7a5\uc774\uc5c8\ub358 \ub9c8\uc774\ud074 \ube14\ub8f8\ubc84\uadf8 \ub294 \ubb34\uc18c\uc18d\uc774\uc5c8\uc73c\ub098 \uacf5\ud654\ub2f9\uacfc \ub274\uc695 \ub3c5\ub9bd\ub2f9\uc758 \uc9c0\uc9c0\ub97c \ubc1b\uace0, 2001\ub144, 2005\ub144, 2009\ub144\uc5d0 3\ubc88 \ub2f9\uc120\ub418\uc5c8\ub2e4. \ube14\ub8f8\ubc84\uadf8\ub294 \ub274\uc695\uc758 \uad50\uc721 \uc2dc\uc2a4\ud15c\uc758 \uad8c\ud55c\uc744 \uad6d\uac00\uc5d0\uc11c \uc774\uad00\uc2dc\ud0a8 \uac83, \ub3c4\uc2dc \uacc4\ud68d, \uacbd\uc81c \uc9c4\ud765, \uc7ac\uc815 \uac74\uc804\ud654, \uacf5\uc911 \ubcf4\uac74 \uc815\ucc45 \ub4f1\uc73c\ub85c \uc720\uba85\ud558\ub2e4. 2\uae30 \ub54c\uc5d0\ub294 \ud559\uad50 \uac1c\ud601, \ube48\uace4 \uac10\uc18c, \uc5c4\uaca9\ud55c \ucd1d\uae30 \uaddc\uc81c \ub4f1\uc744 \ud589\uc815 \uc911\uc2ec \uacfc\uc81c\uc5d0 \ub450\uc5c8\ub2e4. \ubcf4\uc2a4\ud134 \uc2dc\uc7a5 \ud1a0\uba38\uc2a4 \uba54\ub2c8\ub178\ud558\uace0 2006\ub144, \ubd88\ubc95 \ucd1d\uae30\ub97c \ub3c4\uc2dc\uc5d0\uc11c \uc77c\uc18c\ud568\uc73c\ub85c\uc368 \uacf5\uacf5\uc758 \uc548\uc804\uc744 \ub3c4\ubaa8\ud55c\ub2e4\ub294 \ubaa9\ud45c\ub97c \ub0b4\uac78\uace0, \ubd88\ubc95 \ucd1d\uae30\uc640 \uc2f8\uc6b0\ub294 \uc2dc\uc7a5 \ud611\ud68c\ub77c\ub294 \ub2e8\uccb4\ub97c \uacb0\uc131\ud558\uc600\ub2e4. 1924\ub144 \uc774\ud6c4 \ub274\uc695 \ub0b4\uc5d0\uc11c\ub294 \uad6d\uac00 \uc218\uc900\uc758 \uc120\uac70\uc640 \ub300\ud1b5\ub839 \uc120\uac70\uc5d0\uc11c \uacf5\ud654\ub2f9\uc774 \uc774\uae34 \uc801\uc774 \uc5c6\ub2e4. \uc120\uac70 \uacf5\uc57d\uc758 \ud575\uc2ec \ub17c\uc810\uc740 \uc8fc\ud0dd \uac00\uaca9\uc758 \uc778\ud558, \uad50\uc721, \uacbd\uc81c \uc9c4\ud765 \ub4f1\uc774\uba70, \ub178\ub3d9 \uc815\ucc45\uc774 \uc911\uc694\uc131\uc744 \uac00\uc9c0\uace0 \uc788\ub2e4.", "paragraph_answer": "\ud604\uc7ac \uc2dc\uc7a5\uc740 \ubbfc\uc8fc\ub2f9 \uc18c\uc18d\uc758 \ube4c \ub354\ube14\ub77c\uc9c0\uc624\uc774\ub2e4. \uc804 \uc2dc\uc7a5\uc774\uc5c8\ub358 \ub9c8\uc774\ud074 \ube14\ub8f8\ubc84\uadf8 \ub294 \ubb34\uc18c\uc18d\uc774\uc5c8\uc73c\ub098 \uacf5\ud654\ub2f9\uacfc \ub274\uc695 \ub3c5\ub9bd\ub2f9\uc758 \uc9c0\uc9c0\ub97c \ubc1b\uace0, 2001\ub144, 2005\ub144, 2009\ub144\uc5d0 3\ubc88 \ub2f9\uc120\ub418\uc5c8\ub2e4. \ube14\ub8f8\ubc84\uadf8\ub294 \ub274\uc695\uc758 \uad50\uc721 \uc2dc\uc2a4\ud15c\uc758 \uad8c\ud55c\uc744 \uad6d\uac00\uc5d0\uc11c \uc774\uad00\uc2dc\ud0a8 \uac83, \ub3c4\uc2dc \uacc4\ud68d, \uacbd\uc81c \uc9c4\ud765, \uc7ac\uc815 \uac74\uc804\ud654, \uacf5\uc911 \ubcf4\uac74 \uc815\ucc45 \ub4f1\uc73c\ub85c \uc720\uba85\ud558\ub2e4. 2\uae30 \ub54c\uc5d0\ub294 \ud559\uad50 \uac1c\ud601, \ube48\uace4 \uac10\uc18c, \uc5c4\uaca9\ud55c \ucd1d\uae30 \uaddc\uc81c \ub4f1\uc744 \ud589\uc815 \uc911\uc2ec \uacfc\uc81c\uc5d0 \ub450\uc5c8\ub2e4. \ubcf4\uc2a4\ud134 \uc2dc\uc7a5 \ud1a0\uba38\uc2a4 \uba54\ub2c8\ub178\ud558\uace0 2006\ub144, \ubd88\ubc95 \ucd1d\uae30\ub97c \ub3c4\uc2dc\uc5d0\uc11c \uc77c\uc18c\ud568\uc73c\ub85c\uc368 \uacf5\uacf5\uc758 \uc548\uc804\uc744 \ub3c4\ubaa8\ud55c\ub2e4\ub294 \ubaa9\ud45c\ub97c \ub0b4\uac78\uace0, \ubd88\ubc95 \ucd1d\uae30\uc640 \uc2f8\uc6b0\ub294 \uc2dc\uc7a5 \ud611\ud68c\ub77c\ub294 \ub2e8\uccb4\ub97c \uacb0\uc131\ud558\uc600\ub2e4. 1924\ub144 \uc774\ud6c4 \ub274\uc695 \ub0b4\uc5d0\uc11c\ub294 \uad6d\uac00 \uc218\uc900\uc758 \uc120\uac70\uc640 \ub300\ud1b5\ub839 \uc120\uac70\uc5d0\uc11c \uacf5\ud654\ub2f9\uc774 \uc774\uae34 \uc801\uc774 \uc5c6\ub2e4. \uc120\uac70 \uacf5\uc57d\uc758 \ud575\uc2ec \ub17c\uc810\uc740 \uc8fc\ud0dd \uac00\uaca9\uc758 \uc778\ud558, \uad50\uc721, \uacbd\uc81c \uc9c4\ud765 \ub4f1\uc774\uba70, \ub178\ub3d9 \uc815\ucc45\uc774 \uc911\uc694\uc131\uc744 \uac00\uc9c0\uace0 \uc788\ub2e4.", "sentence_answer": "\uc804 \uc2dc\uc7a5\uc774\uc5c8\ub358 \ub9c8\uc774\ud074 \ube14\ub8f8\ubc84\uadf8 \ub294 \ubb34\uc18c\uc18d\uc774\uc5c8\uc73c\ub098 \uacf5\ud654\ub2f9\uacfc \ub274\uc695 \ub3c5\ub9bd\ub2f9\uc758 \uc9c0\uc9c0\ub97c \ubc1b\uace0, 2001\ub144, 2005\ub144, 2009\ub144\uc5d0 3\ubc88 \ub2f9\uc120\ub418\uc5c8\ub2e4.", "paragraph_id": "6546073-13-1", "paragraph_question": "question: \ub274\uc695\uc758 \uc804 \uc2dc\uc7a5\uc73c\ub85c 3\ubc88 \ub2f9\uc120 \ub418\uc5c8\uace0, 2006\ub144\uc5d0 \ubcf4\uc2a4\ud134 \uc2dc\uc7a5\uacfc \ubd88\ubc95 \ucd1d\uae30\uc640 \uc2f8\uc6b0\ub294 \uc2dc\uc7a5 \ud611\ud68c\ub77c\ub294 \ub2e8\uccb4\ub97c \uacb0\uc131\ud55c \uc2dc\uc7a5\uc740?, context: \ud604\uc7ac \uc2dc\uc7a5\uc740 \ubbfc\uc8fc\ub2f9 \uc18c\uc18d\uc758 \ube4c \ub354\ube14\ub77c\uc9c0\uc624\uc774\ub2e4. \uc804 \uc2dc\uc7a5\uc774\uc5c8\ub358 \ub9c8\uc774\ud074 \ube14\ub8f8\ubc84\uadf8\ub294 \ubb34\uc18c\uc18d\uc774\uc5c8\uc73c\ub098 \uacf5\ud654\ub2f9\uacfc \ub274\uc695 \ub3c5\ub9bd\ub2f9\uc758 \uc9c0\uc9c0\ub97c \ubc1b\uace0, 2001\ub144, 2005\ub144, 2009\ub144\uc5d0 3\ubc88 \ub2f9\uc120\ub418\uc5c8\ub2e4. \ube14\ub8f8\ubc84\uadf8\ub294 \ub274\uc695\uc758 \uad50\uc721 \uc2dc\uc2a4\ud15c\uc758 \uad8c\ud55c\uc744 \uad6d\uac00\uc5d0\uc11c \uc774\uad00\uc2dc\ud0a8 \uac83, \ub3c4\uc2dc \uacc4\ud68d, \uacbd\uc81c \uc9c4\ud765, \uc7ac\uc815 \uac74\uc804\ud654, \uacf5\uc911 \ubcf4\uac74 \uc815\ucc45 \ub4f1\uc73c\ub85c \uc720\uba85\ud558\ub2e4. 2\uae30 \ub54c\uc5d0\ub294 \ud559\uad50 \uac1c\ud601, \ube48\uace4 \uac10\uc18c, \uc5c4\uaca9\ud55c \ucd1d\uae30 \uaddc\uc81c \ub4f1\uc744 \ud589\uc815 \uc911\uc2ec \uacfc\uc81c\uc5d0 \ub450\uc5c8\ub2e4. \ubcf4\uc2a4\ud134 \uc2dc\uc7a5 \ud1a0\uba38\uc2a4 \uba54\ub2c8\ub178\ud558\uace0 2006\ub144, \ubd88\ubc95 \ucd1d\uae30\ub97c \ub3c4\uc2dc\uc5d0\uc11c \uc77c\uc18c\ud568\uc73c\ub85c\uc368 \uacf5\uacf5\uc758 \uc548\uc804\uc744 \ub3c4\ubaa8\ud55c\ub2e4\ub294 \ubaa9\ud45c\ub97c \ub0b4\uac78\uace0, \ubd88\ubc95 \ucd1d\uae30\uc640 \uc2f8\uc6b0\ub294 \uc2dc\uc7a5 \ud611\ud68c\ub77c\ub294 \ub2e8\uccb4\ub97c \uacb0\uc131\ud558\uc600\ub2e4. 1924\ub144 \uc774\ud6c4 \ub274\uc695 \ub0b4\uc5d0\uc11c\ub294 \uad6d\uac00 \uc218\uc900\uc758 \uc120\uac70\uc640 \ub300\ud1b5\ub839 \uc120\uac70\uc5d0\uc11c \uacf5\ud654\ub2f9\uc774 \uc774\uae34 \uc801\uc774 \uc5c6\ub2e4. \uc120\uac70 \uacf5\uc57d\uc758 \ud575\uc2ec \ub17c\uc810\uc740 \uc8fc\ud0dd \uac00\uaca9\uc758 \uc778\ud558, \uad50\uc721, \uacbd\uc81c \uc9c4\ud765 \ub4f1\uc774\uba70, \ub178\ub3d9 \uc815\ucc45\uc774 \uc911\uc694\uc131\uc744 \uac00\uc9c0\uace0 \uc788\ub2e4."} {"question": "\ubb38\uc7ac\uc778\uc740 \uacf5\uc57d\uc73c\ub85c \uad6d\ubbfc\ub4e4\uc758 \ubcd1\uc6d0\ube44 \ubd80\ub2f4\uae08 \uc0bc\ud56d\uc120\uc744 \uc5bc\ub9c8\ub85c \ud558\uae30\ub85c \ud558\uc600\ub098?", "paragraph": "\ucc38\uc5ec\uc5f0\ub300\ub294 \uc774\uc5d0 \ub300\ud574 \"OECD \ud68c\uc6d0\uad6d\uc758 \uac74\uac15\ubcf4\ud5d8 \ubcf4\uc7a5\ube44\uc728\uc774 \ud3c9\uade0 81%\uc778 \uac83\uc5d0 \uacac\uc918 \ubcf4\uc7a5\uc728 \ubaa9\ud45c 70%\ub294 \uc801\uc815\ud55c \uc218\uce58\ub85c \ubcf4\uae30 \uc5b4\ub835\ub2e4\"\uba70 \"2012\ub144 \ub300\ud1b5\ub839 \uc120\uac70\uc5d0\uc11c \ubb38\uc7ac\uc778 \ud6c4\ubcf4\uc758 \uacf5\uc57d\uc774\uc5c8\ub358 \ubcd1\uc6d0\ube44 \ubcf8\uc778\ubd80\ub2f4\uae08 100\ub9cc \uc6d0 \uc0c1\ud55c\uc81c\ub97c \uc2e4\uc2dc\ud574\uc57c \ud55c\ub2e4\"\uace0 \ubc1d\ud614\ub2e4. \uae40\ud0dc\ub144 \ub354\ubd88\uc5b4\ubbfc\uc8fc\ub2f9 \uc815\ucc45\uc704\uc758\uc7a5\uc740 \"\ud575\uc2ec\uc740 \uac74\uac15\ubcf4\ud5d8 \ud558\ub098\ub9cc \uc788\uc73c\uba74 \uc544\ud508\ub370\ub3c4 \ub3c8\uc774 \uc5c6\uc5b4\uc11c \uce58\ub8cc\ub97c \ubabb \ubc1b\ub294 \uc77c\uc740 \ub354 \uc774\uc0c1 \uc5c6\ub3c4\ub85d \ud558\uaca0\ub2e4\ub294 \uac83\"\uc774\ub77c\uba70 \"\uc758\ub8cc\ube44 \uc99d\uac00\uc758 \uc8fc\ub41c \uc6d0\uc778\uc774\uc790 \ubbfc\uac04 \uc2e4\uc190\ubcf4\ud5d8 \uac00\uc785\uc758 \uc8fc\uc694\uc6d0\uc778\uc778 \ube44\uae09\uc5ec \ubb38\uc81c\ub97c \ubc14\ub85c \uc7a1\uc744 \uac83\"\uc774\ub77c\uace0 \ubc1d\ud614\ub2e4. \ub610\ud55c \"\ub178\uc778\u00b7\uc5ec\uc131\u00b7\uc544\ub3d9\u00b7\uc7a5\uc560\uc778 \ub4f1 \ucde8\uc57d\uacc4\uce35\uc5d0 \ub300\ud55c \uc9c0\uc6d0\uc744 \ub298\ub9ac\uace0, \ubcf8\uc778\ubd80\ub2f4 \uc0c1\ud55c\uc81c\uc640 \uc7ac\ub09c\uc801 \uc758\ub8cc\ube44 \uc9c0\uc6d0\uc81c\ub3c4\ub97c \uac15\ud654\ud574 \uc800\uc18c\ub4dd\uce35\uc758 \uc758\ub8cc\ube44 \ubd80\ub2f4\uc744 \uc904\uc774\uaca0\ub2e4\"\uace0 \ub367\ubd99\uc600\ub2e4. \ubc18\uba74, \uae40\uad11\ub9bc \uc790\uc720\ud55c\uad6d\ub2f9 \uc815\ucc45\uc704\uc758\uc7a5\uc740 \"\uc7ac\uc6d0 \ub300\ucc45\uc774 \ub450\ub8e8\ubb49\uc2e4\ud574 5\ub144 \ub4a4\uac00 \ubcf4\uc774\uc9c0 \uc54a\ub294\ub2e4\"\uba70 \"13\ud398\uc774\uc9c0\uc758 \uac74\ubcf4 \ub300\ucc45 \ubc1c\ud45c\ubb38\uc5d0 \uc9c0\uc6d0 \ub0b4\uc6a9\uc740 \uae68\uc54c\uac19\uc774 \ub9ce\uc740\ub370 \uc7ac\uc6d0\ub300\ucc45\uc740 3\ubd84\uc758 1 \ubfd0\"\uc774\ub77c\uace0 \ube44\ud310\ud588\ub2e4. \uae40 \uc758\uc7a5\uc740 \"\ubaa8\ub4e0 \ubcd1\uc744 \uc815\ubd80\uc758 \uc758\ub8cc\ubcf4\ud5d8\uc73c\ub85c \uae09\uc5ec\ud654\ud558\uaca0\ub2e4\ub294 \uac83\uc740 \ucc2c\uc131\uc774\uc9c0\ub9cc, \uc758\uc0ac\ub4e4\uc774 \uc81c\ub300\ub85c \ub41c \uc218\uac00\ub97c \ubc1b\uc744 \uc218 \uc788\ub3c4\ub85d \ud574\uc8fc\uc9c0 \uc54a\uace0 \ud1b5\uc81c\ud558\uba74 3\ub9cc \uac1c\uc758 \ubcd1\uc6d0 \uc911 3\ubd84\uc758 1\uc740 5\ub144 \ub4a4 \ubb38\uc744 \ub2eb\uac8c \ub420 \uac83\"\uc774\ub77c\uace0 \ub367\ubd99\uc600\ub2e4. \uc774\uc5d0 \ubb38\uc7ac\uc778\uc740 10\uc77c \uc218\uc11d\u00b7\ubcf4\uc88c\uad00 \ud68c\uc758\ub97c \uc8fc\uc7ac\ud558\uba74\uc11c \"\uc0c8 \uc815\ubd80\uc758 \ubcf5\uc9c0 \ud655\ub300 \uc815\ucc45\uc5d0 \ub300\ud574 \uc138\uae08 \ud3ed\ud0c4\uc774\ub098 \uac74\ubcf4\ub8cc \ud3ed\ud0c4 \ub610\ub294 \ub9c9\ub300\ud55c \uc7ac\uc815\uc801\uc790 \uc5c6\uc774 \uac00\ub2a5\ud560 \uac83\uc778\uac00 \uad81\uae08\ud574\ud558\ub294 \uad6d\ubbfc\uc774 \ub9ce\ub2e4\"\uba70 \"\uae30\ud68d\uc7ac\uc815\ubd80\uc640 \ucda9\ubd84\ud788 \ud611\uc758\ud574 \uc7ac\uc6d0\ub300\ucc45\uc744 \uaf3c\uaf3c\ud788 \uac80\ud1a0\ud588\uace0, \uc62c \ud558\ubc18\uae30\ubd80\ud130 2022\ub144\uae4c\uc9c0 \ub2e8\uacc4\uc801\uc73c\ub85c \uc2dc\ud589\ud558\ub3c4\ub85d \uc124\uacc4\ud574 \ud604\uc2e4\uc801\uc73c\ub85c \uac74\uc804 \uc7ac\uc815\uc744 \uc720\uc9c0\ud558\uba74\uc11c \uac10\ub2ec\ud560 \uc218 \uc788\ub294 \ucd5c\uc120\uc744 \uc120\ud0dd\ud55c \uac83\"\uc774\ub77c\uace0 \uc9c1\uc811 \ubc18\ubc15\uc5d0 \ub098\uc130\ub2e4.", "answer": "100\ub9cc \uc6d0", "sentence": "\ucc38\uc5ec\uc5f0\ub300\ub294 \uc774\uc5d0 \ub300\ud574 \"OECD \ud68c\uc6d0\uad6d\uc758 \uac74\uac15\ubcf4\ud5d8 \ubcf4\uc7a5\ube44\uc728\uc774 \ud3c9\uade0 81%\uc778 \uac83\uc5d0 \uacac\uc918 \ubcf4\uc7a5\uc728 \ubaa9\ud45c 70%\ub294 \uc801\uc815\ud55c \uc218\uce58\ub85c \ubcf4\uae30 \uc5b4\ub835\ub2e4\"\uba70 \"2012\ub144 \ub300\ud1b5\ub839 \uc120\uac70\uc5d0\uc11c \ubb38\uc7ac\uc778 \ud6c4\ubcf4\uc758 \uacf5\uc57d\uc774\uc5c8\ub358 \ubcd1\uc6d0\ube44 \ubcf8\uc778\ubd80\ub2f4\uae08 100\ub9cc \uc6d0 \uc0c1\ud55c\uc81c\ub97c \uc2e4\uc2dc\ud574\uc57c \ud55c\ub2e4\"\uace0 \ubc1d\ud614\ub2e4.", "paragraph_sentence": " \ucc38\uc5ec\uc5f0\ub300\ub294 \uc774\uc5d0 \ub300\ud574 \"OECD \ud68c\uc6d0\uad6d\uc758 \uac74\uac15\ubcf4\ud5d8 \ubcf4\uc7a5\ube44\uc728\uc774 \ud3c9\uade0 81%\uc778 \uac83\uc5d0 \uacac\uc918 \ubcf4\uc7a5\uc728 \ubaa9\ud45c 70%\ub294 \uc801\uc815\ud55c \uc218\uce58\ub85c \ubcf4\uae30 \uc5b4\ub835\ub2e4\"\uba70 \"2012\ub144 \ub300\ud1b5\ub839 \uc120\uac70\uc5d0\uc11c \ubb38\uc7ac\uc778 \ud6c4\ubcf4\uc758 \uacf5\uc57d\uc774\uc5c8\ub358 \ubcd1\uc6d0\ube44 \ubcf8\uc778\ubd80\ub2f4\uae08 100\ub9cc \uc6d0 \uc0c1\ud55c\uc81c\ub97c \uc2e4\uc2dc\ud574\uc57c \ud55c\ub2e4\"\uace0 \ubc1d\ud614\ub2e4. \uae40\ud0dc\ub144 \ub354\ubd88\uc5b4\ubbfc\uc8fc\ub2f9 \uc815\ucc45\uc704\uc758\uc7a5\uc740 \"\ud575\uc2ec\uc740 \uac74\uac15\ubcf4\ud5d8 \ud558\ub098\ub9cc \uc788\uc73c\uba74 \uc544\ud508\ub370\ub3c4 \ub3c8\uc774 \uc5c6\uc5b4\uc11c \uce58\ub8cc\ub97c \ubabb \ubc1b\ub294 \uc77c\uc740 \ub354 \uc774\uc0c1 \uc5c6\ub3c4\ub85d \ud558\uaca0\ub2e4\ub294 \uac83\"\uc774\ub77c\uba70 \"\uc758\ub8cc\ube44 \uc99d\uac00\uc758 \uc8fc\ub41c \uc6d0\uc778\uc774\uc790 \ubbfc\uac04 \uc2e4\uc190\ubcf4\ud5d8 \uac00\uc785\uc758 \uc8fc\uc694\uc6d0\uc778\uc778 \ube44\uae09\uc5ec \ubb38\uc81c\ub97c \ubc14\ub85c \uc7a1\uc744 \uac83\"\uc774\ub77c\uace0 \ubc1d\ud614\ub2e4. \ub610\ud55c \"\ub178\uc778\u00b7\uc5ec\uc131\u00b7\uc544\ub3d9\u00b7\uc7a5\uc560\uc778 \ub4f1 \ucde8\uc57d\uacc4\uce35\uc5d0 \ub300\ud55c \uc9c0\uc6d0\uc744 \ub298\ub9ac\uace0, \ubcf8\uc778\ubd80\ub2f4 \uc0c1\ud55c\uc81c\uc640 \uc7ac\ub09c\uc801 \uc758\ub8cc\ube44 \uc9c0\uc6d0\uc81c\ub3c4\ub97c \uac15\ud654\ud574 \uc800\uc18c\ub4dd\uce35\uc758 \uc758\ub8cc\ube44 \ubd80\ub2f4\uc744 \uc904\uc774\uaca0\ub2e4\"\uace0 \ub367\ubd99\uc600\ub2e4. \ubc18\uba74, \uae40\uad11\ub9bc \uc790\uc720\ud55c\uad6d\ub2f9 \uc815\ucc45\uc704\uc758\uc7a5\uc740 \"\uc7ac\uc6d0 \ub300\ucc45\uc774 \ub450\ub8e8\ubb49\uc2e4\ud574 5\ub144 \ub4a4\uac00 \ubcf4\uc774\uc9c0 \uc54a\ub294\ub2e4\"\uba70 \"13\ud398\uc774\uc9c0\uc758 \uac74\ubcf4 \ub300\ucc45 \ubc1c\ud45c\ubb38\uc5d0 \uc9c0\uc6d0 \ub0b4\uc6a9\uc740 \uae68\uc54c\uac19\uc774 \ub9ce\uc740\ub370 \uc7ac\uc6d0\ub300\ucc45\uc740 3\ubd84\uc758 1 \ubfd0\"\uc774\ub77c\uace0 \ube44\ud310\ud588\ub2e4. \uae40 \uc758\uc7a5\uc740 \"\ubaa8\ub4e0 \ubcd1\uc744 \uc815\ubd80\uc758 \uc758\ub8cc\ubcf4\ud5d8\uc73c\ub85c \uae09\uc5ec\ud654\ud558\uaca0\ub2e4\ub294 \uac83\uc740 \ucc2c\uc131\uc774\uc9c0\ub9cc, \uc758\uc0ac\ub4e4\uc774 \uc81c\ub300\ub85c \ub41c \uc218\uac00\ub97c \ubc1b\uc744 \uc218 \uc788\ub3c4\ub85d \ud574\uc8fc\uc9c0 \uc54a\uace0 \ud1b5\uc81c\ud558\uba74 3\ub9cc \uac1c\uc758 \ubcd1\uc6d0 \uc911 3\ubd84\uc758 1\uc740 5\ub144 \ub4a4 \ubb38\uc744 \ub2eb\uac8c \ub420 \uac83\"\uc774\ub77c\uace0 \ub367\ubd99\uc600\ub2e4. \uc774\uc5d0 \ubb38\uc7ac\uc778\uc740 10\uc77c \uc218\uc11d\u00b7\ubcf4\uc88c\uad00 \ud68c\uc758\ub97c \uc8fc\uc7ac\ud558\uba74\uc11c \"\uc0c8 \uc815\ubd80\uc758 \ubcf5\uc9c0 \ud655\ub300 \uc815\ucc45\uc5d0 \ub300\ud574 \uc138\uae08 \ud3ed\ud0c4\uc774\ub098 \uac74\ubcf4\ub8cc \ud3ed\ud0c4 \ub610\ub294 \ub9c9\ub300\ud55c \uc7ac\uc815\uc801\uc790 \uc5c6\uc774 \uac00\ub2a5\ud560 \uac83\uc778\uac00 \uad81\uae08\ud574\ud558\ub294 \uad6d\ubbfc\uc774 \ub9ce\ub2e4\"\uba70 \"\uae30\ud68d\uc7ac\uc815\ubd80\uc640 \ucda9\ubd84\ud788 \ud611\uc758\ud574 \uc7ac\uc6d0\ub300\ucc45\uc744 \uaf3c\uaf3c\ud788 \uac80\ud1a0\ud588\uace0, \uc62c \ud558\ubc18\uae30\ubd80\ud130 2022\ub144\uae4c\uc9c0 \ub2e8\uacc4\uc801\uc73c\ub85c \uc2dc\ud589\ud558\ub3c4\ub85d \uc124\uacc4\ud574 \ud604\uc2e4\uc801\uc73c\ub85c \uac74\uc804 \uc7ac\uc815\uc744 \uc720\uc9c0\ud558\uba74\uc11c \uac10\ub2ec\ud560 \uc218 \uc788\ub294 \ucd5c\uc120\uc744 \uc120\ud0dd\ud55c \uac83\"\uc774\ub77c\uace0 \uc9c1\uc811 \ubc18\ubc15\uc5d0 \ub098\uc130\ub2e4.", "paragraph_answer": "\ucc38\uc5ec\uc5f0\ub300\ub294 \uc774\uc5d0 \ub300\ud574 \"OECD \ud68c\uc6d0\uad6d\uc758 \uac74\uac15\ubcf4\ud5d8 \ubcf4\uc7a5\ube44\uc728\uc774 \ud3c9\uade0 81%\uc778 \uac83\uc5d0 \uacac\uc918 \ubcf4\uc7a5\uc728 \ubaa9\ud45c 70%\ub294 \uc801\uc815\ud55c \uc218\uce58\ub85c \ubcf4\uae30 \uc5b4\ub835\ub2e4\"\uba70 \"2012\ub144 \ub300\ud1b5\ub839 \uc120\uac70\uc5d0\uc11c \ubb38\uc7ac\uc778 \ud6c4\ubcf4\uc758 \uacf5\uc57d\uc774\uc5c8\ub358 \ubcd1\uc6d0\ube44 \ubcf8\uc778\ubd80\ub2f4\uae08 100\ub9cc \uc6d0 \uc0c1\ud55c\uc81c\ub97c \uc2e4\uc2dc\ud574\uc57c \ud55c\ub2e4\"\uace0 \ubc1d\ud614\ub2e4. \uae40\ud0dc\ub144 \ub354\ubd88\uc5b4\ubbfc\uc8fc\ub2f9 \uc815\ucc45\uc704\uc758\uc7a5\uc740 \"\ud575\uc2ec\uc740 \uac74\uac15\ubcf4\ud5d8 \ud558\ub098\ub9cc \uc788\uc73c\uba74 \uc544\ud508\ub370\ub3c4 \ub3c8\uc774 \uc5c6\uc5b4\uc11c \uce58\ub8cc\ub97c \ubabb \ubc1b\ub294 \uc77c\uc740 \ub354 \uc774\uc0c1 \uc5c6\ub3c4\ub85d \ud558\uaca0\ub2e4\ub294 \uac83\"\uc774\ub77c\uba70 \"\uc758\ub8cc\ube44 \uc99d\uac00\uc758 \uc8fc\ub41c \uc6d0\uc778\uc774\uc790 \ubbfc\uac04 \uc2e4\uc190\ubcf4\ud5d8 \uac00\uc785\uc758 \uc8fc\uc694\uc6d0\uc778\uc778 \ube44\uae09\uc5ec \ubb38\uc81c\ub97c \ubc14\ub85c \uc7a1\uc744 \uac83\"\uc774\ub77c\uace0 \ubc1d\ud614\ub2e4. \ub610\ud55c \"\ub178\uc778\u00b7\uc5ec\uc131\u00b7\uc544\ub3d9\u00b7\uc7a5\uc560\uc778 \ub4f1 \ucde8\uc57d\uacc4\uce35\uc5d0 \ub300\ud55c \uc9c0\uc6d0\uc744 \ub298\ub9ac\uace0, \ubcf8\uc778\ubd80\ub2f4 \uc0c1\ud55c\uc81c\uc640 \uc7ac\ub09c\uc801 \uc758\ub8cc\ube44 \uc9c0\uc6d0\uc81c\ub3c4\ub97c \uac15\ud654\ud574 \uc800\uc18c\ub4dd\uce35\uc758 \uc758\ub8cc\ube44 \ubd80\ub2f4\uc744 \uc904\uc774\uaca0\ub2e4\"\uace0 \ub367\ubd99\uc600\ub2e4. \ubc18\uba74, \uae40\uad11\ub9bc \uc790\uc720\ud55c\uad6d\ub2f9 \uc815\ucc45\uc704\uc758\uc7a5\uc740 \"\uc7ac\uc6d0 \ub300\ucc45\uc774 \ub450\ub8e8\ubb49\uc2e4\ud574 5\ub144 \ub4a4\uac00 \ubcf4\uc774\uc9c0 \uc54a\ub294\ub2e4\"\uba70 \"13\ud398\uc774\uc9c0\uc758 \uac74\ubcf4 \ub300\ucc45 \ubc1c\ud45c\ubb38\uc5d0 \uc9c0\uc6d0 \ub0b4\uc6a9\uc740 \uae68\uc54c\uac19\uc774 \ub9ce\uc740\ub370 \uc7ac\uc6d0\ub300\ucc45\uc740 3\ubd84\uc758 1 \ubfd0\"\uc774\ub77c\uace0 \ube44\ud310\ud588\ub2e4. \uae40 \uc758\uc7a5\uc740 \"\ubaa8\ub4e0 \ubcd1\uc744 \uc815\ubd80\uc758 \uc758\ub8cc\ubcf4\ud5d8\uc73c\ub85c \uae09\uc5ec\ud654\ud558\uaca0\ub2e4\ub294 \uac83\uc740 \ucc2c\uc131\uc774\uc9c0\ub9cc, \uc758\uc0ac\ub4e4\uc774 \uc81c\ub300\ub85c \ub41c \uc218\uac00\ub97c \ubc1b\uc744 \uc218 \uc788\ub3c4\ub85d \ud574\uc8fc\uc9c0 \uc54a\uace0 \ud1b5\uc81c\ud558\uba74 3\ub9cc \uac1c\uc758 \ubcd1\uc6d0 \uc911 3\ubd84\uc758 1\uc740 5\ub144 \ub4a4 \ubb38\uc744 \ub2eb\uac8c \ub420 \uac83\"\uc774\ub77c\uace0 \ub367\ubd99\uc600\ub2e4. \uc774\uc5d0 \ubb38\uc7ac\uc778\uc740 10\uc77c \uc218\uc11d\u00b7\ubcf4\uc88c\uad00 \ud68c\uc758\ub97c \uc8fc\uc7ac\ud558\uba74\uc11c \"\uc0c8 \uc815\ubd80\uc758 \ubcf5\uc9c0 \ud655\ub300 \uc815\ucc45\uc5d0 \ub300\ud574 \uc138\uae08 \ud3ed\ud0c4\uc774\ub098 \uac74\ubcf4\ub8cc \ud3ed\ud0c4 \ub610\ub294 \ub9c9\ub300\ud55c \uc7ac\uc815\uc801\uc790 \uc5c6\uc774 \uac00\ub2a5\ud560 \uac83\uc778\uac00 \uad81\uae08\ud574\ud558\ub294 \uad6d\ubbfc\uc774 \ub9ce\ub2e4\"\uba70 \"\uae30\ud68d\uc7ac\uc815\ubd80\uc640 \ucda9\ubd84\ud788 \ud611\uc758\ud574 \uc7ac\uc6d0\ub300\ucc45\uc744 \uaf3c\uaf3c\ud788 \uac80\ud1a0\ud588\uace0, \uc62c \ud558\ubc18\uae30\ubd80\ud130 2022\ub144\uae4c\uc9c0 \ub2e8\uacc4\uc801\uc73c\ub85c \uc2dc\ud589\ud558\ub3c4\ub85d \uc124\uacc4\ud574 \ud604\uc2e4\uc801\uc73c\ub85c \uac74\uc804 \uc7ac\uc815\uc744 \uc720\uc9c0\ud558\uba74\uc11c \uac10\ub2ec\ud560 \uc218 \uc788\ub294 \ucd5c\uc120\uc744 \uc120\ud0dd\ud55c \uac83\"\uc774\ub77c\uace0 \uc9c1\uc811 \ubc18\ubc15\uc5d0 \ub098\uc130\ub2e4.", "sentence_answer": "\ucc38\uc5ec\uc5f0\ub300\ub294 \uc774\uc5d0 \ub300\ud574 \"OECD \ud68c\uc6d0\uad6d\uc758 \uac74\uac15\ubcf4\ud5d8 \ubcf4\uc7a5\ube44\uc728\uc774 \ud3c9\uade0 81%\uc778 \uac83\uc5d0 \uacac\uc918 \ubcf4\uc7a5\uc728 \ubaa9\ud45c 70%\ub294 \uc801\uc815\ud55c \uc218\uce58\ub85c \ubcf4\uae30 \uc5b4\ub835\ub2e4\"\uba70 \"2012\ub144 \ub300\ud1b5\ub839 \uc120\uac70\uc5d0\uc11c \ubb38\uc7ac\uc778 \ud6c4\ubcf4\uc758 \uacf5\uc57d\uc774\uc5c8\ub358 \ubcd1\uc6d0\ube44 \ubcf8\uc778\ubd80\ub2f4\uae08 100\ub9cc \uc6d0 \uc0c1\ud55c\uc81c\ub97c \uc2e4\uc2dc\ud574\uc57c \ud55c\ub2e4\"\uace0 \ubc1d\ud614\ub2e4.", "paragraph_id": "6546926-124-1", "paragraph_question": "question: \ubb38\uc7ac\uc778\uc740 \uacf5\uc57d\uc73c\ub85c \uad6d\ubbfc\ub4e4\uc758 \ubcd1\uc6d0\ube44 \ubd80\ub2f4\uae08 \uc0bc\ud56d\uc120\uc744 \uc5bc\ub9c8\ub85c \ud558\uae30\ub85c \ud558\uc600\ub098?, context: \ucc38\uc5ec\uc5f0\ub300\ub294 \uc774\uc5d0 \ub300\ud574 \"OECD \ud68c\uc6d0\uad6d\uc758 \uac74\uac15\ubcf4\ud5d8 \ubcf4\uc7a5\ube44\uc728\uc774 \ud3c9\uade0 81%\uc778 \uac83\uc5d0 \uacac\uc918 \ubcf4\uc7a5\uc728 \ubaa9\ud45c 70%\ub294 \uc801\uc815\ud55c \uc218\uce58\ub85c \ubcf4\uae30 \uc5b4\ub835\ub2e4\"\uba70 \"2012\ub144 \ub300\ud1b5\ub839 \uc120\uac70\uc5d0\uc11c \ubb38\uc7ac\uc778 \ud6c4\ubcf4\uc758 \uacf5\uc57d\uc774\uc5c8\ub358 \ubcd1\uc6d0\ube44 \ubcf8\uc778\ubd80\ub2f4\uae08 100\ub9cc \uc6d0 \uc0c1\ud55c\uc81c\ub97c \uc2e4\uc2dc\ud574\uc57c \ud55c\ub2e4\"\uace0 \ubc1d\ud614\ub2e4. \uae40\ud0dc\ub144 \ub354\ubd88\uc5b4\ubbfc\uc8fc\ub2f9 \uc815\ucc45\uc704\uc758\uc7a5\uc740 \"\ud575\uc2ec\uc740 \uac74\uac15\ubcf4\ud5d8 \ud558\ub098\ub9cc \uc788\uc73c\uba74 \uc544\ud508\ub370\ub3c4 \ub3c8\uc774 \uc5c6\uc5b4\uc11c \uce58\ub8cc\ub97c \ubabb \ubc1b\ub294 \uc77c\uc740 \ub354 \uc774\uc0c1 \uc5c6\ub3c4\ub85d \ud558\uaca0\ub2e4\ub294 \uac83\"\uc774\ub77c\uba70 \"\uc758\ub8cc\ube44 \uc99d\uac00\uc758 \uc8fc\ub41c \uc6d0\uc778\uc774\uc790 \ubbfc\uac04 \uc2e4\uc190\ubcf4\ud5d8 \uac00\uc785\uc758 \uc8fc\uc694\uc6d0\uc778\uc778 \ube44\uae09\uc5ec \ubb38\uc81c\ub97c \ubc14\ub85c \uc7a1\uc744 \uac83\"\uc774\ub77c\uace0 \ubc1d\ud614\ub2e4. \ub610\ud55c \"\ub178\uc778\u00b7\uc5ec\uc131\u00b7\uc544\ub3d9\u00b7\uc7a5\uc560\uc778 \ub4f1 \ucde8\uc57d\uacc4\uce35\uc5d0 \ub300\ud55c \uc9c0\uc6d0\uc744 \ub298\ub9ac\uace0, \ubcf8\uc778\ubd80\ub2f4 \uc0c1\ud55c\uc81c\uc640 \uc7ac\ub09c\uc801 \uc758\ub8cc\ube44 \uc9c0\uc6d0\uc81c\ub3c4\ub97c \uac15\ud654\ud574 \uc800\uc18c\ub4dd\uce35\uc758 \uc758\ub8cc\ube44 \ubd80\ub2f4\uc744 \uc904\uc774\uaca0\ub2e4\"\uace0 \ub367\ubd99\uc600\ub2e4. \ubc18\uba74, \uae40\uad11\ub9bc \uc790\uc720\ud55c\uad6d\ub2f9 \uc815\ucc45\uc704\uc758\uc7a5\uc740 \"\uc7ac\uc6d0 \ub300\ucc45\uc774 \ub450\ub8e8\ubb49\uc2e4\ud574 5\ub144 \ub4a4\uac00 \ubcf4\uc774\uc9c0 \uc54a\ub294\ub2e4\"\uba70 \"13\ud398\uc774\uc9c0\uc758 \uac74\ubcf4 \ub300\ucc45 \ubc1c\ud45c\ubb38\uc5d0 \uc9c0\uc6d0 \ub0b4\uc6a9\uc740 \uae68\uc54c\uac19\uc774 \ub9ce\uc740\ub370 \uc7ac\uc6d0\ub300\ucc45\uc740 3\ubd84\uc758 1 \ubfd0\"\uc774\ub77c\uace0 \ube44\ud310\ud588\ub2e4. \uae40 \uc758\uc7a5\uc740 \"\ubaa8\ub4e0 \ubcd1\uc744 \uc815\ubd80\uc758 \uc758\ub8cc\ubcf4\ud5d8\uc73c\ub85c \uae09\uc5ec\ud654\ud558\uaca0\ub2e4\ub294 \uac83\uc740 \ucc2c\uc131\uc774\uc9c0\ub9cc, \uc758\uc0ac\ub4e4\uc774 \uc81c\ub300\ub85c \ub41c \uc218\uac00\ub97c \ubc1b\uc744 \uc218 \uc788\ub3c4\ub85d \ud574\uc8fc\uc9c0 \uc54a\uace0 \ud1b5\uc81c\ud558\uba74 3\ub9cc \uac1c\uc758 \ubcd1\uc6d0 \uc911 3\ubd84\uc758 1\uc740 5\ub144 \ub4a4 \ubb38\uc744 \ub2eb\uac8c \ub420 \uac83\"\uc774\ub77c\uace0 \ub367\ubd99\uc600\ub2e4. \uc774\uc5d0 \ubb38\uc7ac\uc778\uc740 10\uc77c \uc218\uc11d\u00b7\ubcf4\uc88c\uad00 \ud68c\uc758\ub97c \uc8fc\uc7ac\ud558\uba74\uc11c \"\uc0c8 \uc815\ubd80\uc758 \ubcf5\uc9c0 \ud655\ub300 \uc815\ucc45\uc5d0 \ub300\ud574 \uc138\uae08 \ud3ed\ud0c4\uc774\ub098 \uac74\ubcf4\ub8cc \ud3ed\ud0c4 \ub610\ub294 \ub9c9\ub300\ud55c \uc7ac\uc815\uc801\uc790 \uc5c6\uc774 \uac00\ub2a5\ud560 \uac83\uc778\uac00 \uad81\uae08\ud574\ud558\ub294 \uad6d\ubbfc\uc774 \ub9ce\ub2e4\"\uba70 \"\uae30\ud68d\uc7ac\uc815\ubd80\uc640 \ucda9\ubd84\ud788 \ud611\uc758\ud574 \uc7ac\uc6d0\ub300\ucc45\uc744 \uaf3c\uaf3c\ud788 \uac80\ud1a0\ud588\uace0, \uc62c \ud558\ubc18\uae30\ubd80\ud130 2022\ub144\uae4c\uc9c0 \ub2e8\uacc4\uc801\uc73c\ub85c \uc2dc\ud589\ud558\ub3c4\ub85d \uc124\uacc4\ud574 \ud604\uc2e4\uc801\uc73c\ub85c \uac74\uc804 \uc7ac\uc815\uc744 \uc720\uc9c0\ud558\uba74\uc11c \uac10\ub2ec\ud560 \uc218 \uc788\ub294 \ucd5c\uc120\uc744 \uc120\ud0dd\ud55c \uac83\"\uc774\ub77c\uace0 \uc9c1\uc811 \ubc18\ubc15\uc5d0 \ub098\uc130\ub2e4."} {"question": "\ub4dc\ub77c\ub9c8\ud48d\uc758 \ub300\ud654\ub85c \uc8fc\uc694\uc778\ubb3c\uc774 \uc591\uce58\uae30\uc640 \ucc9c\uc0ac\uc774\uba70 \uc911\uc138 \uc131\ud0c4\uc81c\uadf9\uc758 \ubcc0\ud615\uc774\ub77c \ud560 \uc218 \uc788\ub294 \uac83\uc740?", "paragraph": "\uc5d0\uc2a4\ud30c\ub0d0 \uc5f0\uadf9\uc758 \uc544\ubc84\uc9c0\ub85c \ubd88\ub9ac\ub294 \ud658 \ub378 \uc5d4\uc2dc\ub098(1469?-1529?)\uc758 <\uc5d0\uae00\ub85c\uac00\uc2a4(\u7267\u6b4c)>\ub294 \ub4dc\ub77c\ub9c8\ud48d\uc758 \ub300\ud654\ub85c \uc591\uce58\uae30\uc640 \ucc9c\uc0ac\uac00 \uc8fc\uc694\uc778\ubb3c\uc774\uba70 \uc911\uc138 \uc131\ud0c4\uc81c\uadf9\uc758 \ubcc0\ud615\uc774\ub77c \ud558\uaca0\ub2e4. \uadf8 \ud6c4 \ubbf8\ubaa8\uc2a4(momos), \uc989 \uac00\uba74\uc744 \uc4f4 \ubc1c\ub808\uc640 \ud30c\uc2a4\ud1a0\ub77c\ub808\uc2a4(\ubaa9\uac00\uadf9)\uac00 \ud589\ud574\uc84c\uc73c\ub098 \ubc30\uc6b0\ub294 \ube48\uc57d\ud55c \uc7a5\ube44\ub85c \ub3c4\uc2dc\ub098 \uc9c0\ubc29\uc744 \uc21c\ud68c\ud588\ub2e4. \ubc14\ub974\ud1a8\ub85c\uba54 \ub370 \ud1a0\ub974\ub808\uc2a4 \ub098\uc544\ub85c(1521\ub144 \uc0ac\ub9dd)\uc758 \ud76c\uace1\uc9d1 <\ud504\ub85c\ud314\ub77c\ub514\uc544>(1517)\uc758 \uc11c\ubb38\uc740 \ud638\ub77c\ud2f0\uc6b0\uc2a4\ub97c \ub530\ub978 5\ub9c9 \ud615\uc2dd \ub4f1\uc758 \uc5f0\uadf9\ub860\uc774\ub098, \ud76c\uadf9\uacfc \ube44\uadf9\uc758 \uad6c\ubcc4\uc740 \uc791\ud488\uba74\uc5d0\uc11c \uc804\ud600 \ubb34\uc2dc\ub418\uace0 \uc788\uc5c8\ub2e4. <\ud558\uc778\uc758 \uc2dd\ub2f9> <\uc138\ub77c\ud53c\ub098> <\ub77c \uc774\uba54\ub124\uc544> \ub4f1\uc758 \uc791\ud488\uc774 \uc788\uace0, \ud2b9\ud788 \ub9c8\uc9c0\ub9c9 \uac83\uc740 \ud6c4\ub300\uc758 \uc5d0\uc2a4\ud30c\ub0d0\uadf9\uc5d0\uc11c \ud55c \ud2b9\uc0c9\uc774 \ub41c '\uc678\ud22c\uc640 \uce7c\uc758 \ud76c\uace1'\uc758 \ud6a8\uc2dc\ub77c\uace0 \ud558\uaca0\ub2e4. \uc774\ub7ec\ud55c \uc5f0\uadf9\uc740 \ub300\ub3c4\uc2dc\uc758 \uac74\ubb3c \uc548\ub730\uc5d0\uc11c \uac70\ud589\ub418\uc5c8\uc73c\uba70 \uc0c1\uc5f0 \ub54c\ub294 \uc18c\ub780\ud558\uae30 \uc9dd\uc774 \uc5c6\uc5c8\ub2e4\uace0 \ud55c\ub2e4.", "answer": "\uc5d0\uae00\ub85c\uac00\uc2a4", "sentence": "\uc5f0\uadf9\uc758 \uc544\ubc84\uc9c0\ub85c \ubd88\ub9ac\ub294 \ud658 \ub378 \uc5d4\uc2dc\ub098(1469?-1529?)\uc758 < \uc5d0\uae00\ub85c\uac00\uc2a4 (\u7267\u6b4c)", "paragraph_sentence": "\uc5d0\uc2a4\ud30c\ub0d0 \uc5f0\uadf9\uc758 \uc544\ubc84\uc9c0\ub85c \ubd88\ub9ac\ub294 \ud658 \ub378 \uc5d4\uc2dc\ub098(1469?-1529?)\uc758 < \uc5d0\uae00\ub85c\uac00\uc2a4 (\u7267\u6b4c) >\ub294 \ub4dc\ub77c\ub9c8\ud48d\uc758 \ub300\ud654\ub85c \uc591\uce58\uae30\uc640 \ucc9c\uc0ac\uac00 \uc8fc\uc694\uc778\ubb3c\uc774\uba70 \uc911\uc138 \uc131\ud0c4\uc81c\uadf9\uc758 \ubcc0\ud615\uc774\ub77c \ud558\uaca0\ub2e4. \uadf8 \ud6c4 \ubbf8\ubaa8\uc2a4(momos), \uc989 \uac00\uba74\uc744 \uc4f4 \ubc1c\ub808\uc640 \ud30c\uc2a4\ud1a0\ub77c\ub808\uc2a4(\ubaa9\uac00\uadf9)\uac00 \ud589\ud574\uc84c\uc73c\ub098 \ubc30\uc6b0\ub294 \ube48\uc57d\ud55c \uc7a5\ube44\ub85c \ub3c4\uc2dc\ub098 \uc9c0\ubc29\uc744 \uc21c\ud68c\ud588\ub2e4. \ubc14\ub974\ud1a8\ub85c\uba54 \ub370 \ud1a0\ub974\ub808\uc2a4 \ub098\uc544\ub85c(1521\ub144 \uc0ac\ub9dd)\uc758 \ud76c\uace1\uc9d1 <\ud504\ub85c\ud314\ub77c\ub514\uc544>(1517)\uc758 \uc11c\ubb38\uc740 \ud638\ub77c\ud2f0\uc6b0\uc2a4\ub97c \ub530\ub978 5\ub9c9 \ud615\uc2dd \ub4f1\uc758 \uc5f0\uadf9\ub860\uc774\ub098, \ud76c\uadf9\uacfc \ube44\uadf9\uc758 \uad6c\ubcc4\uc740 \uc791\ud488\uba74\uc5d0\uc11c \uc804\ud600 \ubb34\uc2dc\ub418\uace0 \uc788\uc5c8\ub2e4. <\ud558\uc778\uc758 \uc2dd\ub2f9> <\uc138\ub77c\ud53c\ub098> <\ub77c \uc774\uba54\ub124\uc544> \ub4f1\uc758 \uc791\ud488\uc774 \uc788\uace0, \ud2b9\ud788 \ub9c8\uc9c0\ub9c9 \uac83\uc740 \ud6c4\ub300\uc758 \uc5d0\uc2a4\ud30c\ub0d0\uadf9\uc5d0\uc11c \ud55c \ud2b9\uc0c9\uc774 \ub41c '\uc678\ud22c\uc640 \uce7c\uc758 \ud76c\uace1'\uc758 \ud6a8\uc2dc\ub77c\uace0 \ud558\uaca0\ub2e4. \uc774\ub7ec\ud55c \uc5f0\uadf9\uc740 \ub300\ub3c4\uc2dc\uc758 \uac74\ubb3c \uc548\ub730\uc5d0\uc11c \uac70\ud589\ub418\uc5c8\uc73c\uba70 \uc0c1\uc5f0 \ub54c\ub294 \uc18c\ub780\ud558\uae30 \uc9dd\uc774 \uc5c6\uc5c8\ub2e4\uace0 \ud55c\ub2e4.", "paragraph_answer": "\uc5d0\uc2a4\ud30c\ub0d0 \uc5f0\uadf9\uc758 \uc544\ubc84\uc9c0\ub85c \ubd88\ub9ac\ub294 \ud658 \ub378 \uc5d4\uc2dc\ub098(1469?-1529?)\uc758 < \uc5d0\uae00\ub85c\uac00\uc2a4 (\u7267\u6b4c)>\ub294 \ub4dc\ub77c\ub9c8\ud48d\uc758 \ub300\ud654\ub85c \uc591\uce58\uae30\uc640 \ucc9c\uc0ac\uac00 \uc8fc\uc694\uc778\ubb3c\uc774\uba70 \uc911\uc138 \uc131\ud0c4\uc81c\uadf9\uc758 \ubcc0\ud615\uc774\ub77c \ud558\uaca0\ub2e4. \uadf8 \ud6c4 \ubbf8\ubaa8\uc2a4(momos), \uc989 \uac00\uba74\uc744 \uc4f4 \ubc1c\ub808\uc640 \ud30c\uc2a4\ud1a0\ub77c\ub808\uc2a4(\ubaa9\uac00\uadf9)\uac00 \ud589\ud574\uc84c\uc73c\ub098 \ubc30\uc6b0\ub294 \ube48\uc57d\ud55c \uc7a5\ube44\ub85c \ub3c4\uc2dc\ub098 \uc9c0\ubc29\uc744 \uc21c\ud68c\ud588\ub2e4. \ubc14\ub974\ud1a8\ub85c\uba54 \ub370 \ud1a0\ub974\ub808\uc2a4 \ub098\uc544\ub85c(1521\ub144 \uc0ac\ub9dd)\uc758 \ud76c\uace1\uc9d1 <\ud504\ub85c\ud314\ub77c\ub514\uc544>(1517)\uc758 \uc11c\ubb38\uc740 \ud638\ub77c\ud2f0\uc6b0\uc2a4\ub97c \ub530\ub978 5\ub9c9 \ud615\uc2dd \ub4f1\uc758 \uc5f0\uadf9\ub860\uc774\ub098, \ud76c\uadf9\uacfc \ube44\uadf9\uc758 \uad6c\ubcc4\uc740 \uc791\ud488\uba74\uc5d0\uc11c \uc804\ud600 \ubb34\uc2dc\ub418\uace0 \uc788\uc5c8\ub2e4. <\ud558\uc778\uc758 \uc2dd\ub2f9> <\uc138\ub77c\ud53c\ub098> <\ub77c \uc774\uba54\ub124\uc544> \ub4f1\uc758 \uc791\ud488\uc774 \uc788\uace0, \ud2b9\ud788 \ub9c8\uc9c0\ub9c9 \uac83\uc740 \ud6c4\ub300\uc758 \uc5d0\uc2a4\ud30c\ub0d0\uadf9\uc5d0\uc11c \ud55c \ud2b9\uc0c9\uc774 \ub41c '\uc678\ud22c\uc640 \uce7c\uc758 \ud76c\uace1'\uc758 \ud6a8\uc2dc\ub77c\uace0 \ud558\uaca0\ub2e4. \uc774\ub7ec\ud55c \uc5f0\uadf9\uc740 \ub300\ub3c4\uc2dc\uc758 \uac74\ubb3c \uc548\ub730\uc5d0\uc11c \uac70\ud589\ub418\uc5c8\uc73c\uba70 \uc0c1\uc5f0 \ub54c\ub294 \uc18c\ub780\ud558\uae30 \uc9dd\uc774 \uc5c6\uc5c8\ub2e4\uace0 \ud55c\ub2e4.", "sentence_answer": "\uc5f0\uadf9\uc758 \uc544\ubc84\uc9c0\ub85c \ubd88\ub9ac\ub294 \ud658 \ub378 \uc5d4\uc2dc\ub098(1469?-1529?)\uc758 < \uc5d0\uae00\ub85c\uac00\uc2a4 (\u7267\u6b4c)", "paragraph_id": "6529069-1-0", "paragraph_question": "question: \ub4dc\ub77c\ub9c8\ud48d\uc758 \ub300\ud654\ub85c \uc8fc\uc694\uc778\ubb3c\uc774 \uc591\uce58\uae30\uc640 \ucc9c\uc0ac\uc774\uba70 \uc911\uc138 \uc131\ud0c4\uc81c\uadf9\uc758 \ubcc0\ud615\uc774\ub77c \ud560 \uc218 \uc788\ub294 \uac83\uc740?, context: \uc5d0\uc2a4\ud30c\ub0d0 \uc5f0\uadf9\uc758 \uc544\ubc84\uc9c0\ub85c \ubd88\ub9ac\ub294 \ud658 \ub378 \uc5d4\uc2dc\ub098(1469?-1529?)\uc758 <\uc5d0\uae00\ub85c\uac00\uc2a4(\u7267\u6b4c)>\ub294 \ub4dc\ub77c\ub9c8\ud48d\uc758 \ub300\ud654\ub85c \uc591\uce58\uae30\uc640 \ucc9c\uc0ac\uac00 \uc8fc\uc694\uc778\ubb3c\uc774\uba70 \uc911\uc138 \uc131\ud0c4\uc81c\uadf9\uc758 \ubcc0\ud615\uc774\ub77c \ud558\uaca0\ub2e4. \uadf8 \ud6c4 \ubbf8\ubaa8\uc2a4(momos), \uc989 \uac00\uba74\uc744 \uc4f4 \ubc1c\ub808\uc640 \ud30c\uc2a4\ud1a0\ub77c\ub808\uc2a4(\ubaa9\uac00\uadf9)\uac00 \ud589\ud574\uc84c\uc73c\ub098 \ubc30\uc6b0\ub294 \ube48\uc57d\ud55c \uc7a5\ube44\ub85c \ub3c4\uc2dc\ub098 \uc9c0\ubc29\uc744 \uc21c\ud68c\ud588\ub2e4. \ubc14\ub974\ud1a8\ub85c\uba54 \ub370 \ud1a0\ub974\ub808\uc2a4 \ub098\uc544\ub85c(1521\ub144 \uc0ac\ub9dd)\uc758 \ud76c\uace1\uc9d1 <\ud504\ub85c\ud314\ub77c\ub514\uc544>(1517)\uc758 \uc11c\ubb38\uc740 \ud638\ub77c\ud2f0\uc6b0\uc2a4\ub97c \ub530\ub978 5\ub9c9 \ud615\uc2dd \ub4f1\uc758 \uc5f0\uadf9\ub860\uc774\ub098, \ud76c\uadf9\uacfc \ube44\uadf9\uc758 \uad6c\ubcc4\uc740 \uc791\ud488\uba74\uc5d0\uc11c \uc804\ud600 \ubb34\uc2dc\ub418\uace0 \uc788\uc5c8\ub2e4. <\ud558\uc778\uc758 \uc2dd\ub2f9> <\uc138\ub77c\ud53c\ub098> <\ub77c \uc774\uba54\ub124\uc544> \ub4f1\uc758 \uc791\ud488\uc774 \uc788\uace0, \ud2b9\ud788 \ub9c8\uc9c0\ub9c9 \uac83\uc740 \ud6c4\ub300\uc758 \uc5d0\uc2a4\ud30c\ub0d0\uadf9\uc5d0\uc11c \ud55c \ud2b9\uc0c9\uc774 \ub41c '\uc678\ud22c\uc640 \uce7c\uc758 \ud76c\uace1'\uc758 \ud6a8\uc2dc\ub77c\uace0 \ud558\uaca0\ub2e4. \uc774\ub7ec\ud55c \uc5f0\uadf9\uc740 \ub300\ub3c4\uc2dc\uc758 \uac74\ubb3c \uc548\ub730\uc5d0\uc11c \uac70\ud589\ub418\uc5c8\uc73c\uba70 \uc0c1\uc5f0 \ub54c\ub294 \uc18c\ub780\ud558\uae30 \uc9dd\uc774 \uc5c6\uc5c8\ub2e4\uace0 \ud55c\ub2e4."} {"question": "\uc720\ub85c\ud30c\uc758 \uc870\uc11d \uac00\uc18d\uc774 \uc720\ub85c\ud30c\uc758 \ub0b4\ubd80\ub97c \uc6c0\uc9c1\uc774\uac8c \ud574 \ubc1c\uc0dd\ud558\uac8c \ud558\ub294 \uac83\uc740?", "paragraph": "\uc720\ub85c\ud30c\uc758 \uada4\ub3c4\ub294 \uadfc\ucc98 \ub2e4\ub978 \uc704\uc131\ub4e4\uc774 \ubbf8\uce58\ub294 \uc911\ub825 \ub54c\ubb38\uc5d0 \uc57d\uac04 \ucc0c\uadf8\ub7ec\uc838 \uc788\ub294\ub370 \uc774\ub294 \uc720\ub85c\ud30c\uc758 \ubaa9\uc131 \uc9c1\ud558\uc810\uc774 \ud3c9\uade0 \uc704\uce58 \uadfc\ucc98\uc5d0\uc11c \uc654\ub2e4\uac14\ub2e4 \ud558\uac8c \ub9cc\ub4e0\ub2e4. \uc720\ub85c\ud30c\uac00 \ubaa9\uc131\uc5d0 \uac00\uae4c\uc774 \uc624\uba74 \ubaa9\uc131\uc73c\ub85c\ubd80\ud130 \uba40\ub9ac \uc788\uc744 \ub54c\ubcf4\ub2e4 \uc911\ub825\uc774 \uac15\ud574\uc838 \uc720\ub85c\ud30c\uc758 \ubaa8\uc591\uc744 \ubcc0\ud615\uc2dc\ud0a4\uace0, \uc720\ub85c\ud30c\uac00 \ubaa9\uc131\uc73c\ub85c\ubd80\ud130 \uba3c \uc9c0\uc810\uc73c\ub85c \uc774\ub3d9\ud558\uba74 \uc911\ub825\uc774 \uc57d\ud574\uc838\uc11c \uc720\ub85c\ud30c\uac00 \ub2e4\uc2dc \uc6d0\ub798 \ubaa8\uc2b5\uc73c\ub85c \ub418\ub3cc\uc544\uac00\uac8c \ub41c\ub2e4. \uc774\ub7ec\ud55c \uc6b4\ub3d9\uc774 \ud45c\uba74\uc744 \uc950\uc5b4\uc9dc\uba70 \uc870\uc11d \uac00\uc18d \ud604\uc0c1\uc744 \ub9cc\ub4e0\ub2e4. \uc720\ub85c\ud30c\uc758 \uada4\ub3c4 \uc774\uc2ec\ub960\uc740 \uc774\uc624\uc640\uc758 \uada4\ub3c4 \uacf5\uba85\uc73c\ub85c \uc720\uc9c0\ub41c\ub2e4. \ub530\ub77c\uc11c, \uc774 \uc720\ub85c\ud30c\uc758 \uc870\uc11d \uac00\uc18d\uc740 \uc720\ub85c\ud30c\uc758 \ub0b4\ubd80\uac00 \uc6c0\uc9c1\uc774\uba74\uc11c \ub9c8\ucc30\uc5f4\uc744 \ub9cc\ub4e4\uc5b4, \ubc14\ub2e4\uac00 \uc5bc\uc9c0 \uc54a\uac8c \ud558\ub294 \uc5d0\ub108\uc9c0\ub97c \ub9cc\ub4e0\ub2e4. \uc774 \uc5d0\ub108\uc9c0\uc758 \uad81\uadf9\uc801\uc778 \ubc1c\uc0dd \uc6d0\uc778\uc740 \uc774\uc624\uac00 \uacf5\uc804\ud558\uae30 \ub54c\ubb38\uc774\ub2e4. \ubaa9\uc131 \uc8fc\uc704\ub97c \ub3cc \ub54c, \uc774\uc624\uac00 \ubaa9\uc131\uc73c\ub85c\ubd80\ud130 \uc5d0\ub108\uc9c0\ub97c \ubc1b\uace0, \uadf8 \uc5d0\ub108\uc9c0\ub97c \uc720\ub85c\ud30c\uc640 \uac00\ub2c8\uba54\ub370\ub85c \ubcf4\ub0b4\uae30 \ub54c\ubb38\uc774\ub2e4.", "answer": "\ub9c8\ucc30\uc5f4", "sentence": "\ub530\ub77c\uc11c, \uc774 \uc720\ub85c\ud30c\uc758 \uc870\uc11d \uac00\uc18d\uc740 \uc720\ub85c\ud30c\uc758 \ub0b4\ubd80\uac00 \uc6c0\uc9c1\uc774\uba74\uc11c \ub9c8\ucc30\uc5f4 \uc744 \ub9cc\ub4e4\uc5b4, \ubc14\ub2e4\uac00 \uc5bc\uc9c0 \uc54a\uac8c \ud558\ub294 \uc5d0\ub108\uc9c0\ub97c \ub9cc\ub4e0\ub2e4.", "paragraph_sentence": "\uc720\ub85c\ud30c\uc758 \uada4\ub3c4\ub294 \uadfc\ucc98 \ub2e4\ub978 \uc704\uc131\ub4e4\uc774 \ubbf8\uce58\ub294 \uc911\ub825 \ub54c\ubb38\uc5d0 \uc57d\uac04 \ucc0c\uadf8\ub7ec\uc838 \uc788\ub294\ub370 \uc774\ub294 \uc720\ub85c\ud30c\uc758 \ubaa9\uc131 \uc9c1\ud558\uc810\uc774 \ud3c9\uade0 \uc704\uce58 \uadfc\ucc98\uc5d0\uc11c \uc654\ub2e4\uac14\ub2e4 \ud558\uac8c \ub9cc\ub4e0\ub2e4. \uc720\ub85c\ud30c\uac00 \ubaa9\uc131\uc5d0 \uac00\uae4c\uc774 \uc624\uba74 \ubaa9\uc131\uc73c\ub85c\ubd80\ud130 \uba40\ub9ac \uc788\uc744 \ub54c\ubcf4\ub2e4 \uc911\ub825\uc774 \uac15\ud574\uc838 \uc720\ub85c\ud30c\uc758 \ubaa8\uc591\uc744 \ubcc0\ud615\uc2dc\ud0a4\uace0, \uc720\ub85c\ud30c\uac00 \ubaa9\uc131\uc73c\ub85c\ubd80\ud130 \uba3c \uc9c0\uc810\uc73c\ub85c \uc774\ub3d9\ud558\uba74 \uc911\ub825\uc774 \uc57d\ud574\uc838\uc11c \uc720\ub85c\ud30c\uac00 \ub2e4\uc2dc \uc6d0\ub798 \ubaa8\uc2b5\uc73c\ub85c \ub418\ub3cc\uc544\uac00\uac8c \ub41c\ub2e4. \uc774\ub7ec\ud55c \uc6b4\ub3d9\uc774 \ud45c\uba74\uc744 \uc950\uc5b4\uc9dc\uba70 \uc870\uc11d \uac00\uc18d \ud604\uc0c1\uc744 \ub9cc\ub4e0\ub2e4. \uc720\ub85c\ud30c\uc758 \uada4\ub3c4 \uc774\uc2ec\ub960\uc740 \uc774\uc624\uc640\uc758 \uada4\ub3c4 \uacf5\uba85\uc73c\ub85c \uc720\uc9c0\ub41c\ub2e4. \ub530\ub77c\uc11c, \uc774 \uc720\ub85c\ud30c\uc758 \uc870\uc11d \uac00\uc18d\uc740 \uc720\ub85c\ud30c\uc758 \ub0b4\ubd80\uac00 \uc6c0\uc9c1\uc774\uba74\uc11c \ub9c8\ucc30\uc5f4 \uc744 \ub9cc\ub4e4\uc5b4, \ubc14\ub2e4\uac00 \uc5bc\uc9c0 \uc54a\uac8c \ud558\ub294 \uc5d0\ub108\uc9c0\ub97c \ub9cc\ub4e0\ub2e4. \uc774 \uc5d0\ub108\uc9c0\uc758 \uad81\uadf9\uc801\uc778 \ubc1c\uc0dd \uc6d0\uc778\uc740 \uc774\uc624\uac00 \uacf5\uc804\ud558\uae30 \ub54c\ubb38\uc774\ub2e4. \ubaa9\uc131 \uc8fc\uc704\ub97c \ub3cc \ub54c, \uc774\uc624\uac00 \ubaa9\uc131\uc73c\ub85c\ubd80\ud130 \uc5d0\ub108\uc9c0\ub97c \ubc1b\uace0, \uadf8 \uc5d0\ub108\uc9c0\ub97c \uc720\ub85c\ud30c\uc640 \uac00\ub2c8\uba54\ub370\ub85c \ubcf4\ub0b4\uae30 \ub54c\ubb38\uc774\ub2e4.", "paragraph_answer": "\uc720\ub85c\ud30c\uc758 \uada4\ub3c4\ub294 \uadfc\ucc98 \ub2e4\ub978 \uc704\uc131\ub4e4\uc774 \ubbf8\uce58\ub294 \uc911\ub825 \ub54c\ubb38\uc5d0 \uc57d\uac04 \ucc0c\uadf8\ub7ec\uc838 \uc788\ub294\ub370 \uc774\ub294 \uc720\ub85c\ud30c\uc758 \ubaa9\uc131 \uc9c1\ud558\uc810\uc774 \ud3c9\uade0 \uc704\uce58 \uadfc\ucc98\uc5d0\uc11c \uc654\ub2e4\uac14\ub2e4 \ud558\uac8c \ub9cc\ub4e0\ub2e4. \uc720\ub85c\ud30c\uac00 \ubaa9\uc131\uc5d0 \uac00\uae4c\uc774 \uc624\uba74 \ubaa9\uc131\uc73c\ub85c\ubd80\ud130 \uba40\ub9ac \uc788\uc744 \ub54c\ubcf4\ub2e4 \uc911\ub825\uc774 \uac15\ud574\uc838 \uc720\ub85c\ud30c\uc758 \ubaa8\uc591\uc744 \ubcc0\ud615\uc2dc\ud0a4\uace0, \uc720\ub85c\ud30c\uac00 \ubaa9\uc131\uc73c\ub85c\ubd80\ud130 \uba3c \uc9c0\uc810\uc73c\ub85c \uc774\ub3d9\ud558\uba74 \uc911\ub825\uc774 \uc57d\ud574\uc838\uc11c \uc720\ub85c\ud30c\uac00 \ub2e4\uc2dc \uc6d0\ub798 \ubaa8\uc2b5\uc73c\ub85c \ub418\ub3cc\uc544\uac00\uac8c \ub41c\ub2e4. \uc774\ub7ec\ud55c \uc6b4\ub3d9\uc774 \ud45c\uba74\uc744 \uc950\uc5b4\uc9dc\uba70 \uc870\uc11d \uac00\uc18d \ud604\uc0c1\uc744 \ub9cc\ub4e0\ub2e4. \uc720\ub85c\ud30c\uc758 \uada4\ub3c4 \uc774\uc2ec\ub960\uc740 \uc774\uc624\uc640\uc758 \uada4\ub3c4 \uacf5\uba85\uc73c\ub85c \uc720\uc9c0\ub41c\ub2e4. \ub530\ub77c\uc11c, \uc774 \uc720\ub85c\ud30c\uc758 \uc870\uc11d \uac00\uc18d\uc740 \uc720\ub85c\ud30c\uc758 \ub0b4\ubd80\uac00 \uc6c0\uc9c1\uc774\uba74\uc11c \ub9c8\ucc30\uc5f4 \uc744 \ub9cc\ub4e4\uc5b4, \ubc14\ub2e4\uac00 \uc5bc\uc9c0 \uc54a\uac8c \ud558\ub294 \uc5d0\ub108\uc9c0\ub97c \ub9cc\ub4e0\ub2e4. \uc774 \uc5d0\ub108\uc9c0\uc758 \uad81\uadf9\uc801\uc778 \ubc1c\uc0dd \uc6d0\uc778\uc740 \uc774\uc624\uac00 \uacf5\uc804\ud558\uae30 \ub54c\ubb38\uc774\ub2e4. \ubaa9\uc131 \uc8fc\uc704\ub97c \ub3cc \ub54c, \uc774\uc624\uac00 \ubaa9\uc131\uc73c\ub85c\ubd80\ud130 \uc5d0\ub108\uc9c0\ub97c \ubc1b\uace0, \uadf8 \uc5d0\ub108\uc9c0\ub97c \uc720\ub85c\ud30c\uc640 \uac00\ub2c8\uba54\ub370\ub85c \ubcf4\ub0b4\uae30 \ub54c\ubb38\uc774\ub2e4.", "sentence_answer": "\ub530\ub77c\uc11c, \uc774 \uc720\ub85c\ud30c\uc758 \uc870\uc11d \uac00\uc18d\uc740 \uc720\ub85c\ud30c\uc758 \ub0b4\ubd80\uac00 \uc6c0\uc9c1\uc774\uba74\uc11c \ub9c8\ucc30\uc5f4 \uc744 \ub9cc\ub4e4\uc5b4, \ubc14\ub2e4\uac00 \uc5bc\uc9c0 \uc54a\uac8c \ud558\ub294 \uc5d0\ub108\uc9c0\ub97c \ub9cc\ub4e0\ub2e4.", "paragraph_id": "6527070-2-2", "paragraph_question": "question: \uc720\ub85c\ud30c\uc758 \uc870\uc11d \uac00\uc18d\uc774 \uc720\ub85c\ud30c\uc758 \ub0b4\ubd80\ub97c \uc6c0\uc9c1\uc774\uac8c \ud574 \ubc1c\uc0dd\ud558\uac8c \ud558\ub294 \uac83\uc740?, context: \uc720\ub85c\ud30c\uc758 \uada4\ub3c4\ub294 \uadfc\ucc98 \ub2e4\ub978 \uc704\uc131\ub4e4\uc774 \ubbf8\uce58\ub294 \uc911\ub825 \ub54c\ubb38\uc5d0 \uc57d\uac04 \ucc0c\uadf8\ub7ec\uc838 \uc788\ub294\ub370 \uc774\ub294 \uc720\ub85c\ud30c\uc758 \ubaa9\uc131 \uc9c1\ud558\uc810\uc774 \ud3c9\uade0 \uc704\uce58 \uadfc\ucc98\uc5d0\uc11c \uc654\ub2e4\uac14\ub2e4 \ud558\uac8c \ub9cc\ub4e0\ub2e4. \uc720\ub85c\ud30c\uac00 \ubaa9\uc131\uc5d0 \uac00\uae4c\uc774 \uc624\uba74 \ubaa9\uc131\uc73c\ub85c\ubd80\ud130 \uba40\ub9ac \uc788\uc744 \ub54c\ubcf4\ub2e4 \uc911\ub825\uc774 \uac15\ud574\uc838 \uc720\ub85c\ud30c\uc758 \ubaa8\uc591\uc744 \ubcc0\ud615\uc2dc\ud0a4\uace0, \uc720\ub85c\ud30c\uac00 \ubaa9\uc131\uc73c\ub85c\ubd80\ud130 \uba3c \uc9c0\uc810\uc73c\ub85c \uc774\ub3d9\ud558\uba74 \uc911\ub825\uc774 \uc57d\ud574\uc838\uc11c \uc720\ub85c\ud30c\uac00 \ub2e4\uc2dc \uc6d0\ub798 \ubaa8\uc2b5\uc73c\ub85c \ub418\ub3cc\uc544\uac00\uac8c \ub41c\ub2e4. \uc774\ub7ec\ud55c \uc6b4\ub3d9\uc774 \ud45c\uba74\uc744 \uc950\uc5b4\uc9dc\uba70 \uc870\uc11d \uac00\uc18d \ud604\uc0c1\uc744 \ub9cc\ub4e0\ub2e4. \uc720\ub85c\ud30c\uc758 \uada4\ub3c4 \uc774\uc2ec\ub960\uc740 \uc774\uc624\uc640\uc758 \uada4\ub3c4 \uacf5\uba85\uc73c\ub85c \uc720\uc9c0\ub41c\ub2e4. \ub530\ub77c\uc11c, \uc774 \uc720\ub85c\ud30c\uc758 \uc870\uc11d \uac00\uc18d\uc740 \uc720\ub85c\ud30c\uc758 \ub0b4\ubd80\uac00 \uc6c0\uc9c1\uc774\uba74\uc11c \ub9c8\ucc30\uc5f4\uc744 \ub9cc\ub4e4\uc5b4, \ubc14\ub2e4\uac00 \uc5bc\uc9c0 \uc54a\uac8c \ud558\ub294 \uc5d0\ub108\uc9c0\ub97c \ub9cc\ub4e0\ub2e4. \uc774 \uc5d0\ub108\uc9c0\uc758 \uad81\uadf9\uc801\uc778 \ubc1c\uc0dd \uc6d0\uc778\uc740 \uc774\uc624\uac00 \uacf5\uc804\ud558\uae30 \ub54c\ubb38\uc774\ub2e4. \ubaa9\uc131 \uc8fc\uc704\ub97c \ub3cc \ub54c, \uc774\uc624\uac00 \ubaa9\uc131\uc73c\ub85c\ubd80\ud130 \uc5d0\ub108\uc9c0\ub97c \ubc1b\uace0, \uadf8 \uc5d0\ub108\uc9c0\ub97c \uc720\ub85c\ud30c\uc640 \uac00\ub2c8\uba54\ub370\ub85c \ubcf4\ub0b4\uae30 \ub54c\ubb38\uc774\ub2e4."} {"question": "\uae40\uc131\ubbfc\uc774 \ub9c8\uc57d\ub958\uad00\ub9ac\uc5d0 \uad00\ud55c \ubc95\ub960\uc704\ubc18 \ud604\uc758\ub97c \uac16\uac8c\ub41c \ubb38\uc81c\uc758 \ubb3c\uc9c8\uc740 \ubb34\uc5c7\uc778\uac00?", "paragraph": "2010\ub144 12\uc6d4 4\uc77c, \ud544\ub85c\ud3f0\uc744 \ud22c\uc57d\ud55c \ud610\uc758(\ub9c8\uc57d\ub958\uad00\ub9ac\uc5d0 \uad00\ud55c \ubc95\ub960\uc704\ubc18)\ub85c \uc801\ubc1c\ub418\uc5b4 \uad6c\uc18d\ub410\ub2e4. \ub300\ud55c\ubbfc\uad6d \uac80\ucc30\uc5d0 \ub530\ub974\uba74, \ucd5c\uadfc \uc678\uad6d\uc5d0\uc11c \ud544\ub85c\ud3f0\uc744 \ubc00\ubc18\uc785\ud574 \uc790\ud0dd\uc5d0\uc11c \uc0c1\uc2b5\uc801\uc73c\ub85c \ud22c\uc57d\ud55c \ud610\uc758\ub97c \ubc1b\uace0 \uc788\ub2e4. \ub530\ub77c\uc11c, \ud55c\uad6d\ubc29\uc1a1\uacf5\uc0ac \ud574\ud53c\uc120\ub370\uc774 \uc81c\uc791\uc9c4\uc740 \uae40\uc131\ubbfc\uc774 \ub0a8\uc790\uc758 \uc790\uaca9\uc5d0\uc11c \ud558\ucc28\ud558\uac8c \ub420 \uac83\uc774\uba70 12\uc6d4 5\uc77c\uc5d0 \ubc29\uc1a1\ub420 \ud3b8\uc9d1\ubcf8\uc740 \ucd9c\uc5f0 \ubd84\ub7c9\uc744 \uc0ad\uc81c\ud558\uae30\ub85c \ud588\ub2e4. \uc774 \ud6c4 \uae40\uc131\ubbfc\uc740 \ud2b8\uc704\ud130\ub97c \ud1b5\ud574 \uc0ac\uacfc\ud588\ub2e4. \uae40\ud0dc\uc6d0\uc740 \uc774\ubc88 \uc0ac\uac74\uc744 \uad00\ub828\ud574 \uae40\uc131\ubbfc\uc5d0\uac8c \ub530\ub73b\ud55c \ucda9\uace0\ub97c \ud558\uae30\ub3c4 \ud588\ub2e4., 2011\ub144 1\uc6d4 24\uc77c \uc7ac\ud310\uc5d0\uc11c \uac80\ucc30 \uce21\uc774 \uc9d5\uc5ed 4\ub144\uc744 \uad6c\ud615\ud588\ub2e4. \uadf8\ub7ec\ub2e4\uac00 1\uc2ec\uc5d0\uc11c \uc9d5\uc5ed 2\ub144 6\uac1c\uc6d4\uc744 \uc120\uace0 \ubc1b\uc558\uc73c\ub098 \ud56d\uc18c\ud558\uc5ec 2\uc2ec\uc5d0\uc11c\ub294 \uc9d5\uc5ed 2\ub144 6\uac1c\uc6d4, \uc9d1\ud589\uc720\uc608 4\ub144 \ubc0f \ucd94\uc9d5\uae08 904,500\uc6d0, \uc0ac\ud68c\ubd09\uc0ac 120\uc2dc\uac04, \ubcf4\ud638\uac10\ucc30 2\ub144\uacfc \uc57d\ubb3c\uce58\ub8cc 40\uc2dc\uac04\uc744 \uc120\uace0\ubc1b\uc558\ub2e4. \uadf8\ub7ec\ub098 2014\ub144 11\uc6d4 24\uc77c 11\uc6d4 24\uc77c \uc628\ub77c\uc778 \uad11\uace0\ub97c \ubcf4\uace0 \ud544\ub85c\ud3f0\uc744 \ud22c\uc57d\ud55c \ud610\uc758\ub85c \uad6c\uc18d\ub418\uc5b4 \uc9d5\uc5ed 10\uc6d4 \ubc0f \ucd94\uc9d5\uae08 70\ub9cc\uc6d0\uc744 \uc120\uace0\ud588\ub2e4.", "answer": "\ud544\ub85c\ud3f0", "sentence": "2010\ub144 12\uc6d4 4\uc77c, \ud544\ub85c\ud3f0 \uc744 \ud22c\uc57d\ud55c \ud610\uc758(\ub9c8\uc57d\ub958\uad00\ub9ac\uc5d0 \uad00\ud55c \ubc95\ub960\uc704\ubc18)\ub85c \uc801\ubc1c\ub418\uc5b4 \uad6c\uc18d\ub410\ub2e4.", "paragraph_sentence": " 2010\ub144 12\uc6d4 4\uc77c, \ud544\ub85c\ud3f0 \uc744 \ud22c\uc57d\ud55c \ud610\uc758(\ub9c8\uc57d\ub958\uad00\ub9ac\uc5d0 \uad00\ud55c \ubc95\ub960\uc704\ubc18)\ub85c \uc801\ubc1c\ub418\uc5b4 \uad6c\uc18d\ub410\ub2e4. \ub300\ud55c\ubbfc\uad6d \uac80\ucc30\uc5d0 \ub530\ub974\uba74, \ucd5c\uadfc \uc678\uad6d\uc5d0\uc11c \ud544\ub85c\ud3f0\uc744 \ubc00\ubc18\uc785\ud574 \uc790\ud0dd\uc5d0\uc11c \uc0c1\uc2b5\uc801\uc73c\ub85c \ud22c\uc57d\ud55c \ud610\uc758\ub97c \ubc1b\uace0 \uc788\ub2e4. \ub530\ub77c\uc11c, \ud55c\uad6d\ubc29\uc1a1\uacf5\uc0ac \ud574\ud53c\uc120\ub370\uc774 \uc81c\uc791\uc9c4\uc740 \uae40\uc131\ubbfc\uc774 \ub0a8\uc790\uc758 \uc790\uaca9\uc5d0\uc11c \ud558\ucc28\ud558\uac8c \ub420 \uac83\uc774\uba70 12\uc6d4 5\uc77c\uc5d0 \ubc29\uc1a1\ub420 \ud3b8\uc9d1\ubcf8\uc740 \ucd9c\uc5f0 \ubd84\ub7c9\uc744 \uc0ad\uc81c\ud558\uae30\ub85c \ud588\ub2e4. \uc774 \ud6c4 \uae40\uc131\ubbfc\uc740 \ud2b8\uc704\ud130\ub97c \ud1b5\ud574 \uc0ac\uacfc\ud588\ub2e4. \uae40\ud0dc\uc6d0\uc740 \uc774\ubc88 \uc0ac\uac74\uc744 \uad00\ub828\ud574 \uae40\uc131\ubbfc\uc5d0\uac8c \ub530\ub73b\ud55c \ucda9\uace0\ub97c \ud558\uae30\ub3c4 \ud588\ub2e4. , 2011\ub144 1\uc6d4 24\uc77c \uc7ac\ud310\uc5d0\uc11c \uac80\ucc30 \uce21\uc774 \uc9d5\uc5ed 4\ub144\uc744 \uad6c\ud615\ud588\ub2e4. \uadf8\ub7ec\ub2e4\uac00 1\uc2ec\uc5d0\uc11c \uc9d5\uc5ed 2\ub144 6\uac1c\uc6d4\uc744 \uc120\uace0 \ubc1b\uc558\uc73c\ub098 \ud56d\uc18c\ud558\uc5ec 2\uc2ec\uc5d0\uc11c\ub294 \uc9d5\uc5ed 2\ub144 6\uac1c\uc6d4, \uc9d1\ud589\uc720\uc608 4\ub144 \ubc0f \ucd94\uc9d5\uae08 904,500\uc6d0, \uc0ac\ud68c\ubd09\uc0ac 120\uc2dc\uac04, \ubcf4\ud638\uac10\ucc30 2\ub144\uacfc \uc57d\ubb3c\uce58\ub8cc 40\uc2dc\uac04\uc744 \uc120\uace0\ubc1b\uc558\ub2e4. \uadf8\ub7ec\ub098 2014\ub144 11\uc6d4 24\uc77c 11\uc6d4 24\uc77c \uc628\ub77c\uc778 \uad11\uace0\ub97c \ubcf4\uace0 \ud544\ub85c\ud3f0\uc744 \ud22c\uc57d\ud55c \ud610\uc758\ub85c \uad6c\uc18d\ub418\uc5b4 \uc9d5\uc5ed 10\uc6d4 \ubc0f \ucd94\uc9d5\uae08 70\ub9cc\uc6d0\uc744 \uc120\uace0\ud588\ub2e4.", "paragraph_answer": "2010\ub144 12\uc6d4 4\uc77c, \ud544\ub85c\ud3f0 \uc744 \ud22c\uc57d\ud55c \ud610\uc758(\ub9c8\uc57d\ub958\uad00\ub9ac\uc5d0 \uad00\ud55c \ubc95\ub960\uc704\ubc18)\ub85c \uc801\ubc1c\ub418\uc5b4 \uad6c\uc18d\ub410\ub2e4. \ub300\ud55c\ubbfc\uad6d \uac80\ucc30\uc5d0 \ub530\ub974\uba74, \ucd5c\uadfc \uc678\uad6d\uc5d0\uc11c \ud544\ub85c\ud3f0\uc744 \ubc00\ubc18\uc785\ud574 \uc790\ud0dd\uc5d0\uc11c \uc0c1\uc2b5\uc801\uc73c\ub85c \ud22c\uc57d\ud55c \ud610\uc758\ub97c \ubc1b\uace0 \uc788\ub2e4. \ub530\ub77c\uc11c, \ud55c\uad6d\ubc29\uc1a1\uacf5\uc0ac \ud574\ud53c\uc120\ub370\uc774 \uc81c\uc791\uc9c4\uc740 \uae40\uc131\ubbfc\uc774 \ub0a8\uc790\uc758 \uc790\uaca9\uc5d0\uc11c \ud558\ucc28\ud558\uac8c \ub420 \uac83\uc774\uba70 12\uc6d4 5\uc77c\uc5d0 \ubc29\uc1a1\ub420 \ud3b8\uc9d1\ubcf8\uc740 \ucd9c\uc5f0 \ubd84\ub7c9\uc744 \uc0ad\uc81c\ud558\uae30\ub85c \ud588\ub2e4. \uc774 \ud6c4 \uae40\uc131\ubbfc\uc740 \ud2b8\uc704\ud130\ub97c \ud1b5\ud574 \uc0ac\uacfc\ud588\ub2e4. \uae40\ud0dc\uc6d0\uc740 \uc774\ubc88 \uc0ac\uac74\uc744 \uad00\ub828\ud574 \uae40\uc131\ubbfc\uc5d0\uac8c \ub530\ub73b\ud55c \ucda9\uace0\ub97c \ud558\uae30\ub3c4 \ud588\ub2e4., 2011\ub144 1\uc6d4 24\uc77c \uc7ac\ud310\uc5d0\uc11c \uac80\ucc30 \uce21\uc774 \uc9d5\uc5ed 4\ub144\uc744 \uad6c\ud615\ud588\ub2e4. \uadf8\ub7ec\ub2e4\uac00 1\uc2ec\uc5d0\uc11c \uc9d5\uc5ed 2\ub144 6\uac1c\uc6d4\uc744 \uc120\uace0 \ubc1b\uc558\uc73c\ub098 \ud56d\uc18c\ud558\uc5ec 2\uc2ec\uc5d0\uc11c\ub294 \uc9d5\uc5ed 2\ub144 6\uac1c\uc6d4, \uc9d1\ud589\uc720\uc608 4\ub144 \ubc0f \ucd94\uc9d5\uae08 904,500\uc6d0, \uc0ac\ud68c\ubd09\uc0ac 120\uc2dc\uac04, \ubcf4\ud638\uac10\ucc30 2\ub144\uacfc \uc57d\ubb3c\uce58\ub8cc 40\uc2dc\uac04\uc744 \uc120\uace0\ubc1b\uc558\ub2e4. \uadf8\ub7ec\ub098 2014\ub144 11\uc6d4 24\uc77c 11\uc6d4 24\uc77c \uc628\ub77c\uc778 \uad11\uace0\ub97c \ubcf4\uace0 \ud544\ub85c\ud3f0\uc744 \ud22c\uc57d\ud55c \ud610\uc758\ub85c \uad6c\uc18d\ub418\uc5b4 \uc9d5\uc5ed 10\uc6d4 \ubc0f \ucd94\uc9d5\uae08 70\ub9cc\uc6d0\uc744 \uc120\uace0\ud588\ub2e4.", "sentence_answer": "2010\ub144 12\uc6d4 4\uc77c, \ud544\ub85c\ud3f0 \uc744 \ud22c\uc57d\ud55c \ud610\uc758(\ub9c8\uc57d\ub958\uad00\ub9ac\uc5d0 \uad00\ud55c \ubc95\ub960\uc704\ubc18)\ub85c \uc801\ubc1c\ub418\uc5b4 \uad6c\uc18d\ub410\ub2e4.", "paragraph_id": "6481770-1-0", "paragraph_question": "question: \uae40\uc131\ubbfc\uc774 \ub9c8\uc57d\ub958\uad00\ub9ac\uc5d0 \uad00\ud55c \ubc95\ub960\uc704\ubc18 \ud604\uc758\ub97c \uac16\uac8c\ub41c \ubb38\uc81c\uc758 \ubb3c\uc9c8\uc740 \ubb34\uc5c7\uc778\uac00?, context: 2010\ub144 12\uc6d4 4\uc77c, \ud544\ub85c\ud3f0\uc744 \ud22c\uc57d\ud55c \ud610\uc758(\ub9c8\uc57d\ub958\uad00\ub9ac\uc5d0 \uad00\ud55c \ubc95\ub960\uc704\ubc18)\ub85c \uc801\ubc1c\ub418\uc5b4 \uad6c\uc18d\ub410\ub2e4. \ub300\ud55c\ubbfc\uad6d \uac80\ucc30\uc5d0 \ub530\ub974\uba74, \ucd5c\uadfc \uc678\uad6d\uc5d0\uc11c \ud544\ub85c\ud3f0\uc744 \ubc00\ubc18\uc785\ud574 \uc790\ud0dd\uc5d0\uc11c \uc0c1\uc2b5\uc801\uc73c\ub85c \ud22c\uc57d\ud55c \ud610\uc758\ub97c \ubc1b\uace0 \uc788\ub2e4. \ub530\ub77c\uc11c, \ud55c\uad6d\ubc29\uc1a1\uacf5\uc0ac \ud574\ud53c\uc120\ub370\uc774 \uc81c\uc791\uc9c4\uc740 \uae40\uc131\ubbfc\uc774 \ub0a8\uc790\uc758 \uc790\uaca9\uc5d0\uc11c \ud558\ucc28\ud558\uac8c \ub420 \uac83\uc774\uba70 12\uc6d4 5\uc77c\uc5d0 \ubc29\uc1a1\ub420 \ud3b8\uc9d1\ubcf8\uc740 \ucd9c\uc5f0 \ubd84\ub7c9\uc744 \uc0ad\uc81c\ud558\uae30\ub85c \ud588\ub2e4. \uc774 \ud6c4 \uae40\uc131\ubbfc\uc740 \ud2b8\uc704\ud130\ub97c \ud1b5\ud574 \uc0ac\uacfc\ud588\ub2e4. \uae40\ud0dc\uc6d0\uc740 \uc774\ubc88 \uc0ac\uac74\uc744 \uad00\ub828\ud574 \uae40\uc131\ubbfc\uc5d0\uac8c \ub530\ub73b\ud55c \ucda9\uace0\ub97c \ud558\uae30\ub3c4 \ud588\ub2e4., 2011\ub144 1\uc6d4 24\uc77c \uc7ac\ud310\uc5d0\uc11c \uac80\ucc30 \uce21\uc774 \uc9d5\uc5ed 4\ub144\uc744 \uad6c\ud615\ud588\ub2e4. \uadf8\ub7ec\ub2e4\uac00 1\uc2ec\uc5d0\uc11c \uc9d5\uc5ed 2\ub144 6\uac1c\uc6d4\uc744 \uc120\uace0 \ubc1b\uc558\uc73c\ub098 \ud56d\uc18c\ud558\uc5ec 2\uc2ec\uc5d0\uc11c\ub294 \uc9d5\uc5ed 2\ub144 6\uac1c\uc6d4, \uc9d1\ud589\uc720\uc608 4\ub144 \ubc0f \ucd94\uc9d5\uae08 904,500\uc6d0, \uc0ac\ud68c\ubd09\uc0ac 120\uc2dc\uac04, \ubcf4\ud638\uac10\ucc30 2\ub144\uacfc \uc57d\ubb3c\uce58\ub8cc 40\uc2dc\uac04\uc744 \uc120\uace0\ubc1b\uc558\ub2e4. \uadf8\ub7ec\ub098 2014\ub144 11\uc6d4 24\uc77c 11\uc6d4 24\uc77c \uc628\ub77c\uc778 \uad11\uace0\ub97c \ubcf4\uace0 \ud544\ub85c\ud3f0\uc744 \ud22c\uc57d\ud55c \ud610\uc758\ub85c \uad6c\uc18d\ub418\uc5b4 \uc9d5\uc5ed 10\uc6d4 \ubc0f \ucd94\uc9d5\uae08 70\ub9cc\uc6d0\uc744 \uc120\uace0\ud588\ub2e4."} {"question": "\uc601\uad6d\uc758 \uc218\ub3c4\ub294?", "paragraph": "\uadf8\ub808\uc774\ud2b8 \ube0c\ub9ac\ud2bc \ubd81\uc544\uc77c\ub79c\ub4dc \uc5f0\ud569\uc655\uad6d(-\u806f\u5408\u738b\u570b, \uc601\uc5b4: United Kingdom of Great Britain and Northern Ireland \uc720\ub098\uc774\ud2f0\ub4dc \ud0b9\ub364 \uc624\ube0c \uadf8\ub808\uc774\ud2b8 \ube0c\ub9ac\ud2bc \uc564 \ub178\ub358 \uc544\uc77c\ub79c\ub4dc, \ubb38\ud654\uc5b4: \ub300\ube0c\ub9ac\ud150 \ubc0f \ubd81\uc544\uc77c\ub79c\ub4dc \ub828\ud569\uc655\uad6d), \uc57d\uce6d \ube0c\ub9ac\ud2bc(\uc601\uc5b4: Britain) \ub610\ub294 \uc5f0\ud569\uc655\uad6d(\u806f\u5408\u738b\u570b, \uc601\uc5b4: United Kingdom \uc720\ub098\uc774\ud2f0\ub4dc \ud0b9\ub364, UK) \ud639\uc740 \uc601\uad6d(\u82f1\u570b)\uc740 \uc720\ub7fd \ubd81\uc11c\ubd80 \ud574\uc548\uc758 \ube0c\ub9ac\ud2bc \uc81c\ub3c4\uc5d0 \uc704\uce58\ud55c \uc8fc\uad8c\uad6d\uc774\uc790 \uc12c\ub098\ub77c\ub85c, \ubd81\ud574, \uc601\uad6d \ud574\ud611, \uc544\uc77c\ub79c\ub4dc \ud574 \ubc0f \ub300\uc11c\uc591\uc5d0 \uc811\ud558\uc5ec \uc788\uc73c\uba70 \uadf8\ub808\uc774\ud2b8 \ube0c\ub9ac\ud2bc \uc12c\uc758 \uc789\uae00\ub79c\ub4dc, \uc2a4\ucf54\ud2c0\ub79c\ub4dc, \uc6e8\uc77c\uc2a4 \ubc0f \uc544\uc77c\ub79c\ub4dc \uc12c \ubd81\ubd80\uc758 \ubd81\uc544\uc77c\ub79c\ub4dc\ub85c \ub124 \uac1c\uc758 \uad6c\uc131\uad6d\uc73c\ub85c \uc774\ub8e8\uc5b4\uc838 \uc788\ub294 \uc5f0\ud569\uad6d\uac00\uc774\ub2e4. \uc218\ub3c4\ub294 \ub7f0\ub358\uc774\uace0 \uc5f0\ud569\uad6d\uc758 \uad6c\uc131\uc744 \uc774\ub8e8\ub294 \uac01 \ub098\ub77c\ub4e4\uc740 \uc790\uce58\uad8c\uc744 \ubcf4\uc7a5\ubc1b\uace0 \uc788\ub2e4. \uc2a4\ucf54\ud2c0\ub79c\ub4dc, \uc6e8\uc77c\uc2a4, \ubd81\uc544\uc77c\ub79c\ub4dc\uc758 \uc218\ub3c4\ub294 \uac01\uac01 \uc5d0\ub4e0\ubc84\ub7ec, \uce74\ub514\ud504, \ubca8\ud30c\uc2a4\ud2b8\uc774\ub2e4.", "answer": "\ub7f0\ub358", "sentence": "\uc218\ub3c4\ub294 \ub7f0\ub358 \uc774\uace0 \uc5f0\ud569\uad6d\uc758 \uad6c\uc131\uc744 \uc774\ub8e8\ub294 \uac01 \ub098\ub77c\ub4e4\uc740 \uc790\uce58\uad8c\uc744 \ubcf4\uc7a5\ubc1b\uace0 \uc788\ub2e4.", "paragraph_sentence": "\uadf8\ub808\uc774\ud2b8 \ube0c\ub9ac\ud2bc \ubd81\uc544\uc77c\ub79c\ub4dc \uc5f0\ud569\uc655\uad6d(-\u806f\u5408\u738b\u570b, \uc601\uc5b4: United Kingdom of Great Britain and Northern Ireland \uc720\ub098\uc774\ud2f0\ub4dc \ud0b9\ub364 \uc624\ube0c \uadf8\ub808\uc774\ud2b8 \ube0c\ub9ac\ud2bc \uc564 \ub178\ub358 \uc544\uc77c\ub79c\ub4dc, \ubb38\ud654\uc5b4: \ub300\ube0c\ub9ac\ud150 \ubc0f \ubd81\uc544\uc77c\ub79c\ub4dc \ub828\ud569\uc655\uad6d), \uc57d\uce6d \ube0c\ub9ac\ud2bc(\uc601\uc5b4: Britain) \ub610\ub294 \uc5f0\ud569\uc655\uad6d(\u806f\u5408\u738b\u570b, \uc601\uc5b4: United Kingdom \uc720\ub098\uc774\ud2f0\ub4dc \ud0b9\ub364, UK) \ud639\uc740 \uc601\uad6d(\u82f1\u570b)\uc740 \uc720\ub7fd \ubd81\uc11c\ubd80 \ud574\uc548\uc758 \ube0c\ub9ac\ud2bc \uc81c\ub3c4\uc5d0 \uc704\uce58\ud55c \uc8fc\uad8c\uad6d\uc774\uc790 \uc12c\ub098\ub77c\ub85c, \ubd81\ud574, \uc601\uad6d \ud574\ud611, \uc544\uc77c\ub79c\ub4dc \ud574 \ubc0f \ub300\uc11c\uc591\uc5d0 \uc811\ud558\uc5ec \uc788\uc73c\uba70 \uadf8\ub808\uc774\ud2b8 \ube0c\ub9ac\ud2bc \uc12c\uc758 \uc789\uae00\ub79c\ub4dc, \uc2a4\ucf54\ud2c0\ub79c\ub4dc, \uc6e8\uc77c\uc2a4 \ubc0f \uc544\uc77c\ub79c\ub4dc \uc12c \ubd81\ubd80\uc758 \ubd81\uc544\uc77c\ub79c\ub4dc\ub85c \ub124 \uac1c\uc758 \uad6c\uc131\uad6d\uc73c\ub85c \uc774\ub8e8\uc5b4\uc838 \uc788\ub294 \uc5f0\ud569\uad6d\uac00\uc774\ub2e4. \uc218\ub3c4\ub294 \ub7f0\ub358 \uc774\uace0 \uc5f0\ud569\uad6d\uc758 \uad6c\uc131\uc744 \uc774\ub8e8\ub294 \uac01 \ub098\ub77c\ub4e4\uc740 \uc790\uce58\uad8c\uc744 \ubcf4\uc7a5\ubc1b\uace0 \uc788\ub2e4. \uc2a4\ucf54\ud2c0\ub79c\ub4dc, \uc6e8\uc77c\uc2a4, \ubd81\uc544\uc77c\ub79c\ub4dc\uc758 \uc218\ub3c4\ub294 \uac01\uac01 \uc5d0\ub4e0\ubc84\ub7ec, \uce74\ub514\ud504, \ubca8\ud30c\uc2a4\ud2b8\uc774\ub2e4.", "paragraph_answer": "\uadf8\ub808\uc774\ud2b8 \ube0c\ub9ac\ud2bc \ubd81\uc544\uc77c\ub79c\ub4dc \uc5f0\ud569\uc655\uad6d(-\u806f\u5408\u738b\u570b, \uc601\uc5b4: United Kingdom of Great Britain and Northern Ireland \uc720\ub098\uc774\ud2f0\ub4dc \ud0b9\ub364 \uc624\ube0c \uadf8\ub808\uc774\ud2b8 \ube0c\ub9ac\ud2bc \uc564 \ub178\ub358 \uc544\uc77c\ub79c\ub4dc, \ubb38\ud654\uc5b4: \ub300\ube0c\ub9ac\ud150 \ubc0f \ubd81\uc544\uc77c\ub79c\ub4dc \ub828\ud569\uc655\uad6d), \uc57d\uce6d \ube0c\ub9ac\ud2bc(\uc601\uc5b4: Britain) \ub610\ub294 \uc5f0\ud569\uc655\uad6d(\u806f\u5408\u738b\u570b, \uc601\uc5b4: United Kingdom \uc720\ub098\uc774\ud2f0\ub4dc \ud0b9\ub364, UK) \ud639\uc740 \uc601\uad6d(\u82f1\u570b)\uc740 \uc720\ub7fd \ubd81\uc11c\ubd80 \ud574\uc548\uc758 \ube0c\ub9ac\ud2bc \uc81c\ub3c4\uc5d0 \uc704\uce58\ud55c \uc8fc\uad8c\uad6d\uc774\uc790 \uc12c\ub098\ub77c\ub85c, \ubd81\ud574, \uc601\uad6d \ud574\ud611, \uc544\uc77c\ub79c\ub4dc \ud574 \ubc0f \ub300\uc11c\uc591\uc5d0 \uc811\ud558\uc5ec \uc788\uc73c\uba70 \uadf8\ub808\uc774\ud2b8 \ube0c\ub9ac\ud2bc \uc12c\uc758 \uc789\uae00\ub79c\ub4dc, \uc2a4\ucf54\ud2c0\ub79c\ub4dc, \uc6e8\uc77c\uc2a4 \ubc0f \uc544\uc77c\ub79c\ub4dc \uc12c \ubd81\ubd80\uc758 \ubd81\uc544\uc77c\ub79c\ub4dc\ub85c \ub124 \uac1c\uc758 \uad6c\uc131\uad6d\uc73c\ub85c \uc774\ub8e8\uc5b4\uc838 \uc788\ub294 \uc5f0\ud569\uad6d\uac00\uc774\ub2e4. \uc218\ub3c4\ub294 \ub7f0\ub358 \uc774\uace0 \uc5f0\ud569\uad6d\uc758 \uad6c\uc131\uc744 \uc774\ub8e8\ub294 \uac01 \ub098\ub77c\ub4e4\uc740 \uc790\uce58\uad8c\uc744 \ubcf4\uc7a5\ubc1b\uace0 \uc788\ub2e4. \uc2a4\ucf54\ud2c0\ub79c\ub4dc, \uc6e8\uc77c\uc2a4, \ubd81\uc544\uc77c\ub79c\ub4dc\uc758 \uc218\ub3c4\ub294 \uac01\uac01 \uc5d0\ub4e0\ubc84\ub7ec, \uce74\ub514\ud504, \ubca8\ud30c\uc2a4\ud2b8\uc774\ub2e4.", "sentence_answer": "\uc218\ub3c4\ub294 \ub7f0\ub358 \uc774\uace0 \uc5f0\ud569\uad6d\uc758 \uad6c\uc131\uc744 \uc774\ub8e8\ub294 \uac01 \ub098\ub77c\ub4e4\uc740 \uc790\uce58\uad8c\uc744 \ubcf4\uc7a5\ubc1b\uace0 \uc788\ub2e4.", "paragraph_id": "6458616-0-0", "paragraph_question": "question: \uc601\uad6d\uc758 \uc218\ub3c4\ub294?, context: \uadf8\ub808\uc774\ud2b8 \ube0c\ub9ac\ud2bc \ubd81\uc544\uc77c\ub79c\ub4dc \uc5f0\ud569\uc655\uad6d(-\u806f\u5408\u738b\u570b, \uc601\uc5b4: United Kingdom of Great Britain and Northern Ireland \uc720\ub098\uc774\ud2f0\ub4dc \ud0b9\ub364 \uc624\ube0c \uadf8\ub808\uc774\ud2b8 \ube0c\ub9ac\ud2bc \uc564 \ub178\ub358 \uc544\uc77c\ub79c\ub4dc, \ubb38\ud654\uc5b4: \ub300\ube0c\ub9ac\ud150 \ubc0f \ubd81\uc544\uc77c\ub79c\ub4dc \ub828\ud569\uc655\uad6d), \uc57d\uce6d \ube0c\ub9ac\ud2bc(\uc601\uc5b4: Britain) \ub610\ub294 \uc5f0\ud569\uc655\uad6d(\u806f\u5408\u738b\u570b, \uc601\uc5b4: United Kingdom \uc720\ub098\uc774\ud2f0\ub4dc \ud0b9\ub364, UK) \ud639\uc740 \uc601\uad6d(\u82f1\u570b)\uc740 \uc720\ub7fd \ubd81\uc11c\ubd80 \ud574\uc548\uc758 \ube0c\ub9ac\ud2bc \uc81c\ub3c4\uc5d0 \uc704\uce58\ud55c \uc8fc\uad8c\uad6d\uc774\uc790 \uc12c\ub098\ub77c\ub85c, \ubd81\ud574, \uc601\uad6d \ud574\ud611, \uc544\uc77c\ub79c\ub4dc \ud574 \ubc0f \ub300\uc11c\uc591\uc5d0 \uc811\ud558\uc5ec \uc788\uc73c\uba70 \uadf8\ub808\uc774\ud2b8 \ube0c\ub9ac\ud2bc \uc12c\uc758 \uc789\uae00\ub79c\ub4dc, \uc2a4\ucf54\ud2c0\ub79c\ub4dc, \uc6e8\uc77c\uc2a4 \ubc0f \uc544\uc77c\ub79c\ub4dc \uc12c \ubd81\ubd80\uc758 \ubd81\uc544\uc77c\ub79c\ub4dc\ub85c \ub124 \uac1c\uc758 \uad6c\uc131\uad6d\uc73c\ub85c \uc774\ub8e8\uc5b4\uc838 \uc788\ub294 \uc5f0\ud569\uad6d\uac00\uc774\ub2e4. \uc218\ub3c4\ub294 \ub7f0\ub358\uc774\uace0 \uc5f0\ud569\uad6d\uc758 \uad6c\uc131\uc744 \uc774\ub8e8\ub294 \uac01 \ub098\ub77c\ub4e4\uc740 \uc790\uce58\uad8c\uc744 \ubcf4\uc7a5\ubc1b\uace0 \uc788\ub2e4. \uc2a4\ucf54\ud2c0\ub79c\ub4dc, \uc6e8\uc77c\uc2a4, \ubd81\uc544\uc77c\ub79c\ub4dc\uc758 \uc218\ub3c4\ub294 \uac01\uac01 \uc5d0\ub4e0\ubc84\ub7ec, \uce74\ub514\ud504, \ubca8\ud30c\uc2a4\ud2b8\uc774\ub2e4."} {"question": "\ub450\ud23d\uc0c1\uc5b4\uc758 \ucc9c\uc801\uc740 \ubb34\uc5c7\uc778\uac00\uc694?", "paragraph": "\ub450\ud23d\uc0c1\uc5b4\ub294 \uc8fc\ub85c \uc5f0\uccb4\ub3d9\ubb3c\uc744 \uba39\uc73c\uba70, \uadf8 \uc678\uc5d0 \uac11\uac01\ub958, \uacbd\uace8\uc5b4\ub958\ub3c4 \uba39\ub294\ub2e4. \ub450\ud23d\uc0c1\uc5b4\uc758 \ucc9c\uc801\uc740 \ubcf5\uc0c1\uc5b4(Cephaloscyllium umbratile)\uc774\ub2e4. \ubcf5\uc0c1\uc5b4\ub294 \ub450\ud23d\uc0c1\uc5b4 \uac1c\uccb4 \ubfd0 \uc544\ub2c8\ub77c \uadf8 \uc54c\uc9d1\ub3c4 \uba39\ub294\ub2e4. \ub450\ud23d\uc0c1\uc5b4\uc5d0 \uae30\uc0dd\ud558\ub294 \uae30\uc0dd\ucda9\uc73c\ub85c\ub294 \ud074\ub85c\ub85c\ubbf9\uc228 \uc2a4\ud0ac\ub9ac\uc624\ub974\ud788\ub214(Chloromyxum scyliorhinum)\uc774\ub77c\ub294 \uc810\uc561\ud3ec\uc790\ucda9\uc774 \uc788\ub2e4. \uc0dd\uc2dd \ubc29\ubc95\uc740 \ub09c\uc0dd\uc774\ub2e4. \uc131\uc219\ud55c \uc554\ucef7\uc740 \ub09c\uc18c\uac00 \ud55c \uac1c, \ub09c\uad00\uc774 \ub450 \uac1c \uc788\ub2e4. \uad50\ubbf8\ub97c \ud558\uae30 \uc804\uc5d0 \uc218\ucef7\uc740 \uc554\ucef7\uc758 \uac00\uc2b4\uc9c0\ub290\ub7ec\ubbf8, \uc606\uba74, \uc544\uac00\ubbf8\ub97c \ubb3c\uc5b4\ub72f\ub294\ub2e4. \uadf8 \ub4a4 \uc790\uae30 \ubab8\uc73c\ub85c \uc554\ucef7\uc744 \uac10\uc2f8\uace0 \uad50\ubbf8\uae30\ub97c \ucd1d\ubc30\uc124\uac15\uc5d0 \uc0bd\uc785\ud55c\ub2e4. \ud758\ub808 \uc9c0\uc18d\uc2dc\uac04\uc740 15\ucd08 ~ 4\ubd84 \uc0ac\uc774\uc774\ub2e4. \uc218\ucef7\uc758 \uad50\ubbf8\uae30\ub294 \uc548\ucabd \uac00\uc7a5\uc790\ub9ac\ub97c \ub530\ub77c \ubc31\uc5ec \uac1c\uc758 \uac08\uace0\ub9ac\uac00 \ud55c \uc904\ub85c \ub3cb\uc544 \uc788\ub2e4. \uc774 \uac08\uace0\ub9ac\ub294 \ud758\ub808\ub97c \ubd99\ub294 \ub3c4\uc911\uc5d0 \uc554\uc218\uc758 \uacb0\ud569\uc744 \uc720\uc9c0\uc2dc\ud0a4\ub294 \uc5ed\ud560\uc744 \ud55c\ub2e4. \uc554\ucef7\uc740 \uc815\uc561\uc744 \ub09c\ud3ec\uc120\uc5d0 \uc218 \uac1c\uc6d4\uac04 \uc800\uc7a5\ud560 \uc218 \uc788\ub2e4.", "answer": "\ubcf5\uc0c1\uc5b4", "sentence": "\ub450\ud23d\uc0c1\uc5b4\uc758 \ucc9c\uc801\uc740 \ubcf5\uc0c1\uc5b4 (Cephaloscyllium umbratile)\uc774\ub2e4.", "paragraph_sentence": "\ub450\ud23d\uc0c1\uc5b4\ub294 \uc8fc\ub85c \uc5f0\uccb4\ub3d9\ubb3c\uc744 \uba39\uc73c\uba70, \uadf8 \uc678\uc5d0 \uac11\uac01\ub958, \uacbd\uace8\uc5b4\ub958\ub3c4 \uba39\ub294\ub2e4. \ub450\ud23d\uc0c1\uc5b4\uc758 \ucc9c\uc801\uc740 \ubcf5\uc0c1\uc5b4 (Cephaloscyllium umbratile)\uc774\ub2e4. \ubcf5\uc0c1\uc5b4\ub294 \ub450\ud23d\uc0c1\uc5b4 \uac1c\uccb4 \ubfd0 \uc544\ub2c8\ub77c \uadf8 \uc54c\uc9d1\ub3c4 \uba39\ub294\ub2e4. \ub450\ud23d\uc0c1\uc5b4\uc5d0 \uae30\uc0dd\ud558\ub294 \uae30\uc0dd\ucda9\uc73c\ub85c\ub294 \ud074\ub85c\ub85c\ubbf9\uc228 \uc2a4\ud0ac\ub9ac\uc624\ub974\ud788\ub214(Chloromyxum scyliorhinum)\uc774\ub77c\ub294 \uc810\uc561\ud3ec\uc790\ucda9\uc774 \uc788\ub2e4. \uc0dd\uc2dd \ubc29\ubc95\uc740 \ub09c\uc0dd\uc774\ub2e4. \uc131\uc219\ud55c \uc554\ucef7\uc740 \ub09c\uc18c\uac00 \ud55c \uac1c, \ub09c\uad00\uc774 \ub450 \uac1c \uc788\ub2e4. \uad50\ubbf8\ub97c \ud558\uae30 \uc804\uc5d0 \uc218\ucef7\uc740 \uc554\ucef7\uc758 \uac00\uc2b4\uc9c0\ub290\ub7ec\ubbf8, \uc606\uba74, \uc544\uac00\ubbf8\ub97c \ubb3c\uc5b4\ub72f\ub294\ub2e4. \uadf8 \ub4a4 \uc790\uae30 \ubab8\uc73c\ub85c \uc554\ucef7\uc744 \uac10\uc2f8\uace0 \uad50\ubbf8\uae30\ub97c \ucd1d\ubc30\uc124\uac15\uc5d0 \uc0bd\uc785\ud55c\ub2e4. \ud758\ub808 \uc9c0\uc18d\uc2dc\uac04\uc740 15\ucd08 ~ 4\ubd84 \uc0ac\uc774\uc774\ub2e4. \uc218\ucef7\uc758 \uad50\ubbf8\uae30\ub294 \uc548\ucabd \uac00\uc7a5\uc790\ub9ac\ub97c \ub530\ub77c \ubc31\uc5ec \uac1c\uc758 \uac08\uace0\ub9ac\uac00 \ud55c \uc904\ub85c \ub3cb\uc544 \uc788\ub2e4. \uc774 \uac08\uace0\ub9ac\ub294 \ud758\ub808\ub97c \ubd99\ub294 \ub3c4\uc911\uc5d0 \uc554\uc218\uc758 \uacb0\ud569\uc744 \uc720\uc9c0\uc2dc\ud0a4\ub294 \uc5ed\ud560\uc744 \ud55c\ub2e4. \uc554\ucef7\uc740 \uc815\uc561\uc744 \ub09c\ud3ec\uc120\uc5d0 \uc218 \uac1c\uc6d4\uac04 \uc800\uc7a5\ud560 \uc218 \uc788\ub2e4.", "paragraph_answer": "\ub450\ud23d\uc0c1\uc5b4\ub294 \uc8fc\ub85c \uc5f0\uccb4\ub3d9\ubb3c\uc744 \uba39\uc73c\uba70, \uadf8 \uc678\uc5d0 \uac11\uac01\ub958, \uacbd\uace8\uc5b4\ub958\ub3c4 \uba39\ub294\ub2e4. \ub450\ud23d\uc0c1\uc5b4\uc758 \ucc9c\uc801\uc740 \ubcf5\uc0c1\uc5b4 (Cephaloscyllium umbratile)\uc774\ub2e4. \ubcf5\uc0c1\uc5b4\ub294 \ub450\ud23d\uc0c1\uc5b4 \uac1c\uccb4 \ubfd0 \uc544\ub2c8\ub77c \uadf8 \uc54c\uc9d1\ub3c4 \uba39\ub294\ub2e4. \ub450\ud23d\uc0c1\uc5b4\uc5d0 \uae30\uc0dd\ud558\ub294 \uae30\uc0dd\ucda9\uc73c\ub85c\ub294 \ud074\ub85c\ub85c\ubbf9\uc228 \uc2a4\ud0ac\ub9ac\uc624\ub974\ud788\ub214(Chloromyxum scyliorhinum)\uc774\ub77c\ub294 \uc810\uc561\ud3ec\uc790\ucda9\uc774 \uc788\ub2e4. \uc0dd\uc2dd \ubc29\ubc95\uc740 \ub09c\uc0dd\uc774\ub2e4. \uc131\uc219\ud55c \uc554\ucef7\uc740 \ub09c\uc18c\uac00 \ud55c \uac1c, \ub09c\uad00\uc774 \ub450 \uac1c \uc788\ub2e4. \uad50\ubbf8\ub97c \ud558\uae30 \uc804\uc5d0 \uc218\ucef7\uc740 \uc554\ucef7\uc758 \uac00\uc2b4\uc9c0\ub290\ub7ec\ubbf8, \uc606\uba74, \uc544\uac00\ubbf8\ub97c \ubb3c\uc5b4\ub72f\ub294\ub2e4. \uadf8 \ub4a4 \uc790\uae30 \ubab8\uc73c\ub85c \uc554\ucef7\uc744 \uac10\uc2f8\uace0 \uad50\ubbf8\uae30\ub97c \ucd1d\ubc30\uc124\uac15\uc5d0 \uc0bd\uc785\ud55c\ub2e4. \ud758\ub808 \uc9c0\uc18d\uc2dc\uac04\uc740 15\ucd08 ~ 4\ubd84 \uc0ac\uc774\uc774\ub2e4. \uc218\ucef7\uc758 \uad50\ubbf8\uae30\ub294 \uc548\ucabd \uac00\uc7a5\uc790\ub9ac\ub97c \ub530\ub77c \ubc31\uc5ec \uac1c\uc758 \uac08\uace0\ub9ac\uac00 \ud55c \uc904\ub85c \ub3cb\uc544 \uc788\ub2e4. \uc774 \uac08\uace0\ub9ac\ub294 \ud758\ub808\ub97c \ubd99\ub294 \ub3c4\uc911\uc5d0 \uc554\uc218\uc758 \uacb0\ud569\uc744 \uc720\uc9c0\uc2dc\ud0a4\ub294 \uc5ed\ud560\uc744 \ud55c\ub2e4. \uc554\ucef7\uc740 \uc815\uc561\uc744 \ub09c\ud3ec\uc120\uc5d0 \uc218 \uac1c\uc6d4\uac04 \uc800\uc7a5\ud560 \uc218 \uc788\ub2e4.", "sentence_answer": "\ub450\ud23d\uc0c1\uc5b4\uc758 \ucc9c\uc801\uc740 \ubcf5\uc0c1\uc5b4 (Cephaloscyllium umbratile)\uc774\ub2e4.", "paragraph_id": "6492531-2-0", "paragraph_question": "question: \ub450\ud23d\uc0c1\uc5b4\uc758 \ucc9c\uc801\uc740 \ubb34\uc5c7\uc778\uac00\uc694?, context: \ub450\ud23d\uc0c1\uc5b4\ub294 \uc8fc\ub85c \uc5f0\uccb4\ub3d9\ubb3c\uc744 \uba39\uc73c\uba70, \uadf8 \uc678\uc5d0 \uac11\uac01\ub958, \uacbd\uace8\uc5b4\ub958\ub3c4 \uba39\ub294\ub2e4. \ub450\ud23d\uc0c1\uc5b4\uc758 \ucc9c\uc801\uc740 \ubcf5\uc0c1\uc5b4(Cephaloscyllium umbratile)\uc774\ub2e4. \ubcf5\uc0c1\uc5b4\ub294 \ub450\ud23d\uc0c1\uc5b4 \uac1c\uccb4 \ubfd0 \uc544\ub2c8\ub77c \uadf8 \uc54c\uc9d1\ub3c4 \uba39\ub294\ub2e4. \ub450\ud23d\uc0c1\uc5b4\uc5d0 \uae30\uc0dd\ud558\ub294 \uae30\uc0dd\ucda9\uc73c\ub85c\ub294 \ud074\ub85c\ub85c\ubbf9\uc228 \uc2a4\ud0ac\ub9ac\uc624\ub974\ud788\ub214(Chloromyxum scyliorhinum)\uc774\ub77c\ub294 \uc810\uc561\ud3ec\uc790\ucda9\uc774 \uc788\ub2e4. \uc0dd\uc2dd \ubc29\ubc95\uc740 \ub09c\uc0dd\uc774\ub2e4. \uc131\uc219\ud55c \uc554\ucef7\uc740 \ub09c\uc18c\uac00 \ud55c \uac1c, \ub09c\uad00\uc774 \ub450 \uac1c \uc788\ub2e4. \uad50\ubbf8\ub97c \ud558\uae30 \uc804\uc5d0 \uc218\ucef7\uc740 \uc554\ucef7\uc758 \uac00\uc2b4\uc9c0\ub290\ub7ec\ubbf8, \uc606\uba74, \uc544\uac00\ubbf8\ub97c \ubb3c\uc5b4\ub72f\ub294\ub2e4. \uadf8 \ub4a4 \uc790\uae30 \ubab8\uc73c\ub85c \uc554\ucef7\uc744 \uac10\uc2f8\uace0 \uad50\ubbf8\uae30\ub97c \ucd1d\ubc30\uc124\uac15\uc5d0 \uc0bd\uc785\ud55c\ub2e4. \ud758\ub808 \uc9c0\uc18d\uc2dc\uac04\uc740 15\ucd08 ~ 4\ubd84 \uc0ac\uc774\uc774\ub2e4. \uc218\ucef7\uc758 \uad50\ubbf8\uae30\ub294 \uc548\ucabd \uac00\uc7a5\uc790\ub9ac\ub97c \ub530\ub77c \ubc31\uc5ec \uac1c\uc758 \uac08\uace0\ub9ac\uac00 \ud55c \uc904\ub85c \ub3cb\uc544 \uc788\ub2e4. \uc774 \uac08\uace0\ub9ac\ub294 \ud758\ub808\ub97c \ubd99\ub294 \ub3c4\uc911\uc5d0 \uc554\uc218\uc758 \uacb0\ud569\uc744 \uc720\uc9c0\uc2dc\ud0a4\ub294 \uc5ed\ud560\uc744 \ud55c\ub2e4. \uc554\ucef7\uc740 \uc815\uc561\uc744 \ub09c\ud3ec\uc120\uc5d0 \uc218 \uac1c\uc6d4\uac04 \uc800\uc7a5\ud560 \uc218 \uc788\ub2e4."} {"question": "\uc601\ud654 \uc139\uc2a4 \uc564 \ub354 \uc2dc\ud2f0 2\uc5d0\uc11c \ucee4\ubc84 \uacf5\uc5f0\uc744 \ud55c \uac00\uc218\uc774\uc790 \ubc30\uc6b0\ub294?", "paragraph": "2010\ub144 9\uc6d4 26\uc77c \u300a\ub354 \uc5d1\uc2a4\ud329\ud130 \uc624\uc2a4\ud2b8\ub808\uc77c\ub9ac\uc544\u300b \uc2dc\uc98c 2 \ucc38\uac00\uc790 \uc5ec\uc131 \ub4c0\uc624 \uce74\ub9ac\uc988\ub9c8\ub294 \uc790\uc2e0\ub4e4\ub9cc\uc758 \ubc84\uc804\uc73c\ub85c \ubc14\uafd4 \ubd88\ub800\uace0, 2011\ub144 5\uc6d4 31\uc77c\uc5d0\ub294 \u300a\ub354 \uc5d1\uc2a4\ud329\ud130 \ud504\ub791\uc2a4\u300b \uc2dc\uc98c2 \ucc38\uac00\uc790 \ub9e4\ud29c \ub808\uc774\uba3c\ub4dc \ubc14\ucee4\uac00 \ub178\ub798\ub97c \ubd88\ub800\ub2e4. 2011\ub144 5\uc6d4 25\uc77c \u300a\uc544\uba54\ub9ac\uce78 \uc544\uc774\ub3cc\u300b \uc2dc\uc98c 10\uc758 \ub9c8\uc9c0\ub9c9 \ub77c\uc6b4\ub4dc\uc5d0\uc11c \uc5ec\uc131 \ucc38\uac00\uc790\ub4e4\uc774 \u3008Single Ladies\u3009 \uacf5\uc5f0\uc744 \ud3bc\ucce4\ub2e4. \u3008Single Ladies\u3009\ub294 \uc601\ud654 \u300a\uc568\ube48\uacfc \uc288\ud37c\ubc34\ub4dc 2\u300b\uc5d0 \ub098\uc624\ub294 \ub2e4\ub78c\uc950 \uc5ec\uac00\uc218 \ub354 \uce58\ud3ab\uc2a4\uc5d0 \uc758\ud574 \ucee4\ubc84\ub418\uc5c8\uace0 \uc601\ud654\uc758 \uc0ac\uc6b4\ub4dc\ud2b8\ub799\uc5d0\ub3c4 \uc218\ub85d\ub418\uc5c8\ub2e4. \uce58\ud3ab\uc2a4\ub294 \uc138 \ub9c8\ub9ac\uc758 \ub2e4\ub78c\uc950\ub85c \uc774\ub8e8\uc5b4\uc838 \uc788\ub294\ub370, \uc81c\ub2c8\uc2a4 \uce74\ub9e8\uc774 \ubaa8\ub4e0 \ubd80\ubd84\uc744 \ub179\uc74c\ud588\ub2e4. \uce58\ud3ab\uc2a4\ub294 \uc601\ud654\uc5d0\uc11c \ubcf4\ube0c \ud3ec\uc2a4 \uc2a4\ud0c0\uc77c\uc758 \uc548\ubb34\ub97c \ubcf4\uc5ec\uc92c\ub2e4. \uc57c\ud6c4! \ubba4\uc9c1\uc758 \ube4c\ub9ac \uc874\uc2a8\uc740 \ucc98\uc74c \uc601\ud654\ub97c \ubd23\uc744\ub54c \"\ucd5c\uace0\uc758 'Single Ladies' \ud328\ub7ec\ub514 \uc911 \ud558\ub098\uc784\uc774 \ud2c0\ub9bc\uc5c6\ub2e4. \ub2e4\ub78c\uc950\ub4e4\uc774 \ub178\ub798\ub97c \ubd88\ub800\ub2e4\ub294 \uac83\ub9cc\uc73c\ub85c\ub3c4 \uac00\uce58 \uc788\ub2e4\"\uace0 \ub9d0\ud588\ub2e4. \uc601\ud654 \u300a\uc139\uc2a4 \uc564 \ub354 \uc2dc\ud2f0 2\u300b\uc5d0\uc11c\ub294 \ubbf8\uad6d\uc758 \uac00\uc218\uc774\uc790 \ubc30\uc6b0 \ub77c\uc774\uc790 \ubbf8\ub12c\ub9ac\uac00 \ucee4\ubc84 \uacf5\uc5f0\uc744 \ud588\ub2e4. 2011\ub144 10\uc6d4 18\uc77c \uc601 \ub9e8 \uc18c\uc0ac\uc774\uc5b4\ud2f0\ub294 \u300a\ub354 \uc5d1\uc2a4\ud329\ud130 \uc624\uc2a4\ud2b8\ub808\uc77c\ub9ac\uc544\u300b \uc2dc\uc98c 3\uc5d0\uc11c \u3008Single Ladies\u3009\ub97c \ubd88\ub800\ub2e4.", "answer": "\ub77c\uc774\uc790 \ubbf8\ub12c\ub9ac", "sentence": "\uc601\ud654 \u300a\uc139\uc2a4 \uc564 \ub354 \uc2dc\ud2f0 2\u300b\uc5d0\uc11c\ub294 \ubbf8\uad6d\uc758 \uac00\uc218\uc774\uc790 \ubc30\uc6b0 \ub77c\uc774\uc790 \ubbf8\ub12c\ub9ac \uac00 \ucee4\ubc84 \uacf5\uc5f0\uc744 \ud588\ub2e4.", "paragraph_sentence": "2010\ub144 9\uc6d4 26\uc77c \u300a\ub354 \uc5d1\uc2a4\ud329\ud130 \uc624\uc2a4\ud2b8\ub808\uc77c\ub9ac\uc544\u300b \uc2dc\uc98c 2 \ucc38\uac00\uc790 \uc5ec\uc131 \ub4c0\uc624 \uce74\ub9ac\uc988\ub9c8\ub294 \uc790\uc2e0\ub4e4\ub9cc\uc758 \ubc84\uc804\uc73c\ub85c \ubc14\uafd4 \ubd88\ub800\uace0, 2011\ub144 5\uc6d4 31\uc77c\uc5d0\ub294 \u300a\ub354 \uc5d1\uc2a4\ud329\ud130 \ud504\ub791\uc2a4\u300b \uc2dc\uc98c2 \ucc38\uac00\uc790 \ub9e4\ud29c \ub808\uc774\uba3c\ub4dc \ubc14\ucee4\uac00 \ub178\ub798\ub97c \ubd88\ub800\ub2e4. 2011\ub144 5\uc6d4 25\uc77c \u300a\uc544\uba54\ub9ac\uce78 \uc544\uc774\ub3cc\u300b \uc2dc\uc98c 10\uc758 \ub9c8\uc9c0\ub9c9 \ub77c\uc6b4\ub4dc\uc5d0\uc11c \uc5ec\uc131 \ucc38\uac00\uc790\ub4e4\uc774 \u3008Single Ladies\u3009 \uacf5\uc5f0\uc744 \ud3bc\ucce4\ub2e4. \u3008Single Ladies\u3009\ub294 \uc601\ud654 \u300a\uc568\ube48\uacfc \uc288\ud37c\ubc34\ub4dc 2\u300b\uc5d0 \ub098\uc624\ub294 \ub2e4\ub78c\uc950 \uc5ec\uac00\uc218 \ub354 \uce58\ud3ab\uc2a4\uc5d0 \uc758\ud574 \ucee4\ubc84\ub418\uc5c8\uace0 \uc601\ud654\uc758 \uc0ac\uc6b4\ub4dc\ud2b8\ub799\uc5d0\ub3c4 \uc218\ub85d\ub418\uc5c8\ub2e4. \uce58\ud3ab\uc2a4\ub294 \uc138 \ub9c8\ub9ac\uc758 \ub2e4\ub78c\uc950\ub85c \uc774\ub8e8\uc5b4\uc838 \uc788\ub294\ub370, \uc81c\ub2c8\uc2a4 \uce74\ub9e8\uc774 \ubaa8\ub4e0 \ubd80\ubd84\uc744 \ub179\uc74c\ud588\ub2e4. \uce58\ud3ab\uc2a4\ub294 \uc601\ud654\uc5d0\uc11c \ubcf4\ube0c \ud3ec\uc2a4 \uc2a4\ud0c0\uc77c\uc758 \uc548\ubb34\ub97c \ubcf4\uc5ec\uc92c\ub2e4. \uc57c\ud6c4! \ubba4\uc9c1\uc758 \ube4c\ub9ac \uc874\uc2a8\uc740 \ucc98\uc74c \uc601\ud654\ub97c \ubd23\uc744\ub54c \"\ucd5c\uace0\uc758 'Single Ladies' \ud328\ub7ec\ub514 \uc911 \ud558\ub098\uc784\uc774 \ud2c0\ub9bc\uc5c6\ub2e4. \ub2e4\ub78c\uc950\ub4e4\uc774 \ub178\ub798\ub97c \ubd88\ub800\ub2e4\ub294 \uac83\ub9cc\uc73c\ub85c\ub3c4 \uac00\uce58 \uc788\ub2e4\"\uace0 \ub9d0\ud588\ub2e4. \uc601\ud654 \u300a\uc139\uc2a4 \uc564 \ub354 \uc2dc\ud2f0 2\u300b\uc5d0\uc11c\ub294 \ubbf8\uad6d\uc758 \uac00\uc218\uc774\uc790 \ubc30\uc6b0 \ub77c\uc774\uc790 \ubbf8\ub12c\ub9ac \uac00 \ucee4\ubc84 \uacf5\uc5f0\uc744 \ud588\ub2e4. 2011\ub144 10\uc6d4 18\uc77c \uc601 \ub9e8 \uc18c\uc0ac\uc774\uc5b4\ud2f0\ub294 \u300a\ub354 \uc5d1\uc2a4\ud329\ud130 \uc624\uc2a4\ud2b8\ub808\uc77c\ub9ac\uc544\u300b \uc2dc\uc98c 3\uc5d0\uc11c \u3008Single Ladies\u3009\ub97c \ubd88\ub800\ub2e4.", "paragraph_answer": "2010\ub144 9\uc6d4 26\uc77c \u300a\ub354 \uc5d1\uc2a4\ud329\ud130 \uc624\uc2a4\ud2b8\ub808\uc77c\ub9ac\uc544\u300b \uc2dc\uc98c 2 \ucc38\uac00\uc790 \uc5ec\uc131 \ub4c0\uc624 \uce74\ub9ac\uc988\ub9c8\ub294 \uc790\uc2e0\ub4e4\ub9cc\uc758 \ubc84\uc804\uc73c\ub85c \ubc14\uafd4 \ubd88\ub800\uace0, 2011\ub144 5\uc6d4 31\uc77c\uc5d0\ub294 \u300a\ub354 \uc5d1\uc2a4\ud329\ud130 \ud504\ub791\uc2a4\u300b \uc2dc\uc98c2 \ucc38\uac00\uc790 \ub9e4\ud29c \ub808\uc774\uba3c\ub4dc \ubc14\ucee4\uac00 \ub178\ub798\ub97c \ubd88\ub800\ub2e4. 2011\ub144 5\uc6d4 25\uc77c \u300a\uc544\uba54\ub9ac\uce78 \uc544\uc774\ub3cc\u300b \uc2dc\uc98c 10\uc758 \ub9c8\uc9c0\ub9c9 \ub77c\uc6b4\ub4dc\uc5d0\uc11c \uc5ec\uc131 \ucc38\uac00\uc790\ub4e4\uc774 \u3008Single Ladies\u3009 \uacf5\uc5f0\uc744 \ud3bc\ucce4\ub2e4. \u3008Single Ladies\u3009\ub294 \uc601\ud654 \u300a\uc568\ube48\uacfc \uc288\ud37c\ubc34\ub4dc 2\u300b\uc5d0 \ub098\uc624\ub294 \ub2e4\ub78c\uc950 \uc5ec\uac00\uc218 \ub354 \uce58\ud3ab\uc2a4\uc5d0 \uc758\ud574 \ucee4\ubc84\ub418\uc5c8\uace0 \uc601\ud654\uc758 \uc0ac\uc6b4\ub4dc\ud2b8\ub799\uc5d0\ub3c4 \uc218\ub85d\ub418\uc5c8\ub2e4. \uce58\ud3ab\uc2a4\ub294 \uc138 \ub9c8\ub9ac\uc758 \ub2e4\ub78c\uc950\ub85c \uc774\ub8e8\uc5b4\uc838 \uc788\ub294\ub370, \uc81c\ub2c8\uc2a4 \uce74\ub9e8\uc774 \ubaa8\ub4e0 \ubd80\ubd84\uc744 \ub179\uc74c\ud588\ub2e4. \uce58\ud3ab\uc2a4\ub294 \uc601\ud654\uc5d0\uc11c \ubcf4\ube0c \ud3ec\uc2a4 \uc2a4\ud0c0\uc77c\uc758 \uc548\ubb34\ub97c \ubcf4\uc5ec\uc92c\ub2e4. \uc57c\ud6c4! \ubba4\uc9c1\uc758 \ube4c\ub9ac \uc874\uc2a8\uc740 \ucc98\uc74c \uc601\ud654\ub97c \ubd23\uc744\ub54c \"\ucd5c\uace0\uc758 'Single Ladies' \ud328\ub7ec\ub514 \uc911 \ud558\ub098\uc784\uc774 \ud2c0\ub9bc\uc5c6\ub2e4. \ub2e4\ub78c\uc950\ub4e4\uc774 \ub178\ub798\ub97c \ubd88\ub800\ub2e4\ub294 \uac83\ub9cc\uc73c\ub85c\ub3c4 \uac00\uce58 \uc788\ub2e4\"\uace0 \ub9d0\ud588\ub2e4. \uc601\ud654 \u300a\uc139\uc2a4 \uc564 \ub354 \uc2dc\ud2f0 2\u300b\uc5d0\uc11c\ub294 \ubbf8\uad6d\uc758 \uac00\uc218\uc774\uc790 \ubc30\uc6b0 \ub77c\uc774\uc790 \ubbf8\ub12c\ub9ac \uac00 \ucee4\ubc84 \uacf5\uc5f0\uc744 \ud588\ub2e4. 2011\ub144 10\uc6d4 18\uc77c \uc601 \ub9e8 \uc18c\uc0ac\uc774\uc5b4\ud2f0\ub294 \u300a\ub354 \uc5d1\uc2a4\ud329\ud130 \uc624\uc2a4\ud2b8\ub808\uc77c\ub9ac\uc544\u300b \uc2dc\uc98c 3\uc5d0\uc11c \u3008Single Ladies\u3009\ub97c \ubd88\ub800\ub2e4.", "sentence_answer": "\uc601\ud654 \u300a\uc139\uc2a4 \uc564 \ub354 \uc2dc\ud2f0 2\u300b\uc5d0\uc11c\ub294 \ubbf8\uad6d\uc758 \uac00\uc218\uc774\uc790 \ubc30\uc6b0 \ub77c\uc774\uc790 \ubbf8\ub12c\ub9ac \uac00 \ucee4\ubc84 \uacf5\uc5f0\uc744 \ud588\ub2e4.", "paragraph_id": "5808934-32-0", "paragraph_question": "question: \uc601\ud654 \uc139\uc2a4 \uc564 \ub354 \uc2dc\ud2f0 2\uc5d0\uc11c \ucee4\ubc84 \uacf5\uc5f0\uc744 \ud55c \uac00\uc218\uc774\uc790 \ubc30\uc6b0\ub294?, context: 2010\ub144 9\uc6d4 26\uc77c \u300a\ub354 \uc5d1\uc2a4\ud329\ud130 \uc624\uc2a4\ud2b8\ub808\uc77c\ub9ac\uc544\u300b \uc2dc\uc98c 2 \ucc38\uac00\uc790 \uc5ec\uc131 \ub4c0\uc624 \uce74\ub9ac\uc988\ub9c8\ub294 \uc790\uc2e0\ub4e4\ub9cc\uc758 \ubc84\uc804\uc73c\ub85c \ubc14\uafd4 \ubd88\ub800\uace0, 2011\ub144 5\uc6d4 31\uc77c\uc5d0\ub294 \u300a\ub354 \uc5d1\uc2a4\ud329\ud130 \ud504\ub791\uc2a4\u300b \uc2dc\uc98c2 \ucc38\uac00\uc790 \ub9e4\ud29c \ub808\uc774\uba3c\ub4dc \ubc14\ucee4\uac00 \ub178\ub798\ub97c \ubd88\ub800\ub2e4. 2011\ub144 5\uc6d4 25\uc77c \u300a\uc544\uba54\ub9ac\uce78 \uc544\uc774\ub3cc\u300b \uc2dc\uc98c 10\uc758 \ub9c8\uc9c0\ub9c9 \ub77c\uc6b4\ub4dc\uc5d0\uc11c \uc5ec\uc131 \ucc38\uac00\uc790\ub4e4\uc774 \u3008Single Ladies\u3009 \uacf5\uc5f0\uc744 \ud3bc\ucce4\ub2e4. \u3008Single Ladies\u3009\ub294 \uc601\ud654 \u300a\uc568\ube48\uacfc \uc288\ud37c\ubc34\ub4dc 2\u300b\uc5d0 \ub098\uc624\ub294 \ub2e4\ub78c\uc950 \uc5ec\uac00\uc218 \ub354 \uce58\ud3ab\uc2a4\uc5d0 \uc758\ud574 \ucee4\ubc84\ub418\uc5c8\uace0 \uc601\ud654\uc758 \uc0ac\uc6b4\ub4dc\ud2b8\ub799\uc5d0\ub3c4 \uc218\ub85d\ub418\uc5c8\ub2e4. \uce58\ud3ab\uc2a4\ub294 \uc138 \ub9c8\ub9ac\uc758 \ub2e4\ub78c\uc950\ub85c \uc774\ub8e8\uc5b4\uc838 \uc788\ub294\ub370, \uc81c\ub2c8\uc2a4 \uce74\ub9e8\uc774 \ubaa8\ub4e0 \ubd80\ubd84\uc744 \ub179\uc74c\ud588\ub2e4. \uce58\ud3ab\uc2a4\ub294 \uc601\ud654\uc5d0\uc11c \ubcf4\ube0c \ud3ec\uc2a4 \uc2a4\ud0c0\uc77c\uc758 \uc548\ubb34\ub97c \ubcf4\uc5ec\uc92c\ub2e4. \uc57c\ud6c4! \ubba4\uc9c1\uc758 \ube4c\ub9ac \uc874\uc2a8\uc740 \ucc98\uc74c \uc601\ud654\ub97c \ubd23\uc744\ub54c \"\ucd5c\uace0\uc758 'Single Ladies' \ud328\ub7ec\ub514 \uc911 \ud558\ub098\uc784\uc774 \ud2c0\ub9bc\uc5c6\ub2e4. \ub2e4\ub78c\uc950\ub4e4\uc774 \ub178\ub798\ub97c \ubd88\ub800\ub2e4\ub294 \uac83\ub9cc\uc73c\ub85c\ub3c4 \uac00\uce58 \uc788\ub2e4\"\uace0 \ub9d0\ud588\ub2e4. \uc601\ud654 \u300a\uc139\uc2a4 \uc564 \ub354 \uc2dc\ud2f0 2\u300b\uc5d0\uc11c\ub294 \ubbf8\uad6d\uc758 \uac00\uc218\uc774\uc790 \ubc30\uc6b0 \ub77c\uc774\uc790 \ubbf8\ub12c\ub9ac\uac00 \ucee4\ubc84 \uacf5\uc5f0\uc744 \ud588\ub2e4. 2011\ub144 10\uc6d4 18\uc77c \uc601 \ub9e8 \uc18c\uc0ac\uc774\uc5b4\ud2f0\ub294 \u300a\ub354 \uc5d1\uc2a4\ud329\ud130 \uc624\uc2a4\ud2b8\ub808\uc77c\ub9ac\uc544\u300b \uc2dc\uc98c 3\uc5d0\uc11c \u3008Single Ladies\u3009\ub97c \ubd88\ub800\ub2e4."} {"question": "\uc81c2\ucc28 \uc0bc\ub450 \uc815\uce58\uc758 \ubd95\uad34 \ud6c4 \uc625\ud0c0\ube44\uc544\ub204\uc2a4\uac00 \ub300\uc678\uc801\uc73c\ub85c \ubd80\ud65c\uc2dc\ud0a8 \uac83\uc740?", "paragraph": "\uc81c2\ucc28 \uc0bc\ub450 \uc815\uce58\uc758 \ubd95\uad34 \ud6c4 \uc625\ud0c0\ube44\uc544\ub204\uc2a4\ub294 \ub300\uc678\uc801\uc73c\ub85c \ub85c\ub9c8 \uacf5\ud654\uc815\uc744 \ubd80\ud65c\uc2dc\ud0a4\uace0 \uc815\ubd80\uc5d0 \uad00\ud55c \uad8c\ud55c\uc740 \ub85c\ub9c8 \uc6d0\ub85c\uc6d0\uc5d0\uac8c \uc8fc\uc5c8\uc73c\ub098, \uc0ac\uc2e4\uc0c1 \uad8c\ub825\uc740 \uadf8\uc758 \uc190\uc5d0 \ub4e4\uc5b4\uac00\uac8c \ub418\uc5c8\ub2e4. \uc720\uc77c\ud55c \ud1b5\uce58\uc790\uac00 \ub2e4\uc2a4\ub9ac\uc9c0\ub9cc \ub300\uc678\uc801\uc73c\ub85c\ub294 \uacf5\ud654\uad6d \ud615\ud0dc\uc778 \uc815\uce58 \uccb4\uc81c\uc758 \uae30\ud2c0\uc744 \ub2e4\uc9c0\ub294 \ub370\ub294 \uc624\ub79c \uc2dc\uac04\uc774 \uac78\ub838\ub2e4. \uaecd\ub370\uae30\ub9cc \uacf5\ud654\uad6d\uc778 \uc774 \ub098\ub77c\ub294 \ud6d7\ub0a0 \ub85c\ub9c8 \uc81c\uad6d\uc73c\ub85c \ubd88\ub9b0\ub2e4. \ud669\uc81c\uad8c\uc740 \uc625\ud0c0\ube44\uc544\ub204\uc2a4 \uc774\uc804\uc5d0 \ub85c\ub9c8\ub97c \ud1b5\uce58\ud588\ub358 \uce74\uc774\uc0ac\ub974\uc640 \uc220\ub77c\uc758 \ub3c5\uc7ac\uad8c\uacfc\ub294 \uc804\ud600 \ub2ec\ub790\ub2e4. \uc625\ud0c0\ube44\uc544\ub204\uc2a4\ub294 \ub85c\ub9c8\uc758 \uc6d0\ub85c\uc6d0\uacfc \uc2dc\ubbfc\ub4e4\ub85c\ubd80\ud130 \u201c\ub3c5\uc7ac\uad8c\uc744 \ubd80\uc5ec\ubc1b\uc558\uc9c0\ub9cc\u201d \uac70\uc808\ud558\uc600\ub2e4. \ubc95\uc5d0 \ub530\ub974\uba74 \u2018\uc874\uc5c4\uc790\u2019(\uc544\uc6b0\uad6c\uc2a4\ud22c\uc2a4)\ub77c\ub294 \uce6d\ud638\ub97c \ubc1b\uc740 \uc625\ud0c0\ube44\uc544\ub204\uc2a4\uc5d0\uac8c \uc6d0\ub85c\uc6d0\uc740 \ud3c9\uc0dd \ub3d9\uc548 \uad8c\ub825\uc744 \uac00\uc9c0\ub3c4\ub85d \ud558\uc600\uace0 \u201c\ud638\ubbfc\uad00 \uad8c\ud55c\u201d(tribunitia potestas)\uc744 \uac00\uc84c\uc73c\uba70 \uae30\uc6d0\uc804 23\ub144\uae4c\uc9c0 \uc9d1\uc815\uad00\uc744 \uc5ed\uc784\ud558\uc600\ub2e4. \uc544\uc6b0\uad6c\uc2a4\ud22c\uc2a4\ub294 \uc7ac\uc815\uc801\uc778 \uc131\uacf5\uacfc \uc6d0\uc815\uc5d0\uc11c \uc5bb\uc740 \ubb3c\uc790, \uc81c\uad6d \uc804\uccb4\uc5d0 \uac78\uccd0 \ub9fa\uc740 \uc5ec\ub7ec \ud53c\ud638 \uad00\uacc4(clientela), \uad70\uc778\uacfc \uc7ac\ud5a5 \uad70\uc778\uc758 \ucda9\uc131, \uc6d0\ub85c\uc6d0\uc5d0\uc11c \ubd80\uc5ec\ud55c \uc5ec\ub7ec \uad8c\ud55c\uacfc \uba85\uc608 \uadf8\ub9ac\uace0 \uc0ac\ub78c\ub4e4\uc758 \uc874\uacbd\uc744 \ubc1b\uc544 \uc808\ub300\uc801\uc778 \uad8c\ub825\uc744 \ub204\ub838\ub2e4. \uc544\uc6b0\uad6c\uc2a4\ud22c\uc2a4\uac00 \uac00\uc9c4 \ub85c\ub9c8\uc758 \uc815\uc608\ubcd1 \ub85c\ub9c8 \uad70\ub2e8 \ub2e4\uc218\ub97c \ud1b5\uc81c\ud560 \uc218 \uc788\ub294 \uad8c\ud55c\uc740 \uc6d0\ub85c\uc6d0\uc5d0\uac8c \uad70\uc0ac\uc801\uc778 \uc704\ud611\uc774 \ub418\uc5b4 \uc6d0\ub85c\uc6d0\uc758 \uacb0\uc815\uc744 \uc5b5\uc555\ud558\uc600\uace0, \uad70\uc0ac\uc801 \uc218\ub2e8\uc744 \uc0ac\uc6a9\ud558\uc5ec \uc6d0\ub85c\uc6d0\uc758 \uc815\uc801\ub4e4\uc744 \uc81c\uac70\ud558\uc5ec \uc6d0\ub85c\uc6d0\uc774 \uc790\uc2e0\uc5d0 \ubcf5\uc885\ud558\uac8c\ub054 \ud558\uc600\ub2e4.", "answer": "\ub85c\ub9c8 \uacf5\ud654\uc815", "sentence": "\uc81c2\ucc28 \uc0bc\ub450 \uc815\uce58\uc758 \ubd95\uad34 \ud6c4 \uc625\ud0c0\ube44\uc544\ub204\uc2a4\ub294 \ub300\uc678\uc801\uc73c\ub85c \ub85c\ub9c8 \uacf5\ud654\uc815 \uc744 \ubd80\ud65c\uc2dc\ud0a4\uace0 \uc815\ubd80\uc5d0 \uad00\ud55c \uad8c\ud55c\uc740 \ub85c\ub9c8 \uc6d0\ub85c\uc6d0\uc5d0\uac8c \uc8fc\uc5c8\uc73c\ub098, \uc0ac\uc2e4\uc0c1 \uad8c\ub825\uc740 \uadf8\uc758 \uc190\uc5d0 \ub4e4\uc5b4\uac00\uac8c \ub418\uc5c8\ub2e4.", "paragraph_sentence": " \uc81c2\ucc28 \uc0bc\ub450 \uc815\uce58\uc758 \ubd95\uad34 \ud6c4 \uc625\ud0c0\ube44\uc544\ub204\uc2a4\ub294 \ub300\uc678\uc801\uc73c\ub85c \ub85c\ub9c8 \uacf5\ud654\uc815 \uc744 \ubd80\ud65c\uc2dc\ud0a4\uace0 \uc815\ubd80\uc5d0 \uad00\ud55c \uad8c\ud55c\uc740 \ub85c\ub9c8 \uc6d0\ub85c\uc6d0\uc5d0\uac8c \uc8fc\uc5c8\uc73c\ub098, \uc0ac\uc2e4\uc0c1 \uad8c\ub825\uc740 \uadf8\uc758 \uc190\uc5d0 \ub4e4\uc5b4\uac00\uac8c \ub418\uc5c8\ub2e4. \uc720\uc77c\ud55c \ud1b5\uce58\uc790\uac00 \ub2e4\uc2a4\ub9ac\uc9c0\ub9cc \ub300\uc678\uc801\uc73c\ub85c\ub294 \uacf5\ud654\uad6d \ud615\ud0dc\uc778 \uc815\uce58 \uccb4\uc81c\uc758 \uae30\ud2c0\uc744 \ub2e4\uc9c0\ub294 \ub370\ub294 \uc624\ub79c \uc2dc\uac04\uc774 \uac78\ub838\ub2e4. \uaecd\ub370\uae30\ub9cc \uacf5\ud654\uad6d\uc778 \uc774 \ub098\ub77c\ub294 \ud6d7\ub0a0 \ub85c\ub9c8 \uc81c\uad6d\uc73c\ub85c \ubd88\ub9b0\ub2e4. \ud669\uc81c\uad8c\uc740 \uc625\ud0c0\ube44\uc544\ub204\uc2a4 \uc774\uc804\uc5d0 \ub85c\ub9c8\ub97c \ud1b5\uce58\ud588\ub358 \uce74\uc774\uc0ac\ub974\uc640 \uc220\ub77c\uc758 \ub3c5\uc7ac\uad8c\uacfc\ub294 \uc804\ud600 \ub2ec\ub790\ub2e4. \uc625\ud0c0\ube44\uc544\ub204\uc2a4\ub294 \ub85c\ub9c8\uc758 \uc6d0\ub85c\uc6d0\uacfc \uc2dc\ubbfc\ub4e4\ub85c\ubd80\ud130 \u201c\ub3c5\uc7ac\uad8c\uc744 \ubd80\uc5ec\ubc1b\uc558\uc9c0\ub9cc\u201d \uac70\uc808\ud558\uc600\ub2e4. \ubc95\uc5d0 \ub530\ub974\uba74 \u2018\uc874\uc5c4\uc790\u2019(\uc544\uc6b0\uad6c\uc2a4\ud22c\uc2a4)\ub77c\ub294 \uce6d\ud638\ub97c \ubc1b\uc740 \uc625\ud0c0\ube44\uc544\ub204\uc2a4\uc5d0\uac8c \uc6d0\ub85c\uc6d0\uc740 \ud3c9\uc0dd \ub3d9\uc548 \uad8c\ub825\uc744 \uac00\uc9c0\ub3c4\ub85d \ud558\uc600\uace0 \u201c\ud638\ubbfc\uad00 \uad8c\ud55c\u201d(tribunitia potestas)\uc744 \uac00\uc84c\uc73c\uba70 \uae30\uc6d0\uc804 23\ub144\uae4c\uc9c0 \uc9d1\uc815\uad00\uc744 \uc5ed\uc784\ud558\uc600\ub2e4. \uc544\uc6b0\uad6c\uc2a4\ud22c\uc2a4\ub294 \uc7ac\uc815\uc801\uc778 \uc131\uacf5\uacfc \uc6d0\uc815\uc5d0\uc11c \uc5bb\uc740 \ubb3c\uc790, \uc81c\uad6d \uc804\uccb4\uc5d0 \uac78\uccd0 \ub9fa\uc740 \uc5ec\ub7ec \ud53c\ud638 \uad00\uacc4(clientela), \uad70\uc778\uacfc \uc7ac\ud5a5 \uad70\uc778\uc758 \ucda9\uc131, \uc6d0\ub85c\uc6d0\uc5d0\uc11c \ubd80\uc5ec\ud55c \uc5ec\ub7ec \uad8c\ud55c\uacfc \uba85\uc608 \uadf8\ub9ac\uace0 \uc0ac\ub78c\ub4e4\uc758 \uc874\uacbd\uc744 \ubc1b\uc544 \uc808\ub300\uc801\uc778 \uad8c\ub825\uc744 \ub204\ub838\ub2e4. \uc544\uc6b0\uad6c\uc2a4\ud22c\uc2a4\uac00 \uac00\uc9c4 \ub85c\ub9c8\uc758 \uc815\uc608\ubcd1 \ub85c\ub9c8 \uad70\ub2e8 \ub2e4\uc218\ub97c \ud1b5\uc81c\ud560 \uc218 \uc788\ub294 \uad8c\ud55c\uc740 \uc6d0\ub85c\uc6d0\uc5d0\uac8c \uad70\uc0ac\uc801\uc778 \uc704\ud611\uc774 \ub418\uc5b4 \uc6d0\ub85c\uc6d0\uc758 \uacb0\uc815\uc744 \uc5b5\uc555\ud558\uc600\uace0, \uad70\uc0ac\uc801 \uc218\ub2e8\uc744 \uc0ac\uc6a9\ud558\uc5ec \uc6d0\ub85c\uc6d0\uc758 \uc815\uc801\ub4e4\uc744 \uc81c\uac70\ud558\uc5ec \uc6d0\ub85c\uc6d0\uc774 \uc790\uc2e0\uc5d0 \ubcf5\uc885\ud558\uac8c\ub054 \ud558\uc600\ub2e4.", "paragraph_answer": "\uc81c2\ucc28 \uc0bc\ub450 \uc815\uce58\uc758 \ubd95\uad34 \ud6c4 \uc625\ud0c0\ube44\uc544\ub204\uc2a4\ub294 \ub300\uc678\uc801\uc73c\ub85c \ub85c\ub9c8 \uacf5\ud654\uc815 \uc744 \ubd80\ud65c\uc2dc\ud0a4\uace0 \uc815\ubd80\uc5d0 \uad00\ud55c \uad8c\ud55c\uc740 \ub85c\ub9c8 \uc6d0\ub85c\uc6d0\uc5d0\uac8c \uc8fc\uc5c8\uc73c\ub098, \uc0ac\uc2e4\uc0c1 \uad8c\ub825\uc740 \uadf8\uc758 \uc190\uc5d0 \ub4e4\uc5b4\uac00\uac8c \ub418\uc5c8\ub2e4. \uc720\uc77c\ud55c \ud1b5\uce58\uc790\uac00 \ub2e4\uc2a4\ub9ac\uc9c0\ub9cc \ub300\uc678\uc801\uc73c\ub85c\ub294 \uacf5\ud654\uad6d \ud615\ud0dc\uc778 \uc815\uce58 \uccb4\uc81c\uc758 \uae30\ud2c0\uc744 \ub2e4\uc9c0\ub294 \ub370\ub294 \uc624\ub79c \uc2dc\uac04\uc774 \uac78\ub838\ub2e4. \uaecd\ub370\uae30\ub9cc \uacf5\ud654\uad6d\uc778 \uc774 \ub098\ub77c\ub294 \ud6d7\ub0a0 \ub85c\ub9c8 \uc81c\uad6d\uc73c\ub85c \ubd88\ub9b0\ub2e4. \ud669\uc81c\uad8c\uc740 \uc625\ud0c0\ube44\uc544\ub204\uc2a4 \uc774\uc804\uc5d0 \ub85c\ub9c8\ub97c \ud1b5\uce58\ud588\ub358 \uce74\uc774\uc0ac\ub974\uc640 \uc220\ub77c\uc758 \ub3c5\uc7ac\uad8c\uacfc\ub294 \uc804\ud600 \ub2ec\ub790\ub2e4. \uc625\ud0c0\ube44\uc544\ub204\uc2a4\ub294 \ub85c\ub9c8\uc758 \uc6d0\ub85c\uc6d0\uacfc \uc2dc\ubbfc\ub4e4\ub85c\ubd80\ud130 \u201c\ub3c5\uc7ac\uad8c\uc744 \ubd80\uc5ec\ubc1b\uc558\uc9c0\ub9cc\u201d \uac70\uc808\ud558\uc600\ub2e4. \ubc95\uc5d0 \ub530\ub974\uba74 \u2018\uc874\uc5c4\uc790\u2019(\uc544\uc6b0\uad6c\uc2a4\ud22c\uc2a4)\ub77c\ub294 \uce6d\ud638\ub97c \ubc1b\uc740 \uc625\ud0c0\ube44\uc544\ub204\uc2a4\uc5d0\uac8c \uc6d0\ub85c\uc6d0\uc740 \ud3c9\uc0dd \ub3d9\uc548 \uad8c\ub825\uc744 \uac00\uc9c0\ub3c4\ub85d \ud558\uc600\uace0 \u201c\ud638\ubbfc\uad00 \uad8c\ud55c\u201d(tribunitia potestas)\uc744 \uac00\uc84c\uc73c\uba70 \uae30\uc6d0\uc804 23\ub144\uae4c\uc9c0 \uc9d1\uc815\uad00\uc744 \uc5ed\uc784\ud558\uc600\ub2e4. \uc544\uc6b0\uad6c\uc2a4\ud22c\uc2a4\ub294 \uc7ac\uc815\uc801\uc778 \uc131\uacf5\uacfc \uc6d0\uc815\uc5d0\uc11c \uc5bb\uc740 \ubb3c\uc790, \uc81c\uad6d \uc804\uccb4\uc5d0 \uac78\uccd0 \ub9fa\uc740 \uc5ec\ub7ec \ud53c\ud638 \uad00\uacc4(clientela), \uad70\uc778\uacfc \uc7ac\ud5a5 \uad70\uc778\uc758 \ucda9\uc131, \uc6d0\ub85c\uc6d0\uc5d0\uc11c \ubd80\uc5ec\ud55c \uc5ec\ub7ec \uad8c\ud55c\uacfc \uba85\uc608 \uadf8\ub9ac\uace0 \uc0ac\ub78c\ub4e4\uc758 \uc874\uacbd\uc744 \ubc1b\uc544 \uc808\ub300\uc801\uc778 \uad8c\ub825\uc744 \ub204\ub838\ub2e4. \uc544\uc6b0\uad6c\uc2a4\ud22c\uc2a4\uac00 \uac00\uc9c4 \ub85c\ub9c8\uc758 \uc815\uc608\ubcd1 \ub85c\ub9c8 \uad70\ub2e8 \ub2e4\uc218\ub97c \ud1b5\uc81c\ud560 \uc218 \uc788\ub294 \uad8c\ud55c\uc740 \uc6d0\ub85c\uc6d0\uc5d0\uac8c \uad70\uc0ac\uc801\uc778 \uc704\ud611\uc774 \ub418\uc5b4 \uc6d0\ub85c\uc6d0\uc758 \uacb0\uc815\uc744 \uc5b5\uc555\ud558\uc600\uace0, \uad70\uc0ac\uc801 \uc218\ub2e8\uc744 \uc0ac\uc6a9\ud558\uc5ec \uc6d0\ub85c\uc6d0\uc758 \uc815\uc801\ub4e4\uc744 \uc81c\uac70\ud558\uc5ec \uc6d0\ub85c\uc6d0\uc774 \uc790\uc2e0\uc5d0 \ubcf5\uc885\ud558\uac8c\ub054 \ud558\uc600\ub2e4.", "sentence_answer": "\uc81c2\ucc28 \uc0bc\ub450 \uc815\uce58\uc758 \ubd95\uad34 \ud6c4 \uc625\ud0c0\ube44\uc544\ub204\uc2a4\ub294 \ub300\uc678\uc801\uc73c\ub85c \ub85c\ub9c8 \uacf5\ud654\uc815 \uc744 \ubd80\ud65c\uc2dc\ud0a4\uace0 \uc815\ubd80\uc5d0 \uad00\ud55c \uad8c\ud55c\uc740 \ub85c\ub9c8 \uc6d0\ub85c\uc6d0\uc5d0\uac8c \uc8fc\uc5c8\uc73c\ub098, \uc0ac\uc2e4\uc0c1 \uad8c\ub825\uc740 \uadf8\uc758 \uc190\uc5d0 \ub4e4\uc5b4\uac00\uac8c \ub418\uc5c8\ub2e4.", "paragraph_id": "6520922-1-0", "paragraph_question": "question: \uc81c2\ucc28 \uc0bc\ub450 \uc815\uce58\uc758 \ubd95\uad34 \ud6c4 \uc625\ud0c0\ube44\uc544\ub204\uc2a4\uac00 \ub300\uc678\uc801\uc73c\ub85c \ubd80\ud65c\uc2dc\ud0a8 \uac83\uc740?, context: \uc81c2\ucc28 \uc0bc\ub450 \uc815\uce58\uc758 \ubd95\uad34 \ud6c4 \uc625\ud0c0\ube44\uc544\ub204\uc2a4\ub294 \ub300\uc678\uc801\uc73c\ub85c \ub85c\ub9c8 \uacf5\ud654\uc815\uc744 \ubd80\ud65c\uc2dc\ud0a4\uace0 \uc815\ubd80\uc5d0 \uad00\ud55c \uad8c\ud55c\uc740 \ub85c\ub9c8 \uc6d0\ub85c\uc6d0\uc5d0\uac8c \uc8fc\uc5c8\uc73c\ub098, \uc0ac\uc2e4\uc0c1 \uad8c\ub825\uc740 \uadf8\uc758 \uc190\uc5d0 \ub4e4\uc5b4\uac00\uac8c \ub418\uc5c8\ub2e4. \uc720\uc77c\ud55c \ud1b5\uce58\uc790\uac00 \ub2e4\uc2a4\ub9ac\uc9c0\ub9cc \ub300\uc678\uc801\uc73c\ub85c\ub294 \uacf5\ud654\uad6d \ud615\ud0dc\uc778 \uc815\uce58 \uccb4\uc81c\uc758 \uae30\ud2c0\uc744 \ub2e4\uc9c0\ub294 \ub370\ub294 \uc624\ub79c \uc2dc\uac04\uc774 \uac78\ub838\ub2e4. \uaecd\ub370\uae30\ub9cc \uacf5\ud654\uad6d\uc778 \uc774 \ub098\ub77c\ub294 \ud6d7\ub0a0 \ub85c\ub9c8 \uc81c\uad6d\uc73c\ub85c \ubd88\ub9b0\ub2e4. \ud669\uc81c\uad8c\uc740 \uc625\ud0c0\ube44\uc544\ub204\uc2a4 \uc774\uc804\uc5d0 \ub85c\ub9c8\ub97c \ud1b5\uce58\ud588\ub358 \uce74\uc774\uc0ac\ub974\uc640 \uc220\ub77c\uc758 \ub3c5\uc7ac\uad8c\uacfc\ub294 \uc804\ud600 \ub2ec\ub790\ub2e4. \uc625\ud0c0\ube44\uc544\ub204\uc2a4\ub294 \ub85c\ub9c8\uc758 \uc6d0\ub85c\uc6d0\uacfc \uc2dc\ubbfc\ub4e4\ub85c\ubd80\ud130 \u201c\ub3c5\uc7ac\uad8c\uc744 \ubd80\uc5ec\ubc1b\uc558\uc9c0\ub9cc\u201d \uac70\uc808\ud558\uc600\ub2e4. \ubc95\uc5d0 \ub530\ub974\uba74 \u2018\uc874\uc5c4\uc790\u2019(\uc544\uc6b0\uad6c\uc2a4\ud22c\uc2a4)\ub77c\ub294 \uce6d\ud638\ub97c \ubc1b\uc740 \uc625\ud0c0\ube44\uc544\ub204\uc2a4\uc5d0\uac8c \uc6d0\ub85c\uc6d0\uc740 \ud3c9\uc0dd \ub3d9\uc548 \uad8c\ub825\uc744 \uac00\uc9c0\ub3c4\ub85d \ud558\uc600\uace0 \u201c\ud638\ubbfc\uad00 \uad8c\ud55c\u201d(tribunitia potestas)\uc744 \uac00\uc84c\uc73c\uba70 \uae30\uc6d0\uc804 23\ub144\uae4c\uc9c0 \uc9d1\uc815\uad00\uc744 \uc5ed\uc784\ud558\uc600\ub2e4. \uc544\uc6b0\uad6c\uc2a4\ud22c\uc2a4\ub294 \uc7ac\uc815\uc801\uc778 \uc131\uacf5\uacfc \uc6d0\uc815\uc5d0\uc11c \uc5bb\uc740 \ubb3c\uc790, \uc81c\uad6d \uc804\uccb4\uc5d0 \uac78\uccd0 \ub9fa\uc740 \uc5ec\ub7ec \ud53c\ud638 \uad00\uacc4(clientela), \uad70\uc778\uacfc \uc7ac\ud5a5 \uad70\uc778\uc758 \ucda9\uc131, \uc6d0\ub85c\uc6d0\uc5d0\uc11c \ubd80\uc5ec\ud55c \uc5ec\ub7ec \uad8c\ud55c\uacfc \uba85\uc608 \uadf8\ub9ac\uace0 \uc0ac\ub78c\ub4e4\uc758 \uc874\uacbd\uc744 \ubc1b\uc544 \uc808\ub300\uc801\uc778 \uad8c\ub825\uc744 \ub204\ub838\ub2e4. \uc544\uc6b0\uad6c\uc2a4\ud22c\uc2a4\uac00 \uac00\uc9c4 \ub85c\ub9c8\uc758 \uc815\uc608\ubcd1 \ub85c\ub9c8 \uad70\ub2e8 \ub2e4\uc218\ub97c \ud1b5\uc81c\ud560 \uc218 \uc788\ub294 \uad8c\ud55c\uc740 \uc6d0\ub85c\uc6d0\uc5d0\uac8c \uad70\uc0ac\uc801\uc778 \uc704\ud611\uc774 \ub418\uc5b4 \uc6d0\ub85c\uc6d0\uc758 \uacb0\uc815\uc744 \uc5b5\uc555\ud558\uc600\uace0, \uad70\uc0ac\uc801 \uc218\ub2e8\uc744 \uc0ac\uc6a9\ud558\uc5ec \uc6d0\ub85c\uc6d0\uc758 \uc815\uc801\ub4e4\uc744 \uc81c\uac70\ud558\uc5ec \uc6d0\ub85c\uc6d0\uc774 \uc790\uc2e0\uc5d0 \ubcf5\uc885\ud558\uac8c\ub054 \ud558\uc600\ub2e4."} {"question": "\ubc84\ubc8c\uc9c4\ud2b8\ub294 \ubaa8 \uc6f9\uc9c4\uacfc \uc7a1\uc9c0\uac00 \uc4f4 \ub9ac\ubdf0\ub97c \ubcf4\uace0 \ud799\ud569 \uc74c\uc545\uc744 \ubb34\uc5c7\uc774\ub77c\uace0 \uc0dd\uac01 \ud574\uc11c \uc774\ub7f0 \ub9ac\ubdf0\uac00 \ub098\uc654\ub2e4\uace0 \ub9d0\ud558\uc600\ub098?", "paragraph": "\ubc84\ubc8c\uc9c4\ud2b8\ub294 \uba87\uba87\uc758 \ud3c9\uac00\uc5d0 \ubd80\uc815\uc801\uc778 \ubc18\uc751\uc744 \ubcf4\uc600\ub2e4. \ubaa8 \uc6f9\uc9c4\uacfc \uc7a1\uc9c0(\uc815\ud655\ud788 \uaf2c\uc9d1\uc5b4 \ub9d0\ud558\uc9c0\ub294 \uc54a\uc558\ub2e4)\uac00 \uc4f4 \ub9ac\ubdf0\ub97c \uc5b8\uae09\ud558\uba74\uc11c, \u201c\ubaa8\ub450 \ud799\ud569 \uc74c\uc545\uc744 \uc911\ub529 \uc74c\uc545\uc774\ub77c\uace0 \uc0dd\uac01\uc744 \ud558\ub2c8\uae4c \uadf8\ub7f0 \uc2dd\uc758 \ub9ac\ubdf0\uac00 \ub098\uc628 \uac70\ub77c\uace0 \uc0dd\uac01\uc744 \ud55c\ub2e4\u201d\uace0 \ub9d0\ud588\ub2e4. \ubc84\ubc8c\uc9c4\ud2b8\ub294 \ud799\ud569\uc744 \uc5b4\ub9b0\uc560\ub85c \ubcf4\uace0\uc11c \ub9ac\ubdf0\ub97c \uc4f0\uae30 \ub54c\ubb38\uc5d0 \uadf8\ub7f0 \uc6c3\uae34 \uc77c\ub4e4\uc774 \ubc8c\uc5b4\uc9c4\ub2e4\uace0 \ub9d0\ud558\uba74\uc11c, \ud799\ud569\uc774 \uadf8 \uc815\ub3c4\ub85c \uc778\uc2dd\ub418\uace0 \uc788\ub2e4\ub294 \uac83\uc5d0 \uc544\uc26c\uc6c0\uc744 \ub098\ud0c0\ub0c8\ub2e4. \ub610\ud55c \ubc84\ubc8c\uc9c4\ud2b8\ub294 \uba87 \uac1c\uc758 \ub9ac\ubdf0\ub098 \uc77c\ubd80 \ub9ac\uc2a4\ub108\ub4e4\uc774 \uadf8\uc758 \uc601\uac10\uc758 \uc6d0\ucc9c\uc744 \"\uc798\ub09c \uccb4\"\ub85c \uc624\ud574\ud558\ub294\uac8c \uac00\uc7a5 \ud2c0\ub9ac\ub2e4\uace0 \uc9c0\uc801\ud588\ub2e4. \uadf8\ub294 \uc568\ubc94\uc5d0 \uc218\ub85d\ub41c \ud2b8\ub799 \u30082008 \ub300\ud55c\ubbfc\uad6d\u3009, \u3008\uc5ed\uc0ac\uc758 \uac04\uc9c0(\u5978\u667a)\u3009, \u3008Tight\uc774\ub780 \ub0b1\ub9d0\uc758 \uc874\uc7ac\uc774\uc720\u3009, \"1219 Epiphany\"\uc744 \uc5b8\uae09\ud558\uba74\uc11c, \uc774 \ud2b8\ub799\ub4e4\uc744 \ube7c\uba74 \ub098\uba38\uc9c0\ub294 \uc644\uc804 \ub2e4\ub978 \ucc28\uc6d0\uc758 \uc774\uc57c\uae30\ub4e4\uc774\uba70, \ud799\ud569\uc5d4 \"\ud55c \ubc88 \ub4e3\uace0 \ub9d0 \uac00\uc0ac\ubc16\uc5d0 \uc5c6\ub2e4\"\ub294 \uc0dd\uac01\uc73c\ub85c \ud799\ud569\uc744 \ub4e3\uae30 \ub54c\ubb38\uc5d0 \uc0dd\uae30\ub294 \uc624\ud574\uc778 \uac83 \uac19\ub2e4\uace0 \ub9d0\ud588\ub2e4.", "answer": "\uc911\ub529 \uc74c\uc545", "sentence": "\u201c\ubaa8\ub450 \ud799\ud569 \uc74c\uc545\uc744 \uc911\ub529 \uc74c\uc545 \uc774\ub77c\uace0 \uc0dd\uac01\uc744 \ud558\ub2c8\uae4c \uadf8\ub7f0 \uc2dd\uc758 \ub9ac\ubdf0\uac00 \ub098\uc628 \uac70\ub77c\uace0 \uc0dd\uac01\uc744 \ud55c\ub2e4\u201d\uace0 \ub9d0\ud588\ub2e4.", "paragraph_sentence": "\ubc84\ubc8c\uc9c4\ud2b8\ub294 \uba87\uba87\uc758 \ud3c9\uac00\uc5d0 \ubd80\uc815\uc801\uc778 \ubc18\uc751\uc744 \ubcf4\uc600\ub2e4. \ubaa8 \uc6f9\uc9c4\uacfc \uc7a1\uc9c0(\uc815\ud655\ud788 \uaf2c\uc9d1\uc5b4 \ub9d0\ud558\uc9c0\ub294 \uc54a\uc558\ub2e4)\uac00 \uc4f4 \ub9ac\ubdf0\ub97c \uc5b8\uae09\ud558\uba74\uc11c, \u201c\ubaa8\ub450 \ud799\ud569 \uc74c\uc545\uc744 \uc911\ub529 \uc74c\uc545 \uc774\ub77c\uace0 \uc0dd\uac01\uc744 \ud558\ub2c8\uae4c \uadf8\ub7f0 \uc2dd\uc758 \ub9ac\ubdf0\uac00 \ub098\uc628 \uac70\ub77c\uace0 \uc0dd\uac01\uc744 \ud55c\ub2e4\u201d\uace0 \ub9d0\ud588\ub2e4. \ubc84\ubc8c\uc9c4\ud2b8\ub294 \ud799\ud569\uc744 \uc5b4\ub9b0\uc560\ub85c \ubcf4\uace0\uc11c \ub9ac\ubdf0\ub97c \uc4f0\uae30 \ub54c\ubb38\uc5d0 \uadf8\ub7f0 \uc6c3\uae34 \uc77c\ub4e4\uc774 \ubc8c\uc5b4\uc9c4\ub2e4\uace0 \ub9d0\ud558\uba74\uc11c, \ud799\ud569\uc774 \uadf8 \uc815\ub3c4\ub85c \uc778\uc2dd\ub418\uace0 \uc788\ub2e4\ub294 \uac83\uc5d0 \uc544\uc26c\uc6c0\uc744 \ub098\ud0c0\ub0c8\ub2e4. \ub610\ud55c \ubc84\ubc8c\uc9c4\ud2b8\ub294 \uba87 \uac1c\uc758 \ub9ac\ubdf0\ub098 \uc77c\ubd80 \ub9ac\uc2a4\ub108\ub4e4\uc774 \uadf8\uc758 \uc601\uac10\uc758 \uc6d0\ucc9c\uc744 \"\uc798\ub09c \uccb4\"\ub85c \uc624\ud574\ud558\ub294\uac8c \uac00\uc7a5 \ud2c0\ub9ac\ub2e4\uace0 \uc9c0\uc801\ud588\ub2e4. \uadf8\ub294 \uc568\ubc94\uc5d0 \uc218\ub85d\ub41c \ud2b8\ub799 \u30082008 \ub300\ud55c\ubbfc\uad6d\u3009, \u3008\uc5ed\uc0ac\uc758 \uac04\uc9c0(\u5978\u667a)\u3009, \u3008Tight\uc774\ub780 \ub0b1\ub9d0\uc758 \uc874\uc7ac\uc774\uc720\u3009, \"1219 Epiphany\"\uc744 \uc5b8\uae09\ud558\uba74\uc11c, \uc774 \ud2b8\ub799\ub4e4\uc744 \ube7c\uba74 \ub098\uba38\uc9c0\ub294 \uc644\uc804 \ub2e4\ub978 \ucc28\uc6d0\uc758 \uc774\uc57c\uae30\ub4e4\uc774\uba70, \ud799\ud569\uc5d4 \"\ud55c \ubc88 \ub4e3\uace0 \ub9d0 \uac00\uc0ac\ubc16\uc5d0 \uc5c6\ub2e4 \"\ub294 \uc0dd\uac01\uc73c\ub85c \ud799\ud569\uc744 \ub4e3\uae30 \ub54c\ubb38\uc5d0 \uc0dd\uae30\ub294 \uc624\ud574\uc778 \uac83 \uac19\ub2e4\uace0 \ub9d0\ud588\ub2e4.", "paragraph_answer": "\ubc84\ubc8c\uc9c4\ud2b8\ub294 \uba87\uba87\uc758 \ud3c9\uac00\uc5d0 \ubd80\uc815\uc801\uc778 \ubc18\uc751\uc744 \ubcf4\uc600\ub2e4. \ubaa8 \uc6f9\uc9c4\uacfc \uc7a1\uc9c0(\uc815\ud655\ud788 \uaf2c\uc9d1\uc5b4 \ub9d0\ud558\uc9c0\ub294 \uc54a\uc558\ub2e4)\uac00 \uc4f4 \ub9ac\ubdf0\ub97c \uc5b8\uae09\ud558\uba74\uc11c, \u201c\ubaa8\ub450 \ud799\ud569 \uc74c\uc545\uc744 \uc911\ub529 \uc74c\uc545 \uc774\ub77c\uace0 \uc0dd\uac01\uc744 \ud558\ub2c8\uae4c \uadf8\ub7f0 \uc2dd\uc758 \ub9ac\ubdf0\uac00 \ub098\uc628 \uac70\ub77c\uace0 \uc0dd\uac01\uc744 \ud55c\ub2e4\u201d\uace0 \ub9d0\ud588\ub2e4. \ubc84\ubc8c\uc9c4\ud2b8\ub294 \ud799\ud569\uc744 \uc5b4\ub9b0\uc560\ub85c \ubcf4\uace0\uc11c \ub9ac\ubdf0\ub97c \uc4f0\uae30 \ub54c\ubb38\uc5d0 \uadf8\ub7f0 \uc6c3\uae34 \uc77c\ub4e4\uc774 \ubc8c\uc5b4\uc9c4\ub2e4\uace0 \ub9d0\ud558\uba74\uc11c, \ud799\ud569\uc774 \uadf8 \uc815\ub3c4\ub85c \uc778\uc2dd\ub418\uace0 \uc788\ub2e4\ub294 \uac83\uc5d0 \uc544\uc26c\uc6c0\uc744 \ub098\ud0c0\ub0c8\ub2e4. \ub610\ud55c \ubc84\ubc8c\uc9c4\ud2b8\ub294 \uba87 \uac1c\uc758 \ub9ac\ubdf0\ub098 \uc77c\ubd80 \ub9ac\uc2a4\ub108\ub4e4\uc774 \uadf8\uc758 \uc601\uac10\uc758 \uc6d0\ucc9c\uc744 \"\uc798\ub09c \uccb4\"\ub85c \uc624\ud574\ud558\ub294\uac8c \uac00\uc7a5 \ud2c0\ub9ac\ub2e4\uace0 \uc9c0\uc801\ud588\ub2e4. \uadf8\ub294 \uc568\ubc94\uc5d0 \uc218\ub85d\ub41c \ud2b8\ub799 \u30082008 \ub300\ud55c\ubbfc\uad6d\u3009, \u3008\uc5ed\uc0ac\uc758 \uac04\uc9c0(\u5978\u667a)\u3009, \u3008Tight\uc774\ub780 \ub0b1\ub9d0\uc758 \uc874\uc7ac\uc774\uc720\u3009, \"1219 Epiphany\"\uc744 \uc5b8\uae09\ud558\uba74\uc11c, \uc774 \ud2b8\ub799\ub4e4\uc744 \ube7c\uba74 \ub098\uba38\uc9c0\ub294 \uc644\uc804 \ub2e4\ub978 \ucc28\uc6d0\uc758 \uc774\uc57c\uae30\ub4e4\uc774\uba70, \ud799\ud569\uc5d4 \"\ud55c \ubc88 \ub4e3\uace0 \ub9d0 \uac00\uc0ac\ubc16\uc5d0 \uc5c6\ub2e4\"\ub294 \uc0dd\uac01\uc73c\ub85c \ud799\ud569\uc744 \ub4e3\uae30 \ub54c\ubb38\uc5d0 \uc0dd\uae30\ub294 \uc624\ud574\uc778 \uac83 \uac19\ub2e4\uace0 \ub9d0\ud588\ub2e4.", "sentence_answer": "\u201c\ubaa8\ub450 \ud799\ud569 \uc74c\uc545\uc744 \uc911\ub529 \uc74c\uc545 \uc774\ub77c\uace0 \uc0dd\uac01\uc744 \ud558\ub2c8\uae4c \uadf8\ub7f0 \uc2dd\uc758 \ub9ac\ubdf0\uac00 \ub098\uc628 \uac70\ub77c\uace0 \uc0dd\uac01\uc744 \ud55c\ub2e4\u201d\uace0 \ub9d0\ud588\ub2e4.", "paragraph_id": "6527839-3-1", "paragraph_question": "question: \ubc84\ubc8c\uc9c4\ud2b8\ub294 \ubaa8 \uc6f9\uc9c4\uacfc \uc7a1\uc9c0\uac00 \uc4f4 \ub9ac\ubdf0\ub97c \ubcf4\uace0 \ud799\ud569 \uc74c\uc545\uc744 \ubb34\uc5c7\uc774\ub77c\uace0 \uc0dd\uac01 \ud574\uc11c \uc774\ub7f0 \ub9ac\ubdf0\uac00 \ub098\uc654\ub2e4\uace0 \ub9d0\ud558\uc600\ub098?, context: \ubc84\ubc8c\uc9c4\ud2b8\ub294 \uba87\uba87\uc758 \ud3c9\uac00\uc5d0 \ubd80\uc815\uc801\uc778 \ubc18\uc751\uc744 \ubcf4\uc600\ub2e4. \ubaa8 \uc6f9\uc9c4\uacfc \uc7a1\uc9c0(\uc815\ud655\ud788 \uaf2c\uc9d1\uc5b4 \ub9d0\ud558\uc9c0\ub294 \uc54a\uc558\ub2e4)\uac00 \uc4f4 \ub9ac\ubdf0\ub97c \uc5b8\uae09\ud558\uba74\uc11c, \u201c\ubaa8\ub450 \ud799\ud569 \uc74c\uc545\uc744 \uc911\ub529 \uc74c\uc545\uc774\ub77c\uace0 \uc0dd\uac01\uc744 \ud558\ub2c8\uae4c \uadf8\ub7f0 \uc2dd\uc758 \ub9ac\ubdf0\uac00 \ub098\uc628 \uac70\ub77c\uace0 \uc0dd\uac01\uc744 \ud55c\ub2e4\u201d\uace0 \ub9d0\ud588\ub2e4. \ubc84\ubc8c\uc9c4\ud2b8\ub294 \ud799\ud569\uc744 \uc5b4\ub9b0\uc560\ub85c \ubcf4\uace0\uc11c \ub9ac\ubdf0\ub97c \uc4f0\uae30 \ub54c\ubb38\uc5d0 \uadf8\ub7f0 \uc6c3\uae34 \uc77c\ub4e4\uc774 \ubc8c\uc5b4\uc9c4\ub2e4\uace0 \ub9d0\ud558\uba74\uc11c, \ud799\ud569\uc774 \uadf8 \uc815\ub3c4\ub85c \uc778\uc2dd\ub418\uace0 \uc788\ub2e4\ub294 \uac83\uc5d0 \uc544\uc26c\uc6c0\uc744 \ub098\ud0c0\ub0c8\ub2e4. \ub610\ud55c \ubc84\ubc8c\uc9c4\ud2b8\ub294 \uba87 \uac1c\uc758 \ub9ac\ubdf0\ub098 \uc77c\ubd80 \ub9ac\uc2a4\ub108\ub4e4\uc774 \uadf8\uc758 \uc601\uac10\uc758 \uc6d0\ucc9c\uc744 \"\uc798\ub09c \uccb4\"\ub85c \uc624\ud574\ud558\ub294\uac8c \uac00\uc7a5 \ud2c0\ub9ac\ub2e4\uace0 \uc9c0\uc801\ud588\ub2e4. \uadf8\ub294 \uc568\ubc94\uc5d0 \uc218\ub85d\ub41c \ud2b8\ub799 \u30082008 \ub300\ud55c\ubbfc\uad6d\u3009, \u3008\uc5ed\uc0ac\uc758 \uac04\uc9c0(\u5978\u667a)\u3009, \u3008Tight\uc774\ub780 \ub0b1\ub9d0\uc758 \uc874\uc7ac\uc774\uc720\u3009, \"1219 Epiphany\"\uc744 \uc5b8\uae09\ud558\uba74\uc11c, \uc774 \ud2b8\ub799\ub4e4\uc744 \ube7c\uba74 \ub098\uba38\uc9c0\ub294 \uc644\uc804 \ub2e4\ub978 \ucc28\uc6d0\uc758 \uc774\uc57c\uae30\ub4e4\uc774\uba70, \ud799\ud569\uc5d4 \"\ud55c \ubc88 \ub4e3\uace0 \ub9d0 \uac00\uc0ac\ubc16\uc5d0 \uc5c6\ub2e4\"\ub294 \uc0dd\uac01\uc73c\ub85c \ud799\ud569\uc744 \ub4e3\uae30 \ub54c\ubb38\uc5d0 \uc0dd\uae30\ub294 \uc624\ud574\uc778 \uac83 \uac19\ub2e4\uace0 \ub9d0\ud588\ub2e4."} {"question": "\ud574\uad70 \uc804\uc7c1\uacfc \ud504\ub791\uc2a4 \uad70\ud568\uc774\ub77c\ub294 \ucc45\uc740 \uba87\ud398\uc774\uc9c0 \ubd84\ub7c9\uc778\uac00?", "paragraph": "\uc774\ubbf8 \uc5b4\ub8b0\ub098 \uc678\uc7a5 \uc5b4\ub8b0\uc815\uc740 \uc2e4\ud604\ub418\uace0 \uc788\uc5c8\uc73c\uba70, \uc790\uc8fc\uc2dd \uc5b4\ub8b0\ub97c \ubc1c\uba85\ud558\uc5ec \uc774\ub860\uc740 \ub354 \uc815\uad50\ud558\uac8c \ub2e4\ub4ec\uc5b4\uc84c\ub2e4. \uc624\ube0c \uc81c\ub3c5\uc740 1882\ub144 \u300a\ud574\uad70 \uc804\uc7c1\uacfc \ud504\ub791\uc2a4\uc758 \uad70\ud56d\u300b(La guerre maritime et les ports militaires de la France)\uc774\ub77c\ub294 \uc81c\ubaa9\uc758 38 \ud398\uc774\uc9c0\uc758 \ubd84\ub7c9\uc758 \ucc45\uc790\ub97c \ubc1c\uac04\ud588\ub2e4. \uc5ec\uae30\uc5d0\uc11c \uadf8\ub294 \u201c\uc0c8\ub85c\uc6b4 \uae30\uc220\uc758 \uac1c\ubc1c, \uc608\ub97c \ub4e4\uc5b4 \uc5b4\ub8b0\ub97c \uc774\uc6a9\ud558\uc5ec \uc801\uc758 \ud574\uc0c1 \ubd09\uc1c4\ub97c \ud574\uc81c\ud560 \uc218 \uc788\ub2e4. \ub530\ub77c\uc11c, \ud574\uc548\uc120\uc744 \ubcf4\ud638\ud558\uae30 \uc704\ud574\uc11c\ub294 \uc804\ud568\uc744 \ub9cc\ub4dc\ub294 \uac83\ubcf4\ub2e4 \ub354 \ub9ce\uc740 \uc5b4\ub8b0\uc815, \uc5f0\uc548\uacbd\ube44\ud568, \ucda9\uac01\ud568 \ub4f1\uc758 \uc791\uc740 \ud568\uc815\uc774 \uc788\uc5b4\uc57c \ud55c\ub2e4. \uacf5\uaca9\uc5d0 \uad00\ud574\uc11c\ub294 \uc21c\uc591\ud568\uc744 \uc774\uc6a9\ud55c \ud1b5\uc0c1\ud30c\uad34\ub97c \uc2e4\uc2dc\ud574\uc57c \ud558\uba70, \uc5b4\ub5a4 \uacbd\uc6b0\uc5d0\ub3c4 \uace0\uc18d\uc758 \ud568\uc815\uc774 \ud544\uc694\ud558\ub2e4\u201d\uace0 \uc8fc\uc7a5\ud588\ub2e4. 1883\ub144\uc5d0\uc11c 1885\ub144\uae4c\uc9c0 \ubc8c\uc5b4\uc9c4 \uccad\ubd88 \uc804\uc7c1\uc5d0\uc11c \ud504\ub791\uc2a4 \ud574\uad70\uc758 \uc2b9\ub9ac\ub294 \uae30\uc874 \ud574\uad70\uc5d0 \ub300\ud55c \uc5b4\ub8b0\uc815\uc758 \uac00\ub2a5\uc131\uc744 \uc2e4\uc99d\ud55c \uac83\uc73c\ub85c \uc0dd\uac01\ub418\uc5c8\ub2e4.", "answer": "38 \ud398\uc774\uc9c0", "sentence": "\uc624\ube0c \uc81c\ub3c5\uc740 1882\ub144 \u300a\ud574\uad70 \uc804\uc7c1\uacfc \ud504\ub791\uc2a4\uc758 \uad70\ud56d\u300b(La guerre maritime et les ports militaires de la France)\uc774\ub77c\ub294 \uc81c\ubaa9\uc758 38 \ud398\uc774\uc9c0 \uc758 \ubd84\ub7c9\uc758 \ucc45\uc790\ub97c \ubc1c\uac04\ud588\ub2e4.", "paragraph_sentence": "\uc774\ubbf8 \uc5b4\ub8b0\ub098 \uc678\uc7a5 \uc5b4\ub8b0\uc815\uc740 \uc2e4\ud604\ub418\uace0 \uc788\uc5c8\uc73c\uba70, \uc790\uc8fc\uc2dd \uc5b4\ub8b0\ub97c \ubc1c\uba85\ud558\uc5ec \uc774\ub860\uc740 \ub354 \uc815\uad50\ud558\uac8c \ub2e4\ub4ec\uc5b4\uc84c\ub2e4. \uc624\ube0c \uc81c\ub3c5\uc740 1882\ub144 \u300a\ud574\uad70 \uc804\uc7c1\uacfc \ud504\ub791\uc2a4\uc758 \uad70\ud56d\u300b(La guerre maritime et les ports militaires de la France)\uc774\ub77c\ub294 \uc81c\ubaa9\uc758 38 \ud398\uc774\uc9c0 \uc758 \ubd84\ub7c9\uc758 \ucc45\uc790\ub97c \ubc1c\uac04\ud588\ub2e4. \uc5ec\uae30\uc5d0\uc11c \uadf8\ub294 \u201c\uc0c8\ub85c\uc6b4 \uae30\uc220\uc758 \uac1c\ubc1c, \uc608\ub97c \ub4e4\uc5b4 \uc5b4\ub8b0\ub97c \uc774\uc6a9\ud558\uc5ec \uc801\uc758 \ud574\uc0c1 \ubd09\uc1c4\ub97c \ud574\uc81c\ud560 \uc218 \uc788\ub2e4. \ub530\ub77c\uc11c, \ud574\uc548\uc120\uc744 \ubcf4\ud638\ud558\uae30 \uc704\ud574\uc11c\ub294 \uc804\ud568\uc744 \ub9cc\ub4dc\ub294 \uac83\ubcf4\ub2e4 \ub354 \ub9ce\uc740 \uc5b4\ub8b0\uc815, \uc5f0\uc548\uacbd\ube44\ud568, \ucda9\uac01\ud568 \ub4f1\uc758 \uc791\uc740 \ud568\uc815\uc774 \uc788\uc5b4\uc57c \ud55c\ub2e4. \uacf5\uaca9\uc5d0 \uad00\ud574\uc11c\ub294 \uc21c\uc591\ud568\uc744 \uc774\uc6a9\ud55c \ud1b5\uc0c1\ud30c\uad34\ub97c \uc2e4\uc2dc\ud574\uc57c \ud558\uba70, \uc5b4\ub5a4 \uacbd\uc6b0\uc5d0\ub3c4 \uace0\uc18d\uc758 \ud568\uc815\uc774 \ud544\uc694\ud558\ub2e4\u201d\uace0 \uc8fc\uc7a5\ud588\ub2e4. 1883\ub144\uc5d0\uc11c 1885\ub144\uae4c\uc9c0 \ubc8c\uc5b4\uc9c4 \uccad\ubd88 \uc804\uc7c1\uc5d0\uc11c \ud504\ub791\uc2a4 \ud574\uad70\uc758 \uc2b9\ub9ac\ub294 \uae30\uc874 \ud574\uad70\uc5d0 \ub300\ud55c \uc5b4\ub8b0\uc815\uc758 \uac00\ub2a5\uc131\uc744 \uc2e4\uc99d\ud55c \uac83\uc73c\ub85c \uc0dd\uac01\ub418\uc5c8\ub2e4.", "paragraph_answer": "\uc774\ubbf8 \uc5b4\ub8b0\ub098 \uc678\uc7a5 \uc5b4\ub8b0\uc815\uc740 \uc2e4\ud604\ub418\uace0 \uc788\uc5c8\uc73c\uba70, \uc790\uc8fc\uc2dd \uc5b4\ub8b0\ub97c \ubc1c\uba85\ud558\uc5ec \uc774\ub860\uc740 \ub354 \uc815\uad50\ud558\uac8c \ub2e4\ub4ec\uc5b4\uc84c\ub2e4. \uc624\ube0c \uc81c\ub3c5\uc740 1882\ub144 \u300a\ud574\uad70 \uc804\uc7c1\uacfc \ud504\ub791\uc2a4\uc758 \uad70\ud56d\u300b(La guerre maritime et les ports militaires de la France)\uc774\ub77c\ub294 \uc81c\ubaa9\uc758 38 \ud398\uc774\uc9c0 \uc758 \ubd84\ub7c9\uc758 \ucc45\uc790\ub97c \ubc1c\uac04\ud588\ub2e4. \uc5ec\uae30\uc5d0\uc11c \uadf8\ub294 \u201c\uc0c8\ub85c\uc6b4 \uae30\uc220\uc758 \uac1c\ubc1c, \uc608\ub97c \ub4e4\uc5b4 \uc5b4\ub8b0\ub97c \uc774\uc6a9\ud558\uc5ec \uc801\uc758 \ud574\uc0c1 \ubd09\uc1c4\ub97c \ud574\uc81c\ud560 \uc218 \uc788\ub2e4. \ub530\ub77c\uc11c, \ud574\uc548\uc120\uc744 \ubcf4\ud638\ud558\uae30 \uc704\ud574\uc11c\ub294 \uc804\ud568\uc744 \ub9cc\ub4dc\ub294 \uac83\ubcf4\ub2e4 \ub354 \ub9ce\uc740 \uc5b4\ub8b0\uc815, \uc5f0\uc548\uacbd\ube44\ud568, \ucda9\uac01\ud568 \ub4f1\uc758 \uc791\uc740 \ud568\uc815\uc774 \uc788\uc5b4\uc57c \ud55c\ub2e4. \uacf5\uaca9\uc5d0 \uad00\ud574\uc11c\ub294 \uc21c\uc591\ud568\uc744 \uc774\uc6a9\ud55c \ud1b5\uc0c1\ud30c\uad34\ub97c \uc2e4\uc2dc\ud574\uc57c \ud558\uba70, \uc5b4\ub5a4 \uacbd\uc6b0\uc5d0\ub3c4 \uace0\uc18d\uc758 \ud568\uc815\uc774 \ud544\uc694\ud558\ub2e4\u201d\uace0 \uc8fc\uc7a5\ud588\ub2e4. 1883\ub144\uc5d0\uc11c 1885\ub144\uae4c\uc9c0 \ubc8c\uc5b4\uc9c4 \uccad\ubd88 \uc804\uc7c1\uc5d0\uc11c \ud504\ub791\uc2a4 \ud574\uad70\uc758 \uc2b9\ub9ac\ub294 \uae30\uc874 \ud574\uad70\uc5d0 \ub300\ud55c \uc5b4\ub8b0\uc815\uc758 \uac00\ub2a5\uc131\uc744 \uc2e4\uc99d\ud55c \uac83\uc73c\ub85c \uc0dd\uac01\ub418\uc5c8\ub2e4.", "sentence_answer": "\uc624\ube0c \uc81c\ub3c5\uc740 1882\ub144 \u300a\ud574\uad70 \uc804\uc7c1\uacfc \ud504\ub791\uc2a4\uc758 \uad70\ud56d\u300b(La guerre maritime et les ports militaires de la France)\uc774\ub77c\ub294 \uc81c\ubaa9\uc758 38 \ud398\uc774\uc9c0 \uc758 \ubd84\ub7c9\uc758 \ucc45\uc790\ub97c \ubc1c\uac04\ud588\ub2e4.", "paragraph_id": "6596841-0-0", "paragraph_question": "question: \ud574\uad70 \uc804\uc7c1\uacfc \ud504\ub791\uc2a4 \uad70\ud568\uc774\ub77c\ub294 \ucc45\uc740 \uba87\ud398\uc774\uc9c0 \ubd84\ub7c9\uc778\uac00?, context: \uc774\ubbf8 \uc5b4\ub8b0\ub098 \uc678\uc7a5 \uc5b4\ub8b0\uc815\uc740 \uc2e4\ud604\ub418\uace0 \uc788\uc5c8\uc73c\uba70, \uc790\uc8fc\uc2dd \uc5b4\ub8b0\ub97c \ubc1c\uba85\ud558\uc5ec \uc774\ub860\uc740 \ub354 \uc815\uad50\ud558\uac8c \ub2e4\ub4ec\uc5b4\uc84c\ub2e4. \uc624\ube0c \uc81c\ub3c5\uc740 1882\ub144 \u300a\ud574\uad70 \uc804\uc7c1\uacfc \ud504\ub791\uc2a4\uc758 \uad70\ud56d\u300b(La guerre maritime et les ports militaires de la France)\uc774\ub77c\ub294 \uc81c\ubaa9\uc758 38 \ud398\uc774\uc9c0\uc758 \ubd84\ub7c9\uc758 \ucc45\uc790\ub97c \ubc1c\uac04\ud588\ub2e4. \uc5ec\uae30\uc5d0\uc11c \uadf8\ub294 \u201c\uc0c8\ub85c\uc6b4 \uae30\uc220\uc758 \uac1c\ubc1c, \uc608\ub97c \ub4e4\uc5b4 \uc5b4\ub8b0\ub97c \uc774\uc6a9\ud558\uc5ec \uc801\uc758 \ud574\uc0c1 \ubd09\uc1c4\ub97c \ud574\uc81c\ud560 \uc218 \uc788\ub2e4. \ub530\ub77c\uc11c, \ud574\uc548\uc120\uc744 \ubcf4\ud638\ud558\uae30 \uc704\ud574\uc11c\ub294 \uc804\ud568\uc744 \ub9cc\ub4dc\ub294 \uac83\ubcf4\ub2e4 \ub354 \ub9ce\uc740 \uc5b4\ub8b0\uc815, \uc5f0\uc548\uacbd\ube44\ud568, \ucda9\uac01\ud568 \ub4f1\uc758 \uc791\uc740 \ud568\uc815\uc774 \uc788\uc5b4\uc57c \ud55c\ub2e4. \uacf5\uaca9\uc5d0 \uad00\ud574\uc11c\ub294 \uc21c\uc591\ud568\uc744 \uc774\uc6a9\ud55c \ud1b5\uc0c1\ud30c\uad34\ub97c \uc2e4\uc2dc\ud574\uc57c \ud558\uba70, \uc5b4\ub5a4 \uacbd\uc6b0\uc5d0\ub3c4 \uace0\uc18d\uc758 \ud568\uc815\uc774 \ud544\uc694\ud558\ub2e4\u201d\uace0 \uc8fc\uc7a5\ud588\ub2e4. 1883\ub144\uc5d0\uc11c 1885\ub144\uae4c\uc9c0 \ubc8c\uc5b4\uc9c4 \uccad\ubd88 \uc804\uc7c1\uc5d0\uc11c \ud504\ub791\uc2a4 \ud574\uad70\uc758 \uc2b9\ub9ac\ub294 \uae30\uc874 \ud574\uad70\uc5d0 \ub300\ud55c \uc5b4\ub8b0\uc815\uc758 \uac00\ub2a5\uc131\uc744 \uc2e4\uc99d\ud55c \uac83\uc73c\ub85c \uc0dd\uac01\ub418\uc5c8\ub2e4."} {"question": "2012\ub144 9\uc6d4 \uacf5\ud56d\ucca0\ub3c4 \uacf5\uc0ac\ub97c \uc2dc\uc791\ud55c \uc628\ud0c0\ub9ac\uc624 \uc8fc\uc758 \uc8fc\uc9c0\uc0ac\ub294?", "paragraph": "2012\ub144 9\uc6d4, \uc628\ud0c0\ub9ac\uc624 \uc8fc\ub294 \uacf5\ud56d\ucca0\ub3c4 \uacf5\uc0ac\uc5d0 \uccab \uc0bd\uc744 \ub4e4\uc5c8\ub2e4. \ub3cc\ud134 \ub9e5\uadc4\ud2f0 \uc8fc\uc9c0\uc0ac\ub294 \uc131\uba85\ubb38\uc744 \ud1b5\ud574 25km\uc758 \uc774 \ub178\uc120\uc740 1200\uac1c \uc774\uc0c1\uc758 \uc77c\uc790\ub9ac\ub97c \ucc3d\ucd9c\ud558\uace0 \uac1c\ud1b5 \uccab \ud574\uc5d0 120\ub9cc \ub300\uc758 \uc790\uac00\uc6a9 \uc6b4\ud589\uc744 \uc904\uc77c \uac83\uc73c\ub85c \ubcf4\uc778\ub2e4\uba70 \uacf5\ud56d\ucca0\ub3c4\ub294 \uc628\ud0c0\ub9ac\uc624 \uc8fc\uc758 \uacbd\uc81c \ubc1c\uc804\uc5d0 \uc801\uc9c0 \uc54a\uc740 \uae30\uc5ec\ub97c \ud560 \uac83\uc774\ub77c\uace0 \ud3c9\ud588\ub2e4. \uc774\ud6c4 2015\ub144 2\uc6d4 23\uc77c, \uacf5\uc0ac\uac00 \uac70\uc758 \ub9c8\ubb34\ub9ac\ub41c UP \uc775\uc2a4\ud504\ub808\uc2a4\ub294 \ube0c\ub808\uc774\ud06c\uc640 \uc18d\ub825 \uc870\uc808, \uc2a4\uc704\uce58\uc640 \uc2e0\ud638 \ubd88\ub7c9 \ub4f1\uc758 \uc0c1\ud669\uc744 \ub300\ube44\ud574 \uc2dc\ubc94\uc6b4\ud589\uc5d0 \ub3cc\uc785\ud588\ub2e4. \uac19\uc740 \ud574 4\uc6d4 22\uc77c, \uc708 \uc8fc\uc9c0\uc0ac\ub294 \uad50\ud1b5\ubd80 \uc7a5\uad00 \uc2a4\ud2f0\ube10 \ub378 \ub450\uce74\uc640 \uba54\ud2b8\ub85c\ub9c1\uc2a4 \ub300\ud45c \ube0c\ub8e8\uc2a4 \ub9e4\ucfe0\uc5d0\uc774\uadf8\uc640 \ud568\uaed8 \uacf5\ud56d\ucca0\ub3c4\uc5d0 \uc2dc\uc2b9\ud558\uc5ec \uc720\ub2c8\uc5b8 \ud53c\uc5b4\uc2a8 \uae09\ud589\uc758 \ucd5c\uc885 \uac1c\ud1b5\uc77c\uc744 2015\ub144 6\uc6d4 6\uc77c\ub85c \ud655\uc815\uc9c0\uc5c8\ub2e4.", "answer": "\ub3cc\ud134 \ub9e5\uadc4\ud2f0", "sentence": "\ub3cc\ud134 \ub9e5\uadc4\ud2f0 \uc8fc\uc9c0\uc0ac\ub294 \uc131\uba85\ubb38\uc744 \ud1b5\ud574 25km\uc758 \uc774 \ub178\uc120\uc740 1200\uac1c \uc774\uc0c1\uc758 \uc77c\uc790\ub9ac\ub97c \ucc3d\ucd9c\ud558\uace0 \uac1c\ud1b5 \uccab \ud574\uc5d0 120\ub9cc \ub300\uc758 \uc790\uac00\uc6a9 \uc6b4\ud589\uc744 \uc904\uc77c \uac83\uc73c\ub85c \ubcf4\uc778\ub2e4\uba70 \uacf5\ud56d\ucca0\ub3c4\ub294 \uc628\ud0c0\ub9ac\uc624 \uc8fc\uc758 \uacbd\uc81c \ubc1c\uc804\uc5d0 \uc801\uc9c0 \uc54a\uc740 \uae30\uc5ec\ub97c \ud560 \uac83\uc774\ub77c\uace0 \ud3c9\ud588\ub2e4.", "paragraph_sentence": "2012\ub144 9\uc6d4, \uc628\ud0c0\ub9ac\uc624 \uc8fc\ub294 \uacf5\ud56d\ucca0\ub3c4 \uacf5\uc0ac\uc5d0 \uccab \uc0bd\uc744 \ub4e4\uc5c8\ub2e4. \ub3cc\ud134 \ub9e5\uadc4\ud2f0 \uc8fc\uc9c0\uc0ac\ub294 \uc131\uba85\ubb38\uc744 \ud1b5\ud574 25km\uc758 \uc774 \ub178\uc120\uc740 1200\uac1c \uc774\uc0c1\uc758 \uc77c\uc790\ub9ac\ub97c \ucc3d\ucd9c\ud558\uace0 \uac1c\ud1b5 \uccab \ud574\uc5d0 120\ub9cc \ub300\uc758 \uc790\uac00\uc6a9 \uc6b4\ud589\uc744 \uc904\uc77c \uac83\uc73c\ub85c \ubcf4\uc778\ub2e4\uba70 \uacf5\ud56d\ucca0\ub3c4\ub294 \uc628\ud0c0\ub9ac\uc624 \uc8fc\uc758 \uacbd\uc81c \ubc1c\uc804\uc5d0 \uc801\uc9c0 \uc54a\uc740 \uae30\uc5ec\ub97c \ud560 \uac83\uc774\ub77c\uace0 \ud3c9\ud588\ub2e4. \uc774\ud6c4 2015\ub144 2\uc6d4 23\uc77c, \uacf5\uc0ac\uac00 \uac70\uc758 \ub9c8\ubb34\ub9ac\ub41c UP \uc775\uc2a4\ud504\ub808\uc2a4\ub294 \ube0c\ub808\uc774\ud06c\uc640 \uc18d\ub825 \uc870\uc808, \uc2a4\uc704\uce58\uc640 \uc2e0\ud638 \ubd88\ub7c9 \ub4f1\uc758 \uc0c1\ud669\uc744 \ub300\ube44\ud574 \uc2dc\ubc94\uc6b4\ud589\uc5d0 \ub3cc\uc785\ud588\ub2e4. \uac19\uc740 \ud574 4\uc6d4 22\uc77c, \uc708 \uc8fc\uc9c0\uc0ac\ub294 \uad50\ud1b5\ubd80 \uc7a5\uad00 \uc2a4\ud2f0\ube10 \ub378 \ub450\uce74\uc640 \uba54\ud2b8\ub85c\ub9c1\uc2a4 \ub300\ud45c \ube0c\ub8e8\uc2a4 \ub9e4\ucfe0\uc5d0\uc774\uadf8\uc640 \ud568\uaed8 \uacf5\ud56d\ucca0\ub3c4\uc5d0 \uc2dc\uc2b9\ud558\uc5ec \uc720\ub2c8\uc5b8 \ud53c\uc5b4\uc2a8 \uae09\ud589\uc758 \ucd5c\uc885 \uac1c\ud1b5\uc77c\uc744 2015\ub144 6\uc6d4 6\uc77c\ub85c \ud655\uc815\uc9c0\uc5c8\ub2e4.", "paragraph_answer": "2012\ub144 9\uc6d4, \uc628\ud0c0\ub9ac\uc624 \uc8fc\ub294 \uacf5\ud56d\ucca0\ub3c4 \uacf5\uc0ac\uc5d0 \uccab \uc0bd\uc744 \ub4e4\uc5c8\ub2e4. \ub3cc\ud134 \ub9e5\uadc4\ud2f0 \uc8fc\uc9c0\uc0ac\ub294 \uc131\uba85\ubb38\uc744 \ud1b5\ud574 25km\uc758 \uc774 \ub178\uc120\uc740 1200\uac1c \uc774\uc0c1\uc758 \uc77c\uc790\ub9ac\ub97c \ucc3d\ucd9c\ud558\uace0 \uac1c\ud1b5 \uccab \ud574\uc5d0 120\ub9cc \ub300\uc758 \uc790\uac00\uc6a9 \uc6b4\ud589\uc744 \uc904\uc77c \uac83\uc73c\ub85c \ubcf4\uc778\ub2e4\uba70 \uacf5\ud56d\ucca0\ub3c4\ub294 \uc628\ud0c0\ub9ac\uc624 \uc8fc\uc758 \uacbd\uc81c \ubc1c\uc804\uc5d0 \uc801\uc9c0 \uc54a\uc740 \uae30\uc5ec\ub97c \ud560 \uac83\uc774\ub77c\uace0 \ud3c9\ud588\ub2e4. \uc774\ud6c4 2015\ub144 2\uc6d4 23\uc77c, \uacf5\uc0ac\uac00 \uac70\uc758 \ub9c8\ubb34\ub9ac\ub41c UP \uc775\uc2a4\ud504\ub808\uc2a4\ub294 \ube0c\ub808\uc774\ud06c\uc640 \uc18d\ub825 \uc870\uc808, \uc2a4\uc704\uce58\uc640 \uc2e0\ud638 \ubd88\ub7c9 \ub4f1\uc758 \uc0c1\ud669\uc744 \ub300\ube44\ud574 \uc2dc\ubc94\uc6b4\ud589\uc5d0 \ub3cc\uc785\ud588\ub2e4. \uac19\uc740 \ud574 4\uc6d4 22\uc77c, \uc708 \uc8fc\uc9c0\uc0ac\ub294 \uad50\ud1b5\ubd80 \uc7a5\uad00 \uc2a4\ud2f0\ube10 \ub378 \ub450\uce74\uc640 \uba54\ud2b8\ub85c\ub9c1\uc2a4 \ub300\ud45c \ube0c\ub8e8\uc2a4 \ub9e4\ucfe0\uc5d0\uc774\uadf8\uc640 \ud568\uaed8 \uacf5\ud56d\ucca0\ub3c4\uc5d0 \uc2dc\uc2b9\ud558\uc5ec \uc720\ub2c8\uc5b8 \ud53c\uc5b4\uc2a8 \uae09\ud589\uc758 \ucd5c\uc885 \uac1c\ud1b5\uc77c\uc744 2015\ub144 6\uc6d4 6\uc77c\ub85c \ud655\uc815\uc9c0\uc5c8\ub2e4.", "sentence_answer": " \ub3cc\ud134 \ub9e5\uadc4\ud2f0 \uc8fc\uc9c0\uc0ac\ub294 \uc131\uba85\ubb38\uc744 \ud1b5\ud574 25km\uc758 \uc774 \ub178\uc120\uc740 1200\uac1c \uc774\uc0c1\uc758 \uc77c\uc790\ub9ac\ub97c \ucc3d\ucd9c\ud558\uace0 \uac1c\ud1b5 \uccab \ud574\uc5d0 120\ub9cc \ub300\uc758 \uc790\uac00\uc6a9 \uc6b4\ud589\uc744 \uc904\uc77c \uac83\uc73c\ub85c \ubcf4\uc778\ub2e4\uba70 \uacf5\ud56d\ucca0\ub3c4\ub294 \uc628\ud0c0\ub9ac\uc624 \uc8fc\uc758 \uacbd\uc81c \ubc1c\uc804\uc5d0 \uc801\uc9c0 \uc54a\uc740 \uae30\uc5ec\ub97c \ud560 \uac83\uc774\ub77c\uace0 \ud3c9\ud588\ub2e4.", "paragraph_id": "6527049-0-0", "paragraph_question": "question: 2012\ub144 9\uc6d4 \uacf5\ud56d\ucca0\ub3c4 \uacf5\uc0ac\ub97c \uc2dc\uc791\ud55c \uc628\ud0c0\ub9ac\uc624 \uc8fc\uc758 \uc8fc\uc9c0\uc0ac\ub294?, context: 2012\ub144 9\uc6d4, \uc628\ud0c0\ub9ac\uc624 \uc8fc\ub294 \uacf5\ud56d\ucca0\ub3c4 \uacf5\uc0ac\uc5d0 \uccab \uc0bd\uc744 \ub4e4\uc5c8\ub2e4. \ub3cc\ud134 \ub9e5\uadc4\ud2f0 \uc8fc\uc9c0\uc0ac\ub294 \uc131\uba85\ubb38\uc744 \ud1b5\ud574 25km\uc758 \uc774 \ub178\uc120\uc740 1200\uac1c \uc774\uc0c1\uc758 \uc77c\uc790\ub9ac\ub97c \ucc3d\ucd9c\ud558\uace0 \uac1c\ud1b5 \uccab \ud574\uc5d0 120\ub9cc \ub300\uc758 \uc790\uac00\uc6a9 \uc6b4\ud589\uc744 \uc904\uc77c \uac83\uc73c\ub85c \ubcf4\uc778\ub2e4\uba70 \uacf5\ud56d\ucca0\ub3c4\ub294 \uc628\ud0c0\ub9ac\uc624 \uc8fc\uc758 \uacbd\uc81c \ubc1c\uc804\uc5d0 \uc801\uc9c0 \uc54a\uc740 \uae30\uc5ec\ub97c \ud560 \uac83\uc774\ub77c\uace0 \ud3c9\ud588\ub2e4. \uc774\ud6c4 2015\ub144 2\uc6d4 23\uc77c, \uacf5\uc0ac\uac00 \uac70\uc758 \ub9c8\ubb34\ub9ac\ub41c UP \uc775\uc2a4\ud504\ub808\uc2a4\ub294 \ube0c\ub808\uc774\ud06c\uc640 \uc18d\ub825 \uc870\uc808, \uc2a4\uc704\uce58\uc640 \uc2e0\ud638 \ubd88\ub7c9 \ub4f1\uc758 \uc0c1\ud669\uc744 \ub300\ube44\ud574 \uc2dc\ubc94\uc6b4\ud589\uc5d0 \ub3cc\uc785\ud588\ub2e4. \uac19\uc740 \ud574 4\uc6d4 22\uc77c, \uc708 \uc8fc\uc9c0\uc0ac\ub294 \uad50\ud1b5\ubd80 \uc7a5\uad00 \uc2a4\ud2f0\ube10 \ub378 \ub450\uce74\uc640 \uba54\ud2b8\ub85c\ub9c1\uc2a4 \ub300\ud45c \ube0c\ub8e8\uc2a4 \ub9e4\ucfe0\uc5d0\uc774\uadf8\uc640 \ud568\uaed8 \uacf5\ud56d\ucca0\ub3c4\uc5d0 \uc2dc\uc2b9\ud558\uc5ec \uc720\ub2c8\uc5b8 \ud53c\uc5b4\uc2a8 \uae09\ud589\uc758 \ucd5c\uc885 \uac1c\ud1b5\uc77c\uc744 2015\ub144 6\uc6d4 6\uc77c\ub85c \ud655\uc815\uc9c0\uc5c8\ub2e4."} {"question": "\ud53c\uaca8\uc2a4\ucf00\uc774\ud305 \uc0ac\uc0c1 \uac00\uc7a5 \uc758\ubb38\uc2a4\ub7ec\uc6b4 \ud310\uc815\uc774\ub77c\uace0 \ud3c9\ud55c \ubbf8\uad6d\uc758 \uc77c\uac04\uc9c0 \uc774\ub984\uc740?", "paragraph": "\uacbd\uae30 \ud6c4 \ud310\uc815 \ub17c\ub780\uc774 \uc788\uc5c8\ub2e4. \ubbf8\uad6d CBS\ub294 \uae40\uc5f0\uc544\uc758 \ud504\ub9ac \uacbd\uae30\uac00 \ub05d\ub098\uc790 \"\uc62c\ub9bc\ud53d 2\uc5f0\ud328\ub97c \ub2ec\uc131\ud55c \uae08\uba54\ub2ec\ub9ac\uc2a4\ud2b8\uac00 \ub098\uc654\ub2e4\"\ub294 \ubc18\uc751\uc744 \ubcf4\uc600\uc9c0\ub9cc \uacb0\uacfc\uac00 \ubc1c\ud45c\ub418\uc790 \"\uba54\ub2ec\uc744 \ub538 \uc904\uc740 \uc54c\uc558\uc9c0\ub9cc \uba54\ub2ec \uc0c9\uc774 \uc740\ube5b\uc774 \uc544\ub2c8\uc5c8\ub2e4\"\ub294 \ubc18\uc751\uc744 \ubcf4\uc600\ub2e4. \ubbf8\uad6d NBC\ub294 \uacf5\uc2dd\ud2b8\uc704\ud130\ub97c \ud1b5\ud574 \"\uacb0\uacfc\uc5d0 \ub3d9\uc758\ud558\uc2ed\ub2c8\uae4c?\"\uace0 \ubb3c\uc5c8\ub2e4. \ub610\ud55c NBC\uc758 \uc62c\ub9bc\ud53d \ub9ac\uc11c\ucc98\uc778 \uc54c\ub809\uc2a4 \uace8\ub4dc\ubc84\uac70\ub294 \ud2b8\uc704\ud130\ub97c \ud1b5\ud574 \"\uc544\ub378\ub9ac\ub098 \uc18c\ud2b8\ub2c8\ucf54\ubc14\ub294 \uc624\ub298 \ud6cc\ub96d\ud588\ub2e4. \ud558\uc9c0\ub9cc \uae40\uc5f0\uc544\ub294 \uae08\uba54\ub2ec\uc744 \ub3c4\ub451\ub9de\uc558\ub2e4\"\ub294 \uae00\uc744 \uc62c\ub9ac\uba70 \uae40\uc5f0\uc544\uc758 \uc740\uba54\ub2ec\uc5d0 \ub300\ud55c \ubc18\uc751\uc744 \ubcf4\uc600\ub2e4. AFP \ud1b5\uc2e0\uc0ac\ub3c4 \uc774\ub0a0 \uacbd\uae30 \uacb0\uacfc\uc5d0 \ub300\ud574 \"\uc18c\ud2b8\ub2c8\ucf54\ubc14\uac00 \uae40\uc5f0\uc544\ub97c \uc0c1\ub300\ub85c \ub17c\ub780\uc774 \ub9ce\uc740 \uae08\uba54\ub2ec\uc744 \ucc28\uc9c0\ud588\ub2e4\"\uace0 \uc804\ud558\uba70 \"\uc18c\ud2b8\ub2c8\ucf54\ubc14\ub294 \ub354\ube14 \ub8e8\ud504\ub97c \ub6f0\uba74\uc11c \ucc29\uc9c0 \uc2e4\uc218\uac00 \uc788\uc5c8\uc9c0\ub9cc, \uae40\uc5f0\uc544\uc640 \ub3d9\uba54\ub2ec\ub9ac\uc2a4\ud2b8 \uce74\ub864\ub9ac\ub098 \ucf54\uc2a4\ud2b8\ub108\ub294 \uc2e4\uc218\uac00 \uc5c6\ub294 \uc644\ubcbd\ud55c \uc5f0\uae30\ub97c \ud3bc\ucce4\ub2e4\"\uace0 \ud3c9\ud588\ub2e4. \ubbf8\uad6d \uc77c\uac04\uc9c0 \uc2dc\uce74\uace0 \ud2b8\ub9ac\ubdf4\uc740 \"\uc18c\ud2b8\ub2c8\ucf54\ubc14\uac00 \ud3b8\ud30c \ud310\uc815 \ub355\uc5d0 \ub7ec\uc2dc\uc544 \uc120\uc218\ub85c\ub294 \ucd5c\ucd08\ub85c \uc5ec\uc790 \ud53c\uaca8 \uae08\uba54\ub2ec\ub9ac\uc2a4\ud2b8\uac00 \ub410\ub2e4\"\uba70 \"\ud53c\uaca8\uc2a4\ucf00\uc774\ud305 \uc0ac\uc0c1 \uac00\uc7a5 \uc758\ubb38\uc2a4\ub7ec\uc6b4\ud310\uc815\"\uc774\ub77c\uace0 \ud3c9\ud588\ub2e4. \ubbf8\uad6d \uacbd\uc81c \uc804\ubb38\uc9c0 \uc6d4\uc2a4\ud2b8\ub9ac\ud2b8\uc800\ub110 \uacb0\uacfc\ub97c \uc2e4\uc2dc\uac04\uc73c\ub85c \uc804\ud558\ub294 \ub77c\uc774\ube0c \ube14\ub85c\uadf8\uc5d0\uc11c \uae40\uc5f0\uc544\uc758 \uc810\uc218\uac00 \ubc1c\ud45c\ub418\uc790 '\ucda9\uaca9'\uc774\ub77c \ub9d0\ud588\ub2e4.", "answer": "\uc2dc\uce74\uace0 \ud2b8\ub9ac\ubdf4", "sentence": "\ubbf8\uad6d \uc77c\uac04\uc9c0 \uc2dc\uce74\uace0 \ud2b8\ub9ac\ubdf4 \uc740 \"\uc18c\ud2b8\ub2c8\ucf54\ubc14\uac00 \ud3b8\ud30c \ud310\uc815 \ub355\uc5d0 \ub7ec\uc2dc\uc544 \uc120\uc218\ub85c\ub294 \ucd5c\ucd08\ub85c \uc5ec\uc790 \ud53c\uaca8 \uae08\uba54\ub2ec\ub9ac\uc2a4\ud2b8\uac00 \ub410\ub2e4\"\uba70 \"\ud53c\uaca8\uc2a4\ucf00\uc774\ud305 \uc0ac\uc0c1 \uac00\uc7a5 \uc758\ubb38\uc2a4\ub7ec\uc6b4\ud310\uc815\"\uc774\ub77c\uace0 \ud3c9\ud588\ub2e4.", "paragraph_sentence": "\uacbd\uae30 \ud6c4 \ud310\uc815 \ub17c\ub780\uc774 \uc788\uc5c8\ub2e4. \ubbf8\uad6d CBS\ub294 \uae40\uc5f0\uc544\uc758 \ud504\ub9ac \uacbd\uae30\uac00 \ub05d\ub098\uc790 \"\uc62c\ub9bc\ud53d 2\uc5f0\ud328\ub97c \ub2ec\uc131\ud55c \uae08\uba54\ub2ec\ub9ac\uc2a4\ud2b8\uac00 \ub098\uc654\ub2e4\"\ub294 \ubc18\uc751\uc744 \ubcf4\uc600\uc9c0\ub9cc \uacb0\uacfc\uac00 \ubc1c\ud45c\ub418\uc790 \"\uba54\ub2ec\uc744 \ub538 \uc904\uc740 \uc54c\uc558\uc9c0\ub9cc \uba54\ub2ec \uc0c9\uc774 \uc740\ube5b\uc774 \uc544\ub2c8\uc5c8\ub2e4\"\ub294 \ubc18\uc751\uc744 \ubcf4\uc600\ub2e4. \ubbf8\uad6d NBC\ub294 \uacf5\uc2dd\ud2b8\uc704\ud130\ub97c \ud1b5\ud574 \"\uacb0\uacfc\uc5d0 \ub3d9\uc758\ud558\uc2ed\ub2c8\uae4c?\"\uace0 \ubb3c\uc5c8\ub2e4. \ub610\ud55c NBC\uc758 \uc62c\ub9bc\ud53d \ub9ac\uc11c\ucc98\uc778 \uc54c\ub809\uc2a4 \uace8\ub4dc\ubc84\uac70\ub294 \ud2b8\uc704\ud130\ub97c \ud1b5\ud574 \"\uc544\ub378\ub9ac\ub098 \uc18c\ud2b8\ub2c8\ucf54\ubc14\ub294 \uc624\ub298 \ud6cc\ub96d\ud588\ub2e4. \ud558\uc9c0\ub9cc \uae40\uc5f0\uc544\ub294 \uae08\uba54\ub2ec\uc744 \ub3c4\ub451\ub9de\uc558\ub2e4\"\ub294 \uae00\uc744 \uc62c\ub9ac\uba70 \uae40\uc5f0\uc544\uc758 \uc740\uba54\ub2ec\uc5d0 \ub300\ud55c \ubc18\uc751\uc744 \ubcf4\uc600\ub2e4. AFP \ud1b5\uc2e0\uc0ac\ub3c4 \uc774\ub0a0 \uacbd\uae30 \uacb0\uacfc\uc5d0 \ub300\ud574 \"\uc18c\ud2b8\ub2c8\ucf54\ubc14\uac00 \uae40\uc5f0\uc544\ub97c \uc0c1\ub300\ub85c \ub17c\ub780\uc774 \ub9ce\uc740 \uae08\uba54\ub2ec\uc744 \ucc28\uc9c0\ud588\ub2e4\"\uace0 \uc804\ud558\uba70 \"\uc18c\ud2b8\ub2c8\ucf54\ubc14\ub294 \ub354\ube14 \ub8e8\ud504\ub97c \ub6f0\uba74\uc11c \ucc29\uc9c0 \uc2e4\uc218\uac00 \uc788\uc5c8\uc9c0\ub9cc, \uae40\uc5f0\uc544\uc640 \ub3d9\uba54\ub2ec\ub9ac\uc2a4\ud2b8 \uce74\ub864\ub9ac\ub098 \ucf54\uc2a4\ud2b8\ub108\ub294 \uc2e4\uc218\uac00 \uc5c6\ub294 \uc644\ubcbd\ud55c \uc5f0\uae30\ub97c \ud3bc\ucce4\ub2e4\"\uace0 \ud3c9\ud588\ub2e4. \ubbf8\uad6d \uc77c\uac04\uc9c0 \uc2dc\uce74\uace0 \ud2b8\ub9ac\ubdf4 \uc740 \"\uc18c\ud2b8\ub2c8\ucf54\ubc14\uac00 \ud3b8\ud30c \ud310\uc815 \ub355\uc5d0 \ub7ec\uc2dc\uc544 \uc120\uc218\ub85c\ub294 \ucd5c\ucd08\ub85c \uc5ec\uc790 \ud53c\uaca8 \uae08\uba54\ub2ec\ub9ac\uc2a4\ud2b8\uac00 \ub410\ub2e4\"\uba70 \"\ud53c\uaca8\uc2a4\ucf00\uc774\ud305 \uc0ac\uc0c1 \uac00\uc7a5 \uc758\ubb38\uc2a4\ub7ec\uc6b4\ud310\uc815\"\uc774\ub77c\uace0 \ud3c9\ud588\ub2e4. \ubbf8\uad6d \uacbd\uc81c \uc804\ubb38\uc9c0 \uc6d4\uc2a4\ud2b8\ub9ac\ud2b8\uc800\ub110 \uacb0\uacfc\ub97c \uc2e4\uc2dc\uac04\uc73c\ub85c \uc804\ud558\ub294 \ub77c\uc774\ube0c \ube14\ub85c\uadf8\uc5d0\uc11c \uae40\uc5f0\uc544\uc758 \uc810\uc218\uac00 \ubc1c\ud45c\ub418\uc790 '\ucda9\uaca9'\uc774\ub77c \ub9d0\ud588\ub2e4.", "paragraph_answer": "\uacbd\uae30 \ud6c4 \ud310\uc815 \ub17c\ub780\uc774 \uc788\uc5c8\ub2e4. \ubbf8\uad6d CBS\ub294 \uae40\uc5f0\uc544\uc758 \ud504\ub9ac \uacbd\uae30\uac00 \ub05d\ub098\uc790 \"\uc62c\ub9bc\ud53d 2\uc5f0\ud328\ub97c \ub2ec\uc131\ud55c \uae08\uba54\ub2ec\ub9ac\uc2a4\ud2b8\uac00 \ub098\uc654\ub2e4\"\ub294 \ubc18\uc751\uc744 \ubcf4\uc600\uc9c0\ub9cc \uacb0\uacfc\uac00 \ubc1c\ud45c\ub418\uc790 \"\uba54\ub2ec\uc744 \ub538 \uc904\uc740 \uc54c\uc558\uc9c0\ub9cc \uba54\ub2ec \uc0c9\uc774 \uc740\ube5b\uc774 \uc544\ub2c8\uc5c8\ub2e4\"\ub294 \ubc18\uc751\uc744 \ubcf4\uc600\ub2e4. \ubbf8\uad6d NBC\ub294 \uacf5\uc2dd\ud2b8\uc704\ud130\ub97c \ud1b5\ud574 \"\uacb0\uacfc\uc5d0 \ub3d9\uc758\ud558\uc2ed\ub2c8\uae4c?\"\uace0 \ubb3c\uc5c8\ub2e4. \ub610\ud55c NBC\uc758 \uc62c\ub9bc\ud53d \ub9ac\uc11c\ucc98\uc778 \uc54c\ub809\uc2a4 \uace8\ub4dc\ubc84\uac70\ub294 \ud2b8\uc704\ud130\ub97c \ud1b5\ud574 \"\uc544\ub378\ub9ac\ub098 \uc18c\ud2b8\ub2c8\ucf54\ubc14\ub294 \uc624\ub298 \ud6cc\ub96d\ud588\ub2e4. \ud558\uc9c0\ub9cc \uae40\uc5f0\uc544\ub294 \uae08\uba54\ub2ec\uc744 \ub3c4\ub451\ub9de\uc558\ub2e4\"\ub294 \uae00\uc744 \uc62c\ub9ac\uba70 \uae40\uc5f0\uc544\uc758 \uc740\uba54\ub2ec\uc5d0 \ub300\ud55c \ubc18\uc751\uc744 \ubcf4\uc600\ub2e4. AFP \ud1b5\uc2e0\uc0ac\ub3c4 \uc774\ub0a0 \uacbd\uae30 \uacb0\uacfc\uc5d0 \ub300\ud574 \"\uc18c\ud2b8\ub2c8\ucf54\ubc14\uac00 \uae40\uc5f0\uc544\ub97c \uc0c1\ub300\ub85c \ub17c\ub780\uc774 \ub9ce\uc740 \uae08\uba54\ub2ec\uc744 \ucc28\uc9c0\ud588\ub2e4\"\uace0 \uc804\ud558\uba70 \"\uc18c\ud2b8\ub2c8\ucf54\ubc14\ub294 \ub354\ube14 \ub8e8\ud504\ub97c \ub6f0\uba74\uc11c \ucc29\uc9c0 \uc2e4\uc218\uac00 \uc788\uc5c8\uc9c0\ub9cc, \uae40\uc5f0\uc544\uc640 \ub3d9\uba54\ub2ec\ub9ac\uc2a4\ud2b8 \uce74\ub864\ub9ac\ub098 \ucf54\uc2a4\ud2b8\ub108\ub294 \uc2e4\uc218\uac00 \uc5c6\ub294 \uc644\ubcbd\ud55c \uc5f0\uae30\ub97c \ud3bc\ucce4\ub2e4\"\uace0 \ud3c9\ud588\ub2e4. \ubbf8\uad6d \uc77c\uac04\uc9c0 \uc2dc\uce74\uace0 \ud2b8\ub9ac\ubdf4 \uc740 \"\uc18c\ud2b8\ub2c8\ucf54\ubc14\uac00 \ud3b8\ud30c \ud310\uc815 \ub355\uc5d0 \ub7ec\uc2dc\uc544 \uc120\uc218\ub85c\ub294 \ucd5c\ucd08\ub85c \uc5ec\uc790 \ud53c\uaca8 \uae08\uba54\ub2ec\ub9ac\uc2a4\ud2b8\uac00 \ub410\ub2e4\"\uba70 \"\ud53c\uaca8\uc2a4\ucf00\uc774\ud305 \uc0ac\uc0c1 \uac00\uc7a5 \uc758\ubb38\uc2a4\ub7ec\uc6b4\ud310\uc815\"\uc774\ub77c\uace0 \ud3c9\ud588\ub2e4. \ubbf8\uad6d \uacbd\uc81c \uc804\ubb38\uc9c0 \uc6d4\uc2a4\ud2b8\ub9ac\ud2b8\uc800\ub110 \uacb0\uacfc\ub97c \uc2e4\uc2dc\uac04\uc73c\ub85c \uc804\ud558\ub294 \ub77c\uc774\ube0c \ube14\ub85c\uadf8\uc5d0\uc11c \uae40\uc5f0\uc544\uc758 \uc810\uc218\uac00 \ubc1c\ud45c\ub418\uc790 '\ucda9\uaca9'\uc774\ub77c \ub9d0\ud588\ub2e4.", "sentence_answer": "\ubbf8\uad6d \uc77c\uac04\uc9c0 \uc2dc\uce74\uace0 \ud2b8\ub9ac\ubdf4 \uc740 \"\uc18c\ud2b8\ub2c8\ucf54\ubc14\uac00 \ud3b8\ud30c \ud310\uc815 \ub355\uc5d0 \ub7ec\uc2dc\uc544 \uc120\uc218\ub85c\ub294 \ucd5c\ucd08\ub85c \uc5ec\uc790 \ud53c\uaca8 \uae08\uba54\ub2ec\ub9ac\uc2a4\ud2b8\uac00 \ub410\ub2e4\"\uba70 \"\ud53c\uaca8\uc2a4\ucf00\uc774\ud305 \uc0ac\uc0c1 \uac00\uc7a5 \uc758\ubb38\uc2a4\ub7ec\uc6b4\ud310\uc815\"\uc774\ub77c\uace0 \ud3c9\ud588\ub2e4.", "paragraph_id": "6511758-24-0", "paragraph_question": "question: \ud53c\uaca8\uc2a4\ucf00\uc774\ud305 \uc0ac\uc0c1 \uac00\uc7a5 \uc758\ubb38\uc2a4\ub7ec\uc6b4 \ud310\uc815\uc774\ub77c\uace0 \ud3c9\ud55c \ubbf8\uad6d\uc758 \uc77c\uac04\uc9c0 \uc774\ub984\uc740?, context: \uacbd\uae30 \ud6c4 \ud310\uc815 \ub17c\ub780\uc774 \uc788\uc5c8\ub2e4. \ubbf8\uad6d CBS\ub294 \uae40\uc5f0\uc544\uc758 \ud504\ub9ac \uacbd\uae30\uac00 \ub05d\ub098\uc790 \"\uc62c\ub9bc\ud53d 2\uc5f0\ud328\ub97c \ub2ec\uc131\ud55c \uae08\uba54\ub2ec\ub9ac\uc2a4\ud2b8\uac00 \ub098\uc654\ub2e4\"\ub294 \ubc18\uc751\uc744 \ubcf4\uc600\uc9c0\ub9cc \uacb0\uacfc\uac00 \ubc1c\ud45c\ub418\uc790 \"\uba54\ub2ec\uc744 \ub538 \uc904\uc740 \uc54c\uc558\uc9c0\ub9cc \uba54\ub2ec \uc0c9\uc774 \uc740\ube5b\uc774 \uc544\ub2c8\uc5c8\ub2e4\"\ub294 \ubc18\uc751\uc744 \ubcf4\uc600\ub2e4. \ubbf8\uad6d NBC\ub294 \uacf5\uc2dd\ud2b8\uc704\ud130\ub97c \ud1b5\ud574 \"\uacb0\uacfc\uc5d0 \ub3d9\uc758\ud558\uc2ed\ub2c8\uae4c?\"\uace0 \ubb3c\uc5c8\ub2e4. \ub610\ud55c NBC\uc758 \uc62c\ub9bc\ud53d \ub9ac\uc11c\ucc98\uc778 \uc54c\ub809\uc2a4 \uace8\ub4dc\ubc84\uac70\ub294 \ud2b8\uc704\ud130\ub97c \ud1b5\ud574 \"\uc544\ub378\ub9ac\ub098 \uc18c\ud2b8\ub2c8\ucf54\ubc14\ub294 \uc624\ub298 \ud6cc\ub96d\ud588\ub2e4. \ud558\uc9c0\ub9cc \uae40\uc5f0\uc544\ub294 \uae08\uba54\ub2ec\uc744 \ub3c4\ub451\ub9de\uc558\ub2e4\"\ub294 \uae00\uc744 \uc62c\ub9ac\uba70 \uae40\uc5f0\uc544\uc758 \uc740\uba54\ub2ec\uc5d0 \ub300\ud55c \ubc18\uc751\uc744 \ubcf4\uc600\ub2e4. AFP \ud1b5\uc2e0\uc0ac\ub3c4 \uc774\ub0a0 \uacbd\uae30 \uacb0\uacfc\uc5d0 \ub300\ud574 \"\uc18c\ud2b8\ub2c8\ucf54\ubc14\uac00 \uae40\uc5f0\uc544\ub97c \uc0c1\ub300\ub85c \ub17c\ub780\uc774 \ub9ce\uc740 \uae08\uba54\ub2ec\uc744 \ucc28\uc9c0\ud588\ub2e4\"\uace0 \uc804\ud558\uba70 \"\uc18c\ud2b8\ub2c8\ucf54\ubc14\ub294 \ub354\ube14 \ub8e8\ud504\ub97c \ub6f0\uba74\uc11c \ucc29\uc9c0 \uc2e4\uc218\uac00 \uc788\uc5c8\uc9c0\ub9cc, \uae40\uc5f0\uc544\uc640 \ub3d9\uba54\ub2ec\ub9ac\uc2a4\ud2b8 \uce74\ub864\ub9ac\ub098 \ucf54\uc2a4\ud2b8\ub108\ub294 \uc2e4\uc218\uac00 \uc5c6\ub294 \uc644\ubcbd\ud55c \uc5f0\uae30\ub97c \ud3bc\ucce4\ub2e4\"\uace0 \ud3c9\ud588\ub2e4. \ubbf8\uad6d \uc77c\uac04\uc9c0 \uc2dc\uce74\uace0 \ud2b8\ub9ac\ubdf4\uc740 \"\uc18c\ud2b8\ub2c8\ucf54\ubc14\uac00 \ud3b8\ud30c \ud310\uc815 \ub355\uc5d0 \ub7ec\uc2dc\uc544 \uc120\uc218\ub85c\ub294 \ucd5c\ucd08\ub85c \uc5ec\uc790 \ud53c\uaca8 \uae08\uba54\ub2ec\ub9ac\uc2a4\ud2b8\uac00 \ub410\ub2e4\"\uba70 \"\ud53c\uaca8\uc2a4\ucf00\uc774\ud305 \uc0ac\uc0c1 \uac00\uc7a5 \uc758\ubb38\uc2a4\ub7ec\uc6b4\ud310\uc815\"\uc774\ub77c\uace0 \ud3c9\ud588\ub2e4. \ubbf8\uad6d \uacbd\uc81c \uc804\ubb38\uc9c0 \uc6d4\uc2a4\ud2b8\ub9ac\ud2b8\uc800\ub110 \uacb0\uacfc\ub97c \uc2e4\uc2dc\uac04\uc73c\ub85c \uc804\ud558\ub294 \ub77c\uc774\ube0c \ube14\ub85c\uadf8\uc5d0\uc11c \uae40\uc5f0\uc544\uc758 \uc810\uc218\uac00 \ubc1c\ud45c\ub418\uc790 '\ucda9\uaca9'\uc774\ub77c \ub9d0\ud588\ub2e4."} {"question": "1923\ub144 \uc789\uae00\ub79c\ub4dc FA\ucef5 \uacb0\uc2b9\uc804\uc5d0\uc11c \uc870\uc9c0 5\uc138\uc5d0\uac8c \uc6b0\uc2b9 \ud2b8\ub85c\ud53c\ub97c \ubc1b\uc740 \uc120\uc218\ub294?", "paragraph": "\uc6e8\uc2a4\ud2b8 \ud584\uc740 \ud6c4\ubc18\uc804\uc5d0\ub294 \ubcf4\ub2e4 \ub354 \uac15\ud55c \ubaa8\uc2b5\uc744 \ubcf4\uc5ec\uc92c\ub294\ub370, \ud2b9\ud788 \ube45 \uc653\uc2a8\uc774 \uc88b\uc740 \uc790\ub9ac\uc5d0\uc11c \uace8 \ucc2c\uc2a4\ub97c \ub9de\uc558\uc73c\ub098 \uc29b\uc774 \uace8\ubb38\uc744 \ube57\ub098\uac14\ub2e4. \uadf8\ub7ec\ub098 \ud6c4\ubc18 8\ubd84\uc5d0 \ubcfc\ud134\uc774 \uc6e8\uc2a4\ud2b8 \ud584\uc758 \uac70\uc13c \uc800\ud56d\uc744 \ubfcc\ub9ac\uce58\uace0 \uc410\uae30\uace8\uc744 \ub123\uc5c8\ub2e4. \uc544\uc6c3\uc0ac\uc774\ub4dc \ud3ec\uc6cc\ub4dc \ud14c\ub4dc \ube44\uc790\ub4dc\uac00 \uacf5\uc744 \uacbd\uae30\uc7a5 \uc911\uc559\uc5d0 \ud328\uc2a4\ud588\uace0, \ud328\uc2a4\ub97c \ubc1b\uc740 \uc7ad \uc2a4\ubbf8\uc2a4\uac00 \uace8\ud0a4\ud37c \ud5c8\ud504\uc2a8\uc774 \ucc98\ub9ac\ud558\uae30 \uc55e\uc11c \uc29b\uc744 \uc131\uacf5\uc2dc\ucf30\ub2e4 \uace8\uc744 \ub123\uc5c8\ub2e4. \uadf8 \ub54c \uacf5\uc774 \ub2e4\uc2dc \ud295\uaca8 \ub098\uc654\ub294\ub370, \uc6e8\uc2a4\ud2b8 \ud584 \uc120\uc218\ub4e4\uc740 \uacf5\uc774 \uace8\ub77c\uc778\uc744 \uc9c0\ub098\uc9c0 \uc54a\uace0 \uace8\ud3ec\uc2a4\ud2b8\ub97c \ub9de\uace0 \ud280\uc5b4 \ub098\uc654\ub2e4\uace0 \uac70\uc138\uac8c \ud56d\uc758\ud588\uc73c\ub098 \uc8fc\uc2ec D. H. \uc560\uc2a8\uc740 \uc5b4\ud544\uc744 \ubb35\uc0b4\ud558\uace0 \uacf5\uc774 \uace8\ub77c\uc778\uc744 \uc9c0\ub09c \ub4a4\uc5d0 \ub2e4\uc2dc \ud295\uaca8 \ub098\uc654\ub2e4\uace0 \ud310\ub2e8\ud588\ub2e4. \uc6e8\uc2a4\ud2b8 \ud584\uc740 \ub610\ud55c \ubcfc\ud134\uc774 \ubd88\uacf5\ud3c9\ud55c \uc5b4\ub4dc\ubca4\ud2f0\uc9c0\ub97c \ubc1b\uc558\ub2e4\uace0 \ud56d\uc758\ud588\ub294\ub370, \ud56d\uc758 \ub0b4\uc6a9\uc740 \ube44\uc790\ub4dc\uc5d0\uac8c \uacf5\uc744 \ucc28\uc900 \uc0ac\ub78c\uc774 \ud53c\uce58 \uc8fc\ubcc0\uc5d0 \uc11c\uc788\ub358 \ubcfc\ud134\uc758 \ud32c\uc774\uc5c8\ub2e4\ub294 \uac83\uc774\ub2e4. \ud558\uc9c0\ub9cc \uc560\uc2a8\uc740 \uc774 \ud56d\uc758\ub3c4 \uae30\uac01\ud558\uace0 \ubcfc\ud134\uc758 \ub450 \ubc88\uc9f8 \uace8\uc744 \ucd5c\uc885\uc801\uc73c\ub85c \uc778\uc815\ud558\uc600\ub2e4. \uc774\ub54c\ubd80\ud130 \ubcfc\ud134\uc758 \uc2b9\ub9ac\ub97c \uc608\uc0c1\ud55c \ub9ce\uc740 \uad00\uc911\ub4e4\uc774 \uacbd\uae30\uc7a5\uc744 \uc77c\ucc0d \ube60\uc838\ub098\uac14\ub2e4. \ubcfc\ud134\uc758 \ub450 \ubc88\uc9f8 \ub4dd\uc810 \uc774\ud6c4 \uc591 \ud300 \ubaa8\ub450 \uc88b\uc740 \uace8\ucc2c\uc2a4\ub97c \uc7a1\uc9c0 \ubabb\ud558\uace0 \uad50\ucc29 \uc0c1\ud0dc\uc5d0 \ube60\uc838 \ubcc4\ub2e4\ub978 \ud765\ubbf8\ub97c \uc77c\uc73c\ud0a4\uc9c0 \ubabb\ud588\ub2e4. \uacbd\uae30 \ud6c4\ubc18\uc5d0\ub294 \uc6e8\uc2a4\ud2b8 \ud584\uc758 \uc8fc\uc7a5\uc778 \uc870\uc9c0 \ucf00\uc774\uac00 \uc8fc\uc2ec\uc5d0\uac8c \uacbd\uae30\ub97c \uc77c\ucc0d \ub05d\ub0b4\ub77c\uace0 \uc694\uad6c\ud588\uc73c\ub098, \uc18c\ubb38\uc5d0 \uc758\ud558\uba74 \ubcfc\ud134\uc758 \uc8fc\uc7a5 \uc870 \uc2a4\ubbf8\uc2a4\uac00 \uc8fc\uc2ec\uc5d0\uac8c \"\uc6b0\ub9ac\ub294 \uc798 \ud558\uace0 \uc788\uc5b4\uc694. \uc6b0\ub9ac\ub294 \ud544\uc694\ud558\ub2e4\uba74 \ud574\uac00 \uc9c8 \ub54c \uae4c\uc9c0\ub77c\ub3c4 \ud50c\ub808\uc774\ub97c \uacc4\uc18d \ud560 \uac81\ub2c8\ub2e4.\"\ub77c\uace0 \ub300\ub2f5\ud588\ub2e4\uace0 \ud55c\ub2e4. \uc774\uc73d\uace0 \uacbd\uae30 \uc885\ub8cc\uac00 \uac00\uae4c\uc6cc \uc8fc\uc2ec\uc774 \ub9c8\uc9c0\ub9c9\uc744 \ud718\uc2ac\uc744 \ubd88\uc790 2:0\uc73c\ub85c \ubcfc\ud134\uc774 \uc2b9\ub9ac\ud558\uc5ec \uc6b0\uc2b9\ucef5\uc744 \ucc28\uc9c0\ud588\ub2e4. \uc601\uad6d\uc758 \uc655 \uc870\uc9c0 5\uc138\uac00 FA\ucef5 \uc6b0\uc2b9 \ud2b8\ub85c\ud53c\ub97c \uc870 \uc2a4\ubbf8\uc2a4\uc5d0\uac8c \uc218\uc5ec\ud588\uace0 \uacbd\uae30\uc7a5\uc758 \uad00\uc911\ub4e4\uc758 \ud658\ud638\ub97c \uc720\ub3c4\ud588\ub2e4. \ud55c\ud3b8 \uc6e8\uc2a4\ud2b8 \ud584\uc758 \ud2b8\ub808\uc774\ub108 \ucc30\ub9ac \ud398\uc778\ud130\ub294 \"\ud770 \ub9d0\ub4e4\uc774 \uadf8 \ud070 \ubc1c\ub4e4\ub85c \ud53c\uce58(\uadf8\ub77c\uc6b4\ub4dc)\ub97c \ucff5\ucff5\uac70\ub9ac\uba70 \ub3cc\uc544\ub2e4\ub2c8\uba74\uc11c \uacbd\uae30\ub97c \uc808\ub9dd\uc801\uc73c\ub85c \ub9cc\ub4e4\uc5b4\ubc84\ub838\ub2e4. \uc6b0\ub9ac \ud300\uc758 \uc719\uc5b4\ub4e4\uc740 \uacc4\uc18d\ud574\uc11c \ub118\uc5b4\uc84c\uc73c\uba70 \ud53c\uce58\uc5d0 \ud30c\uc778 \ud648\uacfc \uad6c\uba4d\ub4e4\ub85c \uc778\ud574 \ub9ce\uc740 \uc2e4\uc218\ub97c \uc800\uc9c8\ub800\ub2e4\"\uba70 \uacbd\uae30 \uc2dc\uc791 \uc804\uc5d0 \ud6fc\uc190\ub41c \ud53c\uce58\uc758 \uc0c1\ud0dc\ub97c \ud300\uc758 \ud328\ubc30 \uc6d0\uc778\uc73c\ub85c \uaf3d\uc558\ub2e4.", "answer": "\uc870 \uc2a4\ubbf8\uc2a4", "sentence": "\ubcfc\ud134\uc758 \uc8fc\uc7a5 \uc870 \uc2a4\ubbf8\uc2a4 \uac00 \uc8fc\uc2ec\uc5d0\uac8c \"\uc6b0\ub9ac\ub294 \uc798 \ud558\uace0 \uc788\uc5b4\uc694.", "paragraph_sentence": "\uc6e8\uc2a4\ud2b8 \ud584\uc740 \ud6c4\ubc18\uc804\uc5d0\ub294 \ubcf4\ub2e4 \ub354 \uac15\ud55c \ubaa8\uc2b5\uc744 \ubcf4\uc5ec\uc92c\ub294\ub370, \ud2b9\ud788 \ube45 \uc653\uc2a8\uc774 \uc88b\uc740 \uc790\ub9ac\uc5d0\uc11c \uace8 \ucc2c\uc2a4\ub97c \ub9de\uc558\uc73c\ub098 \uc29b\uc774 \uace8\ubb38\uc744 \ube57\ub098\uac14\ub2e4. \uadf8\ub7ec\ub098 \ud6c4\ubc18 8\ubd84\uc5d0 \ubcfc\ud134\uc774 \uc6e8\uc2a4\ud2b8 \ud584\uc758 \uac70\uc13c \uc800\ud56d\uc744 \ubfcc\ub9ac\uce58\uace0 \uc410\uae30\uace8\uc744 \ub123\uc5c8\ub2e4. \uc544\uc6c3\uc0ac\uc774\ub4dc \ud3ec\uc6cc\ub4dc \ud14c\ub4dc \ube44\uc790\ub4dc\uac00 \uacf5\uc744 \uacbd\uae30\uc7a5 \uc911\uc559\uc5d0 \ud328\uc2a4\ud588\uace0, \ud328\uc2a4\ub97c \ubc1b\uc740 \uc7ad \uc2a4\ubbf8\uc2a4\uac00 \uace8\ud0a4\ud37c \ud5c8\ud504\uc2a8\uc774 \ucc98\ub9ac\ud558\uae30 \uc55e\uc11c \uc29b\uc744 \uc131\uacf5\uc2dc\ucf30\ub2e4 \uace8\uc744 \ub123\uc5c8\ub2e4. \uadf8 \ub54c \uacf5\uc774 \ub2e4\uc2dc \ud295\uaca8 \ub098\uc654\ub294\ub370, \uc6e8\uc2a4\ud2b8 \ud584 \uc120\uc218\ub4e4\uc740 \uacf5\uc774 \uace8\ub77c\uc778\uc744 \uc9c0\ub098\uc9c0 \uc54a\uace0 \uace8\ud3ec\uc2a4\ud2b8\ub97c \ub9de\uace0 \ud280\uc5b4 \ub098\uc654\ub2e4\uace0 \uac70\uc138\uac8c \ud56d\uc758\ud588\uc73c\ub098 \uc8fc\uc2ec D. H. \uc560\uc2a8\uc740 \uc5b4\ud544\uc744 \ubb35\uc0b4\ud558\uace0 \uacf5\uc774 \uace8\ub77c\uc778\uc744 \uc9c0\ub09c \ub4a4\uc5d0 \ub2e4\uc2dc \ud295\uaca8 \ub098\uc654\ub2e4\uace0 \ud310\ub2e8\ud588\ub2e4. \uc6e8\uc2a4\ud2b8 \ud584\uc740 \ub610\ud55c \ubcfc\ud134\uc774 \ubd88\uacf5\ud3c9\ud55c \uc5b4\ub4dc\ubca4\ud2f0\uc9c0\ub97c \ubc1b\uc558\ub2e4\uace0 \ud56d\uc758\ud588\ub294\ub370, \ud56d\uc758 \ub0b4\uc6a9\uc740 \ube44\uc790\ub4dc\uc5d0\uac8c \uacf5\uc744 \ucc28\uc900 \uc0ac\ub78c\uc774 \ud53c\uce58 \uc8fc\ubcc0\uc5d0 \uc11c\uc788\ub358 \ubcfc\ud134\uc758 \ud32c\uc774\uc5c8\ub2e4\ub294 \uac83\uc774\ub2e4. \ud558\uc9c0\ub9cc \uc560\uc2a8\uc740 \uc774 \ud56d\uc758\ub3c4 \uae30\uac01\ud558\uace0 \ubcfc\ud134\uc758 \ub450 \ubc88\uc9f8 \uace8\uc744 \ucd5c\uc885\uc801\uc73c\ub85c \uc778\uc815\ud558\uc600\ub2e4. \uc774\ub54c\ubd80\ud130 \ubcfc\ud134\uc758 \uc2b9\ub9ac\ub97c \uc608\uc0c1\ud55c \ub9ce\uc740 \uad00\uc911\ub4e4\uc774 \uacbd\uae30\uc7a5\uc744 \uc77c\ucc0d \ube60\uc838\ub098\uac14\ub2e4. \ubcfc\ud134\uc758 \ub450 \ubc88\uc9f8 \ub4dd\uc810 \uc774\ud6c4 \uc591 \ud300 \ubaa8\ub450 \uc88b\uc740 \uace8\ucc2c\uc2a4\ub97c \uc7a1\uc9c0 \ubabb\ud558\uace0 \uad50\ucc29 \uc0c1\ud0dc\uc5d0 \ube60\uc838 \ubcc4\ub2e4\ub978 \ud765\ubbf8\ub97c \uc77c\uc73c\ud0a4\uc9c0 \ubabb\ud588\ub2e4. \uacbd\uae30 \ud6c4\ubc18\uc5d0\ub294 \uc6e8\uc2a4\ud2b8 \ud584\uc758 \uc8fc\uc7a5\uc778 \uc870\uc9c0 \ucf00\uc774\uac00 \uc8fc\uc2ec\uc5d0\uac8c \uacbd\uae30\ub97c \uc77c\ucc0d \ub05d\ub0b4\ub77c\uace0 \uc694\uad6c\ud588\uc73c\ub098, \uc18c\ubb38\uc5d0 \uc758\ud558\uba74 \ubcfc\ud134\uc758 \uc8fc\uc7a5 \uc870 \uc2a4\ubbf8\uc2a4 \uac00 \uc8fc\uc2ec\uc5d0\uac8c \"\uc6b0\ub9ac\ub294 \uc798 \ud558\uace0 \uc788\uc5b4\uc694. \uc6b0\ub9ac\ub294 \ud544\uc694\ud558\ub2e4\uba74 \ud574\uac00 \uc9c8 \ub54c \uae4c\uc9c0\ub77c\ub3c4 \ud50c\ub808\uc774\ub97c \uacc4\uc18d \ud560 \uac81\ub2c8\ub2e4. \"\ub77c\uace0 \ub300\ub2f5\ud588\ub2e4\uace0 \ud55c\ub2e4. \uc774\uc73d\uace0 \uacbd\uae30 \uc885\ub8cc\uac00 \uac00\uae4c\uc6cc \uc8fc\uc2ec\uc774 \ub9c8\uc9c0\ub9c9\uc744 \ud718\uc2ac\uc744 \ubd88\uc790 2:0\uc73c\ub85c \ubcfc\ud134\uc774 \uc2b9\ub9ac\ud558\uc5ec \uc6b0\uc2b9\ucef5\uc744 \ucc28\uc9c0\ud588\ub2e4. \uc601\uad6d\uc758 \uc655 \uc870\uc9c0 5\uc138\uac00 FA\ucef5 \uc6b0\uc2b9 \ud2b8\ub85c\ud53c\ub97c \uc870 \uc2a4\ubbf8\uc2a4\uc5d0\uac8c \uc218\uc5ec\ud588\uace0 \uacbd\uae30\uc7a5\uc758 \uad00\uc911\ub4e4\uc758 \ud658\ud638\ub97c \uc720\ub3c4\ud588\ub2e4. \ud55c\ud3b8 \uc6e8\uc2a4\ud2b8 \ud584\uc758 \ud2b8\ub808\uc774\ub108 \ucc30\ub9ac \ud398\uc778\ud130\ub294 \"\ud770 \ub9d0\ub4e4\uc774 \uadf8 \ud070 \ubc1c\ub4e4\ub85c \ud53c\uce58(\uadf8\ub77c\uc6b4\ub4dc)\ub97c \ucff5\ucff5\uac70\ub9ac\uba70 \ub3cc\uc544\ub2e4\ub2c8\uba74\uc11c \uacbd\uae30\ub97c \uc808\ub9dd\uc801\uc73c\ub85c \ub9cc\ub4e4\uc5b4\ubc84\ub838\ub2e4. \uc6b0\ub9ac \ud300\uc758 \uc719\uc5b4\ub4e4\uc740 \uacc4\uc18d\ud574\uc11c \ub118\uc5b4\uc84c\uc73c\uba70 \ud53c\uce58\uc5d0 \ud30c\uc778 \ud648\uacfc \uad6c\uba4d\ub4e4\ub85c \uc778\ud574 \ub9ce\uc740 \uc2e4\uc218\ub97c \uc800\uc9c8\ub800\ub2e4\"\uba70 \uacbd\uae30 \uc2dc\uc791 \uc804\uc5d0 \ud6fc\uc190\ub41c \ud53c\uce58\uc758 \uc0c1\ud0dc\ub97c \ud300\uc758 \ud328\ubc30 \uc6d0\uc778\uc73c\ub85c \uaf3d\uc558\ub2e4.", "paragraph_answer": "\uc6e8\uc2a4\ud2b8 \ud584\uc740 \ud6c4\ubc18\uc804\uc5d0\ub294 \ubcf4\ub2e4 \ub354 \uac15\ud55c \ubaa8\uc2b5\uc744 \ubcf4\uc5ec\uc92c\ub294\ub370, \ud2b9\ud788 \ube45 \uc653\uc2a8\uc774 \uc88b\uc740 \uc790\ub9ac\uc5d0\uc11c \uace8 \ucc2c\uc2a4\ub97c \ub9de\uc558\uc73c\ub098 \uc29b\uc774 \uace8\ubb38\uc744 \ube57\ub098\uac14\ub2e4. \uadf8\ub7ec\ub098 \ud6c4\ubc18 8\ubd84\uc5d0 \ubcfc\ud134\uc774 \uc6e8\uc2a4\ud2b8 \ud584\uc758 \uac70\uc13c \uc800\ud56d\uc744 \ubfcc\ub9ac\uce58\uace0 \uc410\uae30\uace8\uc744 \ub123\uc5c8\ub2e4. \uc544\uc6c3\uc0ac\uc774\ub4dc \ud3ec\uc6cc\ub4dc \ud14c\ub4dc \ube44\uc790\ub4dc\uac00 \uacf5\uc744 \uacbd\uae30\uc7a5 \uc911\uc559\uc5d0 \ud328\uc2a4\ud588\uace0, \ud328\uc2a4\ub97c \ubc1b\uc740 \uc7ad \uc2a4\ubbf8\uc2a4\uac00 \uace8\ud0a4\ud37c \ud5c8\ud504\uc2a8\uc774 \ucc98\ub9ac\ud558\uae30 \uc55e\uc11c \uc29b\uc744 \uc131\uacf5\uc2dc\ucf30\ub2e4 \uace8\uc744 \ub123\uc5c8\ub2e4. \uadf8 \ub54c \uacf5\uc774 \ub2e4\uc2dc \ud295\uaca8 \ub098\uc654\ub294\ub370, \uc6e8\uc2a4\ud2b8 \ud584 \uc120\uc218\ub4e4\uc740 \uacf5\uc774 \uace8\ub77c\uc778\uc744 \uc9c0\ub098\uc9c0 \uc54a\uace0 \uace8\ud3ec\uc2a4\ud2b8\ub97c \ub9de\uace0 \ud280\uc5b4 \ub098\uc654\ub2e4\uace0 \uac70\uc138\uac8c \ud56d\uc758\ud588\uc73c\ub098 \uc8fc\uc2ec D. H. \uc560\uc2a8\uc740 \uc5b4\ud544\uc744 \ubb35\uc0b4\ud558\uace0 \uacf5\uc774 \uace8\ub77c\uc778\uc744 \uc9c0\ub09c \ub4a4\uc5d0 \ub2e4\uc2dc \ud295\uaca8 \ub098\uc654\ub2e4\uace0 \ud310\ub2e8\ud588\ub2e4. \uc6e8\uc2a4\ud2b8 \ud584\uc740 \ub610\ud55c \ubcfc\ud134\uc774 \ubd88\uacf5\ud3c9\ud55c \uc5b4\ub4dc\ubca4\ud2f0\uc9c0\ub97c \ubc1b\uc558\ub2e4\uace0 \ud56d\uc758\ud588\ub294\ub370, \ud56d\uc758 \ub0b4\uc6a9\uc740 \ube44\uc790\ub4dc\uc5d0\uac8c \uacf5\uc744 \ucc28\uc900 \uc0ac\ub78c\uc774 \ud53c\uce58 \uc8fc\ubcc0\uc5d0 \uc11c\uc788\ub358 \ubcfc\ud134\uc758 \ud32c\uc774\uc5c8\ub2e4\ub294 \uac83\uc774\ub2e4. \ud558\uc9c0\ub9cc \uc560\uc2a8\uc740 \uc774 \ud56d\uc758\ub3c4 \uae30\uac01\ud558\uace0 \ubcfc\ud134\uc758 \ub450 \ubc88\uc9f8 \uace8\uc744 \ucd5c\uc885\uc801\uc73c\ub85c \uc778\uc815\ud558\uc600\ub2e4. \uc774\ub54c\ubd80\ud130 \ubcfc\ud134\uc758 \uc2b9\ub9ac\ub97c \uc608\uc0c1\ud55c \ub9ce\uc740 \uad00\uc911\ub4e4\uc774 \uacbd\uae30\uc7a5\uc744 \uc77c\ucc0d \ube60\uc838\ub098\uac14\ub2e4. \ubcfc\ud134\uc758 \ub450 \ubc88\uc9f8 \ub4dd\uc810 \uc774\ud6c4 \uc591 \ud300 \ubaa8\ub450 \uc88b\uc740 \uace8\ucc2c\uc2a4\ub97c \uc7a1\uc9c0 \ubabb\ud558\uace0 \uad50\ucc29 \uc0c1\ud0dc\uc5d0 \ube60\uc838 \ubcc4\ub2e4\ub978 \ud765\ubbf8\ub97c \uc77c\uc73c\ud0a4\uc9c0 \ubabb\ud588\ub2e4. \uacbd\uae30 \ud6c4\ubc18\uc5d0\ub294 \uc6e8\uc2a4\ud2b8 \ud584\uc758 \uc8fc\uc7a5\uc778 \uc870\uc9c0 \ucf00\uc774\uac00 \uc8fc\uc2ec\uc5d0\uac8c \uacbd\uae30\ub97c \uc77c\ucc0d \ub05d\ub0b4\ub77c\uace0 \uc694\uad6c\ud588\uc73c\ub098, \uc18c\ubb38\uc5d0 \uc758\ud558\uba74 \ubcfc\ud134\uc758 \uc8fc\uc7a5 \uc870 \uc2a4\ubbf8\uc2a4 \uac00 \uc8fc\uc2ec\uc5d0\uac8c \"\uc6b0\ub9ac\ub294 \uc798 \ud558\uace0 \uc788\uc5b4\uc694. \uc6b0\ub9ac\ub294 \ud544\uc694\ud558\ub2e4\uba74 \ud574\uac00 \uc9c8 \ub54c \uae4c\uc9c0\ub77c\ub3c4 \ud50c\ub808\uc774\ub97c \uacc4\uc18d \ud560 \uac81\ub2c8\ub2e4.\"\ub77c\uace0 \ub300\ub2f5\ud588\ub2e4\uace0 \ud55c\ub2e4. \uc774\uc73d\uace0 \uacbd\uae30 \uc885\ub8cc\uac00 \uac00\uae4c\uc6cc \uc8fc\uc2ec\uc774 \ub9c8\uc9c0\ub9c9\uc744 \ud718\uc2ac\uc744 \ubd88\uc790 2:0\uc73c\ub85c \ubcfc\ud134\uc774 \uc2b9\ub9ac\ud558\uc5ec \uc6b0\uc2b9\ucef5\uc744 \ucc28\uc9c0\ud588\ub2e4. \uc601\uad6d\uc758 \uc655 \uc870\uc9c0 5\uc138\uac00 FA\ucef5 \uc6b0\uc2b9 \ud2b8\ub85c\ud53c\ub97c \uc870 \uc2a4\ubbf8\uc2a4\uc5d0\uac8c \uc218\uc5ec\ud588\uace0 \uacbd\uae30\uc7a5\uc758 \uad00\uc911\ub4e4\uc758 \ud658\ud638\ub97c \uc720\ub3c4\ud588\ub2e4. \ud55c\ud3b8 \uc6e8\uc2a4\ud2b8 \ud584\uc758 \ud2b8\ub808\uc774\ub108 \ucc30\ub9ac \ud398\uc778\ud130\ub294 \"\ud770 \ub9d0\ub4e4\uc774 \uadf8 \ud070 \ubc1c\ub4e4\ub85c \ud53c\uce58(\uadf8\ub77c\uc6b4\ub4dc)\ub97c \ucff5\ucff5\uac70\ub9ac\uba70 \ub3cc\uc544\ub2e4\ub2c8\uba74\uc11c \uacbd\uae30\ub97c \uc808\ub9dd\uc801\uc73c\ub85c \ub9cc\ub4e4\uc5b4\ubc84\ub838\ub2e4. \uc6b0\ub9ac \ud300\uc758 \uc719\uc5b4\ub4e4\uc740 \uacc4\uc18d\ud574\uc11c \ub118\uc5b4\uc84c\uc73c\uba70 \ud53c\uce58\uc5d0 \ud30c\uc778 \ud648\uacfc \uad6c\uba4d\ub4e4\ub85c \uc778\ud574 \ub9ce\uc740 \uc2e4\uc218\ub97c \uc800\uc9c8\ub800\ub2e4\"\uba70 \uacbd\uae30 \uc2dc\uc791 \uc804\uc5d0 \ud6fc\uc190\ub41c \ud53c\uce58\uc758 \uc0c1\ud0dc\ub97c \ud300\uc758 \ud328\ubc30 \uc6d0\uc778\uc73c\ub85c \uaf3d\uc558\ub2e4.", "sentence_answer": "\ubcfc\ud134\uc758 \uc8fc\uc7a5 \uc870 \uc2a4\ubbf8\uc2a4 \uac00 \uc8fc\uc2ec\uc5d0\uac8c \"\uc6b0\ub9ac\ub294 \uc798 \ud558\uace0 \uc788\uc5b4\uc694.", "paragraph_id": "5782571-8-2", "paragraph_question": "question: 1923\ub144 \uc789\uae00\ub79c\ub4dc FA\ucef5 \uacb0\uc2b9\uc804\uc5d0\uc11c \uc870\uc9c0 5\uc138\uc5d0\uac8c \uc6b0\uc2b9 \ud2b8\ub85c\ud53c\ub97c \ubc1b\uc740 \uc120\uc218\ub294?, context: \uc6e8\uc2a4\ud2b8 \ud584\uc740 \ud6c4\ubc18\uc804\uc5d0\ub294 \ubcf4\ub2e4 \ub354 \uac15\ud55c \ubaa8\uc2b5\uc744 \ubcf4\uc5ec\uc92c\ub294\ub370, \ud2b9\ud788 \ube45 \uc653\uc2a8\uc774 \uc88b\uc740 \uc790\ub9ac\uc5d0\uc11c \uace8 \ucc2c\uc2a4\ub97c \ub9de\uc558\uc73c\ub098 \uc29b\uc774 \uace8\ubb38\uc744 \ube57\ub098\uac14\ub2e4. \uadf8\ub7ec\ub098 \ud6c4\ubc18 8\ubd84\uc5d0 \ubcfc\ud134\uc774 \uc6e8\uc2a4\ud2b8 \ud584\uc758 \uac70\uc13c \uc800\ud56d\uc744 \ubfcc\ub9ac\uce58\uace0 \uc410\uae30\uace8\uc744 \ub123\uc5c8\ub2e4. \uc544\uc6c3\uc0ac\uc774\ub4dc \ud3ec\uc6cc\ub4dc \ud14c\ub4dc \ube44\uc790\ub4dc\uac00 \uacf5\uc744 \uacbd\uae30\uc7a5 \uc911\uc559\uc5d0 \ud328\uc2a4\ud588\uace0, \ud328\uc2a4\ub97c \ubc1b\uc740 \uc7ad \uc2a4\ubbf8\uc2a4\uac00 \uace8\ud0a4\ud37c \ud5c8\ud504\uc2a8\uc774 \ucc98\ub9ac\ud558\uae30 \uc55e\uc11c \uc29b\uc744 \uc131\uacf5\uc2dc\ucf30\ub2e4 \uace8\uc744 \ub123\uc5c8\ub2e4. \uadf8 \ub54c \uacf5\uc774 \ub2e4\uc2dc \ud295\uaca8 \ub098\uc654\ub294\ub370, \uc6e8\uc2a4\ud2b8 \ud584 \uc120\uc218\ub4e4\uc740 \uacf5\uc774 \uace8\ub77c\uc778\uc744 \uc9c0\ub098\uc9c0 \uc54a\uace0 \uace8\ud3ec\uc2a4\ud2b8\ub97c \ub9de\uace0 \ud280\uc5b4 \ub098\uc654\ub2e4\uace0 \uac70\uc138\uac8c \ud56d\uc758\ud588\uc73c\ub098 \uc8fc\uc2ec D. H. \uc560\uc2a8\uc740 \uc5b4\ud544\uc744 \ubb35\uc0b4\ud558\uace0 \uacf5\uc774 \uace8\ub77c\uc778\uc744 \uc9c0\ub09c \ub4a4\uc5d0 \ub2e4\uc2dc \ud295\uaca8 \ub098\uc654\ub2e4\uace0 \ud310\ub2e8\ud588\ub2e4. \uc6e8\uc2a4\ud2b8 \ud584\uc740 \ub610\ud55c \ubcfc\ud134\uc774 \ubd88\uacf5\ud3c9\ud55c \uc5b4\ub4dc\ubca4\ud2f0\uc9c0\ub97c \ubc1b\uc558\ub2e4\uace0 \ud56d\uc758\ud588\ub294\ub370, \ud56d\uc758 \ub0b4\uc6a9\uc740 \ube44\uc790\ub4dc\uc5d0\uac8c \uacf5\uc744 \ucc28\uc900 \uc0ac\ub78c\uc774 \ud53c\uce58 \uc8fc\ubcc0\uc5d0 \uc11c\uc788\ub358 \ubcfc\ud134\uc758 \ud32c\uc774\uc5c8\ub2e4\ub294 \uac83\uc774\ub2e4. \ud558\uc9c0\ub9cc \uc560\uc2a8\uc740 \uc774 \ud56d\uc758\ub3c4 \uae30\uac01\ud558\uace0 \ubcfc\ud134\uc758 \ub450 \ubc88\uc9f8 \uace8\uc744 \ucd5c\uc885\uc801\uc73c\ub85c \uc778\uc815\ud558\uc600\ub2e4. \uc774\ub54c\ubd80\ud130 \ubcfc\ud134\uc758 \uc2b9\ub9ac\ub97c \uc608\uc0c1\ud55c \ub9ce\uc740 \uad00\uc911\ub4e4\uc774 \uacbd\uae30\uc7a5\uc744 \uc77c\ucc0d \ube60\uc838\ub098\uac14\ub2e4. \ubcfc\ud134\uc758 \ub450 \ubc88\uc9f8 \ub4dd\uc810 \uc774\ud6c4 \uc591 \ud300 \ubaa8\ub450 \uc88b\uc740 \uace8\ucc2c\uc2a4\ub97c \uc7a1\uc9c0 \ubabb\ud558\uace0 \uad50\ucc29 \uc0c1\ud0dc\uc5d0 \ube60\uc838 \ubcc4\ub2e4\ub978 \ud765\ubbf8\ub97c \uc77c\uc73c\ud0a4\uc9c0 \ubabb\ud588\ub2e4. \uacbd\uae30 \ud6c4\ubc18\uc5d0\ub294 \uc6e8\uc2a4\ud2b8 \ud584\uc758 \uc8fc\uc7a5\uc778 \uc870\uc9c0 \ucf00\uc774\uac00 \uc8fc\uc2ec\uc5d0\uac8c \uacbd\uae30\ub97c \uc77c\ucc0d \ub05d\ub0b4\ub77c\uace0 \uc694\uad6c\ud588\uc73c\ub098, \uc18c\ubb38\uc5d0 \uc758\ud558\uba74 \ubcfc\ud134\uc758 \uc8fc\uc7a5 \uc870 \uc2a4\ubbf8\uc2a4\uac00 \uc8fc\uc2ec\uc5d0\uac8c \"\uc6b0\ub9ac\ub294 \uc798 \ud558\uace0 \uc788\uc5b4\uc694. \uc6b0\ub9ac\ub294 \ud544\uc694\ud558\ub2e4\uba74 \ud574\uac00 \uc9c8 \ub54c \uae4c\uc9c0\ub77c\ub3c4 \ud50c\ub808\uc774\ub97c \uacc4\uc18d \ud560 \uac81\ub2c8\ub2e4.\"\ub77c\uace0 \ub300\ub2f5\ud588\ub2e4\uace0 \ud55c\ub2e4. \uc774\uc73d\uace0 \uacbd\uae30 \uc885\ub8cc\uac00 \uac00\uae4c\uc6cc \uc8fc\uc2ec\uc774 \ub9c8\uc9c0\ub9c9\uc744 \ud718\uc2ac\uc744 \ubd88\uc790 2:0\uc73c\ub85c \ubcfc\ud134\uc774 \uc2b9\ub9ac\ud558\uc5ec \uc6b0\uc2b9\ucef5\uc744 \ucc28\uc9c0\ud588\ub2e4. \uc601\uad6d\uc758 \uc655 \uc870\uc9c0 5\uc138\uac00 FA\ucef5 \uc6b0\uc2b9 \ud2b8\ub85c\ud53c\ub97c \uc870 \uc2a4\ubbf8\uc2a4\uc5d0\uac8c \uc218\uc5ec\ud588\uace0 \uacbd\uae30\uc7a5\uc758 \uad00\uc911\ub4e4\uc758 \ud658\ud638\ub97c \uc720\ub3c4\ud588\ub2e4. \ud55c\ud3b8 \uc6e8\uc2a4\ud2b8 \ud584\uc758 \ud2b8\ub808\uc774\ub108 \ucc30\ub9ac \ud398\uc778\ud130\ub294 \"\ud770 \ub9d0\ub4e4\uc774 \uadf8 \ud070 \ubc1c\ub4e4\ub85c \ud53c\uce58(\uadf8\ub77c\uc6b4\ub4dc)\ub97c \ucff5\ucff5\uac70\ub9ac\uba70 \ub3cc\uc544\ub2e4\ub2c8\uba74\uc11c \uacbd\uae30\ub97c \uc808\ub9dd\uc801\uc73c\ub85c \ub9cc\ub4e4\uc5b4\ubc84\ub838\ub2e4. \uc6b0\ub9ac \ud300\uc758 \uc719\uc5b4\ub4e4\uc740 \uacc4\uc18d\ud574\uc11c \ub118\uc5b4\uc84c\uc73c\uba70 \ud53c\uce58\uc5d0 \ud30c\uc778 \ud648\uacfc \uad6c\uba4d\ub4e4\ub85c \uc778\ud574 \ub9ce\uc740 \uc2e4\uc218\ub97c \uc800\uc9c8\ub800\ub2e4\"\uba70 \uacbd\uae30 \uc2dc\uc791 \uc804\uc5d0 \ud6fc\uc190\ub41c \ud53c\uce58\uc758 \uc0c1\ud0dc\ub97c \ud300\uc758 \ud328\ubc30 \uc6d0\uc778\uc73c\ub85c \uaf3d\uc558\ub2e4."} {"question": "4\u00b719 \ud601\uba85 \uc774\ud6c4 \ub300\ub9ac\ub300\uc0ac\ub85c \uc9c0\uba85\ub41c \uc0ac\ub78c\uc740 \ub204\uad6c\uc778\uac00\uc694?", "paragraph": "4\u00b719 \ud601\uba85 \uc9c1\ud6c4\uc5d0 \uc591\uc720\ucc2c\uc740 \uc0ac\uc9c1\ud558\uc600\ub2e4. \uacfc\ub3c4 \uad6d\ubb34\uc6d0\uc740 4\uc6d4 29\uc77c \uc0c1\uc624 10\uc2dc\ubd80\ud130 \uc5f4\ub9b0 \uccab \uad6d\ubb34\ud68c\uc758\uc5d0\uc11c \uc0ac\ud45c\ub97c \ub0b8 \uc591\uc720\ucc2c \ub300\uc0ac\ub97c \uba74\uc9c1\ud558\uae30\ub85c \uc815\uc2dd\uc758\uacb0\ud558\uc600\ub2e4. 1960\ub144 4\uc6d4 29\uc77c, \uc591\uc720\ucc2c\uc740 \uc6cc\uc2f1\ud134\uc5d0\uc11c \uac1c\uc778\uc0ac\uc5c5\uc744 \ud558\ub824\uace0 \uc0dd\uac01\ud558\uace0 \uc788\ub2e4\uace0 \ub9d0\ud558\uc600\ub2e4. \uc591\uc720\ucc2c\uc740 \uc544\uc9c1 \ud55c\uad6d \uc784\uc2dc\uc815\ubd80\ub85c\ubd80\ud130 \ub204\uac00 \uadf8\uc758 \ud6c4\uc784\uc774 \ub420 \uac83\uc778\uac00\uc5d0 \uad00\ud558\uc5ec \uc544\ubb34\ub7f0 \uc2dc\uc0ac\ub3c4 \ubc1b\uc9c0 \uc54a\uc558\ub2e4. \uadf8\ub294 \uc544\uc9c1 \uba85\ud655\ud55c \uacc4\ud68d\uc740 \uc5c6\uc73c\ub098 \ud55c\uad6d \uc815\uacc4\uc5d0 \ud22c\uc2e0\ud558\uac70\ub098 \uc758\uc5c5\uc744 \uc7ac\uacc4\ud560 \uc0dd\uac01\uc740 \uc5c6\ub2e4\uace0 \ub9d0\ud558\uc600\ub2e4. 5\uc6d4 2\uc77c, \uc591\uc720\ucc2c\uc740 \uc0c8\ub85c\uc6b4 \ud55c\uad6d\uc815\ubd80\uac00 \uc774\ub0a0 \uadf8 \uc0ac\ud45c\ub97c \uc218\ub9ac\ud588\ub2e4\uace0 \ubc1d\ud614\ub2e4. \uc591\uc720\ucc2c\uc740 \"\ub098\ub294 \uadf8\ub4e4\uc774 \ub098\uc758 \uc0ac\ud45c\ub97c \uc218\ub9ac\ud574\uc11c \uae30\uc058\uba70 \uc9d0\uc774 \uac00\ubcbc\uc6cc\uc84c\ub2e4\"\uace0 \ub9d0\ud558\uc600\ub2e4. \ud55c\ud3b8 \ud55c\uad6d \ub300\uc0ac\uad00\uc758 \uc81c2\uc790(\u7b2c\u4e8c\u8005)\uc774\uba70 \ub178\ub828\ud55c \uc678\uad50\uad00\uc778 \ud55c\ud45c\uc6b1 \uacf5\uc0ac\uac00 \ub300\ub9ac\ub300\uc0ac\ub85c \uc9c0\uba85\ub418\uc5c8\ub2e4. \uc774\uc640 \uac19\uc740 \ubcc0\uacbd\uc5d0 \uad00\ud55c \uc815\uc2dd \ud1b5\uace0\ubb38\uc774 2\uc77c \ubbf8\uad6d \uad6d\ubb34\uc131\uacfc \uc678\uad50\ub2e8\uc5d0 \uc804\ub2ec\ub418\uc5c8\ub2e4. \uc591\uc720\ucc2c\uc758 \uc678\uad50\ud65c\ub3d9\uc740 \uc678\ubb34\ubcf8\ubd80\uc758 \uccb4\uacc4\ub97c \ubb34\uc2dc\ud55c \uacbd\ubb34\ub300 \uc9c1\ud1b5 \uc678\uad50\ub85c \ube44\ub09c\uc744 \ubc1b\uc558\ub2e4. \uc678\uad50 \uc635\uc368\ubc84\ub4e4\uc740 \uc591\uc720\ucc2c\uc774 \ud55c\uacb0\uac19\uc774 \uc774\uc2b9\ub9cc \uc815\uad8c\uc758 \ub3c5\uc120\uc801\uc778 \uc678\uad50\ub97c \ud569\ub9ac\ud654\uc2dc\ud0a4\ub294 \ub370 \uc804\ub825\uc744 \uacbd\uc8fc\ud558\uc600\uc73c\uba70 \uc9c0\ub09c 3\u2219 15 \uc815\u2219 \ubd80\ud1b5\ub839\uc120\uac70\uc5d0 \ub300\ud558\uc5ec\ub3c4 \uc790\uc720\uc138\uacc4\uc5d0 \ubaa8\ubc94\uc774 \ub41c \uacf5\uc815\ud55c \uc120\uac70\uc600\ub2e4\ub294 \uac83\uc744 \uc6b0\ubc29 \uc81c\uad6d\uc5d0 \uc65c\uace1 \uc120\uc804\ud558\ub294 \ud55c\ud3b8 \ub9c8\uc0b0\uc5d0\uc11c\uc758 \uad6d\ubbfc\uc758\uac70\ub294 \uacf5\uc0b0\uc624\uc5f4(\u4e94\u5217)\uc758 \uac1c\uc785 \uc6b4\uc6b4\uc73c\ub85c \uc774\uc2b9\ub9cc \uc815\uad8c\uc758 \ubd80\ud328\uc0c1\uc744 \ubb35\uc778, \uad6d\uc81c\uc5ec\ub860\uc744 \uc624\ub3c4\ucf00 \ud558\uc600\uc74c\uc744 \uc0c1\uae30\uc2dc\ucf30\ub2e4. \uc591\uc720\ucc2c\uc740 \uc870\uc6a9\ud55c \uc8fc\ud0dd\uc9c0\uc778 \uba54\uc774\ud50c\ubc00\uac00(\u8857)\uc5d0 \uc788\ub294 \uc790\ud0dd\uc5d0\uc11c \uc790\uae30\uc640 \ud55c(\u97d3) \uacf5\uc0ac\uac00 \uc774\uc2b9\ub9cc\uc758 \ub3c8\uc744 \uc2a4\uc704\uc2a4 \uc740\ud589\uc5d0 \uc608\uae08\ud558\uc600\ub2e4\ub294 \ubaa8 \uc2e0\ubb38\uae30\uc0ac\uc5d0 \uad00\ud558\uc5ec \ud384\ud384\ub6f0\uba74\uc11c \uc774\uac83\uc740 \uc0ac\uc2e4\ubb34\uadfc\uc774\ub77c\uace0 1960\ub144 7\uc6d4 13\uc77c \ubc18\ubc15\ud558\uc600\ub2e4.", "answer": "\ud55c\ud45c\uc6b1 \uacf5\uc0ac", "sentence": "\ud55c\ud3b8 \ud55c\uad6d \ub300\uc0ac\uad00\uc758 \uc81c2\uc790(\u7b2c\u4e8c\u8005)\uc774\uba70 \ub178\ub828\ud55c \uc678\uad50\uad00\uc778 \ud55c\ud45c\uc6b1 \uacf5\uc0ac \uac00 \ub300\ub9ac\ub300\uc0ac\ub85c \uc9c0\uba85\ub418\uc5c8\ub2e4.", "paragraph_sentence": "4\u00b719 \ud601\uba85 \uc9c1\ud6c4\uc5d0 \uc591\uc720\ucc2c\uc740 \uc0ac\uc9c1\ud558\uc600\ub2e4. \uacfc\ub3c4 \uad6d\ubb34\uc6d0\uc740 4\uc6d4 29\uc77c \uc0c1\uc624 10\uc2dc\ubd80\ud130 \uc5f4\ub9b0 \uccab \uad6d\ubb34\ud68c\uc758\uc5d0\uc11c \uc0ac\ud45c\ub97c \ub0b8 \uc591\uc720\ucc2c \ub300\uc0ac\ub97c \uba74\uc9c1\ud558\uae30\ub85c \uc815\uc2dd\uc758\uacb0\ud558\uc600\ub2e4. 1960\ub144 4\uc6d4 29\uc77c, \uc591\uc720\ucc2c\uc740 \uc6cc\uc2f1\ud134\uc5d0\uc11c \uac1c\uc778\uc0ac\uc5c5\uc744 \ud558\ub824\uace0 \uc0dd\uac01\ud558\uace0 \uc788\ub2e4\uace0 \ub9d0\ud558\uc600\ub2e4. \uc591\uc720\ucc2c\uc740 \uc544\uc9c1 \ud55c\uad6d \uc784\uc2dc\uc815\ubd80\ub85c\ubd80\ud130 \ub204\uac00 \uadf8\uc758 \ud6c4\uc784\uc774 \ub420 \uac83\uc778\uac00\uc5d0 \uad00\ud558\uc5ec \uc544\ubb34\ub7f0 \uc2dc\uc0ac\ub3c4 \ubc1b\uc9c0 \uc54a\uc558\ub2e4. \uadf8\ub294 \uc544\uc9c1 \uba85\ud655\ud55c \uacc4\ud68d\uc740 \uc5c6\uc73c\ub098 \ud55c\uad6d \uc815\uacc4\uc5d0 \ud22c\uc2e0\ud558\uac70\ub098 \uc758\uc5c5\uc744 \uc7ac\uacc4\ud560 \uc0dd\uac01\uc740 \uc5c6\ub2e4\uace0 \ub9d0\ud558\uc600\ub2e4. 5\uc6d4 2\uc77c, \uc591\uc720\ucc2c\uc740 \uc0c8\ub85c\uc6b4 \ud55c\uad6d\uc815\ubd80\uac00 \uc774\ub0a0 \uadf8 \uc0ac\ud45c\ub97c \uc218\ub9ac\ud588\ub2e4\uace0 \ubc1d\ud614\ub2e4. \uc591\uc720\ucc2c\uc740 \"\ub098\ub294 \uadf8\ub4e4\uc774 \ub098\uc758 \uc0ac\ud45c\ub97c \uc218\ub9ac\ud574\uc11c \uae30\uc058\uba70 \uc9d0\uc774 \uac00\ubcbc\uc6cc\uc84c\ub2e4\"\uace0 \ub9d0\ud558\uc600\ub2e4. \ud55c\ud3b8 \ud55c\uad6d \ub300\uc0ac\uad00\uc758 \uc81c2\uc790(\u7b2c\u4e8c\u8005)\uc774\uba70 \ub178\ub828\ud55c \uc678\uad50\uad00\uc778 \ud55c\ud45c\uc6b1 \uacf5\uc0ac \uac00 \ub300\ub9ac\ub300\uc0ac\ub85c \uc9c0\uba85\ub418\uc5c8\ub2e4. \uc774\uc640 \uac19\uc740 \ubcc0\uacbd\uc5d0 \uad00\ud55c \uc815\uc2dd \ud1b5\uace0\ubb38\uc774 2\uc77c \ubbf8\uad6d \uad6d\ubb34\uc131\uacfc \uc678\uad50\ub2e8\uc5d0 \uc804\ub2ec\ub418\uc5c8\ub2e4. \uc591\uc720\ucc2c\uc758 \uc678\uad50\ud65c\ub3d9\uc740 \uc678\ubb34\ubcf8\ubd80\uc758 \uccb4\uacc4\ub97c \ubb34\uc2dc\ud55c \uacbd\ubb34\ub300 \uc9c1\ud1b5 \uc678\uad50\ub85c \ube44\ub09c\uc744 \ubc1b\uc558\ub2e4. \uc678\uad50 \uc635\uc368\ubc84\ub4e4\uc740 \uc591\uc720\ucc2c\uc774 \ud55c\uacb0\uac19\uc774 \uc774\uc2b9\ub9cc \uc815\uad8c\uc758 \ub3c5\uc120\uc801\uc778 \uc678\uad50\ub97c \ud569\ub9ac\ud654\uc2dc\ud0a4\ub294 \ub370 \uc804\ub825\uc744 \uacbd\uc8fc\ud558\uc600\uc73c\uba70 \uc9c0\ub09c 3\u2219 15 \uc815\u2219 \ubd80\ud1b5\ub839\uc120\uac70\uc5d0 \ub300\ud558\uc5ec\ub3c4 \uc790\uc720\uc138\uacc4\uc5d0 \ubaa8\ubc94\uc774 \ub41c \uacf5\uc815\ud55c \uc120\uac70\uc600\ub2e4\ub294 \uac83\uc744 \uc6b0\ubc29 \uc81c\uad6d\uc5d0 \uc65c\uace1 \uc120\uc804\ud558\ub294 \ud55c\ud3b8 \ub9c8\uc0b0\uc5d0\uc11c\uc758 \uad6d\ubbfc\uc758\uac70\ub294 \uacf5\uc0b0\uc624\uc5f4(\u4e94\u5217)\uc758 \uac1c\uc785 \uc6b4\uc6b4\uc73c\ub85c \uc774\uc2b9\ub9cc \uc815\uad8c\uc758 \ubd80\ud328\uc0c1\uc744 \ubb35\uc778, \uad6d\uc81c\uc5ec\ub860\uc744 \uc624\ub3c4\ucf00 \ud558\uc600\uc74c\uc744 \uc0c1\uae30\uc2dc\ucf30\ub2e4. \uc591\uc720\ucc2c\uc740 \uc870\uc6a9\ud55c \uc8fc\ud0dd\uc9c0\uc778 \uba54\uc774\ud50c\ubc00\uac00(\u8857)\uc5d0 \uc788\ub294 \uc790\ud0dd\uc5d0\uc11c \uc790\uae30\uc640 \ud55c(\u97d3) \uacf5\uc0ac\uac00 \uc774\uc2b9\ub9cc\uc758 \ub3c8\uc744 \uc2a4\uc704\uc2a4 \uc740\ud589\uc5d0 \uc608\uae08\ud558\uc600\ub2e4\ub294 \ubaa8 \uc2e0\ubb38\uae30\uc0ac\uc5d0 \uad00\ud558\uc5ec \ud384\ud384\ub6f0\uba74\uc11c \uc774\uac83\uc740 \uc0ac\uc2e4\ubb34\uadfc\uc774\ub77c\uace0 1960\ub144 7\uc6d4 13\uc77c \ubc18\ubc15\ud558\uc600\ub2e4.", "paragraph_answer": "4\u00b719 \ud601\uba85 \uc9c1\ud6c4\uc5d0 \uc591\uc720\ucc2c\uc740 \uc0ac\uc9c1\ud558\uc600\ub2e4. \uacfc\ub3c4 \uad6d\ubb34\uc6d0\uc740 4\uc6d4 29\uc77c \uc0c1\uc624 10\uc2dc\ubd80\ud130 \uc5f4\ub9b0 \uccab \uad6d\ubb34\ud68c\uc758\uc5d0\uc11c \uc0ac\ud45c\ub97c \ub0b8 \uc591\uc720\ucc2c \ub300\uc0ac\ub97c \uba74\uc9c1\ud558\uae30\ub85c \uc815\uc2dd\uc758\uacb0\ud558\uc600\ub2e4. 1960\ub144 4\uc6d4 29\uc77c, \uc591\uc720\ucc2c\uc740 \uc6cc\uc2f1\ud134\uc5d0\uc11c \uac1c\uc778\uc0ac\uc5c5\uc744 \ud558\ub824\uace0 \uc0dd\uac01\ud558\uace0 \uc788\ub2e4\uace0 \ub9d0\ud558\uc600\ub2e4. \uc591\uc720\ucc2c\uc740 \uc544\uc9c1 \ud55c\uad6d \uc784\uc2dc\uc815\ubd80\ub85c\ubd80\ud130 \ub204\uac00 \uadf8\uc758 \ud6c4\uc784\uc774 \ub420 \uac83\uc778\uac00\uc5d0 \uad00\ud558\uc5ec \uc544\ubb34\ub7f0 \uc2dc\uc0ac\ub3c4 \ubc1b\uc9c0 \uc54a\uc558\ub2e4. \uadf8\ub294 \uc544\uc9c1 \uba85\ud655\ud55c \uacc4\ud68d\uc740 \uc5c6\uc73c\ub098 \ud55c\uad6d \uc815\uacc4\uc5d0 \ud22c\uc2e0\ud558\uac70\ub098 \uc758\uc5c5\uc744 \uc7ac\uacc4\ud560 \uc0dd\uac01\uc740 \uc5c6\ub2e4\uace0 \ub9d0\ud558\uc600\ub2e4. 5\uc6d4 2\uc77c, \uc591\uc720\ucc2c\uc740 \uc0c8\ub85c\uc6b4 \ud55c\uad6d\uc815\ubd80\uac00 \uc774\ub0a0 \uadf8 \uc0ac\ud45c\ub97c \uc218\ub9ac\ud588\ub2e4\uace0 \ubc1d\ud614\ub2e4. \uc591\uc720\ucc2c\uc740 \"\ub098\ub294 \uadf8\ub4e4\uc774 \ub098\uc758 \uc0ac\ud45c\ub97c \uc218\ub9ac\ud574\uc11c \uae30\uc058\uba70 \uc9d0\uc774 \uac00\ubcbc\uc6cc\uc84c\ub2e4\"\uace0 \ub9d0\ud558\uc600\ub2e4. \ud55c\ud3b8 \ud55c\uad6d \ub300\uc0ac\uad00\uc758 \uc81c2\uc790(\u7b2c\u4e8c\u8005)\uc774\uba70 \ub178\ub828\ud55c \uc678\uad50\uad00\uc778 \ud55c\ud45c\uc6b1 \uacf5\uc0ac \uac00 \ub300\ub9ac\ub300\uc0ac\ub85c \uc9c0\uba85\ub418\uc5c8\ub2e4. \uc774\uc640 \uac19\uc740 \ubcc0\uacbd\uc5d0 \uad00\ud55c \uc815\uc2dd \ud1b5\uace0\ubb38\uc774 2\uc77c \ubbf8\uad6d \uad6d\ubb34\uc131\uacfc \uc678\uad50\ub2e8\uc5d0 \uc804\ub2ec\ub418\uc5c8\ub2e4. \uc591\uc720\ucc2c\uc758 \uc678\uad50\ud65c\ub3d9\uc740 \uc678\ubb34\ubcf8\ubd80\uc758 \uccb4\uacc4\ub97c \ubb34\uc2dc\ud55c \uacbd\ubb34\ub300 \uc9c1\ud1b5 \uc678\uad50\ub85c \ube44\ub09c\uc744 \ubc1b\uc558\ub2e4. \uc678\uad50 \uc635\uc368\ubc84\ub4e4\uc740 \uc591\uc720\ucc2c\uc774 \ud55c\uacb0\uac19\uc774 \uc774\uc2b9\ub9cc \uc815\uad8c\uc758 \ub3c5\uc120\uc801\uc778 \uc678\uad50\ub97c \ud569\ub9ac\ud654\uc2dc\ud0a4\ub294 \ub370 \uc804\ub825\uc744 \uacbd\uc8fc\ud558\uc600\uc73c\uba70 \uc9c0\ub09c 3\u2219 15 \uc815\u2219 \ubd80\ud1b5\ub839\uc120\uac70\uc5d0 \ub300\ud558\uc5ec\ub3c4 \uc790\uc720\uc138\uacc4\uc5d0 \ubaa8\ubc94\uc774 \ub41c \uacf5\uc815\ud55c \uc120\uac70\uc600\ub2e4\ub294 \uac83\uc744 \uc6b0\ubc29 \uc81c\uad6d\uc5d0 \uc65c\uace1 \uc120\uc804\ud558\ub294 \ud55c\ud3b8 \ub9c8\uc0b0\uc5d0\uc11c\uc758 \uad6d\ubbfc\uc758\uac70\ub294 \uacf5\uc0b0\uc624\uc5f4(\u4e94\u5217)\uc758 \uac1c\uc785 \uc6b4\uc6b4\uc73c\ub85c \uc774\uc2b9\ub9cc \uc815\uad8c\uc758 \ubd80\ud328\uc0c1\uc744 \ubb35\uc778, \uad6d\uc81c\uc5ec\ub860\uc744 \uc624\ub3c4\ucf00 \ud558\uc600\uc74c\uc744 \uc0c1\uae30\uc2dc\ucf30\ub2e4. \uc591\uc720\ucc2c\uc740 \uc870\uc6a9\ud55c \uc8fc\ud0dd\uc9c0\uc778 \uba54\uc774\ud50c\ubc00\uac00(\u8857)\uc5d0 \uc788\ub294 \uc790\ud0dd\uc5d0\uc11c \uc790\uae30\uc640 \ud55c(\u97d3) \uacf5\uc0ac\uac00 \uc774\uc2b9\ub9cc\uc758 \ub3c8\uc744 \uc2a4\uc704\uc2a4 \uc740\ud589\uc5d0 \uc608\uae08\ud558\uc600\ub2e4\ub294 \ubaa8 \uc2e0\ubb38\uae30\uc0ac\uc5d0 \uad00\ud558\uc5ec \ud384\ud384\ub6f0\uba74\uc11c \uc774\uac83\uc740 \uc0ac\uc2e4\ubb34\uadfc\uc774\ub77c\uace0 1960\ub144 7\uc6d4 13\uc77c \ubc18\ubc15\ud558\uc600\ub2e4.", "sentence_answer": "\ud55c\ud3b8 \ud55c\uad6d \ub300\uc0ac\uad00\uc758 \uc81c2\uc790(\u7b2c\u4e8c\u8005)\uc774\uba70 \ub178\ub828\ud55c \uc678\uad50\uad00\uc778 \ud55c\ud45c\uc6b1 \uacf5\uc0ac \uac00 \ub300\ub9ac\ub300\uc0ac\ub85c \uc9c0\uba85\ub418\uc5c8\ub2e4.", "paragraph_id": "6522887-3-1", "paragraph_question": "question: 4\u00b719 \ud601\uba85 \uc774\ud6c4 \ub300\ub9ac\ub300\uc0ac\ub85c \uc9c0\uba85\ub41c \uc0ac\ub78c\uc740 \ub204\uad6c\uc778\uac00\uc694?, context: 4\u00b719 \ud601\uba85 \uc9c1\ud6c4\uc5d0 \uc591\uc720\ucc2c\uc740 \uc0ac\uc9c1\ud558\uc600\ub2e4. \uacfc\ub3c4 \uad6d\ubb34\uc6d0\uc740 4\uc6d4 29\uc77c \uc0c1\uc624 10\uc2dc\ubd80\ud130 \uc5f4\ub9b0 \uccab \uad6d\ubb34\ud68c\uc758\uc5d0\uc11c \uc0ac\ud45c\ub97c \ub0b8 \uc591\uc720\ucc2c \ub300\uc0ac\ub97c \uba74\uc9c1\ud558\uae30\ub85c \uc815\uc2dd\uc758\uacb0\ud558\uc600\ub2e4. 1960\ub144 4\uc6d4 29\uc77c, \uc591\uc720\ucc2c\uc740 \uc6cc\uc2f1\ud134\uc5d0\uc11c \uac1c\uc778\uc0ac\uc5c5\uc744 \ud558\ub824\uace0 \uc0dd\uac01\ud558\uace0 \uc788\ub2e4\uace0 \ub9d0\ud558\uc600\ub2e4. \uc591\uc720\ucc2c\uc740 \uc544\uc9c1 \ud55c\uad6d \uc784\uc2dc\uc815\ubd80\ub85c\ubd80\ud130 \ub204\uac00 \uadf8\uc758 \ud6c4\uc784\uc774 \ub420 \uac83\uc778\uac00\uc5d0 \uad00\ud558\uc5ec \uc544\ubb34\ub7f0 \uc2dc\uc0ac\ub3c4 \ubc1b\uc9c0 \uc54a\uc558\ub2e4. \uadf8\ub294 \uc544\uc9c1 \uba85\ud655\ud55c \uacc4\ud68d\uc740 \uc5c6\uc73c\ub098 \ud55c\uad6d \uc815\uacc4\uc5d0 \ud22c\uc2e0\ud558\uac70\ub098 \uc758\uc5c5\uc744 \uc7ac\uacc4\ud560 \uc0dd\uac01\uc740 \uc5c6\ub2e4\uace0 \ub9d0\ud558\uc600\ub2e4. 5\uc6d4 2\uc77c, \uc591\uc720\ucc2c\uc740 \uc0c8\ub85c\uc6b4 \ud55c\uad6d\uc815\ubd80\uac00 \uc774\ub0a0 \uadf8 \uc0ac\ud45c\ub97c \uc218\ub9ac\ud588\ub2e4\uace0 \ubc1d\ud614\ub2e4. \uc591\uc720\ucc2c\uc740 \"\ub098\ub294 \uadf8\ub4e4\uc774 \ub098\uc758 \uc0ac\ud45c\ub97c \uc218\ub9ac\ud574\uc11c \uae30\uc058\uba70 \uc9d0\uc774 \uac00\ubcbc\uc6cc\uc84c\ub2e4\"\uace0 \ub9d0\ud558\uc600\ub2e4. \ud55c\ud3b8 \ud55c\uad6d \ub300\uc0ac\uad00\uc758 \uc81c2\uc790(\u7b2c\u4e8c\u8005)\uc774\uba70 \ub178\ub828\ud55c \uc678\uad50\uad00\uc778 \ud55c\ud45c\uc6b1 \uacf5\uc0ac\uac00 \ub300\ub9ac\ub300\uc0ac\ub85c \uc9c0\uba85\ub418\uc5c8\ub2e4. \uc774\uc640 \uac19\uc740 \ubcc0\uacbd\uc5d0 \uad00\ud55c \uc815\uc2dd \ud1b5\uace0\ubb38\uc774 2\uc77c \ubbf8\uad6d \uad6d\ubb34\uc131\uacfc \uc678\uad50\ub2e8\uc5d0 \uc804\ub2ec\ub418\uc5c8\ub2e4. \uc591\uc720\ucc2c\uc758 \uc678\uad50\ud65c\ub3d9\uc740 \uc678\ubb34\ubcf8\ubd80\uc758 \uccb4\uacc4\ub97c \ubb34\uc2dc\ud55c \uacbd\ubb34\ub300 \uc9c1\ud1b5 \uc678\uad50\ub85c \ube44\ub09c\uc744 \ubc1b\uc558\ub2e4. \uc678\uad50 \uc635\uc368\ubc84\ub4e4\uc740 \uc591\uc720\ucc2c\uc774 \ud55c\uacb0\uac19\uc774 \uc774\uc2b9\ub9cc \uc815\uad8c\uc758 \ub3c5\uc120\uc801\uc778 \uc678\uad50\ub97c \ud569\ub9ac\ud654\uc2dc\ud0a4\ub294 \ub370 \uc804\ub825\uc744 \uacbd\uc8fc\ud558\uc600\uc73c\uba70 \uc9c0\ub09c 3\u2219 15 \uc815\u2219 \ubd80\ud1b5\ub839\uc120\uac70\uc5d0 \ub300\ud558\uc5ec\ub3c4 \uc790\uc720\uc138\uacc4\uc5d0 \ubaa8\ubc94\uc774 \ub41c \uacf5\uc815\ud55c \uc120\uac70\uc600\ub2e4\ub294 \uac83\uc744 \uc6b0\ubc29 \uc81c\uad6d\uc5d0 \uc65c\uace1 \uc120\uc804\ud558\ub294 \ud55c\ud3b8 \ub9c8\uc0b0\uc5d0\uc11c\uc758 \uad6d\ubbfc\uc758\uac70\ub294 \uacf5\uc0b0\uc624\uc5f4(\u4e94\u5217)\uc758 \uac1c\uc785 \uc6b4\uc6b4\uc73c\ub85c \uc774\uc2b9\ub9cc \uc815\uad8c\uc758 \ubd80\ud328\uc0c1\uc744 \ubb35\uc778, \uad6d\uc81c\uc5ec\ub860\uc744 \uc624\ub3c4\ucf00 \ud558\uc600\uc74c\uc744 \uc0c1\uae30\uc2dc\ucf30\ub2e4. \uc591\uc720\ucc2c\uc740 \uc870\uc6a9\ud55c \uc8fc\ud0dd\uc9c0\uc778 \uba54\uc774\ud50c\ubc00\uac00(\u8857)\uc5d0 \uc788\ub294 \uc790\ud0dd\uc5d0\uc11c \uc790\uae30\uc640 \ud55c(\u97d3) \uacf5\uc0ac\uac00 \uc774\uc2b9\ub9cc\uc758 \ub3c8\uc744 \uc2a4\uc704\uc2a4 \uc740\ud589\uc5d0 \uc608\uae08\ud558\uc600\ub2e4\ub294 \ubaa8 \uc2e0\ubb38\uae30\uc0ac\uc5d0 \uad00\ud558\uc5ec \ud384\ud384\ub6f0\uba74\uc11c \uc774\uac83\uc740 \uc0ac\uc2e4\ubb34\uadfc\uc774\ub77c\uace0 1960\ub144 7\uc6d4 13\uc77c \ubc18\ubc15\ud558\uc600\ub2e4."} {"question": "\uc5d0\ubcfc\ub77c\uc758 \uc720\ud589 \ub2f9\uc2dc \ub77c\uc774\ubca0\ub9ac\uc544\uc758 \ub300\ud1b5\ub839 \uc774\ub984\uc740 \ubb34\uc5c7\uc778\uac00?", "paragraph": "7\uc6d4 26\uc77c, \ub77c\uc774\ubca0\ub9ac\uc544\uc5d0\uc11c \ub098\uc774\uc9c0\ub9ac\uc544\ub85c \uc628 \uc0ac\ub78c\ub4e4 \uc911 \uc5d0\ubcfc\ub77c \ubc14\uc774\ub7ec\uc2a4 \uac10\uc5fc \uc758\uc2ec \ud658\uc790\uac00 \ubc1c\uc0dd\ud55c \uc774\ud6c4 \ub098\uc774\uc9c0\ub9ac\uc544\ub294 \ubaa8\ub4e0 \uc785\uad6d \uac00\ub2a5 \uc9c0\uc5ed\uc758 \uac10\uc2dc\ub97c \uac15\ud654\ud588\ub2e4. \ubcf4\uac74 \ub2f9\uad6d\uc740 \uac01 \uc785\uad6d \uad6d\uacbd\uc5d0\uc11c \ub3c4\ucc29\ud558\ub294 \ubaa8\ub4e0 \uc0ac\ub78c\ub4e4\uc5d0\uac8c \uac10\uc5fc \ud14c\uc2a4\ud2b8\ub97c \uac70\uce60 \uac83\uc774\ub77c\uace0 \ub9d0\ud588\ub2e4. \ub098\uc774\uc9c0\ub9ac\uc544\uc758 \uc8fc\uc694 \ud56d\uacf5\uc0ac\uc778 \uc544\ub9ac\ud06c \uc5d0\uc5b4\ub294 \ub77c\uc774\ubca0\ub9ac\uc544\uc640 \uc2dc\uc5d0\ub77c\ub9ac\uc628\uc73c\ub85c \ud5a5\ud558\ub294 \ubaa8\ub4e0 \ube44\ud589\uae30\ud3b8\uc744 \uc911\ub2e8\uc2dc\ucf30\ub2e4. 7\uc6d4 27\uc77c, \ub77c\uc774\ubca0\ub9ac\uc544\uc758 \ub300\ud1b5\ub839\uc778 \uc5d8\ub7f0 \uc874\uc2a8\uc124\ub9ac\ud504\ub294 \ubc14\uc774\ub7ec\uc2a4 \ud14c\uc2a4\ud2b8 \uc13c\ud130\uac00 \uc788\ub294 \uba87\uba87 \uad6d\uacbd\uc744 \uc81c\uc678\ud55c \ubaa8\ub4e0 \uad6d\uac00\uc758 \uc8fc\uc694 \uacf5\ud56d \ubc0f \uad6d\uacbd\uc744 \ud3d0\uc1c4\ud560 \uac83\uc774\ub77c\uace0 \ub9d0\ud588\ub2e4. \ucd5c\uc545\uc758 \uc601\ud5a5\uc744 \ubc1b\ub294 \uc9c0\uc5ed\uc5d0\ub294 \uac80\uc5ed \ud558\uc5d0 \uc788\uc744 \uac83\uc774\ub77c\uace0 \ub9d0\ud588\ub2e4. \ud070 \ud589\uc0ac \ubc0f \uc2a4\ud3ec\uce20 \ud65c\ub3d9\uc740 \uc811\ucd09 \uac10\uc5fc\uc758 \uc704\ud5d8\uc744 \ub192\uc774\uae30 \ub54c\ubb38\uc5d0 \ubaa8\ub4e0 \ucd95\uad6c \uacbd\uae30\ub3c4 \uae08\uc9c0\ub418\uc5c8\ub2e4. 7\uc6d4 30\uc77c, \ub77c\uc774\ubca0\ub9ac\uc544 \ub0b4\uc758 \ubaa8\ub4e0 \ud559\uad50\uac00 \ud734\uad50\ud588\uc73c\uba70, \uc9c0\uc5ed \uc0ac\ud68c\uac00 \uac70\uc758 \uc815\uc9c0\ub418\uc5c8\ub2e4.", "answer": "\uc5d8\ub7f0 \uc874\uc2a8\uc124\ub9ac\ud504", "sentence": "7\uc6d4 27\uc77c, \ub77c\uc774\ubca0\ub9ac\uc544\uc758 \ub300\ud1b5\ub839\uc778 \uc5d8\ub7f0 \uc874\uc2a8\uc124\ub9ac\ud504 \ub294 \ubc14\uc774\ub7ec\uc2a4 \ud14c\uc2a4\ud2b8 \uc13c\ud130\uac00 \uc788\ub294 \uba87\uba87 \uad6d\uacbd\uc744 \uc81c\uc678\ud55c \ubaa8\ub4e0 \uad6d\uac00\uc758 \uc8fc\uc694 \uacf5\ud56d \ubc0f \uad6d\uacbd\uc744 \ud3d0\uc1c4\ud560 \uac83\uc774\ub77c\uace0 \ub9d0\ud588\ub2e4.", "paragraph_sentence": "7\uc6d4 26\uc77c, \ub77c\uc774\ubca0\ub9ac\uc544\uc5d0\uc11c \ub098\uc774\uc9c0\ub9ac\uc544\ub85c \uc628 \uc0ac\ub78c\ub4e4 \uc911 \uc5d0\ubcfc\ub77c \ubc14\uc774\ub7ec\uc2a4 \uac10\uc5fc \uc758\uc2ec \ud658\uc790\uac00 \ubc1c\uc0dd\ud55c \uc774\ud6c4 \ub098\uc774\uc9c0\ub9ac\uc544\ub294 \ubaa8\ub4e0 \uc785\uad6d \uac00\ub2a5 \uc9c0\uc5ed\uc758 \uac10\uc2dc\ub97c \uac15\ud654\ud588\ub2e4. \ubcf4\uac74 \ub2f9\uad6d\uc740 \uac01 \uc785\uad6d \uad6d\uacbd\uc5d0\uc11c \ub3c4\ucc29\ud558\ub294 \ubaa8\ub4e0 \uc0ac\ub78c\ub4e4\uc5d0\uac8c \uac10\uc5fc \ud14c\uc2a4\ud2b8\ub97c \uac70\uce60 \uac83\uc774\ub77c\uace0 \ub9d0\ud588\ub2e4. \ub098\uc774\uc9c0\ub9ac\uc544\uc758 \uc8fc\uc694 \ud56d\uacf5\uc0ac\uc778 \uc544\ub9ac\ud06c \uc5d0\uc5b4\ub294 \ub77c\uc774\ubca0\ub9ac\uc544\uc640 \uc2dc\uc5d0\ub77c\ub9ac\uc628\uc73c\ub85c \ud5a5\ud558\ub294 \ubaa8\ub4e0 \ube44\ud589\uae30\ud3b8\uc744 \uc911\ub2e8\uc2dc\ucf30\ub2e4. 7\uc6d4 27\uc77c, \ub77c\uc774\ubca0\ub9ac\uc544\uc758 \ub300\ud1b5\ub839\uc778 \uc5d8\ub7f0 \uc874\uc2a8\uc124\ub9ac\ud504 \ub294 \ubc14\uc774\ub7ec\uc2a4 \ud14c\uc2a4\ud2b8 \uc13c\ud130\uac00 \uc788\ub294 \uba87\uba87 \uad6d\uacbd\uc744 \uc81c\uc678\ud55c \ubaa8\ub4e0 \uad6d\uac00\uc758 \uc8fc\uc694 \uacf5\ud56d \ubc0f \uad6d\uacbd\uc744 \ud3d0\uc1c4\ud560 \uac83\uc774\ub77c\uace0 \ub9d0\ud588\ub2e4. \ucd5c\uc545\uc758 \uc601\ud5a5\uc744 \ubc1b\ub294 \uc9c0\uc5ed\uc5d0\ub294 \uac80\uc5ed \ud558\uc5d0 \uc788\uc744 \uac83\uc774\ub77c\uace0 \ub9d0\ud588\ub2e4. \ud070 \ud589\uc0ac \ubc0f \uc2a4\ud3ec\uce20 \ud65c\ub3d9\uc740 \uc811\ucd09 \uac10\uc5fc\uc758 \uc704\ud5d8\uc744 \ub192\uc774\uae30 \ub54c\ubb38\uc5d0 \ubaa8\ub4e0 \ucd95\uad6c \uacbd\uae30\ub3c4 \uae08\uc9c0\ub418\uc5c8\ub2e4. 7\uc6d4 30\uc77c, \ub77c\uc774\ubca0\ub9ac\uc544 \ub0b4\uc758 \ubaa8\ub4e0 \ud559\uad50\uac00 \ud734\uad50\ud588\uc73c\uba70, \uc9c0\uc5ed \uc0ac\ud68c\uac00 \uac70\uc758 \uc815\uc9c0\ub418\uc5c8\ub2e4.", "paragraph_answer": "7\uc6d4 26\uc77c, \ub77c\uc774\ubca0\ub9ac\uc544\uc5d0\uc11c \ub098\uc774\uc9c0\ub9ac\uc544\ub85c \uc628 \uc0ac\ub78c\ub4e4 \uc911 \uc5d0\ubcfc\ub77c \ubc14\uc774\ub7ec\uc2a4 \uac10\uc5fc \uc758\uc2ec \ud658\uc790\uac00 \ubc1c\uc0dd\ud55c \uc774\ud6c4 \ub098\uc774\uc9c0\ub9ac\uc544\ub294 \ubaa8\ub4e0 \uc785\uad6d \uac00\ub2a5 \uc9c0\uc5ed\uc758 \uac10\uc2dc\ub97c \uac15\ud654\ud588\ub2e4. \ubcf4\uac74 \ub2f9\uad6d\uc740 \uac01 \uc785\uad6d \uad6d\uacbd\uc5d0\uc11c \ub3c4\ucc29\ud558\ub294 \ubaa8\ub4e0 \uc0ac\ub78c\ub4e4\uc5d0\uac8c \uac10\uc5fc \ud14c\uc2a4\ud2b8\ub97c \uac70\uce60 \uac83\uc774\ub77c\uace0 \ub9d0\ud588\ub2e4. \ub098\uc774\uc9c0\ub9ac\uc544\uc758 \uc8fc\uc694 \ud56d\uacf5\uc0ac\uc778 \uc544\ub9ac\ud06c \uc5d0\uc5b4\ub294 \ub77c\uc774\ubca0\ub9ac\uc544\uc640 \uc2dc\uc5d0\ub77c\ub9ac\uc628\uc73c\ub85c \ud5a5\ud558\ub294 \ubaa8\ub4e0 \ube44\ud589\uae30\ud3b8\uc744 \uc911\ub2e8\uc2dc\ucf30\ub2e4. 7\uc6d4 27\uc77c, \ub77c\uc774\ubca0\ub9ac\uc544\uc758 \ub300\ud1b5\ub839\uc778 \uc5d8\ub7f0 \uc874\uc2a8\uc124\ub9ac\ud504 \ub294 \ubc14\uc774\ub7ec\uc2a4 \ud14c\uc2a4\ud2b8 \uc13c\ud130\uac00 \uc788\ub294 \uba87\uba87 \uad6d\uacbd\uc744 \uc81c\uc678\ud55c \ubaa8\ub4e0 \uad6d\uac00\uc758 \uc8fc\uc694 \uacf5\ud56d \ubc0f \uad6d\uacbd\uc744 \ud3d0\uc1c4\ud560 \uac83\uc774\ub77c\uace0 \ub9d0\ud588\ub2e4. \ucd5c\uc545\uc758 \uc601\ud5a5\uc744 \ubc1b\ub294 \uc9c0\uc5ed\uc5d0\ub294 \uac80\uc5ed \ud558\uc5d0 \uc788\uc744 \uac83\uc774\ub77c\uace0 \ub9d0\ud588\ub2e4. \ud070 \ud589\uc0ac \ubc0f \uc2a4\ud3ec\uce20 \ud65c\ub3d9\uc740 \uc811\ucd09 \uac10\uc5fc\uc758 \uc704\ud5d8\uc744 \ub192\uc774\uae30 \ub54c\ubb38\uc5d0 \ubaa8\ub4e0 \ucd95\uad6c \uacbd\uae30\ub3c4 \uae08\uc9c0\ub418\uc5c8\ub2e4. 7\uc6d4 30\uc77c, \ub77c\uc774\ubca0\ub9ac\uc544 \ub0b4\uc758 \ubaa8\ub4e0 \ud559\uad50\uac00 \ud734\uad50\ud588\uc73c\uba70, \uc9c0\uc5ed \uc0ac\ud68c\uac00 \uac70\uc758 \uc815\uc9c0\ub418\uc5c8\ub2e4.", "sentence_answer": "7\uc6d4 27\uc77c, \ub77c\uc774\ubca0\ub9ac\uc544\uc758 \ub300\ud1b5\ub839\uc778 \uc5d8\ub7f0 \uc874\uc2a8\uc124\ub9ac\ud504 \ub294 \ubc14\uc774\ub7ec\uc2a4 \ud14c\uc2a4\ud2b8 \uc13c\ud130\uac00 \uc788\ub294 \uba87\uba87 \uad6d\uacbd\uc744 \uc81c\uc678\ud55c \ubaa8\ub4e0 \uad6d\uac00\uc758 \uc8fc\uc694 \uacf5\ud56d \ubc0f \uad6d\uacbd\uc744 \ud3d0\uc1c4\ud560 \uac83\uc774\ub77c\uace0 \ub9d0\ud588\ub2e4.", "paragraph_id": "6511508-4-0", "paragraph_question": "question: \uc5d0\ubcfc\ub77c\uc758 \uc720\ud589 \ub2f9\uc2dc \ub77c\uc774\ubca0\ub9ac\uc544\uc758 \ub300\ud1b5\ub839 \uc774\ub984\uc740 \ubb34\uc5c7\uc778\uac00?, context: 7\uc6d4 26\uc77c, \ub77c\uc774\ubca0\ub9ac\uc544\uc5d0\uc11c \ub098\uc774\uc9c0\ub9ac\uc544\ub85c \uc628 \uc0ac\ub78c\ub4e4 \uc911 \uc5d0\ubcfc\ub77c \ubc14\uc774\ub7ec\uc2a4 \uac10\uc5fc \uc758\uc2ec \ud658\uc790\uac00 \ubc1c\uc0dd\ud55c \uc774\ud6c4 \ub098\uc774\uc9c0\ub9ac\uc544\ub294 \ubaa8\ub4e0 \uc785\uad6d \uac00\ub2a5 \uc9c0\uc5ed\uc758 \uac10\uc2dc\ub97c \uac15\ud654\ud588\ub2e4. \ubcf4\uac74 \ub2f9\uad6d\uc740 \uac01 \uc785\uad6d \uad6d\uacbd\uc5d0\uc11c \ub3c4\ucc29\ud558\ub294 \ubaa8\ub4e0 \uc0ac\ub78c\ub4e4\uc5d0\uac8c \uac10\uc5fc \ud14c\uc2a4\ud2b8\ub97c \uac70\uce60 \uac83\uc774\ub77c\uace0 \ub9d0\ud588\ub2e4. \ub098\uc774\uc9c0\ub9ac\uc544\uc758 \uc8fc\uc694 \ud56d\uacf5\uc0ac\uc778 \uc544\ub9ac\ud06c \uc5d0\uc5b4\ub294 \ub77c\uc774\ubca0\ub9ac\uc544\uc640 \uc2dc\uc5d0\ub77c\ub9ac\uc628\uc73c\ub85c \ud5a5\ud558\ub294 \ubaa8\ub4e0 \ube44\ud589\uae30\ud3b8\uc744 \uc911\ub2e8\uc2dc\ucf30\ub2e4. 7\uc6d4 27\uc77c, \ub77c\uc774\ubca0\ub9ac\uc544\uc758 \ub300\ud1b5\ub839\uc778 \uc5d8\ub7f0 \uc874\uc2a8\uc124\ub9ac\ud504\ub294 \ubc14\uc774\ub7ec\uc2a4 \ud14c\uc2a4\ud2b8 \uc13c\ud130\uac00 \uc788\ub294 \uba87\uba87 \uad6d\uacbd\uc744 \uc81c\uc678\ud55c \ubaa8\ub4e0 \uad6d\uac00\uc758 \uc8fc\uc694 \uacf5\ud56d \ubc0f \uad6d\uacbd\uc744 \ud3d0\uc1c4\ud560 \uac83\uc774\ub77c\uace0 \ub9d0\ud588\ub2e4. \ucd5c\uc545\uc758 \uc601\ud5a5\uc744 \ubc1b\ub294 \uc9c0\uc5ed\uc5d0\ub294 \uac80\uc5ed \ud558\uc5d0 \uc788\uc744 \uac83\uc774\ub77c\uace0 \ub9d0\ud588\ub2e4. \ud070 \ud589\uc0ac \ubc0f \uc2a4\ud3ec\uce20 \ud65c\ub3d9\uc740 \uc811\ucd09 \uac10\uc5fc\uc758 \uc704\ud5d8\uc744 \ub192\uc774\uae30 \ub54c\ubb38\uc5d0 \ubaa8\ub4e0 \ucd95\uad6c \uacbd\uae30\ub3c4 \uae08\uc9c0\ub418\uc5c8\ub2e4. 7\uc6d4 30\uc77c, \ub77c\uc774\ubca0\ub9ac\uc544 \ub0b4\uc758 \ubaa8\ub4e0 \ud559\uad50\uac00 \ud734\uad50\ud588\uc73c\uba70, \uc9c0\uc5ed \uc0ac\ud68c\uac00 \uac70\uc758 \uc815\uc9c0\ub418\uc5c8\ub2e4."} {"question": "\ub3d9\uc77c\ubcf8\ub300\uc9c0\uc9c4\uc5d0\uc11c \ub2e8\uce35 \ud30c\uad34\uac00 \uc2dc\uc791\ub41c \uc9c4\uc6d0\uc9c0\ub294 \uc5b4\ub514\uc778\uac00?", "paragraph": "\uc77c\ubcf8 \uae30\uc0c1\uccad\uacfc \ub3c4\ucfc4 \ub300\ud559 \uc9c0\uc9c4\uc5f0\uad6c\uc18c \ub4f1\uc5d0\uc11c\ub294 \ub3d9\uc77c\ubcf8\ub300\uc9c0\uc9c4\uc774 \ub2e8\uce35\uba74\uc758 \uac01\ub3c4\uac00 10\ub3c4 \uc815\ub3c4\ub85c \ub418\uac8c \uc791\uc73c\uba70, \uc11c\ubd81\uc11c-\ub3d9\ub0a8\ub3d9 \ubc29\ud5a5(\ub3d9\uc11c \ubc29\ud5a5\uc5d0 \uac70\uc758 \uac00\uae4c\uc6b4 \ubc29\ud5a5)\uc744 \ucd95\uc73c\ub85c \uc555\ub825\uc744 \ubc1b\uc740 \ub0ae\uc740 \uac01\ub3c4\uc758 \uc5ed\ub2e8\uce35(\uc2a4\ub7ec\uc2a4\ud2b8 \ub2e8\uce35)\uc5d0\uc11c \uc77c\uc5b4\ub0ac\ub2e4\uace0 \ubd84\uc11d\ud588\ub2e4. \ub610\ud55c \ub3d9\uc77c\ubcf8\ub300\uc9c0\uc9c4\uc740 \uc218\ud3c9 \ubc29\ud5a5 \ubcc0\ud654\ub7c9\uc774 \ud070\ub370, \uc774 \uc810\uc740 \ub3c4\ud638\ucfe0 \uc9c0\ubc29 \ud0dc\ud3c9\uc591 \uc5f0\uc548 \uc9c0\uc5ed\uc5d0\uc11c \uc77c\uc5b4\ub098\ub294 \ud574\uad6c\ud615 \uc9c0\uc9c4\uc758 \uc8fc\uc694\ud55c \ud2b9\uc9d5 \uc911 \ud558\ub098\uc774\ub2e4. \ub2e8\uce35 \ud30c\uad34\uac00 \uc2dc\uc791\ub41c \uc9c4\uc6d0\uc9c0\ub294 \uc0b0\ub9ac\ucfe0 \uc55e\ubc14\ub2e4\uc600\uc9c0\ub9cc, \ub2e8\uce35 \ud30c\uad34\uac00 \uc774\ub904\uc9c4 \uc9c4\uc6d0\uc5ed\uc740 \uc77c\ubcf8 \ud574\uad6c \uc544\ub798 \ud310 \uacbd\uacc4\uba74\uc744 \ub530\ub77c \ub0a8\ubd81\uc73c\ub85c \uae38\uac8c \uc774\uc5b4\uc84c\uace0, \uc774\uc640\ud14c\ud604 \uc55e\ubc14\ub2e4\ubd80\ud130 \uc774\ubc14\ub77c\ud0a4\ud604 \uc55e\ubc14\ub2e4\uae4c\uc9c0 \uc774\uc5b4\uc9c0\ub294 \ub0a8\ubd81 \uc57d 500km, \ub3d9\uc11c \uc57d 200km, \uae4a\uc774 \uc57d 4-20km\uc758 \ucd1d 20\ub9cc km\uc5d0 \uc774\ub974\ub294 \uc5c4\uccad\ub09c \ub113\uc774\uc758 \uc601\uc5ed\uc774 \uc9c4\uc6d0\uc5ed\uc774\uc5c8\ub2e4. 2004\ub144\uc758 \ub0a8\uc544\uc2dc\uc544 \ub300\uc9c0\uc9c4\uacfc \ube44\uad50\ud558\uba74, \ub0a8\uc544\uc2dc\uc544 \ub300\uc9c0\uc9c4\uc740 \uc9c4\uc6d0\uc5ed\uc774 1,000km\uc5d0 \ub2ec\ud588\uc73c\ub098 \uc774 \ub3c4\ud638\ucfe0 \uc9c0\uc9c4\uc740 500km\ubc16\uc5d0 \uc548\ub418\ub294 \uc601\uc5ed\uc5d0\uc11c\ub3c4 \ube44\uc2b7\ud55c M9 \uaddc\ubaa8\uc758 \uc9c0\uc9c4\uc774 \uc77c\uc5b4\ub0ac\ub2e4. \uc989 \ubbf8\uc57c\uae30 \ud604 \uc55e\ubc14\ub2e4 \uc9c4\uc559 \ubd80\uadfc\uc5d0\uc11c \uc9c0\uce35 \ubcc0\ud654\ub7c9\uc774 \ub9e4\uc6b0 \ucef8\ub2e4\ub294 \uac83\uc774\ub2e4.", "answer": "\uc0b0\ub9ac\ucfe0 \uc55e\ubc14\ub2e4", "sentence": "\ub2e8\uce35 \ud30c\uad34\uac00 \uc2dc\uc791\ub41c \uc9c4\uc6d0\uc9c0\ub294 \uc0b0\ub9ac\ucfe0 \uc55e\ubc14\ub2e4 \uc600\uc9c0\ub9cc, \ub2e8\uce35 \ud30c\uad34\uac00 \uc774\ub904\uc9c4 \uc9c4\uc6d0\uc5ed\uc740 \uc77c\ubcf8 \ud574\uad6c \uc544\ub798 \ud310 \uacbd\uacc4\uba74\uc744 \ub530\ub77c \ub0a8\ubd81\uc73c\ub85c \uae38\uac8c \uc774\uc5b4\uc84c\uace0, \uc774\uc640\ud14c\ud604 \uc55e\ubc14\ub2e4\ubd80\ud130 \uc774\ubc14\ub77c\ud0a4\ud604 \uc55e\ubc14\ub2e4\uae4c\uc9c0 \uc774\uc5b4\uc9c0\ub294 \ub0a8\ubd81 \uc57d 500km, \ub3d9\uc11c \uc57d 200km, \uae4a\uc774 \uc57d 4-20km\uc758 \ucd1d 20\ub9cc km\uc5d0 \uc774\ub974\ub294 \uc5c4\uccad\ub09c \ub113\uc774\uc758 \uc601\uc5ed\uc774 \uc9c4\uc6d0\uc5ed\uc774\uc5c8\ub2e4.", "paragraph_sentence": "\uc77c\ubcf8 \uae30\uc0c1\uccad\uacfc \ub3c4\ucfc4 \ub300\ud559 \uc9c0\uc9c4\uc5f0\uad6c\uc18c \ub4f1\uc5d0\uc11c\ub294 \ub3d9\uc77c\ubcf8\ub300\uc9c0\uc9c4\uc774 \ub2e8\uce35\uba74\uc758 \uac01\ub3c4\uac00 10\ub3c4 \uc815\ub3c4\ub85c \ub418\uac8c \uc791\uc73c\uba70, \uc11c\ubd81\uc11c-\ub3d9\ub0a8\ub3d9 \ubc29\ud5a5(\ub3d9\uc11c \ubc29\ud5a5\uc5d0 \uac70\uc758 \uac00\uae4c\uc6b4 \ubc29\ud5a5)\uc744 \ucd95\uc73c\ub85c \uc555\ub825\uc744 \ubc1b\uc740 \ub0ae\uc740 \uac01\ub3c4\uc758 \uc5ed\ub2e8\uce35(\uc2a4\ub7ec\uc2a4\ud2b8 \ub2e8\uce35)\uc5d0\uc11c \uc77c\uc5b4\ub0ac\ub2e4\uace0 \ubd84\uc11d\ud588\ub2e4. \ub610\ud55c \ub3d9\uc77c\ubcf8\ub300\uc9c0\uc9c4\uc740 \uc218\ud3c9 \ubc29\ud5a5 \ubcc0\ud654\ub7c9\uc774 \ud070\ub370, \uc774 \uc810\uc740 \ub3c4\ud638\ucfe0 \uc9c0\ubc29 \ud0dc\ud3c9\uc591 \uc5f0\uc548 \uc9c0\uc5ed\uc5d0\uc11c \uc77c\uc5b4\ub098\ub294 \ud574\uad6c\ud615 \uc9c0\uc9c4\uc758 \uc8fc\uc694\ud55c \ud2b9\uc9d5 \uc911 \ud558\ub098\uc774\ub2e4. \ub2e8\uce35 \ud30c\uad34\uac00 \uc2dc\uc791\ub41c \uc9c4\uc6d0\uc9c0\ub294 \uc0b0\ub9ac\ucfe0 \uc55e\ubc14\ub2e4 \uc600\uc9c0\ub9cc, \ub2e8\uce35 \ud30c\uad34\uac00 \uc774\ub904\uc9c4 \uc9c4\uc6d0\uc5ed\uc740 \uc77c\ubcf8 \ud574\uad6c \uc544\ub798 \ud310 \uacbd\uacc4\uba74\uc744 \ub530\ub77c \ub0a8\ubd81\uc73c\ub85c \uae38\uac8c \uc774\uc5b4\uc84c\uace0, \uc774\uc640\ud14c\ud604 \uc55e\ubc14\ub2e4\ubd80\ud130 \uc774\ubc14\ub77c\ud0a4\ud604 \uc55e\ubc14\ub2e4\uae4c\uc9c0 \uc774\uc5b4\uc9c0\ub294 \ub0a8\ubd81 \uc57d 500km, \ub3d9\uc11c \uc57d 200km, \uae4a\uc774 \uc57d 4-20km\uc758 \ucd1d 20\ub9cc km\uc5d0 \uc774\ub974\ub294 \uc5c4\uccad\ub09c \ub113\uc774\uc758 \uc601\uc5ed\uc774 \uc9c4\uc6d0\uc5ed\uc774\uc5c8\ub2e4. 2004\ub144\uc758 \ub0a8\uc544\uc2dc\uc544 \ub300\uc9c0\uc9c4\uacfc \ube44\uad50\ud558\uba74, \ub0a8\uc544\uc2dc\uc544 \ub300\uc9c0\uc9c4\uc740 \uc9c4\uc6d0\uc5ed\uc774 1,000km\uc5d0 \ub2ec\ud588\uc73c\ub098 \uc774 \ub3c4\ud638\ucfe0 \uc9c0\uc9c4\uc740 500km\ubc16\uc5d0 \uc548\ub418\ub294 \uc601\uc5ed\uc5d0\uc11c\ub3c4 \ube44\uc2b7\ud55c M9 \uaddc\ubaa8\uc758 \uc9c0\uc9c4\uc774 \uc77c\uc5b4\ub0ac\ub2e4. \uc989 \ubbf8\uc57c\uae30 \ud604 \uc55e\ubc14\ub2e4 \uc9c4\uc559 \ubd80\uadfc\uc5d0\uc11c \uc9c0\uce35 \ubcc0\ud654\ub7c9\uc774 \ub9e4\uc6b0 \ucef8\ub2e4\ub294 \uac83\uc774\ub2e4.", "paragraph_answer": "\uc77c\ubcf8 \uae30\uc0c1\uccad\uacfc \ub3c4\ucfc4 \ub300\ud559 \uc9c0\uc9c4\uc5f0\uad6c\uc18c \ub4f1\uc5d0\uc11c\ub294 \ub3d9\uc77c\ubcf8\ub300\uc9c0\uc9c4\uc774 \ub2e8\uce35\uba74\uc758 \uac01\ub3c4\uac00 10\ub3c4 \uc815\ub3c4\ub85c \ub418\uac8c \uc791\uc73c\uba70, \uc11c\ubd81\uc11c-\ub3d9\ub0a8\ub3d9 \ubc29\ud5a5(\ub3d9\uc11c \ubc29\ud5a5\uc5d0 \uac70\uc758 \uac00\uae4c\uc6b4 \ubc29\ud5a5)\uc744 \ucd95\uc73c\ub85c \uc555\ub825\uc744 \ubc1b\uc740 \ub0ae\uc740 \uac01\ub3c4\uc758 \uc5ed\ub2e8\uce35(\uc2a4\ub7ec\uc2a4\ud2b8 \ub2e8\uce35)\uc5d0\uc11c \uc77c\uc5b4\ub0ac\ub2e4\uace0 \ubd84\uc11d\ud588\ub2e4. \ub610\ud55c \ub3d9\uc77c\ubcf8\ub300\uc9c0\uc9c4\uc740 \uc218\ud3c9 \ubc29\ud5a5 \ubcc0\ud654\ub7c9\uc774 \ud070\ub370, \uc774 \uc810\uc740 \ub3c4\ud638\ucfe0 \uc9c0\ubc29 \ud0dc\ud3c9\uc591 \uc5f0\uc548 \uc9c0\uc5ed\uc5d0\uc11c \uc77c\uc5b4\ub098\ub294 \ud574\uad6c\ud615 \uc9c0\uc9c4\uc758 \uc8fc\uc694\ud55c \ud2b9\uc9d5 \uc911 \ud558\ub098\uc774\ub2e4. \ub2e8\uce35 \ud30c\uad34\uac00 \uc2dc\uc791\ub41c \uc9c4\uc6d0\uc9c0\ub294 \uc0b0\ub9ac\ucfe0 \uc55e\ubc14\ub2e4 \uc600\uc9c0\ub9cc, \ub2e8\uce35 \ud30c\uad34\uac00 \uc774\ub904\uc9c4 \uc9c4\uc6d0\uc5ed\uc740 \uc77c\ubcf8 \ud574\uad6c \uc544\ub798 \ud310 \uacbd\uacc4\uba74\uc744 \ub530\ub77c \ub0a8\ubd81\uc73c\ub85c \uae38\uac8c \uc774\uc5b4\uc84c\uace0, \uc774\uc640\ud14c\ud604 \uc55e\ubc14\ub2e4\ubd80\ud130 \uc774\ubc14\ub77c\ud0a4\ud604 \uc55e\ubc14\ub2e4\uae4c\uc9c0 \uc774\uc5b4\uc9c0\ub294 \ub0a8\ubd81 \uc57d 500km, \ub3d9\uc11c \uc57d 200km, \uae4a\uc774 \uc57d 4-20km\uc758 \ucd1d 20\ub9cc km\uc5d0 \uc774\ub974\ub294 \uc5c4\uccad\ub09c \ub113\uc774\uc758 \uc601\uc5ed\uc774 \uc9c4\uc6d0\uc5ed\uc774\uc5c8\ub2e4. 2004\ub144\uc758 \ub0a8\uc544\uc2dc\uc544 \ub300\uc9c0\uc9c4\uacfc \ube44\uad50\ud558\uba74, \ub0a8\uc544\uc2dc\uc544 \ub300\uc9c0\uc9c4\uc740 \uc9c4\uc6d0\uc5ed\uc774 1,000km\uc5d0 \ub2ec\ud588\uc73c\ub098 \uc774 \ub3c4\ud638\ucfe0 \uc9c0\uc9c4\uc740 500km\ubc16\uc5d0 \uc548\ub418\ub294 \uc601\uc5ed\uc5d0\uc11c\ub3c4 \ube44\uc2b7\ud55c M9 \uaddc\ubaa8\uc758 \uc9c0\uc9c4\uc774 \uc77c\uc5b4\ub0ac\ub2e4. \uc989 \ubbf8\uc57c\uae30 \ud604 \uc55e\ubc14\ub2e4 \uc9c4\uc559 \ubd80\uadfc\uc5d0\uc11c \uc9c0\uce35 \ubcc0\ud654\ub7c9\uc774 \ub9e4\uc6b0 \ucef8\ub2e4\ub294 \uac83\uc774\ub2e4.", "sentence_answer": "\ub2e8\uce35 \ud30c\uad34\uac00 \uc2dc\uc791\ub41c \uc9c4\uc6d0\uc9c0\ub294 \uc0b0\ub9ac\ucfe0 \uc55e\ubc14\ub2e4 \uc600\uc9c0\ub9cc, \ub2e8\uce35 \ud30c\uad34\uac00 \uc774\ub904\uc9c4 \uc9c4\uc6d0\uc5ed\uc740 \uc77c\ubcf8 \ud574\uad6c \uc544\ub798 \ud310 \uacbd\uacc4\uba74\uc744 \ub530\ub77c \ub0a8\ubd81\uc73c\ub85c \uae38\uac8c \uc774\uc5b4\uc84c\uace0, \uc774\uc640\ud14c\ud604 \uc55e\ubc14\ub2e4\ubd80\ud130 \uc774\ubc14\ub77c\ud0a4\ud604 \uc55e\ubc14\ub2e4\uae4c\uc9c0 \uc774\uc5b4\uc9c0\ub294 \ub0a8\ubd81 \uc57d 500km, \ub3d9\uc11c \uc57d 200km, \uae4a\uc774 \uc57d 4-20km\uc758 \ucd1d 20\ub9cc km\uc5d0 \uc774\ub974\ub294 \uc5c4\uccad\ub09c \ub113\uc774\uc758 \uc601\uc5ed\uc774 \uc9c4\uc6d0\uc5ed\uc774\uc5c8\ub2e4.", "paragraph_id": "6492086-6-2", "paragraph_question": "question: \ub3d9\uc77c\ubcf8\ub300\uc9c0\uc9c4\uc5d0\uc11c \ub2e8\uce35 \ud30c\uad34\uac00 \uc2dc\uc791\ub41c \uc9c4\uc6d0\uc9c0\ub294 \uc5b4\ub514\uc778\uac00?, context: \uc77c\ubcf8 \uae30\uc0c1\uccad\uacfc \ub3c4\ucfc4 \ub300\ud559 \uc9c0\uc9c4\uc5f0\uad6c\uc18c \ub4f1\uc5d0\uc11c\ub294 \ub3d9\uc77c\ubcf8\ub300\uc9c0\uc9c4\uc774 \ub2e8\uce35\uba74\uc758 \uac01\ub3c4\uac00 10\ub3c4 \uc815\ub3c4\ub85c \ub418\uac8c \uc791\uc73c\uba70, \uc11c\ubd81\uc11c-\ub3d9\ub0a8\ub3d9 \ubc29\ud5a5(\ub3d9\uc11c \ubc29\ud5a5\uc5d0 \uac70\uc758 \uac00\uae4c\uc6b4 \ubc29\ud5a5)\uc744 \ucd95\uc73c\ub85c \uc555\ub825\uc744 \ubc1b\uc740 \ub0ae\uc740 \uac01\ub3c4\uc758 \uc5ed\ub2e8\uce35(\uc2a4\ub7ec\uc2a4\ud2b8 \ub2e8\uce35)\uc5d0\uc11c \uc77c\uc5b4\ub0ac\ub2e4\uace0 \ubd84\uc11d\ud588\ub2e4. \ub610\ud55c \ub3d9\uc77c\ubcf8\ub300\uc9c0\uc9c4\uc740 \uc218\ud3c9 \ubc29\ud5a5 \ubcc0\ud654\ub7c9\uc774 \ud070\ub370, \uc774 \uc810\uc740 \ub3c4\ud638\ucfe0 \uc9c0\ubc29 \ud0dc\ud3c9\uc591 \uc5f0\uc548 \uc9c0\uc5ed\uc5d0\uc11c \uc77c\uc5b4\ub098\ub294 \ud574\uad6c\ud615 \uc9c0\uc9c4\uc758 \uc8fc\uc694\ud55c \ud2b9\uc9d5 \uc911 \ud558\ub098\uc774\ub2e4. \ub2e8\uce35 \ud30c\uad34\uac00 \uc2dc\uc791\ub41c \uc9c4\uc6d0\uc9c0\ub294 \uc0b0\ub9ac\ucfe0 \uc55e\ubc14\ub2e4\uc600\uc9c0\ub9cc, \ub2e8\uce35 \ud30c\uad34\uac00 \uc774\ub904\uc9c4 \uc9c4\uc6d0\uc5ed\uc740 \uc77c\ubcf8 \ud574\uad6c \uc544\ub798 \ud310 \uacbd\uacc4\uba74\uc744 \ub530\ub77c \ub0a8\ubd81\uc73c\ub85c \uae38\uac8c \uc774\uc5b4\uc84c\uace0, \uc774\uc640\ud14c\ud604 \uc55e\ubc14\ub2e4\ubd80\ud130 \uc774\ubc14\ub77c\ud0a4\ud604 \uc55e\ubc14\ub2e4\uae4c\uc9c0 \uc774\uc5b4\uc9c0\ub294 \ub0a8\ubd81 \uc57d 500km, \ub3d9\uc11c \uc57d 200km, \uae4a\uc774 \uc57d 4-20km\uc758 \ucd1d 20\ub9cc km\uc5d0 \uc774\ub974\ub294 \uc5c4\uccad\ub09c \ub113\uc774\uc758 \uc601\uc5ed\uc774 \uc9c4\uc6d0\uc5ed\uc774\uc5c8\ub2e4. 2004\ub144\uc758 \ub0a8\uc544\uc2dc\uc544 \ub300\uc9c0\uc9c4\uacfc \ube44\uad50\ud558\uba74, \ub0a8\uc544\uc2dc\uc544 \ub300\uc9c0\uc9c4\uc740 \uc9c4\uc6d0\uc5ed\uc774 1,000km\uc5d0 \ub2ec\ud588\uc73c\ub098 \uc774 \ub3c4\ud638\ucfe0 \uc9c0\uc9c4\uc740 500km\ubc16\uc5d0 \uc548\ub418\ub294 \uc601\uc5ed\uc5d0\uc11c\ub3c4 \ube44\uc2b7\ud55c M9 \uaddc\ubaa8\uc758 \uc9c0\uc9c4\uc774 \uc77c\uc5b4\ub0ac\ub2e4. \uc989 \ubbf8\uc57c\uae30 \ud604 \uc55e\ubc14\ub2e4 \uc9c4\uc559 \ubd80\uadfc\uc5d0\uc11c \uc9c0\uce35 \ubcc0\ud654\ub7c9\uc774 \ub9e4\uc6b0 \ucef8\ub2e4\ub294 \uac83\uc774\ub2e4."} {"question": "\uc774\uc81c\ud6c8\uc774 \uc758\ubb34\uacbd\ucc30\ub85c \ubcf5\ubb34\ud55c \uae30\uad00\uc740?", "paragraph": "\uc904\uace7 \ub2e8\ud3b8\uc601\ud654\uc640 \ub3c5\ub9bd\uc601\ud654\uacc4\uc5d0\uc11c \uc5f0\uae30\ub97c \ud3bc\uce58\ub2e4 2011\ub144 \u300a\ud30c\uc218\uafbc\u300b\uacfc \u300a\uace0\uc9c0\uc804\u300b \ub450 \uc791\ud488\uc73c\ub85c \uc790\uc2e0\uc758 \uc874\uc7ac\ub97c \ucda9\ubb34\ub85c\uc5d0 \uac01\uc778\uc2dc\ucf30\uace0, \uc2e0\uc778\uc0c1 6\uad00\uc655\uc5d0 \uc62c\ub790\ub2e4. \uc774\ud6c4 \uc601\ud654 \u300a\uac74\ucd95\ud559\uac1c\ub860\u300b, \u300a\uc810\uc7c1\uc774\ub4e4\u300b, \u300a\ubd84\ub178\uc758 \uc724\ub9ac\ud559\u300b, \u300a\ud30c\ud30c\ub85c\ud2f0\u300b\uc5d0 \uc8fc\uc5f0\uc73c\ub85c \ucd9c\uc5f0\ud588\uace0, 2012\ub144\ubd80\ud130 2014\ub144\uae4c\uc9c0 \uc11c\uc6b8\uc9c0\ubc29\uacbd\ucc30\uccad\uc5d0\uc11c \uc758\ubb34\uacbd\ucc30\ub85c \ubcf5\ubb34\ud588\ub2e4. 2014\ub144 \u300a\ube44\ubc00\uc758 \ubb38\u300b\uc744 \ud1b5\ud574 \ubcf5\uadc0\ud588\uace0, SBS \uc5f0\uae30\ub300\uc0c1\uc5d0\uc11c \ucd5c\uc6b0\uc218\uc0c1\uc744 \ubc1b\uc558\ub2e4. 2016\ub144\uc5d0\ub294 \u300a\uc2dc\uadf8\ub110\u300b\uc5d0 \ucd9c\uc5f0\ud588\uace0, \uc601\ud654 \ubcf5\uadc0\uc791\uc778 \u300a\ud0d0\uc815 \ud64d\uae38\ub3d9: \uc0ac\ub77c\uc9c4 \ub9c8\uc744\u300b\uc774 \uac1c\ubd09\ud588\ub2e4. \uadf8\ub9ac\uace0 2017\ub144 2\uc6d4 3\uc77c\u300a\ub0b4\uc77c \uadf8\ub300\uc640\u300b\uc5d0\uc11c \uc720\uc18c\uc900 \uc5ed\ud560\uc744 \ub9e1\uc558\uace0, \uadf8 \ud574 6\uc6d4 28\uc77c \uc601\ud654\u300a\ubc15\uc5f4\u300b\ud070 \ud765\ud589\uc744 \uc774\uc5b4\ub098\uac14\ub2e4. 9\uc6d4 21\uc77c \uc601\ud654\u300a\uc544\uc774 \uce94 \uc2a4\ud53c\ud06c\u300b\uc5d0\uc11c \ubc15\ubbfc\uc7ac \uc5ed\uc744 \ub9e1\uc558\ub2e4.", "answer": "\uc11c\uc6b8\uc9c0\ubc29\uacbd\ucc30\uccad", "sentence": "\uc774\ud6c4 \uc601\ud654 \u300a\uac74\ucd95\ud559\uac1c\ub860\u300b, \u300a\uc810\uc7c1\uc774\ub4e4\u300b, \u300a\ubd84\ub178\uc758 \uc724\ub9ac\ud559\u300b, \u300a\ud30c\ud30c\ub85c\ud2f0\u300b\uc5d0 \uc8fc\uc5f0\uc73c\ub85c \ucd9c\uc5f0\ud588\uace0, 2012\ub144\ubd80\ud130 2014\ub144\uae4c\uc9c0 \uc11c\uc6b8\uc9c0\ubc29\uacbd\ucc30\uccad \uc5d0\uc11c \uc758\ubb34\uacbd\ucc30\ub85c \ubcf5\ubb34\ud588\ub2e4.", "paragraph_sentence": "\uc904\uace7 \ub2e8\ud3b8\uc601\ud654\uc640 \ub3c5\ub9bd\uc601\ud654\uacc4\uc5d0\uc11c \uc5f0\uae30\ub97c \ud3bc\uce58\ub2e4 2011\ub144 \u300a\ud30c\uc218\uafbc\u300b\uacfc \u300a\uace0\uc9c0\uc804\u300b \ub450 \uc791\ud488\uc73c\ub85c \uc790\uc2e0\uc758 \uc874\uc7ac\ub97c \ucda9\ubb34\ub85c\uc5d0 \uac01\uc778\uc2dc\ucf30\uace0, \uc2e0\uc778\uc0c1 6\uad00\uc655\uc5d0 \uc62c\ub790\ub2e4. \uc774\ud6c4 \uc601\ud654 \u300a\uac74\ucd95\ud559\uac1c\ub860\u300b, \u300a\uc810\uc7c1\uc774\ub4e4\u300b, \u300a\ubd84\ub178\uc758 \uc724\ub9ac\ud559\u300b, \u300a\ud30c\ud30c\ub85c\ud2f0\u300b\uc5d0 \uc8fc\uc5f0\uc73c\ub85c \ucd9c\uc5f0\ud588\uace0, 2012\ub144\ubd80\ud130 2014\ub144\uae4c\uc9c0 \uc11c\uc6b8\uc9c0\ubc29\uacbd\ucc30\uccad \uc5d0\uc11c \uc758\ubb34\uacbd\ucc30\ub85c \ubcf5\ubb34\ud588\ub2e4. 2014\ub144 \u300a\ube44\ubc00\uc758 \ubb38\u300b\uc744 \ud1b5\ud574 \ubcf5\uadc0\ud588\uace0, SBS \uc5f0\uae30\ub300\uc0c1\uc5d0\uc11c \ucd5c\uc6b0\uc218\uc0c1\uc744 \ubc1b\uc558\ub2e4. 2016\ub144\uc5d0\ub294 \u300a\uc2dc\uadf8\ub110\u300b\uc5d0 \ucd9c\uc5f0\ud588\uace0, \uc601\ud654 \ubcf5\uadc0\uc791\uc778 \u300a\ud0d0\uc815 \ud64d\uae38\ub3d9: \uc0ac\ub77c\uc9c4 \ub9c8\uc744\u300b\uc774 \uac1c\ubd09\ud588\ub2e4. \uadf8\ub9ac\uace0 2017\ub144 2\uc6d4 3\uc77c\u300a\ub0b4\uc77c \uadf8\ub300\uc640\u300b\uc5d0\uc11c \uc720\uc18c\uc900 \uc5ed\ud560\uc744 \ub9e1\uc558\uace0, \uadf8 \ud574 6\uc6d4 28\uc77c \uc601\ud654\u300a\ubc15\uc5f4\u300b\ud070 \ud765\ud589\uc744 \uc774\uc5b4\ub098\uac14\ub2e4. 9\uc6d4 21\uc77c \uc601\ud654\u300a\uc544\uc774 \uce94 \uc2a4\ud53c\ud06c\u300b\uc5d0\uc11c \ubc15\ubbfc\uc7ac \uc5ed\uc744 \ub9e1\uc558\ub2e4.", "paragraph_answer": "\uc904\uace7 \ub2e8\ud3b8\uc601\ud654\uc640 \ub3c5\ub9bd\uc601\ud654\uacc4\uc5d0\uc11c \uc5f0\uae30\ub97c \ud3bc\uce58\ub2e4 2011\ub144 \u300a\ud30c\uc218\uafbc\u300b\uacfc \u300a\uace0\uc9c0\uc804\u300b \ub450 \uc791\ud488\uc73c\ub85c \uc790\uc2e0\uc758 \uc874\uc7ac\ub97c \ucda9\ubb34\ub85c\uc5d0 \uac01\uc778\uc2dc\ucf30\uace0, \uc2e0\uc778\uc0c1 6\uad00\uc655\uc5d0 \uc62c\ub790\ub2e4. \uc774\ud6c4 \uc601\ud654 \u300a\uac74\ucd95\ud559\uac1c\ub860\u300b, \u300a\uc810\uc7c1\uc774\ub4e4\u300b, \u300a\ubd84\ub178\uc758 \uc724\ub9ac\ud559\u300b, \u300a\ud30c\ud30c\ub85c\ud2f0\u300b\uc5d0 \uc8fc\uc5f0\uc73c\ub85c \ucd9c\uc5f0\ud588\uace0, 2012\ub144\ubd80\ud130 2014\ub144\uae4c\uc9c0 \uc11c\uc6b8\uc9c0\ubc29\uacbd\ucc30\uccad \uc5d0\uc11c \uc758\ubb34\uacbd\ucc30\ub85c \ubcf5\ubb34\ud588\ub2e4. 2014\ub144 \u300a\ube44\ubc00\uc758 \ubb38\u300b\uc744 \ud1b5\ud574 \ubcf5\uadc0\ud588\uace0, SBS \uc5f0\uae30\ub300\uc0c1\uc5d0\uc11c \ucd5c\uc6b0\uc218\uc0c1\uc744 \ubc1b\uc558\ub2e4. 2016\ub144\uc5d0\ub294 \u300a\uc2dc\uadf8\ub110\u300b\uc5d0 \ucd9c\uc5f0\ud588\uace0, \uc601\ud654 \ubcf5\uadc0\uc791\uc778 \u300a\ud0d0\uc815 \ud64d\uae38\ub3d9: \uc0ac\ub77c\uc9c4 \ub9c8\uc744\u300b\uc774 \uac1c\ubd09\ud588\ub2e4. \uadf8\ub9ac\uace0 2017\ub144 2\uc6d4 3\uc77c\u300a\ub0b4\uc77c \uadf8\ub300\uc640\u300b\uc5d0\uc11c \uc720\uc18c\uc900 \uc5ed\ud560\uc744 \ub9e1\uc558\uace0, \uadf8 \ud574 6\uc6d4 28\uc77c \uc601\ud654\u300a\ubc15\uc5f4\u300b\ud070 \ud765\ud589\uc744 \uc774\uc5b4\ub098\uac14\ub2e4. 9\uc6d4 21\uc77c \uc601\ud654\u300a\uc544\uc774 \uce94 \uc2a4\ud53c\ud06c\u300b\uc5d0\uc11c \ubc15\ubbfc\uc7ac \uc5ed\uc744 \ub9e1\uc558\ub2e4.", "sentence_answer": "\uc774\ud6c4 \uc601\ud654 \u300a\uac74\ucd95\ud559\uac1c\ub860\u300b, \u300a\uc810\uc7c1\uc774\ub4e4\u300b, \u300a\ubd84\ub178\uc758 \uc724\ub9ac\ud559\u300b, \u300a\ud30c\ud30c\ub85c\ud2f0\u300b\uc5d0 \uc8fc\uc5f0\uc73c\ub85c \ucd9c\uc5f0\ud588\uace0, 2012\ub144\ubd80\ud130 2014\ub144\uae4c\uc9c0 \uc11c\uc6b8\uc9c0\ubc29\uacbd\ucc30\uccad \uc5d0\uc11c \uc758\ubb34\uacbd\ucc30\ub85c \ubcf5\ubb34\ud588\ub2e4.", "paragraph_id": "6547787-0-0", "paragraph_question": "question: \uc774\uc81c\ud6c8\uc774 \uc758\ubb34\uacbd\ucc30\ub85c \ubcf5\ubb34\ud55c \uae30\uad00\uc740?, context: \uc904\uace7 \ub2e8\ud3b8\uc601\ud654\uc640 \ub3c5\ub9bd\uc601\ud654\uacc4\uc5d0\uc11c \uc5f0\uae30\ub97c \ud3bc\uce58\ub2e4 2011\ub144 \u300a\ud30c\uc218\uafbc\u300b\uacfc \u300a\uace0\uc9c0\uc804\u300b \ub450 \uc791\ud488\uc73c\ub85c \uc790\uc2e0\uc758 \uc874\uc7ac\ub97c \ucda9\ubb34\ub85c\uc5d0 \uac01\uc778\uc2dc\ucf30\uace0, \uc2e0\uc778\uc0c1 6\uad00\uc655\uc5d0 \uc62c\ub790\ub2e4. \uc774\ud6c4 \uc601\ud654 \u300a\uac74\ucd95\ud559\uac1c\ub860\u300b, \u300a\uc810\uc7c1\uc774\ub4e4\u300b, \u300a\ubd84\ub178\uc758 \uc724\ub9ac\ud559\u300b, \u300a\ud30c\ud30c\ub85c\ud2f0\u300b\uc5d0 \uc8fc\uc5f0\uc73c\ub85c \ucd9c\uc5f0\ud588\uace0, 2012\ub144\ubd80\ud130 2014\ub144\uae4c\uc9c0 \uc11c\uc6b8\uc9c0\ubc29\uacbd\ucc30\uccad\uc5d0\uc11c \uc758\ubb34\uacbd\ucc30\ub85c \ubcf5\ubb34\ud588\ub2e4. 2014\ub144 \u300a\ube44\ubc00\uc758 \ubb38\u300b\uc744 \ud1b5\ud574 \ubcf5\uadc0\ud588\uace0, SBS \uc5f0\uae30\ub300\uc0c1\uc5d0\uc11c \ucd5c\uc6b0\uc218\uc0c1\uc744 \ubc1b\uc558\ub2e4. 2016\ub144\uc5d0\ub294 \u300a\uc2dc\uadf8\ub110\u300b\uc5d0 \ucd9c\uc5f0\ud588\uace0, \uc601\ud654 \ubcf5\uadc0\uc791\uc778 \u300a\ud0d0\uc815 \ud64d\uae38\ub3d9: \uc0ac\ub77c\uc9c4 \ub9c8\uc744\u300b\uc774 \uac1c\ubd09\ud588\ub2e4. \uadf8\ub9ac\uace0 2017\ub144 2\uc6d4 3\uc77c\u300a\ub0b4\uc77c \uadf8\ub300\uc640\u300b\uc5d0\uc11c \uc720\uc18c\uc900 \uc5ed\ud560\uc744 \ub9e1\uc558\uace0, \uadf8 \ud574 6\uc6d4 28\uc77c \uc601\ud654\u300a\ubc15\uc5f4\u300b\ud070 \ud765\ud589\uc744 \uc774\uc5b4\ub098\uac14\ub2e4. 9\uc6d4 21\uc77c \uc601\ud654\u300a\uc544\uc774 \uce94 \uc2a4\ud53c\ud06c\u300b\uc5d0\uc11c \ubc15\ubbfc\uc7ac \uc5ed\uc744 \ub9e1\uc558\ub2e4."} {"question": "2015\ub144 \ub370\ubdd4 \uc1fc\ucf00\uc774\uc2a4 \ubb34\ub300\ub97c \uac1c\ucd5c\ud55c \uacf3\uc740?", "paragraph": "2015\ub144 10\uc6d4 19\uc77c \uc790\uc815 \uc74c\uc6d0\uacfc \ubba4\uc9c1 \ube44\ub514\uc624\ub97c \ub3d9\uc2dc \uacf5\uac1c\ud558\uc600\uace0, 20\uc77c \uccab \ubc88\uc9f8 EP \uc568\ubc94 \u300aTHE STORY BEGINS\u300b\uc758 \ubc1c\ub9e4\uc640 \ub3d9\uc2dc\uc5d0 \uc545\uc2a4\ucf54\ub9ac\uc544 \uc5d0\uc11c \ub370\ubdd4 \uc1fc\ucf00\uc774\uc2a4 \ubb34\ub300\ub97c \uac1c\ucd5c\ud558\uc600\ub2e4. \ud0c0\uc774\ud2c0 \uace1\uc740 \"OOH-AHH\ud558\uac8c\"\uc774\ub2e4. \ub370\ubdd4 1\uc8fc\uc77c \ub9cc\uc778 27\uc77c\uc5d0\ub294 \ub354 \uc1fc 1\uc704 \ud6c4\ubcf4\uc5d0 \uc62c\ub77c 2\uc704\ub97c \ucc28\uc9c0\ud558\uc600\uace0, 29\uc77c \uc5e0\uce74\uc6b4\ud2b8\ub2e4\uc6b4\uc5d0\uc11c\ub3c4 1\uc704 \ud6c4\ubcf4\uc5d0 \uc62c\ub77c 2\uc704\ub97c \ucc28\uc9c0\ud558\uc600\ub2e4. 2016\ub144 2\uc6d4\ub2ec \uae4c\uc9c0\ub3c4 \uc74c\uc6d0 \ucc28\ud2b8 \uc0c1\uc704\uad8c\uc744 \uc720\uc9c0\ud558\ub294 \uc800\ub825\uc744 \ubcf4\uc5ec\uc8fc\uc5c8\ub2e4. \ud65c\ub3d9 \uc885\ub8cc 3\ub2ec \ud6c4\uc5d0 \ub2e4\uc2dc \uc778\uae30\uac00\uc694\uc5d0\uc11c 1\uc704 \ud6c4\ubcf4\uc5d0 \uc62c\ub77c 2\uc704\ub97c \ucc28\uc9c0\ud558\uc600\ub2e4. 2016\ub144 11\uc6d4 \uae30\uc900 \uc57d 90,000\uc7a5\uc758 \uc74c\ubc18 \ud310\ub9e4\ub7c9\uc744 \uae30\ub85d \ud588\uc73c\uba70, 2016\ub144 \uc2e0\uc778 \uc544\uc774\ub3cc \ub370\ubdd4 \uc74c\ubc18 \ucd08\ub3d9 \uc74c\ubc18 \ud310\ub9e4\ub7c9 1\uc704\ub97c \uae30\ub85d\ud558\uc600\ub2e4. \"OOH-AHH\ud558\uac8c (\uc6b0\uc544\ud558\uac8c)\"\ub294 2015\ub144 \uc544\uc774\ub3cc \ub370\ubdd4 \uace1 \uc911 \uac00\uc7a5 \uc784\ud329\ud2b8 \uc788\ub294 \uace1\uc73c\ub85c \uaf3d\ud614\ub2e4.", "answer": "\uc545\uc2a4\ucf54\ub9ac\uc544", "sentence": "2015\ub144 10\uc6d4 19\uc77c \uc790\uc815 \uc74c\uc6d0\uacfc \ubba4\uc9c1 \ube44\ub514\uc624\ub97c \ub3d9\uc2dc \uacf5\uac1c\ud558\uc600\uace0, 20\uc77c \uccab \ubc88\uc9f8 EP \uc568\ubc94 \u300aTHE STORY BEGINS\u300b\uc758 \ubc1c\ub9e4\uc640 \ub3d9\uc2dc\uc5d0 \uc545\uc2a4\ucf54\ub9ac\uc544 \uc5d0\uc11c \ub370\ubdd4 \uc1fc\ucf00\uc774\uc2a4 \ubb34\ub300\ub97c \uac1c\ucd5c\ud558\uc600\ub2e4.", "paragraph_sentence": " 2015\ub144 10\uc6d4 19\uc77c \uc790\uc815 \uc74c\uc6d0\uacfc \ubba4\uc9c1 \ube44\ub514\uc624\ub97c \ub3d9\uc2dc \uacf5\uac1c\ud558\uc600\uace0, 20\uc77c \uccab \ubc88\uc9f8 EP \uc568\ubc94 \u300aTHE STORY BEGINS\u300b\uc758 \ubc1c\ub9e4\uc640 \ub3d9\uc2dc\uc5d0 \uc545\uc2a4\ucf54\ub9ac\uc544 \uc5d0\uc11c \ub370\ubdd4 \uc1fc\ucf00\uc774\uc2a4 \ubb34\ub300\ub97c \uac1c\ucd5c\ud558\uc600\ub2e4. \ud0c0\uc774\ud2c0 \uace1\uc740 \"OOH-AHH\ud558\uac8c\"\uc774\ub2e4. \ub370\ubdd4 1\uc8fc\uc77c \ub9cc\uc778 27\uc77c\uc5d0\ub294 \ub354 \uc1fc 1\uc704 \ud6c4\ubcf4\uc5d0 \uc62c\ub77c 2\uc704\ub97c \ucc28\uc9c0\ud558\uc600\uace0, 29\uc77c \uc5e0\uce74\uc6b4\ud2b8\ub2e4\uc6b4\uc5d0\uc11c\ub3c4 1\uc704 \ud6c4\ubcf4\uc5d0 \uc62c\ub77c 2\uc704\ub97c \ucc28\uc9c0\ud558\uc600\ub2e4. 2016\ub144 2\uc6d4\ub2ec \uae4c\uc9c0\ub3c4 \uc74c\uc6d0 \ucc28\ud2b8 \uc0c1\uc704\uad8c\uc744 \uc720\uc9c0\ud558\ub294 \uc800\ub825\uc744 \ubcf4\uc5ec\uc8fc\uc5c8\ub2e4. \ud65c\ub3d9 \uc885\ub8cc 3\ub2ec \ud6c4\uc5d0 \ub2e4\uc2dc \uc778\uae30\uac00\uc694\uc5d0\uc11c 1\uc704 \ud6c4\ubcf4\uc5d0 \uc62c\ub77c 2\uc704\ub97c \ucc28\uc9c0\ud558\uc600\ub2e4. 2016\ub144 11\uc6d4 \uae30\uc900 \uc57d 90,000\uc7a5\uc758 \uc74c\ubc18 \ud310\ub9e4\ub7c9\uc744 \uae30\ub85d \ud588\uc73c\uba70, 2016\ub144 \uc2e0\uc778 \uc544\uc774\ub3cc \ub370\ubdd4 \uc74c\ubc18 \ucd08\ub3d9 \uc74c\ubc18 \ud310\ub9e4\ub7c9 1\uc704\ub97c \uae30\ub85d\ud558\uc600\ub2e4. \"OOH-AHH\ud558\uac8c (\uc6b0\uc544\ud558\uac8c)\"\ub294 2015\ub144 \uc544\uc774\ub3cc \ub370\ubdd4 \uace1 \uc911 \uac00\uc7a5 \uc784\ud329\ud2b8 \uc788\ub294 \uace1\uc73c\ub85c \uaf3d\ud614\ub2e4.", "paragraph_answer": "2015\ub144 10\uc6d4 19\uc77c \uc790\uc815 \uc74c\uc6d0\uacfc \ubba4\uc9c1 \ube44\ub514\uc624\ub97c \ub3d9\uc2dc \uacf5\uac1c\ud558\uc600\uace0, 20\uc77c \uccab \ubc88\uc9f8 EP \uc568\ubc94 \u300aTHE STORY BEGINS\u300b\uc758 \ubc1c\ub9e4\uc640 \ub3d9\uc2dc\uc5d0 \uc545\uc2a4\ucf54\ub9ac\uc544 \uc5d0\uc11c \ub370\ubdd4 \uc1fc\ucf00\uc774\uc2a4 \ubb34\ub300\ub97c \uac1c\ucd5c\ud558\uc600\ub2e4. \ud0c0\uc774\ud2c0 \uace1\uc740 \"OOH-AHH\ud558\uac8c\"\uc774\ub2e4. \ub370\ubdd4 1\uc8fc\uc77c \ub9cc\uc778 27\uc77c\uc5d0\ub294 \ub354 \uc1fc 1\uc704 \ud6c4\ubcf4\uc5d0 \uc62c\ub77c 2\uc704\ub97c \ucc28\uc9c0\ud558\uc600\uace0, 29\uc77c \uc5e0\uce74\uc6b4\ud2b8\ub2e4\uc6b4\uc5d0\uc11c\ub3c4 1\uc704 \ud6c4\ubcf4\uc5d0 \uc62c\ub77c 2\uc704\ub97c \ucc28\uc9c0\ud558\uc600\ub2e4. 2016\ub144 2\uc6d4\ub2ec \uae4c\uc9c0\ub3c4 \uc74c\uc6d0 \ucc28\ud2b8 \uc0c1\uc704\uad8c\uc744 \uc720\uc9c0\ud558\ub294 \uc800\ub825\uc744 \ubcf4\uc5ec\uc8fc\uc5c8\ub2e4. \ud65c\ub3d9 \uc885\ub8cc 3\ub2ec \ud6c4\uc5d0 \ub2e4\uc2dc \uc778\uae30\uac00\uc694\uc5d0\uc11c 1\uc704 \ud6c4\ubcf4\uc5d0 \uc62c\ub77c 2\uc704\ub97c \ucc28\uc9c0\ud558\uc600\ub2e4. 2016\ub144 11\uc6d4 \uae30\uc900 \uc57d 90,000\uc7a5\uc758 \uc74c\ubc18 \ud310\ub9e4\ub7c9\uc744 \uae30\ub85d \ud588\uc73c\uba70, 2016\ub144 \uc2e0\uc778 \uc544\uc774\ub3cc \ub370\ubdd4 \uc74c\ubc18 \ucd08\ub3d9 \uc74c\ubc18 \ud310\ub9e4\ub7c9 1\uc704\ub97c \uae30\ub85d\ud558\uc600\ub2e4. \"OOH-AHH\ud558\uac8c (\uc6b0\uc544\ud558\uac8c)\"\ub294 2015\ub144 \uc544\uc774\ub3cc \ub370\ubdd4 \uace1 \uc911 \uac00\uc7a5 \uc784\ud329\ud2b8 \uc788\ub294 \uace1\uc73c\ub85c \uaf3d\ud614\ub2e4.", "sentence_answer": "2015\ub144 10\uc6d4 19\uc77c \uc790\uc815 \uc74c\uc6d0\uacfc \ubba4\uc9c1 \ube44\ub514\uc624\ub97c \ub3d9\uc2dc \uacf5\uac1c\ud558\uc600\uace0, 20\uc77c \uccab \ubc88\uc9f8 EP \uc568\ubc94 \u300aTHE STORY BEGINS\u300b\uc758 \ubc1c\ub9e4\uc640 \ub3d9\uc2dc\uc5d0 \uc545\uc2a4\ucf54\ub9ac\uc544 \uc5d0\uc11c \ub370\ubdd4 \uc1fc\ucf00\uc774\uc2a4 \ubb34\ub300\ub97c \uac1c\ucd5c\ud558\uc600\ub2e4.", "paragraph_id": "6566343-1-0", "paragraph_question": "question: 2015\ub144 \ub370\ubdd4 \uc1fc\ucf00\uc774\uc2a4 \ubb34\ub300\ub97c \uac1c\ucd5c\ud55c \uacf3\uc740?, context: 2015\ub144 10\uc6d4 19\uc77c \uc790\uc815 \uc74c\uc6d0\uacfc \ubba4\uc9c1 \ube44\ub514\uc624\ub97c \ub3d9\uc2dc \uacf5\uac1c\ud558\uc600\uace0, 20\uc77c \uccab \ubc88\uc9f8 EP \uc568\ubc94 \u300aTHE STORY BEGINS\u300b\uc758 \ubc1c\ub9e4\uc640 \ub3d9\uc2dc\uc5d0 \uc545\uc2a4\ucf54\ub9ac\uc544 \uc5d0\uc11c \ub370\ubdd4 \uc1fc\ucf00\uc774\uc2a4 \ubb34\ub300\ub97c \uac1c\ucd5c\ud558\uc600\ub2e4. \ud0c0\uc774\ud2c0 \uace1\uc740 \"OOH-AHH\ud558\uac8c\"\uc774\ub2e4. \ub370\ubdd4 1\uc8fc\uc77c \ub9cc\uc778 27\uc77c\uc5d0\ub294 \ub354 \uc1fc 1\uc704 \ud6c4\ubcf4\uc5d0 \uc62c\ub77c 2\uc704\ub97c \ucc28\uc9c0\ud558\uc600\uace0, 29\uc77c \uc5e0\uce74\uc6b4\ud2b8\ub2e4\uc6b4\uc5d0\uc11c\ub3c4 1\uc704 \ud6c4\ubcf4\uc5d0 \uc62c\ub77c 2\uc704\ub97c \ucc28\uc9c0\ud558\uc600\ub2e4. 2016\ub144 2\uc6d4\ub2ec \uae4c\uc9c0\ub3c4 \uc74c\uc6d0 \ucc28\ud2b8 \uc0c1\uc704\uad8c\uc744 \uc720\uc9c0\ud558\ub294 \uc800\ub825\uc744 \ubcf4\uc5ec\uc8fc\uc5c8\ub2e4. \ud65c\ub3d9 \uc885\ub8cc 3\ub2ec \ud6c4\uc5d0 \ub2e4\uc2dc \uc778\uae30\uac00\uc694\uc5d0\uc11c 1\uc704 \ud6c4\ubcf4\uc5d0 \uc62c\ub77c 2\uc704\ub97c \ucc28\uc9c0\ud558\uc600\ub2e4. 2016\ub144 11\uc6d4 \uae30\uc900 \uc57d 90,000\uc7a5\uc758 \uc74c\ubc18 \ud310\ub9e4\ub7c9\uc744 \uae30\ub85d \ud588\uc73c\uba70, 2016\ub144 \uc2e0\uc778 \uc544\uc774\ub3cc \ub370\ubdd4 \uc74c\ubc18 \ucd08\ub3d9 \uc74c\ubc18 \ud310\ub9e4\ub7c9 1\uc704\ub97c \uae30\ub85d\ud558\uc600\ub2e4. \"OOH-AHH\ud558\uac8c (\uc6b0\uc544\ud558\uac8c)\"\ub294 2015\ub144 \uc544\uc774\ub3cc \ub370\ubdd4 \uace1 \uc911 \uac00\uc7a5 \uc784\ud329\ud2b8 \uc788\ub294 \uace1\uc73c\ub85c \uaf3d\ud614\ub2e4."} {"question": "5.18 \ub2f9\uc2dc \uc5b4\ub290 \ub098\ub77c\uac00 \ub300\ud55c\ubbfc\uad6d\uc758 \ubbfc\uc8fc\ud654\ub97c \uc9c0\uc9c0\ud588\ub098\uc694?", "paragraph": "5\u00b718 \ub2f9\uc2dc \ubbf8\uad6d\uc740 \ub300\ud55c\ubbfc\uad6d\uc758 \ubbfc\uc8fc\ud654\ub97c \uc9c0\uc9c0\ud588\uc73c\ub098, \uad70\ubd80\ub97c \uc7a5\uc545\ud55c \uc804\ub450\ud658\uc744 \uacac\uc81c\ud558\ub294 \ub370 \ud55c\uacc4\uac00 \uc788\uc5c8\ub2e4. \ubbf8\uad6d\uc774 5\u00b718 \ub2f9\uc2dc \uad11\uc8fc\uc5d0\uc11c \uc77c\uc5b4\ub09c \uc77c\ub4e4\uc758 \uc804\ubaa8\ub97c \ud30c\uc545\ud588\uc744 \ub54c \uae00\ub77c\uc774\uc2a4\ud2f4 \uc8fc\ud55c\ubbf8\ub300\uc0ac\ub294 \ud2b9\uc804\uc0ac\uc758 \uc9c0\ub098\uce5c \ubc18\uc751\uc774 \ube44\uadf9\uc758 \uc9c1\uc811\uc801 \uc6d0\uc778\uc774\ub77c\uace0 \ud310\ub2e8\ud588\ub2e4. \ubbf8\uad6d \uad00\ub9ac\ub4e4\uc740 \uadf8\uc640\ub294 \ubc18\ub300\ub85c \uc815\ubd80\uc758 \ud1b5\uc81c\ub97c \ubc1b\ub294 \uad11\uc8fc\uc758 \ub77c\ub514\uc624 \ubc29\uc1a1\uc774 \"\ubbf8\uad6d \uce21\uc774 \ud2b9\uc804\uc0ac\uc758 \uad11\uc8fc \ud30c\uacac\uc744 \uc2b9\uc778\ud588\ub2e4\"\uace0 \ubcf4\ub3c4\ud558\uace0 \uc788\uc74c\uc744 \uc54c\uc558\ub2e4. \uae00\ub77c\uc774\uc2a4\ud2f4 \ub300\uc0ac\ub294 \uc774 \uc5ed\uc815\ubcf4 \ud589\uc704\ub97c \ub300\ud55c\ubbfc\uad6d \uc815\ubd80\uc5d0 \ud56d\uc758\ud558\uace0, \uc774\ub97c \uc815\uc2dd\uc73c\ub85c \ucde8\uc18c\ud560 \uac83\uc744 \uc694\uad6c\ud588\ub2e4. \uadf8\ub7ec\ub098 \uc2e0\uad70\ubd80\ub294 \uc774\ub97c \ucde8\uc18c\ud558\uc9c0 \uc54a\uc558\ub2e4. \ub610\ud55c \ubcf4\uc548\uc0ac\uc758 \uc870\uc885\uc744 \ubc1b\ub294 \ub300\ud55c\ubbfc\uad6d \uc5b8\ub860\uc740 \ubbf8\uad6d \uad00\ub9ac\uc758 \ud56d\uc758\uc5d0\ub3c4 \ubd88\uad6c\ud558\uace0 \"\ubbf8\uad6d\uc5d0\uc11c \uc804\ub450\ud658\uc744 \uc2b9\uc778\ud588\ub2e4\"\ub77c\ub294 \ubcf4\ub3c4\ub97c \uc3df\uc544\ub0b4\uc5c8\ub2e4. \uc774\ub7ec\ud55c \uc2e0\uad70\ubd80\uc5d0 \uc758\ud55c \uc65c\uace1 \ubcf4\ub3c4\ub294 \ub300\ud55c\ubbfc\uad6d \ub0b4 \ubc18\ubbf8 \uac10\uc815\uc774 \ud655\uc0b0\ub418\ub294 \ub370 \uae30\uc5ec\ud588\ub2e4.", "answer": "\ubbf8\uad6d", "sentence": "5\u00b718 \ub2f9\uc2dc \ubbf8\uad6d \uc740 \ub300\ud55c\ubbfc\uad6d\uc758 \ubbfc\uc8fc\ud654\ub97c \uc9c0\uc9c0\ud588\uc73c\ub098, \uad70\ubd80\ub97c \uc7a5\uc545\ud55c \uc804\ub450\ud658\uc744 \uacac\uc81c\ud558\ub294 \ub370 \ud55c\uacc4\uac00 \uc788\uc5c8\ub2e4.", "paragraph_sentence": " 5\u00b718 \ub2f9\uc2dc \ubbf8\uad6d \uc740 \ub300\ud55c\ubbfc\uad6d\uc758 \ubbfc\uc8fc\ud654\ub97c \uc9c0\uc9c0\ud588\uc73c\ub098, \uad70\ubd80\ub97c \uc7a5\uc545\ud55c \uc804\ub450\ud658\uc744 \uacac\uc81c\ud558\ub294 \ub370 \ud55c\uacc4\uac00 \uc788\uc5c8\ub2e4. \ubbf8\uad6d\uc774 5\u00b718 \ub2f9\uc2dc \uad11\uc8fc\uc5d0\uc11c \uc77c\uc5b4\ub09c \uc77c\ub4e4\uc758 \uc804\ubaa8\ub97c \ud30c\uc545\ud588\uc744 \ub54c \uae00\ub77c\uc774\uc2a4\ud2f4 \uc8fc\ud55c\ubbf8\ub300\uc0ac\ub294 \ud2b9\uc804\uc0ac\uc758 \uc9c0\ub098\uce5c \ubc18\uc751\uc774 \ube44\uadf9\uc758 \uc9c1\uc811\uc801 \uc6d0\uc778\uc774\ub77c\uace0 \ud310\ub2e8\ud588\ub2e4. \ubbf8\uad6d \uad00\ub9ac\ub4e4\uc740 \uadf8\uc640\ub294 \ubc18\ub300\ub85c \uc815\ubd80\uc758 \ud1b5\uc81c\ub97c \ubc1b\ub294 \uad11\uc8fc\uc758 \ub77c\ub514\uc624 \ubc29\uc1a1\uc774 \"\ubbf8\uad6d \uce21\uc774 \ud2b9\uc804\uc0ac\uc758 \uad11\uc8fc \ud30c\uacac\uc744 \uc2b9\uc778\ud588\ub2e4\"\uace0 \ubcf4\ub3c4\ud558\uace0 \uc788\uc74c\uc744 \uc54c\uc558\ub2e4. \uae00\ub77c\uc774\uc2a4\ud2f4 \ub300\uc0ac\ub294 \uc774 \uc5ed\uc815\ubcf4 \ud589\uc704\ub97c \ub300\ud55c\ubbfc\uad6d \uc815\ubd80\uc5d0 \ud56d\uc758\ud558\uace0, \uc774\ub97c \uc815\uc2dd\uc73c\ub85c \ucde8\uc18c\ud560 \uac83\uc744 \uc694\uad6c\ud588\ub2e4. \uadf8\ub7ec\ub098 \uc2e0\uad70\ubd80\ub294 \uc774\ub97c \ucde8\uc18c\ud558\uc9c0 \uc54a\uc558\ub2e4. \ub610\ud55c \ubcf4\uc548\uc0ac\uc758 \uc870\uc885\uc744 \ubc1b\ub294 \ub300\ud55c\ubbfc\uad6d \uc5b8\ub860\uc740 \ubbf8\uad6d \uad00\ub9ac\uc758 \ud56d\uc758\uc5d0\ub3c4 \ubd88\uad6c\ud558\uace0 \"\ubbf8\uad6d\uc5d0\uc11c \uc804\ub450\ud658\uc744 \uc2b9\uc778\ud588\ub2e4\"\ub77c\ub294 \ubcf4\ub3c4\ub97c \uc3df\uc544\ub0b4\uc5c8\ub2e4. \uc774\ub7ec\ud55c \uc2e0\uad70\ubd80\uc5d0 \uc758\ud55c \uc65c\uace1 \ubcf4\ub3c4\ub294 \ub300\ud55c\ubbfc\uad6d \ub0b4 \ubc18\ubbf8 \uac10\uc815\uc774 \ud655\uc0b0\ub418\ub294 \ub370 \uae30\uc5ec\ud588\ub2e4.", "paragraph_answer": "5\u00b718 \ub2f9\uc2dc \ubbf8\uad6d \uc740 \ub300\ud55c\ubbfc\uad6d\uc758 \ubbfc\uc8fc\ud654\ub97c \uc9c0\uc9c0\ud588\uc73c\ub098, \uad70\ubd80\ub97c \uc7a5\uc545\ud55c \uc804\ub450\ud658\uc744 \uacac\uc81c\ud558\ub294 \ub370 \ud55c\uacc4\uac00 \uc788\uc5c8\ub2e4. \ubbf8\uad6d\uc774 5\u00b718 \ub2f9\uc2dc \uad11\uc8fc\uc5d0\uc11c \uc77c\uc5b4\ub09c \uc77c\ub4e4\uc758 \uc804\ubaa8\ub97c \ud30c\uc545\ud588\uc744 \ub54c \uae00\ub77c\uc774\uc2a4\ud2f4 \uc8fc\ud55c\ubbf8\ub300\uc0ac\ub294 \ud2b9\uc804\uc0ac\uc758 \uc9c0\ub098\uce5c \ubc18\uc751\uc774 \ube44\uadf9\uc758 \uc9c1\uc811\uc801 \uc6d0\uc778\uc774\ub77c\uace0 \ud310\ub2e8\ud588\ub2e4. \ubbf8\uad6d \uad00\ub9ac\ub4e4\uc740 \uadf8\uc640\ub294 \ubc18\ub300\ub85c \uc815\ubd80\uc758 \ud1b5\uc81c\ub97c \ubc1b\ub294 \uad11\uc8fc\uc758 \ub77c\ub514\uc624 \ubc29\uc1a1\uc774 \"\ubbf8\uad6d \uce21\uc774 \ud2b9\uc804\uc0ac\uc758 \uad11\uc8fc \ud30c\uacac\uc744 \uc2b9\uc778\ud588\ub2e4\"\uace0 \ubcf4\ub3c4\ud558\uace0 \uc788\uc74c\uc744 \uc54c\uc558\ub2e4. \uae00\ub77c\uc774\uc2a4\ud2f4 \ub300\uc0ac\ub294 \uc774 \uc5ed\uc815\ubcf4 \ud589\uc704\ub97c \ub300\ud55c\ubbfc\uad6d \uc815\ubd80\uc5d0 \ud56d\uc758\ud558\uace0, \uc774\ub97c \uc815\uc2dd\uc73c\ub85c \ucde8\uc18c\ud560 \uac83\uc744 \uc694\uad6c\ud588\ub2e4. \uadf8\ub7ec\ub098 \uc2e0\uad70\ubd80\ub294 \uc774\ub97c \ucde8\uc18c\ud558\uc9c0 \uc54a\uc558\ub2e4. \ub610\ud55c \ubcf4\uc548\uc0ac\uc758 \uc870\uc885\uc744 \ubc1b\ub294 \ub300\ud55c\ubbfc\uad6d \uc5b8\ub860\uc740 \ubbf8\uad6d \uad00\ub9ac\uc758 \ud56d\uc758\uc5d0\ub3c4 \ubd88\uad6c\ud558\uace0 \"\ubbf8\uad6d\uc5d0\uc11c \uc804\ub450\ud658\uc744 \uc2b9\uc778\ud588\ub2e4\"\ub77c\ub294 \ubcf4\ub3c4\ub97c \uc3df\uc544\ub0b4\uc5c8\ub2e4. \uc774\ub7ec\ud55c \uc2e0\uad70\ubd80\uc5d0 \uc758\ud55c \uc65c\uace1 \ubcf4\ub3c4\ub294 \ub300\ud55c\ubbfc\uad6d \ub0b4 \ubc18\ubbf8 \uac10\uc815\uc774 \ud655\uc0b0\ub418\ub294 \ub370 \uae30\uc5ec\ud588\ub2e4.", "sentence_answer": "5\u00b718 \ub2f9\uc2dc \ubbf8\uad6d \uc740 \ub300\ud55c\ubbfc\uad6d\uc758 \ubbfc\uc8fc\ud654\ub97c \uc9c0\uc9c0\ud588\uc73c\ub098, \uad70\ubd80\ub97c \uc7a5\uc545\ud55c \uc804\ub450\ud658\uc744 \uacac\uc81c\ud558\ub294 \ub370 \ud55c\uacc4\uac00 \uc788\uc5c8\ub2e4.", "paragraph_id": "5801882-20-0", "paragraph_question": "question: 5.18 \ub2f9\uc2dc \uc5b4\ub290 \ub098\ub77c\uac00 \ub300\ud55c\ubbfc\uad6d\uc758 \ubbfc\uc8fc\ud654\ub97c \uc9c0\uc9c0\ud588\ub098\uc694?, context: 5\u00b718 \ub2f9\uc2dc \ubbf8\uad6d\uc740 \ub300\ud55c\ubbfc\uad6d\uc758 \ubbfc\uc8fc\ud654\ub97c \uc9c0\uc9c0\ud588\uc73c\ub098, \uad70\ubd80\ub97c \uc7a5\uc545\ud55c \uc804\ub450\ud658\uc744 \uacac\uc81c\ud558\ub294 \ub370 \ud55c\uacc4\uac00 \uc788\uc5c8\ub2e4. \ubbf8\uad6d\uc774 5\u00b718 \ub2f9\uc2dc \uad11\uc8fc\uc5d0\uc11c \uc77c\uc5b4\ub09c \uc77c\ub4e4\uc758 \uc804\ubaa8\ub97c \ud30c\uc545\ud588\uc744 \ub54c \uae00\ub77c\uc774\uc2a4\ud2f4 \uc8fc\ud55c\ubbf8\ub300\uc0ac\ub294 \ud2b9\uc804\uc0ac\uc758 \uc9c0\ub098\uce5c \ubc18\uc751\uc774 \ube44\uadf9\uc758 \uc9c1\uc811\uc801 \uc6d0\uc778\uc774\ub77c\uace0 \ud310\ub2e8\ud588\ub2e4. \ubbf8\uad6d \uad00\ub9ac\ub4e4\uc740 \uadf8\uc640\ub294 \ubc18\ub300\ub85c \uc815\ubd80\uc758 \ud1b5\uc81c\ub97c \ubc1b\ub294 \uad11\uc8fc\uc758 \ub77c\ub514\uc624 \ubc29\uc1a1\uc774 \"\ubbf8\uad6d \uce21\uc774 \ud2b9\uc804\uc0ac\uc758 \uad11\uc8fc \ud30c\uacac\uc744 \uc2b9\uc778\ud588\ub2e4\"\uace0 \ubcf4\ub3c4\ud558\uace0 \uc788\uc74c\uc744 \uc54c\uc558\ub2e4. \uae00\ub77c\uc774\uc2a4\ud2f4 \ub300\uc0ac\ub294 \uc774 \uc5ed\uc815\ubcf4 \ud589\uc704\ub97c \ub300\ud55c\ubbfc\uad6d \uc815\ubd80\uc5d0 \ud56d\uc758\ud558\uace0, \uc774\ub97c \uc815\uc2dd\uc73c\ub85c \ucde8\uc18c\ud560 \uac83\uc744 \uc694\uad6c\ud588\ub2e4. \uadf8\ub7ec\ub098 \uc2e0\uad70\ubd80\ub294 \uc774\ub97c \ucde8\uc18c\ud558\uc9c0 \uc54a\uc558\ub2e4. \ub610\ud55c \ubcf4\uc548\uc0ac\uc758 \uc870\uc885\uc744 \ubc1b\ub294 \ub300\ud55c\ubbfc\uad6d \uc5b8\ub860\uc740 \ubbf8\uad6d \uad00\ub9ac\uc758 \ud56d\uc758\uc5d0\ub3c4 \ubd88\uad6c\ud558\uace0 \"\ubbf8\uad6d\uc5d0\uc11c \uc804\ub450\ud658\uc744 \uc2b9\uc778\ud588\ub2e4\"\ub77c\ub294 \ubcf4\ub3c4\ub97c \uc3df\uc544\ub0b4\uc5c8\ub2e4. \uc774\ub7ec\ud55c \uc2e0\uad70\ubd80\uc5d0 \uc758\ud55c \uc65c\uace1 \ubcf4\ub3c4\ub294 \ub300\ud55c\ubbfc\uad6d \ub0b4 \ubc18\ubbf8 \uac10\uc815\uc774 \ud655\uc0b0\ub418\ub294 \ub370 \uae30\uc5ec\ud588\ub2e4."} {"question": "\uc138\uc774\ubd80 \ucca0\ub3c4\uc758 \ucc3d\uc5c5\uc790 \uc774\ub984\uc740?", "paragraph": "\ub610\ud55c \uc624\ud14c \uc0ac\ucca0 \uc911\uc5d0\uc11c\ub294 \uc720\uc77c\ud558\uac8c \ubc31\ud654\uc810\ub4f1 \uc720\ud1b5\uc5c5\uc744 \ud68c\uc0ac \uc18c\uc18d\uc5d0 \ub450\uace0 \uc788\uc9c0 \uc54a\ub2e4. \uc774\uac83\uc740, \ucc3d\uc5c5\uc790\uc778 \uc4f0\uc4f0\ubbf8 \uc57c\uc2a4\uc9c0\ub85c \uc0ac\ud6c4\uc758 \ud6c4\uacc4 \uc2f8\uc6c0\uc73c\ub85c \uc778\ud558\uc5ec, \uc138\uc774\ubd80\ucca0\ub3c4 \uadf8\ub8f9(\ud604 \uc138\uc774\ubd80 \uadf8\ub8f9)\uacfc \uc138\uc774\ubd80 \uc720\ud1b5 \uadf8\ub8f9(\uc138\uc874 \uadf8\ub8f9, 2000\ub144\ub300\uc5d0 \uc2e4\uc9c8\uc801\uc73c\ub85c \ud574\uccb4)\uc73c\ub85c \ubd84\uc5f4\ub418\uc5c8\uae30 \ub54c\ubb38\uc774\ub2e4. \ub2e4\ub9cc, 2003\ub144\uc758 \ubc00\ub808\ub2c8\uc5c4 \ub9ac\ud14c\uc774\ub9c1\uc758 \uc640\ub2e4 \uc2dc\uac8c\ub8e8 \uc544\ud0a4\ub77c \uc0ac\uc7a5(\ub2f9\uc2dc)\uc774 \uc4f0\uc4f0\ubbf8 \uc694\uc2dc\uc544\ud0a4 \ud68c\uc7a5(\ub2f9\uc2dc)\uc5d0\uac8c \uc81c3\uc0ac \ud560\ub2f9 \uc99d\uc790\ub97c \ud0c0\uc9c4\ud558\uc5ec, \uc138\uc774\ubd80 \ucca0\ub3c4\uac00 \ucd9c\uc790\ub97c \ub9e1\uc558\ub2e4. 2006\ub144\uc758 \uc138\uc774\ubd80 \ud640\ub529\uc2a4 \uc124\ub9bd\uc5d0 \uc218\ubc18\ud558\ub294 \uace0\ucfe0\ub3c4(\u30b3\u30af\u30c9)\uc758 \uc81c\uc0bc\uc790 \ud560\ub2f9 \uc99d\uc790\uc2dc\uc5d0\ub294 \ubc00\ub808\ub2c8\uc5c4\uc774 10\uc5b5\uc5d4 \ucd9c\uc790\ub97c \ub9e1\uc544 \uc138\ube10&\uc544\uc774\u00b7\ud640\ub529\uc2a4\uac00 \ubc00\ub808\ub2c8\uc5c4 \uc8fc\uc2dd\uc744 \ud604\uae08\uc73c\ub85c \uc218\ub9e4\ud560 \ub54c\uae4c\uc9c0\ub294 \uc8fc\uc2dd\uc758 \uade0\ud615 \uc0c1\ud0dc\uc600\ub2e4. \uc138\uc774\ubd80 \ubc31\ud654\uc810\uc758 \uc791\uc740 \uc810\ud3ec\ub85c \ub4f1\uc7a5\ud55c \uc138\uc774\uc720\uac00 \uc138\uc774\ubd80 \uc5f0\uc120\uc5d0 \ub9ce\uc740 \uac83\uc740, \uadf8\ub8f9\uc774 \ubd84\uc5f4\ub418\uae30 \uc804\uc758 \ubc1c\uc804 \uacfc\uc815\uc5d0\uc11c \uc810\ud3ec\uc640 \ud1a0\uc9c0 \uc784\ub300\uac00 \ud604\uc7ac\uae4c\uc9c0 \uacc4\uc18d\ud558\uace0 \uc788\uae30 \ub54c\ubb38\uc774\ub2e4. \uadf8\ub7ec\ub098, \uc138\uc774\uc720\uc640\uc758 \uc5f0\uacb0\uc740 \uc544\uc9c1\ub3c4 \uae4a\uc73c\uba70 \uc138\uc774\ubd80 \ucca0\ub3c4\uac00 \uac1c\ubc1c\ud558\ub294 \ub274\ud0c0\uc6b4\uc774\ub098 \uc5ed \uac1c\ub7c9 \uacf5\uc0ac \ud6c4\uc5d0 \uc138\uc774\uc720\uc758 \uc0c8\ub85c\uc6b4 \uc810\ud3ec\uac00 \uac1c\uc5c5\ud558\ub294 \uacbd\uc6b0\uac00 \ub9ce\ub2e4. \ub610\ud55c, \uc138\uc774\uc720\uc758 \uc18c\ub9e4 \uc0ac\uc5c5\uc73c\ub85c\uc11c \ubc1c\uc871\ud55c \ud328\ubc00\ub9ac \ub9c8\ud2b8\ub294 \uc5ed \uad6c\ub0b4 \ud3b8\uc758\uc810\uc778 TOMONY\uc758 \uc804\uac1c\ub85c \uc81c\ud734\ud558\uace0 \uc788\ub2e4.", "answer": "\uc4f0\uc4f0\ubbf8 \uc57c\uc2a4\uc9c0", "sentence": "\uc774\uac83\uc740, \ucc3d\uc5c5\uc790\uc778 \uc4f0\uc4f0\ubbf8 \uc57c\uc2a4\uc9c0 \ub85c \uc0ac\ud6c4\uc758 \ud6c4\uacc4 \uc2f8\uc6c0\uc73c\ub85c \uc778\ud558\uc5ec, \uc138\uc774\ubd80\ucca0\ub3c4 \uadf8\ub8f9(\ud604 \uc138\uc774\ubd80 \uadf8\ub8f9)\uacfc \uc138\uc774\ubd80 \uc720\ud1b5 \uadf8\ub8f9(\uc138\uc874 \uadf8\ub8f9, 2000\ub144\ub300\uc5d0 \uc2e4\uc9c8\uc801\uc73c\ub85c \ud574\uccb4)\uc73c\ub85c \ubd84\uc5f4\ub418\uc5c8\uae30 \ub54c\ubb38\uc774\ub2e4.", "paragraph_sentence": "\ub610\ud55c \uc624\ud14c \uc0ac\ucca0 \uc911\uc5d0\uc11c\ub294 \uc720\uc77c\ud558\uac8c \ubc31\ud654\uc810\ub4f1 \uc720\ud1b5\uc5c5\uc744 \ud68c\uc0ac \uc18c\uc18d\uc5d0 \ub450\uace0 \uc788\uc9c0 \uc54a\ub2e4. \uc774\uac83\uc740, \ucc3d\uc5c5\uc790\uc778 \uc4f0\uc4f0\ubbf8 \uc57c\uc2a4\uc9c0 \ub85c \uc0ac\ud6c4\uc758 \ud6c4\uacc4 \uc2f8\uc6c0\uc73c\ub85c \uc778\ud558\uc5ec, \uc138\uc774\ubd80\ucca0\ub3c4 \uadf8\ub8f9(\ud604 \uc138\uc774\ubd80 \uadf8\ub8f9)\uacfc \uc138\uc774\ubd80 \uc720\ud1b5 \uadf8\ub8f9(\uc138\uc874 \uadf8\ub8f9, 2000\ub144\ub300\uc5d0 \uc2e4\uc9c8\uc801\uc73c\ub85c \ud574\uccb4)\uc73c\ub85c \ubd84\uc5f4\ub418\uc5c8\uae30 \ub54c\ubb38\uc774\ub2e4. \ub2e4\ub9cc, 2003\ub144\uc758 \ubc00\ub808\ub2c8\uc5c4 \ub9ac\ud14c\uc774\ub9c1\uc758 \uc640\ub2e4 \uc2dc\uac8c\ub8e8 \uc544\ud0a4\ub77c \uc0ac\uc7a5(\ub2f9\uc2dc)\uc774 \uc4f0\uc4f0\ubbf8 \uc694\uc2dc\uc544\ud0a4 \ud68c\uc7a5(\ub2f9\uc2dc)\uc5d0\uac8c \uc81c3\uc0ac \ud560\ub2f9 \uc99d\uc790\ub97c \ud0c0\uc9c4\ud558\uc5ec, \uc138\uc774\ubd80 \ucca0\ub3c4\uac00 \ucd9c\uc790\ub97c \ub9e1\uc558\ub2e4. 2006\ub144\uc758 \uc138\uc774\ubd80 \ud640\ub529\uc2a4 \uc124\ub9bd\uc5d0 \uc218\ubc18\ud558\ub294 \uace0\ucfe0\ub3c4(\u30b3\u30af\u30c9)\uc758 \uc81c\uc0bc\uc790 \ud560\ub2f9 \uc99d\uc790\uc2dc\uc5d0\ub294 \ubc00\ub808\ub2c8\uc5c4\uc774 10\uc5b5\uc5d4 \ucd9c\uc790\ub97c \ub9e1\uc544 \uc138\ube10&\uc544\uc774\u00b7\ud640\ub529\uc2a4\uac00 \ubc00\ub808\ub2c8\uc5c4 \uc8fc\uc2dd\uc744 \ud604\uae08\uc73c\ub85c \uc218\ub9e4\ud560 \ub54c\uae4c\uc9c0\ub294 \uc8fc\uc2dd\uc758 \uade0\ud615 \uc0c1\ud0dc\uc600\ub2e4. \uc138\uc774\ubd80 \ubc31\ud654\uc810\uc758 \uc791\uc740 \uc810\ud3ec\ub85c \ub4f1\uc7a5\ud55c \uc138\uc774\uc720\uac00 \uc138\uc774\ubd80 \uc5f0\uc120\uc5d0 \ub9ce\uc740 \uac83\uc740, \uadf8\ub8f9\uc774 \ubd84\uc5f4\ub418\uae30 \uc804\uc758 \ubc1c\uc804 \uacfc\uc815\uc5d0\uc11c \uc810\ud3ec\uc640 \ud1a0\uc9c0 \uc784\ub300\uac00 \ud604\uc7ac\uae4c\uc9c0 \uacc4\uc18d\ud558\uace0 \uc788\uae30 \ub54c\ubb38\uc774\ub2e4. \uadf8\ub7ec\ub098, \uc138\uc774\uc720\uc640\uc758 \uc5f0\uacb0\uc740 \uc544\uc9c1\ub3c4 \uae4a\uc73c\uba70 \uc138\uc774\ubd80 \ucca0\ub3c4\uac00 \uac1c\ubc1c\ud558\ub294 \ub274\ud0c0\uc6b4\uc774\ub098 \uc5ed \uac1c\ub7c9 \uacf5\uc0ac \ud6c4\uc5d0 \uc138\uc774\uc720\uc758 \uc0c8\ub85c\uc6b4 \uc810\ud3ec\uac00 \uac1c\uc5c5\ud558\ub294 \uacbd\uc6b0\uac00 \ub9ce\ub2e4. \ub610\ud55c, \uc138\uc774\uc720\uc758 \uc18c\ub9e4 \uc0ac\uc5c5\uc73c\ub85c\uc11c \ubc1c\uc871\ud55c \ud328\ubc00\ub9ac \ub9c8\ud2b8\ub294 \uc5ed \uad6c\ub0b4 \ud3b8\uc758\uc810\uc778 TOMONY\uc758 \uc804\uac1c\ub85c \uc81c\ud734\ud558\uace0 \uc788\ub2e4.", "paragraph_answer": "\ub610\ud55c \uc624\ud14c \uc0ac\ucca0 \uc911\uc5d0\uc11c\ub294 \uc720\uc77c\ud558\uac8c \ubc31\ud654\uc810\ub4f1 \uc720\ud1b5\uc5c5\uc744 \ud68c\uc0ac \uc18c\uc18d\uc5d0 \ub450\uace0 \uc788\uc9c0 \uc54a\ub2e4. \uc774\uac83\uc740, \ucc3d\uc5c5\uc790\uc778 \uc4f0\uc4f0\ubbf8 \uc57c\uc2a4\uc9c0 \ub85c \uc0ac\ud6c4\uc758 \ud6c4\uacc4 \uc2f8\uc6c0\uc73c\ub85c \uc778\ud558\uc5ec, \uc138\uc774\ubd80\ucca0\ub3c4 \uadf8\ub8f9(\ud604 \uc138\uc774\ubd80 \uadf8\ub8f9)\uacfc \uc138\uc774\ubd80 \uc720\ud1b5 \uadf8\ub8f9(\uc138\uc874 \uadf8\ub8f9, 2000\ub144\ub300\uc5d0 \uc2e4\uc9c8\uc801\uc73c\ub85c \ud574\uccb4)\uc73c\ub85c \ubd84\uc5f4\ub418\uc5c8\uae30 \ub54c\ubb38\uc774\ub2e4. \ub2e4\ub9cc, 2003\ub144\uc758 \ubc00\ub808\ub2c8\uc5c4 \ub9ac\ud14c\uc774\ub9c1\uc758 \uc640\ub2e4 \uc2dc\uac8c\ub8e8 \uc544\ud0a4\ub77c \uc0ac\uc7a5(\ub2f9\uc2dc)\uc774 \uc4f0\uc4f0\ubbf8 \uc694\uc2dc\uc544\ud0a4 \ud68c\uc7a5(\ub2f9\uc2dc)\uc5d0\uac8c \uc81c3\uc0ac \ud560\ub2f9 \uc99d\uc790\ub97c \ud0c0\uc9c4\ud558\uc5ec, \uc138\uc774\ubd80 \ucca0\ub3c4\uac00 \ucd9c\uc790\ub97c \ub9e1\uc558\ub2e4. 2006\ub144\uc758 \uc138\uc774\ubd80 \ud640\ub529\uc2a4 \uc124\ub9bd\uc5d0 \uc218\ubc18\ud558\ub294 \uace0\ucfe0\ub3c4(\u30b3\u30af\u30c9)\uc758 \uc81c\uc0bc\uc790 \ud560\ub2f9 \uc99d\uc790\uc2dc\uc5d0\ub294 \ubc00\ub808\ub2c8\uc5c4\uc774 10\uc5b5\uc5d4 \ucd9c\uc790\ub97c \ub9e1\uc544 \uc138\ube10&\uc544\uc774\u00b7\ud640\ub529\uc2a4\uac00 \ubc00\ub808\ub2c8\uc5c4 \uc8fc\uc2dd\uc744 \ud604\uae08\uc73c\ub85c \uc218\ub9e4\ud560 \ub54c\uae4c\uc9c0\ub294 \uc8fc\uc2dd\uc758 \uade0\ud615 \uc0c1\ud0dc\uc600\ub2e4. \uc138\uc774\ubd80 \ubc31\ud654\uc810\uc758 \uc791\uc740 \uc810\ud3ec\ub85c \ub4f1\uc7a5\ud55c \uc138\uc774\uc720\uac00 \uc138\uc774\ubd80 \uc5f0\uc120\uc5d0 \ub9ce\uc740 \uac83\uc740, \uadf8\ub8f9\uc774 \ubd84\uc5f4\ub418\uae30 \uc804\uc758 \ubc1c\uc804 \uacfc\uc815\uc5d0\uc11c \uc810\ud3ec\uc640 \ud1a0\uc9c0 \uc784\ub300\uac00 \ud604\uc7ac\uae4c\uc9c0 \uacc4\uc18d\ud558\uace0 \uc788\uae30 \ub54c\ubb38\uc774\ub2e4. \uadf8\ub7ec\ub098, \uc138\uc774\uc720\uc640\uc758 \uc5f0\uacb0\uc740 \uc544\uc9c1\ub3c4 \uae4a\uc73c\uba70 \uc138\uc774\ubd80 \ucca0\ub3c4\uac00 \uac1c\ubc1c\ud558\ub294 \ub274\ud0c0\uc6b4\uc774\ub098 \uc5ed \uac1c\ub7c9 \uacf5\uc0ac \ud6c4\uc5d0 \uc138\uc774\uc720\uc758 \uc0c8\ub85c\uc6b4 \uc810\ud3ec\uac00 \uac1c\uc5c5\ud558\ub294 \uacbd\uc6b0\uac00 \ub9ce\ub2e4. \ub610\ud55c, \uc138\uc774\uc720\uc758 \uc18c\ub9e4 \uc0ac\uc5c5\uc73c\ub85c\uc11c \ubc1c\uc871\ud55c \ud328\ubc00\ub9ac \ub9c8\ud2b8\ub294 \uc5ed \uad6c\ub0b4 \ud3b8\uc758\uc810\uc778 TOMONY\uc758 \uc804\uac1c\ub85c \uc81c\ud734\ud558\uace0 \uc788\ub2e4.", "sentence_answer": "\uc774\uac83\uc740, \ucc3d\uc5c5\uc790\uc778 \uc4f0\uc4f0\ubbf8 \uc57c\uc2a4\uc9c0 \ub85c \uc0ac\ud6c4\uc758 \ud6c4\uacc4 \uc2f8\uc6c0\uc73c\ub85c \uc778\ud558\uc5ec, \uc138\uc774\ubd80\ucca0\ub3c4 \uadf8\ub8f9(\ud604 \uc138\uc774\ubd80 \uadf8\ub8f9)\uacfc \uc138\uc774\ubd80 \uc720\ud1b5 \uadf8\ub8f9(\uc138\uc874 \uadf8\ub8f9, 2000\ub144\ub300\uc5d0 \uc2e4\uc9c8\uc801\uc73c\ub85c \ud574\uccb4)\uc73c\ub85c \ubd84\uc5f4\ub418\uc5c8\uae30 \ub54c\ubb38\uc774\ub2e4.", "paragraph_id": "6603770-1-0", "paragraph_question": "question: \uc138\uc774\ubd80 \ucca0\ub3c4\uc758 \ucc3d\uc5c5\uc790 \uc774\ub984\uc740?, context: \ub610\ud55c \uc624\ud14c \uc0ac\ucca0 \uc911\uc5d0\uc11c\ub294 \uc720\uc77c\ud558\uac8c \ubc31\ud654\uc810\ub4f1 \uc720\ud1b5\uc5c5\uc744 \ud68c\uc0ac \uc18c\uc18d\uc5d0 \ub450\uace0 \uc788\uc9c0 \uc54a\ub2e4. \uc774\uac83\uc740, \ucc3d\uc5c5\uc790\uc778 \uc4f0\uc4f0\ubbf8 \uc57c\uc2a4\uc9c0\ub85c \uc0ac\ud6c4\uc758 \ud6c4\uacc4 \uc2f8\uc6c0\uc73c\ub85c \uc778\ud558\uc5ec, \uc138\uc774\ubd80\ucca0\ub3c4 \uadf8\ub8f9(\ud604 \uc138\uc774\ubd80 \uadf8\ub8f9)\uacfc \uc138\uc774\ubd80 \uc720\ud1b5 \uadf8\ub8f9(\uc138\uc874 \uadf8\ub8f9, 2000\ub144\ub300\uc5d0 \uc2e4\uc9c8\uc801\uc73c\ub85c \ud574\uccb4)\uc73c\ub85c \ubd84\uc5f4\ub418\uc5c8\uae30 \ub54c\ubb38\uc774\ub2e4. \ub2e4\ub9cc, 2003\ub144\uc758 \ubc00\ub808\ub2c8\uc5c4 \ub9ac\ud14c\uc774\ub9c1\uc758 \uc640\ub2e4 \uc2dc\uac8c\ub8e8 \uc544\ud0a4\ub77c \uc0ac\uc7a5(\ub2f9\uc2dc)\uc774 \uc4f0\uc4f0\ubbf8 \uc694\uc2dc\uc544\ud0a4 \ud68c\uc7a5(\ub2f9\uc2dc)\uc5d0\uac8c \uc81c3\uc0ac \ud560\ub2f9 \uc99d\uc790\ub97c \ud0c0\uc9c4\ud558\uc5ec, \uc138\uc774\ubd80 \ucca0\ub3c4\uac00 \ucd9c\uc790\ub97c \ub9e1\uc558\ub2e4. 2006\ub144\uc758 \uc138\uc774\ubd80 \ud640\ub529\uc2a4 \uc124\ub9bd\uc5d0 \uc218\ubc18\ud558\ub294 \uace0\ucfe0\ub3c4(\u30b3\u30af\u30c9)\uc758 \uc81c\uc0bc\uc790 \ud560\ub2f9 \uc99d\uc790\uc2dc\uc5d0\ub294 \ubc00\ub808\ub2c8\uc5c4\uc774 10\uc5b5\uc5d4 \ucd9c\uc790\ub97c \ub9e1\uc544 \uc138\ube10&\uc544\uc774\u00b7\ud640\ub529\uc2a4\uac00 \ubc00\ub808\ub2c8\uc5c4 \uc8fc\uc2dd\uc744 \ud604\uae08\uc73c\ub85c \uc218\ub9e4\ud560 \ub54c\uae4c\uc9c0\ub294 \uc8fc\uc2dd\uc758 \uade0\ud615 \uc0c1\ud0dc\uc600\ub2e4. \uc138\uc774\ubd80 \ubc31\ud654\uc810\uc758 \uc791\uc740 \uc810\ud3ec\ub85c \ub4f1\uc7a5\ud55c \uc138\uc774\uc720\uac00 \uc138\uc774\ubd80 \uc5f0\uc120\uc5d0 \ub9ce\uc740 \uac83\uc740, \uadf8\ub8f9\uc774 \ubd84\uc5f4\ub418\uae30 \uc804\uc758 \ubc1c\uc804 \uacfc\uc815\uc5d0\uc11c \uc810\ud3ec\uc640 \ud1a0\uc9c0 \uc784\ub300\uac00 \ud604\uc7ac\uae4c\uc9c0 \uacc4\uc18d\ud558\uace0 \uc788\uae30 \ub54c\ubb38\uc774\ub2e4. \uadf8\ub7ec\ub098, \uc138\uc774\uc720\uc640\uc758 \uc5f0\uacb0\uc740 \uc544\uc9c1\ub3c4 \uae4a\uc73c\uba70 \uc138\uc774\ubd80 \ucca0\ub3c4\uac00 \uac1c\ubc1c\ud558\ub294 \ub274\ud0c0\uc6b4\uc774\ub098 \uc5ed \uac1c\ub7c9 \uacf5\uc0ac \ud6c4\uc5d0 \uc138\uc774\uc720\uc758 \uc0c8\ub85c\uc6b4 \uc810\ud3ec\uac00 \uac1c\uc5c5\ud558\ub294 \uacbd\uc6b0\uac00 \ub9ce\ub2e4. \ub610\ud55c, \uc138\uc774\uc720\uc758 \uc18c\ub9e4 \uc0ac\uc5c5\uc73c\ub85c\uc11c \ubc1c\uc871\ud55c \ud328\ubc00\ub9ac \ub9c8\ud2b8\ub294 \uc5ed \uad6c\ub0b4 \ud3b8\uc758\uc810\uc778 TOMONY\uc758 \uc804\uac1c\ub85c \uc81c\ud734\ud558\uace0 \uc788\ub2e4."} {"question": "\uc804\uad6d\uc5d0\uc11c \ub450 \ubc88\uc9f8\ub85c \ub3c4\uc2dc\ucca0\ub3c4\uac00 \ud070 \uc9c0\uc5ed\uc740?", "paragraph": "\ub300\ud55c\ubbfc\uad6d\uc758 \ub3c4\uc2dc\ucca0\ub3c4\ub294 \uc218\ub3c4\uad8c\uc9c0\uc5ed\uacfc \ubd80\uc0b0\uc9c0\uc5ed, \ub300\uad6c\uad11\uc5ed\uc2dc, \ub300\uc804\uad11\uc5ed\uc2dc, \uad11\uc8fc\uad11\uc5ed\uc2dc \ub4f1\uc5d0\uc11c \uc6b4\ud589\ub418\uace0 \uc788\ub2e4. \uc2dc\ub0b4\ub97c \uc6b4\ud589\ud558\ub294 \ub3c4\uc2dc\ucca0\ub3c4\ub97c \uc77c\uceec\uc5b4 '\uc9c0\ud558\ucca0' \ub610\ub294 '\uc804\ucca0' \ub4f1\uc73c\ub85c \ubd80\ub974\uae30\ub3c4 \ud55c\ub2e4. 1974\ub144 8\uc6d4 15\uc77c \ucc98\uc74c\uc73c\ub85c \uac1c\ud1b5\ub41c \uc218\ub3c4\uad8c\uc9c0\uc5ed\uc740 1~9\ud638\uc120\uacfc \ubd84\ub2f9\uc120, \uc911\uc559\uc120, \uacbd\uc758\uc120, \uacbd\ucd98\uc120, \uc778\ucc9c 1\ud638\uc120, \uc778\ucc9c\uad6d\uc81c\uacf5\ud56d\ucca0\ub3c4, \uc2e0\ubd84\ub2f9\uc120, \uc218\uc778\uc120, \uc6a9\uc778\uacbd\uc804\ucca0, \uc758\uc815\ubd80\uacbd\uc804\ucca0 \uae4c\uc9c0 \ucd1d 19\uac1c \ub178\uc120\uc774 \uc6b4\ud589\ub418\uace0 \uc788\uc73c\uba70 \uc804\uad6d\uc5d0\uc11c \uac00\uc7a5 \uaddc\ubaa8\uac00 \ud06c\ub2e4. \uae30\ubcf8\uc694\uae08\uc740 \ud3c9\uade0\uc801\uc73c\ub85c \uad50\ud1b5\uce74\ub4dc \uc774\uc6a9\uc2dc \ucd5c\ucd08 10km\uae4c\uc9c0 1050\uc6d0, \uadf8\uc774\ud6c4 5;km \ucd08\uacfc\uc2dc\ub9c8\ub2e4 100\uc6d0\uc529 \ucd94\uac00\ub41c\ub2e4. \uadf8\ub7ec\ub098 \ubbfc\uc601 \ub3c4\uc2dc\ucca0\ub3c4\uc778 \uc2e0\ubd84\ub2f9\uc120\uc740 \uae30\ubcf8\uc694\uae08\uc774 1,750\uc6d0\uc774\uace0, \ub2e4\ub978 \ub178\uc120\uc5d0\uc11c \uc2e0\ubd84\ub2f9\uc120\uc73c\ub85c \ud658\uc2b9\ud560 \uacbd\uc6b0 700\uc6d0\uc774 \ucd94\uac00\ub41c\ub2e4. \uc218\ub3c4\uad8c \uc804\ucca0\uc740 \uc778\ucc9c \ub3c4\uc2dc\ucca0\ub3c4, \uc778\ucc9c\uacf5\ud56d, \uae40\ud3ec\uacf5\ud56d\uacfc\ub3c4 \uc5f0\uacb0\ub41c\ub2e4. 1985\ub144 \uac1c\ud1b5\ub41c \ub450 \ubc88\uc9f8\ub85c \ud070 \ubd80\uc0b0 \ub3c4\uc2dc\ucca0\ub3c4\ub294 \ucd1d \uae38\uc774 131.7km\uc5d0 1\ud638\uc120, 2\ud638\uc120, 3\ud638\uc120, 4\ud638\uc120, \ubd80\uc0b0\uae40\ud574\uacbd\uc804\ucca0 5\uac1c \ub178\uc120\uc774 \uc6b4\ud589\ub418\uace0 \uc788\uc73c\uba70, \uc774\uc678\uc5d0\ub3c4 \ub300\uad6c\uc9c0\ud558\ucca0 1\ud638\uc120, 2\ud638\uc120 2\uac1c \ub178\uc120\uc774 \uc6b4\ud589\ub418\uace0 \ub300\uc804\uc9c0\ud558\ucca0 1\ud638\uc120 1\uac1c\ub178\uc120\uc774 \uc6b4\uc601, \uad11\uc8fc1\ud638\uc120 1\uac1c \ub178\uc120 \uc6b4\ud589 \ub4f1 \uc77c\ubd80 \uad11\uc5ed\uc2dc\ub4e4\uc744 \uc911\uc2ec\uc73c\ub85c \ub3c4\uc2dc\ucca0\ub3c4\uac00 \uacc4\uc18d\ub418\uc5b4 \uac74\uc124\uc911\uc774\ub2e4. \uc694\uae08\uc740 \uac01 \uc9c0\uc790\uccb4\uc5d0\uc11c \ub530\ub85c \uaddc\uc815\ud558\uc5ec \uc870\uae08\uc529 \ub2e4\ub974\ub2e4. \ubcf5\uc9c0 \uc815\ucc45\uc73c\ub85c 65\uc138 \uc774\uc0c1\uc758 \ub178\uc778\uacfc \uc7a5\uc560\uc778, \uad6d\uac00\uc720\uacf5\uc790\ub294 \ubb34\uc784\uc73c\ub85c \uc2b9\ucc28\ud560 \uc218 \uc788\ub2e4.(\ubd80\uc0b0-\uae40\ud574 \uacbd\uc804\ucca0\uc740 \ub178\uc778\ub3c4 \uc720\uc784\uc2b9\ucc28)", "answer": "\ubd80\uc0b0", "sentence": "\ub300\ud55c\ubbfc\uad6d\uc758 \ub3c4\uc2dc\ucca0\ub3c4\ub294 \uc218\ub3c4\uad8c\uc9c0\uc5ed\uacfc \ubd80\uc0b0 \uc9c0\uc5ed, \ub300\uad6c\uad11\uc5ed\uc2dc, \ub300\uc804\uad11\uc5ed\uc2dc, \uad11\uc8fc\uad11\uc5ed\uc2dc \ub4f1\uc5d0\uc11c \uc6b4\ud589\ub418\uace0 \uc788\ub2e4.", "paragraph_sentence": " \ub300\ud55c\ubbfc\uad6d\uc758 \ub3c4\uc2dc\ucca0\ub3c4\ub294 \uc218\ub3c4\uad8c\uc9c0\uc5ed\uacfc \ubd80\uc0b0 \uc9c0\uc5ed, \ub300\uad6c\uad11\uc5ed\uc2dc, \ub300\uc804\uad11\uc5ed\uc2dc, \uad11\uc8fc\uad11\uc5ed\uc2dc \ub4f1\uc5d0\uc11c \uc6b4\ud589\ub418\uace0 \uc788\ub2e4. \uc2dc\ub0b4\ub97c \uc6b4\ud589\ud558\ub294 \ub3c4\uc2dc\ucca0\ub3c4\ub97c \uc77c\uceec\uc5b4 '\uc9c0\ud558\ucca0' \ub610\ub294 '\uc804\ucca0' \ub4f1\uc73c\ub85c \ubd80\ub974\uae30\ub3c4 \ud55c\ub2e4. 1974\ub144 8\uc6d4 15\uc77c \ucc98\uc74c\uc73c\ub85c \uac1c\ud1b5\ub41c \uc218\ub3c4\uad8c\uc9c0\uc5ed\uc740 1~9\ud638\uc120\uacfc \ubd84\ub2f9\uc120, \uc911\uc559\uc120, \uacbd\uc758\uc120, \uacbd\ucd98\uc120, \uc778\ucc9c 1\ud638\uc120, \uc778\ucc9c\uad6d\uc81c\uacf5\ud56d\ucca0\ub3c4, \uc2e0\ubd84\ub2f9\uc120, \uc218\uc778\uc120, \uc6a9\uc778\uacbd\uc804\ucca0, \uc758\uc815\ubd80\uacbd\uc804\ucca0 \uae4c\uc9c0 \ucd1d 19\uac1c \ub178\uc120\uc774 \uc6b4\ud589\ub418\uace0 \uc788\uc73c\uba70 \uc804\uad6d\uc5d0\uc11c \uac00\uc7a5 \uaddc\ubaa8\uac00 \ud06c\ub2e4. \uae30\ubcf8\uc694\uae08\uc740 \ud3c9\uade0\uc801\uc73c\ub85c \uad50\ud1b5\uce74\ub4dc \uc774\uc6a9\uc2dc \ucd5c\ucd08 10km\uae4c\uc9c0 1050\uc6d0, \uadf8\uc774\ud6c4 5;km \ucd08\uacfc\uc2dc\ub9c8\ub2e4 100\uc6d0\uc529 \ucd94\uac00\ub41c\ub2e4. \uadf8\ub7ec\ub098 \ubbfc\uc601 \ub3c4\uc2dc\ucca0\ub3c4\uc778 \uc2e0\ubd84\ub2f9\uc120\uc740 \uae30\ubcf8\uc694\uae08\uc774 1,750\uc6d0\uc774\uace0, \ub2e4\ub978 \ub178\uc120\uc5d0\uc11c \uc2e0\ubd84\ub2f9\uc120\uc73c\ub85c \ud658\uc2b9\ud560 \uacbd\uc6b0 700\uc6d0\uc774 \ucd94\uac00\ub41c\ub2e4. \uc218\ub3c4\uad8c \uc804\ucca0\uc740 \uc778\ucc9c \ub3c4\uc2dc\ucca0\ub3c4, \uc778\ucc9c\uacf5\ud56d, \uae40\ud3ec\uacf5\ud56d\uacfc\ub3c4 \uc5f0\uacb0\ub41c\ub2e4. 1985\ub144 \uac1c\ud1b5\ub41c \ub450 \ubc88\uc9f8\ub85c \ud070 \ubd80\uc0b0 \ub3c4\uc2dc\ucca0\ub3c4\ub294 \ucd1d \uae38\uc774 131.7km\uc5d0 1\ud638\uc120, 2\ud638\uc120, 3\ud638\uc120, 4\ud638\uc120, \ubd80\uc0b0\uae40\ud574\uacbd\uc804\ucca0 5\uac1c \ub178\uc120\uc774 \uc6b4\ud589\ub418\uace0 \uc788\uc73c\uba70, \uc774\uc678\uc5d0\ub3c4 \ub300\uad6c\uc9c0\ud558\ucca0 1\ud638\uc120, 2\ud638\uc120 2\uac1c \ub178\uc120\uc774 \uc6b4\ud589\ub418\uace0 \ub300\uc804\uc9c0\ud558\ucca0 1\ud638\uc120 1\uac1c\ub178\uc120\uc774 \uc6b4\uc601, \uad11\uc8fc1\ud638\uc120 1\uac1c \ub178\uc120 \uc6b4\ud589 \ub4f1 \uc77c\ubd80 \uad11\uc5ed\uc2dc\ub4e4\uc744 \uc911\uc2ec\uc73c\ub85c \ub3c4\uc2dc\ucca0\ub3c4\uac00 \uacc4\uc18d\ub418\uc5b4 \uac74\uc124\uc911\uc774\ub2e4. \uc694\uae08\uc740 \uac01 \uc9c0\uc790\uccb4\uc5d0\uc11c \ub530\ub85c \uaddc\uc815\ud558\uc5ec \uc870\uae08\uc529 \ub2e4\ub974\ub2e4. \ubcf5\uc9c0 \uc815\ucc45\uc73c\ub85c 65\uc138 \uc774\uc0c1\uc758 \ub178\uc778\uacfc \uc7a5\uc560\uc778, \uad6d\uac00\uc720\uacf5\uc790\ub294 \ubb34\uc784\uc73c\ub85c \uc2b9\ucc28\ud560 \uc218 \uc788\ub2e4.(\ubd80\uc0b0-\uae40\ud574 \uacbd\uc804\ucca0\uc740 \ub178\uc778\ub3c4 \uc720\uc784\uc2b9\ucc28)", "paragraph_answer": "\ub300\ud55c\ubbfc\uad6d\uc758 \ub3c4\uc2dc\ucca0\ub3c4\ub294 \uc218\ub3c4\uad8c\uc9c0\uc5ed\uacfc \ubd80\uc0b0 \uc9c0\uc5ed, \ub300\uad6c\uad11\uc5ed\uc2dc, \ub300\uc804\uad11\uc5ed\uc2dc, \uad11\uc8fc\uad11\uc5ed\uc2dc \ub4f1\uc5d0\uc11c \uc6b4\ud589\ub418\uace0 \uc788\ub2e4. \uc2dc\ub0b4\ub97c \uc6b4\ud589\ud558\ub294 \ub3c4\uc2dc\ucca0\ub3c4\ub97c \uc77c\uceec\uc5b4 '\uc9c0\ud558\ucca0' \ub610\ub294 '\uc804\ucca0' \ub4f1\uc73c\ub85c \ubd80\ub974\uae30\ub3c4 \ud55c\ub2e4. 1974\ub144 8\uc6d4 15\uc77c \ucc98\uc74c\uc73c\ub85c \uac1c\ud1b5\ub41c \uc218\ub3c4\uad8c\uc9c0\uc5ed\uc740 1~9\ud638\uc120\uacfc \ubd84\ub2f9\uc120, \uc911\uc559\uc120, \uacbd\uc758\uc120, \uacbd\ucd98\uc120, \uc778\ucc9c 1\ud638\uc120, \uc778\ucc9c\uad6d\uc81c\uacf5\ud56d\ucca0\ub3c4, \uc2e0\ubd84\ub2f9\uc120, \uc218\uc778\uc120, \uc6a9\uc778\uacbd\uc804\ucca0, \uc758\uc815\ubd80\uacbd\uc804\ucca0 \uae4c\uc9c0 \ucd1d 19\uac1c \ub178\uc120\uc774 \uc6b4\ud589\ub418\uace0 \uc788\uc73c\uba70 \uc804\uad6d\uc5d0\uc11c \uac00\uc7a5 \uaddc\ubaa8\uac00 \ud06c\ub2e4. \uae30\ubcf8\uc694\uae08\uc740 \ud3c9\uade0\uc801\uc73c\ub85c \uad50\ud1b5\uce74\ub4dc \uc774\uc6a9\uc2dc \ucd5c\ucd08 10km\uae4c\uc9c0 1050\uc6d0, \uadf8\uc774\ud6c4 5;km \ucd08\uacfc\uc2dc\ub9c8\ub2e4 100\uc6d0\uc529 \ucd94\uac00\ub41c\ub2e4. \uadf8\ub7ec\ub098 \ubbfc\uc601 \ub3c4\uc2dc\ucca0\ub3c4\uc778 \uc2e0\ubd84\ub2f9\uc120\uc740 \uae30\ubcf8\uc694\uae08\uc774 1,750\uc6d0\uc774\uace0, \ub2e4\ub978 \ub178\uc120\uc5d0\uc11c \uc2e0\ubd84\ub2f9\uc120\uc73c\ub85c \ud658\uc2b9\ud560 \uacbd\uc6b0 700\uc6d0\uc774 \ucd94\uac00\ub41c\ub2e4. \uc218\ub3c4\uad8c \uc804\ucca0\uc740 \uc778\ucc9c \ub3c4\uc2dc\ucca0\ub3c4, \uc778\ucc9c\uacf5\ud56d, \uae40\ud3ec\uacf5\ud56d\uacfc\ub3c4 \uc5f0\uacb0\ub41c\ub2e4. 1985\ub144 \uac1c\ud1b5\ub41c \ub450 \ubc88\uc9f8\ub85c \ud070 \ubd80\uc0b0 \ub3c4\uc2dc\ucca0\ub3c4\ub294 \ucd1d \uae38\uc774 131.7km\uc5d0 1\ud638\uc120, 2\ud638\uc120, 3\ud638\uc120, 4\ud638\uc120, \ubd80\uc0b0\uae40\ud574\uacbd\uc804\ucca0 5\uac1c \ub178\uc120\uc774 \uc6b4\ud589\ub418\uace0 \uc788\uc73c\uba70, \uc774\uc678\uc5d0\ub3c4 \ub300\uad6c\uc9c0\ud558\ucca0 1\ud638\uc120, 2\ud638\uc120 2\uac1c \ub178\uc120\uc774 \uc6b4\ud589\ub418\uace0 \ub300\uc804\uc9c0\ud558\ucca0 1\ud638\uc120 1\uac1c\ub178\uc120\uc774 \uc6b4\uc601, \uad11\uc8fc1\ud638\uc120 1\uac1c \ub178\uc120 \uc6b4\ud589 \ub4f1 \uc77c\ubd80 \uad11\uc5ed\uc2dc\ub4e4\uc744 \uc911\uc2ec\uc73c\ub85c \ub3c4\uc2dc\ucca0\ub3c4\uac00 \uacc4\uc18d\ub418\uc5b4 \uac74\uc124\uc911\uc774\ub2e4. \uc694\uae08\uc740 \uac01 \uc9c0\uc790\uccb4\uc5d0\uc11c \ub530\ub85c \uaddc\uc815\ud558\uc5ec \uc870\uae08\uc529 \ub2e4\ub974\ub2e4. \ubcf5\uc9c0 \uc815\ucc45\uc73c\ub85c 65\uc138 \uc774\uc0c1\uc758 \ub178\uc778\uacfc \uc7a5\uc560\uc778, \uad6d\uac00\uc720\uacf5\uc790\ub294 \ubb34\uc784\uc73c\ub85c \uc2b9\ucc28\ud560 \uc218 \uc788\ub2e4.(\ubd80\uc0b0-\uae40\ud574 \uacbd\uc804\ucca0\uc740 \ub178\uc778\ub3c4 \uc720\uc784\uc2b9\ucc28)", "sentence_answer": "\ub300\ud55c\ubbfc\uad6d\uc758 \ub3c4\uc2dc\ucca0\ub3c4\ub294 \uc218\ub3c4\uad8c\uc9c0\uc5ed\uacfc \ubd80\uc0b0 \uc9c0\uc5ed, \ub300\uad6c\uad11\uc5ed\uc2dc, \ub300\uc804\uad11\uc5ed\uc2dc, \uad11\uc8fc\uad11\uc5ed\uc2dc \ub4f1\uc5d0\uc11c \uc6b4\ud589\ub418\uace0 \uc788\ub2e4.", "paragraph_id": "6485683-27-1", "paragraph_question": "question: \uc804\uad6d\uc5d0\uc11c \ub450 \ubc88\uc9f8\ub85c \ub3c4\uc2dc\ucca0\ub3c4\uac00 \ud070 \uc9c0\uc5ed\uc740?, context: \ub300\ud55c\ubbfc\uad6d\uc758 \ub3c4\uc2dc\ucca0\ub3c4\ub294 \uc218\ub3c4\uad8c\uc9c0\uc5ed\uacfc \ubd80\uc0b0\uc9c0\uc5ed, \ub300\uad6c\uad11\uc5ed\uc2dc, \ub300\uc804\uad11\uc5ed\uc2dc, \uad11\uc8fc\uad11\uc5ed\uc2dc \ub4f1\uc5d0\uc11c \uc6b4\ud589\ub418\uace0 \uc788\ub2e4. \uc2dc\ub0b4\ub97c \uc6b4\ud589\ud558\ub294 \ub3c4\uc2dc\ucca0\ub3c4\ub97c \uc77c\uceec\uc5b4 '\uc9c0\ud558\ucca0' \ub610\ub294 '\uc804\ucca0' \ub4f1\uc73c\ub85c \ubd80\ub974\uae30\ub3c4 \ud55c\ub2e4. 1974\ub144 8\uc6d4 15\uc77c \ucc98\uc74c\uc73c\ub85c \uac1c\ud1b5\ub41c \uc218\ub3c4\uad8c\uc9c0\uc5ed\uc740 1~9\ud638\uc120\uacfc \ubd84\ub2f9\uc120, \uc911\uc559\uc120, \uacbd\uc758\uc120, \uacbd\ucd98\uc120, \uc778\ucc9c 1\ud638\uc120, \uc778\ucc9c\uad6d\uc81c\uacf5\ud56d\ucca0\ub3c4, \uc2e0\ubd84\ub2f9\uc120, \uc218\uc778\uc120, \uc6a9\uc778\uacbd\uc804\ucca0, \uc758\uc815\ubd80\uacbd\uc804\ucca0 \uae4c\uc9c0 \ucd1d 19\uac1c \ub178\uc120\uc774 \uc6b4\ud589\ub418\uace0 \uc788\uc73c\uba70 \uc804\uad6d\uc5d0\uc11c \uac00\uc7a5 \uaddc\ubaa8\uac00 \ud06c\ub2e4. \uae30\ubcf8\uc694\uae08\uc740 \ud3c9\uade0\uc801\uc73c\ub85c \uad50\ud1b5\uce74\ub4dc \uc774\uc6a9\uc2dc \ucd5c\ucd08 10km\uae4c\uc9c0 1050\uc6d0, \uadf8\uc774\ud6c4 5;km \ucd08\uacfc\uc2dc\ub9c8\ub2e4 100\uc6d0\uc529 \ucd94\uac00\ub41c\ub2e4. \uadf8\ub7ec\ub098 \ubbfc\uc601 \ub3c4\uc2dc\ucca0\ub3c4\uc778 \uc2e0\ubd84\ub2f9\uc120\uc740 \uae30\ubcf8\uc694\uae08\uc774 1,750\uc6d0\uc774\uace0, \ub2e4\ub978 \ub178\uc120\uc5d0\uc11c \uc2e0\ubd84\ub2f9\uc120\uc73c\ub85c \ud658\uc2b9\ud560 \uacbd\uc6b0 700\uc6d0\uc774 \ucd94\uac00\ub41c\ub2e4. \uc218\ub3c4\uad8c \uc804\ucca0\uc740 \uc778\ucc9c \ub3c4\uc2dc\ucca0\ub3c4, \uc778\ucc9c\uacf5\ud56d, \uae40\ud3ec\uacf5\ud56d\uacfc\ub3c4 \uc5f0\uacb0\ub41c\ub2e4. 1985\ub144 \uac1c\ud1b5\ub41c \ub450 \ubc88\uc9f8\ub85c \ud070 \ubd80\uc0b0 \ub3c4\uc2dc\ucca0\ub3c4\ub294 \ucd1d \uae38\uc774 131.7km\uc5d0 1\ud638\uc120, 2\ud638\uc120, 3\ud638\uc120, 4\ud638\uc120, \ubd80\uc0b0\uae40\ud574\uacbd\uc804\ucca0 5\uac1c \ub178\uc120\uc774 \uc6b4\ud589\ub418\uace0 \uc788\uc73c\uba70, \uc774\uc678\uc5d0\ub3c4 \ub300\uad6c\uc9c0\ud558\ucca0 1\ud638\uc120, 2\ud638\uc120 2\uac1c \ub178\uc120\uc774 \uc6b4\ud589\ub418\uace0 \ub300\uc804\uc9c0\ud558\ucca0 1\ud638\uc120 1\uac1c\ub178\uc120\uc774 \uc6b4\uc601, \uad11\uc8fc1\ud638\uc120 1\uac1c \ub178\uc120 \uc6b4\ud589 \ub4f1 \uc77c\ubd80 \uad11\uc5ed\uc2dc\ub4e4\uc744 \uc911\uc2ec\uc73c\ub85c \ub3c4\uc2dc\ucca0\ub3c4\uac00 \uacc4\uc18d\ub418\uc5b4 \uac74\uc124\uc911\uc774\ub2e4. \uc694\uae08\uc740 \uac01 \uc9c0\uc790\uccb4\uc5d0\uc11c \ub530\ub85c \uaddc\uc815\ud558\uc5ec \uc870\uae08\uc529 \ub2e4\ub974\ub2e4. \ubcf5\uc9c0 \uc815\ucc45\uc73c\ub85c 65\uc138 \uc774\uc0c1\uc758 \ub178\uc778\uacfc \uc7a5\uc560\uc778, \uad6d\uac00\uc720\uacf5\uc790\ub294 \ubb34\uc784\uc73c\ub85c \uc2b9\ucc28\ud560 \uc218 \uc788\ub2e4.(\ubd80\uc0b0-\uae40\ud574 \uacbd\uc804\ucca0\uc740 \ub178\uc778\ub3c4 \uc720\uc784\uc2b9\ucc28)"} {"question": "\ub514\uc2a4\uc6cc\uc624\ube0c\ub9c8\uc778\uc5d0\uc11c NPC\ub97c \ub3c4\uc640\uc900 \uce90\ub9ad\ud130\ub294 \uc74c\uc2dd\uc774\ub098 \ubb34\uc5c7\uc744 \uc5bb\uc5b4\uc62c \uc218 \uc788\ub098?", "paragraph": "NPC \uc5ec\ub7ff\uc774 \ubab0\ub824\uc640 \uc77c\uc190\uc744 \uc694\uccad\ud55c\ub2e4. \uc2b9\ub099\ud558\uba74 \uc77c\uc744 \ub3c4\uc640\uc8fc\uace0 \ucd1d\uc54c\uc774\ub098 \uc74c\uc2dd \ub4f1\uc744 \uc5bb\uc5b4\uc62c \uc218 \uc788\ub294 \ubc18\uba74, \ub3c4\uc640\uc8fc\ub7ec \uac04 \uce90\ub9ad\ud130\ub294 \ud558\ub8fb\ubc24 \uc790\ub9ac\ub97c \ube44\uc6b0\uace0 \ub2e4\uc74c\ub0a0 \ud53c\ub85c\uac00 \uc313\uc778 \ucc44 \ub3cc\uc544\uc628\ub2e4. \ub530\ub77c\uc11c \ubcf4\ub0b4\ub294 \uce90\ub9ad\ud130\ub97c \uc798 \uc120\ud0dd\ud574\uc57c \ud55c\ub2e4. \uc704\ud5d8\ud55c \ubd80\ud0c1\uc744 \ud574\uc62c \uacbd\uc6b0 \uc0c1\ud0dc\uac00 \uc88b\uc9c0 \uc54a\uc740 \uce90\ub9ad\ud130\ub97c \ubcf4\ub0b4\uba74 \uc8fd\uc5b4\uc11c \ub3cc\uc544\uc624\uc9c0 \ubabb\ud560 \uac00\ub2a5\uc131\uc774 \ub192\uc73c\ubbc0\ub85c \uc0c1\ud0dc\uac00 \uc88b\uc740 \uce90\ub9ad\ud130\ub97c \ubcf4\ub0b4\uba70 \ubcf4\ub0b4\uae30 \uc804\uc5d0 \ubc25\uc774\ub098 \uc57d\uc744 \uba39\uc774\uace0 \ubcf4\ub0bc \ud544\uc694\uac00 \uc788\ub2e4.\ub3c4\uc6c0 \uc694\uccad\uc758 \uc218\ub77d, \uac70\ubd80\uc5d0 \ub300\ud55c \ubc18\uc751\uc774 \uce90\ub9ad\ud130\ubcc4\ub85c \ub2e4\ub978 \uac83\uc774 \ub208\uc5d0 \ub744\ub294 \uc7ac\ubbf8\uc694\uc18c\uc774\ub2e4. \ube0c\ub8e8\ub178, \uc5d0\ubc00\ub9ac\uc544\ub294 \uc774\uc6c3\uc744 \ub3d5\ub294 \uac83\uc5d0 \ub300\ud574 \ubd80\uc815\uc801\uc778 \ud0dc\ub3c4\ub97c \ub300\uc0ac\ub97c \ud1b5\ud574 \ubcf4\uc774\ub294 \ubc18\uba74, \ud30c\ube14, \uce74\ud2f0\uc544\ub294 \uae30\ubd84\uc774 \uc88b\uc544\uc9c4\ub2e4. \ud2b9\ud788 \ud06c\ubca0\ud0c0\ub294 \uc544\uc774\ub4e4\uc774 \ucc3e\uc544\uc654\ub294\ub370 \uc694\uccad\uc744 \uac70\ubd80\ud560 \uacbd\uc6b0 \uc0ac\uae30\uac00 \ud06c\uac8c \ud558\ub77d\ud55c\ub2e4.", "answer": "\ucd1d\uc54c", "sentence": "\uc2b9\ub099\ud558\uba74 \uc77c\uc744 \ub3c4\uc640\uc8fc\uace0 \ucd1d\uc54c \uc774\ub098 \uc74c\uc2dd \ub4f1\uc744 \uc5bb\uc5b4\uc62c \uc218 \uc788\ub294 \ubc18\uba74, \ub3c4\uc640\uc8fc\ub7ec \uac04 \uce90\ub9ad\ud130\ub294 \ud558\ub8fb\ubc24 \uc790\ub9ac\ub97c \ube44\uc6b0\uace0 \ub2e4\uc74c\ub0a0 \ud53c\ub85c\uac00 \uc313\uc778 \ucc44 \ub3cc\uc544\uc628\ub2e4.", "paragraph_sentence": "NPC \uc5ec\ub7ff\uc774 \ubab0\ub824\uc640 \uc77c\uc190\uc744 \uc694\uccad\ud55c\ub2e4. \uc2b9\ub099\ud558\uba74 \uc77c\uc744 \ub3c4\uc640\uc8fc\uace0 \ucd1d\uc54c \uc774\ub098 \uc74c\uc2dd \ub4f1\uc744 \uc5bb\uc5b4\uc62c \uc218 \uc788\ub294 \ubc18\uba74, \ub3c4\uc640\uc8fc\ub7ec \uac04 \uce90\ub9ad\ud130\ub294 \ud558\ub8fb\ubc24 \uc790\ub9ac\ub97c \ube44\uc6b0\uace0 \ub2e4\uc74c\ub0a0 \ud53c\ub85c\uac00 \uc313\uc778 \ucc44 \ub3cc\uc544\uc628\ub2e4. \ub530\ub77c\uc11c \ubcf4\ub0b4\ub294 \uce90\ub9ad\ud130\ub97c \uc798 \uc120\ud0dd\ud574\uc57c \ud55c\ub2e4. \uc704\ud5d8\ud55c \ubd80\ud0c1\uc744 \ud574\uc62c \uacbd\uc6b0 \uc0c1\ud0dc\uac00 \uc88b\uc9c0 \uc54a\uc740 \uce90\ub9ad\ud130\ub97c \ubcf4\ub0b4\uba74 \uc8fd\uc5b4\uc11c \ub3cc\uc544\uc624\uc9c0 \ubabb\ud560 \uac00\ub2a5\uc131\uc774 \ub192\uc73c\ubbc0\ub85c \uc0c1\ud0dc\uac00 \uc88b\uc740 \uce90\ub9ad\ud130\ub97c \ubcf4\ub0b4\uba70 \ubcf4\ub0b4\uae30 \uc804\uc5d0 \ubc25\uc774\ub098 \uc57d\uc744 \uba39\uc774\uace0 \ubcf4\ub0bc \ud544\uc694\uac00 \uc788\ub2e4.\ub3c4\uc6c0 \uc694\uccad\uc758 \uc218\ub77d, \uac70\ubd80\uc5d0 \ub300\ud55c \ubc18\uc751\uc774 \uce90\ub9ad\ud130\ubcc4\ub85c \ub2e4\ub978 \uac83\uc774 \ub208\uc5d0 \ub744\ub294 \uc7ac\ubbf8\uc694\uc18c\uc774\ub2e4. \ube0c\ub8e8\ub178, \uc5d0\ubc00\ub9ac\uc544\ub294 \uc774\uc6c3\uc744 \ub3d5\ub294 \uac83\uc5d0 \ub300\ud574 \ubd80\uc815\uc801\uc778 \ud0dc\ub3c4\ub97c \ub300\uc0ac\ub97c \ud1b5\ud574 \ubcf4\uc774\ub294 \ubc18\uba74, \ud30c\ube14, \uce74\ud2f0\uc544\ub294 \uae30\ubd84\uc774 \uc88b\uc544\uc9c4\ub2e4. \ud2b9\ud788 \ud06c\ubca0\ud0c0\ub294 \uc544\uc774\ub4e4\uc774 \ucc3e\uc544\uc654\ub294\ub370 \uc694\uccad\uc744 \uac70\ubd80\ud560 \uacbd\uc6b0 \uc0ac\uae30\uac00 \ud06c\uac8c \ud558\ub77d\ud55c\ub2e4.", "paragraph_answer": "NPC \uc5ec\ub7ff\uc774 \ubab0\ub824\uc640 \uc77c\uc190\uc744 \uc694\uccad\ud55c\ub2e4. \uc2b9\ub099\ud558\uba74 \uc77c\uc744 \ub3c4\uc640\uc8fc\uace0 \ucd1d\uc54c \uc774\ub098 \uc74c\uc2dd \ub4f1\uc744 \uc5bb\uc5b4\uc62c \uc218 \uc788\ub294 \ubc18\uba74, \ub3c4\uc640\uc8fc\ub7ec \uac04 \uce90\ub9ad\ud130\ub294 \ud558\ub8fb\ubc24 \uc790\ub9ac\ub97c \ube44\uc6b0\uace0 \ub2e4\uc74c\ub0a0 \ud53c\ub85c\uac00 \uc313\uc778 \ucc44 \ub3cc\uc544\uc628\ub2e4. \ub530\ub77c\uc11c \ubcf4\ub0b4\ub294 \uce90\ub9ad\ud130\ub97c \uc798 \uc120\ud0dd\ud574\uc57c \ud55c\ub2e4. \uc704\ud5d8\ud55c \ubd80\ud0c1\uc744 \ud574\uc62c \uacbd\uc6b0 \uc0c1\ud0dc\uac00 \uc88b\uc9c0 \uc54a\uc740 \uce90\ub9ad\ud130\ub97c \ubcf4\ub0b4\uba74 \uc8fd\uc5b4\uc11c \ub3cc\uc544\uc624\uc9c0 \ubabb\ud560 \uac00\ub2a5\uc131\uc774 \ub192\uc73c\ubbc0\ub85c \uc0c1\ud0dc\uac00 \uc88b\uc740 \uce90\ub9ad\ud130\ub97c \ubcf4\ub0b4\uba70 \ubcf4\ub0b4\uae30 \uc804\uc5d0 \ubc25\uc774\ub098 \uc57d\uc744 \uba39\uc774\uace0 \ubcf4\ub0bc \ud544\uc694\uac00 \uc788\ub2e4.\ub3c4\uc6c0 \uc694\uccad\uc758 \uc218\ub77d, \uac70\ubd80\uc5d0 \ub300\ud55c \ubc18\uc751\uc774 \uce90\ub9ad\ud130\ubcc4\ub85c \ub2e4\ub978 \uac83\uc774 \ub208\uc5d0 \ub744\ub294 \uc7ac\ubbf8\uc694\uc18c\uc774\ub2e4. \ube0c\ub8e8\ub178, \uc5d0\ubc00\ub9ac\uc544\ub294 \uc774\uc6c3\uc744 \ub3d5\ub294 \uac83\uc5d0 \ub300\ud574 \ubd80\uc815\uc801\uc778 \ud0dc\ub3c4\ub97c \ub300\uc0ac\ub97c \ud1b5\ud574 \ubcf4\uc774\ub294 \ubc18\uba74, \ud30c\ube14, \uce74\ud2f0\uc544\ub294 \uae30\ubd84\uc774 \uc88b\uc544\uc9c4\ub2e4. \ud2b9\ud788 \ud06c\ubca0\ud0c0\ub294 \uc544\uc774\ub4e4\uc774 \ucc3e\uc544\uc654\ub294\ub370 \uc694\uccad\uc744 \uac70\ubd80\ud560 \uacbd\uc6b0 \uc0ac\uae30\uac00 \ud06c\uac8c \ud558\ub77d\ud55c\ub2e4.", "sentence_answer": "\uc2b9\ub099\ud558\uba74 \uc77c\uc744 \ub3c4\uc640\uc8fc\uace0 \ucd1d\uc54c \uc774\ub098 \uc74c\uc2dd \ub4f1\uc744 \uc5bb\uc5b4\uc62c \uc218 \uc788\ub294 \ubc18\uba74, \ub3c4\uc640\uc8fc\ub7ec \uac04 \uce90\ub9ad\ud130\ub294 \ud558\ub8fb\ubc24 \uc790\ub9ac\ub97c \ube44\uc6b0\uace0 \ub2e4\uc74c\ub0a0 \ud53c\ub85c\uac00 \uc313\uc778 \ucc44 \ub3cc\uc544\uc628\ub2e4.", "paragraph_id": "6492782-5-0", "paragraph_question": "question: \ub514\uc2a4\uc6cc\uc624\ube0c\ub9c8\uc778\uc5d0\uc11c NPC\ub97c \ub3c4\uc640\uc900 \uce90\ub9ad\ud130\ub294 \uc74c\uc2dd\uc774\ub098 \ubb34\uc5c7\uc744 \uc5bb\uc5b4\uc62c \uc218 \uc788\ub098?, context: NPC \uc5ec\ub7ff\uc774 \ubab0\ub824\uc640 \uc77c\uc190\uc744 \uc694\uccad\ud55c\ub2e4. \uc2b9\ub099\ud558\uba74 \uc77c\uc744 \ub3c4\uc640\uc8fc\uace0 \ucd1d\uc54c\uc774\ub098 \uc74c\uc2dd \ub4f1\uc744 \uc5bb\uc5b4\uc62c \uc218 \uc788\ub294 \ubc18\uba74, \ub3c4\uc640\uc8fc\ub7ec \uac04 \uce90\ub9ad\ud130\ub294 \ud558\ub8fb\ubc24 \uc790\ub9ac\ub97c \ube44\uc6b0\uace0 \ub2e4\uc74c\ub0a0 \ud53c\ub85c\uac00 \uc313\uc778 \ucc44 \ub3cc\uc544\uc628\ub2e4. \ub530\ub77c\uc11c \ubcf4\ub0b4\ub294 \uce90\ub9ad\ud130\ub97c \uc798 \uc120\ud0dd\ud574\uc57c \ud55c\ub2e4. \uc704\ud5d8\ud55c \ubd80\ud0c1\uc744 \ud574\uc62c \uacbd\uc6b0 \uc0c1\ud0dc\uac00 \uc88b\uc9c0 \uc54a\uc740 \uce90\ub9ad\ud130\ub97c \ubcf4\ub0b4\uba74 \uc8fd\uc5b4\uc11c \ub3cc\uc544\uc624\uc9c0 \ubabb\ud560 \uac00\ub2a5\uc131\uc774 \ub192\uc73c\ubbc0\ub85c \uc0c1\ud0dc\uac00 \uc88b\uc740 \uce90\ub9ad\ud130\ub97c \ubcf4\ub0b4\uba70 \ubcf4\ub0b4\uae30 \uc804\uc5d0 \ubc25\uc774\ub098 \uc57d\uc744 \uba39\uc774\uace0 \ubcf4\ub0bc \ud544\uc694\uac00 \uc788\ub2e4.\ub3c4\uc6c0 \uc694\uccad\uc758 \uc218\ub77d, \uac70\ubd80\uc5d0 \ub300\ud55c \ubc18\uc751\uc774 \uce90\ub9ad\ud130\ubcc4\ub85c \ub2e4\ub978 \uac83\uc774 \ub208\uc5d0 \ub744\ub294 \uc7ac\ubbf8\uc694\uc18c\uc774\ub2e4. \ube0c\ub8e8\ub178, \uc5d0\ubc00\ub9ac\uc544\ub294 \uc774\uc6c3\uc744 \ub3d5\ub294 \uac83\uc5d0 \ub300\ud574 \ubd80\uc815\uc801\uc778 \ud0dc\ub3c4\ub97c \ub300\uc0ac\ub97c \ud1b5\ud574 \ubcf4\uc774\ub294 \ubc18\uba74, \ud30c\ube14, \uce74\ud2f0\uc544\ub294 \uae30\ubd84\uc774 \uc88b\uc544\uc9c4\ub2e4. \ud2b9\ud788 \ud06c\ubca0\ud0c0\ub294 \uc544\uc774\ub4e4\uc774 \ucc3e\uc544\uc654\ub294\ub370 \uc694\uccad\uc744 \uac70\ubd80\ud560 \uacbd\uc6b0 \uc0ac\uae30\uac00 \ud06c\uac8c \ud558\ub77d\ud55c\ub2e4."} {"question": "\uc720\ub7fd\ucc0c\ub974\ub808\uae30\uc758 \ub178\ub798\uc5d0\ub294 \ubcf4\ud1b5 \uba87\uac00\uc9c0\uc758 \ubcc0\uc885\uc774 \uc788\ub294\uac00?", "paragraph": "\uc720\ub7fd\ucc0c\ub974\ub808\uae30\ub294 \uc2dc\ub044\ub7ec\uc6b4 \uc0c8\ub2e4. \uadf8 \ub178\ub7ab\uc18c\ub9ac\ub294 \ub2e4\uc18c \uc74c\uc545\uc801\uc778 \uac83\uacfc \uae30\uacc4\uc9c8\uc758 \uc18c\uc74c\uc774 \ud568\uaed8 \uc11e\uc5ec \uc788\ub2e4. \uc8fc\ub85c \ub178\ub798\ub97c \ubd80\ub974\ub294 \uac83\uc740 \uc218\ucef7\uc73c\ub85c, \ud55c\ubc88 \uc2dc\uc791\ud558\uba74 1\ubd84 \uc774\uc0c1 \ub178\ub798\ub97c \ud55c\ubc14\ud0d5 \uc9c0\uc18d\ud55c\ub2e4. \uc774\ub7ec\ud55c \ub178\ub798\ub294 \ubcf4\ud1b5 \ub124 \uac00\uc9c0\uc758 \ubcc0\uc885\uc774 \uc788\ub294\ub370, \uc815\ud574\uc9c4 \uc21c\uc11c\ub300\ub85c \uc911\uac04\uc5d0 \uc27c\uc5c6\uc774 \ub178\ub798\ub97c \uacc4\uc18d\ud55c\ub2e4. \uc6b0\uc120 \uc21c\uc74c\uc73c\ub85c \uc774\ub8e8\uc5b4\uc9c4 \ud718\ud30c\ub78c\uc744 \uba87 \ubc88 \ubd88\uba74\uc11c \ub178\ub798\uac00 \uc2dc\uc791\ub41c\ub2e4. \uadf8 \ub4a4 \uc8fc \ubd80\ubd84\uc5d0\uc11c\ub294 \uc8fc\ub85c \ub2e4\ub978 \uc885\uc758 \uc0c8\uc18c\ub9ac\ub098 \uac01\uc885 \uc790\uc5f0\uc801, \uc778\uacf5\uc801 \uc18c\uc74c\ub4e4\uc744 \ubaa8\ubc29\ud55c\ub2e4. \ub178\ub798\uac00 \ub192\uc544\uc9c8 \uc218\ub85d \ubaa8\ubc29\ud55c \uc18c\ub9ac\uc758 \uad6c\uc870\ub294 \ud3c9\uc774\ud574\uc9c4\ub2e4. \uadf8 \ub2e4\uc74c \ub2e8\uacc4\uc5d0\uc11c\ub294 \ud55c\ub3d9\uc548 \uac19\uc740 \uc18c\ub9ac\ub97c \ubc18\ubcf5\ud558\uace0, \uadf8\ub9ac\uace0 \ub2e4\uc74c \ub2e8\uacc4\ub85c \ub118\uc5b4\uac04\ub2e4. \uc774 \ubcc0\uc8fc \ub4a4\uc5d0\ub294 \uc9e4\uae4d\uac70\ub9ac\ub294 \ud761\ucc29\uc74c\uc744 \uc5ec\ub7ec \ubc88 \ub0b4\uace0 \ub098\uc11c, \ub9c8\uc9c0\ub9c9\uc73c\ub85c \ub9e4\uc6b0 \ub192\uc740 \ub178\ub798\ub97c, \ub2e4\uc2dc \uc5ec\ub7ec \ud615\ud0dc\ub85c \ubcc0\uc8fc\ud558\uba74\uc11c \ubd88\ub7ec\ub304\ub2e4. \uac01\uac01\uc758 \uac1c\uccb4\ub294 \uc790\uae30\ub9cc\uc758 \ub808\ud37c\ud1a0\ub9ac\ub97c \uac16\uace0 \uc788\uc73c\uba70, \ub178\ub798\uc5d0 \ud2b9\ud788 \ub2a5\uc219\ud55c \uac1c\uccb4\ub4e4\uc740 35\uac1c \uc774\ud558\uc758 \ub178\ub7ab\uc18c\ub9ac\uc640 14\uac1c \uc774\ud558\uc758 \ud761\ucc29\uc74c\uc744 \ub0bc \uc218 \uc788\ub2e4.", "answer": "\ub124 \uac00\uc9c0", "sentence": "\uc774\ub7ec\ud55c \ub178\ub798\ub294 \ubcf4\ud1b5 \ub124 \uac00\uc9c0 \uc758 \ubcc0\uc885\uc774 \uc788\ub294\ub370, \uc815\ud574\uc9c4 \uc21c\uc11c\ub300\ub85c \uc911\uac04\uc5d0 \uc27c\uc5c6\uc774 \ub178\ub798\ub97c \uacc4\uc18d\ud55c\ub2e4.", "paragraph_sentence": "\uc720\ub7fd\ucc0c\ub974\ub808\uae30\ub294 \uc2dc\ub044\ub7ec\uc6b4 \uc0c8\ub2e4. \uadf8 \ub178\ub7ab\uc18c\ub9ac\ub294 \ub2e4\uc18c \uc74c\uc545\uc801\uc778 \uac83\uacfc \uae30\uacc4\uc9c8\uc758 \uc18c\uc74c\uc774 \ud568\uaed8 \uc11e\uc5ec \uc788\ub2e4. \uc8fc\ub85c \ub178\ub798\ub97c \ubd80\ub974\ub294 \uac83\uc740 \uc218\ucef7\uc73c\ub85c, \ud55c\ubc88 \uc2dc\uc791\ud558\uba74 1\ubd84 \uc774\uc0c1 \ub178\ub798\ub97c \ud55c\ubc14\ud0d5 \uc9c0\uc18d\ud55c\ub2e4. \uc774\ub7ec\ud55c \ub178\ub798\ub294 \ubcf4\ud1b5 \ub124 \uac00\uc9c0 \uc758 \ubcc0\uc885\uc774 \uc788\ub294\ub370, \uc815\ud574\uc9c4 \uc21c\uc11c\ub300\ub85c \uc911\uac04\uc5d0 \uc27c\uc5c6\uc774 \ub178\ub798\ub97c \uacc4\uc18d\ud55c\ub2e4. \uc6b0\uc120 \uc21c\uc74c\uc73c\ub85c \uc774\ub8e8\uc5b4\uc9c4 \ud718\ud30c\ub78c\uc744 \uba87 \ubc88 \ubd88\uba74\uc11c \ub178\ub798\uac00 \uc2dc\uc791\ub41c\ub2e4. \uadf8 \ub4a4 \uc8fc \ubd80\ubd84\uc5d0\uc11c\ub294 \uc8fc\ub85c \ub2e4\ub978 \uc885\uc758 \uc0c8\uc18c\ub9ac\ub098 \uac01\uc885 \uc790\uc5f0\uc801, \uc778\uacf5\uc801 \uc18c\uc74c\ub4e4\uc744 \ubaa8\ubc29\ud55c\ub2e4. \ub178\ub798\uac00 \ub192\uc544\uc9c8 \uc218\ub85d \ubaa8\ubc29\ud55c \uc18c\ub9ac\uc758 \uad6c\uc870\ub294 \ud3c9\uc774\ud574\uc9c4\ub2e4. \uadf8 \ub2e4\uc74c \ub2e8\uacc4\uc5d0\uc11c\ub294 \ud55c\ub3d9\uc548 \uac19\uc740 \uc18c\ub9ac\ub97c \ubc18\ubcf5\ud558\uace0, \uadf8\ub9ac\uace0 \ub2e4\uc74c \ub2e8\uacc4\ub85c \ub118\uc5b4\uac04\ub2e4. \uc774 \ubcc0\uc8fc \ub4a4\uc5d0\ub294 \uc9e4\uae4d\uac70\ub9ac\ub294 \ud761\ucc29\uc74c\uc744 \uc5ec\ub7ec \ubc88 \ub0b4\uace0 \ub098\uc11c, \ub9c8\uc9c0\ub9c9\uc73c\ub85c \ub9e4\uc6b0 \ub192\uc740 \ub178\ub798\ub97c, \ub2e4\uc2dc \uc5ec\ub7ec \ud615\ud0dc\ub85c \ubcc0\uc8fc\ud558\uba74\uc11c \ubd88\ub7ec\ub304\ub2e4. \uac01\uac01\uc758 \uac1c\uccb4\ub294 \uc790\uae30\ub9cc\uc758 \ub808\ud37c\ud1a0\ub9ac\ub97c \uac16\uace0 \uc788\uc73c\uba70, \ub178\ub798\uc5d0 \ud2b9\ud788 \ub2a5\uc219\ud55c \uac1c\uccb4\ub4e4\uc740 35\uac1c \uc774\ud558\uc758 \ub178\ub7ab\uc18c\ub9ac\uc640 14\uac1c \uc774\ud558\uc758 \ud761\ucc29\uc74c\uc744 \ub0bc \uc218 \uc788\ub2e4.", "paragraph_answer": "\uc720\ub7fd\ucc0c\ub974\ub808\uae30\ub294 \uc2dc\ub044\ub7ec\uc6b4 \uc0c8\ub2e4. \uadf8 \ub178\ub7ab\uc18c\ub9ac\ub294 \ub2e4\uc18c \uc74c\uc545\uc801\uc778 \uac83\uacfc \uae30\uacc4\uc9c8\uc758 \uc18c\uc74c\uc774 \ud568\uaed8 \uc11e\uc5ec \uc788\ub2e4. \uc8fc\ub85c \ub178\ub798\ub97c \ubd80\ub974\ub294 \uac83\uc740 \uc218\ucef7\uc73c\ub85c, \ud55c\ubc88 \uc2dc\uc791\ud558\uba74 1\ubd84 \uc774\uc0c1 \ub178\ub798\ub97c \ud55c\ubc14\ud0d5 \uc9c0\uc18d\ud55c\ub2e4. \uc774\ub7ec\ud55c \ub178\ub798\ub294 \ubcf4\ud1b5 \ub124 \uac00\uc9c0 \uc758 \ubcc0\uc885\uc774 \uc788\ub294\ub370, \uc815\ud574\uc9c4 \uc21c\uc11c\ub300\ub85c \uc911\uac04\uc5d0 \uc27c\uc5c6\uc774 \ub178\ub798\ub97c \uacc4\uc18d\ud55c\ub2e4. \uc6b0\uc120 \uc21c\uc74c\uc73c\ub85c \uc774\ub8e8\uc5b4\uc9c4 \ud718\ud30c\ub78c\uc744 \uba87 \ubc88 \ubd88\uba74\uc11c \ub178\ub798\uac00 \uc2dc\uc791\ub41c\ub2e4. \uadf8 \ub4a4 \uc8fc \ubd80\ubd84\uc5d0\uc11c\ub294 \uc8fc\ub85c \ub2e4\ub978 \uc885\uc758 \uc0c8\uc18c\ub9ac\ub098 \uac01\uc885 \uc790\uc5f0\uc801, \uc778\uacf5\uc801 \uc18c\uc74c\ub4e4\uc744 \ubaa8\ubc29\ud55c\ub2e4. \ub178\ub798\uac00 \ub192\uc544\uc9c8 \uc218\ub85d \ubaa8\ubc29\ud55c \uc18c\ub9ac\uc758 \uad6c\uc870\ub294 \ud3c9\uc774\ud574\uc9c4\ub2e4. \uadf8 \ub2e4\uc74c \ub2e8\uacc4\uc5d0\uc11c\ub294 \ud55c\ub3d9\uc548 \uac19\uc740 \uc18c\ub9ac\ub97c \ubc18\ubcf5\ud558\uace0, \uadf8\ub9ac\uace0 \ub2e4\uc74c \ub2e8\uacc4\ub85c \ub118\uc5b4\uac04\ub2e4. \uc774 \ubcc0\uc8fc \ub4a4\uc5d0\ub294 \uc9e4\uae4d\uac70\ub9ac\ub294 \ud761\ucc29\uc74c\uc744 \uc5ec\ub7ec \ubc88 \ub0b4\uace0 \ub098\uc11c, \ub9c8\uc9c0\ub9c9\uc73c\ub85c \ub9e4\uc6b0 \ub192\uc740 \ub178\ub798\ub97c, \ub2e4\uc2dc \uc5ec\ub7ec \ud615\ud0dc\ub85c \ubcc0\uc8fc\ud558\uba74\uc11c \ubd88\ub7ec\ub304\ub2e4. \uac01\uac01\uc758 \uac1c\uccb4\ub294 \uc790\uae30\ub9cc\uc758 \ub808\ud37c\ud1a0\ub9ac\ub97c \uac16\uace0 \uc788\uc73c\uba70, \ub178\ub798\uc5d0 \ud2b9\ud788 \ub2a5\uc219\ud55c \uac1c\uccb4\ub4e4\uc740 35\uac1c \uc774\ud558\uc758 \ub178\ub7ab\uc18c\ub9ac\uc640 14\uac1c \uc774\ud558\uc758 \ud761\ucc29\uc74c\uc744 \ub0bc \uc218 \uc788\ub2e4.", "sentence_answer": "\uc774\ub7ec\ud55c \ub178\ub798\ub294 \ubcf4\ud1b5 \ub124 \uac00\uc9c0 \uc758 \ubcc0\uc885\uc774 \uc788\ub294\ub370, \uc815\ud574\uc9c4 \uc21c\uc11c\ub300\ub85c \uc911\uac04\uc5d0 \uc27c\uc5c6\uc774 \ub178\ub798\ub97c \uacc4\uc18d\ud55c\ub2e4.", "paragraph_id": "6464640-6-2", "paragraph_question": "question: \uc720\ub7fd\ucc0c\ub974\ub808\uae30\uc758 \ub178\ub798\uc5d0\ub294 \ubcf4\ud1b5 \uba87\uac00\uc9c0\uc758 \ubcc0\uc885\uc774 \uc788\ub294\uac00?, context: \uc720\ub7fd\ucc0c\ub974\ub808\uae30\ub294 \uc2dc\ub044\ub7ec\uc6b4 \uc0c8\ub2e4. \uadf8 \ub178\ub7ab\uc18c\ub9ac\ub294 \ub2e4\uc18c \uc74c\uc545\uc801\uc778 \uac83\uacfc \uae30\uacc4\uc9c8\uc758 \uc18c\uc74c\uc774 \ud568\uaed8 \uc11e\uc5ec \uc788\ub2e4. \uc8fc\ub85c \ub178\ub798\ub97c \ubd80\ub974\ub294 \uac83\uc740 \uc218\ucef7\uc73c\ub85c, \ud55c\ubc88 \uc2dc\uc791\ud558\uba74 1\ubd84 \uc774\uc0c1 \ub178\ub798\ub97c \ud55c\ubc14\ud0d5 \uc9c0\uc18d\ud55c\ub2e4. \uc774\ub7ec\ud55c \ub178\ub798\ub294 \ubcf4\ud1b5 \ub124 \uac00\uc9c0\uc758 \ubcc0\uc885\uc774 \uc788\ub294\ub370, \uc815\ud574\uc9c4 \uc21c\uc11c\ub300\ub85c \uc911\uac04\uc5d0 \uc27c\uc5c6\uc774 \ub178\ub798\ub97c \uacc4\uc18d\ud55c\ub2e4. \uc6b0\uc120 \uc21c\uc74c\uc73c\ub85c \uc774\ub8e8\uc5b4\uc9c4 \ud718\ud30c\ub78c\uc744 \uba87 \ubc88 \ubd88\uba74\uc11c \ub178\ub798\uac00 \uc2dc\uc791\ub41c\ub2e4. \uadf8 \ub4a4 \uc8fc \ubd80\ubd84\uc5d0\uc11c\ub294 \uc8fc\ub85c \ub2e4\ub978 \uc885\uc758 \uc0c8\uc18c\ub9ac\ub098 \uac01\uc885 \uc790\uc5f0\uc801, \uc778\uacf5\uc801 \uc18c\uc74c\ub4e4\uc744 \ubaa8\ubc29\ud55c\ub2e4. \ub178\ub798\uac00 \ub192\uc544\uc9c8 \uc218\ub85d \ubaa8\ubc29\ud55c \uc18c\ub9ac\uc758 \uad6c\uc870\ub294 \ud3c9\uc774\ud574\uc9c4\ub2e4. \uadf8 \ub2e4\uc74c \ub2e8\uacc4\uc5d0\uc11c\ub294 \ud55c\ub3d9\uc548 \uac19\uc740 \uc18c\ub9ac\ub97c \ubc18\ubcf5\ud558\uace0, \uadf8\ub9ac\uace0 \ub2e4\uc74c \ub2e8\uacc4\ub85c \ub118\uc5b4\uac04\ub2e4. \uc774 \ubcc0\uc8fc \ub4a4\uc5d0\ub294 \uc9e4\uae4d\uac70\ub9ac\ub294 \ud761\ucc29\uc74c\uc744 \uc5ec\ub7ec \ubc88 \ub0b4\uace0 \ub098\uc11c, \ub9c8\uc9c0\ub9c9\uc73c\ub85c \ub9e4\uc6b0 \ub192\uc740 \ub178\ub798\ub97c, \ub2e4\uc2dc \uc5ec\ub7ec \ud615\ud0dc\ub85c \ubcc0\uc8fc\ud558\uba74\uc11c \ubd88\ub7ec\ub304\ub2e4. \uac01\uac01\uc758 \uac1c\uccb4\ub294 \uc790\uae30\ub9cc\uc758 \ub808\ud37c\ud1a0\ub9ac\ub97c \uac16\uace0 \uc788\uc73c\uba70, \ub178\ub798\uc5d0 \ud2b9\ud788 \ub2a5\uc219\ud55c \uac1c\uccb4\ub4e4\uc740 35\uac1c \uc774\ud558\uc758 \ub178\ub7ab\uc18c\ub9ac\uc640 14\uac1c \uc774\ud558\uc758 \ud761\ucc29\uc74c\uc744 \ub0bc \uc218 \uc788\ub2e4."} {"question": "\ub85c\ub9c8 \uc2dc\uc5d0\ub294 \uc5b4\ub5a4 \uac74\ucd95 \uae30\uc220\uc744 \ud65c\uc6a9\ud55c \ud070 \uac74\ubb3c\uc774 \ub9ce\uc558\ub294\uac00?", "paragraph": "\uc77c\uacf1 \uc5b8\ub355\uc774 \uc790\ub9ac\uc7a1\uc740 \ub85c\ub9c8 \uc2dc\ub294 \uace0\ub300 \ub85c\ub9c8\uc758 \uc911\uc2ec\uc9c0\uc600\ub2e4. \uc774 \ub3c4\uc2dc\uc5d0\ub294 \ucf5c\ub85c\uc138\uc6c0, \ud2b8\ub77c\uc57c\ub204\uc2a4 \ud3ec\ub8f8, \ud310\ud14c\uc628 \ub4f1 \uc544\uce58\ub97c \ud65c\uc6a9\ud55c \uc7a5\ub300\ud55c \uac74\ubb3c\uc774 \ub9ce\uc558\ub2e4. \ubd84\uc218\uc5d0\uc11c\ub294 \uc218\ubc31 \ub9c8\uc77c \uae38\uc774\uc758 \uc218\ub3c4\uad00\uc744 \ud0c0\uace0 \uc6b4\ubc18\ub418\uc5b4 \uc628 \uc2e0\uc120\ud55c \uc2dd\uc218\uac00 \uc19f\uc558\uc73c\uba70, \uadf9\uc7a5, \uccb4\uc721\uad00, \uadf8\ub9ac\uace0 \ub3c4\uc11c\uad00\uacfc \uac00\uac8c, \uc2dc\uc7a5, \uc0c1\ud558\uc218\ub3c4\ub97c \uac16\ucd98 \ub85c\ub9c8\uc2dd \ubaa9\uc695\ud0d5\ub3c4 \uc788\uc5c8\ub2e4. \uace0\ub300 \ub85c\ub9c8\uc758 \uc9c0\ubc30 \uc601\uc5ed \ub110\ub9ac \uc804\uc6d0 \ubcc4\uc7a5(villa)\uc640 \uac19\uc740 \uac00\uc625 \uad6c\uc870\ub97c \ubcfc \uc218 \uc788\uc5c8\ub2e4. \uc218\ub3c4 \ub85c\ub9c8 \uc2dc\uc5d0\ub294 \ud314\ub77c\ud2f0\ub204\uc2a4 \uc5b8\ub355\uc758 \uad81\uad90\uc774 \uc788\uc5c8\uc73c\uba70, \ud558\uce35 \ud3c9\ubbfc\uc774\ub098 \uc911\ub958 \uae30\uc0ac \uacc4\uae09\uc740 \ub3c4\uc2ec\uc9c0\uc758 \uc544\ud30c\ud2b8(insulae)\uc5d0 \uc0b4\uc558\ub2e4. \uc774\ub7f0 \uacf3\uc740 \uc0c1\ub958\uce35 \uc9c0\uc8fc\ub4e4\uc774 \uc784\ub300\ub8cc \uc218\uc785\uc744 \uc5bb\uae30 \uc704\ud574 \uc9c0\uc740 \uacf3\uc774\uae30\ub3c4 \ud588\ub294\ub370, \ucf5c\ub808\uae30\uc6c0(collegium)\uc774\ub098 \uc120\uc220\uc9d1(taberna)\uc774 \uc0dd\ud65c\uc758 \uc911\uc2ec\uc774\uc5c8\ub2e4. \uc774 \uacf3\uc5d0 \uc0ac\ub294 \uc0ac\ub78c\ub4e4\uc740 \ubb34\ub8cc \uace1\ubb3c \ubc30\uae09\uc744 \ubc1b\uc73c\uba70, \uac80\ud22c\uc0ac \uacbd\uae30\ub97c \uc990\uacbc\uc73c\uba70, \uc0c1\ub958 \uadc0\uc871\uc758 \ud53c\ud638\ubbfc\uc73c\ub85c \ud544\uc694\ud560 \ub54c \ub3c4\uc640\uc8fc\uace0 \uc9c0\ucf1c\uc8fc\ub294 \uad00\uacc4\ub97c \ub9fa\uae30\ub3c4 \ud558\uc600\ub2e4.", "answer": "\uc544\uce58", "sentence": "\uc774 \ub3c4\uc2dc\uc5d0\ub294 \ucf5c\ub85c\uc138\uc6c0, \ud2b8\ub77c\uc57c\ub204\uc2a4 \ud3ec\ub8f8, \ud310\ud14c\uc628 \ub4f1 \uc544\uce58 \ub97c \ud65c\uc6a9\ud55c \uc7a5\ub300\ud55c \uac74\ubb3c\uc774 \ub9ce\uc558\ub2e4.", "paragraph_sentence": "\uc77c\uacf1 \uc5b8\ub355\uc774 \uc790\ub9ac\uc7a1\uc740 \ub85c\ub9c8 \uc2dc\ub294 \uace0\ub300 \ub85c\ub9c8\uc758 \uc911\uc2ec\uc9c0\uc600\ub2e4. \uc774 \ub3c4\uc2dc\uc5d0\ub294 \ucf5c\ub85c\uc138\uc6c0, \ud2b8\ub77c\uc57c\ub204\uc2a4 \ud3ec\ub8f8, \ud310\ud14c\uc628 \ub4f1 \uc544\uce58 \ub97c \ud65c\uc6a9\ud55c \uc7a5\ub300\ud55c \uac74\ubb3c\uc774 \ub9ce\uc558\ub2e4. \ubd84\uc218\uc5d0\uc11c\ub294 \uc218\ubc31 \ub9c8\uc77c \uae38\uc774\uc758 \uc218\ub3c4\uad00\uc744 \ud0c0\uace0 \uc6b4\ubc18\ub418\uc5b4 \uc628 \uc2e0\uc120\ud55c \uc2dd\uc218\uac00 \uc19f\uc558\uc73c\uba70, \uadf9\uc7a5, \uccb4\uc721\uad00, \uadf8\ub9ac\uace0 \ub3c4\uc11c\uad00\uacfc \uac00\uac8c, \uc2dc\uc7a5, \uc0c1\ud558\uc218\ub3c4\ub97c \uac16\ucd98 \ub85c\ub9c8\uc2dd \ubaa9\uc695\ud0d5\ub3c4 \uc788\uc5c8\ub2e4. \uace0\ub300 \ub85c\ub9c8\uc758 \uc9c0\ubc30 \uc601\uc5ed \ub110\ub9ac \uc804\uc6d0 \ubcc4\uc7a5(villa)\uc640 \uac19\uc740 \uac00\uc625 \uad6c\uc870\ub97c \ubcfc \uc218 \uc788\uc5c8\ub2e4. \uc218\ub3c4 \ub85c\ub9c8 \uc2dc\uc5d0\ub294 \ud314\ub77c\ud2f0\ub204\uc2a4 \uc5b8\ub355\uc758 \uad81\uad90\uc774 \uc788\uc5c8\uc73c\uba70, \ud558\uce35 \ud3c9\ubbfc\uc774\ub098 \uc911\ub958 \uae30\uc0ac \uacc4\uae09\uc740 \ub3c4\uc2ec\uc9c0\uc758 \uc544\ud30c\ud2b8(insulae)\uc5d0 \uc0b4\uc558\ub2e4. \uc774\ub7f0 \uacf3\uc740 \uc0c1\ub958\uce35 \uc9c0\uc8fc\ub4e4\uc774 \uc784\ub300\ub8cc \uc218\uc785\uc744 \uc5bb\uae30 \uc704\ud574 \uc9c0\uc740 \uacf3\uc774\uae30\ub3c4 \ud588\ub294\ub370, \ucf5c\ub808\uae30\uc6c0(collegium)\uc774\ub098 \uc120\uc220\uc9d1(taberna)\uc774 \uc0dd\ud65c\uc758 \uc911\uc2ec\uc774\uc5c8\ub2e4. \uc774 \uacf3\uc5d0 \uc0ac\ub294 \uc0ac\ub78c\ub4e4\uc740 \ubb34\ub8cc \uace1\ubb3c \ubc30\uae09\uc744 \ubc1b\uc73c\uba70, \uac80\ud22c\uc0ac \uacbd\uae30\ub97c \uc990\uacbc\uc73c\uba70, \uc0c1\ub958 \uadc0\uc871\uc758 \ud53c\ud638\ubbfc\uc73c\ub85c \ud544\uc694\ud560 \ub54c \ub3c4\uc640\uc8fc\uace0 \uc9c0\ucf1c\uc8fc\ub294 \uad00\uacc4\ub97c \ub9fa\uae30\ub3c4 \ud558\uc600\ub2e4.", "paragraph_answer": "\uc77c\uacf1 \uc5b8\ub355\uc774 \uc790\ub9ac\uc7a1\uc740 \ub85c\ub9c8 \uc2dc\ub294 \uace0\ub300 \ub85c\ub9c8\uc758 \uc911\uc2ec\uc9c0\uc600\ub2e4. \uc774 \ub3c4\uc2dc\uc5d0\ub294 \ucf5c\ub85c\uc138\uc6c0, \ud2b8\ub77c\uc57c\ub204\uc2a4 \ud3ec\ub8f8, \ud310\ud14c\uc628 \ub4f1 \uc544\uce58 \ub97c \ud65c\uc6a9\ud55c \uc7a5\ub300\ud55c \uac74\ubb3c\uc774 \ub9ce\uc558\ub2e4. \ubd84\uc218\uc5d0\uc11c\ub294 \uc218\ubc31 \ub9c8\uc77c \uae38\uc774\uc758 \uc218\ub3c4\uad00\uc744 \ud0c0\uace0 \uc6b4\ubc18\ub418\uc5b4 \uc628 \uc2e0\uc120\ud55c \uc2dd\uc218\uac00 \uc19f\uc558\uc73c\uba70, \uadf9\uc7a5, \uccb4\uc721\uad00, \uadf8\ub9ac\uace0 \ub3c4\uc11c\uad00\uacfc \uac00\uac8c, \uc2dc\uc7a5, \uc0c1\ud558\uc218\ub3c4\ub97c \uac16\ucd98 \ub85c\ub9c8\uc2dd \ubaa9\uc695\ud0d5\ub3c4 \uc788\uc5c8\ub2e4. \uace0\ub300 \ub85c\ub9c8\uc758 \uc9c0\ubc30 \uc601\uc5ed \ub110\ub9ac \uc804\uc6d0 \ubcc4\uc7a5(villa)\uc640 \uac19\uc740 \uac00\uc625 \uad6c\uc870\ub97c \ubcfc \uc218 \uc788\uc5c8\ub2e4. \uc218\ub3c4 \ub85c\ub9c8 \uc2dc\uc5d0\ub294 \ud314\ub77c\ud2f0\ub204\uc2a4 \uc5b8\ub355\uc758 \uad81\uad90\uc774 \uc788\uc5c8\uc73c\uba70, \ud558\uce35 \ud3c9\ubbfc\uc774\ub098 \uc911\ub958 \uae30\uc0ac \uacc4\uae09\uc740 \ub3c4\uc2ec\uc9c0\uc758 \uc544\ud30c\ud2b8(insulae)\uc5d0 \uc0b4\uc558\ub2e4. \uc774\ub7f0 \uacf3\uc740 \uc0c1\ub958\uce35 \uc9c0\uc8fc\ub4e4\uc774 \uc784\ub300\ub8cc \uc218\uc785\uc744 \uc5bb\uae30 \uc704\ud574 \uc9c0\uc740 \uacf3\uc774\uae30\ub3c4 \ud588\ub294\ub370, \ucf5c\ub808\uae30\uc6c0(collegium)\uc774\ub098 \uc120\uc220\uc9d1(taberna)\uc774 \uc0dd\ud65c\uc758 \uc911\uc2ec\uc774\uc5c8\ub2e4. \uc774 \uacf3\uc5d0 \uc0ac\ub294 \uc0ac\ub78c\ub4e4\uc740 \ubb34\ub8cc \uace1\ubb3c \ubc30\uae09\uc744 \ubc1b\uc73c\uba70, \uac80\ud22c\uc0ac \uacbd\uae30\ub97c \uc990\uacbc\uc73c\uba70, \uc0c1\ub958 \uadc0\uc871\uc758 \ud53c\ud638\ubbfc\uc73c\ub85c \ud544\uc694\ud560 \ub54c \ub3c4\uc640\uc8fc\uace0 \uc9c0\ucf1c\uc8fc\ub294 \uad00\uacc4\ub97c \ub9fa\uae30\ub3c4 \ud558\uc600\ub2e4.", "sentence_answer": "\uc774 \ub3c4\uc2dc\uc5d0\ub294 \ucf5c\ub85c\uc138\uc6c0, \ud2b8\ub77c\uc57c\ub204\uc2a4 \ud3ec\ub8f8, \ud310\ud14c\uc628 \ub4f1 \uc544\uce58 \ub97c \ud65c\uc6a9\ud55c \uc7a5\ub300\ud55c \uac74\ubb3c\uc774 \ub9ce\uc558\ub2e4.", "paragraph_id": "6466082-16-1", "paragraph_question": "question: \ub85c\ub9c8 \uc2dc\uc5d0\ub294 \uc5b4\ub5a4 \uac74\ucd95 \uae30\uc220\uc744 \ud65c\uc6a9\ud55c \ud070 \uac74\ubb3c\uc774 \ub9ce\uc558\ub294\uac00?, context: \uc77c\uacf1 \uc5b8\ub355\uc774 \uc790\ub9ac\uc7a1\uc740 \ub85c\ub9c8 \uc2dc\ub294 \uace0\ub300 \ub85c\ub9c8\uc758 \uc911\uc2ec\uc9c0\uc600\ub2e4. \uc774 \ub3c4\uc2dc\uc5d0\ub294 \ucf5c\ub85c\uc138\uc6c0, \ud2b8\ub77c\uc57c\ub204\uc2a4 \ud3ec\ub8f8, \ud310\ud14c\uc628 \ub4f1 \uc544\uce58\ub97c \ud65c\uc6a9\ud55c \uc7a5\ub300\ud55c \uac74\ubb3c\uc774 \ub9ce\uc558\ub2e4. \ubd84\uc218\uc5d0\uc11c\ub294 \uc218\ubc31 \ub9c8\uc77c \uae38\uc774\uc758 \uc218\ub3c4\uad00\uc744 \ud0c0\uace0 \uc6b4\ubc18\ub418\uc5b4 \uc628 \uc2e0\uc120\ud55c \uc2dd\uc218\uac00 \uc19f\uc558\uc73c\uba70, \uadf9\uc7a5, \uccb4\uc721\uad00, \uadf8\ub9ac\uace0 \ub3c4\uc11c\uad00\uacfc \uac00\uac8c, \uc2dc\uc7a5, \uc0c1\ud558\uc218\ub3c4\ub97c \uac16\ucd98 \ub85c\ub9c8\uc2dd \ubaa9\uc695\ud0d5\ub3c4 \uc788\uc5c8\ub2e4. \uace0\ub300 \ub85c\ub9c8\uc758 \uc9c0\ubc30 \uc601\uc5ed \ub110\ub9ac \uc804\uc6d0 \ubcc4\uc7a5(villa)\uc640 \uac19\uc740 \uac00\uc625 \uad6c\uc870\ub97c \ubcfc \uc218 \uc788\uc5c8\ub2e4. \uc218\ub3c4 \ub85c\ub9c8 \uc2dc\uc5d0\ub294 \ud314\ub77c\ud2f0\ub204\uc2a4 \uc5b8\ub355\uc758 \uad81\uad90\uc774 \uc788\uc5c8\uc73c\uba70, \ud558\uce35 \ud3c9\ubbfc\uc774\ub098 \uc911\ub958 \uae30\uc0ac \uacc4\uae09\uc740 \ub3c4\uc2ec\uc9c0\uc758 \uc544\ud30c\ud2b8(insulae)\uc5d0 \uc0b4\uc558\ub2e4. \uc774\ub7f0 \uacf3\uc740 \uc0c1\ub958\uce35 \uc9c0\uc8fc\ub4e4\uc774 \uc784\ub300\ub8cc \uc218\uc785\uc744 \uc5bb\uae30 \uc704\ud574 \uc9c0\uc740 \uacf3\uc774\uae30\ub3c4 \ud588\ub294\ub370, \ucf5c\ub808\uae30\uc6c0(collegium)\uc774\ub098 \uc120\uc220\uc9d1(taberna)\uc774 \uc0dd\ud65c\uc758 \uc911\uc2ec\uc774\uc5c8\ub2e4. \uc774 \uacf3\uc5d0 \uc0ac\ub294 \uc0ac\ub78c\ub4e4\uc740 \ubb34\ub8cc \uace1\ubb3c \ubc30\uae09\uc744 \ubc1b\uc73c\uba70, \uac80\ud22c\uc0ac \uacbd\uae30\ub97c \uc990\uacbc\uc73c\uba70, \uc0c1\ub958 \uadc0\uc871\uc758 \ud53c\ud638\ubbfc\uc73c\ub85c \ud544\uc694\ud560 \ub54c \ub3c4\uc640\uc8fc\uace0 \uc9c0\ucf1c\uc8fc\ub294 \uad00\uacc4\ub97c \ub9fa\uae30\ub3c4 \ud558\uc600\ub2e4."} {"question": "\uc774\ub514 \uc544\ubbfc\uc774 \uc6d0\ub798 \uc6b0\uac04\ub2e4 \ub545\uc774\ub77c\uba70 \ud569\ubcd1\uc744 \uc704\ud574 \uce68\uacf5\ud55c \ud0c4\uc790\ub2c8\uc544\uc758 \uc9c0\ubc29\uc740?", "paragraph": "\uc774\uac83\uc740 \uc544\ubbfc\ud30c\uc758 \uae09\uaca9\ud55c \uc1e0\ud1f4\ub97c \ubd88\ub7ec\uc640, \uc6b0\uac04\ub2e4 \ub0b4\ubd80\uc5d0\uc11c \ub0b4\ubd84\uc774 \uc99d\uac00\ud558\uae30 \uc2dc\uc791\ud588\ub2e4. \uc544\ubbfc\uc758 \ubd80\ud1b5\ub839\uc774\ub358 \ubb34\uc2a4\ud0c0\ud30c \uc544\ub4dc\ub9ac\uc2dc\uac00 \uc218\uc0c1\ud55c \uad50\ud1b5\uc0ac\uace0\ub85c \ubd80\uc0c1\uc744 \ub2f9\ud588\uc744 \ub54c, \uc544\ub4dc\ub9ac\uc2dc\uc5d0\uac8c \ucda9\uc131\uc744 \ub9f9\uc138\ud588\ub358 \ubcd1\uc0ac(\uadf8 \uc678 \ub2e4\ub978 \uc774\uc720\ub85c \ubd88\ub9cc\uc744 \uac00\uc9c4 \ubcd1\uc0ac)\uac00 \ud3ed\ub3d9\uc744 \uc77c\uc73c\ucf30\ub2e4. \uc774\ub514 \uc544\ubbfc\uc740 \ud3ed\ub3d9\uc744 \uc9c4\uc555\ud558\uae30 \uc704\ud574 \uc2ec\ubc14 \uc815\uc608\ubd80\ub300\ub97c \ud3ec\ud568\ud55c \ubcd1\uc0ac\ub97c \ubcf4\ub0b4 \uc77c\ubd80\uac00 \ud0c4\uc790\ub2c8\uc544 \uad6d\uacbd\uc744 \ub118\uc5c8\ub2e4. \ubc18\ub780\uad70\uc774 \ud0c4\uc790\ub2c8\uc544\ub85c \ub118\uc5b4\uac00\uc790 \ud0c4\uc790\ub2c8\uc544\uc5d0 \uc788\ub358 \ubc18\uc544\ubbfc \ud53c\ub09c\ubbfc\ub4e4\uc740 \ubc18\ub780\uad70\uc5d0 \ucc38\uc5ec\ud558\uc5ec \uc774\ub514 \uc544\ubbfc\uad70\uacfc\uc758 \uc804\ud22c\uc5d0 \ucc38\uac00\ud588\ub2e4. \uc6b0\uac04\ub2e4\ub294 \ud0c4\uc790\ub2c8\uc544\uc5d0 \uc120\uc804\uc744 \ud3ec\uace0\ud588\ub294\ub370 \uc774\ub514 \uc544\ubbfc\uc740 \ud0c4\uc790\ub2c8\uc544\uc758 \uce74\uac8c\ub77c \uc9c0\ubc29\uc774 \uc6d0\ub798 \uc6b0\uac04\ub2e4 \ub545\uc774\ub77c\uba70 \ud569\ubcd1\uc744 \uc704\ud574 \ud0c4\uc790\ub2c8\uc544\ub97c\uce68\uacf5\ud588\ub2e4. \uadf8\ub7ec\ub098 \ub2c8\uc5d0\ub808\ub808 \ub300\ud1b5\ub839\uc740 \ud0c4\uc790\ub2c8\uc544 \uad6d\ubbfc\ubc29\uc5b4\uad70\uc744 \ub3d9\uc6d0\ud558\uc5ec \ubc18\uaca9\uc5d0 \ub098\uc130\ub2e4. \ud0c4\uc790\ub2c8\uc544\uad70\uc740 \uba87 \uc8fc\ub9cc\uc5d0 \uacbd\ucc30\uacfc \uc885\uad70\uc0ac\uba74\uc8c4\uc218, \uad6d\ubbfc\ubcd1, \uad70\uc778\uc744 \ud3ec\ud568\ud574 10\ub9cc\uba85\uc73c\ub85c \uc99d\uac00\ud588\ub2e4. \uc6b0\uac04\ub2e4 \ud53c\ub09c\ubbfc\ub4e4\ub85c \uad6c\uc131\ub41c \ubc18 \uc544\ubbfc\ub2e8\uccb4\ub3c4 \ud0c4\uc790\ub2c8\uc544\uc5d0 \uac00\uc138\ud588\ub2e4. \uc774\ub4e4\uc740 \ubaa8\uc2dc \ud68c\uc758\uc5d0 \uc18c\uc18d\ud558\uc5ec \ubaa8\uc2dc \ud68c\uc758\ub294 \uc6b0\uac04\ub2e4 \uad6d\ubbfc\ud574\ubc29\uad70(UNLA)\uc744 \ud615\uc131\ud588\ub2e4. \ud2f0\ud1a0 \uc624\ucf08\ub85c\uc640 \ub370\uc774\ube57 \uc624\uc774\ud14c \uc624\uc870\ud06c\uac00 \uc774\ub044\ub294 \ud0a4\ucf54\uc2dc \ub9d0\ub984(Kikosi Maalum), \uc694\uc6e8\ub808 \ubb34\uc138\ubca0\ub2c8\uac00 \uc774\ub044\ub294 FRONASA, \uc544\ucf00\ub098 \ud3ec\uc871\uacfc \uc70c\ub9ac\uc5c4 \uc624\ub9c8\ub9ac\uc544, \uc544\ud14c\ucf08 \uc5d0\uc798\ub8e8\uac00 \uc774\ub044\ub294 <\uc6b0\uac04\ub2e4 \uad6c\uc6d0\uc6b4\ub3d9>\ub3c4 UNLA\uc5d0 \ucc38\uac00\ud588\ub2e4. \ud0c4\uc790\ub2c8\uc544\uad70\uc740 \uc18c\ub828\uc758 \uce74\ud29c\uc0e4\uc640 \ub85c\ucf13\ud3ec\ub97c \ud68d\ub4dd\ud574 \uc6b0\uac04\ub2e4\uad70\uc744 \ud45c\uc801\uc73c\ub85c \ud3ec\uaca9\uc744 \uac1c\uc2dc\ud588\ub2e4.", "answer": "\uce74\uac8c\ub77c \uc9c0\ubc29", "sentence": "\uc6b0\uac04\ub2e4\ub294 \ud0c4\uc790\ub2c8\uc544\uc5d0 \uc120\uc804\uc744 \ud3ec\uace0\ud588\ub294\ub370 \uc774\ub514 \uc544\ubbfc\uc740 \ud0c4\uc790\ub2c8\uc544\uc758 \uce74\uac8c\ub77c \uc9c0\ubc29 \uc774 \uc6d0\ub798 \uc6b0\uac04\ub2e4 \ub545\uc774\ub77c\uba70 \ud569\ubcd1\uc744 \uc704\ud574 \ud0c4\uc790\ub2c8\uc544\ub97c\uce68\uacf5\ud588\ub2e4.", "paragraph_sentence": "\uc774\uac83\uc740 \uc544\ubbfc\ud30c\uc758 \uae09\uaca9\ud55c \uc1e0\ud1f4\ub97c \ubd88\ub7ec\uc640, \uc6b0\uac04\ub2e4 \ub0b4\ubd80\uc5d0\uc11c \ub0b4\ubd84\uc774 \uc99d\uac00\ud558\uae30 \uc2dc\uc791\ud588\ub2e4. \uc544\ubbfc\uc758 \ubd80\ud1b5\ub839\uc774\ub358 \ubb34\uc2a4\ud0c0\ud30c \uc544\ub4dc\ub9ac\uc2dc\uac00 \uc218\uc0c1\ud55c \uad50\ud1b5\uc0ac\uace0\ub85c \ubd80\uc0c1\uc744 \ub2f9\ud588\uc744 \ub54c, \uc544\ub4dc\ub9ac\uc2dc\uc5d0\uac8c \ucda9\uc131\uc744 \ub9f9\uc138\ud588\ub358 \ubcd1\uc0ac(\uadf8 \uc678 \ub2e4\ub978 \uc774\uc720\ub85c \ubd88\ub9cc\uc744 \uac00\uc9c4 \ubcd1\uc0ac)\uac00 \ud3ed\ub3d9\uc744 \uc77c\uc73c\ucf30\ub2e4. \uc774\ub514 \uc544\ubbfc\uc740 \ud3ed\ub3d9\uc744 \uc9c4\uc555\ud558\uae30 \uc704\ud574 \uc2ec\ubc14 \uc815\uc608\ubd80\ub300\ub97c \ud3ec\ud568\ud55c \ubcd1\uc0ac\ub97c \ubcf4\ub0b4 \uc77c\ubd80\uac00 \ud0c4\uc790\ub2c8\uc544 \uad6d\uacbd\uc744 \ub118\uc5c8\ub2e4. \ubc18\ub780\uad70\uc774 \ud0c4\uc790\ub2c8\uc544\ub85c \ub118\uc5b4\uac00\uc790 \ud0c4\uc790\ub2c8\uc544\uc5d0 \uc788\ub358 \ubc18\uc544\ubbfc \ud53c\ub09c\ubbfc\ub4e4\uc740 \ubc18\ub780\uad70\uc5d0 \ucc38\uc5ec\ud558\uc5ec \uc774\ub514 \uc544\ubbfc\uad70\uacfc\uc758 \uc804\ud22c\uc5d0 \ucc38\uac00\ud588\ub2e4. \uc6b0\uac04\ub2e4\ub294 \ud0c4\uc790\ub2c8\uc544\uc5d0 \uc120\uc804\uc744 \ud3ec\uace0\ud588\ub294\ub370 \uc774\ub514 \uc544\ubbfc\uc740 \ud0c4\uc790\ub2c8\uc544\uc758 \uce74\uac8c\ub77c \uc9c0\ubc29 \uc774 \uc6d0\ub798 \uc6b0\uac04\ub2e4 \ub545\uc774\ub77c\uba70 \ud569\ubcd1\uc744 \uc704\ud574 \ud0c4\uc790\ub2c8\uc544\ub97c\uce68\uacf5\ud588\ub2e4. \uadf8\ub7ec\ub098 \ub2c8\uc5d0\ub808\ub808 \ub300\ud1b5\ub839\uc740 \ud0c4\uc790\ub2c8\uc544 \uad6d\ubbfc\ubc29\uc5b4\uad70\uc744 \ub3d9\uc6d0\ud558\uc5ec \ubc18\uaca9\uc5d0 \ub098\uc130\ub2e4. \ud0c4\uc790\ub2c8\uc544\uad70\uc740 \uba87 \uc8fc\ub9cc\uc5d0 \uacbd\ucc30\uacfc \uc885\uad70\uc0ac\uba74\uc8c4\uc218, \uad6d\ubbfc\ubcd1, \uad70\uc778\uc744 \ud3ec\ud568\ud574 10\ub9cc\uba85\uc73c\ub85c \uc99d\uac00\ud588\ub2e4. \uc6b0\uac04\ub2e4 \ud53c\ub09c\ubbfc\ub4e4\ub85c \uad6c\uc131\ub41c \ubc18 \uc544\ubbfc\ub2e8\uccb4\ub3c4 \ud0c4\uc790\ub2c8\uc544\uc5d0 \uac00\uc138\ud588\ub2e4. \uc774\ub4e4\uc740 \ubaa8\uc2dc \ud68c\uc758\uc5d0 \uc18c\uc18d\ud558\uc5ec \ubaa8\uc2dc \ud68c\uc758\ub294 \uc6b0\uac04\ub2e4 \uad6d\ubbfc\ud574\ubc29\uad70(UNLA)\uc744 \ud615\uc131\ud588\ub2e4. \ud2f0\ud1a0 \uc624\ucf08\ub85c\uc640 \ub370\uc774\ube57 \uc624\uc774\ud14c \uc624\uc870\ud06c\uac00 \uc774\ub044\ub294 \ud0a4\ucf54\uc2dc \ub9d0\ub984(Kikosi Maalum), \uc694\uc6e8\ub808 \ubb34\uc138\ubca0\ub2c8\uac00 \uc774\ub044\ub294 FRONASA, \uc544\ucf00\ub098 \ud3ec\uc871\uacfc \uc70c\ub9ac\uc5c4 \uc624\ub9c8\ub9ac\uc544, \uc544\ud14c\ucf08 \uc5d0\uc798\ub8e8\uac00 \uc774\ub044\ub294 <\uc6b0\uac04\ub2e4 \uad6c\uc6d0\uc6b4\ub3d9>\ub3c4 UNLA\uc5d0 \ucc38\uac00\ud588\ub2e4. \ud0c4\uc790\ub2c8\uc544\uad70\uc740 \uc18c\ub828\uc758 \uce74\ud29c\uc0e4\uc640 \ub85c\ucf13\ud3ec\ub97c \ud68d\ub4dd\ud574 \uc6b0\uac04\ub2e4\uad70\uc744 \ud45c\uc801\uc73c\ub85c \ud3ec\uaca9\uc744 \uac1c\uc2dc\ud588\ub2e4.", "paragraph_answer": "\uc774\uac83\uc740 \uc544\ubbfc\ud30c\uc758 \uae09\uaca9\ud55c \uc1e0\ud1f4\ub97c \ubd88\ub7ec\uc640, \uc6b0\uac04\ub2e4 \ub0b4\ubd80\uc5d0\uc11c \ub0b4\ubd84\uc774 \uc99d\uac00\ud558\uae30 \uc2dc\uc791\ud588\ub2e4. \uc544\ubbfc\uc758 \ubd80\ud1b5\ub839\uc774\ub358 \ubb34\uc2a4\ud0c0\ud30c \uc544\ub4dc\ub9ac\uc2dc\uac00 \uc218\uc0c1\ud55c \uad50\ud1b5\uc0ac\uace0\ub85c \ubd80\uc0c1\uc744 \ub2f9\ud588\uc744 \ub54c, \uc544\ub4dc\ub9ac\uc2dc\uc5d0\uac8c \ucda9\uc131\uc744 \ub9f9\uc138\ud588\ub358 \ubcd1\uc0ac(\uadf8 \uc678 \ub2e4\ub978 \uc774\uc720\ub85c \ubd88\ub9cc\uc744 \uac00\uc9c4 \ubcd1\uc0ac)\uac00 \ud3ed\ub3d9\uc744 \uc77c\uc73c\ucf30\ub2e4. \uc774\ub514 \uc544\ubbfc\uc740 \ud3ed\ub3d9\uc744 \uc9c4\uc555\ud558\uae30 \uc704\ud574 \uc2ec\ubc14 \uc815\uc608\ubd80\ub300\ub97c \ud3ec\ud568\ud55c \ubcd1\uc0ac\ub97c \ubcf4\ub0b4 \uc77c\ubd80\uac00 \ud0c4\uc790\ub2c8\uc544 \uad6d\uacbd\uc744 \ub118\uc5c8\ub2e4. \ubc18\ub780\uad70\uc774 \ud0c4\uc790\ub2c8\uc544\ub85c \ub118\uc5b4\uac00\uc790 \ud0c4\uc790\ub2c8\uc544\uc5d0 \uc788\ub358 \ubc18\uc544\ubbfc \ud53c\ub09c\ubbfc\ub4e4\uc740 \ubc18\ub780\uad70\uc5d0 \ucc38\uc5ec\ud558\uc5ec \uc774\ub514 \uc544\ubbfc\uad70\uacfc\uc758 \uc804\ud22c\uc5d0 \ucc38\uac00\ud588\ub2e4. \uc6b0\uac04\ub2e4\ub294 \ud0c4\uc790\ub2c8\uc544\uc5d0 \uc120\uc804\uc744 \ud3ec\uace0\ud588\ub294\ub370 \uc774\ub514 \uc544\ubbfc\uc740 \ud0c4\uc790\ub2c8\uc544\uc758 \uce74\uac8c\ub77c \uc9c0\ubc29 \uc774 \uc6d0\ub798 \uc6b0\uac04\ub2e4 \ub545\uc774\ub77c\uba70 \ud569\ubcd1\uc744 \uc704\ud574 \ud0c4\uc790\ub2c8\uc544\ub97c\uce68\uacf5\ud588\ub2e4. \uadf8\ub7ec\ub098 \ub2c8\uc5d0\ub808\ub808 \ub300\ud1b5\ub839\uc740 \ud0c4\uc790\ub2c8\uc544 \uad6d\ubbfc\ubc29\uc5b4\uad70\uc744 \ub3d9\uc6d0\ud558\uc5ec \ubc18\uaca9\uc5d0 \ub098\uc130\ub2e4. \ud0c4\uc790\ub2c8\uc544\uad70\uc740 \uba87 \uc8fc\ub9cc\uc5d0 \uacbd\ucc30\uacfc \uc885\uad70\uc0ac\uba74\uc8c4\uc218, \uad6d\ubbfc\ubcd1, \uad70\uc778\uc744 \ud3ec\ud568\ud574 10\ub9cc\uba85\uc73c\ub85c \uc99d\uac00\ud588\ub2e4. \uc6b0\uac04\ub2e4 \ud53c\ub09c\ubbfc\ub4e4\ub85c \uad6c\uc131\ub41c \ubc18 \uc544\ubbfc\ub2e8\uccb4\ub3c4 \ud0c4\uc790\ub2c8\uc544\uc5d0 \uac00\uc138\ud588\ub2e4. \uc774\ub4e4\uc740 \ubaa8\uc2dc \ud68c\uc758\uc5d0 \uc18c\uc18d\ud558\uc5ec \ubaa8\uc2dc \ud68c\uc758\ub294 \uc6b0\uac04\ub2e4 \uad6d\ubbfc\ud574\ubc29\uad70(UNLA)\uc744 \ud615\uc131\ud588\ub2e4. \ud2f0\ud1a0 \uc624\ucf08\ub85c\uc640 \ub370\uc774\ube57 \uc624\uc774\ud14c \uc624\uc870\ud06c\uac00 \uc774\ub044\ub294 \ud0a4\ucf54\uc2dc \ub9d0\ub984(Kikosi Maalum), \uc694\uc6e8\ub808 \ubb34\uc138\ubca0\ub2c8\uac00 \uc774\ub044\ub294 FRONASA, \uc544\ucf00\ub098 \ud3ec\uc871\uacfc \uc70c\ub9ac\uc5c4 \uc624\ub9c8\ub9ac\uc544, \uc544\ud14c\ucf08 \uc5d0\uc798\ub8e8\uac00 \uc774\ub044\ub294 <\uc6b0\uac04\ub2e4 \uad6c\uc6d0\uc6b4\ub3d9>\ub3c4 UNLA\uc5d0 \ucc38\uac00\ud588\ub2e4. \ud0c4\uc790\ub2c8\uc544\uad70\uc740 \uc18c\ub828\uc758 \uce74\ud29c\uc0e4\uc640 \ub85c\ucf13\ud3ec\ub97c \ud68d\ub4dd\ud574 \uc6b0\uac04\ub2e4\uad70\uc744 \ud45c\uc801\uc73c\ub85c \ud3ec\uaca9\uc744 \uac1c\uc2dc\ud588\ub2e4.", "sentence_answer": "\uc6b0\uac04\ub2e4\ub294 \ud0c4\uc790\ub2c8\uc544\uc5d0 \uc120\uc804\uc744 \ud3ec\uace0\ud588\ub294\ub370 \uc774\ub514 \uc544\ubbfc\uc740 \ud0c4\uc790\ub2c8\uc544\uc758 \uce74\uac8c\ub77c \uc9c0\ubc29 \uc774 \uc6d0\ub798 \uc6b0\uac04\ub2e4 \ub545\uc774\ub77c\uba70 \ud569\ubcd1\uc744 \uc704\ud574 \ud0c4\uc790\ub2c8\uc544\ub97c\uce68\uacf5\ud588\ub2e4.", "paragraph_id": "6526192-0-1", "paragraph_question": "question: \uc774\ub514 \uc544\ubbfc\uc774 \uc6d0\ub798 \uc6b0\uac04\ub2e4 \ub545\uc774\ub77c\uba70 \ud569\ubcd1\uc744 \uc704\ud574 \uce68\uacf5\ud55c \ud0c4\uc790\ub2c8\uc544\uc758 \uc9c0\ubc29\uc740?, context: \uc774\uac83\uc740 \uc544\ubbfc\ud30c\uc758 \uae09\uaca9\ud55c \uc1e0\ud1f4\ub97c \ubd88\ub7ec\uc640, \uc6b0\uac04\ub2e4 \ub0b4\ubd80\uc5d0\uc11c \ub0b4\ubd84\uc774 \uc99d\uac00\ud558\uae30 \uc2dc\uc791\ud588\ub2e4. \uc544\ubbfc\uc758 \ubd80\ud1b5\ub839\uc774\ub358 \ubb34\uc2a4\ud0c0\ud30c \uc544\ub4dc\ub9ac\uc2dc\uac00 \uc218\uc0c1\ud55c \uad50\ud1b5\uc0ac\uace0\ub85c \ubd80\uc0c1\uc744 \ub2f9\ud588\uc744 \ub54c, \uc544\ub4dc\ub9ac\uc2dc\uc5d0\uac8c \ucda9\uc131\uc744 \ub9f9\uc138\ud588\ub358 \ubcd1\uc0ac(\uadf8 \uc678 \ub2e4\ub978 \uc774\uc720\ub85c \ubd88\ub9cc\uc744 \uac00\uc9c4 \ubcd1\uc0ac)\uac00 \ud3ed\ub3d9\uc744 \uc77c\uc73c\ucf30\ub2e4. \uc774\ub514 \uc544\ubbfc\uc740 \ud3ed\ub3d9\uc744 \uc9c4\uc555\ud558\uae30 \uc704\ud574 \uc2ec\ubc14 \uc815\uc608\ubd80\ub300\ub97c \ud3ec\ud568\ud55c \ubcd1\uc0ac\ub97c \ubcf4\ub0b4 \uc77c\ubd80\uac00 \ud0c4\uc790\ub2c8\uc544 \uad6d\uacbd\uc744 \ub118\uc5c8\ub2e4. \ubc18\ub780\uad70\uc774 \ud0c4\uc790\ub2c8\uc544\ub85c \ub118\uc5b4\uac00\uc790 \ud0c4\uc790\ub2c8\uc544\uc5d0 \uc788\ub358 \ubc18\uc544\ubbfc \ud53c\ub09c\ubbfc\ub4e4\uc740 \ubc18\ub780\uad70\uc5d0 \ucc38\uc5ec\ud558\uc5ec \uc774\ub514 \uc544\ubbfc\uad70\uacfc\uc758 \uc804\ud22c\uc5d0 \ucc38\uac00\ud588\ub2e4. \uc6b0\uac04\ub2e4\ub294 \ud0c4\uc790\ub2c8\uc544\uc5d0 \uc120\uc804\uc744 \ud3ec\uace0\ud588\ub294\ub370 \uc774\ub514 \uc544\ubbfc\uc740 \ud0c4\uc790\ub2c8\uc544\uc758 \uce74\uac8c\ub77c \uc9c0\ubc29\uc774 \uc6d0\ub798 \uc6b0\uac04\ub2e4 \ub545\uc774\ub77c\uba70 \ud569\ubcd1\uc744 \uc704\ud574 \ud0c4\uc790\ub2c8\uc544\ub97c\uce68\uacf5\ud588\ub2e4. \uadf8\ub7ec\ub098 \ub2c8\uc5d0\ub808\ub808 \ub300\ud1b5\ub839\uc740 \ud0c4\uc790\ub2c8\uc544 \uad6d\ubbfc\ubc29\uc5b4\uad70\uc744 \ub3d9\uc6d0\ud558\uc5ec \ubc18\uaca9\uc5d0 \ub098\uc130\ub2e4. \ud0c4\uc790\ub2c8\uc544\uad70\uc740 \uba87 \uc8fc\ub9cc\uc5d0 \uacbd\ucc30\uacfc \uc885\uad70\uc0ac\uba74\uc8c4\uc218, \uad6d\ubbfc\ubcd1, \uad70\uc778\uc744 \ud3ec\ud568\ud574 10\ub9cc\uba85\uc73c\ub85c \uc99d\uac00\ud588\ub2e4. \uc6b0\uac04\ub2e4 \ud53c\ub09c\ubbfc\ub4e4\ub85c \uad6c\uc131\ub41c \ubc18 \uc544\ubbfc\ub2e8\uccb4\ub3c4 \ud0c4\uc790\ub2c8\uc544\uc5d0 \uac00\uc138\ud588\ub2e4. \uc774\ub4e4\uc740 \ubaa8\uc2dc \ud68c\uc758\uc5d0 \uc18c\uc18d\ud558\uc5ec \ubaa8\uc2dc \ud68c\uc758\ub294 \uc6b0\uac04\ub2e4 \uad6d\ubbfc\ud574\ubc29\uad70(UNLA)\uc744 \ud615\uc131\ud588\ub2e4. \ud2f0\ud1a0 \uc624\ucf08\ub85c\uc640 \ub370\uc774\ube57 \uc624\uc774\ud14c \uc624\uc870\ud06c\uac00 \uc774\ub044\ub294 \ud0a4\ucf54\uc2dc \ub9d0\ub984(Kikosi Maalum), \uc694\uc6e8\ub808 \ubb34\uc138\ubca0\ub2c8\uac00 \uc774\ub044\ub294 FRONASA, \uc544\ucf00\ub098 \ud3ec\uc871\uacfc \uc70c\ub9ac\uc5c4 \uc624\ub9c8\ub9ac\uc544, \uc544\ud14c\ucf08 \uc5d0\uc798\ub8e8\uac00 \uc774\ub044\ub294 <\uc6b0\uac04\ub2e4 \uad6c\uc6d0\uc6b4\ub3d9>\ub3c4 UNLA\uc5d0 \ucc38\uac00\ud588\ub2e4. \ud0c4\uc790\ub2c8\uc544\uad70\uc740 \uc18c\ub828\uc758 \uce74\ud29c\uc0e4\uc640 \ub85c\ucf13\ud3ec\ub97c \ud68d\ub4dd\ud574 \uc6b0\uac04\ub2e4\uad70\uc744 \ud45c\uc801\uc73c\ub85c \ud3ec\uaca9\uc744 \uac1c\uc2dc\ud588\ub2e4."} {"question": "\ub77c\uadf8\ub098\ub85c\ud06c\uc5d0 \ub4f1\uc7a5\ud558\ub294 \ubb34\uc2a4\ud544\ub9ac\uc758 \ub178\ub974\ub4dc\uc5b4\ub294 \ubb34\uc5c7\uc778\uac00?", "paragraph": "9\uc138\uae30\uc5d0 \uc4f0\uc5ec\uc9c4 \uace0\ub300\uace0\uc9c0\ub3c5\uc77c\uc5b4 \uc11c\uc0ac\uc2dc \u300a\ubb34\uc2a4\ud544\ub9ac\u300b\uc640 \ub77c\uadf8\ub098\ub85c\ud06c \uc0ac\uc774\uc758 \uad00\uacc4\uc5d0 \ub300\ud55c \uc774\ub860\ub4e4\uc774 \uc81c\uc2dc\ub418\uace0 \uc788\ub2e4. \u300a\ubb34\uc2a4\ud544\ub9ac\u300b\uc758 \ub0b4\uc6a9\uc740 \uae30\ub3c5\uad50\uc758 \ucd5c\ud6c4\uc758 \uc2ec\ud310\uc5d0 \uad00\ud55c \uac83\uc774\uba70, \ub0b4\uc6a9 \uc911 \"\ubb34\uc2a4\ud544\ub808\"(Muspille)\ub77c\ub294 \ub2e8\uc5b4\uac00 \ub098\ud0c0\ub09c\ub2e4. \ub610\ud55c 9\uc138\uae30\uc5d0 \uc4f0\uc5ec\uc9c4 \uc608\uc218\uc758 \uc0b6\uc5d0 \ub300\ud55c \uace0\ub300 \uc0c9\uc2a8\uc5b4 \uc11c\uc0ac\uc2dc \u300a\ud5ec\ub9ac\uc548\ub4dc\u300b\uc5d0\ub3c4 \"\ubb34\uc2a4\ud544\ub9ac\"\ub77c\ub294 \ub2e8\uc5b4\uc758 \ub2e4\uc591\ud55c \ubcc0\ud615\ub4e4\uc774 \ub098\ud0c0\ub09c\ub2e4. \u300a\ubb34\uc2a4\ud544\ub9ac\u300b\uc640 \u300a\ud5ec\ub9ac\uc548\ub4dc\u300b\uc5d0\uc11c \ubaa8\ub450 \"\ubb34\uc2a4\ud544\ub9ac\"\ub77c\ub294 \ub2e8\uc5b4\ub294 \ubd88\ub85c \uc778\ud55c \uc138\uacc4\uc758 \uba78\ub9dd\uc744 \uac00\ub9ac\ud0a4\ub294 \ub9d0\ub85c \uc0ac\uc6a9\ub41c\ub2e4. \ub77c\uadf8\ub098\ub85c\ud06c\uc5d0\uc11c\ub294 \"\ubb34\uc2a4\ud544\ub9ac\"\uc758 \ub178\ub974\ub4dc\uc5b4\uc5d0 \ud574\ub2f9\ud558\ub294 \"\ubb34\uc2a4\ud3a0\"\uc774\ub77c\ub294 \ub9d0\uc774 \ub77c\uadf8\ub098\ub85c\ud06c \ub54c \ub4f1\uc7a5\ud558\uba70, \ub77c\uadf8\ub098\ub85c\ud06c \uc5ed\uc2dc \uc138\uacc4\uac00 \ubd88\uae38\uc5d0 \uc9d1\uc5b4\uc0bc\ucf1c\uc9c4\ub2e4. \uc774\ub7f0 \uacf5\ud1b5\uc810\uc774 \uc788\uace0 \uc774 \ub2e8\uc5b4\ub4e4\uc758 \uc758\ubbf8 \ubc0f \uae30\uc6d0\uc5d0 \ub300\ud55c \ub2e4\uc591\ud55c \uc774\ub860\ub4e4\uc774 \uc81c\uae30\ub418\uace0 \uc788\uc73c\ub098, \uadf8 \uc5b4\uc6d0\uc740 \uc544\uc9c1\uae4c\uc9c0 \uba85\ud655\ud788 \ubc1d\ud600\uc9c0\uc9c0 \ubabb\ud558\uace0 \uc788\ub2e4.", "answer": "\ubb34\uc2a4\ud3a0", "sentence": "\ub77c\uadf8\ub098\ub85c\ud06c\uc5d0\uc11c\ub294 \"\ubb34\uc2a4\ud544\ub9ac\"\uc758 \ub178\ub974\ub4dc\uc5b4\uc5d0 \ud574\ub2f9\ud558\ub294 \" \ubb34\uc2a4\ud3a0 \"\uc774\ub77c\ub294 \ub9d0\uc774 \ub77c\uadf8\ub098\ub85c\ud06c \ub54c \ub4f1\uc7a5\ud558\uba70, \ub77c\uadf8\ub098\ub85c\ud06c \uc5ed\uc2dc \uc138\uacc4\uac00 \ubd88\uae38\uc5d0 \uc9d1\uc5b4\uc0bc\ucf1c\uc9c4\ub2e4.", "paragraph_sentence": "9\uc138\uae30\uc5d0 \uc4f0\uc5ec\uc9c4 \uace0\ub300\uace0\uc9c0\ub3c5\uc77c\uc5b4 \uc11c\uc0ac\uc2dc \u300a\ubb34\uc2a4\ud544\ub9ac\u300b\uc640 \ub77c\uadf8\ub098\ub85c\ud06c \uc0ac\uc774\uc758 \uad00\uacc4\uc5d0 \ub300\ud55c \uc774\ub860\ub4e4\uc774 \uc81c\uc2dc\ub418\uace0 \uc788\ub2e4. \u300a\ubb34\uc2a4\ud544\ub9ac\u300b\uc758 \ub0b4\uc6a9\uc740 \uae30\ub3c5\uad50\uc758 \ucd5c\ud6c4\uc758 \uc2ec\ud310\uc5d0 \uad00\ud55c \uac83\uc774\uba70, \ub0b4\uc6a9 \uc911 \"\ubb34\uc2a4\ud544\ub808\"(Muspille)\ub77c\ub294 \ub2e8\uc5b4\uac00 \ub098\ud0c0\ub09c\ub2e4. \ub610\ud55c 9\uc138\uae30\uc5d0 \uc4f0\uc5ec\uc9c4 \uc608\uc218\uc758 \uc0b6\uc5d0 \ub300\ud55c \uace0\ub300 \uc0c9\uc2a8\uc5b4 \uc11c\uc0ac\uc2dc \u300a\ud5ec\ub9ac\uc548\ub4dc\u300b\uc5d0\ub3c4 \"\ubb34\uc2a4\ud544\ub9ac\"\ub77c\ub294 \ub2e8\uc5b4\uc758 \ub2e4\uc591\ud55c \ubcc0\ud615\ub4e4\uc774 \ub098\ud0c0\ub09c\ub2e4. \u300a\ubb34\uc2a4\ud544\ub9ac\u300b\uc640 \u300a\ud5ec\ub9ac\uc548\ub4dc\u300b\uc5d0\uc11c \ubaa8\ub450 \"\ubb34\uc2a4\ud544\ub9ac\"\ub77c\ub294 \ub2e8\uc5b4\ub294 \ubd88\ub85c \uc778\ud55c \uc138\uacc4\uc758 \uba78\ub9dd\uc744 \uac00\ub9ac\ud0a4\ub294 \ub9d0\ub85c \uc0ac\uc6a9\ub41c\ub2e4. \ub77c\uadf8\ub098\ub85c\ud06c\uc5d0\uc11c\ub294 \"\ubb34\uc2a4\ud544\ub9ac\"\uc758 \ub178\ub974\ub4dc\uc5b4\uc5d0 \ud574\ub2f9\ud558\ub294 \" \ubb34\uc2a4\ud3a0 \"\uc774\ub77c\ub294 \ub9d0\uc774 \ub77c\uadf8\ub098\ub85c\ud06c \ub54c \ub4f1\uc7a5\ud558\uba70, \ub77c\uadf8\ub098\ub85c\ud06c \uc5ed\uc2dc \uc138\uacc4\uac00 \ubd88\uae38\uc5d0 \uc9d1\uc5b4\uc0bc\ucf1c\uc9c4\ub2e4. \uc774\ub7f0 \uacf5\ud1b5\uc810\uc774 \uc788\uace0 \uc774 \ub2e8\uc5b4\ub4e4\uc758 \uc758\ubbf8 \ubc0f \uae30\uc6d0\uc5d0 \ub300\ud55c \ub2e4\uc591\ud55c \uc774\ub860\ub4e4\uc774 \uc81c\uae30\ub418\uace0 \uc788\uc73c\ub098, \uadf8 \uc5b4\uc6d0\uc740 \uc544\uc9c1\uae4c\uc9c0 \uba85\ud655\ud788 \ubc1d\ud600\uc9c0\uc9c0 \ubabb\ud558\uace0 \uc788\ub2e4.", "paragraph_answer": "9\uc138\uae30\uc5d0 \uc4f0\uc5ec\uc9c4 \uace0\ub300\uace0\uc9c0\ub3c5\uc77c\uc5b4 \uc11c\uc0ac\uc2dc \u300a\ubb34\uc2a4\ud544\ub9ac\u300b\uc640 \ub77c\uadf8\ub098\ub85c\ud06c \uc0ac\uc774\uc758 \uad00\uacc4\uc5d0 \ub300\ud55c \uc774\ub860\ub4e4\uc774 \uc81c\uc2dc\ub418\uace0 \uc788\ub2e4. \u300a\ubb34\uc2a4\ud544\ub9ac\u300b\uc758 \ub0b4\uc6a9\uc740 \uae30\ub3c5\uad50\uc758 \ucd5c\ud6c4\uc758 \uc2ec\ud310\uc5d0 \uad00\ud55c \uac83\uc774\uba70, \ub0b4\uc6a9 \uc911 \"\ubb34\uc2a4\ud544\ub808\"(Muspille)\ub77c\ub294 \ub2e8\uc5b4\uac00 \ub098\ud0c0\ub09c\ub2e4. \ub610\ud55c 9\uc138\uae30\uc5d0 \uc4f0\uc5ec\uc9c4 \uc608\uc218\uc758 \uc0b6\uc5d0 \ub300\ud55c \uace0\ub300 \uc0c9\uc2a8\uc5b4 \uc11c\uc0ac\uc2dc \u300a\ud5ec\ub9ac\uc548\ub4dc\u300b\uc5d0\ub3c4 \"\ubb34\uc2a4\ud544\ub9ac\"\ub77c\ub294 \ub2e8\uc5b4\uc758 \ub2e4\uc591\ud55c \ubcc0\ud615\ub4e4\uc774 \ub098\ud0c0\ub09c\ub2e4. \u300a\ubb34\uc2a4\ud544\ub9ac\u300b\uc640 \u300a\ud5ec\ub9ac\uc548\ub4dc\u300b\uc5d0\uc11c \ubaa8\ub450 \"\ubb34\uc2a4\ud544\ub9ac\"\ub77c\ub294 \ub2e8\uc5b4\ub294 \ubd88\ub85c \uc778\ud55c \uc138\uacc4\uc758 \uba78\ub9dd\uc744 \uac00\ub9ac\ud0a4\ub294 \ub9d0\ub85c \uc0ac\uc6a9\ub41c\ub2e4. \ub77c\uadf8\ub098\ub85c\ud06c\uc5d0\uc11c\ub294 \"\ubb34\uc2a4\ud544\ub9ac\"\uc758 \ub178\ub974\ub4dc\uc5b4\uc5d0 \ud574\ub2f9\ud558\ub294 \" \ubb34\uc2a4\ud3a0 \"\uc774\ub77c\ub294 \ub9d0\uc774 \ub77c\uadf8\ub098\ub85c\ud06c \ub54c \ub4f1\uc7a5\ud558\uba70, \ub77c\uadf8\ub098\ub85c\ud06c \uc5ed\uc2dc \uc138\uacc4\uac00 \ubd88\uae38\uc5d0 \uc9d1\uc5b4\uc0bc\ucf1c\uc9c4\ub2e4. \uc774\ub7f0 \uacf5\ud1b5\uc810\uc774 \uc788\uace0 \uc774 \ub2e8\uc5b4\ub4e4\uc758 \uc758\ubbf8 \ubc0f \uae30\uc6d0\uc5d0 \ub300\ud55c \ub2e4\uc591\ud55c \uc774\ub860\ub4e4\uc774 \uc81c\uae30\ub418\uace0 \uc788\uc73c\ub098, \uadf8 \uc5b4\uc6d0\uc740 \uc544\uc9c1\uae4c\uc9c0 \uba85\ud655\ud788 \ubc1d\ud600\uc9c0\uc9c0 \ubabb\ud558\uace0 \uc788\ub2e4.", "sentence_answer": "\ub77c\uadf8\ub098\ub85c\ud06c\uc5d0\uc11c\ub294 \"\ubb34\uc2a4\ud544\ub9ac\"\uc758 \ub178\ub974\ub4dc\uc5b4\uc5d0 \ud574\ub2f9\ud558\ub294 \" \ubb34\uc2a4\ud3a0 \"\uc774\ub77c\ub294 \ub9d0\uc774 \ub77c\uadf8\ub098\ub85c\ud06c \ub54c \ub4f1\uc7a5\ud558\uba70, \ub77c\uadf8\ub098\ub85c\ud06c \uc5ed\uc2dc \uc138\uacc4\uac00 \ubd88\uae38\uc5d0 \uc9d1\uc5b4\uc0bc\ucf1c\uc9c4\ub2e4.", "paragraph_id": "6493726-12-1", "paragraph_question": "question: \ub77c\uadf8\ub098\ub85c\ud06c\uc5d0 \ub4f1\uc7a5\ud558\ub294 \ubb34\uc2a4\ud544\ub9ac\uc758 \ub178\ub974\ub4dc\uc5b4\ub294 \ubb34\uc5c7\uc778\uac00?, context: 9\uc138\uae30\uc5d0 \uc4f0\uc5ec\uc9c4 \uace0\ub300\uace0\uc9c0\ub3c5\uc77c\uc5b4 \uc11c\uc0ac\uc2dc \u300a\ubb34\uc2a4\ud544\ub9ac\u300b\uc640 \ub77c\uadf8\ub098\ub85c\ud06c \uc0ac\uc774\uc758 \uad00\uacc4\uc5d0 \ub300\ud55c \uc774\ub860\ub4e4\uc774 \uc81c\uc2dc\ub418\uace0 \uc788\ub2e4. \u300a\ubb34\uc2a4\ud544\ub9ac\u300b\uc758 \ub0b4\uc6a9\uc740 \uae30\ub3c5\uad50\uc758 \ucd5c\ud6c4\uc758 \uc2ec\ud310\uc5d0 \uad00\ud55c \uac83\uc774\uba70, \ub0b4\uc6a9 \uc911 \"\ubb34\uc2a4\ud544\ub808\"(Muspille)\ub77c\ub294 \ub2e8\uc5b4\uac00 \ub098\ud0c0\ub09c\ub2e4. \ub610\ud55c 9\uc138\uae30\uc5d0 \uc4f0\uc5ec\uc9c4 \uc608\uc218\uc758 \uc0b6\uc5d0 \ub300\ud55c \uace0\ub300 \uc0c9\uc2a8\uc5b4 \uc11c\uc0ac\uc2dc \u300a\ud5ec\ub9ac\uc548\ub4dc\u300b\uc5d0\ub3c4 \"\ubb34\uc2a4\ud544\ub9ac\"\ub77c\ub294 \ub2e8\uc5b4\uc758 \ub2e4\uc591\ud55c \ubcc0\ud615\ub4e4\uc774 \ub098\ud0c0\ub09c\ub2e4. \u300a\ubb34\uc2a4\ud544\ub9ac\u300b\uc640 \u300a\ud5ec\ub9ac\uc548\ub4dc\u300b\uc5d0\uc11c \ubaa8\ub450 \"\ubb34\uc2a4\ud544\ub9ac\"\ub77c\ub294 \ub2e8\uc5b4\ub294 \ubd88\ub85c \uc778\ud55c \uc138\uacc4\uc758 \uba78\ub9dd\uc744 \uac00\ub9ac\ud0a4\ub294 \ub9d0\ub85c \uc0ac\uc6a9\ub41c\ub2e4. \ub77c\uadf8\ub098\ub85c\ud06c\uc5d0\uc11c\ub294 \"\ubb34\uc2a4\ud544\ub9ac\"\uc758 \ub178\ub974\ub4dc\uc5b4\uc5d0 \ud574\ub2f9\ud558\ub294 \"\ubb34\uc2a4\ud3a0\"\uc774\ub77c\ub294 \ub9d0\uc774 \ub77c\uadf8\ub098\ub85c\ud06c \ub54c \ub4f1\uc7a5\ud558\uba70, \ub77c\uadf8\ub098\ub85c\ud06c \uc5ed\uc2dc \uc138\uacc4\uac00 \ubd88\uae38\uc5d0 \uc9d1\uc5b4\uc0bc\ucf1c\uc9c4\ub2e4. \uc774\ub7f0 \uacf5\ud1b5\uc810\uc774 \uc788\uace0 \uc774 \ub2e8\uc5b4\ub4e4\uc758 \uc758\ubbf8 \ubc0f \uae30\uc6d0\uc5d0 \ub300\ud55c \ub2e4\uc591\ud55c \uc774\ub860\ub4e4\uc774 \uc81c\uae30\ub418\uace0 \uc788\uc73c\ub098, \uadf8 \uc5b4\uc6d0\uc740 \uc544\uc9c1\uae4c\uc9c0 \uba85\ud655\ud788 \ubc1d\ud600\uc9c0\uc9c0 \ubabb\ud558\uace0 \uc788\ub2e4."} {"question": "\ud63c\uce68\uacfc \ub3c4\uac70\ub97c \ub5a0\ub09c \ub9c8\uc74c\uc758 \ud3c9\ub4f1\uc131\uacfc \ubb34\uacbd\uac01\uc131\uc744 \ub73b\ud558\ub294 \uac83\uc758 \uc774\ub984\uc740 \ubb34\uc5c7\uc778\uac00?", "paragraph": "\uc0ac(\u6368, \ub0b4\ub824\ub193\uc74c, \ubc84\ub9bc, \uace0\uc694, \ud3c9\uc815, \ud3c9\uc815\uc2ec, \ud3c9\uc628, \uade0\ud615, \ud3c9\ud615, \uc0b0\uc2a4\ud06c\ub9ac\ud2b8\uc5b4: upeks\u0101, \ud314\ub9ac\uc5b4: upekkh\u0101 \ub610\ub294 upekh\u0101, \uc601\uc5b4: serenity, equilibrium, equanimity, stability, composure)\ub294 \uc124\uc77c\uccb4\uc720\ubd80\uc758 5\uc704 75\ubc95\uc5d0\uc11c \uc2ec\uc18c\ubc95(\u5fc3\u6240\u6cd5: 46\uac00\uc9c0) \uc911 \ub300\uc120\uc9c0\ubc95(\u5927\u5584\u5730\u6cd5: 10\uac00\uc9c0) \uac00\uc6b4\ub370 \ud558\ub098\uc774\uba70, \uc720\uc2dd\uc720\uac00\ud589\ud30c\uc640 \ubc95\uc0c1\uc885\uc758 5\uc704 100\ubc95\uc5d0\uc11c \uc2ec\uc18c\ubc95(\u5fc3\u6240\u6cd5: 51\uac00\uc9c0) \uc911 \uc120\uc2ec\uc18c(\u5584\u5fc3\u6240: 11\uac00\uc9c0) \uac00\uc6b4\ub370 \ud558\ub098\uc778 \ud589\uc0ac(\u884c\u6368)\uc774\ub2e4. \uc0ac\uc218(\u6368\u53d7: \uad34\ub86d\uc9c0\ub3c4 \uc990\uac81\uc9c0\ub3c4 \uc54a\uc740 \uac10\uac01)\uc640 \uad6c\ubcc4\ud558\uae30 \uc704\ud558\uc5ec \ud589\uc0ac(\u884c\u6368)\ub77c \ud55c\ub2e4. \uc124\uc77c\uccb4\uc720\ubd80\uc5d0 \ub530\ub974\uba74, \uc0ac(\u6368)\ub294 \ud63c\uce68(\u60db\u6c88: \ubb34\uae30\ub825, \uce68\uccb4)\uacfc \ub3c4\uac70(\u6389\u64e7: \ud750\ud2b8\ub7ec\uc9d0, \ub4e4\ub738, \ub534\ub370\ub85c \ub2ec\uc544\ub0a8)\ub97c \ub5a0\ub09c \ub9c8\uc74c(6\uc2dd \ub610\ub294 8\uc2dd, \uc989 \uc2ec\uc655, \uc989 \uc2ec\ubc95)\uc758 \ud3c9\ub4f1\uc131(\u5e73\u7b49\u6027: \ud3c9\uc815)\uacfc \ubb34\uacbd\uac01\uc131(\u7121\u8b66\u89ba\u6027: \ub3d9\uc694\ub428\uc774 \uc5c6\ub294 \uac83)\uc744 \ub9d0\ud55c\ub2e4.", "answer": "\uc0ac", "sentence": "\uc0ac (\u6368, \ub0b4\ub824\ub193\uc74c, \ubc84\ub9bc, \uace0\uc694, \ud3c9\uc815, \ud3c9\uc815\uc2ec, \ud3c9\uc628, \uade0\ud615, \ud3c9\ud615, \uc0b0\uc2a4\ud06c\ub9ac\ud2b8\uc5b4: upeks\u0101, \ud314\ub9ac\uc5b4: upekkh\u0101 \ub610\ub294 upekh\u0101, \uc601\uc5b4: serenity, equilibrium, equanimity, stability, composure)\ub294 \uc124\uc77c\uccb4\uc720\ubd80\uc758 5\uc704 75\ubc95\uc5d0\uc11c \uc2ec\uc18c\ubc95(\u5fc3\u6240\u6cd5: 46\uac00\uc9c0) \uc911 \ub300\uc120\uc9c0\ubc95(\u5927\u5584\u5730\u6cd5: 10\uac00\uc9c0) \uac00\uc6b4\ub370 \ud558\ub098\uc774\uba70, \uc720\uc2dd\uc720\uac00\ud589\ud30c\uc640 \ubc95\uc0c1\uc885\uc758 5\uc704 100\ubc95\uc5d0\uc11c \uc2ec\uc18c\ubc95(\u5fc3\u6240\u6cd5: 51\uac00\uc9c0) \uc911 \uc120\uc2ec\uc18c(\u5584\u5fc3\u6240: 11\uac00\uc9c0) \uac00\uc6b4\ub370 \ud558\ub098\uc778 \ud589\uc0ac(\u884c\u6368)\uc774\ub2e4.", "paragraph_sentence": " \uc0ac (\u6368, \ub0b4\ub824\ub193\uc74c, \ubc84\ub9bc, \uace0\uc694, \ud3c9\uc815, \ud3c9\uc815\uc2ec, \ud3c9\uc628, \uade0\ud615, \ud3c9\ud615, \uc0b0\uc2a4\ud06c\ub9ac\ud2b8\uc5b4: upeks\u0101, \ud314\ub9ac\uc5b4: upekkh\u0101 \ub610\ub294 upekh\u0101, \uc601\uc5b4: serenity, equilibrium, equanimity, stability, composure)\ub294 \uc124\uc77c\uccb4\uc720\ubd80\uc758 5\uc704 75\ubc95\uc5d0\uc11c \uc2ec\uc18c\ubc95(\u5fc3\u6240\u6cd5: 46\uac00\uc9c0) \uc911 \ub300\uc120\uc9c0\ubc95(\u5927\u5584\u5730\u6cd5: 10\uac00\uc9c0) \uac00\uc6b4\ub370 \ud558\ub098\uc774\uba70, \uc720\uc2dd\uc720\uac00\ud589\ud30c\uc640 \ubc95\uc0c1\uc885\uc758 5\uc704 100\ubc95\uc5d0\uc11c \uc2ec\uc18c\ubc95(\u5fc3\u6240\u6cd5: 51\uac00\uc9c0) \uc911 \uc120\uc2ec\uc18c(\u5584\u5fc3\u6240: 11\uac00\uc9c0) \uac00\uc6b4\ub370 \ud558\ub098\uc778 \ud589\uc0ac(\u884c\u6368)\uc774\ub2e4. \uc0ac\uc218(\u6368\u53d7: \uad34\ub86d\uc9c0\ub3c4 \uc990\uac81\uc9c0\ub3c4 \uc54a\uc740 \uac10\uac01)\uc640 \uad6c\ubcc4\ud558\uae30 \uc704\ud558\uc5ec \ud589\uc0ac(\u884c\u6368)\ub77c \ud55c\ub2e4. \uc124\uc77c\uccb4\uc720\ubd80\uc5d0 \ub530\ub974\uba74, \uc0ac(\u6368)\ub294 \ud63c\uce68(\u60db\u6c88: \ubb34\uae30\ub825, \uce68\uccb4)\uacfc \ub3c4\uac70(\u6389\u64e7: \ud750\ud2b8\ub7ec\uc9d0, \ub4e4\ub738, \ub534\ub370\ub85c \ub2ec\uc544\ub0a8)\ub97c \ub5a0\ub09c \ub9c8\uc74c(6\uc2dd \ub610\ub294 8\uc2dd, \uc989 \uc2ec\uc655, \uc989 \uc2ec\ubc95)\uc758 \ud3c9\ub4f1\uc131(\u5e73\u7b49\u6027: \ud3c9\uc815)\uacfc \ubb34\uacbd\uac01\uc131(\u7121\u8b66\u89ba\u6027: \ub3d9\uc694\ub428\uc774 \uc5c6\ub294 \uac83)\uc744 \ub9d0\ud55c\ub2e4.", "paragraph_answer": " \uc0ac (\u6368, \ub0b4\ub824\ub193\uc74c, \ubc84\ub9bc, \uace0\uc694, \ud3c9\uc815, \ud3c9\uc815\uc2ec, \ud3c9\uc628, \uade0\ud615, \ud3c9\ud615, \uc0b0\uc2a4\ud06c\ub9ac\ud2b8\uc5b4: upeks\u0101, \ud314\ub9ac\uc5b4: upekkh\u0101 \ub610\ub294 upekh\u0101, \uc601\uc5b4: serenity, equilibrium, equanimity, stability, composure)\ub294 \uc124\uc77c\uccb4\uc720\ubd80\uc758 5\uc704 75\ubc95\uc5d0\uc11c \uc2ec\uc18c\ubc95(\u5fc3\u6240\u6cd5: 46\uac00\uc9c0) \uc911 \ub300\uc120\uc9c0\ubc95(\u5927\u5584\u5730\u6cd5: 10\uac00\uc9c0) \uac00\uc6b4\ub370 \ud558\ub098\uc774\uba70, \uc720\uc2dd\uc720\uac00\ud589\ud30c\uc640 \ubc95\uc0c1\uc885\uc758 5\uc704 100\ubc95\uc5d0\uc11c \uc2ec\uc18c\ubc95(\u5fc3\u6240\u6cd5: 51\uac00\uc9c0) \uc911 \uc120\uc2ec\uc18c(\u5584\u5fc3\u6240: 11\uac00\uc9c0) \uac00\uc6b4\ub370 \ud558\ub098\uc778 \ud589\uc0ac(\u884c\u6368)\uc774\ub2e4. \uc0ac\uc218(\u6368\u53d7: \uad34\ub86d\uc9c0\ub3c4 \uc990\uac81\uc9c0\ub3c4 \uc54a\uc740 \uac10\uac01)\uc640 \uad6c\ubcc4\ud558\uae30 \uc704\ud558\uc5ec \ud589\uc0ac(\u884c\u6368)\ub77c \ud55c\ub2e4. \uc124\uc77c\uccb4\uc720\ubd80\uc5d0 \ub530\ub974\uba74, \uc0ac(\u6368)\ub294 \ud63c\uce68(\u60db\u6c88: \ubb34\uae30\ub825, \uce68\uccb4)\uacfc \ub3c4\uac70(\u6389\u64e7: \ud750\ud2b8\ub7ec\uc9d0, \ub4e4\ub738, \ub534\ub370\ub85c \ub2ec\uc544\ub0a8)\ub97c \ub5a0\ub09c \ub9c8\uc74c(6\uc2dd \ub610\ub294 8\uc2dd, \uc989 \uc2ec\uc655, \uc989 \uc2ec\ubc95)\uc758 \ud3c9\ub4f1\uc131(\u5e73\u7b49\u6027: \ud3c9\uc815)\uacfc \ubb34\uacbd\uac01\uc131(\u7121\u8b66\u89ba\u6027: \ub3d9\uc694\ub428\uc774 \uc5c6\ub294 \uac83)\uc744 \ub9d0\ud55c\ub2e4.", "sentence_answer": " \uc0ac (\u6368, \ub0b4\ub824\ub193\uc74c, \ubc84\ub9bc, \uace0\uc694, \ud3c9\uc815, \ud3c9\uc815\uc2ec, \ud3c9\uc628, \uade0\ud615, \ud3c9\ud615, \uc0b0\uc2a4\ud06c\ub9ac\ud2b8\uc5b4: upeks\u0101, \ud314\ub9ac\uc5b4: upekkh\u0101 \ub610\ub294 upekh\u0101, \uc601\uc5b4: serenity, equilibrium, equanimity, stability, composure)\ub294 \uc124\uc77c\uccb4\uc720\ubd80\uc758 5\uc704 75\ubc95\uc5d0\uc11c \uc2ec\uc18c\ubc95(\u5fc3\u6240\u6cd5: 46\uac00\uc9c0) \uc911 \ub300\uc120\uc9c0\ubc95(\u5927\u5584\u5730\u6cd5: 10\uac00\uc9c0) \uac00\uc6b4\ub370 \ud558\ub098\uc774\uba70, \uc720\uc2dd\uc720\uac00\ud589\ud30c\uc640 \ubc95\uc0c1\uc885\uc758 5\uc704 100\ubc95\uc5d0\uc11c \uc2ec\uc18c\ubc95(\u5fc3\u6240\u6cd5: 51\uac00\uc9c0) \uc911 \uc120\uc2ec\uc18c(\u5584\u5fc3\u6240: 11\uac00\uc9c0) \uac00\uc6b4\ub370 \ud558\ub098\uc778 \ud589\uc0ac(\u884c\u6368)\uc774\ub2e4.", "paragraph_id": "6510488-0-0", "paragraph_question": "question: \ud63c\uce68\uacfc \ub3c4\uac70\ub97c \ub5a0\ub09c \ub9c8\uc74c\uc758 \ud3c9\ub4f1\uc131\uacfc \ubb34\uacbd\uac01\uc131\uc744 \ub73b\ud558\ub294 \uac83\uc758 \uc774\ub984\uc740 \ubb34\uc5c7\uc778\uac00?, context: \uc0ac(\u6368, \ub0b4\ub824\ub193\uc74c, \ubc84\ub9bc, \uace0\uc694, \ud3c9\uc815, \ud3c9\uc815\uc2ec, \ud3c9\uc628, \uade0\ud615, \ud3c9\ud615, \uc0b0\uc2a4\ud06c\ub9ac\ud2b8\uc5b4: upeks\u0101, \ud314\ub9ac\uc5b4: upekkh\u0101 \ub610\ub294 upekh\u0101, \uc601\uc5b4: serenity, equilibrium, equanimity, stability, composure)\ub294 \uc124\uc77c\uccb4\uc720\ubd80\uc758 5\uc704 75\ubc95\uc5d0\uc11c \uc2ec\uc18c\ubc95(\u5fc3\u6240\u6cd5: 46\uac00\uc9c0) \uc911 \ub300\uc120\uc9c0\ubc95(\u5927\u5584\u5730\u6cd5: 10\uac00\uc9c0) \uac00\uc6b4\ub370 \ud558\ub098\uc774\uba70, \uc720\uc2dd\uc720\uac00\ud589\ud30c\uc640 \ubc95\uc0c1\uc885\uc758 5\uc704 100\ubc95\uc5d0\uc11c \uc2ec\uc18c\ubc95(\u5fc3\u6240\u6cd5: 51\uac00\uc9c0) \uc911 \uc120\uc2ec\uc18c(\u5584\u5fc3\u6240: 11\uac00\uc9c0) \uac00\uc6b4\ub370 \ud558\ub098\uc778 \ud589\uc0ac(\u884c\u6368)\uc774\ub2e4. \uc0ac\uc218(\u6368\u53d7: \uad34\ub86d\uc9c0\ub3c4 \uc990\uac81\uc9c0\ub3c4 \uc54a\uc740 \uac10\uac01)\uc640 \uad6c\ubcc4\ud558\uae30 \uc704\ud558\uc5ec \ud589\uc0ac(\u884c\u6368)\ub77c \ud55c\ub2e4. \uc124\uc77c\uccb4\uc720\ubd80\uc5d0 \ub530\ub974\uba74, \uc0ac(\u6368)\ub294 \ud63c\uce68(\u60db\u6c88: \ubb34\uae30\ub825, \uce68\uccb4)\uacfc \ub3c4\uac70(\u6389\u64e7: \ud750\ud2b8\ub7ec\uc9d0, \ub4e4\ub738, \ub534\ub370\ub85c \ub2ec\uc544\ub0a8)\ub97c \ub5a0\ub09c \ub9c8\uc74c(6\uc2dd \ub610\ub294 8\uc2dd, \uc989 \uc2ec\uc655, \uc989 \uc2ec\ubc95)\uc758 \ud3c9\ub4f1\uc131(\u5e73\u7b49\u6027: \ud3c9\uc815)\uacfc \ubb34\uacbd\uac01\uc131(\u7121\u8b66\u89ba\u6027: \ub3d9\uc694\ub428\uc774 \uc5c6\ub294 \uac83)\uc744 \ub9d0\ud55c\ub2e4."} {"question": "\ud504\ub79c\uc2dc\uc2a4\uac00 \uacf5\uc744 \uc7a1\uc740 \uc0c1\ud0dc\uc5d0\uc11c \uc288\ud305,\ud328\uc2a4,\ub4dc\ub9ac\ube14,\uc218\ube44\ud558\uae30 \ud798\ub4e0 \uc120\uc218\ub77c\ud558\uc5ec \ud55c \uc2a4\uce74\uc6b0\ud2b8\ub294 \ud504\ub79c\uc2dc\uc2a4\ub97c \ubb34\uc5c7\uc774\ub77c \ud45c\ud604\ud588\ub294\uac00?", "paragraph": "\uc2a4\ud2f0\ube0c \ud504\ub79c\uc2dc\uc2a4\ub294 \ub2e4\ub978 \uc288\ud305\uac00\ub4dc\uc5d0 \ube44\ud558\uba74 \ud0a4\uac00 \uc791\uc740 \ud3b8\uc774\ub2e4. \ud558\uc9c0\ub9cc \uadf8 \uc791\uc740 \ud0a4\ub85c NBA \uc120\uc218 \uc0dd\ud65c\uc744 \ud558\uba74\uc11c \ud3c9\uade0 5.6\ub9ac\ubc14\uc6b4\ub4dc\ub97c \uae30\ub85d\ud588\uace0 \ub300\ub2e8\ud55c \ub3cc\ud30c\ub825\uc744 \uc120\ubcf4\uc600\ub2e4. NBA \uc2a4\uce74\uc6b0\ud2b8\ub4e4\uc758 \uc758\ud558\uba74 \ud504\ub79c\uc2dc\uc2a4\ub294 \ub6f0\uc5b4\ub09c \uc6b4\ub3d9\uc2e0\uacbd\uc744 \uac00\uc9c0\uace0 \uc788\uace0 \uadf8\uc758 \ubcfc \ud578\ub4e4\ub9c1 \uc19c\uc528\uc640 \uc288\ud305 \uac10\uac01\uc744 \uadf9\ucc2c\ud588\ub2e4. \ud55c \uc2a4\uce74\uc6b0\ud2b8\ub294 \ud504\ub79c\uc2dc\uc2a4\ub97c \ud2b8\ub9ac\ud50c \uc2a4\ub808\ud2b8\ub77c\uace0 \ud45c\ud604\ud588\ub2e4. \uc989 \ud504\ub79c\uc2dc\uc2a4\uac00 \uacf5\uc744 \uc7a1\uc740 \uc0c1\ud0dc\uc5d0\uc11c \uc288\ud305\uc744 \ud558\uac70\ub098 \ud328\uc2a4\ub97c \ud558\uac70\ub098 \ub4dc\ub9ac\ube14\uc744 \ud558\ub358 \uc218\ube44\ud558\uae30 \ud798\ub4e0 \uc120\uc218\ub77c\uace0 \ub9d0\ud588\ub2e4. \ud504\ub79c\uc2dc\uc2a4\ub294 1999\ub144 NBA \ub4dc\ub808\ud504\ud2b8\ub54c \uc0c1\uc704\uac74\uc5d0 \ubf51\ud790\uac70\ub77c\uace0 \ubd84\uc11d\ud588\ub2e4. \ud558\uc9c0\ub9cc \ud504\ub79c\uc2dc\uc2a4\ub3c4 \ub2e8\uc810\uc774 \uc788\uc5c8\ub2e4. \uac00\ub054 \ud3ec\uc778\ud2b8 \uac00\ub4dc \uc5ed\ud560\uc744 \ud558\ub294 \ud504\ub79c\uc2dc\uc2a4\ub294 \ud3c9\uade0 6\ub3c4\uc6c0\uc744 \uae30\ub85d\ud558\ub418 \ub9ce\uc740 \ucf54\uce58\ub4e4\uacfc \uc120\uc218\ub4e4\ub85c\uc11c \uc774\uae30\uc801\uc774\uace0 \uac70\ub9cc\ud558\ub2e4\uace0 \ud3c9\ud310\uc744 \ubc1b\uc558\ub2e4.", "answer": "\ud2b8\ub9ac\ud50c \uc2a4\ub808\ud2b8", "sentence": "\ud55c \uc2a4\uce74\uc6b0\ud2b8\ub294 \ud504\ub79c\uc2dc\uc2a4\ub97c \ud2b8\ub9ac\ud50c \uc2a4\ub808\ud2b8 \ub77c\uace0 \ud45c\ud604\ud588\ub2e4.", "paragraph_sentence": "\uc2a4\ud2f0\ube0c \ud504\ub79c\uc2dc\uc2a4\ub294 \ub2e4\ub978 \uc288\ud305\uac00\ub4dc\uc5d0 \ube44\ud558\uba74 \ud0a4\uac00 \uc791\uc740 \ud3b8\uc774\ub2e4. \ud558\uc9c0\ub9cc \uadf8 \uc791\uc740 \ud0a4\ub85c NBA \uc120\uc218 \uc0dd\ud65c\uc744 \ud558\uba74\uc11c \ud3c9\uade0 5.6\ub9ac\ubc14\uc6b4\ub4dc\ub97c \uae30\ub85d\ud588\uace0 \ub300\ub2e8\ud55c \ub3cc\ud30c\ub825\uc744 \uc120\ubcf4\uc600\ub2e4. NBA \uc2a4\uce74\uc6b0\ud2b8\ub4e4\uc758 \uc758\ud558\uba74 \ud504\ub79c\uc2dc\uc2a4\ub294 \ub6f0\uc5b4\ub09c \uc6b4\ub3d9\uc2e0\uacbd\uc744 \uac00\uc9c0\uace0 \uc788\uace0 \uadf8\uc758 \ubcfc \ud578\ub4e4\ub9c1 \uc19c\uc528\uc640 \uc288\ud305 \uac10\uac01\uc744 \uadf9\ucc2c\ud588\ub2e4. \ud55c \uc2a4\uce74\uc6b0\ud2b8\ub294 \ud504\ub79c\uc2dc\uc2a4\ub97c \ud2b8\ub9ac\ud50c \uc2a4\ub808\ud2b8 \ub77c\uace0 \ud45c\ud604\ud588\ub2e4. \uc989 \ud504\ub79c\uc2dc\uc2a4\uac00 \uacf5\uc744 \uc7a1\uc740 \uc0c1\ud0dc\uc5d0\uc11c \uc288\ud305\uc744 \ud558\uac70\ub098 \ud328\uc2a4\ub97c \ud558\uac70\ub098 \ub4dc\ub9ac\ube14\uc744 \ud558\ub358 \uc218\ube44\ud558\uae30 \ud798\ub4e0 \uc120\uc218\ub77c\uace0 \ub9d0\ud588\ub2e4. \ud504\ub79c\uc2dc\uc2a4\ub294 1999\ub144 NBA \ub4dc\ub808\ud504\ud2b8\ub54c \uc0c1\uc704\uac74\uc5d0 \ubf51\ud790\uac70\ub77c\uace0 \ubd84\uc11d\ud588\ub2e4. \ud558\uc9c0\ub9cc \ud504\ub79c\uc2dc\uc2a4\ub3c4 \ub2e8\uc810\uc774 \uc788\uc5c8\ub2e4. \uac00\ub054 \ud3ec\uc778\ud2b8 \uac00\ub4dc \uc5ed\ud560\uc744 \ud558\ub294 \ud504\ub79c\uc2dc\uc2a4\ub294 \ud3c9\uade0 6\ub3c4\uc6c0\uc744 \uae30\ub85d\ud558\ub418 \ub9ce\uc740 \ucf54\uce58\ub4e4\uacfc \uc120\uc218\ub4e4\ub85c\uc11c \uc774\uae30\uc801\uc774\uace0 \uac70\ub9cc\ud558\ub2e4\uace0 \ud3c9\ud310\uc744 \ubc1b\uc558\ub2e4.", "paragraph_answer": "\uc2a4\ud2f0\ube0c \ud504\ub79c\uc2dc\uc2a4\ub294 \ub2e4\ub978 \uc288\ud305\uac00\ub4dc\uc5d0 \ube44\ud558\uba74 \ud0a4\uac00 \uc791\uc740 \ud3b8\uc774\ub2e4. \ud558\uc9c0\ub9cc \uadf8 \uc791\uc740 \ud0a4\ub85c NBA \uc120\uc218 \uc0dd\ud65c\uc744 \ud558\uba74\uc11c \ud3c9\uade0 5.6\ub9ac\ubc14\uc6b4\ub4dc\ub97c \uae30\ub85d\ud588\uace0 \ub300\ub2e8\ud55c \ub3cc\ud30c\ub825\uc744 \uc120\ubcf4\uc600\ub2e4. NBA \uc2a4\uce74\uc6b0\ud2b8\ub4e4\uc758 \uc758\ud558\uba74 \ud504\ub79c\uc2dc\uc2a4\ub294 \ub6f0\uc5b4\ub09c \uc6b4\ub3d9\uc2e0\uacbd\uc744 \uac00\uc9c0\uace0 \uc788\uace0 \uadf8\uc758 \ubcfc \ud578\ub4e4\ub9c1 \uc19c\uc528\uc640 \uc288\ud305 \uac10\uac01\uc744 \uadf9\ucc2c\ud588\ub2e4. \ud55c \uc2a4\uce74\uc6b0\ud2b8\ub294 \ud504\ub79c\uc2dc\uc2a4\ub97c \ud2b8\ub9ac\ud50c \uc2a4\ub808\ud2b8 \ub77c\uace0 \ud45c\ud604\ud588\ub2e4. \uc989 \ud504\ub79c\uc2dc\uc2a4\uac00 \uacf5\uc744 \uc7a1\uc740 \uc0c1\ud0dc\uc5d0\uc11c \uc288\ud305\uc744 \ud558\uac70\ub098 \ud328\uc2a4\ub97c \ud558\uac70\ub098 \ub4dc\ub9ac\ube14\uc744 \ud558\ub358 \uc218\ube44\ud558\uae30 \ud798\ub4e0 \uc120\uc218\ub77c\uace0 \ub9d0\ud588\ub2e4. \ud504\ub79c\uc2dc\uc2a4\ub294 1999\ub144 NBA \ub4dc\ub808\ud504\ud2b8\ub54c \uc0c1\uc704\uac74\uc5d0 \ubf51\ud790\uac70\ub77c\uace0 \ubd84\uc11d\ud588\ub2e4. \ud558\uc9c0\ub9cc \ud504\ub79c\uc2dc\uc2a4\ub3c4 \ub2e8\uc810\uc774 \uc788\uc5c8\ub2e4. \uac00\ub054 \ud3ec\uc778\ud2b8 \uac00\ub4dc \uc5ed\ud560\uc744 \ud558\ub294 \ud504\ub79c\uc2dc\uc2a4\ub294 \ud3c9\uade0 6\ub3c4\uc6c0\uc744 \uae30\ub85d\ud558\ub418 \ub9ce\uc740 \ucf54\uce58\ub4e4\uacfc \uc120\uc218\ub4e4\ub85c\uc11c \uc774\uae30\uc801\uc774\uace0 \uac70\ub9cc\ud558\ub2e4\uace0 \ud3c9\ud310\uc744 \ubc1b\uc558\ub2e4.", "sentence_answer": "\ud55c \uc2a4\uce74\uc6b0\ud2b8\ub294 \ud504\ub79c\uc2dc\uc2a4\ub97c \ud2b8\ub9ac\ud50c \uc2a4\ub808\ud2b8 \ub77c\uace0 \ud45c\ud604\ud588\ub2e4.", "paragraph_id": "6519214-3-1", "paragraph_question": "question: \ud504\ub79c\uc2dc\uc2a4\uac00 \uacf5\uc744 \uc7a1\uc740 \uc0c1\ud0dc\uc5d0\uc11c \uc288\ud305,\ud328\uc2a4,\ub4dc\ub9ac\ube14,\uc218\ube44\ud558\uae30 \ud798\ub4e0 \uc120\uc218\ub77c\ud558\uc5ec \ud55c \uc2a4\uce74\uc6b0\ud2b8\ub294 \ud504\ub79c\uc2dc\uc2a4\ub97c \ubb34\uc5c7\uc774\ub77c \ud45c\ud604\ud588\ub294\uac00?, context: \uc2a4\ud2f0\ube0c \ud504\ub79c\uc2dc\uc2a4\ub294 \ub2e4\ub978 \uc288\ud305\uac00\ub4dc\uc5d0 \ube44\ud558\uba74 \ud0a4\uac00 \uc791\uc740 \ud3b8\uc774\ub2e4. \ud558\uc9c0\ub9cc \uadf8 \uc791\uc740 \ud0a4\ub85c NBA \uc120\uc218 \uc0dd\ud65c\uc744 \ud558\uba74\uc11c \ud3c9\uade0 5.6\ub9ac\ubc14\uc6b4\ub4dc\ub97c \uae30\ub85d\ud588\uace0 \ub300\ub2e8\ud55c \ub3cc\ud30c\ub825\uc744 \uc120\ubcf4\uc600\ub2e4. NBA \uc2a4\uce74\uc6b0\ud2b8\ub4e4\uc758 \uc758\ud558\uba74 \ud504\ub79c\uc2dc\uc2a4\ub294 \ub6f0\uc5b4\ub09c \uc6b4\ub3d9\uc2e0\uacbd\uc744 \uac00\uc9c0\uace0 \uc788\uace0 \uadf8\uc758 \ubcfc \ud578\ub4e4\ub9c1 \uc19c\uc528\uc640 \uc288\ud305 \uac10\uac01\uc744 \uadf9\ucc2c\ud588\ub2e4. \ud55c \uc2a4\uce74\uc6b0\ud2b8\ub294 \ud504\ub79c\uc2dc\uc2a4\ub97c \ud2b8\ub9ac\ud50c \uc2a4\ub808\ud2b8\ub77c\uace0 \ud45c\ud604\ud588\ub2e4. \uc989 \ud504\ub79c\uc2dc\uc2a4\uac00 \uacf5\uc744 \uc7a1\uc740 \uc0c1\ud0dc\uc5d0\uc11c \uc288\ud305\uc744 \ud558\uac70\ub098 \ud328\uc2a4\ub97c \ud558\uac70\ub098 \ub4dc\ub9ac\ube14\uc744 \ud558\ub358 \uc218\ube44\ud558\uae30 \ud798\ub4e0 \uc120\uc218\ub77c\uace0 \ub9d0\ud588\ub2e4. \ud504\ub79c\uc2dc\uc2a4\ub294 1999\ub144 NBA \ub4dc\ub808\ud504\ud2b8\ub54c \uc0c1\uc704\uac74\uc5d0 \ubf51\ud790\uac70\ub77c\uace0 \ubd84\uc11d\ud588\ub2e4. \ud558\uc9c0\ub9cc \ud504\ub79c\uc2dc\uc2a4\ub3c4 \ub2e8\uc810\uc774 \uc788\uc5c8\ub2e4. \uac00\ub054 \ud3ec\uc778\ud2b8 \uac00\ub4dc \uc5ed\ud560\uc744 \ud558\ub294 \ud504\ub79c\uc2dc\uc2a4\ub294 \ud3c9\uade0 6\ub3c4\uc6c0\uc744 \uae30\ub85d\ud558\ub418 \ub9ce\uc740 \ucf54\uce58\ub4e4\uacfc \uc120\uc218\ub4e4\ub85c\uc11c \uc774\uae30\uc801\uc774\uace0 \uac70\ub9cc\ud558\ub2e4\uace0 \ud3c9\ud310\uc744 \ubc1b\uc558\ub2e4."} {"question": "AV \ub370\ubdd4 \ud6c4 \uc0ac\uce74\ubaa8\ud1a0 \uc720\uc9c0 \uac10\ub3c5\uacfc \uc791\ud488\uc744 \ucd2c\uc601\ud55c \uc2a4\ud29c\ub514\uc624\ub294?", "paragraph": "\uadf8\ub140\ub294 \ud3c9\uc18c AV\ub97c \ubd80\uc815\uc801\uc778 \uc2dc\uc120\uc73c\ub85c \ubcf4\uace0 \uc788\uc5c8\uc9c0\ub9cc, \uc5b4\uba38\ub2c8\ub294 \ub958\uba38\ud2f0\uc2a4 \uad00\uc808\uc5fc\uc744 \uac16\uace0\ub3c4 \uc77c\uc744 \uacc4\uc18d \ud558\uace0 \uc788\uc5c8\uace0 \uc544\ubc84\uc9c0\ub294 \ud38d\uc73c\ub85c \ube5a\uc744 \uc9c4 \uc0c1\ud0dc\uc600\uae30 \ub54c\ubb38\uc5d0, \uac00\uc871\uc744 \ubd80\uc591\ud558\uae30 \uc704\ud574 20\uc138\uac00 \ub420 \ub54c\uae4c\uc9c0\ub9cc \ud65c\ub3d9\ud560 \uc0dd\uac01\uc73c\ub85c AV \ub370\ubdd4\ub97c \uc2b9\ub099\ud588\ub2e4. \ub370\ubdd4\ub294 18\uc0b4\uc774\uc5c8\ub358 2005\ub144 10\uc6d4\uc5d0 \uc774\ub8e8\uc5b4\uc84c\uc73c\uba70, \uc774\ub54c\ubd80\ud130 \ud604\uc7ac\uc758 \uc608\uba85\uc778 \uc544\uc0ac\ubbf8 \uc720\ub9c8\ub97c \uc0ac\uc6a9\ud558\uae30 \uc2dc\uc791\ud588\ub2e4. \uc804\uc18d \uacc4\uc57d\uc744 \ub9fa\uc740 \uacf3\uc740 \uc568\ub9ac\uc2a4 JAPAN \uacfc S1 NO.1 STYLE \uc774\ub77c\ub294 \ub300\ud615 AV \uc2a4\ud29c\ub514\uc624 \ub450 \uacf3\uc774\uc5c8\uace0, \ub450 \uc81c\uc791\uc0ac\uc5d0\uc11c \ud55c \ub2ec\uc5d0 \ud55c \uc791\ud488\uc529 \ub9e4\ub2ec \ub450 \ud3b8 \uc815\ub3c4\ub97c \ucd2c\uc601\ud588\ub2e4. \uc568\ub9ac\uc2a4 JAPAN\uc5d0\uc11c\ub294 \uc0ac\uce74\ubaa8\ud1a0 \uc720\uc9c0 \uac10\ub3c5\uacfc, S1\uc5d0\uc11c\ub294 \uc544\ud0a4 \ud788\ub370\ud1a0 \uac10\ub3c5\uacfc \ud568\uaed8 \ub9ce\uc740 \uc791\ud488\uc744 \ub0c8\ub2e4. \uadf8\ub140\ub294 \ub370\ubdd4 \uc9c1\ud6c4 DVD \ub9e4\ucd9c \ubd80\ubb38\uc5d0\uc11c 1\uc704\ub97c \ucc28\uc9c0\ud558\ub294 \ub4f1 \uc6b0\uc704\ub97c \ubcf4\uc600\uace0, \ub2e8\uc228\uc5d0 \uc778\uae30 \uc5ec\ubc30\uc6b0 \ubc18\uc5f4\uc5d0 \uc62c\ub790\ub2e4. AV\ub97c \ucd2c\uc601\ud558\uba74\uc11c \uc774\ubbf8\uc9c0 \ube44\ub514\uc624 (IV)\ub97c \uba87 \uc791\ud488 \ucd2c\uc601\ud558\uae30\ub3c4 \ud588\ub2e4.", "answer": "\uc568\ub9ac\uc2a4 JAPAN", "sentence": "\uc804\uc18d \uacc4\uc57d\uc744 \ub9fa\uc740 \uacf3\uc740 \uc568\ub9ac\uc2a4 JAPAN \uacfc S1 NO.1 STYLE \uc774\ub77c\ub294 \ub300\ud615 AV \uc2a4\ud29c\ub514\uc624 \ub450 \uacf3\uc774\uc5c8\uace0, \ub450 \uc81c\uc791\uc0ac\uc5d0\uc11c \ud55c \ub2ec\uc5d0 \ud55c \uc791\ud488\uc529 \ub9e4\ub2ec \ub450 \ud3b8 \uc815\ub3c4\ub97c \ucd2c\uc601\ud588\ub2e4.", "paragraph_sentence": "\uadf8\ub140\ub294 \ud3c9\uc18c AV\ub97c \ubd80\uc815\uc801\uc778 \uc2dc\uc120\uc73c\ub85c \ubcf4\uace0 \uc788\uc5c8\uc9c0\ub9cc, \uc5b4\uba38\ub2c8\ub294 \ub958\uba38\ud2f0\uc2a4 \uad00\uc808\uc5fc\uc744 \uac16\uace0\ub3c4 \uc77c\uc744 \uacc4\uc18d \ud558\uace0 \uc788\uc5c8\uace0 \uc544\ubc84\uc9c0\ub294 \ud38d\uc73c\ub85c \ube5a\uc744 \uc9c4 \uc0c1\ud0dc\uc600\uae30 \ub54c\ubb38\uc5d0, \uac00\uc871\uc744 \ubd80\uc591\ud558\uae30 \uc704\ud574 20\uc138\uac00 \ub420 \ub54c\uae4c\uc9c0\ub9cc \ud65c\ub3d9\ud560 \uc0dd\uac01\uc73c\ub85c AV \ub370\ubdd4\ub97c \uc2b9\ub099\ud588\ub2e4. \ub370\ubdd4\ub294 18\uc0b4\uc774\uc5c8\ub358 2005\ub144 10\uc6d4\uc5d0 \uc774\ub8e8\uc5b4\uc84c\uc73c\uba70, \uc774\ub54c\ubd80\ud130 \ud604\uc7ac\uc758 \uc608\uba85\uc778 \uc544\uc0ac\ubbf8 \uc720\ub9c8\ub97c \uc0ac\uc6a9\ud558\uae30 \uc2dc\uc791\ud588\ub2e4. \uc804\uc18d \uacc4\uc57d\uc744 \ub9fa\uc740 \uacf3\uc740 \uc568\ub9ac\uc2a4 JAPAN \uacfc S1 NO.1 STYLE \uc774\ub77c\ub294 \ub300\ud615 AV \uc2a4\ud29c\ub514\uc624 \ub450 \uacf3\uc774\uc5c8\uace0, \ub450 \uc81c\uc791\uc0ac\uc5d0\uc11c \ud55c \ub2ec\uc5d0 \ud55c \uc791\ud488\uc529 \ub9e4\ub2ec \ub450 \ud3b8 \uc815\ub3c4\ub97c \ucd2c\uc601\ud588\ub2e4. \uc568\ub9ac\uc2a4 JAPAN\uc5d0\uc11c\ub294 \uc0ac\uce74\ubaa8\ud1a0 \uc720\uc9c0 \uac10\ub3c5\uacfc, S1\uc5d0\uc11c\ub294 \uc544\ud0a4 \ud788\ub370\ud1a0 \uac10\ub3c5\uacfc \ud568\uaed8 \ub9ce\uc740 \uc791\ud488\uc744 \ub0c8\ub2e4. \uadf8\ub140\ub294 \ub370\ubdd4 \uc9c1\ud6c4 DVD \ub9e4\ucd9c \ubd80\ubb38\uc5d0\uc11c 1\uc704\ub97c \ucc28\uc9c0\ud558\ub294 \ub4f1 \uc6b0\uc704\ub97c \ubcf4\uc600\uace0, \ub2e8\uc228\uc5d0 \uc778\uae30 \uc5ec\ubc30\uc6b0 \ubc18\uc5f4\uc5d0 \uc62c\ub790\ub2e4. AV\ub97c \ucd2c\uc601\ud558\uba74\uc11c \uc774\ubbf8\uc9c0 \ube44\ub514\uc624 (IV)\ub97c \uba87 \uc791\ud488 \ucd2c\uc601\ud558\uae30\ub3c4 \ud588\ub2e4.", "paragraph_answer": "\uadf8\ub140\ub294 \ud3c9\uc18c AV\ub97c \ubd80\uc815\uc801\uc778 \uc2dc\uc120\uc73c\ub85c \ubcf4\uace0 \uc788\uc5c8\uc9c0\ub9cc, \uc5b4\uba38\ub2c8\ub294 \ub958\uba38\ud2f0\uc2a4 \uad00\uc808\uc5fc\uc744 \uac16\uace0\ub3c4 \uc77c\uc744 \uacc4\uc18d \ud558\uace0 \uc788\uc5c8\uace0 \uc544\ubc84\uc9c0\ub294 \ud38d\uc73c\ub85c \ube5a\uc744 \uc9c4 \uc0c1\ud0dc\uc600\uae30 \ub54c\ubb38\uc5d0, \uac00\uc871\uc744 \ubd80\uc591\ud558\uae30 \uc704\ud574 20\uc138\uac00 \ub420 \ub54c\uae4c\uc9c0\ub9cc \ud65c\ub3d9\ud560 \uc0dd\uac01\uc73c\ub85c AV \ub370\ubdd4\ub97c \uc2b9\ub099\ud588\ub2e4. \ub370\ubdd4\ub294 18\uc0b4\uc774\uc5c8\ub358 2005\ub144 10\uc6d4\uc5d0 \uc774\ub8e8\uc5b4\uc84c\uc73c\uba70, \uc774\ub54c\ubd80\ud130 \ud604\uc7ac\uc758 \uc608\uba85\uc778 \uc544\uc0ac\ubbf8 \uc720\ub9c8\ub97c \uc0ac\uc6a9\ud558\uae30 \uc2dc\uc791\ud588\ub2e4. \uc804\uc18d \uacc4\uc57d\uc744 \ub9fa\uc740 \uacf3\uc740 \uc568\ub9ac\uc2a4 JAPAN \uacfc S1 NO.1 STYLE \uc774\ub77c\ub294 \ub300\ud615 AV \uc2a4\ud29c\ub514\uc624 \ub450 \uacf3\uc774\uc5c8\uace0, \ub450 \uc81c\uc791\uc0ac\uc5d0\uc11c \ud55c \ub2ec\uc5d0 \ud55c \uc791\ud488\uc529 \ub9e4\ub2ec \ub450 \ud3b8 \uc815\ub3c4\ub97c \ucd2c\uc601\ud588\ub2e4. \uc568\ub9ac\uc2a4 JAPAN\uc5d0\uc11c\ub294 \uc0ac\uce74\ubaa8\ud1a0 \uc720\uc9c0 \uac10\ub3c5\uacfc, S1\uc5d0\uc11c\ub294 \uc544\ud0a4 \ud788\ub370\ud1a0 \uac10\ub3c5\uacfc \ud568\uaed8 \ub9ce\uc740 \uc791\ud488\uc744 \ub0c8\ub2e4. \uadf8\ub140\ub294 \ub370\ubdd4 \uc9c1\ud6c4 DVD \ub9e4\ucd9c \ubd80\ubb38\uc5d0\uc11c 1\uc704\ub97c \ucc28\uc9c0\ud558\ub294 \ub4f1 \uc6b0\uc704\ub97c \ubcf4\uc600\uace0, \ub2e8\uc228\uc5d0 \uc778\uae30 \uc5ec\ubc30\uc6b0 \ubc18\uc5f4\uc5d0 \uc62c\ub790\ub2e4. AV\ub97c \ucd2c\uc601\ud558\uba74\uc11c \uc774\ubbf8\uc9c0 \ube44\ub514\uc624 (IV)\ub97c \uba87 \uc791\ud488 \ucd2c\uc601\ud558\uae30\ub3c4 \ud588\ub2e4.", "sentence_answer": "\uc804\uc18d \uacc4\uc57d\uc744 \ub9fa\uc740 \uacf3\uc740 \uc568\ub9ac\uc2a4 JAPAN \uacfc S1 NO.1 STYLE \uc774\ub77c\ub294 \ub300\ud615 AV \uc2a4\ud29c\ub514\uc624 \ub450 \uacf3\uc774\uc5c8\uace0, \ub450 \uc81c\uc791\uc0ac\uc5d0\uc11c \ud55c \ub2ec\uc5d0 \ud55c \uc791\ud488\uc529 \ub9e4\ub2ec \ub450 \ud3b8 \uc815\ub3c4\ub97c \ucd2c\uc601\ud588\ub2e4.", "paragraph_id": "6547328-3-1", "paragraph_question": "question: AV \ub370\ubdd4 \ud6c4 \uc0ac\uce74\ubaa8\ud1a0 \uc720\uc9c0 \uac10\ub3c5\uacfc \uc791\ud488\uc744 \ucd2c\uc601\ud55c \uc2a4\ud29c\ub514\uc624\ub294?, context: \uadf8\ub140\ub294 \ud3c9\uc18c AV\ub97c \ubd80\uc815\uc801\uc778 \uc2dc\uc120\uc73c\ub85c \ubcf4\uace0 \uc788\uc5c8\uc9c0\ub9cc, \uc5b4\uba38\ub2c8\ub294 \ub958\uba38\ud2f0\uc2a4 \uad00\uc808\uc5fc\uc744 \uac16\uace0\ub3c4 \uc77c\uc744 \uacc4\uc18d \ud558\uace0 \uc788\uc5c8\uace0 \uc544\ubc84\uc9c0\ub294 \ud38d\uc73c\ub85c \ube5a\uc744 \uc9c4 \uc0c1\ud0dc\uc600\uae30 \ub54c\ubb38\uc5d0, \uac00\uc871\uc744 \ubd80\uc591\ud558\uae30 \uc704\ud574 20\uc138\uac00 \ub420 \ub54c\uae4c\uc9c0\ub9cc \ud65c\ub3d9\ud560 \uc0dd\uac01\uc73c\ub85c AV \ub370\ubdd4\ub97c \uc2b9\ub099\ud588\ub2e4. \ub370\ubdd4\ub294 18\uc0b4\uc774\uc5c8\ub358 2005\ub144 10\uc6d4\uc5d0 \uc774\ub8e8\uc5b4\uc84c\uc73c\uba70, \uc774\ub54c\ubd80\ud130 \ud604\uc7ac\uc758 \uc608\uba85\uc778 \uc544\uc0ac\ubbf8 \uc720\ub9c8\ub97c \uc0ac\uc6a9\ud558\uae30 \uc2dc\uc791\ud588\ub2e4. \uc804\uc18d \uacc4\uc57d\uc744 \ub9fa\uc740 \uacf3\uc740 \uc568\ub9ac\uc2a4 JAPAN \uacfc S1 NO.1 STYLE \uc774\ub77c\ub294 \ub300\ud615 AV \uc2a4\ud29c\ub514\uc624 \ub450 \uacf3\uc774\uc5c8\uace0, \ub450 \uc81c\uc791\uc0ac\uc5d0\uc11c \ud55c \ub2ec\uc5d0 \ud55c \uc791\ud488\uc529 \ub9e4\ub2ec \ub450 \ud3b8 \uc815\ub3c4\ub97c \ucd2c\uc601\ud588\ub2e4. \uc568\ub9ac\uc2a4 JAPAN\uc5d0\uc11c\ub294 \uc0ac\uce74\ubaa8\ud1a0 \uc720\uc9c0 \uac10\ub3c5\uacfc, S1\uc5d0\uc11c\ub294 \uc544\ud0a4 \ud788\ub370\ud1a0 \uac10\ub3c5\uacfc \ud568\uaed8 \ub9ce\uc740 \uc791\ud488\uc744 \ub0c8\ub2e4. \uadf8\ub140\ub294 \ub370\ubdd4 \uc9c1\ud6c4 DVD \ub9e4\ucd9c \ubd80\ubb38\uc5d0\uc11c 1\uc704\ub97c \ucc28\uc9c0\ud558\ub294 \ub4f1 \uc6b0\uc704\ub97c \ubcf4\uc600\uace0, \ub2e8\uc228\uc5d0 \uc778\uae30 \uc5ec\ubc30\uc6b0 \ubc18\uc5f4\uc5d0 \uc62c\ub790\ub2e4. AV\ub97c \ucd2c\uc601\ud558\uba74\uc11c \uc774\ubbf8\uc9c0 \ube44\ub514\uc624 (IV)\ub97c \uba87 \uc791\ud488 \ucd2c\uc601\ud558\uae30\ub3c4 \ud588\ub2e4."} {"question": "\uc720\uae38\uc900\uc774 \uc124\ub9bd.\uc6b4\uc601\ud55c \uad50\uc0ac\uc591\uc131\uae30\uad00\uc740?", "paragraph": "\ub610\ud55c \uadf8\ub294 \uad6d\ubbfc \uac1c\uac1c\uc778\uc744 \uc120\ube44\ub85c \ub9cc\ub4e4\uc790\ub294 \ucde8\uc9c0\ub85c \ud765\uc0ac\ub2e8\uc744 \ucc3d\uc124\ud588\ub2e4. \ud765\uc0ac\ub2e8\uc744 \ud1b5\ud574 \uc720\uae38\uc900\uc740 \u3008\ub300\ud55c\ubb38\uc804 \u5927\u97d3\u6587\u5178\u3009\u00b7\u3008\ub178\ub3d9\uc57c\ud559\ub3c5\ubcf8 \u52de\u52d5\u591c\u5b78\u8b80\u672c\u3009 \ub4f1\uc758 \ucc45\uc744 \uc800\uc220\u00b7\uac04\ud589\ud588\ub2e4. \uadf8\ub9ac\uace0 \uad50\uc0ac\uc591\uc131\uae30\uad00\uc778 \uc0ac\ubc94\ud559\uad50\ub97c \uc124\ub9bd\u00b7\uc6b4\uc601\ud588\uc73c\uba70, \uc18c\ud559\uad50 \uad50\uc721\uc744 \uc815\uc0c1\ud654\ud558\ub824\uace0 \ud588\ub2e4. 1908\ub144 11\uc6d4 \uc758\ubcd1\uc7a5 \uc2e0\ub3cc\uc11d\uc774 \ud55c\uad6d \uc0ac\ub78c\uc758 \ubc00\uace0\ub85c \uc77c\ubcf8\uc5d0 \uccb4\ud3ec\ub418\uc5b4 \ucc98\ud615\ub2f9\ud588\ub2e4\ub294 \uc18c\uc2dd\uc744 \ub4e3\uace0 \uadf8\ub294 \uc88c\uc808\ud55c\ub2e4. \uc2e0\ub3cc\uc11d\uc744 \ubc00\uace0\ud55c \uc790\ub294 \uadf8\uc758 \uce5c\ucc99\uc73c\ub85c \ud604\uc0c1\uae08\uc744 \ub178\ub9b0 \ud615\uc81c\uc640, \uc591\ubc18 \ucd9c\uc2e0\uc73c\ub85c \uc0c1\ubbfc\uc758 \uc9c0\ud718\ub97c \ubc1b\ub294 \uac83\uc744 \ubd88\ucf8c\ud558\uac8c \uc0dd\uac01\ud55c \uc591\ubc18\ub4e4\uc774\uc5c8\ub2e4. \uc2e0\ub3cc\uc11d\uc758 \uccb4\ud3ec \uc18c\uc2dd\uc5d0 \uadf8\ub294 \ub2e4\uc2dc\ud55c\ubc88 \ud06c\uac8c \uc2e4\ub9dd\ud558\uac8c \ub41c\ub2e4. \uc2e0\ub3cc\uc11d\uc758 \uccb4\ud3ec\ub294 \uc544\uc9c1 \uc870\uc120\uc774 \ubbfc\ub3c4\uac00 \ub0ae\ub2e4\ub294 \uc99d\uac70\uc774\uba70 \uc774\ub7f0 \uc218\uc900\uc73c\ub85c\ub294 \ub3c5\ub9bd\uc740 \ub2ec\uc131\ud560 \uc218\ub3c4 \uc5c6\uace0 \ub3c5\ub9bd\ud558\ub354\ub77c\ub3c4 \uacc4\uce35\uac08\ub4f1\uc73c\ub85c \uc0ac\ud68c\uac00 \uacf5\uc911\ubd84\ud574\ub418\uac70\ub098 \ub7ec\uc2dc\uc544\ub098 \ub2e4\uc2dc \uccad\ub098\ub77c\uc758 \ub178\uc608\uac00 \ub420 \uac83\uc774\ub77c\uace0 \uc804\ub9dd\ud588\ub2e4. 1908\ub144\ub9d0 \uc77c\ubcf8 \uace0\uad00\ub4e4\uc774 \ub300\ud55c\uc81c\uad6d\uc744 \ubc29\ubb38\ud558\uc600\ub2e4. \uadf8\ub294 1908\ub144\uc5d0 \uc790\uc2e0\uc774 \uc870\uc9c1\ud55c \ud55c\uc131\ubd80\ubbfc\ud68c\uc758 \ucc28\uc6d0\uc5d0\uc11c \uc77c\ubcf8 \uace0\uad00\ub4e4\uc758 \ud658\uc601\uc2dd\uc5d0 '\ud559\uc0dd\uacfc \uc8fc\ubbfc\ub4e4\uc744 \ubc18\uac15\uc81c\uc801\uc73c\ub85c \ub3d9\uc6d0'\ud558\uc600\ub2e4.", "answer": "\uc0ac\ubc94\ud559\uad50", "sentence": "\uadf8\ub9ac\uace0 \uad50\uc0ac\uc591\uc131\uae30\uad00\uc778 \uc0ac\ubc94\ud559\uad50 \ub97c \uc124\ub9bd\u00b7\uc6b4\uc601\ud588\uc73c\uba70, \uc18c\ud559\uad50 \uad50\uc721\uc744 \uc815\uc0c1\ud654\ud558\ub824\uace0 \ud588\ub2e4.", "paragraph_sentence": "\ub610\ud55c \uadf8\ub294 \uad6d\ubbfc \uac1c\uac1c\uc778\uc744 \uc120\ube44\ub85c \ub9cc\ub4e4\uc790\ub294 \ucde8\uc9c0\ub85c \ud765\uc0ac\ub2e8\uc744 \ucc3d\uc124\ud588\ub2e4. \ud765\uc0ac\ub2e8\uc744 \ud1b5\ud574 \uc720\uae38\uc900\uc740 \u3008\ub300\ud55c\ubb38\uc804 \u5927\u97d3\u6587\u5178\u3009\u00b7\u3008\ub178\ub3d9\uc57c\ud559\ub3c5\ubcf8 \u52de\u52d5\u591c\u5b78\u8b80\u672c\u3009 \ub4f1\uc758 \ucc45\uc744 \uc800\uc220\u00b7\uac04\ud589\ud588\ub2e4. \uadf8\ub9ac\uace0 \uad50\uc0ac\uc591\uc131\uae30\uad00\uc778 \uc0ac\ubc94\ud559\uad50 \ub97c \uc124\ub9bd\u00b7\uc6b4\uc601\ud588\uc73c\uba70, \uc18c\ud559\uad50 \uad50\uc721\uc744 \uc815\uc0c1\ud654\ud558\ub824\uace0 \ud588\ub2e4. 1908\ub144 11\uc6d4 \uc758\ubcd1\uc7a5 \uc2e0\ub3cc\uc11d\uc774 \ud55c\uad6d \uc0ac\ub78c\uc758 \ubc00\uace0\ub85c \uc77c\ubcf8\uc5d0 \uccb4\ud3ec\ub418\uc5b4 \ucc98\ud615\ub2f9\ud588\ub2e4\ub294 \uc18c\uc2dd\uc744 \ub4e3\uace0 \uadf8\ub294 \uc88c\uc808\ud55c\ub2e4. \uc2e0\ub3cc\uc11d\uc744 \ubc00\uace0\ud55c \uc790\ub294 \uadf8\uc758 \uce5c\ucc99\uc73c\ub85c \ud604\uc0c1\uae08\uc744 \ub178\ub9b0 \ud615\uc81c\uc640, \uc591\ubc18 \ucd9c\uc2e0\uc73c\ub85c \uc0c1\ubbfc\uc758 \uc9c0\ud718\ub97c \ubc1b\ub294 \uac83\uc744 \ubd88\ucf8c\ud558\uac8c \uc0dd\uac01\ud55c \uc591\ubc18\ub4e4\uc774\uc5c8\ub2e4. \uc2e0\ub3cc\uc11d\uc758 \uccb4\ud3ec \uc18c\uc2dd\uc5d0 \uadf8\ub294 \ub2e4\uc2dc\ud55c\ubc88 \ud06c\uac8c \uc2e4\ub9dd\ud558\uac8c \ub41c\ub2e4. \uc2e0\ub3cc\uc11d\uc758 \uccb4\ud3ec\ub294 \uc544\uc9c1 \uc870\uc120\uc774 \ubbfc\ub3c4\uac00 \ub0ae\ub2e4\ub294 \uc99d\uac70\uc774\uba70 \uc774\ub7f0 \uc218\uc900\uc73c\ub85c\ub294 \ub3c5\ub9bd\uc740 \ub2ec\uc131\ud560 \uc218\ub3c4 \uc5c6\uace0 \ub3c5\ub9bd\ud558\ub354\ub77c\ub3c4 \uacc4\uce35\uac08\ub4f1\uc73c\ub85c \uc0ac\ud68c\uac00 \uacf5\uc911\ubd84\ud574\ub418\uac70\ub098 \ub7ec\uc2dc\uc544\ub098 \ub2e4\uc2dc \uccad\ub098\ub77c\uc758 \ub178\uc608\uac00 \ub420 \uac83\uc774\ub77c\uace0 \uc804\ub9dd\ud588\ub2e4. 1908\ub144\ub9d0 \uc77c\ubcf8 \uace0\uad00\ub4e4\uc774 \ub300\ud55c\uc81c\uad6d\uc744 \ubc29\ubb38\ud558\uc600\ub2e4. \uadf8\ub294 1908\ub144\uc5d0 \uc790\uc2e0\uc774 \uc870\uc9c1\ud55c \ud55c\uc131\ubd80\ubbfc\ud68c\uc758 \ucc28\uc6d0\uc5d0\uc11c \uc77c\ubcf8 \uace0\uad00\ub4e4\uc758 \ud658\uc601\uc2dd\uc5d0 '\ud559\uc0dd\uacfc \uc8fc\ubbfc\ub4e4\uc744 \ubc18\uac15\uc81c\uc801\uc73c\ub85c \ub3d9\uc6d0'\ud558\uc600\ub2e4.", "paragraph_answer": "\ub610\ud55c \uadf8\ub294 \uad6d\ubbfc \uac1c\uac1c\uc778\uc744 \uc120\ube44\ub85c \ub9cc\ub4e4\uc790\ub294 \ucde8\uc9c0\ub85c \ud765\uc0ac\ub2e8\uc744 \ucc3d\uc124\ud588\ub2e4. \ud765\uc0ac\ub2e8\uc744 \ud1b5\ud574 \uc720\uae38\uc900\uc740 \u3008\ub300\ud55c\ubb38\uc804 \u5927\u97d3\u6587\u5178\u3009\u00b7\u3008\ub178\ub3d9\uc57c\ud559\ub3c5\ubcf8 \u52de\u52d5\u591c\u5b78\u8b80\u672c\u3009 \ub4f1\uc758 \ucc45\uc744 \uc800\uc220\u00b7\uac04\ud589\ud588\ub2e4. \uadf8\ub9ac\uace0 \uad50\uc0ac\uc591\uc131\uae30\uad00\uc778 \uc0ac\ubc94\ud559\uad50 \ub97c \uc124\ub9bd\u00b7\uc6b4\uc601\ud588\uc73c\uba70, \uc18c\ud559\uad50 \uad50\uc721\uc744 \uc815\uc0c1\ud654\ud558\ub824\uace0 \ud588\ub2e4. 1908\ub144 11\uc6d4 \uc758\ubcd1\uc7a5 \uc2e0\ub3cc\uc11d\uc774 \ud55c\uad6d \uc0ac\ub78c\uc758 \ubc00\uace0\ub85c \uc77c\ubcf8\uc5d0 \uccb4\ud3ec\ub418\uc5b4 \ucc98\ud615\ub2f9\ud588\ub2e4\ub294 \uc18c\uc2dd\uc744 \ub4e3\uace0 \uadf8\ub294 \uc88c\uc808\ud55c\ub2e4. \uc2e0\ub3cc\uc11d\uc744 \ubc00\uace0\ud55c \uc790\ub294 \uadf8\uc758 \uce5c\ucc99\uc73c\ub85c \ud604\uc0c1\uae08\uc744 \ub178\ub9b0 \ud615\uc81c\uc640, \uc591\ubc18 \ucd9c\uc2e0\uc73c\ub85c \uc0c1\ubbfc\uc758 \uc9c0\ud718\ub97c \ubc1b\ub294 \uac83\uc744 \ubd88\ucf8c\ud558\uac8c \uc0dd\uac01\ud55c \uc591\ubc18\ub4e4\uc774\uc5c8\ub2e4. \uc2e0\ub3cc\uc11d\uc758 \uccb4\ud3ec \uc18c\uc2dd\uc5d0 \uadf8\ub294 \ub2e4\uc2dc\ud55c\ubc88 \ud06c\uac8c \uc2e4\ub9dd\ud558\uac8c \ub41c\ub2e4. \uc2e0\ub3cc\uc11d\uc758 \uccb4\ud3ec\ub294 \uc544\uc9c1 \uc870\uc120\uc774 \ubbfc\ub3c4\uac00 \ub0ae\ub2e4\ub294 \uc99d\uac70\uc774\uba70 \uc774\ub7f0 \uc218\uc900\uc73c\ub85c\ub294 \ub3c5\ub9bd\uc740 \ub2ec\uc131\ud560 \uc218\ub3c4 \uc5c6\uace0 \ub3c5\ub9bd\ud558\ub354\ub77c\ub3c4 \uacc4\uce35\uac08\ub4f1\uc73c\ub85c \uc0ac\ud68c\uac00 \uacf5\uc911\ubd84\ud574\ub418\uac70\ub098 \ub7ec\uc2dc\uc544\ub098 \ub2e4\uc2dc \uccad\ub098\ub77c\uc758 \ub178\uc608\uac00 \ub420 \uac83\uc774\ub77c\uace0 \uc804\ub9dd\ud588\ub2e4. 1908\ub144\ub9d0 \uc77c\ubcf8 \uace0\uad00\ub4e4\uc774 \ub300\ud55c\uc81c\uad6d\uc744 \ubc29\ubb38\ud558\uc600\ub2e4. \uadf8\ub294 1908\ub144\uc5d0 \uc790\uc2e0\uc774 \uc870\uc9c1\ud55c \ud55c\uc131\ubd80\ubbfc\ud68c\uc758 \ucc28\uc6d0\uc5d0\uc11c \uc77c\ubcf8 \uace0\uad00\ub4e4\uc758 \ud658\uc601\uc2dd\uc5d0 '\ud559\uc0dd\uacfc \uc8fc\ubbfc\ub4e4\uc744 \ubc18\uac15\uc81c\uc801\uc73c\ub85c \ub3d9\uc6d0'\ud558\uc600\ub2e4.", "sentence_answer": "\uadf8\ub9ac\uace0 \uad50\uc0ac\uc591\uc131\uae30\uad00\uc778 \uc0ac\ubc94\ud559\uad50 \ub97c \uc124\ub9bd\u00b7\uc6b4\uc601\ud588\uc73c\uba70, \uc18c\ud559\uad50 \uad50\uc721\uc744 \uc815\uc0c1\ud654\ud558\ub824\uace0 \ud588\ub2e4.", "paragraph_id": "6542752-23-2", "paragraph_question": "question: \uc720\uae38\uc900\uc774 \uc124\ub9bd.\uc6b4\uc601\ud55c \uad50\uc0ac\uc591\uc131\uae30\uad00\uc740?, context: \ub610\ud55c \uadf8\ub294 \uad6d\ubbfc \uac1c\uac1c\uc778\uc744 \uc120\ube44\ub85c \ub9cc\ub4e4\uc790\ub294 \ucde8\uc9c0\ub85c \ud765\uc0ac\ub2e8\uc744 \ucc3d\uc124\ud588\ub2e4. \ud765\uc0ac\ub2e8\uc744 \ud1b5\ud574 \uc720\uae38\uc900\uc740 \u3008\ub300\ud55c\ubb38\uc804 \u5927\u97d3\u6587\u5178\u3009\u00b7\u3008\ub178\ub3d9\uc57c\ud559\ub3c5\ubcf8 \u52de\u52d5\u591c\u5b78\u8b80\u672c\u3009 \ub4f1\uc758 \ucc45\uc744 \uc800\uc220\u00b7\uac04\ud589\ud588\ub2e4. \uadf8\ub9ac\uace0 \uad50\uc0ac\uc591\uc131\uae30\uad00\uc778 \uc0ac\ubc94\ud559\uad50\ub97c \uc124\ub9bd\u00b7\uc6b4\uc601\ud588\uc73c\uba70, \uc18c\ud559\uad50 \uad50\uc721\uc744 \uc815\uc0c1\ud654\ud558\ub824\uace0 \ud588\ub2e4. 1908\ub144 11\uc6d4 \uc758\ubcd1\uc7a5 \uc2e0\ub3cc\uc11d\uc774 \ud55c\uad6d \uc0ac\ub78c\uc758 \ubc00\uace0\ub85c \uc77c\ubcf8\uc5d0 \uccb4\ud3ec\ub418\uc5b4 \ucc98\ud615\ub2f9\ud588\ub2e4\ub294 \uc18c\uc2dd\uc744 \ub4e3\uace0 \uadf8\ub294 \uc88c\uc808\ud55c\ub2e4. \uc2e0\ub3cc\uc11d\uc744 \ubc00\uace0\ud55c \uc790\ub294 \uadf8\uc758 \uce5c\ucc99\uc73c\ub85c \ud604\uc0c1\uae08\uc744 \ub178\ub9b0 \ud615\uc81c\uc640, \uc591\ubc18 \ucd9c\uc2e0\uc73c\ub85c \uc0c1\ubbfc\uc758 \uc9c0\ud718\ub97c \ubc1b\ub294 \uac83\uc744 \ubd88\ucf8c\ud558\uac8c \uc0dd\uac01\ud55c \uc591\ubc18\ub4e4\uc774\uc5c8\ub2e4. \uc2e0\ub3cc\uc11d\uc758 \uccb4\ud3ec \uc18c\uc2dd\uc5d0 \uadf8\ub294 \ub2e4\uc2dc\ud55c\ubc88 \ud06c\uac8c \uc2e4\ub9dd\ud558\uac8c \ub41c\ub2e4. \uc2e0\ub3cc\uc11d\uc758 \uccb4\ud3ec\ub294 \uc544\uc9c1 \uc870\uc120\uc774 \ubbfc\ub3c4\uac00 \ub0ae\ub2e4\ub294 \uc99d\uac70\uc774\uba70 \uc774\ub7f0 \uc218\uc900\uc73c\ub85c\ub294 \ub3c5\ub9bd\uc740 \ub2ec\uc131\ud560 \uc218\ub3c4 \uc5c6\uace0 \ub3c5\ub9bd\ud558\ub354\ub77c\ub3c4 \uacc4\uce35\uac08\ub4f1\uc73c\ub85c \uc0ac\ud68c\uac00 \uacf5\uc911\ubd84\ud574\ub418\uac70\ub098 \ub7ec\uc2dc\uc544\ub098 \ub2e4\uc2dc \uccad\ub098\ub77c\uc758 \ub178\uc608\uac00 \ub420 \uac83\uc774\ub77c\uace0 \uc804\ub9dd\ud588\ub2e4. 1908\ub144\ub9d0 \uc77c\ubcf8 \uace0\uad00\ub4e4\uc774 \ub300\ud55c\uc81c\uad6d\uc744 \ubc29\ubb38\ud558\uc600\ub2e4. \uadf8\ub294 1908\ub144\uc5d0 \uc790\uc2e0\uc774 \uc870\uc9c1\ud55c \ud55c\uc131\ubd80\ubbfc\ud68c\uc758 \ucc28\uc6d0\uc5d0\uc11c \uc77c\ubcf8 \uace0\uad00\ub4e4\uc758 \ud658\uc601\uc2dd\uc5d0 '\ud559\uc0dd\uacfc \uc8fc\ubbfc\ub4e4\uc744 \ubc18\uac15\uc81c\uc801\uc73c\ub85c \ub3d9\uc6d0'\ud558\uc600\ub2e4."} {"question": "\ud1a0\uba38\uc2a4 \uae00\ub7ec\ubc84\uc758 \uc18c\uac1c\ub85c \ud574\ub9ac \ud30c\ud06c\uc2a4\ub294 \uc5b4\ub514\ub97c \ubc29\ubb38\ud588\ub098?", "paragraph": "1866\ub144(\uac8c\uc774\uc624 2\ub144) \uc601\ubd88\ub124 3\uad6d \uc5f0\ud569\uc740 \ub9c9\ubd80\uc640 \uad00\uc138\uc728 \uac1c\uc815\uc5d0 \uc870\uc778\ud588\ub2e4. \ud30c\ud06c\uc2a4\ub294 \ud1a0\uba38\uc2a4 \uae00\ub7ec\ubc84\uc758 \uc18c\uac1c\ub85c \uc0ac\uc4f0\ub9c8 \ubc88\uc744 \ubc29\ubb38\ud574 \ubc88\uc8fc \uc2dc\ub9c8\uc988 \ub2e4\ub2e4\uc694\uc2dc(\u5cf6\u6d25\u5fe0\u7fa9)\ubc0f \uc2dc\ub9c8\uc988 \ud788\uc0ac\ubbf8\uc4f0, \uc0ac\uc774\uace0 \ub2e4\uce74\ubaa8\ub9ac, \ub370\ub77c\uc9c0\ub9c8 \ubb34\ub124\ub178\ub9ac \ub4f1\uacfc \ub9cc\ub0ac\ub2e4. 2\ucc28 \uc870\uc288 \uc815\ubc8c \ubc1c\ubc1c \uc9c1\ud6c4 \ud504\ub791\uc2a4 \uacf5\uc0ac \ub808\uc639 \ub85c\uc288(Michel Jules Marie L\u00e9on Roches,1809\ub144-1900\ub144)\uc640 \ud568\uaed8 \uc2dc\ubaa8\ub178\uc138\ud0a4\uc5d0\uc11c \uae30\ub3c4 \ub2e4\uce74\uc694\uc2dc,\uc774\ud1a0 \ud788\ub85c\ubd80\ubbf8\ub97c, \uc624\ucfe0\ub77c \ubc29\uba74 \ub9c9\ubd80 \uc9c4\uc9c0\uc5d0\uc11c\ub294 \ub85c\uc8fc(\u8001\u4e2d) \uc624\uac00\uc0ac\uc640\ub77c \ub098\uac00\uc720\ud0a4\uc640 \uc5f0\uc774\uc5b4 \ud68c\ub2f4\ud558\ub294 \ub4f1 \uc591\uc790 \ucda9\ub3cc\uc744 \ub9c9\uc73c\ub824 \ud588\uc73c\ub098 \uc2e4\ud328\ud588\ub2e4. \uadf8 \ud6c4 \uc6b0\uc640\uc9c0\ub9c8 \ubc88\uc744 \ubc29\ubb38\ud574 \ub2e4\ud14c \ubb34\ub124\ub098\ub9ac\uc640\ub3c4 \ub9cc\ub0ac\ub2e4. \uc5f0\ub9d0\uc5d0\ub294 \uc601\uad6d\uacf5\uc0ac\uad00\uc758 \uc5d0\ub3c4 \uc774\uc804\uc774 \uc788\uc5c8\ub2e4.", "answer": "\uc0ac\uc4f0\ub9c8 \ubc88", "sentence": "\ud30c\ud06c\uc2a4\ub294 \ud1a0\uba38\uc2a4 \uae00\ub7ec\ubc84\uc758 \uc18c\uac1c\ub85c \uc0ac\uc4f0\ub9c8 \ubc88 \uc744 \ubc29\ubb38\ud574 \ubc88\uc8fc \uc2dc\ub9c8\uc988 \ub2e4\ub2e4\uc694\uc2dc(\u5cf6\u6d25\u5fe0\u7fa9)\ubc0f \uc2dc\ub9c8\uc988 \ud788\uc0ac\ubbf8\uc4f0, \uc0ac\uc774\uace0 \ub2e4\uce74\ubaa8\ub9ac, \ub370\ub77c\uc9c0\ub9c8 \ubb34\ub124\ub178\ub9ac \ub4f1\uacfc \ub9cc\ub0ac\ub2e4.", "paragraph_sentence": "1866\ub144(\uac8c\uc774\uc624 2\ub144) \uc601\ubd88\ub124 3\uad6d \uc5f0\ud569\uc740 \ub9c9\ubd80\uc640 \uad00\uc138\uc728 \uac1c\uc815\uc5d0 \uc870\uc778\ud588\ub2e4. \ud30c\ud06c\uc2a4\ub294 \ud1a0\uba38\uc2a4 \uae00\ub7ec\ubc84\uc758 \uc18c\uac1c\ub85c \uc0ac\uc4f0\ub9c8 \ubc88 \uc744 \ubc29\ubb38\ud574 \ubc88\uc8fc \uc2dc\ub9c8\uc988 \ub2e4\ub2e4\uc694\uc2dc(\u5cf6\u6d25\u5fe0\u7fa9)\ubc0f \uc2dc\ub9c8\uc988 \ud788\uc0ac\ubbf8\uc4f0, \uc0ac\uc774\uace0 \ub2e4\uce74\ubaa8\ub9ac, \ub370\ub77c\uc9c0\ub9c8 \ubb34\ub124\ub178\ub9ac \ub4f1\uacfc \ub9cc\ub0ac\ub2e4. 2\ucc28 \uc870\uc288 \uc815\ubc8c \ubc1c\ubc1c \uc9c1\ud6c4 \ud504\ub791\uc2a4 \uacf5\uc0ac \ub808\uc639 \ub85c\uc288(Michel Jules Marie L\u00e9on Roches,1809\ub144-1900\ub144)\uc640 \ud568\uaed8 \uc2dc\ubaa8\ub178\uc138\ud0a4\uc5d0\uc11c \uae30\ub3c4 \ub2e4\uce74\uc694\uc2dc,\uc774\ud1a0 \ud788\ub85c\ubd80\ubbf8\ub97c, \uc624\ucfe0\ub77c \ubc29\uba74 \ub9c9\ubd80 \uc9c4\uc9c0\uc5d0\uc11c\ub294 \ub85c\uc8fc(\u8001\u4e2d) \uc624\uac00\uc0ac\uc640\ub77c \ub098\uac00\uc720\ud0a4\uc640 \uc5f0\uc774\uc5b4 \ud68c\ub2f4\ud558\ub294 \ub4f1 \uc591\uc790 \ucda9\ub3cc\uc744 \ub9c9\uc73c\ub824 \ud588\uc73c\ub098 \uc2e4\ud328\ud588\ub2e4. \uadf8 \ud6c4 \uc6b0\uc640\uc9c0\ub9c8 \ubc88\uc744 \ubc29\ubb38\ud574 \ub2e4\ud14c \ubb34\ub124\ub098\ub9ac\uc640\ub3c4 \ub9cc\ub0ac\ub2e4. \uc5f0\ub9d0\uc5d0\ub294 \uc601\uad6d\uacf5\uc0ac\uad00\uc758 \uc5d0\ub3c4 \uc774\uc804\uc774 \uc788\uc5c8\ub2e4.", "paragraph_answer": "1866\ub144(\uac8c\uc774\uc624 2\ub144) \uc601\ubd88\ub124 3\uad6d \uc5f0\ud569\uc740 \ub9c9\ubd80\uc640 \uad00\uc138\uc728 \uac1c\uc815\uc5d0 \uc870\uc778\ud588\ub2e4. \ud30c\ud06c\uc2a4\ub294 \ud1a0\uba38\uc2a4 \uae00\ub7ec\ubc84\uc758 \uc18c\uac1c\ub85c \uc0ac\uc4f0\ub9c8 \ubc88 \uc744 \ubc29\ubb38\ud574 \ubc88\uc8fc \uc2dc\ub9c8\uc988 \ub2e4\ub2e4\uc694\uc2dc(\u5cf6\u6d25\u5fe0\u7fa9)\ubc0f \uc2dc\ub9c8\uc988 \ud788\uc0ac\ubbf8\uc4f0, \uc0ac\uc774\uace0 \ub2e4\uce74\ubaa8\ub9ac, \ub370\ub77c\uc9c0\ub9c8 \ubb34\ub124\ub178\ub9ac \ub4f1\uacfc \ub9cc\ub0ac\ub2e4. 2\ucc28 \uc870\uc288 \uc815\ubc8c \ubc1c\ubc1c \uc9c1\ud6c4 \ud504\ub791\uc2a4 \uacf5\uc0ac \ub808\uc639 \ub85c\uc288(Michel Jules Marie L\u00e9on Roches,1809\ub144-1900\ub144)\uc640 \ud568\uaed8 \uc2dc\ubaa8\ub178\uc138\ud0a4\uc5d0\uc11c \uae30\ub3c4 \ub2e4\uce74\uc694\uc2dc,\uc774\ud1a0 \ud788\ub85c\ubd80\ubbf8\ub97c, \uc624\ucfe0\ub77c \ubc29\uba74 \ub9c9\ubd80 \uc9c4\uc9c0\uc5d0\uc11c\ub294 \ub85c\uc8fc(\u8001\u4e2d) \uc624\uac00\uc0ac\uc640\ub77c \ub098\uac00\uc720\ud0a4\uc640 \uc5f0\uc774\uc5b4 \ud68c\ub2f4\ud558\ub294 \ub4f1 \uc591\uc790 \ucda9\ub3cc\uc744 \ub9c9\uc73c\ub824 \ud588\uc73c\ub098 \uc2e4\ud328\ud588\ub2e4. \uadf8 \ud6c4 \uc6b0\uc640\uc9c0\ub9c8 \ubc88\uc744 \ubc29\ubb38\ud574 \ub2e4\ud14c \ubb34\ub124\ub098\ub9ac\uc640\ub3c4 \ub9cc\ub0ac\ub2e4. \uc5f0\ub9d0\uc5d0\ub294 \uc601\uad6d\uacf5\uc0ac\uad00\uc758 \uc5d0\ub3c4 \uc774\uc804\uc774 \uc788\uc5c8\ub2e4.", "sentence_answer": "\ud30c\ud06c\uc2a4\ub294 \ud1a0\uba38\uc2a4 \uae00\ub7ec\ubc84\uc758 \uc18c\uac1c\ub85c \uc0ac\uc4f0\ub9c8 \ubc88 \uc744 \ubc29\ubb38\ud574 \ubc88\uc8fc \uc2dc\ub9c8\uc988 \ub2e4\ub2e4\uc694\uc2dc(\u5cf6\u6d25\u5fe0\u7fa9)\ubc0f \uc2dc\ub9c8\uc988 \ud788\uc0ac\ubbf8\uc4f0, \uc0ac\uc774\uace0 \ub2e4\uce74\ubaa8\ub9ac, \ub370\ub77c\uc9c0\ub9c8 \ubb34\ub124\ub178\ub9ac \ub4f1\uacfc \ub9cc\ub0ac\ub2e4.", "paragraph_id": "6570476-7-1", "paragraph_question": "question: \ud1a0\uba38\uc2a4 \uae00\ub7ec\ubc84\uc758 \uc18c\uac1c\ub85c \ud574\ub9ac \ud30c\ud06c\uc2a4\ub294 \uc5b4\ub514\ub97c \ubc29\ubb38\ud588\ub098?, context: 1866\ub144(\uac8c\uc774\uc624 2\ub144) \uc601\ubd88\ub124 3\uad6d \uc5f0\ud569\uc740 \ub9c9\ubd80\uc640 \uad00\uc138\uc728 \uac1c\uc815\uc5d0 \uc870\uc778\ud588\ub2e4. \ud30c\ud06c\uc2a4\ub294 \ud1a0\uba38\uc2a4 \uae00\ub7ec\ubc84\uc758 \uc18c\uac1c\ub85c \uc0ac\uc4f0\ub9c8 \ubc88\uc744 \ubc29\ubb38\ud574 \ubc88\uc8fc \uc2dc\ub9c8\uc988 \ub2e4\ub2e4\uc694\uc2dc(\u5cf6\u6d25\u5fe0\u7fa9)\ubc0f \uc2dc\ub9c8\uc988 \ud788\uc0ac\ubbf8\uc4f0, \uc0ac\uc774\uace0 \ub2e4\uce74\ubaa8\ub9ac, \ub370\ub77c\uc9c0\ub9c8 \ubb34\ub124\ub178\ub9ac \ub4f1\uacfc \ub9cc\ub0ac\ub2e4. 2\ucc28 \uc870\uc288 \uc815\ubc8c \ubc1c\ubc1c \uc9c1\ud6c4 \ud504\ub791\uc2a4 \uacf5\uc0ac \ub808\uc639 \ub85c\uc288(Michel Jules Marie L\u00e9on Roches,1809\ub144-1900\ub144)\uc640 \ud568\uaed8 \uc2dc\ubaa8\ub178\uc138\ud0a4\uc5d0\uc11c \uae30\ub3c4 \ub2e4\uce74\uc694\uc2dc,\uc774\ud1a0 \ud788\ub85c\ubd80\ubbf8\ub97c, \uc624\ucfe0\ub77c \ubc29\uba74 \ub9c9\ubd80 \uc9c4\uc9c0\uc5d0\uc11c\ub294 \ub85c\uc8fc(\u8001\u4e2d) \uc624\uac00\uc0ac\uc640\ub77c \ub098\uac00\uc720\ud0a4\uc640 \uc5f0\uc774\uc5b4 \ud68c\ub2f4\ud558\ub294 \ub4f1 \uc591\uc790 \ucda9\ub3cc\uc744 \ub9c9\uc73c\ub824 \ud588\uc73c\ub098 \uc2e4\ud328\ud588\ub2e4. \uadf8 \ud6c4 \uc6b0\uc640\uc9c0\ub9c8 \ubc88\uc744 \ubc29\ubb38\ud574 \ub2e4\ud14c \ubb34\ub124\ub098\ub9ac\uc640\ub3c4 \ub9cc\ub0ac\ub2e4. \uc5f0\ub9d0\uc5d0\ub294 \uc601\uad6d\uacf5\uc0ac\uad00\uc758 \uc5d0\ub3c4 \uc774\uc804\uc774 \uc788\uc5c8\ub2e4."} {"question": "\uc0cc\uc988\uac00 \uc120\uac70\uc5d0\uc11c 30,492\ud45c\ub97c \ub4dd\ud45c\ud558\uc5ec \uc785\uc131\ud55c \uacf3\uc740?", "paragraph": "\uacf5\ud654\uc8fc\uc758 \uc6b4\ub3d9 \uce21\uc5d0\uc11c\ub294 \uc81c2\ucc28 \ub2e8\uc2dd\ud22c\uc7c1\uc5d0 \ub300\ud55c \ub300\uc911\uc801 \uc9c0\uc9c0\ub97c \ubaa8\uc73c\uae30 \uc704\ud574 \ud798\uc37c\ub2e4. \uc0cc\uc988\uac00 \ub2e8\uc2dd\ud22c\uc7c1\uc744 \uc2dc\uc791\ud558\uae30 \uc804 \uc8fc \uc77c\uc694\uc77c\uc5d0 3,500\uba85\uc758 \uc0ac\ub78c\ub4e4\uc774 \ubca8\ud30c\uc2a4\ud2b8 \uc11c\ubd80 \uc9c0\uad6c\ub97c \ud589\uc9c4\ud588\ub2e4. 4\uac1c\uc6d4 \uc804\uc778 \uc81c1\ucc28 \ub2e8\uc2dd\ud22c\uc7c1 \ub54c\uc5d0\ub294 10,000\uc5ec \uba85\uc774 \ud589\uc9c4\uc5d0 \ucc38\uc5ec\ud588\uc5c8\ub2e4. \uadf8\ub7f0\ub370 \ub2e8\uc2dd\ud22c\uc7c1 \uc2dc\uc791 5\uc77c \ucc28 \ub54c \ud37c\ub9e4\ub108 \uc0ac\uc6b0\uc2a4\ud2f0\ub860 \uc120\uac70\uad6c\ub97c \uc9c0\uc5ed\uad6c\ub85c \ud558\ub294 \uacf5\ud654\ud30c \uc11c\ubbfc\uc6d0\uc758\uc6d0 \ud504\ub7ad\ud06c \uba38\uacfc\uc774\uc5b4\uac00 \uc0ac\ub9dd\ud588\uace0, \uc774\uc5d0 \ub530\ub77c \ubcf4\uad90\uc120\uac70\uac00 \uc5f4\ub9ac\uac8c \ub418\uc5c8\ub2e4. \ubbfc\uc871\uc8fc\uc758\uc790\ub4e4\uacfc \uacf5\ud654\uc8fc\uc758\uc790\ub4e4 \uc0ac\uc774\uc5d0 \ub204\uad6c\ub97c \ubcf4\uad90\uc120\uac70 \ud6c4\ubcf4\ub85c \ub0b4\ubcf4\ub0b4\uc57c \ud558\ub294\uc9c0\uc5d0 \ub300\ud55c \ub17c\uc7c1\uc774 \uc77c\uc5b4\ub0ac\ub2e4. \uc0ac\ud68c\ubbfc\uc8fc\ub178\ub3d9\ub2f9\uc758 \uc624\uc2a4\ud2f4 \ucee4\ub9ac\uac00 \uad00\uc2ec\uc744 \ud45c\ud588\uace0, \ubca0\ub974\ub098\ub370\ud2b8 \ub9e5\uce7c\ub9ac\uc2a4\ud0a4\uc640 \uc0ac\ub9dd\ud55c \uba38\uacfc\uc774\uc5b4\uc758 \ud615\uc81c \ub178\uc5d8 \uba38\uacfc\uc774\uc5b4\ub3c4 \uc758\uc0ac\ub97c \ubc1d\ud614\ub2e4. \ud611\uc0c1\uc774 \uc774\ub8e8\uc5b4\uc9c0\uace0, \ud55c\ud3b8 \ub178\uc5d8 \uba38\uacfc\uc774\uc5b4\uc5d0\uac8c\ub294 \uc554\ubb35\uc801 \uc704\ud611\uc774 \uac00\ud574\uc9c4 \ub05d\uc5d0 \uadf8\ub4e4\uc740 \uad73\uc774 \uacbd\uc120\uc744 \uce58\ub7ec\uc11c \ud45c\ub97c \ucabc\uac1c\uc9c0 \ub9d0\uc790\uace0 \uacb0\ub860\uc744 \ubcf4\uc558\uace0, \ub300\uc2e0 \ub2e8\uc2dd\ud22c\uc7c1 \uc911\uc778 \uc0cc\uc988\uac00 \ubc18 H \ube14\ub85d \ub2e8\uc77c\ud6c4\ubcf4\ub85c \uc625\uc911\uc5d0\uc11c \ucd9c\ub9c8\ud558\uc5ec \uc5bc\uc2a4\ud130 \uc5f0\ud569\uc8fc\uc758\uc790\ub2f9\uc758 \ud6c4\ubcf4 \ud574\ub9ac \uc6e8\uc2a4\ud2b8\uc640 \uacbd\uc7c1\ud558\uac8c \ub418\uc5c8\ub2e4. \uc138\uac04\uc758 \uc774\ubaa9\uc744 \ub044\ub294 \uc120\uac70\uc6b4\ub3d9 \ub05d\uc5d0 4\uc6d4 9\uc77c \uc120\uac70\uac00 \uce58\ub7ec\uc84c\uace0, \uc6e8\uc2a4\ud2b8\uac00 29,046\ud45c, \uc0cc\uc988\uac00 30,492\ud45c\ub97c \ub4dd\ud45c\ud568\uc5d0 \ub530\ub77c \uc0cc\uc988\ub294 \uc601\uad6d \uc11c\ubbfc\uc6d0\uc5d0 \uc785\uc131\ud588\ub2e4.", "answer": "\uc601\uad6d \uc11c\ubbfc\uc6d0", "sentence": "\uc138\uac04\uc758 \uc774\ubaa9\uc744 \ub044\ub294 \uc120\uac70\uc6b4\ub3d9 \ub05d\uc5d0 4\uc6d4 9\uc77c \uc120\uac70\uac00 \uce58\ub7ec\uc84c\uace0, \uc6e8\uc2a4\ud2b8\uac00 29,046\ud45c, \uc0cc\uc988\uac00 30,492\ud45c\ub97c \ub4dd\ud45c\ud568\uc5d0 \ub530\ub77c \uc0cc\uc988\ub294 \uc601\uad6d \uc11c\ubbfc\uc6d0 \uc5d0 \uc785\uc131\ud588\ub2e4.", "paragraph_sentence": "\uacf5\ud654\uc8fc\uc758 \uc6b4\ub3d9 \uce21\uc5d0\uc11c\ub294 \uc81c2\ucc28 \ub2e8\uc2dd\ud22c\uc7c1\uc5d0 \ub300\ud55c \ub300\uc911\uc801 \uc9c0\uc9c0\ub97c \ubaa8\uc73c\uae30 \uc704\ud574 \ud798\uc37c\ub2e4. \uc0cc\uc988\uac00 \ub2e8\uc2dd\ud22c\uc7c1\uc744 \uc2dc\uc791\ud558\uae30 \uc804 \uc8fc \uc77c\uc694\uc77c\uc5d0 3,500\uba85\uc758 \uc0ac\ub78c\ub4e4\uc774 \ubca8\ud30c\uc2a4\ud2b8 \uc11c\ubd80 \uc9c0\uad6c\ub97c \ud589\uc9c4\ud588\ub2e4. 4\uac1c\uc6d4 \uc804\uc778 \uc81c1\ucc28 \ub2e8\uc2dd\ud22c\uc7c1 \ub54c\uc5d0\ub294 10,000\uc5ec \uba85\uc774 \ud589\uc9c4\uc5d0 \ucc38\uc5ec\ud588\uc5c8\ub2e4. \uadf8\ub7f0\ub370 \ub2e8\uc2dd\ud22c\uc7c1 \uc2dc\uc791 5\uc77c \ucc28 \ub54c \ud37c\ub9e4\ub108 \uc0ac\uc6b0\uc2a4\ud2f0\ub860 \uc120\uac70\uad6c\ub97c \uc9c0\uc5ed\uad6c\ub85c \ud558\ub294 \uacf5\ud654\ud30c \uc11c\ubbfc\uc6d0\uc758\uc6d0 \ud504\ub7ad\ud06c \uba38\uacfc\uc774\uc5b4\uac00 \uc0ac\ub9dd\ud588\uace0, \uc774\uc5d0 \ub530\ub77c \ubcf4\uad90\uc120\uac70\uac00 \uc5f4\ub9ac\uac8c \ub418\uc5c8\ub2e4. \ubbfc\uc871\uc8fc\uc758\uc790\ub4e4\uacfc \uacf5\ud654\uc8fc\uc758\uc790\ub4e4 \uc0ac\uc774\uc5d0 \ub204\uad6c\ub97c \ubcf4\uad90\uc120\uac70 \ud6c4\ubcf4\ub85c \ub0b4\ubcf4\ub0b4\uc57c \ud558\ub294\uc9c0\uc5d0 \ub300\ud55c \ub17c\uc7c1\uc774 \uc77c\uc5b4\ub0ac\ub2e4. \uc0ac\ud68c\ubbfc\uc8fc\ub178\ub3d9\ub2f9\uc758 \uc624\uc2a4\ud2f4 \ucee4\ub9ac\uac00 \uad00\uc2ec\uc744 \ud45c\ud588\uace0, \ubca0\ub974\ub098\ub370\ud2b8 \ub9e5\uce7c\ub9ac\uc2a4\ud0a4\uc640 \uc0ac\ub9dd\ud55c \uba38\uacfc\uc774\uc5b4\uc758 \ud615\uc81c \ub178\uc5d8 \uba38\uacfc\uc774\uc5b4\ub3c4 \uc758\uc0ac\ub97c \ubc1d\ud614\ub2e4. \ud611\uc0c1\uc774 \uc774\ub8e8\uc5b4\uc9c0\uace0, \ud55c\ud3b8 \ub178\uc5d8 \uba38\uacfc\uc774\uc5b4\uc5d0\uac8c\ub294 \uc554\ubb35\uc801 \uc704\ud611\uc774 \uac00\ud574\uc9c4 \ub05d\uc5d0 \uadf8\ub4e4\uc740 \uad73\uc774 \uacbd\uc120\uc744 \uce58\ub7ec\uc11c \ud45c\ub97c \ucabc\uac1c\uc9c0 \ub9d0\uc790\uace0 \uacb0\ub860\uc744 \ubcf4\uc558\uace0, \ub300\uc2e0 \ub2e8\uc2dd\ud22c\uc7c1 \uc911\uc778 \uc0cc\uc988\uac00 \ubc18 H \ube14\ub85d \ub2e8\uc77c\ud6c4\ubcf4\ub85c \uc625\uc911\uc5d0\uc11c \ucd9c\ub9c8\ud558\uc5ec \uc5bc\uc2a4\ud130 \uc5f0\ud569\uc8fc\uc758\uc790\ub2f9\uc758 \ud6c4\ubcf4 \ud574\ub9ac \uc6e8\uc2a4\ud2b8\uc640 \uacbd\uc7c1\ud558\uac8c \ub418\uc5c8\ub2e4. \uc138\uac04\uc758 \uc774\ubaa9\uc744 \ub044\ub294 \uc120\uac70\uc6b4\ub3d9 \ub05d\uc5d0 4\uc6d4 9\uc77c \uc120\uac70\uac00 \uce58\ub7ec\uc84c\uace0, \uc6e8\uc2a4\ud2b8\uac00 29,046\ud45c, \uc0cc\uc988\uac00 30,492\ud45c\ub97c \ub4dd\ud45c\ud568\uc5d0 \ub530\ub77c \uc0cc\uc988\ub294 \uc601\uad6d \uc11c\ubbfc\uc6d0 \uc5d0 \uc785\uc131\ud588\ub2e4. ", "paragraph_answer": "\uacf5\ud654\uc8fc\uc758 \uc6b4\ub3d9 \uce21\uc5d0\uc11c\ub294 \uc81c2\ucc28 \ub2e8\uc2dd\ud22c\uc7c1\uc5d0 \ub300\ud55c \ub300\uc911\uc801 \uc9c0\uc9c0\ub97c \ubaa8\uc73c\uae30 \uc704\ud574 \ud798\uc37c\ub2e4. \uc0cc\uc988\uac00 \ub2e8\uc2dd\ud22c\uc7c1\uc744 \uc2dc\uc791\ud558\uae30 \uc804 \uc8fc \uc77c\uc694\uc77c\uc5d0 3,500\uba85\uc758 \uc0ac\ub78c\ub4e4\uc774 \ubca8\ud30c\uc2a4\ud2b8 \uc11c\ubd80 \uc9c0\uad6c\ub97c \ud589\uc9c4\ud588\ub2e4. 4\uac1c\uc6d4 \uc804\uc778 \uc81c1\ucc28 \ub2e8\uc2dd\ud22c\uc7c1 \ub54c\uc5d0\ub294 10,000\uc5ec \uba85\uc774 \ud589\uc9c4\uc5d0 \ucc38\uc5ec\ud588\uc5c8\ub2e4. \uadf8\ub7f0\ub370 \ub2e8\uc2dd\ud22c\uc7c1 \uc2dc\uc791 5\uc77c \ucc28 \ub54c \ud37c\ub9e4\ub108 \uc0ac\uc6b0\uc2a4\ud2f0\ub860 \uc120\uac70\uad6c\ub97c \uc9c0\uc5ed\uad6c\ub85c \ud558\ub294 \uacf5\ud654\ud30c \uc11c\ubbfc\uc6d0\uc758\uc6d0 \ud504\ub7ad\ud06c \uba38\uacfc\uc774\uc5b4\uac00 \uc0ac\ub9dd\ud588\uace0, \uc774\uc5d0 \ub530\ub77c \ubcf4\uad90\uc120\uac70\uac00 \uc5f4\ub9ac\uac8c \ub418\uc5c8\ub2e4. \ubbfc\uc871\uc8fc\uc758\uc790\ub4e4\uacfc \uacf5\ud654\uc8fc\uc758\uc790\ub4e4 \uc0ac\uc774\uc5d0 \ub204\uad6c\ub97c \ubcf4\uad90\uc120\uac70 \ud6c4\ubcf4\ub85c \ub0b4\ubcf4\ub0b4\uc57c \ud558\ub294\uc9c0\uc5d0 \ub300\ud55c \ub17c\uc7c1\uc774 \uc77c\uc5b4\ub0ac\ub2e4. \uc0ac\ud68c\ubbfc\uc8fc\ub178\ub3d9\ub2f9\uc758 \uc624\uc2a4\ud2f4 \ucee4\ub9ac\uac00 \uad00\uc2ec\uc744 \ud45c\ud588\uace0, \ubca0\ub974\ub098\ub370\ud2b8 \ub9e5\uce7c\ub9ac\uc2a4\ud0a4\uc640 \uc0ac\ub9dd\ud55c \uba38\uacfc\uc774\uc5b4\uc758 \ud615\uc81c \ub178\uc5d8 \uba38\uacfc\uc774\uc5b4\ub3c4 \uc758\uc0ac\ub97c \ubc1d\ud614\ub2e4. \ud611\uc0c1\uc774 \uc774\ub8e8\uc5b4\uc9c0\uace0, \ud55c\ud3b8 \ub178\uc5d8 \uba38\uacfc\uc774\uc5b4\uc5d0\uac8c\ub294 \uc554\ubb35\uc801 \uc704\ud611\uc774 \uac00\ud574\uc9c4 \ub05d\uc5d0 \uadf8\ub4e4\uc740 \uad73\uc774 \uacbd\uc120\uc744 \uce58\ub7ec\uc11c \ud45c\ub97c \ucabc\uac1c\uc9c0 \ub9d0\uc790\uace0 \uacb0\ub860\uc744 \ubcf4\uc558\uace0, \ub300\uc2e0 \ub2e8\uc2dd\ud22c\uc7c1 \uc911\uc778 \uc0cc\uc988\uac00 \ubc18 H \ube14\ub85d \ub2e8\uc77c\ud6c4\ubcf4\ub85c \uc625\uc911\uc5d0\uc11c \ucd9c\ub9c8\ud558\uc5ec \uc5bc\uc2a4\ud130 \uc5f0\ud569\uc8fc\uc758\uc790\ub2f9\uc758 \ud6c4\ubcf4 \ud574\ub9ac \uc6e8\uc2a4\ud2b8\uc640 \uacbd\uc7c1\ud558\uac8c \ub418\uc5c8\ub2e4. \uc138\uac04\uc758 \uc774\ubaa9\uc744 \ub044\ub294 \uc120\uac70\uc6b4\ub3d9 \ub05d\uc5d0 4\uc6d4 9\uc77c \uc120\uac70\uac00 \uce58\ub7ec\uc84c\uace0, \uc6e8\uc2a4\ud2b8\uac00 29,046\ud45c, \uc0cc\uc988\uac00 30,492\ud45c\ub97c \ub4dd\ud45c\ud568\uc5d0 \ub530\ub77c \uc0cc\uc988\ub294 \uc601\uad6d \uc11c\ubbfc\uc6d0 \uc5d0 \uc785\uc131\ud588\ub2e4.", "sentence_answer": "\uc138\uac04\uc758 \uc774\ubaa9\uc744 \ub044\ub294 \uc120\uac70\uc6b4\ub3d9 \ub05d\uc5d0 4\uc6d4 9\uc77c \uc120\uac70\uac00 \uce58\ub7ec\uc84c\uace0, \uc6e8\uc2a4\ud2b8\uac00 29,046\ud45c, \uc0cc\uc988\uac00 30,492\ud45c\ub97c \ub4dd\ud45c\ud568\uc5d0 \ub530\ub77c \uc0cc\uc988\ub294 \uc601\uad6d \uc11c\ubbfc\uc6d0 \uc5d0 \uc785\uc131\ud588\ub2e4.", "paragraph_id": "6585474-5-2", "paragraph_question": "question: \uc0cc\uc988\uac00 \uc120\uac70\uc5d0\uc11c 30,492\ud45c\ub97c \ub4dd\ud45c\ud558\uc5ec \uc785\uc131\ud55c \uacf3\uc740?, context: \uacf5\ud654\uc8fc\uc758 \uc6b4\ub3d9 \uce21\uc5d0\uc11c\ub294 \uc81c2\ucc28 \ub2e8\uc2dd\ud22c\uc7c1\uc5d0 \ub300\ud55c \ub300\uc911\uc801 \uc9c0\uc9c0\ub97c \ubaa8\uc73c\uae30 \uc704\ud574 \ud798\uc37c\ub2e4. \uc0cc\uc988\uac00 \ub2e8\uc2dd\ud22c\uc7c1\uc744 \uc2dc\uc791\ud558\uae30 \uc804 \uc8fc \uc77c\uc694\uc77c\uc5d0 3,500\uba85\uc758 \uc0ac\ub78c\ub4e4\uc774 \ubca8\ud30c\uc2a4\ud2b8 \uc11c\ubd80 \uc9c0\uad6c\ub97c \ud589\uc9c4\ud588\ub2e4. 4\uac1c\uc6d4 \uc804\uc778 \uc81c1\ucc28 \ub2e8\uc2dd\ud22c\uc7c1 \ub54c\uc5d0\ub294 10,000\uc5ec \uba85\uc774 \ud589\uc9c4\uc5d0 \ucc38\uc5ec\ud588\uc5c8\ub2e4. \uadf8\ub7f0\ub370 \ub2e8\uc2dd\ud22c\uc7c1 \uc2dc\uc791 5\uc77c \ucc28 \ub54c \ud37c\ub9e4\ub108 \uc0ac\uc6b0\uc2a4\ud2f0\ub860 \uc120\uac70\uad6c\ub97c \uc9c0\uc5ed\uad6c\ub85c \ud558\ub294 \uacf5\ud654\ud30c \uc11c\ubbfc\uc6d0\uc758\uc6d0 \ud504\ub7ad\ud06c \uba38\uacfc\uc774\uc5b4\uac00 \uc0ac\ub9dd\ud588\uace0, \uc774\uc5d0 \ub530\ub77c \ubcf4\uad90\uc120\uac70\uac00 \uc5f4\ub9ac\uac8c \ub418\uc5c8\ub2e4. \ubbfc\uc871\uc8fc\uc758\uc790\ub4e4\uacfc \uacf5\ud654\uc8fc\uc758\uc790\ub4e4 \uc0ac\uc774\uc5d0 \ub204\uad6c\ub97c \ubcf4\uad90\uc120\uac70 \ud6c4\ubcf4\ub85c \ub0b4\ubcf4\ub0b4\uc57c \ud558\ub294\uc9c0\uc5d0 \ub300\ud55c \ub17c\uc7c1\uc774 \uc77c\uc5b4\ub0ac\ub2e4. \uc0ac\ud68c\ubbfc\uc8fc\ub178\ub3d9\ub2f9\uc758 \uc624\uc2a4\ud2f4 \ucee4\ub9ac\uac00 \uad00\uc2ec\uc744 \ud45c\ud588\uace0, \ubca0\ub974\ub098\ub370\ud2b8 \ub9e5\uce7c\ub9ac\uc2a4\ud0a4\uc640 \uc0ac\ub9dd\ud55c \uba38\uacfc\uc774\uc5b4\uc758 \ud615\uc81c \ub178\uc5d8 \uba38\uacfc\uc774\uc5b4\ub3c4 \uc758\uc0ac\ub97c \ubc1d\ud614\ub2e4. \ud611\uc0c1\uc774 \uc774\ub8e8\uc5b4\uc9c0\uace0, \ud55c\ud3b8 \ub178\uc5d8 \uba38\uacfc\uc774\uc5b4\uc5d0\uac8c\ub294 \uc554\ubb35\uc801 \uc704\ud611\uc774 \uac00\ud574\uc9c4 \ub05d\uc5d0 \uadf8\ub4e4\uc740 \uad73\uc774 \uacbd\uc120\uc744 \uce58\ub7ec\uc11c \ud45c\ub97c \ucabc\uac1c\uc9c0 \ub9d0\uc790\uace0 \uacb0\ub860\uc744 \ubcf4\uc558\uace0, \ub300\uc2e0 \ub2e8\uc2dd\ud22c\uc7c1 \uc911\uc778 \uc0cc\uc988\uac00 \ubc18 H \ube14\ub85d \ub2e8\uc77c\ud6c4\ubcf4\ub85c \uc625\uc911\uc5d0\uc11c \ucd9c\ub9c8\ud558\uc5ec \uc5bc\uc2a4\ud130 \uc5f0\ud569\uc8fc\uc758\uc790\ub2f9\uc758 \ud6c4\ubcf4 \ud574\ub9ac \uc6e8\uc2a4\ud2b8\uc640 \uacbd\uc7c1\ud558\uac8c \ub418\uc5c8\ub2e4. \uc138\uac04\uc758 \uc774\ubaa9\uc744 \ub044\ub294 \uc120\uac70\uc6b4\ub3d9 \ub05d\uc5d0 4\uc6d4 9\uc77c \uc120\uac70\uac00 \uce58\ub7ec\uc84c\uace0, \uc6e8\uc2a4\ud2b8\uac00 29,046\ud45c, \uc0cc\uc988\uac00 30,492\ud45c\ub97c \ub4dd\ud45c\ud568\uc5d0 \ub530\ub77c \uc0cc\uc988\ub294 \uc601\uad6d \uc11c\ubbfc\uc6d0\uc5d0 \uc785\uc131\ud588\ub2e4."} {"question": "\uc774\uc774\ub294 \uc11c\uc778\uacfc \ub3d9\uc778 \uc911 \ube44\uad50\uc801 \ub204\uad6c\uc758 \uc785\uc7a5\uc5d0\uc11c \ub2f9\uc7c1\uc744 \uc870\uc808\ud558\ub824 \ud558\uc600\ub294\uac00?", "paragraph": "\uadf8\ub294 \ub2f9\uc7c1\uc758 \uc870\uc808\uacfc \uc815\uc7c1 \uc911\ub2e8\uc744 \ucd09\uad6c\ud558\uc600\uc9c0\ub9cc, \ub3d9\uc778, \uc11c\uc778 \ubd95\ub2f9\uc740 \uae30\uc874\uc758 \uad70\uc790\uc758 \ubd95\ub2f9\uacfc \uc18c\uc778\uc758 \ubd95\ub2f9\uc774 \uc874\uc7ac\ud55c\ub2e4\ub294 \uacac\ud574\ub97c \uace0\uc218\ud588\uace0, \uc11c\uc778\uc744 \uad70\uc790\uc758 \ubd95\ub2f9, \ub3d9\uc778\uc740 \uc18c\uc778\uc758 \ubd95\ub2f9\uc73c\ub85c \uc0dd\uac01\ud558\uc600\ub2e4. \ub530\ub77c\uc11c \uadf8\ub294 \ub2f9\uc7c1\uc744 \uc870\uc808\ud558\ub294\ub370 \ub178\ub825\ud558\uc600\uc9c0\ub9cc, \ube44\uad50\uc801 \uc11c\uc778\uc758 \uc785\uc7a5\uc5d0 \uc11c\uc11c \ub2f9\uc7c1\uc744 \uc870\uc808\ud558\ub824 \ud558\uc600\uace0, \uc774\ub294 \ub3d9\uc778\uc5d0\uac8c \ub298 \ubd88\ud3c9\ubd88\ub9cc\uacfc \uc801\uac1c\uc2ec\uc758 \ub300\uc0c1\uc774 \ub418\uc5c8\ub2e4. \uadf8\uc758 \ub2f9\uc7c1 \uc870\uc808\uc744 \uc911\ub9bd\uc801\uc774\uc9c0 \ubabb\ud558\ub2e4\uace0 \ud310\ub2e8\ud55c \uc815\uc5ec\ub9bd\uc740 \uc11c\uc778\uc5d0\uc11c \ud0c8\ub2f9\ud558\uc5ec \ub3d9\uc778\uc73c\ub85c \uac74\ub108\uac04\ub2e4. \uadf8\ub294 \uc5b4\ub290 \ubd95\ub2f9\uc5d0\ub098 \uad70\uc790, \uc18c\uc778\uc774 \uc788\uc74c\uc744 \uac70\ub4ed \ubc18\ubcf5\ud558\uc600\ub2e4. \ub610\ud55c \uc790\uc2e0\uc744 \ube44\ud310\ud55c \ub3d9\uc778\uacc4 \uc778\uc0ac\uc5d0 \ub300\ud574\uc11c\ub3c4 \uc11d\ubc29 \uc0ac\uba74\uc744 \ud0c4\uc6d0\ud558\uae30\ub3c4 \ud588\ub2e4. \uadf8\ub7ec\ub098 1583\ub144 \uc790\uc2e0\uc744 \uc18c\uc778\uc73c\ub85c \uacf5\uaca9\ud558\ub294 \uc1a1\uc751\uac1c \ub4f1 \uc0bc\uc0ac\uc758 \uc5b8\uad00\uc5d0 \ub300\ud574\uc11c\ub294 \uc0ac\uba74 \uc694\uad6c\ub97c \uac70\ubd80\ud558\uba70, \uc5c4\uc815\ud55c \uc2dc\ube44\ubd84\ubcc4\uc744 \uc694\uad6c\ud558\uae30\ub3c4 \ud588\ub2e4.", "answer": "\uc11c\uc778", "sentence": "\uadf8\ub294 \ub2f9\uc7c1\uc758 \uc870\uc808\uacfc \uc815\uc7c1 \uc911\ub2e8\uc744 \ucd09\uad6c\ud558\uc600\uc9c0\ub9cc, \ub3d9\uc778, \uc11c\uc778 \ubd95\ub2f9\uc740 \uae30\uc874\uc758 \uad70\uc790\uc758 \ubd95\ub2f9\uacfc \uc18c\uc778\uc758 \ubd95\ub2f9\uc774 \uc874\uc7ac\ud55c\ub2e4\ub294 \uacac\ud574\ub97c \uace0\uc218\ud588\uace0, \uc11c\uc778\uc744 \uad70\uc790\uc758 \ubd95\ub2f9, \ub3d9\uc778\uc740 \uc18c\uc778\uc758 \ubd95\ub2f9\uc73c\ub85c \uc0dd\uac01\ud558\uc600\ub2e4.", "paragraph_sentence": " \uadf8\ub294 \ub2f9\uc7c1\uc758 \uc870\uc808\uacfc \uc815\uc7c1 \uc911\ub2e8\uc744 \ucd09\uad6c\ud558\uc600\uc9c0\ub9cc, \ub3d9\uc778, \uc11c\uc778 \ubd95\ub2f9\uc740 \uae30\uc874\uc758 \uad70\uc790\uc758 \ubd95\ub2f9\uacfc \uc18c\uc778\uc758 \ubd95\ub2f9\uc774 \uc874\uc7ac\ud55c\ub2e4\ub294 \uacac\ud574\ub97c \uace0\uc218\ud588\uace0, \uc11c\uc778\uc744 \uad70\uc790\uc758 \ubd95\ub2f9, \ub3d9\uc778\uc740 \uc18c\uc778\uc758 \ubd95\ub2f9\uc73c\ub85c \uc0dd\uac01\ud558\uc600\ub2e4. \ub530\ub77c\uc11c \uadf8\ub294 \ub2f9\uc7c1\uc744 \uc870\uc808\ud558\ub294\ub370 \ub178\ub825\ud558\uc600\uc9c0\ub9cc, \ube44\uad50\uc801 \uc11c\uc778\uc758 \uc785\uc7a5\uc5d0 \uc11c\uc11c \ub2f9\uc7c1\uc744 \uc870\uc808\ud558\ub824 \ud558\uc600\uace0, \uc774\ub294 \ub3d9\uc778\uc5d0\uac8c \ub298 \ubd88\ud3c9\ubd88\ub9cc\uacfc \uc801\uac1c\uc2ec\uc758 \ub300\uc0c1\uc774 \ub418\uc5c8\ub2e4. \uadf8\uc758 \ub2f9\uc7c1 \uc870\uc808\uc744 \uc911\ub9bd\uc801\uc774\uc9c0 \ubabb\ud558\ub2e4\uace0 \ud310\ub2e8\ud55c \uc815\uc5ec\ub9bd\uc740 \uc11c\uc778\uc5d0\uc11c \ud0c8\ub2f9\ud558\uc5ec \ub3d9\uc778\uc73c\ub85c \uac74\ub108\uac04\ub2e4. \uadf8\ub294 \uc5b4\ub290 \ubd95\ub2f9\uc5d0\ub098 \uad70\uc790, \uc18c\uc778\uc774 \uc788\uc74c\uc744 \uac70\ub4ed \ubc18\ubcf5\ud558\uc600\ub2e4. \ub610\ud55c \uc790\uc2e0\uc744 \ube44\ud310\ud55c \ub3d9\uc778\uacc4 \uc778\uc0ac\uc5d0 \ub300\ud574\uc11c\ub3c4 \uc11d\ubc29 \uc0ac\uba74\uc744 \ud0c4\uc6d0\ud558\uae30\ub3c4 \ud588\ub2e4. \uadf8\ub7ec\ub098 1583\ub144 \uc790\uc2e0\uc744 \uc18c\uc778\uc73c\ub85c \uacf5\uaca9\ud558\ub294 \uc1a1\uc751\uac1c \ub4f1 \uc0bc\uc0ac\uc758 \uc5b8\uad00\uc5d0 \ub300\ud574\uc11c\ub294 \uc0ac\uba74 \uc694\uad6c\ub97c \uac70\ubd80\ud558\uba70, \uc5c4\uc815\ud55c \uc2dc\ube44\ubd84\ubcc4\uc744 \uc694\uad6c\ud558\uae30\ub3c4 \ud588\ub2e4.", "paragraph_answer": "\uadf8\ub294 \ub2f9\uc7c1\uc758 \uc870\uc808\uacfc \uc815\uc7c1 \uc911\ub2e8\uc744 \ucd09\uad6c\ud558\uc600\uc9c0\ub9cc, \ub3d9\uc778, \uc11c\uc778 \ubd95\ub2f9\uc740 \uae30\uc874\uc758 \uad70\uc790\uc758 \ubd95\ub2f9\uacfc \uc18c\uc778\uc758 \ubd95\ub2f9\uc774 \uc874\uc7ac\ud55c\ub2e4\ub294 \uacac\ud574\ub97c \uace0\uc218\ud588\uace0, \uc11c\uc778\uc744 \uad70\uc790\uc758 \ubd95\ub2f9, \ub3d9\uc778\uc740 \uc18c\uc778\uc758 \ubd95\ub2f9\uc73c\ub85c \uc0dd\uac01\ud558\uc600\ub2e4. \ub530\ub77c\uc11c \uadf8\ub294 \ub2f9\uc7c1\uc744 \uc870\uc808\ud558\ub294\ub370 \ub178\ub825\ud558\uc600\uc9c0\ub9cc, \ube44\uad50\uc801 \uc11c\uc778\uc758 \uc785\uc7a5\uc5d0 \uc11c\uc11c \ub2f9\uc7c1\uc744 \uc870\uc808\ud558\ub824 \ud558\uc600\uace0, \uc774\ub294 \ub3d9\uc778\uc5d0\uac8c \ub298 \ubd88\ud3c9\ubd88\ub9cc\uacfc \uc801\uac1c\uc2ec\uc758 \ub300\uc0c1\uc774 \ub418\uc5c8\ub2e4. \uadf8\uc758 \ub2f9\uc7c1 \uc870\uc808\uc744 \uc911\ub9bd\uc801\uc774\uc9c0 \ubabb\ud558\ub2e4\uace0 \ud310\ub2e8\ud55c \uc815\uc5ec\ub9bd\uc740 \uc11c\uc778\uc5d0\uc11c \ud0c8\ub2f9\ud558\uc5ec \ub3d9\uc778\uc73c\ub85c \uac74\ub108\uac04\ub2e4. \uadf8\ub294 \uc5b4\ub290 \ubd95\ub2f9\uc5d0\ub098 \uad70\uc790, \uc18c\uc778\uc774 \uc788\uc74c\uc744 \uac70\ub4ed \ubc18\ubcf5\ud558\uc600\ub2e4. \ub610\ud55c \uc790\uc2e0\uc744 \ube44\ud310\ud55c \ub3d9\uc778\uacc4 \uc778\uc0ac\uc5d0 \ub300\ud574\uc11c\ub3c4 \uc11d\ubc29 \uc0ac\uba74\uc744 \ud0c4\uc6d0\ud558\uae30\ub3c4 \ud588\ub2e4. \uadf8\ub7ec\ub098 1583\ub144 \uc790\uc2e0\uc744 \uc18c\uc778\uc73c\ub85c \uacf5\uaca9\ud558\ub294 \uc1a1\uc751\uac1c \ub4f1 \uc0bc\uc0ac\uc758 \uc5b8\uad00\uc5d0 \ub300\ud574\uc11c\ub294 \uc0ac\uba74 \uc694\uad6c\ub97c \uac70\ubd80\ud558\uba70, \uc5c4\uc815\ud55c \uc2dc\ube44\ubd84\ubcc4\uc744 \uc694\uad6c\ud558\uae30\ub3c4 \ud588\ub2e4.", "sentence_answer": "\uadf8\ub294 \ub2f9\uc7c1\uc758 \uc870\uc808\uacfc \uc815\uc7c1 \uc911\ub2e8\uc744 \ucd09\uad6c\ud558\uc600\uc9c0\ub9cc, \ub3d9\uc778, \uc11c\uc778 \ubd95\ub2f9\uc740 \uae30\uc874\uc758 \uad70\uc790\uc758 \ubd95\ub2f9\uacfc \uc18c\uc778\uc758 \ubd95\ub2f9\uc774 \uc874\uc7ac\ud55c\ub2e4\ub294 \uacac\ud574\ub97c \uace0\uc218\ud588\uace0, \uc11c\uc778\uc744 \uad70\uc790\uc758 \ubd95\ub2f9, \ub3d9\uc778\uc740 \uc18c\uc778\uc758 \ubd95\ub2f9\uc73c\ub85c \uc0dd\uac01\ud558\uc600\ub2e4.", "paragraph_id": "6545935-8-0", "paragraph_question": "question: \uc774\uc774\ub294 \uc11c\uc778\uacfc \ub3d9\uc778 \uc911 \ube44\uad50\uc801 \ub204\uad6c\uc758 \uc785\uc7a5\uc5d0\uc11c \ub2f9\uc7c1\uc744 \uc870\uc808\ud558\ub824 \ud558\uc600\ub294\uac00?, context: \uadf8\ub294 \ub2f9\uc7c1\uc758 \uc870\uc808\uacfc \uc815\uc7c1 \uc911\ub2e8\uc744 \ucd09\uad6c\ud558\uc600\uc9c0\ub9cc, \ub3d9\uc778, \uc11c\uc778 \ubd95\ub2f9\uc740 \uae30\uc874\uc758 \uad70\uc790\uc758 \ubd95\ub2f9\uacfc \uc18c\uc778\uc758 \ubd95\ub2f9\uc774 \uc874\uc7ac\ud55c\ub2e4\ub294 \uacac\ud574\ub97c \uace0\uc218\ud588\uace0, \uc11c\uc778\uc744 \uad70\uc790\uc758 \ubd95\ub2f9, \ub3d9\uc778\uc740 \uc18c\uc778\uc758 \ubd95\ub2f9\uc73c\ub85c \uc0dd\uac01\ud558\uc600\ub2e4. \ub530\ub77c\uc11c \uadf8\ub294 \ub2f9\uc7c1\uc744 \uc870\uc808\ud558\ub294\ub370 \ub178\ub825\ud558\uc600\uc9c0\ub9cc, \ube44\uad50\uc801 \uc11c\uc778\uc758 \uc785\uc7a5\uc5d0 \uc11c\uc11c \ub2f9\uc7c1\uc744 \uc870\uc808\ud558\ub824 \ud558\uc600\uace0, \uc774\ub294 \ub3d9\uc778\uc5d0\uac8c \ub298 \ubd88\ud3c9\ubd88\ub9cc\uacfc \uc801\uac1c\uc2ec\uc758 \ub300\uc0c1\uc774 \ub418\uc5c8\ub2e4. \uadf8\uc758 \ub2f9\uc7c1 \uc870\uc808\uc744 \uc911\ub9bd\uc801\uc774\uc9c0 \ubabb\ud558\ub2e4\uace0 \ud310\ub2e8\ud55c \uc815\uc5ec\ub9bd\uc740 \uc11c\uc778\uc5d0\uc11c \ud0c8\ub2f9\ud558\uc5ec \ub3d9\uc778\uc73c\ub85c \uac74\ub108\uac04\ub2e4. \uadf8\ub294 \uc5b4\ub290 \ubd95\ub2f9\uc5d0\ub098 \uad70\uc790, \uc18c\uc778\uc774 \uc788\uc74c\uc744 \uac70\ub4ed \ubc18\ubcf5\ud558\uc600\ub2e4. \ub610\ud55c \uc790\uc2e0\uc744 \ube44\ud310\ud55c \ub3d9\uc778\uacc4 \uc778\uc0ac\uc5d0 \ub300\ud574\uc11c\ub3c4 \uc11d\ubc29 \uc0ac\uba74\uc744 \ud0c4\uc6d0\ud558\uae30\ub3c4 \ud588\ub2e4. \uadf8\ub7ec\ub098 1583\ub144 \uc790\uc2e0\uc744 \uc18c\uc778\uc73c\ub85c \uacf5\uaca9\ud558\ub294 \uc1a1\uc751\uac1c \ub4f1 \uc0bc\uc0ac\uc758 \uc5b8\uad00\uc5d0 \ub300\ud574\uc11c\ub294 \uc0ac\uba74 \uc694\uad6c\ub97c \uac70\ubd80\ud558\uba70, \uc5c4\uc815\ud55c \uc2dc\ube44\ubd84\ubcc4\uc744 \uc694\uad6c\ud558\uae30\ub3c4 \ud588\ub2e4."} {"question": "\uc2dc\ub9c8 \ub2e4\uc774\uc2a4\ucf00 \uc5ed\ud560\uc744 \ub9e1\uc740 \uc0ac\ub78c\uc740?", "paragraph": "\uc2dc\ub9c8 \ub2e4\uc774\uc2a4\ucf00 \uc5ed\uc744 \ub9e1\uc740 \uc624\uac00\ud0c0 \ub098\uc624\ud1a0\ub294, \"\ub0a8\uc2ec\uc744 \uc790\uadf9\ud558\ub294 \uc774\uc0c1\uc801\uc778 \uc791\ud488 '\uc57c\ub9c8\ud1a0'\ub97c \uc2e4\uc0ac\ud654\ud55c\ub2e4\ub294 \uac83\uc73c\ub85c, \uc790\uc2e0\uc774 \ucd9c\uc5f0\ud558\ub4e0 \uadf8\ub807\uc9c0 \uc54a\ub4e0, \uadf8 \uc601\ud654\ub97c \ubcf4\uace0 \uc2f6\uc5c8\ub2e4\"\ub77c\uace0 \ub9d0\ud588\ub2e4. \uc6d0\uc791\uc5d0\uc11c\ub294 \ub0a8\uc131\uc774\uc5c8\ub358 \uc544\uc774\ud558\ub77c\uac00 \uc5ec\uc790 \uc5ed\ud560\ub85c \ubc14\ub00c\uc5b4 \ucfe8\ud558\uace0 \uac15\ud55c \uc5ec\uc131 \uc5ed\uc744 \ub9e1\uc740 \ub9c8\uc774\ucf54\ub294, \"\ud604\uc7a5\uc5d0\uc11c \ub290\ub080 \uac83\uc744 \uc5f0\uae30\ud558\ub824\uace0 \ud588\ub2e4\"\uace0 \ub9d0\ud588\ub2e4. \uac00\ud1a0 \uc0ac\ubd80\ub85c \uc5ed\uc744 \ub9e1\uc740 \ub098\ubbf8\uc624\uce74 \uce74\uc988\ud0a4\ub294, \"\ud3c9\uc0c1\uc2dc\uc758 \ub098\uc758 \uce90\ub9ad\ud130\uc640\ub3c4 \uac00\uae4c\uc6e0\ub2e4.\"\ub77c\uace0 \ubc30\uc5ed\uc744 \ubb18\uc0ac\ud588\ub2e4. \uc57c\ub9c8\ubaa8\ud1a0 \uc544\ud0a4\ub77c \uc5ed\uc744 \uc5f0\uae30\ud55c \uc0ac\uc774\ud1a0 \ub2e4\ucfe0\ubbf8\ub294, \"\ub098 \uc790\uc2e0\ub3c4 \ub9c8\uce58 \ud560\ub9ac\uc6b0\ub4dc \uc601\ud654\ub97c \ubcf4\uace0 \uc788\ub294 \uac83 \uac19\uc544\uc11c, \uc77c\ubcf8\uc758 \uc601\ud654\ub3c4 \uc9c4\ud654\ud588\ub2e4\uace0 \ub290\uaf08\ub2e4. \uc6b0\uc8fc\uc758 \uc601\uc0c1\uc740 \ubb3c\ub860, \uc9c0\ud558\uc5d0 \uc778\ub958\uac00 \uc0b4\uace0 \uc788\ub294 \ubb18\uc0ac\uac00 \ub300\ub2e8\ud558\ub2e4\uace0 \uc0dd\uac01\ud55c\ub2e4.\"\ub77c\uace0 \ub9d0\ud588\ub2e4. \uc6d0\uc791\uc5d0\uc11c\ub294 \ub0a8\uc131\uc774\uc5c8\ub358 \uc0ac\ub3c4 \uc120\uc0dd\uc774 \uc5ec\uc131\uc73c\ub85c \ubc14\ub00c\uc5c8\ub2e4.", "answer": "\uc624\uac00\ud0c0 \ub098\uc624\ud1a0", "sentence": "\uc2dc\ub9c8 \ub2e4\uc774\uc2a4\ucf00 \uc5ed\uc744 \ub9e1\uc740 \uc624\uac00\ud0c0 \ub098\uc624\ud1a0 \ub294, \"\ub0a8\uc2ec\uc744 \uc790\uadf9\ud558\ub294 \uc774\uc0c1\uc801\uc778 \uc791\ud488 '\uc57c\ub9c8\ud1a0'\ub97c \uc2e4\uc0ac\ud654\ud55c\ub2e4\ub294 \uac83\uc73c\ub85c, \uc790\uc2e0\uc774 \ucd9c\uc5f0\ud558\ub4e0 \uadf8\ub807\uc9c0 \uc54a\ub4e0, \uadf8 \uc601\ud654\ub97c \ubcf4\uace0 \uc2f6\uc5c8\ub2e4\"\ub77c\uace0 \ub9d0\ud588\ub2e4.", "paragraph_sentence": " \uc2dc\ub9c8 \ub2e4\uc774\uc2a4\ucf00 \uc5ed\uc744 \ub9e1\uc740 \uc624\uac00\ud0c0 \ub098\uc624\ud1a0 \ub294, \"\ub0a8\uc2ec\uc744 \uc790\uadf9\ud558\ub294 \uc774\uc0c1\uc801\uc778 \uc791\ud488 '\uc57c\ub9c8\ud1a0'\ub97c \uc2e4\uc0ac\ud654\ud55c\ub2e4\ub294 \uac83\uc73c\ub85c, \uc790\uc2e0\uc774 \ucd9c\uc5f0\ud558\ub4e0 \uadf8\ub807\uc9c0 \uc54a\ub4e0, \uadf8 \uc601\ud654\ub97c \ubcf4\uace0 \uc2f6\uc5c8\ub2e4\"\ub77c\uace0 \ub9d0\ud588\ub2e4. \uc6d0\uc791\uc5d0\uc11c\ub294 \ub0a8\uc131\uc774\uc5c8\ub358 \uc544\uc774\ud558\ub77c\uac00 \uc5ec\uc790 \uc5ed\ud560\ub85c \ubc14\ub00c\uc5b4 \ucfe8\ud558\uace0 \uac15\ud55c \uc5ec\uc131 \uc5ed\uc744 \ub9e1\uc740 \ub9c8\uc774\ucf54\ub294, \"\ud604\uc7a5\uc5d0\uc11c \ub290\ub080 \uac83\uc744 \uc5f0\uae30\ud558\ub824\uace0 \ud588\ub2e4\"\uace0 \ub9d0\ud588\ub2e4. \uac00\ud1a0 \uc0ac\ubd80\ub85c \uc5ed\uc744 \ub9e1\uc740 \ub098\ubbf8\uc624\uce74 \uce74\uc988\ud0a4\ub294, \"\ud3c9\uc0c1\uc2dc\uc758 \ub098\uc758 \uce90\ub9ad\ud130\uc640\ub3c4 \uac00\uae4c\uc6e0\ub2e4. \"\ub77c\uace0 \ubc30\uc5ed\uc744 \ubb18\uc0ac\ud588\ub2e4. \uc57c\ub9c8\ubaa8\ud1a0 \uc544\ud0a4\ub77c \uc5ed\uc744 \uc5f0\uae30\ud55c \uc0ac\uc774\ud1a0 \ub2e4\ucfe0\ubbf8\ub294, \"\ub098 \uc790\uc2e0\ub3c4 \ub9c8\uce58 \ud560\ub9ac\uc6b0\ub4dc \uc601\ud654\ub97c \ubcf4\uace0 \uc788\ub294 \uac83 \uac19\uc544\uc11c, \uc77c\ubcf8\uc758 \uc601\ud654\ub3c4 \uc9c4\ud654\ud588\ub2e4\uace0 \ub290\uaf08\ub2e4. \uc6b0\uc8fc\uc758 \uc601\uc0c1\uc740 \ubb3c\ub860, \uc9c0\ud558\uc5d0 \uc778\ub958\uac00 \uc0b4\uace0 \uc788\ub294 \ubb18\uc0ac\uac00 \ub300\ub2e8\ud558\ub2e4\uace0 \uc0dd\uac01\ud55c\ub2e4. \"\ub77c\uace0 \ub9d0\ud588\ub2e4. \uc6d0\uc791\uc5d0\uc11c\ub294 \ub0a8\uc131\uc774\uc5c8\ub358 \uc0ac\ub3c4 \uc120\uc0dd\uc774 \uc5ec\uc131\uc73c\ub85c \ubc14\ub00c\uc5c8\ub2e4.", "paragraph_answer": "\uc2dc\ub9c8 \ub2e4\uc774\uc2a4\ucf00 \uc5ed\uc744 \ub9e1\uc740 \uc624\uac00\ud0c0 \ub098\uc624\ud1a0 \ub294, \"\ub0a8\uc2ec\uc744 \uc790\uadf9\ud558\ub294 \uc774\uc0c1\uc801\uc778 \uc791\ud488 '\uc57c\ub9c8\ud1a0'\ub97c \uc2e4\uc0ac\ud654\ud55c\ub2e4\ub294 \uac83\uc73c\ub85c, \uc790\uc2e0\uc774 \ucd9c\uc5f0\ud558\ub4e0 \uadf8\ub807\uc9c0 \uc54a\ub4e0, \uadf8 \uc601\ud654\ub97c \ubcf4\uace0 \uc2f6\uc5c8\ub2e4\"\ub77c\uace0 \ub9d0\ud588\ub2e4. \uc6d0\uc791\uc5d0\uc11c\ub294 \ub0a8\uc131\uc774\uc5c8\ub358 \uc544\uc774\ud558\ub77c\uac00 \uc5ec\uc790 \uc5ed\ud560\ub85c \ubc14\ub00c\uc5b4 \ucfe8\ud558\uace0 \uac15\ud55c \uc5ec\uc131 \uc5ed\uc744 \ub9e1\uc740 \ub9c8\uc774\ucf54\ub294, \"\ud604\uc7a5\uc5d0\uc11c \ub290\ub080 \uac83\uc744 \uc5f0\uae30\ud558\ub824\uace0 \ud588\ub2e4\"\uace0 \ub9d0\ud588\ub2e4. \uac00\ud1a0 \uc0ac\ubd80\ub85c \uc5ed\uc744 \ub9e1\uc740 \ub098\ubbf8\uc624\uce74 \uce74\uc988\ud0a4\ub294, \"\ud3c9\uc0c1\uc2dc\uc758 \ub098\uc758 \uce90\ub9ad\ud130\uc640\ub3c4 \uac00\uae4c\uc6e0\ub2e4.\"\ub77c\uace0 \ubc30\uc5ed\uc744 \ubb18\uc0ac\ud588\ub2e4. \uc57c\ub9c8\ubaa8\ud1a0 \uc544\ud0a4\ub77c \uc5ed\uc744 \uc5f0\uae30\ud55c \uc0ac\uc774\ud1a0 \ub2e4\ucfe0\ubbf8\ub294, \"\ub098 \uc790\uc2e0\ub3c4 \ub9c8\uce58 \ud560\ub9ac\uc6b0\ub4dc \uc601\ud654\ub97c \ubcf4\uace0 \uc788\ub294 \uac83 \uac19\uc544\uc11c, \uc77c\ubcf8\uc758 \uc601\ud654\ub3c4 \uc9c4\ud654\ud588\ub2e4\uace0 \ub290\uaf08\ub2e4. \uc6b0\uc8fc\uc758 \uc601\uc0c1\uc740 \ubb3c\ub860, \uc9c0\ud558\uc5d0 \uc778\ub958\uac00 \uc0b4\uace0 \uc788\ub294 \ubb18\uc0ac\uac00 \ub300\ub2e8\ud558\ub2e4\uace0 \uc0dd\uac01\ud55c\ub2e4.\"\ub77c\uace0 \ub9d0\ud588\ub2e4. \uc6d0\uc791\uc5d0\uc11c\ub294 \ub0a8\uc131\uc774\uc5c8\ub358 \uc0ac\ub3c4 \uc120\uc0dd\uc774 \uc5ec\uc131\uc73c\ub85c \ubc14\ub00c\uc5c8\ub2e4.", "sentence_answer": "\uc2dc\ub9c8 \ub2e4\uc774\uc2a4\ucf00 \uc5ed\uc744 \ub9e1\uc740 \uc624\uac00\ud0c0 \ub098\uc624\ud1a0 \ub294, \"\ub0a8\uc2ec\uc744 \uc790\uadf9\ud558\ub294 \uc774\uc0c1\uc801\uc778 \uc791\ud488 '\uc57c\ub9c8\ud1a0'\ub97c \uc2e4\uc0ac\ud654\ud55c\ub2e4\ub294 \uac83\uc73c\ub85c, \uc790\uc2e0\uc774 \ucd9c\uc5f0\ud558\ub4e0 \uadf8\ub807\uc9c0 \uc54a\ub4e0, \uadf8 \uc601\ud654\ub97c \ubcf4\uace0 \uc2f6\uc5c8\ub2e4\"\ub77c\uace0 \ub9d0\ud588\ub2e4.", "paragraph_id": "6526332-8-0", "paragraph_question": "question: \uc2dc\ub9c8 \ub2e4\uc774\uc2a4\ucf00 \uc5ed\ud560\uc744 \ub9e1\uc740 \uc0ac\ub78c\uc740?, context: \uc2dc\ub9c8 \ub2e4\uc774\uc2a4\ucf00 \uc5ed\uc744 \ub9e1\uc740 \uc624\uac00\ud0c0 \ub098\uc624\ud1a0\ub294, \"\ub0a8\uc2ec\uc744 \uc790\uadf9\ud558\ub294 \uc774\uc0c1\uc801\uc778 \uc791\ud488 '\uc57c\ub9c8\ud1a0'\ub97c \uc2e4\uc0ac\ud654\ud55c\ub2e4\ub294 \uac83\uc73c\ub85c, \uc790\uc2e0\uc774 \ucd9c\uc5f0\ud558\ub4e0 \uadf8\ub807\uc9c0 \uc54a\ub4e0, \uadf8 \uc601\ud654\ub97c \ubcf4\uace0 \uc2f6\uc5c8\ub2e4\"\ub77c\uace0 \ub9d0\ud588\ub2e4. \uc6d0\uc791\uc5d0\uc11c\ub294 \ub0a8\uc131\uc774\uc5c8\ub358 \uc544\uc774\ud558\ub77c\uac00 \uc5ec\uc790 \uc5ed\ud560\ub85c \ubc14\ub00c\uc5b4 \ucfe8\ud558\uace0 \uac15\ud55c \uc5ec\uc131 \uc5ed\uc744 \ub9e1\uc740 \ub9c8\uc774\ucf54\ub294, \"\ud604\uc7a5\uc5d0\uc11c \ub290\ub080 \uac83\uc744 \uc5f0\uae30\ud558\ub824\uace0 \ud588\ub2e4\"\uace0 \ub9d0\ud588\ub2e4. \uac00\ud1a0 \uc0ac\ubd80\ub85c \uc5ed\uc744 \ub9e1\uc740 \ub098\ubbf8\uc624\uce74 \uce74\uc988\ud0a4\ub294, \"\ud3c9\uc0c1\uc2dc\uc758 \ub098\uc758 \uce90\ub9ad\ud130\uc640\ub3c4 \uac00\uae4c\uc6e0\ub2e4.\"\ub77c\uace0 \ubc30\uc5ed\uc744 \ubb18\uc0ac\ud588\ub2e4. \uc57c\ub9c8\ubaa8\ud1a0 \uc544\ud0a4\ub77c \uc5ed\uc744 \uc5f0\uae30\ud55c \uc0ac\uc774\ud1a0 \ub2e4\ucfe0\ubbf8\ub294, \"\ub098 \uc790\uc2e0\ub3c4 \ub9c8\uce58 \ud560\ub9ac\uc6b0\ub4dc \uc601\ud654\ub97c \ubcf4\uace0 \uc788\ub294 \uac83 \uac19\uc544\uc11c, \uc77c\ubcf8\uc758 \uc601\ud654\ub3c4 \uc9c4\ud654\ud588\ub2e4\uace0 \ub290\uaf08\ub2e4. \uc6b0\uc8fc\uc758 \uc601\uc0c1\uc740 \ubb3c\ub860, \uc9c0\ud558\uc5d0 \uc778\ub958\uac00 \uc0b4\uace0 \uc788\ub294 \ubb18\uc0ac\uac00 \ub300\ub2e8\ud558\ub2e4\uace0 \uc0dd\uac01\ud55c\ub2e4.\"\ub77c\uace0 \ub9d0\ud588\ub2e4. \uc6d0\uc791\uc5d0\uc11c\ub294 \ub0a8\uc131\uc774\uc5c8\ub358 \uc0ac\ub3c4 \uc120\uc0dd\uc774 \uc5ec\uc131\uc73c\ub85c \ubc14\ub00c\uc5c8\ub2e4."} {"question": "\ubc15\uadfc\ud61c\uac00 \uad6d\ud68c\uc758\uc6d0\uc5d0 \ub2f9\uc120\ub418\uc5c8\uc744 \ub54c \uce5c\ubc15\uc5f0\ub300\ub294 \uc9c0\uc5ed\uad6c\uc5d0\uc11c \uba87 \uc11d\uc744 \ud68d\ub4dd\ud558\uc600\ub294\uac00?", "paragraph": "\ubc15\uadfc\ud61c\ub294 \ud55c\ub098\ub77c\ub2f9 \ud6c4\ubcf4\ub85c \uad6d\ud68c\uc758\uc6d0\uc5d0 \ub2e4\uc2dc \ub2f9\uc120\ub418\uc5c8\uace0, \uc774\uc7ac\uc624, \uc774\ubc29\ud638\ub294 \ub099\uc120\ud558\uc600\uc73c\uba70, \uce5c\ubc15\uc5f0\ub300\ub294 \uc9c0\uc5ed\uad6c 6\uc11d, \uc804\uad6d\uad6c 8\uc11d\uc774\ub77c\ub294 \uc608\uc0c1 \uc678\uc758 \ub192\uc740 \uc9c0\uc9c0\uc728\uc744 \ub2ec\uc131\ud558\uc600\ub2e4. \uce5c\ubc15 \ubb34\uc18c\uc18d \uc5f0\ub300\ub3c4 \ucd1d 12\uba85\uc774 \ub2f9\uc120\ub418\uc5c8\ub2e4. \uc774\ud6c4\uc5d0\ub3c4 \ubc15\uadfc\ud61c\ub294 \uafb8\uc900\ud788 \uce5c\ubc15\uc5f0\ub300 \ubc0f \ubb34\uc18c\uc18d \uce5c\ubc15 \ub2f9\uc120\uc790\ub4e4\uc758 \ubcf5\ub2f9\uc744 \uc694\uad6c\ud558\uc600\ub2e4. \uce5c\ubc15 \uce21\uacfc \uc559\uae08\uc774 \ub0a8\uc544\uc788\ub358 \uac15\uc7ac\uc12d \ub300\ud45c\uc758 \uc784\uae30\uac00 \uc885\ub8cc\ub418\uace0 \ubc15\ud76c\ud0dc \uc804 \uc758\uc6d0\uc774 \ub300\ud45c\ub85c \ubd80\uc784\ud55c \ub4a4 \ub9c8\uce68\ub0b4 \uce5c\ubc15\ubb34\uc18c\uc18d\uc5f0\ub300 \uc18c\uc18d \uc758\uc6d0\uacfc \uce5c\ubc15\uc5f0\ub300\uc758 \uc9c0\uc5ed\uad6c \uc758\uc6d0 \uc804\uc6d0\uc774 \ud55c\ub098\ub77c\ub2f9\uc73c\ub85c \ubcf5\ub2f9\ud558\uc5ec \ud55c\ub098\ub77c\ub2f9 \ub0b4 \uce5c\ubc15\uc758\uc6d0 \uc758\uc11d\uc218\ub294 60\uc5ec\uba85\uc5d0 \uc774\ub974\uac8c \ub418\uc5c8\ub2e4. \uce5c\ubc15\uc5f0\ub300\uc758 \ube44\ub840\ub300\ud45c \uc758\uc6d0 5\uba85\uc740 \ub2f9\uba85 \uac1c\uc815, \ub3c5\uc790 \ud65c\ub3d9\uc744 \uafb8\uc900\ud788 \ud558\uc600\uace0 \uc591\uc815\ub840, \uc11c\uccad\uc6d0, \uae40\ub178\uc2dd\uc740 \ub2f9\uc120\ubb34\ud6a8\ud615\uc774 \ub300\ubc95\uc6d0\uc5d0\uc11c \ud655\uc815\ub418\uc5b4 \uc758\uc6d0\uc9c1\uc744 \uc0c1\uc2e4\ud558\uc600\ub2e4.", "answer": "6\uc11d", "sentence": "\uce5c\ubc15\uc5f0\ub300\ub294 \uc9c0\uc5ed\uad6c 6\uc11d ,", "paragraph_sentence": "\ubc15\uadfc\ud61c\ub294 \ud55c\ub098\ub77c\ub2f9 \ud6c4\ubcf4\ub85c \uad6d\ud68c\uc758\uc6d0\uc5d0 \ub2e4\uc2dc \ub2f9\uc120\ub418\uc5c8\uace0, \uc774\uc7ac\uc624, \uc774\ubc29\ud638\ub294 \ub099\uc120\ud558\uc600\uc73c\uba70, \uce5c\ubc15\uc5f0\ub300\ub294 \uc9c0\uc5ed\uad6c 6\uc11d , \uc804\uad6d\uad6c 8\uc11d\uc774\ub77c\ub294 \uc608\uc0c1 \uc678\uc758 \ub192\uc740 \uc9c0\uc9c0\uc728\uc744 \ub2ec\uc131\ud558\uc600\ub2e4. \uce5c\ubc15 \ubb34\uc18c\uc18d \uc5f0\ub300\ub3c4 \ucd1d 12\uba85\uc774 \ub2f9\uc120\ub418\uc5c8\ub2e4. \uc774\ud6c4\uc5d0\ub3c4 \ubc15\uadfc\ud61c\ub294 \uafb8\uc900\ud788 \uce5c\ubc15\uc5f0\ub300 \ubc0f \ubb34\uc18c\uc18d \uce5c\ubc15 \ub2f9\uc120\uc790\ub4e4\uc758 \ubcf5\ub2f9\uc744 \uc694\uad6c\ud558\uc600\ub2e4. \uce5c\ubc15 \uce21\uacfc \uc559\uae08\uc774 \ub0a8\uc544\uc788\ub358 \uac15\uc7ac\uc12d \ub300\ud45c\uc758 \uc784\uae30\uac00 \uc885\ub8cc\ub418\uace0 \ubc15\ud76c\ud0dc \uc804 \uc758\uc6d0\uc774 \ub300\ud45c\ub85c \ubd80\uc784\ud55c \ub4a4 \ub9c8\uce68\ub0b4 \uce5c\ubc15\ubb34\uc18c\uc18d\uc5f0\ub300 \uc18c\uc18d \uc758\uc6d0\uacfc \uce5c\ubc15\uc5f0\ub300\uc758 \uc9c0\uc5ed\uad6c \uc758\uc6d0 \uc804\uc6d0\uc774 \ud55c\ub098\ub77c\ub2f9\uc73c\ub85c \ubcf5\ub2f9\ud558\uc5ec \ud55c\ub098\ub77c\ub2f9 \ub0b4 \uce5c\ubc15\uc758\uc6d0 \uc758\uc11d\uc218\ub294 60\uc5ec\uba85\uc5d0 \uc774\ub974\uac8c \ub418\uc5c8\ub2e4. \uce5c\ubc15\uc5f0\ub300\uc758 \ube44\ub840\ub300\ud45c \uc758\uc6d0 5\uba85\uc740 \ub2f9\uba85 \uac1c\uc815, \ub3c5\uc790 \ud65c\ub3d9\uc744 \uafb8\uc900\ud788 \ud558\uc600\uace0 \uc591\uc815\ub840, \uc11c\uccad\uc6d0, \uae40\ub178\uc2dd\uc740 \ub2f9\uc120\ubb34\ud6a8\ud615\uc774 \ub300\ubc95\uc6d0\uc5d0\uc11c \ud655\uc815\ub418\uc5b4 \uc758\uc6d0\uc9c1\uc744 \uc0c1\uc2e4\ud558\uc600\ub2e4.", "paragraph_answer": "\ubc15\uadfc\ud61c\ub294 \ud55c\ub098\ub77c\ub2f9 \ud6c4\ubcf4\ub85c \uad6d\ud68c\uc758\uc6d0\uc5d0 \ub2e4\uc2dc \ub2f9\uc120\ub418\uc5c8\uace0, \uc774\uc7ac\uc624, \uc774\ubc29\ud638\ub294 \ub099\uc120\ud558\uc600\uc73c\uba70, \uce5c\ubc15\uc5f0\ub300\ub294 \uc9c0\uc5ed\uad6c 6\uc11d , \uc804\uad6d\uad6c 8\uc11d\uc774\ub77c\ub294 \uc608\uc0c1 \uc678\uc758 \ub192\uc740 \uc9c0\uc9c0\uc728\uc744 \ub2ec\uc131\ud558\uc600\ub2e4. \uce5c\ubc15 \ubb34\uc18c\uc18d \uc5f0\ub300\ub3c4 \ucd1d 12\uba85\uc774 \ub2f9\uc120\ub418\uc5c8\ub2e4. \uc774\ud6c4\uc5d0\ub3c4 \ubc15\uadfc\ud61c\ub294 \uafb8\uc900\ud788 \uce5c\ubc15\uc5f0\ub300 \ubc0f \ubb34\uc18c\uc18d \uce5c\ubc15 \ub2f9\uc120\uc790\ub4e4\uc758 \ubcf5\ub2f9\uc744 \uc694\uad6c\ud558\uc600\ub2e4. \uce5c\ubc15 \uce21\uacfc \uc559\uae08\uc774 \ub0a8\uc544\uc788\ub358 \uac15\uc7ac\uc12d \ub300\ud45c\uc758 \uc784\uae30\uac00 \uc885\ub8cc\ub418\uace0 \ubc15\ud76c\ud0dc \uc804 \uc758\uc6d0\uc774 \ub300\ud45c\ub85c \ubd80\uc784\ud55c \ub4a4 \ub9c8\uce68\ub0b4 \uce5c\ubc15\ubb34\uc18c\uc18d\uc5f0\ub300 \uc18c\uc18d \uc758\uc6d0\uacfc \uce5c\ubc15\uc5f0\ub300\uc758 \uc9c0\uc5ed\uad6c \uc758\uc6d0 \uc804\uc6d0\uc774 \ud55c\ub098\ub77c\ub2f9\uc73c\ub85c \ubcf5\ub2f9\ud558\uc5ec \ud55c\ub098\ub77c\ub2f9 \ub0b4 \uce5c\ubc15\uc758\uc6d0 \uc758\uc11d\uc218\ub294 60\uc5ec\uba85\uc5d0 \uc774\ub974\uac8c \ub418\uc5c8\ub2e4. \uce5c\ubc15\uc5f0\ub300\uc758 \ube44\ub840\ub300\ud45c \uc758\uc6d0 5\uba85\uc740 \ub2f9\uba85 \uac1c\uc815, \ub3c5\uc790 \ud65c\ub3d9\uc744 \uafb8\uc900\ud788 \ud558\uc600\uace0 \uc591\uc815\ub840, \uc11c\uccad\uc6d0, \uae40\ub178\uc2dd\uc740 \ub2f9\uc120\ubb34\ud6a8\ud615\uc774 \ub300\ubc95\uc6d0\uc5d0\uc11c \ud655\uc815\ub418\uc5b4 \uc758\uc6d0\uc9c1\uc744 \uc0c1\uc2e4\ud558\uc600\ub2e4.", "sentence_answer": "\uce5c\ubc15\uc5f0\ub300\ub294 \uc9c0\uc5ed\uad6c 6\uc11d ,", "paragraph_id": "6506540-9-0", "paragraph_question": "question: \ubc15\uadfc\ud61c\uac00 \uad6d\ud68c\uc758\uc6d0\uc5d0 \ub2f9\uc120\ub418\uc5c8\uc744 \ub54c \uce5c\ubc15\uc5f0\ub300\ub294 \uc9c0\uc5ed\uad6c\uc5d0\uc11c \uba87 \uc11d\uc744 \ud68d\ub4dd\ud558\uc600\ub294\uac00?, context: \ubc15\uadfc\ud61c\ub294 \ud55c\ub098\ub77c\ub2f9 \ud6c4\ubcf4\ub85c \uad6d\ud68c\uc758\uc6d0\uc5d0 \ub2e4\uc2dc \ub2f9\uc120\ub418\uc5c8\uace0, \uc774\uc7ac\uc624, \uc774\ubc29\ud638\ub294 \ub099\uc120\ud558\uc600\uc73c\uba70, \uce5c\ubc15\uc5f0\ub300\ub294 \uc9c0\uc5ed\uad6c 6\uc11d, \uc804\uad6d\uad6c 8\uc11d\uc774\ub77c\ub294 \uc608\uc0c1 \uc678\uc758 \ub192\uc740 \uc9c0\uc9c0\uc728\uc744 \ub2ec\uc131\ud558\uc600\ub2e4. \uce5c\ubc15 \ubb34\uc18c\uc18d \uc5f0\ub300\ub3c4 \ucd1d 12\uba85\uc774 \ub2f9\uc120\ub418\uc5c8\ub2e4. \uc774\ud6c4\uc5d0\ub3c4 \ubc15\uadfc\ud61c\ub294 \uafb8\uc900\ud788 \uce5c\ubc15\uc5f0\ub300 \ubc0f \ubb34\uc18c\uc18d \uce5c\ubc15 \ub2f9\uc120\uc790\ub4e4\uc758 \ubcf5\ub2f9\uc744 \uc694\uad6c\ud558\uc600\ub2e4. \uce5c\ubc15 \uce21\uacfc \uc559\uae08\uc774 \ub0a8\uc544\uc788\ub358 \uac15\uc7ac\uc12d \ub300\ud45c\uc758 \uc784\uae30\uac00 \uc885\ub8cc\ub418\uace0 \ubc15\ud76c\ud0dc \uc804 \uc758\uc6d0\uc774 \ub300\ud45c\ub85c \ubd80\uc784\ud55c \ub4a4 \ub9c8\uce68\ub0b4 \uce5c\ubc15\ubb34\uc18c\uc18d\uc5f0\ub300 \uc18c\uc18d \uc758\uc6d0\uacfc \uce5c\ubc15\uc5f0\ub300\uc758 \uc9c0\uc5ed\uad6c \uc758\uc6d0 \uc804\uc6d0\uc774 \ud55c\ub098\ub77c\ub2f9\uc73c\ub85c \ubcf5\ub2f9\ud558\uc5ec \ud55c\ub098\ub77c\ub2f9 \ub0b4 \uce5c\ubc15\uc758\uc6d0 \uc758\uc11d\uc218\ub294 60\uc5ec\uba85\uc5d0 \uc774\ub974\uac8c \ub418\uc5c8\ub2e4. \uce5c\ubc15\uc5f0\ub300\uc758 \ube44\ub840\ub300\ud45c \uc758\uc6d0 5\uba85\uc740 \ub2f9\uba85 \uac1c\uc815, \ub3c5\uc790 \ud65c\ub3d9\uc744 \uafb8\uc900\ud788 \ud558\uc600\uace0 \uc591\uc815\ub840, \uc11c\uccad\uc6d0, \uae40\ub178\uc2dd\uc740 \ub2f9\uc120\ubb34\ud6a8\ud615\uc774 \ub300\ubc95\uc6d0\uc5d0\uc11c \ud655\uc815\ub418\uc5b4 \uc758\uc6d0\uc9c1\uc744 \uc0c1\uc2e4\ud558\uc600\ub2e4."} {"question": "\uc81c\uc784\uc2a4 \ucfe1\uc774 \ub0a8\ubc18\uad6c \ud56d\ud574\uc5d0\uc11c \ub3cc\uc544\uc628 \ub54c\ub294?", "paragraph": "\uc774 \ubc16\uc5d0\ub3c4 \ud504\ud1a8\ub808\ub9c8\uc774\uc624\uc2a4\uc758 \u300a\uc9c0\ub9ac\ud559 geography\u300b\uc740 \ub2e4\ub8e8\ub294 \ub098\ub77c\uc758 \uae30\ud6c4\ub098 \ucc9c\uc5f0\uc0dd\uc0b0\ubb3c, \uc11c\uc2dd\ub3d9\ubb3c\uacfc \uace0\uc720\uc758 \ud2b9\uc9d5 \uac19\uc740 \uac83\uc5d0 \ub300\ud55c \uc5b8\uae09\uc774\ub098 \uc9c0\ub9ac\ud559\uc801\uc73c\ub85c \uc911\uc694\ud55c \uc694\uc18c\uc778 \uac15\uc774\ub098 \uc0b0\ub9e5\uc5d0 \ub300\ud55c \ucde8\uae09\uc740 \uac70\uc758 \uc5c6\uc5c8\ub2e4. \uc774\uc640 \uac19\uc740 \uacb0\uc810\uc5d0\ub3c4 \ubd88\uad6c\ud558\uace0 \uc774 \ucc45\uc740 \uc5ed\uc0ac\uc801\uc73c\ub85c \u300a\uc54c\ub9c8\uac8c\uc2a4\ud2b8\u300b\ucc98\ub7fc \ud6c4\uc138\uc5d0 \ub9e4\uc6b0 \ud070 \uc601\ud5a5\uc744 \ubbf8\ucce4\uae30 \ub54c\ubb38\uc5d0 \uc911\uc694\ud55c \ucc45\uc774\ub2e4. \ud504\ud1a8\ub808\ub9c8\uc774\uc624\uc2a4\ub294 \uc544\uc2dc\uc544\uac00 \uc2e4\uc81c\ubcf4\ub2e4 \ub3d9\ucabd\uc73c\ub85c \ud6e8\uc52c \ub354 \ud655\uc7a5\ub418\uc5b4 \uc788\ub2e4\uace0 \ud588\uae30 \ub54c\ubb38\uc5d0 \ud06c\ub9ac\uc2a4\ud1a0\ud37c \ucf5c\ub7fc\ubc84\uc2a4\ub294 \uc11c\ucabd\uc73c\ub85c \uacc4\uc18d \ud56d\ud574\ud558\uba74 \uc544\uc2dc\uc544\uc5d0 \ub3c4\ucc29\ud560 \uc218 \uc788\uc744 \uac83\uc774\ub77c\uace0 \ubbff\uc5c8\ub2e4. \ub610\ud55c 1775\ub144\uae4c\uc9c0\ub3c4 \ud504\ud1a8\ub808\ub9c8\uc774\uc624\uc2a4\uac00 \ub9d0\ud588\ub358 \uac83\ucc98\ub7fc \uc778\ub3c4\uc591\uc774 \ub0a8\ucabd\uc758 \ub300\ub959\uc5d0 \uc758\ud574 \ub9c9\ud600 \uc788\uc744 \uac83\uc774\ub77c\uace0 \ubbff\uc5c8\ub2e4. \uadf8\ub7ec\ub098 \uc774\uac83\uc740 \uac19\uc740 \ud574 7\uc6d4 \uc81c\uc784\uc2a4 \ucfe1\uc774 \ub0a8\ubc18\uad6c \ud56d\ud574\uc5d0\uc11c \ub3cc\uc544\uc634\uc73c\ub85c\uc368 \ud2c0\ub838\ub2e4\ub294 \uac83\uc774 \uc99d\uba85\ub418\uc5c8\ub2e4.", "answer": "7\uc6d4", "sentence": "\uadf8\ub7ec\ub098 \uc774\uac83\uc740 \uac19\uc740 \ud574 7\uc6d4 \uc81c\uc784\uc2a4 \ucfe1\uc774 \ub0a8\ubc18\uad6c \ud56d\ud574\uc5d0\uc11c \ub3cc\uc544\uc634\uc73c\ub85c\uc368 \ud2c0\ub838\ub2e4\ub294 \uac83\uc774 \uc99d\uba85\ub418\uc5c8\ub2e4.", "paragraph_sentence": "\uc774 \ubc16\uc5d0\ub3c4 \ud504\ud1a8\ub808\ub9c8\uc774\uc624\uc2a4\uc758 \u300a\uc9c0\ub9ac\ud559 geography\u300b\uc740 \ub2e4\ub8e8\ub294 \ub098\ub77c\uc758 \uae30\ud6c4\ub098 \ucc9c\uc5f0\uc0dd\uc0b0\ubb3c, \uc11c\uc2dd\ub3d9\ubb3c\uacfc \uace0\uc720\uc758 \ud2b9\uc9d5 \uac19\uc740 \uac83\uc5d0 \ub300\ud55c \uc5b8\uae09\uc774\ub098 \uc9c0\ub9ac\ud559\uc801\uc73c\ub85c \uc911\uc694\ud55c \uc694\uc18c\uc778 \uac15\uc774\ub098 \uc0b0\ub9e5\uc5d0 \ub300\ud55c \ucde8\uae09\uc740 \uac70\uc758 \uc5c6\uc5c8\ub2e4. \uc774\uc640 \uac19\uc740 \uacb0\uc810\uc5d0\ub3c4 \ubd88\uad6c\ud558\uace0 \uc774 \ucc45\uc740 \uc5ed\uc0ac\uc801\uc73c\ub85c \u300a\uc54c\ub9c8\uac8c\uc2a4\ud2b8\u300b\ucc98\ub7fc \ud6c4\uc138\uc5d0 \ub9e4\uc6b0 \ud070 \uc601\ud5a5\uc744 \ubbf8\ucce4\uae30 \ub54c\ubb38\uc5d0 \uc911\uc694\ud55c \ucc45\uc774\ub2e4. \ud504\ud1a8\ub808\ub9c8\uc774\uc624\uc2a4\ub294 \uc544\uc2dc\uc544\uac00 \uc2e4\uc81c\ubcf4\ub2e4 \ub3d9\ucabd\uc73c\ub85c \ud6e8\uc52c \ub354 \ud655\uc7a5\ub418\uc5b4 \uc788\ub2e4\uace0 \ud588\uae30 \ub54c\ubb38\uc5d0 \ud06c\ub9ac\uc2a4\ud1a0\ud37c \ucf5c\ub7fc\ubc84\uc2a4\ub294 \uc11c\ucabd\uc73c\ub85c \uacc4\uc18d \ud56d\ud574\ud558\uba74 \uc544\uc2dc\uc544\uc5d0 \ub3c4\ucc29\ud560 \uc218 \uc788\uc744 \uac83\uc774\ub77c\uace0 \ubbff\uc5c8\ub2e4. \ub610\ud55c 1775\ub144\uae4c\uc9c0\ub3c4 \ud504\ud1a8\ub808\ub9c8\uc774\uc624\uc2a4\uac00 \ub9d0\ud588\ub358 \uac83\ucc98\ub7fc \uc778\ub3c4\uc591\uc774 \ub0a8\ucabd\uc758 \ub300\ub959\uc5d0 \uc758\ud574 \ub9c9\ud600 \uc788\uc744 \uac83\uc774\ub77c\uace0 \ubbff\uc5c8\ub2e4. \uadf8\ub7ec\ub098 \uc774\uac83\uc740 \uac19\uc740 \ud574 7\uc6d4 \uc81c\uc784\uc2a4 \ucfe1\uc774 \ub0a8\ubc18\uad6c \ud56d\ud574\uc5d0\uc11c \ub3cc\uc544\uc634\uc73c\ub85c\uc368 \ud2c0\ub838\ub2e4\ub294 \uac83\uc774 \uc99d\uba85\ub418\uc5c8\ub2e4. ", "paragraph_answer": "\uc774 \ubc16\uc5d0\ub3c4 \ud504\ud1a8\ub808\ub9c8\uc774\uc624\uc2a4\uc758 \u300a\uc9c0\ub9ac\ud559 geography\u300b\uc740 \ub2e4\ub8e8\ub294 \ub098\ub77c\uc758 \uae30\ud6c4\ub098 \ucc9c\uc5f0\uc0dd\uc0b0\ubb3c, \uc11c\uc2dd\ub3d9\ubb3c\uacfc \uace0\uc720\uc758 \ud2b9\uc9d5 \uac19\uc740 \uac83\uc5d0 \ub300\ud55c \uc5b8\uae09\uc774\ub098 \uc9c0\ub9ac\ud559\uc801\uc73c\ub85c \uc911\uc694\ud55c \uc694\uc18c\uc778 \uac15\uc774\ub098 \uc0b0\ub9e5\uc5d0 \ub300\ud55c \ucde8\uae09\uc740 \uac70\uc758 \uc5c6\uc5c8\ub2e4. \uc774\uc640 \uac19\uc740 \uacb0\uc810\uc5d0\ub3c4 \ubd88\uad6c\ud558\uace0 \uc774 \ucc45\uc740 \uc5ed\uc0ac\uc801\uc73c\ub85c \u300a\uc54c\ub9c8\uac8c\uc2a4\ud2b8\u300b\ucc98\ub7fc \ud6c4\uc138\uc5d0 \ub9e4\uc6b0 \ud070 \uc601\ud5a5\uc744 \ubbf8\ucce4\uae30 \ub54c\ubb38\uc5d0 \uc911\uc694\ud55c \ucc45\uc774\ub2e4. \ud504\ud1a8\ub808\ub9c8\uc774\uc624\uc2a4\ub294 \uc544\uc2dc\uc544\uac00 \uc2e4\uc81c\ubcf4\ub2e4 \ub3d9\ucabd\uc73c\ub85c \ud6e8\uc52c \ub354 \ud655\uc7a5\ub418\uc5b4 \uc788\ub2e4\uace0 \ud588\uae30 \ub54c\ubb38\uc5d0 \ud06c\ub9ac\uc2a4\ud1a0\ud37c \ucf5c\ub7fc\ubc84\uc2a4\ub294 \uc11c\ucabd\uc73c\ub85c \uacc4\uc18d \ud56d\ud574\ud558\uba74 \uc544\uc2dc\uc544\uc5d0 \ub3c4\ucc29\ud560 \uc218 \uc788\uc744 \uac83\uc774\ub77c\uace0 \ubbff\uc5c8\ub2e4. \ub610\ud55c 1775\ub144\uae4c\uc9c0\ub3c4 \ud504\ud1a8\ub808\ub9c8\uc774\uc624\uc2a4\uac00 \ub9d0\ud588\ub358 \uac83\ucc98\ub7fc \uc778\ub3c4\uc591\uc774 \ub0a8\ucabd\uc758 \ub300\ub959\uc5d0 \uc758\ud574 \ub9c9\ud600 \uc788\uc744 \uac83\uc774\ub77c\uace0 \ubbff\uc5c8\ub2e4. \uadf8\ub7ec\ub098 \uc774\uac83\uc740 \uac19\uc740 \ud574 7\uc6d4 \uc81c\uc784\uc2a4 \ucfe1\uc774 \ub0a8\ubc18\uad6c \ud56d\ud574\uc5d0\uc11c \ub3cc\uc544\uc634\uc73c\ub85c\uc368 \ud2c0\ub838\ub2e4\ub294 \uac83\uc774 \uc99d\uba85\ub418\uc5c8\ub2e4.", "sentence_answer": "\uadf8\ub7ec\ub098 \uc774\uac83\uc740 \uac19\uc740 \ud574 7\uc6d4 \uc81c\uc784\uc2a4 \ucfe1\uc774 \ub0a8\ubc18\uad6c \ud56d\ud574\uc5d0\uc11c \ub3cc\uc544\uc634\uc73c\ub85c\uc368 \ud2c0\ub838\ub2e4\ub294 \uac83\uc774 \uc99d\uba85\ub418\uc5c8\ub2e4.", "paragraph_id": "6213662-5-2", "paragraph_question": "question: \uc81c\uc784\uc2a4 \ucfe1\uc774 \ub0a8\ubc18\uad6c \ud56d\ud574\uc5d0\uc11c \ub3cc\uc544\uc628 \ub54c\ub294?, context: \uc774 \ubc16\uc5d0\ub3c4 \ud504\ud1a8\ub808\ub9c8\uc774\uc624\uc2a4\uc758 \u300a\uc9c0\ub9ac\ud559 geography\u300b\uc740 \ub2e4\ub8e8\ub294 \ub098\ub77c\uc758 \uae30\ud6c4\ub098 \ucc9c\uc5f0\uc0dd\uc0b0\ubb3c, \uc11c\uc2dd\ub3d9\ubb3c\uacfc \uace0\uc720\uc758 \ud2b9\uc9d5 \uac19\uc740 \uac83\uc5d0 \ub300\ud55c \uc5b8\uae09\uc774\ub098 \uc9c0\ub9ac\ud559\uc801\uc73c\ub85c \uc911\uc694\ud55c \uc694\uc18c\uc778 \uac15\uc774\ub098 \uc0b0\ub9e5\uc5d0 \ub300\ud55c \ucde8\uae09\uc740 \uac70\uc758 \uc5c6\uc5c8\ub2e4. \uc774\uc640 \uac19\uc740 \uacb0\uc810\uc5d0\ub3c4 \ubd88\uad6c\ud558\uace0 \uc774 \ucc45\uc740 \uc5ed\uc0ac\uc801\uc73c\ub85c \u300a\uc54c\ub9c8\uac8c\uc2a4\ud2b8\u300b\ucc98\ub7fc \ud6c4\uc138\uc5d0 \ub9e4\uc6b0 \ud070 \uc601\ud5a5\uc744 \ubbf8\ucce4\uae30 \ub54c\ubb38\uc5d0 \uc911\uc694\ud55c \ucc45\uc774\ub2e4. \ud504\ud1a8\ub808\ub9c8\uc774\uc624\uc2a4\ub294 \uc544\uc2dc\uc544\uac00 \uc2e4\uc81c\ubcf4\ub2e4 \ub3d9\ucabd\uc73c\ub85c \ud6e8\uc52c \ub354 \ud655\uc7a5\ub418\uc5b4 \uc788\ub2e4\uace0 \ud588\uae30 \ub54c\ubb38\uc5d0 \ud06c\ub9ac\uc2a4\ud1a0\ud37c \ucf5c\ub7fc\ubc84\uc2a4\ub294 \uc11c\ucabd\uc73c\ub85c \uacc4\uc18d \ud56d\ud574\ud558\uba74 \uc544\uc2dc\uc544\uc5d0 \ub3c4\ucc29\ud560 \uc218 \uc788\uc744 \uac83\uc774\ub77c\uace0 \ubbff\uc5c8\ub2e4. \ub610\ud55c 1775\ub144\uae4c\uc9c0\ub3c4 \ud504\ud1a8\ub808\ub9c8\uc774\uc624\uc2a4\uac00 \ub9d0\ud588\ub358 \uac83\ucc98\ub7fc \uc778\ub3c4\uc591\uc774 \ub0a8\ucabd\uc758 \ub300\ub959\uc5d0 \uc758\ud574 \ub9c9\ud600 \uc788\uc744 \uac83\uc774\ub77c\uace0 \ubbff\uc5c8\ub2e4. \uadf8\ub7ec\ub098 \uc774\uac83\uc740 \uac19\uc740 \ud574 7\uc6d4 \uc81c\uc784\uc2a4 \ucfe1\uc774 \ub0a8\ubc18\uad6c \ud56d\ud574\uc5d0\uc11c \ub3cc\uc544\uc634\uc73c\ub85c\uc368 \ud2c0\ub838\ub2e4\ub294 \uac83\uc774 \uc99d\uba85\ub418\uc5c8\ub2e4."} {"question": "\uac15\uc724\uad6c\uc640 \uc724\uc11d\ubbfc\uc758 \ud2b8\ub808\uc774\ub4dc\ub294 \uc5b4\ub5a4 \ud2b8\ub808\uc774\ub4dc\uc600\ub294\uac00?", "paragraph": "\uac15\uc724\uad6c\uc640 \uc724\uc11d\ubbfc\uc758 \ud2b8\ub808\uc774\ub4dc\uac00 \ud604\uae08 \ud2b8\ub808\uc774\ub4dc\ub85c \ubc1d\ud600\uc9d0\uc5d0 \ub530\ub77c, \uacfc\uac70 \ub125\uc13c\uc774 \uc9c4\ud589\ud55c \uc5ec\ub7ec \ud2b8\ub808\uc774\ub4dc\ub3c4 \ud604\uae08\uc774 \ud3ec\ud568\ub418\uc5c8\ub2e4\ub294 \uc758\uc2ec\uc744 \ubc1b\uac8c \ub418\uc5c8\ub2e4. \uc774\uc5d0 \uad00\ub828\ud574\uc11c SK\ub294 \uae40\uc131\ubbfc \u2013 \uae40\ud0dd\ud615 \ud2b8\ub808\uc774\ub4dc\ub294 \ub2e8\uc21c \uc804\ub825 \ubcf4\uac15 \ucc28\uc6d0\uc758 \ud2b8\ub808\uc774\ub4dc\ub77c\uace0 \ud56d\ubcc0\ud588\uc73c\uba70, KIA \uc5ed\uc2dc \uae40\uc138\ud604, \uc720\uc7ac\uc2e0 \u2013 \uc774\uc2b9\ud638, \uc190\ub3d9\uc6b1 \ud2b8\ub808\uc774\ub4dc\uc5d0 \ud604\uae08\uc774 \ud3ec\ud568\ub418\uc9c0 \uc54a\uc558\ub2e4\uace0 \ubc1d\ud614\ub2e4. \uc774\uc5b4\uc11c \uc1a1\uc2e0\uc601 \ud2b8\ub808\uc774\ub4dc\uc5d0\uc11c\ub3c4 \ud604\uae08 15\uc5b5 \uc6d0\uc774 \ud3ec\ud568\ub418\uc5c8\ub2e4\ub294 \ubcf4\ub3c4\uac00 \ub098\uc654\uc73c\uba70, 2010\ub144 7\uc6d4\uc5d0 \uc9c4\ud589\ub41c \ud669\uc7ac\uade0 \ud2b8\ub808\uc774\ub4dc \ub2f9\uc2dc\uc5d0 \ud604\uae08 20\uc5b5 \uc6d0\uc774 \uac74\ub0b4\uc84c\ub2e4\ub294 \uc99d\uc5b8\uc774 \uc804 \ub86f\ub370 \uc790\uc774\uc5b8\uce20 \uad00\uacc4\uc790\uc5d0\uac8c\uc11c \ub098\uc654\ub2e4. \uc774\uc5b4 5\uc6d4 30\uc77c\uc5d0\ub294 SBS\uac00 \uc9c0\uae08\uae4c\uc9c0 \uc5b8\uae09\ub41c \ud2b8\ub808\uc774\ub4dc \uc774\uc678\uc5d0\ub3c4 \ud604\uae08\uc774 \ud3ec\ud568\ub41c \ud2b8\ub808\uc774\ub4dc\uac00 \ub354 \ub9ce\ub2e4\ub294 \ub0b4\uc6a9\uc744 \ub2e8\ub3c5 \ubcf4\ub3c4\ud558\uc600\ub2e4.", "answer": "\ud604\uae08 \ud2b8\ub808\uc774\ub4dc", "sentence": "\uac15\uc724\uad6c\uc640 \uc724\uc11d\ubbfc\uc758 \ud2b8\ub808\uc774\ub4dc\uac00 \ud604\uae08 \ud2b8\ub808\uc774\ub4dc \ub85c \ubc1d\ud600\uc9d0\uc5d0 \ub530\ub77c, \uacfc\uac70 \ub125\uc13c\uc774 \uc9c4\ud589\ud55c \uc5ec\ub7ec \ud2b8\ub808\uc774\ub4dc\ub3c4 \ud604\uae08\uc774 \ud3ec\ud568\ub418\uc5c8\ub2e4\ub294 \uc758\uc2ec\uc744 \ubc1b\uac8c \ub418\uc5c8\ub2e4.", "paragraph_sentence": " \uac15\uc724\uad6c\uc640 \uc724\uc11d\ubbfc\uc758 \ud2b8\ub808\uc774\ub4dc\uac00 \ud604\uae08 \ud2b8\ub808\uc774\ub4dc \ub85c \ubc1d\ud600\uc9d0\uc5d0 \ub530\ub77c, \uacfc\uac70 \ub125\uc13c\uc774 \uc9c4\ud589\ud55c \uc5ec\ub7ec \ud2b8\ub808\uc774\ub4dc\ub3c4 \ud604\uae08\uc774 \ud3ec\ud568\ub418\uc5c8\ub2e4\ub294 \uc758\uc2ec\uc744 \ubc1b\uac8c \ub418\uc5c8\ub2e4. \uc774\uc5d0 \uad00\ub828\ud574\uc11c SK\ub294 \uae40\uc131\ubbfc \u2013 \uae40\ud0dd\ud615 \ud2b8\ub808\uc774\ub4dc\ub294 \ub2e8\uc21c \uc804\ub825 \ubcf4\uac15 \ucc28\uc6d0\uc758 \ud2b8\ub808\uc774\ub4dc\ub77c\uace0 \ud56d\ubcc0\ud588\uc73c\uba70, KIA \uc5ed\uc2dc \uae40\uc138\ud604, \uc720\uc7ac\uc2e0 \u2013 \uc774\uc2b9\ud638, \uc190\ub3d9\uc6b1 \ud2b8\ub808\uc774\ub4dc\uc5d0 \ud604\uae08\uc774 \ud3ec\ud568\ub418\uc9c0 \uc54a\uc558\ub2e4\uace0 \ubc1d\ud614\ub2e4. \uc774\uc5b4\uc11c \uc1a1\uc2e0\uc601 \ud2b8\ub808\uc774\ub4dc\uc5d0\uc11c\ub3c4 \ud604\uae08 15\uc5b5 \uc6d0\uc774 \ud3ec\ud568\ub418\uc5c8\ub2e4\ub294 \ubcf4\ub3c4\uac00 \ub098\uc654\uc73c\uba70, 2010\ub144 7\uc6d4\uc5d0 \uc9c4\ud589\ub41c \ud669\uc7ac\uade0 \ud2b8\ub808\uc774\ub4dc \ub2f9\uc2dc\uc5d0 \ud604\uae08 20\uc5b5 \uc6d0\uc774 \uac74\ub0b4\uc84c\ub2e4\ub294 \uc99d\uc5b8\uc774 \uc804 \ub86f\ub370 \uc790\uc774\uc5b8\uce20 \uad00\uacc4\uc790\uc5d0\uac8c\uc11c \ub098\uc654\ub2e4. \uc774\uc5b4 5\uc6d4 30\uc77c\uc5d0\ub294 SBS\uac00 \uc9c0\uae08\uae4c\uc9c0 \uc5b8\uae09\ub41c \ud2b8\ub808\uc774\ub4dc \uc774\uc678\uc5d0\ub3c4 \ud604\uae08\uc774 \ud3ec\ud568\ub41c \ud2b8\ub808\uc774\ub4dc\uac00 \ub354 \ub9ce\ub2e4\ub294 \ub0b4\uc6a9\uc744 \ub2e8\ub3c5 \ubcf4\ub3c4\ud558\uc600\ub2e4.", "paragraph_answer": "\uac15\uc724\uad6c\uc640 \uc724\uc11d\ubbfc\uc758 \ud2b8\ub808\uc774\ub4dc\uac00 \ud604\uae08 \ud2b8\ub808\uc774\ub4dc \ub85c \ubc1d\ud600\uc9d0\uc5d0 \ub530\ub77c, \uacfc\uac70 \ub125\uc13c\uc774 \uc9c4\ud589\ud55c \uc5ec\ub7ec \ud2b8\ub808\uc774\ub4dc\ub3c4 \ud604\uae08\uc774 \ud3ec\ud568\ub418\uc5c8\ub2e4\ub294 \uc758\uc2ec\uc744 \ubc1b\uac8c \ub418\uc5c8\ub2e4. \uc774\uc5d0 \uad00\ub828\ud574\uc11c SK\ub294 \uae40\uc131\ubbfc \u2013 \uae40\ud0dd\ud615 \ud2b8\ub808\uc774\ub4dc\ub294 \ub2e8\uc21c \uc804\ub825 \ubcf4\uac15 \ucc28\uc6d0\uc758 \ud2b8\ub808\uc774\ub4dc\ub77c\uace0 \ud56d\ubcc0\ud588\uc73c\uba70, KIA \uc5ed\uc2dc \uae40\uc138\ud604, \uc720\uc7ac\uc2e0 \u2013 \uc774\uc2b9\ud638, \uc190\ub3d9\uc6b1 \ud2b8\ub808\uc774\ub4dc\uc5d0 \ud604\uae08\uc774 \ud3ec\ud568\ub418\uc9c0 \uc54a\uc558\ub2e4\uace0 \ubc1d\ud614\ub2e4. \uc774\uc5b4\uc11c \uc1a1\uc2e0\uc601 \ud2b8\ub808\uc774\ub4dc\uc5d0\uc11c\ub3c4 \ud604\uae08 15\uc5b5 \uc6d0\uc774 \ud3ec\ud568\ub418\uc5c8\ub2e4\ub294 \ubcf4\ub3c4\uac00 \ub098\uc654\uc73c\uba70, 2010\ub144 7\uc6d4\uc5d0 \uc9c4\ud589\ub41c \ud669\uc7ac\uade0 \ud2b8\ub808\uc774\ub4dc \ub2f9\uc2dc\uc5d0 \ud604\uae08 20\uc5b5 \uc6d0\uc774 \uac74\ub0b4\uc84c\ub2e4\ub294 \uc99d\uc5b8\uc774 \uc804 \ub86f\ub370 \uc790\uc774\uc5b8\uce20 \uad00\uacc4\uc790\uc5d0\uac8c\uc11c \ub098\uc654\ub2e4. \uc774\uc5b4 5\uc6d4 30\uc77c\uc5d0\ub294 SBS\uac00 \uc9c0\uae08\uae4c\uc9c0 \uc5b8\uae09\ub41c \ud2b8\ub808\uc774\ub4dc \uc774\uc678\uc5d0\ub3c4 \ud604\uae08\uc774 \ud3ec\ud568\ub41c \ud2b8\ub808\uc774\ub4dc\uac00 \ub354 \ub9ce\ub2e4\ub294 \ub0b4\uc6a9\uc744 \ub2e8\ub3c5 \ubcf4\ub3c4\ud558\uc600\ub2e4.", "sentence_answer": "\uac15\uc724\uad6c\uc640 \uc724\uc11d\ubbfc\uc758 \ud2b8\ub808\uc774\ub4dc\uac00 \ud604\uae08 \ud2b8\ub808\uc774\ub4dc \ub85c \ubc1d\ud600\uc9d0\uc5d0 \ub530\ub77c, \uacfc\uac70 \ub125\uc13c\uc774 \uc9c4\ud589\ud55c \uc5ec\ub7ec \ud2b8\ub808\uc774\ub4dc\ub3c4 \ud604\uae08\uc774 \ud3ec\ud568\ub418\uc5c8\ub2e4\ub294 \uc758\uc2ec\uc744 \ubc1b\uac8c \ub418\uc5c8\ub2e4.", "paragraph_id": "6533929-22-0", "paragraph_question": "question: \uac15\uc724\uad6c\uc640 \uc724\uc11d\ubbfc\uc758 \ud2b8\ub808\uc774\ub4dc\ub294 \uc5b4\ub5a4 \ud2b8\ub808\uc774\ub4dc\uc600\ub294\uac00?, context: \uac15\uc724\uad6c\uc640 \uc724\uc11d\ubbfc\uc758 \ud2b8\ub808\uc774\ub4dc\uac00 \ud604\uae08 \ud2b8\ub808\uc774\ub4dc\ub85c \ubc1d\ud600\uc9d0\uc5d0 \ub530\ub77c, \uacfc\uac70 \ub125\uc13c\uc774 \uc9c4\ud589\ud55c \uc5ec\ub7ec \ud2b8\ub808\uc774\ub4dc\ub3c4 \ud604\uae08\uc774 \ud3ec\ud568\ub418\uc5c8\ub2e4\ub294 \uc758\uc2ec\uc744 \ubc1b\uac8c \ub418\uc5c8\ub2e4. \uc774\uc5d0 \uad00\ub828\ud574\uc11c SK\ub294 \uae40\uc131\ubbfc \u2013 \uae40\ud0dd\ud615 \ud2b8\ub808\uc774\ub4dc\ub294 \ub2e8\uc21c \uc804\ub825 \ubcf4\uac15 \ucc28\uc6d0\uc758 \ud2b8\ub808\uc774\ub4dc\ub77c\uace0 \ud56d\ubcc0\ud588\uc73c\uba70, KIA \uc5ed\uc2dc \uae40\uc138\ud604, \uc720\uc7ac\uc2e0 \u2013 \uc774\uc2b9\ud638, \uc190\ub3d9\uc6b1 \ud2b8\ub808\uc774\ub4dc\uc5d0 \ud604\uae08\uc774 \ud3ec\ud568\ub418\uc9c0 \uc54a\uc558\ub2e4\uace0 \ubc1d\ud614\ub2e4. \uc774\uc5b4\uc11c \uc1a1\uc2e0\uc601 \ud2b8\ub808\uc774\ub4dc\uc5d0\uc11c\ub3c4 \ud604\uae08 15\uc5b5 \uc6d0\uc774 \ud3ec\ud568\ub418\uc5c8\ub2e4\ub294 \ubcf4\ub3c4\uac00 \ub098\uc654\uc73c\uba70, 2010\ub144 7\uc6d4\uc5d0 \uc9c4\ud589\ub41c \ud669\uc7ac\uade0 \ud2b8\ub808\uc774\ub4dc \ub2f9\uc2dc\uc5d0 \ud604\uae08 20\uc5b5 \uc6d0\uc774 \uac74\ub0b4\uc84c\ub2e4\ub294 \uc99d\uc5b8\uc774 \uc804 \ub86f\ub370 \uc790\uc774\uc5b8\uce20 \uad00\uacc4\uc790\uc5d0\uac8c\uc11c \ub098\uc654\ub2e4. \uc774\uc5b4 5\uc6d4 30\uc77c\uc5d0\ub294 SBS\uac00 \uc9c0\uae08\uae4c\uc9c0 \uc5b8\uae09\ub41c \ud2b8\ub808\uc774\ub4dc \uc774\uc678\uc5d0\ub3c4 \ud604\uae08\uc774 \ud3ec\ud568\ub41c \ud2b8\ub808\uc774\ub4dc\uac00 \ub354 \ub9ce\ub2e4\ub294 \ub0b4\uc6a9\uc744 \ub2e8\ub3c5 \ubcf4\ub3c4\ud558\uc600\ub2e4."} {"question": "\uc6b0\uc5f0\ud788 \ubc29\ubb38\ud55c \ube59\uc758 \uc624\ub514\uc158\uc5d0\uc11c \ud1a0\ucfe0\ub098\uac00\uac00 \ub9cc\ub4e0 \ub370\ubaa8 \ud14c\uc774\ud504\ub97c \ub4e3\uace0 \ud06c\uac8c \ud638\ud3c9\ud55c \uc0ac\ub78c\uc740 \ub204\uad6c\uc778\uac00?", "paragraph": "A\uc0ac\uc774\ub4dc \uace1\uc740 \ucd5c\uc18c\ud55c 1996\ub144\ubd80\ud130 \uc774\ubbf8 \uc874\uc7ac\ud588\ub358 \ub178\ub798\ub2e4. \uc791\uace1\uc790 \ud1a0\ucfe0\ub098\uac00\ub294 \ub370\ubaa8 \ud14c\uc774\ud504\ub97c \ub9cc\ub4e4\uc5b4 \uac01 \uc74c\ubc18\uc81c\uc791\uc0ac\ub85c \ubcf4\ub0b4\uc9c0\ub9cc \uac70\uc808\ub2f9\ud558\uace0, \uc791\uace1\uc5d0 \uc788\uc5b4 \uc815\uccb4\ub97c \uacaa\uc5b4 \uc74c\uc545\uac00\ub85c\uc11c\uc758 \uc778\uc0dd\uc744 \ub2e8\ub150\ud558\ub824\uace0 \ud55c\ub2e4. \uc774 \ub54c, \ud3c9\uc18c \uc88b\uc544\ud558\ub358 \uc774\uae00\uc2a4\uc758 \u3008Hotel California\u3009\uc758 \uc804\uc8fc \ubd80\ubd84 \uae30\ud0c0\uc18c\ub9ac\ub97c \ub4e3\uace0\uc120 \uc790\uc2e0\uc774 \uc88b\uc544\ud558\ub294 \ub178\ub798\ub97c, \ub9c8\uce58 \ucde8\ubbf8\ub85c \ub9cc\ub4e0 \uac83 \uac19\uc740 \ub178\ub798\ub97c \ub9cc\ub4e4\uc5b4\ub0c8\ub2e4. \uc0ac\uce74\uc774 \uc774\uc988\ubbf8\ub294 \uc6b0\uc5f0\ud788 \ubc29\ubb38\ud55c \ube59\uc758 \uc624\ub514\uc158\uc5d0\uc11c \ud1a0\ucfe0\ub098\uac00\uac00 \uc791\uace1\ud55c \ub370\ubaa8 \ud14c\uc774\ud504\ub97c \ub4e3\uace0\uc11c \ud06c\uac8c \ud638\ud3c9\ud55c \ub4a4 \uadf8\ub97c \ucd94\ucc9c\ud55c\ub2e4. 1996\ub144\uacbd, \ube44\ub85d \uc568\ubc94\uc5d0\ub294 \uc218\ub85d\ub418\uc5b4 \uc788\uc9c0 \uc54a\uc558\uace0 \uace1\uba85\uc740 \u3008Fallin' of the Rain\u3009\uc774\ub780 \uc774\ub984\uc774\uc5c8\uc9c0\ub9cc, \ud6c4\ub834\uad6c\ub9cc\uc740 \uc774\ubbf8 \uc874\uc7ac\ud574\uc11c \uce90\ub17c\uc758 \uce74\uba54\ub77c NEW EOS KISS\uc758 \uad11\uace0 \uc0bd\uc785\uace1\uc73c\ub85c \uc0ac\uc6a9\ub418\uace0 \uc788\uc5c8\ub2e4.", "answer": "\uc0ac\uce74\uc774 \uc774\uc988\ubbf8", "sentence": "\uc0ac\uce74\uc774 \uc774\uc988\ubbf8 \ub294 \uc6b0\uc5f0\ud788 \ubc29\ubb38\ud55c \ube59\uc758 \uc624\ub514\uc158\uc5d0\uc11c \ud1a0\ucfe0\ub098\uac00\uac00 \uc791\uace1\ud55c \ub370\ubaa8 \ud14c\uc774\ud504\ub97c \ub4e3\uace0\uc11c \ud06c\uac8c \ud638\ud3c9\ud55c \ub4a4 \uadf8\ub97c \ucd94\ucc9c\ud55c\ub2e4.", "paragraph_sentence": "A\uc0ac\uc774\ub4dc \uace1\uc740 \ucd5c\uc18c\ud55c 1996\ub144\ubd80\ud130 \uc774\ubbf8 \uc874\uc7ac\ud588\ub358 \ub178\ub798\ub2e4. \uc791\uace1\uc790 \ud1a0\ucfe0\ub098\uac00\ub294 \ub370\ubaa8 \ud14c\uc774\ud504\ub97c \ub9cc\ub4e4\uc5b4 \uac01 \uc74c\ubc18\uc81c\uc791\uc0ac\ub85c \ubcf4\ub0b4\uc9c0\ub9cc \uac70\uc808\ub2f9\ud558\uace0, \uc791\uace1\uc5d0 \uc788\uc5b4 \uc815\uccb4\ub97c \uacaa\uc5b4 \uc74c\uc545\uac00\ub85c\uc11c\uc758 \uc778\uc0dd\uc744 \ub2e8\ub150\ud558\ub824\uace0 \ud55c\ub2e4. \uc774 \ub54c, \ud3c9\uc18c \uc88b\uc544\ud558\ub358 \uc774\uae00\uc2a4\uc758 \u3008Hotel California\u3009\uc758 \uc804\uc8fc \ubd80\ubd84 \uae30\ud0c0\uc18c\ub9ac\ub97c \ub4e3\uace0\uc120 \uc790\uc2e0\uc774 \uc88b\uc544\ud558\ub294 \ub178\ub798\ub97c, \ub9c8\uce58 \ucde8\ubbf8\ub85c \ub9cc\ub4e0 \uac83 \uac19\uc740 \ub178\ub798\ub97c \ub9cc\ub4e4\uc5b4\ub0c8\ub2e4. \uc0ac\uce74\uc774 \uc774\uc988\ubbf8 \ub294 \uc6b0\uc5f0\ud788 \ubc29\ubb38\ud55c \ube59\uc758 \uc624\ub514\uc158\uc5d0\uc11c \ud1a0\ucfe0\ub098\uac00\uac00 \uc791\uace1\ud55c \ub370\ubaa8 \ud14c\uc774\ud504\ub97c \ub4e3\uace0\uc11c \ud06c\uac8c \ud638\ud3c9\ud55c \ub4a4 \uadf8\ub97c \ucd94\ucc9c\ud55c\ub2e4. 1996\ub144\uacbd, \ube44\ub85d \uc568\ubc94\uc5d0\ub294 \uc218\ub85d\ub418\uc5b4 \uc788\uc9c0 \uc54a\uc558\uace0 \uace1\uba85\uc740 \u3008Fallin' of the Rain\u3009\uc774\ub780 \uc774\ub984\uc774\uc5c8\uc9c0\ub9cc, \ud6c4\ub834\uad6c\ub9cc\uc740 \uc774\ubbf8 \uc874\uc7ac\ud574\uc11c \uce90\ub17c\uc758 \uce74\uba54\ub77c NEW EOS KISS\uc758 \uad11\uace0 \uc0bd\uc785\uace1\uc73c\ub85c \uc0ac\uc6a9\ub418\uace0 \uc788\uc5c8\ub2e4.", "paragraph_answer": "A\uc0ac\uc774\ub4dc \uace1\uc740 \ucd5c\uc18c\ud55c 1996\ub144\ubd80\ud130 \uc774\ubbf8 \uc874\uc7ac\ud588\ub358 \ub178\ub798\ub2e4. \uc791\uace1\uc790 \ud1a0\ucfe0\ub098\uac00\ub294 \ub370\ubaa8 \ud14c\uc774\ud504\ub97c \ub9cc\ub4e4\uc5b4 \uac01 \uc74c\ubc18\uc81c\uc791\uc0ac\ub85c \ubcf4\ub0b4\uc9c0\ub9cc \uac70\uc808\ub2f9\ud558\uace0, \uc791\uace1\uc5d0 \uc788\uc5b4 \uc815\uccb4\ub97c \uacaa\uc5b4 \uc74c\uc545\uac00\ub85c\uc11c\uc758 \uc778\uc0dd\uc744 \ub2e8\ub150\ud558\ub824\uace0 \ud55c\ub2e4. \uc774 \ub54c, \ud3c9\uc18c \uc88b\uc544\ud558\ub358 \uc774\uae00\uc2a4\uc758 \u3008Hotel California\u3009\uc758 \uc804\uc8fc \ubd80\ubd84 \uae30\ud0c0\uc18c\ub9ac\ub97c \ub4e3\uace0\uc120 \uc790\uc2e0\uc774 \uc88b\uc544\ud558\ub294 \ub178\ub798\ub97c, \ub9c8\uce58 \ucde8\ubbf8\ub85c \ub9cc\ub4e0 \uac83 \uac19\uc740 \ub178\ub798\ub97c \ub9cc\ub4e4\uc5b4\ub0c8\ub2e4. \uc0ac\uce74\uc774 \uc774\uc988\ubbf8 \ub294 \uc6b0\uc5f0\ud788 \ubc29\ubb38\ud55c \ube59\uc758 \uc624\ub514\uc158\uc5d0\uc11c \ud1a0\ucfe0\ub098\uac00\uac00 \uc791\uace1\ud55c \ub370\ubaa8 \ud14c\uc774\ud504\ub97c \ub4e3\uace0\uc11c \ud06c\uac8c \ud638\ud3c9\ud55c \ub4a4 \uadf8\ub97c \ucd94\ucc9c\ud55c\ub2e4. 1996\ub144\uacbd, \ube44\ub85d \uc568\ubc94\uc5d0\ub294 \uc218\ub85d\ub418\uc5b4 \uc788\uc9c0 \uc54a\uc558\uace0 \uace1\uba85\uc740 \u3008Fallin' of the Rain\u3009\uc774\ub780 \uc774\ub984\uc774\uc5c8\uc9c0\ub9cc, \ud6c4\ub834\uad6c\ub9cc\uc740 \uc774\ubbf8 \uc874\uc7ac\ud574\uc11c \uce90\ub17c\uc758 \uce74\uba54\ub77c NEW EOS KISS\uc758 \uad11\uace0 \uc0bd\uc785\uace1\uc73c\ub85c \uc0ac\uc6a9\ub418\uace0 \uc788\uc5c8\ub2e4.", "sentence_answer": " \uc0ac\uce74\uc774 \uc774\uc988\ubbf8 \ub294 \uc6b0\uc5f0\ud788 \ubc29\ubb38\ud55c \ube59\uc758 \uc624\ub514\uc158\uc5d0\uc11c \ud1a0\ucfe0\ub098\uac00\uac00 \uc791\uace1\ud55c \ub370\ubaa8 \ud14c\uc774\ud504\ub97c \ub4e3\uace0\uc11c \ud06c\uac8c \ud638\ud3c9\ud55c \ub4a4 \uadf8\ub97c \ucd94\ucc9c\ud55c\ub2e4.", "paragraph_id": "6458830-1-1", "paragraph_question": "question: \uc6b0\uc5f0\ud788 \ubc29\ubb38\ud55c \ube59\uc758 \uc624\ub514\uc158\uc5d0\uc11c \ud1a0\ucfe0\ub098\uac00\uac00 \ub9cc\ub4e0 \ub370\ubaa8 \ud14c\uc774\ud504\ub97c \ub4e3\uace0 \ud06c\uac8c \ud638\ud3c9\ud55c \uc0ac\ub78c\uc740 \ub204\uad6c\uc778\uac00?, context: A\uc0ac\uc774\ub4dc \uace1\uc740 \ucd5c\uc18c\ud55c 1996\ub144\ubd80\ud130 \uc774\ubbf8 \uc874\uc7ac\ud588\ub358 \ub178\ub798\ub2e4. \uc791\uace1\uc790 \ud1a0\ucfe0\ub098\uac00\ub294 \ub370\ubaa8 \ud14c\uc774\ud504\ub97c \ub9cc\ub4e4\uc5b4 \uac01 \uc74c\ubc18\uc81c\uc791\uc0ac\ub85c \ubcf4\ub0b4\uc9c0\ub9cc \uac70\uc808\ub2f9\ud558\uace0, \uc791\uace1\uc5d0 \uc788\uc5b4 \uc815\uccb4\ub97c \uacaa\uc5b4 \uc74c\uc545\uac00\ub85c\uc11c\uc758 \uc778\uc0dd\uc744 \ub2e8\ub150\ud558\ub824\uace0 \ud55c\ub2e4. \uc774 \ub54c, \ud3c9\uc18c \uc88b\uc544\ud558\ub358 \uc774\uae00\uc2a4\uc758 \u3008Hotel California\u3009\uc758 \uc804\uc8fc \ubd80\ubd84 \uae30\ud0c0\uc18c\ub9ac\ub97c \ub4e3\uace0\uc120 \uc790\uc2e0\uc774 \uc88b\uc544\ud558\ub294 \ub178\ub798\ub97c, \ub9c8\uce58 \ucde8\ubbf8\ub85c \ub9cc\ub4e0 \uac83 \uac19\uc740 \ub178\ub798\ub97c \ub9cc\ub4e4\uc5b4\ub0c8\ub2e4. \uc0ac\uce74\uc774 \uc774\uc988\ubbf8\ub294 \uc6b0\uc5f0\ud788 \ubc29\ubb38\ud55c \ube59\uc758 \uc624\ub514\uc158\uc5d0\uc11c \ud1a0\ucfe0\ub098\uac00\uac00 \uc791\uace1\ud55c \ub370\ubaa8 \ud14c\uc774\ud504\ub97c \ub4e3\uace0\uc11c \ud06c\uac8c \ud638\ud3c9\ud55c \ub4a4 \uadf8\ub97c \ucd94\ucc9c\ud55c\ub2e4. 1996\ub144\uacbd, \ube44\ub85d \uc568\ubc94\uc5d0\ub294 \uc218\ub85d\ub418\uc5b4 \uc788\uc9c0 \uc54a\uc558\uace0 \uace1\uba85\uc740 \u3008Fallin' of the Rain\u3009\uc774\ub780 \uc774\ub984\uc774\uc5c8\uc9c0\ub9cc, \ud6c4\ub834\uad6c\ub9cc\uc740 \uc774\ubbf8 \uc874\uc7ac\ud574\uc11c \uce90\ub17c\uc758 \uce74\uba54\ub77c NEW EOS KISS\uc758 \uad11\uace0 \uc0bd\uc785\uace1\uc73c\ub85c \uc0ac\uc6a9\ub418\uace0 \uc788\uc5c8\ub2e4."} {"question": "\uce74\uc2a4\ud2f8\ub9ac\uc624\ub124\ub294 \uc5b8\uc81c \uc911\uad6d \uc655\uc2e4\uc5d0 \uc654\ub294\uac00?", "paragraph": "\uccad\ub300 \uad81\uc815\ud68c\ud654\ub294 \uac15\ud76c\uc81c\uc5d0\uc11c \uc639\uc815\uc81c\ub97c \uac70\uccd0 \uac74\ub96d\uc81c \uc2dc\uae30\uc5d0 \uac00\uc7a5 \ubc88\uc131\ud558\uc600\ub2e4. \uac15\ud76c\uc81c \uc7ac\uc704 \ub9d0\uae30\uc5d0 \uce74\uc2a4\ud2f8\ub9ac\uc624\ub124\uac00 \uc911\uad6d \uc655\uc2e4\uc5d0 \uc634\uc5d0 \ub530\ub77c, \uccad\ub300 \uad81\uc815\ud68c\ud654\ub294 \uc11c\uc591\uc758 \uc601\ud5a5\uc744 \ubc1b\uae30 \uc2dc\uc791\ud558\uc600\ub2e4. \uadf8\uc758 \ub4a4\ub97c \uc774\uc5b4 \uc544\ud2f0\ub808(Jean Denis Attiret, \uc911\uad6d\uba85: \u738b\u81f4\u8aa0 \ub610\ub294 \u5df4\u5fb7\u5c3c, 1702\ub144 ~ 1768\ub144), \uc2dc\ucf08\ubc14\ub974\ud2b8(Ignatius Sichelbarth, \uc911\uad6d\uba85: \u827e\u5553\u8499, 1708\ub144 ~ 1780\ub144), \ud504\uc640\ub85c(Louis de Poirot, \uc911\uad6d\uba85: \u8cc0\u6df8\u6cf0, 1735\ub144 ~ 1814\ub144), \ub2e4\ub9c8\uc13c(John Damascene, \uc911\uad6d\uba85: \u5b89\u5fb7\u7fa9, ? ~ 1781\ub144), \ud310\uc9c0(Joseph Panzi, \uc911\uad6d\uba85: \u6f58\u5ef7\u7ae0, 1733\ub144 ~ 1812\ub144?) \ub4f1 \ub9ce\uc740 \uc720\ub7fd\uc778\ub4e4\uc774 \uc911\uad6d\uc5d0 \uc654\uc73c\uba70, \uc774\ub4e4\uc740 \uc911\uad6d \ud654\uac00\ub4e4\uacfc \uc911\uad6d\uacfc \uc11c\uc591\uc758 \uc591\uc2dd\uc744 \uacb0\ud569\ud55c \uc0c8\ub85c\uc6b4 \ud654\ud48d\uc744 \ub9cc\ub4e4\uc5b4\ub098\uac14\ub2e4. \uadf8\ub9ac\ud558\uc5ec \uad81\uc815\ud68c\ud654\ub294 \uba85\uc554\uacfc \uc6d0\uadfc\ubc95\uc744 \uc911\uc2dc\ud558\uace0, \ub2f9\uc2dc\uc758 \uc8fc\uc694 \uc0ac\uac74\uc744 \uadf8\ub9bc\uc5d0 \ub2f4\ub294 \ub4f1 \uc11c\uc591\uc758 \uc601\ud5a5\uc744 \uac15\ud558\uac8c \ubc1b\uc558\ub2e4. \uc774\ub4e4 \uc120\uad50\uc0ac \ud654\uac00\ub4e4\uc774 \uc720\ud654\uc640 \ub3d9\ud310 \uc5d0\uce6d \uae30\ubc95\uc744 \uc911\uad6d\uc5d0 \ub4e4\uc5ec\uc640 \ub450 \uc7a5\ub974\uc758 \ud68c\ud654\uac00 \uccad \uc655\uc2e4\uc5d0 \ucc98\uc74c\uc73c\ub85c \ub098\ud0c0\ub098\uae30 \uc2dc\uc791\ud588\ub2e4.", "answer": "\uac15\ud76c\uc81c \uc7ac\uc704 \ub9d0\uae30", "sentence": "\uac15\ud76c\uc81c \uc7ac\uc704 \ub9d0\uae30 \uc5d0 \uce74\uc2a4\ud2f8\ub9ac\uc624\ub124\uac00 \uc911\uad6d \uc655\uc2e4\uc5d0 \uc634\uc5d0 \ub530\ub77c, \uccad\ub300 \uad81\uc815\ud68c\ud654\ub294 \uc11c\uc591\uc758 \uc601\ud5a5\uc744 \ubc1b\uae30 \uc2dc\uc791\ud558\uc600\ub2e4.", "paragraph_sentence": "\uccad\ub300 \uad81\uc815\ud68c\ud654\ub294 \uac15\ud76c\uc81c\uc5d0\uc11c \uc639\uc815\uc81c\ub97c \uac70\uccd0 \uac74\ub96d\uc81c \uc2dc\uae30\uc5d0 \uac00\uc7a5 \ubc88\uc131\ud558\uc600\ub2e4. \uac15\ud76c\uc81c \uc7ac\uc704 \ub9d0\uae30 \uc5d0 \uce74\uc2a4\ud2f8\ub9ac\uc624\ub124\uac00 \uc911\uad6d \uc655\uc2e4\uc5d0 \uc634\uc5d0 \ub530\ub77c, \uccad\ub300 \uad81\uc815\ud68c\ud654\ub294 \uc11c\uc591\uc758 \uc601\ud5a5\uc744 \ubc1b\uae30 \uc2dc\uc791\ud558\uc600\ub2e4. \uadf8\uc758 \ub4a4\ub97c \uc774\uc5b4 \uc544\ud2f0\ub808(Jean Denis Attiret, \uc911\uad6d\uba85: \u738b\u81f4\u8aa0 \ub610\ub294 \u5df4\u5fb7\u5c3c, 1702\ub144 ~ 1768\ub144), \uc2dc\ucf08\ubc14\ub974\ud2b8(Ignatius Sichelbarth, \uc911\uad6d\uba85: \u827e\u5553\u8499, 1708\ub144 ~ 1780\ub144), \ud504\uc640\ub85c(Louis de Poirot, \uc911\uad6d\uba85: \u8cc0\u6df8\u6cf0, 1735\ub144 ~ 1814\ub144), \ub2e4\ub9c8\uc13c(John Damascene, \uc911\uad6d\uba85: \u5b89\u5fb7\u7fa9, ? ~ 1781\ub144), \ud310\uc9c0(Joseph Panzi, \uc911\uad6d\uba85: \u6f58\u5ef7\u7ae0, 1733\ub144 ~ 1812\ub144?) \ub4f1 \ub9ce\uc740 \uc720\ub7fd\uc778\ub4e4\uc774 \uc911\uad6d\uc5d0 \uc654\uc73c\uba70, \uc774\ub4e4\uc740 \uc911\uad6d \ud654\uac00\ub4e4\uacfc \uc911\uad6d\uacfc \uc11c\uc591\uc758 \uc591\uc2dd\uc744 \uacb0\ud569\ud55c \uc0c8\ub85c\uc6b4 \ud654\ud48d\uc744 \ub9cc\ub4e4\uc5b4\ub098\uac14\ub2e4. \uadf8\ub9ac\ud558\uc5ec \uad81\uc815\ud68c\ud654\ub294 \uba85\uc554\uacfc \uc6d0\uadfc\ubc95\uc744 \uc911\uc2dc\ud558\uace0, \ub2f9\uc2dc\uc758 \uc8fc\uc694 \uc0ac\uac74\uc744 \uadf8\ub9bc\uc5d0 \ub2f4\ub294 \ub4f1 \uc11c\uc591\uc758 \uc601\ud5a5\uc744 \uac15\ud558\uac8c \ubc1b\uc558\ub2e4. \uc774\ub4e4 \uc120\uad50\uc0ac \ud654\uac00\ub4e4\uc774 \uc720\ud654\uc640 \ub3d9\ud310 \uc5d0\uce6d \uae30\ubc95\uc744 \uc911\uad6d\uc5d0 \ub4e4\uc5ec\uc640 \ub450 \uc7a5\ub974\uc758 \ud68c\ud654\uac00 \uccad \uc655\uc2e4\uc5d0 \ucc98\uc74c\uc73c\ub85c \ub098\ud0c0\ub098\uae30 \uc2dc\uc791\ud588\ub2e4.", "paragraph_answer": "\uccad\ub300 \uad81\uc815\ud68c\ud654\ub294 \uac15\ud76c\uc81c\uc5d0\uc11c \uc639\uc815\uc81c\ub97c \uac70\uccd0 \uac74\ub96d\uc81c \uc2dc\uae30\uc5d0 \uac00\uc7a5 \ubc88\uc131\ud558\uc600\ub2e4. \uac15\ud76c\uc81c \uc7ac\uc704 \ub9d0\uae30 \uc5d0 \uce74\uc2a4\ud2f8\ub9ac\uc624\ub124\uac00 \uc911\uad6d \uc655\uc2e4\uc5d0 \uc634\uc5d0 \ub530\ub77c, \uccad\ub300 \uad81\uc815\ud68c\ud654\ub294 \uc11c\uc591\uc758 \uc601\ud5a5\uc744 \ubc1b\uae30 \uc2dc\uc791\ud558\uc600\ub2e4. \uadf8\uc758 \ub4a4\ub97c \uc774\uc5b4 \uc544\ud2f0\ub808(Jean Denis Attiret, \uc911\uad6d\uba85: \u738b\u81f4\u8aa0 \ub610\ub294 \u5df4\u5fb7\u5c3c, 1702\ub144 ~ 1768\ub144), \uc2dc\ucf08\ubc14\ub974\ud2b8(Ignatius Sichelbarth, \uc911\uad6d\uba85: \u827e\u5553\u8499, 1708\ub144 ~ 1780\ub144), \ud504\uc640\ub85c(Louis de Poirot, \uc911\uad6d\uba85: \u8cc0\u6df8\u6cf0, 1735\ub144 ~ 1814\ub144), \ub2e4\ub9c8\uc13c(John Damascene, \uc911\uad6d\uba85: \u5b89\u5fb7\u7fa9, ? ~ 1781\ub144), \ud310\uc9c0(Joseph Panzi, \uc911\uad6d\uba85: \u6f58\u5ef7\u7ae0, 1733\ub144 ~ 1812\ub144?) \ub4f1 \ub9ce\uc740 \uc720\ub7fd\uc778\ub4e4\uc774 \uc911\uad6d\uc5d0 \uc654\uc73c\uba70, \uc774\ub4e4\uc740 \uc911\uad6d \ud654\uac00\ub4e4\uacfc \uc911\uad6d\uacfc \uc11c\uc591\uc758 \uc591\uc2dd\uc744 \uacb0\ud569\ud55c \uc0c8\ub85c\uc6b4 \ud654\ud48d\uc744 \ub9cc\ub4e4\uc5b4\ub098\uac14\ub2e4. \uadf8\ub9ac\ud558\uc5ec \uad81\uc815\ud68c\ud654\ub294 \uba85\uc554\uacfc \uc6d0\uadfc\ubc95\uc744 \uc911\uc2dc\ud558\uace0, \ub2f9\uc2dc\uc758 \uc8fc\uc694 \uc0ac\uac74\uc744 \uadf8\ub9bc\uc5d0 \ub2f4\ub294 \ub4f1 \uc11c\uc591\uc758 \uc601\ud5a5\uc744 \uac15\ud558\uac8c \ubc1b\uc558\ub2e4. \uc774\ub4e4 \uc120\uad50\uc0ac \ud654\uac00\ub4e4\uc774 \uc720\ud654\uc640 \ub3d9\ud310 \uc5d0\uce6d \uae30\ubc95\uc744 \uc911\uad6d\uc5d0 \ub4e4\uc5ec\uc640 \ub450 \uc7a5\ub974\uc758 \ud68c\ud654\uac00 \uccad \uc655\uc2e4\uc5d0 \ucc98\uc74c\uc73c\ub85c \ub098\ud0c0\ub098\uae30 \uc2dc\uc791\ud588\ub2e4.", "sentence_answer": " \uac15\ud76c\uc81c \uc7ac\uc704 \ub9d0\uae30 \uc5d0 \uce74\uc2a4\ud2f8\ub9ac\uc624\ub124\uac00 \uc911\uad6d \uc655\uc2e4\uc5d0 \uc634\uc5d0 \ub530\ub77c, \uccad\ub300 \uad81\uc815\ud68c\ud654\ub294 \uc11c\uc591\uc758 \uc601\ud5a5\uc744 \ubc1b\uae30 \uc2dc\uc791\ud558\uc600\ub2e4.", "paragraph_id": "6555956-7-0", "paragraph_question": "question: \uce74\uc2a4\ud2f8\ub9ac\uc624\ub124\ub294 \uc5b8\uc81c \uc911\uad6d \uc655\uc2e4\uc5d0 \uc654\ub294\uac00?, context: \uccad\ub300 \uad81\uc815\ud68c\ud654\ub294 \uac15\ud76c\uc81c\uc5d0\uc11c \uc639\uc815\uc81c\ub97c \uac70\uccd0 \uac74\ub96d\uc81c \uc2dc\uae30\uc5d0 \uac00\uc7a5 \ubc88\uc131\ud558\uc600\ub2e4. \uac15\ud76c\uc81c \uc7ac\uc704 \ub9d0\uae30\uc5d0 \uce74\uc2a4\ud2f8\ub9ac\uc624\ub124\uac00 \uc911\uad6d \uc655\uc2e4\uc5d0 \uc634\uc5d0 \ub530\ub77c, \uccad\ub300 \uad81\uc815\ud68c\ud654\ub294 \uc11c\uc591\uc758 \uc601\ud5a5\uc744 \ubc1b\uae30 \uc2dc\uc791\ud558\uc600\ub2e4. \uadf8\uc758 \ub4a4\ub97c \uc774\uc5b4 \uc544\ud2f0\ub808(Jean Denis Attiret, \uc911\uad6d\uba85: \u738b\u81f4\u8aa0 \ub610\ub294 \u5df4\u5fb7\u5c3c, 1702\ub144 ~ 1768\ub144), \uc2dc\ucf08\ubc14\ub974\ud2b8(Ignatius Sichelbarth, \uc911\uad6d\uba85: \u827e\u5553\u8499, 1708\ub144 ~ 1780\ub144), \ud504\uc640\ub85c(Louis de Poirot, \uc911\uad6d\uba85: \u8cc0\u6df8\u6cf0, 1735\ub144 ~ 1814\ub144), \ub2e4\ub9c8\uc13c(John Damascene, \uc911\uad6d\uba85: \u5b89\u5fb7\u7fa9, ? ~ 1781\ub144), \ud310\uc9c0(Joseph Panzi, \uc911\uad6d\uba85: \u6f58\u5ef7\u7ae0, 1733\ub144 ~ 1812\ub144?) \ub4f1 \ub9ce\uc740 \uc720\ub7fd\uc778\ub4e4\uc774 \uc911\uad6d\uc5d0 \uc654\uc73c\uba70, \uc774\ub4e4\uc740 \uc911\uad6d \ud654\uac00\ub4e4\uacfc \uc911\uad6d\uacfc \uc11c\uc591\uc758 \uc591\uc2dd\uc744 \uacb0\ud569\ud55c \uc0c8\ub85c\uc6b4 \ud654\ud48d\uc744 \ub9cc\ub4e4\uc5b4\ub098\uac14\ub2e4. \uadf8\ub9ac\ud558\uc5ec \uad81\uc815\ud68c\ud654\ub294 \uba85\uc554\uacfc \uc6d0\uadfc\ubc95\uc744 \uc911\uc2dc\ud558\uace0, \ub2f9\uc2dc\uc758 \uc8fc\uc694 \uc0ac\uac74\uc744 \uadf8\ub9bc\uc5d0 \ub2f4\ub294 \ub4f1 \uc11c\uc591\uc758 \uc601\ud5a5\uc744 \uac15\ud558\uac8c \ubc1b\uc558\ub2e4. \uc774\ub4e4 \uc120\uad50\uc0ac \ud654\uac00\ub4e4\uc774 \uc720\ud654\uc640 \ub3d9\ud310 \uc5d0\uce6d \uae30\ubc95\uc744 \uc911\uad6d\uc5d0 \ub4e4\uc5ec\uc640 \ub450 \uc7a5\ub974\uc758 \ud68c\ud654\uac00 \uccad \uc655\uc2e4\uc5d0 \ucc98\uc74c\uc73c\ub85c \ub098\ud0c0\ub098\uae30 \uc2dc\uc791\ud588\ub2e4."} {"question": "\uc7ac\ub2e8 \uc774\uc0ac\uc7a5\uc758 \uc640\uc774\ud504\ub294 \ub204\uad6c\uc778\uac00?", "paragraph": "\uc5b4\ub9b0 \uc2dc\uc808, \uc790\uc2e0\uc758 \uace0\ud5a5\uc9d1\uc5d0 \uc138 \ub4e4\uc5b4 \uc0b4\ub358 \uc724\ud76c\uac00 \uc81c \uc544\ubc84\uc9c0\ud55c\ud14c \uc5bb\uc5b4\ub9de\uace0 \uc6b8 \ub54c\ub9c8\ub2e4 \uc5c5\uc5b4\uc11c \ub2ec\ub798\uc92c\ub2e4. \uadf8\uc758 \ub4f1 \ub4a4\uc5d0\uc11c \uc724\ud76c\uac00 \uc0ac\ub791\uc744 \ud0a4\uc6b0\uace0 \uc788\ub294 \uc904\uc740 \uafc8\uc5d0\ub3c4 \ubaa8\ub978 \ucc44, \uc9c4\uc11c\uc640 \uc0ac\ub791\uc5d0 \ube60\uc84c\ub2e4. \uadf8\uc5d0\uac8c \uccab\uc0ac\ub791\uc740 \uc544\ub0b4 \uc9c4\uc11c\uc774\uace0 \ub9c8\uc9c0\ub9c9 \uc21c\uac04\uae4c\uc9c0 \uc624\uc9c1 \uc544\ub0b4 \uc9c4\uc11c\ub9cc \uc0ac\ub791\ud560 \uc790\uc2e0 \uc788\ub294\ub370\u2026 \uacb0\ud63c\ud55c \uc774\ub798 \uc904\uace7 \uac00\uc815 \uacbd\uc81c\ub97c \uc544\ub0b4\uac00 \ucc45\uc784\uc9c0\uace0 \uc788\uc73c\ub2c8 \ud658\uc7a5\ud560 \ub178\ub987\uc774\ub2e4. \uc544\ub0b4\ub294 \uac78\ud54f\ud558\uba74 \uadf8\ub97c '\uacbd\uacc4\uac00 \uc5c6\ub2e4, \uc120\uc744 \uae0b\uc9c0 \ubabb\ud55c\ub2e4' \uace0 \ud0c0\ubc15\uc774\ub2e4. \ud558\uae34 \uc9c0\ubc29\ub300 \ucd9c\uc2e0\uc73c\ub85c \uad50\uc218\ub97c \uafc8\uafb8\uba74 \uafc0\uc218\ub85d \ud604\uc2e4\uc774 \uc790\uc2e0\uc744 \ubd84\uc218\ub3c4 \ubaa8\ub974\uace0 \uc790\uafb8\ub9cc \ud559\uad50\uc758 \ube44\ub9ac\ub97c \uaf2c\uc9d1\uc5b4\ub0b4\ub294 \ud22c\uc0ac\ub85c \ub9cc\ub4dc\ub2c8 \uc810\uc810 \ub9c8\uc74c\ub9cc \uc870\uae09\ud574 \uc9c4\ub2e4. \uadf8 \ub54c \uc7ac\ub2e8 \uc774\uc0ac\uc7a5\uc758 \uc640\uc774\ud504\uac00 \ub418\uc5b4\uc788\ub294 \uc724\ud76c\ub97c \ub9cc\ub0ac\ub2e4. \ub355\ubd84\uc5d0 \uc774\uc0ac\uc7a5\uacfc \ub3c5\ub300\ub97c \ud560 \ub9cc\ud07c \uce5c\ud574\uc84c\uc73c\ub2c8 \ud559\uc5f0\uc744 \ub6f0\uc5b4\ub118\uc744 \ub3d9\uc544\uc904\uc744 \uc5bb\uc5c8\ub2e4 \uc2f6\uc5c8\ub294\ub370, \uc774\uc0ac\uc7a5\uc758 \uc18d\uc148\uc744 \ub208\uce58 \ucc44\uace0 \ub9d0\uc558\ub2e4. \uce5c\uc808\uc744 \uac00\uc7a5\ud55c \uadf8\uc758 \uc758\uc2ec\uc744\u2026", "answer": "\uc724\ud76c", "sentence": "\uc5b4\ub9b0 \uc2dc\uc808, \uc790\uc2e0\uc758 \uace0\ud5a5\uc9d1\uc5d0 \uc138 \ub4e4\uc5b4 \uc0b4\ub358 \uc724\ud76c \uac00 \uc81c \uc544\ubc84\uc9c0\ud55c\ud14c \uc5bb\uc5b4\ub9de\uace0 \uc6b8 \ub54c\ub9c8\ub2e4 \uc5c5\uc5b4\uc11c \ub2ec\ub798\uc92c\ub2e4.", "paragraph_sentence": " \uc5b4\ub9b0 \uc2dc\uc808, \uc790\uc2e0\uc758 \uace0\ud5a5\uc9d1\uc5d0 \uc138 \ub4e4\uc5b4 \uc0b4\ub358 \uc724\ud76c \uac00 \uc81c \uc544\ubc84\uc9c0\ud55c\ud14c \uc5bb\uc5b4\ub9de\uace0 \uc6b8 \ub54c\ub9c8\ub2e4 \uc5c5\uc5b4\uc11c \ub2ec\ub798\uc92c\ub2e4. \uadf8\uc758 \ub4f1 \ub4a4\uc5d0\uc11c \uc724\ud76c\uac00 \uc0ac\ub791\uc744 \ud0a4\uc6b0\uace0 \uc788\ub294 \uc904\uc740 \uafc8\uc5d0\ub3c4 \ubaa8\ub978 \ucc44, \uc9c4\uc11c\uc640 \uc0ac\ub791\uc5d0 \ube60\uc84c\ub2e4. \uadf8\uc5d0\uac8c \uccab\uc0ac\ub791\uc740 \uc544\ub0b4 \uc9c4\uc11c\uc774\uace0 \ub9c8\uc9c0\ub9c9 \uc21c\uac04\uae4c\uc9c0 \uc624\uc9c1 \uc544\ub0b4 \uc9c4\uc11c\ub9cc \uc0ac\ub791\ud560 \uc790\uc2e0 \uc788\ub294\ub370\u2026 \uacb0\ud63c\ud55c \uc774\ub798 \uc904\uace7 \uac00\uc815 \uacbd\uc81c\ub97c \uc544\ub0b4\uac00 \ucc45\uc784\uc9c0\uace0 \uc788\uc73c\ub2c8 \ud658\uc7a5\ud560 \ub178\ub987\uc774\ub2e4. \uc544\ub0b4\ub294 \uac78\ud54f\ud558\uba74 \uadf8\ub97c '\uacbd\uacc4\uac00 \uc5c6\ub2e4, \uc120\uc744 \uae0b\uc9c0 \ubabb\ud55c\ub2e4' \uace0 \ud0c0\ubc15\uc774\ub2e4. \ud558\uae34 \uc9c0\ubc29\ub300 \ucd9c\uc2e0\uc73c\ub85c \uad50\uc218\ub97c \uafc8\uafb8\uba74 \uafc0\uc218\ub85d \ud604\uc2e4\uc774 \uc790\uc2e0\uc744 \ubd84\uc218\ub3c4 \ubaa8\ub974\uace0 \uc790\uafb8\ub9cc \ud559\uad50\uc758 \ube44\ub9ac\ub97c \uaf2c\uc9d1\uc5b4\ub0b4\ub294 \ud22c\uc0ac\ub85c \ub9cc\ub4dc\ub2c8 \uc810\uc810 \ub9c8\uc74c\ub9cc \uc870\uae09\ud574 \uc9c4\ub2e4. \uadf8 \ub54c \uc7ac\ub2e8 \uc774\uc0ac\uc7a5\uc758 \uc640\uc774\ud504\uac00 \ub418\uc5b4\uc788\ub294 \uc724\ud76c\ub97c \ub9cc\ub0ac\ub2e4. \ub355\ubd84\uc5d0 \uc774\uc0ac\uc7a5\uacfc \ub3c5\ub300\ub97c \ud560 \ub9cc\ud07c \uce5c\ud574\uc84c\uc73c\ub2c8 \ud559\uc5f0\uc744 \ub6f0\uc5b4\ub118\uc744 \ub3d9\uc544\uc904\uc744 \uc5bb\uc5c8\ub2e4 \uc2f6\uc5c8\ub294\ub370, \uc774\uc0ac\uc7a5\uc758 \uc18d\uc148\uc744 \ub208\uce58 \ucc44\uace0 \ub9d0\uc558\ub2e4. \uce5c\uc808\uc744 \uac00\uc7a5\ud55c \uadf8\uc758 \uc758\uc2ec\uc744\u2026", "paragraph_answer": "\uc5b4\ub9b0 \uc2dc\uc808, \uc790\uc2e0\uc758 \uace0\ud5a5\uc9d1\uc5d0 \uc138 \ub4e4\uc5b4 \uc0b4\ub358 \uc724\ud76c \uac00 \uc81c \uc544\ubc84\uc9c0\ud55c\ud14c \uc5bb\uc5b4\ub9de\uace0 \uc6b8 \ub54c\ub9c8\ub2e4 \uc5c5\uc5b4\uc11c \ub2ec\ub798\uc92c\ub2e4. \uadf8\uc758 \ub4f1 \ub4a4\uc5d0\uc11c \uc724\ud76c\uac00 \uc0ac\ub791\uc744 \ud0a4\uc6b0\uace0 \uc788\ub294 \uc904\uc740 \uafc8\uc5d0\ub3c4 \ubaa8\ub978 \ucc44, \uc9c4\uc11c\uc640 \uc0ac\ub791\uc5d0 \ube60\uc84c\ub2e4. \uadf8\uc5d0\uac8c \uccab\uc0ac\ub791\uc740 \uc544\ub0b4 \uc9c4\uc11c\uc774\uace0 \ub9c8\uc9c0\ub9c9 \uc21c\uac04\uae4c\uc9c0 \uc624\uc9c1 \uc544\ub0b4 \uc9c4\uc11c\ub9cc \uc0ac\ub791\ud560 \uc790\uc2e0 \uc788\ub294\ub370\u2026 \uacb0\ud63c\ud55c \uc774\ub798 \uc904\uace7 \uac00\uc815 \uacbd\uc81c\ub97c \uc544\ub0b4\uac00 \ucc45\uc784\uc9c0\uace0 \uc788\uc73c\ub2c8 \ud658\uc7a5\ud560 \ub178\ub987\uc774\ub2e4. \uc544\ub0b4\ub294 \uac78\ud54f\ud558\uba74 \uadf8\ub97c '\uacbd\uacc4\uac00 \uc5c6\ub2e4, \uc120\uc744 \uae0b\uc9c0 \ubabb\ud55c\ub2e4' \uace0 \ud0c0\ubc15\uc774\ub2e4. \ud558\uae34 \uc9c0\ubc29\ub300 \ucd9c\uc2e0\uc73c\ub85c \uad50\uc218\ub97c \uafc8\uafb8\uba74 \uafc0\uc218\ub85d \ud604\uc2e4\uc774 \uc790\uc2e0\uc744 \ubd84\uc218\ub3c4 \ubaa8\ub974\uace0 \uc790\uafb8\ub9cc \ud559\uad50\uc758 \ube44\ub9ac\ub97c \uaf2c\uc9d1\uc5b4\ub0b4\ub294 \ud22c\uc0ac\ub85c \ub9cc\ub4dc\ub2c8 \uc810\uc810 \ub9c8\uc74c\ub9cc \uc870\uae09\ud574 \uc9c4\ub2e4. \uadf8 \ub54c \uc7ac\ub2e8 \uc774\uc0ac\uc7a5\uc758 \uc640\uc774\ud504\uac00 \ub418\uc5b4\uc788\ub294 \uc724\ud76c\ub97c \ub9cc\ub0ac\ub2e4. \ub355\ubd84\uc5d0 \uc774\uc0ac\uc7a5\uacfc \ub3c5\ub300\ub97c \ud560 \ub9cc\ud07c \uce5c\ud574\uc84c\uc73c\ub2c8 \ud559\uc5f0\uc744 \ub6f0\uc5b4\ub118\uc744 \ub3d9\uc544\uc904\uc744 \uc5bb\uc5c8\ub2e4 \uc2f6\uc5c8\ub294\ub370, \uc774\uc0ac\uc7a5\uc758 \uc18d\uc148\uc744 \ub208\uce58 \ucc44\uace0 \ub9d0\uc558\ub2e4. \uce5c\uc808\uc744 \uac00\uc7a5\ud55c \uadf8\uc758 \uc758\uc2ec\uc744\u2026", "sentence_answer": "\uc5b4\ub9b0 \uc2dc\uc808, \uc790\uc2e0\uc758 \uace0\ud5a5\uc9d1\uc5d0 \uc138 \ub4e4\uc5b4 \uc0b4\ub358 \uc724\ud76c \uac00 \uc81c \uc544\ubc84\uc9c0\ud55c\ud14c \uc5bb\uc5b4\ub9de\uace0 \uc6b8 \ub54c\ub9c8\ub2e4 \uc5c5\uc5b4\uc11c \ub2ec\ub798\uc92c\ub2e4.", "paragraph_id": "6556753-2-0", "paragraph_question": "question: \uc7ac\ub2e8 \uc774\uc0ac\uc7a5\uc758 \uc640\uc774\ud504\ub294 \ub204\uad6c\uc778\uac00?, context: \uc5b4\ub9b0 \uc2dc\uc808, \uc790\uc2e0\uc758 \uace0\ud5a5\uc9d1\uc5d0 \uc138 \ub4e4\uc5b4 \uc0b4\ub358 \uc724\ud76c\uac00 \uc81c \uc544\ubc84\uc9c0\ud55c\ud14c \uc5bb\uc5b4\ub9de\uace0 \uc6b8 \ub54c\ub9c8\ub2e4 \uc5c5\uc5b4\uc11c \ub2ec\ub798\uc92c\ub2e4. \uadf8\uc758 \ub4f1 \ub4a4\uc5d0\uc11c \uc724\ud76c\uac00 \uc0ac\ub791\uc744 \ud0a4\uc6b0\uace0 \uc788\ub294 \uc904\uc740 \uafc8\uc5d0\ub3c4 \ubaa8\ub978 \ucc44, \uc9c4\uc11c\uc640 \uc0ac\ub791\uc5d0 \ube60\uc84c\ub2e4. \uadf8\uc5d0\uac8c \uccab\uc0ac\ub791\uc740 \uc544\ub0b4 \uc9c4\uc11c\uc774\uace0 \ub9c8\uc9c0\ub9c9 \uc21c\uac04\uae4c\uc9c0 \uc624\uc9c1 \uc544\ub0b4 \uc9c4\uc11c\ub9cc \uc0ac\ub791\ud560 \uc790\uc2e0 \uc788\ub294\ub370\u2026 \uacb0\ud63c\ud55c \uc774\ub798 \uc904\uace7 \uac00\uc815 \uacbd\uc81c\ub97c \uc544\ub0b4\uac00 \ucc45\uc784\uc9c0\uace0 \uc788\uc73c\ub2c8 \ud658\uc7a5\ud560 \ub178\ub987\uc774\ub2e4. \uc544\ub0b4\ub294 \uac78\ud54f\ud558\uba74 \uadf8\ub97c '\uacbd\uacc4\uac00 \uc5c6\ub2e4, \uc120\uc744 \uae0b\uc9c0 \ubabb\ud55c\ub2e4' \uace0 \ud0c0\ubc15\uc774\ub2e4. \ud558\uae34 \uc9c0\ubc29\ub300 \ucd9c\uc2e0\uc73c\ub85c \uad50\uc218\ub97c \uafc8\uafb8\uba74 \uafc0\uc218\ub85d \ud604\uc2e4\uc774 \uc790\uc2e0\uc744 \ubd84\uc218\ub3c4 \ubaa8\ub974\uace0 \uc790\uafb8\ub9cc \ud559\uad50\uc758 \ube44\ub9ac\ub97c \uaf2c\uc9d1\uc5b4\ub0b4\ub294 \ud22c\uc0ac\ub85c \ub9cc\ub4dc\ub2c8 \uc810\uc810 \ub9c8\uc74c\ub9cc \uc870\uae09\ud574 \uc9c4\ub2e4. \uadf8 \ub54c \uc7ac\ub2e8 \uc774\uc0ac\uc7a5\uc758 \uc640\uc774\ud504\uac00 \ub418\uc5b4\uc788\ub294 \uc724\ud76c\ub97c \ub9cc\ub0ac\ub2e4. \ub355\ubd84\uc5d0 \uc774\uc0ac\uc7a5\uacfc \ub3c5\ub300\ub97c \ud560 \ub9cc\ud07c \uce5c\ud574\uc84c\uc73c\ub2c8 \ud559\uc5f0\uc744 \ub6f0\uc5b4\ub118\uc744 \ub3d9\uc544\uc904\uc744 \uc5bb\uc5c8\ub2e4 \uc2f6\uc5c8\ub294\ub370, \uc774\uc0ac\uc7a5\uc758 \uc18d\uc148\uc744 \ub208\uce58 \ucc44\uace0 \ub9d0\uc558\ub2e4. \uce5c\uc808\uc744 \uac00\uc7a5\ud55c \uadf8\uc758 \uc758\uc2ec\uc744\u2026"} {"question": "1998\ub144 \uc18c\ub2c9 \uc5b4\ub4dc\ubca4\ucc98\uc758 \uc18c\ub2c9\uc744 \uc0c8\ub86d\uac8c \ub514\uc790\uc778\ud55c \uc0ac\ub78c\uc740?", "paragraph": "\uc18c\ub2c9\uc758 \uc815\ud655\ud55c \ub098\uc774, \ubab8\ubb34\uac8c, \ud0a4\uc640 \ub2e4\ub978 \uc2e0\uccb4\uc801 \ud2b9\uc9d5\uc740 \uadf8\uac00 \ub098\uc628 \uacf3\uacfc \uadf8\uacf3\uc758 \uc2a4\ud0c0\uc77c\uc5d0 \ub530\ub77c\uc11c \ub2ec\ub77c\uc9c4\ub2e4. \uc624\uc624\uc2dc\ub9c8 \ub098\uc624\ud1a0\uc5d0 \uc758\ud574\uc11c \ub514\uc790\uc778\uc774 \ub41c \uc6d0\ub798 \ube44\ub514\uc624 \uac8c\uc784\uc5d0\uc11c\ub294, \uc18c\ub2c9\uc740 \uaf64 \uc791\uace0 \uc5b4\ub9b0\uc774 \uac19\uc73c\uba70, \uc9e7\uc740 \uac00\uc2dc\uc640, \ub465\uadfc \ubab8\uc5d0 \ud64d\uccb4\ub294 \ubcf4\uc774\uc9c0 \uc54a\uc558\ub2e4. \uc640\ud0c0\ub098\ubca0 \uc544\ud0a4\ub77c\uc5d0 \uc758\ud574\uc11c \ub514\uc790\uc778 \ub418\uc5c8\uace0 \uadf8\ub824\uc9c4 \uc774 \uc544\ud2b8\uc6cc\ud06c\ub294 \uc18c\ub2c9 \ub354 \ud5e4\uc9c0\ud639\uc5d0 \ub098\uc654\uc73c\uba70, \uadf8 \ub4a4\ub85c \ub098\uc628 \uc18c\ub2c9 \ube44\ub514\uc624 \uac8c\uc784\uc740 \ube44\uc2b7\ud55c \ub514\uc790\uc778\uc744 \ubcf4\uc5ec\uc8fc\uac8c \ub418\uc5c8\ub2e4. 1998\ub144\uc758 \uc18c\ub2c9 \uc5b4\ub4dc\ubca4\ucc98\ubd80\ud130, \uc18c\ub2c9\uc740 \ub098\uce74 \uc720\uc9c0\uc5d0 \uc758\ud574\uc11c \uc0c8\ub85c \ub514\uc790\uc778\uc774 \ub418\uc5b4\uc11c \ub354 \uae34 \ub2e4\ub9ac\uc640 \ub35c \uad6c\ud615\uc778 \ubab8\uc744 \uc9c0\ub2cc \ub354 \ud0a4\uac00 \ud070 \uce90\ub9ad\ud130\ub85c \ubcc0\ud588\ub2e4. \uadf8\uc758 \uac00\uc2dc\ub294 \ub354 \uae38\uc5b4\uc84c\uace0 \uc544\ub798\ub85c \ub0b4\ub824\uac14\uc73c\uba70, \uadf8\uc758 \ub208\uc740 \ub179\uc0c9\uc73c\ub85c \ubcc0\ud588\ub2e4. \uc774 \uc774\ud6c4\ub85c \ub354 \ubcc0\ud55c \uac83\uc740 \uc18c\ub2c9 \ub354 \ud5e4\uc9c0\ud638\uadf8:\ub125\uc2a4\ud2b8 \uc81c\ub108\ub808\uc774\uc158\uc5d0\uc11c\ubd80\ud130 \uc774\ub2e4. \uadf8\uc758 \ucf54\ubc1c\ud2b8\uc0c9 \ubab8\uc5d0 \ub300\ud574\uc11c\ub294 \uc18c\ub2c9 \uc2dc\ub9ac\uc988 \uc5b4\ub514\uc11c\ub3c4 \uc124\uba85\uc774 \ub418\uc5b4\uc788\uc9c0 \uc54a\ub2e4. \ub610\ud55c \uc18c\ub2c9 \ubd90 \uc2dc\ub9ac\uc988\uc5d0\uc11c \ud0a4\uac00 \ub354 \ud06c\uace0 \ubaa9\uc5d0 \uc2a4\uce74\ud504\ub97c \ub9e8 \uce90\ub9ad\ud130\ub85c \ubcc0\ud588\ub2e4.", "answer": "\ub098\uce74 \uc720\uc9c0", "sentence": "\uc18c\ub2c9\uc740 \ub098\uce74 \uc720\uc9c0 \uc5d0 \uc758\ud574\uc11c \uc0c8\ub85c \ub514\uc790\uc778\uc774 \ub418\uc5b4\uc11c \ub354 \uae34 \ub2e4\ub9ac\uc640 \ub35c \uad6c\ud615\uc778 \ubab8\uc744 \uc9c0\ub2cc \ub354 \ud0a4\uac00 \ud070 \uce90\ub9ad\ud130\ub85c \ubcc0\ud588\ub2e4.", "paragraph_sentence": "\uc18c\ub2c9\uc758 \uc815\ud655\ud55c \ub098\uc774, \ubab8\ubb34\uac8c, \ud0a4\uc640 \ub2e4\ub978 \uc2e0\uccb4\uc801 \ud2b9\uc9d5\uc740 \uadf8\uac00 \ub098\uc628 \uacf3\uacfc \uadf8\uacf3\uc758 \uc2a4\ud0c0\uc77c\uc5d0 \ub530\ub77c\uc11c \ub2ec\ub77c\uc9c4\ub2e4. \uc624\uc624\uc2dc\ub9c8 \ub098\uc624\ud1a0\uc5d0 \uc758\ud574\uc11c \ub514\uc790\uc778\uc774 \ub41c \uc6d0\ub798 \ube44\ub514\uc624 \uac8c\uc784\uc5d0\uc11c\ub294, \uc18c\ub2c9\uc740 \uaf64 \uc791\uace0 \uc5b4\ub9b0\uc774 \uac19\uc73c\uba70, \uc9e7\uc740 \uac00\uc2dc\uc640, \ub465\uadfc \ubab8\uc5d0 \ud64d\uccb4\ub294 \ubcf4\uc774\uc9c0 \uc54a\uc558\ub2e4. \uc640\ud0c0\ub098\ubca0 \uc544\ud0a4\ub77c\uc5d0 \uc758\ud574\uc11c \ub514\uc790\uc778 \ub418\uc5c8\uace0 \uadf8\ub824\uc9c4 \uc774 \uc544\ud2b8\uc6cc\ud06c\ub294 \uc18c\ub2c9 \ub354 \ud5e4\uc9c0\ud639\uc5d0 \ub098\uc654\uc73c\uba70, \uadf8 \ub4a4\ub85c \ub098\uc628 \uc18c\ub2c9 \ube44\ub514\uc624 \uac8c\uc784\uc740 \ube44\uc2b7\ud55c \ub514\uc790\uc778\uc744 \ubcf4\uc5ec\uc8fc\uac8c \ub418\uc5c8\ub2e4. 1998\ub144\uc758 \uc18c\ub2c9 \uc5b4\ub4dc\ubca4\ucc98\ubd80\ud130, \uc18c\ub2c9\uc740 \ub098\uce74 \uc720\uc9c0 \uc5d0 \uc758\ud574\uc11c \uc0c8\ub85c \ub514\uc790\uc778\uc774 \ub418\uc5b4\uc11c \ub354 \uae34 \ub2e4\ub9ac\uc640 \ub35c \uad6c\ud615\uc778 \ubab8\uc744 \uc9c0\ub2cc \ub354 \ud0a4\uac00 \ud070 \uce90\ub9ad\ud130\ub85c \ubcc0\ud588\ub2e4. \uadf8\uc758 \uac00\uc2dc\ub294 \ub354 \uae38\uc5b4\uc84c\uace0 \uc544\ub798\ub85c \ub0b4\ub824\uac14\uc73c\uba70, \uadf8\uc758 \ub208\uc740 \ub179\uc0c9\uc73c\ub85c \ubcc0\ud588\ub2e4. \uc774 \uc774\ud6c4\ub85c \ub354 \ubcc0\ud55c \uac83\uc740 \uc18c\ub2c9 \ub354 \ud5e4\uc9c0\ud638\uadf8:\ub125\uc2a4\ud2b8 \uc81c\ub108\ub808\uc774\uc158\uc5d0\uc11c\ubd80\ud130 \uc774\ub2e4. \uadf8\uc758 \ucf54\ubc1c\ud2b8\uc0c9 \ubab8\uc5d0 \ub300\ud574\uc11c\ub294 \uc18c\ub2c9 \uc2dc\ub9ac\uc988 \uc5b4\ub514\uc11c\ub3c4 \uc124\uba85\uc774 \ub418\uc5b4\uc788\uc9c0 \uc54a\ub2e4. \ub610\ud55c \uc18c\ub2c9 \ubd90 \uc2dc\ub9ac\uc988\uc5d0\uc11c \ud0a4\uac00 \ub354 \ud06c\uace0 \ubaa9\uc5d0 \uc2a4\uce74\ud504\ub97c \ub9e8 \uce90\ub9ad\ud130\ub85c \ubcc0\ud588\ub2e4.", "paragraph_answer": "\uc18c\ub2c9\uc758 \uc815\ud655\ud55c \ub098\uc774, \ubab8\ubb34\uac8c, \ud0a4\uc640 \ub2e4\ub978 \uc2e0\uccb4\uc801 \ud2b9\uc9d5\uc740 \uadf8\uac00 \ub098\uc628 \uacf3\uacfc \uadf8\uacf3\uc758 \uc2a4\ud0c0\uc77c\uc5d0 \ub530\ub77c\uc11c \ub2ec\ub77c\uc9c4\ub2e4. \uc624\uc624\uc2dc\ub9c8 \ub098\uc624\ud1a0\uc5d0 \uc758\ud574\uc11c \ub514\uc790\uc778\uc774 \ub41c \uc6d0\ub798 \ube44\ub514\uc624 \uac8c\uc784\uc5d0\uc11c\ub294, \uc18c\ub2c9\uc740 \uaf64 \uc791\uace0 \uc5b4\ub9b0\uc774 \uac19\uc73c\uba70, \uc9e7\uc740 \uac00\uc2dc\uc640, \ub465\uadfc \ubab8\uc5d0 \ud64d\uccb4\ub294 \ubcf4\uc774\uc9c0 \uc54a\uc558\ub2e4. \uc640\ud0c0\ub098\ubca0 \uc544\ud0a4\ub77c\uc5d0 \uc758\ud574\uc11c \ub514\uc790\uc778 \ub418\uc5c8\uace0 \uadf8\ub824\uc9c4 \uc774 \uc544\ud2b8\uc6cc\ud06c\ub294 \uc18c\ub2c9 \ub354 \ud5e4\uc9c0\ud639\uc5d0 \ub098\uc654\uc73c\uba70, \uadf8 \ub4a4\ub85c \ub098\uc628 \uc18c\ub2c9 \ube44\ub514\uc624 \uac8c\uc784\uc740 \ube44\uc2b7\ud55c \ub514\uc790\uc778\uc744 \ubcf4\uc5ec\uc8fc\uac8c \ub418\uc5c8\ub2e4. 1998\ub144\uc758 \uc18c\ub2c9 \uc5b4\ub4dc\ubca4\ucc98\ubd80\ud130, \uc18c\ub2c9\uc740 \ub098\uce74 \uc720\uc9c0 \uc5d0 \uc758\ud574\uc11c \uc0c8\ub85c \ub514\uc790\uc778\uc774 \ub418\uc5b4\uc11c \ub354 \uae34 \ub2e4\ub9ac\uc640 \ub35c \uad6c\ud615\uc778 \ubab8\uc744 \uc9c0\ub2cc \ub354 \ud0a4\uac00 \ud070 \uce90\ub9ad\ud130\ub85c \ubcc0\ud588\ub2e4. \uadf8\uc758 \uac00\uc2dc\ub294 \ub354 \uae38\uc5b4\uc84c\uace0 \uc544\ub798\ub85c \ub0b4\ub824\uac14\uc73c\uba70, \uadf8\uc758 \ub208\uc740 \ub179\uc0c9\uc73c\ub85c \ubcc0\ud588\ub2e4. \uc774 \uc774\ud6c4\ub85c \ub354 \ubcc0\ud55c \uac83\uc740 \uc18c\ub2c9 \ub354 \ud5e4\uc9c0\ud638\uadf8:\ub125\uc2a4\ud2b8 \uc81c\ub108\ub808\uc774\uc158\uc5d0\uc11c\ubd80\ud130 \uc774\ub2e4. \uadf8\uc758 \ucf54\ubc1c\ud2b8\uc0c9 \ubab8\uc5d0 \ub300\ud574\uc11c\ub294 \uc18c\ub2c9 \uc2dc\ub9ac\uc988 \uc5b4\ub514\uc11c\ub3c4 \uc124\uba85\uc774 \ub418\uc5b4\uc788\uc9c0 \uc54a\ub2e4. \ub610\ud55c \uc18c\ub2c9 \ubd90 \uc2dc\ub9ac\uc988\uc5d0\uc11c \ud0a4\uac00 \ub354 \ud06c\uace0 \ubaa9\uc5d0 \uc2a4\uce74\ud504\ub97c \ub9e8 \uce90\ub9ad\ud130\ub85c \ubcc0\ud588\ub2e4.", "sentence_answer": "\uc18c\ub2c9\uc740 \ub098\uce74 \uc720\uc9c0 \uc5d0 \uc758\ud574\uc11c \uc0c8\ub85c \ub514\uc790\uc778\uc774 \ub418\uc5b4\uc11c \ub354 \uae34 \ub2e4\ub9ac\uc640 \ub35c \uad6c\ud615\uc778 \ubab8\uc744 \uc9c0\ub2cc \ub354 \ud0a4\uac00 \ud070 \uce90\ub9ad\ud130\ub85c \ubcc0\ud588\ub2e4.", "paragraph_id": "6584154-1-0", "paragraph_question": "question: 1998\ub144 \uc18c\ub2c9 \uc5b4\ub4dc\ubca4\ucc98\uc758 \uc18c\ub2c9\uc744 \uc0c8\ub86d\uac8c \ub514\uc790\uc778\ud55c \uc0ac\ub78c\uc740?, context: \uc18c\ub2c9\uc758 \uc815\ud655\ud55c \ub098\uc774, \ubab8\ubb34\uac8c, \ud0a4\uc640 \ub2e4\ub978 \uc2e0\uccb4\uc801 \ud2b9\uc9d5\uc740 \uadf8\uac00 \ub098\uc628 \uacf3\uacfc \uadf8\uacf3\uc758 \uc2a4\ud0c0\uc77c\uc5d0 \ub530\ub77c\uc11c \ub2ec\ub77c\uc9c4\ub2e4. \uc624\uc624\uc2dc\ub9c8 \ub098\uc624\ud1a0\uc5d0 \uc758\ud574\uc11c \ub514\uc790\uc778\uc774 \ub41c \uc6d0\ub798 \ube44\ub514\uc624 \uac8c\uc784\uc5d0\uc11c\ub294, \uc18c\ub2c9\uc740 \uaf64 \uc791\uace0 \uc5b4\ub9b0\uc774 \uac19\uc73c\uba70, \uc9e7\uc740 \uac00\uc2dc\uc640, \ub465\uadfc \ubab8\uc5d0 \ud64d\uccb4\ub294 \ubcf4\uc774\uc9c0 \uc54a\uc558\ub2e4. \uc640\ud0c0\ub098\ubca0 \uc544\ud0a4\ub77c\uc5d0 \uc758\ud574\uc11c \ub514\uc790\uc778 \ub418\uc5c8\uace0 \uadf8\ub824\uc9c4 \uc774 \uc544\ud2b8\uc6cc\ud06c\ub294 \uc18c\ub2c9 \ub354 \ud5e4\uc9c0\ud639\uc5d0 \ub098\uc654\uc73c\uba70, \uadf8 \ub4a4\ub85c \ub098\uc628 \uc18c\ub2c9 \ube44\ub514\uc624 \uac8c\uc784\uc740 \ube44\uc2b7\ud55c \ub514\uc790\uc778\uc744 \ubcf4\uc5ec\uc8fc\uac8c \ub418\uc5c8\ub2e4. 1998\ub144\uc758 \uc18c\ub2c9 \uc5b4\ub4dc\ubca4\ucc98\ubd80\ud130, \uc18c\ub2c9\uc740 \ub098\uce74 \uc720\uc9c0\uc5d0 \uc758\ud574\uc11c \uc0c8\ub85c \ub514\uc790\uc778\uc774 \ub418\uc5b4\uc11c \ub354 \uae34 \ub2e4\ub9ac\uc640 \ub35c \uad6c\ud615\uc778 \ubab8\uc744 \uc9c0\ub2cc \ub354 \ud0a4\uac00 \ud070 \uce90\ub9ad\ud130\ub85c \ubcc0\ud588\ub2e4. \uadf8\uc758 \uac00\uc2dc\ub294 \ub354 \uae38\uc5b4\uc84c\uace0 \uc544\ub798\ub85c \ub0b4\ub824\uac14\uc73c\uba70, \uadf8\uc758 \ub208\uc740 \ub179\uc0c9\uc73c\ub85c \ubcc0\ud588\ub2e4. \uc774 \uc774\ud6c4\ub85c \ub354 \ubcc0\ud55c \uac83\uc740 \uc18c\ub2c9 \ub354 \ud5e4\uc9c0\ud638\uadf8:\ub125\uc2a4\ud2b8 \uc81c\ub108\ub808\uc774\uc158\uc5d0\uc11c\ubd80\ud130 \uc774\ub2e4. \uadf8\uc758 \ucf54\ubc1c\ud2b8\uc0c9 \ubab8\uc5d0 \ub300\ud574\uc11c\ub294 \uc18c\ub2c9 \uc2dc\ub9ac\uc988 \uc5b4\ub514\uc11c\ub3c4 \uc124\uba85\uc774 \ub418\uc5b4\uc788\uc9c0 \uc54a\ub2e4. \ub610\ud55c \uc18c\ub2c9 \ubd90 \uc2dc\ub9ac\uc988\uc5d0\uc11c \ud0a4\uac00 \ub354 \ud06c\uace0 \ubaa9\uc5d0 \uc2a4\uce74\ud504\ub97c \ub9e8 \uce90\ub9ad\ud130\ub85c \ubcc0\ud588\ub2e4."} {"question": "\ub808\uc624\ub098\ub974\ub3c4\ub294 \uc624\ube44\ub514\uc6b0\uc2a4 \uc608\ucc2c\ub860\uc790\ub85c \ubb34\uc5c7\uc744 \uc5f4\uc2ec\ud788 \uc77d\uc5c8\ub294\uac00?", "paragraph": "\ud6c4\uacbd\uc744 \uc798 \uad00\ucc30\ud574\ubcf4\uba74 \ube44\ub17c\ub9ac\uc801\uc778 \uc810\uc774 \ubcf4\uc778\ub2e4 \uad00\ub78c\uc790\uc758 \uc2dc\uc810\uc5d0\uc11c \ubcfc \ub54c \uc624\ub978\ucabd\uc5d0\ub294 \ub192\uc740 \uc0b0\uacfc \uac70\uc6b8\ucc98\ub7fc \ud3c9\ud3c9\ud55c \uc218\ud3c9\uc120\uc774 \uc67c\ucabd\uc5d0 \ube44\ud574 \ub2e4\uc18c \uc62c\ub77c\uc640\uc788\ub294 \ud638\uc218\uac00 \ubcf4\uc778\ub2e4. \ubc18\uba74\uc5d0 \uc67c\ucabd \ud48d\uacbd\uc740 \uc624\ub978\ucabd\ubcf4\ub2e4 \ub0ae\ub2e4 \ub2e4\uc2dc \ub9d0\ud574 \ub450 \ubd80\ubd84\uc774 \ud558\ub098\ub85c \uc774\uc5b4\uc9c4 \uac83\uc774\ub77c\uace0 \ubcfc \uc218 \uc5c6\ub2e4. \uc0ac\uc2e4\uc0c1 \uac70\uae30\uc5d0 \uc778\ubb3c \uc790\uccb4\uc640 \ubaa8\ub098\ub9ac\uc790\uc758 \ubbf8\uc18c\uc5d0 \uc758\ud574 \uac10\ucdb0\uc9c0\uace0 \ubcc0\ud615\ub41c \uac04\uaca9\uc778 \ub2e8\uc808\uc774 \uc788\ub2e4 \ubaa8\ub098\ub9ac\uc790\uc758 \uc785\uc220\uc740 \uc624\ub978\ucabd\uc73c\ub85c \uc57d\uac04 \uc62c\ub77c\uac00 \uc788\ub2e4. (\uc880 \ub354 \ub192\uac8c \uadf8\ub824\uc9c4 \uc624\ub978\ucabd \ud48d\uacbd \ucabd\uc73c\ub85c) \uc591\ucabd \ub450 \ud48d\uacbd \uc0ac\uc774\uc758 \ubd88\uac00\ub2a5\ud55c \uc774\ud589\uc774 \uc778\ubb3c \uc18d\uc5d0\uc11c \ubc14\ub85c \uc778\ubb3c\uc758 \ubbf8\uc18c\ub97c \ud1b5\ud574 \uc2e4\ud604\ub418\uc5c8\ub2e4. \ub808\uc624\ub098\ub974\ub3c4\ub294 \uc624\ube44\ub514\uc6b0\uc2a4 \uc608\ucc2c\ub860\uc790\ub85c <\ubcc0\uc2e0 \uc774\uc57c\uae30>\ub97c \uc5f4\uc2ec\ud788 \uc77d\uc5c8\ub2e4. \ub808\uc624\ub098\ub974\ub3c4\ub294 \uc624\ube44\ub514\uc6b0\uc2a4\uc640 \ub9c8\ucc2c\uac00\uc9c0\ub85c \ucd5c\uace0\uc758 \ubbf8\ub780 \ubc14\ub85c \ub367\uc5c6\uc74c, \uc989 \uc2dc\uac04\uc758 \ubb34\uc0c1\ud568\uc774\ub77c\uace0 \uc0dd\uac01\ud588\ub2e4. <\ubcc0\uc2e0 \uc774\uc57c\uae30>\uc5d0 \ub098\uc624\ub294 \ud5ec\ub808\ub098\ub294 \u2018\uc81c \ubaa8\uc2b5\uc774 \uc624\ub298\uc740 \uc544\ub984\ub2f5\uc9c0\ub9cc \uc2dc\uac04\uc774 \ud750\ub974\uba74 \uacfc\uc5f0 \uc5b4\ub5bb\uac8c \ubcc0\ud560\uae4c\uc694?\u2019\ub77c\ub294 \ub9d0\uc744 \ud55c\ub2e4 \uc774\ub294 \uc2dc\uac04\uc758 \ubb34\uc0c1\ud568\uc744 \ubcf4\uc5ec\uc900\ub2e4. \ub808\uc5b4\ub098\ub974\ub3c4\ub294 <\ubaa8\ub098\ub9ac\uc790>\uc5d0 \ubc14\ub85c \uc774\ub7f0 \uc6b0\uc8fc\uc801 \uc0ac\uc0c9\uacfc \ud1b5\ucc30\uc744 \ud22c\uc601\ud55c \uac83\uc774\ub2e4. \ubc14\ub85c \ubbf8\uc18c\uc758 \ub355\uc774\uae30 \ub54c\ubb38\uc774\ub2e4. \ubbf8\uc18c\ub780 \ub367\uc5c6\ub294 \uac83\uc73c\ub85c \uc9e7\uac8c \uc2a4\uce58\ub294 \uc791\uc740 \uc6c3\uc74c\uc73c\ub85c \uc5bc\uad74\uc5d0 \uc7a0\uc2dc \uba38\ubb3c \ubfd0\uc774\ub2e4. \ub610\ud55c \ud398\ub4dc\ub808\ud2f0\uc5d0 \ub530\ub974\uba74 \ud48d\uacbd\uc5d0\uc11c\uc758 \ub2e4\ub9ac\ub294 \uc2dc\uac04\uc758 \ud750\ub984\uc744 \uc0c1\uc9d5\ud55c\ub2e4 \ub2e4\ub9ac\uac00 \uc788\ub2e4\ub294 \uac83\uc740 \uac15\ubb3c\uc774 \uc788\ub2e4\ub294 \uac83\uc778\ub370 \uac15\ubb3c\uc740 \ub367\uc5c6\uc774 \ud750\ub974\ub294 \uc2dc\uac04\uc758 \uc0c1\uc9d5\uc774\ub2e4 \uc774\uc5d0 \ubaa8\ub098\ub9ac\uc790\uc758 \ud575\uc2ec \uc8fc\uc81c\ub294 \uc2dc\uac04\uc774\ub77c\uace0 \ubcfc \uc218 \uc788\ub2e4.", "answer": "\ubcc0\uc2e0 \uc774\uc57c\uae30", "sentence": "\ub808\uc624\ub098\ub974\ub3c4\ub294 \uc624\ube44\ub514\uc6b0\uc2a4 \uc608\ucc2c\ub860\uc790\ub85c < \ubcc0\uc2e0 \uc774\uc57c\uae30 >", "paragraph_sentence": "\ud6c4\uacbd\uc744 \uc798 \uad00\ucc30\ud574\ubcf4\uba74 \ube44\ub17c\ub9ac\uc801\uc778 \uc810\uc774 \ubcf4\uc778\ub2e4 \uad00\ub78c\uc790\uc758 \uc2dc\uc810\uc5d0\uc11c \ubcfc \ub54c \uc624\ub978\ucabd\uc5d0\ub294 \ub192\uc740 \uc0b0\uacfc \uac70\uc6b8\ucc98\ub7fc \ud3c9\ud3c9\ud55c \uc218\ud3c9\uc120\uc774 \uc67c\ucabd\uc5d0 \ube44\ud574 \ub2e4\uc18c \uc62c\ub77c\uc640\uc788\ub294 \ud638\uc218\uac00 \ubcf4\uc778\ub2e4. \ubc18\uba74\uc5d0 \uc67c\ucabd \ud48d\uacbd\uc740 \uc624\ub978\ucabd\ubcf4\ub2e4 \ub0ae\ub2e4 \ub2e4\uc2dc \ub9d0\ud574 \ub450 \ubd80\ubd84\uc774 \ud558\ub098\ub85c \uc774\uc5b4\uc9c4 \uac83\uc774\ub77c\uace0 \ubcfc \uc218 \uc5c6\ub2e4. \uc0ac\uc2e4\uc0c1 \uac70\uae30\uc5d0 \uc778\ubb3c \uc790\uccb4\uc640 \ubaa8\ub098\ub9ac\uc790\uc758 \ubbf8\uc18c\uc5d0 \uc758\ud574 \uac10\ucdb0\uc9c0\uace0 \ubcc0\ud615\ub41c \uac04\uaca9\uc778 \ub2e8\uc808\uc774 \uc788\ub2e4 \ubaa8\ub098\ub9ac\uc790\uc758 \uc785\uc220\uc740 \uc624\ub978\ucabd\uc73c\ub85c \uc57d\uac04 \uc62c\ub77c\uac00 \uc788\ub2e4. (\uc880 \ub354 \ub192\uac8c \uadf8\ub824\uc9c4 \uc624\ub978\ucabd \ud48d\uacbd \ucabd\uc73c\ub85c) \uc591\ucabd \ub450 \ud48d\uacbd \uc0ac\uc774\uc758 \ubd88\uac00\ub2a5\ud55c \uc774\ud589\uc774 \uc778\ubb3c \uc18d\uc5d0\uc11c \ubc14\ub85c \uc778\ubb3c\uc758 \ubbf8\uc18c\ub97c \ud1b5\ud574 \uc2e4\ud604\ub418\uc5c8\ub2e4. \ub808\uc624\ub098\ub974\ub3c4\ub294 \uc624\ube44\ub514\uc6b0\uc2a4 \uc608\ucc2c\ub860\uc790\ub85c < \ubcc0\uc2e0 \uc774\uc57c\uae30 > \ub97c \uc5f4\uc2ec\ud788 \uc77d\uc5c8\ub2e4. \ub808\uc624\ub098\ub974\ub3c4\ub294 \uc624\ube44\ub514\uc6b0\uc2a4\uc640 \ub9c8\ucc2c\uac00\uc9c0\ub85c \ucd5c\uace0\uc758 \ubbf8\ub780 \ubc14\ub85c \ub367\uc5c6\uc74c, \uc989 \uc2dc\uac04\uc758 \ubb34\uc0c1\ud568\uc774\ub77c\uace0 \uc0dd\uac01\ud588\ub2e4. <\ubcc0\uc2e0 \uc774\uc57c\uae30>\uc5d0 \ub098\uc624\ub294 \ud5ec\ub808\ub098\ub294 \u2018\uc81c \ubaa8\uc2b5\uc774 \uc624\ub298\uc740 \uc544\ub984\ub2f5\uc9c0\ub9cc \uc2dc\uac04\uc774 \ud750\ub974\uba74 \uacfc\uc5f0 \uc5b4\ub5bb\uac8c \ubcc0\ud560\uae4c\uc694?\u2019\ub77c\ub294 \ub9d0\uc744 \ud55c\ub2e4 \uc774\ub294 \uc2dc\uac04\uc758 \ubb34\uc0c1\ud568\uc744 \ubcf4\uc5ec\uc900\ub2e4. \ub808\uc5b4\ub098\ub974\ub3c4\ub294 <\ubaa8\ub098\ub9ac\uc790>\uc5d0 \ubc14\ub85c \uc774\ub7f0 \uc6b0\uc8fc\uc801 \uc0ac\uc0c9\uacfc \ud1b5\ucc30\uc744 \ud22c\uc601\ud55c \uac83\uc774\ub2e4. \ubc14\ub85c \ubbf8\uc18c\uc758 \ub355\uc774\uae30 \ub54c\ubb38\uc774\ub2e4. \ubbf8\uc18c\ub780 \ub367\uc5c6\ub294 \uac83\uc73c\ub85c \uc9e7\uac8c \uc2a4\uce58\ub294 \uc791\uc740 \uc6c3\uc74c\uc73c\ub85c \uc5bc\uad74\uc5d0 \uc7a0\uc2dc \uba38\ubb3c \ubfd0\uc774\ub2e4. \ub610\ud55c \ud398\ub4dc\ub808\ud2f0\uc5d0 \ub530\ub974\uba74 \ud48d\uacbd\uc5d0\uc11c\uc758 \ub2e4\ub9ac\ub294 \uc2dc\uac04\uc758 \ud750\ub984\uc744 \uc0c1\uc9d5\ud55c\ub2e4 \ub2e4\ub9ac\uac00 \uc788\ub2e4\ub294 \uac83\uc740 \uac15\ubb3c\uc774 \uc788\ub2e4\ub294 \uac83\uc778\ub370 \uac15\ubb3c\uc740 \ub367\uc5c6\uc774 \ud750\ub974\ub294 \uc2dc\uac04\uc758 \uc0c1\uc9d5\uc774\ub2e4 \uc774\uc5d0 \ubaa8\ub098\ub9ac\uc790\uc758 \ud575\uc2ec \uc8fc\uc81c\ub294 \uc2dc\uac04\uc774\ub77c\uace0 \ubcfc \uc218 \uc788\ub2e4.", "paragraph_answer": "\ud6c4\uacbd\uc744 \uc798 \uad00\ucc30\ud574\ubcf4\uba74 \ube44\ub17c\ub9ac\uc801\uc778 \uc810\uc774 \ubcf4\uc778\ub2e4 \uad00\ub78c\uc790\uc758 \uc2dc\uc810\uc5d0\uc11c \ubcfc \ub54c \uc624\ub978\ucabd\uc5d0\ub294 \ub192\uc740 \uc0b0\uacfc \uac70\uc6b8\ucc98\ub7fc \ud3c9\ud3c9\ud55c \uc218\ud3c9\uc120\uc774 \uc67c\ucabd\uc5d0 \ube44\ud574 \ub2e4\uc18c \uc62c\ub77c\uc640\uc788\ub294 \ud638\uc218\uac00 \ubcf4\uc778\ub2e4. \ubc18\uba74\uc5d0 \uc67c\ucabd \ud48d\uacbd\uc740 \uc624\ub978\ucabd\ubcf4\ub2e4 \ub0ae\ub2e4 \ub2e4\uc2dc \ub9d0\ud574 \ub450 \ubd80\ubd84\uc774 \ud558\ub098\ub85c \uc774\uc5b4\uc9c4 \uac83\uc774\ub77c\uace0 \ubcfc \uc218 \uc5c6\ub2e4. \uc0ac\uc2e4\uc0c1 \uac70\uae30\uc5d0 \uc778\ubb3c \uc790\uccb4\uc640 \ubaa8\ub098\ub9ac\uc790\uc758 \ubbf8\uc18c\uc5d0 \uc758\ud574 \uac10\ucdb0\uc9c0\uace0 \ubcc0\ud615\ub41c \uac04\uaca9\uc778 \ub2e8\uc808\uc774 \uc788\ub2e4 \ubaa8\ub098\ub9ac\uc790\uc758 \uc785\uc220\uc740 \uc624\ub978\ucabd\uc73c\ub85c \uc57d\uac04 \uc62c\ub77c\uac00 \uc788\ub2e4. (\uc880 \ub354 \ub192\uac8c \uadf8\ub824\uc9c4 \uc624\ub978\ucabd \ud48d\uacbd \ucabd\uc73c\ub85c) \uc591\ucabd \ub450 \ud48d\uacbd \uc0ac\uc774\uc758 \ubd88\uac00\ub2a5\ud55c \uc774\ud589\uc774 \uc778\ubb3c \uc18d\uc5d0\uc11c \ubc14\ub85c \uc778\ubb3c\uc758 \ubbf8\uc18c\ub97c \ud1b5\ud574 \uc2e4\ud604\ub418\uc5c8\ub2e4. \ub808\uc624\ub098\ub974\ub3c4\ub294 \uc624\ube44\ub514\uc6b0\uc2a4 \uc608\ucc2c\ub860\uc790\ub85c < \ubcc0\uc2e0 \uc774\uc57c\uae30 >\ub97c \uc5f4\uc2ec\ud788 \uc77d\uc5c8\ub2e4. \ub808\uc624\ub098\ub974\ub3c4\ub294 \uc624\ube44\ub514\uc6b0\uc2a4\uc640 \ub9c8\ucc2c\uac00\uc9c0\ub85c \ucd5c\uace0\uc758 \ubbf8\ub780 \ubc14\ub85c \ub367\uc5c6\uc74c, \uc989 \uc2dc\uac04\uc758 \ubb34\uc0c1\ud568\uc774\ub77c\uace0 \uc0dd\uac01\ud588\ub2e4. <\ubcc0\uc2e0 \uc774\uc57c\uae30>\uc5d0 \ub098\uc624\ub294 \ud5ec\ub808\ub098\ub294 \u2018\uc81c \ubaa8\uc2b5\uc774 \uc624\ub298\uc740 \uc544\ub984\ub2f5\uc9c0\ub9cc \uc2dc\uac04\uc774 \ud750\ub974\uba74 \uacfc\uc5f0 \uc5b4\ub5bb\uac8c \ubcc0\ud560\uae4c\uc694?\u2019\ub77c\ub294 \ub9d0\uc744 \ud55c\ub2e4 \uc774\ub294 \uc2dc\uac04\uc758 \ubb34\uc0c1\ud568\uc744 \ubcf4\uc5ec\uc900\ub2e4. \ub808\uc5b4\ub098\ub974\ub3c4\ub294 <\ubaa8\ub098\ub9ac\uc790>\uc5d0 \ubc14\ub85c \uc774\ub7f0 \uc6b0\uc8fc\uc801 \uc0ac\uc0c9\uacfc \ud1b5\ucc30\uc744 \ud22c\uc601\ud55c \uac83\uc774\ub2e4. \ubc14\ub85c \ubbf8\uc18c\uc758 \ub355\uc774\uae30 \ub54c\ubb38\uc774\ub2e4. \ubbf8\uc18c\ub780 \ub367\uc5c6\ub294 \uac83\uc73c\ub85c \uc9e7\uac8c \uc2a4\uce58\ub294 \uc791\uc740 \uc6c3\uc74c\uc73c\ub85c \uc5bc\uad74\uc5d0 \uc7a0\uc2dc \uba38\ubb3c \ubfd0\uc774\ub2e4. \ub610\ud55c \ud398\ub4dc\ub808\ud2f0\uc5d0 \ub530\ub974\uba74 \ud48d\uacbd\uc5d0\uc11c\uc758 \ub2e4\ub9ac\ub294 \uc2dc\uac04\uc758 \ud750\ub984\uc744 \uc0c1\uc9d5\ud55c\ub2e4 \ub2e4\ub9ac\uac00 \uc788\ub2e4\ub294 \uac83\uc740 \uac15\ubb3c\uc774 \uc788\ub2e4\ub294 \uac83\uc778\ub370 \uac15\ubb3c\uc740 \ub367\uc5c6\uc774 \ud750\ub974\ub294 \uc2dc\uac04\uc758 \uc0c1\uc9d5\uc774\ub2e4 \uc774\uc5d0 \ubaa8\ub098\ub9ac\uc790\uc758 \ud575\uc2ec \uc8fc\uc81c\ub294 \uc2dc\uac04\uc774\ub77c\uace0 \ubcfc \uc218 \uc788\ub2e4.", "sentence_answer": "\ub808\uc624\ub098\ub974\ub3c4\ub294 \uc624\ube44\ub514\uc6b0\uc2a4 \uc608\ucc2c\ub860\uc790\ub85c < \ubcc0\uc2e0 \uc774\uc57c\uae30 >", "paragraph_id": "6546809-3-0", "paragraph_question": "question: \ub808\uc624\ub098\ub974\ub3c4\ub294 \uc624\ube44\ub514\uc6b0\uc2a4 \uc608\ucc2c\ub860\uc790\ub85c \ubb34\uc5c7\uc744 \uc5f4\uc2ec\ud788 \uc77d\uc5c8\ub294\uac00?, context: \ud6c4\uacbd\uc744 \uc798 \uad00\ucc30\ud574\ubcf4\uba74 \ube44\ub17c\ub9ac\uc801\uc778 \uc810\uc774 \ubcf4\uc778\ub2e4 \uad00\ub78c\uc790\uc758 \uc2dc\uc810\uc5d0\uc11c \ubcfc \ub54c \uc624\ub978\ucabd\uc5d0\ub294 \ub192\uc740 \uc0b0\uacfc \uac70\uc6b8\ucc98\ub7fc \ud3c9\ud3c9\ud55c \uc218\ud3c9\uc120\uc774 \uc67c\ucabd\uc5d0 \ube44\ud574 \ub2e4\uc18c \uc62c\ub77c\uc640\uc788\ub294 \ud638\uc218\uac00 \ubcf4\uc778\ub2e4. \ubc18\uba74\uc5d0 \uc67c\ucabd \ud48d\uacbd\uc740 \uc624\ub978\ucabd\ubcf4\ub2e4 \ub0ae\ub2e4 \ub2e4\uc2dc \ub9d0\ud574 \ub450 \ubd80\ubd84\uc774 \ud558\ub098\ub85c \uc774\uc5b4\uc9c4 \uac83\uc774\ub77c\uace0 \ubcfc \uc218 \uc5c6\ub2e4. \uc0ac\uc2e4\uc0c1 \uac70\uae30\uc5d0 \uc778\ubb3c \uc790\uccb4\uc640 \ubaa8\ub098\ub9ac\uc790\uc758 \ubbf8\uc18c\uc5d0 \uc758\ud574 \uac10\ucdb0\uc9c0\uace0 \ubcc0\ud615\ub41c \uac04\uaca9\uc778 \ub2e8\uc808\uc774 \uc788\ub2e4 \ubaa8\ub098\ub9ac\uc790\uc758 \uc785\uc220\uc740 \uc624\ub978\ucabd\uc73c\ub85c \uc57d\uac04 \uc62c\ub77c\uac00 \uc788\ub2e4. (\uc880 \ub354 \ub192\uac8c \uadf8\ub824\uc9c4 \uc624\ub978\ucabd \ud48d\uacbd \ucabd\uc73c\ub85c) \uc591\ucabd \ub450 \ud48d\uacbd \uc0ac\uc774\uc758 \ubd88\uac00\ub2a5\ud55c \uc774\ud589\uc774 \uc778\ubb3c \uc18d\uc5d0\uc11c \ubc14\ub85c \uc778\ubb3c\uc758 \ubbf8\uc18c\ub97c \ud1b5\ud574 \uc2e4\ud604\ub418\uc5c8\ub2e4. \ub808\uc624\ub098\ub974\ub3c4\ub294 \uc624\ube44\ub514\uc6b0\uc2a4 \uc608\ucc2c\ub860\uc790\ub85c <\ubcc0\uc2e0 \uc774\uc57c\uae30>\ub97c \uc5f4\uc2ec\ud788 \uc77d\uc5c8\ub2e4. \ub808\uc624\ub098\ub974\ub3c4\ub294 \uc624\ube44\ub514\uc6b0\uc2a4\uc640 \ub9c8\ucc2c\uac00\uc9c0\ub85c \ucd5c\uace0\uc758 \ubbf8\ub780 \ubc14\ub85c \ub367\uc5c6\uc74c, \uc989 \uc2dc\uac04\uc758 \ubb34\uc0c1\ud568\uc774\ub77c\uace0 \uc0dd\uac01\ud588\ub2e4. <\ubcc0\uc2e0 \uc774\uc57c\uae30>\uc5d0 \ub098\uc624\ub294 \ud5ec\ub808\ub098\ub294 \u2018\uc81c \ubaa8\uc2b5\uc774 \uc624\ub298\uc740 \uc544\ub984\ub2f5\uc9c0\ub9cc \uc2dc\uac04\uc774 \ud750\ub974\uba74 \uacfc\uc5f0 \uc5b4\ub5bb\uac8c \ubcc0\ud560\uae4c\uc694?\u2019\ub77c\ub294 \ub9d0\uc744 \ud55c\ub2e4 \uc774\ub294 \uc2dc\uac04\uc758 \ubb34\uc0c1\ud568\uc744 \ubcf4\uc5ec\uc900\ub2e4. \ub808\uc5b4\ub098\ub974\ub3c4\ub294 <\ubaa8\ub098\ub9ac\uc790>\uc5d0 \ubc14\ub85c \uc774\ub7f0 \uc6b0\uc8fc\uc801 \uc0ac\uc0c9\uacfc \ud1b5\ucc30\uc744 \ud22c\uc601\ud55c \uac83\uc774\ub2e4. \ubc14\ub85c \ubbf8\uc18c\uc758 \ub355\uc774\uae30 \ub54c\ubb38\uc774\ub2e4. \ubbf8\uc18c\ub780 \ub367\uc5c6\ub294 \uac83\uc73c\ub85c \uc9e7\uac8c \uc2a4\uce58\ub294 \uc791\uc740 \uc6c3\uc74c\uc73c\ub85c \uc5bc\uad74\uc5d0 \uc7a0\uc2dc \uba38\ubb3c \ubfd0\uc774\ub2e4. \ub610\ud55c \ud398\ub4dc\ub808\ud2f0\uc5d0 \ub530\ub974\uba74 \ud48d\uacbd\uc5d0\uc11c\uc758 \ub2e4\ub9ac\ub294 \uc2dc\uac04\uc758 \ud750\ub984\uc744 \uc0c1\uc9d5\ud55c\ub2e4 \ub2e4\ub9ac\uac00 \uc788\ub2e4\ub294 \uac83\uc740 \uac15\ubb3c\uc774 \uc788\ub2e4\ub294 \uac83\uc778\ub370 \uac15\ubb3c\uc740 \ub367\uc5c6\uc774 \ud750\ub974\ub294 \uc2dc\uac04\uc758 \uc0c1\uc9d5\uc774\ub2e4 \uc774\uc5d0 \ubaa8\ub098\ub9ac\uc790\uc758 \ud575\uc2ec \uc8fc\uc81c\ub294 \uc2dc\uac04\uc774\ub77c\uace0 \ubcfc \uc218 \uc788\ub2e4."} {"question": "\ud53c\ud574\uc790\uac00 \uc8fc\ubcd1\uc9c4\uc73c\ub85c\ubd80\ud130 \ubc1b\uc740 \ud569\uc758\uae00\uc744 \ub098\ub220\uc900 \uc774\ub4e4\uc740?", "paragraph": "\uadf8\ub7ec\ub098 \uc8fc\ubcd1\uc9c4\uc740 \ud56d\uc18c\ud588\uace0 2001\ub144 11\uc6d4 28\uc77c\uc5d0 \ubb34\uc8c4 \ud310\uacb0\uc774 \ub0b4\ub824\uc84c\ub2e4. \uc7ac\ud310\ubd80\ub294 \u201c\ud53c\ud574\uc790\uac00 \uc8fc\ubcd1\uc9c4\uc73c\ub85c\ubd80\ud130 \ubc1b\uc740 \ud569\uc758\uae08\uc744 \uc99d\uc778\ub4e4\uc5d0\uac8c \ub098\ub220\uc900 \uacbd\uc704 \ub4f1\uc73c\ub85c \ubcfc \ub54c \ud53c\ud574\uc790\uc640 \uc99d\uc778 \ub4f1\uc758 \uc9c4\uc220\uc744 \ubbff\uae30 \uc5b4\ub835\uace0 \ud53c\ud574\uc790 \ud5c8\ubc85\uc9c0\uc758 \uc0c1\ucc98\ub3c4 \uc0ac\uac74 \ub2f9\uc2dc \uc785\uc740 \uac83\uc73c\ub85c \ubcf4\uae30 \uc5b4\ub835\ub2e4\u201d\uace0 \ub9d0\ud588\ub2e4. \uc8fc\ubcd1\uc9c4\uacfc \ub3d9\ub8cc\ub4e4\uc740 \uc5ec\ub300\uc0dd\uc774 \ud559\uad50\uc5d0\uc11c \uc81c\uc801\uc744 \ub2f9\ud588\uc73c\uba70 \ub8f8\uc0b4\ub871 \uc9c1\uc6d0\uc774\ub77c\ub294 \uc0ac\uc2e4\ub3c4 \ubc1d\ud600\ub0c8\ub2e4. \ub610\ud55c \uc5ec\ub300\uc0dd\uc758 \ub3d9\uc0dd\uc774 \ub611\uac19\uc740 \ubc29\ubc95\uc73c\ub85c \ub8f8\uc0b4\ub871 \uc0ac\uc7a5\uc5d0\uac8c \uc131\ud3ed\ud589 \ub204\uba85\uc744 \uc50c\uc6b0\ub824 \ud55c \uc0ac\uc2e4\ub3c4 \ub4dc\ub7ec\ub0ac\ub2e4. \ub610\ud55c \uc5ec\ub300\uc0dd\uc774 \uce5c\uad6c\uc5d0\uac8c \ub3c8\uc744 \uc8fc\uba70 \uc790\uc2e0\uc744 \ub54c\ub824 \uc0c1\ud574\ub97c \uc785\ud788\ub3c4\ub85d \ud55c\uac83\ub3c4 \ub4dc\ub7ec\ub0ac\uace0 \uc774\ub7ec\ud55c \uc99d\uac70\ubb3c\ub4e4\uc774 \uc778\uc815\ub41c \uac83\uc774\ub2e4. \uadf8\ub9ac\uace0 \uc774\ub4ec\ud574\uc778 2002\ub144 7\uc6d4 12\uc77c \ubb34\uc8c4 \ud655\uc815 \ud310\uacb0\uc744 \ubc1b\uc558\ub2e4.", "answer": "\uc99d\uc778", "sentence": "\uc7ac\ud310\ubd80\ub294 \u201c\ud53c\ud574\uc790\uac00 \uc8fc\ubcd1\uc9c4\uc73c\ub85c\ubd80\ud130 \ubc1b\uc740 \ud569\uc758\uae08\uc744 \uc99d\uc778 \ub4e4\uc5d0\uac8c \ub098\ub220\uc900 \uacbd\uc704 \ub4f1\uc73c\ub85c \ubcfc \ub54c \ud53c\ud574\uc790\uc640 \uc99d\uc778 \ub4f1\uc758 \uc9c4\uc220\uc744 \ubbff\uae30 \uc5b4\ub835\uace0 \ud53c\ud574\uc790 \ud5c8\ubc85\uc9c0\uc758 \uc0c1\ucc98\ub3c4 \uc0ac\uac74 \ub2f9\uc2dc \uc785\uc740 \uac83\uc73c\ub85c \ubcf4\uae30 \uc5b4\ub835\ub2e4\u201d\uace0 \ub9d0\ud588\ub2e4.", "paragraph_sentence": "\uadf8\ub7ec\ub098 \uc8fc\ubcd1\uc9c4\uc740 \ud56d\uc18c\ud588\uace0 2001\ub144 11\uc6d4 28\uc77c\uc5d0 \ubb34\uc8c4 \ud310\uacb0\uc774 \ub0b4\ub824\uc84c\ub2e4. \uc7ac\ud310\ubd80\ub294 \u201c\ud53c\ud574\uc790\uac00 \uc8fc\ubcd1\uc9c4\uc73c\ub85c\ubd80\ud130 \ubc1b\uc740 \ud569\uc758\uae08\uc744 \uc99d\uc778 \ub4e4\uc5d0\uac8c \ub098\ub220\uc900 \uacbd\uc704 \ub4f1\uc73c\ub85c \ubcfc \ub54c \ud53c\ud574\uc790\uc640 \uc99d\uc778 \ub4f1\uc758 \uc9c4\uc220\uc744 \ubbff\uae30 \uc5b4\ub835\uace0 \ud53c\ud574\uc790 \ud5c8\ubc85\uc9c0\uc758 \uc0c1\ucc98\ub3c4 \uc0ac\uac74 \ub2f9\uc2dc \uc785\uc740 \uac83\uc73c\ub85c \ubcf4\uae30 \uc5b4\ub835\ub2e4\u201d\uace0 \ub9d0\ud588\ub2e4. \uc8fc\ubcd1\uc9c4\uacfc \ub3d9\ub8cc\ub4e4\uc740 \uc5ec\ub300\uc0dd\uc774 \ud559\uad50\uc5d0\uc11c \uc81c\uc801\uc744 \ub2f9\ud588\uc73c\uba70 \ub8f8\uc0b4\ub871 \uc9c1\uc6d0\uc774\ub77c\ub294 \uc0ac\uc2e4\ub3c4 \ubc1d\ud600\ub0c8\ub2e4. \ub610\ud55c \uc5ec\ub300\uc0dd\uc758 \ub3d9\uc0dd\uc774 \ub611\uac19\uc740 \ubc29\ubc95\uc73c\ub85c \ub8f8\uc0b4\ub871 \uc0ac\uc7a5\uc5d0\uac8c \uc131\ud3ed\ud589 \ub204\uba85\uc744 \uc50c\uc6b0\ub824 \ud55c \uc0ac\uc2e4\ub3c4 \ub4dc\ub7ec\ub0ac\ub2e4. \ub610\ud55c \uc5ec\ub300\uc0dd\uc774 \uce5c\uad6c\uc5d0\uac8c \ub3c8\uc744 \uc8fc\uba70 \uc790\uc2e0\uc744 \ub54c\ub824 \uc0c1\ud574\ub97c \uc785\ud788\ub3c4\ub85d \ud55c\uac83\ub3c4 \ub4dc\ub7ec\ub0ac\uace0 \uc774\ub7ec\ud55c \uc99d\uac70\ubb3c\ub4e4\uc774 \uc778\uc815\ub41c \uac83\uc774\ub2e4. \uadf8\ub9ac\uace0 \uc774\ub4ec\ud574\uc778 2002\ub144 7\uc6d4 12\uc77c \ubb34\uc8c4 \ud655\uc815 \ud310\uacb0\uc744 \ubc1b\uc558\ub2e4.", "paragraph_answer": "\uadf8\ub7ec\ub098 \uc8fc\ubcd1\uc9c4\uc740 \ud56d\uc18c\ud588\uace0 2001\ub144 11\uc6d4 28\uc77c\uc5d0 \ubb34\uc8c4 \ud310\uacb0\uc774 \ub0b4\ub824\uc84c\ub2e4. \uc7ac\ud310\ubd80\ub294 \u201c\ud53c\ud574\uc790\uac00 \uc8fc\ubcd1\uc9c4\uc73c\ub85c\ubd80\ud130 \ubc1b\uc740 \ud569\uc758\uae08\uc744 \uc99d\uc778 \ub4e4\uc5d0\uac8c \ub098\ub220\uc900 \uacbd\uc704 \ub4f1\uc73c\ub85c \ubcfc \ub54c \ud53c\ud574\uc790\uc640 \uc99d\uc778 \ub4f1\uc758 \uc9c4\uc220\uc744 \ubbff\uae30 \uc5b4\ub835\uace0 \ud53c\ud574\uc790 \ud5c8\ubc85\uc9c0\uc758 \uc0c1\ucc98\ub3c4 \uc0ac\uac74 \ub2f9\uc2dc \uc785\uc740 \uac83\uc73c\ub85c \ubcf4\uae30 \uc5b4\ub835\ub2e4\u201d\uace0 \ub9d0\ud588\ub2e4. \uc8fc\ubcd1\uc9c4\uacfc \ub3d9\ub8cc\ub4e4\uc740 \uc5ec\ub300\uc0dd\uc774 \ud559\uad50\uc5d0\uc11c \uc81c\uc801\uc744 \ub2f9\ud588\uc73c\uba70 \ub8f8\uc0b4\ub871 \uc9c1\uc6d0\uc774\ub77c\ub294 \uc0ac\uc2e4\ub3c4 \ubc1d\ud600\ub0c8\ub2e4. \ub610\ud55c \uc5ec\ub300\uc0dd\uc758 \ub3d9\uc0dd\uc774 \ub611\uac19\uc740 \ubc29\ubc95\uc73c\ub85c \ub8f8\uc0b4\ub871 \uc0ac\uc7a5\uc5d0\uac8c \uc131\ud3ed\ud589 \ub204\uba85\uc744 \uc50c\uc6b0\ub824 \ud55c \uc0ac\uc2e4\ub3c4 \ub4dc\ub7ec\ub0ac\ub2e4. \ub610\ud55c \uc5ec\ub300\uc0dd\uc774 \uce5c\uad6c\uc5d0\uac8c \ub3c8\uc744 \uc8fc\uba70 \uc790\uc2e0\uc744 \ub54c\ub824 \uc0c1\ud574\ub97c \uc785\ud788\ub3c4\ub85d \ud55c\uac83\ub3c4 \ub4dc\ub7ec\ub0ac\uace0 \uc774\ub7ec\ud55c \uc99d\uac70\ubb3c\ub4e4\uc774 \uc778\uc815\ub41c \uac83\uc774\ub2e4. \uadf8\ub9ac\uace0 \uc774\ub4ec\ud574\uc778 2002\ub144 7\uc6d4 12\uc77c \ubb34\uc8c4 \ud655\uc815 \ud310\uacb0\uc744 \ubc1b\uc558\ub2e4.", "sentence_answer": "\uc7ac\ud310\ubd80\ub294 \u201c\ud53c\ud574\uc790\uac00 \uc8fc\ubcd1\uc9c4\uc73c\ub85c\ubd80\ud130 \ubc1b\uc740 \ud569\uc758\uae08\uc744 \uc99d\uc778 \ub4e4\uc5d0\uac8c \ub098\ub220\uc900 \uacbd\uc704 \ub4f1\uc73c\ub85c \ubcfc \ub54c \ud53c\ud574\uc790\uc640 \uc99d\uc778 \ub4f1\uc758 \uc9c4\uc220\uc744 \ubbff\uae30 \uc5b4\ub835\uace0 \ud53c\ud574\uc790 \ud5c8\ubc85\uc9c0\uc758 \uc0c1\ucc98\ub3c4 \uc0ac\uac74 \ub2f9\uc2dc \uc785\uc740 \uac83\uc73c\ub85c \ubcf4\uae30 \uc5b4\ub835\ub2e4\u201d\uace0 \ub9d0\ud588\ub2e4.", "paragraph_id": "6555914-9-0", "paragraph_question": "question: \ud53c\ud574\uc790\uac00 \uc8fc\ubcd1\uc9c4\uc73c\ub85c\ubd80\ud130 \ubc1b\uc740 \ud569\uc758\uae00\uc744 \ub098\ub220\uc900 \uc774\ub4e4\uc740?, context: \uadf8\ub7ec\ub098 \uc8fc\ubcd1\uc9c4\uc740 \ud56d\uc18c\ud588\uace0 2001\ub144 11\uc6d4 28\uc77c\uc5d0 \ubb34\uc8c4 \ud310\uacb0\uc774 \ub0b4\ub824\uc84c\ub2e4. \uc7ac\ud310\ubd80\ub294 \u201c\ud53c\ud574\uc790\uac00 \uc8fc\ubcd1\uc9c4\uc73c\ub85c\ubd80\ud130 \ubc1b\uc740 \ud569\uc758\uae08\uc744 \uc99d\uc778\ub4e4\uc5d0\uac8c \ub098\ub220\uc900 \uacbd\uc704 \ub4f1\uc73c\ub85c \ubcfc \ub54c \ud53c\ud574\uc790\uc640 \uc99d\uc778 \ub4f1\uc758 \uc9c4\uc220\uc744 \ubbff\uae30 \uc5b4\ub835\uace0 \ud53c\ud574\uc790 \ud5c8\ubc85\uc9c0\uc758 \uc0c1\ucc98\ub3c4 \uc0ac\uac74 \ub2f9\uc2dc \uc785\uc740 \uac83\uc73c\ub85c \ubcf4\uae30 \uc5b4\ub835\ub2e4\u201d\uace0 \ub9d0\ud588\ub2e4. \uc8fc\ubcd1\uc9c4\uacfc \ub3d9\ub8cc\ub4e4\uc740 \uc5ec\ub300\uc0dd\uc774 \ud559\uad50\uc5d0\uc11c \uc81c\uc801\uc744 \ub2f9\ud588\uc73c\uba70 \ub8f8\uc0b4\ub871 \uc9c1\uc6d0\uc774\ub77c\ub294 \uc0ac\uc2e4\ub3c4 \ubc1d\ud600\ub0c8\ub2e4. \ub610\ud55c \uc5ec\ub300\uc0dd\uc758 \ub3d9\uc0dd\uc774 \ub611\uac19\uc740 \ubc29\ubc95\uc73c\ub85c \ub8f8\uc0b4\ub871 \uc0ac\uc7a5\uc5d0\uac8c \uc131\ud3ed\ud589 \ub204\uba85\uc744 \uc50c\uc6b0\ub824 \ud55c \uc0ac\uc2e4\ub3c4 \ub4dc\ub7ec\ub0ac\ub2e4. \ub610\ud55c \uc5ec\ub300\uc0dd\uc774 \uce5c\uad6c\uc5d0\uac8c \ub3c8\uc744 \uc8fc\uba70 \uc790\uc2e0\uc744 \ub54c\ub824 \uc0c1\ud574\ub97c \uc785\ud788\ub3c4\ub85d \ud55c\uac83\ub3c4 \ub4dc\ub7ec\ub0ac\uace0 \uc774\ub7ec\ud55c \uc99d\uac70\ubb3c\ub4e4\uc774 \uc778\uc815\ub41c \uac83\uc774\ub2e4. \uadf8\ub9ac\uace0 \uc774\ub4ec\ud574\uc778 2002\ub144 7\uc6d4 12\uc77c \ubb34\uc8c4 \ud655\uc815 \ud310\uacb0\uc744 \ubc1b\uc558\ub2e4."} {"question": "\uae40\ubcf4\ub984\uc740 2015\ub144 11\uc6d4 23\uc77c \ub9e4\uc2a4\uc2a4\ud0c0\ud2b8 \ub514\ube44\uc804A\uc5d0\uc11c \ub9c8\uc9c0\ub9c9 16\ubc14\ud034\ub97c \ub3cc\ub2e4\uac00 \uc77c\ubcf8\uc758 \uc5b4\ub5a4 \uc120\uc218\uc640 \uc811\ucd09\uc73c\ub85c \ub118\uc5b4\uc84c\ub294\uac00?", "paragraph": "2015\ub144 11\uc6d4 13\uc77c \uce90\ub098\ub2e4 \uce98\uac70\ub9ac\uc5d0\uc11c \uc5f4\ub9b0 2015-2016 \uad6d\uc81c\ube59\uc0c1\uacbd\uae30\uc5f0\ub9f9 \uc2a4\ud53c\ub4dc\uc2a4\ucf00\uc774\ud305\uc6d4\ub4dc\ucef5 1\ucc28 \ub300\ud68c 3000m \ub514\ube44\uc804A\uc5d0\uc11c \uae40\ubcf4\ub984\uc740 4\ubd8408\ucd0895\ub85c 15\uc704\ub97c \ucc28\uc9c0\ud588\ub2e4. 14\uc77c \uc5f4\ub9b0 \ud300\ucd94\uc6d4\uc5d0\uc11c\ub294 \uae40\ubcf4\ub984, \ub178\uc120\uc601, \ubc15\ub3c4\uc601\uc774 2\ubd8459\ucd0825\ub85c 5\uc704\ub97c \ucc28\uc9c0\ud588\ub2e4. 15\uc77c \uc5f4\ub9b0 1500m \ub514\ube44\uc804A\uc5d0\uc11c\ub294 \uae40\ubcf4\ub984\uc774 1\ubd8457\ucd0883\uc73c\ub85c 20\uc704\ub97c \ucc28\uc9c0\ud588\ub2e4. 16\uc77c \uc5f4\ub9b0 \ub9e4\uc2a4\uc2a4\ud0c0\ud2b8 \ub514\ube44\uc804A\uc5d0\uc11c\ub294 \uae40\ubcf4\ub984\uc774 8\ubd8436\ucd0804\uc758 \uae30\ub85d\uc73c\ub85c \uac00\uc7a5 \uba3c\uc800 \uacb0\uc2b9\uc120\uc744 \ud1b5\uacfc\ud588\ub2e4. 2\uc704\uc778 \uc774\ub808\uc778 \uc288\uce74\uc6b0\ud150(\ub124\ub35c\ub780\ub4dc-8\ubd8436\ucd0809)\uc744 0.05\ucd08 \ucc28\ub85c \uc81c\ucce4\ub2e4. \uae40\ubcf4\ub984\uc740 2013\ub144 2\uc6d4 \uc6d4\ub4dc\ucef5 7\ucc28 \ub300\ud68c \ub9e4\uc2a4\uc2a4\ud0c0\ud2b8\uc5d0\uc11c \uc6b0\uc2b9\ud55c \uc774\ud6c4 2\ub144 9\uac1c\uc6d4\ub9cc\uc5d0 \uae08\uba54\ub2ec\ub9ac\uc2a4\ud2b8\uac00 \ub410\ub2e4. 2015\ub144 11\uc6d4 20\uc77c \ubbf8\uad6d \uc194\ud2b8\ub808\uc774\ud06c\uc2dc\ud2f0\uc5d0\uc11c \uc5f4\ub9b0 \uc6d4\ub4dc\ucef5 2\ucc28 \ub300\ud68c 5000m \ub514\ube44\uc804B\uc5d0\uc11c \uae40\ubcf4\ub984\uc740 7\ubd845\ucd0855\uc758 \ud55c\uad6d\ucd5c\uace0\uae30\ub85d\uc744 \uc138\uc6b0\uba70 8\uc704\ub97c \ucc28\uc9c0\ud588\ub2e4. 21\uc77c \uc5f4\ub9b0 1500m \ub514\ube44\uc804B\uc5d0\uc11c\ub294 \uae40\ubcf4\ub984\uc774 1\ubd8457\ucd0876\uc73c\ub85c 12\uc704\ub97c \ucc28\uc9c0\ud588\ub2e4. 23\uc77c \uc5f4\ub9b0 \ub9e4\uc2a4\uc2a4\ud0c0\ud2b8 \ub514\ube44\uc804A\uc5d0\uc11c \uae40\ubcf4\ub984\uc740 \ub9c8\uc9c0\ub9c9 16\ubc14\ud034\uc9f8\ub97c \ub3cc\ub2e4\uac00 \ub118\uc5b4\uc838 20\uba85 \uac00\uc6b4\ub370 \ucd5c\ud558\uc704\uc5d0 \uba38\ubb3c\ub800\ub2e4. \uc77c\ubcf8\uc758 \uc624\uc2dc\uae30\ub9ac \ubbf8\uc0ac\ud0a4\uc640 \uc811\ucd09\uc774 \uc788\uc740 \ud6c4, \ub118\uc5b4\uc84c\ub2e4. \uae40\ubcf4\ub984\uc774 \ub9c8\uc9c0\ub9c9 \ubc14\ud034\ub97c \ub3cc \ub54c\uae4c\uc9c0 \uc120\ub450 \uacbd\uc7c1\uc744 \ud3bc\ucce4\uae30\uc5d0 \uc544\uc26c\uc6c0\uc774 \ub354 \ud06c\ub2e4. 15\ubc14\ud034\uae4c\uc9c0\uc758 \uae30\ub85d\ub9cc \uc778\uc815\ubc1b\uc544 8\ubd8401\ucd0885\ub97c \uae30\ub85d\ud588\ub2e4. \uc6b0\uc2b9\uc740 8\ubd8427\ucd0819\ub85c \uacb0\uc2b9\uc120\uc744 \ud1b5\uacfc\ud55c \uc774\ub808\uc778 \uc288\uce74\uc6b0\ud150(\ub124\ub35c\ub780\ub4dc)\uc774 \ucc28\uc9c0\ud588\ub2e4. \uae40\ubcf4\ub984\uacfc \ud568\uaed8 \ucd9c\uc804\ud55c \ub178\uc120\uc601(26-\uac15\uc6d0\ub3c4\uccad)\uc740 8\ubd8428\ucd0829\ub85c 12\uc704\uc5d0 \uc62c\ub790\ub2e4. \uae40\ubcf4\ub984\uc740 \ud5c8\ub9ac \ubd80\uc704\uc758 \ud54f\uc904\uc774 \ud130\uc9c0\uace0 \uc5f0\uace8\uc744 \ub2e4\uccd0 \uc6d4\ub4dc\ucef5 3, 4\ucc28 \ub300\ud68c\uc5d0 \ub098\uc124 \uc218 \uc5c6\uac8c \ub410\ub2e4.", "answer": "\uc624\uc2dc\uae30\ub9ac \ubbf8\uc0ac\ud0a4\uc640", "sentence": "\uc77c\ubcf8\uc758 \uc624\uc2dc\uae30\ub9ac \ubbf8\uc0ac\ud0a4\uc640 \uc811\ucd09\uc774 \uc788\uc740 \ud6c4, \ub118\uc5b4\uc84c\ub2e4.", "paragraph_sentence": "2015\ub144 11\uc6d4 13\uc77c \uce90\ub098\ub2e4 \uce98\uac70\ub9ac\uc5d0\uc11c \uc5f4\ub9b0 2015-2016 \uad6d\uc81c\ube59\uc0c1\uacbd\uae30\uc5f0\ub9f9 \uc2a4\ud53c\ub4dc\uc2a4\ucf00\uc774\ud305\uc6d4\ub4dc\ucef5 1\ucc28 \ub300\ud68c 3000m \ub514\ube44\uc804A\uc5d0\uc11c \uae40\ubcf4\ub984\uc740 4\ubd8408\ucd0895\ub85c 15\uc704\ub97c \ucc28\uc9c0\ud588\ub2e4. 14\uc77c \uc5f4\ub9b0 \ud300\ucd94\uc6d4\uc5d0\uc11c\ub294 \uae40\ubcf4\ub984, \ub178\uc120\uc601, \ubc15\ub3c4\uc601\uc774 2\ubd8459\ucd0825\ub85c 5\uc704\ub97c \ucc28\uc9c0\ud588\ub2e4. 15\uc77c \uc5f4\ub9b0 1500m \ub514\ube44\uc804A\uc5d0\uc11c\ub294 \uae40\ubcf4\ub984\uc774 1\ubd8457\ucd0883\uc73c\ub85c 20\uc704\ub97c \ucc28\uc9c0\ud588\ub2e4. 16\uc77c \uc5f4\ub9b0 \ub9e4\uc2a4\uc2a4\ud0c0\ud2b8 \ub514\ube44\uc804A\uc5d0\uc11c\ub294 \uae40\ubcf4\ub984\uc774 8\ubd8436\ucd0804\uc758 \uae30\ub85d\uc73c\ub85c \uac00\uc7a5 \uba3c\uc800 \uacb0\uc2b9\uc120\uc744 \ud1b5\uacfc\ud588\ub2e4. 2\uc704\uc778 \uc774\ub808\uc778 \uc288\uce74\uc6b0\ud150(\ub124\ub35c\ub780\ub4dc-8\ubd8436\ucd0809)\uc744 0.05\ucd08 \ucc28\ub85c \uc81c\ucce4\ub2e4. \uae40\ubcf4\ub984\uc740 2013\ub144 2\uc6d4 \uc6d4\ub4dc\ucef5 7\ucc28 \ub300\ud68c \ub9e4\uc2a4\uc2a4\ud0c0\ud2b8\uc5d0\uc11c \uc6b0\uc2b9\ud55c \uc774\ud6c4 2\ub144 9\uac1c\uc6d4\ub9cc\uc5d0 \uae08\uba54\ub2ec\ub9ac\uc2a4\ud2b8\uac00 \ub410\ub2e4. 2015\ub144 11\uc6d4 20\uc77c \ubbf8\uad6d \uc194\ud2b8\ub808\uc774\ud06c\uc2dc\ud2f0\uc5d0\uc11c \uc5f4\ub9b0 \uc6d4\ub4dc\ucef5 2\ucc28 \ub300\ud68c 5000m \ub514\ube44\uc804B\uc5d0\uc11c \uae40\ubcf4\ub984\uc740 7\ubd845\ucd0855\uc758 \ud55c\uad6d\ucd5c\uace0\uae30\ub85d\uc744 \uc138\uc6b0\uba70 8\uc704\ub97c \ucc28\uc9c0\ud588\ub2e4. 21\uc77c \uc5f4\ub9b0 1500m \ub514\ube44\uc804B\uc5d0\uc11c\ub294 \uae40\ubcf4\ub984\uc774 1\ubd8457\ucd0876\uc73c\ub85c 12\uc704\ub97c \ucc28\uc9c0\ud588\ub2e4. 23\uc77c \uc5f4\ub9b0 \ub9e4\uc2a4\uc2a4\ud0c0\ud2b8 \ub514\ube44\uc804A\uc5d0\uc11c \uae40\ubcf4\ub984\uc740 \ub9c8\uc9c0\ub9c9 16\ubc14\ud034\uc9f8\ub97c \ub3cc\ub2e4\uac00 \ub118\uc5b4\uc838 20\uba85 \uac00\uc6b4\ub370 \ucd5c\ud558\uc704\uc5d0 \uba38\ubb3c\ub800\ub2e4. \uc77c\ubcf8\uc758 \uc624\uc2dc\uae30\ub9ac \ubbf8\uc0ac\ud0a4\uc640 \uc811\ucd09\uc774 \uc788\uc740 \ud6c4, \ub118\uc5b4\uc84c\ub2e4. \uae40\ubcf4\ub984\uc774 \ub9c8\uc9c0\ub9c9 \ubc14\ud034\ub97c \ub3cc \ub54c\uae4c\uc9c0 \uc120\ub450 \uacbd\uc7c1\uc744 \ud3bc\ucce4\uae30\uc5d0 \uc544\uc26c\uc6c0\uc774 \ub354 \ud06c\ub2e4. 15\ubc14\ud034\uae4c\uc9c0\uc758 \uae30\ub85d\ub9cc \uc778\uc815\ubc1b\uc544 8\ubd8401\ucd0885\ub97c \uae30\ub85d\ud588\ub2e4. \uc6b0\uc2b9\uc740 8\ubd8427\ucd0819\ub85c \uacb0\uc2b9\uc120\uc744 \ud1b5\uacfc\ud55c \uc774\ub808\uc778 \uc288\uce74\uc6b0\ud150(\ub124\ub35c\ub780\ub4dc)\uc774 \ucc28\uc9c0\ud588\ub2e4. \uae40\ubcf4\ub984\uacfc \ud568\uaed8 \ucd9c\uc804\ud55c \ub178\uc120\uc601(26-\uac15\uc6d0\ub3c4\uccad)\uc740 8\ubd8428\ucd0829\ub85c 12\uc704\uc5d0 \uc62c\ub790\ub2e4. \uae40\ubcf4\ub984\uc740 \ud5c8\ub9ac \ubd80\uc704\uc758 \ud54f\uc904\uc774 \ud130\uc9c0\uace0 \uc5f0\uace8\uc744 \ub2e4\uccd0 \uc6d4\ub4dc\ucef5 3, 4\ucc28 \ub300\ud68c\uc5d0 \ub098\uc124 \uc218 \uc5c6\uac8c \ub410\ub2e4.", "paragraph_answer": "2015\ub144 11\uc6d4 13\uc77c \uce90\ub098\ub2e4 \uce98\uac70\ub9ac\uc5d0\uc11c \uc5f4\ub9b0 2015-2016 \uad6d\uc81c\ube59\uc0c1\uacbd\uae30\uc5f0\ub9f9 \uc2a4\ud53c\ub4dc\uc2a4\ucf00\uc774\ud305\uc6d4\ub4dc\ucef5 1\ucc28 \ub300\ud68c 3000m \ub514\ube44\uc804A\uc5d0\uc11c \uae40\ubcf4\ub984\uc740 4\ubd8408\ucd0895\ub85c 15\uc704\ub97c \ucc28\uc9c0\ud588\ub2e4. 14\uc77c \uc5f4\ub9b0 \ud300\ucd94\uc6d4\uc5d0\uc11c\ub294 \uae40\ubcf4\ub984, \ub178\uc120\uc601, \ubc15\ub3c4\uc601\uc774 2\ubd8459\ucd0825\ub85c 5\uc704\ub97c \ucc28\uc9c0\ud588\ub2e4. 15\uc77c \uc5f4\ub9b0 1500m \ub514\ube44\uc804A\uc5d0\uc11c\ub294 \uae40\ubcf4\ub984\uc774 1\ubd8457\ucd0883\uc73c\ub85c 20\uc704\ub97c \ucc28\uc9c0\ud588\ub2e4. 16\uc77c \uc5f4\ub9b0 \ub9e4\uc2a4\uc2a4\ud0c0\ud2b8 \ub514\ube44\uc804A\uc5d0\uc11c\ub294 \uae40\ubcf4\ub984\uc774 8\ubd8436\ucd0804\uc758 \uae30\ub85d\uc73c\ub85c \uac00\uc7a5 \uba3c\uc800 \uacb0\uc2b9\uc120\uc744 \ud1b5\uacfc\ud588\ub2e4. 2\uc704\uc778 \uc774\ub808\uc778 \uc288\uce74\uc6b0\ud150(\ub124\ub35c\ub780\ub4dc-8\ubd8436\ucd0809)\uc744 0.05\ucd08 \ucc28\ub85c \uc81c\ucce4\ub2e4. \uae40\ubcf4\ub984\uc740 2013\ub144 2\uc6d4 \uc6d4\ub4dc\ucef5 7\ucc28 \ub300\ud68c \ub9e4\uc2a4\uc2a4\ud0c0\ud2b8\uc5d0\uc11c \uc6b0\uc2b9\ud55c \uc774\ud6c4 2\ub144 9\uac1c\uc6d4\ub9cc\uc5d0 \uae08\uba54\ub2ec\ub9ac\uc2a4\ud2b8\uac00 \ub410\ub2e4. 2015\ub144 11\uc6d4 20\uc77c \ubbf8\uad6d \uc194\ud2b8\ub808\uc774\ud06c\uc2dc\ud2f0\uc5d0\uc11c \uc5f4\ub9b0 \uc6d4\ub4dc\ucef5 2\ucc28 \ub300\ud68c 5000m \ub514\ube44\uc804B\uc5d0\uc11c \uae40\ubcf4\ub984\uc740 7\ubd845\ucd0855\uc758 \ud55c\uad6d\ucd5c\uace0\uae30\ub85d\uc744 \uc138\uc6b0\uba70 8\uc704\ub97c \ucc28\uc9c0\ud588\ub2e4. 21\uc77c \uc5f4\ub9b0 1500m \ub514\ube44\uc804B\uc5d0\uc11c\ub294 \uae40\ubcf4\ub984\uc774 1\ubd8457\ucd0876\uc73c\ub85c 12\uc704\ub97c \ucc28\uc9c0\ud588\ub2e4. 23\uc77c \uc5f4\ub9b0 \ub9e4\uc2a4\uc2a4\ud0c0\ud2b8 \ub514\ube44\uc804A\uc5d0\uc11c \uae40\ubcf4\ub984\uc740 \ub9c8\uc9c0\ub9c9 16\ubc14\ud034\uc9f8\ub97c \ub3cc\ub2e4\uac00 \ub118\uc5b4\uc838 20\uba85 \uac00\uc6b4\ub370 \ucd5c\ud558\uc704\uc5d0 \uba38\ubb3c\ub800\ub2e4. \uc77c\ubcf8\uc758 \uc624\uc2dc\uae30\ub9ac \ubbf8\uc0ac\ud0a4\uc640 \uc811\ucd09\uc774 \uc788\uc740 \ud6c4, \ub118\uc5b4\uc84c\ub2e4. \uae40\ubcf4\ub984\uc774 \ub9c8\uc9c0\ub9c9 \ubc14\ud034\ub97c \ub3cc \ub54c\uae4c\uc9c0 \uc120\ub450 \uacbd\uc7c1\uc744 \ud3bc\ucce4\uae30\uc5d0 \uc544\uc26c\uc6c0\uc774 \ub354 \ud06c\ub2e4. 15\ubc14\ud034\uae4c\uc9c0\uc758 \uae30\ub85d\ub9cc \uc778\uc815\ubc1b\uc544 8\ubd8401\ucd0885\ub97c \uae30\ub85d\ud588\ub2e4. \uc6b0\uc2b9\uc740 8\ubd8427\ucd0819\ub85c \uacb0\uc2b9\uc120\uc744 \ud1b5\uacfc\ud55c \uc774\ub808\uc778 \uc288\uce74\uc6b0\ud150(\ub124\ub35c\ub780\ub4dc)\uc774 \ucc28\uc9c0\ud588\ub2e4. \uae40\ubcf4\ub984\uacfc \ud568\uaed8 \ucd9c\uc804\ud55c \ub178\uc120\uc601(26-\uac15\uc6d0\ub3c4\uccad)\uc740 8\ubd8428\ucd0829\ub85c 12\uc704\uc5d0 \uc62c\ub790\ub2e4. \uae40\ubcf4\ub984\uc740 \ud5c8\ub9ac \ubd80\uc704\uc758 \ud54f\uc904\uc774 \ud130\uc9c0\uace0 \uc5f0\uace8\uc744 \ub2e4\uccd0 \uc6d4\ub4dc\ucef5 3, 4\ucc28 \ub300\ud68c\uc5d0 \ub098\uc124 \uc218 \uc5c6\uac8c \ub410\ub2e4.", "sentence_answer": "\uc77c\ubcf8\uc758 \uc624\uc2dc\uae30\ub9ac \ubbf8\uc0ac\ud0a4\uc640 \uc811\ucd09\uc774 \uc788\uc740 \ud6c4, \ub118\uc5b4\uc84c\ub2e4.", "paragraph_id": "6488273-12-1", "paragraph_question": "question: \uae40\ubcf4\ub984\uc740 2015\ub144 11\uc6d4 23\uc77c \ub9e4\uc2a4\uc2a4\ud0c0\ud2b8 \ub514\ube44\uc804A\uc5d0\uc11c \ub9c8\uc9c0\ub9c9 16\ubc14\ud034\ub97c \ub3cc\ub2e4\uac00 \uc77c\ubcf8\uc758 \uc5b4\ub5a4 \uc120\uc218\uc640 \uc811\ucd09\uc73c\ub85c \ub118\uc5b4\uc84c\ub294\uac00?, context: 2015\ub144 11\uc6d4 13\uc77c \uce90\ub098\ub2e4 \uce98\uac70\ub9ac\uc5d0\uc11c \uc5f4\ub9b0 2015-2016 \uad6d\uc81c\ube59\uc0c1\uacbd\uae30\uc5f0\ub9f9 \uc2a4\ud53c\ub4dc\uc2a4\ucf00\uc774\ud305\uc6d4\ub4dc\ucef5 1\ucc28 \ub300\ud68c 3000m \ub514\ube44\uc804A\uc5d0\uc11c \uae40\ubcf4\ub984\uc740 4\ubd8408\ucd0895\ub85c 15\uc704\ub97c \ucc28\uc9c0\ud588\ub2e4. 14\uc77c \uc5f4\ub9b0 \ud300\ucd94\uc6d4\uc5d0\uc11c\ub294 \uae40\ubcf4\ub984, \ub178\uc120\uc601, \ubc15\ub3c4\uc601\uc774 2\ubd8459\ucd0825\ub85c 5\uc704\ub97c \ucc28\uc9c0\ud588\ub2e4. 15\uc77c \uc5f4\ub9b0 1500m \ub514\ube44\uc804A\uc5d0\uc11c\ub294 \uae40\ubcf4\ub984\uc774 1\ubd8457\ucd0883\uc73c\ub85c 20\uc704\ub97c \ucc28\uc9c0\ud588\ub2e4. 16\uc77c \uc5f4\ub9b0 \ub9e4\uc2a4\uc2a4\ud0c0\ud2b8 \ub514\ube44\uc804A\uc5d0\uc11c\ub294 \uae40\ubcf4\ub984\uc774 8\ubd8436\ucd0804\uc758 \uae30\ub85d\uc73c\ub85c \uac00\uc7a5 \uba3c\uc800 \uacb0\uc2b9\uc120\uc744 \ud1b5\uacfc\ud588\ub2e4. 2\uc704\uc778 \uc774\ub808\uc778 \uc288\uce74\uc6b0\ud150(\ub124\ub35c\ub780\ub4dc-8\ubd8436\ucd0809)\uc744 0.05\ucd08 \ucc28\ub85c \uc81c\ucce4\ub2e4. \uae40\ubcf4\ub984\uc740 2013\ub144 2\uc6d4 \uc6d4\ub4dc\ucef5 7\ucc28 \ub300\ud68c \ub9e4\uc2a4\uc2a4\ud0c0\ud2b8\uc5d0\uc11c \uc6b0\uc2b9\ud55c \uc774\ud6c4 2\ub144 9\uac1c\uc6d4\ub9cc\uc5d0 \uae08\uba54\ub2ec\ub9ac\uc2a4\ud2b8\uac00 \ub410\ub2e4. 2015\ub144 11\uc6d4 20\uc77c \ubbf8\uad6d \uc194\ud2b8\ub808\uc774\ud06c\uc2dc\ud2f0\uc5d0\uc11c \uc5f4\ub9b0 \uc6d4\ub4dc\ucef5 2\ucc28 \ub300\ud68c 5000m \ub514\ube44\uc804B\uc5d0\uc11c \uae40\ubcf4\ub984\uc740 7\ubd845\ucd0855\uc758 \ud55c\uad6d\ucd5c\uace0\uae30\ub85d\uc744 \uc138\uc6b0\uba70 8\uc704\ub97c \ucc28\uc9c0\ud588\ub2e4. 21\uc77c \uc5f4\ub9b0 1500m \ub514\ube44\uc804B\uc5d0\uc11c\ub294 \uae40\ubcf4\ub984\uc774 1\ubd8457\ucd0876\uc73c\ub85c 12\uc704\ub97c \ucc28\uc9c0\ud588\ub2e4. 23\uc77c \uc5f4\ub9b0 \ub9e4\uc2a4\uc2a4\ud0c0\ud2b8 \ub514\ube44\uc804A\uc5d0\uc11c \uae40\ubcf4\ub984\uc740 \ub9c8\uc9c0\ub9c9 16\ubc14\ud034\uc9f8\ub97c \ub3cc\ub2e4\uac00 \ub118\uc5b4\uc838 20\uba85 \uac00\uc6b4\ub370 \ucd5c\ud558\uc704\uc5d0 \uba38\ubb3c\ub800\ub2e4. \uc77c\ubcf8\uc758 \uc624\uc2dc\uae30\ub9ac \ubbf8\uc0ac\ud0a4\uc640 \uc811\ucd09\uc774 \uc788\uc740 \ud6c4, \ub118\uc5b4\uc84c\ub2e4. \uae40\ubcf4\ub984\uc774 \ub9c8\uc9c0\ub9c9 \ubc14\ud034\ub97c \ub3cc \ub54c\uae4c\uc9c0 \uc120\ub450 \uacbd\uc7c1\uc744 \ud3bc\ucce4\uae30\uc5d0 \uc544\uc26c\uc6c0\uc774 \ub354 \ud06c\ub2e4. 15\ubc14\ud034\uae4c\uc9c0\uc758 \uae30\ub85d\ub9cc \uc778\uc815\ubc1b\uc544 8\ubd8401\ucd0885\ub97c \uae30\ub85d\ud588\ub2e4. \uc6b0\uc2b9\uc740 8\ubd8427\ucd0819\ub85c \uacb0\uc2b9\uc120\uc744 \ud1b5\uacfc\ud55c \uc774\ub808\uc778 \uc288\uce74\uc6b0\ud150(\ub124\ub35c\ub780\ub4dc)\uc774 \ucc28\uc9c0\ud588\ub2e4. \uae40\ubcf4\ub984\uacfc \ud568\uaed8 \ucd9c\uc804\ud55c \ub178\uc120\uc601(26-\uac15\uc6d0\ub3c4\uccad)\uc740 8\ubd8428\ucd0829\ub85c 12\uc704\uc5d0 \uc62c\ub790\ub2e4. \uae40\ubcf4\ub984\uc740 \ud5c8\ub9ac \ubd80\uc704\uc758 \ud54f\uc904\uc774 \ud130\uc9c0\uace0 \uc5f0\uace8\uc744 \ub2e4\uccd0 \uc6d4\ub4dc\ucef5 3, 4\ucc28 \ub300\ud68c\uc5d0 \ub098\uc124 \uc218 \uc5c6\uac8c \ub410\ub2e4."} {"question": "\uba85\uad6c\ud68c\uc5d0\uc11c \uac1c\ucd5c\ud558\ub294 \uccb4\ud5d8 \ud589\uc0ac\uc758 \uba85\uce6d\uc740 \ubb34\uc5c7\uc778\uac00?", "paragraph": "\uc77c\ubcf8 \uc804\uad6d \uac01\uc9c0\uc5d0\uc11c \uc9c0\uc790\uccb4, \uad50\uc721\uc704\uc6d0\ud68c, \uc18c\ub144 \uc57c\uad6c\ud300, \uae30\uc5c5 \ub4f1\uc5d0\uc11c\uc758 \uc758\ub8b0\uc5d0 \ub530\ub974\uace0 \uc6b0\uc120 \uc57c\uad6c\uc5d0 \ud765\ubbf8\ub97c \uac16\uac8c\ud558\uae30 \uc704\ud574 \uc57c\uad6c\uc5d0 \ub300\ud55c \uce5c\uadfc\uac10\uc744 \ub290\ub07c\ub3c4\ub85d \uccb4\ud5d8\ud558\ub294 \u2018\uba85\uad6c\ud68c \ubca0\uc774\uc2a4\ubcfc \ud398\uc2a4\ud2f0\ubc8c\u2019\uc744 \uac1c\ucd5c\ud558\uace0 \uc788\ub2e4. \uac70\uae30\uc5d0\uc120 \ud68c\uc6d0\uc774 \uccb4\ud5d8 \uc9c0\ub3c4\ud558\ub294 \ud22c\uad6c, \ud0c0\uaca9, \uc218\ube44\uc5d0 \ub300\ud574 \uc9c4\uc9c0\ud558\uac8c \ub9c8\uc8fc\ud558\ub294 \uc5b4\ub9b0\uc774\uc640 \ub354\ubd88\uc5b4 \uc751\uc6d0\ud558\ub7ec \uc628 \ud615\uc81c, \ubd80\ubaa8, \uc870\ubd80\ubaa8\uac00 \uac01\uc790 \uc2dc\ub300\uc758 \ud788\uc5b4\ub85c\uc640\uc758 \ub9cc\ub0a8\uc744 \ud1b5\ud574 \uc990\uae30\uac8c\ub054 \ub9cc\ub4e4\uace0 \uc788\ub2e4. \uadf8\ub9ac\uace0 \uc57c\uad6c\ub97c \uc798\ud558\uace0 \uc2f6\uc5b4\ud558\ub294 \uc5b4\ub9b0\uc774\uc5d0\uac8c\ub294 \ud68c\uc6d0\uc774 \uc9c1\uc811 \uc9c0\ub3c4\ud558\ub294 \u2018\uba85\uad6c\ud68c \uc57c\uad6c \uad50\uc2e4\u2019\ub3c4 \uc2e4\uc2dc\ud558\uace0 \uc788\ub2e4. \uc5ec\uae30\uc11c \ud22c\uad6c, \ud0c0\uaca9, \uc218\ube44 \uc804\ubb38\uac00\uac00 \uc5b4\ub9b0\uc774 \ud55c \uc0ac\ub78c \ud55c \uc0ac\ub78c\uc758 \uc18c\ub9ac\ub97c \ub4e3\uace0 \ucee4\ubba4\ub2c8\ucf00\uc774\uc158\uc744 \ucde8\ud558\uba74\uc11c \uc9c0\ub3c4\ud558\uace0 \uc788\ub2e4. \ucd5c\uadfc\uc5d0\ub294 \uae00\ub85c\ubc8c\ud654 \ub418\uba74\uc11c \ud574\uc678\uc5d0\uc11c\ub3c4 \uc57c\uad6c\ub97c \uc990\uae30\uba70 \ubcf4\uae09 \ud65c\ub3d9\uc744 \ud558\uace0 \uc788\ub294 \uc0ac\ub78c\ub4e4\uc758 \uc758\ub8b0\ub3c4 \uc788\uc5b4 \ud604\uc7ac \uc544\uc2dc\uc544 \uc5ec\ub7ec \ub098\ub77c(\ub300\ud55c\ubbfc\uad6d, \uc911\ud654\ubbfc\uad6d, \ud544\ub9ac\ud540)\uc758 \ud68c\uc6d0\ub4e4\uc744 \uc704\ud574 \ud604\uc9c0\uc758 \uc5b4\ub9b0\uc774\ub4e4\uc744 \uc9c0\ub3c4\ud558\uace0 \uc9c0\uc6d0 \ud589\uc0ac\ub3c4 \ud558\uace0 \uc788\ub2e4. \uc774 \uc678\uc5d0\ub3c4 \u2018\uba85\uad6c\ud68c\ubc30\u2019\ub77c\ub294 \uc774\ub984\uc774 \ub4e4\uc5b4\uac04 \uc18c\ub144\uc57c\uad6c\ub300\ud68c\uc758 \uac1c\ucd5c \uc9c0\uc6d0\uacfc \uc57c\uad6c\uc6a9\ud488\uc774 \ubd80\uc871\ud55c \uc9c0\uc5ed\uc758 \uc5b4\ub9b0\uc774\ub4e4\uc5d0\uac8c \uc6a9\ud488\uc744 \uae30\uc99d\ud558\uace0 \uc788\uc73c\uba70 \ub610\ud55c \uadf8\ub77c\uc6b4\ub4dc \uc774\uc678\uc5d0\uc11c\ub3c4 \uc5b4\ub9b0\uc774 \ud559\uc0dd, \uc18c\ub144 \uc57c\uad6c\ud300, \uc18c\ub144 \uad50\ub3c4\uc18c \uc7ac\uc18c\uc790\ub4e4\uc5d0\uac8c \ud68c\uc6d0\uc774 \uc9c1\uc811 \uc790\uc2e0\uc758 \uacbd\ud5d8\uc744 \uac15\uc5f0\ud558\ub294 \ub4f1\uc758 \ud65c\ub3d9\ub3c4 \ud558\uace0 \uc788\ub2e4.", "answer": "\uba85\uad6c\ud68c \ubca0\uc774\uc2a4\ubcfc \ud398\uc2a4\ud2f0\ubc8c", "sentence": "\uc77c\ubcf8 \uc804\uad6d \uac01\uc9c0\uc5d0\uc11c \uc9c0\uc790\uccb4, \uad50\uc721\uc704\uc6d0\ud68c, \uc18c\ub144 \uc57c\uad6c\ud300, \uae30\uc5c5 \ub4f1\uc5d0\uc11c\uc758 \uc758\ub8b0\uc5d0 \ub530\ub974\uace0 \uc6b0\uc120 \uc57c\uad6c\uc5d0 \ud765\ubbf8\ub97c \uac16\uac8c\ud558\uae30 \uc704\ud574 \uc57c\uad6c\uc5d0 \ub300\ud55c \uce5c\uadfc\uac10\uc744 \ub290\ub07c\ub3c4\ub85d \uccb4\ud5d8\ud558\ub294 \u2018 \uba85\uad6c\ud68c \ubca0\uc774\uc2a4\ubcfc \ud398\uc2a4\ud2f0\ubc8c \u2019\uc744 \uac1c\ucd5c\ud558\uace0 \uc788\ub2e4.", "paragraph_sentence": " \uc77c\ubcf8 \uc804\uad6d \uac01\uc9c0\uc5d0\uc11c \uc9c0\uc790\uccb4, \uad50\uc721\uc704\uc6d0\ud68c, \uc18c\ub144 \uc57c\uad6c\ud300, \uae30\uc5c5 \ub4f1\uc5d0\uc11c\uc758 \uc758\ub8b0\uc5d0 \ub530\ub974\uace0 \uc6b0\uc120 \uc57c\uad6c\uc5d0 \ud765\ubbf8\ub97c \uac16\uac8c\ud558\uae30 \uc704\ud574 \uc57c\uad6c\uc5d0 \ub300\ud55c \uce5c\uadfc\uac10\uc744 \ub290\ub07c\ub3c4\ub85d \uccb4\ud5d8\ud558\ub294 \u2018 \uba85\uad6c\ud68c \ubca0\uc774\uc2a4\ubcfc \ud398\uc2a4\ud2f0\ubc8c \u2019\uc744 \uac1c\ucd5c\ud558\uace0 \uc788\ub2e4. \uac70\uae30\uc5d0\uc120 \ud68c\uc6d0\uc774 \uccb4\ud5d8 \uc9c0\ub3c4\ud558\ub294 \ud22c\uad6c, \ud0c0\uaca9, \uc218\ube44\uc5d0 \ub300\ud574 \uc9c4\uc9c0\ud558\uac8c \ub9c8\uc8fc\ud558\ub294 \uc5b4\ub9b0\uc774\uc640 \ub354\ubd88\uc5b4 \uc751\uc6d0\ud558\ub7ec \uc628 \ud615\uc81c, \ubd80\ubaa8, \uc870\ubd80\ubaa8\uac00 \uac01\uc790 \uc2dc\ub300\uc758 \ud788\uc5b4\ub85c\uc640\uc758 \ub9cc\ub0a8\uc744 \ud1b5\ud574 \uc990\uae30\uac8c\ub054 \ub9cc\ub4e4\uace0 \uc788\ub2e4. \uadf8\ub9ac\uace0 \uc57c\uad6c\ub97c \uc798\ud558\uace0 \uc2f6\uc5b4\ud558\ub294 \uc5b4\ub9b0\uc774\uc5d0\uac8c\ub294 \ud68c\uc6d0\uc774 \uc9c1\uc811 \uc9c0\ub3c4\ud558\ub294 \u2018\uba85\uad6c\ud68c \uc57c\uad6c \uad50\uc2e4\u2019\ub3c4 \uc2e4\uc2dc\ud558\uace0 \uc788\ub2e4. \uc5ec\uae30\uc11c \ud22c\uad6c, \ud0c0\uaca9, \uc218\ube44 \uc804\ubb38\uac00\uac00 \uc5b4\ub9b0\uc774 \ud55c \uc0ac\ub78c \ud55c \uc0ac\ub78c\uc758 \uc18c\ub9ac\ub97c \ub4e3\uace0 \ucee4\ubba4\ub2c8\ucf00\uc774\uc158\uc744 \ucde8\ud558\uba74\uc11c \uc9c0\ub3c4\ud558\uace0 \uc788\ub2e4. \ucd5c\uadfc\uc5d0\ub294 \uae00\ub85c\ubc8c\ud654 \ub418\uba74\uc11c \ud574\uc678\uc5d0\uc11c\ub3c4 \uc57c\uad6c\ub97c \uc990\uae30\uba70 \ubcf4\uae09 \ud65c\ub3d9\uc744 \ud558\uace0 \uc788\ub294 \uc0ac\ub78c\ub4e4\uc758 \uc758\ub8b0\ub3c4 \uc788\uc5b4 \ud604\uc7ac \uc544\uc2dc\uc544 \uc5ec\ub7ec \ub098\ub77c(\ub300\ud55c\ubbfc\uad6d, \uc911\ud654\ubbfc\uad6d, \ud544\ub9ac\ud540)\uc758 \ud68c\uc6d0\ub4e4\uc744 \uc704\ud574 \ud604\uc9c0\uc758 \uc5b4\ub9b0\uc774\ub4e4\uc744 \uc9c0\ub3c4\ud558\uace0 \uc9c0\uc6d0 \ud589\uc0ac\ub3c4 \ud558\uace0 \uc788\ub2e4. \uc774 \uc678\uc5d0\ub3c4 \u2018\uba85\uad6c\ud68c\ubc30\u2019\ub77c\ub294 \uc774\ub984\uc774 \ub4e4\uc5b4\uac04 \uc18c\ub144\uc57c\uad6c\ub300\ud68c\uc758 \uac1c\ucd5c \uc9c0\uc6d0\uacfc \uc57c\uad6c\uc6a9\ud488\uc774 \ubd80\uc871\ud55c \uc9c0\uc5ed\uc758 \uc5b4\ub9b0\uc774\ub4e4\uc5d0\uac8c \uc6a9\ud488\uc744 \uae30\uc99d\ud558\uace0 \uc788\uc73c\uba70 \ub610\ud55c \uadf8\ub77c\uc6b4\ub4dc \uc774\uc678\uc5d0\uc11c\ub3c4 \uc5b4\ub9b0\uc774 \ud559\uc0dd, \uc18c\ub144 \uc57c\uad6c\ud300, \uc18c\ub144 \uad50\ub3c4\uc18c \uc7ac\uc18c\uc790\ub4e4\uc5d0\uac8c \ud68c\uc6d0\uc774 \uc9c1\uc811 \uc790\uc2e0\uc758 \uacbd\ud5d8\uc744 \uac15\uc5f0\ud558\ub294 \ub4f1\uc758 \ud65c\ub3d9\ub3c4 \ud558\uace0 \uc788\ub2e4.", "paragraph_answer": "\uc77c\ubcf8 \uc804\uad6d \uac01\uc9c0\uc5d0\uc11c \uc9c0\uc790\uccb4, \uad50\uc721\uc704\uc6d0\ud68c, \uc18c\ub144 \uc57c\uad6c\ud300, \uae30\uc5c5 \ub4f1\uc5d0\uc11c\uc758 \uc758\ub8b0\uc5d0 \ub530\ub974\uace0 \uc6b0\uc120 \uc57c\uad6c\uc5d0 \ud765\ubbf8\ub97c \uac16\uac8c\ud558\uae30 \uc704\ud574 \uc57c\uad6c\uc5d0 \ub300\ud55c \uce5c\uadfc\uac10\uc744 \ub290\ub07c\ub3c4\ub85d \uccb4\ud5d8\ud558\ub294 \u2018 \uba85\uad6c\ud68c \ubca0\uc774\uc2a4\ubcfc \ud398\uc2a4\ud2f0\ubc8c \u2019\uc744 \uac1c\ucd5c\ud558\uace0 \uc788\ub2e4. \uac70\uae30\uc5d0\uc120 \ud68c\uc6d0\uc774 \uccb4\ud5d8 \uc9c0\ub3c4\ud558\ub294 \ud22c\uad6c, \ud0c0\uaca9, \uc218\ube44\uc5d0 \ub300\ud574 \uc9c4\uc9c0\ud558\uac8c \ub9c8\uc8fc\ud558\ub294 \uc5b4\ub9b0\uc774\uc640 \ub354\ubd88\uc5b4 \uc751\uc6d0\ud558\ub7ec \uc628 \ud615\uc81c, \ubd80\ubaa8, \uc870\ubd80\ubaa8\uac00 \uac01\uc790 \uc2dc\ub300\uc758 \ud788\uc5b4\ub85c\uc640\uc758 \ub9cc\ub0a8\uc744 \ud1b5\ud574 \uc990\uae30\uac8c\ub054 \ub9cc\ub4e4\uace0 \uc788\ub2e4. \uadf8\ub9ac\uace0 \uc57c\uad6c\ub97c \uc798\ud558\uace0 \uc2f6\uc5b4\ud558\ub294 \uc5b4\ub9b0\uc774\uc5d0\uac8c\ub294 \ud68c\uc6d0\uc774 \uc9c1\uc811 \uc9c0\ub3c4\ud558\ub294 \u2018\uba85\uad6c\ud68c \uc57c\uad6c \uad50\uc2e4\u2019\ub3c4 \uc2e4\uc2dc\ud558\uace0 \uc788\ub2e4. \uc5ec\uae30\uc11c \ud22c\uad6c, \ud0c0\uaca9, \uc218\ube44 \uc804\ubb38\uac00\uac00 \uc5b4\ub9b0\uc774 \ud55c \uc0ac\ub78c \ud55c \uc0ac\ub78c\uc758 \uc18c\ub9ac\ub97c \ub4e3\uace0 \ucee4\ubba4\ub2c8\ucf00\uc774\uc158\uc744 \ucde8\ud558\uba74\uc11c \uc9c0\ub3c4\ud558\uace0 \uc788\ub2e4. \ucd5c\uadfc\uc5d0\ub294 \uae00\ub85c\ubc8c\ud654 \ub418\uba74\uc11c \ud574\uc678\uc5d0\uc11c\ub3c4 \uc57c\uad6c\ub97c \uc990\uae30\uba70 \ubcf4\uae09 \ud65c\ub3d9\uc744 \ud558\uace0 \uc788\ub294 \uc0ac\ub78c\ub4e4\uc758 \uc758\ub8b0\ub3c4 \uc788\uc5b4 \ud604\uc7ac \uc544\uc2dc\uc544 \uc5ec\ub7ec \ub098\ub77c(\ub300\ud55c\ubbfc\uad6d, \uc911\ud654\ubbfc\uad6d, \ud544\ub9ac\ud540)\uc758 \ud68c\uc6d0\ub4e4\uc744 \uc704\ud574 \ud604\uc9c0\uc758 \uc5b4\ub9b0\uc774\ub4e4\uc744 \uc9c0\ub3c4\ud558\uace0 \uc9c0\uc6d0 \ud589\uc0ac\ub3c4 \ud558\uace0 \uc788\ub2e4. \uc774 \uc678\uc5d0\ub3c4 \u2018\uba85\uad6c\ud68c\ubc30\u2019\ub77c\ub294 \uc774\ub984\uc774 \ub4e4\uc5b4\uac04 \uc18c\ub144\uc57c\uad6c\ub300\ud68c\uc758 \uac1c\ucd5c \uc9c0\uc6d0\uacfc \uc57c\uad6c\uc6a9\ud488\uc774 \ubd80\uc871\ud55c \uc9c0\uc5ed\uc758 \uc5b4\ub9b0\uc774\ub4e4\uc5d0\uac8c \uc6a9\ud488\uc744 \uae30\uc99d\ud558\uace0 \uc788\uc73c\uba70 \ub610\ud55c \uadf8\ub77c\uc6b4\ub4dc \uc774\uc678\uc5d0\uc11c\ub3c4 \uc5b4\ub9b0\uc774 \ud559\uc0dd, \uc18c\ub144 \uc57c\uad6c\ud300, \uc18c\ub144 \uad50\ub3c4\uc18c \uc7ac\uc18c\uc790\ub4e4\uc5d0\uac8c \ud68c\uc6d0\uc774 \uc9c1\uc811 \uc790\uc2e0\uc758 \uacbd\ud5d8\uc744 \uac15\uc5f0\ud558\ub294 \ub4f1\uc758 \ud65c\ub3d9\ub3c4 \ud558\uace0 \uc788\ub2e4.", "sentence_answer": "\uc77c\ubcf8 \uc804\uad6d \uac01\uc9c0\uc5d0\uc11c \uc9c0\uc790\uccb4, \uad50\uc721\uc704\uc6d0\ud68c, \uc18c\ub144 \uc57c\uad6c\ud300, \uae30\uc5c5 \ub4f1\uc5d0\uc11c\uc758 \uc758\ub8b0\uc5d0 \ub530\ub974\uace0 \uc6b0\uc120 \uc57c\uad6c\uc5d0 \ud765\ubbf8\ub97c \uac16\uac8c\ud558\uae30 \uc704\ud574 \uc57c\uad6c\uc5d0 \ub300\ud55c \uce5c\uadfc\uac10\uc744 \ub290\ub07c\ub3c4\ub85d \uccb4\ud5d8\ud558\ub294 \u2018 \uba85\uad6c\ud68c \ubca0\uc774\uc2a4\ubcfc \ud398\uc2a4\ud2f0\ubc8c \u2019\uc744 \uac1c\ucd5c\ud558\uace0 \uc788\ub2e4.", "paragraph_id": "6548427-1-0", "paragraph_question": "question: \uba85\uad6c\ud68c\uc5d0\uc11c \uac1c\ucd5c\ud558\ub294 \uccb4\ud5d8 \ud589\uc0ac\uc758 \uba85\uce6d\uc740 \ubb34\uc5c7\uc778\uac00?, context: \uc77c\ubcf8 \uc804\uad6d \uac01\uc9c0\uc5d0\uc11c \uc9c0\uc790\uccb4, \uad50\uc721\uc704\uc6d0\ud68c, \uc18c\ub144 \uc57c\uad6c\ud300, \uae30\uc5c5 \ub4f1\uc5d0\uc11c\uc758 \uc758\ub8b0\uc5d0 \ub530\ub974\uace0 \uc6b0\uc120 \uc57c\uad6c\uc5d0 \ud765\ubbf8\ub97c \uac16\uac8c\ud558\uae30 \uc704\ud574 \uc57c\uad6c\uc5d0 \ub300\ud55c \uce5c\uadfc\uac10\uc744 \ub290\ub07c\ub3c4\ub85d \uccb4\ud5d8\ud558\ub294 \u2018\uba85\uad6c\ud68c \ubca0\uc774\uc2a4\ubcfc \ud398\uc2a4\ud2f0\ubc8c\u2019\uc744 \uac1c\ucd5c\ud558\uace0 \uc788\ub2e4. \uac70\uae30\uc5d0\uc120 \ud68c\uc6d0\uc774 \uccb4\ud5d8 \uc9c0\ub3c4\ud558\ub294 \ud22c\uad6c, \ud0c0\uaca9, \uc218\ube44\uc5d0 \ub300\ud574 \uc9c4\uc9c0\ud558\uac8c \ub9c8\uc8fc\ud558\ub294 \uc5b4\ub9b0\uc774\uc640 \ub354\ubd88\uc5b4 \uc751\uc6d0\ud558\ub7ec \uc628 \ud615\uc81c, \ubd80\ubaa8, \uc870\ubd80\ubaa8\uac00 \uac01\uc790 \uc2dc\ub300\uc758 \ud788\uc5b4\ub85c\uc640\uc758 \ub9cc\ub0a8\uc744 \ud1b5\ud574 \uc990\uae30\uac8c\ub054 \ub9cc\ub4e4\uace0 \uc788\ub2e4. \uadf8\ub9ac\uace0 \uc57c\uad6c\ub97c \uc798\ud558\uace0 \uc2f6\uc5b4\ud558\ub294 \uc5b4\ub9b0\uc774\uc5d0\uac8c\ub294 \ud68c\uc6d0\uc774 \uc9c1\uc811 \uc9c0\ub3c4\ud558\ub294 \u2018\uba85\uad6c\ud68c \uc57c\uad6c \uad50\uc2e4\u2019\ub3c4 \uc2e4\uc2dc\ud558\uace0 \uc788\ub2e4. \uc5ec\uae30\uc11c \ud22c\uad6c, \ud0c0\uaca9, \uc218\ube44 \uc804\ubb38\uac00\uac00 \uc5b4\ub9b0\uc774 \ud55c \uc0ac\ub78c \ud55c \uc0ac\ub78c\uc758 \uc18c\ub9ac\ub97c \ub4e3\uace0 \ucee4\ubba4\ub2c8\ucf00\uc774\uc158\uc744 \ucde8\ud558\uba74\uc11c \uc9c0\ub3c4\ud558\uace0 \uc788\ub2e4. \ucd5c\uadfc\uc5d0\ub294 \uae00\ub85c\ubc8c\ud654 \ub418\uba74\uc11c \ud574\uc678\uc5d0\uc11c\ub3c4 \uc57c\uad6c\ub97c \uc990\uae30\uba70 \ubcf4\uae09 \ud65c\ub3d9\uc744 \ud558\uace0 \uc788\ub294 \uc0ac\ub78c\ub4e4\uc758 \uc758\ub8b0\ub3c4 \uc788\uc5b4 \ud604\uc7ac \uc544\uc2dc\uc544 \uc5ec\ub7ec \ub098\ub77c(\ub300\ud55c\ubbfc\uad6d, \uc911\ud654\ubbfc\uad6d, \ud544\ub9ac\ud540)\uc758 \ud68c\uc6d0\ub4e4\uc744 \uc704\ud574 \ud604\uc9c0\uc758 \uc5b4\ub9b0\uc774\ub4e4\uc744 \uc9c0\ub3c4\ud558\uace0 \uc9c0\uc6d0 \ud589\uc0ac\ub3c4 \ud558\uace0 \uc788\ub2e4. \uc774 \uc678\uc5d0\ub3c4 \u2018\uba85\uad6c\ud68c\ubc30\u2019\ub77c\ub294 \uc774\ub984\uc774 \ub4e4\uc5b4\uac04 \uc18c\ub144\uc57c\uad6c\ub300\ud68c\uc758 \uac1c\ucd5c \uc9c0\uc6d0\uacfc \uc57c\uad6c\uc6a9\ud488\uc774 \ubd80\uc871\ud55c \uc9c0\uc5ed\uc758 \uc5b4\ub9b0\uc774\ub4e4\uc5d0\uac8c \uc6a9\ud488\uc744 \uae30\uc99d\ud558\uace0 \uc788\uc73c\uba70 \ub610\ud55c \uadf8\ub77c\uc6b4\ub4dc \uc774\uc678\uc5d0\uc11c\ub3c4 \uc5b4\ub9b0\uc774 \ud559\uc0dd, \uc18c\ub144 \uc57c\uad6c\ud300, \uc18c\ub144 \uad50\ub3c4\uc18c \uc7ac\uc18c\uc790\ub4e4\uc5d0\uac8c \ud68c\uc6d0\uc774 \uc9c1\uc811 \uc790\uc2e0\uc758 \uacbd\ud5d8\uc744 \uac15\uc5f0\ud558\ub294 \ub4f1\uc758 \ud65c\ub3d9\ub3c4 \ud558\uace0 \uc788\ub2e4."} {"question": "\uac19\uc774 \ud568\uaed8\ud55c\ub2e4\ub294 \ub73b\uc758 \ub2e8\uc5b4\ub294?", "paragraph": "\uadf8\ub7ec\ub098 \ub0b4\uac01\uc81c \uac1c\ud5cc \ubb38\uc81c\ub97c \ub450\uace0 \uc5ec\uad8c \ubd84\uc5f4\uc774 \uc788\uc5c8\uace0, \uc5ec\ub7ec \uc2a4\uce94\ub4e4\uc774 \ud130\uc838 \ubbfc\uc2ec \uc774\ubc18\uc774 \uc77c\uc5b4\ub0ac\ub2e4. \uc0c8\uc815\uce58\uad6d\ubbfc\ud68c\uc758\ub294 2000\ub144 \uc0c8\ucc9c\ub144\ubbfc\uc8fc\ub2f9\uc73c\ub85c \uac1c\ud3b8\ub418\uc5b4 \ud574\uccb4\ub418\uc5c8\uace0, \uc774\uc5b4 \ucd1d\uc120\uc5d0\uc11c \uc758\uc11d\uc218\ub294 \ub298\uc5b4\ub0ac\uc73c\ub098 \ud55c\ub098\ub77c\ub2f9\uc5d0 \ubc00\ub824 \ud328\ud588\ub2e4. \uc774\ud6c4 \uae40\ub300\uc911\uc740 \ub9ac\ub354\uc2ed\uc744 \uc783\uc5c8\uace0, \uacf5\ub3d9\uc5ec\ub2f9 \uacf5\uc870 \ud30c\uae30\uae4c\uc9c0 \uc77c\uc5b4\ub098\uba74\uc11c \uc815\uad8c\uc758 \uad6d\uc815 \uc6b4\uc601\uc740 \uc5b4\ub824\uc6c0\uc744 \uacaa\uac8c \ub418\uc5c8\ub2e4. \ubbfc\uc8fc\ub2f9\uc740 2002\ub144 \uc9c0\ubc29\uc120\uac70\uc5d0\uc11c \ud328\ud588\uace0, \uacb0\uad6d \uae40\ub300\uc911\uc740 \ud0c8\ub2f9\ud558\uc600\ub2e4(2000\ub144 \ucd1d\uc120 \uc804 \uc0c8\ucc9c\ub144\ubbfc\uc8fc\ub2f9 \ucc3d\ub2f9). \uadf8\ub7ec\ub098 \ub3d9\uad50\ub3d9\uacc4\ub294 \uc5ec\uc804\ud788 \ub2f9\uad8c\uc744 \uc7a5\uc545\ud558\uace0 \uc788\uc5c8\ub2e4. \ud558\uc9c0\ub9cc \uac19\uc740 \ud574 \ub300\uc120 \ud6c4\ubcf4\ub97c \uc120\ucd9c\ud558\uae30 \uc704\ud55c \uad6d\ubbfc \uacbd\uc120\uc81c\uac00 \ub3d9\uad50\ub3d9\uacc4\uc758 \uc808\ub300\uc801 \uad8c\uc704\uac00 \ud754\ub4e4\ub9ac\uae30 \uc2dc\uc791\ud558\ub294\ub370, \ub2f9\uc2dc '\ub178\ud48d'\uc758 \uacb0\uacfc\ub85c \ub178\ubb34\ud604\uc758 \uc9c0\uc9c0\uc728\uc774 \uae09\uc0c1\uc2b9\ud558\uace0 \ub3d9\uad50\ub3d9\uacc4 \ud6c4\ubcf4 \ud55c\ud654\uac11\uc740 \ub099\uc120\ud55c \uac83\uc774\ub2e4. \uacb0\uad6d, \ub300\uc120 \ud6c4\ubcf4\ub85c\ub294 \ub178\ubb34\ud604\uc774 \uc120\ucd9c\ub418\uc5c8\ub2e4. \ub300\uc120 \uacb0\uacfc, \ub178\ubb34\ud604\uc758 \ub2f9\uc120\uc73c\ub85c \uc0c8\ucc9c\ub144\ubbfc\uc8fc\ub2f9\uc740 \uc815\uad8c\uc744 \uc5f0\uc7a5\ud588\uc73c\ub098, \ub2f9\uc2dc \ub2f9\uad8c\uc744 \uc7a5\uc545\ud558\uace0 \uc788\uc5c8\ub358 \ub3d9\uad50\ub3d9\uacc4\uc640 \ub178\ubb34\ud604 \uce21 \uac04\uc758 \uad00\uacc4 \uc545\ud654\ub294 \uacc4\uc18d\ub418\uc5c8\uace0, \uc774 \uc640\uc911\uc5d0 \uc18c\uc704 '\uce5c\ub178'\ub77c\uace0 \ubd88\ub9ac\ub358 \uac1c\ud601\ud30c \ub2f9\uc6d0\ub4e4\uc774 \ud0c8\ub2f9\ud558\uc600\ub2e4. \uc774\ub4e4\uc740 \ud55c\ub098\ub77c\ub2f9 \ud0c8\ub2f9\ud30c \uc77c\ubd80, \uac1c\ud601\uad6d\ubbfc\uc815\ub2f9\uacfc \uc5f0\ub300\ud55c \ub4a4 \uc5f4\ub9b0\uc6b0\ub9ac\ub2f9\uc744 \ucc3d\ub2f9\ud588\ub2e4.", "answer": "\uc5f0\ub300", "sentence": "\uc774\ub4e4\uc740 \ud55c\ub098\ub77c\ub2f9 \ud0c8\ub2f9\ud30c \uc77c\ubd80, \uac1c\ud601\uad6d\ubbfc\uc815\ub2f9\uacfc \uc5f0\ub300 \ud55c \ub4a4 \uc5f4\ub9b0\uc6b0\ub9ac\ub2f9\uc744 \ucc3d\ub2f9\ud588\ub2e4.", "paragraph_sentence": "\uadf8\ub7ec\ub098 \ub0b4\uac01\uc81c \uac1c\ud5cc \ubb38\uc81c\ub97c \ub450\uace0 \uc5ec\uad8c \ubd84\uc5f4\uc774 \uc788\uc5c8\uace0, \uc5ec\ub7ec \uc2a4\uce94\ub4e4\uc774 \ud130\uc838 \ubbfc\uc2ec \uc774\ubc18\uc774 \uc77c\uc5b4\ub0ac\ub2e4. \uc0c8\uc815\uce58\uad6d\ubbfc\ud68c\uc758\ub294 2000\ub144 \uc0c8\ucc9c\ub144\ubbfc\uc8fc\ub2f9\uc73c\ub85c \uac1c\ud3b8\ub418\uc5b4 \ud574\uccb4\ub418\uc5c8\uace0, \uc774\uc5b4 \ucd1d\uc120\uc5d0\uc11c \uc758\uc11d\uc218\ub294 \ub298\uc5b4\ub0ac\uc73c\ub098 \ud55c\ub098\ub77c\ub2f9\uc5d0 \ubc00\ub824 \ud328\ud588\ub2e4. \uc774\ud6c4 \uae40\ub300\uc911\uc740 \ub9ac\ub354\uc2ed\uc744 \uc783\uc5c8\uace0, \uacf5\ub3d9\uc5ec\ub2f9 \uacf5\uc870 \ud30c\uae30\uae4c\uc9c0 \uc77c\uc5b4\ub098\uba74\uc11c \uc815\uad8c\uc758 \uad6d\uc815 \uc6b4\uc601\uc740 \uc5b4\ub824\uc6c0\uc744 \uacaa\uac8c \ub418\uc5c8\ub2e4. \ubbfc\uc8fc\ub2f9\uc740 2002\ub144 \uc9c0\ubc29\uc120\uac70\uc5d0\uc11c \ud328\ud588\uace0, \uacb0\uad6d \uae40\ub300\uc911\uc740 \ud0c8\ub2f9\ud558\uc600\ub2e4(2000\ub144 \ucd1d\uc120 \uc804 \uc0c8\ucc9c\ub144\ubbfc\uc8fc\ub2f9 \ucc3d\ub2f9). \uadf8\ub7ec\ub098 \ub3d9\uad50\ub3d9\uacc4\ub294 \uc5ec\uc804\ud788 \ub2f9\uad8c\uc744 \uc7a5\uc545\ud558\uace0 \uc788\uc5c8\ub2e4. \ud558\uc9c0\ub9cc \uac19\uc740 \ud574 \ub300\uc120 \ud6c4\ubcf4\ub97c \uc120\ucd9c\ud558\uae30 \uc704\ud55c \uad6d\ubbfc \uacbd\uc120\uc81c\uac00 \ub3d9\uad50\ub3d9\uacc4\uc758 \uc808\ub300\uc801 \uad8c\uc704\uac00 \ud754\ub4e4\ub9ac\uae30 \uc2dc\uc791\ud558\ub294\ub370, \ub2f9\uc2dc '\ub178\ud48d'\uc758 \uacb0\uacfc\ub85c \ub178\ubb34\ud604\uc758 \uc9c0\uc9c0\uc728\uc774 \uae09\uc0c1\uc2b9\ud558\uace0 \ub3d9\uad50\ub3d9\uacc4 \ud6c4\ubcf4 \ud55c\ud654\uac11\uc740 \ub099\uc120\ud55c \uac83\uc774\ub2e4. \uacb0\uad6d, \ub300\uc120 \ud6c4\ubcf4\ub85c\ub294 \ub178\ubb34\ud604\uc774 \uc120\ucd9c\ub418\uc5c8\ub2e4. \ub300\uc120 \uacb0\uacfc, \ub178\ubb34\ud604\uc758 \ub2f9\uc120\uc73c\ub85c \uc0c8\ucc9c\ub144\ubbfc\uc8fc\ub2f9\uc740 \uc815\uad8c\uc744 \uc5f0\uc7a5\ud588\uc73c\ub098, \ub2f9\uc2dc \ub2f9\uad8c\uc744 \uc7a5\uc545\ud558\uace0 \uc788\uc5c8\ub358 \ub3d9\uad50\ub3d9\uacc4\uc640 \ub178\ubb34\ud604 \uce21 \uac04\uc758 \uad00\uacc4 \uc545\ud654\ub294 \uacc4\uc18d\ub418\uc5c8\uace0, \uc774 \uc640\uc911\uc5d0 \uc18c\uc704 '\uce5c\ub178'\ub77c\uace0 \ubd88\ub9ac\ub358 \uac1c\ud601\ud30c \ub2f9\uc6d0\ub4e4\uc774 \ud0c8\ub2f9\ud558\uc600\ub2e4. \uc774\ub4e4\uc740 \ud55c\ub098\ub77c\ub2f9 \ud0c8\ub2f9\ud30c \uc77c\ubd80, \uac1c\ud601\uad6d\ubbfc\uc815\ub2f9\uacfc \uc5f0\ub300 \ud55c \ub4a4 \uc5f4\ub9b0\uc6b0\ub9ac\ub2f9\uc744 \ucc3d\ub2f9\ud588\ub2e4. ", "paragraph_answer": "\uadf8\ub7ec\ub098 \ub0b4\uac01\uc81c \uac1c\ud5cc \ubb38\uc81c\ub97c \ub450\uace0 \uc5ec\uad8c \ubd84\uc5f4\uc774 \uc788\uc5c8\uace0, \uc5ec\ub7ec \uc2a4\uce94\ub4e4\uc774 \ud130\uc838 \ubbfc\uc2ec \uc774\ubc18\uc774 \uc77c\uc5b4\ub0ac\ub2e4. \uc0c8\uc815\uce58\uad6d\ubbfc\ud68c\uc758\ub294 2000\ub144 \uc0c8\ucc9c\ub144\ubbfc\uc8fc\ub2f9\uc73c\ub85c \uac1c\ud3b8\ub418\uc5b4 \ud574\uccb4\ub418\uc5c8\uace0, \uc774\uc5b4 \ucd1d\uc120\uc5d0\uc11c \uc758\uc11d\uc218\ub294 \ub298\uc5b4\ub0ac\uc73c\ub098 \ud55c\ub098\ub77c\ub2f9\uc5d0 \ubc00\ub824 \ud328\ud588\ub2e4. \uc774\ud6c4 \uae40\ub300\uc911\uc740 \ub9ac\ub354\uc2ed\uc744 \uc783\uc5c8\uace0, \uacf5\ub3d9\uc5ec\ub2f9 \uacf5\uc870 \ud30c\uae30\uae4c\uc9c0 \uc77c\uc5b4\ub098\uba74\uc11c \uc815\uad8c\uc758 \uad6d\uc815 \uc6b4\uc601\uc740 \uc5b4\ub824\uc6c0\uc744 \uacaa\uac8c \ub418\uc5c8\ub2e4. \ubbfc\uc8fc\ub2f9\uc740 2002\ub144 \uc9c0\ubc29\uc120\uac70\uc5d0\uc11c \ud328\ud588\uace0, \uacb0\uad6d \uae40\ub300\uc911\uc740 \ud0c8\ub2f9\ud558\uc600\ub2e4(2000\ub144 \ucd1d\uc120 \uc804 \uc0c8\ucc9c\ub144\ubbfc\uc8fc\ub2f9 \ucc3d\ub2f9). \uadf8\ub7ec\ub098 \ub3d9\uad50\ub3d9\uacc4\ub294 \uc5ec\uc804\ud788 \ub2f9\uad8c\uc744 \uc7a5\uc545\ud558\uace0 \uc788\uc5c8\ub2e4. \ud558\uc9c0\ub9cc \uac19\uc740 \ud574 \ub300\uc120 \ud6c4\ubcf4\ub97c \uc120\ucd9c\ud558\uae30 \uc704\ud55c \uad6d\ubbfc \uacbd\uc120\uc81c\uac00 \ub3d9\uad50\ub3d9\uacc4\uc758 \uc808\ub300\uc801 \uad8c\uc704\uac00 \ud754\ub4e4\ub9ac\uae30 \uc2dc\uc791\ud558\ub294\ub370, \ub2f9\uc2dc '\ub178\ud48d'\uc758 \uacb0\uacfc\ub85c \ub178\ubb34\ud604\uc758 \uc9c0\uc9c0\uc728\uc774 \uae09\uc0c1\uc2b9\ud558\uace0 \ub3d9\uad50\ub3d9\uacc4 \ud6c4\ubcf4 \ud55c\ud654\uac11\uc740 \ub099\uc120\ud55c \uac83\uc774\ub2e4. \uacb0\uad6d, \ub300\uc120 \ud6c4\ubcf4\ub85c\ub294 \ub178\ubb34\ud604\uc774 \uc120\ucd9c\ub418\uc5c8\ub2e4. \ub300\uc120 \uacb0\uacfc, \ub178\ubb34\ud604\uc758 \ub2f9\uc120\uc73c\ub85c \uc0c8\ucc9c\ub144\ubbfc\uc8fc\ub2f9\uc740 \uc815\uad8c\uc744 \uc5f0\uc7a5\ud588\uc73c\ub098, \ub2f9\uc2dc \ub2f9\uad8c\uc744 \uc7a5\uc545\ud558\uace0 \uc788\uc5c8\ub358 \ub3d9\uad50\ub3d9\uacc4\uc640 \ub178\ubb34\ud604 \uce21 \uac04\uc758 \uad00\uacc4 \uc545\ud654\ub294 \uacc4\uc18d\ub418\uc5c8\uace0, \uc774 \uc640\uc911\uc5d0 \uc18c\uc704 '\uce5c\ub178'\ub77c\uace0 \ubd88\ub9ac\ub358 \uac1c\ud601\ud30c \ub2f9\uc6d0\ub4e4\uc774 \ud0c8\ub2f9\ud558\uc600\ub2e4. \uc774\ub4e4\uc740 \ud55c\ub098\ub77c\ub2f9 \ud0c8\ub2f9\ud30c \uc77c\ubd80, \uac1c\ud601\uad6d\ubbfc\uc815\ub2f9\uacfc \uc5f0\ub300 \ud55c \ub4a4 \uc5f4\ub9b0\uc6b0\ub9ac\ub2f9\uc744 \ucc3d\ub2f9\ud588\ub2e4.", "sentence_answer": "\uc774\ub4e4\uc740 \ud55c\ub098\ub77c\ub2f9 \ud0c8\ub2f9\ud30c \uc77c\ubd80, \uac1c\ud601\uad6d\ubbfc\uc815\ub2f9\uacfc \uc5f0\ub300 \ud55c \ub4a4 \uc5f4\ub9b0\uc6b0\ub9ac\ub2f9\uc744 \ucc3d\ub2f9\ud588\ub2e4.", "paragraph_id": "6512653-3-2", "paragraph_question": "question: \uac19\uc774 \ud568\uaed8\ud55c\ub2e4\ub294 \ub73b\uc758 \ub2e8\uc5b4\ub294?, context: \uadf8\ub7ec\ub098 \ub0b4\uac01\uc81c \uac1c\ud5cc \ubb38\uc81c\ub97c \ub450\uace0 \uc5ec\uad8c \ubd84\uc5f4\uc774 \uc788\uc5c8\uace0, \uc5ec\ub7ec \uc2a4\uce94\ub4e4\uc774 \ud130\uc838 \ubbfc\uc2ec \uc774\ubc18\uc774 \uc77c\uc5b4\ub0ac\ub2e4. \uc0c8\uc815\uce58\uad6d\ubbfc\ud68c\uc758\ub294 2000\ub144 \uc0c8\ucc9c\ub144\ubbfc\uc8fc\ub2f9\uc73c\ub85c \uac1c\ud3b8\ub418\uc5b4 \ud574\uccb4\ub418\uc5c8\uace0, \uc774\uc5b4 \ucd1d\uc120\uc5d0\uc11c \uc758\uc11d\uc218\ub294 \ub298\uc5b4\ub0ac\uc73c\ub098 \ud55c\ub098\ub77c\ub2f9\uc5d0 \ubc00\ub824 \ud328\ud588\ub2e4. \uc774\ud6c4 \uae40\ub300\uc911\uc740 \ub9ac\ub354\uc2ed\uc744 \uc783\uc5c8\uace0, \uacf5\ub3d9\uc5ec\ub2f9 \uacf5\uc870 \ud30c\uae30\uae4c\uc9c0 \uc77c\uc5b4\ub098\uba74\uc11c \uc815\uad8c\uc758 \uad6d\uc815 \uc6b4\uc601\uc740 \uc5b4\ub824\uc6c0\uc744 \uacaa\uac8c \ub418\uc5c8\ub2e4. \ubbfc\uc8fc\ub2f9\uc740 2002\ub144 \uc9c0\ubc29\uc120\uac70\uc5d0\uc11c \ud328\ud588\uace0, \uacb0\uad6d \uae40\ub300\uc911\uc740 \ud0c8\ub2f9\ud558\uc600\ub2e4(2000\ub144 \ucd1d\uc120 \uc804 \uc0c8\ucc9c\ub144\ubbfc\uc8fc\ub2f9 \ucc3d\ub2f9). \uadf8\ub7ec\ub098 \ub3d9\uad50\ub3d9\uacc4\ub294 \uc5ec\uc804\ud788 \ub2f9\uad8c\uc744 \uc7a5\uc545\ud558\uace0 \uc788\uc5c8\ub2e4. \ud558\uc9c0\ub9cc \uac19\uc740 \ud574 \ub300\uc120 \ud6c4\ubcf4\ub97c \uc120\ucd9c\ud558\uae30 \uc704\ud55c \uad6d\ubbfc \uacbd\uc120\uc81c\uac00 \ub3d9\uad50\ub3d9\uacc4\uc758 \uc808\ub300\uc801 \uad8c\uc704\uac00 \ud754\ub4e4\ub9ac\uae30 \uc2dc\uc791\ud558\ub294\ub370, \ub2f9\uc2dc '\ub178\ud48d'\uc758 \uacb0\uacfc\ub85c \ub178\ubb34\ud604\uc758 \uc9c0\uc9c0\uc728\uc774 \uae09\uc0c1\uc2b9\ud558\uace0 \ub3d9\uad50\ub3d9\uacc4 \ud6c4\ubcf4 \ud55c\ud654\uac11\uc740 \ub099\uc120\ud55c \uac83\uc774\ub2e4. \uacb0\uad6d, \ub300\uc120 \ud6c4\ubcf4\ub85c\ub294 \ub178\ubb34\ud604\uc774 \uc120\ucd9c\ub418\uc5c8\ub2e4. \ub300\uc120 \uacb0\uacfc, \ub178\ubb34\ud604\uc758 \ub2f9\uc120\uc73c\ub85c \uc0c8\ucc9c\ub144\ubbfc\uc8fc\ub2f9\uc740 \uc815\uad8c\uc744 \uc5f0\uc7a5\ud588\uc73c\ub098, \ub2f9\uc2dc \ub2f9\uad8c\uc744 \uc7a5\uc545\ud558\uace0 \uc788\uc5c8\ub358 \ub3d9\uad50\ub3d9\uacc4\uc640 \ub178\ubb34\ud604 \uce21 \uac04\uc758 \uad00\uacc4 \uc545\ud654\ub294 \uacc4\uc18d\ub418\uc5c8\uace0, \uc774 \uc640\uc911\uc5d0 \uc18c\uc704 '\uce5c\ub178'\ub77c\uace0 \ubd88\ub9ac\ub358 \uac1c\ud601\ud30c \ub2f9\uc6d0\ub4e4\uc774 \ud0c8\ub2f9\ud558\uc600\ub2e4. \uc774\ub4e4\uc740 \ud55c\ub098\ub77c\ub2f9 \ud0c8\ub2f9\ud30c \uc77c\ubd80, \uac1c\ud601\uad6d\ubbfc\uc815\ub2f9\uacfc \uc5f0\ub300\ud55c \ub4a4 \uc5f4\ub9b0\uc6b0\ub9ac\ub2f9\uc744 \ucc3d\ub2f9\ud588\ub2e4."} {"question": "\ub300\ud55c\ubbfc\uad6d\uc5d0\uc11c \uac00\uc7a5 \ub192\uc740 \uac74\ubb3c\uc778 63\ube4c\ub529\uc774 \uac1c\uc7a5\ud55c \ub144\ub3c4\ub294?", "paragraph": "1969\ub144\uc5d0 KAL \ube4c\ub529(16\uce35, 82m)\uac00 \uc644\uacf5\ub418\uace0 1970\ub144\uc5d0 \uc815\ubd80\uc885\ud569\uccad\uc0ac(19\uce35, 94m)\uac00 \uc644\uacf5\ub418\uace0 1971\ub144\uc5d0\ub294 \uc0bc\uc77c\ube4c\ub529(31\uce35, 114m)\uc774 \uc644\uacf5\ub418\uc5c8\ub2e4. 63\ube4c\ub529(60\uce35, 249m)\uc740 1980\ub144\uc5d0 \ucc29\uacf5\ud558\uc5ec 1983\ub144\uc5d0 \uc644\uacf5\ub41c \ud6c4 1985\ub144\uc5d0 \uac1c\uc7a5\ub418\uc5b4 31\ube4c\ub529(31\uce35, 114m)\uc744 \uc81c\uce58\uace0 \ub300\ud55c\ubbfc\uad6d\uc5d0\uc11c \uac00\uc7a5 \ub192\uc740 \uac74\ubb3c\uc774 \ub418\uc5c8\ub2e4. 1988\ub144\uc5d0\ub294 \ud2b8\ub808\uc774\ub4dc \ud0c0\uc6cc(54\uce35, 229m), 2004\ub144\uc5d0\ub294 \uc0bc\uc131 \ud0c0\uc6cc\ud330\ub9ac\uc2a4 3\ucc28 - G\ub3d9(G\ub3d9 73\uce35, 264m), 2003\ub144\uc5d0 \uc644\uacf5\ub41c \ubaa9\ub3d9 \ud558\uc774\ud398\ub9ac\uc628(101\ub3d9 69\uce35, 256m)\uc740 2003\ub144\ubd80\ud130 2004\ub144\uae4c\uc9c0 \ucd5c\uace0\uce35 \uac74\ubb3c\uc774\uc600\ub2e4. 2011\ub144\uc5d0\ub294 \ud574\uc6b4\ub300 \ub450\uc0b0\uc704\ube0c\ub354\uc81c\ub2c8\uc2a4(101\ub3d9 80\uce35, 301m)\uac00 \uc644\uacf5\ub418\uace0 \ud3ec\uc2a4\ucf54\ud0c0\uc6cc-\uc1a1\ub3c4(69\uce35, 305m)\ub294 \uc778\ucc9c\uad11\uc5ed\uc2dc \uc5f0\uc218\uad6c \uc1a1\ub3c4\uad6d\uc81c\ub3c4\uc2dc\uc5d0 2006\ub144\uc5d0 \ucc29\uacf5\ud558\uc5ec 2011\ub144\uc5d0 \uc678\ubd80\uacf5\uc0ac\uac00 \uc644\ub8cc\ub41c \ud6c4 2014\ub144\uc5d0 \ub0b4\ubd80\uacf5\uc0ac\uac00 \uc644\ub8cc\ub418\uc5b4 \uac1c\uc7a5\ud558\uc600\ub2e4.", "answer": "1985\ub144", "sentence": "63\ube4c\ub529(60\uce35, 249m)\uc740 1980\ub144\uc5d0 \ucc29\uacf5\ud558\uc5ec 1983\ub144\uc5d0 \uc644\uacf5\ub41c \ud6c4 1985\ub144 \uc5d0 \uac1c\uc7a5\ub418\uc5b4 31\ube4c\ub529(31\uce35, 114m)\uc744 \uc81c\uce58\uace0 \ub300\ud55c\ubbfc\uad6d\uc5d0\uc11c \uac00\uc7a5 \ub192\uc740 \uac74\ubb3c\uc774 \ub418\uc5c8\ub2e4.", "paragraph_sentence": "1969\ub144\uc5d0 KAL \ube4c\ub529(16\uce35, 82m)\uac00 \uc644\uacf5\ub418\uace0 1970\ub144\uc5d0 \uc815\ubd80\uc885\ud569\uccad\uc0ac(19\uce35, 94m)\uac00 \uc644\uacf5\ub418\uace0 1971\ub144\uc5d0\ub294 \uc0bc\uc77c\ube4c\ub529(31\uce35, 114m)\uc774 \uc644\uacf5\ub418\uc5c8\ub2e4. 63\ube4c\ub529(60\uce35, 249m)\uc740 1980\ub144\uc5d0 \ucc29\uacf5\ud558\uc5ec 1983\ub144\uc5d0 \uc644\uacf5\ub41c \ud6c4 1985\ub144 \uc5d0 \uac1c\uc7a5\ub418\uc5b4 31\ube4c\ub529(31\uce35, 114m)\uc744 \uc81c\uce58\uace0 \ub300\ud55c\ubbfc\uad6d\uc5d0\uc11c \uac00\uc7a5 \ub192\uc740 \uac74\ubb3c\uc774 \ub418\uc5c8\ub2e4. 1988\ub144\uc5d0\ub294 \ud2b8\ub808\uc774\ub4dc \ud0c0\uc6cc(54\uce35, 229m), 2004\ub144\uc5d0\ub294 \uc0bc\uc131 \ud0c0\uc6cc\ud330\ub9ac\uc2a4 3\ucc28 - G\ub3d9(G\ub3d9 73\uce35, 264m), 2003\ub144\uc5d0 \uc644\uacf5\ub41c \ubaa9\ub3d9 \ud558\uc774\ud398\ub9ac\uc628(101\ub3d9 69\uce35, 256m)\uc740 2003\ub144\ubd80\ud130 2004\ub144\uae4c\uc9c0 \ucd5c\uace0\uce35 \uac74\ubb3c\uc774\uc600\ub2e4. 2011\ub144\uc5d0\ub294 \ud574\uc6b4\ub300 \ub450\uc0b0\uc704\ube0c\ub354\uc81c\ub2c8\uc2a4(101\ub3d9 80\uce35, 301m)\uac00 \uc644\uacf5\ub418\uace0 \ud3ec\uc2a4\ucf54\ud0c0\uc6cc-\uc1a1\ub3c4(69\uce35, 305m)\ub294 \uc778\ucc9c\uad11\uc5ed\uc2dc \uc5f0\uc218\uad6c \uc1a1\ub3c4\uad6d\uc81c\ub3c4\uc2dc\uc5d0 2006\ub144\uc5d0 \ucc29\uacf5\ud558\uc5ec 2011\ub144\uc5d0 \uc678\ubd80\uacf5\uc0ac\uac00 \uc644\ub8cc\ub41c \ud6c4 2014\ub144\uc5d0 \ub0b4\ubd80\uacf5\uc0ac\uac00 \uc644\ub8cc\ub418\uc5b4 \uac1c\uc7a5\ud558\uc600\ub2e4.", "paragraph_answer": "1969\ub144\uc5d0 KAL \ube4c\ub529(16\uce35, 82m)\uac00 \uc644\uacf5\ub418\uace0 1970\ub144\uc5d0 \uc815\ubd80\uc885\ud569\uccad\uc0ac(19\uce35, 94m)\uac00 \uc644\uacf5\ub418\uace0 1971\ub144\uc5d0\ub294 \uc0bc\uc77c\ube4c\ub529(31\uce35, 114m)\uc774 \uc644\uacf5\ub418\uc5c8\ub2e4. 63\ube4c\ub529(60\uce35, 249m)\uc740 1980\ub144\uc5d0 \ucc29\uacf5\ud558\uc5ec 1983\ub144\uc5d0 \uc644\uacf5\ub41c \ud6c4 1985\ub144 \uc5d0 \uac1c\uc7a5\ub418\uc5b4 31\ube4c\ub529(31\uce35, 114m)\uc744 \uc81c\uce58\uace0 \ub300\ud55c\ubbfc\uad6d\uc5d0\uc11c \uac00\uc7a5 \ub192\uc740 \uac74\ubb3c\uc774 \ub418\uc5c8\ub2e4. 1988\ub144\uc5d0\ub294 \ud2b8\ub808\uc774\ub4dc \ud0c0\uc6cc(54\uce35, 229m), 2004\ub144\uc5d0\ub294 \uc0bc\uc131 \ud0c0\uc6cc\ud330\ub9ac\uc2a4 3\ucc28 - G\ub3d9(G\ub3d9 73\uce35, 264m), 2003\ub144\uc5d0 \uc644\uacf5\ub41c \ubaa9\ub3d9 \ud558\uc774\ud398\ub9ac\uc628(101\ub3d9 69\uce35, 256m)\uc740 2003\ub144\ubd80\ud130 2004\ub144\uae4c\uc9c0 \ucd5c\uace0\uce35 \uac74\ubb3c\uc774\uc600\ub2e4. 2011\ub144\uc5d0\ub294 \ud574\uc6b4\ub300 \ub450\uc0b0\uc704\ube0c\ub354\uc81c\ub2c8\uc2a4(101\ub3d9 80\uce35, 301m)\uac00 \uc644\uacf5\ub418\uace0 \ud3ec\uc2a4\ucf54\ud0c0\uc6cc-\uc1a1\ub3c4(69\uce35, 305m)\ub294 \uc778\ucc9c\uad11\uc5ed\uc2dc \uc5f0\uc218\uad6c \uc1a1\ub3c4\uad6d\uc81c\ub3c4\uc2dc\uc5d0 2006\ub144\uc5d0 \ucc29\uacf5\ud558\uc5ec 2011\ub144\uc5d0 \uc678\ubd80\uacf5\uc0ac\uac00 \uc644\ub8cc\ub41c \ud6c4 2014\ub144\uc5d0 \ub0b4\ubd80\uacf5\uc0ac\uac00 \uc644\ub8cc\ub418\uc5b4 \uac1c\uc7a5\ud558\uc600\ub2e4.", "sentence_answer": "63\ube4c\ub529(60\uce35, 249m)\uc740 1980\ub144\uc5d0 \ucc29\uacf5\ud558\uc5ec 1983\ub144\uc5d0 \uc644\uacf5\ub41c \ud6c4 1985\ub144 \uc5d0 \uac1c\uc7a5\ub418\uc5b4 31\ube4c\ub529(31\uce35, 114m)\uc744 \uc81c\uce58\uace0 \ub300\ud55c\ubbfc\uad6d\uc5d0\uc11c \uac00\uc7a5 \ub192\uc740 \uac74\ubb3c\uc774 \ub418\uc5c8\ub2e4.", "paragraph_id": "6546255-2-0", "paragraph_question": "question: \ub300\ud55c\ubbfc\uad6d\uc5d0\uc11c \uac00\uc7a5 \ub192\uc740 \uac74\ubb3c\uc778 63\ube4c\ub529\uc774 \uac1c\uc7a5\ud55c \ub144\ub3c4\ub294?, context: 1969\ub144\uc5d0 KAL \ube4c\ub529(16\uce35, 82m)\uac00 \uc644\uacf5\ub418\uace0 1970\ub144\uc5d0 \uc815\ubd80\uc885\ud569\uccad\uc0ac(19\uce35, 94m)\uac00 \uc644\uacf5\ub418\uace0 1971\ub144\uc5d0\ub294 \uc0bc\uc77c\ube4c\ub529(31\uce35, 114m)\uc774 \uc644\uacf5\ub418\uc5c8\ub2e4. 63\ube4c\ub529(60\uce35, 249m)\uc740 1980\ub144\uc5d0 \ucc29\uacf5\ud558\uc5ec 1983\ub144\uc5d0 \uc644\uacf5\ub41c \ud6c4 1985\ub144\uc5d0 \uac1c\uc7a5\ub418\uc5b4 31\ube4c\ub529(31\uce35, 114m)\uc744 \uc81c\uce58\uace0 \ub300\ud55c\ubbfc\uad6d\uc5d0\uc11c \uac00\uc7a5 \ub192\uc740 \uac74\ubb3c\uc774 \ub418\uc5c8\ub2e4. 1988\ub144\uc5d0\ub294 \ud2b8\ub808\uc774\ub4dc \ud0c0\uc6cc(54\uce35, 229m), 2004\ub144\uc5d0\ub294 \uc0bc\uc131 \ud0c0\uc6cc\ud330\ub9ac\uc2a4 3\ucc28 - G\ub3d9(G\ub3d9 73\uce35, 264m), 2003\ub144\uc5d0 \uc644\uacf5\ub41c \ubaa9\ub3d9 \ud558\uc774\ud398\ub9ac\uc628(101\ub3d9 69\uce35, 256m)\uc740 2003\ub144\ubd80\ud130 2004\ub144\uae4c\uc9c0 \ucd5c\uace0\uce35 \uac74\ubb3c\uc774\uc600\ub2e4. 2011\ub144\uc5d0\ub294 \ud574\uc6b4\ub300 \ub450\uc0b0\uc704\ube0c\ub354\uc81c\ub2c8\uc2a4(101\ub3d9 80\uce35, 301m)\uac00 \uc644\uacf5\ub418\uace0 \ud3ec\uc2a4\ucf54\ud0c0\uc6cc-\uc1a1\ub3c4(69\uce35, 305m)\ub294 \uc778\ucc9c\uad11\uc5ed\uc2dc \uc5f0\uc218\uad6c \uc1a1\ub3c4\uad6d\uc81c\ub3c4\uc2dc\uc5d0 2006\ub144\uc5d0 \ucc29\uacf5\ud558\uc5ec 2011\ub144\uc5d0 \uc678\ubd80\uacf5\uc0ac\uac00 \uc644\ub8cc\ub41c \ud6c4 2014\ub144\uc5d0 \ub0b4\ubd80\uacf5\uc0ac\uac00 \uc644\ub8cc\ub418\uc5b4 \uac1c\uc7a5\ud558\uc600\ub2e4."} {"question": "\uc548\uc0b0\uc2dc\uc758 \uacf5\uc5c5 \uc704\uc131\ub3c4\uc2dc\ud654 \uac00\uc18d\uc73c\ub85c 1976\ub144 \uc548\uc0b0 \uc77c\ub300\uc5d0 \ubc1c\ud45c\ub41c \uac74\uc124\uc0ac\uc5c5\uc740?", "paragraph": "\ud604\uc7ac \uc548\uc0b0\uc2dc\uc758 \uc77c\ubd80\ub85c \uad00\ub9ac\ub418\uace0 \uc788\ub294 \ud48d\ub3c4\uc5d0\uc11c\ub294 \uc870\uc120 \ub9d0 \uccad\uc77c \uc804\uc7c1\uc5d0\uc11c \uc77c\ubcf8\uacfc \uccad\ub098\ub77c \uac04\uc5d0 \ud48d\ub3c4\ud574\uc804\uc774 \uc788\uc5c8\ub2e4. \uc77c\uc81c \uac15\uc810\uae30\uc5d0\ub294 1914\ub144 \ud589\uc815\uad6c\uc5ed \uac1c\ud3b8\uc5d0 \ub530\ub77c \uc2dc\ud765\uad70 \uc218\uc554\uba74, \uad70\uc790\uba74\uc73c\ub85c \uc804\ud658\ub418\uc5c8\uc73c\uba70, \uc77c\ubd80 \uc9c0\uc5ed(\ubc18\uc6d4\uba74)\uc744 \uc218\uc6d0\uad70\uc5d0 \uc591\ub3c4\ud558\uc600\ub2e4. 1937\ub144\uc5d0\ub294 \uc548\uc0b0\uc744 \uad00\ud1b5\ud558\uc5ec \uc218\uc6d0\uacfc \uc778\ucc9c \uac04\uc744 \uc5f0\uacb0\ud558\ub294 \uc218\uc778\uc120 \ud611\uada4 \ucca0\ub3c4\uac00 \uc6b4\ud589\ud558\uae30 \uc2dc\uc791\ud558\uc600\ub2e4. \uadf8\ub7ec\ub098 1972\ub144 \ub300\ud55c\ubbfc\uad6d \uc815\ubd80\uc758 \ucca0\ub3c4 \ub178\uc120 \uc815\ub9ac\uc640 1988\ub144 \uc548\uc0b0\uc120 \uc804\ucca0 \uac1c\ud1b5 \uc774\ud6c4 \uc6b4\uc1a1\uc774 \uae09\uac10\ud558\ub2e4\uac00, 1995\ub144 12\uc6d4 31\uc77c\uc744 \ub05d\uc73c\ub85c \uc6b4\ud589\uc744 \uc911\ub2e8\ud558\uac8c \ub418\uc5c8\ub2e4. \ud55c\ud3b8\uc73c\ub85c 1970\ub144\ub300 \ud6c4\ubc18\ubd80\ud130 \uc548\uc0b0\uc2dc\uc758 \uacf5\uc5c5 \uc704\uc131\ub3c4\uc2dc\ud654\uac00 \uac00\uc18d\ub418\uae30 \uc2dc\uc791\ud558\uc5ec, 1976\ub144 12\uc6d4 \uc2dc\ud765\uad70\uacfc \ud654\uc131\uad70\uc758 \uc77c\ubd80\uc600\ub358 \uc548\uc0b0 \uc77c\ub300\uc5d0 \u2018\ubc18\uc6d4\uc2e0\uacf5\uc5c5\ub3c4\uc2dc \uac74\uc124\uacc4\ud68d\u2019\uc744 \ubc1c\ud45c\ud558\uc600\uc73c\uba70, 1979\ub144 8\uc6d4 31\uc77c\uc5d0 \uacbd\uae30\ub3c4 \ubc18\uc6d4\ucd9c\uc7a5\uc18c\ub97c \uc124\uce58\ud558\uc600\uace0, 1986\ub144 1\uc6d4 1\uc77c\uc5d0\ub294 \ucd9c\uc7a5\uc18c\uc5d0\uc11c \uc9c0\uae08\uc758 \uc548\uc0b0\uc2dc\ub85c \uc2b9\uaca9\ub418\uc5c8\ub2e4. 2002\ub144\uc5d0\ub294 \uc0c1\ub85d\uad6c\uc640 \ub2e8\uc6d0\uad6c \ub4f1 2\uac1c \uad6c\ub97c \uc0c8\ub85c \uc124\uce58\ud558\uba74\uc11c \uad6c\uc81c(\u5340\u5236)\ub97c \uc2e4\uc2dc\ud558\uc600\ub2e4.", "answer": "\ubc18\uc6d4\uc2e0\uacf5\uc5c5\ub3c4\uc2dc \uac74\uc124\uacc4\ud68d", "sentence": "\ud55c\ud3b8\uc73c\ub85c 1970\ub144\ub300 \ud6c4\ubc18\ubd80\ud130 \uc548\uc0b0\uc2dc\uc758 \uacf5\uc5c5 \uc704\uc131\ub3c4\uc2dc\ud654\uac00 \uac00\uc18d\ub418\uae30 \uc2dc\uc791\ud558\uc5ec, 1976\ub144 12\uc6d4 \uc2dc\ud765\uad70\uacfc \ud654\uc131\uad70\uc758 \uc77c\ubd80\uc600\ub358 \uc548\uc0b0 \uc77c\ub300\uc5d0 \u2018 \ubc18\uc6d4\uc2e0\uacf5\uc5c5\ub3c4\uc2dc \uac74\uc124\uacc4\ud68d \u2019\uc744 \ubc1c\ud45c\ud558\uc600\uc73c\uba70, 1979\ub144 8\uc6d4 31\uc77c\uc5d0 \uacbd\uae30\ub3c4 \ubc18\uc6d4\ucd9c\uc7a5\uc18c\ub97c \uc124\uce58\ud558\uc600\uace0, 1986\ub144 1\uc6d4 1\uc77c\uc5d0\ub294 \ucd9c\uc7a5\uc18c\uc5d0\uc11c \uc9c0\uae08\uc758 \uc548\uc0b0\uc2dc\ub85c \uc2b9\uaca9\ub418\uc5c8\ub2e4.", "paragraph_sentence": "\ud604\uc7ac \uc548\uc0b0\uc2dc\uc758 \uc77c\ubd80\ub85c \uad00\ub9ac\ub418\uace0 \uc788\ub294 \ud48d\ub3c4\uc5d0\uc11c\ub294 \uc870\uc120 \ub9d0 \uccad\uc77c \uc804\uc7c1\uc5d0\uc11c \uc77c\ubcf8\uacfc \uccad\ub098\ub77c \uac04\uc5d0 \ud48d\ub3c4\ud574\uc804\uc774 \uc788\uc5c8\ub2e4. \uc77c\uc81c \uac15\uc810\uae30\uc5d0\ub294 1914\ub144 \ud589\uc815\uad6c\uc5ed \uac1c\ud3b8\uc5d0 \ub530\ub77c \uc2dc\ud765\uad70 \uc218\uc554\uba74, \uad70\uc790\uba74\uc73c\ub85c \uc804\ud658\ub418\uc5c8\uc73c\uba70, \uc77c\ubd80 \uc9c0\uc5ed(\ubc18\uc6d4\uba74)\uc744 \uc218\uc6d0\uad70\uc5d0 \uc591\ub3c4\ud558\uc600\ub2e4. 1937\ub144\uc5d0\ub294 \uc548\uc0b0\uc744 \uad00\ud1b5\ud558\uc5ec \uc218\uc6d0\uacfc \uc778\ucc9c \uac04\uc744 \uc5f0\uacb0\ud558\ub294 \uc218\uc778\uc120 \ud611\uada4 \ucca0\ub3c4\uac00 \uc6b4\ud589\ud558\uae30 \uc2dc\uc791\ud558\uc600\ub2e4. \uadf8\ub7ec\ub098 1972\ub144 \ub300\ud55c\ubbfc\uad6d \uc815\ubd80\uc758 \ucca0\ub3c4 \ub178\uc120 \uc815\ub9ac\uc640 1988\ub144 \uc548\uc0b0\uc120 \uc804\ucca0 \uac1c\ud1b5 \uc774\ud6c4 \uc6b4\uc1a1\uc774 \uae09\uac10\ud558\ub2e4\uac00, 1995\ub144 12\uc6d4 31\uc77c\uc744 \ub05d\uc73c\ub85c \uc6b4\ud589\uc744 \uc911\ub2e8\ud558\uac8c \ub418\uc5c8\ub2e4. \ud55c\ud3b8\uc73c\ub85c 1970\ub144\ub300 \ud6c4\ubc18\ubd80\ud130 \uc548\uc0b0\uc2dc\uc758 \uacf5\uc5c5 \uc704\uc131\ub3c4\uc2dc\ud654\uac00 \uac00\uc18d\ub418\uae30 \uc2dc\uc791\ud558\uc5ec, 1976\ub144 12\uc6d4 \uc2dc\ud765\uad70\uacfc \ud654\uc131\uad70\uc758 \uc77c\ubd80\uc600\ub358 \uc548\uc0b0 \uc77c\ub300\uc5d0 \u2018 \ubc18\uc6d4\uc2e0\uacf5\uc5c5\ub3c4\uc2dc \uac74\uc124\uacc4\ud68d \u2019\uc744 \ubc1c\ud45c\ud558\uc600\uc73c\uba70, 1979\ub144 8\uc6d4 31\uc77c\uc5d0 \uacbd\uae30\ub3c4 \ubc18\uc6d4\ucd9c\uc7a5\uc18c\ub97c \uc124\uce58\ud558\uc600\uace0, 1986\ub144 1\uc6d4 1\uc77c\uc5d0\ub294 \ucd9c\uc7a5\uc18c\uc5d0\uc11c \uc9c0\uae08\uc758 \uc548\uc0b0\uc2dc\ub85c \uc2b9\uaca9\ub418\uc5c8\ub2e4. 2002\ub144\uc5d0\ub294 \uc0c1\ub85d\uad6c\uc640 \ub2e8\uc6d0\uad6c \ub4f1 2\uac1c \uad6c\ub97c \uc0c8\ub85c \uc124\uce58\ud558\uba74\uc11c \uad6c\uc81c(\u5340\u5236)\ub97c \uc2e4\uc2dc\ud558\uc600\ub2e4.", "paragraph_answer": "\ud604\uc7ac \uc548\uc0b0\uc2dc\uc758 \uc77c\ubd80\ub85c \uad00\ub9ac\ub418\uace0 \uc788\ub294 \ud48d\ub3c4\uc5d0\uc11c\ub294 \uc870\uc120 \ub9d0 \uccad\uc77c \uc804\uc7c1\uc5d0\uc11c \uc77c\ubcf8\uacfc \uccad\ub098\ub77c \uac04\uc5d0 \ud48d\ub3c4\ud574\uc804\uc774 \uc788\uc5c8\ub2e4. \uc77c\uc81c \uac15\uc810\uae30\uc5d0\ub294 1914\ub144 \ud589\uc815\uad6c\uc5ed \uac1c\ud3b8\uc5d0 \ub530\ub77c \uc2dc\ud765\uad70 \uc218\uc554\uba74, \uad70\uc790\uba74\uc73c\ub85c \uc804\ud658\ub418\uc5c8\uc73c\uba70, \uc77c\ubd80 \uc9c0\uc5ed(\ubc18\uc6d4\uba74)\uc744 \uc218\uc6d0\uad70\uc5d0 \uc591\ub3c4\ud558\uc600\ub2e4. 1937\ub144\uc5d0\ub294 \uc548\uc0b0\uc744 \uad00\ud1b5\ud558\uc5ec \uc218\uc6d0\uacfc \uc778\ucc9c \uac04\uc744 \uc5f0\uacb0\ud558\ub294 \uc218\uc778\uc120 \ud611\uada4 \ucca0\ub3c4\uac00 \uc6b4\ud589\ud558\uae30 \uc2dc\uc791\ud558\uc600\ub2e4. \uadf8\ub7ec\ub098 1972\ub144 \ub300\ud55c\ubbfc\uad6d \uc815\ubd80\uc758 \ucca0\ub3c4 \ub178\uc120 \uc815\ub9ac\uc640 1988\ub144 \uc548\uc0b0\uc120 \uc804\ucca0 \uac1c\ud1b5 \uc774\ud6c4 \uc6b4\uc1a1\uc774 \uae09\uac10\ud558\ub2e4\uac00, 1995\ub144 12\uc6d4 31\uc77c\uc744 \ub05d\uc73c\ub85c \uc6b4\ud589\uc744 \uc911\ub2e8\ud558\uac8c \ub418\uc5c8\ub2e4. \ud55c\ud3b8\uc73c\ub85c 1970\ub144\ub300 \ud6c4\ubc18\ubd80\ud130 \uc548\uc0b0\uc2dc\uc758 \uacf5\uc5c5 \uc704\uc131\ub3c4\uc2dc\ud654\uac00 \uac00\uc18d\ub418\uae30 \uc2dc\uc791\ud558\uc5ec, 1976\ub144 12\uc6d4 \uc2dc\ud765\uad70\uacfc \ud654\uc131\uad70\uc758 \uc77c\ubd80\uc600\ub358 \uc548\uc0b0 \uc77c\ub300\uc5d0 \u2018 \ubc18\uc6d4\uc2e0\uacf5\uc5c5\ub3c4\uc2dc \uac74\uc124\uacc4\ud68d \u2019\uc744 \ubc1c\ud45c\ud558\uc600\uc73c\uba70, 1979\ub144 8\uc6d4 31\uc77c\uc5d0 \uacbd\uae30\ub3c4 \ubc18\uc6d4\ucd9c\uc7a5\uc18c\ub97c \uc124\uce58\ud558\uc600\uace0, 1986\ub144 1\uc6d4 1\uc77c\uc5d0\ub294 \ucd9c\uc7a5\uc18c\uc5d0\uc11c \uc9c0\uae08\uc758 \uc548\uc0b0\uc2dc\ub85c \uc2b9\uaca9\ub418\uc5c8\ub2e4. 2002\ub144\uc5d0\ub294 \uc0c1\ub85d\uad6c\uc640 \ub2e8\uc6d0\uad6c \ub4f1 2\uac1c \uad6c\ub97c \uc0c8\ub85c \uc124\uce58\ud558\uba74\uc11c \uad6c\uc81c(\u5340\u5236)\ub97c \uc2e4\uc2dc\ud558\uc600\ub2e4.", "sentence_answer": "\ud55c\ud3b8\uc73c\ub85c 1970\ub144\ub300 \ud6c4\ubc18\ubd80\ud130 \uc548\uc0b0\uc2dc\uc758 \uacf5\uc5c5 \uc704\uc131\ub3c4\uc2dc\ud654\uac00 \uac00\uc18d\ub418\uae30 \uc2dc\uc791\ud558\uc5ec, 1976\ub144 12\uc6d4 \uc2dc\ud765\uad70\uacfc \ud654\uc131\uad70\uc758 \uc77c\ubd80\uc600\ub358 \uc548\uc0b0 \uc77c\ub300\uc5d0 \u2018 \ubc18\uc6d4\uc2e0\uacf5\uc5c5\ub3c4\uc2dc \uac74\uc124\uacc4\ud68d \u2019\uc744 \ubc1c\ud45c\ud558\uc600\uc73c\uba70, 1979\ub144 8\uc6d4 31\uc77c\uc5d0 \uacbd\uae30\ub3c4 \ubc18\uc6d4\ucd9c\uc7a5\uc18c\ub97c \uc124\uce58\ud558\uc600\uace0, 1986\ub144 1\uc6d4 1\uc77c\uc5d0\ub294 \ucd9c\uc7a5\uc18c\uc5d0\uc11c \uc9c0\uae08\uc758 \uc548\uc0b0\uc2dc\ub85c \uc2b9\uaca9\ub418\uc5c8\ub2e4.", "paragraph_id": "6538931-4-2", "paragraph_question": "question: \uc548\uc0b0\uc2dc\uc758 \uacf5\uc5c5 \uc704\uc131\ub3c4\uc2dc\ud654 \uac00\uc18d\uc73c\ub85c 1976\ub144 \uc548\uc0b0 \uc77c\ub300\uc5d0 \ubc1c\ud45c\ub41c \uac74\uc124\uc0ac\uc5c5\uc740?, context: \ud604\uc7ac \uc548\uc0b0\uc2dc\uc758 \uc77c\ubd80\ub85c \uad00\ub9ac\ub418\uace0 \uc788\ub294 \ud48d\ub3c4\uc5d0\uc11c\ub294 \uc870\uc120 \ub9d0 \uccad\uc77c \uc804\uc7c1\uc5d0\uc11c \uc77c\ubcf8\uacfc \uccad\ub098\ub77c \uac04\uc5d0 \ud48d\ub3c4\ud574\uc804\uc774 \uc788\uc5c8\ub2e4. \uc77c\uc81c \uac15\uc810\uae30\uc5d0\ub294 1914\ub144 \ud589\uc815\uad6c\uc5ed \uac1c\ud3b8\uc5d0 \ub530\ub77c \uc2dc\ud765\uad70 \uc218\uc554\uba74, \uad70\uc790\uba74\uc73c\ub85c \uc804\ud658\ub418\uc5c8\uc73c\uba70, \uc77c\ubd80 \uc9c0\uc5ed(\ubc18\uc6d4\uba74)\uc744 \uc218\uc6d0\uad70\uc5d0 \uc591\ub3c4\ud558\uc600\ub2e4. 1937\ub144\uc5d0\ub294 \uc548\uc0b0\uc744 \uad00\ud1b5\ud558\uc5ec \uc218\uc6d0\uacfc \uc778\ucc9c \uac04\uc744 \uc5f0\uacb0\ud558\ub294 \uc218\uc778\uc120 \ud611\uada4 \ucca0\ub3c4\uac00 \uc6b4\ud589\ud558\uae30 \uc2dc\uc791\ud558\uc600\ub2e4. \uadf8\ub7ec\ub098 1972\ub144 \ub300\ud55c\ubbfc\uad6d \uc815\ubd80\uc758 \ucca0\ub3c4 \ub178\uc120 \uc815\ub9ac\uc640 1988\ub144 \uc548\uc0b0\uc120 \uc804\ucca0 \uac1c\ud1b5 \uc774\ud6c4 \uc6b4\uc1a1\uc774 \uae09\uac10\ud558\ub2e4\uac00, 1995\ub144 12\uc6d4 31\uc77c\uc744 \ub05d\uc73c\ub85c \uc6b4\ud589\uc744 \uc911\ub2e8\ud558\uac8c \ub418\uc5c8\ub2e4. \ud55c\ud3b8\uc73c\ub85c 1970\ub144\ub300 \ud6c4\ubc18\ubd80\ud130 \uc548\uc0b0\uc2dc\uc758 \uacf5\uc5c5 \uc704\uc131\ub3c4\uc2dc\ud654\uac00 \uac00\uc18d\ub418\uae30 \uc2dc\uc791\ud558\uc5ec, 1976\ub144 12\uc6d4 \uc2dc\ud765\uad70\uacfc \ud654\uc131\uad70\uc758 \uc77c\ubd80\uc600\ub358 \uc548\uc0b0 \uc77c\ub300\uc5d0 \u2018\ubc18\uc6d4\uc2e0\uacf5\uc5c5\ub3c4\uc2dc \uac74\uc124\uacc4\ud68d\u2019\uc744 \ubc1c\ud45c\ud558\uc600\uc73c\uba70, 1979\ub144 8\uc6d4 31\uc77c\uc5d0 \uacbd\uae30\ub3c4 \ubc18\uc6d4\ucd9c\uc7a5\uc18c\ub97c \uc124\uce58\ud558\uc600\uace0, 1986\ub144 1\uc6d4 1\uc77c\uc5d0\ub294 \ucd9c\uc7a5\uc18c\uc5d0\uc11c \uc9c0\uae08\uc758 \uc548\uc0b0\uc2dc\ub85c \uc2b9\uaca9\ub418\uc5c8\ub2e4. 2002\ub144\uc5d0\ub294 \uc0c1\ub85d\uad6c\uc640 \ub2e8\uc6d0\uad6c \ub4f1 2\uac1c \uad6c\ub97c \uc0c8\ub85c \uc124\uce58\ud558\uba74\uc11c \uad6c\uc81c(\u5340\u5236)\ub97c \uc2e4\uc2dc\ud558\uc600\ub2e4."} {"question": "\uc9c0\uae08 \ub0a8\ubd81\uc774 \uac19\uc774\ud55c\ub2e4\uace0 \ud574\ub3c4 \uad6d\ubbfc\uc740 \ubd81\ud55c \ud575 \ubb38\uc81c\uac00 \ud574\uacb0\ub410\ub2e4\uace0 \ubcf4\uc9c0 \uc54a\ub294\ub2e4\uace0 \ubd84\uc11d\ud55c \uad50\uc218\ub294?", "paragraph": "\uc5ec\ub860\ub3c4 \uae0d\uc815\uc801\uc774\uc9c0 \ubabb\ud55c \ud3b8\uc774\ub2e4. \uad6d\ud68c\uc758\uc7a5\uc2e4\uacfc SBS\uac00 \ud55c\uad6d\ub9ac\uc11c\uce58\uc5d0 \uc758\ub8b0\ud558\uc5ec \uc804\uad6d 19\uc138 \uc774\uc0c1 \uc131\uc778 \ub0a8\ub140 1,000\uba85\uc744 \ub300\uc0c1\uc73c\ub85c \uc9c4\ud589\ud55c '\ud3c9\ucc3d\uc62c\ub9bc\ud53d \ubc0f \ub0a8\ubd81\uad00\uacc4 \uad00\ub828 \uc5ec\ub860\uc870\uc0ac'\uc5d0\uc11c\ub294 \ubd81\ud55c\uc758 \uc62c\ub9bc\ud53d \ucc38\uac00\uc5d0 \ub300\ud574 \ub9e4\uc6b0 \ucc2c\uc131\uacfc \ub300\uccb4\ub85c \ucc2c\uc131\uc774 81.2%\uac00 \ub098\uc654\uc9c0\ub9cc \ub0a8\ubd81\uc774 \ub2e8\uc77c\ud300\uc73c\ub85c \ucc38\uac00\ud558\ub294 \uac83\uc5d0 \ub300\ud574\uc11c\ub294 \ubb34\ub9ac\ud560 \ud544\uc694\ub294 \uc5c6\ub2e4\ub294 \uc758\uacac\uc774 72.2%\uc5d0 \uc774\ub974\ub800\ub2e4. \ud2b9\ud788 \ud604\uc7ac\uc758 \ub192\uc740 \uc9c0\uc9c0\uc728\uc744 \uc9c0\ud0f1\ud574\uc8fc\ub294 20\u00b730\ub300\uc5d0\uc11c 82%\uac00 \ub118\ub294 \ubc18\ub300 \uc758\uacac\uc774 \ub098\uc654\ub2e4. \ud55c\ubc18\ub3c4\uae30\uc5d0 \ub300\ud574\uc11c\ub3c4 \ucc2c\uc131\uacfc \ubc18\ub300\uac00 50.1%\uc640 49.4%\ub85c \ud33d\ud33d\ud788 \uc758\uacac\uc774 \ub9de\uc130\ub2e4. \uae40\ud615\uc900 \uba85\uc9c0\ub300\ud559\uad50 \uad50\uc218\ub294 \"\ubb38 \ub300\ud1b5\ub839\uc774 \ud6c4\ubcf4 \uc2dc\uc808 '\uae30\ud68c\ub294 \uade0\ub4f1\ud558\uace0 \uacfc\uc815\uc740 \uacf5\uc815\ud558\uace0 \uacb0\uacfc\ub294 \uc815\uc758\ub85c\uc6b8 \uac83'\uc774\ub77c\uace0 \uc6d0\uce59\uc744 \ubc1d\ud614\ub2e4\uba74 \uc774\ub97c \uc9c0\ucf1c\uc57c \ud55c\ub2e4\"\uba70 \"\uae30\ud68c\ub97c \ubc15\ud0c8\ub2f9\ud560 \uc120\uc218\uac00 \uc0dd\uae38 \uc218\ubc16\uc5d0 \uc5c6\ub294\ub370 \uc80a\uc740 \uce35\uc774 \uacf5\uc815\u00b7\uade0\ub4f1\u00b7\uc815\uc758\uc5d0 \uacf5\uac10\ud558\uaca0\ub290\ub0d0\"\uace0 \uac15\uc870\ud588\uc73c\uba70 \uc774\ucc3d\uc21c \uacbd\ud76c\ub300\ud559\uad50 \uad50\uc218\ub294 \"\uc9c0\uae08 \ub0a8\ubd81\uc774 \uac19\uc774\ud55c\ub2e4\uace0 \ud574\ub3c4 \uad6d\ubbfc\uc740 \ubd81\ud55c \ud575 \ubb38\uc81c\uac00 \ud574\uacb0\ub410\ub2e4\uace0 \ubcf4\uc9c0 \uc54a\ub294\ub2e4. \uc774 \uac19\uc740 \uc0c1\ud669\uc5d0\uc11c \ub3d9\uc2dc \uc785\uc7a5\uc774 \uc774\ub904\uc838\ub3c4 \ud574\uacb0\uc5d0\ub294 \ub3c4\uc6c0\uc774 \ub418\uc9c0 \uc54a\ub294\ub2e4\"\uace0 \ubd84\uc11d\ud588\ub2e4. \uc815\uce58\uad8c\uc5d0\uc11c\ub3c4 \uac11\ub860\uc744\ubc15\uc774 \ubc8c\uc5b4\uc84c\ub294\ub370 \ubbfc\uc8fc\ub2f9\uc740 \ub300\ubcc0\uc778 \ub17c\ud3c9\uc744 \ud1b5\ud574 \"\ub0a8\ubd81\ud55c \ub2e8\uc77c\ud300 \uad6c\uc131\uc744 \ud658\uc601\ud55c\ub2e4\"\uba70 \"\ud3c9\ud654\uc640 \ud654\ud569\uc758 \uc62c\ub9bc\ud53d \uc815\uc2e0\uc744 \uc2e4\ud604\ud574\uc57c \ud55c\ub2e4\"\uace0 \ubc1d\ud614\ub2e4. \uadf8\ub7ec\uba74\uc11c \"\uc218\ub144 \ub3d9\uc548 \uc62c\ub9bc\ud53d\uacfc \ud328\ub7f4\ub9bc\ud53d\uc744 \uc720\uce58\ud558\uace0 \uc131\uacf5\ub9ac\uc5d0 \ub300\ud68c\uac00 \uce58\ub7ec\uc9c8 \uc218 \uc788\ub3c4\ub85d \uac01\uace0\uc758 \ub178\ub825\uc744 \uae30\uc6b8\uc774\uace0 \uc788\ub294 \uac15\uc6d0\ub3c4\ubbfc\uacfc \uad6d\ubbfc\uc744 \uc6b0\ub871\ud558\ub294 \ub354 \uc774\uc0c1\uc758 \ubc1c\uc5b8\uc740 \uc0bc\uac00\uc57c \ud55c\ub2e4\"\uace0 \ud55c\uad6d\ub2f9\uc758 '\ud3c9\uc591\uc62c\ub9bc\ud53d' \ud45c\ud604\uc744 \ube44\ud310\ud558\uba70 \"\ud55c\uad6d\ub2f9\uc740 \uc62c\ub9bc\ud53d \uc815\uc2e0 \uc2e4\ucc9c, \ud3c9\ud654 \uc62c\ub9bc\ud53d \uc2e4\ud604\uc5d0 \ub3d9\ucc38\ud560 \uac83\uc744 \ucd09\uad6c\ud55c\ub2e4\"\uace0 \ub367\ubd99\uc600\ub2e4. \ud55c\uad6d\ub2f9\uc740 \"\ubb38\uc7ac\uc778 \uc815\uad8c\uc740 \ubd81\ud55c\uacfc \uc62c\ub9bc\ud53d\uc744 \ub193\uace0 \ubc8c\uc774\ub294 '\uc815\uce58\uc1fc'\uc5d0 \ub300\ud574 \ud639\ub3c5\ud55c \ub300\uac00\ub97c \uce58\ub97c \uac83\"\uc774\ub77c\uba70 \"\ud3c9\ud654\uc62c\ub9bc\ud53d\uc744 \ube4c\ubbf8\ub85c \ud3c9\ucc3d \ub3d9\uacc4\uc62c\ub9bc\ud53d\uc744 \ud3c9\uc591\uc62c\ub9bc\ud53d\uc73c\ub85c \uc0c1\ub0a9\ud558\uba70 \ubd81\ud55c\uc758 \uae40\uc528 \uc655\uc870 \uccb4\uc81c \uc120\uc804\uc744 \ud558\ub7ec \uc628 \ub300\uc88c \ud55c \uba85\uc5d0\uac8c \uc655\ube44 \ub300\uc811\uc744 \ud558\uba70 \uc0ac\uc804\uac80\uc5f4\uae4c\uc9c0 \ubc1b\uace0 \uc788\ub294 \ubb38\uc7ac\uc778 \uc815\uad8c\uc740 \ub3c4\ub300\uccb4 \uc5b4\ub290 \ub098\ub77c \uc815\uad8c\uc778\uac00\"\ub77c\uace0 \uac15\ud558\uac8c \ube44\ud310\ud588\ub2e4. \ub610\ud55c \"'Republic of Korea'\ub77c\ub294 \ub300\ud55c\ubbfc\uad6d\uc758 \uacf5\uc2dd \uad6d\ud638\uc640 \uad6d\uac00\uc758 \uc0c1\uc9d5\uc778 \uc560\uad6d\uac00\uc640 \ud0dc\uadf9\uae30\uac00 \uc0ac\ub77c\uc9c4 \ube48\uc790\ub9ac\uc5d0 \ud604\uc1a1\uc6d4\uc774 \ub4f1\uc7a5\ud588\ub2e4\"\uba70 \ub0a0\uc744 \uc138\uc6e0\ub2e4.", "answer": "\uc774\ucc3d\uc21c", "sentence": "\uae40\ud615\uc900 \uba85\uc9c0\ub300\ud559\uad50 \uad50\uc218\ub294 \"\ubb38 \ub300\ud1b5\ub839\uc774 \ud6c4\ubcf4 \uc2dc\uc808 '\uae30\ud68c\ub294 \uade0\ub4f1\ud558\uace0 \uacfc\uc815\uc740 \uacf5\uc815\ud558\uace0 \uacb0\uacfc\ub294 \uc815\uc758\ub85c\uc6b8 \uac83'\uc774\ub77c\uace0 \uc6d0\uce59\uc744 \ubc1d\ud614\ub2e4\uba74 \uc774\ub97c \uc9c0\ucf1c\uc57c \ud55c\ub2e4\"\uba70 \"\uae30\ud68c\ub97c \ubc15\ud0c8\ub2f9\ud560 \uc120\uc218\uac00 \uc0dd\uae38 \uc218\ubc16\uc5d0 \uc5c6\ub294\ub370 \uc80a\uc740 \uce35\uc774 \uacf5\uc815\u00b7\uade0\ub4f1\u00b7\uc815\uc758\uc5d0 \uacf5\uac10\ud558\uaca0\ub290\ub0d0\"\uace0 \uac15\uc870\ud588\uc73c\uba70 \uc774\ucc3d\uc21c \uacbd\ud76c\ub300\ud559\uad50 \uad50\uc218\ub294 \"\uc9c0\uae08 \ub0a8\ubd81\uc774 \uac19\uc774\ud55c\ub2e4\uace0 \ud574\ub3c4 \uad6d\ubbfc\uc740 \ubd81\ud55c \ud575 \ubb38\uc81c\uac00 \ud574\uacb0\ub410\ub2e4\uace0 \ubcf4\uc9c0 \uc54a\ub294\ub2e4.", "paragraph_sentence": "\uc5ec\ub860\ub3c4 \uae0d\uc815\uc801\uc774\uc9c0 \ubabb\ud55c \ud3b8\uc774\ub2e4. \uad6d\ud68c\uc758\uc7a5\uc2e4\uacfc SBS\uac00 \ud55c\uad6d\ub9ac\uc11c\uce58\uc5d0 \uc758\ub8b0\ud558\uc5ec \uc804\uad6d 19\uc138 \uc774\uc0c1 \uc131\uc778 \ub0a8\ub140 1,000\uba85\uc744 \ub300\uc0c1\uc73c\ub85c \uc9c4\ud589\ud55c '\ud3c9\ucc3d\uc62c\ub9bc\ud53d \ubc0f \ub0a8\ubd81\uad00\uacc4 \uad00\ub828 \uc5ec\ub860\uc870\uc0ac'\uc5d0\uc11c\ub294 \ubd81\ud55c\uc758 \uc62c\ub9bc\ud53d \ucc38\uac00\uc5d0 \ub300\ud574 \ub9e4\uc6b0 \ucc2c\uc131\uacfc \ub300\uccb4\ub85c \ucc2c\uc131\uc774 81.2%\uac00 \ub098\uc654\uc9c0\ub9cc \ub0a8\ubd81\uc774 \ub2e8\uc77c\ud300\uc73c\ub85c \ucc38\uac00\ud558\ub294 \uac83\uc5d0 \ub300\ud574\uc11c\ub294 \ubb34\ub9ac\ud560 \ud544\uc694\ub294 \uc5c6\ub2e4\ub294 \uc758\uacac\uc774 72.2%\uc5d0 \uc774\ub974\ub800\ub2e4. \ud2b9\ud788 \ud604\uc7ac\uc758 \ub192\uc740 \uc9c0\uc9c0\uc728\uc744 \uc9c0\ud0f1\ud574\uc8fc\ub294 20\u00b730\ub300\uc5d0\uc11c 82%\uac00 \ub118\ub294 \ubc18\ub300 \uc758\uacac\uc774 \ub098\uc654\ub2e4. \ud55c\ubc18\ub3c4\uae30\uc5d0 \ub300\ud574\uc11c\ub3c4 \ucc2c\uc131\uacfc \ubc18\ub300\uac00 50.1%\uc640 49.4%\ub85c \ud33d\ud33d\ud788 \uc758\uacac\uc774 \ub9de\uc130\ub2e4. \uae40\ud615\uc900 \uba85\uc9c0\ub300\ud559\uad50 \uad50\uc218\ub294 \"\ubb38 \ub300\ud1b5\ub839\uc774 \ud6c4\ubcf4 \uc2dc\uc808 '\uae30\ud68c\ub294 \uade0\ub4f1\ud558\uace0 \uacfc\uc815\uc740 \uacf5\uc815\ud558\uace0 \uacb0\uacfc\ub294 \uc815\uc758\ub85c\uc6b8 \uac83'\uc774\ub77c\uace0 \uc6d0\uce59\uc744 \ubc1d\ud614\ub2e4\uba74 \uc774\ub97c \uc9c0\ucf1c\uc57c \ud55c\ub2e4\"\uba70 \"\uae30\ud68c\ub97c \ubc15\ud0c8\ub2f9\ud560 \uc120\uc218\uac00 \uc0dd\uae38 \uc218\ubc16\uc5d0 \uc5c6\ub294\ub370 \uc80a\uc740 \uce35\uc774 \uacf5\uc815\u00b7\uade0\ub4f1\u00b7\uc815\uc758\uc5d0 \uacf5\uac10\ud558\uaca0\ub290\ub0d0\"\uace0 \uac15\uc870\ud588\uc73c\uba70 \uc774\ucc3d\uc21c \uacbd\ud76c\ub300\ud559\uad50 \uad50\uc218\ub294 \"\uc9c0\uae08 \ub0a8\ubd81\uc774 \uac19\uc774\ud55c\ub2e4\uace0 \ud574\ub3c4 \uad6d\ubbfc\uc740 \ubd81\ud55c \ud575 \ubb38\uc81c\uac00 \ud574\uacb0\ub410\ub2e4\uace0 \ubcf4\uc9c0 \uc54a\ub294\ub2e4. \uc774 \uac19\uc740 \uc0c1\ud669\uc5d0\uc11c \ub3d9\uc2dc \uc785\uc7a5\uc774 \uc774\ub904\uc838\ub3c4 \ud574\uacb0\uc5d0\ub294 \ub3c4\uc6c0\uc774 \ub418\uc9c0 \uc54a\ub294\ub2e4\"\uace0 \ubd84\uc11d\ud588\ub2e4. \uc815\uce58\uad8c\uc5d0\uc11c\ub3c4 \uac11\ub860\uc744\ubc15\uc774 \ubc8c\uc5b4\uc84c\ub294\ub370 \ubbfc\uc8fc\ub2f9\uc740 \ub300\ubcc0\uc778 \ub17c\ud3c9\uc744 \ud1b5\ud574 \"\ub0a8\ubd81\ud55c \ub2e8\uc77c\ud300 \uad6c\uc131\uc744 \ud658\uc601\ud55c\ub2e4\"\uba70 \"\ud3c9\ud654\uc640 \ud654\ud569\uc758 \uc62c\ub9bc\ud53d \uc815\uc2e0\uc744 \uc2e4\ud604\ud574\uc57c \ud55c\ub2e4\"\uace0 \ubc1d\ud614\ub2e4. \uadf8\ub7ec\uba74\uc11c \"\uc218\ub144 \ub3d9\uc548 \uc62c\ub9bc\ud53d\uacfc \ud328\ub7f4\ub9bc\ud53d\uc744 \uc720\uce58\ud558\uace0 \uc131\uacf5\ub9ac\uc5d0 \ub300\ud68c\uac00 \uce58\ub7ec\uc9c8 \uc218 \uc788\ub3c4\ub85d \uac01\uace0\uc758 \ub178\ub825\uc744 \uae30\uc6b8\uc774\uace0 \uc788\ub294 \uac15\uc6d0\ub3c4\ubbfc\uacfc \uad6d\ubbfc\uc744 \uc6b0\ub871\ud558\ub294 \ub354 \uc774\uc0c1\uc758 \ubc1c\uc5b8\uc740 \uc0bc\uac00\uc57c \ud55c\ub2e4\"\uace0 \ud55c\uad6d\ub2f9\uc758 '\ud3c9\uc591\uc62c\ub9bc\ud53d' \ud45c\ud604\uc744 \ube44\ud310\ud558\uba70 \"\ud55c\uad6d\ub2f9\uc740 \uc62c\ub9bc\ud53d \uc815\uc2e0 \uc2e4\ucc9c, \ud3c9\ud654 \uc62c\ub9bc\ud53d \uc2e4\ud604\uc5d0 \ub3d9\ucc38\ud560 \uac83\uc744 \ucd09\uad6c\ud55c\ub2e4\"\uace0 \ub367\ubd99\uc600\ub2e4. \ud55c\uad6d\ub2f9\uc740 \"\ubb38\uc7ac\uc778 \uc815\uad8c\uc740 \ubd81\ud55c\uacfc \uc62c\ub9bc\ud53d\uc744 \ub193\uace0 \ubc8c\uc774\ub294 '\uc815\uce58\uc1fc'\uc5d0 \ub300\ud574 \ud639\ub3c5\ud55c \ub300\uac00\ub97c \uce58\ub97c \uac83\"\uc774\ub77c\uba70 \"\ud3c9\ud654\uc62c\ub9bc\ud53d\uc744 \ube4c\ubbf8\ub85c \ud3c9\ucc3d \ub3d9\uacc4\uc62c\ub9bc\ud53d\uc744 \ud3c9\uc591\uc62c\ub9bc\ud53d\uc73c\ub85c \uc0c1\ub0a9\ud558\uba70 \ubd81\ud55c\uc758 \uae40\uc528 \uc655\uc870 \uccb4\uc81c \uc120\uc804\uc744 \ud558\ub7ec \uc628 \ub300\uc88c \ud55c \uba85\uc5d0\uac8c \uc655\ube44 \ub300\uc811\uc744 \ud558\uba70 \uc0ac\uc804\uac80\uc5f4\uae4c\uc9c0 \ubc1b\uace0 \uc788\ub294 \ubb38\uc7ac\uc778 \uc815\uad8c\uc740 \ub3c4\ub300\uccb4 \uc5b4\ub290 \ub098\ub77c \uc815\uad8c\uc778\uac00\"\ub77c\uace0 \uac15\ud558\uac8c \ube44\ud310\ud588\ub2e4. \ub610\ud55c \"'Republic of Korea'\ub77c\ub294 \ub300\ud55c\ubbfc\uad6d\uc758 \uacf5\uc2dd \uad6d\ud638\uc640 \uad6d\uac00\uc758 \uc0c1\uc9d5\uc778 \uc560\uad6d\uac00\uc640 \ud0dc\uadf9\uae30\uac00 \uc0ac\ub77c\uc9c4 \ube48\uc790\ub9ac\uc5d0 \ud604\uc1a1\uc6d4\uc774 \ub4f1\uc7a5\ud588\ub2e4\"\uba70 \ub0a0\uc744 \uc138\uc6e0\ub2e4.", "paragraph_answer": "\uc5ec\ub860\ub3c4 \uae0d\uc815\uc801\uc774\uc9c0 \ubabb\ud55c \ud3b8\uc774\ub2e4. \uad6d\ud68c\uc758\uc7a5\uc2e4\uacfc SBS\uac00 \ud55c\uad6d\ub9ac\uc11c\uce58\uc5d0 \uc758\ub8b0\ud558\uc5ec \uc804\uad6d 19\uc138 \uc774\uc0c1 \uc131\uc778 \ub0a8\ub140 1,000\uba85\uc744 \ub300\uc0c1\uc73c\ub85c \uc9c4\ud589\ud55c '\ud3c9\ucc3d\uc62c\ub9bc\ud53d \ubc0f \ub0a8\ubd81\uad00\uacc4 \uad00\ub828 \uc5ec\ub860\uc870\uc0ac'\uc5d0\uc11c\ub294 \ubd81\ud55c\uc758 \uc62c\ub9bc\ud53d \ucc38\uac00\uc5d0 \ub300\ud574 \ub9e4\uc6b0 \ucc2c\uc131\uacfc \ub300\uccb4\ub85c \ucc2c\uc131\uc774 81.2%\uac00 \ub098\uc654\uc9c0\ub9cc \ub0a8\ubd81\uc774 \ub2e8\uc77c\ud300\uc73c\ub85c \ucc38\uac00\ud558\ub294 \uac83\uc5d0 \ub300\ud574\uc11c\ub294 \ubb34\ub9ac\ud560 \ud544\uc694\ub294 \uc5c6\ub2e4\ub294 \uc758\uacac\uc774 72.2%\uc5d0 \uc774\ub974\ub800\ub2e4. \ud2b9\ud788 \ud604\uc7ac\uc758 \ub192\uc740 \uc9c0\uc9c0\uc728\uc744 \uc9c0\ud0f1\ud574\uc8fc\ub294 20\u00b730\ub300\uc5d0\uc11c 82%\uac00 \ub118\ub294 \ubc18\ub300 \uc758\uacac\uc774 \ub098\uc654\ub2e4. \ud55c\ubc18\ub3c4\uae30\uc5d0 \ub300\ud574\uc11c\ub3c4 \ucc2c\uc131\uacfc \ubc18\ub300\uac00 50.1%\uc640 49.4%\ub85c \ud33d\ud33d\ud788 \uc758\uacac\uc774 \ub9de\uc130\ub2e4. \uae40\ud615\uc900 \uba85\uc9c0\ub300\ud559\uad50 \uad50\uc218\ub294 \"\ubb38 \ub300\ud1b5\ub839\uc774 \ud6c4\ubcf4 \uc2dc\uc808 '\uae30\ud68c\ub294 \uade0\ub4f1\ud558\uace0 \uacfc\uc815\uc740 \uacf5\uc815\ud558\uace0 \uacb0\uacfc\ub294 \uc815\uc758\ub85c\uc6b8 \uac83'\uc774\ub77c\uace0 \uc6d0\uce59\uc744 \ubc1d\ud614\ub2e4\uba74 \uc774\ub97c \uc9c0\ucf1c\uc57c \ud55c\ub2e4\"\uba70 \"\uae30\ud68c\ub97c \ubc15\ud0c8\ub2f9\ud560 \uc120\uc218\uac00 \uc0dd\uae38 \uc218\ubc16\uc5d0 \uc5c6\ub294\ub370 \uc80a\uc740 \uce35\uc774 \uacf5\uc815\u00b7\uade0\ub4f1\u00b7\uc815\uc758\uc5d0 \uacf5\uac10\ud558\uaca0\ub290\ub0d0\"\uace0 \uac15\uc870\ud588\uc73c\uba70 \uc774\ucc3d\uc21c \uacbd\ud76c\ub300\ud559\uad50 \uad50\uc218\ub294 \"\uc9c0\uae08 \ub0a8\ubd81\uc774 \uac19\uc774\ud55c\ub2e4\uace0 \ud574\ub3c4 \uad6d\ubbfc\uc740 \ubd81\ud55c \ud575 \ubb38\uc81c\uac00 \ud574\uacb0\ub410\ub2e4\uace0 \ubcf4\uc9c0 \uc54a\ub294\ub2e4. \uc774 \uac19\uc740 \uc0c1\ud669\uc5d0\uc11c \ub3d9\uc2dc \uc785\uc7a5\uc774 \uc774\ub904\uc838\ub3c4 \ud574\uacb0\uc5d0\ub294 \ub3c4\uc6c0\uc774 \ub418\uc9c0 \uc54a\ub294\ub2e4\"\uace0 \ubd84\uc11d\ud588\ub2e4. \uc815\uce58\uad8c\uc5d0\uc11c\ub3c4 \uac11\ub860\uc744\ubc15\uc774 \ubc8c\uc5b4\uc84c\ub294\ub370 \ubbfc\uc8fc\ub2f9\uc740 \ub300\ubcc0\uc778 \ub17c\ud3c9\uc744 \ud1b5\ud574 \"\ub0a8\ubd81\ud55c \ub2e8\uc77c\ud300 \uad6c\uc131\uc744 \ud658\uc601\ud55c\ub2e4\"\uba70 \"\ud3c9\ud654\uc640 \ud654\ud569\uc758 \uc62c\ub9bc\ud53d \uc815\uc2e0\uc744 \uc2e4\ud604\ud574\uc57c \ud55c\ub2e4\"\uace0 \ubc1d\ud614\ub2e4. \uadf8\ub7ec\uba74\uc11c \"\uc218\ub144 \ub3d9\uc548 \uc62c\ub9bc\ud53d\uacfc \ud328\ub7f4\ub9bc\ud53d\uc744 \uc720\uce58\ud558\uace0 \uc131\uacf5\ub9ac\uc5d0 \ub300\ud68c\uac00 \uce58\ub7ec\uc9c8 \uc218 \uc788\ub3c4\ub85d \uac01\uace0\uc758 \ub178\ub825\uc744 \uae30\uc6b8\uc774\uace0 \uc788\ub294 \uac15\uc6d0\ub3c4\ubbfc\uacfc \uad6d\ubbfc\uc744 \uc6b0\ub871\ud558\ub294 \ub354 \uc774\uc0c1\uc758 \ubc1c\uc5b8\uc740 \uc0bc\uac00\uc57c \ud55c\ub2e4\"\uace0 \ud55c\uad6d\ub2f9\uc758 '\ud3c9\uc591\uc62c\ub9bc\ud53d' \ud45c\ud604\uc744 \ube44\ud310\ud558\uba70 \"\ud55c\uad6d\ub2f9\uc740 \uc62c\ub9bc\ud53d \uc815\uc2e0 \uc2e4\ucc9c, \ud3c9\ud654 \uc62c\ub9bc\ud53d \uc2e4\ud604\uc5d0 \ub3d9\ucc38\ud560 \uac83\uc744 \ucd09\uad6c\ud55c\ub2e4\"\uace0 \ub367\ubd99\uc600\ub2e4. \ud55c\uad6d\ub2f9\uc740 \"\ubb38\uc7ac\uc778 \uc815\uad8c\uc740 \ubd81\ud55c\uacfc \uc62c\ub9bc\ud53d\uc744 \ub193\uace0 \ubc8c\uc774\ub294 '\uc815\uce58\uc1fc'\uc5d0 \ub300\ud574 \ud639\ub3c5\ud55c \ub300\uac00\ub97c \uce58\ub97c \uac83\"\uc774\ub77c\uba70 \"\ud3c9\ud654\uc62c\ub9bc\ud53d\uc744 \ube4c\ubbf8\ub85c \ud3c9\ucc3d \ub3d9\uacc4\uc62c\ub9bc\ud53d\uc744 \ud3c9\uc591\uc62c\ub9bc\ud53d\uc73c\ub85c \uc0c1\ub0a9\ud558\uba70 \ubd81\ud55c\uc758 \uae40\uc528 \uc655\uc870 \uccb4\uc81c \uc120\uc804\uc744 \ud558\ub7ec \uc628 \ub300\uc88c \ud55c \uba85\uc5d0\uac8c \uc655\ube44 \ub300\uc811\uc744 \ud558\uba70 \uc0ac\uc804\uac80\uc5f4\uae4c\uc9c0 \ubc1b\uace0 \uc788\ub294 \ubb38\uc7ac\uc778 \uc815\uad8c\uc740 \ub3c4\ub300\uccb4 \uc5b4\ub290 \ub098\ub77c \uc815\uad8c\uc778\uac00\"\ub77c\uace0 \uac15\ud558\uac8c \ube44\ud310\ud588\ub2e4. \ub610\ud55c \"'Republic of Korea'\ub77c\ub294 \ub300\ud55c\ubbfc\uad6d\uc758 \uacf5\uc2dd \uad6d\ud638\uc640 \uad6d\uac00\uc758 \uc0c1\uc9d5\uc778 \uc560\uad6d\uac00\uc640 \ud0dc\uadf9\uae30\uac00 \uc0ac\ub77c\uc9c4 \ube48\uc790\ub9ac\uc5d0 \ud604\uc1a1\uc6d4\uc774 \ub4f1\uc7a5\ud588\ub2e4\"\uba70 \ub0a0\uc744 \uc138\uc6e0\ub2e4.", "sentence_answer": "\uae40\ud615\uc900 \uba85\uc9c0\ub300\ud559\uad50 \uad50\uc218\ub294 \"\ubb38 \ub300\ud1b5\ub839\uc774 \ud6c4\ubcf4 \uc2dc\uc808 '\uae30\ud68c\ub294 \uade0\ub4f1\ud558\uace0 \uacfc\uc815\uc740 \uacf5\uc815\ud558\uace0 \uacb0\uacfc\ub294 \uc815\uc758\ub85c\uc6b8 \uac83'\uc774\ub77c\uace0 \uc6d0\uce59\uc744 \ubc1d\ud614\ub2e4\uba74 \uc774\ub97c \uc9c0\ucf1c\uc57c \ud55c\ub2e4\"\uba70 \"\uae30\ud68c\ub97c \ubc15\ud0c8\ub2f9\ud560 \uc120\uc218\uac00 \uc0dd\uae38 \uc218\ubc16\uc5d0 \uc5c6\ub294\ub370 \uc80a\uc740 \uce35\uc774 \uacf5\uc815\u00b7\uade0\ub4f1\u00b7\uc815\uc758\uc5d0 \uacf5\uac10\ud558\uaca0\ub290\ub0d0\"\uace0 \uac15\uc870\ud588\uc73c\uba70 \uc774\ucc3d\uc21c \uacbd\ud76c\ub300\ud559\uad50 \uad50\uc218\ub294 \"\uc9c0\uae08 \ub0a8\ubd81\uc774 \uac19\uc774\ud55c\ub2e4\uace0 \ud574\ub3c4 \uad6d\ubbfc\uc740 \ubd81\ud55c \ud575 \ubb38\uc81c\uac00 \ud574\uacb0\ub410\ub2e4\uace0 \ubcf4\uc9c0 \uc54a\ub294\ub2e4.", "paragraph_id": "6580792-110-1", "paragraph_question": "question: \uc9c0\uae08 \ub0a8\ubd81\uc774 \uac19\uc774\ud55c\ub2e4\uace0 \ud574\ub3c4 \uad6d\ubbfc\uc740 \ubd81\ud55c \ud575 \ubb38\uc81c\uac00 \ud574\uacb0\ub410\ub2e4\uace0 \ubcf4\uc9c0 \uc54a\ub294\ub2e4\uace0 \ubd84\uc11d\ud55c \uad50\uc218\ub294?, context: \uc5ec\ub860\ub3c4 \uae0d\uc815\uc801\uc774\uc9c0 \ubabb\ud55c \ud3b8\uc774\ub2e4. \uad6d\ud68c\uc758\uc7a5\uc2e4\uacfc SBS\uac00 \ud55c\uad6d\ub9ac\uc11c\uce58\uc5d0 \uc758\ub8b0\ud558\uc5ec \uc804\uad6d 19\uc138 \uc774\uc0c1 \uc131\uc778 \ub0a8\ub140 1,000\uba85\uc744 \ub300\uc0c1\uc73c\ub85c \uc9c4\ud589\ud55c '\ud3c9\ucc3d\uc62c\ub9bc\ud53d \ubc0f \ub0a8\ubd81\uad00\uacc4 \uad00\ub828 \uc5ec\ub860\uc870\uc0ac'\uc5d0\uc11c\ub294 \ubd81\ud55c\uc758 \uc62c\ub9bc\ud53d \ucc38\uac00\uc5d0 \ub300\ud574 \ub9e4\uc6b0 \ucc2c\uc131\uacfc \ub300\uccb4\ub85c \ucc2c\uc131\uc774 81.2%\uac00 \ub098\uc654\uc9c0\ub9cc \ub0a8\ubd81\uc774 \ub2e8\uc77c\ud300\uc73c\ub85c \ucc38\uac00\ud558\ub294 \uac83\uc5d0 \ub300\ud574\uc11c\ub294 \ubb34\ub9ac\ud560 \ud544\uc694\ub294 \uc5c6\ub2e4\ub294 \uc758\uacac\uc774 72.2%\uc5d0 \uc774\ub974\ub800\ub2e4. \ud2b9\ud788 \ud604\uc7ac\uc758 \ub192\uc740 \uc9c0\uc9c0\uc728\uc744 \uc9c0\ud0f1\ud574\uc8fc\ub294 20\u00b730\ub300\uc5d0\uc11c 82%\uac00 \ub118\ub294 \ubc18\ub300 \uc758\uacac\uc774 \ub098\uc654\ub2e4. \ud55c\ubc18\ub3c4\uae30\uc5d0 \ub300\ud574\uc11c\ub3c4 \ucc2c\uc131\uacfc \ubc18\ub300\uac00 50.1%\uc640 49.4%\ub85c \ud33d\ud33d\ud788 \uc758\uacac\uc774 \ub9de\uc130\ub2e4. \uae40\ud615\uc900 \uba85\uc9c0\ub300\ud559\uad50 \uad50\uc218\ub294 \"\ubb38 \ub300\ud1b5\ub839\uc774 \ud6c4\ubcf4 \uc2dc\uc808 '\uae30\ud68c\ub294 \uade0\ub4f1\ud558\uace0 \uacfc\uc815\uc740 \uacf5\uc815\ud558\uace0 \uacb0\uacfc\ub294 \uc815\uc758\ub85c\uc6b8 \uac83'\uc774\ub77c\uace0 \uc6d0\uce59\uc744 \ubc1d\ud614\ub2e4\uba74 \uc774\ub97c \uc9c0\ucf1c\uc57c \ud55c\ub2e4\"\uba70 \"\uae30\ud68c\ub97c \ubc15\ud0c8\ub2f9\ud560 \uc120\uc218\uac00 \uc0dd\uae38 \uc218\ubc16\uc5d0 \uc5c6\ub294\ub370 \uc80a\uc740 \uce35\uc774 \uacf5\uc815\u00b7\uade0\ub4f1\u00b7\uc815\uc758\uc5d0 \uacf5\uac10\ud558\uaca0\ub290\ub0d0\"\uace0 \uac15\uc870\ud588\uc73c\uba70 \uc774\ucc3d\uc21c \uacbd\ud76c\ub300\ud559\uad50 \uad50\uc218\ub294 \"\uc9c0\uae08 \ub0a8\ubd81\uc774 \uac19\uc774\ud55c\ub2e4\uace0 \ud574\ub3c4 \uad6d\ubbfc\uc740 \ubd81\ud55c \ud575 \ubb38\uc81c\uac00 \ud574\uacb0\ub410\ub2e4\uace0 \ubcf4\uc9c0 \uc54a\ub294\ub2e4. \uc774 \uac19\uc740 \uc0c1\ud669\uc5d0\uc11c \ub3d9\uc2dc \uc785\uc7a5\uc774 \uc774\ub904\uc838\ub3c4 \ud574\uacb0\uc5d0\ub294 \ub3c4\uc6c0\uc774 \ub418\uc9c0 \uc54a\ub294\ub2e4\"\uace0 \ubd84\uc11d\ud588\ub2e4. \uc815\uce58\uad8c\uc5d0\uc11c\ub3c4 \uac11\ub860\uc744\ubc15\uc774 \ubc8c\uc5b4\uc84c\ub294\ub370 \ubbfc\uc8fc\ub2f9\uc740 \ub300\ubcc0\uc778 \ub17c\ud3c9\uc744 \ud1b5\ud574 \"\ub0a8\ubd81\ud55c \ub2e8\uc77c\ud300 \uad6c\uc131\uc744 \ud658\uc601\ud55c\ub2e4\"\uba70 \"\ud3c9\ud654\uc640 \ud654\ud569\uc758 \uc62c\ub9bc\ud53d \uc815\uc2e0\uc744 \uc2e4\ud604\ud574\uc57c \ud55c\ub2e4\"\uace0 \ubc1d\ud614\ub2e4. \uadf8\ub7ec\uba74\uc11c \"\uc218\ub144 \ub3d9\uc548 \uc62c\ub9bc\ud53d\uacfc \ud328\ub7f4\ub9bc\ud53d\uc744 \uc720\uce58\ud558\uace0 \uc131\uacf5\ub9ac\uc5d0 \ub300\ud68c\uac00 \uce58\ub7ec\uc9c8 \uc218 \uc788\ub3c4\ub85d \uac01\uace0\uc758 \ub178\ub825\uc744 \uae30\uc6b8\uc774\uace0 \uc788\ub294 \uac15\uc6d0\ub3c4\ubbfc\uacfc \uad6d\ubbfc\uc744 \uc6b0\ub871\ud558\ub294 \ub354 \uc774\uc0c1\uc758 \ubc1c\uc5b8\uc740 \uc0bc\uac00\uc57c \ud55c\ub2e4\"\uace0 \ud55c\uad6d\ub2f9\uc758 '\ud3c9\uc591\uc62c\ub9bc\ud53d' \ud45c\ud604\uc744 \ube44\ud310\ud558\uba70 \"\ud55c\uad6d\ub2f9\uc740 \uc62c\ub9bc\ud53d \uc815\uc2e0 \uc2e4\ucc9c, \ud3c9\ud654 \uc62c\ub9bc\ud53d \uc2e4\ud604\uc5d0 \ub3d9\ucc38\ud560 \uac83\uc744 \ucd09\uad6c\ud55c\ub2e4\"\uace0 \ub367\ubd99\uc600\ub2e4. \ud55c\uad6d\ub2f9\uc740 \"\ubb38\uc7ac\uc778 \uc815\uad8c\uc740 \ubd81\ud55c\uacfc \uc62c\ub9bc\ud53d\uc744 \ub193\uace0 \ubc8c\uc774\ub294 '\uc815\uce58\uc1fc'\uc5d0 \ub300\ud574 \ud639\ub3c5\ud55c \ub300\uac00\ub97c \uce58\ub97c \uac83\"\uc774\ub77c\uba70 \"\ud3c9\ud654\uc62c\ub9bc\ud53d\uc744 \ube4c\ubbf8\ub85c \ud3c9\ucc3d \ub3d9\uacc4\uc62c\ub9bc\ud53d\uc744 \ud3c9\uc591\uc62c\ub9bc\ud53d\uc73c\ub85c \uc0c1\ub0a9\ud558\uba70 \ubd81\ud55c\uc758 \uae40\uc528 \uc655\uc870 \uccb4\uc81c \uc120\uc804\uc744 \ud558\ub7ec \uc628 \ub300\uc88c \ud55c \uba85\uc5d0\uac8c \uc655\ube44 \ub300\uc811\uc744 \ud558\uba70 \uc0ac\uc804\uac80\uc5f4\uae4c\uc9c0 \ubc1b\uace0 \uc788\ub294 \ubb38\uc7ac\uc778 \uc815\uad8c\uc740 \ub3c4\ub300\uccb4 \uc5b4\ub290 \ub098\ub77c \uc815\uad8c\uc778\uac00\"\ub77c\uace0 \uac15\ud558\uac8c \ube44\ud310\ud588\ub2e4. \ub610\ud55c \"'Republic of Korea'\ub77c\ub294 \ub300\ud55c\ubbfc\uad6d\uc758 \uacf5\uc2dd \uad6d\ud638\uc640 \uad6d\uac00\uc758 \uc0c1\uc9d5\uc778 \uc560\uad6d\uac00\uc640 \ud0dc\uadf9\uae30\uac00 \uc0ac\ub77c\uc9c4 \ube48\uc790\ub9ac\uc5d0 \ud604\uc1a1\uc6d4\uc774 \ub4f1\uc7a5\ud588\ub2e4\"\uba70 \ub0a0\uc744 \uc138\uc6e0\ub2e4."} {"question": "1942\ub144 \ub8e8\uc988\ubca8\ud2b8 \ub300\ud1b5\ub839\uc5d0\uac8c \ubcf4\uace0\ub41c \ud575\ud3ed\ud0c4 \uc81c\uc870\ub97c \uc704\ud55c \ud50c\ub79c\ud2b8 \ud6c4\ubcf4\uc9c0\ub85c \uc120\uc815\ub41c \uc9c0\uc5ed\uc740?", "paragraph": "\ub9c8\uc15c\uc740 \ud6d7\ub0a0 \"\ub098\ub294 \ud55c \ubc88\ub3c4, \ub2e8 \ud55c \ubc88\ub3c4 \uc6d0\uc790 \ubd84\uc5f4\uc774\ub780 \uac83\uc744 \ub4e4\uc5b4\ubcf8 \uc801\uc774 \uc5c6\uc5c8\ub2e4. \ub098\ub294 \uadf8\uc800 90\uc5b5 \ub2ec\ub7ec\uc758 \uc608\uc0b0\uc73c\ub85c \ucd5c\uc18c\ud55c 4\uac1c\uc758 \ud50c\ub79c\ud2b8\ub97c \uac74\uc124\ud558\uc5ec\uc57c \ud55c\ub2e4\ub294 \ub9d0\uc744 \ub4e4\uc5c8\uc744 \ubfd0\uc774\ub2e4.\"\ub77c\uace0 \ub9d0\ud558\uc600\ub2e4. \ub2f9\uc2dc\uc5d0\ub294 \ud39c\uc2e4\ubca0\uc774\ub2c8\uc544 \uc8fc\uc5d0 \uc704\uce58\ud558\uc600\ub358 TNT \uacf5\uc7a5 \ud558\ub098\ub97c \uc9d3\ub294\ub370\ub3c4 128\uc5b5 \ub2ec\ub7ec\uac00 \uc18c\uc694\ub418\uc5c8\ub2e4. \uadf8\ub85c\ube0c\uc2a4\uac00 \uc774\ub7f0 \uc77c\uc744 \ub9c8\uce58 \uc5f4 \uba85\uc5d0\uc11c \ucc9c \uba85 \uc815\ub3c4\uc758 \uc190\ub2d8\uc744 \ubaa8\uc2e0 \uc5f0\ud68c\ub97c \uc900\ube44\ud558\ub77c\ub294 \ub4ef\uc774 \uc9c0\uc2dc\ud558\uc600\uae30 \ub54c\ubb38\uc5d0, \ub9c8\uc15c\uc740 \uc2ec\ud55c \uc555\ubc15\uac10\uc744 \ub290\uaef4\uc57c\ud588\ub2e4. \ub9e8\ud574\ud2bc \uacc4\ud68d\uc5d0 \ucc38\uc5ec\ud55c \ubbfc\uac04\ubd80\ubd84 \ud68c\uc0ac\uc600\ub358 \uc2a4\ud1a4 \uc564 \uc6f9\uc2a4\ud130\ub294 \ud50c\ub79c\ud2b8\ub97c \uc9c0\uc744 \uc9c0\uc5ed\uc744 \uc774\ubbf8 \ubb3c\uc0c9\ud558\uc5ec \ub193\uc740 \uc0c1\ud0dc\uc600\ub2e4. 1942\ub144 \ub8e8\uc988\ubca8\ud2b8 \ub300\ud1b5\ub839\uc5d0\uac8c \ubcf4\uace0\ub41c \uad70\uc218\ubb3c\uc790 \uc608\uc0b0\uc5d0\ub294 \ud50c\ub79c\ud2b8\uc758 \ud6c4\ubcf4\uc9c0\ub85c \ud14c\ub124\uc2dc \uc8fc\uc758 \ub179\uc2a4\ube4c\uc774 \uc120\uc815\ub418\uc5b4 \uc788\uc5c8\ub2e4. \uc774 \uc9c0\uc5ed\uc740 \ub2e4\ub978 \uc778\uad6c \ubc00\uc9d1 \uc9c0\uc5ed\uc5d0\uc11c \ub5a8\uc5b4\uc838 \uc788\uc73c\uba74\uc11c \uac15\uc774 \uc778\uc811\ud558\uace0 \uc788\uc5b4 \uc6a9\uc218 \uc870\ub2ec\uc5d0 \uc720\ub9ac\ud55c \uacf3\uc774\uc5c8\ub2e4. \uc870\uc0ac\ud300\uc740 \ucd94\uac00 \uc870\uc0ac\ub97c \ubc8c\uc778 \ud6c4 \ud14c\ub124\uc2dc \uc8fc\uc758 \uc5d8\uc790\ub97c \ucd5c\uc885\uc801\uc73c\ub85c \uc120\uc815\ud558\uc600\ub2e4. \uc6b0\ub77c\ub284\uc758 \uc815\uc81c\uc640 \uad00\ub828\ud558\uc5ec \uc608\uc815\ub418\uc5c8\ub358 \uacf5\uc815\ub4e4 \uac00\uc6b4\ub370\ub294 \ub85c\ub80c\uc2a4\uc758 \uc804\uc790\uae30\uc801 \ubd84\ub9ac \ubc29\ubc95\ub9cc\uc774 \uc81c\uc791\uc5d0 \ucc29\uc218 \ub420\ub9cc\ud07c \ucda9\ubd84\ud55c \uc9c4\ucc99\uc744 \ubcf4\uc600\ub2e4.", "answer": "\ub179\uc2a4\ube4c", "sentence": "1942\ub144 \ub8e8\uc988\ubca8\ud2b8 \ub300\ud1b5\ub839\uc5d0\uac8c \ubcf4\uace0\ub41c \uad70\uc218\ubb3c\uc790 \uc608\uc0b0\uc5d0\ub294 \ud50c\ub79c\ud2b8\uc758 \ud6c4\ubcf4\uc9c0\ub85c \ud14c\ub124\uc2dc \uc8fc\uc758 \ub179\uc2a4\ube4c \uc774 \uc120\uc815\ub418\uc5b4 \uc788\uc5c8\ub2e4.", "paragraph_sentence": "\ub9c8\uc15c\uc740 \ud6d7\ub0a0 \"\ub098\ub294 \ud55c \ubc88\ub3c4, \ub2e8 \ud55c \ubc88\ub3c4 \uc6d0\uc790 \ubd84\uc5f4\uc774\ub780 \uac83\uc744 \ub4e4\uc5b4\ubcf8 \uc801\uc774 \uc5c6\uc5c8\ub2e4. \ub098\ub294 \uadf8\uc800 90\uc5b5 \ub2ec\ub7ec\uc758 \uc608\uc0b0\uc73c\ub85c \ucd5c\uc18c\ud55c 4\uac1c\uc758 \ud50c\ub79c\ud2b8\ub97c \uac74\uc124\ud558\uc5ec\uc57c \ud55c\ub2e4\ub294 \ub9d0\uc744 \ub4e4\uc5c8\uc744 \ubfd0\uc774\ub2e4. \"\ub77c\uace0 \ub9d0\ud558\uc600\ub2e4. \ub2f9\uc2dc\uc5d0\ub294 \ud39c\uc2e4\ubca0\uc774\ub2c8\uc544 \uc8fc\uc5d0 \uc704\uce58\ud558\uc600\ub358 TNT \uacf5\uc7a5 \ud558\ub098\ub97c \uc9d3\ub294\ub370\ub3c4 128\uc5b5 \ub2ec\ub7ec\uac00 \uc18c\uc694\ub418\uc5c8\ub2e4. \uadf8\ub85c\ube0c\uc2a4\uac00 \uc774\ub7f0 \uc77c\uc744 \ub9c8\uce58 \uc5f4 \uba85\uc5d0\uc11c \ucc9c \uba85 \uc815\ub3c4\uc758 \uc190\ub2d8\uc744 \ubaa8\uc2e0 \uc5f0\ud68c\ub97c \uc900\ube44\ud558\ub77c\ub294 \ub4ef\uc774 \uc9c0\uc2dc\ud558\uc600\uae30 \ub54c\ubb38\uc5d0, \ub9c8\uc15c\uc740 \uc2ec\ud55c \uc555\ubc15\uac10\uc744 \ub290\uaef4\uc57c\ud588\ub2e4. \ub9e8\ud574\ud2bc \uacc4\ud68d\uc5d0 \ucc38\uc5ec\ud55c \ubbfc\uac04\ubd80\ubd84 \ud68c\uc0ac\uc600\ub358 \uc2a4\ud1a4 \uc564 \uc6f9\uc2a4\ud130\ub294 \ud50c\ub79c\ud2b8\ub97c \uc9c0\uc744 \uc9c0\uc5ed\uc744 \uc774\ubbf8 \ubb3c\uc0c9\ud558\uc5ec \ub193\uc740 \uc0c1\ud0dc\uc600\ub2e4. 1942\ub144 \ub8e8\uc988\ubca8\ud2b8 \ub300\ud1b5\ub839\uc5d0\uac8c \ubcf4\uace0\ub41c \uad70\uc218\ubb3c\uc790 \uc608\uc0b0\uc5d0\ub294 \ud50c\ub79c\ud2b8\uc758 \ud6c4\ubcf4\uc9c0\ub85c \ud14c\ub124\uc2dc \uc8fc\uc758 \ub179\uc2a4\ube4c \uc774 \uc120\uc815\ub418\uc5b4 \uc788\uc5c8\ub2e4. \uc774 \uc9c0\uc5ed\uc740 \ub2e4\ub978 \uc778\uad6c \ubc00\uc9d1 \uc9c0\uc5ed\uc5d0\uc11c \ub5a8\uc5b4\uc838 \uc788\uc73c\uba74\uc11c \uac15\uc774 \uc778\uc811\ud558\uace0 \uc788\uc5b4 \uc6a9\uc218 \uc870\ub2ec\uc5d0 \uc720\ub9ac\ud55c \uacf3\uc774\uc5c8\ub2e4. \uc870\uc0ac\ud300\uc740 \ucd94\uac00 \uc870\uc0ac\ub97c \ubc8c\uc778 \ud6c4 \ud14c\ub124\uc2dc \uc8fc\uc758 \uc5d8\uc790\ub97c \ucd5c\uc885\uc801\uc73c\ub85c \uc120\uc815\ud558\uc600\ub2e4. \uc6b0\ub77c\ub284\uc758 \uc815\uc81c\uc640 \uad00\ub828\ud558\uc5ec \uc608\uc815\ub418\uc5c8\ub358 \uacf5\uc815\ub4e4 \uac00\uc6b4\ub370\ub294 \ub85c\ub80c\uc2a4\uc758 \uc804\uc790\uae30\uc801 \ubd84\ub9ac \ubc29\ubc95\ub9cc\uc774 \uc81c\uc791\uc5d0 \ucc29\uc218 \ub420\ub9cc\ud07c \ucda9\ubd84\ud55c \uc9c4\ucc99\uc744 \ubcf4\uc600\ub2e4.", "paragraph_answer": "\ub9c8\uc15c\uc740 \ud6d7\ub0a0 \"\ub098\ub294 \ud55c \ubc88\ub3c4, \ub2e8 \ud55c \ubc88\ub3c4 \uc6d0\uc790 \ubd84\uc5f4\uc774\ub780 \uac83\uc744 \ub4e4\uc5b4\ubcf8 \uc801\uc774 \uc5c6\uc5c8\ub2e4. \ub098\ub294 \uadf8\uc800 90\uc5b5 \ub2ec\ub7ec\uc758 \uc608\uc0b0\uc73c\ub85c \ucd5c\uc18c\ud55c 4\uac1c\uc758 \ud50c\ub79c\ud2b8\ub97c \uac74\uc124\ud558\uc5ec\uc57c \ud55c\ub2e4\ub294 \ub9d0\uc744 \ub4e4\uc5c8\uc744 \ubfd0\uc774\ub2e4.\"\ub77c\uace0 \ub9d0\ud558\uc600\ub2e4. \ub2f9\uc2dc\uc5d0\ub294 \ud39c\uc2e4\ubca0\uc774\ub2c8\uc544 \uc8fc\uc5d0 \uc704\uce58\ud558\uc600\ub358 TNT \uacf5\uc7a5 \ud558\ub098\ub97c \uc9d3\ub294\ub370\ub3c4 128\uc5b5 \ub2ec\ub7ec\uac00 \uc18c\uc694\ub418\uc5c8\ub2e4. \uadf8\ub85c\ube0c\uc2a4\uac00 \uc774\ub7f0 \uc77c\uc744 \ub9c8\uce58 \uc5f4 \uba85\uc5d0\uc11c \ucc9c \uba85 \uc815\ub3c4\uc758 \uc190\ub2d8\uc744 \ubaa8\uc2e0 \uc5f0\ud68c\ub97c \uc900\ube44\ud558\ub77c\ub294 \ub4ef\uc774 \uc9c0\uc2dc\ud558\uc600\uae30 \ub54c\ubb38\uc5d0, \ub9c8\uc15c\uc740 \uc2ec\ud55c \uc555\ubc15\uac10\uc744 \ub290\uaef4\uc57c\ud588\ub2e4. \ub9e8\ud574\ud2bc \uacc4\ud68d\uc5d0 \ucc38\uc5ec\ud55c \ubbfc\uac04\ubd80\ubd84 \ud68c\uc0ac\uc600\ub358 \uc2a4\ud1a4 \uc564 \uc6f9\uc2a4\ud130\ub294 \ud50c\ub79c\ud2b8\ub97c \uc9c0\uc744 \uc9c0\uc5ed\uc744 \uc774\ubbf8 \ubb3c\uc0c9\ud558\uc5ec \ub193\uc740 \uc0c1\ud0dc\uc600\ub2e4. 1942\ub144 \ub8e8\uc988\ubca8\ud2b8 \ub300\ud1b5\ub839\uc5d0\uac8c \ubcf4\uace0\ub41c \uad70\uc218\ubb3c\uc790 \uc608\uc0b0\uc5d0\ub294 \ud50c\ub79c\ud2b8\uc758 \ud6c4\ubcf4\uc9c0\ub85c \ud14c\ub124\uc2dc \uc8fc\uc758 \ub179\uc2a4\ube4c \uc774 \uc120\uc815\ub418\uc5b4 \uc788\uc5c8\ub2e4. \uc774 \uc9c0\uc5ed\uc740 \ub2e4\ub978 \uc778\uad6c \ubc00\uc9d1 \uc9c0\uc5ed\uc5d0\uc11c \ub5a8\uc5b4\uc838 \uc788\uc73c\uba74\uc11c \uac15\uc774 \uc778\uc811\ud558\uace0 \uc788\uc5b4 \uc6a9\uc218 \uc870\ub2ec\uc5d0 \uc720\ub9ac\ud55c \uacf3\uc774\uc5c8\ub2e4. \uc870\uc0ac\ud300\uc740 \ucd94\uac00 \uc870\uc0ac\ub97c \ubc8c\uc778 \ud6c4 \ud14c\ub124\uc2dc \uc8fc\uc758 \uc5d8\uc790\ub97c \ucd5c\uc885\uc801\uc73c\ub85c \uc120\uc815\ud558\uc600\ub2e4. \uc6b0\ub77c\ub284\uc758 \uc815\uc81c\uc640 \uad00\ub828\ud558\uc5ec \uc608\uc815\ub418\uc5c8\ub358 \uacf5\uc815\ub4e4 \uac00\uc6b4\ub370\ub294 \ub85c\ub80c\uc2a4\uc758 \uc804\uc790\uae30\uc801 \ubd84\ub9ac \ubc29\ubc95\ub9cc\uc774 \uc81c\uc791\uc5d0 \ucc29\uc218 \ub420\ub9cc\ud07c \ucda9\ubd84\ud55c \uc9c4\ucc99\uc744 \ubcf4\uc600\ub2e4.", "sentence_answer": "1942\ub144 \ub8e8\uc988\ubca8\ud2b8 \ub300\ud1b5\ub839\uc5d0\uac8c \ubcf4\uace0\ub41c \uad70\uc218\ubb3c\uc790 \uc608\uc0b0\uc5d0\ub294 \ud50c\ub79c\ud2b8\uc758 \ud6c4\ubcf4\uc9c0\ub85c \ud14c\ub124\uc2dc \uc8fc\uc758 \ub179\uc2a4\ube4c \uc774 \uc120\uc815\ub418\uc5b4 \uc788\uc5c8\ub2e4.", "paragraph_id": "6502177-8-0", "paragraph_question": "question: 1942\ub144 \ub8e8\uc988\ubca8\ud2b8 \ub300\ud1b5\ub839\uc5d0\uac8c \ubcf4\uace0\ub41c \ud575\ud3ed\ud0c4 \uc81c\uc870\ub97c \uc704\ud55c \ud50c\ub79c\ud2b8 \ud6c4\ubcf4\uc9c0\ub85c \uc120\uc815\ub41c \uc9c0\uc5ed\uc740?, context: \ub9c8\uc15c\uc740 \ud6d7\ub0a0 \"\ub098\ub294 \ud55c \ubc88\ub3c4, \ub2e8 \ud55c \ubc88\ub3c4 \uc6d0\uc790 \ubd84\uc5f4\uc774\ub780 \uac83\uc744 \ub4e4\uc5b4\ubcf8 \uc801\uc774 \uc5c6\uc5c8\ub2e4. \ub098\ub294 \uadf8\uc800 90\uc5b5 \ub2ec\ub7ec\uc758 \uc608\uc0b0\uc73c\ub85c \ucd5c\uc18c\ud55c 4\uac1c\uc758 \ud50c\ub79c\ud2b8\ub97c \uac74\uc124\ud558\uc5ec\uc57c \ud55c\ub2e4\ub294 \ub9d0\uc744 \ub4e4\uc5c8\uc744 \ubfd0\uc774\ub2e4.\"\ub77c\uace0 \ub9d0\ud558\uc600\ub2e4. \ub2f9\uc2dc\uc5d0\ub294 \ud39c\uc2e4\ubca0\uc774\ub2c8\uc544 \uc8fc\uc5d0 \uc704\uce58\ud558\uc600\ub358 TNT \uacf5\uc7a5 \ud558\ub098\ub97c \uc9d3\ub294\ub370\ub3c4 128\uc5b5 \ub2ec\ub7ec\uac00 \uc18c\uc694\ub418\uc5c8\ub2e4. \uadf8\ub85c\ube0c\uc2a4\uac00 \uc774\ub7f0 \uc77c\uc744 \ub9c8\uce58 \uc5f4 \uba85\uc5d0\uc11c \ucc9c \uba85 \uc815\ub3c4\uc758 \uc190\ub2d8\uc744 \ubaa8\uc2e0 \uc5f0\ud68c\ub97c \uc900\ube44\ud558\ub77c\ub294 \ub4ef\uc774 \uc9c0\uc2dc\ud558\uc600\uae30 \ub54c\ubb38\uc5d0, \ub9c8\uc15c\uc740 \uc2ec\ud55c \uc555\ubc15\uac10\uc744 \ub290\uaef4\uc57c\ud588\ub2e4. \ub9e8\ud574\ud2bc \uacc4\ud68d\uc5d0 \ucc38\uc5ec\ud55c \ubbfc\uac04\ubd80\ubd84 \ud68c\uc0ac\uc600\ub358 \uc2a4\ud1a4 \uc564 \uc6f9\uc2a4\ud130\ub294 \ud50c\ub79c\ud2b8\ub97c \uc9c0\uc744 \uc9c0\uc5ed\uc744 \uc774\ubbf8 \ubb3c\uc0c9\ud558\uc5ec \ub193\uc740 \uc0c1\ud0dc\uc600\ub2e4. 1942\ub144 \ub8e8\uc988\ubca8\ud2b8 \ub300\ud1b5\ub839\uc5d0\uac8c \ubcf4\uace0\ub41c \uad70\uc218\ubb3c\uc790 \uc608\uc0b0\uc5d0\ub294 \ud50c\ub79c\ud2b8\uc758 \ud6c4\ubcf4\uc9c0\ub85c \ud14c\ub124\uc2dc \uc8fc\uc758 \ub179\uc2a4\ube4c\uc774 \uc120\uc815\ub418\uc5b4 \uc788\uc5c8\ub2e4. \uc774 \uc9c0\uc5ed\uc740 \ub2e4\ub978 \uc778\uad6c \ubc00\uc9d1 \uc9c0\uc5ed\uc5d0\uc11c \ub5a8\uc5b4\uc838 \uc788\uc73c\uba74\uc11c \uac15\uc774 \uc778\uc811\ud558\uace0 \uc788\uc5b4 \uc6a9\uc218 \uc870\ub2ec\uc5d0 \uc720\ub9ac\ud55c \uacf3\uc774\uc5c8\ub2e4. \uc870\uc0ac\ud300\uc740 \ucd94\uac00 \uc870\uc0ac\ub97c \ubc8c\uc778 \ud6c4 \ud14c\ub124\uc2dc \uc8fc\uc758 \uc5d8\uc790\ub97c \ucd5c\uc885\uc801\uc73c\ub85c \uc120\uc815\ud558\uc600\ub2e4. \uc6b0\ub77c\ub284\uc758 \uc815\uc81c\uc640 \uad00\ub828\ud558\uc5ec \uc608\uc815\ub418\uc5c8\ub358 \uacf5\uc815\ub4e4 \uac00\uc6b4\ub370\ub294 \ub85c\ub80c\uc2a4\uc758 \uc804\uc790\uae30\uc801 \ubd84\ub9ac \ubc29\ubc95\ub9cc\uc774 \uc81c\uc791\uc5d0 \ucc29\uc218 \ub420\ub9cc\ud07c \ucda9\ubd84\ud55c \uc9c4\ucc99\uc744 \ubcf4\uc600\ub2e4."} {"question": "\uc5b4\ub2c8 \ubc45\ud06c\uc2a4\uc758 \uc5f0\uc18d \ucd9c\uc7a5 \uae30\ub85d\uc740?", "paragraph": "1961\ub144, \uc5b4\ub2c8 \ubc45\ud06c\uc2a4\ub294 \ubcf5\ubb34 \uc2dc\uc808 \ub2e4\ucce4\ub358 \ubb34\ub98e\uc5d0 \ubb38\uc81c\ub97c \ub290\uaf08\ub2e4. \uc774\ud6c4 4\uacbd\uae30 \uc815\ub3c4\ub97c \uacb0\uc7a5\ud558\uba74\uc11c \uc5f0\uc18d \ucd9c\uc7a5 \uae30\ub85d\uc774 717\uacbd\uae30\uc5d0\uc11c \uba48\ucd94\uac8c \ub418\uc5c8\uace0, \uc2a4\ud0e0 \ubba4\uc9c0\uc5bc\uc758 895\uacbd\uae30 \uc5f0\uc18d \ucd9c\uc7a5 \uae30\ub85d\uc5d0 \ub3c4\ub2ec\ud558\uc9c0 \ubabb\ud588\ub2e4. 5\uc6d4\uc774 \ub418\uc5b4\uc11c \uc2dc\uce74\uace0 \ucef5\uc2a4\ub294 \uc81c\ub9ac \ud0a8\ub2ec\uc774 \uae30\uc874\uc5d0 \ubc45\ud06c\uc2a4\uac00 \ub9e1\ub358 \uc720\uaca9\uc218\ub97c \ub300\uc2e0\ud558\uae30\ub85c \ud558\uace0 \ubc45\ud06c\uc2a4\ub294 \uc88c\uc775\uc218\ub85c \ud3ec\uc9c0\uc158\uc744 \uc62e\uae30\uae30\ub85c \uacb0\uc815\ud588\ub2e4. \ubc45\ud06c\uc2a4\ub294 \ud6c4\uc77c\uc5d0 \ub2f9\uc2dc \ud3ec\uc9c0\uc158\uc73c\ub85c \uc778\ud574 \uacaa\uc5c8\ub358 \uc5b4\ub824\uc6c0\uc744 \ub450\uace0 \"\uc88c\uce21 \uc794\ub514\uc5d0\uc11c \ub6f8 \ub54c\uba74 \ub9c8\uce58 \ubb3c\uc5d0 \ube60\uc9c4 \uc624\ub9ac \uac19\uc740 \uc678\ub85c\uc6c0\uc744 \ub290\uaf08\ub2e4.\"\uace0 \ub2f5\ud588\ub2e4. \uc911\uacac\uc218 \ub9ac\uce58 \uc560\uc2dc\ubc88\uc5d0\uac8c \uc88c\uc775 \uc218\ube44\uc5d0 \ub300\ud55c \ub3c4\uc6c0\uc744 \ubc1b\uc73c\uba70 23\uacbd\uae30\uc5d0 \ub098\uc124 \uc218 \uc788\uc5c8\uace0 \uc2e4\ucc45\uc740 \ud558\ub098\ub9cc\uc744 \uae30\ub85d\ud588\ub2e4. 6\uc6d4\uc5d0 \uc811\uc5b4\ub4e4\uc5c8\uc744 \ub54c\uac00 \ub418\uc5b4\uc11c\uc57c \uc120\uc218 \uc2dc\uc808 1\ub8e8\ub97c \ub9e1\uc558\uc5c8\ub358 \ucc30\ub9ac \uadf8\ub9bc \ucf54\uce58\uc758 \ub3c4\uc6c0\uc73c\ub85c 1\ub8e8\uc218\ub85c \uc804\ud5a5\ud588\ub2e4. \uba54\uc774\uc800 \ub9ac\uadf8\ub294 1959\ub144\ubd80\ud130 62\ub144\uae4c\uc9c0 \ub9e4\ud574 \ub450 \ucc28\ub840 \uc62c\uc2a4\ud0c0\uc804\uc744 \uac1c\ucd5c\ud588\uace0, 1959\ub144 \uc62c\uc2a4\ud0c0\uc804\uc758 \uccab \uacbd\uae30\uc5d0\uc11c\ub294 \uba85\ub2e8\uc5d0 \ubf51\ud788\uc9c0 \ubabb\ud588\ub2e4. \uadf8\ub7ec\ub098 \ub450 \ubc88\uc9f8 \uacbd\uae30\uc5d0\uc120 \uc608\ube44 \uc120\uc218\ub85c \uc774\ub984\uc744 \uc62c\ub824 \ub300\ud0c0\ub85c \uacbd\uae30\uc5d0 \ub098\uc124 \uc218 \uc788\uc5c8\ub2e4.", "answer": "717\uacbd\uae30", "sentence": "\uc5f0\uc18d \ucd9c\uc7a5 \uae30\ub85d\uc774 717\uacbd\uae30 \uc5d0\uc11c \uba48\ucd94\uac8c \ub418\uc5c8\uace0, \uc2a4\ud0e0 \ubba4\uc9c0\uc5bc\uc758 895\uacbd\uae30 \uc5f0\uc18d \ucd9c\uc7a5 \uae30\ub85d\uc5d0 \ub3c4\ub2ec\ud558\uc9c0 \ubabb\ud588\ub2e4.", "paragraph_sentence": "1961\ub144, \uc5b4\ub2c8 \ubc45\ud06c\uc2a4\ub294 \ubcf5\ubb34 \uc2dc\uc808 \ub2e4\ucce4\ub358 \ubb34\ub98e\uc5d0 \ubb38\uc81c\ub97c \ub290\uaf08\ub2e4. \uc774\ud6c4 4\uacbd\uae30 \uc815\ub3c4\ub97c \uacb0\uc7a5\ud558\uba74\uc11c \uc5f0\uc18d \ucd9c\uc7a5 \uae30\ub85d\uc774 717\uacbd\uae30 \uc5d0\uc11c \uba48\ucd94\uac8c \ub418\uc5c8\uace0, \uc2a4\ud0e0 \ubba4\uc9c0\uc5bc\uc758 895\uacbd\uae30 \uc5f0\uc18d \ucd9c\uc7a5 \uae30\ub85d\uc5d0 \ub3c4\ub2ec\ud558\uc9c0 \ubabb\ud588\ub2e4. 5\uc6d4\uc774 \ub418\uc5b4\uc11c \uc2dc\uce74\uace0 \ucef5\uc2a4\ub294 \uc81c\ub9ac \ud0a8\ub2ec\uc774 \uae30\uc874\uc5d0 \ubc45\ud06c\uc2a4\uac00 \ub9e1\ub358 \uc720\uaca9\uc218\ub97c \ub300\uc2e0\ud558\uae30\ub85c \ud558\uace0 \ubc45\ud06c\uc2a4\ub294 \uc88c\uc775\uc218\ub85c \ud3ec\uc9c0\uc158\uc744 \uc62e\uae30\uae30\ub85c \uacb0\uc815\ud588\ub2e4. \ubc45\ud06c\uc2a4\ub294 \ud6c4\uc77c\uc5d0 \ub2f9\uc2dc \ud3ec\uc9c0\uc158\uc73c\ub85c \uc778\ud574 \uacaa\uc5c8\ub358 \uc5b4\ub824\uc6c0\uc744 \ub450\uace0 \"\uc88c\uce21 \uc794\ub514\uc5d0\uc11c \ub6f8 \ub54c\uba74 \ub9c8\uce58 \ubb3c\uc5d0 \ube60\uc9c4 \uc624\ub9ac \uac19\uc740 \uc678\ub85c\uc6c0\uc744 \ub290\uaf08\ub2e4. \"\uace0 \ub2f5\ud588\ub2e4. \uc911\uacac\uc218 \ub9ac\uce58 \uc560\uc2dc\ubc88\uc5d0\uac8c \uc88c\uc775 \uc218\ube44\uc5d0 \ub300\ud55c \ub3c4\uc6c0\uc744 \ubc1b\uc73c\uba70 23\uacbd\uae30\uc5d0 \ub098\uc124 \uc218 \uc788\uc5c8\uace0 \uc2e4\ucc45\uc740 \ud558\ub098\ub9cc\uc744 \uae30\ub85d\ud588\ub2e4. 6\uc6d4\uc5d0 \uc811\uc5b4\ub4e4\uc5c8\uc744 \ub54c\uac00 \ub418\uc5b4\uc11c\uc57c \uc120\uc218 \uc2dc\uc808 1\ub8e8\ub97c \ub9e1\uc558\uc5c8\ub358 \ucc30\ub9ac \uadf8\ub9bc \ucf54\uce58\uc758 \ub3c4\uc6c0\uc73c\ub85c 1\ub8e8\uc218\ub85c \uc804\ud5a5\ud588\ub2e4. \uba54\uc774\uc800 \ub9ac\uadf8\ub294 1959\ub144\ubd80\ud130 62\ub144\uae4c\uc9c0 \ub9e4\ud574 \ub450 \ucc28\ub840 \uc62c\uc2a4\ud0c0\uc804\uc744 \uac1c\ucd5c\ud588\uace0, 1959\ub144 \uc62c\uc2a4\ud0c0\uc804\uc758 \uccab \uacbd\uae30\uc5d0\uc11c\ub294 \uba85\ub2e8\uc5d0 \ubf51\ud788\uc9c0 \ubabb\ud588\ub2e4. \uadf8\ub7ec\ub098 \ub450 \ubc88\uc9f8 \uacbd\uae30\uc5d0\uc120 \uc608\ube44 \uc120\uc218\ub85c \uc774\ub984\uc744 \uc62c\ub824 \ub300\ud0c0\ub85c \uacbd\uae30\uc5d0 \ub098\uc124 \uc218 \uc788\uc5c8\ub2e4.", "paragraph_answer": "1961\ub144, \uc5b4\ub2c8 \ubc45\ud06c\uc2a4\ub294 \ubcf5\ubb34 \uc2dc\uc808 \ub2e4\ucce4\ub358 \ubb34\ub98e\uc5d0 \ubb38\uc81c\ub97c \ub290\uaf08\ub2e4. \uc774\ud6c4 4\uacbd\uae30 \uc815\ub3c4\ub97c \uacb0\uc7a5\ud558\uba74\uc11c \uc5f0\uc18d \ucd9c\uc7a5 \uae30\ub85d\uc774 717\uacbd\uae30 \uc5d0\uc11c \uba48\ucd94\uac8c \ub418\uc5c8\uace0, \uc2a4\ud0e0 \ubba4\uc9c0\uc5bc\uc758 895\uacbd\uae30 \uc5f0\uc18d \ucd9c\uc7a5 \uae30\ub85d\uc5d0 \ub3c4\ub2ec\ud558\uc9c0 \ubabb\ud588\ub2e4. 5\uc6d4\uc774 \ub418\uc5b4\uc11c \uc2dc\uce74\uace0 \ucef5\uc2a4\ub294 \uc81c\ub9ac \ud0a8\ub2ec\uc774 \uae30\uc874\uc5d0 \ubc45\ud06c\uc2a4\uac00 \ub9e1\ub358 \uc720\uaca9\uc218\ub97c \ub300\uc2e0\ud558\uae30\ub85c \ud558\uace0 \ubc45\ud06c\uc2a4\ub294 \uc88c\uc775\uc218\ub85c \ud3ec\uc9c0\uc158\uc744 \uc62e\uae30\uae30\ub85c \uacb0\uc815\ud588\ub2e4. \ubc45\ud06c\uc2a4\ub294 \ud6c4\uc77c\uc5d0 \ub2f9\uc2dc \ud3ec\uc9c0\uc158\uc73c\ub85c \uc778\ud574 \uacaa\uc5c8\ub358 \uc5b4\ub824\uc6c0\uc744 \ub450\uace0 \"\uc88c\uce21 \uc794\ub514\uc5d0\uc11c \ub6f8 \ub54c\uba74 \ub9c8\uce58 \ubb3c\uc5d0 \ube60\uc9c4 \uc624\ub9ac \uac19\uc740 \uc678\ub85c\uc6c0\uc744 \ub290\uaf08\ub2e4.\"\uace0 \ub2f5\ud588\ub2e4. \uc911\uacac\uc218 \ub9ac\uce58 \uc560\uc2dc\ubc88\uc5d0\uac8c \uc88c\uc775 \uc218\ube44\uc5d0 \ub300\ud55c \ub3c4\uc6c0\uc744 \ubc1b\uc73c\uba70 23\uacbd\uae30\uc5d0 \ub098\uc124 \uc218 \uc788\uc5c8\uace0 \uc2e4\ucc45\uc740 \ud558\ub098\ub9cc\uc744 \uae30\ub85d\ud588\ub2e4. 6\uc6d4\uc5d0 \uc811\uc5b4\ub4e4\uc5c8\uc744 \ub54c\uac00 \ub418\uc5b4\uc11c\uc57c \uc120\uc218 \uc2dc\uc808 1\ub8e8\ub97c \ub9e1\uc558\uc5c8\ub358 \ucc30\ub9ac \uadf8\ub9bc \ucf54\uce58\uc758 \ub3c4\uc6c0\uc73c\ub85c 1\ub8e8\uc218\ub85c \uc804\ud5a5\ud588\ub2e4. \uba54\uc774\uc800 \ub9ac\uadf8\ub294 1959\ub144\ubd80\ud130 62\ub144\uae4c\uc9c0 \ub9e4\ud574 \ub450 \ucc28\ub840 \uc62c\uc2a4\ud0c0\uc804\uc744 \uac1c\ucd5c\ud588\uace0, 1959\ub144 \uc62c\uc2a4\ud0c0\uc804\uc758 \uccab \uacbd\uae30\uc5d0\uc11c\ub294 \uba85\ub2e8\uc5d0 \ubf51\ud788\uc9c0 \ubabb\ud588\ub2e4. \uadf8\ub7ec\ub098 \ub450 \ubc88\uc9f8 \uacbd\uae30\uc5d0\uc120 \uc608\ube44 \uc120\uc218\ub85c \uc774\ub984\uc744 \uc62c\ub824 \ub300\ud0c0\ub85c \uacbd\uae30\uc5d0 \ub098\uc124 \uc218 \uc788\uc5c8\ub2e4.", "sentence_answer": "\uc5f0\uc18d \ucd9c\uc7a5 \uae30\ub85d\uc774 717\uacbd\uae30 \uc5d0\uc11c \uba48\ucd94\uac8c \ub418\uc5c8\uace0, \uc2a4\ud0e0 \ubba4\uc9c0\uc5bc\uc758 895\uacbd\uae30 \uc5f0\uc18d \ucd9c\uc7a5 \uae30\ub85d\uc5d0 \ub3c4\ub2ec\ud558\uc9c0 \ubabb\ud588\ub2e4.", "paragraph_id": "6466051-5-1", "paragraph_question": "question: \uc5b4\ub2c8 \ubc45\ud06c\uc2a4\uc758 \uc5f0\uc18d \ucd9c\uc7a5 \uae30\ub85d\uc740?, context: 1961\ub144, \uc5b4\ub2c8 \ubc45\ud06c\uc2a4\ub294 \ubcf5\ubb34 \uc2dc\uc808 \ub2e4\ucce4\ub358 \ubb34\ub98e\uc5d0 \ubb38\uc81c\ub97c \ub290\uaf08\ub2e4. \uc774\ud6c4 4\uacbd\uae30 \uc815\ub3c4\ub97c \uacb0\uc7a5\ud558\uba74\uc11c \uc5f0\uc18d \ucd9c\uc7a5 \uae30\ub85d\uc774 717\uacbd\uae30\uc5d0\uc11c \uba48\ucd94\uac8c \ub418\uc5c8\uace0, \uc2a4\ud0e0 \ubba4\uc9c0\uc5bc\uc758 895\uacbd\uae30 \uc5f0\uc18d \ucd9c\uc7a5 \uae30\ub85d\uc5d0 \ub3c4\ub2ec\ud558\uc9c0 \ubabb\ud588\ub2e4. 5\uc6d4\uc774 \ub418\uc5b4\uc11c \uc2dc\uce74\uace0 \ucef5\uc2a4\ub294 \uc81c\ub9ac \ud0a8\ub2ec\uc774 \uae30\uc874\uc5d0 \ubc45\ud06c\uc2a4\uac00 \ub9e1\ub358 \uc720\uaca9\uc218\ub97c \ub300\uc2e0\ud558\uae30\ub85c \ud558\uace0 \ubc45\ud06c\uc2a4\ub294 \uc88c\uc775\uc218\ub85c \ud3ec\uc9c0\uc158\uc744 \uc62e\uae30\uae30\ub85c \uacb0\uc815\ud588\ub2e4. \ubc45\ud06c\uc2a4\ub294 \ud6c4\uc77c\uc5d0 \ub2f9\uc2dc \ud3ec\uc9c0\uc158\uc73c\ub85c \uc778\ud574 \uacaa\uc5c8\ub358 \uc5b4\ub824\uc6c0\uc744 \ub450\uace0 \"\uc88c\uce21 \uc794\ub514\uc5d0\uc11c \ub6f8 \ub54c\uba74 \ub9c8\uce58 \ubb3c\uc5d0 \ube60\uc9c4 \uc624\ub9ac \uac19\uc740 \uc678\ub85c\uc6c0\uc744 \ub290\uaf08\ub2e4.\"\uace0 \ub2f5\ud588\ub2e4. \uc911\uacac\uc218 \ub9ac\uce58 \uc560\uc2dc\ubc88\uc5d0\uac8c \uc88c\uc775 \uc218\ube44\uc5d0 \ub300\ud55c \ub3c4\uc6c0\uc744 \ubc1b\uc73c\uba70 23\uacbd\uae30\uc5d0 \ub098\uc124 \uc218 \uc788\uc5c8\uace0 \uc2e4\ucc45\uc740 \ud558\ub098\ub9cc\uc744 \uae30\ub85d\ud588\ub2e4. 6\uc6d4\uc5d0 \uc811\uc5b4\ub4e4\uc5c8\uc744 \ub54c\uac00 \ub418\uc5b4\uc11c\uc57c \uc120\uc218 \uc2dc\uc808 1\ub8e8\ub97c \ub9e1\uc558\uc5c8\ub358 \ucc30\ub9ac \uadf8\ub9bc \ucf54\uce58\uc758 \ub3c4\uc6c0\uc73c\ub85c 1\ub8e8\uc218\ub85c \uc804\ud5a5\ud588\ub2e4. \uba54\uc774\uc800 \ub9ac\uadf8\ub294 1959\ub144\ubd80\ud130 62\ub144\uae4c\uc9c0 \ub9e4\ud574 \ub450 \ucc28\ub840 \uc62c\uc2a4\ud0c0\uc804\uc744 \uac1c\ucd5c\ud588\uace0, 1959\ub144 \uc62c\uc2a4\ud0c0\uc804\uc758 \uccab \uacbd\uae30\uc5d0\uc11c\ub294 \uba85\ub2e8\uc5d0 \ubf51\ud788\uc9c0 \ubabb\ud588\ub2e4. \uadf8\ub7ec\ub098 \ub450 \ubc88\uc9f8 \uacbd\uae30\uc5d0\uc120 \uc608\ube44 \uc120\uc218\ub85c \uc774\ub984\uc744 \uc62c\ub824 \ub300\ud0c0\ub85c \uacbd\uae30\uc5d0 \ub098\uc124 \uc218 \uc788\uc5c8\ub2e4."} {"question": "\uc870\ub974\uc8fc \ub974\uba54\ud2b8\ub974\uac00 1922\ub144 \ubc1d\ud78c \uac00\uc124\uc740?", "paragraph": "\ud504\ub791\uc2a4\uc758 \uc2e0\ud559\uc790\uc774\uc790 \ucc9c\ubb38\ud559\uc790\uc774\ub358 \uc870\ub974\uc8fc \ub974\uba54\ud2b8\ub974\ub294 1922\ub144\uc5d0 \uc6b0\uc8fc\uc758 \uae30\uc6d0\uc5d0 \ub300\ud558\uc5ec, \ud6c4\uc5d0 \ub300\ud3ed\ubc1c \uc774\ub860\uc774\ub77c \ubd88\ub9ac\uac8c \ub418\ub294 \ucd94\uce21\uc744 \ud558\uc600\ub294\ub370, \uadf8\ub294 \uc774\uac83\uc744 \"\uc6d0\uc2dc\uc6d0\uc790\uc5d0 \ub300\ud55c \uac00\uc124\"\uc774\ub77c \ubd88\ub800\ub2e4. \uc774 \ubaa8\ud615\uc758 \ud2c0\uc740 \uc54c\ubca0\ub974\ud2b8 \uc544\uc778\uc288\ud0c0\uc778\uc758 \uc77c\ubc18 \uc0c1\ub300\uc131 \uc774\ub860\uacfc \uacf5\uac04\uc758 \uade0\uc9c8\uc131\uacfc \ub4f1\ubc29\uc131\uacfc \uac19\uc740 \ub2e8\uc21c\ud654 \uac00\uc815\uc744 \uae30\ubc18\uc73c\ub85c \ud55c\ub2e4. \ub300\ud3ed\ubc1c \uc774\ub860\uc758 \uc8fc\uc694 \ubc29\uc815\uc2dd\uc778 \ud504\ub9ac\ub4dc\ub9cc \ubc29\uc815\uc2dd\uc740 \uc54c\ub809\uc0b0\ub4dc\ub974 \ud504\ub9ac\ub4dc\ub9cc\uc5d0 \uc758\ud574 \uacf5\uc2dd\ud654\ub418\uc5c8\ub2e4. \ubbf8\uad6d\uc758 \ucc9c\ubb38\ud559\uc790\uc778 \uc5d0\ub4dc\uc708 \ud5c8\ube14\uc740 1929\ub144 \uba40\ub9ac \ub5a8\uc5b4\uc9c4 \uc740\ud558\ub4e4\uc758 \uac70\ub9ac\uac00 \uadf8\uac83\ub4e4\uc758 \uc801\uc0c9 \ud3b8\uc774\uc640 \ube44\ub840\ud558\ub2e4\ub294 \uac83\uc744 \ubc1c\uacac\ud588\ub2e4. 1964\ub144\uc5d0\ub294 \uc6b0\uc8fc\uc758 \uadf9\ucd08\ub2e8\ud30c\ub97c \uc5f0\uad6c\ud558\ub358 \ub450 \ubbf8\uad6d\uc778 \ucc9c\ubb38\ud559\uc790\ub4e4\uc778 \ub85c\ubc84\ud2b8 \uc6b0\ub4dc\ub85c \uc70c\uc2a8\uacfc \uc544\ub178 \uc568\ub7f0 \ud39c\uc9c0\uc5b4\uc2a4\uac00 \uc6b0\uc8fc\uc5d0\uc11c \uc18c\uc74c\uc774 \ub09c\ub2e4\ub294 \uc0ac\uc2e4\uc744 \ubc1c\uacac\ud588\ub2e4. \uc774 \uc18c\uc74c\uc740 \uc5b4\ub5a4 \ud55c \uc601\uc5ed\uc5d0\uc11c \ub098\uc624\ub294 \uac83\uc774 \uc544\ub2c8\ub77c, \uc6b0\uc8fc\uc758 \uc804\uc5ed\uc5d0\uc11c \ubc1c\uc0dd\ud588\ub2e4. \uc774\uac83\uc774 \uc6b0\uc8fc \ub9c8\uc774\ud06c\ub85c\ud30c \ubc30\uacbd\uc73c\ub85c, \ub300\ud3ed\ubc1c\uc5d0\uc11c \ubc1c\uc0dd\ud55c \uc804\uc790\uae30\ud30c\uac00 \uacf5\uac04\uc758 \ud33d\ucc3d\uacfc \ud568\uaed8 \ub298\uc5b4\ub098 \ud30c\uc7a5\uc774 \uae38\uc5b4\uc9c4 \uac83\uc774\ub2e4. \ub9cc\uc77c \ud604\uc7ac \uc740\ud558 \ud074\ub7ec\uc2a4\ud130\ub4e4 \uac04\uc758 \uac70\ub9ac\uac00 \uc810\ucc28 \uba40\uc5b4\uc9c0\uace0 \uc788\ub2e4\uba74, \uacfc\uac70\uc5d0\ub294 \ubaa8\ub450\uac00 \uc11c\ub85c \uac00\uae4c\uc774 \ubaa8\uc5ec\uc788\uc5c8\uc744 \uac83\uc774\ub2e4. \uc774\ub7ec\ud55c \ubc1c\uc0c1\uc740 \uacb0\uad6d \uadf9\ub3c4\ub85c \ubc00\uc9d1\ub418\uace0 \uadf9\ub3c4\ub85c \ub728\uac70\uc6e0\ub358 \uc2dc\uc810\uc774 \uacfc\uac70\uc5d0 \uc874\uc7ac\ud588\uc744 \uac83\uc774\ub77c\ub294 \ucd94\uce21\uc73c\ub85c \uadc0\uacb0\ub418\uc5c8\uace0, \uc774 \uc774\ub860\uacfc \ube44\uc2b7\ud55c \uc0c1\ud669\uc744 \uc7ac\ud604\ud558\uace0 \ud655\uc778\ud558\uae30 \uc704\ud574 \ucee4\ub2e4\ub780 \uc785\uc790 \uac00\uc18d\uae30\uac00 \ub9cc\ub4e4\uc5b4\uc84c\uc9c0\ub9cc, \uc785\uc790 \uac00\uc18d\uae30\ub294 \uacb0\uad6d \uc774\ub7ec\ud55c \uace0\uc5d0\ub108\uc9c0\uc601\uc5ed\uc744 \uc870\uc0ac\ud558\ub294 \ub370 \uae30\ub2a5\uc801 \ud55c\uacc4\ub97c \ub098\ud0c0\ub0c8\ub2e4. \ub300\ud3ed\ubc1c \uc774\ub860\uc774 \ucd5c\ucd08\uc758 \ud33d\ucc3d \uc774\ud6c4 \uc6b0\uc8fc\uc758 \uc77c\ubc18\uc801\uc778 \ubcc0\ud654\uc5d0 \ub300\ud574 \uc124\uba85\ud574\ub0bc \uc218 \uc788\ub2e4 \ud558\ub354\ub77c\ub3c4, \ud33d\ucc3d \uc9c1\ud6c4\uc640 \uc5f0\uad00\ub41c \uc544\ubb34\ub7f0 \uc99d\uac70\ub3c4 \uc5c6\uc774\ub294 \uc774\ub7ec\ud55c \uae30\ubcf8\uc801\uc778 \uc0c1\ud669\uc5d0 \ub300\ud574 \uc5b4\ub5a0\ud55c \uc785\uc99d\ub3c4 \ud560 \uc218 \uc5c6\ub2e4. \uc6b0\uc8fc\ub97c \ud1b5\ud2c0\uc5b4 \ubcf4\uc774\ub294 \ube5b\uc5d0 \ub300\ud55c \uad00\uce21 \uacb0\uacfc\ub294, \ub300\ud3ed\ubc1c \ud575\ud569\uc131\uc5d0 \ucda9\ubd84\ud788 \ub17c\ub9ac\uc801\uc73c\ub85c \uc124\uba85\ub41c \uc608\uce21, \uc989 \uc6b0\uc8fc \ucc98\uc74c \uba87 \ubd84 \uac04\uc758 \uae09\uc18d\ud55c \ud33d\ucc3d\uacfc \ub0c9\uac01 \uc18d\uc5d0\uc11c \ubc1c\uc0dd\ud55c \ud575\ubc18\uc751\uc73c\ub85c\ubd80\ud130 \ud615\uc131\ub41c \ube5b\uc5d0 \ub300\ud55c \uacc4\uc0b0\uacfc \uac70\uc758 \ub9de\uc544 \ub5a8\uc5b4\uc84c\ub2e4.", "answer": "\uc6d0\uc2dc\uc6d0\uc790\uc5d0 \ub300\ud55c \uac00\uc124", "sentence": "\uadf8\ub294 \uc774\uac83\uc744 \" \uc6d0\uc2dc\uc6d0\uc790\uc5d0 \ub300\ud55c \uac00\uc124 \"\uc774\ub77c \ubd88\ub800\ub2e4.", "paragraph_sentence": "\ud504\ub791\uc2a4\uc758 \uc2e0\ud559\uc790\uc774\uc790 \ucc9c\ubb38\ud559\uc790\uc774\ub358 \uc870\ub974\uc8fc \ub974\uba54\ud2b8\ub974\ub294 1922\ub144\uc5d0 \uc6b0\uc8fc\uc758 \uae30\uc6d0\uc5d0 \ub300\ud558\uc5ec, \ud6c4\uc5d0 \ub300\ud3ed\ubc1c \uc774\ub860\uc774\ub77c \ubd88\ub9ac\uac8c \ub418\ub294 \ucd94\uce21\uc744 \ud558\uc600\ub294\ub370, \uadf8\ub294 \uc774\uac83\uc744 \" \uc6d0\uc2dc\uc6d0\uc790\uc5d0 \ub300\ud55c \uac00\uc124 \"\uc774\ub77c \ubd88\ub800\ub2e4. \uc774 \ubaa8\ud615\uc758 \ud2c0\uc740 \uc54c\ubca0\ub974\ud2b8 \uc544\uc778\uc288\ud0c0\uc778\uc758 \uc77c\ubc18 \uc0c1\ub300\uc131 \uc774\ub860\uacfc \uacf5\uac04\uc758 \uade0\uc9c8\uc131\uacfc \ub4f1\ubc29\uc131\uacfc \uac19\uc740 \ub2e8\uc21c\ud654 \uac00\uc815\uc744 \uae30\ubc18\uc73c\ub85c \ud55c\ub2e4. \ub300\ud3ed\ubc1c \uc774\ub860\uc758 \uc8fc\uc694 \ubc29\uc815\uc2dd\uc778 \ud504\ub9ac\ub4dc\ub9cc \ubc29\uc815\uc2dd\uc740 \uc54c\ub809\uc0b0\ub4dc\ub974 \ud504\ub9ac\ub4dc\ub9cc\uc5d0 \uc758\ud574 \uacf5\uc2dd\ud654\ub418\uc5c8\ub2e4. \ubbf8\uad6d\uc758 \ucc9c\ubb38\ud559\uc790\uc778 \uc5d0\ub4dc\uc708 \ud5c8\ube14\uc740 1929\ub144 \uba40\ub9ac \ub5a8\uc5b4\uc9c4 \uc740\ud558\ub4e4\uc758 \uac70\ub9ac\uac00 \uadf8\uac83\ub4e4\uc758 \uc801\uc0c9 \ud3b8\uc774\uc640 \ube44\ub840\ud558\ub2e4\ub294 \uac83\uc744 \ubc1c\uacac\ud588\ub2e4. 1964\ub144\uc5d0\ub294 \uc6b0\uc8fc\uc758 \uadf9\ucd08\ub2e8\ud30c\ub97c \uc5f0\uad6c\ud558\ub358 \ub450 \ubbf8\uad6d\uc778 \ucc9c\ubb38\ud559\uc790\ub4e4\uc778 \ub85c\ubc84\ud2b8 \uc6b0\ub4dc\ub85c \uc70c\uc2a8\uacfc \uc544\ub178 \uc568\ub7f0 \ud39c\uc9c0\uc5b4\uc2a4\uac00 \uc6b0\uc8fc\uc5d0\uc11c \uc18c\uc74c\uc774 \ub09c\ub2e4\ub294 \uc0ac\uc2e4\uc744 \ubc1c\uacac\ud588\ub2e4. \uc774 \uc18c\uc74c\uc740 \uc5b4\ub5a4 \ud55c \uc601\uc5ed\uc5d0\uc11c \ub098\uc624\ub294 \uac83\uc774 \uc544\ub2c8\ub77c, \uc6b0\uc8fc\uc758 \uc804\uc5ed\uc5d0\uc11c \ubc1c\uc0dd\ud588\ub2e4. \uc774\uac83\uc774 \uc6b0\uc8fc \ub9c8\uc774\ud06c\ub85c\ud30c \ubc30\uacbd\uc73c\ub85c, \ub300\ud3ed\ubc1c\uc5d0\uc11c \ubc1c\uc0dd\ud55c \uc804\uc790\uae30\ud30c\uac00 \uacf5\uac04\uc758 \ud33d\ucc3d\uacfc \ud568\uaed8 \ub298\uc5b4\ub098 \ud30c\uc7a5\uc774 \uae38\uc5b4\uc9c4 \uac83\uc774\ub2e4. \ub9cc\uc77c \ud604\uc7ac \uc740\ud558 \ud074\ub7ec\uc2a4\ud130\ub4e4 \uac04\uc758 \uac70\ub9ac\uac00 \uc810\ucc28 \uba40\uc5b4\uc9c0\uace0 \uc788\ub2e4\uba74, \uacfc\uac70\uc5d0\ub294 \ubaa8\ub450\uac00 \uc11c\ub85c \uac00\uae4c\uc774 \ubaa8\uc5ec\uc788\uc5c8\uc744 \uac83\uc774\ub2e4. \uc774\ub7ec\ud55c \ubc1c\uc0c1\uc740 \uacb0\uad6d \uadf9\ub3c4\ub85c \ubc00\uc9d1\ub418\uace0 \uadf9\ub3c4\ub85c \ub728\uac70\uc6e0\ub358 \uc2dc\uc810\uc774 \uacfc\uac70\uc5d0 \uc874\uc7ac\ud588\uc744 \uac83\uc774\ub77c\ub294 \ucd94\uce21\uc73c\ub85c \uadc0\uacb0\ub418\uc5c8\uace0, \uc774 \uc774\ub860\uacfc \ube44\uc2b7\ud55c \uc0c1\ud669\uc744 \uc7ac\ud604\ud558\uace0 \ud655\uc778\ud558\uae30 \uc704\ud574 \ucee4\ub2e4\ub780 \uc785\uc790 \uac00\uc18d\uae30\uac00 \ub9cc\ub4e4\uc5b4\uc84c\uc9c0\ub9cc, \uc785\uc790 \uac00\uc18d\uae30\ub294 \uacb0\uad6d \uc774\ub7ec\ud55c \uace0\uc5d0\ub108\uc9c0\uc601\uc5ed\uc744 \uc870\uc0ac\ud558\ub294 \ub370 \uae30\ub2a5\uc801 \ud55c\uacc4\ub97c \ub098\ud0c0\ub0c8\ub2e4. \ub300\ud3ed\ubc1c \uc774\ub860\uc774 \ucd5c\ucd08\uc758 \ud33d\ucc3d \uc774\ud6c4 \uc6b0\uc8fc\uc758 \uc77c\ubc18\uc801\uc778 \ubcc0\ud654\uc5d0 \ub300\ud574 \uc124\uba85\ud574\ub0bc \uc218 \uc788\ub2e4 \ud558\ub354\ub77c\ub3c4, \ud33d\ucc3d \uc9c1\ud6c4\uc640 \uc5f0\uad00\ub41c \uc544\ubb34\ub7f0 \uc99d\uac70\ub3c4 \uc5c6\uc774\ub294 \uc774\ub7ec\ud55c \uae30\ubcf8\uc801\uc778 \uc0c1\ud669\uc5d0 \ub300\ud574 \uc5b4\ub5a0\ud55c \uc785\uc99d\ub3c4 \ud560 \uc218 \uc5c6\ub2e4. \uc6b0\uc8fc\ub97c \ud1b5\ud2c0\uc5b4 \ubcf4\uc774\ub294 \ube5b\uc5d0 \ub300\ud55c \uad00\uce21 \uacb0\uacfc\ub294, \ub300\ud3ed\ubc1c \ud575\ud569\uc131\uc5d0 \ucda9\ubd84\ud788 \ub17c\ub9ac\uc801\uc73c\ub85c \uc124\uba85\ub41c \uc608\uce21, \uc989 \uc6b0\uc8fc \ucc98\uc74c \uba87 \ubd84 \uac04\uc758 \uae09\uc18d\ud55c \ud33d\ucc3d\uacfc \ub0c9\uac01 \uc18d\uc5d0\uc11c \ubc1c\uc0dd\ud55c \ud575\ubc18\uc751\uc73c\ub85c\ubd80\ud130 \ud615\uc131\ub41c \ube5b\uc5d0 \ub300\ud55c \uacc4\uc0b0\uacfc \uac70\uc758 \ub9de\uc544 \ub5a8\uc5b4\uc84c\ub2e4.", "paragraph_answer": "\ud504\ub791\uc2a4\uc758 \uc2e0\ud559\uc790\uc774\uc790 \ucc9c\ubb38\ud559\uc790\uc774\ub358 \uc870\ub974\uc8fc \ub974\uba54\ud2b8\ub974\ub294 1922\ub144\uc5d0 \uc6b0\uc8fc\uc758 \uae30\uc6d0\uc5d0 \ub300\ud558\uc5ec, \ud6c4\uc5d0 \ub300\ud3ed\ubc1c \uc774\ub860\uc774\ub77c \ubd88\ub9ac\uac8c \ub418\ub294 \ucd94\uce21\uc744 \ud558\uc600\ub294\ub370, \uadf8\ub294 \uc774\uac83\uc744 \" \uc6d0\uc2dc\uc6d0\uc790\uc5d0 \ub300\ud55c \uac00\uc124 \"\uc774\ub77c \ubd88\ub800\ub2e4. \uc774 \ubaa8\ud615\uc758 \ud2c0\uc740 \uc54c\ubca0\ub974\ud2b8 \uc544\uc778\uc288\ud0c0\uc778\uc758 \uc77c\ubc18 \uc0c1\ub300\uc131 \uc774\ub860\uacfc \uacf5\uac04\uc758 \uade0\uc9c8\uc131\uacfc \ub4f1\ubc29\uc131\uacfc \uac19\uc740 \ub2e8\uc21c\ud654 \uac00\uc815\uc744 \uae30\ubc18\uc73c\ub85c \ud55c\ub2e4. \ub300\ud3ed\ubc1c \uc774\ub860\uc758 \uc8fc\uc694 \ubc29\uc815\uc2dd\uc778 \ud504\ub9ac\ub4dc\ub9cc \ubc29\uc815\uc2dd\uc740 \uc54c\ub809\uc0b0\ub4dc\ub974 \ud504\ub9ac\ub4dc\ub9cc\uc5d0 \uc758\ud574 \uacf5\uc2dd\ud654\ub418\uc5c8\ub2e4. \ubbf8\uad6d\uc758 \ucc9c\ubb38\ud559\uc790\uc778 \uc5d0\ub4dc\uc708 \ud5c8\ube14\uc740 1929\ub144 \uba40\ub9ac \ub5a8\uc5b4\uc9c4 \uc740\ud558\ub4e4\uc758 \uac70\ub9ac\uac00 \uadf8\uac83\ub4e4\uc758 \uc801\uc0c9 \ud3b8\uc774\uc640 \ube44\ub840\ud558\ub2e4\ub294 \uac83\uc744 \ubc1c\uacac\ud588\ub2e4. 1964\ub144\uc5d0\ub294 \uc6b0\uc8fc\uc758 \uadf9\ucd08\ub2e8\ud30c\ub97c \uc5f0\uad6c\ud558\ub358 \ub450 \ubbf8\uad6d\uc778 \ucc9c\ubb38\ud559\uc790\ub4e4\uc778 \ub85c\ubc84\ud2b8 \uc6b0\ub4dc\ub85c \uc70c\uc2a8\uacfc \uc544\ub178 \uc568\ub7f0 \ud39c\uc9c0\uc5b4\uc2a4\uac00 \uc6b0\uc8fc\uc5d0\uc11c \uc18c\uc74c\uc774 \ub09c\ub2e4\ub294 \uc0ac\uc2e4\uc744 \ubc1c\uacac\ud588\ub2e4. \uc774 \uc18c\uc74c\uc740 \uc5b4\ub5a4 \ud55c \uc601\uc5ed\uc5d0\uc11c \ub098\uc624\ub294 \uac83\uc774 \uc544\ub2c8\ub77c, \uc6b0\uc8fc\uc758 \uc804\uc5ed\uc5d0\uc11c \ubc1c\uc0dd\ud588\ub2e4. \uc774\uac83\uc774 \uc6b0\uc8fc \ub9c8\uc774\ud06c\ub85c\ud30c \ubc30\uacbd\uc73c\ub85c, \ub300\ud3ed\ubc1c\uc5d0\uc11c \ubc1c\uc0dd\ud55c \uc804\uc790\uae30\ud30c\uac00 \uacf5\uac04\uc758 \ud33d\ucc3d\uacfc \ud568\uaed8 \ub298\uc5b4\ub098 \ud30c\uc7a5\uc774 \uae38\uc5b4\uc9c4 \uac83\uc774\ub2e4. \ub9cc\uc77c \ud604\uc7ac \uc740\ud558 \ud074\ub7ec\uc2a4\ud130\ub4e4 \uac04\uc758 \uac70\ub9ac\uac00 \uc810\ucc28 \uba40\uc5b4\uc9c0\uace0 \uc788\ub2e4\uba74, \uacfc\uac70\uc5d0\ub294 \ubaa8\ub450\uac00 \uc11c\ub85c \uac00\uae4c\uc774 \ubaa8\uc5ec\uc788\uc5c8\uc744 \uac83\uc774\ub2e4. \uc774\ub7ec\ud55c \ubc1c\uc0c1\uc740 \uacb0\uad6d \uadf9\ub3c4\ub85c \ubc00\uc9d1\ub418\uace0 \uadf9\ub3c4\ub85c \ub728\uac70\uc6e0\ub358 \uc2dc\uc810\uc774 \uacfc\uac70\uc5d0 \uc874\uc7ac\ud588\uc744 \uac83\uc774\ub77c\ub294 \ucd94\uce21\uc73c\ub85c \uadc0\uacb0\ub418\uc5c8\uace0, \uc774 \uc774\ub860\uacfc \ube44\uc2b7\ud55c \uc0c1\ud669\uc744 \uc7ac\ud604\ud558\uace0 \ud655\uc778\ud558\uae30 \uc704\ud574 \ucee4\ub2e4\ub780 \uc785\uc790 \uac00\uc18d\uae30\uac00 \ub9cc\ub4e4\uc5b4\uc84c\uc9c0\ub9cc, \uc785\uc790 \uac00\uc18d\uae30\ub294 \uacb0\uad6d \uc774\ub7ec\ud55c \uace0\uc5d0\ub108\uc9c0\uc601\uc5ed\uc744 \uc870\uc0ac\ud558\ub294 \ub370 \uae30\ub2a5\uc801 \ud55c\uacc4\ub97c \ub098\ud0c0\ub0c8\ub2e4. \ub300\ud3ed\ubc1c \uc774\ub860\uc774 \ucd5c\ucd08\uc758 \ud33d\ucc3d \uc774\ud6c4 \uc6b0\uc8fc\uc758 \uc77c\ubc18\uc801\uc778 \ubcc0\ud654\uc5d0 \ub300\ud574 \uc124\uba85\ud574\ub0bc \uc218 \uc788\ub2e4 \ud558\ub354\ub77c\ub3c4, \ud33d\ucc3d \uc9c1\ud6c4\uc640 \uc5f0\uad00\ub41c \uc544\ubb34\ub7f0 \uc99d\uac70\ub3c4 \uc5c6\uc774\ub294 \uc774\ub7ec\ud55c \uae30\ubcf8\uc801\uc778 \uc0c1\ud669\uc5d0 \ub300\ud574 \uc5b4\ub5a0\ud55c \uc785\uc99d\ub3c4 \ud560 \uc218 \uc5c6\ub2e4. \uc6b0\uc8fc\ub97c \ud1b5\ud2c0\uc5b4 \ubcf4\uc774\ub294 \ube5b\uc5d0 \ub300\ud55c \uad00\uce21 \uacb0\uacfc\ub294, \ub300\ud3ed\ubc1c \ud575\ud569\uc131\uc5d0 \ucda9\ubd84\ud788 \ub17c\ub9ac\uc801\uc73c\ub85c \uc124\uba85\ub41c \uc608\uce21, \uc989 \uc6b0\uc8fc \ucc98\uc74c \uba87 \ubd84 \uac04\uc758 \uae09\uc18d\ud55c \ud33d\ucc3d\uacfc \ub0c9\uac01 \uc18d\uc5d0\uc11c \ubc1c\uc0dd\ud55c \ud575\ubc18\uc751\uc73c\ub85c\ubd80\ud130 \ud615\uc131\ub41c \ube5b\uc5d0 \ub300\ud55c \uacc4\uc0b0\uacfc \uac70\uc758 \ub9de\uc544 \ub5a8\uc5b4\uc84c\ub2e4.", "sentence_answer": "\uadf8\ub294 \uc774\uac83\uc744 \" \uc6d0\uc2dc\uc6d0\uc790\uc5d0 \ub300\ud55c \uac00\uc124 \"\uc774\ub77c \ubd88\ub800\ub2e4.", "paragraph_id": "6526307-1-0", "paragraph_question": "question: \uc870\ub974\uc8fc \ub974\uba54\ud2b8\ub974\uac00 1922\ub144 \ubc1d\ud78c \uac00\uc124\uc740?, context: \ud504\ub791\uc2a4\uc758 \uc2e0\ud559\uc790\uc774\uc790 \ucc9c\ubb38\ud559\uc790\uc774\ub358 \uc870\ub974\uc8fc \ub974\uba54\ud2b8\ub974\ub294 1922\ub144\uc5d0 \uc6b0\uc8fc\uc758 \uae30\uc6d0\uc5d0 \ub300\ud558\uc5ec, \ud6c4\uc5d0 \ub300\ud3ed\ubc1c \uc774\ub860\uc774\ub77c \ubd88\ub9ac\uac8c \ub418\ub294 \ucd94\uce21\uc744 \ud558\uc600\ub294\ub370, \uadf8\ub294 \uc774\uac83\uc744 \"\uc6d0\uc2dc\uc6d0\uc790\uc5d0 \ub300\ud55c \uac00\uc124\"\uc774\ub77c \ubd88\ub800\ub2e4. \uc774 \ubaa8\ud615\uc758 \ud2c0\uc740 \uc54c\ubca0\ub974\ud2b8 \uc544\uc778\uc288\ud0c0\uc778\uc758 \uc77c\ubc18 \uc0c1\ub300\uc131 \uc774\ub860\uacfc \uacf5\uac04\uc758 \uade0\uc9c8\uc131\uacfc \ub4f1\ubc29\uc131\uacfc \uac19\uc740 \ub2e8\uc21c\ud654 \uac00\uc815\uc744 \uae30\ubc18\uc73c\ub85c \ud55c\ub2e4. \ub300\ud3ed\ubc1c \uc774\ub860\uc758 \uc8fc\uc694 \ubc29\uc815\uc2dd\uc778 \ud504\ub9ac\ub4dc\ub9cc \ubc29\uc815\uc2dd\uc740 \uc54c\ub809\uc0b0\ub4dc\ub974 \ud504\ub9ac\ub4dc\ub9cc\uc5d0 \uc758\ud574 \uacf5\uc2dd\ud654\ub418\uc5c8\ub2e4. \ubbf8\uad6d\uc758 \ucc9c\ubb38\ud559\uc790\uc778 \uc5d0\ub4dc\uc708 \ud5c8\ube14\uc740 1929\ub144 \uba40\ub9ac \ub5a8\uc5b4\uc9c4 \uc740\ud558\ub4e4\uc758 \uac70\ub9ac\uac00 \uadf8\uac83\ub4e4\uc758 \uc801\uc0c9 \ud3b8\uc774\uc640 \ube44\ub840\ud558\ub2e4\ub294 \uac83\uc744 \ubc1c\uacac\ud588\ub2e4. 1964\ub144\uc5d0\ub294 \uc6b0\uc8fc\uc758 \uadf9\ucd08\ub2e8\ud30c\ub97c \uc5f0\uad6c\ud558\ub358 \ub450 \ubbf8\uad6d\uc778 \ucc9c\ubb38\ud559\uc790\ub4e4\uc778 \ub85c\ubc84\ud2b8 \uc6b0\ub4dc\ub85c \uc70c\uc2a8\uacfc \uc544\ub178 \uc568\ub7f0 \ud39c\uc9c0\uc5b4\uc2a4\uac00 \uc6b0\uc8fc\uc5d0\uc11c \uc18c\uc74c\uc774 \ub09c\ub2e4\ub294 \uc0ac\uc2e4\uc744 \ubc1c\uacac\ud588\ub2e4. \uc774 \uc18c\uc74c\uc740 \uc5b4\ub5a4 \ud55c \uc601\uc5ed\uc5d0\uc11c \ub098\uc624\ub294 \uac83\uc774 \uc544\ub2c8\ub77c, \uc6b0\uc8fc\uc758 \uc804\uc5ed\uc5d0\uc11c \ubc1c\uc0dd\ud588\ub2e4. \uc774\uac83\uc774 \uc6b0\uc8fc \ub9c8\uc774\ud06c\ub85c\ud30c \ubc30\uacbd\uc73c\ub85c, \ub300\ud3ed\ubc1c\uc5d0\uc11c \ubc1c\uc0dd\ud55c \uc804\uc790\uae30\ud30c\uac00 \uacf5\uac04\uc758 \ud33d\ucc3d\uacfc \ud568\uaed8 \ub298\uc5b4\ub098 \ud30c\uc7a5\uc774 \uae38\uc5b4\uc9c4 \uac83\uc774\ub2e4. \ub9cc\uc77c \ud604\uc7ac \uc740\ud558 \ud074\ub7ec\uc2a4\ud130\ub4e4 \uac04\uc758 \uac70\ub9ac\uac00 \uc810\ucc28 \uba40\uc5b4\uc9c0\uace0 \uc788\ub2e4\uba74, \uacfc\uac70\uc5d0\ub294 \ubaa8\ub450\uac00 \uc11c\ub85c \uac00\uae4c\uc774 \ubaa8\uc5ec\uc788\uc5c8\uc744 \uac83\uc774\ub2e4. \uc774\ub7ec\ud55c \ubc1c\uc0c1\uc740 \uacb0\uad6d \uadf9\ub3c4\ub85c \ubc00\uc9d1\ub418\uace0 \uadf9\ub3c4\ub85c \ub728\uac70\uc6e0\ub358 \uc2dc\uc810\uc774 \uacfc\uac70\uc5d0 \uc874\uc7ac\ud588\uc744 \uac83\uc774\ub77c\ub294 \ucd94\uce21\uc73c\ub85c \uadc0\uacb0\ub418\uc5c8\uace0, \uc774 \uc774\ub860\uacfc \ube44\uc2b7\ud55c \uc0c1\ud669\uc744 \uc7ac\ud604\ud558\uace0 \ud655\uc778\ud558\uae30 \uc704\ud574 \ucee4\ub2e4\ub780 \uc785\uc790 \uac00\uc18d\uae30\uac00 \ub9cc\ub4e4\uc5b4\uc84c\uc9c0\ub9cc, \uc785\uc790 \uac00\uc18d\uae30\ub294 \uacb0\uad6d \uc774\ub7ec\ud55c \uace0\uc5d0\ub108\uc9c0\uc601\uc5ed\uc744 \uc870\uc0ac\ud558\ub294 \ub370 \uae30\ub2a5\uc801 \ud55c\uacc4\ub97c \ub098\ud0c0\ub0c8\ub2e4. \ub300\ud3ed\ubc1c \uc774\ub860\uc774 \ucd5c\ucd08\uc758 \ud33d\ucc3d \uc774\ud6c4 \uc6b0\uc8fc\uc758 \uc77c\ubc18\uc801\uc778 \ubcc0\ud654\uc5d0 \ub300\ud574 \uc124\uba85\ud574\ub0bc \uc218 \uc788\ub2e4 \ud558\ub354\ub77c\ub3c4, \ud33d\ucc3d \uc9c1\ud6c4\uc640 \uc5f0\uad00\ub41c \uc544\ubb34\ub7f0 \uc99d\uac70\ub3c4 \uc5c6\uc774\ub294 \uc774\ub7ec\ud55c \uae30\ubcf8\uc801\uc778 \uc0c1\ud669\uc5d0 \ub300\ud574 \uc5b4\ub5a0\ud55c \uc785\uc99d\ub3c4 \ud560 \uc218 \uc5c6\ub2e4. \uc6b0\uc8fc\ub97c \ud1b5\ud2c0\uc5b4 \ubcf4\uc774\ub294 \ube5b\uc5d0 \ub300\ud55c \uad00\uce21 \uacb0\uacfc\ub294, \ub300\ud3ed\ubc1c \ud575\ud569\uc131\uc5d0 \ucda9\ubd84\ud788 \ub17c\ub9ac\uc801\uc73c\ub85c \uc124\uba85\ub41c \uc608\uce21, \uc989 \uc6b0\uc8fc \ucc98\uc74c \uba87 \ubd84 \uac04\uc758 \uae09\uc18d\ud55c \ud33d\ucc3d\uacfc \ub0c9\uac01 \uc18d\uc5d0\uc11c \ubc1c\uc0dd\ud55c \ud575\ubc18\uc751\uc73c\ub85c\ubd80\ud130 \ud615\uc131\ub41c \ube5b\uc5d0 \ub300\ud55c \uacc4\uc0b0\uacfc \uac70\uc758 \ub9de\uc544 \ub5a8\uc5b4\uc84c\ub2e4."} {"question": "1923\ub144 \uc789\uae00\ub79c\ub4dc FA\ucef5 \uacb0\uc2b9\uc804\uc5d0\uc11c \ubd80\uc0c1\ub2f9\ud55c \uad00\uc911\ub4e4\uc5d0\uac8c \uc751\uae09\ucc98\uce58\ub97c \ud55c \ub2e8\uccb4\ub294?", "paragraph": "\uc591 \ud300 \ubaa8\ub450 \uadf8 \ub2f9\uc2dc\uc5d0 \uac00\uc7a5 \uc804\ud615\uc801\uc778 \ud3ec\uba54\uc774\uc158\uc744 \ub4e4\uace0 \ub098\uc654\ub294\ub370, \uc804\uc220\uc758 \uae30\ubcf8 \ud615\ud0dc\ub294 2\uba85\uc758 \ud480\ubc31\uc774 \ub4a4\uc5d0 \uc704\uce58\ud558\uace0 \ud55c \uba85\uc758 \uc13c\ud130 \ud558\ud504\uc640 \ub450 \uba85\uc758 \uc719 \ud558\ud504\ub85c \uad6c\uc131\ub41c 3\uba85\uc758 \ud558\ud504\ubc31\uc774 \uadf8 \uc55e\uc5d0 \uc11c\uc11c \uc218\ube44\ub97c \ub2f4\ub2f9\ud558\uace0 \ub9c8\uc9c0\ub9c9\uc73c\ub85c 2\uba85\uc758 \uc544\uc6c3\uc0ac\uc774\ub4dc \ud3ec\uc6cc\ub4dc, \ub450 \uba85\uc758 \uc778\uc0ac\uc774\ub4dc \ud3ec\uc6cc\ub4dc, \uadf8\ub9ac\uace0 \ud55c \uba85\uc758 \uc13c\ud130 \ud3ec\uc6cc\ub4dc\ub85c \uad6c\uc131\ub41c 5\uba85\uc758 \ud3ec\uc6cc\ub4dc\uac00 \uacf5\uaca9\uc744 \ub9e1\ub294 \uac83\uc774\ub2e4. \uc6e8\uc2a4\ud2b8 \ud584\uc740 \ub450 \ube60\ub978 \uc544\uc6c3\uc0ac\uc774\ub4dc \ud3ec\uc6cc\ub4dc\uc778 \ub515 \ub9ac\ucc28\uc988\uc640 \uc9c0\ubbf8 \ub7ec\ud3a0\uc744 \uc911\uc2ec\uc73c\ub85c \ud558\ub294 \uc804\ub7b5\uc744 \uad6c\uc131\ud588\uc73c\ub098 \ubcfc\ud134\uc740 \uc2dc\uc791\ud558\uc790\ub9c8\uc790 \uadf8 \ub450 \uc120\uc218\ub97c \uc798 \uc6c0\uc9c1\uc774\uc9c0 \ubabb\ud558\uac8c \ubb36\uc5b4\ub450\uc5c8\uace0, \ub610\ud55c \uadf8 \ub450 \uc120\uc218\uac00 \uacf5\uc744 \uc7a1\uc790\ub9c8\uc790 \ub2ec\ub824\ub4e4\uc5b4 \ubc29\ud574\ud558\ub294 \ub9de\ucda4\uc804\ub7b5\uc744 \uc37c\ub2e4. \uacbd\uae30 \uc2dc\uc791 2\ubd84 \ud6c4 \uc6e8\uc2a4\ud2b8 \ud584\uc758 \ud558\ud504\ubc31\uc778 \uc7ad \ud2b8\ub808\uc0ac\ub358\uc774 \uc2a4\ub85c\uc778\uc744 \ub358\uc9c4 \ud6c4\uc5d0 \uad00\uc911\ub4e4\uc5d0 \ud729\uc4f8\ub824 \uadf8\ub77c\uc6b4\ub4dc\ub85c \ubcf5\uadc0\ud558\uc9c0 \ubabb\ud558\ub294 \uc0c1\ud669\uc774 \ubc8c\uc5b4\uc84c\ub2e4. \uc774 \uae30\ud68c\ub97c \ub193\uce58\uc9c0 \uc54a\uc740 \ubcfc\ud134\uc758 \ub370\uc774\ube44\ub4dc \uc7ad\uc774 \uc120\uc218 \uc22b\uc790\uc758 \uc6b0\uc704\ub97c \uc810\ud55c \uc0c1\ud669\uc5d0\uc11c \uc288\ud305\uc744 \ud558\uc600\ub2e4. \uc6e8\uc2a4\ud2b8 \ud584\uc758 \uace8\ud0a4\ud37c \ud14c\ub4dc \ud5c8\ud504\ud2bc\uc774 \uc774 \uc29b\uc744 \ub9c9\uc9c0 \ubabb\ud558\uace0 \uc2e4\uc810\uc744 \ud5c8\uc6a9\ud574 \ubcfc\ud134\uc774 \uc55e\uc11c\ub098\uac00\uae30 \uc2dc\uc791\ud588\ub2e4. \uadf8\ub54c \uadf8 \uacf5\uc774 \ub2e4\ub978 \uad00\uc911\ub4e4\uc5d0 \ubc00\ub824\ub0b4\ub824\uc640 \uace8\ub124\ud2b8\uc5d0 \ubc14\uc9dd \ubd99\uc5b4\uc788\ub358 \ud55c \uad00\uc911\uc744 \uac15\ud0c0\ud558\uba74\uc11c \uadf8 \uad00\uc911\uc774 \ubb34\uc758\uc2dd \uc0c1\ud0dc\uac00 \ub418\uc5b4\ubc84\ub838\ub2e4. \uc2e4\uc810 3\ubd84 \ud6c4 \uc6e8\uc2a4\ud2b8 \ud584\uc758 \ube45 \uc653\uc2a8\uc774 \ubcfc\ud134\uc758 \uace8\ubb38 \ubc14\ub85c \uc55e\uc5d0\uc11c \uacf5\uc744 \uc7a1\uc544 \ucc9c\uae08\uac19\uc740 \uae30\ud68c\ub97c \uc7a1\uc558\uc73c\ub098 \uc29b\uc774 \ud06c\ub85c\uc2a4\ubc14\ub97c \ub118\uc5b4\uac00\ubc84\ub838\ub2e4. \uc804\ubc18 11\ubd84 \uacbd\uc5d0\ub294 \ub610\ub2e4\uc2dc \uad00\uc911\ub4e4\uc774 \uc55e\uc73c\ub85c \ubab0\ub9ac\uba74\uc11c \ub2e4\uc218\uc758 \ud32c\ub4e4\uc774 \uadf8\ub77c\uc6b4\ub4dc \uc548\uc73c\ub85c \uc9c4\uc785\ud558\ub294 \ubc14\ub78c\uc5d0 \ubd80\uc2ec\ub4e4\uc744 \uc704\ud574 \ub9d0\uc744 \ud0c4 \uacbd\ucc30\ub4e4\uc774 \uc0c1\ud669\uc744 \uc218\uc2b5\ud560 \ub54c\uae4c\uc9c0 \uacbd\uae30\uac00 \uc9c0\uc5f0\ub418\uc5c8\ub2e4. \uc120\uc218\ub4e4\uc774 \uacbd\uae30 \uc7ac\uac1c\ub97c \uae30\ub2e4\ub9ac\ub294 \ub3d9\uc548 \uba87\uba87 \uad00\uc911\ub4e4\uc740 \ub9ce\uc740 \uad00\uc911 \uc18d\uc5d0\uc11c \ub2f9\ud55c \ubd80\uc0c1 \ub54c\ubb38\uc5d0 \uc601\uad6d \uc801\uc2ed\uc790\uc758 \uc751\uae09\ucc98\uce58\uac00 \ud544\uc694\ud558\uae30\ub3c4 \ud588\ub2e4. \uacbd\ucc30\ub4e4\uc740 \uadf8\ub77c\uc6b4\ub4dc \uc8fc\ubcc0\uc744 \ub3cc\uba70 \ub354\uc774\uc0c1 \uad00\uc911\ub4e4\uc774 \uc120\uc744 \ub118\uc5b4 \uc9c4\uc785\ud558\uc9c0 \ubabb\ud558\ub3c4\ub85d \ud588\uace0 \uadf8 \uc0ac\uc774 \uacbd\uae30\uac00 \uc7ac\uac1c\ub418\uc5c8\ub2e4.", "answer": "\uc601\uad6d \uc801\uc2ed\uc790", "sentence": "\uc120\uc218\ub4e4\uc774 \uacbd\uae30 \uc7ac\uac1c\ub97c \uae30\ub2e4\ub9ac\ub294 \ub3d9\uc548 \uba87\uba87 \uad00\uc911\ub4e4\uc740 \ub9ce\uc740 \uad00\uc911 \uc18d\uc5d0\uc11c \ub2f9\ud55c \ubd80\uc0c1 \ub54c\ubb38\uc5d0 \uc601\uad6d \uc801\uc2ed\uc790 \uc758 \uc751\uae09\ucc98\uce58\uac00 \ud544\uc694\ud558\uae30\ub3c4 \ud588\ub2e4.", "paragraph_sentence": "\uc591 \ud300 \ubaa8\ub450 \uadf8 \ub2f9\uc2dc\uc5d0 \uac00\uc7a5 \uc804\ud615\uc801\uc778 \ud3ec\uba54\uc774\uc158\uc744 \ub4e4\uace0 \ub098\uc654\ub294\ub370, \uc804\uc220\uc758 \uae30\ubcf8 \ud615\ud0dc\ub294 2\uba85\uc758 \ud480\ubc31\uc774 \ub4a4\uc5d0 \uc704\uce58\ud558\uace0 \ud55c \uba85\uc758 \uc13c\ud130 \ud558\ud504\uc640 \ub450 \uba85\uc758 \uc719 \ud558\ud504\ub85c \uad6c\uc131\ub41c 3\uba85\uc758 \ud558\ud504\ubc31\uc774 \uadf8 \uc55e\uc5d0 \uc11c\uc11c \uc218\ube44\ub97c \ub2f4\ub2f9\ud558\uace0 \ub9c8\uc9c0\ub9c9\uc73c\ub85c 2\uba85\uc758 \uc544\uc6c3\uc0ac\uc774\ub4dc \ud3ec\uc6cc\ub4dc, \ub450 \uba85\uc758 \uc778\uc0ac\uc774\ub4dc \ud3ec\uc6cc\ub4dc, \uadf8\ub9ac\uace0 \ud55c \uba85\uc758 \uc13c\ud130 \ud3ec\uc6cc\ub4dc\ub85c \uad6c\uc131\ub41c 5\uba85\uc758 \ud3ec\uc6cc\ub4dc\uac00 \uacf5\uaca9\uc744 \ub9e1\ub294 \uac83\uc774\ub2e4. \uc6e8\uc2a4\ud2b8 \ud584\uc740 \ub450 \ube60\ub978 \uc544\uc6c3\uc0ac\uc774\ub4dc \ud3ec\uc6cc\ub4dc\uc778 \ub515 \ub9ac\ucc28\uc988\uc640 \uc9c0\ubbf8 \ub7ec\ud3a0\uc744 \uc911\uc2ec\uc73c\ub85c \ud558\ub294 \uc804\ub7b5\uc744 \uad6c\uc131\ud588\uc73c\ub098 \ubcfc\ud134\uc740 \uc2dc\uc791\ud558\uc790\ub9c8\uc790 \uadf8 \ub450 \uc120\uc218\ub97c \uc798 \uc6c0\uc9c1\uc774\uc9c0 \ubabb\ud558\uac8c \ubb36\uc5b4\ub450\uc5c8\uace0, \ub610\ud55c \uadf8 \ub450 \uc120\uc218\uac00 \uacf5\uc744 \uc7a1\uc790\ub9c8\uc790 \ub2ec\ub824\ub4e4\uc5b4 \ubc29\ud574\ud558\ub294 \ub9de\ucda4\uc804\ub7b5\uc744 \uc37c\ub2e4. \uacbd\uae30 \uc2dc\uc791 2\ubd84 \ud6c4 \uc6e8\uc2a4\ud2b8 \ud584\uc758 \ud558\ud504\ubc31\uc778 \uc7ad \ud2b8\ub808\uc0ac\ub358\uc774 \uc2a4\ub85c\uc778\uc744 \ub358\uc9c4 \ud6c4\uc5d0 \uad00\uc911\ub4e4\uc5d0 \ud729\uc4f8\ub824 \uadf8\ub77c\uc6b4\ub4dc\ub85c \ubcf5\uadc0\ud558\uc9c0 \ubabb\ud558\ub294 \uc0c1\ud669\uc774 \ubc8c\uc5b4\uc84c\ub2e4. \uc774 \uae30\ud68c\ub97c \ub193\uce58\uc9c0 \uc54a\uc740 \ubcfc\ud134\uc758 \ub370\uc774\ube44\ub4dc \uc7ad\uc774 \uc120\uc218 \uc22b\uc790\uc758 \uc6b0\uc704\ub97c \uc810\ud55c \uc0c1\ud669\uc5d0\uc11c \uc288\ud305\uc744 \ud558\uc600\ub2e4. \uc6e8\uc2a4\ud2b8 \ud584\uc758 \uace8\ud0a4\ud37c \ud14c\ub4dc \ud5c8\ud504\ud2bc\uc774 \uc774 \uc29b\uc744 \ub9c9\uc9c0 \ubabb\ud558\uace0 \uc2e4\uc810\uc744 \ud5c8\uc6a9\ud574 \ubcfc\ud134\uc774 \uc55e\uc11c\ub098\uac00\uae30 \uc2dc\uc791\ud588\ub2e4. \uadf8\ub54c \uadf8 \uacf5\uc774 \ub2e4\ub978 \uad00\uc911\ub4e4\uc5d0 \ubc00\ub824\ub0b4\ub824\uc640 \uace8\ub124\ud2b8\uc5d0 \ubc14\uc9dd \ubd99\uc5b4\uc788\ub358 \ud55c \uad00\uc911\uc744 \uac15\ud0c0\ud558\uba74\uc11c \uadf8 \uad00\uc911\uc774 \ubb34\uc758\uc2dd \uc0c1\ud0dc\uac00 \ub418\uc5b4\ubc84\ub838\ub2e4. \uc2e4\uc810 3\ubd84 \ud6c4 \uc6e8\uc2a4\ud2b8 \ud584\uc758 \ube45 \uc653\uc2a8\uc774 \ubcfc\ud134\uc758 \uace8\ubb38 \ubc14\ub85c \uc55e\uc5d0\uc11c \uacf5\uc744 \uc7a1\uc544 \ucc9c\uae08\uac19\uc740 \uae30\ud68c\ub97c \uc7a1\uc558\uc73c\ub098 \uc29b\uc774 \ud06c\ub85c\uc2a4\ubc14\ub97c \ub118\uc5b4\uac00\ubc84\ub838\ub2e4. \uc804\ubc18 11\ubd84 \uacbd\uc5d0\ub294 \ub610\ub2e4\uc2dc \uad00\uc911\ub4e4\uc774 \uc55e\uc73c\ub85c \ubab0\ub9ac\uba74\uc11c \ub2e4\uc218\uc758 \ud32c\ub4e4\uc774 \uadf8\ub77c\uc6b4\ub4dc \uc548\uc73c\ub85c \uc9c4\uc785\ud558\ub294 \ubc14\ub78c\uc5d0 \ubd80\uc2ec\ub4e4\uc744 \uc704\ud574 \ub9d0\uc744 \ud0c4 \uacbd\ucc30\ub4e4\uc774 \uc0c1\ud669\uc744 \uc218\uc2b5\ud560 \ub54c\uae4c\uc9c0 \uacbd\uae30\uac00 \uc9c0\uc5f0\ub418\uc5c8\ub2e4. \uc120\uc218\ub4e4\uc774 \uacbd\uae30 \uc7ac\uac1c\ub97c \uae30\ub2e4\ub9ac\ub294 \ub3d9\uc548 \uba87\uba87 \uad00\uc911\ub4e4\uc740 \ub9ce\uc740 \uad00\uc911 \uc18d\uc5d0\uc11c \ub2f9\ud55c \ubd80\uc0c1 \ub54c\ubb38\uc5d0 \uc601\uad6d \uc801\uc2ed\uc790 \uc758 \uc751\uae09\ucc98\uce58\uac00 \ud544\uc694\ud558\uae30\ub3c4 \ud588\ub2e4. \uacbd\ucc30\ub4e4\uc740 \uadf8\ub77c\uc6b4\ub4dc \uc8fc\ubcc0\uc744 \ub3cc\uba70 \ub354\uc774\uc0c1 \uad00\uc911\ub4e4\uc774 \uc120\uc744 \ub118\uc5b4 \uc9c4\uc785\ud558\uc9c0 \ubabb\ud558\ub3c4\ub85d \ud588\uace0 \uadf8 \uc0ac\uc774 \uacbd\uae30\uac00 \uc7ac\uac1c\ub418\uc5c8\ub2e4.", "paragraph_answer": "\uc591 \ud300 \ubaa8\ub450 \uadf8 \ub2f9\uc2dc\uc5d0 \uac00\uc7a5 \uc804\ud615\uc801\uc778 \ud3ec\uba54\uc774\uc158\uc744 \ub4e4\uace0 \ub098\uc654\ub294\ub370, \uc804\uc220\uc758 \uae30\ubcf8 \ud615\ud0dc\ub294 2\uba85\uc758 \ud480\ubc31\uc774 \ub4a4\uc5d0 \uc704\uce58\ud558\uace0 \ud55c \uba85\uc758 \uc13c\ud130 \ud558\ud504\uc640 \ub450 \uba85\uc758 \uc719 \ud558\ud504\ub85c \uad6c\uc131\ub41c 3\uba85\uc758 \ud558\ud504\ubc31\uc774 \uadf8 \uc55e\uc5d0 \uc11c\uc11c \uc218\ube44\ub97c \ub2f4\ub2f9\ud558\uace0 \ub9c8\uc9c0\ub9c9\uc73c\ub85c 2\uba85\uc758 \uc544\uc6c3\uc0ac\uc774\ub4dc \ud3ec\uc6cc\ub4dc, \ub450 \uba85\uc758 \uc778\uc0ac\uc774\ub4dc \ud3ec\uc6cc\ub4dc, \uadf8\ub9ac\uace0 \ud55c \uba85\uc758 \uc13c\ud130 \ud3ec\uc6cc\ub4dc\ub85c \uad6c\uc131\ub41c 5\uba85\uc758 \ud3ec\uc6cc\ub4dc\uac00 \uacf5\uaca9\uc744 \ub9e1\ub294 \uac83\uc774\ub2e4. \uc6e8\uc2a4\ud2b8 \ud584\uc740 \ub450 \ube60\ub978 \uc544\uc6c3\uc0ac\uc774\ub4dc \ud3ec\uc6cc\ub4dc\uc778 \ub515 \ub9ac\ucc28\uc988\uc640 \uc9c0\ubbf8 \ub7ec\ud3a0\uc744 \uc911\uc2ec\uc73c\ub85c \ud558\ub294 \uc804\ub7b5\uc744 \uad6c\uc131\ud588\uc73c\ub098 \ubcfc\ud134\uc740 \uc2dc\uc791\ud558\uc790\ub9c8\uc790 \uadf8 \ub450 \uc120\uc218\ub97c \uc798 \uc6c0\uc9c1\uc774\uc9c0 \ubabb\ud558\uac8c \ubb36\uc5b4\ub450\uc5c8\uace0, \ub610\ud55c \uadf8 \ub450 \uc120\uc218\uac00 \uacf5\uc744 \uc7a1\uc790\ub9c8\uc790 \ub2ec\ub824\ub4e4\uc5b4 \ubc29\ud574\ud558\ub294 \ub9de\ucda4\uc804\ub7b5\uc744 \uc37c\ub2e4. \uacbd\uae30 \uc2dc\uc791 2\ubd84 \ud6c4 \uc6e8\uc2a4\ud2b8 \ud584\uc758 \ud558\ud504\ubc31\uc778 \uc7ad \ud2b8\ub808\uc0ac\ub358\uc774 \uc2a4\ub85c\uc778\uc744 \ub358\uc9c4 \ud6c4\uc5d0 \uad00\uc911\ub4e4\uc5d0 \ud729\uc4f8\ub824 \uadf8\ub77c\uc6b4\ub4dc\ub85c \ubcf5\uadc0\ud558\uc9c0 \ubabb\ud558\ub294 \uc0c1\ud669\uc774 \ubc8c\uc5b4\uc84c\ub2e4. \uc774 \uae30\ud68c\ub97c \ub193\uce58\uc9c0 \uc54a\uc740 \ubcfc\ud134\uc758 \ub370\uc774\ube44\ub4dc \uc7ad\uc774 \uc120\uc218 \uc22b\uc790\uc758 \uc6b0\uc704\ub97c \uc810\ud55c \uc0c1\ud669\uc5d0\uc11c \uc288\ud305\uc744 \ud558\uc600\ub2e4. \uc6e8\uc2a4\ud2b8 \ud584\uc758 \uace8\ud0a4\ud37c \ud14c\ub4dc \ud5c8\ud504\ud2bc\uc774 \uc774 \uc29b\uc744 \ub9c9\uc9c0 \ubabb\ud558\uace0 \uc2e4\uc810\uc744 \ud5c8\uc6a9\ud574 \ubcfc\ud134\uc774 \uc55e\uc11c\ub098\uac00\uae30 \uc2dc\uc791\ud588\ub2e4. \uadf8\ub54c \uadf8 \uacf5\uc774 \ub2e4\ub978 \uad00\uc911\ub4e4\uc5d0 \ubc00\ub824\ub0b4\ub824\uc640 \uace8\ub124\ud2b8\uc5d0 \ubc14\uc9dd \ubd99\uc5b4\uc788\ub358 \ud55c \uad00\uc911\uc744 \uac15\ud0c0\ud558\uba74\uc11c \uadf8 \uad00\uc911\uc774 \ubb34\uc758\uc2dd \uc0c1\ud0dc\uac00 \ub418\uc5b4\ubc84\ub838\ub2e4. \uc2e4\uc810 3\ubd84 \ud6c4 \uc6e8\uc2a4\ud2b8 \ud584\uc758 \ube45 \uc653\uc2a8\uc774 \ubcfc\ud134\uc758 \uace8\ubb38 \ubc14\ub85c \uc55e\uc5d0\uc11c \uacf5\uc744 \uc7a1\uc544 \ucc9c\uae08\uac19\uc740 \uae30\ud68c\ub97c \uc7a1\uc558\uc73c\ub098 \uc29b\uc774 \ud06c\ub85c\uc2a4\ubc14\ub97c \ub118\uc5b4\uac00\ubc84\ub838\ub2e4. \uc804\ubc18 11\ubd84 \uacbd\uc5d0\ub294 \ub610\ub2e4\uc2dc \uad00\uc911\ub4e4\uc774 \uc55e\uc73c\ub85c \ubab0\ub9ac\uba74\uc11c \ub2e4\uc218\uc758 \ud32c\ub4e4\uc774 \uadf8\ub77c\uc6b4\ub4dc \uc548\uc73c\ub85c \uc9c4\uc785\ud558\ub294 \ubc14\ub78c\uc5d0 \ubd80\uc2ec\ub4e4\uc744 \uc704\ud574 \ub9d0\uc744 \ud0c4 \uacbd\ucc30\ub4e4\uc774 \uc0c1\ud669\uc744 \uc218\uc2b5\ud560 \ub54c\uae4c\uc9c0 \uacbd\uae30\uac00 \uc9c0\uc5f0\ub418\uc5c8\ub2e4. \uc120\uc218\ub4e4\uc774 \uacbd\uae30 \uc7ac\uac1c\ub97c \uae30\ub2e4\ub9ac\ub294 \ub3d9\uc548 \uba87\uba87 \uad00\uc911\ub4e4\uc740 \ub9ce\uc740 \uad00\uc911 \uc18d\uc5d0\uc11c \ub2f9\ud55c \ubd80\uc0c1 \ub54c\ubb38\uc5d0 \uc601\uad6d \uc801\uc2ed\uc790 \uc758 \uc751\uae09\ucc98\uce58\uac00 \ud544\uc694\ud558\uae30\ub3c4 \ud588\ub2e4. \uacbd\ucc30\ub4e4\uc740 \uadf8\ub77c\uc6b4\ub4dc \uc8fc\ubcc0\uc744 \ub3cc\uba70 \ub354\uc774\uc0c1 \uad00\uc911\ub4e4\uc774 \uc120\uc744 \ub118\uc5b4 \uc9c4\uc785\ud558\uc9c0 \ubabb\ud558\ub3c4\ub85d \ud588\uace0 \uadf8 \uc0ac\uc774 \uacbd\uae30\uac00 \uc7ac\uac1c\ub418\uc5c8\ub2e4.", "sentence_answer": "\uc120\uc218\ub4e4\uc774 \uacbd\uae30 \uc7ac\uac1c\ub97c \uae30\ub2e4\ub9ac\ub294 \ub3d9\uc548 \uba87\uba87 \uad00\uc911\ub4e4\uc740 \ub9ce\uc740 \uad00\uc911 \uc18d\uc5d0\uc11c \ub2f9\ud55c \ubd80\uc0c1 \ub54c\ubb38\uc5d0 \uc601\uad6d \uc801\uc2ed\uc790 \uc758 \uc751\uae09\ucc98\uce58\uac00 \ud544\uc694\ud558\uae30\ub3c4 \ud588\ub2e4.", "paragraph_id": "5782571-6-2", "paragraph_question": "question: 1923\ub144 \uc789\uae00\ub79c\ub4dc FA\ucef5 \uacb0\uc2b9\uc804\uc5d0\uc11c \ubd80\uc0c1\ub2f9\ud55c \uad00\uc911\ub4e4\uc5d0\uac8c \uc751\uae09\ucc98\uce58\ub97c \ud55c \ub2e8\uccb4\ub294?, context: \uc591 \ud300 \ubaa8\ub450 \uadf8 \ub2f9\uc2dc\uc5d0 \uac00\uc7a5 \uc804\ud615\uc801\uc778 \ud3ec\uba54\uc774\uc158\uc744 \ub4e4\uace0 \ub098\uc654\ub294\ub370, \uc804\uc220\uc758 \uae30\ubcf8 \ud615\ud0dc\ub294 2\uba85\uc758 \ud480\ubc31\uc774 \ub4a4\uc5d0 \uc704\uce58\ud558\uace0 \ud55c \uba85\uc758 \uc13c\ud130 \ud558\ud504\uc640 \ub450 \uba85\uc758 \uc719 \ud558\ud504\ub85c \uad6c\uc131\ub41c 3\uba85\uc758 \ud558\ud504\ubc31\uc774 \uadf8 \uc55e\uc5d0 \uc11c\uc11c \uc218\ube44\ub97c \ub2f4\ub2f9\ud558\uace0 \ub9c8\uc9c0\ub9c9\uc73c\ub85c 2\uba85\uc758 \uc544\uc6c3\uc0ac\uc774\ub4dc \ud3ec\uc6cc\ub4dc, \ub450 \uba85\uc758 \uc778\uc0ac\uc774\ub4dc \ud3ec\uc6cc\ub4dc, \uadf8\ub9ac\uace0 \ud55c \uba85\uc758 \uc13c\ud130 \ud3ec\uc6cc\ub4dc\ub85c \uad6c\uc131\ub41c 5\uba85\uc758 \ud3ec\uc6cc\ub4dc\uac00 \uacf5\uaca9\uc744 \ub9e1\ub294 \uac83\uc774\ub2e4. \uc6e8\uc2a4\ud2b8 \ud584\uc740 \ub450 \ube60\ub978 \uc544\uc6c3\uc0ac\uc774\ub4dc \ud3ec\uc6cc\ub4dc\uc778 \ub515 \ub9ac\ucc28\uc988\uc640 \uc9c0\ubbf8 \ub7ec\ud3a0\uc744 \uc911\uc2ec\uc73c\ub85c \ud558\ub294 \uc804\ub7b5\uc744 \uad6c\uc131\ud588\uc73c\ub098 \ubcfc\ud134\uc740 \uc2dc\uc791\ud558\uc790\ub9c8\uc790 \uadf8 \ub450 \uc120\uc218\ub97c \uc798 \uc6c0\uc9c1\uc774\uc9c0 \ubabb\ud558\uac8c \ubb36\uc5b4\ub450\uc5c8\uace0, \ub610\ud55c \uadf8 \ub450 \uc120\uc218\uac00 \uacf5\uc744 \uc7a1\uc790\ub9c8\uc790 \ub2ec\ub824\ub4e4\uc5b4 \ubc29\ud574\ud558\ub294 \ub9de\ucda4\uc804\ub7b5\uc744 \uc37c\ub2e4. \uacbd\uae30 \uc2dc\uc791 2\ubd84 \ud6c4 \uc6e8\uc2a4\ud2b8 \ud584\uc758 \ud558\ud504\ubc31\uc778 \uc7ad \ud2b8\ub808\uc0ac\ub358\uc774 \uc2a4\ub85c\uc778\uc744 \ub358\uc9c4 \ud6c4\uc5d0 \uad00\uc911\ub4e4\uc5d0 \ud729\uc4f8\ub824 \uadf8\ub77c\uc6b4\ub4dc\ub85c \ubcf5\uadc0\ud558\uc9c0 \ubabb\ud558\ub294 \uc0c1\ud669\uc774 \ubc8c\uc5b4\uc84c\ub2e4. \uc774 \uae30\ud68c\ub97c \ub193\uce58\uc9c0 \uc54a\uc740 \ubcfc\ud134\uc758 \ub370\uc774\ube44\ub4dc \uc7ad\uc774 \uc120\uc218 \uc22b\uc790\uc758 \uc6b0\uc704\ub97c \uc810\ud55c \uc0c1\ud669\uc5d0\uc11c \uc288\ud305\uc744 \ud558\uc600\ub2e4. \uc6e8\uc2a4\ud2b8 \ud584\uc758 \uace8\ud0a4\ud37c \ud14c\ub4dc \ud5c8\ud504\ud2bc\uc774 \uc774 \uc29b\uc744 \ub9c9\uc9c0 \ubabb\ud558\uace0 \uc2e4\uc810\uc744 \ud5c8\uc6a9\ud574 \ubcfc\ud134\uc774 \uc55e\uc11c\ub098\uac00\uae30 \uc2dc\uc791\ud588\ub2e4. \uadf8\ub54c \uadf8 \uacf5\uc774 \ub2e4\ub978 \uad00\uc911\ub4e4\uc5d0 \ubc00\ub824\ub0b4\ub824\uc640 \uace8\ub124\ud2b8\uc5d0 \ubc14\uc9dd \ubd99\uc5b4\uc788\ub358 \ud55c \uad00\uc911\uc744 \uac15\ud0c0\ud558\uba74\uc11c \uadf8 \uad00\uc911\uc774 \ubb34\uc758\uc2dd \uc0c1\ud0dc\uac00 \ub418\uc5b4\ubc84\ub838\ub2e4. \uc2e4\uc810 3\ubd84 \ud6c4 \uc6e8\uc2a4\ud2b8 \ud584\uc758 \ube45 \uc653\uc2a8\uc774 \ubcfc\ud134\uc758 \uace8\ubb38 \ubc14\ub85c \uc55e\uc5d0\uc11c \uacf5\uc744 \uc7a1\uc544 \ucc9c\uae08\uac19\uc740 \uae30\ud68c\ub97c \uc7a1\uc558\uc73c\ub098 \uc29b\uc774 \ud06c\ub85c\uc2a4\ubc14\ub97c \ub118\uc5b4\uac00\ubc84\ub838\ub2e4. \uc804\ubc18 11\ubd84 \uacbd\uc5d0\ub294 \ub610\ub2e4\uc2dc \uad00\uc911\ub4e4\uc774 \uc55e\uc73c\ub85c \ubab0\ub9ac\uba74\uc11c \ub2e4\uc218\uc758 \ud32c\ub4e4\uc774 \uadf8\ub77c\uc6b4\ub4dc \uc548\uc73c\ub85c \uc9c4\uc785\ud558\ub294 \ubc14\ub78c\uc5d0 \ubd80\uc2ec\ub4e4\uc744 \uc704\ud574 \ub9d0\uc744 \ud0c4 \uacbd\ucc30\ub4e4\uc774 \uc0c1\ud669\uc744 \uc218\uc2b5\ud560 \ub54c\uae4c\uc9c0 \uacbd\uae30\uac00 \uc9c0\uc5f0\ub418\uc5c8\ub2e4. \uc120\uc218\ub4e4\uc774 \uacbd\uae30 \uc7ac\uac1c\ub97c \uae30\ub2e4\ub9ac\ub294 \ub3d9\uc548 \uba87\uba87 \uad00\uc911\ub4e4\uc740 \ub9ce\uc740 \uad00\uc911 \uc18d\uc5d0\uc11c \ub2f9\ud55c \ubd80\uc0c1 \ub54c\ubb38\uc5d0 \uc601\uad6d \uc801\uc2ed\uc790\uc758 \uc751\uae09\ucc98\uce58\uac00 \ud544\uc694\ud558\uae30\ub3c4 \ud588\ub2e4. \uacbd\ucc30\ub4e4\uc740 \uadf8\ub77c\uc6b4\ub4dc \uc8fc\ubcc0\uc744 \ub3cc\uba70 \ub354\uc774\uc0c1 \uad00\uc911\ub4e4\uc774 \uc120\uc744 \ub118\uc5b4 \uc9c4\uc785\ud558\uc9c0 \ubabb\ud558\ub3c4\ub85d \ud588\uace0 \uadf8 \uc0ac\uc774 \uacbd\uae30\uac00 \uc7ac\uac1c\ub418\uc5c8\ub2e4."} {"question": "\ud30c\uc774\uc5b4\ud3ed\uc2a4 2\ub294 \uc774 \uae30\ub2a5\uc744 \ub098\uc911\uc5d0 \ud504\ub85c\uadf8\ub7a8 \uc790\uccb4\uc5d0 \ud1b5\ud569\ub418\ub3c4\ub85d \ud588\ub2e4. \uc774 \uae30\ub2a5\uc740 \ubb34\uc5c7\uc77c\uae4c?", "paragraph": "2006\ub144 10\uc6d4 24\uc77c\uc5d0 \ud30c\uc774\uc5b4\ud3ed\uc2a4 2\ub97c \ubc1c\ud45c\ud558\uc600\ub2e4. \uc774 \ubc84\uc804\uc740 \uac1c\uc120\ub41c \ud0ed \ube0c\ub77c\uc6b0\uc9d5 \ud658\uacbd\uacfc \ubd80\uac00 \uae30\ub2a5 \uad00\ub9ac\uc790, \uac1c\uc120\ub41c \uadf8\ub798\ud53d \uc0ac\uc6a9\uc790 \uc778\ud130\ud398\uc774\uc2a4, \ucc3e\uae30 \ubc0f \uac80\uc0c9\uacfc \uc18c\ud504\ud2b8\uc6e8\uc5b4 \uc5c5\ub370\uc774\ud2b8 \uc5d4\uc9c4, \uc138\uc158 \ubcf5\uad6c \uae30\ub2a5, \uc778\ub77c\uc778 \ub9de\ucda4\ubc95 \uac80\uc0ac, \ucd5c\ucd08\uc5d0 \uad6c\uae00\uc774 \uc81c\uacf5\ud558\ub294 \ud655\uc7a5 \uae30\ub2a5\uc5d0 \uc758\ud574 \uae30\ub2a5\uc774 \uc81c\uacf5\ub418\uc5c8\uace0 \ub098\uc911\uc5d0 \ud504\ub85c\uadf8\ub7a8 \uc790\uccb4\uc5d0 \uae30\ub2a5\uc774 \ud1b5\ud569\ub41c \ud53c\uc2f1 \ubc29\uc9c0 \uae30\ub2a5 \ub4f1\uc774 \ud3ec\ud568\ub418\uc5b4 \uc788\ub2e4. 2007\ub144 \uaca8\uc6b8\uc5d0\ub294 \ud30c\uc774\uc5b4\ud3ed\uc2a4 \ub77c\uc774\ube0c \ucc57\uc774 \ub860\uce6d\ub418\uc5c8\ub2e4. \uc774\ub294 \uc791\ub3d9 \ubcf4\uc99d\uc2dc\uac04\uacfc \uadfc\ubb34\uc2dc\uac04 \uc774\ud6c4\uc5d0 \ub3c4\uc6c0\uc744 \uc904 \uc218 \uc788\ub3c4\ub85d \uc790\uc774\ube0c \uc18c\ud504\ud2b8\uc6e8\uc5b4\uac00 \uc81c\uacf5\ud558\ub294 \uc2dc\uc2a4\ud15c\uc744 \uc774\uc6a9\ud574 \uc0ac\uc6a9\uc790\uac00 \uc790\uc6d0\ubd09\uc0ac\uc790\uc5d0\uac8c \uc9c8\ubb38\ud560 \uc218 \uc788\ub3c4\ub85d \ud5c8\uc6a9\ud55c \uac83\uc774\ub2e4. \ud30c\uc774\uc5b4\ud3ed\uc2a4 2.0.0.20\uc740 \uc708\ub3c4\uc6b0 NT 4.0, \uc708\ub3c4\uc6b0 98\uacfc \uc708\ub3c4\uc6b0 ME\uc5d0\uc11c \uc218\uc815\uc5c6\uc774 \uc124\uce58\ud558\uace0 \uc0ac\uc6a9\ud560 \uc218 \uc788\ub294 \ub9c8\uc9c0\ub9c9 \ubc84\uc804\uc774\ub2e4.", "answer": "\ud53c\uc2f1 \ubc29\uc9c0 \uae30\ub2a5", "sentence": "\uc774 \ubc84\uc804\uc740 \uac1c\uc120\ub41c \ud0ed \ube0c\ub77c\uc6b0\uc9d5 \ud658\uacbd\uacfc \ubd80\uac00 \uae30\ub2a5 \uad00\ub9ac\uc790, \uac1c\uc120\ub41c \uadf8\ub798\ud53d \uc0ac\uc6a9\uc790 \uc778\ud130\ud398\uc774\uc2a4, \ucc3e\uae30 \ubc0f \uac80\uc0c9\uacfc \uc18c\ud504\ud2b8\uc6e8\uc5b4 \uc5c5\ub370\uc774\ud2b8 \uc5d4\uc9c4, \uc138\uc158 \ubcf5\uad6c \uae30\ub2a5, \uc778\ub77c\uc778 \ub9de\ucda4\ubc95 \uac80\uc0ac, \ucd5c\ucd08\uc5d0 \uad6c\uae00\uc774 \uc81c\uacf5\ud558\ub294 \ud655\uc7a5 \uae30\ub2a5\uc5d0 \uc758\ud574 \uae30\ub2a5\uc774 \uc81c\uacf5\ub418\uc5c8\uace0 \ub098\uc911\uc5d0 \ud504\ub85c\uadf8\ub7a8 \uc790\uccb4\uc5d0 \uae30\ub2a5\uc774 \ud1b5\ud569\ub41c \ud53c\uc2f1 \ubc29\uc9c0 \uae30\ub2a5 \ub4f1\uc774 \ud3ec\ud568\ub418\uc5b4 \uc788\ub2e4.", "paragraph_sentence": "2006\ub144 10\uc6d4 24\uc77c\uc5d0 \ud30c\uc774\uc5b4\ud3ed\uc2a4 2\ub97c \ubc1c\ud45c\ud558\uc600\ub2e4. \uc774 \ubc84\uc804\uc740 \uac1c\uc120\ub41c \ud0ed \ube0c\ub77c\uc6b0\uc9d5 \ud658\uacbd\uacfc \ubd80\uac00 \uae30\ub2a5 \uad00\ub9ac\uc790, \uac1c\uc120\ub41c \uadf8\ub798\ud53d \uc0ac\uc6a9\uc790 \uc778\ud130\ud398\uc774\uc2a4, \ucc3e\uae30 \ubc0f \uac80\uc0c9\uacfc \uc18c\ud504\ud2b8\uc6e8\uc5b4 \uc5c5\ub370\uc774\ud2b8 \uc5d4\uc9c4, \uc138\uc158 \ubcf5\uad6c \uae30\ub2a5, \uc778\ub77c\uc778 \ub9de\ucda4\ubc95 \uac80\uc0ac, \ucd5c\ucd08\uc5d0 \uad6c\uae00\uc774 \uc81c\uacf5\ud558\ub294 \ud655\uc7a5 \uae30\ub2a5\uc5d0 \uc758\ud574 \uae30\ub2a5\uc774 \uc81c\uacf5\ub418\uc5c8\uace0 \ub098\uc911\uc5d0 \ud504\ub85c\uadf8\ub7a8 \uc790\uccb4\uc5d0 \uae30\ub2a5\uc774 \ud1b5\ud569\ub41c \ud53c\uc2f1 \ubc29\uc9c0 \uae30\ub2a5 \ub4f1\uc774 \ud3ec\ud568\ub418\uc5b4 \uc788\ub2e4. 2007\ub144 \uaca8\uc6b8\uc5d0\ub294 \ud30c\uc774\uc5b4\ud3ed\uc2a4 \ub77c\uc774\ube0c \ucc57\uc774 \ub860\uce6d\ub418\uc5c8\ub2e4. \uc774\ub294 \uc791\ub3d9 \ubcf4\uc99d\uc2dc\uac04\uacfc \uadfc\ubb34\uc2dc\uac04 \uc774\ud6c4\uc5d0 \ub3c4\uc6c0\uc744 \uc904 \uc218 \uc788\ub3c4\ub85d \uc790\uc774\ube0c \uc18c\ud504\ud2b8\uc6e8\uc5b4\uac00 \uc81c\uacf5\ud558\ub294 \uc2dc\uc2a4\ud15c\uc744 \uc774\uc6a9\ud574 \uc0ac\uc6a9\uc790\uac00 \uc790\uc6d0\ubd09\uc0ac\uc790\uc5d0\uac8c \uc9c8\ubb38\ud560 \uc218 \uc788\ub3c4\ub85d \ud5c8\uc6a9\ud55c \uac83\uc774\ub2e4. \ud30c\uc774\uc5b4\ud3ed\uc2a4 2.0.0.20\uc740 \uc708\ub3c4\uc6b0 NT 4.0, \uc708\ub3c4\uc6b0 98\uacfc \uc708\ub3c4\uc6b0 ME\uc5d0\uc11c \uc218\uc815\uc5c6\uc774 \uc124\uce58\ud558\uace0 \uc0ac\uc6a9\ud560 \uc218 \uc788\ub294 \ub9c8\uc9c0\ub9c9 \ubc84\uc804\uc774\ub2e4.", "paragraph_answer": "2006\ub144 10\uc6d4 24\uc77c\uc5d0 \ud30c\uc774\uc5b4\ud3ed\uc2a4 2\ub97c \ubc1c\ud45c\ud558\uc600\ub2e4. \uc774 \ubc84\uc804\uc740 \uac1c\uc120\ub41c \ud0ed \ube0c\ub77c\uc6b0\uc9d5 \ud658\uacbd\uacfc \ubd80\uac00 \uae30\ub2a5 \uad00\ub9ac\uc790, \uac1c\uc120\ub41c \uadf8\ub798\ud53d \uc0ac\uc6a9\uc790 \uc778\ud130\ud398\uc774\uc2a4, \ucc3e\uae30 \ubc0f \uac80\uc0c9\uacfc \uc18c\ud504\ud2b8\uc6e8\uc5b4 \uc5c5\ub370\uc774\ud2b8 \uc5d4\uc9c4, \uc138\uc158 \ubcf5\uad6c \uae30\ub2a5, \uc778\ub77c\uc778 \ub9de\ucda4\ubc95 \uac80\uc0ac, \ucd5c\ucd08\uc5d0 \uad6c\uae00\uc774 \uc81c\uacf5\ud558\ub294 \ud655\uc7a5 \uae30\ub2a5\uc5d0 \uc758\ud574 \uae30\ub2a5\uc774 \uc81c\uacf5\ub418\uc5c8\uace0 \ub098\uc911\uc5d0 \ud504\ub85c\uadf8\ub7a8 \uc790\uccb4\uc5d0 \uae30\ub2a5\uc774 \ud1b5\ud569\ub41c \ud53c\uc2f1 \ubc29\uc9c0 \uae30\ub2a5 \ub4f1\uc774 \ud3ec\ud568\ub418\uc5b4 \uc788\ub2e4. 2007\ub144 \uaca8\uc6b8\uc5d0\ub294 \ud30c\uc774\uc5b4\ud3ed\uc2a4 \ub77c\uc774\ube0c \ucc57\uc774 \ub860\uce6d\ub418\uc5c8\ub2e4. \uc774\ub294 \uc791\ub3d9 \ubcf4\uc99d\uc2dc\uac04\uacfc \uadfc\ubb34\uc2dc\uac04 \uc774\ud6c4\uc5d0 \ub3c4\uc6c0\uc744 \uc904 \uc218 \uc788\ub3c4\ub85d \uc790\uc774\ube0c \uc18c\ud504\ud2b8\uc6e8\uc5b4\uac00 \uc81c\uacf5\ud558\ub294 \uc2dc\uc2a4\ud15c\uc744 \uc774\uc6a9\ud574 \uc0ac\uc6a9\uc790\uac00 \uc790\uc6d0\ubd09\uc0ac\uc790\uc5d0\uac8c \uc9c8\ubb38\ud560 \uc218 \uc788\ub3c4\ub85d \ud5c8\uc6a9\ud55c \uac83\uc774\ub2e4. \ud30c\uc774\uc5b4\ud3ed\uc2a4 2.0.0.20\uc740 \uc708\ub3c4\uc6b0 NT 4.0, \uc708\ub3c4\uc6b0 98\uacfc \uc708\ub3c4\uc6b0 ME\uc5d0\uc11c \uc218\uc815\uc5c6\uc774 \uc124\uce58\ud558\uace0 \uc0ac\uc6a9\ud560 \uc218 \uc788\ub294 \ub9c8\uc9c0\ub9c9 \ubc84\uc804\uc774\ub2e4.", "sentence_answer": "\uc774 \ubc84\uc804\uc740 \uac1c\uc120\ub41c \ud0ed \ube0c\ub77c\uc6b0\uc9d5 \ud658\uacbd\uacfc \ubd80\uac00 \uae30\ub2a5 \uad00\ub9ac\uc790, \uac1c\uc120\ub41c \uadf8\ub798\ud53d \uc0ac\uc6a9\uc790 \uc778\ud130\ud398\uc774\uc2a4, \ucc3e\uae30 \ubc0f \uac80\uc0c9\uacfc \uc18c\ud504\ud2b8\uc6e8\uc5b4 \uc5c5\ub370\uc774\ud2b8 \uc5d4\uc9c4, \uc138\uc158 \ubcf5\uad6c \uae30\ub2a5, \uc778\ub77c\uc778 \ub9de\ucda4\ubc95 \uac80\uc0ac, \ucd5c\ucd08\uc5d0 \uad6c\uae00\uc774 \uc81c\uacf5\ud558\ub294 \ud655\uc7a5 \uae30\ub2a5\uc5d0 \uc758\ud574 \uae30\ub2a5\uc774 \uc81c\uacf5\ub418\uc5c8\uace0 \ub098\uc911\uc5d0 \ud504\ub85c\uadf8\ub7a8 \uc790\uccb4\uc5d0 \uae30\ub2a5\uc774 \ud1b5\ud569\ub41c \ud53c\uc2f1 \ubc29\uc9c0 \uae30\ub2a5 \ub4f1\uc774 \ud3ec\ud568\ub418\uc5b4 \uc788\ub2e4.", "paragraph_id": "6410157-0-1", "paragraph_question": "question: \ud30c\uc774\uc5b4\ud3ed\uc2a4 2\ub294 \uc774 \uae30\ub2a5\uc744 \ub098\uc911\uc5d0 \ud504\ub85c\uadf8\ub7a8 \uc790\uccb4\uc5d0 \ud1b5\ud569\ub418\ub3c4\ub85d \ud588\ub2e4. \uc774 \uae30\ub2a5\uc740 \ubb34\uc5c7\uc77c\uae4c?, context: 2006\ub144 10\uc6d4 24\uc77c\uc5d0 \ud30c\uc774\uc5b4\ud3ed\uc2a4 2\ub97c \ubc1c\ud45c\ud558\uc600\ub2e4. \uc774 \ubc84\uc804\uc740 \uac1c\uc120\ub41c \ud0ed \ube0c\ub77c\uc6b0\uc9d5 \ud658\uacbd\uacfc \ubd80\uac00 \uae30\ub2a5 \uad00\ub9ac\uc790, \uac1c\uc120\ub41c \uadf8\ub798\ud53d \uc0ac\uc6a9\uc790 \uc778\ud130\ud398\uc774\uc2a4, \ucc3e\uae30 \ubc0f \uac80\uc0c9\uacfc \uc18c\ud504\ud2b8\uc6e8\uc5b4 \uc5c5\ub370\uc774\ud2b8 \uc5d4\uc9c4, \uc138\uc158 \ubcf5\uad6c \uae30\ub2a5, \uc778\ub77c\uc778 \ub9de\ucda4\ubc95 \uac80\uc0ac, \ucd5c\ucd08\uc5d0 \uad6c\uae00\uc774 \uc81c\uacf5\ud558\ub294 \ud655\uc7a5 \uae30\ub2a5\uc5d0 \uc758\ud574 \uae30\ub2a5\uc774 \uc81c\uacf5\ub418\uc5c8\uace0 \ub098\uc911\uc5d0 \ud504\ub85c\uadf8\ub7a8 \uc790\uccb4\uc5d0 \uae30\ub2a5\uc774 \ud1b5\ud569\ub41c \ud53c\uc2f1 \ubc29\uc9c0 \uae30\ub2a5 \ub4f1\uc774 \ud3ec\ud568\ub418\uc5b4 \uc788\ub2e4. 2007\ub144 \uaca8\uc6b8\uc5d0\ub294 \ud30c\uc774\uc5b4\ud3ed\uc2a4 \ub77c\uc774\ube0c \ucc57\uc774 \ub860\uce6d\ub418\uc5c8\ub2e4. \uc774\ub294 \uc791\ub3d9 \ubcf4\uc99d\uc2dc\uac04\uacfc \uadfc\ubb34\uc2dc\uac04 \uc774\ud6c4\uc5d0 \ub3c4\uc6c0\uc744 \uc904 \uc218 \uc788\ub3c4\ub85d \uc790\uc774\ube0c \uc18c\ud504\ud2b8\uc6e8\uc5b4\uac00 \uc81c\uacf5\ud558\ub294 \uc2dc\uc2a4\ud15c\uc744 \uc774\uc6a9\ud574 \uc0ac\uc6a9\uc790\uac00 \uc790\uc6d0\ubd09\uc0ac\uc790\uc5d0\uac8c \uc9c8\ubb38\ud560 \uc218 \uc788\ub3c4\ub85d \ud5c8\uc6a9\ud55c \uac83\uc774\ub2e4. \ud30c\uc774\uc5b4\ud3ed\uc2a4 2.0.0.20\uc740 \uc708\ub3c4\uc6b0 NT 4.0, \uc708\ub3c4\uc6b0 98\uacfc \uc708\ub3c4\uc6b0 ME\uc5d0\uc11c \uc218\uc815\uc5c6\uc774 \uc124\uce58\ud558\uace0 \uc0ac\uc6a9\ud560 \uc218 \uc788\ub294 \ub9c8\uc9c0\ub9c9 \ubc84\uc804\uc774\ub2e4."} {"question": "\ubaa9\ud0c1\uc218\uad6c\ub9ac\uc758 \ucc9c\uc801\uc740?", "paragraph": "\ubaa9\ud0c1\uc218\uad6c\ub9ac\uc758 \ucc9c\uc801\uc73c\ub85c\ub294 \ubc40\uc0c1\uc5b4(Galeocerdo cuvier)\uac00 \uc788\ub2e4. \ubaa9\ud0c1\uc218\uad6c\ub9ac\ub294 \uba38\ub9ac\uc640 \ub4f1\uc5d0 \ub3cb\uce5c \uac00\uc2dc\ub85c \uc2a4\uc2a4\ub85c\ub97c \ubc29\uc5b4\ud558\uace0, \uc704\ud611\uc744 \ub290\ub07c\uba74 \ub4e4\uc774\ubc1b\uae30\ub3c4 \ud55c\ub2e4. \ubaa9\ud0c1\uc218\uad6c\ub9ac\uc5d0 \uae30\uc0dd\ud558\ub294 \uae30\uc0dd\ucda9\uc73c\ub85c\ub294 \ucd0c\ucda9\ub958(Carpobothrium rhinei, Dollfusiella michiae, Nybelinia southwelli, Stoibocephalum arafurense, Tylocephalum carnpanulatum), \uac70\uba38\ub9ac\ub958(Pontobdella macrothela), \ud761\ucda9\ub958(Melogonimus rhodanometra), \ub2e8\uc0dd\ub958(Branchotenthes robinoverstreeti, Monocotyle ancylostomae), \uc694\uac01\ub958(Nesippus vespa, Pandarus cranchii, P. smithii) \ub4f1\uc774 \ubc1d\ud600\uc838 \uc788\ub2e4. \uccad\uc904\uccad\uc18c\ub180\ub798\uae30(Labroides dimidiatus)\uac00 \ubaa9\ud0c1\uc218\uad6c\ub9ac\uc758 \uae30\uc0dd\ucda9\uc744 \uc7a1\uc544\uba39\ub294 \uac83\uc774 \ubc1c\uacac\ub41c \ubc14 \uc788\ub2e4.", "answer": "\ubc40\uc0c1\uc5b4", "sentence": "\ubaa9\ud0c1\uc218\uad6c\ub9ac\uc758 \ucc9c\uc801\uc73c\ub85c\ub294 \ubc40\uc0c1\uc5b4 (Galeocerdo cuvier)", "paragraph_sentence": " \ubaa9\ud0c1\uc218\uad6c\ub9ac\uc758 \ucc9c\uc801\uc73c\ub85c\ub294 \ubc40\uc0c1\uc5b4 (Galeocerdo cuvier) \uac00 \uc788\ub2e4. \ubaa9\ud0c1\uc218\uad6c\ub9ac\ub294 \uba38\ub9ac\uc640 \ub4f1\uc5d0 \ub3cb\uce5c \uac00\uc2dc\ub85c \uc2a4\uc2a4\ub85c\ub97c \ubc29\uc5b4\ud558\uace0, \uc704\ud611\uc744 \ub290\ub07c\uba74 \ub4e4\uc774\ubc1b\uae30\ub3c4 \ud55c\ub2e4. \ubaa9\ud0c1\uc218\uad6c\ub9ac\uc5d0 \uae30\uc0dd\ud558\ub294 \uae30\uc0dd\ucda9\uc73c\ub85c\ub294 \ucd0c\ucda9\ub958(Carpobothrium rhinei, Dollfusiella michiae, Nybelinia southwelli, Stoibocephalum arafurense, Tylocephalum carnpanulatum), \uac70\uba38\ub9ac\ub958(Pontobdella macrothela), \ud761\ucda9\ub958(Melogonimus rhodanometra), \ub2e8\uc0dd\ub958(Branchotenthes robinoverstreeti, Monocotyle ancylostomae), \uc694\uac01\ub958(Nesippus vespa, Pandarus cranchii, P. smithii) \ub4f1\uc774 \ubc1d\ud600\uc838 \uc788\ub2e4. \uccad\uc904\uccad\uc18c\ub180\ub798\uae30(Labroides dimidiatus)\uac00 \ubaa9\ud0c1\uc218\uad6c\ub9ac\uc758 \uae30\uc0dd\ucda9\uc744 \uc7a1\uc544\uba39\ub294 \uac83\uc774 \ubc1c\uacac\ub41c \ubc14 \uc788\ub2e4.", "paragraph_answer": "\ubaa9\ud0c1\uc218\uad6c\ub9ac\uc758 \ucc9c\uc801\uc73c\ub85c\ub294 \ubc40\uc0c1\uc5b4 (Galeocerdo cuvier)\uac00 \uc788\ub2e4. \ubaa9\ud0c1\uc218\uad6c\ub9ac\ub294 \uba38\ub9ac\uc640 \ub4f1\uc5d0 \ub3cb\uce5c \uac00\uc2dc\ub85c \uc2a4\uc2a4\ub85c\ub97c \ubc29\uc5b4\ud558\uace0, \uc704\ud611\uc744 \ub290\ub07c\uba74 \ub4e4\uc774\ubc1b\uae30\ub3c4 \ud55c\ub2e4. \ubaa9\ud0c1\uc218\uad6c\ub9ac\uc5d0 \uae30\uc0dd\ud558\ub294 \uae30\uc0dd\ucda9\uc73c\ub85c\ub294 \ucd0c\ucda9\ub958(Carpobothrium rhinei, Dollfusiella michiae, Nybelinia southwelli, Stoibocephalum arafurense, Tylocephalum carnpanulatum), \uac70\uba38\ub9ac\ub958(Pontobdella macrothela), \ud761\ucda9\ub958(Melogonimus rhodanometra), \ub2e8\uc0dd\ub958(Branchotenthes robinoverstreeti, Monocotyle ancylostomae), \uc694\uac01\ub958(Nesippus vespa, Pandarus cranchii, P. smithii) \ub4f1\uc774 \ubc1d\ud600\uc838 \uc788\ub2e4. \uccad\uc904\uccad\uc18c\ub180\ub798\uae30(Labroides dimidiatus)\uac00 \ubaa9\ud0c1\uc218\uad6c\ub9ac\uc758 \uae30\uc0dd\ucda9\uc744 \uc7a1\uc544\uba39\ub294 \uac83\uc774 \ubc1c\uacac\ub41c \ubc14 \uc788\ub2e4.", "sentence_answer": "\ubaa9\ud0c1\uc218\uad6c\ub9ac\uc758 \ucc9c\uc801\uc73c\ub85c\ub294 \ubc40\uc0c1\uc5b4 (Galeocerdo cuvier)", "paragraph_id": "6546911-8-0", "paragraph_question": "question: \ubaa9\ud0c1\uc218\uad6c\ub9ac\uc758 \ucc9c\uc801\uc740?, context: \ubaa9\ud0c1\uc218\uad6c\ub9ac\uc758 \ucc9c\uc801\uc73c\ub85c\ub294 \ubc40\uc0c1\uc5b4(Galeocerdo cuvier)\uac00 \uc788\ub2e4. \ubaa9\ud0c1\uc218\uad6c\ub9ac\ub294 \uba38\ub9ac\uc640 \ub4f1\uc5d0 \ub3cb\uce5c \uac00\uc2dc\ub85c \uc2a4\uc2a4\ub85c\ub97c \ubc29\uc5b4\ud558\uace0, \uc704\ud611\uc744 \ub290\ub07c\uba74 \ub4e4\uc774\ubc1b\uae30\ub3c4 \ud55c\ub2e4. \ubaa9\ud0c1\uc218\uad6c\ub9ac\uc5d0 \uae30\uc0dd\ud558\ub294 \uae30\uc0dd\ucda9\uc73c\ub85c\ub294 \ucd0c\ucda9\ub958(Carpobothrium rhinei, Dollfusiella michiae, Nybelinia southwelli, Stoibocephalum arafurense, Tylocephalum carnpanulatum), \uac70\uba38\ub9ac\ub958(Pontobdella macrothela), \ud761\ucda9\ub958(Melogonimus rhodanometra), \ub2e8\uc0dd\ub958(Branchotenthes robinoverstreeti, Monocotyle ancylostomae), \uc694\uac01\ub958(Nesippus vespa, Pandarus cranchii, P. smithii) \ub4f1\uc774 \ubc1d\ud600\uc838 \uc788\ub2e4. \uccad\uc904\uccad\uc18c\ub180\ub798\uae30(Labroides dimidiatus)\uac00 \ubaa9\ud0c1\uc218\uad6c\ub9ac\uc758 \uae30\uc0dd\ucda9\uc744 \uc7a1\uc544\uba39\ub294 \uac83\uc774 \ubc1c\uacac\ub41c \ubc14 \uc788\ub2e4."} {"question": "\uc774\uc720\uc601\uc774 2014\ub144 \uad6d\uc81c \uc601\ud654\uc81c\uc5d0\uc11c \uc5ec\uc6b0\uc8fc\uc5f0\uc0c1\uc744 \ubc1b\uc740 \uc601\ud654\uc758 \uc81c\ubaa9\uc740 \ubb34\uc5c7\uc778\uac00?", "paragraph": "\uc774\uc720\uc601\uc740 1989\ub144 12\uc6d4 8\uc77c \uc11c\uc6b8\uc5d0\uc11c \ud0dc\uc5b4\ub0ac\ub2e4. \uadf8\ub140\ub294 \uace0\ub4f1\ud559\uad50 \uc878\uc5c5 \ud6c4 \ub300\ud559\uc5d0 \uc9c4\ud559\ud558\uc9c0 \uc54a\uace0 \ubbf8\uc6a9\uc2e4\uc5d0\uc11c \ubbf8\uc6a9\uc0ac \ubcf4\uc870 \uc77c\uc744 \ud558\ub2e4\uac00 2010\ub144 22\uc138\uc758 \ub098\uc774\uc5d0 \ud55c\uad6d\uc608\uc220\uc885\ud569\ud559\uad50 \uc5f0\uadf9\uc6d0 \uc5f0\uae30\uacfc\uc5d0 \uc9c4\ud559\ud558\uc600\uace0, \ud55c\uad6d\uc608\uc220\uc885\ud569\ud559\uad50 \uc7ac\ud559 \uc911 \uc0c1\uc5c5\uc601\ud654 \ub370\ubdd4\uc791\uc778 2014\ub144 \uc601\ud654 \u300a\ubd04\u300b\uc73c\ub85c \ubc00\ub77c\ub178 \uad6d\uc81c \uc601\ud654\uc81c\uc5d0\uc11c \uc5ec\uc6b0\uc8fc\uc5f0\uc0c1\uc744 \ubc1b\uc73c\uba70 \uc8fc\ubaa9\uc744 \ubc1b\uae30 \uc2dc\uc791\ud588\ub2e4. 1969\ub144 \uacbd\uc0c1\ubd81\ub3c4 \ud3ec\ud56d\uc744 \ubc30\uacbd\uc73c\ub85c \ud55c \uc774 \uc601\ud654\uc5d0\uc11c \uc815\uc219(\uae40\uc11c\ud615 \ubd84)\uc5d0\uac8c \ub3c8\uc744 \ubc1b\uace0 \uc815\uc219\uc758 \ub0a8\ud3b8\uc774\uc790 \ubd88\uce58\ubcd1\uc5d0 \uac78\ub824 \uc2e4\uc758\uc5d0 \ube60\uc9c4 \uc870\uac01\uac00 \uc900\uad6c(\ubc15\uc6a9\uc6b0 \ubd84)\uc758 \ub204\ub4dc\ubaa8\ub378\uc774 \ub418\ub294 \uac00\ub09c\ud55c \uc2dc\uace8 \uc5ec\uc790 \ubbfc\uacbd\uc744 \uc5f0\uae30\ud588\ub2e4. \uc774 \uc5ed\ud560\uc744 \uc5f0\uae30\ud55c \ubc30\uacbd\uc73c\ub85c \uc774\uc720\uc601\uc740 \u201c\uc2dc\ub098\ub9ac\uc624\ub97c \uc77d\uace0 \ud55c\ub208\uc5d0 \uc0ac\ub791\uc5d0 \ube60\uc84c\ub2e4\u201d\uace0 \ub9d0\ud558\uc600\uace0, \ub178\ucd9c \ub54c\ubb38\uc5d0 \uc9c0\uc778\uacfc \uac00\uc871\ub4e4\uc5d0\uac8c \uc601\ud654\ub97c \ubcf4\uc5ec\uc8fc\uae30\uc5d4 \uac71\uc815\uc774 \uc55e\uc130\uc9c0\ub9cc \uc5c4\ub9c8\uc758 \"\ub10c \ub108\uc758 \uc5f0\uae30, \ub108\uc758 \ubab8\uc774 \ucc3d\ud53c\ud558\ub2c8?\"\ub77c\ub294 \ud55c\ub9c8\ub514\uc5d0 \uc6a9\uae30\ub97c \ub0c8\ub2e4\uace0 \ud55c\ub2e4. \uadf8\ub140\ub294 \uc774 \uc791\ud488\uc73c\ub85c \uc774\ub4ec\ud574\uc778 2015\ub144 \uc62c\ud574\uc758 \uc601\ud654\uc0c1, \ubd80\uc77c\uc601\ud654\uc0c1\uacfc \ub300\uc885\uc0c1 \uc601\ud654\uc81c\uc5d0\uc11c \uc5ec\uc6b0\uc2e0\uc778\uc0c1\uc744 \uc218\uc0c1\ud558\uc600\uace0, 2015\ub144\uc5d0\ub294 \uc601\ud654 \u300a\uac04\uc2e0\u300b\uc73c\ub85c \uadf8\ud574 \uccad\ub8e1\uc601\ud654\uc0c1\uc5d0\uc11c \uc2e0\uc778\uc5ec\uc6b0\uc0c1\uc744 \uc218\uc0c1\ud588\ub2e4. 2018\ub144\uc774 \uae30\ub300\ub418\ub294 \ubc30\uc6b0 \uc911 \ud55c \uba85\uc774\ub2e4.", "answer": "\ubd04", "sentence": "\uadf8\ub140\ub294 \uace0\ub4f1\ud559\uad50 \uc878\uc5c5 \ud6c4 \ub300\ud559\uc5d0 \uc9c4\ud559\ud558\uc9c0 \uc54a\uace0 \ubbf8\uc6a9\uc2e4\uc5d0\uc11c \ubbf8\uc6a9\uc0ac \ubcf4\uc870 \uc77c\uc744 \ud558\ub2e4\uac00 2010\ub144 22\uc138\uc758 \ub098\uc774\uc5d0 \ud55c\uad6d\uc608\uc220\uc885\ud569\ud559\uad50 \uc5f0\uadf9\uc6d0 \uc5f0\uae30\uacfc\uc5d0 \uc9c4\ud559\ud558\uc600\uace0, \ud55c\uad6d\uc608\uc220\uc885\ud569\ud559\uad50 \uc7ac\ud559 \uc911 \uc0c1\uc5c5\uc601\ud654 \ub370\ubdd4\uc791\uc778 2014\ub144 \uc601\ud654 \u300a \ubd04 \u300b\uc73c\ub85c \ubc00\ub77c\ub178 \uad6d\uc81c \uc601\ud654\uc81c\uc5d0\uc11c \uc5ec\uc6b0\uc8fc\uc5f0\uc0c1\uc744 \ubc1b\uc73c\uba70 \uc8fc\ubaa9\uc744 \ubc1b\uae30 \uc2dc\uc791\ud588\ub2e4.", "paragraph_sentence": "\uc774\uc720\uc601\uc740 1989\ub144 12\uc6d4 8\uc77c \uc11c\uc6b8\uc5d0\uc11c \ud0dc\uc5b4\ub0ac\ub2e4. \uadf8\ub140\ub294 \uace0\ub4f1\ud559\uad50 \uc878\uc5c5 \ud6c4 \ub300\ud559\uc5d0 \uc9c4\ud559\ud558\uc9c0 \uc54a\uace0 \ubbf8\uc6a9\uc2e4\uc5d0\uc11c \ubbf8\uc6a9\uc0ac \ubcf4\uc870 \uc77c\uc744 \ud558\ub2e4\uac00 2010\ub144 22\uc138\uc758 \ub098\uc774\uc5d0 \ud55c\uad6d\uc608\uc220\uc885\ud569\ud559\uad50 \uc5f0\uadf9\uc6d0 \uc5f0\uae30\uacfc\uc5d0 \uc9c4\ud559\ud558\uc600\uace0, \ud55c\uad6d\uc608\uc220\uc885\ud569\ud559\uad50 \uc7ac\ud559 \uc911 \uc0c1\uc5c5\uc601\ud654 \ub370\ubdd4\uc791\uc778 2014\ub144 \uc601\ud654 \u300a \ubd04 \u300b\uc73c\ub85c \ubc00\ub77c\ub178 \uad6d\uc81c \uc601\ud654\uc81c\uc5d0\uc11c \uc5ec\uc6b0\uc8fc\uc5f0\uc0c1\uc744 \ubc1b\uc73c\uba70 \uc8fc\ubaa9\uc744 \ubc1b\uae30 \uc2dc\uc791\ud588\ub2e4. 1969\ub144 \uacbd\uc0c1\ubd81\ub3c4 \ud3ec\ud56d\uc744 \ubc30\uacbd\uc73c\ub85c \ud55c \uc774 \uc601\ud654\uc5d0\uc11c \uc815\uc219(\uae40\uc11c\ud615 \ubd84)\uc5d0\uac8c \ub3c8\uc744 \ubc1b\uace0 \uc815\uc219\uc758 \ub0a8\ud3b8\uc774\uc790 \ubd88\uce58\ubcd1\uc5d0 \uac78\ub824 \uc2e4\uc758\uc5d0 \ube60\uc9c4 \uc870\uac01\uac00 \uc900\uad6c(\ubc15\uc6a9\uc6b0 \ubd84)\uc758 \ub204\ub4dc\ubaa8\ub378\uc774 \ub418\ub294 \uac00\ub09c\ud55c \uc2dc\uace8 \uc5ec\uc790 \ubbfc\uacbd\uc744 \uc5f0\uae30\ud588\ub2e4. \uc774 \uc5ed\ud560\uc744 \uc5f0\uae30\ud55c \ubc30\uacbd\uc73c\ub85c \uc774\uc720\uc601\uc740 \u201c\uc2dc\ub098\ub9ac\uc624\ub97c \uc77d\uace0 \ud55c\ub208\uc5d0 \uc0ac\ub791\uc5d0 \ube60\uc84c\ub2e4\u201d\uace0 \ub9d0\ud558\uc600\uace0, \ub178\ucd9c \ub54c\ubb38\uc5d0 \uc9c0\uc778\uacfc \uac00\uc871\ub4e4\uc5d0\uac8c \uc601\ud654\ub97c \ubcf4\uc5ec\uc8fc\uae30\uc5d4 \uac71\uc815\uc774 \uc55e\uc130\uc9c0\ub9cc \uc5c4\ub9c8\uc758 \"\ub10c \ub108\uc758 \uc5f0\uae30, \ub108\uc758 \ubab8\uc774 \ucc3d\ud53c\ud558\ub2c8?\"\ub77c\ub294 \ud55c\ub9c8\ub514\uc5d0 \uc6a9\uae30\ub97c \ub0c8\ub2e4\uace0 \ud55c\ub2e4. \uadf8\ub140\ub294 \uc774 \uc791\ud488\uc73c\ub85c \uc774\ub4ec\ud574\uc778 2015\ub144 \uc62c\ud574\uc758 \uc601\ud654\uc0c1, \ubd80\uc77c\uc601\ud654\uc0c1\uacfc \ub300\uc885\uc0c1 \uc601\ud654\uc81c\uc5d0\uc11c \uc5ec\uc6b0\uc2e0\uc778\uc0c1\uc744 \uc218\uc0c1\ud558\uc600\uace0, 2015\ub144\uc5d0\ub294 \uc601\ud654 \u300a\uac04\uc2e0\u300b\uc73c\ub85c \uadf8\ud574 \uccad\ub8e1\uc601\ud654\uc0c1\uc5d0\uc11c \uc2e0\uc778\uc5ec\uc6b0\uc0c1\uc744 \uc218\uc0c1\ud588\ub2e4. 2018\ub144\uc774 \uae30\ub300\ub418\ub294 \ubc30\uc6b0 \uc911 \ud55c \uba85\uc774\ub2e4.", "paragraph_answer": "\uc774\uc720\uc601\uc740 1989\ub144 12\uc6d4 8\uc77c \uc11c\uc6b8\uc5d0\uc11c \ud0dc\uc5b4\ub0ac\ub2e4. \uadf8\ub140\ub294 \uace0\ub4f1\ud559\uad50 \uc878\uc5c5 \ud6c4 \ub300\ud559\uc5d0 \uc9c4\ud559\ud558\uc9c0 \uc54a\uace0 \ubbf8\uc6a9\uc2e4\uc5d0\uc11c \ubbf8\uc6a9\uc0ac \ubcf4\uc870 \uc77c\uc744 \ud558\ub2e4\uac00 2010\ub144 22\uc138\uc758 \ub098\uc774\uc5d0 \ud55c\uad6d\uc608\uc220\uc885\ud569\ud559\uad50 \uc5f0\uadf9\uc6d0 \uc5f0\uae30\uacfc\uc5d0 \uc9c4\ud559\ud558\uc600\uace0, \ud55c\uad6d\uc608\uc220\uc885\ud569\ud559\uad50 \uc7ac\ud559 \uc911 \uc0c1\uc5c5\uc601\ud654 \ub370\ubdd4\uc791\uc778 2014\ub144 \uc601\ud654 \u300a \ubd04 \u300b\uc73c\ub85c \ubc00\ub77c\ub178 \uad6d\uc81c \uc601\ud654\uc81c\uc5d0\uc11c \uc5ec\uc6b0\uc8fc\uc5f0\uc0c1\uc744 \ubc1b\uc73c\uba70 \uc8fc\ubaa9\uc744 \ubc1b\uae30 \uc2dc\uc791\ud588\ub2e4. 1969\ub144 \uacbd\uc0c1\ubd81\ub3c4 \ud3ec\ud56d\uc744 \ubc30\uacbd\uc73c\ub85c \ud55c \uc774 \uc601\ud654\uc5d0\uc11c \uc815\uc219(\uae40\uc11c\ud615 \ubd84)\uc5d0\uac8c \ub3c8\uc744 \ubc1b\uace0 \uc815\uc219\uc758 \ub0a8\ud3b8\uc774\uc790 \ubd88\uce58\ubcd1\uc5d0 \uac78\ub824 \uc2e4\uc758\uc5d0 \ube60\uc9c4 \uc870\uac01\uac00 \uc900\uad6c(\ubc15\uc6a9\uc6b0 \ubd84)\uc758 \ub204\ub4dc\ubaa8\ub378\uc774 \ub418\ub294 \uac00\ub09c\ud55c \uc2dc\uace8 \uc5ec\uc790 \ubbfc\uacbd\uc744 \uc5f0\uae30\ud588\ub2e4. \uc774 \uc5ed\ud560\uc744 \uc5f0\uae30\ud55c \ubc30\uacbd\uc73c\ub85c \uc774\uc720\uc601\uc740 \u201c\uc2dc\ub098\ub9ac\uc624\ub97c \uc77d\uace0 \ud55c\ub208\uc5d0 \uc0ac\ub791\uc5d0 \ube60\uc84c\ub2e4\u201d\uace0 \ub9d0\ud558\uc600\uace0, \ub178\ucd9c \ub54c\ubb38\uc5d0 \uc9c0\uc778\uacfc \uac00\uc871\ub4e4\uc5d0\uac8c \uc601\ud654\ub97c \ubcf4\uc5ec\uc8fc\uae30\uc5d4 \uac71\uc815\uc774 \uc55e\uc130\uc9c0\ub9cc \uc5c4\ub9c8\uc758 \"\ub10c \ub108\uc758 \uc5f0\uae30, \ub108\uc758 \ubab8\uc774 \ucc3d\ud53c\ud558\ub2c8?\"\ub77c\ub294 \ud55c\ub9c8\ub514\uc5d0 \uc6a9\uae30\ub97c \ub0c8\ub2e4\uace0 \ud55c\ub2e4. \uadf8\ub140\ub294 \uc774 \uc791\ud488\uc73c\ub85c \uc774\ub4ec\ud574\uc778 2015\ub144 \uc62c\ud574\uc758 \uc601\ud654\uc0c1, \ubd80\uc77c\uc601\ud654\uc0c1\uacfc \ub300\uc885\uc0c1 \uc601\ud654\uc81c\uc5d0\uc11c \uc5ec\uc6b0\uc2e0\uc778\uc0c1\uc744 \uc218\uc0c1\ud558\uc600\uace0, 2015\ub144\uc5d0\ub294 \uc601\ud654 \u300a\uac04\uc2e0\u300b\uc73c\ub85c \uadf8\ud574 \uccad\ub8e1\uc601\ud654\uc0c1\uc5d0\uc11c \uc2e0\uc778\uc5ec\uc6b0\uc0c1\uc744 \uc218\uc0c1\ud588\ub2e4. 2018\ub144\uc774 \uae30\ub300\ub418\ub294 \ubc30\uc6b0 \uc911 \ud55c \uba85\uc774\ub2e4.", "sentence_answer": "\uadf8\ub140\ub294 \uace0\ub4f1\ud559\uad50 \uc878\uc5c5 \ud6c4 \ub300\ud559\uc5d0 \uc9c4\ud559\ud558\uc9c0 \uc54a\uace0 \ubbf8\uc6a9\uc2e4\uc5d0\uc11c \ubbf8\uc6a9\uc0ac \ubcf4\uc870 \uc77c\uc744 \ud558\ub2e4\uac00 2010\ub144 22\uc138\uc758 \ub098\uc774\uc5d0 \ud55c\uad6d\uc608\uc220\uc885\ud569\ud559\uad50 \uc5f0\uadf9\uc6d0 \uc5f0\uae30\uacfc\uc5d0 \uc9c4\ud559\ud558\uc600\uace0, \ud55c\uad6d\uc608\uc220\uc885\ud569\ud559\uad50 \uc7ac\ud559 \uc911 \uc0c1\uc5c5\uc601\ud654 \ub370\ubdd4\uc791\uc778 2014\ub144 \uc601\ud654 \u300a \ubd04 \u300b\uc73c\ub85c \ubc00\ub77c\ub178 \uad6d\uc81c \uc601\ud654\uc81c\uc5d0\uc11c \uc5ec\uc6b0\uc8fc\uc5f0\uc0c1\uc744 \ubc1b\uc73c\uba70 \uc8fc\ubaa9\uc744 \ubc1b\uae30 \uc2dc\uc791\ud588\ub2e4.", "paragraph_id": "6545882-0-2", "paragraph_question": "question: \uc774\uc720\uc601\uc774 2014\ub144 \uad6d\uc81c \uc601\ud654\uc81c\uc5d0\uc11c \uc5ec\uc6b0\uc8fc\uc5f0\uc0c1\uc744 \ubc1b\uc740 \uc601\ud654\uc758 \uc81c\ubaa9\uc740 \ubb34\uc5c7\uc778\uac00?, context: \uc774\uc720\uc601\uc740 1989\ub144 12\uc6d4 8\uc77c \uc11c\uc6b8\uc5d0\uc11c \ud0dc\uc5b4\ub0ac\ub2e4. \uadf8\ub140\ub294 \uace0\ub4f1\ud559\uad50 \uc878\uc5c5 \ud6c4 \ub300\ud559\uc5d0 \uc9c4\ud559\ud558\uc9c0 \uc54a\uace0 \ubbf8\uc6a9\uc2e4\uc5d0\uc11c \ubbf8\uc6a9\uc0ac \ubcf4\uc870 \uc77c\uc744 \ud558\ub2e4\uac00 2010\ub144 22\uc138\uc758 \ub098\uc774\uc5d0 \ud55c\uad6d\uc608\uc220\uc885\ud569\ud559\uad50 \uc5f0\uadf9\uc6d0 \uc5f0\uae30\uacfc\uc5d0 \uc9c4\ud559\ud558\uc600\uace0, \ud55c\uad6d\uc608\uc220\uc885\ud569\ud559\uad50 \uc7ac\ud559 \uc911 \uc0c1\uc5c5\uc601\ud654 \ub370\ubdd4\uc791\uc778 2014\ub144 \uc601\ud654 \u300a\ubd04\u300b\uc73c\ub85c \ubc00\ub77c\ub178 \uad6d\uc81c \uc601\ud654\uc81c\uc5d0\uc11c \uc5ec\uc6b0\uc8fc\uc5f0\uc0c1\uc744 \ubc1b\uc73c\uba70 \uc8fc\ubaa9\uc744 \ubc1b\uae30 \uc2dc\uc791\ud588\ub2e4. 1969\ub144 \uacbd\uc0c1\ubd81\ub3c4 \ud3ec\ud56d\uc744 \ubc30\uacbd\uc73c\ub85c \ud55c \uc774 \uc601\ud654\uc5d0\uc11c \uc815\uc219(\uae40\uc11c\ud615 \ubd84)\uc5d0\uac8c \ub3c8\uc744 \ubc1b\uace0 \uc815\uc219\uc758 \ub0a8\ud3b8\uc774\uc790 \ubd88\uce58\ubcd1\uc5d0 \uac78\ub824 \uc2e4\uc758\uc5d0 \ube60\uc9c4 \uc870\uac01\uac00 \uc900\uad6c(\ubc15\uc6a9\uc6b0 \ubd84)\uc758 \ub204\ub4dc\ubaa8\ub378\uc774 \ub418\ub294 \uac00\ub09c\ud55c \uc2dc\uace8 \uc5ec\uc790 \ubbfc\uacbd\uc744 \uc5f0\uae30\ud588\ub2e4. \uc774 \uc5ed\ud560\uc744 \uc5f0\uae30\ud55c \ubc30\uacbd\uc73c\ub85c \uc774\uc720\uc601\uc740 \u201c\uc2dc\ub098\ub9ac\uc624\ub97c \uc77d\uace0 \ud55c\ub208\uc5d0 \uc0ac\ub791\uc5d0 \ube60\uc84c\ub2e4\u201d\uace0 \ub9d0\ud558\uc600\uace0, \ub178\ucd9c \ub54c\ubb38\uc5d0 \uc9c0\uc778\uacfc \uac00\uc871\ub4e4\uc5d0\uac8c \uc601\ud654\ub97c \ubcf4\uc5ec\uc8fc\uae30\uc5d4 \uac71\uc815\uc774 \uc55e\uc130\uc9c0\ub9cc \uc5c4\ub9c8\uc758 \"\ub10c \ub108\uc758 \uc5f0\uae30, \ub108\uc758 \ubab8\uc774 \ucc3d\ud53c\ud558\ub2c8?\"\ub77c\ub294 \ud55c\ub9c8\ub514\uc5d0 \uc6a9\uae30\ub97c \ub0c8\ub2e4\uace0 \ud55c\ub2e4. \uadf8\ub140\ub294 \uc774 \uc791\ud488\uc73c\ub85c \uc774\ub4ec\ud574\uc778 2015\ub144 \uc62c\ud574\uc758 \uc601\ud654\uc0c1, \ubd80\uc77c\uc601\ud654\uc0c1\uacfc \ub300\uc885\uc0c1 \uc601\ud654\uc81c\uc5d0\uc11c \uc5ec\uc6b0\uc2e0\uc778\uc0c1\uc744 \uc218\uc0c1\ud558\uc600\uace0, 2015\ub144\uc5d0\ub294 \uc601\ud654 \u300a\uac04\uc2e0\u300b\uc73c\ub85c \uadf8\ud574 \uccad\ub8e1\uc601\ud654\uc0c1\uc5d0\uc11c \uc2e0\uc778\uc5ec\uc6b0\uc0c1\uc744 \uc218\uc0c1\ud588\ub2e4. 2018\ub144\uc774 \uae30\ub300\ub418\ub294 \ubc30\uc6b0 \uc911 \ud55c \uba85\uc774\ub2e4."} {"question": "\uc870\ubd80\uac00 \ub3cc\uc544\uac00\uc2e0 \ud6c4 \uc548\uc911\uadfc\uc774 \ud559\ubb38\uc5d0 \ud798\uc4f0\uc9c0 \uc54a\uace0 \ubab0\ub450\ud55c \uac83\uc740?", "paragraph": "\ud0dc\uc5b4\ub0a0 \ub54c \ub4f1\uc5d0 \uac80\uc740 \uc810\uc774 7\uac1c\uac00 \uc788\uc5b4\uc11c \ubd81\ub450\uce60\uc131\uc758 \uae30\uc6b4\uc73c\ub85c \ud0dc\uc5b4\ub0ac\ub2e4\ub294 \ub73b\uc73c\ub85c \uc5b4\ub9b4 \ub54c\uc5d0\ub294 \uc751\uce60(\u61c9\u4e03)\uc774\ub77c \ubd88\ub800\ub294\ub370, \uc774 \uc774\ub984\uc744 \ud574\uc678\uc5d0 \uc788\uc744 \ub54c \ub9ce\uc774 \uc0ac\uc6a9\ud588\ub2e4. 1884\ub144 \uac11\uc2e0\uc815\ubcc0 \uc774\ud6c4 \uac1c\ud654\ub2f9 \uc815\uac1d\uc758 \uc2dd\uac1d\uc73c\ub85c \uc788\uc5c8\ub358 \uc544\ubc84\uc9c0 \uc548\ud0dc\ud6c8\uc774 \ucc99\uc2e0 \uc815\uad8c\uc5d0 \uc758\ud574 \uc8fd\uc784\uc744 \ub2f9\ud560 \uc704\uae30\uc5d0 \ub193\uc774\uc790 \ud560\uc544\ubc84\uc9c0 \uc548\uc778\uc218\ub294 \uc601\ud2b9\ud55c \uc14b\uc9f8 \uc544\ub4e4\uc744 \uc0b4\ub9ac\uae30 \uc704\ud574 \uc77c\uac00\ub97c \uc774\ub04c\uace0 \ud669\ud574\ub3c4 \uc2e0\ucc9c\uad70 \ub450\ub77c\uba74 \uccad\uacc4\ub3d9\uc73c\ub85c \ud53c\uc2e0\ud588\ub2e4. \uc548\uc911\uadfc\ub3c4 \uac00\uc194\uc744 \ub530\ub77c \uc2e0\ucc9c \uccad\uacc4\ub3d9\uc73c\ub85c \uc774\uc8fc\ud588\uace0, \uc774\uacf3\uc5d0\uc11c \uc544\ubc84\uc9c0\uac00 \uc138\uc6b4 \uc11c\ub2f9\uc5d0\uc11c \ud6c8\uc7a5\uc744 \ucd08\ube59\ud558\uc5ec \uacf5\ubd80\ub97c \ud588\uc73c\ub098 \uc0ac\uc11c\uc624\uacbd\uc5d0\ub294 \uc774\ub974\uc9c0 \ubabb\ud558\uace0 \ud1b5\uac10 9\uad8c\uae4c\uc9c0\ub9cc \ubc30\uc6e0\ub2e4\uace0 \ud55c\ub2e4. \uc548\uc911\uadfc \uc790\uc2e0\uc740 \u300a\uc548\uc751\uce60\uc5ed\uc0ac\u300b\uc5d0\uc11c \"\ub0b4 \ub098\uc774 \uc608\ub2d0\uacf1 \ub54c \uc870\ubd80\ubaa8\uc758 \uc0ac\ub791\uc744 \ubc1b\uc73c\uba70 \uc11c\ub2f9\uc5d0 \ub4e4\uc5b4\uac00 8~9\ub144 \ub3d9\uc548 \ud55c\ubb38\uc744 \uc775\ud614\uc73c\uba70, \uc870\ubd80\uaed8\uc11c \ub3cc\uc544\uac00\uc2e0 \ub4a4\uc5d0\ub294 \uc0ac\ub0e5\uc73c\ub85c \ud559\ubb38\uc5d0 \ud798\uc4f0\uc9c0 \uc54a\uc558\ub2e4\"\uace0 \uc220\ud68c\ud558\uc600\ub2e4.", "answer": "\uc0ac\ub0e5", "sentence": "\uc548\uc911\uadfc \uc790\uc2e0\uc740 \u300a\uc548\uc751\uce60\uc5ed\uc0ac\u300b\uc5d0\uc11c \"\ub0b4 \ub098\uc774 \uc608\ub2d0\uacf1 \ub54c \uc870\ubd80\ubaa8\uc758 \uc0ac\ub791\uc744 \ubc1b\uc73c\uba70 \uc11c\ub2f9\uc5d0 \ub4e4\uc5b4\uac00 8~9\ub144 \ub3d9\uc548 \ud55c\ubb38\uc744 \uc775\ud614\uc73c\uba70, \uc870\ubd80\uaed8\uc11c \ub3cc\uc544\uac00\uc2e0 \ub4a4\uc5d0\ub294 \uc0ac\ub0e5 \uc73c\ub85c \ud559\ubb38\uc5d0 \ud798\uc4f0\uc9c0 \uc54a\uc558\ub2e4\"\uace0 \uc220\ud68c\ud558\uc600\ub2e4.", "paragraph_sentence": "\ud0dc\uc5b4\ub0a0 \ub54c \ub4f1\uc5d0 \uac80\uc740 \uc810\uc774 7\uac1c\uac00 \uc788\uc5b4\uc11c \ubd81\ub450\uce60\uc131\uc758 \uae30\uc6b4\uc73c\ub85c \ud0dc\uc5b4\ub0ac\ub2e4\ub294 \ub73b\uc73c\ub85c \uc5b4\ub9b4 \ub54c\uc5d0\ub294 \uc751\uce60(\u61c9\u4e03)\uc774\ub77c \ubd88\ub800\ub294\ub370, \uc774 \uc774\ub984\uc744 \ud574\uc678\uc5d0 \uc788\uc744 \ub54c \ub9ce\uc774 \uc0ac\uc6a9\ud588\ub2e4. 1884\ub144 \uac11\uc2e0\uc815\ubcc0 \uc774\ud6c4 \uac1c\ud654\ub2f9 \uc815\uac1d\uc758 \uc2dd\uac1d\uc73c\ub85c \uc788\uc5c8\ub358 \uc544\ubc84\uc9c0 \uc548\ud0dc\ud6c8\uc774 \ucc99\uc2e0 \uc815\uad8c\uc5d0 \uc758\ud574 \uc8fd\uc784\uc744 \ub2f9\ud560 \uc704\uae30\uc5d0 \ub193\uc774\uc790 \ud560\uc544\ubc84\uc9c0 \uc548\uc778\uc218\ub294 \uc601\ud2b9\ud55c \uc14b\uc9f8 \uc544\ub4e4\uc744 \uc0b4\ub9ac\uae30 \uc704\ud574 \uc77c\uac00\ub97c \uc774\ub04c\uace0 \ud669\ud574\ub3c4 \uc2e0\ucc9c\uad70 \ub450\ub77c\uba74 \uccad\uacc4\ub3d9\uc73c\ub85c \ud53c\uc2e0\ud588\ub2e4. \uc548\uc911\uadfc\ub3c4 \uac00\uc194\uc744 \ub530\ub77c \uc2e0\ucc9c \uccad\uacc4\ub3d9\uc73c\ub85c \uc774\uc8fc\ud588\uace0, \uc774\uacf3\uc5d0\uc11c \uc544\ubc84\uc9c0\uac00 \uc138\uc6b4 \uc11c\ub2f9\uc5d0\uc11c \ud6c8\uc7a5\uc744 \ucd08\ube59\ud558\uc5ec \uacf5\ubd80\ub97c \ud588\uc73c\ub098 \uc0ac\uc11c\uc624\uacbd\uc5d0\ub294 \uc774\ub974\uc9c0 \ubabb\ud558\uace0 \ud1b5\uac10 9\uad8c\uae4c\uc9c0\ub9cc \ubc30\uc6e0\ub2e4\uace0 \ud55c\ub2e4. \uc548\uc911\uadfc \uc790\uc2e0\uc740 \u300a\uc548\uc751\uce60\uc5ed\uc0ac\u300b\uc5d0\uc11c \"\ub0b4 \ub098\uc774 \uc608\ub2d0\uacf1 \ub54c \uc870\ubd80\ubaa8\uc758 \uc0ac\ub791\uc744 \ubc1b\uc73c\uba70 \uc11c\ub2f9\uc5d0 \ub4e4\uc5b4\uac00 8~9\ub144 \ub3d9\uc548 \ud55c\ubb38\uc744 \uc775\ud614\uc73c\uba70, \uc870\ubd80\uaed8\uc11c \ub3cc\uc544\uac00\uc2e0 \ub4a4\uc5d0\ub294 \uc0ac\ub0e5 \uc73c\ub85c \ud559\ubb38\uc5d0 \ud798\uc4f0\uc9c0 \uc54a\uc558\ub2e4\"\uace0 \uc220\ud68c\ud558\uc600\ub2e4. ", "paragraph_answer": "\ud0dc\uc5b4\ub0a0 \ub54c \ub4f1\uc5d0 \uac80\uc740 \uc810\uc774 7\uac1c\uac00 \uc788\uc5b4\uc11c \ubd81\ub450\uce60\uc131\uc758 \uae30\uc6b4\uc73c\ub85c \ud0dc\uc5b4\ub0ac\ub2e4\ub294 \ub73b\uc73c\ub85c \uc5b4\ub9b4 \ub54c\uc5d0\ub294 \uc751\uce60(\u61c9\u4e03)\uc774\ub77c \ubd88\ub800\ub294\ub370, \uc774 \uc774\ub984\uc744 \ud574\uc678\uc5d0 \uc788\uc744 \ub54c \ub9ce\uc774 \uc0ac\uc6a9\ud588\ub2e4. 1884\ub144 \uac11\uc2e0\uc815\ubcc0 \uc774\ud6c4 \uac1c\ud654\ub2f9 \uc815\uac1d\uc758 \uc2dd\uac1d\uc73c\ub85c \uc788\uc5c8\ub358 \uc544\ubc84\uc9c0 \uc548\ud0dc\ud6c8\uc774 \ucc99\uc2e0 \uc815\uad8c\uc5d0 \uc758\ud574 \uc8fd\uc784\uc744 \ub2f9\ud560 \uc704\uae30\uc5d0 \ub193\uc774\uc790 \ud560\uc544\ubc84\uc9c0 \uc548\uc778\uc218\ub294 \uc601\ud2b9\ud55c \uc14b\uc9f8 \uc544\ub4e4\uc744 \uc0b4\ub9ac\uae30 \uc704\ud574 \uc77c\uac00\ub97c \uc774\ub04c\uace0 \ud669\ud574\ub3c4 \uc2e0\ucc9c\uad70 \ub450\ub77c\uba74 \uccad\uacc4\ub3d9\uc73c\ub85c \ud53c\uc2e0\ud588\ub2e4. \uc548\uc911\uadfc\ub3c4 \uac00\uc194\uc744 \ub530\ub77c \uc2e0\ucc9c \uccad\uacc4\ub3d9\uc73c\ub85c \uc774\uc8fc\ud588\uace0, \uc774\uacf3\uc5d0\uc11c \uc544\ubc84\uc9c0\uac00 \uc138\uc6b4 \uc11c\ub2f9\uc5d0\uc11c \ud6c8\uc7a5\uc744 \ucd08\ube59\ud558\uc5ec \uacf5\ubd80\ub97c \ud588\uc73c\ub098 \uc0ac\uc11c\uc624\uacbd\uc5d0\ub294 \uc774\ub974\uc9c0 \ubabb\ud558\uace0 \ud1b5\uac10 9\uad8c\uae4c\uc9c0\ub9cc \ubc30\uc6e0\ub2e4\uace0 \ud55c\ub2e4. \uc548\uc911\uadfc \uc790\uc2e0\uc740 \u300a\uc548\uc751\uce60\uc5ed\uc0ac\u300b\uc5d0\uc11c \"\ub0b4 \ub098\uc774 \uc608\ub2d0\uacf1 \ub54c \uc870\ubd80\ubaa8\uc758 \uc0ac\ub791\uc744 \ubc1b\uc73c\uba70 \uc11c\ub2f9\uc5d0 \ub4e4\uc5b4\uac00 8~9\ub144 \ub3d9\uc548 \ud55c\ubb38\uc744 \uc775\ud614\uc73c\uba70, \uc870\ubd80\uaed8\uc11c \ub3cc\uc544\uac00\uc2e0 \ub4a4\uc5d0\ub294 \uc0ac\ub0e5 \uc73c\ub85c \ud559\ubb38\uc5d0 \ud798\uc4f0\uc9c0 \uc54a\uc558\ub2e4\"\uace0 \uc220\ud68c\ud558\uc600\ub2e4.", "sentence_answer": "\uc548\uc911\uadfc \uc790\uc2e0\uc740 \u300a\uc548\uc751\uce60\uc5ed\uc0ac\u300b\uc5d0\uc11c \"\ub0b4 \ub098\uc774 \uc608\ub2d0\uacf1 \ub54c \uc870\ubd80\ubaa8\uc758 \uc0ac\ub791\uc744 \ubc1b\uc73c\uba70 \uc11c\ub2f9\uc5d0 \ub4e4\uc5b4\uac00 8~9\ub144 \ub3d9\uc548 \ud55c\ubb38\uc744 \uc775\ud614\uc73c\uba70, \uc870\ubd80\uaed8\uc11c \ub3cc\uc544\uac00\uc2e0 \ub4a4\uc5d0\ub294 \uc0ac\ub0e5 \uc73c\ub85c \ud559\ubb38\uc5d0 \ud798\uc4f0\uc9c0 \uc54a\uc558\ub2e4\"\uace0 \uc220\ud68c\ud558\uc600\ub2e4.", "paragraph_id": "6538987-1-2", "paragraph_question": "question: \uc870\ubd80\uac00 \ub3cc\uc544\uac00\uc2e0 \ud6c4 \uc548\uc911\uadfc\uc774 \ud559\ubb38\uc5d0 \ud798\uc4f0\uc9c0 \uc54a\uace0 \ubab0\ub450\ud55c \uac83\uc740?, context: \ud0dc\uc5b4\ub0a0 \ub54c \ub4f1\uc5d0 \uac80\uc740 \uc810\uc774 7\uac1c\uac00 \uc788\uc5b4\uc11c \ubd81\ub450\uce60\uc131\uc758 \uae30\uc6b4\uc73c\ub85c \ud0dc\uc5b4\ub0ac\ub2e4\ub294 \ub73b\uc73c\ub85c \uc5b4\ub9b4 \ub54c\uc5d0\ub294 \uc751\uce60(\u61c9\u4e03)\uc774\ub77c \ubd88\ub800\ub294\ub370, \uc774 \uc774\ub984\uc744 \ud574\uc678\uc5d0 \uc788\uc744 \ub54c \ub9ce\uc774 \uc0ac\uc6a9\ud588\ub2e4. 1884\ub144 \uac11\uc2e0\uc815\ubcc0 \uc774\ud6c4 \uac1c\ud654\ub2f9 \uc815\uac1d\uc758 \uc2dd\uac1d\uc73c\ub85c \uc788\uc5c8\ub358 \uc544\ubc84\uc9c0 \uc548\ud0dc\ud6c8\uc774 \ucc99\uc2e0 \uc815\uad8c\uc5d0 \uc758\ud574 \uc8fd\uc784\uc744 \ub2f9\ud560 \uc704\uae30\uc5d0 \ub193\uc774\uc790 \ud560\uc544\ubc84\uc9c0 \uc548\uc778\uc218\ub294 \uc601\ud2b9\ud55c \uc14b\uc9f8 \uc544\ub4e4\uc744 \uc0b4\ub9ac\uae30 \uc704\ud574 \uc77c\uac00\ub97c \uc774\ub04c\uace0 \ud669\ud574\ub3c4 \uc2e0\ucc9c\uad70 \ub450\ub77c\uba74 \uccad\uacc4\ub3d9\uc73c\ub85c \ud53c\uc2e0\ud588\ub2e4. \uc548\uc911\uadfc\ub3c4 \uac00\uc194\uc744 \ub530\ub77c \uc2e0\ucc9c \uccad\uacc4\ub3d9\uc73c\ub85c \uc774\uc8fc\ud588\uace0, \uc774\uacf3\uc5d0\uc11c \uc544\ubc84\uc9c0\uac00 \uc138\uc6b4 \uc11c\ub2f9\uc5d0\uc11c \ud6c8\uc7a5\uc744 \ucd08\ube59\ud558\uc5ec \uacf5\ubd80\ub97c \ud588\uc73c\ub098 \uc0ac\uc11c\uc624\uacbd\uc5d0\ub294 \uc774\ub974\uc9c0 \ubabb\ud558\uace0 \ud1b5\uac10 9\uad8c\uae4c\uc9c0\ub9cc \ubc30\uc6e0\ub2e4\uace0 \ud55c\ub2e4. \uc548\uc911\uadfc \uc790\uc2e0\uc740 \u300a\uc548\uc751\uce60\uc5ed\uc0ac\u300b\uc5d0\uc11c \"\ub0b4 \ub098\uc774 \uc608\ub2d0\uacf1 \ub54c \uc870\ubd80\ubaa8\uc758 \uc0ac\ub791\uc744 \ubc1b\uc73c\uba70 \uc11c\ub2f9\uc5d0 \ub4e4\uc5b4\uac00 8~9\ub144 \ub3d9\uc548 \ud55c\ubb38\uc744 \uc775\ud614\uc73c\uba70, \uc870\ubd80\uaed8\uc11c \ub3cc\uc544\uac00\uc2e0 \ub4a4\uc5d0\ub294 \uc0ac\ub0e5\uc73c\ub85c \ud559\ubb38\uc5d0 \ud798\uc4f0\uc9c0 \uc54a\uc558\ub2e4\"\uace0 \uc220\ud68c\ud558\uc600\ub2e4."} {"question": "\uac04\ubc14\ub8e8\uac00\uc758 \uc544\ub4e4\uc740 \ub204\uad6c\uc785\ub2c8\uae4c?", "paragraph": "\ud0a4\ub9ac\uac00\ucfe0\ub808\uac00\uc758 \ub2cc\uc790\ub85c \uc800\uc8fc\uc5d0 \uac78\ub824 \uac1c\uac00 \ub418\uc5c8\uc9c0\ub9cc \ubb34\uc220 \uc2e4\ub825\uc740 \uadf8\ub300\ub85c\uc774\ub2e4. \ud558\uc9c0\ub9cc \uc544\ub4e4 \uac15\uc138\ucc2c\uc5d0\uac8c \ubb34\uc220\uc744 \uac00\ub974\uccd0 \uc8fc\ub824\ud558\uc9c0\ub9cc \ub3c4\ub9dd\uccd0\ubc84\ub9b0\ub2e4. \uac15\uc138\ubbf8\uac00 \uace4\uc798\ub808\uc2a4\ub85c \uc774\ub984\uc744 \uc0c8\ub85c \uc9c0\uc5b4\uc11c \uace4\uc774\ub77c\uace0 \uc790\uc8fc \ubd80\ub978\ub2e4. \uac1c\uac00 \ub418\uba74\uc11c \ud56d\uc0c1 \ub3d9\ub124 \uace0\uc591\uc774\uc640 \uc2f8\uc6b0\uace0 \ubf08\ub2e4\uadc0\ub9cc \ucc3e\ub294 \uac1c\uadf8\uce90\ub9ad\ud130\uac00 \ub418\ubc84\ub9b0\ub2e4. \ud558\uc9c0\ub9cc \uc790\uc2e0\uc774 \uac1c\ucde8\uae09 \ubc1b\ub294 \uac83\uc744 \uc544\uc8fc \uc2eb\uc5b4\ud55c\ub2e4. 15\ud654\uc5d0\uc11c \uac70\ubd81\uc774 \ub418\uac70\ub098 \uae4c\ub9c8\uadc0\uac00 \ub418\uae30\ub3c4 \ud558\uc9c0\ub9cc \uc5d8\ub4dc\ub780\uc758 \uacf5\uaca9\uc744 \ubc1b\uace0 \uc6d0\ub798 \ubaa8\uc2b5\uc73c\ub85c \ub3cc\uc544\uc640 \ubbf8\ub180\ub9ac\uc6b0\uc2a4\uc640 \ud638\uac01\uc73c\ub85c \ub2e4\ud22c\uace0 \ub04a\uc5b4\uc9c4 \ubc27\uc904\uc744 \uac15\ub825 \uc21c\uac04 \uc811\ucc29\uc81c\ub85c \uc774\uc5b4\uc11c \ub178\ub974\uc544\ud06c\ub97c \ubd09\uc778\uc2dc\ud0a8\ub2e4. \uc774 \ub54c \ub5a8\uc5b4\uc838 \ub098\uac04 \ub178\ub974\uc544\ud06c\uc758 \uc67c \ud314\uc744 \ub9cc\uc9c0\ub294 \ubc14\ub78c\uc5d0 \ub610 \ub2e4\uc2dc \uac1c\ub85c \ubcc0\ud55c\ub2e4. \uc610\ub85c\uac10\ub9c8\uac00 \uae30\uc808\ud574 \uc788\ub294 \ub3d9\uc548 \ub9c8\uacc4\uad34\ubb3c\uc774 \ub9cc\ub4e0 \uac00\uc9dc \uc610\ub85c\uac10\ub9c8 \uc637\uc744 \uc785\uace0 \uc2dc\ub178\ube44\uac10\ub9c8\uac00 \ub3fc\uc11c \uac10\ub9c8 \uc5d0\uc774\uc2a4\uc5d0 \uc62c\ub77c\ud0c0 \ud65c\uc57d\ud558\uac70\ub098 \uac10\ub9c8 \uc0bc\ucd1d\uc0ac\uac00 \ub09c\ub85c \ub9c8\uacc4 \uad34\ubb3c\uc5d0\uac8c \uc704\ud5d8\uc5d0 \ucc98\ud588\uc744 \ub54c \uc544\ub4e4 \uac15\uc138\ucc2c\uc5d0\uac8c \ubb34\uc220\uc744 \uac00\ub974\uccd0 \uc704\ud5d8\uc744 \uadf9\ubcf5\ud558\uac8c \ub9cc\ub4dc\ub294 \ud65c\uc57d\uc744 \ud55c\ub2e4. \ub098\uc911\uc5d0\ub294 3\ub300 \ub9c8\uc655\uc774 \uc4f0\ub7ec\uc9c0\uc790 \uc6d0\ub798\ub300\ub85c \ub3cc\uc544\uc628\ub2e4.", "answer": "\uac15\uc138\ucc2c", "sentence": "\uc544\ub4e4 \uac15\uc138\ucc2c \uc5d0\uac8c \ubb34\uc220\uc744 \uac00\ub974\uccd0 \uc8fc\ub824\ud558\uc9c0\ub9cc \ub3c4\ub9dd\uccd0\ubc84\ub9b0\ub2e4.", "paragraph_sentence": "\ud0a4\ub9ac\uac00\ucfe0\ub808\uac00\uc758 \ub2cc\uc790\ub85c \uc800\uc8fc\uc5d0 \uac78\ub824 \uac1c\uac00 \ub418\uc5c8\uc9c0\ub9cc \ubb34\uc220 \uc2e4\ub825\uc740 \uadf8\ub300\ub85c\uc774\ub2e4. \ud558\uc9c0\ub9cc \uc544\ub4e4 \uac15\uc138\ucc2c \uc5d0\uac8c \ubb34\uc220\uc744 \uac00\ub974\uccd0 \uc8fc\ub824\ud558\uc9c0\ub9cc \ub3c4\ub9dd\uccd0\ubc84\ub9b0\ub2e4. \uac15\uc138\ubbf8\uac00 \uace4\uc798\ub808\uc2a4\ub85c \uc774\ub984\uc744 \uc0c8\ub85c \uc9c0\uc5b4\uc11c \uace4\uc774\ub77c\uace0 \uc790\uc8fc \ubd80\ub978\ub2e4. \uac1c\uac00 \ub418\uba74\uc11c \ud56d\uc0c1 \ub3d9\ub124 \uace0\uc591\uc774\uc640 \uc2f8\uc6b0\uace0 \ubf08\ub2e4\uadc0\ub9cc \ucc3e\ub294 \uac1c\uadf8\uce90\ub9ad\ud130\uac00 \ub418\ubc84\ub9b0\ub2e4. \ud558\uc9c0\ub9cc \uc790\uc2e0\uc774 \uac1c\ucde8\uae09 \ubc1b\ub294 \uac83\uc744 \uc544\uc8fc \uc2eb\uc5b4\ud55c\ub2e4. 15\ud654\uc5d0\uc11c \uac70\ubd81\uc774 \ub418\uac70\ub098 \uae4c\ub9c8\uadc0\uac00 \ub418\uae30\ub3c4 \ud558\uc9c0\ub9cc \uc5d8\ub4dc\ub780\uc758 \uacf5\uaca9\uc744 \ubc1b\uace0 \uc6d0\ub798 \ubaa8\uc2b5\uc73c\ub85c \ub3cc\uc544\uc640 \ubbf8\ub180\ub9ac\uc6b0\uc2a4\uc640 \ud638\uac01\uc73c\ub85c \ub2e4\ud22c\uace0 \ub04a\uc5b4\uc9c4 \ubc27\uc904\uc744 \uac15\ub825 \uc21c\uac04 \uc811\ucc29\uc81c\ub85c \uc774\uc5b4\uc11c \ub178\ub974\uc544\ud06c\ub97c \ubd09\uc778\uc2dc\ud0a8\ub2e4. \uc774 \ub54c \ub5a8\uc5b4\uc838 \ub098\uac04 \ub178\ub974\uc544\ud06c\uc758 \uc67c \ud314\uc744 \ub9cc\uc9c0\ub294 \ubc14\ub78c\uc5d0 \ub610 \ub2e4\uc2dc \uac1c\ub85c \ubcc0\ud55c\ub2e4. \uc610\ub85c\uac10\ub9c8\uac00 \uae30\uc808\ud574 \uc788\ub294 \ub3d9\uc548 \ub9c8\uacc4\uad34\ubb3c\uc774 \ub9cc\ub4e0 \uac00\uc9dc \uc610\ub85c\uac10\ub9c8 \uc637\uc744 \uc785\uace0 \uc2dc\ub178\ube44\uac10\ub9c8\uac00 \ub3fc\uc11c \uac10\ub9c8 \uc5d0\uc774\uc2a4\uc5d0 \uc62c\ub77c\ud0c0 \ud65c\uc57d\ud558\uac70\ub098 \uac10\ub9c8 \uc0bc\ucd1d\uc0ac\uac00 \ub09c\ub85c \ub9c8\uacc4 \uad34\ubb3c\uc5d0\uac8c \uc704\ud5d8\uc5d0 \ucc98\ud588\uc744 \ub54c \uc544\ub4e4 \uac15\uc138\ucc2c\uc5d0\uac8c \ubb34\uc220\uc744 \uac00\ub974\uccd0 \uc704\ud5d8\uc744 \uadf9\ubcf5\ud558\uac8c \ub9cc\ub4dc\ub294 \ud65c\uc57d\uc744 \ud55c\ub2e4. \ub098\uc911\uc5d0\ub294 3\ub300 \ub9c8\uc655\uc774 \uc4f0\ub7ec\uc9c0\uc790 \uc6d0\ub798\ub300\ub85c \ub3cc\uc544\uc628\ub2e4.", "paragraph_answer": "\ud0a4\ub9ac\uac00\ucfe0\ub808\uac00\uc758 \ub2cc\uc790\ub85c \uc800\uc8fc\uc5d0 \uac78\ub824 \uac1c\uac00 \ub418\uc5c8\uc9c0\ub9cc \ubb34\uc220 \uc2e4\ub825\uc740 \uadf8\ub300\ub85c\uc774\ub2e4. \ud558\uc9c0\ub9cc \uc544\ub4e4 \uac15\uc138\ucc2c \uc5d0\uac8c \ubb34\uc220\uc744 \uac00\ub974\uccd0 \uc8fc\ub824\ud558\uc9c0\ub9cc \ub3c4\ub9dd\uccd0\ubc84\ub9b0\ub2e4. \uac15\uc138\ubbf8\uac00 \uace4\uc798\ub808\uc2a4\ub85c \uc774\ub984\uc744 \uc0c8\ub85c \uc9c0\uc5b4\uc11c \uace4\uc774\ub77c\uace0 \uc790\uc8fc \ubd80\ub978\ub2e4. \uac1c\uac00 \ub418\uba74\uc11c \ud56d\uc0c1 \ub3d9\ub124 \uace0\uc591\uc774\uc640 \uc2f8\uc6b0\uace0 \ubf08\ub2e4\uadc0\ub9cc \ucc3e\ub294 \uac1c\uadf8\uce90\ub9ad\ud130\uac00 \ub418\ubc84\ub9b0\ub2e4. \ud558\uc9c0\ub9cc \uc790\uc2e0\uc774 \uac1c\ucde8\uae09 \ubc1b\ub294 \uac83\uc744 \uc544\uc8fc \uc2eb\uc5b4\ud55c\ub2e4. 15\ud654\uc5d0\uc11c \uac70\ubd81\uc774 \ub418\uac70\ub098 \uae4c\ub9c8\uadc0\uac00 \ub418\uae30\ub3c4 \ud558\uc9c0\ub9cc \uc5d8\ub4dc\ub780\uc758 \uacf5\uaca9\uc744 \ubc1b\uace0 \uc6d0\ub798 \ubaa8\uc2b5\uc73c\ub85c \ub3cc\uc544\uc640 \ubbf8\ub180\ub9ac\uc6b0\uc2a4\uc640 \ud638\uac01\uc73c\ub85c \ub2e4\ud22c\uace0 \ub04a\uc5b4\uc9c4 \ubc27\uc904\uc744 \uac15\ub825 \uc21c\uac04 \uc811\ucc29\uc81c\ub85c \uc774\uc5b4\uc11c \ub178\ub974\uc544\ud06c\ub97c \ubd09\uc778\uc2dc\ud0a8\ub2e4. \uc774 \ub54c \ub5a8\uc5b4\uc838 \ub098\uac04 \ub178\ub974\uc544\ud06c\uc758 \uc67c \ud314\uc744 \ub9cc\uc9c0\ub294 \ubc14\ub78c\uc5d0 \ub610 \ub2e4\uc2dc \uac1c\ub85c \ubcc0\ud55c\ub2e4. \uc610\ub85c\uac10\ub9c8\uac00 \uae30\uc808\ud574 \uc788\ub294 \ub3d9\uc548 \ub9c8\uacc4\uad34\ubb3c\uc774 \ub9cc\ub4e0 \uac00\uc9dc \uc610\ub85c\uac10\ub9c8 \uc637\uc744 \uc785\uace0 \uc2dc\ub178\ube44\uac10\ub9c8\uac00 \ub3fc\uc11c \uac10\ub9c8 \uc5d0\uc774\uc2a4\uc5d0 \uc62c\ub77c\ud0c0 \ud65c\uc57d\ud558\uac70\ub098 \uac10\ub9c8 \uc0bc\ucd1d\uc0ac\uac00 \ub09c\ub85c \ub9c8\uacc4 \uad34\ubb3c\uc5d0\uac8c \uc704\ud5d8\uc5d0 \ucc98\ud588\uc744 \ub54c \uc544\ub4e4 \uac15\uc138\ucc2c\uc5d0\uac8c \ubb34\uc220\uc744 \uac00\ub974\uccd0 \uc704\ud5d8\uc744 \uadf9\ubcf5\ud558\uac8c \ub9cc\ub4dc\ub294 \ud65c\uc57d\uc744 \ud55c\ub2e4. \ub098\uc911\uc5d0\ub294 3\ub300 \ub9c8\uc655\uc774 \uc4f0\ub7ec\uc9c0\uc790 \uc6d0\ub798\ub300\ub85c \ub3cc\uc544\uc628\ub2e4.", "sentence_answer": "\uc544\ub4e4 \uac15\uc138\ucc2c \uc5d0\uac8c \ubb34\uc220\uc744 \uac00\ub974\uccd0 \uc8fc\ub824\ud558\uc9c0\ub9cc \ub3c4\ub9dd\uccd0\ubc84\ub9b0\ub2e4.", "paragraph_id": "6526611-1-1", "paragraph_question": "question: \uac04\ubc14\ub8e8\uac00\uc758 \uc544\ub4e4\uc740 \ub204\uad6c\uc785\ub2c8\uae4c?, context: \ud0a4\ub9ac\uac00\ucfe0\ub808\uac00\uc758 \ub2cc\uc790\ub85c \uc800\uc8fc\uc5d0 \uac78\ub824 \uac1c\uac00 \ub418\uc5c8\uc9c0\ub9cc \ubb34\uc220 \uc2e4\ub825\uc740 \uadf8\ub300\ub85c\uc774\ub2e4. \ud558\uc9c0\ub9cc \uc544\ub4e4 \uac15\uc138\ucc2c\uc5d0\uac8c \ubb34\uc220\uc744 \uac00\ub974\uccd0 \uc8fc\ub824\ud558\uc9c0\ub9cc \ub3c4\ub9dd\uccd0\ubc84\ub9b0\ub2e4. \uac15\uc138\ubbf8\uac00 \uace4\uc798\ub808\uc2a4\ub85c \uc774\ub984\uc744 \uc0c8\ub85c \uc9c0\uc5b4\uc11c \uace4\uc774\ub77c\uace0 \uc790\uc8fc \ubd80\ub978\ub2e4. \uac1c\uac00 \ub418\uba74\uc11c \ud56d\uc0c1 \ub3d9\ub124 \uace0\uc591\uc774\uc640 \uc2f8\uc6b0\uace0 \ubf08\ub2e4\uadc0\ub9cc \ucc3e\ub294 \uac1c\uadf8\uce90\ub9ad\ud130\uac00 \ub418\ubc84\ub9b0\ub2e4. \ud558\uc9c0\ub9cc \uc790\uc2e0\uc774 \uac1c\ucde8\uae09 \ubc1b\ub294 \uac83\uc744 \uc544\uc8fc \uc2eb\uc5b4\ud55c\ub2e4. 15\ud654\uc5d0\uc11c \uac70\ubd81\uc774 \ub418\uac70\ub098 \uae4c\ub9c8\uadc0\uac00 \ub418\uae30\ub3c4 \ud558\uc9c0\ub9cc \uc5d8\ub4dc\ub780\uc758 \uacf5\uaca9\uc744 \ubc1b\uace0 \uc6d0\ub798 \ubaa8\uc2b5\uc73c\ub85c \ub3cc\uc544\uc640 \ubbf8\ub180\ub9ac\uc6b0\uc2a4\uc640 \ud638\uac01\uc73c\ub85c \ub2e4\ud22c\uace0 \ub04a\uc5b4\uc9c4 \ubc27\uc904\uc744 \uac15\ub825 \uc21c\uac04 \uc811\ucc29\uc81c\ub85c \uc774\uc5b4\uc11c \ub178\ub974\uc544\ud06c\ub97c \ubd09\uc778\uc2dc\ud0a8\ub2e4. \uc774 \ub54c \ub5a8\uc5b4\uc838 \ub098\uac04 \ub178\ub974\uc544\ud06c\uc758 \uc67c \ud314\uc744 \ub9cc\uc9c0\ub294 \ubc14\ub78c\uc5d0 \ub610 \ub2e4\uc2dc \uac1c\ub85c \ubcc0\ud55c\ub2e4. \uc610\ub85c\uac10\ub9c8\uac00 \uae30\uc808\ud574 \uc788\ub294 \ub3d9\uc548 \ub9c8\uacc4\uad34\ubb3c\uc774 \ub9cc\ub4e0 \uac00\uc9dc \uc610\ub85c\uac10\ub9c8 \uc637\uc744 \uc785\uace0 \uc2dc\ub178\ube44\uac10\ub9c8\uac00 \ub3fc\uc11c \uac10\ub9c8 \uc5d0\uc774\uc2a4\uc5d0 \uc62c\ub77c\ud0c0 \ud65c\uc57d\ud558\uac70\ub098 \uac10\ub9c8 \uc0bc\ucd1d\uc0ac\uac00 \ub09c\ub85c \ub9c8\uacc4 \uad34\ubb3c\uc5d0\uac8c \uc704\ud5d8\uc5d0 \ucc98\ud588\uc744 \ub54c \uc544\ub4e4 \uac15\uc138\ucc2c\uc5d0\uac8c \ubb34\uc220\uc744 \uac00\ub974\uccd0 \uc704\ud5d8\uc744 \uadf9\ubcf5\ud558\uac8c \ub9cc\ub4dc\ub294 \ud65c\uc57d\uc744 \ud55c\ub2e4. \ub098\uc911\uc5d0\ub294 3\ub300 \ub9c8\uc655\uc774 \uc4f0\ub7ec\uc9c0\uc790 \uc6d0\ub798\ub300\ub85c \ub3cc\uc544\uc628\ub2e4."} {"question": "\ub274\uc9c8\ub79c\ub4dc\uc5d0 \ucc98\uc74c \uc624\uc2a4\ud2b8\ub808\uc77c\ub9ac\uc544\uae4c\uce58\uac00 \ub3c4\uc785\ub41c \uac83\uc740 \uc5b8\uc81c\uc778\uac00?", "paragraph": "\uc624\uc2a4\ud2b8\ub808\uc77c\ub9ac\uc544\uae4c\uce58\ub294 \ucf00\uc774\ud504\uc694\ud06c \ubc18\ub3c4 \ubd81\ucabd \ub05d\ub2e8, \uae41\uc2a8 \uc0ac\ub9c9, \uadf8\ub808\uc774\ud2b8\uc0cc\ub514 \uc0ac\ub9c9, \ud0dc\uc988\uba54\uc774\ub2c8\uc544 \uc11c\ub0a8\ubd80\ub97c \uc81c\uc678\ud55c \uc624\uc2a4\ud2b8\ub808\uc77c\ub9ac\uc544 \ub300\ubd80\ubd84 \uc9c0\uc5ed\uc5d0 \ubd84\ud3ec\ud558\uba70, \ub0a0\uc544\uc11c \uac08 \uc218 \uc788\ub294 \ub274\uae30\ub2c8 \ub0a8\ubd80, \uc624\ub9ac\uc624\ubaa8 \uac15\uacfc \ud504\ub9b0\uc138\uc2a4\ub9c8\ub9ac\uc548 \ud574\ud611 \uc0ac\uc774\uc5d0\uc11c \ubcfc \uc218 \uc788\ub2e4. 1860\ub144\ub300\uc5d0 \uc624\ud0c0\uace0\uc640 \uce94\ud130\ubca0\ub9ac \uc9c0\uc5ed \uc21c\ud654\ud611\ud68c\ub4e4\uc774 \ud0dc\uc988\uba54\uc774\ub2c8\uc544\uc640 \ube45\ud1a0\ub9ac\uc544\uc5d0\uc11c \uc7a1\ud78c \uc624\uc2a4\ud2b8\ub808\uc77c\ub9ac\uc544\uae4c\uce58\ub97c \ub274\uc9c8\ub79c\ub4dc\uc5d0 \ub3c4\uc785\ud588\uc73c\uba70, 1874\ub144\uc5d0\ub294 \uc6f0\ub9c1\ud134 \uc21c\ud654\ud611\ud68c\uc5d0\uc11c\ub3c4 260\ub9c8\ub9ac\ub97c \ub4e4\uc5ec\uc654\ub2e4. \ud770\ub4f1\uc624\uc2a4\ud2b8\ub808\uc77c\ub9ac\uc544\uae4c\uce58\ub294 \ubd81\uc12c\uacfc \ub0a8\uc12c \ubaa8\ub450\uc5d0 \uc11c\uc2dd\ud558\ub294 \ubc18\uba74 \uac80\uc740\ub4f1\uc624\uc2a4\ud2b8\ub808\uc77c\ub9ac\uc544\uae4c\uce58\ub294 \ud638\ud06c\uc2a4\ubca0\uc774 \uc9c0\uc5ed\uc5d0\uc11c\ub9cc \ubc1c\uacac\ub41c\ub2e4. \ub274\uc9c8\ub79c\ub4dc\uc5d0 \uc624\uc2a4\ud2b8\ub808\uc77c\ub9ac\uc544\uae4c\uce58\ub97c \ub4e4\uc5ec\uc628 \uac83\uc740 \ub18d\uc791\ubb3c\uc5d0 \uc720\ud574\ud55c \ubc8c\ub808\ub4e4\uc744 \uad6c\uc81c\ud558\uae30 \uc704\ud55c \uac83\uc774\uc5c8\uc73c\uba70, \ub54c\ubb38\uc5d0 1951\ub144\uae4c\uc9c0 \ubc95\uc73c\ub85c \ubcf4\ud638\ub418\uc5c8\ub2e4. \uc624\uc2a4\ud2b8\ub808\uc77c\ub9ac\uc544\uae4c\uce58\uac00 \ub274\uc9c8\ub79c\ub4dc \ud1a0\ucc29 \uc0c8\uc778 \ud22c\uc774\ub098 \ucf00\ub808\ub8e8 \ub530\uc704\uc758 \uac1c\uccb4\uc218\uc5d0 \uc601\ud5a5\uc744 \ubbf8\uce58\uace0 \ub54c\ub85c\ub294 \ub465\uc9c0\ub97c \uc2b5\uaca9\ud574 \uc54c\uc744 \ud30c\uad34\ud55c\ub2e4\ub294 \uc18d\uc124\uc774 \uc788\uc73c\ub098, \uc640\uc774\uce74\ud1a0 \ub300\ud559\uad50\uc758 \uc5f0\uad6c\uc5d0\uc11c\ub294 \uc774 \uc18d\uc124\uc5d0 \uc758\ubb38\uc744 \ud45c\ud588\uc73c\uba70, \uc624\uc2a4\ud2b8\ub808\uc77c\ub9ac\uc544\uae4c\uce58\ub97c \ud1a0\ucc29\uc885 \ud3ec\uc2dd\uc790\ub85c \uc9c0\ubaa9\ud55c\ub2e4\ub294 \ud1b5\uc124\uc5d0 \ub300\ud574\uc11c\ub3c4 \ub300\ubd80\ubd84 \ubb3c\uc99d\uc774 \ud655\ubcf4\ub418\uc9c0 \uc54a\uc558\ub2e4. \uc194\ub85c\ubaac \uc81c\ub3c4\uc640 \uc2a4\ub9ac\ub791\uce74\uc5d0\ub294 \ub3c4\uc785 \ud6c4 \uc815\ucc29\ud558\uc9c0 \ubabb\ud588\uc73c\ub098 \ud53c\uc9c0\uc5d0\uc11c\ub294 \ud0c0\ubca0\uc6b0\ub2c8 \uc11c\ubd80\uc5d0 \uc815\ucc29\ud55c \ubc14 \uc788\ub2e4.", "answer": "1860\ub144\ub300", "sentence": "1860\ub144\ub300 \uc5d0 \uc624\ud0c0\uace0\uc640 \uce94\ud130\ubca0\ub9ac \uc9c0\uc5ed \uc21c\ud654\ud611\ud68c\ub4e4\uc774 \ud0dc\uc988\uba54\uc774\ub2c8\uc544\uc640 \ube45\ud1a0\ub9ac\uc544\uc5d0\uc11c \uc7a1\ud78c \uc624\uc2a4\ud2b8\ub808\uc77c\ub9ac\uc544\uae4c\uce58\ub97c \ub274\uc9c8\ub79c\ub4dc\uc5d0 \ub3c4\uc785\ud588\uc73c\uba70, 1874\ub144\uc5d0\ub294 \uc6f0\ub9c1\ud134 \uc21c\ud654\ud611\ud68c\uc5d0\uc11c\ub3c4 260\ub9c8\ub9ac\ub97c \ub4e4\uc5ec\uc654\ub2e4.", "paragraph_sentence": "\uc624\uc2a4\ud2b8\ub808\uc77c\ub9ac\uc544\uae4c\uce58\ub294 \ucf00\uc774\ud504\uc694\ud06c \ubc18\ub3c4 \ubd81\ucabd \ub05d\ub2e8, \uae41\uc2a8 \uc0ac\ub9c9, \uadf8\ub808\uc774\ud2b8\uc0cc\ub514 \uc0ac\ub9c9, \ud0dc\uc988\uba54\uc774\ub2c8\uc544 \uc11c\ub0a8\ubd80\ub97c \uc81c\uc678\ud55c \uc624\uc2a4\ud2b8\ub808\uc77c\ub9ac\uc544 \ub300\ubd80\ubd84 \uc9c0\uc5ed\uc5d0 \ubd84\ud3ec\ud558\uba70, \ub0a0\uc544\uc11c \uac08 \uc218 \uc788\ub294 \ub274\uae30\ub2c8 \ub0a8\ubd80, \uc624\ub9ac\uc624\ubaa8 \uac15\uacfc \ud504\ub9b0\uc138\uc2a4\ub9c8\ub9ac\uc548 \ud574\ud611 \uc0ac\uc774\uc5d0\uc11c \ubcfc \uc218 \uc788\ub2e4. 1860\ub144\ub300 \uc5d0 \uc624\ud0c0\uace0\uc640 \uce94\ud130\ubca0\ub9ac \uc9c0\uc5ed \uc21c\ud654\ud611\ud68c\ub4e4\uc774 \ud0dc\uc988\uba54\uc774\ub2c8\uc544\uc640 \ube45\ud1a0\ub9ac\uc544\uc5d0\uc11c \uc7a1\ud78c \uc624\uc2a4\ud2b8\ub808\uc77c\ub9ac\uc544\uae4c\uce58\ub97c \ub274\uc9c8\ub79c\ub4dc\uc5d0 \ub3c4\uc785\ud588\uc73c\uba70, 1874\ub144\uc5d0\ub294 \uc6f0\ub9c1\ud134 \uc21c\ud654\ud611\ud68c\uc5d0\uc11c\ub3c4 260\ub9c8\ub9ac\ub97c \ub4e4\uc5ec\uc654\ub2e4. \ud770\ub4f1\uc624\uc2a4\ud2b8\ub808\uc77c\ub9ac\uc544\uae4c\uce58\ub294 \ubd81\uc12c\uacfc \ub0a8\uc12c \ubaa8\ub450\uc5d0 \uc11c\uc2dd\ud558\ub294 \ubc18\uba74 \uac80\uc740\ub4f1\uc624\uc2a4\ud2b8\ub808\uc77c\ub9ac\uc544\uae4c\uce58\ub294 \ud638\ud06c\uc2a4\ubca0\uc774 \uc9c0\uc5ed\uc5d0\uc11c\ub9cc \ubc1c\uacac\ub41c\ub2e4. \ub274\uc9c8\ub79c\ub4dc\uc5d0 \uc624\uc2a4\ud2b8\ub808\uc77c\ub9ac\uc544\uae4c\uce58\ub97c \ub4e4\uc5ec\uc628 \uac83\uc740 \ub18d\uc791\ubb3c\uc5d0 \uc720\ud574\ud55c \ubc8c\ub808\ub4e4\uc744 \uad6c\uc81c\ud558\uae30 \uc704\ud55c \uac83\uc774\uc5c8\uc73c\uba70, \ub54c\ubb38\uc5d0 1951\ub144\uae4c\uc9c0 \ubc95\uc73c\ub85c \ubcf4\ud638\ub418\uc5c8\ub2e4. \uc624\uc2a4\ud2b8\ub808\uc77c\ub9ac\uc544\uae4c\uce58\uac00 \ub274\uc9c8\ub79c\ub4dc \ud1a0\ucc29 \uc0c8\uc778 \ud22c\uc774\ub098 \ucf00\ub808\ub8e8 \ub530\uc704\uc758 \uac1c\uccb4\uc218\uc5d0 \uc601\ud5a5\uc744 \ubbf8\uce58\uace0 \ub54c\ub85c\ub294 \ub465\uc9c0\ub97c \uc2b5\uaca9\ud574 \uc54c\uc744 \ud30c\uad34\ud55c\ub2e4\ub294 \uc18d\uc124\uc774 \uc788\uc73c\ub098, \uc640\uc774\uce74\ud1a0 \ub300\ud559\uad50\uc758 \uc5f0\uad6c\uc5d0\uc11c\ub294 \uc774 \uc18d\uc124\uc5d0 \uc758\ubb38\uc744 \ud45c\ud588\uc73c\uba70, \uc624\uc2a4\ud2b8\ub808\uc77c\ub9ac\uc544\uae4c\uce58\ub97c \ud1a0\ucc29\uc885 \ud3ec\uc2dd\uc790\ub85c \uc9c0\ubaa9\ud55c\ub2e4\ub294 \ud1b5\uc124\uc5d0 \ub300\ud574\uc11c\ub3c4 \ub300\ubd80\ubd84 \ubb3c\uc99d\uc774 \ud655\ubcf4\ub418\uc9c0 \uc54a\uc558\ub2e4. \uc194\ub85c\ubaac \uc81c\ub3c4\uc640 \uc2a4\ub9ac\ub791\uce74\uc5d0\ub294 \ub3c4\uc785 \ud6c4 \uc815\ucc29\ud558\uc9c0 \ubabb\ud588\uc73c\ub098 \ud53c\uc9c0\uc5d0\uc11c\ub294 \ud0c0\ubca0\uc6b0\ub2c8 \uc11c\ubd80\uc5d0 \uc815\ucc29\ud55c \ubc14 \uc788\ub2e4.", "paragraph_answer": "\uc624\uc2a4\ud2b8\ub808\uc77c\ub9ac\uc544\uae4c\uce58\ub294 \ucf00\uc774\ud504\uc694\ud06c \ubc18\ub3c4 \ubd81\ucabd \ub05d\ub2e8, \uae41\uc2a8 \uc0ac\ub9c9, \uadf8\ub808\uc774\ud2b8\uc0cc\ub514 \uc0ac\ub9c9, \ud0dc\uc988\uba54\uc774\ub2c8\uc544 \uc11c\ub0a8\ubd80\ub97c \uc81c\uc678\ud55c \uc624\uc2a4\ud2b8\ub808\uc77c\ub9ac\uc544 \ub300\ubd80\ubd84 \uc9c0\uc5ed\uc5d0 \ubd84\ud3ec\ud558\uba70, \ub0a0\uc544\uc11c \uac08 \uc218 \uc788\ub294 \ub274\uae30\ub2c8 \ub0a8\ubd80, \uc624\ub9ac\uc624\ubaa8 \uac15\uacfc \ud504\ub9b0\uc138\uc2a4\ub9c8\ub9ac\uc548 \ud574\ud611 \uc0ac\uc774\uc5d0\uc11c \ubcfc \uc218 \uc788\ub2e4. 1860\ub144\ub300 \uc5d0 \uc624\ud0c0\uace0\uc640 \uce94\ud130\ubca0\ub9ac \uc9c0\uc5ed \uc21c\ud654\ud611\ud68c\ub4e4\uc774 \ud0dc\uc988\uba54\uc774\ub2c8\uc544\uc640 \ube45\ud1a0\ub9ac\uc544\uc5d0\uc11c \uc7a1\ud78c \uc624\uc2a4\ud2b8\ub808\uc77c\ub9ac\uc544\uae4c\uce58\ub97c \ub274\uc9c8\ub79c\ub4dc\uc5d0 \ub3c4\uc785\ud588\uc73c\uba70, 1874\ub144\uc5d0\ub294 \uc6f0\ub9c1\ud134 \uc21c\ud654\ud611\ud68c\uc5d0\uc11c\ub3c4 260\ub9c8\ub9ac\ub97c \ub4e4\uc5ec\uc654\ub2e4. \ud770\ub4f1\uc624\uc2a4\ud2b8\ub808\uc77c\ub9ac\uc544\uae4c\uce58\ub294 \ubd81\uc12c\uacfc \ub0a8\uc12c \ubaa8\ub450\uc5d0 \uc11c\uc2dd\ud558\ub294 \ubc18\uba74 \uac80\uc740\ub4f1\uc624\uc2a4\ud2b8\ub808\uc77c\ub9ac\uc544\uae4c\uce58\ub294 \ud638\ud06c\uc2a4\ubca0\uc774 \uc9c0\uc5ed\uc5d0\uc11c\ub9cc \ubc1c\uacac\ub41c\ub2e4. \ub274\uc9c8\ub79c\ub4dc\uc5d0 \uc624\uc2a4\ud2b8\ub808\uc77c\ub9ac\uc544\uae4c\uce58\ub97c \ub4e4\uc5ec\uc628 \uac83\uc740 \ub18d\uc791\ubb3c\uc5d0 \uc720\ud574\ud55c \ubc8c\ub808\ub4e4\uc744 \uad6c\uc81c\ud558\uae30 \uc704\ud55c \uac83\uc774\uc5c8\uc73c\uba70, \ub54c\ubb38\uc5d0 1951\ub144\uae4c\uc9c0 \ubc95\uc73c\ub85c \ubcf4\ud638\ub418\uc5c8\ub2e4. \uc624\uc2a4\ud2b8\ub808\uc77c\ub9ac\uc544\uae4c\uce58\uac00 \ub274\uc9c8\ub79c\ub4dc \ud1a0\ucc29 \uc0c8\uc778 \ud22c\uc774\ub098 \ucf00\ub808\ub8e8 \ub530\uc704\uc758 \uac1c\uccb4\uc218\uc5d0 \uc601\ud5a5\uc744 \ubbf8\uce58\uace0 \ub54c\ub85c\ub294 \ub465\uc9c0\ub97c \uc2b5\uaca9\ud574 \uc54c\uc744 \ud30c\uad34\ud55c\ub2e4\ub294 \uc18d\uc124\uc774 \uc788\uc73c\ub098, \uc640\uc774\uce74\ud1a0 \ub300\ud559\uad50\uc758 \uc5f0\uad6c\uc5d0\uc11c\ub294 \uc774 \uc18d\uc124\uc5d0 \uc758\ubb38\uc744 \ud45c\ud588\uc73c\uba70, \uc624\uc2a4\ud2b8\ub808\uc77c\ub9ac\uc544\uae4c\uce58\ub97c \ud1a0\ucc29\uc885 \ud3ec\uc2dd\uc790\ub85c \uc9c0\ubaa9\ud55c\ub2e4\ub294 \ud1b5\uc124\uc5d0 \ub300\ud574\uc11c\ub3c4 \ub300\ubd80\ubd84 \ubb3c\uc99d\uc774 \ud655\ubcf4\ub418\uc9c0 \uc54a\uc558\ub2e4. \uc194\ub85c\ubaac \uc81c\ub3c4\uc640 \uc2a4\ub9ac\ub791\uce74\uc5d0\ub294 \ub3c4\uc785 \ud6c4 \uc815\ucc29\ud558\uc9c0 \ubabb\ud588\uc73c\ub098 \ud53c\uc9c0\uc5d0\uc11c\ub294 \ud0c0\ubca0\uc6b0\ub2c8 \uc11c\ubd80\uc5d0 \uc815\ucc29\ud55c \ubc14 \uc788\ub2e4.", "sentence_answer": " 1860\ub144\ub300 \uc5d0 \uc624\ud0c0\uace0\uc640 \uce94\ud130\ubca0\ub9ac \uc9c0\uc5ed \uc21c\ud654\ud611\ud68c\ub4e4\uc774 \ud0dc\uc988\uba54\uc774\ub2c8\uc544\uc640 \ube45\ud1a0\ub9ac\uc544\uc5d0\uc11c \uc7a1\ud78c \uc624\uc2a4\ud2b8\ub808\uc77c\ub9ac\uc544\uae4c\uce58\ub97c \ub274\uc9c8\ub79c\ub4dc\uc5d0 \ub3c4\uc785\ud588\uc73c\uba70, 1874\ub144\uc5d0\ub294 \uc6f0\ub9c1\ud134 \uc21c\ud654\ud611\ud68c\uc5d0\uc11c\ub3c4 260\ub9c8\ub9ac\ub97c \ub4e4\uc5ec\uc654\ub2e4.", "paragraph_id": "5842758-6-0", "paragraph_question": "question: \ub274\uc9c8\ub79c\ub4dc\uc5d0 \ucc98\uc74c \uc624\uc2a4\ud2b8\ub808\uc77c\ub9ac\uc544\uae4c\uce58\uac00 \ub3c4\uc785\ub41c \uac83\uc740 \uc5b8\uc81c\uc778\uac00?, context: \uc624\uc2a4\ud2b8\ub808\uc77c\ub9ac\uc544\uae4c\uce58\ub294 \ucf00\uc774\ud504\uc694\ud06c \ubc18\ub3c4 \ubd81\ucabd \ub05d\ub2e8, \uae41\uc2a8 \uc0ac\ub9c9, \uadf8\ub808\uc774\ud2b8\uc0cc\ub514 \uc0ac\ub9c9, \ud0dc\uc988\uba54\uc774\ub2c8\uc544 \uc11c\ub0a8\ubd80\ub97c \uc81c\uc678\ud55c \uc624\uc2a4\ud2b8\ub808\uc77c\ub9ac\uc544 \ub300\ubd80\ubd84 \uc9c0\uc5ed\uc5d0 \ubd84\ud3ec\ud558\uba70, \ub0a0\uc544\uc11c \uac08 \uc218 \uc788\ub294 \ub274\uae30\ub2c8 \ub0a8\ubd80, \uc624\ub9ac\uc624\ubaa8 \uac15\uacfc \ud504\ub9b0\uc138\uc2a4\ub9c8\ub9ac\uc548 \ud574\ud611 \uc0ac\uc774\uc5d0\uc11c \ubcfc \uc218 \uc788\ub2e4. 1860\ub144\ub300\uc5d0 \uc624\ud0c0\uace0\uc640 \uce94\ud130\ubca0\ub9ac \uc9c0\uc5ed \uc21c\ud654\ud611\ud68c\ub4e4\uc774 \ud0dc\uc988\uba54\uc774\ub2c8\uc544\uc640 \ube45\ud1a0\ub9ac\uc544\uc5d0\uc11c \uc7a1\ud78c \uc624\uc2a4\ud2b8\ub808\uc77c\ub9ac\uc544\uae4c\uce58\ub97c \ub274\uc9c8\ub79c\ub4dc\uc5d0 \ub3c4\uc785\ud588\uc73c\uba70, 1874\ub144\uc5d0\ub294 \uc6f0\ub9c1\ud134 \uc21c\ud654\ud611\ud68c\uc5d0\uc11c\ub3c4 260\ub9c8\ub9ac\ub97c \ub4e4\uc5ec\uc654\ub2e4. \ud770\ub4f1\uc624\uc2a4\ud2b8\ub808\uc77c\ub9ac\uc544\uae4c\uce58\ub294 \ubd81\uc12c\uacfc \ub0a8\uc12c \ubaa8\ub450\uc5d0 \uc11c\uc2dd\ud558\ub294 \ubc18\uba74 \uac80\uc740\ub4f1\uc624\uc2a4\ud2b8\ub808\uc77c\ub9ac\uc544\uae4c\uce58\ub294 \ud638\ud06c\uc2a4\ubca0\uc774 \uc9c0\uc5ed\uc5d0\uc11c\ub9cc \ubc1c\uacac\ub41c\ub2e4. \ub274\uc9c8\ub79c\ub4dc\uc5d0 \uc624\uc2a4\ud2b8\ub808\uc77c\ub9ac\uc544\uae4c\uce58\ub97c \ub4e4\uc5ec\uc628 \uac83\uc740 \ub18d\uc791\ubb3c\uc5d0 \uc720\ud574\ud55c \ubc8c\ub808\ub4e4\uc744 \uad6c\uc81c\ud558\uae30 \uc704\ud55c \uac83\uc774\uc5c8\uc73c\uba70, \ub54c\ubb38\uc5d0 1951\ub144\uae4c\uc9c0 \ubc95\uc73c\ub85c \ubcf4\ud638\ub418\uc5c8\ub2e4. \uc624\uc2a4\ud2b8\ub808\uc77c\ub9ac\uc544\uae4c\uce58\uac00 \ub274\uc9c8\ub79c\ub4dc \ud1a0\ucc29 \uc0c8\uc778 \ud22c\uc774\ub098 \ucf00\ub808\ub8e8 \ub530\uc704\uc758 \uac1c\uccb4\uc218\uc5d0 \uc601\ud5a5\uc744 \ubbf8\uce58\uace0 \ub54c\ub85c\ub294 \ub465\uc9c0\ub97c \uc2b5\uaca9\ud574 \uc54c\uc744 \ud30c\uad34\ud55c\ub2e4\ub294 \uc18d\uc124\uc774 \uc788\uc73c\ub098, \uc640\uc774\uce74\ud1a0 \ub300\ud559\uad50\uc758 \uc5f0\uad6c\uc5d0\uc11c\ub294 \uc774 \uc18d\uc124\uc5d0 \uc758\ubb38\uc744 \ud45c\ud588\uc73c\uba70, \uc624\uc2a4\ud2b8\ub808\uc77c\ub9ac\uc544\uae4c\uce58\ub97c \ud1a0\ucc29\uc885 \ud3ec\uc2dd\uc790\ub85c \uc9c0\ubaa9\ud55c\ub2e4\ub294 \ud1b5\uc124\uc5d0 \ub300\ud574\uc11c\ub3c4 \ub300\ubd80\ubd84 \ubb3c\uc99d\uc774 \ud655\ubcf4\ub418\uc9c0 \uc54a\uc558\ub2e4. \uc194\ub85c\ubaac \uc81c\ub3c4\uc640 \uc2a4\ub9ac\ub791\uce74\uc5d0\ub294 \ub3c4\uc785 \ud6c4 \uc815\ucc29\ud558\uc9c0 \ubabb\ud588\uc73c\ub098 \ud53c\uc9c0\uc5d0\uc11c\ub294 \ud0c0\ubca0\uc6b0\ub2c8 \uc11c\ubd80\uc5d0 \uc815\ucc29\ud55c \ubc14 \uc788\ub2e4."} {"question": "\ubc15\uadfc\ud61c\uac00 \uc911\uc694\ud55c \uc0ac\uc548\uc5d0\uc11c \uc9e7\uc740 \ub9d0\uc744 \ub358\uc9c0\ub294 \uac83\uc740 \uac00\ub9ac\ud0a4\ub294 \ub9d0\uc740 \ubb34\uc5c7\uc778\uac00?", "paragraph": "2008\ub144 \ub300\uc120 \uc774\ud6c4 \ubc15\uadfc\ud61c\ub294 \ubbf8\uad6d\uc18c \uc218\uc785 \ud611\uc0c1\uc5d0 \ub300\ud55c \ubc18\ubc1c\uc774 \uc77c\ub358 \uac00\uc6b4\ub370 \uc774\uba85\ubc15\uc5d0 \ub300\ud55c \ube44\ud310\uc744 \uc81c\uae30\ud558\uac70\ub098, \ubbf8\ub514\uc5b4\ubc95 \uc0c1\uc815\uc2dc \"\ubc18\ub300\ud45c\ub97c \ub358\uc9c8\uac83\"\uc774\ub77c\uace0 \ub9d0\ud558\uac70\ub098, \uc774\uba85\ubc15\uc774 \ub0b4\ub193\uc740 \uc138\uc885\uc2dc \uc218\uc815\uc548\uc5d0 \ub300\ud574 \ubc18\ub300 \uc785\uc7a5\uc744 \ubc1d\ud788\uac70\ub098, \ub3d9\ub0a8\uad8c \uc2e0\uacf5\ud56d \ubc31\uc9c0\ud654 \ub2f9\uc2dc \uc774\uba85\ubc15\uc758 \uacb0\uc815\uc744 \ubc1b\uc544\ub4e4\uc774\uc9c0 \uc54a\uace0 \ucc28\uae30 \uc815\uad8c\uc5d0\uc11c \uacc4\uc18d \ucd94\uc9c4 \ud560 \uac83\uc744 \uc57d\uc18d\ud558\ub294 \ub4f1 \uc911\uc694\ud55c \uc0ac\uc548\uc5d0\uc11c \uc9e7\uc740 \ub9d0\uc744 \ub358\uc9c0\ub294 \uc77c\uba85 '\ud55c\ub9c8\ub514 \uc815\uce58'\ub97c \ud574\uc654\ub2e4. 2010\ub144\uae4c\uc9c0 \ucc28\uae30 \ub300\uc120\ud6c4\ubcf4 \uc9c0\uc9c0\uc728 1\uc704\ub97c \uc9c0\ucf1c\uc628 \ubc15\uadfc\ud61c\uc758 \uc774\ub7f0 \uc758\uacac\ub4e4\uc740 \ub2f9\uc815\uc5d0 \ud070 \uc601\ud5a5\ub825\uc744 \ub07c\ucce4\uc73c\uba70 \uce5c\uc774\uacc4\ub294 \uc815\ubd80\uc5d0 \ud611\uc870\ud558\uc9c0\ub3c4, \uadf8\ub807\ub2e4\uace0 \ubc18\ub300\ud558\uc9c0\ub3c4 \uc54a\ub294 \ubc15\uadfc\ud61c\uc758 \ubaa8\ud638\ud55c \ud589\ubcf4\uc5d0 \ub300\ud574 \ubd88\ub9cc\uc744 \ud1a0\ub85c\ud558\uae30\ub3c4 \ud588\ub2e4. \uadf8\ub3d9\uc548 \ud55c\ub098\ub77c\ub2f9\uacfc \uc774\uba85\ubc15 \uc815\ubd80\ub294 \ubc15\uadfc\ud61c\ub97c \ub2f9\ub300\ud45c\ub098, \uad6d\ubb34\ucd1d\ub9ac\ub85c \ucd94\ub300\ud558\ub294 \ubc29\uc548\uc744 \uace0\ub824\ud574\uc654\uc9c0\ub9cc \ubc15\uadfc\ud61c\ub294 \uc774\ub97c \ubaa8\ub450 \uac70\ubd80\ud558\uc600\uc73c\uba70 \uce5c\ubc15\uacc4 \uc758\uc6d0\uc774 \ub2f9\ub300\ud45c\ub97c \ub9e1\ub294 \uac83\uc870\ucc28 \ubc18\ub300\ud558\ub294 \ub4f1 \uc18c\uadf9\uc801\uc778 \ud589\ubcf4\ub97c \ubcf4\uc5ec\uc654\ub2e4. \ud558\uc9c0\ub9cc \uc9c0\uc9c0\uc728 1\uc704\ub97c \uc9c0\ud0a4\uae30 \uc704\ud574 \ub108\ubb34 \ubab8\uc0ac\ub9ac\uae30\ub9cc \ud558\ub294 \uac83 \uc544\ub2c8\ub0d0\ub294 \ube44\ud310\uc774 \uc81c\uae30\ub418\uace0 \uc788\uc73c\uba70 \uc774\uba85\ubc15\uacfc \ud654\ud574\ub97c \ud558\uac70\ub098, \uc544\uc608 \ub2e4\ub978 \ub178\uc120\uc744 \uc120\ud0dd\ud558\uc5ec \uacb0\ub2e8\uc744 \ub0b4\ub824\uc57c \ud55c\ub2e4\ub294 \uc9c0\uc801\uc774 \uc81c\uae30\ub418\uc5c8\ub2e4. \uc774\ub7ec\ud55c \ube44\ud310\ub4e4 \uc18d\uc5d0\uc11c \uc774\uba85\ubc15 \uc815\uad8c \ucde8\uc784 \ucd08\uae30\uc5d0 \uc2e4\uc2dc\ud588\ub358 \uc790\uae30 \ub300\uc120 \ud6c4\ubcf4\ub4e4 \uc9c0\uc9c0\uc728 \uc870\uc0ac\uc5d0\uc11c 50%\uc5d0 \uc721\ubc15\ud558\ub358 \ubc15\uadfc\ud61c\uc758 \uc9c0\uc9c0\uc728\uc740 2010\ub144 \uc774\ud6c4\ub85c 30%\uae4c\uc9c0 \ud558\ub77d\ud588\ub2e4.", "answer": "\ud55c\ub9c8\ub514 \uc815\uce58", "sentence": "2008\ub144 \ub300\uc120 \uc774\ud6c4 \ubc15\uadfc\ud61c\ub294 \ubbf8\uad6d\uc18c \uc218\uc785 \ud611\uc0c1\uc5d0 \ub300\ud55c \ubc18\ubc1c\uc774 \uc77c\ub358 \uac00\uc6b4\ub370 \uc774\uba85\ubc15\uc5d0 \ub300\ud55c \ube44\ud310\uc744 \uc81c\uae30\ud558\uac70\ub098, \ubbf8\ub514\uc5b4\ubc95 \uc0c1\uc815\uc2dc \"\ubc18\ub300\ud45c\ub97c \ub358\uc9c8\uac83\"\uc774\ub77c\uace0 \ub9d0\ud558\uac70\ub098, \uc774\uba85\ubc15\uc774 \ub0b4\ub193\uc740 \uc138\uc885\uc2dc \uc218\uc815\uc548\uc5d0 \ub300\ud574 \ubc18\ub300 \uc785\uc7a5\uc744 \ubc1d\ud788\uac70\ub098, \ub3d9\ub0a8\uad8c \uc2e0\uacf5\ud56d \ubc31\uc9c0\ud654 \ub2f9\uc2dc \uc774\uba85\ubc15\uc758 \uacb0\uc815\uc744 \ubc1b\uc544\ub4e4\uc774\uc9c0 \uc54a\uace0 \ucc28\uae30 \uc815\uad8c\uc5d0\uc11c \uacc4\uc18d \ucd94\uc9c4 \ud560 \uac83\uc744 \uc57d\uc18d\ud558\ub294 \ub4f1 \uc911\uc694\ud55c \uc0ac\uc548\uc5d0\uc11c \uc9e7\uc740 \ub9d0\uc744 \ub358\uc9c0\ub294 \uc77c\uba85 ' \ud55c\ub9c8\ub514 \uc815\uce58 '\ub97c \ud574\uc654\ub2e4.", "paragraph_sentence": " 2008\ub144 \ub300\uc120 \uc774\ud6c4 \ubc15\uadfc\ud61c\ub294 \ubbf8\uad6d\uc18c \uc218\uc785 \ud611\uc0c1\uc5d0 \ub300\ud55c \ubc18\ubc1c\uc774 \uc77c\ub358 \uac00\uc6b4\ub370 \uc774\uba85\ubc15\uc5d0 \ub300\ud55c \ube44\ud310\uc744 \uc81c\uae30\ud558\uac70\ub098, \ubbf8\ub514\uc5b4\ubc95 \uc0c1\uc815\uc2dc \"\ubc18\ub300\ud45c\ub97c \ub358\uc9c8\uac83\"\uc774\ub77c\uace0 \ub9d0\ud558\uac70\ub098, \uc774\uba85\ubc15\uc774 \ub0b4\ub193\uc740 \uc138\uc885\uc2dc \uc218\uc815\uc548\uc5d0 \ub300\ud574 \ubc18\ub300 \uc785\uc7a5\uc744 \ubc1d\ud788\uac70\ub098, \ub3d9\ub0a8\uad8c \uc2e0\uacf5\ud56d \ubc31\uc9c0\ud654 \ub2f9\uc2dc \uc774\uba85\ubc15\uc758 \uacb0\uc815\uc744 \ubc1b\uc544\ub4e4\uc774\uc9c0 \uc54a\uace0 \ucc28\uae30 \uc815\uad8c\uc5d0\uc11c \uacc4\uc18d \ucd94\uc9c4 \ud560 \uac83\uc744 \uc57d\uc18d\ud558\ub294 \ub4f1 \uc911\uc694\ud55c \uc0ac\uc548\uc5d0\uc11c \uc9e7\uc740 \ub9d0\uc744 \ub358\uc9c0\ub294 \uc77c\uba85 ' \ud55c\ub9c8\ub514 \uc815\uce58 '\ub97c \ud574\uc654\ub2e4. 2010\ub144\uae4c\uc9c0 \ucc28\uae30 \ub300\uc120\ud6c4\ubcf4 \uc9c0\uc9c0\uc728 1\uc704\ub97c \uc9c0\ucf1c\uc628 \ubc15\uadfc\ud61c\uc758 \uc774\ub7f0 \uc758\uacac\ub4e4\uc740 \ub2f9\uc815\uc5d0 \ud070 \uc601\ud5a5\ub825\uc744 \ub07c\ucce4\uc73c\uba70 \uce5c\uc774\uacc4\ub294 \uc815\ubd80\uc5d0 \ud611\uc870\ud558\uc9c0\ub3c4, \uadf8\ub807\ub2e4\uace0 \ubc18\ub300\ud558\uc9c0\ub3c4 \uc54a\ub294 \ubc15\uadfc\ud61c\uc758 \ubaa8\ud638\ud55c \ud589\ubcf4\uc5d0 \ub300\ud574 \ubd88\ub9cc\uc744 \ud1a0\ub85c\ud558\uae30\ub3c4 \ud588\ub2e4. \uadf8\ub3d9\uc548 \ud55c\ub098\ub77c\ub2f9\uacfc \uc774\uba85\ubc15 \uc815\ubd80\ub294 \ubc15\uadfc\ud61c\ub97c \ub2f9\ub300\ud45c\ub098, \uad6d\ubb34\ucd1d\ub9ac\ub85c \ucd94\ub300\ud558\ub294 \ubc29\uc548\uc744 \uace0\ub824\ud574\uc654\uc9c0\ub9cc \ubc15\uadfc\ud61c\ub294 \uc774\ub97c \ubaa8\ub450 \uac70\ubd80\ud558\uc600\uc73c\uba70 \uce5c\ubc15\uacc4 \uc758\uc6d0\uc774 \ub2f9\ub300\ud45c\ub97c \ub9e1\ub294 \uac83\uc870\ucc28 \ubc18\ub300\ud558\ub294 \ub4f1 \uc18c\uadf9\uc801\uc778 \ud589\ubcf4\ub97c \ubcf4\uc5ec\uc654\ub2e4. \ud558\uc9c0\ub9cc \uc9c0\uc9c0\uc728 1\uc704\ub97c \uc9c0\ud0a4\uae30 \uc704\ud574 \ub108\ubb34 \ubab8\uc0ac\ub9ac\uae30\ub9cc \ud558\ub294 \uac83 \uc544\ub2c8\ub0d0\ub294 \ube44\ud310\uc774 \uc81c\uae30\ub418\uace0 \uc788\uc73c\uba70 \uc774\uba85\ubc15\uacfc \ud654\ud574\ub97c \ud558\uac70\ub098, \uc544\uc608 \ub2e4\ub978 \ub178\uc120\uc744 \uc120\ud0dd\ud558\uc5ec \uacb0\ub2e8\uc744 \ub0b4\ub824\uc57c \ud55c\ub2e4\ub294 \uc9c0\uc801\uc774 \uc81c\uae30\ub418\uc5c8\ub2e4. \uc774\ub7ec\ud55c \ube44\ud310\ub4e4 \uc18d\uc5d0\uc11c \uc774\uba85\ubc15 \uc815\uad8c \ucde8\uc784 \ucd08\uae30\uc5d0 \uc2e4\uc2dc\ud588\ub358 \uc790\uae30 \ub300\uc120 \ud6c4\ubcf4\ub4e4 \uc9c0\uc9c0\uc728 \uc870\uc0ac\uc5d0\uc11c 50%\uc5d0 \uc721\ubc15\ud558\ub358 \ubc15\uadfc\ud61c\uc758 \uc9c0\uc9c0\uc728\uc740 2010\ub144 \uc774\ud6c4\ub85c 30%\uae4c\uc9c0 \ud558\ub77d\ud588\ub2e4.", "paragraph_answer": "2008\ub144 \ub300\uc120 \uc774\ud6c4 \ubc15\uadfc\ud61c\ub294 \ubbf8\uad6d\uc18c \uc218\uc785 \ud611\uc0c1\uc5d0 \ub300\ud55c \ubc18\ubc1c\uc774 \uc77c\ub358 \uac00\uc6b4\ub370 \uc774\uba85\ubc15\uc5d0 \ub300\ud55c \ube44\ud310\uc744 \uc81c\uae30\ud558\uac70\ub098, \ubbf8\ub514\uc5b4\ubc95 \uc0c1\uc815\uc2dc \"\ubc18\ub300\ud45c\ub97c \ub358\uc9c8\uac83\"\uc774\ub77c\uace0 \ub9d0\ud558\uac70\ub098, \uc774\uba85\ubc15\uc774 \ub0b4\ub193\uc740 \uc138\uc885\uc2dc \uc218\uc815\uc548\uc5d0 \ub300\ud574 \ubc18\ub300 \uc785\uc7a5\uc744 \ubc1d\ud788\uac70\ub098, \ub3d9\ub0a8\uad8c \uc2e0\uacf5\ud56d \ubc31\uc9c0\ud654 \ub2f9\uc2dc \uc774\uba85\ubc15\uc758 \uacb0\uc815\uc744 \ubc1b\uc544\ub4e4\uc774\uc9c0 \uc54a\uace0 \ucc28\uae30 \uc815\uad8c\uc5d0\uc11c \uacc4\uc18d \ucd94\uc9c4 \ud560 \uac83\uc744 \uc57d\uc18d\ud558\ub294 \ub4f1 \uc911\uc694\ud55c \uc0ac\uc548\uc5d0\uc11c \uc9e7\uc740 \ub9d0\uc744 \ub358\uc9c0\ub294 \uc77c\uba85 ' \ud55c\ub9c8\ub514 \uc815\uce58 '\ub97c \ud574\uc654\ub2e4. 2010\ub144\uae4c\uc9c0 \ucc28\uae30 \ub300\uc120\ud6c4\ubcf4 \uc9c0\uc9c0\uc728 1\uc704\ub97c \uc9c0\ucf1c\uc628 \ubc15\uadfc\ud61c\uc758 \uc774\ub7f0 \uc758\uacac\ub4e4\uc740 \ub2f9\uc815\uc5d0 \ud070 \uc601\ud5a5\ub825\uc744 \ub07c\ucce4\uc73c\uba70 \uce5c\uc774\uacc4\ub294 \uc815\ubd80\uc5d0 \ud611\uc870\ud558\uc9c0\ub3c4, \uadf8\ub807\ub2e4\uace0 \ubc18\ub300\ud558\uc9c0\ub3c4 \uc54a\ub294 \ubc15\uadfc\ud61c\uc758 \ubaa8\ud638\ud55c \ud589\ubcf4\uc5d0 \ub300\ud574 \ubd88\ub9cc\uc744 \ud1a0\ub85c\ud558\uae30\ub3c4 \ud588\ub2e4. \uadf8\ub3d9\uc548 \ud55c\ub098\ub77c\ub2f9\uacfc \uc774\uba85\ubc15 \uc815\ubd80\ub294 \ubc15\uadfc\ud61c\ub97c \ub2f9\ub300\ud45c\ub098, \uad6d\ubb34\ucd1d\ub9ac\ub85c \ucd94\ub300\ud558\ub294 \ubc29\uc548\uc744 \uace0\ub824\ud574\uc654\uc9c0\ub9cc \ubc15\uadfc\ud61c\ub294 \uc774\ub97c \ubaa8\ub450 \uac70\ubd80\ud558\uc600\uc73c\uba70 \uce5c\ubc15\uacc4 \uc758\uc6d0\uc774 \ub2f9\ub300\ud45c\ub97c \ub9e1\ub294 \uac83\uc870\ucc28 \ubc18\ub300\ud558\ub294 \ub4f1 \uc18c\uadf9\uc801\uc778 \ud589\ubcf4\ub97c \ubcf4\uc5ec\uc654\ub2e4. \ud558\uc9c0\ub9cc \uc9c0\uc9c0\uc728 1\uc704\ub97c \uc9c0\ud0a4\uae30 \uc704\ud574 \ub108\ubb34 \ubab8\uc0ac\ub9ac\uae30\ub9cc \ud558\ub294 \uac83 \uc544\ub2c8\ub0d0\ub294 \ube44\ud310\uc774 \uc81c\uae30\ub418\uace0 \uc788\uc73c\uba70 \uc774\uba85\ubc15\uacfc \ud654\ud574\ub97c \ud558\uac70\ub098, \uc544\uc608 \ub2e4\ub978 \ub178\uc120\uc744 \uc120\ud0dd\ud558\uc5ec \uacb0\ub2e8\uc744 \ub0b4\ub824\uc57c \ud55c\ub2e4\ub294 \uc9c0\uc801\uc774 \uc81c\uae30\ub418\uc5c8\ub2e4. \uc774\ub7ec\ud55c \ube44\ud310\ub4e4 \uc18d\uc5d0\uc11c \uc774\uba85\ubc15 \uc815\uad8c \ucde8\uc784 \ucd08\uae30\uc5d0 \uc2e4\uc2dc\ud588\ub358 \uc790\uae30 \ub300\uc120 \ud6c4\ubcf4\ub4e4 \uc9c0\uc9c0\uc728 \uc870\uc0ac\uc5d0\uc11c 50%\uc5d0 \uc721\ubc15\ud558\ub358 \ubc15\uadfc\ud61c\uc758 \uc9c0\uc9c0\uc728\uc740 2010\ub144 \uc774\ud6c4\ub85c 30%\uae4c\uc9c0 \ud558\ub77d\ud588\ub2e4.", "sentence_answer": "2008\ub144 \ub300\uc120 \uc774\ud6c4 \ubc15\uadfc\ud61c\ub294 \ubbf8\uad6d\uc18c \uc218\uc785 \ud611\uc0c1\uc5d0 \ub300\ud55c \ubc18\ubc1c\uc774 \uc77c\ub358 \uac00\uc6b4\ub370 \uc774\uba85\ubc15\uc5d0 \ub300\ud55c \ube44\ud310\uc744 \uc81c\uae30\ud558\uac70\ub098, \ubbf8\ub514\uc5b4\ubc95 \uc0c1\uc815\uc2dc \"\ubc18\ub300\ud45c\ub97c \ub358\uc9c8\uac83\"\uc774\ub77c\uace0 \ub9d0\ud558\uac70\ub098, \uc774\uba85\ubc15\uc774 \ub0b4\ub193\uc740 \uc138\uc885\uc2dc \uc218\uc815\uc548\uc5d0 \ub300\ud574 \ubc18\ub300 \uc785\uc7a5\uc744 \ubc1d\ud788\uac70\ub098, \ub3d9\ub0a8\uad8c \uc2e0\uacf5\ud56d \ubc31\uc9c0\ud654 \ub2f9\uc2dc \uc774\uba85\ubc15\uc758 \uacb0\uc815\uc744 \ubc1b\uc544\ub4e4\uc774\uc9c0 \uc54a\uace0 \ucc28\uae30 \uc815\uad8c\uc5d0\uc11c \uacc4\uc18d \ucd94\uc9c4 \ud560 \uac83\uc744 \uc57d\uc18d\ud558\ub294 \ub4f1 \uc911\uc694\ud55c \uc0ac\uc548\uc5d0\uc11c \uc9e7\uc740 \ub9d0\uc744 \ub358\uc9c0\ub294 \uc77c\uba85 ' \ud55c\ub9c8\ub514 \uc815\uce58 '\ub97c \ud574\uc654\ub2e4.", "paragraph_id": "6581158-31-1", "paragraph_question": "question: \ubc15\uadfc\ud61c\uac00 \uc911\uc694\ud55c \uc0ac\uc548\uc5d0\uc11c \uc9e7\uc740 \ub9d0\uc744 \ub358\uc9c0\ub294 \uac83\uc740 \uac00\ub9ac\ud0a4\ub294 \ub9d0\uc740 \ubb34\uc5c7\uc778\uac00?, context: 2008\ub144 \ub300\uc120 \uc774\ud6c4 \ubc15\uadfc\ud61c\ub294 \ubbf8\uad6d\uc18c \uc218\uc785 \ud611\uc0c1\uc5d0 \ub300\ud55c \ubc18\ubc1c\uc774 \uc77c\ub358 \uac00\uc6b4\ub370 \uc774\uba85\ubc15\uc5d0 \ub300\ud55c \ube44\ud310\uc744 \uc81c\uae30\ud558\uac70\ub098, \ubbf8\ub514\uc5b4\ubc95 \uc0c1\uc815\uc2dc \"\ubc18\ub300\ud45c\ub97c \ub358\uc9c8\uac83\"\uc774\ub77c\uace0 \ub9d0\ud558\uac70\ub098, \uc774\uba85\ubc15\uc774 \ub0b4\ub193\uc740 \uc138\uc885\uc2dc \uc218\uc815\uc548\uc5d0 \ub300\ud574 \ubc18\ub300 \uc785\uc7a5\uc744 \ubc1d\ud788\uac70\ub098, \ub3d9\ub0a8\uad8c \uc2e0\uacf5\ud56d \ubc31\uc9c0\ud654 \ub2f9\uc2dc \uc774\uba85\ubc15\uc758 \uacb0\uc815\uc744 \ubc1b\uc544\ub4e4\uc774\uc9c0 \uc54a\uace0 \ucc28\uae30 \uc815\uad8c\uc5d0\uc11c \uacc4\uc18d \ucd94\uc9c4 \ud560 \uac83\uc744 \uc57d\uc18d\ud558\ub294 \ub4f1 \uc911\uc694\ud55c \uc0ac\uc548\uc5d0\uc11c \uc9e7\uc740 \ub9d0\uc744 \ub358\uc9c0\ub294 \uc77c\uba85 '\ud55c\ub9c8\ub514 \uc815\uce58'\ub97c \ud574\uc654\ub2e4. 2010\ub144\uae4c\uc9c0 \ucc28\uae30 \ub300\uc120\ud6c4\ubcf4 \uc9c0\uc9c0\uc728 1\uc704\ub97c \uc9c0\ucf1c\uc628 \ubc15\uadfc\ud61c\uc758 \uc774\ub7f0 \uc758\uacac\ub4e4\uc740 \ub2f9\uc815\uc5d0 \ud070 \uc601\ud5a5\ub825\uc744 \ub07c\ucce4\uc73c\uba70 \uce5c\uc774\uacc4\ub294 \uc815\ubd80\uc5d0 \ud611\uc870\ud558\uc9c0\ub3c4, \uadf8\ub807\ub2e4\uace0 \ubc18\ub300\ud558\uc9c0\ub3c4 \uc54a\ub294 \ubc15\uadfc\ud61c\uc758 \ubaa8\ud638\ud55c \ud589\ubcf4\uc5d0 \ub300\ud574 \ubd88\ub9cc\uc744 \ud1a0\ub85c\ud558\uae30\ub3c4 \ud588\ub2e4. \uadf8\ub3d9\uc548 \ud55c\ub098\ub77c\ub2f9\uacfc \uc774\uba85\ubc15 \uc815\ubd80\ub294 \ubc15\uadfc\ud61c\ub97c \ub2f9\ub300\ud45c\ub098, \uad6d\ubb34\ucd1d\ub9ac\ub85c \ucd94\ub300\ud558\ub294 \ubc29\uc548\uc744 \uace0\ub824\ud574\uc654\uc9c0\ub9cc \ubc15\uadfc\ud61c\ub294 \uc774\ub97c \ubaa8\ub450 \uac70\ubd80\ud558\uc600\uc73c\uba70 \uce5c\ubc15\uacc4 \uc758\uc6d0\uc774 \ub2f9\ub300\ud45c\ub97c \ub9e1\ub294 \uac83\uc870\ucc28 \ubc18\ub300\ud558\ub294 \ub4f1 \uc18c\uadf9\uc801\uc778 \ud589\ubcf4\ub97c \ubcf4\uc5ec\uc654\ub2e4. \ud558\uc9c0\ub9cc \uc9c0\uc9c0\uc728 1\uc704\ub97c \uc9c0\ud0a4\uae30 \uc704\ud574 \ub108\ubb34 \ubab8\uc0ac\ub9ac\uae30\ub9cc \ud558\ub294 \uac83 \uc544\ub2c8\ub0d0\ub294 \ube44\ud310\uc774 \uc81c\uae30\ub418\uace0 \uc788\uc73c\uba70 \uc774\uba85\ubc15\uacfc \ud654\ud574\ub97c \ud558\uac70\ub098, \uc544\uc608 \ub2e4\ub978 \ub178\uc120\uc744 \uc120\ud0dd\ud558\uc5ec \uacb0\ub2e8\uc744 \ub0b4\ub824\uc57c \ud55c\ub2e4\ub294 \uc9c0\uc801\uc774 \uc81c\uae30\ub418\uc5c8\ub2e4. \uc774\ub7ec\ud55c \ube44\ud310\ub4e4 \uc18d\uc5d0\uc11c \uc774\uba85\ubc15 \uc815\uad8c \ucde8\uc784 \ucd08\uae30\uc5d0 \uc2e4\uc2dc\ud588\ub358 \uc790\uae30 \ub300\uc120 \ud6c4\ubcf4\ub4e4 \uc9c0\uc9c0\uc728 \uc870\uc0ac\uc5d0\uc11c 50%\uc5d0 \uc721\ubc15\ud558\ub358 \ubc15\uadfc\ud61c\uc758 \uc9c0\uc9c0\uc728\uc740 2010\ub144 \uc774\ud6c4\ub85c 30%\uae4c\uc9c0 \ud558\ub77d\ud588\ub2e4."} {"question": "\uae34\uae09 \uc870\uce58\ub97c \ud574\uc9c0\ud55c \uc0ac\ub78c\uc740 \ub204\uad6c\uc778\uac00\uc694?", "paragraph": "1979\ub144 10\uc6d4 26\uc77c \uc911\uc559\uc815\ubcf4\ubd80\uc7a5 \uae40\uc7ac\uaddc\uac00 \ubc15\uc815\ud76c \ub300\ud1b5\ub839\uc744 \uc0b4\ud574\ud55c 10\u00b726 \uc0ac\uac74\uc73c\ub85c \uc720\uc2e0 \uccb4\uc81c\ub294 \ub9c9\uc744 \ub0b4\ub838\ub2e4. \uc720\uc2e0\ud5cc\ubc95\uc744 \uac1c\uc815\ud558\uace0 \ubbfc\uc8fc\uc801\uc778 \ud5cc\ubc95\uc73c\ub85c \ub418\ub3cc\uc544\uc57c \ud55c\ub2e4\ub294 \uc6c0\uc9c1\uc784 \uc18d\uc5d0\uc11c, 11\uc6d4 7\uc77c \ucd5c\uaddc\ud558 \ub300\ud1b5\ub839\uc740 \uae34\uae09\uc870\uce58\ub97c \ud574\uc81c\ud574 \uae34\uae09\uc870\uce58\uc5d0 \uc758\ud574 \uae08\uc9c0\ub410\ub358 \uac1c\ud5cc \ub17c\uc758\ub97c \ud5c8\uc6a9\ud588\ub2e4. \ud558\uc9c0\ub9cc 12\uc6d4 12\uc77c \uacc4\uc5c4\uc0ac\ub839\ubd80 \ud569\ub3d9\uc218\uc0ac\ubcf8\ubd80\uc7a5 \uc804\ub450\ud658 \ubcf4\uc548\uc0ac\ub839\uad00\uc774 \uacc4\uc5c4\uc0ac\ub839\uad00 \uc815\uc2b9\ud654(\uc721\uad70 \ucc38\ubaa8\ucd1d\uc7a5)\ub97c \uccb4\ud3ec\ud574 \ubc18\ub780\uc744 \uc77c\uc73c\ud0b4\uc73c\ub85c\uc368, \uad6d\ubbfc\ub4e4\uc758 \ubbfc\uc8fc \uc815\uad8c \uc218\ub9bd \uc694\uad6c\ub294 \uacb0\uad6d \uc774\ub8e8\uc5b4\uc9c0\uc9c0 \ubabb\ud588\ub2e4. 1980\ub144 2\uc6d4 \uc804\ub450\ud658\uc740 \ubcf4\uc548\uc0ac\ub839\ubd80\uc5d0 \uc9c0\uc2dc\ub97c \ub0b4\ub824 K-\uacf5\uc791\uacc4\ud68d\uc744 \uc2e4\ud589\ud574 \ubbfc\uc8fc\ud654 \uc5ec\ub860\uc744 \uc7a0\uc7ac\uc6b0\uace0 \uad70\ubd80\uc758 \uc815\uce58 \ucc38\uc5ec\ub97c \uc815\ub2f9\ud654\ud558\ub294 \ubc29\ud5a5\uc73c\ub85c \uc5ec\ub860\uc744 \uc870\uc131\ud574 \ub098\uac00\uace0 \uc788\uc5c8\ub2e4.", "answer": "\ucd5c\uaddc\ud558 \ub300\ud1b5\ub839", "sentence": "\uc720\uc2e0\ud5cc\ubc95\uc744 \uac1c\uc815\ud558\uace0 \ubbfc\uc8fc\uc801\uc778 \ud5cc\ubc95\uc73c\ub85c \ub418\ub3cc\uc544\uc57c \ud55c\ub2e4\ub294 \uc6c0\uc9c1\uc784 \uc18d\uc5d0\uc11c, 11\uc6d4 7\uc77c \ucd5c\uaddc\ud558 \ub300\ud1b5\ub839 \uc740 \uae34\uae09\uc870\uce58\ub97c \ud574\uc81c\ud574 \uae34\uae09\uc870\uce58\uc5d0 \uc758\ud574 \uae08\uc9c0\ub410\ub358 \uac1c\ud5cc \ub17c\uc758\ub97c \ud5c8\uc6a9\ud588\ub2e4.", "paragraph_sentence": "1979\ub144 10\uc6d4 26\uc77c \uc911\uc559\uc815\ubcf4\ubd80\uc7a5 \uae40\uc7ac\uaddc\uac00 \ubc15\uc815\ud76c \ub300\ud1b5\ub839\uc744 \uc0b4\ud574\ud55c 10\u00b726 \uc0ac\uac74\uc73c\ub85c \uc720\uc2e0 \uccb4\uc81c\ub294 \ub9c9\uc744 \ub0b4\ub838\ub2e4. \uc720\uc2e0\ud5cc\ubc95\uc744 \uac1c\uc815\ud558\uace0 \ubbfc\uc8fc\uc801\uc778 \ud5cc\ubc95\uc73c\ub85c \ub418\ub3cc\uc544\uc57c \ud55c\ub2e4\ub294 \uc6c0\uc9c1\uc784 \uc18d\uc5d0\uc11c, 11\uc6d4 7\uc77c \ucd5c\uaddc\ud558 \ub300\ud1b5\ub839 \uc740 \uae34\uae09\uc870\uce58\ub97c \ud574\uc81c\ud574 \uae34\uae09\uc870\uce58\uc5d0 \uc758\ud574 \uae08\uc9c0\ub410\ub358 \uac1c\ud5cc \ub17c\uc758\ub97c \ud5c8\uc6a9\ud588\ub2e4. \ud558\uc9c0\ub9cc 12\uc6d4 12\uc77c \uacc4\uc5c4\uc0ac\ub839\ubd80 \ud569\ub3d9\uc218\uc0ac\ubcf8\ubd80\uc7a5 \uc804\ub450\ud658 \ubcf4\uc548\uc0ac\ub839\uad00\uc774 \uacc4\uc5c4\uc0ac\ub839\uad00 \uc815\uc2b9\ud654(\uc721\uad70 \ucc38\ubaa8\ucd1d\uc7a5)\ub97c \uccb4\ud3ec\ud574 \ubc18\ub780\uc744 \uc77c\uc73c\ud0b4\uc73c\ub85c\uc368, \uad6d\ubbfc\ub4e4\uc758 \ubbfc\uc8fc \uc815\uad8c \uc218\ub9bd \uc694\uad6c\ub294 \uacb0\uad6d \uc774\ub8e8\uc5b4\uc9c0\uc9c0 \ubabb\ud588\ub2e4. 1980\ub144 2\uc6d4 \uc804\ub450\ud658\uc740 \ubcf4\uc548\uc0ac\ub839\ubd80\uc5d0 \uc9c0\uc2dc\ub97c \ub0b4\ub824 K-\uacf5\uc791\uacc4\ud68d\uc744 \uc2e4\ud589\ud574 \ubbfc\uc8fc\ud654 \uc5ec\ub860\uc744 \uc7a0\uc7ac\uc6b0\uace0 \uad70\ubd80\uc758 \uc815\uce58 \ucc38\uc5ec\ub97c \uc815\ub2f9\ud654\ud558\ub294 \ubc29\ud5a5\uc73c\ub85c \uc5ec\ub860\uc744 \uc870\uc131\ud574 \ub098\uac00\uace0 \uc788\uc5c8\ub2e4.", "paragraph_answer": "1979\ub144 10\uc6d4 26\uc77c \uc911\uc559\uc815\ubcf4\ubd80\uc7a5 \uae40\uc7ac\uaddc\uac00 \ubc15\uc815\ud76c \ub300\ud1b5\ub839\uc744 \uc0b4\ud574\ud55c 10\u00b726 \uc0ac\uac74\uc73c\ub85c \uc720\uc2e0 \uccb4\uc81c\ub294 \ub9c9\uc744 \ub0b4\ub838\ub2e4. \uc720\uc2e0\ud5cc\ubc95\uc744 \uac1c\uc815\ud558\uace0 \ubbfc\uc8fc\uc801\uc778 \ud5cc\ubc95\uc73c\ub85c \ub418\ub3cc\uc544\uc57c \ud55c\ub2e4\ub294 \uc6c0\uc9c1\uc784 \uc18d\uc5d0\uc11c, 11\uc6d4 7\uc77c \ucd5c\uaddc\ud558 \ub300\ud1b5\ub839 \uc740 \uae34\uae09\uc870\uce58\ub97c \ud574\uc81c\ud574 \uae34\uae09\uc870\uce58\uc5d0 \uc758\ud574 \uae08\uc9c0\ub410\ub358 \uac1c\ud5cc \ub17c\uc758\ub97c \ud5c8\uc6a9\ud588\ub2e4. \ud558\uc9c0\ub9cc 12\uc6d4 12\uc77c \uacc4\uc5c4\uc0ac\ub839\ubd80 \ud569\ub3d9\uc218\uc0ac\ubcf8\ubd80\uc7a5 \uc804\ub450\ud658 \ubcf4\uc548\uc0ac\ub839\uad00\uc774 \uacc4\uc5c4\uc0ac\ub839\uad00 \uc815\uc2b9\ud654(\uc721\uad70 \ucc38\ubaa8\ucd1d\uc7a5)\ub97c \uccb4\ud3ec\ud574 \ubc18\ub780\uc744 \uc77c\uc73c\ud0b4\uc73c\ub85c\uc368, \uad6d\ubbfc\ub4e4\uc758 \ubbfc\uc8fc \uc815\uad8c \uc218\ub9bd \uc694\uad6c\ub294 \uacb0\uad6d \uc774\ub8e8\uc5b4\uc9c0\uc9c0 \ubabb\ud588\ub2e4. 1980\ub144 2\uc6d4 \uc804\ub450\ud658\uc740 \ubcf4\uc548\uc0ac\ub839\ubd80\uc5d0 \uc9c0\uc2dc\ub97c \ub0b4\ub824 K-\uacf5\uc791\uacc4\ud68d\uc744 \uc2e4\ud589\ud574 \ubbfc\uc8fc\ud654 \uc5ec\ub860\uc744 \uc7a0\uc7ac\uc6b0\uace0 \uad70\ubd80\uc758 \uc815\uce58 \ucc38\uc5ec\ub97c \uc815\ub2f9\ud654\ud558\ub294 \ubc29\ud5a5\uc73c\ub85c \uc5ec\ub860\uc744 \uc870\uc131\ud574 \ub098\uac00\uace0 \uc788\uc5c8\ub2e4.", "sentence_answer": "\uc720\uc2e0\ud5cc\ubc95\uc744 \uac1c\uc815\ud558\uace0 \ubbfc\uc8fc\uc801\uc778 \ud5cc\ubc95\uc73c\ub85c \ub418\ub3cc\uc544\uc57c \ud55c\ub2e4\ub294 \uc6c0\uc9c1\uc784 \uc18d\uc5d0\uc11c, 11\uc6d4 7\uc77c \ucd5c\uaddc\ud558 \ub300\ud1b5\ub839 \uc740 \uae34\uae09\uc870\uce58\ub97c \ud574\uc81c\ud574 \uae34\uae09\uc870\uce58\uc5d0 \uc758\ud574 \uae08\uc9c0\ub410\ub358 \uac1c\ud5cc \ub17c\uc758\ub97c \ud5c8\uc6a9\ud588\ub2e4.", "paragraph_id": "5801882-5-1", "paragraph_question": "question: \uae34\uae09 \uc870\uce58\ub97c \ud574\uc9c0\ud55c \uc0ac\ub78c\uc740 \ub204\uad6c\uc778\uac00\uc694?, context: 1979\ub144 10\uc6d4 26\uc77c \uc911\uc559\uc815\ubcf4\ubd80\uc7a5 \uae40\uc7ac\uaddc\uac00 \ubc15\uc815\ud76c \ub300\ud1b5\ub839\uc744 \uc0b4\ud574\ud55c 10\u00b726 \uc0ac\uac74\uc73c\ub85c \uc720\uc2e0 \uccb4\uc81c\ub294 \ub9c9\uc744 \ub0b4\ub838\ub2e4. \uc720\uc2e0\ud5cc\ubc95\uc744 \uac1c\uc815\ud558\uace0 \ubbfc\uc8fc\uc801\uc778 \ud5cc\ubc95\uc73c\ub85c \ub418\ub3cc\uc544\uc57c \ud55c\ub2e4\ub294 \uc6c0\uc9c1\uc784 \uc18d\uc5d0\uc11c, 11\uc6d4 7\uc77c \ucd5c\uaddc\ud558 \ub300\ud1b5\ub839\uc740 \uae34\uae09\uc870\uce58\ub97c \ud574\uc81c\ud574 \uae34\uae09\uc870\uce58\uc5d0 \uc758\ud574 \uae08\uc9c0\ub410\ub358 \uac1c\ud5cc \ub17c\uc758\ub97c \ud5c8\uc6a9\ud588\ub2e4. \ud558\uc9c0\ub9cc 12\uc6d4 12\uc77c \uacc4\uc5c4\uc0ac\ub839\ubd80 \ud569\ub3d9\uc218\uc0ac\ubcf8\ubd80\uc7a5 \uc804\ub450\ud658 \ubcf4\uc548\uc0ac\ub839\uad00\uc774 \uacc4\uc5c4\uc0ac\ub839\uad00 \uc815\uc2b9\ud654(\uc721\uad70 \ucc38\ubaa8\ucd1d\uc7a5)\ub97c \uccb4\ud3ec\ud574 \ubc18\ub780\uc744 \uc77c\uc73c\ud0b4\uc73c\ub85c\uc368, \uad6d\ubbfc\ub4e4\uc758 \ubbfc\uc8fc \uc815\uad8c \uc218\ub9bd \uc694\uad6c\ub294 \uacb0\uad6d \uc774\ub8e8\uc5b4\uc9c0\uc9c0 \ubabb\ud588\ub2e4. 1980\ub144 2\uc6d4 \uc804\ub450\ud658\uc740 \ubcf4\uc548\uc0ac\ub839\ubd80\uc5d0 \uc9c0\uc2dc\ub97c \ub0b4\ub824 K-\uacf5\uc791\uacc4\ud68d\uc744 \uc2e4\ud589\ud574 \ubbfc\uc8fc\ud654 \uc5ec\ub860\uc744 \uc7a0\uc7ac\uc6b0\uace0 \uad70\ubd80\uc758 \uc815\uce58 \ucc38\uc5ec\ub97c \uc815\ub2f9\ud654\ud558\ub294 \ubc29\ud5a5\uc73c\ub85c \uc5ec\ub860\uc744 \uc870\uc131\ud574 \ub098\uac00\uace0 \uc788\uc5c8\ub2e4."} {"question": "\uc815\ubbfc\ucca0\uc774 1\uc2b9\ub3c4 \uac70\ub450\uc9c0 \ubabb\ud55c \ud574\ub294 \uc5b8\uc81c\uc778\uac00?", "paragraph": "\uce5c\uc815 \ud300\uc73c\ub85c \ub3cc\uc544\uc640 2002\ub144 \uc2dc\uc98c\ubd80\ud130 \ub2e4\uc2dc KBO \ub9ac\uadf8\uc5d0 \ucc38\uac00\ud558\uc9c0\ub9cc, \uc804\uacfc \uac19\uc9c0 \uc54a\uc740 \uad6c\uc704\ub85c \ub09c\uc870\ub97c \uacaa\uc5c8\ub2e4. \ud2b9\ud788 2004\ub144\uc5d0\ub294 \ub2e8 1\uc2b9\ub3c4 \uac70\ub450\uc9c0 \ubabb\ud558\uace0, \uadf8\ub3d9\uc548 \uad34\ub86d\ud600 \uc654\uc5c8\ub358 \ud314\uafc8\uce58 \ud1b5\uc99d\uc73c\ub85c \uc778\ud574 \uc218\uc220\uc744 \ubc1b\uae30\ub3c4 \ud558\uc600\ub2e4. \uc774\ud6c4 \uc7ac\uae30\ub97c \ub2e4\uc9d0\ud558\uba74\uc11c 2005\ub144\uc5d0 \uc7ac\uae30\uc5d0 \uac00\ub2a5\uc131\uc744 \ubcf4\uc600\uace0, 2006\ub144\ubd80\ud130 \uae30\uad50\ud30c \ud22c\uc218\ub85c \uc7ac\uae30\uc5d0 \uc131\uacf5\ud558\uc5ec \ud300\uc758 \uc120\ubc1c \uc8fc\ucd95\uc73c\ub85c \ud65c\uc57d\ud588\ub2e4. 2007\ub144 12\uc2b9 5\ud328 \ud3c9\uade0\uc790\ucc45\uc810 2.90\uc73c\ub85c \uc644\ubcbd\ud558\uac8c \uc7ac\uae30\ud588\ub2e4. \uadf8\ub7ec\ub098 2008\ub144 \ub178\uc1e0\ud654\ub85c \uc778\ud574 \ub2e4\uc2dc \ubd80\uc9c4\ud558\uc600\ub2e4. 2009\ub144, \ud55c\ud654 \uc774\uae00\uc2a4\uc5d0\uc11c \ud50c\ub808\uc789 \ucf54\uce58\ub85c \ud65c\ub3d9\ud558\ub2e4\uac00 \ud604\uc5ed \uc740\ud1f4\ub97c \uc120\uc5b8\ud588\uc73c\uba70, 9\uc6d4 12\uc77c \uc740\ud1f4\uc2dd\uacfc \uc601\uad6c \uacb0\ubc88\uc2dd\uc744 \uac70\ud589\ud558\uc600\ub2e4. \uc190\uac00\ub77d \uae38\uc774\uac00 \uc77c\ubc18\uc778\ubcf4\ub2e4 \ud55c \ub9c8\ub514 \uc815\ub3c4 \uae34 \ud2b9\uc9d5\uc774 \uc788\uc73c\uba70, \ud6e4\uce60\ud55c \ubbf8\ub0a8\ud615\uc73c\ub85c \ub9ce\uc740 \uc5ec\uc131\ud32c\ub4e4\uc744 \ubab0\uace0 \ub2e4\ub154\ub2e4.", "answer": "2004\ub144", "sentence": "\ud2b9\ud788 2004\ub144 \uc5d0\ub294 \ub2e8 1\uc2b9\ub3c4 \uac70\ub450\uc9c0 \ubabb\ud558\uace0, \uadf8\ub3d9\uc548 \uad34\ub86d\ud600 \uc654\uc5c8\ub358 \ud314\uafc8\uce58 \ud1b5\uc99d\uc73c\ub85c \uc778\ud574 \uc218\uc220\uc744 \ubc1b\uae30\ub3c4 \ud558\uc600\ub2e4.", "paragraph_sentence": "\uce5c\uc815 \ud300\uc73c\ub85c \ub3cc\uc544\uc640 2002\ub144 \uc2dc\uc98c\ubd80\ud130 \ub2e4\uc2dc KBO \ub9ac\uadf8\uc5d0 \ucc38\uac00\ud558\uc9c0\ub9cc, \uc804\uacfc \uac19\uc9c0 \uc54a\uc740 \uad6c\uc704\ub85c \ub09c\uc870\ub97c \uacaa\uc5c8\ub2e4. \ud2b9\ud788 2004\ub144 \uc5d0\ub294 \ub2e8 1\uc2b9\ub3c4 \uac70\ub450\uc9c0 \ubabb\ud558\uace0, \uadf8\ub3d9\uc548 \uad34\ub86d\ud600 \uc654\uc5c8\ub358 \ud314\uafc8\uce58 \ud1b5\uc99d\uc73c\ub85c \uc778\ud574 \uc218\uc220\uc744 \ubc1b\uae30\ub3c4 \ud558\uc600\ub2e4. \uc774\ud6c4 \uc7ac\uae30\ub97c \ub2e4\uc9d0\ud558\uba74\uc11c 2005\ub144\uc5d0 \uc7ac\uae30\uc5d0 \uac00\ub2a5\uc131\uc744 \ubcf4\uc600\uace0, 2006\ub144\ubd80\ud130 \uae30\uad50\ud30c \ud22c\uc218\ub85c \uc7ac\uae30\uc5d0 \uc131\uacf5\ud558\uc5ec \ud300\uc758 \uc120\ubc1c \uc8fc\ucd95\uc73c\ub85c \ud65c\uc57d\ud588\ub2e4. 2007\ub144 12\uc2b9 5\ud328 \ud3c9\uade0\uc790\ucc45\uc810 2.90\uc73c\ub85c \uc644\ubcbd\ud558\uac8c \uc7ac\uae30\ud588\ub2e4. \uadf8\ub7ec\ub098 2008\ub144 \ub178\uc1e0\ud654\ub85c \uc778\ud574 \ub2e4\uc2dc \ubd80\uc9c4\ud558\uc600\ub2e4. 2009\ub144, \ud55c\ud654 \uc774\uae00\uc2a4\uc5d0\uc11c \ud50c\ub808\uc789 \ucf54\uce58\ub85c \ud65c\ub3d9\ud558\ub2e4\uac00 \ud604\uc5ed \uc740\ud1f4\ub97c \uc120\uc5b8\ud588\uc73c\uba70, 9\uc6d4 12\uc77c \uc740\ud1f4\uc2dd\uacfc \uc601\uad6c \uacb0\ubc88\uc2dd\uc744 \uac70\ud589\ud558\uc600\ub2e4. \uc190\uac00\ub77d \uae38\uc774\uac00 \uc77c\ubc18\uc778\ubcf4\ub2e4 \ud55c \ub9c8\ub514 \uc815\ub3c4 \uae34 \ud2b9\uc9d5\uc774 \uc788\uc73c\uba70, \ud6e4\uce60\ud55c \ubbf8\ub0a8\ud615\uc73c\ub85c \ub9ce\uc740 \uc5ec\uc131\ud32c\ub4e4\uc744 \ubab0\uace0 \ub2e4\ub154\ub2e4.", "paragraph_answer": "\uce5c\uc815 \ud300\uc73c\ub85c \ub3cc\uc544\uc640 2002\ub144 \uc2dc\uc98c\ubd80\ud130 \ub2e4\uc2dc KBO \ub9ac\uadf8\uc5d0 \ucc38\uac00\ud558\uc9c0\ub9cc, \uc804\uacfc \uac19\uc9c0 \uc54a\uc740 \uad6c\uc704\ub85c \ub09c\uc870\ub97c \uacaa\uc5c8\ub2e4. \ud2b9\ud788 2004\ub144 \uc5d0\ub294 \ub2e8 1\uc2b9\ub3c4 \uac70\ub450\uc9c0 \ubabb\ud558\uace0, \uadf8\ub3d9\uc548 \uad34\ub86d\ud600 \uc654\uc5c8\ub358 \ud314\uafc8\uce58 \ud1b5\uc99d\uc73c\ub85c \uc778\ud574 \uc218\uc220\uc744 \ubc1b\uae30\ub3c4 \ud558\uc600\ub2e4. \uc774\ud6c4 \uc7ac\uae30\ub97c \ub2e4\uc9d0\ud558\uba74\uc11c 2005\ub144\uc5d0 \uc7ac\uae30\uc5d0 \uac00\ub2a5\uc131\uc744 \ubcf4\uc600\uace0, 2006\ub144\ubd80\ud130 \uae30\uad50\ud30c \ud22c\uc218\ub85c \uc7ac\uae30\uc5d0 \uc131\uacf5\ud558\uc5ec \ud300\uc758 \uc120\ubc1c \uc8fc\ucd95\uc73c\ub85c \ud65c\uc57d\ud588\ub2e4. 2007\ub144 12\uc2b9 5\ud328 \ud3c9\uade0\uc790\ucc45\uc810 2.90\uc73c\ub85c \uc644\ubcbd\ud558\uac8c \uc7ac\uae30\ud588\ub2e4. \uadf8\ub7ec\ub098 2008\ub144 \ub178\uc1e0\ud654\ub85c \uc778\ud574 \ub2e4\uc2dc \ubd80\uc9c4\ud558\uc600\ub2e4. 2009\ub144, \ud55c\ud654 \uc774\uae00\uc2a4\uc5d0\uc11c \ud50c\ub808\uc789 \ucf54\uce58\ub85c \ud65c\ub3d9\ud558\ub2e4\uac00 \ud604\uc5ed \uc740\ud1f4\ub97c \uc120\uc5b8\ud588\uc73c\uba70, 9\uc6d4 12\uc77c \uc740\ud1f4\uc2dd\uacfc \uc601\uad6c \uacb0\ubc88\uc2dd\uc744 \uac70\ud589\ud558\uc600\ub2e4. \uc190\uac00\ub77d \uae38\uc774\uac00 \uc77c\ubc18\uc778\ubcf4\ub2e4 \ud55c \ub9c8\ub514 \uc815\ub3c4 \uae34 \ud2b9\uc9d5\uc774 \uc788\uc73c\uba70, \ud6e4\uce60\ud55c \ubbf8\ub0a8\ud615\uc73c\ub85c \ub9ce\uc740 \uc5ec\uc131\ud32c\ub4e4\uc744 \ubab0\uace0 \ub2e4\ub154\ub2e4.", "sentence_answer": "\ud2b9\ud788 2004\ub144 \uc5d0\ub294 \ub2e8 1\uc2b9\ub3c4 \uac70\ub450\uc9c0 \ubabb\ud558\uace0, \uadf8\ub3d9\uc548 \uad34\ub86d\ud600 \uc654\uc5c8\ub358 \ud314\uafc8\uce58 \ud1b5\uc99d\uc73c\ub85c \uc778\ud574 \uc218\uc220\uc744 \ubc1b\uae30\ub3c4 \ud558\uc600\ub2e4.", "paragraph_id": "6550102-3-0", "paragraph_question": "question: \uc815\ubbfc\ucca0\uc774 1\uc2b9\ub3c4 \uac70\ub450\uc9c0 \ubabb\ud55c \ud574\ub294 \uc5b8\uc81c\uc778\uac00?, context: \uce5c\uc815 \ud300\uc73c\ub85c \ub3cc\uc544\uc640 2002\ub144 \uc2dc\uc98c\ubd80\ud130 \ub2e4\uc2dc KBO \ub9ac\uadf8\uc5d0 \ucc38\uac00\ud558\uc9c0\ub9cc, \uc804\uacfc \uac19\uc9c0 \uc54a\uc740 \uad6c\uc704\ub85c \ub09c\uc870\ub97c \uacaa\uc5c8\ub2e4. \ud2b9\ud788 2004\ub144\uc5d0\ub294 \ub2e8 1\uc2b9\ub3c4 \uac70\ub450\uc9c0 \ubabb\ud558\uace0, \uadf8\ub3d9\uc548 \uad34\ub86d\ud600 \uc654\uc5c8\ub358 \ud314\uafc8\uce58 \ud1b5\uc99d\uc73c\ub85c \uc778\ud574 \uc218\uc220\uc744 \ubc1b\uae30\ub3c4 \ud558\uc600\ub2e4. \uc774\ud6c4 \uc7ac\uae30\ub97c \ub2e4\uc9d0\ud558\uba74\uc11c 2005\ub144\uc5d0 \uc7ac\uae30\uc5d0 \uac00\ub2a5\uc131\uc744 \ubcf4\uc600\uace0, 2006\ub144\ubd80\ud130 \uae30\uad50\ud30c \ud22c\uc218\ub85c \uc7ac\uae30\uc5d0 \uc131\uacf5\ud558\uc5ec \ud300\uc758 \uc120\ubc1c \uc8fc\ucd95\uc73c\ub85c \ud65c\uc57d\ud588\ub2e4. 2007\ub144 12\uc2b9 5\ud328 \ud3c9\uade0\uc790\ucc45\uc810 2.90\uc73c\ub85c \uc644\ubcbd\ud558\uac8c \uc7ac\uae30\ud588\ub2e4. \uadf8\ub7ec\ub098 2008\ub144 \ub178\uc1e0\ud654\ub85c \uc778\ud574 \ub2e4\uc2dc \ubd80\uc9c4\ud558\uc600\ub2e4. 2009\ub144, \ud55c\ud654 \uc774\uae00\uc2a4\uc5d0\uc11c \ud50c\ub808\uc789 \ucf54\uce58\ub85c \ud65c\ub3d9\ud558\ub2e4\uac00 \ud604\uc5ed \uc740\ud1f4\ub97c \uc120\uc5b8\ud588\uc73c\uba70, 9\uc6d4 12\uc77c \uc740\ud1f4\uc2dd\uacfc \uc601\uad6c \uacb0\ubc88\uc2dd\uc744 \uac70\ud589\ud558\uc600\ub2e4. \uc190\uac00\ub77d \uae38\uc774\uac00 \uc77c\ubc18\uc778\ubcf4\ub2e4 \ud55c \ub9c8\ub514 \uc815\ub3c4 \uae34 \ud2b9\uc9d5\uc774 \uc788\uc73c\uba70, \ud6e4\uce60\ud55c \ubbf8\ub0a8\ud615\uc73c\ub85c \ub9ce\uc740 \uc5ec\uc131\ud32c\ub4e4\uc744 \ubab0\uace0 \ub2e4\ub154\ub2e4."} {"question": "\ubb38\uc7ac\uc778 \ub300\ud1b5\ub839\uc774 \ud55c\ubbf8\uc815\uc0c1\ud68c\ub2f4\uc744 \uc704\ud574 \ubbf8\uad6d\uc744 \uacf5\uc2dd\uc2e4\ubb34\ubc29\ubb38\ud55c \ub0a0\uc740?", "paragraph": "6\uc6d4 29\uc77c \ubbf8\uad6d\uc744 \uacf5\uc2dd\uc2e4\ubb34\ubc29\ubb38\ud558\uba74\uc11c \uc2dc\uc791\ub41c 3\ubc15 5\uc77c \ub3d9\uc548\uc758 \ud55c\ubbf8\uc815\uc0c1\ud68c\ub2f4\uc740 \ube44\uad50\uc801 \uc131\uacfc\ub97c \uac70\ub480\ub2e4\ub294 \ud3c9\uac00\ub2e4. \uacf5\ub3d9\uc131\uba85\uc5d0\uc11c \uc0ac\ub4dc \ubb38\uc81c\ub97c \uc81c\uc678\ud558\ub294 \ub4f1 \ubbf8\uad6d\uc758 \uc774\ud574\ub97c \ud655\ubcf4\ud558\uba74\uc11c \uc911\uad6d\uacfc\ub294 \ub300\ud654\ub97c \ud560 \uc218 \uc788\ub294 \uc2dc\uac04\uc744 \ubc8c\uc5c8\uc73c\uba70, \ubd81\ud575 \ubb38\uc81c\ub294 \ub300\ud654\ub97c \ud1b5\ud574 \ud574\uacb0\ud560 \uac00\ub2a5\uc131\uc744 \ub0a8\uaca8\ub450\uba70 \ud55c\uad6d\uc758 \uc8fc\ub3c4\uc801 \uc5ed\ud560\uacfc \uc774\uc5d0 \ub300\ud55c \ud2b8\ub7fc\ud504 \ub300\ud1b5\ub839\uc758 \uc9c0\uc9c0\ub97c \uc774\ub04c\uc5b4\ub0c8\ub2e4. \ud1b5\uc0c1 \ubb38\uc81c\ub3c4 \uac70\ub860\ub418\uc5c8\ub294\ub370 \ubbf8\uad6d\uc740 \ud55c\ubbf8 FTA\uac00 \uacf5\uc815\ud558\uc9c0 \uc54a\uc740 \ud611\uc815\uc774\ub77c\uba70 \uc790\ub3d9\ucc28\uc640 \ucca0\uac15 \ubd84\uc57c\uc5d0\uc11c \uc801\uc790\uac00 \ub9ce\uc774 \ubc1c\uc0dd\ud558\uace0 \uc788\ub2e4\uace0 \uc8fc\uc7a5\ud588\uc73c\uba70, \ud55c\uad6d\uc740 \ubb34\uc5ed\uade0\ud615\uc774 \ub9de\uc544\uac00\uace0 \uc788\ub2e4\uba70 FTA \uaddc\uc815\uc774 \uc81c\ub300\ub85c \uc9c0\ucf1c\uc9c0\uc9c0 \uc54a\uace0 \uc788\ub294\uc9c0\ub97c \uc2a4\ud130\ub514\ud558\uace0 \uc591\uad6d \ubb34\uc5ed\uc5d0 \ubbf8\uce58\ub294 \uc601\ud5a5\uc744 \uacf5\ub3d9 \uc870\uc0ac\ubd84\uc11d\ud560 \uac83\uc744 \uc5ed\uc81c\uc548\ud588\ub2e4. \ub610 \uc774\uc640 \uad00\ub828\ud574\uc11c\ub294 \ud55c\uad6d\uc774 \ubbf8\uad6d\uc0b0 \uc804\ud22c\uae30\ub97c \uad6c\ub9e4\ud558\uba74\uc11c \ubbf8\uad6d\uc758 \ubb34\uc5ed\uc5ed\uc870 \ubb38\uc81c\ub97c \uc644\ud654\ud558\uae30\ub85c \ud569\uc758\ud588\ub2e4. \ud558\uc9c0\ub9cc \ud2b8\ub7fc\ud504 \ub300\ud1b5\ub839\uc774 \uc815\uc0c1\ud68c\ub2f4 \uc9c1\ud6c4\uc758 \uc5b8\ub860\ubc1c\ud45c\uc5d0\uc11c \"\ud55c\ubbf8FTA \uccb4\uacb0 \uc774\ud6c4 \ubbf8\uad6d \uc801\uc790\uac00 110\uc5b5 \ub2ec\ub7ec \uc774\uc0c1 \ub298\uc5c8\ub2e4\"\uace0 \uac15\uc870\ud558\uc600\uace0 \uacb0\uad6d 10\uc6d4\uc5d0\ub294 \uc7ac\ud611\uc0c1\uc5d0 \ub3cc\uc785\ud558\ub294 \uacc4\uae30\uac00 \ub418\uba74\uc11c \uacfc\uc81c\ub3c4 \uc548\uac8c \ub418\uc5c8\ub2e4. \uc8fc\ud55c \ubbf8\uad70\uacfc \uad00\ub828\ud558\uc5ec \ubc29\uc704\ube44 \ubd84\ub2f4\uae08 \ubb38\uc81c\ub3c4 \uc5b8\uae09\ub418\uc5c8\ub294\ub370 \ud2b8\ub7fc\ud504\uac00 \uc8fc\uc7a5\ud55c \ubb34\uc784\uc2b9\ucc28\ub860\uc740 \uc0ac\uc2e4\uc774 \uc544\ub2c8\ub77c\uba70 \"\ud55c\uad6d\uc740 \ubbf8\uad6d\ub9cc\ud07c\uc740 \uc544\ub2c8\uc9c0\ub9cc GDP \ub300\ube44 \uac00\uc7a5\ub192\uc740 \uad6d\ubc29\ube44\ub97c \uc9c0\ucd9c\ud558\ub294 \ub3d9\ub9f9\uad6d\"\uc784\uc744 \uac15\uc870\ud588\ub2e4.", "answer": "6\uc6d4 29\uc77c", "sentence": "6\uc6d4 29\uc77c \ubbf8\uad6d\uc744 \uacf5\uc2dd\uc2e4\ubb34\ubc29\ubb38\ud558\uba74\uc11c \uc2dc\uc791\ub41c 3\ubc15 5\uc77c \ub3d9\uc548\uc758 \ud55c\ubbf8\uc815\uc0c1\ud68c\ub2f4\uc740 \ube44\uad50\uc801 \uc131\uacfc\ub97c \uac70\ub480\ub2e4\ub294 \ud3c9\uac00\ub2e4.", "paragraph_sentence": " 6\uc6d4 29\uc77c \ubbf8\uad6d\uc744 \uacf5\uc2dd\uc2e4\ubb34\ubc29\ubb38\ud558\uba74\uc11c \uc2dc\uc791\ub41c 3\ubc15 5\uc77c \ub3d9\uc548\uc758 \ud55c\ubbf8\uc815\uc0c1\ud68c\ub2f4\uc740 \ube44\uad50\uc801 \uc131\uacfc\ub97c \uac70\ub480\ub2e4\ub294 \ud3c9\uac00\ub2e4. \uacf5\ub3d9\uc131\uba85\uc5d0\uc11c \uc0ac\ub4dc \ubb38\uc81c\ub97c \uc81c\uc678\ud558\ub294 \ub4f1 \ubbf8\uad6d\uc758 \uc774\ud574\ub97c \ud655\ubcf4\ud558\uba74\uc11c \uc911\uad6d\uacfc\ub294 \ub300\ud654\ub97c \ud560 \uc218 \uc788\ub294 \uc2dc\uac04\uc744 \ubc8c\uc5c8\uc73c\uba70, \ubd81\ud575 \ubb38\uc81c\ub294 \ub300\ud654\ub97c \ud1b5\ud574 \ud574\uacb0\ud560 \uac00\ub2a5\uc131\uc744 \ub0a8\uaca8\ub450\uba70 \ud55c\uad6d\uc758 \uc8fc\ub3c4\uc801 \uc5ed\ud560\uacfc \uc774\uc5d0 \ub300\ud55c \ud2b8\ub7fc\ud504 \ub300\ud1b5\ub839\uc758 \uc9c0\uc9c0\ub97c \uc774\ub04c\uc5b4\ub0c8\ub2e4. \ud1b5\uc0c1 \ubb38\uc81c\ub3c4 \uac70\ub860\ub418\uc5c8\ub294\ub370 \ubbf8\uad6d\uc740 \ud55c\ubbf8 FTA\uac00 \uacf5\uc815\ud558\uc9c0 \uc54a\uc740 \ud611\uc815\uc774\ub77c\uba70 \uc790\ub3d9\ucc28\uc640 \ucca0\uac15 \ubd84\uc57c\uc5d0\uc11c \uc801\uc790\uac00 \ub9ce\uc774 \ubc1c\uc0dd\ud558\uace0 \uc788\ub2e4\uace0 \uc8fc\uc7a5\ud588\uc73c\uba70, \ud55c\uad6d\uc740 \ubb34\uc5ed\uade0\ud615\uc774 \ub9de\uc544\uac00\uace0 \uc788\ub2e4\uba70 FTA \uaddc\uc815\uc774 \uc81c\ub300\ub85c \uc9c0\ucf1c\uc9c0\uc9c0 \uc54a\uace0 \uc788\ub294\uc9c0\ub97c \uc2a4\ud130\ub514\ud558\uace0 \uc591\uad6d \ubb34\uc5ed\uc5d0 \ubbf8\uce58\ub294 \uc601\ud5a5\uc744 \uacf5\ub3d9 \uc870\uc0ac\ubd84\uc11d\ud560 \uac83\uc744 \uc5ed\uc81c\uc548\ud588\ub2e4. \ub610 \uc774\uc640 \uad00\ub828\ud574\uc11c\ub294 \ud55c\uad6d\uc774 \ubbf8\uad6d\uc0b0 \uc804\ud22c\uae30\ub97c \uad6c\ub9e4\ud558\uba74\uc11c \ubbf8\uad6d\uc758 \ubb34\uc5ed\uc5ed\uc870 \ubb38\uc81c\ub97c \uc644\ud654\ud558\uae30\ub85c \ud569\uc758\ud588\ub2e4. \ud558\uc9c0\ub9cc \ud2b8\ub7fc\ud504 \ub300\ud1b5\ub839\uc774 \uc815\uc0c1\ud68c\ub2f4 \uc9c1\ud6c4\uc758 \uc5b8\ub860\ubc1c\ud45c\uc5d0\uc11c \"\ud55c\ubbf8FTA \uccb4\uacb0 \uc774\ud6c4 \ubbf8\uad6d \uc801\uc790\uac00 110\uc5b5 \ub2ec\ub7ec \uc774\uc0c1 \ub298\uc5c8\ub2e4\"\uace0 \uac15\uc870\ud558\uc600\uace0 \uacb0\uad6d 10\uc6d4\uc5d0\ub294 \uc7ac\ud611\uc0c1\uc5d0 \ub3cc\uc785\ud558\ub294 \uacc4\uae30\uac00 \ub418\uba74\uc11c \uacfc\uc81c\ub3c4 \uc548\uac8c \ub418\uc5c8\ub2e4. \uc8fc\ud55c \ubbf8\uad70\uacfc \uad00\ub828\ud558\uc5ec \ubc29\uc704\ube44 \ubd84\ub2f4\uae08 \ubb38\uc81c\ub3c4 \uc5b8\uae09\ub418\uc5c8\ub294\ub370 \ud2b8\ub7fc\ud504\uac00 \uc8fc\uc7a5\ud55c \ubb34\uc784\uc2b9\ucc28\ub860\uc740 \uc0ac\uc2e4\uc774 \uc544\ub2c8\ub77c\uba70 \"\ud55c\uad6d\uc740 \ubbf8\uad6d\ub9cc\ud07c\uc740 \uc544\ub2c8\uc9c0\ub9cc GDP \ub300\ube44 \uac00\uc7a5\ub192\uc740 \uad6d\ubc29\ube44\ub97c \uc9c0\ucd9c\ud558\ub294 \ub3d9\ub9f9\uad6d\"\uc784\uc744 \uac15\uc870\ud588\ub2e4.", "paragraph_answer": " 6\uc6d4 29\uc77c \ubbf8\uad6d\uc744 \uacf5\uc2dd\uc2e4\ubb34\ubc29\ubb38\ud558\uba74\uc11c \uc2dc\uc791\ub41c 3\ubc15 5\uc77c \ub3d9\uc548\uc758 \ud55c\ubbf8\uc815\uc0c1\ud68c\ub2f4\uc740 \ube44\uad50\uc801 \uc131\uacfc\ub97c \uac70\ub480\ub2e4\ub294 \ud3c9\uac00\ub2e4. \uacf5\ub3d9\uc131\uba85\uc5d0\uc11c \uc0ac\ub4dc \ubb38\uc81c\ub97c \uc81c\uc678\ud558\ub294 \ub4f1 \ubbf8\uad6d\uc758 \uc774\ud574\ub97c \ud655\ubcf4\ud558\uba74\uc11c \uc911\uad6d\uacfc\ub294 \ub300\ud654\ub97c \ud560 \uc218 \uc788\ub294 \uc2dc\uac04\uc744 \ubc8c\uc5c8\uc73c\uba70, \ubd81\ud575 \ubb38\uc81c\ub294 \ub300\ud654\ub97c \ud1b5\ud574 \ud574\uacb0\ud560 \uac00\ub2a5\uc131\uc744 \ub0a8\uaca8\ub450\uba70 \ud55c\uad6d\uc758 \uc8fc\ub3c4\uc801 \uc5ed\ud560\uacfc \uc774\uc5d0 \ub300\ud55c \ud2b8\ub7fc\ud504 \ub300\ud1b5\ub839\uc758 \uc9c0\uc9c0\ub97c \uc774\ub04c\uc5b4\ub0c8\ub2e4. \ud1b5\uc0c1 \ubb38\uc81c\ub3c4 \uac70\ub860\ub418\uc5c8\ub294\ub370 \ubbf8\uad6d\uc740 \ud55c\ubbf8 FTA\uac00 \uacf5\uc815\ud558\uc9c0 \uc54a\uc740 \ud611\uc815\uc774\ub77c\uba70 \uc790\ub3d9\ucc28\uc640 \ucca0\uac15 \ubd84\uc57c\uc5d0\uc11c \uc801\uc790\uac00 \ub9ce\uc774 \ubc1c\uc0dd\ud558\uace0 \uc788\ub2e4\uace0 \uc8fc\uc7a5\ud588\uc73c\uba70, \ud55c\uad6d\uc740 \ubb34\uc5ed\uade0\ud615\uc774 \ub9de\uc544\uac00\uace0 \uc788\ub2e4\uba70 FTA \uaddc\uc815\uc774 \uc81c\ub300\ub85c \uc9c0\ucf1c\uc9c0\uc9c0 \uc54a\uace0 \uc788\ub294\uc9c0\ub97c \uc2a4\ud130\ub514\ud558\uace0 \uc591\uad6d \ubb34\uc5ed\uc5d0 \ubbf8\uce58\ub294 \uc601\ud5a5\uc744 \uacf5\ub3d9 \uc870\uc0ac\ubd84\uc11d\ud560 \uac83\uc744 \uc5ed\uc81c\uc548\ud588\ub2e4. \ub610 \uc774\uc640 \uad00\ub828\ud574\uc11c\ub294 \ud55c\uad6d\uc774 \ubbf8\uad6d\uc0b0 \uc804\ud22c\uae30\ub97c \uad6c\ub9e4\ud558\uba74\uc11c \ubbf8\uad6d\uc758 \ubb34\uc5ed\uc5ed\uc870 \ubb38\uc81c\ub97c \uc644\ud654\ud558\uae30\ub85c \ud569\uc758\ud588\ub2e4. \ud558\uc9c0\ub9cc \ud2b8\ub7fc\ud504 \ub300\ud1b5\ub839\uc774 \uc815\uc0c1\ud68c\ub2f4 \uc9c1\ud6c4\uc758 \uc5b8\ub860\ubc1c\ud45c\uc5d0\uc11c \"\ud55c\ubbf8FTA \uccb4\uacb0 \uc774\ud6c4 \ubbf8\uad6d \uc801\uc790\uac00 110\uc5b5 \ub2ec\ub7ec \uc774\uc0c1 \ub298\uc5c8\ub2e4\"\uace0 \uac15\uc870\ud558\uc600\uace0 \uacb0\uad6d 10\uc6d4\uc5d0\ub294 \uc7ac\ud611\uc0c1\uc5d0 \ub3cc\uc785\ud558\ub294 \uacc4\uae30\uac00 \ub418\uba74\uc11c \uacfc\uc81c\ub3c4 \uc548\uac8c \ub418\uc5c8\ub2e4. \uc8fc\ud55c \ubbf8\uad70\uacfc \uad00\ub828\ud558\uc5ec \ubc29\uc704\ube44 \ubd84\ub2f4\uae08 \ubb38\uc81c\ub3c4 \uc5b8\uae09\ub418\uc5c8\ub294\ub370 \ud2b8\ub7fc\ud504\uac00 \uc8fc\uc7a5\ud55c \ubb34\uc784\uc2b9\ucc28\ub860\uc740 \uc0ac\uc2e4\uc774 \uc544\ub2c8\ub77c\uba70 \"\ud55c\uad6d\uc740 \ubbf8\uad6d\ub9cc\ud07c\uc740 \uc544\ub2c8\uc9c0\ub9cc GDP \ub300\ube44 \uac00\uc7a5\ub192\uc740 \uad6d\ubc29\ube44\ub97c \uc9c0\ucd9c\ud558\ub294 \ub3d9\ub9f9\uad6d\"\uc784\uc744 \uac15\uc870\ud588\ub2e4.", "sentence_answer": " 6\uc6d4 29\uc77c \ubbf8\uad6d\uc744 \uacf5\uc2dd\uc2e4\ubb34\ubc29\ubb38\ud558\uba74\uc11c \uc2dc\uc791\ub41c 3\ubc15 5\uc77c \ub3d9\uc548\uc758 \ud55c\ubbf8\uc815\uc0c1\ud68c\ub2f4\uc740 \ube44\uad50\uc801 \uc131\uacfc\ub97c \uac70\ub480\ub2e4\ub294 \ud3c9\uac00\ub2e4.", "paragraph_id": "6580792-5-0", "paragraph_question": "question: \ubb38\uc7ac\uc778 \ub300\ud1b5\ub839\uc774 \ud55c\ubbf8\uc815\uc0c1\ud68c\ub2f4\uc744 \uc704\ud574 \ubbf8\uad6d\uc744 \uacf5\uc2dd\uc2e4\ubb34\ubc29\ubb38\ud55c \ub0a0\uc740?, context: 6\uc6d4 29\uc77c \ubbf8\uad6d\uc744 \uacf5\uc2dd\uc2e4\ubb34\ubc29\ubb38\ud558\uba74\uc11c \uc2dc\uc791\ub41c 3\ubc15 5\uc77c \ub3d9\uc548\uc758 \ud55c\ubbf8\uc815\uc0c1\ud68c\ub2f4\uc740 \ube44\uad50\uc801 \uc131\uacfc\ub97c \uac70\ub480\ub2e4\ub294 \ud3c9\uac00\ub2e4. \uacf5\ub3d9\uc131\uba85\uc5d0\uc11c \uc0ac\ub4dc \ubb38\uc81c\ub97c \uc81c\uc678\ud558\ub294 \ub4f1 \ubbf8\uad6d\uc758 \uc774\ud574\ub97c \ud655\ubcf4\ud558\uba74\uc11c \uc911\uad6d\uacfc\ub294 \ub300\ud654\ub97c \ud560 \uc218 \uc788\ub294 \uc2dc\uac04\uc744 \ubc8c\uc5c8\uc73c\uba70, \ubd81\ud575 \ubb38\uc81c\ub294 \ub300\ud654\ub97c \ud1b5\ud574 \ud574\uacb0\ud560 \uac00\ub2a5\uc131\uc744 \ub0a8\uaca8\ub450\uba70 \ud55c\uad6d\uc758 \uc8fc\ub3c4\uc801 \uc5ed\ud560\uacfc \uc774\uc5d0 \ub300\ud55c \ud2b8\ub7fc\ud504 \ub300\ud1b5\ub839\uc758 \uc9c0\uc9c0\ub97c \uc774\ub04c\uc5b4\ub0c8\ub2e4. \ud1b5\uc0c1 \ubb38\uc81c\ub3c4 \uac70\ub860\ub418\uc5c8\ub294\ub370 \ubbf8\uad6d\uc740 \ud55c\ubbf8 FTA\uac00 \uacf5\uc815\ud558\uc9c0 \uc54a\uc740 \ud611\uc815\uc774\ub77c\uba70 \uc790\ub3d9\ucc28\uc640 \ucca0\uac15 \ubd84\uc57c\uc5d0\uc11c \uc801\uc790\uac00 \ub9ce\uc774 \ubc1c\uc0dd\ud558\uace0 \uc788\ub2e4\uace0 \uc8fc\uc7a5\ud588\uc73c\uba70, \ud55c\uad6d\uc740 \ubb34\uc5ed\uade0\ud615\uc774 \ub9de\uc544\uac00\uace0 \uc788\ub2e4\uba70 FTA \uaddc\uc815\uc774 \uc81c\ub300\ub85c \uc9c0\ucf1c\uc9c0\uc9c0 \uc54a\uace0 \uc788\ub294\uc9c0\ub97c \uc2a4\ud130\ub514\ud558\uace0 \uc591\uad6d \ubb34\uc5ed\uc5d0 \ubbf8\uce58\ub294 \uc601\ud5a5\uc744 \uacf5\ub3d9 \uc870\uc0ac\ubd84\uc11d\ud560 \uac83\uc744 \uc5ed\uc81c\uc548\ud588\ub2e4. \ub610 \uc774\uc640 \uad00\ub828\ud574\uc11c\ub294 \ud55c\uad6d\uc774 \ubbf8\uad6d\uc0b0 \uc804\ud22c\uae30\ub97c \uad6c\ub9e4\ud558\uba74\uc11c \ubbf8\uad6d\uc758 \ubb34\uc5ed\uc5ed\uc870 \ubb38\uc81c\ub97c \uc644\ud654\ud558\uae30\ub85c \ud569\uc758\ud588\ub2e4. \ud558\uc9c0\ub9cc \ud2b8\ub7fc\ud504 \ub300\ud1b5\ub839\uc774 \uc815\uc0c1\ud68c\ub2f4 \uc9c1\ud6c4\uc758 \uc5b8\ub860\ubc1c\ud45c\uc5d0\uc11c \"\ud55c\ubbf8FTA \uccb4\uacb0 \uc774\ud6c4 \ubbf8\uad6d \uc801\uc790\uac00 110\uc5b5 \ub2ec\ub7ec \uc774\uc0c1 \ub298\uc5c8\ub2e4\"\uace0 \uac15\uc870\ud558\uc600\uace0 \uacb0\uad6d 10\uc6d4\uc5d0\ub294 \uc7ac\ud611\uc0c1\uc5d0 \ub3cc\uc785\ud558\ub294 \uacc4\uae30\uac00 \ub418\uba74\uc11c \uacfc\uc81c\ub3c4 \uc548\uac8c \ub418\uc5c8\ub2e4. \uc8fc\ud55c \ubbf8\uad70\uacfc \uad00\ub828\ud558\uc5ec \ubc29\uc704\ube44 \ubd84\ub2f4\uae08 \ubb38\uc81c\ub3c4 \uc5b8\uae09\ub418\uc5c8\ub294\ub370 \ud2b8\ub7fc\ud504\uac00 \uc8fc\uc7a5\ud55c \ubb34\uc784\uc2b9\ucc28\ub860\uc740 \uc0ac\uc2e4\uc774 \uc544\ub2c8\ub77c\uba70 \"\ud55c\uad6d\uc740 \ubbf8\uad6d\ub9cc\ud07c\uc740 \uc544\ub2c8\uc9c0\ub9cc GDP \ub300\ube44 \uac00\uc7a5\ub192\uc740 \uad6d\ubc29\ube44\ub97c \uc9c0\ucd9c\ud558\ub294 \ub3d9\ub9f9\uad6d\"\uc784\uc744 \uac15\uc870\ud588\ub2e4."} {"question": "\uc7ad \ucff0\uc2a4\uac00 \ud544\ub77c\ub378\ud53c\uc544 \ud544\ub9ac\uc2a4\uc758 \uac10\ub3c5\uc73c\ub85c \uc120\uc784\ub41c \ub54c\ub294?", "paragraph": "1918\ub144 12\uc6d4 17\uc77c, \uc7ad \ucff0\uc2a4\ub294 \ud544\ub77c\ub378\ud53c\uc544 \ud544\ub9ac\uc2a4\uc758 \uc0c8\ub85c\uc6b4 \uac10\ub3c5\uc73c\ub85c \uc120\uc784\ub418\uc5c8\ub2e4. \uc7ad \ucff0\uc2a4\uac00 \uc774\ub044\ub294 1919\ub144\uc758 \ud544\ub9ac\uc2a4\ub294 \uc2dc\uc98c \uc911\ubc18\uae4c\uc9c0 18\uc2b9 44\ud328\ub97c \uac70\ub450\uba70, 3\ud560\uc5d0\ub3c4 \ubbf8\uce58\uc9c0 \ubabb\ud558\ub294 \uc2b9\ub960\uc744 \uae30\ub85d\ud588\ub2e4. \uacb0\uad6d 7\uc6d4 9\uc77c \uac10\ub3c5\uc9c1\uc5d0\uc11c \uc0ac\uc784\ud588\ub2e4. \uc0ac\uc784 \ub2f9\uc2dc \uadf8\ub294 \"\ub098\ub294 \ub0b4\uac00 \ud560 \uc218 \uc788\ub294 \ud55c\uc5d0\uc11c \ubaa8\ub4e0 \uac83\uc744 \ud558\ub824\uace0 \ub178\ub825\ud588\ub2e4. \u2026 \ud300\uc744 \ub354\uc6b1 \uac15\ud558\uac8c \ub9cc\ub4e4\uae30 \uc704\ud574 \uba87 \ucc28\ub840\uc758 \uc2dc\ub3c4\ub97c \ud574\ubd24\uc9c0\ub9cc \ud5c8\uc0ac\uc600\ub2e4. \uc774 \ubaa8\ub4e0 \uac83\uc774 \ub0b4 \uc798\ubabb\uc774\ub77c\uace0\ub294 \uc0dd\uac01\ud558\uc9c0 \uc54a\ub294\ub2e4.\"\ub77c\uace0 \uc774\uc57c\uae30\ud558\uba70 \uc544\uc26c\uc6c0\uc744 \ub098\ud0c0\ub0c8\ub2e4. \ud558\uc9c0\ub9cc \uadf8\ud574 12\uc6d4, \uadf8\ub294 \ub2e4\uc2dc \ub514\ud2b8\ub85c\uc774\ud2b8 \ud0c0\uc774\uac70\uc2a4\uc758 \ubd80\ub984\uc744 \ubc1b\uace0 \ud22c\uc218 \ucf54\uce58\ub85c \ubd80\uc784\ud588\ub2e4. 1920\ub144 6\uc6d4 30\uc77c, \ud0c0\uc774\uac70\uc2a4\uc640 \ud654\uc774\ud2b8\uc0ad\uc2a4\uc758 \uacbd\uae30\uc5d0\uc11c 4\ud68c\uae4c\uc9c0 \uc810\uc218\uac00 0-12\ub85c \uc0c1\ub300\ud3b8\uc5d0\uac8c \uae30\uc6b8\uc790 \uc7ad \ucff0\uc2a4\ub294 5\ud68c \ud328\uc804 \ucc98\ub9ac\ub85c \ub4f1\ud310\ud588\ub2e4. 21\uac1c\uc6d4\ub9cc\uc758 \ub4f1\ud310\uc784\uc5d0\ub3c4 \ubd88\uad6c\ud558\uace0, 5\uc774\ub2dd \ub3d9\uc548 5\ud53c\uc548\ud0c0 2\uc2e4\uc810\uc73c\ub85c \ub9c9\uc558\ub2e4. 7\uc6d4 18\uc77c, \uc138\ub108\ud130\uc2a4\uc640\uc758 \uacbd\uae30\uc5d0\uc11c \ub2e4\uc2dc \ud328\uc804 \ucc98\ub9ac\ub85c \ub4f1\ud310\ud574 \u20443\uc774\ub2dd \ub3d9\uc548 2\ud53c\uc548\ud0c0 1\ubcfc\ub137 3\uc2e4\uc810\ud588\ub2e4. \uc774 \uacbd\uae30\uac00 \uc7ad \ucff0\uc2a4\uc758 \uba54\uc774\uc800 \ub9ac\uadf8 \ub9c8\uc9c0\ub9c9 \uacbd\uae30\uac00 \ub418\uc5c8\ub2e4.", "answer": "1918\ub144 12\uc6d4 17\uc77c", "sentence": "1918\ub144 12\uc6d4 17\uc77c ,", "paragraph_sentence": " 1918\ub144 12\uc6d4 17\uc77c , \uc7ad \ucff0\uc2a4\ub294 \ud544\ub77c\ub378\ud53c\uc544 \ud544\ub9ac\uc2a4\uc758 \uc0c8\ub85c\uc6b4 \uac10\ub3c5\uc73c\ub85c \uc120\uc784\ub418\uc5c8\ub2e4. \uc7ad \ucff0\uc2a4\uac00 \uc774\ub044\ub294 1919\ub144\uc758 \ud544\ub9ac\uc2a4\ub294 \uc2dc\uc98c \uc911\ubc18\uae4c\uc9c0 18\uc2b9 44\ud328\ub97c \uac70\ub450\uba70, 3\ud560\uc5d0\ub3c4 \ubbf8\uce58\uc9c0 \ubabb\ud558\ub294 \uc2b9\ub960\uc744 \uae30\ub85d\ud588\ub2e4. \uacb0\uad6d 7\uc6d4 9\uc77c \uac10\ub3c5\uc9c1\uc5d0\uc11c \uc0ac\uc784\ud588\ub2e4. \uc0ac\uc784 \ub2f9\uc2dc \uadf8\ub294 \"\ub098\ub294 \ub0b4\uac00 \ud560 \uc218 \uc788\ub294 \ud55c\uc5d0\uc11c \ubaa8\ub4e0 \uac83\uc744 \ud558\ub824\uace0 \ub178\ub825\ud588\ub2e4. \u2026 \ud300\uc744 \ub354\uc6b1 \uac15\ud558\uac8c \ub9cc\ub4e4\uae30 \uc704\ud574 \uba87 \ucc28\ub840\uc758 \uc2dc\ub3c4\ub97c \ud574\ubd24\uc9c0\ub9cc \ud5c8\uc0ac\uc600\ub2e4. \uc774 \ubaa8\ub4e0 \uac83\uc774 \ub0b4 \uc798\ubabb\uc774\ub77c\uace0\ub294 \uc0dd\uac01\ud558\uc9c0 \uc54a\ub294\ub2e4. \"\ub77c\uace0 \uc774\uc57c\uae30\ud558\uba70 \uc544\uc26c\uc6c0\uc744 \ub098\ud0c0\ub0c8\ub2e4. \ud558\uc9c0\ub9cc \uadf8\ud574 12\uc6d4, \uadf8\ub294 \ub2e4\uc2dc \ub514\ud2b8\ub85c\uc774\ud2b8 \ud0c0\uc774\uac70\uc2a4\uc758 \ubd80\ub984\uc744 \ubc1b\uace0 \ud22c\uc218 \ucf54\uce58\ub85c \ubd80\uc784\ud588\ub2e4. 1920\ub144 6\uc6d4 30\uc77c, \ud0c0\uc774\uac70\uc2a4\uc640 \ud654\uc774\ud2b8\uc0ad\uc2a4\uc758 \uacbd\uae30\uc5d0\uc11c 4\ud68c\uae4c\uc9c0 \uc810\uc218\uac00 0-12\ub85c \uc0c1\ub300\ud3b8\uc5d0\uac8c \uae30\uc6b8\uc790 \uc7ad \ucff0\uc2a4\ub294 5\ud68c \ud328\uc804 \ucc98\ub9ac\ub85c \ub4f1\ud310\ud588\ub2e4. 21\uac1c\uc6d4\ub9cc\uc758 \ub4f1\ud310\uc784\uc5d0\ub3c4 \ubd88\uad6c\ud558\uace0, 5\uc774\ub2dd \ub3d9\uc548 5\ud53c\uc548\ud0c0 2\uc2e4\uc810\uc73c\ub85c \ub9c9\uc558\ub2e4. 7\uc6d4 18\uc77c, \uc138\ub108\ud130\uc2a4\uc640\uc758 \uacbd\uae30\uc5d0\uc11c \ub2e4\uc2dc \ud328\uc804 \ucc98\ub9ac\ub85c \ub4f1\ud310\ud574 \u20443\uc774\ub2dd \ub3d9\uc548 2\ud53c\uc548\ud0c0 1\ubcfc\ub137 3\uc2e4\uc810\ud588\ub2e4. \uc774 \uacbd\uae30\uac00 \uc7ad \ucff0\uc2a4\uc758 \uba54\uc774\uc800 \ub9ac\uadf8 \ub9c8\uc9c0\ub9c9 \uacbd\uae30\uac00 \ub418\uc5c8\ub2e4.", "paragraph_answer": " 1918\ub144 12\uc6d4 17\uc77c , \uc7ad \ucff0\uc2a4\ub294 \ud544\ub77c\ub378\ud53c\uc544 \ud544\ub9ac\uc2a4\uc758 \uc0c8\ub85c\uc6b4 \uac10\ub3c5\uc73c\ub85c \uc120\uc784\ub418\uc5c8\ub2e4. \uc7ad \ucff0\uc2a4\uac00 \uc774\ub044\ub294 1919\ub144\uc758 \ud544\ub9ac\uc2a4\ub294 \uc2dc\uc98c \uc911\ubc18\uae4c\uc9c0 18\uc2b9 44\ud328\ub97c \uac70\ub450\uba70, 3\ud560\uc5d0\ub3c4 \ubbf8\uce58\uc9c0 \ubabb\ud558\ub294 \uc2b9\ub960\uc744 \uae30\ub85d\ud588\ub2e4. \uacb0\uad6d 7\uc6d4 9\uc77c \uac10\ub3c5\uc9c1\uc5d0\uc11c \uc0ac\uc784\ud588\ub2e4. \uc0ac\uc784 \ub2f9\uc2dc \uadf8\ub294 \"\ub098\ub294 \ub0b4\uac00 \ud560 \uc218 \uc788\ub294 \ud55c\uc5d0\uc11c \ubaa8\ub4e0 \uac83\uc744 \ud558\ub824\uace0 \ub178\ub825\ud588\ub2e4. \u2026 \ud300\uc744 \ub354\uc6b1 \uac15\ud558\uac8c \ub9cc\ub4e4\uae30 \uc704\ud574 \uba87 \ucc28\ub840\uc758 \uc2dc\ub3c4\ub97c \ud574\ubd24\uc9c0\ub9cc \ud5c8\uc0ac\uc600\ub2e4. \uc774 \ubaa8\ub4e0 \uac83\uc774 \ub0b4 \uc798\ubabb\uc774\ub77c\uace0\ub294 \uc0dd\uac01\ud558\uc9c0 \uc54a\ub294\ub2e4.\"\ub77c\uace0 \uc774\uc57c\uae30\ud558\uba70 \uc544\uc26c\uc6c0\uc744 \ub098\ud0c0\ub0c8\ub2e4. \ud558\uc9c0\ub9cc \uadf8\ud574 12\uc6d4, \uadf8\ub294 \ub2e4\uc2dc \ub514\ud2b8\ub85c\uc774\ud2b8 \ud0c0\uc774\uac70\uc2a4\uc758 \ubd80\ub984\uc744 \ubc1b\uace0 \ud22c\uc218 \ucf54\uce58\ub85c \ubd80\uc784\ud588\ub2e4. 1920\ub144 6\uc6d4 30\uc77c, \ud0c0\uc774\uac70\uc2a4\uc640 \ud654\uc774\ud2b8\uc0ad\uc2a4\uc758 \uacbd\uae30\uc5d0\uc11c 4\ud68c\uae4c\uc9c0 \uc810\uc218\uac00 0-12\ub85c \uc0c1\ub300\ud3b8\uc5d0\uac8c \uae30\uc6b8\uc790 \uc7ad \ucff0\uc2a4\ub294 5\ud68c \ud328\uc804 \ucc98\ub9ac\ub85c \ub4f1\ud310\ud588\ub2e4. 21\uac1c\uc6d4\ub9cc\uc758 \ub4f1\ud310\uc784\uc5d0\ub3c4 \ubd88\uad6c\ud558\uace0, 5\uc774\ub2dd \ub3d9\uc548 5\ud53c\uc548\ud0c0 2\uc2e4\uc810\uc73c\ub85c \ub9c9\uc558\ub2e4. 7\uc6d4 18\uc77c, \uc138\ub108\ud130\uc2a4\uc640\uc758 \uacbd\uae30\uc5d0\uc11c \ub2e4\uc2dc \ud328\uc804 \ucc98\ub9ac\ub85c \ub4f1\ud310\ud574 \u20443\uc774\ub2dd \ub3d9\uc548 2\ud53c\uc548\ud0c0 1\ubcfc\ub137 3\uc2e4\uc810\ud588\ub2e4. \uc774 \uacbd\uae30\uac00 \uc7ad \ucff0\uc2a4\uc758 \uba54\uc774\uc800 \ub9ac\uadf8 \ub9c8\uc9c0\ub9c9 \uacbd\uae30\uac00 \ub418\uc5c8\ub2e4.", "sentence_answer": " 1918\ub144 12\uc6d4 17\uc77c ,", "paragraph_id": "6608933-13-0", "paragraph_question": "question: \uc7ad \ucff0\uc2a4\uac00 \ud544\ub77c\ub378\ud53c\uc544 \ud544\ub9ac\uc2a4\uc758 \uac10\ub3c5\uc73c\ub85c \uc120\uc784\ub41c \ub54c\ub294?, context: 1918\ub144 12\uc6d4 17\uc77c, \uc7ad \ucff0\uc2a4\ub294 \ud544\ub77c\ub378\ud53c\uc544 \ud544\ub9ac\uc2a4\uc758 \uc0c8\ub85c\uc6b4 \uac10\ub3c5\uc73c\ub85c \uc120\uc784\ub418\uc5c8\ub2e4. \uc7ad \ucff0\uc2a4\uac00 \uc774\ub044\ub294 1919\ub144\uc758 \ud544\ub9ac\uc2a4\ub294 \uc2dc\uc98c \uc911\ubc18\uae4c\uc9c0 18\uc2b9 44\ud328\ub97c \uac70\ub450\uba70, 3\ud560\uc5d0\ub3c4 \ubbf8\uce58\uc9c0 \ubabb\ud558\ub294 \uc2b9\ub960\uc744 \uae30\ub85d\ud588\ub2e4. \uacb0\uad6d 7\uc6d4 9\uc77c \uac10\ub3c5\uc9c1\uc5d0\uc11c \uc0ac\uc784\ud588\ub2e4. \uc0ac\uc784 \ub2f9\uc2dc \uadf8\ub294 \"\ub098\ub294 \ub0b4\uac00 \ud560 \uc218 \uc788\ub294 \ud55c\uc5d0\uc11c \ubaa8\ub4e0 \uac83\uc744 \ud558\ub824\uace0 \ub178\ub825\ud588\ub2e4. \u2026 \ud300\uc744 \ub354\uc6b1 \uac15\ud558\uac8c \ub9cc\ub4e4\uae30 \uc704\ud574 \uba87 \ucc28\ub840\uc758 \uc2dc\ub3c4\ub97c \ud574\ubd24\uc9c0\ub9cc \ud5c8\uc0ac\uc600\ub2e4. \uc774 \ubaa8\ub4e0 \uac83\uc774 \ub0b4 \uc798\ubabb\uc774\ub77c\uace0\ub294 \uc0dd\uac01\ud558\uc9c0 \uc54a\ub294\ub2e4.\"\ub77c\uace0 \uc774\uc57c\uae30\ud558\uba70 \uc544\uc26c\uc6c0\uc744 \ub098\ud0c0\ub0c8\ub2e4. \ud558\uc9c0\ub9cc \uadf8\ud574 12\uc6d4, \uadf8\ub294 \ub2e4\uc2dc \ub514\ud2b8\ub85c\uc774\ud2b8 \ud0c0\uc774\uac70\uc2a4\uc758 \ubd80\ub984\uc744 \ubc1b\uace0 \ud22c\uc218 \ucf54\uce58\ub85c \ubd80\uc784\ud588\ub2e4. 1920\ub144 6\uc6d4 30\uc77c, \ud0c0\uc774\uac70\uc2a4\uc640 \ud654\uc774\ud2b8\uc0ad\uc2a4\uc758 \uacbd\uae30\uc5d0\uc11c 4\ud68c\uae4c\uc9c0 \uc810\uc218\uac00 0-12\ub85c \uc0c1\ub300\ud3b8\uc5d0\uac8c \uae30\uc6b8\uc790 \uc7ad \ucff0\uc2a4\ub294 5\ud68c \ud328\uc804 \ucc98\ub9ac\ub85c \ub4f1\ud310\ud588\ub2e4. 21\uac1c\uc6d4\ub9cc\uc758 \ub4f1\ud310\uc784\uc5d0\ub3c4 \ubd88\uad6c\ud558\uace0, 5\uc774\ub2dd \ub3d9\uc548 5\ud53c\uc548\ud0c0 2\uc2e4\uc810\uc73c\ub85c \ub9c9\uc558\ub2e4. 7\uc6d4 18\uc77c, \uc138\ub108\ud130\uc2a4\uc640\uc758 \uacbd\uae30\uc5d0\uc11c \ub2e4\uc2dc \ud328\uc804 \ucc98\ub9ac\ub85c \ub4f1\ud310\ud574 \u20443\uc774\ub2dd \ub3d9\uc548 2\ud53c\uc548\ud0c0 1\ubcfc\ub137 3\uc2e4\uc810\ud588\ub2e4. \uc774 \uacbd\uae30\uac00 \uc7ad \ucff0\uc2a4\uc758 \uba54\uc774\uc800 \ub9ac\uadf8 \ub9c8\uc9c0\ub9c9 \uacbd\uae30\uac00 \ub418\uc5c8\ub2e4."} {"question": "\uc544\ud3b8 \uae08\uc9c0\ub839\uc774 \ub0b4\ub824\uc9c4 \ud574\ub294?", "paragraph": "\uc774 \uc2dc\uae30\uc5d0 \uc544\ud3b8\ub3c4 \ubcf8\uaca9\uc801\uc73c\ub85c \ub4e4\uc5b4\uc624\uae30 \uc2dc\uc791\ud588\ub294\ub370 \uc601\uad6d\uacfc\uc758 \uad50\uc5ed\uc774 \uc2dc\uc791\ub41c \uc774\ud6c4 \uc601\uad6d \uc0c1\uc778\ub4e4\uc740 \ub3c8\uc744 \ub354 \ubc8c\uae30 \uc704\ud574 \uc778\ub3c4\uc5d0\uc11c \uc544\ud3b8\uc744 \uac00\uc838\uc640 \uc911\uad6d\uc5d0 \ud314\uae30 \uc2dc\uc791\ud558\uc600\ub2e4. \ubcf8\ub798 1729\ub144(\uc639\uc815 7\ub144)\uc5d0 \uc774\ubbf8 \uc544\ud3b8 \uae08\uc9c0\ub839\uc774 \ub0b4\ub9ac\uace0 2\ub144 \ub4a4\uc778 1731\ub144(\uc639\uc815 9\ub144)\uc5d0\ub294 \uc544\ud3b8 \ud761\uc5f0\uc790\uc640 \uc544\ud3b8 \ud310\ub9e4\uc790\ub97c \uac00\ucc28\uc5c6\uc774 \ucc98\ud615\ud558\ub294 \ub4f1\uc758 \ub2e8\uc18d\uc744 \ub300\ud3ed \uac15\ud654\ud558\ub294 \ub4f1 \uac15\uacbd\ucc45\uc744 \uc37c\uc73c\ub098 \uac74\ub96d\uc81c\uc758 \uce58\uc138\ub97c \uac70\uce58\uba74\uc11c \uc815\ucc45\ub4e4\uc774 \ub290\uc2a8\ud574\uc9c0\uace0 \uc9c0\ubc29\uc758 \uad00\ub9ac\ub4e4 \uc5ed\uc2dc \uc601\uad6d \uc0c1\uc778\ub4e4\uc5d0\uac8c \ub1cc\ubb3c\uc744 \ubc1b\uc73c\uba74\uc11c \uc544\ud3b8\uc5d0 \ub300\ud55c \uc81c\uc7ac\uac00 \ud06c\uac8c \ub290\uc2a8\ud574\uc838 \uc544\ud3b8\uc774 \ub2e4\uc2dc \ub4e4\uc5b4\uc624\uae30 \uc2dc\uc791\ud558\uc600\uace0 \uac00\uacbd\uc81c \uce58\uc138 \ub54c\uc5d0\ub294 \ubbfc\uac04\uc740 \ubb3c\ub860 \uc911\uc559\uacfc \uc9c0\ubc29 \uad00\ub9ac\ub4e4\ub3c4 \uc544\ud3b8\uc744 \ud761\uc5f0\ud558\uc600\ub2e4. \uc544\ud3b8 \uc218\uc785\ub7c9\uc740 1816\ub144(\uac00\uacbd 21\ub144) 5000\uc0c1\uc790\ub97c \uc2dc\uc791\uc73c\ub85c \uac00\uacbd\uc81c \ub9d0\ub144\uc778 1820\ub144(\uac00\uacbd 25\ub144)\uc5d0\ub294 \uc774\ubbf8 1\ub9cc \uc0c1\uc790\uc5d0 \uc721\ubc15\ud560 \uc815\ub3c4\ub85c \ub9ce\uc740 \uc591\uc758 \uc544\ud3b8\uc774 \ub4e4\uc5b4\uc654\ub2e4.", "answer": "1729\ub144", "sentence": "\ubcf8\ub798 1729\ub144 (\uc639\uc815 7\ub144)\uc5d0 \uc774\ubbf8 \uc544\ud3b8 \uae08\uc9c0\ub839\uc774 \ub0b4\ub9ac\uace0 2\ub144 \ub4a4\uc778 1731\ub144(\uc639\uc815 9\ub144)\uc5d0\ub294 \uc544\ud3b8 \ud761\uc5f0\uc790\uc640 \uc544\ud3b8 \ud310\ub9e4\uc790\ub97c \uac00\ucc28\uc5c6\uc774 \ucc98\ud615\ud558\ub294 \ub4f1\uc758 \ub2e8\uc18d\uc744 \ub300\ud3ed \uac15\ud654\ud558\ub294 \ub4f1 \uac15\uacbd\ucc45\uc744 \uc37c\uc73c\ub098 \uac74\ub96d\uc81c\uc758 \uce58\uc138\ub97c \uac70\uce58\uba74\uc11c \uc815\ucc45\ub4e4\uc774 \ub290\uc2a8\ud574\uc9c0\uace0 \uc9c0\ubc29\uc758 \uad00\ub9ac\ub4e4 \uc5ed\uc2dc \uc601\uad6d \uc0c1\uc778\ub4e4\uc5d0\uac8c \ub1cc\ubb3c\uc744 \ubc1b\uc73c\uba74\uc11c \uc544\ud3b8\uc5d0 \ub300\ud55c \uc81c\uc7ac\uac00 \ud06c\uac8c \ub290\uc2a8\ud574\uc838 \uc544\ud3b8\uc774 \ub2e4\uc2dc \ub4e4\uc5b4\uc624\uae30 \uc2dc\uc791\ud558\uc600\uace0 \uac00\uacbd\uc81c \uce58\uc138 \ub54c\uc5d0\ub294 \ubbfc\uac04\uc740 \ubb3c\ub860 \uc911\uc559\uacfc \uc9c0\ubc29 \uad00\ub9ac\ub4e4\ub3c4 \uc544\ud3b8\uc744 \ud761\uc5f0\ud558\uc600\ub2e4.", "paragraph_sentence": "\uc774 \uc2dc\uae30\uc5d0 \uc544\ud3b8\ub3c4 \ubcf8\uaca9\uc801\uc73c\ub85c \ub4e4\uc5b4\uc624\uae30 \uc2dc\uc791\ud588\ub294\ub370 \uc601\uad6d\uacfc\uc758 \uad50\uc5ed\uc774 \uc2dc\uc791\ub41c \uc774\ud6c4 \uc601\uad6d \uc0c1\uc778\ub4e4\uc740 \ub3c8\uc744 \ub354 \ubc8c\uae30 \uc704\ud574 \uc778\ub3c4\uc5d0\uc11c \uc544\ud3b8\uc744 \uac00\uc838\uc640 \uc911\uad6d\uc5d0 \ud314\uae30 \uc2dc\uc791\ud558\uc600\ub2e4. \ubcf8\ub798 1729\ub144 (\uc639\uc815 7\ub144)\uc5d0 \uc774\ubbf8 \uc544\ud3b8 \uae08\uc9c0\ub839\uc774 \ub0b4\ub9ac\uace0 2\ub144 \ub4a4\uc778 1731\ub144(\uc639\uc815 9\ub144)\uc5d0\ub294 \uc544\ud3b8 \ud761\uc5f0\uc790\uc640 \uc544\ud3b8 \ud310\ub9e4\uc790\ub97c \uac00\ucc28\uc5c6\uc774 \ucc98\ud615\ud558\ub294 \ub4f1\uc758 \ub2e8\uc18d\uc744 \ub300\ud3ed \uac15\ud654\ud558\ub294 \ub4f1 \uac15\uacbd\ucc45\uc744 \uc37c\uc73c\ub098 \uac74\ub96d\uc81c\uc758 \uce58\uc138\ub97c \uac70\uce58\uba74\uc11c \uc815\ucc45\ub4e4\uc774 \ub290\uc2a8\ud574\uc9c0\uace0 \uc9c0\ubc29\uc758 \uad00\ub9ac\ub4e4 \uc5ed\uc2dc \uc601\uad6d \uc0c1\uc778\ub4e4\uc5d0\uac8c \ub1cc\ubb3c\uc744 \ubc1b\uc73c\uba74\uc11c \uc544\ud3b8\uc5d0 \ub300\ud55c \uc81c\uc7ac\uac00 \ud06c\uac8c \ub290\uc2a8\ud574\uc838 \uc544\ud3b8\uc774 \ub2e4\uc2dc \ub4e4\uc5b4\uc624\uae30 \uc2dc\uc791\ud558\uc600\uace0 \uac00\uacbd\uc81c \uce58\uc138 \ub54c\uc5d0\ub294 \ubbfc\uac04\uc740 \ubb3c\ub860 \uc911\uc559\uacfc \uc9c0\ubc29 \uad00\ub9ac\ub4e4\ub3c4 \uc544\ud3b8\uc744 \ud761\uc5f0\ud558\uc600\ub2e4. \uc544\ud3b8 \uc218\uc785\ub7c9\uc740 1816\ub144(\uac00\uacbd 21\ub144) 5000\uc0c1\uc790\ub97c \uc2dc\uc791\uc73c\ub85c \uac00\uacbd\uc81c \ub9d0\ub144\uc778 1820\ub144(\uac00\uacbd 25\ub144)\uc5d0\ub294 \uc774\ubbf8 1\ub9cc \uc0c1\uc790\uc5d0 \uc721\ubc15\ud560 \uc815\ub3c4\ub85c \ub9ce\uc740 \uc591\uc758 \uc544\ud3b8\uc774 \ub4e4\uc5b4\uc654\ub2e4.", "paragraph_answer": "\uc774 \uc2dc\uae30\uc5d0 \uc544\ud3b8\ub3c4 \ubcf8\uaca9\uc801\uc73c\ub85c \ub4e4\uc5b4\uc624\uae30 \uc2dc\uc791\ud588\ub294\ub370 \uc601\uad6d\uacfc\uc758 \uad50\uc5ed\uc774 \uc2dc\uc791\ub41c \uc774\ud6c4 \uc601\uad6d \uc0c1\uc778\ub4e4\uc740 \ub3c8\uc744 \ub354 \ubc8c\uae30 \uc704\ud574 \uc778\ub3c4\uc5d0\uc11c \uc544\ud3b8\uc744 \uac00\uc838\uc640 \uc911\uad6d\uc5d0 \ud314\uae30 \uc2dc\uc791\ud558\uc600\ub2e4. \ubcf8\ub798 1729\ub144 (\uc639\uc815 7\ub144)\uc5d0 \uc774\ubbf8 \uc544\ud3b8 \uae08\uc9c0\ub839\uc774 \ub0b4\ub9ac\uace0 2\ub144 \ub4a4\uc778 1731\ub144(\uc639\uc815 9\ub144)\uc5d0\ub294 \uc544\ud3b8 \ud761\uc5f0\uc790\uc640 \uc544\ud3b8 \ud310\ub9e4\uc790\ub97c \uac00\ucc28\uc5c6\uc774 \ucc98\ud615\ud558\ub294 \ub4f1\uc758 \ub2e8\uc18d\uc744 \ub300\ud3ed \uac15\ud654\ud558\ub294 \ub4f1 \uac15\uacbd\ucc45\uc744 \uc37c\uc73c\ub098 \uac74\ub96d\uc81c\uc758 \uce58\uc138\ub97c \uac70\uce58\uba74\uc11c \uc815\ucc45\ub4e4\uc774 \ub290\uc2a8\ud574\uc9c0\uace0 \uc9c0\ubc29\uc758 \uad00\ub9ac\ub4e4 \uc5ed\uc2dc \uc601\uad6d \uc0c1\uc778\ub4e4\uc5d0\uac8c \ub1cc\ubb3c\uc744 \ubc1b\uc73c\uba74\uc11c \uc544\ud3b8\uc5d0 \ub300\ud55c \uc81c\uc7ac\uac00 \ud06c\uac8c \ub290\uc2a8\ud574\uc838 \uc544\ud3b8\uc774 \ub2e4\uc2dc \ub4e4\uc5b4\uc624\uae30 \uc2dc\uc791\ud558\uc600\uace0 \uac00\uacbd\uc81c \uce58\uc138 \ub54c\uc5d0\ub294 \ubbfc\uac04\uc740 \ubb3c\ub860 \uc911\uc559\uacfc \uc9c0\ubc29 \uad00\ub9ac\ub4e4\ub3c4 \uc544\ud3b8\uc744 \ud761\uc5f0\ud558\uc600\ub2e4. \uc544\ud3b8 \uc218\uc785\ub7c9\uc740 1816\ub144(\uac00\uacbd 21\ub144) 5000\uc0c1\uc790\ub97c \uc2dc\uc791\uc73c\ub85c \uac00\uacbd\uc81c \ub9d0\ub144\uc778 1820\ub144(\uac00\uacbd 25\ub144)\uc5d0\ub294 \uc774\ubbf8 1\ub9cc \uc0c1\uc790\uc5d0 \uc721\ubc15\ud560 \uc815\ub3c4\ub85c \ub9ce\uc740 \uc591\uc758 \uc544\ud3b8\uc774 \ub4e4\uc5b4\uc654\ub2e4.", "sentence_answer": "\ubcf8\ub798 1729\ub144 (\uc639\uc815 7\ub144)\uc5d0 \uc774\ubbf8 \uc544\ud3b8 \uae08\uc9c0\ub839\uc774 \ub0b4\ub9ac\uace0 2\ub144 \ub4a4\uc778 1731\ub144(\uc639\uc815 9\ub144)\uc5d0\ub294 \uc544\ud3b8 \ud761\uc5f0\uc790\uc640 \uc544\ud3b8 \ud310\ub9e4\uc790\ub97c \uac00\ucc28\uc5c6\uc774 \ucc98\ud615\ud558\ub294 \ub4f1\uc758 \ub2e8\uc18d\uc744 \ub300\ud3ed \uac15\ud654\ud558\ub294 \ub4f1 \uac15\uacbd\ucc45\uc744 \uc37c\uc73c\ub098 \uac74\ub96d\uc81c\uc758 \uce58\uc138\ub97c \uac70\uce58\uba74\uc11c \uc815\ucc45\ub4e4\uc774 \ub290\uc2a8\ud574\uc9c0\uace0 \uc9c0\ubc29\uc758 \uad00\ub9ac\ub4e4 \uc5ed\uc2dc \uc601\uad6d \uc0c1\uc778\ub4e4\uc5d0\uac8c \ub1cc\ubb3c\uc744 \ubc1b\uc73c\uba74\uc11c \uc544\ud3b8\uc5d0 \ub300\ud55c \uc81c\uc7ac\uac00 \ud06c\uac8c \ub290\uc2a8\ud574\uc838 \uc544\ud3b8\uc774 \ub2e4\uc2dc \ub4e4\uc5b4\uc624\uae30 \uc2dc\uc791\ud558\uc600\uace0 \uac00\uacbd\uc81c \uce58\uc138 \ub54c\uc5d0\ub294 \ubbfc\uac04\uc740 \ubb3c\ub860 \uc911\uc559\uacfc \uc9c0\ubc29 \uad00\ub9ac\ub4e4\ub3c4 \uc544\ud3b8\uc744 \ud761\uc5f0\ud558\uc600\ub2e4.", "paragraph_id": "6500266-10-1", "paragraph_question": "question: \uc544\ud3b8 \uae08\uc9c0\ub839\uc774 \ub0b4\ub824\uc9c4 \ud574\ub294?, context: \uc774 \uc2dc\uae30\uc5d0 \uc544\ud3b8\ub3c4 \ubcf8\uaca9\uc801\uc73c\ub85c \ub4e4\uc5b4\uc624\uae30 \uc2dc\uc791\ud588\ub294\ub370 \uc601\uad6d\uacfc\uc758 \uad50\uc5ed\uc774 \uc2dc\uc791\ub41c \uc774\ud6c4 \uc601\uad6d \uc0c1\uc778\ub4e4\uc740 \ub3c8\uc744 \ub354 \ubc8c\uae30 \uc704\ud574 \uc778\ub3c4\uc5d0\uc11c \uc544\ud3b8\uc744 \uac00\uc838\uc640 \uc911\uad6d\uc5d0 \ud314\uae30 \uc2dc\uc791\ud558\uc600\ub2e4. \ubcf8\ub798 1729\ub144(\uc639\uc815 7\ub144)\uc5d0 \uc774\ubbf8 \uc544\ud3b8 \uae08\uc9c0\ub839\uc774 \ub0b4\ub9ac\uace0 2\ub144 \ub4a4\uc778 1731\ub144(\uc639\uc815 9\ub144)\uc5d0\ub294 \uc544\ud3b8 \ud761\uc5f0\uc790\uc640 \uc544\ud3b8 \ud310\ub9e4\uc790\ub97c \uac00\ucc28\uc5c6\uc774 \ucc98\ud615\ud558\ub294 \ub4f1\uc758 \ub2e8\uc18d\uc744 \ub300\ud3ed \uac15\ud654\ud558\ub294 \ub4f1 \uac15\uacbd\ucc45\uc744 \uc37c\uc73c\ub098 \uac74\ub96d\uc81c\uc758 \uce58\uc138\ub97c \uac70\uce58\uba74\uc11c \uc815\ucc45\ub4e4\uc774 \ub290\uc2a8\ud574\uc9c0\uace0 \uc9c0\ubc29\uc758 \uad00\ub9ac\ub4e4 \uc5ed\uc2dc \uc601\uad6d \uc0c1\uc778\ub4e4\uc5d0\uac8c \ub1cc\ubb3c\uc744 \ubc1b\uc73c\uba74\uc11c \uc544\ud3b8\uc5d0 \ub300\ud55c \uc81c\uc7ac\uac00 \ud06c\uac8c \ub290\uc2a8\ud574\uc838 \uc544\ud3b8\uc774 \ub2e4\uc2dc \ub4e4\uc5b4\uc624\uae30 \uc2dc\uc791\ud558\uc600\uace0 \uac00\uacbd\uc81c \uce58\uc138 \ub54c\uc5d0\ub294 \ubbfc\uac04\uc740 \ubb3c\ub860 \uc911\uc559\uacfc \uc9c0\ubc29 \uad00\ub9ac\ub4e4\ub3c4 \uc544\ud3b8\uc744 \ud761\uc5f0\ud558\uc600\ub2e4. \uc544\ud3b8 \uc218\uc785\ub7c9\uc740 1816\ub144(\uac00\uacbd 21\ub144) 5000\uc0c1\uc790\ub97c \uc2dc\uc791\uc73c\ub85c \uac00\uacbd\uc81c \ub9d0\ub144\uc778 1820\ub144(\uac00\uacbd 25\ub144)\uc5d0\ub294 \uc774\ubbf8 1\ub9cc \uc0c1\uc790\uc5d0 \uc721\ubc15\ud560 \uc815\ub3c4\ub85c \ub9ce\uc740 \uc591\uc758 \uc544\ud3b8\uc774 \ub4e4\uc5b4\uc654\ub2e4."} {"question": "\uc774\ubd80\uce74\uc640 \ubaa8\ub9ac\ud0c0\ub294 \ud2b8\ub79c\uc9c0\uc2a4\ud130\ub97c \uad70\uc0ac\uc6a9\uc774 \uc544\ub2cc \ubb34\uc5c7\uc5d0 \uc751\uc6a9\ud558\ub824\uace0 \ud588\ub294\uac00?", "paragraph": "1950\ub144\ub300 \ucd08\ubc18 \uc774\ubd80\uce74\ub294 \ubbf8\uad6d\uc744 \uc5ec\ud589\ud558\ub2e4\uac00 \ubca8 \uc5f0\uad6c\uc18c\uc758 \ud2b8\ub79c\uc9c0\uc2a4\ud130 \ubc1c\uba85 \uc18c\uc2dd\uc744 \ub4e3\uac8c \ub418\uc5c8\ub2e4. \uadf8\ub294 \ubca8 \uc5f0\uad6c\uc18c\uac00 \ud2b8\ub79c\uc9c0\uc2a4\ud130 \uae30\uc220\uc744 \uc77c\ubcf8\uc5d0\uac8c \uc0ac\uc6a9\ud560 \uc218 \uc788\ub3c4\ub85d \ud5c8\uac00\ud560 \uac83\uc774\ub77c\uace0 \ud655\uc2e0\ud588\ub2e4. \ub300\ubd80\ubd84\uc758 \ubbf8\uad6d \ud68c\uc0ac\ub4e4\uc774 \ud2b8\ub79c\uc9c0\uc2a4\ud130\ub97c \uad70\uc0ac\uc6a9\uc73c\ub85c \uc5f0\uad6c\ud558\uace0 \uc788\uc5c8\ub358 \ubc18\uba74, \uc774\ubd80\uce74\uc640 \ubaa8\ub9ac\ud0c0\ub294 \uadf8\uac83\uc744 \ud1b5\uc2e0\uc5d0 \uc751\uc6a9\ud558\ub824\uace0 \ud588\ub2e4. \ube44\ub85d \ubbf8\uad6d \ud68c\uc0ac \ub808\uc804\uc2dc\uc640 \ud14d\uc0ac\uc2a4 \uc778\uc2a4\ud2b8\ub8e8\uba3c\ud2b8\uac00 \uccab \ubc88\uc9f8 \ud2b8\ub79c\uc9c0\uc2a4\ud130 \ub77c\ub514\uc624\ub97c \ub9cc\ub4e4\uc5c8\uc9c0\ub9cc, \uc774\ubd80\uce74\uc758 \ud68c\uc0ac\uac00 \uadf8\uac83\uc744 \ucc98\uc74c\uc73c\ub85c \uc0c1\uc5c5\uc801\uc73c\ub85c \uc131\uacf5\ud558\ub3c4\ub85d \ub9cc\ub4e4\uc5c8\ub2e4. 1958\ub144 \uc9c0\uae08\uc758 \uc774\ub984\uc73c\ub85c \ubc14\uafb8\uc5c8\ub2e4. \uc774 \ud68c\uc0ac\ub294 \uc138\uacc4 \ucd5c\ucd08\ub85c 1957\ub144 \ud3ec\ucf13\ud615 \ud2b8\ub79c\uc9c0\uc2a4\ud130 \ub77c\ub514\uc624\ub97c \uac1c\ubc1c\ud558\uc600\uc73c\uba70 1969\ub144\uc5d0\ub294 \uc0b0\uc5c5\uc6a9 \ubc0f \uc18c\ube44\uc790\uc6a9 \uceec\ub7ec\ube44\ub514\uc624\ud14c\uc774\ud504 \ub9ac\ucf54\ub354\ub97c \uc2dc\ud310\ud558\uc600\ub2e4. \uc18c\ub2c8\ub294 \uc77c\ubcf8\uc774 20\uc138\uae30 \ud6c4\ubc18 \uc138\uacc4\uc801\uc778 \uc804\uc790\uc81c\ud488 \uc0dd\uc0b0\uad6d\uc774 \ub418\ub294 \ub370 \uc8fc\ub3c4\uc801\uc778 \uc5ed\ud560\uc744 \ud558\uc600\uc73c\uba70, 1987~88\ub144 CBS\ub808\ucf54\ub4dc\uadf8\ub8f9\uc744 \ub9e4\uc785\ud568\uc73c\ub85c\uc368 \uc138\uacc4 \ucd5c\ub300\uc758 \ub808\ucf54\ub4dc\ud68c\uc0ac\ub85c\ub3c4 \uc790\ub9ac\uc7a1\uac8c \ub418\uc5c8\ub2e4.", "answer": "\ud1b5\uc2e0", "sentence": "\uadf8\uac83\uc744 \ud1b5\uc2e0 \uc5d0 \uc751\uc6a9\ud558\ub824\uace0 \ud588\ub2e4.", "paragraph_sentence": "1950\ub144\ub300 \ucd08\ubc18 \uc774\ubd80\uce74\ub294 \ubbf8\uad6d\uc744 \uc5ec\ud589\ud558\ub2e4\uac00 \ubca8 \uc5f0\uad6c\uc18c\uc758 \ud2b8\ub79c\uc9c0\uc2a4\ud130 \ubc1c\uba85 \uc18c\uc2dd\uc744 \ub4e3\uac8c \ub418\uc5c8\ub2e4. \uadf8\ub294 \ubca8 \uc5f0\uad6c\uc18c\uac00 \ud2b8\ub79c\uc9c0\uc2a4\ud130 \uae30\uc220\uc744 \uc77c\ubcf8\uc5d0\uac8c \uc0ac\uc6a9\ud560 \uc218 \uc788\ub3c4\ub85d \ud5c8\uac00\ud560 \uac83\uc774\ub77c\uace0 \ud655\uc2e0\ud588\ub2e4. \ub300\ubd80\ubd84\uc758 \ubbf8\uad6d \ud68c\uc0ac\ub4e4\uc774 \ud2b8\ub79c\uc9c0\uc2a4\ud130\ub97c \uad70\uc0ac\uc6a9\uc73c\ub85c \uc5f0\uad6c\ud558\uace0 \uc788\uc5c8\ub358 \ubc18\uba74, \uc774\ubd80\uce74\uc640 \ubaa8\ub9ac\ud0c0\ub294 \uadf8\uac83\uc744 \ud1b5\uc2e0 \uc5d0 \uc751\uc6a9\ud558\ub824\uace0 \ud588\ub2e4. \ube44\ub85d \ubbf8\uad6d \ud68c\uc0ac \ub808\uc804\uc2dc\uc640 \ud14d\uc0ac\uc2a4 \uc778\uc2a4\ud2b8\ub8e8\uba3c\ud2b8\uac00 \uccab \ubc88\uc9f8 \ud2b8\ub79c\uc9c0\uc2a4\ud130 \ub77c\ub514\uc624\ub97c \ub9cc\ub4e4\uc5c8\uc9c0\ub9cc, \uc774\ubd80\uce74\uc758 \ud68c\uc0ac\uac00 \uadf8\uac83\uc744 \ucc98\uc74c\uc73c\ub85c \uc0c1\uc5c5\uc801\uc73c\ub85c \uc131\uacf5\ud558\ub3c4\ub85d \ub9cc\ub4e4\uc5c8\ub2e4. 1958\ub144 \uc9c0\uae08\uc758 \uc774\ub984\uc73c\ub85c \ubc14\uafb8\uc5c8\ub2e4. \uc774 \ud68c\uc0ac\ub294 \uc138\uacc4 \ucd5c\ucd08\ub85c 1957\ub144 \ud3ec\ucf13\ud615 \ud2b8\ub79c\uc9c0\uc2a4\ud130 \ub77c\ub514\uc624\ub97c \uac1c\ubc1c\ud558\uc600\uc73c\uba70 1969\ub144\uc5d0\ub294 \uc0b0\uc5c5\uc6a9 \ubc0f \uc18c\ube44\uc790\uc6a9 \uceec\ub7ec\ube44\ub514\uc624\ud14c\uc774\ud504 \ub9ac\ucf54\ub354\ub97c \uc2dc\ud310\ud558\uc600\ub2e4. \uc18c\ub2c8\ub294 \uc77c\ubcf8\uc774 20\uc138\uae30 \ud6c4\ubc18 \uc138\uacc4\uc801\uc778 \uc804\uc790\uc81c\ud488 \uc0dd\uc0b0\uad6d\uc774 \ub418\ub294 \ub370 \uc8fc\ub3c4\uc801\uc778 \uc5ed\ud560\uc744 \ud558\uc600\uc73c\uba70, 1987~88\ub144 CBS\ub808\ucf54\ub4dc\uadf8\ub8f9\uc744 \ub9e4\uc785\ud568\uc73c\ub85c\uc368 \uc138\uacc4 \ucd5c\ub300\uc758 \ub808\ucf54\ub4dc\ud68c\uc0ac\ub85c\ub3c4 \uc790\ub9ac\uc7a1\uac8c \ub418\uc5c8\ub2e4.", "paragraph_answer": "1950\ub144\ub300 \ucd08\ubc18 \uc774\ubd80\uce74\ub294 \ubbf8\uad6d\uc744 \uc5ec\ud589\ud558\ub2e4\uac00 \ubca8 \uc5f0\uad6c\uc18c\uc758 \ud2b8\ub79c\uc9c0\uc2a4\ud130 \ubc1c\uba85 \uc18c\uc2dd\uc744 \ub4e3\uac8c \ub418\uc5c8\ub2e4. \uadf8\ub294 \ubca8 \uc5f0\uad6c\uc18c\uac00 \ud2b8\ub79c\uc9c0\uc2a4\ud130 \uae30\uc220\uc744 \uc77c\ubcf8\uc5d0\uac8c \uc0ac\uc6a9\ud560 \uc218 \uc788\ub3c4\ub85d \ud5c8\uac00\ud560 \uac83\uc774\ub77c\uace0 \ud655\uc2e0\ud588\ub2e4. \ub300\ubd80\ubd84\uc758 \ubbf8\uad6d \ud68c\uc0ac\ub4e4\uc774 \ud2b8\ub79c\uc9c0\uc2a4\ud130\ub97c \uad70\uc0ac\uc6a9\uc73c\ub85c \uc5f0\uad6c\ud558\uace0 \uc788\uc5c8\ub358 \ubc18\uba74, \uc774\ubd80\uce74\uc640 \ubaa8\ub9ac\ud0c0\ub294 \uadf8\uac83\uc744 \ud1b5\uc2e0 \uc5d0 \uc751\uc6a9\ud558\ub824\uace0 \ud588\ub2e4. \ube44\ub85d \ubbf8\uad6d \ud68c\uc0ac \ub808\uc804\uc2dc\uc640 \ud14d\uc0ac\uc2a4 \uc778\uc2a4\ud2b8\ub8e8\uba3c\ud2b8\uac00 \uccab \ubc88\uc9f8 \ud2b8\ub79c\uc9c0\uc2a4\ud130 \ub77c\ub514\uc624\ub97c \ub9cc\ub4e4\uc5c8\uc9c0\ub9cc, \uc774\ubd80\uce74\uc758 \ud68c\uc0ac\uac00 \uadf8\uac83\uc744 \ucc98\uc74c\uc73c\ub85c \uc0c1\uc5c5\uc801\uc73c\ub85c \uc131\uacf5\ud558\ub3c4\ub85d \ub9cc\ub4e4\uc5c8\ub2e4. 1958\ub144 \uc9c0\uae08\uc758 \uc774\ub984\uc73c\ub85c \ubc14\uafb8\uc5c8\ub2e4. \uc774 \ud68c\uc0ac\ub294 \uc138\uacc4 \ucd5c\ucd08\ub85c 1957\ub144 \ud3ec\ucf13\ud615 \ud2b8\ub79c\uc9c0\uc2a4\ud130 \ub77c\ub514\uc624\ub97c \uac1c\ubc1c\ud558\uc600\uc73c\uba70 1969\ub144\uc5d0\ub294 \uc0b0\uc5c5\uc6a9 \ubc0f \uc18c\ube44\uc790\uc6a9 \uceec\ub7ec\ube44\ub514\uc624\ud14c\uc774\ud504 \ub9ac\ucf54\ub354\ub97c \uc2dc\ud310\ud558\uc600\ub2e4. \uc18c\ub2c8\ub294 \uc77c\ubcf8\uc774 20\uc138\uae30 \ud6c4\ubc18 \uc138\uacc4\uc801\uc778 \uc804\uc790\uc81c\ud488 \uc0dd\uc0b0\uad6d\uc774 \ub418\ub294 \ub370 \uc8fc\ub3c4\uc801\uc778 \uc5ed\ud560\uc744 \ud558\uc600\uc73c\uba70, 1987~88\ub144 CBS\ub808\ucf54\ub4dc\uadf8\ub8f9\uc744 \ub9e4\uc785\ud568\uc73c\ub85c\uc368 \uc138\uacc4 \ucd5c\ub300\uc758 \ub808\ucf54\ub4dc\ud68c\uc0ac\ub85c\ub3c4 \uc790\ub9ac\uc7a1\uac8c \ub418\uc5c8\ub2e4.", "sentence_answer": "\uadf8\uac83\uc744 \ud1b5\uc2e0 \uc5d0 \uc751\uc6a9\ud558\ub824\uace0 \ud588\ub2e4.", "paragraph_id": "6445677-1-1", "paragraph_question": "question: \uc774\ubd80\uce74\uc640 \ubaa8\ub9ac\ud0c0\ub294 \ud2b8\ub79c\uc9c0\uc2a4\ud130\ub97c \uad70\uc0ac\uc6a9\uc774 \uc544\ub2cc \ubb34\uc5c7\uc5d0 \uc751\uc6a9\ud558\ub824\uace0 \ud588\ub294\uac00?, context: 1950\ub144\ub300 \ucd08\ubc18 \uc774\ubd80\uce74\ub294 \ubbf8\uad6d\uc744 \uc5ec\ud589\ud558\ub2e4\uac00 \ubca8 \uc5f0\uad6c\uc18c\uc758 \ud2b8\ub79c\uc9c0\uc2a4\ud130 \ubc1c\uba85 \uc18c\uc2dd\uc744 \ub4e3\uac8c \ub418\uc5c8\ub2e4. \uadf8\ub294 \ubca8 \uc5f0\uad6c\uc18c\uac00 \ud2b8\ub79c\uc9c0\uc2a4\ud130 \uae30\uc220\uc744 \uc77c\ubcf8\uc5d0\uac8c \uc0ac\uc6a9\ud560 \uc218 \uc788\ub3c4\ub85d \ud5c8\uac00\ud560 \uac83\uc774\ub77c\uace0 \ud655\uc2e0\ud588\ub2e4. \ub300\ubd80\ubd84\uc758 \ubbf8\uad6d \ud68c\uc0ac\ub4e4\uc774 \ud2b8\ub79c\uc9c0\uc2a4\ud130\ub97c \uad70\uc0ac\uc6a9\uc73c\ub85c \uc5f0\uad6c\ud558\uace0 \uc788\uc5c8\ub358 \ubc18\uba74, \uc774\ubd80\uce74\uc640 \ubaa8\ub9ac\ud0c0\ub294 \uadf8\uac83\uc744 \ud1b5\uc2e0\uc5d0 \uc751\uc6a9\ud558\ub824\uace0 \ud588\ub2e4. \ube44\ub85d \ubbf8\uad6d \ud68c\uc0ac \ub808\uc804\uc2dc\uc640 \ud14d\uc0ac\uc2a4 \uc778\uc2a4\ud2b8\ub8e8\uba3c\ud2b8\uac00 \uccab \ubc88\uc9f8 \ud2b8\ub79c\uc9c0\uc2a4\ud130 \ub77c\ub514\uc624\ub97c \ub9cc\ub4e4\uc5c8\uc9c0\ub9cc, \uc774\ubd80\uce74\uc758 \ud68c\uc0ac\uac00 \uadf8\uac83\uc744 \ucc98\uc74c\uc73c\ub85c \uc0c1\uc5c5\uc801\uc73c\ub85c \uc131\uacf5\ud558\ub3c4\ub85d \ub9cc\ub4e4\uc5c8\ub2e4. 1958\ub144 \uc9c0\uae08\uc758 \uc774\ub984\uc73c\ub85c \ubc14\uafb8\uc5c8\ub2e4. \uc774 \ud68c\uc0ac\ub294 \uc138\uacc4 \ucd5c\ucd08\ub85c 1957\ub144 \ud3ec\ucf13\ud615 \ud2b8\ub79c\uc9c0\uc2a4\ud130 \ub77c\ub514\uc624\ub97c \uac1c\ubc1c\ud558\uc600\uc73c\uba70 1969\ub144\uc5d0\ub294 \uc0b0\uc5c5\uc6a9 \ubc0f \uc18c\ube44\uc790\uc6a9 \uceec\ub7ec\ube44\ub514\uc624\ud14c\uc774\ud504 \ub9ac\ucf54\ub354\ub97c \uc2dc\ud310\ud558\uc600\ub2e4. \uc18c\ub2c8\ub294 \uc77c\ubcf8\uc774 20\uc138\uae30 \ud6c4\ubc18 \uc138\uacc4\uc801\uc778 \uc804\uc790\uc81c\ud488 \uc0dd\uc0b0\uad6d\uc774 \ub418\ub294 \ub370 \uc8fc\ub3c4\uc801\uc778 \uc5ed\ud560\uc744 \ud558\uc600\uc73c\uba70, 1987~88\ub144 CBS\ub808\ucf54\ub4dc\uadf8\ub8f9\uc744 \ub9e4\uc785\ud568\uc73c\ub85c\uc368 \uc138\uacc4 \ucd5c\ub300\uc758 \ub808\ucf54\ub4dc\ud68c\uc0ac\ub85c\ub3c4 \uc790\ub9ac\uc7a1\uac8c \ub418\uc5c8\ub2e4."} {"question": "17\uac1c\uc6d4 \uac00\ub7c9 \ubd81\ud55c\uc5d0 \uac15\uc81c \uc5b5\ub958 \ub418\uc5c8\ub2e4 \ud574\ubc29\ub41c \ubbf8\uad6d\uc778\uc740 \ub204\uad6c\uc778\uac00?", "paragraph": "\uc774\ub97c \uc704\ud574 \ud55c\ubbf8 \uc815\uc0c1\ud68c\ub2f4\uc758 \uc870\uae30 \uc131\uc0ac\uac00 \ucd94\uc9c4\ub418\uc5c8\ub2e4. \uac00\uc7a5 \uc911\uc694\ud55c \uc7c1\uc810\uc740 \uc0ac\ub4dc \ubc30\uce58\uc5d0 \uad00\ud55c \uac83\uc774\uc5c8\ub294\ub370, 6\uc6d4 9\uc77c \uc815\uc758\uc6a9 \uad6d\uac00\uc548\ubcf4\uc2e4\uc7a5\uc740 \"\uc0ac\ub4dc\ub294 \ubd81\ud55c\uc758 \uc810\uc99d\ud558\ub294 \uc704\ud611\uc73c\ub85c\ubd80\ud130 \ud55c\uad6d\uacfc \uc8fc\ud55c\ubbf8\uad70\uc744 \ubcf4\ud638\ud558\uae30 \uc704\ud574 \uacb0\uc815\ud55c \uac83\"\uc774\ub77c\uba70 \"\uc815\uad8c\uc774 \uad50\uccb4\ub418\uc5c8\ub2e4\uace0 \ud574\uc11c \uc774 \uacb0\uc815\uc744 \uacb0\ucf54 \uac00\ubccd\uac8c \uc5ec\uae30\uc9c0 \uc54a\uc744 \uac83\uc774\uba70 \ubbf8\uad6d\uacfc \uacc4\uc18d \uae34\ubc00\ud788 \ud611\uc758\ud560 \uac83\"\uc774\ub77c\uace0 \ubc1d\ud788\uba74\uc11c \"\uc815\ubd80\ub294 \ud55c\ubbf8\ub3d9\ub9f9 \ucc28\uc6d0\uc5d0\uc11c \uc57d\uc18d\ud55c \ub0b4\uc6a9\uc744 \uadfc\ubcf8\uc801\uc73c\ub85c \ubc14\uafb8\ub824\ub294 \uc758\ub3c4\ub294 \uc5c6\ub2e4\"\uace0 \uac15\uc870\ud588\ub2e4. \ub2e4\ub9cc, \ubbfc\uc8fc\uc801\u00b7\uc808\ucc28\uc801 \uc815\ub2f9\uc131\uc744 \uc704\ud574 \ud658\uacbd\uc601\ud5a5\ud3c9\uac00\uc758 \ud544\uc694\uc131\ub3c4 \uc5b8\uae09\ud588\ub2e4. \uc774\uc5d0 \ub300\ud574 \ubbf8\uad6d \ub0b4\uc5d0\uc11c\ub294 \"\uc0ac\ub4dc\uc758 \uc644\uc804\ud55c \ubc30\uce58\uc640 \uad00\ub828\ud55c \uc5b4\ub5a4 \ud658\uacbd\uc801 \uc6b0\ub824\ub3c4 \uc2e0\uc18d\ud558\uace0 \ucca0\uc800\ud55c \uac80\ud1a0\ub97c \ud1b5\ud574 \ud574\uc18c\ub418\uae38 \ubc14\ub780\ub2e4\"\uba74\uc11c\ub3c4 \ud658\uacbd\uc601\ud5a5\ud3c9\uac00\ub97c \uc774\ud574\ud558\uc9c0 \ubabb\ud558\uaca0\ub2e4\ub294 \uc758\uacac\uc774 \ub098\uc624\uae30\ub3c4 \ud588\ub2e4. 16\uc77c\uc5d0\ub294 \ubbf8\uad6d\uc744 \ubc29\ubb38 \uc911\uc778 \ubb38\uc815\uc778 \ud1b5\uc77c\uc678\uad50\uc548\ubcf4\ud2b9\ubcc4\ubcf4\uc88c\uad00\uc774 \"\ubd81\ud55c\uc774 \ud575 \ubbf8\uc0ac\uc77c \ud65c\ub3d9\uc744 \uc911\ub2e8\ud558\uba74 \ubbf8\uad6d\uc758 \ud55c\ubc18\ub3c4 \uc804\ub7b5\uc790\uc0b0\uacfc \ud55c\ubbf8\ud569\ub3d9\uad70\uc0ac\ud6c8\ub828\uc744 \ucd95\uc18c\ud560 \uc218 \uc788\ub2e4\"\uace0 \ubc1d\ud788\uace0 \ud559\uc790\ub85c\uc11c\uc758 \uacac\ud574\uc784\uc744 \uc804\uc81c\ud55c \ub4a4 \"\uc0ac\ub4dc \ubb38\uc81c\uac00 \ud574\uacb0\ub418\uc9c0 \uc54a\uc73c\uba74 \ud55c\ubbf8\ub3d9\ub9f9\uc774 \uae68\uc9c4\ub2e4\ub294 \uc778\uc2dd\uc774 \uc788\ub294\ub370 \uadf8\ub807\ub2e4\uba74 \uadf8\uac8c \ubb34\uc2a8 \ub3d9\ub9f9\uc774\ub0d0\"\ub294 \ubc1c\uc5b8\uc744 \ud558\uc600\ub2e4. \uc774\ub294 '\uc120 \ubd81\ud575\ub3d9\uacb0, \ud6c4 \uc644\uc804\ud55c \ube44\ud575\ud654'\ub97c \uc8fc\ucc3d\ud55c \ubb38\uc7ac\uc778 \uc815\ubd80\uc758 \uacf5\uc57d\uc5d0\uc11c \ud06c\uac8c \ubc97\uc5b4\ub098\uc9c0 \uc54a\uc9c0\ub9cc \ud55c\ubbf8\uc815\uc0c1\ud68c\ub2f4\uc744 \uc55e\ub450\ub85c \ud55c\ubbf8 \uac04 \uc774\uacac\uc73c\ub85c \ubc88\uc9c0\uba74\uc11c \ub17c\ub780\uc744 \uc77c\uc73c\ucf30\ub2e4. \uc774\uc5d0 \uad6d\ubc29\ubd80\ub294 \"\ud559\uc790 \uac1c\uc778\uc801 \uacac\ud574\uc784\uc744 \uc804\uc81c\ub85c \uc774\ub904\uc9c4 \uac83\uc774\uae30 \ub54c\ubb38\uc5d0 \uc815\ubd80\uc640 \uc870\uc728\ub41c \uc785\uc7a5\uc740 \uc544\ub2cc \uac83\uc73c\ub85c \uc54c\uace0 \uc788\ub2e4\"\uace0 \ud588\uc73c\uba70, \uccad\uc640\ub300\ub3c4 \"\ubb38 \ud2b9\ubcf4 \ubc1c\uc5b8\uc740 \uac1c\uc778 \uc544\uc774\ub514\uc5b4\"\ub77c\uba70 \uc0ac\uc804 \uc870\uc728\uc774 \uc5c6\uc5c8\uc74c\uc744 \uac15\uc870\ud588\ub2e4. \ubbf8\uad6d \uad6d\ubb34\ubd80\ub3c4 \"\uc6b0\ub9ac\ub294 \uc774\ub7f0 \uc2dc\uac01\uc774 \ubb38 \ud2b9\ubcf4\uc758 \uac1c\uc778\uc801 \uacac\ud574\ub85c \ud55c\uad6d \uc815\ubd80\uc758 \uacf5\uc2dd\uc801\uc778 \uc815\ucc45\uc744 \ubc18\uc601\ud55c \uac8c \uc544\ub2d0 \uc218 \uc788\ub294 \uac83\uc73c\ub85c \uc774\ud574\ud558\uace0 \uc788\ub2e4\"\ub294 \ubc18\uc751\uc744 \ubcf4\uc600\ub2e4. \uc774\ud6c4 20\uc77c\uc5d0\ub294 \ubd81\ud55c\uc5d0 1\ub144 5\uac1c\uc6d4\uac00\ub7c9 \uc5b5\ub958\ub418\uc5b4 \uc788\ub2e4\uac00 \ud574\ubc29\ub41c \uc624\ud1a0 \uc6dc\ube44\uc5b4\uac00 \ud63c\uc218\uc0c1\ud0dc\ub85c \ub3cc\uc544\uc628 \uc9c0 \uc77c\uc8fc\uc77c \ub9cc\uc5d0 \uc0ac\ub9dd\ud558\uba74\uc11c \ub610 \ub2e4\ub978 \ubb38\uc81c\ub97c \ub9cc\ub4e4\uc5c8\ub2e4. \ud558\uc9c0\ub9cc \uc815\uc0c1\ud68c\ub2f4\uc744 \uc55e\ub450\uace0 \uc5ec\ub7ec \uc554\ucd08\ub85c \ud55c\ubbf8 \uac08\ub4f1\uc774 \ud45c\uba74\ud654\ub418\ub294 \uc6b0\ub824\uc5d0 \ub300\ud574 \uccad\uc640\ub300\ub294 '\uc678\uad50\uc548\ubcf4\ub77c\uc778\uc774 \uacf5\uace0\ud558\uae30 \ub54c\ubb38\uc5d0 \uc0ac\ub4dc \ub17c\ub780\uc740 \uc624\ud574 \uc5c6\uc774 \uc798 \ud480\uc5c8\uace0, \ubb38 \ub300\ud1b5\ub839\uc774 \ubd81\ud55c \uc778\uad8c\ubb38\uc81c\uc5d0 \ub300\ud574 \ub2e8\ud638\ud55c \ud0dc\ub3c4\ub97c \ubc1d\ud78c \ub9cc\ud07c \ubbf8\uad6d\uacfc \ubd88\ud611\ud654\uc74c\uc774 \uc0dd\uae38 \uc5ec\uc9c0\uac00 \uc5c6\ub2e4'\ub294 \uc785\uc7a5\uc744 \ubcf4\uc600\ub2e4. \ub2e4\ub9cc, \uc0c8 \uc815\ubd80\uc758 \ub300\ubd81\uc811\uadfc\ubc95\uc774 \ud070 \ud2c0\uc5d0\uc11c \uc774\uc804 \uc815\uad8c\uc5d0 \ub2e4\ub97c \uc218 \uc788\ub2e4\ub294 \uc810\uc740 \ubbf8\uad6d\ub3c4 \uc778\uc815\ud574\uc57c \ud55c\ub2e4\uace0 \ub367\ubd99\uc600\ub2e4.", "answer": "\uc624\ud1a0 \uc6dc\ube44\uc5b4", "sentence": "\uc774\ud6c4 20\uc77c\uc5d0\ub294 \ubd81\ud55c\uc5d0 1\ub144 5\uac1c\uc6d4\uac00\ub7c9 \uc5b5\ub958\ub418\uc5b4 \uc788\ub2e4\uac00 \ud574\ubc29\ub41c \uc624\ud1a0 \uc6dc\ube44\uc5b4 \uac00 \ud63c\uc218\uc0c1\ud0dc\ub85c \ub3cc\uc544\uc628 \uc9c0 \uc77c\uc8fc\uc77c \ub9cc\uc5d0 \uc0ac\ub9dd\ud558\uba74\uc11c \ub610 \ub2e4\ub978 \ubb38\uc81c\ub97c \ub9cc\ub4e4\uc5c8\ub2e4.", "paragraph_sentence": "\uc774\ub97c \uc704\ud574 \ud55c\ubbf8 \uc815\uc0c1\ud68c\ub2f4\uc758 \uc870\uae30 \uc131\uc0ac\uac00 \ucd94\uc9c4\ub418\uc5c8\ub2e4. \uac00\uc7a5 \uc911\uc694\ud55c \uc7c1\uc810\uc740 \uc0ac\ub4dc \ubc30\uce58\uc5d0 \uad00\ud55c \uac83\uc774\uc5c8\ub294\ub370, 6\uc6d4 9\uc77c \uc815\uc758\uc6a9 \uad6d\uac00\uc548\ubcf4\uc2e4\uc7a5\uc740 \"\uc0ac\ub4dc\ub294 \ubd81\ud55c\uc758 \uc810\uc99d\ud558\ub294 \uc704\ud611\uc73c\ub85c\ubd80\ud130 \ud55c\uad6d\uacfc \uc8fc\ud55c\ubbf8\uad70\uc744 \ubcf4\ud638\ud558\uae30 \uc704\ud574 \uacb0\uc815\ud55c \uac83\"\uc774\ub77c\uba70 \"\uc815\uad8c\uc774 \uad50\uccb4\ub418\uc5c8\ub2e4\uace0 \ud574\uc11c \uc774 \uacb0\uc815\uc744 \uacb0\ucf54 \uac00\ubccd\uac8c \uc5ec\uae30\uc9c0 \uc54a\uc744 \uac83\uc774\uba70 \ubbf8\uad6d\uacfc \uacc4\uc18d \uae34\ubc00\ud788 \ud611\uc758\ud560 \uac83\"\uc774\ub77c\uace0 \ubc1d\ud788\uba74\uc11c \"\uc815\ubd80\ub294 \ud55c\ubbf8\ub3d9\ub9f9 \ucc28\uc6d0\uc5d0\uc11c \uc57d\uc18d\ud55c \ub0b4\uc6a9\uc744 \uadfc\ubcf8\uc801\uc73c\ub85c \ubc14\uafb8\ub824\ub294 \uc758\ub3c4\ub294 \uc5c6\ub2e4\"\uace0 \uac15\uc870\ud588\ub2e4. \ub2e4\ub9cc, \ubbfc\uc8fc\uc801\u00b7\uc808\ucc28\uc801 \uc815\ub2f9\uc131\uc744 \uc704\ud574 \ud658\uacbd\uc601\ud5a5\ud3c9\uac00\uc758 \ud544\uc694\uc131\ub3c4 \uc5b8\uae09\ud588\ub2e4. \uc774\uc5d0 \ub300\ud574 \ubbf8\uad6d \ub0b4\uc5d0\uc11c\ub294 \"\uc0ac\ub4dc\uc758 \uc644\uc804\ud55c \ubc30\uce58\uc640 \uad00\ub828\ud55c \uc5b4\ub5a4 \ud658\uacbd\uc801 \uc6b0\ub824\ub3c4 \uc2e0\uc18d\ud558\uace0 \ucca0\uc800\ud55c \uac80\ud1a0\ub97c \ud1b5\ud574 \ud574\uc18c\ub418\uae38 \ubc14\ub780\ub2e4\"\uba74\uc11c\ub3c4 \ud658\uacbd\uc601\ud5a5\ud3c9\uac00\ub97c \uc774\ud574\ud558\uc9c0 \ubabb\ud558\uaca0\ub2e4\ub294 \uc758\uacac\uc774 \ub098\uc624\uae30\ub3c4 \ud588\ub2e4. 16\uc77c\uc5d0\ub294 \ubbf8\uad6d\uc744 \ubc29\ubb38 \uc911\uc778 \ubb38\uc815\uc778 \ud1b5\uc77c\uc678\uad50\uc548\ubcf4\ud2b9\ubcc4\ubcf4\uc88c\uad00\uc774 \"\ubd81\ud55c\uc774 \ud575 \ubbf8\uc0ac\uc77c \ud65c\ub3d9\uc744 \uc911\ub2e8\ud558\uba74 \ubbf8\uad6d\uc758 \ud55c\ubc18\ub3c4 \uc804\ub7b5\uc790\uc0b0\uacfc \ud55c\ubbf8\ud569\ub3d9\uad70\uc0ac\ud6c8\ub828\uc744 \ucd95\uc18c\ud560 \uc218 \uc788\ub2e4\"\uace0 \ubc1d\ud788\uace0 \ud559\uc790\ub85c\uc11c\uc758 \uacac\ud574\uc784\uc744 \uc804\uc81c\ud55c \ub4a4 \"\uc0ac\ub4dc \ubb38\uc81c\uac00 \ud574\uacb0\ub418\uc9c0 \uc54a\uc73c\uba74 \ud55c\ubbf8\ub3d9\ub9f9\uc774 \uae68\uc9c4\ub2e4\ub294 \uc778\uc2dd\uc774 \uc788\ub294\ub370 \uadf8\ub807\ub2e4\uba74 \uadf8\uac8c \ubb34\uc2a8 \ub3d9\ub9f9\uc774\ub0d0\"\ub294 \ubc1c\uc5b8\uc744 \ud558\uc600\ub2e4. \uc774\ub294 '\uc120 \ubd81\ud575\ub3d9\uacb0, \ud6c4 \uc644\uc804\ud55c \ube44\ud575\ud654'\ub97c \uc8fc\ucc3d\ud55c \ubb38\uc7ac\uc778 \uc815\ubd80\uc758 \uacf5\uc57d\uc5d0\uc11c \ud06c\uac8c \ubc97\uc5b4\ub098\uc9c0 \uc54a\uc9c0\ub9cc \ud55c\ubbf8\uc815\uc0c1\ud68c\ub2f4\uc744 \uc55e\ub450\ub85c \ud55c\ubbf8 \uac04 \uc774\uacac\uc73c\ub85c \ubc88\uc9c0\uba74\uc11c \ub17c\ub780\uc744 \uc77c\uc73c\ucf30\ub2e4. \uc774\uc5d0 \uad6d\ubc29\ubd80\ub294 \"\ud559\uc790 \uac1c\uc778\uc801 \uacac\ud574\uc784\uc744 \uc804\uc81c\ub85c \uc774\ub904\uc9c4 \uac83\uc774\uae30 \ub54c\ubb38\uc5d0 \uc815\ubd80\uc640 \uc870\uc728\ub41c \uc785\uc7a5\uc740 \uc544\ub2cc \uac83\uc73c\ub85c \uc54c\uace0 \uc788\ub2e4\"\uace0 \ud588\uc73c\uba70, \uccad\uc640\ub300\ub3c4 \"\ubb38 \ud2b9\ubcf4 \ubc1c\uc5b8\uc740 \uac1c\uc778 \uc544\uc774\ub514\uc5b4\"\ub77c\uba70 \uc0ac\uc804 \uc870\uc728\uc774 \uc5c6\uc5c8\uc74c\uc744 \uac15\uc870\ud588\ub2e4. \ubbf8\uad6d \uad6d\ubb34\ubd80\ub3c4 \"\uc6b0\ub9ac\ub294 \uc774\ub7f0 \uc2dc\uac01\uc774 \ubb38 \ud2b9\ubcf4\uc758 \uac1c\uc778\uc801 \uacac\ud574\ub85c \ud55c\uad6d \uc815\ubd80\uc758 \uacf5\uc2dd\uc801\uc778 \uc815\ucc45\uc744 \ubc18\uc601\ud55c \uac8c \uc544\ub2d0 \uc218 \uc788\ub294 \uac83\uc73c\ub85c \uc774\ud574\ud558\uace0 \uc788\ub2e4\"\ub294 \ubc18\uc751\uc744 \ubcf4\uc600\ub2e4. \uc774\ud6c4 20\uc77c\uc5d0\ub294 \ubd81\ud55c\uc5d0 1\ub144 5\uac1c\uc6d4\uac00\ub7c9 \uc5b5\ub958\ub418\uc5b4 \uc788\ub2e4\uac00 \ud574\ubc29\ub41c \uc624\ud1a0 \uc6dc\ube44\uc5b4 \uac00 \ud63c\uc218\uc0c1\ud0dc\ub85c \ub3cc\uc544\uc628 \uc9c0 \uc77c\uc8fc\uc77c \ub9cc\uc5d0 \uc0ac\ub9dd\ud558\uba74\uc11c \ub610 \ub2e4\ub978 \ubb38\uc81c\ub97c \ub9cc\ub4e4\uc5c8\ub2e4. \ud558\uc9c0\ub9cc \uc815\uc0c1\ud68c\ub2f4\uc744 \uc55e\ub450\uace0 \uc5ec\ub7ec \uc554\ucd08\ub85c \ud55c\ubbf8 \uac08\ub4f1\uc774 \ud45c\uba74\ud654\ub418\ub294 \uc6b0\ub824\uc5d0 \ub300\ud574 \uccad\uc640\ub300\ub294 '\uc678\uad50\uc548\ubcf4\ub77c\uc778\uc774 \uacf5\uace0\ud558\uae30 \ub54c\ubb38\uc5d0 \uc0ac\ub4dc \ub17c\ub780\uc740 \uc624\ud574 \uc5c6\uc774 \uc798 \ud480\uc5c8\uace0, \ubb38 \ub300\ud1b5\ub839\uc774 \ubd81\ud55c \uc778\uad8c\ubb38\uc81c\uc5d0 \ub300\ud574 \ub2e8\ud638\ud55c \ud0dc\ub3c4\ub97c \ubc1d\ud78c \ub9cc\ud07c \ubbf8\uad6d\uacfc \ubd88\ud611\ud654\uc74c\uc774 \uc0dd\uae38 \uc5ec\uc9c0\uac00 \uc5c6\ub2e4'\ub294 \uc785\uc7a5\uc744 \ubcf4\uc600\ub2e4. \ub2e4\ub9cc, \uc0c8 \uc815\ubd80\uc758 \ub300\ubd81\uc811\uadfc\ubc95\uc774 \ud070 \ud2c0\uc5d0\uc11c \uc774\uc804 \uc815\uad8c\uc5d0 \ub2e4\ub97c \uc218 \uc788\ub2e4\ub294 \uc810\uc740 \ubbf8\uad6d\ub3c4 \uc778\uc815\ud574\uc57c \ud55c\ub2e4\uace0 \ub367\ubd99\uc600\ub2e4.", "paragraph_answer": "\uc774\ub97c \uc704\ud574 \ud55c\ubbf8 \uc815\uc0c1\ud68c\ub2f4\uc758 \uc870\uae30 \uc131\uc0ac\uac00 \ucd94\uc9c4\ub418\uc5c8\ub2e4. \uac00\uc7a5 \uc911\uc694\ud55c \uc7c1\uc810\uc740 \uc0ac\ub4dc \ubc30\uce58\uc5d0 \uad00\ud55c \uac83\uc774\uc5c8\ub294\ub370, 6\uc6d4 9\uc77c \uc815\uc758\uc6a9 \uad6d\uac00\uc548\ubcf4\uc2e4\uc7a5\uc740 \"\uc0ac\ub4dc\ub294 \ubd81\ud55c\uc758 \uc810\uc99d\ud558\ub294 \uc704\ud611\uc73c\ub85c\ubd80\ud130 \ud55c\uad6d\uacfc \uc8fc\ud55c\ubbf8\uad70\uc744 \ubcf4\ud638\ud558\uae30 \uc704\ud574 \uacb0\uc815\ud55c \uac83\"\uc774\ub77c\uba70 \"\uc815\uad8c\uc774 \uad50\uccb4\ub418\uc5c8\ub2e4\uace0 \ud574\uc11c \uc774 \uacb0\uc815\uc744 \uacb0\ucf54 \uac00\ubccd\uac8c \uc5ec\uae30\uc9c0 \uc54a\uc744 \uac83\uc774\uba70 \ubbf8\uad6d\uacfc \uacc4\uc18d \uae34\ubc00\ud788 \ud611\uc758\ud560 \uac83\"\uc774\ub77c\uace0 \ubc1d\ud788\uba74\uc11c \"\uc815\ubd80\ub294 \ud55c\ubbf8\ub3d9\ub9f9 \ucc28\uc6d0\uc5d0\uc11c \uc57d\uc18d\ud55c \ub0b4\uc6a9\uc744 \uadfc\ubcf8\uc801\uc73c\ub85c \ubc14\uafb8\ub824\ub294 \uc758\ub3c4\ub294 \uc5c6\ub2e4\"\uace0 \uac15\uc870\ud588\ub2e4. \ub2e4\ub9cc, \ubbfc\uc8fc\uc801\u00b7\uc808\ucc28\uc801 \uc815\ub2f9\uc131\uc744 \uc704\ud574 \ud658\uacbd\uc601\ud5a5\ud3c9\uac00\uc758 \ud544\uc694\uc131\ub3c4 \uc5b8\uae09\ud588\ub2e4. \uc774\uc5d0 \ub300\ud574 \ubbf8\uad6d \ub0b4\uc5d0\uc11c\ub294 \"\uc0ac\ub4dc\uc758 \uc644\uc804\ud55c \ubc30\uce58\uc640 \uad00\ub828\ud55c \uc5b4\ub5a4 \ud658\uacbd\uc801 \uc6b0\ub824\ub3c4 \uc2e0\uc18d\ud558\uace0 \ucca0\uc800\ud55c \uac80\ud1a0\ub97c \ud1b5\ud574 \ud574\uc18c\ub418\uae38 \ubc14\ub780\ub2e4\"\uba74\uc11c\ub3c4 \ud658\uacbd\uc601\ud5a5\ud3c9\uac00\ub97c \uc774\ud574\ud558\uc9c0 \ubabb\ud558\uaca0\ub2e4\ub294 \uc758\uacac\uc774 \ub098\uc624\uae30\ub3c4 \ud588\ub2e4. 16\uc77c\uc5d0\ub294 \ubbf8\uad6d\uc744 \ubc29\ubb38 \uc911\uc778 \ubb38\uc815\uc778 \ud1b5\uc77c\uc678\uad50\uc548\ubcf4\ud2b9\ubcc4\ubcf4\uc88c\uad00\uc774 \"\ubd81\ud55c\uc774 \ud575 \ubbf8\uc0ac\uc77c \ud65c\ub3d9\uc744 \uc911\ub2e8\ud558\uba74 \ubbf8\uad6d\uc758 \ud55c\ubc18\ub3c4 \uc804\ub7b5\uc790\uc0b0\uacfc \ud55c\ubbf8\ud569\ub3d9\uad70\uc0ac\ud6c8\ub828\uc744 \ucd95\uc18c\ud560 \uc218 \uc788\ub2e4\"\uace0 \ubc1d\ud788\uace0 \ud559\uc790\ub85c\uc11c\uc758 \uacac\ud574\uc784\uc744 \uc804\uc81c\ud55c \ub4a4 \"\uc0ac\ub4dc \ubb38\uc81c\uac00 \ud574\uacb0\ub418\uc9c0 \uc54a\uc73c\uba74 \ud55c\ubbf8\ub3d9\ub9f9\uc774 \uae68\uc9c4\ub2e4\ub294 \uc778\uc2dd\uc774 \uc788\ub294\ub370 \uadf8\ub807\ub2e4\uba74 \uadf8\uac8c \ubb34\uc2a8 \ub3d9\ub9f9\uc774\ub0d0\"\ub294 \ubc1c\uc5b8\uc744 \ud558\uc600\ub2e4. \uc774\ub294 '\uc120 \ubd81\ud575\ub3d9\uacb0, \ud6c4 \uc644\uc804\ud55c \ube44\ud575\ud654'\ub97c \uc8fc\ucc3d\ud55c \ubb38\uc7ac\uc778 \uc815\ubd80\uc758 \uacf5\uc57d\uc5d0\uc11c \ud06c\uac8c \ubc97\uc5b4\ub098\uc9c0 \uc54a\uc9c0\ub9cc \ud55c\ubbf8\uc815\uc0c1\ud68c\ub2f4\uc744 \uc55e\ub450\ub85c \ud55c\ubbf8 \uac04 \uc774\uacac\uc73c\ub85c \ubc88\uc9c0\uba74\uc11c \ub17c\ub780\uc744 \uc77c\uc73c\ucf30\ub2e4. \uc774\uc5d0 \uad6d\ubc29\ubd80\ub294 \"\ud559\uc790 \uac1c\uc778\uc801 \uacac\ud574\uc784\uc744 \uc804\uc81c\ub85c \uc774\ub904\uc9c4 \uac83\uc774\uae30 \ub54c\ubb38\uc5d0 \uc815\ubd80\uc640 \uc870\uc728\ub41c \uc785\uc7a5\uc740 \uc544\ub2cc \uac83\uc73c\ub85c \uc54c\uace0 \uc788\ub2e4\"\uace0 \ud588\uc73c\uba70, \uccad\uc640\ub300\ub3c4 \"\ubb38 \ud2b9\ubcf4 \ubc1c\uc5b8\uc740 \uac1c\uc778 \uc544\uc774\ub514\uc5b4\"\ub77c\uba70 \uc0ac\uc804 \uc870\uc728\uc774 \uc5c6\uc5c8\uc74c\uc744 \uac15\uc870\ud588\ub2e4. \ubbf8\uad6d \uad6d\ubb34\ubd80\ub3c4 \"\uc6b0\ub9ac\ub294 \uc774\ub7f0 \uc2dc\uac01\uc774 \ubb38 \ud2b9\ubcf4\uc758 \uac1c\uc778\uc801 \uacac\ud574\ub85c \ud55c\uad6d \uc815\ubd80\uc758 \uacf5\uc2dd\uc801\uc778 \uc815\ucc45\uc744 \ubc18\uc601\ud55c \uac8c \uc544\ub2d0 \uc218 \uc788\ub294 \uac83\uc73c\ub85c \uc774\ud574\ud558\uace0 \uc788\ub2e4\"\ub294 \ubc18\uc751\uc744 \ubcf4\uc600\ub2e4. \uc774\ud6c4 20\uc77c\uc5d0\ub294 \ubd81\ud55c\uc5d0 1\ub144 5\uac1c\uc6d4\uac00\ub7c9 \uc5b5\ub958\ub418\uc5b4 \uc788\ub2e4\uac00 \ud574\ubc29\ub41c \uc624\ud1a0 \uc6dc\ube44\uc5b4 \uac00 \ud63c\uc218\uc0c1\ud0dc\ub85c \ub3cc\uc544\uc628 \uc9c0 \uc77c\uc8fc\uc77c \ub9cc\uc5d0 \uc0ac\ub9dd\ud558\uba74\uc11c \ub610 \ub2e4\ub978 \ubb38\uc81c\ub97c \ub9cc\ub4e4\uc5c8\ub2e4. \ud558\uc9c0\ub9cc \uc815\uc0c1\ud68c\ub2f4\uc744 \uc55e\ub450\uace0 \uc5ec\ub7ec \uc554\ucd08\ub85c \ud55c\ubbf8 \uac08\ub4f1\uc774 \ud45c\uba74\ud654\ub418\ub294 \uc6b0\ub824\uc5d0 \ub300\ud574 \uccad\uc640\ub300\ub294 '\uc678\uad50\uc548\ubcf4\ub77c\uc778\uc774 \uacf5\uace0\ud558\uae30 \ub54c\ubb38\uc5d0 \uc0ac\ub4dc \ub17c\ub780\uc740 \uc624\ud574 \uc5c6\uc774 \uc798 \ud480\uc5c8\uace0, \ubb38 \ub300\ud1b5\ub839\uc774 \ubd81\ud55c \uc778\uad8c\ubb38\uc81c\uc5d0 \ub300\ud574 \ub2e8\ud638\ud55c \ud0dc\ub3c4\ub97c \ubc1d\ud78c \ub9cc\ud07c \ubbf8\uad6d\uacfc \ubd88\ud611\ud654\uc74c\uc774 \uc0dd\uae38 \uc5ec\uc9c0\uac00 \uc5c6\ub2e4'\ub294 \uc785\uc7a5\uc744 \ubcf4\uc600\ub2e4. \ub2e4\ub9cc, \uc0c8 \uc815\ubd80\uc758 \ub300\ubd81\uc811\uadfc\ubc95\uc774 \ud070 \ud2c0\uc5d0\uc11c \uc774\uc804 \uc815\uad8c\uc5d0 \ub2e4\ub97c \uc218 \uc788\ub2e4\ub294 \uc810\uc740 \ubbf8\uad6d\ub3c4 \uc778\uc815\ud574\uc57c \ud55c\ub2e4\uace0 \ub367\ubd99\uc600\ub2e4.", "sentence_answer": "\uc774\ud6c4 20\uc77c\uc5d0\ub294 \ubd81\ud55c\uc5d0 1\ub144 5\uac1c\uc6d4\uac00\ub7c9 \uc5b5\ub958\ub418\uc5b4 \uc788\ub2e4\uac00 \ud574\ubc29\ub41c \uc624\ud1a0 \uc6dc\ube44\uc5b4 \uac00 \ud63c\uc218\uc0c1\ud0dc\ub85c \ub3cc\uc544\uc628 \uc9c0 \uc77c\uc8fc\uc77c \ub9cc\uc5d0 \uc0ac\ub9dd\ud558\uba74\uc11c \ub610 \ub2e4\ub978 \ubb38\uc81c\ub97c \ub9cc\ub4e4\uc5c8\ub2e4.", "paragraph_id": "6546926-4-1", "paragraph_question": "question: 17\uac1c\uc6d4 \uac00\ub7c9 \ubd81\ud55c\uc5d0 \uac15\uc81c \uc5b5\ub958 \ub418\uc5c8\ub2e4 \ud574\ubc29\ub41c \ubbf8\uad6d\uc778\uc740 \ub204\uad6c\uc778\uac00?, context: \uc774\ub97c \uc704\ud574 \ud55c\ubbf8 \uc815\uc0c1\ud68c\ub2f4\uc758 \uc870\uae30 \uc131\uc0ac\uac00 \ucd94\uc9c4\ub418\uc5c8\ub2e4. \uac00\uc7a5 \uc911\uc694\ud55c \uc7c1\uc810\uc740 \uc0ac\ub4dc \ubc30\uce58\uc5d0 \uad00\ud55c \uac83\uc774\uc5c8\ub294\ub370, 6\uc6d4 9\uc77c \uc815\uc758\uc6a9 \uad6d\uac00\uc548\ubcf4\uc2e4\uc7a5\uc740 \"\uc0ac\ub4dc\ub294 \ubd81\ud55c\uc758 \uc810\uc99d\ud558\ub294 \uc704\ud611\uc73c\ub85c\ubd80\ud130 \ud55c\uad6d\uacfc \uc8fc\ud55c\ubbf8\uad70\uc744 \ubcf4\ud638\ud558\uae30 \uc704\ud574 \uacb0\uc815\ud55c \uac83\"\uc774\ub77c\uba70 \"\uc815\uad8c\uc774 \uad50\uccb4\ub418\uc5c8\ub2e4\uace0 \ud574\uc11c \uc774 \uacb0\uc815\uc744 \uacb0\ucf54 \uac00\ubccd\uac8c \uc5ec\uae30\uc9c0 \uc54a\uc744 \uac83\uc774\uba70 \ubbf8\uad6d\uacfc \uacc4\uc18d \uae34\ubc00\ud788 \ud611\uc758\ud560 \uac83\"\uc774\ub77c\uace0 \ubc1d\ud788\uba74\uc11c \"\uc815\ubd80\ub294 \ud55c\ubbf8\ub3d9\ub9f9 \ucc28\uc6d0\uc5d0\uc11c \uc57d\uc18d\ud55c \ub0b4\uc6a9\uc744 \uadfc\ubcf8\uc801\uc73c\ub85c \ubc14\uafb8\ub824\ub294 \uc758\ub3c4\ub294 \uc5c6\ub2e4\"\uace0 \uac15\uc870\ud588\ub2e4. \ub2e4\ub9cc, \ubbfc\uc8fc\uc801\u00b7\uc808\ucc28\uc801 \uc815\ub2f9\uc131\uc744 \uc704\ud574 \ud658\uacbd\uc601\ud5a5\ud3c9\uac00\uc758 \ud544\uc694\uc131\ub3c4 \uc5b8\uae09\ud588\ub2e4. \uc774\uc5d0 \ub300\ud574 \ubbf8\uad6d \ub0b4\uc5d0\uc11c\ub294 \"\uc0ac\ub4dc\uc758 \uc644\uc804\ud55c \ubc30\uce58\uc640 \uad00\ub828\ud55c \uc5b4\ub5a4 \ud658\uacbd\uc801 \uc6b0\ub824\ub3c4 \uc2e0\uc18d\ud558\uace0 \ucca0\uc800\ud55c \uac80\ud1a0\ub97c \ud1b5\ud574 \ud574\uc18c\ub418\uae38 \ubc14\ub780\ub2e4\"\uba74\uc11c\ub3c4 \ud658\uacbd\uc601\ud5a5\ud3c9\uac00\ub97c \uc774\ud574\ud558\uc9c0 \ubabb\ud558\uaca0\ub2e4\ub294 \uc758\uacac\uc774 \ub098\uc624\uae30\ub3c4 \ud588\ub2e4. 16\uc77c\uc5d0\ub294 \ubbf8\uad6d\uc744 \ubc29\ubb38 \uc911\uc778 \ubb38\uc815\uc778 \ud1b5\uc77c\uc678\uad50\uc548\ubcf4\ud2b9\ubcc4\ubcf4\uc88c\uad00\uc774 \"\ubd81\ud55c\uc774 \ud575 \ubbf8\uc0ac\uc77c \ud65c\ub3d9\uc744 \uc911\ub2e8\ud558\uba74 \ubbf8\uad6d\uc758 \ud55c\ubc18\ub3c4 \uc804\ub7b5\uc790\uc0b0\uacfc \ud55c\ubbf8\ud569\ub3d9\uad70\uc0ac\ud6c8\ub828\uc744 \ucd95\uc18c\ud560 \uc218 \uc788\ub2e4\"\uace0 \ubc1d\ud788\uace0 \ud559\uc790\ub85c\uc11c\uc758 \uacac\ud574\uc784\uc744 \uc804\uc81c\ud55c \ub4a4 \"\uc0ac\ub4dc \ubb38\uc81c\uac00 \ud574\uacb0\ub418\uc9c0 \uc54a\uc73c\uba74 \ud55c\ubbf8\ub3d9\ub9f9\uc774 \uae68\uc9c4\ub2e4\ub294 \uc778\uc2dd\uc774 \uc788\ub294\ub370 \uadf8\ub807\ub2e4\uba74 \uadf8\uac8c \ubb34\uc2a8 \ub3d9\ub9f9\uc774\ub0d0\"\ub294 \ubc1c\uc5b8\uc744 \ud558\uc600\ub2e4. \uc774\ub294 '\uc120 \ubd81\ud575\ub3d9\uacb0, \ud6c4 \uc644\uc804\ud55c \ube44\ud575\ud654'\ub97c \uc8fc\ucc3d\ud55c \ubb38\uc7ac\uc778 \uc815\ubd80\uc758 \uacf5\uc57d\uc5d0\uc11c \ud06c\uac8c \ubc97\uc5b4\ub098\uc9c0 \uc54a\uc9c0\ub9cc \ud55c\ubbf8\uc815\uc0c1\ud68c\ub2f4\uc744 \uc55e\ub450\ub85c \ud55c\ubbf8 \uac04 \uc774\uacac\uc73c\ub85c \ubc88\uc9c0\uba74\uc11c \ub17c\ub780\uc744 \uc77c\uc73c\ucf30\ub2e4. \uc774\uc5d0 \uad6d\ubc29\ubd80\ub294 \"\ud559\uc790 \uac1c\uc778\uc801 \uacac\ud574\uc784\uc744 \uc804\uc81c\ub85c \uc774\ub904\uc9c4 \uac83\uc774\uae30 \ub54c\ubb38\uc5d0 \uc815\ubd80\uc640 \uc870\uc728\ub41c \uc785\uc7a5\uc740 \uc544\ub2cc \uac83\uc73c\ub85c \uc54c\uace0 \uc788\ub2e4\"\uace0 \ud588\uc73c\uba70, \uccad\uc640\ub300\ub3c4 \"\ubb38 \ud2b9\ubcf4 \ubc1c\uc5b8\uc740 \uac1c\uc778 \uc544\uc774\ub514\uc5b4\"\ub77c\uba70 \uc0ac\uc804 \uc870\uc728\uc774 \uc5c6\uc5c8\uc74c\uc744 \uac15\uc870\ud588\ub2e4. \ubbf8\uad6d \uad6d\ubb34\ubd80\ub3c4 \"\uc6b0\ub9ac\ub294 \uc774\ub7f0 \uc2dc\uac01\uc774 \ubb38 \ud2b9\ubcf4\uc758 \uac1c\uc778\uc801 \uacac\ud574\ub85c \ud55c\uad6d \uc815\ubd80\uc758 \uacf5\uc2dd\uc801\uc778 \uc815\ucc45\uc744 \ubc18\uc601\ud55c \uac8c \uc544\ub2d0 \uc218 \uc788\ub294 \uac83\uc73c\ub85c \uc774\ud574\ud558\uace0 \uc788\ub2e4\"\ub294 \ubc18\uc751\uc744 \ubcf4\uc600\ub2e4. \uc774\ud6c4 20\uc77c\uc5d0\ub294 \ubd81\ud55c\uc5d0 1\ub144 5\uac1c\uc6d4\uac00\ub7c9 \uc5b5\ub958\ub418\uc5b4 \uc788\ub2e4\uac00 \ud574\ubc29\ub41c \uc624\ud1a0 \uc6dc\ube44\uc5b4\uac00 \ud63c\uc218\uc0c1\ud0dc\ub85c \ub3cc\uc544\uc628 \uc9c0 \uc77c\uc8fc\uc77c \ub9cc\uc5d0 \uc0ac\ub9dd\ud558\uba74\uc11c \ub610 \ub2e4\ub978 \ubb38\uc81c\ub97c \ub9cc\ub4e4\uc5c8\ub2e4. \ud558\uc9c0\ub9cc \uc815\uc0c1\ud68c\ub2f4\uc744 \uc55e\ub450\uace0 \uc5ec\ub7ec \uc554\ucd08\ub85c \ud55c\ubbf8 \uac08\ub4f1\uc774 \ud45c\uba74\ud654\ub418\ub294 \uc6b0\ub824\uc5d0 \ub300\ud574 \uccad\uc640\ub300\ub294 '\uc678\uad50\uc548\ubcf4\ub77c\uc778\uc774 \uacf5\uace0\ud558\uae30 \ub54c\ubb38\uc5d0 \uc0ac\ub4dc \ub17c\ub780\uc740 \uc624\ud574 \uc5c6\uc774 \uc798 \ud480\uc5c8\uace0, \ubb38 \ub300\ud1b5\ub839\uc774 \ubd81\ud55c \uc778\uad8c\ubb38\uc81c\uc5d0 \ub300\ud574 \ub2e8\ud638\ud55c \ud0dc\ub3c4\ub97c \ubc1d\ud78c \ub9cc\ud07c \ubbf8\uad6d\uacfc \ubd88\ud611\ud654\uc74c\uc774 \uc0dd\uae38 \uc5ec\uc9c0\uac00 \uc5c6\ub2e4'\ub294 \uc785\uc7a5\uc744 \ubcf4\uc600\ub2e4. \ub2e4\ub9cc, \uc0c8 \uc815\ubd80\uc758 \ub300\ubd81\uc811\uadfc\ubc95\uc774 \ud070 \ud2c0\uc5d0\uc11c \uc774\uc804 \uc815\uad8c\uc5d0 \ub2e4\ub97c \uc218 \uc788\ub2e4\ub294 \uc810\uc740 \ubbf8\uad6d\ub3c4 \uc778\uc815\ud574\uc57c \ud55c\ub2e4\uace0 \ub367\ubd99\uc600\ub2e4."} {"question": "\ud3ec\uc640 \uacb0\ud63c\ud55c \uc0ac\ub78c\uc740 \ub204\uad6c\uc778\uac00", "paragraph": "\ud3ec\ub294 \uc0b0\ubb38\ubb38\ud559\uc73c\ub85c \uad00\uc2ec\uc0ac\ub97c \uc62e\uaca8 \uadf8 \ub4a4 \uba87 \ub144 \ub3d9\uc548 \ubb38\ud559\ud559\uc220\uc9c0 \ubc0f \uc815\uae30\uac04\ud589\ubb3c\uc5d0 \uae00\uc744 \uae30\uace0\ud588\uace0, \ud2b9\uc720\uc758 \ubb38\ud559\ube44\ud3c9\uc73c\ub85c \uc774\ub984\uc744 \uc54c\ub9ac\uac8c \ub418\uc5c8\ub2e4. \uc9c1\uc5c5 \uad00\uacc4\ub85c \ud3ec\ub294 \ubcfc\ud2f0\ubaa8\uc5b4\u00b7\ud544\ub77c\ub378\ud53c\uc544\u00b7\ub274\uc695 \uc2dc \ub4f1\uc9c0\ub97c \uc804\uc804\ud588\ub2e4. 1835\ub144 \ubcfc\ud2f0\ubaa8\uc5b4\uc5d0\uc11c 13\uc0b4\uc9dc\ub9ac \uc0ac\ucd0c \ubc84\uc9c0\ub2c8\uc544 \ud074\ub818\uacfc \uacb0\ud63c\ud588\ub2e4. 1845\ub144 \ud3ec\ub294 \uc2dc\uc9d1 \u300a\ub3c4\ub798\uae4c\ub9c8\uadc0\u300b\ub97c \ucd9c\uac04\ud574 \uc77c\uc2dc\uc801\uc778 \uc131\uacf5\uc744 \uac70\ub450\uc5c8\ub2e4. 2\ub144 \ub4a4 \uc544\ub0b4 \ubc84\uc9c0\ub2c8\uc544\uac00 \ud3d0\uacb0\ud575\uc73c\ub85c \uc8fd\uc5c8\ub2e4. \uadf8 \ub4a4 \uba87 \ub144 \ub3d9\uc548 \ud3ec\ub294 \u300a\ud39c\u300b\uc774\ub77c\ub294 \uc774\ub984\uc758 \uc790\uae30 \ubb38\ud559\uc9c0\ub97c \ub9cc\ub4e4\uae30\ub85c \uacc4\ud68d\ud588\uc73c\ub098(\uc774\ud6c4 \uc81c\ubaa9\uc740 \u300a\uc2a4\ud0c0\uc77c\ub7ec\uc2a4\u300b\ub85c \ubcc0\uacbd), \uacc4\ud68d\uc744 \uc2e4\ud604\uc73c\ub85c \uc62e\uae30\uae30 \uc804\uc5d0 \uc0ac\ub9dd\ud588\ub2e4. 1849\ub144 10\uc6d4 7\uc77c, \ud5a5\ub144 40\uc138\uc600\ub2e4. \ud3ec\ub294 \ubcfc\ud2f0\ubaa8\uc5b4\uc5d0\uc11c \uc8fd\uc5c8\ub294\ub370, \uadf8 \uc0ac\uc778\uc774 \ubd88\ubd84\uba85\ud558\ub2e4. \uc54c\ucf54\uc62c\u00b7\ub1cc\ub0b4\ucd9c\ud608\u00b7\ucf5c\ub808\ub77c\u00b7\ub9c8\uc57d\u00b7\uc2ec\uc7a5\ubcd1\u00b7\uad11\uacac\ubcd1\u00b7\uc790\uc0b4\u00b7\ud3d0\uacb0\ud575 \ub4f1 \ub2e4\uc591\ud55c \uc6d0\uc778\uc774 \uac70\ub860\ub418\uace0 \uc788\ub2e4.", "answer": "\ubc84\uc9c0\ub2c8\uc544 \ud074\ub818", "sentence": "1835\ub144 \ubcfc\ud2f0\ubaa8\uc5b4\uc5d0\uc11c 13\uc0b4\uc9dc\ub9ac \uc0ac\ucd0c \ubc84\uc9c0\ub2c8\uc544 \ud074\ub818 \uacfc \uacb0\ud63c\ud588\ub2e4.", "paragraph_sentence": "\ud3ec\ub294 \uc0b0\ubb38\ubb38\ud559\uc73c\ub85c \uad00\uc2ec\uc0ac\ub97c \uc62e\uaca8 \uadf8 \ub4a4 \uba87 \ub144 \ub3d9\uc548 \ubb38\ud559\ud559\uc220\uc9c0 \ubc0f \uc815\uae30\uac04\ud589\ubb3c\uc5d0 \uae00\uc744 \uae30\uace0\ud588\uace0, \ud2b9\uc720\uc758 \ubb38\ud559\ube44\ud3c9\uc73c\ub85c \uc774\ub984\uc744 \uc54c\ub9ac\uac8c \ub418\uc5c8\ub2e4. \uc9c1\uc5c5 \uad00\uacc4\ub85c \ud3ec\ub294 \ubcfc\ud2f0\ubaa8\uc5b4\u00b7\ud544\ub77c\ub378\ud53c\uc544\u00b7\ub274\uc695 \uc2dc \ub4f1\uc9c0\ub97c \uc804\uc804\ud588\ub2e4. 1835\ub144 \ubcfc\ud2f0\ubaa8\uc5b4\uc5d0\uc11c 13\uc0b4\uc9dc\ub9ac \uc0ac\ucd0c \ubc84\uc9c0\ub2c8\uc544 \ud074\ub818 \uacfc \uacb0\ud63c\ud588\ub2e4. 1845\ub144 \ud3ec\ub294 \uc2dc\uc9d1 \u300a\ub3c4\ub798\uae4c\ub9c8\uadc0\u300b\ub97c \ucd9c\uac04\ud574 \uc77c\uc2dc\uc801\uc778 \uc131\uacf5\uc744 \uac70\ub450\uc5c8\ub2e4. 2\ub144 \ub4a4 \uc544\ub0b4 \ubc84\uc9c0\ub2c8\uc544\uac00 \ud3d0\uacb0\ud575\uc73c\ub85c \uc8fd\uc5c8\ub2e4. \uadf8 \ub4a4 \uba87 \ub144 \ub3d9\uc548 \ud3ec\ub294 \u300a\ud39c\u300b\uc774\ub77c\ub294 \uc774\ub984\uc758 \uc790\uae30 \ubb38\ud559\uc9c0\ub97c \ub9cc\ub4e4\uae30\ub85c \uacc4\ud68d\ud588\uc73c\ub098(\uc774\ud6c4 \uc81c\ubaa9\uc740 \u300a\uc2a4\ud0c0\uc77c\ub7ec\uc2a4\u300b\ub85c \ubcc0\uacbd), \uacc4\ud68d\uc744 \uc2e4\ud604\uc73c\ub85c \uc62e\uae30\uae30 \uc804\uc5d0 \uc0ac\ub9dd\ud588\ub2e4. 1849\ub144 10\uc6d4 7\uc77c, \ud5a5\ub144 40\uc138\uc600\ub2e4. \ud3ec\ub294 \ubcfc\ud2f0\ubaa8\uc5b4\uc5d0\uc11c \uc8fd\uc5c8\ub294\ub370, \uadf8 \uc0ac\uc778\uc774 \ubd88\ubd84\uba85\ud558\ub2e4. \uc54c\ucf54\uc62c\u00b7\ub1cc\ub0b4\ucd9c\ud608\u00b7\ucf5c\ub808\ub77c\u00b7\ub9c8\uc57d\u00b7\uc2ec\uc7a5\ubcd1\u00b7\uad11\uacac\ubcd1\u00b7\uc790\uc0b4\u00b7\ud3d0\uacb0\ud575 \ub4f1 \ub2e4\uc591\ud55c \uc6d0\uc778\uc774 \uac70\ub860\ub418\uace0 \uc788\ub2e4.", "paragraph_answer": "\ud3ec\ub294 \uc0b0\ubb38\ubb38\ud559\uc73c\ub85c \uad00\uc2ec\uc0ac\ub97c \uc62e\uaca8 \uadf8 \ub4a4 \uba87 \ub144 \ub3d9\uc548 \ubb38\ud559\ud559\uc220\uc9c0 \ubc0f \uc815\uae30\uac04\ud589\ubb3c\uc5d0 \uae00\uc744 \uae30\uace0\ud588\uace0, \ud2b9\uc720\uc758 \ubb38\ud559\ube44\ud3c9\uc73c\ub85c \uc774\ub984\uc744 \uc54c\ub9ac\uac8c \ub418\uc5c8\ub2e4. \uc9c1\uc5c5 \uad00\uacc4\ub85c \ud3ec\ub294 \ubcfc\ud2f0\ubaa8\uc5b4\u00b7\ud544\ub77c\ub378\ud53c\uc544\u00b7\ub274\uc695 \uc2dc \ub4f1\uc9c0\ub97c \uc804\uc804\ud588\ub2e4. 1835\ub144 \ubcfc\ud2f0\ubaa8\uc5b4\uc5d0\uc11c 13\uc0b4\uc9dc\ub9ac \uc0ac\ucd0c \ubc84\uc9c0\ub2c8\uc544 \ud074\ub818 \uacfc \uacb0\ud63c\ud588\ub2e4. 1845\ub144 \ud3ec\ub294 \uc2dc\uc9d1 \u300a\ub3c4\ub798\uae4c\ub9c8\uadc0\u300b\ub97c \ucd9c\uac04\ud574 \uc77c\uc2dc\uc801\uc778 \uc131\uacf5\uc744 \uac70\ub450\uc5c8\ub2e4. 2\ub144 \ub4a4 \uc544\ub0b4 \ubc84\uc9c0\ub2c8\uc544\uac00 \ud3d0\uacb0\ud575\uc73c\ub85c \uc8fd\uc5c8\ub2e4. \uadf8 \ub4a4 \uba87 \ub144 \ub3d9\uc548 \ud3ec\ub294 \u300a\ud39c\u300b\uc774\ub77c\ub294 \uc774\ub984\uc758 \uc790\uae30 \ubb38\ud559\uc9c0\ub97c \ub9cc\ub4e4\uae30\ub85c \uacc4\ud68d\ud588\uc73c\ub098(\uc774\ud6c4 \uc81c\ubaa9\uc740 \u300a\uc2a4\ud0c0\uc77c\ub7ec\uc2a4\u300b\ub85c \ubcc0\uacbd), \uacc4\ud68d\uc744 \uc2e4\ud604\uc73c\ub85c \uc62e\uae30\uae30 \uc804\uc5d0 \uc0ac\ub9dd\ud588\ub2e4. 1849\ub144 10\uc6d4 7\uc77c, \ud5a5\ub144 40\uc138\uc600\ub2e4. \ud3ec\ub294 \ubcfc\ud2f0\ubaa8\uc5b4\uc5d0\uc11c \uc8fd\uc5c8\ub294\ub370, \uadf8 \uc0ac\uc778\uc774 \ubd88\ubd84\uba85\ud558\ub2e4. \uc54c\ucf54\uc62c\u00b7\ub1cc\ub0b4\ucd9c\ud608\u00b7\ucf5c\ub808\ub77c\u00b7\ub9c8\uc57d\u00b7\uc2ec\uc7a5\ubcd1\u00b7\uad11\uacac\ubcd1\u00b7\uc790\uc0b4\u00b7\ud3d0\uacb0\ud575 \ub4f1 \ub2e4\uc591\ud55c \uc6d0\uc778\uc774 \uac70\ub860\ub418\uace0 \uc788\ub2e4.", "sentence_answer": "1835\ub144 \ubcfc\ud2f0\ubaa8\uc5b4\uc5d0\uc11c 13\uc0b4\uc9dc\ub9ac \uc0ac\ucd0c \ubc84\uc9c0\ub2c8\uc544 \ud074\ub818 \uacfc \uacb0\ud63c\ud588\ub2e4.", "paragraph_id": "6523010-1-2", "paragraph_question": "question: \ud3ec\uc640 \uacb0\ud63c\ud55c \uc0ac\ub78c\uc740 \ub204\uad6c\uc778\uac00, context: \ud3ec\ub294 \uc0b0\ubb38\ubb38\ud559\uc73c\ub85c \uad00\uc2ec\uc0ac\ub97c \uc62e\uaca8 \uadf8 \ub4a4 \uba87 \ub144 \ub3d9\uc548 \ubb38\ud559\ud559\uc220\uc9c0 \ubc0f \uc815\uae30\uac04\ud589\ubb3c\uc5d0 \uae00\uc744 \uae30\uace0\ud588\uace0, \ud2b9\uc720\uc758 \ubb38\ud559\ube44\ud3c9\uc73c\ub85c \uc774\ub984\uc744 \uc54c\ub9ac\uac8c \ub418\uc5c8\ub2e4. \uc9c1\uc5c5 \uad00\uacc4\ub85c \ud3ec\ub294 \ubcfc\ud2f0\ubaa8\uc5b4\u00b7\ud544\ub77c\ub378\ud53c\uc544\u00b7\ub274\uc695 \uc2dc \ub4f1\uc9c0\ub97c \uc804\uc804\ud588\ub2e4. 1835\ub144 \ubcfc\ud2f0\ubaa8\uc5b4\uc5d0\uc11c 13\uc0b4\uc9dc\ub9ac \uc0ac\ucd0c \ubc84\uc9c0\ub2c8\uc544 \ud074\ub818\uacfc \uacb0\ud63c\ud588\ub2e4. 1845\ub144 \ud3ec\ub294 \uc2dc\uc9d1 \u300a\ub3c4\ub798\uae4c\ub9c8\uadc0\u300b\ub97c \ucd9c\uac04\ud574 \uc77c\uc2dc\uc801\uc778 \uc131\uacf5\uc744 \uac70\ub450\uc5c8\ub2e4. 2\ub144 \ub4a4 \uc544\ub0b4 \ubc84\uc9c0\ub2c8\uc544\uac00 \ud3d0\uacb0\ud575\uc73c\ub85c \uc8fd\uc5c8\ub2e4. \uadf8 \ub4a4 \uba87 \ub144 \ub3d9\uc548 \ud3ec\ub294 \u300a\ud39c\u300b\uc774\ub77c\ub294 \uc774\ub984\uc758 \uc790\uae30 \ubb38\ud559\uc9c0\ub97c \ub9cc\ub4e4\uae30\ub85c \uacc4\ud68d\ud588\uc73c\ub098(\uc774\ud6c4 \uc81c\ubaa9\uc740 \u300a\uc2a4\ud0c0\uc77c\ub7ec\uc2a4\u300b\ub85c \ubcc0\uacbd), \uacc4\ud68d\uc744 \uc2e4\ud604\uc73c\ub85c \uc62e\uae30\uae30 \uc804\uc5d0 \uc0ac\ub9dd\ud588\ub2e4. 1849\ub144 10\uc6d4 7\uc77c, \ud5a5\ub144 40\uc138\uc600\ub2e4. \ud3ec\ub294 \ubcfc\ud2f0\ubaa8\uc5b4\uc5d0\uc11c \uc8fd\uc5c8\ub294\ub370, \uadf8 \uc0ac\uc778\uc774 \ubd88\ubd84\uba85\ud558\ub2e4. \uc54c\ucf54\uc62c\u00b7\ub1cc\ub0b4\ucd9c\ud608\u00b7\ucf5c\ub808\ub77c\u00b7\ub9c8\uc57d\u00b7\uc2ec\uc7a5\ubcd1\u00b7\uad11\uacac\ubcd1\u00b7\uc790\uc0b4\u00b7\ud3d0\uacb0\ud575 \ub4f1 \ub2e4\uc591\ud55c \uc6d0\uc778\uc774 \uac70\ub860\ub418\uace0 \uc788\ub2e4."} {"question": "\uc601\uad6d\uad70\uc758 \uc8fc\uacf5\uc744 \ub9e1\uc740 7\uae30\uac11\uc774 \uc0ac\ub2e8\uc774 \uacf5\uaca9\ud55c \uacf3\uc740?", "paragraph": "\uc601\uad6d\uad70\uc758 \uc8fc\uacf5\uc744 \ub9e1\uc740 7 \uae30\uac11 \uc0ac\ub2e8\uc740 \ud568\ubd80\ub974\ud06c\ub97c \uacf5\uaca9\ud558\uace0 \uc774\uacf3\uc5d0\uc11c \uc5d8\ubca0 \uac15\uc73c\ub85c \uc9c4\ucd9c\ud588\uc73c\uba70, 15 \ubcf4\ubcd1\uc0ac\ub2e8\uc740 \ub3c4\uc2dc \ub0a8\ucabd\uc758 \uc6b8\uc820\uc5d0\uc11c \uacf5\uc138\ub97c \uac00\ud588\ub2e4. \uc601\uad6d 8\uad70\ub2e8\uc758 \uc77c\ubd80\ub294 \ud568\ubd80\ub974\ud06c \ubd81\uc11c\ucabd\uc744 \uc9c1\uc811 \uacf5\uaca9\ud588\ub2e4. \ud558\ub974\ubd80\ub974\ud06c\ub85c \uac00\ub294 \uae38\uc5d0 7 \uc0ac\ub2e8\uc740 4\uc6d4 18\uc77c \ubca8\ub808\uc640 \ud1a0\uc2a4\ud14c\ud2b8\ub97c \uc810\ub839\ud588\uace0, \ub2e4\uc74c \ub0a0 \ud640\ub80c\uc2a4\ud14c\ud2b8\ub85c \uc9c4\uaca9\ud588\ub2e4. \uc774\ub54c\uae4c\uc9c0 \ub3c5\uc77c\uad70\uc740 \uac00\uae4c\uc774 \uc9c4\uaca9\ud558\ub294 \uc601\uad6d\uad70\uc744 \ub9c9\uae30 \uc704\ud574 \ubc29\uc5b4\uc120\uc744 \uad6c\ucd95\ud588\ub2e4. 4\uc6d4 20\uc77c, \uc0ac\ub2e8\uc774 \ub370\ub974\uc2a4\ud1a0\ub974\ud504\ub97c \uc810\ub839\ud558\uba74\uc11c \ud568\ubd80\ub974\ud06c \uc11c\ucabd 8\ub9c8\uc77c(12.9km)\uae4c\uc9c0 \ub2e4\uac00\uc654\ub2e4. RHA \uc804\ubc29 \uad00\uce21 \uc7a5\uad50(FOOS)\ub294 \uc5d8\ubca0 \uac15\uc5d0 \ub3c4\ub2ec\ud588\uc73c\uba70, \uad70\ub300\ub97c \ub3cc\ub824 \uac15 \uac74\ub108\ud3b8\uc758 \ud3c9\uc6d0\uc744 \uc810\ub839\ud588\ub2e4. \uac19\uc740 \ub0a0, 131 \ubcf4\ubcd1 \uc5ec\ub2e8\uc740 \ud558\ub974\ubd80\ub974\ud06c \ub0a8\ucabd 2\ub9c8\uc77c(3.2km)\uc758 \ubc14\ud750\ub80c\ub3cc\ud504\ub97c \uc810\ub839\ud588\ub2e4. \uc0ac\ub2e8\uc740 5\uc77c\uac04 \ud568\ubd80\ub974\ud06c\ub85c\uc758 \uc9c4\uaca9\uc744 \uc815\uc9c0\ud588\ub2e4. \uc774 \uc9c4\uaca9 \uc815\uc9c0 \uae30\uac04 \ub3d9\uc548 \ub3c4\uc2dc\uc758 \ubc29\uc5b4\uc120\uc744 \uad6c\ucd95\ud558\uace0 \uacf5\uaca9\uc5d0 \ub300\ube44\ud560 \uc2dc\uac04\uc744 \ubc8c\uc600\ub2e4. \uadf8\ub7ec\ub098, 4\uc6d4 26\uc77c \ud788\ud2c0\ub7ec \uc720\uac90\ud2b8 \uad70\uc778 12 SS \uc5f0\ub300\uc758 \uc9c0\uc6d0\uc744 \ubc1b\uc740 \uc120\uc6d0\uacfc \uacbd\ucc30\uc774 \ubc14\ud750\ub80c\ub3cc\ud504\ub85c \ubc18\uaca9\ud588\ub2e4. \uadf8\ub4e4\uc740 88mm FlaK\uc640 75mm \uace1\uc0ac\ud3ec\uc758 \uc9c0\uc6d0\uc744 \ubc1b\uace0 \uc2dc\ub0b4 \uc911\uc2ec\uac00\uae4c\uc9c0 \uc9c4\ucd9c\ud588\uc73c\ub098 \uc601\uad6d \uc804\ucc28\uc758 \ubc18\uaca9\uc73c\ub85c \ub2e4\uc2dc \ud6c4\ud1f4\ud588\ub2e4. \uc804\ud22c\ub294 \ub2e4\uc74c\ub0a0\uae4c\uc9c0 \uacc4\uc18d\ub418\uc5c8\uace0, \ub3c5\uc77c\uad70\uc740 60\uba85\uc774 \uc0ac\ub9dd\ud558\uace0 70\uba85\uc774 \ud3ec\ub85c\uac00 \ub41c \uc774\ud6c4 \ud558\ub974\ubd80\ub974\ud06c\ub85c \ub2e4\uc2dc \ud6c4\ud1f4\ud588\ub2e4.", "answer": "\ud568\ubd80\ub974\ud06c", "sentence": "\uc601\uad6d\uad70\uc758 \uc8fc\uacf5\uc744 \ub9e1\uc740 7 \uae30\uac11 \uc0ac\ub2e8\uc740 \ud568\ubd80\ub974\ud06c \ub97c \uacf5\uaca9\ud558\uace0 \uc774\uacf3\uc5d0\uc11c \uc5d8\ubca0 \uac15\uc73c\ub85c \uc9c4\ucd9c\ud588\uc73c\uba70, 15 \ubcf4\ubcd1\uc0ac\ub2e8\uc740 \ub3c4\uc2dc \ub0a8\ucabd\uc758 \uc6b8\uc820\uc5d0\uc11c \uacf5\uc138\ub97c \uac00\ud588\ub2e4.", "paragraph_sentence": " \uc601\uad6d\uad70\uc758 \uc8fc\uacf5\uc744 \ub9e1\uc740 7 \uae30\uac11 \uc0ac\ub2e8\uc740 \ud568\ubd80\ub974\ud06c \ub97c \uacf5\uaca9\ud558\uace0 \uc774\uacf3\uc5d0\uc11c \uc5d8\ubca0 \uac15\uc73c\ub85c \uc9c4\ucd9c\ud588\uc73c\uba70, 15 \ubcf4\ubcd1\uc0ac\ub2e8\uc740 \ub3c4\uc2dc \ub0a8\ucabd\uc758 \uc6b8\uc820\uc5d0\uc11c \uacf5\uc138\ub97c \uac00\ud588\ub2e4. \uc601\uad6d 8\uad70\ub2e8\uc758 \uc77c\ubd80\ub294 \ud568\ubd80\ub974\ud06c \ubd81\uc11c\ucabd\uc744 \uc9c1\uc811 \uacf5\uaca9\ud588\ub2e4. \ud558\ub974\ubd80\ub974\ud06c\ub85c \uac00\ub294 \uae38\uc5d0 7 \uc0ac\ub2e8\uc740 4\uc6d4 18\uc77c \ubca8\ub808\uc640 \ud1a0\uc2a4\ud14c\ud2b8\ub97c \uc810\ub839\ud588\uace0, \ub2e4\uc74c \ub0a0 \ud640\ub80c\uc2a4\ud14c\ud2b8\ub85c \uc9c4\uaca9\ud588\ub2e4. \uc774\ub54c\uae4c\uc9c0 \ub3c5\uc77c\uad70\uc740 \uac00\uae4c\uc774 \uc9c4\uaca9\ud558\ub294 \uc601\uad6d\uad70\uc744 \ub9c9\uae30 \uc704\ud574 \ubc29\uc5b4\uc120\uc744 \uad6c\ucd95\ud588\ub2e4. 4\uc6d4 20\uc77c, \uc0ac\ub2e8\uc774 \ub370\ub974\uc2a4\ud1a0\ub974\ud504\ub97c \uc810\ub839\ud558\uba74\uc11c \ud568\ubd80\ub974\ud06c \uc11c\ucabd 8\ub9c8\uc77c(12.9km)\uae4c\uc9c0 \ub2e4\uac00\uc654\ub2e4. RHA \uc804\ubc29 \uad00\uce21 \uc7a5\uad50(FOOS)\ub294 \uc5d8\ubca0 \uac15\uc5d0 \ub3c4\ub2ec\ud588\uc73c\uba70, \uad70\ub300\ub97c \ub3cc\ub824 \uac15 \uac74\ub108\ud3b8\uc758 \ud3c9\uc6d0\uc744 \uc810\ub839\ud588\ub2e4. \uac19\uc740 \ub0a0, 131 \ubcf4\ubcd1 \uc5ec\ub2e8\uc740 \ud558\ub974\ubd80\ub974\ud06c \ub0a8\ucabd 2\ub9c8\uc77c(3.2km)\uc758 \ubc14\ud750\ub80c\ub3cc\ud504\ub97c \uc810\ub839\ud588\ub2e4. \uc0ac\ub2e8\uc740 5\uc77c\uac04 \ud568\ubd80\ub974\ud06c\ub85c\uc758 \uc9c4\uaca9\uc744 \uc815\uc9c0\ud588\ub2e4. \uc774 \uc9c4\uaca9 \uc815\uc9c0 \uae30\uac04 \ub3d9\uc548 \ub3c4\uc2dc\uc758 \ubc29\uc5b4\uc120\uc744 \uad6c\ucd95\ud558\uace0 \uacf5\uaca9\uc5d0 \ub300\ube44\ud560 \uc2dc\uac04\uc744 \ubc8c\uc600\ub2e4. \uadf8\ub7ec\ub098, 4\uc6d4 26\uc77c \ud788\ud2c0\ub7ec \uc720\uac90\ud2b8 \uad70\uc778 12 SS \uc5f0\ub300\uc758 \uc9c0\uc6d0\uc744 \ubc1b\uc740 \uc120\uc6d0\uacfc \uacbd\ucc30\uc774 \ubc14\ud750\ub80c\ub3cc\ud504\ub85c \ubc18\uaca9\ud588\ub2e4. \uadf8\ub4e4\uc740 88mm FlaK\uc640 75mm \uace1\uc0ac\ud3ec\uc758 \uc9c0\uc6d0\uc744 \ubc1b\uace0 \uc2dc\ub0b4 \uc911\uc2ec\uac00\uae4c\uc9c0 \uc9c4\ucd9c\ud588\uc73c\ub098 \uc601\uad6d \uc804\ucc28\uc758 \ubc18\uaca9\uc73c\ub85c \ub2e4\uc2dc \ud6c4\ud1f4\ud588\ub2e4. \uc804\ud22c\ub294 \ub2e4\uc74c\ub0a0\uae4c\uc9c0 \uacc4\uc18d\ub418\uc5c8\uace0, \ub3c5\uc77c\uad70\uc740 60\uba85\uc774 \uc0ac\ub9dd\ud558\uace0 70\uba85\uc774 \ud3ec\ub85c\uac00 \ub41c \uc774\ud6c4 \ud558\ub974\ubd80\ub974\ud06c\ub85c \ub2e4\uc2dc \ud6c4\ud1f4\ud588\ub2e4.", "paragraph_answer": "\uc601\uad6d\uad70\uc758 \uc8fc\uacf5\uc744 \ub9e1\uc740 7 \uae30\uac11 \uc0ac\ub2e8\uc740 \ud568\ubd80\ub974\ud06c \ub97c \uacf5\uaca9\ud558\uace0 \uc774\uacf3\uc5d0\uc11c \uc5d8\ubca0 \uac15\uc73c\ub85c \uc9c4\ucd9c\ud588\uc73c\uba70, 15 \ubcf4\ubcd1\uc0ac\ub2e8\uc740 \ub3c4\uc2dc \ub0a8\ucabd\uc758 \uc6b8\uc820\uc5d0\uc11c \uacf5\uc138\ub97c \uac00\ud588\ub2e4. \uc601\uad6d 8\uad70\ub2e8\uc758 \uc77c\ubd80\ub294 \ud568\ubd80\ub974\ud06c \ubd81\uc11c\ucabd\uc744 \uc9c1\uc811 \uacf5\uaca9\ud588\ub2e4. \ud558\ub974\ubd80\ub974\ud06c\ub85c \uac00\ub294 \uae38\uc5d0 7 \uc0ac\ub2e8\uc740 4\uc6d4 18\uc77c \ubca8\ub808\uc640 \ud1a0\uc2a4\ud14c\ud2b8\ub97c \uc810\ub839\ud588\uace0, \ub2e4\uc74c \ub0a0 \ud640\ub80c\uc2a4\ud14c\ud2b8\ub85c \uc9c4\uaca9\ud588\ub2e4. \uc774\ub54c\uae4c\uc9c0 \ub3c5\uc77c\uad70\uc740 \uac00\uae4c\uc774 \uc9c4\uaca9\ud558\ub294 \uc601\uad6d\uad70\uc744 \ub9c9\uae30 \uc704\ud574 \ubc29\uc5b4\uc120\uc744 \uad6c\ucd95\ud588\ub2e4. 4\uc6d4 20\uc77c, \uc0ac\ub2e8\uc774 \ub370\ub974\uc2a4\ud1a0\ub974\ud504\ub97c \uc810\ub839\ud558\uba74\uc11c \ud568\ubd80\ub974\ud06c \uc11c\ucabd 8\ub9c8\uc77c(12.9km)\uae4c\uc9c0 \ub2e4\uac00\uc654\ub2e4. RHA \uc804\ubc29 \uad00\uce21 \uc7a5\uad50(FOOS)\ub294 \uc5d8\ubca0 \uac15\uc5d0 \ub3c4\ub2ec\ud588\uc73c\uba70, \uad70\ub300\ub97c \ub3cc\ub824 \uac15 \uac74\ub108\ud3b8\uc758 \ud3c9\uc6d0\uc744 \uc810\ub839\ud588\ub2e4. \uac19\uc740 \ub0a0, 131 \ubcf4\ubcd1 \uc5ec\ub2e8\uc740 \ud558\ub974\ubd80\ub974\ud06c \ub0a8\ucabd 2\ub9c8\uc77c(3.2km)\uc758 \ubc14\ud750\ub80c\ub3cc\ud504\ub97c \uc810\ub839\ud588\ub2e4. \uc0ac\ub2e8\uc740 5\uc77c\uac04 \ud568\ubd80\ub974\ud06c\ub85c\uc758 \uc9c4\uaca9\uc744 \uc815\uc9c0\ud588\ub2e4. \uc774 \uc9c4\uaca9 \uc815\uc9c0 \uae30\uac04 \ub3d9\uc548 \ub3c4\uc2dc\uc758 \ubc29\uc5b4\uc120\uc744 \uad6c\ucd95\ud558\uace0 \uacf5\uaca9\uc5d0 \ub300\ube44\ud560 \uc2dc\uac04\uc744 \ubc8c\uc600\ub2e4. \uadf8\ub7ec\ub098, 4\uc6d4 26\uc77c \ud788\ud2c0\ub7ec \uc720\uac90\ud2b8 \uad70\uc778 12 SS \uc5f0\ub300\uc758 \uc9c0\uc6d0\uc744 \ubc1b\uc740 \uc120\uc6d0\uacfc \uacbd\ucc30\uc774 \ubc14\ud750\ub80c\ub3cc\ud504\ub85c \ubc18\uaca9\ud588\ub2e4. \uadf8\ub4e4\uc740 88mm FlaK\uc640 75mm \uace1\uc0ac\ud3ec\uc758 \uc9c0\uc6d0\uc744 \ubc1b\uace0 \uc2dc\ub0b4 \uc911\uc2ec\uac00\uae4c\uc9c0 \uc9c4\ucd9c\ud588\uc73c\ub098 \uc601\uad6d \uc804\ucc28\uc758 \ubc18\uaca9\uc73c\ub85c \ub2e4\uc2dc \ud6c4\ud1f4\ud588\ub2e4. \uc804\ud22c\ub294 \ub2e4\uc74c\ub0a0\uae4c\uc9c0 \uacc4\uc18d\ub418\uc5c8\uace0, \ub3c5\uc77c\uad70\uc740 60\uba85\uc774 \uc0ac\ub9dd\ud558\uace0 70\uba85\uc774 \ud3ec\ub85c\uac00 \ub41c \uc774\ud6c4 \ud558\ub974\ubd80\ub974\ud06c\ub85c \ub2e4\uc2dc \ud6c4\ud1f4\ud588\ub2e4.", "sentence_answer": "\uc601\uad6d\uad70\uc758 \uc8fc\uacf5\uc744 \ub9e1\uc740 7 \uae30\uac11 \uc0ac\ub2e8\uc740 \ud568\ubd80\ub974\ud06c \ub97c \uacf5\uaca9\ud558\uace0 \uc774\uacf3\uc5d0\uc11c \uc5d8\ubca0 \uac15\uc73c\ub85c \uc9c4\ucd9c\ud588\uc73c\uba70, 15 \ubcf4\ubcd1\uc0ac\ub2e8\uc740 \ub3c4\uc2dc \ub0a8\ucabd\uc758 \uc6b8\uc820\uc5d0\uc11c \uacf5\uc138\ub97c \uac00\ud588\ub2e4.", "paragraph_id": "6570227-1-0", "paragraph_question": "question: \uc601\uad6d\uad70\uc758 \uc8fc\uacf5\uc744 \ub9e1\uc740 7\uae30\uac11\uc774 \uc0ac\ub2e8\uc774 \uacf5\uaca9\ud55c \uacf3\uc740?, context: \uc601\uad6d\uad70\uc758 \uc8fc\uacf5\uc744 \ub9e1\uc740 7 \uae30\uac11 \uc0ac\ub2e8\uc740 \ud568\ubd80\ub974\ud06c\ub97c \uacf5\uaca9\ud558\uace0 \uc774\uacf3\uc5d0\uc11c \uc5d8\ubca0 \uac15\uc73c\ub85c \uc9c4\ucd9c\ud588\uc73c\uba70, 15 \ubcf4\ubcd1\uc0ac\ub2e8\uc740 \ub3c4\uc2dc \ub0a8\ucabd\uc758 \uc6b8\uc820\uc5d0\uc11c \uacf5\uc138\ub97c \uac00\ud588\ub2e4. \uc601\uad6d 8\uad70\ub2e8\uc758 \uc77c\ubd80\ub294 \ud568\ubd80\ub974\ud06c \ubd81\uc11c\ucabd\uc744 \uc9c1\uc811 \uacf5\uaca9\ud588\ub2e4. \ud558\ub974\ubd80\ub974\ud06c\ub85c \uac00\ub294 \uae38\uc5d0 7 \uc0ac\ub2e8\uc740 4\uc6d4 18\uc77c \ubca8\ub808\uc640 \ud1a0\uc2a4\ud14c\ud2b8\ub97c \uc810\ub839\ud588\uace0, \ub2e4\uc74c \ub0a0 \ud640\ub80c\uc2a4\ud14c\ud2b8\ub85c \uc9c4\uaca9\ud588\ub2e4. \uc774\ub54c\uae4c\uc9c0 \ub3c5\uc77c\uad70\uc740 \uac00\uae4c\uc774 \uc9c4\uaca9\ud558\ub294 \uc601\uad6d\uad70\uc744 \ub9c9\uae30 \uc704\ud574 \ubc29\uc5b4\uc120\uc744 \uad6c\ucd95\ud588\ub2e4. 4\uc6d4 20\uc77c, \uc0ac\ub2e8\uc774 \ub370\ub974\uc2a4\ud1a0\ub974\ud504\ub97c \uc810\ub839\ud558\uba74\uc11c \ud568\ubd80\ub974\ud06c \uc11c\ucabd 8\ub9c8\uc77c(12.9km)\uae4c\uc9c0 \ub2e4\uac00\uc654\ub2e4. RHA \uc804\ubc29 \uad00\uce21 \uc7a5\uad50(FOOS)\ub294 \uc5d8\ubca0 \uac15\uc5d0 \ub3c4\ub2ec\ud588\uc73c\uba70, \uad70\ub300\ub97c \ub3cc\ub824 \uac15 \uac74\ub108\ud3b8\uc758 \ud3c9\uc6d0\uc744 \uc810\ub839\ud588\ub2e4. \uac19\uc740 \ub0a0, 131 \ubcf4\ubcd1 \uc5ec\ub2e8\uc740 \ud558\ub974\ubd80\ub974\ud06c \ub0a8\ucabd 2\ub9c8\uc77c(3.2km)\uc758 \ubc14\ud750\ub80c\ub3cc\ud504\ub97c \uc810\ub839\ud588\ub2e4. \uc0ac\ub2e8\uc740 5\uc77c\uac04 \ud568\ubd80\ub974\ud06c\ub85c\uc758 \uc9c4\uaca9\uc744 \uc815\uc9c0\ud588\ub2e4. \uc774 \uc9c4\uaca9 \uc815\uc9c0 \uae30\uac04 \ub3d9\uc548 \ub3c4\uc2dc\uc758 \ubc29\uc5b4\uc120\uc744 \uad6c\ucd95\ud558\uace0 \uacf5\uaca9\uc5d0 \ub300\ube44\ud560 \uc2dc\uac04\uc744 \ubc8c\uc600\ub2e4. \uadf8\ub7ec\ub098, 4\uc6d4 26\uc77c \ud788\ud2c0\ub7ec \uc720\uac90\ud2b8 \uad70\uc778 12 SS \uc5f0\ub300\uc758 \uc9c0\uc6d0\uc744 \ubc1b\uc740 \uc120\uc6d0\uacfc \uacbd\ucc30\uc774 \ubc14\ud750\ub80c\ub3cc\ud504\ub85c \ubc18\uaca9\ud588\ub2e4. \uadf8\ub4e4\uc740 88mm FlaK\uc640 75mm \uace1\uc0ac\ud3ec\uc758 \uc9c0\uc6d0\uc744 \ubc1b\uace0 \uc2dc\ub0b4 \uc911\uc2ec\uac00\uae4c\uc9c0 \uc9c4\ucd9c\ud588\uc73c\ub098 \uc601\uad6d \uc804\ucc28\uc758 \ubc18\uaca9\uc73c\ub85c \ub2e4\uc2dc \ud6c4\ud1f4\ud588\ub2e4. \uc804\ud22c\ub294 \ub2e4\uc74c\ub0a0\uae4c\uc9c0 \uacc4\uc18d\ub418\uc5c8\uace0, \ub3c5\uc77c\uad70\uc740 60\uba85\uc774 \uc0ac\ub9dd\ud558\uace0 70\uba85\uc774 \ud3ec\ub85c\uac00 \ub41c \uc774\ud6c4 \ud558\ub974\ubd80\ub974\ud06c\ub85c \ub2e4\uc2dc \ud6c4\ud1f4\ud588\ub2e4."} {"question": "\ub9cc\uc57d \uc138\ub4dc\ub098\uac00 \ud604\uc7ac\uc758 \uc704\uce58\uc5d0\uc11c \ud615\uc131\ub418\uc5c8\uc744 \uacbd\uc6b0 \ucd08\uae30 \uada4\ub3c4\uc758 \ud615\ud0dc\ub294 \ub300\ub7b5 \uc5b4\ub5a4 \ubaa8\uc591\uc774\uc5c8\uc744\uae4c?", "paragraph": "\ub9cc\uc57d \uc138\ub4dc\ub098\uac00 \ud604\uc7ac\uc758 \uc704\uce58\uc5d0\uc11c \ud615\uc131\ub418\uc5c8\ub2e4\uba74 \ud0dc\uc591\uc758 \uc6d0\uc2dc\ud589\uc131\uacc4 \uc6d0\ubc18 \ud06c\uae30\ub294 75 AU \uc774\uc0c1\uc774\uc5c8\ub2e4\ub294 \uc774\uc57c\uae30\uac00 \ub41c\ub2e4. \ub610\ud55c \uc138\ub4dc\ub098\uc758 \ucd08\uae30 \uada4\ub3c4\ub294 \uc6d0\ud615\uc5d0 \uac00\uae4c\uc6e0\uc744 \uac83\uc774\ub2e4. \uadf8\ub807\uc9c0 \uc54a\uc73c\uba74 \ubbf8\ud589\uc131\ub4e4 \uc0ac\uc774\uc758 \ud070 \uc0c1\ub300\uc18d\ub3c4\ub85c \uc778\ud55c \ubc29\ud574\uac00 \uadf9\uc2ec\ud560 \uac83\uc774\uae30\uc5d0 \uc791\uc740 \ucc9c\uccb4\uc758 \uac15\ucc29\uc744 \ud1b5\ud55c \ud070 \ucc9c\uccb4\uc758 \ud615\uc131\uc774\ub77c\ub294 \uacfc\uc815\uc740 \ubd88\uac00\ub2a5\ud558\ub2e4. \uadf8\ub7ec\ubbc0\ub85c \uc138\ub4dc\ub098\ub294 \ub2e4\ub978 \ucc9c\uccb4\uc640\uc758 \uc911\ub825\uc801 \uc0c1\ud638\uc791\uc6a9\uc744 \ud1b5\ud574 \ud604\uc7ac\uc758 \uc9dc\ubd80\ub77c\uc9c4 \uada4\ub3c4\ub97c \uac00\uc9c0\uac8c \ub418\uc5c8\ub2e4\uace0 \ubcf4\uc544\uc57c \ud560 \uac83\uc774\ub2e4. \ube0c\ub77c\uc6b4, \ub77c\ube44\ub178\ube44\uce20 \uc678 \uadf8 \ub3d9\ub8cc\ub4e4\uc740 \ucd5c\ucd08 \ub17c\ubb38\uc5d0\uc11c \uc138\ub4dc\ub098\uc5d0\uac8c \uc12d\ub3d9\uc744 \uc77c\uc73c\ud0a8 \ucc9c\uccb4\uc758 \ud6c4\ubcf4\ub97c \uc138 \uac00\uc9c0\ub85c \ucd94\ub824 \uc81c\uc2dc\ud588\ub2e4. \uccab\uc9f8\ub294 \uce74\uc774\ud37c\ub300 \ub108\uba38\uc758 \ubcf4\uc774\uc9c0 \uc54a\ub294 \ud589\uc131\uc774\uace0, \ub458\uc9f8\ub294 \ud0dc\uc591\uacc4 \uc606\uc744 \uc9c0\ub098\uac00\ub358 \ud56d\uc131\uc774\uba70, \uc14b\uc9f8\ub294 \ud0dc\uc591\uacfc \ud568\uaed8 \ud615\uc131\ub418\uc5b4 \uc0b0\uac1c\uc131\ub2e8\uc774 \ub2e4 \ud769\uc5b4\uc9c0\uae30 \uc804\uc5d0 \uac19\uc774 \uc788\uc5c8\ub358 \uc80a\uc740 \ud56d\uc131\uc774\ub2e4.", "answer": "\uc6d0\ud615", "sentence": "\ub610\ud55c \uc138\ub4dc\ub098\uc758 \ucd08\uae30 \uada4\ub3c4\ub294 \uc6d0\ud615 \uc5d0 \uac00\uae4c\uc6e0\uc744 \uac83\uc774\ub2e4.", "paragraph_sentence": "\ub9cc\uc57d \uc138\ub4dc\ub098\uac00 \ud604\uc7ac\uc758 \uc704\uce58\uc5d0\uc11c \ud615\uc131\ub418\uc5c8\ub2e4\uba74 \ud0dc\uc591\uc758 \uc6d0\uc2dc\ud589\uc131\uacc4 \uc6d0\ubc18 \ud06c\uae30\ub294 75 AU \uc774\uc0c1\uc774\uc5c8\ub2e4\ub294 \uc774\uc57c\uae30\uac00 \ub41c\ub2e4. \ub610\ud55c \uc138\ub4dc\ub098\uc758 \ucd08\uae30 \uada4\ub3c4\ub294 \uc6d0\ud615 \uc5d0 \uac00\uae4c\uc6e0\uc744 \uac83\uc774\ub2e4. \uadf8\ub807\uc9c0 \uc54a\uc73c\uba74 \ubbf8\ud589\uc131\ub4e4 \uc0ac\uc774\uc758 \ud070 \uc0c1\ub300\uc18d\ub3c4\ub85c \uc778\ud55c \ubc29\ud574\uac00 \uadf9\uc2ec\ud560 \uac83\uc774\uae30\uc5d0 \uc791\uc740 \ucc9c\uccb4\uc758 \uac15\ucc29\uc744 \ud1b5\ud55c \ud070 \ucc9c\uccb4\uc758 \ud615\uc131\uc774\ub77c\ub294 \uacfc\uc815\uc740 \ubd88\uac00\ub2a5\ud558\ub2e4. \uadf8\ub7ec\ubbc0\ub85c \uc138\ub4dc\ub098\ub294 \ub2e4\ub978 \ucc9c\uccb4\uc640\uc758 \uc911\ub825\uc801 \uc0c1\ud638\uc791\uc6a9\uc744 \ud1b5\ud574 \ud604\uc7ac\uc758 \uc9dc\ubd80\ub77c\uc9c4 \uada4\ub3c4\ub97c \uac00\uc9c0\uac8c \ub418\uc5c8\ub2e4\uace0 \ubcf4\uc544\uc57c \ud560 \uac83\uc774\ub2e4. \ube0c\ub77c\uc6b4, \ub77c\ube44\ub178\ube44\uce20 \uc678 \uadf8 \ub3d9\ub8cc\ub4e4\uc740 \ucd5c\ucd08 \ub17c\ubb38\uc5d0\uc11c \uc138\ub4dc\ub098\uc5d0\uac8c \uc12d\ub3d9\uc744 \uc77c\uc73c\ud0a8 \ucc9c\uccb4\uc758 \ud6c4\ubcf4\ub97c \uc138 \uac00\uc9c0\ub85c \ucd94\ub824 \uc81c\uc2dc\ud588\ub2e4. \uccab\uc9f8\ub294 \uce74\uc774\ud37c\ub300 \ub108\uba38\uc758 \ubcf4\uc774\uc9c0 \uc54a\ub294 \ud589\uc131\uc774\uace0, \ub458\uc9f8\ub294 \ud0dc\uc591\uacc4 \uc606\uc744 \uc9c0\ub098\uac00\ub358 \ud56d\uc131\uc774\uba70, \uc14b\uc9f8\ub294 \ud0dc\uc591\uacfc \ud568\uaed8 \ud615\uc131\ub418\uc5b4 \uc0b0\uac1c\uc131\ub2e8\uc774 \ub2e4 \ud769\uc5b4\uc9c0\uae30 \uc804\uc5d0 \uac19\uc774 \uc788\uc5c8\ub358 \uc80a\uc740 \ud56d\uc131\uc774\ub2e4.", "paragraph_answer": "\ub9cc\uc57d \uc138\ub4dc\ub098\uac00 \ud604\uc7ac\uc758 \uc704\uce58\uc5d0\uc11c \ud615\uc131\ub418\uc5c8\ub2e4\uba74 \ud0dc\uc591\uc758 \uc6d0\uc2dc\ud589\uc131\uacc4 \uc6d0\ubc18 \ud06c\uae30\ub294 75 AU \uc774\uc0c1\uc774\uc5c8\ub2e4\ub294 \uc774\uc57c\uae30\uac00 \ub41c\ub2e4. \ub610\ud55c \uc138\ub4dc\ub098\uc758 \ucd08\uae30 \uada4\ub3c4\ub294 \uc6d0\ud615 \uc5d0 \uac00\uae4c\uc6e0\uc744 \uac83\uc774\ub2e4. \uadf8\ub807\uc9c0 \uc54a\uc73c\uba74 \ubbf8\ud589\uc131\ub4e4 \uc0ac\uc774\uc758 \ud070 \uc0c1\ub300\uc18d\ub3c4\ub85c \uc778\ud55c \ubc29\ud574\uac00 \uadf9\uc2ec\ud560 \uac83\uc774\uae30\uc5d0 \uc791\uc740 \ucc9c\uccb4\uc758 \uac15\ucc29\uc744 \ud1b5\ud55c \ud070 \ucc9c\uccb4\uc758 \ud615\uc131\uc774\ub77c\ub294 \uacfc\uc815\uc740 \ubd88\uac00\ub2a5\ud558\ub2e4. \uadf8\ub7ec\ubbc0\ub85c \uc138\ub4dc\ub098\ub294 \ub2e4\ub978 \ucc9c\uccb4\uc640\uc758 \uc911\ub825\uc801 \uc0c1\ud638\uc791\uc6a9\uc744 \ud1b5\ud574 \ud604\uc7ac\uc758 \uc9dc\ubd80\ub77c\uc9c4 \uada4\ub3c4\ub97c \uac00\uc9c0\uac8c \ub418\uc5c8\ub2e4\uace0 \ubcf4\uc544\uc57c \ud560 \uac83\uc774\ub2e4. \ube0c\ub77c\uc6b4, \ub77c\ube44\ub178\ube44\uce20 \uc678 \uadf8 \ub3d9\ub8cc\ub4e4\uc740 \ucd5c\ucd08 \ub17c\ubb38\uc5d0\uc11c \uc138\ub4dc\ub098\uc5d0\uac8c \uc12d\ub3d9\uc744 \uc77c\uc73c\ud0a8 \ucc9c\uccb4\uc758 \ud6c4\ubcf4\ub97c \uc138 \uac00\uc9c0\ub85c \ucd94\ub824 \uc81c\uc2dc\ud588\ub2e4. \uccab\uc9f8\ub294 \uce74\uc774\ud37c\ub300 \ub108\uba38\uc758 \ubcf4\uc774\uc9c0 \uc54a\ub294 \ud589\uc131\uc774\uace0, \ub458\uc9f8\ub294 \ud0dc\uc591\uacc4 \uc606\uc744 \uc9c0\ub098\uac00\ub358 \ud56d\uc131\uc774\uba70, \uc14b\uc9f8\ub294 \ud0dc\uc591\uacfc \ud568\uaed8 \ud615\uc131\ub418\uc5b4 \uc0b0\uac1c\uc131\ub2e8\uc774 \ub2e4 \ud769\uc5b4\uc9c0\uae30 \uc804\uc5d0 \uac19\uc774 \uc788\uc5c8\ub358 \uc80a\uc740 \ud56d\uc131\uc774\ub2e4.", "sentence_answer": "\ub610\ud55c \uc138\ub4dc\ub098\uc758 \ucd08\uae30 \uada4\ub3c4\ub294 \uc6d0\ud615 \uc5d0 \uac00\uae4c\uc6e0\uc744 \uac83\uc774\ub2e4.", "paragraph_id": "6568339-7-0", "paragraph_question": "question: \ub9cc\uc57d \uc138\ub4dc\ub098\uac00 \ud604\uc7ac\uc758 \uc704\uce58\uc5d0\uc11c \ud615\uc131\ub418\uc5c8\uc744 \uacbd\uc6b0 \ucd08\uae30 \uada4\ub3c4\uc758 \ud615\ud0dc\ub294 \ub300\ub7b5 \uc5b4\ub5a4 \ubaa8\uc591\uc774\uc5c8\uc744\uae4c?, context: \ub9cc\uc57d \uc138\ub4dc\ub098\uac00 \ud604\uc7ac\uc758 \uc704\uce58\uc5d0\uc11c \ud615\uc131\ub418\uc5c8\ub2e4\uba74 \ud0dc\uc591\uc758 \uc6d0\uc2dc\ud589\uc131\uacc4 \uc6d0\ubc18 \ud06c\uae30\ub294 75 AU \uc774\uc0c1\uc774\uc5c8\ub2e4\ub294 \uc774\uc57c\uae30\uac00 \ub41c\ub2e4. \ub610\ud55c \uc138\ub4dc\ub098\uc758 \ucd08\uae30 \uada4\ub3c4\ub294 \uc6d0\ud615\uc5d0 \uac00\uae4c\uc6e0\uc744 \uac83\uc774\ub2e4. \uadf8\ub807\uc9c0 \uc54a\uc73c\uba74 \ubbf8\ud589\uc131\ub4e4 \uc0ac\uc774\uc758 \ud070 \uc0c1\ub300\uc18d\ub3c4\ub85c \uc778\ud55c \ubc29\ud574\uac00 \uadf9\uc2ec\ud560 \uac83\uc774\uae30\uc5d0 \uc791\uc740 \ucc9c\uccb4\uc758 \uac15\ucc29\uc744 \ud1b5\ud55c \ud070 \ucc9c\uccb4\uc758 \ud615\uc131\uc774\ub77c\ub294 \uacfc\uc815\uc740 \ubd88\uac00\ub2a5\ud558\ub2e4. \uadf8\ub7ec\ubbc0\ub85c \uc138\ub4dc\ub098\ub294 \ub2e4\ub978 \ucc9c\uccb4\uc640\uc758 \uc911\ub825\uc801 \uc0c1\ud638\uc791\uc6a9\uc744 \ud1b5\ud574 \ud604\uc7ac\uc758 \uc9dc\ubd80\ub77c\uc9c4 \uada4\ub3c4\ub97c \uac00\uc9c0\uac8c \ub418\uc5c8\ub2e4\uace0 \ubcf4\uc544\uc57c \ud560 \uac83\uc774\ub2e4. \ube0c\ub77c\uc6b4, \ub77c\ube44\ub178\ube44\uce20 \uc678 \uadf8 \ub3d9\ub8cc\ub4e4\uc740 \ucd5c\ucd08 \ub17c\ubb38\uc5d0\uc11c \uc138\ub4dc\ub098\uc5d0\uac8c \uc12d\ub3d9\uc744 \uc77c\uc73c\ud0a8 \ucc9c\uccb4\uc758 \ud6c4\ubcf4\ub97c \uc138 \uac00\uc9c0\ub85c \ucd94\ub824 \uc81c\uc2dc\ud588\ub2e4. \uccab\uc9f8\ub294 \uce74\uc774\ud37c\ub300 \ub108\uba38\uc758 \ubcf4\uc774\uc9c0 \uc54a\ub294 \ud589\uc131\uc774\uace0, \ub458\uc9f8\ub294 \ud0dc\uc591\uacc4 \uc606\uc744 \uc9c0\ub098\uac00\ub358 \ud56d\uc131\uc774\uba70, \uc14b\uc9f8\ub294 \ud0dc\uc591\uacfc \ud568\uaed8 \ud615\uc131\ub418\uc5b4 \uc0b0\uac1c\uc131\ub2e8\uc774 \ub2e4 \ud769\uc5b4\uc9c0\uae30 \uc804\uc5d0 \uac19\uc774 \uc788\uc5c8\ub358 \uc80a\uc740 \ud56d\uc131\uc774\ub2e4."} {"question": "\ud5c8\ub9ac\ucf00\uc778 \uae38\ubc84\ud2b8\uc5d0 \uc758\ud574 \uce89\ucfe4 \uc9c0\ubc29\uc5d0\uc11c\ub294 \uc5bc\ub9c8\uc758 \ud53c\ud574\uc561\uc774 \ubc1c\uc0dd\ub418\uc5c8\ub098?", "paragraph": "\ud5c8\ub9ac\ucf00\uc778 \uae38\ubc84\ud2b8\uc758 \ucd1d \uc0ac\ub9dd\uc790 433\uba85 \uc911\uc5d0 240\uba85\uc740 \uba55\uc2dc\ucf54\uc5d0\uc11c \ub098\uc654\uc744 \ub9cc\ud07c \uba55\uc2dc\ucf54\uc5d0\uc11c\ub294 \ud53c\ud574\uac00 \uadf9\uc2ec\ud558\uc600\ub2e4. 3\ub9cc5\ucc9c\uba85\uc758 \uc774\uc7ac\ubbfc\uc774 \uc0dd\uacbc\uace0 \uae38\ubc84\ud2b8\uac00 \uc720\uce74\ud0c4 \ubc18\ub3c4\ub97c \ub36e\ucce4\uc744 \ub54c, 83\ucc99\uc758 \ubc30\ub4e4\uc774 \uac00\ub77c\uc549\uc558\uace0 6\ub9cc\ucc44\uc758 \uc9d1\uc774 \ud30c\uad34, \ucd1d \uc7ac\uc0b0\ud53c\ud574\uc561\uc740 10\uc5b5\uc5d0\uc11c 20\uc5b5\ub2ec\ub7ec(1989\ub144 \uae30\uc900)\uc73c\ub85c \ucd94\uc0b0\ub418\uc5c8\ub2e4. \uce89\ucfe4 \uc9c0\ubc29\uc5d0\uc11c\ub294 8\ucc9c7\ubc31\ub9cc\ub2ec\ub7ec(1989\ub144 \uae30\uc900)\uc758 \ud53c\ud574\uc561\uc774 \uc9d1\uacc4\ub418\uc5c8\uace0 \ud5c8\ub9ac\ucf00\uc778\uc758 \ud53c\ud574\ub85c \uc778\ud574 \ub3c4\uc2dc\uc758 \uad00\uad11\uc0b0\uc5c5\uc5d0 \uce58\uba85\uc801\uc778 \ud0c0\uaca9\uc744 \uc8fc\uc5c8\ub2e4. \uc720\uce74\ud0c4 \ubc18\ub3c4\uc5d0 \uc704\uce58\ud55c \ud504\ub85c\uadf8\ub808\uc18c\ub77c\ub294 \ub3c4\uc2dc\uc5d0\uc11c\ub294 350mm\uac00 \ub118\ub294 \ube44\uac00 \uc3df\uc544\uc84c\uae30\ub3c4 \ud558\uc600\ub2e4. \uba55\uc2dc\ucf54 \ubd81\ub3d9\ubd80\uc5d0\uc11c\ub294 \ubaac\ud14c\ub808\uc774 \uc8fc\ubcc0\uc5d0 \ud3ed\uc6b0\uac00 \ub0b4\ub824 \uc9c0\uc5ed\uc774 \ubb3c\uc5d0 \uc7a0\uacbc\ub2e4. 100\uba85 \uc774\uc0c1\uc758 \uc0ac\ub78c\ub4e4\uc774 \uc228\uc84c\uace0 \ub300\ud53c\uc790\ub97c \ub098\ub974\ub358 5\ub300\uc758 \ubc84\uc2a4\uac00 \ub4a4\uc9d1\ud614\ub2e4. \ud0c0\ub9c8\uc6b8\ub9ac\ud30c\uc2a4 \uc8fc \ub0b4\ub959\uc5d0\uc11c\ub294 250mm\uac00 \ub118\ub294 \uac15\uc218\ub7c9\uc744 \uae30\ub85d\ud558\uc600\ub2e4.", "answer": "8\ucc9c7\ubc31\ub9cc\ub2ec\ub7ec", "sentence": "\uce89\ucfe4 \uc9c0\ubc29\uc5d0\uc11c\ub294 8\ucc9c7\ubc31\ub9cc\ub2ec\ub7ec (1989\ub144 \uae30\uc900)\uc758 \ud53c\ud574\uc561\uc774 \uc9d1\uacc4\ub418\uc5c8\uace0 \ud5c8\ub9ac\ucf00\uc778\uc758 \ud53c\ud574\ub85c \uc778\ud574 \ub3c4\uc2dc\uc758 \uad00\uad11\uc0b0\uc5c5\uc5d0 \uce58\uba85\uc801\uc778 \ud0c0\uaca9\uc744 \uc8fc\uc5c8\ub2e4.", "paragraph_sentence": "\ud5c8\ub9ac\ucf00\uc778 \uae38\ubc84\ud2b8\uc758 \ucd1d \uc0ac\ub9dd\uc790 433\uba85 \uc911\uc5d0 240\uba85\uc740 \uba55\uc2dc\ucf54\uc5d0\uc11c \ub098\uc654\uc744 \ub9cc\ud07c \uba55\uc2dc\ucf54\uc5d0\uc11c\ub294 \ud53c\ud574\uac00 \uadf9\uc2ec\ud558\uc600\ub2e4. 3\ub9cc5\ucc9c\uba85\uc758 \uc774\uc7ac\ubbfc\uc774 \uc0dd\uacbc\uace0 \uae38\ubc84\ud2b8\uac00 \uc720\uce74\ud0c4 \ubc18\ub3c4\ub97c \ub36e\ucce4\uc744 \ub54c, 83\ucc99\uc758 \ubc30\ub4e4\uc774 \uac00\ub77c\uc549\uc558\uace0 6\ub9cc\ucc44\uc758 \uc9d1\uc774 \ud30c\uad34, \ucd1d \uc7ac\uc0b0\ud53c\ud574\uc561\uc740 10\uc5b5\uc5d0\uc11c 20\uc5b5\ub2ec\ub7ec(1989\ub144 \uae30\uc900)\uc73c\ub85c \ucd94\uc0b0\ub418\uc5c8\ub2e4. \uce89\ucfe4 \uc9c0\ubc29\uc5d0\uc11c\ub294 8\ucc9c7\ubc31\ub9cc\ub2ec\ub7ec (1989\ub144 \uae30\uc900)\uc758 \ud53c\ud574\uc561\uc774 \uc9d1\uacc4\ub418\uc5c8\uace0 \ud5c8\ub9ac\ucf00\uc778\uc758 \ud53c\ud574\ub85c \uc778\ud574 \ub3c4\uc2dc\uc758 \uad00\uad11\uc0b0\uc5c5\uc5d0 \uce58\uba85\uc801\uc778 \ud0c0\uaca9\uc744 \uc8fc\uc5c8\ub2e4. \uc720\uce74\ud0c4 \ubc18\ub3c4\uc5d0 \uc704\uce58\ud55c \ud504\ub85c\uadf8\ub808\uc18c\ub77c\ub294 \ub3c4\uc2dc\uc5d0\uc11c\ub294 350mm\uac00 \ub118\ub294 \ube44\uac00 \uc3df\uc544\uc84c\uae30\ub3c4 \ud558\uc600\ub2e4. \uba55\uc2dc\ucf54 \ubd81\ub3d9\ubd80\uc5d0\uc11c\ub294 \ubaac\ud14c\ub808\uc774 \uc8fc\ubcc0\uc5d0 \ud3ed\uc6b0\uac00 \ub0b4\ub824 \uc9c0\uc5ed\uc774 \ubb3c\uc5d0 \uc7a0\uacbc\ub2e4. 100\uba85 \uc774\uc0c1\uc758 \uc0ac\ub78c\ub4e4\uc774 \uc228\uc84c\uace0 \ub300\ud53c\uc790\ub97c \ub098\ub974\ub358 5\ub300\uc758 \ubc84\uc2a4\uac00 \ub4a4\uc9d1\ud614\ub2e4. \ud0c0\ub9c8\uc6b8\ub9ac\ud30c\uc2a4 \uc8fc \ub0b4\ub959\uc5d0\uc11c\ub294 250mm\uac00 \ub118\ub294 \uac15\uc218\ub7c9\uc744 \uae30\ub85d\ud558\uc600\ub2e4.", "paragraph_answer": "\ud5c8\ub9ac\ucf00\uc778 \uae38\ubc84\ud2b8\uc758 \ucd1d \uc0ac\ub9dd\uc790 433\uba85 \uc911\uc5d0 240\uba85\uc740 \uba55\uc2dc\ucf54\uc5d0\uc11c \ub098\uc654\uc744 \ub9cc\ud07c \uba55\uc2dc\ucf54\uc5d0\uc11c\ub294 \ud53c\ud574\uac00 \uadf9\uc2ec\ud558\uc600\ub2e4. 3\ub9cc5\ucc9c\uba85\uc758 \uc774\uc7ac\ubbfc\uc774 \uc0dd\uacbc\uace0 \uae38\ubc84\ud2b8\uac00 \uc720\uce74\ud0c4 \ubc18\ub3c4\ub97c \ub36e\ucce4\uc744 \ub54c, 83\ucc99\uc758 \ubc30\ub4e4\uc774 \uac00\ub77c\uc549\uc558\uace0 6\ub9cc\ucc44\uc758 \uc9d1\uc774 \ud30c\uad34, \ucd1d \uc7ac\uc0b0\ud53c\ud574\uc561\uc740 10\uc5b5\uc5d0\uc11c 20\uc5b5\ub2ec\ub7ec(1989\ub144 \uae30\uc900)\uc73c\ub85c \ucd94\uc0b0\ub418\uc5c8\ub2e4. \uce89\ucfe4 \uc9c0\ubc29\uc5d0\uc11c\ub294 8\ucc9c7\ubc31\ub9cc\ub2ec\ub7ec (1989\ub144 \uae30\uc900)\uc758 \ud53c\ud574\uc561\uc774 \uc9d1\uacc4\ub418\uc5c8\uace0 \ud5c8\ub9ac\ucf00\uc778\uc758 \ud53c\ud574\ub85c \uc778\ud574 \ub3c4\uc2dc\uc758 \uad00\uad11\uc0b0\uc5c5\uc5d0 \uce58\uba85\uc801\uc778 \ud0c0\uaca9\uc744 \uc8fc\uc5c8\ub2e4. \uc720\uce74\ud0c4 \ubc18\ub3c4\uc5d0 \uc704\uce58\ud55c \ud504\ub85c\uadf8\ub808\uc18c\ub77c\ub294 \ub3c4\uc2dc\uc5d0\uc11c\ub294 350mm\uac00 \ub118\ub294 \ube44\uac00 \uc3df\uc544\uc84c\uae30\ub3c4 \ud558\uc600\ub2e4. \uba55\uc2dc\ucf54 \ubd81\ub3d9\ubd80\uc5d0\uc11c\ub294 \ubaac\ud14c\ub808\uc774 \uc8fc\ubcc0\uc5d0 \ud3ed\uc6b0\uac00 \ub0b4\ub824 \uc9c0\uc5ed\uc774 \ubb3c\uc5d0 \uc7a0\uacbc\ub2e4. 100\uba85 \uc774\uc0c1\uc758 \uc0ac\ub78c\ub4e4\uc774 \uc228\uc84c\uace0 \ub300\ud53c\uc790\ub97c \ub098\ub974\ub358 5\ub300\uc758 \ubc84\uc2a4\uac00 \ub4a4\uc9d1\ud614\ub2e4. \ud0c0\ub9c8\uc6b8\ub9ac\ud30c\uc2a4 \uc8fc \ub0b4\ub959\uc5d0\uc11c\ub294 250mm\uac00 \ub118\ub294 \uac15\uc218\ub7c9\uc744 \uae30\ub85d\ud558\uc600\ub2e4.", "sentence_answer": "\uce89\ucfe4 \uc9c0\ubc29\uc5d0\uc11c\ub294 8\ucc9c7\ubc31\ub9cc\ub2ec\ub7ec (1989\ub144 \uae30\uc900)\uc758 \ud53c\ud574\uc561\uc774 \uc9d1\uacc4\ub418\uc5c8\uace0 \ud5c8\ub9ac\ucf00\uc778\uc758 \ud53c\ud574\ub85c \uc778\ud574 \ub3c4\uc2dc\uc758 \uad00\uad11\uc0b0\uc5c5\uc5d0 \uce58\uba85\uc801\uc778 \ud0c0\uaca9\uc744 \uc8fc\uc5c8\ub2e4.", "paragraph_id": "6571067-2-2", "paragraph_question": "question: \ud5c8\ub9ac\ucf00\uc778 \uae38\ubc84\ud2b8\uc5d0 \uc758\ud574 \uce89\ucfe4 \uc9c0\ubc29\uc5d0\uc11c\ub294 \uc5bc\ub9c8\uc758 \ud53c\ud574\uc561\uc774 \ubc1c\uc0dd\ub418\uc5c8\ub098?, context: \ud5c8\ub9ac\ucf00\uc778 \uae38\ubc84\ud2b8\uc758 \ucd1d \uc0ac\ub9dd\uc790 433\uba85 \uc911\uc5d0 240\uba85\uc740 \uba55\uc2dc\ucf54\uc5d0\uc11c \ub098\uc654\uc744 \ub9cc\ud07c \uba55\uc2dc\ucf54\uc5d0\uc11c\ub294 \ud53c\ud574\uac00 \uadf9\uc2ec\ud558\uc600\ub2e4. 3\ub9cc5\ucc9c\uba85\uc758 \uc774\uc7ac\ubbfc\uc774 \uc0dd\uacbc\uace0 \uae38\ubc84\ud2b8\uac00 \uc720\uce74\ud0c4 \ubc18\ub3c4\ub97c \ub36e\ucce4\uc744 \ub54c, 83\ucc99\uc758 \ubc30\ub4e4\uc774 \uac00\ub77c\uc549\uc558\uace0 6\ub9cc\ucc44\uc758 \uc9d1\uc774 \ud30c\uad34, \ucd1d \uc7ac\uc0b0\ud53c\ud574\uc561\uc740 10\uc5b5\uc5d0\uc11c 20\uc5b5\ub2ec\ub7ec(1989\ub144 \uae30\uc900)\uc73c\ub85c \ucd94\uc0b0\ub418\uc5c8\ub2e4. \uce89\ucfe4 \uc9c0\ubc29\uc5d0\uc11c\ub294 8\ucc9c7\ubc31\ub9cc\ub2ec\ub7ec(1989\ub144 \uae30\uc900)\uc758 \ud53c\ud574\uc561\uc774 \uc9d1\uacc4\ub418\uc5c8\uace0 \ud5c8\ub9ac\ucf00\uc778\uc758 \ud53c\ud574\ub85c \uc778\ud574 \ub3c4\uc2dc\uc758 \uad00\uad11\uc0b0\uc5c5\uc5d0 \uce58\uba85\uc801\uc778 \ud0c0\uaca9\uc744 \uc8fc\uc5c8\ub2e4. \uc720\uce74\ud0c4 \ubc18\ub3c4\uc5d0 \uc704\uce58\ud55c \ud504\ub85c\uadf8\ub808\uc18c\ub77c\ub294 \ub3c4\uc2dc\uc5d0\uc11c\ub294 350mm\uac00 \ub118\ub294 \ube44\uac00 \uc3df\uc544\uc84c\uae30\ub3c4 \ud558\uc600\ub2e4. \uba55\uc2dc\ucf54 \ubd81\ub3d9\ubd80\uc5d0\uc11c\ub294 \ubaac\ud14c\ub808\uc774 \uc8fc\ubcc0\uc5d0 \ud3ed\uc6b0\uac00 \ub0b4\ub824 \uc9c0\uc5ed\uc774 \ubb3c\uc5d0 \uc7a0\uacbc\ub2e4. 100\uba85 \uc774\uc0c1\uc758 \uc0ac\ub78c\ub4e4\uc774 \uc228\uc84c\uace0 \ub300\ud53c\uc790\ub97c \ub098\ub974\ub358 5\ub300\uc758 \ubc84\uc2a4\uac00 \ub4a4\uc9d1\ud614\ub2e4. \ud0c0\ub9c8\uc6b8\ub9ac\ud30c\uc2a4 \uc8fc \ub0b4\ub959\uc5d0\uc11c\ub294 250mm\uac00 \ub118\ub294 \uac15\uc218\ub7c9\uc744 \uae30\ub85d\ud558\uc600\ub2e4."} {"question": "\uc870\uc120\uc2dc\ub300, \ub0a8\ud55c\uc0b0\uc131\uc5d0 \uc720\uc0ac\uc2dc \uc655\uc774 \ud53c\ub09c\ud560 \uc218 \uc788\ub3c4\ub85d \uc900\ube44\ud55c \uc608\ube44\uad81\uad90\uc758 \uc774\ub984\uc740?", "paragraph": "\uad11\uc8fc \ud589\uad81, \ub0a8\ud55c\uc0b0\uc131 \ud589\uad81\uc774\ub77c \ubd88\ub9ac\ub294 \ud589\uad81\uc740 1626\ub144 6\uc6d4 \ucd1d\uc735\uc0ac \uc774\uc11c\uc758 \uacc4\ucc45\uc5d0 \ub530\ub77c \uc644\uc131\ub41c \uac83\uc774\ub2e4. \uc870\uc120 \uc2dc\ub300\uc758 \ub0a8\ud55c\ud589\uad81\uc740 \uc720\uc0ac\uc2dc \uc655\uc774 \ud53c\ub09c\ud560 \uc218 \uc788\ub3c4\ub85d \uc900\ube44\ub41c \uc608\ube44 \uad81\uad90\uc774\uc5c8\ub2e4. \ub0a8\ud55c\uc0b0\uc131\uc758 \ud589\uad81\uc5d0 \ub300\ud55c \ubc1c\uad74\uc870\uc0ac\uac00 1999\ub144 \ubd80\ud130 \ud55c\uad6d\ud1a0\uc9c0\uacf5\uc0ac \ubc15\ubb3c\uad00\uc5d0 \uc758\ud574\uc11c \uc2dc\ud589\ub418\uc5c8\ub2e4. 2007\ub144\uae4c\uc9c0 \ucd1d 8\ucc28\uc5d0 \uac78\uce5c \uc870\uc0ac\uc5d0\uc11c, \uc2e0\ub77c \uc2dc\ub300 \ub300\ud615 \uac74\ubb3c\ud130\uac00 \ubc1c\uacac\ub418\uc5c8\ub2e4. \ud1b5\uc77c\uc2e0\ub77c \uc2dc\ub300 \ub300\ud615 \uac74\ubb3c\ud130\ub294 \ub3d9\uc11c \uc57d 18m, \ub0a8\ubd81 53.5m \ucd1d 290\ud3c9\uc758 \ub9e4\uc6b0 \ud070 \uaddc\ubaa8\ub85c, \uc548\ud559\uad81\uc758 \uc678\uc804\uc774 \uc815\uba74 11\uce78 49m, \uce21\uba74 4\uce78 16.3m\ub85c \uc57d 242\ud3c9\uc774\ub77c\ub294 \uac83\uc5d0 \ube44\ucd94\uc5b4 \uc0dd\uac01\ud574 \ubcf4\uba74 \uad49\uc7a5\ud55c \uaddc\ubaa8\uc784\uc744 \uc54c \uc218\uac00 \uc788\ub2e4. \uac74\ubb3c\uc758 \uaddc\ubaa8\ub85c \uc778\ud574 \uad81\uad90\ub85c \uc4f0\uc600\uc744 \uac83\uc774\ub77c\ub294 \uc758\uacac\uc774 \uc788\uc73c\ub098, \ub450\uaebc\uc6b4 \ubcbd\uacfc \ubc29\ucda9\uc744 \uc704\ud55c \uc2dc\uc124\uc774 \uac16\ucd94\uc5b4\uc9c4 \uac74\ubb3c\uc758 \uad6c\uc870\ub97c \ubcfc \ub54c \ub300\uccb4\ub85c \ucc3d\uace0\ub85c \uc4f0\uc600\uc744 \uac83\uc774\ub77c\ub294 \uc758\uacac\uc774\ub2e4. \uac19\uc774 \ubc1c\uacac\ub41c \uae30\uc640\ub294 \ud06c\uae30\uac00 \ub9e4\uc6b0 \ud070\ub370 \ubb34\uac8c\uac00 \ubcf4\ud1b5\uc758 \uae30\uc640\uc758 4\ubc30\uac00 \ub118\uc5b4 \uc554\uae30\uc640\ub294 \ud55c \uc7a5\uc774 \uc57d 19kg\uc774\uace0, \uc22b\ud0a4\uc640\ub294 \uc57d 15kg\uc774\ub098 \ub41c\ub2e4. \uc774\uac83\uc740 \uc870\uc120 \uc2dc\ub300\uc758 \uc911\uae30\uc640\uac00 \uc57d 4 kg, 3.3kg\uc778 \uac83\uc5d0 \ube44\ud558\uba74 \uc0c1\ub2f9\ud55c \ubb34\uac8c\uc774\ub2e4.", "answer": "\ub0a8\ud55c\ud589\uad81", "sentence": "\uc870\uc120 \uc2dc\ub300\uc758 \ub0a8\ud55c\ud589\uad81 \uc740 \uc720\uc0ac\uc2dc \uc655\uc774 \ud53c\ub09c\ud560 \uc218 \uc788\ub3c4\ub85d \uc900\ube44\ub41c \uc608\ube44 \uad81\uad90\uc774\uc5c8\ub2e4.", "paragraph_sentence": "\uad11\uc8fc \ud589\uad81, \ub0a8\ud55c\uc0b0\uc131 \ud589\uad81\uc774\ub77c \ubd88\ub9ac\ub294 \ud589\uad81\uc740 1626\ub144 6\uc6d4 \ucd1d\uc735\uc0ac \uc774\uc11c\uc758 \uacc4\ucc45\uc5d0 \ub530\ub77c \uc644\uc131\ub41c \uac83\uc774\ub2e4. \uc870\uc120 \uc2dc\ub300\uc758 \ub0a8\ud55c\ud589\uad81 \uc740 \uc720\uc0ac\uc2dc \uc655\uc774 \ud53c\ub09c\ud560 \uc218 \uc788\ub3c4\ub85d \uc900\ube44\ub41c \uc608\ube44 \uad81\uad90\uc774\uc5c8\ub2e4. \ub0a8\ud55c\uc0b0\uc131\uc758 \ud589\uad81\uc5d0 \ub300\ud55c \ubc1c\uad74\uc870\uc0ac\uac00 1999\ub144 \ubd80\ud130 \ud55c\uad6d\ud1a0\uc9c0\uacf5\uc0ac \ubc15\ubb3c\uad00\uc5d0 \uc758\ud574\uc11c \uc2dc\ud589\ub418\uc5c8\ub2e4. 2007\ub144\uae4c\uc9c0 \ucd1d 8\ucc28\uc5d0 \uac78\uce5c \uc870\uc0ac\uc5d0\uc11c, \uc2e0\ub77c \uc2dc\ub300 \ub300\ud615 \uac74\ubb3c\ud130\uac00 \ubc1c\uacac\ub418\uc5c8\ub2e4. \ud1b5\uc77c\uc2e0\ub77c \uc2dc\ub300 \ub300\ud615 \uac74\ubb3c\ud130\ub294 \ub3d9\uc11c \uc57d 18m, \ub0a8\ubd81 53.5m \ucd1d 290\ud3c9\uc758 \ub9e4\uc6b0 \ud070 \uaddc\ubaa8\ub85c, \uc548\ud559\uad81\uc758 \uc678\uc804\uc774 \uc815\uba74 11\uce78 49m, \uce21\uba74 4\uce78 16.3m\ub85c \uc57d 242\ud3c9\uc774\ub77c\ub294 \uac83\uc5d0 \ube44\ucd94\uc5b4 \uc0dd\uac01\ud574 \ubcf4\uba74 \uad49\uc7a5\ud55c \uaddc\ubaa8\uc784\uc744 \uc54c \uc218\uac00 \uc788\ub2e4. \uac74\ubb3c\uc758 \uaddc\ubaa8\ub85c \uc778\ud574 \uad81\uad90\ub85c \uc4f0\uc600\uc744 \uac83\uc774\ub77c\ub294 \uc758\uacac\uc774 \uc788\uc73c\ub098, \ub450\uaebc\uc6b4 \ubcbd\uacfc \ubc29\ucda9\uc744 \uc704\ud55c \uc2dc\uc124\uc774 \uac16\ucd94\uc5b4\uc9c4 \uac74\ubb3c\uc758 \uad6c\uc870\ub97c \ubcfc \ub54c \ub300\uccb4\ub85c \ucc3d\uace0\ub85c \uc4f0\uc600\uc744 \uac83\uc774\ub77c\ub294 \uc758\uacac\uc774\ub2e4. \uac19\uc774 \ubc1c\uacac\ub41c \uae30\uc640\ub294 \ud06c\uae30\uac00 \ub9e4\uc6b0 \ud070\ub370 \ubb34\uac8c\uac00 \ubcf4\ud1b5\uc758 \uae30\uc640\uc758 4\ubc30\uac00 \ub118\uc5b4 \uc554\uae30\uc640\ub294 \ud55c \uc7a5\uc774 \uc57d 19kg\uc774\uace0, \uc22b\ud0a4\uc640\ub294 \uc57d 15kg\uc774\ub098 \ub41c\ub2e4. \uc774\uac83\uc740 \uc870\uc120 \uc2dc\ub300\uc758 \uc911\uae30\uc640\uac00 \uc57d 4 kg, 3.3kg\uc778 \uac83\uc5d0 \ube44\ud558\uba74 \uc0c1\ub2f9\ud55c \ubb34\uac8c\uc774\ub2e4.", "paragraph_answer": "\uad11\uc8fc \ud589\uad81, \ub0a8\ud55c\uc0b0\uc131 \ud589\uad81\uc774\ub77c \ubd88\ub9ac\ub294 \ud589\uad81\uc740 1626\ub144 6\uc6d4 \ucd1d\uc735\uc0ac \uc774\uc11c\uc758 \uacc4\ucc45\uc5d0 \ub530\ub77c \uc644\uc131\ub41c \uac83\uc774\ub2e4. \uc870\uc120 \uc2dc\ub300\uc758 \ub0a8\ud55c\ud589\uad81 \uc740 \uc720\uc0ac\uc2dc \uc655\uc774 \ud53c\ub09c\ud560 \uc218 \uc788\ub3c4\ub85d \uc900\ube44\ub41c \uc608\ube44 \uad81\uad90\uc774\uc5c8\ub2e4. \ub0a8\ud55c\uc0b0\uc131\uc758 \ud589\uad81\uc5d0 \ub300\ud55c \ubc1c\uad74\uc870\uc0ac\uac00 1999\ub144 \ubd80\ud130 \ud55c\uad6d\ud1a0\uc9c0\uacf5\uc0ac \ubc15\ubb3c\uad00\uc5d0 \uc758\ud574\uc11c \uc2dc\ud589\ub418\uc5c8\ub2e4. 2007\ub144\uae4c\uc9c0 \ucd1d 8\ucc28\uc5d0 \uac78\uce5c \uc870\uc0ac\uc5d0\uc11c, \uc2e0\ub77c \uc2dc\ub300 \ub300\ud615 \uac74\ubb3c\ud130\uac00 \ubc1c\uacac\ub418\uc5c8\ub2e4. \ud1b5\uc77c\uc2e0\ub77c \uc2dc\ub300 \ub300\ud615 \uac74\ubb3c\ud130\ub294 \ub3d9\uc11c \uc57d 18m, \ub0a8\ubd81 53.5m \ucd1d 290\ud3c9\uc758 \ub9e4\uc6b0 \ud070 \uaddc\ubaa8\ub85c, \uc548\ud559\uad81\uc758 \uc678\uc804\uc774 \uc815\uba74 11\uce78 49m, \uce21\uba74 4\uce78 16.3m\ub85c \uc57d 242\ud3c9\uc774\ub77c\ub294 \uac83\uc5d0 \ube44\ucd94\uc5b4 \uc0dd\uac01\ud574 \ubcf4\uba74 \uad49\uc7a5\ud55c \uaddc\ubaa8\uc784\uc744 \uc54c \uc218\uac00 \uc788\ub2e4. \uac74\ubb3c\uc758 \uaddc\ubaa8\ub85c \uc778\ud574 \uad81\uad90\ub85c \uc4f0\uc600\uc744 \uac83\uc774\ub77c\ub294 \uc758\uacac\uc774 \uc788\uc73c\ub098, \ub450\uaebc\uc6b4 \ubcbd\uacfc \ubc29\ucda9\uc744 \uc704\ud55c \uc2dc\uc124\uc774 \uac16\ucd94\uc5b4\uc9c4 \uac74\ubb3c\uc758 \uad6c\uc870\ub97c \ubcfc \ub54c \ub300\uccb4\ub85c \ucc3d\uace0\ub85c \uc4f0\uc600\uc744 \uac83\uc774\ub77c\ub294 \uc758\uacac\uc774\ub2e4. \uac19\uc774 \ubc1c\uacac\ub41c \uae30\uc640\ub294 \ud06c\uae30\uac00 \ub9e4\uc6b0 \ud070\ub370 \ubb34\uac8c\uac00 \ubcf4\ud1b5\uc758 \uae30\uc640\uc758 4\ubc30\uac00 \ub118\uc5b4 \uc554\uae30\uc640\ub294 \ud55c \uc7a5\uc774 \uc57d 19kg\uc774\uace0, \uc22b\ud0a4\uc640\ub294 \uc57d 15kg\uc774\ub098 \ub41c\ub2e4. \uc774\uac83\uc740 \uc870\uc120 \uc2dc\ub300\uc758 \uc911\uae30\uc640\uac00 \uc57d 4 kg, 3.3kg\uc778 \uac83\uc5d0 \ube44\ud558\uba74 \uc0c1\ub2f9\ud55c \ubb34\uac8c\uc774\ub2e4.", "sentence_answer": "\uc870\uc120 \uc2dc\ub300\uc758 \ub0a8\ud55c\ud589\uad81 \uc740 \uc720\uc0ac\uc2dc \uc655\uc774 \ud53c\ub09c\ud560 \uc218 \uc788\ub3c4\ub85d \uc900\ube44\ub41c \uc608\ube44 \uad81\uad90\uc774\uc5c8\ub2e4.", "paragraph_id": "6022976-16-0", "paragraph_question": "question: \uc870\uc120\uc2dc\ub300, \ub0a8\ud55c\uc0b0\uc131\uc5d0 \uc720\uc0ac\uc2dc \uc655\uc774 \ud53c\ub09c\ud560 \uc218 \uc788\ub3c4\ub85d \uc900\ube44\ud55c \uc608\ube44\uad81\uad90\uc758 \uc774\ub984\uc740?, context: \uad11\uc8fc \ud589\uad81, \ub0a8\ud55c\uc0b0\uc131 \ud589\uad81\uc774\ub77c \ubd88\ub9ac\ub294 \ud589\uad81\uc740 1626\ub144 6\uc6d4 \ucd1d\uc735\uc0ac \uc774\uc11c\uc758 \uacc4\ucc45\uc5d0 \ub530\ub77c \uc644\uc131\ub41c \uac83\uc774\ub2e4. \uc870\uc120 \uc2dc\ub300\uc758 \ub0a8\ud55c\ud589\uad81\uc740 \uc720\uc0ac\uc2dc \uc655\uc774 \ud53c\ub09c\ud560 \uc218 \uc788\ub3c4\ub85d \uc900\ube44\ub41c \uc608\ube44 \uad81\uad90\uc774\uc5c8\ub2e4. \ub0a8\ud55c\uc0b0\uc131\uc758 \ud589\uad81\uc5d0 \ub300\ud55c \ubc1c\uad74\uc870\uc0ac\uac00 1999\ub144 \ubd80\ud130 \ud55c\uad6d\ud1a0\uc9c0\uacf5\uc0ac \ubc15\ubb3c\uad00\uc5d0 \uc758\ud574\uc11c \uc2dc\ud589\ub418\uc5c8\ub2e4. 2007\ub144\uae4c\uc9c0 \ucd1d 8\ucc28\uc5d0 \uac78\uce5c \uc870\uc0ac\uc5d0\uc11c, \uc2e0\ub77c \uc2dc\ub300 \ub300\ud615 \uac74\ubb3c\ud130\uac00 \ubc1c\uacac\ub418\uc5c8\ub2e4. \ud1b5\uc77c\uc2e0\ub77c \uc2dc\ub300 \ub300\ud615 \uac74\ubb3c\ud130\ub294 \ub3d9\uc11c \uc57d 18m, \ub0a8\ubd81 53.5m \ucd1d 290\ud3c9\uc758 \ub9e4\uc6b0 \ud070 \uaddc\ubaa8\ub85c, \uc548\ud559\uad81\uc758 \uc678\uc804\uc774 \uc815\uba74 11\uce78 49m, \uce21\uba74 4\uce78 16.3m\ub85c \uc57d 242\ud3c9\uc774\ub77c\ub294 \uac83\uc5d0 \ube44\ucd94\uc5b4 \uc0dd\uac01\ud574 \ubcf4\uba74 \uad49\uc7a5\ud55c \uaddc\ubaa8\uc784\uc744 \uc54c \uc218\uac00 \uc788\ub2e4. \uac74\ubb3c\uc758 \uaddc\ubaa8\ub85c \uc778\ud574 \uad81\uad90\ub85c \uc4f0\uc600\uc744 \uac83\uc774\ub77c\ub294 \uc758\uacac\uc774 \uc788\uc73c\ub098, \ub450\uaebc\uc6b4 \ubcbd\uacfc \ubc29\ucda9\uc744 \uc704\ud55c \uc2dc\uc124\uc774 \uac16\ucd94\uc5b4\uc9c4 \uac74\ubb3c\uc758 \uad6c\uc870\ub97c \ubcfc \ub54c \ub300\uccb4\ub85c \ucc3d\uace0\ub85c \uc4f0\uc600\uc744 \uac83\uc774\ub77c\ub294 \uc758\uacac\uc774\ub2e4. \uac19\uc774 \ubc1c\uacac\ub41c \uae30\uc640\ub294 \ud06c\uae30\uac00 \ub9e4\uc6b0 \ud070\ub370 \ubb34\uac8c\uac00 \ubcf4\ud1b5\uc758 \uae30\uc640\uc758 4\ubc30\uac00 \ub118\uc5b4 \uc554\uae30\uc640\ub294 \ud55c \uc7a5\uc774 \uc57d 19kg\uc774\uace0, \uc22b\ud0a4\uc640\ub294 \uc57d 15kg\uc774\ub098 \ub41c\ub2e4. \uc774\uac83\uc740 \uc870\uc120 \uc2dc\ub300\uc758 \uc911\uae30\uc640\uac00 \uc57d 4 kg, 3.3kg\uc778 \uac83\uc5d0 \ube44\ud558\uba74 \uc0c1\ub2f9\ud55c \ubb34\uac8c\uc774\ub2e4."} {"question": "\uce60\ub808\uac00 \uad6d\uc81c \uc6d0\uc790\ub825\uae30\uad6c \uc774\uc0ac\ud68c\uc5d0 \ubd09\uc0ac\ud55c \ud574\ub294?", "paragraph": "\uce60\ub808\ub294 2005\ub144 \ud638\uc138 \ubbf8\uac94 Insulza, \ubbf8\uc8fc\uae30\uad6c\uc758 \uce60\ub808 \uad6d\uac00, \uc120\ucd9c\ub418\uc5c8\ub2e4. \uc0ac\ubb34 \ucd1d\uc7a5\uc774 2005\ub144 5\uc6d4\uc5d0 \uc720\uc5d4 \uc548\uc804 \ubcf4\uc7a5 \uc774\uc0ac\ud68c\uc5d0 2\ub144\uc774 \uc544\ub2cc \uc601\uad6c\uc801\uc778 \uc704\uce58\ub97c \uc644\ub8cc\ud558\uace0 \uc7ac\uc120\ucd9c\ub418\uace0, \uc790\uc2e0\uc758 \uc704\uce58\uc5d0\uc11c \ud655\uc778\ub418\uc5c8\ub2e4. 2009\ub144 \uce60\ub808\ub294 \ud604\uc7ac \ucd1d\uc7ac\uc758 \uad6d\uc81c \uc6d0\uc790\ub825\uae30\uad6c (IAEA) \uc774\uc0ac\ud68c\uc5d0 \ubd09\uc0ac\ud558\uace0, \uc774\uc0ac\ud68c\uc758 2007-2008 \uc704\uc6d0\uc7a5\uc740 \uad6d\uc81c \uc6d0\uc790\ub825\uae30\uad6c (IAEA), Milenko E. Skoknic \uce60\ub808 \ub300\uc0ac\uc774\ub2e4. \uad6d\uac00 \uae30\uad00\uc758 \uc720\uc5d4 \uac00\uc871\uc758 \ud68c\uc6d0\uc774\uba70, UN \ud3c9\ud654 \uc720\uc9c0 \ud65c\ub3d9\uc5d0 \ucc38\uc5ec\ud558\uace0 \uc788\ub2e4. \uc774\ub294 3\ub144 \uc784\uae30\ub85c 2011\ub144 \uc720\uc5d4 \uc778\uad8c \uc774\uc0ac\ud68c\uc758 \uad6c\uc131\uc6d0\uc73c\ub85c \uc7ac \uc120\ucd9c\ub418\uc5c8\ub2e4. \ub610\ud55c 2013\ub144\uc5d0 \uc720\uc5d4 \uc548\uc804 \ubcf4\uc7a5 \uc774\uc0ac\ud68c\uc5d0 5 \ube44\uc0c1\uc784 \uc88c\uc11d \uc911 \ud558\ub098\uc5d0 \uc120\uc784\ub418\uc5c8\ub2e4.", "answer": "2009\ub144", "sentence": "2009\ub144 \uce60\ub808\ub294 \ud604\uc7ac \ucd1d\uc7ac\uc758 \uad6d\uc81c \uc6d0\uc790\ub825\uae30\uad6c (IAEA) \uc774\uc0ac\ud68c\uc5d0 \ubd09\uc0ac\ud558\uace0, \uc774\uc0ac\ud68c\uc758 2007-2008 \uc704\uc6d0\uc7a5\uc740 \uad6d\uc81c \uc6d0\uc790\ub825\uae30\uad6c (IAEA), Milenko E. Skoknic \uce60\ub808 \ub300\uc0ac\uc774\ub2e4.", "paragraph_sentence": "\uce60\ub808\ub294 2005\ub144 \ud638\uc138 \ubbf8\uac94 Insulza, \ubbf8\uc8fc\uae30\uad6c\uc758 \uce60\ub808 \uad6d\uac00, \uc120\ucd9c\ub418\uc5c8\ub2e4. \uc0ac\ubb34 \ucd1d\uc7a5\uc774 2005\ub144 5\uc6d4\uc5d0 \uc720\uc5d4 \uc548\uc804 \ubcf4\uc7a5 \uc774\uc0ac\ud68c\uc5d0 2\ub144\uc774 \uc544\ub2cc \uc601\uad6c\uc801\uc778 \uc704\uce58\ub97c \uc644\ub8cc\ud558\uace0 \uc7ac\uc120\ucd9c\ub418\uace0, \uc790\uc2e0\uc758 \uc704\uce58\uc5d0\uc11c \ud655\uc778\ub418\uc5c8\ub2e4. 2009\ub144 \uce60\ub808\ub294 \ud604\uc7ac \ucd1d\uc7ac\uc758 \uad6d\uc81c \uc6d0\uc790\ub825\uae30\uad6c (IAEA) \uc774\uc0ac\ud68c\uc5d0 \ubd09\uc0ac\ud558\uace0, \uc774\uc0ac\ud68c\uc758 2007-2008 \uc704\uc6d0\uc7a5\uc740 \uad6d\uc81c \uc6d0\uc790\ub825\uae30\uad6c (IAEA), Milenko E. Skoknic \uce60\ub808 \ub300\uc0ac\uc774\ub2e4. \uad6d\uac00 \uae30\uad00\uc758 \uc720\uc5d4 \uac00\uc871\uc758 \ud68c\uc6d0\uc774\uba70, UN \ud3c9\ud654 \uc720\uc9c0 \ud65c\ub3d9\uc5d0 \ucc38\uc5ec\ud558\uace0 \uc788\ub2e4. \uc774\ub294 3\ub144 \uc784\uae30\ub85c 2011\ub144 \uc720\uc5d4 \uc778\uad8c \uc774\uc0ac\ud68c\uc758 \uad6c\uc131\uc6d0\uc73c\ub85c \uc7ac \uc120\ucd9c\ub418\uc5c8\ub2e4. \ub610\ud55c 2013\ub144\uc5d0 \uc720\uc5d4 \uc548\uc804 \ubcf4\uc7a5 \uc774\uc0ac\ud68c\uc5d0 5 \ube44\uc0c1\uc784 \uc88c\uc11d \uc911 \ud558\ub098\uc5d0 \uc120\uc784\ub418\uc5c8\ub2e4.", "paragraph_answer": "\uce60\ub808\ub294 2005\ub144 \ud638\uc138 \ubbf8\uac94 Insulza, \ubbf8\uc8fc\uae30\uad6c\uc758 \uce60\ub808 \uad6d\uac00, \uc120\ucd9c\ub418\uc5c8\ub2e4. \uc0ac\ubb34 \ucd1d\uc7a5\uc774 2005\ub144 5\uc6d4\uc5d0 \uc720\uc5d4 \uc548\uc804 \ubcf4\uc7a5 \uc774\uc0ac\ud68c\uc5d0 2\ub144\uc774 \uc544\ub2cc \uc601\uad6c\uc801\uc778 \uc704\uce58\ub97c \uc644\ub8cc\ud558\uace0 \uc7ac\uc120\ucd9c\ub418\uace0, \uc790\uc2e0\uc758 \uc704\uce58\uc5d0\uc11c \ud655\uc778\ub418\uc5c8\ub2e4. 2009\ub144 \uce60\ub808\ub294 \ud604\uc7ac \ucd1d\uc7ac\uc758 \uad6d\uc81c \uc6d0\uc790\ub825\uae30\uad6c (IAEA) \uc774\uc0ac\ud68c\uc5d0 \ubd09\uc0ac\ud558\uace0, \uc774\uc0ac\ud68c\uc758 2007-2008 \uc704\uc6d0\uc7a5\uc740 \uad6d\uc81c \uc6d0\uc790\ub825\uae30\uad6c (IAEA), Milenko E. Skoknic \uce60\ub808 \ub300\uc0ac\uc774\ub2e4. \uad6d\uac00 \uae30\uad00\uc758 \uc720\uc5d4 \uac00\uc871\uc758 \ud68c\uc6d0\uc774\uba70, UN \ud3c9\ud654 \uc720\uc9c0 \ud65c\ub3d9\uc5d0 \ucc38\uc5ec\ud558\uace0 \uc788\ub2e4. \uc774\ub294 3\ub144 \uc784\uae30\ub85c 2011\ub144 \uc720\uc5d4 \uc778\uad8c \uc774\uc0ac\ud68c\uc758 \uad6c\uc131\uc6d0\uc73c\ub85c \uc7ac \uc120\ucd9c\ub418\uc5c8\ub2e4. \ub610\ud55c 2013\ub144\uc5d0 \uc720\uc5d4 \uc548\uc804 \ubcf4\uc7a5 \uc774\uc0ac\ud68c\uc5d0 5 \ube44\uc0c1\uc784 \uc88c\uc11d \uc911 \ud558\ub098\uc5d0 \uc120\uc784\ub418\uc5c8\ub2e4.", "sentence_answer": " 2009\ub144 \uce60\ub808\ub294 \ud604\uc7ac \ucd1d\uc7ac\uc758 \uad6d\uc81c \uc6d0\uc790\ub825\uae30\uad6c (IAEA) \uc774\uc0ac\ud68c\uc5d0 \ubd09\uc0ac\ud558\uace0, \uc774\uc0ac\ud68c\uc758 2007-2008 \uc704\uc6d0\uc7a5\uc740 \uad6d\uc81c \uc6d0\uc790\ub825\uae30\uad6c (IAEA), Milenko E. Skoknic \uce60\ub808 \ub300\uc0ac\uc774\ub2e4.", "paragraph_id": "6464200-7-0", "paragraph_question": "question: \uce60\ub808\uac00 \uad6d\uc81c \uc6d0\uc790\ub825\uae30\uad6c \uc774\uc0ac\ud68c\uc5d0 \ubd09\uc0ac\ud55c \ud574\ub294?, context: \uce60\ub808\ub294 2005\ub144 \ud638\uc138 \ubbf8\uac94 Insulza, \ubbf8\uc8fc\uae30\uad6c\uc758 \uce60\ub808 \uad6d\uac00, \uc120\ucd9c\ub418\uc5c8\ub2e4. \uc0ac\ubb34 \ucd1d\uc7a5\uc774 2005\ub144 5\uc6d4\uc5d0 \uc720\uc5d4 \uc548\uc804 \ubcf4\uc7a5 \uc774\uc0ac\ud68c\uc5d0 2\ub144\uc774 \uc544\ub2cc \uc601\uad6c\uc801\uc778 \uc704\uce58\ub97c \uc644\ub8cc\ud558\uace0 \uc7ac\uc120\ucd9c\ub418\uace0, \uc790\uc2e0\uc758 \uc704\uce58\uc5d0\uc11c \ud655\uc778\ub418\uc5c8\ub2e4. 2009\ub144 \uce60\ub808\ub294 \ud604\uc7ac \ucd1d\uc7ac\uc758 \uad6d\uc81c \uc6d0\uc790\ub825\uae30\uad6c (IAEA) \uc774\uc0ac\ud68c\uc5d0 \ubd09\uc0ac\ud558\uace0, \uc774\uc0ac\ud68c\uc758 2007-2008 \uc704\uc6d0\uc7a5\uc740 \uad6d\uc81c \uc6d0\uc790\ub825\uae30\uad6c (IAEA), Milenko E. Skoknic \uce60\ub808 \ub300\uc0ac\uc774\ub2e4. \uad6d\uac00 \uae30\uad00\uc758 \uc720\uc5d4 \uac00\uc871\uc758 \ud68c\uc6d0\uc774\uba70, UN \ud3c9\ud654 \uc720\uc9c0 \ud65c\ub3d9\uc5d0 \ucc38\uc5ec\ud558\uace0 \uc788\ub2e4. \uc774\ub294 3\ub144 \uc784\uae30\ub85c 2011\ub144 \uc720\uc5d4 \uc778\uad8c \uc774\uc0ac\ud68c\uc758 \uad6c\uc131\uc6d0\uc73c\ub85c \uc7ac \uc120\ucd9c\ub418\uc5c8\ub2e4. \ub610\ud55c 2013\ub144\uc5d0 \uc720\uc5d4 \uc548\uc804 \ubcf4\uc7a5 \uc774\uc0ac\ud68c\uc5d0 5 \ube44\uc0c1\uc784 \uc88c\uc11d \uc911 \ud558\ub098\uc5d0 \uc120\uc784\ub418\uc5c8\ub2e4."} {"question": "661\ub144\uc5d0 \uc2e0\ub77c\uc758 \uc655\uc740 \ub204\uad6c\uc778\uac00?", "paragraph": "\uc8fc\ub958\uc131\uc758 \uc704\uce58 \ube44\uc815\uacfc \uad00\ub828\ud574 \u300a\uc0bc\uad6d\uc0ac\uae30\u300b \uad8c\uc81c5 \uc2e0\ub77c\ubcf8\uae305 \ud0dc\uc885\ubb34\uc5f4\uc655 8\ub144(661\ub144)\uc870\uc5d0 \ub450\ub989\uc724(\u8c46\u9675\u5c39) \u00b7 \ub450\ub989\uc774(\u8c46\u9675\u4f0a) \u00b7 \ub450\ub7c9\uc724(\u8c46\u826f\u5c39)\uc774\ub77c\ub294 \ud45c\uae30\ub85c \uc5b8\uae09\ub418\ub294 \uc131\uc774 \ub4f1\uc7a5\ud558\ub294\ub370, \u300a\uc0bc\uad6d\uc0ac\uae30\u300b\uad8c\uc81c37 \uc9c0\ub9ac\uc9c0 \ubc31\uc81c\uc870\uc5d0\uc11c \ubc31\uc81c \uc5f4\uc774\ud604(\u6085\u5df2\u7e23)\uc758 \ub2e4\ub978 \uc774\ub984\uc744 \ub450\ub989\uc724\uc131, \ub610\ub294 \ub450\uacf6\uc131, \uc724\uc131(\u5c39\u57ce)\uc774\ub77c\uace0 \ud55c \uae30\ub85d\uacfc \u300a\uc2e0\uc99d\ub3d9\uad6d\uc5ec\uc9c0\uc2b9\ub78c\u300b\uad8c18 \uc815\uc0b0(\u5b9a\u5c71, \uc9c0\uae08\uc758 \ucda9\uccad\ub0a8\ub3c4 \uccad\uc591\uad70 \uc815\uc0b0\ud604)\uc870\uc5d0\uc11c \"\ubcf8\ub798 \ubc31\uc81c\uc758 \uc5f4\uc774\ud604\uc778\ub370 \ub450\ub989\uc724\uc131\uc774\ub77c\uace0\ub3c4 \ud55c\ub2e4\"\uace0 \ud55c \uae30\ub85d\uc774 \uc788\uc5b4, \ub450\ub989\uc724\uc131\uc740 \ub300\uccb4\ub85c \ubc31\uc81c\uc758 \uc5f4\uc774\ud604\uc774 \uc788\uc5c8\ub358 \uc9c0\uae08\uc758 \ucda9\uccad\ub0a8\ub3c4 \uccad\uc591\uad70 \uc815\uc0b0\ud604\uc5d0 \uc704\uce58\ud55c \uacc4\ubd09\uc0b0\uc131(\u9dc4\u9cf3\u5c71\u57ce)\uc73c\ub85c \ube44\uc815\ub41c\ub2e4. \u300a\uad6c\ub2f9\uc11c\u300b \ubc0f \u300a\uc790\uce58\ud1b5\uac10\u300b\uc758 \uae30\ub85d, \u300a\uc0bc\uad6d\uc0ac\uae30\u300b \ubcf8\uae30\uc640 \u300c\ubb38\ubb34\uc655\ubcf4\uc11c\u300d\uc758 \uae30\ub85d\uc744 \ub300\uc870\ud558\uc5ec \ub098 \u00b7 \ub2f9 \uc5f0\ud569\uad70\uc774 \uacf5\uaca9\ud55c \ub450\ub989\uc724\uc131\uc774 \ubc14\ub85c \uc8fc\ub958\uc131\uc774\uace0 \ub450\ub989\uc724\uc131\uacfc \uc8fc\ub958\uc131\uc740 \uc11c\ub85c \uac19\uc740 \uc9c0\uba85\uc744 \ub2e4\ub978 \ud55c\uc790\ub97c \uc368\uc11c \ud45c\uae30\ud55c \uc774\ub984\uc73c\ub85c \ubcf4\uae30\ub3c4 \ud55c\ub2e4.", "answer": "\ud0dc\uc885\ubb34\uc5f4\uc655", "sentence": "\uc8fc\ub958\uc131\uc758 \uc704\uce58 \ube44\uc815\uacfc \uad00\ub828\ud574 \u300a\uc0bc\uad6d\uc0ac\uae30\u300b \uad8c\uc81c5 \uc2e0\ub77c\ubcf8\uae305 \ud0dc\uc885\ubb34\uc5f4\uc655 8\ub144(661\ub144)\uc870\uc5d0 \ub450\ub989\uc724(\u8c46\u9675\u5c39) \u00b7 \ub450\ub989\uc774(\u8c46\u9675\u4f0a) \u00b7 \ub450\ub7c9\uc724(\u8c46\u826f\u5c39)\uc774\ub77c\ub294 \ud45c\uae30\ub85c \uc5b8\uae09\ub418\ub294 \uc131\uc774 \ub4f1\uc7a5\ud558\ub294\ub370, \u300a\uc0bc\uad6d\uc0ac\uae30\u300b\uad8c\uc81c37 \uc9c0\ub9ac\uc9c0 \ubc31\uc81c\uc870\uc5d0\uc11c \ubc31\uc81c \uc5f4\uc774\ud604(\u6085\u5df2\u7e23)\uc758 \ub2e4\ub978 \uc774\ub984\uc744 \ub450\ub989\uc724\uc131, \ub610\ub294 \ub450\uacf6\uc131, \uc724\uc131(\u5c39\u57ce)\uc774\ub77c\uace0 \ud55c \uae30\ub85d\uacfc \u300a\uc2e0\uc99d\ub3d9\uad6d\uc5ec\uc9c0\uc2b9\ub78c\u300b\uad8c18 \uc815\uc0b0(\u5b9a\u5c71, \uc9c0\uae08\uc758 \ucda9\uccad\ub0a8\ub3c4 \uccad\uc591\uad70 \uc815\uc0b0\ud604)\uc870\uc5d0\uc11c \"\ubcf8\ub798 \ubc31\uc81c\uc758 \uc5f4\uc774\ud604\uc778\ub370 \ub450\ub989\uc724\uc131\uc774\ub77c\uace0\ub3c4 \ud55c\ub2e4\"\uace0 \ud55c \uae30\ub85d\uc774 \uc788\uc5b4, \ub450\ub989\uc724\uc131\uc740 \ub300\uccb4\ub85c \ubc31\uc81c\uc758 \uc5f4\uc774\ud604\uc774 \uc788\uc5c8\ub358 \uc9c0\uae08\uc758 \ucda9\uccad\ub0a8\ub3c4 \uccad\uc591\uad70 \uc815\uc0b0\ud604\uc5d0 \uc704\uce58\ud55c \uacc4\ubd09\uc0b0\uc131(\u9dc4\u9cf3\u5c71\u57ce)\uc73c\ub85c \ube44\uc815\ub41c\ub2e4.", "paragraph_sentence": " \uc8fc\ub958\uc131\uc758 \uc704\uce58 \ube44\uc815\uacfc \uad00\ub828\ud574 \u300a\uc0bc\uad6d\uc0ac\uae30\u300b \uad8c\uc81c5 \uc2e0\ub77c\ubcf8\uae305 \ud0dc\uc885\ubb34\uc5f4\uc655 8\ub144(661\ub144)\uc870\uc5d0 \ub450\ub989\uc724(\u8c46\u9675\u5c39) \u00b7 \ub450\ub989\uc774(\u8c46\u9675\u4f0a) \u00b7 \ub450\ub7c9\uc724(\u8c46\u826f\u5c39)\uc774\ub77c\ub294 \ud45c\uae30\ub85c \uc5b8\uae09\ub418\ub294 \uc131\uc774 \ub4f1\uc7a5\ud558\ub294\ub370, \u300a\uc0bc\uad6d\uc0ac\uae30\u300b\uad8c\uc81c37 \uc9c0\ub9ac\uc9c0 \ubc31\uc81c\uc870\uc5d0\uc11c \ubc31\uc81c \uc5f4\uc774\ud604(\u6085\u5df2\u7e23)\uc758 \ub2e4\ub978 \uc774\ub984\uc744 \ub450\ub989\uc724\uc131, \ub610\ub294 \ub450\uacf6\uc131, \uc724\uc131(\u5c39\u57ce)\uc774\ub77c\uace0 \ud55c \uae30\ub85d\uacfc \u300a\uc2e0\uc99d\ub3d9\uad6d\uc5ec\uc9c0\uc2b9\ub78c\u300b\uad8c18 \uc815\uc0b0(\u5b9a\u5c71, \uc9c0\uae08\uc758 \ucda9\uccad\ub0a8\ub3c4 \uccad\uc591\uad70 \uc815\uc0b0\ud604)\uc870\uc5d0\uc11c \"\ubcf8\ub798 \ubc31\uc81c\uc758 \uc5f4\uc774\ud604\uc778\ub370 \ub450\ub989\uc724\uc131\uc774\ub77c\uace0\ub3c4 \ud55c\ub2e4\"\uace0 \ud55c \uae30\ub85d\uc774 \uc788\uc5b4, \ub450\ub989\uc724\uc131\uc740 \ub300\uccb4\ub85c \ubc31\uc81c\uc758 \uc5f4\uc774\ud604\uc774 \uc788\uc5c8\ub358 \uc9c0\uae08\uc758 \ucda9\uccad\ub0a8\ub3c4 \uccad\uc591\uad70 \uc815\uc0b0\ud604\uc5d0 \uc704\uce58\ud55c \uacc4\ubd09\uc0b0\uc131(\u9dc4\u9cf3\u5c71\u57ce)\uc73c\ub85c \ube44\uc815\ub41c\ub2e4. \u300a\uad6c\ub2f9\uc11c\u300b \ubc0f \u300a\uc790\uce58\ud1b5\uac10\u300b\uc758 \uae30\ub85d, \u300a\uc0bc\uad6d\uc0ac\uae30\u300b \ubcf8\uae30\uc640 \u300c\ubb38\ubb34\uc655\ubcf4\uc11c\u300d\uc758 \uae30\ub85d\uc744 \ub300\uc870\ud558\uc5ec \ub098 \u00b7 \ub2f9 \uc5f0\ud569\uad70\uc774 \uacf5\uaca9\ud55c \ub450\ub989\uc724\uc131\uc774 \ubc14\ub85c \uc8fc\ub958\uc131\uc774\uace0 \ub450\ub989\uc724\uc131\uacfc \uc8fc\ub958\uc131\uc740 \uc11c\ub85c \uac19\uc740 \uc9c0\uba85\uc744 \ub2e4\ub978 \ud55c\uc790\ub97c \uc368\uc11c \ud45c\uae30\ud55c \uc774\ub984\uc73c\ub85c \ubcf4\uae30\ub3c4 \ud55c\ub2e4.", "paragraph_answer": "\uc8fc\ub958\uc131\uc758 \uc704\uce58 \ube44\uc815\uacfc \uad00\ub828\ud574 \u300a\uc0bc\uad6d\uc0ac\uae30\u300b \uad8c\uc81c5 \uc2e0\ub77c\ubcf8\uae305 \ud0dc\uc885\ubb34\uc5f4\uc655 8\ub144(661\ub144)\uc870\uc5d0 \ub450\ub989\uc724(\u8c46\u9675\u5c39) \u00b7 \ub450\ub989\uc774(\u8c46\u9675\u4f0a) \u00b7 \ub450\ub7c9\uc724(\u8c46\u826f\u5c39)\uc774\ub77c\ub294 \ud45c\uae30\ub85c \uc5b8\uae09\ub418\ub294 \uc131\uc774 \ub4f1\uc7a5\ud558\ub294\ub370, \u300a\uc0bc\uad6d\uc0ac\uae30\u300b\uad8c\uc81c37 \uc9c0\ub9ac\uc9c0 \ubc31\uc81c\uc870\uc5d0\uc11c \ubc31\uc81c \uc5f4\uc774\ud604(\u6085\u5df2\u7e23)\uc758 \ub2e4\ub978 \uc774\ub984\uc744 \ub450\ub989\uc724\uc131, \ub610\ub294 \ub450\uacf6\uc131, \uc724\uc131(\u5c39\u57ce)\uc774\ub77c\uace0 \ud55c \uae30\ub85d\uacfc \u300a\uc2e0\uc99d\ub3d9\uad6d\uc5ec\uc9c0\uc2b9\ub78c\u300b\uad8c18 \uc815\uc0b0(\u5b9a\u5c71, \uc9c0\uae08\uc758 \ucda9\uccad\ub0a8\ub3c4 \uccad\uc591\uad70 \uc815\uc0b0\ud604)\uc870\uc5d0\uc11c \"\ubcf8\ub798 \ubc31\uc81c\uc758 \uc5f4\uc774\ud604\uc778\ub370 \ub450\ub989\uc724\uc131\uc774\ub77c\uace0\ub3c4 \ud55c\ub2e4\"\uace0 \ud55c \uae30\ub85d\uc774 \uc788\uc5b4, \ub450\ub989\uc724\uc131\uc740 \ub300\uccb4\ub85c \ubc31\uc81c\uc758 \uc5f4\uc774\ud604\uc774 \uc788\uc5c8\ub358 \uc9c0\uae08\uc758 \ucda9\uccad\ub0a8\ub3c4 \uccad\uc591\uad70 \uc815\uc0b0\ud604\uc5d0 \uc704\uce58\ud55c \uacc4\ubd09\uc0b0\uc131(\u9dc4\u9cf3\u5c71\u57ce)\uc73c\ub85c \ube44\uc815\ub41c\ub2e4. \u300a\uad6c\ub2f9\uc11c\u300b \ubc0f \u300a\uc790\uce58\ud1b5\uac10\u300b\uc758 \uae30\ub85d, \u300a\uc0bc\uad6d\uc0ac\uae30\u300b \ubcf8\uae30\uc640 \u300c\ubb38\ubb34\uc655\ubcf4\uc11c\u300d\uc758 \uae30\ub85d\uc744 \ub300\uc870\ud558\uc5ec \ub098 \u00b7 \ub2f9 \uc5f0\ud569\uad70\uc774 \uacf5\uaca9\ud55c \ub450\ub989\uc724\uc131\uc774 \ubc14\ub85c \uc8fc\ub958\uc131\uc774\uace0 \ub450\ub989\uc724\uc131\uacfc \uc8fc\ub958\uc131\uc740 \uc11c\ub85c \uac19\uc740 \uc9c0\uba85\uc744 \ub2e4\ub978 \ud55c\uc790\ub97c \uc368\uc11c \ud45c\uae30\ud55c \uc774\ub984\uc73c\ub85c \ubcf4\uae30\ub3c4 \ud55c\ub2e4.", "sentence_answer": "\uc8fc\ub958\uc131\uc758 \uc704\uce58 \ube44\uc815\uacfc \uad00\ub828\ud574 \u300a\uc0bc\uad6d\uc0ac\uae30\u300b \uad8c\uc81c5 \uc2e0\ub77c\ubcf8\uae305 \ud0dc\uc885\ubb34\uc5f4\uc655 8\ub144(661\ub144)\uc870\uc5d0 \ub450\ub989\uc724(\u8c46\u9675\u5c39) \u00b7 \ub450\ub989\uc774(\u8c46\u9675\u4f0a) \u00b7 \ub450\ub7c9\uc724(\u8c46\u826f\u5c39)\uc774\ub77c\ub294 \ud45c\uae30\ub85c \uc5b8\uae09\ub418\ub294 \uc131\uc774 \ub4f1\uc7a5\ud558\ub294\ub370, \u300a\uc0bc\uad6d\uc0ac\uae30\u300b\uad8c\uc81c37 \uc9c0\ub9ac\uc9c0 \ubc31\uc81c\uc870\uc5d0\uc11c \ubc31\uc81c \uc5f4\uc774\ud604(\u6085\u5df2\u7e23)\uc758 \ub2e4\ub978 \uc774\ub984\uc744 \ub450\ub989\uc724\uc131, \ub610\ub294 \ub450\uacf6\uc131, \uc724\uc131(\u5c39\u57ce)\uc774\ub77c\uace0 \ud55c \uae30\ub85d\uacfc \u300a\uc2e0\uc99d\ub3d9\uad6d\uc5ec\uc9c0\uc2b9\ub78c\u300b\uad8c18 \uc815\uc0b0(\u5b9a\u5c71, \uc9c0\uae08\uc758 \ucda9\uccad\ub0a8\ub3c4 \uccad\uc591\uad70 \uc815\uc0b0\ud604)\uc870\uc5d0\uc11c \"\ubcf8\ub798 \ubc31\uc81c\uc758 \uc5f4\uc774\ud604\uc778\ub370 \ub450\ub989\uc724\uc131\uc774\ub77c\uace0\ub3c4 \ud55c\ub2e4\"\uace0 \ud55c \uae30\ub85d\uc774 \uc788\uc5b4, \ub450\ub989\uc724\uc131\uc740 \ub300\uccb4\ub85c \ubc31\uc81c\uc758 \uc5f4\uc774\ud604\uc774 \uc788\uc5c8\ub358 \uc9c0\uae08\uc758 \ucda9\uccad\ub0a8\ub3c4 \uccad\uc591\uad70 \uc815\uc0b0\ud604\uc5d0 \uc704\uce58\ud55c \uacc4\ubd09\uc0b0\uc131(\u9dc4\u9cf3\u5c71\u57ce)\uc73c\ub85c \ube44\uc815\ub41c\ub2e4.", "paragraph_id": "6555572-5-0", "paragraph_question": "question: 661\ub144\uc5d0 \uc2e0\ub77c\uc758 \uc655\uc740 \ub204\uad6c\uc778\uac00?, context: \uc8fc\ub958\uc131\uc758 \uc704\uce58 \ube44\uc815\uacfc \uad00\ub828\ud574 \u300a\uc0bc\uad6d\uc0ac\uae30\u300b \uad8c\uc81c5 \uc2e0\ub77c\ubcf8\uae305 \ud0dc\uc885\ubb34\uc5f4\uc655 8\ub144(661\ub144)\uc870\uc5d0 \ub450\ub989\uc724(\u8c46\u9675\u5c39) \u00b7 \ub450\ub989\uc774(\u8c46\u9675\u4f0a) \u00b7 \ub450\ub7c9\uc724(\u8c46\u826f\u5c39)\uc774\ub77c\ub294 \ud45c\uae30\ub85c \uc5b8\uae09\ub418\ub294 \uc131\uc774 \ub4f1\uc7a5\ud558\ub294\ub370, \u300a\uc0bc\uad6d\uc0ac\uae30\u300b\uad8c\uc81c37 \uc9c0\ub9ac\uc9c0 \ubc31\uc81c\uc870\uc5d0\uc11c \ubc31\uc81c \uc5f4\uc774\ud604(\u6085\u5df2\u7e23)\uc758 \ub2e4\ub978 \uc774\ub984\uc744 \ub450\ub989\uc724\uc131, \ub610\ub294 \ub450\uacf6\uc131, \uc724\uc131(\u5c39\u57ce)\uc774\ub77c\uace0 \ud55c \uae30\ub85d\uacfc \u300a\uc2e0\uc99d\ub3d9\uad6d\uc5ec\uc9c0\uc2b9\ub78c\u300b\uad8c18 \uc815\uc0b0(\u5b9a\u5c71, \uc9c0\uae08\uc758 \ucda9\uccad\ub0a8\ub3c4 \uccad\uc591\uad70 \uc815\uc0b0\ud604)\uc870\uc5d0\uc11c \"\ubcf8\ub798 \ubc31\uc81c\uc758 \uc5f4\uc774\ud604\uc778\ub370 \ub450\ub989\uc724\uc131\uc774\ub77c\uace0\ub3c4 \ud55c\ub2e4\"\uace0 \ud55c \uae30\ub85d\uc774 \uc788\uc5b4, \ub450\ub989\uc724\uc131\uc740 \ub300\uccb4\ub85c \ubc31\uc81c\uc758 \uc5f4\uc774\ud604\uc774 \uc788\uc5c8\ub358 \uc9c0\uae08\uc758 \ucda9\uccad\ub0a8\ub3c4 \uccad\uc591\uad70 \uc815\uc0b0\ud604\uc5d0 \uc704\uce58\ud55c \uacc4\ubd09\uc0b0\uc131(\u9dc4\u9cf3\u5c71\u57ce)\uc73c\ub85c \ube44\uc815\ub41c\ub2e4. \u300a\uad6c\ub2f9\uc11c\u300b \ubc0f \u300a\uc790\uce58\ud1b5\uac10\u300b\uc758 \uae30\ub85d, \u300a\uc0bc\uad6d\uc0ac\uae30\u300b \ubcf8\uae30\uc640 \u300c\ubb38\ubb34\uc655\ubcf4\uc11c\u300d\uc758 \uae30\ub85d\uc744 \ub300\uc870\ud558\uc5ec \ub098 \u00b7 \ub2f9 \uc5f0\ud569\uad70\uc774 \uacf5\uaca9\ud55c \ub450\ub989\uc724\uc131\uc774 \ubc14\ub85c \uc8fc\ub958\uc131\uc774\uace0 \ub450\ub989\uc724\uc131\uacfc \uc8fc\ub958\uc131\uc740 \uc11c\ub85c \uac19\uc740 \uc9c0\uba85\uc744 \ub2e4\ub978 \ud55c\uc790\ub97c \uc368\uc11c \ud45c\uae30\ud55c \uc774\ub984\uc73c\ub85c \ubcf4\uae30\ub3c4 \ud55c\ub2e4."} {"question": "2002\ub144 \ub3d9\uacc4 \uc62c\ub9bc\ud53d \uc9c1\uc804 \ubc8c\uc5b4\uc838 \ud14c\ub7ec\uacf5\uaca9 \ubc29\uc9c0\ub97c \uc704\ud55c \ubcf4\uc548\uc744 \uac15\ud654\ud558\ub294 \uacc4\uae30\uac00 \ub41c \uc0ac\uac74\uc740?", "paragraph": "2002\ub144 \ub3d9\uacc4 \uc62c\ub9bc\ud53d\uc740 \ubbf8\uad6d \uc720\ud0c0 \uc8fc \uc194\ud2b8\ub808\uc774\ud06c\uc2dc\ud2f0\uc5d0\uc11c \uc5f4\ub838\ub2e4. \ucd1d 77\uac1c \ucc38\uac00\uad6d 2,399\uba85 \uc120\uc218\uac00 7\uac1c \uc885\ubaa9 78\uac1c \uacbd\uae30\uc5d0 \ucd9c\uc804\ud558\uc600\ub2e4. \uc774 \ub300\ud68c\ub294 9\u00b711 \ud14c\ub7ec\ub97c \uacaa\uc740 \uc9c1\ud6c4\uc5d0 \uc5f4\ub9b0 \uccab \uc62c\ub9bc\ud53d \ub300\ud68c\uc600\uae30 \ub54c\ubb38\uc5d0 \ud14c\ub7ec\uacf5\uaca9 \ubc29\uc9c0\ub97c \uc704\ud55c \ubcf4\uc548\uc774 \ud55c\uce35 \uac15\ud654\ub418\uc5c8\ub2e4. \ub610 \ub300\ud68c \uac1c\ub9c9\uc2dd\uc5d0\uc11c\ub294 \uad6d\uae30 \uac8c\uc591\uacfc \uad6d\uac00 \uc81c\ucc3d \ucc28\ub840\uc5d0, \uadf8\ub77c\uc6b4\ub4dc \uc81c\ub85c\uc5d0\uc11c \ubc1c\uacac\ub41c \ucc22\uaca8\uc9c4 \uc131\uc870\uae30\ub97c \ub274\uc695 \uba85\uc608\uacbd\ucc30\ub300\uc640 \ub274\uc695 \uc18c\ubc29\ub300\uc6d0\ub4e4\uc774 \ub4e4\uace0 \uc788\ub294 \ud37c\ud3ec\uba3c\uc2a4\ub97c \ud3bc\ucce4\ub294\ub370, \uc774\uc5d0 \ub530\ub978 \ub17c\ub780\ub3c4 \uc788\uc5c8\ub2e4. \ub3c5\uc77c\uc758 \uac8c\uc624\ub974\uadf8 \ud558\ud074\uc740 \uc774\ubc88 \ub300\ud68c \ub8e8\uc9c0 \uc2f1\uae00 \uc885\ubaa9\uc5d0\uc11c \uc740\uba54\ub2ec\uc744 \ud68d\ub4dd\ud568\uc73c\ub85c\uc11c, \uac1c\uc778\uc885\ubaa9\uc5d0\uc11c \uc62c\ub9bc\ud53d \ub300\ud68c 5\uc5f0\uc18d \uba54\ub2ec \ud68d\ub4dd\uc5d0 \uc131\uacf5\ud55c \ucd5c\ucd08\uc758 \uc120\uc218\uac00 \ub418\uc5c8\ub2e4. \uce90\ub098\ub2e4\ub294 \ub0a8\uc790 \uc544\uc774\uc2a4\ud558\ud0a4\uc640 \uc5ec\uc790 \uc544\uc774\uc2a4\ud558\ud0a4\ub97c \ubaa8\ub450 \uc11d\uad8c\ud558\ub294, \uc804\ub840\uc5c6\ub294 \uae30\ub85d\uc744 \ub2ec\uc131\ud558\uba70 \uac15\uad6d\uc758 \uc785\uc9c0\ub97c \uad73\ud614\ub2e4. \ud55c\ud3b8 \uce90\ub098\ub2e4\ub294 \ub7ec\uc2dc\uc544\uc640 \ud568\uaed8 \ud53c\uaca8\uc2a4\ucf00\uc774\ud305 \ud398\uc5b4 \uacbd\uae30\uc5d0\uc11c \uc11d\uc5f0\uce58 \uc54a\uc740 \uc2ec\ud310 \ud310\uc815\uc73c\ub85c \ud070 \ub17c\ub780\uc758 \ub2f9\uc0ac\uc790\uac00 \ub418\uc5c8\ub2e4. \ub2f9\uc2dc \ub7ec\uc2dc\uc544\uc758 \uc610\ub808\ub098 \ubca0\ub808\uc988\ubc14\ub098, \uc548\ud1a4 \uc2dc\ud558\ub8f0\ub9ac\uc81c\uc640 \uce90\ub098\ub2e4\uc758 \uc81c\uc774\ubbf8 \uc0b4\ub808, \ub370\uc774\ube44\ub4dc \ud3a0\ud2f0\uc5b4\uac00 \uae08\uba54\ub2ec\uc744 \ub193\uace0 \uacb0\uc804\uc744 \uce58\ub918\ub294\ub370, \uce90\ub098\ub2e4 \uc120\uc218\ub4e4\uc774 \uba85\ubc31\ud558\uac8c \ub354 \ub098\uc740 \uc5f0\uae30\ub97c \ud3bc\ucce4\uc74c\uc5d0\ub3c4 \ubd88\uad6c\ud558\uace0 \ub7ec\uc2dc\uc544\uac00 \uc6b0\uc2b9\uc744 \ucc28\uc9c0\ud558\uc600\ub2e4. \uacbd\uae30 \ub2f9\uc2dc \uc2ec\ud310\uc9c4\ub4e4\uc740 \uc644\ubcbd\ud55c \ub0c9\uc804 \uad6c\ub3c4\ub85c \uac08\ub77c\uc838, \uc61b \uacf5\uc0b0\uad8c \uad6d\uac00 \ucd9c\uc2e0 \uc2ec\ud310\ub4e4\uc740 \ub7ec\uc2dc\uc544\uc758 \uc190\uc744, \uc11c\ubc29 \uad6d\uac00 \ucd9c\uc2e0 \uc2ec\ud310\ub4e4\uc740 \uce90\ub098\ub2e4\uc758 \uc190\uc744 \ub4e4\uc5b4\uc8fc\uc5c8\ub2e4. \ud504\ub791\uc2a4\uc758 \ub9c8\ub9ac\ub80c \ub974 \uad6c\ub274 \uc2ec\ud310\ub9cc\uc740 \uc608\uc678\ub85c \ub7ec\uc2dc\uc544\uc758 \uc6b0\uc2b9\uc744 \ud655\uc815\ud558\uc600\ub294\ub370 \ub300\ud68c \ud6c4 \uc804\uc218\uc870\uc0ac\ub97c \uc9c4\ud589\ud55c \uacb0\uacfc, \uc544\uc774\uc2a4\ub304\uc2f1\uc5d0 \ucc38\uac00\ud558\ub294 \ud504\ub791\uc2a4 \uc120\uc218\uc5d0\uac8c \ub7ec\uc2dc\uc544 \uc2ec\ud310\uc9c4\uc774 \ud2b9\ud61c\ub97c \uc8fc\ub294 \ub300\uac00\ub85c, \uad6c\ub274 \uc2ec\ud310\uc774 \uacbd\uae30\uc9c4\ud589\uacfc\ub294 \ubb34\uad00\ud558\uac8c \ub7ec\uc2dc\uc544\uc5d0\uac8c \uae08\uba54\ub2ec\uc774 \ub3cc\uc544\uac00\ub3c4\ub85d \uc555\ub825\ubc1b\uc558\ub2e4\ub294 \uc0ac\uc2e4\uc774 \ub4dc\ub7ec\ub098 \ucda9\uaca9\uc744 \uc92c\ub2e4. \uc774\uc5d0 IOC\ub294 2\ucc28 \uba54\ub2ec \uc218\uc5ec\uc2dd\uc744 \uc5f4\uc5b4 \uce90\ub098\ub2e4 \uc120\uc218\ub4e4\uc5d0\uac8c\ub3c4 \uae08\uba54\ub2ec\uc744 \uc218\uc5ec\ud558\uace0, \ub450 \uad6d\uac00 \ubaa8\ub450 \uc6b0\uc2b9 \ucc98\ub9ac\ud558\ub294 \uc870\uce58\ub97c \ucde8\ud588\ub2e4. \uc1fc\ud2b8\ud2b8\ub799 \ub0a8\uc790 1,500m\uc5d0\uc11c\ub3c4 \ub300\ud55c\ubbfc\uad6d\uc758 \uae40\ub3d9\uc131 \uc120\uc218\uac00 \ubbf8\uad6d\uc758 \uc548\ud1a4 \uc624\ub178 \uc120\uc218\uc5d0\uac8c \ubc00\ub824 \ud0c8\ub77d \ucc98\ub9ac\ub41c, \uc774\ub978\ubc14 '\ud5d0\ub9ac\uc6b0\ub4dc \uc561\uc158' \ub17c\ub780\uc774 \uc77c\uc5c8\uc73c\uba70, \ub0a8\uc790 1,000m\uc5d0\uc11c\ub294 \uc624\uc2a4\ud2b8\ub808\uc77c\ub9ac\uc544\uc758 \uc2a4\ud2f0\ube10 \ube0c\ub798\ub4dc\ubc84\ub9ac\uac00 \uc0c1\ub2f9\ud788 \uadf9\uc801\uc778 \uacbd\uae30 \ub05d\uc5d0 \ub0a8\ubc18\uad6c \uad6d\uac00\ub85c\uc11c\ub294 \ucd5c\ucd08\uc758 \ub3d9\uacc4\uc62c\ub9bc\ud53d \uae08\uba54\ub2ec\ub9ac\uc2a4\ud2b8\uac00 \ub418\ub294 \uc601\uad11\uc744 \uc548\uc558\ub2e4.", "answer": "9\u00b711 \ud14c\ub7ec", "sentence": "\uc774 \ub300\ud68c\ub294 9\u00b711 \ud14c\ub7ec \ub97c \uacaa\uc740 \uc9c1\ud6c4\uc5d0 \uc5f4\ub9b0 \uccab \uc62c\ub9bc\ud53d \ub300\ud68c\uc600\uae30 \ub54c\ubb38\uc5d0 \ud14c\ub7ec\uacf5\uaca9 \ubc29\uc9c0\ub97c \uc704\ud55c \ubcf4\uc548\uc774 \ud55c\uce35 \uac15\ud654\ub418\uc5c8\ub2e4.", "paragraph_sentence": "2002\ub144 \ub3d9\uacc4 \uc62c\ub9bc\ud53d\uc740 \ubbf8\uad6d \uc720\ud0c0 \uc8fc \uc194\ud2b8\ub808\uc774\ud06c\uc2dc\ud2f0\uc5d0\uc11c \uc5f4\ub838\ub2e4. \ucd1d 77\uac1c \ucc38\uac00\uad6d 2,399\uba85 \uc120\uc218\uac00 7\uac1c \uc885\ubaa9 78\uac1c \uacbd\uae30\uc5d0 \ucd9c\uc804\ud558\uc600\ub2e4. \uc774 \ub300\ud68c\ub294 9\u00b711 \ud14c\ub7ec \ub97c \uacaa\uc740 \uc9c1\ud6c4\uc5d0 \uc5f4\ub9b0 \uccab \uc62c\ub9bc\ud53d \ub300\ud68c\uc600\uae30 \ub54c\ubb38\uc5d0 \ud14c\ub7ec\uacf5\uaca9 \ubc29\uc9c0\ub97c \uc704\ud55c \ubcf4\uc548\uc774 \ud55c\uce35 \uac15\ud654\ub418\uc5c8\ub2e4. \ub610 \ub300\ud68c \uac1c\ub9c9\uc2dd\uc5d0\uc11c\ub294 \uad6d\uae30 \uac8c\uc591\uacfc \uad6d\uac00 \uc81c\ucc3d \ucc28\ub840\uc5d0, \uadf8\ub77c\uc6b4\ub4dc \uc81c\ub85c\uc5d0\uc11c \ubc1c\uacac\ub41c \ucc22\uaca8\uc9c4 \uc131\uc870\uae30\ub97c \ub274\uc695 \uba85\uc608\uacbd\ucc30\ub300\uc640 \ub274\uc695 \uc18c\ubc29\ub300\uc6d0\ub4e4\uc774 \ub4e4\uace0 \uc788\ub294 \ud37c\ud3ec\uba3c\uc2a4\ub97c \ud3bc\ucce4\ub294\ub370, \uc774\uc5d0 \ub530\ub978 \ub17c\ub780\ub3c4 \uc788\uc5c8\ub2e4. \ub3c5\uc77c\uc758 \uac8c\uc624\ub974\uadf8 \ud558\ud074\uc740 \uc774\ubc88 \ub300\ud68c \ub8e8\uc9c0 \uc2f1\uae00 \uc885\ubaa9\uc5d0\uc11c \uc740\uba54\ub2ec\uc744 \ud68d\ub4dd\ud568\uc73c\ub85c\uc11c, \uac1c\uc778\uc885\ubaa9\uc5d0\uc11c \uc62c\ub9bc\ud53d \ub300\ud68c 5\uc5f0\uc18d \uba54\ub2ec \ud68d\ub4dd\uc5d0 \uc131\uacf5\ud55c \ucd5c\ucd08\uc758 \uc120\uc218\uac00 \ub418\uc5c8\ub2e4. \uce90\ub098\ub2e4\ub294 \ub0a8\uc790 \uc544\uc774\uc2a4\ud558\ud0a4\uc640 \uc5ec\uc790 \uc544\uc774\uc2a4\ud558\ud0a4\ub97c \ubaa8\ub450 \uc11d\uad8c\ud558\ub294, \uc804\ub840\uc5c6\ub294 \uae30\ub85d\uc744 \ub2ec\uc131\ud558\uba70 \uac15\uad6d\uc758 \uc785\uc9c0\ub97c \uad73\ud614\ub2e4. \ud55c\ud3b8 \uce90\ub098\ub2e4\ub294 \ub7ec\uc2dc\uc544\uc640 \ud568\uaed8 \ud53c\uaca8\uc2a4\ucf00\uc774\ud305 \ud398\uc5b4 \uacbd\uae30\uc5d0\uc11c \uc11d\uc5f0\uce58 \uc54a\uc740 \uc2ec\ud310 \ud310\uc815\uc73c\ub85c \ud070 \ub17c\ub780\uc758 \ub2f9\uc0ac\uc790\uac00 \ub418\uc5c8\ub2e4. \ub2f9\uc2dc \ub7ec\uc2dc\uc544\uc758 \uc610\ub808\ub098 \ubca0\ub808\uc988\ubc14\ub098, \uc548\ud1a4 \uc2dc\ud558\ub8f0\ub9ac\uc81c\uc640 \uce90\ub098\ub2e4\uc758 \uc81c\uc774\ubbf8 \uc0b4\ub808, \ub370\uc774\ube44\ub4dc \ud3a0\ud2f0\uc5b4\uac00 \uae08\uba54\ub2ec\uc744 \ub193\uace0 \uacb0\uc804\uc744 \uce58\ub918\ub294\ub370, \uce90\ub098\ub2e4 \uc120\uc218\ub4e4\uc774 \uba85\ubc31\ud558\uac8c \ub354 \ub098\uc740 \uc5f0\uae30\ub97c \ud3bc\ucce4\uc74c\uc5d0\ub3c4 \ubd88\uad6c\ud558\uace0 \ub7ec\uc2dc\uc544\uac00 \uc6b0\uc2b9\uc744 \ucc28\uc9c0\ud558\uc600\ub2e4. \uacbd\uae30 \ub2f9\uc2dc \uc2ec\ud310\uc9c4\ub4e4\uc740 \uc644\ubcbd\ud55c \ub0c9\uc804 \uad6c\ub3c4\ub85c \uac08\ub77c\uc838, \uc61b \uacf5\uc0b0\uad8c \uad6d\uac00 \ucd9c\uc2e0 \uc2ec\ud310\ub4e4\uc740 \ub7ec\uc2dc\uc544\uc758 \uc190\uc744, \uc11c\ubc29 \uad6d\uac00 \ucd9c\uc2e0 \uc2ec\ud310\ub4e4\uc740 \uce90\ub098\ub2e4\uc758 \uc190\uc744 \ub4e4\uc5b4\uc8fc\uc5c8\ub2e4. \ud504\ub791\uc2a4\uc758 \ub9c8\ub9ac\ub80c \ub974 \uad6c\ub274 \uc2ec\ud310\ub9cc\uc740 \uc608\uc678\ub85c \ub7ec\uc2dc\uc544\uc758 \uc6b0\uc2b9\uc744 \ud655\uc815\ud558\uc600\ub294\ub370 \ub300\ud68c \ud6c4 \uc804\uc218\uc870\uc0ac\ub97c \uc9c4\ud589\ud55c \uacb0\uacfc, \uc544\uc774\uc2a4\ub304\uc2f1\uc5d0 \ucc38\uac00\ud558\ub294 \ud504\ub791\uc2a4 \uc120\uc218\uc5d0\uac8c \ub7ec\uc2dc\uc544 \uc2ec\ud310\uc9c4\uc774 \ud2b9\ud61c\ub97c \uc8fc\ub294 \ub300\uac00\ub85c, \uad6c\ub274 \uc2ec\ud310\uc774 \uacbd\uae30\uc9c4\ud589\uacfc\ub294 \ubb34\uad00\ud558\uac8c \ub7ec\uc2dc\uc544\uc5d0\uac8c \uae08\uba54\ub2ec\uc774 \ub3cc\uc544\uac00\ub3c4\ub85d \uc555\ub825\ubc1b\uc558\ub2e4\ub294 \uc0ac\uc2e4\uc774 \ub4dc\ub7ec\ub098 \ucda9\uaca9\uc744 \uc92c\ub2e4. \uc774\uc5d0 IOC\ub294 2\ucc28 \uba54\ub2ec \uc218\uc5ec\uc2dd\uc744 \uc5f4\uc5b4 \uce90\ub098\ub2e4 \uc120\uc218\ub4e4\uc5d0\uac8c\ub3c4 \uae08\uba54\ub2ec\uc744 \uc218\uc5ec\ud558\uace0, \ub450 \uad6d\uac00 \ubaa8\ub450 \uc6b0\uc2b9 \ucc98\ub9ac\ud558\ub294 \uc870\uce58\ub97c \ucde8\ud588\ub2e4. \uc1fc\ud2b8\ud2b8\ub799 \ub0a8\uc790 1,500m\uc5d0\uc11c\ub3c4 \ub300\ud55c\ubbfc\uad6d\uc758 \uae40\ub3d9\uc131 \uc120\uc218\uac00 \ubbf8\uad6d\uc758 \uc548\ud1a4 \uc624\ub178 \uc120\uc218\uc5d0\uac8c \ubc00\ub824 \ud0c8\ub77d \ucc98\ub9ac\ub41c, \uc774\ub978\ubc14 '\ud5d0\ub9ac\uc6b0\ub4dc \uc561\uc158' \ub17c\ub780\uc774 \uc77c\uc5c8\uc73c\uba70, \ub0a8\uc790 1,000m\uc5d0\uc11c\ub294 \uc624\uc2a4\ud2b8\ub808\uc77c\ub9ac\uc544\uc758 \uc2a4\ud2f0\ube10 \ube0c\ub798\ub4dc\ubc84\ub9ac\uac00 \uc0c1\ub2f9\ud788 \uadf9\uc801\uc778 \uacbd\uae30 \ub05d\uc5d0 \ub0a8\ubc18\uad6c \uad6d\uac00\ub85c\uc11c\ub294 \ucd5c\ucd08\uc758 \ub3d9\uacc4\uc62c\ub9bc\ud53d \uae08\uba54\ub2ec\ub9ac\uc2a4\ud2b8\uac00 \ub418\ub294 \uc601\uad11\uc744 \uc548\uc558\ub2e4.", "paragraph_answer": "2002\ub144 \ub3d9\uacc4 \uc62c\ub9bc\ud53d\uc740 \ubbf8\uad6d \uc720\ud0c0 \uc8fc \uc194\ud2b8\ub808\uc774\ud06c\uc2dc\ud2f0\uc5d0\uc11c \uc5f4\ub838\ub2e4. \ucd1d 77\uac1c \ucc38\uac00\uad6d 2,399\uba85 \uc120\uc218\uac00 7\uac1c \uc885\ubaa9 78\uac1c \uacbd\uae30\uc5d0 \ucd9c\uc804\ud558\uc600\ub2e4. \uc774 \ub300\ud68c\ub294 9\u00b711 \ud14c\ub7ec \ub97c \uacaa\uc740 \uc9c1\ud6c4\uc5d0 \uc5f4\ub9b0 \uccab \uc62c\ub9bc\ud53d \ub300\ud68c\uc600\uae30 \ub54c\ubb38\uc5d0 \ud14c\ub7ec\uacf5\uaca9 \ubc29\uc9c0\ub97c \uc704\ud55c \ubcf4\uc548\uc774 \ud55c\uce35 \uac15\ud654\ub418\uc5c8\ub2e4. \ub610 \ub300\ud68c \uac1c\ub9c9\uc2dd\uc5d0\uc11c\ub294 \uad6d\uae30 \uac8c\uc591\uacfc \uad6d\uac00 \uc81c\ucc3d \ucc28\ub840\uc5d0, \uadf8\ub77c\uc6b4\ub4dc \uc81c\ub85c\uc5d0\uc11c \ubc1c\uacac\ub41c \ucc22\uaca8\uc9c4 \uc131\uc870\uae30\ub97c \ub274\uc695 \uba85\uc608\uacbd\ucc30\ub300\uc640 \ub274\uc695 \uc18c\ubc29\ub300\uc6d0\ub4e4\uc774 \ub4e4\uace0 \uc788\ub294 \ud37c\ud3ec\uba3c\uc2a4\ub97c \ud3bc\ucce4\ub294\ub370, \uc774\uc5d0 \ub530\ub978 \ub17c\ub780\ub3c4 \uc788\uc5c8\ub2e4. \ub3c5\uc77c\uc758 \uac8c\uc624\ub974\uadf8 \ud558\ud074\uc740 \uc774\ubc88 \ub300\ud68c \ub8e8\uc9c0 \uc2f1\uae00 \uc885\ubaa9\uc5d0\uc11c \uc740\uba54\ub2ec\uc744 \ud68d\ub4dd\ud568\uc73c\ub85c\uc11c, \uac1c\uc778\uc885\ubaa9\uc5d0\uc11c \uc62c\ub9bc\ud53d \ub300\ud68c 5\uc5f0\uc18d \uba54\ub2ec \ud68d\ub4dd\uc5d0 \uc131\uacf5\ud55c \ucd5c\ucd08\uc758 \uc120\uc218\uac00 \ub418\uc5c8\ub2e4. \uce90\ub098\ub2e4\ub294 \ub0a8\uc790 \uc544\uc774\uc2a4\ud558\ud0a4\uc640 \uc5ec\uc790 \uc544\uc774\uc2a4\ud558\ud0a4\ub97c \ubaa8\ub450 \uc11d\uad8c\ud558\ub294, \uc804\ub840\uc5c6\ub294 \uae30\ub85d\uc744 \ub2ec\uc131\ud558\uba70 \uac15\uad6d\uc758 \uc785\uc9c0\ub97c \uad73\ud614\ub2e4. \ud55c\ud3b8 \uce90\ub098\ub2e4\ub294 \ub7ec\uc2dc\uc544\uc640 \ud568\uaed8 \ud53c\uaca8\uc2a4\ucf00\uc774\ud305 \ud398\uc5b4 \uacbd\uae30\uc5d0\uc11c \uc11d\uc5f0\uce58 \uc54a\uc740 \uc2ec\ud310 \ud310\uc815\uc73c\ub85c \ud070 \ub17c\ub780\uc758 \ub2f9\uc0ac\uc790\uac00 \ub418\uc5c8\ub2e4. \ub2f9\uc2dc \ub7ec\uc2dc\uc544\uc758 \uc610\ub808\ub098 \ubca0\ub808\uc988\ubc14\ub098, \uc548\ud1a4 \uc2dc\ud558\ub8f0\ub9ac\uc81c\uc640 \uce90\ub098\ub2e4\uc758 \uc81c\uc774\ubbf8 \uc0b4\ub808, \ub370\uc774\ube44\ub4dc \ud3a0\ud2f0\uc5b4\uac00 \uae08\uba54\ub2ec\uc744 \ub193\uace0 \uacb0\uc804\uc744 \uce58\ub918\ub294\ub370, \uce90\ub098\ub2e4 \uc120\uc218\ub4e4\uc774 \uba85\ubc31\ud558\uac8c \ub354 \ub098\uc740 \uc5f0\uae30\ub97c \ud3bc\ucce4\uc74c\uc5d0\ub3c4 \ubd88\uad6c\ud558\uace0 \ub7ec\uc2dc\uc544\uac00 \uc6b0\uc2b9\uc744 \ucc28\uc9c0\ud558\uc600\ub2e4. \uacbd\uae30 \ub2f9\uc2dc \uc2ec\ud310\uc9c4\ub4e4\uc740 \uc644\ubcbd\ud55c \ub0c9\uc804 \uad6c\ub3c4\ub85c \uac08\ub77c\uc838, \uc61b \uacf5\uc0b0\uad8c \uad6d\uac00 \ucd9c\uc2e0 \uc2ec\ud310\ub4e4\uc740 \ub7ec\uc2dc\uc544\uc758 \uc190\uc744, \uc11c\ubc29 \uad6d\uac00 \ucd9c\uc2e0 \uc2ec\ud310\ub4e4\uc740 \uce90\ub098\ub2e4\uc758 \uc190\uc744 \ub4e4\uc5b4\uc8fc\uc5c8\ub2e4. \ud504\ub791\uc2a4\uc758 \ub9c8\ub9ac\ub80c \ub974 \uad6c\ub274 \uc2ec\ud310\ub9cc\uc740 \uc608\uc678\ub85c \ub7ec\uc2dc\uc544\uc758 \uc6b0\uc2b9\uc744 \ud655\uc815\ud558\uc600\ub294\ub370 \ub300\ud68c \ud6c4 \uc804\uc218\uc870\uc0ac\ub97c \uc9c4\ud589\ud55c \uacb0\uacfc, \uc544\uc774\uc2a4\ub304\uc2f1\uc5d0 \ucc38\uac00\ud558\ub294 \ud504\ub791\uc2a4 \uc120\uc218\uc5d0\uac8c \ub7ec\uc2dc\uc544 \uc2ec\ud310\uc9c4\uc774 \ud2b9\ud61c\ub97c \uc8fc\ub294 \ub300\uac00\ub85c, \uad6c\ub274 \uc2ec\ud310\uc774 \uacbd\uae30\uc9c4\ud589\uacfc\ub294 \ubb34\uad00\ud558\uac8c \ub7ec\uc2dc\uc544\uc5d0\uac8c \uae08\uba54\ub2ec\uc774 \ub3cc\uc544\uac00\ub3c4\ub85d \uc555\ub825\ubc1b\uc558\ub2e4\ub294 \uc0ac\uc2e4\uc774 \ub4dc\ub7ec\ub098 \ucda9\uaca9\uc744 \uc92c\ub2e4. \uc774\uc5d0 IOC\ub294 2\ucc28 \uba54\ub2ec \uc218\uc5ec\uc2dd\uc744 \uc5f4\uc5b4 \uce90\ub098\ub2e4 \uc120\uc218\ub4e4\uc5d0\uac8c\ub3c4 \uae08\uba54\ub2ec\uc744 \uc218\uc5ec\ud558\uace0, \ub450 \uad6d\uac00 \ubaa8\ub450 \uc6b0\uc2b9 \ucc98\ub9ac\ud558\ub294 \uc870\uce58\ub97c \ucde8\ud588\ub2e4. \uc1fc\ud2b8\ud2b8\ub799 \ub0a8\uc790 1,500m\uc5d0\uc11c\ub3c4 \ub300\ud55c\ubbfc\uad6d\uc758 \uae40\ub3d9\uc131 \uc120\uc218\uac00 \ubbf8\uad6d\uc758 \uc548\ud1a4 \uc624\ub178 \uc120\uc218\uc5d0\uac8c \ubc00\ub824 \ud0c8\ub77d \ucc98\ub9ac\ub41c, \uc774\ub978\ubc14 '\ud5d0\ub9ac\uc6b0\ub4dc \uc561\uc158' \ub17c\ub780\uc774 \uc77c\uc5c8\uc73c\uba70, \ub0a8\uc790 1,000m\uc5d0\uc11c\ub294 \uc624\uc2a4\ud2b8\ub808\uc77c\ub9ac\uc544\uc758 \uc2a4\ud2f0\ube10 \ube0c\ub798\ub4dc\ubc84\ub9ac\uac00 \uc0c1\ub2f9\ud788 \uadf9\uc801\uc778 \uacbd\uae30 \ub05d\uc5d0 \ub0a8\ubc18\uad6c \uad6d\uac00\ub85c\uc11c\ub294 \ucd5c\ucd08\uc758 \ub3d9\uacc4\uc62c\ub9bc\ud53d \uae08\uba54\ub2ec\ub9ac\uc2a4\ud2b8\uac00 \ub418\ub294 \uc601\uad11\uc744 \uc548\uc558\ub2e4.", "sentence_answer": "\uc774 \ub300\ud68c\ub294 9\u00b711 \ud14c\ub7ec \ub97c \uacaa\uc740 \uc9c1\ud6c4\uc5d0 \uc5f4\ub9b0 \uccab \uc62c\ub9bc\ud53d \ub300\ud68c\uc600\uae30 \ub54c\ubb38\uc5d0 \ud14c\ub7ec\uacf5\uaca9 \ubc29\uc9c0\ub97c \uc704\ud55c \ubcf4\uc548\uc774 \ud55c\uce35 \uac15\ud654\ub418\uc5c8\ub2e4.", "paragraph_id": "6534018-14-0", "paragraph_question": "question: 2002\ub144 \ub3d9\uacc4 \uc62c\ub9bc\ud53d \uc9c1\uc804 \ubc8c\uc5b4\uc838 \ud14c\ub7ec\uacf5\uaca9 \ubc29\uc9c0\ub97c \uc704\ud55c \ubcf4\uc548\uc744 \uac15\ud654\ud558\ub294 \uacc4\uae30\uac00 \ub41c \uc0ac\uac74\uc740?, context: 2002\ub144 \ub3d9\uacc4 \uc62c\ub9bc\ud53d\uc740 \ubbf8\uad6d \uc720\ud0c0 \uc8fc \uc194\ud2b8\ub808\uc774\ud06c\uc2dc\ud2f0\uc5d0\uc11c \uc5f4\ub838\ub2e4. \ucd1d 77\uac1c \ucc38\uac00\uad6d 2,399\uba85 \uc120\uc218\uac00 7\uac1c \uc885\ubaa9 78\uac1c \uacbd\uae30\uc5d0 \ucd9c\uc804\ud558\uc600\ub2e4. \uc774 \ub300\ud68c\ub294 9\u00b711 \ud14c\ub7ec\ub97c \uacaa\uc740 \uc9c1\ud6c4\uc5d0 \uc5f4\ub9b0 \uccab \uc62c\ub9bc\ud53d \ub300\ud68c\uc600\uae30 \ub54c\ubb38\uc5d0 \ud14c\ub7ec\uacf5\uaca9 \ubc29\uc9c0\ub97c \uc704\ud55c \ubcf4\uc548\uc774 \ud55c\uce35 \uac15\ud654\ub418\uc5c8\ub2e4. \ub610 \ub300\ud68c \uac1c\ub9c9\uc2dd\uc5d0\uc11c\ub294 \uad6d\uae30 \uac8c\uc591\uacfc \uad6d\uac00 \uc81c\ucc3d \ucc28\ub840\uc5d0, \uadf8\ub77c\uc6b4\ub4dc \uc81c\ub85c\uc5d0\uc11c \ubc1c\uacac\ub41c \ucc22\uaca8\uc9c4 \uc131\uc870\uae30\ub97c \ub274\uc695 \uba85\uc608\uacbd\ucc30\ub300\uc640 \ub274\uc695 \uc18c\ubc29\ub300\uc6d0\ub4e4\uc774 \ub4e4\uace0 \uc788\ub294 \ud37c\ud3ec\uba3c\uc2a4\ub97c \ud3bc\ucce4\ub294\ub370, \uc774\uc5d0 \ub530\ub978 \ub17c\ub780\ub3c4 \uc788\uc5c8\ub2e4. \ub3c5\uc77c\uc758 \uac8c\uc624\ub974\uadf8 \ud558\ud074\uc740 \uc774\ubc88 \ub300\ud68c \ub8e8\uc9c0 \uc2f1\uae00 \uc885\ubaa9\uc5d0\uc11c \uc740\uba54\ub2ec\uc744 \ud68d\ub4dd\ud568\uc73c\ub85c\uc11c, \uac1c\uc778\uc885\ubaa9\uc5d0\uc11c \uc62c\ub9bc\ud53d \ub300\ud68c 5\uc5f0\uc18d \uba54\ub2ec \ud68d\ub4dd\uc5d0 \uc131\uacf5\ud55c \ucd5c\ucd08\uc758 \uc120\uc218\uac00 \ub418\uc5c8\ub2e4. \uce90\ub098\ub2e4\ub294 \ub0a8\uc790 \uc544\uc774\uc2a4\ud558\ud0a4\uc640 \uc5ec\uc790 \uc544\uc774\uc2a4\ud558\ud0a4\ub97c \ubaa8\ub450 \uc11d\uad8c\ud558\ub294, \uc804\ub840\uc5c6\ub294 \uae30\ub85d\uc744 \ub2ec\uc131\ud558\uba70 \uac15\uad6d\uc758 \uc785\uc9c0\ub97c \uad73\ud614\ub2e4. \ud55c\ud3b8 \uce90\ub098\ub2e4\ub294 \ub7ec\uc2dc\uc544\uc640 \ud568\uaed8 \ud53c\uaca8\uc2a4\ucf00\uc774\ud305 \ud398\uc5b4 \uacbd\uae30\uc5d0\uc11c \uc11d\uc5f0\uce58 \uc54a\uc740 \uc2ec\ud310 \ud310\uc815\uc73c\ub85c \ud070 \ub17c\ub780\uc758 \ub2f9\uc0ac\uc790\uac00 \ub418\uc5c8\ub2e4. \ub2f9\uc2dc \ub7ec\uc2dc\uc544\uc758 \uc610\ub808\ub098 \ubca0\ub808\uc988\ubc14\ub098, \uc548\ud1a4 \uc2dc\ud558\ub8f0\ub9ac\uc81c\uc640 \uce90\ub098\ub2e4\uc758 \uc81c\uc774\ubbf8 \uc0b4\ub808, \ub370\uc774\ube44\ub4dc \ud3a0\ud2f0\uc5b4\uac00 \uae08\uba54\ub2ec\uc744 \ub193\uace0 \uacb0\uc804\uc744 \uce58\ub918\ub294\ub370, \uce90\ub098\ub2e4 \uc120\uc218\ub4e4\uc774 \uba85\ubc31\ud558\uac8c \ub354 \ub098\uc740 \uc5f0\uae30\ub97c \ud3bc\ucce4\uc74c\uc5d0\ub3c4 \ubd88\uad6c\ud558\uace0 \ub7ec\uc2dc\uc544\uac00 \uc6b0\uc2b9\uc744 \ucc28\uc9c0\ud558\uc600\ub2e4. \uacbd\uae30 \ub2f9\uc2dc \uc2ec\ud310\uc9c4\ub4e4\uc740 \uc644\ubcbd\ud55c \ub0c9\uc804 \uad6c\ub3c4\ub85c \uac08\ub77c\uc838, \uc61b \uacf5\uc0b0\uad8c \uad6d\uac00 \ucd9c\uc2e0 \uc2ec\ud310\ub4e4\uc740 \ub7ec\uc2dc\uc544\uc758 \uc190\uc744, \uc11c\ubc29 \uad6d\uac00 \ucd9c\uc2e0 \uc2ec\ud310\ub4e4\uc740 \uce90\ub098\ub2e4\uc758 \uc190\uc744 \ub4e4\uc5b4\uc8fc\uc5c8\ub2e4. \ud504\ub791\uc2a4\uc758 \ub9c8\ub9ac\ub80c \ub974 \uad6c\ub274 \uc2ec\ud310\ub9cc\uc740 \uc608\uc678\ub85c \ub7ec\uc2dc\uc544\uc758 \uc6b0\uc2b9\uc744 \ud655\uc815\ud558\uc600\ub294\ub370 \ub300\ud68c \ud6c4 \uc804\uc218\uc870\uc0ac\ub97c \uc9c4\ud589\ud55c \uacb0\uacfc, \uc544\uc774\uc2a4\ub304\uc2f1\uc5d0 \ucc38\uac00\ud558\ub294 \ud504\ub791\uc2a4 \uc120\uc218\uc5d0\uac8c \ub7ec\uc2dc\uc544 \uc2ec\ud310\uc9c4\uc774 \ud2b9\ud61c\ub97c \uc8fc\ub294 \ub300\uac00\ub85c, \uad6c\ub274 \uc2ec\ud310\uc774 \uacbd\uae30\uc9c4\ud589\uacfc\ub294 \ubb34\uad00\ud558\uac8c \ub7ec\uc2dc\uc544\uc5d0\uac8c \uae08\uba54\ub2ec\uc774 \ub3cc\uc544\uac00\ub3c4\ub85d \uc555\ub825\ubc1b\uc558\ub2e4\ub294 \uc0ac\uc2e4\uc774 \ub4dc\ub7ec\ub098 \ucda9\uaca9\uc744 \uc92c\ub2e4. \uc774\uc5d0 IOC\ub294 2\ucc28 \uba54\ub2ec \uc218\uc5ec\uc2dd\uc744 \uc5f4\uc5b4 \uce90\ub098\ub2e4 \uc120\uc218\ub4e4\uc5d0\uac8c\ub3c4 \uae08\uba54\ub2ec\uc744 \uc218\uc5ec\ud558\uace0, \ub450 \uad6d\uac00 \ubaa8\ub450 \uc6b0\uc2b9 \ucc98\ub9ac\ud558\ub294 \uc870\uce58\ub97c \ucde8\ud588\ub2e4. \uc1fc\ud2b8\ud2b8\ub799 \ub0a8\uc790 1,500m\uc5d0\uc11c\ub3c4 \ub300\ud55c\ubbfc\uad6d\uc758 \uae40\ub3d9\uc131 \uc120\uc218\uac00 \ubbf8\uad6d\uc758 \uc548\ud1a4 \uc624\ub178 \uc120\uc218\uc5d0\uac8c \ubc00\ub824 \ud0c8\ub77d \ucc98\ub9ac\ub41c, \uc774\ub978\ubc14 '\ud5d0\ub9ac\uc6b0\ub4dc \uc561\uc158' \ub17c\ub780\uc774 \uc77c\uc5c8\uc73c\uba70, \ub0a8\uc790 1,000m\uc5d0\uc11c\ub294 \uc624\uc2a4\ud2b8\ub808\uc77c\ub9ac\uc544\uc758 \uc2a4\ud2f0\ube10 \ube0c\ub798\ub4dc\ubc84\ub9ac\uac00 \uc0c1\ub2f9\ud788 \uadf9\uc801\uc778 \uacbd\uae30 \ub05d\uc5d0 \ub0a8\ubc18\uad6c \uad6d\uac00\ub85c\uc11c\ub294 \ucd5c\ucd08\uc758 \ub3d9\uacc4\uc62c\ub9bc\ud53d \uae08\uba54\ub2ec\ub9ac\uc2a4\ud2b8\uac00 \ub418\ub294 \uc601\uad11\uc744 \uc548\uc558\ub2e4."} {"question": "\uc774\ub9cc\uc12d\uc774 \uc8fc\ub3c4\ud55c \uc601\uacb0\uc2dd\uc740 \uc5b4\ub514\uc5d0\uc11c \uc9c4\ud589\ub418\uc5c8\ub294\uac00?", "paragraph": "\uc720\ud574\ub294 \ube44\ud589\uae30\ud3b8\uc73c\ub85c \uadc0\uad6d, 1994\ub144 1\uc6d4 22\uc77c \uc624\uc804 \uad6d\ud68c\uc758\uc0ac\ub2f9\uc55e \uad11\uc7a5 \uc55e\uc5d0\uc11c \uad6d\ud68c\uc758\uc6d0 \uc774\ub9cc\uc12d\uc758 \uc8fc\ub3c4\ub85c \uc601\uacb0\uc2dd\uacfc \ub178\uc81c\ub97c \uac70\ud589\ud55c \ub4a4, \uacbd\ucc30\ubcd1\ub825\uc758 \ud638\uc1a1\ud558\uc5d0 \uc11c\uc6b8\ud2b9\ubcc4\uc2dc \ub3d9\uc791\uad6c \ub3d9\uc791\ub3d9 \uc0b044-7 \ub3d9\uc791\ub3d9 \ud604\ucda9\uc6d0 \uc7a5\uad70 \uc81c3 \ubb18\uc5ed\uc5d0 \ubb3b\ud614\ub2e4. \ud55c\ud3b8 \uadf8\uc758 \uad11\uba85\uc911\ud559\uad50 \ub3d9\ubb38\uc778 \ubb38\uc775\ud658\uc774 \uac19\uc740 \ub0a0 \uc138\uc0c1\uc744 \ub5a0\ub0ac\ub294\ub370, \ubb38\uc775\ud658\ubaa9\uc0ac\uc758 \uc601\uacb0\uc2dd\uc740 \ud55c\uc2e0\ub300\uc5d0\uc11c \uc2dc\uc791\ud574, \ub300\ud559\ub85c, \ub3d9\ub300\ubb38\uc5d0 \uc774\ub974\uae30\uae4c\uc9c0 \uc218\uc2ed\ub9cc\uba85\uc774 \ucc38\uc5ec\ud55c \ub178\uc81c\ub85c \uce58\ub7ec\uc9c4 \ubc18\uba74 \uc815\uc77c\uad8c\uc758 \uc7a5\ub840\uc2dd\uc740 \ud604\ucda9\uc6d0\uc5d0\uc11c \uc815\uce58\uc778\ub4e4\uc774 \ucc38\uac00\ud55c \uac00\uc6b4\ub370 \uc774\ub8e8\uc5b4\uc838 TV\ub274\uc2a4\uc5d0\uc11c \ub300\ube44\ub418\uc5c8\ub2e4. \uadf8\ub4e4\uc740 \ud0dc\uc5b4\ub09c \ud574\ub3c4 \uac19\uace0, \uac19\uc740 \uc911\ud559\uad50\ub97c \ub2e4\ub154\uc73c\ub098 \uc11c\ub85c \ub2e4\ub978 \uc778\uc0dd\uc744 \uc0b4\uc544\uac14\uc73c\uba70, \uac19\uc740 \ub0a0 \uc0ac\ub9dd\ud558\uba70 \ub9ce\uc740 \uc0ac\ub78c\ub4e4\uc5d0\uac8c \uae4a\uc740 \uc778\uc0c1\uc744 \ub0a8\uacbc\ub2e4.", "answer": "\uad6d\ud68c\uc758\uc0ac\ub2f9\uc55e", "sentence": "\uc720\ud574\ub294 \ube44\ud589\uae30\ud3b8\uc73c\ub85c \uadc0\uad6d, 1994\ub144 1\uc6d4 22\uc77c \uc624\uc804 \uad6d\ud68c\uc758\uc0ac\ub2f9\uc55e \uad11\uc7a5 \uc55e\uc5d0\uc11c \uad6d\ud68c\uc758\uc6d0 \uc774\ub9cc\uc12d\uc758 \uc8fc\ub3c4\ub85c \uc601\uacb0\uc2dd\uacfc \ub178\uc81c\ub97c \uac70\ud589\ud55c \ub4a4, \uacbd\ucc30\ubcd1\ub825\uc758 \ud638\uc1a1\ud558\uc5d0 \uc11c\uc6b8\ud2b9\ubcc4\uc2dc \ub3d9\uc791\uad6c \ub3d9\uc791\ub3d9 \uc0b044-7 \ub3d9\uc791\ub3d9 \ud604\ucda9\uc6d0 \uc7a5\uad70 \uc81c3 \ubb18\uc5ed\uc5d0 \ubb3b\ud614\ub2e4.", "paragraph_sentence": " \uc720\ud574\ub294 \ube44\ud589\uae30\ud3b8\uc73c\ub85c \uadc0\uad6d, 1994\ub144 1\uc6d4 22\uc77c \uc624\uc804 \uad6d\ud68c\uc758\uc0ac\ub2f9\uc55e \uad11\uc7a5 \uc55e\uc5d0\uc11c \uad6d\ud68c\uc758\uc6d0 \uc774\ub9cc\uc12d\uc758 \uc8fc\ub3c4\ub85c \uc601\uacb0\uc2dd\uacfc \ub178\uc81c\ub97c \uac70\ud589\ud55c \ub4a4, \uacbd\ucc30\ubcd1\ub825\uc758 \ud638\uc1a1\ud558\uc5d0 \uc11c\uc6b8\ud2b9\ubcc4\uc2dc \ub3d9\uc791\uad6c \ub3d9\uc791\ub3d9 \uc0b044-7 \ub3d9\uc791\ub3d9 \ud604\ucda9\uc6d0 \uc7a5\uad70 \uc81c3 \ubb18\uc5ed\uc5d0 \ubb3b\ud614\ub2e4. \ud55c\ud3b8 \uadf8\uc758 \uad11\uba85\uc911\ud559\uad50 \ub3d9\ubb38\uc778 \ubb38\uc775\ud658\uc774 \uac19\uc740 \ub0a0 \uc138\uc0c1\uc744 \ub5a0\ub0ac\ub294\ub370, \ubb38\uc775\ud658\ubaa9\uc0ac\uc758 \uc601\uacb0\uc2dd\uc740 \ud55c\uc2e0\ub300\uc5d0\uc11c \uc2dc\uc791\ud574, \ub300\ud559\ub85c, \ub3d9\ub300\ubb38\uc5d0 \uc774\ub974\uae30\uae4c\uc9c0 \uc218\uc2ed\ub9cc\uba85\uc774 \ucc38\uc5ec\ud55c \ub178\uc81c\ub85c \uce58\ub7ec\uc9c4 \ubc18\uba74 \uc815\uc77c\uad8c\uc758 \uc7a5\ub840\uc2dd\uc740 \ud604\ucda9\uc6d0\uc5d0\uc11c \uc815\uce58\uc778\ub4e4\uc774 \ucc38\uac00\ud55c \uac00\uc6b4\ub370 \uc774\ub8e8\uc5b4\uc838 TV\ub274\uc2a4\uc5d0\uc11c \ub300\ube44\ub418\uc5c8\ub2e4. \uadf8\ub4e4\uc740 \ud0dc\uc5b4\ub09c \ud574\ub3c4 \uac19\uace0, \uac19\uc740 \uc911\ud559\uad50\ub97c \ub2e4\ub154\uc73c\ub098 \uc11c\ub85c \ub2e4\ub978 \uc778\uc0dd\uc744 \uc0b4\uc544\uac14\uc73c\uba70, \uac19\uc740 \ub0a0 \uc0ac\ub9dd\ud558\uba70 \ub9ce\uc740 \uc0ac\ub78c\ub4e4\uc5d0\uac8c \uae4a\uc740 \uc778\uc0c1\uc744 \ub0a8\uacbc\ub2e4.", "paragraph_answer": "\uc720\ud574\ub294 \ube44\ud589\uae30\ud3b8\uc73c\ub85c \uadc0\uad6d, 1994\ub144 1\uc6d4 22\uc77c \uc624\uc804 \uad6d\ud68c\uc758\uc0ac\ub2f9\uc55e \uad11\uc7a5 \uc55e\uc5d0\uc11c \uad6d\ud68c\uc758\uc6d0 \uc774\ub9cc\uc12d\uc758 \uc8fc\ub3c4\ub85c \uc601\uacb0\uc2dd\uacfc \ub178\uc81c\ub97c \uac70\ud589\ud55c \ub4a4, \uacbd\ucc30\ubcd1\ub825\uc758 \ud638\uc1a1\ud558\uc5d0 \uc11c\uc6b8\ud2b9\ubcc4\uc2dc \ub3d9\uc791\uad6c \ub3d9\uc791\ub3d9 \uc0b044-7 \ub3d9\uc791\ub3d9 \ud604\ucda9\uc6d0 \uc7a5\uad70 \uc81c3 \ubb18\uc5ed\uc5d0 \ubb3b\ud614\ub2e4. \ud55c\ud3b8 \uadf8\uc758 \uad11\uba85\uc911\ud559\uad50 \ub3d9\ubb38\uc778 \ubb38\uc775\ud658\uc774 \uac19\uc740 \ub0a0 \uc138\uc0c1\uc744 \ub5a0\ub0ac\ub294\ub370, \ubb38\uc775\ud658\ubaa9\uc0ac\uc758 \uc601\uacb0\uc2dd\uc740 \ud55c\uc2e0\ub300\uc5d0\uc11c \uc2dc\uc791\ud574, \ub300\ud559\ub85c, \ub3d9\ub300\ubb38\uc5d0 \uc774\ub974\uae30\uae4c\uc9c0 \uc218\uc2ed\ub9cc\uba85\uc774 \ucc38\uc5ec\ud55c \ub178\uc81c\ub85c \uce58\ub7ec\uc9c4 \ubc18\uba74 \uc815\uc77c\uad8c\uc758 \uc7a5\ub840\uc2dd\uc740 \ud604\ucda9\uc6d0\uc5d0\uc11c \uc815\uce58\uc778\ub4e4\uc774 \ucc38\uac00\ud55c \uac00\uc6b4\ub370 \uc774\ub8e8\uc5b4\uc838 TV\ub274\uc2a4\uc5d0\uc11c \ub300\ube44\ub418\uc5c8\ub2e4. \uadf8\ub4e4\uc740 \ud0dc\uc5b4\ub09c \ud574\ub3c4 \uac19\uace0, \uac19\uc740 \uc911\ud559\uad50\ub97c \ub2e4\ub154\uc73c\ub098 \uc11c\ub85c \ub2e4\ub978 \uc778\uc0dd\uc744 \uc0b4\uc544\uac14\uc73c\uba70, \uac19\uc740 \ub0a0 \uc0ac\ub9dd\ud558\uba70 \ub9ce\uc740 \uc0ac\ub78c\ub4e4\uc5d0\uac8c \uae4a\uc740 \uc778\uc0c1\uc744 \ub0a8\uacbc\ub2e4.", "sentence_answer": "\uc720\ud574\ub294 \ube44\ud589\uae30\ud3b8\uc73c\ub85c \uadc0\uad6d, 1994\ub144 1\uc6d4 22\uc77c \uc624\uc804 \uad6d\ud68c\uc758\uc0ac\ub2f9\uc55e \uad11\uc7a5 \uc55e\uc5d0\uc11c \uad6d\ud68c\uc758\uc6d0 \uc774\ub9cc\uc12d\uc758 \uc8fc\ub3c4\ub85c \uc601\uacb0\uc2dd\uacfc \ub178\uc81c\ub97c \uac70\ud589\ud55c \ub4a4, \uacbd\ucc30\ubcd1\ub825\uc758 \ud638\uc1a1\ud558\uc5d0 \uc11c\uc6b8\ud2b9\ubcc4\uc2dc \ub3d9\uc791\uad6c \ub3d9\uc791\ub3d9 \uc0b044-7 \ub3d9\uc791\ub3d9 \ud604\ucda9\uc6d0 \uc7a5\uad70 \uc81c3 \ubb18\uc5ed\uc5d0 \ubb3b\ud614\ub2e4.", "paragraph_id": "5868390-8-1", "paragraph_question": "question: \uc774\ub9cc\uc12d\uc774 \uc8fc\ub3c4\ud55c \uc601\uacb0\uc2dd\uc740 \uc5b4\ub514\uc5d0\uc11c \uc9c4\ud589\ub418\uc5c8\ub294\uac00?, context: \uc720\ud574\ub294 \ube44\ud589\uae30\ud3b8\uc73c\ub85c \uadc0\uad6d, 1994\ub144 1\uc6d4 22\uc77c \uc624\uc804 \uad6d\ud68c\uc758\uc0ac\ub2f9\uc55e \uad11\uc7a5 \uc55e\uc5d0\uc11c \uad6d\ud68c\uc758\uc6d0 \uc774\ub9cc\uc12d\uc758 \uc8fc\ub3c4\ub85c \uc601\uacb0\uc2dd\uacfc \ub178\uc81c\ub97c \uac70\ud589\ud55c \ub4a4, \uacbd\ucc30\ubcd1\ub825\uc758 \ud638\uc1a1\ud558\uc5d0 \uc11c\uc6b8\ud2b9\ubcc4\uc2dc \ub3d9\uc791\uad6c \ub3d9\uc791\ub3d9 \uc0b044-7 \ub3d9\uc791\ub3d9 \ud604\ucda9\uc6d0 \uc7a5\uad70 \uc81c3 \ubb18\uc5ed\uc5d0 \ubb3b\ud614\ub2e4. \ud55c\ud3b8 \uadf8\uc758 \uad11\uba85\uc911\ud559\uad50 \ub3d9\ubb38\uc778 \ubb38\uc775\ud658\uc774 \uac19\uc740 \ub0a0 \uc138\uc0c1\uc744 \ub5a0\ub0ac\ub294\ub370, \ubb38\uc775\ud658\ubaa9\uc0ac\uc758 \uc601\uacb0\uc2dd\uc740 \ud55c\uc2e0\ub300\uc5d0\uc11c \uc2dc\uc791\ud574, \ub300\ud559\ub85c, \ub3d9\ub300\ubb38\uc5d0 \uc774\ub974\uae30\uae4c\uc9c0 \uc218\uc2ed\ub9cc\uba85\uc774 \ucc38\uc5ec\ud55c \ub178\uc81c\ub85c \uce58\ub7ec\uc9c4 \ubc18\uba74 \uc815\uc77c\uad8c\uc758 \uc7a5\ub840\uc2dd\uc740 \ud604\ucda9\uc6d0\uc5d0\uc11c \uc815\uce58\uc778\ub4e4\uc774 \ucc38\uac00\ud55c \uac00\uc6b4\ub370 \uc774\ub8e8\uc5b4\uc838 TV\ub274\uc2a4\uc5d0\uc11c \ub300\ube44\ub418\uc5c8\ub2e4. \uadf8\ub4e4\uc740 \ud0dc\uc5b4\ub09c \ud574\ub3c4 \uac19\uace0, \uac19\uc740 \uc911\ud559\uad50\ub97c \ub2e4\ub154\uc73c\ub098 \uc11c\ub85c \ub2e4\ub978 \uc778\uc0dd\uc744 \uc0b4\uc544\uac14\uc73c\uba70, \uac19\uc740 \ub0a0 \uc0ac\ub9dd\ud558\uba70 \ub9ce\uc740 \uc0ac\ub78c\ub4e4\uc5d0\uac8c \uae4a\uc740 \uc778\uc0c1\uc744 \ub0a8\uacbc\ub2e4."} {"question": "\uc774\uc885\uc6b1\uc774 \ub9ac\uadf8 \ub0b4 \ucd5c\uc815\uc0c1\uae09\uc778 \ub2a5\ub825\uc740?", "paragraph": "\uc804\ud615\uc801\uc778 \ud14c\uc774\ube14 \uc138\ud130 \uc720\ud615\uc758 \uc120\uc218\ub85c\uc11c \ub9ac\uadf8\ub97c \ub300\ud45c\ud558\ub294 1\ubc88 \ud0c0\uc790\ub85c \uc77c\uceec\uc5b4\uc9c8 \uc815\ub3c4\ub85c \ube60\ub978 \ubc1c\uacfc \ud0c1\uc6d4\ud55c \uc8fc\ub8e8 \uc13c\uc2a4\ub97c \ubcf4\uc720\ud558\uace0 \uc788\uc5b4, \ubcd1\uc0b4\ud0c0\uac00 \uc801\uc740 \ud3b8\uc774\ub2e4. \uacf5\uaca9\uc801\uc73c\ub85c \ud0c0\uaca9\ud558\uba70 \uc120\uad6c\uc548\uc774 \uc88b\uace0 \ucee8\ud0dd \ub2a5\ub825\uc774 \ub6f0\uc5b4\ub098\uba70, \ubc00\uc5b4\uce58\ub294 \ub2a5\ub825\uc774 \ub6f0\uc5b4\ub098\ub2e4. \ubcc0\ud654\uad6c\uc5d0\ub294 \uc57d\uc810\uc774 \uc788\uc9c0\ub9cc \ucee4\ud2b8\ub97c \uc798\ud574 \ub048\uc9c8\uae34 \uc2b9\ubd80\ub97c \ud55c\ub2e4. \ub3c4\ub8e8 \ub2a5\ub825\uc740 \ub9ac\uadf8 \ub0b4 \ucd5c\uc815\uc0c1\uae09\uc774\ub2e4. 2006\ub144\ubd80\ud130 2011\ub144\uae4c\uc9c0 6\uc2dc\uc98c \ub3d9\uc548\uc758 \ub3c4\ub8e8 \uac2f\uc218\uac00 \uc774\ub300\ud615\uc758 \ub4a4\ub97c \uc774\uc740 2\uc704(212\uac1c)\ub97c \uae30\ub85d\ud588\ub2e4. \uadf8\ub7ec\ub098 \ucd5c\uadfc\uc5d0 \ub4e4\uc5b4\uc11c \uc810\uc810 \ub3c4\ub8e8 \uc2dc\ub3c4\uac00 \uc904\uc5b4\ub4e4\uace0 \uc788\ub294\ub370, \uadf8\ub294 \uc774\uc5d0 \ub300\ud574 \"\ub3c4\ub8e8\ub85c \uc88b\uc9c0\ub9cc 1\ub8e8\uc5d0\uc11c \uc0c1\ub300 \ubc30\ud130\ub9ac\ub97c \ud754\ub4e4\uace0 \ud6c4\uc18d\ud0c0 \ub54c 3\ub8e8\ub85c \uc9c4\ub8e8\ud558\ub294 \uac8c \ud0c0\uc790\uc5d0\uac8c\ub3c4 \uc720\ub9ac\ud558\ub2e4.\"\ub77c\uace0 \ub9d0\ud558\uae30\ub3c4 \ud588\uace0, \ub610\ud55c 2011\ub144 \uc2dc\uc98c \ucd08\uc5d0\ub294 \"\uccb4\ub825\uc801\uc778 \ubd80\ub2f4\uc774 \uc788\uc5b4\uc11c \uc548 \ub6f0\ub294 \uac83\uc740 \uc544\ub2c8\ub2e4. \uc7ac\uc791\ub144\uc5d0\ub3c4 \ud06c\uac8c \ub2e4\uce58\uace0 \uc62c \uc2dc\uc98c\ub3c4 \ubd80\uc0c1\uc73c\ub85c \uc880 \ub6f0\uae30 \uc5b4\ub835\ub2e4. \uc870\uae08 \uc870\uc2ec\ud558\ub824\uace0 \ud588\ub2e4.\"\ub77c\uace0 \ub9d0\ud588\ub2e4. \ub610\ud55c \uadfc\uc131\uc774 \uac15\ud55c \uc120\uc218\ub85c \uc798 \uc54c\ub824\uc838 \uc788\ub294\ub370, \uadf8 \uc608\ub85c 2009\ub144 6\uc6d4\ucd08\uc5d0\ub294 \ud131\uad00\uc808 \ubd80\uc0c1\uc73c\ub85c 2~3\uac1c\uc6d4\uc758 \uce58\ub8cc\uc640 \uc7ac\ud65c\uc774 \uc608\uc0c1\ub418\uc5c8\uc73c\ub098 \ubd88\uacfc \ud55c \ub2ec \ubc18 \ub9cc\uc5d0 \ubcf5\uadc0\ud55c \uc801\uc774 \uc788\ub2e4. \ub610\ud55c \uac19\uc740 \ud300 \ub3d9\ub8cc\uc778 \uc774\ud61c\ucc9c\uc740 2011\ub144 \uc2dc\uc98c \uc804\uc5d0 \uadf8\ub97c \ubcf4\uace0 \"\ub9c8\uce58 \ub3c5\uc57d \uba39\uace0 \ub6f0\ub294 \uac83 \uac19\ub2e4. \uc77c\ubcf8 \uc120\uc218\ub4e4\ub3c4 \ub3d9\uc601\uc0c1\uc744 \ubcf4\uc5ec\uc8fc\ub2c8 \uce6d\ucc2c\uc744 \uc544\ub07c\uc9c0 \uc54a\uc558\ub2e4. \ub2e4\uce60 \uac83 \uac19\uc544\uc11c \uacbd\uae30 \ub54c\ub9c8\ub2e4 \uad6c\uae09\ucc28\ub97c \ubd88\ub7ec\ub193\uace0 \ud574\uc57c\ud560 \uac83 \uac19\ub2e4.\"\ub77c\uace0 \ub9d0\ud558\uae30\ub3c4 \ud588\ub2e4. \uadf8\ub7ec\ub098 \ud3c9\uade0 \ud0c0\uc728\uc5d0 \ube44\ud574 \uc88c\uc644\ud22c\uc218 \uc0c1\ub300 \ud0c0\uc728\uc774 \uc57d\uac04 \ub5a8\uc5b4\uc9c0\ub294 \ud3b8\uc774\ub2e4. 2007\ub144 \uc2dc\uc98c\uc5d0 \uc88c\uc644 \uc0c1\ub300\ub85c 0.324\ub97c \uae30\ub85d\ud55c \uac83\uc744 \uc81c\uc678\ud558\uace0 2008\ub144 \uc2dc\uc98c 0.247, 2009\ub144 \uc2dc\uc98c 0.248, 2010\ub144 \uc2dc\uc98c 0.291\uc758 \ud0c0\uc728\uc744 \uae30\ub85d\ud574 \ubcf8\uc778\uc758 \uc2dc\uc98c \ud0c0\uc728\uc5d0 \ube44\ud574 \uc800\uc870\ud55c \uc131\uc801\uc744 \uae30\ub85d\ud588\ub2e4. \uadf8\ub9ac\uace0 \uc678\uc57c\uc218\ube44\uc5d0 \uc788\uc5b4\uc11c \ube60\ub978 \ubc1c\uc744 \uc0b4\ub9b0 \uc218\ube44 \ubc94\uc704\ub294 \ub113\uc73c\ub098 \uc5b4\uae68\ub294 \ud3c9\ubc94\ud558\ub2e4\uace0 \ud3c9\uac00\ubc1b\ub294\ub2e4. \ud604\ub300 \uc720\ub2c8\ucf58\uc2a4\uc640 \uc120\uc218\ub2e8 \uc2b9\uacc4\ub97c \ud55c \uc6b0\ub9ac, \ub125\uc13c \ud788\uc5b4\ub85c\uc988\ub97c \uc0c1\ub300\ub85c \uc9d5\ud06c\uc2a4\uac00 \uc788\uc5c8\ub2e4. 2007\ub144 \uc2dc\uc98c \ud604\ub300\ub97c \uc0c1\ub300\ub85c 0.246\uc758 \ud0c0\uc728\uc744 \uae30\ub85d\ud588\uace0, \uc774\ub4ec\ud574 \uc6b0\ub9ac \ud788\uc5b4\ub85c\uc988\ub97c \uc0c1\ub300\ub85c 0.189, 2009\ub144 \uc2dc\uc98c\uc5d0\ub294 0.286\ub97c \uae30\ub85d\ud588\ub2e4. \uadf8\ub7ec\ub098 2010\ub144 \uc2dc\uc98c\uc5d0\ub294 \ub125\uc13c\uc744 \uc0c1\ub300\ub85c \ub9f9\ud0c0\ub97c \ud718\ub458\ub800\uace0, \uc0c1\ub300\ud300 \uc911 \uac00\uc7a5 \ub192\uc740 0.511\uc758 \ud0c0\uc728\uc744 \uae30\ub85d\ud588\ub2e4. \ud14c\uc774\ube14 \uc138\ud130\uc774\uae34 \ud558\uc9c0\ub9cc \uc774\ub840\uc801\uc73c\ub85c \ub4dd\uc810\uad8c\uc5d0\uc11c\uc758 \ud0c0\uc728\uc774 \ub192\uc544 2007\ub144 \uc2dc\uc98c\uc5d0 0.385(\ub9ac\uadf8 2\uc704), 2010\ub144 \uc2dc\uc98c\uc5d0 0.359(\ub9ac\uadf8 8\uc704)\ub97c \uae30\ub85d\ud558\uc600\ub2e4. \ub610\ud55c 1\ubc88 \ud0c0\uc790\ub2f5\uac8c \ud648\ub7f0 \uc218\ub294 \uc801\uc740 \ud3b8\uc774\ub098, \uc758\uc678\ub85c \uad6d\uac00\ub300\ud45c \uacbd\uae30\ub098 \ud3ec\uc2a4\ud2b8 \uc2dc\uc98c\ub4f1 \uc911\uc694\ud55c \uacbd\uae30\uc5d0\uc11c \ud648\ub7f0\uc744 \uce58\ub294 \uacbd\uc6b0\uac00 \uc788\ub2e4. 2007\ub144 \ud50c\ub808\uc774\uc624\ud504 2\ucc28\uc804\uc5d0\uc11c\uc758 \uc194\ub85c \ud648\ub7f0, 2008\ub144 \ubca0\uc774\uc9d5 \uc62c\ub9bc\ud53d \uc57c\uad6c \uc608\uc120 \ub300\ub9cc\uc804\uc5d0\uc11c\uc758 3\uc810 \ud648\ub7f0, 2010\ub144 \uc900\ud50c\ub808\uc774\uc624\ud504 3\ucc28\uc804\uc5d0\uc11c\uc758 \ud648\ub7f0\uc774 \uadf8 \uc608\uc774\ub2e4. 2010\ub144 \uc2dc\uc98c\uc5d0 5\uac1c\uc758 \ud648\ub7f0\uc744 \uae30\ub85d\ud558\uc5ec \uc774\uc804 \uc2dc\uc98c\uae4c\uc9c0\uc758 \ud648\ub7f0 \ucd1d \ud569\ubcf4\ub2e4 \ub354 \ub9ce\uc740 \ud648\ub7f0 \uac1c\uc218\ub97c \uae30\ub85d\ud558\uc600\ub2e4. \ud0c0\uc11d\uc5d0 \ub4f1\uc7a5\ud560 \uc2dc\uc758 \ub178\ub798\ub294 \ud50c\ub85c \ub77c\uc774\ub354\uc758 'In The Ayer'\ub77c\ub294 \uace1\uc774\ub2e4. \uc751\uc6d0\uac00\ub294 \ud2b8\ub79c\uc2a4\ud53d\uc158\uc758 '\ub77c\ub514\uc624'\ub97c \uac1c\uc0ac\ud55c \uac83\uc73c\ub85c, '\ub0a0\ub824\ub77c \uc774\uc885\uc6b1 \uc774\uc885\uc6b1 \uc774\uc885\uc6b1 \ub0a0\ub824\ubc84\ub824 \ub450\uc0b0\uc758 \uc774\uc885\uc6b1\uc774\ub2e4'\ub97c \ubc18\ubcf5\ud55c\ub2e4. \ub610\ud55c \uadf8\ub294 \uc601\ud654 \uc639\ubc15\uc758 \uc8fc\uc778\uacf5 \ubc30\uc5ed\uc744 \ub9e1\uc740 \ud1a0\ub2c8 \uc790\uc640 \ub2ee\uc558\ub2e4\uace0 \ud558\uc5ec \ud32c\ub4e4 \uc0ac\uc774\uc5d0\uc11c \u2018\uc639\ubc15\u2019 \ud639\uc740 \u2018\uc885\ubc15(\uc885\uc6b1+\uc639\ubc15)\u2019\uc774\ub77c\ub294 \uc560\uce6d\uc73c\ub85c \ubd88\ub9ac\uace0 \uc788\ub2e4.", "answer": "\ub3c4\ub8e8 \ub2a5\ub825", "sentence": "\ub3c4\ub8e8 \ub2a5\ub825 \uc740 \ub9ac\uadf8 \ub0b4 \ucd5c\uc815\uc0c1\uae09\uc774\ub2e4.", "paragraph_sentence": "\uc804\ud615\uc801\uc778 \ud14c\uc774\ube14 \uc138\ud130 \uc720\ud615\uc758 \uc120\uc218\ub85c\uc11c \ub9ac\uadf8\ub97c \ub300\ud45c\ud558\ub294 1\ubc88 \ud0c0\uc790\ub85c \uc77c\uceec\uc5b4\uc9c8 \uc815\ub3c4\ub85c \ube60\ub978 \ubc1c\uacfc \ud0c1\uc6d4\ud55c \uc8fc\ub8e8 \uc13c\uc2a4\ub97c \ubcf4\uc720\ud558\uace0 \uc788\uc5b4, \ubcd1\uc0b4\ud0c0\uac00 \uc801\uc740 \ud3b8\uc774\ub2e4. \uacf5\uaca9\uc801\uc73c\ub85c \ud0c0\uaca9\ud558\uba70 \uc120\uad6c\uc548\uc774 \uc88b\uace0 \ucee8\ud0dd \ub2a5\ub825\uc774 \ub6f0\uc5b4\ub098\uba70, \ubc00\uc5b4\uce58\ub294 \ub2a5\ub825\uc774 \ub6f0\uc5b4\ub098\ub2e4. \ubcc0\ud654\uad6c\uc5d0\ub294 \uc57d\uc810\uc774 \uc788\uc9c0\ub9cc \ucee4\ud2b8\ub97c \uc798\ud574 \ub048\uc9c8\uae34 \uc2b9\ubd80\ub97c \ud55c\ub2e4. \ub3c4\ub8e8 \ub2a5\ub825 \uc740 \ub9ac\uadf8 \ub0b4 \ucd5c\uc815\uc0c1\uae09\uc774\ub2e4. 2006\ub144\ubd80\ud130 2011\ub144\uae4c\uc9c0 6\uc2dc\uc98c \ub3d9\uc548\uc758 \ub3c4\ub8e8 \uac2f\uc218\uac00 \uc774\ub300\ud615\uc758 \ub4a4\ub97c \uc774\uc740 2\uc704(212\uac1c)\ub97c \uae30\ub85d\ud588\ub2e4. \uadf8\ub7ec\ub098 \ucd5c\uadfc\uc5d0 \ub4e4\uc5b4\uc11c \uc810\uc810 \ub3c4\ub8e8 \uc2dc\ub3c4\uac00 \uc904\uc5b4\ub4e4\uace0 \uc788\ub294\ub370, \uadf8\ub294 \uc774\uc5d0 \ub300\ud574 \"\ub3c4\ub8e8\ub85c \uc88b\uc9c0\ub9cc 1\ub8e8\uc5d0\uc11c \uc0c1\ub300 \ubc30\ud130\ub9ac\ub97c \ud754\ub4e4\uace0 \ud6c4\uc18d\ud0c0 \ub54c 3\ub8e8\ub85c \uc9c4\ub8e8\ud558\ub294 \uac8c \ud0c0\uc790\uc5d0\uac8c\ub3c4 \uc720\ub9ac\ud558\ub2e4. \"\ub77c\uace0 \ub9d0\ud558\uae30\ub3c4 \ud588\uace0, \ub610\ud55c 2011\ub144 \uc2dc\uc98c \ucd08\uc5d0\ub294 \"\uccb4\ub825\uc801\uc778 \ubd80\ub2f4\uc774 \uc788\uc5b4\uc11c \uc548 \ub6f0\ub294 \uac83\uc740 \uc544\ub2c8\ub2e4. \uc7ac\uc791\ub144\uc5d0\ub3c4 \ud06c\uac8c \ub2e4\uce58\uace0 \uc62c \uc2dc\uc98c\ub3c4 \ubd80\uc0c1\uc73c\ub85c \uc880 \ub6f0\uae30 \uc5b4\ub835\ub2e4. \uc870\uae08 \uc870\uc2ec\ud558\ub824\uace0 \ud588\ub2e4. \"\ub77c\uace0 \ub9d0\ud588\ub2e4. \ub610\ud55c \uadfc\uc131\uc774 \uac15\ud55c \uc120\uc218\ub85c \uc798 \uc54c\ub824\uc838 \uc788\ub294\ub370, \uadf8 \uc608\ub85c 2009\ub144 6\uc6d4\ucd08\uc5d0\ub294 \ud131\uad00\uc808 \ubd80\uc0c1\uc73c\ub85c 2~3\uac1c\uc6d4\uc758 \uce58\ub8cc\uc640 \uc7ac\ud65c\uc774 \uc608\uc0c1\ub418\uc5c8\uc73c\ub098 \ubd88\uacfc \ud55c \ub2ec \ubc18 \ub9cc\uc5d0 \ubcf5\uadc0\ud55c \uc801\uc774 \uc788\ub2e4. \ub610\ud55c \uac19\uc740 \ud300 \ub3d9\ub8cc\uc778 \uc774\ud61c\ucc9c\uc740 2011\ub144 \uc2dc\uc98c \uc804\uc5d0 \uadf8\ub97c \ubcf4\uace0 \"\ub9c8\uce58 \ub3c5\uc57d \uba39\uace0 \ub6f0\ub294 \uac83 \uac19\ub2e4. \uc77c\ubcf8 \uc120\uc218\ub4e4\ub3c4 \ub3d9\uc601\uc0c1\uc744 \ubcf4\uc5ec\uc8fc\ub2c8 \uce6d\ucc2c\uc744 \uc544\ub07c\uc9c0 \uc54a\uc558\ub2e4. \ub2e4\uce60 \uac83 \uac19\uc544\uc11c \uacbd\uae30 \ub54c\ub9c8\ub2e4 \uad6c\uae09\ucc28\ub97c \ubd88\ub7ec\ub193\uace0 \ud574\uc57c\ud560 \uac83 \uac19\ub2e4. \"\ub77c\uace0 \ub9d0\ud558\uae30\ub3c4 \ud588\ub2e4. \uadf8\ub7ec\ub098 \ud3c9\uade0 \ud0c0\uc728\uc5d0 \ube44\ud574 \uc88c\uc644\ud22c\uc218 \uc0c1\ub300 \ud0c0\uc728\uc774 \uc57d\uac04 \ub5a8\uc5b4\uc9c0\ub294 \ud3b8\uc774\ub2e4. 2007\ub144 \uc2dc\uc98c\uc5d0 \uc88c\uc644 \uc0c1\ub300\ub85c 0.324\ub97c \uae30\ub85d\ud55c \uac83\uc744 \uc81c\uc678\ud558\uace0 2008\ub144 \uc2dc\uc98c 0.247, 2009\ub144 \uc2dc\uc98c 0.248, 2010\ub144 \uc2dc\uc98c 0.291\uc758 \ud0c0\uc728\uc744 \uae30\ub85d\ud574 \ubcf8\uc778\uc758 \uc2dc\uc98c \ud0c0\uc728\uc5d0 \ube44\ud574 \uc800\uc870\ud55c \uc131\uc801\uc744 \uae30\ub85d\ud588\ub2e4. \uadf8\ub9ac\uace0 \uc678\uc57c\uc218\ube44\uc5d0 \uc788\uc5b4\uc11c \ube60\ub978 \ubc1c\uc744 \uc0b4\ub9b0 \uc218\ube44 \ubc94\uc704\ub294 \ub113\uc73c\ub098 \uc5b4\uae68\ub294 \ud3c9\ubc94\ud558\ub2e4\uace0 \ud3c9\uac00\ubc1b\ub294\ub2e4. \ud604\ub300 \uc720\ub2c8\ucf58\uc2a4\uc640 \uc120\uc218\ub2e8 \uc2b9\uacc4\ub97c \ud55c \uc6b0\ub9ac, \ub125\uc13c \ud788\uc5b4\ub85c\uc988\ub97c \uc0c1\ub300\ub85c \uc9d5\ud06c\uc2a4\uac00 \uc788\uc5c8\ub2e4. 2007\ub144 \uc2dc\uc98c \ud604\ub300\ub97c \uc0c1\ub300\ub85c 0.246\uc758 \ud0c0\uc728\uc744 \uae30\ub85d\ud588\uace0, \uc774\ub4ec\ud574 \uc6b0\ub9ac \ud788\uc5b4\ub85c\uc988\ub97c \uc0c1\ub300\ub85c 0.189, 2009\ub144 \uc2dc\uc98c\uc5d0\ub294 0.286\ub97c \uae30\ub85d\ud588\ub2e4. \uadf8\ub7ec\ub098 2010\ub144 \uc2dc\uc98c\uc5d0\ub294 \ub125\uc13c\uc744 \uc0c1\ub300\ub85c \ub9f9\ud0c0\ub97c \ud718\ub458\ub800\uace0, \uc0c1\ub300\ud300 \uc911 \uac00\uc7a5 \ub192\uc740 0.511\uc758 \ud0c0\uc728\uc744 \uae30\ub85d\ud588\ub2e4. \ud14c\uc774\ube14 \uc138\ud130\uc774\uae34 \ud558\uc9c0\ub9cc \uc774\ub840\uc801\uc73c\ub85c \ub4dd\uc810\uad8c\uc5d0\uc11c\uc758 \ud0c0\uc728\uc774 \ub192\uc544 2007\ub144 \uc2dc\uc98c\uc5d0 0.385(\ub9ac\uadf8 2\uc704), 2010\ub144 \uc2dc\uc98c\uc5d0 0.359(\ub9ac\uadf8 8\uc704)\ub97c \uae30\ub85d\ud558\uc600\ub2e4. \ub610\ud55c 1\ubc88 \ud0c0\uc790\ub2f5\uac8c \ud648\ub7f0 \uc218\ub294 \uc801\uc740 \ud3b8\uc774\ub098, \uc758\uc678\ub85c \uad6d\uac00\ub300\ud45c \uacbd\uae30\ub098 \ud3ec\uc2a4\ud2b8 \uc2dc\uc98c\ub4f1 \uc911\uc694\ud55c \uacbd\uae30\uc5d0\uc11c \ud648\ub7f0\uc744 \uce58\ub294 \uacbd\uc6b0\uac00 \uc788\ub2e4. 2007\ub144 \ud50c\ub808\uc774\uc624\ud504 2\ucc28\uc804\uc5d0\uc11c\uc758 \uc194\ub85c \ud648\ub7f0, 2008\ub144 \ubca0\uc774\uc9d5 \uc62c\ub9bc\ud53d \uc57c\uad6c \uc608\uc120 \ub300\ub9cc\uc804\uc5d0\uc11c\uc758 3\uc810 \ud648\ub7f0, 2010\ub144 \uc900\ud50c\ub808\uc774\uc624\ud504 3\ucc28\uc804\uc5d0\uc11c\uc758 \ud648\ub7f0\uc774 \uadf8 \uc608\uc774\ub2e4. 2010\ub144 \uc2dc\uc98c\uc5d0 5\uac1c\uc758 \ud648\ub7f0\uc744 \uae30\ub85d\ud558\uc5ec \uc774\uc804 \uc2dc\uc98c\uae4c\uc9c0\uc758 \ud648\ub7f0 \ucd1d \ud569\ubcf4\ub2e4 \ub354 \ub9ce\uc740 \ud648\ub7f0 \uac1c\uc218\ub97c \uae30\ub85d\ud558\uc600\ub2e4. \ud0c0\uc11d\uc5d0 \ub4f1\uc7a5\ud560 \uc2dc\uc758 \ub178\ub798\ub294 \ud50c\ub85c \ub77c\uc774\ub354\uc758 'In The Ayer'\ub77c\ub294 \uace1\uc774\ub2e4. \uc751\uc6d0\uac00\ub294 \ud2b8\ub79c\uc2a4\ud53d\uc158\uc758 '\ub77c\ub514\uc624'\ub97c \uac1c\uc0ac\ud55c \uac83\uc73c\ub85c, '\ub0a0\ub824\ub77c \uc774\uc885\uc6b1 \uc774\uc885\uc6b1 \uc774\uc885\uc6b1 \ub0a0\ub824\ubc84\ub824 \ub450\uc0b0\uc758 \uc774\uc885\uc6b1\uc774\ub2e4'\ub97c \ubc18\ubcf5\ud55c\ub2e4. \ub610\ud55c \uadf8\ub294 \uc601\ud654 \uc639\ubc15\uc758 \uc8fc\uc778\uacf5 \ubc30\uc5ed\uc744 \ub9e1\uc740 \ud1a0\ub2c8 \uc790\uc640 \ub2ee\uc558\ub2e4\uace0 \ud558\uc5ec \ud32c\ub4e4 \uc0ac\uc774\uc5d0\uc11c \u2018\uc639\ubc15\u2019 \ud639\uc740 \u2018\uc885\ubc15(\uc885\uc6b1+\uc639\ubc15)\u2019\uc774\ub77c\ub294 \uc560\uce6d\uc73c\ub85c \ubd88\ub9ac\uace0 \uc788\ub2e4.", "paragraph_answer": "\uc804\ud615\uc801\uc778 \ud14c\uc774\ube14 \uc138\ud130 \uc720\ud615\uc758 \uc120\uc218\ub85c\uc11c \ub9ac\uadf8\ub97c \ub300\ud45c\ud558\ub294 1\ubc88 \ud0c0\uc790\ub85c \uc77c\uceec\uc5b4\uc9c8 \uc815\ub3c4\ub85c \ube60\ub978 \ubc1c\uacfc \ud0c1\uc6d4\ud55c \uc8fc\ub8e8 \uc13c\uc2a4\ub97c \ubcf4\uc720\ud558\uace0 \uc788\uc5b4, \ubcd1\uc0b4\ud0c0\uac00 \uc801\uc740 \ud3b8\uc774\ub2e4. \uacf5\uaca9\uc801\uc73c\ub85c \ud0c0\uaca9\ud558\uba70 \uc120\uad6c\uc548\uc774 \uc88b\uace0 \ucee8\ud0dd \ub2a5\ub825\uc774 \ub6f0\uc5b4\ub098\uba70, \ubc00\uc5b4\uce58\ub294 \ub2a5\ub825\uc774 \ub6f0\uc5b4\ub098\ub2e4. \ubcc0\ud654\uad6c\uc5d0\ub294 \uc57d\uc810\uc774 \uc788\uc9c0\ub9cc \ucee4\ud2b8\ub97c \uc798\ud574 \ub048\uc9c8\uae34 \uc2b9\ubd80\ub97c \ud55c\ub2e4. \ub3c4\ub8e8 \ub2a5\ub825 \uc740 \ub9ac\uadf8 \ub0b4 \ucd5c\uc815\uc0c1\uae09\uc774\ub2e4. 2006\ub144\ubd80\ud130 2011\ub144\uae4c\uc9c0 6\uc2dc\uc98c \ub3d9\uc548\uc758 \ub3c4\ub8e8 \uac2f\uc218\uac00 \uc774\ub300\ud615\uc758 \ub4a4\ub97c \uc774\uc740 2\uc704(212\uac1c)\ub97c \uae30\ub85d\ud588\ub2e4. \uadf8\ub7ec\ub098 \ucd5c\uadfc\uc5d0 \ub4e4\uc5b4\uc11c \uc810\uc810 \ub3c4\ub8e8 \uc2dc\ub3c4\uac00 \uc904\uc5b4\ub4e4\uace0 \uc788\ub294\ub370, \uadf8\ub294 \uc774\uc5d0 \ub300\ud574 \"\ub3c4\ub8e8\ub85c \uc88b\uc9c0\ub9cc 1\ub8e8\uc5d0\uc11c \uc0c1\ub300 \ubc30\ud130\ub9ac\ub97c \ud754\ub4e4\uace0 \ud6c4\uc18d\ud0c0 \ub54c 3\ub8e8\ub85c \uc9c4\ub8e8\ud558\ub294 \uac8c \ud0c0\uc790\uc5d0\uac8c\ub3c4 \uc720\ub9ac\ud558\ub2e4.\"\ub77c\uace0 \ub9d0\ud558\uae30\ub3c4 \ud588\uace0, \ub610\ud55c 2011\ub144 \uc2dc\uc98c \ucd08\uc5d0\ub294 \"\uccb4\ub825\uc801\uc778 \ubd80\ub2f4\uc774 \uc788\uc5b4\uc11c \uc548 \ub6f0\ub294 \uac83\uc740 \uc544\ub2c8\ub2e4. \uc7ac\uc791\ub144\uc5d0\ub3c4 \ud06c\uac8c \ub2e4\uce58\uace0 \uc62c \uc2dc\uc98c\ub3c4 \ubd80\uc0c1\uc73c\ub85c \uc880 \ub6f0\uae30 \uc5b4\ub835\ub2e4. \uc870\uae08 \uc870\uc2ec\ud558\ub824\uace0 \ud588\ub2e4.\"\ub77c\uace0 \ub9d0\ud588\ub2e4. \ub610\ud55c \uadfc\uc131\uc774 \uac15\ud55c \uc120\uc218\ub85c \uc798 \uc54c\ub824\uc838 \uc788\ub294\ub370, \uadf8 \uc608\ub85c 2009\ub144 6\uc6d4\ucd08\uc5d0\ub294 \ud131\uad00\uc808 \ubd80\uc0c1\uc73c\ub85c 2~3\uac1c\uc6d4\uc758 \uce58\ub8cc\uc640 \uc7ac\ud65c\uc774 \uc608\uc0c1\ub418\uc5c8\uc73c\ub098 \ubd88\uacfc \ud55c \ub2ec \ubc18 \ub9cc\uc5d0 \ubcf5\uadc0\ud55c \uc801\uc774 \uc788\ub2e4. \ub610\ud55c \uac19\uc740 \ud300 \ub3d9\ub8cc\uc778 \uc774\ud61c\ucc9c\uc740 2011\ub144 \uc2dc\uc98c \uc804\uc5d0 \uadf8\ub97c \ubcf4\uace0 \"\ub9c8\uce58 \ub3c5\uc57d \uba39\uace0 \ub6f0\ub294 \uac83 \uac19\ub2e4. \uc77c\ubcf8 \uc120\uc218\ub4e4\ub3c4 \ub3d9\uc601\uc0c1\uc744 \ubcf4\uc5ec\uc8fc\ub2c8 \uce6d\ucc2c\uc744 \uc544\ub07c\uc9c0 \uc54a\uc558\ub2e4. \ub2e4\uce60 \uac83 \uac19\uc544\uc11c \uacbd\uae30 \ub54c\ub9c8\ub2e4 \uad6c\uae09\ucc28\ub97c \ubd88\ub7ec\ub193\uace0 \ud574\uc57c\ud560 \uac83 \uac19\ub2e4.\"\ub77c\uace0 \ub9d0\ud558\uae30\ub3c4 \ud588\ub2e4. \uadf8\ub7ec\ub098 \ud3c9\uade0 \ud0c0\uc728\uc5d0 \ube44\ud574 \uc88c\uc644\ud22c\uc218 \uc0c1\ub300 \ud0c0\uc728\uc774 \uc57d\uac04 \ub5a8\uc5b4\uc9c0\ub294 \ud3b8\uc774\ub2e4. 2007\ub144 \uc2dc\uc98c\uc5d0 \uc88c\uc644 \uc0c1\ub300\ub85c 0.324\ub97c \uae30\ub85d\ud55c \uac83\uc744 \uc81c\uc678\ud558\uace0 2008\ub144 \uc2dc\uc98c 0.247, 2009\ub144 \uc2dc\uc98c 0.248, 2010\ub144 \uc2dc\uc98c 0.291\uc758 \ud0c0\uc728\uc744 \uae30\ub85d\ud574 \ubcf8\uc778\uc758 \uc2dc\uc98c \ud0c0\uc728\uc5d0 \ube44\ud574 \uc800\uc870\ud55c \uc131\uc801\uc744 \uae30\ub85d\ud588\ub2e4. \uadf8\ub9ac\uace0 \uc678\uc57c\uc218\ube44\uc5d0 \uc788\uc5b4\uc11c \ube60\ub978 \ubc1c\uc744 \uc0b4\ub9b0 \uc218\ube44 \ubc94\uc704\ub294 \ub113\uc73c\ub098 \uc5b4\uae68\ub294 \ud3c9\ubc94\ud558\ub2e4\uace0 \ud3c9\uac00\ubc1b\ub294\ub2e4. \ud604\ub300 \uc720\ub2c8\ucf58\uc2a4\uc640 \uc120\uc218\ub2e8 \uc2b9\uacc4\ub97c \ud55c \uc6b0\ub9ac, \ub125\uc13c \ud788\uc5b4\ub85c\uc988\ub97c \uc0c1\ub300\ub85c \uc9d5\ud06c\uc2a4\uac00 \uc788\uc5c8\ub2e4. 2007\ub144 \uc2dc\uc98c \ud604\ub300\ub97c \uc0c1\ub300\ub85c 0.246\uc758 \ud0c0\uc728\uc744 \uae30\ub85d\ud588\uace0, \uc774\ub4ec\ud574 \uc6b0\ub9ac \ud788\uc5b4\ub85c\uc988\ub97c \uc0c1\ub300\ub85c 0.189, 2009\ub144 \uc2dc\uc98c\uc5d0\ub294 0.286\ub97c \uae30\ub85d\ud588\ub2e4. \uadf8\ub7ec\ub098 2010\ub144 \uc2dc\uc98c\uc5d0\ub294 \ub125\uc13c\uc744 \uc0c1\ub300\ub85c \ub9f9\ud0c0\ub97c \ud718\ub458\ub800\uace0, \uc0c1\ub300\ud300 \uc911 \uac00\uc7a5 \ub192\uc740 0.511\uc758 \ud0c0\uc728\uc744 \uae30\ub85d\ud588\ub2e4. \ud14c\uc774\ube14 \uc138\ud130\uc774\uae34 \ud558\uc9c0\ub9cc \uc774\ub840\uc801\uc73c\ub85c \ub4dd\uc810\uad8c\uc5d0\uc11c\uc758 \ud0c0\uc728\uc774 \ub192\uc544 2007\ub144 \uc2dc\uc98c\uc5d0 0.385(\ub9ac\uadf8 2\uc704), 2010\ub144 \uc2dc\uc98c\uc5d0 0.359(\ub9ac\uadf8 8\uc704)\ub97c \uae30\ub85d\ud558\uc600\ub2e4. \ub610\ud55c 1\ubc88 \ud0c0\uc790\ub2f5\uac8c \ud648\ub7f0 \uc218\ub294 \uc801\uc740 \ud3b8\uc774\ub098, \uc758\uc678\ub85c \uad6d\uac00\ub300\ud45c \uacbd\uae30\ub098 \ud3ec\uc2a4\ud2b8 \uc2dc\uc98c\ub4f1 \uc911\uc694\ud55c \uacbd\uae30\uc5d0\uc11c \ud648\ub7f0\uc744 \uce58\ub294 \uacbd\uc6b0\uac00 \uc788\ub2e4. 2007\ub144 \ud50c\ub808\uc774\uc624\ud504 2\ucc28\uc804\uc5d0\uc11c\uc758 \uc194\ub85c \ud648\ub7f0, 2008\ub144 \ubca0\uc774\uc9d5 \uc62c\ub9bc\ud53d \uc57c\uad6c \uc608\uc120 \ub300\ub9cc\uc804\uc5d0\uc11c\uc758 3\uc810 \ud648\ub7f0, 2010\ub144 \uc900\ud50c\ub808\uc774\uc624\ud504 3\ucc28\uc804\uc5d0\uc11c\uc758 \ud648\ub7f0\uc774 \uadf8 \uc608\uc774\ub2e4. 2010\ub144 \uc2dc\uc98c\uc5d0 5\uac1c\uc758 \ud648\ub7f0\uc744 \uae30\ub85d\ud558\uc5ec \uc774\uc804 \uc2dc\uc98c\uae4c\uc9c0\uc758 \ud648\ub7f0 \ucd1d \ud569\ubcf4\ub2e4 \ub354 \ub9ce\uc740 \ud648\ub7f0 \uac1c\uc218\ub97c \uae30\ub85d\ud558\uc600\ub2e4. \ud0c0\uc11d\uc5d0 \ub4f1\uc7a5\ud560 \uc2dc\uc758 \ub178\ub798\ub294 \ud50c\ub85c \ub77c\uc774\ub354\uc758 'In The Ayer'\ub77c\ub294 \uace1\uc774\ub2e4. \uc751\uc6d0\uac00\ub294 \ud2b8\ub79c\uc2a4\ud53d\uc158\uc758 '\ub77c\ub514\uc624'\ub97c \uac1c\uc0ac\ud55c \uac83\uc73c\ub85c, '\ub0a0\ub824\ub77c \uc774\uc885\uc6b1 \uc774\uc885\uc6b1 \uc774\uc885\uc6b1 \ub0a0\ub824\ubc84\ub824 \ub450\uc0b0\uc758 \uc774\uc885\uc6b1\uc774\ub2e4'\ub97c \ubc18\ubcf5\ud55c\ub2e4. \ub610\ud55c \uadf8\ub294 \uc601\ud654 \uc639\ubc15\uc758 \uc8fc\uc778\uacf5 \ubc30\uc5ed\uc744 \ub9e1\uc740 \ud1a0\ub2c8 \uc790\uc640 \ub2ee\uc558\ub2e4\uace0 \ud558\uc5ec \ud32c\ub4e4 \uc0ac\uc774\uc5d0\uc11c \u2018\uc639\ubc15\u2019 \ud639\uc740 \u2018\uc885\ubc15(\uc885\uc6b1+\uc639\ubc15)\u2019\uc774\ub77c\ub294 \uc560\uce6d\uc73c\ub85c \ubd88\ub9ac\uace0 \uc788\ub2e4.", "sentence_answer": " \ub3c4\ub8e8 \ub2a5\ub825 \uc740 \ub9ac\uadf8 \ub0b4 \ucd5c\uc815\uc0c1\uae09\uc774\ub2e4.", "paragraph_id": "6547838-14-0", "paragraph_question": "question: \uc774\uc885\uc6b1\uc774 \ub9ac\uadf8 \ub0b4 \ucd5c\uc815\uc0c1\uae09\uc778 \ub2a5\ub825\uc740?, context: \uc804\ud615\uc801\uc778 \ud14c\uc774\ube14 \uc138\ud130 \uc720\ud615\uc758 \uc120\uc218\ub85c\uc11c \ub9ac\uadf8\ub97c \ub300\ud45c\ud558\ub294 1\ubc88 \ud0c0\uc790\ub85c \uc77c\uceec\uc5b4\uc9c8 \uc815\ub3c4\ub85c \ube60\ub978 \ubc1c\uacfc \ud0c1\uc6d4\ud55c \uc8fc\ub8e8 \uc13c\uc2a4\ub97c \ubcf4\uc720\ud558\uace0 \uc788\uc5b4, \ubcd1\uc0b4\ud0c0\uac00 \uc801\uc740 \ud3b8\uc774\ub2e4. \uacf5\uaca9\uc801\uc73c\ub85c \ud0c0\uaca9\ud558\uba70 \uc120\uad6c\uc548\uc774 \uc88b\uace0 \ucee8\ud0dd \ub2a5\ub825\uc774 \ub6f0\uc5b4\ub098\uba70, \ubc00\uc5b4\uce58\ub294 \ub2a5\ub825\uc774 \ub6f0\uc5b4\ub098\ub2e4. \ubcc0\ud654\uad6c\uc5d0\ub294 \uc57d\uc810\uc774 \uc788\uc9c0\ub9cc \ucee4\ud2b8\ub97c \uc798\ud574 \ub048\uc9c8\uae34 \uc2b9\ubd80\ub97c \ud55c\ub2e4. \ub3c4\ub8e8 \ub2a5\ub825\uc740 \ub9ac\uadf8 \ub0b4 \ucd5c\uc815\uc0c1\uae09\uc774\ub2e4. 2006\ub144\ubd80\ud130 2011\ub144\uae4c\uc9c0 6\uc2dc\uc98c \ub3d9\uc548\uc758 \ub3c4\ub8e8 \uac2f\uc218\uac00 \uc774\ub300\ud615\uc758 \ub4a4\ub97c \uc774\uc740 2\uc704(212\uac1c)\ub97c \uae30\ub85d\ud588\ub2e4. \uadf8\ub7ec\ub098 \ucd5c\uadfc\uc5d0 \ub4e4\uc5b4\uc11c \uc810\uc810 \ub3c4\ub8e8 \uc2dc\ub3c4\uac00 \uc904\uc5b4\ub4e4\uace0 \uc788\ub294\ub370, \uadf8\ub294 \uc774\uc5d0 \ub300\ud574 \"\ub3c4\ub8e8\ub85c \uc88b\uc9c0\ub9cc 1\ub8e8\uc5d0\uc11c \uc0c1\ub300 \ubc30\ud130\ub9ac\ub97c \ud754\ub4e4\uace0 \ud6c4\uc18d\ud0c0 \ub54c 3\ub8e8\ub85c \uc9c4\ub8e8\ud558\ub294 \uac8c \ud0c0\uc790\uc5d0\uac8c\ub3c4 \uc720\ub9ac\ud558\ub2e4.\"\ub77c\uace0 \ub9d0\ud558\uae30\ub3c4 \ud588\uace0, \ub610\ud55c 2011\ub144 \uc2dc\uc98c \ucd08\uc5d0\ub294 \"\uccb4\ub825\uc801\uc778 \ubd80\ub2f4\uc774 \uc788\uc5b4\uc11c \uc548 \ub6f0\ub294 \uac83\uc740 \uc544\ub2c8\ub2e4. \uc7ac\uc791\ub144\uc5d0\ub3c4 \ud06c\uac8c \ub2e4\uce58\uace0 \uc62c \uc2dc\uc98c\ub3c4 \ubd80\uc0c1\uc73c\ub85c \uc880 \ub6f0\uae30 \uc5b4\ub835\ub2e4. \uc870\uae08 \uc870\uc2ec\ud558\ub824\uace0 \ud588\ub2e4.\"\ub77c\uace0 \ub9d0\ud588\ub2e4. \ub610\ud55c \uadfc\uc131\uc774 \uac15\ud55c \uc120\uc218\ub85c \uc798 \uc54c\ub824\uc838 \uc788\ub294\ub370, \uadf8 \uc608\ub85c 2009\ub144 6\uc6d4\ucd08\uc5d0\ub294 \ud131\uad00\uc808 \ubd80\uc0c1\uc73c\ub85c 2~3\uac1c\uc6d4\uc758 \uce58\ub8cc\uc640 \uc7ac\ud65c\uc774 \uc608\uc0c1\ub418\uc5c8\uc73c\ub098 \ubd88\uacfc \ud55c \ub2ec \ubc18 \ub9cc\uc5d0 \ubcf5\uadc0\ud55c \uc801\uc774 \uc788\ub2e4. \ub610\ud55c \uac19\uc740 \ud300 \ub3d9\ub8cc\uc778 \uc774\ud61c\ucc9c\uc740 2011\ub144 \uc2dc\uc98c \uc804\uc5d0 \uadf8\ub97c \ubcf4\uace0 \"\ub9c8\uce58 \ub3c5\uc57d \uba39\uace0 \ub6f0\ub294 \uac83 \uac19\ub2e4. \uc77c\ubcf8 \uc120\uc218\ub4e4\ub3c4 \ub3d9\uc601\uc0c1\uc744 \ubcf4\uc5ec\uc8fc\ub2c8 \uce6d\ucc2c\uc744 \uc544\ub07c\uc9c0 \uc54a\uc558\ub2e4. \ub2e4\uce60 \uac83 \uac19\uc544\uc11c \uacbd\uae30 \ub54c\ub9c8\ub2e4 \uad6c\uae09\ucc28\ub97c \ubd88\ub7ec\ub193\uace0 \ud574\uc57c\ud560 \uac83 \uac19\ub2e4.\"\ub77c\uace0 \ub9d0\ud558\uae30\ub3c4 \ud588\ub2e4. \uadf8\ub7ec\ub098 \ud3c9\uade0 \ud0c0\uc728\uc5d0 \ube44\ud574 \uc88c\uc644\ud22c\uc218 \uc0c1\ub300 \ud0c0\uc728\uc774 \uc57d\uac04 \ub5a8\uc5b4\uc9c0\ub294 \ud3b8\uc774\ub2e4. 2007\ub144 \uc2dc\uc98c\uc5d0 \uc88c\uc644 \uc0c1\ub300\ub85c 0.324\ub97c \uae30\ub85d\ud55c \uac83\uc744 \uc81c\uc678\ud558\uace0 2008\ub144 \uc2dc\uc98c 0.247, 2009\ub144 \uc2dc\uc98c 0.248, 2010\ub144 \uc2dc\uc98c 0.291\uc758 \ud0c0\uc728\uc744 \uae30\ub85d\ud574 \ubcf8\uc778\uc758 \uc2dc\uc98c \ud0c0\uc728\uc5d0 \ube44\ud574 \uc800\uc870\ud55c \uc131\uc801\uc744 \uae30\ub85d\ud588\ub2e4. \uadf8\ub9ac\uace0 \uc678\uc57c\uc218\ube44\uc5d0 \uc788\uc5b4\uc11c \ube60\ub978 \ubc1c\uc744 \uc0b4\ub9b0 \uc218\ube44 \ubc94\uc704\ub294 \ub113\uc73c\ub098 \uc5b4\uae68\ub294 \ud3c9\ubc94\ud558\ub2e4\uace0 \ud3c9\uac00\ubc1b\ub294\ub2e4. \ud604\ub300 \uc720\ub2c8\ucf58\uc2a4\uc640 \uc120\uc218\ub2e8 \uc2b9\uacc4\ub97c \ud55c \uc6b0\ub9ac, \ub125\uc13c \ud788\uc5b4\ub85c\uc988\ub97c \uc0c1\ub300\ub85c \uc9d5\ud06c\uc2a4\uac00 \uc788\uc5c8\ub2e4. 2007\ub144 \uc2dc\uc98c \ud604\ub300\ub97c \uc0c1\ub300\ub85c 0.246\uc758 \ud0c0\uc728\uc744 \uae30\ub85d\ud588\uace0, \uc774\ub4ec\ud574 \uc6b0\ub9ac \ud788\uc5b4\ub85c\uc988\ub97c \uc0c1\ub300\ub85c 0.189, 2009\ub144 \uc2dc\uc98c\uc5d0\ub294 0.286\ub97c \uae30\ub85d\ud588\ub2e4. \uadf8\ub7ec\ub098 2010\ub144 \uc2dc\uc98c\uc5d0\ub294 \ub125\uc13c\uc744 \uc0c1\ub300\ub85c \ub9f9\ud0c0\ub97c \ud718\ub458\ub800\uace0, \uc0c1\ub300\ud300 \uc911 \uac00\uc7a5 \ub192\uc740 0.511\uc758 \ud0c0\uc728\uc744 \uae30\ub85d\ud588\ub2e4. \ud14c\uc774\ube14 \uc138\ud130\uc774\uae34 \ud558\uc9c0\ub9cc \uc774\ub840\uc801\uc73c\ub85c \ub4dd\uc810\uad8c\uc5d0\uc11c\uc758 \ud0c0\uc728\uc774 \ub192\uc544 2007\ub144 \uc2dc\uc98c\uc5d0 0.385(\ub9ac\uadf8 2\uc704), 2010\ub144 \uc2dc\uc98c\uc5d0 0.359(\ub9ac\uadf8 8\uc704)\ub97c \uae30\ub85d\ud558\uc600\ub2e4. \ub610\ud55c 1\ubc88 \ud0c0\uc790\ub2f5\uac8c \ud648\ub7f0 \uc218\ub294 \uc801\uc740 \ud3b8\uc774\ub098, \uc758\uc678\ub85c \uad6d\uac00\ub300\ud45c \uacbd\uae30\ub098 \ud3ec\uc2a4\ud2b8 \uc2dc\uc98c\ub4f1 \uc911\uc694\ud55c \uacbd\uae30\uc5d0\uc11c \ud648\ub7f0\uc744 \uce58\ub294 \uacbd\uc6b0\uac00 \uc788\ub2e4. 2007\ub144 \ud50c\ub808\uc774\uc624\ud504 2\ucc28\uc804\uc5d0\uc11c\uc758 \uc194\ub85c \ud648\ub7f0, 2008\ub144 \ubca0\uc774\uc9d5 \uc62c\ub9bc\ud53d \uc57c\uad6c \uc608\uc120 \ub300\ub9cc\uc804\uc5d0\uc11c\uc758 3\uc810 \ud648\ub7f0, 2010\ub144 \uc900\ud50c\ub808\uc774\uc624\ud504 3\ucc28\uc804\uc5d0\uc11c\uc758 \ud648\ub7f0\uc774 \uadf8 \uc608\uc774\ub2e4. 2010\ub144 \uc2dc\uc98c\uc5d0 5\uac1c\uc758 \ud648\ub7f0\uc744 \uae30\ub85d\ud558\uc5ec \uc774\uc804 \uc2dc\uc98c\uae4c\uc9c0\uc758 \ud648\ub7f0 \ucd1d \ud569\ubcf4\ub2e4 \ub354 \ub9ce\uc740 \ud648\ub7f0 \uac1c\uc218\ub97c \uae30\ub85d\ud558\uc600\ub2e4. \ud0c0\uc11d\uc5d0 \ub4f1\uc7a5\ud560 \uc2dc\uc758 \ub178\ub798\ub294 \ud50c\ub85c \ub77c\uc774\ub354\uc758 'In The Ayer'\ub77c\ub294 \uace1\uc774\ub2e4. \uc751\uc6d0\uac00\ub294 \ud2b8\ub79c\uc2a4\ud53d\uc158\uc758 '\ub77c\ub514\uc624'\ub97c \uac1c\uc0ac\ud55c \uac83\uc73c\ub85c, '\ub0a0\ub824\ub77c \uc774\uc885\uc6b1 \uc774\uc885\uc6b1 \uc774\uc885\uc6b1 \ub0a0\ub824\ubc84\ub824 \ub450\uc0b0\uc758 \uc774\uc885\uc6b1\uc774\ub2e4'\ub97c \ubc18\ubcf5\ud55c\ub2e4. \ub610\ud55c \uadf8\ub294 \uc601\ud654 \uc639\ubc15\uc758 \uc8fc\uc778\uacf5 \ubc30\uc5ed\uc744 \ub9e1\uc740 \ud1a0\ub2c8 \uc790\uc640 \ub2ee\uc558\ub2e4\uace0 \ud558\uc5ec \ud32c\ub4e4 \uc0ac\uc774\uc5d0\uc11c \u2018\uc639\ubc15\u2019 \ud639\uc740 \u2018\uc885\ubc15(\uc885\uc6b1+\uc639\ubc15)\u2019\uc774\ub77c\ub294 \uc560\uce6d\uc73c\ub85c \ubd88\ub9ac\uace0 \uc788\ub2e4."} {"question": "\ubb38\uc7ac\uc778 \ub300\ud1b5\ub839\uc740 \uc0ac\ub4dc\ubc30\uce58 \uacb0\uc815\uc740 \ubb34\uc5c7\uc5d0 \uadfc\uac70\ud574, \ud55c.\ubbf8 \ud569\uc758\ud558\uc5d0 \uacb0\uc815\ub41c\uac83\uc774\ub77c \ud558\uc600\ub098\uc694?", "paragraph": "\ubb38\uc7ac\uc778 \uc815\ubd80 \uad6d\uac00\uc548\ubcf4\uc2e4\uc7a5 \uc815\uc758\uc6a9\uc740 2017\ub144 6\uc6d4 9\uc77c \uccad\uc640\ub300\uc5d0\uc11c \u201c\uc0ac\ub4dc\ub294 \ubd81\ud55c\uc758 \uc810\uc99d\ud558\ub294 \uc704\ud611\uc73c\ub85c\ubd80\ud130 \ud55c\uad6d\uacfc \uc8fc\ud55c\ubbf8\uad70\uc744 \ubcf4\ud638\ud558\uae30 \uc704\ud574 \uacb0\uc815\ud55c \uac83\"\uc774\ub77c\uace0 \uacf5\uc2dd\uc801\uc73c\ub85c \ubc1c\ud45c\ud588\ub2e4. \ubb38\uc7ac\uc778 \ub300\ud1b5\ub839\uc740 2017\ub144 6\uc6d4 20\uc77c \uccad\uc640\ub300\uc5d0\uc11c CBS, \uc6cc\uc2f1\ud134\ud3ec\uc2a4\ud2b8 \uc778\ud130\ubdf0\ub97c \uac00\uc84c\uace0 \uc0ac\ub4dc \ubc30\uce58 \uad00\ub828 \uc9c8\ubb38\uc5d0\uc11c \u201c\uc0ac\ub4dc \ubc30\uce58 \uacb0\uc815\uc740 \uc6b0\ub9ac \ud55c\uad6d\uacfc \uc8fc\ud55c\ubbf8\uad70\uc758 \uc548\uc804\uc744 \uc704\ud574\uc11c \ud55c\u00b7\ubbf8\ub3d9\ub9f9\uc5d0 \uadfc\uac70\ud574 \ud55c\uad6d\uacfc \ubbf8\uad6d\uc774 \ud569\uc758\ud574\uc11c \uacb0\uc815\ud55c \uac83\u201d\uc774\ub77c\uace0 \ub300\ub2f5\ud588\ub2e4. 2017\ub144 6\uc6d4 29\uc77c(\ubbf8\uad6d \ub3d9\ubd80\uc2dc\uac04), \ubb38\uc7ac\uc778 \ub300\ud1b5\ub839\uc740 \uc6cc\uc2f1\ud134D.C. \ubbf8\uad6d \uc758\ud68c \ud3f4 \ub77c\uc774\uc5b8 \ud558\uc6d0\uc758\uc7a5 \ub4f1 \ud558\uc6d0 \uc9c0\ub3c4\ubd80\uc640 \uac00\uc9c4 \uac04\ub2f4\ud68c\uc5d0\uc11c \u201c\uc0ac\ub4dc\ub294 \ud55c\ubbf8\ub3d9\ub9f9\uc5d0 \uae30\ucd08\ud55c \ud569\uc758\uc774\uace0 \ud55c\uad6d \uad6d\ubbfc\uacfc \uc8fc\ud55c\ubbf8\uad70\uc758 \uc0dd\uba85\uc744 \ubcf4\ud638\ud558\uae30 \uc704\ud55c \uac83\"\uc774\ub77c\uace0 \uac15\uc870\ud588\ub2e4. \ud55c\ubc18\ub3c4 \uc0ac\ub4dc \ubc30\uce58\ub294 \ubbf8\uad6d\ubcf8\ud1a0 \ubc29\uc5b4\uc6a9\uc774 \uc544\ub2c8\ub77c \ud55c\ubc18\ub3c4\uc5d0 \uc8fc\ub454\ud558\uace0 \uc788\ub294 \uc8fc\ud55c\ubbf8\uad70\uacfc \ud55c\uad6d \uad6d\ubbfc\ub4e4\uc744 \ubcf4\ud638\ud558\uae30 \uc704\ud574 \uacb0\uc815\ud588\ub2e4\ub294 \uac83\uc744 \ubb38\uc7ac\uc778 \uc815\ubd80\uac00 \uc778\uc815\ud55c \uac83\uc774\ub2e4.", "answer": "\ud55c\ubbf8\ub3d9\ub9f9", "sentence": "2017\ub144 6\uc6d4 29\uc77c(\ubbf8\uad6d \ub3d9\ubd80\uc2dc\uac04), \ubb38\uc7ac\uc778 \ub300\ud1b5\ub839\uc740 \uc6cc\uc2f1\ud134D.C. \ubbf8\uad6d \uc758\ud68c \ud3f4 \ub77c\uc774\uc5b8 \ud558\uc6d0\uc758\uc7a5 \ub4f1 \ud558\uc6d0 \uc9c0\ub3c4\ubd80\uc640 \uac00\uc9c4 \uac04\ub2f4\ud68c\uc5d0\uc11c \u201c\uc0ac\ub4dc\ub294 \ud55c\ubbf8\ub3d9\ub9f9 \uc5d0 \uae30\ucd08\ud55c \ud569\uc758\uc774\uace0 \ud55c\uad6d \uad6d\ubbfc\uacfc \uc8fc\ud55c\ubbf8\uad70\uc758 \uc0dd\uba85\uc744 \ubcf4\ud638\ud558\uae30 \uc704\ud55c \uac83\"\uc774\ub77c\uace0 \uac15\uc870\ud588\ub2e4.", "paragraph_sentence": "\ubb38\uc7ac\uc778 \uc815\ubd80 \uad6d\uac00\uc548\ubcf4\uc2e4\uc7a5 \uc815\uc758\uc6a9\uc740 2017\ub144 6\uc6d4 9\uc77c \uccad\uc640\ub300\uc5d0\uc11c \u201c\uc0ac\ub4dc\ub294 \ubd81\ud55c\uc758 \uc810\uc99d\ud558\ub294 \uc704\ud611\uc73c\ub85c\ubd80\ud130 \ud55c\uad6d\uacfc \uc8fc\ud55c\ubbf8\uad70\uc744 \ubcf4\ud638\ud558\uae30 \uc704\ud574 \uacb0\uc815\ud55c \uac83\"\uc774\ub77c\uace0 \uacf5\uc2dd\uc801\uc73c\ub85c \ubc1c\ud45c\ud588\ub2e4. \ubb38\uc7ac\uc778 \ub300\ud1b5\ub839\uc740 2017\ub144 6\uc6d4 20\uc77c \uccad\uc640\ub300\uc5d0\uc11c CBS, \uc6cc\uc2f1\ud134\ud3ec\uc2a4\ud2b8 \uc778\ud130\ubdf0\ub97c \uac00\uc84c\uace0 \uc0ac\ub4dc \ubc30\uce58 \uad00\ub828 \uc9c8\ubb38\uc5d0\uc11c \u201c\uc0ac\ub4dc \ubc30\uce58 \uacb0\uc815\uc740 \uc6b0\ub9ac \ud55c\uad6d\uacfc \uc8fc\ud55c\ubbf8\uad70\uc758 \uc548\uc804\uc744 \uc704\ud574\uc11c \ud55c\u00b7\ubbf8\ub3d9\ub9f9\uc5d0 \uadfc\uac70\ud574 \ud55c\uad6d\uacfc \ubbf8\uad6d\uc774 \ud569\uc758\ud574\uc11c \uacb0\uc815\ud55c \uac83\u201d\uc774\ub77c\uace0 \ub300\ub2f5\ud588\ub2e4. 2017\ub144 6\uc6d4 29\uc77c(\ubbf8\uad6d \ub3d9\ubd80\uc2dc\uac04), \ubb38\uc7ac\uc778 \ub300\ud1b5\ub839\uc740 \uc6cc\uc2f1\ud134D.C. \ubbf8\uad6d \uc758\ud68c \ud3f4 \ub77c\uc774\uc5b8 \ud558\uc6d0\uc758\uc7a5 \ub4f1 \ud558\uc6d0 \uc9c0\ub3c4\ubd80\uc640 \uac00\uc9c4 \uac04\ub2f4\ud68c\uc5d0\uc11c \u201c\uc0ac\ub4dc\ub294 \ud55c\ubbf8\ub3d9\ub9f9 \uc5d0 \uae30\ucd08\ud55c \ud569\uc758\uc774\uace0 \ud55c\uad6d \uad6d\ubbfc\uacfc \uc8fc\ud55c\ubbf8\uad70\uc758 \uc0dd\uba85\uc744 \ubcf4\ud638\ud558\uae30 \uc704\ud55c \uac83\"\uc774\ub77c\uace0 \uac15\uc870\ud588\ub2e4. \ud55c\ubc18\ub3c4 \uc0ac\ub4dc \ubc30\uce58\ub294 \ubbf8\uad6d\ubcf8\ud1a0 \ubc29\uc5b4\uc6a9\uc774 \uc544\ub2c8\ub77c \ud55c\ubc18\ub3c4\uc5d0 \uc8fc\ub454\ud558\uace0 \uc788\ub294 \uc8fc\ud55c\ubbf8\uad70\uacfc \ud55c\uad6d \uad6d\ubbfc\ub4e4\uc744 \ubcf4\ud638\ud558\uae30 \uc704\ud574 \uacb0\uc815\ud588\ub2e4\ub294 \uac83\uc744 \ubb38\uc7ac\uc778 \uc815\ubd80\uac00 \uc778\uc815\ud55c \uac83\uc774\ub2e4.", "paragraph_answer": "\ubb38\uc7ac\uc778 \uc815\ubd80 \uad6d\uac00\uc548\ubcf4\uc2e4\uc7a5 \uc815\uc758\uc6a9\uc740 2017\ub144 6\uc6d4 9\uc77c \uccad\uc640\ub300\uc5d0\uc11c \u201c\uc0ac\ub4dc\ub294 \ubd81\ud55c\uc758 \uc810\uc99d\ud558\ub294 \uc704\ud611\uc73c\ub85c\ubd80\ud130 \ud55c\uad6d\uacfc \uc8fc\ud55c\ubbf8\uad70\uc744 \ubcf4\ud638\ud558\uae30 \uc704\ud574 \uacb0\uc815\ud55c \uac83\"\uc774\ub77c\uace0 \uacf5\uc2dd\uc801\uc73c\ub85c \ubc1c\ud45c\ud588\ub2e4. \ubb38\uc7ac\uc778 \ub300\ud1b5\ub839\uc740 2017\ub144 6\uc6d4 20\uc77c \uccad\uc640\ub300\uc5d0\uc11c CBS, \uc6cc\uc2f1\ud134\ud3ec\uc2a4\ud2b8 \uc778\ud130\ubdf0\ub97c \uac00\uc84c\uace0 \uc0ac\ub4dc \ubc30\uce58 \uad00\ub828 \uc9c8\ubb38\uc5d0\uc11c \u201c\uc0ac\ub4dc \ubc30\uce58 \uacb0\uc815\uc740 \uc6b0\ub9ac \ud55c\uad6d\uacfc \uc8fc\ud55c\ubbf8\uad70\uc758 \uc548\uc804\uc744 \uc704\ud574\uc11c \ud55c\u00b7\ubbf8\ub3d9\ub9f9\uc5d0 \uadfc\uac70\ud574 \ud55c\uad6d\uacfc \ubbf8\uad6d\uc774 \ud569\uc758\ud574\uc11c \uacb0\uc815\ud55c \uac83\u201d\uc774\ub77c\uace0 \ub300\ub2f5\ud588\ub2e4. 2017\ub144 6\uc6d4 29\uc77c(\ubbf8\uad6d \ub3d9\ubd80\uc2dc\uac04), \ubb38\uc7ac\uc778 \ub300\ud1b5\ub839\uc740 \uc6cc\uc2f1\ud134D.C. \ubbf8\uad6d \uc758\ud68c \ud3f4 \ub77c\uc774\uc5b8 \ud558\uc6d0\uc758\uc7a5 \ub4f1 \ud558\uc6d0 \uc9c0\ub3c4\ubd80\uc640 \uac00\uc9c4 \uac04\ub2f4\ud68c\uc5d0\uc11c \u201c\uc0ac\ub4dc\ub294 \ud55c\ubbf8\ub3d9\ub9f9 \uc5d0 \uae30\ucd08\ud55c \ud569\uc758\uc774\uace0 \ud55c\uad6d \uad6d\ubbfc\uacfc \uc8fc\ud55c\ubbf8\uad70\uc758 \uc0dd\uba85\uc744 \ubcf4\ud638\ud558\uae30 \uc704\ud55c \uac83\"\uc774\ub77c\uace0 \uac15\uc870\ud588\ub2e4. \ud55c\ubc18\ub3c4 \uc0ac\ub4dc \ubc30\uce58\ub294 \ubbf8\uad6d\ubcf8\ud1a0 \ubc29\uc5b4\uc6a9\uc774 \uc544\ub2c8\ub77c \ud55c\ubc18\ub3c4\uc5d0 \uc8fc\ub454\ud558\uace0 \uc788\ub294 \uc8fc\ud55c\ubbf8\uad70\uacfc \ud55c\uad6d \uad6d\ubbfc\ub4e4\uc744 \ubcf4\ud638\ud558\uae30 \uc704\ud574 \uacb0\uc815\ud588\ub2e4\ub294 \uac83\uc744 \ubb38\uc7ac\uc778 \uc815\ubd80\uac00 \uc778\uc815\ud55c \uac83\uc774\ub2e4.", "sentence_answer": "2017\ub144 6\uc6d4 29\uc77c(\ubbf8\uad6d \ub3d9\ubd80\uc2dc\uac04), \ubb38\uc7ac\uc778 \ub300\ud1b5\ub839\uc740 \uc6cc\uc2f1\ud134D.C. \ubbf8\uad6d \uc758\ud68c \ud3f4 \ub77c\uc774\uc5b8 \ud558\uc6d0\uc758\uc7a5 \ub4f1 \ud558\uc6d0 \uc9c0\ub3c4\ubd80\uc640 \uac00\uc9c4 \uac04\ub2f4\ud68c\uc5d0\uc11c \u201c\uc0ac\ub4dc\ub294 \ud55c\ubbf8\ub3d9\ub9f9 \uc5d0 \uae30\ucd08\ud55c \ud569\uc758\uc774\uace0 \ud55c\uad6d \uad6d\ubbfc\uacfc \uc8fc\ud55c\ubbf8\uad70\uc758 \uc0dd\uba85\uc744 \ubcf4\ud638\ud558\uae30 \uc704\ud55c \uac83\"\uc774\ub77c\uace0 \uac15\uc870\ud588\ub2e4.", "paragraph_id": "6546295-0-1", "paragraph_question": "question: \ubb38\uc7ac\uc778 \ub300\ud1b5\ub839\uc740 \uc0ac\ub4dc\ubc30\uce58 \uacb0\uc815\uc740 \ubb34\uc5c7\uc5d0 \uadfc\uac70\ud574, \ud55c.\ubbf8 \ud569\uc758\ud558\uc5d0 \uacb0\uc815\ub41c\uac83\uc774\ub77c \ud558\uc600\ub098\uc694?, context: \ubb38\uc7ac\uc778 \uc815\ubd80 \uad6d\uac00\uc548\ubcf4\uc2e4\uc7a5 \uc815\uc758\uc6a9\uc740 2017\ub144 6\uc6d4 9\uc77c \uccad\uc640\ub300\uc5d0\uc11c \u201c\uc0ac\ub4dc\ub294 \ubd81\ud55c\uc758 \uc810\uc99d\ud558\ub294 \uc704\ud611\uc73c\ub85c\ubd80\ud130 \ud55c\uad6d\uacfc \uc8fc\ud55c\ubbf8\uad70\uc744 \ubcf4\ud638\ud558\uae30 \uc704\ud574 \uacb0\uc815\ud55c \uac83\"\uc774\ub77c\uace0 \uacf5\uc2dd\uc801\uc73c\ub85c \ubc1c\ud45c\ud588\ub2e4. \ubb38\uc7ac\uc778 \ub300\ud1b5\ub839\uc740 2017\ub144 6\uc6d4 20\uc77c \uccad\uc640\ub300\uc5d0\uc11c CBS, \uc6cc\uc2f1\ud134\ud3ec\uc2a4\ud2b8 \uc778\ud130\ubdf0\ub97c \uac00\uc84c\uace0 \uc0ac\ub4dc \ubc30\uce58 \uad00\ub828 \uc9c8\ubb38\uc5d0\uc11c \u201c\uc0ac\ub4dc \ubc30\uce58 \uacb0\uc815\uc740 \uc6b0\ub9ac \ud55c\uad6d\uacfc \uc8fc\ud55c\ubbf8\uad70\uc758 \uc548\uc804\uc744 \uc704\ud574\uc11c \ud55c\u00b7\ubbf8\ub3d9\ub9f9\uc5d0 \uadfc\uac70\ud574 \ud55c\uad6d\uacfc \ubbf8\uad6d\uc774 \ud569\uc758\ud574\uc11c \uacb0\uc815\ud55c \uac83\u201d\uc774\ub77c\uace0 \ub300\ub2f5\ud588\ub2e4. 2017\ub144 6\uc6d4 29\uc77c(\ubbf8\uad6d \ub3d9\ubd80\uc2dc\uac04), \ubb38\uc7ac\uc778 \ub300\ud1b5\ub839\uc740 \uc6cc\uc2f1\ud134D.C. \ubbf8\uad6d \uc758\ud68c \ud3f4 \ub77c\uc774\uc5b8 \ud558\uc6d0\uc758\uc7a5 \ub4f1 \ud558\uc6d0 \uc9c0\ub3c4\ubd80\uc640 \uac00\uc9c4 \uac04\ub2f4\ud68c\uc5d0\uc11c \u201c\uc0ac\ub4dc\ub294 \ud55c\ubbf8\ub3d9\ub9f9\uc5d0 \uae30\ucd08\ud55c \ud569\uc758\uc774\uace0 \ud55c\uad6d \uad6d\ubbfc\uacfc \uc8fc\ud55c\ubbf8\uad70\uc758 \uc0dd\uba85\uc744 \ubcf4\ud638\ud558\uae30 \uc704\ud55c \uac83\"\uc774\ub77c\uace0 \uac15\uc870\ud588\ub2e4. \ud55c\ubc18\ub3c4 \uc0ac\ub4dc \ubc30\uce58\ub294 \ubbf8\uad6d\ubcf8\ud1a0 \ubc29\uc5b4\uc6a9\uc774 \uc544\ub2c8\ub77c \ud55c\ubc18\ub3c4\uc5d0 \uc8fc\ub454\ud558\uace0 \uc788\ub294 \uc8fc\ud55c\ubbf8\uad70\uacfc \ud55c\uad6d \uad6d\ubbfc\ub4e4\uc744 \ubcf4\ud638\ud558\uae30 \uc704\ud574 \uacb0\uc815\ud588\ub2e4\ub294 \uac83\uc744 \ubb38\uc7ac\uc778 \uc815\ubd80\uac00 \uc778\uc815\ud55c \uac83\uc774\ub2e4."} {"question": "\uc81c\ud6c4\uad6d\uc774\ub098 \uc9c0\uc5ed\uad70\ubc8c \uc544\ub798\uc5d0 \uc788\ub358 \uac01 \uc9c0\uc5ed\uc758 \uad70\uc0ac\ub97c \ub3d9\uc778\ub3c4 \ud68c\uc0ac\uac00 \uc9c1\uc811 \ud1b5\uc194\ud558\ub3c4\ub85d \ud558\ub294 \uac83\uc740?", "paragraph": "19\uc138\uae30\uc5d0 \ub4e4\uc5b4 \ucd1d\ub3c5 \ub9ac\ucc98\ub4dc \uc6e8\uc2ac\ub9ac\ub294 \ub3d9\uc778\ub3c4 \ud68c\uc0ac\uc758 \uc9c0\ubc30 \uc9c0\uc5ed\uc744 \ud655\uc7a5\ud558\uae30 \uc704\ud55c \uc0c8\ub85c\uc6b4 \uc815\ucc45\uc744 \uc2dc\ud589\ud558\uc600\ub2e4. \uc774 \uc815\ucc45\uc5d0 \ub530\ub77c \uc601\uad6d \ub3d9\uc778\ub3c4 \ud68c\uc0ac\ub294 \uc778\ub3c4 \ub0b4\uc758 \uc81c\ud6c4\uad6d\ub4e4\uacfc \uc885\uc18d \ub3d9\ub9f9\uc744 \uacb0\uc131\ud558\uc600\ub2e4. \uc885\uc18d \ub3d9\ub9f9\uc740 \uc81c\ud6c4\uad6d\uc774\ub098 \uc9c0\uc5ed \uad70\ubc8c\uc758 \ud718\ud558\uc5d0 \uc788\ub358 \uac01 \uc9c0\uc5ed\uc758 \uad70\uc0ac\ub97c \ub3d9\uc778\ub3c4 \ud68c\uc0ac\uac00 \uc9c1\uc811 \ud1b5\uc194\ud558\ub3c4\ub85d \ud558\ub294 \uac83\uc73c\ub85c, \uc81c2\ucc28 \uc601\uad6d-\uc2dc\ud06c \uc804\uc7c1 \uc774\ud6c4 \uce74\uc774\ubca0\ub974\ud30c\ud06c\ud230\ud06c\uc640 \uc8fc\uc640 \uce74\uc288\ubbf8\ub974, \ud380\uc790\ube0c \uc9c0\uc5ed\uc5d0 \uc874\uc7ac\ud558\ub358 \uc81c\ud6c4\uad6d\ub4e4\uc744 \ub300\ud56d\uc73c\ub85c \ud558\uc600\ub294\ub370 \uc774 \uc9c0\uc5ed\uc758 \uc218\uc7a5\uc774\uc5c8\ub358 \ud78c\ub450\uad50 \uc9c0\uc5ed\uc758 \ub9c8\ud558\ub77c\uc790\ub098 \uc774\uc2ac\ub78c\uad50 \uc9c0\uc5ed\uc758 \ub098\uc640\ube0c \ub4f1 \uc81c\ud6c4\ub4e4\uc744 \uc885\uc18d \ub3d9\ub9f9 \uc548\uc73c\ub85c \ud3b8\uc81c\ud558\uc600\ub2e4. \ud2b9\ud788 \uce74\uc2dc\ubbf8\ub974 \uc9c0\uc5ed\uc740 \uc2dc\ud06c \uc804\uc7c1 \uc9c1\ud6c4\uc778 1850\ub144 \uc554\ub2c8\ucc28\ub974 \uc870\uc57d\uc744 \ud1b5\ud574 \uc7a0\ubb34 \uc9c0\ubc29\uc758 \ub3c4\uadf8\ub9ac \uc655\uc870\uc5d0\uac8c \ud560\uc591\ub418\uc5b4 \uc81c\ud6c4\uad6d\uc774 \ub418\uc5c8\ub2e4. \ud55c\ud3b8 1814\ub144\uc5d0\uc11c 1816\ub144\uae4c\uc9c0 \uc601\uad6d\ub839 \uc778\ub3c4\uc640 \ub124\ud314 \uc0ac\uc774\uc758 \uad6d\uacbd \ubd84\uc7c1\uc73c\ub85c \uc778\ud574 \uc77c\uc5b4\ub09c \uc601\uad6d-\ub124\ud314 \uc804\uc7c1\uc758 \uacb0\uacfc \uad6c\ub974\uce74\uc2a4 \uc9c0\uc5ed\uc774 \uc601\uad6d\uc758 \uc9c0\ubc30\uc5d0 \ub193\uc774\uac8c \ub418\uc5c8\ub2e4. \ub610\ud55c 1854\ub144\uc5d0\ub294 \uc9c0\uae08\uc758 \ub9c8\ud558\ub77c\uc288\ud2b8\ub77c \uc8fc\uc5d0 \uc18d\ud558\ub294 \ubca0\ub77c\ub974 \uc9c0\ubc29\uc774 \uc885\uc18d \ub3d9\ub9f9\uc5d0 \ud3ec\ud568\ub418\uc5c8\uace0, 2\ub144 \ub4a4\uc5d0\ub294 \uc544\uc640\ub4dc\uac00 \uc885\uc18d \ub3d9\ub9f9\uc5d0 \ud3b8\uc785\ub418\uc5c8\ub2e4.", "answer": "\uc885\uc18d \ub3d9\ub9f9", "sentence": "\uc774 \uc815\ucc45\uc5d0 \ub530\ub77c \uc601\uad6d \ub3d9\uc778\ub3c4 \ud68c\uc0ac\ub294 \uc778\ub3c4 \ub0b4\uc758 \uc81c\ud6c4\uad6d\ub4e4\uacfc \uc885\uc18d \ub3d9\ub9f9 \uc744", "paragraph_sentence": "19\uc138\uae30\uc5d0 \ub4e4\uc5b4 \ucd1d\ub3c5 \ub9ac\ucc98\ub4dc \uc6e8\uc2ac\ub9ac\ub294 \ub3d9\uc778\ub3c4 \ud68c\uc0ac\uc758 \uc9c0\ubc30 \uc9c0\uc5ed\uc744 \ud655\uc7a5\ud558\uae30 \uc704\ud55c \uc0c8\ub85c\uc6b4 \uc815\ucc45\uc744 \uc2dc\ud589\ud558\uc600\ub2e4. \uc774 \uc815\ucc45\uc5d0 \ub530\ub77c \uc601\uad6d \ub3d9\uc778\ub3c4 \ud68c\uc0ac\ub294 \uc778\ub3c4 \ub0b4\uc758 \uc81c\ud6c4\uad6d\ub4e4\uacfc \uc885\uc18d \ub3d9\ub9f9 \uc744 \uacb0\uc131\ud558\uc600\ub2e4. \uc885\uc18d \ub3d9\ub9f9\uc740 \uc81c\ud6c4\uad6d\uc774\ub098 \uc9c0\uc5ed \uad70\ubc8c\uc758 \ud718\ud558\uc5d0 \uc788\ub358 \uac01 \uc9c0\uc5ed\uc758 \uad70\uc0ac\ub97c \ub3d9\uc778\ub3c4 \ud68c\uc0ac\uac00 \uc9c1\uc811 \ud1b5\uc194\ud558\ub3c4\ub85d \ud558\ub294 \uac83\uc73c\ub85c, \uc81c2\ucc28 \uc601\uad6d-\uc2dc\ud06c \uc804\uc7c1 \uc774\ud6c4 \uce74\uc774\ubca0\ub974\ud30c\ud06c\ud230\ud06c\uc640 \uc8fc\uc640 \uce74\uc288\ubbf8\ub974, \ud380\uc790\ube0c \uc9c0\uc5ed\uc5d0 \uc874\uc7ac\ud558\ub358 \uc81c\ud6c4\uad6d\ub4e4\uc744 \ub300\ud56d\uc73c\ub85c \ud558\uc600\ub294\ub370 \uc774 \uc9c0\uc5ed\uc758 \uc218\uc7a5\uc774\uc5c8\ub358 \ud78c\ub450\uad50 \uc9c0\uc5ed\uc758 \ub9c8\ud558\ub77c\uc790\ub098 \uc774\uc2ac\ub78c\uad50 \uc9c0\uc5ed\uc758 \ub098\uc640\ube0c \ub4f1 \uc81c\ud6c4\ub4e4\uc744 \uc885\uc18d \ub3d9\ub9f9 \uc548\uc73c\ub85c \ud3b8\uc81c\ud558\uc600\ub2e4. \ud2b9\ud788 \uce74\uc2dc\ubbf8\ub974 \uc9c0\uc5ed\uc740 \uc2dc\ud06c \uc804\uc7c1 \uc9c1\ud6c4\uc778 1850\ub144 \uc554\ub2c8\ucc28\ub974 \uc870\uc57d\uc744 \ud1b5\ud574 \uc7a0\ubb34 \uc9c0\ubc29\uc758 \ub3c4\uadf8\ub9ac \uc655\uc870\uc5d0\uac8c \ud560\uc591\ub418\uc5b4 \uc81c\ud6c4\uad6d\uc774 \ub418\uc5c8\ub2e4. \ud55c\ud3b8 1814\ub144\uc5d0\uc11c 1816\ub144\uae4c\uc9c0 \uc601\uad6d\ub839 \uc778\ub3c4\uc640 \ub124\ud314 \uc0ac\uc774\uc758 \uad6d\uacbd \ubd84\uc7c1\uc73c\ub85c \uc778\ud574 \uc77c\uc5b4\ub09c \uc601\uad6d-\ub124\ud314 \uc804\uc7c1\uc758 \uacb0\uacfc \uad6c\ub974\uce74\uc2a4 \uc9c0\uc5ed\uc774 \uc601\uad6d\uc758 \uc9c0\ubc30\uc5d0 \ub193\uc774\uac8c \ub418\uc5c8\ub2e4. \ub610\ud55c 1854\ub144\uc5d0\ub294 \uc9c0\uae08\uc758 \ub9c8\ud558\ub77c\uc288\ud2b8\ub77c \uc8fc\uc5d0 \uc18d\ud558\ub294 \ubca0\ub77c\ub974 \uc9c0\ubc29\uc774 \uc885\uc18d \ub3d9\ub9f9\uc5d0 \ud3ec\ud568\ub418\uc5c8\uace0, 2\ub144 \ub4a4\uc5d0\ub294 \uc544\uc640\ub4dc\uac00 \uc885\uc18d \ub3d9\ub9f9\uc5d0 \ud3b8\uc785\ub418\uc5c8\ub2e4.", "paragraph_answer": "19\uc138\uae30\uc5d0 \ub4e4\uc5b4 \ucd1d\ub3c5 \ub9ac\ucc98\ub4dc \uc6e8\uc2ac\ub9ac\ub294 \ub3d9\uc778\ub3c4 \ud68c\uc0ac\uc758 \uc9c0\ubc30 \uc9c0\uc5ed\uc744 \ud655\uc7a5\ud558\uae30 \uc704\ud55c \uc0c8\ub85c\uc6b4 \uc815\ucc45\uc744 \uc2dc\ud589\ud558\uc600\ub2e4. \uc774 \uc815\ucc45\uc5d0 \ub530\ub77c \uc601\uad6d \ub3d9\uc778\ub3c4 \ud68c\uc0ac\ub294 \uc778\ub3c4 \ub0b4\uc758 \uc81c\ud6c4\uad6d\ub4e4\uacfc \uc885\uc18d \ub3d9\ub9f9 \uc744 \uacb0\uc131\ud558\uc600\ub2e4. \uc885\uc18d \ub3d9\ub9f9\uc740 \uc81c\ud6c4\uad6d\uc774\ub098 \uc9c0\uc5ed \uad70\ubc8c\uc758 \ud718\ud558\uc5d0 \uc788\ub358 \uac01 \uc9c0\uc5ed\uc758 \uad70\uc0ac\ub97c \ub3d9\uc778\ub3c4 \ud68c\uc0ac\uac00 \uc9c1\uc811 \ud1b5\uc194\ud558\ub3c4\ub85d \ud558\ub294 \uac83\uc73c\ub85c, \uc81c2\ucc28 \uc601\uad6d-\uc2dc\ud06c \uc804\uc7c1 \uc774\ud6c4 \uce74\uc774\ubca0\ub974\ud30c\ud06c\ud230\ud06c\uc640 \uc8fc\uc640 \uce74\uc288\ubbf8\ub974, \ud380\uc790\ube0c \uc9c0\uc5ed\uc5d0 \uc874\uc7ac\ud558\ub358 \uc81c\ud6c4\uad6d\ub4e4\uc744 \ub300\ud56d\uc73c\ub85c \ud558\uc600\ub294\ub370 \uc774 \uc9c0\uc5ed\uc758 \uc218\uc7a5\uc774\uc5c8\ub358 \ud78c\ub450\uad50 \uc9c0\uc5ed\uc758 \ub9c8\ud558\ub77c\uc790\ub098 \uc774\uc2ac\ub78c\uad50 \uc9c0\uc5ed\uc758 \ub098\uc640\ube0c \ub4f1 \uc81c\ud6c4\ub4e4\uc744 \uc885\uc18d \ub3d9\ub9f9 \uc548\uc73c\ub85c \ud3b8\uc81c\ud558\uc600\ub2e4. \ud2b9\ud788 \uce74\uc2dc\ubbf8\ub974 \uc9c0\uc5ed\uc740 \uc2dc\ud06c \uc804\uc7c1 \uc9c1\ud6c4\uc778 1850\ub144 \uc554\ub2c8\ucc28\ub974 \uc870\uc57d\uc744 \ud1b5\ud574 \uc7a0\ubb34 \uc9c0\ubc29\uc758 \ub3c4\uadf8\ub9ac \uc655\uc870\uc5d0\uac8c \ud560\uc591\ub418\uc5b4 \uc81c\ud6c4\uad6d\uc774 \ub418\uc5c8\ub2e4. \ud55c\ud3b8 1814\ub144\uc5d0\uc11c 1816\ub144\uae4c\uc9c0 \uc601\uad6d\ub839 \uc778\ub3c4\uc640 \ub124\ud314 \uc0ac\uc774\uc758 \uad6d\uacbd \ubd84\uc7c1\uc73c\ub85c \uc778\ud574 \uc77c\uc5b4\ub09c \uc601\uad6d-\ub124\ud314 \uc804\uc7c1\uc758 \uacb0\uacfc \uad6c\ub974\uce74\uc2a4 \uc9c0\uc5ed\uc774 \uc601\uad6d\uc758 \uc9c0\ubc30\uc5d0 \ub193\uc774\uac8c \ub418\uc5c8\ub2e4. \ub610\ud55c 1854\ub144\uc5d0\ub294 \uc9c0\uae08\uc758 \ub9c8\ud558\ub77c\uc288\ud2b8\ub77c \uc8fc\uc5d0 \uc18d\ud558\ub294 \ubca0\ub77c\ub974 \uc9c0\ubc29\uc774 \uc885\uc18d \ub3d9\ub9f9\uc5d0 \ud3ec\ud568\ub418\uc5c8\uace0, 2\ub144 \ub4a4\uc5d0\ub294 \uc544\uc640\ub4dc\uac00 \uc885\uc18d \ub3d9\ub9f9\uc5d0 \ud3b8\uc785\ub418\uc5c8\ub2e4.", "sentence_answer": "\uc774 \uc815\ucc45\uc5d0 \ub530\ub77c \uc601\uad6d \ub3d9\uc778\ub3c4 \ud68c\uc0ac\ub294 \uc778\ub3c4 \ub0b4\uc758 \uc81c\ud6c4\uad6d\ub4e4\uacfc \uc885\uc18d \ub3d9\ub9f9 \uc744", "paragraph_id": "6532248-0-1", "paragraph_question": "question: \uc81c\ud6c4\uad6d\uc774\ub098 \uc9c0\uc5ed\uad70\ubc8c \uc544\ub798\uc5d0 \uc788\ub358 \uac01 \uc9c0\uc5ed\uc758 \uad70\uc0ac\ub97c \ub3d9\uc778\ub3c4 \ud68c\uc0ac\uac00 \uc9c1\uc811 \ud1b5\uc194\ud558\ub3c4\ub85d \ud558\ub294 \uac83\uc740?, context: 19\uc138\uae30\uc5d0 \ub4e4\uc5b4 \ucd1d\ub3c5 \ub9ac\ucc98\ub4dc \uc6e8\uc2ac\ub9ac\ub294 \ub3d9\uc778\ub3c4 \ud68c\uc0ac\uc758 \uc9c0\ubc30 \uc9c0\uc5ed\uc744 \ud655\uc7a5\ud558\uae30 \uc704\ud55c \uc0c8\ub85c\uc6b4 \uc815\ucc45\uc744 \uc2dc\ud589\ud558\uc600\ub2e4. \uc774 \uc815\ucc45\uc5d0 \ub530\ub77c \uc601\uad6d \ub3d9\uc778\ub3c4 \ud68c\uc0ac\ub294 \uc778\ub3c4 \ub0b4\uc758 \uc81c\ud6c4\uad6d\ub4e4\uacfc \uc885\uc18d \ub3d9\ub9f9\uc744 \uacb0\uc131\ud558\uc600\ub2e4. \uc885\uc18d \ub3d9\ub9f9\uc740 \uc81c\ud6c4\uad6d\uc774\ub098 \uc9c0\uc5ed \uad70\ubc8c\uc758 \ud718\ud558\uc5d0 \uc788\ub358 \uac01 \uc9c0\uc5ed\uc758 \uad70\uc0ac\ub97c \ub3d9\uc778\ub3c4 \ud68c\uc0ac\uac00 \uc9c1\uc811 \ud1b5\uc194\ud558\ub3c4\ub85d \ud558\ub294 \uac83\uc73c\ub85c, \uc81c2\ucc28 \uc601\uad6d-\uc2dc\ud06c \uc804\uc7c1 \uc774\ud6c4 \uce74\uc774\ubca0\ub974\ud30c\ud06c\ud230\ud06c\uc640 \uc8fc\uc640 \uce74\uc288\ubbf8\ub974, \ud380\uc790\ube0c \uc9c0\uc5ed\uc5d0 \uc874\uc7ac\ud558\ub358 \uc81c\ud6c4\uad6d\ub4e4\uc744 \ub300\ud56d\uc73c\ub85c \ud558\uc600\ub294\ub370 \uc774 \uc9c0\uc5ed\uc758 \uc218\uc7a5\uc774\uc5c8\ub358 \ud78c\ub450\uad50 \uc9c0\uc5ed\uc758 \ub9c8\ud558\ub77c\uc790\ub098 \uc774\uc2ac\ub78c\uad50 \uc9c0\uc5ed\uc758 \ub098\uc640\ube0c \ub4f1 \uc81c\ud6c4\ub4e4\uc744 \uc885\uc18d \ub3d9\ub9f9 \uc548\uc73c\ub85c \ud3b8\uc81c\ud558\uc600\ub2e4. \ud2b9\ud788 \uce74\uc2dc\ubbf8\ub974 \uc9c0\uc5ed\uc740 \uc2dc\ud06c \uc804\uc7c1 \uc9c1\ud6c4\uc778 1850\ub144 \uc554\ub2c8\ucc28\ub974 \uc870\uc57d\uc744 \ud1b5\ud574 \uc7a0\ubb34 \uc9c0\ubc29\uc758 \ub3c4\uadf8\ub9ac \uc655\uc870\uc5d0\uac8c \ud560\uc591\ub418\uc5b4 \uc81c\ud6c4\uad6d\uc774 \ub418\uc5c8\ub2e4. \ud55c\ud3b8 1814\ub144\uc5d0\uc11c 1816\ub144\uae4c\uc9c0 \uc601\uad6d\ub839 \uc778\ub3c4\uc640 \ub124\ud314 \uc0ac\uc774\uc758 \uad6d\uacbd \ubd84\uc7c1\uc73c\ub85c \uc778\ud574 \uc77c\uc5b4\ub09c \uc601\uad6d-\ub124\ud314 \uc804\uc7c1\uc758 \uacb0\uacfc \uad6c\ub974\uce74\uc2a4 \uc9c0\uc5ed\uc774 \uc601\uad6d\uc758 \uc9c0\ubc30\uc5d0 \ub193\uc774\uac8c \ub418\uc5c8\ub2e4. \ub610\ud55c 1854\ub144\uc5d0\ub294 \uc9c0\uae08\uc758 \ub9c8\ud558\ub77c\uc288\ud2b8\ub77c \uc8fc\uc5d0 \uc18d\ud558\ub294 \ubca0\ub77c\ub974 \uc9c0\ubc29\uc774 \uc885\uc18d \ub3d9\ub9f9\uc5d0 \ud3ec\ud568\ub418\uc5c8\uace0, 2\ub144 \ub4a4\uc5d0\ub294 \uc544\uc640\ub4dc\uac00 \uc885\uc18d \ub3d9\ub9f9\uc5d0 \ud3b8\uc785\ub418\uc5c8\ub2e4."} {"question": "\ud56d\uc77c\uc6b4\ub3d9\uc744 \uc704\ud55c \uacf5\uc0b0\ub2f9\uacfc\uc758 \ud734\uc804\uacfc \uc81c2\ucc28 \uad6d\uacf5\ud569\uc791\uc5d0 \ud569\uc758\ud55c \uc778\ubb3c\uc740 \ub204\uad6c\uc778\uac00?", "paragraph": "1936\ub144 \uc7a5\uc81c\uc2a4\ub294 \ub9cc\uc8fc\uc0ac\ubcc0\uc774\ud6c4 \uc77c\ubcf8\uad70\uc5d0\uac8c \uadfc\uac70\uc9c0\ub97c \uc783\uace0 \uadfc\ucc98\uc5d0 \uc8fc\ub454\ud55c \uc7a5\uc250\ub7c9\uc758 \uad70\ub300\uc5d0\uac8c \ub3c4\uc8fc\ud574\uc628 \uacf5\uc0b0\uad70\uc758 \ud1a0\ubc8c\uc744 \uba85\ub839\ud588\ub2e4. \ud1a0\ubc8c\uc804\uc744 \ub3c5\uc804\ud558\uae30 \uc704\ud574 \uc7a5\uc81c\uc2a4\ub294 \uc7a5\uc250\ub7c9\uc774 \uc8fc\ub454\ud574 \uc788\ub358 \uc2dc\uc548\uc744 \ubc29\ubb38\ud558\uc600\ub2e4. \uacf5\uc0b0\ub2f9\uc5d0 \ub3d9\uc815\uc801\uc774\uc5c8\ub358 \uc7a5\uc250\ub7c9\uc740 \uc77c\ubcf8\uc81c\uad6d\uc758 \uce68\ub7b5\uc5d0 \ubb34 \ub300\uc751\uc73c\ub85c \uc77c\uad00\ud558\uba74\uc11c\ub3c4 \uacf5\uc0b0\uad70 \ud1a0\ubc8c\ub9cc \uc8fc\uc7a5\ud558\ub294 \uc7a5\uc81c\uc2a4\uc5d0 \ubd88\ub9cc\uc744 \ud488\uace0 \ub3c4\ub9ac\uc5b4 \uc7a5\uc81c\uc2a4\ub97c \uccb4\ud3ec\ud558\ub294 \uc2dc\uc548 \uc0ac\ud0dc\ub97c \uc77c\uc73c\ucf30\ub2e4. \uc7a5\uc81c\uc2a4\ub294 \ud480\ub824\ub098\ub294 \uc870\uac74\uc73c\ub85c \ud56d\uc77c\uc744 \uc704\ud55c \uacf5\uc0b0\ub2f9\uacfc\uc758 \ud734\uc804\uacfc \uc81c2\ucc28 \uad6d\uacf5\ud569\uc791\uc5d0 \ud569\uc758\ud588\ub2e4. \uc774\uc5d0 \ub530\ub77c \ud64d\uad70\uc740 \uad6d\ubbfc\ud601\uba85\uad70 \uc81c8\ub85c\uad70\uacfc \uc2e0\uc0ac\uad70\uc73c\ub85c \uac1c\ud3b8\ub418\uc5b4 \ud615\uc2dd\uc0c1 \uad6d\ubbfc\ub2f9 \uad70\uc5d0 \uc18c\uc18d\ub418\uc5c8\uc9c0\ub9cc, \uc2e4\uc81c\ub85c\ub294 \ub3c5\ub9bd\uc801\uc73c\ub85c \uc874\uc7ac\ud588\ub2e4. \uc774\ud6c4 \uacf5\uc0b0\ub2f9\uc740 \uc77c\ubcf8\uad70\uc774 \uc810\ub839\ud55c \ud654\ubd81\uc9c0\uc5ed\uc744 \uc911\uc2ec\uc73c\ub85c \uc138\ub825\uc744 \ud655\uc7a5\ud558\uc600\ub2e4.", "answer": "\uc7a5\uc81c\uc2a4", "sentence": "1936\ub144 \uc7a5\uc81c\uc2a4 \ub294 \ub9cc\uc8fc\uc0ac\ubcc0\uc774\ud6c4 \uc77c\ubcf8\uad70\uc5d0\uac8c \uadfc\uac70\uc9c0\ub97c \uc783\uace0 \uadfc\ucc98\uc5d0 \uc8fc\ub454\ud55c \uc7a5\uc250\ub7c9\uc758 \uad70\ub300\uc5d0\uac8c \ub3c4\uc8fc\ud574\uc628 \uacf5\uc0b0\uad70\uc758 \ud1a0\ubc8c\uc744 \uba85\ub839\ud588\ub2e4.", "paragraph_sentence": " 1936\ub144 \uc7a5\uc81c\uc2a4 \ub294 \ub9cc\uc8fc\uc0ac\ubcc0\uc774\ud6c4 \uc77c\ubcf8\uad70\uc5d0\uac8c \uadfc\uac70\uc9c0\ub97c \uc783\uace0 \uadfc\ucc98\uc5d0 \uc8fc\ub454\ud55c \uc7a5\uc250\ub7c9\uc758 \uad70\ub300\uc5d0\uac8c \ub3c4\uc8fc\ud574\uc628 \uacf5\uc0b0\uad70\uc758 \ud1a0\ubc8c\uc744 \uba85\ub839\ud588\ub2e4. \ud1a0\ubc8c\uc804\uc744 \ub3c5\uc804\ud558\uae30 \uc704\ud574 \uc7a5\uc81c\uc2a4\ub294 \uc7a5\uc250\ub7c9\uc774 \uc8fc\ub454\ud574 \uc788\ub358 \uc2dc\uc548\uc744 \ubc29\ubb38\ud558\uc600\ub2e4. \uacf5\uc0b0\ub2f9\uc5d0 \ub3d9\uc815\uc801\uc774\uc5c8\ub358 \uc7a5\uc250\ub7c9\uc740 \uc77c\ubcf8\uc81c\uad6d\uc758 \uce68\ub7b5\uc5d0 \ubb34 \ub300\uc751\uc73c\ub85c \uc77c\uad00\ud558\uba74\uc11c\ub3c4 \uacf5\uc0b0\uad70 \ud1a0\ubc8c\ub9cc \uc8fc\uc7a5\ud558\ub294 \uc7a5\uc81c\uc2a4\uc5d0 \ubd88\ub9cc\uc744 \ud488\uace0 \ub3c4\ub9ac\uc5b4 \uc7a5\uc81c\uc2a4\ub97c \uccb4\ud3ec\ud558\ub294 \uc2dc\uc548 \uc0ac\ud0dc\ub97c \uc77c\uc73c\ucf30\ub2e4. \uc7a5\uc81c\uc2a4\ub294 \ud480\ub824\ub098\ub294 \uc870\uac74\uc73c\ub85c \ud56d\uc77c\uc744 \uc704\ud55c \uacf5\uc0b0\ub2f9\uacfc\uc758 \ud734\uc804\uacfc \uc81c2\ucc28 \uad6d\uacf5\ud569\uc791\uc5d0 \ud569\uc758\ud588\ub2e4. \uc774\uc5d0 \ub530\ub77c \ud64d\uad70\uc740 \uad6d\ubbfc\ud601\uba85\uad70 \uc81c8\ub85c\uad70\uacfc \uc2e0\uc0ac\uad70\uc73c\ub85c \uac1c\ud3b8\ub418\uc5b4 \ud615\uc2dd\uc0c1 \uad6d\ubbfc\ub2f9 \uad70\uc5d0 \uc18c\uc18d\ub418\uc5c8\uc9c0\ub9cc, \uc2e4\uc81c\ub85c\ub294 \ub3c5\ub9bd\uc801\uc73c\ub85c \uc874\uc7ac\ud588\ub2e4. \uc774\ud6c4 \uacf5\uc0b0\ub2f9\uc740 \uc77c\ubcf8\uad70\uc774 \uc810\ub839\ud55c \ud654\ubd81\uc9c0\uc5ed\uc744 \uc911\uc2ec\uc73c\ub85c \uc138\ub825\uc744 \ud655\uc7a5\ud558\uc600\ub2e4.", "paragraph_answer": "1936\ub144 \uc7a5\uc81c\uc2a4 \ub294 \ub9cc\uc8fc\uc0ac\ubcc0\uc774\ud6c4 \uc77c\ubcf8\uad70\uc5d0\uac8c \uadfc\uac70\uc9c0\ub97c \uc783\uace0 \uadfc\ucc98\uc5d0 \uc8fc\ub454\ud55c \uc7a5\uc250\ub7c9\uc758 \uad70\ub300\uc5d0\uac8c \ub3c4\uc8fc\ud574\uc628 \uacf5\uc0b0\uad70\uc758 \ud1a0\ubc8c\uc744 \uba85\ub839\ud588\ub2e4. \ud1a0\ubc8c\uc804\uc744 \ub3c5\uc804\ud558\uae30 \uc704\ud574 \uc7a5\uc81c\uc2a4\ub294 \uc7a5\uc250\ub7c9\uc774 \uc8fc\ub454\ud574 \uc788\ub358 \uc2dc\uc548\uc744 \ubc29\ubb38\ud558\uc600\ub2e4. \uacf5\uc0b0\ub2f9\uc5d0 \ub3d9\uc815\uc801\uc774\uc5c8\ub358 \uc7a5\uc250\ub7c9\uc740 \uc77c\ubcf8\uc81c\uad6d\uc758 \uce68\ub7b5\uc5d0 \ubb34 \ub300\uc751\uc73c\ub85c \uc77c\uad00\ud558\uba74\uc11c\ub3c4 \uacf5\uc0b0\uad70 \ud1a0\ubc8c\ub9cc \uc8fc\uc7a5\ud558\ub294 \uc7a5\uc81c\uc2a4\uc5d0 \ubd88\ub9cc\uc744 \ud488\uace0 \ub3c4\ub9ac\uc5b4 \uc7a5\uc81c\uc2a4\ub97c \uccb4\ud3ec\ud558\ub294 \uc2dc\uc548 \uc0ac\ud0dc\ub97c \uc77c\uc73c\ucf30\ub2e4. \uc7a5\uc81c\uc2a4\ub294 \ud480\ub824\ub098\ub294 \uc870\uac74\uc73c\ub85c \ud56d\uc77c\uc744 \uc704\ud55c \uacf5\uc0b0\ub2f9\uacfc\uc758 \ud734\uc804\uacfc \uc81c2\ucc28 \uad6d\uacf5\ud569\uc791\uc5d0 \ud569\uc758\ud588\ub2e4. \uc774\uc5d0 \ub530\ub77c \ud64d\uad70\uc740 \uad6d\ubbfc\ud601\uba85\uad70 \uc81c8\ub85c\uad70\uacfc \uc2e0\uc0ac\uad70\uc73c\ub85c \uac1c\ud3b8\ub418\uc5b4 \ud615\uc2dd\uc0c1 \uad6d\ubbfc\ub2f9 \uad70\uc5d0 \uc18c\uc18d\ub418\uc5c8\uc9c0\ub9cc, \uc2e4\uc81c\ub85c\ub294 \ub3c5\ub9bd\uc801\uc73c\ub85c \uc874\uc7ac\ud588\ub2e4. \uc774\ud6c4 \uacf5\uc0b0\ub2f9\uc740 \uc77c\ubcf8\uad70\uc774 \uc810\ub839\ud55c \ud654\ubd81\uc9c0\uc5ed\uc744 \uc911\uc2ec\uc73c\ub85c \uc138\ub825\uc744 \ud655\uc7a5\ud558\uc600\ub2e4.", "sentence_answer": "1936\ub144 \uc7a5\uc81c\uc2a4 \ub294 \ub9cc\uc8fc\uc0ac\ubcc0\uc774\ud6c4 \uc77c\ubcf8\uad70\uc5d0\uac8c \uadfc\uac70\uc9c0\ub97c \uc783\uace0 \uadfc\ucc98\uc5d0 \uc8fc\ub454\ud55c \uc7a5\uc250\ub7c9\uc758 \uad70\ub300\uc5d0\uac8c \ub3c4\uc8fc\ud574\uc628 \uacf5\uc0b0\uad70\uc758 \ud1a0\ubc8c\uc744 \uba85\ub839\ud588\ub2e4.", "paragraph_id": "6499916-5-1", "paragraph_question": "question: \ud56d\uc77c\uc6b4\ub3d9\uc744 \uc704\ud55c \uacf5\uc0b0\ub2f9\uacfc\uc758 \ud734\uc804\uacfc \uc81c2\ucc28 \uad6d\uacf5\ud569\uc791\uc5d0 \ud569\uc758\ud55c \uc778\ubb3c\uc740 \ub204\uad6c\uc778\uac00?, context: 1936\ub144 \uc7a5\uc81c\uc2a4\ub294 \ub9cc\uc8fc\uc0ac\ubcc0\uc774\ud6c4 \uc77c\ubcf8\uad70\uc5d0\uac8c \uadfc\uac70\uc9c0\ub97c \uc783\uace0 \uadfc\ucc98\uc5d0 \uc8fc\ub454\ud55c \uc7a5\uc250\ub7c9\uc758 \uad70\ub300\uc5d0\uac8c \ub3c4\uc8fc\ud574\uc628 \uacf5\uc0b0\uad70\uc758 \ud1a0\ubc8c\uc744 \uba85\ub839\ud588\ub2e4. \ud1a0\ubc8c\uc804\uc744 \ub3c5\uc804\ud558\uae30 \uc704\ud574 \uc7a5\uc81c\uc2a4\ub294 \uc7a5\uc250\ub7c9\uc774 \uc8fc\ub454\ud574 \uc788\ub358 \uc2dc\uc548\uc744 \ubc29\ubb38\ud558\uc600\ub2e4. \uacf5\uc0b0\ub2f9\uc5d0 \ub3d9\uc815\uc801\uc774\uc5c8\ub358 \uc7a5\uc250\ub7c9\uc740 \uc77c\ubcf8\uc81c\uad6d\uc758 \uce68\ub7b5\uc5d0 \ubb34 \ub300\uc751\uc73c\ub85c \uc77c\uad00\ud558\uba74\uc11c\ub3c4 \uacf5\uc0b0\uad70 \ud1a0\ubc8c\ub9cc \uc8fc\uc7a5\ud558\ub294 \uc7a5\uc81c\uc2a4\uc5d0 \ubd88\ub9cc\uc744 \ud488\uace0 \ub3c4\ub9ac\uc5b4 \uc7a5\uc81c\uc2a4\ub97c \uccb4\ud3ec\ud558\ub294 \uc2dc\uc548 \uc0ac\ud0dc\ub97c \uc77c\uc73c\ucf30\ub2e4. \uc7a5\uc81c\uc2a4\ub294 \ud480\ub824\ub098\ub294 \uc870\uac74\uc73c\ub85c \ud56d\uc77c\uc744 \uc704\ud55c \uacf5\uc0b0\ub2f9\uacfc\uc758 \ud734\uc804\uacfc \uc81c2\ucc28 \uad6d\uacf5\ud569\uc791\uc5d0 \ud569\uc758\ud588\ub2e4. \uc774\uc5d0 \ub530\ub77c \ud64d\uad70\uc740 \uad6d\ubbfc\ud601\uba85\uad70 \uc81c8\ub85c\uad70\uacfc \uc2e0\uc0ac\uad70\uc73c\ub85c \uac1c\ud3b8\ub418\uc5b4 \ud615\uc2dd\uc0c1 \uad6d\ubbfc\ub2f9 \uad70\uc5d0 \uc18c\uc18d\ub418\uc5c8\uc9c0\ub9cc, \uc2e4\uc81c\ub85c\ub294 \ub3c5\ub9bd\uc801\uc73c\ub85c \uc874\uc7ac\ud588\ub2e4. \uc774\ud6c4 \uacf5\uc0b0\ub2f9\uc740 \uc77c\ubcf8\uad70\uc774 \uc810\ub839\ud55c \ud654\ubd81\uc9c0\uc5ed\uc744 \uc911\uc2ec\uc73c\ub85c \uc138\ub825\uc744 \ud655\uc7a5\ud558\uc600\ub2e4."} {"question": "\uce7c\ub9ac\uc2a4\ud1a0\ub294 \ubb34\uc5c7\uc73c\ub85c \uad6c\uc131\ub418\uc5c8\uae30 \ub54c\ubb38\uc5d0 \ubbf8\uc57d\ud55c \ub300\uae30\uad8c\uc744 \uac16\uace0 \uc788\ub294\uac00?", "paragraph": "\uce7c\ub9ac\uc2a4\ud1a0\ub294 \uc774\uc0b0\ud654 \ud0c4\uc18c\ub85c \uad6c\uc131\ub41c \ub9e4\uc6b0 \ubbf8\uc57d\ud55c \ub300\uae30\uad8c\uc744 \uac00\uc9c0\uace0 \uc788\ub2e4. \uc774\ub294 \uac08\ub9b4\ub808\uc624 \ud0d0\uc0ac\uc120\uc758 \uadfc\uc801\uc678\uc120 \ubd84\uad11 \uc9c0\ub3c4\uc791\uc131\uae30(Near Infrared Mapping Spectrometer, NIMS)\uac00 4.2 \u00b5m \ub300\uc5ed\uc5d0\uc11c \ub300\uae30\uad8c\uc774 \ub9cc\ub4e4\uc5b4 \ub0b8 \ud30c\uc7a5 \ud761\uc218\ub97c \uac10\uc9c0\ud558\uc5ec \ubc1c\uacac\ub41c \uac83\uc774\ub2e4. \uce7c\ub9ac\uc2a4\ud1a0 \ud45c\uba74\uc5d0\uc11c\uc758 \uae30\uc555\uc740 \uc57d 7.5\u00d710^ bar (0.75 \u00b5Pa) \ub85c \ucd94\uc815\ub418\uace0, \uc785\uc790\ub4e4\uc758 \ubc00\ub3c4\ub294 4\u00d710^ cm \uc774\ub2e4. \uc774 \uc815\ub3c4\ub85c \uc605\uc740 \ub300\uae30\uad8c\uc740 4\uc77c \uc774\ub0b4\ub85c \ubaa8\ub450 \uce7c\ub9ac\uc2a4\ud1a0\ub97c \ud0c8\ucd9c\ud574 \ubc84\ub9b4 \uac83\uc774\uae30 \ub54c\ubb38\uc5d0, \ub300\uae30\uad8c\uc740 \uce7c\ub9ac\uc2a4\ud1a0\uc758 \uc5bc\uc74c \uc9c0\uac01\uc5d0\uc11c \ucc9c\ucc9c\ud788 \uc2b9\ud654\ub418\ub294 \uc774\uc0b0\ud654 \ud0c4\uc18c\uc5d0 \uc758\ud574 \uc9c0\uc18d\uc801\uc73c\ub85c \ubcf4\ucda9\ub418\uace0 \uc788\ub2e4\uace0 \uc5ec\uaca8\uc9c4\ub2e4. \uc774 \uc774\ub860\uc740 \ud45c\uba74\uc758 \ud639\ub4e4\uc744 \uc124\uba85\ud558\ub294 \uc2b9\ud654-\ubd84\ud574 \uc774\ub860\uacfc \uc5b4\ub290 \uc815\ub3c4 \uc77c\uce58\ud55c\ub2e4.", "answer": "\uc774\uc0b0\ud654 \ud0c4\uc18c", "sentence": "\uce7c\ub9ac\uc2a4\ud1a0\ub294 \uc774\uc0b0\ud654 \ud0c4\uc18c \ub85c \uad6c\uc131\ub41c \ub9e4\uc6b0 \ubbf8\uc57d\ud55c \ub300\uae30\uad8c\uc744 \uac00\uc9c0\uace0 \uc788\ub2e4.", "paragraph_sentence": " \uce7c\ub9ac\uc2a4\ud1a0\ub294 \uc774\uc0b0\ud654 \ud0c4\uc18c \ub85c \uad6c\uc131\ub41c \ub9e4\uc6b0 \ubbf8\uc57d\ud55c \ub300\uae30\uad8c\uc744 \uac00\uc9c0\uace0 \uc788\ub2e4. \uc774\ub294 \uac08\ub9b4\ub808\uc624 \ud0d0\uc0ac\uc120\uc758 \uadfc\uc801\uc678\uc120 \ubd84\uad11 \uc9c0\ub3c4\uc791\uc131\uae30(Near Infrared Mapping Spectrometer, NIMS)\uac00 4.2 \u00b5m \ub300\uc5ed\uc5d0\uc11c \ub300\uae30\uad8c\uc774 \ub9cc\ub4e4\uc5b4 \ub0b8 \ud30c\uc7a5 \ud761\uc218\ub97c \uac10\uc9c0\ud558\uc5ec \ubc1c\uacac\ub41c \uac83\uc774\ub2e4. \uce7c\ub9ac\uc2a4\ud1a0 \ud45c\uba74\uc5d0\uc11c\uc758 \uae30\uc555\uc740 \uc57d 7.5\u00d710^ bar (0.75 \u00b5Pa) \ub85c \ucd94\uc815\ub418\uace0, \uc785\uc790\ub4e4\uc758 \ubc00\ub3c4\ub294 4\u00d710^ cm \uc774\ub2e4. \uc774 \uc815\ub3c4\ub85c \uc605\uc740 \ub300\uae30\uad8c\uc740 4\uc77c \uc774\ub0b4\ub85c \ubaa8\ub450 \uce7c\ub9ac\uc2a4\ud1a0\ub97c \ud0c8\ucd9c\ud574 \ubc84\ub9b4 \uac83\uc774\uae30 \ub54c\ubb38\uc5d0, \ub300\uae30\uad8c\uc740 \uce7c\ub9ac\uc2a4\ud1a0\uc758 \uc5bc\uc74c \uc9c0\uac01\uc5d0\uc11c \ucc9c\ucc9c\ud788 \uc2b9\ud654\ub418\ub294 \uc774\uc0b0\ud654 \ud0c4\uc18c\uc5d0 \uc758\ud574 \uc9c0\uc18d\uc801\uc73c\ub85c \ubcf4\ucda9\ub418\uace0 \uc788\ub2e4\uace0 \uc5ec\uaca8\uc9c4\ub2e4. \uc774 \uc774\ub860\uc740 \ud45c\uba74\uc758 \ud639\ub4e4\uc744 \uc124\uba85\ud558\ub294 \uc2b9\ud654-\ubd84\ud574 \uc774\ub860\uacfc \uc5b4\ub290 \uc815\ub3c4 \uc77c\uce58\ud55c\ub2e4.", "paragraph_answer": "\uce7c\ub9ac\uc2a4\ud1a0\ub294 \uc774\uc0b0\ud654 \ud0c4\uc18c \ub85c \uad6c\uc131\ub41c \ub9e4\uc6b0 \ubbf8\uc57d\ud55c \ub300\uae30\uad8c\uc744 \uac00\uc9c0\uace0 \uc788\ub2e4. \uc774\ub294 \uac08\ub9b4\ub808\uc624 \ud0d0\uc0ac\uc120\uc758 \uadfc\uc801\uc678\uc120 \ubd84\uad11 \uc9c0\ub3c4\uc791\uc131\uae30(Near Infrared Mapping Spectrometer, NIMS)\uac00 4.2 \u00b5m \ub300\uc5ed\uc5d0\uc11c \ub300\uae30\uad8c\uc774 \ub9cc\ub4e4\uc5b4 \ub0b8 \ud30c\uc7a5 \ud761\uc218\ub97c \uac10\uc9c0\ud558\uc5ec \ubc1c\uacac\ub41c \uac83\uc774\ub2e4. \uce7c\ub9ac\uc2a4\ud1a0 \ud45c\uba74\uc5d0\uc11c\uc758 \uae30\uc555\uc740 \uc57d 7.5\u00d710^ bar (0.75 \u00b5Pa) \ub85c \ucd94\uc815\ub418\uace0, \uc785\uc790\ub4e4\uc758 \ubc00\ub3c4\ub294 4\u00d710^ cm \uc774\ub2e4. \uc774 \uc815\ub3c4\ub85c \uc605\uc740 \ub300\uae30\uad8c\uc740 4\uc77c \uc774\ub0b4\ub85c \ubaa8\ub450 \uce7c\ub9ac\uc2a4\ud1a0\ub97c \ud0c8\ucd9c\ud574 \ubc84\ub9b4 \uac83\uc774\uae30 \ub54c\ubb38\uc5d0, \ub300\uae30\uad8c\uc740 \uce7c\ub9ac\uc2a4\ud1a0\uc758 \uc5bc\uc74c \uc9c0\uac01\uc5d0\uc11c \ucc9c\ucc9c\ud788 \uc2b9\ud654\ub418\ub294 \uc774\uc0b0\ud654 \ud0c4\uc18c\uc5d0 \uc758\ud574 \uc9c0\uc18d\uc801\uc73c\ub85c \ubcf4\ucda9\ub418\uace0 \uc788\ub2e4\uace0 \uc5ec\uaca8\uc9c4\ub2e4. \uc774 \uc774\ub860\uc740 \ud45c\uba74\uc758 \ud639\ub4e4\uc744 \uc124\uba85\ud558\ub294 \uc2b9\ud654-\ubd84\ud574 \uc774\ub860\uacfc \uc5b4\ub290 \uc815\ub3c4 \uc77c\uce58\ud55c\ub2e4.", "sentence_answer": "\uce7c\ub9ac\uc2a4\ud1a0\ub294 \uc774\uc0b0\ud654 \ud0c4\uc18c \ub85c \uad6c\uc131\ub41c \ub9e4\uc6b0 \ubbf8\uc57d\ud55c \ub300\uae30\uad8c\uc744 \uac00\uc9c0\uace0 \uc788\ub2e4.", "paragraph_id": "6561311-8-0", "paragraph_question": "question: \uce7c\ub9ac\uc2a4\ud1a0\ub294 \ubb34\uc5c7\uc73c\ub85c \uad6c\uc131\ub418\uc5c8\uae30 \ub54c\ubb38\uc5d0 \ubbf8\uc57d\ud55c \ub300\uae30\uad8c\uc744 \uac16\uace0 \uc788\ub294\uac00?, context: \uce7c\ub9ac\uc2a4\ud1a0\ub294 \uc774\uc0b0\ud654 \ud0c4\uc18c\ub85c \uad6c\uc131\ub41c \ub9e4\uc6b0 \ubbf8\uc57d\ud55c \ub300\uae30\uad8c\uc744 \uac00\uc9c0\uace0 \uc788\ub2e4. \uc774\ub294 \uac08\ub9b4\ub808\uc624 \ud0d0\uc0ac\uc120\uc758 \uadfc\uc801\uc678\uc120 \ubd84\uad11 \uc9c0\ub3c4\uc791\uc131\uae30(Near Infrared Mapping Spectrometer, NIMS)\uac00 4.2 \u00b5m \ub300\uc5ed\uc5d0\uc11c \ub300\uae30\uad8c\uc774 \ub9cc\ub4e4\uc5b4 \ub0b8 \ud30c\uc7a5 \ud761\uc218\ub97c \uac10\uc9c0\ud558\uc5ec \ubc1c\uacac\ub41c \uac83\uc774\ub2e4. \uce7c\ub9ac\uc2a4\ud1a0 \ud45c\uba74\uc5d0\uc11c\uc758 \uae30\uc555\uc740 \uc57d 7.5\u00d710^ bar (0.75 \u00b5Pa) \ub85c \ucd94\uc815\ub418\uace0, \uc785\uc790\ub4e4\uc758 \ubc00\ub3c4\ub294 4\u00d710^ cm \uc774\ub2e4. \uc774 \uc815\ub3c4\ub85c \uc605\uc740 \ub300\uae30\uad8c\uc740 4\uc77c \uc774\ub0b4\ub85c \ubaa8\ub450 \uce7c\ub9ac\uc2a4\ud1a0\ub97c \ud0c8\ucd9c\ud574 \ubc84\ub9b4 \uac83\uc774\uae30 \ub54c\ubb38\uc5d0, \ub300\uae30\uad8c\uc740 \uce7c\ub9ac\uc2a4\ud1a0\uc758 \uc5bc\uc74c \uc9c0\uac01\uc5d0\uc11c \ucc9c\ucc9c\ud788 \uc2b9\ud654\ub418\ub294 \uc774\uc0b0\ud654 \ud0c4\uc18c\uc5d0 \uc758\ud574 \uc9c0\uc18d\uc801\uc73c\ub85c \ubcf4\ucda9\ub418\uace0 \uc788\ub2e4\uace0 \uc5ec\uaca8\uc9c4\ub2e4. \uc774 \uc774\ub860\uc740 \ud45c\uba74\uc758 \ud639\ub4e4\uc744 \uc124\uba85\ud558\ub294 \uc2b9\ud654-\ubd84\ud574 \uc774\ub860\uacfc \uc5b4\ub290 \uc815\ub3c4 \uc77c\uce58\ud55c\ub2e4."} {"question": "\uc9c0\uae09\uc704\ud0c1\uc758 \ucde8\uc18c \uc81c\ub3c4\uac00 \uc0dd\ub7b5\ud558\ub294 \uc808\ucc28\ub294?", "paragraph": "\uc9c0\uae09\uc704\ud0c1\uc758 \ucde8\uc18c\ub294 \uc218\ud45c\uc758 \ub3c4\ub09c\u00b7\uc0c1\uc2e4\u00b7\ubcc0\uc870 \ub4f1\uc758 \uacbd\uc6b0\uc5d0 \ubc88\uc7a1\ud55c \uacf5\uc2dc\ucd5c\uace0\uc808\ucc28\ub97c \uc0dd\ub7b5\ud558\uae30 \uc704\ud558\uc5ec \uc778\uc815\ub41c \ud3b8\ub9ac\ud55c \uc81c\ub3c4\uc774\ub2e4. \uc5ec\uae30\uc5d0\uc11c \uc9c0\uae09\uc704\ud0c1\uc758 \ucde8\uc18c\ub77c \ud568\uc740 \uc218\ud45c\uacc4\uc57d\uc758 \ucde8\uc18c\ub97c \ub9d0\ud558\ub294 \uac83\uc774 \uc544\ub2c8\uace0 \uac1c\uac1c\uc758 \uc218\ud45c\ubc1c\ud589\ud589\uc704\uc778 \uc9c0\uae09\uc9c0\uc2dc\ub97c \ucca0\ud68c\ud558\ub294 \uac83\uc744 \ub9d0\ud55c\ub2e4. \ubc1c\ud589\uc778\uc740 \uc9c0\uae09\uc778\uacfc \uc218\ud45c\uacc4\uc57d\uc5d0 \ub530\ub77c \uc218\ud45c\ub97c \ubc1c\ud589\ud558\uace0 \uc9c0\uae09\uc778\uc740 \uadf8 \uc704\ud0c1\uc5d0 \ub530\ub77c \ubc1c\ud589\uc778\uc758 \uc790\uae08\uc5d0\uc11c \uc218\ud45c\uae08\uc561\uc744 \uc9c0\uae09\ud55c\ub2e4. \uc774 \uacbd\uc6b0\uc5d0 \uc218\ud45c\uc758 \ubc1c\ud589\ud589\uc704\ub294 \uc9c0\uae09\uc758 \uc9c0\uc2dc\uc774\uba70 \uc9c0\uae09\uc9c0\uc2dc\ub294 \uc9c0\uc2dc\uc778\uc778 \uc218\ud45c\ubc1c\ud589\uc778\uc774 \uc784\uc758\ub85c \ucca0\ud68c\ud568\uc73c\ub85c\uc368 \uadf8 \uc9c0\uae09\uc744 \uc911\uc9c0\uc2dc\ud0ac \uc218 \uc788\ub2e4. \ub530\ub77c\uc11c \ucde8\uc18c \ud6c4\uc5d0\ub294 \uc9c0\uae09\uc778\uc774 \uc774 \uc218\ud45c\uc5d0 \ub300\ud558\uc5ec \uc9c0\uae09\uc744 \ud558\uc5ec\ub3c4 \ubc1c\ud589\uc778\uc5d0\uac8c \uadf8 \uacc4\uc0b0\uc744 \ub3cc\ub9ac\uc9c0 \ubabb\ud558\uac8c \ub41c\ub2e4. \uadf8\ub7ec\ub098 \ud55c\ud3b8 \ubc1c\ud589\uc778\uc774 \uc9c0\uae09\uc704\ud0c1\uc744 \uc5b8\uc81c\ub4e0\uc9c0 \ucde8\uc18c\ud560 \uc218 \uc788\ub2e4\uace0 \ud558\uba74 \uc218\ud45c\uc18c\uc9c0\uc778\uc758 \uc9c0\uc704\ub97c \ubd88\uc548\ud558\uac8c \ud558\uc5ec \uc624\ud788\ub824 \uac70\ub798\uc758 \uc548\uc804\uc744 \ud574\uce58\uac8c \ub41c\ub2e4. \uadf8\ub7ec\ubbc0\ub85c \ubc95\uc740 \ub610\ud55c \uc9c0\uae09\uc704\ud0c1\uc758 \ucde8\uc18c\ub97c \uc81c\ud55c\ud55c\ub2e4. \uc774\uc640 \uac19\uc774 \uc9c0\uae09\uc704\ud0c1\uc758 \ucde8\uc18c\ub294 \ubc1c\ud589\uc778\uc5d0\uac8c\ub294 \uc720\ub9ac\ud558\uc9c0\ub9cc \uc18c\uc9c0\uc778\uc5d0\uac8c\ub294 \ubd88\ub9ac\ud55c \uc81c\ub3c4\uc774\ubbc0\ub85c \uadf8 \ucde8\uc18c\ub97c \ubb34\uc81c\ud55c\uc73c\ub85c \ud5c8\uc6a9\ud558\uba74 \uc218\ud45c\uc758 \uc720\ud1b5\uc744 \ud574\uce58\uac8c \ub41c\ub2e4. \ub530\ub77c\uc11c \ucde8\uc18c\ub294 \uc81c\uc2dc\uae30\uac04 \uacbd\uacfc \ud6c4\uc5d0\ub9cc \ud6a8\ub825\uc744 \ubc1c\uc0dd\ud558\ub3c4\ub85d \uc81c\ud55c\ud558\uc5ec \uc9c0\uae09\uc778\uc740 \uc81c\uc2dc\uae30\uac04 \uc548\uc5d0\ub294 \ubc1c\ud589\uc778\uc774 \ucde8\uc18c\ub97c \ud558\uc5ec\ub3c4 \uc9c0\uae09\uc744 \ud560 \uc218 \uc788\uc73c\uba70, \uadf8 \uacc4\uc0b0\uc744 \ubc1c\ud589\uc778\uc5d0\uac8c \uadc0\uc18d\uc2dc\ud0ac \uc218 \uc788\ub3c4\ub85d \ud558\uc600\ub2e4(\uc218\ud45c 32\uc870 1\ud56d). \uc9c0\uae09\uc778\uc740 \uc218\ud45c\uc0c1\uc758 \uc758\ubb34\uc790\uac00 \uc544\ub2c8\uace0 \uc9c0\uae09\uc5ec\ubd80\ub294 \uadf8\uc758 \uc784\uc758\uc774\ubbc0\ub85c \ubc1c\ud589\uc778\uc758 \uc81c\uc2dc\uae30\uac04 \ub0b4\uc758 \ubb34\ud6a8\uc778 \uc704\ud0c1\ucde8\uc18c\uc5d0 \ub530\ub77c\uc11c \uc18c\uc9c0\uc778\uc5d0 \ub300\ud55c \uc218\ud45c\uc9c0\uae09\uc744 \uac70\uc808\ud55c \uacbd\uc6b0\uc5d0\ub3c4 \uc18c\uc9c0\uc778\uc740 \uc9c0\uae09\uc778\uc5d0\uac8c \uc218\ud45c\uc758 \uc9c0\uae09\uc744 \uac15\uc694\ud560 \uc218 \uc5c6\uc73c\uba70 \uacb0\uad6d \uc804\uc790\uc5d0 \ub300\ud558\uc5ec \uc0c1\ud658 \uccad\uad6c\ud558\ub294 \uc218\ubc16\uc5d0 \uc5c6\ub2e4. \ub530\ub77c\uc11c \uc218\ud45c\uc758 \ub3c4\ub09c\u00b7\uc720\uc2e4(\u907a\u5931)\uc758 \uacbd\uc6b0\uc5d0\ub3c4 \uadf8 \ubc94\uc704 \ub0b4\uc5d0\uc11c\ub294 \ubc1c\ud589\uc778\uc740 \ubcf4\ud638\ub418\ub294 \uc148\uc774\ub2e4. \uc774\uc5d0 \ubc18\ud558\uc5ec \ubc1c\ud589\uc778\uc740 \uc81c\uc2dc\uae30\uac04 \uacbd\uacfc \ud6c4\uc5d0\ub294 \uc784\uc758\ub85c \uc9c0\uae09\uc704\ud0c1\uc758 \ucde8\uc18c\ub97c \ud560 \uc218 \uc788\ub2e4. \uadf8\ub7ec\ub098 \uc81c\uc2dc\uae30\uac04\uc774 \uacbd\uacfc\ud568\uc73c\ub85c\uc368 \ub2f9\uc5f0\ud788 \uc9c0\uae09\uc704\ud0c1\uc774 \uc18c\uba78\ud558\ub294 \uac83\uc774 \uc544\ub2c8\ubbc0\ub85c \ubc1c\ud589\uc778\uc5d0 \uc758\ud55c \ucde8\uc18c\uac00 \uc5c6\ub294 \ud55c \uc9c0\uae09\uc778\uc740 \ubc1c\ud589\uc778\uc758 \uacc4\uc0b0\uc73c\ub85c \uc720\ud6a8\ud55c \uc9c0\uae09\uc744 \ud560 \uc218 \uc788\ub2e4(\uc218\ud45c 32\uc870 2\ud56d). \uc9c0\uae09\uc704\ud0c1\uc758 \ucde8\uc18c\uc758 \ud1b5\uc9c0\ubc29\ubc95\uc5d0\ub294 \uc81c\ud55c\uc774 \uc5c6\ub2e4. \uc11c\uba74\uc774\ub098 \uad6c\uc220\ub85c\ub3c4 \ud560 \uc218 \uc788\uc73c\uba70 \ucde8\uc18c\ub294 \uadf8 \ud1b5\uc9c0\uac00 \uc9c0\uae09\uc778\uc5d0\uac8c \ub3c4\ub2ec\ud55c \ub54c\uc5d0 \ud6a8\ub825\uc744 \ubc1c\uc0dd\ud55c\ub2e4. \uc704", "answer": "\uacf5\uc2dc\ucd5c\uace0\uc808\ucc28", "sentence": "\uc9c0\uae09\uc704\ud0c1\uc758 \ucde8\uc18c\ub294 \uc218\ud45c\uc758 \ub3c4\ub09c\u00b7\uc0c1\uc2e4\u00b7\ubcc0\uc870 \ub4f1\uc758 \uacbd\uc6b0\uc5d0 \ubc88\uc7a1\ud55c \uacf5\uc2dc\ucd5c\uace0\uc808\ucc28 \ub97c \uc0dd\ub7b5\ud558\uae30 \uc704\ud558\uc5ec \uc778\uc815\ub41c \ud3b8\ub9ac\ud55c \uc81c\ub3c4\uc774\ub2e4.", "paragraph_sentence": " \uc9c0\uae09\uc704\ud0c1\uc758 \ucde8\uc18c\ub294 \uc218\ud45c\uc758 \ub3c4\ub09c\u00b7\uc0c1\uc2e4\u00b7\ubcc0\uc870 \ub4f1\uc758 \uacbd\uc6b0\uc5d0 \ubc88\uc7a1\ud55c \uacf5\uc2dc\ucd5c\uace0\uc808\ucc28 \ub97c \uc0dd\ub7b5\ud558\uae30 \uc704\ud558\uc5ec \uc778\uc815\ub41c \ud3b8\ub9ac\ud55c \uc81c\ub3c4\uc774\ub2e4. \uc5ec\uae30\uc5d0\uc11c \uc9c0\uae09\uc704\ud0c1\uc758 \ucde8\uc18c\ub77c \ud568\uc740 \uc218\ud45c\uacc4\uc57d\uc758 \ucde8\uc18c\ub97c \ub9d0\ud558\ub294 \uac83\uc774 \uc544\ub2c8\uace0 \uac1c\uac1c\uc758 \uc218\ud45c\ubc1c\ud589\ud589\uc704\uc778 \uc9c0\uae09\uc9c0\uc2dc\ub97c \ucca0\ud68c\ud558\ub294 \uac83\uc744 \ub9d0\ud55c\ub2e4. \ubc1c\ud589\uc778\uc740 \uc9c0\uae09\uc778\uacfc \uc218\ud45c\uacc4\uc57d\uc5d0 \ub530\ub77c \uc218\ud45c\ub97c \ubc1c\ud589\ud558\uace0 \uc9c0\uae09\uc778\uc740 \uadf8 \uc704\ud0c1\uc5d0 \ub530\ub77c \ubc1c\ud589\uc778\uc758 \uc790\uae08\uc5d0\uc11c \uc218\ud45c\uae08\uc561\uc744 \uc9c0\uae09\ud55c\ub2e4. \uc774 \uacbd\uc6b0\uc5d0 \uc218\ud45c\uc758 \ubc1c\ud589\ud589\uc704\ub294 \uc9c0\uae09\uc758 \uc9c0\uc2dc\uc774\uba70 \uc9c0\uae09\uc9c0\uc2dc\ub294 \uc9c0\uc2dc\uc778\uc778 \uc218\ud45c\ubc1c\ud589\uc778\uc774 \uc784\uc758\ub85c \ucca0\ud68c\ud568\uc73c\ub85c\uc368 \uadf8 \uc9c0\uae09\uc744 \uc911\uc9c0\uc2dc\ud0ac \uc218 \uc788\ub2e4. \ub530\ub77c\uc11c \ucde8\uc18c \ud6c4\uc5d0\ub294 \uc9c0\uae09\uc778\uc774 \uc774 \uc218\ud45c\uc5d0 \ub300\ud558\uc5ec \uc9c0\uae09\uc744 \ud558\uc5ec\ub3c4 \ubc1c\ud589\uc778\uc5d0\uac8c \uadf8 \uacc4\uc0b0\uc744 \ub3cc\ub9ac\uc9c0 \ubabb\ud558\uac8c \ub41c\ub2e4. \uadf8\ub7ec\ub098 \ud55c\ud3b8 \ubc1c\ud589\uc778\uc774 \uc9c0\uae09\uc704\ud0c1\uc744 \uc5b8\uc81c\ub4e0\uc9c0 \ucde8\uc18c\ud560 \uc218 \uc788\ub2e4\uace0 \ud558\uba74 \uc218\ud45c\uc18c\uc9c0\uc778\uc758 \uc9c0\uc704\ub97c \ubd88\uc548\ud558\uac8c \ud558\uc5ec \uc624\ud788\ub824 \uac70\ub798\uc758 \uc548\uc804\uc744 \ud574\uce58\uac8c \ub41c\ub2e4. \uadf8\ub7ec\ubbc0\ub85c \ubc95\uc740 \ub610\ud55c \uc9c0\uae09\uc704\ud0c1\uc758 \ucde8\uc18c\ub97c \uc81c\ud55c\ud55c\ub2e4. \uc774\uc640 \uac19\uc774 \uc9c0\uae09\uc704\ud0c1\uc758 \ucde8\uc18c\ub294 \ubc1c\ud589\uc778\uc5d0\uac8c\ub294 \uc720\ub9ac\ud558\uc9c0\ub9cc \uc18c\uc9c0\uc778\uc5d0\uac8c\ub294 \ubd88\ub9ac\ud55c \uc81c\ub3c4\uc774\ubbc0\ub85c \uadf8 \ucde8\uc18c\ub97c \ubb34\uc81c\ud55c\uc73c\ub85c \ud5c8\uc6a9\ud558\uba74 \uc218\ud45c\uc758 \uc720\ud1b5\uc744 \ud574\uce58\uac8c \ub41c\ub2e4. \ub530\ub77c\uc11c \ucde8\uc18c\ub294 \uc81c\uc2dc\uae30\uac04 \uacbd\uacfc \ud6c4\uc5d0\ub9cc \ud6a8\ub825\uc744 \ubc1c\uc0dd\ud558\ub3c4\ub85d \uc81c\ud55c\ud558\uc5ec \uc9c0\uae09\uc778\uc740 \uc81c\uc2dc\uae30\uac04 \uc548\uc5d0\ub294 \ubc1c\ud589\uc778\uc774 \ucde8\uc18c\ub97c \ud558\uc5ec\ub3c4 \uc9c0\uae09\uc744 \ud560 \uc218 \uc788\uc73c\uba70, \uadf8 \uacc4\uc0b0\uc744 \ubc1c\ud589\uc778\uc5d0\uac8c \uadc0\uc18d\uc2dc\ud0ac \uc218 \uc788\ub3c4\ub85d \ud558\uc600\ub2e4(\uc218\ud45c 32\uc870 1\ud56d). \uc9c0\uae09\uc778\uc740 \uc218\ud45c\uc0c1\uc758 \uc758\ubb34\uc790\uac00 \uc544\ub2c8\uace0 \uc9c0\uae09\uc5ec\ubd80\ub294 \uadf8\uc758 \uc784\uc758\uc774\ubbc0\ub85c \ubc1c\ud589\uc778\uc758 \uc81c\uc2dc\uae30\uac04 \ub0b4\uc758 \ubb34\ud6a8\uc778 \uc704\ud0c1\ucde8\uc18c\uc5d0 \ub530\ub77c\uc11c \uc18c\uc9c0\uc778\uc5d0 \ub300\ud55c \uc218\ud45c\uc9c0\uae09\uc744 \uac70\uc808\ud55c \uacbd\uc6b0\uc5d0\ub3c4 \uc18c\uc9c0\uc778\uc740 \uc9c0\uae09\uc778\uc5d0\uac8c \uc218\ud45c\uc758 \uc9c0\uae09\uc744 \uac15\uc694\ud560 \uc218 \uc5c6\uc73c\uba70 \uacb0\uad6d \uc804\uc790\uc5d0 \ub300\ud558\uc5ec \uc0c1\ud658 \uccad\uad6c\ud558\ub294 \uc218\ubc16\uc5d0 \uc5c6\ub2e4. \ub530\ub77c\uc11c \uc218\ud45c\uc758 \ub3c4\ub09c\u00b7\uc720\uc2e4(\u907a\u5931)\uc758 \uacbd\uc6b0\uc5d0\ub3c4 \uadf8 \ubc94\uc704 \ub0b4\uc5d0\uc11c\ub294 \ubc1c\ud589\uc778\uc740 \ubcf4\ud638\ub418\ub294 \uc148\uc774\ub2e4. \uc774\uc5d0 \ubc18\ud558\uc5ec \ubc1c\ud589\uc778\uc740 \uc81c\uc2dc\uae30\uac04 \uacbd\uacfc \ud6c4\uc5d0\ub294 \uc784\uc758\ub85c \uc9c0\uae09\uc704\ud0c1\uc758 \ucde8\uc18c\ub97c \ud560 \uc218 \uc788\ub2e4. \uadf8\ub7ec\ub098 \uc81c\uc2dc\uae30\uac04\uc774 \uacbd\uacfc\ud568\uc73c\ub85c\uc368 \ub2f9\uc5f0\ud788 \uc9c0\uae09\uc704\ud0c1\uc774 \uc18c\uba78\ud558\ub294 \uac83\uc774 \uc544\ub2c8\ubbc0\ub85c \ubc1c\ud589\uc778\uc5d0 \uc758\ud55c \ucde8\uc18c\uac00 \uc5c6\ub294 \ud55c \uc9c0\uae09\uc778\uc740 \ubc1c\ud589\uc778\uc758 \uacc4\uc0b0\uc73c\ub85c \uc720\ud6a8\ud55c \uc9c0\uae09\uc744 \ud560 \uc218 \uc788\ub2e4(\uc218\ud45c 32\uc870 2\ud56d). \uc9c0\uae09\uc704\ud0c1\uc758 \ucde8\uc18c\uc758 \ud1b5\uc9c0\ubc29\ubc95\uc5d0\ub294 \uc81c\ud55c\uc774 \uc5c6\ub2e4. \uc11c\uba74\uc774\ub098 \uad6c\uc220\ub85c\ub3c4 \ud560 \uc218 \uc788\uc73c\uba70 \ucde8\uc18c\ub294 \uadf8 \ud1b5\uc9c0\uac00 \uc9c0\uae09\uc778\uc5d0\uac8c \ub3c4\ub2ec\ud55c \ub54c\uc5d0 \ud6a8\ub825\uc744 \ubc1c\uc0dd\ud55c\ub2e4. \uc704", "paragraph_answer": "\uc9c0\uae09\uc704\ud0c1\uc758 \ucde8\uc18c\ub294 \uc218\ud45c\uc758 \ub3c4\ub09c\u00b7\uc0c1\uc2e4\u00b7\ubcc0\uc870 \ub4f1\uc758 \uacbd\uc6b0\uc5d0 \ubc88\uc7a1\ud55c \uacf5\uc2dc\ucd5c\uace0\uc808\ucc28 \ub97c \uc0dd\ub7b5\ud558\uae30 \uc704\ud558\uc5ec \uc778\uc815\ub41c \ud3b8\ub9ac\ud55c \uc81c\ub3c4\uc774\ub2e4. \uc5ec\uae30\uc5d0\uc11c \uc9c0\uae09\uc704\ud0c1\uc758 \ucde8\uc18c\ub77c \ud568\uc740 \uc218\ud45c\uacc4\uc57d\uc758 \ucde8\uc18c\ub97c \ub9d0\ud558\ub294 \uac83\uc774 \uc544\ub2c8\uace0 \uac1c\uac1c\uc758 \uc218\ud45c\ubc1c\ud589\ud589\uc704\uc778 \uc9c0\uae09\uc9c0\uc2dc\ub97c \ucca0\ud68c\ud558\ub294 \uac83\uc744 \ub9d0\ud55c\ub2e4. \ubc1c\ud589\uc778\uc740 \uc9c0\uae09\uc778\uacfc \uc218\ud45c\uacc4\uc57d\uc5d0 \ub530\ub77c \uc218\ud45c\ub97c \ubc1c\ud589\ud558\uace0 \uc9c0\uae09\uc778\uc740 \uadf8 \uc704\ud0c1\uc5d0 \ub530\ub77c \ubc1c\ud589\uc778\uc758 \uc790\uae08\uc5d0\uc11c \uc218\ud45c\uae08\uc561\uc744 \uc9c0\uae09\ud55c\ub2e4. \uc774 \uacbd\uc6b0\uc5d0 \uc218\ud45c\uc758 \ubc1c\ud589\ud589\uc704\ub294 \uc9c0\uae09\uc758 \uc9c0\uc2dc\uc774\uba70 \uc9c0\uae09\uc9c0\uc2dc\ub294 \uc9c0\uc2dc\uc778\uc778 \uc218\ud45c\ubc1c\ud589\uc778\uc774 \uc784\uc758\ub85c \ucca0\ud68c\ud568\uc73c\ub85c\uc368 \uadf8 \uc9c0\uae09\uc744 \uc911\uc9c0\uc2dc\ud0ac \uc218 \uc788\ub2e4. \ub530\ub77c\uc11c \ucde8\uc18c \ud6c4\uc5d0\ub294 \uc9c0\uae09\uc778\uc774 \uc774 \uc218\ud45c\uc5d0 \ub300\ud558\uc5ec \uc9c0\uae09\uc744 \ud558\uc5ec\ub3c4 \ubc1c\ud589\uc778\uc5d0\uac8c \uadf8 \uacc4\uc0b0\uc744 \ub3cc\ub9ac\uc9c0 \ubabb\ud558\uac8c \ub41c\ub2e4. \uadf8\ub7ec\ub098 \ud55c\ud3b8 \ubc1c\ud589\uc778\uc774 \uc9c0\uae09\uc704\ud0c1\uc744 \uc5b8\uc81c\ub4e0\uc9c0 \ucde8\uc18c\ud560 \uc218 \uc788\ub2e4\uace0 \ud558\uba74 \uc218\ud45c\uc18c\uc9c0\uc778\uc758 \uc9c0\uc704\ub97c \ubd88\uc548\ud558\uac8c \ud558\uc5ec \uc624\ud788\ub824 \uac70\ub798\uc758 \uc548\uc804\uc744 \ud574\uce58\uac8c \ub41c\ub2e4. \uadf8\ub7ec\ubbc0\ub85c \ubc95\uc740 \ub610\ud55c \uc9c0\uae09\uc704\ud0c1\uc758 \ucde8\uc18c\ub97c \uc81c\ud55c\ud55c\ub2e4. \uc774\uc640 \uac19\uc774 \uc9c0\uae09\uc704\ud0c1\uc758 \ucde8\uc18c\ub294 \ubc1c\ud589\uc778\uc5d0\uac8c\ub294 \uc720\ub9ac\ud558\uc9c0\ub9cc \uc18c\uc9c0\uc778\uc5d0\uac8c\ub294 \ubd88\ub9ac\ud55c \uc81c\ub3c4\uc774\ubbc0\ub85c \uadf8 \ucde8\uc18c\ub97c \ubb34\uc81c\ud55c\uc73c\ub85c \ud5c8\uc6a9\ud558\uba74 \uc218\ud45c\uc758 \uc720\ud1b5\uc744 \ud574\uce58\uac8c \ub41c\ub2e4. \ub530\ub77c\uc11c \ucde8\uc18c\ub294 \uc81c\uc2dc\uae30\uac04 \uacbd\uacfc \ud6c4\uc5d0\ub9cc \ud6a8\ub825\uc744 \ubc1c\uc0dd\ud558\ub3c4\ub85d \uc81c\ud55c\ud558\uc5ec \uc9c0\uae09\uc778\uc740 \uc81c\uc2dc\uae30\uac04 \uc548\uc5d0\ub294 \ubc1c\ud589\uc778\uc774 \ucde8\uc18c\ub97c \ud558\uc5ec\ub3c4 \uc9c0\uae09\uc744 \ud560 \uc218 \uc788\uc73c\uba70, \uadf8 \uacc4\uc0b0\uc744 \ubc1c\ud589\uc778\uc5d0\uac8c \uadc0\uc18d\uc2dc\ud0ac \uc218 \uc788\ub3c4\ub85d \ud558\uc600\ub2e4(\uc218\ud45c 32\uc870 1\ud56d). \uc9c0\uae09\uc778\uc740 \uc218\ud45c\uc0c1\uc758 \uc758\ubb34\uc790\uac00 \uc544\ub2c8\uace0 \uc9c0\uae09\uc5ec\ubd80\ub294 \uadf8\uc758 \uc784\uc758\uc774\ubbc0\ub85c \ubc1c\ud589\uc778\uc758 \uc81c\uc2dc\uae30\uac04 \ub0b4\uc758 \ubb34\ud6a8\uc778 \uc704\ud0c1\ucde8\uc18c\uc5d0 \ub530\ub77c\uc11c \uc18c\uc9c0\uc778\uc5d0 \ub300\ud55c \uc218\ud45c\uc9c0\uae09\uc744 \uac70\uc808\ud55c \uacbd\uc6b0\uc5d0\ub3c4 \uc18c\uc9c0\uc778\uc740 \uc9c0\uae09\uc778\uc5d0\uac8c \uc218\ud45c\uc758 \uc9c0\uae09\uc744 \uac15\uc694\ud560 \uc218 \uc5c6\uc73c\uba70 \uacb0\uad6d \uc804\uc790\uc5d0 \ub300\ud558\uc5ec \uc0c1\ud658 \uccad\uad6c\ud558\ub294 \uc218\ubc16\uc5d0 \uc5c6\ub2e4. \ub530\ub77c\uc11c \uc218\ud45c\uc758 \ub3c4\ub09c\u00b7\uc720\uc2e4(\u907a\u5931)\uc758 \uacbd\uc6b0\uc5d0\ub3c4 \uadf8 \ubc94\uc704 \ub0b4\uc5d0\uc11c\ub294 \ubc1c\ud589\uc778\uc740 \ubcf4\ud638\ub418\ub294 \uc148\uc774\ub2e4. \uc774\uc5d0 \ubc18\ud558\uc5ec \ubc1c\ud589\uc778\uc740 \uc81c\uc2dc\uae30\uac04 \uacbd\uacfc \ud6c4\uc5d0\ub294 \uc784\uc758\ub85c \uc9c0\uae09\uc704\ud0c1\uc758 \ucde8\uc18c\ub97c \ud560 \uc218 \uc788\ub2e4. \uadf8\ub7ec\ub098 \uc81c\uc2dc\uae30\uac04\uc774 \uacbd\uacfc\ud568\uc73c\ub85c\uc368 \ub2f9\uc5f0\ud788 \uc9c0\uae09\uc704\ud0c1\uc774 \uc18c\uba78\ud558\ub294 \uac83\uc774 \uc544\ub2c8\ubbc0\ub85c \ubc1c\ud589\uc778\uc5d0 \uc758\ud55c \ucde8\uc18c\uac00 \uc5c6\ub294 \ud55c \uc9c0\uae09\uc778\uc740 \ubc1c\ud589\uc778\uc758 \uacc4\uc0b0\uc73c\ub85c \uc720\ud6a8\ud55c \uc9c0\uae09\uc744 \ud560 \uc218 \uc788\ub2e4(\uc218\ud45c 32\uc870 2\ud56d). \uc9c0\uae09\uc704\ud0c1\uc758 \ucde8\uc18c\uc758 \ud1b5\uc9c0\ubc29\ubc95\uc5d0\ub294 \uc81c\ud55c\uc774 \uc5c6\ub2e4. \uc11c\uba74\uc774\ub098 \uad6c\uc220\ub85c\ub3c4 \ud560 \uc218 \uc788\uc73c\uba70 \ucde8\uc18c\ub294 \uadf8 \ud1b5\uc9c0\uac00 \uc9c0\uae09\uc778\uc5d0\uac8c \ub3c4\ub2ec\ud55c \ub54c\uc5d0 \ud6a8\ub825\uc744 \ubc1c\uc0dd\ud55c\ub2e4. \uc704", "sentence_answer": "\uc9c0\uae09\uc704\ud0c1\uc758 \ucde8\uc18c\ub294 \uc218\ud45c\uc758 \ub3c4\ub09c\u00b7\uc0c1\uc2e4\u00b7\ubcc0\uc870 \ub4f1\uc758 \uacbd\uc6b0\uc5d0 \ubc88\uc7a1\ud55c \uacf5\uc2dc\ucd5c\uace0\uc808\ucc28 \ub97c \uc0dd\ub7b5\ud558\uae30 \uc704\ud558\uc5ec \uc778\uc815\ub41c \ud3b8\ub9ac\ud55c \uc81c\ub3c4\uc774\ub2e4.", "paragraph_id": "6536952-8-0", "paragraph_question": "question: \uc9c0\uae09\uc704\ud0c1\uc758 \ucde8\uc18c \uc81c\ub3c4\uac00 \uc0dd\ub7b5\ud558\ub294 \uc808\ucc28\ub294?, context: \uc9c0\uae09\uc704\ud0c1\uc758 \ucde8\uc18c\ub294 \uc218\ud45c\uc758 \ub3c4\ub09c\u00b7\uc0c1\uc2e4\u00b7\ubcc0\uc870 \ub4f1\uc758 \uacbd\uc6b0\uc5d0 \ubc88\uc7a1\ud55c \uacf5\uc2dc\ucd5c\uace0\uc808\ucc28\ub97c \uc0dd\ub7b5\ud558\uae30 \uc704\ud558\uc5ec \uc778\uc815\ub41c \ud3b8\ub9ac\ud55c \uc81c\ub3c4\uc774\ub2e4. \uc5ec\uae30\uc5d0\uc11c \uc9c0\uae09\uc704\ud0c1\uc758 \ucde8\uc18c\ub77c \ud568\uc740 \uc218\ud45c\uacc4\uc57d\uc758 \ucde8\uc18c\ub97c \ub9d0\ud558\ub294 \uac83\uc774 \uc544\ub2c8\uace0 \uac1c\uac1c\uc758 \uc218\ud45c\ubc1c\ud589\ud589\uc704\uc778 \uc9c0\uae09\uc9c0\uc2dc\ub97c \ucca0\ud68c\ud558\ub294 \uac83\uc744 \ub9d0\ud55c\ub2e4. \ubc1c\ud589\uc778\uc740 \uc9c0\uae09\uc778\uacfc \uc218\ud45c\uacc4\uc57d\uc5d0 \ub530\ub77c \uc218\ud45c\ub97c \ubc1c\ud589\ud558\uace0 \uc9c0\uae09\uc778\uc740 \uadf8 \uc704\ud0c1\uc5d0 \ub530\ub77c \ubc1c\ud589\uc778\uc758 \uc790\uae08\uc5d0\uc11c \uc218\ud45c\uae08\uc561\uc744 \uc9c0\uae09\ud55c\ub2e4. \uc774 \uacbd\uc6b0\uc5d0 \uc218\ud45c\uc758 \ubc1c\ud589\ud589\uc704\ub294 \uc9c0\uae09\uc758 \uc9c0\uc2dc\uc774\uba70 \uc9c0\uae09\uc9c0\uc2dc\ub294 \uc9c0\uc2dc\uc778\uc778 \uc218\ud45c\ubc1c\ud589\uc778\uc774 \uc784\uc758\ub85c \ucca0\ud68c\ud568\uc73c\ub85c\uc368 \uadf8 \uc9c0\uae09\uc744 \uc911\uc9c0\uc2dc\ud0ac \uc218 \uc788\ub2e4. \ub530\ub77c\uc11c \ucde8\uc18c \ud6c4\uc5d0\ub294 \uc9c0\uae09\uc778\uc774 \uc774 \uc218\ud45c\uc5d0 \ub300\ud558\uc5ec \uc9c0\uae09\uc744 \ud558\uc5ec\ub3c4 \ubc1c\ud589\uc778\uc5d0\uac8c \uadf8 \uacc4\uc0b0\uc744 \ub3cc\ub9ac\uc9c0 \ubabb\ud558\uac8c \ub41c\ub2e4. \uadf8\ub7ec\ub098 \ud55c\ud3b8 \ubc1c\ud589\uc778\uc774 \uc9c0\uae09\uc704\ud0c1\uc744 \uc5b8\uc81c\ub4e0\uc9c0 \ucde8\uc18c\ud560 \uc218 \uc788\ub2e4\uace0 \ud558\uba74 \uc218\ud45c\uc18c\uc9c0\uc778\uc758 \uc9c0\uc704\ub97c \ubd88\uc548\ud558\uac8c \ud558\uc5ec \uc624\ud788\ub824 \uac70\ub798\uc758 \uc548\uc804\uc744 \ud574\uce58\uac8c \ub41c\ub2e4. \uadf8\ub7ec\ubbc0\ub85c \ubc95\uc740 \ub610\ud55c \uc9c0\uae09\uc704\ud0c1\uc758 \ucde8\uc18c\ub97c \uc81c\ud55c\ud55c\ub2e4. \uc774\uc640 \uac19\uc774 \uc9c0\uae09\uc704\ud0c1\uc758 \ucde8\uc18c\ub294 \ubc1c\ud589\uc778\uc5d0\uac8c\ub294 \uc720\ub9ac\ud558\uc9c0\ub9cc \uc18c\uc9c0\uc778\uc5d0\uac8c\ub294 \ubd88\ub9ac\ud55c \uc81c\ub3c4\uc774\ubbc0\ub85c \uadf8 \ucde8\uc18c\ub97c \ubb34\uc81c\ud55c\uc73c\ub85c \ud5c8\uc6a9\ud558\uba74 \uc218\ud45c\uc758 \uc720\ud1b5\uc744 \ud574\uce58\uac8c \ub41c\ub2e4. \ub530\ub77c\uc11c \ucde8\uc18c\ub294 \uc81c\uc2dc\uae30\uac04 \uacbd\uacfc \ud6c4\uc5d0\ub9cc \ud6a8\ub825\uc744 \ubc1c\uc0dd\ud558\ub3c4\ub85d \uc81c\ud55c\ud558\uc5ec \uc9c0\uae09\uc778\uc740 \uc81c\uc2dc\uae30\uac04 \uc548\uc5d0\ub294 \ubc1c\ud589\uc778\uc774 \ucde8\uc18c\ub97c \ud558\uc5ec\ub3c4 \uc9c0\uae09\uc744 \ud560 \uc218 \uc788\uc73c\uba70, \uadf8 \uacc4\uc0b0\uc744 \ubc1c\ud589\uc778\uc5d0\uac8c \uadc0\uc18d\uc2dc\ud0ac \uc218 \uc788\ub3c4\ub85d \ud558\uc600\ub2e4(\uc218\ud45c 32\uc870 1\ud56d). \uc9c0\uae09\uc778\uc740 \uc218\ud45c\uc0c1\uc758 \uc758\ubb34\uc790\uac00 \uc544\ub2c8\uace0 \uc9c0\uae09\uc5ec\ubd80\ub294 \uadf8\uc758 \uc784\uc758\uc774\ubbc0\ub85c \ubc1c\ud589\uc778\uc758 \uc81c\uc2dc\uae30\uac04 \ub0b4\uc758 \ubb34\ud6a8\uc778 \uc704\ud0c1\ucde8\uc18c\uc5d0 \ub530\ub77c\uc11c \uc18c\uc9c0\uc778\uc5d0 \ub300\ud55c \uc218\ud45c\uc9c0\uae09\uc744 \uac70\uc808\ud55c \uacbd\uc6b0\uc5d0\ub3c4 \uc18c\uc9c0\uc778\uc740 \uc9c0\uae09\uc778\uc5d0\uac8c \uc218\ud45c\uc758 \uc9c0\uae09\uc744 \uac15\uc694\ud560 \uc218 \uc5c6\uc73c\uba70 \uacb0\uad6d \uc804\uc790\uc5d0 \ub300\ud558\uc5ec \uc0c1\ud658 \uccad\uad6c\ud558\ub294 \uc218\ubc16\uc5d0 \uc5c6\ub2e4. \ub530\ub77c\uc11c \uc218\ud45c\uc758 \ub3c4\ub09c\u00b7\uc720\uc2e4(\u907a\u5931)\uc758 \uacbd\uc6b0\uc5d0\ub3c4 \uadf8 \ubc94\uc704 \ub0b4\uc5d0\uc11c\ub294 \ubc1c\ud589\uc778\uc740 \ubcf4\ud638\ub418\ub294 \uc148\uc774\ub2e4. \uc774\uc5d0 \ubc18\ud558\uc5ec \ubc1c\ud589\uc778\uc740 \uc81c\uc2dc\uae30\uac04 \uacbd\uacfc \ud6c4\uc5d0\ub294 \uc784\uc758\ub85c \uc9c0\uae09\uc704\ud0c1\uc758 \ucde8\uc18c\ub97c \ud560 \uc218 \uc788\ub2e4. \uadf8\ub7ec\ub098 \uc81c\uc2dc\uae30\uac04\uc774 \uacbd\uacfc\ud568\uc73c\ub85c\uc368 \ub2f9\uc5f0\ud788 \uc9c0\uae09\uc704\ud0c1\uc774 \uc18c\uba78\ud558\ub294 \uac83\uc774 \uc544\ub2c8\ubbc0\ub85c \ubc1c\ud589\uc778\uc5d0 \uc758\ud55c \ucde8\uc18c\uac00 \uc5c6\ub294 \ud55c \uc9c0\uae09\uc778\uc740 \ubc1c\ud589\uc778\uc758 \uacc4\uc0b0\uc73c\ub85c \uc720\ud6a8\ud55c \uc9c0\uae09\uc744 \ud560 \uc218 \uc788\ub2e4(\uc218\ud45c 32\uc870 2\ud56d). \uc9c0\uae09\uc704\ud0c1\uc758 \ucde8\uc18c\uc758 \ud1b5\uc9c0\ubc29\ubc95\uc5d0\ub294 \uc81c\ud55c\uc774 \uc5c6\ub2e4. \uc11c\uba74\uc774\ub098 \uad6c\uc220\ub85c\ub3c4 \ud560 \uc218 \uc788\uc73c\uba70 \ucde8\uc18c\ub294 \uadf8 \ud1b5\uc9c0\uac00 \uc9c0\uae09\uc778\uc5d0\uac8c \ub3c4\ub2ec\ud55c \ub54c\uc5d0 \ud6a8\ub825\uc744 \ubc1c\uc0dd\ud55c\ub2e4. \uc704"} {"question": "\ub85c\uc800\uc2a4 \ud63c\uc988\ube44\uc758 \ubcc4\uba85\uc740?", "paragraph": "\ub85c\uc800\uc2a4 \ud63c\uc988\ube44(\uc601\uc5b4: Rogers Hornsby, 1896\ub144 4\uc6d4 27\uc77c ~ 1963\ub144 1\uc6d4 5\uc77c, \ubbf8\uad6d \ud14d\uc0ac\uc2a4 \uc8fc \uc708\ud130\uc2a4)\ub294 \ubbf8\uad6d\uc758 \uc804 \uba54\uc774\uc800 \ub9ac\uadf8 \ubca0\uc774\uc2a4\ubcfc \ub0b4\uc57c\uc218\uc774\uc790 \ucf54\uce58, \uac10\ub3c5\uc774\ub2e4. \ubcc4\uba85\uc740 \ub354 \ub77c\uc790(\uc601\uc5b4: The Rajah)\uc774\uba70, \uc138\uc778\ud2b8\ub8e8\uc774\uc2a4 \uce74\ub514\ub110\uc2a4 (1915-1926\ub144, 1933\ub144), \ub274\uc695 \uc790\uc774\uc5b8\uce20 (1927\ub144), \ubcf4\uc2a4\ud134 \ube0c\ub808\uc774\ube0c\uc2a4 (1928), \uc2dc\uce74\uace0 \ucef5\uc2a4 (1929-1932\ub144), \uc138\uc778\ud2b8\ub8e8\uc774\uc2a4 \ube0c\ub77c\uc6b4\uc2a4 (1933-1937\ub144)\uc5d0\uc11c 23\uac1c \uc2dc\uc98c\uc744 \ub6f0\uc5c8\ub2e4. \ud63c\uc988\ube44\ub294 2390\uc548\ud0c0, 301\ud648\ub7f0, \ud1b5\uc0b0 \ud0c0\uc728 0.358\uc744 \uae30\ub85d\ud588\uace0, \ub0b4\uc154\ub110 \ub9ac\uadf8 \ucd5c\uc6b0\uc218 \uc120\uc218 (MVP)\uc0c1\uc744 \ub450 \ubc88 \ubc1b\uc558\uace0 \uc6d4\ub4dc \uc2dc\ub9ac\uc988\uc5d0\uc11c\ub3c4 \ud55c \ubc88 \uc6b0\uc2b9\ud588\ub2e4.", "answer": "\ub354 \ub77c\uc790", "sentence": "\ubcc4\uba85\uc740 \ub354 \ub77c\uc790 (\uc601\uc5b4: The Rajah)", "paragraph_sentence": "\ub85c\uc800\uc2a4 \ud63c\uc988\ube44(\uc601\uc5b4: Rogers Hornsby, 1896\ub144 4\uc6d4 27\uc77c ~ 1963\ub144 1\uc6d4 5\uc77c, \ubbf8\uad6d \ud14d\uc0ac\uc2a4 \uc8fc \uc708\ud130\uc2a4)\ub294 \ubbf8\uad6d\uc758 \uc804 \uba54\uc774\uc800 \ub9ac\uadf8 \ubca0\uc774\uc2a4\ubcfc \ub0b4\uc57c\uc218\uc774\uc790 \ucf54\uce58, \uac10\ub3c5\uc774\ub2e4. \ubcc4\uba85\uc740 \ub354 \ub77c\uc790 (\uc601\uc5b4: The Rajah) \uc774\uba70, \uc138\uc778\ud2b8\ub8e8\uc774\uc2a4 \uce74\ub514\ub110\uc2a4 (1915-1926\ub144, 1933\ub144), \ub274\uc695 \uc790\uc774\uc5b8\uce20 (1927\ub144), \ubcf4\uc2a4\ud134 \ube0c\ub808\uc774\ube0c\uc2a4 (1928), \uc2dc\uce74\uace0 \ucef5\uc2a4 (1929-1932\ub144), \uc138\uc778\ud2b8\ub8e8\uc774\uc2a4 \ube0c\ub77c\uc6b4\uc2a4 (1933-1937\ub144)\uc5d0\uc11c 23\uac1c \uc2dc\uc98c\uc744 \ub6f0\uc5c8\ub2e4. \ud63c\uc988\ube44\ub294 2390\uc548\ud0c0, 301\ud648\ub7f0, \ud1b5\uc0b0 \ud0c0\uc728 0.358\uc744 \uae30\ub85d\ud588\uace0, \ub0b4\uc154\ub110 \ub9ac\uadf8 \ucd5c\uc6b0\uc218 \uc120\uc218 (MVP)\uc0c1\uc744 \ub450 \ubc88 \ubc1b\uc558\uace0 \uc6d4\ub4dc \uc2dc\ub9ac\uc988\uc5d0\uc11c\ub3c4 \ud55c \ubc88 \uc6b0\uc2b9\ud588\ub2e4.", "paragraph_answer": "\ub85c\uc800\uc2a4 \ud63c\uc988\ube44(\uc601\uc5b4: Rogers Hornsby, 1896\ub144 4\uc6d4 27\uc77c ~ 1963\ub144 1\uc6d4 5\uc77c, \ubbf8\uad6d \ud14d\uc0ac\uc2a4 \uc8fc \uc708\ud130\uc2a4)\ub294 \ubbf8\uad6d\uc758 \uc804 \uba54\uc774\uc800 \ub9ac\uadf8 \ubca0\uc774\uc2a4\ubcfc \ub0b4\uc57c\uc218\uc774\uc790 \ucf54\uce58, \uac10\ub3c5\uc774\ub2e4. \ubcc4\uba85\uc740 \ub354 \ub77c\uc790 (\uc601\uc5b4: The Rajah)\uc774\uba70, \uc138\uc778\ud2b8\ub8e8\uc774\uc2a4 \uce74\ub514\ub110\uc2a4 (1915-1926\ub144, 1933\ub144), \ub274\uc695 \uc790\uc774\uc5b8\uce20 (1927\ub144), \ubcf4\uc2a4\ud134 \ube0c\ub808\uc774\ube0c\uc2a4 (1928), \uc2dc\uce74\uace0 \ucef5\uc2a4 (1929-1932\ub144), \uc138\uc778\ud2b8\ub8e8\uc774\uc2a4 \ube0c\ub77c\uc6b4\uc2a4 (1933-1937\ub144)\uc5d0\uc11c 23\uac1c \uc2dc\uc98c\uc744 \ub6f0\uc5c8\ub2e4. \ud63c\uc988\ube44\ub294 2390\uc548\ud0c0, 301\ud648\ub7f0, \ud1b5\uc0b0 \ud0c0\uc728 0.358\uc744 \uae30\ub85d\ud588\uace0, \ub0b4\uc154\ub110 \ub9ac\uadf8 \ucd5c\uc6b0\uc218 \uc120\uc218 (MVP)\uc0c1\uc744 \ub450 \ubc88 \ubc1b\uc558\uace0 \uc6d4\ub4dc \uc2dc\ub9ac\uc988\uc5d0\uc11c\ub3c4 \ud55c \ubc88 \uc6b0\uc2b9\ud588\ub2e4.", "sentence_answer": "\ubcc4\uba85\uc740 \ub354 \ub77c\uc790 (\uc601\uc5b4: The Rajah)", "paragraph_id": "6496568-0-0", "paragraph_question": "question: \ub85c\uc800\uc2a4 \ud63c\uc988\ube44\uc758 \ubcc4\uba85\uc740?, context: \ub85c\uc800\uc2a4 \ud63c\uc988\ube44(\uc601\uc5b4: Rogers Hornsby, 1896\ub144 4\uc6d4 27\uc77c ~ 1963\ub144 1\uc6d4 5\uc77c, \ubbf8\uad6d \ud14d\uc0ac\uc2a4 \uc8fc \uc708\ud130\uc2a4)\ub294 \ubbf8\uad6d\uc758 \uc804 \uba54\uc774\uc800 \ub9ac\uadf8 \ubca0\uc774\uc2a4\ubcfc \ub0b4\uc57c\uc218\uc774\uc790 \ucf54\uce58, \uac10\ub3c5\uc774\ub2e4. \ubcc4\uba85\uc740 \ub354 \ub77c\uc790(\uc601\uc5b4: The Rajah)\uc774\uba70, \uc138\uc778\ud2b8\ub8e8\uc774\uc2a4 \uce74\ub514\ub110\uc2a4 (1915-1926\ub144, 1933\ub144), \ub274\uc695 \uc790\uc774\uc5b8\uce20 (1927\ub144), \ubcf4\uc2a4\ud134 \ube0c\ub808\uc774\ube0c\uc2a4 (1928), \uc2dc\uce74\uace0 \ucef5\uc2a4 (1929-1932\ub144), \uc138\uc778\ud2b8\ub8e8\uc774\uc2a4 \ube0c\ub77c\uc6b4\uc2a4 (1933-1937\ub144)\uc5d0\uc11c 23\uac1c \uc2dc\uc98c\uc744 \ub6f0\uc5c8\ub2e4. \ud63c\uc988\ube44\ub294 2390\uc548\ud0c0, 301\ud648\ub7f0, \ud1b5\uc0b0 \ud0c0\uc728 0.358\uc744 \uae30\ub85d\ud588\uace0, \ub0b4\uc154\ub110 \ub9ac\uadf8 \ucd5c\uc6b0\uc218 \uc120\uc218 (MVP)\uc0c1\uc744 \ub450 \ubc88 \ubc1b\uc558\uace0 \uc6d4\ub4dc \uc2dc\ub9ac\uc988\uc5d0\uc11c\ub3c4 \ud55c \ubc88 \uc6b0\uc2b9\ud588\ub2e4."} {"question": "\uc0ac\uc774\ud37c\uc758 \ud0c4\uc555\uc5d0 \ubc18\ud558\uc5ec \uc124\ub9bd\ub41c \ud68c\uc0ac\uc758 \uc774\ub984\uc740?", "paragraph": "1860\ub144 \uac70\ub300\ud55c \uc77c\uc2dd \uc774\ud6c4 \uc138\uacc4 \ub3c4\ucc98\uc5d0\uc11c\ub294 \ud669\ubb34\uc9c0 \uc704\uc5d0 \ub3c4\uc2dc\uac00 \uc0dd\uaca8\ub098\uac70\ub098 \ud2b9\ubcc4\ud55c \ub2a5\ub825\uc744 \uac00\uc9c4 \uc0ac\ub78c\ub4e4\uc774 \ub098\ud0c0\ub098\uac8c \ub41c\ub2e4. \uc774\ub4e4\uc740 \uc0ac\uc774\ud37c(Cypher)\ub77c\uace0 \ubd88\ub9ac\uba70 \ud0c4\uc555\uc758 \ub300\uc0c1\uc774 \ub418\ub294\ub370, \uc5ec\uae30 \ubc18\ud558\uc5ec \ud5ec\ub9ac\uc624\uc2a4\ub77c\ub294 \ud68c\uc0ac\uac00 \uc124\ub9bd\ub41c\ub2e4. \ubfd0\ub9cc \uc544\ub2c8\ub77c \uc81c\ub3c4\uc758 \uc190\uc774 \ubbf8\uce58\uce58 \uc54a\ub294 \uacf3\uc5d0\uc11c\ub3c4 \uc2a4\uc2a4\ub85c\uc758 \uad8c\ub9ac\ub97c \ubcf4\ud638\ud558\uae30 \uc704\ud574 \uc9c0\ud558\uc5f0\ud569\uc774 \uacb0\uc131\ub41c\ub2e4. \ub450 \uc138\ub825\uc740 \uac01\uc790\uc758 \uc601\uc5ed\uc5d0\uc11c \uacf5\uc874\ud574\uc654\uc9c0\ub9cc, \uba87 \ucc28\ub840 \ub9c8\ucc30\ub3c4 \uc874\uc7ac\ud558\uc600\ub2e4. \uadf8\ub7ec\ub358 \ub3c4\uc911 \ub178\uc778\uc774 \ub9cc\ub4e0 \uc548\ud0c0\ub9ac\uc6b0\uc2a4\ub77c\ub294 \uc9d1\ub2e8\uc774 \uc0dd\uaca8\ub098 \uad11\uc2e0\uc801\uc778 \uc9c0\uc9c0\ub97c \ubc14\ud0d5\uc73c\ub85c \uae09\uaca9\ud558\uac8c \uc131\uc7a5\ud588\uace0, \uc778\uac04\uc744 \uc0c1\ub300\ub85c \ub054\ucc0d\ud55c \uc2e4\ud5d8\uc744 \uc790\ud589\ud558\uae30\uc5d0 \uc774\ub978\ub2e4. \uc138\ub825\uc744 \ud0a4\uc6b4 \uc548\ud0c0\ub9ac\uc6b0\uc2a4\ub294 \uc138\uacc4\ub97c \ud5a5\ud574 \uc804\uba74\uc804\uc744 \uc120\ud3ec\ud558\uc9c0\ub9cc, \ud68c\uc0ac\uc640 \uc5f0\ud569\uc740 \ub3d9\ub9f9\uc744 \ub9fa\uc5b4 \uc778\ud615\uc2e4 \ub04a\uae30 \uc791\uc804\uc744 \ud1b5\ud574 \ub178\uc778\uc744 \uc81c\uac70\ud558\uace0 \uc548\ud0c0\ub9ac\uc6b0\uc2a4\ub97c \uada4\uba78\uc2dc\ud0a8\ub2e4. \uada4\uba78 \uc774\ud6c4 \uc7a0\uae50\ub3d9\uc548 \ud3c9\ud654\uc758 \uc2dc\ub300\uac00 \ucc3e\uc544\uc654\uc73c\ub098, \ud3c9\ud654\ub294 \ub2e4\uc2dc\uae08 \uae68\uc9c0\ub294 \uae30\ubbf8\uac00 \ubcf4\uc774\uace0, \ud769\uc5b4\uc9c4 \uc548\ud0c0\ub9ac\uc6b0\uc2a4 \uc138\ub825\ub9c8\uc800 \uacf3\uacf3\uc5d0\uc11c \ubaa8\uc2b5\uc744 \ub4dc\ub7ec\ub0b8\ub2e4.", "answer": "\ud5ec\ub9ac\uc624\uc2a4", "sentence": "\uc774\ub4e4\uc740 \uc0ac\uc774\ud37c(Cypher)\ub77c\uace0 \ubd88\ub9ac\uba70 \ud0c4\uc555\uc758 \ub300\uc0c1\uc774 \ub418\ub294\ub370, \uc5ec\uae30 \ubc18\ud558\uc5ec \ud5ec\ub9ac\uc624\uc2a4 \ub77c\ub294 \ud68c\uc0ac\uac00 \uc124\ub9bd\ub41c\ub2e4.", "paragraph_sentence": "1860\ub144 \uac70\ub300\ud55c \uc77c\uc2dd \uc774\ud6c4 \uc138\uacc4 \ub3c4\ucc98\uc5d0\uc11c\ub294 \ud669\ubb34\uc9c0 \uc704\uc5d0 \ub3c4\uc2dc\uac00 \uc0dd\uaca8\ub098\uac70\ub098 \ud2b9\ubcc4\ud55c \ub2a5\ub825\uc744 \uac00\uc9c4 \uc0ac\ub78c\ub4e4\uc774 \ub098\ud0c0\ub098\uac8c \ub41c\ub2e4. \uc774\ub4e4\uc740 \uc0ac\uc774\ud37c(Cypher)\ub77c\uace0 \ubd88\ub9ac\uba70 \ud0c4\uc555\uc758 \ub300\uc0c1\uc774 \ub418\ub294\ub370, \uc5ec\uae30 \ubc18\ud558\uc5ec \ud5ec\ub9ac\uc624\uc2a4 \ub77c\ub294 \ud68c\uc0ac\uac00 \uc124\ub9bd\ub41c\ub2e4. \ubfd0\ub9cc \uc544\ub2c8\ub77c \uc81c\ub3c4\uc758 \uc190\uc774 \ubbf8\uce58\uce58 \uc54a\ub294 \uacf3\uc5d0\uc11c\ub3c4 \uc2a4\uc2a4\ub85c\uc758 \uad8c\ub9ac\ub97c \ubcf4\ud638\ud558\uae30 \uc704\ud574 \uc9c0\ud558\uc5f0\ud569\uc774 \uacb0\uc131\ub41c\ub2e4. \ub450 \uc138\ub825\uc740 \uac01\uc790\uc758 \uc601\uc5ed\uc5d0\uc11c \uacf5\uc874\ud574\uc654\uc9c0\ub9cc, \uba87 \ucc28\ub840 \ub9c8\ucc30\ub3c4 \uc874\uc7ac\ud558\uc600\ub2e4. \uadf8\ub7ec\ub358 \ub3c4\uc911 \ub178\uc778\uc774 \ub9cc\ub4e0 \uc548\ud0c0\ub9ac\uc6b0\uc2a4\ub77c\ub294 \uc9d1\ub2e8\uc774 \uc0dd\uaca8\ub098 \uad11\uc2e0\uc801\uc778 \uc9c0\uc9c0\ub97c \ubc14\ud0d5\uc73c\ub85c \uae09\uaca9\ud558\uac8c \uc131\uc7a5\ud588\uace0, \uc778\uac04\uc744 \uc0c1\ub300\ub85c \ub054\ucc0d\ud55c \uc2e4\ud5d8\uc744 \uc790\ud589\ud558\uae30\uc5d0 \uc774\ub978\ub2e4. \uc138\ub825\uc744 \ud0a4\uc6b4 \uc548\ud0c0\ub9ac\uc6b0\uc2a4\ub294 \uc138\uacc4\ub97c \ud5a5\ud574 \uc804\uba74\uc804\uc744 \uc120\ud3ec\ud558\uc9c0\ub9cc, \ud68c\uc0ac\uc640 \uc5f0\ud569\uc740 \ub3d9\ub9f9\uc744 \ub9fa\uc5b4 \uc778\ud615\uc2e4 \ub04a\uae30 \uc791\uc804\uc744 \ud1b5\ud574 \ub178\uc778\uc744 \uc81c\uac70\ud558\uace0 \uc548\ud0c0\ub9ac\uc6b0\uc2a4\ub97c \uada4\uba78\uc2dc\ud0a8\ub2e4. \uada4\uba78 \uc774\ud6c4 \uc7a0\uae50\ub3d9\uc548 \ud3c9\ud654\uc758 \uc2dc\ub300\uac00 \ucc3e\uc544\uc654\uc73c\ub098, \ud3c9\ud654\ub294 \ub2e4\uc2dc\uae08 \uae68\uc9c0\ub294 \uae30\ubbf8\uac00 \ubcf4\uc774\uace0, \ud769\uc5b4\uc9c4 \uc548\ud0c0\ub9ac\uc6b0\uc2a4 \uc138\ub825\ub9c8\uc800 \uacf3\uacf3\uc5d0\uc11c \ubaa8\uc2b5\uc744 \ub4dc\ub7ec\ub0b8\ub2e4.", "paragraph_answer": "1860\ub144 \uac70\ub300\ud55c \uc77c\uc2dd \uc774\ud6c4 \uc138\uacc4 \ub3c4\ucc98\uc5d0\uc11c\ub294 \ud669\ubb34\uc9c0 \uc704\uc5d0 \ub3c4\uc2dc\uac00 \uc0dd\uaca8\ub098\uac70\ub098 \ud2b9\ubcc4\ud55c \ub2a5\ub825\uc744 \uac00\uc9c4 \uc0ac\ub78c\ub4e4\uc774 \ub098\ud0c0\ub098\uac8c \ub41c\ub2e4. \uc774\ub4e4\uc740 \uc0ac\uc774\ud37c(Cypher)\ub77c\uace0 \ubd88\ub9ac\uba70 \ud0c4\uc555\uc758 \ub300\uc0c1\uc774 \ub418\ub294\ub370, \uc5ec\uae30 \ubc18\ud558\uc5ec \ud5ec\ub9ac\uc624\uc2a4 \ub77c\ub294 \ud68c\uc0ac\uac00 \uc124\ub9bd\ub41c\ub2e4. \ubfd0\ub9cc \uc544\ub2c8\ub77c \uc81c\ub3c4\uc758 \uc190\uc774 \ubbf8\uce58\uce58 \uc54a\ub294 \uacf3\uc5d0\uc11c\ub3c4 \uc2a4\uc2a4\ub85c\uc758 \uad8c\ub9ac\ub97c \ubcf4\ud638\ud558\uae30 \uc704\ud574 \uc9c0\ud558\uc5f0\ud569\uc774 \uacb0\uc131\ub41c\ub2e4. \ub450 \uc138\ub825\uc740 \uac01\uc790\uc758 \uc601\uc5ed\uc5d0\uc11c \uacf5\uc874\ud574\uc654\uc9c0\ub9cc, \uba87 \ucc28\ub840 \ub9c8\ucc30\ub3c4 \uc874\uc7ac\ud558\uc600\ub2e4. \uadf8\ub7ec\ub358 \ub3c4\uc911 \ub178\uc778\uc774 \ub9cc\ub4e0 \uc548\ud0c0\ub9ac\uc6b0\uc2a4\ub77c\ub294 \uc9d1\ub2e8\uc774 \uc0dd\uaca8\ub098 \uad11\uc2e0\uc801\uc778 \uc9c0\uc9c0\ub97c \ubc14\ud0d5\uc73c\ub85c \uae09\uaca9\ud558\uac8c \uc131\uc7a5\ud588\uace0, \uc778\uac04\uc744 \uc0c1\ub300\ub85c \ub054\ucc0d\ud55c \uc2e4\ud5d8\uc744 \uc790\ud589\ud558\uae30\uc5d0 \uc774\ub978\ub2e4. \uc138\ub825\uc744 \ud0a4\uc6b4 \uc548\ud0c0\ub9ac\uc6b0\uc2a4\ub294 \uc138\uacc4\ub97c \ud5a5\ud574 \uc804\uba74\uc804\uc744 \uc120\ud3ec\ud558\uc9c0\ub9cc, \ud68c\uc0ac\uc640 \uc5f0\ud569\uc740 \ub3d9\ub9f9\uc744 \ub9fa\uc5b4 \uc778\ud615\uc2e4 \ub04a\uae30 \uc791\uc804\uc744 \ud1b5\ud574 \ub178\uc778\uc744 \uc81c\uac70\ud558\uace0 \uc548\ud0c0\ub9ac\uc6b0\uc2a4\ub97c \uada4\uba78\uc2dc\ud0a8\ub2e4. \uada4\uba78 \uc774\ud6c4 \uc7a0\uae50\ub3d9\uc548 \ud3c9\ud654\uc758 \uc2dc\ub300\uac00 \ucc3e\uc544\uc654\uc73c\ub098, \ud3c9\ud654\ub294 \ub2e4\uc2dc\uae08 \uae68\uc9c0\ub294 \uae30\ubbf8\uac00 \ubcf4\uc774\uace0, \ud769\uc5b4\uc9c4 \uc548\ud0c0\ub9ac\uc6b0\uc2a4 \uc138\ub825\ub9c8\uc800 \uacf3\uacf3\uc5d0\uc11c \ubaa8\uc2b5\uc744 \ub4dc\ub7ec\ub0b8\ub2e4.", "sentence_answer": "\uc774\ub4e4\uc740 \uc0ac\uc774\ud37c(Cypher)\ub77c\uace0 \ubd88\ub9ac\uba70 \ud0c4\uc555\uc758 \ub300\uc0c1\uc774 \ub418\ub294\ub370, \uc5ec\uae30 \ubc18\ud558\uc5ec \ud5ec\ub9ac\uc624\uc2a4 \ub77c\ub294 \ud68c\uc0ac\uac00 \uc124\ub9bd\ub41c\ub2e4.", "paragraph_id": "6657185-0-1", "paragraph_question": "question: \uc0ac\uc774\ud37c\uc758 \ud0c4\uc555\uc5d0 \ubc18\ud558\uc5ec \uc124\ub9bd\ub41c \ud68c\uc0ac\uc758 \uc774\ub984\uc740?, context: 1860\ub144 \uac70\ub300\ud55c \uc77c\uc2dd \uc774\ud6c4 \uc138\uacc4 \ub3c4\ucc98\uc5d0\uc11c\ub294 \ud669\ubb34\uc9c0 \uc704\uc5d0 \ub3c4\uc2dc\uac00 \uc0dd\uaca8\ub098\uac70\ub098 \ud2b9\ubcc4\ud55c \ub2a5\ub825\uc744 \uac00\uc9c4 \uc0ac\ub78c\ub4e4\uc774 \ub098\ud0c0\ub098\uac8c \ub41c\ub2e4. \uc774\ub4e4\uc740 \uc0ac\uc774\ud37c(Cypher)\ub77c\uace0 \ubd88\ub9ac\uba70 \ud0c4\uc555\uc758 \ub300\uc0c1\uc774 \ub418\ub294\ub370, \uc5ec\uae30 \ubc18\ud558\uc5ec \ud5ec\ub9ac\uc624\uc2a4\ub77c\ub294 \ud68c\uc0ac\uac00 \uc124\ub9bd\ub41c\ub2e4. \ubfd0\ub9cc \uc544\ub2c8\ub77c \uc81c\ub3c4\uc758 \uc190\uc774 \ubbf8\uce58\uce58 \uc54a\ub294 \uacf3\uc5d0\uc11c\ub3c4 \uc2a4\uc2a4\ub85c\uc758 \uad8c\ub9ac\ub97c \ubcf4\ud638\ud558\uae30 \uc704\ud574 \uc9c0\ud558\uc5f0\ud569\uc774 \uacb0\uc131\ub41c\ub2e4. \ub450 \uc138\ub825\uc740 \uac01\uc790\uc758 \uc601\uc5ed\uc5d0\uc11c \uacf5\uc874\ud574\uc654\uc9c0\ub9cc, \uba87 \ucc28\ub840 \ub9c8\ucc30\ub3c4 \uc874\uc7ac\ud558\uc600\ub2e4. \uadf8\ub7ec\ub358 \ub3c4\uc911 \ub178\uc778\uc774 \ub9cc\ub4e0 \uc548\ud0c0\ub9ac\uc6b0\uc2a4\ub77c\ub294 \uc9d1\ub2e8\uc774 \uc0dd\uaca8\ub098 \uad11\uc2e0\uc801\uc778 \uc9c0\uc9c0\ub97c \ubc14\ud0d5\uc73c\ub85c \uae09\uaca9\ud558\uac8c \uc131\uc7a5\ud588\uace0, \uc778\uac04\uc744 \uc0c1\ub300\ub85c \ub054\ucc0d\ud55c \uc2e4\ud5d8\uc744 \uc790\ud589\ud558\uae30\uc5d0 \uc774\ub978\ub2e4. \uc138\ub825\uc744 \ud0a4\uc6b4 \uc548\ud0c0\ub9ac\uc6b0\uc2a4\ub294 \uc138\uacc4\ub97c \ud5a5\ud574 \uc804\uba74\uc804\uc744 \uc120\ud3ec\ud558\uc9c0\ub9cc, \ud68c\uc0ac\uc640 \uc5f0\ud569\uc740 \ub3d9\ub9f9\uc744 \ub9fa\uc5b4 \uc778\ud615\uc2e4 \ub04a\uae30 \uc791\uc804\uc744 \ud1b5\ud574 \ub178\uc778\uc744 \uc81c\uac70\ud558\uace0 \uc548\ud0c0\ub9ac\uc6b0\uc2a4\ub97c \uada4\uba78\uc2dc\ud0a8\ub2e4. \uada4\uba78 \uc774\ud6c4 \uc7a0\uae50\ub3d9\uc548 \ud3c9\ud654\uc758 \uc2dc\ub300\uac00 \ucc3e\uc544\uc654\uc73c\ub098, \ud3c9\ud654\ub294 \ub2e4\uc2dc\uae08 \uae68\uc9c0\ub294 \uae30\ubbf8\uac00 \ubcf4\uc774\uace0, \ud769\uc5b4\uc9c4 \uc548\ud0c0\ub9ac\uc6b0\uc2a4 \uc138\ub825\ub9c8\uc800 \uacf3\uacf3\uc5d0\uc11c \ubaa8\uc2b5\uc744 \ub4dc\ub7ec\ub0b8\ub2e4."} {"question": "\ud2f8\ub9ac\uac00 \uc774\ub044\ub294 \uc81c\uad6d\uad70\uacfc \uc2a4\uc6e8\ub374 \uad70\uc774 \ubc8c\uc778 \uc804\ud22c\uac00 \ubc8c\uc5b4\uc9c4 \uc7a5\uc18c\ub294?", "paragraph": "1631\ub144 9\uc6d4 17\uc77c \ud669\uc81c\uad70\uc758 \uba85\uc7a5 \ud558\uc778\ub9ac\ud788 \ud30c\ud39c\ud558\uc784 \uae30\ubcd1\uc7a5\uad70(1594~1632)\uc740 \uad50\ubb18\ud55c \uae30\ub3d9\uc73c\ub85c \uc2a4\uc6e8\ub374\uad70\uc744 \uc720\uc778\ud558\uc5ec \ub77c\uc774\ud504\uce58\ud788 \ubd81\ucabd \uc57d 4\ub9c8\uc77c(\uc57d6.4km)\uc758 \ube0c\ub77c\uc774\ud150\ud3a0\ud2b8 \ud3c9\uc6d0\uc5d0 \ud3ec\uc9c4\ud55c \ud2f8\ub9ac\uad70\uacfc\uc758 \uacb0\uc804\uc73c\ub85c \uad6c\uc2a4\ud0c0\ube0c \uc655\uc744 \ub04c\uc5b4\ub4e4\uc600\ub2e4. \uadf8\ub7ec\ub098 \uad6c\uc2a4\ud0c0\ube0c\ub294 \uc790\uc2e0\uc758 \uad70\uc744 \uc0c8\ub85c\uc6b4 \uad70\uc81c(\u8ecd\u5236), \uc7a5\ube44, \uc804\uc220\ub85c \ud0c8\ubc14\uafc8\uc2dc\ud0a4\uace0, \uc2e0 \uc804\uc220\uc758 \uc804\ud22c\ub300\ud615\uc73c\ub85c \ubc30\uce58\ud588\ub2e4. \uacb0\uacfc\uc801\uc73c\ub85c \uc2e0 \uc804\uc220\uc758 \uc704\ub825\uc744 \uc720\uac10\uc5c6\uc774 \ubc1c\ud718\ud574 \uad6c\uccb4\uc81c\uc758 \ud14c\ub974\uc2dc\uc624\ub85c \uc870\uc9c1\ub41c \ud2f8\ub9ac\uad70\uc5d0\uac8c \uacb0\uc815\uc801\uc778 \ud328\ubc30\ub97c \uc548\uaca8\uc8fc\uc5c8\ub2e4. \uc774 \uc804\ud22c\ub85c \uc778\ud574 \uc138\ub825\uad00\uacc4\ub294 \ub2e8\uc228\uc5d0 \uc5ed\uc804\ub418\uc5b4, \ud669\uc81c\uce21\uc774 \uc218\uc138\ub85c \ubab0\ub9ac\uac8c \ub41c \uac83\uacfc \ub354\ubd88\uc5b4, \uc774 \ube0c\ub77c\uc774\ud150\ud3a0\ud2b8 \uc804\ud22c\ub294 \uad6c\uc2a4\ud0c0\ube0c\uac00 \ub124\ub35c\ub780\ub4dc\uc758 \ub9c8\uc6b0\ub9ac\uce20 \ubaa8\ub378\uc758 \uc804\uc220\uc744 \uacc4\uc2b9 \ubc1c\uc804\uc2dc\ucf1c \ud6a1\ub300, \uae30\ubcd1, \ud3ec\ubcd1\uc758 3\ubcd1\uc804\uc220\uc758 \uc6b0\uc218\uc131\uc744 \uc2e4\uc99d\ud558\uc5ec \ubcf4\uc5ec\uc900 \uc804\ud22c\ub77c\uace0 \ud560 \uc218 \uc788\ub2e4.", "answer": "\ube0c\ub77c\uc774\ud150\ud3a0\ud2b8 \ud3c9\uc6d0", "sentence": "1631\ub144 9\uc6d4 17\uc77c \ud669\uc81c\uad70\uc758 \uba85\uc7a5 \ud558\uc778\ub9ac\ud788 \ud30c\ud39c\ud558\uc784 \uae30\ubcd1\uc7a5\uad70(1594~1632)\uc740 \uad50\ubb18\ud55c \uae30\ub3d9\uc73c\ub85c \uc2a4\uc6e8\ub374\uad70\uc744 \uc720\uc778\ud558\uc5ec \ub77c\uc774\ud504\uce58\ud788 \ubd81\ucabd \uc57d 4\ub9c8\uc77c(\uc57d6.4km)\uc758 \ube0c\ub77c\uc774\ud150\ud3a0\ud2b8 \ud3c9\uc6d0 \uc5d0 \ud3ec\uc9c4\ud55c \ud2f8\ub9ac\uad70\uacfc\uc758 \uacb0\uc804\uc73c\ub85c \uad6c\uc2a4\ud0c0\ube0c \uc655\uc744 \ub04c\uc5b4\ub4e4\uc600\ub2e4.", "paragraph_sentence": " 1631\ub144 9\uc6d4 17\uc77c \ud669\uc81c\uad70\uc758 \uba85\uc7a5 \ud558\uc778\ub9ac\ud788 \ud30c\ud39c\ud558\uc784 \uae30\ubcd1\uc7a5\uad70(1594~1632)\uc740 \uad50\ubb18\ud55c \uae30\ub3d9\uc73c\ub85c \uc2a4\uc6e8\ub374\uad70\uc744 \uc720\uc778\ud558\uc5ec \ub77c\uc774\ud504\uce58\ud788 \ubd81\ucabd \uc57d 4\ub9c8\uc77c(\uc57d6.4km)\uc758 \ube0c\ub77c\uc774\ud150\ud3a0\ud2b8 \ud3c9\uc6d0 \uc5d0 \ud3ec\uc9c4\ud55c \ud2f8\ub9ac\uad70\uacfc\uc758 \uacb0\uc804\uc73c\ub85c \uad6c\uc2a4\ud0c0\ube0c \uc655\uc744 \ub04c\uc5b4\ub4e4\uc600\ub2e4. \uadf8\ub7ec\ub098 \uad6c\uc2a4\ud0c0\ube0c\ub294 \uc790\uc2e0\uc758 \uad70\uc744 \uc0c8\ub85c\uc6b4 \uad70\uc81c(\u8ecd\u5236), \uc7a5\ube44, \uc804\uc220\ub85c \ud0c8\ubc14\uafc8\uc2dc\ud0a4\uace0, \uc2e0 \uc804\uc220\uc758 \uc804\ud22c\ub300\ud615\uc73c\ub85c \ubc30\uce58\ud588\ub2e4. \uacb0\uacfc\uc801\uc73c\ub85c \uc2e0 \uc804\uc220\uc758 \uc704\ub825\uc744 \uc720\uac10\uc5c6\uc774 \ubc1c\ud718\ud574 \uad6c\uccb4\uc81c\uc758 \ud14c\ub974\uc2dc\uc624\ub85c \uc870\uc9c1\ub41c \ud2f8\ub9ac\uad70\uc5d0\uac8c \uacb0\uc815\uc801\uc778 \ud328\ubc30\ub97c \uc548\uaca8\uc8fc\uc5c8\ub2e4. \uc774 \uc804\ud22c\ub85c \uc778\ud574 \uc138\ub825\uad00\uacc4\ub294 \ub2e8\uc228\uc5d0 \uc5ed\uc804\ub418\uc5b4, \ud669\uc81c\uce21\uc774 \uc218\uc138\ub85c \ubab0\ub9ac\uac8c \ub41c \uac83\uacfc \ub354\ubd88\uc5b4, \uc774 \ube0c\ub77c\uc774\ud150\ud3a0\ud2b8 \uc804\ud22c\ub294 \uad6c\uc2a4\ud0c0\ube0c\uac00 \ub124\ub35c\ub780\ub4dc\uc758 \ub9c8\uc6b0\ub9ac\uce20 \ubaa8\ub378\uc758 \uc804\uc220\uc744 \uacc4\uc2b9 \ubc1c\uc804\uc2dc\ucf1c \ud6a1\ub300, \uae30\ubcd1, \ud3ec\ubcd1\uc758 3\ubcd1\uc804\uc220\uc758 \uc6b0\uc218\uc131\uc744 \uc2e4\uc99d\ud558\uc5ec \ubcf4\uc5ec\uc900 \uc804\ud22c\ub77c\uace0 \ud560 \uc218 \uc788\ub2e4.", "paragraph_answer": "1631\ub144 9\uc6d4 17\uc77c \ud669\uc81c\uad70\uc758 \uba85\uc7a5 \ud558\uc778\ub9ac\ud788 \ud30c\ud39c\ud558\uc784 \uae30\ubcd1\uc7a5\uad70(1594~1632)\uc740 \uad50\ubb18\ud55c \uae30\ub3d9\uc73c\ub85c \uc2a4\uc6e8\ub374\uad70\uc744 \uc720\uc778\ud558\uc5ec \ub77c\uc774\ud504\uce58\ud788 \ubd81\ucabd \uc57d 4\ub9c8\uc77c(\uc57d6.4km)\uc758 \ube0c\ub77c\uc774\ud150\ud3a0\ud2b8 \ud3c9\uc6d0 \uc5d0 \ud3ec\uc9c4\ud55c \ud2f8\ub9ac\uad70\uacfc\uc758 \uacb0\uc804\uc73c\ub85c \uad6c\uc2a4\ud0c0\ube0c \uc655\uc744 \ub04c\uc5b4\ub4e4\uc600\ub2e4. \uadf8\ub7ec\ub098 \uad6c\uc2a4\ud0c0\ube0c\ub294 \uc790\uc2e0\uc758 \uad70\uc744 \uc0c8\ub85c\uc6b4 \uad70\uc81c(\u8ecd\u5236), \uc7a5\ube44, \uc804\uc220\ub85c \ud0c8\ubc14\uafc8\uc2dc\ud0a4\uace0, \uc2e0 \uc804\uc220\uc758 \uc804\ud22c\ub300\ud615\uc73c\ub85c \ubc30\uce58\ud588\ub2e4. \uacb0\uacfc\uc801\uc73c\ub85c \uc2e0 \uc804\uc220\uc758 \uc704\ub825\uc744 \uc720\uac10\uc5c6\uc774 \ubc1c\ud718\ud574 \uad6c\uccb4\uc81c\uc758 \ud14c\ub974\uc2dc\uc624\ub85c \uc870\uc9c1\ub41c \ud2f8\ub9ac\uad70\uc5d0\uac8c \uacb0\uc815\uc801\uc778 \ud328\ubc30\ub97c \uc548\uaca8\uc8fc\uc5c8\ub2e4. \uc774 \uc804\ud22c\ub85c \uc778\ud574 \uc138\ub825\uad00\uacc4\ub294 \ub2e8\uc228\uc5d0 \uc5ed\uc804\ub418\uc5b4, \ud669\uc81c\uce21\uc774 \uc218\uc138\ub85c \ubab0\ub9ac\uac8c \ub41c \uac83\uacfc \ub354\ubd88\uc5b4, \uc774 \ube0c\ub77c\uc774\ud150\ud3a0\ud2b8 \uc804\ud22c\ub294 \uad6c\uc2a4\ud0c0\ube0c\uac00 \ub124\ub35c\ub780\ub4dc\uc758 \ub9c8\uc6b0\ub9ac\uce20 \ubaa8\ub378\uc758 \uc804\uc220\uc744 \uacc4\uc2b9 \ubc1c\uc804\uc2dc\ucf1c \ud6a1\ub300, \uae30\ubcd1, \ud3ec\ubcd1\uc758 3\ubcd1\uc804\uc220\uc758 \uc6b0\uc218\uc131\uc744 \uc2e4\uc99d\ud558\uc5ec \ubcf4\uc5ec\uc900 \uc804\ud22c\ub77c\uace0 \ud560 \uc218 \uc788\ub2e4.", "sentence_answer": "1631\ub144 9\uc6d4 17\uc77c \ud669\uc81c\uad70\uc758 \uba85\uc7a5 \ud558\uc778\ub9ac\ud788 \ud30c\ud39c\ud558\uc784 \uae30\ubcd1\uc7a5\uad70(1594~1632)\uc740 \uad50\ubb18\ud55c \uae30\ub3d9\uc73c\ub85c \uc2a4\uc6e8\ub374\uad70\uc744 \uc720\uc778\ud558\uc5ec \ub77c\uc774\ud504\uce58\ud788 \ubd81\ucabd \uc57d 4\ub9c8\uc77c(\uc57d6.4km)\uc758 \ube0c\ub77c\uc774\ud150\ud3a0\ud2b8 \ud3c9\uc6d0 \uc5d0 \ud3ec\uc9c4\ud55c \ud2f8\ub9ac\uad70\uacfc\uc758 \uacb0\uc804\uc73c\ub85c \uad6c\uc2a4\ud0c0\ube0c \uc655\uc744 \ub04c\uc5b4\ub4e4\uc600\ub2e4.", "paragraph_id": "5782587-17-0", "paragraph_question": "question: \ud2f8\ub9ac\uac00 \uc774\ub044\ub294 \uc81c\uad6d\uad70\uacfc \uc2a4\uc6e8\ub374 \uad70\uc774 \ubc8c\uc778 \uc804\ud22c\uac00 \ubc8c\uc5b4\uc9c4 \uc7a5\uc18c\ub294?, context: 1631\ub144 9\uc6d4 17\uc77c \ud669\uc81c\uad70\uc758 \uba85\uc7a5 \ud558\uc778\ub9ac\ud788 \ud30c\ud39c\ud558\uc784 \uae30\ubcd1\uc7a5\uad70(1594~1632)\uc740 \uad50\ubb18\ud55c \uae30\ub3d9\uc73c\ub85c \uc2a4\uc6e8\ub374\uad70\uc744 \uc720\uc778\ud558\uc5ec \ub77c\uc774\ud504\uce58\ud788 \ubd81\ucabd \uc57d 4\ub9c8\uc77c(\uc57d6.4km)\uc758 \ube0c\ub77c\uc774\ud150\ud3a0\ud2b8 \ud3c9\uc6d0\uc5d0 \ud3ec\uc9c4\ud55c \ud2f8\ub9ac\uad70\uacfc\uc758 \uacb0\uc804\uc73c\ub85c \uad6c\uc2a4\ud0c0\ube0c \uc655\uc744 \ub04c\uc5b4\ub4e4\uc600\ub2e4. \uadf8\ub7ec\ub098 \uad6c\uc2a4\ud0c0\ube0c\ub294 \uc790\uc2e0\uc758 \uad70\uc744 \uc0c8\ub85c\uc6b4 \uad70\uc81c(\u8ecd\u5236), \uc7a5\ube44, \uc804\uc220\ub85c \ud0c8\ubc14\uafc8\uc2dc\ud0a4\uace0, \uc2e0 \uc804\uc220\uc758 \uc804\ud22c\ub300\ud615\uc73c\ub85c \ubc30\uce58\ud588\ub2e4. \uacb0\uacfc\uc801\uc73c\ub85c \uc2e0 \uc804\uc220\uc758 \uc704\ub825\uc744 \uc720\uac10\uc5c6\uc774 \ubc1c\ud718\ud574 \uad6c\uccb4\uc81c\uc758 \ud14c\ub974\uc2dc\uc624\ub85c \uc870\uc9c1\ub41c \ud2f8\ub9ac\uad70\uc5d0\uac8c \uacb0\uc815\uc801\uc778 \ud328\ubc30\ub97c \uc548\uaca8\uc8fc\uc5c8\ub2e4. \uc774 \uc804\ud22c\ub85c \uc778\ud574 \uc138\ub825\uad00\uacc4\ub294 \ub2e8\uc228\uc5d0 \uc5ed\uc804\ub418\uc5b4, \ud669\uc81c\uce21\uc774 \uc218\uc138\ub85c \ubab0\ub9ac\uac8c \ub41c \uac83\uacfc \ub354\ubd88\uc5b4, \uc774 \ube0c\ub77c\uc774\ud150\ud3a0\ud2b8 \uc804\ud22c\ub294 \uad6c\uc2a4\ud0c0\ube0c\uac00 \ub124\ub35c\ub780\ub4dc\uc758 \ub9c8\uc6b0\ub9ac\uce20 \ubaa8\ub378\uc758 \uc804\uc220\uc744 \uacc4\uc2b9 \ubc1c\uc804\uc2dc\ucf1c \ud6a1\ub300, \uae30\ubcd1, \ud3ec\ubcd1\uc758 3\ubcd1\uc804\uc220\uc758 \uc6b0\uc218\uc131\uc744 \uc2e4\uc99d\ud558\uc5ec \ubcf4\uc5ec\uc900 \uc804\ud22c\ub77c\uace0 \ud560 \uc218 \uc788\ub2e4."} {"question": "\ucd0c\ub77d\uc758 \uc5d8\ub9ac\ud2b8 \uacc4\uce35 \ub2e4\uc218\uac00 \ubc29\ud574\ud558\uae30 \uc2dc\uc791\ud55c \uae30\uad00\uc740?", "paragraph": "\uc804\uc6d0 \uacf5\ub3d9\uccb4, \ud2b9\ud788 \uadf8 \uc911\uc5d0\uc11c\ub3c4 \uc789\uae00\ub79c\ub4dc \ub3d9\ub0a8\ubd80\ub294 \ub18d\ub178\uc81c\uc758 \uc791\ub3d9\uacfc \uc9c0\uc5ed \uc7a5\uc6d0 \uc7ac\ud310\uc18c\uc758 \uc138\uae08 \uc9d5\uc218\ub97c \ubd88\ub9cc\uc2a4\ub7ec\uc6cc\ud588\ub2e4. \ud2b9\ud788 \uc7ac\ud310\uc18c\ub97c \uc6b4\uc601\ud558\ub294 \uc601\uc8fc\ub4e4\uc774 \uace7 \uc6d0\uc131\uc744 \uc0ac\ub294 \ub178\ub3d9 \uc870\ub840\ub098 \uc655\uc2e4\uc5d0\uc11c \uc81c\uc815\ud55c \ubc95\uc758 \ud604\uc7a5 \uc9d1\ud589\uc790\ub85c \uae30\ub2a5\ud558\ub294 \uacbd\uc6b0\uac00 \uc7a6\uc558\uae30 \ub54c\ubb38\uc5d0 \ubd88\ub9cc\uc740 \uac00\uc911\ub418\uc5c8\ub2e4. \ucd0c\ub77d\uc758 \uc5d8\ub9ac\ud2b8 \uacc4\uce35 \ub2e4\uc218\ub294 \uc9c0\ubc29\uc815\ubd80\uc758 \uc5ed\ud560\uc744 \ub9e1\ub294 \uac83\uc744 \uac70\ubd80\ud588\uace0, \uc7ac\ud310\uc18c\uc758 \ud65c\ub3d9\uc744 \ubc29\ud574\ud558\uae30 \uc2dc\uc791\ud588\ub2e4. \uc7ac\ud310\uc18c\uc5d0 \uc758\ud574 \ubab0\uc218\ub41c \uac00\ucd95\ub4e4\uc740 \uadf8 \uc8fc\uc778\ub4e4\uc5d0 \uc758\ud574 \u2018\ud574\ubc29\u2019\ub418\uc5c8\uace0, \ubc95\uad00\ub4e4\uc740 \ud3ed\ud589\uc744 \ub2f9\ud588\ub2e4. \uc77c\ubd80\ub294 \uc804\ud1b5\uc801\uc778 \ubc95\uc744 \uc874\uc911\ud558\ub098 \ub7f0\ub358\uc5d0\uc11c \ub0b4\ub824\uc624\ub294 \uc99d\uc624\ubc1b\ub294 \uc911\uc559 \ubc95\ub960\uc5d0\uc11c\ub294 \ubd84\ub9ac\ub41c, \ub3c5\ub9bd\uc801\uc778 \ucd0c\ub77d \uacf5\ub3d9\uccb4\uc758 \ud0c4\uc0dd\uc744 \uc9c0\uc9c0\ud558\uae30 \uc2dc\uc791\ud588\ub2e4. \uc0ac\ud559\uc790 \ubbf8\ub9ac \ub8e8\ube48\uc740 \uc774 \uc0c1\ud669\uc744, \u201c\ubb38\uc81c\ub294 \ub098\ub78f\ubc95 \uadf8 \uc790\uccb4\ub77c\uae30\ubcf4\ub2e4, \uadf8 \ubc95\uc744 \uc801\uc6a9\ud558\uace0 \uc639\ud638\ud558\ub294 \ub370 \uc788\uc5c8\ub2e4\u201d\uace0 \ubb18\uc0ac\ud55c\ub2e4.", "answer": "\uc7ac\ud310\uc18c", "sentence": "\uc804\uc6d0 \uacf5\ub3d9\uccb4, \ud2b9\ud788 \uadf8 \uc911\uc5d0\uc11c\ub3c4 \uc789\uae00\ub79c\ub4dc \ub3d9\ub0a8\ubd80\ub294 \ub18d\ub178\uc81c\uc758 \uc791\ub3d9\uacfc \uc9c0\uc5ed \uc7a5\uc6d0 \uc7ac\ud310\uc18c \uc758 \uc138\uae08 \uc9d5\uc218\ub97c \ubd88\ub9cc\uc2a4\ub7ec\uc6cc\ud588\ub2e4.", "paragraph_sentence": " \uc804\uc6d0 \uacf5\ub3d9\uccb4, \ud2b9\ud788 \uadf8 \uc911\uc5d0\uc11c\ub3c4 \uc789\uae00\ub79c\ub4dc \ub3d9\ub0a8\ubd80\ub294 \ub18d\ub178\uc81c\uc758 \uc791\ub3d9\uacfc \uc9c0\uc5ed \uc7a5\uc6d0 \uc7ac\ud310\uc18c \uc758 \uc138\uae08 \uc9d5\uc218\ub97c \ubd88\ub9cc\uc2a4\ub7ec\uc6cc\ud588\ub2e4. \ud2b9\ud788 \uc7ac\ud310\uc18c\ub97c \uc6b4\uc601\ud558\ub294 \uc601\uc8fc\ub4e4\uc774 \uace7 \uc6d0\uc131\uc744 \uc0ac\ub294 \ub178\ub3d9 \uc870\ub840\ub098 \uc655\uc2e4\uc5d0\uc11c \uc81c\uc815\ud55c \ubc95\uc758 \ud604\uc7a5 \uc9d1\ud589\uc790\ub85c \uae30\ub2a5\ud558\ub294 \uacbd\uc6b0\uac00 \uc7a6\uc558\uae30 \ub54c\ubb38\uc5d0 \ubd88\ub9cc\uc740 \uac00\uc911\ub418\uc5c8\ub2e4. \ucd0c\ub77d\uc758 \uc5d8\ub9ac\ud2b8 \uacc4\uce35 \ub2e4\uc218\ub294 \uc9c0\ubc29\uc815\ubd80\uc758 \uc5ed\ud560\uc744 \ub9e1\ub294 \uac83\uc744 \uac70\ubd80\ud588\uace0, \uc7ac\ud310\uc18c\uc758 \ud65c\ub3d9\uc744 \ubc29\ud574\ud558\uae30 \uc2dc\uc791\ud588\ub2e4. \uc7ac\ud310\uc18c\uc5d0 \uc758\ud574 \ubab0\uc218\ub41c \uac00\ucd95\ub4e4\uc740 \uadf8 \uc8fc\uc778\ub4e4\uc5d0 \uc758\ud574 \u2018\ud574\ubc29\u2019\ub418\uc5c8\uace0, \ubc95\uad00\ub4e4\uc740 \ud3ed\ud589\uc744 \ub2f9\ud588\ub2e4. \uc77c\ubd80\ub294 \uc804\ud1b5\uc801\uc778 \ubc95\uc744 \uc874\uc911\ud558\ub098 \ub7f0\ub358\uc5d0\uc11c \ub0b4\ub824\uc624\ub294 \uc99d\uc624\ubc1b\ub294 \uc911\uc559 \ubc95\ub960\uc5d0\uc11c\ub294 \ubd84\ub9ac\ub41c, \ub3c5\ub9bd\uc801\uc778 \ucd0c\ub77d \uacf5\ub3d9\uccb4\uc758 \ud0c4\uc0dd\uc744 \uc9c0\uc9c0\ud558\uae30 \uc2dc\uc791\ud588\ub2e4. \uc0ac\ud559\uc790 \ubbf8\ub9ac \ub8e8\ube48\uc740 \uc774 \uc0c1\ud669\uc744, \u201c\ubb38\uc81c\ub294 \ub098\ub78f\ubc95 \uadf8 \uc790\uccb4\ub77c\uae30\ubcf4\ub2e4, \uadf8 \ubc95\uc744 \uc801\uc6a9\ud558\uace0 \uc639\ud638\ud558\ub294 \ub370 \uc788\uc5c8\ub2e4\u201d\uace0 \ubb18\uc0ac\ud55c\ub2e4.", "paragraph_answer": "\uc804\uc6d0 \uacf5\ub3d9\uccb4, \ud2b9\ud788 \uadf8 \uc911\uc5d0\uc11c\ub3c4 \uc789\uae00\ub79c\ub4dc \ub3d9\ub0a8\ubd80\ub294 \ub18d\ub178\uc81c\uc758 \uc791\ub3d9\uacfc \uc9c0\uc5ed \uc7a5\uc6d0 \uc7ac\ud310\uc18c \uc758 \uc138\uae08 \uc9d5\uc218\ub97c \ubd88\ub9cc\uc2a4\ub7ec\uc6cc\ud588\ub2e4. \ud2b9\ud788 \uc7ac\ud310\uc18c\ub97c \uc6b4\uc601\ud558\ub294 \uc601\uc8fc\ub4e4\uc774 \uace7 \uc6d0\uc131\uc744 \uc0ac\ub294 \ub178\ub3d9 \uc870\ub840\ub098 \uc655\uc2e4\uc5d0\uc11c \uc81c\uc815\ud55c \ubc95\uc758 \ud604\uc7a5 \uc9d1\ud589\uc790\ub85c \uae30\ub2a5\ud558\ub294 \uacbd\uc6b0\uac00 \uc7a6\uc558\uae30 \ub54c\ubb38\uc5d0 \ubd88\ub9cc\uc740 \uac00\uc911\ub418\uc5c8\ub2e4. \ucd0c\ub77d\uc758 \uc5d8\ub9ac\ud2b8 \uacc4\uce35 \ub2e4\uc218\ub294 \uc9c0\ubc29\uc815\ubd80\uc758 \uc5ed\ud560\uc744 \ub9e1\ub294 \uac83\uc744 \uac70\ubd80\ud588\uace0, \uc7ac\ud310\uc18c\uc758 \ud65c\ub3d9\uc744 \ubc29\ud574\ud558\uae30 \uc2dc\uc791\ud588\ub2e4. \uc7ac\ud310\uc18c\uc5d0 \uc758\ud574 \ubab0\uc218\ub41c \uac00\ucd95\ub4e4\uc740 \uadf8 \uc8fc\uc778\ub4e4\uc5d0 \uc758\ud574 \u2018\ud574\ubc29\u2019\ub418\uc5c8\uace0, \ubc95\uad00\ub4e4\uc740 \ud3ed\ud589\uc744 \ub2f9\ud588\ub2e4. \uc77c\ubd80\ub294 \uc804\ud1b5\uc801\uc778 \ubc95\uc744 \uc874\uc911\ud558\ub098 \ub7f0\ub358\uc5d0\uc11c \ub0b4\ub824\uc624\ub294 \uc99d\uc624\ubc1b\ub294 \uc911\uc559 \ubc95\ub960\uc5d0\uc11c\ub294 \ubd84\ub9ac\ub41c, \ub3c5\ub9bd\uc801\uc778 \ucd0c\ub77d \uacf5\ub3d9\uccb4\uc758 \ud0c4\uc0dd\uc744 \uc9c0\uc9c0\ud558\uae30 \uc2dc\uc791\ud588\ub2e4. \uc0ac\ud559\uc790 \ubbf8\ub9ac \ub8e8\ube48\uc740 \uc774 \uc0c1\ud669\uc744, \u201c\ubb38\uc81c\ub294 \ub098\ub78f\ubc95 \uadf8 \uc790\uccb4\ub77c\uae30\ubcf4\ub2e4, \uadf8 \ubc95\uc744 \uc801\uc6a9\ud558\uace0 \uc639\ud638\ud558\ub294 \ub370 \uc788\uc5c8\ub2e4\u201d\uace0 \ubb18\uc0ac\ud55c\ub2e4.", "sentence_answer": "\uc804\uc6d0 \uacf5\ub3d9\uccb4, \ud2b9\ud788 \uadf8 \uc911\uc5d0\uc11c\ub3c4 \uc789\uae00\ub79c\ub4dc \ub3d9\ub0a8\ubd80\ub294 \ub18d\ub178\uc81c\uc758 \uc791\ub3d9\uacfc \uc9c0\uc5ed \uc7a5\uc6d0 \uc7ac\ud310\uc18c \uc758 \uc138\uae08 \uc9d5\uc218\ub97c \ubd88\ub9cc\uc2a4\ub7ec\uc6cc\ud588\ub2e4.", "paragraph_id": "6541477-12-1", "paragraph_question": "question: \ucd0c\ub77d\uc758 \uc5d8\ub9ac\ud2b8 \uacc4\uce35 \ub2e4\uc218\uac00 \ubc29\ud574\ud558\uae30 \uc2dc\uc791\ud55c \uae30\uad00\uc740?, context: \uc804\uc6d0 \uacf5\ub3d9\uccb4, \ud2b9\ud788 \uadf8 \uc911\uc5d0\uc11c\ub3c4 \uc789\uae00\ub79c\ub4dc \ub3d9\ub0a8\ubd80\ub294 \ub18d\ub178\uc81c\uc758 \uc791\ub3d9\uacfc \uc9c0\uc5ed \uc7a5\uc6d0 \uc7ac\ud310\uc18c\uc758 \uc138\uae08 \uc9d5\uc218\ub97c \ubd88\ub9cc\uc2a4\ub7ec\uc6cc\ud588\ub2e4. \ud2b9\ud788 \uc7ac\ud310\uc18c\ub97c \uc6b4\uc601\ud558\ub294 \uc601\uc8fc\ub4e4\uc774 \uace7 \uc6d0\uc131\uc744 \uc0ac\ub294 \ub178\ub3d9 \uc870\ub840\ub098 \uc655\uc2e4\uc5d0\uc11c \uc81c\uc815\ud55c \ubc95\uc758 \ud604\uc7a5 \uc9d1\ud589\uc790\ub85c \uae30\ub2a5\ud558\ub294 \uacbd\uc6b0\uac00 \uc7a6\uc558\uae30 \ub54c\ubb38\uc5d0 \ubd88\ub9cc\uc740 \uac00\uc911\ub418\uc5c8\ub2e4. \ucd0c\ub77d\uc758 \uc5d8\ub9ac\ud2b8 \uacc4\uce35 \ub2e4\uc218\ub294 \uc9c0\ubc29\uc815\ubd80\uc758 \uc5ed\ud560\uc744 \ub9e1\ub294 \uac83\uc744 \uac70\ubd80\ud588\uace0, \uc7ac\ud310\uc18c\uc758 \ud65c\ub3d9\uc744 \ubc29\ud574\ud558\uae30 \uc2dc\uc791\ud588\ub2e4. \uc7ac\ud310\uc18c\uc5d0 \uc758\ud574 \ubab0\uc218\ub41c \uac00\ucd95\ub4e4\uc740 \uadf8 \uc8fc\uc778\ub4e4\uc5d0 \uc758\ud574 \u2018\ud574\ubc29\u2019\ub418\uc5c8\uace0, \ubc95\uad00\ub4e4\uc740 \ud3ed\ud589\uc744 \ub2f9\ud588\ub2e4. \uc77c\ubd80\ub294 \uc804\ud1b5\uc801\uc778 \ubc95\uc744 \uc874\uc911\ud558\ub098 \ub7f0\ub358\uc5d0\uc11c \ub0b4\ub824\uc624\ub294 \uc99d\uc624\ubc1b\ub294 \uc911\uc559 \ubc95\ub960\uc5d0\uc11c\ub294 \ubd84\ub9ac\ub41c, \ub3c5\ub9bd\uc801\uc778 \ucd0c\ub77d \uacf5\ub3d9\uccb4\uc758 \ud0c4\uc0dd\uc744 \uc9c0\uc9c0\ud558\uae30 \uc2dc\uc791\ud588\ub2e4. \uc0ac\ud559\uc790 \ubbf8\ub9ac \ub8e8\ube48\uc740 \uc774 \uc0c1\ud669\uc744, \u201c\ubb38\uc81c\ub294 \ub098\ub78f\ubc95 \uadf8 \uc790\uccb4\ub77c\uae30\ubcf4\ub2e4, \uadf8 \ubc95\uc744 \uc801\uc6a9\ud558\uace0 \uc639\ud638\ud558\ub294 \ub370 \uc788\uc5c8\ub2e4\u201d\uace0 \ubb18\uc0ac\ud55c\ub2e4."} {"question": "\uc624\ub514\uc158 \ud504\ub85c\uadf8\ub7a8 \uc2a4\ud0c0 \uc11c\ud074 \ud018\uc2a4\ud2b8\uc5d0\uc11c\uc758 \uacbd\ud5d8\uc744 \ub2f4\uc740 \ubc15\uc0b0\ub2e4\ub77c\uc758 \ub304\uc2a4\uace1\uc740?", "paragraph": "\uace0\ub4f1\ud559\uad50 \ud559\uad50 \ud589\uc0ac\uc5d0\uc11c \uc5f0\uc608\uacc4\uc5d0\uc11c \ud65c\ub3d9\ud558\ub358 \uce5c\uad6c\uac00 \ud544\ub9ac\ud540 \ubc29\uc1a1\uad6d ABS-CBN\uc758 \"\uc2a4\ud0c0 \uc11c\ud074 \ud018\uc2a4\ud2b8\"\ub77c\uace0 \ud558\ub294 \uc11c\ubc14\uc774\ubc8c \uc624\ub514\uc158\uc5d0 \uc751\uc2dc\ud574 \ubcfc \uac83\uc744 \uad8c\ud558\uc5ec \uadf8\ub140\ub294 \uc624\ub514\uc158\uc5d0 \uc751\uc2dc\ud558\uac8c \ub41c\ub2e4. \ucd5c\uc885 5\uba85\uc774 \ud655\uc815\ub418\ub294 \ub9c8\uc9c0\ub9c9 \ud0c8\ub77d\uc790 \uc120\ubc1c\uc804\uc5d0\uc11c \uc57d 50\ub9cc \ud1b5\uc758 \ud734\ub300\uc804\ud654 \ubb38\uc790 \ub4dd\ud45c\ub97c \uae30\ub85d\ud558\uc600\uace0, \uc774\ub294 \uc5ec\uc131 \ucd9c\uc5f0\uc790 \uc0ac\uc0c1 \ucd5c\ub300\uc758 \uae30\ub85d\uc774\uc5c8\ub2e4. \ucc38\uac00 \uc774\ud6c4 \ud55c\uad6d \ub4dc\ub77c\ub9c8\ub97c \uc18c\uac1c\ud558\ub294\u300a\uc0b0\ub2e4\ub77c\uc758 \ub85c\ub9e8\uc2a4\u300b\ub97c \ube44\ub86f\ud55c \uc5ec\ub7ec \ub2e4\uc591\ud55c TV \ud504\ub85c\uadf8\ub7a8\uc5d0 \ucd9c\uc5f0\ud558\uc600\ub2e4. \ub610\ud55c, \uadf8\ub140\uc758 \uc774\ub984\uc774 \ubd99\uc5ec\uc9c4 \uc568\ubc94 \"Sandara\"\ub97c \ubc1c\ub9e4\ud558\uba70 \uac00\uc218 \ud65c\ub3d9\ub3c4 \uacb8\ud558\uac8c \ub418\uc5c8\ub2e4. \uc2e0\uc120\ud55c \ub304\uc2a4\uace1\uc73c\ub85c \ud788\ud2b8\ud55c \"In Or Out\"\uc740 \uc2a4\ud0c0 \uc11c\ud074 \ud018\uc2a4\ud2b8\uc5d0\uc11c\uc758 \uacbd\ud5d8\uc744 \ub2f4\uc740 \uace1\uc774\ub2e4. \ubc15\uc0b0\ub2e4\ub77c\uc758 \uc778\uae30\ub97c \uc99d\uba85\ud558\ub4ef, \uc568\ubc94\uc740 6\uac1c\uc6d4 \ud6c4 \ub354\ube14 \ud50c\ub798\ud2f0\ub118\uc73c\ub85c \uc778\uc815\ubc1b\uc558\ub2e4. \uc774 \uc678\uc5d0\ub3c4 4\ud3b8\uc758 \uc601\ud654\uc5d0 \ucd9c\uc5f0\ud558\uc5ec \ud604\uc9c0\uc758 \uc80a\uc740 \ubc30\uc6b0\ub4e4\uacfc \uc5f4\uc5f0\uc744 \ud3bc\uce58\ub294 \ub4f1 \ub2e4\uc591\ud55c \uc601\uc5ed\uc5d0\uc11c \ud65c\ubc1c\ud558\uac8c \ud65c\ub3d9\ud558\uba70 \uc2a4\ud0c0\ub85c\uc11c\uc758 \uc785\uc9c0\ub97c \ud655\uace0\ud558\uac8c \uc313\uc544 \ub098\uac14\ub2e4.", "answer": "In Or Out", "sentence": "\uc2e0\uc120\ud55c \ub304\uc2a4\uace1\uc73c\ub85c \ud788\ud2b8\ud55c \" In Or Out \"\uc740 \uc2a4\ud0c0 \uc11c\ud074 \ud018\uc2a4\ud2b8\uc5d0\uc11c\uc758 \uacbd\ud5d8\uc744 \ub2f4\uc740 \uace1\uc774\ub2e4.", "paragraph_sentence": "\uace0\ub4f1\ud559\uad50 \ud559\uad50 \ud589\uc0ac\uc5d0\uc11c \uc5f0\uc608\uacc4\uc5d0\uc11c \ud65c\ub3d9\ud558\ub358 \uce5c\uad6c\uac00 \ud544\ub9ac\ud540 \ubc29\uc1a1\uad6d ABS-CBN\uc758 \"\uc2a4\ud0c0 \uc11c\ud074 \ud018\uc2a4\ud2b8\"\ub77c\uace0 \ud558\ub294 \uc11c\ubc14\uc774\ubc8c \uc624\ub514\uc158\uc5d0 \uc751\uc2dc\ud574 \ubcfc \uac83\uc744 \uad8c\ud558\uc5ec \uadf8\ub140\ub294 \uc624\ub514\uc158\uc5d0 \uc751\uc2dc\ud558\uac8c \ub41c\ub2e4. \ucd5c\uc885 5\uba85\uc774 \ud655\uc815\ub418\ub294 \ub9c8\uc9c0\ub9c9 \ud0c8\ub77d\uc790 \uc120\ubc1c\uc804\uc5d0\uc11c \uc57d 50\ub9cc \ud1b5\uc758 \ud734\ub300\uc804\ud654 \ubb38\uc790 \ub4dd\ud45c\ub97c \uae30\ub85d\ud558\uc600\uace0, \uc774\ub294 \uc5ec\uc131 \ucd9c\uc5f0\uc790 \uc0ac\uc0c1 \ucd5c\ub300\uc758 \uae30\ub85d\uc774\uc5c8\ub2e4. \ucc38\uac00 \uc774\ud6c4 \ud55c\uad6d \ub4dc\ub77c\ub9c8\ub97c \uc18c\uac1c\ud558\ub294\u300a\uc0b0\ub2e4\ub77c\uc758 \ub85c\ub9e8\uc2a4\u300b\ub97c \ube44\ub86f\ud55c \uc5ec\ub7ec \ub2e4\uc591\ud55c TV \ud504\ub85c\uadf8\ub7a8\uc5d0 \ucd9c\uc5f0\ud558\uc600\ub2e4. \ub610\ud55c, \uadf8\ub140\uc758 \uc774\ub984\uc774 \ubd99\uc5ec\uc9c4 \uc568\ubc94 \"Sandara\"\ub97c \ubc1c\ub9e4\ud558\uba70 \uac00\uc218 \ud65c\ub3d9\ub3c4 \uacb8\ud558\uac8c \ub418\uc5c8\ub2e4. \uc2e0\uc120\ud55c \ub304\uc2a4\uace1\uc73c\ub85c \ud788\ud2b8\ud55c \" In Or Out \"\uc740 \uc2a4\ud0c0 \uc11c\ud074 \ud018\uc2a4\ud2b8\uc5d0\uc11c\uc758 \uacbd\ud5d8\uc744 \ub2f4\uc740 \uace1\uc774\ub2e4. \ubc15\uc0b0\ub2e4\ub77c\uc758 \uc778\uae30\ub97c \uc99d\uba85\ud558\ub4ef, \uc568\ubc94\uc740 6\uac1c\uc6d4 \ud6c4 \ub354\ube14 \ud50c\ub798\ud2f0\ub118\uc73c\ub85c \uc778\uc815\ubc1b\uc558\ub2e4. \uc774 \uc678\uc5d0\ub3c4 4\ud3b8\uc758 \uc601\ud654\uc5d0 \ucd9c\uc5f0\ud558\uc5ec \ud604\uc9c0\uc758 \uc80a\uc740 \ubc30\uc6b0\ub4e4\uacfc \uc5f4\uc5f0\uc744 \ud3bc\uce58\ub294 \ub4f1 \ub2e4\uc591\ud55c \uc601\uc5ed\uc5d0\uc11c \ud65c\ubc1c\ud558\uac8c \ud65c\ub3d9\ud558\uba70 \uc2a4\ud0c0\ub85c\uc11c\uc758 \uc785\uc9c0\ub97c \ud655\uace0\ud558\uac8c \uc313\uc544 \ub098\uac14\ub2e4.", "paragraph_answer": "\uace0\ub4f1\ud559\uad50 \ud559\uad50 \ud589\uc0ac\uc5d0\uc11c \uc5f0\uc608\uacc4\uc5d0\uc11c \ud65c\ub3d9\ud558\ub358 \uce5c\uad6c\uac00 \ud544\ub9ac\ud540 \ubc29\uc1a1\uad6d ABS-CBN\uc758 \"\uc2a4\ud0c0 \uc11c\ud074 \ud018\uc2a4\ud2b8\"\ub77c\uace0 \ud558\ub294 \uc11c\ubc14\uc774\ubc8c \uc624\ub514\uc158\uc5d0 \uc751\uc2dc\ud574 \ubcfc \uac83\uc744 \uad8c\ud558\uc5ec \uadf8\ub140\ub294 \uc624\ub514\uc158\uc5d0 \uc751\uc2dc\ud558\uac8c \ub41c\ub2e4. \ucd5c\uc885 5\uba85\uc774 \ud655\uc815\ub418\ub294 \ub9c8\uc9c0\ub9c9 \ud0c8\ub77d\uc790 \uc120\ubc1c\uc804\uc5d0\uc11c \uc57d 50\ub9cc \ud1b5\uc758 \ud734\ub300\uc804\ud654 \ubb38\uc790 \ub4dd\ud45c\ub97c \uae30\ub85d\ud558\uc600\uace0, \uc774\ub294 \uc5ec\uc131 \ucd9c\uc5f0\uc790 \uc0ac\uc0c1 \ucd5c\ub300\uc758 \uae30\ub85d\uc774\uc5c8\ub2e4. \ucc38\uac00 \uc774\ud6c4 \ud55c\uad6d \ub4dc\ub77c\ub9c8\ub97c \uc18c\uac1c\ud558\ub294\u300a\uc0b0\ub2e4\ub77c\uc758 \ub85c\ub9e8\uc2a4\u300b\ub97c \ube44\ub86f\ud55c \uc5ec\ub7ec \ub2e4\uc591\ud55c TV \ud504\ub85c\uadf8\ub7a8\uc5d0 \ucd9c\uc5f0\ud558\uc600\ub2e4. \ub610\ud55c, \uadf8\ub140\uc758 \uc774\ub984\uc774 \ubd99\uc5ec\uc9c4 \uc568\ubc94 \"Sandara\"\ub97c \ubc1c\ub9e4\ud558\uba70 \uac00\uc218 \ud65c\ub3d9\ub3c4 \uacb8\ud558\uac8c \ub418\uc5c8\ub2e4. \uc2e0\uc120\ud55c \ub304\uc2a4\uace1\uc73c\ub85c \ud788\ud2b8\ud55c \" In Or Out \"\uc740 \uc2a4\ud0c0 \uc11c\ud074 \ud018\uc2a4\ud2b8\uc5d0\uc11c\uc758 \uacbd\ud5d8\uc744 \ub2f4\uc740 \uace1\uc774\ub2e4. \ubc15\uc0b0\ub2e4\ub77c\uc758 \uc778\uae30\ub97c \uc99d\uba85\ud558\ub4ef, \uc568\ubc94\uc740 6\uac1c\uc6d4 \ud6c4 \ub354\ube14 \ud50c\ub798\ud2f0\ub118\uc73c\ub85c \uc778\uc815\ubc1b\uc558\ub2e4. \uc774 \uc678\uc5d0\ub3c4 4\ud3b8\uc758 \uc601\ud654\uc5d0 \ucd9c\uc5f0\ud558\uc5ec \ud604\uc9c0\uc758 \uc80a\uc740 \ubc30\uc6b0\ub4e4\uacfc \uc5f4\uc5f0\uc744 \ud3bc\uce58\ub294 \ub4f1 \ub2e4\uc591\ud55c \uc601\uc5ed\uc5d0\uc11c \ud65c\ubc1c\ud558\uac8c \ud65c\ub3d9\ud558\uba70 \uc2a4\ud0c0\ub85c\uc11c\uc758 \uc785\uc9c0\ub97c \ud655\uace0\ud558\uac8c \uc313\uc544 \ub098\uac14\ub2e4.", "sentence_answer": "\uc2e0\uc120\ud55c \ub304\uc2a4\uace1\uc73c\ub85c \ud788\ud2b8\ud55c \" In Or Out \"\uc740 \uc2a4\ud0c0 \uc11c\ud074 \ud018\uc2a4\ud2b8\uc5d0\uc11c\uc758 \uacbd\ud5d8\uc744 \ub2f4\uc740 \uace1\uc774\ub2e4.", "paragraph_id": "6507456-0-2", "paragraph_question": "question: \uc624\ub514\uc158 \ud504\ub85c\uadf8\ub7a8 \uc2a4\ud0c0 \uc11c\ud074 \ud018\uc2a4\ud2b8\uc5d0\uc11c\uc758 \uacbd\ud5d8\uc744 \ub2f4\uc740 \ubc15\uc0b0\ub2e4\ub77c\uc758 \ub304\uc2a4\uace1\uc740?, context: \uace0\ub4f1\ud559\uad50 \ud559\uad50 \ud589\uc0ac\uc5d0\uc11c \uc5f0\uc608\uacc4\uc5d0\uc11c \ud65c\ub3d9\ud558\ub358 \uce5c\uad6c\uac00 \ud544\ub9ac\ud540 \ubc29\uc1a1\uad6d ABS-CBN\uc758 \"\uc2a4\ud0c0 \uc11c\ud074 \ud018\uc2a4\ud2b8\"\ub77c\uace0 \ud558\ub294 \uc11c\ubc14\uc774\ubc8c \uc624\ub514\uc158\uc5d0 \uc751\uc2dc\ud574 \ubcfc \uac83\uc744 \uad8c\ud558\uc5ec \uadf8\ub140\ub294 \uc624\ub514\uc158\uc5d0 \uc751\uc2dc\ud558\uac8c \ub41c\ub2e4. \ucd5c\uc885 5\uba85\uc774 \ud655\uc815\ub418\ub294 \ub9c8\uc9c0\ub9c9 \ud0c8\ub77d\uc790 \uc120\ubc1c\uc804\uc5d0\uc11c \uc57d 50\ub9cc \ud1b5\uc758 \ud734\ub300\uc804\ud654 \ubb38\uc790 \ub4dd\ud45c\ub97c \uae30\ub85d\ud558\uc600\uace0, \uc774\ub294 \uc5ec\uc131 \ucd9c\uc5f0\uc790 \uc0ac\uc0c1 \ucd5c\ub300\uc758 \uae30\ub85d\uc774\uc5c8\ub2e4. \ucc38\uac00 \uc774\ud6c4 \ud55c\uad6d \ub4dc\ub77c\ub9c8\ub97c \uc18c\uac1c\ud558\ub294\u300a\uc0b0\ub2e4\ub77c\uc758 \ub85c\ub9e8\uc2a4\u300b\ub97c \ube44\ub86f\ud55c \uc5ec\ub7ec \ub2e4\uc591\ud55c TV \ud504\ub85c\uadf8\ub7a8\uc5d0 \ucd9c\uc5f0\ud558\uc600\ub2e4. \ub610\ud55c, \uadf8\ub140\uc758 \uc774\ub984\uc774 \ubd99\uc5ec\uc9c4 \uc568\ubc94 \"Sandara\"\ub97c \ubc1c\ub9e4\ud558\uba70 \uac00\uc218 \ud65c\ub3d9\ub3c4 \uacb8\ud558\uac8c \ub418\uc5c8\ub2e4. \uc2e0\uc120\ud55c \ub304\uc2a4\uace1\uc73c\ub85c \ud788\ud2b8\ud55c \"In Or Out\"\uc740 \uc2a4\ud0c0 \uc11c\ud074 \ud018\uc2a4\ud2b8\uc5d0\uc11c\uc758 \uacbd\ud5d8\uc744 \ub2f4\uc740 \uace1\uc774\ub2e4. \ubc15\uc0b0\ub2e4\ub77c\uc758 \uc778\uae30\ub97c \uc99d\uba85\ud558\ub4ef, \uc568\ubc94\uc740 6\uac1c\uc6d4 \ud6c4 \ub354\ube14 \ud50c\ub798\ud2f0\ub118\uc73c\ub85c \uc778\uc815\ubc1b\uc558\ub2e4. \uc774 \uc678\uc5d0\ub3c4 4\ud3b8\uc758 \uc601\ud654\uc5d0 \ucd9c\uc5f0\ud558\uc5ec \ud604\uc9c0\uc758 \uc80a\uc740 \ubc30\uc6b0\ub4e4\uacfc \uc5f4\uc5f0\uc744 \ud3bc\uce58\ub294 \ub4f1 \ub2e4\uc591\ud55c \uc601\uc5ed\uc5d0\uc11c \ud65c\ubc1c\ud558\uac8c \ud65c\ub3d9\ud558\uba70 \uc2a4\ud0c0\ub85c\uc11c\uc758 \uc785\uc9c0\ub97c \ud655\uace0\ud558\uac8c \uc313\uc544 \ub098\uac14\ub2e4."} {"question": "1997\ub144 \ub300\uc120 \ub2f9\uc2dc \uc0c8\uc815\uce58\uad6d\ubbfc\ud68c\uc758\ub294 \uc5b4\ub5a4 \uc815\ub2f9\uacfc \uc5f0\ub300\ud588\ub294\uac00?", "paragraph": "\uadf8\ub7ec\ub098 \uc77c\ubd80 \ud1b5\uc77c\ubbfc\uc8fc\ub2f9 \uc758\uc6d0\ub4e4\uc740 \ub3c5\uc790 \ud589\ubcf4\ub97c \ub0b4\uac78\uace0 \uaf2c\ub9c8\ubbfc\uc8fc\ub2f9\uc744 \ucc3d\ub2f9\ud588\ub2e4. \ud3c9\ud654\ubbfc\uc8fc\ub2f9\uc740 \uc774\ud6c4 \uc2e0\ubbfc\uc8fc\uc5f0\ud569\ub2f9\uc73c\ub85c \uac1c\uba85\ud55c \ub4a4 \uc904\uace7 \ud65c\ub3d9\ud574 \uc654\uc73c\uba70, \uc774\ud6c4 1992\ub144 \uaf2c\ub9c8\ubbfc\uc8fc\ub2f9\uc744 \ud761\uc218\ud55c \ub4a4 \ubbfc\uc8fc\ub2f9\uc744 \ucc3d\ub2f9\ud588\ub2e4. \uc0c8\ub85c\uc6b4 \ubbfc\uc8fc\ub2f9\uc740 1992\ub144 \ucd1d\uc120\uc5d0\uc11c \ud328\ud588\uc73c\ub098 \ubbfc\uc8fc\uc790\uc720\ub2f9\uc758 \uc704\uae30 \uc18d\uc5d0\uc11c \ub098\ub984 \uc120\uc804\uc744 \ud588\uc73c\uba70, \uc774\ud6c4 \uac19\uc740 \ud574 \ub300\uc120\uc5d0 \uae40\ub300\uc911\uc744 \ud6c4\ubcf4\ub85c \ub0b4\uc138\uc6cc \ub3c4\uc804\ud588\uc73c\ub098 \ud328\ud558\uc5ec, \uc57c\ub2f9\uc73c\ub85c \uacc4\uc18d \ub0a8\uac8c \ub418\uc5c8\ub2e4. \uae40\ub300\uc911\uc758 \uc740\ud1f4\ub85c \uae40\ub300\uc911\uc740 \ubaa8\ub4e0 \ub2f9\uad8c\uc744 \ub0b4\ub193\uc558\uc73c\ub098, \ub3d9\uad50\ub3d9\uacc4\uc758 \ub2f9\uad8c \uc7a5\uc545\uc740 \uacc4\uc18d\ub418\uc5c8\ub2e4. 1995\ub144 \uc9c0\ubc29\uc120\uac70\uc5d0\uc11c \uc120\uc804\ud588\uace0, \uc774\uc5b4 \uc815\uacc4\uc5d0 \ubcf5\uadc0\ud55c \uae40\ub300\uc911\uc744 \ub530\ub77c \ub3d9\uad50\ub3d9\uacc4 \ub4f1\uc774 \ud0c8\ub2f9\ud558\uba74\uc11c \uc0c8\uc815\uce58\uad6d\ubbfc\ud68c\uc758\ub97c \ucc3d\ub2f9\ud558\uc600\ub2e4. \uc0c8\uc815\uce58\uad6d\ubbfc\ud68c\uc758\ub294 1996\ub144 \ucd1d\uc120\uc5d0\uc11c \ubd80\uc9c4\ud588\uc73c\ub098 1997\ub144 \ub300\uc120\uc5d0\uc11c \uc790\uc720\ubbfc\uc8fc\uc5f0\ud569\uacfc\uc758 \uc5f0\ub300\ub85c \uc2b9\ub9ac\ud558\uc5ec, \uc0ac\uc0c1 \ucd5c\ucd08\uc758 \uc5ec\uc57c \uc815\uad8c\uad50\uccb4\ub97c \uc774\ub04c\uc5b4\ub0c8\ub2e4. \uc774\ub85c\uc368 \ud5cc\uc815 \uc774\ud6c4 \ucd5c\ucd08\uc774\uc790 1960\ub144 \uc774\ud6c4 38\ub144\ub9cc\uc5d0 \ubbfc\uc8fc\ub2f9\uacc4 \uc815\ub2f9\uc774 \uc9d1\uad8c\ud558\uc600\ub2e4.", "answer": "\uc790\uc720\ubbfc\uc8fc\uc5f0\ud569", "sentence": "\uc0c8\uc815\uce58\uad6d\ubbfc\ud68c\uc758\ub294 1996\ub144 \ucd1d\uc120\uc5d0\uc11c \ubd80\uc9c4\ud588\uc73c\ub098 1997\ub144 \ub300\uc120\uc5d0\uc11c \uc790\uc720\ubbfc\uc8fc\uc5f0\ud569 \uacfc\uc758 \uc5f0\ub300\ub85c \uc2b9\ub9ac\ud558\uc5ec, \uc0ac\uc0c1 \ucd5c\ucd08\uc758 \uc5ec\uc57c \uc815\uad8c\uad50\uccb4\ub97c \uc774\ub04c\uc5b4\ub0c8\ub2e4.", "paragraph_sentence": "\uadf8\ub7ec\ub098 \uc77c\ubd80 \ud1b5\uc77c\ubbfc\uc8fc\ub2f9 \uc758\uc6d0\ub4e4\uc740 \ub3c5\uc790 \ud589\ubcf4\ub97c \ub0b4\uac78\uace0 \uaf2c\ub9c8\ubbfc\uc8fc\ub2f9\uc744 \ucc3d\ub2f9\ud588\ub2e4. \ud3c9\ud654\ubbfc\uc8fc\ub2f9\uc740 \uc774\ud6c4 \uc2e0\ubbfc\uc8fc\uc5f0\ud569\ub2f9\uc73c\ub85c \uac1c\uba85\ud55c \ub4a4 \uc904\uace7 \ud65c\ub3d9\ud574 \uc654\uc73c\uba70, \uc774\ud6c4 1992\ub144 \uaf2c\ub9c8\ubbfc\uc8fc\ub2f9\uc744 \ud761\uc218\ud55c \ub4a4 \ubbfc\uc8fc\ub2f9\uc744 \ucc3d\ub2f9\ud588\ub2e4. \uc0c8\ub85c\uc6b4 \ubbfc\uc8fc\ub2f9\uc740 1992\ub144 \ucd1d\uc120\uc5d0\uc11c \ud328\ud588\uc73c\ub098 \ubbfc\uc8fc\uc790\uc720\ub2f9\uc758 \uc704\uae30 \uc18d\uc5d0\uc11c \ub098\ub984 \uc120\uc804\uc744 \ud588\uc73c\uba70, \uc774\ud6c4 \uac19\uc740 \ud574 \ub300\uc120\uc5d0 \uae40\ub300\uc911\uc744 \ud6c4\ubcf4\ub85c \ub0b4\uc138\uc6cc \ub3c4\uc804\ud588\uc73c\ub098 \ud328\ud558\uc5ec, \uc57c\ub2f9\uc73c\ub85c \uacc4\uc18d \ub0a8\uac8c \ub418\uc5c8\ub2e4. \uae40\ub300\uc911\uc758 \uc740\ud1f4\ub85c \uae40\ub300\uc911\uc740 \ubaa8\ub4e0 \ub2f9\uad8c\uc744 \ub0b4\ub193\uc558\uc73c\ub098, \ub3d9\uad50\ub3d9\uacc4\uc758 \ub2f9\uad8c \uc7a5\uc545\uc740 \uacc4\uc18d\ub418\uc5c8\ub2e4. 1995\ub144 \uc9c0\ubc29\uc120\uac70\uc5d0\uc11c \uc120\uc804\ud588\uace0, \uc774\uc5b4 \uc815\uacc4\uc5d0 \ubcf5\uadc0\ud55c \uae40\ub300\uc911\uc744 \ub530\ub77c \ub3d9\uad50\ub3d9\uacc4 \ub4f1\uc774 \ud0c8\ub2f9\ud558\uba74\uc11c \uc0c8\uc815\uce58\uad6d\ubbfc\ud68c\uc758\ub97c \ucc3d\ub2f9\ud558\uc600\ub2e4. \uc0c8\uc815\uce58\uad6d\ubbfc\ud68c\uc758\ub294 1996\ub144 \ucd1d\uc120\uc5d0\uc11c \ubd80\uc9c4\ud588\uc73c\ub098 1997\ub144 \ub300\uc120\uc5d0\uc11c \uc790\uc720\ubbfc\uc8fc\uc5f0\ud569 \uacfc\uc758 \uc5f0\ub300\ub85c \uc2b9\ub9ac\ud558\uc5ec, \uc0ac\uc0c1 \ucd5c\ucd08\uc758 \uc5ec\uc57c \uc815\uad8c\uad50\uccb4\ub97c \uc774\ub04c\uc5b4\ub0c8\ub2e4. \uc774\ub85c\uc368 \ud5cc\uc815 \uc774\ud6c4 \ucd5c\ucd08\uc774\uc790 1960\ub144 \uc774\ud6c4 38\ub144\ub9cc\uc5d0 \ubbfc\uc8fc\ub2f9\uacc4 \uc815\ub2f9\uc774 \uc9d1\uad8c\ud558\uc600\ub2e4.", "paragraph_answer": "\uadf8\ub7ec\ub098 \uc77c\ubd80 \ud1b5\uc77c\ubbfc\uc8fc\ub2f9 \uc758\uc6d0\ub4e4\uc740 \ub3c5\uc790 \ud589\ubcf4\ub97c \ub0b4\uac78\uace0 \uaf2c\ub9c8\ubbfc\uc8fc\ub2f9\uc744 \ucc3d\ub2f9\ud588\ub2e4. \ud3c9\ud654\ubbfc\uc8fc\ub2f9\uc740 \uc774\ud6c4 \uc2e0\ubbfc\uc8fc\uc5f0\ud569\ub2f9\uc73c\ub85c \uac1c\uba85\ud55c \ub4a4 \uc904\uace7 \ud65c\ub3d9\ud574 \uc654\uc73c\uba70, \uc774\ud6c4 1992\ub144 \uaf2c\ub9c8\ubbfc\uc8fc\ub2f9\uc744 \ud761\uc218\ud55c \ub4a4 \ubbfc\uc8fc\ub2f9\uc744 \ucc3d\ub2f9\ud588\ub2e4. \uc0c8\ub85c\uc6b4 \ubbfc\uc8fc\ub2f9\uc740 1992\ub144 \ucd1d\uc120\uc5d0\uc11c \ud328\ud588\uc73c\ub098 \ubbfc\uc8fc\uc790\uc720\ub2f9\uc758 \uc704\uae30 \uc18d\uc5d0\uc11c \ub098\ub984 \uc120\uc804\uc744 \ud588\uc73c\uba70, \uc774\ud6c4 \uac19\uc740 \ud574 \ub300\uc120\uc5d0 \uae40\ub300\uc911\uc744 \ud6c4\ubcf4\ub85c \ub0b4\uc138\uc6cc \ub3c4\uc804\ud588\uc73c\ub098 \ud328\ud558\uc5ec, \uc57c\ub2f9\uc73c\ub85c \uacc4\uc18d \ub0a8\uac8c \ub418\uc5c8\ub2e4. \uae40\ub300\uc911\uc758 \uc740\ud1f4\ub85c \uae40\ub300\uc911\uc740 \ubaa8\ub4e0 \ub2f9\uad8c\uc744 \ub0b4\ub193\uc558\uc73c\ub098, \ub3d9\uad50\ub3d9\uacc4\uc758 \ub2f9\uad8c \uc7a5\uc545\uc740 \uacc4\uc18d\ub418\uc5c8\ub2e4. 1995\ub144 \uc9c0\ubc29\uc120\uac70\uc5d0\uc11c \uc120\uc804\ud588\uace0, \uc774\uc5b4 \uc815\uacc4\uc5d0 \ubcf5\uadc0\ud55c \uae40\ub300\uc911\uc744 \ub530\ub77c \ub3d9\uad50\ub3d9\uacc4 \ub4f1\uc774 \ud0c8\ub2f9\ud558\uba74\uc11c \uc0c8\uc815\uce58\uad6d\ubbfc\ud68c\uc758\ub97c \ucc3d\ub2f9\ud558\uc600\ub2e4. \uc0c8\uc815\uce58\uad6d\ubbfc\ud68c\uc758\ub294 1996\ub144 \ucd1d\uc120\uc5d0\uc11c \ubd80\uc9c4\ud588\uc73c\ub098 1997\ub144 \ub300\uc120\uc5d0\uc11c \uc790\uc720\ubbfc\uc8fc\uc5f0\ud569 \uacfc\uc758 \uc5f0\ub300\ub85c \uc2b9\ub9ac\ud558\uc5ec, \uc0ac\uc0c1 \ucd5c\ucd08\uc758 \uc5ec\uc57c \uc815\uad8c\uad50\uccb4\ub97c \uc774\ub04c\uc5b4\ub0c8\ub2e4. \uc774\ub85c\uc368 \ud5cc\uc815 \uc774\ud6c4 \ucd5c\ucd08\uc774\uc790 1960\ub144 \uc774\ud6c4 38\ub144\ub9cc\uc5d0 \ubbfc\uc8fc\ub2f9\uacc4 \uc815\ub2f9\uc774 \uc9d1\uad8c\ud558\uc600\ub2e4.", "sentence_answer": "\uc0c8\uc815\uce58\uad6d\ubbfc\ud68c\uc758\ub294 1996\ub144 \ucd1d\uc120\uc5d0\uc11c \ubd80\uc9c4\ud588\uc73c\ub098 1997\ub144 \ub300\uc120\uc5d0\uc11c \uc790\uc720\ubbfc\uc8fc\uc5f0\ud569 \uacfc\uc758 \uc5f0\ub300\ub85c \uc2b9\ub9ac\ud558\uc5ec, \uc0ac\uc0c1 \ucd5c\ucd08\uc758 \uc5ec\uc57c \uc815\uad8c\uad50\uccb4\ub97c \uc774\ub04c\uc5b4\ub0c8\ub2e4.", "paragraph_id": "6486812-2-1", "paragraph_question": "question: 1997\ub144 \ub300\uc120 \ub2f9\uc2dc \uc0c8\uc815\uce58\uad6d\ubbfc\ud68c\uc758\ub294 \uc5b4\ub5a4 \uc815\ub2f9\uacfc \uc5f0\ub300\ud588\ub294\uac00?, context: \uadf8\ub7ec\ub098 \uc77c\ubd80 \ud1b5\uc77c\ubbfc\uc8fc\ub2f9 \uc758\uc6d0\ub4e4\uc740 \ub3c5\uc790 \ud589\ubcf4\ub97c \ub0b4\uac78\uace0 \uaf2c\ub9c8\ubbfc\uc8fc\ub2f9\uc744 \ucc3d\ub2f9\ud588\ub2e4. \ud3c9\ud654\ubbfc\uc8fc\ub2f9\uc740 \uc774\ud6c4 \uc2e0\ubbfc\uc8fc\uc5f0\ud569\ub2f9\uc73c\ub85c \uac1c\uba85\ud55c \ub4a4 \uc904\uace7 \ud65c\ub3d9\ud574 \uc654\uc73c\uba70, \uc774\ud6c4 1992\ub144 \uaf2c\ub9c8\ubbfc\uc8fc\ub2f9\uc744 \ud761\uc218\ud55c \ub4a4 \ubbfc\uc8fc\ub2f9\uc744 \ucc3d\ub2f9\ud588\ub2e4. \uc0c8\ub85c\uc6b4 \ubbfc\uc8fc\ub2f9\uc740 1992\ub144 \ucd1d\uc120\uc5d0\uc11c \ud328\ud588\uc73c\ub098 \ubbfc\uc8fc\uc790\uc720\ub2f9\uc758 \uc704\uae30 \uc18d\uc5d0\uc11c \ub098\ub984 \uc120\uc804\uc744 \ud588\uc73c\uba70, \uc774\ud6c4 \uac19\uc740 \ud574 \ub300\uc120\uc5d0 \uae40\ub300\uc911\uc744 \ud6c4\ubcf4\ub85c \ub0b4\uc138\uc6cc \ub3c4\uc804\ud588\uc73c\ub098 \ud328\ud558\uc5ec, \uc57c\ub2f9\uc73c\ub85c \uacc4\uc18d \ub0a8\uac8c \ub418\uc5c8\ub2e4. \uae40\ub300\uc911\uc758 \uc740\ud1f4\ub85c \uae40\ub300\uc911\uc740 \ubaa8\ub4e0 \ub2f9\uad8c\uc744 \ub0b4\ub193\uc558\uc73c\ub098, \ub3d9\uad50\ub3d9\uacc4\uc758 \ub2f9\uad8c \uc7a5\uc545\uc740 \uacc4\uc18d\ub418\uc5c8\ub2e4. 1995\ub144 \uc9c0\ubc29\uc120\uac70\uc5d0\uc11c \uc120\uc804\ud588\uace0, \uc774\uc5b4 \uc815\uacc4\uc5d0 \ubcf5\uadc0\ud55c \uae40\ub300\uc911\uc744 \ub530\ub77c \ub3d9\uad50\ub3d9\uacc4 \ub4f1\uc774 \ud0c8\ub2f9\ud558\uba74\uc11c \uc0c8\uc815\uce58\uad6d\ubbfc\ud68c\uc758\ub97c \ucc3d\ub2f9\ud558\uc600\ub2e4. \uc0c8\uc815\uce58\uad6d\ubbfc\ud68c\uc758\ub294 1996\ub144 \ucd1d\uc120\uc5d0\uc11c \ubd80\uc9c4\ud588\uc73c\ub098 1997\ub144 \ub300\uc120\uc5d0\uc11c \uc790\uc720\ubbfc\uc8fc\uc5f0\ud569\uacfc\uc758 \uc5f0\ub300\ub85c \uc2b9\ub9ac\ud558\uc5ec, \uc0ac\uc0c1 \ucd5c\ucd08\uc758 \uc5ec\uc57c \uc815\uad8c\uad50\uccb4\ub97c \uc774\ub04c\uc5b4\ub0c8\ub2e4. \uc774\ub85c\uc368 \ud5cc\uc815 \uc774\ud6c4 \ucd5c\ucd08\uc774\uc790 1960\ub144 \uc774\ud6c4 38\ub144\ub9cc\uc5d0 \ubbfc\uc8fc\ub2f9\uacc4 \uc815\ub2f9\uc774 \uc9d1\uad8c\ud558\uc600\ub2e4."} {"question": "\ubc15\uc5f0\ucc28\uac00 \uad8c\uc591\uc219\uc5d0\uac8c \uc900 \uc790\uae08\uc774 \ube4c\ub9b0 \ub3c8\uc774\ub77c\uace0 \uc801\uc2dc\ud55c \uc720\uc8c4 \ud310\uacb0\ubb38\uc744 \ubc1b\uc740 \uc0ac\ub78c\uc740?", "paragraph": "\uc57c\ub2f9\uacfc \uc9c4\ubcf4 \uc131\ud5a5\uc758 \uc2dc\ubbfc \ub2e8\uccb4\ub4e4\uc740 \uac80\ucc30 \uc218\uc0ac\uc5d0 \ub300\ud55c \ub178\uace8\uc801\uc778 \ube44\ud310\uc744 \uac00\ud588\uc73c\uba70, \uac80\ucc30 \uc218\uc0ac\uc640 \uad00\ub828\ub41c \uc2dc\uad6d \uc120\uc5b8\ub3c4 \uc904\uc744 \uc774\uc5c8\ub2e4. \ub300\uac80\ucc30\uccad \ud648\ud398\uc774\uc9c0\uc5d0\ub294 \uac80\ucc30\uc744 \ube44\ud558\ud558\ub294 '\ub5a1\uac80'\uc774\ub77c\ub294 \ud45c\ud604\uc774 \ub118\uce58\uba70, \uac80\ucc30\uc744 \uacac\uc81c\ud558\uae30 \uc704\ud55c \uacf5\uc9c1\uc790 \ubd80\ud328 \uc218\uc0ac\ucc98\ub97c \uc2e0\uc124\ud574\uc57c \ud55c\ub2e4\ub294 \uc8fc\uc7a5\uacfc \ube44\ud310\uc774 \uc81c\uae30\ub418\uc5c8\ub2e4. \uadf8\ub7ec\ub098 \ubcf4\uc218 \uc5b8\ub860 \ubc0f \uc2dc\ubbfc\ub2e8\uccb4\uc5d0\uc11c\ub294 \uc804\uc9c1 \ub300\ud1b5\ub839\uc774\ub77c\ub3c4 \ubc95 \uc704\uc5d0 \uad70\ub9bc\ud560 \uc218 \uc5c6\uace0 \uc8c4\ub97c \uc9c0\uc73c\uba74 \ub204\uad6c\ub098 \ucc98\ubc8c\ubc1b\uc744 \uc218 \uc788\ub2e4\ub294 \uc6d0\uce59\uc744 \ub2e4\uc2dc \uc138\uc6c0\uc73c\ub85c\uc368 \ub300\ud55c\ubbfc\uad6d\uc774 \ubc95\uce58\uad6d\uac00\uc784\uc744 \uc7ac\ud655\uc778\ud588\ub2e4\uace0 \uc8fc\uc7a5\ud588\ub2e4. \uadf8\ub7fc\uc5d0\ub3c4 \ubd88\uad6c\ud558\uace0 \uac80\ucc30\uc740 \ub178\ubb34\ud604\uacfc \uad00\ub828\ud55c \uc778\ubb3c\uc5d0 \ub300\ud574 \uadf8\uac00 \uc8fd\uc5c8\ub2e4\ub294 \uc774\uc720\ub85c \uae30\uc18c\ud558\uc9c0 \uc54a\uc740 \ub370 \ub300\ud574 \uc774\uc758\uac00 \uc81c\uae30\ub418\uc5c8\ub2e4. \uc774\ub294 \uc608\uc804\uc5d0 \ub178\uac74\ud3c9\uacfc \ub0a8\uc0c1\uad6d \uc0ac\uc774\uc5d0 \ubc8c\uc5b4\uc84c\ub358 \ub1cc\ubb3c \uc218\uc218 \uc0ac\uac74\uc5d0\uc11c \ub0a8\uc0c1\uad6d\uc774 \uc790\uc0b4\ud588\uc74c\uc5d0\ub3c4 \ub178\uac74\ud3c9\uc744 \uae30\uc18c\ud55c \uc608 \uc640\ub3c4 \ubaa8\uc21c\uc774 \ub41c\ub2e4\ub294 \uc758\uacac\ub3c4 \uc788\uc73c\uba70, \uac80\ucc30\uc774 \ub178\ubb34\ud604\uacfc \uad00\ub828\ud55c \uc0ac\ud56d\uc5d0\uc11c \uc720\uc8c4\uac00 \ud655\uc2e4\ud558\uc9c0 \uc54a\uc558\uae30 \ub54c\ubb38\uc5d0 \uae30\uc18c\ud558\uc9c0 \uc54a\uc558\ub2e4\ub294 \ube44\ud310\uc774 \uc788\ub2e4. \uc774\uc640 \ubcc4\uac1c\ub85c \uac80\ucc30\uc740 \ub178\ubb34\ud604\uc758 \uc544\ub0b4\uc778 \uad8c\uc591\uc219\uc758 \uac70\uc9d3 \uc99d\uc5b8\uc774 \uc0ac\ubc95 \ubc29\ud574\uc5d0 \ud574\ub2f9\ud55c\ub2e4\uace0 \uc8fc\uc7a5\ud558\uba74\uc11c \uc774\ub97c \uadfc\uac70\ub85c \ub1cc\ubb3c \uc218\uc218 \ud610\uc758\ub85c \uae30\uc18c\ud558\ub824 \ud588\uc73c\ub098 \ub178\ubb34\ud604\uc758 \uc790\uc0b4\uc5d0 \ub530\ub978 \ub3d9\uc815 \uc5ec\ub860\uc73c\ub85c \uc778\ud574 \uae30\uc18c\uc870\ucc28 \ud558\uc9c0 \ubabb\ud588\ub2e4\ub294 \ube44\ud310\ub3c4 \uc788\ub2e4. \uc55e\uc11c\uc758 \ub0a8\uc0c1\uad6d \uc608\uc640 \uac19\uc774 \uacfc\uac70\uc5d0 \uc790\uc0b4\ud55c \uc0ac\ub78c\uc5d0 \ub300\ud574 \uadf8 \uc0c1\ub300\ubc29\uc744 \uae30\uc18c\ud55c \uc804\ub840\uac00 \uc788\uae30 \ub54c\ubb38\uc774\ub2e4. \ub2e4\ub9cc \ubc15\uc5f0\ucc28\uc640 \uad8c\uc591\uc219\uc774 \uad00\ub828\ub41c \uc790\uae08\uc5d0 \ub300\ud574\uc11c\ub294 \ub1cc\ubb3c\uc774 \uc544\ub2cc \ube4c\ub9b0 \ub3c8\uc774\ub77c\uace0 \uc815\uc0c1\ubb38 \uc720\uc8c4 \ud310\uacb0\ubb38\uc5d0\uc11c \uc801\uc2dc\ud588\ub2e4.", "answer": "\uc815\uc0c1\ubb38", "sentence": "\ub2e4\ub9cc \ubc15\uc5f0\ucc28\uc640 \uad8c\uc591\uc219\uc774 \uad00\ub828\ub41c \uc790\uae08\uc5d0 \ub300\ud574\uc11c\ub294 \ub1cc\ubb3c\uc774 \uc544\ub2cc \ube4c\ub9b0 \ub3c8\uc774\ub77c\uace0 \uc815\uc0c1\ubb38 \uc720\uc8c4 \ud310\uacb0\ubb38\uc5d0\uc11c \uc801\uc2dc\ud588\ub2e4.", "paragraph_sentence": "\uc57c\ub2f9\uacfc \uc9c4\ubcf4 \uc131\ud5a5\uc758 \uc2dc\ubbfc \ub2e8\uccb4\ub4e4\uc740 \uac80\ucc30 \uc218\uc0ac\uc5d0 \ub300\ud55c \ub178\uace8\uc801\uc778 \ube44\ud310\uc744 \uac00\ud588\uc73c\uba70, \uac80\ucc30 \uc218\uc0ac\uc640 \uad00\ub828\ub41c \uc2dc\uad6d \uc120\uc5b8\ub3c4 \uc904\uc744 \uc774\uc5c8\ub2e4. \ub300\uac80\ucc30\uccad \ud648\ud398\uc774\uc9c0\uc5d0\ub294 \uac80\ucc30\uc744 \ube44\ud558\ud558\ub294 '\ub5a1\uac80'\uc774\ub77c\ub294 \ud45c\ud604\uc774 \ub118\uce58\uba70, \uac80\ucc30\uc744 \uacac\uc81c\ud558\uae30 \uc704\ud55c \uacf5\uc9c1\uc790 \ubd80\ud328 \uc218\uc0ac\ucc98\ub97c \uc2e0\uc124\ud574\uc57c \ud55c\ub2e4\ub294 \uc8fc\uc7a5\uacfc \ube44\ud310\uc774 \uc81c\uae30\ub418\uc5c8\ub2e4. \uadf8\ub7ec\ub098 \ubcf4\uc218 \uc5b8\ub860 \ubc0f \uc2dc\ubbfc\ub2e8\uccb4\uc5d0\uc11c\ub294 \uc804\uc9c1 \ub300\ud1b5\ub839\uc774\ub77c\ub3c4 \ubc95 \uc704\uc5d0 \uad70\ub9bc\ud560 \uc218 \uc5c6\uace0 \uc8c4\ub97c \uc9c0\uc73c\uba74 \ub204\uad6c\ub098 \ucc98\ubc8c\ubc1b\uc744 \uc218 \uc788\ub2e4\ub294 \uc6d0\uce59\uc744 \ub2e4\uc2dc \uc138\uc6c0\uc73c\ub85c\uc368 \ub300\ud55c\ubbfc\uad6d\uc774 \ubc95\uce58\uad6d\uac00\uc784\uc744 \uc7ac\ud655\uc778\ud588\ub2e4\uace0 \uc8fc\uc7a5\ud588\ub2e4. \uadf8\ub7fc\uc5d0\ub3c4 \ubd88\uad6c\ud558\uace0 \uac80\ucc30\uc740 \ub178\ubb34\ud604\uacfc \uad00\ub828\ud55c \uc778\ubb3c\uc5d0 \ub300\ud574 \uadf8\uac00 \uc8fd\uc5c8\ub2e4\ub294 \uc774\uc720\ub85c \uae30\uc18c\ud558\uc9c0 \uc54a\uc740 \ub370 \ub300\ud574 \uc774\uc758\uac00 \uc81c\uae30\ub418\uc5c8\ub2e4. \uc774\ub294 \uc608\uc804\uc5d0 \ub178\uac74\ud3c9\uacfc \ub0a8\uc0c1\uad6d \uc0ac\uc774\uc5d0 \ubc8c\uc5b4\uc84c\ub358 \ub1cc\ubb3c \uc218\uc218 \uc0ac\uac74\uc5d0\uc11c \ub0a8\uc0c1\uad6d\uc774 \uc790\uc0b4\ud588\uc74c\uc5d0\ub3c4 \ub178\uac74\ud3c9\uc744 \uae30\uc18c\ud55c \uc608 \uc640\ub3c4 \ubaa8\uc21c\uc774 \ub41c\ub2e4\ub294 \uc758\uacac\ub3c4 \uc788\uc73c\uba70, \uac80\ucc30\uc774 \ub178\ubb34\ud604\uacfc \uad00\ub828\ud55c \uc0ac\ud56d\uc5d0\uc11c \uc720\uc8c4\uac00 \ud655\uc2e4\ud558\uc9c0 \uc54a\uc558\uae30 \ub54c\ubb38\uc5d0 \uae30\uc18c\ud558\uc9c0 \uc54a\uc558\ub2e4\ub294 \ube44\ud310\uc774 \uc788\ub2e4. \uc774\uc640 \ubcc4\uac1c\ub85c \uac80\ucc30\uc740 \ub178\ubb34\ud604\uc758 \uc544\ub0b4\uc778 \uad8c\uc591\uc219\uc758 \uac70\uc9d3 \uc99d\uc5b8\uc774 \uc0ac\ubc95 \ubc29\ud574\uc5d0 \ud574\ub2f9\ud55c\ub2e4\uace0 \uc8fc\uc7a5\ud558\uba74\uc11c \uc774\ub97c \uadfc\uac70\ub85c \ub1cc\ubb3c \uc218\uc218 \ud610\uc758\ub85c \uae30\uc18c\ud558\ub824 \ud588\uc73c\ub098 \ub178\ubb34\ud604\uc758 \uc790\uc0b4\uc5d0 \ub530\ub978 \ub3d9\uc815 \uc5ec\ub860\uc73c\ub85c \uc778\ud574 \uae30\uc18c\uc870\ucc28 \ud558\uc9c0 \ubabb\ud588\ub2e4\ub294 \ube44\ud310\ub3c4 \uc788\ub2e4. \uc55e\uc11c\uc758 \ub0a8\uc0c1\uad6d \uc608\uc640 \uac19\uc774 \uacfc\uac70\uc5d0 \uc790\uc0b4\ud55c \uc0ac\ub78c\uc5d0 \ub300\ud574 \uadf8 \uc0c1\ub300\ubc29\uc744 \uae30\uc18c\ud55c \uc804\ub840\uac00 \uc788\uae30 \ub54c\ubb38\uc774\ub2e4. \ub2e4\ub9cc \ubc15\uc5f0\ucc28\uc640 \uad8c\uc591\uc219\uc774 \uad00\ub828\ub41c \uc790\uae08\uc5d0 \ub300\ud574\uc11c\ub294 \ub1cc\ubb3c\uc774 \uc544\ub2cc \ube4c\ub9b0 \ub3c8\uc774\ub77c\uace0 \uc815\uc0c1\ubb38 \uc720\uc8c4 \ud310\uacb0\ubb38\uc5d0\uc11c \uc801\uc2dc\ud588\ub2e4. ", "paragraph_answer": "\uc57c\ub2f9\uacfc \uc9c4\ubcf4 \uc131\ud5a5\uc758 \uc2dc\ubbfc \ub2e8\uccb4\ub4e4\uc740 \uac80\ucc30 \uc218\uc0ac\uc5d0 \ub300\ud55c \ub178\uace8\uc801\uc778 \ube44\ud310\uc744 \uac00\ud588\uc73c\uba70, \uac80\ucc30 \uc218\uc0ac\uc640 \uad00\ub828\ub41c \uc2dc\uad6d \uc120\uc5b8\ub3c4 \uc904\uc744 \uc774\uc5c8\ub2e4. \ub300\uac80\ucc30\uccad \ud648\ud398\uc774\uc9c0\uc5d0\ub294 \uac80\ucc30\uc744 \ube44\ud558\ud558\ub294 '\ub5a1\uac80'\uc774\ub77c\ub294 \ud45c\ud604\uc774 \ub118\uce58\uba70, \uac80\ucc30\uc744 \uacac\uc81c\ud558\uae30 \uc704\ud55c \uacf5\uc9c1\uc790 \ubd80\ud328 \uc218\uc0ac\ucc98\ub97c \uc2e0\uc124\ud574\uc57c \ud55c\ub2e4\ub294 \uc8fc\uc7a5\uacfc \ube44\ud310\uc774 \uc81c\uae30\ub418\uc5c8\ub2e4. \uadf8\ub7ec\ub098 \ubcf4\uc218 \uc5b8\ub860 \ubc0f \uc2dc\ubbfc\ub2e8\uccb4\uc5d0\uc11c\ub294 \uc804\uc9c1 \ub300\ud1b5\ub839\uc774\ub77c\ub3c4 \ubc95 \uc704\uc5d0 \uad70\ub9bc\ud560 \uc218 \uc5c6\uace0 \uc8c4\ub97c \uc9c0\uc73c\uba74 \ub204\uad6c\ub098 \ucc98\ubc8c\ubc1b\uc744 \uc218 \uc788\ub2e4\ub294 \uc6d0\uce59\uc744 \ub2e4\uc2dc \uc138\uc6c0\uc73c\ub85c\uc368 \ub300\ud55c\ubbfc\uad6d\uc774 \ubc95\uce58\uad6d\uac00\uc784\uc744 \uc7ac\ud655\uc778\ud588\ub2e4\uace0 \uc8fc\uc7a5\ud588\ub2e4. \uadf8\ub7fc\uc5d0\ub3c4 \ubd88\uad6c\ud558\uace0 \uac80\ucc30\uc740 \ub178\ubb34\ud604\uacfc \uad00\ub828\ud55c \uc778\ubb3c\uc5d0 \ub300\ud574 \uadf8\uac00 \uc8fd\uc5c8\ub2e4\ub294 \uc774\uc720\ub85c \uae30\uc18c\ud558\uc9c0 \uc54a\uc740 \ub370 \ub300\ud574 \uc774\uc758\uac00 \uc81c\uae30\ub418\uc5c8\ub2e4. \uc774\ub294 \uc608\uc804\uc5d0 \ub178\uac74\ud3c9\uacfc \ub0a8\uc0c1\uad6d \uc0ac\uc774\uc5d0 \ubc8c\uc5b4\uc84c\ub358 \ub1cc\ubb3c \uc218\uc218 \uc0ac\uac74\uc5d0\uc11c \ub0a8\uc0c1\uad6d\uc774 \uc790\uc0b4\ud588\uc74c\uc5d0\ub3c4 \ub178\uac74\ud3c9\uc744 \uae30\uc18c\ud55c \uc608 \uc640\ub3c4 \ubaa8\uc21c\uc774 \ub41c\ub2e4\ub294 \uc758\uacac\ub3c4 \uc788\uc73c\uba70, \uac80\ucc30\uc774 \ub178\ubb34\ud604\uacfc \uad00\ub828\ud55c \uc0ac\ud56d\uc5d0\uc11c \uc720\uc8c4\uac00 \ud655\uc2e4\ud558\uc9c0 \uc54a\uc558\uae30 \ub54c\ubb38\uc5d0 \uae30\uc18c\ud558\uc9c0 \uc54a\uc558\ub2e4\ub294 \ube44\ud310\uc774 \uc788\ub2e4. \uc774\uc640 \ubcc4\uac1c\ub85c \uac80\ucc30\uc740 \ub178\ubb34\ud604\uc758 \uc544\ub0b4\uc778 \uad8c\uc591\uc219\uc758 \uac70\uc9d3 \uc99d\uc5b8\uc774 \uc0ac\ubc95 \ubc29\ud574\uc5d0 \ud574\ub2f9\ud55c\ub2e4\uace0 \uc8fc\uc7a5\ud558\uba74\uc11c \uc774\ub97c \uadfc\uac70\ub85c \ub1cc\ubb3c \uc218\uc218 \ud610\uc758\ub85c \uae30\uc18c\ud558\ub824 \ud588\uc73c\ub098 \ub178\ubb34\ud604\uc758 \uc790\uc0b4\uc5d0 \ub530\ub978 \ub3d9\uc815 \uc5ec\ub860\uc73c\ub85c \uc778\ud574 \uae30\uc18c\uc870\ucc28 \ud558\uc9c0 \ubabb\ud588\ub2e4\ub294 \ube44\ud310\ub3c4 \uc788\ub2e4. \uc55e\uc11c\uc758 \ub0a8\uc0c1\uad6d \uc608\uc640 \uac19\uc774 \uacfc\uac70\uc5d0 \uc790\uc0b4\ud55c \uc0ac\ub78c\uc5d0 \ub300\ud574 \uadf8 \uc0c1\ub300\ubc29\uc744 \uae30\uc18c\ud55c \uc804\ub840\uac00 \uc788\uae30 \ub54c\ubb38\uc774\ub2e4. \ub2e4\ub9cc \ubc15\uc5f0\ucc28\uc640 \uad8c\uc591\uc219\uc774 \uad00\ub828\ub41c \uc790\uae08\uc5d0 \ub300\ud574\uc11c\ub294 \ub1cc\ubb3c\uc774 \uc544\ub2cc \ube4c\ub9b0 \ub3c8\uc774\ub77c\uace0 \uc815\uc0c1\ubb38 \uc720\uc8c4 \ud310\uacb0\ubb38\uc5d0\uc11c \uc801\uc2dc\ud588\ub2e4.", "sentence_answer": "\ub2e4\ub9cc \ubc15\uc5f0\ucc28\uc640 \uad8c\uc591\uc219\uc774 \uad00\ub828\ub41c \uc790\uae08\uc5d0 \ub300\ud574\uc11c\ub294 \ub1cc\ubb3c\uc774 \uc544\ub2cc \ube4c\ub9b0 \ub3c8\uc774\ub77c\uace0 \uc815\uc0c1\ubb38 \uc720\uc8c4 \ud310\uacb0\ubb38\uc5d0\uc11c \uc801\uc2dc\ud588\ub2e4.", "paragraph_id": "6512090-22-1", "paragraph_question": "question: \ubc15\uc5f0\ucc28\uac00 \uad8c\uc591\uc219\uc5d0\uac8c \uc900 \uc790\uae08\uc774 \ube4c\ub9b0 \ub3c8\uc774\ub77c\uace0 \uc801\uc2dc\ud55c \uc720\uc8c4 \ud310\uacb0\ubb38\uc744 \ubc1b\uc740 \uc0ac\ub78c\uc740?, context: \uc57c\ub2f9\uacfc \uc9c4\ubcf4 \uc131\ud5a5\uc758 \uc2dc\ubbfc \ub2e8\uccb4\ub4e4\uc740 \uac80\ucc30 \uc218\uc0ac\uc5d0 \ub300\ud55c \ub178\uace8\uc801\uc778 \ube44\ud310\uc744 \uac00\ud588\uc73c\uba70, \uac80\ucc30 \uc218\uc0ac\uc640 \uad00\ub828\ub41c \uc2dc\uad6d \uc120\uc5b8\ub3c4 \uc904\uc744 \uc774\uc5c8\ub2e4. \ub300\uac80\ucc30\uccad \ud648\ud398\uc774\uc9c0\uc5d0\ub294 \uac80\ucc30\uc744 \ube44\ud558\ud558\ub294 '\ub5a1\uac80'\uc774\ub77c\ub294 \ud45c\ud604\uc774 \ub118\uce58\uba70, \uac80\ucc30\uc744 \uacac\uc81c\ud558\uae30 \uc704\ud55c \uacf5\uc9c1\uc790 \ubd80\ud328 \uc218\uc0ac\ucc98\ub97c \uc2e0\uc124\ud574\uc57c \ud55c\ub2e4\ub294 \uc8fc\uc7a5\uacfc \ube44\ud310\uc774 \uc81c\uae30\ub418\uc5c8\ub2e4. \uadf8\ub7ec\ub098 \ubcf4\uc218 \uc5b8\ub860 \ubc0f \uc2dc\ubbfc\ub2e8\uccb4\uc5d0\uc11c\ub294 \uc804\uc9c1 \ub300\ud1b5\ub839\uc774\ub77c\ub3c4 \ubc95 \uc704\uc5d0 \uad70\ub9bc\ud560 \uc218 \uc5c6\uace0 \uc8c4\ub97c \uc9c0\uc73c\uba74 \ub204\uad6c\ub098 \ucc98\ubc8c\ubc1b\uc744 \uc218 \uc788\ub2e4\ub294 \uc6d0\uce59\uc744 \ub2e4\uc2dc \uc138\uc6c0\uc73c\ub85c\uc368 \ub300\ud55c\ubbfc\uad6d\uc774 \ubc95\uce58\uad6d\uac00\uc784\uc744 \uc7ac\ud655\uc778\ud588\ub2e4\uace0 \uc8fc\uc7a5\ud588\ub2e4. \uadf8\ub7fc\uc5d0\ub3c4 \ubd88\uad6c\ud558\uace0 \uac80\ucc30\uc740 \ub178\ubb34\ud604\uacfc \uad00\ub828\ud55c \uc778\ubb3c\uc5d0 \ub300\ud574 \uadf8\uac00 \uc8fd\uc5c8\ub2e4\ub294 \uc774\uc720\ub85c \uae30\uc18c\ud558\uc9c0 \uc54a\uc740 \ub370 \ub300\ud574 \uc774\uc758\uac00 \uc81c\uae30\ub418\uc5c8\ub2e4. \uc774\ub294 \uc608\uc804\uc5d0 \ub178\uac74\ud3c9\uacfc \ub0a8\uc0c1\uad6d \uc0ac\uc774\uc5d0 \ubc8c\uc5b4\uc84c\ub358 \ub1cc\ubb3c \uc218\uc218 \uc0ac\uac74\uc5d0\uc11c \ub0a8\uc0c1\uad6d\uc774 \uc790\uc0b4\ud588\uc74c\uc5d0\ub3c4 \ub178\uac74\ud3c9\uc744 \uae30\uc18c\ud55c \uc608 \uc640\ub3c4 \ubaa8\uc21c\uc774 \ub41c\ub2e4\ub294 \uc758\uacac\ub3c4 \uc788\uc73c\uba70, \uac80\ucc30\uc774 \ub178\ubb34\ud604\uacfc \uad00\ub828\ud55c \uc0ac\ud56d\uc5d0\uc11c \uc720\uc8c4\uac00 \ud655\uc2e4\ud558\uc9c0 \uc54a\uc558\uae30 \ub54c\ubb38\uc5d0 \uae30\uc18c\ud558\uc9c0 \uc54a\uc558\ub2e4\ub294 \ube44\ud310\uc774 \uc788\ub2e4. \uc774\uc640 \ubcc4\uac1c\ub85c \uac80\ucc30\uc740 \ub178\ubb34\ud604\uc758 \uc544\ub0b4\uc778 \uad8c\uc591\uc219\uc758 \uac70\uc9d3 \uc99d\uc5b8\uc774 \uc0ac\ubc95 \ubc29\ud574\uc5d0 \ud574\ub2f9\ud55c\ub2e4\uace0 \uc8fc\uc7a5\ud558\uba74\uc11c \uc774\ub97c \uadfc\uac70\ub85c \ub1cc\ubb3c \uc218\uc218 \ud610\uc758\ub85c \uae30\uc18c\ud558\ub824 \ud588\uc73c\ub098 \ub178\ubb34\ud604\uc758 \uc790\uc0b4\uc5d0 \ub530\ub978 \ub3d9\uc815 \uc5ec\ub860\uc73c\ub85c \uc778\ud574 \uae30\uc18c\uc870\ucc28 \ud558\uc9c0 \ubabb\ud588\ub2e4\ub294 \ube44\ud310\ub3c4 \uc788\ub2e4. \uc55e\uc11c\uc758 \ub0a8\uc0c1\uad6d \uc608\uc640 \uac19\uc774 \uacfc\uac70\uc5d0 \uc790\uc0b4\ud55c \uc0ac\ub78c\uc5d0 \ub300\ud574 \uadf8 \uc0c1\ub300\ubc29\uc744 \uae30\uc18c\ud55c \uc804\ub840\uac00 \uc788\uae30 \ub54c\ubb38\uc774\ub2e4. \ub2e4\ub9cc \ubc15\uc5f0\ucc28\uc640 \uad8c\uc591\uc219\uc774 \uad00\ub828\ub41c \uc790\uae08\uc5d0 \ub300\ud574\uc11c\ub294 \ub1cc\ubb3c\uc774 \uc544\ub2cc \ube4c\ub9b0 \ub3c8\uc774\ub77c\uace0 \uc815\uc0c1\ubb38 \uc720\uc8c4 \ud310\uacb0\ubb38\uc5d0\uc11c \uc801\uc2dc\ud588\ub2e4."} {"question": "\ub9d0\uac12\uc744 \ubb3c\uc5b4\ub0b4\ub824\ub2e4 \ud30c\uc0b0\ud558\ub294 \uacbd\uc6b0\uac00 \ub9ce\uc558\ub358 \uc774\ub4e4\uc740?", "paragraph": "\uc870\uc120 \uc2dc\ub300\uc758 \uc81c\uc8fc\ubaa9\ub9c8\uc7a5\uc740 \uc758\uc815\ubd80, \ubcd1\uc870 \ubc0f \uc0ac\ubcf5\uc2dc(\u53f8\u50d5\u5bfa)\uc758 \uc9c0\ud718 \uac10\ub3c5 \uc544\ub798\uc11c \uc804\ub77c\ub3c4\uac10\ucc30\uc0ac, \uc81c\uc8fc\ubaa9\uc0ac, \uac10\ubaa9\uad00(\uc81c\uc8fc \ud310\uad00 \u00b7 \uc815\uc758\ud604\uac10 \u00b7 \ub300\uc815\ud604\uac10 \uacb8\uc784), \ub9c8\uac10, \uad70\ub450, \uad70\ubd80, \ubaa9\uc790 \ub4f1 \uc21c\uc11c\uc758 \uacc4\uae09\uc73c\ub85c \ubc30\uce58\ub3fc \uc6b4\uc601\ub418\uc5c8\ub2e4. \uc81c\uc8fc\ub9c8\ub294 \uc655\uc774 \ud0c0\ub294 \uc5b4\uc2b9\ub9c8\ub97c \ube44\ub86f\ud574 \uad70\ub9c8, \uc885\ub9c8, \uc5ed\ub9c8, \ud30c\ubc1c\ub9c8, \ud0dc\ub9c8, \ub9cc\ub9c8, \ubcf5\ub9c8 \ub4f1 \ub2e4\uc591\ud558\uac8c \ud65c\uc6a9\ub418\uc5c8\uc73c\uba70, \ud55c\uc591\uc758 \uc870\uc815\uc5d0 \ud574\ub9c8\ub2e4 200\ud544\uacfc \uc9c4\uc0c1\ub9c8 60\ud544\uc744 \ubc14\ucce4\uc744 \ubfd0 \uc544\ub2c8\ub77c \uc81c\uc8fc\ubaa9\uc0ac \ub4f1 \uad00\ub9ac\uc5d0\uac8c\ub3c4 \ubc14\uce58\ub3c4\ub85d \uaddc\uc815\ub418\uc5b4, 3\ub144 \uc8fc\uae30\ub85c \ub9d0\uc744 \ucc28\ucd9c\ud588\ub2e4. \ub9d0\uc758 \uc0c1\ud0dc\uac00 \uc88b\uc9c0 \uc54a\uc744 \uacbd\uc6b0 \ubaa9\uc790\uc758 \ucc45\uc784\uc73c\ub85c \uc804\uac00\ub418\uc5b4 \ub9d0\uac12\uc744 \ubcf8\uc778\ubfd0 \uc544\ub2c8\ub77c \uce5c\ucc99\ub4e4\uae4c\uc9c0 \ub098\uc11c\uc11c \ubc30\uc0c1 \ucc45\uc784\uc744 \uc9c0\uace0 \ubb3c\uc5b4\ub0b4\uc57c \ud588\ub294\ub370, \ud480\uc774 \uc5c6\ub294 \ucd08\ubd04\uc774\ub098 \uaca8\uc6b8\uc774 \ub418\uba74 \uad76\uc8fc\ub824 \uc8fd\ub294 \ub9d0\uc774 \ub9ce\uc558\uace0, \uc774\ub54c \ubaa9\uc790\ub294 \uc8fd\uc740 \ub9d0\uc758 \uac00\uc8fd\uc744 \ubc97\uaca8\uc11c \ub9d0\uc774 \uc790\uc5f0\uc0ac\ud588\ub2e4\ub294 \uc99d\uac70\ub85c \uad00\uac00\uc5d0 \ubc14\uce58\ub3c4\ub85d \ub418\uc5b4 \uc788\uc5c8\uc73c\uba70, \uad00\uac00\uc5d0\uc11c\ub294 \uc7a5\ubd80\uc758 \uae30\ub85d\uacfc \ub300\uc870\ud558\uc5ec \ub9d0\uc774 \uc790\uc5f0\uc0ac\ud55c \uac83\uc774 \ud655\uc778\ub418\uba74 \uac00\uc8fd\uc744 \ub3cc\ub824\uc8fc\uc5c8\uc9c0\ub9cc, \ub300\ubd80\ubd84 \uad00\ub9ac\ub4e4\uc5d0 \uc758\ud574 \ubaa9\uc790\ub4e4\uc758 \uacfc\uc2e4\ub85c \ucc98\ub9ac\ub418\uc5b4 \ubaa9\uc790\ub4e4\uc774 \ub9d0\uac12\uc744 \ubb3c\uc5b4\ub0b4\ub824\ub2e4 \ud30c\uc0b0\ud558\ub294 \uacbd\uc6b0\uac00 \ub9ce\uc558\ub2e4.", "answer": "\ubaa9\uc790", "sentence": "\uc870\uc120 \uc2dc\ub300\uc758 \uc81c\uc8fc\ubaa9\ub9c8\uc7a5\uc740 \uc758\uc815\ubd80, \ubcd1\uc870 \ubc0f \uc0ac\ubcf5\uc2dc(\u53f8\u50d5\u5bfa)\uc758 \uc9c0\ud718 \uac10\ub3c5 \uc544\ub798\uc11c \uc804\ub77c\ub3c4\uac10\ucc30\uc0ac, \uc81c\uc8fc\ubaa9\uc0ac, \uac10\ubaa9\uad00(\uc81c\uc8fc \ud310\uad00 \u00b7 \uc815\uc758\ud604\uac10 \u00b7 \ub300\uc815\ud604\uac10 \uacb8\uc784), \ub9c8\uac10, \uad70\ub450, \uad70\ubd80, \ubaa9\uc790 \ub4f1 \uc21c\uc11c\uc758 \uacc4\uae09\uc73c\ub85c \ubc30\uce58\ub3fc \uc6b4\uc601\ub418\uc5c8\ub2e4.", "paragraph_sentence": " \uc870\uc120 \uc2dc\ub300\uc758 \uc81c\uc8fc\ubaa9\ub9c8\uc7a5\uc740 \uc758\uc815\ubd80, \ubcd1\uc870 \ubc0f \uc0ac\ubcf5\uc2dc(\u53f8\u50d5\u5bfa)\uc758 \uc9c0\ud718 \uac10\ub3c5 \uc544\ub798\uc11c \uc804\ub77c\ub3c4\uac10\ucc30\uc0ac, \uc81c\uc8fc\ubaa9\uc0ac, \uac10\ubaa9\uad00(\uc81c\uc8fc \ud310\uad00 \u00b7 \uc815\uc758\ud604\uac10 \u00b7 \ub300\uc815\ud604\uac10 \uacb8\uc784), \ub9c8\uac10, \uad70\ub450, \uad70\ubd80, \ubaa9\uc790 \ub4f1 \uc21c\uc11c\uc758 \uacc4\uae09\uc73c\ub85c \ubc30\uce58\ub3fc \uc6b4\uc601\ub418\uc5c8\ub2e4. \uc81c\uc8fc\ub9c8\ub294 \uc655\uc774 \ud0c0\ub294 \uc5b4\uc2b9\ub9c8\ub97c \ube44\ub86f\ud574 \uad70\ub9c8, \uc885\ub9c8, \uc5ed\ub9c8, \ud30c\ubc1c\ub9c8, \ud0dc\ub9c8, \ub9cc\ub9c8, \ubcf5\ub9c8 \ub4f1 \ub2e4\uc591\ud558\uac8c \ud65c\uc6a9\ub418\uc5c8\uc73c\uba70, \ud55c\uc591\uc758 \uc870\uc815\uc5d0 \ud574\ub9c8\ub2e4 200\ud544\uacfc \uc9c4\uc0c1\ub9c8 60\ud544\uc744 \ubc14\ucce4\uc744 \ubfd0 \uc544\ub2c8\ub77c \uc81c\uc8fc\ubaa9\uc0ac \ub4f1 \uad00\ub9ac\uc5d0\uac8c\ub3c4 \ubc14\uce58\ub3c4\ub85d \uaddc\uc815\ub418\uc5b4, 3\ub144 \uc8fc\uae30\ub85c \ub9d0\uc744 \ucc28\ucd9c\ud588\ub2e4. \ub9d0\uc758 \uc0c1\ud0dc\uac00 \uc88b\uc9c0 \uc54a\uc744 \uacbd\uc6b0 \ubaa9\uc790\uc758 \ucc45\uc784\uc73c\ub85c \uc804\uac00\ub418\uc5b4 \ub9d0\uac12\uc744 \ubcf8\uc778\ubfd0 \uc544\ub2c8\ub77c \uce5c\ucc99\ub4e4\uae4c\uc9c0 \ub098\uc11c\uc11c \ubc30\uc0c1 \ucc45\uc784\uc744 \uc9c0\uace0 \ubb3c\uc5b4\ub0b4\uc57c \ud588\ub294\ub370, \ud480\uc774 \uc5c6\ub294 \ucd08\ubd04\uc774\ub098 \uaca8\uc6b8\uc774 \ub418\uba74 \uad76\uc8fc\ub824 \uc8fd\ub294 \ub9d0\uc774 \ub9ce\uc558\uace0, \uc774\ub54c \ubaa9\uc790\ub294 \uc8fd\uc740 \ub9d0\uc758 \uac00\uc8fd\uc744 \ubc97\uaca8\uc11c \ub9d0\uc774 \uc790\uc5f0\uc0ac\ud588\ub2e4\ub294 \uc99d\uac70\ub85c \uad00\uac00\uc5d0 \ubc14\uce58\ub3c4\ub85d \ub418\uc5b4 \uc788\uc5c8\uc73c\uba70, \uad00\uac00\uc5d0\uc11c\ub294 \uc7a5\ubd80\uc758 \uae30\ub85d\uacfc \ub300\uc870\ud558\uc5ec \ub9d0\uc774 \uc790\uc5f0\uc0ac\ud55c \uac83\uc774 \ud655\uc778\ub418\uba74 \uac00\uc8fd\uc744 \ub3cc\ub824\uc8fc\uc5c8\uc9c0\ub9cc, \ub300\ubd80\ubd84 \uad00\ub9ac\ub4e4\uc5d0 \uc758\ud574 \ubaa9\uc790\ub4e4\uc758 \uacfc\uc2e4\ub85c \ucc98\ub9ac\ub418\uc5b4 \ubaa9\uc790\ub4e4\uc774 \ub9d0\uac12\uc744 \ubb3c\uc5b4\ub0b4\ub824\ub2e4 \ud30c\uc0b0\ud558\ub294 \uacbd\uc6b0\uac00 \ub9ce\uc558\ub2e4.", "paragraph_answer": "\uc870\uc120 \uc2dc\ub300\uc758 \uc81c\uc8fc\ubaa9\ub9c8\uc7a5\uc740 \uc758\uc815\ubd80, \ubcd1\uc870 \ubc0f \uc0ac\ubcf5\uc2dc(\u53f8\u50d5\u5bfa)\uc758 \uc9c0\ud718 \uac10\ub3c5 \uc544\ub798\uc11c \uc804\ub77c\ub3c4\uac10\ucc30\uc0ac, \uc81c\uc8fc\ubaa9\uc0ac, \uac10\ubaa9\uad00(\uc81c\uc8fc \ud310\uad00 \u00b7 \uc815\uc758\ud604\uac10 \u00b7 \ub300\uc815\ud604\uac10 \uacb8\uc784), \ub9c8\uac10, \uad70\ub450, \uad70\ubd80, \ubaa9\uc790 \ub4f1 \uc21c\uc11c\uc758 \uacc4\uae09\uc73c\ub85c \ubc30\uce58\ub3fc \uc6b4\uc601\ub418\uc5c8\ub2e4. \uc81c\uc8fc\ub9c8\ub294 \uc655\uc774 \ud0c0\ub294 \uc5b4\uc2b9\ub9c8\ub97c \ube44\ub86f\ud574 \uad70\ub9c8, \uc885\ub9c8, \uc5ed\ub9c8, \ud30c\ubc1c\ub9c8, \ud0dc\ub9c8, \ub9cc\ub9c8, \ubcf5\ub9c8 \ub4f1 \ub2e4\uc591\ud558\uac8c \ud65c\uc6a9\ub418\uc5c8\uc73c\uba70, \ud55c\uc591\uc758 \uc870\uc815\uc5d0 \ud574\ub9c8\ub2e4 200\ud544\uacfc \uc9c4\uc0c1\ub9c8 60\ud544\uc744 \ubc14\ucce4\uc744 \ubfd0 \uc544\ub2c8\ub77c \uc81c\uc8fc\ubaa9\uc0ac \ub4f1 \uad00\ub9ac\uc5d0\uac8c\ub3c4 \ubc14\uce58\ub3c4\ub85d \uaddc\uc815\ub418\uc5b4, 3\ub144 \uc8fc\uae30\ub85c \ub9d0\uc744 \ucc28\ucd9c\ud588\ub2e4. \ub9d0\uc758 \uc0c1\ud0dc\uac00 \uc88b\uc9c0 \uc54a\uc744 \uacbd\uc6b0 \ubaa9\uc790\uc758 \ucc45\uc784\uc73c\ub85c \uc804\uac00\ub418\uc5b4 \ub9d0\uac12\uc744 \ubcf8\uc778\ubfd0 \uc544\ub2c8\ub77c \uce5c\ucc99\ub4e4\uae4c\uc9c0 \ub098\uc11c\uc11c \ubc30\uc0c1 \ucc45\uc784\uc744 \uc9c0\uace0 \ubb3c\uc5b4\ub0b4\uc57c \ud588\ub294\ub370, \ud480\uc774 \uc5c6\ub294 \ucd08\ubd04\uc774\ub098 \uaca8\uc6b8\uc774 \ub418\uba74 \uad76\uc8fc\ub824 \uc8fd\ub294 \ub9d0\uc774 \ub9ce\uc558\uace0, \uc774\ub54c \ubaa9\uc790\ub294 \uc8fd\uc740 \ub9d0\uc758 \uac00\uc8fd\uc744 \ubc97\uaca8\uc11c \ub9d0\uc774 \uc790\uc5f0\uc0ac\ud588\ub2e4\ub294 \uc99d\uac70\ub85c \uad00\uac00\uc5d0 \ubc14\uce58\ub3c4\ub85d \ub418\uc5b4 \uc788\uc5c8\uc73c\uba70, \uad00\uac00\uc5d0\uc11c\ub294 \uc7a5\ubd80\uc758 \uae30\ub85d\uacfc \ub300\uc870\ud558\uc5ec \ub9d0\uc774 \uc790\uc5f0\uc0ac\ud55c \uac83\uc774 \ud655\uc778\ub418\uba74 \uac00\uc8fd\uc744 \ub3cc\ub824\uc8fc\uc5c8\uc9c0\ub9cc, \ub300\ubd80\ubd84 \uad00\ub9ac\ub4e4\uc5d0 \uc758\ud574 \ubaa9\uc790\ub4e4\uc758 \uacfc\uc2e4\ub85c \ucc98\ub9ac\ub418\uc5b4 \ubaa9\uc790\ub4e4\uc774 \ub9d0\uac12\uc744 \ubb3c\uc5b4\ub0b4\ub824\ub2e4 \ud30c\uc0b0\ud558\ub294 \uacbd\uc6b0\uac00 \ub9ce\uc558\ub2e4.", "sentence_answer": "\uc870\uc120 \uc2dc\ub300\uc758 \uc81c\uc8fc\ubaa9\ub9c8\uc7a5\uc740 \uc758\uc815\ubd80, \ubcd1\uc870 \ubc0f \uc0ac\ubcf5\uc2dc(\u53f8\u50d5\u5bfa)\uc758 \uc9c0\ud718 \uac10\ub3c5 \uc544\ub798\uc11c \uc804\ub77c\ub3c4\uac10\ucc30\uc0ac, \uc81c\uc8fc\ubaa9\uc0ac, \uac10\ubaa9\uad00(\uc81c\uc8fc \ud310\uad00 \u00b7 \uc815\uc758\ud604\uac10 \u00b7 \ub300\uc815\ud604\uac10 \uacb8\uc784), \ub9c8\uac10, \uad70\ub450, \uad70\ubd80, \ubaa9\uc790 \ub4f1 \uc21c\uc11c\uc758 \uacc4\uae09\uc73c\ub85c \ubc30\uce58\ub3fc \uc6b4\uc601\ub418\uc5c8\ub2e4.", "paragraph_id": "6594323-2-2", "paragraph_question": "question: \ub9d0\uac12\uc744 \ubb3c\uc5b4\ub0b4\ub824\ub2e4 \ud30c\uc0b0\ud558\ub294 \uacbd\uc6b0\uac00 \ub9ce\uc558\ub358 \uc774\ub4e4\uc740?, context: \uc870\uc120 \uc2dc\ub300\uc758 \uc81c\uc8fc\ubaa9\ub9c8\uc7a5\uc740 \uc758\uc815\ubd80, \ubcd1\uc870 \ubc0f \uc0ac\ubcf5\uc2dc(\u53f8\u50d5\u5bfa)\uc758 \uc9c0\ud718 \uac10\ub3c5 \uc544\ub798\uc11c \uc804\ub77c\ub3c4\uac10\ucc30\uc0ac, \uc81c\uc8fc\ubaa9\uc0ac, \uac10\ubaa9\uad00(\uc81c\uc8fc \ud310\uad00 \u00b7 \uc815\uc758\ud604\uac10 \u00b7 \ub300\uc815\ud604\uac10 \uacb8\uc784), \ub9c8\uac10, \uad70\ub450, \uad70\ubd80, \ubaa9\uc790 \ub4f1 \uc21c\uc11c\uc758 \uacc4\uae09\uc73c\ub85c \ubc30\uce58\ub3fc \uc6b4\uc601\ub418\uc5c8\ub2e4. \uc81c\uc8fc\ub9c8\ub294 \uc655\uc774 \ud0c0\ub294 \uc5b4\uc2b9\ub9c8\ub97c \ube44\ub86f\ud574 \uad70\ub9c8, \uc885\ub9c8, \uc5ed\ub9c8, \ud30c\ubc1c\ub9c8, \ud0dc\ub9c8, \ub9cc\ub9c8, \ubcf5\ub9c8 \ub4f1 \ub2e4\uc591\ud558\uac8c \ud65c\uc6a9\ub418\uc5c8\uc73c\uba70, \ud55c\uc591\uc758 \uc870\uc815\uc5d0 \ud574\ub9c8\ub2e4 200\ud544\uacfc \uc9c4\uc0c1\ub9c8 60\ud544\uc744 \ubc14\ucce4\uc744 \ubfd0 \uc544\ub2c8\ub77c \uc81c\uc8fc\ubaa9\uc0ac \ub4f1 \uad00\ub9ac\uc5d0\uac8c\ub3c4 \ubc14\uce58\ub3c4\ub85d \uaddc\uc815\ub418\uc5b4, 3\ub144 \uc8fc\uae30\ub85c \ub9d0\uc744 \ucc28\ucd9c\ud588\ub2e4. \ub9d0\uc758 \uc0c1\ud0dc\uac00 \uc88b\uc9c0 \uc54a\uc744 \uacbd\uc6b0 \ubaa9\uc790\uc758 \ucc45\uc784\uc73c\ub85c \uc804\uac00\ub418\uc5b4 \ub9d0\uac12\uc744 \ubcf8\uc778\ubfd0 \uc544\ub2c8\ub77c \uce5c\ucc99\ub4e4\uae4c\uc9c0 \ub098\uc11c\uc11c \ubc30\uc0c1 \ucc45\uc784\uc744 \uc9c0\uace0 \ubb3c\uc5b4\ub0b4\uc57c \ud588\ub294\ub370, \ud480\uc774 \uc5c6\ub294 \ucd08\ubd04\uc774\ub098 \uaca8\uc6b8\uc774 \ub418\uba74 \uad76\uc8fc\ub824 \uc8fd\ub294 \ub9d0\uc774 \ub9ce\uc558\uace0, \uc774\ub54c \ubaa9\uc790\ub294 \uc8fd\uc740 \ub9d0\uc758 \uac00\uc8fd\uc744 \ubc97\uaca8\uc11c \ub9d0\uc774 \uc790\uc5f0\uc0ac\ud588\ub2e4\ub294 \uc99d\uac70\ub85c \uad00\uac00\uc5d0 \ubc14\uce58\ub3c4\ub85d \ub418\uc5b4 \uc788\uc5c8\uc73c\uba70, \uad00\uac00\uc5d0\uc11c\ub294 \uc7a5\ubd80\uc758 \uae30\ub85d\uacfc \ub300\uc870\ud558\uc5ec \ub9d0\uc774 \uc790\uc5f0\uc0ac\ud55c \uac83\uc774 \ud655\uc778\ub418\uba74 \uac00\uc8fd\uc744 \ub3cc\ub824\uc8fc\uc5c8\uc9c0\ub9cc, \ub300\ubd80\ubd84 \uad00\ub9ac\ub4e4\uc5d0 \uc758\ud574 \ubaa9\uc790\ub4e4\uc758 \uacfc\uc2e4\ub85c \ucc98\ub9ac\ub418\uc5b4 \ubaa9\uc790\ub4e4\uc774 \ub9d0\uac12\uc744 \ubb3c\uc5b4\ub0b4\ub824\ub2e4 \ud30c\uc0b0\ud558\ub294 \uacbd\uc6b0\uac00 \ub9ce\uc558\ub2e4."} {"question": "\uc704\uc131\uc774 \ub2ec \uacf5\uc804 \uada4\ub3c4\uc5d0 \uc9c4\uc785\ud55c \ub0a0\uc740 \uc5b8\uc81c\uc778\uac00?", "paragraph": "\uc77c\ubcf8 \uc6b0\uc8fc\ud56d\uacf5\uc5f0\uad6c\uac1c\ubc1c\uae30\uad6c(JAXA)\ub294 \uc77c\ubcf8\uc758 \uc6b0\uc8fc\uae30\uad00\uc73c\ub85c \uc6b0\uc8fc\uc640 \ud589\uc131 \uc5f0\uad6c\ub97c \uac10\ub3c5\ud558\uace0, \ud56d\uacf5\uae30 \uc0b0\uc5c5\uacfc \ub85c\ucf13\uacfc \uc704\uc131 \uac1c\ubc1c\uc744 \uc9c0\ud718\ud558\uace0 \uc788\ub2e4. \uc774 \uae30\uad6c\ub294 \uad6d\uc81c \uc6b0\uc8fc\uc815\uac70\uc7a5\uc5d0 \ucc38\uc5ec\ud558\uace0 \uc788\uc73c\uba70 \uc77c\ubcf8 \uc2e4\ud5d8 \ubaa8\ub4c8\uc744 \uad6d\uc81c\uc6b0\uc8fc\uc815\uac70\uc7a5\uc5d0 \ucd94\uac00\ud558\uc600\ub2e4. JAXA\ub294 \uae08\uc131 \uae30\ud6c4 \ud0d0\uc0ac \uc778\uacf5\uc704\uc131(PLANET-C)\uc744 2010\ub144\uc5d0 \uc3d8\uc544\uc62c\ub9ac\ub294 \uac83\uacfc \uac19\uc740 \uc6b0\uc8fc \ud0d0\ud5d8 \uacc4\ud68d\uc744 \uac16\uace0 \uc788\ub2e4. \ub610\ud55c JAXA\ub294 2013\ub144\uc5d0 \uc218\uc131\ud0d0\uc0ac\uc704\uc131\uc744 \uc3d8\uc544\uc62c\ub9ac\ub824\uace0 \uacc4\ud68d\ud558\uace0 \uc788\uc73c\uba70 2030\ub144\uc5d0\ub294 \ub2ec\uc5d0 \uc2dd\ubbfc\uc9c0\ub97c \uac74\uc124\ud558\ub824\uace0 \ud558\uace0 \uc788\ub2e4. JAXA\ub294 2007\ub144 9\uc6d4 14\uc77c\uc5d0 \ub2ec \ud0d0\uc0ac \uc704\uc131\uc778 \uc140\ub808\ub124(SELENE, \uc77c\ubcf8\uc5b4: \u304b\u3050\u3084 \uac00\uad6c\uc57c/\uce74\uad6c\uc57c)\ub97c \ubc1c\uc0ac\ud588\ub2e4. \uc140\ub808\ub124\ub294 \ubbf8\uad6d\uc758 \uc544\ud3f4\ub85c \uacc4\ud68d \uc774\ud6c4\uc5d0 \uac00\uc7a5 \ud070 \uaddc\ubaa8\uc758 \ub2ec \ud0d0\uc0ac \uacc4\ud68d\uc774\ub2e4. \uc774 \uacc4\ud68d\uc758 \ubaa9\ud45c\ub294 \ub2ec\uc758 \uae30\uc6d0\uacfc \ubc1c\ub2ec\uc5d0 \ub300\ud55c \uc790\ub8cc\ub97c \ubaa8\uc73c\ub294 \uac83\uc774\ub2e4. \uc704\uc131\uc740 2007\ub144 10\uc6d4 4\uc77c\uc5d0 \ub2ec \uacf5\uc804 \uada4\ub3c4\uc5d0 \uc9c4\uc785\ud558\uc600\ub2e4.", "answer": "2007\ub144 10\uc6d4 4\uc77c", "sentence": "\uc704\uc131\uc740 2007\ub144 10\uc6d4 4\uc77c \uc5d0 \ub2ec \uacf5\uc804 \uada4\ub3c4\uc5d0 \uc9c4\uc785\ud558\uc600\ub2e4.", "paragraph_sentence": "\uc77c\ubcf8 \uc6b0\uc8fc\ud56d\uacf5\uc5f0\uad6c\uac1c\ubc1c\uae30\uad6c(JAXA)\ub294 \uc77c\ubcf8\uc758 \uc6b0\uc8fc\uae30\uad00\uc73c\ub85c \uc6b0\uc8fc\uc640 \ud589\uc131 \uc5f0\uad6c\ub97c \uac10\ub3c5\ud558\uace0, \ud56d\uacf5\uae30 \uc0b0\uc5c5\uacfc \ub85c\ucf13\uacfc \uc704\uc131 \uac1c\ubc1c\uc744 \uc9c0\ud718\ud558\uace0 \uc788\ub2e4. \uc774 \uae30\uad6c\ub294 \uad6d\uc81c \uc6b0\uc8fc\uc815\uac70\uc7a5\uc5d0 \ucc38\uc5ec\ud558\uace0 \uc788\uc73c\uba70 \uc77c\ubcf8 \uc2e4\ud5d8 \ubaa8\ub4c8\uc744 \uad6d\uc81c\uc6b0\uc8fc\uc815\uac70\uc7a5\uc5d0 \ucd94\uac00\ud558\uc600\ub2e4. JAXA\ub294 \uae08\uc131 \uae30\ud6c4 \ud0d0\uc0ac \uc778\uacf5\uc704\uc131(PLANET-C)\uc744 2010\ub144\uc5d0 \uc3d8\uc544\uc62c\ub9ac\ub294 \uac83\uacfc \uac19\uc740 \uc6b0\uc8fc \ud0d0\ud5d8 \uacc4\ud68d\uc744 \uac16\uace0 \uc788\ub2e4. \ub610\ud55c JAXA\ub294 2013\ub144\uc5d0 \uc218\uc131\ud0d0\uc0ac\uc704\uc131\uc744 \uc3d8\uc544\uc62c\ub9ac\ub824\uace0 \uacc4\ud68d\ud558\uace0 \uc788\uc73c\uba70 2030\ub144\uc5d0\ub294 \ub2ec\uc5d0 \uc2dd\ubbfc\uc9c0\ub97c \uac74\uc124\ud558\ub824\uace0 \ud558\uace0 \uc788\ub2e4. JAXA\ub294 2007\ub144 9\uc6d4 14\uc77c\uc5d0 \ub2ec \ud0d0\uc0ac \uc704\uc131\uc778 \uc140\ub808\ub124(SELENE, \uc77c\ubcf8\uc5b4: \u304b\u3050\u3084 \uac00\uad6c\uc57c/\uce74\uad6c\uc57c)\ub97c \ubc1c\uc0ac\ud588\ub2e4. \uc140\ub808\ub124\ub294 \ubbf8\uad6d\uc758 \uc544\ud3f4\ub85c \uacc4\ud68d \uc774\ud6c4\uc5d0 \uac00\uc7a5 \ud070 \uaddc\ubaa8\uc758 \ub2ec \ud0d0\uc0ac \uacc4\ud68d\uc774\ub2e4. \uc774 \uacc4\ud68d\uc758 \ubaa9\ud45c\ub294 \ub2ec\uc758 \uae30\uc6d0\uacfc \ubc1c\ub2ec\uc5d0 \ub300\ud55c \uc790\ub8cc\ub97c \ubaa8\uc73c\ub294 \uac83\uc774\ub2e4. \uc704\uc131\uc740 2007\ub144 10\uc6d4 4\uc77c \uc5d0 \ub2ec \uacf5\uc804 \uada4\ub3c4\uc5d0 \uc9c4\uc785\ud558\uc600\ub2e4. ", "paragraph_answer": "\uc77c\ubcf8 \uc6b0\uc8fc\ud56d\uacf5\uc5f0\uad6c\uac1c\ubc1c\uae30\uad6c(JAXA)\ub294 \uc77c\ubcf8\uc758 \uc6b0\uc8fc\uae30\uad00\uc73c\ub85c \uc6b0\uc8fc\uc640 \ud589\uc131 \uc5f0\uad6c\ub97c \uac10\ub3c5\ud558\uace0, \ud56d\uacf5\uae30 \uc0b0\uc5c5\uacfc \ub85c\ucf13\uacfc \uc704\uc131 \uac1c\ubc1c\uc744 \uc9c0\ud718\ud558\uace0 \uc788\ub2e4. \uc774 \uae30\uad6c\ub294 \uad6d\uc81c \uc6b0\uc8fc\uc815\uac70\uc7a5\uc5d0 \ucc38\uc5ec\ud558\uace0 \uc788\uc73c\uba70 \uc77c\ubcf8 \uc2e4\ud5d8 \ubaa8\ub4c8\uc744 \uad6d\uc81c\uc6b0\uc8fc\uc815\uac70\uc7a5\uc5d0 \ucd94\uac00\ud558\uc600\ub2e4. JAXA\ub294 \uae08\uc131 \uae30\ud6c4 \ud0d0\uc0ac \uc778\uacf5\uc704\uc131(PLANET-C)\uc744 2010\ub144\uc5d0 \uc3d8\uc544\uc62c\ub9ac\ub294 \uac83\uacfc \uac19\uc740 \uc6b0\uc8fc \ud0d0\ud5d8 \uacc4\ud68d\uc744 \uac16\uace0 \uc788\ub2e4. \ub610\ud55c JAXA\ub294 2013\ub144\uc5d0 \uc218\uc131\ud0d0\uc0ac\uc704\uc131\uc744 \uc3d8\uc544\uc62c\ub9ac\ub824\uace0 \uacc4\ud68d\ud558\uace0 \uc788\uc73c\uba70 2030\ub144\uc5d0\ub294 \ub2ec\uc5d0 \uc2dd\ubbfc\uc9c0\ub97c \uac74\uc124\ud558\ub824\uace0 \ud558\uace0 \uc788\ub2e4. JAXA\ub294 2007\ub144 9\uc6d4 14\uc77c\uc5d0 \ub2ec \ud0d0\uc0ac \uc704\uc131\uc778 \uc140\ub808\ub124(SELENE, \uc77c\ubcf8\uc5b4: \u304b\u3050\u3084 \uac00\uad6c\uc57c/\uce74\uad6c\uc57c)\ub97c \ubc1c\uc0ac\ud588\ub2e4. \uc140\ub808\ub124\ub294 \ubbf8\uad6d\uc758 \uc544\ud3f4\ub85c \uacc4\ud68d \uc774\ud6c4\uc5d0 \uac00\uc7a5 \ud070 \uaddc\ubaa8\uc758 \ub2ec \ud0d0\uc0ac \uacc4\ud68d\uc774\ub2e4. \uc774 \uacc4\ud68d\uc758 \ubaa9\ud45c\ub294 \ub2ec\uc758 \uae30\uc6d0\uacfc \ubc1c\ub2ec\uc5d0 \ub300\ud55c \uc790\ub8cc\ub97c \ubaa8\uc73c\ub294 \uac83\uc774\ub2e4. \uc704\uc131\uc740 2007\ub144 10\uc6d4 4\uc77c \uc5d0 \ub2ec \uacf5\uc804 \uada4\ub3c4\uc5d0 \uc9c4\uc785\ud558\uc600\ub2e4.", "sentence_answer": "\uc704\uc131\uc740 2007\ub144 10\uc6d4 4\uc77c \uc5d0 \ub2ec \uacf5\uc804 \uada4\ub3c4\uc5d0 \uc9c4\uc785\ud558\uc600\ub2e4.", "paragraph_id": "6591477-26-1", "paragraph_question": "question: \uc704\uc131\uc774 \ub2ec \uacf5\uc804 \uada4\ub3c4\uc5d0 \uc9c4\uc785\ud55c \ub0a0\uc740 \uc5b8\uc81c\uc778\uac00?, context: \uc77c\ubcf8 \uc6b0\uc8fc\ud56d\uacf5\uc5f0\uad6c\uac1c\ubc1c\uae30\uad6c(JAXA)\ub294 \uc77c\ubcf8\uc758 \uc6b0\uc8fc\uae30\uad00\uc73c\ub85c \uc6b0\uc8fc\uc640 \ud589\uc131 \uc5f0\uad6c\ub97c \uac10\ub3c5\ud558\uace0, \ud56d\uacf5\uae30 \uc0b0\uc5c5\uacfc \ub85c\ucf13\uacfc \uc704\uc131 \uac1c\ubc1c\uc744 \uc9c0\ud718\ud558\uace0 \uc788\ub2e4. \uc774 \uae30\uad6c\ub294 \uad6d\uc81c \uc6b0\uc8fc\uc815\uac70\uc7a5\uc5d0 \ucc38\uc5ec\ud558\uace0 \uc788\uc73c\uba70 \uc77c\ubcf8 \uc2e4\ud5d8 \ubaa8\ub4c8\uc744 \uad6d\uc81c\uc6b0\uc8fc\uc815\uac70\uc7a5\uc5d0 \ucd94\uac00\ud558\uc600\ub2e4. JAXA\ub294 \uae08\uc131 \uae30\ud6c4 \ud0d0\uc0ac \uc778\uacf5\uc704\uc131(PLANET-C)\uc744 2010\ub144\uc5d0 \uc3d8\uc544\uc62c\ub9ac\ub294 \uac83\uacfc \uac19\uc740 \uc6b0\uc8fc \ud0d0\ud5d8 \uacc4\ud68d\uc744 \uac16\uace0 \uc788\ub2e4. \ub610\ud55c JAXA\ub294 2013\ub144\uc5d0 \uc218\uc131\ud0d0\uc0ac\uc704\uc131\uc744 \uc3d8\uc544\uc62c\ub9ac\ub824\uace0 \uacc4\ud68d\ud558\uace0 \uc788\uc73c\uba70 2030\ub144\uc5d0\ub294 \ub2ec\uc5d0 \uc2dd\ubbfc\uc9c0\ub97c \uac74\uc124\ud558\ub824\uace0 \ud558\uace0 \uc788\ub2e4. JAXA\ub294 2007\ub144 9\uc6d4 14\uc77c\uc5d0 \ub2ec \ud0d0\uc0ac \uc704\uc131\uc778 \uc140\ub808\ub124(SELENE, \uc77c\ubcf8\uc5b4: \u304b\u3050\u3084 \uac00\uad6c\uc57c/\uce74\uad6c\uc57c)\ub97c \ubc1c\uc0ac\ud588\ub2e4. \uc140\ub808\ub124\ub294 \ubbf8\uad6d\uc758 \uc544\ud3f4\ub85c \uacc4\ud68d \uc774\ud6c4\uc5d0 \uac00\uc7a5 \ud070 \uaddc\ubaa8\uc758 \ub2ec \ud0d0\uc0ac \uacc4\ud68d\uc774\ub2e4. \uc774 \uacc4\ud68d\uc758 \ubaa9\ud45c\ub294 \ub2ec\uc758 \uae30\uc6d0\uacfc \ubc1c\ub2ec\uc5d0 \ub300\ud55c \uc790\ub8cc\ub97c \ubaa8\uc73c\ub294 \uac83\uc774\ub2e4. \uc704\uc131\uc740 2007\ub144 10\uc6d4 4\uc77c\uc5d0 \ub2ec \uacf5\uc804 \uada4\ub3c4\uc5d0 \uc9c4\uc785\ud558\uc600\ub2e4."} {"question": "\uc5ed\ub300 \ub0a8\uc790 \uc120\uc218\ub4e4 \uc911 20\uac1c\uc758 \uadf8\ub79c\ub4dc \uc2ac\ub7a8 \ub2e8\uc2dd \ud0c0\uc774\ud2c0\uc744 \ud68d\ub4dd\ud55c \ud14c\ub2c8\uc2a4 \uc120\uc218\ub294?", "paragraph": "\ud398\ub354\ub7ec\ub294 \ub0a8\uc790 \ud14c\ub2c8\uc2a4\uc640 \uad00\ub828\ud574 \ub9ce\uc740 \uc138\uacc4 \uae30\ub85d\uc744 \ubcf4\uc720\ud558\uace0 \uc788\ub2e4. \uc5ed\ub300 \ub0a8\uc790 \uc120\uc218\ub4e4 \uc911 \uac00\uc7a5 \ub9ce\uc740 \ucd1d 20\uac1c\uc758 \uadf8\ub79c\ub4dc \uc2ac\ub7a8 \ub2e8\uc2dd \ud0c0\uc774\ud2c0\uc744 \ud68d\ub4dd\ud588\ub2e4. \ub610\ud55c \uadf8\ub294 2009\ub144 \ud504\ub791\uc2a4 \uc624\ud508\uc5d0\uc11c \uc6b0\uc2b9\ud558\uba74\uc11c \ucee4\ub9ac\uc5b4 \uadf8\ub79c\ub4dc \uc2ac\ub7a8\uc744 \ub2ec\uc131\ud55c \uc5ed\ub300 7\ubc88\uc9f8 \ub0a8\uc790 \uc120\uc218\uac00 \ub418\uc5c8\ub2e4. \uadf8\ub294 \uc5ed\ub300 \ub0a8\uc790 \uc120\uc218\ub4e4 \uc911 \uac00\uc7a5 \ub9ce\uc740 \ucd1d 30\ud68c\uc758 \uadf8\ub79c\ub4dc \uc2ac\ub7a8 \uacb0\uc2b9\uc5d0 \uc9c4\ucd9c\ud588\ub2e4. \ub610\ud55c 2004\ub144 \uc714\ube14\ub358 \uc6b0\uc2b9\uc744 \uc2dc\uc791\uc73c\ub85c 2010\ub144 1\uc6d4 \ud638\uc8fc \uc624\ud508\uc5d0\uc11c \uc6b0\uc2b9\ud558\uba74\uc11c \uc57d 6\ub144\uc5d0 \uac78\uccd0 \uadf8\ub79c\ub4dc \uc2ac\ub7a8 \uc900\uacb0\uc2b9 23\ud68c \uc5f0\uc18d \uc9c4\ucd9c\uc744 \uae30\ub85d, \uc774 \ubd80\ubb38 \ucd5c\uace0 \uae30\ub85d\uc744 \uc138\uc6e0\uc73c\uba70 \uc774\ub294 \uc885\uc804 \ucd5c\uace0 \uae30\ub85d\ubcf4\ub2e4 \ub450 \ubc30 \uc774\uc0c1 \ub9ce\uc740 \uac83\uc774\uc5c8\ub2e4. \uadf8\uac00 \uc120\uc218 \uc0dd\ud65c\ub3d9\uc548 \uc774\ub904\ub0b8 \ub6f0\uc5b4\ub09c \uc131\uc801\ub4e4\uacfc \uae30\ub85d\ub4e4\uc744 \uc778\uc815\ubc1b\uc544 \uadf8\ub294 2005\ub144\ubd80\ud130 2008\ub144\uae4c\uc9c0 4\ub144 \uc5f0\uc18d \ub85c\ub808\uc6b0\uc2a4 \uc62c\ud574\uc758 \uc138\uacc4 \uc2a4\ud3ec\uce20\uc778\uc0c1\uc744 \uc218\uc0c1\ud558\uc600\ub2e4.", "answer": "\ud398\ub354\ub7ec", "sentence": "\ud398\ub354\ub7ec \ub294 \ub0a8\uc790 \ud14c\ub2c8\uc2a4\uc640 \uad00\ub828\ud574 \ub9ce\uc740 \uc138\uacc4 \uae30\ub85d\uc744 \ubcf4\uc720\ud558\uace0 \uc788\ub2e4.", "paragraph_sentence": " \ud398\ub354\ub7ec \ub294 \ub0a8\uc790 \ud14c\ub2c8\uc2a4\uc640 \uad00\ub828\ud574 \ub9ce\uc740 \uc138\uacc4 \uae30\ub85d\uc744 \ubcf4\uc720\ud558\uace0 \uc788\ub2e4. \uc5ed\ub300 \ub0a8\uc790 \uc120\uc218\ub4e4 \uc911 \uac00\uc7a5 \ub9ce\uc740 \ucd1d 20\uac1c\uc758 \uadf8\ub79c\ub4dc \uc2ac\ub7a8 \ub2e8\uc2dd \ud0c0\uc774\ud2c0\uc744 \ud68d\ub4dd\ud588\ub2e4. \ub610\ud55c \uadf8\ub294 2009\ub144 \ud504\ub791\uc2a4 \uc624\ud508\uc5d0\uc11c \uc6b0\uc2b9\ud558\uba74\uc11c \ucee4\ub9ac\uc5b4 \uadf8\ub79c\ub4dc \uc2ac\ub7a8\uc744 \ub2ec\uc131\ud55c \uc5ed\ub300 7\ubc88\uc9f8 \ub0a8\uc790 \uc120\uc218\uac00 \ub418\uc5c8\ub2e4. \uadf8\ub294 \uc5ed\ub300 \ub0a8\uc790 \uc120\uc218\ub4e4 \uc911 \uac00\uc7a5 \ub9ce\uc740 \ucd1d 30\ud68c\uc758 \uadf8\ub79c\ub4dc \uc2ac\ub7a8 \uacb0\uc2b9\uc5d0 \uc9c4\ucd9c\ud588\ub2e4. \ub610\ud55c 2004\ub144 \uc714\ube14\ub358 \uc6b0\uc2b9\uc744 \uc2dc\uc791\uc73c\ub85c 2010\ub144 1\uc6d4 \ud638\uc8fc \uc624\ud508\uc5d0\uc11c \uc6b0\uc2b9\ud558\uba74\uc11c \uc57d 6\ub144\uc5d0 \uac78\uccd0 \uadf8\ub79c\ub4dc \uc2ac\ub7a8 \uc900\uacb0\uc2b9 23\ud68c \uc5f0\uc18d \uc9c4\ucd9c\uc744 \uae30\ub85d, \uc774 \ubd80\ubb38 \ucd5c\uace0 \uae30\ub85d\uc744 \uc138\uc6e0\uc73c\uba70 \uc774\ub294 \uc885\uc804 \ucd5c\uace0 \uae30\ub85d\ubcf4\ub2e4 \ub450 \ubc30 \uc774\uc0c1 \ub9ce\uc740 \uac83\uc774\uc5c8\ub2e4. \uadf8\uac00 \uc120\uc218 \uc0dd\ud65c\ub3d9\uc548 \uc774\ub904\ub0b8 \ub6f0\uc5b4\ub09c \uc131\uc801\ub4e4\uacfc \uae30\ub85d\ub4e4\uc744 \uc778\uc815\ubc1b\uc544 \uadf8\ub294 2005\ub144\ubd80\ud130 2008\ub144\uae4c\uc9c0 4\ub144 \uc5f0\uc18d \ub85c\ub808\uc6b0\uc2a4 \uc62c\ud574\uc758 \uc138\uacc4 \uc2a4\ud3ec\uce20\uc778\uc0c1\uc744 \uc218\uc0c1\ud558\uc600\ub2e4.", "paragraph_answer": " \ud398\ub354\ub7ec \ub294 \ub0a8\uc790 \ud14c\ub2c8\uc2a4\uc640 \uad00\ub828\ud574 \ub9ce\uc740 \uc138\uacc4 \uae30\ub85d\uc744 \ubcf4\uc720\ud558\uace0 \uc788\ub2e4. \uc5ed\ub300 \ub0a8\uc790 \uc120\uc218\ub4e4 \uc911 \uac00\uc7a5 \ub9ce\uc740 \ucd1d 20\uac1c\uc758 \uadf8\ub79c\ub4dc \uc2ac\ub7a8 \ub2e8\uc2dd \ud0c0\uc774\ud2c0\uc744 \ud68d\ub4dd\ud588\ub2e4. \ub610\ud55c \uadf8\ub294 2009\ub144 \ud504\ub791\uc2a4 \uc624\ud508\uc5d0\uc11c \uc6b0\uc2b9\ud558\uba74\uc11c \ucee4\ub9ac\uc5b4 \uadf8\ub79c\ub4dc \uc2ac\ub7a8\uc744 \ub2ec\uc131\ud55c \uc5ed\ub300 7\ubc88\uc9f8 \ub0a8\uc790 \uc120\uc218\uac00 \ub418\uc5c8\ub2e4. \uadf8\ub294 \uc5ed\ub300 \ub0a8\uc790 \uc120\uc218\ub4e4 \uc911 \uac00\uc7a5 \ub9ce\uc740 \ucd1d 30\ud68c\uc758 \uadf8\ub79c\ub4dc \uc2ac\ub7a8 \uacb0\uc2b9\uc5d0 \uc9c4\ucd9c\ud588\ub2e4. \ub610\ud55c 2004\ub144 \uc714\ube14\ub358 \uc6b0\uc2b9\uc744 \uc2dc\uc791\uc73c\ub85c 2010\ub144 1\uc6d4 \ud638\uc8fc \uc624\ud508\uc5d0\uc11c \uc6b0\uc2b9\ud558\uba74\uc11c \uc57d 6\ub144\uc5d0 \uac78\uccd0 \uadf8\ub79c\ub4dc \uc2ac\ub7a8 \uc900\uacb0\uc2b9 23\ud68c \uc5f0\uc18d \uc9c4\ucd9c\uc744 \uae30\ub85d, \uc774 \ubd80\ubb38 \ucd5c\uace0 \uae30\ub85d\uc744 \uc138\uc6e0\uc73c\uba70 \uc774\ub294 \uc885\uc804 \ucd5c\uace0 \uae30\ub85d\ubcf4\ub2e4 \ub450 \ubc30 \uc774\uc0c1 \ub9ce\uc740 \uac83\uc774\uc5c8\ub2e4. \uadf8\uac00 \uc120\uc218 \uc0dd\ud65c\ub3d9\uc548 \uc774\ub904\ub0b8 \ub6f0\uc5b4\ub09c \uc131\uc801\ub4e4\uacfc \uae30\ub85d\ub4e4\uc744 \uc778\uc815\ubc1b\uc544 \uadf8\ub294 2005\ub144\ubd80\ud130 2008\ub144\uae4c\uc9c0 4\ub144 \uc5f0\uc18d \ub85c\ub808\uc6b0\uc2a4 \uc62c\ud574\uc758 \uc138\uacc4 \uc2a4\ud3ec\uce20\uc778\uc0c1\uc744 \uc218\uc0c1\ud558\uc600\ub2e4.", "sentence_answer": " \ud398\ub354\ub7ec \ub294 \ub0a8\uc790 \ud14c\ub2c8\uc2a4\uc640 \uad00\ub828\ud574 \ub9ce\uc740 \uc138\uacc4 \uae30\ub85d\uc744 \ubcf4\uc720\ud558\uace0 \uc788\ub2e4.", "paragraph_id": "6496518-0-0", "paragraph_question": "question: \uc5ed\ub300 \ub0a8\uc790 \uc120\uc218\ub4e4 \uc911 20\uac1c\uc758 \uadf8\ub79c\ub4dc \uc2ac\ub7a8 \ub2e8\uc2dd \ud0c0\uc774\ud2c0\uc744 \ud68d\ub4dd\ud55c \ud14c\ub2c8\uc2a4 \uc120\uc218\ub294?, context: \ud398\ub354\ub7ec\ub294 \ub0a8\uc790 \ud14c\ub2c8\uc2a4\uc640 \uad00\ub828\ud574 \ub9ce\uc740 \uc138\uacc4 \uae30\ub85d\uc744 \ubcf4\uc720\ud558\uace0 \uc788\ub2e4. \uc5ed\ub300 \ub0a8\uc790 \uc120\uc218\ub4e4 \uc911 \uac00\uc7a5 \ub9ce\uc740 \ucd1d 20\uac1c\uc758 \uadf8\ub79c\ub4dc \uc2ac\ub7a8 \ub2e8\uc2dd \ud0c0\uc774\ud2c0\uc744 \ud68d\ub4dd\ud588\ub2e4. \ub610\ud55c \uadf8\ub294 2009\ub144 \ud504\ub791\uc2a4 \uc624\ud508\uc5d0\uc11c \uc6b0\uc2b9\ud558\uba74\uc11c \ucee4\ub9ac\uc5b4 \uadf8\ub79c\ub4dc \uc2ac\ub7a8\uc744 \ub2ec\uc131\ud55c \uc5ed\ub300 7\ubc88\uc9f8 \ub0a8\uc790 \uc120\uc218\uac00 \ub418\uc5c8\ub2e4. \uadf8\ub294 \uc5ed\ub300 \ub0a8\uc790 \uc120\uc218\ub4e4 \uc911 \uac00\uc7a5 \ub9ce\uc740 \ucd1d 30\ud68c\uc758 \uadf8\ub79c\ub4dc \uc2ac\ub7a8 \uacb0\uc2b9\uc5d0 \uc9c4\ucd9c\ud588\ub2e4. \ub610\ud55c 2004\ub144 \uc714\ube14\ub358 \uc6b0\uc2b9\uc744 \uc2dc\uc791\uc73c\ub85c 2010\ub144 1\uc6d4 \ud638\uc8fc \uc624\ud508\uc5d0\uc11c \uc6b0\uc2b9\ud558\uba74\uc11c \uc57d 6\ub144\uc5d0 \uac78\uccd0 \uadf8\ub79c\ub4dc \uc2ac\ub7a8 \uc900\uacb0\uc2b9 23\ud68c \uc5f0\uc18d \uc9c4\ucd9c\uc744 \uae30\ub85d, \uc774 \ubd80\ubb38 \ucd5c\uace0 \uae30\ub85d\uc744 \uc138\uc6e0\uc73c\uba70 \uc774\ub294 \uc885\uc804 \ucd5c\uace0 \uae30\ub85d\ubcf4\ub2e4 \ub450 \ubc30 \uc774\uc0c1 \ub9ce\uc740 \uac83\uc774\uc5c8\ub2e4. \uadf8\uac00 \uc120\uc218 \uc0dd\ud65c\ub3d9\uc548 \uc774\ub904\ub0b8 \ub6f0\uc5b4\ub09c \uc131\uc801\ub4e4\uacfc \uae30\ub85d\ub4e4\uc744 \uc778\uc815\ubc1b\uc544 \uadf8\ub294 2005\ub144\ubd80\ud130 2008\ub144\uae4c\uc9c0 4\ub144 \uc5f0\uc18d \ub85c\ub808\uc6b0\uc2a4 \uc62c\ud574\uc758 \uc138\uacc4 \uc2a4\ud3ec\uce20\uc778\uc0c1\uc744 \uc218\uc0c1\ud558\uc600\ub2e4."} {"question": "\ub0b4\uc678\uc7a5\uc744 \ub9ac\ub274\uc5bc\ud55c \ucc28\ub7c9\uc774 \ub4f1\uc7a5\ud55c \ud574\ub294?", "paragraph": "2001\ub144\ubd80\ud130 \uc870\uc5d0\uc4f0 \uc2e0\uce78\uc13c\uacfc \ub098\uac00\ub178 \uc2e0\uce78\uc13c \ub178\uc120\uc744 \uc6b4\ud589\ud558\uae30 \uc2dc\uc791\ud588\ub2e4. \ub2f9\uc2dc Max \uc544\uc0ac\ub9c8\ub85c \ub3c4\ucfc4 ~ \uce74\ub8e8\uc774\uc790\uc640 \uad6c\uac04\uc744 \uc6b4\ud589 \ud588\uc73c\ub098 2003\ub144 9\uc6d4\uc5d0 \uc6b4\ud589\uc744 \uc885\ub8cc\ud588\ub2e4. 2005\ub144 12\uc6d4 10\uc77c\uc5d0 \uc13c\ub2e4\uc774 \uc774\ubd81 \uad6c\uac04\uc5d0\uc11c \uc815\uae30 \uc6b4\uc6a9\uc744 \uc885\ub8cc\ud588\ub2e4. 2011\ub144\ubd80\ud130 E5\uacc4\uc758 \ub3c4\uc785\uc73c\ub85c \uc6b4\ud589\uc774 \uc904\uc5b4\ub4e4\uc5c8\ub2e4. 2012\ub144 3\uc6d4\uc5d0 \uc870\uc5d0\uc4f0 \uc2e0\uce78\uc13c \uc5d0\uce58\uace0\uc720\uc790\uc640 \uc774\ubd81 \uad6c\uac04\uc5d0 16\ub7c9 \ud3b8\uc131\uc744 \uc601\uc5c5 \uc6b4\uc804\uc744 \uc2dc\uc791\ud558\uba74\uc11c \uc624\ubbf8\uc57c \uc774\ubd81 \uad6c\uac04 16\ub7c9 \ud3b8\uc131\uc774 \uc6b4\ud589\uc774 \uc885\ub8cc\ub418\uc5c8\ub2e4. \uac19\uc740 \ud574 9\uc6d4 28\uc77c\uc5d0 \uc624\ubbf8\uc57c \uc774\ubd81 \uad6c\uac04 \uc6b4\ud589\uc774 \uc885\ub8cc\ub418\uba74\uc11c \uc57c\ub9c8\ube44\ucf54, \ub098\uc2a4\ub178\uc758 \uacbd\uc6b0 E2\uacc4\ub85c \ubcc0\uacbd\uacfc \ub3d9\uc2dc\uc5d0 \uc57c\ub9c8\uac00\ud0c0 \uc2e0\uce78\uc13c \uc4f0\ubc14\uc0ac\uc640 \uc911\ub828 \uc6b4\ud589 \uc2dc E2\uacc4\ub85c \ubcc0\uacbd\ub418\uc5c8\ub2e4. \uc2dc\uac04\ud45c \uac1c\uc815 \uc774\uc804\uc5d0 \ub3c4\ud638\ucfe0 \uc2e0\uce78\uc13c \uc6b4\ud589 \uc885\ub8cc \uae30\ub150\uc73c\ub85c \uace0\ub9c8\uc6cc\uc694! Max \uc57c\ub9c8\ube44\ucf54\ud638(\uc77c\ubcf8\uc5b4: \u3042\u308a\u304c\u3068\u3046!Max\u3084\u307e\u3073\u3053\u53f7)\ub85c 8\ub7c9 \ud3b8\uc131\uc73c\ub85c \uc6b4\ud589\ud588\ub2e4. 2014\ub144\uc5d0 \ub0b4\uc678\uc7a5\uc744 \ub9ac\ub274\uc5bc\ud55c \ucc28\ub7c9\uc774 \ub4f1\uc7a5\ud588\ub2e4. \ub3c4\uc7a5\uc740 E1\uacc4, E2\uacc4\uc640 \uac19\uc774 \ud770\uc0c9\uacfc \ud30c\ub780\uc0c9\uc758 \ub450 \uc0c9\uc744 \uc911\uc2ec\uc73c\ub85c \uacbd\uacc4\uc120\uc5d0 \ubd84\ud64d\uc0c9\uc758 \ub760\uac00 \uc788\ub294 \ud615\ud0dc\ub85c \ubcc0\uacbd\ub418\uc5c8\ub2e4. \uc544\uc6b8\ub7ec \ub85c\uace0\uc5d0\ub3c4 \ub530\uc624\uae30\uc758 \uc77c\ub7ec\uc2a4\ud2b8\uac00 \ucd94\uac00\ub418\uc5c8\ub2e4. 2016\ub144 \ud558\ubc18\uae30\uc5d0 \uc804 \ud3b8\uc131\uc774 \ub0b4\uc678\uc7a5\uc744 \ub9ac\ub274\uc5bc \uc791\uc5c5\uc774 \uc644\ub8cc\ub418\uc5c8\ub2e4.", "answer": "2014\ub144", "sentence": "2014\ub144 \uc5d0 \ub0b4\uc678\uc7a5\uc744 \ub9ac\ub274\uc5bc\ud55c \ucc28\ub7c9\uc774 \ub4f1\uc7a5\ud588\ub2e4.", "paragraph_sentence": "2001\ub144\ubd80\ud130 \uc870\uc5d0\uc4f0 \uc2e0\uce78\uc13c\uacfc \ub098\uac00\ub178 \uc2e0\uce78\uc13c \ub178\uc120\uc744 \uc6b4\ud589\ud558\uae30 \uc2dc\uc791\ud588\ub2e4. \ub2f9\uc2dc Max \uc544\uc0ac\ub9c8\ub85c \ub3c4\ucfc4 ~ \uce74\ub8e8\uc774\uc790\uc640 \uad6c\uac04\uc744 \uc6b4\ud589 \ud588\uc73c\ub098 2003\ub144 9\uc6d4\uc5d0 \uc6b4\ud589\uc744 \uc885\ub8cc\ud588\ub2e4. 2005\ub144 12\uc6d4 10\uc77c\uc5d0 \uc13c\ub2e4\uc774 \uc774\ubd81 \uad6c\uac04\uc5d0\uc11c \uc815\uae30 \uc6b4\uc6a9\uc744 \uc885\ub8cc\ud588\ub2e4. 2011\ub144\ubd80\ud130 E5\uacc4\uc758 \ub3c4\uc785\uc73c\ub85c \uc6b4\ud589\uc774 \uc904\uc5b4\ub4e4\uc5c8\ub2e4. 2012\ub144 3\uc6d4\uc5d0 \uc870\uc5d0\uc4f0 \uc2e0\uce78\uc13c \uc5d0\uce58\uace0\uc720\uc790\uc640 \uc774\ubd81 \uad6c\uac04\uc5d0 16\ub7c9 \ud3b8\uc131\uc744 \uc601\uc5c5 \uc6b4\uc804\uc744 \uc2dc\uc791\ud558\uba74\uc11c \uc624\ubbf8\uc57c \uc774\ubd81 \uad6c\uac04 16\ub7c9 \ud3b8\uc131\uc774 \uc6b4\ud589\uc774 \uc885\ub8cc\ub418\uc5c8\ub2e4. \uac19\uc740 \ud574 9\uc6d4 28\uc77c\uc5d0 \uc624\ubbf8\uc57c \uc774\ubd81 \uad6c\uac04 \uc6b4\ud589\uc774 \uc885\ub8cc\ub418\uba74\uc11c \uc57c\ub9c8\ube44\ucf54, \ub098\uc2a4\ub178\uc758 \uacbd\uc6b0 E2\uacc4\ub85c \ubcc0\uacbd\uacfc \ub3d9\uc2dc\uc5d0 \uc57c\ub9c8\uac00\ud0c0 \uc2e0\uce78\uc13c \uc4f0\ubc14\uc0ac\uc640 \uc911\ub828 \uc6b4\ud589 \uc2dc E2\uacc4\ub85c \ubcc0\uacbd\ub418\uc5c8\ub2e4. \uc2dc\uac04\ud45c \uac1c\uc815 \uc774\uc804\uc5d0 \ub3c4\ud638\ucfe0 \uc2e0\uce78\uc13c \uc6b4\ud589 \uc885\ub8cc \uae30\ub150\uc73c\ub85c \uace0\ub9c8\uc6cc\uc694! Max \uc57c\ub9c8\ube44\ucf54\ud638(\uc77c\ubcf8\uc5b4: \u3042\u308a\u304c\u3068\u3046!Max\u3084\u307e\u3073\u3053\u53f7)\ub85c 8\ub7c9 \ud3b8\uc131\uc73c\ub85c \uc6b4\ud589\ud588\ub2e4. 2014\ub144 \uc5d0 \ub0b4\uc678\uc7a5\uc744 \ub9ac\ub274\uc5bc\ud55c \ucc28\ub7c9\uc774 \ub4f1\uc7a5\ud588\ub2e4. \ub3c4\uc7a5\uc740 E1\uacc4, E2\uacc4\uc640 \uac19\uc774 \ud770\uc0c9\uacfc \ud30c\ub780\uc0c9\uc758 \ub450 \uc0c9\uc744 \uc911\uc2ec\uc73c\ub85c \uacbd\uacc4\uc120\uc5d0 \ubd84\ud64d\uc0c9\uc758 \ub760\uac00 \uc788\ub294 \ud615\ud0dc\ub85c \ubcc0\uacbd\ub418\uc5c8\ub2e4. \uc544\uc6b8\ub7ec \ub85c\uace0\uc5d0\ub3c4 \ub530\uc624\uae30\uc758 \uc77c\ub7ec\uc2a4\ud2b8\uac00 \ucd94\uac00\ub418\uc5c8\ub2e4. 2016\ub144 \ud558\ubc18\uae30\uc5d0 \uc804 \ud3b8\uc131\uc774 \ub0b4\uc678\uc7a5\uc744 \ub9ac\ub274\uc5bc \uc791\uc5c5\uc774 \uc644\ub8cc\ub418\uc5c8\ub2e4.", "paragraph_answer": "2001\ub144\ubd80\ud130 \uc870\uc5d0\uc4f0 \uc2e0\uce78\uc13c\uacfc \ub098\uac00\ub178 \uc2e0\uce78\uc13c \ub178\uc120\uc744 \uc6b4\ud589\ud558\uae30 \uc2dc\uc791\ud588\ub2e4. \ub2f9\uc2dc Max \uc544\uc0ac\ub9c8\ub85c \ub3c4\ucfc4 ~ \uce74\ub8e8\uc774\uc790\uc640 \uad6c\uac04\uc744 \uc6b4\ud589 \ud588\uc73c\ub098 2003\ub144 9\uc6d4\uc5d0 \uc6b4\ud589\uc744 \uc885\ub8cc\ud588\ub2e4. 2005\ub144 12\uc6d4 10\uc77c\uc5d0 \uc13c\ub2e4\uc774 \uc774\ubd81 \uad6c\uac04\uc5d0\uc11c \uc815\uae30 \uc6b4\uc6a9\uc744 \uc885\ub8cc\ud588\ub2e4. 2011\ub144\ubd80\ud130 E5\uacc4\uc758 \ub3c4\uc785\uc73c\ub85c \uc6b4\ud589\uc774 \uc904\uc5b4\ub4e4\uc5c8\ub2e4. 2012\ub144 3\uc6d4\uc5d0 \uc870\uc5d0\uc4f0 \uc2e0\uce78\uc13c \uc5d0\uce58\uace0\uc720\uc790\uc640 \uc774\ubd81 \uad6c\uac04\uc5d0 16\ub7c9 \ud3b8\uc131\uc744 \uc601\uc5c5 \uc6b4\uc804\uc744 \uc2dc\uc791\ud558\uba74\uc11c \uc624\ubbf8\uc57c \uc774\ubd81 \uad6c\uac04 16\ub7c9 \ud3b8\uc131\uc774 \uc6b4\ud589\uc774 \uc885\ub8cc\ub418\uc5c8\ub2e4. \uac19\uc740 \ud574 9\uc6d4 28\uc77c\uc5d0 \uc624\ubbf8\uc57c \uc774\ubd81 \uad6c\uac04 \uc6b4\ud589\uc774 \uc885\ub8cc\ub418\uba74\uc11c \uc57c\ub9c8\ube44\ucf54, \ub098\uc2a4\ub178\uc758 \uacbd\uc6b0 E2\uacc4\ub85c \ubcc0\uacbd\uacfc \ub3d9\uc2dc\uc5d0 \uc57c\ub9c8\uac00\ud0c0 \uc2e0\uce78\uc13c \uc4f0\ubc14\uc0ac\uc640 \uc911\ub828 \uc6b4\ud589 \uc2dc E2\uacc4\ub85c \ubcc0\uacbd\ub418\uc5c8\ub2e4. \uc2dc\uac04\ud45c \uac1c\uc815 \uc774\uc804\uc5d0 \ub3c4\ud638\ucfe0 \uc2e0\uce78\uc13c \uc6b4\ud589 \uc885\ub8cc \uae30\ub150\uc73c\ub85c \uace0\ub9c8\uc6cc\uc694! Max \uc57c\ub9c8\ube44\ucf54\ud638(\uc77c\ubcf8\uc5b4: \u3042\u308a\u304c\u3068\u3046!Max\u3084\u307e\u3073\u3053\u53f7)\ub85c 8\ub7c9 \ud3b8\uc131\uc73c\ub85c \uc6b4\ud589\ud588\ub2e4. 2014\ub144 \uc5d0 \ub0b4\uc678\uc7a5\uc744 \ub9ac\ub274\uc5bc\ud55c \ucc28\ub7c9\uc774 \ub4f1\uc7a5\ud588\ub2e4. \ub3c4\uc7a5\uc740 E1\uacc4, E2\uacc4\uc640 \uac19\uc774 \ud770\uc0c9\uacfc \ud30c\ub780\uc0c9\uc758 \ub450 \uc0c9\uc744 \uc911\uc2ec\uc73c\ub85c \uacbd\uacc4\uc120\uc5d0 \ubd84\ud64d\uc0c9\uc758 \ub760\uac00 \uc788\ub294 \ud615\ud0dc\ub85c \ubcc0\uacbd\ub418\uc5c8\ub2e4. \uc544\uc6b8\ub7ec \ub85c\uace0\uc5d0\ub3c4 \ub530\uc624\uae30\uc758 \uc77c\ub7ec\uc2a4\ud2b8\uac00 \ucd94\uac00\ub418\uc5c8\ub2e4. 2016\ub144 \ud558\ubc18\uae30\uc5d0 \uc804 \ud3b8\uc131\uc774 \ub0b4\uc678\uc7a5\uc744 \ub9ac\ub274\uc5bc \uc791\uc5c5\uc774 \uc644\ub8cc\ub418\uc5c8\ub2e4.", "sentence_answer": " 2014\ub144 \uc5d0 \ub0b4\uc678\uc7a5\uc744 \ub9ac\ub274\uc5bc\ud55c \ucc28\ub7c9\uc774 \ub4f1\uc7a5\ud588\ub2e4.", "paragraph_id": "6547255-2-1", "paragraph_question": "question: \ub0b4\uc678\uc7a5\uc744 \ub9ac\ub274\uc5bc\ud55c \ucc28\ub7c9\uc774 \ub4f1\uc7a5\ud55c \ud574\ub294?, context: 2001\ub144\ubd80\ud130 \uc870\uc5d0\uc4f0 \uc2e0\uce78\uc13c\uacfc \ub098\uac00\ub178 \uc2e0\uce78\uc13c \ub178\uc120\uc744 \uc6b4\ud589\ud558\uae30 \uc2dc\uc791\ud588\ub2e4. \ub2f9\uc2dc Max \uc544\uc0ac\ub9c8\ub85c \ub3c4\ucfc4 ~ \uce74\ub8e8\uc774\uc790\uc640 \uad6c\uac04\uc744 \uc6b4\ud589 \ud588\uc73c\ub098 2003\ub144 9\uc6d4\uc5d0 \uc6b4\ud589\uc744 \uc885\ub8cc\ud588\ub2e4. 2005\ub144 12\uc6d4 10\uc77c\uc5d0 \uc13c\ub2e4\uc774 \uc774\ubd81 \uad6c\uac04\uc5d0\uc11c \uc815\uae30 \uc6b4\uc6a9\uc744 \uc885\ub8cc\ud588\ub2e4. 2011\ub144\ubd80\ud130 E5\uacc4\uc758 \ub3c4\uc785\uc73c\ub85c \uc6b4\ud589\uc774 \uc904\uc5b4\ub4e4\uc5c8\ub2e4. 2012\ub144 3\uc6d4\uc5d0 \uc870\uc5d0\uc4f0 \uc2e0\uce78\uc13c \uc5d0\uce58\uace0\uc720\uc790\uc640 \uc774\ubd81 \uad6c\uac04\uc5d0 16\ub7c9 \ud3b8\uc131\uc744 \uc601\uc5c5 \uc6b4\uc804\uc744 \uc2dc\uc791\ud558\uba74\uc11c \uc624\ubbf8\uc57c \uc774\ubd81 \uad6c\uac04 16\ub7c9 \ud3b8\uc131\uc774 \uc6b4\ud589\uc774 \uc885\ub8cc\ub418\uc5c8\ub2e4. \uac19\uc740 \ud574 9\uc6d4 28\uc77c\uc5d0 \uc624\ubbf8\uc57c \uc774\ubd81 \uad6c\uac04 \uc6b4\ud589\uc774 \uc885\ub8cc\ub418\uba74\uc11c \uc57c\ub9c8\ube44\ucf54, \ub098\uc2a4\ub178\uc758 \uacbd\uc6b0 E2\uacc4\ub85c \ubcc0\uacbd\uacfc \ub3d9\uc2dc\uc5d0 \uc57c\ub9c8\uac00\ud0c0 \uc2e0\uce78\uc13c \uc4f0\ubc14\uc0ac\uc640 \uc911\ub828 \uc6b4\ud589 \uc2dc E2\uacc4\ub85c \ubcc0\uacbd\ub418\uc5c8\ub2e4. \uc2dc\uac04\ud45c \uac1c\uc815 \uc774\uc804\uc5d0 \ub3c4\ud638\ucfe0 \uc2e0\uce78\uc13c \uc6b4\ud589 \uc885\ub8cc \uae30\ub150\uc73c\ub85c \uace0\ub9c8\uc6cc\uc694! Max \uc57c\ub9c8\ube44\ucf54\ud638(\uc77c\ubcf8\uc5b4: \u3042\u308a\u304c\u3068\u3046!Max\u3084\u307e\u3073\u3053\u53f7)\ub85c 8\ub7c9 \ud3b8\uc131\uc73c\ub85c \uc6b4\ud589\ud588\ub2e4. 2014\ub144\uc5d0 \ub0b4\uc678\uc7a5\uc744 \ub9ac\ub274\uc5bc\ud55c \ucc28\ub7c9\uc774 \ub4f1\uc7a5\ud588\ub2e4. \ub3c4\uc7a5\uc740 E1\uacc4, E2\uacc4\uc640 \uac19\uc774 \ud770\uc0c9\uacfc \ud30c\ub780\uc0c9\uc758 \ub450 \uc0c9\uc744 \uc911\uc2ec\uc73c\ub85c \uacbd\uacc4\uc120\uc5d0 \ubd84\ud64d\uc0c9\uc758 \ub760\uac00 \uc788\ub294 \ud615\ud0dc\ub85c \ubcc0\uacbd\ub418\uc5c8\ub2e4. \uc544\uc6b8\ub7ec \ub85c\uace0\uc5d0\ub3c4 \ub530\uc624\uae30\uc758 \uc77c\ub7ec\uc2a4\ud2b8\uac00 \ucd94\uac00\ub418\uc5c8\ub2e4. 2016\ub144 \ud558\ubc18\uae30\uc5d0 \uc804 \ud3b8\uc131\uc774 \ub0b4\uc678\uc7a5\uc744 \ub9ac\ub274\uc5bc \uc791\uc5c5\uc774 \uc644\ub8cc\ub418\uc5c8\ub2e4."} {"question": "\uc219\uc885\uc758 \uc5b4\uba38\ub2c8\ub294 \ub204\uad6c\uc778\uac00\uc694?", "paragraph": "\uc219\uc885(\u8085\u5b97, 1661\ub144 10\uc6d4 7\uc77c(\uc74c\ub825 8\uc6d4 15\uc77c) ~ 1720\ub144 7\uc6d4 12\uc77c(\uc74c\ub825 6\uc6d4 8\uc77c))\uc740 \uc870\uc120\uc758 \uc81c19\ub300 \uc655\uc774\ub2e4. \uc131\uc740 \uc774(\u674e), \ud718\ub294 \ub3c8(\u711e), \ubcf8\uad00\uc740 \uc804\uc8fc(\u5168\u5dde)., \ucd08\uba85\uc740 \uc6a9\uc0c1(\u9f8d\u7965), \uc790\ub294 \uba85\ubcf4(\u660e\u8b5c), \uc0ac\ud6c4 \uc2dc\ud638\ub294 \uc219\uc885\ud604\uc758\uad11\ub95c\uc608\uc131\uc601\ub82c\uc7a5\ubb38\ud5cc\ubb34\uacbd\uba85\uc6d0\ud6a8\ub300\uc655(\u8085\u5b97\u986f\u7fa9\u5149\u502b\u777f\u8056\u82f1\u70c8\u7ae0\u6587\u61b2\u6b66\u656c\u660e\u5143\u5b5d\u5927\u738b)\uc774\uba70 \uc774\ud6c4 \uc874\ud638\uac00 \ub354\ud574\uc838 \uc815\uc2dd \uc2dc\ud638\ub294 \uc219\uc885\ud604\uc758\uad11\ub95c\uc608\uc131\uc601\ub82c\uc720\ubaa8\uc601\uc6b4\ud64d\uc778\uc900\ub355\ubc30\ucc9c\ud569\ub3c4\uacc4\ud734\ub3c5\uacbd\uc815\uc911\ud611\uadf9\uc2e0\uc758\ub300\ud6c8\uc7a5\ubb38\ud5cc\ubb34\uacbd\uba85\uc6d0\ud6a8\ub300\uc655(\u8085\u5b97\u986f\u7fa9\u5149\u502b\u777f\u8056\u82f1\u70c8\u88d5\u8b28\u6c38\u904b\u6d2a\u4ec1\u5cfb\u5fb7\u914d\u5929\u5408\u9053\u5553\u4f11\u7be4\u6176\u6b63\u4e2d\u604a\u6975\u795e\u6bc5\u5927\u52f3\u7ae0\u6587\u61b2\u6b66\u656c\u660e\u5143\u5b5d\u5927\u738b)\uc774\ub2e4. \ud604\uc885\uacfc \uba85\uc131\uc655\ud6c4\uc758 \uc678\uc544\ub4e4\ub85c \ube44\ub294 \uae40\ub9cc\uae30\uc758 \ub538 \uc778\uacbd\uc655\ud6c4, \uacc4\ube44\ub294 \ubbfc\uc720\uc911\uc758 \ub538 \uc778\ud604\uc655\ud6c4, \uc81c2\uacc4\ube44\ub294 \uae40\uc8fc\uc2e0\uc758 \ub538 \uc778\uc6d0\uc655\ud6c4\uc774\ub2e4.", "answer": "\uba85\uc131\uc655\ud6c4", "sentence": "\ud604\uc885\uacfc \uba85\uc131\uc655\ud6c4 \uc758 \uc678\uc544\ub4e4\ub85c \ube44\ub294 \uae40\ub9cc\uae30\uc758 \ub538 \uc778\uacbd\uc655\ud6c4, \uacc4\ube44\ub294 \ubbfc\uc720\uc911\uc758 \ub538 \uc778\ud604\uc655\ud6c4, \uc81c2\uacc4\ube44\ub294 \uae40\uc8fc\uc2e0\uc758 \ub538 \uc778\uc6d0\uc655\ud6c4\uc774\ub2e4.", "paragraph_sentence": "\uc219\uc885(\u8085\u5b97, 1661\ub144 10\uc6d4 7\uc77c(\uc74c\ub825 8\uc6d4 15\uc77c) ~ 1720\ub144 7\uc6d4 12\uc77c(\uc74c\ub825 6\uc6d4 8\uc77c))\uc740 \uc870\uc120\uc758 \uc81c19\ub300 \uc655\uc774\ub2e4. \uc131\uc740 \uc774(\u674e), \ud718\ub294 \ub3c8(\u711e), \ubcf8\uad00\uc740 \uc804\uc8fc(\u5168\u5dde)., \ucd08\uba85\uc740 \uc6a9\uc0c1(\u9f8d\u7965), \uc790\ub294 \uba85\ubcf4(\u660e\u8b5c), \uc0ac\ud6c4 \uc2dc\ud638\ub294 \uc219\uc885\ud604\uc758\uad11\ub95c\uc608\uc131\uc601\ub82c\uc7a5\ubb38\ud5cc\ubb34\uacbd\uba85\uc6d0\ud6a8\ub300\uc655(\u8085\u5b97\u986f\u7fa9\u5149\u502b\u777f\u8056\u82f1\u70c8\u7ae0\u6587\u61b2\u6b66\u656c\u660e\u5143\u5b5d\u5927\u738b)\uc774\uba70 \uc774\ud6c4 \uc874\ud638\uac00 \ub354\ud574\uc838 \uc815\uc2dd \uc2dc\ud638\ub294 \uc219\uc885\ud604\uc758\uad11\ub95c\uc608\uc131\uc601\ub82c\uc720\ubaa8\uc601\uc6b4\ud64d\uc778\uc900\ub355\ubc30\ucc9c\ud569\ub3c4\uacc4\ud734\ub3c5\uacbd\uc815\uc911\ud611\uadf9\uc2e0\uc758\ub300\ud6c8\uc7a5\ubb38\ud5cc\ubb34\uacbd\uba85\uc6d0\ud6a8\ub300\uc655(\u8085\u5b97\u986f\u7fa9\u5149\u502b\u777f\u8056\u82f1\u70c8\u88d5\u8b28\u6c38\u904b\u6d2a\u4ec1\u5cfb\u5fb7\u914d\u5929\u5408\u9053\u5553\u4f11\u7be4\u6176\u6b63\u4e2d\u604a\u6975\u795e\u6bc5\u5927\u52f3\u7ae0\u6587\u61b2\u6b66\u656c\u660e\u5143\u5b5d\u5927\u738b)\uc774\ub2e4. \ud604\uc885\uacfc \uba85\uc131\uc655\ud6c4 \uc758 \uc678\uc544\ub4e4\ub85c \ube44\ub294 \uae40\ub9cc\uae30\uc758 \ub538 \uc778\uacbd\uc655\ud6c4, \uacc4\ube44\ub294 \ubbfc\uc720\uc911\uc758 \ub538 \uc778\ud604\uc655\ud6c4, \uc81c2\uacc4\ube44\ub294 \uae40\uc8fc\uc2e0\uc758 \ub538 \uc778\uc6d0\uc655\ud6c4\uc774\ub2e4. ", "paragraph_answer": "\uc219\uc885(\u8085\u5b97, 1661\ub144 10\uc6d4 7\uc77c(\uc74c\ub825 8\uc6d4 15\uc77c) ~ 1720\ub144 7\uc6d4 12\uc77c(\uc74c\ub825 6\uc6d4 8\uc77c))\uc740 \uc870\uc120\uc758 \uc81c19\ub300 \uc655\uc774\ub2e4. \uc131\uc740 \uc774(\u674e), \ud718\ub294 \ub3c8(\u711e), \ubcf8\uad00\uc740 \uc804\uc8fc(\u5168\u5dde)., \ucd08\uba85\uc740 \uc6a9\uc0c1(\u9f8d\u7965), \uc790\ub294 \uba85\ubcf4(\u660e\u8b5c), \uc0ac\ud6c4 \uc2dc\ud638\ub294 \uc219\uc885\ud604\uc758\uad11\ub95c\uc608\uc131\uc601\ub82c\uc7a5\ubb38\ud5cc\ubb34\uacbd\uba85\uc6d0\ud6a8\ub300\uc655(\u8085\u5b97\u986f\u7fa9\u5149\u502b\u777f\u8056\u82f1\u70c8\u7ae0\u6587\u61b2\u6b66\u656c\u660e\u5143\u5b5d\u5927\u738b)\uc774\uba70 \uc774\ud6c4 \uc874\ud638\uac00 \ub354\ud574\uc838 \uc815\uc2dd \uc2dc\ud638\ub294 \uc219\uc885\ud604\uc758\uad11\ub95c\uc608\uc131\uc601\ub82c\uc720\ubaa8\uc601\uc6b4\ud64d\uc778\uc900\ub355\ubc30\ucc9c\ud569\ub3c4\uacc4\ud734\ub3c5\uacbd\uc815\uc911\ud611\uadf9\uc2e0\uc758\ub300\ud6c8\uc7a5\ubb38\ud5cc\ubb34\uacbd\uba85\uc6d0\ud6a8\ub300\uc655(\u8085\u5b97\u986f\u7fa9\u5149\u502b\u777f\u8056\u82f1\u70c8\u88d5\u8b28\u6c38\u904b\u6d2a\u4ec1\u5cfb\u5fb7\u914d\u5929\u5408\u9053\u5553\u4f11\u7be4\u6176\u6b63\u4e2d\u604a\u6975\u795e\u6bc5\u5927\u52f3\u7ae0\u6587\u61b2\u6b66\u656c\u660e\u5143\u5b5d\u5927\u738b)\uc774\ub2e4. \ud604\uc885\uacfc \uba85\uc131\uc655\ud6c4 \uc758 \uc678\uc544\ub4e4\ub85c \ube44\ub294 \uae40\ub9cc\uae30\uc758 \ub538 \uc778\uacbd\uc655\ud6c4, \uacc4\ube44\ub294 \ubbfc\uc720\uc911\uc758 \ub538 \uc778\ud604\uc655\ud6c4, \uc81c2\uacc4\ube44\ub294 \uae40\uc8fc\uc2e0\uc758 \ub538 \uc778\uc6d0\uc655\ud6c4\uc774\ub2e4.", "sentence_answer": "\ud604\uc885\uacfc \uba85\uc131\uc655\ud6c4 \uc758 \uc678\uc544\ub4e4\ub85c \ube44\ub294 \uae40\ub9cc\uae30\uc758 \ub538 \uc778\uacbd\uc655\ud6c4, \uacc4\ube44\ub294 \ubbfc\uc720\uc911\uc758 \ub538 \uc778\ud604\uc655\ud6c4, \uc81c2\uacc4\ube44\ub294 \uae40\uc8fc\uc2e0\uc758 \ub538 \uc778\uc6d0\uc655\ud6c4\uc774\ub2e4.", "paragraph_id": "5876576-0-2", "paragraph_question": "question: \uc219\uc885\uc758 \uc5b4\uba38\ub2c8\ub294 \ub204\uad6c\uc778\uac00\uc694?, context: \uc219\uc885(\u8085\u5b97, 1661\ub144 10\uc6d4 7\uc77c(\uc74c\ub825 8\uc6d4 15\uc77c) ~ 1720\ub144 7\uc6d4 12\uc77c(\uc74c\ub825 6\uc6d4 8\uc77c))\uc740 \uc870\uc120\uc758 \uc81c19\ub300 \uc655\uc774\ub2e4. \uc131\uc740 \uc774(\u674e), \ud718\ub294 \ub3c8(\u711e), \ubcf8\uad00\uc740 \uc804\uc8fc(\u5168\u5dde)., \ucd08\uba85\uc740 \uc6a9\uc0c1(\u9f8d\u7965), \uc790\ub294 \uba85\ubcf4(\u660e\u8b5c), \uc0ac\ud6c4 \uc2dc\ud638\ub294 \uc219\uc885\ud604\uc758\uad11\ub95c\uc608\uc131\uc601\ub82c\uc7a5\ubb38\ud5cc\ubb34\uacbd\uba85\uc6d0\ud6a8\ub300\uc655(\u8085\u5b97\u986f\u7fa9\u5149\u502b\u777f\u8056\u82f1\u70c8\u7ae0\u6587\u61b2\u6b66\u656c\u660e\u5143\u5b5d\u5927\u738b)\uc774\uba70 \uc774\ud6c4 \uc874\ud638\uac00 \ub354\ud574\uc838 \uc815\uc2dd \uc2dc\ud638\ub294 \uc219\uc885\ud604\uc758\uad11\ub95c\uc608\uc131\uc601\ub82c\uc720\ubaa8\uc601\uc6b4\ud64d\uc778\uc900\ub355\ubc30\ucc9c\ud569\ub3c4\uacc4\ud734\ub3c5\uacbd\uc815\uc911\ud611\uadf9\uc2e0\uc758\ub300\ud6c8\uc7a5\ubb38\ud5cc\ubb34\uacbd\uba85\uc6d0\ud6a8\ub300\uc655(\u8085\u5b97\u986f\u7fa9\u5149\u502b\u777f\u8056\u82f1\u70c8\u88d5\u8b28\u6c38\u904b\u6d2a\u4ec1\u5cfb\u5fb7\u914d\u5929\u5408\u9053\u5553\u4f11\u7be4\u6176\u6b63\u4e2d\u604a\u6975\u795e\u6bc5\u5927\u52f3\u7ae0\u6587\u61b2\u6b66\u656c\u660e\u5143\u5b5d\u5927\u738b)\uc774\ub2e4. \ud604\uc885\uacfc \uba85\uc131\uc655\ud6c4\uc758 \uc678\uc544\ub4e4\ub85c \ube44\ub294 \uae40\ub9cc\uae30\uc758 \ub538 \uc778\uacbd\uc655\ud6c4, \uacc4\ube44\ub294 \ubbfc\uc720\uc911\uc758 \ub538 \uc778\ud604\uc655\ud6c4, \uc81c2\uacc4\ube44\ub294 \uae40\uc8fc\uc2e0\uc758 \ub538 \uc778\uc6d0\uc655\ud6c4\uc774\ub2e4."} {"question": "\uc0bc\ub3c4\uc218\uad70\ud1b5\uc81c\uc0ac\uc5d0 \uc624\ub978 \uc6d0\uade0\uc774 \uc77c\ubcf8\uad70\uc5d0\uac8c \ud06c\uac8c \ud328\ud55c \uc804\ud22c\ub294?", "paragraph": "1597\ub144 8\uc6d4 28\uc77c(\uc74c\ub825 7\uc6d4 16\uc77c)\uc5d0 \uc0bc\ub3c4\uc218\uad70\ud1b5\uc81c\uc0ac\uc5d0 \uc624\ub978 \uc6d0\uade0\uc774 \uc774\ub044\ub294 \uc870\uc120 \ud568\ub300\uac00 \uce60\ucc9c\ub7c9 \ud574\uc804\uc5d0\uc11c \uc77c\ubcf8\uad70\uc758 \uae30\uc2b5\uc744 \ubc1b\uc544 \ucd98\uc6d0\ud3ec\ub85c \ud6c4\ud1f4, \uc218\uad70\ub4e4\uc740 \uc0c1\ub959\ud558\uc5ec \ub3c4\uc8fc\ud558\uace0 \ud310\uc625\uc120 \ub300\ubd80\ubd84\uc774 \ubd88\ud0c0\uac70\ub098 \uc65c\uad70\uc5d0\uac8c \ub178\ud68d\ub2f9\ud574 \uc624\uc0ac\uce74\ub85c \ub04c\ub824\uac04\ub2e4. \uc774\ub97c \uc218\uc2b5\ud558\uae30 \uc704\ud558\uc5ec \uc870\uc120 \uc870\uc815\uc5d0\uc11c\ub294 \uacbd\ub9bc\uad70(\u6176\u6797\u541b) \uae40\uba85\uc6d0(\u91d1\u547d\u5143), \ubcd1\uc870 \ud310\uc11c \uc774\ud56d\ubcf5(\u674e\u6052\u798f)\uc758 \uac74\uc758 \ub85c \uc774\uc21c\uc2e0\uc744 \ub2e4\uc2dc \uc0bc\ub3c4\uc218\uad70\ud1b5\uc81c\uc0ac\ub85c \uc784\uba85\ud558\uc600\ub2e4. \ud558\uc9c0\ub9cc \uc774\uc21c\uc2e0\uc774 \ub2e4\uc2dc \uc870\uc120 \uc218\uad70\uc744 \ubaa8\uc544 \uc815\ube44\ud588\uc744 \ub54c \ud568\uc120\uc740 12\ucc99\ubc16\uc5d0 \ub0a8\uc544 \uc788\uc9c0 \uc54a\uc558\ub2e4. \uc870\uc120 \uc815\ubd80\uc5d0\uc11c\ub294 \uc774 \ubcd1\ub825\uc73c\ub85c \uc801\uc744 \ub300\ud56d\ud0a4 \uc5b4\ub835\ub2e4 \ud558\uc5ec \uc218\uad70\uc744 \ud3d0\uc9c0\ud558\ub77c\ub294 \uba85\ub839\uc744 \ub0b4\ub838\uc73c\ub098, \uc774\uc21c\uc2e0\uc740 \uc544\uc9c1\ub3c4 12\ucc99\uc758 \ubc30\uac00 \ub0a8\uc544 \uc788\uc73c\uba70 \ub0b4\uac00 \uc8fd\uc9c0 \uc54a\ub294 \ud55c \uc801\uc774 \uac10\ud788 \uc6b0\ub9ac\uc758 \uc218\uad70\uc744 \uc5c5\uc2e0\uc5ec\uae30\uc9c0 \ubabb\ud560 \uac83\uc774\ub77c\ub294 \ube44\uc7a5\ud55c \uacb0\uc758\ub97c \ud45c\ud558\uc600\ub2e4\uace0 \ud55c\ub2e4. \uadf8 \ub4a4 \uc804\uc5f4\uc744 \uc7ac\uc815\ube44\ud558\uae30 \uc704\ud574 10\uc6d4 9\uc77c(\uc74c\ub825 8\uc6d4 29\uc77c)\uc5d0 \uc9c4\ub3c4 \ubcbd\ud30c\uc9c4\uc73c\ub85c \uc9c4\uc744 \uc62e\uacbc\ub2e4.", "answer": "\uce60\ucc9c\ub7c9 \ud574\uc804", "sentence": "1597\ub144 8\uc6d4 28\uc77c(\uc74c\ub825 7\uc6d4 16\uc77c)\uc5d0 \uc0bc\ub3c4\uc218\uad70\ud1b5\uc81c\uc0ac\uc5d0 \uc624\ub978 \uc6d0\uade0\uc774 \uc774\ub044\ub294 \uc870\uc120 \ud568\ub300\uac00 \uce60\ucc9c\ub7c9 \ud574\uc804 \uc5d0\uc11c \uc77c\ubcf8\uad70\uc758 \uae30\uc2b5\uc744 \ubc1b\uc544 \ucd98\uc6d0\ud3ec\ub85c \ud6c4\ud1f4, \uc218\uad70\ub4e4\uc740 \uc0c1\ub959\ud558\uc5ec \ub3c4\uc8fc\ud558\uace0 \ud310\uc625\uc120 \ub300\ubd80\ubd84\uc774 \ubd88\ud0c0\uac70\ub098 \uc65c\uad70\uc5d0\uac8c \ub178\ud68d\ub2f9\ud574 \uc624\uc0ac\uce74\ub85c \ub04c\ub824\uac04\ub2e4.", "paragraph_sentence": " 1597\ub144 8\uc6d4 28\uc77c(\uc74c\ub825 7\uc6d4 16\uc77c)\uc5d0 \uc0bc\ub3c4\uc218\uad70\ud1b5\uc81c\uc0ac\uc5d0 \uc624\ub978 \uc6d0\uade0\uc774 \uc774\ub044\ub294 \uc870\uc120 \ud568\ub300\uac00 \uce60\ucc9c\ub7c9 \ud574\uc804 \uc5d0\uc11c \uc77c\ubcf8\uad70\uc758 \uae30\uc2b5\uc744 \ubc1b\uc544 \ucd98\uc6d0\ud3ec\ub85c \ud6c4\ud1f4, \uc218\uad70\ub4e4\uc740 \uc0c1\ub959\ud558\uc5ec \ub3c4\uc8fc\ud558\uace0 \ud310\uc625\uc120 \ub300\ubd80\ubd84\uc774 \ubd88\ud0c0\uac70\ub098 \uc65c\uad70\uc5d0\uac8c \ub178\ud68d\ub2f9\ud574 \uc624\uc0ac\uce74\ub85c \ub04c\ub824\uac04\ub2e4. \uc774\ub97c \uc218\uc2b5\ud558\uae30 \uc704\ud558\uc5ec \uc870\uc120 \uc870\uc815\uc5d0\uc11c\ub294 \uacbd\ub9bc\uad70(\u6176\u6797\u541b) \uae40\uba85\uc6d0(\u91d1\u547d\u5143), \ubcd1\uc870 \ud310\uc11c \uc774\ud56d\ubcf5(\u674e\u6052\u798f)\uc758 \uac74\uc758 \ub85c \uc774\uc21c\uc2e0\uc744 \ub2e4\uc2dc \uc0bc\ub3c4\uc218\uad70\ud1b5\uc81c\uc0ac\ub85c \uc784\uba85\ud558\uc600\ub2e4. \ud558\uc9c0\ub9cc \uc774\uc21c\uc2e0\uc774 \ub2e4\uc2dc \uc870\uc120 \uc218\uad70\uc744 \ubaa8\uc544 \uc815\ube44\ud588\uc744 \ub54c \ud568\uc120\uc740 12\ucc99\ubc16\uc5d0 \ub0a8\uc544 \uc788\uc9c0 \uc54a\uc558\ub2e4. \uc870\uc120 \uc815\ubd80\uc5d0\uc11c\ub294 \uc774 \ubcd1\ub825\uc73c\ub85c \uc801\uc744 \ub300\ud56d\ud0a4 \uc5b4\ub835\ub2e4 \ud558\uc5ec \uc218\uad70\uc744 \ud3d0\uc9c0\ud558\ub77c\ub294 \uba85\ub839\uc744 \ub0b4\ub838\uc73c\ub098, \uc774\uc21c\uc2e0\uc740 \uc544\uc9c1\ub3c4 12\ucc99\uc758 \ubc30\uac00 \ub0a8\uc544 \uc788\uc73c\uba70 \ub0b4\uac00 \uc8fd\uc9c0 \uc54a\ub294 \ud55c \uc801\uc774 \uac10\ud788 \uc6b0\ub9ac\uc758 \uc218\uad70\uc744 \uc5c5\uc2e0\uc5ec\uae30\uc9c0 \ubabb\ud560 \uac83\uc774\ub77c\ub294 \ube44\uc7a5\ud55c \uacb0\uc758\ub97c \ud45c\ud558\uc600\ub2e4\uace0 \ud55c\ub2e4. \uadf8 \ub4a4 \uc804\uc5f4\uc744 \uc7ac\uc815\ube44\ud558\uae30 \uc704\ud574 10\uc6d4 9\uc77c(\uc74c\ub825 8\uc6d4 29\uc77c)\uc5d0 \uc9c4\ub3c4 \ubcbd\ud30c\uc9c4\uc73c\ub85c \uc9c4\uc744 \uc62e\uacbc\ub2e4.", "paragraph_answer": "1597\ub144 8\uc6d4 28\uc77c(\uc74c\ub825 7\uc6d4 16\uc77c)\uc5d0 \uc0bc\ub3c4\uc218\uad70\ud1b5\uc81c\uc0ac\uc5d0 \uc624\ub978 \uc6d0\uade0\uc774 \uc774\ub044\ub294 \uc870\uc120 \ud568\ub300\uac00 \uce60\ucc9c\ub7c9 \ud574\uc804 \uc5d0\uc11c \uc77c\ubcf8\uad70\uc758 \uae30\uc2b5\uc744 \ubc1b\uc544 \ucd98\uc6d0\ud3ec\ub85c \ud6c4\ud1f4, \uc218\uad70\ub4e4\uc740 \uc0c1\ub959\ud558\uc5ec \ub3c4\uc8fc\ud558\uace0 \ud310\uc625\uc120 \ub300\ubd80\ubd84\uc774 \ubd88\ud0c0\uac70\ub098 \uc65c\uad70\uc5d0\uac8c \ub178\ud68d\ub2f9\ud574 \uc624\uc0ac\uce74\ub85c \ub04c\ub824\uac04\ub2e4. \uc774\ub97c \uc218\uc2b5\ud558\uae30 \uc704\ud558\uc5ec \uc870\uc120 \uc870\uc815\uc5d0\uc11c\ub294 \uacbd\ub9bc\uad70(\u6176\u6797\u541b) \uae40\uba85\uc6d0(\u91d1\u547d\u5143), \ubcd1\uc870 \ud310\uc11c \uc774\ud56d\ubcf5(\u674e\u6052\u798f)\uc758 \uac74\uc758 \ub85c \uc774\uc21c\uc2e0\uc744 \ub2e4\uc2dc \uc0bc\ub3c4\uc218\uad70\ud1b5\uc81c\uc0ac\ub85c \uc784\uba85\ud558\uc600\ub2e4. \ud558\uc9c0\ub9cc \uc774\uc21c\uc2e0\uc774 \ub2e4\uc2dc \uc870\uc120 \uc218\uad70\uc744 \ubaa8\uc544 \uc815\ube44\ud588\uc744 \ub54c \ud568\uc120\uc740 12\ucc99\ubc16\uc5d0 \ub0a8\uc544 \uc788\uc9c0 \uc54a\uc558\ub2e4. \uc870\uc120 \uc815\ubd80\uc5d0\uc11c\ub294 \uc774 \ubcd1\ub825\uc73c\ub85c \uc801\uc744 \ub300\ud56d\ud0a4 \uc5b4\ub835\ub2e4 \ud558\uc5ec \uc218\uad70\uc744 \ud3d0\uc9c0\ud558\ub77c\ub294 \uba85\ub839\uc744 \ub0b4\ub838\uc73c\ub098, \uc774\uc21c\uc2e0\uc740 \uc544\uc9c1\ub3c4 12\ucc99\uc758 \ubc30\uac00 \ub0a8\uc544 \uc788\uc73c\uba70 \ub0b4\uac00 \uc8fd\uc9c0 \uc54a\ub294 \ud55c \uc801\uc774 \uac10\ud788 \uc6b0\ub9ac\uc758 \uc218\uad70\uc744 \uc5c5\uc2e0\uc5ec\uae30\uc9c0 \ubabb\ud560 \uac83\uc774\ub77c\ub294 \ube44\uc7a5\ud55c \uacb0\uc758\ub97c \ud45c\ud558\uc600\ub2e4\uace0 \ud55c\ub2e4. \uadf8 \ub4a4 \uc804\uc5f4\uc744 \uc7ac\uc815\ube44\ud558\uae30 \uc704\ud574 10\uc6d4 9\uc77c(\uc74c\ub825 8\uc6d4 29\uc77c)\uc5d0 \uc9c4\ub3c4 \ubcbd\ud30c\uc9c4\uc73c\ub85c \uc9c4\uc744 \uc62e\uacbc\ub2e4.", "sentence_answer": "1597\ub144 8\uc6d4 28\uc77c(\uc74c\ub825 7\uc6d4 16\uc77c)\uc5d0 \uc0bc\ub3c4\uc218\uad70\ud1b5\uc81c\uc0ac\uc5d0 \uc624\ub978 \uc6d0\uade0\uc774 \uc774\ub044\ub294 \uc870\uc120 \ud568\ub300\uac00 \uce60\ucc9c\ub7c9 \ud574\uc804 \uc5d0\uc11c \uc77c\ubcf8\uad70\uc758 \uae30\uc2b5\uc744 \ubc1b\uc544 \ucd98\uc6d0\ud3ec\ub85c \ud6c4\ud1f4, \uc218\uad70\ub4e4\uc740 \uc0c1\ub959\ud558\uc5ec \ub3c4\uc8fc\ud558\uace0 \ud310\uc625\uc120 \ub300\ubd80\ubd84\uc774 \ubd88\ud0c0\uac70\ub098 \uc65c\uad70\uc5d0\uac8c \ub178\ud68d\ub2f9\ud574 \uc624\uc0ac\uce74\ub85c \ub04c\ub824\uac04\ub2e4.", "paragraph_id": "6161989-10-1", "paragraph_question": "question: \uc0bc\ub3c4\uc218\uad70\ud1b5\uc81c\uc0ac\uc5d0 \uc624\ub978 \uc6d0\uade0\uc774 \uc77c\ubcf8\uad70\uc5d0\uac8c \ud06c\uac8c \ud328\ud55c \uc804\ud22c\ub294?, context: 1597\ub144 8\uc6d4 28\uc77c(\uc74c\ub825 7\uc6d4 16\uc77c)\uc5d0 \uc0bc\ub3c4\uc218\uad70\ud1b5\uc81c\uc0ac\uc5d0 \uc624\ub978 \uc6d0\uade0\uc774 \uc774\ub044\ub294 \uc870\uc120 \ud568\ub300\uac00 \uce60\ucc9c\ub7c9 \ud574\uc804\uc5d0\uc11c \uc77c\ubcf8\uad70\uc758 \uae30\uc2b5\uc744 \ubc1b\uc544 \ucd98\uc6d0\ud3ec\ub85c \ud6c4\ud1f4, \uc218\uad70\ub4e4\uc740 \uc0c1\ub959\ud558\uc5ec \ub3c4\uc8fc\ud558\uace0 \ud310\uc625\uc120 \ub300\ubd80\ubd84\uc774 \ubd88\ud0c0\uac70\ub098 \uc65c\uad70\uc5d0\uac8c \ub178\ud68d\ub2f9\ud574 \uc624\uc0ac\uce74\ub85c \ub04c\ub824\uac04\ub2e4. \uc774\ub97c \uc218\uc2b5\ud558\uae30 \uc704\ud558\uc5ec \uc870\uc120 \uc870\uc815\uc5d0\uc11c\ub294 \uacbd\ub9bc\uad70(\u6176\u6797\u541b) \uae40\uba85\uc6d0(\u91d1\u547d\u5143), \ubcd1\uc870 \ud310\uc11c \uc774\ud56d\ubcf5(\u674e\u6052\u798f)\uc758 \uac74\uc758 \ub85c \uc774\uc21c\uc2e0\uc744 \ub2e4\uc2dc \uc0bc\ub3c4\uc218\uad70\ud1b5\uc81c\uc0ac\ub85c \uc784\uba85\ud558\uc600\ub2e4. \ud558\uc9c0\ub9cc \uc774\uc21c\uc2e0\uc774 \ub2e4\uc2dc \uc870\uc120 \uc218\uad70\uc744 \ubaa8\uc544 \uc815\ube44\ud588\uc744 \ub54c \ud568\uc120\uc740 12\ucc99\ubc16\uc5d0 \ub0a8\uc544 \uc788\uc9c0 \uc54a\uc558\ub2e4. \uc870\uc120 \uc815\ubd80\uc5d0\uc11c\ub294 \uc774 \ubcd1\ub825\uc73c\ub85c \uc801\uc744 \ub300\ud56d\ud0a4 \uc5b4\ub835\ub2e4 \ud558\uc5ec \uc218\uad70\uc744 \ud3d0\uc9c0\ud558\ub77c\ub294 \uba85\ub839\uc744 \ub0b4\ub838\uc73c\ub098, \uc774\uc21c\uc2e0\uc740 \uc544\uc9c1\ub3c4 12\ucc99\uc758 \ubc30\uac00 \ub0a8\uc544 \uc788\uc73c\uba70 \ub0b4\uac00 \uc8fd\uc9c0 \uc54a\ub294 \ud55c \uc801\uc774 \uac10\ud788 \uc6b0\ub9ac\uc758 \uc218\uad70\uc744 \uc5c5\uc2e0\uc5ec\uae30\uc9c0 \ubabb\ud560 \uac83\uc774\ub77c\ub294 \ube44\uc7a5\ud55c \uacb0\uc758\ub97c \ud45c\ud558\uc600\ub2e4\uace0 \ud55c\ub2e4. \uadf8 \ub4a4 \uc804\uc5f4\uc744 \uc7ac\uc815\ube44\ud558\uae30 \uc704\ud574 10\uc6d4 9\uc77c(\uc74c\ub825 8\uc6d4 29\uc77c)\uc5d0 \uc9c4\ub3c4 \ubcbd\ud30c\uc9c4\uc73c\ub85c \uc9c4\uc744 \uc62e\uacbc\ub2e4."} {"question": "\uad50\ud669 \ud558\ub4dc\ub9ac\uc544\ub178 2\uc138\ub294 \uc774\ud63c\uc744 \ud5c8\ub77d\ud558\ub294 \ub300\uc2e0 \ub85c\ud0c0\ub974 2\uc138\uc758 \ubb34\uc5c7\ub3c4 \uae08\uc9c0\ud558\uc600\ub294\uac00?", "paragraph": "865\ub144 \ub85c\ud0c0\ub974 2\uc138\uc758 \uc0bc\ucd0c\ub4e4\uc778 \ub300\uba38\ub9ac \uce74\ub97c\uacfc \ub3c5\uc77c\uc778 \ub8e8\ud2b8\ube44\ud788\ub294 \ud30c\ubb38\uc758 \uc704\ud611\uc744 \ud558\uba74\uc11c \ub85c\ud0c0\ub974\uc5d0\uac8c \ud1a0\uc774\ud2b8\ubca0\ub974\uac00\ub97c \ub2e4\uc2dc \ubc1b\uc544\ub4e4\uc774\ub77c\uace0 \uac15\uc694\ud588\ub2e4. \ud544\uc694\ud558\ub2e4\uba74 \uadf8\ub97c \uad8c\uc88c\uc5d0\uc11c \ucad3\uc544\ub0b4\uc11c\ub77c\ub3c4 \uad50\ud669\uc758 \uacb0\uc815\uc744 \uc2e4\ud589\ud558\uac8c \ub9cc\ub4e4\uaca0\ub2e4\uace0 \ud611\ubc15\ud558\uc790 \ub85c\ud0c0\ub974\ub294 \ud1a0\uc774\ud2b8\ubca0\ub974\uac00\ub97c \ub2e4\uc2dc \ubc1b\uc544\ub4e4\uc600\ub2e4. \uadf8\ub7ec\ub098 \ud1a0\uc774\ud2b8\ubca0\ub974\uac00\uac00 \uc5ec\uc804\ud788 \uc544\uc774\ub97c \ub0b3\uc9c0 \ubabb\ud558\uc790 \ub85c\ud0c0\ub974\ub294 1\ub144 \ub4a4 \uadf8\ub140\uc5d0\uac8c \uc73d\ubc15\uc9c8\ub7ec \uc774\ud63c\uc744 \uc2e0\uccad\ud558\uac8c \ub9cc\ub4e4\uc5c8\ub2e4. \uadf8\ud574 6\uc6d4 15\uc77c\uc5d0\ub294 \uad50\ud669\uccad\uc5d0\uc11c \ub85c\ud0c0\ub974 2\uc138\uc5d0\uac8c \uacbd\uace0\ub97c \ud558\uae30 \uc704\ud55c \ud2b9\uc0ac\uac00 \ub85c\ud0c0\ub9c1\uae30\uc544\ub85c \ud30c\uacac\ub418\uc5c8\ub2e4. \uadf8\ub7ec\ub098 \ub85c\ud0c0\ub974 2\uc138\ub294 \uac70\uc808\ud588\uace0 \uac19\uc740 \ud574 \ud30c\ubb38\ub2f9\ud588\ub2e4. 867\ub144 \uad50\ud669 \ub2c8\ucf5c\ub77c\uc624\uc758 \ub4a4\ub97c \uc774\uc5b4 \uc880 \ub354 \uc735\ud1b5\uc131 \uc788\ub294 \uad50\ud669 \ud558\ub4dc\ub9ac\uc544\ub178 2\uc138\uac00 \uc0c8\ub85c\uc6b4 \uad50\ud669\uc774 \ub418\uc790 \ub85c\ud0c0\ub974\ub294 \uc544\ub0b4\uc5d0\uac8c \uc9c1\uc811 \uc0c8 \uad50\ud669\uc5d0\uac8c \uc774\ud63c\uc744 \uc694\uccad\ud558\ub3c4\ub85d \uc555\ub825\uc744 \ub123\uc5c8\uace0, \ud1a0\uc774\ud2b8\ubca0\ub974\uac00\uac00 \uc774\ud63c\uc744 \uc694\uad6c\ud558\uace0 \ub098\uc130\ub2e4. \uad50\ud669 \ud558\ub4dc\ub9ac\uc544\ub178 2\uc138\ub294 \ubc1c\ud2b8\ub77c\ub2e4\uc5d0\uac8c \uac00\ud574\uc9c4 \ud30c\ubb38\uc744 \ud480\uc5b4\uc8fc\uc5c8\ub2e4. \uadf8\ub7ec\ub098 \uad50\ud669 \ud558\ub4dc\ub9ac\uc544\ub178 2\uc138\ub294 \ud1a0\uc774\ud2b8\ubca0\ub974\uac00\uac00 \ub85c\ud0c0\ub974 2\uc138\uc640 \uc774\ud63c\ud558\ub294 \uac83\uc740 \ud5c8\uc6a9\ud558\uc9c0\ub9cc, \uc790\ub140\ub97c \ub0b3\uc9c0 \ubabb\ud558\ub294 \uac83\uc740 \ucda9\ubd84\ud55c \uc774\ud63c \uc0ac\uc720\uac00 \ub420 \uc218 \uc5c6\uc5b4\uc11c \ub85c\ud0c0\ub974 2\uc138\ub3c4 \uc7ac\ud63c\ud560 \uc218 \uc5c6\ub2e4\ub294 \ucd5c\uc885 \ud310\uacb0\uc744 \ub0b4\ub838\ub2e4.", "answer": "\uc7ac\ud63c", "sentence": "\uadf8\ub7ec\ub098 \uad50\ud669 \ud558\ub4dc\ub9ac\uc544\ub178 2\uc138\ub294 \ud1a0\uc774\ud2b8\ubca0\ub974\uac00\uac00 \ub85c\ud0c0\ub974 2\uc138\uc640 \uc774\ud63c\ud558\ub294 \uac83\uc740 \ud5c8\uc6a9\ud558\uc9c0\ub9cc, \uc790\ub140\ub97c \ub0b3\uc9c0 \ubabb\ud558\ub294 \uac83\uc740 \ucda9\ubd84\ud55c \uc774\ud63c \uc0ac\uc720\uac00 \ub420 \uc218 \uc5c6\uc5b4\uc11c \ub85c\ud0c0\ub974 2\uc138\ub3c4 \uc7ac\ud63c \ud560 \uc218 \uc5c6\ub2e4\ub294 \ucd5c\uc885 \ud310\uacb0\uc744 \ub0b4\ub838\ub2e4.", "paragraph_sentence": "865\ub144 \ub85c\ud0c0\ub974 2\uc138\uc758 \uc0bc\ucd0c\ub4e4\uc778 \ub300\uba38\ub9ac \uce74\ub97c\uacfc \ub3c5\uc77c\uc778 \ub8e8\ud2b8\ube44\ud788\ub294 \ud30c\ubb38\uc758 \uc704\ud611\uc744 \ud558\uba74\uc11c \ub85c\ud0c0\ub974\uc5d0\uac8c \ud1a0\uc774\ud2b8\ubca0\ub974\uac00\ub97c \ub2e4\uc2dc \ubc1b\uc544\ub4e4\uc774\ub77c\uace0 \uac15\uc694\ud588\ub2e4. \ud544\uc694\ud558\ub2e4\uba74 \uadf8\ub97c \uad8c\uc88c\uc5d0\uc11c \ucad3\uc544\ub0b4\uc11c\ub77c\ub3c4 \uad50\ud669\uc758 \uacb0\uc815\uc744 \uc2e4\ud589\ud558\uac8c \ub9cc\ub4e4\uaca0\ub2e4\uace0 \ud611\ubc15\ud558\uc790 \ub85c\ud0c0\ub974\ub294 \ud1a0\uc774\ud2b8\ubca0\ub974\uac00\ub97c \ub2e4\uc2dc \ubc1b\uc544\ub4e4\uc600\ub2e4. \uadf8\ub7ec\ub098 \ud1a0\uc774\ud2b8\ubca0\ub974\uac00\uac00 \uc5ec\uc804\ud788 \uc544\uc774\ub97c \ub0b3\uc9c0 \ubabb\ud558\uc790 \ub85c\ud0c0\ub974\ub294 1\ub144 \ub4a4 \uadf8\ub140\uc5d0\uac8c \uc73d\ubc15\uc9c8\ub7ec \uc774\ud63c\uc744 \uc2e0\uccad\ud558\uac8c \ub9cc\ub4e4\uc5c8\ub2e4. \uadf8\ud574 6\uc6d4 15\uc77c\uc5d0\ub294 \uad50\ud669\uccad\uc5d0\uc11c \ub85c\ud0c0\ub974 2\uc138\uc5d0\uac8c \uacbd\uace0\ub97c \ud558\uae30 \uc704\ud55c \ud2b9\uc0ac\uac00 \ub85c\ud0c0\ub9c1\uae30\uc544\ub85c \ud30c\uacac\ub418\uc5c8\ub2e4. \uadf8\ub7ec\ub098 \ub85c\ud0c0\ub974 2\uc138\ub294 \uac70\uc808\ud588\uace0 \uac19\uc740 \ud574 \ud30c\ubb38\ub2f9\ud588\ub2e4. 867\ub144 \uad50\ud669 \ub2c8\ucf5c\ub77c\uc624\uc758 \ub4a4\ub97c \uc774\uc5b4 \uc880 \ub354 \uc735\ud1b5\uc131 \uc788\ub294 \uad50\ud669 \ud558\ub4dc\ub9ac\uc544\ub178 2\uc138\uac00 \uc0c8\ub85c\uc6b4 \uad50\ud669\uc774 \ub418\uc790 \ub85c\ud0c0\ub974\ub294 \uc544\ub0b4\uc5d0\uac8c \uc9c1\uc811 \uc0c8 \uad50\ud669\uc5d0\uac8c \uc774\ud63c\uc744 \uc694\uccad\ud558\ub3c4\ub85d \uc555\ub825\uc744 \ub123\uc5c8\uace0, \ud1a0\uc774\ud2b8\ubca0\ub974\uac00\uac00 \uc774\ud63c\uc744 \uc694\uad6c\ud558\uace0 \ub098\uc130\ub2e4. \uad50\ud669 \ud558\ub4dc\ub9ac\uc544\ub178 2\uc138\ub294 \ubc1c\ud2b8\ub77c\ub2e4\uc5d0\uac8c \uac00\ud574\uc9c4 \ud30c\ubb38\uc744 \ud480\uc5b4\uc8fc\uc5c8\ub2e4. \uadf8\ub7ec\ub098 \uad50\ud669 \ud558\ub4dc\ub9ac\uc544\ub178 2\uc138\ub294 \ud1a0\uc774\ud2b8\ubca0\ub974\uac00\uac00 \ub85c\ud0c0\ub974 2\uc138\uc640 \uc774\ud63c\ud558\ub294 \uac83\uc740 \ud5c8\uc6a9\ud558\uc9c0\ub9cc, \uc790\ub140\ub97c \ub0b3\uc9c0 \ubabb\ud558\ub294 \uac83\uc740 \ucda9\ubd84\ud55c \uc774\ud63c \uc0ac\uc720\uac00 \ub420 \uc218 \uc5c6\uc5b4\uc11c \ub85c\ud0c0\ub974 2\uc138\ub3c4 \uc7ac\ud63c \ud560 \uc218 \uc5c6\ub2e4\ub294 \ucd5c\uc885 \ud310\uacb0\uc744 \ub0b4\ub838\ub2e4. ", "paragraph_answer": "865\ub144 \ub85c\ud0c0\ub974 2\uc138\uc758 \uc0bc\ucd0c\ub4e4\uc778 \ub300\uba38\ub9ac \uce74\ub97c\uacfc \ub3c5\uc77c\uc778 \ub8e8\ud2b8\ube44\ud788\ub294 \ud30c\ubb38\uc758 \uc704\ud611\uc744 \ud558\uba74\uc11c \ub85c\ud0c0\ub974\uc5d0\uac8c \ud1a0\uc774\ud2b8\ubca0\ub974\uac00\ub97c \ub2e4\uc2dc \ubc1b\uc544\ub4e4\uc774\ub77c\uace0 \uac15\uc694\ud588\ub2e4. \ud544\uc694\ud558\ub2e4\uba74 \uadf8\ub97c \uad8c\uc88c\uc5d0\uc11c \ucad3\uc544\ub0b4\uc11c\ub77c\ub3c4 \uad50\ud669\uc758 \uacb0\uc815\uc744 \uc2e4\ud589\ud558\uac8c \ub9cc\ub4e4\uaca0\ub2e4\uace0 \ud611\ubc15\ud558\uc790 \ub85c\ud0c0\ub974\ub294 \ud1a0\uc774\ud2b8\ubca0\ub974\uac00\ub97c \ub2e4\uc2dc \ubc1b\uc544\ub4e4\uc600\ub2e4. \uadf8\ub7ec\ub098 \ud1a0\uc774\ud2b8\ubca0\ub974\uac00\uac00 \uc5ec\uc804\ud788 \uc544\uc774\ub97c \ub0b3\uc9c0 \ubabb\ud558\uc790 \ub85c\ud0c0\ub974\ub294 1\ub144 \ub4a4 \uadf8\ub140\uc5d0\uac8c \uc73d\ubc15\uc9c8\ub7ec \uc774\ud63c\uc744 \uc2e0\uccad\ud558\uac8c \ub9cc\ub4e4\uc5c8\ub2e4. \uadf8\ud574 6\uc6d4 15\uc77c\uc5d0\ub294 \uad50\ud669\uccad\uc5d0\uc11c \ub85c\ud0c0\ub974 2\uc138\uc5d0\uac8c \uacbd\uace0\ub97c \ud558\uae30 \uc704\ud55c \ud2b9\uc0ac\uac00 \ub85c\ud0c0\ub9c1\uae30\uc544\ub85c \ud30c\uacac\ub418\uc5c8\ub2e4. \uadf8\ub7ec\ub098 \ub85c\ud0c0\ub974 2\uc138\ub294 \uac70\uc808\ud588\uace0 \uac19\uc740 \ud574 \ud30c\ubb38\ub2f9\ud588\ub2e4. 867\ub144 \uad50\ud669 \ub2c8\ucf5c\ub77c\uc624\uc758 \ub4a4\ub97c \uc774\uc5b4 \uc880 \ub354 \uc735\ud1b5\uc131 \uc788\ub294 \uad50\ud669 \ud558\ub4dc\ub9ac\uc544\ub178 2\uc138\uac00 \uc0c8\ub85c\uc6b4 \uad50\ud669\uc774 \ub418\uc790 \ub85c\ud0c0\ub974\ub294 \uc544\ub0b4\uc5d0\uac8c \uc9c1\uc811 \uc0c8 \uad50\ud669\uc5d0\uac8c \uc774\ud63c\uc744 \uc694\uccad\ud558\ub3c4\ub85d \uc555\ub825\uc744 \ub123\uc5c8\uace0, \ud1a0\uc774\ud2b8\ubca0\ub974\uac00\uac00 \uc774\ud63c\uc744 \uc694\uad6c\ud558\uace0 \ub098\uc130\ub2e4. \uad50\ud669 \ud558\ub4dc\ub9ac\uc544\ub178 2\uc138\ub294 \ubc1c\ud2b8\ub77c\ub2e4\uc5d0\uac8c \uac00\ud574\uc9c4 \ud30c\ubb38\uc744 \ud480\uc5b4\uc8fc\uc5c8\ub2e4. \uadf8\ub7ec\ub098 \uad50\ud669 \ud558\ub4dc\ub9ac\uc544\ub178 2\uc138\ub294 \ud1a0\uc774\ud2b8\ubca0\ub974\uac00\uac00 \ub85c\ud0c0\ub974 2\uc138\uc640 \uc774\ud63c\ud558\ub294 \uac83\uc740 \ud5c8\uc6a9\ud558\uc9c0\ub9cc, \uc790\ub140\ub97c \ub0b3\uc9c0 \ubabb\ud558\ub294 \uac83\uc740 \ucda9\ubd84\ud55c \uc774\ud63c \uc0ac\uc720\uac00 \ub420 \uc218 \uc5c6\uc5b4\uc11c \ub85c\ud0c0\ub974 2\uc138\ub3c4 \uc7ac\ud63c \ud560 \uc218 \uc5c6\ub2e4\ub294 \ucd5c\uc885 \ud310\uacb0\uc744 \ub0b4\ub838\ub2e4.", "sentence_answer": "\uadf8\ub7ec\ub098 \uad50\ud669 \ud558\ub4dc\ub9ac\uc544\ub178 2\uc138\ub294 \ud1a0\uc774\ud2b8\ubca0\ub974\uac00\uac00 \ub85c\ud0c0\ub974 2\uc138\uc640 \uc774\ud63c\ud558\ub294 \uac83\uc740 \ud5c8\uc6a9\ud558\uc9c0\ub9cc, \uc790\ub140\ub97c \ub0b3\uc9c0 \ubabb\ud558\ub294 \uac83\uc740 \ucda9\ubd84\ud55c \uc774\ud63c \uc0ac\uc720\uac00 \ub420 \uc218 \uc5c6\uc5b4\uc11c \ub85c\ud0c0\ub974 2\uc138\ub3c4 \uc7ac\ud63c \ud560 \uc218 \uc5c6\ub2e4\ub294 \ucd5c\uc885 \ud310\uacb0\uc744 \ub0b4\ub838\ub2e4.", "paragraph_id": "6497078-6-2", "paragraph_question": "question: \uad50\ud669 \ud558\ub4dc\ub9ac\uc544\ub178 2\uc138\ub294 \uc774\ud63c\uc744 \ud5c8\ub77d\ud558\ub294 \ub300\uc2e0 \ub85c\ud0c0\ub974 2\uc138\uc758 \ubb34\uc5c7\ub3c4 \uae08\uc9c0\ud558\uc600\ub294\uac00?, context: 865\ub144 \ub85c\ud0c0\ub974 2\uc138\uc758 \uc0bc\ucd0c\ub4e4\uc778 \ub300\uba38\ub9ac \uce74\ub97c\uacfc \ub3c5\uc77c\uc778 \ub8e8\ud2b8\ube44\ud788\ub294 \ud30c\ubb38\uc758 \uc704\ud611\uc744 \ud558\uba74\uc11c \ub85c\ud0c0\ub974\uc5d0\uac8c \ud1a0\uc774\ud2b8\ubca0\ub974\uac00\ub97c \ub2e4\uc2dc \ubc1b\uc544\ub4e4\uc774\ub77c\uace0 \uac15\uc694\ud588\ub2e4. \ud544\uc694\ud558\ub2e4\uba74 \uadf8\ub97c \uad8c\uc88c\uc5d0\uc11c \ucad3\uc544\ub0b4\uc11c\ub77c\ub3c4 \uad50\ud669\uc758 \uacb0\uc815\uc744 \uc2e4\ud589\ud558\uac8c \ub9cc\ub4e4\uaca0\ub2e4\uace0 \ud611\ubc15\ud558\uc790 \ub85c\ud0c0\ub974\ub294 \ud1a0\uc774\ud2b8\ubca0\ub974\uac00\ub97c \ub2e4\uc2dc \ubc1b\uc544\ub4e4\uc600\ub2e4. \uadf8\ub7ec\ub098 \ud1a0\uc774\ud2b8\ubca0\ub974\uac00\uac00 \uc5ec\uc804\ud788 \uc544\uc774\ub97c \ub0b3\uc9c0 \ubabb\ud558\uc790 \ub85c\ud0c0\ub974\ub294 1\ub144 \ub4a4 \uadf8\ub140\uc5d0\uac8c \uc73d\ubc15\uc9c8\ub7ec \uc774\ud63c\uc744 \uc2e0\uccad\ud558\uac8c \ub9cc\ub4e4\uc5c8\ub2e4. \uadf8\ud574 6\uc6d4 15\uc77c\uc5d0\ub294 \uad50\ud669\uccad\uc5d0\uc11c \ub85c\ud0c0\ub974 2\uc138\uc5d0\uac8c \uacbd\uace0\ub97c \ud558\uae30 \uc704\ud55c \ud2b9\uc0ac\uac00 \ub85c\ud0c0\ub9c1\uae30\uc544\ub85c \ud30c\uacac\ub418\uc5c8\ub2e4. \uadf8\ub7ec\ub098 \ub85c\ud0c0\ub974 2\uc138\ub294 \uac70\uc808\ud588\uace0 \uac19\uc740 \ud574 \ud30c\ubb38\ub2f9\ud588\ub2e4. 867\ub144 \uad50\ud669 \ub2c8\ucf5c\ub77c\uc624\uc758 \ub4a4\ub97c \uc774\uc5b4 \uc880 \ub354 \uc735\ud1b5\uc131 \uc788\ub294 \uad50\ud669 \ud558\ub4dc\ub9ac\uc544\ub178 2\uc138\uac00 \uc0c8\ub85c\uc6b4 \uad50\ud669\uc774 \ub418\uc790 \ub85c\ud0c0\ub974\ub294 \uc544\ub0b4\uc5d0\uac8c \uc9c1\uc811 \uc0c8 \uad50\ud669\uc5d0\uac8c \uc774\ud63c\uc744 \uc694\uccad\ud558\ub3c4\ub85d \uc555\ub825\uc744 \ub123\uc5c8\uace0, \ud1a0\uc774\ud2b8\ubca0\ub974\uac00\uac00 \uc774\ud63c\uc744 \uc694\uad6c\ud558\uace0 \ub098\uc130\ub2e4. \uad50\ud669 \ud558\ub4dc\ub9ac\uc544\ub178 2\uc138\ub294 \ubc1c\ud2b8\ub77c\ub2e4\uc5d0\uac8c \uac00\ud574\uc9c4 \ud30c\ubb38\uc744 \ud480\uc5b4\uc8fc\uc5c8\ub2e4. \uadf8\ub7ec\ub098 \uad50\ud669 \ud558\ub4dc\ub9ac\uc544\ub178 2\uc138\ub294 \ud1a0\uc774\ud2b8\ubca0\ub974\uac00\uac00 \ub85c\ud0c0\ub974 2\uc138\uc640 \uc774\ud63c\ud558\ub294 \uac83\uc740 \ud5c8\uc6a9\ud558\uc9c0\ub9cc, \uc790\ub140\ub97c \ub0b3\uc9c0 \ubabb\ud558\ub294 \uac83\uc740 \ucda9\ubd84\ud55c \uc774\ud63c \uc0ac\uc720\uac00 \ub420 \uc218 \uc5c6\uc5b4\uc11c \ub85c\ud0c0\ub974 2\uc138\ub3c4 \uc7ac\ud63c\ud560 \uc218 \uc5c6\ub2e4\ub294 \ucd5c\uc885 \ud310\uacb0\uc744 \ub0b4\ub838\ub2e4."} {"question": "<\uc774\ud074\ub9bd\uc2a4> \uc601\ud654\uc758 \uac1c\ubd09\uccab\uc8fc \uc218\uc785\uc740?", "paragraph": "2007\ub144, \uc601\ud654 \uc81c\uc791\uc790\ub4e4\uc740 \u300a\ud2b8\uc640\uc77c\ub77c\uc787 \uc601\ud654 \uc2dc\ub9ac\uc988\u300b\uc758 \uccab \ubc88\uc9f8 \uc791\ud488\uc778 \u300a\ud2b8\uc640\uc77c\ub77c\uc787\u300b\uc5d0\uc11c \ubca8\ub77c \uc2a4\uc644\uc758 \uc6d0\uc8fc\ubbfc \uce5c\uad6c \uc81c\uc774\ucf65 \ube14\ub799\uc744 \ub9e1\uc744 \ubc30\uc6b0\ub97c \ucc3e\uace0 \uc788\uc5c8\ub2e4. 2008\ub144 1\uc6d4, \ud3ec\ud2c0\ub79c\ub4dc\uc5d0\uc11c \uc624\ub514\uc158\uc774 \uc5f4\ub838\uace0, \ub85c\ud2b8\ub108\ub294 \u300a\ud2b8\uc640\uc77c\ub77c\uc787\u300b\uc5d0 \ub300\ud574 \uc804\ud600 \ubab0\ub790\uc9c0\ub9cc \uae30\ud68d\uc0ac\uc5d0 \uc758\ud574 \uc624\ub514\uc158\uc744 \ubcf4\uac8c \ub418\uc5c8\ub2e4. \uc624\ub514\uc158\uc5d0\uc11c \uc774\ubbf8 \ubca8\ub77c \uc5ed\uc73c\ub85c \uce90\uc2a4\ud305\ub41c \ud06c\ub9ac\uc2a4\ud2f4 \uc2a4\ud29c\uc5b4\ud2b8\uc640 \ub300\uc0ac\ub97c \uc77d\uc5c8\uace0 \uadf8\ub4e4\uc740 \u300a\ub274\ubb38\u300b, \u300a\uc774\ud074\ub9bd\uc2a4\u300b\uc758 \uc7a5\uba74\ub4e4\uc744 \uc5f0\uae30\ud558\uc600\ub2e4. \uacb0\uacfc\ub294 \uc81c\uc774\ucf65 \ube14\ub799 \uc5ed\uc73c\ub85c \uce90\uc2a4\ud305\ub418\uc5c8\uace0, \uc601\ud654\ub294 2008\ub144\uc5d0 \uac1c\ubd09\ud558\uc600\ub2e4. \uc601\ud654\ub294 \uc131\uacf5\uc801\uc774\uc600\uace0, \uccab \uc8fc\uc5d0 6\ucc9c 9\ubc31\ub9cc\ub2ec\ub7ec\ub97c \ubc8c\uc5c8\uc73c\uba70, \uc804\uc138\uacc4\ub85c\ub294 3\uc5b5 9\ucc9c 2\ubc31\ub9cc\ub2ec\ub7ec\uc758 \uc218\uc785\uc744 \ub0c8\ub2e4. \ub85c\ud2bc \ud1a0\ub9c8\ud1a0\uc5d0\uc11c\ub294 10\uc810 \uc911 5.5\uc810\uc744 \ud68d\ub4dd\ud558\uc600\uace0, \uba54\ud0c0\ud06c\ub9ac\ud2f1\uc5d0\uc11c\ub3c4 \ud3c9\uade0\uc801\uc778 \ud3c9\uc744 \ubc1b\uc558\ub2e4. 2009\ub144 MTV \ubb34\ube44 \uc5b4\uc6cc\ub4dc\uc5d0\uc11c \ub85c\ud2b8\ub108\ub294 \u300a\ud2b8\uc640\uc77c\ub77c\uc787\u300b\uc5d0 \uac19\uc774 \ucd9c\uc5f0\ud55c \ub85c\ubc84\ud2b8 \ud328\ud2f4\uc2a8\uacfc \ub0a8\uc6b0\uc2e0\uc778\uc0c1\uc5d0 \ud6c4\ubcf4\ub85c \uc62c\ub790\uc73c\ub098, \ud328\ud2f4\uc2a8\uc774 \uc218\uc0c1\ud558\uc600\ub2e4.", "answer": "6\ucc9c 9\ubc31\ub9cc\ub2ec\ub7ec", "sentence": "\uc601\ud654\ub294 \uc131\uacf5\uc801\uc774\uc600\uace0, \uccab \uc8fc\uc5d0 6\ucc9c 9\ubc31\ub9cc\ub2ec\ub7ec \ub97c \ubc8c\uc5c8\uc73c\uba70, \uc804\uc138\uacc4\ub85c\ub294 3\uc5b5 9\ucc9c 2\ubc31\ub9cc\ub2ec\ub7ec\uc758 \uc218\uc785\uc744 \ub0c8\ub2e4.", "paragraph_sentence": "2007\ub144, \uc601\ud654 \uc81c\uc791\uc790\ub4e4\uc740 \u300a\ud2b8\uc640\uc77c\ub77c\uc787 \uc601\ud654 \uc2dc\ub9ac\uc988\u300b\uc758 \uccab \ubc88\uc9f8 \uc791\ud488\uc778 \u300a\ud2b8\uc640\uc77c\ub77c\uc787\u300b\uc5d0\uc11c \ubca8\ub77c \uc2a4\uc644\uc758 \uc6d0\uc8fc\ubbfc \uce5c\uad6c \uc81c\uc774\ucf65 \ube14\ub799\uc744 \ub9e1\uc744 \ubc30\uc6b0\ub97c \ucc3e\uace0 \uc788\uc5c8\ub2e4. 2008\ub144 1\uc6d4, \ud3ec\ud2c0\ub79c\ub4dc\uc5d0\uc11c \uc624\ub514\uc158\uc774 \uc5f4\ub838\uace0, \ub85c\ud2b8\ub108\ub294 \u300a\ud2b8\uc640\uc77c\ub77c\uc787\u300b\uc5d0 \ub300\ud574 \uc804\ud600 \ubab0\ub790\uc9c0\ub9cc \uae30\ud68d\uc0ac\uc5d0 \uc758\ud574 \uc624\ub514\uc158\uc744 \ubcf4\uac8c \ub418\uc5c8\ub2e4. \uc624\ub514\uc158\uc5d0\uc11c \uc774\ubbf8 \ubca8\ub77c \uc5ed\uc73c\ub85c \uce90\uc2a4\ud305\ub41c \ud06c\ub9ac\uc2a4\ud2f4 \uc2a4\ud29c\uc5b4\ud2b8\uc640 \ub300\uc0ac\ub97c \uc77d\uc5c8\uace0 \uadf8\ub4e4\uc740 \u300a\ub274\ubb38\u300b, \u300a\uc774\ud074\ub9bd\uc2a4\u300b\uc758 \uc7a5\uba74\ub4e4\uc744 \uc5f0\uae30\ud558\uc600\ub2e4. \uacb0\uacfc\ub294 \uc81c\uc774\ucf65 \ube14\ub799 \uc5ed\uc73c\ub85c \uce90\uc2a4\ud305\ub418\uc5c8\uace0, \uc601\ud654\ub294 2008\ub144\uc5d0 \uac1c\ubd09\ud558\uc600\ub2e4. \uc601\ud654\ub294 \uc131\uacf5\uc801\uc774\uc600\uace0, \uccab \uc8fc\uc5d0 6\ucc9c 9\ubc31\ub9cc\ub2ec\ub7ec \ub97c \ubc8c\uc5c8\uc73c\uba70, \uc804\uc138\uacc4\ub85c\ub294 3\uc5b5 9\ucc9c 2\ubc31\ub9cc\ub2ec\ub7ec\uc758 \uc218\uc785\uc744 \ub0c8\ub2e4. \ub85c\ud2bc \ud1a0\ub9c8\ud1a0\uc5d0\uc11c\ub294 10\uc810 \uc911 5.5\uc810\uc744 \ud68d\ub4dd\ud558\uc600\uace0, \uba54\ud0c0\ud06c\ub9ac\ud2f1\uc5d0\uc11c\ub3c4 \ud3c9\uade0\uc801\uc778 \ud3c9\uc744 \ubc1b\uc558\ub2e4. 2009\ub144 MTV \ubb34\ube44 \uc5b4\uc6cc\ub4dc\uc5d0\uc11c \ub85c\ud2b8\ub108\ub294 \u300a\ud2b8\uc640\uc77c\ub77c\uc787\u300b\uc5d0 \uac19\uc774 \ucd9c\uc5f0\ud55c \ub85c\ubc84\ud2b8 \ud328\ud2f4\uc2a8\uacfc \ub0a8\uc6b0\uc2e0\uc778\uc0c1\uc5d0 \ud6c4\ubcf4\ub85c \uc62c\ub790\uc73c\ub098, \ud328\ud2f4\uc2a8\uc774 \uc218\uc0c1\ud558\uc600\ub2e4.", "paragraph_answer": "2007\ub144, \uc601\ud654 \uc81c\uc791\uc790\ub4e4\uc740 \u300a\ud2b8\uc640\uc77c\ub77c\uc787 \uc601\ud654 \uc2dc\ub9ac\uc988\u300b\uc758 \uccab \ubc88\uc9f8 \uc791\ud488\uc778 \u300a\ud2b8\uc640\uc77c\ub77c\uc787\u300b\uc5d0\uc11c \ubca8\ub77c \uc2a4\uc644\uc758 \uc6d0\uc8fc\ubbfc \uce5c\uad6c \uc81c\uc774\ucf65 \ube14\ub799\uc744 \ub9e1\uc744 \ubc30\uc6b0\ub97c \ucc3e\uace0 \uc788\uc5c8\ub2e4. 2008\ub144 1\uc6d4, \ud3ec\ud2c0\ub79c\ub4dc\uc5d0\uc11c \uc624\ub514\uc158\uc774 \uc5f4\ub838\uace0, \ub85c\ud2b8\ub108\ub294 \u300a\ud2b8\uc640\uc77c\ub77c\uc787\u300b\uc5d0 \ub300\ud574 \uc804\ud600 \ubab0\ub790\uc9c0\ub9cc \uae30\ud68d\uc0ac\uc5d0 \uc758\ud574 \uc624\ub514\uc158\uc744 \ubcf4\uac8c \ub418\uc5c8\ub2e4. \uc624\ub514\uc158\uc5d0\uc11c \uc774\ubbf8 \ubca8\ub77c \uc5ed\uc73c\ub85c \uce90\uc2a4\ud305\ub41c \ud06c\ub9ac\uc2a4\ud2f4 \uc2a4\ud29c\uc5b4\ud2b8\uc640 \ub300\uc0ac\ub97c \uc77d\uc5c8\uace0 \uadf8\ub4e4\uc740 \u300a\ub274\ubb38\u300b, \u300a\uc774\ud074\ub9bd\uc2a4\u300b\uc758 \uc7a5\uba74\ub4e4\uc744 \uc5f0\uae30\ud558\uc600\ub2e4. \uacb0\uacfc\ub294 \uc81c\uc774\ucf65 \ube14\ub799 \uc5ed\uc73c\ub85c \uce90\uc2a4\ud305\ub418\uc5c8\uace0, \uc601\ud654\ub294 2008\ub144\uc5d0 \uac1c\ubd09\ud558\uc600\ub2e4. \uc601\ud654\ub294 \uc131\uacf5\uc801\uc774\uc600\uace0, \uccab \uc8fc\uc5d0 6\ucc9c 9\ubc31\ub9cc\ub2ec\ub7ec \ub97c \ubc8c\uc5c8\uc73c\uba70, \uc804\uc138\uacc4\ub85c\ub294 3\uc5b5 9\ucc9c 2\ubc31\ub9cc\ub2ec\ub7ec\uc758 \uc218\uc785\uc744 \ub0c8\ub2e4. \ub85c\ud2bc \ud1a0\ub9c8\ud1a0\uc5d0\uc11c\ub294 10\uc810 \uc911 5.5\uc810\uc744 \ud68d\ub4dd\ud558\uc600\uace0, \uba54\ud0c0\ud06c\ub9ac\ud2f1\uc5d0\uc11c\ub3c4 \ud3c9\uade0\uc801\uc778 \ud3c9\uc744 \ubc1b\uc558\ub2e4. 2009\ub144 MTV \ubb34\ube44 \uc5b4\uc6cc\ub4dc\uc5d0\uc11c \ub85c\ud2b8\ub108\ub294 \u300a\ud2b8\uc640\uc77c\ub77c\uc787\u300b\uc5d0 \uac19\uc774 \ucd9c\uc5f0\ud55c \ub85c\ubc84\ud2b8 \ud328\ud2f4\uc2a8\uacfc \ub0a8\uc6b0\uc2e0\uc778\uc0c1\uc5d0 \ud6c4\ubcf4\ub85c \uc62c\ub790\uc73c\ub098, \ud328\ud2f4\uc2a8\uc774 \uc218\uc0c1\ud558\uc600\ub2e4.", "sentence_answer": "\uc601\ud654\ub294 \uc131\uacf5\uc801\uc774\uc600\uace0, \uccab \uc8fc\uc5d0 6\ucc9c 9\ubc31\ub9cc\ub2ec\ub7ec \ub97c \ubc8c\uc5c8\uc73c\uba70, \uc804\uc138\uacc4\ub85c\ub294 3\uc5b5 9\ucc9c 2\ubc31\ub9cc\ub2ec\ub7ec\uc758 \uc218\uc785\uc744 \ub0c8\ub2e4.", "paragraph_id": "5912999-1-1", "paragraph_question": "question: <\uc774\ud074\ub9bd\uc2a4> \uc601\ud654\uc758 \uac1c\ubd09\uccab\uc8fc \uc218\uc785\uc740?, context: 2007\ub144, \uc601\ud654 \uc81c\uc791\uc790\ub4e4\uc740 \u300a\ud2b8\uc640\uc77c\ub77c\uc787 \uc601\ud654 \uc2dc\ub9ac\uc988\u300b\uc758 \uccab \ubc88\uc9f8 \uc791\ud488\uc778 \u300a\ud2b8\uc640\uc77c\ub77c\uc787\u300b\uc5d0\uc11c \ubca8\ub77c \uc2a4\uc644\uc758 \uc6d0\uc8fc\ubbfc \uce5c\uad6c \uc81c\uc774\ucf65 \ube14\ub799\uc744 \ub9e1\uc744 \ubc30\uc6b0\ub97c \ucc3e\uace0 \uc788\uc5c8\ub2e4. 2008\ub144 1\uc6d4, \ud3ec\ud2c0\ub79c\ub4dc\uc5d0\uc11c \uc624\ub514\uc158\uc774 \uc5f4\ub838\uace0, \ub85c\ud2b8\ub108\ub294 \u300a\ud2b8\uc640\uc77c\ub77c\uc787\u300b\uc5d0 \ub300\ud574 \uc804\ud600 \ubab0\ub790\uc9c0\ub9cc \uae30\ud68d\uc0ac\uc5d0 \uc758\ud574 \uc624\ub514\uc158\uc744 \ubcf4\uac8c \ub418\uc5c8\ub2e4. \uc624\ub514\uc158\uc5d0\uc11c \uc774\ubbf8 \ubca8\ub77c \uc5ed\uc73c\ub85c \uce90\uc2a4\ud305\ub41c \ud06c\ub9ac\uc2a4\ud2f4 \uc2a4\ud29c\uc5b4\ud2b8\uc640 \ub300\uc0ac\ub97c \uc77d\uc5c8\uace0 \uadf8\ub4e4\uc740 \u300a\ub274\ubb38\u300b, \u300a\uc774\ud074\ub9bd\uc2a4\u300b\uc758 \uc7a5\uba74\ub4e4\uc744 \uc5f0\uae30\ud558\uc600\ub2e4. \uacb0\uacfc\ub294 \uc81c\uc774\ucf65 \ube14\ub799 \uc5ed\uc73c\ub85c \uce90\uc2a4\ud305\ub418\uc5c8\uace0, \uc601\ud654\ub294 2008\ub144\uc5d0 \uac1c\ubd09\ud558\uc600\ub2e4. \uc601\ud654\ub294 \uc131\uacf5\uc801\uc774\uc600\uace0, \uccab \uc8fc\uc5d0 6\ucc9c 9\ubc31\ub9cc\ub2ec\ub7ec\ub97c \ubc8c\uc5c8\uc73c\uba70, \uc804\uc138\uacc4\ub85c\ub294 3\uc5b5 9\ucc9c 2\ubc31\ub9cc\ub2ec\ub7ec\uc758 \uc218\uc785\uc744 \ub0c8\ub2e4. \ub85c\ud2bc \ud1a0\ub9c8\ud1a0\uc5d0\uc11c\ub294 10\uc810 \uc911 5.5\uc810\uc744 \ud68d\ub4dd\ud558\uc600\uace0, \uba54\ud0c0\ud06c\ub9ac\ud2f1\uc5d0\uc11c\ub3c4 \ud3c9\uade0\uc801\uc778 \ud3c9\uc744 \ubc1b\uc558\ub2e4. 2009\ub144 MTV \ubb34\ube44 \uc5b4\uc6cc\ub4dc\uc5d0\uc11c \ub85c\ud2b8\ub108\ub294 \u300a\ud2b8\uc640\uc77c\ub77c\uc787\u300b\uc5d0 \uac19\uc774 \ucd9c\uc5f0\ud55c \ub85c\ubc84\ud2b8 \ud328\ud2f4\uc2a8\uacfc \ub0a8\uc6b0\uc2e0\uc778\uc0c1\uc5d0 \ud6c4\ubcf4\ub85c \uc62c\ub790\uc73c\ub098, \ud328\ud2f4\uc2a8\uc774 \uc218\uc0c1\ud558\uc600\ub2e4."} {"question": "\ucc9c\uc6b0\ud76c\uc758 \uc544\ubc84\uc9c0\ub294 \ubb34\uc5c7\uc744 \ud558\ub294 \uc0ac\ub78c\uc774\uc5c8\ub098?", "paragraph": "\ucc9c\uc6b0\ud76c\ub294 1987\ub144 4\uc6d4 20\uc77c \uacbd\uae30\ub3c4 \uc774\ucc9c\uc2dc\uc5d0\uc11c 1\ub0a8 1\ub140 \uc911 \ub458\uc9f8\ub85c \ud0dc\uc5b4\ub0ac\ub2e4. \uc624\ube60\uc640\ub294 \uc138\uc0b4 \ud130\uc6b8\uc774\uc5c8\ub2e4. \uc544\ubc84\uc9c0\ub294 \uc774\ucc9c\uc5d0\uc11c \ub3c4\uc608\ub97c \ud558\ub294 \uc0ac\ub78c\uc774\uc5c8\ub2e4. \ucc9c\uc6b0\ud76c\ub294 \ud559\ucc3d \uc2dc\uc808 \uce5c\uad6c\ub4e4\uacfc \ub450\ub8e8 \uc798 \uc5b4\uc6b8\ub9ac\ub294 \ucf8c\ud65c\ud55c \uc131\uaca9\uc73c\ub85c \ucd08\ub4f1\ud559\uad50 \uc2dc\uc808 \ubc18\uc7a5\uc744 \ub3c4\ub9e1\uc544 \ud558\uc600\uace0 \uc804\uad50\ud68c\uc7a5\ub3c4 \ud558\uc600\uc73c\uba70 \uc911\ud559\uad50 \ub54c\ub294 \uc804\uad50\ubd80\ud68c\uc7a5\uc744 \ud558\uc600\ub2e4. \uc774\ucc9c\uc591\uc815\uc5ec\uc790\uace0\ub4f1\ud559\uad50\uc5d0 \uc9c4\ud559\ud574\uc11c\ub294 \uc5f0\uadf9\ubc18 \ud65c\ub3d9\uc744 \ud558\uc600\ub294\ub370, \uadf8\ub140\uac00 \ucc98\uc74c\uc73c\ub85c \ubb34\ub300\uc5d0 \uc624\ub978 \uc791\ud488\uc740 \uc704\uc548\ubd80 \ud560\uba38\ub2c8\uc758 \uc774\uc57c\uae30\ub97c \ub2e4\ub8ec \u300a\ubc18\ucabd \ub0a0\uac1c\ub85c \ub0a0\uc544\uc628 \uc0c8\u300b\uc600\ub2e4. \ucc9c\uc6b0\ud76c\ub294 \uace0\uad50 \uc2dc\uc808 \uccad\uc18c\ub144 \uc5f0\uae30\ub300\ud68c\uc5d0\uc11c \uc5f0\uae30\uc0c1\uc744 \ubc1b\uae30\ub3c4 \ud558\uc600\ub2e4. \ucc9c\uc6b0\ud76c\ub294 \uc5f0\uadf9\ubc18 \ud65c\ub3d9\uc744 \ud558\uba70 \uc5f0\uae30\uc5d0 \ud765\ubbf8\ub97c \ub290\uaef4 2006\ub144 \uacbd\uae30\ub300\ud559\uad50 \ub2e4\uc911\ub9e4\uccb4\ud559\ubd80\uc5d0 \uc5f0\uae30\uc804\uacf5\uc73c\ub85c \uc9c4\ud559\ud558\uc600\uace0 \uc774\ud6c4 2011\ub144 8\uc6d4 \ub300\ud559\uc744 \uc878\uc5c5\ud558\uc600\ub2e4.", "answer": "\ub3c4\uc608", "sentence": "\uc544\ubc84\uc9c0\ub294 \uc774\ucc9c\uc5d0\uc11c \ub3c4\uc608 \ub97c \ud558\ub294 \uc0ac\ub78c\uc774\uc5c8\ub2e4.", "paragraph_sentence": "\ucc9c\uc6b0\ud76c\ub294 1987\ub144 4\uc6d4 20\uc77c \uacbd\uae30\ub3c4 \uc774\ucc9c\uc2dc\uc5d0\uc11c 1\ub0a8 1\ub140 \uc911 \ub458\uc9f8\ub85c \ud0dc\uc5b4\ub0ac\ub2e4. \uc624\ube60\uc640\ub294 \uc138\uc0b4 \ud130\uc6b8\uc774\uc5c8\ub2e4. \uc544\ubc84\uc9c0\ub294 \uc774\ucc9c\uc5d0\uc11c \ub3c4\uc608 \ub97c \ud558\ub294 \uc0ac\ub78c\uc774\uc5c8\ub2e4. \ucc9c\uc6b0\ud76c\ub294 \ud559\ucc3d \uc2dc\uc808 \uce5c\uad6c\ub4e4\uacfc \ub450\ub8e8 \uc798 \uc5b4\uc6b8\ub9ac\ub294 \ucf8c\ud65c\ud55c \uc131\uaca9\uc73c\ub85c \ucd08\ub4f1\ud559\uad50 \uc2dc\uc808 \ubc18\uc7a5\uc744 \ub3c4\ub9e1\uc544 \ud558\uc600\uace0 \uc804\uad50\ud68c\uc7a5\ub3c4 \ud558\uc600\uc73c\uba70 \uc911\ud559\uad50 \ub54c\ub294 \uc804\uad50\ubd80\ud68c\uc7a5\uc744 \ud558\uc600\ub2e4. \uc774\ucc9c\uc591\uc815\uc5ec\uc790\uace0\ub4f1\ud559\uad50\uc5d0 \uc9c4\ud559\ud574\uc11c\ub294 \uc5f0\uadf9\ubc18 \ud65c\ub3d9\uc744 \ud558\uc600\ub294\ub370, \uadf8\ub140\uac00 \ucc98\uc74c\uc73c\ub85c \ubb34\ub300\uc5d0 \uc624\ub978 \uc791\ud488\uc740 \uc704\uc548\ubd80 \ud560\uba38\ub2c8\uc758 \uc774\uc57c\uae30\ub97c \ub2e4\ub8ec \u300a\ubc18\ucabd \ub0a0\uac1c\ub85c \ub0a0\uc544\uc628 \uc0c8\u300b\uc600\ub2e4. \ucc9c\uc6b0\ud76c\ub294 \uace0\uad50 \uc2dc\uc808 \uccad\uc18c\ub144 \uc5f0\uae30\ub300\ud68c\uc5d0\uc11c \uc5f0\uae30\uc0c1\uc744 \ubc1b\uae30\ub3c4 \ud558\uc600\ub2e4. \ucc9c\uc6b0\ud76c\ub294 \uc5f0\uadf9\ubc18 \ud65c\ub3d9\uc744 \ud558\uba70 \uc5f0\uae30\uc5d0 \ud765\ubbf8\ub97c \ub290\uaef4 2006\ub144 \uacbd\uae30\ub300\ud559\uad50 \ub2e4\uc911\ub9e4\uccb4\ud559\ubd80\uc5d0 \uc5f0\uae30\uc804\uacf5\uc73c\ub85c \uc9c4\ud559\ud558\uc600\uace0 \uc774\ud6c4 2011\ub144 8\uc6d4 \ub300\ud559\uc744 \uc878\uc5c5\ud558\uc600\ub2e4.", "paragraph_answer": "\ucc9c\uc6b0\ud76c\ub294 1987\ub144 4\uc6d4 20\uc77c \uacbd\uae30\ub3c4 \uc774\ucc9c\uc2dc\uc5d0\uc11c 1\ub0a8 1\ub140 \uc911 \ub458\uc9f8\ub85c \ud0dc\uc5b4\ub0ac\ub2e4. \uc624\ube60\uc640\ub294 \uc138\uc0b4 \ud130\uc6b8\uc774\uc5c8\ub2e4. \uc544\ubc84\uc9c0\ub294 \uc774\ucc9c\uc5d0\uc11c \ub3c4\uc608 \ub97c \ud558\ub294 \uc0ac\ub78c\uc774\uc5c8\ub2e4. \ucc9c\uc6b0\ud76c\ub294 \ud559\ucc3d \uc2dc\uc808 \uce5c\uad6c\ub4e4\uacfc \ub450\ub8e8 \uc798 \uc5b4\uc6b8\ub9ac\ub294 \ucf8c\ud65c\ud55c \uc131\uaca9\uc73c\ub85c \ucd08\ub4f1\ud559\uad50 \uc2dc\uc808 \ubc18\uc7a5\uc744 \ub3c4\ub9e1\uc544 \ud558\uc600\uace0 \uc804\uad50\ud68c\uc7a5\ub3c4 \ud558\uc600\uc73c\uba70 \uc911\ud559\uad50 \ub54c\ub294 \uc804\uad50\ubd80\ud68c\uc7a5\uc744 \ud558\uc600\ub2e4. \uc774\ucc9c\uc591\uc815\uc5ec\uc790\uace0\ub4f1\ud559\uad50\uc5d0 \uc9c4\ud559\ud574\uc11c\ub294 \uc5f0\uadf9\ubc18 \ud65c\ub3d9\uc744 \ud558\uc600\ub294\ub370, \uadf8\ub140\uac00 \ucc98\uc74c\uc73c\ub85c \ubb34\ub300\uc5d0 \uc624\ub978 \uc791\ud488\uc740 \uc704\uc548\ubd80 \ud560\uba38\ub2c8\uc758 \uc774\uc57c\uae30\ub97c \ub2e4\ub8ec \u300a\ubc18\ucabd \ub0a0\uac1c\ub85c \ub0a0\uc544\uc628 \uc0c8\u300b\uc600\ub2e4. \ucc9c\uc6b0\ud76c\ub294 \uace0\uad50 \uc2dc\uc808 \uccad\uc18c\ub144 \uc5f0\uae30\ub300\ud68c\uc5d0\uc11c \uc5f0\uae30\uc0c1\uc744 \ubc1b\uae30\ub3c4 \ud558\uc600\ub2e4. \ucc9c\uc6b0\ud76c\ub294 \uc5f0\uadf9\ubc18 \ud65c\ub3d9\uc744 \ud558\uba70 \uc5f0\uae30\uc5d0 \ud765\ubbf8\ub97c \ub290\uaef4 2006\ub144 \uacbd\uae30\ub300\ud559\uad50 \ub2e4\uc911\ub9e4\uccb4\ud559\ubd80\uc5d0 \uc5f0\uae30\uc804\uacf5\uc73c\ub85c \uc9c4\ud559\ud558\uc600\uace0 \uc774\ud6c4 2011\ub144 8\uc6d4 \ub300\ud559\uc744 \uc878\uc5c5\ud558\uc600\ub2e4.", "sentence_answer": "\uc544\ubc84\uc9c0\ub294 \uc774\ucc9c\uc5d0\uc11c \ub3c4\uc608 \ub97c \ud558\ub294 \uc0ac\ub78c\uc774\uc5c8\ub2e4.", "paragraph_id": "6596757-1-2", "paragraph_question": "question: \ucc9c\uc6b0\ud76c\uc758 \uc544\ubc84\uc9c0\ub294 \ubb34\uc5c7\uc744 \ud558\ub294 \uc0ac\ub78c\uc774\uc5c8\ub098?, context: \ucc9c\uc6b0\ud76c\ub294 1987\ub144 4\uc6d4 20\uc77c \uacbd\uae30\ub3c4 \uc774\ucc9c\uc2dc\uc5d0\uc11c 1\ub0a8 1\ub140 \uc911 \ub458\uc9f8\ub85c \ud0dc\uc5b4\ub0ac\ub2e4. \uc624\ube60\uc640\ub294 \uc138\uc0b4 \ud130\uc6b8\uc774\uc5c8\ub2e4. \uc544\ubc84\uc9c0\ub294 \uc774\ucc9c\uc5d0\uc11c \ub3c4\uc608\ub97c \ud558\ub294 \uc0ac\ub78c\uc774\uc5c8\ub2e4. \ucc9c\uc6b0\ud76c\ub294 \ud559\ucc3d \uc2dc\uc808 \uce5c\uad6c\ub4e4\uacfc \ub450\ub8e8 \uc798 \uc5b4\uc6b8\ub9ac\ub294 \ucf8c\ud65c\ud55c \uc131\uaca9\uc73c\ub85c \ucd08\ub4f1\ud559\uad50 \uc2dc\uc808 \ubc18\uc7a5\uc744 \ub3c4\ub9e1\uc544 \ud558\uc600\uace0 \uc804\uad50\ud68c\uc7a5\ub3c4 \ud558\uc600\uc73c\uba70 \uc911\ud559\uad50 \ub54c\ub294 \uc804\uad50\ubd80\ud68c\uc7a5\uc744 \ud558\uc600\ub2e4. \uc774\ucc9c\uc591\uc815\uc5ec\uc790\uace0\ub4f1\ud559\uad50\uc5d0 \uc9c4\ud559\ud574\uc11c\ub294 \uc5f0\uadf9\ubc18 \ud65c\ub3d9\uc744 \ud558\uc600\ub294\ub370, \uadf8\ub140\uac00 \ucc98\uc74c\uc73c\ub85c \ubb34\ub300\uc5d0 \uc624\ub978 \uc791\ud488\uc740 \uc704\uc548\ubd80 \ud560\uba38\ub2c8\uc758 \uc774\uc57c\uae30\ub97c \ub2e4\ub8ec \u300a\ubc18\ucabd \ub0a0\uac1c\ub85c \ub0a0\uc544\uc628 \uc0c8\u300b\uc600\ub2e4. \ucc9c\uc6b0\ud76c\ub294 \uace0\uad50 \uc2dc\uc808 \uccad\uc18c\ub144 \uc5f0\uae30\ub300\ud68c\uc5d0\uc11c \uc5f0\uae30\uc0c1\uc744 \ubc1b\uae30\ub3c4 \ud558\uc600\ub2e4. \ucc9c\uc6b0\ud76c\ub294 \uc5f0\uadf9\ubc18 \ud65c\ub3d9\uc744 \ud558\uba70 \uc5f0\uae30\uc5d0 \ud765\ubbf8\ub97c \ub290\uaef4 2006\ub144 \uacbd\uae30\ub300\ud559\uad50 \ub2e4\uc911\ub9e4\uccb4\ud559\ubd80\uc5d0 \uc5f0\uae30\uc804\uacf5\uc73c\ub85c \uc9c4\ud559\ud558\uc600\uace0 \uc774\ud6c4 2011\ub144 8\uc6d4 \ub300\ud559\uc744 \uc878\uc5c5\ud558\uc600\ub2e4."} {"question": "\ud33d\ub098\ubb34\uc5d0 \uc0b0\ub780\uc744 \ud558\uba70 \uba39\uc774\ub85c \uc88b\uc544\ud558\ub294 \uace4\ucda9\uc740?", "paragraph": "\ud33d\ub098\ubb34\ub97c \uba39\uc774\ub85c \uc88b\uc544\ud558\ub294 \ub098\ube44\uac00 \ub9ce\ub2e4. \ud64d\uc810\uc54c\ub77d\ub098\ube44, \uc218\ub178\ub791\ub098\ube44, \ud751\ubc31\uc54c\ub77d\ub098\ube44, \uc655\uc624\uc0c9\ub098\ube44, \ubfd4\ub098\ube44 \ub4f1\uc758 \uc560\ubc8c\ub808\uac00 \ud33d\ub098\ubb34\uc758 \uc78e\uc744 \uba39\uace0 \uc790\ub77c\uba70, \uc5ec\ub984\uc774 \ub418\uba74 \uc131\ucda9\uc774 \ub418\uc5b4 \uc9dd\uc9d3\uae30\ub97c \ub9c8\uce58\uba74 8\uc6d4 \uc911\uc21c \ubb34\ub835 \ud33d\ub098\ubb34\uc640 \ud48d\uac8c\ub098\ubb34\uc5d0 \uc0b0\ub780\ud55c\ub2e4. \ubfd4\ub098\ube44\ub97c \uc81c\uc678\ud558\uace4 \ubaa8\ub450 \uc560\ubc8c\ub808\ub85c \ud33d\ub098\ubb34 \ubc11\ub465 \ubd80\uadfc\uc758 \ub099\uc5fd \uc544\ub798\uc5d0\uc11c \ub3d9\uba74\uc744 \ud55c\ub2e4. \ud2b9\uc774\ud558\uac8c\ub3c4 \uc5ec\ub7ec \uc885\ub958\uc758 \ub099\uc5fd \uc911 \uba39\uc774\uc778 \ud33d\ub098\ubb34\ub098 \ud48d\uac8c\ub098\ubb34 \uc78e \uc544\ub798\uc5d0\uc11c\ub9cc \uaca8\uc6b8\uc7a0\uc744 \uc790\ub294\ub370, \uadf8 \uc774\uc720\ub294 \uc544\uc9c1 \ubc1d\ud600\uc9c0\uc9c0 \uc54a\uc558\ub2e4. \ud33d\uc774\ubc84\uc12f\uc740 \ud33d\ub098\ubb34 \uace0\ubaa9\uc5d0\uc11c \uc790\ub780\ub2e4. \ud33d\ub098\ubb34\uc54c\ub77d\uc9c4\ub527\ubb3c, \ud070\ud33d\ub098\ubb34\uc774 \ub4f1\uc774 \ud33d\ub098\ubb34\uc5d0 \uae30\uc0dd\ud558\uba70 \ud574\ub97c \ub07c\uce5c\ub2e4. \uc774\ubc16\uc5d0, \uae30\uc0dd \uc2dd\ubb3c\ub85c\ub294 \uaca8\uc6b0\uc0b4\uc774\uac00 \uc788\ub2e4. \ub098\ubb47\uc78e\uc774 \ub2e4 \ub5a8\uc5b4\uc9c4 \uaca8\uc6b8\uc5d0\uc57c \uad00\ucc30\uc774 \ub418\uba70, \ud0a4 \ud070 \ub098\ubb47\uac00\uc9c0\uc758 \ub05d\uc5d0 \uae30\uc0dd\ud558\uc5ec \uc5bc\ud54f \ubcf4\uba74 \uc0c8\ub465\uc9c0\ucc98\ub7fc \ubcf4\uc778\ub2e4.", "answer": "\ub098\ube44", "sentence": "\ud33d\ub098\ubb34\ub97c \uba39\uc774\ub85c \uc88b\uc544\ud558\ub294 \ub098\ube44 \uac00 \ub9ce\ub2e4.", "paragraph_sentence": " \ud33d\ub098\ubb34\ub97c \uba39\uc774\ub85c \uc88b\uc544\ud558\ub294 \ub098\ube44 \uac00 \ub9ce\ub2e4. \ud64d\uc810\uc54c\ub77d\ub098\ube44, \uc218\ub178\ub791\ub098\ube44, \ud751\ubc31\uc54c\ub77d\ub098\ube44, \uc655\uc624\uc0c9\ub098\ube44, \ubfd4\ub098\ube44 \ub4f1\uc758 \uc560\ubc8c\ub808\uac00 \ud33d\ub098\ubb34\uc758 \uc78e\uc744 \uba39\uace0 \uc790\ub77c\uba70, \uc5ec\ub984\uc774 \ub418\uba74 \uc131\ucda9\uc774 \ub418\uc5b4 \uc9dd\uc9d3\uae30\ub97c \ub9c8\uce58\uba74 8\uc6d4 \uc911\uc21c \ubb34\ub835 \ud33d\ub098\ubb34\uc640 \ud48d\uac8c\ub098\ubb34\uc5d0 \uc0b0\ub780\ud55c\ub2e4. \ubfd4\ub098\ube44\ub97c \uc81c\uc678\ud558\uace4 \ubaa8\ub450 \uc560\ubc8c\ub808\ub85c \ud33d\ub098\ubb34 \ubc11\ub465 \ubd80\uadfc\uc758 \ub099\uc5fd \uc544\ub798\uc5d0\uc11c \ub3d9\uba74\uc744 \ud55c\ub2e4. \ud2b9\uc774\ud558\uac8c\ub3c4 \uc5ec\ub7ec \uc885\ub958\uc758 \ub099\uc5fd \uc911 \uba39\uc774\uc778 \ud33d\ub098\ubb34\ub098 \ud48d\uac8c\ub098\ubb34 \uc78e \uc544\ub798\uc5d0\uc11c\ub9cc \uaca8\uc6b8\uc7a0\uc744 \uc790\ub294\ub370, \uadf8 \uc774\uc720\ub294 \uc544\uc9c1 \ubc1d\ud600\uc9c0\uc9c0 \uc54a\uc558\ub2e4. \ud33d\uc774\ubc84\uc12f\uc740 \ud33d\ub098\ubb34 \uace0\ubaa9\uc5d0\uc11c \uc790\ub780\ub2e4. \ud33d\ub098\ubb34\uc54c\ub77d\uc9c4\ub527\ubb3c, \ud070\ud33d\ub098\ubb34\uc774 \ub4f1\uc774 \ud33d\ub098\ubb34\uc5d0 \uae30\uc0dd\ud558\uba70 \ud574\ub97c \ub07c\uce5c\ub2e4. \uc774\ubc16\uc5d0, \uae30\uc0dd \uc2dd\ubb3c\ub85c\ub294 \uaca8\uc6b0\uc0b4\uc774\uac00 \uc788\ub2e4. \ub098\ubb47\uc78e\uc774 \ub2e4 \ub5a8\uc5b4\uc9c4 \uaca8\uc6b8\uc5d0\uc57c \uad00\ucc30\uc774 \ub418\uba70, \ud0a4 \ud070 \ub098\ubb47\uac00\uc9c0\uc758 \ub05d\uc5d0 \uae30\uc0dd\ud558\uc5ec \uc5bc\ud54f \ubcf4\uba74 \uc0c8\ub465\uc9c0\ucc98\ub7fc \ubcf4\uc778\ub2e4.", "paragraph_answer": "\ud33d\ub098\ubb34\ub97c \uba39\uc774\ub85c \uc88b\uc544\ud558\ub294 \ub098\ube44 \uac00 \ub9ce\ub2e4. \ud64d\uc810\uc54c\ub77d\ub098\ube44, \uc218\ub178\ub791\ub098\ube44, \ud751\ubc31\uc54c\ub77d\ub098\ube44, \uc655\uc624\uc0c9\ub098\ube44, \ubfd4\ub098\ube44 \ub4f1\uc758 \uc560\ubc8c\ub808\uac00 \ud33d\ub098\ubb34\uc758 \uc78e\uc744 \uba39\uace0 \uc790\ub77c\uba70, \uc5ec\ub984\uc774 \ub418\uba74 \uc131\ucda9\uc774 \ub418\uc5b4 \uc9dd\uc9d3\uae30\ub97c \ub9c8\uce58\uba74 8\uc6d4 \uc911\uc21c \ubb34\ub835 \ud33d\ub098\ubb34\uc640 \ud48d\uac8c\ub098\ubb34\uc5d0 \uc0b0\ub780\ud55c\ub2e4. \ubfd4\ub098\ube44\ub97c \uc81c\uc678\ud558\uace4 \ubaa8\ub450 \uc560\ubc8c\ub808\ub85c \ud33d\ub098\ubb34 \ubc11\ub465 \ubd80\uadfc\uc758 \ub099\uc5fd \uc544\ub798\uc5d0\uc11c \ub3d9\uba74\uc744 \ud55c\ub2e4. \ud2b9\uc774\ud558\uac8c\ub3c4 \uc5ec\ub7ec \uc885\ub958\uc758 \ub099\uc5fd \uc911 \uba39\uc774\uc778 \ud33d\ub098\ubb34\ub098 \ud48d\uac8c\ub098\ubb34 \uc78e \uc544\ub798\uc5d0\uc11c\ub9cc \uaca8\uc6b8\uc7a0\uc744 \uc790\ub294\ub370, \uadf8 \uc774\uc720\ub294 \uc544\uc9c1 \ubc1d\ud600\uc9c0\uc9c0 \uc54a\uc558\ub2e4. \ud33d\uc774\ubc84\uc12f\uc740 \ud33d\ub098\ubb34 \uace0\ubaa9\uc5d0\uc11c \uc790\ub780\ub2e4. \ud33d\ub098\ubb34\uc54c\ub77d\uc9c4\ub527\ubb3c, \ud070\ud33d\ub098\ubb34\uc774 \ub4f1\uc774 \ud33d\ub098\ubb34\uc5d0 \uae30\uc0dd\ud558\uba70 \ud574\ub97c \ub07c\uce5c\ub2e4. \uc774\ubc16\uc5d0, \uae30\uc0dd \uc2dd\ubb3c\ub85c\ub294 \uaca8\uc6b0\uc0b4\uc774\uac00 \uc788\ub2e4. \ub098\ubb47\uc78e\uc774 \ub2e4 \ub5a8\uc5b4\uc9c4 \uaca8\uc6b8\uc5d0\uc57c \uad00\ucc30\uc774 \ub418\uba70, \ud0a4 \ud070 \ub098\ubb47\uac00\uc9c0\uc758 \ub05d\uc5d0 \uae30\uc0dd\ud558\uc5ec \uc5bc\ud54f \ubcf4\uba74 \uc0c8\ub465\uc9c0\ucc98\ub7fc \ubcf4\uc778\ub2e4.", "sentence_answer": "\ud33d\ub098\ubb34\ub97c \uba39\uc774\ub85c \uc88b\uc544\ud558\ub294 \ub098\ube44 \uac00 \ub9ce\ub2e4.", "paragraph_id": "5918454-2-2", "paragraph_question": "question: \ud33d\ub098\ubb34\uc5d0 \uc0b0\ub780\uc744 \ud558\uba70 \uba39\uc774\ub85c \uc88b\uc544\ud558\ub294 \uace4\ucda9\uc740?, context: \ud33d\ub098\ubb34\ub97c \uba39\uc774\ub85c \uc88b\uc544\ud558\ub294 \ub098\ube44\uac00 \ub9ce\ub2e4. \ud64d\uc810\uc54c\ub77d\ub098\ube44, \uc218\ub178\ub791\ub098\ube44, \ud751\ubc31\uc54c\ub77d\ub098\ube44, \uc655\uc624\uc0c9\ub098\ube44, \ubfd4\ub098\ube44 \ub4f1\uc758 \uc560\ubc8c\ub808\uac00 \ud33d\ub098\ubb34\uc758 \uc78e\uc744 \uba39\uace0 \uc790\ub77c\uba70, \uc5ec\ub984\uc774 \ub418\uba74 \uc131\ucda9\uc774 \ub418\uc5b4 \uc9dd\uc9d3\uae30\ub97c \ub9c8\uce58\uba74 8\uc6d4 \uc911\uc21c \ubb34\ub835 \ud33d\ub098\ubb34\uc640 \ud48d\uac8c\ub098\ubb34\uc5d0 \uc0b0\ub780\ud55c\ub2e4. \ubfd4\ub098\ube44\ub97c \uc81c\uc678\ud558\uace4 \ubaa8\ub450 \uc560\ubc8c\ub808\ub85c \ud33d\ub098\ubb34 \ubc11\ub465 \ubd80\uadfc\uc758 \ub099\uc5fd \uc544\ub798\uc5d0\uc11c \ub3d9\uba74\uc744 \ud55c\ub2e4. \ud2b9\uc774\ud558\uac8c\ub3c4 \uc5ec\ub7ec \uc885\ub958\uc758 \ub099\uc5fd \uc911 \uba39\uc774\uc778 \ud33d\ub098\ubb34\ub098 \ud48d\uac8c\ub098\ubb34 \uc78e \uc544\ub798\uc5d0\uc11c\ub9cc \uaca8\uc6b8\uc7a0\uc744 \uc790\ub294\ub370, \uadf8 \uc774\uc720\ub294 \uc544\uc9c1 \ubc1d\ud600\uc9c0\uc9c0 \uc54a\uc558\ub2e4. \ud33d\uc774\ubc84\uc12f\uc740 \ud33d\ub098\ubb34 \uace0\ubaa9\uc5d0\uc11c \uc790\ub780\ub2e4. \ud33d\ub098\ubb34\uc54c\ub77d\uc9c4\ub527\ubb3c, \ud070\ud33d\ub098\ubb34\uc774 \ub4f1\uc774 \ud33d\ub098\ubb34\uc5d0 \uae30\uc0dd\ud558\uba70 \ud574\ub97c \ub07c\uce5c\ub2e4. \uc774\ubc16\uc5d0, \uae30\uc0dd \uc2dd\ubb3c\ub85c\ub294 \uaca8\uc6b0\uc0b4\uc774\uac00 \uc788\ub2e4. \ub098\ubb47\uc78e\uc774 \ub2e4 \ub5a8\uc5b4\uc9c4 \uaca8\uc6b8\uc5d0\uc57c \uad00\ucc30\uc774 \ub418\uba70, \ud0a4 \ud070 \ub098\ubb47\uac00\uc9c0\uc758 \ub05d\uc5d0 \uae30\uc0dd\ud558\uc5ec \uc5bc\ud54f \ubcf4\uba74 \uc0c8\ub465\uc9c0\ucc98\ub7fc \ubcf4\uc778\ub2e4."} {"question": "\uae40\ub300\uc911\uc758 \ubaa9\ud3ec\ud574\uc0c1\ubc29\uc704\ub300 \uacbd\ub825\uc744 \uc639\ud638\ud558\uace0\uc790 \ubbf8\uad6d\uc73c\ub85c \uac04 \uad6d\ubbfc\ud68c\uc758 \uc758\uc6d0\uc740?", "paragraph": "\uc774\uc5b4 \ud574\uad70 \uacbd\ube44\ub300 \uc18c\uc18d \ubaa9\ud3ec\ud574\uc0c1\ubc29\uc704\ub300 \uc804\ub0a8\uc9c0\uad6c \ubd80\ub300\uc7a5\uc744 \ub9e1\uc544 \ub2f9\uc2dc \uc790\uc2e0\uc758 \ud574\uc6b4\ud68c\uc0ac\uc5d0\uc11c \ubcf4\uc720\ud558\uace0 \uc788\ub358 \uc120\ubc15\uc744 \ub3d9\uc6d0\ud574 \ucc38\uc804\ud588\ub2e4\uace0 \uc99d\uc5b8\ud588\ub2e4. \uadf8\ub7ec\ub098 \uc2e0\ud55c\uad6d\ub2f9\uc740 \ubaa9\ud3ec\ud574\uc0c1\ubc29\uc704\ub300\uac00 \uc720\ub839 \uad70\ubd80\ub300\ub77c\uace0 \uc8fc\uc7a5\ud588\uace0, \uad6d\ubbfc\ud68c\uc758 \uce21\uc5d0\uc120 \ud574\uc0c1\ubc29\uc704\ub300\ub294 1950\ub144 \uc721\uad70\ubcf8\ubd80 \uc791\uc804\uba85\ub839 \uc81c38\ud638\uc640 \uad6d\ubc29\uad70\uc0ac\uc5f0\uad6c\uc18c\uc5d0\uc11c \ubc1c\ud589\ud55c \uad6d\ubc29\uc0ac\uc5f0\ud45c\uc5d0 \ubd84\uba85\ud788 \uadfc\uac70\uac00 \uc788\ub2e4\uace0 \ubc18\ubc15\ud558\ub294 \ub4f1 \uc591\uce21\uc758 \uacf5\ubc29\uc774 \uc774\uc5b4\uc84c\ub2e4. \uc774\ud6c4 \uad6d\ubbfc\ud68c\uc758 \ucc9c\uc6a9\ud0dd \uc758\uc6d0\uc774 \ubbf8\uad6d\uc5d0 \uac70\uc8fc\uc911\uc778 \ud574\ubcd1\ub300 \uc608\ube44\uc5ed \uc900\uc7a5 \uc1a1\uc778\uba85\uc744 \ub9cc\ub098 \uadf8\uc758 \uc99d\uc5b8\ub85d\uacfc \uc9c1\uc811 \uc4f4 \uc99d\uba85\uc11c \ub4f1\uc744 \uac00\uc838\uc640 \uacf5\uac1c\ud558\uace0, \uc1a1\uc778\uba85\uc740 \uc804 \ubaa9\ud3ec\uacbd\ube44\ubd80\uc0ac\ub839\uad00\uc73c\ub85c \ubd80\uc784\ud588\uc744 \ub54c \uad70\uacfc \uacbd\ucc30\ub9cc\uc73c\ub85c \uacf5\uc0b0\uc138\ub825\uc758 \uc794\ub2f9\uc18c\ud0d5\uacfc \uc548\ub155\uc9c8\uc11c \uc720\uc9c0\uc5d0 \uc5b4\ub824\uc6c0\uc774 \ub9ce\uc558\uae30\ub54c\ubb38\uc5d0 \ubaa9\ud3ec\uc2dc\ub0b4 \uccad\ub144\ub2e8\uacfc \uc5ec\uc131\uccad\ub144\ub2e8 \ub4f1 \uc560\uad6d\ub2e8\uccb4\ub4e4\uc744 \ubaa8\ub450 \ub3d9\uc6d0\ud574 \uc791\uc804\uc744 \ubcf4\uc870\ud558\uac8c \ud558\uc600\uc73c\uba70, \ud2b9\ud788 \ud574\uc0c1\uc791\uc804\uc774 \uc911\uc694\ud588\ub2e4\uace0 \ud310\ub2e8\ud588\uae30 \ub54c\ubb38\uc5d0 \ub2f9\uc2dc \uc9c0\uc5ed\uc720\uc9c0 \uc624\uc7ac\uade0\uc5d0\uac8c \uc9c0\uc2dc, \ud574\uc0c1\ubc29\uc704\ub300\ub97c \ucc3d\uc124\ud574 \uae40\ub300\uc911\uc5d0\uac8c \ubd80\ub300\uc7a5\uc744 \ub9e1\uacbc\ub2e4\uace0 \ud574\uba85\ud588\ub2e4.", "answer": "\ucc9c\uc6a9\ud0dd", "sentence": "\uc774\ud6c4 \uad6d\ubbfc\ud68c\uc758 \ucc9c\uc6a9\ud0dd \uc758\uc6d0\uc774 \ubbf8\uad6d\uc5d0 \uac70\uc8fc\uc911\uc778 \ud574\ubcd1\ub300 \uc608\ube44\uc5ed \uc900\uc7a5 \uc1a1\uc778\uba85\uc744 \ub9cc\ub098 \uadf8\uc758 \uc99d\uc5b8\ub85d\uacfc \uc9c1\uc811 \uc4f4 \uc99d\uba85\uc11c \ub4f1\uc744 \uac00\uc838\uc640 \uacf5\uac1c\ud558\uace0, \uc1a1\uc778\uba85\uc740 \uc804 \ubaa9\ud3ec\uacbd\ube44\ubd80\uc0ac\ub839\uad00\uc73c\ub85c \ubd80\uc784\ud588\uc744 \ub54c \uad70\uacfc \uacbd\ucc30\ub9cc\uc73c\ub85c \uacf5\uc0b0\uc138\ub825\uc758 \uc794\ub2f9\uc18c\ud0d5\uacfc \uc548\ub155\uc9c8\uc11c \uc720\uc9c0\uc5d0 \uc5b4\ub824\uc6c0\uc774 \ub9ce\uc558\uae30\ub54c\ubb38\uc5d0 \ubaa9\ud3ec\uc2dc\ub0b4 \uccad\ub144\ub2e8\uacfc \uc5ec\uc131\uccad\ub144\ub2e8 \ub4f1 \uc560\uad6d\ub2e8\uccb4\ub4e4\uc744 \ubaa8\ub450 \ub3d9\uc6d0\ud574 \uc791\uc804\uc744 \ubcf4\uc870\ud558\uac8c \ud558\uc600\uc73c\uba70, \ud2b9\ud788 \ud574\uc0c1\uc791\uc804\uc774 \uc911\uc694\ud588\ub2e4\uace0 \ud310\ub2e8\ud588\uae30 \ub54c\ubb38\uc5d0 \ub2f9\uc2dc \uc9c0\uc5ed\uc720\uc9c0 \uc624\uc7ac\uade0\uc5d0\uac8c \uc9c0\uc2dc, \ud574\uc0c1\ubc29\uc704\ub300\ub97c \ucc3d\uc124\ud574 \uae40\ub300\uc911\uc5d0\uac8c \ubd80\ub300\uc7a5\uc744 \ub9e1\uacbc\ub2e4\uace0 \ud574\uba85\ud588\ub2e4.", "paragraph_sentence": "\uc774\uc5b4 \ud574\uad70 \uacbd\ube44\ub300 \uc18c\uc18d \ubaa9\ud3ec\ud574\uc0c1\ubc29\uc704\ub300 \uc804\ub0a8\uc9c0\uad6c \ubd80\ub300\uc7a5\uc744 \ub9e1\uc544 \ub2f9\uc2dc \uc790\uc2e0\uc758 \ud574\uc6b4\ud68c\uc0ac\uc5d0\uc11c \ubcf4\uc720\ud558\uace0 \uc788\ub358 \uc120\ubc15\uc744 \ub3d9\uc6d0\ud574 \ucc38\uc804\ud588\ub2e4\uace0 \uc99d\uc5b8\ud588\ub2e4. \uadf8\ub7ec\ub098 \uc2e0\ud55c\uad6d\ub2f9\uc740 \ubaa9\ud3ec\ud574\uc0c1\ubc29\uc704\ub300\uac00 \uc720\ub839 \uad70\ubd80\ub300\ub77c\uace0 \uc8fc\uc7a5\ud588\uace0, \uad6d\ubbfc\ud68c\uc758 \uce21\uc5d0\uc120 \ud574\uc0c1\ubc29\uc704\ub300\ub294 1950\ub144 \uc721\uad70\ubcf8\ubd80 \uc791\uc804\uba85\ub839 \uc81c38\ud638\uc640 \uad6d\ubc29\uad70\uc0ac\uc5f0\uad6c\uc18c\uc5d0\uc11c \ubc1c\ud589\ud55c \uad6d\ubc29\uc0ac\uc5f0\ud45c\uc5d0 \ubd84\uba85\ud788 \uadfc\uac70\uac00 \uc788\ub2e4\uace0 \ubc18\ubc15\ud558\ub294 \ub4f1 \uc591\uce21\uc758 \uacf5\ubc29\uc774 \uc774\uc5b4\uc84c\ub2e4. \uc774\ud6c4 \uad6d\ubbfc\ud68c\uc758 \ucc9c\uc6a9\ud0dd \uc758\uc6d0\uc774 \ubbf8\uad6d\uc5d0 \uac70\uc8fc\uc911\uc778 \ud574\ubcd1\ub300 \uc608\ube44\uc5ed \uc900\uc7a5 \uc1a1\uc778\uba85\uc744 \ub9cc\ub098 \uadf8\uc758 \uc99d\uc5b8\ub85d\uacfc \uc9c1\uc811 \uc4f4 \uc99d\uba85\uc11c \ub4f1\uc744 \uac00\uc838\uc640 \uacf5\uac1c\ud558\uace0, \uc1a1\uc778\uba85\uc740 \uc804 \ubaa9\ud3ec\uacbd\ube44\ubd80\uc0ac\ub839\uad00\uc73c\ub85c \ubd80\uc784\ud588\uc744 \ub54c \uad70\uacfc \uacbd\ucc30\ub9cc\uc73c\ub85c \uacf5\uc0b0\uc138\ub825\uc758 \uc794\ub2f9\uc18c\ud0d5\uacfc \uc548\ub155\uc9c8\uc11c \uc720\uc9c0\uc5d0 \uc5b4\ub824\uc6c0\uc774 \ub9ce\uc558\uae30\ub54c\ubb38\uc5d0 \ubaa9\ud3ec\uc2dc\ub0b4 \uccad\ub144\ub2e8\uacfc \uc5ec\uc131\uccad\ub144\ub2e8 \ub4f1 \uc560\uad6d\ub2e8\uccb4\ub4e4\uc744 \ubaa8\ub450 \ub3d9\uc6d0\ud574 \uc791\uc804\uc744 \ubcf4\uc870\ud558\uac8c \ud558\uc600\uc73c\uba70, \ud2b9\ud788 \ud574\uc0c1\uc791\uc804\uc774 \uc911\uc694\ud588\ub2e4\uace0 \ud310\ub2e8\ud588\uae30 \ub54c\ubb38\uc5d0 \ub2f9\uc2dc \uc9c0\uc5ed\uc720\uc9c0 \uc624\uc7ac\uade0\uc5d0\uac8c \uc9c0\uc2dc, \ud574\uc0c1\ubc29\uc704\ub300\ub97c \ucc3d\uc124\ud574 \uae40\ub300\uc911\uc5d0\uac8c \ubd80\ub300\uc7a5\uc744 \ub9e1\uacbc\ub2e4\uace0 \ud574\uba85\ud588\ub2e4. ", "paragraph_answer": "\uc774\uc5b4 \ud574\uad70 \uacbd\ube44\ub300 \uc18c\uc18d \ubaa9\ud3ec\ud574\uc0c1\ubc29\uc704\ub300 \uc804\ub0a8\uc9c0\uad6c \ubd80\ub300\uc7a5\uc744 \ub9e1\uc544 \ub2f9\uc2dc \uc790\uc2e0\uc758 \ud574\uc6b4\ud68c\uc0ac\uc5d0\uc11c \ubcf4\uc720\ud558\uace0 \uc788\ub358 \uc120\ubc15\uc744 \ub3d9\uc6d0\ud574 \ucc38\uc804\ud588\ub2e4\uace0 \uc99d\uc5b8\ud588\ub2e4. \uadf8\ub7ec\ub098 \uc2e0\ud55c\uad6d\ub2f9\uc740 \ubaa9\ud3ec\ud574\uc0c1\ubc29\uc704\ub300\uac00 \uc720\ub839 \uad70\ubd80\ub300\ub77c\uace0 \uc8fc\uc7a5\ud588\uace0, \uad6d\ubbfc\ud68c\uc758 \uce21\uc5d0\uc120 \ud574\uc0c1\ubc29\uc704\ub300\ub294 1950\ub144 \uc721\uad70\ubcf8\ubd80 \uc791\uc804\uba85\ub839 \uc81c38\ud638\uc640 \uad6d\ubc29\uad70\uc0ac\uc5f0\uad6c\uc18c\uc5d0\uc11c \ubc1c\ud589\ud55c \uad6d\ubc29\uc0ac\uc5f0\ud45c\uc5d0 \ubd84\uba85\ud788 \uadfc\uac70\uac00 \uc788\ub2e4\uace0 \ubc18\ubc15\ud558\ub294 \ub4f1 \uc591\uce21\uc758 \uacf5\ubc29\uc774 \uc774\uc5b4\uc84c\ub2e4. \uc774\ud6c4 \uad6d\ubbfc\ud68c\uc758 \ucc9c\uc6a9\ud0dd \uc758\uc6d0\uc774 \ubbf8\uad6d\uc5d0 \uac70\uc8fc\uc911\uc778 \ud574\ubcd1\ub300 \uc608\ube44\uc5ed \uc900\uc7a5 \uc1a1\uc778\uba85\uc744 \ub9cc\ub098 \uadf8\uc758 \uc99d\uc5b8\ub85d\uacfc \uc9c1\uc811 \uc4f4 \uc99d\uba85\uc11c \ub4f1\uc744 \uac00\uc838\uc640 \uacf5\uac1c\ud558\uace0, \uc1a1\uc778\uba85\uc740 \uc804 \ubaa9\ud3ec\uacbd\ube44\ubd80\uc0ac\ub839\uad00\uc73c\ub85c \ubd80\uc784\ud588\uc744 \ub54c \uad70\uacfc \uacbd\ucc30\ub9cc\uc73c\ub85c \uacf5\uc0b0\uc138\ub825\uc758 \uc794\ub2f9\uc18c\ud0d5\uacfc \uc548\ub155\uc9c8\uc11c \uc720\uc9c0\uc5d0 \uc5b4\ub824\uc6c0\uc774 \ub9ce\uc558\uae30\ub54c\ubb38\uc5d0 \ubaa9\ud3ec\uc2dc\ub0b4 \uccad\ub144\ub2e8\uacfc \uc5ec\uc131\uccad\ub144\ub2e8 \ub4f1 \uc560\uad6d\ub2e8\uccb4\ub4e4\uc744 \ubaa8\ub450 \ub3d9\uc6d0\ud574 \uc791\uc804\uc744 \ubcf4\uc870\ud558\uac8c \ud558\uc600\uc73c\uba70, \ud2b9\ud788 \ud574\uc0c1\uc791\uc804\uc774 \uc911\uc694\ud588\ub2e4\uace0 \ud310\ub2e8\ud588\uae30 \ub54c\ubb38\uc5d0 \ub2f9\uc2dc \uc9c0\uc5ed\uc720\uc9c0 \uc624\uc7ac\uade0\uc5d0\uac8c \uc9c0\uc2dc, \ud574\uc0c1\ubc29\uc704\ub300\ub97c \ucc3d\uc124\ud574 \uae40\ub300\uc911\uc5d0\uac8c \ubd80\ub300\uc7a5\uc744 \ub9e1\uacbc\ub2e4\uace0 \ud574\uba85\ud588\ub2e4.", "sentence_answer": "\uc774\ud6c4 \uad6d\ubbfc\ud68c\uc758 \ucc9c\uc6a9\ud0dd \uc758\uc6d0\uc774 \ubbf8\uad6d\uc5d0 \uac70\uc8fc\uc911\uc778 \ud574\ubcd1\ub300 \uc608\ube44\uc5ed \uc900\uc7a5 \uc1a1\uc778\uba85\uc744 \ub9cc\ub098 \uadf8\uc758 \uc99d\uc5b8\ub85d\uacfc \uc9c1\uc811 \uc4f4 \uc99d\uba85\uc11c \ub4f1\uc744 \uac00\uc838\uc640 \uacf5\uac1c\ud558\uace0, \uc1a1\uc778\uba85\uc740 \uc804 \ubaa9\ud3ec\uacbd\ube44\ubd80\uc0ac\ub839\uad00\uc73c\ub85c \ubd80\uc784\ud588\uc744 \ub54c \uad70\uacfc \uacbd\ucc30\ub9cc\uc73c\ub85c \uacf5\uc0b0\uc138\ub825\uc758 \uc794\ub2f9\uc18c\ud0d5\uacfc \uc548\ub155\uc9c8\uc11c \uc720\uc9c0\uc5d0 \uc5b4\ub824\uc6c0\uc774 \ub9ce\uc558\uae30\ub54c\ubb38\uc5d0 \ubaa9\ud3ec\uc2dc\ub0b4 \uccad\ub144\ub2e8\uacfc \uc5ec\uc131\uccad\ub144\ub2e8 \ub4f1 \uc560\uad6d\ub2e8\uccb4\ub4e4\uc744 \ubaa8\ub450 \ub3d9\uc6d0\ud574 \uc791\uc804\uc744 \ubcf4\uc870\ud558\uac8c \ud558\uc600\uc73c\uba70, \ud2b9\ud788 \ud574\uc0c1\uc791\uc804\uc774 \uc911\uc694\ud588\ub2e4\uace0 \ud310\ub2e8\ud588\uae30 \ub54c\ubb38\uc5d0 \ub2f9\uc2dc \uc9c0\uc5ed\uc720\uc9c0 \uc624\uc7ac\uade0\uc5d0\uac8c \uc9c0\uc2dc, \ud574\uc0c1\ubc29\uc704\ub300\ub97c \ucc3d\uc124\ud574 \uae40\ub300\uc911\uc5d0\uac8c \ubd80\ub300\uc7a5\uc744 \ub9e1\uacbc\ub2e4\uace0 \ud574\uba85\ud588\ub2e4.", "paragraph_id": "6488203-85-0", "paragraph_question": "question: \uae40\ub300\uc911\uc758 \ubaa9\ud3ec\ud574\uc0c1\ubc29\uc704\ub300 \uacbd\ub825\uc744 \uc639\ud638\ud558\uace0\uc790 \ubbf8\uad6d\uc73c\ub85c \uac04 \uad6d\ubbfc\ud68c\uc758 \uc758\uc6d0\uc740?, context: \uc774\uc5b4 \ud574\uad70 \uacbd\ube44\ub300 \uc18c\uc18d \ubaa9\ud3ec\ud574\uc0c1\ubc29\uc704\ub300 \uc804\ub0a8\uc9c0\uad6c \ubd80\ub300\uc7a5\uc744 \ub9e1\uc544 \ub2f9\uc2dc \uc790\uc2e0\uc758 \ud574\uc6b4\ud68c\uc0ac\uc5d0\uc11c \ubcf4\uc720\ud558\uace0 \uc788\ub358 \uc120\ubc15\uc744 \ub3d9\uc6d0\ud574 \ucc38\uc804\ud588\ub2e4\uace0 \uc99d\uc5b8\ud588\ub2e4. \uadf8\ub7ec\ub098 \uc2e0\ud55c\uad6d\ub2f9\uc740 \ubaa9\ud3ec\ud574\uc0c1\ubc29\uc704\ub300\uac00 \uc720\ub839 \uad70\ubd80\ub300\ub77c\uace0 \uc8fc\uc7a5\ud588\uace0, \uad6d\ubbfc\ud68c\uc758 \uce21\uc5d0\uc120 \ud574\uc0c1\ubc29\uc704\ub300\ub294 1950\ub144 \uc721\uad70\ubcf8\ubd80 \uc791\uc804\uba85\ub839 \uc81c38\ud638\uc640 \uad6d\ubc29\uad70\uc0ac\uc5f0\uad6c\uc18c\uc5d0\uc11c \ubc1c\ud589\ud55c \uad6d\ubc29\uc0ac\uc5f0\ud45c\uc5d0 \ubd84\uba85\ud788 \uadfc\uac70\uac00 \uc788\ub2e4\uace0 \ubc18\ubc15\ud558\ub294 \ub4f1 \uc591\uce21\uc758 \uacf5\ubc29\uc774 \uc774\uc5b4\uc84c\ub2e4. \uc774\ud6c4 \uad6d\ubbfc\ud68c\uc758 \ucc9c\uc6a9\ud0dd \uc758\uc6d0\uc774 \ubbf8\uad6d\uc5d0 \uac70\uc8fc\uc911\uc778 \ud574\ubcd1\ub300 \uc608\ube44\uc5ed \uc900\uc7a5 \uc1a1\uc778\uba85\uc744 \ub9cc\ub098 \uadf8\uc758 \uc99d\uc5b8\ub85d\uacfc \uc9c1\uc811 \uc4f4 \uc99d\uba85\uc11c \ub4f1\uc744 \uac00\uc838\uc640 \uacf5\uac1c\ud558\uace0, \uc1a1\uc778\uba85\uc740 \uc804 \ubaa9\ud3ec\uacbd\ube44\ubd80\uc0ac\ub839\uad00\uc73c\ub85c \ubd80\uc784\ud588\uc744 \ub54c \uad70\uacfc \uacbd\ucc30\ub9cc\uc73c\ub85c \uacf5\uc0b0\uc138\ub825\uc758 \uc794\ub2f9\uc18c\ud0d5\uacfc \uc548\ub155\uc9c8\uc11c \uc720\uc9c0\uc5d0 \uc5b4\ub824\uc6c0\uc774 \ub9ce\uc558\uae30\ub54c\ubb38\uc5d0 \ubaa9\ud3ec\uc2dc\ub0b4 \uccad\ub144\ub2e8\uacfc \uc5ec\uc131\uccad\ub144\ub2e8 \ub4f1 \uc560\uad6d\ub2e8\uccb4\ub4e4\uc744 \ubaa8\ub450 \ub3d9\uc6d0\ud574 \uc791\uc804\uc744 \ubcf4\uc870\ud558\uac8c \ud558\uc600\uc73c\uba70, \ud2b9\ud788 \ud574\uc0c1\uc791\uc804\uc774 \uc911\uc694\ud588\ub2e4\uace0 \ud310\ub2e8\ud588\uae30 \ub54c\ubb38\uc5d0 \ub2f9\uc2dc \uc9c0\uc5ed\uc720\uc9c0 \uc624\uc7ac\uade0\uc5d0\uac8c \uc9c0\uc2dc, \ud574\uc0c1\ubc29\uc704\ub300\ub97c \ucc3d\uc124\ud574 \uae40\ub300\uc911\uc5d0\uac8c \ubd80\ub300\uc7a5\uc744 \ub9e1\uacbc\ub2e4\uace0 \ud574\uba85\ud588\ub2e4."} {"question": "\uc218\uc544\ub808\uc2a4\uac00 2014\ub144 \uc6d4\ub4dc\ucef5 \ub0a8\ubbf8 \uc608\uc120\uc5d0\uc11c 4\uace8 \ud574\ud2b8\ud2b8\ub9ad\uc744 \uae30\ub85d\ud588\ub358 \uacbd\uae30\ub294 \uc5b4\ub5a4 \ub098\ub77c\ub97c \uc0c1\ub300\ub85c \ud55c \uacbd\uae30\uc600\ub294\uac00?", "paragraph": "\uc218\uc544\ub808\uc2a4\ub294 2014\ub144 FIFA \uc6d4\ub4dc\ucef5 \ub0a8\uc544\uba54\ub9ac\uce74 \uc9c0\uc5ed \uc608\uc120\uc5d0\uc11c 3\ub77c\uc6b4\ub4dc \uce60\ub808\uc804\uc5d0\uc11c \ud574\ud2b8\ud2b8\ub9ad(4\uace8)\uc744 \uae30\ub85d\ud558\ub294 \ub4f1 \ub2e8 \uc138 \uacbd\uae30\ub9cc\uc5d0 5\uace8\uc744 \uae30\ub85d\ud55c \uc0c1\ud0dc\uc774\uba70 \ud604\uc7ac \uae30\uc874\uc758 \ub0a8\ubbf8 \uc9c0\uc5ed\uc608\uc120 \ucd5c\ub2e4\ub4dd\uc810\uc790\uc778 \uc544\ub974\ud5e8\ud2f0\ub098\uc758 \uc5d0\ub974\ub09c \ud06c\ub808\uc2a4\ud3ec\uac00 2002\ub144\uc5d0 \uae30\ub85d\ud55c \ub0a8\uc544\uba54\ub9ac\uce74 \uc9c0\uc5ed\uc608\uc120 \uc5ed\uc0ac\uc0c1 \ucd5c\ub2e4\ub4dd\uc810\uae30\ub85d\uc778 9\uace8\uc744 4\uace8 \ub0a8\uaca8\ub193\uc740 \uc0c1\ud0dc\uac00 \ub418\uc5c8\ub2e4. \uadf8\ub7ec\ub098 \uadf8 \uc774\ud6c4 \ud65c\uc57d\uc774 \uc904\uc5b4\ub4e4\uba74\uc11c 9\ucc28\uc804\uc774 \uc885\ub8cc\ub41c \uc2dc\uc810\uc5d0\uc11c\ub294 6\uace8\ub85c \ub9ac\uc624\ub12c \uba54\uc2dc, \uace4\uc0b4\ub85c \uc774\uacfc\uc778\uacfc \uacf5\ub3d9\uc120\ub450\ub97c \ub2ec\ub9ac\uace0 \uc788\ub2e4. 2014\ub144 FIFA \uc6d4\ub4dc\ucef5 \ub0a8\uc544\uba54\ub9ac\uce74 \uc9c0\uc5ed \uc608\uc120 \uc544\ub974\ud5e8\ud2f0\ub098\uc804\uc744 \uc55e\ub450\uace0 \uc218\uc544\ub808\uc2a4\ub294 \uc544\ub974\ud5e8\ud2f0\ub098\ub294 \uc218\ube44\uac00 \uc57d\ud55c \ud300\uc774\ub77c \uc5ed\uc2b5\uc744 \uc81c\ub300\ub85c \uc804\uac1c\ud558\uba74 \uc5bc\ub9c8\ub4e0\uc9c0 \uc774\uae38 \uc790\uc2e0\uc774 \uc788\ub2e4.\uace0 \uacf5\uc5b8\ud588\uc73c\ub098 \ud300\uc740 3-0\uc73c\ub85c \ud328\ubc30\ud588\ub2e4. \uacb0\uad6d 2014\ub144 FIFA \uc6d4\ub4dc\ucef5 \ub0a8\uc544\uba54\ub9ac\uce74 \uc9c0\uc5ed \uc608\uc120\uc5d0\uc11c\ub294 \ucd1d\ud569 11\uace8\uc744 \uae30\ub85d\ud558\uc5ec \uadf8\uc804\uc758 \ucd5c\ub2e4\ub4dd\uc810\uc790\uc778 \uc5d0\ub974\ub09c \ud06c\ub808\uc2a4\ud3ec\uc758 9\uace8\uc744 \uc81c\uce58\uace0 \ub0a8\uc544\uba54\ub9ac\uce74 \uc9c0\uc5ed \uc608\uc120 \uc5ed\uc0ac\uc0c1 \ucd5c\ub2e4\ub4dd\uc810\uc790\uac00 \ub418\uc5c8\ub2e4. \uc774 \uace8\uc758 \uacbd\uc7c1\uc790\uac00 \ub9ac\uc624\ub12c \uba54\uc2dc(10\uace8), \uace4\uc0b4\ub85c \uc774\uacfc\uc778(9\uace8)\uac19\uc740 \uc7c1\uc7c1\ud55c \uace8\uc7a1\uc774\ub4e4\uc774\ub77c\ub294 \uc810\uc5d0\uc11c \ub8e8\uc774\uc2a4 \uc54c\ubca0\ub974\ud1a0 \uc218\uc544\ub808\uc2a4\uc758 \uc774 \uae30\ub85d\uc740 \ub354\uc6b1 \ube5b\ub0ac\ub2e4.", "answer": "\uce60\ub808", "sentence": "\uc218\uc544\ub808\uc2a4\ub294 2014\ub144 FIFA \uc6d4\ub4dc\ucef5 \ub0a8\uc544\uba54\ub9ac\uce74 \uc9c0\uc5ed \uc608\uc120\uc5d0\uc11c 3\ub77c\uc6b4\ub4dc \uce60\ub808 \uc804\uc5d0\uc11c \ud574\ud2b8\ud2b8\ub9ad(4\uace8)\uc744 \uae30\ub85d\ud558\ub294 \ub4f1 \ub2e8 \uc138 \uacbd\uae30\ub9cc\uc5d0 5\uace8\uc744 \uae30\ub85d\ud55c \uc0c1\ud0dc\uc774\uba70 \ud604\uc7ac \uae30\uc874\uc758 \ub0a8\ubbf8 \uc9c0\uc5ed\uc608\uc120 \ucd5c\ub2e4\ub4dd\uc810\uc790\uc778 \uc544\ub974\ud5e8\ud2f0\ub098\uc758 \uc5d0\ub974\ub09c \ud06c\ub808\uc2a4\ud3ec\uac00 2002\ub144\uc5d0 \uae30\ub85d\ud55c \ub0a8\uc544\uba54\ub9ac\uce74 \uc9c0\uc5ed\uc608\uc120 \uc5ed\uc0ac\uc0c1 \ucd5c\ub2e4\ub4dd\uc810\uae30\ub85d\uc778 9\uace8\uc744 4\uace8 \ub0a8\uaca8\ub193\uc740 \uc0c1\ud0dc\uac00 \ub418\uc5c8\ub2e4.", "paragraph_sentence": " \uc218\uc544\ub808\uc2a4\ub294 2014\ub144 FIFA \uc6d4\ub4dc\ucef5 \ub0a8\uc544\uba54\ub9ac\uce74 \uc9c0\uc5ed \uc608\uc120\uc5d0\uc11c 3\ub77c\uc6b4\ub4dc \uce60\ub808 \uc804\uc5d0\uc11c \ud574\ud2b8\ud2b8\ub9ad(4\uace8)\uc744 \uae30\ub85d\ud558\ub294 \ub4f1 \ub2e8 \uc138 \uacbd\uae30\ub9cc\uc5d0 5\uace8\uc744 \uae30\ub85d\ud55c \uc0c1\ud0dc\uc774\uba70 \ud604\uc7ac \uae30\uc874\uc758 \ub0a8\ubbf8 \uc9c0\uc5ed\uc608\uc120 \ucd5c\ub2e4\ub4dd\uc810\uc790\uc778 \uc544\ub974\ud5e8\ud2f0\ub098\uc758 \uc5d0\ub974\ub09c \ud06c\ub808\uc2a4\ud3ec\uac00 2002\ub144\uc5d0 \uae30\ub85d\ud55c \ub0a8\uc544\uba54\ub9ac\uce74 \uc9c0\uc5ed\uc608\uc120 \uc5ed\uc0ac\uc0c1 \ucd5c\ub2e4\ub4dd\uc810\uae30\ub85d\uc778 9\uace8\uc744 4\uace8 \ub0a8\uaca8\ub193\uc740 \uc0c1\ud0dc\uac00 \ub418\uc5c8\ub2e4. \uadf8\ub7ec\ub098 \uadf8 \uc774\ud6c4 \ud65c\uc57d\uc774 \uc904\uc5b4\ub4e4\uba74\uc11c 9\ucc28\uc804\uc774 \uc885\ub8cc\ub41c \uc2dc\uc810\uc5d0\uc11c\ub294 6\uace8\ub85c \ub9ac\uc624\ub12c \uba54\uc2dc, \uace4\uc0b4\ub85c \uc774\uacfc\uc778\uacfc \uacf5\ub3d9\uc120\ub450\ub97c \ub2ec\ub9ac\uace0 \uc788\ub2e4. 2014\ub144 FIFA \uc6d4\ub4dc\ucef5 \ub0a8\uc544\uba54\ub9ac\uce74 \uc9c0\uc5ed \uc608\uc120 \uc544\ub974\ud5e8\ud2f0\ub098\uc804\uc744 \uc55e\ub450\uace0 \uc218\uc544\ub808\uc2a4\ub294 \uc544\ub974\ud5e8\ud2f0\ub098\ub294 \uc218\ube44\uac00 \uc57d\ud55c \ud300\uc774\ub77c \uc5ed\uc2b5\uc744 \uc81c\ub300\ub85c \uc804\uac1c\ud558\uba74 \uc5bc\ub9c8\ub4e0\uc9c0 \uc774\uae38 \uc790\uc2e0\uc774 \uc788\ub2e4. \uace0 \uacf5\uc5b8\ud588\uc73c\ub098 \ud300\uc740 3-0\uc73c\ub85c \ud328\ubc30\ud588\ub2e4. \uacb0\uad6d 2014\ub144 FIFA \uc6d4\ub4dc\ucef5 \ub0a8\uc544\uba54\ub9ac\uce74 \uc9c0\uc5ed \uc608\uc120\uc5d0\uc11c\ub294 \ucd1d\ud569 11\uace8\uc744 \uae30\ub85d\ud558\uc5ec \uadf8\uc804\uc758 \ucd5c\ub2e4\ub4dd\uc810\uc790\uc778 \uc5d0\ub974\ub09c \ud06c\ub808\uc2a4\ud3ec\uc758 9\uace8\uc744 \uc81c\uce58\uace0 \ub0a8\uc544\uba54\ub9ac\uce74 \uc9c0\uc5ed \uc608\uc120 \uc5ed\uc0ac\uc0c1 \ucd5c\ub2e4\ub4dd\uc810\uc790\uac00 \ub418\uc5c8\ub2e4. \uc774 \uace8\uc758 \uacbd\uc7c1\uc790\uac00 \ub9ac\uc624\ub12c \uba54\uc2dc(10\uace8), \uace4\uc0b4\ub85c \uc774\uacfc\uc778(9\uace8)\uac19\uc740 \uc7c1\uc7c1\ud55c \uace8\uc7a1\uc774\ub4e4\uc774\ub77c\ub294 \uc810\uc5d0\uc11c \ub8e8\uc774\uc2a4 \uc54c\ubca0\ub974\ud1a0 \uc218\uc544\ub808\uc2a4\uc758 \uc774 \uae30\ub85d\uc740 \ub354\uc6b1 \ube5b\ub0ac\ub2e4.", "paragraph_answer": "\uc218\uc544\ub808\uc2a4\ub294 2014\ub144 FIFA \uc6d4\ub4dc\ucef5 \ub0a8\uc544\uba54\ub9ac\uce74 \uc9c0\uc5ed \uc608\uc120\uc5d0\uc11c 3\ub77c\uc6b4\ub4dc \uce60\ub808 \uc804\uc5d0\uc11c \ud574\ud2b8\ud2b8\ub9ad(4\uace8)\uc744 \uae30\ub85d\ud558\ub294 \ub4f1 \ub2e8 \uc138 \uacbd\uae30\ub9cc\uc5d0 5\uace8\uc744 \uae30\ub85d\ud55c \uc0c1\ud0dc\uc774\uba70 \ud604\uc7ac \uae30\uc874\uc758 \ub0a8\ubbf8 \uc9c0\uc5ed\uc608\uc120 \ucd5c\ub2e4\ub4dd\uc810\uc790\uc778 \uc544\ub974\ud5e8\ud2f0\ub098\uc758 \uc5d0\ub974\ub09c \ud06c\ub808\uc2a4\ud3ec\uac00 2002\ub144\uc5d0 \uae30\ub85d\ud55c \ub0a8\uc544\uba54\ub9ac\uce74 \uc9c0\uc5ed\uc608\uc120 \uc5ed\uc0ac\uc0c1 \ucd5c\ub2e4\ub4dd\uc810\uae30\ub85d\uc778 9\uace8\uc744 4\uace8 \ub0a8\uaca8\ub193\uc740 \uc0c1\ud0dc\uac00 \ub418\uc5c8\ub2e4. \uadf8\ub7ec\ub098 \uadf8 \uc774\ud6c4 \ud65c\uc57d\uc774 \uc904\uc5b4\ub4e4\uba74\uc11c 9\ucc28\uc804\uc774 \uc885\ub8cc\ub41c \uc2dc\uc810\uc5d0\uc11c\ub294 6\uace8\ub85c \ub9ac\uc624\ub12c \uba54\uc2dc, \uace4\uc0b4\ub85c \uc774\uacfc\uc778\uacfc \uacf5\ub3d9\uc120\ub450\ub97c \ub2ec\ub9ac\uace0 \uc788\ub2e4. 2014\ub144 FIFA \uc6d4\ub4dc\ucef5 \ub0a8\uc544\uba54\ub9ac\uce74 \uc9c0\uc5ed \uc608\uc120 \uc544\ub974\ud5e8\ud2f0\ub098\uc804\uc744 \uc55e\ub450\uace0 \uc218\uc544\ub808\uc2a4\ub294 \uc544\ub974\ud5e8\ud2f0\ub098\ub294 \uc218\ube44\uac00 \uc57d\ud55c \ud300\uc774\ub77c \uc5ed\uc2b5\uc744 \uc81c\ub300\ub85c \uc804\uac1c\ud558\uba74 \uc5bc\ub9c8\ub4e0\uc9c0 \uc774\uae38 \uc790\uc2e0\uc774 \uc788\ub2e4.\uace0 \uacf5\uc5b8\ud588\uc73c\ub098 \ud300\uc740 3-0\uc73c\ub85c \ud328\ubc30\ud588\ub2e4. \uacb0\uad6d 2014\ub144 FIFA \uc6d4\ub4dc\ucef5 \ub0a8\uc544\uba54\ub9ac\uce74 \uc9c0\uc5ed \uc608\uc120\uc5d0\uc11c\ub294 \ucd1d\ud569 11\uace8\uc744 \uae30\ub85d\ud558\uc5ec \uadf8\uc804\uc758 \ucd5c\ub2e4\ub4dd\uc810\uc790\uc778 \uc5d0\ub974\ub09c \ud06c\ub808\uc2a4\ud3ec\uc758 9\uace8\uc744 \uc81c\uce58\uace0 \ub0a8\uc544\uba54\ub9ac\uce74 \uc9c0\uc5ed \uc608\uc120 \uc5ed\uc0ac\uc0c1 \ucd5c\ub2e4\ub4dd\uc810\uc790\uac00 \ub418\uc5c8\ub2e4. \uc774 \uace8\uc758 \uacbd\uc7c1\uc790\uac00 \ub9ac\uc624\ub12c \uba54\uc2dc(10\uace8), \uace4\uc0b4\ub85c \uc774\uacfc\uc778(9\uace8)\uac19\uc740 \uc7c1\uc7c1\ud55c \uace8\uc7a1\uc774\ub4e4\uc774\ub77c\ub294 \uc810\uc5d0\uc11c \ub8e8\uc774\uc2a4 \uc54c\ubca0\ub974\ud1a0 \uc218\uc544\ub808\uc2a4\uc758 \uc774 \uae30\ub85d\uc740 \ub354\uc6b1 \ube5b\ub0ac\ub2e4.", "sentence_answer": "\uc218\uc544\ub808\uc2a4\ub294 2014\ub144 FIFA \uc6d4\ub4dc\ucef5 \ub0a8\uc544\uba54\ub9ac\uce74 \uc9c0\uc5ed \uc608\uc120\uc5d0\uc11c 3\ub77c\uc6b4\ub4dc \uce60\ub808 \uc804\uc5d0\uc11c \ud574\ud2b8\ud2b8\ub9ad(4\uace8)\uc744 \uae30\ub85d\ud558\ub294 \ub4f1 \ub2e8 \uc138 \uacbd\uae30\ub9cc\uc5d0 5\uace8\uc744 \uae30\ub85d\ud55c \uc0c1\ud0dc\uc774\uba70 \ud604\uc7ac \uae30\uc874\uc758 \ub0a8\ubbf8 \uc9c0\uc5ed\uc608\uc120 \ucd5c\ub2e4\ub4dd\uc810\uc790\uc778 \uc544\ub974\ud5e8\ud2f0\ub098\uc758 \uc5d0\ub974\ub09c \ud06c\ub808\uc2a4\ud3ec\uac00 2002\ub144\uc5d0 \uae30\ub85d\ud55c \ub0a8\uc544\uba54\ub9ac\uce74 \uc9c0\uc5ed\uc608\uc120 \uc5ed\uc0ac\uc0c1 \ucd5c\ub2e4\ub4dd\uc810\uae30\ub85d\uc778 9\uace8\uc744 4\uace8 \ub0a8\uaca8\ub193\uc740 \uc0c1\ud0dc\uac00 \ub418\uc5c8\ub2e4.", "paragraph_id": "6497528-1-0", "paragraph_question": "question: \uc218\uc544\ub808\uc2a4\uac00 2014\ub144 \uc6d4\ub4dc\ucef5 \ub0a8\ubbf8 \uc608\uc120\uc5d0\uc11c 4\uace8 \ud574\ud2b8\ud2b8\ub9ad\uc744 \uae30\ub85d\ud588\ub358 \uacbd\uae30\ub294 \uc5b4\ub5a4 \ub098\ub77c\ub97c \uc0c1\ub300\ub85c \ud55c \uacbd\uae30\uc600\ub294\uac00?, context: \uc218\uc544\ub808\uc2a4\ub294 2014\ub144 FIFA \uc6d4\ub4dc\ucef5 \ub0a8\uc544\uba54\ub9ac\uce74 \uc9c0\uc5ed \uc608\uc120\uc5d0\uc11c 3\ub77c\uc6b4\ub4dc \uce60\ub808\uc804\uc5d0\uc11c \ud574\ud2b8\ud2b8\ub9ad(4\uace8)\uc744 \uae30\ub85d\ud558\ub294 \ub4f1 \ub2e8 \uc138 \uacbd\uae30\ub9cc\uc5d0 5\uace8\uc744 \uae30\ub85d\ud55c \uc0c1\ud0dc\uc774\uba70 \ud604\uc7ac \uae30\uc874\uc758 \ub0a8\ubbf8 \uc9c0\uc5ed\uc608\uc120 \ucd5c\ub2e4\ub4dd\uc810\uc790\uc778 \uc544\ub974\ud5e8\ud2f0\ub098\uc758 \uc5d0\ub974\ub09c \ud06c\ub808\uc2a4\ud3ec\uac00 2002\ub144\uc5d0 \uae30\ub85d\ud55c \ub0a8\uc544\uba54\ub9ac\uce74 \uc9c0\uc5ed\uc608\uc120 \uc5ed\uc0ac\uc0c1 \ucd5c\ub2e4\ub4dd\uc810\uae30\ub85d\uc778 9\uace8\uc744 4\uace8 \ub0a8\uaca8\ub193\uc740 \uc0c1\ud0dc\uac00 \ub418\uc5c8\ub2e4. \uadf8\ub7ec\ub098 \uadf8 \uc774\ud6c4 \ud65c\uc57d\uc774 \uc904\uc5b4\ub4e4\uba74\uc11c 9\ucc28\uc804\uc774 \uc885\ub8cc\ub41c \uc2dc\uc810\uc5d0\uc11c\ub294 6\uace8\ub85c \ub9ac\uc624\ub12c \uba54\uc2dc, \uace4\uc0b4\ub85c \uc774\uacfc\uc778\uacfc \uacf5\ub3d9\uc120\ub450\ub97c \ub2ec\ub9ac\uace0 \uc788\ub2e4. 2014\ub144 FIFA \uc6d4\ub4dc\ucef5 \ub0a8\uc544\uba54\ub9ac\uce74 \uc9c0\uc5ed \uc608\uc120 \uc544\ub974\ud5e8\ud2f0\ub098\uc804\uc744 \uc55e\ub450\uace0 \uc218\uc544\ub808\uc2a4\ub294 \uc544\ub974\ud5e8\ud2f0\ub098\ub294 \uc218\ube44\uac00 \uc57d\ud55c \ud300\uc774\ub77c \uc5ed\uc2b5\uc744 \uc81c\ub300\ub85c \uc804\uac1c\ud558\uba74 \uc5bc\ub9c8\ub4e0\uc9c0 \uc774\uae38 \uc790\uc2e0\uc774 \uc788\ub2e4.\uace0 \uacf5\uc5b8\ud588\uc73c\ub098 \ud300\uc740 3-0\uc73c\ub85c \ud328\ubc30\ud588\ub2e4. \uacb0\uad6d 2014\ub144 FIFA \uc6d4\ub4dc\ucef5 \ub0a8\uc544\uba54\ub9ac\uce74 \uc9c0\uc5ed \uc608\uc120\uc5d0\uc11c\ub294 \ucd1d\ud569 11\uace8\uc744 \uae30\ub85d\ud558\uc5ec \uadf8\uc804\uc758 \ucd5c\ub2e4\ub4dd\uc810\uc790\uc778 \uc5d0\ub974\ub09c \ud06c\ub808\uc2a4\ud3ec\uc758 9\uace8\uc744 \uc81c\uce58\uace0 \ub0a8\uc544\uba54\ub9ac\uce74 \uc9c0\uc5ed \uc608\uc120 \uc5ed\uc0ac\uc0c1 \ucd5c\ub2e4\ub4dd\uc810\uc790\uac00 \ub418\uc5c8\ub2e4. \uc774 \uace8\uc758 \uacbd\uc7c1\uc790\uac00 \ub9ac\uc624\ub12c \uba54\uc2dc(10\uace8), \uace4\uc0b4\ub85c \uc774\uacfc\uc778(9\uace8)\uac19\uc740 \uc7c1\uc7c1\ud55c \uace8\uc7a1\uc774\ub4e4\uc774\ub77c\ub294 \uc810\uc5d0\uc11c \ub8e8\uc774\uc2a4 \uc54c\ubca0\ub974\ud1a0 \uc218\uc544\ub808\uc2a4\uc758 \uc774 \uae30\ub85d\uc740 \ub354\uc6b1 \ube5b\ub0ac\ub2e4."} {"question": "\ud64d\uac00\uad8c\uc758 \uac00\uc7a5 \ud070 \ud2b9\uc9d5\uc740?", "paragraph": "\ud64d\uac00\uad8c\uc758 \ub300\ud45c\uc801\uc778 \ud2b9\uc9d5\uc740 \uc77c\uc9c0\uc218(\u4e00\u6307\u624b)\uc778\ub370, \uc774\uac83\uc740 \ud64d\uac00\uad8c\uc774 \uc18c\ub9bc \uad8c\ubc95\uc73c\ub85c\ubd80\ud130 \ube44\ub86f\ub418\uc5c8\uae30\uc5d0 \uc18c\ub9bc \uad8c\ubc95\uacfc\uc758 \ucc28\ubcc4\uc744 \uaf80\ud558\uae30 \uc704\ud574 \ucc3d\uc548\ud574\ub0b8 \ud64d\uac00\uad8c\ub9cc\uc758 \uace0\uc720 \uc218\ud615(\u624b\u5f62)\uc774\ub2e4. \ud64d\uac00\uad8c\uc758 \ubb34\uc220 \ud48d\uaca9\uc744 \ubcf4\uba74 \uac15(\u525b)\uc744 \uc704\uc8fc\ub85c \ud558\uace0 \uc788\uc73c\uba70 \uc7a5\uad50\ub300\ub9c8(\u9577\u6a4b\u5927\u99ac), \uc7a5\uc218\uc7a5\uacf5(\u9577\u624b\u9577\u653b), \ub300\uac1c\ub300\ud569(\u5927\u958b\u5927\u5408), \ub2e8\uad50\ud611\ub9c8(\u77ed\u6a4b\u72f9\u99ac), \ub2e8\uad50\uacf5\uaca9(\u77ed\u6a4b\u653b\u64ca), \uc810\uc2e0\uacf5\ubc29(\u8cbc\u8eab\u653b\u9632) \ub4f1\uc744 \ud2b9\uc9d5\uc73c\ub85c \ud558\uace0 \uc788\ub2e4. \ud64d\uac00\uad8c\uc744 \ucc98\uc74c \uc5f0\ub9c8\ud560 \ub54c\ub294 \ub3d9\uad50\ucca0\ub9c8(\u9285\u6a4b\u9435\u99ac)\ub97c \ub2e8\ub828\ud574\uc57c \ud558\uba70 \uae30\ubcf8 \uc790\uc138\uc778 \ucc38\uc7a5\uc744 \ub9ce\uc774 \uc5f0\uc2b5\ud574\uc57c \ud55c\ub2e4. \uba3c\uc800 \ud558\uccb4(\u9435\u99ac)\ub97c \ud2bc\ud2bc\ud558\uac8c \ub2e8\ub828\uc2dc\ucf1c\uc57c \ud558\uba70, \uc591\ud314(\u9285\u6a4b)\uc744 \ud798 \uc788\uac8c \ub2e8\ub828\ud574\uc57c \ud55c\ub2e4. \uae30\ub97c \ub2e8\uc804\uae4c\uc9c0 \ub04c\uc5b4\ub0b4\ub9b4 \uc218 \uc788\uc744 \uc815\ub3c4\uae4c\uc9c0 \ud558\uace0(\u6c23\u606f\u4e0b\u6c88) \uadf8 \ub2e4\uc74c\uc5d0 \uc218\uc758 \ubcc0\ud654\uc640 \ubbfc\ucca9\uc131(\u8b8a\u5316\u9748\u6d3b)\uc744 \uc5f0\ub9c8\ud558\uace0, \ub2e4\uc74c\uc5d0 \uc0c1\uccb4\uc640 \ud558\uccb4\ub97c \ubc30\ud569(\u914d\u5408)\uc2dc\ud0a8\ub2e4. \uc2e0(\u8eab), \uc218(\u624b), \ubcf4(\u6b65), \ubc95(\u6cd5), \uc694(\u8170), \ub9c8(\u99ac), \uc548(\u773c), \uc815(\u7cbe), \uae30(\u6c23), \uc2e0(\u795e)\uc774 \ud558\ub098\ub85c \ud569\uccb4(\u5408\u9ad4)\ud560 \uc218 \uc788\uc5b4\uc57c \ud55c\ub2e4. \ud56d\uc0c1 \ubab8\uacfc \uc790\uc138\uac00 \uac19\uc774 \uc6c0\uc9c1\uc5ec\uc57c \ud55c\ub2e4. \uc774\uac83\uc744 \uc2e0\ud615 \ub300\ub3d9(\u8eab\u5f62\u5e36\u52d5)\uc774\ub77c \ud55c\ub2e4.", "answer": "\uc77c\uc9c0\uc218(\u4e00\u6307\u624b)", "sentence": "\ud64d\uac00\uad8c\uc758 \ub300\ud45c\uc801\uc778 \ud2b9\uc9d5\uc740 \uc77c\uc9c0\uc218(\u4e00\u6307\u624b) \uc778\ub370, \uc774\uac83\uc740 \ud64d\uac00\uad8c\uc774 \uc18c\ub9bc \uad8c\ubc95\uc73c\ub85c\ubd80\ud130 \ube44\ub86f\ub418\uc5c8\uae30\uc5d0 \uc18c\ub9bc \uad8c\ubc95\uacfc\uc758 \ucc28\ubcc4\uc744 \uaf80\ud558\uae30 \uc704\ud574 \ucc3d\uc548\ud574\ub0b8 \ud64d\uac00\uad8c\ub9cc\uc758 \uace0\uc720 \uc218\ud615(\u624b\u5f62)\uc774\ub2e4.", "paragraph_sentence": " \ud64d\uac00\uad8c\uc758 \ub300\ud45c\uc801\uc778 \ud2b9\uc9d5\uc740 \uc77c\uc9c0\uc218(\u4e00\u6307\u624b) \uc778\ub370, \uc774\uac83\uc740 \ud64d\uac00\uad8c\uc774 \uc18c\ub9bc \uad8c\ubc95\uc73c\ub85c\ubd80\ud130 \ube44\ub86f\ub418\uc5c8\uae30\uc5d0 \uc18c\ub9bc \uad8c\ubc95\uacfc\uc758 \ucc28\ubcc4\uc744 \uaf80\ud558\uae30 \uc704\ud574 \ucc3d\uc548\ud574\ub0b8 \ud64d\uac00\uad8c\ub9cc\uc758 \uace0\uc720 \uc218\ud615(\u624b\u5f62)\uc774\ub2e4. \ud64d\uac00\uad8c\uc758 \ubb34\uc220 \ud48d\uaca9\uc744 \ubcf4\uba74 \uac15(\u525b)\uc744 \uc704\uc8fc\ub85c \ud558\uace0 \uc788\uc73c\uba70 \uc7a5\uad50\ub300\ub9c8(\u9577\u6a4b\u5927\u99ac), \uc7a5\uc218\uc7a5\uacf5(\u9577\u624b\u9577\u653b), \ub300\uac1c\ub300\ud569(\u5927\u958b\u5927\u5408), \ub2e8\uad50\ud611\ub9c8(\u77ed\u6a4b\u72f9\u99ac), \ub2e8\uad50\uacf5\uaca9(\u77ed\u6a4b\u653b\u64ca), \uc810\uc2e0\uacf5\ubc29(\u8cbc\u8eab\u653b\u9632) \ub4f1\uc744 \ud2b9\uc9d5\uc73c\ub85c \ud558\uace0 \uc788\ub2e4. \ud64d\uac00\uad8c\uc744 \ucc98\uc74c \uc5f0\ub9c8\ud560 \ub54c\ub294 \ub3d9\uad50\ucca0\ub9c8(\u9285\u6a4b\u9435\u99ac)\ub97c \ub2e8\ub828\ud574\uc57c \ud558\uba70 \uae30\ubcf8 \uc790\uc138\uc778 \ucc38\uc7a5\uc744 \ub9ce\uc774 \uc5f0\uc2b5\ud574\uc57c \ud55c\ub2e4. \uba3c\uc800 \ud558\uccb4(\u9435\u99ac)\ub97c \ud2bc\ud2bc\ud558\uac8c \ub2e8\ub828\uc2dc\ucf1c\uc57c \ud558\uba70, \uc591\ud314(\u9285\u6a4b)\uc744 \ud798 \uc788\uac8c \ub2e8\ub828\ud574\uc57c \ud55c\ub2e4. \uae30\ub97c \ub2e8\uc804\uae4c\uc9c0 \ub04c\uc5b4\ub0b4\ub9b4 \uc218 \uc788\uc744 \uc815\ub3c4\uae4c\uc9c0 \ud558\uace0(\u6c23\u606f\u4e0b\u6c88) \uadf8 \ub2e4\uc74c\uc5d0 \uc218\uc758 \ubcc0\ud654\uc640 \ubbfc\ucca9\uc131(\u8b8a\u5316\u9748\u6d3b)\uc744 \uc5f0\ub9c8\ud558\uace0, \ub2e4\uc74c\uc5d0 \uc0c1\uccb4\uc640 \ud558\uccb4\ub97c \ubc30\ud569(\u914d\u5408)\uc2dc\ud0a8\ub2e4. \uc2e0(\u8eab), \uc218(\u624b), \ubcf4(\u6b65), \ubc95(\u6cd5), \uc694(\u8170), \ub9c8(\u99ac), \uc548(\u773c), \uc815(\u7cbe), \uae30(\u6c23), \uc2e0(\u795e)\uc774 \ud558\ub098\ub85c \ud569\uccb4(\u5408\u9ad4)\ud560 \uc218 \uc788\uc5b4\uc57c \ud55c\ub2e4. \ud56d\uc0c1 \ubab8\uacfc \uc790\uc138\uac00 \uac19\uc774 \uc6c0\uc9c1\uc5ec\uc57c \ud55c\ub2e4. \uc774\uac83\uc744 \uc2e0\ud615 \ub300\ub3d9(\u8eab\u5f62\u5e36\u52d5)\uc774\ub77c \ud55c\ub2e4.", "paragraph_answer": "\ud64d\uac00\uad8c\uc758 \ub300\ud45c\uc801\uc778 \ud2b9\uc9d5\uc740 \uc77c\uc9c0\uc218(\u4e00\u6307\u624b) \uc778\ub370, \uc774\uac83\uc740 \ud64d\uac00\uad8c\uc774 \uc18c\ub9bc \uad8c\ubc95\uc73c\ub85c\ubd80\ud130 \ube44\ub86f\ub418\uc5c8\uae30\uc5d0 \uc18c\ub9bc \uad8c\ubc95\uacfc\uc758 \ucc28\ubcc4\uc744 \uaf80\ud558\uae30 \uc704\ud574 \ucc3d\uc548\ud574\ub0b8 \ud64d\uac00\uad8c\ub9cc\uc758 \uace0\uc720 \uc218\ud615(\u624b\u5f62)\uc774\ub2e4. \ud64d\uac00\uad8c\uc758 \ubb34\uc220 \ud48d\uaca9\uc744 \ubcf4\uba74 \uac15(\u525b)\uc744 \uc704\uc8fc\ub85c \ud558\uace0 \uc788\uc73c\uba70 \uc7a5\uad50\ub300\ub9c8(\u9577\u6a4b\u5927\u99ac), \uc7a5\uc218\uc7a5\uacf5(\u9577\u624b\u9577\u653b), \ub300\uac1c\ub300\ud569(\u5927\u958b\u5927\u5408), \ub2e8\uad50\ud611\ub9c8(\u77ed\u6a4b\u72f9\u99ac), \ub2e8\uad50\uacf5\uaca9(\u77ed\u6a4b\u653b\u64ca), \uc810\uc2e0\uacf5\ubc29(\u8cbc\u8eab\u653b\u9632) \ub4f1\uc744 \ud2b9\uc9d5\uc73c\ub85c \ud558\uace0 \uc788\ub2e4. \ud64d\uac00\uad8c\uc744 \ucc98\uc74c \uc5f0\ub9c8\ud560 \ub54c\ub294 \ub3d9\uad50\ucca0\ub9c8(\u9285\u6a4b\u9435\u99ac)\ub97c \ub2e8\ub828\ud574\uc57c \ud558\uba70 \uae30\ubcf8 \uc790\uc138\uc778 \ucc38\uc7a5\uc744 \ub9ce\uc774 \uc5f0\uc2b5\ud574\uc57c \ud55c\ub2e4. \uba3c\uc800 \ud558\uccb4(\u9435\u99ac)\ub97c \ud2bc\ud2bc\ud558\uac8c \ub2e8\ub828\uc2dc\ucf1c\uc57c \ud558\uba70, \uc591\ud314(\u9285\u6a4b)\uc744 \ud798 \uc788\uac8c \ub2e8\ub828\ud574\uc57c \ud55c\ub2e4. \uae30\ub97c \ub2e8\uc804\uae4c\uc9c0 \ub04c\uc5b4\ub0b4\ub9b4 \uc218 \uc788\uc744 \uc815\ub3c4\uae4c\uc9c0 \ud558\uace0(\u6c23\u606f\u4e0b\u6c88) \uadf8 \ub2e4\uc74c\uc5d0 \uc218\uc758 \ubcc0\ud654\uc640 \ubbfc\ucca9\uc131(\u8b8a\u5316\u9748\u6d3b)\uc744 \uc5f0\ub9c8\ud558\uace0, \ub2e4\uc74c\uc5d0 \uc0c1\uccb4\uc640 \ud558\uccb4\ub97c \ubc30\ud569(\u914d\u5408)\uc2dc\ud0a8\ub2e4. \uc2e0(\u8eab), \uc218(\u624b), \ubcf4(\u6b65), \ubc95(\u6cd5), \uc694(\u8170), \ub9c8(\u99ac), \uc548(\u773c), \uc815(\u7cbe), \uae30(\u6c23), \uc2e0(\u795e)\uc774 \ud558\ub098\ub85c \ud569\uccb4(\u5408\u9ad4)\ud560 \uc218 \uc788\uc5b4\uc57c \ud55c\ub2e4. \ud56d\uc0c1 \ubab8\uacfc \uc790\uc138\uac00 \uac19\uc774 \uc6c0\uc9c1\uc5ec\uc57c \ud55c\ub2e4. \uc774\uac83\uc744 \uc2e0\ud615 \ub300\ub3d9(\u8eab\u5f62\u5e36\u52d5)\uc774\ub77c \ud55c\ub2e4.", "sentence_answer": "\ud64d\uac00\uad8c\uc758 \ub300\ud45c\uc801\uc778 \ud2b9\uc9d5\uc740 \uc77c\uc9c0\uc218(\u4e00\u6307\u624b) \uc778\ub370, \uc774\uac83\uc740 \ud64d\uac00\uad8c\uc774 \uc18c\ub9bc \uad8c\ubc95\uc73c\ub85c\ubd80\ud130 \ube44\ub86f\ub418\uc5c8\uae30\uc5d0 \uc18c\ub9bc \uad8c\ubc95\uacfc\uc758 \ucc28\ubcc4\uc744 \uaf80\ud558\uae30 \uc704\ud574 \ucc3d\uc548\ud574\ub0b8 \ud64d\uac00\uad8c\ub9cc\uc758 \uace0\uc720 \uc218\ud615(\u624b\u5f62)\uc774\ub2e4.", "paragraph_id": "5948035-2-0", "paragraph_question": "question: \ud64d\uac00\uad8c\uc758 \uac00\uc7a5 \ud070 \ud2b9\uc9d5\uc740?, context: \ud64d\uac00\uad8c\uc758 \ub300\ud45c\uc801\uc778 \ud2b9\uc9d5\uc740 \uc77c\uc9c0\uc218(\u4e00\u6307\u624b)\uc778\ub370, \uc774\uac83\uc740 \ud64d\uac00\uad8c\uc774 \uc18c\ub9bc \uad8c\ubc95\uc73c\ub85c\ubd80\ud130 \ube44\ub86f\ub418\uc5c8\uae30\uc5d0 \uc18c\ub9bc \uad8c\ubc95\uacfc\uc758 \ucc28\ubcc4\uc744 \uaf80\ud558\uae30 \uc704\ud574 \ucc3d\uc548\ud574\ub0b8 \ud64d\uac00\uad8c\ub9cc\uc758 \uace0\uc720 \uc218\ud615(\u624b\u5f62)\uc774\ub2e4. \ud64d\uac00\uad8c\uc758 \ubb34\uc220 \ud48d\uaca9\uc744 \ubcf4\uba74 \uac15(\u525b)\uc744 \uc704\uc8fc\ub85c \ud558\uace0 \uc788\uc73c\uba70 \uc7a5\uad50\ub300\ub9c8(\u9577\u6a4b\u5927\u99ac), \uc7a5\uc218\uc7a5\uacf5(\u9577\u624b\u9577\u653b), \ub300\uac1c\ub300\ud569(\u5927\u958b\u5927\u5408), \ub2e8\uad50\ud611\ub9c8(\u77ed\u6a4b\u72f9\u99ac), \ub2e8\uad50\uacf5\uaca9(\u77ed\u6a4b\u653b\u64ca), \uc810\uc2e0\uacf5\ubc29(\u8cbc\u8eab\u653b\u9632) \ub4f1\uc744 \ud2b9\uc9d5\uc73c\ub85c \ud558\uace0 \uc788\ub2e4. \ud64d\uac00\uad8c\uc744 \ucc98\uc74c \uc5f0\ub9c8\ud560 \ub54c\ub294 \ub3d9\uad50\ucca0\ub9c8(\u9285\u6a4b\u9435\u99ac)\ub97c \ub2e8\ub828\ud574\uc57c \ud558\uba70 \uae30\ubcf8 \uc790\uc138\uc778 \ucc38\uc7a5\uc744 \ub9ce\uc774 \uc5f0\uc2b5\ud574\uc57c \ud55c\ub2e4. \uba3c\uc800 \ud558\uccb4(\u9435\u99ac)\ub97c \ud2bc\ud2bc\ud558\uac8c \ub2e8\ub828\uc2dc\ucf1c\uc57c \ud558\uba70, \uc591\ud314(\u9285\u6a4b)\uc744 \ud798 \uc788\uac8c \ub2e8\ub828\ud574\uc57c \ud55c\ub2e4. \uae30\ub97c \ub2e8\uc804\uae4c\uc9c0 \ub04c\uc5b4\ub0b4\ub9b4 \uc218 \uc788\uc744 \uc815\ub3c4\uae4c\uc9c0 \ud558\uace0(\u6c23\u606f\u4e0b\u6c88) \uadf8 \ub2e4\uc74c\uc5d0 \uc218\uc758 \ubcc0\ud654\uc640 \ubbfc\ucca9\uc131(\u8b8a\u5316\u9748\u6d3b)\uc744 \uc5f0\ub9c8\ud558\uace0, \ub2e4\uc74c\uc5d0 \uc0c1\uccb4\uc640 \ud558\uccb4\ub97c \ubc30\ud569(\u914d\u5408)\uc2dc\ud0a8\ub2e4. \uc2e0(\u8eab), \uc218(\u624b), \ubcf4(\u6b65), \ubc95(\u6cd5), \uc694(\u8170), \ub9c8(\u99ac), \uc548(\u773c), \uc815(\u7cbe), \uae30(\u6c23), \uc2e0(\u795e)\uc774 \ud558\ub098\ub85c \ud569\uccb4(\u5408\u9ad4)\ud560 \uc218 \uc788\uc5b4\uc57c \ud55c\ub2e4. \ud56d\uc0c1 \ubab8\uacfc \uc790\uc138\uac00 \uac19\uc774 \uc6c0\uc9c1\uc5ec\uc57c \ud55c\ub2e4. \uc774\uac83\uc744 \uc2e0\ud615 \ub300\ub3d9(\u8eab\u5f62\u5e36\u52d5)\uc774\ub77c \ud55c\ub2e4."} {"question": "\uc138\ubb34\uc870\uc0ac \ub2f9\uc2dc \ub178\ubb34\ud604\uc744 \ubcc0\ud638\ud55c \uc0ac\ub78c\uc740 \uc804\ud574\ucca0\uacfc \ub204\uad6c\uc778\uac00?", "paragraph": "2009\ub144 4\uc6d4 30\uc77c \uc624\uc804 8\uc2dc \ub178\ubb34\ud604\uc740 \uae40\ud574 \ubd09\ud558\ub9c8\uc744 \uc0ac\uc800\ub97c \ucd9c\ubc1c, \uc624\ud6c4 1\uc2dc 20\ubd84 \uacbd \ub300\uac80\ucc30\uccad\uc5d0 \ud53c\uc758\uc790 \uc2e0\ubd84\uc73c\ub85c \ucd9c\uc11d\ud574 \ub300\uac80\ucc30\uccad \uc911\uc559\uc218\uc0ac\ubd80 1120\ud638 \ud2b9\ubcc4\uc870\uc0ac\uc2e4\uc5d0\uc11c \uc624\ud6c4 11\uc2dc 20\ubd84 \uacbd\uae4c\uc9c0 \uc870\uc0ac\ub97c \ubc1b\uc558\ub2e4. \ub178\ubb34\ud604\uc740 \uc9c4\uc220\uc870\uc11c\ub97c \uac80\ud1a0\ud55c \ub4a4 \uc775\uc77c \uc624\uc804 2\uc2dc 10\ubd84\ucbe4 \uccad\uc0ac\ub97c \ub098\uc640 \uadc0\uac00\ud588\ub2e4. \ub178\ubb34\ud604\uc740 \ubc15\uc5f0\ucc28\ub85c\ubd80\ud130 2007\ub144 6\uc6d4 \ub9d0 \uc815\uc0c1\ubb38 \uc804 \ucd1d\ubb34\ube44\uc11c\uad00\uc744 \ud1b5\ud574 100\ub9cc \ub2ec\ub7ec\ub97c \ubc1b\uace0, 2008\ub144 2\uc6d4 \ub9d0 \uc870\uce74\uc0ac\uc704 \uc5f0\ucca0\ud638\ub97c \ud1b5\ud574 500\ub9cc \ub2ec\ub7ec\ub97c \ubc1b\uc740 \ud610\uc758\ub97c \ubc1b\uc558\ub2e4. \ub610\ud55c \uac80\ucc30\uc740 \uc815\uc0c1\ubb38 \uc804 \ucd1d\ubb34\ube44\uc11c\uad00\uc774 \ud6a1\ub839\ud55c \ub300\ud1b5\ub839 \ud2b9\uc218\ud65c\ub3d9\ube44 12\uc5b5 5\ucc9c\ub9cc\uc6d0\uc5d0 \ub300\ud574 \ubcf4\uace0\ub97c \ubc1b\uc558\ub294\uc9c0, \ubc15\uc5f0\ucc28\uc758 \uba85\ud488 \uc2dc\uacc4 \uc120\ubb3c\uacfc 15\uc5b5\uc6d0\uc758 \ucc28\uc6a9\uc99d\uc744 \uc4f0\uac8c \ub41c \uacbd\uc704\uc5d0 \ub300\ud574\uc11c\ub3c4 \uc870\uc0ac\ud588\ub2e4. \ub178\ubb34\ud604\uc758 \uc870\uc0ac\ub294 \uc8fc\uc784\uac80\uc0ac\uc778 \uc6b0\ubcd1\uc6b0 \uc911\uc2181\uacfc\uc7a5\uc774 \uc2e0\ubb38\uc744 \ud558\uc600\uace0, \uc9c1\ubb34\uad00\ub828\uc131 \ubd80\ubd84\uc740 \uae40\ud615\uc6b1 \uac80\uc0ac\uac00, 100\ub9cc\ub2ec\ub7ec \uc218\uc218 \ud610\uc758\uc5d0 \ub300\ud574\uc11c\ub294 \uc774\uc8fc\ud615 \uac80\uc0ac\uac00 \uc870\uc0ac\ud588\ub2e4. \ub610\ud55c 500\ub9cc\ub2ec\ub7ec \uc218\uc218 \ud610\uc758\uc640 \ud2b9\uc218\ud65c\ub3d9\ube44 12\uc5b5 5000\ub9cc\uc6d0 \uc218\uc218 \ud610\uc758\uc5d0 \ub300\ud574\uc11c\ub294 \uc774\uc120\ubd09 \uac80\uc0ac\uac00 \uc870\uc0ac\ud588\uace0, \uc774\uc778\uaddc \uc911\uc218\ubd80\uc7a5\uacfc \ud64d\ub9cc\ud45c \uc218\uc0ac\uae30\ud68d\uad00\uc740 \uc870\uc0ac \uc0c1\ud669\uc744 \ubaa8\ub2c8\ud130\ub85c \uc9c0\ucf1c\ubcf4\uba70 \uc2e4\uc2dc\uac04\uc73c\ub85c \uc9c0\ud718\ud588\ub2e4. \uc784\ucc44\uc9c4 \uac80\ucc30\ucd1d\uc7a5\uc740 \uc790\uc815\uae4c\uc9c0 \uccad\uc0ac\uc5d0 \ub0a8\uc544 \uc218\uc0ac \ub0b4\uc6a9\uc744 \ubcf4\uace0 \ubc1b\uc558\uc73c\uba70, \ub178\ubb34\ud604\uc758 \ubcc0\ud638\uc778 \uce21\uc740 \ubb38\uc7ac\uc778\uacfc \uc804\ud574\ucca0 \ubcc0\ud638\uc0ac\uac00 \ubc88\uac08\uc544 \uac00\uba70 \ubcc0\ud638\ud588\ub2e4. \uc218\uc0ac\ud300\uc740 \uc624\ud6c4 1\uc2dc 40\ubd84 \uacbd\ubd80\ud130 4\uc2dc 10\ubd84\uae4c\uc9c0 \ub300\ud1b5\ub839\uc758 \uc9c1\ubb34\uc640 \uad8c\ud55c, \ubc15\uc5f0\ucc28\uc640\uc758 \uad00\uacc4\uc5d0 \ub300\ud574 \uc870\uc0ac\ud558\uc600\uc73c\uba70, 10\ubd84\uac04 \ud734\uc2dd\ud55c \ub4a4 \ubc15\uc5f0\ucc28\ub85c\ubd80\ud130 100\ub9cc\ub2ec\ub7ec\ub97c \uc218\uc218\ud55c \ud610\uc758\uc5d0 \ub300\ud574 \uc624\ud6c4 6\uc2dc 30\ubd84\uae4c\uc9c0 \uc870\uc0ac\ub97c \uc9c4\ud589\ud588\ub2e4. \uc800\ub141 \uc2dd\uc0ac \ud6c4 \uc624\ud6c4 7\uc2dc35\ubd84\ubd80\ud130 \ubc15\uc5f0\ucc28\ub85c \ubd80\ud130 500\ub9cc \ub2ec\ub7ec\ub97c \uc218\uc218\ud55c \ud610\uc758, \uc815\uc0c1\ubb38 \uc804 \ub300\ud1b5\ub839 \ucd1d\ubb34\ube44\uc11c\uad00\uc774 \ud6a1\ub839\ud55c \ub300\ud1b5\ub839 \ud2b9\uc218\ud65c\ub3d9\ube44 12\uc5b55\ucc9c\ub9cc\uc6d0, \ubc15\uc5f0\ucc28\uac00 \ub178\ubb34\ud604 \uce21\uc5d0\uac8c \uc120\ubb3c\ud55c \uba85\ud488\uc2dc\uacc4 \ub4f1\uc5d0 \ub300\ud574 \uc870\uc0ac\ud588\ub2e4. \uac80\ucc30\uc740 \ubc15\uc5f0\ucc28\uc758 \ubca0\ud2b8\ub0a8 \ud654\ub825\ubc1c\uc804\uc18c \uc0ac\uc5c5 \ub4f1\uc5d0 \ub3c4\uc6c0\uc744 \uc900 \ub300\uac00\ub85c 600\ub9cc \ub2ec\ub7ec\ub97c \ubc1b\uc740 \uac8c \uc544\ub2c8\ub0d0\uace0 \uc2e0\ubb38\ud588\uc73c\ub098, \ub178\ubb34\ud604\uc740 \ub300\ud1b5\ub839\uc774 \ud55c\uad6d \uae30\uc5c5\uc758 \ud574\uc678 \uc0ac\uc5c5\uc5d0 \ub3c4\uc6c0\uc744 \uc900 \uac8c \ubb38\uc81c\uac00 \ub420 \uc218 \uc5c6\ub2e4\uace0 \uc8fc\uc7a5\ud588\ub2e4. \ub178\ubb34\ud604\uc740 2007\ub144 \ubc15\uc5f0\ucc28\ub85c\ubd80\ud130 \ubc1b\uc740 100\ub9cc \ub2ec\ub7ec\uc5d0 \ub300\ud574 \ucc44\ubb34\ub97c \uac1a\ub294\ub370 \uc790\uc2e0\uc758 \ubc30\uc6b0\uc790 \uad8c\uc591\uc219\uc774 \uc4f4 \uac83\uc774\uba70, \ub300\ud1b5\ub839 \uc7ac\uc784 \uc911 \uc774\uac19\uc740 \uc0ac\uc2e4\uc744 \ubab0\ub790\ub2e4\uace0 \uc9c4\uc220\ud588\ub2e4. 100\ub9cc \ub2ec\ub7ec\uc758 \uad6c\uccb4\uc801\uc778 \uc0ac\uc6a9\ucc98\uc5d0 \ub300\ud574\uc11c\ub294 \uc11c\uba74 \ub2f5\ubcc0\uacfc \ub9c8\ucc2c\uac00\uc9c0\ub85c \ubc1d\ud790 \uc218 \uc5c6\ub2e4\uace0 \uc9c4\uc220\ud588\ub2e4. 2008\ub144 500\ub9cc \ub2ec\ub7ec\ub97c \ubc1b\uc740 \uc0ac\uc2e4\uc5d0 \ub300\ud574\uc11c\ub294 \ub300\ud1b5\ub839 \ud1f4\uc784 \ud6c4\uc5d0 \uc54c\uac8c \ub41c \uc0ac\uc2e4\uc774\uba70, \ud638\uc758\uc801\uc778 \ud22c\uc790\uc77c \ubfd0 \uc790\uc2e0\uacfc\ub294 \ubb34\uad00\ud558\ub2e4\uace0 \uc8fc\uc7a5\ud588\ub2e4. \uc815\uc0c1\ubb38\uc774 \ud6a1\ub839\ud55c 12\uc5b5 5000\ub9cc\uc6d0\uc5d0 \ub300\ud574\uc11c\ub3c4 \ubab0\ub790\ub2e4\uace0 \uc9c4\uc220\ud588\ub2e4. \uac80\ucc30\uc740 \ub178\ubb34\ud604\uc758 \uc870\uc0ac\uc5d0 \uc55e\uc11c \uae08\uc735\uc815\ubcf4\ubd84\uc11d\uc6d0(FIU)\uc5d0\uc11c \uc678\ud654\uc1a1\uae08 \uac70\ub798 \ub0b4\uc5ed\uc744 \uac74\ub124\ubc1b\uc544 2006\u223c2007\ub144 \uad8c\uc591\uc219\uc774 \ub2e4\ub978 \uc0ac\ub78c\uc744 \uc2dc\ucf1c 30\ub9cc \ub2ec\ub7ec \uc774\uc0c1\uc744 \ubbf8\uad6d\uc5d0 \uccb4\ub958\ud558\ub358 \uc7a5\ub0a8 \ub178\uac74\ud638\uc640 \ub538 \ub178\uc815\uc5f0\uc5d0\uac8c \uc1a1\uae08\ud55c \uc0ac\uc2e4\uc744 \ud655\uc778\ud588\ub2e4. \ub178\ubb34\ud604\uc740 \uae30\uc874 \uc11c\uba74 \ub2f5\ubcc0\uacfc \uac19\uc774 \uc0ac\uc2e4\uc774 \uc544\ub2c8\uba70 \uae30\uc5b5\uc774 \ub098\uc9c0 \uc54a\ub294\ub2e4\ub294 \uc785\uc7a5\uc744 \uc720\uc9c0\ud588\uc73c\ub098, \uac80\ucc30\uc774 \ud655\ubcf4\ud55c \uae08\uc735\uc815\ubcf4\ubd84\uc11d\uc6d0 \uc790\ub8cc \uc55e\uc5d0\uc11c\ub294 \ud754\ub4e4\ub9ac\ub294 \ubaa8\uc2b5\uc744 \ubcf4\uc778 \uac83\uc73c\ub85c \uc804\ud574\uc84c\ub2e4. \ub178\ubb34\ud604\uc774 \uc804\ubc18\uc801\uc73c\ub85c \ud610\uc758\ub97c \ubd80\uc778\ud568\uc5d0 \ub530\ub77c \uac80\ucc30\uc740 \uc624\ud6c4 11\uc2dc\uacbd \ub178\ubb34\ud604\uacfc \ubc15\uc5f0\ucc28\uc758 \ub300\uc9c8 \uc870\uc0ac\ub97c \uc2dc\ub3c4\ud588\uc73c\ub098, \ub178\ubb34\ud604\uacfc \ubcc0\ud638\uc0ac \ubb38\uc7ac\uc778\uc740 \uc804\uc9c1 \ub300\ud1b5\ub839\uc5d0 \ub300\ud55c \uc608\uc6b0\uac00 \uc544\ub2c8\uba70 \uc2dc\uac04\uc774 \ub108\ubb34 \ub2a6\uc5c8\ub2e4\uace0 \uac70\ubd80\ud574 11\uc2dc 20\ubd84 \uacbd \uc870\uc0ac\ub97c \uc885\ub8cc\ud588\ub2e4.", "answer": "\ubb38\uc7ac\uc778", "sentence": "\ub178\ubb34\ud604\uc758 \ubcc0\ud638\uc778 \uce21\uc740 \ubb38\uc7ac\uc778 \uacfc \uc804\ud574\ucca0 \ubcc0\ud638\uc0ac\uac00 \ubc88\uac08\uc544 \uac00\uba70 \ubcc0\ud638\ud588\ub2e4.", "paragraph_sentence": "2009\ub144 4\uc6d4 30\uc77c \uc624\uc804 8\uc2dc \ub178\ubb34\ud604\uc740 \uae40\ud574 \ubd09\ud558\ub9c8\uc744 \uc0ac\uc800\ub97c \ucd9c\ubc1c, \uc624\ud6c4 1\uc2dc 20\ubd84 \uacbd \ub300\uac80\ucc30\uccad\uc5d0 \ud53c\uc758\uc790 \uc2e0\ubd84\uc73c\ub85c \ucd9c\uc11d\ud574 \ub300\uac80\ucc30\uccad \uc911\uc559\uc218\uc0ac\ubd80 1120\ud638 \ud2b9\ubcc4\uc870\uc0ac\uc2e4\uc5d0\uc11c \uc624\ud6c4 11\uc2dc 20\ubd84 \uacbd\uae4c\uc9c0 \uc870\uc0ac\ub97c \ubc1b\uc558\ub2e4. \ub178\ubb34\ud604\uc740 \uc9c4\uc220\uc870\uc11c\ub97c \uac80\ud1a0\ud55c \ub4a4 \uc775\uc77c \uc624\uc804 2\uc2dc 10\ubd84\ucbe4 \uccad\uc0ac\ub97c \ub098\uc640 \uadc0\uac00\ud588\ub2e4. \ub178\ubb34\ud604\uc740 \ubc15\uc5f0\ucc28\ub85c\ubd80\ud130 2007\ub144 6\uc6d4 \ub9d0 \uc815\uc0c1\ubb38 \uc804 \ucd1d\ubb34\ube44\uc11c\uad00\uc744 \ud1b5\ud574 100\ub9cc \ub2ec\ub7ec\ub97c \ubc1b\uace0, 2008\ub144 2\uc6d4 \ub9d0 \uc870\uce74\uc0ac\uc704 \uc5f0\ucca0\ud638\ub97c \ud1b5\ud574 500\ub9cc \ub2ec\ub7ec\ub97c \ubc1b\uc740 \ud610\uc758\ub97c \ubc1b\uc558\ub2e4. \ub610\ud55c \uac80\ucc30\uc740 \uc815\uc0c1\ubb38 \uc804 \ucd1d\ubb34\ube44\uc11c\uad00\uc774 \ud6a1\ub839\ud55c \ub300\ud1b5\ub839 \ud2b9\uc218\ud65c\ub3d9\ube44 12\uc5b5 5\ucc9c\ub9cc\uc6d0\uc5d0 \ub300\ud574 \ubcf4\uace0\ub97c \ubc1b\uc558\ub294\uc9c0, \ubc15\uc5f0\ucc28\uc758 \uba85\ud488 \uc2dc\uacc4 \uc120\ubb3c\uacfc 15\uc5b5\uc6d0\uc758 \ucc28\uc6a9\uc99d\uc744 \uc4f0\uac8c \ub41c \uacbd\uc704\uc5d0 \ub300\ud574\uc11c\ub3c4 \uc870\uc0ac\ud588\ub2e4. \ub178\ubb34\ud604\uc758 \uc870\uc0ac\ub294 \uc8fc\uc784\uac80\uc0ac\uc778 \uc6b0\ubcd1\uc6b0 \uc911\uc2181\uacfc\uc7a5\uc774 \uc2e0\ubb38\uc744 \ud558\uc600\uace0, \uc9c1\ubb34\uad00\ub828\uc131 \ubd80\ubd84\uc740 \uae40\ud615\uc6b1 \uac80\uc0ac\uac00, 100\ub9cc\ub2ec\ub7ec \uc218\uc218 \ud610\uc758\uc5d0 \ub300\ud574\uc11c\ub294 \uc774\uc8fc\ud615 \uac80\uc0ac\uac00 \uc870\uc0ac\ud588\ub2e4. \ub610\ud55c 500\ub9cc\ub2ec\ub7ec \uc218\uc218 \ud610\uc758\uc640 \ud2b9\uc218\ud65c\ub3d9\ube44 12\uc5b5 5000\ub9cc\uc6d0 \uc218\uc218 \ud610\uc758\uc5d0 \ub300\ud574\uc11c\ub294 \uc774\uc120\ubd09 \uac80\uc0ac\uac00 \uc870\uc0ac\ud588\uace0, \uc774\uc778\uaddc \uc911\uc218\ubd80\uc7a5\uacfc \ud64d\ub9cc\ud45c \uc218\uc0ac\uae30\ud68d\uad00\uc740 \uc870\uc0ac \uc0c1\ud669\uc744 \ubaa8\ub2c8\ud130\ub85c \uc9c0\ucf1c\ubcf4\uba70 \uc2e4\uc2dc\uac04\uc73c\ub85c \uc9c0\ud718\ud588\ub2e4. \uc784\ucc44\uc9c4 \uac80\ucc30\ucd1d\uc7a5\uc740 \uc790\uc815\uae4c\uc9c0 \uccad\uc0ac\uc5d0 \ub0a8\uc544 \uc218\uc0ac \ub0b4\uc6a9\uc744 \ubcf4\uace0 \ubc1b\uc558\uc73c\uba70, \ub178\ubb34\ud604\uc758 \ubcc0\ud638\uc778 \uce21\uc740 \ubb38\uc7ac\uc778 \uacfc \uc804\ud574\ucca0 \ubcc0\ud638\uc0ac\uac00 \ubc88\uac08\uc544 \uac00\uba70 \ubcc0\ud638\ud588\ub2e4. \uc218\uc0ac\ud300\uc740 \uc624\ud6c4 1\uc2dc 40\ubd84 \uacbd\ubd80\ud130 4\uc2dc 10\ubd84\uae4c\uc9c0 \ub300\ud1b5\ub839\uc758 \uc9c1\ubb34\uc640 \uad8c\ud55c, \ubc15\uc5f0\ucc28\uc640\uc758 \uad00\uacc4\uc5d0 \ub300\ud574 \uc870\uc0ac\ud558\uc600\uc73c\uba70, 10\ubd84\uac04 \ud734\uc2dd\ud55c \ub4a4 \ubc15\uc5f0\ucc28\ub85c\ubd80\ud130 100\ub9cc\ub2ec\ub7ec\ub97c \uc218\uc218\ud55c \ud610\uc758\uc5d0 \ub300\ud574 \uc624\ud6c4 6\uc2dc 30\ubd84\uae4c\uc9c0 \uc870\uc0ac\ub97c \uc9c4\ud589\ud588\ub2e4. \uc800\ub141 \uc2dd\uc0ac \ud6c4 \uc624\ud6c4 7\uc2dc35\ubd84\ubd80\ud130 \ubc15\uc5f0\ucc28\ub85c \ubd80\ud130 500\ub9cc \ub2ec\ub7ec\ub97c \uc218\uc218\ud55c \ud610\uc758, \uc815\uc0c1\ubb38 \uc804 \ub300\ud1b5\ub839 \ucd1d\ubb34\ube44\uc11c\uad00\uc774 \ud6a1\ub839\ud55c \ub300\ud1b5\ub839 \ud2b9\uc218\ud65c\ub3d9\ube44 12\uc5b55\ucc9c\ub9cc\uc6d0, \ubc15\uc5f0\ucc28\uac00 \ub178\ubb34\ud604 \uce21\uc5d0\uac8c \uc120\ubb3c\ud55c \uba85\ud488\uc2dc\uacc4 \ub4f1\uc5d0 \ub300\ud574 \uc870\uc0ac\ud588\ub2e4. \uac80\ucc30\uc740 \ubc15\uc5f0\ucc28\uc758 \ubca0\ud2b8\ub0a8 \ud654\ub825\ubc1c\uc804\uc18c \uc0ac\uc5c5 \ub4f1\uc5d0 \ub3c4\uc6c0\uc744 \uc900 \ub300\uac00\ub85c 600\ub9cc \ub2ec\ub7ec\ub97c \ubc1b\uc740 \uac8c \uc544\ub2c8\ub0d0\uace0 \uc2e0\ubb38\ud588\uc73c\ub098, \ub178\ubb34\ud604\uc740 \ub300\ud1b5\ub839\uc774 \ud55c\uad6d \uae30\uc5c5\uc758 \ud574\uc678 \uc0ac\uc5c5\uc5d0 \ub3c4\uc6c0\uc744 \uc900 \uac8c \ubb38\uc81c\uac00 \ub420 \uc218 \uc5c6\ub2e4\uace0 \uc8fc\uc7a5\ud588\ub2e4. \ub178\ubb34\ud604\uc740 2007\ub144 \ubc15\uc5f0\ucc28\ub85c\ubd80\ud130 \ubc1b\uc740 100\ub9cc \ub2ec\ub7ec\uc5d0 \ub300\ud574 \ucc44\ubb34\ub97c \uac1a\ub294\ub370 \uc790\uc2e0\uc758 \ubc30\uc6b0\uc790 \uad8c\uc591\uc219\uc774 \uc4f4 \uac83\uc774\uba70, \ub300\ud1b5\ub839 \uc7ac\uc784 \uc911 \uc774\uac19\uc740 \uc0ac\uc2e4\uc744 \ubab0\ub790\ub2e4\uace0 \uc9c4\uc220\ud588\ub2e4. 100\ub9cc \ub2ec\ub7ec\uc758 \uad6c\uccb4\uc801\uc778 \uc0ac\uc6a9\ucc98\uc5d0 \ub300\ud574\uc11c\ub294 \uc11c\uba74 \ub2f5\ubcc0\uacfc \ub9c8\ucc2c\uac00\uc9c0\ub85c \ubc1d\ud790 \uc218 \uc5c6\ub2e4\uace0 \uc9c4\uc220\ud588\ub2e4. 2008\ub144 500\ub9cc \ub2ec\ub7ec\ub97c \ubc1b\uc740 \uc0ac\uc2e4\uc5d0 \ub300\ud574\uc11c\ub294 \ub300\ud1b5\ub839 \ud1f4\uc784 \ud6c4\uc5d0 \uc54c\uac8c \ub41c \uc0ac\uc2e4\uc774\uba70, \ud638\uc758\uc801\uc778 \ud22c\uc790\uc77c \ubfd0 \uc790\uc2e0\uacfc\ub294 \ubb34\uad00\ud558\ub2e4\uace0 \uc8fc\uc7a5\ud588\ub2e4. \uc815\uc0c1\ubb38\uc774 \ud6a1\ub839\ud55c 12\uc5b5 5000\ub9cc\uc6d0\uc5d0 \ub300\ud574\uc11c\ub3c4 \ubab0\ub790\ub2e4\uace0 \uc9c4\uc220\ud588\ub2e4. \uac80\ucc30\uc740 \ub178\ubb34\ud604\uc758 \uc870\uc0ac\uc5d0 \uc55e\uc11c \uae08\uc735\uc815\ubcf4\ubd84\uc11d\uc6d0(FIU)\uc5d0\uc11c \uc678\ud654\uc1a1\uae08 \uac70\ub798 \ub0b4\uc5ed\uc744 \uac74\ub124\ubc1b\uc544 2006\u223c2007\ub144 \uad8c\uc591\uc219\uc774 \ub2e4\ub978 \uc0ac\ub78c\uc744 \uc2dc\ucf1c 30\ub9cc \ub2ec\ub7ec \uc774\uc0c1\uc744 \ubbf8\uad6d\uc5d0 \uccb4\ub958\ud558\ub358 \uc7a5\ub0a8 \ub178\uac74\ud638\uc640 \ub538 \ub178\uc815\uc5f0\uc5d0\uac8c \uc1a1\uae08\ud55c \uc0ac\uc2e4\uc744 \ud655\uc778\ud588\ub2e4. \ub178\ubb34\ud604\uc740 \uae30\uc874 \uc11c\uba74 \ub2f5\ubcc0\uacfc \uac19\uc774 \uc0ac\uc2e4\uc774 \uc544\ub2c8\uba70 \uae30\uc5b5\uc774 \ub098\uc9c0 \uc54a\ub294\ub2e4\ub294 \uc785\uc7a5\uc744 \uc720\uc9c0\ud588\uc73c\ub098, \uac80\ucc30\uc774 \ud655\ubcf4\ud55c \uae08\uc735\uc815\ubcf4\ubd84\uc11d\uc6d0 \uc790\ub8cc \uc55e\uc5d0\uc11c\ub294 \ud754\ub4e4\ub9ac\ub294 \ubaa8\uc2b5\uc744 \ubcf4\uc778 \uac83\uc73c\ub85c \uc804\ud574\uc84c\ub2e4. \ub178\ubb34\ud604\uc774 \uc804\ubc18\uc801\uc73c\ub85c \ud610\uc758\ub97c \ubd80\uc778\ud568\uc5d0 \ub530\ub77c \uac80\ucc30\uc740 \uc624\ud6c4 11\uc2dc\uacbd \ub178\ubb34\ud604\uacfc \ubc15\uc5f0\ucc28\uc758 \ub300\uc9c8 \uc870\uc0ac\ub97c \uc2dc\ub3c4\ud588\uc73c\ub098, \ub178\ubb34\ud604\uacfc \ubcc0\ud638\uc0ac \ubb38\uc7ac\uc778\uc740 \uc804\uc9c1 \ub300\ud1b5\ub839\uc5d0 \ub300\ud55c \uc608\uc6b0\uac00 \uc544\ub2c8\uba70 \uc2dc\uac04\uc774 \ub108\ubb34 \ub2a6\uc5c8\ub2e4\uace0 \uac70\ubd80\ud574 11\uc2dc 20\ubd84 \uacbd \uc870\uc0ac\ub97c \uc885\ub8cc\ud588\ub2e4.", "paragraph_answer": "2009\ub144 4\uc6d4 30\uc77c \uc624\uc804 8\uc2dc \ub178\ubb34\ud604\uc740 \uae40\ud574 \ubd09\ud558\ub9c8\uc744 \uc0ac\uc800\ub97c \ucd9c\ubc1c, \uc624\ud6c4 1\uc2dc 20\ubd84 \uacbd \ub300\uac80\ucc30\uccad\uc5d0 \ud53c\uc758\uc790 \uc2e0\ubd84\uc73c\ub85c \ucd9c\uc11d\ud574 \ub300\uac80\ucc30\uccad \uc911\uc559\uc218\uc0ac\ubd80 1120\ud638 \ud2b9\ubcc4\uc870\uc0ac\uc2e4\uc5d0\uc11c \uc624\ud6c4 11\uc2dc 20\ubd84 \uacbd\uae4c\uc9c0 \uc870\uc0ac\ub97c \ubc1b\uc558\ub2e4. \ub178\ubb34\ud604\uc740 \uc9c4\uc220\uc870\uc11c\ub97c \uac80\ud1a0\ud55c \ub4a4 \uc775\uc77c \uc624\uc804 2\uc2dc 10\ubd84\ucbe4 \uccad\uc0ac\ub97c \ub098\uc640 \uadc0\uac00\ud588\ub2e4. \ub178\ubb34\ud604\uc740 \ubc15\uc5f0\ucc28\ub85c\ubd80\ud130 2007\ub144 6\uc6d4 \ub9d0 \uc815\uc0c1\ubb38 \uc804 \ucd1d\ubb34\ube44\uc11c\uad00\uc744 \ud1b5\ud574 100\ub9cc \ub2ec\ub7ec\ub97c \ubc1b\uace0, 2008\ub144 2\uc6d4 \ub9d0 \uc870\uce74\uc0ac\uc704 \uc5f0\ucca0\ud638\ub97c \ud1b5\ud574 500\ub9cc \ub2ec\ub7ec\ub97c \ubc1b\uc740 \ud610\uc758\ub97c \ubc1b\uc558\ub2e4. \ub610\ud55c \uac80\ucc30\uc740 \uc815\uc0c1\ubb38 \uc804 \ucd1d\ubb34\ube44\uc11c\uad00\uc774 \ud6a1\ub839\ud55c \ub300\ud1b5\ub839 \ud2b9\uc218\ud65c\ub3d9\ube44 12\uc5b5 5\ucc9c\ub9cc\uc6d0\uc5d0 \ub300\ud574 \ubcf4\uace0\ub97c \ubc1b\uc558\ub294\uc9c0, \ubc15\uc5f0\ucc28\uc758 \uba85\ud488 \uc2dc\uacc4 \uc120\ubb3c\uacfc 15\uc5b5\uc6d0\uc758 \ucc28\uc6a9\uc99d\uc744 \uc4f0\uac8c \ub41c \uacbd\uc704\uc5d0 \ub300\ud574\uc11c\ub3c4 \uc870\uc0ac\ud588\ub2e4. \ub178\ubb34\ud604\uc758 \uc870\uc0ac\ub294 \uc8fc\uc784\uac80\uc0ac\uc778 \uc6b0\ubcd1\uc6b0 \uc911\uc2181\uacfc\uc7a5\uc774 \uc2e0\ubb38\uc744 \ud558\uc600\uace0, \uc9c1\ubb34\uad00\ub828\uc131 \ubd80\ubd84\uc740 \uae40\ud615\uc6b1 \uac80\uc0ac\uac00, 100\ub9cc\ub2ec\ub7ec \uc218\uc218 \ud610\uc758\uc5d0 \ub300\ud574\uc11c\ub294 \uc774\uc8fc\ud615 \uac80\uc0ac\uac00 \uc870\uc0ac\ud588\ub2e4. \ub610\ud55c 500\ub9cc\ub2ec\ub7ec \uc218\uc218 \ud610\uc758\uc640 \ud2b9\uc218\ud65c\ub3d9\ube44 12\uc5b5 5000\ub9cc\uc6d0 \uc218\uc218 \ud610\uc758\uc5d0 \ub300\ud574\uc11c\ub294 \uc774\uc120\ubd09 \uac80\uc0ac\uac00 \uc870\uc0ac\ud588\uace0, \uc774\uc778\uaddc \uc911\uc218\ubd80\uc7a5\uacfc \ud64d\ub9cc\ud45c \uc218\uc0ac\uae30\ud68d\uad00\uc740 \uc870\uc0ac \uc0c1\ud669\uc744 \ubaa8\ub2c8\ud130\ub85c \uc9c0\ucf1c\ubcf4\uba70 \uc2e4\uc2dc\uac04\uc73c\ub85c \uc9c0\ud718\ud588\ub2e4. \uc784\ucc44\uc9c4 \uac80\ucc30\ucd1d\uc7a5\uc740 \uc790\uc815\uae4c\uc9c0 \uccad\uc0ac\uc5d0 \ub0a8\uc544 \uc218\uc0ac \ub0b4\uc6a9\uc744 \ubcf4\uace0 \ubc1b\uc558\uc73c\uba70, \ub178\ubb34\ud604\uc758 \ubcc0\ud638\uc778 \uce21\uc740 \ubb38\uc7ac\uc778 \uacfc \uc804\ud574\ucca0 \ubcc0\ud638\uc0ac\uac00 \ubc88\uac08\uc544 \uac00\uba70 \ubcc0\ud638\ud588\ub2e4. \uc218\uc0ac\ud300\uc740 \uc624\ud6c4 1\uc2dc 40\ubd84 \uacbd\ubd80\ud130 4\uc2dc 10\ubd84\uae4c\uc9c0 \ub300\ud1b5\ub839\uc758 \uc9c1\ubb34\uc640 \uad8c\ud55c, \ubc15\uc5f0\ucc28\uc640\uc758 \uad00\uacc4\uc5d0 \ub300\ud574 \uc870\uc0ac\ud558\uc600\uc73c\uba70, 10\ubd84\uac04 \ud734\uc2dd\ud55c \ub4a4 \ubc15\uc5f0\ucc28\ub85c\ubd80\ud130 100\ub9cc\ub2ec\ub7ec\ub97c \uc218\uc218\ud55c \ud610\uc758\uc5d0 \ub300\ud574 \uc624\ud6c4 6\uc2dc 30\ubd84\uae4c\uc9c0 \uc870\uc0ac\ub97c \uc9c4\ud589\ud588\ub2e4. \uc800\ub141 \uc2dd\uc0ac \ud6c4 \uc624\ud6c4 7\uc2dc35\ubd84\ubd80\ud130 \ubc15\uc5f0\ucc28\ub85c \ubd80\ud130 500\ub9cc \ub2ec\ub7ec\ub97c \uc218\uc218\ud55c \ud610\uc758, \uc815\uc0c1\ubb38 \uc804 \ub300\ud1b5\ub839 \ucd1d\ubb34\ube44\uc11c\uad00\uc774 \ud6a1\ub839\ud55c \ub300\ud1b5\ub839 \ud2b9\uc218\ud65c\ub3d9\ube44 12\uc5b55\ucc9c\ub9cc\uc6d0, \ubc15\uc5f0\ucc28\uac00 \ub178\ubb34\ud604 \uce21\uc5d0\uac8c \uc120\ubb3c\ud55c \uba85\ud488\uc2dc\uacc4 \ub4f1\uc5d0 \ub300\ud574 \uc870\uc0ac\ud588\ub2e4. \uac80\ucc30\uc740 \ubc15\uc5f0\ucc28\uc758 \ubca0\ud2b8\ub0a8 \ud654\ub825\ubc1c\uc804\uc18c \uc0ac\uc5c5 \ub4f1\uc5d0 \ub3c4\uc6c0\uc744 \uc900 \ub300\uac00\ub85c 600\ub9cc \ub2ec\ub7ec\ub97c \ubc1b\uc740 \uac8c \uc544\ub2c8\ub0d0\uace0 \uc2e0\ubb38\ud588\uc73c\ub098, \ub178\ubb34\ud604\uc740 \ub300\ud1b5\ub839\uc774 \ud55c\uad6d \uae30\uc5c5\uc758 \ud574\uc678 \uc0ac\uc5c5\uc5d0 \ub3c4\uc6c0\uc744 \uc900 \uac8c \ubb38\uc81c\uac00 \ub420 \uc218 \uc5c6\ub2e4\uace0 \uc8fc\uc7a5\ud588\ub2e4. \ub178\ubb34\ud604\uc740 2007\ub144 \ubc15\uc5f0\ucc28\ub85c\ubd80\ud130 \ubc1b\uc740 100\ub9cc \ub2ec\ub7ec\uc5d0 \ub300\ud574 \ucc44\ubb34\ub97c \uac1a\ub294\ub370 \uc790\uc2e0\uc758 \ubc30\uc6b0\uc790 \uad8c\uc591\uc219\uc774 \uc4f4 \uac83\uc774\uba70, \ub300\ud1b5\ub839 \uc7ac\uc784 \uc911 \uc774\uac19\uc740 \uc0ac\uc2e4\uc744 \ubab0\ub790\ub2e4\uace0 \uc9c4\uc220\ud588\ub2e4. 100\ub9cc \ub2ec\ub7ec\uc758 \uad6c\uccb4\uc801\uc778 \uc0ac\uc6a9\ucc98\uc5d0 \ub300\ud574\uc11c\ub294 \uc11c\uba74 \ub2f5\ubcc0\uacfc \ub9c8\ucc2c\uac00\uc9c0\ub85c \ubc1d\ud790 \uc218 \uc5c6\ub2e4\uace0 \uc9c4\uc220\ud588\ub2e4. 2008\ub144 500\ub9cc \ub2ec\ub7ec\ub97c \ubc1b\uc740 \uc0ac\uc2e4\uc5d0 \ub300\ud574\uc11c\ub294 \ub300\ud1b5\ub839 \ud1f4\uc784 \ud6c4\uc5d0 \uc54c\uac8c \ub41c \uc0ac\uc2e4\uc774\uba70, \ud638\uc758\uc801\uc778 \ud22c\uc790\uc77c \ubfd0 \uc790\uc2e0\uacfc\ub294 \ubb34\uad00\ud558\ub2e4\uace0 \uc8fc\uc7a5\ud588\ub2e4. \uc815\uc0c1\ubb38\uc774 \ud6a1\ub839\ud55c 12\uc5b5 5000\ub9cc\uc6d0\uc5d0 \ub300\ud574\uc11c\ub3c4 \ubab0\ub790\ub2e4\uace0 \uc9c4\uc220\ud588\ub2e4. \uac80\ucc30\uc740 \ub178\ubb34\ud604\uc758 \uc870\uc0ac\uc5d0 \uc55e\uc11c \uae08\uc735\uc815\ubcf4\ubd84\uc11d\uc6d0(FIU)\uc5d0\uc11c \uc678\ud654\uc1a1\uae08 \uac70\ub798 \ub0b4\uc5ed\uc744 \uac74\ub124\ubc1b\uc544 2006\u223c2007\ub144 \uad8c\uc591\uc219\uc774 \ub2e4\ub978 \uc0ac\ub78c\uc744 \uc2dc\ucf1c 30\ub9cc \ub2ec\ub7ec \uc774\uc0c1\uc744 \ubbf8\uad6d\uc5d0 \uccb4\ub958\ud558\ub358 \uc7a5\ub0a8 \ub178\uac74\ud638\uc640 \ub538 \ub178\uc815\uc5f0\uc5d0\uac8c \uc1a1\uae08\ud55c \uc0ac\uc2e4\uc744 \ud655\uc778\ud588\ub2e4. \ub178\ubb34\ud604\uc740 \uae30\uc874 \uc11c\uba74 \ub2f5\ubcc0\uacfc \uac19\uc774 \uc0ac\uc2e4\uc774 \uc544\ub2c8\uba70 \uae30\uc5b5\uc774 \ub098\uc9c0 \uc54a\ub294\ub2e4\ub294 \uc785\uc7a5\uc744 \uc720\uc9c0\ud588\uc73c\ub098, \uac80\ucc30\uc774 \ud655\ubcf4\ud55c \uae08\uc735\uc815\ubcf4\ubd84\uc11d\uc6d0 \uc790\ub8cc \uc55e\uc5d0\uc11c\ub294 \ud754\ub4e4\ub9ac\ub294 \ubaa8\uc2b5\uc744 \ubcf4\uc778 \uac83\uc73c\ub85c \uc804\ud574\uc84c\ub2e4. \ub178\ubb34\ud604\uc774 \uc804\ubc18\uc801\uc73c\ub85c \ud610\uc758\ub97c \ubd80\uc778\ud568\uc5d0 \ub530\ub77c \uac80\ucc30\uc740 \uc624\ud6c4 11\uc2dc\uacbd \ub178\ubb34\ud604\uacfc \ubc15\uc5f0\ucc28\uc758 \ub300\uc9c8 \uc870\uc0ac\ub97c \uc2dc\ub3c4\ud588\uc73c\ub098, \ub178\ubb34\ud604\uacfc \ubcc0\ud638\uc0ac \ubb38\uc7ac\uc778\uc740 \uc804\uc9c1 \ub300\ud1b5\ub839\uc5d0 \ub300\ud55c \uc608\uc6b0\uac00 \uc544\ub2c8\uba70 \uc2dc\uac04\uc774 \ub108\ubb34 \ub2a6\uc5c8\ub2e4\uace0 \uac70\ubd80\ud574 11\uc2dc 20\ubd84 \uacbd \uc870\uc0ac\ub97c \uc885\ub8cc\ud588\ub2e4.", "sentence_answer": "\ub178\ubb34\ud604\uc758 \ubcc0\ud638\uc778 \uce21\uc740 \ubb38\uc7ac\uc778 \uacfc \uc804\ud574\ucca0 \ubcc0\ud638\uc0ac\uac00 \ubc88\uac08\uc544 \uac00\uba70 \ubcc0\ud638\ud588\ub2e4.", "paragraph_id": "6512090-20-1", "paragraph_question": "question: \uc138\ubb34\uc870\uc0ac \ub2f9\uc2dc \ub178\ubb34\ud604\uc744 \ubcc0\ud638\ud55c \uc0ac\ub78c\uc740 \uc804\ud574\ucca0\uacfc \ub204\uad6c\uc778\uac00?, context: 2009\ub144 4\uc6d4 30\uc77c \uc624\uc804 8\uc2dc \ub178\ubb34\ud604\uc740 \uae40\ud574 \ubd09\ud558\ub9c8\uc744 \uc0ac\uc800\ub97c \ucd9c\ubc1c, \uc624\ud6c4 1\uc2dc 20\ubd84 \uacbd \ub300\uac80\ucc30\uccad\uc5d0 \ud53c\uc758\uc790 \uc2e0\ubd84\uc73c\ub85c \ucd9c\uc11d\ud574 \ub300\uac80\ucc30\uccad \uc911\uc559\uc218\uc0ac\ubd80 1120\ud638 \ud2b9\ubcc4\uc870\uc0ac\uc2e4\uc5d0\uc11c \uc624\ud6c4 11\uc2dc 20\ubd84 \uacbd\uae4c\uc9c0 \uc870\uc0ac\ub97c \ubc1b\uc558\ub2e4. \ub178\ubb34\ud604\uc740 \uc9c4\uc220\uc870\uc11c\ub97c \uac80\ud1a0\ud55c \ub4a4 \uc775\uc77c \uc624\uc804 2\uc2dc 10\ubd84\ucbe4 \uccad\uc0ac\ub97c \ub098\uc640 \uadc0\uac00\ud588\ub2e4. \ub178\ubb34\ud604\uc740 \ubc15\uc5f0\ucc28\ub85c\ubd80\ud130 2007\ub144 6\uc6d4 \ub9d0 \uc815\uc0c1\ubb38 \uc804 \ucd1d\ubb34\ube44\uc11c\uad00\uc744 \ud1b5\ud574 100\ub9cc \ub2ec\ub7ec\ub97c \ubc1b\uace0, 2008\ub144 2\uc6d4 \ub9d0 \uc870\uce74\uc0ac\uc704 \uc5f0\ucca0\ud638\ub97c \ud1b5\ud574 500\ub9cc \ub2ec\ub7ec\ub97c \ubc1b\uc740 \ud610\uc758\ub97c \ubc1b\uc558\ub2e4. \ub610\ud55c \uac80\ucc30\uc740 \uc815\uc0c1\ubb38 \uc804 \ucd1d\ubb34\ube44\uc11c\uad00\uc774 \ud6a1\ub839\ud55c \ub300\ud1b5\ub839 \ud2b9\uc218\ud65c\ub3d9\ube44 12\uc5b5 5\ucc9c\ub9cc\uc6d0\uc5d0 \ub300\ud574 \ubcf4\uace0\ub97c \ubc1b\uc558\ub294\uc9c0, \ubc15\uc5f0\ucc28\uc758 \uba85\ud488 \uc2dc\uacc4 \uc120\ubb3c\uacfc 15\uc5b5\uc6d0\uc758 \ucc28\uc6a9\uc99d\uc744 \uc4f0\uac8c \ub41c \uacbd\uc704\uc5d0 \ub300\ud574\uc11c\ub3c4 \uc870\uc0ac\ud588\ub2e4. \ub178\ubb34\ud604\uc758 \uc870\uc0ac\ub294 \uc8fc\uc784\uac80\uc0ac\uc778 \uc6b0\ubcd1\uc6b0 \uc911\uc2181\uacfc\uc7a5\uc774 \uc2e0\ubb38\uc744 \ud558\uc600\uace0, \uc9c1\ubb34\uad00\ub828\uc131 \ubd80\ubd84\uc740 \uae40\ud615\uc6b1 \uac80\uc0ac\uac00, 100\ub9cc\ub2ec\ub7ec \uc218\uc218 \ud610\uc758\uc5d0 \ub300\ud574\uc11c\ub294 \uc774\uc8fc\ud615 \uac80\uc0ac\uac00 \uc870\uc0ac\ud588\ub2e4. \ub610\ud55c 500\ub9cc\ub2ec\ub7ec \uc218\uc218 \ud610\uc758\uc640 \ud2b9\uc218\ud65c\ub3d9\ube44 12\uc5b5 5000\ub9cc\uc6d0 \uc218\uc218 \ud610\uc758\uc5d0 \ub300\ud574\uc11c\ub294 \uc774\uc120\ubd09 \uac80\uc0ac\uac00 \uc870\uc0ac\ud588\uace0, \uc774\uc778\uaddc \uc911\uc218\ubd80\uc7a5\uacfc \ud64d\ub9cc\ud45c \uc218\uc0ac\uae30\ud68d\uad00\uc740 \uc870\uc0ac \uc0c1\ud669\uc744 \ubaa8\ub2c8\ud130\ub85c \uc9c0\ucf1c\ubcf4\uba70 \uc2e4\uc2dc\uac04\uc73c\ub85c \uc9c0\ud718\ud588\ub2e4. \uc784\ucc44\uc9c4 \uac80\ucc30\ucd1d\uc7a5\uc740 \uc790\uc815\uae4c\uc9c0 \uccad\uc0ac\uc5d0 \ub0a8\uc544 \uc218\uc0ac \ub0b4\uc6a9\uc744 \ubcf4\uace0 \ubc1b\uc558\uc73c\uba70, \ub178\ubb34\ud604\uc758 \ubcc0\ud638\uc778 \uce21\uc740 \ubb38\uc7ac\uc778\uacfc \uc804\ud574\ucca0 \ubcc0\ud638\uc0ac\uac00 \ubc88\uac08\uc544 \uac00\uba70 \ubcc0\ud638\ud588\ub2e4. \uc218\uc0ac\ud300\uc740 \uc624\ud6c4 1\uc2dc 40\ubd84 \uacbd\ubd80\ud130 4\uc2dc 10\ubd84\uae4c\uc9c0 \ub300\ud1b5\ub839\uc758 \uc9c1\ubb34\uc640 \uad8c\ud55c, \ubc15\uc5f0\ucc28\uc640\uc758 \uad00\uacc4\uc5d0 \ub300\ud574 \uc870\uc0ac\ud558\uc600\uc73c\uba70, 10\ubd84\uac04 \ud734\uc2dd\ud55c \ub4a4 \ubc15\uc5f0\ucc28\ub85c\ubd80\ud130 100\ub9cc\ub2ec\ub7ec\ub97c \uc218\uc218\ud55c \ud610\uc758\uc5d0 \ub300\ud574 \uc624\ud6c4 6\uc2dc 30\ubd84\uae4c\uc9c0 \uc870\uc0ac\ub97c \uc9c4\ud589\ud588\ub2e4. \uc800\ub141 \uc2dd\uc0ac \ud6c4 \uc624\ud6c4 7\uc2dc35\ubd84\ubd80\ud130 \ubc15\uc5f0\ucc28\ub85c \ubd80\ud130 500\ub9cc \ub2ec\ub7ec\ub97c \uc218\uc218\ud55c \ud610\uc758, \uc815\uc0c1\ubb38 \uc804 \ub300\ud1b5\ub839 \ucd1d\ubb34\ube44\uc11c\uad00\uc774 \ud6a1\ub839\ud55c \ub300\ud1b5\ub839 \ud2b9\uc218\ud65c\ub3d9\ube44 12\uc5b55\ucc9c\ub9cc\uc6d0, \ubc15\uc5f0\ucc28\uac00 \ub178\ubb34\ud604 \uce21\uc5d0\uac8c \uc120\ubb3c\ud55c \uba85\ud488\uc2dc\uacc4 \ub4f1\uc5d0 \ub300\ud574 \uc870\uc0ac\ud588\ub2e4. \uac80\ucc30\uc740 \ubc15\uc5f0\ucc28\uc758 \ubca0\ud2b8\ub0a8 \ud654\ub825\ubc1c\uc804\uc18c \uc0ac\uc5c5 \ub4f1\uc5d0 \ub3c4\uc6c0\uc744 \uc900 \ub300\uac00\ub85c 600\ub9cc \ub2ec\ub7ec\ub97c \ubc1b\uc740 \uac8c \uc544\ub2c8\ub0d0\uace0 \uc2e0\ubb38\ud588\uc73c\ub098, \ub178\ubb34\ud604\uc740 \ub300\ud1b5\ub839\uc774 \ud55c\uad6d \uae30\uc5c5\uc758 \ud574\uc678 \uc0ac\uc5c5\uc5d0 \ub3c4\uc6c0\uc744 \uc900 \uac8c \ubb38\uc81c\uac00 \ub420 \uc218 \uc5c6\ub2e4\uace0 \uc8fc\uc7a5\ud588\ub2e4. \ub178\ubb34\ud604\uc740 2007\ub144 \ubc15\uc5f0\ucc28\ub85c\ubd80\ud130 \ubc1b\uc740 100\ub9cc \ub2ec\ub7ec\uc5d0 \ub300\ud574 \ucc44\ubb34\ub97c \uac1a\ub294\ub370 \uc790\uc2e0\uc758 \ubc30\uc6b0\uc790 \uad8c\uc591\uc219\uc774 \uc4f4 \uac83\uc774\uba70, \ub300\ud1b5\ub839 \uc7ac\uc784 \uc911 \uc774\uac19\uc740 \uc0ac\uc2e4\uc744 \ubab0\ub790\ub2e4\uace0 \uc9c4\uc220\ud588\ub2e4. 100\ub9cc \ub2ec\ub7ec\uc758 \uad6c\uccb4\uc801\uc778 \uc0ac\uc6a9\ucc98\uc5d0 \ub300\ud574\uc11c\ub294 \uc11c\uba74 \ub2f5\ubcc0\uacfc \ub9c8\ucc2c\uac00\uc9c0\ub85c \ubc1d\ud790 \uc218 \uc5c6\ub2e4\uace0 \uc9c4\uc220\ud588\ub2e4. 2008\ub144 500\ub9cc \ub2ec\ub7ec\ub97c \ubc1b\uc740 \uc0ac\uc2e4\uc5d0 \ub300\ud574\uc11c\ub294 \ub300\ud1b5\ub839 \ud1f4\uc784 \ud6c4\uc5d0 \uc54c\uac8c \ub41c \uc0ac\uc2e4\uc774\uba70, \ud638\uc758\uc801\uc778 \ud22c\uc790\uc77c \ubfd0 \uc790\uc2e0\uacfc\ub294 \ubb34\uad00\ud558\ub2e4\uace0 \uc8fc\uc7a5\ud588\ub2e4. \uc815\uc0c1\ubb38\uc774 \ud6a1\ub839\ud55c 12\uc5b5 5000\ub9cc\uc6d0\uc5d0 \ub300\ud574\uc11c\ub3c4 \ubab0\ub790\ub2e4\uace0 \uc9c4\uc220\ud588\ub2e4. \uac80\ucc30\uc740 \ub178\ubb34\ud604\uc758 \uc870\uc0ac\uc5d0 \uc55e\uc11c \uae08\uc735\uc815\ubcf4\ubd84\uc11d\uc6d0(FIU)\uc5d0\uc11c \uc678\ud654\uc1a1\uae08 \uac70\ub798 \ub0b4\uc5ed\uc744 \uac74\ub124\ubc1b\uc544 2006\u223c2007\ub144 \uad8c\uc591\uc219\uc774 \ub2e4\ub978 \uc0ac\ub78c\uc744 \uc2dc\ucf1c 30\ub9cc \ub2ec\ub7ec \uc774\uc0c1\uc744 \ubbf8\uad6d\uc5d0 \uccb4\ub958\ud558\ub358 \uc7a5\ub0a8 \ub178\uac74\ud638\uc640 \ub538 \ub178\uc815\uc5f0\uc5d0\uac8c \uc1a1\uae08\ud55c \uc0ac\uc2e4\uc744 \ud655\uc778\ud588\ub2e4. \ub178\ubb34\ud604\uc740 \uae30\uc874 \uc11c\uba74 \ub2f5\ubcc0\uacfc \uac19\uc774 \uc0ac\uc2e4\uc774 \uc544\ub2c8\uba70 \uae30\uc5b5\uc774 \ub098\uc9c0 \uc54a\ub294\ub2e4\ub294 \uc785\uc7a5\uc744 \uc720\uc9c0\ud588\uc73c\ub098, \uac80\ucc30\uc774 \ud655\ubcf4\ud55c \uae08\uc735\uc815\ubcf4\ubd84\uc11d\uc6d0 \uc790\ub8cc \uc55e\uc5d0\uc11c\ub294 \ud754\ub4e4\ub9ac\ub294 \ubaa8\uc2b5\uc744 \ubcf4\uc778 \uac83\uc73c\ub85c \uc804\ud574\uc84c\ub2e4. \ub178\ubb34\ud604\uc774 \uc804\ubc18\uc801\uc73c\ub85c \ud610\uc758\ub97c \ubd80\uc778\ud568\uc5d0 \ub530\ub77c \uac80\ucc30\uc740 \uc624\ud6c4 11\uc2dc\uacbd \ub178\ubb34\ud604\uacfc \ubc15\uc5f0\ucc28\uc758 \ub300\uc9c8 \uc870\uc0ac\ub97c \uc2dc\ub3c4\ud588\uc73c\ub098, \ub178\ubb34\ud604\uacfc \ubcc0\ud638\uc0ac \ubb38\uc7ac\uc778\uc740 \uc804\uc9c1 \ub300\ud1b5\ub839\uc5d0 \ub300\ud55c \uc608\uc6b0\uac00 \uc544\ub2c8\uba70 \uc2dc\uac04\uc774 \ub108\ubb34 \ub2a6\uc5c8\ub2e4\uace0 \uac70\ubd80\ud574 11\uc2dc 20\ubd84 \uacbd \uc870\uc0ac\ub97c \uc885\ub8cc\ud588\ub2e4."} {"question": "\uc138\uc6d4\ud638 \ucc38\uc0ac \ub2f9\uc2dc \ud604\uc7a5\uc9c0\ud718\uad00\uc758 \uc774\ub984\uc740?", "paragraph": "2014\ub144 10\uc6d4 6\uc77c \uac80\ucc30\uc740 \uae40\uacbd\uc77c\uc744 \ubd88\uad6c\uc18d \uae30\uc18c\ud558\uc600\ub2e4. 2014\ub144 4\uc6d4 16\uc77c \uc138\uc6d4\ud638 \uce68\ubab0\uc0ac\uace0 \ub2f9\uc2dc \ud604\uc7a5\uc9c0\ud718\uad00(OSC)\uc774\uc5c8\ub358 \ubaa9\ud3ec\ud574\uc591\uacbd\ucc30\uc11c 123\uc815\uc7a5 \uae40\uacbd\uc77c\uc740 \uc138\uc6d4\ud638\uc640 \uad50\uc2e0\ud558\ub294 \ub4f1 \uc120\ub0b4 \uc2b9\uac1d \uc0c1\ud669\uc744 \ud655\uc778\ud558\uace0 \uc2b9\uac1d\uc5d0 \ub300\ud55c \ud1f4\uc120 \uc548\ub0b4\uc640 \uc720\ub3c4 \uc870\uce58\ub97c \ud588\uc5b4\uc57c \ud558\ub098, 123\uc815 \uc2b9\uc870\uc6d0 \ubc0f \ud574\uacbd \ud5ec\uae30\uc758 \uad6c\uc870 \ud65c\ub3d9\uc744 \uc9c0\ud718\ud558\uba74\uc11c \ud1f4\uc120 \uc720\ub3c4\ub97c \uc704\ud55c \uc544\ubb34\ub7f0 \uc870\uce58\ub97c \ucde8\ud558\uc9c0 \uc54a\uc558\ub2e4. \ub610\ud55c \uc11c\ud574\uc9c0\ubc29\ud574\uc591\uacbd\ucc30\uccad \ubc0f \ubaa9\ud3ec\ud574\uc591\uacbd\ucc30\uc11c \uc0c1\ud669\uc2e4\ub85c\ubd80\ud130 \uc218\ucc28\ub840\uc5d0 \uac78\uccd0 \uc2b9\uac1d\ub4e4\uc774 \ubc30\uc5d0\uc11c \ub6f0\uc5b4\ub0b4\ub9ac\ub3c4\ub85d \uace0\ud568\uc744 \uce58\uac70\ub098 123\uc815 \ub0b4 \ub300\uacf5\ub9c8\uc774\ud06c\ub97c \uc774\uc6a9\ud558\uc5ec \ud1f4\uc120\uc744 \uc720\ub3c4\ud558\ub77c\ub294 \uc9c0\uc2dc\ub97c \ubc1b\uc558\uc74c\uc5d0\ub3c4, \uac1c\uc778\uc801\uc778 \ud310\ub2e8 \ud558\uc5d0 \uc0c1\uae09\uc9c0\ud718\uad00\uc11c\uc758 \uc9c0\uc2dc\uc870\ucc28 \uc774\ud589\ud558\uc9c0 \uc54a\uc544 \uc2b9\uac1d\uc774 \uc0ac\ub9dd\uc5d0 \uc774\ub974\uac8c\ud558\uac70\ub098 \uc0c1\ud574\ub97c \uc785\ub3c4\ub85d \ubc29\uce58(\uc5c5\ubb34\uc0c1\uacfc\uc2e4\uce58\uc0ac, \uc5c5\ubb34\uc0c1\uacfc\uc2e4\uce58\uc0c1)\ud558\uc600\ub2e4. \uae40\uacbd\uc77c\uc740 2014\ub144 5\uc6d4 \uacbd \uc138\uc6d4\ud638 \uce68\ubab0 \uc0ac\uace0 \uc77c\uc778 4\uc6d4 16\uc77c\uc790 \ud568\uc815\uc77c\uc9c0\uc758 \uc77c\ubd80\ub97c \ub5bc\uc5b4\ub0b8 \ub2e4\uc74c, \ub9c8\uce58 \ub300\uacf5 \ub9c8\uc774\ud06c\ub97c \uc774\uc6a9\ud558\uc5ec \ud1f4\uc120\ubc29\uc1a1\uc744 \ud558\uace0 123\uc815 \uc2b9\uc870\uc6d0\uc5d0\uac8c \uc120\ub0b4 \uc9c4\uc785\uc744 \uc2dc\ub3c4\ud558\ub3c4\ub85d \uc9c0\uc2dc\ud55c \uc0ac\uc2e4\uc774 \uc788\ub294 \uac83\ucc98\ub7fc \ud568\uc815\uc77c\uc9c0\ub97c \ud5c8\uc704\ub85c \uc7ac\uc791\uc131\ud558\uc5ec \ucca8\ubd80\ud558\ub294 \ub4f1 \uc870\uc791(\ud5c8\uc704\uacf5\ubb38\uc11c\uc791\uc131, \ud5c8\uc704\uc791\uc131\uacf5\ubb38\uc11c\ud589\uc0ac, \uacf5\uc6a9\uc11c\ub958\uc190\uc0c1)\ud558\uc600\ub2e4. \uae40\uacbd\uc77c\uc740 \uac10\uc0ac\uc6d0 \uc870\uc0ac\uc640 \uac80\ucc30 \uc18c\ud658 \uc870\uc0ac\ub97c \uc55e\ub450\uace0 \uc2b9\uc870\uc6d0\ub4e4\uacfc \ub300\ucc45\ud68c\uc758\ub97c \uc5f4\uc5b4 123\uc815 \uc2b9\uc870\uc6d0\ub4e4\ub85c \ud558\uc5ec\uae08 \ud5c8\uc704\ub85c \uc9c4\uc220\ud558\ub3c4\ub85d \uc9c0\uc2dc\ud55c \uc0ac\uc2e4\ub3c4 \ud655\uc778\ub418\uc5c8\ub2e4.", "answer": "\uae40\uacbd\uc77c", "sentence": "2014\ub144 10\uc6d4 6\uc77c \uac80\ucc30\uc740 \uae40\uacbd\uc77c \uc744 \ubd88\uad6c\uc18d \uae30\uc18c\ud558\uc600\ub2e4.", "paragraph_sentence": " 2014\ub144 10\uc6d4 6\uc77c \uac80\ucc30\uc740 \uae40\uacbd\uc77c \uc744 \ubd88\uad6c\uc18d \uae30\uc18c\ud558\uc600\ub2e4. 2014\ub144 4\uc6d4 16\uc77c \uc138\uc6d4\ud638 \uce68\ubab0\uc0ac\uace0 \ub2f9\uc2dc \ud604\uc7a5\uc9c0\ud718\uad00(OSC)\uc774\uc5c8\ub358 \ubaa9\ud3ec\ud574\uc591\uacbd\ucc30\uc11c 123\uc815\uc7a5 \uae40\uacbd\uc77c\uc740 \uc138\uc6d4\ud638\uc640 \uad50\uc2e0\ud558\ub294 \ub4f1 \uc120\ub0b4 \uc2b9\uac1d \uc0c1\ud669\uc744 \ud655\uc778\ud558\uace0 \uc2b9\uac1d\uc5d0 \ub300\ud55c \ud1f4\uc120 \uc548\ub0b4\uc640 \uc720\ub3c4 \uc870\uce58\ub97c \ud588\uc5b4\uc57c \ud558\ub098, 123\uc815 \uc2b9\uc870\uc6d0 \ubc0f \ud574\uacbd \ud5ec\uae30\uc758 \uad6c\uc870 \ud65c\ub3d9\uc744 \uc9c0\ud718\ud558\uba74\uc11c \ud1f4\uc120 \uc720\ub3c4\ub97c \uc704\ud55c \uc544\ubb34\ub7f0 \uc870\uce58\ub97c \ucde8\ud558\uc9c0 \uc54a\uc558\ub2e4. \ub610\ud55c \uc11c\ud574\uc9c0\ubc29\ud574\uc591\uacbd\ucc30\uccad \ubc0f \ubaa9\ud3ec\ud574\uc591\uacbd\ucc30\uc11c \uc0c1\ud669\uc2e4\ub85c\ubd80\ud130 \uc218\ucc28\ub840\uc5d0 \uac78\uccd0 \uc2b9\uac1d\ub4e4\uc774 \ubc30\uc5d0\uc11c \ub6f0\uc5b4\ub0b4\ub9ac\ub3c4\ub85d \uace0\ud568\uc744 \uce58\uac70\ub098 123\uc815 \ub0b4 \ub300\uacf5\ub9c8\uc774\ud06c\ub97c \uc774\uc6a9\ud558\uc5ec \ud1f4\uc120\uc744 \uc720\ub3c4\ud558\ub77c\ub294 \uc9c0\uc2dc\ub97c \ubc1b\uc558\uc74c\uc5d0\ub3c4, \uac1c\uc778\uc801\uc778 \ud310\ub2e8 \ud558\uc5d0 \uc0c1\uae09\uc9c0\ud718\uad00\uc11c\uc758 \uc9c0\uc2dc\uc870\ucc28 \uc774\ud589\ud558\uc9c0 \uc54a\uc544 \uc2b9\uac1d\uc774 \uc0ac\ub9dd\uc5d0 \uc774\ub974\uac8c\ud558\uac70\ub098 \uc0c1\ud574\ub97c \uc785\ub3c4\ub85d \ubc29\uce58(\uc5c5\ubb34\uc0c1\uacfc\uc2e4\uce58\uc0ac, \uc5c5\ubb34\uc0c1\uacfc\uc2e4\uce58\uc0c1)\ud558\uc600\ub2e4. \uae40\uacbd\uc77c\uc740 2014\ub144 5\uc6d4 \uacbd \uc138\uc6d4\ud638 \uce68\ubab0 \uc0ac\uace0 \uc77c\uc778 4\uc6d4 16\uc77c\uc790 \ud568\uc815\uc77c\uc9c0\uc758 \uc77c\ubd80\ub97c \ub5bc\uc5b4\ub0b8 \ub2e4\uc74c, \ub9c8\uce58 \ub300\uacf5 \ub9c8\uc774\ud06c\ub97c \uc774\uc6a9\ud558\uc5ec \ud1f4\uc120\ubc29\uc1a1\uc744 \ud558\uace0 123\uc815 \uc2b9\uc870\uc6d0\uc5d0\uac8c \uc120\ub0b4 \uc9c4\uc785\uc744 \uc2dc\ub3c4\ud558\ub3c4\ub85d \uc9c0\uc2dc\ud55c \uc0ac\uc2e4\uc774 \uc788\ub294 \uac83\ucc98\ub7fc \ud568\uc815\uc77c\uc9c0\ub97c \ud5c8\uc704\ub85c \uc7ac\uc791\uc131\ud558\uc5ec \ucca8\ubd80\ud558\ub294 \ub4f1 \uc870\uc791(\ud5c8\uc704\uacf5\ubb38\uc11c\uc791\uc131, \ud5c8\uc704\uc791\uc131\uacf5\ubb38\uc11c\ud589\uc0ac, \uacf5\uc6a9\uc11c\ub958\uc190\uc0c1)\ud558\uc600\ub2e4. \uae40\uacbd\uc77c\uc740 \uac10\uc0ac\uc6d0 \uc870\uc0ac\uc640 \uac80\ucc30 \uc18c\ud658 \uc870\uc0ac\ub97c \uc55e\ub450\uace0 \uc2b9\uc870\uc6d0\ub4e4\uacfc \ub300\ucc45\ud68c\uc758\ub97c \uc5f4\uc5b4 123\uc815 \uc2b9\uc870\uc6d0\ub4e4\ub85c \ud558\uc5ec\uae08 \ud5c8\uc704\ub85c \uc9c4\uc220\ud558\ub3c4\ub85d \uc9c0\uc2dc\ud55c \uc0ac\uc2e4\ub3c4 \ud655\uc778\ub418\uc5c8\ub2e4.", "paragraph_answer": "2014\ub144 10\uc6d4 6\uc77c \uac80\ucc30\uc740 \uae40\uacbd\uc77c \uc744 \ubd88\uad6c\uc18d \uae30\uc18c\ud558\uc600\ub2e4. 2014\ub144 4\uc6d4 16\uc77c \uc138\uc6d4\ud638 \uce68\ubab0\uc0ac\uace0 \ub2f9\uc2dc \ud604\uc7a5\uc9c0\ud718\uad00(OSC)\uc774\uc5c8\ub358 \ubaa9\ud3ec\ud574\uc591\uacbd\ucc30\uc11c 123\uc815\uc7a5 \uae40\uacbd\uc77c\uc740 \uc138\uc6d4\ud638\uc640 \uad50\uc2e0\ud558\ub294 \ub4f1 \uc120\ub0b4 \uc2b9\uac1d \uc0c1\ud669\uc744 \ud655\uc778\ud558\uace0 \uc2b9\uac1d\uc5d0 \ub300\ud55c \ud1f4\uc120 \uc548\ub0b4\uc640 \uc720\ub3c4 \uc870\uce58\ub97c \ud588\uc5b4\uc57c \ud558\ub098, 123\uc815 \uc2b9\uc870\uc6d0 \ubc0f \ud574\uacbd \ud5ec\uae30\uc758 \uad6c\uc870 \ud65c\ub3d9\uc744 \uc9c0\ud718\ud558\uba74\uc11c \ud1f4\uc120 \uc720\ub3c4\ub97c \uc704\ud55c \uc544\ubb34\ub7f0 \uc870\uce58\ub97c \ucde8\ud558\uc9c0 \uc54a\uc558\ub2e4. \ub610\ud55c \uc11c\ud574\uc9c0\ubc29\ud574\uc591\uacbd\ucc30\uccad \ubc0f \ubaa9\ud3ec\ud574\uc591\uacbd\ucc30\uc11c \uc0c1\ud669\uc2e4\ub85c\ubd80\ud130 \uc218\ucc28\ub840\uc5d0 \uac78\uccd0 \uc2b9\uac1d\ub4e4\uc774 \ubc30\uc5d0\uc11c \ub6f0\uc5b4\ub0b4\ub9ac\ub3c4\ub85d \uace0\ud568\uc744 \uce58\uac70\ub098 123\uc815 \ub0b4 \ub300\uacf5\ub9c8\uc774\ud06c\ub97c \uc774\uc6a9\ud558\uc5ec \ud1f4\uc120\uc744 \uc720\ub3c4\ud558\ub77c\ub294 \uc9c0\uc2dc\ub97c \ubc1b\uc558\uc74c\uc5d0\ub3c4, \uac1c\uc778\uc801\uc778 \ud310\ub2e8 \ud558\uc5d0 \uc0c1\uae09\uc9c0\ud718\uad00\uc11c\uc758 \uc9c0\uc2dc\uc870\ucc28 \uc774\ud589\ud558\uc9c0 \uc54a\uc544 \uc2b9\uac1d\uc774 \uc0ac\ub9dd\uc5d0 \uc774\ub974\uac8c\ud558\uac70\ub098 \uc0c1\ud574\ub97c \uc785\ub3c4\ub85d \ubc29\uce58(\uc5c5\ubb34\uc0c1\uacfc\uc2e4\uce58\uc0ac, \uc5c5\ubb34\uc0c1\uacfc\uc2e4\uce58\uc0c1)\ud558\uc600\ub2e4. \uae40\uacbd\uc77c\uc740 2014\ub144 5\uc6d4 \uacbd \uc138\uc6d4\ud638 \uce68\ubab0 \uc0ac\uace0 \uc77c\uc778 4\uc6d4 16\uc77c\uc790 \ud568\uc815\uc77c\uc9c0\uc758 \uc77c\ubd80\ub97c \ub5bc\uc5b4\ub0b8 \ub2e4\uc74c, \ub9c8\uce58 \ub300\uacf5 \ub9c8\uc774\ud06c\ub97c \uc774\uc6a9\ud558\uc5ec \ud1f4\uc120\ubc29\uc1a1\uc744 \ud558\uace0 123\uc815 \uc2b9\uc870\uc6d0\uc5d0\uac8c \uc120\ub0b4 \uc9c4\uc785\uc744 \uc2dc\ub3c4\ud558\ub3c4\ub85d \uc9c0\uc2dc\ud55c \uc0ac\uc2e4\uc774 \uc788\ub294 \uac83\ucc98\ub7fc \ud568\uc815\uc77c\uc9c0\ub97c \ud5c8\uc704\ub85c \uc7ac\uc791\uc131\ud558\uc5ec \ucca8\ubd80\ud558\ub294 \ub4f1 \uc870\uc791(\ud5c8\uc704\uacf5\ubb38\uc11c\uc791\uc131, \ud5c8\uc704\uc791\uc131\uacf5\ubb38\uc11c\ud589\uc0ac, \uacf5\uc6a9\uc11c\ub958\uc190\uc0c1)\ud558\uc600\ub2e4. \uae40\uacbd\uc77c\uc740 \uac10\uc0ac\uc6d0 \uc870\uc0ac\uc640 \uac80\ucc30 \uc18c\ud658 \uc870\uc0ac\ub97c \uc55e\ub450\uace0 \uc2b9\uc870\uc6d0\ub4e4\uacfc \ub300\ucc45\ud68c\uc758\ub97c \uc5f4\uc5b4 123\uc815 \uc2b9\uc870\uc6d0\ub4e4\ub85c \ud558\uc5ec\uae08 \ud5c8\uc704\ub85c \uc9c4\uc220\ud558\ub3c4\ub85d \uc9c0\uc2dc\ud55c \uc0ac\uc2e4\ub3c4 \ud655\uc778\ub418\uc5c8\ub2e4.", "sentence_answer": "2014\ub144 10\uc6d4 6\uc77c \uac80\ucc30\uc740 \uae40\uacbd\uc77c \uc744 \ubd88\uad6c\uc18d \uae30\uc18c\ud558\uc600\ub2e4.", "paragraph_id": "6536361-12-0", "paragraph_question": "question: \uc138\uc6d4\ud638 \ucc38\uc0ac \ub2f9\uc2dc \ud604\uc7a5\uc9c0\ud718\uad00\uc758 \uc774\ub984\uc740?, context: 2014\ub144 10\uc6d4 6\uc77c \uac80\ucc30\uc740 \uae40\uacbd\uc77c\uc744 \ubd88\uad6c\uc18d \uae30\uc18c\ud558\uc600\ub2e4. 2014\ub144 4\uc6d4 16\uc77c \uc138\uc6d4\ud638 \uce68\ubab0\uc0ac\uace0 \ub2f9\uc2dc \ud604\uc7a5\uc9c0\ud718\uad00(OSC)\uc774\uc5c8\ub358 \ubaa9\ud3ec\ud574\uc591\uacbd\ucc30\uc11c 123\uc815\uc7a5 \uae40\uacbd\uc77c\uc740 \uc138\uc6d4\ud638\uc640 \uad50\uc2e0\ud558\ub294 \ub4f1 \uc120\ub0b4 \uc2b9\uac1d \uc0c1\ud669\uc744 \ud655\uc778\ud558\uace0 \uc2b9\uac1d\uc5d0 \ub300\ud55c \ud1f4\uc120 \uc548\ub0b4\uc640 \uc720\ub3c4 \uc870\uce58\ub97c \ud588\uc5b4\uc57c \ud558\ub098, 123\uc815 \uc2b9\uc870\uc6d0 \ubc0f \ud574\uacbd \ud5ec\uae30\uc758 \uad6c\uc870 \ud65c\ub3d9\uc744 \uc9c0\ud718\ud558\uba74\uc11c \ud1f4\uc120 \uc720\ub3c4\ub97c \uc704\ud55c \uc544\ubb34\ub7f0 \uc870\uce58\ub97c \ucde8\ud558\uc9c0 \uc54a\uc558\ub2e4. \ub610\ud55c \uc11c\ud574\uc9c0\ubc29\ud574\uc591\uacbd\ucc30\uccad \ubc0f \ubaa9\ud3ec\ud574\uc591\uacbd\ucc30\uc11c \uc0c1\ud669\uc2e4\ub85c\ubd80\ud130 \uc218\ucc28\ub840\uc5d0 \uac78\uccd0 \uc2b9\uac1d\ub4e4\uc774 \ubc30\uc5d0\uc11c \ub6f0\uc5b4\ub0b4\ub9ac\ub3c4\ub85d \uace0\ud568\uc744 \uce58\uac70\ub098 123\uc815 \ub0b4 \ub300\uacf5\ub9c8\uc774\ud06c\ub97c \uc774\uc6a9\ud558\uc5ec \ud1f4\uc120\uc744 \uc720\ub3c4\ud558\ub77c\ub294 \uc9c0\uc2dc\ub97c \ubc1b\uc558\uc74c\uc5d0\ub3c4, \uac1c\uc778\uc801\uc778 \ud310\ub2e8 \ud558\uc5d0 \uc0c1\uae09\uc9c0\ud718\uad00\uc11c\uc758 \uc9c0\uc2dc\uc870\ucc28 \uc774\ud589\ud558\uc9c0 \uc54a\uc544 \uc2b9\uac1d\uc774 \uc0ac\ub9dd\uc5d0 \uc774\ub974\uac8c\ud558\uac70\ub098 \uc0c1\ud574\ub97c \uc785\ub3c4\ub85d \ubc29\uce58(\uc5c5\ubb34\uc0c1\uacfc\uc2e4\uce58\uc0ac, \uc5c5\ubb34\uc0c1\uacfc\uc2e4\uce58\uc0c1)\ud558\uc600\ub2e4. \uae40\uacbd\uc77c\uc740 2014\ub144 5\uc6d4 \uacbd \uc138\uc6d4\ud638 \uce68\ubab0 \uc0ac\uace0 \uc77c\uc778 4\uc6d4 16\uc77c\uc790 \ud568\uc815\uc77c\uc9c0\uc758 \uc77c\ubd80\ub97c \ub5bc\uc5b4\ub0b8 \ub2e4\uc74c, \ub9c8\uce58 \ub300\uacf5 \ub9c8\uc774\ud06c\ub97c \uc774\uc6a9\ud558\uc5ec \ud1f4\uc120\ubc29\uc1a1\uc744 \ud558\uace0 123\uc815 \uc2b9\uc870\uc6d0\uc5d0\uac8c \uc120\ub0b4 \uc9c4\uc785\uc744 \uc2dc\ub3c4\ud558\ub3c4\ub85d \uc9c0\uc2dc\ud55c \uc0ac\uc2e4\uc774 \uc788\ub294 \uac83\ucc98\ub7fc \ud568\uc815\uc77c\uc9c0\ub97c \ud5c8\uc704\ub85c \uc7ac\uc791\uc131\ud558\uc5ec \ucca8\ubd80\ud558\ub294 \ub4f1 \uc870\uc791(\ud5c8\uc704\uacf5\ubb38\uc11c\uc791\uc131, \ud5c8\uc704\uc791\uc131\uacf5\ubb38\uc11c\ud589\uc0ac, \uacf5\uc6a9\uc11c\ub958\uc190\uc0c1)\ud558\uc600\ub2e4. \uae40\uacbd\uc77c\uc740 \uac10\uc0ac\uc6d0 \uc870\uc0ac\uc640 \uac80\ucc30 \uc18c\ud658 \uc870\uc0ac\ub97c \uc55e\ub450\uace0 \uc2b9\uc870\uc6d0\ub4e4\uacfc \ub300\ucc45\ud68c\uc758\ub97c \uc5f4\uc5b4 123\uc815 \uc2b9\uc870\uc6d0\ub4e4\ub85c \ud558\uc5ec\uae08 \ud5c8\uc704\ub85c \uc9c4\uc220\ud558\ub3c4\ub85d \uc9c0\uc2dc\ud55c \uc0ac\uc2e4\ub3c4 \ud655\uc778\ub418\uc5c8\ub2e4."} {"question": "\ud574\ubbf8\ub294 \uc870\uc120 \ucd08\uae30 \ubb34\uc5c7\uc774 \uc704\uce58\ud558\uace0 \uc788\uc5c8\ub294\uac00?", "paragraph": "\ud574\ubbf8\ub294 \uc870\uc120 \ucd08\uae30 \ucda9\uccad \ubcd1\ub9c8\uc808\ub3c4\uc0ac\uc601\uc774 \uc704\uce58\ud55c \uacf3\uc774\uc600\ub2e4. \uc870\uc120 \uc911\uae30\uc5d0\ub294 1651\ub144 \ubcd1\ub9c8\uc808\ub3c4\uc0ac\uc601\uc774 \uccad\uc8fc\ub85c \uc62e\uaca8\uc9c0\uace0 \ud574\ubbf8\ub294 \ud604\uc73c\ub85c \ucd95\uc18c\ub418\uc9c0\ub9cc, \uc5ec\uc804\ud788 1,400~1,500\uc5ec \uba85\uc758 \uad70\uc0ac\uac00 \uc8fc\ub454\ud55c \uc9c4\uc601\uc774 \uc874\uc7ac\ud558\ub294 \uad70\uc0ac\uc801 \uc694\ucda9\uc9c0\uc600\ub2e4. \uad70\uc0ac\ub97c \uac70\ub290\ub9b0 \ubb34\uad00 \uc601\uc7a5\uc740 \ud574\ubbf8 \ud604\uac10\uc744 \uacb8\ud558\uc600\uace0, \uc774\uc5d0 \ud574\ubbf8 \ud604\uac10\uc740 \ucda9\uccad\uc88c\ub3c4\uc758 \ub0b4\ud3ec \uc9c0\ubc29 \ud574\uc548\uc218\ube44 \uba85\ubaa9\uc73c\ub85c \uad6d\uc0ac\ubc94\uc744 \ucc98\ud615\ud560 \uc218 \uc788\ub294 \uad8c\ud55c\uc744 \uac16\uac8c \ub41c\ub2e4. \ub2f9\uc2dc \ud574\ubbf8 \uad00\uc544\uc758 \ub2f4\ub2f9 \uc9c0\uc5ed\uc740 \ucda9\uccad\ub3c4\ub294 \ubb3c\ub860\uc774\uace0 \uacbd\uae30\ub3c4 \ud3c9\ud0dd\uc5d0 \uc774\ub974\ub294 \ub113\uc740 \uc9c0\uc5ed\uc744 \uc544\uc6b8\ub800\ub2e4. \uc774\uc5d0 \ub2f4\ub2f9 \uc9c0\uc5ed\uc758 \uc2e0\uc790\ub4e4\uc774 \uccb4\ud3ec\ub418\uba74 \ubaa8\ub450 \ud574\ubbf8\uc74d\uc131\uc73c\ub85c \ub04c\ub824\uc624\uac8c \ub418\uc5c8\ub2e4. \uc774\ub4e4\uc774 \ud574\ubbf8\uc74d\uc131\uc5d0\uc11c \uac16\uc740 \uace0\ubb38\uc744 \ub2f9\ud55c \ud6c4 \ub9c8\uc9c0\ub9c9\uc73c\ub85c \uc8fd\uc74c\uc744 \ub9de\uc73c\ub7ec \uac00\ub294 \uacf3\uc774 \ud574\ubbf8\uc74d\uc131 \uc11c\ubb38 \ubc16\uacfc \ubc14\ub85c \uc774\uacf3 \ud574\ubbf8\uc21c\uad50\uc131\uc9c0\uc758 \uc7a5\uc18c\uc600\ub2e4.", "answer": "\ucda9\uccad \ubcd1\ub9c8\uc808\ub3c4\uc0ac\uc601", "sentence": "\ud574\ubbf8\ub294 \uc870\uc120 \ucd08\uae30 \ucda9\uccad \ubcd1\ub9c8\uc808\ub3c4\uc0ac\uc601 \uc774 \uc704\uce58\ud55c \uacf3\uc774\uc600\ub2e4.", "paragraph_sentence": " \ud574\ubbf8\ub294 \uc870\uc120 \ucd08\uae30 \ucda9\uccad \ubcd1\ub9c8\uc808\ub3c4\uc0ac\uc601 \uc774 \uc704\uce58\ud55c \uacf3\uc774\uc600\ub2e4. \uc870\uc120 \uc911\uae30\uc5d0\ub294 1651\ub144 \ubcd1\ub9c8\uc808\ub3c4\uc0ac\uc601\uc774 \uccad\uc8fc\ub85c \uc62e\uaca8\uc9c0\uace0 \ud574\ubbf8\ub294 \ud604\uc73c\ub85c \ucd95\uc18c\ub418\uc9c0\ub9cc, \uc5ec\uc804\ud788 1,400~1,500\uc5ec \uba85\uc758 \uad70\uc0ac\uac00 \uc8fc\ub454\ud55c \uc9c4\uc601\uc774 \uc874\uc7ac\ud558\ub294 \uad70\uc0ac\uc801 \uc694\ucda9\uc9c0\uc600\ub2e4. \uad70\uc0ac\ub97c \uac70\ub290\ub9b0 \ubb34\uad00 \uc601\uc7a5\uc740 \ud574\ubbf8 \ud604\uac10\uc744 \uacb8\ud558\uc600\uace0, \uc774\uc5d0 \ud574\ubbf8 \ud604\uac10\uc740 \ucda9\uccad\uc88c\ub3c4\uc758 \ub0b4\ud3ec \uc9c0\ubc29 \ud574\uc548\uc218\ube44 \uba85\ubaa9\uc73c\ub85c \uad6d\uc0ac\ubc94\uc744 \ucc98\ud615\ud560 \uc218 \uc788\ub294 \uad8c\ud55c\uc744 \uac16\uac8c \ub41c\ub2e4. \ub2f9\uc2dc \ud574\ubbf8 \uad00\uc544\uc758 \ub2f4\ub2f9 \uc9c0\uc5ed\uc740 \ucda9\uccad\ub3c4\ub294 \ubb3c\ub860\uc774\uace0 \uacbd\uae30\ub3c4 \ud3c9\ud0dd\uc5d0 \uc774\ub974\ub294 \ub113\uc740 \uc9c0\uc5ed\uc744 \uc544\uc6b8\ub800\ub2e4. \uc774\uc5d0 \ub2f4\ub2f9 \uc9c0\uc5ed\uc758 \uc2e0\uc790\ub4e4\uc774 \uccb4\ud3ec\ub418\uba74 \ubaa8\ub450 \ud574\ubbf8\uc74d\uc131\uc73c\ub85c \ub04c\ub824\uc624\uac8c \ub418\uc5c8\ub2e4. \uc774\ub4e4\uc774 \ud574\ubbf8\uc74d\uc131\uc5d0\uc11c \uac16\uc740 \uace0\ubb38\uc744 \ub2f9\ud55c \ud6c4 \ub9c8\uc9c0\ub9c9\uc73c\ub85c \uc8fd\uc74c\uc744 \ub9de\uc73c\ub7ec \uac00\ub294 \uacf3\uc774 \ud574\ubbf8\uc74d\uc131 \uc11c\ubb38 \ubc16\uacfc \ubc14\ub85c \uc774\uacf3 \ud574\ubbf8\uc21c\uad50\uc131\uc9c0\uc758 \uc7a5\uc18c\uc600\ub2e4.", "paragraph_answer": "\ud574\ubbf8\ub294 \uc870\uc120 \ucd08\uae30 \ucda9\uccad \ubcd1\ub9c8\uc808\ub3c4\uc0ac\uc601 \uc774 \uc704\uce58\ud55c \uacf3\uc774\uc600\ub2e4. \uc870\uc120 \uc911\uae30\uc5d0\ub294 1651\ub144 \ubcd1\ub9c8\uc808\ub3c4\uc0ac\uc601\uc774 \uccad\uc8fc\ub85c \uc62e\uaca8\uc9c0\uace0 \ud574\ubbf8\ub294 \ud604\uc73c\ub85c \ucd95\uc18c\ub418\uc9c0\ub9cc, \uc5ec\uc804\ud788 1,400~1,500\uc5ec \uba85\uc758 \uad70\uc0ac\uac00 \uc8fc\ub454\ud55c \uc9c4\uc601\uc774 \uc874\uc7ac\ud558\ub294 \uad70\uc0ac\uc801 \uc694\ucda9\uc9c0\uc600\ub2e4. \uad70\uc0ac\ub97c \uac70\ub290\ub9b0 \ubb34\uad00 \uc601\uc7a5\uc740 \ud574\ubbf8 \ud604\uac10\uc744 \uacb8\ud558\uc600\uace0, \uc774\uc5d0 \ud574\ubbf8 \ud604\uac10\uc740 \ucda9\uccad\uc88c\ub3c4\uc758 \ub0b4\ud3ec \uc9c0\ubc29 \ud574\uc548\uc218\ube44 \uba85\ubaa9\uc73c\ub85c \uad6d\uc0ac\ubc94\uc744 \ucc98\ud615\ud560 \uc218 \uc788\ub294 \uad8c\ud55c\uc744 \uac16\uac8c \ub41c\ub2e4. \ub2f9\uc2dc \ud574\ubbf8 \uad00\uc544\uc758 \ub2f4\ub2f9 \uc9c0\uc5ed\uc740 \ucda9\uccad\ub3c4\ub294 \ubb3c\ub860\uc774\uace0 \uacbd\uae30\ub3c4 \ud3c9\ud0dd\uc5d0 \uc774\ub974\ub294 \ub113\uc740 \uc9c0\uc5ed\uc744 \uc544\uc6b8\ub800\ub2e4. \uc774\uc5d0 \ub2f4\ub2f9 \uc9c0\uc5ed\uc758 \uc2e0\uc790\ub4e4\uc774 \uccb4\ud3ec\ub418\uba74 \ubaa8\ub450 \ud574\ubbf8\uc74d\uc131\uc73c\ub85c \ub04c\ub824\uc624\uac8c \ub418\uc5c8\ub2e4. \uc774\ub4e4\uc774 \ud574\ubbf8\uc74d\uc131\uc5d0\uc11c \uac16\uc740 \uace0\ubb38\uc744 \ub2f9\ud55c \ud6c4 \ub9c8\uc9c0\ub9c9\uc73c\ub85c \uc8fd\uc74c\uc744 \ub9de\uc73c\ub7ec \uac00\ub294 \uacf3\uc774 \ud574\ubbf8\uc74d\uc131 \uc11c\ubb38 \ubc16\uacfc \ubc14\ub85c \uc774\uacf3 \ud574\ubbf8\uc21c\uad50\uc131\uc9c0\uc758 \uc7a5\uc18c\uc600\ub2e4.", "sentence_answer": "\ud574\ubbf8\ub294 \uc870\uc120 \ucd08\uae30 \ucda9\uccad \ubcd1\ub9c8\uc808\ub3c4\uc0ac\uc601 \uc774 \uc704\uce58\ud55c \uacf3\uc774\uc600\ub2e4.", "paragraph_id": "6570705-0-0", "paragraph_question": "question: \ud574\ubbf8\ub294 \uc870\uc120 \ucd08\uae30 \ubb34\uc5c7\uc774 \uc704\uce58\ud558\uace0 \uc788\uc5c8\ub294\uac00?, context: \ud574\ubbf8\ub294 \uc870\uc120 \ucd08\uae30 \ucda9\uccad \ubcd1\ub9c8\uc808\ub3c4\uc0ac\uc601\uc774 \uc704\uce58\ud55c \uacf3\uc774\uc600\ub2e4. \uc870\uc120 \uc911\uae30\uc5d0\ub294 1651\ub144 \ubcd1\ub9c8\uc808\ub3c4\uc0ac\uc601\uc774 \uccad\uc8fc\ub85c \uc62e\uaca8\uc9c0\uace0 \ud574\ubbf8\ub294 \ud604\uc73c\ub85c \ucd95\uc18c\ub418\uc9c0\ub9cc, \uc5ec\uc804\ud788 1,400~1,500\uc5ec \uba85\uc758 \uad70\uc0ac\uac00 \uc8fc\ub454\ud55c \uc9c4\uc601\uc774 \uc874\uc7ac\ud558\ub294 \uad70\uc0ac\uc801 \uc694\ucda9\uc9c0\uc600\ub2e4. \uad70\uc0ac\ub97c \uac70\ub290\ub9b0 \ubb34\uad00 \uc601\uc7a5\uc740 \ud574\ubbf8 \ud604\uac10\uc744 \uacb8\ud558\uc600\uace0, \uc774\uc5d0 \ud574\ubbf8 \ud604\uac10\uc740 \ucda9\uccad\uc88c\ub3c4\uc758 \ub0b4\ud3ec \uc9c0\ubc29 \ud574\uc548\uc218\ube44 \uba85\ubaa9\uc73c\ub85c \uad6d\uc0ac\ubc94\uc744 \ucc98\ud615\ud560 \uc218 \uc788\ub294 \uad8c\ud55c\uc744 \uac16\uac8c \ub41c\ub2e4. \ub2f9\uc2dc \ud574\ubbf8 \uad00\uc544\uc758 \ub2f4\ub2f9 \uc9c0\uc5ed\uc740 \ucda9\uccad\ub3c4\ub294 \ubb3c\ub860\uc774\uace0 \uacbd\uae30\ub3c4 \ud3c9\ud0dd\uc5d0 \uc774\ub974\ub294 \ub113\uc740 \uc9c0\uc5ed\uc744 \uc544\uc6b8\ub800\ub2e4. \uc774\uc5d0 \ub2f4\ub2f9 \uc9c0\uc5ed\uc758 \uc2e0\uc790\ub4e4\uc774 \uccb4\ud3ec\ub418\uba74 \ubaa8\ub450 \ud574\ubbf8\uc74d\uc131\uc73c\ub85c \ub04c\ub824\uc624\uac8c \ub418\uc5c8\ub2e4. \uc774\ub4e4\uc774 \ud574\ubbf8\uc74d\uc131\uc5d0\uc11c \uac16\uc740 \uace0\ubb38\uc744 \ub2f9\ud55c \ud6c4 \ub9c8\uc9c0\ub9c9\uc73c\ub85c \uc8fd\uc74c\uc744 \ub9de\uc73c\ub7ec \uac00\ub294 \uacf3\uc774 \ud574\ubbf8\uc74d\uc131 \uc11c\ubb38 \ubc16\uacfc \ubc14\ub85c \uc774\uacf3 \ud574\ubbf8\uc21c\uad50\uc131\uc9c0\uc758 \uc7a5\uc18c\uc600\ub2e4."} {"question": "\uc624\ud638\ub3c4\uc655\uc790\ub294 \uc5b4\ub290 \ub098\ub77c\uc5d0\uc11c \uc654\ub294\uac00?", "paragraph": "\ubd80\ub808\uc4f0(\u6b66\u70c8) \ub54c \uc624\uc624\ubb34\ub77c\uce58\uac00 \ub41c \uac00\ub124\ub9c8\ub85c(\u91d1\u6751)\uc758 \uc2dc\ub300\uc5d0 \uc624\ud1a0\ubaa8 \uc528\ub294 \uc804\uc131\uae30\ub97c \ub9de\uc774\ud558\uc600\uace0, \uadf8 \ub4a4 \uac8c\uc774\ud0c0\uc774(\u7d99\u4f53) \u30fb \uc548\uce78(\u5b89\u9591) \u30fb \uc13c\uce74(\u5ba3\u5316) \u30fb \uae34\uba54\uc774(\u6b3d\u660e)\uc5d0 \uc774\ub974\ub294 \ub2e4\uc12f \uc624\ud0a4\ubbf8\uc758 \uc870\uc815\uc5d0\uc11c \uc624\uc624\ubb34\ub77c\uce58\ub97c \ub9e1\uc558\ub2e4. \ud2b9\ud788 \uac00\ub124\ub9c8\ub85c\ub294 \uc5d0\uce58\uc820 \uad6d(\u8d8a\u524d\u56fd)\uc5d0\uc11c \uc624\ud638\ub3c4(\u5f1f) \uc655\uc790\ub97c \ub9de\uc774\ud574 \uc640\uc11c \uc0c8\ub85c\uc6b4 \uc624\ud0a4\ubbf8\ub85c \uc639\ub9bd\ud558\ub294 \uacf5\uc744 \uc138\uc6b0\uace0 \uc57c\ub9c8\ud1a0 \uc655\uad8c \ub0b4\uc5d0\uc11c\ub3c4 \ud655\uace0\ud55c \uc9c0\uc704\ub97c \uad6c\ucd95\ud558\uc600\ub2e4. \uadf8\ub7ec\ub098 \ubc31\uc81c(\u767e\u6fdf)\uc758 \uac00\uc57c \uc9c4\ucd9c \uacfc\uc815\uc5d0\uc11c \uc784\ub098 4\ud604\uc774 \ubc31\uc81c\uc5d0 \ub118\uc5b4\uac04 \ub4a4 \uac19\uc740 \uc624\uc624\ubb34\ub77c\uce58\uc600\ub358 \ubaa8\ub178\ub178\ubca0 \uc528\uc758 \ucc38\uc18c\ub97c \ub2f9\ud574 \uc2e4\uac01\ud55c\ub2e4 \uc14b\uc4f0 \uad6d \uc2a4\ubbf8\uc694\uc2dc \uad70(\u4f4f\u5409\u90e1, \uc9c0\uae08\uc758 \uc624\uc0ac\uce74 \uc2dc \uc2a4\ubbf8\uc694\uc2dc \uad6c \ubbf8\uce74\ub3c4\uc988\uce74 \uc0b0\u5e1d\u585a\u5c71)\uc758 \uc800\ud0dd\uc73c\ub85c \ubb3c\ub7ec\ub0ac\ub2e4. \uadf8 \ub4a4 \uc18c\uac00 \uc528(\u8607\u6211\u6c0f)\uc640 \ubaa8\ub178\ub178\ubca0 \uc528(\u7269\u90e8\u6c0f)\uc758 \ub300\ub9bd\uc774 \uc2dc\uc791\ub41c\ub2e4.", "answer": "\uc5d0\uce58\uc820", "sentence": "\ud2b9\ud788 \uac00\ub124\ub9c8\ub85c\ub294 \uc5d0\uce58\uc820 \uad6d(\u8d8a\u524d\u56fd)\uc5d0\uc11c \uc624\ud638\ub3c4(\u5f1f) \uc655\uc790\ub97c \ub9de\uc774\ud574 \uc640\uc11c \uc0c8\ub85c\uc6b4 \uc624\ud0a4\ubbf8\ub85c \uc639\ub9bd\ud558\ub294 \uacf5\uc744 \uc138\uc6b0\uace0 \uc57c\ub9c8\ud1a0 \uc655\uad8c \ub0b4\uc5d0\uc11c\ub3c4 \ud655\uace0\ud55c \uc9c0\uc704\ub97c \uad6c\ucd95\ud558\uc600\ub2e4.", "paragraph_sentence": "\ubd80\ub808\uc4f0(\u6b66\u70c8) \ub54c \uc624\uc624\ubb34\ub77c\uce58\uac00 \ub41c \uac00\ub124\ub9c8\ub85c(\u91d1\u6751)\uc758 \uc2dc\ub300\uc5d0 \uc624\ud1a0\ubaa8 \uc528\ub294 \uc804\uc131\uae30\ub97c \ub9de\uc774\ud558\uc600\uace0, \uadf8 \ub4a4 \uac8c\uc774\ud0c0\uc774(\u7d99\u4f53) \u30fb \uc548\uce78(\u5b89\u9591) \u30fb \uc13c\uce74(\u5ba3\u5316) \u30fb \uae34\uba54\uc774(\u6b3d\u660e)\uc5d0 \uc774\ub974\ub294 \ub2e4\uc12f \uc624\ud0a4\ubbf8\uc758 \uc870\uc815\uc5d0\uc11c \uc624\uc624\ubb34\ub77c\uce58\ub97c \ub9e1\uc558\ub2e4. \ud2b9\ud788 \uac00\ub124\ub9c8\ub85c\ub294 \uc5d0\uce58\uc820 \uad6d(\u8d8a\u524d\u56fd)\uc5d0\uc11c \uc624\ud638\ub3c4(\u5f1f) \uc655\uc790\ub97c \ub9de\uc774\ud574 \uc640\uc11c \uc0c8\ub85c\uc6b4 \uc624\ud0a4\ubbf8\ub85c \uc639\ub9bd\ud558\ub294 \uacf5\uc744 \uc138\uc6b0\uace0 \uc57c\ub9c8\ud1a0 \uc655\uad8c \ub0b4\uc5d0\uc11c\ub3c4 \ud655\uace0\ud55c \uc9c0\uc704\ub97c \uad6c\ucd95\ud558\uc600\ub2e4. \uadf8\ub7ec\ub098 \ubc31\uc81c(\u767e\u6fdf)\uc758 \uac00\uc57c \uc9c4\ucd9c \uacfc\uc815\uc5d0\uc11c \uc784\ub098 4\ud604\uc774 \ubc31\uc81c\uc5d0 \ub118\uc5b4\uac04 \ub4a4 \uac19\uc740 \uc624\uc624\ubb34\ub77c\uce58\uc600\ub358 \ubaa8\ub178\ub178\ubca0 \uc528\uc758 \ucc38\uc18c\ub97c \ub2f9\ud574 \uc2e4\uac01\ud55c\ub2e4 \uc14b\uc4f0 \uad6d \uc2a4\ubbf8\uc694\uc2dc \uad70(\u4f4f\u5409\u90e1, \uc9c0\uae08\uc758 \uc624\uc0ac\uce74 \uc2dc \uc2a4\ubbf8\uc694\uc2dc \uad6c \ubbf8\uce74\ub3c4\uc988\uce74 \uc0b0\u5e1d\u585a\u5c71)\uc758 \uc800\ud0dd\uc73c\ub85c \ubb3c\ub7ec\ub0ac\ub2e4. \uadf8 \ub4a4 \uc18c\uac00 \uc528(\u8607\u6211\u6c0f)\uc640 \ubaa8\ub178\ub178\ubca0 \uc528(\u7269\u90e8\u6c0f)\uc758 \ub300\ub9bd\uc774 \uc2dc\uc791\ub41c\ub2e4.", "paragraph_answer": "\ubd80\ub808\uc4f0(\u6b66\u70c8) \ub54c \uc624\uc624\ubb34\ub77c\uce58\uac00 \ub41c \uac00\ub124\ub9c8\ub85c(\u91d1\u6751)\uc758 \uc2dc\ub300\uc5d0 \uc624\ud1a0\ubaa8 \uc528\ub294 \uc804\uc131\uae30\ub97c \ub9de\uc774\ud558\uc600\uace0, \uadf8 \ub4a4 \uac8c\uc774\ud0c0\uc774(\u7d99\u4f53) \u30fb \uc548\uce78(\u5b89\u9591) \u30fb \uc13c\uce74(\u5ba3\u5316) \u30fb \uae34\uba54\uc774(\u6b3d\u660e)\uc5d0 \uc774\ub974\ub294 \ub2e4\uc12f \uc624\ud0a4\ubbf8\uc758 \uc870\uc815\uc5d0\uc11c \uc624\uc624\ubb34\ub77c\uce58\ub97c \ub9e1\uc558\ub2e4. \ud2b9\ud788 \uac00\ub124\ub9c8\ub85c\ub294 \uc5d0\uce58\uc820 \uad6d(\u8d8a\u524d\u56fd)\uc5d0\uc11c \uc624\ud638\ub3c4(\u5f1f) \uc655\uc790\ub97c \ub9de\uc774\ud574 \uc640\uc11c \uc0c8\ub85c\uc6b4 \uc624\ud0a4\ubbf8\ub85c \uc639\ub9bd\ud558\ub294 \uacf5\uc744 \uc138\uc6b0\uace0 \uc57c\ub9c8\ud1a0 \uc655\uad8c \ub0b4\uc5d0\uc11c\ub3c4 \ud655\uace0\ud55c \uc9c0\uc704\ub97c \uad6c\ucd95\ud558\uc600\ub2e4. \uadf8\ub7ec\ub098 \ubc31\uc81c(\u767e\u6fdf)\uc758 \uac00\uc57c \uc9c4\ucd9c \uacfc\uc815\uc5d0\uc11c \uc784\ub098 4\ud604\uc774 \ubc31\uc81c\uc5d0 \ub118\uc5b4\uac04 \ub4a4 \uac19\uc740 \uc624\uc624\ubb34\ub77c\uce58\uc600\ub358 \ubaa8\ub178\ub178\ubca0 \uc528\uc758 \ucc38\uc18c\ub97c \ub2f9\ud574 \uc2e4\uac01\ud55c\ub2e4 \uc14b\uc4f0 \uad6d \uc2a4\ubbf8\uc694\uc2dc \uad70(\u4f4f\u5409\u90e1, \uc9c0\uae08\uc758 \uc624\uc0ac\uce74 \uc2dc \uc2a4\ubbf8\uc694\uc2dc \uad6c \ubbf8\uce74\ub3c4\uc988\uce74 \uc0b0\u5e1d\u585a\u5c71)\uc758 \uc800\ud0dd\uc73c\ub85c \ubb3c\ub7ec\ub0ac\ub2e4. \uadf8 \ub4a4 \uc18c\uac00 \uc528(\u8607\u6211\u6c0f)\uc640 \ubaa8\ub178\ub178\ubca0 \uc528(\u7269\u90e8\u6c0f)\uc758 \ub300\ub9bd\uc774 \uc2dc\uc791\ub41c\ub2e4.", "sentence_answer": "\ud2b9\ud788 \uac00\ub124\ub9c8\ub85c\ub294 \uc5d0\uce58\uc820 \uad6d(\u8d8a\u524d\u56fd)\uc5d0\uc11c \uc624\ud638\ub3c4(\u5f1f) \uc655\uc790\ub97c \ub9de\uc774\ud574 \uc640\uc11c \uc0c8\ub85c\uc6b4 \uc624\ud0a4\ubbf8\ub85c \uc639\ub9bd\ud558\ub294 \uacf5\uc744 \uc138\uc6b0\uace0 \uc57c\ub9c8\ud1a0 \uc655\uad8c \ub0b4\uc5d0\uc11c\ub3c4 \ud655\uace0\ud55c \uc9c0\uc704\ub97c \uad6c\ucd95\ud558\uc600\ub2e4.", "paragraph_id": "6657437-0-0", "paragraph_question": "question: \uc624\ud638\ub3c4\uc655\uc790\ub294 \uc5b4\ub290 \ub098\ub77c\uc5d0\uc11c \uc654\ub294\uac00?, context: \ubd80\ub808\uc4f0(\u6b66\u70c8) \ub54c \uc624\uc624\ubb34\ub77c\uce58\uac00 \ub41c \uac00\ub124\ub9c8\ub85c(\u91d1\u6751)\uc758 \uc2dc\ub300\uc5d0 \uc624\ud1a0\ubaa8 \uc528\ub294 \uc804\uc131\uae30\ub97c \ub9de\uc774\ud558\uc600\uace0, \uadf8 \ub4a4 \uac8c\uc774\ud0c0\uc774(\u7d99\u4f53) \u30fb \uc548\uce78(\u5b89\u9591) \u30fb \uc13c\uce74(\u5ba3\u5316) \u30fb \uae34\uba54\uc774(\u6b3d\u660e)\uc5d0 \uc774\ub974\ub294 \ub2e4\uc12f \uc624\ud0a4\ubbf8\uc758 \uc870\uc815\uc5d0\uc11c \uc624\uc624\ubb34\ub77c\uce58\ub97c \ub9e1\uc558\ub2e4. \ud2b9\ud788 \uac00\ub124\ub9c8\ub85c\ub294 \uc5d0\uce58\uc820 \uad6d(\u8d8a\u524d\u56fd)\uc5d0\uc11c \uc624\ud638\ub3c4(\u5f1f) \uc655\uc790\ub97c \ub9de\uc774\ud574 \uc640\uc11c \uc0c8\ub85c\uc6b4 \uc624\ud0a4\ubbf8\ub85c \uc639\ub9bd\ud558\ub294 \uacf5\uc744 \uc138\uc6b0\uace0 \uc57c\ub9c8\ud1a0 \uc655\uad8c \ub0b4\uc5d0\uc11c\ub3c4 \ud655\uace0\ud55c \uc9c0\uc704\ub97c \uad6c\ucd95\ud558\uc600\ub2e4. \uadf8\ub7ec\ub098 \ubc31\uc81c(\u767e\u6fdf)\uc758 \uac00\uc57c \uc9c4\ucd9c \uacfc\uc815\uc5d0\uc11c \uc784\ub098 4\ud604\uc774 \ubc31\uc81c\uc5d0 \ub118\uc5b4\uac04 \ub4a4 \uac19\uc740 \uc624\uc624\ubb34\ub77c\uce58\uc600\ub358 \ubaa8\ub178\ub178\ubca0 \uc528\uc758 \ucc38\uc18c\ub97c \ub2f9\ud574 \uc2e4\uac01\ud55c\ub2e4 \uc14b\uc4f0 \uad6d \uc2a4\ubbf8\uc694\uc2dc \uad70(\u4f4f\u5409\u90e1, \uc9c0\uae08\uc758 \uc624\uc0ac\uce74 \uc2dc \uc2a4\ubbf8\uc694\uc2dc \uad6c \ubbf8\uce74\ub3c4\uc988\uce74 \uc0b0\u5e1d\u585a\u5c71)\uc758 \uc800\ud0dd\uc73c\ub85c \ubb3c\ub7ec\ub0ac\ub2e4. \uadf8 \ub4a4 \uc18c\uac00 \uc528(\u8607\u6211\u6c0f)\uc640 \ubaa8\ub178\ub178\ubca0 \uc528(\u7269\u90e8\u6c0f)\uc758 \ub300\ub9bd\uc774 \uc2dc\uc791\ub41c\ub2e4."} {"question": "\ub178\ubb34\ud604 \ub300\ud1b5\ub839\uc758 \uae30\ub85d\ubb3c\uc744 \uc62e\uae34 \uacf3\uc740?", "paragraph": "2008\ub144 7\uc6d4, \uad6d\uac00\uae30\ub85d\uc6d0\uacfc \ub274\ub77c\uc774\ud2b8 \uc804\uad6d\uc5f0\ud569\uc5d0\uc11c \ub300\ud1b5\ub839 \uae30\ub85d\ubb3c\uc744 \uc0ac\uc0ac\ub85c\uc774 \ubd09\ud558 \ub9c8\uc744\ub85c \uc62e\uae34 \uac74\uc5d0 \ub300\ud558\uc5ec \uac80\ucc30\uc5d0 \ubd88\ubc95\uc801\uc778 '\ubb34\ub2e8 \uc720\ucd9c'\ub85c \uae30\ub85d\ubb3c\uc5d0 \uad00\ub828\ub41c \uc804 \ube44\uc11c\uad00\uacfc \ud589\uc815\uad00\ub4e4\uc744 \uace0\ubc1c\ud558\uc5ec \uc218\uc0ac\uac00 \uc9c4\ud589\ub418\uc5c8\ub2e4. \uae30\ub85d\uc6d0\uc758 \uace0\ubc1c \uc870\uce58\uc5d0 \ub300\ud574 \ub178\ubb34\ud604 \uce21 \ube44\uc11c\uad00\uc778 \uae40\uacbd\uc218\ub294 \"\uccad\uc640\ub300\uc640 \uc815\ubd80\uc758 \ubaa9\uc801\uc774 \uae30\ub85d \ud68c\uc218\uac00 \uc544\ub2cc \ucc38\uc5ec\uc815\ubd80 \ud760\uc9d1 \ub0b4\uae30\uc600\uc74c\uc774 \ubd84\uba85\ud574\uc9c4 \uac83\"\uc774\ub77c\uace0 \ubc1d\ud788\uba70 \"\ucc38\ubaa8\uc9c4\uacfc \ud5a5\ud6c4 \ub300\uc751 \ubc29\uc548\uc744 \ub17c\uc758\ud558\uaca0\ub2e4\"\ub77c\uace0 \ub9d0\ud588\ub2e4. \uac80\ucc30\uc774 \ub300\ud1b5\ub839 \uae30\ub85d\ubb3c \uc720\ucd9c \uc2e4\uccb4 \uaddc\uba85\uc5d0 \ub098\uc11c\uac8c \ub428\uc5d0 \ub530\ub77c \uc2e0\u00b7\uad6c \uc815\uad8c \uac04 \ub300\ub9bd\uacfc \uac08\ub4f1\uc774 \uaca9\ud574\uc9c8 \uac83\uc774\ub77c\ub294 \uc2dc\uac01\ub3c4 \uc788\ub2e4. \ud55c\ud3b8 \ud68c\uc218 \uc870\uce58\ub97c \ud558\ub294 \uc640\uc911\uc5d0 \uae30\ub85d\uc774 \ub2f4\uae34 \ud558\ub4dc\ub514\uc2a4\ud06c(\ub370\uc774\ud130)\ubfd0\ub9cc \uc544\ub2c8\ub77c \ub178\ubb34\ud604\uc774 \uac1c\uc778 \uc790\uae08\uc73c\ub85c \uad6c\ub9e4\ud55c e-\uc9c0\uc6d0 \uc2dc\uc2a4\ud15c \uc11c\ubc84(\ud558\ub4dc\uc6e8\uc5b4)\uae4c\uc9c0 \ubc18\ud658\ud558\ub77c\uace0 \uc694\uad6c\ud588\uace0, \ub178\ubb34\ud604 \uce21\uc740 \uac1c\uc778 \uc7ac\uc0b0\uc774\ub77c\ub294 \uc774\uc720\ub85c \uac70\ubd80\ud588\ub2e4. \uc774\ub54c \ub300\ud1b5\ub839\uae30\ub85d\ubb3c \uad00\ub9ac\uc5d0 \uad00\ud55c \ubc95\ub960 \uc81c17\uc870 5\ud56d\uc5d0 \ub530\ub974\uba74, \uc804\uc9c1 \ub300\ud1b5\ub839 \ubc0f \uc804\uc9c1 \ub300\ud1b5\ub839\uc774 \uc9c0\uc815\ud55c \ub300\ub9ac\uc778\uc740 \ub300\ud1b5\ub839 \uae30\ub85d\ubb3c\uc744 \uc5f4\ub78c\ud560 \uc218 \uc788\uace0, \uadf8\uac83\uc774 \ube44\ubc00\ub85c \uc9c0\uc815\ub418\uc5b4 \uc788\uc9c0 \uc54a\ub2e4\uba74, \uadf8\uac83\uc744 \ucd9c\ud310\ud558\uac70\ub098 \uc5b8\ub860\ub9e4\uccb4\uc5d0 \uacf5\ud45c\ud560 \uc218 \uc788\ub2e4\uace0 \uaddc\uc815\ub418\uc5b4 \uc788\ub2e4(\ub2e4\ub9cc \uc774 \uc870\ud56d\uc740 2010\ub144 2\uc6d4 4\uc77c \uac1c\uc815\ub418\uc5c8\ub2e4). \uadf8\ub7ec\ub098 \ubc95\ub960 \ub0b4\uc6a9\uc5d0\ub3c4 \ubd88\uad6c\ud558\uace0 \uc678\ubd80\ub85c \ubc18\ucd9c\ud558\uc5ec \uc9c0\uc815\ub418\uc9c0 \uc54a\uc740 \uc7a5\uc18c\uc5d0 \ubcf4\uad00\ud558\ub294 \uac83\uc740 \uae30\ub85d\ubb3c\uc758 \uad00\ub9ac \ubc0f \ubcf4\uc548\uc0c1 \uc720\ucd9c \uc6b0\ub824\uac00 \uc788\uc5b4 \uc778\uc815\ud560 \uc218 \uc5c6\ub2e4\ub294 \uac83\uc774 \ubc95\ud559\uc790\ub4e4\uc758 \ub2e4\uc218 \uacac\ud574\uac00 \uc788\uc5c8\ub2e4.", "answer": "\ubd09\ud558 \ub9c8\uc744", "sentence": "2008\ub144 7\uc6d4, \uad6d\uac00\uae30\ub85d\uc6d0\uacfc \ub274\ub77c\uc774\ud2b8 \uc804\uad6d\uc5f0\ud569\uc5d0\uc11c \ub300\ud1b5\ub839 \uae30\ub85d\ubb3c\uc744 \uc0ac\uc0ac\ub85c\uc774 \ubd09\ud558 \ub9c8\uc744 \ub85c \uc62e\uae34 \uac74\uc5d0 \ub300\ud558\uc5ec \uac80\ucc30\uc5d0 \ubd88\ubc95\uc801\uc778 '\ubb34\ub2e8 \uc720\ucd9c'\ub85c \uae30\ub85d\ubb3c\uc5d0 \uad00\ub828\ub41c \uc804 \ube44\uc11c\uad00\uacfc \ud589\uc815\uad00\ub4e4\uc744 \uace0\ubc1c\ud558\uc5ec \uc218\uc0ac\uac00 \uc9c4\ud589\ub418\uc5c8\ub2e4.", "paragraph_sentence": " 2008\ub144 7\uc6d4, \uad6d\uac00\uae30\ub85d\uc6d0\uacfc \ub274\ub77c\uc774\ud2b8 \uc804\uad6d\uc5f0\ud569\uc5d0\uc11c \ub300\ud1b5\ub839 \uae30\ub85d\ubb3c\uc744 \uc0ac\uc0ac\ub85c\uc774 \ubd09\ud558 \ub9c8\uc744 \ub85c \uc62e\uae34 \uac74\uc5d0 \ub300\ud558\uc5ec \uac80\ucc30\uc5d0 \ubd88\ubc95\uc801\uc778 '\ubb34\ub2e8 \uc720\ucd9c'\ub85c \uae30\ub85d\ubb3c\uc5d0 \uad00\ub828\ub41c \uc804 \ube44\uc11c\uad00\uacfc \ud589\uc815\uad00\ub4e4\uc744 \uace0\ubc1c\ud558\uc5ec \uc218\uc0ac\uac00 \uc9c4\ud589\ub418\uc5c8\ub2e4. \uae30\ub85d\uc6d0\uc758 \uace0\ubc1c \uc870\uce58\uc5d0 \ub300\ud574 \ub178\ubb34\ud604 \uce21 \ube44\uc11c\uad00\uc778 \uae40\uacbd\uc218\ub294 \"\uccad\uc640\ub300\uc640 \uc815\ubd80\uc758 \ubaa9\uc801\uc774 \uae30\ub85d \ud68c\uc218\uac00 \uc544\ub2cc \ucc38\uc5ec\uc815\ubd80 \ud760\uc9d1 \ub0b4\uae30\uc600\uc74c\uc774 \ubd84\uba85\ud574\uc9c4 \uac83\"\uc774\ub77c\uace0 \ubc1d\ud788\uba70 \"\ucc38\ubaa8\uc9c4\uacfc \ud5a5\ud6c4 \ub300\uc751 \ubc29\uc548\uc744 \ub17c\uc758\ud558\uaca0\ub2e4\"\ub77c\uace0 \ub9d0\ud588\ub2e4. \uac80\ucc30\uc774 \ub300\ud1b5\ub839 \uae30\ub85d\ubb3c \uc720\ucd9c \uc2e4\uccb4 \uaddc\uba85\uc5d0 \ub098\uc11c\uac8c \ub428\uc5d0 \ub530\ub77c \uc2e0\u00b7\uad6c \uc815\uad8c \uac04 \ub300\ub9bd\uacfc \uac08\ub4f1\uc774 \uaca9\ud574\uc9c8 \uac83\uc774\ub77c\ub294 \uc2dc\uac01\ub3c4 \uc788\ub2e4. \ud55c\ud3b8 \ud68c\uc218 \uc870\uce58\ub97c \ud558\ub294 \uc640\uc911\uc5d0 \uae30\ub85d\uc774 \ub2f4\uae34 \ud558\ub4dc\ub514\uc2a4\ud06c(\ub370\uc774\ud130)\ubfd0\ub9cc \uc544\ub2c8\ub77c \ub178\ubb34\ud604\uc774 \uac1c\uc778 \uc790\uae08\uc73c\ub85c \uad6c\ub9e4\ud55c e-\uc9c0\uc6d0 \uc2dc\uc2a4\ud15c \uc11c\ubc84(\ud558\ub4dc\uc6e8\uc5b4)\uae4c\uc9c0 \ubc18\ud658\ud558\ub77c\uace0 \uc694\uad6c\ud588\uace0, \ub178\ubb34\ud604 \uce21\uc740 \uac1c\uc778 \uc7ac\uc0b0\uc774\ub77c\ub294 \uc774\uc720\ub85c \uac70\ubd80\ud588\ub2e4. \uc774\ub54c \ub300\ud1b5\ub839\uae30\ub85d\ubb3c \uad00\ub9ac\uc5d0 \uad00\ud55c \ubc95\ub960 \uc81c17\uc870 5\ud56d\uc5d0 \ub530\ub974\uba74, \uc804\uc9c1 \ub300\ud1b5\ub839 \ubc0f \uc804\uc9c1 \ub300\ud1b5\ub839\uc774 \uc9c0\uc815\ud55c \ub300\ub9ac\uc778\uc740 \ub300\ud1b5\ub839 \uae30\ub85d\ubb3c\uc744 \uc5f4\ub78c\ud560 \uc218 \uc788\uace0, \uadf8\uac83\uc774 \ube44\ubc00\ub85c \uc9c0\uc815\ub418\uc5b4 \uc788\uc9c0 \uc54a\ub2e4\uba74, \uadf8\uac83\uc744 \ucd9c\ud310\ud558\uac70\ub098 \uc5b8\ub860\ub9e4\uccb4\uc5d0 \uacf5\ud45c\ud560 \uc218 \uc788\ub2e4\uace0 \uaddc\uc815\ub418\uc5b4 \uc788\ub2e4(\ub2e4\ub9cc \uc774 \uc870\ud56d\uc740 2010\ub144 2\uc6d4 4\uc77c \uac1c\uc815\ub418\uc5c8\ub2e4). \uadf8\ub7ec\ub098 \ubc95\ub960 \ub0b4\uc6a9\uc5d0\ub3c4 \ubd88\uad6c\ud558\uace0 \uc678\ubd80\ub85c \ubc18\ucd9c\ud558\uc5ec \uc9c0\uc815\ub418\uc9c0 \uc54a\uc740 \uc7a5\uc18c\uc5d0 \ubcf4\uad00\ud558\ub294 \uac83\uc740 \uae30\ub85d\ubb3c\uc758 \uad00\ub9ac \ubc0f \ubcf4\uc548\uc0c1 \uc720\ucd9c \uc6b0\ub824\uac00 \uc788\uc5b4 \uc778\uc815\ud560 \uc218 \uc5c6\ub2e4\ub294 \uac83\uc774 \ubc95\ud559\uc790\ub4e4\uc758 \ub2e4\uc218 \uacac\ud574\uac00 \uc788\uc5c8\ub2e4.", "paragraph_answer": "2008\ub144 7\uc6d4, \uad6d\uac00\uae30\ub85d\uc6d0\uacfc \ub274\ub77c\uc774\ud2b8 \uc804\uad6d\uc5f0\ud569\uc5d0\uc11c \ub300\ud1b5\ub839 \uae30\ub85d\ubb3c\uc744 \uc0ac\uc0ac\ub85c\uc774 \ubd09\ud558 \ub9c8\uc744 \ub85c \uc62e\uae34 \uac74\uc5d0 \ub300\ud558\uc5ec \uac80\ucc30\uc5d0 \ubd88\ubc95\uc801\uc778 '\ubb34\ub2e8 \uc720\ucd9c'\ub85c \uae30\ub85d\ubb3c\uc5d0 \uad00\ub828\ub41c \uc804 \ube44\uc11c\uad00\uacfc \ud589\uc815\uad00\ub4e4\uc744 \uace0\ubc1c\ud558\uc5ec \uc218\uc0ac\uac00 \uc9c4\ud589\ub418\uc5c8\ub2e4. \uae30\ub85d\uc6d0\uc758 \uace0\ubc1c \uc870\uce58\uc5d0 \ub300\ud574 \ub178\ubb34\ud604 \uce21 \ube44\uc11c\uad00\uc778 \uae40\uacbd\uc218\ub294 \"\uccad\uc640\ub300\uc640 \uc815\ubd80\uc758 \ubaa9\uc801\uc774 \uae30\ub85d \ud68c\uc218\uac00 \uc544\ub2cc \ucc38\uc5ec\uc815\ubd80 \ud760\uc9d1 \ub0b4\uae30\uc600\uc74c\uc774 \ubd84\uba85\ud574\uc9c4 \uac83\"\uc774\ub77c\uace0 \ubc1d\ud788\uba70 \"\ucc38\ubaa8\uc9c4\uacfc \ud5a5\ud6c4 \ub300\uc751 \ubc29\uc548\uc744 \ub17c\uc758\ud558\uaca0\ub2e4\"\ub77c\uace0 \ub9d0\ud588\ub2e4. \uac80\ucc30\uc774 \ub300\ud1b5\ub839 \uae30\ub85d\ubb3c \uc720\ucd9c \uc2e4\uccb4 \uaddc\uba85\uc5d0 \ub098\uc11c\uac8c \ub428\uc5d0 \ub530\ub77c \uc2e0\u00b7\uad6c \uc815\uad8c \uac04 \ub300\ub9bd\uacfc \uac08\ub4f1\uc774 \uaca9\ud574\uc9c8 \uac83\uc774\ub77c\ub294 \uc2dc\uac01\ub3c4 \uc788\ub2e4. \ud55c\ud3b8 \ud68c\uc218 \uc870\uce58\ub97c \ud558\ub294 \uc640\uc911\uc5d0 \uae30\ub85d\uc774 \ub2f4\uae34 \ud558\ub4dc\ub514\uc2a4\ud06c(\ub370\uc774\ud130)\ubfd0\ub9cc \uc544\ub2c8\ub77c \ub178\ubb34\ud604\uc774 \uac1c\uc778 \uc790\uae08\uc73c\ub85c \uad6c\ub9e4\ud55c e-\uc9c0\uc6d0 \uc2dc\uc2a4\ud15c \uc11c\ubc84(\ud558\ub4dc\uc6e8\uc5b4)\uae4c\uc9c0 \ubc18\ud658\ud558\ub77c\uace0 \uc694\uad6c\ud588\uace0, \ub178\ubb34\ud604 \uce21\uc740 \uac1c\uc778 \uc7ac\uc0b0\uc774\ub77c\ub294 \uc774\uc720\ub85c \uac70\ubd80\ud588\ub2e4. \uc774\ub54c \ub300\ud1b5\ub839\uae30\ub85d\ubb3c \uad00\ub9ac\uc5d0 \uad00\ud55c \ubc95\ub960 \uc81c17\uc870 5\ud56d\uc5d0 \ub530\ub974\uba74, \uc804\uc9c1 \ub300\ud1b5\ub839 \ubc0f \uc804\uc9c1 \ub300\ud1b5\ub839\uc774 \uc9c0\uc815\ud55c \ub300\ub9ac\uc778\uc740 \ub300\ud1b5\ub839 \uae30\ub85d\ubb3c\uc744 \uc5f4\ub78c\ud560 \uc218 \uc788\uace0, \uadf8\uac83\uc774 \ube44\ubc00\ub85c \uc9c0\uc815\ub418\uc5b4 \uc788\uc9c0 \uc54a\ub2e4\uba74, \uadf8\uac83\uc744 \ucd9c\ud310\ud558\uac70\ub098 \uc5b8\ub860\ub9e4\uccb4\uc5d0 \uacf5\ud45c\ud560 \uc218 \uc788\ub2e4\uace0 \uaddc\uc815\ub418\uc5b4 \uc788\ub2e4(\ub2e4\ub9cc \uc774 \uc870\ud56d\uc740 2010\ub144 2\uc6d4 4\uc77c \uac1c\uc815\ub418\uc5c8\ub2e4). \uadf8\ub7ec\ub098 \ubc95\ub960 \ub0b4\uc6a9\uc5d0\ub3c4 \ubd88\uad6c\ud558\uace0 \uc678\ubd80\ub85c \ubc18\ucd9c\ud558\uc5ec \uc9c0\uc815\ub418\uc9c0 \uc54a\uc740 \uc7a5\uc18c\uc5d0 \ubcf4\uad00\ud558\ub294 \uac83\uc740 \uae30\ub85d\ubb3c\uc758 \uad00\ub9ac \ubc0f \ubcf4\uc548\uc0c1 \uc720\ucd9c \uc6b0\ub824\uac00 \uc788\uc5b4 \uc778\uc815\ud560 \uc218 \uc5c6\ub2e4\ub294 \uac83\uc774 \ubc95\ud559\uc790\ub4e4\uc758 \ub2e4\uc218 \uacac\ud574\uac00 \uc788\uc5c8\ub2e4.", "sentence_answer": "2008\ub144 7\uc6d4, \uad6d\uac00\uae30\ub85d\uc6d0\uacfc \ub274\ub77c\uc774\ud2b8 \uc804\uad6d\uc5f0\ud569\uc5d0\uc11c \ub300\ud1b5\ub839 \uae30\ub85d\ubb3c\uc744 \uc0ac\uc0ac\ub85c\uc774 \ubd09\ud558 \ub9c8\uc744 \ub85c \uc62e\uae34 \uac74\uc5d0 \ub300\ud558\uc5ec \uac80\ucc30\uc5d0 \ubd88\ubc95\uc801\uc778 '\ubb34\ub2e8 \uc720\ucd9c'\ub85c \uae30\ub85d\ubb3c\uc5d0 \uad00\ub828\ub41c \uc804 \ube44\uc11c\uad00\uacfc \ud589\uc815\uad00\ub4e4\uc744 \uace0\ubc1c\ud558\uc5ec \uc218\uc0ac\uac00 \uc9c4\ud589\ub418\uc5c8\ub2e4.", "paragraph_id": "6512090-18-0", "paragraph_question": "question: \ub178\ubb34\ud604 \ub300\ud1b5\ub839\uc758 \uae30\ub85d\ubb3c\uc744 \uc62e\uae34 \uacf3\uc740?, context: 2008\ub144 7\uc6d4, \uad6d\uac00\uae30\ub85d\uc6d0\uacfc \ub274\ub77c\uc774\ud2b8 \uc804\uad6d\uc5f0\ud569\uc5d0\uc11c \ub300\ud1b5\ub839 \uae30\ub85d\ubb3c\uc744 \uc0ac\uc0ac\ub85c\uc774 \ubd09\ud558 \ub9c8\uc744\ub85c \uc62e\uae34 \uac74\uc5d0 \ub300\ud558\uc5ec \uac80\ucc30\uc5d0 \ubd88\ubc95\uc801\uc778 '\ubb34\ub2e8 \uc720\ucd9c'\ub85c \uae30\ub85d\ubb3c\uc5d0 \uad00\ub828\ub41c \uc804 \ube44\uc11c\uad00\uacfc \ud589\uc815\uad00\ub4e4\uc744 \uace0\ubc1c\ud558\uc5ec \uc218\uc0ac\uac00 \uc9c4\ud589\ub418\uc5c8\ub2e4. \uae30\ub85d\uc6d0\uc758 \uace0\ubc1c \uc870\uce58\uc5d0 \ub300\ud574 \ub178\ubb34\ud604 \uce21 \ube44\uc11c\uad00\uc778 \uae40\uacbd\uc218\ub294 \"\uccad\uc640\ub300\uc640 \uc815\ubd80\uc758 \ubaa9\uc801\uc774 \uae30\ub85d \ud68c\uc218\uac00 \uc544\ub2cc \ucc38\uc5ec\uc815\ubd80 \ud760\uc9d1 \ub0b4\uae30\uc600\uc74c\uc774 \ubd84\uba85\ud574\uc9c4 \uac83\"\uc774\ub77c\uace0 \ubc1d\ud788\uba70 \"\ucc38\ubaa8\uc9c4\uacfc \ud5a5\ud6c4 \ub300\uc751 \ubc29\uc548\uc744 \ub17c\uc758\ud558\uaca0\ub2e4\"\ub77c\uace0 \ub9d0\ud588\ub2e4. \uac80\ucc30\uc774 \ub300\ud1b5\ub839 \uae30\ub85d\ubb3c \uc720\ucd9c \uc2e4\uccb4 \uaddc\uba85\uc5d0 \ub098\uc11c\uac8c \ub428\uc5d0 \ub530\ub77c \uc2e0\u00b7\uad6c \uc815\uad8c \uac04 \ub300\ub9bd\uacfc \uac08\ub4f1\uc774 \uaca9\ud574\uc9c8 \uac83\uc774\ub77c\ub294 \uc2dc\uac01\ub3c4 \uc788\ub2e4. \ud55c\ud3b8 \ud68c\uc218 \uc870\uce58\ub97c \ud558\ub294 \uc640\uc911\uc5d0 \uae30\ub85d\uc774 \ub2f4\uae34 \ud558\ub4dc\ub514\uc2a4\ud06c(\ub370\uc774\ud130)\ubfd0\ub9cc \uc544\ub2c8\ub77c \ub178\ubb34\ud604\uc774 \uac1c\uc778 \uc790\uae08\uc73c\ub85c \uad6c\ub9e4\ud55c e-\uc9c0\uc6d0 \uc2dc\uc2a4\ud15c \uc11c\ubc84(\ud558\ub4dc\uc6e8\uc5b4)\uae4c\uc9c0 \ubc18\ud658\ud558\ub77c\uace0 \uc694\uad6c\ud588\uace0, \ub178\ubb34\ud604 \uce21\uc740 \uac1c\uc778 \uc7ac\uc0b0\uc774\ub77c\ub294 \uc774\uc720\ub85c \uac70\ubd80\ud588\ub2e4. \uc774\ub54c \ub300\ud1b5\ub839\uae30\ub85d\ubb3c \uad00\ub9ac\uc5d0 \uad00\ud55c \ubc95\ub960 \uc81c17\uc870 5\ud56d\uc5d0 \ub530\ub974\uba74, \uc804\uc9c1 \ub300\ud1b5\ub839 \ubc0f \uc804\uc9c1 \ub300\ud1b5\ub839\uc774 \uc9c0\uc815\ud55c \ub300\ub9ac\uc778\uc740 \ub300\ud1b5\ub839 \uae30\ub85d\ubb3c\uc744 \uc5f4\ub78c\ud560 \uc218 \uc788\uace0, \uadf8\uac83\uc774 \ube44\ubc00\ub85c \uc9c0\uc815\ub418\uc5b4 \uc788\uc9c0 \uc54a\ub2e4\uba74, \uadf8\uac83\uc744 \ucd9c\ud310\ud558\uac70\ub098 \uc5b8\ub860\ub9e4\uccb4\uc5d0 \uacf5\ud45c\ud560 \uc218 \uc788\ub2e4\uace0 \uaddc\uc815\ub418\uc5b4 \uc788\ub2e4(\ub2e4\ub9cc \uc774 \uc870\ud56d\uc740 2010\ub144 2\uc6d4 4\uc77c \uac1c\uc815\ub418\uc5c8\ub2e4). \uadf8\ub7ec\ub098 \ubc95\ub960 \ub0b4\uc6a9\uc5d0\ub3c4 \ubd88\uad6c\ud558\uace0 \uc678\ubd80\ub85c \ubc18\ucd9c\ud558\uc5ec \uc9c0\uc815\ub418\uc9c0 \uc54a\uc740 \uc7a5\uc18c\uc5d0 \ubcf4\uad00\ud558\ub294 \uac83\uc740 \uae30\ub85d\ubb3c\uc758 \uad00\ub9ac \ubc0f \ubcf4\uc548\uc0c1 \uc720\ucd9c \uc6b0\ub824\uac00 \uc788\uc5b4 \uc778\uc815\ud560 \uc218 \uc5c6\ub2e4\ub294 \uac83\uc774 \ubc95\ud559\uc790\ub4e4\uc758 \ub2e4\uc218 \uacac\ud574\uac00 \uc788\uc5c8\ub2e4."} {"question": "\ud398\ud37c \uc0c1\uc0ac\uac00 \ub179\uc74c\uc5d0 \ud65c\uc6a9\ud55c \ub179\uc74c\uae30\ub294 \ubb34\uc5c7\uc778\uac00?", "paragraph": "\ud398\ud37c \uc0c1\uc0ac\ub294 \ud3ec \ud2b8\ub799 \ub179\uc74c\uae30\ub97c \uc774\uc6a9\ud558\uc5ec \ub179\uc74c\ud588\ub2e4. \uc5d0\uc774\ud2b8 \ud2b8\ub799 \ud14c\uc774\ud504 \ub179\uc74c\uae30\uac00 \ubbf8\uad6d\uc5d0 \uc788\uae34 \ud588\uc9c0\ub9cc, 1967\ub144 \ub9d0\uae4c\uc9c0 \ub7f0\ub358\uc758 \uc0c1\uc5c5 \uc2a4\ud29c\ub514\uc624\uc5d0\uc11c\ub294 \uac00\ub3d9\uc774 \ubd88\uac00\ub2a5\ud588\ub2e4. \uc774\uc804\uc758 \ube44\ud2c0\uc988 \uc74c\ubc18\ucc98\ub7fc, \ud398\ud37c \uc0c1\uc0ac\ub294 \ud558\ub098\uc5d0\uc11c \ub124 \uac1c\uae4c\uc9c0\uc758 \ud2b8\ub799\uc744 \ud558\ub098\uc758 \ub179\uc74c\uae30\uc5d0\uc11c \ubbf9\uc2a4\ud558\uace0 \ub354\ube59\ud558\ub294 \ud3ec \ud2b8\ub799 \ub179\uc74c\uae30\ub85c, \ub9ac\ub355\uc158 \ubbf9\uc2f1\uc774\ub77c\uace0 \uc54c\ub824\uc9c4 \uae30\uc220\uc744 \ubc94\uc6a9\ud588\ub2e4. \uc774\ub7f0 \uc900 \uba40\ud2f0 \ud2b8\ub799 \uc2a4\ud29c\ub514\uc624\ub294 \uc560\ube44 \ub85c\ub4dc\uc758 \uc5d4\uc9c0\ub2c8\uc5b4 \ub355\ubd84\uc5d0 \uac00\ub2a5\ud55c \uc77c\uc774\uc5c8\ub2e4. EMI\uc758 \uc2a4\ud29c\ub354 J37 \ud3ec \ud2b8\ub799 \uba38\uc2e0\uc740 \uc81c\uc791 \uc911 \ubc1c\uc0dd\ud558\ub294 \uc18c\uc74c\uc744 \ucd5c\uc18c\ud654\ud558\uc5ec \ub192\uc740 \ud488\uc9c8\uc758 \ub808\ucf54\ub529\uc744 \ud560 \uc218 \uc788\uc5c8\uae30\uc5d0, \ub9ac\ub355\uc158 \ubbf9\uc2f1\uc5d0 \uc801\ud569\ud588\ub2e4. \ub9e4\uce74\ud2b8\ub2c8\ub294 \uc790\uc2e0\uc758 \ubca0\uc774\uc2a4 \ud30c\ud2b8\ub97c \ub9c8\uc9c0\ub9c9\uc5d0 \uc624\ubc84\ub354\ube59\ud558\ub294 \uac83\uc744 \uc120\ud638\ud588\uace0, \uace1\uc758 \ubc31 \ud2b8\ub799\uc744 \ub179\uc74c\ud558\uace0 \uc788\uc744 \ub54c, \ub2e4\ub978 \uc545\uae30\ub97c \uc5f0\uc8fc\ud558\ub294 \uacbd\ud5a5\uc774 \uc788\uc5c8\ub2e4. \uc774\ub7ec\ud55c \uc811\uadfc \ubc29\uc2dd\uc740 \uadf8\uc5d0\uac8c \uace1\uc758 \ucd5c\uc885 \ud3b8\uace1\uc744 \ubcf4\uc644\ud558\uae30 \uc704\ud574, \uc791\uc0ac\uc640 \uc120\uc728\uc801 \ubca0\uc774\uc2a4\ub77c\uc778\uc744 \ub179\uc74c\ud558\ub294\ub370 \ud544\uc694\ud55c \uc5ec\ubd84\uc758 \uc2dc\uac04\uc744 \uc8fc\uc5c8\ub2e4. \u3008A Day in the Life\u3009\ub97c \uc704\ud574 \uc624\ucf00\uc2a4\ud2b8\ub77c\ub97c \ub179\uc74c\ud560 \ub54c, \ub9c8\ud2f4\uc740 \ube44\ud2c0\uc988\uac00 \uc5f0\uc8fc\ud55c \ubc18\uc8fc\uc640 \uc624\ucf00\uc2a4\ud2b8\ub77c \uc624\ubc84\ub354\ube59 \ud14c\uc774\ud504\ub97c \ub3d9\uae30\ud588\ub2e4. \uc5d4\uc9c0\ub2c8\uc5b4 \ucf04 \ud0c0\uc6b4\uc13c\ub4dc\ub294 \ub450 \uba38\uc2e0 \uc0ac\uc774\uc758 50 Hz \uc81c\uc5b4 \uc2e0\ud638\ub97c \uc0ac\uc6a9\ud558\uc5ec \uc774\ub97c \ub2ec\uc131\ud558\ub294 \ubc29\ubc95\uc744 \uace0\uc548\ud588\ub2e4.", "answer": "\ud3ec \ud2b8\ub799 \ub179\uc74c\uae30", "sentence": "\ud398\ud37c \uc0c1\uc0ac\ub294 \ud3ec \ud2b8\ub799 \ub179\uc74c\uae30 \ub97c \uc774\uc6a9\ud558\uc5ec \ub179\uc74c\ud588\ub2e4.", "paragraph_sentence": " \ud398\ud37c \uc0c1\uc0ac\ub294 \ud3ec \ud2b8\ub799 \ub179\uc74c\uae30 \ub97c \uc774\uc6a9\ud558\uc5ec \ub179\uc74c\ud588\ub2e4. \uc5d0\uc774\ud2b8 \ud2b8\ub799 \ud14c\uc774\ud504 \ub179\uc74c\uae30\uac00 \ubbf8\uad6d\uc5d0 \uc788\uae34 \ud588\uc9c0\ub9cc, 1967\ub144 \ub9d0\uae4c\uc9c0 \ub7f0\ub358\uc758 \uc0c1\uc5c5 \uc2a4\ud29c\ub514\uc624\uc5d0\uc11c\ub294 \uac00\ub3d9\uc774 \ubd88\uac00\ub2a5\ud588\ub2e4. \uc774\uc804\uc758 \ube44\ud2c0\uc988 \uc74c\ubc18\ucc98\ub7fc, \ud398\ud37c \uc0c1\uc0ac\ub294 \ud558\ub098\uc5d0\uc11c \ub124 \uac1c\uae4c\uc9c0\uc758 \ud2b8\ub799\uc744 \ud558\ub098\uc758 \ub179\uc74c\uae30\uc5d0\uc11c \ubbf9\uc2a4\ud558\uace0 \ub354\ube59\ud558\ub294 \ud3ec \ud2b8\ub799 \ub179\uc74c\uae30\ub85c, \ub9ac\ub355\uc158 \ubbf9\uc2f1\uc774\ub77c\uace0 \uc54c\ub824\uc9c4 \uae30\uc220\uc744 \ubc94\uc6a9\ud588\ub2e4. \uc774\ub7f0 \uc900 \uba40\ud2f0 \ud2b8\ub799 \uc2a4\ud29c\ub514\uc624\ub294 \uc560\ube44 \ub85c\ub4dc\uc758 \uc5d4\uc9c0\ub2c8\uc5b4 \ub355\ubd84\uc5d0 \uac00\ub2a5\ud55c \uc77c\uc774\uc5c8\ub2e4. EMI\uc758 \uc2a4\ud29c\ub354 J37 \ud3ec \ud2b8\ub799 \uba38\uc2e0\uc740 \uc81c\uc791 \uc911 \ubc1c\uc0dd\ud558\ub294 \uc18c\uc74c\uc744 \ucd5c\uc18c\ud654\ud558\uc5ec \ub192\uc740 \ud488\uc9c8\uc758 \ub808\ucf54\ub529\uc744 \ud560 \uc218 \uc788\uc5c8\uae30\uc5d0, \ub9ac\ub355\uc158 \ubbf9\uc2f1\uc5d0 \uc801\ud569\ud588\ub2e4. \ub9e4\uce74\ud2b8\ub2c8\ub294 \uc790\uc2e0\uc758 \ubca0\uc774\uc2a4 \ud30c\ud2b8\ub97c \ub9c8\uc9c0\ub9c9\uc5d0 \uc624\ubc84\ub354\ube59\ud558\ub294 \uac83\uc744 \uc120\ud638\ud588\uace0, \uace1\uc758 \ubc31 \ud2b8\ub799\uc744 \ub179\uc74c\ud558\uace0 \uc788\uc744 \ub54c, \ub2e4\ub978 \uc545\uae30\ub97c \uc5f0\uc8fc\ud558\ub294 \uacbd\ud5a5\uc774 \uc788\uc5c8\ub2e4. \uc774\ub7ec\ud55c \uc811\uadfc \ubc29\uc2dd\uc740 \uadf8\uc5d0\uac8c \uace1\uc758 \ucd5c\uc885 \ud3b8\uace1\uc744 \ubcf4\uc644\ud558\uae30 \uc704\ud574, \uc791\uc0ac\uc640 \uc120\uc728\uc801 \ubca0\uc774\uc2a4\ub77c\uc778\uc744 \ub179\uc74c\ud558\ub294\ub370 \ud544\uc694\ud55c \uc5ec\ubd84\uc758 \uc2dc\uac04\uc744 \uc8fc\uc5c8\ub2e4. \u3008A Day in the Life\u3009\ub97c \uc704\ud574 \uc624\ucf00\uc2a4\ud2b8\ub77c\ub97c \ub179\uc74c\ud560 \ub54c, \ub9c8\ud2f4\uc740 \ube44\ud2c0\uc988\uac00 \uc5f0\uc8fc\ud55c \ubc18\uc8fc\uc640 \uc624\ucf00\uc2a4\ud2b8\ub77c \uc624\ubc84\ub354\ube59 \ud14c\uc774\ud504\ub97c \ub3d9\uae30\ud588\ub2e4. \uc5d4\uc9c0\ub2c8\uc5b4 \ucf04 \ud0c0\uc6b4\uc13c\ub4dc\ub294 \ub450 \uba38\uc2e0 \uc0ac\uc774\uc758 50 Hz \uc81c\uc5b4 \uc2e0\ud638\ub97c \uc0ac\uc6a9\ud558\uc5ec \uc774\ub97c \ub2ec\uc131\ud558\ub294 \ubc29\ubc95\uc744 \uace0\uc548\ud588\ub2e4.", "paragraph_answer": "\ud398\ud37c \uc0c1\uc0ac\ub294 \ud3ec \ud2b8\ub799 \ub179\uc74c\uae30 \ub97c \uc774\uc6a9\ud558\uc5ec \ub179\uc74c\ud588\ub2e4. \uc5d0\uc774\ud2b8 \ud2b8\ub799 \ud14c\uc774\ud504 \ub179\uc74c\uae30\uac00 \ubbf8\uad6d\uc5d0 \uc788\uae34 \ud588\uc9c0\ub9cc, 1967\ub144 \ub9d0\uae4c\uc9c0 \ub7f0\ub358\uc758 \uc0c1\uc5c5 \uc2a4\ud29c\ub514\uc624\uc5d0\uc11c\ub294 \uac00\ub3d9\uc774 \ubd88\uac00\ub2a5\ud588\ub2e4. \uc774\uc804\uc758 \ube44\ud2c0\uc988 \uc74c\ubc18\ucc98\ub7fc, \ud398\ud37c \uc0c1\uc0ac\ub294 \ud558\ub098\uc5d0\uc11c \ub124 \uac1c\uae4c\uc9c0\uc758 \ud2b8\ub799\uc744 \ud558\ub098\uc758 \ub179\uc74c\uae30\uc5d0\uc11c \ubbf9\uc2a4\ud558\uace0 \ub354\ube59\ud558\ub294 \ud3ec \ud2b8\ub799 \ub179\uc74c\uae30\ub85c, \ub9ac\ub355\uc158 \ubbf9\uc2f1\uc774\ub77c\uace0 \uc54c\ub824\uc9c4 \uae30\uc220\uc744 \ubc94\uc6a9\ud588\ub2e4. \uc774\ub7f0 \uc900 \uba40\ud2f0 \ud2b8\ub799 \uc2a4\ud29c\ub514\uc624\ub294 \uc560\ube44 \ub85c\ub4dc\uc758 \uc5d4\uc9c0\ub2c8\uc5b4 \ub355\ubd84\uc5d0 \uac00\ub2a5\ud55c \uc77c\uc774\uc5c8\ub2e4. EMI\uc758 \uc2a4\ud29c\ub354 J37 \ud3ec \ud2b8\ub799 \uba38\uc2e0\uc740 \uc81c\uc791 \uc911 \ubc1c\uc0dd\ud558\ub294 \uc18c\uc74c\uc744 \ucd5c\uc18c\ud654\ud558\uc5ec \ub192\uc740 \ud488\uc9c8\uc758 \ub808\ucf54\ub529\uc744 \ud560 \uc218 \uc788\uc5c8\uae30\uc5d0, \ub9ac\ub355\uc158 \ubbf9\uc2f1\uc5d0 \uc801\ud569\ud588\ub2e4. \ub9e4\uce74\ud2b8\ub2c8\ub294 \uc790\uc2e0\uc758 \ubca0\uc774\uc2a4 \ud30c\ud2b8\ub97c \ub9c8\uc9c0\ub9c9\uc5d0 \uc624\ubc84\ub354\ube59\ud558\ub294 \uac83\uc744 \uc120\ud638\ud588\uace0, \uace1\uc758 \ubc31 \ud2b8\ub799\uc744 \ub179\uc74c\ud558\uace0 \uc788\uc744 \ub54c, \ub2e4\ub978 \uc545\uae30\ub97c \uc5f0\uc8fc\ud558\ub294 \uacbd\ud5a5\uc774 \uc788\uc5c8\ub2e4. \uc774\ub7ec\ud55c \uc811\uadfc \ubc29\uc2dd\uc740 \uadf8\uc5d0\uac8c \uace1\uc758 \ucd5c\uc885 \ud3b8\uace1\uc744 \ubcf4\uc644\ud558\uae30 \uc704\ud574, \uc791\uc0ac\uc640 \uc120\uc728\uc801 \ubca0\uc774\uc2a4\ub77c\uc778\uc744 \ub179\uc74c\ud558\ub294\ub370 \ud544\uc694\ud55c \uc5ec\ubd84\uc758 \uc2dc\uac04\uc744 \uc8fc\uc5c8\ub2e4. \u3008A Day in the Life\u3009\ub97c \uc704\ud574 \uc624\ucf00\uc2a4\ud2b8\ub77c\ub97c \ub179\uc74c\ud560 \ub54c, \ub9c8\ud2f4\uc740 \ube44\ud2c0\uc988\uac00 \uc5f0\uc8fc\ud55c \ubc18\uc8fc\uc640 \uc624\ucf00\uc2a4\ud2b8\ub77c \uc624\ubc84\ub354\ube59 \ud14c\uc774\ud504\ub97c \ub3d9\uae30\ud588\ub2e4. \uc5d4\uc9c0\ub2c8\uc5b4 \ucf04 \ud0c0\uc6b4\uc13c\ub4dc\ub294 \ub450 \uba38\uc2e0 \uc0ac\uc774\uc758 50 Hz \uc81c\uc5b4 \uc2e0\ud638\ub97c \uc0ac\uc6a9\ud558\uc5ec \uc774\ub97c \ub2ec\uc131\ud558\ub294 \ubc29\ubc95\uc744 \uace0\uc548\ud588\ub2e4.", "sentence_answer": "\ud398\ud37c \uc0c1\uc0ac\ub294 \ud3ec \ud2b8\ub799 \ub179\uc74c\uae30 \ub97c \uc774\uc6a9\ud558\uc5ec \ub179\uc74c\ud588\ub2e4.", "paragraph_id": "6568043-8-0", "paragraph_question": "question: \ud398\ud37c \uc0c1\uc0ac\uac00 \ub179\uc74c\uc5d0 \ud65c\uc6a9\ud55c \ub179\uc74c\uae30\ub294 \ubb34\uc5c7\uc778\uac00?, context: \ud398\ud37c \uc0c1\uc0ac\ub294 \ud3ec \ud2b8\ub799 \ub179\uc74c\uae30\ub97c \uc774\uc6a9\ud558\uc5ec \ub179\uc74c\ud588\ub2e4. \uc5d0\uc774\ud2b8 \ud2b8\ub799 \ud14c\uc774\ud504 \ub179\uc74c\uae30\uac00 \ubbf8\uad6d\uc5d0 \uc788\uae34 \ud588\uc9c0\ub9cc, 1967\ub144 \ub9d0\uae4c\uc9c0 \ub7f0\ub358\uc758 \uc0c1\uc5c5 \uc2a4\ud29c\ub514\uc624\uc5d0\uc11c\ub294 \uac00\ub3d9\uc774 \ubd88\uac00\ub2a5\ud588\ub2e4. \uc774\uc804\uc758 \ube44\ud2c0\uc988 \uc74c\ubc18\ucc98\ub7fc, \ud398\ud37c \uc0c1\uc0ac\ub294 \ud558\ub098\uc5d0\uc11c \ub124 \uac1c\uae4c\uc9c0\uc758 \ud2b8\ub799\uc744 \ud558\ub098\uc758 \ub179\uc74c\uae30\uc5d0\uc11c \ubbf9\uc2a4\ud558\uace0 \ub354\ube59\ud558\ub294 \ud3ec \ud2b8\ub799 \ub179\uc74c\uae30\ub85c, \ub9ac\ub355\uc158 \ubbf9\uc2f1\uc774\ub77c\uace0 \uc54c\ub824\uc9c4 \uae30\uc220\uc744 \ubc94\uc6a9\ud588\ub2e4. \uc774\ub7f0 \uc900 \uba40\ud2f0 \ud2b8\ub799 \uc2a4\ud29c\ub514\uc624\ub294 \uc560\ube44 \ub85c\ub4dc\uc758 \uc5d4\uc9c0\ub2c8\uc5b4 \ub355\ubd84\uc5d0 \uac00\ub2a5\ud55c \uc77c\uc774\uc5c8\ub2e4. EMI\uc758 \uc2a4\ud29c\ub354 J37 \ud3ec \ud2b8\ub799 \uba38\uc2e0\uc740 \uc81c\uc791 \uc911 \ubc1c\uc0dd\ud558\ub294 \uc18c\uc74c\uc744 \ucd5c\uc18c\ud654\ud558\uc5ec \ub192\uc740 \ud488\uc9c8\uc758 \ub808\ucf54\ub529\uc744 \ud560 \uc218 \uc788\uc5c8\uae30\uc5d0, \ub9ac\ub355\uc158 \ubbf9\uc2f1\uc5d0 \uc801\ud569\ud588\ub2e4. \ub9e4\uce74\ud2b8\ub2c8\ub294 \uc790\uc2e0\uc758 \ubca0\uc774\uc2a4 \ud30c\ud2b8\ub97c \ub9c8\uc9c0\ub9c9\uc5d0 \uc624\ubc84\ub354\ube59\ud558\ub294 \uac83\uc744 \uc120\ud638\ud588\uace0, \uace1\uc758 \ubc31 \ud2b8\ub799\uc744 \ub179\uc74c\ud558\uace0 \uc788\uc744 \ub54c, \ub2e4\ub978 \uc545\uae30\ub97c \uc5f0\uc8fc\ud558\ub294 \uacbd\ud5a5\uc774 \uc788\uc5c8\ub2e4. \uc774\ub7ec\ud55c \uc811\uadfc \ubc29\uc2dd\uc740 \uadf8\uc5d0\uac8c \uace1\uc758 \ucd5c\uc885 \ud3b8\uace1\uc744 \ubcf4\uc644\ud558\uae30 \uc704\ud574, \uc791\uc0ac\uc640 \uc120\uc728\uc801 \ubca0\uc774\uc2a4\ub77c\uc778\uc744 \ub179\uc74c\ud558\ub294\ub370 \ud544\uc694\ud55c \uc5ec\ubd84\uc758 \uc2dc\uac04\uc744 \uc8fc\uc5c8\ub2e4. \u3008A Day in the Life\u3009\ub97c \uc704\ud574 \uc624\ucf00\uc2a4\ud2b8\ub77c\ub97c \ub179\uc74c\ud560 \ub54c, \ub9c8\ud2f4\uc740 \ube44\ud2c0\uc988\uac00 \uc5f0\uc8fc\ud55c \ubc18\uc8fc\uc640 \uc624\ucf00\uc2a4\ud2b8\ub77c \uc624\ubc84\ub354\ube59 \ud14c\uc774\ud504\ub97c \ub3d9\uae30\ud588\ub2e4. \uc5d4\uc9c0\ub2c8\uc5b4 \ucf04 \ud0c0\uc6b4\uc13c\ub4dc\ub294 \ub450 \uba38\uc2e0 \uc0ac\uc774\uc758 50 Hz \uc81c\uc5b4 \uc2e0\ud638\ub97c \uc0ac\uc6a9\ud558\uc5ec \uc774\ub97c \ub2ec\uc131\ud558\ub294 \ubc29\ubc95\uc744 \uace0\uc548\ud588\ub2e4."} {"question": "\uce98\ub9ac\ud3ec\ub284\uc758 \ub3d9\uc704 \uc6d0\uc18c \uc911 \uc9c8\ub7c9\uc218\uac00 \uc81c\uc77c \uc791\uc740 \uac83\uc758 \uc9c8\ub7c9\uc218\ub294 \uba87\uc778\uac00?", "paragraph": "\uc9c8\ub7c9\uc218\uac00 237\ubd80\ud130 256\uae4c\uc9c0\uc778 \uce98\ub9ac\ud3ec\ub284 \ub3d9\uc704 \uc6d0\uc18c\ub294 \ud575 \ubc18\uc751\uae30\uc5d0\uc11c \ub9cc\ub4e4\uc5b4\uc9c4\ub2e4. \uce98\ub9ac\ud3ec\ub284-253\uc740 \u03b2-\ubc29\uc0ac\ub97c \ud558\ub294 \ub3d9\uc704 \uc6d0\uc18c\uc774\uace0 \ub098\uba38\uc9c0\ub294 \u03b1-\ubc29\uc0ac\ub97c \ud558\ub294 \ub3d9\uc704 \uc6d0\uc18c\uc774\ub2e4. \uc9c8\ub7c9\uc218\uac00 \uc9dd\uc218\uc778 \ub3d9\uc704 \uc6d0\uc18c (Cf, Cf and Cf)\ub294 \uc790\ubc1c \ud575\ubd84\uc5f4\uc744 \ud560 \ud655\ub960\uc774 \ub192\ub2e4. \ud2b9\ud788 Cf\ub294 \uc790\ubc1c \ud575\ubd84\uc5f4\ud560 \ud655\ub960\uc774 99.7%\uc774\ub2e4. \uce98\ub9ac\ud3ec\ub284-249\ub294 \uadf8\ub098\ub9c8 \uae34 \ubc18\uac10\uae30 (352\ub144)\uc744 \uac00\uc9c0\uace0 \uc788\uace0, \ub0ae\uc740 \uc790\ubc1c \ud575\ubd84\uc5f4 \ud655\ub960\uacfc \uac15\ud55c \u03b3-\ubc29\uc0ac\ub97c \ud558\uc5ec \uc778\uc2dd\uc744 \uac00\ub2a5\ud558\uac8c \ud55c\ub2e4. Cf\ub294 \ud575 \ubc18\uc751\uae30\uc5d0\uc11c \ub9ce\uc774 \ub9cc\ub4e4\uc5b4\uc9c0\uc9c0 \uc54a\ub294\ub2e4. \uc65c\ub0d0\ud558\uba74 \ubd95\uad34\ub418\uae30 \uc804 Bk\uc758 \ub290\ub9b0 \u03b2-\ubd95\uad34\uc640 \uc911\uc131\uc790\uc640 \uc0c1\ud638 \uc791\uc6a9\uc744 \ud560 \ub54c\uc758 \ud070 \ub2e8\uba74\uc801 \ub54c\ubb38\uc5d0 \ub9cc\ub4e4\uae30\uac00 \uc27d\uc9c0 \uc54a\ub2e4. \uadf8\ub7ec\ub098 Bk\uc758 \u03b2-\ubd95\uad34\uc758 \uc0b0\ubb3c\ub85c \uc778\ud574 \uc21c\uc218\ud55c \ub3d9\uc704 \uc6d0\uc18c\ub85c \ucd95\uc801\ub41c\ub2e4. \uce98\ub9ac\ud3ec\ub284\uc740 \ud50c\ub8e8\ud1a0\ub284\uc758 \ubc18\uc751\uae30 \uc870\uc0ac\uc5d0 \uc758\ud574 \ub9cc\ub4e4\uc5b4\uc9c4\ub2e4. \uc774\uac83\uc740 \uc8fc\ub85c Cf\uacfc Cf\uac00 \uc874\uc7ac\ud558\ub294\ub370, \ud6c4\uc5d0 \ud070 \uc911\uc131\uc790\uc758 \uc601\ud5a5\uc73c\ub85c \uc810\uc810 \ub69c\ub837\ud574\uc9c0\uace0, \uc774\uac83\uc758 \ud2b9\uc9d5\uc740 \uac15\ud55c \uc911\uc131\uc790\uc120\uc5d0 \uc758\ud574 \ubc18\uc751\uc774 \uc800\ud574\ub41c\ub2e4.", "answer": "237", "sentence": "\uc9c8\ub7c9\uc218\uac00 237 \ubd80\ud130 256\uae4c\uc9c0\uc778 \uce98\ub9ac\ud3ec\ub284 \ub3d9\uc704 \uc6d0\uc18c\ub294 \ud575 \ubc18\uc751\uae30\uc5d0\uc11c \ub9cc\ub4e4\uc5b4\uc9c4\ub2e4.", "paragraph_sentence": " \uc9c8\ub7c9\uc218\uac00 237 \ubd80\ud130 256\uae4c\uc9c0\uc778 \uce98\ub9ac\ud3ec\ub284 \ub3d9\uc704 \uc6d0\uc18c\ub294 \ud575 \ubc18\uc751\uae30\uc5d0\uc11c \ub9cc\ub4e4\uc5b4\uc9c4\ub2e4. \uce98\ub9ac\ud3ec\ub284-253\uc740 \u03b2-\ubc29\uc0ac\ub97c \ud558\ub294 \ub3d9\uc704 \uc6d0\uc18c\uc774\uace0 \ub098\uba38\uc9c0\ub294 \u03b1-\ubc29\uc0ac\ub97c \ud558\ub294 \ub3d9\uc704 \uc6d0\uc18c\uc774\ub2e4. \uc9c8\ub7c9\uc218\uac00 \uc9dd\uc218\uc778 \ub3d9\uc704 \uc6d0\uc18c (Cf, Cf and Cf)\ub294 \uc790\ubc1c \ud575\ubd84\uc5f4\uc744 \ud560 \ud655\ub960\uc774 \ub192\ub2e4. \ud2b9\ud788 Cf\ub294 \uc790\ubc1c \ud575\ubd84\uc5f4\ud560 \ud655\ub960\uc774 99.7%\uc774\ub2e4. \uce98\ub9ac\ud3ec\ub284-249\ub294 \uadf8\ub098\ub9c8 \uae34 \ubc18\uac10\uae30 (352\ub144)\uc744 \uac00\uc9c0\uace0 \uc788\uace0, \ub0ae\uc740 \uc790\ubc1c \ud575\ubd84\uc5f4 \ud655\ub960\uacfc \uac15\ud55c \u03b3-\ubc29\uc0ac\ub97c \ud558\uc5ec \uc778\uc2dd\uc744 \uac00\ub2a5\ud558\uac8c \ud55c\ub2e4. Cf\ub294 \ud575 \ubc18\uc751\uae30\uc5d0\uc11c \ub9ce\uc774 \ub9cc\ub4e4\uc5b4\uc9c0\uc9c0 \uc54a\ub294\ub2e4. \uc65c\ub0d0\ud558\uba74 \ubd95\uad34\ub418\uae30 \uc804 Bk\uc758 \ub290\ub9b0 \u03b2-\ubd95\uad34\uc640 \uc911\uc131\uc790\uc640 \uc0c1\ud638 \uc791\uc6a9\uc744 \ud560 \ub54c\uc758 \ud070 \ub2e8\uba74\uc801 \ub54c\ubb38\uc5d0 \ub9cc\ub4e4\uae30\uac00 \uc27d\uc9c0 \uc54a\ub2e4. \uadf8\ub7ec\ub098 Bk\uc758 \u03b2-\ubd95\uad34\uc758 \uc0b0\ubb3c\ub85c \uc778\ud574 \uc21c\uc218\ud55c \ub3d9\uc704 \uc6d0\uc18c\ub85c \ucd95\uc801\ub41c\ub2e4. \uce98\ub9ac\ud3ec\ub284\uc740 \ud50c\ub8e8\ud1a0\ub284\uc758 \ubc18\uc751\uae30 \uc870\uc0ac\uc5d0 \uc758\ud574 \ub9cc\ub4e4\uc5b4\uc9c4\ub2e4. \uc774\uac83\uc740 \uc8fc\ub85c Cf\uacfc Cf\uac00 \uc874\uc7ac\ud558\ub294\ub370, \ud6c4\uc5d0 \ud070 \uc911\uc131\uc790\uc758 \uc601\ud5a5\uc73c\ub85c \uc810\uc810 \ub69c\ub837\ud574\uc9c0\uace0, \uc774\uac83\uc758 \ud2b9\uc9d5\uc740 \uac15\ud55c \uc911\uc131\uc790\uc120\uc5d0 \uc758\ud574 \ubc18\uc751\uc774 \uc800\ud574\ub41c\ub2e4.", "paragraph_answer": "\uc9c8\ub7c9\uc218\uac00 237 \ubd80\ud130 256\uae4c\uc9c0\uc778 \uce98\ub9ac\ud3ec\ub284 \ub3d9\uc704 \uc6d0\uc18c\ub294 \ud575 \ubc18\uc751\uae30\uc5d0\uc11c \ub9cc\ub4e4\uc5b4\uc9c4\ub2e4. \uce98\ub9ac\ud3ec\ub284-253\uc740 \u03b2-\ubc29\uc0ac\ub97c \ud558\ub294 \ub3d9\uc704 \uc6d0\uc18c\uc774\uace0 \ub098\uba38\uc9c0\ub294 \u03b1-\ubc29\uc0ac\ub97c \ud558\ub294 \ub3d9\uc704 \uc6d0\uc18c\uc774\ub2e4. \uc9c8\ub7c9\uc218\uac00 \uc9dd\uc218\uc778 \ub3d9\uc704 \uc6d0\uc18c (Cf, Cf and Cf)\ub294 \uc790\ubc1c \ud575\ubd84\uc5f4\uc744 \ud560 \ud655\ub960\uc774 \ub192\ub2e4. \ud2b9\ud788 Cf\ub294 \uc790\ubc1c \ud575\ubd84\uc5f4\ud560 \ud655\ub960\uc774 99.7%\uc774\ub2e4. \uce98\ub9ac\ud3ec\ub284-249\ub294 \uadf8\ub098\ub9c8 \uae34 \ubc18\uac10\uae30 (352\ub144)\uc744 \uac00\uc9c0\uace0 \uc788\uace0, \ub0ae\uc740 \uc790\ubc1c \ud575\ubd84\uc5f4 \ud655\ub960\uacfc \uac15\ud55c \u03b3-\ubc29\uc0ac\ub97c \ud558\uc5ec \uc778\uc2dd\uc744 \uac00\ub2a5\ud558\uac8c \ud55c\ub2e4. Cf\ub294 \ud575 \ubc18\uc751\uae30\uc5d0\uc11c \ub9ce\uc774 \ub9cc\ub4e4\uc5b4\uc9c0\uc9c0 \uc54a\ub294\ub2e4. \uc65c\ub0d0\ud558\uba74 \ubd95\uad34\ub418\uae30 \uc804 Bk\uc758 \ub290\ub9b0 \u03b2-\ubd95\uad34\uc640 \uc911\uc131\uc790\uc640 \uc0c1\ud638 \uc791\uc6a9\uc744 \ud560 \ub54c\uc758 \ud070 \ub2e8\uba74\uc801 \ub54c\ubb38\uc5d0 \ub9cc\ub4e4\uae30\uac00 \uc27d\uc9c0 \uc54a\ub2e4. \uadf8\ub7ec\ub098 Bk\uc758 \u03b2-\ubd95\uad34\uc758 \uc0b0\ubb3c\ub85c \uc778\ud574 \uc21c\uc218\ud55c \ub3d9\uc704 \uc6d0\uc18c\ub85c \ucd95\uc801\ub41c\ub2e4. \uce98\ub9ac\ud3ec\ub284\uc740 \ud50c\ub8e8\ud1a0\ub284\uc758 \ubc18\uc751\uae30 \uc870\uc0ac\uc5d0 \uc758\ud574 \ub9cc\ub4e4\uc5b4\uc9c4\ub2e4. \uc774\uac83\uc740 \uc8fc\ub85c Cf\uacfc Cf\uac00 \uc874\uc7ac\ud558\ub294\ub370, \ud6c4\uc5d0 \ud070 \uc911\uc131\uc790\uc758 \uc601\ud5a5\uc73c\ub85c \uc810\uc810 \ub69c\ub837\ud574\uc9c0\uace0, \uc774\uac83\uc758 \ud2b9\uc9d5\uc740 \uac15\ud55c \uc911\uc131\uc790\uc120\uc5d0 \uc758\ud574 \ubc18\uc751\uc774 \uc800\ud574\ub41c\ub2e4.", "sentence_answer": "\uc9c8\ub7c9\uc218\uac00 237 \ubd80\ud130 256\uae4c\uc9c0\uc778 \uce98\ub9ac\ud3ec\ub284 \ub3d9\uc704 \uc6d0\uc18c\ub294 \ud575 \ubc18\uc751\uae30\uc5d0\uc11c \ub9cc\ub4e4\uc5b4\uc9c4\ub2e4.", "paragraph_id": "6585607-7-0", "paragraph_question": "question: \uce98\ub9ac\ud3ec\ub284\uc758 \ub3d9\uc704 \uc6d0\uc18c \uc911 \uc9c8\ub7c9\uc218\uac00 \uc81c\uc77c \uc791\uc740 \uac83\uc758 \uc9c8\ub7c9\uc218\ub294 \uba87\uc778\uac00?, context: \uc9c8\ub7c9\uc218\uac00 237\ubd80\ud130 256\uae4c\uc9c0\uc778 \uce98\ub9ac\ud3ec\ub284 \ub3d9\uc704 \uc6d0\uc18c\ub294 \ud575 \ubc18\uc751\uae30\uc5d0\uc11c \ub9cc\ub4e4\uc5b4\uc9c4\ub2e4. \uce98\ub9ac\ud3ec\ub284-253\uc740 \u03b2-\ubc29\uc0ac\ub97c \ud558\ub294 \ub3d9\uc704 \uc6d0\uc18c\uc774\uace0 \ub098\uba38\uc9c0\ub294 \u03b1-\ubc29\uc0ac\ub97c \ud558\ub294 \ub3d9\uc704 \uc6d0\uc18c\uc774\ub2e4. \uc9c8\ub7c9\uc218\uac00 \uc9dd\uc218\uc778 \ub3d9\uc704 \uc6d0\uc18c (Cf, Cf and Cf)\ub294 \uc790\ubc1c \ud575\ubd84\uc5f4\uc744 \ud560 \ud655\ub960\uc774 \ub192\ub2e4. \ud2b9\ud788 Cf\ub294 \uc790\ubc1c \ud575\ubd84\uc5f4\ud560 \ud655\ub960\uc774 99.7%\uc774\ub2e4. \uce98\ub9ac\ud3ec\ub284-249\ub294 \uadf8\ub098\ub9c8 \uae34 \ubc18\uac10\uae30 (352\ub144)\uc744 \uac00\uc9c0\uace0 \uc788\uace0, \ub0ae\uc740 \uc790\ubc1c \ud575\ubd84\uc5f4 \ud655\ub960\uacfc \uac15\ud55c \u03b3-\ubc29\uc0ac\ub97c \ud558\uc5ec \uc778\uc2dd\uc744 \uac00\ub2a5\ud558\uac8c \ud55c\ub2e4. Cf\ub294 \ud575 \ubc18\uc751\uae30\uc5d0\uc11c \ub9ce\uc774 \ub9cc\ub4e4\uc5b4\uc9c0\uc9c0 \uc54a\ub294\ub2e4. \uc65c\ub0d0\ud558\uba74 \ubd95\uad34\ub418\uae30 \uc804 Bk\uc758 \ub290\ub9b0 \u03b2-\ubd95\uad34\uc640 \uc911\uc131\uc790\uc640 \uc0c1\ud638 \uc791\uc6a9\uc744 \ud560 \ub54c\uc758 \ud070 \ub2e8\uba74\uc801 \ub54c\ubb38\uc5d0 \ub9cc\ub4e4\uae30\uac00 \uc27d\uc9c0 \uc54a\ub2e4. \uadf8\ub7ec\ub098 Bk\uc758 \u03b2-\ubd95\uad34\uc758 \uc0b0\ubb3c\ub85c \uc778\ud574 \uc21c\uc218\ud55c \ub3d9\uc704 \uc6d0\uc18c\ub85c \ucd95\uc801\ub41c\ub2e4. \uce98\ub9ac\ud3ec\ub284\uc740 \ud50c\ub8e8\ud1a0\ub284\uc758 \ubc18\uc751\uae30 \uc870\uc0ac\uc5d0 \uc758\ud574 \ub9cc\ub4e4\uc5b4\uc9c4\ub2e4. \uc774\uac83\uc740 \uc8fc\ub85c Cf\uacfc Cf\uac00 \uc874\uc7ac\ud558\ub294\ub370, \ud6c4\uc5d0 \ud070 \uc911\uc131\uc790\uc758 \uc601\ud5a5\uc73c\ub85c \uc810\uc810 \ub69c\ub837\ud574\uc9c0\uace0, \uc774\uac83\uc758 \ud2b9\uc9d5\uc740 \uac15\ud55c \uc911\uc131\uc790\uc120\uc5d0 \uc758\ud574 \ubc18\uc751\uc774 \uc800\ud574\ub41c\ub2e4."} {"question": "\ud669\uad6d\uc758 \uc6a9\uad70\uc740 \ub9c8\uc4f0\uc5d0 \uacbd\ucc30\uc11c\uc758 \uc5b4\ub514\uc5d0 \uc218\uc6a9\ub418\uc5c8\uc2b5\ub2c8\uae4c?", "paragraph": "\ud669\uad6d\uc758\uc6a9\uad70\uc740 \ub9c8\uc4f0\uc5d0 \uacbd\ucc30\uc11c\uc758 \uac80\ub3c4\uc7a5\uc5d0 \uc218\uc6a9\ub418\uc5c8\ub2e4. \uc624\uce74\uc790\ud0a4\uc640 \ud2b9\ubcc4\uace0\ub4f1\uacbd\ucc30 \uacfc\uc7a5\uc740 \ubcc4\uc2e4\uc5d0\uc11c \ucc98\uc6b0\uc5d0 \ub300\ud574 \uc774\uc57c\uae30\ub97c \ub098\ub204\uc5c8\ub2e4. \uacfc\uc7a5\uc740 \uac80\uc0ac\uc815\uc73c\ub85c\ubd80\ud130 \"\ud3ed\ub3c4\uc758 \uc11d\ubc29\uc740 \ubd88\uac00\ub2a5\ud558\ub2e4\"\ub294 \ud1b5\uace0\ub97c \ubc1b\uc544 \uba3c\uc800\uc758 \uc57d\uc18d\uc744 \ucca0\ud68c\ud574\ub2ec\ub77c\uace0 \uc624\uce74\uc790\ud0a4\uc5d0\uac8c \uc694\uccad\ud588\ub2e4. \uc624\uce74\uc790\ud0a4\ub294 \ud56d\uc758\ud558\uba70, \ub3d9\uc9c0\ub4e4\uacfc \uc774\uc57c\uae30\ud558\uae30 \uc704\ud574 \uac80\ub3c4\uc7a5\uc5d0 \ub4e4\uc5b4\uc130\uc9c0\ub9cc, \uc774\ubbf8 \ub54c\ub294 \ub2a6\uc5c8\uc74c\uc744 \uae68\ub2ec\uc558\ub2e4. \uadf8 \ub54c\ubb38\uc5d0 \uc624\uce74\uc790\ud0a4\ub294 \uc120\uc57d\ub300\ub85c \uc790\uc2e0\uc758 \uc0dd\uba85\uc744 \uac00\uc9c0\uace0 \ub2e4\ub978 \ub3d9\uc9c0\ub4e4\uc744 \uc11d\ubc29\uc2dc\ud0a4\ub294 \uac83\uc744 \uae30\ub3c4\ud588\ub2e4. \uc624\uce74\uc790\ud0a4\ub294 \uba64\ubc84\ub4e4\uc5d0\uac8c \"\uc870\uc0ac\ub97c \ub2f9\ub2f9\ud788 \ubc1b\uace0 \uc11d\ubc29 \ud6c4\uc5d0\ub294 \uc77c\ubcf8\uc758 \ubd80\ud765\uc744 \uc704\ud574 \uc9c4\ub825\uc744 \ub2e4\ud574\ub2ec\ub77c\"\uba70, \ubd09\uae30\uc758 \uc2e4\ud328\uc5d0 \ub300\ud574 \uc0ac\uacfc\ud588\ub2e4. \uadf8\ub9ac\uace0 \ub2e8\uac80\uc73c\ub85c \uc790\uc2e0\uc758 \ubc30\ub97c \ub450 \ubc88 \ucc14\ub800\ub2e4. \ud2b9\ubcc4\uace0\ub4f1\uacbd\ucc30 \uacfc\uc7a5\uacfc \ubc29\uc1a1\uad6d\uc7a5\uc774 \uc774\ub97c \ub9d0\ub9ac\ub7ec \ub2ec\ub824\uc624\uc790, \ud558\uc138\uac00\uc640\uac00 \uc77c\ubcf8\ub3c4\ub85c \uadf8\ub4e4\uc744 \uc81c\uc9c0\ud588\ub2e4. \uc624\uce74\uc790\ud0a4\ub294 \"\ucc9c\ud669 \ud3d0\ud558 \ub9cc\uc138\"\ub97c \uc678\uce58\uba70 \uc774\ubc88\uc5d0\ub294 \ub2e8\uac80\uc744 \ubaa9\ub35c\ubbf8\uc5d0 \uaf42\uc558\ub2e4. \uac80\ub3c4\uc7a5\uc5d0 \uc788\uc5c8\ub358 \ubaa8\ub4e0 \uba64\ubc84\ub4e4\uc740 \ud1b5\uace1\ud558\uc600\uc73c\uba70, \uc624\uce74\uc790\ud0a4\ub294 \uc758\uc2dd \ubd88\uba85 \uc0c1\ud0dc\ub85c \ub9c8\uc4f0\uc5d0 \uc77c\ubcf8 \uc801\uc2ed\uc790\uc0ac \ubcd1\uc6d0\uc5d0 \uc62e\uaca8\uc838 \ubaa9\uc228\uc744 \uac74\uc84c\ub2e4. \uadf8 \ud6c4, \uc804\uc6d0\uc774 \uc804\uc2dc \uc18c\uc694\u00b7\uc8fc\uac70\uce68\uc785\u00b7\uc804\uc2e0\uac00\uc2a4\uc774\uc6a9\ubc29\ud574\u00b7\ud3ed\ubc1c\ubb3c\ub2e8\uc18d\ubc8c\uce59 \uc704\ubc18 \ub4f1\uc758 \ud610\uc758\ub85c \uc870\uc0ac\ub97c \ubc1b\uc558\uc73c\uba70, \uc8fc\ub3d9\uc790\ub97c \uc81c\uc678\ud55c \uc77c\ubc18 \ub300\uc6d0\ub4e4\uc740 \uadf8 \ub2e4\ub2e4\uc74c\ub0a0\uae4c\uc9c0 \ubaa8\ub450 \uc11d\ubc29\ub418\uc5c8\ub2e4.", "answer": "\uac80\ub3c4\uc7a5", "sentence": "\ud669\uad6d\uc758\uc6a9\uad70\uc740 \ub9c8\uc4f0\uc5d0 \uacbd\ucc30\uc11c\uc758 \uac80\ub3c4\uc7a5 \uc5d0 \uc218\uc6a9\ub418\uc5c8\ub2e4.", "paragraph_sentence": " \ud669\uad6d\uc758\uc6a9\uad70\uc740 \ub9c8\uc4f0\uc5d0 \uacbd\ucc30\uc11c\uc758 \uac80\ub3c4\uc7a5 \uc5d0 \uc218\uc6a9\ub418\uc5c8\ub2e4. \uc624\uce74\uc790\ud0a4\uc640 \ud2b9\ubcc4\uace0\ub4f1\uacbd\ucc30 \uacfc\uc7a5\uc740 \ubcc4\uc2e4\uc5d0\uc11c \ucc98\uc6b0\uc5d0 \ub300\ud574 \uc774\uc57c\uae30\ub97c \ub098\ub204\uc5c8\ub2e4. \uacfc\uc7a5\uc740 \uac80\uc0ac\uc815\uc73c\ub85c\ubd80\ud130 \"\ud3ed\ub3c4\uc758 \uc11d\ubc29\uc740 \ubd88\uac00\ub2a5\ud558\ub2e4\"\ub294 \ud1b5\uace0\ub97c \ubc1b\uc544 \uba3c\uc800\uc758 \uc57d\uc18d\uc744 \ucca0\ud68c\ud574\ub2ec\ub77c\uace0 \uc624\uce74\uc790\ud0a4\uc5d0\uac8c \uc694\uccad\ud588\ub2e4. \uc624\uce74\uc790\ud0a4\ub294 \ud56d\uc758\ud558\uba70, \ub3d9\uc9c0\ub4e4\uacfc \uc774\uc57c\uae30\ud558\uae30 \uc704\ud574 \uac80\ub3c4\uc7a5\uc5d0 \ub4e4\uc5b4\uc130\uc9c0\ub9cc, \uc774\ubbf8 \ub54c\ub294 \ub2a6\uc5c8\uc74c\uc744 \uae68\ub2ec\uc558\ub2e4. \uadf8 \ub54c\ubb38\uc5d0 \uc624\uce74\uc790\ud0a4\ub294 \uc120\uc57d\ub300\ub85c \uc790\uc2e0\uc758 \uc0dd\uba85\uc744 \uac00\uc9c0\uace0 \ub2e4\ub978 \ub3d9\uc9c0\ub4e4\uc744 \uc11d\ubc29\uc2dc\ud0a4\ub294 \uac83\uc744 \uae30\ub3c4\ud588\ub2e4. \uc624\uce74\uc790\ud0a4\ub294 \uba64\ubc84\ub4e4\uc5d0\uac8c \"\uc870\uc0ac\ub97c \ub2f9\ub2f9\ud788 \ubc1b\uace0 \uc11d\ubc29 \ud6c4\uc5d0\ub294 \uc77c\ubcf8\uc758 \ubd80\ud765\uc744 \uc704\ud574 \uc9c4\ub825\uc744 \ub2e4\ud574\ub2ec\ub77c\"\uba70, \ubd09\uae30\uc758 \uc2e4\ud328\uc5d0 \ub300\ud574 \uc0ac\uacfc\ud588\ub2e4. \uadf8\ub9ac\uace0 \ub2e8\uac80\uc73c\ub85c \uc790\uc2e0\uc758 \ubc30\ub97c \ub450 \ubc88 \ucc14\ub800\ub2e4. \ud2b9\ubcc4\uace0\ub4f1\uacbd\ucc30 \uacfc\uc7a5\uacfc \ubc29\uc1a1\uad6d\uc7a5\uc774 \uc774\ub97c \ub9d0\ub9ac\ub7ec \ub2ec\ub824\uc624\uc790, \ud558\uc138\uac00\uc640\uac00 \uc77c\ubcf8\ub3c4\ub85c \uadf8\ub4e4\uc744 \uc81c\uc9c0\ud588\ub2e4. \uc624\uce74\uc790\ud0a4\ub294 \"\ucc9c\ud669 \ud3d0\ud558 \ub9cc\uc138\"\ub97c \uc678\uce58\uba70 \uc774\ubc88\uc5d0\ub294 \ub2e8\uac80\uc744 \ubaa9\ub35c\ubbf8\uc5d0 \uaf42\uc558\ub2e4. \uac80\ub3c4\uc7a5\uc5d0 \uc788\uc5c8\ub358 \ubaa8\ub4e0 \uba64\ubc84\ub4e4\uc740 \ud1b5\uace1\ud558\uc600\uc73c\uba70, \uc624\uce74\uc790\ud0a4\ub294 \uc758\uc2dd \ubd88\uba85 \uc0c1\ud0dc\ub85c \ub9c8\uc4f0\uc5d0 \uc77c\ubcf8 \uc801\uc2ed\uc790\uc0ac \ubcd1\uc6d0\uc5d0 \uc62e\uaca8\uc838 \ubaa9\uc228\uc744 \uac74\uc84c\ub2e4. \uadf8 \ud6c4, \uc804\uc6d0\uc774 \uc804\uc2dc \uc18c\uc694\u00b7\uc8fc\uac70\uce68\uc785\u00b7\uc804\uc2e0\uac00\uc2a4\uc774\uc6a9\ubc29\ud574\u00b7\ud3ed\ubc1c\ubb3c\ub2e8\uc18d\ubc8c\uce59 \uc704\ubc18 \ub4f1\uc758 \ud610\uc758\ub85c \uc870\uc0ac\ub97c \ubc1b\uc558\uc73c\uba70, \uc8fc\ub3d9\uc790\ub97c \uc81c\uc678\ud55c \uc77c\ubc18 \ub300\uc6d0\ub4e4\uc740 \uadf8 \ub2e4\ub2e4\uc74c\ub0a0\uae4c\uc9c0 \ubaa8\ub450 \uc11d\ubc29\ub418\uc5c8\ub2e4.", "paragraph_answer": "\ud669\uad6d\uc758\uc6a9\uad70\uc740 \ub9c8\uc4f0\uc5d0 \uacbd\ucc30\uc11c\uc758 \uac80\ub3c4\uc7a5 \uc5d0 \uc218\uc6a9\ub418\uc5c8\ub2e4. \uc624\uce74\uc790\ud0a4\uc640 \ud2b9\ubcc4\uace0\ub4f1\uacbd\ucc30 \uacfc\uc7a5\uc740 \ubcc4\uc2e4\uc5d0\uc11c \ucc98\uc6b0\uc5d0 \ub300\ud574 \uc774\uc57c\uae30\ub97c \ub098\ub204\uc5c8\ub2e4. \uacfc\uc7a5\uc740 \uac80\uc0ac\uc815\uc73c\ub85c\ubd80\ud130 \"\ud3ed\ub3c4\uc758 \uc11d\ubc29\uc740 \ubd88\uac00\ub2a5\ud558\ub2e4\"\ub294 \ud1b5\uace0\ub97c \ubc1b\uc544 \uba3c\uc800\uc758 \uc57d\uc18d\uc744 \ucca0\ud68c\ud574\ub2ec\ub77c\uace0 \uc624\uce74\uc790\ud0a4\uc5d0\uac8c \uc694\uccad\ud588\ub2e4. \uc624\uce74\uc790\ud0a4\ub294 \ud56d\uc758\ud558\uba70, \ub3d9\uc9c0\ub4e4\uacfc \uc774\uc57c\uae30\ud558\uae30 \uc704\ud574 \uac80\ub3c4\uc7a5\uc5d0 \ub4e4\uc5b4\uc130\uc9c0\ub9cc, \uc774\ubbf8 \ub54c\ub294 \ub2a6\uc5c8\uc74c\uc744 \uae68\ub2ec\uc558\ub2e4. \uadf8 \ub54c\ubb38\uc5d0 \uc624\uce74\uc790\ud0a4\ub294 \uc120\uc57d\ub300\ub85c \uc790\uc2e0\uc758 \uc0dd\uba85\uc744 \uac00\uc9c0\uace0 \ub2e4\ub978 \ub3d9\uc9c0\ub4e4\uc744 \uc11d\ubc29\uc2dc\ud0a4\ub294 \uac83\uc744 \uae30\ub3c4\ud588\ub2e4. \uc624\uce74\uc790\ud0a4\ub294 \uba64\ubc84\ub4e4\uc5d0\uac8c \"\uc870\uc0ac\ub97c \ub2f9\ub2f9\ud788 \ubc1b\uace0 \uc11d\ubc29 \ud6c4\uc5d0\ub294 \uc77c\ubcf8\uc758 \ubd80\ud765\uc744 \uc704\ud574 \uc9c4\ub825\uc744 \ub2e4\ud574\ub2ec\ub77c\"\uba70, \ubd09\uae30\uc758 \uc2e4\ud328\uc5d0 \ub300\ud574 \uc0ac\uacfc\ud588\ub2e4. \uadf8\ub9ac\uace0 \ub2e8\uac80\uc73c\ub85c \uc790\uc2e0\uc758 \ubc30\ub97c \ub450 \ubc88 \ucc14\ub800\ub2e4. \ud2b9\ubcc4\uace0\ub4f1\uacbd\ucc30 \uacfc\uc7a5\uacfc \ubc29\uc1a1\uad6d\uc7a5\uc774 \uc774\ub97c \ub9d0\ub9ac\ub7ec \ub2ec\ub824\uc624\uc790, \ud558\uc138\uac00\uc640\uac00 \uc77c\ubcf8\ub3c4\ub85c \uadf8\ub4e4\uc744 \uc81c\uc9c0\ud588\ub2e4. \uc624\uce74\uc790\ud0a4\ub294 \"\ucc9c\ud669 \ud3d0\ud558 \ub9cc\uc138\"\ub97c \uc678\uce58\uba70 \uc774\ubc88\uc5d0\ub294 \ub2e8\uac80\uc744 \ubaa9\ub35c\ubbf8\uc5d0 \uaf42\uc558\ub2e4. \uac80\ub3c4\uc7a5\uc5d0 \uc788\uc5c8\ub358 \ubaa8\ub4e0 \uba64\ubc84\ub4e4\uc740 \ud1b5\uace1\ud558\uc600\uc73c\uba70, \uc624\uce74\uc790\ud0a4\ub294 \uc758\uc2dd \ubd88\uba85 \uc0c1\ud0dc\ub85c \ub9c8\uc4f0\uc5d0 \uc77c\ubcf8 \uc801\uc2ed\uc790\uc0ac \ubcd1\uc6d0\uc5d0 \uc62e\uaca8\uc838 \ubaa9\uc228\uc744 \uac74\uc84c\ub2e4. \uadf8 \ud6c4, \uc804\uc6d0\uc774 \uc804\uc2dc \uc18c\uc694\u00b7\uc8fc\uac70\uce68\uc785\u00b7\uc804\uc2e0\uac00\uc2a4\uc774\uc6a9\ubc29\ud574\u00b7\ud3ed\ubc1c\ubb3c\ub2e8\uc18d\ubc8c\uce59 \uc704\ubc18 \ub4f1\uc758 \ud610\uc758\ub85c \uc870\uc0ac\ub97c \ubc1b\uc558\uc73c\uba70, \uc8fc\ub3d9\uc790\ub97c \uc81c\uc678\ud55c \uc77c\ubc18 \ub300\uc6d0\ub4e4\uc740 \uadf8 \ub2e4\ub2e4\uc74c\ub0a0\uae4c\uc9c0 \ubaa8\ub450 \uc11d\ubc29\ub418\uc5c8\ub2e4.", "sentence_answer": "\ud669\uad6d\uc758\uc6a9\uad70\uc740 \ub9c8\uc4f0\uc5d0 \uacbd\ucc30\uc11c\uc758 \uac80\ub3c4\uc7a5 \uc5d0 \uc218\uc6a9\ub418\uc5c8\ub2e4.", "paragraph_id": "6534177-17-0", "paragraph_question": "question: \ud669\uad6d\uc758 \uc6a9\uad70\uc740 \ub9c8\uc4f0\uc5d0 \uacbd\ucc30\uc11c\uc758 \uc5b4\ub514\uc5d0 \uc218\uc6a9\ub418\uc5c8\uc2b5\ub2c8\uae4c?, context: \ud669\uad6d\uc758\uc6a9\uad70\uc740 \ub9c8\uc4f0\uc5d0 \uacbd\ucc30\uc11c\uc758 \uac80\ub3c4\uc7a5\uc5d0 \uc218\uc6a9\ub418\uc5c8\ub2e4. \uc624\uce74\uc790\ud0a4\uc640 \ud2b9\ubcc4\uace0\ub4f1\uacbd\ucc30 \uacfc\uc7a5\uc740 \ubcc4\uc2e4\uc5d0\uc11c \ucc98\uc6b0\uc5d0 \ub300\ud574 \uc774\uc57c\uae30\ub97c \ub098\ub204\uc5c8\ub2e4. \uacfc\uc7a5\uc740 \uac80\uc0ac\uc815\uc73c\ub85c\ubd80\ud130 \"\ud3ed\ub3c4\uc758 \uc11d\ubc29\uc740 \ubd88\uac00\ub2a5\ud558\ub2e4\"\ub294 \ud1b5\uace0\ub97c \ubc1b\uc544 \uba3c\uc800\uc758 \uc57d\uc18d\uc744 \ucca0\ud68c\ud574\ub2ec\ub77c\uace0 \uc624\uce74\uc790\ud0a4\uc5d0\uac8c \uc694\uccad\ud588\ub2e4. \uc624\uce74\uc790\ud0a4\ub294 \ud56d\uc758\ud558\uba70, \ub3d9\uc9c0\ub4e4\uacfc \uc774\uc57c\uae30\ud558\uae30 \uc704\ud574 \uac80\ub3c4\uc7a5\uc5d0 \ub4e4\uc5b4\uc130\uc9c0\ub9cc, \uc774\ubbf8 \ub54c\ub294 \ub2a6\uc5c8\uc74c\uc744 \uae68\ub2ec\uc558\ub2e4. \uadf8 \ub54c\ubb38\uc5d0 \uc624\uce74\uc790\ud0a4\ub294 \uc120\uc57d\ub300\ub85c \uc790\uc2e0\uc758 \uc0dd\uba85\uc744 \uac00\uc9c0\uace0 \ub2e4\ub978 \ub3d9\uc9c0\ub4e4\uc744 \uc11d\ubc29\uc2dc\ud0a4\ub294 \uac83\uc744 \uae30\ub3c4\ud588\ub2e4. \uc624\uce74\uc790\ud0a4\ub294 \uba64\ubc84\ub4e4\uc5d0\uac8c \"\uc870\uc0ac\ub97c \ub2f9\ub2f9\ud788 \ubc1b\uace0 \uc11d\ubc29 \ud6c4\uc5d0\ub294 \uc77c\ubcf8\uc758 \ubd80\ud765\uc744 \uc704\ud574 \uc9c4\ub825\uc744 \ub2e4\ud574\ub2ec\ub77c\"\uba70, \ubd09\uae30\uc758 \uc2e4\ud328\uc5d0 \ub300\ud574 \uc0ac\uacfc\ud588\ub2e4. \uadf8\ub9ac\uace0 \ub2e8\uac80\uc73c\ub85c \uc790\uc2e0\uc758 \ubc30\ub97c \ub450 \ubc88 \ucc14\ub800\ub2e4. \ud2b9\ubcc4\uace0\ub4f1\uacbd\ucc30 \uacfc\uc7a5\uacfc \ubc29\uc1a1\uad6d\uc7a5\uc774 \uc774\ub97c \ub9d0\ub9ac\ub7ec \ub2ec\ub824\uc624\uc790, \ud558\uc138\uac00\uc640\uac00 \uc77c\ubcf8\ub3c4\ub85c \uadf8\ub4e4\uc744 \uc81c\uc9c0\ud588\ub2e4. \uc624\uce74\uc790\ud0a4\ub294 \"\ucc9c\ud669 \ud3d0\ud558 \ub9cc\uc138\"\ub97c \uc678\uce58\uba70 \uc774\ubc88\uc5d0\ub294 \ub2e8\uac80\uc744 \ubaa9\ub35c\ubbf8\uc5d0 \uaf42\uc558\ub2e4. \uac80\ub3c4\uc7a5\uc5d0 \uc788\uc5c8\ub358 \ubaa8\ub4e0 \uba64\ubc84\ub4e4\uc740 \ud1b5\uace1\ud558\uc600\uc73c\uba70, \uc624\uce74\uc790\ud0a4\ub294 \uc758\uc2dd \ubd88\uba85 \uc0c1\ud0dc\ub85c \ub9c8\uc4f0\uc5d0 \uc77c\ubcf8 \uc801\uc2ed\uc790\uc0ac \ubcd1\uc6d0\uc5d0 \uc62e\uaca8\uc838 \ubaa9\uc228\uc744 \uac74\uc84c\ub2e4. \uadf8 \ud6c4, \uc804\uc6d0\uc774 \uc804\uc2dc \uc18c\uc694\u00b7\uc8fc\uac70\uce68\uc785\u00b7\uc804\uc2e0\uac00\uc2a4\uc774\uc6a9\ubc29\ud574\u00b7\ud3ed\ubc1c\ubb3c\ub2e8\uc18d\ubc8c\uce59 \uc704\ubc18 \ub4f1\uc758 \ud610\uc758\ub85c \uc870\uc0ac\ub97c \ubc1b\uc558\uc73c\uba70, \uc8fc\ub3d9\uc790\ub97c \uc81c\uc678\ud55c \uc77c\ubc18 \ub300\uc6d0\ub4e4\uc740 \uadf8 \ub2e4\ub2e4\uc74c\ub0a0\uae4c\uc9c0 \ubaa8\ub450 \uc11d\ubc29\ub418\uc5c8\ub2e4."} {"question": "\uc7ad \ucff0\uc2a4\ub294 \uc5b8\uc81c \uc8fd\uc5c8\ub294\uac00?", "paragraph": "1931\ub144, \uc57c\uad6c\uc758 \uaddc\uce59\uacfc \uc804\uc220\uc744 \ub2e4\ub8ec \ucc45\uc778 \u300aBaseball: Individual Play and Team Strategy\u300b\ub97c \ud3b4\ub0c8\ub2e4. 1937\ub144\uacfc 38\ub144, 1946\ub144\uacfc 48\ub144, \uadf8\ub9ac\uace0 1951\ub144\uc5d0\ub294 \ubbf8\uad6d \uc57c\uad6c \uba85\uc608\uc758 \uc804\ub2f9 \ud22c\ud45c\uc5d0 \ud6c4\ubcf4\ub85c \uc774\ub984\uc744 \uc62c\ub824 \ud55c \ud45c \uc774\uc0c1\uc744 \ud68d\ub4dd\ud588\ub2e4. 1957\ub144 4\uc6d4 15\uc77c, \ud314\ub808\uc2a4\ud0c0\uc778\uc5d0 \uc788\ub294 \uc790\uc2e0\uc758 \uc9d1\uc5d0\uc11c \uc2ec\uc7a5\ub9c8\ube44\ub85c \uc0ac\ub9dd\ud588\ub2e4. \uc2dc\uc2e0\uc740 \ud314\ub808\uc2a4\ud0c0\uc778\uc758 \uc138\uc778\ud2b8 \uc870\uc9c0\ud504 \uacf5\ub3d9\ubb18\uc9c0\uc5d0 \uc548\uc7a5\ub418\uc5c8\ub2e4. 1987\ub144, \ub178\uc2a4\uce90\ub864\ub77c\uc774\ub098 \uc2a4\ud3ec\uce20 \uba85\uc608\uc758 \uc804\ub2f9\uc5d0 \ud5cc\uc815\ub418\uc5c8\ub2e4. 1992\ub144, \ud544\ub77c\ub378\ud53c\uc544\uc5d0\uc11c \ub6f0\uc5c8\ub358 \uc57c\uad6c \uc120\uc218\ub4e4\uc744 \uae30\ub9ac\uae30 \uc704\ud55c \uc804\ub2f9\uc778 \ud544\ub77c\ub378\ud53c\uc544 \uc57c\uad6c \uba85\uc608\uc758 \ubcbd\uc5d0 \ud5cc\uc815\ub418\uc5c8\ub2e4. \ud604\uc7ac \ub4c0\ud06c \ub300\ud559\uad50 \uc57c\uad6c\ubd80\uac00 \uc0ac\uc6a9\ud558\uace0 \uc788\ub294 \uc57c\uad6c\uc7a5\uc758 \uba85\uce6d\uc740 \uc7ad \ucff0\uc2a4 \ud544\ub4dc\uc774\ub2e4.", "answer": "1957\ub144 4\uc6d4 15\uc77c", "sentence": "1957\ub144 4\uc6d4 15\uc77c ,", "paragraph_sentence": "1931\ub144, \uc57c\uad6c\uc758 \uaddc\uce59\uacfc \uc804\uc220\uc744 \ub2e4\ub8ec \ucc45\uc778 \u300aBaseball: Individual Play and Team Strategy\u300b\ub97c \ud3b4\ub0c8\ub2e4. 1937\ub144\uacfc 38\ub144, 1946\ub144\uacfc 48\ub144, \uadf8\ub9ac\uace0 1951\ub144\uc5d0\ub294 \ubbf8\uad6d \uc57c\uad6c \uba85\uc608\uc758 \uc804\ub2f9 \ud22c\ud45c\uc5d0 \ud6c4\ubcf4\ub85c \uc774\ub984\uc744 \uc62c\ub824 \ud55c \ud45c \uc774\uc0c1\uc744 \ud68d\ub4dd\ud588\ub2e4. 1957\ub144 4\uc6d4 15\uc77c , \ud314\ub808\uc2a4\ud0c0\uc778\uc5d0 \uc788\ub294 \uc790\uc2e0\uc758 \uc9d1\uc5d0\uc11c \uc2ec\uc7a5\ub9c8\ube44\ub85c \uc0ac\ub9dd\ud588\ub2e4. \uc2dc\uc2e0\uc740 \ud314\ub808\uc2a4\ud0c0\uc778\uc758 \uc138\uc778\ud2b8 \uc870\uc9c0\ud504 \uacf5\ub3d9\ubb18\uc9c0\uc5d0 \uc548\uc7a5\ub418\uc5c8\ub2e4. 1987\ub144, \ub178\uc2a4\uce90\ub864\ub77c\uc774\ub098 \uc2a4\ud3ec\uce20 \uba85\uc608\uc758 \uc804\ub2f9\uc5d0 \ud5cc\uc815\ub418\uc5c8\ub2e4. 1992\ub144, \ud544\ub77c\ub378\ud53c\uc544\uc5d0\uc11c \ub6f0\uc5c8\ub358 \uc57c\uad6c \uc120\uc218\ub4e4\uc744 \uae30\ub9ac\uae30 \uc704\ud55c \uc804\ub2f9\uc778 \ud544\ub77c\ub378\ud53c\uc544 \uc57c\uad6c \uba85\uc608\uc758 \ubcbd\uc5d0 \ud5cc\uc815\ub418\uc5c8\ub2e4. \ud604\uc7ac \ub4c0\ud06c \ub300\ud559\uad50 \uc57c\uad6c\ubd80\uac00 \uc0ac\uc6a9\ud558\uace0 \uc788\ub294 \uc57c\uad6c\uc7a5\uc758 \uba85\uce6d\uc740 \uc7ad \ucff0\uc2a4 \ud544\ub4dc\uc774\ub2e4.", "paragraph_answer": "1931\ub144, \uc57c\uad6c\uc758 \uaddc\uce59\uacfc \uc804\uc220\uc744 \ub2e4\ub8ec \ucc45\uc778 \u300aBaseball: Individual Play and Team Strategy\u300b\ub97c \ud3b4\ub0c8\ub2e4. 1937\ub144\uacfc 38\ub144, 1946\ub144\uacfc 48\ub144, \uadf8\ub9ac\uace0 1951\ub144\uc5d0\ub294 \ubbf8\uad6d \uc57c\uad6c \uba85\uc608\uc758 \uc804\ub2f9 \ud22c\ud45c\uc5d0 \ud6c4\ubcf4\ub85c \uc774\ub984\uc744 \uc62c\ub824 \ud55c \ud45c \uc774\uc0c1\uc744 \ud68d\ub4dd\ud588\ub2e4. 1957\ub144 4\uc6d4 15\uc77c , \ud314\ub808\uc2a4\ud0c0\uc778\uc5d0 \uc788\ub294 \uc790\uc2e0\uc758 \uc9d1\uc5d0\uc11c \uc2ec\uc7a5\ub9c8\ube44\ub85c \uc0ac\ub9dd\ud588\ub2e4. \uc2dc\uc2e0\uc740 \ud314\ub808\uc2a4\ud0c0\uc778\uc758 \uc138\uc778\ud2b8 \uc870\uc9c0\ud504 \uacf5\ub3d9\ubb18\uc9c0\uc5d0 \uc548\uc7a5\ub418\uc5c8\ub2e4. 1987\ub144, \ub178\uc2a4\uce90\ub864\ub77c\uc774\ub098 \uc2a4\ud3ec\uce20 \uba85\uc608\uc758 \uc804\ub2f9\uc5d0 \ud5cc\uc815\ub418\uc5c8\ub2e4. 1992\ub144, \ud544\ub77c\ub378\ud53c\uc544\uc5d0\uc11c \ub6f0\uc5c8\ub358 \uc57c\uad6c \uc120\uc218\ub4e4\uc744 \uae30\ub9ac\uae30 \uc704\ud55c \uc804\ub2f9\uc778 \ud544\ub77c\ub378\ud53c\uc544 \uc57c\uad6c \uba85\uc608\uc758 \ubcbd\uc5d0 \ud5cc\uc815\ub418\uc5c8\ub2e4. \ud604\uc7ac \ub4c0\ud06c \ub300\ud559\uad50 \uc57c\uad6c\ubd80\uac00 \uc0ac\uc6a9\ud558\uace0 \uc788\ub294 \uc57c\uad6c\uc7a5\uc758 \uba85\uce6d\uc740 \uc7ad \ucff0\uc2a4 \ud544\ub4dc\uc774\ub2e4.", "sentence_answer": " 1957\ub144 4\uc6d4 15\uc77c ,", "paragraph_id": "6608933-14-1", "paragraph_question": "question: \uc7ad \ucff0\uc2a4\ub294 \uc5b8\uc81c \uc8fd\uc5c8\ub294\uac00?, context: 1931\ub144, \uc57c\uad6c\uc758 \uaddc\uce59\uacfc \uc804\uc220\uc744 \ub2e4\ub8ec \ucc45\uc778 \u300aBaseball: Individual Play and Team Strategy\u300b\ub97c \ud3b4\ub0c8\ub2e4. 1937\ub144\uacfc 38\ub144, 1946\ub144\uacfc 48\ub144, \uadf8\ub9ac\uace0 1951\ub144\uc5d0\ub294 \ubbf8\uad6d \uc57c\uad6c \uba85\uc608\uc758 \uc804\ub2f9 \ud22c\ud45c\uc5d0 \ud6c4\ubcf4\ub85c \uc774\ub984\uc744 \uc62c\ub824 \ud55c \ud45c \uc774\uc0c1\uc744 \ud68d\ub4dd\ud588\ub2e4. 1957\ub144 4\uc6d4 15\uc77c, \ud314\ub808\uc2a4\ud0c0\uc778\uc5d0 \uc788\ub294 \uc790\uc2e0\uc758 \uc9d1\uc5d0\uc11c \uc2ec\uc7a5\ub9c8\ube44\ub85c \uc0ac\ub9dd\ud588\ub2e4. \uc2dc\uc2e0\uc740 \ud314\ub808\uc2a4\ud0c0\uc778\uc758 \uc138\uc778\ud2b8 \uc870\uc9c0\ud504 \uacf5\ub3d9\ubb18\uc9c0\uc5d0 \uc548\uc7a5\ub418\uc5c8\ub2e4. 1987\ub144, \ub178\uc2a4\uce90\ub864\ub77c\uc774\ub098 \uc2a4\ud3ec\uce20 \uba85\uc608\uc758 \uc804\ub2f9\uc5d0 \ud5cc\uc815\ub418\uc5c8\ub2e4. 1992\ub144, \ud544\ub77c\ub378\ud53c\uc544\uc5d0\uc11c \ub6f0\uc5c8\ub358 \uc57c\uad6c \uc120\uc218\ub4e4\uc744 \uae30\ub9ac\uae30 \uc704\ud55c \uc804\ub2f9\uc778 \ud544\ub77c\ub378\ud53c\uc544 \uc57c\uad6c \uba85\uc608\uc758 \ubcbd\uc5d0 \ud5cc\uc815\ub418\uc5c8\ub2e4. \ud604\uc7ac \ub4c0\ud06c \ub300\ud559\uad50 \uc57c\uad6c\ubd80\uac00 \uc0ac\uc6a9\ud558\uace0 \uc788\ub294 \uc57c\uad6c\uc7a5\uc758 \uba85\uce6d\uc740 \uc7ad \ucff0\uc2a4 \ud544\ub4dc\uc774\ub2e4."} {"question": "\uacf5\uc790\uc758 \uc0ac\uc0c1\uc744 \uc774\uc5b4 \ubc1b\uc740 \uc0ac\ub78c\uc740?", "paragraph": "\ub9f9\uc790\ub294 \uacf5\uc790\uc758 \uc0ac\uc0c1\uc744 \uacc4\uc2b9\ud558\uace0 \uacf5\uc790\uac00 \uc218\ub9bd\ud55c \uc778\uac04\uc758 \uc2e4\ucc9c\uc801 \uc8fc\uccb4\uc131\uc774\ub098 \ub355\uc5d0 \uc758\ud55c \uc815\uce58\ub77c\ub294 \uc0ac\uace0\ubc29\uc2dd\uc744 \uc804\ud1b5\uc801\uc778 \ud558\ub298\uc758 \uc2e0\uc559\uacfc \uacb0\ubd80\uc2dc\ud0b4\uc73c\ub85c\uc368 \uc774\ub97c \ubc1c\uc804\uc2dc\ucf30\ub2e4. \uc131\uc120\uc124\uc774\ub098 \uc655\ub3c4\ub860(\u738b\u9053\u8ad6)\uc5d0\uc11c \uadf8 \uacbd\uc704\ub97c \ubcfc \uc218 \uc788\ub2e4. \uadf8\ub7ec\ub098 \ub9f9\uc790\ub294 \ub610\ud55c 5\ubc31\ub144\ub9c8\ub2e4 \uc131\uc778\uc774 \ucd9c\ud604\ud55c\ub2e4\uace0 \ud558\ub294 \uc77c\uc885\uc758 \uc21c\ud658\ub860\uc801 \uc5ed\uc0ac\uad00\uc5d0 \uc758\uac70\ud558\uc5ec \uacf5\uc790\uc758 \uc815\ub2f9\uc131\uc744 \uc99d\uba85\ud558\ub824\uace0 \ud55c\ub2e4. \uc131\uc778\uc758 \uc804\ud615\uc774\ub77c\ub294 \uc804\uc124\uc0c1\uc758 \uc81c\uc655(\u5e1d\u738b)\uc778 \uc694\u00b7\uc21c(\u582f\u00b7\u821c)\ubd80\ud130 5\ubc31\ub144\ucbe4 \uc9c0\ub098 \uc740\uc758 \ud0d5\uc655(\u6e6f\u738b)\uc774 \ub098\uc624\uace0, \ud0d5\uc655\uc5d0\uc11c 5\ubc31\ub144\ucbe4 \uc9c0\ub098 \uc8fc\ub098\ub77c\uc758 \ubb38\uc655(\u6587\u738b), \ubb38\uc655\uc5d0\uc11c 5\ubc31\ub144\ucbe4 \uc9c0\ub098 \uacf5\uc790\uac00 \ub098\uc640\uc11c \uc120\uc655(\u5148\u738b)\uc758 \ub3c4(\u9053)\ub97c \uc804\ud588\ub2e4\ub294 \uac83\uc774\ub2e4. \uadf8\ub9ac\uace0 \ub9f9\uc790 \uc790\uc2e0\uc740 \uacf5\uc790\ubd80\ud130 \ub2f9\uc2dc\uae4c\uc9c0 1\ubc31\ub144\ucbe4, \uacf5\uc790\uac00 \uc138\uc0c1\uc744 \ub5a0\ub09c \ub4a4\ubd80\ud130\ub294 \uc544\uc9c1 \uc5bc\ub9c8 \ub418\uc9c0 \uc54a\uc544\uc11c \uc790\uae30\ub294 \uacf5\uc790\uc758 \uae38\uc744 \uc720\uc9c0 \ud655\ubcf4\ud558\ub294 \uc790\ub85c \uc704\uce58\uac00 \uc815\ud574\uc838 \uc788\ub2e4\ub294 \uac83\uc774\ub2e4.", "answer": "\ub9f9\uc790", "sentence": "\ub9f9\uc790 \ub294 \uacf5\uc790\uc758 \uc0ac\uc0c1\uc744 \uacc4\uc2b9\ud558\uace0 \uacf5\uc790\uac00 \uc218\ub9bd\ud55c \uc778\uac04\uc758 \uc2e4\ucc9c\uc801 \uc8fc\uccb4\uc131\uc774\ub098 \ub355\uc5d0 \uc758\ud55c \uc815\uce58\ub77c\ub294 \uc0ac\uace0\ubc29\uc2dd\uc744 \uc804\ud1b5\uc801\uc778 \ud558\ub298\uc758 \uc2e0\uc559\uacfc \uacb0\ubd80\uc2dc\ud0b4\uc73c\ub85c\uc368 \uc774\ub97c \ubc1c\uc804\uc2dc\ucf30\ub2e4.", "paragraph_sentence": " \ub9f9\uc790 \ub294 \uacf5\uc790\uc758 \uc0ac\uc0c1\uc744 \uacc4\uc2b9\ud558\uace0 \uacf5\uc790\uac00 \uc218\ub9bd\ud55c \uc778\uac04\uc758 \uc2e4\ucc9c\uc801 \uc8fc\uccb4\uc131\uc774\ub098 \ub355\uc5d0 \uc758\ud55c \uc815\uce58\ub77c\ub294 \uc0ac\uace0\ubc29\uc2dd\uc744 \uc804\ud1b5\uc801\uc778 \ud558\ub298\uc758 \uc2e0\uc559\uacfc \uacb0\ubd80\uc2dc\ud0b4\uc73c\ub85c\uc368 \uc774\ub97c \ubc1c\uc804\uc2dc\ucf30\ub2e4. \uc131\uc120\uc124\uc774\ub098 \uc655\ub3c4\ub860(\u738b\u9053\u8ad6)\uc5d0\uc11c \uadf8 \uacbd\uc704\ub97c \ubcfc \uc218 \uc788\ub2e4. \uadf8\ub7ec\ub098 \ub9f9\uc790\ub294 \ub610\ud55c 5\ubc31\ub144\ub9c8\ub2e4 \uc131\uc778\uc774 \ucd9c\ud604\ud55c\ub2e4\uace0 \ud558\ub294 \uc77c\uc885\uc758 \uc21c\ud658\ub860\uc801 \uc5ed\uc0ac\uad00\uc5d0 \uc758\uac70\ud558\uc5ec \uacf5\uc790\uc758 \uc815\ub2f9\uc131\uc744 \uc99d\uba85\ud558\ub824\uace0 \ud55c\ub2e4. \uc131\uc778\uc758 \uc804\ud615\uc774\ub77c\ub294 \uc804\uc124\uc0c1\uc758 \uc81c\uc655(\u5e1d\u738b)\uc778 \uc694\u00b7\uc21c(\u582f\u00b7\u821c)\ubd80\ud130 5\ubc31\ub144\ucbe4 \uc9c0\ub098 \uc740\uc758 \ud0d5\uc655(\u6e6f\u738b)\uc774 \ub098\uc624\uace0, \ud0d5\uc655\uc5d0\uc11c 5\ubc31\ub144\ucbe4 \uc9c0\ub098 \uc8fc\ub098\ub77c\uc758 \ubb38\uc655(\u6587\u738b), \ubb38\uc655\uc5d0\uc11c 5\ubc31\ub144\ucbe4 \uc9c0\ub098 \uacf5\uc790\uac00 \ub098\uc640\uc11c \uc120\uc655(\u5148\u738b)\uc758 \ub3c4(\u9053)\ub97c \uc804\ud588\ub2e4\ub294 \uac83\uc774\ub2e4. \uadf8\ub9ac\uace0 \ub9f9\uc790 \uc790\uc2e0\uc740 \uacf5\uc790\ubd80\ud130 \ub2f9\uc2dc\uae4c\uc9c0 1\ubc31\ub144\ucbe4, \uacf5\uc790\uac00 \uc138\uc0c1\uc744 \ub5a0\ub09c \ub4a4\ubd80\ud130\ub294 \uc544\uc9c1 \uc5bc\ub9c8 \ub418\uc9c0 \uc54a\uc544\uc11c \uc790\uae30\ub294 \uacf5\uc790\uc758 \uae38\uc744 \uc720\uc9c0 \ud655\ubcf4\ud558\ub294 \uc790\ub85c \uc704\uce58\uac00 \uc815\ud574\uc838 \uc788\ub2e4\ub294 \uac83\uc774\ub2e4.", "paragraph_answer": " \ub9f9\uc790 \ub294 \uacf5\uc790\uc758 \uc0ac\uc0c1\uc744 \uacc4\uc2b9\ud558\uace0 \uacf5\uc790\uac00 \uc218\ub9bd\ud55c \uc778\uac04\uc758 \uc2e4\ucc9c\uc801 \uc8fc\uccb4\uc131\uc774\ub098 \ub355\uc5d0 \uc758\ud55c \uc815\uce58\ub77c\ub294 \uc0ac\uace0\ubc29\uc2dd\uc744 \uc804\ud1b5\uc801\uc778 \ud558\ub298\uc758 \uc2e0\uc559\uacfc \uacb0\ubd80\uc2dc\ud0b4\uc73c\ub85c\uc368 \uc774\ub97c \ubc1c\uc804\uc2dc\ucf30\ub2e4. \uc131\uc120\uc124\uc774\ub098 \uc655\ub3c4\ub860(\u738b\u9053\u8ad6)\uc5d0\uc11c \uadf8 \uacbd\uc704\ub97c \ubcfc \uc218 \uc788\ub2e4. \uadf8\ub7ec\ub098 \ub9f9\uc790\ub294 \ub610\ud55c 5\ubc31\ub144\ub9c8\ub2e4 \uc131\uc778\uc774 \ucd9c\ud604\ud55c\ub2e4\uace0 \ud558\ub294 \uc77c\uc885\uc758 \uc21c\ud658\ub860\uc801 \uc5ed\uc0ac\uad00\uc5d0 \uc758\uac70\ud558\uc5ec \uacf5\uc790\uc758 \uc815\ub2f9\uc131\uc744 \uc99d\uba85\ud558\ub824\uace0 \ud55c\ub2e4. \uc131\uc778\uc758 \uc804\ud615\uc774\ub77c\ub294 \uc804\uc124\uc0c1\uc758 \uc81c\uc655(\u5e1d\u738b)\uc778 \uc694\u00b7\uc21c(\u582f\u00b7\u821c)\ubd80\ud130 5\ubc31\ub144\ucbe4 \uc9c0\ub098 \uc740\uc758 \ud0d5\uc655(\u6e6f\u738b)\uc774 \ub098\uc624\uace0, \ud0d5\uc655\uc5d0\uc11c 5\ubc31\ub144\ucbe4 \uc9c0\ub098 \uc8fc\ub098\ub77c\uc758 \ubb38\uc655(\u6587\u738b), \ubb38\uc655\uc5d0\uc11c 5\ubc31\ub144\ucbe4 \uc9c0\ub098 \uacf5\uc790\uac00 \ub098\uc640\uc11c \uc120\uc655(\u5148\u738b)\uc758 \ub3c4(\u9053)\ub97c \uc804\ud588\ub2e4\ub294 \uac83\uc774\ub2e4. \uadf8\ub9ac\uace0 \ub9f9\uc790 \uc790\uc2e0\uc740 \uacf5\uc790\ubd80\ud130 \ub2f9\uc2dc\uae4c\uc9c0 1\ubc31\ub144\ucbe4, \uacf5\uc790\uac00 \uc138\uc0c1\uc744 \ub5a0\ub09c \ub4a4\ubd80\ud130\ub294 \uc544\uc9c1 \uc5bc\ub9c8 \ub418\uc9c0 \uc54a\uc544\uc11c \uc790\uae30\ub294 \uacf5\uc790\uc758 \uae38\uc744 \uc720\uc9c0 \ud655\ubcf4\ud558\ub294 \uc790\ub85c \uc704\uce58\uac00 \uc815\ud574\uc838 \uc788\ub2e4\ub294 \uac83\uc774\ub2e4.", "sentence_answer": " \ub9f9\uc790 \ub294 \uacf5\uc790\uc758 \uc0ac\uc0c1\uc744 \uacc4\uc2b9\ud558\uace0 \uacf5\uc790\uac00 \uc218\ub9bd\ud55c \uc778\uac04\uc758 \uc2e4\ucc9c\uc801 \uc8fc\uccb4\uc131\uc774\ub098 \ub355\uc5d0 \uc758\ud55c \uc815\uce58\ub77c\ub294 \uc0ac\uace0\ubc29\uc2dd\uc744 \uc804\ud1b5\uc801\uc778 \ud558\ub298\uc758 \uc2e0\uc559\uacfc \uacb0\ubd80\uc2dc\ud0b4\uc73c\ub85c\uc368 \uc774\ub97c \ubc1c\uc804\uc2dc\ucf30\ub2e4.", "paragraph_id": "6502322-2-0", "paragraph_question": "question: \uacf5\uc790\uc758 \uc0ac\uc0c1\uc744 \uc774\uc5b4 \ubc1b\uc740 \uc0ac\ub78c\uc740?, context: \ub9f9\uc790\ub294 \uacf5\uc790\uc758 \uc0ac\uc0c1\uc744 \uacc4\uc2b9\ud558\uace0 \uacf5\uc790\uac00 \uc218\ub9bd\ud55c \uc778\uac04\uc758 \uc2e4\ucc9c\uc801 \uc8fc\uccb4\uc131\uc774\ub098 \ub355\uc5d0 \uc758\ud55c \uc815\uce58\ub77c\ub294 \uc0ac\uace0\ubc29\uc2dd\uc744 \uc804\ud1b5\uc801\uc778 \ud558\ub298\uc758 \uc2e0\uc559\uacfc \uacb0\ubd80\uc2dc\ud0b4\uc73c\ub85c\uc368 \uc774\ub97c \ubc1c\uc804\uc2dc\ucf30\ub2e4. \uc131\uc120\uc124\uc774\ub098 \uc655\ub3c4\ub860(\u738b\u9053\u8ad6)\uc5d0\uc11c \uadf8 \uacbd\uc704\ub97c \ubcfc \uc218 \uc788\ub2e4. \uadf8\ub7ec\ub098 \ub9f9\uc790\ub294 \ub610\ud55c 5\ubc31\ub144\ub9c8\ub2e4 \uc131\uc778\uc774 \ucd9c\ud604\ud55c\ub2e4\uace0 \ud558\ub294 \uc77c\uc885\uc758 \uc21c\ud658\ub860\uc801 \uc5ed\uc0ac\uad00\uc5d0 \uc758\uac70\ud558\uc5ec \uacf5\uc790\uc758 \uc815\ub2f9\uc131\uc744 \uc99d\uba85\ud558\ub824\uace0 \ud55c\ub2e4. \uc131\uc778\uc758 \uc804\ud615\uc774\ub77c\ub294 \uc804\uc124\uc0c1\uc758 \uc81c\uc655(\u5e1d\u738b)\uc778 \uc694\u00b7\uc21c(\u582f\u00b7\u821c)\ubd80\ud130 5\ubc31\ub144\ucbe4 \uc9c0\ub098 \uc740\uc758 \ud0d5\uc655(\u6e6f\u738b)\uc774 \ub098\uc624\uace0, \ud0d5\uc655\uc5d0\uc11c 5\ubc31\ub144\ucbe4 \uc9c0\ub098 \uc8fc\ub098\ub77c\uc758 \ubb38\uc655(\u6587\u738b), \ubb38\uc655\uc5d0\uc11c 5\ubc31\ub144\ucbe4 \uc9c0\ub098 \uacf5\uc790\uac00 \ub098\uc640\uc11c \uc120\uc655(\u5148\u738b)\uc758 \ub3c4(\u9053)\ub97c \uc804\ud588\ub2e4\ub294 \uac83\uc774\ub2e4. \uadf8\ub9ac\uace0 \ub9f9\uc790 \uc790\uc2e0\uc740 \uacf5\uc790\ubd80\ud130 \ub2f9\uc2dc\uae4c\uc9c0 1\ubc31\ub144\ucbe4, \uacf5\uc790\uac00 \uc138\uc0c1\uc744 \ub5a0\ub09c \ub4a4\ubd80\ud130\ub294 \uc544\uc9c1 \uc5bc\ub9c8 \ub418\uc9c0 \uc54a\uc544\uc11c \uc790\uae30\ub294 \uacf5\uc790\uc758 \uae38\uc744 \uc720\uc9c0 \ud655\ubcf4\ud558\ub294 \uc790\ub85c \uc704\uce58\uac00 \uc815\ud574\uc838 \uc788\ub2e4\ub294 \uac83\uc774\ub2e4."} {"question": "1914\ub144 11\uc6d4 16\uc77c \ub9cc\uc288\ud0c0\uc778\uc740 \ubb34\uc5c7\uc744 \uacf5\uaca9\ud558\ub2e4\uac00 \ucd1d\uc5d0 \ub9de\ub294 \ubd80\uc0c1\uc744 \uc785\uc5c8\ub098?", "paragraph": "1\ucc28\ub300\uc804 \ub2f9\uc2dc \ub9cc\uc288\ud0c0\uc778\uc740 \uc11c\ubd80\uc804\uc120\uacfc \ub3d9\ubd80\uc804\uc120\uc5d0\uc11c \ubaa8\ub450 \ubcf5\ubb34\ud588\ub2e4. \uc804\uc7c1 \uac1c\uc804 \uc2dc\uc810\uc5d0\uc11c \uc911\uc704\uc600\ub358 \ub9cc\uc288\ud0c0\uc778\uc740 \uc81c2\uce5c\uc704\uc608\ube44\ubcf4\ubcd1\uc5f0\ub300\uc5d0 \uc18d\ud558\uc5ec \ubca8\uae30\uc5d0 \uce68\uacf5\uc5d0 \uc885\uad70\ud588\ub2e4. \ub9cc\uc288\ud0c0\uc778\uc758 \ubd80\ub300\ub294 1914\ub144 8\uc6d4 \ub098\ubb88\ub974\ub97c \ud568\ub77d\uc2dc\ucf30\uace0, 9\uc6d4\uc5d0\ub294 \ub3d9\ud504\ub85c\uc774\uc13c\uc73c\ub85c \uc774\ub3d9\ud558\uc5ec \uc774\ubaa8\ubd80 \ud78c\ub374\ubd80\ub974\ud06c\uac00 \uc9c0\ud718\ud558\ub294 \uc81c8\uad70\uc5d0 \ubc30\uc18d\ub418\uc5c8\ub2e4. \uc81c1\ucc28 \ub9c8\uc218\ub9ac\uc548 \ud638 \uc804\ud22c\uc5d0 \ucc38\uc5ec\ud55c \ub4a4 \ub9cc\uc288\ud0c0\uc778\uc758 \ubd80\ub300\ub294 \uc81c9\uad70\uc5d0 \ubc30\uc18d\ub418\uc5b4 \uc624\ubca0\uc5b4\uc290\ub808\uc9c0\uc5d4\uc5d0\uc11c \ubc14\ub974\uc0e4\ubc14\ub85c \uc9c4\uad70\ud588\ub2e4. \uadf8\ub7ec\ub098 \uc81c9\uad70\uc740 \ub7ec\uc2dc\uc544\uad70\uc758 \ubc18\uaca9\uc744 \ubc1b\uc544 \ud6c4\ud1f4\ud588\uace0, \ub9cc\uc288\ud0c0\uc778\uc740 \ud6c4\ud1f4 \uc640\uc911 \ubd84\uacac\ub300\ub97c \uc774\ub04c\uace0 \ub7ec\uc2dc\uc544 \ucc38\ud638\uc120\uc744 \uacf5\uaca9\ud558\ub2e4\uac00 \uc67c\ucabd \uc5b4\uae68\uc640 \uc67c\ucabd \ubb34\ub98e\uc5d0 \ucd1d\uc744 \ub9de\ub294 \ubd80\uc0c1\uc744 \uc785\uc5c8\ub2e4(11\uc6d4 16\uc77c). \ucd1d\uc54c \ud55c \ubc1c\uc774 \uc88c\uace8\uc2e0\uacbd\uc744 \ub04a\uc5b4\uc11c \ub9cc\uc288\ud0c0\uc778\uc740 \ub2e4\ub9ac\uac00 \ub9c8\ube44\ub418\uc5c8\ub2e4. \ud68c\ubcf5\ub418\ub294 \ub370\ub294 6\uac1c\uc6d4\uc774 \uac78\ub838\uace0 \uadf8 \ub3d9\uc548 \ub9cc\uc288\ud0c0\uc778\uc740 \ube44\ud1b0\uacfc \ube44\uc2a4\ubc14\ub374\uc5d0\uc11c \ubcd1\uc6d0\uc2e0\uc138\ub97c \uc84c\ub2e4.", "answer": "\ub7ec\uc2dc\uc544 \ucc38\ud638\uc120", "sentence": "\uadf8\ub7ec\ub098 \uc81c9\uad70\uc740 \ub7ec\uc2dc\uc544\uad70\uc758 \ubc18\uaca9\uc744 \ubc1b\uc544 \ud6c4\ud1f4\ud588\uace0, \ub9cc\uc288\ud0c0\uc778\uc740 \ud6c4\ud1f4 \uc640\uc911 \ubd84\uacac\ub300\ub97c \uc774\ub04c\uace0 \ub7ec\uc2dc\uc544 \ucc38\ud638\uc120 \uc744 \uacf5\uaca9\ud558\ub2e4\uac00 \uc67c\ucabd \uc5b4\uae68\uc640 \uc67c\ucabd \ubb34\ub98e\uc5d0 \ucd1d\uc744 \ub9de\ub294 \ubd80\uc0c1\uc744 \uc785\uc5c8\ub2e4(11\uc6d4 16\uc77c).", "paragraph_sentence": "1\ucc28\ub300\uc804 \ub2f9\uc2dc \ub9cc\uc288\ud0c0\uc778\uc740 \uc11c\ubd80\uc804\uc120\uacfc \ub3d9\ubd80\uc804\uc120\uc5d0\uc11c \ubaa8\ub450 \ubcf5\ubb34\ud588\ub2e4. \uc804\uc7c1 \uac1c\uc804 \uc2dc\uc810\uc5d0\uc11c \uc911\uc704\uc600\ub358 \ub9cc\uc288\ud0c0\uc778\uc740 \uc81c2\uce5c\uc704\uc608\ube44\ubcf4\ubcd1\uc5f0\ub300\uc5d0 \uc18d\ud558\uc5ec \ubca8\uae30\uc5d0 \uce68\uacf5\uc5d0 \uc885\uad70\ud588\ub2e4. \ub9cc\uc288\ud0c0\uc778\uc758 \ubd80\ub300\ub294 1914\ub144 8\uc6d4 \ub098\ubb88\ub974\ub97c \ud568\ub77d\uc2dc\ucf30\uace0, 9\uc6d4\uc5d0\ub294 \ub3d9\ud504\ub85c\uc774\uc13c\uc73c\ub85c \uc774\ub3d9\ud558\uc5ec \uc774\ubaa8\ubd80 \ud78c\ub374\ubd80\ub974\ud06c\uac00 \uc9c0\ud718\ud558\ub294 \uc81c8\uad70\uc5d0 \ubc30\uc18d\ub418\uc5c8\ub2e4. \uc81c1\ucc28 \ub9c8\uc218\ub9ac\uc548 \ud638 \uc804\ud22c\uc5d0 \ucc38\uc5ec\ud55c \ub4a4 \ub9cc\uc288\ud0c0\uc778\uc758 \ubd80\ub300\ub294 \uc81c9\uad70\uc5d0 \ubc30\uc18d\ub418\uc5b4 \uc624\ubca0\uc5b4\uc290\ub808\uc9c0\uc5d4\uc5d0\uc11c \ubc14\ub974\uc0e4\ubc14\ub85c \uc9c4\uad70\ud588\ub2e4. \uadf8\ub7ec\ub098 \uc81c9\uad70\uc740 \ub7ec\uc2dc\uc544\uad70\uc758 \ubc18\uaca9\uc744 \ubc1b\uc544 \ud6c4\ud1f4\ud588\uace0, \ub9cc\uc288\ud0c0\uc778\uc740 \ud6c4\ud1f4 \uc640\uc911 \ubd84\uacac\ub300\ub97c \uc774\ub04c\uace0 \ub7ec\uc2dc\uc544 \ucc38\ud638\uc120 \uc744 \uacf5\uaca9\ud558\ub2e4\uac00 \uc67c\ucabd \uc5b4\uae68\uc640 \uc67c\ucabd \ubb34\ub98e\uc5d0 \ucd1d\uc744 \ub9de\ub294 \ubd80\uc0c1\uc744 \uc785\uc5c8\ub2e4(11\uc6d4 16\uc77c). \ucd1d\uc54c \ud55c \ubc1c\uc774 \uc88c\uace8\uc2e0\uacbd\uc744 \ub04a\uc5b4\uc11c \ub9cc\uc288\ud0c0\uc778\uc740 \ub2e4\ub9ac\uac00 \ub9c8\ube44\ub418\uc5c8\ub2e4. \ud68c\ubcf5\ub418\ub294 \ub370\ub294 6\uac1c\uc6d4\uc774 \uac78\ub838\uace0 \uadf8 \ub3d9\uc548 \ub9cc\uc288\ud0c0\uc778\uc740 \ube44\ud1b0\uacfc \ube44\uc2a4\ubc14\ub374\uc5d0\uc11c \ubcd1\uc6d0\uc2e0\uc138\ub97c \uc84c\ub2e4.", "paragraph_answer": "1\ucc28\ub300\uc804 \ub2f9\uc2dc \ub9cc\uc288\ud0c0\uc778\uc740 \uc11c\ubd80\uc804\uc120\uacfc \ub3d9\ubd80\uc804\uc120\uc5d0\uc11c \ubaa8\ub450 \ubcf5\ubb34\ud588\ub2e4. \uc804\uc7c1 \uac1c\uc804 \uc2dc\uc810\uc5d0\uc11c \uc911\uc704\uc600\ub358 \ub9cc\uc288\ud0c0\uc778\uc740 \uc81c2\uce5c\uc704\uc608\ube44\ubcf4\ubcd1\uc5f0\ub300\uc5d0 \uc18d\ud558\uc5ec \ubca8\uae30\uc5d0 \uce68\uacf5\uc5d0 \uc885\uad70\ud588\ub2e4. \ub9cc\uc288\ud0c0\uc778\uc758 \ubd80\ub300\ub294 1914\ub144 8\uc6d4 \ub098\ubb88\ub974\ub97c \ud568\ub77d\uc2dc\ucf30\uace0, 9\uc6d4\uc5d0\ub294 \ub3d9\ud504\ub85c\uc774\uc13c\uc73c\ub85c \uc774\ub3d9\ud558\uc5ec \uc774\ubaa8\ubd80 \ud78c\ub374\ubd80\ub974\ud06c\uac00 \uc9c0\ud718\ud558\ub294 \uc81c8\uad70\uc5d0 \ubc30\uc18d\ub418\uc5c8\ub2e4. \uc81c1\ucc28 \ub9c8\uc218\ub9ac\uc548 \ud638 \uc804\ud22c\uc5d0 \ucc38\uc5ec\ud55c \ub4a4 \ub9cc\uc288\ud0c0\uc778\uc758 \ubd80\ub300\ub294 \uc81c9\uad70\uc5d0 \ubc30\uc18d\ub418\uc5b4 \uc624\ubca0\uc5b4\uc290\ub808\uc9c0\uc5d4\uc5d0\uc11c \ubc14\ub974\uc0e4\ubc14\ub85c \uc9c4\uad70\ud588\ub2e4. \uadf8\ub7ec\ub098 \uc81c9\uad70\uc740 \ub7ec\uc2dc\uc544\uad70\uc758 \ubc18\uaca9\uc744 \ubc1b\uc544 \ud6c4\ud1f4\ud588\uace0, \ub9cc\uc288\ud0c0\uc778\uc740 \ud6c4\ud1f4 \uc640\uc911 \ubd84\uacac\ub300\ub97c \uc774\ub04c\uace0 \ub7ec\uc2dc\uc544 \ucc38\ud638\uc120 \uc744 \uacf5\uaca9\ud558\ub2e4\uac00 \uc67c\ucabd \uc5b4\uae68\uc640 \uc67c\ucabd \ubb34\ub98e\uc5d0 \ucd1d\uc744 \ub9de\ub294 \ubd80\uc0c1\uc744 \uc785\uc5c8\ub2e4(11\uc6d4 16\uc77c). \ucd1d\uc54c \ud55c \ubc1c\uc774 \uc88c\uace8\uc2e0\uacbd\uc744 \ub04a\uc5b4\uc11c \ub9cc\uc288\ud0c0\uc778\uc740 \ub2e4\ub9ac\uac00 \ub9c8\ube44\ub418\uc5c8\ub2e4. \ud68c\ubcf5\ub418\ub294 \ub370\ub294 6\uac1c\uc6d4\uc774 \uac78\ub838\uace0 \uadf8 \ub3d9\uc548 \ub9cc\uc288\ud0c0\uc778\uc740 \ube44\ud1b0\uacfc \ube44\uc2a4\ubc14\ub374\uc5d0\uc11c \ubcd1\uc6d0\uc2e0\uc138\ub97c \uc84c\ub2e4.", "sentence_answer": "\uadf8\ub7ec\ub098 \uc81c9\uad70\uc740 \ub7ec\uc2dc\uc544\uad70\uc758 \ubc18\uaca9\uc744 \ubc1b\uc544 \ud6c4\ud1f4\ud588\uace0, \ub9cc\uc288\ud0c0\uc778\uc740 \ud6c4\ud1f4 \uc640\uc911 \ubd84\uacac\ub300\ub97c \uc774\ub04c\uace0 \ub7ec\uc2dc\uc544 \ucc38\ud638\uc120 \uc744 \uacf5\uaca9\ud558\ub2e4\uac00 \uc67c\ucabd \uc5b4\uae68\uc640 \uc67c\ucabd \ubb34\ub98e\uc5d0 \ucd1d\uc744 \ub9de\ub294 \ubd80\uc0c1\uc744 \uc785\uc5c8\ub2e4(11\uc6d4 16\uc77c).", "paragraph_id": "6540644-3-2", "paragraph_question": "question: 1914\ub144 11\uc6d4 16\uc77c \ub9cc\uc288\ud0c0\uc778\uc740 \ubb34\uc5c7\uc744 \uacf5\uaca9\ud558\ub2e4\uac00 \ucd1d\uc5d0 \ub9de\ub294 \ubd80\uc0c1\uc744 \uc785\uc5c8\ub098?, context: 1\ucc28\ub300\uc804 \ub2f9\uc2dc \ub9cc\uc288\ud0c0\uc778\uc740 \uc11c\ubd80\uc804\uc120\uacfc \ub3d9\ubd80\uc804\uc120\uc5d0\uc11c \ubaa8\ub450 \ubcf5\ubb34\ud588\ub2e4. \uc804\uc7c1 \uac1c\uc804 \uc2dc\uc810\uc5d0\uc11c \uc911\uc704\uc600\ub358 \ub9cc\uc288\ud0c0\uc778\uc740 \uc81c2\uce5c\uc704\uc608\ube44\ubcf4\ubcd1\uc5f0\ub300\uc5d0 \uc18d\ud558\uc5ec \ubca8\uae30\uc5d0 \uce68\uacf5\uc5d0 \uc885\uad70\ud588\ub2e4. \ub9cc\uc288\ud0c0\uc778\uc758 \ubd80\ub300\ub294 1914\ub144 8\uc6d4 \ub098\ubb88\ub974\ub97c \ud568\ub77d\uc2dc\ucf30\uace0, 9\uc6d4\uc5d0\ub294 \ub3d9\ud504\ub85c\uc774\uc13c\uc73c\ub85c \uc774\ub3d9\ud558\uc5ec \uc774\ubaa8\ubd80 \ud78c\ub374\ubd80\ub974\ud06c\uac00 \uc9c0\ud718\ud558\ub294 \uc81c8\uad70\uc5d0 \ubc30\uc18d\ub418\uc5c8\ub2e4. \uc81c1\ucc28 \ub9c8\uc218\ub9ac\uc548 \ud638 \uc804\ud22c\uc5d0 \ucc38\uc5ec\ud55c \ub4a4 \ub9cc\uc288\ud0c0\uc778\uc758 \ubd80\ub300\ub294 \uc81c9\uad70\uc5d0 \ubc30\uc18d\ub418\uc5b4 \uc624\ubca0\uc5b4\uc290\ub808\uc9c0\uc5d4\uc5d0\uc11c \ubc14\ub974\uc0e4\ubc14\ub85c \uc9c4\uad70\ud588\ub2e4. \uadf8\ub7ec\ub098 \uc81c9\uad70\uc740 \ub7ec\uc2dc\uc544\uad70\uc758 \ubc18\uaca9\uc744 \ubc1b\uc544 \ud6c4\ud1f4\ud588\uace0, \ub9cc\uc288\ud0c0\uc778\uc740 \ud6c4\ud1f4 \uc640\uc911 \ubd84\uacac\ub300\ub97c \uc774\ub04c\uace0 \ub7ec\uc2dc\uc544 \ucc38\ud638\uc120\uc744 \uacf5\uaca9\ud558\ub2e4\uac00 \uc67c\ucabd \uc5b4\uae68\uc640 \uc67c\ucabd \ubb34\ub98e\uc5d0 \ucd1d\uc744 \ub9de\ub294 \ubd80\uc0c1\uc744 \uc785\uc5c8\ub2e4(11\uc6d4 16\uc77c). \ucd1d\uc54c \ud55c \ubc1c\uc774 \uc88c\uace8\uc2e0\uacbd\uc744 \ub04a\uc5b4\uc11c \ub9cc\uc288\ud0c0\uc778\uc740 \ub2e4\ub9ac\uac00 \ub9c8\ube44\ub418\uc5c8\ub2e4. \ud68c\ubcf5\ub418\ub294 \ub370\ub294 6\uac1c\uc6d4\uc774 \uac78\ub838\uace0 \uadf8 \ub3d9\uc548 \ub9cc\uc288\ud0c0\uc778\uc740 \ube44\ud1b0\uacfc \ube44\uc2a4\ubc14\ub374\uc5d0\uc11c \ubcd1\uc6d0\uc2e0\uc138\ub97c \uc84c\ub2e4."} {"question": "\uccad\ub098\ub77c\ub97c \uc57c\ub9cc\uad6d\uc73c\ub85c \ucde8\uae09\ud558\ub77c\ub294 \uc5f0\uc124\uc744 \ud55c \uc0ac\ub78c\uc740?", "paragraph": "\ubca0\uc774\uc9d5\uc744 \uc810\ub839\ud558\uace0 \uba38\ubb3c\ub800\ub358 \uc5f0\ud569\uad70\uc758 \uc218\ub294 \ub298\uc5b4 9\uc6d4\uc5d0\ub294 \uc57d 11\ub9cc\uba85\uc774\ub098 \ub418\uc5c8\ub2e4. \uc774\uc5b4 '\uc758\ud654\ub2e8\uc744 \uc0c9\ucd9c\ud55c\ub2e4'\ub294 \uba85\ubd84\ud558\uc5d0 '\uc758\ud654\ub2e8 \uc794\ub2f9 \uc18c\ud0d5 \uc791\uc804'\uc744 \uc804\uac1c\ud588\ub2e4. \uc774\ub294 \ubcf4\ubcf5\uc131\uc744 \ub744\uace0 \uc788\uc5b4\uc11c \uc57d\ud0c8\uacfc \ud3ed\ud589\uc740 \uad11\ubd84\uc5d0 \ub2ec\ud560 \uc815\ub3c4\ub85c \uc2ec\uac01\ud588\ub294\ub370, \ud2b9\ud788 \ub3c5\uc77c\uad70\uc774 \uac00\uc7a5 \uc2ec\ud588\ub2e4. \uc774\ub294 \ub3c5\uc77c \uc678\uad50 \uacf5\uc0ac\uad00 \ucf00\ud154\ub7ec \ub0a8\uc791\uc774 \uc0b4\ud574\ub2f9\ud55c \uc774\ud6c4 \ub3c5\uc77c \uc6d0\uc815\ub300\uac00 \ucd9c\uc815\ud560 \ub2f9\uc2dc \ud669\uc81c \ube4c\ud5ec\ub984 2\uc138\uac00 \uc5f0\uc124\uc5d0\uc11c \uccad\ub098\ub77c\ub97c '\uc57c\ub9cc\uad6d\uc73c\ub85c \ucde8\uae09\ud558\ub77c'\ub294 \uc774\ub978\ubc14 '\ud6c8\uc871\uc5f0\uc124'\uc5d0\uc11c \uc790\uadf9\ubc1b\uc740 \uc601\ud5a5\uc774 \ucef8\ub2e4. \uc601\uad6d, \ud504\ub791\uc2a4, \ub7ec\uc2dc\uc544, \uc774\ud0c8\ub9ac\uc544, \uc624\uc2a4\ud2b8\ub9ac\uc544 \ub4f1\ub3c4 \ub3c5\uc77c\uc5d0 \ubabb\uc9c0 \uc54a\uc740 \uc57d\ud0c8\uacfc \ubbfc\uac04\uc778 \ud559\uc0b4 \ud589\uc704\ub97c \uc790\ud589\ud558\uc600\uace0, \ubbf8\uad6d\uacfc \uc77c\ubcf8\uc774 \ube44\uad50\uc801 \uae30\uc728\uc744 \uc9c0\ucf30\ub2e4. \ubca0\uc774\uc9d5\uc744 \uc810\ub839\ud55c \uc5f0\ud569\uad70\uc740 1901\ub144 9\uc6d4 \uc2e0\ucd95 \uc870\uc57d\uc774 \uccb4\uacb0\ub418\uae30\uae4c\uc9c0 13\uac1c\uc6d4\uac04 \uc8fc\ub454\ud588\ub2e4.", "answer": "\ube4c\ud5ec\ub984 2\uc138", "sentence": "\uc774\ub294 \ub3c5\uc77c \uc678\uad50 \uacf5\uc0ac\uad00 \ucf00\ud154\ub7ec \ub0a8\uc791\uc774 \uc0b4\ud574\ub2f9\ud55c \uc774\ud6c4 \ub3c5\uc77c \uc6d0\uc815\ub300\uac00 \ucd9c\uc815\ud560 \ub2f9\uc2dc \ud669\uc81c \ube4c\ud5ec\ub984 2\uc138 \uac00 \uc5f0\uc124\uc5d0\uc11c \uccad\ub098\ub77c\ub97c '\uc57c\ub9cc\uad6d\uc73c\ub85c \ucde8\uae09\ud558\ub77c'\ub294 \uc774\ub978\ubc14 '\ud6c8\uc871\uc5f0\uc124'\uc5d0\uc11c \uc790\uadf9\ubc1b\uc740 \uc601\ud5a5\uc774 \ucef8\ub2e4.", "paragraph_sentence": "\ubca0\uc774\uc9d5\uc744 \uc810\ub839\ud558\uace0 \uba38\ubb3c\ub800\ub358 \uc5f0\ud569\uad70\uc758 \uc218\ub294 \ub298\uc5b4 9\uc6d4\uc5d0\ub294 \uc57d 11\ub9cc\uba85\uc774\ub098 \ub418\uc5c8\ub2e4. \uc774\uc5b4 '\uc758\ud654\ub2e8\uc744 \uc0c9\ucd9c\ud55c\ub2e4'\ub294 \uba85\ubd84\ud558\uc5d0 '\uc758\ud654\ub2e8 \uc794\ub2f9 \uc18c\ud0d5 \uc791\uc804'\uc744 \uc804\uac1c\ud588\ub2e4. \uc774\ub294 \ubcf4\ubcf5\uc131\uc744 \ub744\uace0 \uc788\uc5b4\uc11c \uc57d\ud0c8\uacfc \ud3ed\ud589\uc740 \uad11\ubd84\uc5d0 \ub2ec\ud560 \uc815\ub3c4\ub85c \uc2ec\uac01\ud588\ub294\ub370, \ud2b9\ud788 \ub3c5\uc77c\uad70\uc774 \uac00\uc7a5 \uc2ec\ud588\ub2e4. \uc774\ub294 \ub3c5\uc77c \uc678\uad50 \uacf5\uc0ac\uad00 \ucf00\ud154\ub7ec \ub0a8\uc791\uc774 \uc0b4\ud574\ub2f9\ud55c \uc774\ud6c4 \ub3c5\uc77c \uc6d0\uc815\ub300\uac00 \ucd9c\uc815\ud560 \ub2f9\uc2dc \ud669\uc81c \ube4c\ud5ec\ub984 2\uc138 \uac00 \uc5f0\uc124\uc5d0\uc11c \uccad\ub098\ub77c\ub97c '\uc57c\ub9cc\uad6d\uc73c\ub85c \ucde8\uae09\ud558\ub77c'\ub294 \uc774\ub978\ubc14 '\ud6c8\uc871\uc5f0\uc124'\uc5d0\uc11c \uc790\uadf9\ubc1b\uc740 \uc601\ud5a5\uc774 \ucef8\ub2e4. \uc601\uad6d, \ud504\ub791\uc2a4, \ub7ec\uc2dc\uc544, \uc774\ud0c8\ub9ac\uc544, \uc624\uc2a4\ud2b8\ub9ac\uc544 \ub4f1\ub3c4 \ub3c5\uc77c\uc5d0 \ubabb\uc9c0 \uc54a\uc740 \uc57d\ud0c8\uacfc \ubbfc\uac04\uc778 \ud559\uc0b4 \ud589\uc704\ub97c \uc790\ud589\ud558\uc600\uace0, \ubbf8\uad6d\uacfc \uc77c\ubcf8\uc774 \ube44\uad50\uc801 \uae30\uc728\uc744 \uc9c0\ucf30\ub2e4. \ubca0\uc774\uc9d5\uc744 \uc810\ub839\ud55c \uc5f0\ud569\uad70\uc740 1901\ub144 9\uc6d4 \uc2e0\ucd95 \uc870\uc57d\uc774 \uccb4\uacb0\ub418\uae30\uae4c\uc9c0 13\uac1c\uc6d4\uac04 \uc8fc\ub454\ud588\ub2e4.", "paragraph_answer": "\ubca0\uc774\uc9d5\uc744 \uc810\ub839\ud558\uace0 \uba38\ubb3c\ub800\ub358 \uc5f0\ud569\uad70\uc758 \uc218\ub294 \ub298\uc5b4 9\uc6d4\uc5d0\ub294 \uc57d 11\ub9cc\uba85\uc774\ub098 \ub418\uc5c8\ub2e4. \uc774\uc5b4 '\uc758\ud654\ub2e8\uc744 \uc0c9\ucd9c\ud55c\ub2e4'\ub294 \uba85\ubd84\ud558\uc5d0 '\uc758\ud654\ub2e8 \uc794\ub2f9 \uc18c\ud0d5 \uc791\uc804'\uc744 \uc804\uac1c\ud588\ub2e4. \uc774\ub294 \ubcf4\ubcf5\uc131\uc744 \ub744\uace0 \uc788\uc5b4\uc11c \uc57d\ud0c8\uacfc \ud3ed\ud589\uc740 \uad11\ubd84\uc5d0 \ub2ec\ud560 \uc815\ub3c4\ub85c \uc2ec\uac01\ud588\ub294\ub370, \ud2b9\ud788 \ub3c5\uc77c\uad70\uc774 \uac00\uc7a5 \uc2ec\ud588\ub2e4. \uc774\ub294 \ub3c5\uc77c \uc678\uad50 \uacf5\uc0ac\uad00 \ucf00\ud154\ub7ec \ub0a8\uc791\uc774 \uc0b4\ud574\ub2f9\ud55c \uc774\ud6c4 \ub3c5\uc77c \uc6d0\uc815\ub300\uac00 \ucd9c\uc815\ud560 \ub2f9\uc2dc \ud669\uc81c \ube4c\ud5ec\ub984 2\uc138 \uac00 \uc5f0\uc124\uc5d0\uc11c \uccad\ub098\ub77c\ub97c '\uc57c\ub9cc\uad6d\uc73c\ub85c \ucde8\uae09\ud558\ub77c'\ub294 \uc774\ub978\ubc14 '\ud6c8\uc871\uc5f0\uc124'\uc5d0\uc11c \uc790\uadf9\ubc1b\uc740 \uc601\ud5a5\uc774 \ucef8\ub2e4. \uc601\uad6d, \ud504\ub791\uc2a4, \ub7ec\uc2dc\uc544, \uc774\ud0c8\ub9ac\uc544, \uc624\uc2a4\ud2b8\ub9ac\uc544 \ub4f1\ub3c4 \ub3c5\uc77c\uc5d0 \ubabb\uc9c0 \uc54a\uc740 \uc57d\ud0c8\uacfc \ubbfc\uac04\uc778 \ud559\uc0b4 \ud589\uc704\ub97c \uc790\ud589\ud558\uc600\uace0, \ubbf8\uad6d\uacfc \uc77c\ubcf8\uc774 \ube44\uad50\uc801 \uae30\uc728\uc744 \uc9c0\ucf30\ub2e4. \ubca0\uc774\uc9d5\uc744 \uc810\ub839\ud55c \uc5f0\ud569\uad70\uc740 1901\ub144 9\uc6d4 \uc2e0\ucd95 \uc870\uc57d\uc774 \uccb4\uacb0\ub418\uae30\uae4c\uc9c0 13\uac1c\uc6d4\uac04 \uc8fc\ub454\ud588\ub2e4.", "sentence_answer": "\uc774\ub294 \ub3c5\uc77c \uc678\uad50 \uacf5\uc0ac\uad00 \ucf00\ud154\ub7ec \ub0a8\uc791\uc774 \uc0b4\ud574\ub2f9\ud55c \uc774\ud6c4 \ub3c5\uc77c \uc6d0\uc815\ub300\uac00 \ucd9c\uc815\ud560 \ub2f9\uc2dc \ud669\uc81c \ube4c\ud5ec\ub984 2\uc138 \uac00 \uc5f0\uc124\uc5d0\uc11c \uccad\ub098\ub77c\ub97c '\uc57c\ub9cc\uad6d\uc73c\ub85c \ucde8\uae09\ud558\ub77c'\ub294 \uc774\ub978\ubc14 '\ud6c8\uc871\uc5f0\uc124'\uc5d0\uc11c \uc790\uadf9\ubc1b\uc740 \uc601\ud5a5\uc774 \ucef8\ub2e4.", "paragraph_id": "6544419-17-0", "paragraph_question": "question: \uccad\ub098\ub77c\ub97c \uc57c\ub9cc\uad6d\uc73c\ub85c \ucde8\uae09\ud558\ub77c\ub294 \uc5f0\uc124\uc744 \ud55c \uc0ac\ub78c\uc740?, context: \ubca0\uc774\uc9d5\uc744 \uc810\ub839\ud558\uace0 \uba38\ubb3c\ub800\ub358 \uc5f0\ud569\uad70\uc758 \uc218\ub294 \ub298\uc5b4 9\uc6d4\uc5d0\ub294 \uc57d 11\ub9cc\uba85\uc774\ub098 \ub418\uc5c8\ub2e4. \uc774\uc5b4 '\uc758\ud654\ub2e8\uc744 \uc0c9\ucd9c\ud55c\ub2e4'\ub294 \uba85\ubd84\ud558\uc5d0 '\uc758\ud654\ub2e8 \uc794\ub2f9 \uc18c\ud0d5 \uc791\uc804'\uc744 \uc804\uac1c\ud588\ub2e4. \uc774\ub294 \ubcf4\ubcf5\uc131\uc744 \ub744\uace0 \uc788\uc5b4\uc11c \uc57d\ud0c8\uacfc \ud3ed\ud589\uc740 \uad11\ubd84\uc5d0 \ub2ec\ud560 \uc815\ub3c4\ub85c \uc2ec\uac01\ud588\ub294\ub370, \ud2b9\ud788 \ub3c5\uc77c\uad70\uc774 \uac00\uc7a5 \uc2ec\ud588\ub2e4. \uc774\ub294 \ub3c5\uc77c \uc678\uad50 \uacf5\uc0ac\uad00 \ucf00\ud154\ub7ec \ub0a8\uc791\uc774 \uc0b4\ud574\ub2f9\ud55c \uc774\ud6c4 \ub3c5\uc77c \uc6d0\uc815\ub300\uac00 \ucd9c\uc815\ud560 \ub2f9\uc2dc \ud669\uc81c \ube4c\ud5ec\ub984 2\uc138\uac00 \uc5f0\uc124\uc5d0\uc11c \uccad\ub098\ub77c\ub97c '\uc57c\ub9cc\uad6d\uc73c\ub85c \ucde8\uae09\ud558\ub77c'\ub294 \uc774\ub978\ubc14 '\ud6c8\uc871\uc5f0\uc124'\uc5d0\uc11c \uc790\uadf9\ubc1b\uc740 \uc601\ud5a5\uc774 \ucef8\ub2e4. \uc601\uad6d, \ud504\ub791\uc2a4, \ub7ec\uc2dc\uc544, \uc774\ud0c8\ub9ac\uc544, \uc624\uc2a4\ud2b8\ub9ac\uc544 \ub4f1\ub3c4 \ub3c5\uc77c\uc5d0 \ubabb\uc9c0 \uc54a\uc740 \uc57d\ud0c8\uacfc \ubbfc\uac04\uc778 \ud559\uc0b4 \ud589\uc704\ub97c \uc790\ud589\ud558\uc600\uace0, \ubbf8\uad6d\uacfc \uc77c\ubcf8\uc774 \ube44\uad50\uc801 \uae30\uc728\uc744 \uc9c0\ucf30\ub2e4. \ubca0\uc774\uc9d5\uc744 \uc810\ub839\ud55c \uc5f0\ud569\uad70\uc740 1901\ub144 9\uc6d4 \uc2e0\ucd95 \uc870\uc57d\uc774 \uccb4\uacb0\ub418\uae30\uae4c\uc9c0 13\uac1c\uc6d4\uac04 \uc8fc\ub454\ud588\ub2e4."} {"question": "T \uacf5\uaca9 \ub2f9\uc2dc \uc5f0\ud569\ud568\ub300 \uc0ac\ub839\uc7a5\uad00\uc740?", "paragraph": "1944\ub144 7\uc6d4 23\uc77c \ub3c4\uc0c1 \uc5f0\uc2b5\uad70\ub839\ubd80\uc5d0\uc11c \uc545\ucc9c\ud6c4\ub85c \uc778\ud574 \ucd9c\ubc1c\uc774 \uc5b4\ub824\uc6b4 \uc8fc\uac04 \uacf5\uaca9\uc744 \uc8fc\ub85c \ud558\uace0, \uae30\ud68c\uac00 \uc5c6\uc73c\uba74 \uc801\uc758 \ud65c\ub3d9\uc774 \ub738\ud55c \uc57c\uac04\uc5d0 \uacf5\uaca9\ud558\ub294 T \uacf5\uaca9 \ubd80\ub300 \uc548\uc774 \ub098\uc654\ub2e4. \ud55c\ud3b8\uc73c\ub85c \uc9c0\ud718\uad8c\uc744 \uac00\uc9c4 \uc81c2\ud56d\uacf5\ud568\ub300\uc5d0\uc11c \ube44\ud589 \uc9d1\ub2e8 \uad6c\ubd84\uc744 \uc784\ubb34\ubcc4\ub85c \uc815\ud558\uc5ec T \uacf5\uaca9 \ubd80\ub300\uc5d0 \uc758\ud55c \uc57c\uac04 \uacf5\uaca9\uc744 \uc911\uc2ec\uc73c\ub85c \ud558\ub294 \uc8fc\uac04 \uacf5\uaca9, \ud669\ud63c \uacf5\uaca9\uc758 \uc138 \uac00\uc9c0\ub97c \ubd80\uc11c\ub85c \ud3b8\uc131\ud588\ub2e4. \uc81c1\uc5d0\uc11c \uc81c4\uae4c\uc9c0 \uc815\ud574\uc9c4 \uc0c1\ud669\uc5d0 \ub530\ub77c \uadf8 \uc911 \ud558\ub098\ub97c \uc801\uc6a9\ud558\ub294 \uc804\ubc95\uc774 \ub4f1\uc7a5\ud588\ub2e4. \uc774\uac83\uc740 \ub098\uc911\uc5d0 \uc81c6\uae30\uc9c0 \ud56d\uacf5\ubd80\ub300\uac00 \uaddc\uc815\ud55c \uc804\ud22c \ubc29\ubc95\uc73c\ub85c \ubc1c\uc804\ud588\ub2e4. \ub610\ud55c 1944\ub144 9\uc6d4 \ucd08, T \uacf5\uaca9\uc758 \ubc29\uc2dd\uc5d0 \uc758\ubb38\uc744 \uc81c\uae30\ud55c \uc81c2\ud56d\uacf5\ud568\ub300 \uc0ac\ub839\uad00 \ud6c4\ucfe0\ub3c4\uba54 \uc2dc\uac8c\ub8e8 \uc911\uc7a5\uc740 T \uacf5\uaca9 \ubd80\ub300\ub294 \uacb0\uc804\uc758 \uc77c\uaca9\uc744 \uc704\ud574 \uc57c\uac04 \uacf5\uaca9\uc774\ub098, \uc545\ucc9c\ud6c4 \uc18d\uc5d0 \uc6b4\uc6a9\ud558\ub294 \uac83\uc740 \ucd5c\ud6c4\uc758 \uce74\ub4dc\ub85c \uacb0\ud589\ud558\uaca0\ub2e4\uace0 \ud588\ub2e4. \uc5f0\ud569\ud568\ub300 \uc0ac\ub839\uc7a5\uad00 \ub3c4\uc694\ub2e4 \uc18c\uc5d0\ubb34\ub294 \ubd80\ub300 \uc6b4\uc6a9\uc5d0 \ub300\ud574 \uc81c2\ud56d\uacf5\ud568\ub300 \uc0ac\ub839\uc7a5\uad00\uc778 \ud6c4\ucfe0\ub3c4\uba54\uc5d0\uac8c \uc77c\uc784\ud558\uace0, \ud798\ub4e4 \ub54c\ub294 \ubb34\ub9ac\ub97c \ud558\uc9c0 \ub9d0\ub77c\uace0 \ud588\ub2e4.", "answer": "\ub3c4\uc694\ub2e4 \uc18c\uc5d0\ubb34", "sentence": "\uc5f0\ud569\ud568\ub300 \uc0ac\ub839\uc7a5\uad00 \ub3c4\uc694\ub2e4 \uc18c\uc5d0\ubb34 \ub294 \ubd80\ub300 \uc6b4\uc6a9\uc5d0 \ub300\ud574 \uc81c2\ud56d\uacf5\ud568\ub300 \uc0ac\ub839\uc7a5\uad00\uc778 \ud6c4\ucfe0\ub3c4\uba54\uc5d0\uac8c \uc77c\uc784\ud558\uace0, \ud798\ub4e4 \ub54c\ub294 \ubb34\ub9ac\ub97c \ud558\uc9c0 \ub9d0\ub77c\uace0 \ud588\ub2e4.", "paragraph_sentence": "1944\ub144 7\uc6d4 23\uc77c \ub3c4\uc0c1 \uc5f0\uc2b5\uad70\ub839\ubd80\uc5d0\uc11c \uc545\ucc9c\ud6c4\ub85c \uc778\ud574 \ucd9c\ubc1c\uc774 \uc5b4\ub824\uc6b4 \uc8fc\uac04 \uacf5\uaca9\uc744 \uc8fc\ub85c \ud558\uace0, \uae30\ud68c\uac00 \uc5c6\uc73c\uba74 \uc801\uc758 \ud65c\ub3d9\uc774 \ub738\ud55c \uc57c\uac04\uc5d0 \uacf5\uaca9\ud558\ub294 T \uacf5\uaca9 \ubd80\ub300 \uc548\uc774 \ub098\uc654\ub2e4. \ud55c\ud3b8\uc73c\ub85c \uc9c0\ud718\uad8c\uc744 \uac00\uc9c4 \uc81c2\ud56d\uacf5\ud568\ub300\uc5d0\uc11c \ube44\ud589 \uc9d1\ub2e8 \uad6c\ubd84\uc744 \uc784\ubb34\ubcc4\ub85c \uc815\ud558\uc5ec T \uacf5\uaca9 \ubd80\ub300\uc5d0 \uc758\ud55c \uc57c\uac04 \uacf5\uaca9\uc744 \uc911\uc2ec\uc73c\ub85c \ud558\ub294 \uc8fc\uac04 \uacf5\uaca9, \ud669\ud63c \uacf5\uaca9\uc758 \uc138 \uac00\uc9c0\ub97c \ubd80\uc11c\ub85c \ud3b8\uc131\ud588\ub2e4. \uc81c1\uc5d0\uc11c \uc81c4\uae4c\uc9c0 \uc815\ud574\uc9c4 \uc0c1\ud669\uc5d0 \ub530\ub77c \uadf8 \uc911 \ud558\ub098\ub97c \uc801\uc6a9\ud558\ub294 \uc804\ubc95\uc774 \ub4f1\uc7a5\ud588\ub2e4. \uc774\uac83\uc740 \ub098\uc911\uc5d0 \uc81c6\uae30\uc9c0 \ud56d\uacf5\ubd80\ub300\uac00 \uaddc\uc815\ud55c \uc804\ud22c \ubc29\ubc95\uc73c\ub85c \ubc1c\uc804\ud588\ub2e4. \ub610\ud55c 1944\ub144 9\uc6d4 \ucd08, T \uacf5\uaca9\uc758 \ubc29\uc2dd\uc5d0 \uc758\ubb38\uc744 \uc81c\uae30\ud55c \uc81c2\ud56d\uacf5\ud568\ub300 \uc0ac\ub839\uad00 \ud6c4\ucfe0\ub3c4\uba54 \uc2dc\uac8c\ub8e8 \uc911\uc7a5\uc740 T \uacf5\uaca9 \ubd80\ub300\ub294 \uacb0\uc804\uc758 \uc77c\uaca9\uc744 \uc704\ud574 \uc57c\uac04 \uacf5\uaca9\uc774\ub098, \uc545\ucc9c\ud6c4 \uc18d\uc5d0 \uc6b4\uc6a9\ud558\ub294 \uac83\uc740 \ucd5c\ud6c4\uc758 \uce74\ub4dc\ub85c \uacb0\ud589\ud558\uaca0\ub2e4\uace0 \ud588\ub2e4. \uc5f0\ud569\ud568\ub300 \uc0ac\ub839\uc7a5\uad00 \ub3c4\uc694\ub2e4 \uc18c\uc5d0\ubb34 \ub294 \ubd80\ub300 \uc6b4\uc6a9\uc5d0 \ub300\ud574 \uc81c2\ud56d\uacf5\ud568\ub300 \uc0ac\ub839\uc7a5\uad00\uc778 \ud6c4\ucfe0\ub3c4\uba54\uc5d0\uac8c \uc77c\uc784\ud558\uace0, \ud798\ub4e4 \ub54c\ub294 \ubb34\ub9ac\ub97c \ud558\uc9c0 \ub9d0\ub77c\uace0 \ud588\ub2e4. ", "paragraph_answer": "1944\ub144 7\uc6d4 23\uc77c \ub3c4\uc0c1 \uc5f0\uc2b5\uad70\ub839\ubd80\uc5d0\uc11c \uc545\ucc9c\ud6c4\ub85c \uc778\ud574 \ucd9c\ubc1c\uc774 \uc5b4\ub824\uc6b4 \uc8fc\uac04 \uacf5\uaca9\uc744 \uc8fc\ub85c \ud558\uace0, \uae30\ud68c\uac00 \uc5c6\uc73c\uba74 \uc801\uc758 \ud65c\ub3d9\uc774 \ub738\ud55c \uc57c\uac04\uc5d0 \uacf5\uaca9\ud558\ub294 T \uacf5\uaca9 \ubd80\ub300 \uc548\uc774 \ub098\uc654\ub2e4. \ud55c\ud3b8\uc73c\ub85c \uc9c0\ud718\uad8c\uc744 \uac00\uc9c4 \uc81c2\ud56d\uacf5\ud568\ub300\uc5d0\uc11c \ube44\ud589 \uc9d1\ub2e8 \uad6c\ubd84\uc744 \uc784\ubb34\ubcc4\ub85c \uc815\ud558\uc5ec T \uacf5\uaca9 \ubd80\ub300\uc5d0 \uc758\ud55c \uc57c\uac04 \uacf5\uaca9\uc744 \uc911\uc2ec\uc73c\ub85c \ud558\ub294 \uc8fc\uac04 \uacf5\uaca9, \ud669\ud63c \uacf5\uaca9\uc758 \uc138 \uac00\uc9c0\ub97c \ubd80\uc11c\ub85c \ud3b8\uc131\ud588\ub2e4. \uc81c1\uc5d0\uc11c \uc81c4\uae4c\uc9c0 \uc815\ud574\uc9c4 \uc0c1\ud669\uc5d0 \ub530\ub77c \uadf8 \uc911 \ud558\ub098\ub97c \uc801\uc6a9\ud558\ub294 \uc804\ubc95\uc774 \ub4f1\uc7a5\ud588\ub2e4. \uc774\uac83\uc740 \ub098\uc911\uc5d0 \uc81c6\uae30\uc9c0 \ud56d\uacf5\ubd80\ub300\uac00 \uaddc\uc815\ud55c \uc804\ud22c \ubc29\ubc95\uc73c\ub85c \ubc1c\uc804\ud588\ub2e4. \ub610\ud55c 1944\ub144 9\uc6d4 \ucd08, T \uacf5\uaca9\uc758 \ubc29\uc2dd\uc5d0 \uc758\ubb38\uc744 \uc81c\uae30\ud55c \uc81c2\ud56d\uacf5\ud568\ub300 \uc0ac\ub839\uad00 \ud6c4\ucfe0\ub3c4\uba54 \uc2dc\uac8c\ub8e8 \uc911\uc7a5\uc740 T \uacf5\uaca9 \ubd80\ub300\ub294 \uacb0\uc804\uc758 \uc77c\uaca9\uc744 \uc704\ud574 \uc57c\uac04 \uacf5\uaca9\uc774\ub098, \uc545\ucc9c\ud6c4 \uc18d\uc5d0 \uc6b4\uc6a9\ud558\ub294 \uac83\uc740 \ucd5c\ud6c4\uc758 \uce74\ub4dc\ub85c \uacb0\ud589\ud558\uaca0\ub2e4\uace0 \ud588\ub2e4. \uc5f0\ud569\ud568\ub300 \uc0ac\ub839\uc7a5\uad00 \ub3c4\uc694\ub2e4 \uc18c\uc5d0\ubb34 \ub294 \ubd80\ub300 \uc6b4\uc6a9\uc5d0 \ub300\ud574 \uc81c2\ud56d\uacf5\ud568\ub300 \uc0ac\ub839\uc7a5\uad00\uc778 \ud6c4\ucfe0\ub3c4\uba54\uc5d0\uac8c \uc77c\uc784\ud558\uace0, \ud798\ub4e4 \ub54c\ub294 \ubb34\ub9ac\ub97c \ud558\uc9c0 \ub9d0\ub77c\uace0 \ud588\ub2e4.", "sentence_answer": "\uc5f0\ud569\ud568\ub300 \uc0ac\ub839\uc7a5\uad00 \ub3c4\uc694\ub2e4 \uc18c\uc5d0\ubb34 \ub294 \ubd80\ub300 \uc6b4\uc6a9\uc5d0 \ub300\ud574 \uc81c2\ud56d\uacf5\ud568\ub300 \uc0ac\ub839\uc7a5\uad00\uc778 \ud6c4\ucfe0\ub3c4\uba54\uc5d0\uac8c \uc77c\uc784\ud558\uace0, \ud798\ub4e4 \ub54c\ub294 \ubb34\ub9ac\ub97c \ud558\uc9c0 \ub9d0\ub77c\uace0 \ud588\ub2e4.", "paragraph_id": "6600262-0-2", "paragraph_question": "question: T \uacf5\uaca9 \ub2f9\uc2dc \uc5f0\ud569\ud568\ub300 \uc0ac\ub839\uc7a5\uad00\uc740?, context: 1944\ub144 7\uc6d4 23\uc77c \ub3c4\uc0c1 \uc5f0\uc2b5\uad70\ub839\ubd80\uc5d0\uc11c \uc545\ucc9c\ud6c4\ub85c \uc778\ud574 \ucd9c\ubc1c\uc774 \uc5b4\ub824\uc6b4 \uc8fc\uac04 \uacf5\uaca9\uc744 \uc8fc\ub85c \ud558\uace0, \uae30\ud68c\uac00 \uc5c6\uc73c\uba74 \uc801\uc758 \ud65c\ub3d9\uc774 \ub738\ud55c \uc57c\uac04\uc5d0 \uacf5\uaca9\ud558\ub294 T \uacf5\uaca9 \ubd80\ub300 \uc548\uc774 \ub098\uc654\ub2e4. \ud55c\ud3b8\uc73c\ub85c \uc9c0\ud718\uad8c\uc744 \uac00\uc9c4 \uc81c2\ud56d\uacf5\ud568\ub300\uc5d0\uc11c \ube44\ud589 \uc9d1\ub2e8 \uad6c\ubd84\uc744 \uc784\ubb34\ubcc4\ub85c \uc815\ud558\uc5ec T \uacf5\uaca9 \ubd80\ub300\uc5d0 \uc758\ud55c \uc57c\uac04 \uacf5\uaca9\uc744 \uc911\uc2ec\uc73c\ub85c \ud558\ub294 \uc8fc\uac04 \uacf5\uaca9, \ud669\ud63c \uacf5\uaca9\uc758 \uc138 \uac00\uc9c0\ub97c \ubd80\uc11c\ub85c \ud3b8\uc131\ud588\ub2e4. \uc81c1\uc5d0\uc11c \uc81c4\uae4c\uc9c0 \uc815\ud574\uc9c4 \uc0c1\ud669\uc5d0 \ub530\ub77c \uadf8 \uc911 \ud558\ub098\ub97c \uc801\uc6a9\ud558\ub294 \uc804\ubc95\uc774 \ub4f1\uc7a5\ud588\ub2e4. \uc774\uac83\uc740 \ub098\uc911\uc5d0 \uc81c6\uae30\uc9c0 \ud56d\uacf5\ubd80\ub300\uac00 \uaddc\uc815\ud55c \uc804\ud22c \ubc29\ubc95\uc73c\ub85c \ubc1c\uc804\ud588\ub2e4. \ub610\ud55c 1944\ub144 9\uc6d4 \ucd08, T \uacf5\uaca9\uc758 \ubc29\uc2dd\uc5d0 \uc758\ubb38\uc744 \uc81c\uae30\ud55c \uc81c2\ud56d\uacf5\ud568\ub300 \uc0ac\ub839\uad00 \ud6c4\ucfe0\ub3c4\uba54 \uc2dc\uac8c\ub8e8 \uc911\uc7a5\uc740 T \uacf5\uaca9 \ubd80\ub300\ub294 \uacb0\uc804\uc758 \uc77c\uaca9\uc744 \uc704\ud574 \uc57c\uac04 \uacf5\uaca9\uc774\ub098, \uc545\ucc9c\ud6c4 \uc18d\uc5d0 \uc6b4\uc6a9\ud558\ub294 \uac83\uc740 \ucd5c\ud6c4\uc758 \uce74\ub4dc\ub85c \uacb0\ud589\ud558\uaca0\ub2e4\uace0 \ud588\ub2e4. \uc5f0\ud569\ud568\ub300 \uc0ac\ub839\uc7a5\uad00 \ub3c4\uc694\ub2e4 \uc18c\uc5d0\ubb34\ub294 \ubd80\ub300 \uc6b4\uc6a9\uc5d0 \ub300\ud574 \uc81c2\ud56d\uacf5\ud568\ub300 \uc0ac\ub839\uc7a5\uad00\uc778 \ud6c4\ucfe0\ub3c4\uba54\uc5d0\uac8c \uc77c\uc784\ud558\uace0, \ud798\ub4e4 \ub54c\ub294 \ubb34\ub9ac\ub97c \ud558\uc9c0 \ub9d0\ub77c\uace0 \ud588\ub2e4."} {"question": "\ubb3c \ubd84\uc790\uc758 \uad6c\uc131 \uc131\ubd84 \uc911 \uc720\ub85c\ud30c\uc758 \uc911\ub825\uc5d0\uc11c \ubc97\uc5b4\ub098 \ub300\uae30 \ud45c\uba74\uc73c\ub85c \ub3cc\uc544\uac00\uc9c0 \ubabb\ud558\ub294 \uc131\ubd84\uc740?", "paragraph": "\uc9c0\uad6c\uc758 \ub300\uae30\uad8c\uacfc\ub294 \ub2e4\ub974\uac8c, \uc720\ub85c\ud30c\uc758 \ub300\uae30\uad8c\uc740 \uc0dd\ubb3c\ud559\uc801\uc73c\ub85c \uc0dd\uaca8\ub09c \uac83\uc774 \uc544\ub2c8\ub77c \ubd84\uc790\ub4e4\uc774 \ubc29\uc0ac\uc120 \ubd84\ud574\uac00 \ub418\uc5b4 \uc0dd\uc131\ub418\uc5c8\ub2e4. \ud0dc\uc591\uc5d0\uc11c \uc624\ub294 \uc790\uc678\uc120\uacfc \ubaa9\uc131\uc758 \uc790\uae30\uad8c\uc5d0\uc11c \uc624\ub294 \ub300\uc804 \uc785\uc790(\uc774\uc628\uc774\ub098 \uc804\uc790)\uac00 \uc720\ub85c\ud30c\uc758 \ubb3c \ubd84\uc790\ub97c \ub54c\ub824, \uc218\uc18c\uc640 \uc0b0\uc18c\ub85c \ub098\ub220 \ub300\uae30 \uc911\uc73c\ub85c \uc2a4\ud37c\ud130\ub9c1\ub418\uac70\ub098 \ud761\uc218\ub418\uac8c \ub9cc\ub4e0\ub2e4. \uac19\uc740 \ubc29\uc0ac\uc120\uc774 \uc774 \ud761\uc218\ub41c \ubb3c\uc9c8\ub4e4\uc744 \ub54c\uc5b4\ub0b4\uace0, \ub450 \uc791\uc6a9\uc774 \uc54c\ub9de\uac8c \uade0\ud615\uc744 \uc774\ub8e8\ub294 \ub300\uae30\uad8c\uc744 \ub9cc\ub4e4\uc5b4\ub0b8\ub2e4. \uc0b0\uc18c \ubd84\uc790\ub294 \uc218\uba85\uc774 \uae38\uae30 \ub54c\ubb38\uc5d0 \ub300\uae30\uc758 \uc8fc\uc694 \uad6c\uc131 \uc131\ubd84\uc774 \ub41c\ub2e4. \uc65c\ub0d0\ud558\uba74 \uc0b0\uc18c \ubd84\uc790\uc640 \uac19\uc740 \ubb3c\uc9c8\ub4e4\uc774 \ud45c\uba74\uc73c\ub85c \ub3cc\uc544\uac00\uba74 \ubb3c\uc774\ub098 \uacfc\uc0b0\ud654\uc218\uc18c\uc640 \uac19\uc740 \ubd84\uc790\ub85c \ub9cc\ub4e4\uc5b4\uc9c0\ub294 \uac83\uc774 \uc544\ub2c8\ub77c \ube60\uc838\ub098\uc640 \ub2e4\ub978 \uc791\uc6a9\uc744 \uc2dc\uc791\ud558\uae30 \ub54c\ubb38\uc774\ub2e4. \uc218\uc18c \ubd84\uc790\ub294 \uac00\ubccd\uae30 \ub54c\ubb38\uc5d0 \uc720\ub85c\ud30c\uc758 \uc911\ub825\uc5d0\uc11c \ubc97\uc5b4\ub098\uae30 \uc27d\uace0, \ub2e4\uc2dc\ub294 \ud45c\uba74\uc73c\ub85c \ub3cc\uc544\uac00\uc9c0 \ubabb\ud55c\ub2e4.", "answer": "\uc218\uc18c \ubd84\uc790", "sentence": "\uc218\uc18c \ubd84\uc790 \ub294 \uac00\ubccd\uae30 \ub54c\ubb38\uc5d0 \uc720\ub85c\ud30c\uc758 \uc911\ub825\uc5d0\uc11c \ubc97\uc5b4\ub098\uae30 \uc27d\uace0, \ub2e4\uc2dc\ub294 \ud45c\uba74\uc73c\ub85c \ub3cc\uc544\uac00\uc9c0 \ubabb\ud55c\ub2e4.", "paragraph_sentence": "\uc9c0\uad6c\uc758 \ub300\uae30\uad8c\uacfc\ub294 \ub2e4\ub974\uac8c, \uc720\ub85c\ud30c\uc758 \ub300\uae30\uad8c\uc740 \uc0dd\ubb3c\ud559\uc801\uc73c\ub85c \uc0dd\uaca8\ub09c \uac83\uc774 \uc544\ub2c8\ub77c \ubd84\uc790\ub4e4\uc774 \ubc29\uc0ac\uc120 \ubd84\ud574\uac00 \ub418\uc5b4 \uc0dd\uc131\ub418\uc5c8\ub2e4. \ud0dc\uc591\uc5d0\uc11c \uc624\ub294 \uc790\uc678\uc120\uacfc \ubaa9\uc131\uc758 \uc790\uae30\uad8c\uc5d0\uc11c \uc624\ub294 \ub300\uc804 \uc785\uc790(\uc774\uc628\uc774\ub098 \uc804\uc790)\uac00 \uc720\ub85c\ud30c\uc758 \ubb3c \ubd84\uc790\ub97c \ub54c\ub824, \uc218\uc18c\uc640 \uc0b0\uc18c\ub85c \ub098\ub220 \ub300\uae30 \uc911\uc73c\ub85c \uc2a4\ud37c\ud130\ub9c1\ub418\uac70\ub098 \ud761\uc218\ub418\uac8c \ub9cc\ub4e0\ub2e4. \uac19\uc740 \ubc29\uc0ac\uc120\uc774 \uc774 \ud761\uc218\ub41c \ubb3c\uc9c8\ub4e4\uc744 \ub54c\uc5b4\ub0b4\uace0, \ub450 \uc791\uc6a9\uc774 \uc54c\ub9de\uac8c \uade0\ud615\uc744 \uc774\ub8e8\ub294 \ub300\uae30\uad8c\uc744 \ub9cc\ub4e4\uc5b4\ub0b8\ub2e4. \uc0b0\uc18c \ubd84\uc790\ub294 \uc218\uba85\uc774 \uae38\uae30 \ub54c\ubb38\uc5d0 \ub300\uae30\uc758 \uc8fc\uc694 \uad6c\uc131 \uc131\ubd84\uc774 \ub41c\ub2e4. \uc65c\ub0d0\ud558\uba74 \uc0b0\uc18c \ubd84\uc790\uc640 \uac19\uc740 \ubb3c\uc9c8\ub4e4\uc774 \ud45c\uba74\uc73c\ub85c \ub3cc\uc544\uac00\uba74 \ubb3c\uc774\ub098 \uacfc\uc0b0\ud654\uc218\uc18c\uc640 \uac19\uc740 \ubd84\uc790\ub85c \ub9cc\ub4e4\uc5b4\uc9c0\ub294 \uac83\uc774 \uc544\ub2c8\ub77c \ube60\uc838\ub098\uc640 \ub2e4\ub978 \uc791\uc6a9\uc744 \uc2dc\uc791\ud558\uae30 \ub54c\ubb38\uc774\ub2e4. \uc218\uc18c \ubd84\uc790 \ub294 \uac00\ubccd\uae30 \ub54c\ubb38\uc5d0 \uc720\ub85c\ud30c\uc758 \uc911\ub825\uc5d0\uc11c \ubc97\uc5b4\ub098\uae30 \uc27d\uace0, \ub2e4\uc2dc\ub294 \ud45c\uba74\uc73c\ub85c \ub3cc\uc544\uac00\uc9c0 \ubabb\ud55c\ub2e4. ", "paragraph_answer": "\uc9c0\uad6c\uc758 \ub300\uae30\uad8c\uacfc\ub294 \ub2e4\ub974\uac8c, \uc720\ub85c\ud30c\uc758 \ub300\uae30\uad8c\uc740 \uc0dd\ubb3c\ud559\uc801\uc73c\ub85c \uc0dd\uaca8\ub09c \uac83\uc774 \uc544\ub2c8\ub77c \ubd84\uc790\ub4e4\uc774 \ubc29\uc0ac\uc120 \ubd84\ud574\uac00 \ub418\uc5b4 \uc0dd\uc131\ub418\uc5c8\ub2e4. \ud0dc\uc591\uc5d0\uc11c \uc624\ub294 \uc790\uc678\uc120\uacfc \ubaa9\uc131\uc758 \uc790\uae30\uad8c\uc5d0\uc11c \uc624\ub294 \ub300\uc804 \uc785\uc790(\uc774\uc628\uc774\ub098 \uc804\uc790)\uac00 \uc720\ub85c\ud30c\uc758 \ubb3c \ubd84\uc790\ub97c \ub54c\ub824, \uc218\uc18c\uc640 \uc0b0\uc18c\ub85c \ub098\ub220 \ub300\uae30 \uc911\uc73c\ub85c \uc2a4\ud37c\ud130\ub9c1\ub418\uac70\ub098 \ud761\uc218\ub418\uac8c \ub9cc\ub4e0\ub2e4. \uac19\uc740 \ubc29\uc0ac\uc120\uc774 \uc774 \ud761\uc218\ub41c \ubb3c\uc9c8\ub4e4\uc744 \ub54c\uc5b4\ub0b4\uace0, \ub450 \uc791\uc6a9\uc774 \uc54c\ub9de\uac8c \uade0\ud615\uc744 \uc774\ub8e8\ub294 \ub300\uae30\uad8c\uc744 \ub9cc\ub4e4\uc5b4\ub0b8\ub2e4. \uc0b0\uc18c \ubd84\uc790\ub294 \uc218\uba85\uc774 \uae38\uae30 \ub54c\ubb38\uc5d0 \ub300\uae30\uc758 \uc8fc\uc694 \uad6c\uc131 \uc131\ubd84\uc774 \ub41c\ub2e4. \uc65c\ub0d0\ud558\uba74 \uc0b0\uc18c \ubd84\uc790\uc640 \uac19\uc740 \ubb3c\uc9c8\ub4e4\uc774 \ud45c\uba74\uc73c\ub85c \ub3cc\uc544\uac00\uba74 \ubb3c\uc774\ub098 \uacfc\uc0b0\ud654\uc218\uc18c\uc640 \uac19\uc740 \ubd84\uc790\ub85c \ub9cc\ub4e4\uc5b4\uc9c0\ub294 \uac83\uc774 \uc544\ub2c8\ub77c \ube60\uc838\ub098\uc640 \ub2e4\ub978 \uc791\uc6a9\uc744 \uc2dc\uc791\ud558\uae30 \ub54c\ubb38\uc774\ub2e4. \uc218\uc18c \ubd84\uc790 \ub294 \uac00\ubccd\uae30 \ub54c\ubb38\uc5d0 \uc720\ub85c\ud30c\uc758 \uc911\ub825\uc5d0\uc11c \ubc97\uc5b4\ub098\uae30 \uc27d\uace0, \ub2e4\uc2dc\ub294 \ud45c\uba74\uc73c\ub85c \ub3cc\uc544\uac00\uc9c0 \ubabb\ud55c\ub2e4.", "sentence_answer": " \uc218\uc18c \ubd84\uc790 \ub294 \uac00\ubccd\uae30 \ub54c\ubb38\uc5d0 \uc720\ub85c\ud30c\uc758 \uc911\ub825\uc5d0\uc11c \ubc97\uc5b4\ub098\uae30 \uc27d\uace0, \ub2e4\uc2dc\ub294 \ud45c\uba74\uc73c\ub85c \ub3cc\uc544\uac00\uc9c0 \ubabb\ud55c\ub2e4.", "paragraph_id": "6527070-9-1", "paragraph_question": "question: \ubb3c \ubd84\uc790\uc758 \uad6c\uc131 \uc131\ubd84 \uc911 \uc720\ub85c\ud30c\uc758 \uc911\ub825\uc5d0\uc11c \ubc97\uc5b4\ub098 \ub300\uae30 \ud45c\uba74\uc73c\ub85c \ub3cc\uc544\uac00\uc9c0 \ubabb\ud558\ub294 \uc131\ubd84\uc740?, context: \uc9c0\uad6c\uc758 \ub300\uae30\uad8c\uacfc\ub294 \ub2e4\ub974\uac8c, \uc720\ub85c\ud30c\uc758 \ub300\uae30\uad8c\uc740 \uc0dd\ubb3c\ud559\uc801\uc73c\ub85c \uc0dd\uaca8\ub09c \uac83\uc774 \uc544\ub2c8\ub77c \ubd84\uc790\ub4e4\uc774 \ubc29\uc0ac\uc120 \ubd84\ud574\uac00 \ub418\uc5b4 \uc0dd\uc131\ub418\uc5c8\ub2e4. \ud0dc\uc591\uc5d0\uc11c \uc624\ub294 \uc790\uc678\uc120\uacfc \ubaa9\uc131\uc758 \uc790\uae30\uad8c\uc5d0\uc11c \uc624\ub294 \ub300\uc804 \uc785\uc790(\uc774\uc628\uc774\ub098 \uc804\uc790)\uac00 \uc720\ub85c\ud30c\uc758 \ubb3c \ubd84\uc790\ub97c \ub54c\ub824, \uc218\uc18c\uc640 \uc0b0\uc18c\ub85c \ub098\ub220 \ub300\uae30 \uc911\uc73c\ub85c \uc2a4\ud37c\ud130\ub9c1\ub418\uac70\ub098 \ud761\uc218\ub418\uac8c \ub9cc\ub4e0\ub2e4. \uac19\uc740 \ubc29\uc0ac\uc120\uc774 \uc774 \ud761\uc218\ub41c \ubb3c\uc9c8\ub4e4\uc744 \ub54c\uc5b4\ub0b4\uace0, \ub450 \uc791\uc6a9\uc774 \uc54c\ub9de\uac8c \uade0\ud615\uc744 \uc774\ub8e8\ub294 \ub300\uae30\uad8c\uc744 \ub9cc\ub4e4\uc5b4\ub0b8\ub2e4. \uc0b0\uc18c \ubd84\uc790\ub294 \uc218\uba85\uc774 \uae38\uae30 \ub54c\ubb38\uc5d0 \ub300\uae30\uc758 \uc8fc\uc694 \uad6c\uc131 \uc131\ubd84\uc774 \ub41c\ub2e4. \uc65c\ub0d0\ud558\uba74 \uc0b0\uc18c \ubd84\uc790\uc640 \uac19\uc740 \ubb3c\uc9c8\ub4e4\uc774 \ud45c\uba74\uc73c\ub85c \ub3cc\uc544\uac00\uba74 \ubb3c\uc774\ub098 \uacfc\uc0b0\ud654\uc218\uc18c\uc640 \uac19\uc740 \ubd84\uc790\ub85c \ub9cc\ub4e4\uc5b4\uc9c0\ub294 \uac83\uc774 \uc544\ub2c8\ub77c \ube60\uc838\ub098\uc640 \ub2e4\ub978 \uc791\uc6a9\uc744 \uc2dc\uc791\ud558\uae30 \ub54c\ubb38\uc774\ub2e4. \uc218\uc18c \ubd84\uc790\ub294 \uac00\ubccd\uae30 \ub54c\ubb38\uc5d0 \uc720\ub85c\ud30c\uc758 \uc911\ub825\uc5d0\uc11c \ubc97\uc5b4\ub098\uae30 \uc27d\uace0, \ub2e4\uc2dc\ub294 \ud45c\uba74\uc73c\ub85c \ub3cc\uc544\uac00\uc9c0 \ubabb\ud55c\ub2e4."} {"question": "\ubd81\ud55c\uc5d0\uc11c \uc778\ubbfc\uc704\uc6d0\ud68c \uc704\uc6d0 \uc120\uac70\uac00 \uc774\ub8e8\uc5b4\uc9c4 \uc5f0\ub3c4\ub294?", "paragraph": "1946\ub144 11\uc6d4 \ubd81\ud55c\uc5d0\uc11c\ub294 \uc778\ubbfc\uc704\uc6d0\ud68c \uc704\uc6d0 \uc120\uac70\uac00 \uc788\uc5c8\ub2e4. \uc774\ub54c \uc6d4\ub0a8\ud588\ub358 \ubc18\uacf5\uc138\ub825\uc774 \uc77c\ubd80\ub2e4\uc2dc \uc6d4\ubd81\ud558\uc5ec \uc120\uac70\ubc29\ud574\ub97c \ub3c4\ubaa8\ud558\uc600\ub2e4. \uae40\uc77c\uc131\uc740 \uc774\ub97c \uc774\uc2b9\ub9cc\uacfc \uae40\uad6c\uac00 \ubc30\ud6c4\uc5d0\uc11c \uc870\uc885\ud55c \uc9d3\uc774\ub77c\uace0 \uc120\uc5b8\ud588\ub2e4. \uc2ec\uc9c0\uc5b4 \uc774\ub4e4\uc758 \ubc29\ud574\ub85c \uc120\uc804\ub300\uc6d0 \uba87 \uba85\uc774 \uc0ac\ub9dd\ud55c \uacbd\uc6b0\ub3c4 \uc788\uc5c8\ub2e4. \uc774\ub4e4\uc740 \u201c\uc120\uac70\uac00 \ube44\ubbfc\uc8fc\uc801\uc774\ub2e4\u201d, \u201c\uc2b9\ub824\ub4e4\uacfc \ubaa9\uc0ac\ub4e4\uc740 \uc120\uac70\ud558\uc9c0 \ub9d0\ub77c\u201d, \u201c\uacf5\ub3d9\ud6c4\ubcf4\uac00 \uc544\ub2c8\ub77c \uc790\uc720\uacbd\uc7c1\u201d \ub4f1\uc73c\ub85c \uc120\uac70\uc5d0 \ube44\ud310\uc801\uc774\uc5c8\uace0, \uc720\uad8c\uc790\ub4e4\uc5d0\uac8c \uc120\uac70\uc5d0\uc11c \ucc2c\uc131\ud558\uba74 \ud751\ud568\uc5d0 \ub123\uc73c\ub77c\ub294 \ud751\ud568\uc6b4\ub3d9\uc744 \ud558\uae30\ub3c4 \ud558\uc600\ub2e4. \uc774\ub4e4\uc5d0 \ub300\ud558\uc5ec \uae40\uc77c\uc131\uc740 \uc778\ubbfc\uc758 \uc6d0\uc218\uc774\uba70 \ubc18\ub3d9\ud30c\ub4e4\uc5d0\uac8c \ub9e4\uc218\ub418\uc5b4 \uadf8\ub4e4\uc758 \uac04\ucca9\ubc30\uac00 \ub41c \uc138\ub825\uc73c\ub85c \ubab0\uc558\ub2e4. \uae40\uc77c\uc131\uc740 \uc120\uac70\uae30\uac04\uc5d0 \uc0dd\uc0b0\uae30\uad00\uc758 \ubc29\ud654, \uc6b4\uc218\ubd80\ubd84\uc5d0\uc11c \ucda9\ub3cc\uc0ac\uac74, \uc8fc\uc694 \uc778\uc0ac\uc758 \uc554\uc0b4\ubbf8\uc218\uc0ac\uac74 \ub4f1\uc774 \ubc18\ub3d9\uc138\ub825\uc758 \ubc18\ub300\uc6b4\ub3d9\uc758 \uc77c\ud658\uc73c\ub85c \ud30c\uc545\ud558\uc600\uace0, \ub098\uc544\uac00\uc11c \ubd81\ud55c\uc5d0\uc11c \uc77c\uc5b4\ub09c \ubc29\ud654\ub97c \uc774\uc2b9\ub9cc\uacfc \uae40\uad6c\uac00 \ud30c\uacac\ud55c \ubc29\ud654\ub2e8\uc73c\ub85c \uaddc\uc815\ud558\uc600\ub2e4.", "answer": "1946\ub144", "sentence": "1946\ub144 11\uc6d4 \ubd81\ud55c\uc5d0\uc11c\ub294 \uc778\ubbfc\uc704\uc6d0\ud68c \uc704\uc6d0 \uc120\uac70\uac00 \uc788\uc5c8\ub2e4.", "paragraph_sentence": " 1946\ub144 11\uc6d4 \ubd81\ud55c\uc5d0\uc11c\ub294 \uc778\ubbfc\uc704\uc6d0\ud68c \uc704\uc6d0 \uc120\uac70\uac00 \uc788\uc5c8\ub2e4. \uc774\ub54c \uc6d4\ub0a8\ud588\ub358 \ubc18\uacf5\uc138\ub825\uc774 \uc77c\ubd80\ub2e4\uc2dc \uc6d4\ubd81\ud558\uc5ec \uc120\uac70\ubc29\ud574\ub97c \ub3c4\ubaa8\ud558\uc600\ub2e4. \uae40\uc77c\uc131\uc740 \uc774\ub97c \uc774\uc2b9\ub9cc\uacfc \uae40\uad6c\uac00 \ubc30\ud6c4\uc5d0\uc11c \uc870\uc885\ud55c \uc9d3\uc774\ub77c\uace0 \uc120\uc5b8\ud588\ub2e4. \uc2ec\uc9c0\uc5b4 \uc774\ub4e4\uc758 \ubc29\ud574\ub85c \uc120\uc804\ub300\uc6d0 \uba87 \uba85\uc774 \uc0ac\ub9dd\ud55c \uacbd\uc6b0\ub3c4 \uc788\uc5c8\ub2e4. \uc774\ub4e4\uc740 \u201c\uc120\uac70\uac00 \ube44\ubbfc\uc8fc\uc801\uc774\ub2e4\u201d, \u201c\uc2b9\ub824\ub4e4\uacfc \ubaa9\uc0ac\ub4e4\uc740 \uc120\uac70\ud558\uc9c0 \ub9d0\ub77c\u201d, \u201c\uacf5\ub3d9\ud6c4\ubcf4\uac00 \uc544\ub2c8\ub77c \uc790\uc720\uacbd\uc7c1\u201d \ub4f1\uc73c\ub85c \uc120\uac70\uc5d0 \ube44\ud310\uc801\uc774\uc5c8\uace0, \uc720\uad8c\uc790\ub4e4\uc5d0\uac8c \uc120\uac70\uc5d0\uc11c \ucc2c\uc131\ud558\uba74 \ud751\ud568\uc5d0 \ub123\uc73c\ub77c\ub294 \ud751\ud568\uc6b4\ub3d9\uc744 \ud558\uae30\ub3c4 \ud558\uc600\ub2e4. \uc774\ub4e4\uc5d0 \ub300\ud558\uc5ec \uae40\uc77c\uc131\uc740 \uc778\ubbfc\uc758 \uc6d0\uc218\uc774\uba70 \ubc18\ub3d9\ud30c\ub4e4\uc5d0\uac8c \ub9e4\uc218\ub418\uc5b4 \uadf8\ub4e4\uc758 \uac04\ucca9\ubc30\uac00 \ub41c \uc138\ub825\uc73c\ub85c \ubab0\uc558\ub2e4. \uae40\uc77c\uc131\uc740 \uc120\uac70\uae30\uac04\uc5d0 \uc0dd\uc0b0\uae30\uad00\uc758 \ubc29\ud654, \uc6b4\uc218\ubd80\ubd84\uc5d0\uc11c \ucda9\ub3cc\uc0ac\uac74, \uc8fc\uc694 \uc778\uc0ac\uc758 \uc554\uc0b4\ubbf8\uc218\uc0ac\uac74 \ub4f1\uc774 \ubc18\ub3d9\uc138\ub825\uc758 \ubc18\ub300\uc6b4\ub3d9\uc758 \uc77c\ud658\uc73c\ub85c \ud30c\uc545\ud558\uc600\uace0, \ub098\uc544\uac00\uc11c \ubd81\ud55c\uc5d0\uc11c \uc77c\uc5b4\ub09c \ubc29\ud654\ub97c \uc774\uc2b9\ub9cc\uacfc \uae40\uad6c\uac00 \ud30c\uacac\ud55c \ubc29\ud654\ub2e8\uc73c\ub85c \uaddc\uc815\ud558\uc600\ub2e4.", "paragraph_answer": " 1946\ub144 11\uc6d4 \ubd81\ud55c\uc5d0\uc11c\ub294 \uc778\ubbfc\uc704\uc6d0\ud68c \uc704\uc6d0 \uc120\uac70\uac00 \uc788\uc5c8\ub2e4. \uc774\ub54c \uc6d4\ub0a8\ud588\ub358 \ubc18\uacf5\uc138\ub825\uc774 \uc77c\ubd80\ub2e4\uc2dc \uc6d4\ubd81\ud558\uc5ec \uc120\uac70\ubc29\ud574\ub97c \ub3c4\ubaa8\ud558\uc600\ub2e4. \uae40\uc77c\uc131\uc740 \uc774\ub97c \uc774\uc2b9\ub9cc\uacfc \uae40\uad6c\uac00 \ubc30\ud6c4\uc5d0\uc11c \uc870\uc885\ud55c \uc9d3\uc774\ub77c\uace0 \uc120\uc5b8\ud588\ub2e4. \uc2ec\uc9c0\uc5b4 \uc774\ub4e4\uc758 \ubc29\ud574\ub85c \uc120\uc804\ub300\uc6d0 \uba87 \uba85\uc774 \uc0ac\ub9dd\ud55c \uacbd\uc6b0\ub3c4 \uc788\uc5c8\ub2e4. \uc774\ub4e4\uc740 \u201c\uc120\uac70\uac00 \ube44\ubbfc\uc8fc\uc801\uc774\ub2e4\u201d, \u201c\uc2b9\ub824\ub4e4\uacfc \ubaa9\uc0ac\ub4e4\uc740 \uc120\uac70\ud558\uc9c0 \ub9d0\ub77c\u201d, \u201c\uacf5\ub3d9\ud6c4\ubcf4\uac00 \uc544\ub2c8\ub77c \uc790\uc720\uacbd\uc7c1\u201d \ub4f1\uc73c\ub85c \uc120\uac70\uc5d0 \ube44\ud310\uc801\uc774\uc5c8\uace0, \uc720\uad8c\uc790\ub4e4\uc5d0\uac8c \uc120\uac70\uc5d0\uc11c \ucc2c\uc131\ud558\uba74 \ud751\ud568\uc5d0 \ub123\uc73c\ub77c\ub294 \ud751\ud568\uc6b4\ub3d9\uc744 \ud558\uae30\ub3c4 \ud558\uc600\ub2e4. \uc774\ub4e4\uc5d0 \ub300\ud558\uc5ec \uae40\uc77c\uc131\uc740 \uc778\ubbfc\uc758 \uc6d0\uc218\uc774\uba70 \ubc18\ub3d9\ud30c\ub4e4\uc5d0\uac8c \ub9e4\uc218\ub418\uc5b4 \uadf8\ub4e4\uc758 \uac04\ucca9\ubc30\uac00 \ub41c \uc138\ub825\uc73c\ub85c \ubab0\uc558\ub2e4. \uae40\uc77c\uc131\uc740 \uc120\uac70\uae30\uac04\uc5d0 \uc0dd\uc0b0\uae30\uad00\uc758 \ubc29\ud654, \uc6b4\uc218\ubd80\ubd84\uc5d0\uc11c \ucda9\ub3cc\uc0ac\uac74, \uc8fc\uc694 \uc778\uc0ac\uc758 \uc554\uc0b4\ubbf8\uc218\uc0ac\uac74 \ub4f1\uc774 \ubc18\ub3d9\uc138\ub825\uc758 \ubc18\ub300\uc6b4\ub3d9\uc758 \uc77c\ud658\uc73c\ub85c \ud30c\uc545\ud558\uc600\uace0, \ub098\uc544\uac00\uc11c \ubd81\ud55c\uc5d0\uc11c \uc77c\uc5b4\ub09c \ubc29\ud654\ub97c \uc774\uc2b9\ub9cc\uacfc \uae40\uad6c\uac00 \ud30c\uacac\ud55c \ubc29\ud654\ub2e8\uc73c\ub85c \uaddc\uc815\ud558\uc600\ub2e4.", "sentence_answer": " 1946\ub144 11\uc6d4 \ubd81\ud55c\uc5d0\uc11c\ub294 \uc778\ubbfc\uc704\uc6d0\ud68c \uc704\uc6d0 \uc120\uac70\uac00 \uc788\uc5c8\ub2e4.", "paragraph_id": "6487890-0-0", "paragraph_question": "question: \ubd81\ud55c\uc5d0\uc11c \uc778\ubbfc\uc704\uc6d0\ud68c \uc704\uc6d0 \uc120\uac70\uac00 \uc774\ub8e8\uc5b4\uc9c4 \uc5f0\ub3c4\ub294?, context: 1946\ub144 11\uc6d4 \ubd81\ud55c\uc5d0\uc11c\ub294 \uc778\ubbfc\uc704\uc6d0\ud68c \uc704\uc6d0 \uc120\uac70\uac00 \uc788\uc5c8\ub2e4. \uc774\ub54c \uc6d4\ub0a8\ud588\ub358 \ubc18\uacf5\uc138\ub825\uc774 \uc77c\ubd80\ub2e4\uc2dc \uc6d4\ubd81\ud558\uc5ec \uc120\uac70\ubc29\ud574\ub97c \ub3c4\ubaa8\ud558\uc600\ub2e4. \uae40\uc77c\uc131\uc740 \uc774\ub97c \uc774\uc2b9\ub9cc\uacfc \uae40\uad6c\uac00 \ubc30\ud6c4\uc5d0\uc11c \uc870\uc885\ud55c \uc9d3\uc774\ub77c\uace0 \uc120\uc5b8\ud588\ub2e4. \uc2ec\uc9c0\uc5b4 \uc774\ub4e4\uc758 \ubc29\ud574\ub85c \uc120\uc804\ub300\uc6d0 \uba87 \uba85\uc774 \uc0ac\ub9dd\ud55c \uacbd\uc6b0\ub3c4 \uc788\uc5c8\ub2e4. \uc774\ub4e4\uc740 \u201c\uc120\uac70\uac00 \ube44\ubbfc\uc8fc\uc801\uc774\ub2e4\u201d, \u201c\uc2b9\ub824\ub4e4\uacfc \ubaa9\uc0ac\ub4e4\uc740 \uc120\uac70\ud558\uc9c0 \ub9d0\ub77c\u201d, \u201c\uacf5\ub3d9\ud6c4\ubcf4\uac00 \uc544\ub2c8\ub77c \uc790\uc720\uacbd\uc7c1\u201d \ub4f1\uc73c\ub85c \uc120\uac70\uc5d0 \ube44\ud310\uc801\uc774\uc5c8\uace0, \uc720\uad8c\uc790\ub4e4\uc5d0\uac8c \uc120\uac70\uc5d0\uc11c \ucc2c\uc131\ud558\uba74 \ud751\ud568\uc5d0 \ub123\uc73c\ub77c\ub294 \ud751\ud568\uc6b4\ub3d9\uc744 \ud558\uae30\ub3c4 \ud558\uc600\ub2e4. \uc774\ub4e4\uc5d0 \ub300\ud558\uc5ec \uae40\uc77c\uc131\uc740 \uc778\ubbfc\uc758 \uc6d0\uc218\uc774\uba70 \ubc18\ub3d9\ud30c\ub4e4\uc5d0\uac8c \ub9e4\uc218\ub418\uc5b4 \uadf8\ub4e4\uc758 \uac04\ucca9\ubc30\uac00 \ub41c \uc138\ub825\uc73c\ub85c \ubab0\uc558\ub2e4. \uae40\uc77c\uc131\uc740 \uc120\uac70\uae30\uac04\uc5d0 \uc0dd\uc0b0\uae30\uad00\uc758 \ubc29\ud654, \uc6b4\uc218\ubd80\ubd84\uc5d0\uc11c \ucda9\ub3cc\uc0ac\uac74, \uc8fc\uc694 \uc778\uc0ac\uc758 \uc554\uc0b4\ubbf8\uc218\uc0ac\uac74 \ub4f1\uc774 \ubc18\ub3d9\uc138\ub825\uc758 \ubc18\ub300\uc6b4\ub3d9\uc758 \uc77c\ud658\uc73c\ub85c \ud30c\uc545\ud558\uc600\uace0, \ub098\uc544\uac00\uc11c \ubd81\ud55c\uc5d0\uc11c \uc77c\uc5b4\ub09c \ubc29\ud654\ub97c \uc774\uc2b9\ub9cc\uacfc \uae40\uad6c\uac00 \ud30c\uacac\ud55c \ubc29\ud654\ub2e8\uc73c\ub85c \uaddc\uc815\ud558\uc600\ub2e4."} {"question": "\uc790\uc720\ubbfc\uc8fc\uc8fc\uc758\uac00 \ub300\ud55c\ubbfc\uad6d\uc758 \uc815\uce58 \uc774\ub150\uc774 \ub41c \uc5f0\ub3c4\ub294?", "paragraph": "\uc815\uce58\uc801 \uc790\uc720\uc8fc\uc758\ub77c\ub294 \uc758\ubbf8\uc5d0\uc11c\uc758 \uc790\uc720\ubbfc\uc8fc\uc8fc\uc758\ub294 1919\ub144 \ub300\ud55c\ubbfc\uad6d \uc784\uc2dc \uc815\ubd80\uc758 \ub300\ud55c\ubbfc\uad6d \uc784\uc2dc \ud5cc\uc7a5\uc5d0\uc11c \uad6d\uccb4\ub97c \ubbfc\uc8fc\uacf5\ud654\uc81c\ub85c \uaddc\uc815\ud558\uace0, \uc815\uce58\uc801 \uc0ac\ud68c\uc801 \uc790\uc720 \uad8c\ub9ac\ub97c \uba85\uc2dc\ud55c \ub370\uc11c \ucd9c\ubc1c\ud558\uc5ec 1948\ub144 \ub300\ud55c\ubbfc\uad6d \uc81c\ud5cc \ud5cc\ubc95\uc758 \uacf5\uc2dd \uc81c\uc815\uc73c\ub85c \ub300\ud55c\ubbfc\uad6d \ubbfc\uc8fc\uacf5\ud654\uad6d\uc758 \uc815\uce58 \uc774\ub150\uc774 \ub418\uc5c8\ub2e4. \ud558\uc9c0\ub9cc \uc774\uc2b9\ub9cc\uacfc \uad70\uc0ac\uc815\uad8c\uc744 \uac70\uce58\uba74\uc11c \uc790\uc720\ubbfc\uc8fc\uc8fc\uc758\ub97c \uac15\uc870\ud558\ub294 \uad8c\ub825\uc5d0 \ub9de\uc11c \ubc18\ub3c5\uc7ac \ubbfc\uc8fc\ud654 \uc6b4\ub3d9\uc744 \ud558\ub358 \uc9c4\ubcf4 \uc9c4\uc601\uc744 \uc911\uc2ec\uc73c\ub85c \uc790\uc720\ubbfc\uc8fc\uc8fc\uc758\ubcf4\ub2e4 \ubbfc\uc8fc\uc8fc\uc758\ub97c \uac15\uc870\ud558\uc600\ub2e4. 2018\ub144\uc5d0 \ubb38\uc7ac\uc778 \uc815\ubd80\uac00 \ub300\ud1b5\ub839 \uac1c\ud5cc\uc548\uc744 \uc81c\uc2dc\ud558\uba74\uc11c \uc790\uc720\ubbfc\uc8fc\uc8fc\uc758 \uc218\ud638\ub97c \uc911\uc694\ud55c \uac00\uce58\ub85c \uc5ec\uae30\ub294 \ubcf4\uc218 \uc9c4\uc601\uc73c\ub85c\ubd80\ud130 \uc790\uc720\uac00 \ube60\uc9c4 \ubbfc\uc8fc\uc8fc\uc758\uc5d0 \ub300\ud574 \ube44\ud310\uc744 \ubc1b\uc558\ub2e4. \ub300\ud55c\ubbfc\uad6d \ud5cc\ubc95\uc5d0\uc11c\ub294 \uc790\uc720\ubbfc\uc8fc\uc8fc\uc758\ubcf4\ub2e4 \uc790\uc720\ubbfc\uc8fc\uc801 \uae30\ubcf8\uc9c8\uc11c\ub77c\uace0 \ud45c\ud604\ud558\uace0 \uc788\uc73c\uba70 \ub300\ud55c\ubbfc\uad6d\uc758 \ud5cc\ubc95\uc7ac\ud310\uc18c\ub294 \uc774\uc5d0 \ub300\ud574 \"\uc790\uc758\uc801 \uc9c0\ubc30\uc640 \ud3ed\ub825\uc801 \uc9c0\ubc30\ub97c \ubc30\uc81c\ud558\ub294 \uac83\uc744 \uc758\ubbf8\ud55c\ub2e4\"\uace0 \ubc1d\ud614\ub2e4. \ubcf4\uc218 \uc9c4\uc601\uc774 \ube44\ud310\ud558\ub294 \uc774\uc11d\uae30 \uc758\uc6d0\uc5d0 \ub300\ud574 \ubbf8\ud569\uc911\uad6d \uc815\ubd80\ub294 \uc790\uc758\uc801 \uad6c\uc18d\uc774\ub77c\uace0 \ub17c\ud3c9\ud588\ub2e4.", "answer": "1948\ub144", "sentence": "\uc815\uce58\uc801 \uc790\uc720\uc8fc\uc758\ub77c\ub294 \uc758\ubbf8\uc5d0\uc11c\uc758 \uc790\uc720\ubbfc\uc8fc\uc8fc\uc758\ub294 1919\ub144 \ub300\ud55c\ubbfc\uad6d \uc784\uc2dc \uc815\ubd80\uc758 \ub300\ud55c\ubbfc\uad6d \uc784\uc2dc \ud5cc\uc7a5\uc5d0\uc11c \uad6d\uccb4\ub97c \ubbfc\uc8fc\uacf5\ud654\uc81c\ub85c \uaddc\uc815\ud558\uace0, \uc815\uce58\uc801 \uc0ac\ud68c\uc801 \uc790\uc720 \uad8c\ub9ac\ub97c \uba85\uc2dc\ud55c \ub370\uc11c \ucd9c\ubc1c\ud558\uc5ec 1948\ub144 \ub300\ud55c\ubbfc\uad6d \uc81c\ud5cc \ud5cc\ubc95\uc758 \uacf5\uc2dd \uc81c\uc815\uc73c\ub85c \ub300\ud55c\ubbfc\uad6d \ubbfc\uc8fc\uacf5\ud654\uad6d\uc758 \uc815\uce58 \uc774\ub150\uc774 \ub418\uc5c8\ub2e4.", "paragraph_sentence": " \uc815\uce58\uc801 \uc790\uc720\uc8fc\uc758\ub77c\ub294 \uc758\ubbf8\uc5d0\uc11c\uc758 \uc790\uc720\ubbfc\uc8fc\uc8fc\uc758\ub294 1919\ub144 \ub300\ud55c\ubbfc\uad6d \uc784\uc2dc \uc815\ubd80\uc758 \ub300\ud55c\ubbfc\uad6d \uc784\uc2dc \ud5cc\uc7a5\uc5d0\uc11c \uad6d\uccb4\ub97c \ubbfc\uc8fc\uacf5\ud654\uc81c\ub85c \uaddc\uc815\ud558\uace0, \uc815\uce58\uc801 \uc0ac\ud68c\uc801 \uc790\uc720 \uad8c\ub9ac\ub97c \uba85\uc2dc\ud55c \ub370\uc11c \ucd9c\ubc1c\ud558\uc5ec 1948\ub144 \ub300\ud55c\ubbfc\uad6d \uc81c\ud5cc \ud5cc\ubc95\uc758 \uacf5\uc2dd \uc81c\uc815\uc73c\ub85c \ub300\ud55c\ubbfc\uad6d \ubbfc\uc8fc\uacf5\ud654\uad6d\uc758 \uc815\uce58 \uc774\ub150\uc774 \ub418\uc5c8\ub2e4. \ud558\uc9c0\ub9cc \uc774\uc2b9\ub9cc\uacfc \uad70\uc0ac\uc815\uad8c\uc744 \uac70\uce58\uba74\uc11c \uc790\uc720\ubbfc\uc8fc\uc8fc\uc758\ub97c \uac15\uc870\ud558\ub294 \uad8c\ub825\uc5d0 \ub9de\uc11c \ubc18\ub3c5\uc7ac \ubbfc\uc8fc\ud654 \uc6b4\ub3d9\uc744 \ud558\ub358 \uc9c4\ubcf4 \uc9c4\uc601\uc744 \uc911\uc2ec\uc73c\ub85c \uc790\uc720\ubbfc\uc8fc\uc8fc\uc758\ubcf4\ub2e4 \ubbfc\uc8fc\uc8fc\uc758\ub97c \uac15\uc870\ud558\uc600\ub2e4. 2018\ub144\uc5d0 \ubb38\uc7ac\uc778 \uc815\ubd80\uac00 \ub300\ud1b5\ub839 \uac1c\ud5cc\uc548\uc744 \uc81c\uc2dc\ud558\uba74\uc11c \uc790\uc720\ubbfc\uc8fc\uc8fc\uc758 \uc218\ud638\ub97c \uc911\uc694\ud55c \uac00\uce58\ub85c \uc5ec\uae30\ub294 \ubcf4\uc218 \uc9c4\uc601\uc73c\ub85c\ubd80\ud130 \uc790\uc720\uac00 \ube60\uc9c4 \ubbfc\uc8fc\uc8fc\uc758\uc5d0 \ub300\ud574 \ube44\ud310\uc744 \ubc1b\uc558\ub2e4. \ub300\ud55c\ubbfc\uad6d \ud5cc\ubc95\uc5d0\uc11c\ub294 \uc790\uc720\ubbfc\uc8fc\uc8fc\uc758\ubcf4\ub2e4 \uc790\uc720\ubbfc\uc8fc\uc801 \uae30\ubcf8\uc9c8\uc11c\ub77c\uace0 \ud45c\ud604\ud558\uace0 \uc788\uc73c\uba70 \ub300\ud55c\ubbfc\uad6d\uc758 \ud5cc\ubc95\uc7ac\ud310\uc18c\ub294 \uc774\uc5d0 \ub300\ud574 \"\uc790\uc758\uc801 \uc9c0\ubc30\uc640 \ud3ed\ub825\uc801 \uc9c0\ubc30\ub97c \ubc30\uc81c\ud558\ub294 \uac83\uc744 \uc758\ubbf8\ud55c\ub2e4\"\uace0 \ubc1d\ud614\ub2e4. \ubcf4\uc218 \uc9c4\uc601\uc774 \ube44\ud310\ud558\ub294 \uc774\uc11d\uae30 \uc758\uc6d0\uc5d0 \ub300\ud574 \ubbf8\ud569\uc911\uad6d \uc815\ubd80\ub294 \uc790\uc758\uc801 \uad6c\uc18d\uc774\ub77c\uace0 \ub17c\ud3c9\ud588\ub2e4.", "paragraph_answer": "\uc815\uce58\uc801 \uc790\uc720\uc8fc\uc758\ub77c\ub294 \uc758\ubbf8\uc5d0\uc11c\uc758 \uc790\uc720\ubbfc\uc8fc\uc8fc\uc758\ub294 1919\ub144 \ub300\ud55c\ubbfc\uad6d \uc784\uc2dc \uc815\ubd80\uc758 \ub300\ud55c\ubbfc\uad6d \uc784\uc2dc \ud5cc\uc7a5\uc5d0\uc11c \uad6d\uccb4\ub97c \ubbfc\uc8fc\uacf5\ud654\uc81c\ub85c \uaddc\uc815\ud558\uace0, \uc815\uce58\uc801 \uc0ac\ud68c\uc801 \uc790\uc720 \uad8c\ub9ac\ub97c \uba85\uc2dc\ud55c \ub370\uc11c \ucd9c\ubc1c\ud558\uc5ec 1948\ub144 \ub300\ud55c\ubbfc\uad6d \uc81c\ud5cc \ud5cc\ubc95\uc758 \uacf5\uc2dd \uc81c\uc815\uc73c\ub85c \ub300\ud55c\ubbfc\uad6d \ubbfc\uc8fc\uacf5\ud654\uad6d\uc758 \uc815\uce58 \uc774\ub150\uc774 \ub418\uc5c8\ub2e4. \ud558\uc9c0\ub9cc \uc774\uc2b9\ub9cc\uacfc \uad70\uc0ac\uc815\uad8c\uc744 \uac70\uce58\uba74\uc11c \uc790\uc720\ubbfc\uc8fc\uc8fc\uc758\ub97c \uac15\uc870\ud558\ub294 \uad8c\ub825\uc5d0 \ub9de\uc11c \ubc18\ub3c5\uc7ac \ubbfc\uc8fc\ud654 \uc6b4\ub3d9\uc744 \ud558\ub358 \uc9c4\ubcf4 \uc9c4\uc601\uc744 \uc911\uc2ec\uc73c\ub85c \uc790\uc720\ubbfc\uc8fc\uc8fc\uc758\ubcf4\ub2e4 \ubbfc\uc8fc\uc8fc\uc758\ub97c \uac15\uc870\ud558\uc600\ub2e4. 2018\ub144\uc5d0 \ubb38\uc7ac\uc778 \uc815\ubd80\uac00 \ub300\ud1b5\ub839 \uac1c\ud5cc\uc548\uc744 \uc81c\uc2dc\ud558\uba74\uc11c \uc790\uc720\ubbfc\uc8fc\uc8fc\uc758 \uc218\ud638\ub97c \uc911\uc694\ud55c \uac00\uce58\ub85c \uc5ec\uae30\ub294 \ubcf4\uc218 \uc9c4\uc601\uc73c\ub85c\ubd80\ud130 \uc790\uc720\uac00 \ube60\uc9c4 \ubbfc\uc8fc\uc8fc\uc758\uc5d0 \ub300\ud574 \ube44\ud310\uc744 \ubc1b\uc558\ub2e4. \ub300\ud55c\ubbfc\uad6d \ud5cc\ubc95\uc5d0\uc11c\ub294 \uc790\uc720\ubbfc\uc8fc\uc8fc\uc758\ubcf4\ub2e4 \uc790\uc720\ubbfc\uc8fc\uc801 \uae30\ubcf8\uc9c8\uc11c\ub77c\uace0 \ud45c\ud604\ud558\uace0 \uc788\uc73c\uba70 \ub300\ud55c\ubbfc\uad6d\uc758 \ud5cc\ubc95\uc7ac\ud310\uc18c\ub294 \uc774\uc5d0 \ub300\ud574 \"\uc790\uc758\uc801 \uc9c0\ubc30\uc640 \ud3ed\ub825\uc801 \uc9c0\ubc30\ub97c \ubc30\uc81c\ud558\ub294 \uac83\uc744 \uc758\ubbf8\ud55c\ub2e4\"\uace0 \ubc1d\ud614\ub2e4. \ubcf4\uc218 \uc9c4\uc601\uc774 \ube44\ud310\ud558\ub294 \uc774\uc11d\uae30 \uc758\uc6d0\uc5d0 \ub300\ud574 \ubbf8\ud569\uc911\uad6d \uc815\ubd80\ub294 \uc790\uc758\uc801 \uad6c\uc18d\uc774\ub77c\uace0 \ub17c\ud3c9\ud588\ub2e4.", "sentence_answer": "\uc815\uce58\uc801 \uc790\uc720\uc8fc\uc758\ub77c\ub294 \uc758\ubbf8\uc5d0\uc11c\uc758 \uc790\uc720\ubbfc\uc8fc\uc8fc\uc758\ub294 1919\ub144 \ub300\ud55c\ubbfc\uad6d \uc784\uc2dc \uc815\ubd80\uc758 \ub300\ud55c\ubbfc\uad6d \uc784\uc2dc \ud5cc\uc7a5\uc5d0\uc11c \uad6d\uccb4\ub97c \ubbfc\uc8fc\uacf5\ud654\uc81c\ub85c \uaddc\uc815\ud558\uace0, \uc815\uce58\uc801 \uc0ac\ud68c\uc801 \uc790\uc720 \uad8c\ub9ac\ub97c \uba85\uc2dc\ud55c \ub370\uc11c \ucd9c\ubc1c\ud558\uc5ec 1948\ub144 \ub300\ud55c\ubbfc\uad6d \uc81c\ud5cc \ud5cc\ubc95\uc758 \uacf5\uc2dd \uc81c\uc815\uc73c\ub85c \ub300\ud55c\ubbfc\uad6d \ubbfc\uc8fc\uacf5\ud654\uad6d\uc758 \uc815\uce58 \uc774\ub150\uc774 \ub418\uc5c8\ub2e4.", "paragraph_id": "6657592-3-0", "paragraph_question": "question: \uc790\uc720\ubbfc\uc8fc\uc8fc\uc758\uac00 \ub300\ud55c\ubbfc\uad6d\uc758 \uc815\uce58 \uc774\ub150\uc774 \ub41c \uc5f0\ub3c4\ub294?, context: \uc815\uce58\uc801 \uc790\uc720\uc8fc\uc758\ub77c\ub294 \uc758\ubbf8\uc5d0\uc11c\uc758 \uc790\uc720\ubbfc\uc8fc\uc8fc\uc758\ub294 1919\ub144 \ub300\ud55c\ubbfc\uad6d \uc784\uc2dc \uc815\ubd80\uc758 \ub300\ud55c\ubbfc\uad6d \uc784\uc2dc \ud5cc\uc7a5\uc5d0\uc11c \uad6d\uccb4\ub97c \ubbfc\uc8fc\uacf5\ud654\uc81c\ub85c \uaddc\uc815\ud558\uace0, \uc815\uce58\uc801 \uc0ac\ud68c\uc801 \uc790\uc720 \uad8c\ub9ac\ub97c \uba85\uc2dc\ud55c \ub370\uc11c \ucd9c\ubc1c\ud558\uc5ec 1948\ub144 \ub300\ud55c\ubbfc\uad6d \uc81c\ud5cc \ud5cc\ubc95\uc758 \uacf5\uc2dd \uc81c\uc815\uc73c\ub85c \ub300\ud55c\ubbfc\uad6d \ubbfc\uc8fc\uacf5\ud654\uad6d\uc758 \uc815\uce58 \uc774\ub150\uc774 \ub418\uc5c8\ub2e4. \ud558\uc9c0\ub9cc \uc774\uc2b9\ub9cc\uacfc \uad70\uc0ac\uc815\uad8c\uc744 \uac70\uce58\uba74\uc11c \uc790\uc720\ubbfc\uc8fc\uc8fc\uc758\ub97c \uac15\uc870\ud558\ub294 \uad8c\ub825\uc5d0 \ub9de\uc11c \ubc18\ub3c5\uc7ac \ubbfc\uc8fc\ud654 \uc6b4\ub3d9\uc744 \ud558\ub358 \uc9c4\ubcf4 \uc9c4\uc601\uc744 \uc911\uc2ec\uc73c\ub85c \uc790\uc720\ubbfc\uc8fc\uc8fc\uc758\ubcf4\ub2e4 \ubbfc\uc8fc\uc8fc\uc758\ub97c \uac15\uc870\ud558\uc600\ub2e4. 2018\ub144\uc5d0 \ubb38\uc7ac\uc778 \uc815\ubd80\uac00 \ub300\ud1b5\ub839 \uac1c\ud5cc\uc548\uc744 \uc81c\uc2dc\ud558\uba74\uc11c \uc790\uc720\ubbfc\uc8fc\uc8fc\uc758 \uc218\ud638\ub97c \uc911\uc694\ud55c \uac00\uce58\ub85c \uc5ec\uae30\ub294 \ubcf4\uc218 \uc9c4\uc601\uc73c\ub85c\ubd80\ud130 \uc790\uc720\uac00 \ube60\uc9c4 \ubbfc\uc8fc\uc8fc\uc758\uc5d0 \ub300\ud574 \ube44\ud310\uc744 \ubc1b\uc558\ub2e4. \ub300\ud55c\ubbfc\uad6d \ud5cc\ubc95\uc5d0\uc11c\ub294 \uc790\uc720\ubbfc\uc8fc\uc8fc\uc758\ubcf4\ub2e4 \uc790\uc720\ubbfc\uc8fc\uc801 \uae30\ubcf8\uc9c8\uc11c\ub77c\uace0 \ud45c\ud604\ud558\uace0 \uc788\uc73c\uba70 \ub300\ud55c\ubbfc\uad6d\uc758 \ud5cc\ubc95\uc7ac\ud310\uc18c\ub294 \uc774\uc5d0 \ub300\ud574 \"\uc790\uc758\uc801 \uc9c0\ubc30\uc640 \ud3ed\ub825\uc801 \uc9c0\ubc30\ub97c \ubc30\uc81c\ud558\ub294 \uac83\uc744 \uc758\ubbf8\ud55c\ub2e4\"\uace0 \ubc1d\ud614\ub2e4. \ubcf4\uc218 \uc9c4\uc601\uc774 \ube44\ud310\ud558\ub294 \uc774\uc11d\uae30 \uc758\uc6d0\uc5d0 \ub300\ud574 \ubbf8\ud569\uc911\uad6d \uc815\ubd80\ub294 \uc790\uc758\uc801 \uad6c\uc18d\uc774\ub77c\uace0 \ub17c\ud3c9\ud588\ub2e4."} {"question": "\uc804\uc778\ubc94\uc774 \uc804\uc5ed \ud6c4 \ud2b9\uc804\ub3d9\uc9c0\ud68c\uc5d0\uc11c \ucde8\uc784\ud55c \uc9c1\uc704\ub294?", "paragraph": "2016\ub144 10\uc6d4\ubd80\ud130 \ube0c\ub8e8\ud0b9\uc2a4 \uc5f0\uad6c\uc18c \ub3d9\uc544\uc2dc\uc544\uc815\ucc45\uc5f0\uad6c\uc13c\ud130 \ubc29\ubb38\uc5f0\uad6c\uc6d0, \uc874\uc2a4 \ud649\ud0a8\uc2a4 \ub300\ud559\uad50 \uad6d\uc81c\uad00\uacc4\ub300\ud559\uc6d0 \uac1d\uc6d0\uc5f0\uad6c\uc6d0\uc73c\ub85c 1\ub144 \ub3d9\uc548 \ud65c\ub3d9\ud588\ub2e4. \ubbf8\uad6d \uc5f0\uad6c\uae30\uad00\uc758 \uc5f0\uc218\ub294 \uc5f0\uc218\uc790\uac00 \uc8fc\uc81c\ub97c \uc120\uc815\ud574 \uadf8 \ubd84\uc57c\ub97c \uc5f0\uad6c\ud558\uba70 \uc790\uc2e0\uc758 \uc804\ubb38 \ubd84\uc57c\ub97c \ub2e4\ub978 \uc5f0\uad6c\uc790\ub4e4\uacfc \uacf5\uc720\ud558\ub294 \ud65c\ub3d9\uc744 \ud558\ub294\ub370, \uadf8\ub294 \uc804\uc2dc\uc791\uc804\ud1b5\uc81c\uad8c \ubb38\uc81c, \ud55c\uad6d\uc758 \ud575\ubb34\uc7a5, \ud55c\ubbf8 \uad70\uc0ac\ub3d9\ub9f9 \uac15\ud654 \ubc29\uc548 \ub4f1\uc744 \uc911\uc2ec\uc73c\ub85c \uc5f0\uad6c\ud558\uc600\ub2e4. \uc5f0\uc218 3\uac1c\uc6d4 \ub3d9\uc548 \ud504\ub9b0\uc2a4\ud134 \ub300\ud559\uad50 \ub4f1 \ud559\uad50\uc5d0\uc11c \ud2b9\uac15, \ube0c\ub8e8\ud0b9\uc2a4 \uc5f0\uad6c\uc18c\uc5d0\uc11c \uc8fc\uad00\ud558\ub294 \uc138\ubbf8\ub098 \ucc38\uc5ec, \uc6cc\uc2f1\ud134 \ub0b4\uc758 \ub9ce\uc740 \uc5f0\uad6c\uc18c\uc640 \uae30\uad00 \ubc1c\ud45c, \uc138\ubbf8\ub098, \ud68c\uc758\uc5d0 \ucc38\uc11d\ud574 \ubbf8\uad6d\uc758 \uc0dd\uac01\uc744 \uc774\ud574\ud558\uace0 \ud55c\uad6d\uc758 \uc785\uc7a5\uc744 \ubbf8\uad6d \uc804\ubb38\uac00\ub4e4\uc5d0\uac8c \uc801\uadf9 \uc124\uba85\ud558\uc600\ub2e4\uace0 \ub9d0\ud588\ub2e4. 2017\ub144 1\uc6d4\uc5d0\ub294 \ud2b9\uc804\uc0ac \uc608\ube44\uc5ed\ub4e4\uc744 \uc911\uc2ec\uc73c\ub85c \uacb0\uc131\ub418\uc5b4 \uc804\uad6d\uc801\uc73c\ub85c 2\ub9cc 6\ucc9c\uc5ec \uba85\uc758 \ud68c\uc6d0\uc774 \ud65c\ub3d9\ud558\uace0 \uc788\ub294 \ube44\uc601\ub9ac \uc0ac\ub2e8\ubc95\uc778 \ubd09\uc0ac\ub2e8\uccb4\uc778 \ud2b9\uc804\ub3d9\uc9c0\ud68c\uc758 \uc81c2\ub300 \ucd1d\uc7ac\ub85c \ucde8\uc784\ud558\uc600\ub2e4. \uc804 \ucd1d\uc7ac\ub294 \ucde8\uc784\uc0ac\ub97c \ud1b5\ud574 \u201c\uc0c8\ub85c\uc6b4 \uc2dc\ub300 \ubcc0\ud654\uc5d0 \ubd80\uc751\ud558\ub294 \ub300\ud55c\ubbfc\uad6d \uce5c\ubaa9\ub2e8\uccb4\uc758 \uc0c8\ub85c\uc6b4 \u7ae0\uc744 \uc5f4\uaca0\ub2e4\u201d\uace0 \ub2e4\uc9d0\ud558\uba70, \u201c\ud604\uc5ed \ud6c4\ubc30\ub4e4\uc758 \uad8c\uc775\uc744 \uc704\ud574 \uc55e\uc7a5\uc11c\uace0, \ud68c\uc6d0\uc744 \ud3b8\uc560\ud558\uc9c0 \uc54a\ub294 \ubaa8\ub450\uc758 \ud2b9\uc804\ub3d9\uc9c0\ud68c\ub85c \ub9cc\ub4e4\uc5b4 \ub098\uac00\uaca0\ub2e4\u201d\uace0 \ud3ec\ubd80\ub97c \uc5ed\uc124\ud588\ub2e4. \uac19\uc740 \ub2ec \ubd81\ud55c\uc758 \uae40\uc815\uc740 \ub178\ub3d9\ub2f9 \uc704\uc6d0\uc7a5\uc5d0\uac8c \ub300\ub959\uac04\ud0c4\ub3c4\ubbf8\uc0ac\uc77c \uac1c\ubc1c \ub4f1\uc758 \uc7ac\uace0\ub97c \ucd09\uad6c\ud558\ub294 \ub0b4\uc6a9\uc73c\ub85c A4 \uc6a9\uc9c0 4\uc7a5 \ubd84\ub7c9\uc758 \uc601\ubb38\ud3b8\uc9c0\ub97c \ubcf4\ub0b4\uae30\ub3c4 \ud558\uc600\ub2e4. \u2018\uae40\uc815\uc740 \uc704\uc6d0\uc7a5\ub2d8\uaed8: \uc804 \ub300\ud55c\ubbfc\uad6d \uc721\uad70 \uc911\uc7a5\uc758 \uacf5\uac1c\ud3b8\uc9c0\u2019\ub77c\ub294 \uc81c\ubaa9\uc758 \uc774 \ud3b8\uc9c0\uc5d0\uc11c \uadf8\ub294 \u201c\ud3c9\ud654\ub97c \ucd94\uad6c\ud560 \uae30\ud68c\ub294 \uc5b8\uc81c\ub098 \uc5f4\ub824 \uc788\uc2b5\ub2c8\ub2e4\u201d\ub77c\uba74\uc11c \u201c\uac00\uc7a5 \ud544\uc218\uc801\uc778 \uccab \ub2e8\uacc4\ub294 \uacf5\uacf5\uc5f0\ud55c \uc704\ud611\uc758 \ud68c\ud53c, \ud575\uacfc \ubbf8\uc0ac\uc77c \uc2e4\ud5d8\uc758 \uc911\ub2e8, \ud575\ubb34\uae30 \uac1c\ubc1c\ubd80\ud130 \uc778\uad8c\uc5d0 \uc774\ub974\uae30\uae4c\uc9c0 \uc2e0\ub8b0\ub97c \uc99d\uc9c4\uc2dc\ud0ac \ud611\uc0c1\uc5d0 \ucc38\uc5ec\ud558\ub294 \uac83 \ub4f1\uc744 \ud3ec\ud568\ud569\ub2c8\ub2e4\u201d\ub77c\uace0 \ud3c9\ud654 \ud504\ub85c\uc138\uc2a4\ub97c \uc81c\uc548\ud588\ub2e4.", "answer": "\uc81c2\ub300 \ucd1d\uc7ac", "sentence": "2017\ub144 1\uc6d4\uc5d0\ub294 \ud2b9\uc804\uc0ac \uc608\ube44\uc5ed\ub4e4\uc744 \uc911\uc2ec\uc73c\ub85c \uacb0\uc131\ub418\uc5b4 \uc804\uad6d\uc801\uc73c\ub85c 2\ub9cc 6\ucc9c\uc5ec \uba85\uc758 \ud68c\uc6d0\uc774 \ud65c\ub3d9\ud558\uace0 \uc788\ub294 \ube44\uc601\ub9ac \uc0ac\ub2e8\ubc95\uc778 \ubd09\uc0ac\ub2e8\uccb4\uc778 \ud2b9\uc804\ub3d9\uc9c0\ud68c\uc758 \uc81c2\ub300 \ucd1d\uc7ac \ub85c \ucde8\uc784\ud558\uc600\ub2e4.", "paragraph_sentence": "2016\ub144 10\uc6d4\ubd80\ud130 \ube0c\ub8e8\ud0b9\uc2a4 \uc5f0\uad6c\uc18c \ub3d9\uc544\uc2dc\uc544\uc815\ucc45\uc5f0\uad6c\uc13c\ud130 \ubc29\ubb38\uc5f0\uad6c\uc6d0, \uc874\uc2a4 \ud649\ud0a8\uc2a4 \ub300\ud559\uad50 \uad6d\uc81c\uad00\uacc4\ub300\ud559\uc6d0 \uac1d\uc6d0\uc5f0\uad6c\uc6d0\uc73c\ub85c 1\ub144 \ub3d9\uc548 \ud65c\ub3d9\ud588\ub2e4. \ubbf8\uad6d \uc5f0\uad6c\uae30\uad00\uc758 \uc5f0\uc218\ub294 \uc5f0\uc218\uc790\uac00 \uc8fc\uc81c\ub97c \uc120\uc815\ud574 \uadf8 \ubd84\uc57c\ub97c \uc5f0\uad6c\ud558\uba70 \uc790\uc2e0\uc758 \uc804\ubb38 \ubd84\uc57c\ub97c \ub2e4\ub978 \uc5f0\uad6c\uc790\ub4e4\uacfc \uacf5\uc720\ud558\ub294 \ud65c\ub3d9\uc744 \ud558\ub294\ub370, \uadf8\ub294 \uc804\uc2dc\uc791\uc804\ud1b5\uc81c\uad8c \ubb38\uc81c, \ud55c\uad6d\uc758 \ud575\ubb34\uc7a5, \ud55c\ubbf8 \uad70\uc0ac\ub3d9\ub9f9 \uac15\ud654 \ubc29\uc548 \ub4f1\uc744 \uc911\uc2ec\uc73c\ub85c \uc5f0\uad6c\ud558\uc600\ub2e4. \uc5f0\uc218 3\uac1c\uc6d4 \ub3d9\uc548 \ud504\ub9b0\uc2a4\ud134 \ub300\ud559\uad50 \ub4f1 \ud559\uad50\uc5d0\uc11c \ud2b9\uac15, \ube0c\ub8e8\ud0b9\uc2a4 \uc5f0\uad6c\uc18c\uc5d0\uc11c \uc8fc\uad00\ud558\ub294 \uc138\ubbf8\ub098 \ucc38\uc5ec, \uc6cc\uc2f1\ud134 \ub0b4\uc758 \ub9ce\uc740 \uc5f0\uad6c\uc18c\uc640 \uae30\uad00 \ubc1c\ud45c, \uc138\ubbf8\ub098, \ud68c\uc758\uc5d0 \ucc38\uc11d\ud574 \ubbf8\uad6d\uc758 \uc0dd\uac01\uc744 \uc774\ud574\ud558\uace0 \ud55c\uad6d\uc758 \uc785\uc7a5\uc744 \ubbf8\uad6d \uc804\ubb38\uac00\ub4e4\uc5d0\uac8c \uc801\uadf9 \uc124\uba85\ud558\uc600\ub2e4\uace0 \ub9d0\ud588\ub2e4. 2017\ub144 1\uc6d4\uc5d0\ub294 \ud2b9\uc804\uc0ac \uc608\ube44\uc5ed\ub4e4\uc744 \uc911\uc2ec\uc73c\ub85c \uacb0\uc131\ub418\uc5b4 \uc804\uad6d\uc801\uc73c\ub85c 2\ub9cc 6\ucc9c\uc5ec \uba85\uc758 \ud68c\uc6d0\uc774 \ud65c\ub3d9\ud558\uace0 \uc788\ub294 \ube44\uc601\ub9ac \uc0ac\ub2e8\ubc95\uc778 \ubd09\uc0ac\ub2e8\uccb4\uc778 \ud2b9\uc804\ub3d9\uc9c0\ud68c\uc758 \uc81c2\ub300 \ucd1d\uc7ac \ub85c \ucde8\uc784\ud558\uc600\ub2e4. \uc804 \ucd1d\uc7ac\ub294 \ucde8\uc784\uc0ac\ub97c \ud1b5\ud574 \u201c\uc0c8\ub85c\uc6b4 \uc2dc\ub300 \ubcc0\ud654\uc5d0 \ubd80\uc751\ud558\ub294 \ub300\ud55c\ubbfc\uad6d \uce5c\ubaa9\ub2e8\uccb4\uc758 \uc0c8\ub85c\uc6b4 \u7ae0\uc744 \uc5f4\uaca0\ub2e4\u201d\uace0 \ub2e4\uc9d0\ud558\uba70, \u201c\ud604\uc5ed \ud6c4\ubc30\ub4e4\uc758 \uad8c\uc775\uc744 \uc704\ud574 \uc55e\uc7a5\uc11c\uace0, \ud68c\uc6d0\uc744 \ud3b8\uc560\ud558\uc9c0 \uc54a\ub294 \ubaa8\ub450\uc758 \ud2b9\uc804\ub3d9\uc9c0\ud68c\ub85c \ub9cc\ub4e4\uc5b4 \ub098\uac00\uaca0\ub2e4\u201d\uace0 \ud3ec\ubd80\ub97c \uc5ed\uc124\ud588\ub2e4. \uac19\uc740 \ub2ec \ubd81\ud55c\uc758 \uae40\uc815\uc740 \ub178\ub3d9\ub2f9 \uc704\uc6d0\uc7a5\uc5d0\uac8c \ub300\ub959\uac04\ud0c4\ub3c4\ubbf8\uc0ac\uc77c \uac1c\ubc1c \ub4f1\uc758 \uc7ac\uace0\ub97c \ucd09\uad6c\ud558\ub294 \ub0b4\uc6a9\uc73c\ub85c A4 \uc6a9\uc9c0 4\uc7a5 \ubd84\ub7c9\uc758 \uc601\ubb38\ud3b8\uc9c0\ub97c \ubcf4\ub0b4\uae30\ub3c4 \ud558\uc600\ub2e4. \u2018\uae40\uc815\uc740 \uc704\uc6d0\uc7a5\ub2d8\uaed8: \uc804 \ub300\ud55c\ubbfc\uad6d \uc721\uad70 \uc911\uc7a5\uc758 \uacf5\uac1c\ud3b8\uc9c0\u2019\ub77c\ub294 \uc81c\ubaa9\uc758 \uc774 \ud3b8\uc9c0\uc5d0\uc11c \uadf8\ub294 \u201c\ud3c9\ud654\ub97c \ucd94\uad6c\ud560 \uae30\ud68c\ub294 \uc5b8\uc81c\ub098 \uc5f4\ub824 \uc788\uc2b5\ub2c8\ub2e4\u201d\ub77c\uba74\uc11c \u201c\uac00\uc7a5 \ud544\uc218\uc801\uc778 \uccab \ub2e8\uacc4\ub294 \uacf5\uacf5\uc5f0\ud55c \uc704\ud611\uc758 \ud68c\ud53c, \ud575\uacfc \ubbf8\uc0ac\uc77c \uc2e4\ud5d8\uc758 \uc911\ub2e8, \ud575\ubb34\uae30 \uac1c\ubc1c\ubd80\ud130 \uc778\uad8c\uc5d0 \uc774\ub974\uae30\uae4c\uc9c0 \uc2e0\ub8b0\ub97c \uc99d\uc9c4\uc2dc\ud0ac \ud611\uc0c1\uc5d0 \ucc38\uc5ec\ud558\ub294 \uac83 \ub4f1\uc744 \ud3ec\ud568\ud569\ub2c8\ub2e4\u201d\ub77c\uace0 \ud3c9\ud654 \ud504\ub85c\uc138\uc2a4\ub97c \uc81c\uc548\ud588\ub2e4.", "paragraph_answer": "2016\ub144 10\uc6d4\ubd80\ud130 \ube0c\ub8e8\ud0b9\uc2a4 \uc5f0\uad6c\uc18c \ub3d9\uc544\uc2dc\uc544\uc815\ucc45\uc5f0\uad6c\uc13c\ud130 \ubc29\ubb38\uc5f0\uad6c\uc6d0, \uc874\uc2a4 \ud649\ud0a8\uc2a4 \ub300\ud559\uad50 \uad6d\uc81c\uad00\uacc4\ub300\ud559\uc6d0 \uac1d\uc6d0\uc5f0\uad6c\uc6d0\uc73c\ub85c 1\ub144 \ub3d9\uc548 \ud65c\ub3d9\ud588\ub2e4. \ubbf8\uad6d \uc5f0\uad6c\uae30\uad00\uc758 \uc5f0\uc218\ub294 \uc5f0\uc218\uc790\uac00 \uc8fc\uc81c\ub97c \uc120\uc815\ud574 \uadf8 \ubd84\uc57c\ub97c \uc5f0\uad6c\ud558\uba70 \uc790\uc2e0\uc758 \uc804\ubb38 \ubd84\uc57c\ub97c \ub2e4\ub978 \uc5f0\uad6c\uc790\ub4e4\uacfc \uacf5\uc720\ud558\ub294 \ud65c\ub3d9\uc744 \ud558\ub294\ub370, \uadf8\ub294 \uc804\uc2dc\uc791\uc804\ud1b5\uc81c\uad8c \ubb38\uc81c, \ud55c\uad6d\uc758 \ud575\ubb34\uc7a5, \ud55c\ubbf8 \uad70\uc0ac\ub3d9\ub9f9 \uac15\ud654 \ubc29\uc548 \ub4f1\uc744 \uc911\uc2ec\uc73c\ub85c \uc5f0\uad6c\ud558\uc600\ub2e4. \uc5f0\uc218 3\uac1c\uc6d4 \ub3d9\uc548 \ud504\ub9b0\uc2a4\ud134 \ub300\ud559\uad50 \ub4f1 \ud559\uad50\uc5d0\uc11c \ud2b9\uac15, \ube0c\ub8e8\ud0b9\uc2a4 \uc5f0\uad6c\uc18c\uc5d0\uc11c \uc8fc\uad00\ud558\ub294 \uc138\ubbf8\ub098 \ucc38\uc5ec, \uc6cc\uc2f1\ud134 \ub0b4\uc758 \ub9ce\uc740 \uc5f0\uad6c\uc18c\uc640 \uae30\uad00 \ubc1c\ud45c, \uc138\ubbf8\ub098, \ud68c\uc758\uc5d0 \ucc38\uc11d\ud574 \ubbf8\uad6d\uc758 \uc0dd\uac01\uc744 \uc774\ud574\ud558\uace0 \ud55c\uad6d\uc758 \uc785\uc7a5\uc744 \ubbf8\uad6d \uc804\ubb38\uac00\ub4e4\uc5d0\uac8c \uc801\uadf9 \uc124\uba85\ud558\uc600\ub2e4\uace0 \ub9d0\ud588\ub2e4. 2017\ub144 1\uc6d4\uc5d0\ub294 \ud2b9\uc804\uc0ac \uc608\ube44\uc5ed\ub4e4\uc744 \uc911\uc2ec\uc73c\ub85c \uacb0\uc131\ub418\uc5b4 \uc804\uad6d\uc801\uc73c\ub85c 2\ub9cc 6\ucc9c\uc5ec \uba85\uc758 \ud68c\uc6d0\uc774 \ud65c\ub3d9\ud558\uace0 \uc788\ub294 \ube44\uc601\ub9ac \uc0ac\ub2e8\ubc95\uc778 \ubd09\uc0ac\ub2e8\uccb4\uc778 \ud2b9\uc804\ub3d9\uc9c0\ud68c\uc758 \uc81c2\ub300 \ucd1d\uc7ac \ub85c \ucde8\uc784\ud558\uc600\ub2e4. \uc804 \ucd1d\uc7ac\ub294 \ucde8\uc784\uc0ac\ub97c \ud1b5\ud574 \u201c\uc0c8\ub85c\uc6b4 \uc2dc\ub300 \ubcc0\ud654\uc5d0 \ubd80\uc751\ud558\ub294 \ub300\ud55c\ubbfc\uad6d \uce5c\ubaa9\ub2e8\uccb4\uc758 \uc0c8\ub85c\uc6b4 \u7ae0\uc744 \uc5f4\uaca0\ub2e4\u201d\uace0 \ub2e4\uc9d0\ud558\uba70, \u201c\ud604\uc5ed \ud6c4\ubc30\ub4e4\uc758 \uad8c\uc775\uc744 \uc704\ud574 \uc55e\uc7a5\uc11c\uace0, \ud68c\uc6d0\uc744 \ud3b8\uc560\ud558\uc9c0 \uc54a\ub294 \ubaa8\ub450\uc758 \ud2b9\uc804\ub3d9\uc9c0\ud68c\ub85c \ub9cc\ub4e4\uc5b4 \ub098\uac00\uaca0\ub2e4\u201d\uace0 \ud3ec\ubd80\ub97c \uc5ed\uc124\ud588\ub2e4. \uac19\uc740 \ub2ec \ubd81\ud55c\uc758 \uae40\uc815\uc740 \ub178\ub3d9\ub2f9 \uc704\uc6d0\uc7a5\uc5d0\uac8c \ub300\ub959\uac04\ud0c4\ub3c4\ubbf8\uc0ac\uc77c \uac1c\ubc1c \ub4f1\uc758 \uc7ac\uace0\ub97c \ucd09\uad6c\ud558\ub294 \ub0b4\uc6a9\uc73c\ub85c A4 \uc6a9\uc9c0 4\uc7a5 \ubd84\ub7c9\uc758 \uc601\ubb38\ud3b8\uc9c0\ub97c \ubcf4\ub0b4\uae30\ub3c4 \ud558\uc600\ub2e4. \u2018\uae40\uc815\uc740 \uc704\uc6d0\uc7a5\ub2d8\uaed8: \uc804 \ub300\ud55c\ubbfc\uad6d \uc721\uad70 \uc911\uc7a5\uc758 \uacf5\uac1c\ud3b8\uc9c0\u2019\ub77c\ub294 \uc81c\ubaa9\uc758 \uc774 \ud3b8\uc9c0\uc5d0\uc11c \uadf8\ub294 \u201c\ud3c9\ud654\ub97c \ucd94\uad6c\ud560 \uae30\ud68c\ub294 \uc5b8\uc81c\ub098 \uc5f4\ub824 \uc788\uc2b5\ub2c8\ub2e4\u201d\ub77c\uba74\uc11c \u201c\uac00\uc7a5 \ud544\uc218\uc801\uc778 \uccab \ub2e8\uacc4\ub294 \uacf5\uacf5\uc5f0\ud55c \uc704\ud611\uc758 \ud68c\ud53c, \ud575\uacfc \ubbf8\uc0ac\uc77c \uc2e4\ud5d8\uc758 \uc911\ub2e8, \ud575\ubb34\uae30 \uac1c\ubc1c\ubd80\ud130 \uc778\uad8c\uc5d0 \uc774\ub974\uae30\uae4c\uc9c0 \uc2e0\ub8b0\ub97c \uc99d\uc9c4\uc2dc\ud0ac \ud611\uc0c1\uc5d0 \ucc38\uc5ec\ud558\ub294 \uac83 \ub4f1\uc744 \ud3ec\ud568\ud569\ub2c8\ub2e4\u201d\ub77c\uace0 \ud3c9\ud654 \ud504\ub85c\uc138\uc2a4\ub97c \uc81c\uc548\ud588\ub2e4.", "sentence_answer": "2017\ub144 1\uc6d4\uc5d0\ub294 \ud2b9\uc804\uc0ac \uc608\ube44\uc5ed\ub4e4\uc744 \uc911\uc2ec\uc73c\ub85c \uacb0\uc131\ub418\uc5b4 \uc804\uad6d\uc801\uc73c\ub85c 2\ub9cc 6\ucc9c\uc5ec \uba85\uc758 \ud68c\uc6d0\uc774 \ud65c\ub3d9\ud558\uace0 \uc788\ub294 \ube44\uc601\ub9ac \uc0ac\ub2e8\ubc95\uc778 \ubd09\uc0ac\ub2e8\uccb4\uc778 \ud2b9\uc804\ub3d9\uc9c0\ud68c\uc758 \uc81c2\ub300 \ucd1d\uc7ac \ub85c \ucde8\uc784\ud558\uc600\ub2e4.", "paragraph_id": "6593261-11-1", "paragraph_question": "question: \uc804\uc778\ubc94\uc774 \uc804\uc5ed \ud6c4 \ud2b9\uc804\ub3d9\uc9c0\ud68c\uc5d0\uc11c \ucde8\uc784\ud55c \uc9c1\uc704\ub294?, context: 2016\ub144 10\uc6d4\ubd80\ud130 \ube0c\ub8e8\ud0b9\uc2a4 \uc5f0\uad6c\uc18c \ub3d9\uc544\uc2dc\uc544\uc815\ucc45\uc5f0\uad6c\uc13c\ud130 \ubc29\ubb38\uc5f0\uad6c\uc6d0, \uc874\uc2a4 \ud649\ud0a8\uc2a4 \ub300\ud559\uad50 \uad6d\uc81c\uad00\uacc4\ub300\ud559\uc6d0 \uac1d\uc6d0\uc5f0\uad6c\uc6d0\uc73c\ub85c 1\ub144 \ub3d9\uc548 \ud65c\ub3d9\ud588\ub2e4. \ubbf8\uad6d \uc5f0\uad6c\uae30\uad00\uc758 \uc5f0\uc218\ub294 \uc5f0\uc218\uc790\uac00 \uc8fc\uc81c\ub97c \uc120\uc815\ud574 \uadf8 \ubd84\uc57c\ub97c \uc5f0\uad6c\ud558\uba70 \uc790\uc2e0\uc758 \uc804\ubb38 \ubd84\uc57c\ub97c \ub2e4\ub978 \uc5f0\uad6c\uc790\ub4e4\uacfc \uacf5\uc720\ud558\ub294 \ud65c\ub3d9\uc744 \ud558\ub294\ub370, \uadf8\ub294 \uc804\uc2dc\uc791\uc804\ud1b5\uc81c\uad8c \ubb38\uc81c, \ud55c\uad6d\uc758 \ud575\ubb34\uc7a5, \ud55c\ubbf8 \uad70\uc0ac\ub3d9\ub9f9 \uac15\ud654 \ubc29\uc548 \ub4f1\uc744 \uc911\uc2ec\uc73c\ub85c \uc5f0\uad6c\ud558\uc600\ub2e4. \uc5f0\uc218 3\uac1c\uc6d4 \ub3d9\uc548 \ud504\ub9b0\uc2a4\ud134 \ub300\ud559\uad50 \ub4f1 \ud559\uad50\uc5d0\uc11c \ud2b9\uac15, \ube0c\ub8e8\ud0b9\uc2a4 \uc5f0\uad6c\uc18c\uc5d0\uc11c \uc8fc\uad00\ud558\ub294 \uc138\ubbf8\ub098 \ucc38\uc5ec, \uc6cc\uc2f1\ud134 \ub0b4\uc758 \ub9ce\uc740 \uc5f0\uad6c\uc18c\uc640 \uae30\uad00 \ubc1c\ud45c, \uc138\ubbf8\ub098, \ud68c\uc758\uc5d0 \ucc38\uc11d\ud574 \ubbf8\uad6d\uc758 \uc0dd\uac01\uc744 \uc774\ud574\ud558\uace0 \ud55c\uad6d\uc758 \uc785\uc7a5\uc744 \ubbf8\uad6d \uc804\ubb38\uac00\ub4e4\uc5d0\uac8c \uc801\uadf9 \uc124\uba85\ud558\uc600\ub2e4\uace0 \ub9d0\ud588\ub2e4. 2017\ub144 1\uc6d4\uc5d0\ub294 \ud2b9\uc804\uc0ac \uc608\ube44\uc5ed\ub4e4\uc744 \uc911\uc2ec\uc73c\ub85c \uacb0\uc131\ub418\uc5b4 \uc804\uad6d\uc801\uc73c\ub85c 2\ub9cc 6\ucc9c\uc5ec \uba85\uc758 \ud68c\uc6d0\uc774 \ud65c\ub3d9\ud558\uace0 \uc788\ub294 \ube44\uc601\ub9ac \uc0ac\ub2e8\ubc95\uc778 \ubd09\uc0ac\ub2e8\uccb4\uc778 \ud2b9\uc804\ub3d9\uc9c0\ud68c\uc758 \uc81c2\ub300 \ucd1d\uc7ac\ub85c \ucde8\uc784\ud558\uc600\ub2e4. \uc804 \ucd1d\uc7ac\ub294 \ucde8\uc784\uc0ac\ub97c \ud1b5\ud574 \u201c\uc0c8\ub85c\uc6b4 \uc2dc\ub300 \ubcc0\ud654\uc5d0 \ubd80\uc751\ud558\ub294 \ub300\ud55c\ubbfc\uad6d \uce5c\ubaa9\ub2e8\uccb4\uc758 \uc0c8\ub85c\uc6b4 \u7ae0\uc744 \uc5f4\uaca0\ub2e4\u201d\uace0 \ub2e4\uc9d0\ud558\uba70, \u201c\ud604\uc5ed \ud6c4\ubc30\ub4e4\uc758 \uad8c\uc775\uc744 \uc704\ud574 \uc55e\uc7a5\uc11c\uace0, \ud68c\uc6d0\uc744 \ud3b8\uc560\ud558\uc9c0 \uc54a\ub294 \ubaa8\ub450\uc758 \ud2b9\uc804\ub3d9\uc9c0\ud68c\ub85c \ub9cc\ub4e4\uc5b4 \ub098\uac00\uaca0\ub2e4\u201d\uace0 \ud3ec\ubd80\ub97c \uc5ed\uc124\ud588\ub2e4. \uac19\uc740 \ub2ec \ubd81\ud55c\uc758 \uae40\uc815\uc740 \ub178\ub3d9\ub2f9 \uc704\uc6d0\uc7a5\uc5d0\uac8c \ub300\ub959\uac04\ud0c4\ub3c4\ubbf8\uc0ac\uc77c \uac1c\ubc1c \ub4f1\uc758 \uc7ac\uace0\ub97c \ucd09\uad6c\ud558\ub294 \ub0b4\uc6a9\uc73c\ub85c A4 \uc6a9\uc9c0 4\uc7a5 \ubd84\ub7c9\uc758 \uc601\ubb38\ud3b8\uc9c0\ub97c \ubcf4\ub0b4\uae30\ub3c4 \ud558\uc600\ub2e4. \u2018\uae40\uc815\uc740 \uc704\uc6d0\uc7a5\ub2d8\uaed8: \uc804 \ub300\ud55c\ubbfc\uad6d \uc721\uad70 \uc911\uc7a5\uc758 \uacf5\uac1c\ud3b8\uc9c0\u2019\ub77c\ub294 \uc81c\ubaa9\uc758 \uc774 \ud3b8\uc9c0\uc5d0\uc11c \uadf8\ub294 \u201c\ud3c9\ud654\ub97c \ucd94\uad6c\ud560 \uae30\ud68c\ub294 \uc5b8\uc81c\ub098 \uc5f4\ub824 \uc788\uc2b5\ub2c8\ub2e4\u201d\ub77c\uba74\uc11c \u201c\uac00\uc7a5 \ud544\uc218\uc801\uc778 \uccab \ub2e8\uacc4\ub294 \uacf5\uacf5\uc5f0\ud55c \uc704\ud611\uc758 \ud68c\ud53c, \ud575\uacfc \ubbf8\uc0ac\uc77c \uc2e4\ud5d8\uc758 \uc911\ub2e8, \ud575\ubb34\uae30 \uac1c\ubc1c\ubd80\ud130 \uc778\uad8c\uc5d0 \uc774\ub974\uae30\uae4c\uc9c0 \uc2e0\ub8b0\ub97c \uc99d\uc9c4\uc2dc\ud0ac \ud611\uc0c1\uc5d0 \ucc38\uc5ec\ud558\ub294 \uac83 \ub4f1\uc744 \ud3ec\ud568\ud569\ub2c8\ub2e4\u201d\ub77c\uace0 \ud3c9\ud654 \ud504\ub85c\uc138\uc2a4\ub97c \uc81c\uc548\ud588\ub2e4."} {"question": "\uc601\uad6d \ub3d9\uc778\ub3c4 \ud68c\uc0ac\uac00 \uc778\ub3c4 \ub0b4\uc758 \uc81c\ud6c4\uad6d\ub4e4\uacfc \uccb4\uacb0\ud55c \uac83\uc73c\ub85c \uc81c\ud6c4\uad6d\uc758 \uad70\ub300\ub97c \ub3d9\uc778\ub3c4 \ud68c\uc0ac\uac00 \ud1b5\uc194\ud560 \uc218 \uc788\uac8c \ud55c \uac83\uc740?", "paragraph": "19\uc138\uae30\uc5d0 \ub4e4\uc5b4 \ucd1d\ub3c5 \ub9ac\ucc98\ub4dc \uc6e8\uc2ac\ub9ac\ub294 \ub3d9\uc778\ub3c4 \ud68c\uc0ac\uc758 \uc9c0\ubc30 \uc9c0\uc5ed\uc744 \ud655\uc7a5\ud558\uae30 \uc704\ud55c \uc0c8\ub85c\uc6b4 \uc815\ucc45\uc744 \uc2dc\ud589\ud558\uc600\ub2e4. \uc774 \uc815\ucc45\uc5d0 \ub530\ub77c \uc601\uad6d \ub3d9\uc778\ub3c4 \ud68c\uc0ac\ub294 \uc778\ub3c4 \ub0b4\uc758 \uc81c\ud6c4\uad6d\ub4e4\uacfc \uc885\uc18d \ub3d9\ub9f9\uc744 \uacb0\uc131\ud558\uc600\ub2e4. \uc885\uc18d \ub3d9\ub9f9\uc740 \uc81c\ud6c4\uad6d\uc774\ub098 \uc9c0\uc5ed \uad70\ubc8c\uc758 \ud718\ud558\uc5d0 \uc788\ub358 \uac01 \uc9c0\uc5ed\uc758 \uad70\uc0ac\ub97c \ub3d9\uc778\ub3c4 \ud68c\uc0ac\uac00 \uc9c1\uc811 \ud1b5\uc194\ud558\ub3c4\ub85d \ud558\ub294 \uac83\uc73c\ub85c, \uc81c2\ucc28 \uc601\uad6d-\uc2dc\ud06c \uc804\uc7c1 \uc774\ud6c4 \uce74\uc774\ubca0\ub974\ud30c\ud06c\ud230\ud06c\uc640 \uc8fc\uc640 \uce74\uc288\ubbf8\ub974, \ud380\uc790\ube0c \uc9c0\uc5ed\uc5d0 \uc874\uc7ac\ud558\ub358 \uc81c\ud6c4\uad6d\ub4e4\uc744 \ub300\ud56d\uc73c\ub85c \ud558\uc600\ub294\ub370 \uc774 \uc9c0\uc5ed\uc758 \uc218\uc7a5\uc774\uc5c8\ub358 \ud78c\ub450\uad50 \uc9c0\uc5ed\uc758 \ub9c8\ud558\ub77c\uc790\ub098 \uc774\uc2ac\ub78c\uad50 \uc9c0\uc5ed\uc758 \ub098\uc640\ube0c \ub4f1 \uc81c\ud6c4\ub4e4\uc744 \uc885\uc18d \ub3d9\ub9f9 \uc548\uc73c\ub85c \ud3b8\uc81c\ud558\uc600\ub2e4. \ud2b9\ud788 \uce74\uc2dc\ubbf8\ub974 \uc9c0\uc5ed\uc740 \uc2dc\ud06c \uc804\uc7c1 \uc9c1\ud6c4\uc778 1850\ub144 \uc554\ub2c8\ucc28\ub974 \uc870\uc57d\uc744 \ud1b5\ud574 \uc7a0\ubb34 \uc9c0\ubc29\uc758 \ub3c4\uadf8\ub9ac \uc655\uc870\uc5d0\uac8c \ud560\uc591\ub418\uc5b4 \uc81c\ud6c4\uad6d\uc774 \ub418\uc5c8\ub2e4. \ud55c\ud3b8 1814\ub144\uc5d0\uc11c 1816\ub144\uae4c\uc9c0 \uc601\uad6d\ub839 \uc778\ub3c4\uc640 \ub124\ud314 \uc0ac\uc774\uc758 \uad6d\uacbd \ubd84\uc7c1\uc73c\ub85c \uc778\ud574 \uc77c\uc5b4\ub09c \uc601\uad6d-\ub124\ud314 \uc804\uc7c1\uc758 \uacb0\uacfc \uad6c\ub974\uce74\uc2a4 \uc9c0\uc5ed\uc774 \uc601\uad6d\uc758 \uc9c0\ubc30\uc5d0 \ub193\uc774\uac8c \ub418\uc5c8\ub2e4. \ub610\ud55c 1854\ub144\uc5d0\ub294 \uc9c0\uae08\uc758 \ub9c8\ud558\ub77c\uc288\ud2b8\ub77c \uc8fc\uc5d0 \uc18d\ud558\ub294 \ubca0\ub77c\ub974 \uc9c0\ubc29\uc774 \uc885\uc18d \ub3d9\ub9f9\uc5d0 \ud3ec\ud568\ub418\uc5c8\uace0, 2\ub144 \ub4a4\uc5d0\ub294 \uc544\uc640\ub4dc\uac00 \uc885\uc18d \ub3d9\ub9f9\uc5d0 \ud3b8\uc785\ub418\uc5c8\ub2e4.", "answer": "\uc885\uc18d \ub3d9\ub9f9", "sentence": "\uc774 \uc815\ucc45\uc5d0 \ub530\ub77c \uc601\uad6d \ub3d9\uc778\ub3c4 \ud68c\uc0ac\ub294 \uc778\ub3c4 \ub0b4\uc758 \uc81c\ud6c4\uad6d\ub4e4\uacfc \uc885\uc18d \ub3d9\ub9f9 \uc744", "paragraph_sentence": "19\uc138\uae30\uc5d0 \ub4e4\uc5b4 \ucd1d\ub3c5 \ub9ac\ucc98\ub4dc \uc6e8\uc2ac\ub9ac\ub294 \ub3d9\uc778\ub3c4 \ud68c\uc0ac\uc758 \uc9c0\ubc30 \uc9c0\uc5ed\uc744 \ud655\uc7a5\ud558\uae30 \uc704\ud55c \uc0c8\ub85c\uc6b4 \uc815\ucc45\uc744 \uc2dc\ud589\ud558\uc600\ub2e4. \uc774 \uc815\ucc45\uc5d0 \ub530\ub77c \uc601\uad6d \ub3d9\uc778\ub3c4 \ud68c\uc0ac\ub294 \uc778\ub3c4 \ub0b4\uc758 \uc81c\ud6c4\uad6d\ub4e4\uacfc \uc885\uc18d \ub3d9\ub9f9 \uc744 \uacb0\uc131\ud558\uc600\ub2e4. \uc885\uc18d \ub3d9\ub9f9\uc740 \uc81c\ud6c4\uad6d\uc774\ub098 \uc9c0\uc5ed \uad70\ubc8c\uc758 \ud718\ud558\uc5d0 \uc788\ub358 \uac01 \uc9c0\uc5ed\uc758 \uad70\uc0ac\ub97c \ub3d9\uc778\ub3c4 \ud68c\uc0ac\uac00 \uc9c1\uc811 \ud1b5\uc194\ud558\ub3c4\ub85d \ud558\ub294 \uac83\uc73c\ub85c, \uc81c2\ucc28 \uc601\uad6d-\uc2dc\ud06c \uc804\uc7c1 \uc774\ud6c4 \uce74\uc774\ubca0\ub974\ud30c\ud06c\ud230\ud06c\uc640 \uc8fc\uc640 \uce74\uc288\ubbf8\ub974, \ud380\uc790\ube0c \uc9c0\uc5ed\uc5d0 \uc874\uc7ac\ud558\ub358 \uc81c\ud6c4\uad6d\ub4e4\uc744 \ub300\ud56d\uc73c\ub85c \ud558\uc600\ub294\ub370 \uc774 \uc9c0\uc5ed\uc758 \uc218\uc7a5\uc774\uc5c8\ub358 \ud78c\ub450\uad50 \uc9c0\uc5ed\uc758 \ub9c8\ud558\ub77c\uc790\ub098 \uc774\uc2ac\ub78c\uad50 \uc9c0\uc5ed\uc758 \ub098\uc640\ube0c \ub4f1 \uc81c\ud6c4\ub4e4\uc744 \uc885\uc18d \ub3d9\ub9f9 \uc548\uc73c\ub85c \ud3b8\uc81c\ud558\uc600\ub2e4. \ud2b9\ud788 \uce74\uc2dc\ubbf8\ub974 \uc9c0\uc5ed\uc740 \uc2dc\ud06c \uc804\uc7c1 \uc9c1\ud6c4\uc778 1850\ub144 \uc554\ub2c8\ucc28\ub974 \uc870\uc57d\uc744 \ud1b5\ud574 \uc7a0\ubb34 \uc9c0\ubc29\uc758 \ub3c4\uadf8\ub9ac \uc655\uc870\uc5d0\uac8c \ud560\uc591\ub418\uc5b4 \uc81c\ud6c4\uad6d\uc774 \ub418\uc5c8\ub2e4. \ud55c\ud3b8 1814\ub144\uc5d0\uc11c 1816\ub144\uae4c\uc9c0 \uc601\uad6d\ub839 \uc778\ub3c4\uc640 \ub124\ud314 \uc0ac\uc774\uc758 \uad6d\uacbd \ubd84\uc7c1\uc73c\ub85c \uc778\ud574 \uc77c\uc5b4\ub09c \uc601\uad6d-\ub124\ud314 \uc804\uc7c1\uc758 \uacb0\uacfc \uad6c\ub974\uce74\uc2a4 \uc9c0\uc5ed\uc774 \uc601\uad6d\uc758 \uc9c0\ubc30\uc5d0 \ub193\uc774\uac8c \ub418\uc5c8\ub2e4. \ub610\ud55c 1854\ub144\uc5d0\ub294 \uc9c0\uae08\uc758 \ub9c8\ud558\ub77c\uc288\ud2b8\ub77c \uc8fc\uc5d0 \uc18d\ud558\ub294 \ubca0\ub77c\ub974 \uc9c0\ubc29\uc774 \uc885\uc18d \ub3d9\ub9f9\uc5d0 \ud3ec\ud568\ub418\uc5c8\uace0, 2\ub144 \ub4a4\uc5d0\ub294 \uc544\uc640\ub4dc\uac00 \uc885\uc18d \ub3d9\ub9f9\uc5d0 \ud3b8\uc785\ub418\uc5c8\ub2e4.", "paragraph_answer": "19\uc138\uae30\uc5d0 \ub4e4\uc5b4 \ucd1d\ub3c5 \ub9ac\ucc98\ub4dc \uc6e8\uc2ac\ub9ac\ub294 \ub3d9\uc778\ub3c4 \ud68c\uc0ac\uc758 \uc9c0\ubc30 \uc9c0\uc5ed\uc744 \ud655\uc7a5\ud558\uae30 \uc704\ud55c \uc0c8\ub85c\uc6b4 \uc815\ucc45\uc744 \uc2dc\ud589\ud558\uc600\ub2e4. \uc774 \uc815\ucc45\uc5d0 \ub530\ub77c \uc601\uad6d \ub3d9\uc778\ub3c4 \ud68c\uc0ac\ub294 \uc778\ub3c4 \ub0b4\uc758 \uc81c\ud6c4\uad6d\ub4e4\uacfc \uc885\uc18d \ub3d9\ub9f9 \uc744 \uacb0\uc131\ud558\uc600\ub2e4. \uc885\uc18d \ub3d9\ub9f9\uc740 \uc81c\ud6c4\uad6d\uc774\ub098 \uc9c0\uc5ed \uad70\ubc8c\uc758 \ud718\ud558\uc5d0 \uc788\ub358 \uac01 \uc9c0\uc5ed\uc758 \uad70\uc0ac\ub97c \ub3d9\uc778\ub3c4 \ud68c\uc0ac\uac00 \uc9c1\uc811 \ud1b5\uc194\ud558\ub3c4\ub85d \ud558\ub294 \uac83\uc73c\ub85c, \uc81c2\ucc28 \uc601\uad6d-\uc2dc\ud06c \uc804\uc7c1 \uc774\ud6c4 \uce74\uc774\ubca0\ub974\ud30c\ud06c\ud230\ud06c\uc640 \uc8fc\uc640 \uce74\uc288\ubbf8\ub974, \ud380\uc790\ube0c \uc9c0\uc5ed\uc5d0 \uc874\uc7ac\ud558\ub358 \uc81c\ud6c4\uad6d\ub4e4\uc744 \ub300\ud56d\uc73c\ub85c \ud558\uc600\ub294\ub370 \uc774 \uc9c0\uc5ed\uc758 \uc218\uc7a5\uc774\uc5c8\ub358 \ud78c\ub450\uad50 \uc9c0\uc5ed\uc758 \ub9c8\ud558\ub77c\uc790\ub098 \uc774\uc2ac\ub78c\uad50 \uc9c0\uc5ed\uc758 \ub098\uc640\ube0c \ub4f1 \uc81c\ud6c4\ub4e4\uc744 \uc885\uc18d \ub3d9\ub9f9 \uc548\uc73c\ub85c \ud3b8\uc81c\ud558\uc600\ub2e4. \ud2b9\ud788 \uce74\uc2dc\ubbf8\ub974 \uc9c0\uc5ed\uc740 \uc2dc\ud06c \uc804\uc7c1 \uc9c1\ud6c4\uc778 1850\ub144 \uc554\ub2c8\ucc28\ub974 \uc870\uc57d\uc744 \ud1b5\ud574 \uc7a0\ubb34 \uc9c0\ubc29\uc758 \ub3c4\uadf8\ub9ac \uc655\uc870\uc5d0\uac8c \ud560\uc591\ub418\uc5b4 \uc81c\ud6c4\uad6d\uc774 \ub418\uc5c8\ub2e4. \ud55c\ud3b8 1814\ub144\uc5d0\uc11c 1816\ub144\uae4c\uc9c0 \uc601\uad6d\ub839 \uc778\ub3c4\uc640 \ub124\ud314 \uc0ac\uc774\uc758 \uad6d\uacbd \ubd84\uc7c1\uc73c\ub85c \uc778\ud574 \uc77c\uc5b4\ub09c \uc601\uad6d-\ub124\ud314 \uc804\uc7c1\uc758 \uacb0\uacfc \uad6c\ub974\uce74\uc2a4 \uc9c0\uc5ed\uc774 \uc601\uad6d\uc758 \uc9c0\ubc30\uc5d0 \ub193\uc774\uac8c \ub418\uc5c8\ub2e4. \ub610\ud55c 1854\ub144\uc5d0\ub294 \uc9c0\uae08\uc758 \ub9c8\ud558\ub77c\uc288\ud2b8\ub77c \uc8fc\uc5d0 \uc18d\ud558\ub294 \ubca0\ub77c\ub974 \uc9c0\ubc29\uc774 \uc885\uc18d \ub3d9\ub9f9\uc5d0 \ud3ec\ud568\ub418\uc5c8\uace0, 2\ub144 \ub4a4\uc5d0\ub294 \uc544\uc640\ub4dc\uac00 \uc885\uc18d \ub3d9\ub9f9\uc5d0 \ud3b8\uc785\ub418\uc5c8\ub2e4.", "sentence_answer": "\uc774 \uc815\ucc45\uc5d0 \ub530\ub77c \uc601\uad6d \ub3d9\uc778\ub3c4 \ud68c\uc0ac\ub294 \uc778\ub3c4 \ub0b4\uc758 \uc81c\ud6c4\uad6d\ub4e4\uacfc \uc885\uc18d \ub3d9\ub9f9 \uc744", "paragraph_id": "6444921-0-0", "paragraph_question": "question: \uc601\uad6d \ub3d9\uc778\ub3c4 \ud68c\uc0ac\uac00 \uc778\ub3c4 \ub0b4\uc758 \uc81c\ud6c4\uad6d\ub4e4\uacfc \uccb4\uacb0\ud55c \uac83\uc73c\ub85c \uc81c\ud6c4\uad6d\uc758 \uad70\ub300\ub97c \ub3d9\uc778\ub3c4 \ud68c\uc0ac\uac00 \ud1b5\uc194\ud560 \uc218 \uc788\uac8c \ud55c \uac83\uc740?, context: 19\uc138\uae30\uc5d0 \ub4e4\uc5b4 \ucd1d\ub3c5 \ub9ac\ucc98\ub4dc \uc6e8\uc2ac\ub9ac\ub294 \ub3d9\uc778\ub3c4 \ud68c\uc0ac\uc758 \uc9c0\ubc30 \uc9c0\uc5ed\uc744 \ud655\uc7a5\ud558\uae30 \uc704\ud55c \uc0c8\ub85c\uc6b4 \uc815\ucc45\uc744 \uc2dc\ud589\ud558\uc600\ub2e4. \uc774 \uc815\ucc45\uc5d0 \ub530\ub77c \uc601\uad6d \ub3d9\uc778\ub3c4 \ud68c\uc0ac\ub294 \uc778\ub3c4 \ub0b4\uc758 \uc81c\ud6c4\uad6d\ub4e4\uacfc \uc885\uc18d \ub3d9\ub9f9\uc744 \uacb0\uc131\ud558\uc600\ub2e4. \uc885\uc18d \ub3d9\ub9f9\uc740 \uc81c\ud6c4\uad6d\uc774\ub098 \uc9c0\uc5ed \uad70\ubc8c\uc758 \ud718\ud558\uc5d0 \uc788\ub358 \uac01 \uc9c0\uc5ed\uc758 \uad70\uc0ac\ub97c \ub3d9\uc778\ub3c4 \ud68c\uc0ac\uac00 \uc9c1\uc811 \ud1b5\uc194\ud558\ub3c4\ub85d \ud558\ub294 \uac83\uc73c\ub85c, \uc81c2\ucc28 \uc601\uad6d-\uc2dc\ud06c \uc804\uc7c1 \uc774\ud6c4 \uce74\uc774\ubca0\ub974\ud30c\ud06c\ud230\ud06c\uc640 \uc8fc\uc640 \uce74\uc288\ubbf8\ub974, \ud380\uc790\ube0c \uc9c0\uc5ed\uc5d0 \uc874\uc7ac\ud558\ub358 \uc81c\ud6c4\uad6d\ub4e4\uc744 \ub300\ud56d\uc73c\ub85c \ud558\uc600\ub294\ub370 \uc774 \uc9c0\uc5ed\uc758 \uc218\uc7a5\uc774\uc5c8\ub358 \ud78c\ub450\uad50 \uc9c0\uc5ed\uc758 \ub9c8\ud558\ub77c\uc790\ub098 \uc774\uc2ac\ub78c\uad50 \uc9c0\uc5ed\uc758 \ub098\uc640\ube0c \ub4f1 \uc81c\ud6c4\ub4e4\uc744 \uc885\uc18d \ub3d9\ub9f9 \uc548\uc73c\ub85c \ud3b8\uc81c\ud558\uc600\ub2e4. \ud2b9\ud788 \uce74\uc2dc\ubbf8\ub974 \uc9c0\uc5ed\uc740 \uc2dc\ud06c \uc804\uc7c1 \uc9c1\ud6c4\uc778 1850\ub144 \uc554\ub2c8\ucc28\ub974 \uc870\uc57d\uc744 \ud1b5\ud574 \uc7a0\ubb34 \uc9c0\ubc29\uc758 \ub3c4\uadf8\ub9ac \uc655\uc870\uc5d0\uac8c \ud560\uc591\ub418\uc5b4 \uc81c\ud6c4\uad6d\uc774 \ub418\uc5c8\ub2e4. \ud55c\ud3b8 1814\ub144\uc5d0\uc11c 1816\ub144\uae4c\uc9c0 \uc601\uad6d\ub839 \uc778\ub3c4\uc640 \ub124\ud314 \uc0ac\uc774\uc758 \uad6d\uacbd \ubd84\uc7c1\uc73c\ub85c \uc778\ud574 \uc77c\uc5b4\ub09c \uc601\uad6d-\ub124\ud314 \uc804\uc7c1\uc758 \uacb0\uacfc \uad6c\ub974\uce74\uc2a4 \uc9c0\uc5ed\uc774 \uc601\uad6d\uc758 \uc9c0\ubc30\uc5d0 \ub193\uc774\uac8c \ub418\uc5c8\ub2e4. \ub610\ud55c 1854\ub144\uc5d0\ub294 \uc9c0\uae08\uc758 \ub9c8\ud558\ub77c\uc288\ud2b8\ub77c \uc8fc\uc5d0 \uc18d\ud558\ub294 \ubca0\ub77c\ub974 \uc9c0\ubc29\uc774 \uc885\uc18d \ub3d9\ub9f9\uc5d0 \ud3ec\ud568\ub418\uc5c8\uace0, 2\ub144 \ub4a4\uc5d0\ub294 \uc544\uc640\ub4dc\uac00 \uc885\uc18d \ub3d9\ub9f9\uc5d0 \ud3b8\uc785\ub418\uc5c8\ub2e4."} {"question": "\ubd09\ud558\ub9c8\uc744 \ub178\ubb34\ud604 \uc804 \ub300\ud1b5\ub839 \ubb18\uc18c\uc5d0 \uc778\ubd84\uc774 \ud22c\ucc99\ub41c \uc0ac\uac74\uc774 \uc77c\uc5b4\ub09c \ub0a0\uc9dc\ub294?", "paragraph": "2010\ub144 11\uc6d4 14\uc77c \uc624\ud6c4 1\uc2dc\uacbd \ubd09\ud558\ub9c8\uc744 \ub178\ubb34\ud604 \ubb18\uc18c\uc5d0 \uc778\ubd84\uc774 \ud22c\ucc99\ub418\ub294 \uc0ac\uac74\uc774 \ubc1c\uc0dd\ud588\ub2e4. 62\uc138\uc758 \uc815\ubaa8\uc528\ub294 \ub178\ubb34\ud604 \ubb18\uc18c\uc5d0 \uc778\ubd84\uc744 \ud22c\ucc99\ud558\uace0 \uc720\uc778\ubb3c 22\uc7a5\uc744 \uc0b4\ud3ec\ud558\uc600\ub294\ub370 \uadf8\uac00 \ubfcc\ub9b0 \uc720\uc778\ubb3c\uc740 \ub178\ubb34\ud604 \uadf8\ub300 \ubb34\ub364\uc5d0 \ub625\ubb3c\uc744 \ubd80\uc73c\uba70\"\ub77c\ub294 \uc81c\ubaa9\uc73c\ub85c, \"\uc804\uad50\uc870\u00b7\uc804\uacf5\ub178\u00b7\ubbfc\uc8fc\ub178\ucd1d \uac19\uc740 \uc88c\ud30c\uc138\ub825\ub4e4\uc774 \uc0dd\uc131\ub418\ub3c4\ub85d \ub3c4\uc640 \uccad\uc18c\ub144\ub4e4\uc758 \uc815\uc2e0\uc744 \uc138\ub1cc\uc2dc\ud0a4\uace0, \uad6d\uac00 \uc815\uccb4\uc131\uc744 \ud63c\ub3c8\uc5d0 \ube60\ub728\ub838\uc73c\uba70, \uad6d\ubbfc\uc744 \ubd88\uc548\ud558\uac8c \ud588\ub2e4\"\ub294 \ub0b4\uc6a9\uc774\uc5c8\ub2e4. \uc815\ubaa8\uc528\ub294 \uc778\ubd84 \ud22c\ucc99\ud6c4 \ud604\uc7a5\uc5d0\uc11c \ubc14\ub85c \uacbd\ucc30\uc5d0 \ubd99\uc7a1\ud614\uc73c\uba70 \uacbd\ucc30 \uc9c4\uc220\uc5d0\uc11c \"\uace0 \ub178\ubb34\ud604 \uc804 \ub300\ud1b5\ub839\uc774 \uc7ac\uc9c1 \uc911 \uc88c\uc775\uc138\ub825\uc774 \ud310\uc744 \uce58\ub294 \ub370 \ub300\ud574 \ubd88\ub9cc\uc744 \ud488\uace0 \ubc94\ud589\uc744 \uacc4\ud68d\ud588\ub2e4\" \uace0 \ubc1d\ud614\ub2e4. \ud55c\ud3b8 \uacbd\ucc30\uc740 \uc815\ubaa8\uc528\uc758 \ub2e8\ub3c5\ubc94\ud589\uc73c\ub85c \uacb0\ub860 \uc9d3\uace0 \uc7ac\ubb3c\uc190\uad34 \ub4f1\uc758 \ud610\uc758\ub85c \uc785\uac74\ud558\uace0, \uc778\ubd84\uc774 \ud22c\ucc99\ub41c \ubd09\ubd84\uc744 \uc138\ucc99 \uc870\uce58\ud588\ub2e4. \uc774\uc5d0 \ub300\ud574 \ubbfc\uc8fc\ub2f9 \ucc28\uc601 \ub300\ubcc0\uc778\uc740 \"\ubbff\uae30\uc9c0 \uc54a\ub294 \uc774\ubc88 \uc0ac\uac74\uc5d0 \ub300\ud574 \ubbfc\uc8fc\ub2f9\uc740 \uad6d\ubbfc\uacfc \ud568\uaed8 \ubd84\ub178\ud558\uba70 \uc720\uac10\uc758 \ub73b\uc744 \ubc1d\ud78c\ub2e4. \uc544\uc6b8\ub7ec \uacbd\ucc30\uc740 \ucca0\uc800\ud55c \uc218\uc0ac\ub97c \ud1b5\ud574 \uc0ac\uac74\uc758 \uc804\ub9d0\uc744 \ubc1d\ud600\uc57c \ud558\uace0, \ub9cc\uc57d \ubc30\ud6c4\uac00 \uc788\ub2e4\uba74 \ucca0\uc800\ud788 \uac00\ub824\ub0b4 \uc5c4\ubc8c\ud574\uc57c \ud560 \uac83\"\uc774\ub77c\uace0 \ubc1d\ud614\uc73c\uba70, \uc18c\uc124\uac00 \uc774\uc678\uc218\ub294 \uc790\uc2e0\uc758 \ud2b8\uc704\ud130\uc5d0\uc11c \"\uc774\ub7f0 \uc0ac\ub78c\ub4e4\uc77c\uc218\ub85d \uad6d\uaca9 \uc790\uc8fc \ub4e4\uba39\uac70\ub9ac\uba74\uc11c \uc560\uad6d\uc790 \ud589\uc138\ub97c \ud55c\ub2e4. \ub2e8\uc138\ud3ec\uc801\uc778 \uad6c\ud1a0\uc720\ubc1c\uc790\ub4e4\"\uc774\ub77c\uace0 \ub9f9\ube44\ub09c\ud588\ub2e4. \ud55c\ub098\ub77c\ub2f9 \uc548\uc0c1\uc218 \ub300\ud45c\ub3c4 \"\uc774\ubc88 \ubd88\uc0c1\uc0ac\ub294 \uae4a\uc774 \uac1c\ud0c4\ud560 \uc218 \ubc16\uc5d0 \uc5c6\ub294 \uc0c1\ud669\"\uc774\ub77c\uba70 \"\ubc95\uc5d0 \ub530\ub77c \uc5c4\uc815\ucc98\ub9ac\ud574\uc57c \ud55c\ub2e4\"\uace0 \uc8fc\uc7a5\ud588\ub2e4.", "answer": "2010\ub144 11\uc6d4 14\uc77c", "sentence": "2010\ub144 11\uc6d4 14\uc77c \uc624\ud6c4 1\uc2dc\uacbd \ubd09\ud558\ub9c8\uc744 \ub178\ubb34\ud604 \ubb18\uc18c\uc5d0 \uc778\ubd84\uc774 \ud22c\ucc99\ub418\ub294 \uc0ac\uac74\uc774 \ubc1c\uc0dd\ud588\ub2e4.", "paragraph_sentence": " 2010\ub144 11\uc6d4 14\uc77c \uc624\ud6c4 1\uc2dc\uacbd \ubd09\ud558\ub9c8\uc744 \ub178\ubb34\ud604 \ubb18\uc18c\uc5d0 \uc778\ubd84\uc774 \ud22c\ucc99\ub418\ub294 \uc0ac\uac74\uc774 \ubc1c\uc0dd\ud588\ub2e4. 62\uc138\uc758 \uc815\ubaa8\uc528\ub294 \ub178\ubb34\ud604 \ubb18\uc18c\uc5d0 \uc778\ubd84\uc744 \ud22c\ucc99\ud558\uace0 \uc720\uc778\ubb3c 22\uc7a5\uc744 \uc0b4\ud3ec\ud558\uc600\ub294\ub370 \uadf8\uac00 \ubfcc\ub9b0 \uc720\uc778\ubb3c\uc740 \ub178\ubb34\ud604 \uadf8\ub300 \ubb34\ub364\uc5d0 \ub625\ubb3c\uc744 \ubd80\uc73c\uba70\"\ub77c\ub294 \uc81c\ubaa9\uc73c\ub85c, \"\uc804\uad50\uc870\u00b7\uc804\uacf5\ub178\u00b7\ubbfc\uc8fc\ub178\ucd1d \uac19\uc740 \uc88c\ud30c\uc138\ub825\ub4e4\uc774 \uc0dd\uc131\ub418\ub3c4\ub85d \ub3c4\uc640 \uccad\uc18c\ub144\ub4e4\uc758 \uc815\uc2e0\uc744 \uc138\ub1cc\uc2dc\ud0a4\uace0, \uad6d\uac00 \uc815\uccb4\uc131\uc744 \ud63c\ub3c8\uc5d0 \ube60\ub728\ub838\uc73c\uba70, \uad6d\ubbfc\uc744 \ubd88\uc548\ud558\uac8c \ud588\ub2e4\"\ub294 \ub0b4\uc6a9\uc774\uc5c8\ub2e4. \uc815\ubaa8\uc528\ub294 \uc778\ubd84 \ud22c\ucc99\ud6c4 \ud604\uc7a5\uc5d0\uc11c \ubc14\ub85c \uacbd\ucc30\uc5d0 \ubd99\uc7a1\ud614\uc73c\uba70 \uacbd\ucc30 \uc9c4\uc220\uc5d0\uc11c \"\uace0 \ub178\ubb34\ud604 \uc804 \ub300\ud1b5\ub839\uc774 \uc7ac\uc9c1 \uc911 \uc88c\uc775\uc138\ub825\uc774 \ud310\uc744 \uce58\ub294 \ub370 \ub300\ud574 \ubd88\ub9cc\uc744 \ud488\uace0 \ubc94\ud589\uc744 \uacc4\ud68d\ud588\ub2e4\" \uace0 \ubc1d\ud614\ub2e4. \ud55c\ud3b8 \uacbd\ucc30\uc740 \uc815\ubaa8\uc528\uc758 \ub2e8\ub3c5\ubc94\ud589\uc73c\ub85c \uacb0\ub860 \uc9d3\uace0 \uc7ac\ubb3c\uc190\uad34 \ub4f1\uc758 \ud610\uc758\ub85c \uc785\uac74\ud558\uace0, \uc778\ubd84\uc774 \ud22c\ucc99\ub41c \ubd09\ubd84\uc744 \uc138\ucc99 \uc870\uce58\ud588\ub2e4. \uc774\uc5d0 \ub300\ud574 \ubbfc\uc8fc\ub2f9 \ucc28\uc601 \ub300\ubcc0\uc778\uc740 \"\ubbff\uae30\uc9c0 \uc54a\ub294 \uc774\ubc88 \uc0ac\uac74\uc5d0 \ub300\ud574 \ubbfc\uc8fc\ub2f9\uc740 \uad6d\ubbfc\uacfc \ud568\uaed8 \ubd84\ub178\ud558\uba70 \uc720\uac10\uc758 \ub73b\uc744 \ubc1d\ud78c\ub2e4. \uc544\uc6b8\ub7ec \uacbd\ucc30\uc740 \ucca0\uc800\ud55c \uc218\uc0ac\ub97c \ud1b5\ud574 \uc0ac\uac74\uc758 \uc804\ub9d0\uc744 \ubc1d\ud600\uc57c \ud558\uace0, \ub9cc\uc57d \ubc30\ud6c4\uac00 \uc788\ub2e4\uba74 \ucca0\uc800\ud788 \uac00\ub824\ub0b4 \uc5c4\ubc8c\ud574\uc57c \ud560 \uac83\"\uc774\ub77c\uace0 \ubc1d\ud614\uc73c\uba70, \uc18c\uc124\uac00 \uc774\uc678\uc218\ub294 \uc790\uc2e0\uc758 \ud2b8\uc704\ud130\uc5d0\uc11c \"\uc774\ub7f0 \uc0ac\ub78c\ub4e4\uc77c\uc218\ub85d \uad6d\uaca9 \uc790\uc8fc \ub4e4\uba39\uac70\ub9ac\uba74\uc11c \uc560\uad6d\uc790 \ud589\uc138\ub97c \ud55c\ub2e4. \ub2e8\uc138\ud3ec\uc801\uc778 \uad6c\ud1a0\uc720\ubc1c\uc790\ub4e4\"\uc774\ub77c\uace0 \ub9f9\ube44\ub09c\ud588\ub2e4. \ud55c\ub098\ub77c\ub2f9 \uc548\uc0c1\uc218 \ub300\ud45c\ub3c4 \"\uc774\ubc88 \ubd88\uc0c1\uc0ac\ub294 \uae4a\uc774 \uac1c\ud0c4\ud560 \uc218 \ubc16\uc5d0 \uc5c6\ub294 \uc0c1\ud669\"\uc774\ub77c\uba70 \"\ubc95\uc5d0 \ub530\ub77c \uc5c4\uc815\ucc98\ub9ac\ud574\uc57c \ud55c\ub2e4\"\uace0 \uc8fc\uc7a5\ud588\ub2e4.", "paragraph_answer": " 2010\ub144 11\uc6d4 14\uc77c \uc624\ud6c4 1\uc2dc\uacbd \ubd09\ud558\ub9c8\uc744 \ub178\ubb34\ud604 \ubb18\uc18c\uc5d0 \uc778\ubd84\uc774 \ud22c\ucc99\ub418\ub294 \uc0ac\uac74\uc774 \ubc1c\uc0dd\ud588\ub2e4. 62\uc138\uc758 \uc815\ubaa8\uc528\ub294 \ub178\ubb34\ud604 \ubb18\uc18c\uc5d0 \uc778\ubd84\uc744 \ud22c\ucc99\ud558\uace0 \uc720\uc778\ubb3c 22\uc7a5\uc744 \uc0b4\ud3ec\ud558\uc600\ub294\ub370 \uadf8\uac00 \ubfcc\ub9b0 \uc720\uc778\ubb3c\uc740 \ub178\ubb34\ud604 \uadf8\ub300 \ubb34\ub364\uc5d0 \ub625\ubb3c\uc744 \ubd80\uc73c\uba70\"\ub77c\ub294 \uc81c\ubaa9\uc73c\ub85c, \"\uc804\uad50\uc870\u00b7\uc804\uacf5\ub178\u00b7\ubbfc\uc8fc\ub178\ucd1d \uac19\uc740 \uc88c\ud30c\uc138\ub825\ub4e4\uc774 \uc0dd\uc131\ub418\ub3c4\ub85d \ub3c4\uc640 \uccad\uc18c\ub144\ub4e4\uc758 \uc815\uc2e0\uc744 \uc138\ub1cc\uc2dc\ud0a4\uace0, \uad6d\uac00 \uc815\uccb4\uc131\uc744 \ud63c\ub3c8\uc5d0 \ube60\ub728\ub838\uc73c\uba70, \uad6d\ubbfc\uc744 \ubd88\uc548\ud558\uac8c \ud588\ub2e4\"\ub294 \ub0b4\uc6a9\uc774\uc5c8\ub2e4. \uc815\ubaa8\uc528\ub294 \uc778\ubd84 \ud22c\ucc99\ud6c4 \ud604\uc7a5\uc5d0\uc11c \ubc14\ub85c \uacbd\ucc30\uc5d0 \ubd99\uc7a1\ud614\uc73c\uba70 \uacbd\ucc30 \uc9c4\uc220\uc5d0\uc11c \"\uace0 \ub178\ubb34\ud604 \uc804 \ub300\ud1b5\ub839\uc774 \uc7ac\uc9c1 \uc911 \uc88c\uc775\uc138\ub825\uc774 \ud310\uc744 \uce58\ub294 \ub370 \ub300\ud574 \ubd88\ub9cc\uc744 \ud488\uace0 \ubc94\ud589\uc744 \uacc4\ud68d\ud588\ub2e4\" \uace0 \ubc1d\ud614\ub2e4. \ud55c\ud3b8 \uacbd\ucc30\uc740 \uc815\ubaa8\uc528\uc758 \ub2e8\ub3c5\ubc94\ud589\uc73c\ub85c \uacb0\ub860 \uc9d3\uace0 \uc7ac\ubb3c\uc190\uad34 \ub4f1\uc758 \ud610\uc758\ub85c \uc785\uac74\ud558\uace0, \uc778\ubd84\uc774 \ud22c\ucc99\ub41c \ubd09\ubd84\uc744 \uc138\ucc99 \uc870\uce58\ud588\ub2e4. \uc774\uc5d0 \ub300\ud574 \ubbfc\uc8fc\ub2f9 \ucc28\uc601 \ub300\ubcc0\uc778\uc740 \"\ubbff\uae30\uc9c0 \uc54a\ub294 \uc774\ubc88 \uc0ac\uac74\uc5d0 \ub300\ud574 \ubbfc\uc8fc\ub2f9\uc740 \uad6d\ubbfc\uacfc \ud568\uaed8 \ubd84\ub178\ud558\uba70 \uc720\uac10\uc758 \ub73b\uc744 \ubc1d\ud78c\ub2e4. \uc544\uc6b8\ub7ec \uacbd\ucc30\uc740 \ucca0\uc800\ud55c \uc218\uc0ac\ub97c \ud1b5\ud574 \uc0ac\uac74\uc758 \uc804\ub9d0\uc744 \ubc1d\ud600\uc57c \ud558\uace0, \ub9cc\uc57d \ubc30\ud6c4\uac00 \uc788\ub2e4\uba74 \ucca0\uc800\ud788 \uac00\ub824\ub0b4 \uc5c4\ubc8c\ud574\uc57c \ud560 \uac83\"\uc774\ub77c\uace0 \ubc1d\ud614\uc73c\uba70, \uc18c\uc124\uac00 \uc774\uc678\uc218\ub294 \uc790\uc2e0\uc758 \ud2b8\uc704\ud130\uc5d0\uc11c \"\uc774\ub7f0 \uc0ac\ub78c\ub4e4\uc77c\uc218\ub85d \uad6d\uaca9 \uc790\uc8fc \ub4e4\uba39\uac70\ub9ac\uba74\uc11c \uc560\uad6d\uc790 \ud589\uc138\ub97c \ud55c\ub2e4. \ub2e8\uc138\ud3ec\uc801\uc778 \uad6c\ud1a0\uc720\ubc1c\uc790\ub4e4\"\uc774\ub77c\uace0 \ub9f9\ube44\ub09c\ud588\ub2e4. \ud55c\ub098\ub77c\ub2f9 \uc548\uc0c1\uc218 \ub300\ud45c\ub3c4 \"\uc774\ubc88 \ubd88\uc0c1\uc0ac\ub294 \uae4a\uc774 \uac1c\ud0c4\ud560 \uc218 \ubc16\uc5d0 \uc5c6\ub294 \uc0c1\ud669\"\uc774\ub77c\uba70 \"\ubc95\uc5d0 \ub530\ub77c \uc5c4\uc815\ucc98\ub9ac\ud574\uc57c \ud55c\ub2e4\"\uace0 \uc8fc\uc7a5\ud588\ub2e4.", "sentence_answer": " 2010\ub144 11\uc6d4 14\uc77c \uc624\ud6c4 1\uc2dc\uacbd \ubd09\ud558\ub9c8\uc744 \ub178\ubb34\ud604 \ubb18\uc18c\uc5d0 \uc778\ubd84\uc774 \ud22c\ucc99\ub418\ub294 \uc0ac\uac74\uc774 \ubc1c\uc0dd\ud588\ub2e4.", "paragraph_id": "6483948-32-0", "paragraph_question": "question: \ubd09\ud558\ub9c8\uc744 \ub178\ubb34\ud604 \uc804 \ub300\ud1b5\ub839 \ubb18\uc18c\uc5d0 \uc778\ubd84\uc774 \ud22c\ucc99\ub41c \uc0ac\uac74\uc774 \uc77c\uc5b4\ub09c \ub0a0\uc9dc\ub294?, context: 2010\ub144 11\uc6d4 14\uc77c \uc624\ud6c4 1\uc2dc\uacbd \ubd09\ud558\ub9c8\uc744 \ub178\ubb34\ud604 \ubb18\uc18c\uc5d0 \uc778\ubd84\uc774 \ud22c\ucc99\ub418\ub294 \uc0ac\uac74\uc774 \ubc1c\uc0dd\ud588\ub2e4. 62\uc138\uc758 \uc815\ubaa8\uc528\ub294 \ub178\ubb34\ud604 \ubb18\uc18c\uc5d0 \uc778\ubd84\uc744 \ud22c\ucc99\ud558\uace0 \uc720\uc778\ubb3c 22\uc7a5\uc744 \uc0b4\ud3ec\ud558\uc600\ub294\ub370 \uadf8\uac00 \ubfcc\ub9b0 \uc720\uc778\ubb3c\uc740 \ub178\ubb34\ud604 \uadf8\ub300 \ubb34\ub364\uc5d0 \ub625\ubb3c\uc744 \ubd80\uc73c\uba70\"\ub77c\ub294 \uc81c\ubaa9\uc73c\ub85c, \"\uc804\uad50\uc870\u00b7\uc804\uacf5\ub178\u00b7\ubbfc\uc8fc\ub178\ucd1d \uac19\uc740 \uc88c\ud30c\uc138\ub825\ub4e4\uc774 \uc0dd\uc131\ub418\ub3c4\ub85d \ub3c4\uc640 \uccad\uc18c\ub144\ub4e4\uc758 \uc815\uc2e0\uc744 \uc138\ub1cc\uc2dc\ud0a4\uace0, \uad6d\uac00 \uc815\uccb4\uc131\uc744 \ud63c\ub3c8\uc5d0 \ube60\ub728\ub838\uc73c\uba70, \uad6d\ubbfc\uc744 \ubd88\uc548\ud558\uac8c \ud588\ub2e4\"\ub294 \ub0b4\uc6a9\uc774\uc5c8\ub2e4. \uc815\ubaa8\uc528\ub294 \uc778\ubd84 \ud22c\ucc99\ud6c4 \ud604\uc7a5\uc5d0\uc11c \ubc14\ub85c \uacbd\ucc30\uc5d0 \ubd99\uc7a1\ud614\uc73c\uba70 \uacbd\ucc30 \uc9c4\uc220\uc5d0\uc11c \"\uace0 \ub178\ubb34\ud604 \uc804 \ub300\ud1b5\ub839\uc774 \uc7ac\uc9c1 \uc911 \uc88c\uc775\uc138\ub825\uc774 \ud310\uc744 \uce58\ub294 \ub370 \ub300\ud574 \ubd88\ub9cc\uc744 \ud488\uace0 \ubc94\ud589\uc744 \uacc4\ud68d\ud588\ub2e4\" \uace0 \ubc1d\ud614\ub2e4. \ud55c\ud3b8 \uacbd\ucc30\uc740 \uc815\ubaa8\uc528\uc758 \ub2e8\ub3c5\ubc94\ud589\uc73c\ub85c \uacb0\ub860 \uc9d3\uace0 \uc7ac\ubb3c\uc190\uad34 \ub4f1\uc758 \ud610\uc758\ub85c \uc785\uac74\ud558\uace0, \uc778\ubd84\uc774 \ud22c\ucc99\ub41c \ubd09\ubd84\uc744 \uc138\ucc99 \uc870\uce58\ud588\ub2e4. \uc774\uc5d0 \ub300\ud574 \ubbfc\uc8fc\ub2f9 \ucc28\uc601 \ub300\ubcc0\uc778\uc740 \"\ubbff\uae30\uc9c0 \uc54a\ub294 \uc774\ubc88 \uc0ac\uac74\uc5d0 \ub300\ud574 \ubbfc\uc8fc\ub2f9\uc740 \uad6d\ubbfc\uacfc \ud568\uaed8 \ubd84\ub178\ud558\uba70 \uc720\uac10\uc758 \ub73b\uc744 \ubc1d\ud78c\ub2e4. \uc544\uc6b8\ub7ec \uacbd\ucc30\uc740 \ucca0\uc800\ud55c \uc218\uc0ac\ub97c \ud1b5\ud574 \uc0ac\uac74\uc758 \uc804\ub9d0\uc744 \ubc1d\ud600\uc57c \ud558\uace0, \ub9cc\uc57d \ubc30\ud6c4\uac00 \uc788\ub2e4\uba74 \ucca0\uc800\ud788 \uac00\ub824\ub0b4 \uc5c4\ubc8c\ud574\uc57c \ud560 \uac83\"\uc774\ub77c\uace0 \ubc1d\ud614\uc73c\uba70, \uc18c\uc124\uac00 \uc774\uc678\uc218\ub294 \uc790\uc2e0\uc758 \ud2b8\uc704\ud130\uc5d0\uc11c \"\uc774\ub7f0 \uc0ac\ub78c\ub4e4\uc77c\uc218\ub85d \uad6d\uaca9 \uc790\uc8fc \ub4e4\uba39\uac70\ub9ac\uba74\uc11c \uc560\uad6d\uc790 \ud589\uc138\ub97c \ud55c\ub2e4. \ub2e8\uc138\ud3ec\uc801\uc778 \uad6c\ud1a0\uc720\ubc1c\uc790\ub4e4\"\uc774\ub77c\uace0 \ub9f9\ube44\ub09c\ud588\ub2e4. \ud55c\ub098\ub77c\ub2f9 \uc548\uc0c1\uc218 \ub300\ud45c\ub3c4 \"\uc774\ubc88 \ubd88\uc0c1\uc0ac\ub294 \uae4a\uc774 \uac1c\ud0c4\ud560 \uc218 \ubc16\uc5d0 \uc5c6\ub294 \uc0c1\ud669\"\uc774\ub77c\uba70 \"\ubc95\uc5d0 \ub530\ub77c \uc5c4\uc815\ucc98\ub9ac\ud574\uc57c \ud55c\ub2e4\"\uace0 \uc8fc\uc7a5\ud588\ub2e4."} {"question": "2016\ub144 8\uc6d4 18\uc77c \ubbf8\uad6d\uc774 \ucd5c\ucd08\ub85c \uad0c\uc758 \uc5b4\ub290 \uae30\uc9c0\uc5d0 \ubc30\uce58\ub41c \uc0ac\ub4dc \uae30\uc9c0\ub97c \uacf5\uac1c\ud588\ub098\uc694?", "paragraph": "2016\ub144 8\uc6d4 18\uc77c \ubbf8\uad6d\uc774 \ud55c\uad6d \uad6d\ubc29\uad00\uacc4\uc790\uc640 \uc5b8\ub860\uc5d0 \ucd5c\ucd08\ub85c \uad0c\uc758 \uc564\ub354\uc2a8 \uacf5\uad70\uae30\uc9c0\uc5d0 \ubc30\uce58\ub41c \uc0ac\ub4dc \uae30\uc9c0\ub97c \uacf5\uac1c\ud558\uc600\uace0 \uad0c \uc0ac\ub4dc\uae30\uc9c0\uc758 \ub808\uc774\ub354 \uc804\uc790\ud30c\ub97c \uce21\uc815\ud55c \uacb0\uacfc\ub294 \ud55c\uad6d\ubc29\uc1a1\ud1b5\uc2e0\uc704\uc6d0\ud68c \uc804\uc790\ud30c \uc778\uccb4\ubcf4\ud638 \uae30\uc900\uce58\uc778 10W/m\uc758 0.007% \uc218\uc900\uc774\uc5c8\ub2e4. \ud55c\uad6d\uc740 \ud604\uc7ac \uc774\uc2a4\ub77c\uc5d8 \uc288\ud37c\uadf8\ub9b0\ud30c\uc778 \ub808\uc774\ub2e4 2\uae30\ub97c \ubc30\uce58\uc911\uc774\ub2e4. \uc0ac\ub4dc \ub808\uc774\ub2e4 \ubcf4\ub2e4 \uace0\ucd9c\ub825\uc778\ub370\ub3c4, \uadf8 \ub450 \uc9c0\uc5ed \uc8fc\ubbfc\ub4e4\uc740 \uc544\ubb34\ub7f0 \uc2dc\uc704\ub3c4 \ubc18\ub300\ub3c4 \ud558\uc9c0 \uc54a\uace0 \uc788\uc73c\uba70, \uc804\uc790\ud30c \ubbfc\uc6d0\ub3c4 \uc5c6\ub2e4. \ud55c\uad6d\uc740 2025\ub144\uae4c\uc9c0 \uc288\ud37c\uadf8\ub9b0\ud30c\uc778 \ub808\uc774\ub2e4 \ubcf4\ub2e4 \uace0\ucd9c\ub825\uc778 \ud55c\ud654\ud0c8\ub808\uc2a4 L-SAM \ub808\uc774\ub2e4 4\uae30\ub97c \ubc30\uce58\ud560 \uac83\uc774\ub2e4. 2016\ub144 \uc804\uc790\ud30c \ubb38\uc81c\ub85c \uc0ac\ub4dc\ub97c \ubc18\ub300\ud558\ub294 \ub354\ubd88\uc5b4 \ubbfc\uc8fc\ub2f9\uc740 \ub178\ubb34\ud604 \uc815\ubd80 \ub2f9\uc2dc L-SAM \uac1c\ubc1c\uc744 \ucd94\uc9c4\ud588\ub358 \uc9d1\uad8c\ub2f9\uc774\uc5c8\ub2e4.", "answer": "\uc564\ub354\uc2a8 \uacf5\uad70\uae30\uc9c0", "sentence": "2016\ub144 8\uc6d4 18\uc77c \ubbf8\uad6d\uc774 \ud55c\uad6d \uad6d\ubc29\uad00\uacc4\uc790\uc640 \uc5b8\ub860\uc5d0 \ucd5c\ucd08\ub85c \uad0c\uc758 \uc564\ub354\uc2a8 \uacf5\uad70\uae30\uc9c0 \uc5d0 \ubc30\uce58\ub41c \uc0ac\ub4dc \uae30\uc9c0\ub97c \uacf5\uac1c\ud558\uc600\uace0 \uad0c \uc0ac\ub4dc\uae30\uc9c0\uc758 \ub808\uc774\ub354 \uc804\uc790\ud30c\ub97c \uce21\uc815\ud55c \uacb0\uacfc\ub294 \ud55c\uad6d\ubc29\uc1a1\ud1b5\uc2e0\uc704\uc6d0\ud68c \uc804\uc790\ud30c \uc778\uccb4\ubcf4\ud638 \uae30\uc900\uce58\uc778 10W/m\uc758 0.007% \uc218\uc900\uc774\uc5c8\ub2e4.", "paragraph_sentence": " 2016\ub144 8\uc6d4 18\uc77c \ubbf8\uad6d\uc774 \ud55c\uad6d \uad6d\ubc29\uad00\uacc4\uc790\uc640 \uc5b8\ub860\uc5d0 \ucd5c\ucd08\ub85c \uad0c\uc758 \uc564\ub354\uc2a8 \uacf5\uad70\uae30\uc9c0 \uc5d0 \ubc30\uce58\ub41c \uc0ac\ub4dc \uae30\uc9c0\ub97c \uacf5\uac1c\ud558\uc600\uace0 \uad0c \uc0ac\ub4dc\uae30\uc9c0\uc758 \ub808\uc774\ub354 \uc804\uc790\ud30c\ub97c \uce21\uc815\ud55c \uacb0\uacfc\ub294 \ud55c\uad6d\ubc29\uc1a1\ud1b5\uc2e0\uc704\uc6d0\ud68c \uc804\uc790\ud30c \uc778\uccb4\ubcf4\ud638 \uae30\uc900\uce58\uc778 10W/m\uc758 0.007% \uc218\uc900\uc774\uc5c8\ub2e4. \ud55c\uad6d\uc740 \ud604\uc7ac \uc774\uc2a4\ub77c\uc5d8 \uc288\ud37c\uadf8\ub9b0\ud30c\uc778 \ub808\uc774\ub2e4 2\uae30\ub97c \ubc30\uce58\uc911\uc774\ub2e4. \uc0ac\ub4dc \ub808\uc774\ub2e4 \ubcf4\ub2e4 \uace0\ucd9c\ub825\uc778\ub370\ub3c4, \uadf8 \ub450 \uc9c0\uc5ed \uc8fc\ubbfc\ub4e4\uc740 \uc544\ubb34\ub7f0 \uc2dc\uc704\ub3c4 \ubc18\ub300\ub3c4 \ud558\uc9c0 \uc54a\uace0 \uc788\uc73c\uba70, \uc804\uc790\ud30c \ubbfc\uc6d0\ub3c4 \uc5c6\ub2e4. \ud55c\uad6d\uc740 2025\ub144\uae4c\uc9c0 \uc288\ud37c\uadf8\ub9b0\ud30c\uc778 \ub808\uc774\ub2e4 \ubcf4\ub2e4 \uace0\ucd9c\ub825\uc778 \ud55c\ud654\ud0c8\ub808\uc2a4 L-SAM \ub808\uc774\ub2e4 4\uae30\ub97c \ubc30\uce58\ud560 \uac83\uc774\ub2e4. 2016\ub144 \uc804\uc790\ud30c \ubb38\uc81c\ub85c \uc0ac\ub4dc\ub97c \ubc18\ub300\ud558\ub294 \ub354\ubd88\uc5b4 \ubbfc\uc8fc\ub2f9\uc740 \ub178\ubb34\ud604 \uc815\ubd80 \ub2f9\uc2dc L-SAM \uac1c\ubc1c\uc744 \ucd94\uc9c4\ud588\ub358 \uc9d1\uad8c\ub2f9\uc774\uc5c8\ub2e4.", "paragraph_answer": "2016\ub144 8\uc6d4 18\uc77c \ubbf8\uad6d\uc774 \ud55c\uad6d \uad6d\ubc29\uad00\uacc4\uc790\uc640 \uc5b8\ub860\uc5d0 \ucd5c\ucd08\ub85c \uad0c\uc758 \uc564\ub354\uc2a8 \uacf5\uad70\uae30\uc9c0 \uc5d0 \ubc30\uce58\ub41c \uc0ac\ub4dc \uae30\uc9c0\ub97c \uacf5\uac1c\ud558\uc600\uace0 \uad0c \uc0ac\ub4dc\uae30\uc9c0\uc758 \ub808\uc774\ub354 \uc804\uc790\ud30c\ub97c \uce21\uc815\ud55c \uacb0\uacfc\ub294 \ud55c\uad6d\ubc29\uc1a1\ud1b5\uc2e0\uc704\uc6d0\ud68c \uc804\uc790\ud30c \uc778\uccb4\ubcf4\ud638 \uae30\uc900\uce58\uc778 10W/m\uc758 0.007% \uc218\uc900\uc774\uc5c8\ub2e4. \ud55c\uad6d\uc740 \ud604\uc7ac \uc774\uc2a4\ub77c\uc5d8 \uc288\ud37c\uadf8\ub9b0\ud30c\uc778 \ub808\uc774\ub2e4 2\uae30\ub97c \ubc30\uce58\uc911\uc774\ub2e4. \uc0ac\ub4dc \ub808\uc774\ub2e4 \ubcf4\ub2e4 \uace0\ucd9c\ub825\uc778\ub370\ub3c4, \uadf8 \ub450 \uc9c0\uc5ed \uc8fc\ubbfc\ub4e4\uc740 \uc544\ubb34\ub7f0 \uc2dc\uc704\ub3c4 \ubc18\ub300\ub3c4 \ud558\uc9c0 \uc54a\uace0 \uc788\uc73c\uba70, \uc804\uc790\ud30c \ubbfc\uc6d0\ub3c4 \uc5c6\ub2e4. \ud55c\uad6d\uc740 2025\ub144\uae4c\uc9c0 \uc288\ud37c\uadf8\ub9b0\ud30c\uc778 \ub808\uc774\ub2e4 \ubcf4\ub2e4 \uace0\ucd9c\ub825\uc778 \ud55c\ud654\ud0c8\ub808\uc2a4 L-SAM \ub808\uc774\ub2e4 4\uae30\ub97c \ubc30\uce58\ud560 \uac83\uc774\ub2e4. 2016\ub144 \uc804\uc790\ud30c \ubb38\uc81c\ub85c \uc0ac\ub4dc\ub97c \ubc18\ub300\ud558\ub294 \ub354\ubd88\uc5b4 \ubbfc\uc8fc\ub2f9\uc740 \ub178\ubb34\ud604 \uc815\ubd80 \ub2f9\uc2dc L-SAM \uac1c\ubc1c\uc744 \ucd94\uc9c4\ud588\ub358 \uc9d1\uad8c\ub2f9\uc774\uc5c8\ub2e4.", "sentence_answer": "2016\ub144 8\uc6d4 18\uc77c \ubbf8\uad6d\uc774 \ud55c\uad6d \uad6d\ubc29\uad00\uacc4\uc790\uc640 \uc5b8\ub860\uc5d0 \ucd5c\ucd08\ub85c \uad0c\uc758 \uc564\ub354\uc2a8 \uacf5\uad70\uae30\uc9c0 \uc5d0 \ubc30\uce58\ub41c \uc0ac\ub4dc \uae30\uc9c0\ub97c \uacf5\uac1c\ud558\uc600\uace0 \uad0c \uc0ac\ub4dc\uae30\uc9c0\uc758 \ub808\uc774\ub354 \uc804\uc790\ud30c\ub97c \uce21\uc815\ud55c \uacb0\uacfc\ub294 \ud55c\uad6d\ubc29\uc1a1\ud1b5\uc2e0\uc704\uc6d0\ud68c \uc804\uc790\ud30c \uc778\uccb4\ubcf4\ud638 \uae30\uc900\uce58\uc778 10W/m\uc758 0.007% \uc218\uc900\uc774\uc5c8\ub2e4.", "paragraph_id": "6546295-32-0", "paragraph_question": "question: 2016\ub144 8\uc6d4 18\uc77c \ubbf8\uad6d\uc774 \ucd5c\ucd08\ub85c \uad0c\uc758 \uc5b4\ub290 \uae30\uc9c0\uc5d0 \ubc30\uce58\ub41c \uc0ac\ub4dc \uae30\uc9c0\ub97c \uacf5\uac1c\ud588\ub098\uc694?, context: 2016\ub144 8\uc6d4 18\uc77c \ubbf8\uad6d\uc774 \ud55c\uad6d \uad6d\ubc29\uad00\uacc4\uc790\uc640 \uc5b8\ub860\uc5d0 \ucd5c\ucd08\ub85c \uad0c\uc758 \uc564\ub354\uc2a8 \uacf5\uad70\uae30\uc9c0\uc5d0 \ubc30\uce58\ub41c \uc0ac\ub4dc \uae30\uc9c0\ub97c \uacf5\uac1c\ud558\uc600\uace0 \uad0c \uc0ac\ub4dc\uae30\uc9c0\uc758 \ub808\uc774\ub354 \uc804\uc790\ud30c\ub97c \uce21\uc815\ud55c \uacb0\uacfc\ub294 \ud55c\uad6d\ubc29\uc1a1\ud1b5\uc2e0\uc704\uc6d0\ud68c \uc804\uc790\ud30c \uc778\uccb4\ubcf4\ud638 \uae30\uc900\uce58\uc778 10W/m\uc758 0.007% \uc218\uc900\uc774\uc5c8\ub2e4. \ud55c\uad6d\uc740 \ud604\uc7ac \uc774\uc2a4\ub77c\uc5d8 \uc288\ud37c\uadf8\ub9b0\ud30c\uc778 \ub808\uc774\ub2e4 2\uae30\ub97c \ubc30\uce58\uc911\uc774\ub2e4. \uc0ac\ub4dc \ub808\uc774\ub2e4 \ubcf4\ub2e4 \uace0\ucd9c\ub825\uc778\ub370\ub3c4, \uadf8 \ub450 \uc9c0\uc5ed \uc8fc\ubbfc\ub4e4\uc740 \uc544\ubb34\ub7f0 \uc2dc\uc704\ub3c4 \ubc18\ub300\ub3c4 \ud558\uc9c0 \uc54a\uace0 \uc788\uc73c\uba70, \uc804\uc790\ud30c \ubbfc\uc6d0\ub3c4 \uc5c6\ub2e4. \ud55c\uad6d\uc740 2025\ub144\uae4c\uc9c0 \uc288\ud37c\uadf8\ub9b0\ud30c\uc778 \ub808\uc774\ub2e4 \ubcf4\ub2e4 \uace0\ucd9c\ub825\uc778 \ud55c\ud654\ud0c8\ub808\uc2a4 L-SAM \ub808\uc774\ub2e4 4\uae30\ub97c \ubc30\uce58\ud560 \uac83\uc774\ub2e4. 2016\ub144 \uc804\uc790\ud30c \ubb38\uc81c\ub85c \uc0ac\ub4dc\ub97c \ubc18\ub300\ud558\ub294 \ub354\ubd88\uc5b4 \ubbfc\uc8fc\ub2f9\uc740 \ub178\ubb34\ud604 \uc815\ubd80 \ub2f9\uc2dc L-SAM \uac1c\ubc1c\uc744 \ucd94\uc9c4\ud588\ub358 \uc9d1\uad8c\ub2f9\uc774\uc5c8\ub2e4."} {"question": "\ub300\ud55c\ud56d\uacf5 \uc77c\ubc18\uc11d\uc5d0\uc11c IFE\uc5d0 \uc81c\uacf5\ud558\ub294 \uc11c\ube44\uc2a4\ub294?", "paragraph": "\ub300\ud55c\ud56d\uacf5 \uc77c\ubc18\uc11d\uc758 \uacbd\uc6b0 IFE(\uae30\ub0b4 \uc5d4\ud130\ud14c\uc778\uba3c\ud2b8)\uc758 \ub300\ud45c\uaca9\uc778 \uac1c\uc778\uc6a9 \ub514\uc2a4\ud50c\ub808\uc774\uac00 2000\ub144\ub300\uc5d0 \uc640\uc11c\uc57c \uc7a5\ucc29\ub418\uc5b4, \ud0c0 \ud56d\uacf5\uc0ac\uc5d0 \ube44\ud574 \ud55c \ubc1c \ub2a6\uac8c \uc2dc\uc791\ud558\uc600\ub2e4. \ud558\uc9c0\ub9cc AVOD \uc11c\ube44\uc2a4\ub85c \uc120\ub3c4\uaca9\uc778 IFE\ub97c \uc81c\uacf5\ud568\uc73c\ub85c\uc368 \uc774\ub97c \ubb34\ub9c8\ud558\uace0 \uc788\ub2e4. \ud558\uc9c0\ub9cc \uc544\uc9c1\uae4c\uc9c0\ub3c4 \uc77c\ubd80 \ub2e8\uac70\ub9ac\ub178\uc120(\uc77c\ubcf8, \uc911\uad6d, \ub3d9\ub0a8\uc544 \uc77c\ubd80\ub178\uc120)\uc5d0\uc11c\ub294 \uc774 \uc11c\ube44\uc2a4\uac00 \uc81c\uacf5\ub418\uc9c0 \uc54a\ub294\ub2e4. \ud2b9\ud788 \uc5d0\uc5b4\ubc84\uc2a4 A330 \uae30\uc885\uc758 \uacbd\uc6b0 \ub0b4\ubd80 \uc778\ud14c\ub9ac\uc5b4\ub294 \ubc14\uafe8\uc74c\uc5d0\ub3c4 \ubd88\uad6c\ud558\uace0 \uac1c\uc778\uc6a9 \ub514\uc2a4\ud50c\ub808\uc774\ub294 \uc7a5\ucc29\ud558\uc9c0 \uc54a\uc558\uc5c8\uc73c\ub098, 2010\ub144\uc744 \uae30\uc810\uc73c\ub85c \uac1c\uc870\ub418\uae30 \uc2dc\uc791\ud558\uc600\ub2e4. \"\ub274 \uc774\ucf54\ub178\ubbf8\"\ub294 \ubcf4\uc789 737-900ER, \ubcf4\uc789 777-200ER, \ubcf4\uc789 777-300, \ubcf4\uc789 777-300ER, \uc5d0\uc5b4\ubc84\uc2a4 A330-200\uc640 \uc5d0\uc5b4\ubc84\uc2a4 A330-300, \uc5d0\uc5b4\ubc84\uc2a4 A380-800 \uc804\uc88c\uc11d\uc5d0 \uc7a5\ucc29\uc644\ub8cc\ub418\uc5c8\uc73c\uba70 \uc88c\uc11d \ub4b7\uba74\uc744 \uc2ac\ub9bc\ud654\ud574 \uc55e\ub4a4 \uacf5\uac04\uc744 \ud655\ubcf4\ud588\uace0 \ub4f1\ubc1b\uc774\ub97c \ub4a4\ub85c \uae30\uc6b8\uc77c \uacbd\uc6b0 \ubc29\uc11d \ubd80\ubd84\uc774 \uc55e\uc73c\ub85c \uc774\ub3d9\ud558\ub3c4\ub85d \uc124\uacc4\ud574 \uc55e \uc0ac\ub78c\uc758 \uc6c0\uc9c1\uc784\uc73c\ub85c\ubd80\ud130 \ubc29\ud574\ub97c \ubc1b\uc9c0 \uc54a\ub3c4\ub85d \ud588\ub2e4. \ubd04\ubc14\ub514\uc5b4 CS300\uc5d0 \uc0c8\ub85c\uc6b4 \uc774\ucf54\ub178\ubbf8 \ud50c\ub7ec\uc2a4\ub97c \uc7a5\ucc29\ud558\uc600\ub2e4.", "answer": "AVOD \uc11c\ube44\uc2a4", "sentence": "\ud558\uc9c0\ub9cc AVOD \uc11c\ube44\uc2a4 \ub85c \uc120\ub3c4\uaca9\uc778 IFE\ub97c \uc81c\uacf5\ud568\uc73c\ub85c\uc368 \uc774\ub97c \ubb34\ub9c8\ud558\uace0 \uc788\ub2e4.", "paragraph_sentence": "\ub300\ud55c\ud56d\uacf5 \uc77c\ubc18\uc11d\uc758 \uacbd\uc6b0 IFE(\uae30\ub0b4 \uc5d4\ud130\ud14c\uc778\uba3c\ud2b8)\uc758 \ub300\ud45c\uaca9\uc778 \uac1c\uc778\uc6a9 \ub514\uc2a4\ud50c\ub808\uc774\uac00 2000\ub144\ub300\uc5d0 \uc640\uc11c\uc57c \uc7a5\ucc29\ub418\uc5b4, \ud0c0 \ud56d\uacf5\uc0ac\uc5d0 \ube44\ud574 \ud55c \ubc1c \ub2a6\uac8c \uc2dc\uc791\ud558\uc600\ub2e4. \ud558\uc9c0\ub9cc AVOD \uc11c\ube44\uc2a4 \ub85c \uc120\ub3c4\uaca9\uc778 IFE\ub97c \uc81c\uacf5\ud568\uc73c\ub85c\uc368 \uc774\ub97c \ubb34\ub9c8\ud558\uace0 \uc788\ub2e4. \ud558\uc9c0\ub9cc \uc544\uc9c1\uae4c\uc9c0\ub3c4 \uc77c\ubd80 \ub2e8\uac70\ub9ac\ub178\uc120(\uc77c\ubcf8, \uc911\uad6d, \ub3d9\ub0a8\uc544 \uc77c\ubd80\ub178\uc120)\uc5d0\uc11c\ub294 \uc774 \uc11c\ube44\uc2a4\uac00 \uc81c\uacf5\ub418\uc9c0 \uc54a\ub294\ub2e4. \ud2b9\ud788 \uc5d0\uc5b4\ubc84\uc2a4 A330 \uae30\uc885\uc758 \uacbd\uc6b0 \ub0b4\ubd80 \uc778\ud14c\ub9ac\uc5b4\ub294 \ubc14\uafe8\uc74c\uc5d0\ub3c4 \ubd88\uad6c\ud558\uace0 \uac1c\uc778\uc6a9 \ub514\uc2a4\ud50c\ub808\uc774\ub294 \uc7a5\ucc29\ud558\uc9c0 \uc54a\uc558\uc5c8\uc73c\ub098, 2010\ub144\uc744 \uae30\uc810\uc73c\ub85c \uac1c\uc870\ub418\uae30 \uc2dc\uc791\ud558\uc600\ub2e4. \"\ub274 \uc774\ucf54\ub178\ubbf8\"\ub294 \ubcf4\uc789 737-900ER, \ubcf4\uc789 777-200ER, \ubcf4\uc789 777-300, \ubcf4\uc789 777-300ER, \uc5d0\uc5b4\ubc84\uc2a4 A330-200\uc640 \uc5d0\uc5b4\ubc84\uc2a4 A330-300, \uc5d0\uc5b4\ubc84\uc2a4 A380-800 \uc804\uc88c\uc11d\uc5d0 \uc7a5\ucc29\uc644\ub8cc\ub418\uc5c8\uc73c\uba70 \uc88c\uc11d \ub4b7\uba74\uc744 \uc2ac\ub9bc\ud654\ud574 \uc55e\ub4a4 \uacf5\uac04\uc744 \ud655\ubcf4\ud588\uace0 \ub4f1\ubc1b\uc774\ub97c \ub4a4\ub85c \uae30\uc6b8\uc77c \uacbd\uc6b0 \ubc29\uc11d \ubd80\ubd84\uc774 \uc55e\uc73c\ub85c \uc774\ub3d9\ud558\ub3c4\ub85d \uc124\uacc4\ud574 \uc55e \uc0ac\ub78c\uc758 \uc6c0\uc9c1\uc784\uc73c\ub85c\ubd80\ud130 \ubc29\ud574\ub97c \ubc1b\uc9c0 \uc54a\ub3c4\ub85d \ud588\ub2e4. \ubd04\ubc14\ub514\uc5b4 CS300\uc5d0 \uc0c8\ub85c\uc6b4 \uc774\ucf54\ub178\ubbf8 \ud50c\ub7ec\uc2a4\ub97c \uc7a5\ucc29\ud558\uc600\ub2e4.", "paragraph_answer": "\ub300\ud55c\ud56d\uacf5 \uc77c\ubc18\uc11d\uc758 \uacbd\uc6b0 IFE(\uae30\ub0b4 \uc5d4\ud130\ud14c\uc778\uba3c\ud2b8)\uc758 \ub300\ud45c\uaca9\uc778 \uac1c\uc778\uc6a9 \ub514\uc2a4\ud50c\ub808\uc774\uac00 2000\ub144\ub300\uc5d0 \uc640\uc11c\uc57c \uc7a5\ucc29\ub418\uc5b4, \ud0c0 \ud56d\uacf5\uc0ac\uc5d0 \ube44\ud574 \ud55c \ubc1c \ub2a6\uac8c \uc2dc\uc791\ud558\uc600\ub2e4. \ud558\uc9c0\ub9cc AVOD \uc11c\ube44\uc2a4 \ub85c \uc120\ub3c4\uaca9\uc778 IFE\ub97c \uc81c\uacf5\ud568\uc73c\ub85c\uc368 \uc774\ub97c \ubb34\ub9c8\ud558\uace0 \uc788\ub2e4. \ud558\uc9c0\ub9cc \uc544\uc9c1\uae4c\uc9c0\ub3c4 \uc77c\ubd80 \ub2e8\uac70\ub9ac\ub178\uc120(\uc77c\ubcf8, \uc911\uad6d, \ub3d9\ub0a8\uc544 \uc77c\ubd80\ub178\uc120)\uc5d0\uc11c\ub294 \uc774 \uc11c\ube44\uc2a4\uac00 \uc81c\uacf5\ub418\uc9c0 \uc54a\ub294\ub2e4. \ud2b9\ud788 \uc5d0\uc5b4\ubc84\uc2a4 A330 \uae30\uc885\uc758 \uacbd\uc6b0 \ub0b4\ubd80 \uc778\ud14c\ub9ac\uc5b4\ub294 \ubc14\uafe8\uc74c\uc5d0\ub3c4 \ubd88\uad6c\ud558\uace0 \uac1c\uc778\uc6a9 \ub514\uc2a4\ud50c\ub808\uc774\ub294 \uc7a5\ucc29\ud558\uc9c0 \uc54a\uc558\uc5c8\uc73c\ub098, 2010\ub144\uc744 \uae30\uc810\uc73c\ub85c \uac1c\uc870\ub418\uae30 \uc2dc\uc791\ud558\uc600\ub2e4. \"\ub274 \uc774\ucf54\ub178\ubbf8\"\ub294 \ubcf4\uc789 737-900ER, \ubcf4\uc789 777-200ER, \ubcf4\uc789 777-300, \ubcf4\uc789 777-300ER, \uc5d0\uc5b4\ubc84\uc2a4 A330-200\uc640 \uc5d0\uc5b4\ubc84\uc2a4 A330-300, \uc5d0\uc5b4\ubc84\uc2a4 A380-800 \uc804\uc88c\uc11d\uc5d0 \uc7a5\ucc29\uc644\ub8cc\ub418\uc5c8\uc73c\uba70 \uc88c\uc11d \ub4b7\uba74\uc744 \uc2ac\ub9bc\ud654\ud574 \uc55e\ub4a4 \uacf5\uac04\uc744 \ud655\ubcf4\ud588\uace0 \ub4f1\ubc1b\uc774\ub97c \ub4a4\ub85c \uae30\uc6b8\uc77c \uacbd\uc6b0 \ubc29\uc11d \ubd80\ubd84\uc774 \uc55e\uc73c\ub85c \uc774\ub3d9\ud558\ub3c4\ub85d \uc124\uacc4\ud574 \uc55e \uc0ac\ub78c\uc758 \uc6c0\uc9c1\uc784\uc73c\ub85c\ubd80\ud130 \ubc29\ud574\ub97c \ubc1b\uc9c0 \uc54a\ub3c4\ub85d \ud588\ub2e4. \ubd04\ubc14\ub514\uc5b4 CS300\uc5d0 \uc0c8\ub85c\uc6b4 \uc774\ucf54\ub178\ubbf8 \ud50c\ub7ec\uc2a4\ub97c \uc7a5\ucc29\ud558\uc600\ub2e4.", "sentence_answer": "\ud558\uc9c0\ub9cc AVOD \uc11c\ube44\uc2a4 \ub85c \uc120\ub3c4\uaca9\uc778 IFE\ub97c \uc81c\uacf5\ud568\uc73c\ub85c\uc368 \uc774\ub97c \ubb34\ub9c8\ud558\uace0 \uc788\ub2e4.", "paragraph_id": "6491032-8-0", "paragraph_question": "question: \ub300\ud55c\ud56d\uacf5 \uc77c\ubc18\uc11d\uc5d0\uc11c IFE\uc5d0 \uc81c\uacf5\ud558\ub294 \uc11c\ube44\uc2a4\ub294?, context: \ub300\ud55c\ud56d\uacf5 \uc77c\ubc18\uc11d\uc758 \uacbd\uc6b0 IFE(\uae30\ub0b4 \uc5d4\ud130\ud14c\uc778\uba3c\ud2b8)\uc758 \ub300\ud45c\uaca9\uc778 \uac1c\uc778\uc6a9 \ub514\uc2a4\ud50c\ub808\uc774\uac00 2000\ub144\ub300\uc5d0 \uc640\uc11c\uc57c \uc7a5\ucc29\ub418\uc5b4, \ud0c0 \ud56d\uacf5\uc0ac\uc5d0 \ube44\ud574 \ud55c \ubc1c \ub2a6\uac8c \uc2dc\uc791\ud558\uc600\ub2e4. \ud558\uc9c0\ub9cc AVOD \uc11c\ube44\uc2a4\ub85c \uc120\ub3c4\uaca9\uc778 IFE\ub97c \uc81c\uacf5\ud568\uc73c\ub85c\uc368 \uc774\ub97c \ubb34\ub9c8\ud558\uace0 \uc788\ub2e4. \ud558\uc9c0\ub9cc \uc544\uc9c1\uae4c\uc9c0\ub3c4 \uc77c\ubd80 \ub2e8\uac70\ub9ac\ub178\uc120(\uc77c\ubcf8, \uc911\uad6d, \ub3d9\ub0a8\uc544 \uc77c\ubd80\ub178\uc120)\uc5d0\uc11c\ub294 \uc774 \uc11c\ube44\uc2a4\uac00 \uc81c\uacf5\ub418\uc9c0 \uc54a\ub294\ub2e4. \ud2b9\ud788 \uc5d0\uc5b4\ubc84\uc2a4 A330 \uae30\uc885\uc758 \uacbd\uc6b0 \ub0b4\ubd80 \uc778\ud14c\ub9ac\uc5b4\ub294 \ubc14\uafe8\uc74c\uc5d0\ub3c4 \ubd88\uad6c\ud558\uace0 \uac1c\uc778\uc6a9 \ub514\uc2a4\ud50c\ub808\uc774\ub294 \uc7a5\ucc29\ud558\uc9c0 \uc54a\uc558\uc5c8\uc73c\ub098, 2010\ub144\uc744 \uae30\uc810\uc73c\ub85c \uac1c\uc870\ub418\uae30 \uc2dc\uc791\ud558\uc600\ub2e4. \"\ub274 \uc774\ucf54\ub178\ubbf8\"\ub294 \ubcf4\uc789 737-900ER, \ubcf4\uc789 777-200ER, \ubcf4\uc789 777-300, \ubcf4\uc789 777-300ER, \uc5d0\uc5b4\ubc84\uc2a4 A330-200\uc640 \uc5d0\uc5b4\ubc84\uc2a4 A330-300, \uc5d0\uc5b4\ubc84\uc2a4 A380-800 \uc804\uc88c\uc11d\uc5d0 \uc7a5\ucc29\uc644\ub8cc\ub418\uc5c8\uc73c\uba70 \uc88c\uc11d \ub4b7\uba74\uc744 \uc2ac\ub9bc\ud654\ud574 \uc55e\ub4a4 \uacf5\uac04\uc744 \ud655\ubcf4\ud588\uace0 \ub4f1\ubc1b\uc774\ub97c \ub4a4\ub85c \uae30\uc6b8\uc77c \uacbd\uc6b0 \ubc29\uc11d \ubd80\ubd84\uc774 \uc55e\uc73c\ub85c \uc774\ub3d9\ud558\ub3c4\ub85d \uc124\uacc4\ud574 \uc55e \uc0ac\ub78c\uc758 \uc6c0\uc9c1\uc784\uc73c\ub85c\ubd80\ud130 \ubc29\ud574\ub97c \ubc1b\uc9c0 \uc54a\ub3c4\ub85d \ud588\ub2e4. \ubd04\ubc14\ub514\uc5b4 CS300\uc5d0 \uc0c8\ub85c\uc6b4 \uc774\ucf54\ub178\ubbf8 \ud50c\ub7ec\uc2a4\ub97c \uc7a5\ucc29\ud558\uc600\ub2e4."} {"question": "1961\ub144 \ucd5c\ucd08\ub85c \uc720\uc778 \uc6b0\uc8fc\uc120\uc744 \uc3d8\uc544\uc62c\ub9b0 \uad6d\uac00\ub294?", "paragraph": "1961\ub144 \uc18c\ub828\uc774 \ucd5c\ucd08\uc758 \uc720\uc778 \uc6b0\uc8fc\uc120\uc744 \uc3d8\uc544\uc62c\ub9ac\uba74\uc11c, \uc874 F. \ucf00\ub124\ub514 \ub300\ud1b5\ub839\uc740 \ubbf8\uad6d\uc774 \ucd5c\ucd08\ub85c \"\ub2ec\uc5d0 \uc0ac\ub78c\uc744 \ubcf4\ub0b4\uaca0\ub2e4\"\ub294 \uacc4\ud68d\uc744 \ub0b4\uc138\uc6cc 1969\ub144\uc5d0 \uae30\uc5b4\uc774 \uc2e4\ud604\ud588\ub2e4. \ub610 \ucf00\ub124\ub514\ub294 \ucfe0\ubc14\uc5d0\uc11c \uc18c\ub828\uad70\uacfc \ud575 \uc704\uae30\uc5d0 \uc9c1\uba74\ud558\uae30\ub3c4 \ud558\uc600\ub2e4. \uadf8\ub7f0 \uac00\uc6b4\ub370 \ubbf8\uad6d \uacbd\uc81c\ub294 \uacc4\uc18d \ubc1c\uc804\ud588\ub2e4. \ub9c8\ud2f4 \ub8e8\uc11c \ud0b9 2\uc138\uc758 \ube44\ud3ed\ub825 \uc800\ud56d\uc8fc\uc758\uc640 \ub9ec\ucef4 \uc5d1\uc2a4\uc640 \uc2a4\ud1a0\ud074\ub9ac \uce74\ub9c8\uc774\ud074\uc774 \uc8fc\ub3c4\ud55c \uacfc\uaca9\uc8fc\uc758\ub85c \uc544\ud504\ub9ac\uce74\uacc4 \ubbf8\uad6d\uc778\ub4e4\uc740 \ud751\uc778 \uc778\uad8c\uc6b4\ub3d9\uc744 \uc804\uac1c\ud588\ub2e4. 1963\ub144\uc5d0 \ucf00\ub124\ub514\uac00 \uc554\uc0b4\ub2f9\ud55c \ub4a4, \ub9b0\ub4e0 B. \uc874\uc2a8 \ub300\ud1b5\ub839\uc774 1964\ub144 \ubbfc\uad8c\ubc95, 1965\ub144 \ud22c\ud45c\uad8c\ubc95\uc744 \ud1b5\uacfc\uc2dc\ucf1c \ud751\uc778\uc758 \ubbfc\uad8c\uc774 \uc131\uc7a5\ud588\ub2e4. \ud558\uc9c0\ub9cc \uc874 F. \ucf00\ub124\ub514 \ub300\ud1b5\ub839\uacfc \uc5ec\ub7ec \uc778\uad8c\uc6b4\ub3d9 \uc9c0\ub3c4\uc790\ub4e4\uc774 \uc554\uc0b4 \ub2f9\ud558\ub294 \ub4f1, \uc0ac\ud68c\uac00 \ud63c\ub780\uc2a4\ub7ec\uc6e0\ub2e4. \ud6c4\uc784 \ub300\ud1b5\ub839\uc778 \ub9ac\ucc98\ub4dc \ub2c9\uc2a8\uc740 \ub3d9\ub0a8 \uc544\uc2dc\uc544\ub85c \ub300\ub9ac\uc804\uc744 \ud655\ub300\ud588\ub2e4\uac00 \ubca0\ud2b8\ub0a8 \uc804\uc7c1\uc5d0\uc11c \ud328\ubc30\ud588\ub2e4. \uadf8\ub7f0 \uac00\uc6b4\ub370 \ubc18\ubb38\ud654 \uc6b4\ub3d9\uc774 \ubc18\uc804 \uc6b4\ub3d9\uacfc \ud751\uc778 \ubbfc\uc871\uc8fc\uc758, \uc131 \ud601\uba85\uc758 \uc601\ud5a5\uc744 \ubc1b\uc544 \ub110\ub9ac \uc131\uc7a5\ud588\ub2e4. 1960\ub144\ub300\ub294 \uac01\uc885 \uc554\uc0b4\uc0ac\uac74, \ud751\uc778 \ubbfc\uc871\uc8fc\uc758, \uc5ec\uc131\ud574\ubc29\uc6b4\ub3d9, \ud788\ud53c \ubb38\ud654, \ub3d9\uc131\uc560\ubb38\ud654\uac00 \ub4f1\uc7a5\ud55c \ubcc0\ud601\uacfc \uaca9\ub3d9\uc758 \uc2dc\uae30\uc600\ub2e4.", "answer": "\uc18c\ub828", "sentence": "1961\ub144 \uc18c\ub828 \uc774 \ucd5c\ucd08\uc758 \uc720\uc778 \uc6b0\uc8fc\uc120\uc744 \uc3d8\uc544\uc62c\ub9ac\uba74\uc11c, \uc874 F. \ucf00\ub124\ub514 \ub300\ud1b5\ub839\uc740 \ubbf8\uad6d\uc774 \ucd5c\ucd08\ub85c \"\ub2ec\uc5d0 \uc0ac\ub78c\uc744 \ubcf4\ub0b4\uaca0\ub2e4\"\ub294 \uacc4\ud68d\uc744 \ub0b4\uc138\uc6cc 1969\ub144\uc5d0 \uae30\uc5b4\uc774 \uc2e4\ud604\ud588\ub2e4.", "paragraph_sentence": " 1961\ub144 \uc18c\ub828 \uc774 \ucd5c\ucd08\uc758 \uc720\uc778 \uc6b0\uc8fc\uc120\uc744 \uc3d8\uc544\uc62c\ub9ac\uba74\uc11c, \uc874 F. \ucf00\ub124\ub514 \ub300\ud1b5\ub839\uc740 \ubbf8\uad6d\uc774 \ucd5c\ucd08\ub85c \"\ub2ec\uc5d0 \uc0ac\ub78c\uc744 \ubcf4\ub0b4\uaca0\ub2e4\"\ub294 \uacc4\ud68d\uc744 \ub0b4\uc138\uc6cc 1969\ub144\uc5d0 \uae30\uc5b4\uc774 \uc2e4\ud604\ud588\ub2e4. \ub610 \ucf00\ub124\ub514\ub294 \ucfe0\ubc14\uc5d0\uc11c \uc18c\ub828\uad70\uacfc \ud575 \uc704\uae30\uc5d0 \uc9c1\uba74\ud558\uae30\ub3c4 \ud558\uc600\ub2e4. \uadf8\ub7f0 \uac00\uc6b4\ub370 \ubbf8\uad6d \uacbd\uc81c\ub294 \uacc4\uc18d \ubc1c\uc804\ud588\ub2e4. \ub9c8\ud2f4 \ub8e8\uc11c \ud0b9 2\uc138\uc758 \ube44\ud3ed\ub825 \uc800\ud56d\uc8fc\uc758\uc640 \ub9ec\ucef4 \uc5d1\uc2a4\uc640 \uc2a4\ud1a0\ud074\ub9ac \uce74\ub9c8\uc774\ud074\uc774 \uc8fc\ub3c4\ud55c \uacfc\uaca9\uc8fc\uc758\ub85c \uc544\ud504\ub9ac\uce74\uacc4 \ubbf8\uad6d\uc778\ub4e4\uc740 \ud751\uc778 \uc778\uad8c\uc6b4\ub3d9\uc744 \uc804\uac1c\ud588\ub2e4. 1963\ub144\uc5d0 \ucf00\ub124\ub514\uac00 \uc554\uc0b4\ub2f9\ud55c \ub4a4, \ub9b0\ub4e0 B. \uc874\uc2a8 \ub300\ud1b5\ub839\uc774 1964\ub144 \ubbfc\uad8c\ubc95, 1965\ub144 \ud22c\ud45c\uad8c\ubc95\uc744 \ud1b5\uacfc\uc2dc\ucf1c \ud751\uc778\uc758 \ubbfc\uad8c\uc774 \uc131\uc7a5\ud588\ub2e4. \ud558\uc9c0\ub9cc \uc874 F. \ucf00\ub124\ub514 \ub300\ud1b5\ub839\uacfc \uc5ec\ub7ec \uc778\uad8c\uc6b4\ub3d9 \uc9c0\ub3c4\uc790\ub4e4\uc774 \uc554\uc0b4 \ub2f9\ud558\ub294 \ub4f1, \uc0ac\ud68c\uac00 \ud63c\ub780\uc2a4\ub7ec\uc6e0\ub2e4. \ud6c4\uc784 \ub300\ud1b5\ub839\uc778 \ub9ac\ucc98\ub4dc \ub2c9\uc2a8\uc740 \ub3d9\ub0a8 \uc544\uc2dc\uc544\ub85c \ub300\ub9ac\uc804\uc744 \ud655\ub300\ud588\ub2e4\uac00 \ubca0\ud2b8\ub0a8 \uc804\uc7c1\uc5d0\uc11c \ud328\ubc30\ud588\ub2e4. \uadf8\ub7f0 \uac00\uc6b4\ub370 \ubc18\ubb38\ud654 \uc6b4\ub3d9\uc774 \ubc18\uc804 \uc6b4\ub3d9\uacfc \ud751\uc778 \ubbfc\uc871\uc8fc\uc758, \uc131 \ud601\uba85\uc758 \uc601\ud5a5\uc744 \ubc1b\uc544 \ub110\ub9ac \uc131\uc7a5\ud588\ub2e4. 1960\ub144\ub300\ub294 \uac01\uc885 \uc554\uc0b4\uc0ac\uac74, \ud751\uc778 \ubbfc\uc871\uc8fc\uc758, \uc5ec\uc131\ud574\ubc29\uc6b4\ub3d9, \ud788\ud53c \ubb38\ud654, \ub3d9\uc131\uc560\ubb38\ud654\uac00 \ub4f1\uc7a5\ud55c \ubcc0\ud601\uacfc \uaca9\ub3d9\uc758 \uc2dc\uae30\uc600\ub2e4.", "paragraph_answer": "1961\ub144 \uc18c\ub828 \uc774 \ucd5c\ucd08\uc758 \uc720\uc778 \uc6b0\uc8fc\uc120\uc744 \uc3d8\uc544\uc62c\ub9ac\uba74\uc11c, \uc874 F. \ucf00\ub124\ub514 \ub300\ud1b5\ub839\uc740 \ubbf8\uad6d\uc774 \ucd5c\ucd08\ub85c \"\ub2ec\uc5d0 \uc0ac\ub78c\uc744 \ubcf4\ub0b4\uaca0\ub2e4\"\ub294 \uacc4\ud68d\uc744 \ub0b4\uc138\uc6cc 1969\ub144\uc5d0 \uae30\uc5b4\uc774 \uc2e4\ud604\ud588\ub2e4. \ub610 \ucf00\ub124\ub514\ub294 \ucfe0\ubc14\uc5d0\uc11c \uc18c\ub828\uad70\uacfc \ud575 \uc704\uae30\uc5d0 \uc9c1\uba74\ud558\uae30\ub3c4 \ud558\uc600\ub2e4. \uadf8\ub7f0 \uac00\uc6b4\ub370 \ubbf8\uad6d \uacbd\uc81c\ub294 \uacc4\uc18d \ubc1c\uc804\ud588\ub2e4. \ub9c8\ud2f4 \ub8e8\uc11c \ud0b9 2\uc138\uc758 \ube44\ud3ed\ub825 \uc800\ud56d\uc8fc\uc758\uc640 \ub9ec\ucef4 \uc5d1\uc2a4\uc640 \uc2a4\ud1a0\ud074\ub9ac \uce74\ub9c8\uc774\ud074\uc774 \uc8fc\ub3c4\ud55c \uacfc\uaca9\uc8fc\uc758\ub85c \uc544\ud504\ub9ac\uce74\uacc4 \ubbf8\uad6d\uc778\ub4e4\uc740 \ud751\uc778 \uc778\uad8c\uc6b4\ub3d9\uc744 \uc804\uac1c\ud588\ub2e4. 1963\ub144\uc5d0 \ucf00\ub124\ub514\uac00 \uc554\uc0b4\ub2f9\ud55c \ub4a4, \ub9b0\ub4e0 B. \uc874\uc2a8 \ub300\ud1b5\ub839\uc774 1964\ub144 \ubbfc\uad8c\ubc95, 1965\ub144 \ud22c\ud45c\uad8c\ubc95\uc744 \ud1b5\uacfc\uc2dc\ucf1c \ud751\uc778\uc758 \ubbfc\uad8c\uc774 \uc131\uc7a5\ud588\ub2e4. \ud558\uc9c0\ub9cc \uc874 F. \ucf00\ub124\ub514 \ub300\ud1b5\ub839\uacfc \uc5ec\ub7ec \uc778\uad8c\uc6b4\ub3d9 \uc9c0\ub3c4\uc790\ub4e4\uc774 \uc554\uc0b4 \ub2f9\ud558\ub294 \ub4f1, \uc0ac\ud68c\uac00 \ud63c\ub780\uc2a4\ub7ec\uc6e0\ub2e4. \ud6c4\uc784 \ub300\ud1b5\ub839\uc778 \ub9ac\ucc98\ub4dc \ub2c9\uc2a8\uc740 \ub3d9\ub0a8 \uc544\uc2dc\uc544\ub85c \ub300\ub9ac\uc804\uc744 \ud655\ub300\ud588\ub2e4\uac00 \ubca0\ud2b8\ub0a8 \uc804\uc7c1\uc5d0\uc11c \ud328\ubc30\ud588\ub2e4. \uadf8\ub7f0 \uac00\uc6b4\ub370 \ubc18\ubb38\ud654 \uc6b4\ub3d9\uc774 \ubc18\uc804 \uc6b4\ub3d9\uacfc \ud751\uc778 \ubbfc\uc871\uc8fc\uc758, \uc131 \ud601\uba85\uc758 \uc601\ud5a5\uc744 \ubc1b\uc544 \ub110\ub9ac \uc131\uc7a5\ud588\ub2e4. 1960\ub144\ub300\ub294 \uac01\uc885 \uc554\uc0b4\uc0ac\uac74, \ud751\uc778 \ubbfc\uc871\uc8fc\uc758, \uc5ec\uc131\ud574\ubc29\uc6b4\ub3d9, \ud788\ud53c \ubb38\ud654, \ub3d9\uc131\uc560\ubb38\ud654\uac00 \ub4f1\uc7a5\ud55c \ubcc0\ud601\uacfc \uaca9\ub3d9\uc758 \uc2dc\uae30\uc600\ub2e4.", "sentence_answer": "1961\ub144 \uc18c\ub828 \uc774 \ucd5c\ucd08\uc758 \uc720\uc778 \uc6b0\uc8fc\uc120\uc744 \uc3d8\uc544\uc62c\ub9ac\uba74\uc11c, \uc874 F. \ucf00\ub124\ub514 \ub300\ud1b5\ub839\uc740 \ubbf8\uad6d\uc774 \ucd5c\ucd08\ub85c \"\ub2ec\uc5d0 \uc0ac\ub78c\uc744 \ubcf4\ub0b4\uaca0\ub2e4\"\ub294 \uacc4\ud68d\uc744 \ub0b4\uc138\uc6cc 1969\ub144\uc5d0 \uae30\uc5b4\uc774 \uc2e4\ud604\ud588\ub2e4.", "paragraph_id": "6514860-13-0", "paragraph_question": "question: 1961\ub144 \ucd5c\ucd08\ub85c \uc720\uc778 \uc6b0\uc8fc\uc120\uc744 \uc3d8\uc544\uc62c\ub9b0 \uad6d\uac00\ub294?, context: 1961\ub144 \uc18c\ub828\uc774 \ucd5c\ucd08\uc758 \uc720\uc778 \uc6b0\uc8fc\uc120\uc744 \uc3d8\uc544\uc62c\ub9ac\uba74\uc11c, \uc874 F. \ucf00\ub124\ub514 \ub300\ud1b5\ub839\uc740 \ubbf8\uad6d\uc774 \ucd5c\ucd08\ub85c \"\ub2ec\uc5d0 \uc0ac\ub78c\uc744 \ubcf4\ub0b4\uaca0\ub2e4\"\ub294 \uacc4\ud68d\uc744 \ub0b4\uc138\uc6cc 1969\ub144\uc5d0 \uae30\uc5b4\uc774 \uc2e4\ud604\ud588\ub2e4. \ub610 \ucf00\ub124\ub514\ub294 \ucfe0\ubc14\uc5d0\uc11c \uc18c\ub828\uad70\uacfc \ud575 \uc704\uae30\uc5d0 \uc9c1\uba74\ud558\uae30\ub3c4 \ud558\uc600\ub2e4. \uadf8\ub7f0 \uac00\uc6b4\ub370 \ubbf8\uad6d \uacbd\uc81c\ub294 \uacc4\uc18d \ubc1c\uc804\ud588\ub2e4. \ub9c8\ud2f4 \ub8e8\uc11c \ud0b9 2\uc138\uc758 \ube44\ud3ed\ub825 \uc800\ud56d\uc8fc\uc758\uc640 \ub9ec\ucef4 \uc5d1\uc2a4\uc640 \uc2a4\ud1a0\ud074\ub9ac \uce74\ub9c8\uc774\ud074\uc774 \uc8fc\ub3c4\ud55c \uacfc\uaca9\uc8fc\uc758\ub85c \uc544\ud504\ub9ac\uce74\uacc4 \ubbf8\uad6d\uc778\ub4e4\uc740 \ud751\uc778 \uc778\uad8c\uc6b4\ub3d9\uc744 \uc804\uac1c\ud588\ub2e4. 1963\ub144\uc5d0 \ucf00\ub124\ub514\uac00 \uc554\uc0b4\ub2f9\ud55c \ub4a4, \ub9b0\ub4e0 B. \uc874\uc2a8 \ub300\ud1b5\ub839\uc774 1964\ub144 \ubbfc\uad8c\ubc95, 1965\ub144 \ud22c\ud45c\uad8c\ubc95\uc744 \ud1b5\uacfc\uc2dc\ucf1c \ud751\uc778\uc758 \ubbfc\uad8c\uc774 \uc131\uc7a5\ud588\ub2e4. \ud558\uc9c0\ub9cc \uc874 F. \ucf00\ub124\ub514 \ub300\ud1b5\ub839\uacfc \uc5ec\ub7ec \uc778\uad8c\uc6b4\ub3d9 \uc9c0\ub3c4\uc790\ub4e4\uc774 \uc554\uc0b4 \ub2f9\ud558\ub294 \ub4f1, \uc0ac\ud68c\uac00 \ud63c\ub780\uc2a4\ub7ec\uc6e0\ub2e4. \ud6c4\uc784 \ub300\ud1b5\ub839\uc778 \ub9ac\ucc98\ub4dc \ub2c9\uc2a8\uc740 \ub3d9\ub0a8 \uc544\uc2dc\uc544\ub85c \ub300\ub9ac\uc804\uc744 \ud655\ub300\ud588\ub2e4\uac00 \ubca0\ud2b8\ub0a8 \uc804\uc7c1\uc5d0\uc11c \ud328\ubc30\ud588\ub2e4. \uadf8\ub7f0 \uac00\uc6b4\ub370 \ubc18\ubb38\ud654 \uc6b4\ub3d9\uc774 \ubc18\uc804 \uc6b4\ub3d9\uacfc \ud751\uc778 \ubbfc\uc871\uc8fc\uc758, \uc131 \ud601\uba85\uc758 \uc601\ud5a5\uc744 \ubc1b\uc544 \ub110\ub9ac \uc131\uc7a5\ud588\ub2e4. 1960\ub144\ub300\ub294 \uac01\uc885 \uc554\uc0b4\uc0ac\uac74, \ud751\uc778 \ubbfc\uc871\uc8fc\uc758, \uc5ec\uc131\ud574\ubc29\uc6b4\ub3d9, \ud788\ud53c \ubb38\ud654, \ub3d9\uc131\uc560\ubb38\ud654\uac00 \ub4f1\uc7a5\ud55c \ubcc0\ud601\uacfc \uaca9\ub3d9\uc758 \uc2dc\uae30\uc600\ub2e4."} {"question": "\uc77c\ubcf8 \ub3c4\ucfc4\uc758 \uc778\uad6c\ub294?", "paragraph": "\ub3c4\ucfc4\ub294 \uc77c\ubcf8 \ud63c\uc288 \uc12c\uc758 \uc911\uc559, \uac04\ud1a0 \uc9c0\ubc29\uc758 \ub0a8\uc11c\ubd80\uc5d0 \uc788\ub294 \uc77c\ubcf8\uc758 \uc218\ub3c4\uc774\ub2e4. 17\uc138\uae30 \ub2f9\uc2dc \uc5d0\ub3c4\ub77c\uace0 \ubd88\ub9ac\ub358 \uc774 \uc9c0\uc5ed\uc5d0 \ub9c9\ubd80\uac00 \ub4e4\uc5b4\uc120 \uc774\ud6c4 \uc77c\ubcf8\uc758 \uc815\uce58, \uacbd\uc81c, \ubb38\ud654, \uad50\ud1b5\uc758 \uc911\uc2ec\uc9c0\ub85c\uc11c \ubc1c\uc804\ud574 \uc654\uace0, \uc0b0\uc5c5\uacfc \uc778\uad6c\ub3c4 \uc9d1\uc911\ub418\uc5b4 \uc788\ub2e4. \uc804\uad6d\uc5d0\uc11c \uc138 \ubc88\uc9f8\ub85c \uba74\uc801\uc774 \uc791\uc740 \uc9c0\uc790\uccb4\ub85c\uc11c 2000m\uae09 \uc0b0\uc5d0\uc11c \uc624\uac00\uc0ac\uc640\ub77c \uc81c\ub3c4\uc640 \uac19\uc740 \uc544\uc5f4\ub300 \uc12c\ub4e4\uae4c\uc9c0 \ub2e4\uc591\ud55c \uc9c0\ud615 \ubd84\ud3ec\ub97c \uc9c0\ub2c8\uace0 \uc788\uc73c\uba70 \uc2e4\uc81c \uba74\uc801\uc740 \uc791\uc9c0\ub9cc \ubc94\uc704\ub294 \uc0c1\ub2f9\ud788 \ub113\uc5b4 \uc77c\ubcf8\uc758 \ucd5c\ub3d9\ub2e8\uacfc \ucd5c\ub0a8\ub2e8 \uc9c0\uc5ed\uc744 \ud3ec\ud568\ud558\uace0 \uc788\ub2e4. \uc778\uad6c\ub294 \uc57d 1,300\ub9cc \uba85\uc73c\ub85c \uc77c\ubcf8 \ucd1d \uc778\uad6c\uc758 10%\uc5d0 \ud574\ub2f9\ud55c\ub2e4. \uc774\ub7ec\ud55c \uc778\uad6c \uacfc\ubc00\uc744 \ub9c9\uae30 \uc704\ud574 \uc815\ubd80\ub294 \ubd80\ub3c4\uc2ec\uc758 \uac1c\ubc1c\uacfc \ub354\ubd88\uc5b4 \ub3c4\ucfc4 23\uad6c \uc774\uc678 \uc9c0\uc5ed\uc5d0 \uce68\uc0c1 \ub3c4\uc2dc\ub97c \uac74\uc124\ud558\uace0 \uc788\ub2e4. \uac04\ub2e4, \uc2e0\uc8fc\ucfe0, \uc544\ud0a4\ud558\ubc14\ub77c, \uc2dc\ubd80\uc57c, \uae34\uc790 \ub4f1\uc758 \uc5ec\ub7ec \uac70\ub9ac\uac00 \uc138\uacc4\uc801\uc73c\ub85c \uc54c\ub824\uc9c4 \uba85\uc18c\uc774\uba70 \ub3c4\ucfc4 \uc5ed \uc778\uadfc\uc758 \ub2c8\ud63c\ubc14\uc2dc\ub294 \ub3c4\ucfc4\ub97c \uae30\uc900\uc73c\ub85c \ud55c \uac70\ub9ac \uce21\uc815\uc758 \uae30\uc810\uc73c\ub85c \ud65c\uc6a9\ub418\uace0 \uc788\ub2e4. \ub3c4\ucfc4\ub294 \ud55c \ub54c \uc9c0\ub098\uce5c \uac1c\ubc1c\ub85c \uc778\ud574 \uaddc\uc288\uc640 \ub354\ubd88\uc5b4 \uac01\uc885 \ud658\uacbd \uc624\uc5fc\uc758 \uc628\uc0c1\uc774\uc5c8\uc73c\ub098, 1960\ub144\ub300 \uc774\ud6c4 \uc2a4\ubbf8\ub2e4 \uac15\uc758 \uc815\ud654 \ub4f1\uc744 \ube44\ub86f\ud55c \ub9ce\uc740 \ub178\ub825\uc744 \ud558\uace0 \uc788\ub2e4.", "answer": "\uc57d 1,300\ub9cc \uba85", "sentence": "\uc778\uad6c\ub294 \uc57d 1,300\ub9cc \uba85 \uc73c\ub85c \uc77c\ubcf8 \ucd1d \uc778\uad6c\uc758 10%\uc5d0 \ud574\ub2f9\ud55c\ub2e4.", "paragraph_sentence": "\ub3c4\ucfc4\ub294 \uc77c\ubcf8 \ud63c\uc288 \uc12c\uc758 \uc911\uc559, \uac04\ud1a0 \uc9c0\ubc29\uc758 \ub0a8\uc11c\ubd80\uc5d0 \uc788\ub294 \uc77c\ubcf8\uc758 \uc218\ub3c4\uc774\ub2e4. 17\uc138\uae30 \ub2f9\uc2dc \uc5d0\ub3c4\ub77c\uace0 \ubd88\ub9ac\ub358 \uc774 \uc9c0\uc5ed\uc5d0 \ub9c9\ubd80\uac00 \ub4e4\uc5b4\uc120 \uc774\ud6c4 \uc77c\ubcf8\uc758 \uc815\uce58, \uacbd\uc81c, \ubb38\ud654, \uad50\ud1b5\uc758 \uc911\uc2ec\uc9c0\ub85c\uc11c \ubc1c\uc804\ud574 \uc654\uace0, \uc0b0\uc5c5\uacfc \uc778\uad6c\ub3c4 \uc9d1\uc911\ub418\uc5b4 \uc788\ub2e4. \uc804\uad6d\uc5d0\uc11c \uc138 \ubc88\uc9f8\ub85c \uba74\uc801\uc774 \uc791\uc740 \uc9c0\uc790\uccb4\ub85c\uc11c 2000m\uae09 \uc0b0\uc5d0\uc11c \uc624\uac00\uc0ac\uc640\ub77c \uc81c\ub3c4\uc640 \uac19\uc740 \uc544\uc5f4\ub300 \uc12c\ub4e4\uae4c\uc9c0 \ub2e4\uc591\ud55c \uc9c0\ud615 \ubd84\ud3ec\ub97c \uc9c0\ub2c8\uace0 \uc788\uc73c\uba70 \uc2e4\uc81c \uba74\uc801\uc740 \uc791\uc9c0\ub9cc \ubc94\uc704\ub294 \uc0c1\ub2f9\ud788 \ub113\uc5b4 \uc77c\ubcf8\uc758 \ucd5c\ub3d9\ub2e8\uacfc \ucd5c\ub0a8\ub2e8 \uc9c0\uc5ed\uc744 \ud3ec\ud568\ud558\uace0 \uc788\ub2e4. \uc778\uad6c\ub294 \uc57d 1,300\ub9cc \uba85 \uc73c\ub85c \uc77c\ubcf8 \ucd1d \uc778\uad6c\uc758 10%\uc5d0 \ud574\ub2f9\ud55c\ub2e4. \uc774\ub7ec\ud55c \uc778\uad6c \uacfc\ubc00\uc744 \ub9c9\uae30 \uc704\ud574 \uc815\ubd80\ub294 \ubd80\ub3c4\uc2ec\uc758 \uac1c\ubc1c\uacfc \ub354\ubd88\uc5b4 \ub3c4\ucfc4 23\uad6c \uc774\uc678 \uc9c0\uc5ed\uc5d0 \uce68\uc0c1 \ub3c4\uc2dc\ub97c \uac74\uc124\ud558\uace0 \uc788\ub2e4. \uac04\ub2e4, \uc2e0\uc8fc\ucfe0, \uc544\ud0a4\ud558\ubc14\ub77c, \uc2dc\ubd80\uc57c, \uae34\uc790 \ub4f1\uc758 \uc5ec\ub7ec \uac70\ub9ac\uac00 \uc138\uacc4\uc801\uc73c\ub85c \uc54c\ub824\uc9c4 \uba85\uc18c\uc774\uba70 \ub3c4\ucfc4 \uc5ed \uc778\uadfc\uc758 \ub2c8\ud63c\ubc14\uc2dc\ub294 \ub3c4\ucfc4\ub97c \uae30\uc900\uc73c\ub85c \ud55c \uac70\ub9ac \uce21\uc815\uc758 \uae30\uc810\uc73c\ub85c \ud65c\uc6a9\ub418\uace0 \uc788\ub2e4. \ub3c4\ucfc4\ub294 \ud55c \ub54c \uc9c0\ub098\uce5c \uac1c\ubc1c\ub85c \uc778\ud574 \uaddc\uc288\uc640 \ub354\ubd88\uc5b4 \uac01\uc885 \ud658\uacbd \uc624\uc5fc\uc758 \uc628\uc0c1\uc774\uc5c8\uc73c\ub098, 1960\ub144\ub300 \uc774\ud6c4 \uc2a4\ubbf8\ub2e4 \uac15\uc758 \uc815\ud654 \ub4f1\uc744 \ube44\ub86f\ud55c \ub9ce\uc740 \ub178\ub825\uc744 \ud558\uace0 \uc788\ub2e4.", "paragraph_answer": "\ub3c4\ucfc4\ub294 \uc77c\ubcf8 \ud63c\uc288 \uc12c\uc758 \uc911\uc559, \uac04\ud1a0 \uc9c0\ubc29\uc758 \ub0a8\uc11c\ubd80\uc5d0 \uc788\ub294 \uc77c\ubcf8\uc758 \uc218\ub3c4\uc774\ub2e4. 17\uc138\uae30 \ub2f9\uc2dc \uc5d0\ub3c4\ub77c\uace0 \ubd88\ub9ac\ub358 \uc774 \uc9c0\uc5ed\uc5d0 \ub9c9\ubd80\uac00 \ub4e4\uc5b4\uc120 \uc774\ud6c4 \uc77c\ubcf8\uc758 \uc815\uce58, \uacbd\uc81c, \ubb38\ud654, \uad50\ud1b5\uc758 \uc911\uc2ec\uc9c0\ub85c\uc11c \ubc1c\uc804\ud574 \uc654\uace0, \uc0b0\uc5c5\uacfc \uc778\uad6c\ub3c4 \uc9d1\uc911\ub418\uc5b4 \uc788\ub2e4. \uc804\uad6d\uc5d0\uc11c \uc138 \ubc88\uc9f8\ub85c \uba74\uc801\uc774 \uc791\uc740 \uc9c0\uc790\uccb4\ub85c\uc11c 2000m\uae09 \uc0b0\uc5d0\uc11c \uc624\uac00\uc0ac\uc640\ub77c \uc81c\ub3c4\uc640 \uac19\uc740 \uc544\uc5f4\ub300 \uc12c\ub4e4\uae4c\uc9c0 \ub2e4\uc591\ud55c \uc9c0\ud615 \ubd84\ud3ec\ub97c \uc9c0\ub2c8\uace0 \uc788\uc73c\uba70 \uc2e4\uc81c \uba74\uc801\uc740 \uc791\uc9c0\ub9cc \ubc94\uc704\ub294 \uc0c1\ub2f9\ud788 \ub113\uc5b4 \uc77c\ubcf8\uc758 \ucd5c\ub3d9\ub2e8\uacfc \ucd5c\ub0a8\ub2e8 \uc9c0\uc5ed\uc744 \ud3ec\ud568\ud558\uace0 \uc788\ub2e4. \uc778\uad6c\ub294 \uc57d 1,300\ub9cc \uba85 \uc73c\ub85c \uc77c\ubcf8 \ucd1d \uc778\uad6c\uc758 10%\uc5d0 \ud574\ub2f9\ud55c\ub2e4. \uc774\ub7ec\ud55c \uc778\uad6c \uacfc\ubc00\uc744 \ub9c9\uae30 \uc704\ud574 \uc815\ubd80\ub294 \ubd80\ub3c4\uc2ec\uc758 \uac1c\ubc1c\uacfc \ub354\ubd88\uc5b4 \ub3c4\ucfc4 23\uad6c \uc774\uc678 \uc9c0\uc5ed\uc5d0 \uce68\uc0c1 \ub3c4\uc2dc\ub97c \uac74\uc124\ud558\uace0 \uc788\ub2e4. \uac04\ub2e4, \uc2e0\uc8fc\ucfe0, \uc544\ud0a4\ud558\ubc14\ub77c, \uc2dc\ubd80\uc57c, \uae34\uc790 \ub4f1\uc758 \uc5ec\ub7ec \uac70\ub9ac\uac00 \uc138\uacc4\uc801\uc73c\ub85c \uc54c\ub824\uc9c4 \uba85\uc18c\uc774\uba70 \ub3c4\ucfc4 \uc5ed \uc778\uadfc\uc758 \ub2c8\ud63c\ubc14\uc2dc\ub294 \ub3c4\ucfc4\ub97c \uae30\uc900\uc73c\ub85c \ud55c \uac70\ub9ac \uce21\uc815\uc758 \uae30\uc810\uc73c\ub85c \ud65c\uc6a9\ub418\uace0 \uc788\ub2e4. \ub3c4\ucfc4\ub294 \ud55c \ub54c \uc9c0\ub098\uce5c \uac1c\ubc1c\ub85c \uc778\ud574 \uaddc\uc288\uc640 \ub354\ubd88\uc5b4 \uac01\uc885 \ud658\uacbd \uc624\uc5fc\uc758 \uc628\uc0c1\uc774\uc5c8\uc73c\ub098, 1960\ub144\ub300 \uc774\ud6c4 \uc2a4\ubbf8\ub2e4 \uac15\uc758 \uc815\ud654 \ub4f1\uc744 \ube44\ub86f\ud55c \ub9ce\uc740 \ub178\ub825\uc744 \ud558\uace0 \uc788\ub2e4.", "sentence_answer": "\uc778\uad6c\ub294 \uc57d 1,300\ub9cc \uba85 \uc73c\ub85c \uc77c\ubcf8 \ucd1d \uc778\uad6c\uc758 10%\uc5d0 \ud574\ub2f9\ud55c\ub2e4.", "paragraph_id": "6657580-17-1", "paragraph_question": "question: \uc77c\ubcf8 \ub3c4\ucfc4\uc758 \uc778\uad6c\ub294?, context: \ub3c4\ucfc4\ub294 \uc77c\ubcf8 \ud63c\uc288 \uc12c\uc758 \uc911\uc559, \uac04\ud1a0 \uc9c0\ubc29\uc758 \ub0a8\uc11c\ubd80\uc5d0 \uc788\ub294 \uc77c\ubcf8\uc758 \uc218\ub3c4\uc774\ub2e4. 17\uc138\uae30 \ub2f9\uc2dc \uc5d0\ub3c4\ub77c\uace0 \ubd88\ub9ac\ub358 \uc774 \uc9c0\uc5ed\uc5d0 \ub9c9\ubd80\uac00 \ub4e4\uc5b4\uc120 \uc774\ud6c4 \uc77c\ubcf8\uc758 \uc815\uce58, \uacbd\uc81c, \ubb38\ud654, \uad50\ud1b5\uc758 \uc911\uc2ec\uc9c0\ub85c\uc11c \ubc1c\uc804\ud574 \uc654\uace0, \uc0b0\uc5c5\uacfc \uc778\uad6c\ub3c4 \uc9d1\uc911\ub418\uc5b4 \uc788\ub2e4. \uc804\uad6d\uc5d0\uc11c \uc138 \ubc88\uc9f8\ub85c \uba74\uc801\uc774 \uc791\uc740 \uc9c0\uc790\uccb4\ub85c\uc11c 2000m\uae09 \uc0b0\uc5d0\uc11c \uc624\uac00\uc0ac\uc640\ub77c \uc81c\ub3c4\uc640 \uac19\uc740 \uc544\uc5f4\ub300 \uc12c\ub4e4\uae4c\uc9c0 \ub2e4\uc591\ud55c \uc9c0\ud615 \ubd84\ud3ec\ub97c \uc9c0\ub2c8\uace0 \uc788\uc73c\uba70 \uc2e4\uc81c \uba74\uc801\uc740 \uc791\uc9c0\ub9cc \ubc94\uc704\ub294 \uc0c1\ub2f9\ud788 \ub113\uc5b4 \uc77c\ubcf8\uc758 \ucd5c\ub3d9\ub2e8\uacfc \ucd5c\ub0a8\ub2e8 \uc9c0\uc5ed\uc744 \ud3ec\ud568\ud558\uace0 \uc788\ub2e4. \uc778\uad6c\ub294 \uc57d 1,300\ub9cc \uba85\uc73c\ub85c \uc77c\ubcf8 \ucd1d \uc778\uad6c\uc758 10%\uc5d0 \ud574\ub2f9\ud55c\ub2e4. \uc774\ub7ec\ud55c \uc778\uad6c \uacfc\ubc00\uc744 \ub9c9\uae30 \uc704\ud574 \uc815\ubd80\ub294 \ubd80\ub3c4\uc2ec\uc758 \uac1c\ubc1c\uacfc \ub354\ubd88\uc5b4 \ub3c4\ucfc4 23\uad6c \uc774\uc678 \uc9c0\uc5ed\uc5d0 \uce68\uc0c1 \ub3c4\uc2dc\ub97c \uac74\uc124\ud558\uace0 \uc788\ub2e4. \uac04\ub2e4, \uc2e0\uc8fc\ucfe0, \uc544\ud0a4\ud558\ubc14\ub77c, \uc2dc\ubd80\uc57c, \uae34\uc790 \ub4f1\uc758 \uc5ec\ub7ec \uac70\ub9ac\uac00 \uc138\uacc4\uc801\uc73c\ub85c \uc54c\ub824\uc9c4 \uba85\uc18c\uc774\uba70 \ub3c4\ucfc4 \uc5ed \uc778\uadfc\uc758 \ub2c8\ud63c\ubc14\uc2dc\ub294 \ub3c4\ucfc4\ub97c \uae30\uc900\uc73c\ub85c \ud55c \uac70\ub9ac \uce21\uc815\uc758 \uae30\uc810\uc73c\ub85c \ud65c\uc6a9\ub418\uace0 \uc788\ub2e4. \ub3c4\ucfc4\ub294 \ud55c \ub54c \uc9c0\ub098\uce5c \uac1c\ubc1c\ub85c \uc778\ud574 \uaddc\uc288\uc640 \ub354\ubd88\uc5b4 \uac01\uc885 \ud658\uacbd \uc624\uc5fc\uc758 \uc628\uc0c1\uc774\uc5c8\uc73c\ub098, 1960\ub144\ub300 \uc774\ud6c4 \uc2a4\ubbf8\ub2e4 \uac15\uc758 \uc815\ud654 \ub4f1\uc744 \ube44\ub86f\ud55c \ub9ce\uc740 \ub178\ub825\uc744 \ud558\uace0 \uc788\ub2e4."} {"question": "\ub304\uc2a4 \uc544\uce74\ub370\ubbf8\ub97c \uc124\ub9bd\ud55c \uba64\ubc84\uc758 \uc774\ub984\uc740?", "paragraph": "2014\ub144 12\uc6d4 21\uc77c SBS \uac00\uc694\ub300\uc804\uc744 \ub9c8\uc9c0\ub9c9\uc73c\ub85c 2NE1\uc740 \uba64\ubc84\ubcc4 \uac1c\uc778\ud65c\ub3d9\uc5d0 \uc9d1\uc911\ud558\uba74\uc11c, \ud574\uccb4\uc124\uc774 \ub3cc\uae30 \uc2dc\uc791\ud588\ub2e4. \ub17c\ub780\uc744 \uc77c\uc73c\ucf30\ub358 \uba64\ubc84 \ubc15\ubd04\uc740 \uc790\uc219, \uba64\ubc84 \uc0b0\ub2e4\ub77c\ubc15\uc740 \uc5f0\uae30 \ud65c\ub3d9, \uc528\uc5d8\uc740 \ubbf8\uad6d\uc9c4\ucd9c\uc744 \uc704\ud574 YG\uc5d4\ud130\ud14c\uc778\uba3c\ud2b8\uc640 \uc7ac\uacc4\uc57d\uc744 1\ub144 \uc55e\ub2f9\uae30\uace0 \uc74c\uc545 \uc791\uc5c5\uc5d0 \ub3cc\uc785, \uacf5\ubbfc\uc9c0\ub294 \ub304\uc2a4 \uc544\uce74\ub370\ubbf8\ub97c \uc124\ub9bd\ud588\ub2e4. \ubcc4\ub2e4\ub978 \uc18c\uc2dd\uc774 \uc5c6\ub358 \ub3c4\uc911 2015\ub144 11\uc6d4 9\uc77c \uc18c\uc18d\uc0ac YG\uc5d4\ud130\ud14c\uc778\uba3c\ud2b8\ub294 \ud2f0\uc800 \uc774\ubbf8\uc9c0\ub97c \uacf5\uac1c\ud55c\ub2e4. \uacf5\uac1c\ub41c \ud2f0\uc800 \uc774\ubbf8\uc9c0 \uc18d\uc5d0\ub294 \uc22b\uc790 21\uc774 \ubc18\ubcf5\ub3fc 2NE1\uc758 \uc644\uc804\uccb4 \ucef4\ubc31\uc774 \uc544\ub2c8\ub0d0\ub294 \ucd94\uce21\uc774 \ub098\uc654\ub294\ub370, \uba64\ubc84 \uc528\uc5d8\uc758 \ubbf8\uad6d \ub370\ubdd4\uc568\ubc94 \uc218\ub85d\uace1 \"Hello Bitches\" \uc120\uacf5\uac1c\ub85c \ubc1d\ud600\uc84c\ub2e4. YG\uc5d4\ud130\ud14c\uc778\uba3c\ud2b8\ub294 2015 MAMA \uc5d0\uc11c \"Hello Bitches\"\uc758 \uccab \ubb34\ub300\ub97c \uac00\uc9c8 \uac83\uc774\ub77c\ub294 \uacf5\uc2dd \uc785\uc7a5\uc744 \ub0b4\ub193\uace0, \uc528\uc5d8\uc740 \ub9c8\ub9c8\uc5d0\uc11c \ub2e8\ub3c5 \ubb34\ub300\ub97c \uac00\uc84c\ub2e4. \uc528\uc5d8\uc758 \ubb34\ub300\uac00 \ub05d\ub09c \ud6c4, \uc608\uace0\ub3c4 \uc5c6\uc774 2NE1 \uba64\ubc84 \uc804\uc6d0\uc774 \uae5c\uc9dd \ucd9c\uc5f0\ud574 \ub370\ubdd4\uace1\"Fire\"\uc640 \ub300\ud45c\uace1 \"\ub0b4\uac00 \uc81c\uc77c \uc798 \ub098\uac00\"\ub97c \ubd88\ub800\ub2e4. \uc774 \ubb34\ub300\uc5d0\ub294 \uc790\uc219\uc911\uc774\ub358 \uba64\ubc84 \ubc15\ubd04\ub3c4 \ucd9c\uc5f0\ud574, \uc57d 1\ub144 8\uac1c\uc6d4 \ub9cc\uc5d0 \ubc29\uc1a1\uc5d0\uc11c 4\uba85 \ubaa8\ub450\uac00 \ud568\uaed8\ud55c \ubb34\ub300\ub97c \ubcf4\uc5ec\uc8fc\uc5c8\ub2e4.", "answer": "\uacf5\ubbfc\uc9c0", "sentence": "\ub17c\ub780\uc744 \uc77c\uc73c\ucf30\ub358 \uba64\ubc84 \ubc15\ubd04\uc740 \uc790\uc219, \uba64\ubc84 \uc0b0\ub2e4\ub77c\ubc15\uc740 \uc5f0\uae30 \ud65c\ub3d9, \uc528\uc5d8\uc740 \ubbf8\uad6d\uc9c4\ucd9c\uc744 \uc704\ud574 YG\uc5d4\ud130\ud14c\uc778\uba3c\ud2b8\uc640 \uc7ac\uacc4\uc57d\uc744 1\ub144 \uc55e\ub2f9\uae30\uace0 \uc74c\uc545 \uc791\uc5c5\uc5d0 \ub3cc\uc785, \uacf5\ubbfc\uc9c0 \ub294 \ub304\uc2a4 \uc544\uce74\ub370\ubbf8\ub97c \uc124\ub9bd\ud588\ub2e4.", "paragraph_sentence": "2014\ub144 12\uc6d4 21\uc77c SBS \uac00\uc694\ub300\uc804\uc744 \ub9c8\uc9c0\ub9c9\uc73c\ub85c 2NE1\uc740 \uba64\ubc84\ubcc4 \uac1c\uc778\ud65c\ub3d9\uc5d0 \uc9d1\uc911\ud558\uba74\uc11c, \ud574\uccb4\uc124\uc774 \ub3cc\uae30 \uc2dc\uc791\ud588\ub2e4. \ub17c\ub780\uc744 \uc77c\uc73c\ucf30\ub358 \uba64\ubc84 \ubc15\ubd04\uc740 \uc790\uc219, \uba64\ubc84 \uc0b0\ub2e4\ub77c\ubc15\uc740 \uc5f0\uae30 \ud65c\ub3d9, \uc528\uc5d8\uc740 \ubbf8\uad6d\uc9c4\ucd9c\uc744 \uc704\ud574 YG\uc5d4\ud130\ud14c\uc778\uba3c\ud2b8\uc640 \uc7ac\uacc4\uc57d\uc744 1\ub144 \uc55e\ub2f9\uae30\uace0 \uc74c\uc545 \uc791\uc5c5\uc5d0 \ub3cc\uc785, \uacf5\ubbfc\uc9c0 \ub294 \ub304\uc2a4 \uc544\uce74\ub370\ubbf8\ub97c \uc124\ub9bd\ud588\ub2e4. \ubcc4\ub2e4\ub978 \uc18c\uc2dd\uc774 \uc5c6\ub358 \ub3c4\uc911 2015\ub144 11\uc6d4 9\uc77c \uc18c\uc18d\uc0ac YG\uc5d4\ud130\ud14c\uc778\uba3c\ud2b8\ub294 \ud2f0\uc800 \uc774\ubbf8\uc9c0\ub97c \uacf5\uac1c\ud55c\ub2e4. \uacf5\uac1c\ub41c \ud2f0\uc800 \uc774\ubbf8\uc9c0 \uc18d\uc5d0\ub294 \uc22b\uc790 21\uc774 \ubc18\ubcf5\ub3fc 2NE1\uc758 \uc644\uc804\uccb4 \ucef4\ubc31\uc774 \uc544\ub2c8\ub0d0\ub294 \ucd94\uce21\uc774 \ub098\uc654\ub294\ub370, \uba64\ubc84 \uc528\uc5d8\uc758 \ubbf8\uad6d \ub370\ubdd4\uc568\ubc94 \uc218\ub85d\uace1 \"Hello Bitches\" \uc120\uacf5\uac1c\ub85c \ubc1d\ud600\uc84c\ub2e4. YG\uc5d4\ud130\ud14c\uc778\uba3c\ud2b8\ub294 2015 MAMA \uc5d0\uc11c \"Hello Bitches\"\uc758 \uccab \ubb34\ub300\ub97c \uac00\uc9c8 \uac83\uc774\ub77c\ub294 \uacf5\uc2dd \uc785\uc7a5\uc744 \ub0b4\ub193\uace0, \uc528\uc5d8\uc740 \ub9c8\ub9c8\uc5d0\uc11c \ub2e8\ub3c5 \ubb34\ub300\ub97c \uac00\uc84c\ub2e4. \uc528\uc5d8\uc758 \ubb34\ub300\uac00 \ub05d\ub09c \ud6c4, \uc608\uace0\ub3c4 \uc5c6\uc774 2NE1 \uba64\ubc84 \uc804\uc6d0\uc774 \uae5c\uc9dd \ucd9c\uc5f0\ud574 \ub370\ubdd4\uace1\"Fire\"\uc640 \ub300\ud45c\uace1 \"\ub0b4\uac00 \uc81c\uc77c \uc798 \ub098\uac00\"\ub97c \ubd88\ub800\ub2e4. \uc774 \ubb34\ub300\uc5d0\ub294 \uc790\uc219\uc911\uc774\ub358 \uba64\ubc84 \ubc15\ubd04\ub3c4 \ucd9c\uc5f0\ud574, \uc57d 1\ub144 8\uac1c\uc6d4 \ub9cc\uc5d0 \ubc29\uc1a1\uc5d0\uc11c 4\uba85 \ubaa8\ub450\uac00 \ud568\uaed8\ud55c \ubb34\ub300\ub97c \ubcf4\uc5ec\uc8fc\uc5c8\ub2e4.", "paragraph_answer": "2014\ub144 12\uc6d4 21\uc77c SBS \uac00\uc694\ub300\uc804\uc744 \ub9c8\uc9c0\ub9c9\uc73c\ub85c 2NE1\uc740 \uba64\ubc84\ubcc4 \uac1c\uc778\ud65c\ub3d9\uc5d0 \uc9d1\uc911\ud558\uba74\uc11c, \ud574\uccb4\uc124\uc774 \ub3cc\uae30 \uc2dc\uc791\ud588\ub2e4. \ub17c\ub780\uc744 \uc77c\uc73c\ucf30\ub358 \uba64\ubc84 \ubc15\ubd04\uc740 \uc790\uc219, \uba64\ubc84 \uc0b0\ub2e4\ub77c\ubc15\uc740 \uc5f0\uae30 \ud65c\ub3d9, \uc528\uc5d8\uc740 \ubbf8\uad6d\uc9c4\ucd9c\uc744 \uc704\ud574 YG\uc5d4\ud130\ud14c\uc778\uba3c\ud2b8\uc640 \uc7ac\uacc4\uc57d\uc744 1\ub144 \uc55e\ub2f9\uae30\uace0 \uc74c\uc545 \uc791\uc5c5\uc5d0 \ub3cc\uc785, \uacf5\ubbfc\uc9c0 \ub294 \ub304\uc2a4 \uc544\uce74\ub370\ubbf8\ub97c \uc124\ub9bd\ud588\ub2e4. \ubcc4\ub2e4\ub978 \uc18c\uc2dd\uc774 \uc5c6\ub358 \ub3c4\uc911 2015\ub144 11\uc6d4 9\uc77c \uc18c\uc18d\uc0ac YG\uc5d4\ud130\ud14c\uc778\uba3c\ud2b8\ub294 \ud2f0\uc800 \uc774\ubbf8\uc9c0\ub97c \uacf5\uac1c\ud55c\ub2e4. \uacf5\uac1c\ub41c \ud2f0\uc800 \uc774\ubbf8\uc9c0 \uc18d\uc5d0\ub294 \uc22b\uc790 21\uc774 \ubc18\ubcf5\ub3fc 2NE1\uc758 \uc644\uc804\uccb4 \ucef4\ubc31\uc774 \uc544\ub2c8\ub0d0\ub294 \ucd94\uce21\uc774 \ub098\uc654\ub294\ub370, \uba64\ubc84 \uc528\uc5d8\uc758 \ubbf8\uad6d \ub370\ubdd4\uc568\ubc94 \uc218\ub85d\uace1 \"Hello Bitches\" \uc120\uacf5\uac1c\ub85c \ubc1d\ud600\uc84c\ub2e4. YG\uc5d4\ud130\ud14c\uc778\uba3c\ud2b8\ub294 2015 MAMA \uc5d0\uc11c \"Hello Bitches\"\uc758 \uccab \ubb34\ub300\ub97c \uac00\uc9c8 \uac83\uc774\ub77c\ub294 \uacf5\uc2dd \uc785\uc7a5\uc744 \ub0b4\ub193\uace0, \uc528\uc5d8\uc740 \ub9c8\ub9c8\uc5d0\uc11c \ub2e8\ub3c5 \ubb34\ub300\ub97c \uac00\uc84c\ub2e4. \uc528\uc5d8\uc758 \ubb34\ub300\uac00 \ub05d\ub09c \ud6c4, \uc608\uace0\ub3c4 \uc5c6\uc774 2NE1 \uba64\ubc84 \uc804\uc6d0\uc774 \uae5c\uc9dd \ucd9c\uc5f0\ud574 \ub370\ubdd4\uace1\"Fire\"\uc640 \ub300\ud45c\uace1 \"\ub0b4\uac00 \uc81c\uc77c \uc798 \ub098\uac00\"\ub97c \ubd88\ub800\ub2e4. \uc774 \ubb34\ub300\uc5d0\ub294 \uc790\uc219\uc911\uc774\ub358 \uba64\ubc84 \ubc15\ubd04\ub3c4 \ucd9c\uc5f0\ud574, \uc57d 1\ub144 8\uac1c\uc6d4 \ub9cc\uc5d0 \ubc29\uc1a1\uc5d0\uc11c 4\uba85 \ubaa8\ub450\uac00 \ud568\uaed8\ud55c \ubb34\ub300\ub97c \ubcf4\uc5ec\uc8fc\uc5c8\ub2e4.", "sentence_answer": "\ub17c\ub780\uc744 \uc77c\uc73c\ucf30\ub358 \uba64\ubc84 \ubc15\ubd04\uc740 \uc790\uc219, \uba64\ubc84 \uc0b0\ub2e4\ub77c\ubc15\uc740 \uc5f0\uae30 \ud65c\ub3d9, \uc528\uc5d8\uc740 \ubbf8\uad6d\uc9c4\ucd9c\uc744 \uc704\ud574 YG\uc5d4\ud130\ud14c\uc778\uba3c\ud2b8\uc640 \uc7ac\uacc4\uc57d\uc744 1\ub144 \uc55e\ub2f9\uae30\uace0 \uc74c\uc545 \uc791\uc5c5\uc5d0 \ub3cc\uc785, \uacf5\ubbfc\uc9c0 \ub294 \ub304\uc2a4 \uc544\uce74\ub370\ubbf8\ub97c \uc124\ub9bd\ud588\ub2e4.", "paragraph_id": "6471356-8-0", "paragraph_question": "question: \ub304\uc2a4 \uc544\uce74\ub370\ubbf8\ub97c \uc124\ub9bd\ud55c \uba64\ubc84\uc758 \uc774\ub984\uc740?, context: 2014\ub144 12\uc6d4 21\uc77c SBS \uac00\uc694\ub300\uc804\uc744 \ub9c8\uc9c0\ub9c9\uc73c\ub85c 2NE1\uc740 \uba64\ubc84\ubcc4 \uac1c\uc778\ud65c\ub3d9\uc5d0 \uc9d1\uc911\ud558\uba74\uc11c, \ud574\uccb4\uc124\uc774 \ub3cc\uae30 \uc2dc\uc791\ud588\ub2e4. \ub17c\ub780\uc744 \uc77c\uc73c\ucf30\ub358 \uba64\ubc84 \ubc15\ubd04\uc740 \uc790\uc219, \uba64\ubc84 \uc0b0\ub2e4\ub77c\ubc15\uc740 \uc5f0\uae30 \ud65c\ub3d9, \uc528\uc5d8\uc740 \ubbf8\uad6d\uc9c4\ucd9c\uc744 \uc704\ud574 YG\uc5d4\ud130\ud14c\uc778\uba3c\ud2b8\uc640 \uc7ac\uacc4\uc57d\uc744 1\ub144 \uc55e\ub2f9\uae30\uace0 \uc74c\uc545 \uc791\uc5c5\uc5d0 \ub3cc\uc785, \uacf5\ubbfc\uc9c0\ub294 \ub304\uc2a4 \uc544\uce74\ub370\ubbf8\ub97c \uc124\ub9bd\ud588\ub2e4. \ubcc4\ub2e4\ub978 \uc18c\uc2dd\uc774 \uc5c6\ub358 \ub3c4\uc911 2015\ub144 11\uc6d4 9\uc77c \uc18c\uc18d\uc0ac YG\uc5d4\ud130\ud14c\uc778\uba3c\ud2b8\ub294 \ud2f0\uc800 \uc774\ubbf8\uc9c0\ub97c \uacf5\uac1c\ud55c\ub2e4. \uacf5\uac1c\ub41c \ud2f0\uc800 \uc774\ubbf8\uc9c0 \uc18d\uc5d0\ub294 \uc22b\uc790 21\uc774 \ubc18\ubcf5\ub3fc 2NE1\uc758 \uc644\uc804\uccb4 \ucef4\ubc31\uc774 \uc544\ub2c8\ub0d0\ub294 \ucd94\uce21\uc774 \ub098\uc654\ub294\ub370, \uba64\ubc84 \uc528\uc5d8\uc758 \ubbf8\uad6d \ub370\ubdd4\uc568\ubc94 \uc218\ub85d\uace1 \"Hello Bitches\" \uc120\uacf5\uac1c\ub85c \ubc1d\ud600\uc84c\ub2e4. YG\uc5d4\ud130\ud14c\uc778\uba3c\ud2b8\ub294 2015 MAMA \uc5d0\uc11c \"Hello Bitches\"\uc758 \uccab \ubb34\ub300\ub97c \uac00\uc9c8 \uac83\uc774\ub77c\ub294 \uacf5\uc2dd \uc785\uc7a5\uc744 \ub0b4\ub193\uace0, \uc528\uc5d8\uc740 \ub9c8\ub9c8\uc5d0\uc11c \ub2e8\ub3c5 \ubb34\ub300\ub97c \uac00\uc84c\ub2e4. \uc528\uc5d8\uc758 \ubb34\ub300\uac00 \ub05d\ub09c \ud6c4, \uc608\uace0\ub3c4 \uc5c6\uc774 2NE1 \uba64\ubc84 \uc804\uc6d0\uc774 \uae5c\uc9dd \ucd9c\uc5f0\ud574 \ub370\ubdd4\uace1\"Fire\"\uc640 \ub300\ud45c\uace1 \"\ub0b4\uac00 \uc81c\uc77c \uc798 \ub098\uac00\"\ub97c \ubd88\ub800\ub2e4. \uc774 \ubb34\ub300\uc5d0\ub294 \uc790\uc219\uc911\uc774\ub358 \uba64\ubc84 \ubc15\ubd04\ub3c4 \ucd9c\uc5f0\ud574, \uc57d 1\ub144 8\uac1c\uc6d4 \ub9cc\uc5d0 \ubc29\uc1a1\uc5d0\uc11c 4\uba85 \ubaa8\ub450\uac00 \ud568\uaed8\ud55c \ubb34\ub300\ub97c \ubcf4\uc5ec\uc8fc\uc5c8\ub2e4."} {"question": "\uc7a5\uc774\uba38\uc6b0\uc758 \uc778\uc5f0\uc740 \uc5b8\uc81c \ubc1c\ud45c\ub418\uc5c8\ub294\uac00?", "paragraph": "2000\ub144 \ub300 \uc774\ud6c4, \uc7a5\uc774\uba38\uc6b0\ub294 \ub9ac\uc5bc\ub9ac\uc998 \ubc14\ud0d5\uc758 \ubbfc\uc911\uc758 \uc0b6\uc744 \uadf8\ub9ac\ub294 \uac83\ubcf4\ub2e4 \uba3c \uacfc\uac70\uc758 \uc911\uad6d \uc655\uc870 \uc2dc\ub300\ub85c \uce74\uba54\ub77c\uc758 \ucd08\uc810\uc744 \ubc14\uafbc\ub2e4. \uadf8\ub9ac\uace0 \uc601\uc0c1\uc5d0 \uc788\uc5b4 \uacfc\uac70\uc640\ub294 \ub9e4\uc6b0 \ucc28\ubcc4\ud654\ub418\ub294 \uc2a4\ud0c0\uc77c\uc758 \ubcc0\ud654\uac00 \uc0dd\uacbc\ub2e4. \uacfc\uac70 \uc911\uad6d \uc655\uc870 \uc2dc\ub300\ub97c \ubc30\uacbd\uc73c\ub85c \ubb34\ud611\uc801\uc778 \uc694\uc18c\ub97c \ub3c4\uc785\ud558\uc5ec \ud560\ub9ac\uc6b0\ub4dc\uc640 \uac19\uc740 \uc2a4\ud399\ud130\ud074\ud55c \uc7a5\uba74\ub4e4\uc744 \uc5f0\ucd9c\ud558\uace0 \uc601\uc0c1\ubbf8\ub97c \uadf9\ub300\ud654\ud558\uc600\ub2e4. 2002\ub144 \u300a\uc601\uc6c5\u300b\uc740 \uc774\ub7ec\ud55c \uacbd\ud5a5\uc744 \uc798 \ub098\ud0c0\ub0b4\ub294 \uc791\ud488\uc73c\ub85c \uc804\uad6d\uc2dc\ub300 \ub9d0, \uc9c4\uc2dc\ud669 \uc2dc\ub300\ub97c \ubc30\uacbd\uc73c\ub85c \uc9c4\uc2dc\ud669\uc744 \uc554\uc0b4\ud558\ub824\uace0 \ud558\ub294 \ud611\uac1d\ub4e4\uc758 \uc774\uc57c\uae30\uc774\ub2e4. \u300a\uc601\uc6c5\u300b\uc740 \uc911\uad6d\uacfc \ud574\uc678\uc5d0\uc11c \uc0c1\uc5c5\uc601\ud654\ub85c\uc11c \uacf5\uc804\uc758 \ud788\ud2b8\ub97c \uae30\ub85d\ud558\uc600\uace0, 2003\ub144 \uc544\uce74\ub370\ubbf8 \uc218\uc0c1\uc744 \ud558\uc600\ub2e4. \ub4a4\uc774\uc5b4 \u300a\uc778\uc5f0\u300b(2004)\uacfc \u300a\ud669\ud6c4\ud654\u300b(2006)\uc5d0\uc11c\ub3c4 \uacfc\uac70 \uc911\uad6d \uc655\uc870\ub97c \ubc30\uacbd\uc73c\ub85c \uc0c9\ucc44\uc911\uc2ec\uc758 \ud654\ub824\ud55c \uc601\uc0c1\ubbf8\ub97c \uc120\ubcf4\uc600\ub2e4. \uc774 \uc2dc\uae30\uc758 \uc791\ud488\uc740 \uc601\uc0c1\ubbf8\ud559\uc801\uc778 \uce21\uba74\uc5d0\uc11c\ub294 \uc804\uc791\ub4e4\uacfc \ube44\uad50\ud558\uc5ec \ube44\uc57d\uc801\uc778 \ubc1c\uc804\uc774 \uc788\uc5c8\uc9c0\ub9cc, \ub0b4\uc6a9\uc801\uc778 \uce21\uba74\uc5d0\uc11c\ub294 \uc18c\uc2dc\ubbfc\uc758 \uc77c\uc0c1\uc5d0 \ucd08\uc810\uc744 \ub9de\ucd94\uc5b4 \uc601\uc0c1\uc744 \ub2f4\uc544\ub0b4\ub358 \uacfc\uac70\uc640\ub294 \ub2e4\ub974\uac8c \uad6d\uac00\uad8c\ub825\uc744 \uc639\ud638\ud558\ub294 \ubaa8\uc2b5\uc744 \ubcf4\uc774\uae30\ub3c4 \ud558\uc600\ub2e4.", "answer": "2004", "sentence": "\ub4a4\uc774\uc5b4 \u300a\uc778\uc5f0\u300b( 2004 )", "paragraph_sentence": "2000\ub144 \ub300 \uc774\ud6c4, \uc7a5\uc774\uba38\uc6b0\ub294 \ub9ac\uc5bc\ub9ac\uc998 \ubc14\ud0d5\uc758 \ubbfc\uc911\uc758 \uc0b6\uc744 \uadf8\ub9ac\ub294 \uac83\ubcf4\ub2e4 \uba3c \uacfc\uac70\uc758 \uc911\uad6d \uc655\uc870 \uc2dc\ub300\ub85c \uce74\uba54\ub77c\uc758 \ucd08\uc810\uc744 \ubc14\uafbc\ub2e4. \uadf8\ub9ac\uace0 \uc601\uc0c1\uc5d0 \uc788\uc5b4 \uacfc\uac70\uc640\ub294 \ub9e4\uc6b0 \ucc28\ubcc4\ud654\ub418\ub294 \uc2a4\ud0c0\uc77c\uc758 \ubcc0\ud654\uac00 \uc0dd\uacbc\ub2e4. \uacfc\uac70 \uc911\uad6d \uc655\uc870 \uc2dc\ub300\ub97c \ubc30\uacbd\uc73c\ub85c \ubb34\ud611\uc801\uc778 \uc694\uc18c\ub97c \ub3c4\uc785\ud558\uc5ec \ud560\ub9ac\uc6b0\ub4dc\uc640 \uac19\uc740 \uc2a4\ud399\ud130\ud074\ud55c \uc7a5\uba74\ub4e4\uc744 \uc5f0\ucd9c\ud558\uace0 \uc601\uc0c1\ubbf8\ub97c \uadf9\ub300\ud654\ud558\uc600\ub2e4. 2002\ub144 \u300a\uc601\uc6c5\u300b\uc740 \uc774\ub7ec\ud55c \uacbd\ud5a5\uc744 \uc798 \ub098\ud0c0\ub0b4\ub294 \uc791\ud488\uc73c\ub85c \uc804\uad6d\uc2dc\ub300 \ub9d0, \uc9c4\uc2dc\ud669 \uc2dc\ub300\ub97c \ubc30\uacbd\uc73c\ub85c \uc9c4\uc2dc\ud669\uc744 \uc554\uc0b4\ud558\ub824\uace0 \ud558\ub294 \ud611\uac1d\ub4e4\uc758 \uc774\uc57c\uae30\uc774\ub2e4. \u300a\uc601\uc6c5\u300b\uc740 \uc911\uad6d\uacfc \ud574\uc678\uc5d0\uc11c \uc0c1\uc5c5\uc601\ud654\ub85c\uc11c \uacf5\uc804\uc758 \ud788\ud2b8\ub97c \uae30\ub85d\ud558\uc600\uace0, 2003\ub144 \uc544\uce74\ub370\ubbf8 \uc218\uc0c1\uc744 \ud558\uc600\ub2e4. \ub4a4\uc774\uc5b4 \u300a\uc778\uc5f0\u300b( 2004 ) \uacfc \u300a\ud669\ud6c4\ud654\u300b(2006)\uc5d0\uc11c\ub3c4 \uacfc\uac70 \uc911\uad6d \uc655\uc870\ub97c \ubc30\uacbd\uc73c\ub85c \uc0c9\ucc44\uc911\uc2ec\uc758 \ud654\ub824\ud55c \uc601\uc0c1\ubbf8\ub97c \uc120\ubcf4\uc600\ub2e4. \uc774 \uc2dc\uae30\uc758 \uc791\ud488\uc740 \uc601\uc0c1\ubbf8\ud559\uc801\uc778 \uce21\uba74\uc5d0\uc11c\ub294 \uc804\uc791\ub4e4\uacfc \ube44\uad50\ud558\uc5ec \ube44\uc57d\uc801\uc778 \ubc1c\uc804\uc774 \uc788\uc5c8\uc9c0\ub9cc, \ub0b4\uc6a9\uc801\uc778 \uce21\uba74\uc5d0\uc11c\ub294 \uc18c\uc2dc\ubbfc\uc758 \uc77c\uc0c1\uc5d0 \ucd08\uc810\uc744 \ub9de\ucd94\uc5b4 \uc601\uc0c1\uc744 \ub2f4\uc544\ub0b4\ub358 \uacfc\uac70\uc640\ub294 \ub2e4\ub974\uac8c \uad6d\uac00\uad8c\ub825\uc744 \uc639\ud638\ud558\ub294 \ubaa8\uc2b5\uc744 \ubcf4\uc774\uae30\ub3c4 \ud558\uc600\ub2e4.", "paragraph_answer": "2000\ub144 \ub300 \uc774\ud6c4, \uc7a5\uc774\uba38\uc6b0\ub294 \ub9ac\uc5bc\ub9ac\uc998 \ubc14\ud0d5\uc758 \ubbfc\uc911\uc758 \uc0b6\uc744 \uadf8\ub9ac\ub294 \uac83\ubcf4\ub2e4 \uba3c \uacfc\uac70\uc758 \uc911\uad6d \uc655\uc870 \uc2dc\ub300\ub85c \uce74\uba54\ub77c\uc758 \ucd08\uc810\uc744 \ubc14\uafbc\ub2e4. \uadf8\ub9ac\uace0 \uc601\uc0c1\uc5d0 \uc788\uc5b4 \uacfc\uac70\uc640\ub294 \ub9e4\uc6b0 \ucc28\ubcc4\ud654\ub418\ub294 \uc2a4\ud0c0\uc77c\uc758 \ubcc0\ud654\uac00 \uc0dd\uacbc\ub2e4. \uacfc\uac70 \uc911\uad6d \uc655\uc870 \uc2dc\ub300\ub97c \ubc30\uacbd\uc73c\ub85c \ubb34\ud611\uc801\uc778 \uc694\uc18c\ub97c \ub3c4\uc785\ud558\uc5ec \ud560\ub9ac\uc6b0\ub4dc\uc640 \uac19\uc740 \uc2a4\ud399\ud130\ud074\ud55c \uc7a5\uba74\ub4e4\uc744 \uc5f0\ucd9c\ud558\uace0 \uc601\uc0c1\ubbf8\ub97c \uadf9\ub300\ud654\ud558\uc600\ub2e4. 2002\ub144 \u300a\uc601\uc6c5\u300b\uc740 \uc774\ub7ec\ud55c \uacbd\ud5a5\uc744 \uc798 \ub098\ud0c0\ub0b4\ub294 \uc791\ud488\uc73c\ub85c \uc804\uad6d\uc2dc\ub300 \ub9d0, \uc9c4\uc2dc\ud669 \uc2dc\ub300\ub97c \ubc30\uacbd\uc73c\ub85c \uc9c4\uc2dc\ud669\uc744 \uc554\uc0b4\ud558\ub824\uace0 \ud558\ub294 \ud611\uac1d\ub4e4\uc758 \uc774\uc57c\uae30\uc774\ub2e4. \u300a\uc601\uc6c5\u300b\uc740 \uc911\uad6d\uacfc \ud574\uc678\uc5d0\uc11c \uc0c1\uc5c5\uc601\ud654\ub85c\uc11c \uacf5\uc804\uc758 \ud788\ud2b8\ub97c \uae30\ub85d\ud558\uc600\uace0, 2003\ub144 \uc544\uce74\ub370\ubbf8 \uc218\uc0c1\uc744 \ud558\uc600\ub2e4. \ub4a4\uc774\uc5b4 \u300a\uc778\uc5f0\u300b( 2004 )\uacfc \u300a\ud669\ud6c4\ud654\u300b(2006)\uc5d0\uc11c\ub3c4 \uacfc\uac70 \uc911\uad6d \uc655\uc870\ub97c \ubc30\uacbd\uc73c\ub85c \uc0c9\ucc44\uc911\uc2ec\uc758 \ud654\ub824\ud55c \uc601\uc0c1\ubbf8\ub97c \uc120\ubcf4\uc600\ub2e4. \uc774 \uc2dc\uae30\uc758 \uc791\ud488\uc740 \uc601\uc0c1\ubbf8\ud559\uc801\uc778 \uce21\uba74\uc5d0\uc11c\ub294 \uc804\uc791\ub4e4\uacfc \ube44\uad50\ud558\uc5ec \ube44\uc57d\uc801\uc778 \ubc1c\uc804\uc774 \uc788\uc5c8\uc9c0\ub9cc, \ub0b4\uc6a9\uc801\uc778 \uce21\uba74\uc5d0\uc11c\ub294 \uc18c\uc2dc\ubbfc\uc758 \uc77c\uc0c1\uc5d0 \ucd08\uc810\uc744 \ub9de\ucd94\uc5b4 \uc601\uc0c1\uc744 \ub2f4\uc544\ub0b4\ub358 \uacfc\uac70\uc640\ub294 \ub2e4\ub974\uac8c \uad6d\uac00\uad8c\ub825\uc744 \uc639\ud638\ud558\ub294 \ubaa8\uc2b5\uc744 \ubcf4\uc774\uae30\ub3c4 \ud558\uc600\ub2e4.", "sentence_answer": "\ub4a4\uc774\uc5b4 \u300a\uc778\uc5f0\u300b( 2004 )", "paragraph_id": "6549099-6-1", "paragraph_question": "question: \uc7a5\uc774\uba38\uc6b0\uc758 \uc778\uc5f0\uc740 \uc5b8\uc81c \ubc1c\ud45c\ub418\uc5c8\ub294\uac00?, context: 2000\ub144 \ub300 \uc774\ud6c4, \uc7a5\uc774\uba38\uc6b0\ub294 \ub9ac\uc5bc\ub9ac\uc998 \ubc14\ud0d5\uc758 \ubbfc\uc911\uc758 \uc0b6\uc744 \uadf8\ub9ac\ub294 \uac83\ubcf4\ub2e4 \uba3c \uacfc\uac70\uc758 \uc911\uad6d \uc655\uc870 \uc2dc\ub300\ub85c \uce74\uba54\ub77c\uc758 \ucd08\uc810\uc744 \ubc14\uafbc\ub2e4. \uadf8\ub9ac\uace0 \uc601\uc0c1\uc5d0 \uc788\uc5b4 \uacfc\uac70\uc640\ub294 \ub9e4\uc6b0 \ucc28\ubcc4\ud654\ub418\ub294 \uc2a4\ud0c0\uc77c\uc758 \ubcc0\ud654\uac00 \uc0dd\uacbc\ub2e4. \uacfc\uac70 \uc911\uad6d \uc655\uc870 \uc2dc\ub300\ub97c \ubc30\uacbd\uc73c\ub85c \ubb34\ud611\uc801\uc778 \uc694\uc18c\ub97c \ub3c4\uc785\ud558\uc5ec \ud560\ub9ac\uc6b0\ub4dc\uc640 \uac19\uc740 \uc2a4\ud399\ud130\ud074\ud55c \uc7a5\uba74\ub4e4\uc744 \uc5f0\ucd9c\ud558\uace0 \uc601\uc0c1\ubbf8\ub97c \uadf9\ub300\ud654\ud558\uc600\ub2e4. 2002\ub144 \u300a\uc601\uc6c5\u300b\uc740 \uc774\ub7ec\ud55c \uacbd\ud5a5\uc744 \uc798 \ub098\ud0c0\ub0b4\ub294 \uc791\ud488\uc73c\ub85c \uc804\uad6d\uc2dc\ub300 \ub9d0, \uc9c4\uc2dc\ud669 \uc2dc\ub300\ub97c \ubc30\uacbd\uc73c\ub85c \uc9c4\uc2dc\ud669\uc744 \uc554\uc0b4\ud558\ub824\uace0 \ud558\ub294 \ud611\uac1d\ub4e4\uc758 \uc774\uc57c\uae30\uc774\ub2e4. \u300a\uc601\uc6c5\u300b\uc740 \uc911\uad6d\uacfc \ud574\uc678\uc5d0\uc11c \uc0c1\uc5c5\uc601\ud654\ub85c\uc11c \uacf5\uc804\uc758 \ud788\ud2b8\ub97c \uae30\ub85d\ud558\uc600\uace0, 2003\ub144 \uc544\uce74\ub370\ubbf8 \uc218\uc0c1\uc744 \ud558\uc600\ub2e4. \ub4a4\uc774\uc5b4 \u300a\uc778\uc5f0\u300b(2004)\uacfc \u300a\ud669\ud6c4\ud654\u300b(2006)\uc5d0\uc11c\ub3c4 \uacfc\uac70 \uc911\uad6d \uc655\uc870\ub97c \ubc30\uacbd\uc73c\ub85c \uc0c9\ucc44\uc911\uc2ec\uc758 \ud654\ub824\ud55c \uc601\uc0c1\ubbf8\ub97c \uc120\ubcf4\uc600\ub2e4. \uc774 \uc2dc\uae30\uc758 \uc791\ud488\uc740 \uc601\uc0c1\ubbf8\ud559\uc801\uc778 \uce21\uba74\uc5d0\uc11c\ub294 \uc804\uc791\ub4e4\uacfc \ube44\uad50\ud558\uc5ec \ube44\uc57d\uc801\uc778 \ubc1c\uc804\uc774 \uc788\uc5c8\uc9c0\ub9cc, \ub0b4\uc6a9\uc801\uc778 \uce21\uba74\uc5d0\uc11c\ub294 \uc18c\uc2dc\ubbfc\uc758 \uc77c\uc0c1\uc5d0 \ucd08\uc810\uc744 \ub9de\ucd94\uc5b4 \uc601\uc0c1\uc744 \ub2f4\uc544\ub0b4\ub358 \uacfc\uac70\uc640\ub294 \ub2e4\ub974\uac8c \uad6d\uac00\uad8c\ub825\uc744 \uc639\ud638\ud558\ub294 \ubaa8\uc2b5\uc744 \ubcf4\uc774\uae30\ub3c4 \ud558\uc600\ub2e4."} {"question": "\uc2a4\ud53c\uc5b4\uc2a4\uac00 2008\ub144 \uacf5\uc5f0\uc5d0\uc11c \uc785\uc5c8\ub358 \ub4dc\ub808\uc2a4\uc758 \uacbd\ub9e4 \uc218\uc775\uae08\uc744 \uae30\ubd80\ud55c \uacf3\uc740?", "paragraph": "\ube0c\ub9ac\ud2b8\ub2c8\ub294 2001\ub144\uc5d0 \ub3c8\uc774 \uc5c6\uc5b4 \uacf5\ubd80\ub97c \ubabb\ud558\ub294 \uc544\uc774\ub4e4\uc744 \ub3c4\uc640\uc8fc\uae30\uc704\ud574 \uc790\uc2e0\uc758 \uc7ac\uc0b0 50\ub9cc \ub2ec\ub7ec\ub97c \uae30\ubd80\ud574 \ube0c\ub9ac\ud2b8\ub2c8\uc2a4\ud53c\uc5b4\uc2a4\uc7ac\ub2e8\uc744 \uc138\uc6e0\ub2e4. \ud53c\ud50c \uba54\uac70\uc9c4\uacfc MTV\uac00 \ubc1c\ud45c\ud55c \uae30\uc0ac\uc5d0 \ub530\ub974\uba74 2008\ub144 10\uc6d4 1\uc77c \ub274\uc695 \ube0c\ub871\ud06c\uc2a4\uc758 '\uc874 \ud544\ub9bd \uc18c\uc0ac \ubbf8\ub4e4 \uc2a4\ucfe8'\uc5d0 \ubc29\ubb38\ud574 \uc74c\uc545 \ud504\ub85c\uadf8\ub7a8\uc744 \uc704\ud574 1\ub9cc\ub2ec\ub7ec\ub97c \uae30\ubd80\ud588\ub2e4\uace0 \ud55c\ub2e4. 2010\ub144\uc5d0\ub294 2008 MTV \ube44\ub514\uc624 \ubba4\uc9c1 \uc5b4\uc6cc\ub4dc\uc5d0\uc11c \uc785\uc788\ub358 \ub4dc\ub808\uc2a4\ub97c \uacbd\ub9e4\uc5d0 \ub0b4\ub193\uc544 \uc218\uc775\uae08\uc744 \uc544\uc774\ud2f0\uc5d0 \uae30\ubd80\ud588\ub2e4. 2011\ub144\uc5d0\ub294 \ubbf8\uad6d \ub300\ud1b5\ub839 \uc624\ubc14\ub9c8\uc640 \uc601\ubd80\uc778\uc758 \uad8c\uc720\ub85c \"\uc57d\ud55c \uce5c\uad6c\ub97c \ub530\ub3cc\ub9ac\uace0 \uad34\ub86d\ud788\ub294 \ud589\uc704\ub294 \uc0ac\ub77c\uc838\uc57c\ub9cc \ud55c\ub2e4\"\uba70 \ud559\uad50 \ud3ed\ub825 \ubc29\uc9c0 \ucea0\ud398\uc778\uc5d0 \ucc38\uc5ec\ud588\ub2e4. \ud55c\ud3b8, \ube0c\ub9ac\ud2b8\ub2c8\uc758 \uc7ac\uc0b0\uc740 1\uc5b5 \ub2ec\ub7ec\uac00 \ub118\ub294 \uac83\uc73c\ub85c \uc54c\ub824\uc838 \uc788\ub2e4.", "answer": "\uc544\uc774\ud2f0", "sentence": "2010\ub144\uc5d0\ub294 2008 MTV \ube44\ub514\uc624 \ubba4\uc9c1 \uc5b4\uc6cc\ub4dc\uc5d0\uc11c \uc785\uc788\ub358 \ub4dc\ub808\uc2a4\ub97c \uacbd\ub9e4\uc5d0 \ub0b4\ub193\uc544 \uc218\uc775\uae08\uc744 \uc544\uc774\ud2f0 \uc5d0 \uae30\ubd80\ud588\ub2e4.", "paragraph_sentence": "\ube0c\ub9ac\ud2b8\ub2c8\ub294 2001\ub144\uc5d0 \ub3c8\uc774 \uc5c6\uc5b4 \uacf5\ubd80\ub97c \ubabb\ud558\ub294 \uc544\uc774\ub4e4\uc744 \ub3c4\uc640\uc8fc\uae30\uc704\ud574 \uc790\uc2e0\uc758 \uc7ac\uc0b0 50\ub9cc \ub2ec\ub7ec\ub97c \uae30\ubd80\ud574 \ube0c\ub9ac\ud2b8\ub2c8\uc2a4\ud53c\uc5b4\uc2a4\uc7ac\ub2e8\uc744 \uc138\uc6e0\ub2e4. \ud53c\ud50c \uba54\uac70\uc9c4\uacfc MTV\uac00 \ubc1c\ud45c\ud55c \uae30\uc0ac\uc5d0 \ub530\ub974\uba74 2008\ub144 10\uc6d4 1\uc77c \ub274\uc695 \ube0c\ub871\ud06c\uc2a4\uc758 '\uc874 \ud544\ub9bd \uc18c\uc0ac \ubbf8\ub4e4 \uc2a4\ucfe8'\uc5d0 \ubc29\ubb38\ud574 \uc74c\uc545 \ud504\ub85c\uadf8\ub7a8\uc744 \uc704\ud574 1\ub9cc\ub2ec\ub7ec\ub97c \uae30\ubd80\ud588\ub2e4\uace0 \ud55c\ub2e4. 2010\ub144\uc5d0\ub294 2008 MTV \ube44\ub514\uc624 \ubba4\uc9c1 \uc5b4\uc6cc\ub4dc\uc5d0\uc11c \uc785\uc788\ub358 \ub4dc\ub808\uc2a4\ub97c \uacbd\ub9e4\uc5d0 \ub0b4\ub193\uc544 \uc218\uc775\uae08\uc744 \uc544\uc774\ud2f0 \uc5d0 \uae30\ubd80\ud588\ub2e4. 2011\ub144\uc5d0\ub294 \ubbf8\uad6d \ub300\ud1b5\ub839 \uc624\ubc14\ub9c8\uc640 \uc601\ubd80\uc778\uc758 \uad8c\uc720\ub85c \"\uc57d\ud55c \uce5c\uad6c\ub97c \ub530\ub3cc\ub9ac\uace0 \uad34\ub86d\ud788\ub294 \ud589\uc704\ub294 \uc0ac\ub77c\uc838\uc57c\ub9cc \ud55c\ub2e4\"\uba70 \ud559\uad50 \ud3ed\ub825 \ubc29\uc9c0 \ucea0\ud398\uc778\uc5d0 \ucc38\uc5ec\ud588\ub2e4. \ud55c\ud3b8, \ube0c\ub9ac\ud2b8\ub2c8\uc758 \uc7ac\uc0b0\uc740 1\uc5b5 \ub2ec\ub7ec\uac00 \ub118\ub294 \uac83\uc73c\ub85c \uc54c\ub824\uc838 \uc788\ub2e4.", "paragraph_answer": "\ube0c\ub9ac\ud2b8\ub2c8\ub294 2001\ub144\uc5d0 \ub3c8\uc774 \uc5c6\uc5b4 \uacf5\ubd80\ub97c \ubabb\ud558\ub294 \uc544\uc774\ub4e4\uc744 \ub3c4\uc640\uc8fc\uae30\uc704\ud574 \uc790\uc2e0\uc758 \uc7ac\uc0b0 50\ub9cc \ub2ec\ub7ec\ub97c \uae30\ubd80\ud574 \ube0c\ub9ac\ud2b8\ub2c8\uc2a4\ud53c\uc5b4\uc2a4\uc7ac\ub2e8\uc744 \uc138\uc6e0\ub2e4. \ud53c\ud50c \uba54\uac70\uc9c4\uacfc MTV\uac00 \ubc1c\ud45c\ud55c \uae30\uc0ac\uc5d0 \ub530\ub974\uba74 2008\ub144 10\uc6d4 1\uc77c \ub274\uc695 \ube0c\ub871\ud06c\uc2a4\uc758 '\uc874 \ud544\ub9bd \uc18c\uc0ac \ubbf8\ub4e4 \uc2a4\ucfe8'\uc5d0 \ubc29\ubb38\ud574 \uc74c\uc545 \ud504\ub85c\uadf8\ub7a8\uc744 \uc704\ud574 1\ub9cc\ub2ec\ub7ec\ub97c \uae30\ubd80\ud588\ub2e4\uace0 \ud55c\ub2e4. 2010\ub144\uc5d0\ub294 2008 MTV \ube44\ub514\uc624 \ubba4\uc9c1 \uc5b4\uc6cc\ub4dc\uc5d0\uc11c \uc785\uc788\ub358 \ub4dc\ub808\uc2a4\ub97c \uacbd\ub9e4\uc5d0 \ub0b4\ub193\uc544 \uc218\uc775\uae08\uc744 \uc544\uc774\ud2f0 \uc5d0 \uae30\ubd80\ud588\ub2e4. 2011\ub144\uc5d0\ub294 \ubbf8\uad6d \ub300\ud1b5\ub839 \uc624\ubc14\ub9c8\uc640 \uc601\ubd80\uc778\uc758 \uad8c\uc720\ub85c \"\uc57d\ud55c \uce5c\uad6c\ub97c \ub530\ub3cc\ub9ac\uace0 \uad34\ub86d\ud788\ub294 \ud589\uc704\ub294 \uc0ac\ub77c\uc838\uc57c\ub9cc \ud55c\ub2e4\"\uba70 \ud559\uad50 \ud3ed\ub825 \ubc29\uc9c0 \ucea0\ud398\uc778\uc5d0 \ucc38\uc5ec\ud588\ub2e4. \ud55c\ud3b8, \ube0c\ub9ac\ud2b8\ub2c8\uc758 \uc7ac\uc0b0\uc740 1\uc5b5 \ub2ec\ub7ec\uac00 \ub118\ub294 \uac83\uc73c\ub85c \uc54c\ub824\uc838 \uc788\ub2e4.", "sentence_answer": "2010\ub144\uc5d0\ub294 2008 MTV \ube44\ub514\uc624 \ubba4\uc9c1 \uc5b4\uc6cc\ub4dc\uc5d0\uc11c \uc785\uc788\ub358 \ub4dc\ub808\uc2a4\ub97c \uacbd\ub9e4\uc5d0 \ub0b4\ub193\uc544 \uc218\uc775\uae08\uc744 \uc544\uc774\ud2f0 \uc5d0 \uae30\ubd80\ud588\ub2e4.", "paragraph_id": "6509466-30-1", "paragraph_question": "question: \uc2a4\ud53c\uc5b4\uc2a4\uac00 2008\ub144 \uacf5\uc5f0\uc5d0\uc11c \uc785\uc5c8\ub358 \ub4dc\ub808\uc2a4\uc758 \uacbd\ub9e4 \uc218\uc775\uae08\uc744 \uae30\ubd80\ud55c \uacf3\uc740?, context: \ube0c\ub9ac\ud2b8\ub2c8\ub294 2001\ub144\uc5d0 \ub3c8\uc774 \uc5c6\uc5b4 \uacf5\ubd80\ub97c \ubabb\ud558\ub294 \uc544\uc774\ub4e4\uc744 \ub3c4\uc640\uc8fc\uae30\uc704\ud574 \uc790\uc2e0\uc758 \uc7ac\uc0b0 50\ub9cc \ub2ec\ub7ec\ub97c \uae30\ubd80\ud574 \ube0c\ub9ac\ud2b8\ub2c8\uc2a4\ud53c\uc5b4\uc2a4\uc7ac\ub2e8\uc744 \uc138\uc6e0\ub2e4. \ud53c\ud50c \uba54\uac70\uc9c4\uacfc MTV\uac00 \ubc1c\ud45c\ud55c \uae30\uc0ac\uc5d0 \ub530\ub974\uba74 2008\ub144 10\uc6d4 1\uc77c \ub274\uc695 \ube0c\ub871\ud06c\uc2a4\uc758 '\uc874 \ud544\ub9bd \uc18c\uc0ac \ubbf8\ub4e4 \uc2a4\ucfe8'\uc5d0 \ubc29\ubb38\ud574 \uc74c\uc545 \ud504\ub85c\uadf8\ub7a8\uc744 \uc704\ud574 1\ub9cc\ub2ec\ub7ec\ub97c \uae30\ubd80\ud588\ub2e4\uace0 \ud55c\ub2e4. 2010\ub144\uc5d0\ub294 2008 MTV \ube44\ub514\uc624 \ubba4\uc9c1 \uc5b4\uc6cc\ub4dc\uc5d0\uc11c \uc785\uc788\ub358 \ub4dc\ub808\uc2a4\ub97c \uacbd\ub9e4\uc5d0 \ub0b4\ub193\uc544 \uc218\uc775\uae08\uc744 \uc544\uc774\ud2f0\uc5d0 \uae30\ubd80\ud588\ub2e4. 2011\ub144\uc5d0\ub294 \ubbf8\uad6d \ub300\ud1b5\ub839 \uc624\ubc14\ub9c8\uc640 \uc601\ubd80\uc778\uc758 \uad8c\uc720\ub85c \"\uc57d\ud55c \uce5c\uad6c\ub97c \ub530\ub3cc\ub9ac\uace0 \uad34\ub86d\ud788\ub294 \ud589\uc704\ub294 \uc0ac\ub77c\uc838\uc57c\ub9cc \ud55c\ub2e4\"\uba70 \ud559\uad50 \ud3ed\ub825 \ubc29\uc9c0 \ucea0\ud398\uc778\uc5d0 \ucc38\uc5ec\ud588\ub2e4. \ud55c\ud3b8, \ube0c\ub9ac\ud2b8\ub2c8\uc758 \uc7ac\uc0b0\uc740 1\uc5b5 \ub2ec\ub7ec\uac00 \ub118\ub294 \uac83\uc73c\ub85c \uc54c\ub824\uc838 \uc788\ub2e4."} {"question": "\uc624\uacf5\uc758 \uce5c\ud615\uc740?", "paragraph": "\ub9c8\uc96c\ub2c8\uc5b4\uc640\uc758 \uc2f8\uc6c0\uc73c\ub85c\ubd80\ud130 5\ub144 \ud6c4, \ud3c9\ud654\ub85c\uc6b4 \ub098\ub0a0\uc744 \ubcf4\ub0b4\uace0 \uc788\ub358 \uc624\uacf5 \uc55e\uc5d0 \uce5c\ud615 \ub77c\ub370\uce20\uac00 \uc6b0\uc8fc\uc5d0\uc11c \ub0b4\uc2b5, \uc624\uacf5\uc740 \uc790\uc2e0\uc774 \ud589\uc131 \ubca0\uc9c0\ud130\uc758 \uc804\ud22c\ubbfc\uc871 \uc0ac\uc774\uc5b4\uc778\uc784\uc744 \uc54c\uac8c \ub41c\ub2e4. \ub0a9\uce58\ub41c \uc544\ub4e4 \uc190\uc624\ubc18\uc744 \uad6c\ud558\uae30 \uc704\ud574 \uc624\uacf5\uc740 \ud55c\ub54c \uc219\uc801\uc774\ub358 \ud53c\ucf5c\ub85c(\ub9c8\uc96c\ub2c8\uc5b4)\uc640 \uc190\uc744 \uc7a1\uace0, \uc790\uc2e0\uc758 \ubaa9\uc228\uacfc \ubc14\uafb8\uc5b4\uc11c \ub77c\ub370\uce20\ub97c \ubb34\ucc0c\ub974\uc9c0\ub9cc, 1\ub144 \ud6c4\uc5d0\ub294 \ub354\uc6b1 \uac15\ud55c \uc0ac\uc774\uc5b4\uc778\ub4e4\uc774 \ub4dc\ub798\uace4\ubcfc\uc744 \uc190\uc5d0 \ub123\uae30 \uc704\ud574 \uc9c0\uad6c\ub85c \uc628\ub2e4\ub294 \uac83\uc744 \uc54c\uac8c \ub41c\ub2e4. \uc624\uacf5\uc740 \ub4dc\ub798\uace4\ubcfc\uc5d0 \uc758\ud574 \uc0b4\uc544\ub0a0 \ub54c\uae4c\uc9c0 \uc800 \uc138\uc0c1\uc758 \uacc4\uc655\uc758 \ubc11\uc5d0\uc11c \uc218\ud589\uc744 \ud55c\ub2e4. \uadf8\ub9ac\uace0 \ub3d9\ub8cc\ub4e4\uacfc \ud568\uaed8 \uc9c0\uad6c\uc5d0 \uc628 \uc0ac\uc774\uc5b4\uc778\uc758 \uc655\uc790 \ubca0\uc9c0\ud130\ub97c \ud328\ud1f4\uc2dc\ud0a4\uc9c0\ub9cc, \ub9ce\uc740 \ub3d9\ub8cc\ub97c \uc783\ub294\ub2e4. \uac8c\ub2e4\uac00 \ud53c\ucf5c\ub85c\uc758 \uc8fd\uc74c\uc73c\ub85c \uadf8\uc640 \uc77c\uc2ec\ub3d9\uccb4\uc778 \uc2e0\ub3c4 \uc0ac\ub9dd, \uc9c0\uad6c\uc758 \ub4dc\ub798\uace4\ubcfc\ub3c4 \uc18c\uba78\ud55c\ub2e4.", "answer": "\ub77c\ub370\uce20", "sentence": "\ub9c8\uc96c\ub2c8\uc5b4\uc640\uc758 \uc2f8\uc6c0\uc73c\ub85c\ubd80\ud130 5\ub144 \ud6c4, \ud3c9\ud654\ub85c\uc6b4 \ub098\ub0a0\uc744 \ubcf4\ub0b4\uace0 \uc788\ub358 \uc624\uacf5 \uc55e\uc5d0 \uce5c\ud615 \ub77c\ub370\uce20 \uac00 \uc6b0\uc8fc\uc5d0\uc11c \ub0b4\uc2b5, \uc624\uacf5\uc740 \uc790\uc2e0\uc774 \ud589\uc131 \ubca0\uc9c0\ud130\uc758 \uc804\ud22c\ubbfc\uc871 \uc0ac\uc774\uc5b4\uc778\uc784\uc744 \uc54c\uac8c \ub41c\ub2e4.", "paragraph_sentence": " \ub9c8\uc96c\ub2c8\uc5b4\uc640\uc758 \uc2f8\uc6c0\uc73c\ub85c\ubd80\ud130 5\ub144 \ud6c4, \ud3c9\ud654\ub85c\uc6b4 \ub098\ub0a0\uc744 \ubcf4\ub0b4\uace0 \uc788\ub358 \uc624\uacf5 \uc55e\uc5d0 \uce5c\ud615 \ub77c\ub370\uce20 \uac00 \uc6b0\uc8fc\uc5d0\uc11c \ub0b4\uc2b5, \uc624\uacf5\uc740 \uc790\uc2e0\uc774 \ud589\uc131 \ubca0\uc9c0\ud130\uc758 \uc804\ud22c\ubbfc\uc871 \uc0ac\uc774\uc5b4\uc778\uc784\uc744 \uc54c\uac8c \ub41c\ub2e4. \ub0a9\uce58\ub41c \uc544\ub4e4 \uc190\uc624\ubc18\uc744 \uad6c\ud558\uae30 \uc704\ud574 \uc624\uacf5\uc740 \ud55c\ub54c \uc219\uc801\uc774\ub358 \ud53c\ucf5c\ub85c(\ub9c8\uc96c\ub2c8\uc5b4)\uc640 \uc190\uc744 \uc7a1\uace0, \uc790\uc2e0\uc758 \ubaa9\uc228\uacfc \ubc14\uafb8\uc5b4\uc11c \ub77c\ub370\uce20\ub97c \ubb34\ucc0c\ub974\uc9c0\ub9cc, 1\ub144 \ud6c4\uc5d0\ub294 \ub354\uc6b1 \uac15\ud55c \uc0ac\uc774\uc5b4\uc778\ub4e4\uc774 \ub4dc\ub798\uace4\ubcfc\uc744 \uc190\uc5d0 \ub123\uae30 \uc704\ud574 \uc9c0\uad6c\ub85c \uc628\ub2e4\ub294 \uac83\uc744 \uc54c\uac8c \ub41c\ub2e4. \uc624\uacf5\uc740 \ub4dc\ub798\uace4\ubcfc\uc5d0 \uc758\ud574 \uc0b4\uc544\ub0a0 \ub54c\uae4c\uc9c0 \uc800 \uc138\uc0c1\uc758 \uacc4\uc655\uc758 \ubc11\uc5d0\uc11c \uc218\ud589\uc744 \ud55c\ub2e4. \uadf8\ub9ac\uace0 \ub3d9\ub8cc\ub4e4\uacfc \ud568\uaed8 \uc9c0\uad6c\uc5d0 \uc628 \uc0ac\uc774\uc5b4\uc778\uc758 \uc655\uc790 \ubca0\uc9c0\ud130\ub97c \ud328\ud1f4\uc2dc\ud0a4\uc9c0\ub9cc, \ub9ce\uc740 \ub3d9\ub8cc\ub97c \uc783\ub294\ub2e4. \uac8c\ub2e4\uac00 \ud53c\ucf5c\ub85c\uc758 \uc8fd\uc74c\uc73c\ub85c \uadf8\uc640 \uc77c\uc2ec\ub3d9\uccb4\uc778 \uc2e0\ub3c4 \uc0ac\ub9dd, \uc9c0\uad6c\uc758 \ub4dc\ub798\uace4\ubcfc\ub3c4 \uc18c\uba78\ud55c\ub2e4.", "paragraph_answer": "\ub9c8\uc96c\ub2c8\uc5b4\uc640\uc758 \uc2f8\uc6c0\uc73c\ub85c\ubd80\ud130 5\ub144 \ud6c4, \ud3c9\ud654\ub85c\uc6b4 \ub098\ub0a0\uc744 \ubcf4\ub0b4\uace0 \uc788\ub358 \uc624\uacf5 \uc55e\uc5d0 \uce5c\ud615 \ub77c\ub370\uce20 \uac00 \uc6b0\uc8fc\uc5d0\uc11c \ub0b4\uc2b5, \uc624\uacf5\uc740 \uc790\uc2e0\uc774 \ud589\uc131 \ubca0\uc9c0\ud130\uc758 \uc804\ud22c\ubbfc\uc871 \uc0ac\uc774\uc5b4\uc778\uc784\uc744 \uc54c\uac8c \ub41c\ub2e4. \ub0a9\uce58\ub41c \uc544\ub4e4 \uc190\uc624\ubc18\uc744 \uad6c\ud558\uae30 \uc704\ud574 \uc624\uacf5\uc740 \ud55c\ub54c \uc219\uc801\uc774\ub358 \ud53c\ucf5c\ub85c(\ub9c8\uc96c\ub2c8\uc5b4)\uc640 \uc190\uc744 \uc7a1\uace0, \uc790\uc2e0\uc758 \ubaa9\uc228\uacfc \ubc14\uafb8\uc5b4\uc11c \ub77c\ub370\uce20\ub97c \ubb34\ucc0c\ub974\uc9c0\ub9cc, 1\ub144 \ud6c4\uc5d0\ub294 \ub354\uc6b1 \uac15\ud55c \uc0ac\uc774\uc5b4\uc778\ub4e4\uc774 \ub4dc\ub798\uace4\ubcfc\uc744 \uc190\uc5d0 \ub123\uae30 \uc704\ud574 \uc9c0\uad6c\ub85c \uc628\ub2e4\ub294 \uac83\uc744 \uc54c\uac8c \ub41c\ub2e4. \uc624\uacf5\uc740 \ub4dc\ub798\uace4\ubcfc\uc5d0 \uc758\ud574 \uc0b4\uc544\ub0a0 \ub54c\uae4c\uc9c0 \uc800 \uc138\uc0c1\uc758 \uacc4\uc655\uc758 \ubc11\uc5d0\uc11c \uc218\ud589\uc744 \ud55c\ub2e4. \uadf8\ub9ac\uace0 \ub3d9\ub8cc\ub4e4\uacfc \ud568\uaed8 \uc9c0\uad6c\uc5d0 \uc628 \uc0ac\uc774\uc5b4\uc778\uc758 \uc655\uc790 \ubca0\uc9c0\ud130\ub97c \ud328\ud1f4\uc2dc\ud0a4\uc9c0\ub9cc, \ub9ce\uc740 \ub3d9\ub8cc\ub97c \uc783\ub294\ub2e4. \uac8c\ub2e4\uac00 \ud53c\ucf5c\ub85c\uc758 \uc8fd\uc74c\uc73c\ub85c \uadf8\uc640 \uc77c\uc2ec\ub3d9\uccb4\uc778 \uc2e0\ub3c4 \uc0ac\ub9dd, \uc9c0\uad6c\uc758 \ub4dc\ub798\uace4\ubcfc\ub3c4 \uc18c\uba78\ud55c\ub2e4.", "sentence_answer": "\ub9c8\uc96c\ub2c8\uc5b4\uc640\uc758 \uc2f8\uc6c0\uc73c\ub85c\ubd80\ud130 5\ub144 \ud6c4, \ud3c9\ud654\ub85c\uc6b4 \ub098\ub0a0\uc744 \ubcf4\ub0b4\uace0 \uc788\ub358 \uc624\uacf5 \uc55e\uc5d0 \uce5c\ud615 \ub77c\ub370\uce20 \uac00 \uc6b0\uc8fc\uc5d0\uc11c \ub0b4\uc2b5, \uc624\uacf5\uc740 \uc790\uc2e0\uc774 \ud589\uc131 \ubca0\uc9c0\ud130\uc758 \uc804\ud22c\ubbfc\uc871 \uc0ac\uc774\uc5b4\uc778\uc784\uc744 \uc54c\uac8c \ub41c\ub2e4.", "paragraph_id": "6048697-1-0", "paragraph_question": "question: \uc624\uacf5\uc758 \uce5c\ud615\uc740?, context: \ub9c8\uc96c\ub2c8\uc5b4\uc640\uc758 \uc2f8\uc6c0\uc73c\ub85c\ubd80\ud130 5\ub144 \ud6c4, \ud3c9\ud654\ub85c\uc6b4 \ub098\ub0a0\uc744 \ubcf4\ub0b4\uace0 \uc788\ub358 \uc624\uacf5 \uc55e\uc5d0 \uce5c\ud615 \ub77c\ub370\uce20\uac00 \uc6b0\uc8fc\uc5d0\uc11c \ub0b4\uc2b5, \uc624\uacf5\uc740 \uc790\uc2e0\uc774 \ud589\uc131 \ubca0\uc9c0\ud130\uc758 \uc804\ud22c\ubbfc\uc871 \uc0ac\uc774\uc5b4\uc778\uc784\uc744 \uc54c\uac8c \ub41c\ub2e4. \ub0a9\uce58\ub41c \uc544\ub4e4 \uc190\uc624\ubc18\uc744 \uad6c\ud558\uae30 \uc704\ud574 \uc624\uacf5\uc740 \ud55c\ub54c \uc219\uc801\uc774\ub358 \ud53c\ucf5c\ub85c(\ub9c8\uc96c\ub2c8\uc5b4)\uc640 \uc190\uc744 \uc7a1\uace0, \uc790\uc2e0\uc758 \ubaa9\uc228\uacfc \ubc14\uafb8\uc5b4\uc11c \ub77c\ub370\uce20\ub97c \ubb34\ucc0c\ub974\uc9c0\ub9cc, 1\ub144 \ud6c4\uc5d0\ub294 \ub354\uc6b1 \uac15\ud55c \uc0ac\uc774\uc5b4\uc778\ub4e4\uc774 \ub4dc\ub798\uace4\ubcfc\uc744 \uc190\uc5d0 \ub123\uae30 \uc704\ud574 \uc9c0\uad6c\ub85c \uc628\ub2e4\ub294 \uac83\uc744 \uc54c\uac8c \ub41c\ub2e4. \uc624\uacf5\uc740 \ub4dc\ub798\uace4\ubcfc\uc5d0 \uc758\ud574 \uc0b4\uc544\ub0a0 \ub54c\uae4c\uc9c0 \uc800 \uc138\uc0c1\uc758 \uacc4\uc655\uc758 \ubc11\uc5d0\uc11c \uc218\ud589\uc744 \ud55c\ub2e4. \uadf8\ub9ac\uace0 \ub3d9\ub8cc\ub4e4\uacfc \ud568\uaed8 \uc9c0\uad6c\uc5d0 \uc628 \uc0ac\uc774\uc5b4\uc778\uc758 \uc655\uc790 \ubca0\uc9c0\ud130\ub97c \ud328\ud1f4\uc2dc\ud0a4\uc9c0\ub9cc, \ub9ce\uc740 \ub3d9\ub8cc\ub97c \uc783\ub294\ub2e4. \uac8c\ub2e4\uac00 \ud53c\ucf5c\ub85c\uc758 \uc8fd\uc74c\uc73c\ub85c \uadf8\uc640 \uc77c\uc2ec\ub3d9\uccb4\uc778 \uc2e0\ub3c4 \uc0ac\ub9dd, \uc9c0\uad6c\uc758 \ub4dc\ub798\uace4\ubcfc\ub3c4 \uc18c\uba78\ud55c\ub2e4."} {"question": "\ub3c5\uc77c \uacf5\uc911 \uc0ac\uc6b0\ub098\uc5d0\uc11c \uc0ac\uc6b0\ub098\ub9c8\uc2a4\ud130\uc5d0 \uc758\ud574\uc11c \uc218\ud589\ub418\ub294 \uac83\uc740?", "paragraph": "\uacf5\uc911\uc0ac\uc6b0\ub098\uc5d0\uc11c\ub294 \"Saunameister\"\ub77c\uace0 \ubd88\ub9ac\ub294 \uc0ac\uc6b0\ub098\ub9c8\uc2a4\ud130\uc5d0 \uc758\ud574\uc11c \uc544\uc6b0\ud504\uad6c\uc2a4\uac00 \uc218\ud589\ub41c\ub2e4. \ubc29\ubb38\uc790\uac00 \uc9c1\uc811 \uc774\ub97c \uc218\ud589\ud558\ub294 \uacbd\uc6b0\ub294 \uc77c\ubc18\uc801\uc774\uc9c0 \uc54a\uac70\ub098 \uae08\uc9c0\ub3fc\uc788\ub2e4. \uc544\uc6b0\ud504\uad6c\uc2a4\ub4e4\uc740 \ub300\ubd80\ubd84 \ube44\uc2b7\ud55c \ubc29\uc2dd\uc73c\ub85c \uc218\ud589\ub41c\ub2e4. \ub2e8\uc9c0 \uc4f0\uc774\ub294 \ud5a5\uc774 \ub2e4\uc591\ud560 \ubfd0\uc774\ub2e4. \uc544\uc6b0\ud504\uad6c\uc2a4\ud589\uc0ac\uac00 \uc9c4\ud589\ub420 \ub54c\uc5d0\ub294 \uc0ac\uc6b0\ub098\ub97c \ub098\uac08 \uc218 \uc5c6\ub2e4. \ubb38\uc744 \uc5ec\ub294 \uac83\uc774 \uc544\uc6b0\ud504\uad6c\uc2a4\uac00 \uac00\uc838\ub2e4\uc8fc\ub294 \ud6a8\uacfc\ub97c \ubc29\ud574\ud558\uae30 \ub54c\ubb38\uc774\ub2e4. \ub108\ubb34 \ub365\ub2e4\uace0 \ub290\ub07c\ub294 \ubc29\ubb38\uc790\ub4e4\uc740 \uc880 \ub354 \uae4a\uc774 \uc549\ub294 \uac83\uc744 \ucd94\ucc9c\ubc1b\ub294\ub2e4. \ubb3c\ub860 \uc815\ub9d0 \uc911\uc694\ud55c \ud544\uc694\uc131\uc774\ub098 \uac74\uac15\uc0c1\uc758 \ubb38\uc81c(\ud608\uc561\uc21c\ud658\uacc4\ud1b5)\uac00 \uc788\ub294 \uacbd\uc6b0\uc5d0\ub294 \uc0ac\uc6b0\ub098\ub8f8\uc744 \ub5a0\ub098\ub294 \uac83\uc774 \ud5c8\uc6a9\ub41c\ub2e4. \ub367\ubd99\uc5ec, \uc0ac\uc6b0\ub098\uc5d0\uc11c\ub294 \uc544\uc6b0\ud504\uad6c\uc2a4 \uc2dc \ubcf4\ud1b5 \uc870\uc6a9\ud788 \uc784\ud558\ub294 \uac83\uc774 \ud734\uc2dd\uc744 \ucd09\uc9c4\uc2dc\ud0a4\ub294 \uae38\uc774\uba70 \ubc29\ubb38\uc790\uc758 \uc608\uc758\uc774\ub2e4.", "answer": "\uc544\uc6b0\ud504\uad6c\uc2a4", "sentence": "\uacf5\uc911\uc0ac\uc6b0\ub098\uc5d0\uc11c\ub294 \"Saunameister\"\ub77c\uace0 \ubd88\ub9ac\ub294 \uc0ac\uc6b0\ub098\ub9c8\uc2a4\ud130\uc5d0 \uc758\ud574\uc11c \uc544\uc6b0\ud504\uad6c\uc2a4 \uac00 \uc218\ud589\ub41c\ub2e4.", "paragraph_sentence": " \uacf5\uc911\uc0ac\uc6b0\ub098\uc5d0\uc11c\ub294 \"Saunameister\"\ub77c\uace0 \ubd88\ub9ac\ub294 \uc0ac\uc6b0\ub098\ub9c8\uc2a4\ud130\uc5d0 \uc758\ud574\uc11c \uc544\uc6b0\ud504\uad6c\uc2a4 \uac00 \uc218\ud589\ub41c\ub2e4. \ubc29\ubb38\uc790\uac00 \uc9c1\uc811 \uc774\ub97c \uc218\ud589\ud558\ub294 \uacbd\uc6b0\ub294 \uc77c\ubc18\uc801\uc774\uc9c0 \uc54a\uac70\ub098 \uae08\uc9c0\ub3fc\uc788\ub2e4. \uc544\uc6b0\ud504\uad6c\uc2a4\ub4e4\uc740 \ub300\ubd80\ubd84 \ube44\uc2b7\ud55c \ubc29\uc2dd\uc73c\ub85c \uc218\ud589\ub41c\ub2e4. \ub2e8\uc9c0 \uc4f0\uc774\ub294 \ud5a5\uc774 \ub2e4\uc591\ud560 \ubfd0\uc774\ub2e4. \uc544\uc6b0\ud504\uad6c\uc2a4\ud589\uc0ac\uac00 \uc9c4\ud589\ub420 \ub54c\uc5d0\ub294 \uc0ac\uc6b0\ub098\ub97c \ub098\uac08 \uc218 \uc5c6\ub2e4. \ubb38\uc744 \uc5ec\ub294 \uac83\uc774 \uc544\uc6b0\ud504\uad6c\uc2a4\uac00 \uac00\uc838\ub2e4\uc8fc\ub294 \ud6a8\uacfc\ub97c \ubc29\ud574\ud558\uae30 \ub54c\ubb38\uc774\ub2e4. \ub108\ubb34 \ub365\ub2e4\uace0 \ub290\ub07c\ub294 \ubc29\ubb38\uc790\ub4e4\uc740 \uc880 \ub354 \uae4a\uc774 \uc549\ub294 \uac83\uc744 \ucd94\ucc9c\ubc1b\ub294\ub2e4. \ubb3c\ub860 \uc815\ub9d0 \uc911\uc694\ud55c \ud544\uc694\uc131\uc774\ub098 \uac74\uac15\uc0c1\uc758 \ubb38\uc81c(\ud608\uc561\uc21c\ud658\uacc4\ud1b5)\uac00 \uc788\ub294 \uacbd\uc6b0\uc5d0\ub294 \uc0ac\uc6b0\ub098\ub8f8\uc744 \ub5a0\ub098\ub294 \uac83\uc774 \ud5c8\uc6a9\ub41c\ub2e4. \ub367\ubd99\uc5ec, \uc0ac\uc6b0\ub098\uc5d0\uc11c\ub294 \uc544\uc6b0\ud504\uad6c\uc2a4 \uc2dc \ubcf4\ud1b5 \uc870\uc6a9\ud788 \uc784\ud558\ub294 \uac83\uc774 \ud734\uc2dd\uc744 \ucd09\uc9c4\uc2dc\ud0a4\ub294 \uae38\uc774\uba70 \ubc29\ubb38\uc790\uc758 \uc608\uc758\uc774\ub2e4.", "paragraph_answer": "\uacf5\uc911\uc0ac\uc6b0\ub098\uc5d0\uc11c\ub294 \"Saunameister\"\ub77c\uace0 \ubd88\ub9ac\ub294 \uc0ac\uc6b0\ub098\ub9c8\uc2a4\ud130\uc5d0 \uc758\ud574\uc11c \uc544\uc6b0\ud504\uad6c\uc2a4 \uac00 \uc218\ud589\ub41c\ub2e4. \ubc29\ubb38\uc790\uac00 \uc9c1\uc811 \uc774\ub97c \uc218\ud589\ud558\ub294 \uacbd\uc6b0\ub294 \uc77c\ubc18\uc801\uc774\uc9c0 \uc54a\uac70\ub098 \uae08\uc9c0\ub3fc\uc788\ub2e4. \uc544\uc6b0\ud504\uad6c\uc2a4\ub4e4\uc740 \ub300\ubd80\ubd84 \ube44\uc2b7\ud55c \ubc29\uc2dd\uc73c\ub85c \uc218\ud589\ub41c\ub2e4. \ub2e8\uc9c0 \uc4f0\uc774\ub294 \ud5a5\uc774 \ub2e4\uc591\ud560 \ubfd0\uc774\ub2e4. \uc544\uc6b0\ud504\uad6c\uc2a4\ud589\uc0ac\uac00 \uc9c4\ud589\ub420 \ub54c\uc5d0\ub294 \uc0ac\uc6b0\ub098\ub97c \ub098\uac08 \uc218 \uc5c6\ub2e4. \ubb38\uc744 \uc5ec\ub294 \uac83\uc774 \uc544\uc6b0\ud504\uad6c\uc2a4\uac00 \uac00\uc838\ub2e4\uc8fc\ub294 \ud6a8\uacfc\ub97c \ubc29\ud574\ud558\uae30 \ub54c\ubb38\uc774\ub2e4. \ub108\ubb34 \ub365\ub2e4\uace0 \ub290\ub07c\ub294 \ubc29\ubb38\uc790\ub4e4\uc740 \uc880 \ub354 \uae4a\uc774 \uc549\ub294 \uac83\uc744 \ucd94\ucc9c\ubc1b\ub294\ub2e4. \ubb3c\ub860 \uc815\ub9d0 \uc911\uc694\ud55c \ud544\uc694\uc131\uc774\ub098 \uac74\uac15\uc0c1\uc758 \ubb38\uc81c(\ud608\uc561\uc21c\ud658\uacc4\ud1b5)\uac00 \uc788\ub294 \uacbd\uc6b0\uc5d0\ub294 \uc0ac\uc6b0\ub098\ub8f8\uc744 \ub5a0\ub098\ub294 \uac83\uc774 \ud5c8\uc6a9\ub41c\ub2e4. \ub367\ubd99\uc5ec, \uc0ac\uc6b0\ub098\uc5d0\uc11c\ub294 \uc544\uc6b0\ud504\uad6c\uc2a4 \uc2dc \ubcf4\ud1b5 \uc870\uc6a9\ud788 \uc784\ud558\ub294 \uac83\uc774 \ud734\uc2dd\uc744 \ucd09\uc9c4\uc2dc\ud0a4\ub294 \uae38\uc774\uba70 \ubc29\ubb38\uc790\uc758 \uc608\uc758\uc774\ub2e4.", "sentence_answer": "\uacf5\uc911\uc0ac\uc6b0\ub098\uc5d0\uc11c\ub294 \"Saunameister\"\ub77c\uace0 \ubd88\ub9ac\ub294 \uc0ac\uc6b0\ub098\ub9c8\uc2a4\ud130\uc5d0 \uc758\ud574\uc11c \uc544\uc6b0\ud504\uad6c\uc2a4 \uac00 \uc218\ud589\ub41c\ub2e4.", "paragraph_id": "6046382-5-0", "paragraph_question": "question: \ub3c5\uc77c \uacf5\uc911 \uc0ac\uc6b0\ub098\uc5d0\uc11c \uc0ac\uc6b0\ub098\ub9c8\uc2a4\ud130\uc5d0 \uc758\ud574\uc11c \uc218\ud589\ub418\ub294 \uac83\uc740?, context: \uacf5\uc911\uc0ac\uc6b0\ub098\uc5d0\uc11c\ub294 \"Saunameister\"\ub77c\uace0 \ubd88\ub9ac\ub294 \uc0ac\uc6b0\ub098\ub9c8\uc2a4\ud130\uc5d0 \uc758\ud574\uc11c \uc544\uc6b0\ud504\uad6c\uc2a4\uac00 \uc218\ud589\ub41c\ub2e4. \ubc29\ubb38\uc790\uac00 \uc9c1\uc811 \uc774\ub97c \uc218\ud589\ud558\ub294 \uacbd\uc6b0\ub294 \uc77c\ubc18\uc801\uc774\uc9c0 \uc54a\uac70\ub098 \uae08\uc9c0\ub3fc\uc788\ub2e4. \uc544\uc6b0\ud504\uad6c\uc2a4\ub4e4\uc740 \ub300\ubd80\ubd84 \ube44\uc2b7\ud55c \ubc29\uc2dd\uc73c\ub85c \uc218\ud589\ub41c\ub2e4. \ub2e8\uc9c0 \uc4f0\uc774\ub294 \ud5a5\uc774 \ub2e4\uc591\ud560 \ubfd0\uc774\ub2e4. \uc544\uc6b0\ud504\uad6c\uc2a4\ud589\uc0ac\uac00 \uc9c4\ud589\ub420 \ub54c\uc5d0\ub294 \uc0ac\uc6b0\ub098\ub97c \ub098\uac08 \uc218 \uc5c6\ub2e4. \ubb38\uc744 \uc5ec\ub294 \uac83\uc774 \uc544\uc6b0\ud504\uad6c\uc2a4\uac00 \uac00\uc838\ub2e4\uc8fc\ub294 \ud6a8\uacfc\ub97c \ubc29\ud574\ud558\uae30 \ub54c\ubb38\uc774\ub2e4. \ub108\ubb34 \ub365\ub2e4\uace0 \ub290\ub07c\ub294 \ubc29\ubb38\uc790\ub4e4\uc740 \uc880 \ub354 \uae4a\uc774 \uc549\ub294 \uac83\uc744 \ucd94\ucc9c\ubc1b\ub294\ub2e4. \ubb3c\ub860 \uc815\ub9d0 \uc911\uc694\ud55c \ud544\uc694\uc131\uc774\ub098 \uac74\uac15\uc0c1\uc758 \ubb38\uc81c(\ud608\uc561\uc21c\ud658\uacc4\ud1b5)\uac00 \uc788\ub294 \uacbd\uc6b0\uc5d0\ub294 \uc0ac\uc6b0\ub098\ub8f8\uc744 \ub5a0\ub098\ub294 \uac83\uc774 \ud5c8\uc6a9\ub41c\ub2e4. \ub367\ubd99\uc5ec, \uc0ac\uc6b0\ub098\uc5d0\uc11c\ub294 \uc544\uc6b0\ud504\uad6c\uc2a4 \uc2dc \ubcf4\ud1b5 \uc870\uc6a9\ud788 \uc784\ud558\ub294 \uac83\uc774 \ud734\uc2dd\uc744 \ucd09\uc9c4\uc2dc\ud0a4\ub294 \uae38\uc774\uba70 \ubc29\ubb38\uc790\uc758 \uc608\uc758\uc774\ub2e4."} {"question": "'\uc544\ub9c8\ub8e8\ubca0 \uc0ac\uace0 \uae30\uc220\uc870\uc0ac\uc704\uc6d0\ud68c'\uac00 \ucd9c\ubc94\ud55c \ub0a0\uc9dc\ub294 \uc5b8\uc81c\uc778\uac00?", "paragraph": "1987\ub144 2\uc6d4 9\uc77c '\uc544\ub9c8\ub8e8\ubca0 \uc0ac\uace0 \uae30\uc220\uc870\uc0ac\uc704\uc6d0\ud68c'\uac00 \ucd9c\ubc94\ud558\uc5ec \ub3c4\ucfc4 \ub300\ud559\uc758 \ub9c8\uc4f0\ubaa8\ud1a0 \uc694\uc2dc\uc9c0 \uad50\uc218\uac00 \uc704\uc6d0\uc7a5\uc744 \ub9e1\uc558\ub2e4. \uae30\uc220\uc870\uc0ac\uc704\uc6d0\ud68c\ub294 \ubbfc\uc601\ud654 \uc774\ud6c4\uc778 1988\ub144 2\uc6d4 5\uc77c \uc870\uc0ac \ubcf4\uace0\uc11c\ub97c \ub0b4\ub193\uc558\ub2e4. \uc870\uc0ac \ubcf4\uace0\uc11c\uc5d0 \ub530\ub974\uba74 \ub2e4\ub9ac\uc5d0 \uc124\uce58\ub418\uc5b4 \uc788\ub358 \ub450 \ub300\uc758 \ud48d\uc18d\uacc4 \uc911 \ud55c \ub300\uac00 \uace0\uc7a5\uc774 \ub098\uc788\uc5c8\uc73c\uba70, \ub2e4\ub978 \ud55c \ub300\ub3c4 \uc624\ub798\ub418\uc5b4 \ubd88\uc548\uc815\ud55c \uc0c1\ud0dc\uc600\ub2e4. \uae30\uc220\uc870\uc0ac\uc704\uc6d0\ud68c\ub294 \ud48d\uc18d\uacc4\uac00 \uacbd\ubcf4\ub97c \uc6b8\ub9b0 \ub4a4\uc5d0 \uc9c0\ub839\uc6d0\uc758 \ud310\ub2e8\uc744 \ud1b5\ud574 \uc5f4\ucc28\ub97c \uc815\uc9c0\uc2dc\ud0a4\uae30\ub85c \ud55c \uac83\ub3c4 \ubb38\uc81c \uc0bc\uc558\uc73c\uba70 \uc790\ub3d9\uc801\uc73c\ub85c \uc5f4\ucc28\ub97c \uc815\uc9c0\uc2dc\ud0a4\ub3c4\ub85d \ud588\uc5b4\uc57c \ud55c\ub2e4\uace0 \ubc1d\ud614\ub2e4. \ud55c\ud3b8 \uad6c\ubcf4\ud0c0 \ud788\ub85c\uc2dc\uac00 \uc4f4 \u300a\uc911\ub300\ud55c \ucca0\ub3c4 \uc0ac\uace0\uc758 \uc5ed\uc0ac\u300b(\u9244\u9053\u91cd\u5927\u4e8b\u6545\u306e\u6b74\u53f2)\uc5d0\ub294 \ud48d\uc18d 33 m/s\uc758 \ubc14\ub78c\uc73c\ub85c\ub294 \uacc4\uc0b0 \uc0c1 \uc5f4\ucc28\uac00 \ub4a4\uc9d1\uc5b4\uc9c0\uc9c0 \uc54a\uc73c\uba70, \ub2e4\ub9ac \uc704\uc758 \uada4\ub3c4\uac00 \ubc14\ub78c\uc774 \ubd80\ub294 \ubc29\ud5a5\uacfc\ub294 \uc815\ubc18\ub300\uc778 \ubc14\ub2e4 \ucabd\uc73c\ub85c \ud718\uc5b4\uc838 \uc788\ub358 \uc810\uc73c\ub85c \ubcf4\uc544 \uc0ac\uace0\uc758 \uc6d0\uc778\uc740 \ubc14\ub78c\uc5d0 \uc788\uc9c0 \uc54a\uc73c\uba70, \ubd80\uc2e4\ud55c \ubcf4\uc218 \uacf5\uc0ac\uc5d0 \uc788\ub2e4\ub294 \uc704\uc6d0\ud68c\uc758 \uc9c0\uc801\uacfc\ub294 \ub2e4\uc18c \ub2e4\ub978 \uc8fc\uc7a5\uc774 \uc2e4\ub824 \uc788\ub2e4.", "answer": "1987\ub144 2\uc6d4 9\uc77c", "sentence": "1987\ub144 2\uc6d4 9\uc77c '\uc544\ub9c8\ub8e8\ubca0 \uc0ac\uace0 \uae30\uc220\uc870\uc0ac\uc704\uc6d0\ud68c'\uac00 \ucd9c\ubc94\ud558\uc5ec \ub3c4\ucfc4 \ub300\ud559\uc758 \ub9c8\uc4f0\ubaa8\ud1a0 \uc694\uc2dc\uc9c0 \uad50\uc218\uac00 \uc704\uc6d0\uc7a5\uc744 \ub9e1\uc558\ub2e4.", "paragraph_sentence": " 1987\ub144 2\uc6d4 9\uc77c '\uc544\ub9c8\ub8e8\ubca0 \uc0ac\uace0 \uae30\uc220\uc870\uc0ac\uc704\uc6d0\ud68c'\uac00 \ucd9c\ubc94\ud558\uc5ec \ub3c4\ucfc4 \ub300\ud559\uc758 \ub9c8\uc4f0\ubaa8\ud1a0 \uc694\uc2dc\uc9c0 \uad50\uc218\uac00 \uc704\uc6d0\uc7a5\uc744 \ub9e1\uc558\ub2e4. \uae30\uc220\uc870\uc0ac\uc704\uc6d0\ud68c\ub294 \ubbfc\uc601\ud654 \uc774\ud6c4\uc778 1988\ub144 2\uc6d4 5\uc77c \uc870\uc0ac \ubcf4\uace0\uc11c\ub97c \ub0b4\ub193\uc558\ub2e4. \uc870\uc0ac \ubcf4\uace0\uc11c\uc5d0 \ub530\ub974\uba74 \ub2e4\ub9ac\uc5d0 \uc124\uce58\ub418\uc5b4 \uc788\ub358 \ub450 \ub300\uc758 \ud48d\uc18d\uacc4 \uc911 \ud55c \ub300\uac00 \uace0\uc7a5\uc774 \ub098\uc788\uc5c8\uc73c\uba70, \ub2e4\ub978 \ud55c \ub300\ub3c4 \uc624\ub798\ub418\uc5b4 \ubd88\uc548\uc815\ud55c \uc0c1\ud0dc\uc600\ub2e4. \uae30\uc220\uc870\uc0ac\uc704\uc6d0\ud68c\ub294 \ud48d\uc18d\uacc4\uac00 \uacbd\ubcf4\ub97c \uc6b8\ub9b0 \ub4a4\uc5d0 \uc9c0\ub839\uc6d0\uc758 \ud310\ub2e8\uc744 \ud1b5\ud574 \uc5f4\ucc28\ub97c \uc815\uc9c0\uc2dc\ud0a4\uae30\ub85c \ud55c \uac83\ub3c4 \ubb38\uc81c \uc0bc\uc558\uc73c\uba70 \uc790\ub3d9\uc801\uc73c\ub85c \uc5f4\ucc28\ub97c \uc815\uc9c0\uc2dc\ud0a4\ub3c4\ub85d \ud588\uc5b4\uc57c \ud55c\ub2e4\uace0 \ubc1d\ud614\ub2e4. \ud55c\ud3b8 \uad6c\ubcf4\ud0c0 \ud788\ub85c\uc2dc\uac00 \uc4f4 \u300a\uc911\ub300\ud55c \ucca0\ub3c4 \uc0ac\uace0\uc758 \uc5ed\uc0ac\u300b(\u9244\u9053\u91cd\u5927\u4e8b\u6545\u306e\u6b74\u53f2)\uc5d0\ub294 \ud48d\uc18d 33 m/s\uc758 \ubc14\ub78c\uc73c\ub85c\ub294 \uacc4\uc0b0 \uc0c1 \uc5f4\ucc28\uac00 \ub4a4\uc9d1\uc5b4\uc9c0\uc9c0 \uc54a\uc73c\uba70, \ub2e4\ub9ac \uc704\uc758 \uada4\ub3c4\uac00 \ubc14\ub78c\uc774 \ubd80\ub294 \ubc29\ud5a5\uacfc\ub294 \uc815\ubc18\ub300\uc778 \ubc14\ub2e4 \ucabd\uc73c\ub85c \ud718\uc5b4\uc838 \uc788\ub358 \uc810\uc73c\ub85c \ubcf4\uc544 \uc0ac\uace0\uc758 \uc6d0\uc778\uc740 \ubc14\ub78c\uc5d0 \uc788\uc9c0 \uc54a\uc73c\uba70, \ubd80\uc2e4\ud55c \ubcf4\uc218 \uacf5\uc0ac\uc5d0 \uc788\ub2e4\ub294 \uc704\uc6d0\ud68c\uc758 \uc9c0\uc801\uacfc\ub294 \ub2e4\uc18c \ub2e4\ub978 \uc8fc\uc7a5\uc774 \uc2e4\ub824 \uc788\ub2e4.", "paragraph_answer": " 1987\ub144 2\uc6d4 9\uc77c '\uc544\ub9c8\ub8e8\ubca0 \uc0ac\uace0 \uae30\uc220\uc870\uc0ac\uc704\uc6d0\ud68c'\uac00 \ucd9c\ubc94\ud558\uc5ec \ub3c4\ucfc4 \ub300\ud559\uc758 \ub9c8\uc4f0\ubaa8\ud1a0 \uc694\uc2dc\uc9c0 \uad50\uc218\uac00 \uc704\uc6d0\uc7a5\uc744 \ub9e1\uc558\ub2e4. \uae30\uc220\uc870\uc0ac\uc704\uc6d0\ud68c\ub294 \ubbfc\uc601\ud654 \uc774\ud6c4\uc778 1988\ub144 2\uc6d4 5\uc77c \uc870\uc0ac \ubcf4\uace0\uc11c\ub97c \ub0b4\ub193\uc558\ub2e4. \uc870\uc0ac \ubcf4\uace0\uc11c\uc5d0 \ub530\ub974\uba74 \ub2e4\ub9ac\uc5d0 \uc124\uce58\ub418\uc5b4 \uc788\ub358 \ub450 \ub300\uc758 \ud48d\uc18d\uacc4 \uc911 \ud55c \ub300\uac00 \uace0\uc7a5\uc774 \ub098\uc788\uc5c8\uc73c\uba70, \ub2e4\ub978 \ud55c \ub300\ub3c4 \uc624\ub798\ub418\uc5b4 \ubd88\uc548\uc815\ud55c \uc0c1\ud0dc\uc600\ub2e4. \uae30\uc220\uc870\uc0ac\uc704\uc6d0\ud68c\ub294 \ud48d\uc18d\uacc4\uac00 \uacbd\ubcf4\ub97c \uc6b8\ub9b0 \ub4a4\uc5d0 \uc9c0\ub839\uc6d0\uc758 \ud310\ub2e8\uc744 \ud1b5\ud574 \uc5f4\ucc28\ub97c \uc815\uc9c0\uc2dc\ud0a4\uae30\ub85c \ud55c \uac83\ub3c4 \ubb38\uc81c \uc0bc\uc558\uc73c\uba70 \uc790\ub3d9\uc801\uc73c\ub85c \uc5f4\ucc28\ub97c \uc815\uc9c0\uc2dc\ud0a4\ub3c4\ub85d \ud588\uc5b4\uc57c \ud55c\ub2e4\uace0 \ubc1d\ud614\ub2e4. \ud55c\ud3b8 \uad6c\ubcf4\ud0c0 \ud788\ub85c\uc2dc\uac00 \uc4f4 \u300a\uc911\ub300\ud55c \ucca0\ub3c4 \uc0ac\uace0\uc758 \uc5ed\uc0ac\u300b(\u9244\u9053\u91cd\u5927\u4e8b\u6545\u306e\u6b74\u53f2)\uc5d0\ub294 \ud48d\uc18d 33 m/s\uc758 \ubc14\ub78c\uc73c\ub85c\ub294 \uacc4\uc0b0 \uc0c1 \uc5f4\ucc28\uac00 \ub4a4\uc9d1\uc5b4\uc9c0\uc9c0 \uc54a\uc73c\uba70, \ub2e4\ub9ac \uc704\uc758 \uada4\ub3c4\uac00 \ubc14\ub78c\uc774 \ubd80\ub294 \ubc29\ud5a5\uacfc\ub294 \uc815\ubc18\ub300\uc778 \ubc14\ub2e4 \ucabd\uc73c\ub85c \ud718\uc5b4\uc838 \uc788\ub358 \uc810\uc73c\ub85c \ubcf4\uc544 \uc0ac\uace0\uc758 \uc6d0\uc778\uc740 \ubc14\ub78c\uc5d0 \uc788\uc9c0 \uc54a\uc73c\uba70, \ubd80\uc2e4\ud55c \ubcf4\uc218 \uacf5\uc0ac\uc5d0 \uc788\ub2e4\ub294 \uc704\uc6d0\ud68c\uc758 \uc9c0\uc801\uacfc\ub294 \ub2e4\uc18c \ub2e4\ub978 \uc8fc\uc7a5\uc774 \uc2e4\ub824 \uc788\ub2e4.", "sentence_answer": " 1987\ub144 2\uc6d4 9\uc77c '\uc544\ub9c8\ub8e8\ubca0 \uc0ac\uace0 \uae30\uc220\uc870\uc0ac\uc704\uc6d0\ud68c'\uac00 \ucd9c\ubc94\ud558\uc5ec \ub3c4\ucfc4 \ub300\ud559\uc758 \ub9c8\uc4f0\ubaa8\ud1a0 \uc694\uc2dc\uc9c0 \uad50\uc218\uac00 \uc704\uc6d0\uc7a5\uc744 \ub9e1\uc558\ub2e4.", "paragraph_id": "6547282-15-1", "paragraph_question": "question: '\uc544\ub9c8\ub8e8\ubca0 \uc0ac\uace0 \uae30\uc220\uc870\uc0ac\uc704\uc6d0\ud68c'\uac00 \ucd9c\ubc94\ud55c \ub0a0\uc9dc\ub294 \uc5b8\uc81c\uc778\uac00?, context: 1987\ub144 2\uc6d4 9\uc77c '\uc544\ub9c8\ub8e8\ubca0 \uc0ac\uace0 \uae30\uc220\uc870\uc0ac\uc704\uc6d0\ud68c'\uac00 \ucd9c\ubc94\ud558\uc5ec \ub3c4\ucfc4 \ub300\ud559\uc758 \ub9c8\uc4f0\ubaa8\ud1a0 \uc694\uc2dc\uc9c0 \uad50\uc218\uac00 \uc704\uc6d0\uc7a5\uc744 \ub9e1\uc558\ub2e4. \uae30\uc220\uc870\uc0ac\uc704\uc6d0\ud68c\ub294 \ubbfc\uc601\ud654 \uc774\ud6c4\uc778 1988\ub144 2\uc6d4 5\uc77c \uc870\uc0ac \ubcf4\uace0\uc11c\ub97c \ub0b4\ub193\uc558\ub2e4. \uc870\uc0ac \ubcf4\uace0\uc11c\uc5d0 \ub530\ub974\uba74 \ub2e4\ub9ac\uc5d0 \uc124\uce58\ub418\uc5b4 \uc788\ub358 \ub450 \ub300\uc758 \ud48d\uc18d\uacc4 \uc911 \ud55c \ub300\uac00 \uace0\uc7a5\uc774 \ub098\uc788\uc5c8\uc73c\uba70, \ub2e4\ub978 \ud55c \ub300\ub3c4 \uc624\ub798\ub418\uc5b4 \ubd88\uc548\uc815\ud55c \uc0c1\ud0dc\uc600\ub2e4. \uae30\uc220\uc870\uc0ac\uc704\uc6d0\ud68c\ub294 \ud48d\uc18d\uacc4\uac00 \uacbd\ubcf4\ub97c \uc6b8\ub9b0 \ub4a4\uc5d0 \uc9c0\ub839\uc6d0\uc758 \ud310\ub2e8\uc744 \ud1b5\ud574 \uc5f4\ucc28\ub97c \uc815\uc9c0\uc2dc\ud0a4\uae30\ub85c \ud55c \uac83\ub3c4 \ubb38\uc81c \uc0bc\uc558\uc73c\uba70 \uc790\ub3d9\uc801\uc73c\ub85c \uc5f4\ucc28\ub97c \uc815\uc9c0\uc2dc\ud0a4\ub3c4\ub85d \ud588\uc5b4\uc57c \ud55c\ub2e4\uace0 \ubc1d\ud614\ub2e4. \ud55c\ud3b8 \uad6c\ubcf4\ud0c0 \ud788\ub85c\uc2dc\uac00 \uc4f4 \u300a\uc911\ub300\ud55c \ucca0\ub3c4 \uc0ac\uace0\uc758 \uc5ed\uc0ac\u300b(\u9244\u9053\u91cd\u5927\u4e8b\u6545\u306e\u6b74\u53f2)\uc5d0\ub294 \ud48d\uc18d 33 m/s\uc758 \ubc14\ub78c\uc73c\ub85c\ub294 \uacc4\uc0b0 \uc0c1 \uc5f4\ucc28\uac00 \ub4a4\uc9d1\uc5b4\uc9c0\uc9c0 \uc54a\uc73c\uba70, \ub2e4\ub9ac \uc704\uc758 \uada4\ub3c4\uac00 \ubc14\ub78c\uc774 \ubd80\ub294 \ubc29\ud5a5\uacfc\ub294 \uc815\ubc18\ub300\uc778 \ubc14\ub2e4 \ucabd\uc73c\ub85c \ud718\uc5b4\uc838 \uc788\ub358 \uc810\uc73c\ub85c \ubcf4\uc544 \uc0ac\uace0\uc758 \uc6d0\uc778\uc740 \ubc14\ub78c\uc5d0 \uc788\uc9c0 \uc54a\uc73c\uba70, \ubd80\uc2e4\ud55c \ubcf4\uc218 \uacf5\uc0ac\uc5d0 \uc788\ub2e4\ub294 \uc704\uc6d0\ud68c\uc758 \uc9c0\uc801\uacfc\ub294 \ub2e4\uc18c \ub2e4\ub978 \uc8fc\uc7a5\uc774 \uc2e4\ub824 \uc788\ub2e4."} {"question": "11\uc6d4 27\uc77c JELLY JELLY\uc758 \uc2a4\ud398\uc15c \ubb34\ub300\ub97c \uac00\uc9c4 \ubc29\uc1a1\uad6d\uc740?", "paragraph": "11\uc6d4 14\uc77c, \"TT\"\uc758 \uc720\ud29c\ube0c \ubba4\uc9c1 \ube44\ub514\uc624 \uc870\ud68c\uc218\uac00 \uacf5\uac1c 22\uc77c \ub9cc\uc5d0 5,000\ub9cc\ubdf0\ub97c \ub3cc\ud30c\ud558\uba70 K-Pop \uc544\uc774\ub3cc \uc0ac\uc0c1 \ucd5c\ub2e8 \uae30\uac04 \ub3cc\ud30c\ub77c\ub294 \uae30\ub85d\uc744 \uc138\uc6e0\ub2e4. 17\uc77c \uc624\uc804 6\uc2dc 40\ubd84\uacbd \"CHEER UP\"\uc758 \uc720\ud29c\ube0c \ubba4\uc9c1 \ube44\ub514\uc624 \uc870\ud68c\uc218\uac00 1\uc5b5 \ubdf0\ub97c \ub3cc\ud30c\ud558\uc600\uc73c\uba70, \uc774\ub294 \ucf00\uc774\ud31d \uc544\uc774\ub3cc \uac00\uc6b4\ub370 \ucd5c\ub2e8 \uae30\uac04\uc778 207\uc77c \ub9cc\uc5d0 1\uc5b5\ubdf0\ub97c \ub3cc\ud30c \ud55c \uae30\ub85d\uc774\ub2e4. \ub610\ud55c \"OOH-AHH\ud558\uac8c (\uc6b0\uc544\ud558\uac8c)\"\uc5d0 \uc774\uc5b4 \"CHEER UP\"\uc758 \uc720\ud29c\ube0c \ubba4\uc9c1 \ube44\ub514\uc624\uac00 \uc870\ud68c\uc218\uac00 1\uc5b5\ubdf0\ub97c \ub3cc\ud30c\ud568\uc73c\ub85c\uc11c, 2\uace1 \uc5f0\uc18d 1\uc5b5\ubdf0 \ub3cc\ud30c\ub77c\ub294 \uae30\ub85d\uc744 \uc138\uc6e0\ub2e4. 11\uc6d4 26\uc77c MBC \u300a\uc1fc! \uc74c\uc545\uc911\uc2ec\u300b\uc5d0\uc11c \"TT\"\uc758 \ubaa8\ub4e0 \uc74c\uc545 \ubc29\uc1a1 \ud65c\ub3d9\uc744 \ub9c8\ubb34\ub9ac \ud558\uc600\uace0, 11\uc6d4 27\uc77c SBS \u300a\uc778\uae30\uac00\uc694\u300b\uc5d0\uc11c \uc568\ubc94\uc758 \uc218\ub85d\uace1\uc778 \"JELLY JELLY\"\uc758 \uc2a4\ud398\uc15c \ubb34\ub300\ub97c \uac00\uc84c\ub2e4.", "answer": "SBS", "sentence": "\uc74c\uc545\uc911\uc2ec\u300b\uc5d0\uc11c \"TT\"\uc758 \ubaa8\ub4e0 \uc74c\uc545 \ubc29\uc1a1 \ud65c\ub3d9\uc744 \ub9c8\ubb34\ub9ac \ud558\uc600\uace0, 11\uc6d4 27\uc77c SBS \u300a\uc778\uae30\uac00\uc694\u300b\uc5d0\uc11c \uc568\ubc94\uc758 \uc218\ub85d\uace1\uc778 \"JELLY JELLY\"\uc758 \uc2a4\ud398\uc15c \ubb34\ub300\ub97c \uac00\uc84c\ub2e4.", "paragraph_sentence": "11\uc6d4 14\uc77c, \"TT\"\uc758 \uc720\ud29c\ube0c \ubba4\uc9c1 \ube44\ub514\uc624 \uc870\ud68c\uc218\uac00 \uacf5\uac1c 22\uc77c \ub9cc\uc5d0 5,000\ub9cc\ubdf0\ub97c \ub3cc\ud30c\ud558\uba70 K-Pop \uc544\uc774\ub3cc \uc0ac\uc0c1 \ucd5c\ub2e8 \uae30\uac04 \ub3cc\ud30c\ub77c\ub294 \uae30\ub85d\uc744 \uc138\uc6e0\ub2e4. 17\uc77c \uc624\uc804 6\uc2dc 40\ubd84\uacbd \"CHEER UP\"\uc758 \uc720\ud29c\ube0c \ubba4\uc9c1 \ube44\ub514\uc624 \uc870\ud68c\uc218\uac00 1\uc5b5 \ubdf0\ub97c \ub3cc\ud30c\ud558\uc600\uc73c\uba70, \uc774\ub294 \ucf00\uc774\ud31d \uc544\uc774\ub3cc \uac00\uc6b4\ub370 \ucd5c\ub2e8 \uae30\uac04\uc778 207\uc77c \ub9cc\uc5d0 1\uc5b5\ubdf0\ub97c \ub3cc\ud30c \ud55c \uae30\ub85d\uc774\ub2e4. \ub610\ud55c \"OOH-AHH\ud558\uac8c (\uc6b0\uc544\ud558\uac8c)\"\uc5d0 \uc774\uc5b4 \"CHEER UP\"\uc758 \uc720\ud29c\ube0c \ubba4\uc9c1 \ube44\ub514\uc624\uac00 \uc870\ud68c\uc218\uac00 1\uc5b5\ubdf0\ub97c \ub3cc\ud30c\ud568\uc73c\ub85c\uc11c, 2\uace1 \uc5f0\uc18d 1\uc5b5\ubdf0 \ub3cc\ud30c\ub77c\ub294 \uae30\ub85d\uc744 \uc138\uc6e0\ub2e4. 11\uc6d4 26\uc77c MBC \u300a\uc1fc! \uc74c\uc545\uc911\uc2ec\u300b\uc5d0\uc11c \"TT\"\uc758 \ubaa8\ub4e0 \uc74c\uc545 \ubc29\uc1a1 \ud65c\ub3d9\uc744 \ub9c8\ubb34\ub9ac \ud558\uc600\uace0, 11\uc6d4 27\uc77c SBS \u300a\uc778\uae30\uac00\uc694\u300b\uc5d0\uc11c \uc568\ubc94\uc758 \uc218\ub85d\uace1\uc778 \"JELLY JELLY\"\uc758 \uc2a4\ud398\uc15c \ubb34\ub300\ub97c \uac00\uc84c\ub2e4. ", "paragraph_answer": "11\uc6d4 14\uc77c, \"TT\"\uc758 \uc720\ud29c\ube0c \ubba4\uc9c1 \ube44\ub514\uc624 \uc870\ud68c\uc218\uac00 \uacf5\uac1c 22\uc77c \ub9cc\uc5d0 5,000\ub9cc\ubdf0\ub97c \ub3cc\ud30c\ud558\uba70 K-Pop \uc544\uc774\ub3cc \uc0ac\uc0c1 \ucd5c\ub2e8 \uae30\uac04 \ub3cc\ud30c\ub77c\ub294 \uae30\ub85d\uc744 \uc138\uc6e0\ub2e4. 17\uc77c \uc624\uc804 6\uc2dc 40\ubd84\uacbd \"CHEER UP\"\uc758 \uc720\ud29c\ube0c \ubba4\uc9c1 \ube44\ub514\uc624 \uc870\ud68c\uc218\uac00 1\uc5b5 \ubdf0\ub97c \ub3cc\ud30c\ud558\uc600\uc73c\uba70, \uc774\ub294 \ucf00\uc774\ud31d \uc544\uc774\ub3cc \uac00\uc6b4\ub370 \ucd5c\ub2e8 \uae30\uac04\uc778 207\uc77c \ub9cc\uc5d0 1\uc5b5\ubdf0\ub97c \ub3cc\ud30c \ud55c \uae30\ub85d\uc774\ub2e4. \ub610\ud55c \"OOH-AHH\ud558\uac8c (\uc6b0\uc544\ud558\uac8c)\"\uc5d0 \uc774\uc5b4 \"CHEER UP\"\uc758 \uc720\ud29c\ube0c \ubba4\uc9c1 \ube44\ub514\uc624\uac00 \uc870\ud68c\uc218\uac00 1\uc5b5\ubdf0\ub97c \ub3cc\ud30c\ud568\uc73c\ub85c\uc11c, 2\uace1 \uc5f0\uc18d 1\uc5b5\ubdf0 \ub3cc\ud30c\ub77c\ub294 \uae30\ub85d\uc744 \uc138\uc6e0\ub2e4. 11\uc6d4 26\uc77c MBC \u300a\uc1fc! \uc74c\uc545\uc911\uc2ec\u300b\uc5d0\uc11c \"TT\"\uc758 \ubaa8\ub4e0 \uc74c\uc545 \ubc29\uc1a1 \ud65c\ub3d9\uc744 \ub9c8\ubb34\ub9ac \ud558\uc600\uace0, 11\uc6d4 27\uc77c SBS \u300a\uc778\uae30\uac00\uc694\u300b\uc5d0\uc11c \uc568\ubc94\uc758 \uc218\ub85d\uace1\uc778 \"JELLY JELLY\"\uc758 \uc2a4\ud398\uc15c \ubb34\ub300\ub97c \uac00\uc84c\ub2e4.", "sentence_answer": "\uc74c\uc545\uc911\uc2ec\u300b\uc5d0\uc11c \"TT\"\uc758 \ubaa8\ub4e0 \uc74c\uc545 \ubc29\uc1a1 \ud65c\ub3d9\uc744 \ub9c8\ubb34\ub9ac \ud558\uc600\uace0, 11\uc6d4 27\uc77c SBS \u300a\uc778\uae30\uac00\uc694\u300b\uc5d0\uc11c \uc568\ubc94\uc758 \uc218\ub85d\uace1\uc778 \"JELLY JELLY\"\uc758 \uc2a4\ud398\uc15c \ubb34\ub300\ub97c \uac00\uc84c\ub2e4.", "paragraph_id": "6566343-3-2", "paragraph_question": "question: 11\uc6d4 27\uc77c JELLY JELLY\uc758 \uc2a4\ud398\uc15c \ubb34\ub300\ub97c \uac00\uc9c4 \ubc29\uc1a1\uad6d\uc740?, context: 11\uc6d4 14\uc77c, \"TT\"\uc758 \uc720\ud29c\ube0c \ubba4\uc9c1 \ube44\ub514\uc624 \uc870\ud68c\uc218\uac00 \uacf5\uac1c 22\uc77c \ub9cc\uc5d0 5,000\ub9cc\ubdf0\ub97c \ub3cc\ud30c\ud558\uba70 K-Pop \uc544\uc774\ub3cc \uc0ac\uc0c1 \ucd5c\ub2e8 \uae30\uac04 \ub3cc\ud30c\ub77c\ub294 \uae30\ub85d\uc744 \uc138\uc6e0\ub2e4. 17\uc77c \uc624\uc804 6\uc2dc 40\ubd84\uacbd \"CHEER UP\"\uc758 \uc720\ud29c\ube0c \ubba4\uc9c1 \ube44\ub514\uc624 \uc870\ud68c\uc218\uac00 1\uc5b5 \ubdf0\ub97c \ub3cc\ud30c\ud558\uc600\uc73c\uba70, \uc774\ub294 \ucf00\uc774\ud31d \uc544\uc774\ub3cc \uac00\uc6b4\ub370 \ucd5c\ub2e8 \uae30\uac04\uc778 207\uc77c \ub9cc\uc5d0 1\uc5b5\ubdf0\ub97c \ub3cc\ud30c \ud55c \uae30\ub85d\uc774\ub2e4. \ub610\ud55c \"OOH-AHH\ud558\uac8c (\uc6b0\uc544\ud558\uac8c)\"\uc5d0 \uc774\uc5b4 \"CHEER UP\"\uc758 \uc720\ud29c\ube0c \ubba4\uc9c1 \ube44\ub514\uc624\uac00 \uc870\ud68c\uc218\uac00 1\uc5b5\ubdf0\ub97c \ub3cc\ud30c\ud568\uc73c\ub85c\uc11c, 2\uace1 \uc5f0\uc18d 1\uc5b5\ubdf0 \ub3cc\ud30c\ub77c\ub294 \uae30\ub85d\uc744 \uc138\uc6e0\ub2e4. 11\uc6d4 26\uc77c MBC \u300a\uc1fc! \uc74c\uc545\uc911\uc2ec\u300b\uc5d0\uc11c \"TT\"\uc758 \ubaa8\ub4e0 \uc74c\uc545 \ubc29\uc1a1 \ud65c\ub3d9\uc744 \ub9c8\ubb34\ub9ac \ud558\uc600\uace0, 11\uc6d4 27\uc77c SBS \u300a\uc778\uae30\uac00\uc694\u300b\uc5d0\uc11c \uc568\ubc94\uc758 \uc218\ub85d\uace1\uc778 \"JELLY JELLY\"\uc758 \uc2a4\ud398\uc15c \ubb34\ub300\ub97c \uac00\uc84c\ub2e4."} {"question": "\uae30\uc6d0\uc804 23\ub144 \uc544\uc6b0\uad6c\uc2a4\uc5d0 \ub300\ud56d\ud558\uc600\uc73c\ub098 \uc784\uae30\ub97c \ub9c8\uce58\uc9c0 \ubabb\ud558\uace0 \uc9d1\uc815\uad00\uc5d0\uc11c \ub0b4\ub824\uc628 \uc778\ubb3c\uc740 \ub204\uad6c\uc778\uac00?", "paragraph": "\uae30\uc6d0\uc804 23\ub144 \uc544\uc6b0\uad6c\uc2a4\ud22c\uc2a4\uc758 \ub3d9\ub8cc \uc9d1\uc815\uad00\uc774\uc5c8\ub358 \ud14c\ub80c\ud2f0\uc6b0\uc2a4 \ubc14\ub85c \ubb34\ub808\ub098(Terentius Varro Murena)\uac00 \uc544\uc6b0\uad6c\uc2a4\ud22c\uc2a4\uc5d0 \ub300\ud56d\ud558\ub824 \ud55c\ub2e4. \uadf8 \ubc29\uc2dd\uc740 \uc815\ud655\ud558\uac8c \uc804\ud574\uc9c0\uace0 \uc788\uc9c0 \uc54a\uc73c\ub098 \ubb34\ub808\ub098\ub294 \uc784\uae30\ub97c \ub9c8\uce58\uc9c0 \ubabb\ud558\uace0 \uce7c\ud478\ub974\ub2c8\uc6b0\uc2a4 \ud53c\uc18c(Calpurnius Piso)\uac00 \uadf8 \uc790\ub9ac\ub97c \ub300\uc2e0\ud558\uac8c \ub41c\ub2e4. \ud53c\uc18c\ub294 \ub110\ub9ac \uc54c\ub824\uc9c4 \uacf5\ud654\uc815 \uc9c0\uc9c0\uc790\uc600\ub294\ub370, \uc544\uc6b0\uad6c\uc2a4\ud22c\uc2a4\ub294 \ub3d9\ub8cc \uc9d1\uc815\uad00\uacfc \ud611\ub825\ud558\uba74\uc11c \ud30c\ubc8c\uc5d0 \uad00\uacc4\uc5c6\uc774 \ubaa8\ub4e0 \uc774\ub4e4\uacfc \ud611\ub825\ud558\uc5ec \uad6d\uc815\uc744 \uc6b4\uc601\ud558\uace0\uc790 \ud558\ub294 \ubaa8\uc2b5\uc744 \ubcf4\uc5ec\uc8fc\uc5c8\ub2e4. \ub2a6\ubd04\uc5d0 \uc544\uc6b0\uad6c\uc2a4\ud22c\uc2a4\ub294 \uc790\uce6b\ud558\uba74 \uc8fd\uc744 \uc218 \uc788\uc5c8\ub358 \uc815\ub3c4\ub85c \uc2ec\ud558\uac8c \ubcd1\uc744 \uc553\uc558\uace0, \uc0ac\ub78c\ub4e4\uc740 \uc544\uc6b0\uad6c\uc2a4\ud22c\uc2a4\uac00 \uce5c\uad6c\uc778 \uc544\uadf8\ub9ac\ud30c\uc5d0\uac8c \uc790\uc2e0\uc758 \uc778\uc7a5\uc744 \ub118\uae30\uace0 \uc0ac\uc704\uc778 \ub9c8\ub974\ucf08\ub8e8\uc2a4\uc5d0\uac8c \ubcd1\uad8c\uc744 \uc704\uc784\ud560 \uac83\uc774\ub77c \uc0dd\uac01\ud588\ub2e4. \ud558\uc9c0\ub9cc \uc544\uc6b0\uad6c\uc2a4\ud22c\uc2a4\ub294 \uc790\uc2e0\uc758 \uc778\uc7a5\uacfc, \uad00\ub9ac\ud558\uace0 \uc788\ub358 \uacf5\uae08 \ubc0f \uad70\ub2e8 \ud1b5\uc81c\uad8c\uc744 \ub3d9\ub8cc \uc9d1\uc815\uad00\uc778 \ud53c\uc18c\uc5d0\uac8c \uc704\uc784\ud588\ub2e4. \uc774\ub7ec\ud55c \ud589\ub3d9\uc740 \uc544\uc6b0\uad6c\uc2a4\ud22c\uc2a4\uac00 \uc0ac\uc2e4\uc0c1\uc758 \ud669\uc81c\ub77c\uace0 \ubbff\uace0 \uc788\ub358 \uc0ac\ub78c\ub4e4\uc5d0\uac8c\ub294 \ub180\ub77c\uc6b4 \uc77c\uc774\uc5c8\uace0, \uc6d0\ub85c\uc6d0\uc740 \uc544\uc6b0\uad6c\uc2a4\ud22c\uc2a4\uac00 \uc81c\uc815\uc73c\ub85c \ub098\uc544\uac00\uace0 \uc788\ub2e4\ub294 \uc758\uc2ec\uc744 \uc7a0\uc2dc \uac70\ub454\ub2e4. \uc544\uc6b0\uad6c\uc2a4\ud22c\uc2a4\ub294 \uc0ac\uc720 \uc7ac\uc0b0\ub9cc \uc790\uc2e0\uc774 \uc9c0\ubaa9\ud55c \uc0c1\uc18d\uc790\ub4e4\uc5d0\uac8c \uc218\uc5ec\ud558\ub824 \ud588\ub2e4. \ub2f9\uc2dc \ub85c\ub9c8\uc758 \uc2dc\ubbfc\ub4e4\uc740 \uc5ec\uc804\ud788 \uad70\uc8fc\uc815\uc744 \uc88b\uac8c \ubcf4\uc9c0 \uc54a\uc558\uae30 \ub54c\ubb38\uc5d0 \uc544\uc6b0\uad6c\uc2a4\ud22c\uc2a4\uac00 \ub9cc\uc57d \uad8c\ub825\uc744 \uc790\uc2e0\uc774 \uc9c0\uba85\ud55c \ud6c4\uacc4\uc790\uc5d0\uac8c \ubb3c\ub824\uc8fc\ub824 \ud588\ub2e4\uba74 \ubc18\ub780\uc774 \uc77c\uc5b4\ub0ac\uc744 \uac83\uc774\ub2e4.", "answer": "\ud14c\ub80c\ud2f0\uc6b0\uc2a4 \ubc14\ub85c \ubb34\ub808\ub098", "sentence": "\uae30\uc6d0\uc804 23\ub144 \uc544\uc6b0\uad6c\uc2a4\ud22c\uc2a4\uc758 \ub3d9\ub8cc \uc9d1\uc815\uad00\uc774\uc5c8\ub358 \ud14c\ub80c\ud2f0\uc6b0\uc2a4 \ubc14\ub85c \ubb34\ub808\ub098 (Terentius Varro Murena)", "paragraph_sentence": " \uae30\uc6d0\uc804 23\ub144 \uc544\uc6b0\uad6c\uc2a4\ud22c\uc2a4\uc758 \ub3d9\ub8cc \uc9d1\uc815\uad00\uc774\uc5c8\ub358 \ud14c\ub80c\ud2f0\uc6b0\uc2a4 \ubc14\ub85c \ubb34\ub808\ub098 (Terentius Varro Murena) \uac00 \uc544\uc6b0\uad6c\uc2a4\ud22c\uc2a4\uc5d0 \ub300\ud56d\ud558\ub824 \ud55c\ub2e4. \uadf8 \ubc29\uc2dd\uc740 \uc815\ud655\ud558\uac8c \uc804\ud574\uc9c0\uace0 \uc788\uc9c0 \uc54a\uc73c\ub098 \ubb34\ub808\ub098\ub294 \uc784\uae30\ub97c \ub9c8\uce58\uc9c0 \ubabb\ud558\uace0 \uce7c\ud478\ub974\ub2c8\uc6b0\uc2a4 \ud53c\uc18c(Calpurnius Piso)\uac00 \uadf8 \uc790\ub9ac\ub97c \ub300\uc2e0\ud558\uac8c \ub41c\ub2e4. \ud53c\uc18c\ub294 \ub110\ub9ac \uc54c\ub824\uc9c4 \uacf5\ud654\uc815 \uc9c0\uc9c0\uc790\uc600\ub294\ub370, \uc544\uc6b0\uad6c\uc2a4\ud22c\uc2a4\ub294 \ub3d9\ub8cc \uc9d1\uc815\uad00\uacfc \ud611\ub825\ud558\uba74\uc11c \ud30c\ubc8c\uc5d0 \uad00\uacc4\uc5c6\uc774 \ubaa8\ub4e0 \uc774\ub4e4\uacfc \ud611\ub825\ud558\uc5ec \uad6d\uc815\uc744 \uc6b4\uc601\ud558\uace0\uc790 \ud558\ub294 \ubaa8\uc2b5\uc744 \ubcf4\uc5ec\uc8fc\uc5c8\ub2e4. \ub2a6\ubd04\uc5d0 \uc544\uc6b0\uad6c\uc2a4\ud22c\uc2a4\ub294 \uc790\uce6b\ud558\uba74 \uc8fd\uc744 \uc218 \uc788\uc5c8\ub358 \uc815\ub3c4\ub85c \uc2ec\ud558\uac8c \ubcd1\uc744 \uc553\uc558\uace0, \uc0ac\ub78c\ub4e4\uc740 \uc544\uc6b0\uad6c\uc2a4\ud22c\uc2a4\uac00 \uce5c\uad6c\uc778 \uc544\uadf8\ub9ac\ud30c\uc5d0\uac8c \uc790\uc2e0\uc758 \uc778\uc7a5\uc744 \ub118\uae30\uace0 \uc0ac\uc704\uc778 \ub9c8\ub974\ucf08\ub8e8\uc2a4\uc5d0\uac8c \ubcd1\uad8c\uc744 \uc704\uc784\ud560 \uac83\uc774\ub77c \uc0dd\uac01\ud588\ub2e4. \ud558\uc9c0\ub9cc \uc544\uc6b0\uad6c\uc2a4\ud22c\uc2a4\ub294 \uc790\uc2e0\uc758 \uc778\uc7a5\uacfc, \uad00\ub9ac\ud558\uace0 \uc788\ub358 \uacf5\uae08 \ubc0f \uad70\ub2e8 \ud1b5\uc81c\uad8c\uc744 \ub3d9\ub8cc \uc9d1\uc815\uad00\uc778 \ud53c\uc18c\uc5d0\uac8c \uc704\uc784\ud588\ub2e4. \uc774\ub7ec\ud55c \ud589\ub3d9\uc740 \uc544\uc6b0\uad6c\uc2a4\ud22c\uc2a4\uac00 \uc0ac\uc2e4\uc0c1\uc758 \ud669\uc81c\ub77c\uace0 \ubbff\uace0 \uc788\ub358 \uc0ac\ub78c\ub4e4\uc5d0\uac8c\ub294 \ub180\ub77c\uc6b4 \uc77c\uc774\uc5c8\uace0, \uc6d0\ub85c\uc6d0\uc740 \uc544\uc6b0\uad6c\uc2a4\ud22c\uc2a4\uac00 \uc81c\uc815\uc73c\ub85c \ub098\uc544\uac00\uace0 \uc788\ub2e4\ub294 \uc758\uc2ec\uc744 \uc7a0\uc2dc \uac70\ub454\ub2e4. \uc544\uc6b0\uad6c\uc2a4\ud22c\uc2a4\ub294 \uc0ac\uc720 \uc7ac\uc0b0\ub9cc \uc790\uc2e0\uc774 \uc9c0\ubaa9\ud55c \uc0c1\uc18d\uc790\ub4e4\uc5d0\uac8c \uc218\uc5ec\ud558\ub824 \ud588\ub2e4. \ub2f9\uc2dc \ub85c\ub9c8\uc758 \uc2dc\ubbfc\ub4e4\uc740 \uc5ec\uc804\ud788 \uad70\uc8fc\uc815\uc744 \uc88b\uac8c \ubcf4\uc9c0 \uc54a\uc558\uae30 \ub54c\ubb38\uc5d0 \uc544\uc6b0\uad6c\uc2a4\ud22c\uc2a4\uac00 \ub9cc\uc57d \uad8c\ub825\uc744 \uc790\uc2e0\uc774 \uc9c0\uba85\ud55c \ud6c4\uacc4\uc790\uc5d0\uac8c \ubb3c\ub824\uc8fc\ub824 \ud588\ub2e4\uba74 \ubc18\ub780\uc774 \uc77c\uc5b4\ub0ac\uc744 \uac83\uc774\ub2e4.", "paragraph_answer": "\uae30\uc6d0\uc804 23\ub144 \uc544\uc6b0\uad6c\uc2a4\ud22c\uc2a4\uc758 \ub3d9\ub8cc \uc9d1\uc815\uad00\uc774\uc5c8\ub358 \ud14c\ub80c\ud2f0\uc6b0\uc2a4 \ubc14\ub85c \ubb34\ub808\ub098 (Terentius Varro Murena)\uac00 \uc544\uc6b0\uad6c\uc2a4\ud22c\uc2a4\uc5d0 \ub300\ud56d\ud558\ub824 \ud55c\ub2e4. \uadf8 \ubc29\uc2dd\uc740 \uc815\ud655\ud558\uac8c \uc804\ud574\uc9c0\uace0 \uc788\uc9c0 \uc54a\uc73c\ub098 \ubb34\ub808\ub098\ub294 \uc784\uae30\ub97c \ub9c8\uce58\uc9c0 \ubabb\ud558\uace0 \uce7c\ud478\ub974\ub2c8\uc6b0\uc2a4 \ud53c\uc18c(Calpurnius Piso)\uac00 \uadf8 \uc790\ub9ac\ub97c \ub300\uc2e0\ud558\uac8c \ub41c\ub2e4. \ud53c\uc18c\ub294 \ub110\ub9ac \uc54c\ub824\uc9c4 \uacf5\ud654\uc815 \uc9c0\uc9c0\uc790\uc600\ub294\ub370, \uc544\uc6b0\uad6c\uc2a4\ud22c\uc2a4\ub294 \ub3d9\ub8cc \uc9d1\uc815\uad00\uacfc \ud611\ub825\ud558\uba74\uc11c \ud30c\ubc8c\uc5d0 \uad00\uacc4\uc5c6\uc774 \ubaa8\ub4e0 \uc774\ub4e4\uacfc \ud611\ub825\ud558\uc5ec \uad6d\uc815\uc744 \uc6b4\uc601\ud558\uace0\uc790 \ud558\ub294 \ubaa8\uc2b5\uc744 \ubcf4\uc5ec\uc8fc\uc5c8\ub2e4. \ub2a6\ubd04\uc5d0 \uc544\uc6b0\uad6c\uc2a4\ud22c\uc2a4\ub294 \uc790\uce6b\ud558\uba74 \uc8fd\uc744 \uc218 \uc788\uc5c8\ub358 \uc815\ub3c4\ub85c \uc2ec\ud558\uac8c \ubcd1\uc744 \uc553\uc558\uace0, \uc0ac\ub78c\ub4e4\uc740 \uc544\uc6b0\uad6c\uc2a4\ud22c\uc2a4\uac00 \uce5c\uad6c\uc778 \uc544\uadf8\ub9ac\ud30c\uc5d0\uac8c \uc790\uc2e0\uc758 \uc778\uc7a5\uc744 \ub118\uae30\uace0 \uc0ac\uc704\uc778 \ub9c8\ub974\ucf08\ub8e8\uc2a4\uc5d0\uac8c \ubcd1\uad8c\uc744 \uc704\uc784\ud560 \uac83\uc774\ub77c \uc0dd\uac01\ud588\ub2e4. \ud558\uc9c0\ub9cc \uc544\uc6b0\uad6c\uc2a4\ud22c\uc2a4\ub294 \uc790\uc2e0\uc758 \uc778\uc7a5\uacfc, \uad00\ub9ac\ud558\uace0 \uc788\ub358 \uacf5\uae08 \ubc0f \uad70\ub2e8 \ud1b5\uc81c\uad8c\uc744 \ub3d9\ub8cc \uc9d1\uc815\uad00\uc778 \ud53c\uc18c\uc5d0\uac8c \uc704\uc784\ud588\ub2e4. \uc774\ub7ec\ud55c \ud589\ub3d9\uc740 \uc544\uc6b0\uad6c\uc2a4\ud22c\uc2a4\uac00 \uc0ac\uc2e4\uc0c1\uc758 \ud669\uc81c\ub77c\uace0 \ubbff\uace0 \uc788\ub358 \uc0ac\ub78c\ub4e4\uc5d0\uac8c\ub294 \ub180\ub77c\uc6b4 \uc77c\uc774\uc5c8\uace0, \uc6d0\ub85c\uc6d0\uc740 \uc544\uc6b0\uad6c\uc2a4\ud22c\uc2a4\uac00 \uc81c\uc815\uc73c\ub85c \ub098\uc544\uac00\uace0 \uc788\ub2e4\ub294 \uc758\uc2ec\uc744 \uc7a0\uc2dc \uac70\ub454\ub2e4. \uc544\uc6b0\uad6c\uc2a4\ud22c\uc2a4\ub294 \uc0ac\uc720 \uc7ac\uc0b0\ub9cc \uc790\uc2e0\uc774 \uc9c0\ubaa9\ud55c \uc0c1\uc18d\uc790\ub4e4\uc5d0\uac8c \uc218\uc5ec\ud558\ub824 \ud588\ub2e4. \ub2f9\uc2dc \ub85c\ub9c8\uc758 \uc2dc\ubbfc\ub4e4\uc740 \uc5ec\uc804\ud788 \uad70\uc8fc\uc815\uc744 \uc88b\uac8c \ubcf4\uc9c0 \uc54a\uc558\uae30 \ub54c\ubb38\uc5d0 \uc544\uc6b0\uad6c\uc2a4\ud22c\uc2a4\uac00 \ub9cc\uc57d \uad8c\ub825\uc744 \uc790\uc2e0\uc774 \uc9c0\uba85\ud55c \ud6c4\uacc4\uc790\uc5d0\uac8c \ubb3c\ub824\uc8fc\ub824 \ud588\ub2e4\uba74 \ubc18\ub780\uc774 \uc77c\uc5b4\ub0ac\uc744 \uac83\uc774\ub2e4.", "sentence_answer": "\uae30\uc6d0\uc804 23\ub144 \uc544\uc6b0\uad6c\uc2a4\ud22c\uc2a4\uc758 \ub3d9\ub8cc \uc9d1\uc815\uad00\uc774\uc5c8\ub358 \ud14c\ub80c\ud2f0\uc6b0\uc2a4 \ubc14\ub85c \ubb34\ub808\ub098 (Terentius Varro Murena)", "paragraph_id": "6657258-22-0", "paragraph_question": "question: \uae30\uc6d0\uc804 23\ub144 \uc544\uc6b0\uad6c\uc2a4\uc5d0 \ub300\ud56d\ud558\uc600\uc73c\ub098 \uc784\uae30\ub97c \ub9c8\uce58\uc9c0 \ubabb\ud558\uace0 \uc9d1\uc815\uad00\uc5d0\uc11c \ub0b4\ub824\uc628 \uc778\ubb3c\uc740 \ub204\uad6c\uc778\uac00?, context: \uae30\uc6d0\uc804 23\ub144 \uc544\uc6b0\uad6c\uc2a4\ud22c\uc2a4\uc758 \ub3d9\ub8cc \uc9d1\uc815\uad00\uc774\uc5c8\ub358 \ud14c\ub80c\ud2f0\uc6b0\uc2a4 \ubc14\ub85c \ubb34\ub808\ub098(Terentius Varro Murena)\uac00 \uc544\uc6b0\uad6c\uc2a4\ud22c\uc2a4\uc5d0 \ub300\ud56d\ud558\ub824 \ud55c\ub2e4. \uadf8 \ubc29\uc2dd\uc740 \uc815\ud655\ud558\uac8c \uc804\ud574\uc9c0\uace0 \uc788\uc9c0 \uc54a\uc73c\ub098 \ubb34\ub808\ub098\ub294 \uc784\uae30\ub97c \ub9c8\uce58\uc9c0 \ubabb\ud558\uace0 \uce7c\ud478\ub974\ub2c8\uc6b0\uc2a4 \ud53c\uc18c(Calpurnius Piso)\uac00 \uadf8 \uc790\ub9ac\ub97c \ub300\uc2e0\ud558\uac8c \ub41c\ub2e4. \ud53c\uc18c\ub294 \ub110\ub9ac \uc54c\ub824\uc9c4 \uacf5\ud654\uc815 \uc9c0\uc9c0\uc790\uc600\ub294\ub370, \uc544\uc6b0\uad6c\uc2a4\ud22c\uc2a4\ub294 \ub3d9\ub8cc \uc9d1\uc815\uad00\uacfc \ud611\ub825\ud558\uba74\uc11c \ud30c\ubc8c\uc5d0 \uad00\uacc4\uc5c6\uc774 \ubaa8\ub4e0 \uc774\ub4e4\uacfc \ud611\ub825\ud558\uc5ec \uad6d\uc815\uc744 \uc6b4\uc601\ud558\uace0\uc790 \ud558\ub294 \ubaa8\uc2b5\uc744 \ubcf4\uc5ec\uc8fc\uc5c8\ub2e4. \ub2a6\ubd04\uc5d0 \uc544\uc6b0\uad6c\uc2a4\ud22c\uc2a4\ub294 \uc790\uce6b\ud558\uba74 \uc8fd\uc744 \uc218 \uc788\uc5c8\ub358 \uc815\ub3c4\ub85c \uc2ec\ud558\uac8c \ubcd1\uc744 \uc553\uc558\uace0, \uc0ac\ub78c\ub4e4\uc740 \uc544\uc6b0\uad6c\uc2a4\ud22c\uc2a4\uac00 \uce5c\uad6c\uc778 \uc544\uadf8\ub9ac\ud30c\uc5d0\uac8c \uc790\uc2e0\uc758 \uc778\uc7a5\uc744 \ub118\uae30\uace0 \uc0ac\uc704\uc778 \ub9c8\ub974\ucf08\ub8e8\uc2a4\uc5d0\uac8c \ubcd1\uad8c\uc744 \uc704\uc784\ud560 \uac83\uc774\ub77c \uc0dd\uac01\ud588\ub2e4. \ud558\uc9c0\ub9cc \uc544\uc6b0\uad6c\uc2a4\ud22c\uc2a4\ub294 \uc790\uc2e0\uc758 \uc778\uc7a5\uacfc, \uad00\ub9ac\ud558\uace0 \uc788\ub358 \uacf5\uae08 \ubc0f \uad70\ub2e8 \ud1b5\uc81c\uad8c\uc744 \ub3d9\ub8cc \uc9d1\uc815\uad00\uc778 \ud53c\uc18c\uc5d0\uac8c \uc704\uc784\ud588\ub2e4. \uc774\ub7ec\ud55c \ud589\ub3d9\uc740 \uc544\uc6b0\uad6c\uc2a4\ud22c\uc2a4\uac00 \uc0ac\uc2e4\uc0c1\uc758 \ud669\uc81c\ub77c\uace0 \ubbff\uace0 \uc788\ub358 \uc0ac\ub78c\ub4e4\uc5d0\uac8c\ub294 \ub180\ub77c\uc6b4 \uc77c\uc774\uc5c8\uace0, \uc6d0\ub85c\uc6d0\uc740 \uc544\uc6b0\uad6c\uc2a4\ud22c\uc2a4\uac00 \uc81c\uc815\uc73c\ub85c \ub098\uc544\uac00\uace0 \uc788\ub2e4\ub294 \uc758\uc2ec\uc744 \uc7a0\uc2dc \uac70\ub454\ub2e4. \uc544\uc6b0\uad6c\uc2a4\ud22c\uc2a4\ub294 \uc0ac\uc720 \uc7ac\uc0b0\ub9cc \uc790\uc2e0\uc774 \uc9c0\ubaa9\ud55c \uc0c1\uc18d\uc790\ub4e4\uc5d0\uac8c \uc218\uc5ec\ud558\ub824 \ud588\ub2e4. \ub2f9\uc2dc \ub85c\ub9c8\uc758 \uc2dc\ubbfc\ub4e4\uc740 \uc5ec\uc804\ud788 \uad70\uc8fc\uc815\uc744 \uc88b\uac8c \ubcf4\uc9c0 \uc54a\uc558\uae30 \ub54c\ubb38\uc5d0 \uc544\uc6b0\uad6c\uc2a4\ud22c\uc2a4\uac00 \ub9cc\uc57d \uad8c\ub825\uc744 \uc790\uc2e0\uc774 \uc9c0\uba85\ud55c \ud6c4\uacc4\uc790\uc5d0\uac8c \ubb3c\ub824\uc8fc\ub824 \ud588\ub2e4\uba74 \ubc18\ub780\uc774 \uc77c\uc5b4\ub0ac\uc744 \uac83\uc774\ub2e4."} {"question": "\ube0c\ub8e8\ud06c\ub108\ub294 \uc8fc\ub85c \uc5b4\ub5a4\uace1\uc744 \uc791\uace1\ud558\uc600\ub294\uac00?", "paragraph": "\uc5ed\uc2dc \ube48 \ucd9c\uc2e0\uc778 \ube0c\ub8e8\ud06c\ub108(1824-1896)\ub294 \ub0ad\ub9cc\ud30c\uc5d0\uc11c \uc885\uad50\uc801\uc778 \uc138\uacc4\ub97c \uc9c4\uc9c0\ud558\uac8c \uc74c\uc545 \uc18d\uc5d0 \ubcf4\uc778 \uc720\uc77c\ud55c \uc0ac\ub78c\uc774\ub2e4. \ubbfc\uc871\uc801, \uac1c\uc778\uc801\uc73c\ub85c \ubd84\uc5f4\ud55c 19\uc138\uae30\ub97c \ud1b5\ud558\uc5ec \uc791\uace1\uac00\ub294 \ud56d\uc0c1 \uc885\uad50\uc801\uc778 \uac83\uc744 \ub3d9\uacbd\ud558\uc600\ub2e4. \ubc14\uadf8\ub108\ub098 \ub9ac\uc2a4\ud2b8, \uadf8\ub9ac\uace0 \ube0c\ub78c\uc2a4\ub97c \ube0c\ub8e8\ud06c\ub108\uc640 \ube44\uad50\ud558\uc5ec \ubcf4\uba74, \uadf8\ub4e4\uc740 \ub108\ubb34\ub098 \uc778\uac04\uc911\uc2ec\uc774\uba70 \uc8fc\uad00\uc8fc\uc758\uc801\uc778 \uac10\uc815\uc5d0\uc11c \ube0c\ub8e8\ud06c\ub108\uc640 \uac19\uc774 \uc18c\ubc15\ud558\uac8c \ub5a8\uc5b4\uc9c0\uc9c0 \ubabb\ud558\uace0 \uc788\ub2e4. \ube0c\ub8e8\ud06c\ub108\ub294 \uc8fc\ub85c \uad50\ud5a5\uace1\uacfc \ubbf8\uc0ac\uace1\uc744 \uc791\uace1\ud558\uc600\ub2e4. \uadf8\uc758 \uad50\ud5a5\uace1\uc740 \uadf8 \ubc29\ub300\ud55c \ud615\uc2dd\uc774\ub098 \ud654\uc131\uc758 \uba74\uc5d0\uc11c \ub9ac\uc2a4\ud2b8, \ubc14\uadf8\ub108\uc758 \uc601\ud5a5\uc744 \uac15\ud558\uac8c \ubcf4\uc774\uace0 \uc788\uc73c\ub098 \uc644\uc804\ud788 \uad50\ud5a5\uc801\uc774\ub77c\ub294 \uc810\uc5d0\uc11c \ud754\ud788 \ube0c\ub78c\uc2a4\uc758 \uad50\ud5a5\uace1\uacfc \ube44\uad50\ub418\uace0 \uc788\ub2e4. \uc774 \ube44\uad50\ub85c\ub294 \uac00\ud1a8\ub9ad\uacfc \ud504\ub85c\ud14c\uc2a4\ud0c4\ud2b8, \ube44\uc11c\uc815\uc801\uacfc \uc11c\uc815\uc801, \uad11\ub300\ud55c \uc6b8\ub9bc\uacfc \uc2e4\ub0b4\uc545\uc801\uc778 \uc6b8\ub9bc \ub4f1\uc758 \ud070 \ucc28\uc774\ub97c \ubcfc \uc218 \uc788\uc73c\uba70, \ub0ad\ub9cc\ud30c\uc758 \uc5ed\uc0ac\ub97c \uc0dd\uac01\ud560 \ub54c \ub9e4\uc6b0 \ud765\ubbf8\uac00 \uc788\ub2e4. \ube0c\ub8e8\ud06c\ub108\uc758 \uad50\ud5a5\uace1\uc740 \uc0ac\ud6c4 \ub9d0\ub7ec\ub098 \ubcfc\ud504, \ub2c8\ud0a4\uc2dc \ub4f1\uc774 \uce58\ub978 \ub178\ub825\uc5d0 \uc758\ud574 \ud574\ub97c \uac70\ub4ed\ud560\uc218\ub85d \ub192\uc774 \ud3c9\uac00\ubc1b\uace0 \uc788\ub2e4.", "answer": "\uad50\ud5a5\uace1\uacfc \ubbf8\uc0ac\uace1", "sentence": "\ube0c\ub8e8\ud06c\ub108\ub294 \uc8fc\ub85c \uad50\ud5a5\uace1\uacfc \ubbf8\uc0ac\uace1 \uc744 \uc791\uace1\ud558\uc600\ub2e4.", "paragraph_sentence": "\uc5ed\uc2dc \ube48 \ucd9c\uc2e0\uc778 \ube0c\ub8e8\ud06c\ub108(1824-1896)\ub294 \ub0ad\ub9cc\ud30c\uc5d0\uc11c \uc885\uad50\uc801\uc778 \uc138\uacc4\ub97c \uc9c4\uc9c0\ud558\uac8c \uc74c\uc545 \uc18d\uc5d0 \ubcf4\uc778 \uc720\uc77c\ud55c \uc0ac\ub78c\uc774\ub2e4. \ubbfc\uc871\uc801, \uac1c\uc778\uc801\uc73c\ub85c \ubd84\uc5f4\ud55c 19\uc138\uae30\ub97c \ud1b5\ud558\uc5ec \uc791\uace1\uac00\ub294 \ud56d\uc0c1 \uc885\uad50\uc801\uc778 \uac83\uc744 \ub3d9\uacbd\ud558\uc600\ub2e4. \ubc14\uadf8\ub108\ub098 \ub9ac\uc2a4\ud2b8, \uadf8\ub9ac\uace0 \ube0c\ub78c\uc2a4\ub97c \ube0c\ub8e8\ud06c\ub108\uc640 \ube44\uad50\ud558\uc5ec \ubcf4\uba74, \uadf8\ub4e4\uc740 \ub108\ubb34\ub098 \uc778\uac04\uc911\uc2ec\uc774\uba70 \uc8fc\uad00\uc8fc\uc758\uc801\uc778 \uac10\uc815\uc5d0\uc11c \ube0c\ub8e8\ud06c\ub108\uc640 \uac19\uc774 \uc18c\ubc15\ud558\uac8c \ub5a8\uc5b4\uc9c0\uc9c0 \ubabb\ud558\uace0 \uc788\ub2e4. \ube0c\ub8e8\ud06c\ub108\ub294 \uc8fc\ub85c \uad50\ud5a5\uace1\uacfc \ubbf8\uc0ac\uace1 \uc744 \uc791\uace1\ud558\uc600\ub2e4. \uadf8\uc758 \uad50\ud5a5\uace1\uc740 \uadf8 \ubc29\ub300\ud55c \ud615\uc2dd\uc774\ub098 \ud654\uc131\uc758 \uba74\uc5d0\uc11c \ub9ac\uc2a4\ud2b8, \ubc14\uadf8\ub108\uc758 \uc601\ud5a5\uc744 \uac15\ud558\uac8c \ubcf4\uc774\uace0 \uc788\uc73c\ub098 \uc644\uc804\ud788 \uad50\ud5a5\uc801\uc774\ub77c\ub294 \uc810\uc5d0\uc11c \ud754\ud788 \ube0c\ub78c\uc2a4\uc758 \uad50\ud5a5\uace1\uacfc \ube44\uad50\ub418\uace0 \uc788\ub2e4. \uc774 \ube44\uad50\ub85c\ub294 \uac00\ud1a8\ub9ad\uacfc \ud504\ub85c\ud14c\uc2a4\ud0c4\ud2b8, \ube44\uc11c\uc815\uc801\uacfc \uc11c\uc815\uc801, \uad11\ub300\ud55c \uc6b8\ub9bc\uacfc \uc2e4\ub0b4\uc545\uc801\uc778 \uc6b8\ub9bc \ub4f1\uc758 \ud070 \ucc28\uc774\ub97c \ubcfc \uc218 \uc788\uc73c\uba70, \ub0ad\ub9cc\ud30c\uc758 \uc5ed\uc0ac\ub97c \uc0dd\uac01\ud560 \ub54c \ub9e4\uc6b0 \ud765\ubbf8\uac00 \uc788\ub2e4. \ube0c\ub8e8\ud06c\ub108\uc758 \uad50\ud5a5\uace1\uc740 \uc0ac\ud6c4 \ub9d0\ub7ec\ub098 \ubcfc\ud504, \ub2c8\ud0a4\uc2dc \ub4f1\uc774 \uce58\ub978 \ub178\ub825\uc5d0 \uc758\ud574 \ud574\ub97c \uac70\ub4ed\ud560\uc218\ub85d \ub192\uc774 \ud3c9\uac00\ubc1b\uace0 \uc788\ub2e4.", "paragraph_answer": "\uc5ed\uc2dc \ube48 \ucd9c\uc2e0\uc778 \ube0c\ub8e8\ud06c\ub108(1824-1896)\ub294 \ub0ad\ub9cc\ud30c\uc5d0\uc11c \uc885\uad50\uc801\uc778 \uc138\uacc4\ub97c \uc9c4\uc9c0\ud558\uac8c \uc74c\uc545 \uc18d\uc5d0 \ubcf4\uc778 \uc720\uc77c\ud55c \uc0ac\ub78c\uc774\ub2e4. \ubbfc\uc871\uc801, \uac1c\uc778\uc801\uc73c\ub85c \ubd84\uc5f4\ud55c 19\uc138\uae30\ub97c \ud1b5\ud558\uc5ec \uc791\uace1\uac00\ub294 \ud56d\uc0c1 \uc885\uad50\uc801\uc778 \uac83\uc744 \ub3d9\uacbd\ud558\uc600\ub2e4. \ubc14\uadf8\ub108\ub098 \ub9ac\uc2a4\ud2b8, \uadf8\ub9ac\uace0 \ube0c\ub78c\uc2a4\ub97c \ube0c\ub8e8\ud06c\ub108\uc640 \ube44\uad50\ud558\uc5ec \ubcf4\uba74, \uadf8\ub4e4\uc740 \ub108\ubb34\ub098 \uc778\uac04\uc911\uc2ec\uc774\uba70 \uc8fc\uad00\uc8fc\uc758\uc801\uc778 \uac10\uc815\uc5d0\uc11c \ube0c\ub8e8\ud06c\ub108\uc640 \uac19\uc774 \uc18c\ubc15\ud558\uac8c \ub5a8\uc5b4\uc9c0\uc9c0 \ubabb\ud558\uace0 \uc788\ub2e4. \ube0c\ub8e8\ud06c\ub108\ub294 \uc8fc\ub85c \uad50\ud5a5\uace1\uacfc \ubbf8\uc0ac\uace1 \uc744 \uc791\uace1\ud558\uc600\ub2e4. \uadf8\uc758 \uad50\ud5a5\uace1\uc740 \uadf8 \ubc29\ub300\ud55c \ud615\uc2dd\uc774\ub098 \ud654\uc131\uc758 \uba74\uc5d0\uc11c \ub9ac\uc2a4\ud2b8, \ubc14\uadf8\ub108\uc758 \uc601\ud5a5\uc744 \uac15\ud558\uac8c \ubcf4\uc774\uace0 \uc788\uc73c\ub098 \uc644\uc804\ud788 \uad50\ud5a5\uc801\uc774\ub77c\ub294 \uc810\uc5d0\uc11c \ud754\ud788 \ube0c\ub78c\uc2a4\uc758 \uad50\ud5a5\uace1\uacfc \ube44\uad50\ub418\uace0 \uc788\ub2e4. \uc774 \ube44\uad50\ub85c\ub294 \uac00\ud1a8\ub9ad\uacfc \ud504\ub85c\ud14c\uc2a4\ud0c4\ud2b8, \ube44\uc11c\uc815\uc801\uacfc \uc11c\uc815\uc801, \uad11\ub300\ud55c \uc6b8\ub9bc\uacfc \uc2e4\ub0b4\uc545\uc801\uc778 \uc6b8\ub9bc \ub4f1\uc758 \ud070 \ucc28\uc774\ub97c \ubcfc \uc218 \uc788\uc73c\uba70, \ub0ad\ub9cc\ud30c\uc758 \uc5ed\uc0ac\ub97c \uc0dd\uac01\ud560 \ub54c \ub9e4\uc6b0 \ud765\ubbf8\uac00 \uc788\ub2e4. \ube0c\ub8e8\ud06c\ub108\uc758 \uad50\ud5a5\uace1\uc740 \uc0ac\ud6c4 \ub9d0\ub7ec\ub098 \ubcfc\ud504, \ub2c8\ud0a4\uc2dc \ub4f1\uc774 \uce58\ub978 \ub178\ub825\uc5d0 \uc758\ud574 \ud574\ub97c \uac70\ub4ed\ud560\uc218\ub85d \ub192\uc774 \ud3c9\uac00\ubc1b\uace0 \uc788\ub2e4.", "sentence_answer": "\ube0c\ub8e8\ud06c\ub108\ub294 \uc8fc\ub85c \uad50\ud5a5\uace1\uacfc \ubbf8\uc0ac\uace1 \uc744 \uc791\uace1\ud558\uc600\ub2e4.", "paragraph_id": "6464953-12-1", "paragraph_question": "question: \ube0c\ub8e8\ud06c\ub108\ub294 \uc8fc\ub85c \uc5b4\ub5a4\uace1\uc744 \uc791\uace1\ud558\uc600\ub294\uac00?, context: \uc5ed\uc2dc \ube48 \ucd9c\uc2e0\uc778 \ube0c\ub8e8\ud06c\ub108(1824-1896)\ub294 \ub0ad\ub9cc\ud30c\uc5d0\uc11c \uc885\uad50\uc801\uc778 \uc138\uacc4\ub97c \uc9c4\uc9c0\ud558\uac8c \uc74c\uc545 \uc18d\uc5d0 \ubcf4\uc778 \uc720\uc77c\ud55c \uc0ac\ub78c\uc774\ub2e4. \ubbfc\uc871\uc801, \uac1c\uc778\uc801\uc73c\ub85c \ubd84\uc5f4\ud55c 19\uc138\uae30\ub97c \ud1b5\ud558\uc5ec \uc791\uace1\uac00\ub294 \ud56d\uc0c1 \uc885\uad50\uc801\uc778 \uac83\uc744 \ub3d9\uacbd\ud558\uc600\ub2e4. \ubc14\uadf8\ub108\ub098 \ub9ac\uc2a4\ud2b8, \uadf8\ub9ac\uace0 \ube0c\ub78c\uc2a4\ub97c \ube0c\ub8e8\ud06c\ub108\uc640 \ube44\uad50\ud558\uc5ec \ubcf4\uba74, \uadf8\ub4e4\uc740 \ub108\ubb34\ub098 \uc778\uac04\uc911\uc2ec\uc774\uba70 \uc8fc\uad00\uc8fc\uc758\uc801\uc778 \uac10\uc815\uc5d0\uc11c \ube0c\ub8e8\ud06c\ub108\uc640 \uac19\uc774 \uc18c\ubc15\ud558\uac8c \ub5a8\uc5b4\uc9c0\uc9c0 \ubabb\ud558\uace0 \uc788\ub2e4. \ube0c\ub8e8\ud06c\ub108\ub294 \uc8fc\ub85c \uad50\ud5a5\uace1\uacfc \ubbf8\uc0ac\uace1\uc744 \uc791\uace1\ud558\uc600\ub2e4. \uadf8\uc758 \uad50\ud5a5\uace1\uc740 \uadf8 \ubc29\ub300\ud55c \ud615\uc2dd\uc774\ub098 \ud654\uc131\uc758 \uba74\uc5d0\uc11c \ub9ac\uc2a4\ud2b8, \ubc14\uadf8\ub108\uc758 \uc601\ud5a5\uc744 \uac15\ud558\uac8c \ubcf4\uc774\uace0 \uc788\uc73c\ub098 \uc644\uc804\ud788 \uad50\ud5a5\uc801\uc774\ub77c\ub294 \uc810\uc5d0\uc11c \ud754\ud788 \ube0c\ub78c\uc2a4\uc758 \uad50\ud5a5\uace1\uacfc \ube44\uad50\ub418\uace0 \uc788\ub2e4. \uc774 \ube44\uad50\ub85c\ub294 \uac00\ud1a8\ub9ad\uacfc \ud504\ub85c\ud14c\uc2a4\ud0c4\ud2b8, \ube44\uc11c\uc815\uc801\uacfc \uc11c\uc815\uc801, \uad11\ub300\ud55c \uc6b8\ub9bc\uacfc \uc2e4\ub0b4\uc545\uc801\uc778 \uc6b8\ub9bc \ub4f1\uc758 \ud070 \ucc28\uc774\ub97c \ubcfc \uc218 \uc788\uc73c\uba70, \ub0ad\ub9cc\ud30c\uc758 \uc5ed\uc0ac\ub97c \uc0dd\uac01\ud560 \ub54c \ub9e4\uc6b0 \ud765\ubbf8\uac00 \uc788\ub2e4. \ube0c\ub8e8\ud06c\ub108\uc758 \uad50\ud5a5\uace1\uc740 \uc0ac\ud6c4 \ub9d0\ub7ec\ub098 \ubcfc\ud504, \ub2c8\ud0a4\uc2dc \ub4f1\uc774 \uce58\ub978 \ub178\ub825\uc5d0 \uc758\ud574 \ud574\ub97c \uac70\ub4ed\ud560\uc218\ub85d \ub192\uc774 \ud3c9\uac00\ubc1b\uace0 \uc788\ub2e4."} {"question": "\uc2e0\ucc3d\uc6d0\uc758 \uc218\ubc30 \uc2dc\uc808 \uadf8\uc5d0\uac8c \ucd1d\uc744 \uc3dc \ub2f4\ub2f9\ud615\uc0ac\uc758 \uc774\ub984\uc740?", "paragraph": "1997\ub144 1\uc6d4 20\uc77c \ubd80\uc0b0\uad50\ub3c4\uc18c\uc758 \ud654\uc7a5\uc2e4 \uc1e0\ucc3d\uc0b4\uc744 \uc1e0\ud1b1\ub0a0\ub85c \uc808\ub2e8\ud558\uace0 \ud0c8\ucd9c\ud558\uc600\ub2e4. \uc774\ud6c4 \uc57d 2\ub144 \ub3d9\uc548, \uc804\uad6d\uc744 \uc624\uac00\uba70 \uc57d 9\uc5b5 8000\uc5ec\ub9cc\uc6d0\uc744 \ud6d4\ucce4\uace0, \ud6d4\uce5c \ub3c8\uc73c\ub85c \uc720\ud765\uc5c5\uc18c \uc5ec\uc885\uc5c5\uc6d0\ub4e4\uc744 \uc720\ud639\ud574 \ub3d9\uac70\ud558\uba70 \uc544\uc9c0\ud2b8\ub85c \uc0bc\uc558\ub2e4. \uadf8\ub7ec\ub098, \ub3d9\uac70\ud560 \uacf3\uc774 \uc5c6\uc744 \ub54c\ub294 \ud1a0\uad74 \ub4f1\uc5d0\uc11c \uc0dd\ud65c\ud558\uba70 \uc950\uace0\uae30\ub85c \uc5f0\uba85\ud55c \uc801\ub3c4 \uc788\uc5c8\ub2e4\uace0 \ud55c\ub2e4. \ub610\ud55c \uacbd\ucc30\uc758 \ucd94\uaca9\uc5d0 \ucad3\uae30\ub294 \uc911 \uc2e0\ucc3d\uc6d0\uc758 \ub2f4\ub2f9\ud615\uc0ac\uc778 \uc6d0\uc885\uc5f4 \uacbd\uc7a5\uc774 \uc3dc \ucd1d\uc5d0 \ub9de\uc544 \ubd80\uc0c1\ub2f9\ud55c \uc0c1\ud0dc\uc784\uc5d0\ub3c4 \ubd88\uad6c\ud558\uace0 \uacbd\ucc30\uc758 \ucd94\uaca9\uc744 \ub05d\ub0b4 \ub530\ub3cc\ub824 \uc2e0\ucc3d\uc6d0\uc740 \uc790\uc2e0\uc758 \ucd08\uc778\uc801\uc778 \uccb4\ub825\uc744 \uacfc\uc2dc\ud558\uae30\ub3c4 \ud588\ub2e4. \uacbd\ucc30\uc740 \uc2e0\ucc3d\uc6d0\uc758 \uac80\uac70\ub97c \uc704\ud574 \ud5ec\ub9ac\ucf65\ud130\ub97c \ub744\uc6b0\uace0 \uc804\uacbd\uc744 \ub3d9\uc6d0\ud588\uc73c\ub098 \ubc88\ubc88\uc774 \uc18d\uc218\ubb34\ucc45\uc774\uc5c8\uc73c\uba70 \uc5f4\uc138 \ubc88\uc744 \ub208\uc55e\uc5d0\uc11c \ub193\uccd0 \ub9ce\uc740 \uacbd\ucc30\uad00\ub4e4\uc774 \uc774\uc5d0 \ucc45\uc784\uc744 \uc9c0\uace0 \uc0ac\ud1f4\ud588\ub2e4. 1999\ub144 7\uc6d4 16\uc77c, \uadf8\uac00 \uc228\uc5b4\uc788\ub358 \uc804\ub0a8 \uc21c\ucc9c\uc758 \uc544\ud30c\ud2b8\uc5d0\uc11c TV\uc218\ub9ac\ub97c \uc758\ub8b0\ubc1b\uc740 \uc218\ub9ac\uacf5\uc758 \uc81c\ubcf4\ub97c \ubc1b\uc740 \uacbd\ucc30\uc5d0\uac8c \uac80\uac70 \ub418\uc5c8\uace0 2\ub144 6\uac1c\uc6d4\uac04\uc758 \ud0c8\uc625 \uc0dd\ud65c\uc740 \ub9c8\uce68\ud45c\ub97c \ucc0d\uac8c \ub418\uc5c8\ub2e4. \uc774\ud6c4 \ub2f9\uc2dc \uc218\uc0ac\uacfc\uc7a5 \uae40\uc9c4\ud76c\ub294 \uc81c63\ub300 \uad11\uc8fc\ub3d9\ubd80\uacbd\ucc30\uc11c\uc7a5\uc73c\ub85c \ubd80\uc784, \uc2e0\uace0\ud55c \uc218\ub9ac\uacf5 \uae40\uc601\uad70\uc740 \uacbd\ucc30 \ud2b9\ucc44\ub418\uc5c8\ub2e4. \uad6d\ub0b4 \uc5ec\uc131 \uacbd\ucc30\uacf5\ubb34\uc6d0 \ucd5c\ucd08 \uac15\ub825\uacc4\uc7a5\uc73c\ub85c \uc784\uba85\ub41c \ubc15\ubbf8\uc625\ub3c4 \ub2f9\uc2dc \ud2b9\ubcc4\uac80\uac70\ud300 \uc18c\uc18d\uc73c\ub85c \ud65c\uc57d\ud558\uc600\ub2e4.", "answer": "\uc6d0\uc885\uc5f4", "sentence": "\ub610\ud55c \uacbd\ucc30\uc758 \ucd94\uaca9\uc5d0 \ucad3\uae30\ub294 \uc911 \uc2e0\ucc3d\uc6d0\uc758 \ub2f4\ub2f9\ud615\uc0ac\uc778 \uc6d0\uc885\uc5f4 \uacbd\uc7a5\uc774 \uc3dc \ucd1d\uc5d0 \ub9de\uc544 \ubd80\uc0c1\ub2f9\ud55c \uc0c1\ud0dc\uc784\uc5d0\ub3c4 \ubd88\uad6c\ud558\uace0 \uacbd\ucc30\uc758 \ucd94\uaca9\uc744 \ub05d\ub0b4 \ub530\ub3cc\ub824 \uc2e0\ucc3d\uc6d0\uc740 \uc790\uc2e0\uc758 \ucd08\uc778\uc801\uc778 \uccb4\ub825\uc744 \uacfc\uc2dc\ud558\uae30\ub3c4 \ud588\ub2e4.", "paragraph_sentence": "1997\ub144 1\uc6d4 20\uc77c \ubd80\uc0b0\uad50\ub3c4\uc18c\uc758 \ud654\uc7a5\uc2e4 \uc1e0\ucc3d\uc0b4\uc744 \uc1e0\ud1b1\ub0a0\ub85c \uc808\ub2e8\ud558\uace0 \ud0c8\ucd9c\ud558\uc600\ub2e4. \uc774\ud6c4 \uc57d 2\ub144 \ub3d9\uc548, \uc804\uad6d\uc744 \uc624\uac00\uba70 \uc57d 9\uc5b5 8000\uc5ec\ub9cc\uc6d0\uc744 \ud6d4\ucce4\uace0, \ud6d4\uce5c \ub3c8\uc73c\ub85c \uc720\ud765\uc5c5\uc18c \uc5ec\uc885\uc5c5\uc6d0\ub4e4\uc744 \uc720\ud639\ud574 \ub3d9\uac70\ud558\uba70 \uc544\uc9c0\ud2b8\ub85c \uc0bc\uc558\ub2e4. \uadf8\ub7ec\ub098, \ub3d9\uac70\ud560 \uacf3\uc774 \uc5c6\uc744 \ub54c\ub294 \ud1a0\uad74 \ub4f1\uc5d0\uc11c \uc0dd\ud65c\ud558\uba70 \uc950\uace0\uae30\ub85c \uc5f0\uba85\ud55c \uc801\ub3c4 \uc788\uc5c8\ub2e4\uace0 \ud55c\ub2e4. \ub610\ud55c \uacbd\ucc30\uc758 \ucd94\uaca9\uc5d0 \ucad3\uae30\ub294 \uc911 \uc2e0\ucc3d\uc6d0\uc758 \ub2f4\ub2f9\ud615\uc0ac\uc778 \uc6d0\uc885\uc5f4 \uacbd\uc7a5\uc774 \uc3dc \ucd1d\uc5d0 \ub9de\uc544 \ubd80\uc0c1\ub2f9\ud55c \uc0c1\ud0dc\uc784\uc5d0\ub3c4 \ubd88\uad6c\ud558\uace0 \uacbd\ucc30\uc758 \ucd94\uaca9\uc744 \ub05d\ub0b4 \ub530\ub3cc\ub824 \uc2e0\ucc3d\uc6d0\uc740 \uc790\uc2e0\uc758 \ucd08\uc778\uc801\uc778 \uccb4\ub825\uc744 \uacfc\uc2dc\ud558\uae30\ub3c4 \ud588\ub2e4. \uacbd\ucc30\uc740 \uc2e0\ucc3d\uc6d0\uc758 \uac80\uac70\ub97c \uc704\ud574 \ud5ec\ub9ac\ucf65\ud130\ub97c \ub744\uc6b0\uace0 \uc804\uacbd\uc744 \ub3d9\uc6d0\ud588\uc73c\ub098 \ubc88\ubc88\uc774 \uc18d\uc218\ubb34\ucc45\uc774\uc5c8\uc73c\uba70 \uc5f4\uc138 \ubc88\uc744 \ub208\uc55e\uc5d0\uc11c \ub193\uccd0 \ub9ce\uc740 \uacbd\ucc30\uad00\ub4e4\uc774 \uc774\uc5d0 \ucc45\uc784\uc744 \uc9c0\uace0 \uc0ac\ud1f4\ud588\ub2e4. 1999\ub144 7\uc6d4 16\uc77c, \uadf8\uac00 \uc228\uc5b4\uc788\ub358 \uc804\ub0a8 \uc21c\ucc9c\uc758 \uc544\ud30c\ud2b8\uc5d0\uc11c TV\uc218\ub9ac\ub97c \uc758\ub8b0\ubc1b\uc740 \uc218\ub9ac\uacf5\uc758 \uc81c\ubcf4\ub97c \ubc1b\uc740 \uacbd\ucc30\uc5d0\uac8c \uac80\uac70 \ub418\uc5c8\uace0 2\ub144 6\uac1c\uc6d4\uac04\uc758 \ud0c8\uc625 \uc0dd\ud65c\uc740 \ub9c8\uce68\ud45c\ub97c \ucc0d\uac8c \ub418\uc5c8\ub2e4. \uc774\ud6c4 \ub2f9\uc2dc \uc218\uc0ac\uacfc\uc7a5 \uae40\uc9c4\ud76c\ub294 \uc81c63\ub300 \uad11\uc8fc\ub3d9\ubd80\uacbd\ucc30\uc11c\uc7a5\uc73c\ub85c \ubd80\uc784, \uc2e0\uace0\ud55c \uc218\ub9ac\uacf5 \uae40\uc601\uad70\uc740 \uacbd\ucc30 \ud2b9\ucc44\ub418\uc5c8\ub2e4. \uad6d\ub0b4 \uc5ec\uc131 \uacbd\ucc30\uacf5\ubb34\uc6d0 \ucd5c\ucd08 \uac15\ub825\uacc4\uc7a5\uc73c\ub85c \uc784\uba85\ub41c \ubc15\ubbf8\uc625\ub3c4 \ub2f9\uc2dc \ud2b9\ubcc4\uac80\uac70\ud300 \uc18c\uc18d\uc73c\ub85c \ud65c\uc57d\ud558\uc600\ub2e4.", "paragraph_answer": "1997\ub144 1\uc6d4 20\uc77c \ubd80\uc0b0\uad50\ub3c4\uc18c\uc758 \ud654\uc7a5\uc2e4 \uc1e0\ucc3d\uc0b4\uc744 \uc1e0\ud1b1\ub0a0\ub85c \uc808\ub2e8\ud558\uace0 \ud0c8\ucd9c\ud558\uc600\ub2e4. \uc774\ud6c4 \uc57d 2\ub144 \ub3d9\uc548, \uc804\uad6d\uc744 \uc624\uac00\uba70 \uc57d 9\uc5b5 8000\uc5ec\ub9cc\uc6d0\uc744 \ud6d4\ucce4\uace0, \ud6d4\uce5c \ub3c8\uc73c\ub85c \uc720\ud765\uc5c5\uc18c \uc5ec\uc885\uc5c5\uc6d0\ub4e4\uc744 \uc720\ud639\ud574 \ub3d9\uac70\ud558\uba70 \uc544\uc9c0\ud2b8\ub85c \uc0bc\uc558\ub2e4. \uadf8\ub7ec\ub098, \ub3d9\uac70\ud560 \uacf3\uc774 \uc5c6\uc744 \ub54c\ub294 \ud1a0\uad74 \ub4f1\uc5d0\uc11c \uc0dd\ud65c\ud558\uba70 \uc950\uace0\uae30\ub85c \uc5f0\uba85\ud55c \uc801\ub3c4 \uc788\uc5c8\ub2e4\uace0 \ud55c\ub2e4. \ub610\ud55c \uacbd\ucc30\uc758 \ucd94\uaca9\uc5d0 \ucad3\uae30\ub294 \uc911 \uc2e0\ucc3d\uc6d0\uc758 \ub2f4\ub2f9\ud615\uc0ac\uc778 \uc6d0\uc885\uc5f4 \uacbd\uc7a5\uc774 \uc3dc \ucd1d\uc5d0 \ub9de\uc544 \ubd80\uc0c1\ub2f9\ud55c \uc0c1\ud0dc\uc784\uc5d0\ub3c4 \ubd88\uad6c\ud558\uace0 \uacbd\ucc30\uc758 \ucd94\uaca9\uc744 \ub05d\ub0b4 \ub530\ub3cc\ub824 \uc2e0\ucc3d\uc6d0\uc740 \uc790\uc2e0\uc758 \ucd08\uc778\uc801\uc778 \uccb4\ub825\uc744 \uacfc\uc2dc\ud558\uae30\ub3c4 \ud588\ub2e4. \uacbd\ucc30\uc740 \uc2e0\ucc3d\uc6d0\uc758 \uac80\uac70\ub97c \uc704\ud574 \ud5ec\ub9ac\ucf65\ud130\ub97c \ub744\uc6b0\uace0 \uc804\uacbd\uc744 \ub3d9\uc6d0\ud588\uc73c\ub098 \ubc88\ubc88\uc774 \uc18d\uc218\ubb34\ucc45\uc774\uc5c8\uc73c\uba70 \uc5f4\uc138 \ubc88\uc744 \ub208\uc55e\uc5d0\uc11c \ub193\uccd0 \ub9ce\uc740 \uacbd\ucc30\uad00\ub4e4\uc774 \uc774\uc5d0 \ucc45\uc784\uc744 \uc9c0\uace0 \uc0ac\ud1f4\ud588\ub2e4. 1999\ub144 7\uc6d4 16\uc77c, \uadf8\uac00 \uc228\uc5b4\uc788\ub358 \uc804\ub0a8 \uc21c\ucc9c\uc758 \uc544\ud30c\ud2b8\uc5d0\uc11c TV\uc218\ub9ac\ub97c \uc758\ub8b0\ubc1b\uc740 \uc218\ub9ac\uacf5\uc758 \uc81c\ubcf4\ub97c \ubc1b\uc740 \uacbd\ucc30\uc5d0\uac8c \uac80\uac70 \ub418\uc5c8\uace0 2\ub144 6\uac1c\uc6d4\uac04\uc758 \ud0c8\uc625 \uc0dd\ud65c\uc740 \ub9c8\uce68\ud45c\ub97c \ucc0d\uac8c \ub418\uc5c8\ub2e4. \uc774\ud6c4 \ub2f9\uc2dc \uc218\uc0ac\uacfc\uc7a5 \uae40\uc9c4\ud76c\ub294 \uc81c63\ub300 \uad11\uc8fc\ub3d9\ubd80\uacbd\ucc30\uc11c\uc7a5\uc73c\ub85c \ubd80\uc784, \uc2e0\uace0\ud55c \uc218\ub9ac\uacf5 \uae40\uc601\uad70\uc740 \uacbd\ucc30 \ud2b9\ucc44\ub418\uc5c8\ub2e4. \uad6d\ub0b4 \uc5ec\uc131 \uacbd\ucc30\uacf5\ubb34\uc6d0 \ucd5c\ucd08 \uac15\ub825\uacc4\uc7a5\uc73c\ub85c \uc784\uba85\ub41c \ubc15\ubbf8\uc625\ub3c4 \ub2f9\uc2dc \ud2b9\ubcc4\uac80\uac70\ud300 \uc18c\uc18d\uc73c\ub85c \ud65c\uc57d\ud558\uc600\ub2e4.", "sentence_answer": "\ub610\ud55c \uacbd\ucc30\uc758 \ucd94\uaca9\uc5d0 \ucad3\uae30\ub294 \uc911 \uc2e0\ucc3d\uc6d0\uc758 \ub2f4\ub2f9\ud615\uc0ac\uc778 \uc6d0\uc885\uc5f4 \uacbd\uc7a5\uc774 \uc3dc \ucd1d\uc5d0 \ub9de\uc544 \ubd80\uc0c1\ub2f9\ud55c \uc0c1\ud0dc\uc784\uc5d0\ub3c4 \ubd88\uad6c\ud558\uace0 \uacbd\ucc30\uc758 \ucd94\uaca9\uc744 \ub05d\ub0b4 \ub530\ub3cc\ub824 \uc2e0\ucc3d\uc6d0\uc740 \uc790\uc2e0\uc758 \ucd08\uc778\uc801\uc778 \uccb4\ub825\uc744 \uacfc\uc2dc\ud558\uae30\ub3c4 \ud588\ub2e4.", "paragraph_id": "6463940-1-1", "paragraph_question": "question: \uc2e0\ucc3d\uc6d0\uc758 \uc218\ubc30 \uc2dc\uc808 \uadf8\uc5d0\uac8c \ucd1d\uc744 \uc3dc \ub2f4\ub2f9\ud615\uc0ac\uc758 \uc774\ub984\uc740?, context: 1997\ub144 1\uc6d4 20\uc77c \ubd80\uc0b0\uad50\ub3c4\uc18c\uc758 \ud654\uc7a5\uc2e4 \uc1e0\ucc3d\uc0b4\uc744 \uc1e0\ud1b1\ub0a0\ub85c \uc808\ub2e8\ud558\uace0 \ud0c8\ucd9c\ud558\uc600\ub2e4. \uc774\ud6c4 \uc57d 2\ub144 \ub3d9\uc548, \uc804\uad6d\uc744 \uc624\uac00\uba70 \uc57d 9\uc5b5 8000\uc5ec\ub9cc\uc6d0\uc744 \ud6d4\ucce4\uace0, \ud6d4\uce5c \ub3c8\uc73c\ub85c \uc720\ud765\uc5c5\uc18c \uc5ec\uc885\uc5c5\uc6d0\ub4e4\uc744 \uc720\ud639\ud574 \ub3d9\uac70\ud558\uba70 \uc544\uc9c0\ud2b8\ub85c \uc0bc\uc558\ub2e4. \uadf8\ub7ec\ub098, \ub3d9\uac70\ud560 \uacf3\uc774 \uc5c6\uc744 \ub54c\ub294 \ud1a0\uad74 \ub4f1\uc5d0\uc11c \uc0dd\ud65c\ud558\uba70 \uc950\uace0\uae30\ub85c \uc5f0\uba85\ud55c \uc801\ub3c4 \uc788\uc5c8\ub2e4\uace0 \ud55c\ub2e4. \ub610\ud55c \uacbd\ucc30\uc758 \ucd94\uaca9\uc5d0 \ucad3\uae30\ub294 \uc911 \uc2e0\ucc3d\uc6d0\uc758 \ub2f4\ub2f9\ud615\uc0ac\uc778 \uc6d0\uc885\uc5f4 \uacbd\uc7a5\uc774 \uc3dc \ucd1d\uc5d0 \ub9de\uc544 \ubd80\uc0c1\ub2f9\ud55c \uc0c1\ud0dc\uc784\uc5d0\ub3c4 \ubd88\uad6c\ud558\uace0 \uacbd\ucc30\uc758 \ucd94\uaca9\uc744 \ub05d\ub0b4 \ub530\ub3cc\ub824 \uc2e0\ucc3d\uc6d0\uc740 \uc790\uc2e0\uc758 \ucd08\uc778\uc801\uc778 \uccb4\ub825\uc744 \uacfc\uc2dc\ud558\uae30\ub3c4 \ud588\ub2e4. \uacbd\ucc30\uc740 \uc2e0\ucc3d\uc6d0\uc758 \uac80\uac70\ub97c \uc704\ud574 \ud5ec\ub9ac\ucf65\ud130\ub97c \ub744\uc6b0\uace0 \uc804\uacbd\uc744 \ub3d9\uc6d0\ud588\uc73c\ub098 \ubc88\ubc88\uc774 \uc18d\uc218\ubb34\ucc45\uc774\uc5c8\uc73c\uba70 \uc5f4\uc138 \ubc88\uc744 \ub208\uc55e\uc5d0\uc11c \ub193\uccd0 \ub9ce\uc740 \uacbd\ucc30\uad00\ub4e4\uc774 \uc774\uc5d0 \ucc45\uc784\uc744 \uc9c0\uace0 \uc0ac\ud1f4\ud588\ub2e4. 1999\ub144 7\uc6d4 16\uc77c, \uadf8\uac00 \uc228\uc5b4\uc788\ub358 \uc804\ub0a8 \uc21c\ucc9c\uc758 \uc544\ud30c\ud2b8\uc5d0\uc11c TV\uc218\ub9ac\ub97c \uc758\ub8b0\ubc1b\uc740 \uc218\ub9ac\uacf5\uc758 \uc81c\ubcf4\ub97c \ubc1b\uc740 \uacbd\ucc30\uc5d0\uac8c \uac80\uac70 \ub418\uc5c8\uace0 2\ub144 6\uac1c\uc6d4\uac04\uc758 \ud0c8\uc625 \uc0dd\ud65c\uc740 \ub9c8\uce68\ud45c\ub97c \ucc0d\uac8c \ub418\uc5c8\ub2e4. \uc774\ud6c4 \ub2f9\uc2dc \uc218\uc0ac\uacfc\uc7a5 \uae40\uc9c4\ud76c\ub294 \uc81c63\ub300 \uad11\uc8fc\ub3d9\ubd80\uacbd\ucc30\uc11c\uc7a5\uc73c\ub85c \ubd80\uc784, \uc2e0\uace0\ud55c \uc218\ub9ac\uacf5 \uae40\uc601\uad70\uc740 \uacbd\ucc30 \ud2b9\ucc44\ub418\uc5c8\ub2e4. \uad6d\ub0b4 \uc5ec\uc131 \uacbd\ucc30\uacf5\ubb34\uc6d0 \ucd5c\ucd08 \uac15\ub825\uacc4\uc7a5\uc73c\ub85c \uc784\uba85\ub41c \ubc15\ubbf8\uc625\ub3c4 \ub2f9\uc2dc \ud2b9\ubcc4\uac80\uac70\ud300 \uc18c\uc18d\uc73c\ub85c \ud65c\uc57d\ud558\uc600\ub2e4."} {"question": "\ubcf5\uc9c0\ubd80\uac00 20\uc870\uc6d0\uc5d0 \ub2ec\ud558\ub294 \uac74\ubcf4 \ub204\uc801\uc801\ub9bd\uae08\uc744 \uc801\uc808\ud788 \ud65c\uc6a9\ud574 \uad6d\ubbfc\uc5d0\uac8c \uac00\uae09\uc801 \ub298\ub9ac\uc9c0 \uc54a\uaca0\ub2e4\uace0 \ud55c \uac83\uc740?", "paragraph": "2017\ub144 8\uc6d4 9\uc77c \uac00\ud1a8\ub9ad\ub300\ud559\uad50 \uc11c\uc6b8\uc131\ubaa8\ubcd1\uc6d0\uc744 \ubc29\ubb38\ud55c \uc790\ub9ac\uc5d0\uc11c \ud658\uc790\ub4e4\uc758 \uc758\ub8cc\ube44 \ubd80\ub2f4\uc744 \ud68d\uae30\uc801\uc73c\ub85c \ub0ae\ucdb0 \uac00\uacc4\ud30c\ud0c4\uc744 \ub9c9\ub294 \ub0b4\uc6a9\uc744 \ub2f4\uc740 \uad6d\ubbfc\uac74\uac15\ubcf4\ud5d8 \ubcf4\uc7a5\uc131 \uac15\ud654\ub300\ucc45\uc744 \ubc1c\ud45c\ud588\ub294\ub370 \ubbf8\uc6a9\u00b7\uc131\ud615\uc744 \uc81c\uc678\ud55c \uc804 \uc758\ub8cc \ubd84\uc57c\uc5d0 \ub300\ud574 \uac74\uac15\ubcf4\ud5d8\uc744 \uc801\uc6a9\ud558\ub294 \uac83\uc744 \uace8\uc790\ub85c \ud55c\ub2e4. \uad6c\uccb4\uc801\uc73c\ub85c \uace0\uac00 \ud56d\uc554\uc81c\u00b7\ucd08\uc74c\ud30c\u00b7\uc790\uae30\uacf5\uba85\uc601\uc0c1\uc7a5\uce58(MRI) \ub4f1 3800\uc5ec \uac1c\uc758 \ube44\uae09\uc5ec \uc9c4\ub8cc \ud56d\ubaa9\uc758 \uc758\ub8cc\ube44\ub97c \uae30\uc874\uc758 100%\uc5d0\uc11c \ud658\uc790 \ubd80\ub2f4\ub960\uc744 30% ~ 90%\ub9cc \ubd80\ub2f4\ud558\ub294 \uc870\uac74\uc73c\ub85c \uac74\uac15\ubcf4\ud5d8 \ud61c\ud0dd \ub300\uc0c1\uc758 \uc608\ube44\uae09\uc5ec\ub85c \ub123\uc5b4 \ube44\uae09\uc5ec \uc9c4\ub8cc\ud56d\ubaa9\uc744 1/3 \uc218\uc900\uc73c\ub85c \ub0ae\ucd94\uace0 \uac04\uacbd\ube44\u00b7\uc120\ud0dd\uc9c4\ub8cc\ube44\u00b7\uc0c1\uae09\ubcd1\uc2e4\ub8cc \ub4f1 3\ub300 \ube44\uae09\uc5ec \ubd80\ub2f4\ub3c4 \uc904\uc774\uba70, \uc120\ud0dd\uc9c4\ub8cc\ub97c \ubc1b\uc744 \ub54c \uc9c0\ubd88\ud558\ub294 \ube44\uc6a9\ub3c4 \ud3d0\uc9c0\ud558\uace0 4\uc778\uc2e4\uae4c\uc9c0 \uc801\uc6a9\ud558\ub358 \uc785\uc6d0\ub8cc\ub97c 2 ~ 3\uc778\uc2e4\uae4c\uc9c0 \ud655\ub300\ud558\uba70 \ubcf4\ud638\uc790\ub098 \uac04\ubcd1\uc778 \uc5c6\uc774 \uc804\ubb38 \uac04\ud638\uc0ac\uac00 \uac04\ud638\uc640 \uac04\ubcd1\uc744 \uc804\ub2f4\ud558\ub294 \uac04\ud638\uac04\ubcd1\ud1b5\ud569\uc11c\ube44\uc2a4 \uc81c\uacf5 \ubcd1\uc0c1\ub3c4 \uae30\uc874\uc758 2\ub9cc 5\ucc9c \uac1c \uc218\uc900\uc5d0\uc11c 10\ub9cc \uac1c\ub85c \ud655\ub300\ud560 \uac83\uc744 \uacc4\ud68d\ud588\ub2e4. \uc774\ub97c \uc704\ud574 2022\ub144\uae4c\uc9c0 30\uc870 \uc6d0\uac00\ub7c9\uc744 \ud22c\uc785\ud558\uc5ec \uad6d\ubbfc\uc758 \ube44\uae09\uc5ec \uc758\ub8cc\ube44 \ubd80\ub2f4\uc744 13\uc870 \uc6d0\uc5d0\uc11c 5\uc870 \uc6d0 \uc218\uc900\uc73c\ub85c \ub0ae\ucd9c \uacc4\ud68d\uc774\uba70 \ub300\ucc45\uc774 \uc644\ub8cc\ub418\uba74 \uac74\uac15\ubcf4\ud5d8 \ubcf4\uc7a5\ub960\uc740 70% \uc218\uc900\uc774 \ub420 \uac83\uc73c\ub85c \ubcf4\uc778\ub2e4\uace0 \ubc1d\ud614\ub2e4. \ubcf5\uc9c0\ubd80\ub294 \"20\uc870 \uc6d0\uc5d0 \ub2ec\ud558\ub294 \uac74\ubcf4 \ub204\uc801\uc801\ub9bd\uae08\uc744 \uc801\uc808\ud788 \ud65c\uc6a9\ud574 \uad6d\ubbfc\uc758 \ubcf4\ud5d8\ub8cc \ubd80\ub2f4\uc744 \uac00\uae09\uc801 \ub298\ub9ac\uc9c0 \uc54a\ub3c4\ub85d \ud558\uaca0\ub2e4\"\uace0 \uc57d\uc18d\ud588\ub2e4. \ub610 \uac74\uac15\ubcf4\ud5d8\uc5d0 \ub300\ud55c \uad6d\uace0 \uc9c0\uc6d0\uacfc \ubcf4\ud5d8\ub8cc \ubd80\uacfc\uae30\ubc18 \ud655\ub300 \ub4f1\uc73c\ub85c \ubcf4\ud5d8\ub8cc \uc218\uc785\uc744 \ud655\ucda9\ud558\uace0 \uac01\uc885 \ub3c4\ub355\uc801 \ud574\uc774\uc640 \uc7ac\uc815 \ub204\uc218 \uc694\uc778\uc744 \ucc28\ub2e8\ud558\uc5ec \uc7ac\uc815\uc808\uac10\ub300\ucc45\uc744 \ubcd1\ud589\ud558\uae30\ub85c \ud588\ub2e4. \ud558\uc9c0\ub9cc \"\ubcf4\ud5d8\ub8cc\ub294 \uc7ac\uc815\uc9c0\ucd9c\uc744 \uc904\uc774\uace0 \uad6d\uace0 \uc9c0\uc6d0\uc744 \ub354 \ubc1b\ub294 \ub4f1 \ub2e4\uc591\ud55c \uc815\ucc45\uc801 \ub178\ub825\uc744 \uae30\uc6b8\uc774\uace0 \ub098\uc11c \uac00\uacc4\uc5d0 \ud070 \ubd80\ub2f4\uc774 \uc5c6\ub294 \uc218\uc900\uc5d0\uc11c \uad00\ub9ac\ud574\ub098\uac08 \uac83\"\uc774\ub77c\uace0 \ub9d0\ud574 \uc911\uc7a5\uae30\uc801\uc73c\ub85c\ub294 \ubcf4\ud5d8\ub8cc \ubd80\ub2f4\uc774 \ub298 \uc218 \uc788\uc74c\ub3c4 \ud568\uaed8 \uc2dc\uc0ac\ud588\ub2e4.", "answer": "\ubcf4\ud5d8\ub8cc \ubd80\ub2f4", "sentence": "\uc801\uc808\ud788 \ud65c\uc6a9\ud574 \uad6d\ubbfc\uc758 \ubcf4\ud5d8\ub8cc \ubd80\ub2f4 \uc744 \uac00\uae09\uc801 \ub298\ub9ac\uc9c0 \uc54a\ub3c4\ub85d \ud558\uaca0\ub2e4\"\uace0 \uc57d\uc18d\ud588\ub2e4.", "paragraph_sentence": "2017\ub144 8\uc6d4 9\uc77c \uac00\ud1a8\ub9ad\ub300\ud559\uad50 \uc11c\uc6b8\uc131\ubaa8\ubcd1\uc6d0\uc744 \ubc29\ubb38\ud55c \uc790\ub9ac\uc5d0\uc11c \ud658\uc790\ub4e4\uc758 \uc758\ub8cc\ube44 \ubd80\ub2f4\uc744 \ud68d\uae30\uc801\uc73c\ub85c \ub0ae\ucdb0 \uac00\uacc4\ud30c\ud0c4\uc744 \ub9c9\ub294 \ub0b4\uc6a9\uc744 \ub2f4\uc740 \uad6d\ubbfc\uac74\uac15\ubcf4\ud5d8 \ubcf4\uc7a5\uc131 \uac15\ud654\ub300\ucc45\uc744 \ubc1c\ud45c\ud588\ub294\ub370 \ubbf8\uc6a9\u00b7\uc131\ud615\uc744 \uc81c\uc678\ud55c \uc804 \uc758\ub8cc \ubd84\uc57c\uc5d0 \ub300\ud574 \uac74\uac15\ubcf4\ud5d8\uc744 \uc801\uc6a9\ud558\ub294 \uac83\uc744 \uace8\uc790\ub85c \ud55c\ub2e4. \uad6c\uccb4\uc801\uc73c\ub85c \uace0\uac00 \ud56d\uc554\uc81c\u00b7\ucd08\uc74c\ud30c\u00b7\uc790\uae30\uacf5\uba85\uc601\uc0c1\uc7a5\uce58(MRI) \ub4f1 3800\uc5ec \uac1c\uc758 \ube44\uae09\uc5ec \uc9c4\ub8cc \ud56d\ubaa9\uc758 \uc758\ub8cc\ube44\ub97c \uae30\uc874\uc758 100%\uc5d0\uc11c \ud658\uc790 \ubd80\ub2f4\ub960\uc744 30% ~ 90%\ub9cc \ubd80\ub2f4\ud558\ub294 \uc870\uac74\uc73c\ub85c \uac74\uac15\ubcf4\ud5d8 \ud61c\ud0dd \ub300\uc0c1\uc758 \uc608\ube44\uae09\uc5ec\ub85c \ub123\uc5b4 \ube44\uae09\uc5ec \uc9c4\ub8cc\ud56d\ubaa9\uc744 1/3 \uc218\uc900\uc73c\ub85c \ub0ae\ucd94\uace0 \uac04\uacbd\ube44\u00b7\uc120\ud0dd\uc9c4\ub8cc\ube44\u00b7\uc0c1\uae09\ubcd1\uc2e4\ub8cc \ub4f1 3\ub300 \ube44\uae09\uc5ec \ubd80\ub2f4\ub3c4 \uc904\uc774\uba70, \uc120\ud0dd\uc9c4\ub8cc\ub97c \ubc1b\uc744 \ub54c \uc9c0\ubd88\ud558\ub294 \ube44\uc6a9\ub3c4 \ud3d0\uc9c0\ud558\uace0 4\uc778\uc2e4\uae4c\uc9c0 \uc801\uc6a9\ud558\ub358 \uc785\uc6d0\ub8cc\ub97c 2 ~ 3\uc778\uc2e4\uae4c\uc9c0 \ud655\ub300\ud558\uba70 \ubcf4\ud638\uc790\ub098 \uac04\ubcd1\uc778 \uc5c6\uc774 \uc804\ubb38 \uac04\ud638\uc0ac\uac00 \uac04\ud638\uc640 \uac04\ubcd1\uc744 \uc804\ub2f4\ud558\ub294 \uac04\ud638\uac04\ubcd1\ud1b5\ud569\uc11c\ube44\uc2a4 \uc81c\uacf5 \ubcd1\uc0c1\ub3c4 \uae30\uc874\uc758 2\ub9cc 5\ucc9c \uac1c \uc218\uc900\uc5d0\uc11c 10\ub9cc \uac1c\ub85c \ud655\ub300\ud560 \uac83\uc744 \uacc4\ud68d\ud588\ub2e4. \uc774\ub97c \uc704\ud574 2022\ub144\uae4c\uc9c0 30\uc870 \uc6d0\uac00\ub7c9\uc744 \ud22c\uc785\ud558\uc5ec \uad6d\ubbfc\uc758 \ube44\uae09\uc5ec \uc758\ub8cc\ube44 \ubd80\ub2f4\uc744 13\uc870 \uc6d0\uc5d0\uc11c 5\uc870 \uc6d0 \uc218\uc900\uc73c\ub85c \ub0ae\ucd9c \uacc4\ud68d\uc774\uba70 \ub300\ucc45\uc774 \uc644\ub8cc\ub418\uba74 \uac74\uac15\ubcf4\ud5d8 \ubcf4\uc7a5\ub960\uc740 70% \uc218\uc900\uc774 \ub420 \uac83\uc73c\ub85c \ubcf4\uc778\ub2e4\uace0 \ubc1d\ud614\ub2e4. \ubcf5\uc9c0\ubd80\ub294 \"20\uc870 \uc6d0\uc5d0 \ub2ec\ud558\ub294 \uac74\ubcf4 \ub204\uc801\uc801\ub9bd\uae08\uc744 \uc801\uc808\ud788 \ud65c\uc6a9\ud574 \uad6d\ubbfc\uc758 \ubcf4\ud5d8\ub8cc \ubd80\ub2f4 \uc744 \uac00\uae09\uc801 \ub298\ub9ac\uc9c0 \uc54a\ub3c4\ub85d \ud558\uaca0\ub2e4\"\uace0 \uc57d\uc18d\ud588\ub2e4. \ub610 \uac74\uac15\ubcf4\ud5d8\uc5d0 \ub300\ud55c \uad6d\uace0 \uc9c0\uc6d0\uacfc \ubcf4\ud5d8\ub8cc \ubd80\uacfc\uae30\ubc18 \ud655\ub300 \ub4f1\uc73c\ub85c \ubcf4\ud5d8\ub8cc \uc218\uc785\uc744 \ud655\ucda9\ud558\uace0 \uac01\uc885 \ub3c4\ub355\uc801 \ud574\uc774\uc640 \uc7ac\uc815 \ub204\uc218 \uc694\uc778\uc744 \ucc28\ub2e8\ud558\uc5ec \uc7ac\uc815\uc808\uac10\ub300\ucc45\uc744 \ubcd1\ud589\ud558\uae30\ub85c \ud588\ub2e4. \ud558\uc9c0\ub9cc \"\ubcf4\ud5d8\ub8cc\ub294 \uc7ac\uc815\uc9c0\ucd9c\uc744 \uc904\uc774\uace0 \uad6d\uace0 \uc9c0\uc6d0\uc744 \ub354 \ubc1b\ub294 \ub4f1 \ub2e4\uc591\ud55c \uc815\ucc45\uc801 \ub178\ub825\uc744 \uae30\uc6b8\uc774\uace0 \ub098\uc11c \uac00\uacc4\uc5d0 \ud070 \ubd80\ub2f4\uc774 \uc5c6\ub294 \uc218\uc900\uc5d0\uc11c \uad00\ub9ac\ud574\ub098\uac08 \uac83\"\uc774\ub77c\uace0 \ub9d0\ud574 \uc911\uc7a5\uae30\uc801\uc73c\ub85c\ub294 \ubcf4\ud5d8\ub8cc \ubd80\ub2f4\uc774 \ub298 \uc218 \uc788\uc74c\ub3c4 \ud568\uaed8 \uc2dc\uc0ac\ud588\ub2e4.", "paragraph_answer": "2017\ub144 8\uc6d4 9\uc77c \uac00\ud1a8\ub9ad\ub300\ud559\uad50 \uc11c\uc6b8\uc131\ubaa8\ubcd1\uc6d0\uc744 \ubc29\ubb38\ud55c \uc790\ub9ac\uc5d0\uc11c \ud658\uc790\ub4e4\uc758 \uc758\ub8cc\ube44 \ubd80\ub2f4\uc744 \ud68d\uae30\uc801\uc73c\ub85c \ub0ae\ucdb0 \uac00\uacc4\ud30c\ud0c4\uc744 \ub9c9\ub294 \ub0b4\uc6a9\uc744 \ub2f4\uc740 \uad6d\ubbfc\uac74\uac15\ubcf4\ud5d8 \ubcf4\uc7a5\uc131 \uac15\ud654\ub300\ucc45\uc744 \ubc1c\ud45c\ud588\ub294\ub370 \ubbf8\uc6a9\u00b7\uc131\ud615\uc744 \uc81c\uc678\ud55c \uc804 \uc758\ub8cc \ubd84\uc57c\uc5d0 \ub300\ud574 \uac74\uac15\ubcf4\ud5d8\uc744 \uc801\uc6a9\ud558\ub294 \uac83\uc744 \uace8\uc790\ub85c \ud55c\ub2e4. \uad6c\uccb4\uc801\uc73c\ub85c \uace0\uac00 \ud56d\uc554\uc81c\u00b7\ucd08\uc74c\ud30c\u00b7\uc790\uae30\uacf5\uba85\uc601\uc0c1\uc7a5\uce58(MRI) \ub4f1 3800\uc5ec \uac1c\uc758 \ube44\uae09\uc5ec \uc9c4\ub8cc \ud56d\ubaa9\uc758 \uc758\ub8cc\ube44\ub97c \uae30\uc874\uc758 100%\uc5d0\uc11c \ud658\uc790 \ubd80\ub2f4\ub960\uc744 30% ~ 90%\ub9cc \ubd80\ub2f4\ud558\ub294 \uc870\uac74\uc73c\ub85c \uac74\uac15\ubcf4\ud5d8 \ud61c\ud0dd \ub300\uc0c1\uc758 \uc608\ube44\uae09\uc5ec\ub85c \ub123\uc5b4 \ube44\uae09\uc5ec \uc9c4\ub8cc\ud56d\ubaa9\uc744 1/3 \uc218\uc900\uc73c\ub85c \ub0ae\ucd94\uace0 \uac04\uacbd\ube44\u00b7\uc120\ud0dd\uc9c4\ub8cc\ube44\u00b7\uc0c1\uae09\ubcd1\uc2e4\ub8cc \ub4f1 3\ub300 \ube44\uae09\uc5ec \ubd80\ub2f4\ub3c4 \uc904\uc774\uba70, \uc120\ud0dd\uc9c4\ub8cc\ub97c \ubc1b\uc744 \ub54c \uc9c0\ubd88\ud558\ub294 \ube44\uc6a9\ub3c4 \ud3d0\uc9c0\ud558\uace0 4\uc778\uc2e4\uae4c\uc9c0 \uc801\uc6a9\ud558\ub358 \uc785\uc6d0\ub8cc\ub97c 2 ~ 3\uc778\uc2e4\uae4c\uc9c0 \ud655\ub300\ud558\uba70 \ubcf4\ud638\uc790\ub098 \uac04\ubcd1\uc778 \uc5c6\uc774 \uc804\ubb38 \uac04\ud638\uc0ac\uac00 \uac04\ud638\uc640 \uac04\ubcd1\uc744 \uc804\ub2f4\ud558\ub294 \uac04\ud638\uac04\ubcd1\ud1b5\ud569\uc11c\ube44\uc2a4 \uc81c\uacf5 \ubcd1\uc0c1\ub3c4 \uae30\uc874\uc758 2\ub9cc 5\ucc9c \uac1c \uc218\uc900\uc5d0\uc11c 10\ub9cc \uac1c\ub85c \ud655\ub300\ud560 \uac83\uc744 \uacc4\ud68d\ud588\ub2e4. \uc774\ub97c \uc704\ud574 2022\ub144\uae4c\uc9c0 30\uc870 \uc6d0\uac00\ub7c9\uc744 \ud22c\uc785\ud558\uc5ec \uad6d\ubbfc\uc758 \ube44\uae09\uc5ec \uc758\ub8cc\ube44 \ubd80\ub2f4\uc744 13\uc870 \uc6d0\uc5d0\uc11c 5\uc870 \uc6d0 \uc218\uc900\uc73c\ub85c \ub0ae\ucd9c \uacc4\ud68d\uc774\uba70 \ub300\ucc45\uc774 \uc644\ub8cc\ub418\uba74 \uac74\uac15\ubcf4\ud5d8 \ubcf4\uc7a5\ub960\uc740 70% \uc218\uc900\uc774 \ub420 \uac83\uc73c\ub85c \ubcf4\uc778\ub2e4\uace0 \ubc1d\ud614\ub2e4. \ubcf5\uc9c0\ubd80\ub294 \"20\uc870 \uc6d0\uc5d0 \ub2ec\ud558\ub294 \uac74\ubcf4 \ub204\uc801\uc801\ub9bd\uae08\uc744 \uc801\uc808\ud788 \ud65c\uc6a9\ud574 \uad6d\ubbfc\uc758 \ubcf4\ud5d8\ub8cc \ubd80\ub2f4 \uc744 \uac00\uae09\uc801 \ub298\ub9ac\uc9c0 \uc54a\ub3c4\ub85d \ud558\uaca0\ub2e4\"\uace0 \uc57d\uc18d\ud588\ub2e4. \ub610 \uac74\uac15\ubcf4\ud5d8\uc5d0 \ub300\ud55c \uad6d\uace0 \uc9c0\uc6d0\uacfc \ubcf4\ud5d8\ub8cc \ubd80\uacfc\uae30\ubc18 \ud655\ub300 \ub4f1\uc73c\ub85c \ubcf4\ud5d8\ub8cc \uc218\uc785\uc744 \ud655\ucda9\ud558\uace0 \uac01\uc885 \ub3c4\ub355\uc801 \ud574\uc774\uc640 \uc7ac\uc815 \ub204\uc218 \uc694\uc778\uc744 \ucc28\ub2e8\ud558\uc5ec \uc7ac\uc815\uc808\uac10\ub300\ucc45\uc744 \ubcd1\ud589\ud558\uae30\ub85c \ud588\ub2e4. \ud558\uc9c0\ub9cc \"\ubcf4\ud5d8\ub8cc\ub294 \uc7ac\uc815\uc9c0\ucd9c\uc744 \uc904\uc774\uace0 \uad6d\uace0 \uc9c0\uc6d0\uc744 \ub354 \ubc1b\ub294 \ub4f1 \ub2e4\uc591\ud55c \uc815\ucc45\uc801 \ub178\ub825\uc744 \uae30\uc6b8\uc774\uace0 \ub098\uc11c \uac00\uacc4\uc5d0 \ud070 \ubd80\ub2f4\uc774 \uc5c6\ub294 \uc218\uc900\uc5d0\uc11c \uad00\ub9ac\ud574\ub098\uac08 \uac83\"\uc774\ub77c\uace0 \ub9d0\ud574 \uc911\uc7a5\uae30\uc801\uc73c\ub85c\ub294 \ubcf4\ud5d8\ub8cc \ubd80\ub2f4\uc774 \ub298 \uc218 \uc788\uc74c\ub3c4 \ud568\uaed8 \uc2dc\uc0ac\ud588\ub2e4.", "sentence_answer": "\uc801\uc808\ud788 \ud65c\uc6a9\ud574 \uad6d\ubbfc\uc758 \ubcf4\ud5d8\ub8cc \ubd80\ub2f4 \uc744 \uac00\uae09\uc801 \ub298\ub9ac\uc9c0 \uc54a\ub3c4\ub85d \ud558\uaca0\ub2e4\"\uace0 \uc57d\uc18d\ud588\ub2e4.", "paragraph_id": "6580792-123-1", "paragraph_question": "question: \ubcf5\uc9c0\ubd80\uac00 20\uc870\uc6d0\uc5d0 \ub2ec\ud558\ub294 \uac74\ubcf4 \ub204\uc801\uc801\ub9bd\uae08\uc744 \uc801\uc808\ud788 \ud65c\uc6a9\ud574 \uad6d\ubbfc\uc5d0\uac8c \uac00\uae09\uc801 \ub298\ub9ac\uc9c0 \uc54a\uaca0\ub2e4\uace0 \ud55c \uac83\uc740?, context: 2017\ub144 8\uc6d4 9\uc77c \uac00\ud1a8\ub9ad\ub300\ud559\uad50 \uc11c\uc6b8\uc131\ubaa8\ubcd1\uc6d0\uc744 \ubc29\ubb38\ud55c \uc790\ub9ac\uc5d0\uc11c \ud658\uc790\ub4e4\uc758 \uc758\ub8cc\ube44 \ubd80\ub2f4\uc744 \ud68d\uae30\uc801\uc73c\ub85c \ub0ae\ucdb0 \uac00\uacc4\ud30c\ud0c4\uc744 \ub9c9\ub294 \ub0b4\uc6a9\uc744 \ub2f4\uc740 \uad6d\ubbfc\uac74\uac15\ubcf4\ud5d8 \ubcf4\uc7a5\uc131 \uac15\ud654\ub300\ucc45\uc744 \ubc1c\ud45c\ud588\ub294\ub370 \ubbf8\uc6a9\u00b7\uc131\ud615\uc744 \uc81c\uc678\ud55c \uc804 \uc758\ub8cc \ubd84\uc57c\uc5d0 \ub300\ud574 \uac74\uac15\ubcf4\ud5d8\uc744 \uc801\uc6a9\ud558\ub294 \uac83\uc744 \uace8\uc790\ub85c \ud55c\ub2e4. \uad6c\uccb4\uc801\uc73c\ub85c \uace0\uac00 \ud56d\uc554\uc81c\u00b7\ucd08\uc74c\ud30c\u00b7\uc790\uae30\uacf5\uba85\uc601\uc0c1\uc7a5\uce58(MRI) \ub4f1 3800\uc5ec \uac1c\uc758 \ube44\uae09\uc5ec \uc9c4\ub8cc \ud56d\ubaa9\uc758 \uc758\ub8cc\ube44\ub97c \uae30\uc874\uc758 100%\uc5d0\uc11c \ud658\uc790 \ubd80\ub2f4\ub960\uc744 30% ~ 90%\ub9cc \ubd80\ub2f4\ud558\ub294 \uc870\uac74\uc73c\ub85c \uac74\uac15\ubcf4\ud5d8 \ud61c\ud0dd \ub300\uc0c1\uc758 \uc608\ube44\uae09\uc5ec\ub85c \ub123\uc5b4 \ube44\uae09\uc5ec \uc9c4\ub8cc\ud56d\ubaa9\uc744 1/3 \uc218\uc900\uc73c\ub85c \ub0ae\ucd94\uace0 \uac04\uacbd\ube44\u00b7\uc120\ud0dd\uc9c4\ub8cc\ube44\u00b7\uc0c1\uae09\ubcd1\uc2e4\ub8cc \ub4f1 3\ub300 \ube44\uae09\uc5ec \ubd80\ub2f4\ub3c4 \uc904\uc774\uba70, \uc120\ud0dd\uc9c4\ub8cc\ub97c \ubc1b\uc744 \ub54c \uc9c0\ubd88\ud558\ub294 \ube44\uc6a9\ub3c4 \ud3d0\uc9c0\ud558\uace0 4\uc778\uc2e4\uae4c\uc9c0 \uc801\uc6a9\ud558\ub358 \uc785\uc6d0\ub8cc\ub97c 2 ~ 3\uc778\uc2e4\uae4c\uc9c0 \ud655\ub300\ud558\uba70 \ubcf4\ud638\uc790\ub098 \uac04\ubcd1\uc778 \uc5c6\uc774 \uc804\ubb38 \uac04\ud638\uc0ac\uac00 \uac04\ud638\uc640 \uac04\ubcd1\uc744 \uc804\ub2f4\ud558\ub294 \uac04\ud638\uac04\ubcd1\ud1b5\ud569\uc11c\ube44\uc2a4 \uc81c\uacf5 \ubcd1\uc0c1\ub3c4 \uae30\uc874\uc758 2\ub9cc 5\ucc9c \uac1c \uc218\uc900\uc5d0\uc11c 10\ub9cc \uac1c\ub85c \ud655\ub300\ud560 \uac83\uc744 \uacc4\ud68d\ud588\ub2e4. \uc774\ub97c \uc704\ud574 2022\ub144\uae4c\uc9c0 30\uc870 \uc6d0\uac00\ub7c9\uc744 \ud22c\uc785\ud558\uc5ec \uad6d\ubbfc\uc758 \ube44\uae09\uc5ec \uc758\ub8cc\ube44 \ubd80\ub2f4\uc744 13\uc870 \uc6d0\uc5d0\uc11c 5\uc870 \uc6d0 \uc218\uc900\uc73c\ub85c \ub0ae\ucd9c \uacc4\ud68d\uc774\uba70 \ub300\ucc45\uc774 \uc644\ub8cc\ub418\uba74 \uac74\uac15\ubcf4\ud5d8 \ubcf4\uc7a5\ub960\uc740 70% \uc218\uc900\uc774 \ub420 \uac83\uc73c\ub85c \ubcf4\uc778\ub2e4\uace0 \ubc1d\ud614\ub2e4. \ubcf5\uc9c0\ubd80\ub294 \"20\uc870 \uc6d0\uc5d0 \ub2ec\ud558\ub294 \uac74\ubcf4 \ub204\uc801\uc801\ub9bd\uae08\uc744 \uc801\uc808\ud788 \ud65c\uc6a9\ud574 \uad6d\ubbfc\uc758 \ubcf4\ud5d8\ub8cc \ubd80\ub2f4\uc744 \uac00\uae09\uc801 \ub298\ub9ac\uc9c0 \uc54a\ub3c4\ub85d \ud558\uaca0\ub2e4\"\uace0 \uc57d\uc18d\ud588\ub2e4. \ub610 \uac74\uac15\ubcf4\ud5d8\uc5d0 \ub300\ud55c \uad6d\uace0 \uc9c0\uc6d0\uacfc \ubcf4\ud5d8\ub8cc \ubd80\uacfc\uae30\ubc18 \ud655\ub300 \ub4f1\uc73c\ub85c \ubcf4\ud5d8\ub8cc \uc218\uc785\uc744 \ud655\ucda9\ud558\uace0 \uac01\uc885 \ub3c4\ub355\uc801 \ud574\uc774\uc640 \uc7ac\uc815 \ub204\uc218 \uc694\uc778\uc744 \ucc28\ub2e8\ud558\uc5ec \uc7ac\uc815\uc808\uac10\ub300\ucc45\uc744 \ubcd1\ud589\ud558\uae30\ub85c \ud588\ub2e4. \ud558\uc9c0\ub9cc \"\ubcf4\ud5d8\ub8cc\ub294 \uc7ac\uc815\uc9c0\ucd9c\uc744 \uc904\uc774\uace0 \uad6d\uace0 \uc9c0\uc6d0\uc744 \ub354 \ubc1b\ub294 \ub4f1 \ub2e4\uc591\ud55c \uc815\ucc45\uc801 \ub178\ub825\uc744 \uae30\uc6b8\uc774\uace0 \ub098\uc11c \uac00\uacc4\uc5d0 \ud070 \ubd80\ub2f4\uc774 \uc5c6\ub294 \uc218\uc900\uc5d0\uc11c \uad00\ub9ac\ud574\ub098\uac08 \uac83\"\uc774\ub77c\uace0 \ub9d0\ud574 \uc911\uc7a5\uae30\uc801\uc73c\ub85c\ub294 \ubcf4\ud5d8\ub8cc \ubd80\ub2f4\uc774 \ub298 \uc218 \uc788\uc74c\ub3c4 \ud568\uaed8 \uc2dc\uc0ac\ud588\ub2e4."} {"question": "\ub9cc\uc288\ud0c0\uc778\uc740 \uc804\ubc94\ud610\uc758\ub85c \uba87\uac1c\uc758 \uc0ac\ud56d\uc5d0 \uae30\uc18c \ub418\uc5c8\ub294\uac00?", "paragraph": "\ud06c\ub9bc \uc804\uc5ed\uc5d0\uc11c \ub9cc\uc288\ud0c0\uc778\uc740 \uc18c\ub828\uc778\ub4e4\uc5d0 \ub300\ud55c \uc794\ud559\ud589\uc704, \ud2b9\ud788 \uc544\uc778\uc790\uce20\uadf8\ub8e8\ud39c D\ubd80\ub300\uc758 \ud559\uc0b4\ud589\uc704\uc5d0 \uac04\uc811\uc801\uc73c\ub85c \uc5f0\ub8e8\ub418\uc5c8\ub2e4. \uc544\uc778\uc790\uce20\uadf8\ub8e8\ud39c\uc740 \ub098\uce58 \uce5c\uc704\ub300(SS) \uc9d1\ub2e8 \uc911 \ud558\ub098\ub85c\uc11c, \uc720\ub7fd\uc758 \uc720\ub300\uc778 \ubc15\uba78\uc744 \uadf8 \uc784\ubb34\ub85c \uc0bc\uace0 \uc788\uc5c8\ub2e4. \uc544\uc778\uc790\uce20\uadf8\ub8e8\ud39c D\ubd80\ub300\ub294 \ub9cc\uc288\ud0c0\uc778\uc758 \uc81c11\uad70\uc774 \uac00\ub294 \uae38\uc744 \ub530\ub77c\ub2e4\ub154\uace0, \ub9cc\uc288\ud0c0\uc778\uc5d0\uac8c \ucc28\ub7c9, \uc5f0\ub8cc, \uc6b4\uc804\ubcd1 \ub4f1\uc744 \uc81c\uacf5\ubc1b\uc558\ub2e4. \uc544\uc778\uc790\uce20\uadf8\ub8e8\ud39c\uc774 \uc720\ub300\uc778\ub4e4\uc744 \ucd1d\uc0b4\ud558\ub824\uace0 \uacc4\ud68d\ud55c \uc9c0\uc5ed\uc5d0 \ud5cc\ubcd1\ub4e4\uc774 \uc904\uc744 \ucce4\ub294\ub370 \uc774\ub294 \ud0c8\ucd9c\ud558\ub294 \uc0ac\ub78c\uc774 \uc5c6\ub3c4\ub85d \ud558\uae30 \uc704\ud568\uc774\uc5c8\ub2e4. \uc544\uc778\uc790\uce20\uadf8\ub8e8\ud39c D\ubd80\ub300\uac00 \uc720\ub300\uc778 \uc5ec\uc131\uacfc \uc544\ub3d9\ub4e4\uc744 \ud559\uc0b4\ud558\ub294 \uac83\uc744 \ubcf4\uace0 \ucda9\uaca9\uc744 \ubc1b\uc740 \uc6b8\ub9ac\ud788 \uad70\uccb4\ub974\ud2b8(Ulrich Gunzert) \ub300\uc704\ub294 \ub9cc\uc288\ud0c0\uc778\uc5d0\uac8c \uac00\uc11c \ud559\uc0b4\uc744 \uba48\ucd94\uae30 \uc704\ud574 \uc870\uce58\ub97c \ucde8\ud574 \ub2ec\ub77c\uace0 \uc9c4\uc5b8\ud588\ub2e4. \uad70\uccb4\ub974\ud2b8\uc758 \uc99d\uc5b8\uc5d0 \ub530\ub974\uba74 \ub9cc\uc288\ud0c0\uc778\uc740 \uadf8\uc5d0\uac8c \ubc29\uae08 \ubcf8 \uac83\uc740 \uc78a\uace0 \ubd89\uc740 \uad70\ub300\uc640\uc758 \uc2f8\uc6c0\uc5d0\ub098 \uc9d1\uc911\ud558\ub77c\uace0 \ub9d0\ud588\ub2e4. \uad70\uccb4\ub974\ud2b8\ub294 \ub9cc\uc288\ud0c0\uc778\uc758 \ud589\uc704\ub97c \u201c\ucc45\uc784\uc73c\ub85c\ubd80\ud130\uc758 \ub3c4\ud53c, \ub3c4\ub355\uc801 \uc2e4\uaca9\u201d\uc774\ub77c\uace0 \ud3c9\ud588\ub2e4. \ub9cc\uc288\ud0c0\uc778\uc774 \uc804\ubc94\ud610\uc758\ub85c \uae30\uc18c\ub418\uc5c8\uc744 \ub2f9\uc2dc 17\uac1c \uae30\uc18c\uc0ac\ud56d \uc911 11\uac1c\uac00 \ud06c\ub9bc \uc804\uc5ed\uc5d0\uc11c\uc758 \uc720\ub300\uc778\uacfc \ud3ec\ub85c \ud559\ub300 \ubc0f \ud559\uc0b4\uc5d0 \uad00\ub828\ub41c \uac83\uc774\ub2e4.", "answer": "17\uac1c", "sentence": "\ub2f9\uc2dc 17\uac1c \uae30\uc18c\uc0ac\ud56d \uc911 11\uac1c\uac00 \ud06c\ub9bc \uc804\uc5ed\uc5d0\uc11c\uc758 \uc720\ub300\uc778\uacfc \ud3ec\ub85c \ud559\ub300 \ubc0f \ud559\uc0b4\uc5d0 \uad00\ub828\ub41c \uac83\uc774\ub2e4.", "paragraph_sentence": "\ud06c\ub9bc \uc804\uc5ed\uc5d0\uc11c \ub9cc\uc288\ud0c0\uc778\uc740 \uc18c\ub828\uc778\ub4e4\uc5d0 \ub300\ud55c \uc794\ud559\ud589\uc704, \ud2b9\ud788 \uc544\uc778\uc790\uce20\uadf8\ub8e8\ud39c D\ubd80\ub300\uc758 \ud559\uc0b4\ud589\uc704\uc5d0 \uac04\uc811\uc801\uc73c\ub85c \uc5f0\ub8e8\ub418\uc5c8\ub2e4. \uc544\uc778\uc790\uce20\uadf8\ub8e8\ud39c\uc740 \ub098\uce58 \uce5c\uc704\ub300(SS) \uc9d1\ub2e8 \uc911 \ud558\ub098\ub85c\uc11c, \uc720\ub7fd\uc758 \uc720\ub300\uc778 \ubc15\uba78\uc744 \uadf8 \uc784\ubb34\ub85c \uc0bc\uace0 \uc788\uc5c8\ub2e4. \uc544\uc778\uc790\uce20\uadf8\ub8e8\ud39c D\ubd80\ub300\ub294 \ub9cc\uc288\ud0c0\uc778\uc758 \uc81c11\uad70\uc774 \uac00\ub294 \uae38\uc744 \ub530\ub77c\ub2e4\ub154\uace0, \ub9cc\uc288\ud0c0\uc778\uc5d0\uac8c \ucc28\ub7c9, \uc5f0\ub8cc, \uc6b4\uc804\ubcd1 \ub4f1\uc744 \uc81c\uacf5\ubc1b\uc558\ub2e4. \uc544\uc778\uc790\uce20\uadf8\ub8e8\ud39c\uc774 \uc720\ub300\uc778\ub4e4\uc744 \ucd1d\uc0b4\ud558\ub824\uace0 \uacc4\ud68d\ud55c \uc9c0\uc5ed\uc5d0 \ud5cc\ubcd1\ub4e4\uc774 \uc904\uc744 \ucce4\ub294\ub370 \uc774\ub294 \ud0c8\ucd9c\ud558\ub294 \uc0ac\ub78c\uc774 \uc5c6\ub3c4\ub85d \ud558\uae30 \uc704\ud568\uc774\uc5c8\ub2e4. \uc544\uc778\uc790\uce20\uadf8\ub8e8\ud39c D\ubd80\ub300\uac00 \uc720\ub300\uc778 \uc5ec\uc131\uacfc \uc544\ub3d9\ub4e4\uc744 \ud559\uc0b4\ud558\ub294 \uac83\uc744 \ubcf4\uace0 \ucda9\uaca9\uc744 \ubc1b\uc740 \uc6b8\ub9ac\ud788 \uad70\uccb4\ub974\ud2b8(Ulrich Gunzert) \ub300\uc704\ub294 \ub9cc\uc288\ud0c0\uc778\uc5d0\uac8c \uac00\uc11c \ud559\uc0b4\uc744 \uba48\ucd94\uae30 \uc704\ud574 \uc870\uce58\ub97c \ucde8\ud574 \ub2ec\ub77c\uace0 \uc9c4\uc5b8\ud588\ub2e4. \uad70\uccb4\ub974\ud2b8\uc758 \uc99d\uc5b8\uc5d0 \ub530\ub974\uba74 \ub9cc\uc288\ud0c0\uc778\uc740 \uadf8\uc5d0\uac8c \ubc29\uae08 \ubcf8 \uac83\uc740 \uc78a\uace0 \ubd89\uc740 \uad70\ub300\uc640\uc758 \uc2f8\uc6c0\uc5d0\ub098 \uc9d1\uc911\ud558\ub77c\uace0 \ub9d0\ud588\ub2e4. \uad70\uccb4\ub974\ud2b8\ub294 \ub9cc\uc288\ud0c0\uc778\uc758 \ud589\uc704\ub97c \u201c\ucc45\uc784\uc73c\ub85c\ubd80\ud130\uc758 \ub3c4\ud53c, \ub3c4\ub355\uc801 \uc2e4\uaca9\u201d\uc774\ub77c\uace0 \ud3c9\ud588\ub2e4. \ub9cc\uc288\ud0c0\uc778\uc774 \uc804\ubc94\ud610\uc758\ub85c \uae30\uc18c\ub418\uc5c8\uc744 \ub2f9\uc2dc 17\uac1c \uae30\uc18c\uc0ac\ud56d \uc911 11\uac1c\uac00 \ud06c\ub9bc \uc804\uc5ed\uc5d0\uc11c\uc758 \uc720\ub300\uc778\uacfc \ud3ec\ub85c \ud559\ub300 \ubc0f \ud559\uc0b4\uc5d0 \uad00\ub828\ub41c \uac83\uc774\ub2e4. ", "paragraph_answer": "\ud06c\ub9bc \uc804\uc5ed\uc5d0\uc11c \ub9cc\uc288\ud0c0\uc778\uc740 \uc18c\ub828\uc778\ub4e4\uc5d0 \ub300\ud55c \uc794\ud559\ud589\uc704, \ud2b9\ud788 \uc544\uc778\uc790\uce20\uadf8\ub8e8\ud39c D\ubd80\ub300\uc758 \ud559\uc0b4\ud589\uc704\uc5d0 \uac04\uc811\uc801\uc73c\ub85c \uc5f0\ub8e8\ub418\uc5c8\ub2e4. \uc544\uc778\uc790\uce20\uadf8\ub8e8\ud39c\uc740 \ub098\uce58 \uce5c\uc704\ub300(SS) \uc9d1\ub2e8 \uc911 \ud558\ub098\ub85c\uc11c, \uc720\ub7fd\uc758 \uc720\ub300\uc778 \ubc15\uba78\uc744 \uadf8 \uc784\ubb34\ub85c \uc0bc\uace0 \uc788\uc5c8\ub2e4. \uc544\uc778\uc790\uce20\uadf8\ub8e8\ud39c D\ubd80\ub300\ub294 \ub9cc\uc288\ud0c0\uc778\uc758 \uc81c11\uad70\uc774 \uac00\ub294 \uae38\uc744 \ub530\ub77c\ub2e4\ub154\uace0, \ub9cc\uc288\ud0c0\uc778\uc5d0\uac8c \ucc28\ub7c9, \uc5f0\ub8cc, \uc6b4\uc804\ubcd1 \ub4f1\uc744 \uc81c\uacf5\ubc1b\uc558\ub2e4. \uc544\uc778\uc790\uce20\uadf8\ub8e8\ud39c\uc774 \uc720\ub300\uc778\ub4e4\uc744 \ucd1d\uc0b4\ud558\ub824\uace0 \uacc4\ud68d\ud55c \uc9c0\uc5ed\uc5d0 \ud5cc\ubcd1\ub4e4\uc774 \uc904\uc744 \ucce4\ub294\ub370 \uc774\ub294 \ud0c8\ucd9c\ud558\ub294 \uc0ac\ub78c\uc774 \uc5c6\ub3c4\ub85d \ud558\uae30 \uc704\ud568\uc774\uc5c8\ub2e4. \uc544\uc778\uc790\uce20\uadf8\ub8e8\ud39c D\ubd80\ub300\uac00 \uc720\ub300\uc778 \uc5ec\uc131\uacfc \uc544\ub3d9\ub4e4\uc744 \ud559\uc0b4\ud558\ub294 \uac83\uc744 \ubcf4\uace0 \ucda9\uaca9\uc744 \ubc1b\uc740 \uc6b8\ub9ac\ud788 \uad70\uccb4\ub974\ud2b8(Ulrich Gunzert) \ub300\uc704\ub294 \ub9cc\uc288\ud0c0\uc778\uc5d0\uac8c \uac00\uc11c \ud559\uc0b4\uc744 \uba48\ucd94\uae30 \uc704\ud574 \uc870\uce58\ub97c \ucde8\ud574 \ub2ec\ub77c\uace0 \uc9c4\uc5b8\ud588\ub2e4. \uad70\uccb4\ub974\ud2b8\uc758 \uc99d\uc5b8\uc5d0 \ub530\ub974\uba74 \ub9cc\uc288\ud0c0\uc778\uc740 \uadf8\uc5d0\uac8c \ubc29\uae08 \ubcf8 \uac83\uc740 \uc78a\uace0 \ubd89\uc740 \uad70\ub300\uc640\uc758 \uc2f8\uc6c0\uc5d0\ub098 \uc9d1\uc911\ud558\ub77c\uace0 \ub9d0\ud588\ub2e4. \uad70\uccb4\ub974\ud2b8\ub294 \ub9cc\uc288\ud0c0\uc778\uc758 \ud589\uc704\ub97c \u201c\ucc45\uc784\uc73c\ub85c\ubd80\ud130\uc758 \ub3c4\ud53c, \ub3c4\ub355\uc801 \uc2e4\uaca9\u201d\uc774\ub77c\uace0 \ud3c9\ud588\ub2e4. \ub9cc\uc288\ud0c0\uc778\uc774 \uc804\ubc94\ud610\uc758\ub85c \uae30\uc18c\ub418\uc5c8\uc744 \ub2f9\uc2dc 17\uac1c \uae30\uc18c\uc0ac\ud56d \uc911 11\uac1c\uac00 \ud06c\ub9bc \uc804\uc5ed\uc5d0\uc11c\uc758 \uc720\ub300\uc778\uacfc \ud3ec\ub85c \ud559\ub300 \ubc0f \ud559\uc0b4\uc5d0 \uad00\ub828\ub41c \uac83\uc774\ub2e4.", "sentence_answer": "\ub2f9\uc2dc 17\uac1c \uae30\uc18c\uc0ac\ud56d \uc911 11\uac1c\uac00 \ud06c\ub9bc \uc804\uc5ed\uc5d0\uc11c\uc758 \uc720\ub300\uc778\uacfc \ud3ec\ub85c \ud559\ub300 \ubc0f \ud559\uc0b4\uc5d0 \uad00\ub828\ub41c \uac83\uc774\ub2e4.", "paragraph_id": "6586570-18-1", "paragraph_question": "question: \ub9cc\uc288\ud0c0\uc778\uc740 \uc804\ubc94\ud610\uc758\ub85c \uba87\uac1c\uc758 \uc0ac\ud56d\uc5d0 \uae30\uc18c \ub418\uc5c8\ub294\uac00?, context: \ud06c\ub9bc \uc804\uc5ed\uc5d0\uc11c \ub9cc\uc288\ud0c0\uc778\uc740 \uc18c\ub828\uc778\ub4e4\uc5d0 \ub300\ud55c \uc794\ud559\ud589\uc704, \ud2b9\ud788 \uc544\uc778\uc790\uce20\uadf8\ub8e8\ud39c D\ubd80\ub300\uc758 \ud559\uc0b4\ud589\uc704\uc5d0 \uac04\uc811\uc801\uc73c\ub85c \uc5f0\ub8e8\ub418\uc5c8\ub2e4. \uc544\uc778\uc790\uce20\uadf8\ub8e8\ud39c\uc740 \ub098\uce58 \uce5c\uc704\ub300(SS) \uc9d1\ub2e8 \uc911 \ud558\ub098\ub85c\uc11c, \uc720\ub7fd\uc758 \uc720\ub300\uc778 \ubc15\uba78\uc744 \uadf8 \uc784\ubb34\ub85c \uc0bc\uace0 \uc788\uc5c8\ub2e4. \uc544\uc778\uc790\uce20\uadf8\ub8e8\ud39c D\ubd80\ub300\ub294 \ub9cc\uc288\ud0c0\uc778\uc758 \uc81c11\uad70\uc774 \uac00\ub294 \uae38\uc744 \ub530\ub77c\ub2e4\ub154\uace0, \ub9cc\uc288\ud0c0\uc778\uc5d0\uac8c \ucc28\ub7c9, \uc5f0\ub8cc, \uc6b4\uc804\ubcd1 \ub4f1\uc744 \uc81c\uacf5\ubc1b\uc558\ub2e4. \uc544\uc778\uc790\uce20\uadf8\ub8e8\ud39c\uc774 \uc720\ub300\uc778\ub4e4\uc744 \ucd1d\uc0b4\ud558\ub824\uace0 \uacc4\ud68d\ud55c \uc9c0\uc5ed\uc5d0 \ud5cc\ubcd1\ub4e4\uc774 \uc904\uc744 \ucce4\ub294\ub370 \uc774\ub294 \ud0c8\ucd9c\ud558\ub294 \uc0ac\ub78c\uc774 \uc5c6\ub3c4\ub85d \ud558\uae30 \uc704\ud568\uc774\uc5c8\ub2e4. \uc544\uc778\uc790\uce20\uadf8\ub8e8\ud39c D\ubd80\ub300\uac00 \uc720\ub300\uc778 \uc5ec\uc131\uacfc \uc544\ub3d9\ub4e4\uc744 \ud559\uc0b4\ud558\ub294 \uac83\uc744 \ubcf4\uace0 \ucda9\uaca9\uc744 \ubc1b\uc740 \uc6b8\ub9ac\ud788 \uad70\uccb4\ub974\ud2b8(Ulrich Gunzert) \ub300\uc704\ub294 \ub9cc\uc288\ud0c0\uc778\uc5d0\uac8c \uac00\uc11c \ud559\uc0b4\uc744 \uba48\ucd94\uae30 \uc704\ud574 \uc870\uce58\ub97c \ucde8\ud574 \ub2ec\ub77c\uace0 \uc9c4\uc5b8\ud588\ub2e4. \uad70\uccb4\ub974\ud2b8\uc758 \uc99d\uc5b8\uc5d0 \ub530\ub974\uba74 \ub9cc\uc288\ud0c0\uc778\uc740 \uadf8\uc5d0\uac8c \ubc29\uae08 \ubcf8 \uac83\uc740 \uc78a\uace0 \ubd89\uc740 \uad70\ub300\uc640\uc758 \uc2f8\uc6c0\uc5d0\ub098 \uc9d1\uc911\ud558\ub77c\uace0 \ub9d0\ud588\ub2e4. \uad70\uccb4\ub974\ud2b8\ub294 \ub9cc\uc288\ud0c0\uc778\uc758 \ud589\uc704\ub97c \u201c\ucc45\uc784\uc73c\ub85c\ubd80\ud130\uc758 \ub3c4\ud53c, \ub3c4\ub355\uc801 \uc2e4\uaca9\u201d\uc774\ub77c\uace0 \ud3c9\ud588\ub2e4. \ub9cc\uc288\ud0c0\uc778\uc774 \uc804\ubc94\ud610\uc758\ub85c \uae30\uc18c\ub418\uc5c8\uc744 \ub2f9\uc2dc 17\uac1c \uae30\uc18c\uc0ac\ud56d \uc911 11\uac1c\uac00 \ud06c\ub9bc \uc804\uc5ed\uc5d0\uc11c\uc758 \uc720\ub300\uc778\uacfc \ud3ec\ub85c \ud559\ub300 \ubc0f \ud559\uc0b4\uc5d0 \uad00\ub828\ub41c \uac83\uc774\ub2e4."} {"question": "18\ub300 \ub300\uc120 \ud6c4\ubcf4\uc778 \ubb38\uc7ac\uc778\uc774 \uc18d\ud55c \uc815\ub2f9\uc740?", "paragraph": "\uc120\uac70\ubc29\uc1a1\ud1a0\ub860\uc704\uc6d0\ud68c\uac00 \u300c\uacf5\uc9c1\uc120\uac70\ubc95\u300d\uc81c82\uc870\uc7582, \uc81c82\uc870\uc7583 \ubc0f \u300c\uc815\ub2f9\ubc95\u300d39\uc870\uc5d0 \uc758\ud574\uc11c 2012\ub144 12\uc6d4 4\uc77c\uacfc 10\uc77c, 16\uc77c \uad6d\ud68c\uc5d0 \uacf5\uc9c1\uc120\uac70\ubc95\uc5d0 \ub530\ub978 \ucd08\uccad \uc790\uaca9\uc744 \uac16\ucd98 \ub300\uc120 \ud6c4\ubcf4\ub4e4\uc744 \ucd94\ucc9c\ud558\uc5ec, \uac01\uac01 \uc815\uce58, \uc678\uad50, \uc548\ubcf4, \ud1b5\uc77c, \uacbd\uc81c, \ubcf5\uc9c0, \ub178\ub3d9, \ud658\uacbd, \uc0ac\ud68c, \uad50\uc721, \uacfc\ud559, \ubb38\ud654 \uc5ec\uc131 \ub4f1\uc744 \uc8fc\uc81c\ub85c \uc120\uac70\ubc29\uc1a1\ud1a0\ub860\ud68c\ub97c \uac1c\ucd5c\ud558\uc600\ub2e4. 5\uc11d \uc774\uc0c1 \uad6d\ud68c \uc758\uc11d\uc744 \uac00\uc9c4 \uc815\ub2f9 \ud6c4\ubcf4\uc774\uc678\uc5d0 \uc5ec\ub860\uc870\uc0ac\uc5d0\uc11c 5% \uc774\uc0c1 \uc9c0\uc9c0\ub97c \uc5bb\uc740 \ud6c4\ubcf4\uc640 \uc774\uc804 \uc120\uac70\uc5d0\uc11c 3%\uc774\uc0c1 \ub4dd\ud45c\ud55c \ud6c4\ubcf4\uac00 \uc5c6\uc5b4, 5\uc11d \uc774\uc0c1 \uc758\uc11d\uc744 \uac00\uc9c4 \uc815\ub2f9 \ud6c4\ubcf4\uc778 \ubc15\uadfc\ud61c, \ubb38\uc7ac\uc778, \uc774\uc815\ud76c\ud6c4\ubcf4\uac00 \ud1a0\ub860\ud68c\uc5d0 \ucd08\uccad\ub418\uc5c8\ub2e4. \uc138 \ubc88\uc9f8 \ud1a0\ub860\uc5d0\uc120 \uc774\uc815\ud76c \ud1b5\ud569\uc9c4\ubcf4\ub2f9 \ub300\uc120 \ud6c4\ubcf4\uac00 \uc0ac\ud1f4\ud558\uc5ec, \uc0c8\ub204\ub9ac\ub2f9 \ub300\uc120 \ud6c4\ubcf4 \ubc15\uadfc\ud61c\uc640 \ubbfc\uc8fc\ud1b5\ud569\ub2f9 \ub300\uc120 \ud6c4\ubcf4 \ubb38\uc7ac\uc778\uc758 \uc591\uc790\ud1a0\ub860\uc73c\ub85c \uc9c4\ud589\ub418\uc5c8\ub2e4.", "answer": "\ubbfc\uc8fc\ud1b5\ud569\ub2f9", "sentence": "\uc138 \ubc88\uc9f8 \ud1a0\ub860\uc5d0\uc120 \uc774\uc815\ud76c \ud1b5\ud569\uc9c4\ubcf4\ub2f9 \ub300\uc120 \ud6c4\ubcf4\uac00 \uc0ac\ud1f4\ud558\uc5ec, \uc0c8\ub204\ub9ac\ub2f9 \ub300\uc120 \ud6c4\ubcf4 \ubc15\uadfc\ud61c\uc640 \ubbfc\uc8fc\ud1b5\ud569\ub2f9 \ub300\uc120 \ud6c4\ubcf4 \ubb38\uc7ac\uc778\uc758 \uc591\uc790\ud1a0\ub860\uc73c\ub85c \uc9c4\ud589\ub418\uc5c8\ub2e4.", "paragraph_sentence": "\uc120\uac70\ubc29\uc1a1\ud1a0\ub860\uc704\uc6d0\ud68c\uac00 \u300c\uacf5\uc9c1\uc120\uac70\ubc95\u300d\uc81c82\uc870\uc7582, \uc81c82\uc870\uc7583 \ubc0f \u300c\uc815\ub2f9\ubc95\u300d39\uc870\uc5d0 \uc758\ud574\uc11c 2012\ub144 12\uc6d4 4\uc77c\uacfc 10\uc77c, 16\uc77c \uad6d\ud68c\uc5d0 \uacf5\uc9c1\uc120\uac70\ubc95\uc5d0 \ub530\ub978 \ucd08\uccad \uc790\uaca9\uc744 \uac16\ucd98 \ub300\uc120 \ud6c4\ubcf4\ub4e4\uc744 \ucd94\ucc9c\ud558\uc5ec, \uac01\uac01 \uc815\uce58, \uc678\uad50, \uc548\ubcf4, \ud1b5\uc77c, \uacbd\uc81c, \ubcf5\uc9c0, \ub178\ub3d9, \ud658\uacbd, \uc0ac\ud68c, \uad50\uc721, \uacfc\ud559, \ubb38\ud654 \uc5ec\uc131 \ub4f1\uc744 \uc8fc\uc81c\ub85c \uc120\uac70\ubc29\uc1a1\ud1a0\ub860\ud68c\ub97c \uac1c\ucd5c\ud558\uc600\ub2e4. 5\uc11d \uc774\uc0c1 \uad6d\ud68c \uc758\uc11d\uc744 \uac00\uc9c4 \uc815\ub2f9 \ud6c4\ubcf4\uc774\uc678\uc5d0 \uc5ec\ub860\uc870\uc0ac\uc5d0\uc11c 5% \uc774\uc0c1 \uc9c0\uc9c0\ub97c \uc5bb\uc740 \ud6c4\ubcf4\uc640 \uc774\uc804 \uc120\uac70\uc5d0\uc11c 3%\uc774\uc0c1 \ub4dd\ud45c\ud55c \ud6c4\ubcf4\uac00 \uc5c6\uc5b4, 5\uc11d \uc774\uc0c1 \uc758\uc11d\uc744 \uac00\uc9c4 \uc815\ub2f9 \ud6c4\ubcf4\uc778 \ubc15\uadfc\ud61c, \ubb38\uc7ac\uc778, \uc774\uc815\ud76c\ud6c4\ubcf4\uac00 \ud1a0\ub860\ud68c\uc5d0 \ucd08\uccad\ub418\uc5c8\ub2e4. \uc138 \ubc88\uc9f8 \ud1a0\ub860\uc5d0\uc120 \uc774\uc815\ud76c \ud1b5\ud569\uc9c4\ubcf4\ub2f9 \ub300\uc120 \ud6c4\ubcf4\uac00 \uc0ac\ud1f4\ud558\uc5ec, \uc0c8\ub204\ub9ac\ub2f9 \ub300\uc120 \ud6c4\ubcf4 \ubc15\uadfc\ud61c\uc640 \ubbfc\uc8fc\ud1b5\ud569\ub2f9 \ub300\uc120 \ud6c4\ubcf4 \ubb38\uc7ac\uc778\uc758 \uc591\uc790\ud1a0\ub860\uc73c\ub85c \uc9c4\ud589\ub418\uc5c8\ub2e4. ", "paragraph_answer": "\uc120\uac70\ubc29\uc1a1\ud1a0\ub860\uc704\uc6d0\ud68c\uac00 \u300c\uacf5\uc9c1\uc120\uac70\ubc95\u300d\uc81c82\uc870\uc7582, \uc81c82\uc870\uc7583 \ubc0f \u300c\uc815\ub2f9\ubc95\u300d39\uc870\uc5d0 \uc758\ud574\uc11c 2012\ub144 12\uc6d4 4\uc77c\uacfc 10\uc77c, 16\uc77c \uad6d\ud68c\uc5d0 \uacf5\uc9c1\uc120\uac70\ubc95\uc5d0 \ub530\ub978 \ucd08\uccad \uc790\uaca9\uc744 \uac16\ucd98 \ub300\uc120 \ud6c4\ubcf4\ub4e4\uc744 \ucd94\ucc9c\ud558\uc5ec, \uac01\uac01 \uc815\uce58, \uc678\uad50, \uc548\ubcf4, \ud1b5\uc77c, \uacbd\uc81c, \ubcf5\uc9c0, \ub178\ub3d9, \ud658\uacbd, \uc0ac\ud68c, \uad50\uc721, \uacfc\ud559, \ubb38\ud654 \uc5ec\uc131 \ub4f1\uc744 \uc8fc\uc81c\ub85c \uc120\uac70\ubc29\uc1a1\ud1a0\ub860\ud68c\ub97c \uac1c\ucd5c\ud558\uc600\ub2e4. 5\uc11d \uc774\uc0c1 \uad6d\ud68c \uc758\uc11d\uc744 \uac00\uc9c4 \uc815\ub2f9 \ud6c4\ubcf4\uc774\uc678\uc5d0 \uc5ec\ub860\uc870\uc0ac\uc5d0\uc11c 5% \uc774\uc0c1 \uc9c0\uc9c0\ub97c \uc5bb\uc740 \ud6c4\ubcf4\uc640 \uc774\uc804 \uc120\uac70\uc5d0\uc11c 3%\uc774\uc0c1 \ub4dd\ud45c\ud55c \ud6c4\ubcf4\uac00 \uc5c6\uc5b4, 5\uc11d \uc774\uc0c1 \uc758\uc11d\uc744 \uac00\uc9c4 \uc815\ub2f9 \ud6c4\ubcf4\uc778 \ubc15\uadfc\ud61c, \ubb38\uc7ac\uc778, \uc774\uc815\ud76c\ud6c4\ubcf4\uac00 \ud1a0\ub860\ud68c\uc5d0 \ucd08\uccad\ub418\uc5c8\ub2e4. \uc138 \ubc88\uc9f8 \ud1a0\ub860\uc5d0\uc120 \uc774\uc815\ud76c \ud1b5\ud569\uc9c4\ubcf4\ub2f9 \ub300\uc120 \ud6c4\ubcf4\uac00 \uc0ac\ud1f4\ud558\uc5ec, \uc0c8\ub204\ub9ac\ub2f9 \ub300\uc120 \ud6c4\ubcf4 \ubc15\uadfc\ud61c\uc640 \ubbfc\uc8fc\ud1b5\ud569\ub2f9 \ub300\uc120 \ud6c4\ubcf4 \ubb38\uc7ac\uc778\uc758 \uc591\uc790\ud1a0\ub860\uc73c\ub85c \uc9c4\ud589\ub418\uc5c8\ub2e4.", "sentence_answer": "\uc138 \ubc88\uc9f8 \ud1a0\ub860\uc5d0\uc120 \uc774\uc815\ud76c \ud1b5\ud569\uc9c4\ubcf4\ub2f9 \ub300\uc120 \ud6c4\ubcf4\uac00 \uc0ac\ud1f4\ud558\uc5ec, \uc0c8\ub204\ub9ac\ub2f9 \ub300\uc120 \ud6c4\ubcf4 \ubc15\uadfc\ud61c\uc640 \ubbfc\uc8fc\ud1b5\ud569\ub2f9 \ub300\uc120 \ud6c4\ubcf4 \ubb38\uc7ac\uc778\uc758 \uc591\uc790\ud1a0\ub860\uc73c\ub85c \uc9c4\ud589\ub418\uc5c8\ub2e4.", "paragraph_id": "6486088-1-1", "paragraph_question": "question: 18\ub300 \ub300\uc120 \ud6c4\ubcf4\uc778 \ubb38\uc7ac\uc778\uc774 \uc18d\ud55c \uc815\ub2f9\uc740?, context: \uc120\uac70\ubc29\uc1a1\ud1a0\ub860\uc704\uc6d0\ud68c\uac00 \u300c\uacf5\uc9c1\uc120\uac70\ubc95\u300d\uc81c82\uc870\uc7582, \uc81c82\uc870\uc7583 \ubc0f \u300c\uc815\ub2f9\ubc95\u300d39\uc870\uc5d0 \uc758\ud574\uc11c 2012\ub144 12\uc6d4 4\uc77c\uacfc 10\uc77c, 16\uc77c \uad6d\ud68c\uc5d0 \uacf5\uc9c1\uc120\uac70\ubc95\uc5d0 \ub530\ub978 \ucd08\uccad \uc790\uaca9\uc744 \uac16\ucd98 \ub300\uc120 \ud6c4\ubcf4\ub4e4\uc744 \ucd94\ucc9c\ud558\uc5ec, \uac01\uac01 \uc815\uce58, \uc678\uad50, \uc548\ubcf4, \ud1b5\uc77c, \uacbd\uc81c, \ubcf5\uc9c0, \ub178\ub3d9, \ud658\uacbd, \uc0ac\ud68c, \uad50\uc721, \uacfc\ud559, \ubb38\ud654 \uc5ec\uc131 \ub4f1\uc744 \uc8fc\uc81c\ub85c \uc120\uac70\ubc29\uc1a1\ud1a0\ub860\ud68c\ub97c \uac1c\ucd5c\ud558\uc600\ub2e4. 5\uc11d \uc774\uc0c1 \uad6d\ud68c \uc758\uc11d\uc744 \uac00\uc9c4 \uc815\ub2f9 \ud6c4\ubcf4\uc774\uc678\uc5d0 \uc5ec\ub860\uc870\uc0ac\uc5d0\uc11c 5% \uc774\uc0c1 \uc9c0\uc9c0\ub97c \uc5bb\uc740 \ud6c4\ubcf4\uc640 \uc774\uc804 \uc120\uac70\uc5d0\uc11c 3%\uc774\uc0c1 \ub4dd\ud45c\ud55c \ud6c4\ubcf4\uac00 \uc5c6\uc5b4, 5\uc11d \uc774\uc0c1 \uc758\uc11d\uc744 \uac00\uc9c4 \uc815\ub2f9 \ud6c4\ubcf4\uc778 \ubc15\uadfc\ud61c, \ubb38\uc7ac\uc778, \uc774\uc815\ud76c\ud6c4\ubcf4\uac00 \ud1a0\ub860\ud68c\uc5d0 \ucd08\uccad\ub418\uc5c8\ub2e4. \uc138 \ubc88\uc9f8 \ud1a0\ub860\uc5d0\uc120 \uc774\uc815\ud76c \ud1b5\ud569\uc9c4\ubcf4\ub2f9 \ub300\uc120 \ud6c4\ubcf4\uac00 \uc0ac\ud1f4\ud558\uc5ec, \uc0c8\ub204\ub9ac\ub2f9 \ub300\uc120 \ud6c4\ubcf4 \ubc15\uadfc\ud61c\uc640 \ubbfc\uc8fc\ud1b5\ud569\ub2f9 \ub300\uc120 \ud6c4\ubcf4 \ubb38\uc7ac\uc778\uc758 \uc591\uc790\ud1a0\ub860\uc73c\ub85c \uc9c4\ud589\ub418\uc5c8\ub2e4."} {"question": "\ud0a4\ud504\ub85c\uc2a4\uac00 \uc624\uc2a4\ub9cc\uc758 \uc601\ud1a0\ub85c \ub0a8\uac8c\ub41c \uae30\uac04\uc740?", "paragraph": "\uadf8\ub7ec\ub098 \uc911\uc694\ud55c \uc2b9\ub9ac\uc5d0\ub3c4 \ubd88\uad6c\ud558\uace0 \uc2e0\uc131 \ub3d9\ub9f9\uc758 \ubd88\ud611\ud654\uc74c\uc740 \uc774 \uc2b9\ub9ac\ub85c\ubd80\ud130 \uc5bb\uc740 \uc774\uc775\uc744 \uadf9\ub300\ud654\ud558\ub294 \ub370 \uc2e4\ud328\ud588\ub2e4. \uae30\ub3c5\uad50 \uc138\uacc4\ub97c \uc704\ud574 \ucf58\uc2a4\ud0c4\ud2f0\ub178\ud3f4\ub9ac\uc2a4\ub97c \ud68c\ubcf5\ud558\uae30 \uc704\ud55c \ubc29\uc548\uc73c\ub85c \ub2e4\ub974\ub2e4\ub12c\uc2a4 \ud574\ud611\uc744 \uacf5\uaca9\ud558\ub294 \uacc4\ud68d\uc740 \ub3d9\ub9f9\uad70\ub4e4\ub07c\ub9ac\uc758 \uc5b8\uc7c1\uc73c\ub85c \uc5c6\uc5c8\ub358 \uac83\uc774 \ub418\uace0 \ub9d0\uc558\ub2e4. \ub180\ub77c\uc6b8 \ub9cc\ud55c \ub178\ub825\uc73c\ub85c \uc624\uc2a4\ub9cc \uad70\uc740 \uadf8\ub4e4\uc758 \ud574\uad70\uc744 \uc218\ubcf5\ud558\uace0, \uc131\uacf5\uc801\uc73c\ub85c \ubca0\ub124\uce58\uc544\uc758 \uac24\ub9ac\uc5b4\uc2a4\ub97c \ubcf5\uc81c\ud558\uc600\ub2e4. 1572\ub144 \uc9c0\uc911\ud574\uc5d0\uc11c \uc774\uc81c\uae4c\uc9c0 \ub098\ud0c0\ub09c \ud568\uc120 \uc911 \uac00\uc7a5 \ud070 \ucd5c\uace0 \uc131\ub2a5\uc758 \ud568\uc120\ub4e4\uc744 \ub354\ud558\uc5ec 150\ucc99\uc758 \uac24\ub9ac\uc120\uacfc 8\ucc99\uc758 \uac24\ub9ac\uc5b4\uc2a4\uac00 \ub9cc\ub4e4\uc5b4\uc84c\ub2e4. 6\ub2ec \ub3d9\uc548 250\ucc99(8\ucc99\uc758 \uac24\ub9ac\uc5b4\uc2a4\ub97c \ud3ec\ud568\ud55c)\uc758 \uc0c8\ub85c\uc6b4 \ud568\ub300\uac00 \ub3d9\uc9c0\uc911\ud574\uc5d0\uc11c \uc624\uc2a4\ub9cc \ud574\uad70\uc774 \uc6b0\uc704\ub97c \uc8fc\uc7a5\ud558\ub294 \uac83\uc744 \uac00\ub2a5\ucf00 \ud558\uc600\ub2e4. 1573\ub144 5\uc6d4 7\uc77c \ubca0\ub124\uce58\uc544\uad70\uc740 \ub808\ud310\ud1a0 \ud574\uc804\uc5d0\uc11c \uaca8\uc6b0 \ub450 \ub2ec \uc804\uc778 1571\ub144 8\uc6d4 3\uc77c \ud53c\uc584\ub808 \ud30c\uc0e4 \ud718\ud558\uc758 \uc624\uc2a4\ub9cc \ud22c\ub974\ud06c \uad70\uc5d0 \ud568\ub77d\ub41c \ud0a4\ud504\ub85c\uc2a4\uc5d0 \ub300\ud55c \uc624\uc2a4\ub9cc\uc758 \uc810\uc720\ub97c \uc778\uc815\ud558\ub294 \uc870\uc57d\uc744 \ub9fa\uc5c8\uace0, \uc774 \uc12c\uc740 \ub2e4\uc74c 3\uc138\uae30 \ub3d9\uc548 \uc624\uc2a4\ub9cc\uc758 \uc601\ud1a0\ub85c \ub0a8\uac8c \ub418\uc5c8\ub2e4. \uadf8\ub9ac\uace0 \uc5ec\ub984\uc5d0 \uc624\uc2a4\ub9cc \ud574\uad70\uc740 \uc9c0\uc9c8\ud559\uc801\uc73c\ub85c \ucde8\uc57d\ud55c \uc2dc\uce60\ub9ac\uc544\uc758 \ud574\uc548\uacfc \uc774\ud0c8\ub9ac\uc544 \ub0a8\ubd80\ub97c \uc57d\ud0c8\ud558\uc5ec \ud53c\ud574\ub97c \uc785\ud614\ub2e4.", "answer": "3\uc138\uae30 \ub3d9\uc548", "sentence": "\uc774 \uc12c\uc740 \ub2e4\uc74c 3\uc138\uae30 \ub3d9\uc548 \uc624\uc2a4\ub9cc\uc758 \uc601\ud1a0\ub85c \ub0a8\uac8c \ub418\uc5c8\ub2e4.", "paragraph_sentence": "\uadf8\ub7ec\ub098 \uc911\uc694\ud55c \uc2b9\ub9ac\uc5d0\ub3c4 \ubd88\uad6c\ud558\uace0 \uc2e0\uc131 \ub3d9\ub9f9\uc758 \ubd88\ud611\ud654\uc74c\uc740 \uc774 \uc2b9\ub9ac\ub85c\ubd80\ud130 \uc5bb\uc740 \uc774\uc775\uc744 \uadf9\ub300\ud654\ud558\ub294 \ub370 \uc2e4\ud328\ud588\ub2e4. \uae30\ub3c5\uad50 \uc138\uacc4\ub97c \uc704\ud574 \ucf58\uc2a4\ud0c4\ud2f0\ub178\ud3f4\ub9ac\uc2a4\ub97c \ud68c\ubcf5\ud558\uae30 \uc704\ud55c \ubc29\uc548\uc73c\ub85c \ub2e4\ub974\ub2e4\ub12c\uc2a4 \ud574\ud611\uc744 \uacf5\uaca9\ud558\ub294 \uacc4\ud68d\uc740 \ub3d9\ub9f9\uad70\ub4e4\ub07c\ub9ac\uc758 \uc5b8\uc7c1\uc73c\ub85c \uc5c6\uc5c8\ub358 \uac83\uc774 \ub418\uace0 \ub9d0\uc558\ub2e4. \ub180\ub77c\uc6b8 \ub9cc\ud55c \ub178\ub825\uc73c\ub85c \uc624\uc2a4\ub9cc \uad70\uc740 \uadf8\ub4e4\uc758 \ud574\uad70\uc744 \uc218\ubcf5\ud558\uace0, \uc131\uacf5\uc801\uc73c\ub85c \ubca0\ub124\uce58\uc544\uc758 \uac24\ub9ac\uc5b4\uc2a4\ub97c \ubcf5\uc81c\ud558\uc600\ub2e4. 1572\ub144 \uc9c0\uc911\ud574\uc5d0\uc11c \uc774\uc81c\uae4c\uc9c0 \ub098\ud0c0\ub09c \ud568\uc120 \uc911 \uac00\uc7a5 \ud070 \ucd5c\uace0 \uc131\ub2a5\uc758 \ud568\uc120\ub4e4\uc744 \ub354\ud558\uc5ec 150\ucc99\uc758 \uac24\ub9ac\uc120\uacfc 8\ucc99\uc758 \uac24\ub9ac\uc5b4\uc2a4\uac00 \ub9cc\ub4e4\uc5b4\uc84c\ub2e4. 6\ub2ec \ub3d9\uc548 250\ucc99(8\ucc99\uc758 \uac24\ub9ac\uc5b4\uc2a4\ub97c \ud3ec\ud568\ud55c)\uc758 \uc0c8\ub85c\uc6b4 \ud568\ub300\uac00 \ub3d9\uc9c0\uc911\ud574\uc5d0\uc11c \uc624\uc2a4\ub9cc \ud574\uad70\uc774 \uc6b0\uc704\ub97c \uc8fc\uc7a5\ud558\ub294 \uac83\uc744 \uac00\ub2a5\ucf00 \ud558\uc600\ub2e4. 1573\ub144 5\uc6d4 7\uc77c \ubca0\ub124\uce58\uc544\uad70\uc740 \ub808\ud310\ud1a0 \ud574\uc804\uc5d0\uc11c \uaca8\uc6b0 \ub450 \ub2ec \uc804\uc778 1571\ub144 8\uc6d4 3\uc77c \ud53c\uc584\ub808 \ud30c\uc0e4 \ud718\ud558\uc758 \uc624\uc2a4\ub9cc \ud22c\ub974\ud06c \uad70\uc5d0 \ud568\ub77d\ub41c \ud0a4\ud504\ub85c\uc2a4\uc5d0 \ub300\ud55c \uc624\uc2a4\ub9cc\uc758 \uc810\uc720\ub97c \uc778\uc815\ud558\ub294 \uc870\uc57d\uc744 \ub9fa\uc5c8\uace0, \uc774 \uc12c\uc740 \ub2e4\uc74c 3\uc138\uae30 \ub3d9\uc548 \uc624\uc2a4\ub9cc\uc758 \uc601\ud1a0\ub85c \ub0a8\uac8c \ub418\uc5c8\ub2e4. \uadf8\ub9ac\uace0 \uc5ec\ub984\uc5d0 \uc624\uc2a4\ub9cc \ud574\uad70\uc740 \uc9c0\uc9c8\ud559\uc801\uc73c\ub85c \ucde8\uc57d\ud55c \uc2dc\uce60\ub9ac\uc544\uc758 \ud574\uc548\uacfc \uc774\ud0c8\ub9ac\uc544 \ub0a8\ubd80\ub97c \uc57d\ud0c8\ud558\uc5ec \ud53c\ud574\ub97c \uc785\ud614\ub2e4.", "paragraph_answer": "\uadf8\ub7ec\ub098 \uc911\uc694\ud55c \uc2b9\ub9ac\uc5d0\ub3c4 \ubd88\uad6c\ud558\uace0 \uc2e0\uc131 \ub3d9\ub9f9\uc758 \ubd88\ud611\ud654\uc74c\uc740 \uc774 \uc2b9\ub9ac\ub85c\ubd80\ud130 \uc5bb\uc740 \uc774\uc775\uc744 \uadf9\ub300\ud654\ud558\ub294 \ub370 \uc2e4\ud328\ud588\ub2e4. \uae30\ub3c5\uad50 \uc138\uacc4\ub97c \uc704\ud574 \ucf58\uc2a4\ud0c4\ud2f0\ub178\ud3f4\ub9ac\uc2a4\ub97c \ud68c\ubcf5\ud558\uae30 \uc704\ud55c \ubc29\uc548\uc73c\ub85c \ub2e4\ub974\ub2e4\ub12c\uc2a4 \ud574\ud611\uc744 \uacf5\uaca9\ud558\ub294 \uacc4\ud68d\uc740 \ub3d9\ub9f9\uad70\ub4e4\ub07c\ub9ac\uc758 \uc5b8\uc7c1\uc73c\ub85c \uc5c6\uc5c8\ub358 \uac83\uc774 \ub418\uace0 \ub9d0\uc558\ub2e4. \ub180\ub77c\uc6b8 \ub9cc\ud55c \ub178\ub825\uc73c\ub85c \uc624\uc2a4\ub9cc \uad70\uc740 \uadf8\ub4e4\uc758 \ud574\uad70\uc744 \uc218\ubcf5\ud558\uace0, \uc131\uacf5\uc801\uc73c\ub85c \ubca0\ub124\uce58\uc544\uc758 \uac24\ub9ac\uc5b4\uc2a4\ub97c \ubcf5\uc81c\ud558\uc600\ub2e4. 1572\ub144 \uc9c0\uc911\ud574\uc5d0\uc11c \uc774\uc81c\uae4c\uc9c0 \ub098\ud0c0\ub09c \ud568\uc120 \uc911 \uac00\uc7a5 \ud070 \ucd5c\uace0 \uc131\ub2a5\uc758 \ud568\uc120\ub4e4\uc744 \ub354\ud558\uc5ec 150\ucc99\uc758 \uac24\ub9ac\uc120\uacfc 8\ucc99\uc758 \uac24\ub9ac\uc5b4\uc2a4\uac00 \ub9cc\ub4e4\uc5b4\uc84c\ub2e4. 6\ub2ec \ub3d9\uc548 250\ucc99(8\ucc99\uc758 \uac24\ub9ac\uc5b4\uc2a4\ub97c \ud3ec\ud568\ud55c)\uc758 \uc0c8\ub85c\uc6b4 \ud568\ub300\uac00 \ub3d9\uc9c0\uc911\ud574\uc5d0\uc11c \uc624\uc2a4\ub9cc \ud574\uad70\uc774 \uc6b0\uc704\ub97c \uc8fc\uc7a5\ud558\ub294 \uac83\uc744 \uac00\ub2a5\ucf00 \ud558\uc600\ub2e4. 1573\ub144 5\uc6d4 7\uc77c \ubca0\ub124\uce58\uc544\uad70\uc740 \ub808\ud310\ud1a0 \ud574\uc804\uc5d0\uc11c \uaca8\uc6b0 \ub450 \ub2ec \uc804\uc778 1571\ub144 8\uc6d4 3\uc77c \ud53c\uc584\ub808 \ud30c\uc0e4 \ud718\ud558\uc758 \uc624\uc2a4\ub9cc \ud22c\ub974\ud06c \uad70\uc5d0 \ud568\ub77d\ub41c \ud0a4\ud504\ub85c\uc2a4\uc5d0 \ub300\ud55c \uc624\uc2a4\ub9cc\uc758 \uc810\uc720\ub97c \uc778\uc815\ud558\ub294 \uc870\uc57d\uc744 \ub9fa\uc5c8\uace0, \uc774 \uc12c\uc740 \ub2e4\uc74c 3\uc138\uae30 \ub3d9\uc548 \uc624\uc2a4\ub9cc\uc758 \uc601\ud1a0\ub85c \ub0a8\uac8c \ub418\uc5c8\ub2e4. \uadf8\ub9ac\uace0 \uc5ec\ub984\uc5d0 \uc624\uc2a4\ub9cc \ud574\uad70\uc740 \uc9c0\uc9c8\ud559\uc801\uc73c\ub85c \ucde8\uc57d\ud55c \uc2dc\uce60\ub9ac\uc544\uc758 \ud574\uc548\uacfc \uc774\ud0c8\ub9ac\uc544 \ub0a8\ubd80\ub97c \uc57d\ud0c8\ud558\uc5ec \ud53c\ud574\ub97c \uc785\ud614\ub2e4.", "sentence_answer": "\uc774 \uc12c\uc740 \ub2e4\uc74c 3\uc138\uae30 \ub3d9\uc548 \uc624\uc2a4\ub9cc\uc758 \uc601\ud1a0\ub85c \ub0a8\uac8c \ub418\uc5c8\ub2e4.", "paragraph_id": "6306274-7-2", "paragraph_question": "question: \ud0a4\ud504\ub85c\uc2a4\uac00 \uc624\uc2a4\ub9cc\uc758 \uc601\ud1a0\ub85c \ub0a8\uac8c\ub41c \uae30\uac04\uc740?, context: \uadf8\ub7ec\ub098 \uc911\uc694\ud55c \uc2b9\ub9ac\uc5d0\ub3c4 \ubd88\uad6c\ud558\uace0 \uc2e0\uc131 \ub3d9\ub9f9\uc758 \ubd88\ud611\ud654\uc74c\uc740 \uc774 \uc2b9\ub9ac\ub85c\ubd80\ud130 \uc5bb\uc740 \uc774\uc775\uc744 \uadf9\ub300\ud654\ud558\ub294 \ub370 \uc2e4\ud328\ud588\ub2e4. \uae30\ub3c5\uad50 \uc138\uacc4\ub97c \uc704\ud574 \ucf58\uc2a4\ud0c4\ud2f0\ub178\ud3f4\ub9ac\uc2a4\ub97c \ud68c\ubcf5\ud558\uae30 \uc704\ud55c \ubc29\uc548\uc73c\ub85c \ub2e4\ub974\ub2e4\ub12c\uc2a4 \ud574\ud611\uc744 \uacf5\uaca9\ud558\ub294 \uacc4\ud68d\uc740 \ub3d9\ub9f9\uad70\ub4e4\ub07c\ub9ac\uc758 \uc5b8\uc7c1\uc73c\ub85c \uc5c6\uc5c8\ub358 \uac83\uc774 \ub418\uace0 \ub9d0\uc558\ub2e4. \ub180\ub77c\uc6b8 \ub9cc\ud55c \ub178\ub825\uc73c\ub85c \uc624\uc2a4\ub9cc \uad70\uc740 \uadf8\ub4e4\uc758 \ud574\uad70\uc744 \uc218\ubcf5\ud558\uace0, \uc131\uacf5\uc801\uc73c\ub85c \ubca0\ub124\uce58\uc544\uc758 \uac24\ub9ac\uc5b4\uc2a4\ub97c \ubcf5\uc81c\ud558\uc600\ub2e4. 1572\ub144 \uc9c0\uc911\ud574\uc5d0\uc11c \uc774\uc81c\uae4c\uc9c0 \ub098\ud0c0\ub09c \ud568\uc120 \uc911 \uac00\uc7a5 \ud070 \ucd5c\uace0 \uc131\ub2a5\uc758 \ud568\uc120\ub4e4\uc744 \ub354\ud558\uc5ec 150\ucc99\uc758 \uac24\ub9ac\uc120\uacfc 8\ucc99\uc758 \uac24\ub9ac\uc5b4\uc2a4\uac00 \ub9cc\ub4e4\uc5b4\uc84c\ub2e4. 6\ub2ec \ub3d9\uc548 250\ucc99(8\ucc99\uc758 \uac24\ub9ac\uc5b4\uc2a4\ub97c \ud3ec\ud568\ud55c)\uc758 \uc0c8\ub85c\uc6b4 \ud568\ub300\uac00 \ub3d9\uc9c0\uc911\ud574\uc5d0\uc11c \uc624\uc2a4\ub9cc \ud574\uad70\uc774 \uc6b0\uc704\ub97c \uc8fc\uc7a5\ud558\ub294 \uac83\uc744 \uac00\ub2a5\ucf00 \ud558\uc600\ub2e4. 1573\ub144 5\uc6d4 7\uc77c \ubca0\ub124\uce58\uc544\uad70\uc740 \ub808\ud310\ud1a0 \ud574\uc804\uc5d0\uc11c \uaca8\uc6b0 \ub450 \ub2ec \uc804\uc778 1571\ub144 8\uc6d4 3\uc77c \ud53c\uc584\ub808 \ud30c\uc0e4 \ud718\ud558\uc758 \uc624\uc2a4\ub9cc \ud22c\ub974\ud06c \uad70\uc5d0 \ud568\ub77d\ub41c \ud0a4\ud504\ub85c\uc2a4\uc5d0 \ub300\ud55c \uc624\uc2a4\ub9cc\uc758 \uc810\uc720\ub97c \uc778\uc815\ud558\ub294 \uc870\uc57d\uc744 \ub9fa\uc5c8\uace0, \uc774 \uc12c\uc740 \ub2e4\uc74c 3\uc138\uae30 \ub3d9\uc548 \uc624\uc2a4\ub9cc\uc758 \uc601\ud1a0\ub85c \ub0a8\uac8c \ub418\uc5c8\ub2e4. \uadf8\ub9ac\uace0 \uc5ec\ub984\uc5d0 \uc624\uc2a4\ub9cc \ud574\uad70\uc740 \uc9c0\uc9c8\ud559\uc801\uc73c\ub85c \ucde8\uc57d\ud55c \uc2dc\uce60\ub9ac\uc544\uc758 \ud574\uc548\uacfc \uc774\ud0c8\ub9ac\uc544 \ub0a8\ubd80\ub97c \uc57d\ud0c8\ud558\uc5ec \ud53c\ud574\ub97c \uc785\ud614\ub2e4."} {"question": "Fry song\uc744 \ub9cc\ub4e0 \uc0ac\ub78c\uc740?", "paragraph": "\u300a\uc5b4\ub4dc\ubca4\ucc98 \ud0c0\uc784\u300b\uc5d0\ub294 \uc138\uc774\ub354, \ucf00\ub2c8, \uc62c\uc2a8 \ub4f1\uc758 \uc131\uc6b0\ub4e4\uc774 \uc9c1\uc811 \ubd80\ub978 \ub178\ub798\uac00 \uc790\uc8fc \ub4f1\uc7a5\ud55c\ub2e4. \uadf9\uc911 \ub4f1\uc7a5\uc778\ubb3c\ub4e4\uc740 \ub178\ub798\ub97c \ud1b5\ud574 \uc790\uc2e0\uc758 \uac10\uc815\uc744 \ud45c\ud604\ud558\ub294 \uacbd\uc6b0\uac00 \uc788\ub294\ub370, \ub300\ud45c\uc801\uc73c\ub85c \ub9c8\ub974\uc140\ub9b0\uc758 \u3008I'm Just Your Problem\u3009\uc774\ub098 \ud540\uc758 \u3008All Gummed Up Inside\u3009 \ub4f1\uc774 \uc788\ub2e4. \ubc30\uacbd \uc74c\uc545\uc744 \ub9e1\ub294 \ubc30\uc774\uc2dc\ucee4\uc2a4\uc640 \ud0a4\ud37c\uac00 \uc791\uace1\ud55c \uace1 \uc678\uc5d0\ub3c4 \uc2a4\ud1a0\ub9ac\ubcf4\ub4dc \uc791\uac00\uac00 \uc81c\uc791\ud55c \ub178\ub798\ub3c4 \uc788\ub2e4. \uc608\ub97c \ub4e4\uc5b4 \u3008Fry Song\u3009\uc740 \ub9ac\ubca0\uce74 \uc288\uac70\uac00 \ub9cc\ub4e4\uc5c8\ub2e4. \uc774 \uace1\uc740 \uc288\uac70\uac00 \ucc98\uc74c\uc73c\ub85c \uc81c\uc791\ud55c \uc5d0\ud53c\uc18c\ub4dc\uc778 \u3008It Came from the Nightosphere\u3009\uc5d0 \uc0bd\uc785\ub418\uc5c8\ub2e4. \ud504\ub808\ub4dc\ub808\uc774\ud130 \uc2a4\ud29c\ub514\uc624\ub294 \u3008Fry Song\u3009\uc758 \ub370\ubaa8 \ubc0f \uc804\uccb4 \ubc84\uc804\uc744 \uacf5\uac1c\ud558\uae30\ub3c4 \ud588\ub2e4.", "answer": "\ub9ac\ubca0\uce74 \uc288\uac70", "sentence": "\uc608\ub97c \ub4e4\uc5b4 \u3008Fry Song\u3009\uc740 \ub9ac\ubca0\uce74 \uc288\uac70 \uac00 \ub9cc\ub4e4\uc5c8\ub2e4.", "paragraph_sentence": "\u300a\uc5b4\ub4dc\ubca4\ucc98 \ud0c0\uc784\u300b\uc5d0\ub294 \uc138\uc774\ub354, \ucf00\ub2c8, \uc62c\uc2a8 \ub4f1\uc758 \uc131\uc6b0\ub4e4\uc774 \uc9c1\uc811 \ubd80\ub978 \ub178\ub798\uac00 \uc790\uc8fc \ub4f1\uc7a5\ud55c\ub2e4. \uadf9\uc911 \ub4f1\uc7a5\uc778\ubb3c\ub4e4\uc740 \ub178\ub798\ub97c \ud1b5\ud574 \uc790\uc2e0\uc758 \uac10\uc815\uc744 \ud45c\ud604\ud558\ub294 \uacbd\uc6b0\uac00 \uc788\ub294\ub370, \ub300\ud45c\uc801\uc73c\ub85c \ub9c8\ub974\uc140\ub9b0\uc758 \u3008I'm Just Your Problem\u3009\uc774\ub098 \ud540\uc758 \u3008All Gummed Up Inside\u3009 \ub4f1\uc774 \uc788\ub2e4. \ubc30\uacbd \uc74c\uc545\uc744 \ub9e1\ub294 \ubc30\uc774\uc2dc\ucee4\uc2a4\uc640 \ud0a4\ud37c\uac00 \uc791\uace1\ud55c \uace1 \uc678\uc5d0\ub3c4 \uc2a4\ud1a0\ub9ac\ubcf4\ub4dc \uc791\uac00\uac00 \uc81c\uc791\ud55c \ub178\ub798\ub3c4 \uc788\ub2e4. \uc608\ub97c \ub4e4\uc5b4 \u3008Fry Song\u3009\uc740 \ub9ac\ubca0\uce74 \uc288\uac70 \uac00 \ub9cc\ub4e4\uc5c8\ub2e4. \uc774 \uace1\uc740 \uc288\uac70\uac00 \ucc98\uc74c\uc73c\ub85c \uc81c\uc791\ud55c \uc5d0\ud53c\uc18c\ub4dc\uc778 \u3008It Came from the Nightosphere\u3009\uc5d0 \uc0bd\uc785\ub418\uc5c8\ub2e4. \ud504\ub808\ub4dc\ub808\uc774\ud130 \uc2a4\ud29c\ub514\uc624\ub294 \u3008Fry Song\u3009\uc758 \ub370\ubaa8 \ubc0f \uc804\uccb4 \ubc84\uc804\uc744 \uacf5\uac1c\ud558\uae30\ub3c4 \ud588\ub2e4.", "paragraph_answer": "\u300a\uc5b4\ub4dc\ubca4\ucc98 \ud0c0\uc784\u300b\uc5d0\ub294 \uc138\uc774\ub354, \ucf00\ub2c8, \uc62c\uc2a8 \ub4f1\uc758 \uc131\uc6b0\ub4e4\uc774 \uc9c1\uc811 \ubd80\ub978 \ub178\ub798\uac00 \uc790\uc8fc \ub4f1\uc7a5\ud55c\ub2e4. \uadf9\uc911 \ub4f1\uc7a5\uc778\ubb3c\ub4e4\uc740 \ub178\ub798\ub97c \ud1b5\ud574 \uc790\uc2e0\uc758 \uac10\uc815\uc744 \ud45c\ud604\ud558\ub294 \uacbd\uc6b0\uac00 \uc788\ub294\ub370, \ub300\ud45c\uc801\uc73c\ub85c \ub9c8\ub974\uc140\ub9b0\uc758 \u3008I'm Just Your Problem\u3009\uc774\ub098 \ud540\uc758 \u3008All Gummed Up Inside\u3009 \ub4f1\uc774 \uc788\ub2e4. \ubc30\uacbd \uc74c\uc545\uc744 \ub9e1\ub294 \ubc30\uc774\uc2dc\ucee4\uc2a4\uc640 \ud0a4\ud37c\uac00 \uc791\uace1\ud55c \uace1 \uc678\uc5d0\ub3c4 \uc2a4\ud1a0\ub9ac\ubcf4\ub4dc \uc791\uac00\uac00 \uc81c\uc791\ud55c \ub178\ub798\ub3c4 \uc788\ub2e4. \uc608\ub97c \ub4e4\uc5b4 \u3008Fry Song\u3009\uc740 \ub9ac\ubca0\uce74 \uc288\uac70 \uac00 \ub9cc\ub4e4\uc5c8\ub2e4. \uc774 \uace1\uc740 \uc288\uac70\uac00 \ucc98\uc74c\uc73c\ub85c \uc81c\uc791\ud55c \uc5d0\ud53c\uc18c\ub4dc\uc778 \u3008It Came from the Nightosphere\u3009\uc5d0 \uc0bd\uc785\ub418\uc5c8\ub2e4. \ud504\ub808\ub4dc\ub808\uc774\ud130 \uc2a4\ud29c\ub514\uc624\ub294 \u3008Fry Song\u3009\uc758 \ub370\ubaa8 \ubc0f \uc804\uccb4 \ubc84\uc804\uc744 \uacf5\uac1c\ud558\uae30\ub3c4 \ud588\ub2e4.", "sentence_answer": "\uc608\ub97c \ub4e4\uc5b4 \u3008Fry Song\u3009\uc740 \ub9ac\ubca0\uce74 \uc288\uac70 \uac00 \ub9cc\ub4e4\uc5c8\ub2e4.", "paragraph_id": "6569097-14-0", "paragraph_question": "question: Fry song\uc744 \ub9cc\ub4e0 \uc0ac\ub78c\uc740?, context: \u300a\uc5b4\ub4dc\ubca4\ucc98 \ud0c0\uc784\u300b\uc5d0\ub294 \uc138\uc774\ub354, \ucf00\ub2c8, \uc62c\uc2a8 \ub4f1\uc758 \uc131\uc6b0\ub4e4\uc774 \uc9c1\uc811 \ubd80\ub978 \ub178\ub798\uac00 \uc790\uc8fc \ub4f1\uc7a5\ud55c\ub2e4. \uadf9\uc911 \ub4f1\uc7a5\uc778\ubb3c\ub4e4\uc740 \ub178\ub798\ub97c \ud1b5\ud574 \uc790\uc2e0\uc758 \uac10\uc815\uc744 \ud45c\ud604\ud558\ub294 \uacbd\uc6b0\uac00 \uc788\ub294\ub370, \ub300\ud45c\uc801\uc73c\ub85c \ub9c8\ub974\uc140\ub9b0\uc758 \u3008I'm Just Your Problem\u3009\uc774\ub098 \ud540\uc758 \u3008All Gummed Up Inside\u3009 \ub4f1\uc774 \uc788\ub2e4. \ubc30\uacbd \uc74c\uc545\uc744 \ub9e1\ub294 \ubc30\uc774\uc2dc\ucee4\uc2a4\uc640 \ud0a4\ud37c\uac00 \uc791\uace1\ud55c \uace1 \uc678\uc5d0\ub3c4 \uc2a4\ud1a0\ub9ac\ubcf4\ub4dc \uc791\uac00\uac00 \uc81c\uc791\ud55c \ub178\ub798\ub3c4 \uc788\ub2e4. \uc608\ub97c \ub4e4\uc5b4 \u3008Fry Song\u3009\uc740 \ub9ac\ubca0\uce74 \uc288\uac70\uac00 \ub9cc\ub4e4\uc5c8\ub2e4. \uc774 \uace1\uc740 \uc288\uac70\uac00 \ucc98\uc74c\uc73c\ub85c \uc81c\uc791\ud55c \uc5d0\ud53c\uc18c\ub4dc\uc778 \u3008It Came from the Nightosphere\u3009\uc5d0 \uc0bd\uc785\ub418\uc5c8\ub2e4. \ud504\ub808\ub4dc\ub808\uc774\ud130 \uc2a4\ud29c\ub514\uc624\ub294 \u3008Fry Song\u3009\uc758 \ub370\ubaa8 \ubc0f \uc804\uccb4 \ubc84\uc804\uc744 \uacf5\uac1c\ud558\uae30\ub3c4 \ud588\ub2e4."} {"question": "\uc6f9\ud230 \uc791\uac00\uac00 \uc790\uc2e0\uc758 \uc791\ud488\uc5d0 \ub300\ud55c \ud53c\ub4dc\ubc31\uc744 \ubc1b\ub294 \uc218\ub2e8\uc740?", "paragraph": "\uc2a4\ub9c8\ud2b8\ud3f0\uacfc \ud0dc\ube14\ub9bf PC\uc758 \ub4f1\uc7a5\uc740 \ubb34\uc120\uc778\ud130\ub137\uc758 \ud655\uc0b0 \ubc0f \uc790\uc728\uc131, \uc560\ud50c\ub9ac\ucf00\uc774\uc158 \uc0ac\uc6a9\uc73c\ub85c \uc778\ud55c \uc778\ud130\ub137 \uc811\uc18d\uc758 \uac04\uc18c\ud654, \uc18c\uc15c \ub124\ud2b8\uc6cc\ud06c \uc774\uc6a9\uc73c\ub85c \uc778\ud55c \uc815\ubcf4\uad50\ud658\uc758 \uc6a9\uc774\uc131 \ub4f1\uc758 \uacb0\uacfc\ub97c \ubcf4\uc5ec\uc8fc\uace0 \uc788\ub2e4. \ud0dc\ube14\ub9bf\uc758 \ub4f1\uc7a5\uc73c\ub85c \ud654\uba74\uc758 \uc0ac\uc774\uc988\uac00 \ub300\ud615\ud654\ub418\uace0 \ud574\uc0c1\ub3c4\uac00 \uac1c\uc120\ub418\ub294 \ub4f1 \uc11c\ube44\uc2a4 \ud658\uacbd\uc774 \uac1c\uc120\ub418\uc5b4 \uc6f9\uc2a4\ud1a0\uc5b4 \uae30\ubc18\uc758 \ubaa8\ubc14\uc77c \ub9cc\ud654 \uc11c\ube44\uc2a4 \uc2dc\uc7a5\uc758 \uac00\ub2a5\uc131\uc740 \ub354\uc6b1 \ucee4\uc9c0\uace0 \uc788\uc73c\uba70, \uc774\ub4e4 \uae30\uae30\uc758 \u2018\uc774\ub3d9\uc131\u2019\uc740 \uc6f9\ud230 \uad6c\ub3c5\uc758 \uc7a5\uc18c, \uc2dc\uac04\uc758 \uc790\uc728\uc131\uc744 \ud655\ubcf4\ud558\uac8c \ud574 \uc8fc\uc5c8\ub2e4. \uc608\ub97c \ub4e4\uc5b4, \u2018\uc774\ub3d9\uc131\u2019\uc740 \uc9c1\uc7a5\uc778\uc758 \ucd9c\ud1f4\uadfc \uc2dc\uac04, \ud559\uc0dd\ub4e4\uc758 \uc26c\ub294 \uc2dc\uac04 \ub4f1\uc758 \uc6f9\ud230 \uad6c\ub3c5\uc744 \uac00\ub2a5\ud558\uac8c \ud560 \ubfd0 \uc544\ub2c8\ub77c \uc6f9\ud230\uc758 \uc5c5\ub370\uc774\ud2b8\uc640 \ub3d9\uc2dc\uc5d0 \uc2e4\uc2dc\uac04 \uc6f9\ud230 \uad6c\ub3c5\uc744 \uac00\ub2a5\ud558\uac8c \ud574 \uc8fc\uc5c8\ub2e4. \ud55c\ud3b8, SNS\ub97c \uc774\uc6a9\ud558\uc5ec \uc2e4\uc2dc\uac04\uc73c\ub85c \uc778\uae30 \uc788\ub294 \uc6f9\ud230\uc758 \uc18c\uc2dd\uc744 \uad50\ub958\ud560 \uc218 \uc788\uc73c\uba70, \uc791\uac00\ub294 \uc2e4\uc2dc\uac04\uc73c\ub85c \ubc18\uc751 \ubc0f \uc791\ud488\uc5d0 \ub300\ud55c \ud3c9\uac00\ub3c4 \uc5bb\uc744 \uc218 \uc788\ub2e4.", "answer": "SNS", "sentence": "\ud55c\ud3b8, SNS \ub97c \uc774\uc6a9\ud558\uc5ec \uc2e4\uc2dc\uac04\uc73c\ub85c \uc778\uae30 \uc788\ub294 \uc6f9\ud230\uc758 \uc18c\uc2dd\uc744 \uad50\ub958\ud560 \uc218 \uc788\uc73c\uba70, \uc791\uac00\ub294 \uc2e4\uc2dc\uac04\uc73c\ub85c \ubc18\uc751 \ubc0f \uc791\ud488\uc5d0 \ub300\ud55c \ud3c9\uac00\ub3c4 \uc5bb\uc744 \uc218 \uc788\ub2e4.", "paragraph_sentence": "\uc2a4\ub9c8\ud2b8\ud3f0\uacfc \ud0dc\ube14\ub9bf PC\uc758 \ub4f1\uc7a5\uc740 \ubb34\uc120\uc778\ud130\ub137\uc758 \ud655\uc0b0 \ubc0f \uc790\uc728\uc131, \uc560\ud50c\ub9ac\ucf00\uc774\uc158 \uc0ac\uc6a9\uc73c\ub85c \uc778\ud55c \uc778\ud130\ub137 \uc811\uc18d\uc758 \uac04\uc18c\ud654, \uc18c\uc15c \ub124\ud2b8\uc6cc\ud06c \uc774\uc6a9\uc73c\ub85c \uc778\ud55c \uc815\ubcf4\uad50\ud658\uc758 \uc6a9\uc774\uc131 \ub4f1\uc758 \uacb0\uacfc\ub97c \ubcf4\uc5ec\uc8fc\uace0 \uc788\ub2e4. \ud0dc\ube14\ub9bf\uc758 \ub4f1\uc7a5\uc73c\ub85c \ud654\uba74\uc758 \uc0ac\uc774\uc988\uac00 \ub300\ud615\ud654\ub418\uace0 \ud574\uc0c1\ub3c4\uac00 \uac1c\uc120\ub418\ub294 \ub4f1 \uc11c\ube44\uc2a4 \ud658\uacbd\uc774 \uac1c\uc120\ub418\uc5b4 \uc6f9\uc2a4\ud1a0\uc5b4 \uae30\ubc18\uc758 \ubaa8\ubc14\uc77c \ub9cc\ud654 \uc11c\ube44\uc2a4 \uc2dc\uc7a5\uc758 \uac00\ub2a5\uc131\uc740 \ub354\uc6b1 \ucee4\uc9c0\uace0 \uc788\uc73c\uba70, \uc774\ub4e4 \uae30\uae30\uc758 \u2018\uc774\ub3d9\uc131\u2019\uc740 \uc6f9\ud230 \uad6c\ub3c5\uc758 \uc7a5\uc18c, \uc2dc\uac04\uc758 \uc790\uc728\uc131\uc744 \ud655\ubcf4\ud558\uac8c \ud574 \uc8fc\uc5c8\ub2e4. \uc608\ub97c \ub4e4\uc5b4, \u2018\uc774\ub3d9\uc131\u2019\uc740 \uc9c1\uc7a5\uc778\uc758 \ucd9c\ud1f4\uadfc \uc2dc\uac04, \ud559\uc0dd\ub4e4\uc758 \uc26c\ub294 \uc2dc\uac04 \ub4f1\uc758 \uc6f9\ud230 \uad6c\ub3c5\uc744 \uac00\ub2a5\ud558\uac8c \ud560 \ubfd0 \uc544\ub2c8\ub77c \uc6f9\ud230\uc758 \uc5c5\ub370\uc774\ud2b8\uc640 \ub3d9\uc2dc\uc5d0 \uc2e4\uc2dc\uac04 \uc6f9\ud230 \uad6c\ub3c5\uc744 \uac00\ub2a5\ud558\uac8c \ud574 \uc8fc\uc5c8\ub2e4. \ud55c\ud3b8, SNS \ub97c \uc774\uc6a9\ud558\uc5ec \uc2e4\uc2dc\uac04\uc73c\ub85c \uc778\uae30 \uc788\ub294 \uc6f9\ud230\uc758 \uc18c\uc2dd\uc744 \uad50\ub958\ud560 \uc218 \uc788\uc73c\uba70, \uc791\uac00\ub294 \uc2e4\uc2dc\uac04\uc73c\ub85c \ubc18\uc751 \ubc0f \uc791\ud488\uc5d0 \ub300\ud55c \ud3c9\uac00\ub3c4 \uc5bb\uc744 \uc218 \uc788\ub2e4. ", "paragraph_answer": "\uc2a4\ub9c8\ud2b8\ud3f0\uacfc \ud0dc\ube14\ub9bf PC\uc758 \ub4f1\uc7a5\uc740 \ubb34\uc120\uc778\ud130\ub137\uc758 \ud655\uc0b0 \ubc0f \uc790\uc728\uc131, \uc560\ud50c\ub9ac\ucf00\uc774\uc158 \uc0ac\uc6a9\uc73c\ub85c \uc778\ud55c \uc778\ud130\ub137 \uc811\uc18d\uc758 \uac04\uc18c\ud654, \uc18c\uc15c \ub124\ud2b8\uc6cc\ud06c \uc774\uc6a9\uc73c\ub85c \uc778\ud55c \uc815\ubcf4\uad50\ud658\uc758 \uc6a9\uc774\uc131 \ub4f1\uc758 \uacb0\uacfc\ub97c \ubcf4\uc5ec\uc8fc\uace0 \uc788\ub2e4. \ud0dc\ube14\ub9bf\uc758 \ub4f1\uc7a5\uc73c\ub85c \ud654\uba74\uc758 \uc0ac\uc774\uc988\uac00 \ub300\ud615\ud654\ub418\uace0 \ud574\uc0c1\ub3c4\uac00 \uac1c\uc120\ub418\ub294 \ub4f1 \uc11c\ube44\uc2a4 \ud658\uacbd\uc774 \uac1c\uc120\ub418\uc5b4 \uc6f9\uc2a4\ud1a0\uc5b4 \uae30\ubc18\uc758 \ubaa8\ubc14\uc77c \ub9cc\ud654 \uc11c\ube44\uc2a4 \uc2dc\uc7a5\uc758 \uac00\ub2a5\uc131\uc740 \ub354\uc6b1 \ucee4\uc9c0\uace0 \uc788\uc73c\uba70, \uc774\ub4e4 \uae30\uae30\uc758 \u2018\uc774\ub3d9\uc131\u2019\uc740 \uc6f9\ud230 \uad6c\ub3c5\uc758 \uc7a5\uc18c, \uc2dc\uac04\uc758 \uc790\uc728\uc131\uc744 \ud655\ubcf4\ud558\uac8c \ud574 \uc8fc\uc5c8\ub2e4. \uc608\ub97c \ub4e4\uc5b4, \u2018\uc774\ub3d9\uc131\u2019\uc740 \uc9c1\uc7a5\uc778\uc758 \ucd9c\ud1f4\uadfc \uc2dc\uac04, \ud559\uc0dd\ub4e4\uc758 \uc26c\ub294 \uc2dc\uac04 \ub4f1\uc758 \uc6f9\ud230 \uad6c\ub3c5\uc744 \uac00\ub2a5\ud558\uac8c \ud560 \ubfd0 \uc544\ub2c8\ub77c \uc6f9\ud230\uc758 \uc5c5\ub370\uc774\ud2b8\uc640 \ub3d9\uc2dc\uc5d0 \uc2e4\uc2dc\uac04 \uc6f9\ud230 \uad6c\ub3c5\uc744 \uac00\ub2a5\ud558\uac8c \ud574 \uc8fc\uc5c8\ub2e4. \ud55c\ud3b8, SNS \ub97c \uc774\uc6a9\ud558\uc5ec \uc2e4\uc2dc\uac04\uc73c\ub85c \uc778\uae30 \uc788\ub294 \uc6f9\ud230\uc758 \uc18c\uc2dd\uc744 \uad50\ub958\ud560 \uc218 \uc788\uc73c\uba70, \uc791\uac00\ub294 \uc2e4\uc2dc\uac04\uc73c\ub85c \ubc18\uc751 \ubc0f \uc791\ud488\uc5d0 \ub300\ud55c \ud3c9\uac00\ub3c4 \uc5bb\uc744 \uc218 \uc788\ub2e4.", "sentence_answer": "\ud55c\ud3b8, SNS \ub97c \uc774\uc6a9\ud558\uc5ec \uc2e4\uc2dc\uac04\uc73c\ub85c \uc778\uae30 \uc788\ub294 \uc6f9\ud230\uc758 \uc18c\uc2dd\uc744 \uad50\ub958\ud560 \uc218 \uc788\uc73c\uba70, \uc791\uac00\ub294 \uc2e4\uc2dc\uac04\uc73c\ub85c \ubc18\uc751 \ubc0f \uc791\ud488\uc5d0 \ub300\ud55c \ud3c9\uac00\ub3c4 \uc5bb\uc744 \uc218 \uc788\ub2e4.", "paragraph_id": "5849228-1-1", "paragraph_question": "question: \uc6f9\ud230 \uc791\uac00\uac00 \uc790\uc2e0\uc758 \uc791\ud488\uc5d0 \ub300\ud55c \ud53c\ub4dc\ubc31\uc744 \ubc1b\ub294 \uc218\ub2e8\uc740?, context: \uc2a4\ub9c8\ud2b8\ud3f0\uacfc \ud0dc\ube14\ub9bf PC\uc758 \ub4f1\uc7a5\uc740 \ubb34\uc120\uc778\ud130\ub137\uc758 \ud655\uc0b0 \ubc0f \uc790\uc728\uc131, \uc560\ud50c\ub9ac\ucf00\uc774\uc158 \uc0ac\uc6a9\uc73c\ub85c \uc778\ud55c \uc778\ud130\ub137 \uc811\uc18d\uc758 \uac04\uc18c\ud654, \uc18c\uc15c \ub124\ud2b8\uc6cc\ud06c \uc774\uc6a9\uc73c\ub85c \uc778\ud55c \uc815\ubcf4\uad50\ud658\uc758 \uc6a9\uc774\uc131 \ub4f1\uc758 \uacb0\uacfc\ub97c \ubcf4\uc5ec\uc8fc\uace0 \uc788\ub2e4. \ud0dc\ube14\ub9bf\uc758 \ub4f1\uc7a5\uc73c\ub85c \ud654\uba74\uc758 \uc0ac\uc774\uc988\uac00 \ub300\ud615\ud654\ub418\uace0 \ud574\uc0c1\ub3c4\uac00 \uac1c\uc120\ub418\ub294 \ub4f1 \uc11c\ube44\uc2a4 \ud658\uacbd\uc774 \uac1c\uc120\ub418\uc5b4 \uc6f9\uc2a4\ud1a0\uc5b4 \uae30\ubc18\uc758 \ubaa8\ubc14\uc77c \ub9cc\ud654 \uc11c\ube44\uc2a4 \uc2dc\uc7a5\uc758 \uac00\ub2a5\uc131\uc740 \ub354\uc6b1 \ucee4\uc9c0\uace0 \uc788\uc73c\uba70, \uc774\ub4e4 \uae30\uae30\uc758 \u2018\uc774\ub3d9\uc131\u2019\uc740 \uc6f9\ud230 \uad6c\ub3c5\uc758 \uc7a5\uc18c, \uc2dc\uac04\uc758 \uc790\uc728\uc131\uc744 \ud655\ubcf4\ud558\uac8c \ud574 \uc8fc\uc5c8\ub2e4. \uc608\ub97c \ub4e4\uc5b4, \u2018\uc774\ub3d9\uc131\u2019\uc740 \uc9c1\uc7a5\uc778\uc758 \ucd9c\ud1f4\uadfc \uc2dc\uac04, \ud559\uc0dd\ub4e4\uc758 \uc26c\ub294 \uc2dc\uac04 \ub4f1\uc758 \uc6f9\ud230 \uad6c\ub3c5\uc744 \uac00\ub2a5\ud558\uac8c \ud560 \ubfd0 \uc544\ub2c8\ub77c \uc6f9\ud230\uc758 \uc5c5\ub370\uc774\ud2b8\uc640 \ub3d9\uc2dc\uc5d0 \uc2e4\uc2dc\uac04 \uc6f9\ud230 \uad6c\ub3c5\uc744 \uac00\ub2a5\ud558\uac8c \ud574 \uc8fc\uc5c8\ub2e4. \ud55c\ud3b8, SNS\ub97c \uc774\uc6a9\ud558\uc5ec \uc2e4\uc2dc\uac04\uc73c\ub85c \uc778\uae30 \uc788\ub294 \uc6f9\ud230\uc758 \uc18c\uc2dd\uc744 \uad50\ub958\ud560 \uc218 \uc788\uc73c\uba70, \uc791\uac00\ub294 \uc2e4\uc2dc\uac04\uc73c\ub85c \ubc18\uc751 \ubc0f \uc791\ud488\uc5d0 \ub300\ud55c \ud3c9\uac00\ub3c4 \uc5bb\uc744 \uc218 \uc788\ub2e4."} {"question": "\uc120\uc218\ud611\uc774 \uc9c4\ud589\ud588\ub358 \ucd1d 6\ucc28\ub840\uc758 \uacbd\ub9e4\uc5d0\uc11c \uc5bb\uc740 \uc784\uc218\ud601 \ub3d5\uae30 \uc131\uae08\uc758 \uae08\uc561\uc740?", "paragraph": "2002\ub144\uc5d0\ub294, \uc784\uc218\ud601\uc774 \uc4f0\ub7ec\uc838\uc11c \ubcd1\uc0c1\uc5d0 \uc788\ub294\uc9c0 2\ub144\uc9f8\uac00 \ub418\uc5c8\uc9c0\ub9cc, 2001\ub144\uae4c\uc9c0\ub9cc \ud574\ub3c4, \uc4f0\ub824\uc84c\uc744 \ub54c\uc640 \ub2f9\uc2dc \uae30\uc0ac\uc640 \uad00\uc2ec\uc774 \uc904\uc5b4\ub4e4\uc5c8\uc73c\ub098, \ub86f\ub370 \uc790\uc774\uc5b8\uce20\uc758 \ub530\ub73b\ud55c \ubc30\ub824\uc640, \uad00\uc2ec\uc18d\uc5d0 \ubcd1\uc218\ubc1c\uc5d0\ub294 \ud070 \uc5b4\ub824\uc6c0\uc744 \uacaa\uc9c0 \uc54a\uc558\uc5c8\uace0, 2001\ub144 4\uc6d4 18\uc77c\uc5d0\ub294 \uc784\uc218\ud601\uc758 \ub0a0\ub85c \uc9c0\uc815\ud558\uc5ec, \uc544\ub4e4\uc774 \uc2dc\uad6c\ub97c \ud558\uace0, 4\uc6d4 18\uc77c\uc5d0 \uc788\uc5c8\ub358, \uacbd\uae30\uc758 \uc218\uc775\uae08\uc778 4100\ub9cc \uc6d0\uc744 \uc784\uc218\ud601\uc758 \uce58\ub8cc\ube44\ub85c \uc804\ud588\ub2e4. \uadf8\ub7ec\ub098, 2002\ub144\ubd80\ud130\ub294 \ub86f\ub370 \uc790\uc774\uc5b8\uce20\uc5d0\uc11c \uc0dd\ud65c\ube44 \uc9c0\uc6d0\uc5d0 \ub300\ud55c \uc5b4\ub5a0\ud55c \uc5b8\uc9c8\ub3c4 \ubc1b\uc9c0 \ubabb\ud588\uace0, \ubcd1\uc6d0\ube44\uc5d0 \ub300\ud574\uc11c\ub3c4 \uc9c0\uc18d\uc801\uc73c\ub85c \uc9c0\uae09\ud558\uae30 \ud798\ub4e4\ub2e4\ub294 \uc758\uacac\uc744 \uc804\ub2ec\ubc1b\uc558\ub2e4. \ub9cc\uc57d, \ub86f\ub370 \uc790\uc774\uc5b8\uce20\uac00 \uc0dd\ud65c\ube44 \uc9c0\uc6d0\uacfc \uce58\ub8cc\ube44 \uc9c0\uc6d0\uc744 \ud558\uc9c0 \uc54a\ub294\ub2e4\uba74, \uac00\uc871\ub4e4\uc774 \uc5c4\uccad\ub09c \uc0dd\ud65c\ube44\uc640 \uce58\ub8cc\ube44\ub97c \uac10\ub2f9\ud574\uc57c \ub418\uc5c8\ub2e4. \uadf8\ub7ec\ub098, \ub450\uc0b0 \ubca0\uc5b4\uc2a4\uc758 \ud32c\ub4e4\uc774 \uc131\uae08\uc744 \uc9c0\uc6d0\ud558\uace0, \uacc4\uc18d\ub418\ub294 \uad00\uc2ec\uc73c\ub85c \uc5b4\ub824\uc6b4 \uc77c\uc5d0 \ud76c\ub9dd\uc744 \uac00\uc9c0\uac8c \ub418\uc5c8\ub2e4. \ud558\uc9c0\ub9cc, \uad00\uc2ec\ub3c4 \uc77c\uc2dc\uc801\uc774\uc5c8\uace0, \uc784\uc218\ud601\uc5d0 \uad00\ud55c \uae30\uc0ac\ub294 2002\ub144 4\uc6d4\uc5d0 \uc62c\ub77c\uc628 \uc774\ud6c4, 11\uc6d4\uae4c\uc9c0 \uad00\ub828 \uae30\uc0ac\ub294\ucee4\ub155, \uc784\uc218\ud601\uc5d0 \uad00\ud55c 1\uc904\ub3c4 \ub098\uc624\uc9c0 \uc54a\uc558\ub2e4. \uae30\uc0ac\uac00 \ub098\uc624\uc9c0 \uc54a\ub2e4\uac00, \uc120\uc218\ud611\uc774 \uc784\uc218\ud601 \ub3d5\uae30 \uc790\uc120 \uacbd\ub9e4\ub97c \uc5f4\uba74\uc11c \uae30\uc0ac\uac00 \ub098\uc624\uae30 \uc2dc\uc791\ud588\ub294\ub370, \uc774\ub54c \uc120\uc218\ud611\uc740 \uc784\uc218\ud601\uc758 \uce58\ub8cc\ube44\ub97c \uc704\ud574, \ubc15\ucc2c\ud638\uc758 \uae00\ub7ec\ube0c\uc640 \uc790\ud544 \uc0ac\uc778\uacf5, \ub2f9\uc2dc \uc2dc\uce74\uace0 \ud654\uc774\ud2b8\uc0ad\uc2a4\uc758 \ucf54\uce58\uc600\ub358 \uc774\ub9cc\uc218 \ucf54\uce58\uc758 \ubaa8\uc790, \ud3ec\uc218 \ubbf8\ud2b8, \uc54c\ub809\uc2a4 \ub85c\ub4dc\ub9ac\uac8c\uc2a4, \ub370\ub9ad \uc9c0\ud130 \ub4f1\uc758 \uc790\ud544 \uc0ac\uc778\ubcfc \ub4f1\uc774 \uc784\uc218\ud601\uc758 \uce58\ub8cc\ube44\ub97c \uc704\ud574 \ucd9c\ud488\ub418\uc5c8\ub2e4. \uc774\ud6c4, \uace8\ud504 \uc120\uc218\uc778 \ucd5c\uacbd\uc8fc\ub3c4 \uc784\uc218\ud601 \ub3d5\uae30\uc5d0 \uc790\uc2e0\uc758 \uc18c\uc7a5\uc6a9\ud488\uc744 \uacbd\ub9e4\uc5d0 \uae30\ud0c1\ud558\uba74\uc11c \uc784\uc218\ud601 \ub3d5\uae30\uc5d0 \ub3d9\ucc38\ud558\uc600\uace0, \ud22c\uc218 \ub79c\ub514 \uc874\uc2a8, \uc1fc\ud2b8\ud2b8\ub799 \uc120\uc218 \uae40\ub3d9\uc131\uae4c\uc9c0 \uc784\uc218\ud601 \ub3d5\uae30\uc5d0 \uc790\uc2e0\uc5d0 \uc560\uc7a5\ud488\uc744 \uacbd\ub9e4\uc5d0 \uae30\ud0c1\ud558\uba74\uc11c \ucc38\uac00\ud558\uac8c \ub418\uc5c8\ub2e4. \uacc4\uc18d\ud574\uc11c \uacbd\ub9e4\uc5d0 \ucc38\uac00\ud558\ub824\ub294 \uc2a4\ud3ec\uce20 \uc2a4\ud0c0\ub4e4\uc774 \ub298\uc5b4\ub098\uba74\uc11c, \uc120\uc218\ud611\uc740 \uc784\uc218\ud601 \ub3d5\uae30 \uc790\uc120 \uacbd\ub9e4\uae30\uac04\uc744 \uc5f0\uc7a5\ud558\uc600\uace0, \uc5f0\uc7a5\ud55c \uc774\ud6c4\uc5d0\ub3c4, \ub2f9\uc2dc \ub300\ud55c\ubbfc\uad6d \ucd95\uad6c \ub3cc\ud48d\uc744 \uc77c\uc73c\ucf30\ub358 \uac70\uc2a4 \ud788\ub529\ud06c, \ud64d\uba85\ubcf4, \uc548\uc815\ud658\uae4c\uc9c0 \uc790\uc2e0\uc758 \uc560\uc7a5\ud488\uc744 \uc784\uc218\ud601 \ub3d5\uae30 \uacbd\ub9e4\uc5d0 \uae30\ud0c1\ud558\uc600\ub2e4.\ubc15\ucc2c\ud638\ub294, 4\ucc28 \uacbd\ub9e4\uac00 \uc9c4\ud589\ub418\uace0 \uc788\ub294 \ub3c4\uc911, \ubbf8\uad6d\uc5d0\uc11c \uadc0\uad6d\ud558\uc5ec, \uc784\uc218\ud601\uc758 \ubcd1\uc0c1\uc5d0 \ubc29\ubb38\ud558\uc600\uace0, \uc774\ud6c4, 4\ucc28 \uacbd\ub9e4\uac00 \ub05d\ub09c \ud6c4, 5\ucc28 \uacbd\ub9e4\uac00 \uc5f4\ub9ac\uac8c \ub418\uc5c8\ub294\ub370, \ub2f9\uc2dc \uc77c\ubcf8 \ud504\ub85c \uc57c\uad6c\uc758 \uc120\uc218\uc600\ub358 \uad6c\ub300\uc131\uc774 \uc784\uc218\ud601\uc758 \uce58\ub8cc\ube44\ub97c \uc704\ud574 \ud55c\ud654 \uc774\uae00\uc2a4\ub97c \ud1b5\ud574\uc11c 2\ucc9c\ub9cc \uc6d0\uc744 \uc804\ub2ec\ud558\uc600\ub2e4. \uc784\uc218\ud601\uc758 \uce58\ub8cc\ube44\ub97c \uc704\ud55c 5\ucc28 \uacbd\ub9e4\uac00 \uc9c4\ud589\ub418\ub358 \ub3c4\uc911, \uace0\ub824\ub300\ud559\uad50 \uc57c\uad6c\ubd80 \uad50\uc6b0\ud68c\ub294, \uc784\uc218\ud601\uc744 \uc704\ud574 \uc784\uc218\ud601 \ub3d5\uae30 \uc131\uae08\uc73c\ub85c \uae08\uc77c\ubd09\uc744 \uc804\ub2ec\ud588\uace0, \uacc4\uc18d\ud574\uc11c \uc9c4\ud589\ub418\ub358 5\ucc28 \uacbd\ub9e4\ub294, 4\ucc28 \uacbd\ub9e4\uc5d0 \ube44\ud574 \ud070 \uc778\uae30\ub97c \ub04c\uc9c0 \ubabb\ud558\uace0 \ub05d\ub098\uac8c \ub418\uc5c8\ub2e4. \uc774\ud6c4, 5\ucc28 \uacbd\ub9e4\uac00 \ub05d\ub09c \ub4a4, 6\ucc28 \uacbd\ub9e4\uc5d0\uc11c\ub294 \ud788\uc5b4\ub85c\uc988\uac00 2002\ub144\uc5d0 \uc2e4\uc2dc\ud588\ub358 '\uc0ac\ub791\uc758 \uc131\uae08 \uc2dc\uad6c\ud589\uc0ac'\uc5d0\uc11c \ub9c8\ub828\ub41c 420\ub9cc \uc6d0\uc5d0\uc11c \uadf8 \uc808\ubc18\uc778 210\ub9cc \uc6d0\uc744 \uc784\uc218\ud601\uc758 \uce58\ub8cc\ube44\ub85c \uc9c0\uc6d0\ud588\uace0, \uc120\uc218\ud611\uc774 \ucd1d 6\ucc28\ub840\uc758 \uacbd\ub9e4\uc5d0\uc11c \uc5bb\uc740 \uc784\uc218\ud601 \ub3d5\uae30 \uc131\uae08\uc740 \ucd1d 2\ucc9c 9\ubc31\ub9cc \uc6d0\uc774\uc5c8\uace0, \uc774 \uc131\uae08\uc740 \uc804\uc561 \uc784\uc218\ud601\uc758 \uac00\uc871\ub4e4\uc5d0\uac8c \uc804\ub2ec\ub418\uc5c8\ub2e4.", "answer": "2\ucc9c 9\ubc31\ub9cc \uc6d0", "sentence": "\ucd1d 2\ucc9c 9\ubc31\ub9cc \uc6d0 \uc774\uc5c8\uace0, \uc774 \uc131\uae08\uc740 \uc804\uc561 \uc784\uc218\ud601\uc758 \uac00\uc871\ub4e4\uc5d0\uac8c \uc804\ub2ec\ub418\uc5c8\ub2e4.", "paragraph_sentence": "2002\ub144\uc5d0\ub294, \uc784\uc218\ud601\uc774 \uc4f0\ub7ec\uc838\uc11c \ubcd1\uc0c1\uc5d0 \uc788\ub294\uc9c0 2\ub144\uc9f8\uac00 \ub418\uc5c8\uc9c0\ub9cc, 2001\ub144\uae4c\uc9c0\ub9cc \ud574\ub3c4, \uc4f0\ub824\uc84c\uc744 \ub54c\uc640 \ub2f9\uc2dc \uae30\uc0ac\uc640 \uad00\uc2ec\uc774 \uc904\uc5b4\ub4e4\uc5c8\uc73c\ub098, \ub86f\ub370 \uc790\uc774\uc5b8\uce20\uc758 \ub530\ub73b\ud55c \ubc30\ub824\uc640, \uad00\uc2ec\uc18d\uc5d0 \ubcd1\uc218\ubc1c\uc5d0\ub294 \ud070 \uc5b4\ub824\uc6c0\uc744 \uacaa\uc9c0 \uc54a\uc558\uc5c8\uace0, 2001\ub144 4\uc6d4 18\uc77c\uc5d0\ub294 \uc784\uc218\ud601\uc758 \ub0a0\ub85c \uc9c0\uc815\ud558\uc5ec, \uc544\ub4e4\uc774 \uc2dc\uad6c\ub97c \ud558\uace0, 4\uc6d4 18\uc77c\uc5d0 \uc788\uc5c8\ub358, \uacbd\uae30\uc758 \uc218\uc775\uae08\uc778 4100\ub9cc \uc6d0\uc744 \uc784\uc218\ud601\uc758 \uce58\ub8cc\ube44\ub85c \uc804\ud588\ub2e4. \uadf8\ub7ec\ub098, 2002\ub144\ubd80\ud130\ub294 \ub86f\ub370 \uc790\uc774\uc5b8\uce20\uc5d0\uc11c \uc0dd\ud65c\ube44 \uc9c0\uc6d0\uc5d0 \ub300\ud55c \uc5b4\ub5a0\ud55c \uc5b8\uc9c8\ub3c4 \ubc1b\uc9c0 \ubabb\ud588\uace0, \ubcd1\uc6d0\ube44\uc5d0 \ub300\ud574\uc11c\ub3c4 \uc9c0\uc18d\uc801\uc73c\ub85c \uc9c0\uae09\ud558\uae30 \ud798\ub4e4\ub2e4\ub294 \uc758\uacac\uc744 \uc804\ub2ec\ubc1b\uc558\ub2e4. \ub9cc\uc57d, \ub86f\ub370 \uc790\uc774\uc5b8\uce20\uac00 \uc0dd\ud65c\ube44 \uc9c0\uc6d0\uacfc \uce58\ub8cc\ube44 \uc9c0\uc6d0\uc744 \ud558\uc9c0 \uc54a\ub294\ub2e4\uba74, \uac00\uc871\ub4e4\uc774 \uc5c4\uccad\ub09c \uc0dd\ud65c\ube44\uc640 \uce58\ub8cc\ube44\ub97c \uac10\ub2f9\ud574\uc57c \ub418\uc5c8\ub2e4. \uadf8\ub7ec\ub098, \ub450\uc0b0 \ubca0\uc5b4\uc2a4\uc758 \ud32c\ub4e4\uc774 \uc131\uae08\uc744 \uc9c0\uc6d0\ud558\uace0, \uacc4\uc18d\ub418\ub294 \uad00\uc2ec\uc73c\ub85c \uc5b4\ub824\uc6b4 \uc77c\uc5d0 \ud76c\ub9dd\uc744 \uac00\uc9c0\uac8c \ub418\uc5c8\ub2e4. \ud558\uc9c0\ub9cc, \uad00\uc2ec\ub3c4 \uc77c\uc2dc\uc801\uc774\uc5c8\uace0, \uc784\uc218\ud601\uc5d0 \uad00\ud55c \uae30\uc0ac\ub294 2002\ub144 4\uc6d4\uc5d0 \uc62c\ub77c\uc628 \uc774\ud6c4, 11\uc6d4\uae4c\uc9c0 \uad00\ub828 \uae30\uc0ac\ub294\ucee4\ub155, \uc784\uc218\ud601\uc5d0 \uad00\ud55c 1\uc904\ub3c4 \ub098\uc624\uc9c0 \uc54a\uc558\ub2e4. \uae30\uc0ac\uac00 \ub098\uc624\uc9c0 \uc54a\ub2e4\uac00, \uc120\uc218\ud611\uc774 \uc784\uc218\ud601 \ub3d5\uae30 \uc790\uc120 \uacbd\ub9e4\ub97c \uc5f4\uba74\uc11c \uae30\uc0ac\uac00 \ub098\uc624\uae30 \uc2dc\uc791\ud588\ub294\ub370, \uc774\ub54c \uc120\uc218\ud611\uc740 \uc784\uc218\ud601\uc758 \uce58\ub8cc\ube44\ub97c \uc704\ud574, \ubc15\ucc2c\ud638\uc758 \uae00\ub7ec\ube0c\uc640 \uc790\ud544 \uc0ac\uc778\uacf5, \ub2f9\uc2dc \uc2dc\uce74\uace0 \ud654\uc774\ud2b8\uc0ad\uc2a4\uc758 \ucf54\uce58\uc600\ub358 \uc774\ub9cc\uc218 \ucf54\uce58\uc758 \ubaa8\uc790, \ud3ec\uc218 \ubbf8\ud2b8, \uc54c\ub809\uc2a4 \ub85c\ub4dc\ub9ac\uac8c\uc2a4, \ub370\ub9ad \uc9c0\ud130 \ub4f1\uc758 \uc790\ud544 \uc0ac\uc778\ubcfc \ub4f1\uc774 \uc784\uc218\ud601\uc758 \uce58\ub8cc\ube44\ub97c \uc704\ud574 \ucd9c\ud488\ub418\uc5c8\ub2e4. \uc774\ud6c4, \uace8\ud504 \uc120\uc218\uc778 \ucd5c\uacbd\uc8fc\ub3c4 \uc784\uc218\ud601 \ub3d5\uae30\uc5d0 \uc790\uc2e0\uc758 \uc18c\uc7a5\uc6a9\ud488\uc744 \uacbd\ub9e4\uc5d0 \uae30\ud0c1\ud558\uba74\uc11c \uc784\uc218\ud601 \ub3d5\uae30\uc5d0 \ub3d9\ucc38\ud558\uc600\uace0, \ud22c\uc218 \ub79c\ub514 \uc874\uc2a8, \uc1fc\ud2b8\ud2b8\ub799 \uc120\uc218 \uae40\ub3d9\uc131\uae4c\uc9c0 \uc784\uc218\ud601 \ub3d5\uae30\uc5d0 \uc790\uc2e0\uc5d0 \uc560\uc7a5\ud488\uc744 \uacbd\ub9e4\uc5d0 \uae30\ud0c1\ud558\uba74\uc11c \ucc38\uac00\ud558\uac8c \ub418\uc5c8\ub2e4. \uacc4\uc18d\ud574\uc11c \uacbd\ub9e4\uc5d0 \ucc38\uac00\ud558\ub824\ub294 \uc2a4\ud3ec\uce20 \uc2a4\ud0c0\ub4e4\uc774 \ub298\uc5b4\ub098\uba74\uc11c, \uc120\uc218\ud611\uc740 \uc784\uc218\ud601 \ub3d5\uae30 \uc790\uc120 \uacbd\ub9e4\uae30\uac04\uc744 \uc5f0\uc7a5\ud558\uc600\uace0, \uc5f0\uc7a5\ud55c \uc774\ud6c4\uc5d0\ub3c4, \ub2f9\uc2dc \ub300\ud55c\ubbfc\uad6d \ucd95\uad6c \ub3cc\ud48d\uc744 \uc77c\uc73c\ucf30\ub358 \uac70\uc2a4 \ud788\ub529\ud06c, \ud64d\uba85\ubcf4, \uc548\uc815\ud658\uae4c\uc9c0 \uc790\uc2e0\uc758 \uc560\uc7a5\ud488\uc744 \uc784\uc218\ud601 \ub3d5\uae30 \uacbd\ub9e4\uc5d0 \uae30\ud0c1\ud558\uc600\ub2e4.\ubc15\ucc2c\ud638\ub294, 4\ucc28 \uacbd\ub9e4\uac00 \uc9c4\ud589\ub418\uace0 \uc788\ub294 \ub3c4\uc911, \ubbf8\uad6d\uc5d0\uc11c \uadc0\uad6d\ud558\uc5ec, \uc784\uc218\ud601\uc758 \ubcd1\uc0c1\uc5d0 \ubc29\ubb38\ud558\uc600\uace0, \uc774\ud6c4, 4\ucc28 \uacbd\ub9e4\uac00 \ub05d\ub09c \ud6c4, 5\ucc28 \uacbd\ub9e4\uac00 \uc5f4\ub9ac\uac8c \ub418\uc5c8\ub294\ub370, \ub2f9\uc2dc \uc77c\ubcf8 \ud504\ub85c \uc57c\uad6c\uc758 \uc120\uc218\uc600\ub358 \uad6c\ub300\uc131\uc774 \uc784\uc218\ud601\uc758 \uce58\ub8cc\ube44\ub97c \uc704\ud574 \ud55c\ud654 \uc774\uae00\uc2a4\ub97c \ud1b5\ud574\uc11c 2\ucc9c\ub9cc \uc6d0\uc744 \uc804\ub2ec\ud558\uc600\ub2e4. \uc784\uc218\ud601\uc758 \uce58\ub8cc\ube44\ub97c \uc704\ud55c 5\ucc28 \uacbd\ub9e4\uac00 \uc9c4\ud589\ub418\ub358 \ub3c4\uc911, \uace0\ub824\ub300\ud559\uad50 \uc57c\uad6c\ubd80 \uad50\uc6b0\ud68c\ub294, \uc784\uc218\ud601\uc744 \uc704\ud574 \uc784\uc218\ud601 \ub3d5\uae30 \uc131\uae08\uc73c\ub85c \uae08\uc77c\ubd09\uc744 \uc804\ub2ec\ud588\uace0, \uacc4\uc18d\ud574\uc11c \uc9c4\ud589\ub418\ub358 5\ucc28 \uacbd\ub9e4\ub294, 4\ucc28 \uacbd\ub9e4\uc5d0 \ube44\ud574 \ud070 \uc778\uae30\ub97c \ub04c\uc9c0 \ubabb\ud558\uace0 \ub05d\ub098\uac8c \ub418\uc5c8\ub2e4. \uc774\ud6c4, 5\ucc28 \uacbd\ub9e4\uac00 \ub05d\ub09c \ub4a4, 6\ucc28 \uacbd\ub9e4\uc5d0\uc11c\ub294 \ud788\uc5b4\ub85c\uc988\uac00 2002\ub144\uc5d0 \uc2e4\uc2dc\ud588\ub358 '\uc0ac\ub791\uc758 \uc131\uae08 \uc2dc\uad6c\ud589\uc0ac'\uc5d0\uc11c \ub9c8\ub828\ub41c 420\ub9cc \uc6d0\uc5d0\uc11c \uadf8 \uc808\ubc18\uc778 210\ub9cc \uc6d0\uc744 \uc784\uc218\ud601\uc758 \uce58\ub8cc\ube44\ub85c \uc9c0\uc6d0\ud588\uace0, \uc120\uc218\ud611\uc774 \ucd1d 6\ucc28\ub840\uc758 \uacbd\ub9e4\uc5d0\uc11c \uc5bb\uc740 \uc784\uc218\ud601 \ub3d5\uae30 \uc131\uae08\uc740 \ucd1d 2\ucc9c 9\ubc31\ub9cc \uc6d0 \uc774\uc5c8\uace0, \uc774 \uc131\uae08\uc740 \uc804\uc561 \uc784\uc218\ud601\uc758 \uac00\uc871\ub4e4\uc5d0\uac8c \uc804\ub2ec\ub418\uc5c8\ub2e4. ", "paragraph_answer": "2002\ub144\uc5d0\ub294, \uc784\uc218\ud601\uc774 \uc4f0\ub7ec\uc838\uc11c \ubcd1\uc0c1\uc5d0 \uc788\ub294\uc9c0 2\ub144\uc9f8\uac00 \ub418\uc5c8\uc9c0\ub9cc, 2001\ub144\uae4c\uc9c0\ub9cc \ud574\ub3c4, \uc4f0\ub824\uc84c\uc744 \ub54c\uc640 \ub2f9\uc2dc \uae30\uc0ac\uc640 \uad00\uc2ec\uc774 \uc904\uc5b4\ub4e4\uc5c8\uc73c\ub098, \ub86f\ub370 \uc790\uc774\uc5b8\uce20\uc758 \ub530\ub73b\ud55c \ubc30\ub824\uc640, \uad00\uc2ec\uc18d\uc5d0 \ubcd1\uc218\ubc1c\uc5d0\ub294 \ud070 \uc5b4\ub824\uc6c0\uc744 \uacaa\uc9c0 \uc54a\uc558\uc5c8\uace0, 2001\ub144 4\uc6d4 18\uc77c\uc5d0\ub294 \uc784\uc218\ud601\uc758 \ub0a0\ub85c \uc9c0\uc815\ud558\uc5ec, \uc544\ub4e4\uc774 \uc2dc\uad6c\ub97c \ud558\uace0, 4\uc6d4 18\uc77c\uc5d0 \uc788\uc5c8\ub358, \uacbd\uae30\uc758 \uc218\uc775\uae08\uc778 4100\ub9cc \uc6d0\uc744 \uc784\uc218\ud601\uc758 \uce58\ub8cc\ube44\ub85c \uc804\ud588\ub2e4. \uadf8\ub7ec\ub098, 2002\ub144\ubd80\ud130\ub294 \ub86f\ub370 \uc790\uc774\uc5b8\uce20\uc5d0\uc11c \uc0dd\ud65c\ube44 \uc9c0\uc6d0\uc5d0 \ub300\ud55c \uc5b4\ub5a0\ud55c \uc5b8\uc9c8\ub3c4 \ubc1b\uc9c0 \ubabb\ud588\uace0, \ubcd1\uc6d0\ube44\uc5d0 \ub300\ud574\uc11c\ub3c4 \uc9c0\uc18d\uc801\uc73c\ub85c \uc9c0\uae09\ud558\uae30 \ud798\ub4e4\ub2e4\ub294 \uc758\uacac\uc744 \uc804\ub2ec\ubc1b\uc558\ub2e4. \ub9cc\uc57d, \ub86f\ub370 \uc790\uc774\uc5b8\uce20\uac00 \uc0dd\ud65c\ube44 \uc9c0\uc6d0\uacfc \uce58\ub8cc\ube44 \uc9c0\uc6d0\uc744 \ud558\uc9c0 \uc54a\ub294\ub2e4\uba74, \uac00\uc871\ub4e4\uc774 \uc5c4\uccad\ub09c \uc0dd\ud65c\ube44\uc640 \uce58\ub8cc\ube44\ub97c \uac10\ub2f9\ud574\uc57c \ub418\uc5c8\ub2e4. \uadf8\ub7ec\ub098, \ub450\uc0b0 \ubca0\uc5b4\uc2a4\uc758 \ud32c\ub4e4\uc774 \uc131\uae08\uc744 \uc9c0\uc6d0\ud558\uace0, \uacc4\uc18d\ub418\ub294 \uad00\uc2ec\uc73c\ub85c \uc5b4\ub824\uc6b4 \uc77c\uc5d0 \ud76c\ub9dd\uc744 \uac00\uc9c0\uac8c \ub418\uc5c8\ub2e4. \ud558\uc9c0\ub9cc, \uad00\uc2ec\ub3c4 \uc77c\uc2dc\uc801\uc774\uc5c8\uace0, \uc784\uc218\ud601\uc5d0 \uad00\ud55c \uae30\uc0ac\ub294 2002\ub144 4\uc6d4\uc5d0 \uc62c\ub77c\uc628 \uc774\ud6c4, 11\uc6d4\uae4c\uc9c0 \uad00\ub828 \uae30\uc0ac\ub294\ucee4\ub155, \uc784\uc218\ud601\uc5d0 \uad00\ud55c 1\uc904\ub3c4 \ub098\uc624\uc9c0 \uc54a\uc558\ub2e4. \uae30\uc0ac\uac00 \ub098\uc624\uc9c0 \uc54a\ub2e4\uac00, \uc120\uc218\ud611\uc774 \uc784\uc218\ud601 \ub3d5\uae30 \uc790\uc120 \uacbd\ub9e4\ub97c \uc5f4\uba74\uc11c \uae30\uc0ac\uac00 \ub098\uc624\uae30 \uc2dc\uc791\ud588\ub294\ub370, \uc774\ub54c \uc120\uc218\ud611\uc740 \uc784\uc218\ud601\uc758 \uce58\ub8cc\ube44\ub97c \uc704\ud574, \ubc15\ucc2c\ud638\uc758 \uae00\ub7ec\ube0c\uc640 \uc790\ud544 \uc0ac\uc778\uacf5, \ub2f9\uc2dc \uc2dc\uce74\uace0 \ud654\uc774\ud2b8\uc0ad\uc2a4\uc758 \ucf54\uce58\uc600\ub358 \uc774\ub9cc\uc218 \ucf54\uce58\uc758 \ubaa8\uc790, \ud3ec\uc218 \ubbf8\ud2b8, \uc54c\ub809\uc2a4 \ub85c\ub4dc\ub9ac\uac8c\uc2a4, \ub370\ub9ad \uc9c0\ud130 \ub4f1\uc758 \uc790\ud544 \uc0ac\uc778\ubcfc \ub4f1\uc774 \uc784\uc218\ud601\uc758 \uce58\ub8cc\ube44\ub97c \uc704\ud574 \ucd9c\ud488\ub418\uc5c8\ub2e4. \uc774\ud6c4, \uace8\ud504 \uc120\uc218\uc778 \ucd5c\uacbd\uc8fc\ub3c4 \uc784\uc218\ud601 \ub3d5\uae30\uc5d0 \uc790\uc2e0\uc758 \uc18c\uc7a5\uc6a9\ud488\uc744 \uacbd\ub9e4\uc5d0 \uae30\ud0c1\ud558\uba74\uc11c \uc784\uc218\ud601 \ub3d5\uae30\uc5d0 \ub3d9\ucc38\ud558\uc600\uace0, \ud22c\uc218 \ub79c\ub514 \uc874\uc2a8, \uc1fc\ud2b8\ud2b8\ub799 \uc120\uc218 \uae40\ub3d9\uc131\uae4c\uc9c0 \uc784\uc218\ud601 \ub3d5\uae30\uc5d0 \uc790\uc2e0\uc5d0 \uc560\uc7a5\ud488\uc744 \uacbd\ub9e4\uc5d0 \uae30\ud0c1\ud558\uba74\uc11c \ucc38\uac00\ud558\uac8c \ub418\uc5c8\ub2e4. \uacc4\uc18d\ud574\uc11c \uacbd\ub9e4\uc5d0 \ucc38\uac00\ud558\ub824\ub294 \uc2a4\ud3ec\uce20 \uc2a4\ud0c0\ub4e4\uc774 \ub298\uc5b4\ub098\uba74\uc11c, \uc120\uc218\ud611\uc740 \uc784\uc218\ud601 \ub3d5\uae30 \uc790\uc120 \uacbd\ub9e4\uae30\uac04\uc744 \uc5f0\uc7a5\ud558\uc600\uace0, \uc5f0\uc7a5\ud55c \uc774\ud6c4\uc5d0\ub3c4, \ub2f9\uc2dc \ub300\ud55c\ubbfc\uad6d \ucd95\uad6c \ub3cc\ud48d\uc744 \uc77c\uc73c\ucf30\ub358 \uac70\uc2a4 \ud788\ub529\ud06c, \ud64d\uba85\ubcf4, \uc548\uc815\ud658\uae4c\uc9c0 \uc790\uc2e0\uc758 \uc560\uc7a5\ud488\uc744 \uc784\uc218\ud601 \ub3d5\uae30 \uacbd\ub9e4\uc5d0 \uae30\ud0c1\ud558\uc600\ub2e4.\ubc15\ucc2c\ud638\ub294, 4\ucc28 \uacbd\ub9e4\uac00 \uc9c4\ud589\ub418\uace0 \uc788\ub294 \ub3c4\uc911, \ubbf8\uad6d\uc5d0\uc11c \uadc0\uad6d\ud558\uc5ec, \uc784\uc218\ud601\uc758 \ubcd1\uc0c1\uc5d0 \ubc29\ubb38\ud558\uc600\uace0, \uc774\ud6c4, 4\ucc28 \uacbd\ub9e4\uac00 \ub05d\ub09c \ud6c4, 5\ucc28 \uacbd\ub9e4\uac00 \uc5f4\ub9ac\uac8c \ub418\uc5c8\ub294\ub370, \ub2f9\uc2dc \uc77c\ubcf8 \ud504\ub85c \uc57c\uad6c\uc758 \uc120\uc218\uc600\ub358 \uad6c\ub300\uc131\uc774 \uc784\uc218\ud601\uc758 \uce58\ub8cc\ube44\ub97c \uc704\ud574 \ud55c\ud654 \uc774\uae00\uc2a4\ub97c \ud1b5\ud574\uc11c 2\ucc9c\ub9cc \uc6d0\uc744 \uc804\ub2ec\ud558\uc600\ub2e4. \uc784\uc218\ud601\uc758 \uce58\ub8cc\ube44\ub97c \uc704\ud55c 5\ucc28 \uacbd\ub9e4\uac00 \uc9c4\ud589\ub418\ub358 \ub3c4\uc911, \uace0\ub824\ub300\ud559\uad50 \uc57c\uad6c\ubd80 \uad50\uc6b0\ud68c\ub294, \uc784\uc218\ud601\uc744 \uc704\ud574 \uc784\uc218\ud601 \ub3d5\uae30 \uc131\uae08\uc73c\ub85c \uae08\uc77c\ubd09\uc744 \uc804\ub2ec\ud588\uace0, \uacc4\uc18d\ud574\uc11c \uc9c4\ud589\ub418\ub358 5\ucc28 \uacbd\ub9e4\ub294, 4\ucc28 \uacbd\ub9e4\uc5d0 \ube44\ud574 \ud070 \uc778\uae30\ub97c \ub04c\uc9c0 \ubabb\ud558\uace0 \ub05d\ub098\uac8c \ub418\uc5c8\ub2e4. \uc774\ud6c4, 5\ucc28 \uacbd\ub9e4\uac00 \ub05d\ub09c \ub4a4, 6\ucc28 \uacbd\ub9e4\uc5d0\uc11c\ub294 \ud788\uc5b4\ub85c\uc988\uac00 2002\ub144\uc5d0 \uc2e4\uc2dc\ud588\ub358 '\uc0ac\ub791\uc758 \uc131\uae08 \uc2dc\uad6c\ud589\uc0ac'\uc5d0\uc11c \ub9c8\ub828\ub41c 420\ub9cc \uc6d0\uc5d0\uc11c \uadf8 \uc808\ubc18\uc778 210\ub9cc \uc6d0\uc744 \uc784\uc218\ud601\uc758 \uce58\ub8cc\ube44\ub85c \uc9c0\uc6d0\ud588\uace0, \uc120\uc218\ud611\uc774 \ucd1d 6\ucc28\ub840\uc758 \uacbd\ub9e4\uc5d0\uc11c \uc5bb\uc740 \uc784\uc218\ud601 \ub3d5\uae30 \uc131\uae08\uc740 \ucd1d 2\ucc9c 9\ubc31\ub9cc \uc6d0 \uc774\uc5c8\uace0, \uc774 \uc131\uae08\uc740 \uc804\uc561 \uc784\uc218\ud601\uc758 \uac00\uc871\ub4e4\uc5d0\uac8c \uc804\ub2ec\ub418\uc5c8\ub2e4.", "sentence_answer": "\ucd1d 2\ucc9c 9\ubc31\ub9cc \uc6d0 \uc774\uc5c8\uace0, \uc774 \uc131\uae08\uc740 \uc804\uc561 \uc784\uc218\ud601\uc758 \uac00\uc871\ub4e4\uc5d0\uac8c \uc804\ub2ec\ub418\uc5c8\ub2e4.", "paragraph_id": "6548835-4-0", "paragraph_question": "question: \uc120\uc218\ud611\uc774 \uc9c4\ud589\ud588\ub358 \ucd1d 6\ucc28\ub840\uc758 \uacbd\ub9e4\uc5d0\uc11c \uc5bb\uc740 \uc784\uc218\ud601 \ub3d5\uae30 \uc131\uae08\uc758 \uae08\uc561\uc740?, context: 2002\ub144\uc5d0\ub294, \uc784\uc218\ud601\uc774 \uc4f0\ub7ec\uc838\uc11c \ubcd1\uc0c1\uc5d0 \uc788\ub294\uc9c0 2\ub144\uc9f8\uac00 \ub418\uc5c8\uc9c0\ub9cc, 2001\ub144\uae4c\uc9c0\ub9cc \ud574\ub3c4, \uc4f0\ub824\uc84c\uc744 \ub54c\uc640 \ub2f9\uc2dc \uae30\uc0ac\uc640 \uad00\uc2ec\uc774 \uc904\uc5b4\ub4e4\uc5c8\uc73c\ub098, \ub86f\ub370 \uc790\uc774\uc5b8\uce20\uc758 \ub530\ub73b\ud55c \ubc30\ub824\uc640, \uad00\uc2ec\uc18d\uc5d0 \ubcd1\uc218\ubc1c\uc5d0\ub294 \ud070 \uc5b4\ub824\uc6c0\uc744 \uacaa\uc9c0 \uc54a\uc558\uc5c8\uace0, 2001\ub144 4\uc6d4 18\uc77c\uc5d0\ub294 \uc784\uc218\ud601\uc758 \ub0a0\ub85c \uc9c0\uc815\ud558\uc5ec, \uc544\ub4e4\uc774 \uc2dc\uad6c\ub97c \ud558\uace0, 4\uc6d4 18\uc77c\uc5d0 \uc788\uc5c8\ub358, \uacbd\uae30\uc758 \uc218\uc775\uae08\uc778 4100\ub9cc \uc6d0\uc744 \uc784\uc218\ud601\uc758 \uce58\ub8cc\ube44\ub85c \uc804\ud588\ub2e4. \uadf8\ub7ec\ub098, 2002\ub144\ubd80\ud130\ub294 \ub86f\ub370 \uc790\uc774\uc5b8\uce20\uc5d0\uc11c \uc0dd\ud65c\ube44 \uc9c0\uc6d0\uc5d0 \ub300\ud55c \uc5b4\ub5a0\ud55c \uc5b8\uc9c8\ub3c4 \ubc1b\uc9c0 \ubabb\ud588\uace0, \ubcd1\uc6d0\ube44\uc5d0 \ub300\ud574\uc11c\ub3c4 \uc9c0\uc18d\uc801\uc73c\ub85c \uc9c0\uae09\ud558\uae30 \ud798\ub4e4\ub2e4\ub294 \uc758\uacac\uc744 \uc804\ub2ec\ubc1b\uc558\ub2e4. \ub9cc\uc57d, \ub86f\ub370 \uc790\uc774\uc5b8\uce20\uac00 \uc0dd\ud65c\ube44 \uc9c0\uc6d0\uacfc \uce58\ub8cc\ube44 \uc9c0\uc6d0\uc744 \ud558\uc9c0 \uc54a\ub294\ub2e4\uba74, \uac00\uc871\ub4e4\uc774 \uc5c4\uccad\ub09c \uc0dd\ud65c\ube44\uc640 \uce58\ub8cc\ube44\ub97c \uac10\ub2f9\ud574\uc57c \ub418\uc5c8\ub2e4. \uadf8\ub7ec\ub098, \ub450\uc0b0 \ubca0\uc5b4\uc2a4\uc758 \ud32c\ub4e4\uc774 \uc131\uae08\uc744 \uc9c0\uc6d0\ud558\uace0, \uacc4\uc18d\ub418\ub294 \uad00\uc2ec\uc73c\ub85c \uc5b4\ub824\uc6b4 \uc77c\uc5d0 \ud76c\ub9dd\uc744 \uac00\uc9c0\uac8c \ub418\uc5c8\ub2e4. \ud558\uc9c0\ub9cc, \uad00\uc2ec\ub3c4 \uc77c\uc2dc\uc801\uc774\uc5c8\uace0, \uc784\uc218\ud601\uc5d0 \uad00\ud55c \uae30\uc0ac\ub294 2002\ub144 4\uc6d4\uc5d0 \uc62c\ub77c\uc628 \uc774\ud6c4, 11\uc6d4\uae4c\uc9c0 \uad00\ub828 \uae30\uc0ac\ub294\ucee4\ub155, \uc784\uc218\ud601\uc5d0 \uad00\ud55c 1\uc904\ub3c4 \ub098\uc624\uc9c0 \uc54a\uc558\ub2e4. \uae30\uc0ac\uac00 \ub098\uc624\uc9c0 \uc54a\ub2e4\uac00, \uc120\uc218\ud611\uc774 \uc784\uc218\ud601 \ub3d5\uae30 \uc790\uc120 \uacbd\ub9e4\ub97c \uc5f4\uba74\uc11c \uae30\uc0ac\uac00 \ub098\uc624\uae30 \uc2dc\uc791\ud588\ub294\ub370, \uc774\ub54c \uc120\uc218\ud611\uc740 \uc784\uc218\ud601\uc758 \uce58\ub8cc\ube44\ub97c \uc704\ud574, \ubc15\ucc2c\ud638\uc758 \uae00\ub7ec\ube0c\uc640 \uc790\ud544 \uc0ac\uc778\uacf5, \ub2f9\uc2dc \uc2dc\uce74\uace0 \ud654\uc774\ud2b8\uc0ad\uc2a4\uc758 \ucf54\uce58\uc600\ub358 \uc774\ub9cc\uc218 \ucf54\uce58\uc758 \ubaa8\uc790, \ud3ec\uc218 \ubbf8\ud2b8, \uc54c\ub809\uc2a4 \ub85c\ub4dc\ub9ac\uac8c\uc2a4, \ub370\ub9ad \uc9c0\ud130 \ub4f1\uc758 \uc790\ud544 \uc0ac\uc778\ubcfc \ub4f1\uc774 \uc784\uc218\ud601\uc758 \uce58\ub8cc\ube44\ub97c \uc704\ud574 \ucd9c\ud488\ub418\uc5c8\ub2e4. \uc774\ud6c4, \uace8\ud504 \uc120\uc218\uc778 \ucd5c\uacbd\uc8fc\ub3c4 \uc784\uc218\ud601 \ub3d5\uae30\uc5d0 \uc790\uc2e0\uc758 \uc18c\uc7a5\uc6a9\ud488\uc744 \uacbd\ub9e4\uc5d0 \uae30\ud0c1\ud558\uba74\uc11c \uc784\uc218\ud601 \ub3d5\uae30\uc5d0 \ub3d9\ucc38\ud558\uc600\uace0, \ud22c\uc218 \ub79c\ub514 \uc874\uc2a8, \uc1fc\ud2b8\ud2b8\ub799 \uc120\uc218 \uae40\ub3d9\uc131\uae4c\uc9c0 \uc784\uc218\ud601 \ub3d5\uae30\uc5d0 \uc790\uc2e0\uc5d0 \uc560\uc7a5\ud488\uc744 \uacbd\ub9e4\uc5d0 \uae30\ud0c1\ud558\uba74\uc11c \ucc38\uac00\ud558\uac8c \ub418\uc5c8\ub2e4. \uacc4\uc18d\ud574\uc11c \uacbd\ub9e4\uc5d0 \ucc38\uac00\ud558\ub824\ub294 \uc2a4\ud3ec\uce20 \uc2a4\ud0c0\ub4e4\uc774 \ub298\uc5b4\ub098\uba74\uc11c, \uc120\uc218\ud611\uc740 \uc784\uc218\ud601 \ub3d5\uae30 \uc790\uc120 \uacbd\ub9e4\uae30\uac04\uc744 \uc5f0\uc7a5\ud558\uc600\uace0, \uc5f0\uc7a5\ud55c \uc774\ud6c4\uc5d0\ub3c4, \ub2f9\uc2dc \ub300\ud55c\ubbfc\uad6d \ucd95\uad6c \ub3cc\ud48d\uc744 \uc77c\uc73c\ucf30\ub358 \uac70\uc2a4 \ud788\ub529\ud06c, \ud64d\uba85\ubcf4, \uc548\uc815\ud658\uae4c\uc9c0 \uc790\uc2e0\uc758 \uc560\uc7a5\ud488\uc744 \uc784\uc218\ud601 \ub3d5\uae30 \uacbd\ub9e4\uc5d0 \uae30\ud0c1\ud558\uc600\ub2e4.\ubc15\ucc2c\ud638\ub294, 4\ucc28 \uacbd\ub9e4\uac00 \uc9c4\ud589\ub418\uace0 \uc788\ub294 \ub3c4\uc911, \ubbf8\uad6d\uc5d0\uc11c \uadc0\uad6d\ud558\uc5ec, \uc784\uc218\ud601\uc758 \ubcd1\uc0c1\uc5d0 \ubc29\ubb38\ud558\uc600\uace0, \uc774\ud6c4, 4\ucc28 \uacbd\ub9e4\uac00 \ub05d\ub09c \ud6c4, 5\ucc28 \uacbd\ub9e4\uac00 \uc5f4\ub9ac\uac8c \ub418\uc5c8\ub294\ub370, \ub2f9\uc2dc \uc77c\ubcf8 \ud504\ub85c \uc57c\uad6c\uc758 \uc120\uc218\uc600\ub358 \uad6c\ub300\uc131\uc774 \uc784\uc218\ud601\uc758 \uce58\ub8cc\ube44\ub97c \uc704\ud574 \ud55c\ud654 \uc774\uae00\uc2a4\ub97c \ud1b5\ud574\uc11c 2\ucc9c\ub9cc \uc6d0\uc744 \uc804\ub2ec\ud558\uc600\ub2e4. \uc784\uc218\ud601\uc758 \uce58\ub8cc\ube44\ub97c \uc704\ud55c 5\ucc28 \uacbd\ub9e4\uac00 \uc9c4\ud589\ub418\ub358 \ub3c4\uc911, \uace0\ub824\ub300\ud559\uad50 \uc57c\uad6c\ubd80 \uad50\uc6b0\ud68c\ub294, \uc784\uc218\ud601\uc744 \uc704\ud574 \uc784\uc218\ud601 \ub3d5\uae30 \uc131\uae08\uc73c\ub85c \uae08\uc77c\ubd09\uc744 \uc804\ub2ec\ud588\uace0, \uacc4\uc18d\ud574\uc11c \uc9c4\ud589\ub418\ub358 5\ucc28 \uacbd\ub9e4\ub294, 4\ucc28 \uacbd\ub9e4\uc5d0 \ube44\ud574 \ud070 \uc778\uae30\ub97c \ub04c\uc9c0 \ubabb\ud558\uace0 \ub05d\ub098\uac8c \ub418\uc5c8\ub2e4. \uc774\ud6c4, 5\ucc28 \uacbd\ub9e4\uac00 \ub05d\ub09c \ub4a4, 6\ucc28 \uacbd\ub9e4\uc5d0\uc11c\ub294 \ud788\uc5b4\ub85c\uc988\uac00 2002\ub144\uc5d0 \uc2e4\uc2dc\ud588\ub358 '\uc0ac\ub791\uc758 \uc131\uae08 \uc2dc\uad6c\ud589\uc0ac'\uc5d0\uc11c \ub9c8\ub828\ub41c 420\ub9cc \uc6d0\uc5d0\uc11c \uadf8 \uc808\ubc18\uc778 210\ub9cc \uc6d0\uc744 \uc784\uc218\ud601\uc758 \uce58\ub8cc\ube44\ub85c \uc9c0\uc6d0\ud588\uace0, \uc120\uc218\ud611\uc774 \ucd1d 6\ucc28\ub840\uc758 \uacbd\ub9e4\uc5d0\uc11c \uc5bb\uc740 \uc784\uc218\ud601 \ub3d5\uae30 \uc131\uae08\uc740 \ucd1d 2\ucc9c 9\ubc31\ub9cc \uc6d0\uc774\uc5c8\uace0, \uc774 \uc131\uae08\uc740 \uc804\uc561 \uc784\uc218\ud601\uc758 \uac00\uc871\ub4e4\uc5d0\uac8c \uc804\ub2ec\ub418\uc5c8\ub2e4."} {"question": "1941\ub144\uc5d0 \uc124\ub9bd\ub41c \uacfc\ud559 \uc5f0\uad6c \uac1c\ubc1c \uc0ac\ubb34\uc18c\uc758 \uc18c\uc7a5\uc740?", "paragraph": "\ube0c\ub9ac\uadf8\ub294 \uc6b0\ub77c\ub284, \ud2b9\ud788 \uc6b0\ub77c\ub284-235\ub97c \ucd94\ucd9c\ud558\uace0 \ud50c\ub8e8\ud1a0\ub284\uc744 \ubc1c\uacac\ud558\uc5ec\uc57c \ud558\uba70, \uc774\ub97c \uc704\ud574 167,000 \ub2ec\ub7ec\uc758 \uc608\uc0b0\uc73c\ub85c \uad6d\uac00 \ubc29\uc704 \uc5f0\uad6c \uc704\uc6d0\ud68c(\uc601\uc5b4: National Defense Research Committee, NDRC)\ub97c \uc124\ub9bd\ud558\uc5ec\uc57c \ud55c\ub2e4\uace0 \uc694\uccad\ud558\uc600\ub2e4. 1941\ub144 6\uc6d4 28\uc77c, \ub8e8\uc988\ubca8\ud2b8\ub294 \ud589\uc815\uba85\ub839 \uc81c8807\ud638\uc5d0 \uc11c\uba85\ud558\uc600\uace0, \uadf8\uc5d0 \ub530\ub77c \uacfc\ud559 \uc5f0\uad6c \uac1c\ubc1c \uc0ac\ubb34\uc18c(\uc601\uc5b4: Office of Scientific Research and Development, OSRD)\uac00 \ub9cc\ub4e4\uc5b4\uc84c\ub2e4. \uc0ac\ubb34\uc18c\uc758 \uc18c\uc7a5\uc740 \ubc84\ub2c8\ubc14 \ubd80\uc2dc\uac00 \ub9e1\uc558\ub2e4. \uc0ac\ubb34\uc18c\ub294 \ucd94\uac00\uc801\uc73c\ub85c \ubc29\ub300\ud55c \uacf5\ud559 \ud504\ub85c\uc81d\ud2b8\ub97c \uc218\ud589\ud558\uac8c \ub418\uc5c8\uace0 \uad6d\uac00 \ubc29\uc704 \uc5f0\uad6c \uc704\uc6d0\ud68c\ub294 \uacfc\ud559 \uc5f0\uad6c \uac1c\ubc1c \uc0ac\ubb34\uc18c\uc758 \ubd80\uc11c\ub85c \ud761\uc218\ub418\uc5b4 S-1 \uc6b0\ub77c\ub284 \uc704\uc6d0\ud68c\uac00 \ub418\uc5c8\ub2e4. \uc5bc\ub9c8 \uc9c0\ub098\uc9c0 \uc54a\uc544, \uc774 \uc704\uc6d0\ud68c\ub294 \ubcf4\uc548\uc744 \uc704\ud574 \uc6b0\ub77c\ub284\uc774\ub77c\ub294 \ub9d0\uc744 \ube7c\uace0 S-1 \uc704\uc6d0\ud68c\ub77c \ubd88\ub9ac\uac8c \ub418\uc5c8\ub2e4.", "answer": "\ubc84\ub2c8\ubc14 \ubd80\uc2dc", "sentence": "\uc0ac\ubb34\uc18c\uc758 \uc18c\uc7a5\uc740 \ubc84\ub2c8\ubc14 \ubd80\uc2dc \uac00 \ub9e1\uc558\ub2e4.", "paragraph_sentence": "\ube0c\ub9ac\uadf8\ub294 \uc6b0\ub77c\ub284, \ud2b9\ud788 \uc6b0\ub77c\ub284-235\ub97c \ucd94\ucd9c\ud558\uace0 \ud50c\ub8e8\ud1a0\ub284\uc744 \ubc1c\uacac\ud558\uc5ec\uc57c \ud558\uba70, \uc774\ub97c \uc704\ud574 167,000 \ub2ec\ub7ec\uc758 \uc608\uc0b0\uc73c\ub85c \uad6d\uac00 \ubc29\uc704 \uc5f0\uad6c \uc704\uc6d0\ud68c(\uc601\uc5b4: National Defense Research Committee, NDRC)\ub97c \uc124\ub9bd\ud558\uc5ec\uc57c \ud55c\ub2e4\uace0 \uc694\uccad\ud558\uc600\ub2e4. 1941\ub144 6\uc6d4 28\uc77c, \ub8e8\uc988\ubca8\ud2b8\ub294 \ud589\uc815\uba85\ub839 \uc81c8807\ud638\uc5d0 \uc11c\uba85\ud558\uc600\uace0, \uadf8\uc5d0 \ub530\ub77c \uacfc\ud559 \uc5f0\uad6c \uac1c\ubc1c \uc0ac\ubb34\uc18c(\uc601\uc5b4: Office of Scientific Research and Development, OSRD)\uac00 \ub9cc\ub4e4\uc5b4\uc84c\ub2e4. \uc0ac\ubb34\uc18c\uc758 \uc18c\uc7a5\uc740 \ubc84\ub2c8\ubc14 \ubd80\uc2dc \uac00 \ub9e1\uc558\ub2e4. \uc0ac\ubb34\uc18c\ub294 \ucd94\uac00\uc801\uc73c\ub85c \ubc29\ub300\ud55c \uacf5\ud559 \ud504\ub85c\uc81d\ud2b8\ub97c \uc218\ud589\ud558\uac8c \ub418\uc5c8\uace0 \uad6d\uac00 \ubc29\uc704 \uc5f0\uad6c \uc704\uc6d0\ud68c\ub294 \uacfc\ud559 \uc5f0\uad6c \uac1c\ubc1c \uc0ac\ubb34\uc18c\uc758 \ubd80\uc11c\ub85c \ud761\uc218\ub418\uc5b4 S-1 \uc6b0\ub77c\ub284 \uc704\uc6d0\ud68c\uac00 \ub418\uc5c8\ub2e4. \uc5bc\ub9c8 \uc9c0\ub098\uc9c0 \uc54a\uc544, \uc774 \uc704\uc6d0\ud68c\ub294 \ubcf4\uc548\uc744 \uc704\ud574 \uc6b0\ub77c\ub284\uc774\ub77c\ub294 \ub9d0\uc744 \ube7c\uace0 S-1 \uc704\uc6d0\ud68c\ub77c \ubd88\ub9ac\uac8c \ub418\uc5c8\ub2e4.", "paragraph_answer": "\ube0c\ub9ac\uadf8\ub294 \uc6b0\ub77c\ub284, \ud2b9\ud788 \uc6b0\ub77c\ub284-235\ub97c \ucd94\ucd9c\ud558\uace0 \ud50c\ub8e8\ud1a0\ub284\uc744 \ubc1c\uacac\ud558\uc5ec\uc57c \ud558\uba70, \uc774\ub97c \uc704\ud574 167,000 \ub2ec\ub7ec\uc758 \uc608\uc0b0\uc73c\ub85c \uad6d\uac00 \ubc29\uc704 \uc5f0\uad6c \uc704\uc6d0\ud68c(\uc601\uc5b4: National Defense Research Committee, NDRC)\ub97c \uc124\ub9bd\ud558\uc5ec\uc57c \ud55c\ub2e4\uace0 \uc694\uccad\ud558\uc600\ub2e4. 1941\ub144 6\uc6d4 28\uc77c, \ub8e8\uc988\ubca8\ud2b8\ub294 \ud589\uc815\uba85\ub839 \uc81c8807\ud638\uc5d0 \uc11c\uba85\ud558\uc600\uace0, \uadf8\uc5d0 \ub530\ub77c \uacfc\ud559 \uc5f0\uad6c \uac1c\ubc1c \uc0ac\ubb34\uc18c(\uc601\uc5b4: Office of Scientific Research and Development, OSRD)\uac00 \ub9cc\ub4e4\uc5b4\uc84c\ub2e4. \uc0ac\ubb34\uc18c\uc758 \uc18c\uc7a5\uc740 \ubc84\ub2c8\ubc14 \ubd80\uc2dc \uac00 \ub9e1\uc558\ub2e4. \uc0ac\ubb34\uc18c\ub294 \ucd94\uac00\uc801\uc73c\ub85c \ubc29\ub300\ud55c \uacf5\ud559 \ud504\ub85c\uc81d\ud2b8\ub97c \uc218\ud589\ud558\uac8c \ub418\uc5c8\uace0 \uad6d\uac00 \ubc29\uc704 \uc5f0\uad6c \uc704\uc6d0\ud68c\ub294 \uacfc\ud559 \uc5f0\uad6c \uac1c\ubc1c \uc0ac\ubb34\uc18c\uc758 \ubd80\uc11c\ub85c \ud761\uc218\ub418\uc5b4 S-1 \uc6b0\ub77c\ub284 \uc704\uc6d0\ud68c\uac00 \ub418\uc5c8\ub2e4. \uc5bc\ub9c8 \uc9c0\ub098\uc9c0 \uc54a\uc544, \uc774 \uc704\uc6d0\ud68c\ub294 \ubcf4\uc548\uc744 \uc704\ud574 \uc6b0\ub77c\ub284\uc774\ub77c\ub294 \ub9d0\uc744 \ube7c\uace0 S-1 \uc704\uc6d0\ud68c\ub77c \ubd88\ub9ac\uac8c \ub418\uc5c8\ub2e4.", "sentence_answer": "\uc0ac\ubb34\uc18c\uc758 \uc18c\uc7a5\uc740 \ubc84\ub2c8\ubc14 \ubd80\uc2dc \uac00 \ub9e1\uc558\ub2e4.", "paragraph_id": "6502200-2-1", "paragraph_question": "question: 1941\ub144\uc5d0 \uc124\ub9bd\ub41c \uacfc\ud559 \uc5f0\uad6c \uac1c\ubc1c \uc0ac\ubb34\uc18c\uc758 \uc18c\uc7a5\uc740?, context: \ube0c\ub9ac\uadf8\ub294 \uc6b0\ub77c\ub284, \ud2b9\ud788 \uc6b0\ub77c\ub284-235\ub97c \ucd94\ucd9c\ud558\uace0 \ud50c\ub8e8\ud1a0\ub284\uc744 \ubc1c\uacac\ud558\uc5ec\uc57c \ud558\uba70, \uc774\ub97c \uc704\ud574 167,000 \ub2ec\ub7ec\uc758 \uc608\uc0b0\uc73c\ub85c \uad6d\uac00 \ubc29\uc704 \uc5f0\uad6c \uc704\uc6d0\ud68c(\uc601\uc5b4: National Defense Research Committee, NDRC)\ub97c \uc124\ub9bd\ud558\uc5ec\uc57c \ud55c\ub2e4\uace0 \uc694\uccad\ud558\uc600\ub2e4. 1941\ub144 6\uc6d4 28\uc77c, \ub8e8\uc988\ubca8\ud2b8\ub294 \ud589\uc815\uba85\ub839 \uc81c8807\ud638\uc5d0 \uc11c\uba85\ud558\uc600\uace0, \uadf8\uc5d0 \ub530\ub77c \uacfc\ud559 \uc5f0\uad6c \uac1c\ubc1c \uc0ac\ubb34\uc18c(\uc601\uc5b4: Office of Scientific Research and Development, OSRD)\uac00 \ub9cc\ub4e4\uc5b4\uc84c\ub2e4. \uc0ac\ubb34\uc18c\uc758 \uc18c\uc7a5\uc740 \ubc84\ub2c8\ubc14 \ubd80\uc2dc\uac00 \ub9e1\uc558\ub2e4. \uc0ac\ubb34\uc18c\ub294 \ucd94\uac00\uc801\uc73c\ub85c \ubc29\ub300\ud55c \uacf5\ud559 \ud504\ub85c\uc81d\ud2b8\ub97c \uc218\ud589\ud558\uac8c \ub418\uc5c8\uace0 \uad6d\uac00 \ubc29\uc704 \uc5f0\uad6c \uc704\uc6d0\ud68c\ub294 \uacfc\ud559 \uc5f0\uad6c \uac1c\ubc1c \uc0ac\ubb34\uc18c\uc758 \ubd80\uc11c\ub85c \ud761\uc218\ub418\uc5b4 S-1 \uc6b0\ub77c\ub284 \uc704\uc6d0\ud68c\uac00 \ub418\uc5c8\ub2e4. \uc5bc\ub9c8 \uc9c0\ub098\uc9c0 \uc54a\uc544, \uc774 \uc704\uc6d0\ud68c\ub294 \ubcf4\uc548\uc744 \uc704\ud574 \uc6b0\ub77c\ub284\uc774\ub77c\ub294 \ub9d0\uc744 \ube7c\uace0 S-1 \uc704\uc6d0\ud68c\ub77c \ubd88\ub9ac\uac8c \ub418\uc5c8\ub2e4."} {"question": "\uc9c0\uc815\ud559\uad50 \uc2b9\uaca9\uc744 \uc704\ud574 \uc2e0\ud765\ud559\uad50\uc758 \uc120\uad50\uc0ac\ub4e4\uc774 \uae30\uae08 \ub9c8\ub828\uc744 \uc704\ud574 \uac74\ub108\uac04 \ub098\ub77c\ub294 \ubb34\uc5c7\uc778\uac00?", "paragraph": "\uc804\uc8fc\uc2e0\ud765\uace0\ub4f1\ud559\uad50 \uad50\ub0b4(\uc18c\uc7ac\uc9c0: \uc804\ub77c\ubd81\ub3c4 \uc804\uc8fc\uc2dc \uc644\uc0b0\uad6c \uc11c\uc6d0\ub85c 399)\uc5d0 \uc788\uc73c\uba70 \ub4f1\ub85d\ubb38\ud654\uc7ac \uc81c172\ud638 \ub85c \uc9c0\uc815\ub418\uc5c8\ub2e4. \uc77c\uc81c\uac15\uc810\uae30\uc778 1923\ub144\uc5d0 \uc77c\uc815 \uc218\uc900\uc758 \uad50\uc0ac \ud655\ubcf4\uc640 \uc2dc\uc124 \ubc0f \uad50\uc721\uacfc\uc815\uc774 \uac16\ucd94\uc5b4\uc9c4 \uc0ac\ub9bd\ud559\uad50\uc5d0\uac8c \uace0\ub4f1\ubcf4\ud1b5\ud559\uad50\uc640 \uac19\uc740 \uc790\uaca9\uc744 \uc778\uc815\ud558\ub294 \uc18c\uc704 \uc9c0\uc815\ud559\uad50\ub77c\ub294 \uc81c\ub3c4\uac00 \ub9cc\ub4e4\uc5b4\uc84c\ub2e4. \uc2e0\ud765\ud559\uad50\ub294 \uc9c0\uc815\ud559\uad50 \uc2b9\uaca9\uc744 \uc704\ud558\uc5ec \uad50\uc0ac \uc2e0\ucd95\uc744 \ucd94\uc9c4\ud558\uc600\uace0, \uc120\uad50\uc0ac\ub4e4\uc740 \ubbf8\uad6d\uc73c\ub85c \uac74\ub108\uac00 \uae30\uae08\uc744 \ub9c8\ub828\ud558\uae30 \uc704\ud55c \ubaa8\uae08\ud65c\ub3d9\uc744 \ubc8c\uc600\ub2e4. \ub9c8\uce68\ub0b4 \ub178\uc2a4\uce90\ub864\ub77c\uc774\ub098 \uc8fc\uc758 \ub9ac\ucc28\ub4dc\uc2a8 \uc5ec\uc0ac \uac00 9,000\ub2ec\ub7ec\uc758 \uac74\ucd95\ube44\ub97c \uc9c0\uc6d0\ud558\uba74\uc11c 1927\ub144 10\uc6d4 \uc2e0\ud765\ud559\uad50 \ubcf8\uad00\uc744 \uc2e0\ucd95\ud558\uac8c \ub418\uc5c8\uc73c\uba70, \uc774\ub97c \uae30\ub150\ud558\uc5ec \uba85\uce6d\uc744 \ub9ac\ucc28\ub4dc\uc2a8 \ud640(Richardson Hall)\ub85c \uc815\ud558\uc600\ub2e4. \ubcf8\uad00 \uac74\ubb3c\uc758 \uc124\uacc4\ub294 \uac74\ucd95\uc744 \uc804\uacf5\ud55c \uc11c\ub85c\ub4dd \uc120\uad50\uc0ac\uac00 \ub2f4\ub2f9\ud558\uc600\uc73c\uba70, \uad11\uc8fc\uc218\ud53c\uc544\uc5ec\uc790\uace0\ub4f1\ud559\uad50\uc758 \uc708\uc2a4\ube0c\ub85c\uc6b0 \ud640(Winsborough Hall)\uacfc \uad6c\uc870\uac00 \uac19\uc740 \uc30d\ub465\uc774 \uac74\ubb3c\ub85c \uac74\ucd95\ub418\uc5c8\ub2e4.", "answer": "\ubbf8\uad6d", "sentence": "\uc120\uad50\uc0ac\ub4e4\uc740 \ubbf8\uad6d \uc73c\ub85c \uac74\ub108\uac00 \uae30\uae08\uc744 \ub9c8\ub828\ud558\uae30 \uc704\ud55c \ubaa8\uae08\ud65c\ub3d9\uc744 \ubc8c\uc600\ub2e4.", "paragraph_sentence": "\uc804\uc8fc\uc2e0\ud765\uace0\ub4f1\ud559\uad50 \uad50\ub0b4(\uc18c\uc7ac\uc9c0: \uc804\ub77c\ubd81\ub3c4 \uc804\uc8fc\uc2dc \uc644\uc0b0\uad6c \uc11c\uc6d0\ub85c 399)\uc5d0 \uc788\uc73c\uba70 \ub4f1\ub85d\ubb38\ud654\uc7ac \uc81c172\ud638 \ub85c \uc9c0\uc815\ub418\uc5c8\ub2e4. \uc77c\uc81c\uac15\uc810\uae30\uc778 1923\ub144\uc5d0 \uc77c\uc815 \uc218\uc900\uc758 \uad50\uc0ac \ud655\ubcf4\uc640 \uc2dc\uc124 \ubc0f \uad50\uc721\uacfc\uc815\uc774 \uac16\ucd94\uc5b4\uc9c4 \uc0ac\ub9bd\ud559\uad50\uc5d0\uac8c \uace0\ub4f1\ubcf4\ud1b5\ud559\uad50\uc640 \uac19\uc740 \uc790\uaca9\uc744 \uc778\uc815\ud558\ub294 \uc18c\uc704 \uc9c0\uc815\ud559\uad50\ub77c\ub294 \uc81c\ub3c4\uac00 \ub9cc\ub4e4\uc5b4\uc84c\ub2e4. \uc2e0\ud765\ud559\uad50\ub294 \uc9c0\uc815\ud559\uad50 \uc2b9\uaca9\uc744 \uc704\ud558\uc5ec \uad50\uc0ac \uc2e0\ucd95\uc744 \ucd94\uc9c4\ud558\uc600\uace0, \uc120\uad50\uc0ac\ub4e4\uc740 \ubbf8\uad6d \uc73c\ub85c \uac74\ub108\uac00 \uae30\uae08\uc744 \ub9c8\ub828\ud558\uae30 \uc704\ud55c \ubaa8\uae08\ud65c\ub3d9\uc744 \ubc8c\uc600\ub2e4. \ub9c8\uce68\ub0b4 \ub178\uc2a4\uce90\ub864\ub77c\uc774\ub098 \uc8fc\uc758 \ub9ac\ucc28\ub4dc\uc2a8 \uc5ec\uc0ac \uac00 9,000\ub2ec\ub7ec\uc758 \uac74\ucd95\ube44\ub97c \uc9c0\uc6d0\ud558\uba74\uc11c 1927\ub144 10\uc6d4 \uc2e0\ud765\ud559\uad50 \ubcf8\uad00\uc744 \uc2e0\ucd95\ud558\uac8c \ub418\uc5c8\uc73c\uba70, \uc774\ub97c \uae30\ub150\ud558\uc5ec \uba85\uce6d\uc744 \ub9ac\ucc28\ub4dc\uc2a8 \ud640(Richardson Hall)\ub85c \uc815\ud558\uc600\ub2e4. \ubcf8\uad00 \uac74\ubb3c\uc758 \uc124\uacc4\ub294 \uac74\ucd95\uc744 \uc804\uacf5\ud55c \uc11c\ub85c\ub4dd \uc120\uad50\uc0ac\uac00 \ub2f4\ub2f9\ud558\uc600\uc73c\uba70, \uad11\uc8fc\uc218\ud53c\uc544\uc5ec\uc790\uace0\ub4f1\ud559\uad50\uc758 \uc708\uc2a4\ube0c\ub85c\uc6b0 \ud640(Winsborough Hall)\uacfc \uad6c\uc870\uac00 \uac19\uc740 \uc30d\ub465\uc774 \uac74\ubb3c\ub85c \uac74\ucd95\ub418\uc5c8\ub2e4.", "paragraph_answer": "\uc804\uc8fc\uc2e0\ud765\uace0\ub4f1\ud559\uad50 \uad50\ub0b4(\uc18c\uc7ac\uc9c0: \uc804\ub77c\ubd81\ub3c4 \uc804\uc8fc\uc2dc \uc644\uc0b0\uad6c \uc11c\uc6d0\ub85c 399)\uc5d0 \uc788\uc73c\uba70 \ub4f1\ub85d\ubb38\ud654\uc7ac \uc81c172\ud638 \ub85c \uc9c0\uc815\ub418\uc5c8\ub2e4. \uc77c\uc81c\uac15\uc810\uae30\uc778 1923\ub144\uc5d0 \uc77c\uc815 \uc218\uc900\uc758 \uad50\uc0ac \ud655\ubcf4\uc640 \uc2dc\uc124 \ubc0f \uad50\uc721\uacfc\uc815\uc774 \uac16\ucd94\uc5b4\uc9c4 \uc0ac\ub9bd\ud559\uad50\uc5d0\uac8c \uace0\ub4f1\ubcf4\ud1b5\ud559\uad50\uc640 \uac19\uc740 \uc790\uaca9\uc744 \uc778\uc815\ud558\ub294 \uc18c\uc704 \uc9c0\uc815\ud559\uad50\ub77c\ub294 \uc81c\ub3c4\uac00 \ub9cc\ub4e4\uc5b4\uc84c\ub2e4. \uc2e0\ud765\ud559\uad50\ub294 \uc9c0\uc815\ud559\uad50 \uc2b9\uaca9\uc744 \uc704\ud558\uc5ec \uad50\uc0ac \uc2e0\ucd95\uc744 \ucd94\uc9c4\ud558\uc600\uace0, \uc120\uad50\uc0ac\ub4e4\uc740 \ubbf8\uad6d \uc73c\ub85c \uac74\ub108\uac00 \uae30\uae08\uc744 \ub9c8\ub828\ud558\uae30 \uc704\ud55c \ubaa8\uae08\ud65c\ub3d9\uc744 \ubc8c\uc600\ub2e4. \ub9c8\uce68\ub0b4 \ub178\uc2a4\uce90\ub864\ub77c\uc774\ub098 \uc8fc\uc758 \ub9ac\ucc28\ub4dc\uc2a8 \uc5ec\uc0ac \uac00 9,000\ub2ec\ub7ec\uc758 \uac74\ucd95\ube44\ub97c \uc9c0\uc6d0\ud558\uba74\uc11c 1927\ub144 10\uc6d4 \uc2e0\ud765\ud559\uad50 \ubcf8\uad00\uc744 \uc2e0\ucd95\ud558\uac8c \ub418\uc5c8\uc73c\uba70, \uc774\ub97c \uae30\ub150\ud558\uc5ec \uba85\uce6d\uc744 \ub9ac\ucc28\ub4dc\uc2a8 \ud640(Richardson Hall)\ub85c \uc815\ud558\uc600\ub2e4. \ubcf8\uad00 \uac74\ubb3c\uc758 \uc124\uacc4\ub294 \uac74\ucd95\uc744 \uc804\uacf5\ud55c \uc11c\ub85c\ub4dd \uc120\uad50\uc0ac\uac00 \ub2f4\ub2f9\ud558\uc600\uc73c\uba70, \uad11\uc8fc\uc218\ud53c\uc544\uc5ec\uc790\uace0\ub4f1\ud559\uad50\uc758 \uc708\uc2a4\ube0c\ub85c\uc6b0 \ud640(Winsborough Hall)\uacfc \uad6c\uc870\uac00 \uac19\uc740 \uc30d\ub465\uc774 \uac74\ubb3c\ub85c \uac74\ucd95\ub418\uc5c8\ub2e4.", "sentence_answer": "\uc120\uad50\uc0ac\ub4e4\uc740 \ubbf8\uad6d \uc73c\ub85c \uac74\ub108\uac00 \uae30\uae08\uc744 \ub9c8\ub828\ud558\uae30 \uc704\ud55c \ubaa8\uae08\ud65c\ub3d9\uc744 \ubc8c\uc600\ub2e4.", "paragraph_id": "6593502-5-1", "paragraph_question": "question: \uc9c0\uc815\ud559\uad50 \uc2b9\uaca9\uc744 \uc704\ud574 \uc2e0\ud765\ud559\uad50\uc758 \uc120\uad50\uc0ac\ub4e4\uc774 \uae30\uae08 \ub9c8\ub828\uc744 \uc704\ud574 \uac74\ub108\uac04 \ub098\ub77c\ub294 \ubb34\uc5c7\uc778\uac00?, context: \uc804\uc8fc\uc2e0\ud765\uace0\ub4f1\ud559\uad50 \uad50\ub0b4(\uc18c\uc7ac\uc9c0: \uc804\ub77c\ubd81\ub3c4 \uc804\uc8fc\uc2dc \uc644\uc0b0\uad6c \uc11c\uc6d0\ub85c 399)\uc5d0 \uc788\uc73c\uba70 \ub4f1\ub85d\ubb38\ud654\uc7ac \uc81c172\ud638 \ub85c \uc9c0\uc815\ub418\uc5c8\ub2e4. \uc77c\uc81c\uac15\uc810\uae30\uc778 1923\ub144\uc5d0 \uc77c\uc815 \uc218\uc900\uc758 \uad50\uc0ac \ud655\ubcf4\uc640 \uc2dc\uc124 \ubc0f \uad50\uc721\uacfc\uc815\uc774 \uac16\ucd94\uc5b4\uc9c4 \uc0ac\ub9bd\ud559\uad50\uc5d0\uac8c \uace0\ub4f1\ubcf4\ud1b5\ud559\uad50\uc640 \uac19\uc740 \uc790\uaca9\uc744 \uc778\uc815\ud558\ub294 \uc18c\uc704 \uc9c0\uc815\ud559\uad50\ub77c\ub294 \uc81c\ub3c4\uac00 \ub9cc\ub4e4\uc5b4\uc84c\ub2e4. \uc2e0\ud765\ud559\uad50\ub294 \uc9c0\uc815\ud559\uad50 \uc2b9\uaca9\uc744 \uc704\ud558\uc5ec \uad50\uc0ac \uc2e0\ucd95\uc744 \ucd94\uc9c4\ud558\uc600\uace0, \uc120\uad50\uc0ac\ub4e4\uc740 \ubbf8\uad6d\uc73c\ub85c \uac74\ub108\uac00 \uae30\uae08\uc744 \ub9c8\ub828\ud558\uae30 \uc704\ud55c \ubaa8\uae08\ud65c\ub3d9\uc744 \ubc8c\uc600\ub2e4. \ub9c8\uce68\ub0b4 \ub178\uc2a4\uce90\ub864\ub77c\uc774\ub098 \uc8fc\uc758 \ub9ac\ucc28\ub4dc\uc2a8 \uc5ec\uc0ac \uac00 9,000\ub2ec\ub7ec\uc758 \uac74\ucd95\ube44\ub97c \uc9c0\uc6d0\ud558\uba74\uc11c 1927\ub144 10\uc6d4 \uc2e0\ud765\ud559\uad50 \ubcf8\uad00\uc744 \uc2e0\ucd95\ud558\uac8c \ub418\uc5c8\uc73c\uba70, \uc774\ub97c \uae30\ub150\ud558\uc5ec \uba85\uce6d\uc744 \ub9ac\ucc28\ub4dc\uc2a8 \ud640(Richardson Hall)\ub85c \uc815\ud558\uc600\ub2e4. \ubcf8\uad00 \uac74\ubb3c\uc758 \uc124\uacc4\ub294 \uac74\ucd95\uc744 \uc804\uacf5\ud55c \uc11c\ub85c\ub4dd \uc120\uad50\uc0ac\uac00 \ub2f4\ub2f9\ud558\uc600\uc73c\uba70, \uad11\uc8fc\uc218\ud53c\uc544\uc5ec\uc790\uace0\ub4f1\ud559\uad50\uc758 \uc708\uc2a4\ube0c\ub85c\uc6b0 \ud640(Winsborough Hall)\uacfc \uad6c\uc870\uac00 \uac19\uc740 \uc30d\ub465\uc774 \uac74\ubb3c\ub85c \uac74\ucd95\ub418\uc5c8\ub2e4."} {"question": "\ub300\ub2e4\uc218 \uc544\ud504\ub9ac\uce74 \uad6d\uac00\uc758 \uc815\ubd80\ud615\ud0dc\ub294?", "paragraph": "\uc624\ub298\ub0a0 \uc544\ud504\ub9ac\uce74\uc5d0\ub294 54\uac1c \ub3c5\ub9bd \uc8fc\uad8c \uad6d\uac00\uac00 \uc788\uc73c\uba70, \uc774\ub4e4 \ub098\ub77c\ub97c \uac00\ub974\ub294 \uad6d\uacbd\uc120\uc740 \ub300\ubd80\ubd84 \uc720\ub7fd \uc2dd\ubbfc \uc9c0\ubc30 \uc2dc\ub300\uc5d0 \ud68d\uc815\ub41c \uac83\uc774\ub2e4. \uc2dd\ubbfc \uc9c0\ubc30 \uc774\ub798 \uc544\ud504\ub9ac\uce74 \ub098\ub77c\ub4e4\uc740 \ubd88\uc548\uc815, \ubd80\ud328, \ud3ed\ub825, \uad8c\uc704\uc8fc\uc758\uc5d0 \uc2dc\ub2ec\ub838\ub2e4. \ub300\ub2e4\uc218 \uc544\ud504\ub9ac\uce74 \uad6d\uac00\ub294 \ub300\ud1b5\ub839\uc81c \ud615\ud0dc\uc758 \uacf5\ud654\uad6d\uc774\ub2e4. \uadf8\ub7ec\ub098 \uc774 \uac00\uc6b4\ub370 \ubbfc\uc8fc\uc801\uc778 \uc815\ubd80\ub97c \uc774\ub8ec \ub098\ub77c\ub294 \uac70\uc758 \uc5c6\uc73c\uba70, \ub300\uac1c \ucfe0\ub370\ud0c0\uac00 \ub418\ud480\uc774\ub418\uba70 \uad70\uc0ac \ub3c5\uc7ac \uc815\uad8c\uc774 \ub4e4\uc5b4\uc130\ub2e4. \ub3c5\ub9bd \uc774\ud6c4 \uc544\ud504\ub9ac\uce74\uc758 \uc815\uce58 \uc9c0\ub3c4\uc790\ub294 \ub300\ubd80\ubd84 \uc7a5\uad70 \ucd9c\uc2e0\uc73c\ub85c, \uad50\uc721 \uc218\uc900\ub3c4 \ub0ae\uace0 \ubbfc\uc8fc\uc801\uc778 \ud1b5\uce58\ub97c \ubb34\uc2dc\ud558\uc600\ub2e4. \uadf8\ub7ec\ub098 \uc774\uc9c8\uc801\uc778 \uc885\uc871\ub4e4\uc774 \uc774\ub4e4 \uc9c0\ubc30\uc790 \ubc11\uc5d0\uc11c \ud1b5\ud569\ub41c \uac83\uc774\uc57c\ub9d0\ub85c \uac00\uc7a5 \ud070 \uc815\uce58 \ubd88\uc548\uc815\uc758 \uc694\uc778\uc774\ub2e4. \uc5ec\ub7ec \uc815\uce58\uc778\ub4e4\uc740 \uc815\uce58\uc801 \uc774\uc775(\"\ubd84\ud560\ud558\uc5ec \ud1b5\uce58\ud558\ub77c\")\uc744 \uc704\ud574 \uc2dd\ubbfc \ud1b5\uce58\ub85c \ubc1c\uc0dd\ud558\uac70\ub098 \uc545\ud654\ub41c \uc885\uc871 \uac08\ub4f1\uc744 \ub354\uc6b1 \ubd80\ucc44\uc9c8\ud558\uc600\ub2e4. \uc5ec\ub7ec \ub098\ub77c\uc5d0\uc11c \uad70\ub300\ub294 \uc9c8\uc11c\ub97c \uc81c\ub300\ub85c \uc720\uc9c0\ud560 \uc720\uc77c\ud55c \uc9d1\ub2e8\uc73c\ub85c \uc5ec\uaca8\uc9c0\uace0 \uc788\uc73c\uba70, 1970\ub144\ub300\uc640 1980\ub144\ub300\uc5d0 \uc544\ud504\ub9ac\uce74\uc758 \uc5ec\ub7ec \ub098\ub77c\uc5d0\uc11c \uad70\uc0ac \uc9c0\ubc30\uac00 \ub098\ud0c0\ub0ac\ub2e4. 1960\ub144\ub300 \ucd08\uc5d0\uc11c 1980\ub144\ub300 \ud6c4\ubc18\uae4c\uc9c0 \uc544\ud504\ub9ac\uce74\ub294 70\ubc88 \uc774\uc0c1\uc758 \ucfe0\ub370\ud0c0\uc640 13\ubc88\uc758 \ub300\ud1b5\ub839 \uc554\uc0b4\uc774 \ubc1c\uc0dd\ud558\uc600\ub2e4. \uc720\ub7fd\uc778\ub4e4\uc774 \uadf8\uc5b4\ub193\uc740 \uad6d\uacbd\uc120 \ub54c\ubb38\uc5d0 \uad6d\uacbd \ubc0f \uc601\ud1a0 \ubd84\uc7c1\ub3c4 \ud754\ud558\uba70, \ubb34\ub825 \uac08\ub4f1\uc774 \ub110\ub9ac \uc77c\uc5b4\ub098\uace0 \uc788\ub2e4.", "answer": "\ub300\ud1b5\ub839\uc81c \ud615\ud0dc\uc758 \uacf5\ud654\uad6d", "sentence": "\ub300\ub2e4\uc218 \uc544\ud504\ub9ac\uce74 \uad6d\uac00\ub294 \ub300\ud1b5\ub839\uc81c \ud615\ud0dc\uc758 \uacf5\ud654\uad6d \uc774\ub2e4.", "paragraph_sentence": "\uc624\ub298\ub0a0 \uc544\ud504\ub9ac\uce74\uc5d0\ub294 54\uac1c \ub3c5\ub9bd \uc8fc\uad8c \uad6d\uac00\uac00 \uc788\uc73c\uba70, \uc774\ub4e4 \ub098\ub77c\ub97c \uac00\ub974\ub294 \uad6d\uacbd\uc120\uc740 \ub300\ubd80\ubd84 \uc720\ub7fd \uc2dd\ubbfc \uc9c0\ubc30 \uc2dc\ub300\uc5d0 \ud68d\uc815\ub41c \uac83\uc774\ub2e4. \uc2dd\ubbfc \uc9c0\ubc30 \uc774\ub798 \uc544\ud504\ub9ac\uce74 \ub098\ub77c\ub4e4\uc740 \ubd88\uc548\uc815, \ubd80\ud328, \ud3ed\ub825, \uad8c\uc704\uc8fc\uc758\uc5d0 \uc2dc\ub2ec\ub838\ub2e4. \ub300\ub2e4\uc218 \uc544\ud504\ub9ac\uce74 \uad6d\uac00\ub294 \ub300\ud1b5\ub839\uc81c \ud615\ud0dc\uc758 \uacf5\ud654\uad6d \uc774\ub2e4. \uadf8\ub7ec\ub098 \uc774 \uac00\uc6b4\ub370 \ubbfc\uc8fc\uc801\uc778 \uc815\ubd80\ub97c \uc774\ub8ec \ub098\ub77c\ub294 \uac70\uc758 \uc5c6\uc73c\uba70, \ub300\uac1c \ucfe0\ub370\ud0c0\uac00 \ub418\ud480\uc774\ub418\uba70 \uad70\uc0ac \ub3c5\uc7ac \uc815\uad8c\uc774 \ub4e4\uc5b4\uc130\ub2e4. \ub3c5\ub9bd \uc774\ud6c4 \uc544\ud504\ub9ac\uce74\uc758 \uc815\uce58 \uc9c0\ub3c4\uc790\ub294 \ub300\ubd80\ubd84 \uc7a5\uad70 \ucd9c\uc2e0\uc73c\ub85c, \uad50\uc721 \uc218\uc900\ub3c4 \ub0ae\uace0 \ubbfc\uc8fc\uc801\uc778 \ud1b5\uce58\ub97c \ubb34\uc2dc\ud558\uc600\ub2e4. \uadf8\ub7ec\ub098 \uc774\uc9c8\uc801\uc778 \uc885\uc871\ub4e4\uc774 \uc774\ub4e4 \uc9c0\ubc30\uc790 \ubc11\uc5d0\uc11c \ud1b5\ud569\ub41c \uac83\uc774\uc57c\ub9d0\ub85c \uac00\uc7a5 \ud070 \uc815\uce58 \ubd88\uc548\uc815\uc758 \uc694\uc778\uc774\ub2e4. \uc5ec\ub7ec \uc815\uce58\uc778\ub4e4\uc740 \uc815\uce58\uc801 \uc774\uc775(\"\ubd84\ud560\ud558\uc5ec \ud1b5\uce58\ud558\ub77c\")\uc744 \uc704\ud574 \uc2dd\ubbfc \ud1b5\uce58\ub85c \ubc1c\uc0dd\ud558\uac70\ub098 \uc545\ud654\ub41c \uc885\uc871 \uac08\ub4f1\uc744 \ub354\uc6b1 \ubd80\ucc44\uc9c8\ud558\uc600\ub2e4. \uc5ec\ub7ec \ub098\ub77c\uc5d0\uc11c \uad70\ub300\ub294 \uc9c8\uc11c\ub97c \uc81c\ub300\ub85c \uc720\uc9c0\ud560 \uc720\uc77c\ud55c \uc9d1\ub2e8\uc73c\ub85c \uc5ec\uaca8\uc9c0\uace0 \uc788\uc73c\uba70, 1970\ub144\ub300\uc640 1980\ub144\ub300\uc5d0 \uc544\ud504\ub9ac\uce74\uc758 \uc5ec\ub7ec \ub098\ub77c\uc5d0\uc11c \uad70\uc0ac \uc9c0\ubc30\uac00 \ub098\ud0c0\ub0ac\ub2e4. 1960\ub144\ub300 \ucd08\uc5d0\uc11c 1980\ub144\ub300 \ud6c4\ubc18\uae4c\uc9c0 \uc544\ud504\ub9ac\uce74\ub294 70\ubc88 \uc774\uc0c1\uc758 \ucfe0\ub370\ud0c0\uc640 13\ubc88\uc758 \ub300\ud1b5\ub839 \uc554\uc0b4\uc774 \ubc1c\uc0dd\ud558\uc600\ub2e4. \uc720\ub7fd\uc778\ub4e4\uc774 \uadf8\uc5b4\ub193\uc740 \uad6d\uacbd\uc120 \ub54c\ubb38\uc5d0 \uad6d\uacbd \ubc0f \uc601\ud1a0 \ubd84\uc7c1\ub3c4 \ud754\ud558\uba70, \ubb34\ub825 \uac08\ub4f1\uc774 \ub110\ub9ac \uc77c\uc5b4\ub098\uace0 \uc788\ub2e4.", "paragraph_answer": "\uc624\ub298\ub0a0 \uc544\ud504\ub9ac\uce74\uc5d0\ub294 54\uac1c \ub3c5\ub9bd \uc8fc\uad8c \uad6d\uac00\uac00 \uc788\uc73c\uba70, \uc774\ub4e4 \ub098\ub77c\ub97c \uac00\ub974\ub294 \uad6d\uacbd\uc120\uc740 \ub300\ubd80\ubd84 \uc720\ub7fd \uc2dd\ubbfc \uc9c0\ubc30 \uc2dc\ub300\uc5d0 \ud68d\uc815\ub41c \uac83\uc774\ub2e4. \uc2dd\ubbfc \uc9c0\ubc30 \uc774\ub798 \uc544\ud504\ub9ac\uce74 \ub098\ub77c\ub4e4\uc740 \ubd88\uc548\uc815, \ubd80\ud328, \ud3ed\ub825, \uad8c\uc704\uc8fc\uc758\uc5d0 \uc2dc\ub2ec\ub838\ub2e4. \ub300\ub2e4\uc218 \uc544\ud504\ub9ac\uce74 \uad6d\uac00\ub294 \ub300\ud1b5\ub839\uc81c \ud615\ud0dc\uc758 \uacf5\ud654\uad6d \uc774\ub2e4. \uadf8\ub7ec\ub098 \uc774 \uac00\uc6b4\ub370 \ubbfc\uc8fc\uc801\uc778 \uc815\ubd80\ub97c \uc774\ub8ec \ub098\ub77c\ub294 \uac70\uc758 \uc5c6\uc73c\uba70, \ub300\uac1c \ucfe0\ub370\ud0c0\uac00 \ub418\ud480\uc774\ub418\uba70 \uad70\uc0ac \ub3c5\uc7ac \uc815\uad8c\uc774 \ub4e4\uc5b4\uc130\ub2e4. \ub3c5\ub9bd \uc774\ud6c4 \uc544\ud504\ub9ac\uce74\uc758 \uc815\uce58 \uc9c0\ub3c4\uc790\ub294 \ub300\ubd80\ubd84 \uc7a5\uad70 \ucd9c\uc2e0\uc73c\ub85c, \uad50\uc721 \uc218\uc900\ub3c4 \ub0ae\uace0 \ubbfc\uc8fc\uc801\uc778 \ud1b5\uce58\ub97c \ubb34\uc2dc\ud558\uc600\ub2e4. \uadf8\ub7ec\ub098 \uc774\uc9c8\uc801\uc778 \uc885\uc871\ub4e4\uc774 \uc774\ub4e4 \uc9c0\ubc30\uc790 \ubc11\uc5d0\uc11c \ud1b5\ud569\ub41c \uac83\uc774\uc57c\ub9d0\ub85c \uac00\uc7a5 \ud070 \uc815\uce58 \ubd88\uc548\uc815\uc758 \uc694\uc778\uc774\ub2e4. \uc5ec\ub7ec \uc815\uce58\uc778\ub4e4\uc740 \uc815\uce58\uc801 \uc774\uc775(\"\ubd84\ud560\ud558\uc5ec \ud1b5\uce58\ud558\ub77c\")\uc744 \uc704\ud574 \uc2dd\ubbfc \ud1b5\uce58\ub85c \ubc1c\uc0dd\ud558\uac70\ub098 \uc545\ud654\ub41c \uc885\uc871 \uac08\ub4f1\uc744 \ub354\uc6b1 \ubd80\ucc44\uc9c8\ud558\uc600\ub2e4. \uc5ec\ub7ec \ub098\ub77c\uc5d0\uc11c \uad70\ub300\ub294 \uc9c8\uc11c\ub97c \uc81c\ub300\ub85c \uc720\uc9c0\ud560 \uc720\uc77c\ud55c \uc9d1\ub2e8\uc73c\ub85c \uc5ec\uaca8\uc9c0\uace0 \uc788\uc73c\uba70, 1970\ub144\ub300\uc640 1980\ub144\ub300\uc5d0 \uc544\ud504\ub9ac\uce74\uc758 \uc5ec\ub7ec \ub098\ub77c\uc5d0\uc11c \uad70\uc0ac \uc9c0\ubc30\uac00 \ub098\ud0c0\ub0ac\ub2e4. 1960\ub144\ub300 \ucd08\uc5d0\uc11c 1980\ub144\ub300 \ud6c4\ubc18\uae4c\uc9c0 \uc544\ud504\ub9ac\uce74\ub294 70\ubc88 \uc774\uc0c1\uc758 \ucfe0\ub370\ud0c0\uc640 13\ubc88\uc758 \ub300\ud1b5\ub839 \uc554\uc0b4\uc774 \ubc1c\uc0dd\ud558\uc600\ub2e4. \uc720\ub7fd\uc778\ub4e4\uc774 \uadf8\uc5b4\ub193\uc740 \uad6d\uacbd\uc120 \ub54c\ubb38\uc5d0 \uad6d\uacbd \ubc0f \uc601\ud1a0 \ubd84\uc7c1\ub3c4 \ud754\ud558\uba70, \ubb34\ub825 \uac08\ub4f1\uc774 \ub110\ub9ac \uc77c\uc5b4\ub098\uace0 \uc788\ub2e4.", "sentence_answer": "\ub300\ub2e4\uc218 \uc544\ud504\ub9ac\uce74 \uad6d\uac00\ub294 \ub300\ud1b5\ub839\uc81c \ud615\ud0dc\uc758 \uacf5\ud654\uad6d \uc774\ub2e4.", "paragraph_id": "6585600-2-2", "paragraph_question": "question: \ub300\ub2e4\uc218 \uc544\ud504\ub9ac\uce74 \uad6d\uac00\uc758 \uc815\ubd80\ud615\ud0dc\ub294?, context: \uc624\ub298\ub0a0 \uc544\ud504\ub9ac\uce74\uc5d0\ub294 54\uac1c \ub3c5\ub9bd \uc8fc\uad8c \uad6d\uac00\uac00 \uc788\uc73c\uba70, \uc774\ub4e4 \ub098\ub77c\ub97c \uac00\ub974\ub294 \uad6d\uacbd\uc120\uc740 \ub300\ubd80\ubd84 \uc720\ub7fd \uc2dd\ubbfc \uc9c0\ubc30 \uc2dc\ub300\uc5d0 \ud68d\uc815\ub41c \uac83\uc774\ub2e4. \uc2dd\ubbfc \uc9c0\ubc30 \uc774\ub798 \uc544\ud504\ub9ac\uce74 \ub098\ub77c\ub4e4\uc740 \ubd88\uc548\uc815, \ubd80\ud328, \ud3ed\ub825, \uad8c\uc704\uc8fc\uc758\uc5d0 \uc2dc\ub2ec\ub838\ub2e4. \ub300\ub2e4\uc218 \uc544\ud504\ub9ac\uce74 \uad6d\uac00\ub294 \ub300\ud1b5\ub839\uc81c \ud615\ud0dc\uc758 \uacf5\ud654\uad6d\uc774\ub2e4. \uadf8\ub7ec\ub098 \uc774 \uac00\uc6b4\ub370 \ubbfc\uc8fc\uc801\uc778 \uc815\ubd80\ub97c \uc774\ub8ec \ub098\ub77c\ub294 \uac70\uc758 \uc5c6\uc73c\uba70, \ub300\uac1c \ucfe0\ub370\ud0c0\uac00 \ub418\ud480\uc774\ub418\uba70 \uad70\uc0ac \ub3c5\uc7ac \uc815\uad8c\uc774 \ub4e4\uc5b4\uc130\ub2e4. \ub3c5\ub9bd \uc774\ud6c4 \uc544\ud504\ub9ac\uce74\uc758 \uc815\uce58 \uc9c0\ub3c4\uc790\ub294 \ub300\ubd80\ubd84 \uc7a5\uad70 \ucd9c\uc2e0\uc73c\ub85c, \uad50\uc721 \uc218\uc900\ub3c4 \ub0ae\uace0 \ubbfc\uc8fc\uc801\uc778 \ud1b5\uce58\ub97c \ubb34\uc2dc\ud558\uc600\ub2e4. \uadf8\ub7ec\ub098 \uc774\uc9c8\uc801\uc778 \uc885\uc871\ub4e4\uc774 \uc774\ub4e4 \uc9c0\ubc30\uc790 \ubc11\uc5d0\uc11c \ud1b5\ud569\ub41c \uac83\uc774\uc57c\ub9d0\ub85c \uac00\uc7a5 \ud070 \uc815\uce58 \ubd88\uc548\uc815\uc758 \uc694\uc778\uc774\ub2e4. \uc5ec\ub7ec \uc815\uce58\uc778\ub4e4\uc740 \uc815\uce58\uc801 \uc774\uc775(\"\ubd84\ud560\ud558\uc5ec \ud1b5\uce58\ud558\ub77c\")\uc744 \uc704\ud574 \uc2dd\ubbfc \ud1b5\uce58\ub85c \ubc1c\uc0dd\ud558\uac70\ub098 \uc545\ud654\ub41c \uc885\uc871 \uac08\ub4f1\uc744 \ub354\uc6b1 \ubd80\ucc44\uc9c8\ud558\uc600\ub2e4. \uc5ec\ub7ec \ub098\ub77c\uc5d0\uc11c \uad70\ub300\ub294 \uc9c8\uc11c\ub97c \uc81c\ub300\ub85c \uc720\uc9c0\ud560 \uc720\uc77c\ud55c \uc9d1\ub2e8\uc73c\ub85c \uc5ec\uaca8\uc9c0\uace0 \uc788\uc73c\uba70, 1970\ub144\ub300\uc640 1980\ub144\ub300\uc5d0 \uc544\ud504\ub9ac\uce74\uc758 \uc5ec\ub7ec \ub098\ub77c\uc5d0\uc11c \uad70\uc0ac \uc9c0\ubc30\uac00 \ub098\ud0c0\ub0ac\ub2e4. 1960\ub144\ub300 \ucd08\uc5d0\uc11c 1980\ub144\ub300 \ud6c4\ubc18\uae4c\uc9c0 \uc544\ud504\ub9ac\uce74\ub294 70\ubc88 \uc774\uc0c1\uc758 \ucfe0\ub370\ud0c0\uc640 13\ubc88\uc758 \ub300\ud1b5\ub839 \uc554\uc0b4\uc774 \ubc1c\uc0dd\ud558\uc600\ub2e4. \uc720\ub7fd\uc778\ub4e4\uc774 \uadf8\uc5b4\ub193\uc740 \uad6d\uacbd\uc120 \ub54c\ubb38\uc5d0 \uad6d\uacbd \ubc0f \uc601\ud1a0 \ubd84\uc7c1\ub3c4 \ud754\ud558\uba70, \ubb34\ub825 \uac08\ub4f1\uc774 \ub110\ub9ac \uc77c\uc5b4\ub098\uace0 \uc788\ub2e4."} {"question": "\ub124\uc774\ubc84\uc5d0\uc11c \uc2e0\uc778 \uc791\uac00\ub4e4\uc758 \ub370\ubdd4\ub97c \uc704\ud574 \ub9cc\ub4e0 \ucf54\ub108\ub294?", "paragraph": "\uc6f9\ud230 \ub9cc\ud654\uac00\ub4e4\uc740 \ub300\uac1c \uc790\uccb4\uc801\uc778 \uc218\uc775 \ucc3d\ucd9c\ubcf4\ub2e4\ub294 \uae09\uc5ec\ub97c \ubc1b\uc73c\uba74\uc11c \uc6d0\uace0\ub97c \uc81c\ucd9c\ud558\ub294 \ubc29\uc2dd\uc73c\ub85c \uc0dd\ud65c\ud558\uace0 \uc788\ub2e4. \uae09\uc5ec \uc81c\uacf5\uc758 \uc8fc\ub41c \uc8fc\uccb4\ub294 \ud3ec\ud138 \uc0ac\uc774\ud2b8\uc774\uba70, \ub300\ud45c\uc801\uc778 \ud3ec\ud138\ub85c\ub294 \ub124\uc774\ubc84\uc640 \ub2e4\uc74c\uc774 \uc788\ub2e4. \uc6f9\ud230 \uc2dc\uc7a5\uc5d0\uc11c \uc791\uac00\ub294 \ud3ec\ud138\uacfc \uacc4\uc57d\uad00\uacc4\ub85c \ub9fa\uc5b4\uc9c0\uba70 \uc2e0\uc778 \uc791\uac00\uc758 \uacbd\uc6b0 \ub300\ubd80\ubd84 \uc778\ud130\ub137 \ud3ec\ud138\uc0ac\uc774\ud2b8\uc758 \u2018\ub098\ub3c4 \ub9cc\ud654\uac00\u2019(\ub2e4\uc74c), \u2018\ub3c4\uc804 \ub9cc\ud654\u2019(\ub124\uc774\ubc84)\ub4f1\uc758 \ucf54\ub108\ub97c \ud1b5\ud574 \ub370\ubdd4\ub97c \ud55c\ub2e4. \uc791\uac00\ub294 \ucf54\ub108\uc5d0 \uc790\uc2e0\uc758 \uc791\ud488\uc744 \uc62c\ub9ac\uace0 \ud3ec\ud138\uc740 \uc870\ud68c\uc218\uac00 \ub9ce\uc740 \uc791\ud488\uc744 \uc120\ubcc4\ud558\uc5ec \uacc4\uc57d\uc744 \ud558\uac8c \ub418\ub294\ub370, \uacc4\uc57d\uc774\ud6c4 \ubaa8\ub2c8\ud130\ub9c1 \uae30\uac04\uc744 \uac00\uc9c0\uac8c \ub41c\ub2e4. \uc774 \uae30\uac04 \ub3d9\uc548 \ud3ec\ud138\uc740 \uc791\ud488\uc758 \ud765\ud589\uc5ec\ubd80, \uc0c1\ud488\uac00\uce58 \uc720\ubb34\ub97c \ud655\uc778\ud558\uace0 \uc791\uac00\uc758 \uc6d0\uace0\ub8cc\ub97c \uc0b0\ucd9c\ud55c\ub2e4. \ub124\uc774\ubc84\uc758 \uacbd\uc6b0 \uc870\ud68c\uc218, \ud68c\ucc28, \ubc30\uacbd\uc720\ubb34, \ubd84\ub7c9 \ub4f1\uc744 \uae30\uc900\uc73c\ub85c \uc6d0\uace0\ub8cc\ub97c \uc0b0\ucd9c\ud558\uba70, \ub2e4\uc74c\uc758 \uacbd\uc6b0 \uc870\ud68c\uc218\ub85c \uc6d0\uace0\ub8cc\ub97c \uc0b0\ucd9c\ud55c\ub2e4. \ub124\uc774\ubc84\uc640 \ub2e4\uc74c\uc758 \uacbd\uc6b0\uc5d0\ub294 \uc218\uc2ed \uba85 \uc815\ub3c4\uc758 \ub9cc\ud654\uac00\ub4e4\uc744 \ud504\ub85c\ub85c\uc368 \uace0\uc6a9, \ub9cc\ud654\ub97c \ubb34\ub8cc\ub85c \ub204\ub9ac\uafbc\ub4e4\uc5d0\uac8c \uc81c\uacf5\ud558\uace0 \uc788\ub2e4. \ub124\uc774\ubc84\uc758 \uc6f9\ud230 \uc11c\ube44\uc2a4\uc758 \ud3c9\uade0 \uc870\ud68c\uc218\ub294 300\ub9cc\uc73c\ub85c \ucd94\uce21\ub41c\ub2e4. \uc778\uae30\uc788\ub294 \uc6f9\ud230\uc758 \uacbd\uc6b0\uc5d0\ub294 \ub313\uae00\uc774 5,000\uac1c\ub97c \ub118\ub294 \ub4f1 \uc6ec\ub9cc\ud55c \uc624\ud504\ub77c\uc778 \ub9cc\ud654\uac00\ub4e4 \ubabb\uc9c0\uc54a\uc740 \uc778\uae30\ub97c \ub204\ub9ac\uace0 \uc788\ub2e4. \ud558\uc9c0\ub9cc \uc6f9\ud230\uc774 \ubb34\ub8cc \ucee8\ud14c\uce20\ub85c \uc11c\ube44\uc2a4 \ub41c\ub2e4\ub294 \uc810\uc5d0\uc11c \uc77c\ubd80 \uc720\uba85 \uc791\uac00\ub97c \uc81c\uc678\ud588\uc744 \ub54c \ub9cc\ud654\uac00\uc5d0\uac8c \uc9c1\uc811\uc801\uc778 \uc218\uc775\uc740 \ud604\uc800\ud788 \ub0ae\uc740 \uac83\uc774 \uc0ac\uc2e4\uc774\ub2e4. \uc2e0\uc778 \uc791\uac00\uac00 \ub124\uc774\ubc84\u00b7\ub2e4\uc74c\u00b7\ud30c\ub780 \ub4f1 \uc8fc\uc694 \uc778\ud130\ub137 \ud3ec\ud138\uc5d0\uc11c \uc5f0\uc7ac\ub97c \uc2dc\uc791\ud560 \uacbd\uc6b0 \ucd5c\uc800 \uace0\ub8cc\ub294 \ud68c\ub2f9 10\ub9cc\u223c20\ub9cc\uc6d0\uc120. \uac01 \ud3ec\ud138\ub9c8\ub2e4 \uc791\uac00\uc758 \uc9c0\uba85\ub3c4\uc640 \uc9c1\uc804 \uc791\ud488\uc758 \uc6d0\uace0\ub8cc, \uc5f0\uc7ac \uc8fc\uae30, \uaddc\ubaa8 \ub4f1\uc758 \ucd94\uac00 \uc694\uc778\uc744 \ud569\uc0b0\ud574 \uc791\ud488 \ub2e8\uc704 \ud639\uc740 3\uac1c\uc6d4, 6\uac1c\uc6d4 \uc8fc\uae30\uc5d0 \ud55c \ubc88\uc529 \uc6d0\uace0\ub8cc\ub97c \uac31\uc2e0\ud558\ub294 \uad6c\uc870\ub2e4. \ucd9c\ud310\uc0ac \uc9c1\uc601\uc0ac\uc774\ud2b8\ub294 \uc720\ub8cc \uc11c\ube44\uc2a4\ub97c \ud1b5\ud574 \uc218\uc775\uc744 \ub0b4\ub294 \uad6c\uc870\uc774\uba70, \ud3ec\ud138\uc0ac\uc774\ud2b8\ub294 \uc11c\ube44\uc2a4\ub9e4\ucd9c \uc774\uc678\uc5d0 \ud310\uad8c\ub8cc \ubc0f \ub7ec\ub2dd\ub85c\uc5f4\ud2f0\ub97c \ubc1b\uac8c \ub41c\ub2e4. \uc778\uae30 \ub9cc\ud654\uc758 \uacbd\uc6b0 \ucd9c\ud310 \ub9cc\ud654, \uce90\ub9ad\ud130 \uc0c1\ud488, \ub4dc\ub77c\ub9c8, \uc601\ud654, \uc5f0\uadf9 \ub4f1\uc758 \ub2e4\uc591\ud55c \uc7a5\ub974\uc758 \ucee8\ud150\uce20\ub85c \ud65c\uc6a9\ub418\uba74\uc11c \uc0c8\ub85c\uc6b4 \uc218\uc775\uc744 \ub0b3\ub294 \uacbd\uc6b0\ub3c4 \uc788\ub2e4. \ub2e8\ud589\ubcf8\uc740 \uc791\ud488\uc774 \uc628\ub77c\uc778\uc73c\ub85c \uac8c\uc7ac\ub41c \ud6c4 \uc120\ubcc4\ub418\uc5b4 \ucd9c\uac04\ub418\uba70 \ucc45\uac12\uc740 \ub300\ub7b5 9\ucc9c\uc6d0 \ub0b4\uc678\ub2e4. \uc774\uc678\uc5d0\ub3c4 \uc791\ud488\uc774 \uc778\uae30\ub97c \uc5bb\uac8c \ub418\uba74 2\ucc28 \uc2dc\uc7a5\uc778 \uc601\ud654\ub098 \ub4dc\ub77c\ub9c8, \uc560\ub2c8\uba54\uc774\uc158 \ub4f1\uc73c\ub85c \ub098\uc624\uae30\ub3c4 \ud558\uba70, 3\ucc28 \uc2dc\uc7a5\uc778 \ubb38\uad6c, \uc644\uad6c, \uc758\ub958 \ub4f1 \uc0c1\ud488\uc73c\ub85c \ub098\uc624\ub294 \uacbd\uc6b0\ub3c4 \uc788\ub2e4.", "answer": "\ub3c4\uc804 \ub9cc\ud654", "sentence": "\uc6f9\ud230 \uc2dc\uc7a5\uc5d0\uc11c \uc791\uac00\ub294 \ud3ec\ud138\uacfc \uacc4\uc57d\uad00\uacc4\ub85c \ub9fa\uc5b4\uc9c0\uba70 \uc2e0\uc778 \uc791\uac00\uc758 \uacbd\uc6b0 \ub300\ubd80\ubd84 \uc778\ud130\ub137 \ud3ec\ud138\uc0ac\uc774\ud2b8\uc758 \u2018\ub098\ub3c4 \ub9cc\ud654\uac00\u2019(\ub2e4\uc74c), \u2018 \ub3c4\uc804 \ub9cc\ud654 \u2019(\ub124\uc774\ubc84)\ub4f1\uc758 \ucf54\ub108\ub97c \ud1b5\ud574 \ub370\ubdd4\ub97c \ud55c\ub2e4.", "paragraph_sentence": "\uc6f9\ud230 \ub9cc\ud654\uac00\ub4e4\uc740 \ub300\uac1c \uc790\uccb4\uc801\uc778 \uc218\uc775 \ucc3d\ucd9c\ubcf4\ub2e4\ub294 \uae09\uc5ec\ub97c \ubc1b\uc73c\uba74\uc11c \uc6d0\uace0\ub97c \uc81c\ucd9c\ud558\ub294 \ubc29\uc2dd\uc73c\ub85c \uc0dd\ud65c\ud558\uace0 \uc788\ub2e4. \uae09\uc5ec \uc81c\uacf5\uc758 \uc8fc\ub41c \uc8fc\uccb4\ub294 \ud3ec\ud138 \uc0ac\uc774\ud2b8\uc774\uba70, \ub300\ud45c\uc801\uc778 \ud3ec\ud138\ub85c\ub294 \ub124\uc774\ubc84\uc640 \ub2e4\uc74c\uc774 \uc788\ub2e4. \uc6f9\ud230 \uc2dc\uc7a5\uc5d0\uc11c \uc791\uac00\ub294 \ud3ec\ud138\uacfc \uacc4\uc57d\uad00\uacc4\ub85c \ub9fa\uc5b4\uc9c0\uba70 \uc2e0\uc778 \uc791\uac00\uc758 \uacbd\uc6b0 \ub300\ubd80\ubd84 \uc778\ud130\ub137 \ud3ec\ud138\uc0ac\uc774\ud2b8\uc758 \u2018\ub098\ub3c4 \ub9cc\ud654\uac00\u2019(\ub2e4\uc74c), \u2018 \ub3c4\uc804 \ub9cc\ud654 \u2019(\ub124\uc774\ubc84)\ub4f1\uc758 \ucf54\ub108\ub97c \ud1b5\ud574 \ub370\ubdd4\ub97c \ud55c\ub2e4. \uc791\uac00\ub294 \ucf54\ub108\uc5d0 \uc790\uc2e0\uc758 \uc791\ud488\uc744 \uc62c\ub9ac\uace0 \ud3ec\ud138\uc740 \uc870\ud68c\uc218\uac00 \ub9ce\uc740 \uc791\ud488\uc744 \uc120\ubcc4\ud558\uc5ec \uacc4\uc57d\uc744 \ud558\uac8c \ub418\ub294\ub370, \uacc4\uc57d\uc774\ud6c4 \ubaa8\ub2c8\ud130\ub9c1 \uae30\uac04\uc744 \uac00\uc9c0\uac8c \ub41c\ub2e4. \uc774 \uae30\uac04 \ub3d9\uc548 \ud3ec\ud138\uc740 \uc791\ud488\uc758 \ud765\ud589\uc5ec\ubd80, \uc0c1\ud488\uac00\uce58 \uc720\ubb34\ub97c \ud655\uc778\ud558\uace0 \uc791\uac00\uc758 \uc6d0\uace0\ub8cc\ub97c \uc0b0\ucd9c\ud55c\ub2e4. \ub124\uc774\ubc84\uc758 \uacbd\uc6b0 \uc870\ud68c\uc218, \ud68c\ucc28, \ubc30\uacbd\uc720\ubb34, \ubd84\ub7c9 \ub4f1\uc744 \uae30\uc900\uc73c\ub85c \uc6d0\uace0\ub8cc\ub97c \uc0b0\ucd9c\ud558\uba70, \ub2e4\uc74c\uc758 \uacbd\uc6b0 \uc870\ud68c\uc218\ub85c \uc6d0\uace0\ub8cc\ub97c \uc0b0\ucd9c\ud55c\ub2e4. \ub124\uc774\ubc84\uc640 \ub2e4\uc74c\uc758 \uacbd\uc6b0\uc5d0\ub294 \uc218\uc2ed \uba85 \uc815\ub3c4\uc758 \ub9cc\ud654\uac00\ub4e4\uc744 \ud504\ub85c\ub85c\uc368 \uace0\uc6a9, \ub9cc\ud654\ub97c \ubb34\ub8cc\ub85c \ub204\ub9ac\uafbc\ub4e4\uc5d0\uac8c \uc81c\uacf5\ud558\uace0 \uc788\ub2e4. \ub124\uc774\ubc84\uc758 \uc6f9\ud230 \uc11c\ube44\uc2a4\uc758 \ud3c9\uade0 \uc870\ud68c\uc218\ub294 300\ub9cc\uc73c\ub85c \ucd94\uce21\ub41c\ub2e4. \uc778\uae30\uc788\ub294 \uc6f9\ud230\uc758 \uacbd\uc6b0\uc5d0\ub294 \ub313\uae00\uc774 5,000\uac1c\ub97c \ub118\ub294 \ub4f1 \uc6ec\ub9cc\ud55c \uc624\ud504\ub77c\uc778 \ub9cc\ud654\uac00\ub4e4 \ubabb\uc9c0\uc54a\uc740 \uc778\uae30\ub97c \ub204\ub9ac\uace0 \uc788\ub2e4. \ud558\uc9c0\ub9cc \uc6f9\ud230\uc774 \ubb34\ub8cc \ucee8\ud14c\uce20\ub85c \uc11c\ube44\uc2a4 \ub41c\ub2e4\ub294 \uc810\uc5d0\uc11c \uc77c\ubd80 \uc720\uba85 \uc791\uac00\ub97c \uc81c\uc678\ud588\uc744 \ub54c \ub9cc\ud654\uac00\uc5d0\uac8c \uc9c1\uc811\uc801\uc778 \uc218\uc775\uc740 \ud604\uc800\ud788 \ub0ae\uc740 \uac83\uc774 \uc0ac\uc2e4\uc774\ub2e4. \uc2e0\uc778 \uc791\uac00\uac00 \ub124\uc774\ubc84\u00b7\ub2e4\uc74c\u00b7\ud30c\ub780 \ub4f1 \uc8fc\uc694 \uc778\ud130\ub137 \ud3ec\ud138\uc5d0\uc11c \uc5f0\uc7ac\ub97c \uc2dc\uc791\ud560 \uacbd\uc6b0 \ucd5c\uc800 \uace0\ub8cc\ub294 \ud68c\ub2f9 10\ub9cc\u223c20\ub9cc\uc6d0\uc120. \uac01 \ud3ec\ud138\ub9c8\ub2e4 \uc791\uac00\uc758 \uc9c0\uba85\ub3c4\uc640 \uc9c1\uc804 \uc791\ud488\uc758 \uc6d0\uace0\ub8cc, \uc5f0\uc7ac \uc8fc\uae30, \uaddc\ubaa8 \ub4f1\uc758 \ucd94\uac00 \uc694\uc778\uc744 \ud569\uc0b0\ud574 \uc791\ud488 \ub2e8\uc704 \ud639\uc740 3\uac1c\uc6d4, 6\uac1c\uc6d4 \uc8fc\uae30\uc5d0 \ud55c \ubc88\uc529 \uc6d0\uace0\ub8cc\ub97c \uac31\uc2e0\ud558\ub294 \uad6c\uc870\ub2e4. \ucd9c\ud310\uc0ac \uc9c1\uc601\uc0ac\uc774\ud2b8\ub294 \uc720\ub8cc \uc11c\ube44\uc2a4\ub97c \ud1b5\ud574 \uc218\uc775\uc744 \ub0b4\ub294 \uad6c\uc870\uc774\uba70, \ud3ec\ud138\uc0ac\uc774\ud2b8\ub294 \uc11c\ube44\uc2a4\ub9e4\ucd9c \uc774\uc678\uc5d0 \ud310\uad8c\ub8cc \ubc0f \ub7ec\ub2dd\ub85c\uc5f4\ud2f0\ub97c \ubc1b\uac8c \ub41c\ub2e4. \uc778\uae30 \ub9cc\ud654\uc758 \uacbd\uc6b0 \ucd9c\ud310 \ub9cc\ud654, \uce90\ub9ad\ud130 \uc0c1\ud488, \ub4dc\ub77c\ub9c8, \uc601\ud654, \uc5f0\uadf9 \ub4f1\uc758 \ub2e4\uc591\ud55c \uc7a5\ub974\uc758 \ucee8\ud150\uce20\ub85c \ud65c\uc6a9\ub418\uba74\uc11c \uc0c8\ub85c\uc6b4 \uc218\uc775\uc744 \ub0b3\ub294 \uacbd\uc6b0\ub3c4 \uc788\ub2e4. \ub2e8\ud589\ubcf8\uc740 \uc791\ud488\uc774 \uc628\ub77c\uc778\uc73c\ub85c \uac8c\uc7ac\ub41c \ud6c4 \uc120\ubcc4\ub418\uc5b4 \ucd9c\uac04\ub418\uba70 \ucc45\uac12\uc740 \ub300\ub7b5 9\ucc9c\uc6d0 \ub0b4\uc678\ub2e4. \uc774\uc678\uc5d0\ub3c4 \uc791\ud488\uc774 \uc778\uae30\ub97c \uc5bb\uac8c \ub418\uba74 2\ucc28 \uc2dc\uc7a5\uc778 \uc601\ud654\ub098 \ub4dc\ub77c\ub9c8, \uc560\ub2c8\uba54\uc774\uc158 \ub4f1\uc73c\ub85c \ub098\uc624\uae30\ub3c4 \ud558\uba70, 3\ucc28 \uc2dc\uc7a5\uc778 \ubb38\uad6c, \uc644\uad6c, \uc758\ub958 \ub4f1 \uc0c1\ud488\uc73c\ub85c \ub098\uc624\ub294 \uacbd\uc6b0\ub3c4 \uc788\ub2e4.", "paragraph_answer": "\uc6f9\ud230 \ub9cc\ud654\uac00\ub4e4\uc740 \ub300\uac1c \uc790\uccb4\uc801\uc778 \uc218\uc775 \ucc3d\ucd9c\ubcf4\ub2e4\ub294 \uae09\uc5ec\ub97c \ubc1b\uc73c\uba74\uc11c \uc6d0\uace0\ub97c \uc81c\ucd9c\ud558\ub294 \ubc29\uc2dd\uc73c\ub85c \uc0dd\ud65c\ud558\uace0 \uc788\ub2e4. \uae09\uc5ec \uc81c\uacf5\uc758 \uc8fc\ub41c \uc8fc\uccb4\ub294 \ud3ec\ud138 \uc0ac\uc774\ud2b8\uc774\uba70, \ub300\ud45c\uc801\uc778 \ud3ec\ud138\ub85c\ub294 \ub124\uc774\ubc84\uc640 \ub2e4\uc74c\uc774 \uc788\ub2e4. \uc6f9\ud230 \uc2dc\uc7a5\uc5d0\uc11c \uc791\uac00\ub294 \ud3ec\ud138\uacfc \uacc4\uc57d\uad00\uacc4\ub85c \ub9fa\uc5b4\uc9c0\uba70 \uc2e0\uc778 \uc791\uac00\uc758 \uacbd\uc6b0 \ub300\ubd80\ubd84 \uc778\ud130\ub137 \ud3ec\ud138\uc0ac\uc774\ud2b8\uc758 \u2018\ub098\ub3c4 \ub9cc\ud654\uac00\u2019(\ub2e4\uc74c), \u2018 \ub3c4\uc804 \ub9cc\ud654 \u2019(\ub124\uc774\ubc84)\ub4f1\uc758 \ucf54\ub108\ub97c \ud1b5\ud574 \ub370\ubdd4\ub97c \ud55c\ub2e4. \uc791\uac00\ub294 \ucf54\ub108\uc5d0 \uc790\uc2e0\uc758 \uc791\ud488\uc744 \uc62c\ub9ac\uace0 \ud3ec\ud138\uc740 \uc870\ud68c\uc218\uac00 \ub9ce\uc740 \uc791\ud488\uc744 \uc120\ubcc4\ud558\uc5ec \uacc4\uc57d\uc744 \ud558\uac8c \ub418\ub294\ub370, \uacc4\uc57d\uc774\ud6c4 \ubaa8\ub2c8\ud130\ub9c1 \uae30\uac04\uc744 \uac00\uc9c0\uac8c \ub41c\ub2e4. \uc774 \uae30\uac04 \ub3d9\uc548 \ud3ec\ud138\uc740 \uc791\ud488\uc758 \ud765\ud589\uc5ec\ubd80, \uc0c1\ud488\uac00\uce58 \uc720\ubb34\ub97c \ud655\uc778\ud558\uace0 \uc791\uac00\uc758 \uc6d0\uace0\ub8cc\ub97c \uc0b0\ucd9c\ud55c\ub2e4. \ub124\uc774\ubc84\uc758 \uacbd\uc6b0 \uc870\ud68c\uc218, \ud68c\ucc28, \ubc30\uacbd\uc720\ubb34, \ubd84\ub7c9 \ub4f1\uc744 \uae30\uc900\uc73c\ub85c \uc6d0\uace0\ub8cc\ub97c \uc0b0\ucd9c\ud558\uba70, \ub2e4\uc74c\uc758 \uacbd\uc6b0 \uc870\ud68c\uc218\ub85c \uc6d0\uace0\ub8cc\ub97c \uc0b0\ucd9c\ud55c\ub2e4. \ub124\uc774\ubc84\uc640 \ub2e4\uc74c\uc758 \uacbd\uc6b0\uc5d0\ub294 \uc218\uc2ed \uba85 \uc815\ub3c4\uc758 \ub9cc\ud654\uac00\ub4e4\uc744 \ud504\ub85c\ub85c\uc368 \uace0\uc6a9, \ub9cc\ud654\ub97c \ubb34\ub8cc\ub85c \ub204\ub9ac\uafbc\ub4e4\uc5d0\uac8c \uc81c\uacf5\ud558\uace0 \uc788\ub2e4. \ub124\uc774\ubc84\uc758 \uc6f9\ud230 \uc11c\ube44\uc2a4\uc758 \ud3c9\uade0 \uc870\ud68c\uc218\ub294 300\ub9cc\uc73c\ub85c \ucd94\uce21\ub41c\ub2e4. \uc778\uae30\uc788\ub294 \uc6f9\ud230\uc758 \uacbd\uc6b0\uc5d0\ub294 \ub313\uae00\uc774 5,000\uac1c\ub97c \ub118\ub294 \ub4f1 \uc6ec\ub9cc\ud55c \uc624\ud504\ub77c\uc778 \ub9cc\ud654\uac00\ub4e4 \ubabb\uc9c0\uc54a\uc740 \uc778\uae30\ub97c \ub204\ub9ac\uace0 \uc788\ub2e4. \ud558\uc9c0\ub9cc \uc6f9\ud230\uc774 \ubb34\ub8cc \ucee8\ud14c\uce20\ub85c \uc11c\ube44\uc2a4 \ub41c\ub2e4\ub294 \uc810\uc5d0\uc11c \uc77c\ubd80 \uc720\uba85 \uc791\uac00\ub97c \uc81c\uc678\ud588\uc744 \ub54c \ub9cc\ud654\uac00\uc5d0\uac8c \uc9c1\uc811\uc801\uc778 \uc218\uc775\uc740 \ud604\uc800\ud788 \ub0ae\uc740 \uac83\uc774 \uc0ac\uc2e4\uc774\ub2e4. \uc2e0\uc778 \uc791\uac00\uac00 \ub124\uc774\ubc84\u00b7\ub2e4\uc74c\u00b7\ud30c\ub780 \ub4f1 \uc8fc\uc694 \uc778\ud130\ub137 \ud3ec\ud138\uc5d0\uc11c \uc5f0\uc7ac\ub97c \uc2dc\uc791\ud560 \uacbd\uc6b0 \ucd5c\uc800 \uace0\ub8cc\ub294 \ud68c\ub2f9 10\ub9cc\u223c20\ub9cc\uc6d0\uc120. \uac01 \ud3ec\ud138\ub9c8\ub2e4 \uc791\uac00\uc758 \uc9c0\uba85\ub3c4\uc640 \uc9c1\uc804 \uc791\ud488\uc758 \uc6d0\uace0\ub8cc, \uc5f0\uc7ac \uc8fc\uae30, \uaddc\ubaa8 \ub4f1\uc758 \ucd94\uac00 \uc694\uc778\uc744 \ud569\uc0b0\ud574 \uc791\ud488 \ub2e8\uc704 \ud639\uc740 3\uac1c\uc6d4, 6\uac1c\uc6d4 \uc8fc\uae30\uc5d0 \ud55c \ubc88\uc529 \uc6d0\uace0\ub8cc\ub97c \uac31\uc2e0\ud558\ub294 \uad6c\uc870\ub2e4. \ucd9c\ud310\uc0ac \uc9c1\uc601\uc0ac\uc774\ud2b8\ub294 \uc720\ub8cc \uc11c\ube44\uc2a4\ub97c \ud1b5\ud574 \uc218\uc775\uc744 \ub0b4\ub294 \uad6c\uc870\uc774\uba70, \ud3ec\ud138\uc0ac\uc774\ud2b8\ub294 \uc11c\ube44\uc2a4\ub9e4\ucd9c \uc774\uc678\uc5d0 \ud310\uad8c\ub8cc \ubc0f \ub7ec\ub2dd\ub85c\uc5f4\ud2f0\ub97c \ubc1b\uac8c \ub41c\ub2e4. \uc778\uae30 \ub9cc\ud654\uc758 \uacbd\uc6b0 \ucd9c\ud310 \ub9cc\ud654, \uce90\ub9ad\ud130 \uc0c1\ud488, \ub4dc\ub77c\ub9c8, \uc601\ud654, \uc5f0\uadf9 \ub4f1\uc758 \ub2e4\uc591\ud55c \uc7a5\ub974\uc758 \ucee8\ud150\uce20\ub85c \ud65c\uc6a9\ub418\uba74\uc11c \uc0c8\ub85c\uc6b4 \uc218\uc775\uc744 \ub0b3\ub294 \uacbd\uc6b0\ub3c4 \uc788\ub2e4. \ub2e8\ud589\ubcf8\uc740 \uc791\ud488\uc774 \uc628\ub77c\uc778\uc73c\ub85c \uac8c\uc7ac\ub41c \ud6c4 \uc120\ubcc4\ub418\uc5b4 \ucd9c\uac04\ub418\uba70 \ucc45\uac12\uc740 \ub300\ub7b5 9\ucc9c\uc6d0 \ub0b4\uc678\ub2e4. \uc774\uc678\uc5d0\ub3c4 \uc791\ud488\uc774 \uc778\uae30\ub97c \uc5bb\uac8c \ub418\uba74 2\ucc28 \uc2dc\uc7a5\uc778 \uc601\ud654\ub098 \ub4dc\ub77c\ub9c8, \uc560\ub2c8\uba54\uc774\uc158 \ub4f1\uc73c\ub85c \ub098\uc624\uae30\ub3c4 \ud558\uba70, 3\ucc28 \uc2dc\uc7a5\uc778 \ubb38\uad6c, \uc644\uad6c, \uc758\ub958 \ub4f1 \uc0c1\ud488\uc73c\ub85c \ub098\uc624\ub294 \uacbd\uc6b0\ub3c4 \uc788\ub2e4.", "sentence_answer": "\uc6f9\ud230 \uc2dc\uc7a5\uc5d0\uc11c \uc791\uac00\ub294 \ud3ec\ud138\uacfc \uacc4\uc57d\uad00\uacc4\ub85c \ub9fa\uc5b4\uc9c0\uba70 \uc2e0\uc778 \uc791\uac00\uc758 \uacbd\uc6b0 \ub300\ubd80\ubd84 \uc778\ud130\ub137 \ud3ec\ud138\uc0ac\uc774\ud2b8\uc758 \u2018\ub098\ub3c4 \ub9cc\ud654\uac00\u2019(\ub2e4\uc74c), \u2018 \ub3c4\uc804 \ub9cc\ud654 \u2019(\ub124\uc774\ubc84)\ub4f1\uc758 \ucf54\ub108\ub97c \ud1b5\ud574 \ub370\ubdd4\ub97c \ud55c\ub2e4.", "paragraph_id": "6526920-4-0", "paragraph_question": "question: \ub124\uc774\ubc84\uc5d0\uc11c \uc2e0\uc778 \uc791\uac00\ub4e4\uc758 \ub370\ubdd4\ub97c \uc704\ud574 \ub9cc\ub4e0 \ucf54\ub108\ub294?, context: \uc6f9\ud230 \ub9cc\ud654\uac00\ub4e4\uc740 \ub300\uac1c \uc790\uccb4\uc801\uc778 \uc218\uc775 \ucc3d\ucd9c\ubcf4\ub2e4\ub294 \uae09\uc5ec\ub97c \ubc1b\uc73c\uba74\uc11c \uc6d0\uace0\ub97c \uc81c\ucd9c\ud558\ub294 \ubc29\uc2dd\uc73c\ub85c \uc0dd\ud65c\ud558\uace0 \uc788\ub2e4. \uae09\uc5ec \uc81c\uacf5\uc758 \uc8fc\ub41c \uc8fc\uccb4\ub294 \ud3ec\ud138 \uc0ac\uc774\ud2b8\uc774\uba70, \ub300\ud45c\uc801\uc778 \ud3ec\ud138\ub85c\ub294 \ub124\uc774\ubc84\uc640 \ub2e4\uc74c\uc774 \uc788\ub2e4. \uc6f9\ud230 \uc2dc\uc7a5\uc5d0\uc11c \uc791\uac00\ub294 \ud3ec\ud138\uacfc \uacc4\uc57d\uad00\uacc4\ub85c \ub9fa\uc5b4\uc9c0\uba70 \uc2e0\uc778 \uc791\uac00\uc758 \uacbd\uc6b0 \ub300\ubd80\ubd84 \uc778\ud130\ub137 \ud3ec\ud138\uc0ac\uc774\ud2b8\uc758 \u2018\ub098\ub3c4 \ub9cc\ud654\uac00\u2019(\ub2e4\uc74c), \u2018\ub3c4\uc804 \ub9cc\ud654\u2019(\ub124\uc774\ubc84)\ub4f1\uc758 \ucf54\ub108\ub97c \ud1b5\ud574 \ub370\ubdd4\ub97c \ud55c\ub2e4. \uc791\uac00\ub294 \ucf54\ub108\uc5d0 \uc790\uc2e0\uc758 \uc791\ud488\uc744 \uc62c\ub9ac\uace0 \ud3ec\ud138\uc740 \uc870\ud68c\uc218\uac00 \ub9ce\uc740 \uc791\ud488\uc744 \uc120\ubcc4\ud558\uc5ec \uacc4\uc57d\uc744 \ud558\uac8c \ub418\ub294\ub370, \uacc4\uc57d\uc774\ud6c4 \ubaa8\ub2c8\ud130\ub9c1 \uae30\uac04\uc744 \uac00\uc9c0\uac8c \ub41c\ub2e4. \uc774 \uae30\uac04 \ub3d9\uc548 \ud3ec\ud138\uc740 \uc791\ud488\uc758 \ud765\ud589\uc5ec\ubd80, \uc0c1\ud488\uac00\uce58 \uc720\ubb34\ub97c \ud655\uc778\ud558\uace0 \uc791\uac00\uc758 \uc6d0\uace0\ub8cc\ub97c \uc0b0\ucd9c\ud55c\ub2e4. \ub124\uc774\ubc84\uc758 \uacbd\uc6b0 \uc870\ud68c\uc218, \ud68c\ucc28, \ubc30\uacbd\uc720\ubb34, \ubd84\ub7c9 \ub4f1\uc744 \uae30\uc900\uc73c\ub85c \uc6d0\uace0\ub8cc\ub97c \uc0b0\ucd9c\ud558\uba70, \ub2e4\uc74c\uc758 \uacbd\uc6b0 \uc870\ud68c\uc218\ub85c \uc6d0\uace0\ub8cc\ub97c \uc0b0\ucd9c\ud55c\ub2e4. \ub124\uc774\ubc84\uc640 \ub2e4\uc74c\uc758 \uacbd\uc6b0\uc5d0\ub294 \uc218\uc2ed \uba85 \uc815\ub3c4\uc758 \ub9cc\ud654\uac00\ub4e4\uc744 \ud504\ub85c\ub85c\uc368 \uace0\uc6a9, \ub9cc\ud654\ub97c \ubb34\ub8cc\ub85c \ub204\ub9ac\uafbc\ub4e4\uc5d0\uac8c \uc81c\uacf5\ud558\uace0 \uc788\ub2e4. \ub124\uc774\ubc84\uc758 \uc6f9\ud230 \uc11c\ube44\uc2a4\uc758 \ud3c9\uade0 \uc870\ud68c\uc218\ub294 300\ub9cc\uc73c\ub85c \ucd94\uce21\ub41c\ub2e4. \uc778\uae30\uc788\ub294 \uc6f9\ud230\uc758 \uacbd\uc6b0\uc5d0\ub294 \ub313\uae00\uc774 5,000\uac1c\ub97c \ub118\ub294 \ub4f1 \uc6ec\ub9cc\ud55c \uc624\ud504\ub77c\uc778 \ub9cc\ud654\uac00\ub4e4 \ubabb\uc9c0\uc54a\uc740 \uc778\uae30\ub97c \ub204\ub9ac\uace0 \uc788\ub2e4. \ud558\uc9c0\ub9cc \uc6f9\ud230\uc774 \ubb34\ub8cc \ucee8\ud14c\uce20\ub85c \uc11c\ube44\uc2a4 \ub41c\ub2e4\ub294 \uc810\uc5d0\uc11c \uc77c\ubd80 \uc720\uba85 \uc791\uac00\ub97c \uc81c\uc678\ud588\uc744 \ub54c \ub9cc\ud654\uac00\uc5d0\uac8c \uc9c1\uc811\uc801\uc778 \uc218\uc775\uc740 \ud604\uc800\ud788 \ub0ae\uc740 \uac83\uc774 \uc0ac\uc2e4\uc774\ub2e4. \uc2e0\uc778 \uc791\uac00\uac00 \ub124\uc774\ubc84\u00b7\ub2e4\uc74c\u00b7\ud30c\ub780 \ub4f1 \uc8fc\uc694 \uc778\ud130\ub137 \ud3ec\ud138\uc5d0\uc11c \uc5f0\uc7ac\ub97c \uc2dc\uc791\ud560 \uacbd\uc6b0 \ucd5c\uc800 \uace0\ub8cc\ub294 \ud68c\ub2f9 10\ub9cc\u223c20\ub9cc\uc6d0\uc120. \uac01 \ud3ec\ud138\ub9c8\ub2e4 \uc791\uac00\uc758 \uc9c0\uba85\ub3c4\uc640 \uc9c1\uc804 \uc791\ud488\uc758 \uc6d0\uace0\ub8cc, \uc5f0\uc7ac \uc8fc\uae30, \uaddc\ubaa8 \ub4f1\uc758 \ucd94\uac00 \uc694\uc778\uc744 \ud569\uc0b0\ud574 \uc791\ud488 \ub2e8\uc704 \ud639\uc740 3\uac1c\uc6d4, 6\uac1c\uc6d4 \uc8fc\uae30\uc5d0 \ud55c \ubc88\uc529 \uc6d0\uace0\ub8cc\ub97c \uac31\uc2e0\ud558\ub294 \uad6c\uc870\ub2e4. \ucd9c\ud310\uc0ac \uc9c1\uc601\uc0ac\uc774\ud2b8\ub294 \uc720\ub8cc \uc11c\ube44\uc2a4\ub97c \ud1b5\ud574 \uc218\uc775\uc744 \ub0b4\ub294 \uad6c\uc870\uc774\uba70, \ud3ec\ud138\uc0ac\uc774\ud2b8\ub294 \uc11c\ube44\uc2a4\ub9e4\ucd9c \uc774\uc678\uc5d0 \ud310\uad8c\ub8cc \ubc0f \ub7ec\ub2dd\ub85c\uc5f4\ud2f0\ub97c \ubc1b\uac8c \ub41c\ub2e4. \uc778\uae30 \ub9cc\ud654\uc758 \uacbd\uc6b0 \ucd9c\ud310 \ub9cc\ud654, \uce90\ub9ad\ud130 \uc0c1\ud488, \ub4dc\ub77c\ub9c8, \uc601\ud654, \uc5f0\uadf9 \ub4f1\uc758 \ub2e4\uc591\ud55c \uc7a5\ub974\uc758 \ucee8\ud150\uce20\ub85c \ud65c\uc6a9\ub418\uba74\uc11c \uc0c8\ub85c\uc6b4 \uc218\uc775\uc744 \ub0b3\ub294 \uacbd\uc6b0\ub3c4 \uc788\ub2e4. \ub2e8\ud589\ubcf8\uc740 \uc791\ud488\uc774 \uc628\ub77c\uc778\uc73c\ub85c \uac8c\uc7ac\ub41c \ud6c4 \uc120\ubcc4\ub418\uc5b4 \ucd9c\uac04\ub418\uba70 \ucc45\uac12\uc740 \ub300\ub7b5 9\ucc9c\uc6d0 \ub0b4\uc678\ub2e4. \uc774\uc678\uc5d0\ub3c4 \uc791\ud488\uc774 \uc778\uae30\ub97c \uc5bb\uac8c \ub418\uba74 2\ucc28 \uc2dc\uc7a5\uc778 \uc601\ud654\ub098 \ub4dc\ub77c\ub9c8, \uc560\ub2c8\uba54\uc774\uc158 \ub4f1\uc73c\ub85c \ub098\uc624\uae30\ub3c4 \ud558\uba70, 3\ucc28 \uc2dc\uc7a5\uc778 \ubb38\uad6c, \uc644\uad6c, \uc758\ub958 \ub4f1 \uc0c1\ud488\uc73c\ub85c \ub098\uc624\ub294 \uacbd\uc6b0\ub3c4 \uc788\ub2e4."} {"question": "'\uc138\uacc4\uc758 \uad50\ucc28\ub85c' \ub77c\uace0 \ubd88\ub9ac\ub294 \ub274\uc695\uc758 \ub79c\ub4dc\ub9c8\ud06c\ub294?", "paragraph": "\ub274\uc695\uc5d0\ub294 5\ubc88\ub85c\ub97c \ud3ec\ud568\ud55c \uac70\ub9ac, \uc790\uc720\uc758 \uc5ec\uc2e0\uc0c1\uc744 \ud3ec\ud568\ud55c \ub79c\ub4dc\ub9c8\ud06c\uac00 \ub9ce\uc774 \uc788\uc73c\uba70, \uc5f0\uac04 5\ucc9c\ub9cc \uba85\uc758 \uad00\uad11\uac1d\uc774 \ubc29\ubb38\ud55c\ub2e4. \ud0c0\uc784\uc2a4 \uc2a4\ud018\uc5b4\ub294 '\uc138\uacc4\uc758 \uad50\ucc28\ub85c'(The Crossroads of the World)\ub77c\uace0 \ubd88\ub9ac\uace0 \uc788\ub2e4. \ud0c0\uc784\uc2a4 \uc2a4\ud018\uc5b4 \ubd80\uadfc\uc5d0\uc11c\ub294 \ube0c\ub85c\ub4dc\uc6e8\uc774 \uc5f0\uadf9\uc774 \uc0c1\uc5f0\ub418\uba70, \ub274\uc695\uc740 \uc5d4\ud130\ud14c\uc778\uba3c\ud2b8 \uc0b0\uc5c5\uc758 \uc911\uc2ec\uc9c0\ub85c \uc77c\uceec\uc5b4\uc9c4\ub2e4. \uc5e0\ud30c\uc774\uc5b4 \uc2a4\ud14c\uc774\ud2b8 \ube4c\ub529, \ub85d\ud3a0\ub7ec\uc13c\ud130, \ud06c\ub77c\uc774\uc2ac\ub7ec \ube4c\ub529\uc744 \ud3ec\ud568\ud55c \ucd08\uace0\uce35 \uac74\ubb3c, \uc13c\ud2b8\ub7f4 \ud30c\ud06c\ub97c \ud3ec\ud568\ud55c \uacf5\uc6d0, \ube0c\ub8e8\ud074\ub9b0 \ub2e4\ub9ac \ub4f1\uc744 \ud3ec\ud568\ud55c \ub2e4\ub9ac\ub3c4 \ub9ce\uc774 \uc788\ub2e4. \uacbd\uc81c \uc218\ub3c4\ub85c\ub3c4 \ubd88\ub9ac\ub294 \ub274\uc695\uc5d0\ub294 \uc6d4 \uac00\uac00 \uc788\uc73c\uba70, \ub274\uc695 \uc99d\uad8c\uac70\ub798\uc18c(NYSE)\uc640 NASDAQ\uc774 \uc774 \uac70\ub9ac\uc5d0 \uc788\ub2e4. \ub9e8\ud574\ud2bc\uc758 \ubd80\ub3d9\uc0b0 \uc2dc\uc7a5\uc740 \uc138\uacc4\uc5d0\uc11c \uac00\uc7a5 \ube44\uc2f8\ub2e4. \ub274\uc695 \uc9c0\ud558\ucca0\uc740 \uc138\uacc4 \ucd5c\ub300\uc758 \uc9c0\ud558\ucca0\ub9dd \uc911 \ud558\ub098\uc774\uba70, \uceec\ub7fc\ube44\uc544 \ub300\ud559\uad50, \ub274\uc695 \ub300\ud559\uad50\ub97c \ud3ec\ud568\ud55c \uc218\ub9ce\uc740 \ub300\ud559\uad50\ub3c4 \ub274\uc695\uc5d0 \uc788\ub2e4.", "answer": "\ud0c0\uc784\uc2a4 \uc2a4\ud018\uc5b4", "sentence": "\ud0c0\uc784\uc2a4 \uc2a4\ud018\uc5b4 \ub294 '\uc138\uacc4\uc758 \uad50\ucc28\ub85c'(The Crossroads of the World)\ub77c\uace0 \ubd88\ub9ac\uace0 \uc788\ub2e4.", "paragraph_sentence": "\ub274\uc695\uc5d0\ub294 5\ubc88\ub85c\ub97c \ud3ec\ud568\ud55c \uac70\ub9ac, \uc790\uc720\uc758 \uc5ec\uc2e0\uc0c1\uc744 \ud3ec\ud568\ud55c \ub79c\ub4dc\ub9c8\ud06c\uac00 \ub9ce\uc774 \uc788\uc73c\uba70, \uc5f0\uac04 5\ucc9c\ub9cc \uba85\uc758 \uad00\uad11\uac1d\uc774 \ubc29\ubb38\ud55c\ub2e4. \ud0c0\uc784\uc2a4 \uc2a4\ud018\uc5b4 \ub294 '\uc138\uacc4\uc758 \uad50\ucc28\ub85c'(The Crossroads of the World)\ub77c\uace0 \ubd88\ub9ac\uace0 \uc788\ub2e4. \ud0c0\uc784\uc2a4 \uc2a4\ud018\uc5b4 \ubd80\uadfc\uc5d0\uc11c\ub294 \ube0c\ub85c\ub4dc\uc6e8\uc774 \uc5f0\uadf9\uc774 \uc0c1\uc5f0\ub418\uba70, \ub274\uc695\uc740 \uc5d4\ud130\ud14c\uc778\uba3c\ud2b8 \uc0b0\uc5c5\uc758 \uc911\uc2ec\uc9c0\ub85c \uc77c\uceec\uc5b4\uc9c4\ub2e4. \uc5e0\ud30c\uc774\uc5b4 \uc2a4\ud14c\uc774\ud2b8 \ube4c\ub529, \ub85d\ud3a0\ub7ec\uc13c\ud130, \ud06c\ub77c\uc774\uc2ac\ub7ec \ube4c\ub529\uc744 \ud3ec\ud568\ud55c \ucd08\uace0\uce35 \uac74\ubb3c, \uc13c\ud2b8\ub7f4 \ud30c\ud06c\ub97c \ud3ec\ud568\ud55c \uacf5\uc6d0, \ube0c\ub8e8\ud074\ub9b0 \ub2e4\ub9ac \ub4f1\uc744 \ud3ec\ud568\ud55c \ub2e4\ub9ac\ub3c4 \ub9ce\uc774 \uc788\ub2e4. \uacbd\uc81c \uc218\ub3c4\ub85c\ub3c4 \ubd88\ub9ac\ub294 \ub274\uc695\uc5d0\ub294 \uc6d4 \uac00\uac00 \uc788\uc73c\uba70, \ub274\uc695 \uc99d\uad8c\uac70\ub798\uc18c(NYSE)\uc640 NASDAQ\uc774 \uc774 \uac70\ub9ac\uc5d0 \uc788\ub2e4. \ub9e8\ud574\ud2bc\uc758 \ubd80\ub3d9\uc0b0 \uc2dc\uc7a5\uc740 \uc138\uacc4\uc5d0\uc11c \uac00\uc7a5 \ube44\uc2f8\ub2e4. \ub274\uc695 \uc9c0\ud558\ucca0\uc740 \uc138\uacc4 \ucd5c\ub300\uc758 \uc9c0\ud558\ucca0\ub9dd \uc911 \ud558\ub098\uc774\uba70, \uceec\ub7fc\ube44\uc544 \ub300\ud559\uad50, \ub274\uc695 \ub300\ud559\uad50\ub97c \ud3ec\ud568\ud55c \uc218\ub9ce\uc740 \ub300\ud559\uad50\ub3c4 \ub274\uc695\uc5d0 \uc788\ub2e4.", "paragraph_answer": "\ub274\uc695\uc5d0\ub294 5\ubc88\ub85c\ub97c \ud3ec\ud568\ud55c \uac70\ub9ac, \uc790\uc720\uc758 \uc5ec\uc2e0\uc0c1\uc744 \ud3ec\ud568\ud55c \ub79c\ub4dc\ub9c8\ud06c\uac00 \ub9ce\uc774 \uc788\uc73c\uba70, \uc5f0\uac04 5\ucc9c\ub9cc \uba85\uc758 \uad00\uad11\uac1d\uc774 \ubc29\ubb38\ud55c\ub2e4. \ud0c0\uc784\uc2a4 \uc2a4\ud018\uc5b4 \ub294 '\uc138\uacc4\uc758 \uad50\ucc28\ub85c'(The Crossroads of the World)\ub77c\uace0 \ubd88\ub9ac\uace0 \uc788\ub2e4. \ud0c0\uc784\uc2a4 \uc2a4\ud018\uc5b4 \ubd80\uadfc\uc5d0\uc11c\ub294 \ube0c\ub85c\ub4dc\uc6e8\uc774 \uc5f0\uadf9\uc774 \uc0c1\uc5f0\ub418\uba70, \ub274\uc695\uc740 \uc5d4\ud130\ud14c\uc778\uba3c\ud2b8 \uc0b0\uc5c5\uc758 \uc911\uc2ec\uc9c0\ub85c \uc77c\uceec\uc5b4\uc9c4\ub2e4. \uc5e0\ud30c\uc774\uc5b4 \uc2a4\ud14c\uc774\ud2b8 \ube4c\ub529, \ub85d\ud3a0\ub7ec\uc13c\ud130, \ud06c\ub77c\uc774\uc2ac\ub7ec \ube4c\ub529\uc744 \ud3ec\ud568\ud55c \ucd08\uace0\uce35 \uac74\ubb3c, \uc13c\ud2b8\ub7f4 \ud30c\ud06c\ub97c \ud3ec\ud568\ud55c \uacf5\uc6d0, \ube0c\ub8e8\ud074\ub9b0 \ub2e4\ub9ac \ub4f1\uc744 \ud3ec\ud568\ud55c \ub2e4\ub9ac\ub3c4 \ub9ce\uc774 \uc788\ub2e4. \uacbd\uc81c \uc218\ub3c4\ub85c\ub3c4 \ubd88\ub9ac\ub294 \ub274\uc695\uc5d0\ub294 \uc6d4 \uac00\uac00 \uc788\uc73c\uba70, \ub274\uc695 \uc99d\uad8c\uac70\ub798\uc18c(NYSE)\uc640 NASDAQ\uc774 \uc774 \uac70\ub9ac\uc5d0 \uc788\ub2e4. \ub9e8\ud574\ud2bc\uc758 \ubd80\ub3d9\uc0b0 \uc2dc\uc7a5\uc740 \uc138\uacc4\uc5d0\uc11c \uac00\uc7a5 \ube44\uc2f8\ub2e4. \ub274\uc695 \uc9c0\ud558\ucca0\uc740 \uc138\uacc4 \ucd5c\ub300\uc758 \uc9c0\ud558\ucca0\ub9dd \uc911 \ud558\ub098\uc774\uba70, \uceec\ub7fc\ube44\uc544 \ub300\ud559\uad50, \ub274\uc695 \ub300\ud559\uad50\ub97c \ud3ec\ud568\ud55c \uc218\ub9ce\uc740 \ub300\ud559\uad50\ub3c4 \ub274\uc695\uc5d0 \uc788\ub2e4.", "sentence_answer": " \ud0c0\uc784\uc2a4 \uc2a4\ud018\uc5b4 \ub294 '\uc138\uacc4\uc758 \uad50\ucc28\ub85c'(The Crossroads of the World)\ub77c\uace0 \ubd88\ub9ac\uace0 \uc788\ub2e4.", "paragraph_id": "6546073-0-1", "paragraph_question": "question: '\uc138\uacc4\uc758 \uad50\ucc28\ub85c' \ub77c\uace0 \ubd88\ub9ac\ub294 \ub274\uc695\uc758 \ub79c\ub4dc\ub9c8\ud06c\ub294?, context: \ub274\uc695\uc5d0\ub294 5\ubc88\ub85c\ub97c \ud3ec\ud568\ud55c \uac70\ub9ac, \uc790\uc720\uc758 \uc5ec\uc2e0\uc0c1\uc744 \ud3ec\ud568\ud55c \ub79c\ub4dc\ub9c8\ud06c\uac00 \ub9ce\uc774 \uc788\uc73c\uba70, \uc5f0\uac04 5\ucc9c\ub9cc \uba85\uc758 \uad00\uad11\uac1d\uc774 \ubc29\ubb38\ud55c\ub2e4. \ud0c0\uc784\uc2a4 \uc2a4\ud018\uc5b4\ub294 '\uc138\uacc4\uc758 \uad50\ucc28\ub85c'(The Crossroads of the World)\ub77c\uace0 \ubd88\ub9ac\uace0 \uc788\ub2e4. \ud0c0\uc784\uc2a4 \uc2a4\ud018\uc5b4 \ubd80\uadfc\uc5d0\uc11c\ub294 \ube0c\ub85c\ub4dc\uc6e8\uc774 \uc5f0\uadf9\uc774 \uc0c1\uc5f0\ub418\uba70, \ub274\uc695\uc740 \uc5d4\ud130\ud14c\uc778\uba3c\ud2b8 \uc0b0\uc5c5\uc758 \uc911\uc2ec\uc9c0\ub85c \uc77c\uceec\uc5b4\uc9c4\ub2e4. \uc5e0\ud30c\uc774\uc5b4 \uc2a4\ud14c\uc774\ud2b8 \ube4c\ub529, \ub85d\ud3a0\ub7ec\uc13c\ud130, \ud06c\ub77c\uc774\uc2ac\ub7ec \ube4c\ub529\uc744 \ud3ec\ud568\ud55c \ucd08\uace0\uce35 \uac74\ubb3c, \uc13c\ud2b8\ub7f4 \ud30c\ud06c\ub97c \ud3ec\ud568\ud55c \uacf5\uc6d0, \ube0c\ub8e8\ud074\ub9b0 \ub2e4\ub9ac \ub4f1\uc744 \ud3ec\ud568\ud55c \ub2e4\ub9ac\ub3c4 \ub9ce\uc774 \uc788\ub2e4. \uacbd\uc81c \uc218\ub3c4\ub85c\ub3c4 \ubd88\ub9ac\ub294 \ub274\uc695\uc5d0\ub294 \uc6d4 \uac00\uac00 \uc788\uc73c\uba70, \ub274\uc695 \uc99d\uad8c\uac70\ub798\uc18c(NYSE)\uc640 NASDAQ\uc774 \uc774 \uac70\ub9ac\uc5d0 \uc788\ub2e4. \ub9e8\ud574\ud2bc\uc758 \ubd80\ub3d9\uc0b0 \uc2dc\uc7a5\uc740 \uc138\uacc4\uc5d0\uc11c \uac00\uc7a5 \ube44\uc2f8\ub2e4. \ub274\uc695 \uc9c0\ud558\ucca0\uc740 \uc138\uacc4 \ucd5c\ub300\uc758 \uc9c0\ud558\ucca0\ub9dd \uc911 \ud558\ub098\uc774\uba70, \uceec\ub7fc\ube44\uc544 \ub300\ud559\uad50, \ub274\uc695 \ub300\ud559\uad50\ub97c \ud3ec\ud568\ud55c \uc218\ub9ce\uc740 \ub300\ud559\uad50\ub3c4 \ub274\uc695\uc5d0 \uc788\ub2e4."} {"question": "\ucef4\ud4e8\ud130\uc640 \ube44\ub514\uc624 \uac8c\uc784\uc5d0\uc11c \ubba4\ub97c \ud68d\ub4dd\ud560 \uc218 \uc788\ub294 \ubc29\ubc95\uc740?", "paragraph": "\ucca0\uc800\ud788 \uae30\ud68d\ub41c \uc774\ubca4\ud2b8\ub97c \ud1b5\ud55c \ubba4\uc758 \uacf5\uac1c \ubc0f \ubc30\ud3ec\ub294 \ud3ec\ucf13\ubaac \uc2dc\ub9ac\uc988\uc758 \uc77c\ubcf8 \ub0b4 \uc131\uacf5\uc758 \uc8fc \uc694\uc778\uc73c\ub85c \ud3c9\uac00\ub418\uc5c8\ub2e4. \uc77c\ubcf8\uc5d0\uc11c \uc9c4\ud589\ub41c '\uc804\uc124\uc758 \ud3ec\ucf13\ubaac \uc624\ud37c' \uc5d0\uc11c\ub294 \uc560\ucd08 \uc608\uc0c1\ud588\ub358 3,000\uba85\uc744 \ud06c\uac8c \ub118\uc5b4\uc11c\ub294 78,000\uba85 \uc774\uc0c1\uc758 \uc751\ubaa8\uc790\uac00 \ub098\uc654\ub2e4. \ub2cc\ud150\ub3c4\uc758 \uc0ac\uc7a5\uc778 \uc774\uc640\ud0c0 \uc0ac\ud1a0\ub8e8\ub3c4 \uc774 \uac8c\uc784 \uc2dc\ub9ac\uc988\uc758 \uc131\uacf5\uc740 \"\uc804\uc124\uc758 \ud3ec\ucf13\ubaac \uc624\ud37c\" \ub355\ubd84\uc774\uc5c8\ub2e4\uace0 \ubc1d\ud614\ub2e4. \uc774\ubca4\ud2b8 \uc774\ud6c4, \ud3ec\ucf13\ubaac \uc801\ub179\uc758 \uc8fc\uac04 \ud310\ub9e4\ub7c9\uc774 \uc804\uc6d4\uc758 \uc6d4\uac04 \ud310\ub9e4\ub7c9\uacfc \ub3d9\ub4f1\ud55c \uc218\uc900\uc774 \ub418\uc5c8\uc73c\uba70, \uacc4\uc18d \ub298\uc5b4\ub098 3~4\ubc30\uc5d0 \uc774\ub974\uac8c \ub418\uc5c8\ub2e4. \uadf8\ub7ec\ub098, \uac8c\uc784 \uad00\ub828 \uc7a1\uc9c0 '\ucef4\ud4e8\ud130\uc640 \ube44\ub514\uc624\uac8c\uc784'\uc5d0\uc11c\ub294 \ub2cc\ud150\ub3c4 \uc774\ubca4\ud2b8\ub97c \ud1b5\ud574\uc11c\ub9cc \ubba4\ub97c \ud68d\ub4dd\ud560 \uc218 \uc788\ub294 \ubc30\ud0c0\uc131\uc744 \ud3ec\ucf13\ubaac \uc2dc\ub9ac\uc988 \ucd5c\uc545\uc758 \ub2e8\uc810\uc73c\ub85c \uaf3d\uc73c\uba70 \ube44\ud310\ud558\uc600\ub2e4. \ud504\ub85c \uc561\uc158 \ub9ac\ud50c\ub808\uc774\uc640 \uac19\uc740 \uce58\ud2b8 \ud504\ub85c\uadf8\ub7a8\uc744 \uc0ac\uc6a9\ud558\uba74 \uc774\ubca4\ud2b8\uc640 \ubb34\uad00\ud558\uac8c \ubba4\ub97c \uc5bb\uc744 \uc218 \uc788\uc5c8\uae30 \ub54c\ubb38\uc774\uc5c8\ub2e4. \ub9ce\uc740 \ud3ec\ucf13\ubaac \ub9e4\ub2c8\uc544\ub4e4\uc774 \ub2e8\uc9c0 \ubba4\ub97c \uc190\uc5d0 \ub123\uae30 \uc704\ud574 \uce58\ud2b8 \ud504\ub85c\uadf8\ub7a8\uc744 \uad6c\ub9e4\ud558\uc600\ub2e4. UGO.com \uc740 '\uac00\uc7a5 \uba4b\uc9c4 \uc228\uaca8\uc9c4 \uce90\ub9ad\ud130 25' \uc21c\uc704\uc5d0\uc11c \ubba4\ub97c 6\uc704\ub85c \uc120\uc815\ud588\ub2e4. \uc791\uac00 \ud2b8\ub808\uc774\uc2dc \uc6e8\uc2a4\ud2b8\uc640 \uce90\uc11c\ub9b0 \ub180 \ub4f1\uc740 \ubba4\ub97c \ucd5c\uace0\uc758 \uc804\uc124\uc758 \ud3ec\ucf13\ubaac\uc774\ub77c \uce6d\ud558\uc600\uc73c\uba70, \uc804 \ud3ec\ucf13\ubaac \uc911\uc5d0\uc11c\ub3c4 5\uc704\ub85c \uaf3d\uc558\ub2e4.", "answer": "\ub2cc\ud150\ub3c4 \uc774\ubca4\ud2b8", "sentence": "\uadf8\ub7ec\ub098, \uac8c\uc784 \uad00\ub828 \uc7a1\uc9c0 '\ucef4\ud4e8\ud130\uc640 \ube44\ub514\uc624\uac8c\uc784'\uc5d0\uc11c\ub294 \ub2cc\ud150\ub3c4 \uc774\ubca4\ud2b8 \ub97c \ud1b5\ud574\uc11c\ub9cc \ubba4\ub97c \ud68d\ub4dd\ud560 \uc218 \uc788\ub294 \ubc30\ud0c0\uc131\uc744 \ud3ec\ucf13\ubaac \uc2dc\ub9ac\uc988 \ucd5c\uc545\uc758 \ub2e8\uc810\uc73c\ub85c \uaf3d\uc73c\uba70 \ube44\ud310\ud558\uc600\ub2e4.", "paragraph_sentence": "\ucca0\uc800\ud788 \uae30\ud68d\ub41c \uc774\ubca4\ud2b8\ub97c \ud1b5\ud55c \ubba4\uc758 \uacf5\uac1c \ubc0f \ubc30\ud3ec\ub294 \ud3ec\ucf13\ubaac \uc2dc\ub9ac\uc988\uc758 \uc77c\ubcf8 \ub0b4 \uc131\uacf5\uc758 \uc8fc \uc694\uc778\uc73c\ub85c \ud3c9\uac00\ub418\uc5c8\ub2e4. \uc77c\ubcf8\uc5d0\uc11c \uc9c4\ud589\ub41c '\uc804\uc124\uc758 \ud3ec\ucf13\ubaac \uc624\ud37c' \uc5d0\uc11c\ub294 \uc560\ucd08 \uc608\uc0c1\ud588\ub358 3,000\uba85\uc744 \ud06c\uac8c \ub118\uc5b4\uc11c\ub294 78,000\uba85 \uc774\uc0c1\uc758 \uc751\ubaa8\uc790\uac00 \ub098\uc654\ub2e4. \ub2cc\ud150\ub3c4\uc758 \uc0ac\uc7a5\uc778 \uc774\uc640\ud0c0 \uc0ac\ud1a0\ub8e8\ub3c4 \uc774 \uac8c\uc784 \uc2dc\ub9ac\uc988\uc758 \uc131\uacf5\uc740 \"\uc804\uc124\uc758 \ud3ec\ucf13\ubaac \uc624\ud37c\" \ub355\ubd84\uc774\uc5c8\ub2e4\uace0 \ubc1d\ud614\ub2e4. \uc774\ubca4\ud2b8 \uc774\ud6c4, \ud3ec\ucf13\ubaac \uc801\ub179\uc758 \uc8fc\uac04 \ud310\ub9e4\ub7c9\uc774 \uc804\uc6d4\uc758 \uc6d4\uac04 \ud310\ub9e4\ub7c9\uacfc \ub3d9\ub4f1\ud55c \uc218\uc900\uc774 \ub418\uc5c8\uc73c\uba70, \uacc4\uc18d \ub298\uc5b4\ub098 3~4\ubc30\uc5d0 \uc774\ub974\uac8c \ub418\uc5c8\ub2e4. \uadf8\ub7ec\ub098, \uac8c\uc784 \uad00\ub828 \uc7a1\uc9c0 '\ucef4\ud4e8\ud130\uc640 \ube44\ub514\uc624\uac8c\uc784'\uc5d0\uc11c\ub294 \ub2cc\ud150\ub3c4 \uc774\ubca4\ud2b8 \ub97c \ud1b5\ud574\uc11c\ub9cc \ubba4\ub97c \ud68d\ub4dd\ud560 \uc218 \uc788\ub294 \ubc30\ud0c0\uc131\uc744 \ud3ec\ucf13\ubaac \uc2dc\ub9ac\uc988 \ucd5c\uc545\uc758 \ub2e8\uc810\uc73c\ub85c \uaf3d\uc73c\uba70 \ube44\ud310\ud558\uc600\ub2e4. \ud504\ub85c \uc561\uc158 \ub9ac\ud50c\ub808\uc774\uc640 \uac19\uc740 \uce58\ud2b8 \ud504\ub85c\uadf8\ub7a8\uc744 \uc0ac\uc6a9\ud558\uba74 \uc774\ubca4\ud2b8\uc640 \ubb34\uad00\ud558\uac8c \ubba4\ub97c \uc5bb\uc744 \uc218 \uc788\uc5c8\uae30 \ub54c\ubb38\uc774\uc5c8\ub2e4. \ub9ce\uc740 \ud3ec\ucf13\ubaac \ub9e4\ub2c8\uc544\ub4e4\uc774 \ub2e8\uc9c0 \ubba4\ub97c \uc190\uc5d0 \ub123\uae30 \uc704\ud574 \uce58\ud2b8 \ud504\ub85c\uadf8\ub7a8\uc744 \uad6c\ub9e4\ud558\uc600\ub2e4. UGO.com \uc740 '\uac00\uc7a5 \uba4b\uc9c4 \uc228\uaca8\uc9c4 \uce90\ub9ad\ud130 25' \uc21c\uc704\uc5d0\uc11c \ubba4\ub97c 6\uc704\ub85c \uc120\uc815\ud588\ub2e4. \uc791\uac00 \ud2b8\ub808\uc774\uc2dc \uc6e8\uc2a4\ud2b8\uc640 \uce90\uc11c\ub9b0 \ub180 \ub4f1\uc740 \ubba4\ub97c \ucd5c\uace0\uc758 \uc804\uc124\uc758 \ud3ec\ucf13\ubaac\uc774\ub77c \uce6d\ud558\uc600\uc73c\uba70, \uc804 \ud3ec\ucf13\ubaac \uc911\uc5d0\uc11c\ub3c4 5\uc704\ub85c \uaf3d\uc558\ub2e4.", "paragraph_answer": "\ucca0\uc800\ud788 \uae30\ud68d\ub41c \uc774\ubca4\ud2b8\ub97c \ud1b5\ud55c \ubba4\uc758 \uacf5\uac1c \ubc0f \ubc30\ud3ec\ub294 \ud3ec\ucf13\ubaac \uc2dc\ub9ac\uc988\uc758 \uc77c\ubcf8 \ub0b4 \uc131\uacf5\uc758 \uc8fc \uc694\uc778\uc73c\ub85c \ud3c9\uac00\ub418\uc5c8\ub2e4. \uc77c\ubcf8\uc5d0\uc11c \uc9c4\ud589\ub41c '\uc804\uc124\uc758 \ud3ec\ucf13\ubaac \uc624\ud37c' \uc5d0\uc11c\ub294 \uc560\ucd08 \uc608\uc0c1\ud588\ub358 3,000\uba85\uc744 \ud06c\uac8c \ub118\uc5b4\uc11c\ub294 78,000\uba85 \uc774\uc0c1\uc758 \uc751\ubaa8\uc790\uac00 \ub098\uc654\ub2e4. \ub2cc\ud150\ub3c4\uc758 \uc0ac\uc7a5\uc778 \uc774\uc640\ud0c0 \uc0ac\ud1a0\ub8e8\ub3c4 \uc774 \uac8c\uc784 \uc2dc\ub9ac\uc988\uc758 \uc131\uacf5\uc740 \"\uc804\uc124\uc758 \ud3ec\ucf13\ubaac \uc624\ud37c\" \ub355\ubd84\uc774\uc5c8\ub2e4\uace0 \ubc1d\ud614\ub2e4. \uc774\ubca4\ud2b8 \uc774\ud6c4, \ud3ec\ucf13\ubaac \uc801\ub179\uc758 \uc8fc\uac04 \ud310\ub9e4\ub7c9\uc774 \uc804\uc6d4\uc758 \uc6d4\uac04 \ud310\ub9e4\ub7c9\uacfc \ub3d9\ub4f1\ud55c \uc218\uc900\uc774 \ub418\uc5c8\uc73c\uba70, \uacc4\uc18d \ub298\uc5b4\ub098 3~4\ubc30\uc5d0 \uc774\ub974\uac8c \ub418\uc5c8\ub2e4. \uadf8\ub7ec\ub098, \uac8c\uc784 \uad00\ub828 \uc7a1\uc9c0 '\ucef4\ud4e8\ud130\uc640 \ube44\ub514\uc624\uac8c\uc784'\uc5d0\uc11c\ub294 \ub2cc\ud150\ub3c4 \uc774\ubca4\ud2b8 \ub97c \ud1b5\ud574\uc11c\ub9cc \ubba4\ub97c \ud68d\ub4dd\ud560 \uc218 \uc788\ub294 \ubc30\ud0c0\uc131\uc744 \ud3ec\ucf13\ubaac \uc2dc\ub9ac\uc988 \ucd5c\uc545\uc758 \ub2e8\uc810\uc73c\ub85c \uaf3d\uc73c\uba70 \ube44\ud310\ud558\uc600\ub2e4. \ud504\ub85c \uc561\uc158 \ub9ac\ud50c\ub808\uc774\uc640 \uac19\uc740 \uce58\ud2b8 \ud504\ub85c\uadf8\ub7a8\uc744 \uc0ac\uc6a9\ud558\uba74 \uc774\ubca4\ud2b8\uc640 \ubb34\uad00\ud558\uac8c \ubba4\ub97c \uc5bb\uc744 \uc218 \uc788\uc5c8\uae30 \ub54c\ubb38\uc774\uc5c8\ub2e4. \ub9ce\uc740 \ud3ec\ucf13\ubaac \ub9e4\ub2c8\uc544\ub4e4\uc774 \ub2e8\uc9c0 \ubba4\ub97c \uc190\uc5d0 \ub123\uae30 \uc704\ud574 \uce58\ud2b8 \ud504\ub85c\uadf8\ub7a8\uc744 \uad6c\ub9e4\ud558\uc600\ub2e4. UGO.com \uc740 '\uac00\uc7a5 \uba4b\uc9c4 \uc228\uaca8\uc9c4 \uce90\ub9ad\ud130 25' \uc21c\uc704\uc5d0\uc11c \ubba4\ub97c 6\uc704\ub85c \uc120\uc815\ud588\ub2e4. \uc791\uac00 \ud2b8\ub808\uc774\uc2dc \uc6e8\uc2a4\ud2b8\uc640 \uce90\uc11c\ub9b0 \ub180 \ub4f1\uc740 \ubba4\ub97c \ucd5c\uace0\uc758 \uc804\uc124\uc758 \ud3ec\ucf13\ubaac\uc774\ub77c \uce6d\ud558\uc600\uc73c\uba70, \uc804 \ud3ec\ucf13\ubaac \uc911\uc5d0\uc11c\ub3c4 5\uc704\ub85c \uaf3d\uc558\ub2e4.", "sentence_answer": "\uadf8\ub7ec\ub098, \uac8c\uc784 \uad00\ub828 \uc7a1\uc9c0 '\ucef4\ud4e8\ud130\uc640 \ube44\ub514\uc624\uac8c\uc784'\uc5d0\uc11c\ub294 \ub2cc\ud150\ub3c4 \uc774\ubca4\ud2b8 \ub97c \ud1b5\ud574\uc11c\ub9cc \ubba4\ub97c \ud68d\ub4dd\ud560 \uc218 \uc788\ub294 \ubc30\ud0c0\uc131\uc744 \ud3ec\ucf13\ubaac \uc2dc\ub9ac\uc988 \ucd5c\uc545\uc758 \ub2e8\uc810\uc73c\ub85c \uaf3d\uc73c\uba70 \ube44\ud310\ud558\uc600\ub2e4.", "paragraph_id": "6514803-4-0", "paragraph_question": "question: \ucef4\ud4e8\ud130\uc640 \ube44\ub514\uc624 \uac8c\uc784\uc5d0\uc11c \ubba4\ub97c \ud68d\ub4dd\ud560 \uc218 \uc788\ub294 \ubc29\ubc95\uc740?, context: \ucca0\uc800\ud788 \uae30\ud68d\ub41c \uc774\ubca4\ud2b8\ub97c \ud1b5\ud55c \ubba4\uc758 \uacf5\uac1c \ubc0f \ubc30\ud3ec\ub294 \ud3ec\ucf13\ubaac \uc2dc\ub9ac\uc988\uc758 \uc77c\ubcf8 \ub0b4 \uc131\uacf5\uc758 \uc8fc \uc694\uc778\uc73c\ub85c \ud3c9\uac00\ub418\uc5c8\ub2e4. \uc77c\ubcf8\uc5d0\uc11c \uc9c4\ud589\ub41c '\uc804\uc124\uc758 \ud3ec\ucf13\ubaac \uc624\ud37c' \uc5d0\uc11c\ub294 \uc560\ucd08 \uc608\uc0c1\ud588\ub358 3,000\uba85\uc744 \ud06c\uac8c \ub118\uc5b4\uc11c\ub294 78,000\uba85 \uc774\uc0c1\uc758 \uc751\ubaa8\uc790\uac00 \ub098\uc654\ub2e4. \ub2cc\ud150\ub3c4\uc758 \uc0ac\uc7a5\uc778 \uc774\uc640\ud0c0 \uc0ac\ud1a0\ub8e8\ub3c4 \uc774 \uac8c\uc784 \uc2dc\ub9ac\uc988\uc758 \uc131\uacf5\uc740 \"\uc804\uc124\uc758 \ud3ec\ucf13\ubaac \uc624\ud37c\" \ub355\ubd84\uc774\uc5c8\ub2e4\uace0 \ubc1d\ud614\ub2e4. \uc774\ubca4\ud2b8 \uc774\ud6c4, \ud3ec\ucf13\ubaac \uc801\ub179\uc758 \uc8fc\uac04 \ud310\ub9e4\ub7c9\uc774 \uc804\uc6d4\uc758 \uc6d4\uac04 \ud310\ub9e4\ub7c9\uacfc \ub3d9\ub4f1\ud55c \uc218\uc900\uc774 \ub418\uc5c8\uc73c\uba70, \uacc4\uc18d \ub298\uc5b4\ub098 3~4\ubc30\uc5d0 \uc774\ub974\uac8c \ub418\uc5c8\ub2e4. \uadf8\ub7ec\ub098, \uac8c\uc784 \uad00\ub828 \uc7a1\uc9c0 '\ucef4\ud4e8\ud130\uc640 \ube44\ub514\uc624\uac8c\uc784'\uc5d0\uc11c\ub294 \ub2cc\ud150\ub3c4 \uc774\ubca4\ud2b8\ub97c \ud1b5\ud574\uc11c\ub9cc \ubba4\ub97c \ud68d\ub4dd\ud560 \uc218 \uc788\ub294 \ubc30\ud0c0\uc131\uc744 \ud3ec\ucf13\ubaac \uc2dc\ub9ac\uc988 \ucd5c\uc545\uc758 \ub2e8\uc810\uc73c\ub85c \uaf3d\uc73c\uba70 \ube44\ud310\ud558\uc600\ub2e4. \ud504\ub85c \uc561\uc158 \ub9ac\ud50c\ub808\uc774\uc640 \uac19\uc740 \uce58\ud2b8 \ud504\ub85c\uadf8\ub7a8\uc744 \uc0ac\uc6a9\ud558\uba74 \uc774\ubca4\ud2b8\uc640 \ubb34\uad00\ud558\uac8c \ubba4\ub97c \uc5bb\uc744 \uc218 \uc788\uc5c8\uae30 \ub54c\ubb38\uc774\uc5c8\ub2e4. \ub9ce\uc740 \ud3ec\ucf13\ubaac \ub9e4\ub2c8\uc544\ub4e4\uc774 \ub2e8\uc9c0 \ubba4\ub97c \uc190\uc5d0 \ub123\uae30 \uc704\ud574 \uce58\ud2b8 \ud504\ub85c\uadf8\ub7a8\uc744 \uad6c\ub9e4\ud558\uc600\ub2e4. UGO.com \uc740 '\uac00\uc7a5 \uba4b\uc9c4 \uc228\uaca8\uc9c4 \uce90\ub9ad\ud130 25' \uc21c\uc704\uc5d0\uc11c \ubba4\ub97c 6\uc704\ub85c \uc120\uc815\ud588\ub2e4. \uc791\uac00 \ud2b8\ub808\uc774\uc2dc \uc6e8\uc2a4\ud2b8\uc640 \uce90\uc11c\ub9b0 \ub180 \ub4f1\uc740 \ubba4\ub97c \ucd5c\uace0\uc758 \uc804\uc124\uc758 \ud3ec\ucf13\ubaac\uc774\ub77c \uce6d\ud558\uc600\uc73c\uba70, \uc804 \ud3ec\ucf13\ubaac \uc911\uc5d0\uc11c\ub3c4 5\uc704\ub85c \uaf3d\uc558\ub2e4."} {"question": "\ub9e5\ud30c\ub97c\ub79c\ub4dc\ub294 \uc624\ucc28\ub4dc\uc758 \ubb34\uc5c7\uc744 \uc774\uc6a9\ud574 WMF \uc9c0\ub3c4\ubd80\uc5d0 \ub204\uba85\uc744 \ub4a4\uc9d1\uc5b4\uc50c\uc6b8 \uc218 \uc788\uc5c8\ub098?", "paragraph": "\ub9e5\ud30c\ub97c\ub79c\ub4dc\ub294 \uc624\ucc28\ub4dc\uc758 \uc704\uc99d \uc790\ubc31\uc11c\ub97c \uc774\uc6a9\ud574 WFM \uc9c0\ub3c4\ubd80\uac00 \uc288\ud1a0\ub128\ubc84\uadf8 \ubaa8\uc0b4 \ub2f9\uc2dc \uadf8 \ud604\uc7a5\uc5d0 \uc788\uc5c8\ub2e4\ub294 \ub204\uba85\uc744 \ub4a4\uc9d1\uc5b4\uc50c\uc6e0\ub2e4. \uadf8\ub9ac\uace0 \uc8fc \uacbd\uacc4\ub97c \ub118\uc5b4 \ucf5c\ub85c\ub77c\ub3c4 \uc8fc \ub374\ubc84 \uc2dc\uc5d0\uc11c \ud5e4\uc774\uc6b0\ub4dc, \ubaa8\uc774\uc5b4, \uc870\uc9c0 \ud504\ud2f0\ubcf8 3\uc778\uc744 \uccb4\ud3ec\ud588\ub2e4. 1906\ub144 2\uc6d4 17\uc77c, \ub9e5\ud30c\ub97c\ub79c\ub4dc\ub294 3\uc778\uc744 \ud2b9\uc218 \uc5f4\ucc28\uc5d0 \ud0dc\uc6cc \ub374\ubc84 \uc2dc\ubc95\uc6d0\uc774 \uac1c\uc785\ud558\uae30 \uc804\uc5d0 \uc544\uc774\ub2e4\ud638\ub85c \ube7c\ub3cc\ub838\ub2e4. \ub178\uc870 \uc870\uc9c1\uac00\ub4e4\uc740 \uc774 \uacfc\uc815\uc744 \ub0a9\uce58 \uacc4\ud68d\uc73c\ub85c \uac04\uc8fc\ud588\ub2e4. \uc774 \uc720\uad34\ud589\uc704\ub294 \ub108\ubb34 \ud130\ubb34\ub2c8\uc5c6\uc5b4\uc11c WFM\uacfc \uc804\ud600 \uc0ac\uc774\uac00 \uc88b\uc9c0 \uc54a\uc740 \ubbf8\uad6d\ub178\ucd1d(AFL) \ucd1d\uc7ac \uc0c8\ubba4\uc5bc \uacf0\ud37c\uc2a4\uac00 \uc790\uae30 \uc870\uc9c1\uc5d0 \ubcc0\ud638\ub97c \uc704\ud55c \uae30\uae08\uc744 \ubaa8\uc744 \uac83\uc744 \uc9c0\uc2dc\ud588\uc744 \uc815\ub3c4\uc600\ub2e4. \ub300\ubc95\uc6d0\uc5d0 \uccad\uc6d0\ud55c \uc778\uc2e0\ubcf4\ud638\ub294 \uae30\uac01\ub418\uc5c8\ub2e4. \uc774\uc5d0 \ubc18\ub300\ud55c \uc0ac\ub78c\uc740 \uc870\uc9c0\ud504 \ub9e5\ucf00\ub098 \ub300\ubc95\uad00 \ubfd0\uc774\uc5c8\ub2e4.", "answer": "\uc704\uc99d \uc790\ubc31\uc11c", "sentence": "\ub9e5\ud30c\ub97c\ub79c\ub4dc\ub294 \uc624\ucc28\ub4dc\uc758 \uc704\uc99d \uc790\ubc31\uc11c \ub97c \uc774\uc6a9\ud574 WFM \uc9c0\ub3c4\ubd80\uac00 \uc288\ud1a0\ub128\ubc84\uadf8 \ubaa8\uc0b4 \ub2f9\uc2dc \uadf8 \ud604\uc7a5\uc5d0 \uc788\uc5c8\ub2e4\ub294 \ub204\uba85\uc744 \ub4a4\uc9d1\uc5b4\uc50c\uc6e0\ub2e4.", "paragraph_sentence": " \ub9e5\ud30c\ub97c\ub79c\ub4dc\ub294 \uc624\ucc28\ub4dc\uc758 \uc704\uc99d \uc790\ubc31\uc11c \ub97c \uc774\uc6a9\ud574 WFM \uc9c0\ub3c4\ubd80\uac00 \uc288\ud1a0\ub128\ubc84\uadf8 \ubaa8\uc0b4 \ub2f9\uc2dc \uadf8 \ud604\uc7a5\uc5d0 \uc788\uc5c8\ub2e4\ub294 \ub204\uba85\uc744 \ub4a4\uc9d1\uc5b4\uc50c\uc6e0\ub2e4. \uadf8\ub9ac\uace0 \uc8fc \uacbd\uacc4\ub97c \ub118\uc5b4 \ucf5c\ub85c\ub77c\ub3c4 \uc8fc \ub374\ubc84 \uc2dc\uc5d0\uc11c \ud5e4\uc774\uc6b0\ub4dc, \ubaa8\uc774\uc5b4, \uc870\uc9c0 \ud504\ud2f0\ubcf8 3\uc778\uc744 \uccb4\ud3ec\ud588\ub2e4. 1906\ub144 2\uc6d4 17\uc77c, \ub9e5\ud30c\ub97c\ub79c\ub4dc\ub294 3\uc778\uc744 \ud2b9\uc218 \uc5f4\ucc28\uc5d0 \ud0dc\uc6cc \ub374\ubc84 \uc2dc\ubc95\uc6d0\uc774 \uac1c\uc785\ud558\uae30 \uc804\uc5d0 \uc544\uc774\ub2e4\ud638\ub85c \ube7c\ub3cc\ub838\ub2e4. \ub178\uc870 \uc870\uc9c1\uac00\ub4e4\uc740 \uc774 \uacfc\uc815\uc744 \ub0a9\uce58 \uacc4\ud68d\uc73c\ub85c \uac04\uc8fc\ud588\ub2e4. \uc774 \uc720\uad34\ud589\uc704\ub294 \ub108\ubb34 \ud130\ubb34\ub2c8\uc5c6\uc5b4\uc11c WFM\uacfc \uc804\ud600 \uc0ac\uc774\uac00 \uc88b\uc9c0 \uc54a\uc740 \ubbf8\uad6d\ub178\ucd1d(AFL) \ucd1d\uc7ac \uc0c8\ubba4\uc5bc \uacf0\ud37c\uc2a4\uac00 \uc790\uae30 \uc870\uc9c1\uc5d0 \ubcc0\ud638\ub97c \uc704\ud55c \uae30\uae08\uc744 \ubaa8\uc744 \uac83\uc744 \uc9c0\uc2dc\ud588\uc744 \uc815\ub3c4\uc600\ub2e4. \ub300\ubc95\uc6d0\uc5d0 \uccad\uc6d0\ud55c \uc778\uc2e0\ubcf4\ud638\ub294 \uae30\uac01\ub418\uc5c8\ub2e4. \uc774\uc5d0 \ubc18\ub300\ud55c \uc0ac\ub78c\uc740 \uc870\uc9c0\ud504 \ub9e5\ucf00\ub098 \ub300\ubc95\uad00 \ubfd0\uc774\uc5c8\ub2e4.", "paragraph_answer": "\ub9e5\ud30c\ub97c\ub79c\ub4dc\ub294 \uc624\ucc28\ub4dc\uc758 \uc704\uc99d \uc790\ubc31\uc11c \ub97c \uc774\uc6a9\ud574 WFM \uc9c0\ub3c4\ubd80\uac00 \uc288\ud1a0\ub128\ubc84\uadf8 \ubaa8\uc0b4 \ub2f9\uc2dc \uadf8 \ud604\uc7a5\uc5d0 \uc788\uc5c8\ub2e4\ub294 \ub204\uba85\uc744 \ub4a4\uc9d1\uc5b4\uc50c\uc6e0\ub2e4. \uadf8\ub9ac\uace0 \uc8fc \uacbd\uacc4\ub97c \ub118\uc5b4 \ucf5c\ub85c\ub77c\ub3c4 \uc8fc \ub374\ubc84 \uc2dc\uc5d0\uc11c \ud5e4\uc774\uc6b0\ub4dc, \ubaa8\uc774\uc5b4, \uc870\uc9c0 \ud504\ud2f0\ubcf8 3\uc778\uc744 \uccb4\ud3ec\ud588\ub2e4. 1906\ub144 2\uc6d4 17\uc77c, \ub9e5\ud30c\ub97c\ub79c\ub4dc\ub294 3\uc778\uc744 \ud2b9\uc218 \uc5f4\ucc28\uc5d0 \ud0dc\uc6cc \ub374\ubc84 \uc2dc\ubc95\uc6d0\uc774 \uac1c\uc785\ud558\uae30 \uc804\uc5d0 \uc544\uc774\ub2e4\ud638\ub85c \ube7c\ub3cc\ub838\ub2e4. \ub178\uc870 \uc870\uc9c1\uac00\ub4e4\uc740 \uc774 \uacfc\uc815\uc744 \ub0a9\uce58 \uacc4\ud68d\uc73c\ub85c \uac04\uc8fc\ud588\ub2e4. \uc774 \uc720\uad34\ud589\uc704\ub294 \ub108\ubb34 \ud130\ubb34\ub2c8\uc5c6\uc5b4\uc11c WFM\uacfc \uc804\ud600 \uc0ac\uc774\uac00 \uc88b\uc9c0 \uc54a\uc740 \ubbf8\uad6d\ub178\ucd1d(AFL) \ucd1d\uc7ac \uc0c8\ubba4\uc5bc \uacf0\ud37c\uc2a4\uac00 \uc790\uae30 \uc870\uc9c1\uc5d0 \ubcc0\ud638\ub97c \uc704\ud55c \uae30\uae08\uc744 \ubaa8\uc744 \uac83\uc744 \uc9c0\uc2dc\ud588\uc744 \uc815\ub3c4\uc600\ub2e4. \ub300\ubc95\uc6d0\uc5d0 \uccad\uc6d0\ud55c \uc778\uc2e0\ubcf4\ud638\ub294 \uae30\uac01\ub418\uc5c8\ub2e4. \uc774\uc5d0 \ubc18\ub300\ud55c \uc0ac\ub78c\uc740 \uc870\uc9c0\ud504 \ub9e5\ucf00\ub098 \ub300\ubc95\uad00 \ubfd0\uc774\uc5c8\ub2e4.", "sentence_answer": "\ub9e5\ud30c\ub97c\ub79c\ub4dc\ub294 \uc624\ucc28\ub4dc\uc758 \uc704\uc99d \uc790\ubc31\uc11c \ub97c \uc774\uc6a9\ud574 WFM \uc9c0\ub3c4\ubd80\uac00 \uc288\ud1a0\ub128\ubc84\uadf8 \ubaa8\uc0b4 \ub2f9\uc2dc \uadf8 \ud604\uc7a5\uc5d0 \uc788\uc5c8\ub2e4\ub294 \ub204\uba85\uc744 \ub4a4\uc9d1\uc5b4\uc50c\uc6e0\ub2e4.", "paragraph_id": "6657171-3-0", "paragraph_question": "question: \ub9e5\ud30c\ub97c\ub79c\ub4dc\ub294 \uc624\ucc28\ub4dc\uc758 \ubb34\uc5c7\uc744 \uc774\uc6a9\ud574 WMF \uc9c0\ub3c4\ubd80\uc5d0 \ub204\uba85\uc744 \ub4a4\uc9d1\uc5b4\uc50c\uc6b8 \uc218 \uc788\uc5c8\ub098?, context: \ub9e5\ud30c\ub97c\ub79c\ub4dc\ub294 \uc624\ucc28\ub4dc\uc758 \uc704\uc99d \uc790\ubc31\uc11c\ub97c \uc774\uc6a9\ud574 WFM \uc9c0\ub3c4\ubd80\uac00 \uc288\ud1a0\ub128\ubc84\uadf8 \ubaa8\uc0b4 \ub2f9\uc2dc \uadf8 \ud604\uc7a5\uc5d0 \uc788\uc5c8\ub2e4\ub294 \ub204\uba85\uc744 \ub4a4\uc9d1\uc5b4\uc50c\uc6e0\ub2e4. \uadf8\ub9ac\uace0 \uc8fc \uacbd\uacc4\ub97c \ub118\uc5b4 \ucf5c\ub85c\ub77c\ub3c4 \uc8fc \ub374\ubc84 \uc2dc\uc5d0\uc11c \ud5e4\uc774\uc6b0\ub4dc, \ubaa8\uc774\uc5b4, \uc870\uc9c0 \ud504\ud2f0\ubcf8 3\uc778\uc744 \uccb4\ud3ec\ud588\ub2e4. 1906\ub144 2\uc6d4 17\uc77c, \ub9e5\ud30c\ub97c\ub79c\ub4dc\ub294 3\uc778\uc744 \ud2b9\uc218 \uc5f4\ucc28\uc5d0 \ud0dc\uc6cc \ub374\ubc84 \uc2dc\ubc95\uc6d0\uc774 \uac1c\uc785\ud558\uae30 \uc804\uc5d0 \uc544\uc774\ub2e4\ud638\ub85c \ube7c\ub3cc\ub838\ub2e4. \ub178\uc870 \uc870\uc9c1\uac00\ub4e4\uc740 \uc774 \uacfc\uc815\uc744 \ub0a9\uce58 \uacc4\ud68d\uc73c\ub85c \uac04\uc8fc\ud588\ub2e4. \uc774 \uc720\uad34\ud589\uc704\ub294 \ub108\ubb34 \ud130\ubb34\ub2c8\uc5c6\uc5b4\uc11c WFM\uacfc \uc804\ud600 \uc0ac\uc774\uac00 \uc88b\uc9c0 \uc54a\uc740 \ubbf8\uad6d\ub178\ucd1d(AFL) \ucd1d\uc7ac \uc0c8\ubba4\uc5bc \uacf0\ud37c\uc2a4\uac00 \uc790\uae30 \uc870\uc9c1\uc5d0 \ubcc0\ud638\ub97c \uc704\ud55c \uae30\uae08\uc744 \ubaa8\uc744 \uac83\uc744 \uc9c0\uc2dc\ud588\uc744 \uc815\ub3c4\uc600\ub2e4. \ub300\ubc95\uc6d0\uc5d0 \uccad\uc6d0\ud55c \uc778\uc2e0\ubcf4\ud638\ub294 \uae30\uac01\ub418\uc5c8\ub2e4. \uc774\uc5d0 \ubc18\ub300\ud55c \uc0ac\ub78c\uc740 \uc870\uc9c0\ud504 \ub9e5\ucf00\ub098 \ub300\ubc95\uad00 \ubfd0\uc774\uc5c8\ub2e4."} {"question": "\uace0\ub300\uc758 \uc9c4\ub9ac\uac00 \uc218\uc138\uae30\ub97c \uac70\uce58\uba74\uc11c \ubaa8\ubc29\uc790\ub4e4\uacfc \uc8fc\uc11d\uc790\ub4e4\uc5d0 \uc758\ud574 \uc190\uc0c1\ub418\uc5c8\ub2e4\uace0 \uc0dd\uac01\ud55c \uc778\ubb38\uc8fc\uc758\uc790\ub4e4\uc758 \uc601\ud5a5\uc774\ub77c\uace0 \ub9d0\ud55c \uc0ac\ub78c\uc740 \ub204\uad6c\uc778\uac00?", "paragraph": "\ud55c\ud3b8, \ucf00\ud50c\ub7ec\ub294 \ub2e4\ub978 16 ~ 17\uc138\uae30\uc758 \ucc9c\ubb38\ud559\uc790\ub4e4\ucc98\ub7fc \uadfc\ub300\uc5d0 \ube44\ud574 \uace0\ub300\uc758 \ucc9c\ubb38\ud559\uc774 \uc870\uc7a1\ud558\ub2e4\ub294 \uac83\uc744 \uac15\uc870\ud588\ub294\ub370, \uc2a4\ud2f0\ube10 \uc0e4\ud540\uc740 \uc774\uac83\uc774 \uace0\ub300\uc758 \uc9c4\ub9ac\uac00 \uc218\uc138\uae30\ub97c \uac70\uce58\uba74\uc11c \ubaa8\ubc29\uc790\ub4e4\uacfc \uc8fc\uc11d\uc790\ub4e4\uc5d0 \uc758\ud574 \uc190\uc0c1\ub418\uc5c8\ub2e4\uace0 \uc0dd\uac01\ud55c \uc778\ubb38\uc8fc\uc758\uc758 \uc601\ud5a5\uc774\ub77c\uace0 \ub9d0\ud55c\ub2e4. \uc0e4\ud540\uc5d0 \uc758\ud558\uba74 \ucf00\ud50c\ub7ec\uc758 \uc774\ub7f0 \uc0ac\uc0c1\uc740 \u300a\ub8e8\ub3cc\ud504 \ud45c\u300b\uc758 \uad8c\ub450\ud654\ub85c \uc0ac\uc6a9\ub41c \uc720\uba85\ud55c \ud310\ud654 \uc791\ud488\uc5d0\uc11c\ub3c4 \uc798 \ub4dc\ub7ec\ub098\ub294\ub370, \ud310\ud654\uc5d0\uc11c \ucc9c\ubb38\ud559\uc758 \uc5ec\uc2e0\uc758 \uc2e0\uc804\uc758 \ub4a4\ucabd\uc5d0\ub294 \ud22c\ubc15\ud558\uace0 \uad6c\uc870\uc801\uc778 \uc591\uc2dd\uc758 \uae30\ub465\uc774 \uace0\ub300 \ucc9c\ubb38\ud559\uc758 \uc870\uc7a1\ud568\uc744 \ub4dc\ub7ec\ub0b4\uace0 \uc788\uace0, \uc2e0\uc804\uc758 \uc55e\uc5d0 \uc788\ub294 \ucf54\ud398\ub974\ub2c8\ucfe0\uc2a4\uc640 \ud280\ucf54\ub294 \uc774\uc624\ub2c8\uc544\uc2dd\u00b7\ucf54\ub9b0\ud2b8\uc2dd\uc758 \uc6b0\uc544\ud55c \uae30\ub465 \uc606\uc5d0 \uc11c \uc788\ub2e4. \ucf00\ud50c\ub7ec\ub294 \uc778\ubb38\uc8fc\uc758\uc790\uba74\uc11c\ub3c4 \uc5f4\uad11\uc801\uc778 \ud50c\ub77c\ud1a4\uc8fc\uc758\uc790\uc600\ub294\ub370, \uadf8\uc758 \ucd5c\ucd08\uc758 \ud559\uc124\uc778 \u2018\ucf54\uc2a4\ubaa8\uc2a4\uc758 \uc2e0\ube44\u2019\ub860\uc5d0\uc11c \uc774\ub7f0 \uc810\uc744 \ucda9\ubd84\ud788 \ud655\uc778\ud560 \uc218 \uc788\ub2e4.", "answer": "\uc2a4\ud2f0\ube10 \uc0e4\ud540", "sentence": "\ud55c\ud3b8, \ucf00\ud50c\ub7ec\ub294 \ub2e4\ub978 16 ~ 17\uc138\uae30\uc758 \ucc9c\ubb38\ud559\uc790\ub4e4\ucc98\ub7fc \uadfc\ub300\uc5d0 \ube44\ud574 \uace0\ub300\uc758 \ucc9c\ubb38\ud559\uc774 \uc870\uc7a1\ud558\ub2e4\ub294 \uac83\uc744 \uac15\uc870\ud588\ub294\ub370, \uc2a4\ud2f0\ube10 \uc0e4\ud540 \uc740 \uc774\uac83\uc774 \uace0\ub300\uc758 \uc9c4\ub9ac\uac00 \uc218\uc138\uae30\ub97c \uac70\uce58\uba74\uc11c \ubaa8\ubc29\uc790\ub4e4\uacfc \uc8fc\uc11d\uc790\ub4e4\uc5d0 \uc758\ud574 \uc190\uc0c1\ub418\uc5c8\ub2e4\uace0 \uc0dd\uac01\ud55c \uc778\ubb38\uc8fc\uc758\uc758 \uc601\ud5a5\uc774\ub77c\uace0 \ub9d0\ud55c\ub2e4.", "paragraph_sentence": " \ud55c\ud3b8, \ucf00\ud50c\ub7ec\ub294 \ub2e4\ub978 16 ~ 17\uc138\uae30\uc758 \ucc9c\ubb38\ud559\uc790\ub4e4\ucc98\ub7fc \uadfc\ub300\uc5d0 \ube44\ud574 \uace0\ub300\uc758 \ucc9c\ubb38\ud559\uc774 \uc870\uc7a1\ud558\ub2e4\ub294 \uac83\uc744 \uac15\uc870\ud588\ub294\ub370, \uc2a4\ud2f0\ube10 \uc0e4\ud540 \uc740 \uc774\uac83\uc774 \uace0\ub300\uc758 \uc9c4\ub9ac\uac00 \uc218\uc138\uae30\ub97c \uac70\uce58\uba74\uc11c \ubaa8\ubc29\uc790\ub4e4\uacfc \uc8fc\uc11d\uc790\ub4e4\uc5d0 \uc758\ud574 \uc190\uc0c1\ub418\uc5c8\ub2e4\uace0 \uc0dd\uac01\ud55c \uc778\ubb38\uc8fc\uc758\uc758 \uc601\ud5a5\uc774\ub77c\uace0 \ub9d0\ud55c\ub2e4. \uc0e4\ud540\uc5d0 \uc758\ud558\uba74 \ucf00\ud50c\ub7ec\uc758 \uc774\ub7f0 \uc0ac\uc0c1\uc740 \u300a\ub8e8\ub3cc\ud504 \ud45c\u300b\uc758 \uad8c\ub450\ud654\ub85c \uc0ac\uc6a9\ub41c \uc720\uba85\ud55c \ud310\ud654 \uc791\ud488\uc5d0\uc11c\ub3c4 \uc798 \ub4dc\ub7ec\ub098\ub294\ub370, \ud310\ud654\uc5d0\uc11c \ucc9c\ubb38\ud559\uc758 \uc5ec\uc2e0\uc758 \uc2e0\uc804\uc758 \ub4a4\ucabd\uc5d0\ub294 \ud22c\ubc15\ud558\uace0 \uad6c\uc870\uc801\uc778 \uc591\uc2dd\uc758 \uae30\ub465\uc774 \uace0\ub300 \ucc9c\ubb38\ud559\uc758 \uc870\uc7a1\ud568\uc744 \ub4dc\ub7ec\ub0b4\uace0 \uc788\uace0, \uc2e0\uc804\uc758 \uc55e\uc5d0 \uc788\ub294 \ucf54\ud398\ub974\ub2c8\ucfe0\uc2a4\uc640 \ud280\ucf54\ub294 \uc774\uc624\ub2c8\uc544\uc2dd\u00b7\ucf54\ub9b0\ud2b8\uc2dd\uc758 \uc6b0\uc544\ud55c \uae30\ub465 \uc606\uc5d0 \uc11c \uc788\ub2e4. \ucf00\ud50c\ub7ec\ub294 \uc778\ubb38\uc8fc\uc758\uc790\uba74\uc11c\ub3c4 \uc5f4\uad11\uc801\uc778 \ud50c\ub77c\ud1a4\uc8fc\uc758\uc790\uc600\ub294\ub370, \uadf8\uc758 \ucd5c\ucd08\uc758 \ud559\uc124\uc778 \u2018\ucf54\uc2a4\ubaa8\uc2a4\uc758 \uc2e0\ube44\u2019\ub860\uc5d0\uc11c \uc774\ub7f0 \uc810\uc744 \ucda9\ubd84\ud788 \ud655\uc778\ud560 \uc218 \uc788\ub2e4.", "paragraph_answer": "\ud55c\ud3b8, \ucf00\ud50c\ub7ec\ub294 \ub2e4\ub978 16 ~ 17\uc138\uae30\uc758 \ucc9c\ubb38\ud559\uc790\ub4e4\ucc98\ub7fc \uadfc\ub300\uc5d0 \ube44\ud574 \uace0\ub300\uc758 \ucc9c\ubb38\ud559\uc774 \uc870\uc7a1\ud558\ub2e4\ub294 \uac83\uc744 \uac15\uc870\ud588\ub294\ub370, \uc2a4\ud2f0\ube10 \uc0e4\ud540 \uc740 \uc774\uac83\uc774 \uace0\ub300\uc758 \uc9c4\ub9ac\uac00 \uc218\uc138\uae30\ub97c \uac70\uce58\uba74\uc11c \ubaa8\ubc29\uc790\ub4e4\uacfc \uc8fc\uc11d\uc790\ub4e4\uc5d0 \uc758\ud574 \uc190\uc0c1\ub418\uc5c8\ub2e4\uace0 \uc0dd\uac01\ud55c \uc778\ubb38\uc8fc\uc758\uc758 \uc601\ud5a5\uc774\ub77c\uace0 \ub9d0\ud55c\ub2e4. \uc0e4\ud540\uc5d0 \uc758\ud558\uba74 \ucf00\ud50c\ub7ec\uc758 \uc774\ub7f0 \uc0ac\uc0c1\uc740 \u300a\ub8e8\ub3cc\ud504 \ud45c\u300b\uc758 \uad8c\ub450\ud654\ub85c \uc0ac\uc6a9\ub41c \uc720\uba85\ud55c \ud310\ud654 \uc791\ud488\uc5d0\uc11c\ub3c4 \uc798 \ub4dc\ub7ec\ub098\ub294\ub370, \ud310\ud654\uc5d0\uc11c \ucc9c\ubb38\ud559\uc758 \uc5ec\uc2e0\uc758 \uc2e0\uc804\uc758 \ub4a4\ucabd\uc5d0\ub294 \ud22c\ubc15\ud558\uace0 \uad6c\uc870\uc801\uc778 \uc591\uc2dd\uc758 \uae30\ub465\uc774 \uace0\ub300 \ucc9c\ubb38\ud559\uc758 \uc870\uc7a1\ud568\uc744 \ub4dc\ub7ec\ub0b4\uace0 \uc788\uace0, \uc2e0\uc804\uc758 \uc55e\uc5d0 \uc788\ub294 \ucf54\ud398\ub974\ub2c8\ucfe0\uc2a4\uc640 \ud280\ucf54\ub294 \uc774\uc624\ub2c8\uc544\uc2dd\u00b7\ucf54\ub9b0\ud2b8\uc2dd\uc758 \uc6b0\uc544\ud55c \uae30\ub465 \uc606\uc5d0 \uc11c \uc788\ub2e4. \ucf00\ud50c\ub7ec\ub294 \uc778\ubb38\uc8fc\uc758\uc790\uba74\uc11c\ub3c4 \uc5f4\uad11\uc801\uc778 \ud50c\ub77c\ud1a4\uc8fc\uc758\uc790\uc600\ub294\ub370, \uadf8\uc758 \ucd5c\ucd08\uc758 \ud559\uc124\uc778 \u2018\ucf54\uc2a4\ubaa8\uc2a4\uc758 \uc2e0\ube44\u2019\ub860\uc5d0\uc11c \uc774\ub7f0 \uc810\uc744 \ucda9\ubd84\ud788 \ud655\uc778\ud560 \uc218 \uc788\ub2e4.", "sentence_answer": "\ud55c\ud3b8, \ucf00\ud50c\ub7ec\ub294 \ub2e4\ub978 16 ~ 17\uc138\uae30\uc758 \ucc9c\ubb38\ud559\uc790\ub4e4\ucc98\ub7fc \uadfc\ub300\uc5d0 \ube44\ud574 \uace0\ub300\uc758 \ucc9c\ubb38\ud559\uc774 \uc870\uc7a1\ud558\ub2e4\ub294 \uac83\uc744 \uac15\uc870\ud588\ub294\ub370, \uc2a4\ud2f0\ube10 \uc0e4\ud540 \uc740 \uc774\uac83\uc774 \uace0\ub300\uc758 \uc9c4\ub9ac\uac00 \uc218\uc138\uae30\ub97c \uac70\uce58\uba74\uc11c \ubaa8\ubc29\uc790\ub4e4\uacfc \uc8fc\uc11d\uc790\ub4e4\uc5d0 \uc758\ud574 \uc190\uc0c1\ub418\uc5c8\ub2e4\uace0 \uc0dd\uac01\ud55c \uc778\ubb38\uc8fc\uc758\uc758 \uc601\ud5a5\uc774\ub77c\uace0 \ub9d0\ud55c\ub2e4.", "paragraph_id": "6384301-34-1", "paragraph_question": "question: \uace0\ub300\uc758 \uc9c4\ub9ac\uac00 \uc218\uc138\uae30\ub97c \uac70\uce58\uba74\uc11c \ubaa8\ubc29\uc790\ub4e4\uacfc \uc8fc\uc11d\uc790\ub4e4\uc5d0 \uc758\ud574 \uc190\uc0c1\ub418\uc5c8\ub2e4\uace0 \uc0dd\uac01\ud55c \uc778\ubb38\uc8fc\uc758\uc790\ub4e4\uc758 \uc601\ud5a5\uc774\ub77c\uace0 \ub9d0\ud55c \uc0ac\ub78c\uc740 \ub204\uad6c\uc778\uac00?, context: \ud55c\ud3b8, \ucf00\ud50c\ub7ec\ub294 \ub2e4\ub978 16 ~ 17\uc138\uae30\uc758 \ucc9c\ubb38\ud559\uc790\ub4e4\ucc98\ub7fc \uadfc\ub300\uc5d0 \ube44\ud574 \uace0\ub300\uc758 \ucc9c\ubb38\ud559\uc774 \uc870\uc7a1\ud558\ub2e4\ub294 \uac83\uc744 \uac15\uc870\ud588\ub294\ub370, \uc2a4\ud2f0\ube10 \uc0e4\ud540\uc740 \uc774\uac83\uc774 \uace0\ub300\uc758 \uc9c4\ub9ac\uac00 \uc218\uc138\uae30\ub97c \uac70\uce58\uba74\uc11c \ubaa8\ubc29\uc790\ub4e4\uacfc \uc8fc\uc11d\uc790\ub4e4\uc5d0 \uc758\ud574 \uc190\uc0c1\ub418\uc5c8\ub2e4\uace0 \uc0dd\uac01\ud55c \uc778\ubb38\uc8fc\uc758\uc758 \uc601\ud5a5\uc774\ub77c\uace0 \ub9d0\ud55c\ub2e4. \uc0e4\ud540\uc5d0 \uc758\ud558\uba74 \ucf00\ud50c\ub7ec\uc758 \uc774\ub7f0 \uc0ac\uc0c1\uc740 \u300a\ub8e8\ub3cc\ud504 \ud45c\u300b\uc758 \uad8c\ub450\ud654\ub85c \uc0ac\uc6a9\ub41c \uc720\uba85\ud55c \ud310\ud654 \uc791\ud488\uc5d0\uc11c\ub3c4 \uc798 \ub4dc\ub7ec\ub098\ub294\ub370, \ud310\ud654\uc5d0\uc11c \ucc9c\ubb38\ud559\uc758 \uc5ec\uc2e0\uc758 \uc2e0\uc804\uc758 \ub4a4\ucabd\uc5d0\ub294 \ud22c\ubc15\ud558\uace0 \uad6c\uc870\uc801\uc778 \uc591\uc2dd\uc758 \uae30\ub465\uc774 \uace0\ub300 \ucc9c\ubb38\ud559\uc758 \uc870\uc7a1\ud568\uc744 \ub4dc\ub7ec\ub0b4\uace0 \uc788\uace0, \uc2e0\uc804\uc758 \uc55e\uc5d0 \uc788\ub294 \ucf54\ud398\ub974\ub2c8\ucfe0\uc2a4\uc640 \ud280\ucf54\ub294 \uc774\uc624\ub2c8\uc544\uc2dd\u00b7\ucf54\ub9b0\ud2b8\uc2dd\uc758 \uc6b0\uc544\ud55c \uae30\ub465 \uc606\uc5d0 \uc11c \uc788\ub2e4. \ucf00\ud50c\ub7ec\ub294 \uc778\ubb38\uc8fc\uc758\uc790\uba74\uc11c\ub3c4 \uc5f4\uad11\uc801\uc778 \ud50c\ub77c\ud1a4\uc8fc\uc758\uc790\uc600\ub294\ub370, \uadf8\uc758 \ucd5c\ucd08\uc758 \ud559\uc124\uc778 \u2018\ucf54\uc2a4\ubaa8\uc2a4\uc758 \uc2e0\ube44\u2019\ub860\uc5d0\uc11c \uc774\ub7f0 \uc810\uc744 \ucda9\ubd84\ud788 \ud655\uc778\ud560 \uc218 \uc788\ub2e4."} {"question": "\uae30\uc874\uc758 \ud074\ub8e8\ub3c4\uc5d0\ub294 11\uac1c\uc758 \ubc29\uacfc \ud568\uaed8 \uba87 \uac1c\uc758 \ubb34\uae30\uac00 \uc788\ub294 \uc124\uc815\uc774\uc5c8\ub294\uac00?", "paragraph": "\uadf8\ub807\uc9c0\ub9cc \ud074\ub8e8\ub3c4\ub294 1949\ub144\uc5d0 \ucd9c\uc2dc\ub418\uc5c8\ub358 \uc694\uc18c\ud558\uace0 \ub9e8\ucc98\uc74c\uc5d0 \uad6c\uc0c1\ud588\ub358 \uc694\uc18c\ud558\uace0 \uc11c\ub85c \ucc28\uc774\uac00 \uc788\ub2e4. \ud504\ub81b\uac00 \ubcf8\ub798 \uc81c\uc548\ud558\uc600\ub358 \ub4f1\uc7a5\uc778\ubb3c\uc758 \uc22b\uc790\ub294 10\uba85\uc774\uba70 \uac8c\uc784\uc744 \uc2dc\uc791\ud558\uae30 \uc55e\uc11c \ud76c\uc0dd\uc790 1\uba85\ub97c \ub4f1\uc7a5\uc778\ubb3c 10\uba85 \uc911\uc5d0 \uc784\uc758\uc801\uc73c\ub85c \uacb0\uc815\ud558\ub294 \uacc4\ud68d\uc774\uc5c8\ub2e4. \uadf8 \ub4f1\uc7a5\uc778\ubb3c 10\uba85\uc740 \uc0ac\ub9dd\ud55c \ube0c\ub77c\uc6b4 \uc528(Mr. Brown), \uace8\ub4dc \uc528(Mr. Gold), \uadf8\ub808\uc774 \uc528(Miss Grey) \uc2e4\ubc84 \ubd80\uc778(Mrs. Silver)\uacfc \ub354\ubd88\uc5b4 \ud654\uc774\ud2b8 \uac04\ud638\uc0ac(Nurse White)\uc640 \uc610\ub85c\uc6b0 \ub300\ub839(Colonel Yellow)\uc774 \ub4f1\uc7a5\ud558\uc600\ub2e4. \uc6d0\ub798 \ucd5c\ub300 9\uba85\uc758 \uc6a9\uc758\uc790\uac00 \uc788\uc744 \ub54c\uc5d0 \ub0a8\uc544 \uc788\ub294 \uc8fc\uc778\uacf5 8\uba85\uc73c\ub85c \uac8c\uc784\uc758 \uc9c4\ud589\ud55c\ub2e4. \uc6d0\ub798 \uc218\ub835\uc2e4(gun room)\uacfc \uc800\uc7a5\uace0(cellar)\ub97c \ud3ec\ud568\ud55c \ubc29 11\uac1c\uac00 \uc788\uc5c8\ub2e4. \uc774\uc640 \ub354\ubd88\uc5b4 \ub3c4\ub07c, \ud3ed\ud0c4, \uc8fc\uc0ac\uae30, \ub3c5\uadf9\ubb3c, \uace4\ubd09, \uadf8\ub9ac\uace0 \ubd80\uc9c0\uae7d\uc774\ub97c \ud3ec\ud568\ud55c \ucd1d \ubb34\uac00 9\uac1c\uac00 \uc788\uc5c8\ub2e4. \uadf8 \uc77c\ubd80\uc758 \ubb34\uae30\ub294 \ub098\uc911\uc5d0 \ud074\ub8e8\uc5d0\uc11c \ud30c\uc0dd\ub41c \ubcf4\ub4dc\uac8c\uc784\uc5d0\uc11c \ub4f1\uc7a5\ud55c\ub2e4.", "answer": "9\uac1c", "sentence": "\uc774\uc640 \ub354\ubd88\uc5b4 \ub3c4\ub07c, \ud3ed\ud0c4, \uc8fc\uc0ac\uae30, \ub3c5\uadf9\ubb3c, \uace4\ubd09, \uadf8\ub9ac\uace0 \ubd80\uc9c0\uae7d\uc774\ub97c \ud3ec\ud568\ud55c \ucd1d \ubb34\uac00 9\uac1c \uac00 \uc788\uc5c8\ub2e4.", "paragraph_sentence": "\uadf8\ub807\uc9c0\ub9cc \ud074\ub8e8\ub3c4\ub294 1949\ub144\uc5d0 \ucd9c\uc2dc\ub418\uc5c8\ub358 \uc694\uc18c\ud558\uace0 \ub9e8\ucc98\uc74c\uc5d0 \uad6c\uc0c1\ud588\ub358 \uc694\uc18c\ud558\uace0 \uc11c\ub85c \ucc28\uc774\uac00 \uc788\ub2e4. \ud504\ub81b\uac00 \ubcf8\ub798 \uc81c\uc548\ud558\uc600\ub358 \ub4f1\uc7a5\uc778\ubb3c\uc758 \uc22b\uc790\ub294 10\uba85\uc774\uba70 \uac8c\uc784\uc744 \uc2dc\uc791\ud558\uae30 \uc55e\uc11c \ud76c\uc0dd\uc790 1\uba85\ub97c \ub4f1\uc7a5\uc778\ubb3c 10\uba85 \uc911\uc5d0 \uc784\uc758\uc801\uc73c\ub85c \uacb0\uc815\ud558\ub294 \uacc4\ud68d\uc774\uc5c8\ub2e4. \uadf8 \ub4f1\uc7a5\uc778\ubb3c 10\uba85\uc740 \uc0ac\ub9dd\ud55c \ube0c\ub77c\uc6b4 \uc528(Mr. Brown), \uace8\ub4dc \uc528(Mr. Gold), \uadf8\ub808\uc774 \uc528(Miss Grey) \uc2e4\ubc84 \ubd80\uc778(Mrs. Silver)\uacfc \ub354\ubd88\uc5b4 \ud654\uc774\ud2b8 \uac04\ud638\uc0ac(Nurse White)\uc640 \uc610\ub85c\uc6b0 \ub300\ub839(Colonel Yellow)\uc774 \ub4f1\uc7a5\ud558\uc600\ub2e4. \uc6d0\ub798 \ucd5c\ub300 9\uba85\uc758 \uc6a9\uc758\uc790\uac00 \uc788\uc744 \ub54c\uc5d0 \ub0a8\uc544 \uc788\ub294 \uc8fc\uc778\uacf5 8\uba85\uc73c\ub85c \uac8c\uc784\uc758 \uc9c4\ud589\ud55c\ub2e4. \uc6d0\ub798 \uc218\ub835\uc2e4(gun room)\uacfc \uc800\uc7a5\uace0(cellar)\ub97c \ud3ec\ud568\ud55c \ubc29 11\uac1c\uac00 \uc788\uc5c8\ub2e4. \uc774\uc640 \ub354\ubd88\uc5b4 \ub3c4\ub07c, \ud3ed\ud0c4, \uc8fc\uc0ac\uae30, \ub3c5\uadf9\ubb3c, \uace4\ubd09, \uadf8\ub9ac\uace0 \ubd80\uc9c0\uae7d\uc774\ub97c \ud3ec\ud568\ud55c \ucd1d \ubb34\uac00 9\uac1c \uac00 \uc788\uc5c8\ub2e4. \uadf8 \uc77c\ubd80\uc758 \ubb34\uae30\ub294 \ub098\uc911\uc5d0 \ud074\ub8e8\uc5d0\uc11c \ud30c\uc0dd\ub41c \ubcf4\ub4dc\uac8c\uc784\uc5d0\uc11c \ub4f1\uc7a5\ud55c\ub2e4.", "paragraph_answer": "\uadf8\ub807\uc9c0\ub9cc \ud074\ub8e8\ub3c4\ub294 1949\ub144\uc5d0 \ucd9c\uc2dc\ub418\uc5c8\ub358 \uc694\uc18c\ud558\uace0 \ub9e8\ucc98\uc74c\uc5d0 \uad6c\uc0c1\ud588\ub358 \uc694\uc18c\ud558\uace0 \uc11c\ub85c \ucc28\uc774\uac00 \uc788\ub2e4. \ud504\ub81b\uac00 \ubcf8\ub798 \uc81c\uc548\ud558\uc600\ub358 \ub4f1\uc7a5\uc778\ubb3c\uc758 \uc22b\uc790\ub294 10\uba85\uc774\uba70 \uac8c\uc784\uc744 \uc2dc\uc791\ud558\uae30 \uc55e\uc11c \ud76c\uc0dd\uc790 1\uba85\ub97c \ub4f1\uc7a5\uc778\ubb3c 10\uba85 \uc911\uc5d0 \uc784\uc758\uc801\uc73c\ub85c \uacb0\uc815\ud558\ub294 \uacc4\ud68d\uc774\uc5c8\ub2e4. \uadf8 \ub4f1\uc7a5\uc778\ubb3c 10\uba85\uc740 \uc0ac\ub9dd\ud55c \ube0c\ub77c\uc6b4 \uc528(Mr. Brown), \uace8\ub4dc \uc528(Mr. Gold), \uadf8\ub808\uc774 \uc528(Miss Grey) \uc2e4\ubc84 \ubd80\uc778(Mrs. Silver)\uacfc \ub354\ubd88\uc5b4 \ud654\uc774\ud2b8 \uac04\ud638\uc0ac(Nurse White)\uc640 \uc610\ub85c\uc6b0 \ub300\ub839(Colonel Yellow)\uc774 \ub4f1\uc7a5\ud558\uc600\ub2e4. \uc6d0\ub798 \ucd5c\ub300 9\uba85\uc758 \uc6a9\uc758\uc790\uac00 \uc788\uc744 \ub54c\uc5d0 \ub0a8\uc544 \uc788\ub294 \uc8fc\uc778\uacf5 8\uba85\uc73c\ub85c \uac8c\uc784\uc758 \uc9c4\ud589\ud55c\ub2e4. \uc6d0\ub798 \uc218\ub835\uc2e4(gun room)\uacfc \uc800\uc7a5\uace0(cellar)\ub97c \ud3ec\ud568\ud55c \ubc29 11\uac1c\uac00 \uc788\uc5c8\ub2e4. \uc774\uc640 \ub354\ubd88\uc5b4 \ub3c4\ub07c, \ud3ed\ud0c4, \uc8fc\uc0ac\uae30, \ub3c5\uadf9\ubb3c, \uace4\ubd09, \uadf8\ub9ac\uace0 \ubd80\uc9c0\uae7d\uc774\ub97c \ud3ec\ud568\ud55c \ucd1d \ubb34\uac00 9\uac1c \uac00 \uc788\uc5c8\ub2e4. \uadf8 \uc77c\ubd80\uc758 \ubb34\uae30\ub294 \ub098\uc911\uc5d0 \ud074\ub8e8\uc5d0\uc11c \ud30c\uc0dd\ub41c \ubcf4\ub4dc\uac8c\uc784\uc5d0\uc11c \ub4f1\uc7a5\ud55c\ub2e4.", "sentence_answer": "\uc774\uc640 \ub354\ubd88\uc5b4 \ub3c4\ub07c, \ud3ed\ud0c4, \uc8fc\uc0ac\uae30, \ub3c5\uadf9\ubb3c, \uace4\ubd09, \uadf8\ub9ac\uace0 \ubd80\uc9c0\uae7d\uc774\ub97c \ud3ec\ud568\ud55c \ucd1d \ubb34\uac00 9\uac1c \uac00 \uc788\uc5c8\ub2e4.", "paragraph_id": "6564692-1-1", "paragraph_question": "question: \uae30\uc874\uc758 \ud074\ub8e8\ub3c4\uc5d0\ub294 11\uac1c\uc758 \ubc29\uacfc \ud568\uaed8 \uba87 \uac1c\uc758 \ubb34\uae30\uac00 \uc788\ub294 \uc124\uc815\uc774\uc5c8\ub294\uac00?, context: \uadf8\ub807\uc9c0\ub9cc \ud074\ub8e8\ub3c4\ub294 1949\ub144\uc5d0 \ucd9c\uc2dc\ub418\uc5c8\ub358 \uc694\uc18c\ud558\uace0 \ub9e8\ucc98\uc74c\uc5d0 \uad6c\uc0c1\ud588\ub358 \uc694\uc18c\ud558\uace0 \uc11c\ub85c \ucc28\uc774\uac00 \uc788\ub2e4. \ud504\ub81b\uac00 \ubcf8\ub798 \uc81c\uc548\ud558\uc600\ub358 \ub4f1\uc7a5\uc778\ubb3c\uc758 \uc22b\uc790\ub294 10\uba85\uc774\uba70 \uac8c\uc784\uc744 \uc2dc\uc791\ud558\uae30 \uc55e\uc11c \ud76c\uc0dd\uc790 1\uba85\ub97c \ub4f1\uc7a5\uc778\ubb3c 10\uba85 \uc911\uc5d0 \uc784\uc758\uc801\uc73c\ub85c \uacb0\uc815\ud558\ub294 \uacc4\ud68d\uc774\uc5c8\ub2e4. \uadf8 \ub4f1\uc7a5\uc778\ubb3c 10\uba85\uc740 \uc0ac\ub9dd\ud55c \ube0c\ub77c\uc6b4 \uc528(Mr. Brown), \uace8\ub4dc \uc528(Mr. Gold), \uadf8\ub808\uc774 \uc528(Miss Grey) \uc2e4\ubc84 \ubd80\uc778(Mrs. Silver)\uacfc \ub354\ubd88\uc5b4 \ud654\uc774\ud2b8 \uac04\ud638\uc0ac(Nurse White)\uc640 \uc610\ub85c\uc6b0 \ub300\ub839(Colonel Yellow)\uc774 \ub4f1\uc7a5\ud558\uc600\ub2e4. \uc6d0\ub798 \ucd5c\ub300 9\uba85\uc758 \uc6a9\uc758\uc790\uac00 \uc788\uc744 \ub54c\uc5d0 \ub0a8\uc544 \uc788\ub294 \uc8fc\uc778\uacf5 8\uba85\uc73c\ub85c \uac8c\uc784\uc758 \uc9c4\ud589\ud55c\ub2e4. \uc6d0\ub798 \uc218\ub835\uc2e4(gun room)\uacfc \uc800\uc7a5\uace0(cellar)\ub97c \ud3ec\ud568\ud55c \ubc29 11\uac1c\uac00 \uc788\uc5c8\ub2e4. \uc774\uc640 \ub354\ubd88\uc5b4 \ub3c4\ub07c, \ud3ed\ud0c4, \uc8fc\uc0ac\uae30, \ub3c5\uadf9\ubb3c, \uace4\ubd09, \uadf8\ub9ac\uace0 \ubd80\uc9c0\uae7d\uc774\ub97c \ud3ec\ud568\ud55c \ucd1d \ubb34\uac00 9\uac1c\uac00 \uc788\uc5c8\ub2e4. \uadf8 \uc77c\ubd80\uc758 \ubb34\uae30\ub294 \ub098\uc911\uc5d0 \ud074\ub8e8\uc5d0\uc11c \ud30c\uc0dd\ub41c \ubcf4\ub4dc\uac8c\uc784\uc5d0\uc11c \ub4f1\uc7a5\ud55c\ub2e4."} {"question": "\ubbfc\uc8fc\uc8fc\uc758\uac00 \ubd80\ud65c\ud558\ub3c4\ub85d \ub3c4\uc6b4 87\ub144\uc5d0 \uc77c\uc5b4\ub09c \ud56d\uc7c1\uc740?", "paragraph": "\ud55c\uad6d\uc5d0\uc11c\ub3c4 \uc591\ubc18\uacfc \uc0c1\ubbfc\u00b7\ub178\ube44\ub85c \uac08\ub824 \uc18c\uc218\uc758 \uc591\ubc18\uce35\uc774 \ubbfc\uc911\uc744 \uac00\ud639\ud558\uac8c \uc5b5\uc555,\ucc29\ucde8\ud558\ub358 \uc57c\ub9cc\uc2dc\ub300\ub97c \ub3d9\ud559\uc0ac\uc0c1\uc758 \ub300\ub450\ub85c \ud601\uba85\uc774 \ubc1c\ubc1c\ud558\uc5ec \uc138\uc0c1\uc774 \ub4a4\ubc14\ub00c\uae30 \uc2dc\uc791\ud588\ub2e4.\ub3d9\ud559\ub3c4\ub4e4\ub07c\ub9ac\ub294 \ub0a8\ub140\ub178\uc18c,\ube48\ubd80\uadc0\ucc9c \uac00\ub9ac\uc9c0 \uc54a\uace0 \uc11c\ub85c\ub97c \ud3c9\ub4f1\ud558\uac8c \ub300\uc6b0\ud558\uace0 \ud558\ub298\ucc98\ub7fc \uc12c\uacbc\ub2e4(\uc0ac\uc778\uc5ec\ucc9c,\uc778\ub0b4\ucc9c).\uadf8\ub7ec\ub098 \uc2dd\ubbfc\uae30\ub97c \uac70\uce58\uba74\uc11c \ud3c9\ub4f1\uc0ac\uc0c1\uc774 \ud76c\ubc15\ud574\uc84c\uace0 \uc624\ub79c \ub3c5\uc7ac\uc815\uce58\ub85c \uc644\uc804\ud788 \ud3c9\ub4f1\uc0ac\uc0c1\uc740 \uc0ac\ub77c\uc84c\ub2e4.\uadf8\ub9ac\uace0 5,18\uad11\uc8fc\ud56d\uc7c1\uacfc 87\ub144 6\uc6d4\ubbfc\uc911\ud56d\uc7c1\uc73c\ub85c \ubbfc\uc8fc\uc8fc\uc758\uac00 \ubd80\ud65c\ud568\uc73c\ub85c\uc368 \ud3c9\ub4f1\uc0ac\uc0c1\ub3c4 \uaca8\uc6b0 \ube5b\uc740 \ubcf4\uac8c \ub418\uc5c8\ub2e4.\uadf8\ub7ec\ub098 \uc774\ub7f0 \ub3c4\ub3c4\ud55c \ud3c9\ub4f1\uc758 \uc138\uacc4\uc0ac\uc640 \uc0dd\uc0dd\ud55c \ubbfc\uc871\uc801 \uccb4\ud5d8\uc774 \uc788\uc5c8\uc74c\uc5d0\ub3c4,\ud604\uc7ac \ud55c\uad6d\uc0ac\ud68c\ub294 \uc804\ud600 \ud3c9\ub4f1\ud558\uc9c0\uac00 \uc54a\ub2e4.\uacbd\uc81c,\uad70\uc0ac\uc801\uc73c\ub85c\ub294 \uc120\uc9c4\uc744 \ub118\uc5b4 \uc77c\ub958\uad6d\uac00\ub97c \ubd80\ub974\uc9d6\uc73c\uba74\uc11c\ub3c4 \uc778\uac04\uc758 \uc0c1\ub2f9\ud55c \uacbd\uc81c\uc801 \ud3c9\ub4f1\uc740 \uace0\uc0ac\ud558\uace0 \ud3c9\ub4f1\uc758 \ucd08\ubcf4\ub2e8\uacc4\uc778 \ubc95\uc801 \uc815\uce58\uad8c\ub825\uc801 \ud3c9\ub4f1\ub3c4 \uc815\ucc29\uc2dc\ud0a4\uc9c0 \ubabb\ud558\uace0 \uc788\ub2e4.\uc0ac\ud68c\ubcf5\uc9c0 \uc218\uc900,\uc0ac\ud68c\uad6c\uc131\uc6d0\uc758 \ub300\ub2e4\uc218\uc778 \ub178\ub3d9\uc790\uc758 \uc9c0\uc704\uc640 \uad8c\ud55c,\uc5ec\uc131\uc758 \uad8c\ub9ac \ub4f1\uc5d0 \uc788\uc5b4 \uc77c\ub958\ub294\ucee4\ub155 \uac70\uc758 \uc911\ud6c4\uc9c4\uad6d \uc218\uc900\uc744 \ub9f4\ub3cc\uace0 \uc788\ub294 \uac83\uc774\ub2e4.\uacbd\uc81c,\uad70\uc0ac \uc120\uc9c4\uad6d\ub2f5\uac8c \uc815\uce58\u00b7\uacbd\uc81c\uc801 \ud3c9\ub4f1\uc218\uc900\uacfc \uc0ac\ub78c\ub4e4 \ubaa8\ub450\uc758 \uad8c\uc775 \ubcf4\uc7a5\ub3c4 \uc120\uc9c4\uad6d\ub2e4\uc6cc\uc57c \uc815\uc0c1\uc774\uc9c0 \uc774\ub807\uac8c \ubd88\uade0\ud615\ud55c \uae30\ud615\uc801\uc778 \ub098\ub77c\uc5d0\uc11c \ubb34\uc2a8 \ud76c\ub9dd\uacfc \ubbff\uc74c\uc774 \uc788\uaca0\ub294\uac00!", "answer": "6\uc6d4\ubbfc\uc911\ud56d\uc7c1", "sentence": "\ud55c\uad6d\uc5d0\uc11c\ub3c4 \uc591\ubc18\uacfc \uc0c1\ubbfc\u00b7\ub178\ube44\ub85c \uac08\ub824 \uc18c\uc218\uc758 \uc591\ubc18\uce35\uc774 \ubbfc\uc911\uc744 \uac00\ud639\ud558\uac8c \uc5b5\uc555,\ucc29\ucde8\ud558\ub358 \uc57c\ub9cc\uc2dc\ub300\ub97c \ub3d9\ud559\uc0ac\uc0c1\uc758 \ub300\ub450\ub85c \ud601\uba85\uc774 \ubc1c\ubc1c\ud558\uc5ec \uc138\uc0c1\uc774 \ub4a4\ubc14\ub00c\uae30 \uc2dc\uc791\ud588\ub2e4.\ub3d9\ud559\ub3c4\ub4e4\ub07c\ub9ac\ub294 \ub0a8\ub140\ub178\uc18c,\ube48\ubd80\uadc0\ucc9c \uac00\ub9ac\uc9c0 \uc54a\uace0 \uc11c\ub85c\ub97c \ud3c9\ub4f1\ud558\uac8c \ub300\uc6b0\ud558\uace0 \ud558\ub298\ucc98\ub7fc \uc12c\uacbc\ub2e4(\uc0ac\uc778\uc5ec\ucc9c,\uc778\ub0b4\ucc9c).\uadf8\ub7ec\ub098 \uc2dd\ubbfc\uae30\ub97c \uac70\uce58\uba74\uc11c \ud3c9\ub4f1\uc0ac\uc0c1\uc774 \ud76c\ubc15\ud574\uc84c\uace0 \uc624\ub79c \ub3c5\uc7ac\uc815\uce58\ub85c \uc644\uc804\ud788 \ud3c9\ub4f1\uc0ac\uc0c1\uc740 \uc0ac\ub77c\uc84c\ub2e4.\uadf8\ub9ac\uace0 5,18\uad11\uc8fc\ud56d\uc7c1\uacfc 87\ub144 6\uc6d4\ubbfc\uc911\ud56d\uc7c1 \uc73c\ub85c \ubbfc\uc8fc\uc8fc\uc758\uac00 \ubd80\ud65c\ud568\uc73c\ub85c\uc368 \ud3c9\ub4f1\uc0ac\uc0c1\ub3c4 \uaca8\uc6b0 \ube5b\uc740 \ubcf4\uac8c \ub418\uc5c8\ub2e4.\uadf8\ub7ec\ub098 \uc774\ub7f0 \ub3c4\ub3c4\ud55c \ud3c9\ub4f1\uc758 \uc138\uacc4\uc0ac\uc640 \uc0dd\uc0dd\ud55c \ubbfc\uc871\uc801 \uccb4\ud5d8\uc774 \uc788\uc5c8\uc74c\uc5d0\ub3c4,\ud604\uc7ac \ud55c\uad6d\uc0ac\ud68c\ub294 \uc804\ud600 \ud3c9\ub4f1\ud558\uc9c0\uac00 \uc54a\ub2e4.", "paragraph_sentence": " \ud55c\uad6d\uc5d0\uc11c\ub3c4 \uc591\ubc18\uacfc \uc0c1\ubbfc\u00b7\ub178\ube44\ub85c \uac08\ub824 \uc18c\uc218\uc758 \uc591\ubc18\uce35\uc774 \ubbfc\uc911\uc744 \uac00\ud639\ud558\uac8c \uc5b5\uc555,\ucc29\ucde8\ud558\ub358 \uc57c\ub9cc\uc2dc\ub300\ub97c \ub3d9\ud559\uc0ac\uc0c1\uc758 \ub300\ub450\ub85c \ud601\uba85\uc774 \ubc1c\ubc1c\ud558\uc5ec \uc138\uc0c1\uc774 \ub4a4\ubc14\ub00c\uae30 \uc2dc\uc791\ud588\ub2e4.\ub3d9\ud559\ub3c4\ub4e4\ub07c\ub9ac\ub294 \ub0a8\ub140\ub178\uc18c,\ube48\ubd80\uadc0\ucc9c \uac00\ub9ac\uc9c0 \uc54a\uace0 \uc11c\ub85c\ub97c \ud3c9\ub4f1\ud558\uac8c \ub300\uc6b0\ud558\uace0 \ud558\ub298\ucc98\ub7fc \uc12c\uacbc\ub2e4(\uc0ac\uc778\uc5ec\ucc9c,\uc778\ub0b4\ucc9c).\uadf8\ub7ec\ub098 \uc2dd\ubbfc\uae30\ub97c \uac70\uce58\uba74\uc11c \ud3c9\ub4f1\uc0ac\uc0c1\uc774 \ud76c\ubc15\ud574\uc84c\uace0 \uc624\ub79c \ub3c5\uc7ac\uc815\uce58\ub85c \uc644\uc804\ud788 \ud3c9\ub4f1\uc0ac\uc0c1\uc740 \uc0ac\ub77c\uc84c\ub2e4.\uadf8\ub9ac\uace0 5,18\uad11\uc8fc\ud56d\uc7c1\uacfc 87\ub144 6\uc6d4\ubbfc\uc911\ud56d\uc7c1 \uc73c\ub85c \ubbfc\uc8fc\uc8fc\uc758\uac00 \ubd80\ud65c\ud568\uc73c\ub85c\uc368 \ud3c9\ub4f1\uc0ac\uc0c1\ub3c4 \uaca8\uc6b0 \ube5b\uc740 \ubcf4\uac8c \ub418\uc5c8\ub2e4.\uadf8\ub7ec\ub098 \uc774\ub7f0 \ub3c4\ub3c4\ud55c \ud3c9\ub4f1\uc758 \uc138\uacc4\uc0ac\uc640 \uc0dd\uc0dd\ud55c \ubbfc\uc871\uc801 \uccb4\ud5d8\uc774 \uc788\uc5c8\uc74c\uc5d0\ub3c4,\ud604\uc7ac \ud55c\uad6d\uc0ac\ud68c\ub294 \uc804\ud600 \ud3c9\ub4f1\ud558\uc9c0\uac00 \uc54a\ub2e4. \uacbd\uc81c,\uad70\uc0ac\uc801\uc73c\ub85c\ub294 \uc120\uc9c4\uc744 \ub118\uc5b4 \uc77c\ub958\uad6d\uac00\ub97c \ubd80\ub974\uc9d6\uc73c\uba74\uc11c\ub3c4 \uc778\uac04\uc758 \uc0c1\ub2f9\ud55c \uacbd\uc81c\uc801 \ud3c9\ub4f1\uc740 \uace0\uc0ac\ud558\uace0 \ud3c9\ub4f1\uc758 \ucd08\ubcf4\ub2e8\uacc4\uc778 \ubc95\uc801 \uc815\uce58\uad8c\ub825\uc801 \ud3c9\ub4f1\ub3c4 \uc815\ucc29\uc2dc\ud0a4\uc9c0 \ubabb\ud558\uace0 \uc788\ub2e4.\uc0ac\ud68c\ubcf5\uc9c0 \uc218\uc900,\uc0ac\ud68c\uad6c\uc131\uc6d0\uc758 \ub300\ub2e4\uc218\uc778 \ub178\ub3d9\uc790\uc758 \uc9c0\uc704\uc640 \uad8c\ud55c,\uc5ec\uc131\uc758 \uad8c\ub9ac \ub4f1\uc5d0 \uc788\uc5b4 \uc77c\ub958\ub294\ucee4\ub155 \uac70\uc758 \uc911\ud6c4\uc9c4\uad6d \uc218\uc900\uc744 \ub9f4\ub3cc\uace0 \uc788\ub294 \uac83\uc774\ub2e4. \uacbd\uc81c,\uad70\uc0ac \uc120\uc9c4\uad6d\ub2f5\uac8c \uc815\uce58\u00b7\uacbd\uc81c\uc801 \ud3c9\ub4f1\uc218\uc900\uacfc \uc0ac\ub78c\ub4e4 \ubaa8\ub450\uc758 \uad8c\uc775 \ubcf4\uc7a5\ub3c4 \uc120\uc9c4\uad6d\ub2e4\uc6cc\uc57c \uc815\uc0c1\uc774\uc9c0 \uc774\ub807\uac8c \ubd88\uade0\ud615\ud55c \uae30\ud615\uc801\uc778 \ub098\ub77c\uc5d0\uc11c \ubb34\uc2a8 \ud76c\ub9dd\uacfc \ubbff\uc74c\uc774 \uc788\uaca0\ub294\uac00!", "paragraph_answer": "\ud55c\uad6d\uc5d0\uc11c\ub3c4 \uc591\ubc18\uacfc \uc0c1\ubbfc\u00b7\ub178\ube44\ub85c \uac08\ub824 \uc18c\uc218\uc758 \uc591\ubc18\uce35\uc774 \ubbfc\uc911\uc744 \uac00\ud639\ud558\uac8c \uc5b5\uc555,\ucc29\ucde8\ud558\ub358 \uc57c\ub9cc\uc2dc\ub300\ub97c \ub3d9\ud559\uc0ac\uc0c1\uc758 \ub300\ub450\ub85c \ud601\uba85\uc774 \ubc1c\ubc1c\ud558\uc5ec \uc138\uc0c1\uc774 \ub4a4\ubc14\ub00c\uae30 \uc2dc\uc791\ud588\ub2e4.\ub3d9\ud559\ub3c4\ub4e4\ub07c\ub9ac\ub294 \ub0a8\ub140\ub178\uc18c,\ube48\ubd80\uadc0\ucc9c \uac00\ub9ac\uc9c0 \uc54a\uace0 \uc11c\ub85c\ub97c \ud3c9\ub4f1\ud558\uac8c \ub300\uc6b0\ud558\uace0 \ud558\ub298\ucc98\ub7fc \uc12c\uacbc\ub2e4(\uc0ac\uc778\uc5ec\ucc9c,\uc778\ub0b4\ucc9c).\uadf8\ub7ec\ub098 \uc2dd\ubbfc\uae30\ub97c \uac70\uce58\uba74\uc11c \ud3c9\ub4f1\uc0ac\uc0c1\uc774 \ud76c\ubc15\ud574\uc84c\uace0 \uc624\ub79c \ub3c5\uc7ac\uc815\uce58\ub85c \uc644\uc804\ud788 \ud3c9\ub4f1\uc0ac\uc0c1\uc740 \uc0ac\ub77c\uc84c\ub2e4.\uadf8\ub9ac\uace0 5,18\uad11\uc8fc\ud56d\uc7c1\uacfc 87\ub144 6\uc6d4\ubbfc\uc911\ud56d\uc7c1 \uc73c\ub85c \ubbfc\uc8fc\uc8fc\uc758\uac00 \ubd80\ud65c\ud568\uc73c\ub85c\uc368 \ud3c9\ub4f1\uc0ac\uc0c1\ub3c4 \uaca8\uc6b0 \ube5b\uc740 \ubcf4\uac8c \ub418\uc5c8\ub2e4.\uadf8\ub7ec\ub098 \uc774\ub7f0 \ub3c4\ub3c4\ud55c \ud3c9\ub4f1\uc758 \uc138\uacc4\uc0ac\uc640 \uc0dd\uc0dd\ud55c \ubbfc\uc871\uc801 \uccb4\ud5d8\uc774 \uc788\uc5c8\uc74c\uc5d0\ub3c4,\ud604\uc7ac \ud55c\uad6d\uc0ac\ud68c\ub294 \uc804\ud600 \ud3c9\ub4f1\ud558\uc9c0\uac00 \uc54a\ub2e4.\uacbd\uc81c,\uad70\uc0ac\uc801\uc73c\ub85c\ub294 \uc120\uc9c4\uc744 \ub118\uc5b4 \uc77c\ub958\uad6d\uac00\ub97c \ubd80\ub974\uc9d6\uc73c\uba74\uc11c\ub3c4 \uc778\uac04\uc758 \uc0c1\ub2f9\ud55c \uacbd\uc81c\uc801 \ud3c9\ub4f1\uc740 \uace0\uc0ac\ud558\uace0 \ud3c9\ub4f1\uc758 \ucd08\ubcf4\ub2e8\uacc4\uc778 \ubc95\uc801 \uc815\uce58\uad8c\ub825\uc801 \ud3c9\ub4f1\ub3c4 \uc815\ucc29\uc2dc\ud0a4\uc9c0 \ubabb\ud558\uace0 \uc788\ub2e4.\uc0ac\ud68c\ubcf5\uc9c0 \uc218\uc900,\uc0ac\ud68c\uad6c\uc131\uc6d0\uc758 \ub300\ub2e4\uc218\uc778 \ub178\ub3d9\uc790\uc758 \uc9c0\uc704\uc640 \uad8c\ud55c,\uc5ec\uc131\uc758 \uad8c\ub9ac \ub4f1\uc5d0 \uc788\uc5b4 \uc77c\ub958\ub294\ucee4\ub155 \uac70\uc758 \uc911\ud6c4\uc9c4\uad6d \uc218\uc900\uc744 \ub9f4\ub3cc\uace0 \uc788\ub294 \uac83\uc774\ub2e4.\uacbd\uc81c,\uad70\uc0ac \uc120\uc9c4\uad6d\ub2f5\uac8c \uc815\uce58\u00b7\uacbd\uc81c\uc801 \ud3c9\ub4f1\uc218\uc900\uacfc \uc0ac\ub78c\ub4e4 \ubaa8\ub450\uc758 \uad8c\uc775 \ubcf4\uc7a5\ub3c4 \uc120\uc9c4\uad6d\ub2e4\uc6cc\uc57c \uc815\uc0c1\uc774\uc9c0 \uc774\ub807\uac8c \ubd88\uade0\ud615\ud55c \uae30\ud615\uc801\uc778 \ub098\ub77c\uc5d0\uc11c \ubb34\uc2a8 \ud76c\ub9dd\uacfc \ubbff\uc74c\uc774 \uc788\uaca0\ub294\uac00!", "sentence_answer": "\ud55c\uad6d\uc5d0\uc11c\ub3c4 \uc591\ubc18\uacfc \uc0c1\ubbfc\u00b7\ub178\ube44\ub85c \uac08\ub824 \uc18c\uc218\uc758 \uc591\ubc18\uce35\uc774 \ubbfc\uc911\uc744 \uac00\ud639\ud558\uac8c \uc5b5\uc555,\ucc29\ucde8\ud558\ub358 \uc57c\ub9cc\uc2dc\ub300\ub97c \ub3d9\ud559\uc0ac\uc0c1\uc758 \ub300\ub450\ub85c \ud601\uba85\uc774 \ubc1c\ubc1c\ud558\uc5ec \uc138\uc0c1\uc774 \ub4a4\ubc14\ub00c\uae30 \uc2dc\uc791\ud588\ub2e4.\ub3d9\ud559\ub3c4\ub4e4\ub07c\ub9ac\ub294 \ub0a8\ub140\ub178\uc18c,\ube48\ubd80\uadc0\ucc9c \uac00\ub9ac\uc9c0 \uc54a\uace0 \uc11c\ub85c\ub97c \ud3c9\ub4f1\ud558\uac8c \ub300\uc6b0\ud558\uace0 \ud558\ub298\ucc98\ub7fc \uc12c\uacbc\ub2e4(\uc0ac\uc778\uc5ec\ucc9c,\uc778\ub0b4\ucc9c).\uadf8\ub7ec\ub098 \uc2dd\ubbfc\uae30\ub97c \uac70\uce58\uba74\uc11c \ud3c9\ub4f1\uc0ac\uc0c1\uc774 \ud76c\ubc15\ud574\uc84c\uace0 \uc624\ub79c \ub3c5\uc7ac\uc815\uce58\ub85c \uc644\uc804\ud788 \ud3c9\ub4f1\uc0ac\uc0c1\uc740 \uc0ac\ub77c\uc84c\ub2e4.\uadf8\ub9ac\uace0 5,18\uad11\uc8fc\ud56d\uc7c1\uacfc 87\ub144 6\uc6d4\ubbfc\uc911\ud56d\uc7c1 \uc73c\ub85c \ubbfc\uc8fc\uc8fc\uc758\uac00 \ubd80\ud65c\ud568\uc73c\ub85c\uc368 \ud3c9\ub4f1\uc0ac\uc0c1\ub3c4 \uaca8\uc6b0 \ube5b\uc740 \ubcf4\uac8c \ub418\uc5c8\ub2e4.\uadf8\ub7ec\ub098 \uc774\ub7f0 \ub3c4\ub3c4\ud55c \ud3c9\ub4f1\uc758 \uc138\uacc4\uc0ac\uc640 \uc0dd\uc0dd\ud55c \ubbfc\uc871\uc801 \uccb4\ud5d8\uc774 \uc788\uc5c8\uc74c\uc5d0\ub3c4,\ud604\uc7ac \ud55c\uad6d\uc0ac\ud68c\ub294 \uc804\ud600 \ud3c9\ub4f1\ud558\uc9c0\uac00 \uc54a\ub2e4.", "paragraph_id": "6511017-4-0", "paragraph_question": "question: \ubbfc\uc8fc\uc8fc\uc758\uac00 \ubd80\ud65c\ud558\ub3c4\ub85d \ub3c4\uc6b4 87\ub144\uc5d0 \uc77c\uc5b4\ub09c \ud56d\uc7c1\uc740?, context: \ud55c\uad6d\uc5d0\uc11c\ub3c4 \uc591\ubc18\uacfc \uc0c1\ubbfc\u00b7\ub178\ube44\ub85c \uac08\ub824 \uc18c\uc218\uc758 \uc591\ubc18\uce35\uc774 \ubbfc\uc911\uc744 \uac00\ud639\ud558\uac8c \uc5b5\uc555,\ucc29\ucde8\ud558\ub358 \uc57c\ub9cc\uc2dc\ub300\ub97c \ub3d9\ud559\uc0ac\uc0c1\uc758 \ub300\ub450\ub85c \ud601\uba85\uc774 \ubc1c\ubc1c\ud558\uc5ec \uc138\uc0c1\uc774 \ub4a4\ubc14\ub00c\uae30 \uc2dc\uc791\ud588\ub2e4.\ub3d9\ud559\ub3c4\ub4e4\ub07c\ub9ac\ub294 \ub0a8\ub140\ub178\uc18c,\ube48\ubd80\uadc0\ucc9c \uac00\ub9ac\uc9c0 \uc54a\uace0 \uc11c\ub85c\ub97c \ud3c9\ub4f1\ud558\uac8c \ub300\uc6b0\ud558\uace0 \ud558\ub298\ucc98\ub7fc \uc12c\uacbc\ub2e4(\uc0ac\uc778\uc5ec\ucc9c,\uc778\ub0b4\ucc9c).\uadf8\ub7ec\ub098 \uc2dd\ubbfc\uae30\ub97c \uac70\uce58\uba74\uc11c \ud3c9\ub4f1\uc0ac\uc0c1\uc774 \ud76c\ubc15\ud574\uc84c\uace0 \uc624\ub79c \ub3c5\uc7ac\uc815\uce58\ub85c \uc644\uc804\ud788 \ud3c9\ub4f1\uc0ac\uc0c1\uc740 \uc0ac\ub77c\uc84c\ub2e4.\uadf8\ub9ac\uace0 5,18\uad11\uc8fc\ud56d\uc7c1\uacfc 87\ub144 6\uc6d4\ubbfc\uc911\ud56d\uc7c1\uc73c\ub85c \ubbfc\uc8fc\uc8fc\uc758\uac00 \ubd80\ud65c\ud568\uc73c\ub85c\uc368 \ud3c9\ub4f1\uc0ac\uc0c1\ub3c4 \uaca8\uc6b0 \ube5b\uc740 \ubcf4\uac8c \ub418\uc5c8\ub2e4.\uadf8\ub7ec\ub098 \uc774\ub7f0 \ub3c4\ub3c4\ud55c \ud3c9\ub4f1\uc758 \uc138\uacc4\uc0ac\uc640 \uc0dd\uc0dd\ud55c \ubbfc\uc871\uc801 \uccb4\ud5d8\uc774 \uc788\uc5c8\uc74c\uc5d0\ub3c4,\ud604\uc7ac \ud55c\uad6d\uc0ac\ud68c\ub294 \uc804\ud600 \ud3c9\ub4f1\ud558\uc9c0\uac00 \uc54a\ub2e4.\uacbd\uc81c,\uad70\uc0ac\uc801\uc73c\ub85c\ub294 \uc120\uc9c4\uc744 \ub118\uc5b4 \uc77c\ub958\uad6d\uac00\ub97c \ubd80\ub974\uc9d6\uc73c\uba74\uc11c\ub3c4 \uc778\uac04\uc758 \uc0c1\ub2f9\ud55c \uacbd\uc81c\uc801 \ud3c9\ub4f1\uc740 \uace0\uc0ac\ud558\uace0 \ud3c9\ub4f1\uc758 \ucd08\ubcf4\ub2e8\uacc4\uc778 \ubc95\uc801 \uc815\uce58\uad8c\ub825\uc801 \ud3c9\ub4f1\ub3c4 \uc815\ucc29\uc2dc\ud0a4\uc9c0 \ubabb\ud558\uace0 \uc788\ub2e4.\uc0ac\ud68c\ubcf5\uc9c0 \uc218\uc900,\uc0ac\ud68c\uad6c\uc131\uc6d0\uc758 \ub300\ub2e4\uc218\uc778 \ub178\ub3d9\uc790\uc758 \uc9c0\uc704\uc640 \uad8c\ud55c,\uc5ec\uc131\uc758 \uad8c\ub9ac \ub4f1\uc5d0 \uc788\uc5b4 \uc77c\ub958\ub294\ucee4\ub155 \uac70\uc758 \uc911\ud6c4\uc9c4\uad6d \uc218\uc900\uc744 \ub9f4\ub3cc\uace0 \uc788\ub294 \uac83\uc774\ub2e4.\uacbd\uc81c,\uad70\uc0ac \uc120\uc9c4\uad6d\ub2f5\uac8c \uc815\uce58\u00b7\uacbd\uc81c\uc801 \ud3c9\ub4f1\uc218\uc900\uacfc \uc0ac\ub78c\ub4e4 \ubaa8\ub450\uc758 \uad8c\uc775 \ubcf4\uc7a5\ub3c4 \uc120\uc9c4\uad6d\ub2e4\uc6cc\uc57c \uc815\uc0c1\uc774\uc9c0 \uc774\ub807\uac8c \ubd88\uade0\ud615\ud55c \uae30\ud615\uc801\uc778 \ub098\ub77c\uc5d0\uc11c \ubb34\uc2a8 \ud76c\ub9dd\uacfc \ubbff\uc74c\uc774 \uc788\uaca0\ub294\uac00!"} {"question": "\ub9ac\uc988 \uc704\ub354\uc2a4\ud47c\uc774 \ubaa9\uc18c\ub9ac \uc5f0\uae30\ub85c \ucc38\uac00\ud55c \ub4dc\ub9bc\uc6cd\uc2a4 \uc560\ub2c8\uba54\uc774\uc158 \uc601\ud654\ub294 \ubb34\uc5c7\uc778\uac00?", "paragraph": "2008\ub144 \uc911\ubc18\uae30\uc5d0 \uc811\uc5b4\ub4e4\uba74\uc11c \uadf8\ub140\ub294 \uc720\ub2c8\ubc84\uc124 \uc2a4\ud29c\ub514\uc624\uc758 1939\ub144 \uc791\ud488\uc778 \u300a\ubbf8\ub4dc\ub098\uc774\ud2b8\u300b\ub97c \ub9ac\uba54\uc774\ud06c\ud55c \uc791\ud488\uc5d0 \ucd9c\ud604\ud558\uae30\ub85c \uacb0\uc815\ud588\ub2e4. 2009\ub144\uc5d0\ub294 \ucc98\uc74c\uc73c\ub85c \ud638\ub7ec\ubb3c\uc5d0 \ub3c4\uc804\ud558\uac8c \ud588\uc73c\uba70, \u300a\uae08\ubc1c\uc774 \ub108\ubb34\ud574 2\u300b\uc5d0 \uacf5\ub3d9 \uc81c\uc791\uc790\ub85c \ud568\uaed8 \ud588\ub358 \uc81c\ub2c8\ud37c \uc2ec\uc2a8\uacfc \ud568\uaed8\ud558\uac8c \ub418\uc5c8\ub2e4. \uc704\ub354\uc2a4\ud47c\uc740 \uc790\uc2e0\uc774 \uc9c1\uc811 \uc18c\uc720\ud558\uace0 \uc788\ub294 \ud0c0\uc774\ud504 \uc5d0\uc774\uc0ac\uac00 \uc9c1\uc811 \uc81c\uc791\uc5d0 \ucc38\uc5ec\ud558\uba70, \uc774\uc640 \ud568\uaed8 2009\ub144 \uac1c\ubd09\ud558\ub294 \ub4dc\ub9bc\uc6cd\uc2a4\uc758 \uc560\ub2c8\uba54\uc774\uc158\uc778 \u300a\ubaac\uc2a4\ud130 vs \uc5d0\uc77c\ub9ac\uc5b8\u300b\uc758 \ubaa9\uc18c\ub9ac \uc5f0\uae30\uc5d0\ub3c4 \ucc38\uac00\ud558\uace0 \uc788\ub294 \uac83\uc73c\ub85c \uc54c\ub824\uc838 \uc788\ub2e4. \ub2e4\ub978 \uc131\uc6b0 \uc5f0\uae30\ub85c\ub294 2011\ub144 \ud06c\ub9ac\uc2a4\ub9c8\uc2a4\uc5d0 \ub9de\ucdb0 \uac1c\ubd09\ud560 \ud53d\uc0ac \uc560\ub2c8\uba54\uc774\uc158 \uc2a4\ud29c\ub514\uc624\uc758 \u300a\uba54\ub9ac\ub2e4\uc640 \ub9c8\ubc95\uc758 \uc232\u300b\uc774 \uc788\uc73c\uba70, \uc774 \uc601\ud654\ub294 \ud53d\uc0ac\uc5d0\uc11c \uc81c\uc791, \uc6d4\ud2b8 \ub514\uc988\ub2c8\uc5d0\uc11c \ubc30\uae09\uc774 \uacb0\uc815\ub418\uc5c8\ub2e4.", "answer": "\ubaac\uc2a4\ud130 vs \uc5d0\uc77c\ub9ac\uc5b8", "sentence": "\uc704\ub354\uc2a4\ud47c\uc740 \uc790\uc2e0\uc774 \uc9c1\uc811 \uc18c\uc720\ud558\uace0 \uc788\ub294 \ud0c0\uc774\ud504 \uc5d0\uc774\uc0ac\uac00 \uc9c1\uc811 \uc81c\uc791\uc5d0 \ucc38\uc5ec\ud558\uba70, \uc774\uc640 \ud568\uaed8 2009\ub144 \uac1c\ubd09\ud558\ub294 \ub4dc\ub9bc\uc6cd\uc2a4\uc758 \uc560\ub2c8\uba54\uc774\uc158\uc778 \u300a \ubaac\uc2a4\ud130 vs \uc5d0\uc77c\ub9ac\uc5b8 \u300b\uc758 \ubaa9\uc18c\ub9ac \uc5f0\uae30\uc5d0\ub3c4 \ucc38\uac00\ud558\uace0 \uc788\ub294 \uac83\uc73c\ub85c \uc54c\ub824\uc838 \uc788\ub2e4.", "paragraph_sentence": "2008\ub144 \uc911\ubc18\uae30\uc5d0 \uc811\uc5b4\ub4e4\uba74\uc11c \uadf8\ub140\ub294 \uc720\ub2c8\ubc84\uc124 \uc2a4\ud29c\ub514\uc624\uc758 1939\ub144 \uc791\ud488\uc778 \u300a\ubbf8\ub4dc\ub098\uc774\ud2b8\u300b\ub97c \ub9ac\uba54\uc774\ud06c\ud55c \uc791\ud488\uc5d0 \ucd9c\ud604\ud558\uae30\ub85c \uacb0\uc815\ud588\ub2e4. 2009\ub144\uc5d0\ub294 \ucc98\uc74c\uc73c\ub85c \ud638\ub7ec\ubb3c\uc5d0 \ub3c4\uc804\ud558\uac8c \ud588\uc73c\uba70, \u300a\uae08\ubc1c\uc774 \ub108\ubb34\ud574 2\u300b\uc5d0 \uacf5\ub3d9 \uc81c\uc791\uc790\ub85c \ud568\uaed8 \ud588\ub358 \uc81c\ub2c8\ud37c \uc2ec\uc2a8\uacfc \ud568\uaed8\ud558\uac8c \ub418\uc5c8\ub2e4. \uc704\ub354\uc2a4\ud47c\uc740 \uc790\uc2e0\uc774 \uc9c1\uc811 \uc18c\uc720\ud558\uace0 \uc788\ub294 \ud0c0\uc774\ud504 \uc5d0\uc774\uc0ac\uac00 \uc9c1\uc811 \uc81c\uc791\uc5d0 \ucc38\uc5ec\ud558\uba70, \uc774\uc640 \ud568\uaed8 2009\ub144 \uac1c\ubd09\ud558\ub294 \ub4dc\ub9bc\uc6cd\uc2a4\uc758 \uc560\ub2c8\uba54\uc774\uc158\uc778 \u300a \ubaac\uc2a4\ud130 vs \uc5d0\uc77c\ub9ac\uc5b8 \u300b\uc758 \ubaa9\uc18c\ub9ac \uc5f0\uae30\uc5d0\ub3c4 \ucc38\uac00\ud558\uace0 \uc788\ub294 \uac83\uc73c\ub85c \uc54c\ub824\uc838 \uc788\ub2e4. \ub2e4\ub978 \uc131\uc6b0 \uc5f0\uae30\ub85c\ub294 2011\ub144 \ud06c\ub9ac\uc2a4\ub9c8\uc2a4\uc5d0 \ub9de\ucdb0 \uac1c\ubd09\ud560 \ud53d\uc0ac \uc560\ub2c8\uba54\uc774\uc158 \uc2a4\ud29c\ub514\uc624\uc758 \u300a\uba54\ub9ac\ub2e4\uc640 \ub9c8\ubc95\uc758 \uc232\u300b\uc774 \uc788\uc73c\uba70, \uc774 \uc601\ud654\ub294 \ud53d\uc0ac\uc5d0\uc11c \uc81c\uc791, \uc6d4\ud2b8 \ub514\uc988\ub2c8\uc5d0\uc11c \ubc30\uae09\uc774 \uacb0\uc815\ub418\uc5c8\ub2e4.", "paragraph_answer": "2008\ub144 \uc911\ubc18\uae30\uc5d0 \uc811\uc5b4\ub4e4\uba74\uc11c \uadf8\ub140\ub294 \uc720\ub2c8\ubc84\uc124 \uc2a4\ud29c\ub514\uc624\uc758 1939\ub144 \uc791\ud488\uc778 \u300a\ubbf8\ub4dc\ub098\uc774\ud2b8\u300b\ub97c \ub9ac\uba54\uc774\ud06c\ud55c \uc791\ud488\uc5d0 \ucd9c\ud604\ud558\uae30\ub85c \uacb0\uc815\ud588\ub2e4. 2009\ub144\uc5d0\ub294 \ucc98\uc74c\uc73c\ub85c \ud638\ub7ec\ubb3c\uc5d0 \ub3c4\uc804\ud558\uac8c \ud588\uc73c\uba70, \u300a\uae08\ubc1c\uc774 \ub108\ubb34\ud574 2\u300b\uc5d0 \uacf5\ub3d9 \uc81c\uc791\uc790\ub85c \ud568\uaed8 \ud588\ub358 \uc81c\ub2c8\ud37c \uc2ec\uc2a8\uacfc \ud568\uaed8\ud558\uac8c \ub418\uc5c8\ub2e4. \uc704\ub354\uc2a4\ud47c\uc740 \uc790\uc2e0\uc774 \uc9c1\uc811 \uc18c\uc720\ud558\uace0 \uc788\ub294 \ud0c0\uc774\ud504 \uc5d0\uc774\uc0ac\uac00 \uc9c1\uc811 \uc81c\uc791\uc5d0 \ucc38\uc5ec\ud558\uba70, \uc774\uc640 \ud568\uaed8 2009\ub144 \uac1c\ubd09\ud558\ub294 \ub4dc\ub9bc\uc6cd\uc2a4\uc758 \uc560\ub2c8\uba54\uc774\uc158\uc778 \u300a \ubaac\uc2a4\ud130 vs \uc5d0\uc77c\ub9ac\uc5b8 \u300b\uc758 \ubaa9\uc18c\ub9ac \uc5f0\uae30\uc5d0\ub3c4 \ucc38\uac00\ud558\uace0 \uc788\ub294 \uac83\uc73c\ub85c \uc54c\ub824\uc838 \uc788\ub2e4. \ub2e4\ub978 \uc131\uc6b0 \uc5f0\uae30\ub85c\ub294 2011\ub144 \ud06c\ub9ac\uc2a4\ub9c8\uc2a4\uc5d0 \ub9de\ucdb0 \uac1c\ubd09\ud560 \ud53d\uc0ac \uc560\ub2c8\uba54\uc774\uc158 \uc2a4\ud29c\ub514\uc624\uc758 \u300a\uba54\ub9ac\ub2e4\uc640 \ub9c8\ubc95\uc758 \uc232\u300b\uc774 \uc788\uc73c\uba70, \uc774 \uc601\ud654\ub294 \ud53d\uc0ac\uc5d0\uc11c \uc81c\uc791, \uc6d4\ud2b8 \ub514\uc988\ub2c8\uc5d0\uc11c \ubc30\uae09\uc774 \uacb0\uc815\ub418\uc5c8\ub2e4.", "sentence_answer": "\uc704\ub354\uc2a4\ud47c\uc740 \uc790\uc2e0\uc774 \uc9c1\uc811 \uc18c\uc720\ud558\uace0 \uc788\ub294 \ud0c0\uc774\ud504 \uc5d0\uc774\uc0ac\uac00 \uc9c1\uc811 \uc81c\uc791\uc5d0 \ucc38\uc5ec\ud558\uba70, \uc774\uc640 \ud568\uaed8 2009\ub144 \uac1c\ubd09\ud558\ub294 \ub4dc\ub9bc\uc6cd\uc2a4\uc758 \uc560\ub2c8\uba54\uc774\uc158\uc778 \u300a \ubaac\uc2a4\ud130 vs \uc5d0\uc77c\ub9ac\uc5b8 \u300b\uc758 \ubaa9\uc18c\ub9ac \uc5f0\uae30\uc5d0\ub3c4 \ucc38\uac00\ud558\uace0 \uc788\ub294 \uac83\uc73c\ub85c \uc54c\ub824\uc838 \uc788\ub2e4.", "paragraph_id": "6513888-9-1", "paragraph_question": "question: \ub9ac\uc988 \uc704\ub354\uc2a4\ud47c\uc774 \ubaa9\uc18c\ub9ac \uc5f0\uae30\ub85c \ucc38\uac00\ud55c \ub4dc\ub9bc\uc6cd\uc2a4 \uc560\ub2c8\uba54\uc774\uc158 \uc601\ud654\ub294 \ubb34\uc5c7\uc778\uac00?, context: 2008\ub144 \uc911\ubc18\uae30\uc5d0 \uc811\uc5b4\ub4e4\uba74\uc11c \uadf8\ub140\ub294 \uc720\ub2c8\ubc84\uc124 \uc2a4\ud29c\ub514\uc624\uc758 1939\ub144 \uc791\ud488\uc778 \u300a\ubbf8\ub4dc\ub098\uc774\ud2b8\u300b\ub97c \ub9ac\uba54\uc774\ud06c\ud55c \uc791\ud488\uc5d0 \ucd9c\ud604\ud558\uae30\ub85c \uacb0\uc815\ud588\ub2e4. 2009\ub144\uc5d0\ub294 \ucc98\uc74c\uc73c\ub85c \ud638\ub7ec\ubb3c\uc5d0 \ub3c4\uc804\ud558\uac8c \ud588\uc73c\uba70, \u300a\uae08\ubc1c\uc774 \ub108\ubb34\ud574 2\u300b\uc5d0 \uacf5\ub3d9 \uc81c\uc791\uc790\ub85c \ud568\uaed8 \ud588\ub358 \uc81c\ub2c8\ud37c \uc2ec\uc2a8\uacfc \ud568\uaed8\ud558\uac8c \ub418\uc5c8\ub2e4. \uc704\ub354\uc2a4\ud47c\uc740 \uc790\uc2e0\uc774 \uc9c1\uc811 \uc18c\uc720\ud558\uace0 \uc788\ub294 \ud0c0\uc774\ud504 \uc5d0\uc774\uc0ac\uac00 \uc9c1\uc811 \uc81c\uc791\uc5d0 \ucc38\uc5ec\ud558\uba70, \uc774\uc640 \ud568\uaed8 2009\ub144 \uac1c\ubd09\ud558\ub294 \ub4dc\ub9bc\uc6cd\uc2a4\uc758 \uc560\ub2c8\uba54\uc774\uc158\uc778 \u300a\ubaac\uc2a4\ud130 vs \uc5d0\uc77c\ub9ac\uc5b8\u300b\uc758 \ubaa9\uc18c\ub9ac \uc5f0\uae30\uc5d0\ub3c4 \ucc38\uac00\ud558\uace0 \uc788\ub294 \uac83\uc73c\ub85c \uc54c\ub824\uc838 \uc788\ub2e4. \ub2e4\ub978 \uc131\uc6b0 \uc5f0\uae30\ub85c\ub294 2011\ub144 \ud06c\ub9ac\uc2a4\ub9c8\uc2a4\uc5d0 \ub9de\ucdb0 \uac1c\ubd09\ud560 \ud53d\uc0ac \uc560\ub2c8\uba54\uc774\uc158 \uc2a4\ud29c\ub514\uc624\uc758 \u300a\uba54\ub9ac\ub2e4\uc640 \ub9c8\ubc95\uc758 \uc232\u300b\uc774 \uc788\uc73c\uba70, \uc774 \uc601\ud654\ub294 \ud53d\uc0ac\uc5d0\uc11c \uc81c\uc791, \uc6d4\ud2b8 \ub514\uc988\ub2c8\uc5d0\uc11c \ubc30\uae09\uc774 \uacb0\uc815\ub418\uc5c8\ub2e4."} {"question": "\ucfe0\ub370\ud0c0\ub97c \uacc4\ud68d\ud55c \uc77c\uc2ec\ud68c\uc758 \uc8fc\ub3d9\uc778\ubb3c\uc740 \uc720\uae38\uc900\uacfc \uc5b4\ub5a4 \ud559\uad50 \ucd9c\uc2e0 \uc7a5\uad50\ub4e4\uc778\uac00?", "paragraph": "\uadf8\ub294 \uc678\uad6d \uc138\ub825\uc5d0 \ub300\ud55c \ub9f9\uc2e0\uacfc \ub9f9\ubaa9\uc801\uc778 \ube44\ud310\uc740 \uc704\ud5d8\ud558\ub2e4\uace0 \ubd24\ub2e4. \uc720\uae38\uc900\uc740 \ubbf8\uad6d\uc740 \ud1b5\uc0c1 \uc0c1\ub300\uc5d0 \ubd88\uacfc\ud560 \ubfd0\uc774\uba70, \u201c\uc704\uae09\ud568\uc744 \ubc18\ub4dc\uc2dc \uad6c\ud574\uc8fc\ub294 \uc6b0\ubc29\uc73c\ub85c \ubbff\uc744 \ubc14 \ubabb \ub41c\ub2e4\u201d\uace0\ub77c \uc77c\uac08\ud558\uc5ec, \u201c\uc57d\uc790\ub97c \ub3d5\ub294 \uc815\uc758\uc758 \ub098\ub77c\u201d\ub77c\ub294 \ubbf8\uad6d\uc5d0 \ub300\ud55c \ub3d9\uc2dc\ub300\uc778\ub4e4\uc758 \ud53c\uc0c1\uc801 \uc778\uc2dd\uc5d0 \uacbd\uc885\uc744 \uc6b8\ub838\ub2e4. \ubbf8\uad6d\ub3c4 \uc790\uad6d\uc758 \uc774\uc775\uc5d0 \ubd80\ud569\ud558\uc9c0 \uc54a\ub294\ub2e4\uba74 \uc870\uc120\uc744 \ub3c4\uc6b8 \uc774\uc720\uac00 \uc5c6\ub2e4\ub294 \uac83\uc774 \uadf8\uc758 \uacac\ud574\uc600\ub2e4. \uc744\uc0ac\uc870\uc57d\uacfc \uac00\uc4f0\ub77c-\ud0dc\ud504\ud2b8 \ubc00\uc57d \uc774\ud6c4 \ubbf8\uad6d\uc5d0 \ub300\ud55c \uadf8\uc758 \uc758\ud639\uc740 \ud55c\uce35 \ub354\ud574\uac14\ub2e4. \ud55c\ud3b8 \uadf8\ub294 \uc77c\ubcf8\uc721\uad70\uc0ac\uad00\ud559\uad50 \ucd9c\uc2e0\uc758 \ud55c\uad6d\uc778 \uccad\ub144\uc7a5\uad50\ub4e4\uc774 \uc870\uc9c1\ud55c \uc77c\uc2ec\ud68c(\u4e00\u5fc3\u6703)\uc640 \uc5f0\uacb0\uc744 \ucd94\uc9c4, \uc774\ub4e4\uacfc \ud568\uaed8 \ucfe0\ub370\ud0c0\ub97c \uae30\ub3c4\ud55c\ub2e4. \uc774\ub7ec\ud55c \uadf8\uc758 \uc74c\ubaa8 \uacc4\ud68d\uc774 \ubc88\ubc88\ud788 \uc218\ud3ec\ub85c \ub3cc\uc544\uac00\uace0, \uc774\uac83\uc774 \uc870\uc120\uc73c\ub85c \uc54c\ub824\uc9c0\uba74\uc11c \uc77c\ubcf8\uacfc \uc870\uc120 \uc591\uad6d\uac04\uc758 \uc678\uad50 \ubd84\uaddc\ub85c \ube44\ud654\ub41c\ub2e4.", "answer": "\uc77c\ubcf8\uc721\uad70\uc0ac\uad00\ud559\uad50", "sentence": "\uadf8\ub294 \uc77c\ubcf8\uc721\uad70\uc0ac\uad00\ud559\uad50 \ucd9c\uc2e0\uc758 \ud55c\uad6d\uc778 \uccad\ub144\uc7a5\uad50\ub4e4\uc774 \uc870\uc9c1\ud55c \uc77c\uc2ec\ud68c(\u4e00\u5fc3\u6703)\uc640 \uc5f0\uacb0\uc744 \ucd94\uc9c4, \uc774\ub4e4\uacfc \ud568\uaed8 \ucfe0\ub370\ud0c0\ub97c \uae30\ub3c4\ud55c\ub2e4.", "paragraph_sentence": "\uadf8\ub294 \uc678\uad6d \uc138\ub825\uc5d0 \ub300\ud55c \ub9f9\uc2e0\uacfc \ub9f9\ubaa9\uc801\uc778 \ube44\ud310\uc740 \uc704\ud5d8\ud558\ub2e4\uace0 \ubd24\ub2e4. \uc720\uae38\uc900\uc740 \ubbf8\uad6d\uc740 \ud1b5\uc0c1 \uc0c1\ub300\uc5d0 \ubd88\uacfc\ud560 \ubfd0\uc774\uba70, \u201c\uc704\uae09\ud568\uc744 \ubc18\ub4dc\uc2dc \uad6c\ud574\uc8fc\ub294 \uc6b0\ubc29\uc73c\ub85c \ubbff\uc744 \ubc14 \ubabb \ub41c\ub2e4\u201d\uace0\ub77c \uc77c\uac08\ud558\uc5ec, \u201c\uc57d\uc790\ub97c \ub3d5\ub294 \uc815\uc758\uc758 \ub098\ub77c\u201d\ub77c\ub294 \ubbf8\uad6d\uc5d0 \ub300\ud55c \ub3d9\uc2dc\ub300\uc778\ub4e4\uc758 \ud53c\uc0c1\uc801 \uc778\uc2dd\uc5d0 \uacbd\uc885\uc744 \uc6b8\ub838\ub2e4. \ubbf8\uad6d\ub3c4 \uc790\uad6d\uc758 \uc774\uc775\uc5d0 \ubd80\ud569\ud558\uc9c0 \uc54a\ub294\ub2e4\uba74 \uc870\uc120\uc744 \ub3c4\uc6b8 \uc774\uc720\uac00 \uc5c6\ub2e4\ub294 \uac83\uc774 \uadf8\uc758 \uacac\ud574\uc600\ub2e4. \uc744\uc0ac\uc870\uc57d\uacfc \uac00\uc4f0\ub77c-\ud0dc\ud504\ud2b8 \ubc00\uc57d \uc774\ud6c4 \ubbf8\uad6d\uc5d0 \ub300\ud55c \uadf8\uc758 \uc758\ud639\uc740 \ud55c\uce35 \ub354\ud574\uac14\ub2e4. \ud55c\ud3b8 \uadf8\ub294 \uc77c\ubcf8\uc721\uad70\uc0ac\uad00\ud559\uad50 \ucd9c\uc2e0\uc758 \ud55c\uad6d\uc778 \uccad\ub144\uc7a5\uad50\ub4e4\uc774 \uc870\uc9c1\ud55c \uc77c\uc2ec\ud68c(\u4e00\u5fc3\u6703)\uc640 \uc5f0\uacb0\uc744 \ucd94\uc9c4, \uc774\ub4e4\uacfc \ud568\uaed8 \ucfe0\ub370\ud0c0\ub97c \uae30\ub3c4\ud55c\ub2e4. \uc774\ub7ec\ud55c \uadf8\uc758 \uc74c\ubaa8 \uacc4\ud68d\uc774 \ubc88\ubc88\ud788 \uc218\ud3ec\ub85c \ub3cc\uc544\uac00\uace0, \uc774\uac83\uc774 \uc870\uc120\uc73c\ub85c \uc54c\ub824\uc9c0\uba74\uc11c \uc77c\ubcf8\uacfc \uc870\uc120 \uc591\uad6d\uac04\uc758 \uc678\uad50 \ubd84\uaddc\ub85c \ube44\ud654\ub41c\ub2e4.", "paragraph_answer": "\uadf8\ub294 \uc678\uad6d \uc138\ub825\uc5d0 \ub300\ud55c \ub9f9\uc2e0\uacfc \ub9f9\ubaa9\uc801\uc778 \ube44\ud310\uc740 \uc704\ud5d8\ud558\ub2e4\uace0 \ubd24\ub2e4. \uc720\uae38\uc900\uc740 \ubbf8\uad6d\uc740 \ud1b5\uc0c1 \uc0c1\ub300\uc5d0 \ubd88\uacfc\ud560 \ubfd0\uc774\uba70, \u201c\uc704\uae09\ud568\uc744 \ubc18\ub4dc\uc2dc \uad6c\ud574\uc8fc\ub294 \uc6b0\ubc29\uc73c\ub85c \ubbff\uc744 \ubc14 \ubabb \ub41c\ub2e4\u201d\uace0\ub77c \uc77c\uac08\ud558\uc5ec, \u201c\uc57d\uc790\ub97c \ub3d5\ub294 \uc815\uc758\uc758 \ub098\ub77c\u201d\ub77c\ub294 \ubbf8\uad6d\uc5d0 \ub300\ud55c \ub3d9\uc2dc\ub300\uc778\ub4e4\uc758 \ud53c\uc0c1\uc801 \uc778\uc2dd\uc5d0 \uacbd\uc885\uc744 \uc6b8\ub838\ub2e4. \ubbf8\uad6d\ub3c4 \uc790\uad6d\uc758 \uc774\uc775\uc5d0 \ubd80\ud569\ud558\uc9c0 \uc54a\ub294\ub2e4\uba74 \uc870\uc120\uc744 \ub3c4\uc6b8 \uc774\uc720\uac00 \uc5c6\ub2e4\ub294 \uac83\uc774 \uadf8\uc758 \uacac\ud574\uc600\ub2e4. \uc744\uc0ac\uc870\uc57d\uacfc \uac00\uc4f0\ub77c-\ud0dc\ud504\ud2b8 \ubc00\uc57d \uc774\ud6c4 \ubbf8\uad6d\uc5d0 \ub300\ud55c \uadf8\uc758 \uc758\ud639\uc740 \ud55c\uce35 \ub354\ud574\uac14\ub2e4. \ud55c\ud3b8 \uadf8\ub294 \uc77c\ubcf8\uc721\uad70\uc0ac\uad00\ud559\uad50 \ucd9c\uc2e0\uc758 \ud55c\uad6d\uc778 \uccad\ub144\uc7a5\uad50\ub4e4\uc774 \uc870\uc9c1\ud55c \uc77c\uc2ec\ud68c(\u4e00\u5fc3\u6703)\uc640 \uc5f0\uacb0\uc744 \ucd94\uc9c4, \uc774\ub4e4\uacfc \ud568\uaed8 \ucfe0\ub370\ud0c0\ub97c \uae30\ub3c4\ud55c\ub2e4. \uc774\ub7ec\ud55c \uadf8\uc758 \uc74c\ubaa8 \uacc4\ud68d\uc774 \ubc88\ubc88\ud788 \uc218\ud3ec\ub85c \ub3cc\uc544\uac00\uace0, \uc774\uac83\uc774 \uc870\uc120\uc73c\ub85c \uc54c\ub824\uc9c0\uba74\uc11c \uc77c\ubcf8\uacfc \uc870\uc120 \uc591\uad6d\uac04\uc758 \uc678\uad50 \ubd84\uaddc\ub85c \ube44\ud654\ub41c\ub2e4.", "sentence_answer": "\uadf8\ub294 \uc77c\ubcf8\uc721\uad70\uc0ac\uad00\ud559\uad50 \ucd9c\uc2e0\uc758 \ud55c\uad6d\uc778 \uccad\ub144\uc7a5\uad50\ub4e4\uc774 \uc870\uc9c1\ud55c \uc77c\uc2ec\ud68c(\u4e00\u5fc3\u6703)\uc640 \uc5f0\uacb0\uc744 \ucd94\uc9c4, \uc774\ub4e4\uacfc \ud568\uaed8 \ucfe0\ub370\ud0c0\ub97c \uae30\ub3c4\ud55c\ub2e4.", "paragraph_id": "6542749-19-1", "paragraph_question": "question: \ucfe0\ub370\ud0c0\ub97c \uacc4\ud68d\ud55c \uc77c\uc2ec\ud68c\uc758 \uc8fc\ub3d9\uc778\ubb3c\uc740 \uc720\uae38\uc900\uacfc \uc5b4\ub5a4 \ud559\uad50 \ucd9c\uc2e0 \uc7a5\uad50\ub4e4\uc778\uac00?, context: \uadf8\ub294 \uc678\uad6d \uc138\ub825\uc5d0 \ub300\ud55c \ub9f9\uc2e0\uacfc \ub9f9\ubaa9\uc801\uc778 \ube44\ud310\uc740 \uc704\ud5d8\ud558\ub2e4\uace0 \ubd24\ub2e4. \uc720\uae38\uc900\uc740 \ubbf8\uad6d\uc740 \ud1b5\uc0c1 \uc0c1\ub300\uc5d0 \ubd88\uacfc\ud560 \ubfd0\uc774\uba70, \u201c\uc704\uae09\ud568\uc744 \ubc18\ub4dc\uc2dc \uad6c\ud574\uc8fc\ub294 \uc6b0\ubc29\uc73c\ub85c \ubbff\uc744 \ubc14 \ubabb \ub41c\ub2e4\u201d\uace0\ub77c \uc77c\uac08\ud558\uc5ec, \u201c\uc57d\uc790\ub97c \ub3d5\ub294 \uc815\uc758\uc758 \ub098\ub77c\u201d\ub77c\ub294 \ubbf8\uad6d\uc5d0 \ub300\ud55c \ub3d9\uc2dc\ub300\uc778\ub4e4\uc758 \ud53c\uc0c1\uc801 \uc778\uc2dd\uc5d0 \uacbd\uc885\uc744 \uc6b8\ub838\ub2e4. \ubbf8\uad6d\ub3c4 \uc790\uad6d\uc758 \uc774\uc775\uc5d0 \ubd80\ud569\ud558\uc9c0 \uc54a\ub294\ub2e4\uba74 \uc870\uc120\uc744 \ub3c4\uc6b8 \uc774\uc720\uac00 \uc5c6\ub2e4\ub294 \uac83\uc774 \uadf8\uc758 \uacac\ud574\uc600\ub2e4. \uc744\uc0ac\uc870\uc57d\uacfc \uac00\uc4f0\ub77c-\ud0dc\ud504\ud2b8 \ubc00\uc57d \uc774\ud6c4 \ubbf8\uad6d\uc5d0 \ub300\ud55c \uadf8\uc758 \uc758\ud639\uc740 \ud55c\uce35 \ub354\ud574\uac14\ub2e4. \ud55c\ud3b8 \uadf8\ub294 \uc77c\ubcf8\uc721\uad70\uc0ac\uad00\ud559\uad50 \ucd9c\uc2e0\uc758 \ud55c\uad6d\uc778 \uccad\ub144\uc7a5\uad50\ub4e4\uc774 \uc870\uc9c1\ud55c \uc77c\uc2ec\ud68c(\u4e00\u5fc3\u6703)\uc640 \uc5f0\uacb0\uc744 \ucd94\uc9c4, \uc774\ub4e4\uacfc \ud568\uaed8 \ucfe0\ub370\ud0c0\ub97c \uae30\ub3c4\ud55c\ub2e4. \uc774\ub7ec\ud55c \uadf8\uc758 \uc74c\ubaa8 \uacc4\ud68d\uc774 \ubc88\ubc88\ud788 \uc218\ud3ec\ub85c \ub3cc\uc544\uac00\uace0, \uc774\uac83\uc774 \uc870\uc120\uc73c\ub85c \uc54c\ub824\uc9c0\uba74\uc11c \uc77c\ubcf8\uacfc \uc870\uc120 \uc591\uad6d\uac04\uc758 \uc678\uad50 \ubd84\uaddc\ub85c \ube44\ud654\ub41c\ub2e4."} {"question": "2018\ub144 3\uc6d4 30\uc77c \uc2dc\uc9c4\ud551 \uc8fc\uc11d\uc758 \ud2b9\ubcc4\ub300\ud45c \uc790\uaca9\uc73c\ub85c \ubb38\uc7ac\uc778\uc744 \ub9cc\ub09c \uc0ac\ub78c\uc740?", "paragraph": "2018\ub144 3\uc6d4 30\uc77c \uc2dc\uc9c4\ud551 \uc8fc\uc11d\uc758 \ud2b9\ubcc4\ub300\ud45c \uc790\uaca9\uc73c\ub85c \uc591\uc81c\uce20 \uc678\uad50\ub2f4\ub2f9 \uc911\uad6d\uacf5\uc0b0\ub2f9 \uc911\uc559\uc815\uce58\uad6d \uc704\uc6d0\uc774 \ubc29\ud55c\ud558\uc5ec \ubb38\uc7ac\uc778\uc744 \ub9cc\ub0ac\ub294\ub370 \uc774 \uc790\ub9ac\uc5d0\uc11c \uc911\uad6d\uc758 \ub2e8\uccb4\uad00\uad11 \uc815\uc0c1\ud654 \ub4f1 \ubb38\uc7ac\uc778\uc758 \uad00\uc2ec\uc0ac\ud56d\uc744 \uc911\uad6d\ub3c4 \uc911\uc694\uc2dc\ud558\uace0 \uc788\ub2e4\uba70 \ube60\ub978 \uc2dc\uc77c \uc548\uc5d0 \uac00\uc2dc\uc801 \uc131\uacfc\ub97c \ubcf4\uac8c\ub420 \uac83\uc774\ub77c\uace0 \ud588\ub2e4. \uc774\ub294 \uc0ac\ub4dc \ubc30\uce58\uc5d0 \ub300\ud55c \uacbd\uc81c\ubcf4\ubcf5 \uc870\uce58\ub97c \uc0ac\uc2e4\uc0c1 \ucca0\ud68c\ud55c\ub2e4\ub294 \uc785\uc7a5\uc73c\ub85c \ubc1b\uc544\ub4e4\uc5ec\uc84c\ub2e4. \ud558\uc9c0\ub9cc 18\uac1c\uc6d4 \uc911\ub2e8\ub418\uc5c8\ub358 \uc120\uc591\uc2dc\uc758 \ub86f\ub370\uc6d4\ub4dc \uac74\uc124\ub3c4 \uc7ac\uac1c\ud558\uae30\ub85c \ud588\ub294\ub370 \ub178\uc601\ubbfc \uc911\uad6d \ub300\uc0ac\uac00 4\uc6d4 23\uc77c \ub7b4\uc624\ub2dd\uc131\uc744 \ubc29\ubb38\ud558\ub824\ub358 \uacc4\ud68d\uc774 \uc5f0\uae30\ub418\uc5c8\ub2e4. \uc774\uc5d0 \ub86f\ub370\uc6d4\ub4dc \uac74\uc124\uacfc \uad00\ub828\ud574 \uc544\uc9c1 \ud574\uacb0\ud558\uae30\uc5d0 \uc2dc\uac04\uc774 \ud544\uc694\ud55c\ub370 \uc774 \ubb38\uc81c\ub97c \ub2f9\uc7a5 \ub2e4\ub8e8\uae30\ub97c \uc911\uad6d\uc774 \uaebc\ub824\ud55c \uac83\uc774 \uc544\ub2c8\ub0d0\ub294 \uc758\uacac\uc774 \uc81c\uae30\ub418\uc5c8\ub2e4. \ud558\uc9c0\ub9cc \ub178\uc601\ubbfc\uc758 \uc120\uc591\uc2dc \ubc29\ubb38\uc740 2017\ub144 11\uc6d4\uc5d0\ub3c4 \uc608\uc815\ub418\uc5b4 \uc788\uc5c8\uc9c0\ub9cc \uc774\ub54c\ub3c4 \ucd5c\uc18c\ub418\uc5c8\uc5c8\ub2e4. \ub450 \ucc28\ub840 \ubaa8\ub450 \ub0a9\ub4dd\ud560 \ub9cc\ud55c \uc774\uc720\ub97c \uc81c\uc2dc\ud558\uc9c0\ub3c4 \uc54a\uc558\ub294\ub370 \ub300\uc0ac\uad00\uc740 \"\uc911\uad6d\uc740 \uc6d0\ub798 \uadf8\ub807\ub2e4\"\ub294 \ubc18\uc751\uc744 \ubcf4\uc600\uc73c\uba70, \ub610\ud55c \uc0ac\ub4dc \uc598\uae30\uac00 \uad6d\uc775\uc5d0 \ub3c4\uc6c0\uc774 \uc548 \ub41c\ub2e4\uba70 \uc5b8\ub860\uc5d0\ub3c4 \uc5b8\uae09\uc744 \uc790\uc81c\ud574 \ub2ec\ub77c\uace0 \ud558\uc5ec \uc544\uc9c1 \uacbd\uc81c\ubcf4\ubcf5 \ud574\uc81c\ub294 \uba40\uc5c8\ub2e4\ub294 \uc598\uae30\uac00 \ub098\uc654\ub2e4.}}", "answer": "\uc591\uc81c\uce20", "sentence": "\ud2b9\ubcc4\ub300\ud45c \uc790\uaca9\uc73c\ub85c \uc591\uc81c\uce20 \uc678\uad50\ub2f4\ub2f9 \uc911\uad6d\uacf5\uc0b0\ub2f9 \uc911\uc559\uc815\uce58\uad6d \uc704\uc6d0\uc774 \ubc29\ud55c\ud558\uc5ec \ubb38\uc7ac\uc778\uc744 \ub9cc\ub0ac\ub294\ub370 \uc774 \uc790\ub9ac\uc5d0\uc11c \uc911\uad6d\uc758 \ub2e8\uccb4\uad00\uad11 \uc815\uc0c1\ud654 \ub4f1 \ubb38\uc7ac\uc778\uc758 \uad00\uc2ec\uc0ac\ud56d\uc744 \uc911\uad6d\ub3c4 \uc911\uc694\uc2dc\ud558\uace0 \uc788\ub2e4\uba70 \ube60\ub978 \uc2dc\uc77c \uc548\uc5d0 \uac00\uc2dc\uc801 \uc131\uacfc\ub97c \ubcf4\uac8c\ub420 \uac83\uc774\ub77c\uace0 \ud588\ub2e4.", "paragraph_sentence": "2018\ub144 3\uc6d4 30\uc77c \uc2dc\uc9c4\ud551 \uc8fc\uc11d\uc758 \ud2b9\ubcc4\ub300\ud45c \uc790\uaca9\uc73c\ub85c \uc591\uc81c\uce20 \uc678\uad50\ub2f4\ub2f9 \uc911\uad6d\uacf5\uc0b0\ub2f9 \uc911\uc559\uc815\uce58\uad6d \uc704\uc6d0\uc774 \ubc29\ud55c\ud558\uc5ec \ubb38\uc7ac\uc778\uc744 \ub9cc\ub0ac\ub294\ub370 \uc774 \uc790\ub9ac\uc5d0\uc11c \uc911\uad6d\uc758 \ub2e8\uccb4\uad00\uad11 \uc815\uc0c1\ud654 \ub4f1 \ubb38\uc7ac\uc778\uc758 \uad00\uc2ec\uc0ac\ud56d\uc744 \uc911\uad6d\ub3c4 \uc911\uc694\uc2dc\ud558\uace0 \uc788\ub2e4\uba70 \ube60\ub978 \uc2dc\uc77c \uc548\uc5d0 \uac00\uc2dc\uc801 \uc131\uacfc\ub97c \ubcf4\uac8c\ub420 \uac83\uc774\ub77c\uace0 \ud588\ub2e4. \uc774\ub294 \uc0ac\ub4dc \ubc30\uce58\uc5d0 \ub300\ud55c \uacbd\uc81c\ubcf4\ubcf5 \uc870\uce58\ub97c \uc0ac\uc2e4\uc0c1 \ucca0\ud68c\ud55c\ub2e4\ub294 \uc785\uc7a5\uc73c\ub85c \ubc1b\uc544\ub4e4\uc5ec\uc84c\ub2e4. \ud558\uc9c0\ub9cc 18\uac1c\uc6d4 \uc911\ub2e8\ub418\uc5c8\ub358 \uc120\uc591\uc2dc\uc758 \ub86f\ub370\uc6d4\ub4dc \uac74\uc124\ub3c4 \uc7ac\uac1c\ud558\uae30\ub85c \ud588\ub294\ub370 \ub178\uc601\ubbfc \uc911\uad6d \ub300\uc0ac\uac00 4\uc6d4 23\uc77c \ub7b4\uc624\ub2dd\uc131\uc744 \ubc29\ubb38\ud558\ub824\ub358 \uacc4\ud68d\uc774 \uc5f0\uae30\ub418\uc5c8\ub2e4. \uc774\uc5d0 \ub86f\ub370\uc6d4\ub4dc \uac74\uc124\uacfc \uad00\ub828\ud574 \uc544\uc9c1 \ud574\uacb0\ud558\uae30\uc5d0 \uc2dc\uac04\uc774 \ud544\uc694\ud55c\ub370 \uc774 \ubb38\uc81c\ub97c \ub2f9\uc7a5 \ub2e4\ub8e8\uae30\ub97c \uc911\uad6d\uc774 \uaebc\ub824\ud55c \uac83\uc774 \uc544\ub2c8\ub0d0\ub294 \uc758\uacac\uc774 \uc81c\uae30\ub418\uc5c8\ub2e4. \ud558\uc9c0\ub9cc \ub178\uc601\ubbfc\uc758 \uc120\uc591\uc2dc \ubc29\ubb38\uc740 2017\ub144 11\uc6d4\uc5d0\ub3c4 \uc608\uc815\ub418\uc5b4 \uc788\uc5c8\uc9c0\ub9cc \uc774\ub54c\ub3c4 \ucd5c\uc18c\ub418\uc5c8\uc5c8\ub2e4. \ub450 \ucc28\ub840 \ubaa8\ub450 \ub0a9\ub4dd\ud560 \ub9cc\ud55c \uc774\uc720\ub97c \uc81c\uc2dc\ud558\uc9c0\ub3c4 \uc54a\uc558\ub294\ub370 \ub300\uc0ac\uad00\uc740 \"\uc911\uad6d\uc740 \uc6d0\ub798 \uadf8\ub807\ub2e4\"\ub294 \ubc18\uc751\uc744 \ubcf4\uc600\uc73c\uba70, \ub610\ud55c \uc0ac\ub4dc \uc598\uae30\uac00 \uad6d\uc775\uc5d0 \ub3c4\uc6c0\uc774 \uc548 \ub41c\ub2e4\uba70 \uc5b8\ub860\uc5d0\ub3c4 \uc5b8\uae09\uc744 \uc790\uc81c\ud574 \ub2ec\ub77c\uace0 \ud558\uc5ec \uc544\uc9c1 \uacbd\uc81c\ubcf4\ubcf5 \ud574\uc81c\ub294 \uba40\uc5c8\ub2e4\ub294 \uc598\uae30\uac00 \ub098\uc654\ub2e4. }}", "paragraph_answer": "2018\ub144 3\uc6d4 30\uc77c \uc2dc\uc9c4\ud551 \uc8fc\uc11d\uc758 \ud2b9\ubcc4\ub300\ud45c \uc790\uaca9\uc73c\ub85c \uc591\uc81c\uce20 \uc678\uad50\ub2f4\ub2f9 \uc911\uad6d\uacf5\uc0b0\ub2f9 \uc911\uc559\uc815\uce58\uad6d \uc704\uc6d0\uc774 \ubc29\ud55c\ud558\uc5ec \ubb38\uc7ac\uc778\uc744 \ub9cc\ub0ac\ub294\ub370 \uc774 \uc790\ub9ac\uc5d0\uc11c \uc911\uad6d\uc758 \ub2e8\uccb4\uad00\uad11 \uc815\uc0c1\ud654 \ub4f1 \ubb38\uc7ac\uc778\uc758 \uad00\uc2ec\uc0ac\ud56d\uc744 \uc911\uad6d\ub3c4 \uc911\uc694\uc2dc\ud558\uace0 \uc788\ub2e4\uba70 \ube60\ub978 \uc2dc\uc77c \uc548\uc5d0 \uac00\uc2dc\uc801 \uc131\uacfc\ub97c \ubcf4\uac8c\ub420 \uac83\uc774\ub77c\uace0 \ud588\ub2e4. \uc774\ub294 \uc0ac\ub4dc \ubc30\uce58\uc5d0 \ub300\ud55c \uacbd\uc81c\ubcf4\ubcf5 \uc870\uce58\ub97c \uc0ac\uc2e4\uc0c1 \ucca0\ud68c\ud55c\ub2e4\ub294 \uc785\uc7a5\uc73c\ub85c \ubc1b\uc544\ub4e4\uc5ec\uc84c\ub2e4. \ud558\uc9c0\ub9cc 18\uac1c\uc6d4 \uc911\ub2e8\ub418\uc5c8\ub358 \uc120\uc591\uc2dc\uc758 \ub86f\ub370\uc6d4\ub4dc \uac74\uc124\ub3c4 \uc7ac\uac1c\ud558\uae30\ub85c \ud588\ub294\ub370 \ub178\uc601\ubbfc \uc911\uad6d \ub300\uc0ac\uac00 4\uc6d4 23\uc77c \ub7b4\uc624\ub2dd\uc131\uc744 \ubc29\ubb38\ud558\ub824\ub358 \uacc4\ud68d\uc774 \uc5f0\uae30\ub418\uc5c8\ub2e4. \uc774\uc5d0 \ub86f\ub370\uc6d4\ub4dc \uac74\uc124\uacfc \uad00\ub828\ud574 \uc544\uc9c1 \ud574\uacb0\ud558\uae30\uc5d0 \uc2dc\uac04\uc774 \ud544\uc694\ud55c\ub370 \uc774 \ubb38\uc81c\ub97c \ub2f9\uc7a5 \ub2e4\ub8e8\uae30\ub97c \uc911\uad6d\uc774 \uaebc\ub824\ud55c \uac83\uc774 \uc544\ub2c8\ub0d0\ub294 \uc758\uacac\uc774 \uc81c\uae30\ub418\uc5c8\ub2e4. \ud558\uc9c0\ub9cc \ub178\uc601\ubbfc\uc758 \uc120\uc591\uc2dc \ubc29\ubb38\uc740 2017\ub144 11\uc6d4\uc5d0\ub3c4 \uc608\uc815\ub418\uc5b4 \uc788\uc5c8\uc9c0\ub9cc \uc774\ub54c\ub3c4 \ucd5c\uc18c\ub418\uc5c8\uc5c8\ub2e4. \ub450 \ucc28\ub840 \ubaa8\ub450 \ub0a9\ub4dd\ud560 \ub9cc\ud55c \uc774\uc720\ub97c \uc81c\uc2dc\ud558\uc9c0\ub3c4 \uc54a\uc558\ub294\ub370 \ub300\uc0ac\uad00\uc740 \"\uc911\uad6d\uc740 \uc6d0\ub798 \uadf8\ub807\ub2e4\"\ub294 \ubc18\uc751\uc744 \ubcf4\uc600\uc73c\uba70, \ub610\ud55c \uc0ac\ub4dc \uc598\uae30\uac00 \uad6d\uc775\uc5d0 \ub3c4\uc6c0\uc774 \uc548 \ub41c\ub2e4\uba70 \uc5b8\ub860\uc5d0\ub3c4 \uc5b8\uae09\uc744 \uc790\uc81c\ud574 \ub2ec\ub77c\uace0 \ud558\uc5ec \uc544\uc9c1 \uacbd\uc81c\ubcf4\ubcf5 \ud574\uc81c\ub294 \uba40\uc5c8\ub2e4\ub294 \uc598\uae30\uac00 \ub098\uc654\ub2e4.}}", "sentence_answer": "\ud2b9\ubcc4\ub300\ud45c \uc790\uaca9\uc73c\ub85c \uc591\uc81c\uce20 \uc678\uad50\ub2f4\ub2f9 \uc911\uad6d\uacf5\uc0b0\ub2f9 \uc911\uc559\uc815\uce58\uad6d \uc704\uc6d0\uc774 \ubc29\ud55c\ud558\uc5ec \ubb38\uc7ac\uc778\uc744 \ub9cc\ub0ac\ub294\ub370 \uc774 \uc790\ub9ac\uc5d0\uc11c \uc911\uad6d\uc758 \ub2e8\uccb4\uad00\uad11 \uc815\uc0c1\ud654 \ub4f1 \ubb38\uc7ac\uc778\uc758 \uad00\uc2ec\uc0ac\ud56d\uc744 \uc911\uad6d\ub3c4 \uc911\uc694\uc2dc\ud558\uace0 \uc788\ub2e4\uba70 \ube60\ub978 \uc2dc\uc77c \uc548\uc5d0 \uac00\uc2dc\uc801 \uc131\uacfc\ub97c \ubcf4\uac8c\ub420 \uac83\uc774\ub77c\uace0 \ud588\ub2e4.", "paragraph_id": "6580792-12-0", "paragraph_question": "question: 2018\ub144 3\uc6d4 30\uc77c \uc2dc\uc9c4\ud551 \uc8fc\uc11d\uc758 \ud2b9\ubcc4\ub300\ud45c \uc790\uaca9\uc73c\ub85c \ubb38\uc7ac\uc778\uc744 \ub9cc\ub09c \uc0ac\ub78c\uc740?, context: 2018\ub144 3\uc6d4 30\uc77c \uc2dc\uc9c4\ud551 \uc8fc\uc11d\uc758 \ud2b9\ubcc4\ub300\ud45c \uc790\uaca9\uc73c\ub85c \uc591\uc81c\uce20 \uc678\uad50\ub2f4\ub2f9 \uc911\uad6d\uacf5\uc0b0\ub2f9 \uc911\uc559\uc815\uce58\uad6d \uc704\uc6d0\uc774 \ubc29\ud55c\ud558\uc5ec \ubb38\uc7ac\uc778\uc744 \ub9cc\ub0ac\ub294\ub370 \uc774 \uc790\ub9ac\uc5d0\uc11c \uc911\uad6d\uc758 \ub2e8\uccb4\uad00\uad11 \uc815\uc0c1\ud654 \ub4f1 \ubb38\uc7ac\uc778\uc758 \uad00\uc2ec\uc0ac\ud56d\uc744 \uc911\uad6d\ub3c4 \uc911\uc694\uc2dc\ud558\uace0 \uc788\ub2e4\uba70 \ube60\ub978 \uc2dc\uc77c \uc548\uc5d0 \uac00\uc2dc\uc801 \uc131\uacfc\ub97c \ubcf4\uac8c\ub420 \uac83\uc774\ub77c\uace0 \ud588\ub2e4. \uc774\ub294 \uc0ac\ub4dc \ubc30\uce58\uc5d0 \ub300\ud55c \uacbd\uc81c\ubcf4\ubcf5 \uc870\uce58\ub97c \uc0ac\uc2e4\uc0c1 \ucca0\ud68c\ud55c\ub2e4\ub294 \uc785\uc7a5\uc73c\ub85c \ubc1b\uc544\ub4e4\uc5ec\uc84c\ub2e4. \ud558\uc9c0\ub9cc 18\uac1c\uc6d4 \uc911\ub2e8\ub418\uc5c8\ub358 \uc120\uc591\uc2dc\uc758 \ub86f\ub370\uc6d4\ub4dc \uac74\uc124\ub3c4 \uc7ac\uac1c\ud558\uae30\ub85c \ud588\ub294\ub370 \ub178\uc601\ubbfc \uc911\uad6d \ub300\uc0ac\uac00 4\uc6d4 23\uc77c \ub7b4\uc624\ub2dd\uc131\uc744 \ubc29\ubb38\ud558\ub824\ub358 \uacc4\ud68d\uc774 \uc5f0\uae30\ub418\uc5c8\ub2e4. \uc774\uc5d0 \ub86f\ub370\uc6d4\ub4dc \uac74\uc124\uacfc \uad00\ub828\ud574 \uc544\uc9c1 \ud574\uacb0\ud558\uae30\uc5d0 \uc2dc\uac04\uc774 \ud544\uc694\ud55c\ub370 \uc774 \ubb38\uc81c\ub97c \ub2f9\uc7a5 \ub2e4\ub8e8\uae30\ub97c \uc911\uad6d\uc774 \uaebc\ub824\ud55c \uac83\uc774 \uc544\ub2c8\ub0d0\ub294 \uc758\uacac\uc774 \uc81c\uae30\ub418\uc5c8\ub2e4. \ud558\uc9c0\ub9cc \ub178\uc601\ubbfc\uc758 \uc120\uc591\uc2dc \ubc29\ubb38\uc740 2017\ub144 11\uc6d4\uc5d0\ub3c4 \uc608\uc815\ub418\uc5b4 \uc788\uc5c8\uc9c0\ub9cc \uc774\ub54c\ub3c4 \ucd5c\uc18c\ub418\uc5c8\uc5c8\ub2e4. \ub450 \ucc28\ub840 \ubaa8\ub450 \ub0a9\ub4dd\ud560 \ub9cc\ud55c \uc774\uc720\ub97c \uc81c\uc2dc\ud558\uc9c0\ub3c4 \uc54a\uc558\ub294\ub370 \ub300\uc0ac\uad00\uc740 \"\uc911\uad6d\uc740 \uc6d0\ub798 \uadf8\ub807\ub2e4\"\ub294 \ubc18\uc751\uc744 \ubcf4\uc600\uc73c\uba70, \ub610\ud55c \uc0ac\ub4dc \uc598\uae30\uac00 \uad6d\uc775\uc5d0 \ub3c4\uc6c0\uc774 \uc548 \ub41c\ub2e4\uba70 \uc5b8\ub860\uc5d0\ub3c4 \uc5b8\uae09\uc744 \uc790\uc81c\ud574 \ub2ec\ub77c\uace0 \ud558\uc5ec \uc544\uc9c1 \uacbd\uc81c\ubcf4\ubcf5 \ud574\uc81c\ub294 \uba40\uc5c8\ub2e4\ub294 \uc598\uae30\uac00 \ub098\uc654\ub2e4.}}"} {"question": "\ud5cc\ubc95\uc758 \uc774\ub150\uacfc \uac00\uce58\ub294 \ubb34\uc2a8 \uc120\uc5b8\uc744 \ud1a0\ub300\ub85c \ud55c \uac83\uc778\uac00?", "paragraph": "\uadf8\ub7ec\ub2e4 1789\ub144 \ud504\ub791\uc2a4\ud601\uba85\uc758 \uc131\uacf5\uc73c\ub85c \uc5ed\uc0ac\uc0c1 \ucd5c\ucd08\ub85c \ubaa8\ub4e0 \uc0ac\ub78c\ub4e4\uc774 \uc2e0\ubd84\uc758 \uc871\uc1c4\ub85c\ubd80\ud130 \ud574\ubc29\ub418\uc5b4 \uc9d1\ub2e8\uc801\uc73c\ub85c \ud3c9\ub4f1\ud558\uac8c \uc695\ub9dd\uc758 \ubd84\ucd9c\uc744 \uacbd\ud5d8\ud558\uac8c \ub418\uace0 \uc790\uc720\uc640 \ud3c9\ub4f1\uc774\ub77c\ub294 \uad00\ub150\uc774 \uc2f9\ud2bc \uac83\uc774\ub2e4.\uc624\ub298\ub0a0 \uc138\uacc4\uc0ac\ud68c\ub294,\ud3c9\ub4f1\uacfc \uc790\uc720\ub77c\ub294 \uac1c\ub150\uc790\uccb4\uac00 \uc5c6\uc5c8\uc744\uc815\ub3c4\ub85c \ub300\ub2e8\ud788 \ubd88\ud589\ud558\uace0 \ube44\uc778\uac04\uc801\uc778,\ubd80\uc790\uc720\ud558\uace0 \ubd88\ud3c9\ub4f1\ud55c \uc5ed\uc0ac\ub97c \ub2e4\uc2dc\ub294 \ub418\ud480\uc774 \ud558\uc9c0\uc54a\uae30\uc704\ud574 \uc790\uc720\uc640 \ud3c9\ub4f1\uc744 \ucc9c\ubd80\uc778\uad8c\uc801 \uad8c\ub9ac\ub85c \uaddc\uc815\ud558\uace0 \ud5cc\ubc95\uc5d0 \uac01\uc778\uc2dc\ucf1c \uc5b4\ub5a0\ud55c \uc774\uc720\ub85c\ub3c4 \uce68\ud574\ud558\uac70\ub098 \ubd80\uc815\ud560 \uc218\uc5c6\ub3c4\ub85d\ud588\ub2e4.\ud5cc\ubc95\uc758 \uc774\ub150\uacfc \uac00\uce58\ub294 \uc138\uacc4\uc778\uad8c\uc120\uc5b8\uc744 \ud1a0\ub300\ub85c \ud55c \uac83\uc73c\ub85c \uc0ac\ud68c\uac00 \uc9c0\ud5a5\ud574\uc57c \ud560 \ucd5c\uace0 \uc9c0\ud45c\uc778 \ub3d9\uc2dc\uc5d0 \ub204\uad6c\ub098 \ubc18\ub4dc\uc2dc \uc778\uc815\ud558\uace0 \uc9c0\ud0a4\uc9c0 \uc54a\uc73c\uba74 \uc548\ub418\ub294 \uac15\uc81c\uaddc\ubc94\uc774\ub2e4. \ud5cc\ubc95\uc740 \uc778\uac04\uc758 \ud3c9\ub4f1\uacfc \uc790\uc720(\uad8c)\uc758 \ubcf4\uc7a5\uc5d0 \ub300\ud574 \ubb34\ub824 30\uc5ec \uc870\ud56d\uc5d0 \uac78\uccd0 \ub9e4\uc6b0 \uad6c\uccb4\uc801\uc774\uace0 \uc2e4\uc99d\uc801\uc73c\ub85c \uac15\uc870\ud558\uace0 \uc788\uc744 \ubfd0\ub9cc \uc544\ub2c8\ub77c \uadf8\uc678 \uc804 \uc870\ud56d\ub3c4 \uc778\uac04\uc758 \uc790\uc720\uc640 \ud3c9\ub4f1\uc744 \uc2e4\ud604\ud558\uae30 \uc704\ud55c \uac83\uc784\uc740 \uc790\uba85\ud558\ub2e4.", "answer": "\uc138\uacc4\uc778\uad8c\uc120\uc5b8", "sentence": "\uadf8\ub7ec\ub2e4 1789\ub144 \ud504\ub791\uc2a4\ud601\uba85\uc758 \uc131\uacf5\uc73c\ub85c \uc5ed\uc0ac\uc0c1 \ucd5c\ucd08\ub85c \ubaa8\ub4e0 \uc0ac\ub78c\ub4e4\uc774 \uc2e0\ubd84\uc758 \uc871\uc1c4\ub85c\ubd80\ud130 \ud574\ubc29\ub418\uc5b4 \uc9d1\ub2e8\uc801\uc73c\ub85c \ud3c9\ub4f1\ud558\uac8c \uc695\ub9dd\uc758 \ubd84\ucd9c\uc744 \uacbd\ud5d8\ud558\uac8c \ub418\uace0 \uc790\uc720\uc640 \ud3c9\ub4f1\uc774\ub77c\ub294 \uad00\ub150\uc774 \uc2f9\ud2bc \uac83\uc774\ub2e4.\uc624\ub298\ub0a0 \uc138\uacc4\uc0ac\ud68c\ub294,\ud3c9\ub4f1\uacfc \uc790\uc720\ub77c\ub294 \uac1c\ub150\uc790\uccb4\uac00 \uc5c6\uc5c8\uc744\uc815\ub3c4\ub85c \ub300\ub2e8\ud788 \ubd88\ud589\ud558\uace0 \ube44\uc778\uac04\uc801\uc778,\ubd80\uc790\uc720\ud558\uace0 \ubd88\ud3c9\ub4f1\ud55c \uc5ed\uc0ac\ub97c \ub2e4\uc2dc\ub294 \ub418\ud480\uc774 \ud558\uc9c0\uc54a\uae30\uc704\ud574 \uc790\uc720\uc640 \ud3c9\ub4f1\uc744 \ucc9c\ubd80\uc778\uad8c\uc801 \uad8c\ub9ac\ub85c \uaddc\uc815\ud558\uace0 \ud5cc\ubc95\uc5d0 \uac01\uc778\uc2dc\ucf1c \uc5b4\ub5a0\ud55c \uc774\uc720\ub85c\ub3c4 \uce68\ud574\ud558\uac70\ub098 \ubd80\uc815\ud560 \uc218\uc5c6\ub3c4\ub85d\ud588\ub2e4.\ud5cc\ubc95\uc758 \uc774\ub150\uacfc \uac00\uce58\ub294 \uc138\uacc4\uc778\uad8c\uc120\uc5b8 \uc744 \ud1a0\ub300\ub85c \ud55c \uac83\uc73c\ub85c \uc0ac\ud68c\uac00 \uc9c0\ud5a5\ud574\uc57c \ud560 \ucd5c\uace0 \uc9c0\ud45c\uc778 \ub3d9\uc2dc\uc5d0 \ub204\uad6c\ub098 \ubc18\ub4dc\uc2dc \uc778\uc815\ud558\uace0 \uc9c0\ud0a4\uc9c0 \uc54a\uc73c\uba74 \uc548\ub418\ub294 \uac15\uc81c\uaddc\ubc94\uc774\ub2e4.", "paragraph_sentence": " \uadf8\ub7ec\ub2e4 1789\ub144 \ud504\ub791\uc2a4\ud601\uba85\uc758 \uc131\uacf5\uc73c\ub85c \uc5ed\uc0ac\uc0c1 \ucd5c\ucd08\ub85c \ubaa8\ub4e0 \uc0ac\ub78c\ub4e4\uc774 \uc2e0\ubd84\uc758 \uc871\uc1c4\ub85c\ubd80\ud130 \ud574\ubc29\ub418\uc5b4 \uc9d1\ub2e8\uc801\uc73c\ub85c \ud3c9\ub4f1\ud558\uac8c \uc695\ub9dd\uc758 \ubd84\ucd9c\uc744 \uacbd\ud5d8\ud558\uac8c \ub418\uace0 \uc790\uc720\uc640 \ud3c9\ub4f1\uc774\ub77c\ub294 \uad00\ub150\uc774 \uc2f9\ud2bc \uac83\uc774\ub2e4.\uc624\ub298\ub0a0 \uc138\uacc4\uc0ac\ud68c\ub294,\ud3c9\ub4f1\uacfc \uc790\uc720\ub77c\ub294 \uac1c\ub150\uc790\uccb4\uac00 \uc5c6\uc5c8\uc744\uc815\ub3c4\ub85c \ub300\ub2e8\ud788 \ubd88\ud589\ud558\uace0 \ube44\uc778\uac04\uc801\uc778,\ubd80\uc790\uc720\ud558\uace0 \ubd88\ud3c9\ub4f1\ud55c \uc5ed\uc0ac\ub97c \ub2e4\uc2dc\ub294 \ub418\ud480\uc774 \ud558\uc9c0\uc54a\uae30\uc704\ud574 \uc790\uc720\uc640 \ud3c9\ub4f1\uc744 \ucc9c\ubd80\uc778\uad8c\uc801 \uad8c\ub9ac\ub85c \uaddc\uc815\ud558\uace0 \ud5cc\ubc95\uc5d0 \uac01\uc778\uc2dc\ucf1c \uc5b4\ub5a0\ud55c \uc774\uc720\ub85c\ub3c4 \uce68\ud574\ud558\uac70\ub098 \ubd80\uc815\ud560 \uc218\uc5c6\ub3c4\ub85d\ud588\ub2e4.\ud5cc\ubc95\uc758 \uc774\ub150\uacfc \uac00\uce58\ub294 \uc138\uacc4\uc778\uad8c\uc120\uc5b8 \uc744 \ud1a0\ub300\ub85c \ud55c \uac83\uc73c\ub85c \uc0ac\ud68c\uac00 \uc9c0\ud5a5\ud574\uc57c \ud560 \ucd5c\uace0 \uc9c0\ud45c\uc778 \ub3d9\uc2dc\uc5d0 \ub204\uad6c\ub098 \ubc18\ub4dc\uc2dc \uc778\uc815\ud558\uace0 \uc9c0\ud0a4\uc9c0 \uc54a\uc73c\uba74 \uc548\ub418\ub294 \uac15\uc81c\uaddc\ubc94\uc774\ub2e4. \ud5cc\ubc95\uc740 \uc778\uac04\uc758 \ud3c9\ub4f1\uacfc \uc790\uc720(\uad8c)\uc758 \ubcf4\uc7a5\uc5d0 \ub300\ud574 \ubb34\ub824 30\uc5ec \uc870\ud56d\uc5d0 \uac78\uccd0 \ub9e4\uc6b0 \uad6c\uccb4\uc801\uc774\uace0 \uc2e4\uc99d\uc801\uc73c\ub85c \uac15\uc870\ud558\uace0 \uc788\uc744 \ubfd0\ub9cc \uc544\ub2c8\ub77c \uadf8\uc678 \uc804 \uc870\ud56d\ub3c4 \uc778\uac04\uc758 \uc790\uc720\uc640 \ud3c9\ub4f1\uc744 \uc2e4\ud604\ud558\uae30 \uc704\ud55c \uac83\uc784\uc740 \uc790\uba85\ud558\ub2e4.", "paragraph_answer": "\uadf8\ub7ec\ub2e4 1789\ub144 \ud504\ub791\uc2a4\ud601\uba85\uc758 \uc131\uacf5\uc73c\ub85c \uc5ed\uc0ac\uc0c1 \ucd5c\ucd08\ub85c \ubaa8\ub4e0 \uc0ac\ub78c\ub4e4\uc774 \uc2e0\ubd84\uc758 \uc871\uc1c4\ub85c\ubd80\ud130 \ud574\ubc29\ub418\uc5b4 \uc9d1\ub2e8\uc801\uc73c\ub85c \ud3c9\ub4f1\ud558\uac8c \uc695\ub9dd\uc758 \ubd84\ucd9c\uc744 \uacbd\ud5d8\ud558\uac8c \ub418\uace0 \uc790\uc720\uc640 \ud3c9\ub4f1\uc774\ub77c\ub294 \uad00\ub150\uc774 \uc2f9\ud2bc \uac83\uc774\ub2e4.\uc624\ub298\ub0a0 \uc138\uacc4\uc0ac\ud68c\ub294,\ud3c9\ub4f1\uacfc \uc790\uc720\ub77c\ub294 \uac1c\ub150\uc790\uccb4\uac00 \uc5c6\uc5c8\uc744\uc815\ub3c4\ub85c \ub300\ub2e8\ud788 \ubd88\ud589\ud558\uace0 \ube44\uc778\uac04\uc801\uc778,\ubd80\uc790\uc720\ud558\uace0 \ubd88\ud3c9\ub4f1\ud55c \uc5ed\uc0ac\ub97c \ub2e4\uc2dc\ub294 \ub418\ud480\uc774 \ud558\uc9c0\uc54a\uae30\uc704\ud574 \uc790\uc720\uc640 \ud3c9\ub4f1\uc744 \ucc9c\ubd80\uc778\uad8c\uc801 \uad8c\ub9ac\ub85c \uaddc\uc815\ud558\uace0 \ud5cc\ubc95\uc5d0 \uac01\uc778\uc2dc\ucf1c \uc5b4\ub5a0\ud55c \uc774\uc720\ub85c\ub3c4 \uce68\ud574\ud558\uac70\ub098 \ubd80\uc815\ud560 \uc218\uc5c6\ub3c4\ub85d\ud588\ub2e4.\ud5cc\ubc95\uc758 \uc774\ub150\uacfc \uac00\uce58\ub294 \uc138\uacc4\uc778\uad8c\uc120\uc5b8 \uc744 \ud1a0\ub300\ub85c \ud55c \uac83\uc73c\ub85c \uc0ac\ud68c\uac00 \uc9c0\ud5a5\ud574\uc57c \ud560 \ucd5c\uace0 \uc9c0\ud45c\uc778 \ub3d9\uc2dc\uc5d0 \ub204\uad6c\ub098 \ubc18\ub4dc\uc2dc \uc778\uc815\ud558\uace0 \uc9c0\ud0a4\uc9c0 \uc54a\uc73c\uba74 \uc548\ub418\ub294 \uac15\uc81c\uaddc\ubc94\uc774\ub2e4. \ud5cc\ubc95\uc740 \uc778\uac04\uc758 \ud3c9\ub4f1\uacfc \uc790\uc720(\uad8c)\uc758 \ubcf4\uc7a5\uc5d0 \ub300\ud574 \ubb34\ub824 30\uc5ec \uc870\ud56d\uc5d0 \uac78\uccd0 \ub9e4\uc6b0 \uad6c\uccb4\uc801\uc774\uace0 \uc2e4\uc99d\uc801\uc73c\ub85c \uac15\uc870\ud558\uace0 \uc788\uc744 \ubfd0\ub9cc \uc544\ub2c8\ub77c \uadf8\uc678 \uc804 \uc870\ud56d\ub3c4 \uc778\uac04\uc758 \uc790\uc720\uc640 \ud3c9\ub4f1\uc744 \uc2e4\ud604\ud558\uae30 \uc704\ud55c \uac83\uc784\uc740 \uc790\uba85\ud558\ub2e4.", "sentence_answer": "\uadf8\ub7ec\ub2e4 1789\ub144 \ud504\ub791\uc2a4\ud601\uba85\uc758 \uc131\uacf5\uc73c\ub85c \uc5ed\uc0ac\uc0c1 \ucd5c\ucd08\ub85c \ubaa8\ub4e0 \uc0ac\ub78c\ub4e4\uc774 \uc2e0\ubd84\uc758 \uc871\uc1c4\ub85c\ubd80\ud130 \ud574\ubc29\ub418\uc5b4 \uc9d1\ub2e8\uc801\uc73c\ub85c \ud3c9\ub4f1\ud558\uac8c \uc695\ub9dd\uc758 \ubd84\ucd9c\uc744 \uacbd\ud5d8\ud558\uac8c \ub418\uace0 \uc790\uc720\uc640 \ud3c9\ub4f1\uc774\ub77c\ub294 \uad00\ub150\uc774 \uc2f9\ud2bc \uac83\uc774\ub2e4.\uc624\ub298\ub0a0 \uc138\uacc4\uc0ac\ud68c\ub294,\ud3c9\ub4f1\uacfc \uc790\uc720\ub77c\ub294 \uac1c\ub150\uc790\uccb4\uac00 \uc5c6\uc5c8\uc744\uc815\ub3c4\ub85c \ub300\ub2e8\ud788 \ubd88\ud589\ud558\uace0 \ube44\uc778\uac04\uc801\uc778,\ubd80\uc790\uc720\ud558\uace0 \ubd88\ud3c9\ub4f1\ud55c \uc5ed\uc0ac\ub97c \ub2e4\uc2dc\ub294 \ub418\ud480\uc774 \ud558\uc9c0\uc54a\uae30\uc704\ud574 \uc790\uc720\uc640 \ud3c9\ub4f1\uc744 \ucc9c\ubd80\uc778\uad8c\uc801 \uad8c\ub9ac\ub85c \uaddc\uc815\ud558\uace0 \ud5cc\ubc95\uc5d0 \uac01\uc778\uc2dc\ucf1c \uc5b4\ub5a0\ud55c \uc774\uc720\ub85c\ub3c4 \uce68\ud574\ud558\uac70\ub098 \ubd80\uc815\ud560 \uc218\uc5c6\ub3c4\ub85d\ud588\ub2e4.\ud5cc\ubc95\uc758 \uc774\ub150\uacfc \uac00\uce58\ub294 \uc138\uacc4\uc778\uad8c\uc120\uc5b8 \uc744 \ud1a0\ub300\ub85c \ud55c \uac83\uc73c\ub85c \uc0ac\ud68c\uac00 \uc9c0\ud5a5\ud574\uc57c \ud560 \ucd5c\uace0 \uc9c0\ud45c\uc778 \ub3d9\uc2dc\uc5d0 \ub204\uad6c\ub098 \ubc18\ub4dc\uc2dc \uc778\uc815\ud558\uace0 \uc9c0\ud0a4\uc9c0 \uc54a\uc73c\uba74 \uc548\ub418\ub294 \uac15\uc81c\uaddc\ubc94\uc774\ub2e4.", "paragraph_id": "6508096-3-0", "paragraph_question": "question: \ud5cc\ubc95\uc758 \uc774\ub150\uacfc \uac00\uce58\ub294 \ubb34\uc2a8 \uc120\uc5b8\uc744 \ud1a0\ub300\ub85c \ud55c \uac83\uc778\uac00?, context: \uadf8\ub7ec\ub2e4 1789\ub144 \ud504\ub791\uc2a4\ud601\uba85\uc758 \uc131\uacf5\uc73c\ub85c \uc5ed\uc0ac\uc0c1 \ucd5c\ucd08\ub85c \ubaa8\ub4e0 \uc0ac\ub78c\ub4e4\uc774 \uc2e0\ubd84\uc758 \uc871\uc1c4\ub85c\ubd80\ud130 \ud574\ubc29\ub418\uc5b4 \uc9d1\ub2e8\uc801\uc73c\ub85c \ud3c9\ub4f1\ud558\uac8c \uc695\ub9dd\uc758 \ubd84\ucd9c\uc744 \uacbd\ud5d8\ud558\uac8c \ub418\uace0 \uc790\uc720\uc640 \ud3c9\ub4f1\uc774\ub77c\ub294 \uad00\ub150\uc774 \uc2f9\ud2bc \uac83\uc774\ub2e4.\uc624\ub298\ub0a0 \uc138\uacc4\uc0ac\ud68c\ub294,\ud3c9\ub4f1\uacfc \uc790\uc720\ub77c\ub294 \uac1c\ub150\uc790\uccb4\uac00 \uc5c6\uc5c8\uc744\uc815\ub3c4\ub85c \ub300\ub2e8\ud788 \ubd88\ud589\ud558\uace0 \ube44\uc778\uac04\uc801\uc778,\ubd80\uc790\uc720\ud558\uace0 \ubd88\ud3c9\ub4f1\ud55c \uc5ed\uc0ac\ub97c \ub2e4\uc2dc\ub294 \ub418\ud480\uc774 \ud558\uc9c0\uc54a\uae30\uc704\ud574 \uc790\uc720\uc640 \ud3c9\ub4f1\uc744 \ucc9c\ubd80\uc778\uad8c\uc801 \uad8c\ub9ac\ub85c \uaddc\uc815\ud558\uace0 \ud5cc\ubc95\uc5d0 \uac01\uc778\uc2dc\ucf1c \uc5b4\ub5a0\ud55c \uc774\uc720\ub85c\ub3c4 \uce68\ud574\ud558\uac70\ub098 \ubd80\uc815\ud560 \uc218\uc5c6\ub3c4\ub85d\ud588\ub2e4.\ud5cc\ubc95\uc758 \uc774\ub150\uacfc \uac00\uce58\ub294 \uc138\uacc4\uc778\uad8c\uc120\uc5b8\uc744 \ud1a0\ub300\ub85c \ud55c \uac83\uc73c\ub85c \uc0ac\ud68c\uac00 \uc9c0\ud5a5\ud574\uc57c \ud560 \ucd5c\uace0 \uc9c0\ud45c\uc778 \ub3d9\uc2dc\uc5d0 \ub204\uad6c\ub098 \ubc18\ub4dc\uc2dc \uc778\uc815\ud558\uace0 \uc9c0\ud0a4\uc9c0 \uc54a\uc73c\uba74 \uc548\ub418\ub294 \uac15\uc81c\uaddc\ubc94\uc774\ub2e4. \ud5cc\ubc95\uc740 \uc778\uac04\uc758 \ud3c9\ub4f1\uacfc \uc790\uc720(\uad8c)\uc758 \ubcf4\uc7a5\uc5d0 \ub300\ud574 \ubb34\ub824 30\uc5ec \uc870\ud56d\uc5d0 \uac78\uccd0 \ub9e4\uc6b0 \uad6c\uccb4\uc801\uc774\uace0 \uc2e4\uc99d\uc801\uc73c\ub85c \uac15\uc870\ud558\uace0 \uc788\uc744 \ubfd0\ub9cc \uc544\ub2c8\ub77c \uadf8\uc678 \uc804 \uc870\ud56d\ub3c4 \uc778\uac04\uc758 \uc790\uc720\uc640 \ud3c9\ub4f1\uc744 \uc2e4\ud604\ud558\uae30 \uc704\ud55c \uac83\uc784\uc740 \uc790\uba85\ud558\ub2e4."} {"question": "\ub370\ud2c0\ub808\ud504 \ud3ec\uc774\ucf00\ub974\ud2b8\uc758 \uc5b4\ub290\ubd84\uc57c \ud559\uc790\uc778\uac00?", "paragraph": "19\uc138\uae30 \ud6c4\ubc18\ubd80\ud130 20\uc138\uae30 \ucd08 \uae4c\uc9c0 \ub3c5\uc77c\ubcf5\uc9c0\uc758 \ubc1c\uc804\uacfc \ud568\uaed8 \ub098\ud0c0\ub09c \uc5c4\uccad\ub09c \uacfc\ud559\uc801, \uae30\uc220\uc801 \ubcc0\ud654\ub294 \ubaa8\ub4e0 \uc0ac\ud68c\ubb38\uc81c\uc758 \ud574\uacb0\uc774 \uba40\uc9c0 \uc54a\uc558\ub2e4\ub294 \uc720\ud1a0\ud53c\uc544\uc801 \ud76c\ub9dd\uc744 \ub9cc\uc5f0\ud558\uac8c \ud588\ub2e4. \uadf8\uc640 \ub3d9\uc2dc\uc5d0 \uba87\uba87 \uc0ac\ub78c\ub4e4\uc740 \ub2e4\ub978 \uc774\ub4e4\ubcf4\ub2e4 \uc0dd\ubb3c\ud559\uc801\uc73c\ub85c \ub354 \uac00\uce58 \uc788\ub2e4\uace0 \uc5ec\uae30\ub294 \uc138\uacc4\uad00\uc744 \uac00\uc9c4 \uc778\uc885\ucc28\ubcc4\uc8fc\uc758\uc790, \uc0ac\ud68c\uc9c4\ud654\ub860 \uc8fc\uc7a5\uc790, \uc6b0\uc0dd\ud559\uc790\ub4e4\ub3c4 \ub9ce\uc774 \uc788\uc5c8\ub2e4. \uc5ed\uc0ac\ud559\uc790 \ub370\ud2c0\ub808\ud504 \ud3ec\uc774\ucf00\ub974\ud2b8(Detlev Peukert)\ub294 \ubc18\uc720\ub300\uc8fc\uc758\uac00 \uc720\ub300\uc778 \ub300\ud559\uc0b4\uc774 \uc77c\uc5b4\ub098\uac8c \ub41c \uc720\uc77c\ud55c \uc774\uc720\uac00 \uc544\ub2c8\ub77c \uad49\uc7a5\ud788 \ub9ce\uc740 \uc791\uc740 \ud750\ub984\uc774 \ud070 \ud750\ub984\uc73c\ub85c \uc774\uc5b4\uc84c\uace0 \uc774\uac83\uc774 \ub300\ud559\uc0b4\uc744 \uc77c\uc73c\ud0a4\uac8c \ub41c \ucd95\uc801\ub41c \uacfc\uaca9\ud654\uc758 \uc0b0\ubb3c\uc774\ub77c\uace0 \ub9d0\ud588\ub2e4. \uc81c 1\ucc28 \uc138\uacc4\ub300\uc804 \uc774\ud6c4\uc5d0 \uc804\uc804\uc758 \ub099\uad00\uc801\uc778 \ubd84\uc704\uae30\ub294 \ub3c5\uc77c \uad00\ub8cc\ub4e4\uc774 \uc774\uc804\uc5d0 \uc0dd\uac01\ud588\ub358 \uac83\ubcf4\ub2e4 \uc0ac\ud68c\uc801\ubb38\uc81c\uac00 \ub354 \uc798 \ud574\uacb0\ub420 \uc218 \uc5c6\uc74c\uc744 \uc54c\uace0 \ud658\uba78\ub85c \ubcc0\ud588\ub2e4. \uc810\ucc28 \uc0dd\ubb3c\ud559\uc801\uc73c\ub85c \ub9de\uc9c0 \uc54a\ub294 \uc0b6\uc744 \uad6c\uc81c\ud558\ub294 \uac83\ubcf4\ub2e4 \uc0dd\ubb3c\ud559\uc801\uc73c\ub85c \uc54c\ub9de\uc740 \uc0b6\ub4e4\uc744 \uad6c\uc81c\ud558\ub294 \uac83\uc5d0 \ub354\uc6b1 \ucd08\uc810\uc744 \ub9de\ucd94\uac8c \ub418\uc5c8\ub2e4.", "answer": "\uc5ed\uc0ac\ud559\uc790", "sentence": "\uc5ed\uc0ac\ud559\uc790 \ub370\ud2c0\ub808\ud504 \ud3ec\uc774\ucf00\ub974\ud2b8(Detlev Peukert)\ub294 \ubc18\uc720\ub300\uc8fc\uc758\uac00 \uc720\ub300\uc778 \ub300\ud559\uc0b4\uc774 \uc77c\uc5b4\ub098\uac8c \ub41c \uc720\uc77c\ud55c \uc774\uc720\uac00 \uc544\ub2c8\ub77c \uad49\uc7a5\ud788 \ub9ce\uc740 \uc791\uc740 \ud750\ub984\uc774 \ud070 \ud750\ub984\uc73c\ub85c \uc774\uc5b4\uc84c\uace0 \uc774\uac83\uc774 \ub300\ud559\uc0b4\uc744 \uc77c\uc73c\ud0a4\uac8c \ub41c \ucd95\uc801\ub41c \uacfc\uaca9\ud654\uc758 \uc0b0\ubb3c\uc774\ub77c\uace0 \ub9d0\ud588\ub2e4.", "paragraph_sentence": "19\uc138\uae30 \ud6c4\ubc18\ubd80\ud130 20\uc138\uae30 \ucd08 \uae4c\uc9c0 \ub3c5\uc77c\ubcf5\uc9c0\uc758 \ubc1c\uc804\uacfc \ud568\uaed8 \ub098\ud0c0\ub09c \uc5c4\uccad\ub09c \uacfc\ud559\uc801, \uae30\uc220\uc801 \ubcc0\ud654\ub294 \ubaa8\ub4e0 \uc0ac\ud68c\ubb38\uc81c\uc758 \ud574\uacb0\uc774 \uba40\uc9c0 \uc54a\uc558\ub2e4\ub294 \uc720\ud1a0\ud53c\uc544\uc801 \ud76c\ub9dd\uc744 \ub9cc\uc5f0\ud558\uac8c \ud588\ub2e4. \uadf8\uc640 \ub3d9\uc2dc\uc5d0 \uba87\uba87 \uc0ac\ub78c\ub4e4\uc740 \ub2e4\ub978 \uc774\ub4e4\ubcf4\ub2e4 \uc0dd\ubb3c\ud559\uc801\uc73c\ub85c \ub354 \uac00\uce58 \uc788\ub2e4\uace0 \uc5ec\uae30\ub294 \uc138\uacc4\uad00\uc744 \uac00\uc9c4 \uc778\uc885\ucc28\ubcc4\uc8fc\uc758\uc790, \uc0ac\ud68c\uc9c4\ud654\ub860 \uc8fc\uc7a5\uc790, \uc6b0\uc0dd\ud559\uc790\ub4e4\ub3c4 \ub9ce\uc774 \uc788\uc5c8\ub2e4. \uc5ed\uc0ac\ud559\uc790 \ub370\ud2c0\ub808\ud504 \ud3ec\uc774\ucf00\ub974\ud2b8(Detlev Peukert)\ub294 \ubc18\uc720\ub300\uc8fc\uc758\uac00 \uc720\ub300\uc778 \ub300\ud559\uc0b4\uc774 \uc77c\uc5b4\ub098\uac8c \ub41c \uc720\uc77c\ud55c \uc774\uc720\uac00 \uc544\ub2c8\ub77c \uad49\uc7a5\ud788 \ub9ce\uc740 \uc791\uc740 \ud750\ub984\uc774 \ud070 \ud750\ub984\uc73c\ub85c \uc774\uc5b4\uc84c\uace0 \uc774\uac83\uc774 \ub300\ud559\uc0b4\uc744 \uc77c\uc73c\ud0a4\uac8c \ub41c \ucd95\uc801\ub41c \uacfc\uaca9\ud654\uc758 \uc0b0\ubb3c\uc774\ub77c\uace0 \ub9d0\ud588\ub2e4. \uc81c 1\ucc28 \uc138\uacc4\ub300\uc804 \uc774\ud6c4\uc5d0 \uc804\uc804\uc758 \ub099\uad00\uc801\uc778 \ubd84\uc704\uae30\ub294 \ub3c5\uc77c \uad00\ub8cc\ub4e4\uc774 \uc774\uc804\uc5d0 \uc0dd\uac01\ud588\ub358 \uac83\ubcf4\ub2e4 \uc0ac\ud68c\uc801\ubb38\uc81c\uac00 \ub354 \uc798 \ud574\uacb0\ub420 \uc218 \uc5c6\uc74c\uc744 \uc54c\uace0 \ud658\uba78\ub85c \ubcc0\ud588\ub2e4. \uc810\ucc28 \uc0dd\ubb3c\ud559\uc801\uc73c\ub85c \ub9de\uc9c0 \uc54a\ub294 \uc0b6\uc744 \uad6c\uc81c\ud558\ub294 \uac83\ubcf4\ub2e4 \uc0dd\ubb3c\ud559\uc801\uc73c\ub85c \uc54c\ub9de\uc740 \uc0b6\ub4e4\uc744 \uad6c\uc81c\ud558\ub294 \uac83\uc5d0 \ub354\uc6b1 \ucd08\uc810\uc744 \ub9de\ucd94\uac8c \ub418\uc5c8\ub2e4.", "paragraph_answer": "19\uc138\uae30 \ud6c4\ubc18\ubd80\ud130 20\uc138\uae30 \ucd08 \uae4c\uc9c0 \ub3c5\uc77c\ubcf5\uc9c0\uc758 \ubc1c\uc804\uacfc \ud568\uaed8 \ub098\ud0c0\ub09c \uc5c4\uccad\ub09c \uacfc\ud559\uc801, \uae30\uc220\uc801 \ubcc0\ud654\ub294 \ubaa8\ub4e0 \uc0ac\ud68c\ubb38\uc81c\uc758 \ud574\uacb0\uc774 \uba40\uc9c0 \uc54a\uc558\ub2e4\ub294 \uc720\ud1a0\ud53c\uc544\uc801 \ud76c\ub9dd\uc744 \ub9cc\uc5f0\ud558\uac8c \ud588\ub2e4. \uadf8\uc640 \ub3d9\uc2dc\uc5d0 \uba87\uba87 \uc0ac\ub78c\ub4e4\uc740 \ub2e4\ub978 \uc774\ub4e4\ubcf4\ub2e4 \uc0dd\ubb3c\ud559\uc801\uc73c\ub85c \ub354 \uac00\uce58 \uc788\ub2e4\uace0 \uc5ec\uae30\ub294 \uc138\uacc4\uad00\uc744 \uac00\uc9c4 \uc778\uc885\ucc28\ubcc4\uc8fc\uc758\uc790, \uc0ac\ud68c\uc9c4\ud654\ub860 \uc8fc\uc7a5\uc790, \uc6b0\uc0dd\ud559\uc790\ub4e4\ub3c4 \ub9ce\uc774 \uc788\uc5c8\ub2e4. \uc5ed\uc0ac\ud559\uc790 \ub370\ud2c0\ub808\ud504 \ud3ec\uc774\ucf00\ub974\ud2b8(Detlev Peukert)\ub294 \ubc18\uc720\ub300\uc8fc\uc758\uac00 \uc720\ub300\uc778 \ub300\ud559\uc0b4\uc774 \uc77c\uc5b4\ub098\uac8c \ub41c \uc720\uc77c\ud55c \uc774\uc720\uac00 \uc544\ub2c8\ub77c \uad49\uc7a5\ud788 \ub9ce\uc740 \uc791\uc740 \ud750\ub984\uc774 \ud070 \ud750\ub984\uc73c\ub85c \uc774\uc5b4\uc84c\uace0 \uc774\uac83\uc774 \ub300\ud559\uc0b4\uc744 \uc77c\uc73c\ud0a4\uac8c \ub41c \ucd95\uc801\ub41c \uacfc\uaca9\ud654\uc758 \uc0b0\ubb3c\uc774\ub77c\uace0 \ub9d0\ud588\ub2e4. \uc81c 1\ucc28 \uc138\uacc4\ub300\uc804 \uc774\ud6c4\uc5d0 \uc804\uc804\uc758 \ub099\uad00\uc801\uc778 \ubd84\uc704\uae30\ub294 \ub3c5\uc77c \uad00\ub8cc\ub4e4\uc774 \uc774\uc804\uc5d0 \uc0dd\uac01\ud588\ub358 \uac83\ubcf4\ub2e4 \uc0ac\ud68c\uc801\ubb38\uc81c\uac00 \ub354 \uc798 \ud574\uacb0\ub420 \uc218 \uc5c6\uc74c\uc744 \uc54c\uace0 \ud658\uba78\ub85c \ubcc0\ud588\ub2e4. \uc810\ucc28 \uc0dd\ubb3c\ud559\uc801\uc73c\ub85c \ub9de\uc9c0 \uc54a\ub294 \uc0b6\uc744 \uad6c\uc81c\ud558\ub294 \uac83\ubcf4\ub2e4 \uc0dd\ubb3c\ud559\uc801\uc73c\ub85c \uc54c\ub9de\uc740 \uc0b6\ub4e4\uc744 \uad6c\uc81c\ud558\ub294 \uac83\uc5d0 \ub354\uc6b1 \ucd08\uc810\uc744 \ub9de\ucd94\uac8c \ub418\uc5c8\ub2e4.", "sentence_answer": " \uc5ed\uc0ac\ud559\uc790 \ub370\ud2c0\ub808\ud504 \ud3ec\uc774\ucf00\ub974\ud2b8(Detlev Peukert)\ub294 \ubc18\uc720\ub300\uc8fc\uc758\uac00 \uc720\ub300\uc778 \ub300\ud559\uc0b4\uc774 \uc77c\uc5b4\ub098\uac8c \ub41c \uc720\uc77c\ud55c \uc774\uc720\uac00 \uc544\ub2c8\ub77c \uad49\uc7a5\ud788 \ub9ce\uc740 \uc791\uc740 \ud750\ub984\uc774 \ud070 \ud750\ub984\uc73c\ub85c \uc774\uc5b4\uc84c\uace0 \uc774\uac83\uc774 \ub300\ud559\uc0b4\uc744 \uc77c\uc73c\ud0a4\uac8c \ub41c \ucd95\uc801\ub41c \uacfc\uaca9\ud654\uc758 \uc0b0\ubb3c\uc774\ub77c\uace0 \ub9d0\ud588\ub2e4.", "paragraph_id": "6571969-4-0", "paragraph_question": "question: \ub370\ud2c0\ub808\ud504 \ud3ec\uc774\ucf00\ub974\ud2b8\uc758 \uc5b4\ub290\ubd84\uc57c \ud559\uc790\uc778\uac00?, context: 19\uc138\uae30 \ud6c4\ubc18\ubd80\ud130 20\uc138\uae30 \ucd08 \uae4c\uc9c0 \ub3c5\uc77c\ubcf5\uc9c0\uc758 \ubc1c\uc804\uacfc \ud568\uaed8 \ub098\ud0c0\ub09c \uc5c4\uccad\ub09c \uacfc\ud559\uc801, \uae30\uc220\uc801 \ubcc0\ud654\ub294 \ubaa8\ub4e0 \uc0ac\ud68c\ubb38\uc81c\uc758 \ud574\uacb0\uc774 \uba40\uc9c0 \uc54a\uc558\ub2e4\ub294 \uc720\ud1a0\ud53c\uc544\uc801 \ud76c\ub9dd\uc744 \ub9cc\uc5f0\ud558\uac8c \ud588\ub2e4. \uadf8\uc640 \ub3d9\uc2dc\uc5d0 \uba87\uba87 \uc0ac\ub78c\ub4e4\uc740 \ub2e4\ub978 \uc774\ub4e4\ubcf4\ub2e4 \uc0dd\ubb3c\ud559\uc801\uc73c\ub85c \ub354 \uac00\uce58 \uc788\ub2e4\uace0 \uc5ec\uae30\ub294 \uc138\uacc4\uad00\uc744 \uac00\uc9c4 \uc778\uc885\ucc28\ubcc4\uc8fc\uc758\uc790, \uc0ac\ud68c\uc9c4\ud654\ub860 \uc8fc\uc7a5\uc790, \uc6b0\uc0dd\ud559\uc790\ub4e4\ub3c4 \ub9ce\uc774 \uc788\uc5c8\ub2e4. \uc5ed\uc0ac\ud559\uc790 \ub370\ud2c0\ub808\ud504 \ud3ec\uc774\ucf00\ub974\ud2b8(Detlev Peukert)\ub294 \ubc18\uc720\ub300\uc8fc\uc758\uac00 \uc720\ub300\uc778 \ub300\ud559\uc0b4\uc774 \uc77c\uc5b4\ub098\uac8c \ub41c \uc720\uc77c\ud55c \uc774\uc720\uac00 \uc544\ub2c8\ub77c \uad49\uc7a5\ud788 \ub9ce\uc740 \uc791\uc740 \ud750\ub984\uc774 \ud070 \ud750\ub984\uc73c\ub85c \uc774\uc5b4\uc84c\uace0 \uc774\uac83\uc774 \ub300\ud559\uc0b4\uc744 \uc77c\uc73c\ud0a4\uac8c \ub41c \ucd95\uc801\ub41c \uacfc\uaca9\ud654\uc758 \uc0b0\ubb3c\uc774\ub77c\uace0 \ub9d0\ud588\ub2e4. \uc81c 1\ucc28 \uc138\uacc4\ub300\uc804 \uc774\ud6c4\uc5d0 \uc804\uc804\uc758 \ub099\uad00\uc801\uc778 \ubd84\uc704\uae30\ub294 \ub3c5\uc77c \uad00\ub8cc\ub4e4\uc774 \uc774\uc804\uc5d0 \uc0dd\uac01\ud588\ub358 \uac83\ubcf4\ub2e4 \uc0ac\ud68c\uc801\ubb38\uc81c\uac00 \ub354 \uc798 \ud574\uacb0\ub420 \uc218 \uc5c6\uc74c\uc744 \uc54c\uace0 \ud658\uba78\ub85c \ubcc0\ud588\ub2e4. \uc810\ucc28 \uc0dd\ubb3c\ud559\uc801\uc73c\ub85c \ub9de\uc9c0 \uc54a\ub294 \uc0b6\uc744 \uad6c\uc81c\ud558\ub294 \uac83\ubcf4\ub2e4 \uc0dd\ubb3c\ud559\uc801\uc73c\ub85c \uc54c\ub9de\uc740 \uc0b6\ub4e4\uc744 \uad6c\uc81c\ud558\ub294 \uac83\uc5d0 \ub354\uc6b1 \ucd08\uc810\uc744 \ub9de\ucd94\uac8c \ub418\uc5c8\ub2e4."} {"question": "12\uc138\uae30 \uc2dc\uce60\ub9ac\uc544 \uc0ac\ub78c\ub4e4\uc774 \ub9cc\ub4e4\ub358 \ud30c\uc2a4\ud0c0\ub294 \ubb34\uc5c7\uc73c\ub85c \ub9cc\ub4e4\uc5b4\uc84c\ub294\uac00?", "paragraph": "\ubb34\uc2ac\ub9bc\uc774 9\uc138\uae30\uae4c\uc9c0 \uc2dc\uce60\ub9ac\uc544\ub97c \uce68\ub7b5\ud558\uc5ec \ub2e4\uc2a4\ub838\uace0 \uadf8 \ub4a4\uc5d0\ub294 \ubc14\uc774\ud0b9 \uc138\ub825\uc774 \ub0a8\ud558\ud558\uc5ec \uc9c0\uc911\ud574 \uc77c\ub300\ub294 \uc0c1\ub2f9\ud55c \ud63c\ub780\uae30\ub97c \uacaa\uc5c8\ub2e4. \uc544\ub78d\uc778\ub4e4\uc740 \uc2dc\uae08\uce58, \uc544\ubaac\ub4dc, \uc300 \ub4f1\uc744 \ub3c4\uc785\ud588\uc73c\uba70, \uc774\ub7ec\ud55c \uc7ac\ub8cc\uc758 \ucd9c\ud604\uc740 12\uc138\uae30 \ub178\ub974\uc6e8\uc774\uc758 \uc655\uc774 \uc2dc\uce60\ub9ac\uc544\uc5d0 \uc654\uc744 \ub54c \uc0ac\ub78c\ub4e4\uc774 \uae30\ub2e4\ub780 \uba74\ubc1c\uc744 \ub9cc\ub4e4\uc5b4 \uba39\uc5c8\ub2e4\ub294 \uac83\uc744 \ubcf4\uc558\ub2e4\uace0 \uae30\ub85d\ud55c \uac83\uc744 \ud1a0\ub300\ub85c \uc54c \uc218 \uc788\ub2e4. \uc774 \uc694\ub9ac\ub97c \uc77c\ubd80\uc5d0\uc11c\ub294 \ud30c\uc2a4\ud0c0\uc758 \uccab \ucd9c\ud604\uc774\ub77c\uace0 \ubcf4\uace0 \uc788\ub2e4. \ub2f9\uc2dc\uc5d0\ub294 \uba74\uc744 \ub2e8\uc21c\ud788 \ubc00\uac00\ub8e8\uc640 \ubb3c\ub85c \ub9cc\ub4e4\uc5c8\ub2e4\uace0 \ud558\ub294\ub370, \uc0ac\ub78c\ub4e4\uc774 \u201c\uc544\ud2b8\ub9ac\uc57c\u201d(atriya)\ub77c\uace0 \ubd88\ub800\ub2e4\uac00 \ub098\uc911\uc5d0\ub294 \u201c\ud2b8\ub9ac\u201d(trii)\ub85c \uad73\uc5b4\uc84c\ub2e4\uace0 \ud55c\ub2e4. \ud2b8\ub9ac(Trii)\ub77c\ub294 \uc774 \ub0b1\ub9d0\uc740 \ub0a8\ubd80 \uc774\ud0c8\ub9ac\uc544\uc5d0\uc11c \uc2a4\ud30c\uac8c\ud2f0\uc758 \ub2e4\ub978 \uc774\ub984\uc73c\ub85c \uc4f0\uc774\uae30\ub3c4 \ud588\ub2e4. \ub178\ub974\ub9cc \uc871\ub4e4\uc740 \uccad\uc5b4\ub098 \ub300\uad6c \ub530\uc704\ub97c \uc808\uc5ec \uba39\ub294 \ubc95\ub3c4 \ub4e4\uc5ec\uc654\uc73c\uba70, \uc774 \ubc29\ubc95\uc740 \uc544\uc9c1\uae4c\uc9c0\ub3c4 \uc774\ud0c8\ub9ac\uc544\uc5d0\uc11c \uc0ac\uc6a9\ub418\uace0 \uc788\ub2e4.", "answer": "\ubc00\uac00\ub8e8\uc640 \ubb3c", "sentence": "\ub2f9\uc2dc\uc5d0\ub294 \uba74\uc744 \ub2e8\uc21c\ud788 \ubc00\uac00\ub8e8\uc640 \ubb3c \ub85c \ub9cc\ub4e4\uc5c8\ub2e4\uace0 \ud558\ub294\ub370, \uc0ac\ub78c\ub4e4\uc774 \u201c\uc544\ud2b8\ub9ac\uc57c\u201d(atriya)\ub77c\uace0 \ubd88\ub800\ub2e4\uac00 \ub098\uc911\uc5d0\ub294 \u201c\ud2b8\ub9ac\u201d(trii)\ub85c \uad73\uc5b4\uc84c\ub2e4\uace0 \ud55c\ub2e4.", "paragraph_sentence": "\ubb34\uc2ac\ub9bc\uc774 9\uc138\uae30\uae4c\uc9c0 \uc2dc\uce60\ub9ac\uc544\ub97c \uce68\ub7b5\ud558\uc5ec \ub2e4\uc2a4\ub838\uace0 \uadf8 \ub4a4\uc5d0\ub294 \ubc14\uc774\ud0b9 \uc138\ub825\uc774 \ub0a8\ud558\ud558\uc5ec \uc9c0\uc911\ud574 \uc77c\ub300\ub294 \uc0c1\ub2f9\ud55c \ud63c\ub780\uae30\ub97c \uacaa\uc5c8\ub2e4. \uc544\ub78d\uc778\ub4e4\uc740 \uc2dc\uae08\uce58, \uc544\ubaac\ub4dc, \uc300 \ub4f1\uc744 \ub3c4\uc785\ud588\uc73c\uba70, \uc774\ub7ec\ud55c \uc7ac\ub8cc\uc758 \ucd9c\ud604\uc740 12\uc138\uae30 \ub178\ub974\uc6e8\uc774\uc758 \uc655\uc774 \uc2dc\uce60\ub9ac\uc544\uc5d0 \uc654\uc744 \ub54c \uc0ac\ub78c\ub4e4\uc774 \uae30\ub2e4\ub780 \uba74\ubc1c\uc744 \ub9cc\ub4e4\uc5b4 \uba39\uc5c8\ub2e4\ub294 \uac83\uc744 \ubcf4\uc558\ub2e4\uace0 \uae30\ub85d\ud55c \uac83\uc744 \ud1a0\ub300\ub85c \uc54c \uc218 \uc788\ub2e4. \uc774 \uc694\ub9ac\ub97c \uc77c\ubd80\uc5d0\uc11c\ub294 \ud30c\uc2a4\ud0c0\uc758 \uccab \ucd9c\ud604\uc774\ub77c\uace0 \ubcf4\uace0 \uc788\ub2e4. \ub2f9\uc2dc\uc5d0\ub294 \uba74\uc744 \ub2e8\uc21c\ud788 \ubc00\uac00\ub8e8\uc640 \ubb3c \ub85c \ub9cc\ub4e4\uc5c8\ub2e4\uace0 \ud558\ub294\ub370, \uc0ac\ub78c\ub4e4\uc774 \u201c\uc544\ud2b8\ub9ac\uc57c\u201d(atriya)\ub77c\uace0 \ubd88\ub800\ub2e4\uac00 \ub098\uc911\uc5d0\ub294 \u201c\ud2b8\ub9ac\u201d(trii)\ub85c \uad73\uc5b4\uc84c\ub2e4\uace0 \ud55c\ub2e4. \ud2b8\ub9ac(Trii)\ub77c\ub294 \uc774 \ub0b1\ub9d0\uc740 \ub0a8\ubd80 \uc774\ud0c8\ub9ac\uc544\uc5d0\uc11c \uc2a4\ud30c\uac8c\ud2f0\uc758 \ub2e4\ub978 \uc774\ub984\uc73c\ub85c \uc4f0\uc774\uae30\ub3c4 \ud588\ub2e4. \ub178\ub974\ub9cc \uc871\ub4e4\uc740 \uccad\uc5b4\ub098 \ub300\uad6c \ub530\uc704\ub97c \uc808\uc5ec \uba39\ub294 \ubc95\ub3c4 \ub4e4\uc5ec\uc654\uc73c\uba70, \uc774 \ubc29\ubc95\uc740 \uc544\uc9c1\uae4c\uc9c0\ub3c4 \uc774\ud0c8\ub9ac\uc544\uc5d0\uc11c \uc0ac\uc6a9\ub418\uace0 \uc788\ub2e4.", "paragraph_answer": "\ubb34\uc2ac\ub9bc\uc774 9\uc138\uae30\uae4c\uc9c0 \uc2dc\uce60\ub9ac\uc544\ub97c \uce68\ub7b5\ud558\uc5ec \ub2e4\uc2a4\ub838\uace0 \uadf8 \ub4a4\uc5d0\ub294 \ubc14\uc774\ud0b9 \uc138\ub825\uc774 \ub0a8\ud558\ud558\uc5ec \uc9c0\uc911\ud574 \uc77c\ub300\ub294 \uc0c1\ub2f9\ud55c \ud63c\ub780\uae30\ub97c \uacaa\uc5c8\ub2e4. \uc544\ub78d\uc778\ub4e4\uc740 \uc2dc\uae08\uce58, \uc544\ubaac\ub4dc, \uc300 \ub4f1\uc744 \ub3c4\uc785\ud588\uc73c\uba70, \uc774\ub7ec\ud55c \uc7ac\ub8cc\uc758 \ucd9c\ud604\uc740 12\uc138\uae30 \ub178\ub974\uc6e8\uc774\uc758 \uc655\uc774 \uc2dc\uce60\ub9ac\uc544\uc5d0 \uc654\uc744 \ub54c \uc0ac\ub78c\ub4e4\uc774 \uae30\ub2e4\ub780 \uba74\ubc1c\uc744 \ub9cc\ub4e4\uc5b4 \uba39\uc5c8\ub2e4\ub294 \uac83\uc744 \ubcf4\uc558\ub2e4\uace0 \uae30\ub85d\ud55c \uac83\uc744 \ud1a0\ub300\ub85c \uc54c \uc218 \uc788\ub2e4. \uc774 \uc694\ub9ac\ub97c \uc77c\ubd80\uc5d0\uc11c\ub294 \ud30c\uc2a4\ud0c0\uc758 \uccab \ucd9c\ud604\uc774\ub77c\uace0 \ubcf4\uace0 \uc788\ub2e4. \ub2f9\uc2dc\uc5d0\ub294 \uba74\uc744 \ub2e8\uc21c\ud788 \ubc00\uac00\ub8e8\uc640 \ubb3c \ub85c \ub9cc\ub4e4\uc5c8\ub2e4\uace0 \ud558\ub294\ub370, \uc0ac\ub78c\ub4e4\uc774 \u201c\uc544\ud2b8\ub9ac\uc57c\u201d(atriya)\ub77c\uace0 \ubd88\ub800\ub2e4\uac00 \ub098\uc911\uc5d0\ub294 \u201c\ud2b8\ub9ac\u201d(trii)\ub85c \uad73\uc5b4\uc84c\ub2e4\uace0 \ud55c\ub2e4. \ud2b8\ub9ac(Trii)\ub77c\ub294 \uc774 \ub0b1\ub9d0\uc740 \ub0a8\ubd80 \uc774\ud0c8\ub9ac\uc544\uc5d0\uc11c \uc2a4\ud30c\uac8c\ud2f0\uc758 \ub2e4\ub978 \uc774\ub984\uc73c\ub85c \uc4f0\uc774\uae30\ub3c4 \ud588\ub2e4. \ub178\ub974\ub9cc \uc871\ub4e4\uc740 \uccad\uc5b4\ub098 \ub300\uad6c \ub530\uc704\ub97c \uc808\uc5ec \uba39\ub294 \ubc95\ub3c4 \ub4e4\uc5ec\uc654\uc73c\uba70, \uc774 \ubc29\ubc95\uc740 \uc544\uc9c1\uae4c\uc9c0\ub3c4 \uc774\ud0c8\ub9ac\uc544\uc5d0\uc11c \uc0ac\uc6a9\ub418\uace0 \uc788\ub2e4.", "sentence_answer": "\ub2f9\uc2dc\uc5d0\ub294 \uba74\uc744 \ub2e8\uc21c\ud788 \ubc00\uac00\ub8e8\uc640 \ubb3c \ub85c \ub9cc\ub4e4\uc5c8\ub2e4\uace0 \ud558\ub294\ub370, \uc0ac\ub78c\ub4e4\uc774 \u201c\uc544\ud2b8\ub9ac\uc57c\u201d(atriya)\ub77c\uace0 \ubd88\ub800\ub2e4\uac00 \ub098\uc911\uc5d0\ub294 \u201c\ud2b8\ub9ac\u201d(trii)\ub85c \uad73\uc5b4\uc84c\ub2e4\uace0 \ud55c\ub2e4.", "paragraph_id": "6590622-1-2", "paragraph_question": "question: 12\uc138\uae30 \uc2dc\uce60\ub9ac\uc544 \uc0ac\ub78c\ub4e4\uc774 \ub9cc\ub4e4\ub358 \ud30c\uc2a4\ud0c0\ub294 \ubb34\uc5c7\uc73c\ub85c \ub9cc\ub4e4\uc5b4\uc84c\ub294\uac00?, context: \ubb34\uc2ac\ub9bc\uc774 9\uc138\uae30\uae4c\uc9c0 \uc2dc\uce60\ub9ac\uc544\ub97c \uce68\ub7b5\ud558\uc5ec \ub2e4\uc2a4\ub838\uace0 \uadf8 \ub4a4\uc5d0\ub294 \ubc14\uc774\ud0b9 \uc138\ub825\uc774 \ub0a8\ud558\ud558\uc5ec \uc9c0\uc911\ud574 \uc77c\ub300\ub294 \uc0c1\ub2f9\ud55c \ud63c\ub780\uae30\ub97c \uacaa\uc5c8\ub2e4. \uc544\ub78d\uc778\ub4e4\uc740 \uc2dc\uae08\uce58, \uc544\ubaac\ub4dc, \uc300 \ub4f1\uc744 \ub3c4\uc785\ud588\uc73c\uba70, \uc774\ub7ec\ud55c \uc7ac\ub8cc\uc758 \ucd9c\ud604\uc740 12\uc138\uae30 \ub178\ub974\uc6e8\uc774\uc758 \uc655\uc774 \uc2dc\uce60\ub9ac\uc544\uc5d0 \uc654\uc744 \ub54c \uc0ac\ub78c\ub4e4\uc774 \uae30\ub2e4\ub780 \uba74\ubc1c\uc744 \ub9cc\ub4e4\uc5b4 \uba39\uc5c8\ub2e4\ub294 \uac83\uc744 \ubcf4\uc558\ub2e4\uace0 \uae30\ub85d\ud55c \uac83\uc744 \ud1a0\ub300\ub85c \uc54c \uc218 \uc788\ub2e4. \uc774 \uc694\ub9ac\ub97c \uc77c\ubd80\uc5d0\uc11c\ub294 \ud30c\uc2a4\ud0c0\uc758 \uccab \ucd9c\ud604\uc774\ub77c\uace0 \ubcf4\uace0 \uc788\ub2e4. \ub2f9\uc2dc\uc5d0\ub294 \uba74\uc744 \ub2e8\uc21c\ud788 \ubc00\uac00\ub8e8\uc640 \ubb3c\ub85c \ub9cc\ub4e4\uc5c8\ub2e4\uace0 \ud558\ub294\ub370, \uc0ac\ub78c\ub4e4\uc774 \u201c\uc544\ud2b8\ub9ac\uc57c\u201d(atriya)\ub77c\uace0 \ubd88\ub800\ub2e4\uac00 \ub098\uc911\uc5d0\ub294 \u201c\ud2b8\ub9ac\u201d(trii)\ub85c \uad73\uc5b4\uc84c\ub2e4\uace0 \ud55c\ub2e4. \ud2b8\ub9ac(Trii)\ub77c\ub294 \uc774 \ub0b1\ub9d0\uc740 \ub0a8\ubd80 \uc774\ud0c8\ub9ac\uc544\uc5d0\uc11c \uc2a4\ud30c\uac8c\ud2f0\uc758 \ub2e4\ub978 \uc774\ub984\uc73c\ub85c \uc4f0\uc774\uae30\ub3c4 \ud588\ub2e4. \ub178\ub974\ub9cc \uc871\ub4e4\uc740 \uccad\uc5b4\ub098 \ub300\uad6c \ub530\uc704\ub97c \uc808\uc5ec \uba39\ub294 \ubc95\ub3c4 \ub4e4\uc5ec\uc654\uc73c\uba70, \uc774 \ubc29\ubc95\uc740 \uc544\uc9c1\uae4c\uc9c0\ub3c4 \uc774\ud0c8\ub9ac\uc544\uc5d0\uc11c \uc0ac\uc6a9\ub418\uace0 \uc788\ub2e4."} {"question": "\uc911\ubd80 \ubc0f \ub3d9\ubd80 \uc804\uc120\uc758 \uc720\uc5d4\uad70\uc774 \ubc29\uc5b4\uc9c4\uc9c0\ub97c \uc124\uce58\ud55c \uc7a5\uc18c\ub294?", "paragraph": "\uc911\uad6d\uad70\uacfc \ubd81\ud55c\uad70\uc758 \ucd08\uae30 \uacf5\uaca9\uc774 \uc131\uacf5\ud558\uc5ec \ub300\ud55c\ubbfc\uad6d \uc81c1\uad70\ub2e8\uc740 \uc6d0\ub798 \ubc29\uc5b4\uc9c0\uc810\uc774\uc5c8\ub358 \ud604\ub9ac\uc5d0\uc11c\uc758 \uc0c1\ud669 \ud68c\ubcf5 \uc2dc\ub3c4\ub97c \ud3ec\uae30\ud560 \uc218\ubc16\uc5d0 \uc5c6\uc5c8\ub2e4. \uc774\ud6c4 \ubd81\ud55c\uad70 \ubcd1\ub825 \ub2e4\uc218\uac00 \uc6d0\uc8fc\uc758 \ubbf8\uad6d \uc81c2\ubcf4\ubcd1\uc0ac\ub2e8\uacfc \ub3d9\ud574\uc548\uc758 \ub300\ud55c\ubbfc\uad6d \uc81c1\uad70\ub2e8 \uc0ac\uc774\uc5d0 \uc0dd\uae34 \ud2c8 \uc0ac\uc774\ub85c \ubc00\uace0 \ub4e4\uc5b4\uc654\ub2e4. \ub9ac\uc9c0\uc6e8\uc774\ub294 \uc911\ubd80 \uc804\uc120\uc5d0 \ub300\ud55c \uc870\uc911 \uc5f0\ud569\uad70\uc758 \uacf5\uaca9\uc774 \uc11c\uc6b8\uc758 \uc720\uc5d4\uad70 \ubc29\uc5b4\uc790\ub4e4\uc744 \ud3ec\uc704\ud558\uae30 \uc704\ud55c \uc2dc\ub3c4\ub77c\uace0 \ud574\uc11d\ud558\uace0 \uadf8\ub294 1\uc6d4 3\uc77c \uc11c\uc6b8 \uc8fc\ub454 \uc720\uc5d4\uad70\uc5d0\uac8c \uc989\uac01\uc801\uc778 \ucca0\uc218\ub97c \uba85\ub839\ud588\ub2e4. 1\uc6d4 5\uc77c, \ub9ac\uc9c0\uc6e8\uc774\ub294 D \ub77c\uc778\uc774\ub77c \uba85\uba85\ub41c \uc0c8\ub85c\uc6b4 \ubc29\uc5b4\uc120\uc744 \uc124\ub9bd\ud558\uae30 \uc704\ud574 \ubaa8\ub4e0 \uc720\uc5d4\uad70 \ubcd1\ub825\uc5d0\uac8c 37\ub3c4\uc120 \uc774\ub0a8\uc744 \ucca0\uc218\ud560 \uac83\uc744 \uc9c0\uc2dc\ud588\uace0, \uc911\ubd80 \ubc0f \ub3d9\ubd80 \uc804\uc120\uc758 \uc720\uc5d4\uad70\uc740 \uc6d0\uc8fc\uc640 \ud574\uc548\ub9c8\uc744\uc778 \uc8fc\ubb38\uc9c4 \uc0ac\uc774\uc5d0 \ubc29\uc5b4\uc9c4\uc9c0\ub4e4\uc744 \uc124\uce58\ud588\ub2e4. \uac19\uc740 \uc2dc\uae30\uc5d0 \uc911\uad6d\uc778\ubbfc\uc9c0\uc6d0\uad70 \uc81c42\uad70\ub2e8\uacfc \uc81c66\uad70\ub2e8\uc744 \uad6c\ud638\ud558\uae30 \uc704\ud574 \uc911\uad6d\uad70\uc740 \uc870\uc120\uc778\ubbfc\uad70 \uc81c2\uad70\ub2e8\uacfc \uc81c5\uad70\ub2e8\uacfc \ud568\uaed8 \uacf5\uc138 \uc791\uc804\uc744 \uc911\ub2e8\ud588\ub2e4.", "answer": "\uc6d0\uc8fc\uc640 \ud574\uc548\ub9c8\uc744\uc778 \uc8fc\ubb38\uc9c4 \uc0ac\uc774", "sentence": "1\uc6d4 5\uc77c, \ub9ac\uc9c0\uc6e8\uc774\ub294 D \ub77c\uc778\uc774\ub77c \uba85\uba85\ub41c \uc0c8\ub85c\uc6b4 \ubc29\uc5b4\uc120\uc744 \uc124\ub9bd\ud558\uae30 \uc704\ud574 \ubaa8\ub4e0 \uc720\uc5d4\uad70 \ubcd1\ub825\uc5d0\uac8c 37\ub3c4\uc120 \uc774\ub0a8\uc744 \ucca0\uc218\ud560 \uac83\uc744 \uc9c0\uc2dc\ud588\uace0, \uc911\ubd80 \ubc0f \ub3d9\ubd80 \uc804\uc120\uc758 \uc720\uc5d4\uad70\uc740 \uc6d0\uc8fc\uc640 \ud574\uc548\ub9c8\uc744\uc778 \uc8fc\ubb38\uc9c4 \uc0ac\uc774 \uc5d0", "paragraph_sentence": "\uc911\uad6d\uad70\uacfc \ubd81\ud55c\uad70\uc758 \ucd08\uae30 \uacf5\uaca9\uc774 \uc131\uacf5\ud558\uc5ec \ub300\ud55c\ubbfc\uad6d \uc81c1\uad70\ub2e8\uc740 \uc6d0\ub798 \ubc29\uc5b4\uc9c0\uc810\uc774\uc5c8\ub358 \ud604\ub9ac\uc5d0\uc11c\uc758 \uc0c1\ud669 \ud68c\ubcf5 \uc2dc\ub3c4\ub97c \ud3ec\uae30\ud560 \uc218\ubc16\uc5d0 \uc5c6\uc5c8\ub2e4. \uc774\ud6c4 \ubd81\ud55c\uad70 \ubcd1\ub825 \ub2e4\uc218\uac00 \uc6d0\uc8fc\uc758 \ubbf8\uad6d \uc81c2\ubcf4\ubcd1\uc0ac\ub2e8\uacfc \ub3d9\ud574\uc548\uc758 \ub300\ud55c\ubbfc\uad6d \uc81c1\uad70\ub2e8 \uc0ac\uc774\uc5d0 \uc0dd\uae34 \ud2c8 \uc0ac\uc774\ub85c \ubc00\uace0 \ub4e4\uc5b4\uc654\ub2e4. \ub9ac\uc9c0\uc6e8\uc774\ub294 \uc911\ubd80 \uc804\uc120\uc5d0 \ub300\ud55c \uc870\uc911 \uc5f0\ud569\uad70\uc758 \uacf5\uaca9\uc774 \uc11c\uc6b8\uc758 \uc720\uc5d4\uad70 \ubc29\uc5b4\uc790\ub4e4\uc744 \ud3ec\uc704\ud558\uae30 \uc704\ud55c \uc2dc\ub3c4\ub77c\uace0 \ud574\uc11d\ud558\uace0 \uadf8\ub294 1\uc6d4 3\uc77c \uc11c\uc6b8 \uc8fc\ub454 \uc720\uc5d4\uad70\uc5d0\uac8c \uc989\uac01\uc801\uc778 \ucca0\uc218\ub97c \uba85\ub839\ud588\ub2e4. 1\uc6d4 5\uc77c, \ub9ac\uc9c0\uc6e8\uc774\ub294 D \ub77c\uc778\uc774\ub77c \uba85\uba85\ub41c \uc0c8\ub85c\uc6b4 \ubc29\uc5b4\uc120\uc744 \uc124\ub9bd\ud558\uae30 \uc704\ud574 \ubaa8\ub4e0 \uc720\uc5d4\uad70 \ubcd1\ub825\uc5d0\uac8c 37\ub3c4\uc120 \uc774\ub0a8\uc744 \ucca0\uc218\ud560 \uac83\uc744 \uc9c0\uc2dc\ud588\uace0, \uc911\ubd80 \ubc0f \ub3d9\ubd80 \uc804\uc120\uc758 \uc720\uc5d4\uad70\uc740 \uc6d0\uc8fc\uc640 \ud574\uc548\ub9c8\uc744\uc778 \uc8fc\ubb38\uc9c4 \uc0ac\uc774 \uc5d0 \ubc29\uc5b4\uc9c4\uc9c0\ub4e4\uc744 \uc124\uce58\ud588\ub2e4. \uac19\uc740 \uc2dc\uae30\uc5d0 \uc911\uad6d\uc778\ubbfc\uc9c0\uc6d0\uad70 \uc81c42\uad70\ub2e8\uacfc \uc81c66\uad70\ub2e8\uc744 \uad6c\ud638\ud558\uae30 \uc704\ud574 \uc911\uad6d\uad70\uc740 \uc870\uc120\uc778\ubbfc\uad70 \uc81c2\uad70\ub2e8\uacfc \uc81c5\uad70\ub2e8\uacfc \ud568\uaed8 \uacf5\uc138 \uc791\uc804\uc744 \uc911\ub2e8\ud588\ub2e4.", "paragraph_answer": "\uc911\uad6d\uad70\uacfc \ubd81\ud55c\uad70\uc758 \ucd08\uae30 \uacf5\uaca9\uc774 \uc131\uacf5\ud558\uc5ec \ub300\ud55c\ubbfc\uad6d \uc81c1\uad70\ub2e8\uc740 \uc6d0\ub798 \ubc29\uc5b4\uc9c0\uc810\uc774\uc5c8\ub358 \ud604\ub9ac\uc5d0\uc11c\uc758 \uc0c1\ud669 \ud68c\ubcf5 \uc2dc\ub3c4\ub97c \ud3ec\uae30\ud560 \uc218\ubc16\uc5d0 \uc5c6\uc5c8\ub2e4. \uc774\ud6c4 \ubd81\ud55c\uad70 \ubcd1\ub825 \ub2e4\uc218\uac00 \uc6d0\uc8fc\uc758 \ubbf8\uad6d \uc81c2\ubcf4\ubcd1\uc0ac\ub2e8\uacfc \ub3d9\ud574\uc548\uc758 \ub300\ud55c\ubbfc\uad6d \uc81c1\uad70\ub2e8 \uc0ac\uc774\uc5d0 \uc0dd\uae34 \ud2c8 \uc0ac\uc774\ub85c \ubc00\uace0 \ub4e4\uc5b4\uc654\ub2e4. \ub9ac\uc9c0\uc6e8\uc774\ub294 \uc911\ubd80 \uc804\uc120\uc5d0 \ub300\ud55c \uc870\uc911 \uc5f0\ud569\uad70\uc758 \uacf5\uaca9\uc774 \uc11c\uc6b8\uc758 \uc720\uc5d4\uad70 \ubc29\uc5b4\uc790\ub4e4\uc744 \ud3ec\uc704\ud558\uae30 \uc704\ud55c \uc2dc\ub3c4\ub77c\uace0 \ud574\uc11d\ud558\uace0 \uadf8\ub294 1\uc6d4 3\uc77c \uc11c\uc6b8 \uc8fc\ub454 \uc720\uc5d4\uad70\uc5d0\uac8c \uc989\uac01\uc801\uc778 \ucca0\uc218\ub97c \uba85\ub839\ud588\ub2e4. 1\uc6d4 5\uc77c, \ub9ac\uc9c0\uc6e8\uc774\ub294 D \ub77c\uc778\uc774\ub77c \uba85\uba85\ub41c \uc0c8\ub85c\uc6b4 \ubc29\uc5b4\uc120\uc744 \uc124\ub9bd\ud558\uae30 \uc704\ud574 \ubaa8\ub4e0 \uc720\uc5d4\uad70 \ubcd1\ub825\uc5d0\uac8c 37\ub3c4\uc120 \uc774\ub0a8\uc744 \ucca0\uc218\ud560 \uac83\uc744 \uc9c0\uc2dc\ud588\uace0, \uc911\ubd80 \ubc0f \ub3d9\ubd80 \uc804\uc120\uc758 \uc720\uc5d4\uad70\uc740 \uc6d0\uc8fc\uc640 \ud574\uc548\ub9c8\uc744\uc778 \uc8fc\ubb38\uc9c4 \uc0ac\uc774 \uc5d0 \ubc29\uc5b4\uc9c4\uc9c0\ub4e4\uc744 \uc124\uce58\ud588\ub2e4. \uac19\uc740 \uc2dc\uae30\uc5d0 \uc911\uad6d\uc778\ubbfc\uc9c0\uc6d0\uad70 \uc81c42\uad70\ub2e8\uacfc \uc81c66\uad70\ub2e8\uc744 \uad6c\ud638\ud558\uae30 \uc704\ud574 \uc911\uad6d\uad70\uc740 \uc870\uc120\uc778\ubbfc\uad70 \uc81c2\uad70\ub2e8\uacfc \uc81c5\uad70\ub2e8\uacfc \ud568\uaed8 \uacf5\uc138 \uc791\uc804\uc744 \uc911\ub2e8\ud588\ub2e4.", "sentence_answer": "1\uc6d4 5\uc77c, \ub9ac\uc9c0\uc6e8\uc774\ub294 D \ub77c\uc778\uc774\ub77c \uba85\uba85\ub41c \uc0c8\ub85c\uc6b4 \ubc29\uc5b4\uc120\uc744 \uc124\ub9bd\ud558\uae30 \uc704\ud574 \ubaa8\ub4e0 \uc720\uc5d4\uad70 \ubcd1\ub825\uc5d0\uac8c 37\ub3c4\uc120 \uc774\ub0a8\uc744 \ucca0\uc218\ud560 \uac83\uc744 \uc9c0\uc2dc\ud588\uace0, \uc911\ubd80 \ubc0f \ub3d9\ubd80 \uc804\uc120\uc758 \uc720\uc5d4\uad70\uc740 \uc6d0\uc8fc\uc640 \ud574\uc548\ub9c8\uc744\uc778 \uc8fc\ubb38\uc9c4 \uc0ac\uc774 \uc5d0", "paragraph_id": "6550565-7-2", "paragraph_question": "question: \uc911\ubd80 \ubc0f \ub3d9\ubd80 \uc804\uc120\uc758 \uc720\uc5d4\uad70\uc774 \ubc29\uc5b4\uc9c4\uc9c0\ub97c \uc124\uce58\ud55c \uc7a5\uc18c\ub294?, context: \uc911\uad6d\uad70\uacfc \ubd81\ud55c\uad70\uc758 \ucd08\uae30 \uacf5\uaca9\uc774 \uc131\uacf5\ud558\uc5ec \ub300\ud55c\ubbfc\uad6d \uc81c1\uad70\ub2e8\uc740 \uc6d0\ub798 \ubc29\uc5b4\uc9c0\uc810\uc774\uc5c8\ub358 \ud604\ub9ac\uc5d0\uc11c\uc758 \uc0c1\ud669 \ud68c\ubcf5 \uc2dc\ub3c4\ub97c \ud3ec\uae30\ud560 \uc218\ubc16\uc5d0 \uc5c6\uc5c8\ub2e4. \uc774\ud6c4 \ubd81\ud55c\uad70 \ubcd1\ub825 \ub2e4\uc218\uac00 \uc6d0\uc8fc\uc758 \ubbf8\uad6d \uc81c2\ubcf4\ubcd1\uc0ac\ub2e8\uacfc \ub3d9\ud574\uc548\uc758 \ub300\ud55c\ubbfc\uad6d \uc81c1\uad70\ub2e8 \uc0ac\uc774\uc5d0 \uc0dd\uae34 \ud2c8 \uc0ac\uc774\ub85c \ubc00\uace0 \ub4e4\uc5b4\uc654\ub2e4. \ub9ac\uc9c0\uc6e8\uc774\ub294 \uc911\ubd80 \uc804\uc120\uc5d0 \ub300\ud55c \uc870\uc911 \uc5f0\ud569\uad70\uc758 \uacf5\uaca9\uc774 \uc11c\uc6b8\uc758 \uc720\uc5d4\uad70 \ubc29\uc5b4\uc790\ub4e4\uc744 \ud3ec\uc704\ud558\uae30 \uc704\ud55c \uc2dc\ub3c4\ub77c\uace0 \ud574\uc11d\ud558\uace0 \uadf8\ub294 1\uc6d4 3\uc77c \uc11c\uc6b8 \uc8fc\ub454 \uc720\uc5d4\uad70\uc5d0\uac8c \uc989\uac01\uc801\uc778 \ucca0\uc218\ub97c \uba85\ub839\ud588\ub2e4. 1\uc6d4 5\uc77c, \ub9ac\uc9c0\uc6e8\uc774\ub294 D \ub77c\uc778\uc774\ub77c \uba85\uba85\ub41c \uc0c8\ub85c\uc6b4 \ubc29\uc5b4\uc120\uc744 \uc124\ub9bd\ud558\uae30 \uc704\ud574 \ubaa8\ub4e0 \uc720\uc5d4\uad70 \ubcd1\ub825\uc5d0\uac8c 37\ub3c4\uc120 \uc774\ub0a8\uc744 \ucca0\uc218\ud560 \uac83\uc744 \uc9c0\uc2dc\ud588\uace0, \uc911\ubd80 \ubc0f \ub3d9\ubd80 \uc804\uc120\uc758 \uc720\uc5d4\uad70\uc740 \uc6d0\uc8fc\uc640 \ud574\uc548\ub9c8\uc744\uc778 \uc8fc\ubb38\uc9c4 \uc0ac\uc774\uc5d0 \ubc29\uc5b4\uc9c4\uc9c0\ub4e4\uc744 \uc124\uce58\ud588\ub2e4. \uac19\uc740 \uc2dc\uae30\uc5d0 \uc911\uad6d\uc778\ubbfc\uc9c0\uc6d0\uad70 \uc81c42\uad70\ub2e8\uacfc \uc81c66\uad70\ub2e8\uc744 \uad6c\ud638\ud558\uae30 \uc704\ud574 \uc911\uad6d\uad70\uc740 \uc870\uc120\uc778\ubbfc\uad70 \uc81c2\uad70\ub2e8\uacfc \uc81c5\uad70\ub2e8\uacfc \ud568\uaed8 \uacf5\uc138 \uc791\uc804\uc744 \uc911\ub2e8\ud588\ub2e4."} {"question": "\uc544\uc2a4\ud398\ub978 \uc5d0\uc2ac\ub9c1 \uc804\ud22c\uc5d0\uc11c \uc544\uc2a4\ud398\ub978\uc758 \ud575\uc2ec\uc694\uc0c8\ub97c \ub048\uae30\uc788\uac8c \ubc29\uc5b4\ud55c \uc0ac\ub78c\uc740?", "paragraph": "\uc138 \uac1c\uc758 \uc624\uc2a4\ud2b8\ub9ac\uc544 \ubd80\ub300\ub294 \ub9c8\uc744\uc758 \ubc18\ub3c4 \uc810\uac70\ud558\uc9c0 \ubabb\ud588\ub2e4. \uc544\uc2a4\ud398\ub978\uc758 \uc8fc\uc694 \uac70\uc810 \uc694\uc0c8\ub294 \ub9c8\uc138\ub098\uac00 \ub048\uc9c8\uae30\uac8c \ubc29\uc5b4\ud558\uace0 \uc788\uc5c8\uace0, \ubc24\uc774 \ub418\uc5b4\uc11c\uc57c \ud568\ub77d\ub418\uc5c8\ub2e4. \uadf8\ub7ec\ub294 \ub3d9\uc548 \ub450 \ub9c8\uc744 \uc0ac\uc774\uc640 \ub2e4\ub9ac\uc758 \uc55e\ubd80\ubd84\uc5d0 \uc788\ub358 \ud504\ub791\uc2a4\uc758 \uc804\uad70\uc774 \uc804\uc7a5\uc5d0 \ud22c\uc785\ub418\uc5b4 \uc624\uc2a4\ud2b8\ub9ac\uc544\uad70\uc758 \uce21\uba74\uc744 \uacf5\uaca9\ud558\uc600\ub2e4. \ub098\ud3f4\ub808\uc639\uc740 \ubd80\ub300\ub97c \ub098\ub204\uc5b4 \uc77c\ubd80 \ubcf4\ubcd1\ub300\ub97c \uc911\uc559\uc758 \uc804\uba74\uc73c\ub85c \uc774\ub3d9\uc2dc\ucf30\uace0, \uae30\ubcd1\ub9cc\uc73c\ub85c \uc774\ub8e8\uc5b4\uc9c4 \ubd80\ub300\uc5d0\uac8c \uba85\ub839\uc744 \ub0b4\ub824 \uae34 \uc5f4\uc744 \uc774\ub8e8\uc5b4 \uc544\uc2a4\ud398\ub978\uc744 \uacf5\uaca9\ud558\ub358 \uc624\uc2a4\ud2b8\ub9ac\uc544 \ud3ec\ubcd1\ub300\uc5d0\uac8c \uacf5\uaca9\ud560 \uac83\uc744 \uba85\ud588\ub2e4. \ud504\ub791\uc2a4\uad70\uc758 1\ucc28 \ub3cc\uaca9\uc740 \uc624\uc2a4\ud2b8\ub9ac\uc544\uad70\uc5d0\uac8c \uaca9\ud1f4 \ub2f9\ud588\uc73c\ub098, \ub450 \ubc88\uc9f8 \ud749\uac11\uae30\ubcd1(cuirassires)\uc758 \uacf5\uaca9\uc740 \uc131\uacf5\ud558\uc600\ub2e4. \ud504\ub791\uc2a4 \uae30\ubcd1\ub300\ub294 \uc624\uc2a4\ud2b8\ub9ac\uc544\uad70\uc758 \ud3ec\ub97c \ud0c8\ucde8\ud558\uace0 \uc5ec\uc138\ub97c \ubab0\uc544 \ud638\uc5d4\ucd10\ub808\ub978 \ud718\ud558\uc758 \ubcf4\ubcd1 \ubc29\uc9c4\uc5d0 \uacf5\uaca9\uc744 \uac00\ud558\uace0 \ub9ac\ud788\ud150\uc288\ud0c0\uc778(Lichtenstein)\uc758 \uae30\ubcd1 \ubd80\ub300\uc758 \uacf5\uaca9\uc5d0 \uc800\ud56d\ud558\uc600\ub2e4. \uadf8\ub7ec\ub098 \uc774\ub4e4\uc740 \uc774 \uc774\uc0c1\uc758 \ud589\ub3d9\uc744 \ud560 \uc218\ub294 \uc5c6\uc5c8\uae30\uc5d0 \uc885\uad6d\uc5d0\ub294 \ud1f4\uac01\ud558\uc5ec \uc6d0\ub798 \uc704\uce58\ub85c \ub3cc\uc544\uac14\ub2e4.", "answer": "\ub9c8\uc138\ub098", "sentence": "\uc544\uc2a4\ud398\ub978\uc758 \uc8fc\uc694 \uac70\uc810 \uc694\uc0c8\ub294 \ub9c8\uc138\ub098 \uac00 \ub048\uc9c8\uae30\uac8c \ubc29\uc5b4\ud558\uace0 \uc788\uc5c8\uace0, \ubc24\uc774 \ub418\uc5b4\uc11c\uc57c \ud568\ub77d\ub418\uc5c8\ub2e4.", "paragraph_sentence": "\uc138 \uac1c\uc758 \uc624\uc2a4\ud2b8\ub9ac\uc544 \ubd80\ub300\ub294 \ub9c8\uc744\uc758 \ubc18\ub3c4 \uc810\uac70\ud558\uc9c0 \ubabb\ud588\ub2e4. \uc544\uc2a4\ud398\ub978\uc758 \uc8fc\uc694 \uac70\uc810 \uc694\uc0c8\ub294 \ub9c8\uc138\ub098 \uac00 \ub048\uc9c8\uae30\uac8c \ubc29\uc5b4\ud558\uace0 \uc788\uc5c8\uace0, \ubc24\uc774 \ub418\uc5b4\uc11c\uc57c \ud568\ub77d\ub418\uc5c8\ub2e4. \uadf8\ub7ec\ub294 \ub3d9\uc548 \ub450 \ub9c8\uc744 \uc0ac\uc774\uc640 \ub2e4\ub9ac\uc758 \uc55e\ubd80\ubd84\uc5d0 \uc788\ub358 \ud504\ub791\uc2a4\uc758 \uc804\uad70\uc774 \uc804\uc7a5\uc5d0 \ud22c\uc785\ub418\uc5b4 \uc624\uc2a4\ud2b8\ub9ac\uc544\uad70\uc758 \uce21\uba74\uc744 \uacf5\uaca9\ud558\uc600\ub2e4. \ub098\ud3f4\ub808\uc639\uc740 \ubd80\ub300\ub97c \ub098\ub204\uc5b4 \uc77c\ubd80 \ubcf4\ubcd1\ub300\ub97c \uc911\uc559\uc758 \uc804\uba74\uc73c\ub85c \uc774\ub3d9\uc2dc\ucf30\uace0, \uae30\ubcd1\ub9cc\uc73c\ub85c \uc774\ub8e8\uc5b4\uc9c4 \ubd80\ub300\uc5d0\uac8c \uba85\ub839\uc744 \ub0b4\ub824 \uae34 \uc5f4\uc744 \uc774\ub8e8\uc5b4 \uc544\uc2a4\ud398\ub978\uc744 \uacf5\uaca9\ud558\ub358 \uc624\uc2a4\ud2b8\ub9ac\uc544 \ud3ec\ubcd1\ub300\uc5d0\uac8c \uacf5\uaca9\ud560 \uac83\uc744 \uba85\ud588\ub2e4. \ud504\ub791\uc2a4\uad70\uc758 1\ucc28 \ub3cc\uaca9\uc740 \uc624\uc2a4\ud2b8\ub9ac\uc544\uad70\uc5d0\uac8c \uaca9\ud1f4 \ub2f9\ud588\uc73c\ub098, \ub450 \ubc88\uc9f8 \ud749\uac11\uae30\ubcd1(cuirassires)\uc758 \uacf5\uaca9\uc740 \uc131\uacf5\ud558\uc600\ub2e4. \ud504\ub791\uc2a4 \uae30\ubcd1\ub300\ub294 \uc624\uc2a4\ud2b8\ub9ac\uc544\uad70\uc758 \ud3ec\ub97c \ud0c8\ucde8\ud558\uace0 \uc5ec\uc138\ub97c \ubab0\uc544 \ud638\uc5d4\ucd10\ub808\ub978 \ud718\ud558\uc758 \ubcf4\ubcd1 \ubc29\uc9c4\uc5d0 \uacf5\uaca9\uc744 \uac00\ud558\uace0 \ub9ac\ud788\ud150\uc288\ud0c0\uc778(Lichtenstein)\uc758 \uae30\ubcd1 \ubd80\ub300\uc758 \uacf5\uaca9\uc5d0 \uc800\ud56d\ud558\uc600\ub2e4. \uadf8\ub7ec\ub098 \uc774\ub4e4\uc740 \uc774 \uc774\uc0c1\uc758 \ud589\ub3d9\uc744 \ud560 \uc218\ub294 \uc5c6\uc5c8\uae30\uc5d0 \uc885\uad6d\uc5d0\ub294 \ud1f4\uac01\ud558\uc5ec \uc6d0\ub798 \uc704\uce58\ub85c \ub3cc\uc544\uac14\ub2e4.", "paragraph_answer": "\uc138 \uac1c\uc758 \uc624\uc2a4\ud2b8\ub9ac\uc544 \ubd80\ub300\ub294 \ub9c8\uc744\uc758 \ubc18\ub3c4 \uc810\uac70\ud558\uc9c0 \ubabb\ud588\ub2e4. \uc544\uc2a4\ud398\ub978\uc758 \uc8fc\uc694 \uac70\uc810 \uc694\uc0c8\ub294 \ub9c8\uc138\ub098 \uac00 \ub048\uc9c8\uae30\uac8c \ubc29\uc5b4\ud558\uace0 \uc788\uc5c8\uace0, \ubc24\uc774 \ub418\uc5b4\uc11c\uc57c \ud568\ub77d\ub418\uc5c8\ub2e4. \uadf8\ub7ec\ub294 \ub3d9\uc548 \ub450 \ub9c8\uc744 \uc0ac\uc774\uc640 \ub2e4\ub9ac\uc758 \uc55e\ubd80\ubd84\uc5d0 \uc788\ub358 \ud504\ub791\uc2a4\uc758 \uc804\uad70\uc774 \uc804\uc7a5\uc5d0 \ud22c\uc785\ub418\uc5b4 \uc624\uc2a4\ud2b8\ub9ac\uc544\uad70\uc758 \uce21\uba74\uc744 \uacf5\uaca9\ud558\uc600\ub2e4. \ub098\ud3f4\ub808\uc639\uc740 \ubd80\ub300\ub97c \ub098\ub204\uc5b4 \uc77c\ubd80 \ubcf4\ubcd1\ub300\ub97c \uc911\uc559\uc758 \uc804\uba74\uc73c\ub85c \uc774\ub3d9\uc2dc\ucf30\uace0, \uae30\ubcd1\ub9cc\uc73c\ub85c \uc774\ub8e8\uc5b4\uc9c4 \ubd80\ub300\uc5d0\uac8c \uba85\ub839\uc744 \ub0b4\ub824 \uae34 \uc5f4\uc744 \uc774\ub8e8\uc5b4 \uc544\uc2a4\ud398\ub978\uc744 \uacf5\uaca9\ud558\ub358 \uc624\uc2a4\ud2b8\ub9ac\uc544 \ud3ec\ubcd1\ub300\uc5d0\uac8c \uacf5\uaca9\ud560 \uac83\uc744 \uba85\ud588\ub2e4. \ud504\ub791\uc2a4\uad70\uc758 1\ucc28 \ub3cc\uaca9\uc740 \uc624\uc2a4\ud2b8\ub9ac\uc544\uad70\uc5d0\uac8c \uaca9\ud1f4 \ub2f9\ud588\uc73c\ub098, \ub450 \ubc88\uc9f8 \ud749\uac11\uae30\ubcd1(cuirassires)\uc758 \uacf5\uaca9\uc740 \uc131\uacf5\ud558\uc600\ub2e4. \ud504\ub791\uc2a4 \uae30\ubcd1\ub300\ub294 \uc624\uc2a4\ud2b8\ub9ac\uc544\uad70\uc758 \ud3ec\ub97c \ud0c8\ucde8\ud558\uace0 \uc5ec\uc138\ub97c \ubab0\uc544 \ud638\uc5d4\ucd10\ub808\ub978 \ud718\ud558\uc758 \ubcf4\ubcd1 \ubc29\uc9c4\uc5d0 \uacf5\uaca9\uc744 \uac00\ud558\uace0 \ub9ac\ud788\ud150\uc288\ud0c0\uc778(Lichtenstein)\uc758 \uae30\ubcd1 \ubd80\ub300\uc758 \uacf5\uaca9\uc5d0 \uc800\ud56d\ud558\uc600\ub2e4. \uadf8\ub7ec\ub098 \uc774\ub4e4\uc740 \uc774 \uc774\uc0c1\uc758 \ud589\ub3d9\uc744 \ud560 \uc218\ub294 \uc5c6\uc5c8\uae30\uc5d0 \uc885\uad6d\uc5d0\ub294 \ud1f4\uac01\ud558\uc5ec \uc6d0\ub798 \uc704\uce58\ub85c \ub3cc\uc544\uac14\ub2e4.", "sentence_answer": "\uc544\uc2a4\ud398\ub978\uc758 \uc8fc\uc694 \uac70\uc810 \uc694\uc0c8\ub294 \ub9c8\uc138\ub098 \uac00 \ub048\uc9c8\uae30\uac8c \ubc29\uc5b4\ud558\uace0 \uc788\uc5c8\uace0, \ubc24\uc774 \ub418\uc5b4\uc11c\uc57c \ud568\ub77d\ub418\uc5c8\ub2e4.", "paragraph_id": "6533702-3-0", "paragraph_question": "question: \uc544\uc2a4\ud398\ub978 \uc5d0\uc2ac\ub9c1 \uc804\ud22c\uc5d0\uc11c \uc544\uc2a4\ud398\ub978\uc758 \ud575\uc2ec\uc694\uc0c8\ub97c \ub048\uae30\uc788\uac8c \ubc29\uc5b4\ud55c \uc0ac\ub78c\uc740?, context: \uc138 \uac1c\uc758 \uc624\uc2a4\ud2b8\ub9ac\uc544 \ubd80\ub300\ub294 \ub9c8\uc744\uc758 \ubc18\ub3c4 \uc810\uac70\ud558\uc9c0 \ubabb\ud588\ub2e4. \uc544\uc2a4\ud398\ub978\uc758 \uc8fc\uc694 \uac70\uc810 \uc694\uc0c8\ub294 \ub9c8\uc138\ub098\uac00 \ub048\uc9c8\uae30\uac8c \ubc29\uc5b4\ud558\uace0 \uc788\uc5c8\uace0, \ubc24\uc774 \ub418\uc5b4\uc11c\uc57c \ud568\ub77d\ub418\uc5c8\ub2e4. \uadf8\ub7ec\ub294 \ub3d9\uc548 \ub450 \ub9c8\uc744 \uc0ac\uc774\uc640 \ub2e4\ub9ac\uc758 \uc55e\ubd80\ubd84\uc5d0 \uc788\ub358 \ud504\ub791\uc2a4\uc758 \uc804\uad70\uc774 \uc804\uc7a5\uc5d0 \ud22c\uc785\ub418\uc5b4 \uc624\uc2a4\ud2b8\ub9ac\uc544\uad70\uc758 \uce21\uba74\uc744 \uacf5\uaca9\ud558\uc600\ub2e4. \ub098\ud3f4\ub808\uc639\uc740 \ubd80\ub300\ub97c \ub098\ub204\uc5b4 \uc77c\ubd80 \ubcf4\ubcd1\ub300\ub97c \uc911\uc559\uc758 \uc804\uba74\uc73c\ub85c \uc774\ub3d9\uc2dc\ucf30\uace0, \uae30\ubcd1\ub9cc\uc73c\ub85c \uc774\ub8e8\uc5b4\uc9c4 \ubd80\ub300\uc5d0\uac8c \uba85\ub839\uc744 \ub0b4\ub824 \uae34 \uc5f4\uc744 \uc774\ub8e8\uc5b4 \uc544\uc2a4\ud398\ub978\uc744 \uacf5\uaca9\ud558\ub358 \uc624\uc2a4\ud2b8\ub9ac\uc544 \ud3ec\ubcd1\ub300\uc5d0\uac8c \uacf5\uaca9\ud560 \uac83\uc744 \uba85\ud588\ub2e4. \ud504\ub791\uc2a4\uad70\uc758 1\ucc28 \ub3cc\uaca9\uc740 \uc624\uc2a4\ud2b8\ub9ac\uc544\uad70\uc5d0\uac8c \uaca9\ud1f4 \ub2f9\ud588\uc73c\ub098, \ub450 \ubc88\uc9f8 \ud749\uac11\uae30\ubcd1(cuirassires)\uc758 \uacf5\uaca9\uc740 \uc131\uacf5\ud558\uc600\ub2e4. \ud504\ub791\uc2a4 \uae30\ubcd1\ub300\ub294 \uc624\uc2a4\ud2b8\ub9ac\uc544\uad70\uc758 \ud3ec\ub97c \ud0c8\ucde8\ud558\uace0 \uc5ec\uc138\ub97c \ubab0\uc544 \ud638\uc5d4\ucd10\ub808\ub978 \ud718\ud558\uc758 \ubcf4\ubcd1 \ubc29\uc9c4\uc5d0 \uacf5\uaca9\uc744 \uac00\ud558\uace0 \ub9ac\ud788\ud150\uc288\ud0c0\uc778(Lichtenstein)\uc758 \uae30\ubcd1 \ubd80\ub300\uc758 \uacf5\uaca9\uc5d0 \uc800\ud56d\ud558\uc600\ub2e4. \uadf8\ub7ec\ub098 \uc774\ub4e4\uc740 \uc774 \uc774\uc0c1\uc758 \ud589\ub3d9\uc744 \ud560 \uc218\ub294 \uc5c6\uc5c8\uae30\uc5d0 \uc885\uad6d\uc5d0\ub294 \ud1f4\uac01\ud558\uc5ec \uc6d0\ub798 \uc704\uce58\ub85c \ub3cc\uc544\uac14\ub2e4."} {"question": "\ucd5c\uc9c4\uc2e4\uc774 \uc5f0\ub300 \uc758\uc0ac\ub97c \ubc1d\ud78c \ub2e8\uccb4\ub294?", "paragraph": "\uc774 \uc18c\uc1a1\uacfc \uad00\ub828\ud558\uc5ec \uc2e0\ud55c\uc758 \uc18c\uc7a5\uc5d0\ub294 '\uc774 \uc544\ud30c\ud2b8\uc5d0 \ub4e4\uc5b4\uac00\uba74 \uba40\uca61\ud55c \ubd80\ubd80\ub3c4 \uac08\ub77c\uc11c\uaca0\ub2e4'\ub294 \uad6c\uc808\uc774 \uba85\uc2dc\ub418\uc5b4 \uc788\uc5c8\ub2e4. 2004\ub144 11\uc6d4 24\uc77c, \ucd5c\uc9c4\uc2e4\uc740 \uc5ec\uc131\uc2e0\ubb38\uacfc\uc758 \uc778\ud130\ubdf0\uc5d0\uc11c \uc774 \uc0ac\uac74\uc5d0 \ub300\ud574 '\uc774\ud63c\ub140\uc5d0 \ub300\ud55c \ud3b8\uacac\uacfc \ucc28\ubcc4\uc5d0 \uc758\ud55c \uc5ec\uc131\uc778\uad8c \uce68\ud574'\ub77c\uace0 \uaddc\uc815\ud558\uc600\uace0 \uc5ec\uc131\uc6b4\ub3d9\ub2e8\uccb4\uc640 \uc5f0\ub300\ud558\uaca0\ub2e4\ub294 \uc758\uc0ac\ub97c \ubc1d\ud614\ub2e4. \ucd5c\uc9c4\uc2e4\uc740 \uc774 \uc778\ud130\ubdf0\uc5d0\uc11c \"\uc774\ubc88 \uc0ac\uac74\uc73c\ub85c \uc778\ud574 \uc774\ud63c\ub140\uac00 '\uc0ac\ud68c\uc801 \uba85\uc608\uac00 \uc2e4\ucd94\ub41c \uc0ac\ub78c', '\uc0ac\uc0dd\ud65c \uad00\ub9ac\ub97c \ubabb\ud55c \uc0ac\ub78c'\uc774\ub77c\ub294 '\uc8fc\ud64d\uae00\uc528' \uc120\ub840\ub97c \ub0a8\uae30\uc9c0 \uc54a\uac8c \ucd5c\uc120\uc744 \ub2e4\ud574 \ub300\uc751\ud558\uaca0\ub2e4. (\uc2e0\ud55c\uc774 \ub0b4\uac8c \uc18c\uc1a1\uc744 \uac74 \uc774\uc720\ub85c \ubbf8\ub8e8\uc5b4\ubcf4\uc544) \uadf8\ub807\ub2e4\uba74 \uc2e0\ud55c\uc740 \uc774\ud63c\ub140\uc5d0\uac90 \uc544\uc608 \uc544\ud30c\ud2b8 \ubd84\uc591 \uc790\uaca9\uc744 \uc8fc\uc9c0 \ub9d0\uc544\uc57c \ud558\ub294 \uac83 \uc544\ub2cc\uac00. \uc774\ubc88 \uc0ac\uac74\uc744 \ud1b5\ud574 '\uc5ec\uc131'\uc73c\ub85c\uc11c\uc758 \uc544\ud514\uc744 \uc808\uac10\ud588\ub2e4. \uc5ec\uc131\uc778\uad8c \ud5a5\uc0c1\uc744 \uc704\ud574 \uc560\uc4f0\ub294 \uc5ec\uc131\uc6b4\ub3d9\ub2e8\uccb4\ub4e4\uc774 \ub0b4 \ub3c4\uc6c0\uc744 \ud544\uc694\ub85c \ud55c\ub2e4\uba74 \uc5b8\uc81c\ub4e0 \ub3d5\uace0 \uc2f6\ub2e4\"\ub77c\uace0 \ub9d0\ud588\ub2e4.", "answer": "\uc5ec\uc131\uc6b4\ub3d9\ub2e8\uccb4", "sentence": "2004\ub144 11\uc6d4 24\uc77c, \ucd5c\uc9c4\uc2e4\uc740 \uc5ec\uc131\uc2e0\ubb38\uacfc\uc758 \uc778\ud130\ubdf0\uc5d0\uc11c \uc774 \uc0ac\uac74\uc5d0 \ub300\ud574 '\uc774\ud63c\ub140\uc5d0 \ub300\ud55c \ud3b8\uacac\uacfc \ucc28\ubcc4\uc5d0 \uc758\ud55c \uc5ec\uc131\uc778\uad8c \uce68\ud574'\ub77c\uace0 \uaddc\uc815\ud558\uc600\uace0 \uc5ec\uc131\uc6b4\ub3d9\ub2e8\uccb4 \uc640 \uc5f0\ub300\ud558\uaca0\ub2e4\ub294 \uc758\uc0ac\ub97c \ubc1d\ud614\ub2e4.", "paragraph_sentence": "\uc774 \uc18c\uc1a1\uacfc \uad00\ub828\ud558\uc5ec \uc2e0\ud55c\uc758 \uc18c\uc7a5\uc5d0\ub294 '\uc774 \uc544\ud30c\ud2b8\uc5d0 \ub4e4\uc5b4\uac00\uba74 \uba40\uca61\ud55c \ubd80\ubd80\ub3c4 \uac08\ub77c\uc11c\uaca0\ub2e4'\ub294 \uad6c\uc808\uc774 \uba85\uc2dc\ub418\uc5b4 \uc788\uc5c8\ub2e4. 2004\ub144 11\uc6d4 24\uc77c, \ucd5c\uc9c4\uc2e4\uc740 \uc5ec\uc131\uc2e0\ubb38\uacfc\uc758 \uc778\ud130\ubdf0\uc5d0\uc11c \uc774 \uc0ac\uac74\uc5d0 \ub300\ud574 '\uc774\ud63c\ub140\uc5d0 \ub300\ud55c \ud3b8\uacac\uacfc \ucc28\ubcc4\uc5d0 \uc758\ud55c \uc5ec\uc131\uc778\uad8c \uce68\ud574'\ub77c\uace0 \uaddc\uc815\ud558\uc600\uace0 \uc5ec\uc131\uc6b4\ub3d9\ub2e8\uccb4 \uc640 \uc5f0\ub300\ud558\uaca0\ub2e4\ub294 \uc758\uc0ac\ub97c \ubc1d\ud614\ub2e4. \ucd5c\uc9c4\uc2e4\uc740 \uc774 \uc778\ud130\ubdf0\uc5d0\uc11c \"\uc774\ubc88 \uc0ac\uac74\uc73c\ub85c \uc778\ud574 \uc774\ud63c\ub140\uac00 '\uc0ac\ud68c\uc801 \uba85\uc608\uac00 \uc2e4\ucd94\ub41c \uc0ac\ub78c', '\uc0ac\uc0dd\ud65c \uad00\ub9ac\ub97c \ubabb\ud55c \uc0ac\ub78c'\uc774\ub77c\ub294 '\uc8fc\ud64d\uae00\uc528' \uc120\ub840\ub97c \ub0a8\uae30\uc9c0 \uc54a\uac8c \ucd5c\uc120\uc744 \ub2e4\ud574 \ub300\uc751\ud558\uaca0\ub2e4. (\uc2e0\ud55c\uc774 \ub0b4\uac8c \uc18c\uc1a1\uc744 \uac74 \uc774\uc720\ub85c \ubbf8\ub8e8\uc5b4\ubcf4\uc544) \uadf8\ub807\ub2e4\uba74 \uc2e0\ud55c\uc740 \uc774\ud63c\ub140\uc5d0\uac90 \uc544\uc608 \uc544\ud30c\ud2b8 \ubd84\uc591 \uc790\uaca9\uc744 \uc8fc\uc9c0 \ub9d0\uc544\uc57c \ud558\ub294 \uac83 \uc544\ub2cc\uac00. \uc774\ubc88 \uc0ac\uac74\uc744 \ud1b5\ud574 '\uc5ec\uc131'\uc73c\ub85c\uc11c\uc758 \uc544\ud514\uc744 \uc808\uac10\ud588\ub2e4. \uc5ec\uc131\uc778\uad8c \ud5a5\uc0c1\uc744 \uc704\ud574 \uc560\uc4f0\ub294 \uc5ec\uc131\uc6b4\ub3d9\ub2e8\uccb4\ub4e4\uc774 \ub0b4 \ub3c4\uc6c0\uc744 \ud544\uc694\ub85c \ud55c\ub2e4\uba74 \uc5b8\uc81c\ub4e0 \ub3d5\uace0 \uc2f6\ub2e4\"\ub77c\uace0 \ub9d0\ud588\ub2e4.", "paragraph_answer": "\uc774 \uc18c\uc1a1\uacfc \uad00\ub828\ud558\uc5ec \uc2e0\ud55c\uc758 \uc18c\uc7a5\uc5d0\ub294 '\uc774 \uc544\ud30c\ud2b8\uc5d0 \ub4e4\uc5b4\uac00\uba74 \uba40\uca61\ud55c \ubd80\ubd80\ub3c4 \uac08\ub77c\uc11c\uaca0\ub2e4'\ub294 \uad6c\uc808\uc774 \uba85\uc2dc\ub418\uc5b4 \uc788\uc5c8\ub2e4. 2004\ub144 11\uc6d4 24\uc77c, \ucd5c\uc9c4\uc2e4\uc740 \uc5ec\uc131\uc2e0\ubb38\uacfc\uc758 \uc778\ud130\ubdf0\uc5d0\uc11c \uc774 \uc0ac\uac74\uc5d0 \ub300\ud574 '\uc774\ud63c\ub140\uc5d0 \ub300\ud55c \ud3b8\uacac\uacfc \ucc28\ubcc4\uc5d0 \uc758\ud55c \uc5ec\uc131\uc778\uad8c \uce68\ud574'\ub77c\uace0 \uaddc\uc815\ud558\uc600\uace0 \uc5ec\uc131\uc6b4\ub3d9\ub2e8\uccb4 \uc640 \uc5f0\ub300\ud558\uaca0\ub2e4\ub294 \uc758\uc0ac\ub97c \ubc1d\ud614\ub2e4. \ucd5c\uc9c4\uc2e4\uc740 \uc774 \uc778\ud130\ubdf0\uc5d0\uc11c \"\uc774\ubc88 \uc0ac\uac74\uc73c\ub85c \uc778\ud574 \uc774\ud63c\ub140\uac00 '\uc0ac\ud68c\uc801 \uba85\uc608\uac00 \uc2e4\ucd94\ub41c \uc0ac\ub78c', '\uc0ac\uc0dd\ud65c \uad00\ub9ac\ub97c \ubabb\ud55c \uc0ac\ub78c'\uc774\ub77c\ub294 '\uc8fc\ud64d\uae00\uc528' \uc120\ub840\ub97c \ub0a8\uae30\uc9c0 \uc54a\uac8c \ucd5c\uc120\uc744 \ub2e4\ud574 \ub300\uc751\ud558\uaca0\ub2e4. (\uc2e0\ud55c\uc774 \ub0b4\uac8c \uc18c\uc1a1\uc744 \uac74 \uc774\uc720\ub85c \ubbf8\ub8e8\uc5b4\ubcf4\uc544) \uadf8\ub807\ub2e4\uba74 \uc2e0\ud55c\uc740 \uc774\ud63c\ub140\uc5d0\uac90 \uc544\uc608 \uc544\ud30c\ud2b8 \ubd84\uc591 \uc790\uaca9\uc744 \uc8fc\uc9c0 \ub9d0\uc544\uc57c \ud558\ub294 \uac83 \uc544\ub2cc\uac00. \uc774\ubc88 \uc0ac\uac74\uc744 \ud1b5\ud574 '\uc5ec\uc131'\uc73c\ub85c\uc11c\uc758 \uc544\ud514\uc744 \uc808\uac10\ud588\ub2e4. \uc5ec\uc131\uc778\uad8c \ud5a5\uc0c1\uc744 \uc704\ud574 \uc560\uc4f0\ub294 \uc5ec\uc131\uc6b4\ub3d9\ub2e8\uccb4\ub4e4\uc774 \ub0b4 \ub3c4\uc6c0\uc744 \ud544\uc694\ub85c \ud55c\ub2e4\uba74 \uc5b8\uc81c\ub4e0 \ub3d5\uace0 \uc2f6\ub2e4\"\ub77c\uace0 \ub9d0\ud588\ub2e4.", "sentence_answer": "2004\ub144 11\uc6d4 24\uc77c, \ucd5c\uc9c4\uc2e4\uc740 \uc5ec\uc131\uc2e0\ubb38\uacfc\uc758 \uc778\ud130\ubdf0\uc5d0\uc11c \uc774 \uc0ac\uac74\uc5d0 \ub300\ud574 '\uc774\ud63c\ub140\uc5d0 \ub300\ud55c \ud3b8\uacac\uacfc \ucc28\ubcc4\uc5d0 \uc758\ud55c \uc5ec\uc131\uc778\uad8c \uce68\ud574'\ub77c\uace0 \uaddc\uc815\ud558\uc600\uace0 \uc5ec\uc131\uc6b4\ub3d9\ub2e8\uccb4 \uc640 \uc5f0\ub300\ud558\uaca0\ub2e4\ub294 \uc758\uc0ac\ub97c \ubc1d\ud614\ub2e4.", "paragraph_id": "6560193-16-1", "paragraph_question": "question: \ucd5c\uc9c4\uc2e4\uc774 \uc5f0\ub300 \uc758\uc0ac\ub97c \ubc1d\ud78c \ub2e8\uccb4\ub294?, context: \uc774 \uc18c\uc1a1\uacfc \uad00\ub828\ud558\uc5ec \uc2e0\ud55c\uc758 \uc18c\uc7a5\uc5d0\ub294 '\uc774 \uc544\ud30c\ud2b8\uc5d0 \ub4e4\uc5b4\uac00\uba74 \uba40\uca61\ud55c \ubd80\ubd80\ub3c4 \uac08\ub77c\uc11c\uaca0\ub2e4'\ub294 \uad6c\uc808\uc774 \uba85\uc2dc\ub418\uc5b4 \uc788\uc5c8\ub2e4. 2004\ub144 11\uc6d4 24\uc77c, \ucd5c\uc9c4\uc2e4\uc740 \uc5ec\uc131\uc2e0\ubb38\uacfc\uc758 \uc778\ud130\ubdf0\uc5d0\uc11c \uc774 \uc0ac\uac74\uc5d0 \ub300\ud574 '\uc774\ud63c\ub140\uc5d0 \ub300\ud55c \ud3b8\uacac\uacfc \ucc28\ubcc4\uc5d0 \uc758\ud55c \uc5ec\uc131\uc778\uad8c \uce68\ud574'\ub77c\uace0 \uaddc\uc815\ud558\uc600\uace0 \uc5ec\uc131\uc6b4\ub3d9\ub2e8\uccb4\uc640 \uc5f0\ub300\ud558\uaca0\ub2e4\ub294 \uc758\uc0ac\ub97c \ubc1d\ud614\ub2e4. \ucd5c\uc9c4\uc2e4\uc740 \uc774 \uc778\ud130\ubdf0\uc5d0\uc11c \"\uc774\ubc88 \uc0ac\uac74\uc73c\ub85c \uc778\ud574 \uc774\ud63c\ub140\uac00 '\uc0ac\ud68c\uc801 \uba85\uc608\uac00 \uc2e4\ucd94\ub41c \uc0ac\ub78c', '\uc0ac\uc0dd\ud65c \uad00\ub9ac\ub97c \ubabb\ud55c \uc0ac\ub78c'\uc774\ub77c\ub294 '\uc8fc\ud64d\uae00\uc528' \uc120\ub840\ub97c \ub0a8\uae30\uc9c0 \uc54a\uac8c \ucd5c\uc120\uc744 \ub2e4\ud574 \ub300\uc751\ud558\uaca0\ub2e4. (\uc2e0\ud55c\uc774 \ub0b4\uac8c \uc18c\uc1a1\uc744 \uac74 \uc774\uc720\ub85c \ubbf8\ub8e8\uc5b4\ubcf4\uc544) \uadf8\ub807\ub2e4\uba74 \uc2e0\ud55c\uc740 \uc774\ud63c\ub140\uc5d0\uac90 \uc544\uc608 \uc544\ud30c\ud2b8 \ubd84\uc591 \uc790\uaca9\uc744 \uc8fc\uc9c0 \ub9d0\uc544\uc57c \ud558\ub294 \uac83 \uc544\ub2cc\uac00. \uc774\ubc88 \uc0ac\uac74\uc744 \ud1b5\ud574 '\uc5ec\uc131'\uc73c\ub85c\uc11c\uc758 \uc544\ud514\uc744 \uc808\uac10\ud588\ub2e4. \uc5ec\uc131\uc778\uad8c \ud5a5\uc0c1\uc744 \uc704\ud574 \uc560\uc4f0\ub294 \uc5ec\uc131\uc6b4\ub3d9\ub2e8\uccb4\ub4e4\uc774 \ub0b4 \ub3c4\uc6c0\uc744 \ud544\uc694\ub85c \ud55c\ub2e4\uba74 \uc5b8\uc81c\ub4e0 \ub3d5\uace0 \uc2f6\ub2e4\"\ub77c\uace0 \ub9d0\ud588\ub2e4."} {"question": "\ub0a8\ud3b8\uc740 \uba87\uac1c\uc6d4\ub3d9\uc548 \uc9c4\uc11c\uc5d0\uac8c \uc0c1\ub2f4\ubc1b\uc544 \uc654\ub294\uac00?", "paragraph": "\ud56d\uc0c1 \ub298 \uc8fc\ubaa9\uc744 \ubc1b\uc544\uc57c \ud560 \uc790\uc2e0\uc778\ub370\u2026\uac00\ub09c\ubc45\uc774 \uc8fc\uc81c\uc5d0 \uac78\ud54f\ud558\uba74 \uc5c4\ub9c8\uc640 \uc790\uc2e0\uc744 \ub450\ub4e4\uaca8 \ud328\ub300\ub358 \uc544\ubc84\uc9c0 \ub54c\ubb38\uc5d0 \ub298 \ubb34\uc2dc\ub2f9\ud558\uba70 \uc790\ub790\ub2e4. \uc724\ud0dd\ud55c \ud658\uacbd\uc5d0\uc11c \ubd80\ubaa8\ub2d8\uc758 \uc804\ud3ed\uc801 \uc0ac\ub791\uc744 \ubc1b\ub294 \uc9c4\uc11c\uac00 \ubab9\uc2dc\ub3c4 \ubd80\ub7ec\uc6b0\uba74\uc11c \ub298 \uc9c8\ud22c\uc758 \ub300\uc0c1\uc774\uc5c8\ub2e4. \uadf8\ub7f0\ub370 \uadf8\ub140\uac00 \uccab\uc0ac\ub791\ub9c8\uc800 \ube7c\uc557\uc544\ubc84\ub838\ub2e4. \uadf8\uac83\ub3c4 \uc544\uc8fc \ube44\uc5f4\ud55c \ubc29\ubc95\uc73c\ub85c\u2026\ubaa9\uc228 \uac19\uc740 \ub0a8\uc790\ub97c \uac00\ub85c\ucc44 \ub193\uace0 \uaca8\uc6b0 \ucf54\ud758\ub9ac\uac1c \uacc4\uc9d1\uc560\ud55c\ud14c \ube7c\uc557\uaca8? \ub0a8\ud3b8\uad00\ub9ac \uc798 \ud558\ub77c\uace0 \ud638\uc758\ub97c \ubca0\ud47c \uc77c\uc744 \uc6d0\uc218 \uc0bc\ub354\ub2c8, \uae09\uae30\uc57c \uc8fd\uc740 \ub0a8\ud3b8\uc744 8\uac1c\uc6d4\uc529\uc774\ub098 \uc0c1\ub2f4\ud574 \uc654\ub2e8\ub2e4. \uc0b4\uc544\uc0dd\uc804 \uc790\uc2e0\uc740 \uba85\ubb38\uac70\uc871\uc758 \uc790\uc190\uc774\ub78d\uc2dc\uace0 \uc790\uc2e0\uc744 \ubb34\uc2dc\ud588\ub358 \ub0a8\ud3b8! \uadf8\uc758 \uc8fd\uc74c\uc740 \ubc18\ub4dc\uc2dc \ubb34\uc2a8 \uc77c\uc774 \uc788\uc5b4\ub3c4 \uc0ac\uace0\uc0ac\ub85c \ubb3b\ud600\uc57c \ud588\ub2e4. \uadf8\ub7f0\ub370 \ucca0\ucc9c\uc9c0 \uc6d0\uc218 \uac19\uc740 \uc9c4\uc11c\uac00 \uadf8 \uc0ac\uac74\uc758 \uc2e4\uccb4\ub97c \ud30c\ud5e4\uce58\uaca0\ub2e4\uace0??? \ub0b4\uc2ec \ub450\ub824\uc6b0\uba74\uc11c\ub3c4 \uc774\ubc88\uc5d0\uc57c\ub9d0\ub85c \uae30\ud544\ucf54 \uc9c4\uc11c\ub97c \uc774\uaca8\ub0b4\ub9ac\ub77c \uc804\uc758\ub97c \ubd88\ud0dc\uc6b0\ub294\ub370\u2026", "answer": "8\uac1c\uc6d4", "sentence": "\ub0a8\ud3b8\uad00\ub9ac \uc798 \ud558\ub77c\uace0 \ud638\uc758\ub97c \ubca0\ud47c \uc77c\uc744 \uc6d0\uc218 \uc0bc\ub354\ub2c8, \uae09\uae30\uc57c \uc8fd\uc740 \ub0a8\ud3b8\uc744 8\uac1c\uc6d4 \uc529\uc774\ub098 \uc0c1\ub2f4\ud574 \uc654\ub2e8\ub2e4.", "paragraph_sentence": "\ud56d\uc0c1 \ub298 \uc8fc\ubaa9\uc744 \ubc1b\uc544\uc57c \ud560 \uc790\uc2e0\uc778\ub370\u2026\uac00\ub09c\ubc45\uc774 \uc8fc\uc81c\uc5d0 \uac78\ud54f\ud558\uba74 \uc5c4\ub9c8\uc640 \uc790\uc2e0\uc744 \ub450\ub4e4\uaca8 \ud328\ub300\ub358 \uc544\ubc84\uc9c0 \ub54c\ubb38\uc5d0 \ub298 \ubb34\uc2dc\ub2f9\ud558\uba70 \uc790\ub790\ub2e4. \uc724\ud0dd\ud55c \ud658\uacbd\uc5d0\uc11c \ubd80\ubaa8\ub2d8\uc758 \uc804\ud3ed\uc801 \uc0ac\ub791\uc744 \ubc1b\ub294 \uc9c4\uc11c\uac00 \ubab9\uc2dc\ub3c4 \ubd80\ub7ec\uc6b0\uba74\uc11c \ub298 \uc9c8\ud22c\uc758 \ub300\uc0c1\uc774\uc5c8\ub2e4. \uadf8\ub7f0\ub370 \uadf8\ub140\uac00 \uccab\uc0ac\ub791\ub9c8\uc800 \ube7c\uc557\uc544\ubc84\ub838\ub2e4. \uadf8\uac83\ub3c4 \uc544\uc8fc \ube44\uc5f4\ud55c \ubc29\ubc95\uc73c\ub85c\u2026\ubaa9\uc228 \uac19\uc740 \ub0a8\uc790\ub97c \uac00\ub85c\ucc44 \ub193\uace0 \uaca8\uc6b0 \ucf54\ud758\ub9ac\uac1c \uacc4\uc9d1\uc560\ud55c\ud14c \ube7c\uc557\uaca8? \ub0a8\ud3b8\uad00\ub9ac \uc798 \ud558\ub77c\uace0 \ud638\uc758\ub97c \ubca0\ud47c \uc77c\uc744 \uc6d0\uc218 \uc0bc\ub354\ub2c8, \uae09\uae30\uc57c \uc8fd\uc740 \ub0a8\ud3b8\uc744 8\uac1c\uc6d4 \uc529\uc774\ub098 \uc0c1\ub2f4\ud574 \uc654\ub2e8\ub2e4. \uc0b4\uc544\uc0dd\uc804 \uc790\uc2e0\uc740 \uba85\ubb38\uac70\uc871\uc758 \uc790\uc190\uc774\ub78d\uc2dc\uace0 \uc790\uc2e0\uc744 \ubb34\uc2dc\ud588\ub358 \ub0a8\ud3b8! \uadf8\uc758 \uc8fd\uc74c\uc740 \ubc18\ub4dc\uc2dc \ubb34\uc2a8 \uc77c\uc774 \uc788\uc5b4\ub3c4 \uc0ac\uace0\uc0ac\ub85c \ubb3b\ud600\uc57c \ud588\ub2e4. \uadf8\ub7f0\ub370 \ucca0\ucc9c\uc9c0 \uc6d0\uc218 \uac19\uc740 \uc9c4\uc11c\uac00 \uadf8 \uc0ac\uac74\uc758 \uc2e4\uccb4\ub97c \ud30c\ud5e4\uce58\uaca0\ub2e4\uace0? ?? \ub0b4\uc2ec \ub450\ub824\uc6b0\uba74\uc11c\ub3c4 \uc774\ubc88\uc5d0\uc57c\ub9d0\ub85c \uae30\ud544\ucf54 \uc9c4\uc11c\ub97c \uc774\uaca8\ub0b4\ub9ac\ub77c \uc804\uc758\ub97c \ubd88\ud0dc\uc6b0\ub294\ub370\u2026", "paragraph_answer": "\ud56d\uc0c1 \ub298 \uc8fc\ubaa9\uc744 \ubc1b\uc544\uc57c \ud560 \uc790\uc2e0\uc778\ub370\u2026\uac00\ub09c\ubc45\uc774 \uc8fc\uc81c\uc5d0 \uac78\ud54f\ud558\uba74 \uc5c4\ub9c8\uc640 \uc790\uc2e0\uc744 \ub450\ub4e4\uaca8 \ud328\ub300\ub358 \uc544\ubc84\uc9c0 \ub54c\ubb38\uc5d0 \ub298 \ubb34\uc2dc\ub2f9\ud558\uba70 \uc790\ub790\ub2e4. \uc724\ud0dd\ud55c \ud658\uacbd\uc5d0\uc11c \ubd80\ubaa8\ub2d8\uc758 \uc804\ud3ed\uc801 \uc0ac\ub791\uc744 \ubc1b\ub294 \uc9c4\uc11c\uac00 \ubab9\uc2dc\ub3c4 \ubd80\ub7ec\uc6b0\uba74\uc11c \ub298 \uc9c8\ud22c\uc758 \ub300\uc0c1\uc774\uc5c8\ub2e4. \uadf8\ub7f0\ub370 \uadf8\ub140\uac00 \uccab\uc0ac\ub791\ub9c8\uc800 \ube7c\uc557\uc544\ubc84\ub838\ub2e4. \uadf8\uac83\ub3c4 \uc544\uc8fc \ube44\uc5f4\ud55c \ubc29\ubc95\uc73c\ub85c\u2026\ubaa9\uc228 \uac19\uc740 \ub0a8\uc790\ub97c \uac00\ub85c\ucc44 \ub193\uace0 \uaca8\uc6b0 \ucf54\ud758\ub9ac\uac1c \uacc4\uc9d1\uc560\ud55c\ud14c \ube7c\uc557\uaca8? \ub0a8\ud3b8\uad00\ub9ac \uc798 \ud558\ub77c\uace0 \ud638\uc758\ub97c \ubca0\ud47c \uc77c\uc744 \uc6d0\uc218 \uc0bc\ub354\ub2c8, \uae09\uae30\uc57c \uc8fd\uc740 \ub0a8\ud3b8\uc744 8\uac1c\uc6d4 \uc529\uc774\ub098 \uc0c1\ub2f4\ud574 \uc654\ub2e8\ub2e4. \uc0b4\uc544\uc0dd\uc804 \uc790\uc2e0\uc740 \uba85\ubb38\uac70\uc871\uc758 \uc790\uc190\uc774\ub78d\uc2dc\uace0 \uc790\uc2e0\uc744 \ubb34\uc2dc\ud588\ub358 \ub0a8\ud3b8! \uadf8\uc758 \uc8fd\uc74c\uc740 \ubc18\ub4dc\uc2dc \ubb34\uc2a8 \uc77c\uc774 \uc788\uc5b4\ub3c4 \uc0ac\uace0\uc0ac\ub85c \ubb3b\ud600\uc57c \ud588\ub2e4. \uadf8\ub7f0\ub370 \ucca0\ucc9c\uc9c0 \uc6d0\uc218 \uac19\uc740 \uc9c4\uc11c\uac00 \uadf8 \uc0ac\uac74\uc758 \uc2e4\uccb4\ub97c \ud30c\ud5e4\uce58\uaca0\ub2e4\uace0??? \ub0b4\uc2ec \ub450\ub824\uc6b0\uba74\uc11c\ub3c4 \uc774\ubc88\uc5d0\uc57c\ub9d0\ub85c \uae30\ud544\ucf54 \uc9c4\uc11c\ub97c \uc774\uaca8\ub0b4\ub9ac\ub77c \uc804\uc758\ub97c \ubd88\ud0dc\uc6b0\ub294\ub370\u2026", "sentence_answer": "\ub0a8\ud3b8\uad00\ub9ac \uc798 \ud558\ub77c\uace0 \ud638\uc758\ub97c \ubca0\ud47c \uc77c\uc744 \uc6d0\uc218 \uc0bc\ub354\ub2c8, \uae09\uae30\uc57c \uc8fd\uc740 \ub0a8\ud3b8\uc744 8\uac1c\uc6d4 \uc529\uc774\ub098 \uc0c1\ub2f4\ud574 \uc654\ub2e8\ub2e4.", "paragraph_id": "6556753-1-1", "paragraph_question": "question: \ub0a8\ud3b8\uc740 \uba87\uac1c\uc6d4\ub3d9\uc548 \uc9c4\uc11c\uc5d0\uac8c \uc0c1\ub2f4\ubc1b\uc544 \uc654\ub294\uac00?, context: \ud56d\uc0c1 \ub298 \uc8fc\ubaa9\uc744 \ubc1b\uc544\uc57c \ud560 \uc790\uc2e0\uc778\ub370\u2026\uac00\ub09c\ubc45\uc774 \uc8fc\uc81c\uc5d0 \uac78\ud54f\ud558\uba74 \uc5c4\ub9c8\uc640 \uc790\uc2e0\uc744 \ub450\ub4e4\uaca8 \ud328\ub300\ub358 \uc544\ubc84\uc9c0 \ub54c\ubb38\uc5d0 \ub298 \ubb34\uc2dc\ub2f9\ud558\uba70 \uc790\ub790\ub2e4. \uc724\ud0dd\ud55c \ud658\uacbd\uc5d0\uc11c \ubd80\ubaa8\ub2d8\uc758 \uc804\ud3ed\uc801 \uc0ac\ub791\uc744 \ubc1b\ub294 \uc9c4\uc11c\uac00 \ubab9\uc2dc\ub3c4 \ubd80\ub7ec\uc6b0\uba74\uc11c \ub298 \uc9c8\ud22c\uc758 \ub300\uc0c1\uc774\uc5c8\ub2e4. \uadf8\ub7f0\ub370 \uadf8\ub140\uac00 \uccab\uc0ac\ub791\ub9c8\uc800 \ube7c\uc557\uc544\ubc84\ub838\ub2e4. \uadf8\uac83\ub3c4 \uc544\uc8fc \ube44\uc5f4\ud55c \ubc29\ubc95\uc73c\ub85c\u2026\ubaa9\uc228 \uac19\uc740 \ub0a8\uc790\ub97c \uac00\ub85c\ucc44 \ub193\uace0 \uaca8\uc6b0 \ucf54\ud758\ub9ac\uac1c \uacc4\uc9d1\uc560\ud55c\ud14c \ube7c\uc557\uaca8? \ub0a8\ud3b8\uad00\ub9ac \uc798 \ud558\ub77c\uace0 \ud638\uc758\ub97c \ubca0\ud47c \uc77c\uc744 \uc6d0\uc218 \uc0bc\ub354\ub2c8, \uae09\uae30\uc57c \uc8fd\uc740 \ub0a8\ud3b8\uc744 8\uac1c\uc6d4\uc529\uc774\ub098 \uc0c1\ub2f4\ud574 \uc654\ub2e8\ub2e4. \uc0b4\uc544\uc0dd\uc804 \uc790\uc2e0\uc740 \uba85\ubb38\uac70\uc871\uc758 \uc790\uc190\uc774\ub78d\uc2dc\uace0 \uc790\uc2e0\uc744 \ubb34\uc2dc\ud588\ub358 \ub0a8\ud3b8! \uadf8\uc758 \uc8fd\uc74c\uc740 \ubc18\ub4dc\uc2dc \ubb34\uc2a8 \uc77c\uc774 \uc788\uc5b4\ub3c4 \uc0ac\uace0\uc0ac\ub85c \ubb3b\ud600\uc57c \ud588\ub2e4. \uadf8\ub7f0\ub370 \ucca0\ucc9c\uc9c0 \uc6d0\uc218 \uac19\uc740 \uc9c4\uc11c\uac00 \uadf8 \uc0ac\uac74\uc758 \uc2e4\uccb4\ub97c \ud30c\ud5e4\uce58\uaca0\ub2e4\uace0??? \ub0b4\uc2ec \ub450\ub824\uc6b0\uba74\uc11c\ub3c4 \uc774\ubc88\uc5d0\uc57c\ub9d0\ub85c \uae30\ud544\ucf54 \uc9c4\uc11c\ub97c \uc774\uaca8\ub0b4\ub9ac\ub77c \uc804\uc758\ub97c \ubd88\ud0dc\uc6b0\ub294\ub370\u2026"} {"question": "1959\ub144 \uc81c\uc0ac\ud574\uc6b4\ub3d9\uc73c\ub85c \uba78\uc885 \uc9c1\uc804\uc5d0 \uc774\ub974\ub978 \uc0c8 \uc774\ub984\uc740 \ubb34\uc5c7\uc778\uac00?", "paragraph": "\u300a\ub9ac\ub354\uc2a4 \ub2e4\uc774\uc81c\uc2a4\ud2b8\u300b\uc5d0 \ub530\ub974\uba74 1958\ub144\uc5d0 \ub9c8\uc624\ub294 \ucc38\uc0c8\uac00 \ubcbc\ub97c \uba39\ub294\ubaa8\uc2b5\uc744 \ubcf4\uace0 \ubd84\ub178\ud558\uc600\ub2e4. \ucc38\uc0c8\ub97c \uac00\ub9ac\ud0a4\uba70 \uc0ac\ub78c\uc774 \uba39\ub294 \uace1\uc2dd\uc744 \uc0c8\uac00 \uba39\uace0\uc788\ub2e4\ub2c8! \u201c\uc800 \uc0c8\ub294 \ud574\ub85c\uc6b4 \uc0c8\ub2e4.\u201d\ub77c\uace0 \ub9d0\ud588\ub294\ub370, \uc774 \ub9d0 \ub54c\ubb38\uc5d0 \uc911\uad6d\uc5d0\ub294 \ucc38\uc0c8\uc7a1\uc774 \uad11\ud48d\uc774 \ubd88\uc5b4 1959\ub144\uc5d0\ub294 \uc81c\uc0ac\ud574 \uc6b4\ub3d9\uc73c\ub85c \uc778\ud574 \uc911\uad6d \ucc38\uc0c8\uc758 \uc528\uac00 \ub9d0\ub790\ub2e4. \uace4\ucda9\uc744 \uba39\uc774\ub85c \ud558\ub294 \ucc38\uc0c8\uac00 \uc5c6\uc5b4\uc9c0\uba74\uc11c \uac01\uc885 \ud574\ucda9\uc774 \ucc3d\uad90\ud558\uc5ec \ub300\ud749\ub144\uc758 \ubc1c\uc0dd\uc73c\ub85c 4\ucc9c\ub9cc \uba85\uc774 \uad76\uc5b4\uc8fd\uc5c8\ub2e4. 1959\ub144\uc5d0 \uc5f4\ub9b0 \ub8e8\uc0b0 \ud68c\uc758\uc5d0\uc11c \uad6d\ubc29\ubd80\uc7a5\uc744 \ub9e1\uace0 \uc788\ub358 \ud391\ub354\ud654\uc774\uac00 \ub300\uc57d\uc9c4 \uc6b4\ub3d9\uc744 \uc804\uac1c\ud55c \ub9c8\uc624\ub97c \ube44\ud310\ud558\uc9c0\ub9cc, \ub3c4\ub9ac\uc5b4 \ud391\ub354\ud654\uc774 \uc790\uc2e0\uc774 \ud574\uc784\ub2f9\ud558\uace0 \ub9d0\uc558\ub2e4. \ub300\uc57d\uc9c4 \uc6b4\ub3d9\uc758 \uc2e4\ud328\ub85c \uc544\uc0ac\uc790\uc640 \ucc98\ud615\ub2f9\ud55c \uc218\ub294 \uc57d 2\ucc9c 500\ub9cc \uba85\uc744 \ud5e4\uc544\ub838\ub2e4. 1959\ub144 \ub300\uc57d\uc9c4 \uc6b4\ub3d9\uc5d0 \ub300\ud55c \ube44\ub09c \uc5ec\ub860\uc774 \ud655\ub300\ub420 \uc870\uc9d0\uc744 \ubcf4\uc774\uc790 \ub9c8\uc624\uca4c\ub465\uc740 \uad6d\uac00\uc8fc\uc11d\uc9c1\uc744 \uc0ac\uc784\ud558\uc600\ub2e4. \uadf8\ub7ec\ub098 \ub9c8\uc624\uc758 \ub4a4\ub97c \uc774\uc5b4 \uad6d\uac00\uc8fc\uc11d\uc5d0 \ucde8\uc784\ud55c \ub958\uc0ac\uc624\uce58\ub294 \ub300\uc57d\uc9c4 \uc6b4\ub3d9\uc744 \uac15\ud558\uac8c \ube44\ud310\ud558\uc600\uace0, \ub369\uc0e4\uc624\ud551\uacfc \ud568\uaed8 \uacbd\uc81c\uac1c\ud601\uc744 \uc8fc\ub3c4\ud558\uac8c \ub418\uc5c8\ub2e4.", "answer": "\ucc38\uc0c8", "sentence": "\u300a\ub9ac\ub354\uc2a4 \ub2e4\uc774\uc81c\uc2a4\ud2b8\u300b\uc5d0 \ub530\ub974\uba74 1958\ub144\uc5d0 \ub9c8\uc624\ub294 \ucc38\uc0c8 \uac00 \ubcbc\ub97c \uba39\ub294\ubaa8\uc2b5\uc744 \ubcf4\uace0 \ubd84\ub178\ud558\uc600\ub2e4.", "paragraph_sentence": " \u300a\ub9ac\ub354\uc2a4 \ub2e4\uc774\uc81c\uc2a4\ud2b8\u300b\uc5d0 \ub530\ub974\uba74 1958\ub144\uc5d0 \ub9c8\uc624\ub294 \ucc38\uc0c8 \uac00 \ubcbc\ub97c \uba39\ub294\ubaa8\uc2b5\uc744 \ubcf4\uace0 \ubd84\ub178\ud558\uc600\ub2e4. \ucc38\uc0c8\ub97c \uac00\ub9ac\ud0a4\uba70 \uc0ac\ub78c\uc774 \uba39\ub294 \uace1\uc2dd\uc744 \uc0c8\uac00 \uba39\uace0\uc788\ub2e4\ub2c8! \u201c\uc800 \uc0c8\ub294 \ud574\ub85c\uc6b4 \uc0c8\ub2e4. \u201d\ub77c\uace0 \ub9d0\ud588\ub294\ub370, \uc774 \ub9d0 \ub54c\ubb38\uc5d0 \uc911\uad6d\uc5d0\ub294 \ucc38\uc0c8\uc7a1\uc774 \uad11\ud48d\uc774 \ubd88\uc5b4 1959\ub144\uc5d0\ub294 \uc81c\uc0ac\ud574 \uc6b4\ub3d9\uc73c\ub85c \uc778\ud574 \uc911\uad6d \ucc38\uc0c8\uc758 \uc528\uac00 \ub9d0\ub790\ub2e4. \uace4\ucda9\uc744 \uba39\uc774\ub85c \ud558\ub294 \ucc38\uc0c8\uac00 \uc5c6\uc5b4\uc9c0\uba74\uc11c \uac01\uc885 \ud574\ucda9\uc774 \ucc3d\uad90\ud558\uc5ec \ub300\ud749\ub144\uc758 \ubc1c\uc0dd\uc73c\ub85c 4\ucc9c\ub9cc \uba85\uc774 \uad76\uc5b4\uc8fd\uc5c8\ub2e4. 1959\ub144\uc5d0 \uc5f4\ub9b0 \ub8e8\uc0b0 \ud68c\uc758\uc5d0\uc11c \uad6d\ubc29\ubd80\uc7a5\uc744 \ub9e1\uace0 \uc788\ub358 \ud391\ub354\ud654\uc774\uac00 \ub300\uc57d\uc9c4 \uc6b4\ub3d9\uc744 \uc804\uac1c\ud55c \ub9c8\uc624\ub97c \ube44\ud310\ud558\uc9c0\ub9cc, \ub3c4\ub9ac\uc5b4 \ud391\ub354\ud654\uc774 \uc790\uc2e0\uc774 \ud574\uc784\ub2f9\ud558\uace0 \ub9d0\uc558\ub2e4. \ub300\uc57d\uc9c4 \uc6b4\ub3d9\uc758 \uc2e4\ud328\ub85c \uc544\uc0ac\uc790\uc640 \ucc98\ud615\ub2f9\ud55c \uc218\ub294 \uc57d 2\ucc9c 500\ub9cc \uba85\uc744 \ud5e4\uc544\ub838\ub2e4. 1959\ub144 \ub300\uc57d\uc9c4 \uc6b4\ub3d9\uc5d0 \ub300\ud55c \ube44\ub09c \uc5ec\ub860\uc774 \ud655\ub300\ub420 \uc870\uc9d0\uc744 \ubcf4\uc774\uc790 \ub9c8\uc624\uca4c\ub465\uc740 \uad6d\uac00\uc8fc\uc11d\uc9c1\uc744 \uc0ac\uc784\ud558\uc600\ub2e4. \uadf8\ub7ec\ub098 \ub9c8\uc624\uc758 \ub4a4\ub97c \uc774\uc5b4 \uad6d\uac00\uc8fc\uc11d\uc5d0 \ucde8\uc784\ud55c \ub958\uc0ac\uc624\uce58\ub294 \ub300\uc57d\uc9c4 \uc6b4\ub3d9\uc744 \uac15\ud558\uac8c \ube44\ud310\ud558\uc600\uace0, \ub369\uc0e4\uc624\ud551\uacfc \ud568\uaed8 \uacbd\uc81c\uac1c\ud601\uc744 \uc8fc\ub3c4\ud558\uac8c \ub418\uc5c8\ub2e4.", "paragraph_answer": "\u300a\ub9ac\ub354\uc2a4 \ub2e4\uc774\uc81c\uc2a4\ud2b8\u300b\uc5d0 \ub530\ub974\uba74 1958\ub144\uc5d0 \ub9c8\uc624\ub294 \ucc38\uc0c8 \uac00 \ubcbc\ub97c \uba39\ub294\ubaa8\uc2b5\uc744 \ubcf4\uace0 \ubd84\ub178\ud558\uc600\ub2e4. \ucc38\uc0c8\ub97c \uac00\ub9ac\ud0a4\uba70 \uc0ac\ub78c\uc774 \uba39\ub294 \uace1\uc2dd\uc744 \uc0c8\uac00 \uba39\uace0\uc788\ub2e4\ub2c8! \u201c\uc800 \uc0c8\ub294 \ud574\ub85c\uc6b4 \uc0c8\ub2e4.\u201d\ub77c\uace0 \ub9d0\ud588\ub294\ub370, \uc774 \ub9d0 \ub54c\ubb38\uc5d0 \uc911\uad6d\uc5d0\ub294 \ucc38\uc0c8\uc7a1\uc774 \uad11\ud48d\uc774 \ubd88\uc5b4 1959\ub144\uc5d0\ub294 \uc81c\uc0ac\ud574 \uc6b4\ub3d9\uc73c\ub85c \uc778\ud574 \uc911\uad6d \ucc38\uc0c8\uc758 \uc528\uac00 \ub9d0\ub790\ub2e4. \uace4\ucda9\uc744 \uba39\uc774\ub85c \ud558\ub294 \ucc38\uc0c8\uac00 \uc5c6\uc5b4\uc9c0\uba74\uc11c \uac01\uc885 \ud574\ucda9\uc774 \ucc3d\uad90\ud558\uc5ec \ub300\ud749\ub144\uc758 \ubc1c\uc0dd\uc73c\ub85c 4\ucc9c\ub9cc \uba85\uc774 \uad76\uc5b4\uc8fd\uc5c8\ub2e4. 1959\ub144\uc5d0 \uc5f4\ub9b0 \ub8e8\uc0b0 \ud68c\uc758\uc5d0\uc11c \uad6d\ubc29\ubd80\uc7a5\uc744 \ub9e1\uace0 \uc788\ub358 \ud391\ub354\ud654\uc774\uac00 \ub300\uc57d\uc9c4 \uc6b4\ub3d9\uc744 \uc804\uac1c\ud55c \ub9c8\uc624\ub97c \ube44\ud310\ud558\uc9c0\ub9cc, \ub3c4\ub9ac\uc5b4 \ud391\ub354\ud654\uc774 \uc790\uc2e0\uc774 \ud574\uc784\ub2f9\ud558\uace0 \ub9d0\uc558\ub2e4. \ub300\uc57d\uc9c4 \uc6b4\ub3d9\uc758 \uc2e4\ud328\ub85c \uc544\uc0ac\uc790\uc640 \ucc98\ud615\ub2f9\ud55c \uc218\ub294 \uc57d 2\ucc9c 500\ub9cc \uba85\uc744 \ud5e4\uc544\ub838\ub2e4. 1959\ub144 \ub300\uc57d\uc9c4 \uc6b4\ub3d9\uc5d0 \ub300\ud55c \ube44\ub09c \uc5ec\ub860\uc774 \ud655\ub300\ub420 \uc870\uc9d0\uc744 \ubcf4\uc774\uc790 \ub9c8\uc624\uca4c\ub465\uc740 \uad6d\uac00\uc8fc\uc11d\uc9c1\uc744 \uc0ac\uc784\ud558\uc600\ub2e4. \uadf8\ub7ec\ub098 \ub9c8\uc624\uc758 \ub4a4\ub97c \uc774\uc5b4 \uad6d\uac00\uc8fc\uc11d\uc5d0 \ucde8\uc784\ud55c \ub958\uc0ac\uc624\uce58\ub294 \ub300\uc57d\uc9c4 \uc6b4\ub3d9\uc744 \uac15\ud558\uac8c \ube44\ud310\ud558\uc600\uace0, \ub369\uc0e4\uc624\ud551\uacfc \ud568\uaed8 \uacbd\uc81c\uac1c\ud601\uc744 \uc8fc\ub3c4\ud558\uac8c \ub418\uc5c8\ub2e4.", "sentence_answer": "\u300a\ub9ac\ub354\uc2a4 \ub2e4\uc774\uc81c\uc2a4\ud2b8\u300b\uc5d0 \ub530\ub974\uba74 1958\ub144\uc5d0 \ub9c8\uc624\ub294 \ucc38\uc0c8 \uac00 \ubcbc\ub97c \uba39\ub294\ubaa8\uc2b5\uc744 \ubcf4\uace0 \ubd84\ub178\ud558\uc600\ub2e4.", "paragraph_id": "6499916-9-1", "paragraph_question": "question: 1959\ub144 \uc81c\uc0ac\ud574\uc6b4\ub3d9\uc73c\ub85c \uba78\uc885 \uc9c1\uc804\uc5d0 \uc774\ub974\ub978 \uc0c8 \uc774\ub984\uc740 \ubb34\uc5c7\uc778\uac00?, context: \u300a\ub9ac\ub354\uc2a4 \ub2e4\uc774\uc81c\uc2a4\ud2b8\u300b\uc5d0 \ub530\ub974\uba74 1958\ub144\uc5d0 \ub9c8\uc624\ub294 \ucc38\uc0c8\uac00 \ubcbc\ub97c \uba39\ub294\ubaa8\uc2b5\uc744 \ubcf4\uace0 \ubd84\ub178\ud558\uc600\ub2e4. \ucc38\uc0c8\ub97c \uac00\ub9ac\ud0a4\uba70 \uc0ac\ub78c\uc774 \uba39\ub294 \uace1\uc2dd\uc744 \uc0c8\uac00 \uba39\uace0\uc788\ub2e4\ub2c8! \u201c\uc800 \uc0c8\ub294 \ud574\ub85c\uc6b4 \uc0c8\ub2e4.\u201d\ub77c\uace0 \ub9d0\ud588\ub294\ub370, \uc774 \ub9d0 \ub54c\ubb38\uc5d0 \uc911\uad6d\uc5d0\ub294 \ucc38\uc0c8\uc7a1\uc774 \uad11\ud48d\uc774 \ubd88\uc5b4 1959\ub144\uc5d0\ub294 \uc81c\uc0ac\ud574 \uc6b4\ub3d9\uc73c\ub85c \uc778\ud574 \uc911\uad6d \ucc38\uc0c8\uc758 \uc528\uac00 \ub9d0\ub790\ub2e4. \uace4\ucda9\uc744 \uba39\uc774\ub85c \ud558\ub294 \ucc38\uc0c8\uac00 \uc5c6\uc5b4\uc9c0\uba74\uc11c \uac01\uc885 \ud574\ucda9\uc774 \ucc3d\uad90\ud558\uc5ec \ub300\ud749\ub144\uc758 \ubc1c\uc0dd\uc73c\ub85c 4\ucc9c\ub9cc \uba85\uc774 \uad76\uc5b4\uc8fd\uc5c8\ub2e4. 1959\ub144\uc5d0 \uc5f4\ub9b0 \ub8e8\uc0b0 \ud68c\uc758\uc5d0\uc11c \uad6d\ubc29\ubd80\uc7a5\uc744 \ub9e1\uace0 \uc788\ub358 \ud391\ub354\ud654\uc774\uac00 \ub300\uc57d\uc9c4 \uc6b4\ub3d9\uc744 \uc804\uac1c\ud55c \ub9c8\uc624\ub97c \ube44\ud310\ud558\uc9c0\ub9cc, \ub3c4\ub9ac\uc5b4 \ud391\ub354\ud654\uc774 \uc790\uc2e0\uc774 \ud574\uc784\ub2f9\ud558\uace0 \ub9d0\uc558\ub2e4. \ub300\uc57d\uc9c4 \uc6b4\ub3d9\uc758 \uc2e4\ud328\ub85c \uc544\uc0ac\uc790\uc640 \ucc98\ud615\ub2f9\ud55c \uc218\ub294 \uc57d 2\ucc9c 500\ub9cc \uba85\uc744 \ud5e4\uc544\ub838\ub2e4. 1959\ub144 \ub300\uc57d\uc9c4 \uc6b4\ub3d9\uc5d0 \ub300\ud55c \ube44\ub09c \uc5ec\ub860\uc774 \ud655\ub300\ub420 \uc870\uc9d0\uc744 \ubcf4\uc774\uc790 \ub9c8\uc624\uca4c\ub465\uc740 \uad6d\uac00\uc8fc\uc11d\uc9c1\uc744 \uc0ac\uc784\ud558\uc600\ub2e4. \uadf8\ub7ec\ub098 \ub9c8\uc624\uc758 \ub4a4\ub97c \uc774\uc5b4 \uad6d\uac00\uc8fc\uc11d\uc5d0 \ucde8\uc784\ud55c \ub958\uc0ac\uc624\uce58\ub294 \ub300\uc57d\uc9c4 \uc6b4\ub3d9\uc744 \uac15\ud558\uac8c \ube44\ud310\ud558\uc600\uace0, \ub369\uc0e4\uc624\ud551\uacfc \ud568\uaed8 \uacbd\uc81c\uac1c\ud601\uc744 \uc8fc\ub3c4\ud558\uac8c \ub418\uc5c8\ub2e4."} {"question": "\uc219\uba85\uc5ec\ub300\uc758 \ud559\uacfc,\ud559\ubd80,\uc804\uacf5 \ud559\ud68c\uc7a5\uacfc \ubd80\ud559\ud68c\uc7a5\uc73c\ub85c \uad6c\uc131\ub418\ub294 \uae30\uad6c\ub294?", "paragraph": "\ucd1d\ud559\uc0dd\ud68c\ub294 \uc219\uba85\uc5ec\ub300\uc758 \uc7ac\ud559\uc0dd \ubaa8\uc784\uc774\uba70 \ud559\uc0dd\ucd1d\ud68c, \ub300\uc758\uc6d0\ud68c, \ucd1d\ud559\uc0dd\ud68c, \uc6b4\uc601\uc704\uc6d0\ud68c, \ub2e8\uacfc \ub300\ud559 \ud559\uc0dd\ud68c, \ud559\uacfc\u2219\ud559\ubd80\u2219\uc804\uacf5 \ud559\uc0dd\ud68c, \uae30\ud0c0 \ud559\uc0dd \ubb38\ud654 \ud65c\ub3d9 \ub2e8\uccb4\ub85c \uad6c\uc131\ub41c\ub2e4. \ud559\uc0dd\ucd1d\ud68c\ub294 \ucd1d\ud559\uc0dd\ud68c\uc5d0 \uad00\ub828\ub41c \uc911\ub300\ud55c \uc0ac\ud56d\uc744 \uc758\uacb0\ud558\ub294 \uae30\uad6c\ub85c\uc11c \ub300\uc758\uc6d0\ud68c\uc758 \uc694\uad6c \ub610\ub294 \ucd1d\ud559\uc0dd\ud68c \ud68c\uc6d0 10\ubd84\uc758 1 \uc774\uc0c1\uc758 \uc694\uad6c\uac00 \uc788\uc744 \ub54c \ucd1d\ud559\uc0dd\ud68c\uc7a5\uc774 \uc18c\uc9d1\ud55c\ub2e4. \ub300\uc758\uc6d0\ud68c\ub294 \uc219\uba85\uc5ec\ub300\uc758 \ud559\uacfc\u2219\ud559\ubd80\u2219\uc804\uacf5 \ud559\ud68c\uc7a5\uacfc \ubd80\ud559\ud68c\uc7a5\ub85c \uad6c\uc131\ub418\ub294 \uae30\uad6c\ub85c\uc11c \ud559\uc0dd\ucd1d\ud68c\uc758 \uc18c\uc9d1\uc744 \uc694\uad6c\ud560 \uad8c\ud55c \ubfd0\ub9cc \uc544\ub098\ub77c \ucd1d\ud559\uc0dd\ud68c \ud68c\uce59\uc758 \uac1c\uc815\uacfc \ubc1c\uc758\ub97c \ud655\uc815\ud560 \uad8c\ud55c, \ucd1d\ud559\uc0dd\ud68c\uc758 \uc5c5\ubb34\ub97c \uac10\uc0ac\ud560 \uad8c\ud55c, \ucd1d\ud559\uc0dd\ud68c\uc7a5\ub2e8\uc5d0 \ub300\ud55c \ubd88\uc2e0\uc784\uc744 \uacb0\uc758\ud560 \uad8c\ud55c \ub4f1\uc744 \uac00\uc9c4\ub2e4. \ucd1d\ud559\uc0dd\ud68c\uc7a5\uc740 \ucd1d\ud559\uc0dd\ud68c\ub97c \ub300\ud45c\ud558\uba70 \uc6b4\uc601\uc704\uc6d0\ud68c\uc640 \uc9d1\ud589\ubd80\uc758 \uc7a5\uc774 \ub41c\ub2e4. \ucd1d\ud559\uc0dd\ud68c\uc7a5\uc740 \ucd1d\ud559\uc0dd\ud68c\uc758 \uc0ac\uc5c5 \uacc4\ud68d\uc744 \uc218\ub9bd\ud558\uace0 \ud3b8\uc131\ub41c \uc608\uc0b0\uacfc \uacb0\uc0b0\uc744 \ub300\uc758\uc6d0\ud68c\uc5d0 \uc81c\ucd9c\ud55c\ub2e4. \ub610\ud55c \ucd1d\ud559\uc0dd\ud68c\uc5d0 \uc911\ub300\ud55c \uc601\ud5a5\uc744 \ubbf8\uce58\ub294 \uc0ac\ud56d\uc5d0 \uad00\ud558\uc5ec \ud559\uad50\uc640 \uad00\uacc4 \uae30\uad00\uc5d0 \uc758\uacac\uc11c\ub97c \uc81c\ucd9c\ud560 \uc218 \uc788\ub2e4. \uc6b4\uc601 \uc704\uc6d0\ud68c\ub294 \ucd1d\ud559\uc0dd\ud68c\uc758 \uc2ec\uc758, \uc790\ubb38 \uae30\uad6c\ub85c\uc11c \ucd1d\ud559\uc0dd\ud68c\uc7a5, \ubd80\ud559\uc0dd\ud68c\uc7a5, \ub2e8\uacfc \ub300\ud559 \ud559\uc0dd\ud68c\uc7a5, \ucd1d\ud559\uc0dd\ud68c \uc9d1\ud589\ubd80\uc758 \ubd80\uc7a5\uc73c\ub85c \uad6c\uc131\ub418\uba70, \ucd1d\ud559\uc0dd\ud68c\uac00 \uc6b4\uc601\ud558\ub294 \uc81c\ubc18 \uc0ac\uc5c5, \ucd1d\ud559\uc0dd\ud68c\uc7a5\uc774 \ub300\uc758\uc6d0\ud68c\uc5d0 \uc81c\ucd9c\ud560 \uc608\uc0b0\uacfc \uacb0\uc0b0, \ucd1d\ud559\uc0dd\ud68c\uc7a5\uc758 \ub300\uc758\uc6d0\ud68c\uc5d0 \ub300\ud55c \uc18c\uc9d1 \uc694\uad6c \ub4f1\uc744 \uc2ec\uc758, \uc790\ubb38\ud55c\ub2e4.", "answer": "\ub300\uc758\uc6d0\ud68c", "sentence": "\ucd1d\ud559\uc0dd\ud68c\ub294 \uc219\uba85\uc5ec\ub300\uc758 \uc7ac\ud559\uc0dd \ubaa8\uc784\uc774\uba70 \ud559\uc0dd\ucd1d\ud68c, \ub300\uc758\uc6d0\ud68c ,", "paragraph_sentence": " \ucd1d\ud559\uc0dd\ud68c\ub294 \uc219\uba85\uc5ec\ub300\uc758 \uc7ac\ud559\uc0dd \ubaa8\uc784\uc774\uba70 \ud559\uc0dd\ucd1d\ud68c, \ub300\uc758\uc6d0\ud68c , \ucd1d\ud559\uc0dd\ud68c, \uc6b4\uc601\uc704\uc6d0\ud68c, \ub2e8\uacfc \ub300\ud559 \ud559\uc0dd\ud68c, \ud559\uacfc\u2219\ud559\ubd80\u2219\uc804\uacf5 \ud559\uc0dd\ud68c, \uae30\ud0c0 \ud559\uc0dd \ubb38\ud654 \ud65c\ub3d9 \ub2e8\uccb4\ub85c \uad6c\uc131\ub41c\ub2e4. \ud559\uc0dd\ucd1d\ud68c\ub294 \ucd1d\ud559\uc0dd\ud68c\uc5d0 \uad00\ub828\ub41c \uc911\ub300\ud55c \uc0ac\ud56d\uc744 \uc758\uacb0\ud558\ub294 \uae30\uad6c\ub85c\uc11c \ub300\uc758\uc6d0\ud68c\uc758 \uc694\uad6c \ub610\ub294 \ucd1d\ud559\uc0dd\ud68c \ud68c\uc6d0 10\ubd84\uc758 1 \uc774\uc0c1\uc758 \uc694\uad6c\uac00 \uc788\uc744 \ub54c \ucd1d\ud559\uc0dd\ud68c\uc7a5\uc774 \uc18c\uc9d1\ud55c\ub2e4. \ub300\uc758\uc6d0\ud68c\ub294 \uc219\uba85\uc5ec\ub300\uc758 \ud559\uacfc\u2219\ud559\ubd80\u2219\uc804\uacf5 \ud559\ud68c\uc7a5\uacfc \ubd80\ud559\ud68c\uc7a5\ub85c \uad6c\uc131\ub418\ub294 \uae30\uad6c\ub85c\uc11c \ud559\uc0dd\ucd1d\ud68c\uc758 \uc18c\uc9d1\uc744 \uc694\uad6c\ud560 \uad8c\ud55c \ubfd0\ub9cc \uc544\ub098\ub77c \ucd1d\ud559\uc0dd\ud68c \ud68c\uce59\uc758 \uac1c\uc815\uacfc \ubc1c\uc758\ub97c \ud655\uc815\ud560 \uad8c\ud55c, \ucd1d\ud559\uc0dd\ud68c\uc758 \uc5c5\ubb34\ub97c \uac10\uc0ac\ud560 \uad8c\ud55c, \ucd1d\ud559\uc0dd\ud68c\uc7a5\ub2e8\uc5d0 \ub300\ud55c \ubd88\uc2e0\uc784\uc744 \uacb0\uc758\ud560 \uad8c\ud55c \ub4f1\uc744 \uac00\uc9c4\ub2e4. \ucd1d\ud559\uc0dd\ud68c\uc7a5\uc740 \ucd1d\ud559\uc0dd\ud68c\ub97c \ub300\ud45c\ud558\uba70 \uc6b4\uc601\uc704\uc6d0\ud68c\uc640 \uc9d1\ud589\ubd80\uc758 \uc7a5\uc774 \ub41c\ub2e4. \ucd1d\ud559\uc0dd\ud68c\uc7a5\uc740 \ucd1d\ud559\uc0dd\ud68c\uc758 \uc0ac\uc5c5 \uacc4\ud68d\uc744 \uc218\ub9bd\ud558\uace0 \ud3b8\uc131\ub41c \uc608\uc0b0\uacfc \uacb0\uc0b0\uc744 \ub300\uc758\uc6d0\ud68c\uc5d0 \uc81c\ucd9c\ud55c\ub2e4. \ub610\ud55c \ucd1d\ud559\uc0dd\ud68c\uc5d0 \uc911\ub300\ud55c \uc601\ud5a5\uc744 \ubbf8\uce58\ub294 \uc0ac\ud56d\uc5d0 \uad00\ud558\uc5ec \ud559\uad50\uc640 \uad00\uacc4 \uae30\uad00\uc5d0 \uc758\uacac\uc11c\ub97c \uc81c\ucd9c\ud560 \uc218 \uc788\ub2e4. \uc6b4\uc601 \uc704\uc6d0\ud68c\ub294 \ucd1d\ud559\uc0dd\ud68c\uc758 \uc2ec\uc758, \uc790\ubb38 \uae30\uad6c\ub85c\uc11c \ucd1d\ud559\uc0dd\ud68c\uc7a5, \ubd80\ud559\uc0dd\ud68c\uc7a5, \ub2e8\uacfc \ub300\ud559 \ud559\uc0dd\ud68c\uc7a5, \ucd1d\ud559\uc0dd\ud68c \uc9d1\ud589\ubd80\uc758 \ubd80\uc7a5\uc73c\ub85c \uad6c\uc131\ub418\uba70, \ucd1d\ud559\uc0dd\ud68c\uac00 \uc6b4\uc601\ud558\ub294 \uc81c\ubc18 \uc0ac\uc5c5, \ucd1d\ud559\uc0dd\ud68c\uc7a5\uc774 \ub300\uc758\uc6d0\ud68c\uc5d0 \uc81c\ucd9c\ud560 \uc608\uc0b0\uacfc \uacb0\uc0b0, \ucd1d\ud559\uc0dd\ud68c\uc7a5\uc758 \ub300\uc758\uc6d0\ud68c\uc5d0 \ub300\ud55c \uc18c\uc9d1 \uc694\uad6c \ub4f1\uc744 \uc2ec\uc758, \uc790\ubb38\ud55c\ub2e4.", "paragraph_answer": "\ucd1d\ud559\uc0dd\ud68c\ub294 \uc219\uba85\uc5ec\ub300\uc758 \uc7ac\ud559\uc0dd \ubaa8\uc784\uc774\uba70 \ud559\uc0dd\ucd1d\ud68c, \ub300\uc758\uc6d0\ud68c , \ucd1d\ud559\uc0dd\ud68c, \uc6b4\uc601\uc704\uc6d0\ud68c, \ub2e8\uacfc \ub300\ud559 \ud559\uc0dd\ud68c, \ud559\uacfc\u2219\ud559\ubd80\u2219\uc804\uacf5 \ud559\uc0dd\ud68c, \uae30\ud0c0 \ud559\uc0dd \ubb38\ud654 \ud65c\ub3d9 \ub2e8\uccb4\ub85c \uad6c\uc131\ub41c\ub2e4. \ud559\uc0dd\ucd1d\ud68c\ub294 \ucd1d\ud559\uc0dd\ud68c\uc5d0 \uad00\ub828\ub41c \uc911\ub300\ud55c \uc0ac\ud56d\uc744 \uc758\uacb0\ud558\ub294 \uae30\uad6c\ub85c\uc11c \ub300\uc758\uc6d0\ud68c\uc758 \uc694\uad6c \ub610\ub294 \ucd1d\ud559\uc0dd\ud68c \ud68c\uc6d0 10\ubd84\uc758 1 \uc774\uc0c1\uc758 \uc694\uad6c\uac00 \uc788\uc744 \ub54c \ucd1d\ud559\uc0dd\ud68c\uc7a5\uc774 \uc18c\uc9d1\ud55c\ub2e4. \ub300\uc758\uc6d0\ud68c\ub294 \uc219\uba85\uc5ec\ub300\uc758 \ud559\uacfc\u2219\ud559\ubd80\u2219\uc804\uacf5 \ud559\ud68c\uc7a5\uacfc \ubd80\ud559\ud68c\uc7a5\ub85c \uad6c\uc131\ub418\ub294 \uae30\uad6c\ub85c\uc11c \ud559\uc0dd\ucd1d\ud68c\uc758 \uc18c\uc9d1\uc744 \uc694\uad6c\ud560 \uad8c\ud55c \ubfd0\ub9cc \uc544\ub098\ub77c \ucd1d\ud559\uc0dd\ud68c \ud68c\uce59\uc758 \uac1c\uc815\uacfc \ubc1c\uc758\ub97c \ud655\uc815\ud560 \uad8c\ud55c, \ucd1d\ud559\uc0dd\ud68c\uc758 \uc5c5\ubb34\ub97c \uac10\uc0ac\ud560 \uad8c\ud55c, \ucd1d\ud559\uc0dd\ud68c\uc7a5\ub2e8\uc5d0 \ub300\ud55c \ubd88\uc2e0\uc784\uc744 \uacb0\uc758\ud560 \uad8c\ud55c \ub4f1\uc744 \uac00\uc9c4\ub2e4. \ucd1d\ud559\uc0dd\ud68c\uc7a5\uc740 \ucd1d\ud559\uc0dd\ud68c\ub97c \ub300\ud45c\ud558\uba70 \uc6b4\uc601\uc704\uc6d0\ud68c\uc640 \uc9d1\ud589\ubd80\uc758 \uc7a5\uc774 \ub41c\ub2e4. \ucd1d\ud559\uc0dd\ud68c\uc7a5\uc740 \ucd1d\ud559\uc0dd\ud68c\uc758 \uc0ac\uc5c5 \uacc4\ud68d\uc744 \uc218\ub9bd\ud558\uace0 \ud3b8\uc131\ub41c \uc608\uc0b0\uacfc \uacb0\uc0b0\uc744 \ub300\uc758\uc6d0\ud68c\uc5d0 \uc81c\ucd9c\ud55c\ub2e4. \ub610\ud55c \ucd1d\ud559\uc0dd\ud68c\uc5d0 \uc911\ub300\ud55c \uc601\ud5a5\uc744 \ubbf8\uce58\ub294 \uc0ac\ud56d\uc5d0 \uad00\ud558\uc5ec \ud559\uad50\uc640 \uad00\uacc4 \uae30\uad00\uc5d0 \uc758\uacac\uc11c\ub97c \uc81c\ucd9c\ud560 \uc218 \uc788\ub2e4. \uc6b4\uc601 \uc704\uc6d0\ud68c\ub294 \ucd1d\ud559\uc0dd\ud68c\uc758 \uc2ec\uc758, \uc790\ubb38 \uae30\uad6c\ub85c\uc11c \ucd1d\ud559\uc0dd\ud68c\uc7a5, \ubd80\ud559\uc0dd\ud68c\uc7a5, \ub2e8\uacfc \ub300\ud559 \ud559\uc0dd\ud68c\uc7a5, \ucd1d\ud559\uc0dd\ud68c \uc9d1\ud589\ubd80\uc758 \ubd80\uc7a5\uc73c\ub85c \uad6c\uc131\ub418\uba70, \ucd1d\ud559\uc0dd\ud68c\uac00 \uc6b4\uc601\ud558\ub294 \uc81c\ubc18 \uc0ac\uc5c5, \ucd1d\ud559\uc0dd\ud68c\uc7a5\uc774 \ub300\uc758\uc6d0\ud68c\uc5d0 \uc81c\ucd9c\ud560 \uc608\uc0b0\uacfc \uacb0\uc0b0, \ucd1d\ud559\uc0dd\ud68c\uc7a5\uc758 \ub300\uc758\uc6d0\ud68c\uc5d0 \ub300\ud55c \uc18c\uc9d1 \uc694\uad6c \ub4f1\uc744 \uc2ec\uc758, \uc790\ubb38\ud55c\ub2e4.", "sentence_answer": "\ucd1d\ud559\uc0dd\ud68c\ub294 \uc219\uba85\uc5ec\ub300\uc758 \uc7ac\ud559\uc0dd \ubaa8\uc784\uc774\uba70 \ud559\uc0dd\ucd1d\ud68c, \ub300\uc758\uc6d0\ud68c ,", "paragraph_id": "6537087-5-0", "paragraph_question": "question: \uc219\uba85\uc5ec\ub300\uc758 \ud559\uacfc,\ud559\ubd80,\uc804\uacf5 \ud559\ud68c\uc7a5\uacfc \ubd80\ud559\ud68c\uc7a5\uc73c\ub85c \uad6c\uc131\ub418\ub294 \uae30\uad6c\ub294?, context: \ucd1d\ud559\uc0dd\ud68c\ub294 \uc219\uba85\uc5ec\ub300\uc758 \uc7ac\ud559\uc0dd \ubaa8\uc784\uc774\uba70 \ud559\uc0dd\ucd1d\ud68c, \ub300\uc758\uc6d0\ud68c, \ucd1d\ud559\uc0dd\ud68c, \uc6b4\uc601\uc704\uc6d0\ud68c, \ub2e8\uacfc \ub300\ud559 \ud559\uc0dd\ud68c, \ud559\uacfc\u2219\ud559\ubd80\u2219\uc804\uacf5 \ud559\uc0dd\ud68c, \uae30\ud0c0 \ud559\uc0dd \ubb38\ud654 \ud65c\ub3d9 \ub2e8\uccb4\ub85c \uad6c\uc131\ub41c\ub2e4. \ud559\uc0dd\ucd1d\ud68c\ub294 \ucd1d\ud559\uc0dd\ud68c\uc5d0 \uad00\ub828\ub41c \uc911\ub300\ud55c \uc0ac\ud56d\uc744 \uc758\uacb0\ud558\ub294 \uae30\uad6c\ub85c\uc11c \ub300\uc758\uc6d0\ud68c\uc758 \uc694\uad6c \ub610\ub294 \ucd1d\ud559\uc0dd\ud68c \ud68c\uc6d0 10\ubd84\uc758 1 \uc774\uc0c1\uc758 \uc694\uad6c\uac00 \uc788\uc744 \ub54c \ucd1d\ud559\uc0dd\ud68c\uc7a5\uc774 \uc18c\uc9d1\ud55c\ub2e4. \ub300\uc758\uc6d0\ud68c\ub294 \uc219\uba85\uc5ec\ub300\uc758 \ud559\uacfc\u2219\ud559\ubd80\u2219\uc804\uacf5 \ud559\ud68c\uc7a5\uacfc \ubd80\ud559\ud68c\uc7a5\ub85c \uad6c\uc131\ub418\ub294 \uae30\uad6c\ub85c\uc11c \ud559\uc0dd\ucd1d\ud68c\uc758 \uc18c\uc9d1\uc744 \uc694\uad6c\ud560 \uad8c\ud55c \ubfd0\ub9cc \uc544\ub098\ub77c \ucd1d\ud559\uc0dd\ud68c \ud68c\uce59\uc758 \uac1c\uc815\uacfc \ubc1c\uc758\ub97c \ud655\uc815\ud560 \uad8c\ud55c, \ucd1d\ud559\uc0dd\ud68c\uc758 \uc5c5\ubb34\ub97c \uac10\uc0ac\ud560 \uad8c\ud55c, \ucd1d\ud559\uc0dd\ud68c\uc7a5\ub2e8\uc5d0 \ub300\ud55c \ubd88\uc2e0\uc784\uc744 \uacb0\uc758\ud560 \uad8c\ud55c \ub4f1\uc744 \uac00\uc9c4\ub2e4. \ucd1d\ud559\uc0dd\ud68c\uc7a5\uc740 \ucd1d\ud559\uc0dd\ud68c\ub97c \ub300\ud45c\ud558\uba70 \uc6b4\uc601\uc704\uc6d0\ud68c\uc640 \uc9d1\ud589\ubd80\uc758 \uc7a5\uc774 \ub41c\ub2e4. \ucd1d\ud559\uc0dd\ud68c\uc7a5\uc740 \ucd1d\ud559\uc0dd\ud68c\uc758 \uc0ac\uc5c5 \uacc4\ud68d\uc744 \uc218\ub9bd\ud558\uace0 \ud3b8\uc131\ub41c \uc608\uc0b0\uacfc \uacb0\uc0b0\uc744 \ub300\uc758\uc6d0\ud68c\uc5d0 \uc81c\ucd9c\ud55c\ub2e4. \ub610\ud55c \ucd1d\ud559\uc0dd\ud68c\uc5d0 \uc911\ub300\ud55c \uc601\ud5a5\uc744 \ubbf8\uce58\ub294 \uc0ac\ud56d\uc5d0 \uad00\ud558\uc5ec \ud559\uad50\uc640 \uad00\uacc4 \uae30\uad00\uc5d0 \uc758\uacac\uc11c\ub97c \uc81c\ucd9c\ud560 \uc218 \uc788\ub2e4. \uc6b4\uc601 \uc704\uc6d0\ud68c\ub294 \ucd1d\ud559\uc0dd\ud68c\uc758 \uc2ec\uc758, \uc790\ubb38 \uae30\uad6c\ub85c\uc11c \ucd1d\ud559\uc0dd\ud68c\uc7a5, \ubd80\ud559\uc0dd\ud68c\uc7a5, \ub2e8\uacfc \ub300\ud559 \ud559\uc0dd\ud68c\uc7a5, \ucd1d\ud559\uc0dd\ud68c \uc9d1\ud589\ubd80\uc758 \ubd80\uc7a5\uc73c\ub85c \uad6c\uc131\ub418\uba70, \ucd1d\ud559\uc0dd\ud68c\uac00 \uc6b4\uc601\ud558\ub294 \uc81c\ubc18 \uc0ac\uc5c5, \ucd1d\ud559\uc0dd\ud68c\uc7a5\uc774 \ub300\uc758\uc6d0\ud68c\uc5d0 \uc81c\ucd9c\ud560 \uc608\uc0b0\uacfc \uacb0\uc0b0, \ucd1d\ud559\uc0dd\ud68c\uc7a5\uc758 \ub300\uc758\uc6d0\ud68c\uc5d0 \ub300\ud55c \uc18c\uc9d1 \uc694\uad6c \ub4f1\uc744 \uc2ec\uc758, \uc790\ubb38\ud55c\ub2e4."} {"question": "\ub0b4\uac00 \ud669\uc81c\ub77c\uba74 \ucc45\uc774 \ubc1c\uac04\ub41c \ub144\ub3c4\ub294?", "paragraph": "1895\ub144, \uc61b \ub3c5\uc77c\uc81c\uad6d\uc758 \uc758\ud68c \uc774\uc804\uc5d0 \ud589\ud574\uc9c4 \uc5f0\uc124\uc5d0\uc11c \ubbfc\uc871\uc8fc\uc758 \uc9c0\ub3c4\uc790 \ud5e4\ub974\ub9cc \uc54c\ubc14\ub974\ud2b8(Hermann Ahlwardt)\ub294 \uc720\ub300\uc778\uc744 \u201c\uc721\uc2dd \ub3d9\ubb3c\u201d\uacfc \u201c\ucf5c\ub808\ub77c\uade0\u201d\uc774\ub77c \uce6d\ud558\uba70 \ub3c5\uc77c\uad6d\ubbfc\uc758 \uc774\uc775\uc744 \uc704\ud574 \ucc98\ud615\ud574\uc57c \ud55c\ub2e4\uace0 \ub9d0\ud588\ub2e4. 1912\ub144 \ubc1c\uac04 \ub41c \uadf8\uc758 \ubca0\uc2a4\ud2b8\uc140\ub7ec \"\ub0b4\uac00 \ud669\uc81c\ub77c\uba74 Wenn ich der Kaiser w\u00e4re\"\uc5d0\uc11c \ud558\uc778\ub9ac\ud788 \uadf8\ub8f9\uc758 \ubbfc\uc871\uc8fc\uc758 \uadf8\ub8f9 \uc9c0\ub3c4\uc790 \uc54c \ub3c4\uc774 \ubc84\ubc34\uc740 \ubaa8\ub4e0 \ub3c5\uc77c \uc720\ub300\uc778\ub4e4\uc5d0\uac8c\uc11c \ub3c5\uc77c\uc5b4 \uc2dc\ubbfc\uad8c\uc744 \ubc15\ud0c8\ud558\uace0 \uc678\uad6d\uc778 \ubc95\uc5d0 \ub530\ub77c \uc774\ub4e4\uc758 \uad8c\ub9ac\ub97c \ucd95\uc18c\uc2dc\ud0ac \uac83\uc744 \ucd09\uad6c\ud588\ub2e4. \uc774 \uadf8\ub8f9\uc740 \uc720\ub300\uc778\ub4e4\uc5d0\uac8c \uc18c\uc720\uc9c0\ub97c \uac16\uac70\ub098 \uacf5\uc9c1\uc5d0 \uc9c4\ucd9c\ud558\uac70\ub098 \ud639\uc740 \uc5b8\ub860\uc5d0 \ucc38\uc5ec\ud558\uac70\ub098 \uc740\ud589\uc5c5\ubb34 \uadf8\ub9ac\uace0 \uc790\uc720\ub85c\uc6b4 \uc9c1\uc5c5\ud65c\ub3d9 \ub4f1\uc774 \ubaa8\ub450 \uae08\uc9c0\ub418\uc5b4 \ub3c5\uc77c\uc758 \ubaa8\ub4e0 \uc0b6\uc5d0\uc11c \uc81c\uc678\ub418\uc5b4\uc57c \ud560 \uac83\uc744 \ucd09\uad6c\ud588\ub2e4. \uc774 \uadf8\ub8f9\uc740 1871\ub144 \ub3c5\uc77c \uc81c\uad6d\uc774 \uc120\ud3ec\ub41c \uc774\ub798\ub85c \uc720\ub300\uc778\uc774\uc600\ub358 \uc801\uc774 \uc788\uac70\ub098 \uc801\uc5b4\ub3c4 \uc870\ubd80\ubaa8\ub2d8\uc774 \uc720\ub300\uc778\uc774\uc5c8\ub358 \ubaa8\ub4e0 \uc0ac\ub78c\uc744 \uc720\ub300\uc778\uc73c\ub85c \uba85\uba85\ud558\uc600\ub2e4.", "answer": "1912\ub144", "sentence": "1912\ub144 \ubc1c\uac04 \ub41c \uadf8\uc758 \ubca0\uc2a4\ud2b8\uc140\ub7ec \"\ub0b4\uac00 \ud669\uc81c\ub77c\uba74 Wenn ich der Kaiser w\u00e4re\"\uc5d0\uc11c \ud558\uc778\ub9ac\ud788 \uadf8\ub8f9\uc758 \ubbfc\uc871\uc8fc\uc758 \uadf8\ub8f9 \uc9c0\ub3c4\uc790 \uc54c \ub3c4\uc774 \ubc84\ubc34\uc740 \ubaa8\ub4e0 \ub3c5\uc77c \uc720\ub300\uc778\ub4e4\uc5d0\uac8c\uc11c \ub3c5\uc77c\uc5b4 \uc2dc\ubbfc\uad8c\uc744 \ubc15\ud0c8\ud558\uace0 \uc678\uad6d\uc778 \ubc95\uc5d0 \ub530\ub77c \uc774\ub4e4\uc758 \uad8c\ub9ac\ub97c \ucd95\uc18c\uc2dc\ud0ac \uac83\uc744 \ucd09\uad6c\ud588\ub2e4.", "paragraph_sentence": "1895\ub144, \uc61b \ub3c5\uc77c\uc81c\uad6d\uc758 \uc758\ud68c \uc774\uc804\uc5d0 \ud589\ud574\uc9c4 \uc5f0\uc124\uc5d0\uc11c \ubbfc\uc871\uc8fc\uc758 \uc9c0\ub3c4\uc790 \ud5e4\ub974\ub9cc \uc54c\ubc14\ub974\ud2b8(Hermann Ahlwardt)\ub294 \uc720\ub300\uc778\uc744 \u201c\uc721\uc2dd \ub3d9\ubb3c\u201d\uacfc \u201c\ucf5c\ub808\ub77c\uade0\u201d\uc774\ub77c \uce6d\ud558\uba70 \ub3c5\uc77c\uad6d\ubbfc\uc758 \uc774\uc775\uc744 \uc704\ud574 \ucc98\ud615\ud574\uc57c \ud55c\ub2e4\uace0 \ub9d0\ud588\ub2e4. 1912\ub144 \ubc1c\uac04 \ub41c \uadf8\uc758 \ubca0\uc2a4\ud2b8\uc140\ub7ec \"\ub0b4\uac00 \ud669\uc81c\ub77c\uba74 Wenn ich der Kaiser w\u00e4re\"\uc5d0\uc11c \ud558\uc778\ub9ac\ud788 \uadf8\ub8f9\uc758 \ubbfc\uc871\uc8fc\uc758 \uadf8\ub8f9 \uc9c0\ub3c4\uc790 \uc54c \ub3c4\uc774 \ubc84\ubc34\uc740 \ubaa8\ub4e0 \ub3c5\uc77c \uc720\ub300\uc778\ub4e4\uc5d0\uac8c\uc11c \ub3c5\uc77c\uc5b4 \uc2dc\ubbfc\uad8c\uc744 \ubc15\ud0c8\ud558\uace0 \uc678\uad6d\uc778 \ubc95\uc5d0 \ub530\ub77c \uc774\ub4e4\uc758 \uad8c\ub9ac\ub97c \ucd95\uc18c\uc2dc\ud0ac \uac83\uc744 \ucd09\uad6c\ud588\ub2e4. \uc774 \uadf8\ub8f9\uc740 \uc720\ub300\uc778\ub4e4\uc5d0\uac8c \uc18c\uc720\uc9c0\ub97c \uac16\uac70\ub098 \uacf5\uc9c1\uc5d0 \uc9c4\ucd9c\ud558\uac70\ub098 \ud639\uc740 \uc5b8\ub860\uc5d0 \ucc38\uc5ec\ud558\uac70\ub098 \uc740\ud589\uc5c5\ubb34 \uadf8\ub9ac\uace0 \uc790\uc720\ub85c\uc6b4 \uc9c1\uc5c5\ud65c\ub3d9 \ub4f1\uc774 \ubaa8\ub450 \uae08\uc9c0\ub418\uc5b4 \ub3c5\uc77c\uc758 \ubaa8\ub4e0 \uc0b6\uc5d0\uc11c \uc81c\uc678\ub418\uc5b4\uc57c \ud560 \uac83\uc744 \ucd09\uad6c\ud588\ub2e4. \uc774 \uadf8\ub8f9\uc740 1871\ub144 \ub3c5\uc77c \uc81c\uad6d\uc774 \uc120\ud3ec\ub41c \uc774\ub798\ub85c \uc720\ub300\uc778\uc774\uc600\ub358 \uc801\uc774 \uc788\uac70\ub098 \uc801\uc5b4\ub3c4 \uc870\ubd80\ubaa8\ub2d8\uc774 \uc720\ub300\uc778\uc774\uc5c8\ub358 \ubaa8\ub4e0 \uc0ac\ub78c\uc744 \uc720\ub300\uc778\uc73c\ub85c \uba85\uba85\ud558\uc600\ub2e4.", "paragraph_answer": "1895\ub144, \uc61b \ub3c5\uc77c\uc81c\uad6d\uc758 \uc758\ud68c \uc774\uc804\uc5d0 \ud589\ud574\uc9c4 \uc5f0\uc124\uc5d0\uc11c \ubbfc\uc871\uc8fc\uc758 \uc9c0\ub3c4\uc790 \ud5e4\ub974\ub9cc \uc54c\ubc14\ub974\ud2b8(Hermann Ahlwardt)\ub294 \uc720\ub300\uc778\uc744 \u201c\uc721\uc2dd \ub3d9\ubb3c\u201d\uacfc \u201c\ucf5c\ub808\ub77c\uade0\u201d\uc774\ub77c \uce6d\ud558\uba70 \ub3c5\uc77c\uad6d\ubbfc\uc758 \uc774\uc775\uc744 \uc704\ud574 \ucc98\ud615\ud574\uc57c \ud55c\ub2e4\uace0 \ub9d0\ud588\ub2e4. 1912\ub144 \ubc1c\uac04 \ub41c \uadf8\uc758 \ubca0\uc2a4\ud2b8\uc140\ub7ec \"\ub0b4\uac00 \ud669\uc81c\ub77c\uba74 Wenn ich der Kaiser w\u00e4re\"\uc5d0\uc11c \ud558\uc778\ub9ac\ud788 \uadf8\ub8f9\uc758 \ubbfc\uc871\uc8fc\uc758 \uadf8\ub8f9 \uc9c0\ub3c4\uc790 \uc54c \ub3c4\uc774 \ubc84\ubc34\uc740 \ubaa8\ub4e0 \ub3c5\uc77c \uc720\ub300\uc778\ub4e4\uc5d0\uac8c\uc11c \ub3c5\uc77c\uc5b4 \uc2dc\ubbfc\uad8c\uc744 \ubc15\ud0c8\ud558\uace0 \uc678\uad6d\uc778 \ubc95\uc5d0 \ub530\ub77c \uc774\ub4e4\uc758 \uad8c\ub9ac\ub97c \ucd95\uc18c\uc2dc\ud0ac \uac83\uc744 \ucd09\uad6c\ud588\ub2e4. \uc774 \uadf8\ub8f9\uc740 \uc720\ub300\uc778\ub4e4\uc5d0\uac8c \uc18c\uc720\uc9c0\ub97c \uac16\uac70\ub098 \uacf5\uc9c1\uc5d0 \uc9c4\ucd9c\ud558\uac70\ub098 \ud639\uc740 \uc5b8\ub860\uc5d0 \ucc38\uc5ec\ud558\uac70\ub098 \uc740\ud589\uc5c5\ubb34 \uadf8\ub9ac\uace0 \uc790\uc720\ub85c\uc6b4 \uc9c1\uc5c5\ud65c\ub3d9 \ub4f1\uc774 \ubaa8\ub450 \uae08\uc9c0\ub418\uc5b4 \ub3c5\uc77c\uc758 \ubaa8\ub4e0 \uc0b6\uc5d0\uc11c \uc81c\uc678\ub418\uc5b4\uc57c \ud560 \uac83\uc744 \ucd09\uad6c\ud588\ub2e4. \uc774 \uadf8\ub8f9\uc740 1871\ub144 \ub3c5\uc77c \uc81c\uad6d\uc774 \uc120\ud3ec\ub41c \uc774\ub798\ub85c \uc720\ub300\uc778\uc774\uc600\ub358 \uc801\uc774 \uc788\uac70\ub098 \uc801\uc5b4\ub3c4 \uc870\ubd80\ubaa8\ub2d8\uc774 \uc720\ub300\uc778\uc774\uc5c8\ub358 \ubaa8\ub4e0 \uc0ac\ub78c\uc744 \uc720\ub300\uc778\uc73c\ub85c \uba85\uba85\ud558\uc600\ub2e4.", "sentence_answer": " 1912\ub144 \ubc1c\uac04 \ub41c \uadf8\uc758 \ubca0\uc2a4\ud2b8\uc140\ub7ec \"\ub0b4\uac00 \ud669\uc81c\ub77c\uba74 Wenn ich der Kaiser w\u00e4re\"\uc5d0\uc11c \ud558\uc778\ub9ac\ud788 \uadf8\ub8f9\uc758 \ubbfc\uc871\uc8fc\uc758 \uadf8\ub8f9 \uc9c0\ub3c4\uc790 \uc54c \ub3c4\uc774 \ubc84\ubc34\uc740 \ubaa8\ub4e0 \ub3c5\uc77c \uc720\ub300\uc778\ub4e4\uc5d0\uac8c\uc11c \ub3c5\uc77c\uc5b4 \uc2dc\ubbfc\uad8c\uc744 \ubc15\ud0c8\ud558\uace0 \uc678\uad6d\uc778 \ubc95\uc5d0 \ub530\ub77c \uc774\ub4e4\uc758 \uad8c\ub9ac\ub97c \ucd95\uc18c\uc2dc\ud0ac \uac83\uc744 \ucd09\uad6c\ud588\ub2e4.", "paragraph_id": "6571969-3-1", "paragraph_question": "question: \ub0b4\uac00 \ud669\uc81c\ub77c\uba74 \ucc45\uc774 \ubc1c\uac04\ub41c \ub144\ub3c4\ub294?, context: 1895\ub144, \uc61b \ub3c5\uc77c\uc81c\uad6d\uc758 \uc758\ud68c \uc774\uc804\uc5d0 \ud589\ud574\uc9c4 \uc5f0\uc124\uc5d0\uc11c \ubbfc\uc871\uc8fc\uc758 \uc9c0\ub3c4\uc790 \ud5e4\ub974\ub9cc \uc54c\ubc14\ub974\ud2b8(Hermann Ahlwardt)\ub294 \uc720\ub300\uc778\uc744 \u201c\uc721\uc2dd \ub3d9\ubb3c\u201d\uacfc \u201c\ucf5c\ub808\ub77c\uade0\u201d\uc774\ub77c \uce6d\ud558\uba70 \ub3c5\uc77c\uad6d\ubbfc\uc758 \uc774\uc775\uc744 \uc704\ud574 \ucc98\ud615\ud574\uc57c \ud55c\ub2e4\uace0 \ub9d0\ud588\ub2e4. 1912\ub144 \ubc1c\uac04 \ub41c \uadf8\uc758 \ubca0\uc2a4\ud2b8\uc140\ub7ec \"\ub0b4\uac00 \ud669\uc81c\ub77c\uba74 Wenn ich der Kaiser w\u00e4re\"\uc5d0\uc11c \ud558\uc778\ub9ac\ud788 \uadf8\ub8f9\uc758 \ubbfc\uc871\uc8fc\uc758 \uadf8\ub8f9 \uc9c0\ub3c4\uc790 \uc54c \ub3c4\uc774 \ubc84\ubc34\uc740 \ubaa8\ub4e0 \ub3c5\uc77c \uc720\ub300\uc778\ub4e4\uc5d0\uac8c\uc11c \ub3c5\uc77c\uc5b4 \uc2dc\ubbfc\uad8c\uc744 \ubc15\ud0c8\ud558\uace0 \uc678\uad6d\uc778 \ubc95\uc5d0 \ub530\ub77c \uc774\ub4e4\uc758 \uad8c\ub9ac\ub97c \ucd95\uc18c\uc2dc\ud0ac \uac83\uc744 \ucd09\uad6c\ud588\ub2e4. \uc774 \uadf8\ub8f9\uc740 \uc720\ub300\uc778\ub4e4\uc5d0\uac8c \uc18c\uc720\uc9c0\ub97c \uac16\uac70\ub098 \uacf5\uc9c1\uc5d0 \uc9c4\ucd9c\ud558\uac70\ub098 \ud639\uc740 \uc5b8\ub860\uc5d0 \ucc38\uc5ec\ud558\uac70\ub098 \uc740\ud589\uc5c5\ubb34 \uadf8\ub9ac\uace0 \uc790\uc720\ub85c\uc6b4 \uc9c1\uc5c5\ud65c\ub3d9 \ub4f1\uc774 \ubaa8\ub450 \uae08\uc9c0\ub418\uc5b4 \ub3c5\uc77c\uc758 \ubaa8\ub4e0 \uc0b6\uc5d0\uc11c \uc81c\uc678\ub418\uc5b4\uc57c \ud560 \uac83\uc744 \ucd09\uad6c\ud588\ub2e4. \uc774 \uadf8\ub8f9\uc740 1871\ub144 \ub3c5\uc77c \uc81c\uad6d\uc774 \uc120\ud3ec\ub41c \uc774\ub798\ub85c \uc720\ub300\uc778\uc774\uc600\ub358 \uc801\uc774 \uc788\uac70\ub098 \uc801\uc5b4\ub3c4 \uc870\ubd80\ubaa8\ub2d8\uc774 \uc720\ub300\uc778\uc774\uc5c8\ub358 \ubaa8\ub4e0 \uc0ac\ub78c\uc744 \uc720\ub300\uc778\uc73c\ub85c \uba85\uba85\ud558\uc600\ub2e4."} {"question": "\ud0dc\uc591\uc758 \ud6c4\uc608\uac00 \ud0c4\ud0c4\ud55c \uc2a4\ud1a0\ub9ac\uc640 \uc778\ub958\uc560 \ubc0f \ucee4\ud50c\ub4e4\uc758 \uc0ac\ub791\uc774\ub77c\ub294 \uc694\uc18c\uac00 \uc801\uc808\ud788 \ubc30\ud569\ub418\uc5b4 \uc778\uae30 \uc788\ub2e4\uace0 \ud55c \uc0ac\ub78c\uc740?", "paragraph": "\uc601\uad6d BBC\ub294\u300a\ud0dc\uc591\uc758 \ud6c4\uc608\u300b\uac00 \ud55c\ub958\uc758 \uc815\uc810\uc744 \ucc0d\uc744 \uac83\uc774\ub77c \uc608\uc0c1\ud588\ub2e4. \uc720\uac74\uc2dd\uc740 \"\ud0c4\ud0c4\ud558\uba74\uc11c\ub3c4 \ube60\ub978 \uc2a4\ud1a0\ub9ac \uc804\uac1c\uc640 \uc601\ud654 \uac19\uc740 \uc9c8\uac10, \uc778\ub958\uc560\uc5d0\uc11c \ud53c\uc5b4\ub098\ub294 \ucee4\ud50c\ub4e4\uc758 \uc0ac\ub791\uc774 \uc801\uc808\ud788 \ubc30\ubd84\ub41c \uac83\uc774 \uc778\uae30\uc694\uc778\uc73c\ub85c \uaf3d\ud788\uace0 \uc788\ub2e4.\u201d\uace0 \uc804\ud588\ub2e4. \ub0a8\uad81\uc778\uc740 \ud3c9\ub860\uc5d0\uc11c \u300a\ud0dc\uc591\uc758 \ud6c4\uc608\u300b\uc758 \uc778\uae30\uc694\uc778\uc5d0 \ub300\ud574 \ub2e4\uc74c\uacfc \uac19\uc774 \ubd84\uc11d\ud588\ub2e4. \u300a\ud0dc\uc591\uc758 \ud6c4\uc608\u300b\ub294 \"\uc8fc\uc5b4\uc9c4 \uc2dc\uac04\uc5d0 \uadf9\uc801\uc778 \uc774\uc57c\uae30\ub97c \ubc30\uce58\ud558\ub294 \ud6a8\uc728\uc131\uc73c\ub85c \ud1b1\ub2c8\ubc14\ud034\ucc98\ub7fc \ub9de\ubb3c\ub824 \uc788\ub2e4. \uc774\uc57c\uae30\ud558\uace0\uc790 \ud558\ub294 \ubc14\uc640, \uadf8 \uc774\uc57c\uae30\ub97c \uc2dc\uac01\ud654\ud558\uc5ec \ubcf4\uc5ec\uc8fc\uace0\uc790 \ud558\ub294 \uc695\ub9dd\uc744 \uc5b4\ub514\uae4c\uc9c0 \uc2e4\ud604\ud560 \uc218 \uc788\ub290\ub0d0\ub294 \uc9c8\ubb38\uc5d0 \ub2f5\ud558\ub294, \uadf9\ub2e8\uc801\uc778 \ubc1c\uc804\uc758 \ud55c \uc608\ub2e4. \u300a\ud0dc\uc591\uc758 \ud6c4\uc608\u300b\uc5d0\uc11c \ub300\uc911\uc740 \"\ub2e4\uc774\ub0b4\ubbf9\ud55c \ud074\ub77c\uc774\ub9e5\uc2a4, \uadf8\ub9ac\uace0 \uae30\uc6d0\ud558\ub358 \uac83\uc774 \uc774\ub8e8\uc5b4\uc84c\uc744 \ub54c\uc758 \uce74\ud0c0\ub974\uc2dc\uc2a4\"\ub97c \ub290\ub07c\uac8c \ub41c\ub2e4. \u300a\ud0dc\uc591\uc758 \ud6c4\uc608\u300b\ub294 \"\uc774 \uc695\ub9dd\uc744 \uc2ed\ubd84 \ucda9\uc871\uc2dc\ud0a4\uae30 \uc704\ud574 \ud604\uc2e4\uc774\ub098 \uace0\uc99d\uc740 \uacfc\uac10\ud788 \ubb34\uc2dc\ud558\uace0, \uc77c\ubc18 \ub300\uc911\uc758 \ub208\ub192\uc774\uc5d0 \uc11c\uc11c, \uadf8\ub4e4\uc774 \ubcf4\uace0 \uc2f6\uc740 \uc7a5\uba74\uc774\ub77c\uba74 \ubb34\uc5c7\uc774\ub4e0 \ubcf4\uc5ec\uc900\ub2e4. (\uc911\ub7b5) \ub300\uc911\uc774 \ubcf4\uace0 \uc2f6\uc5b4 \ud558\ub294 \uac83\uc740 \ucd1d\ud0c4\uc774 \ubc30\uc5d0 \ubc15\ud614\ub2e4\uace0 \ubc14\uc774\ud0c8\uc744 \uc7ac\uba74\uc11c \ubcd1\uc6d0\uc73c\ub85c \uc2e0\uc18d\ud788 \ud6c4\uc1a1\ud558\ub294 \uc7a5\uba74\ub3c4 \uc544\ub2c8\uace0, \ub2e4\uc774\uc544\ubaac\ub4dc\ub294 \uc548 \ucc0d\ud614\uc9c0\ub9cc \ud544\ub984\uc744 \ub4e4\uace0 \uc640\uc11c \uc815\ud669\uc0c1 \uc758\uc2ec \uac00\ub2a5\ud560 \uc218 \uc788\ub2e4\uace0 \ub9dd\uc124\uc774\ub294 \uc7a5\uba74\ub3c4, \uccad\uc9c4\uae30\ub97c \uc548 \ub300\uace0 \ubc30\ub97c \uc5f4\uc5b4 \uc870\uc2ec\uc870\uc2ec \ucd09\uc9c4\ud574 \ud658\uc790\uc758 \ubc18\uc751\uc744 \uc0b4\ud53c\ub294 \uac83\ub3c4 \uc544\ub2c8\ub2e4. \ub300\uc911\uacfc TV \uc548\uc5d0\ub294 \uc9c1\uad00\uc801\uc778 \uc138\uc0c1\uc774 \uc788\uc744 \ubfd0, \uc544\ubb34\ub3c4 \uadf8\uac83\uc774 \ud604\uc2e4\uc801\uc778 \uacbd\uacc4\uc5d0 \uc788\uc5b4\uc57c \ud55c\ub2e4\uace0 \uac15\uc694\ud558\uc9c0 \uc54a\ub294\ub2e4. \uadf8\ub798\uc11c, \uc774 \ub4dc\ub77c\ub9c8\ub294 \uacfc\uac10\ud788 \ud604\uc2e4\uc744 \ud0c8\ud53c\ud558\uace0 \ubd95\uad34\uc2dc\ucf1c \uc790\uae30\ub4e4\uc774 \uac00\uc7a5 \uc798 \ud558\ub294 \uc774\uc57c\uae30\ub97c \ud568\uc73c\ub85c\uc368 \ubaa8\ub450\uc758 \uc695\ub9dd\uc744 \ubc1c\ud604\uc2dc\ud0a8\ub2e4. (\uc911\ub7b5) \ucd1d\ud0c4\uc740 \uc989\uc2dc \ud5e4\uc9d1\uc5b4 \ubf51\uc544\uc57c \ud558\uba70, \uc5d1\uc2a4\ub808\uc774\uc5d0\ub294 \ubd84\uba85\ud788 \ud558\uc580 \ub2e4\uc774\uc544\ubaac\ub4dc\uac00 \ucc0d\ud600\uc57c \ud55c\ub2e4. \ud1b5\ub150\uc0c1 \ubc18\uad70 \uc9c0\ub3c4\uc790\ub294 \uc544\ud504\ub9ac\uce74 \uc778\uc774\uace0, \ud30c\ubcd1\ub41c \ub098\ub77c \uc0ac\ub78c\ub4e4\uc740 \uc544\ub78d\uc5b4\ub97c \uc4f0\uace0, \uac31\uc740 \uc601\uc5b4\ub97c \uc4f0\ub294 \ubc31\uc778\uc774\uc5b4\uc57c \ud560 \uac83 \uac19\"\ub2e4\uace0 \ub300\uc911\uc774 \uc0dd\uac01\ud558\uae30 \ub54c\ubb38\uc5d0 \u300a\ud0dc\uc591\uc758 \ud6c4\uc608\u300b\ub294 \"\uadf8\ub7f0 \ub098\ub77c\uc640 \uc2dc\uacf5\uc744 \uad6c\ud604\ud574\ubc84\"\ub838\ub2e4. \"\uc81c\uc791\uc9c4\uc774 \uc758\ub3c4\uc801\uc73c\ub85c \ud3bc\uccd0\ub0b8, \ubb3c\ub9ac\uc801\uc73c\ub85c \ub4a4\uc8fd\ubc15\uc8fd\uc778 \uc774 \uacf5\uac04\uc5d0\uc11c, \uc774\uc57c\uae30\ub294 \ub0a0\uac1c\uac00 \ub3cb\uc544 \uad00\uac1d\uc758 \ub208\ub3d9\uc790\ub97c \ud33d\ud33d \ub3cc\ub9ac\uba70 \ubc1c\ud604\ub41c\ub2e4. \uc5ec\uc8fc\uc778\uacf5\uc758 \ucc28\ub294 \ub9e4\ud68c \uace4\ub450\ubc15\uc9c8\uce58\uace0, \ucd94\ub77d\ud558\uace0, \uc9c0\ub8b0\ubc2d\uc5d0 \ub4e4\uc5b4\uac04\ub2e4. \ub354\ubd88\uc5b4 \uc9c0\uc9c4\ub3c4 \ub098\uace0, \uc801\uad70\ub3c4 \uccd0\ub4e4\uc5b4 \uc624\uace0, \uc804\uc5fc\ubcd1\ub3c4 \ub3cc\uace0, \uac31\uc740 \ub9e4\ubc88 \ub3c4\ub07c\ub208\uc73c\ub85c \ucd1d\uc744 \uc3dc\ub2e4. \uadf8 \uc7a5\uce58\ub85c \uacf5\uace0\ud574\uc9c0\ub294 \uac83\uc740 \uc8fc\uc5b4\uc9c4 \uc721\uc2ed \ubd84\uc5d0\ub3c4 \uba87 \ubc88\uc529 \ud074\ub77c\uc774\ub9e5\uc2a4\ub97c \ub9cc\ub4e4 \uc218 \uc788\ub294 \uc2dc\uac01\uc801 \uad6c\ud604\uacfc, \uba87 \ubc88\uc529 \uba5c\ub85c\uc801 \uce74\ud0c0\ub974\uc2dc\uc2a4\ub97c \ub290\ub084 \uc218 \uc788\ub294 \ub2ec\ub2ec\ud55c \uc7a5\uba74\uc774\ub2e4.\" \ub610\ud55c \ub0a8\uad81\uc778\uc740 \"\uadf8 \uc640\uc911\uc5d0 \uc8fc\uc778\uacf5\ub4e4\uc774 \uc8fc\uace0\ubc1b\ub294 \ub300\uc0ac\ub4e4\uc740 \uc704\ud2b8\ub098 \uae34\ubc15\ud568\uc5d0\uc11c \ud765\ubbf8\ub86d\uace0 \uae30\ubc1c\"\ud558\uace0 \uc7a5\uba74\uc740 \"\ucc38\uc2e0\"\ud558\ub2e4 \ud3c9\ud588\ub2e4. \uadf8\ub798\uc11c \u300a\ud0dc\uc591\uc758 \ud6c4\uc608\u300b\uc5d0\ub294 \"\uc27d\uac8c \ub118\ubcf4\uae30 \ud798\ub4e0 \uba85\ub791\ud568\uc774 \uac00\ub4dd\ud558\ub2e4.\" \ub0a8\uad81\uc778\uc740 \ud0dc\uc591\uc758 \ud6c4\uc608\uac00 \ub300\uc911\uc758 \"\uc695\ub9dd\uc758 \ud604\uc7ac \uc8fc\uc18c\"\ub97c \ud3ec\ucc29\ud55c \ub4a4 \uc2dc\ub3c4\ud55c \"\ub098\ub984\ub300\ub85c\uc758 \uc791\ud488\uc138\uacc4\uc640 \uc791\ud654 \ubc29\ubc95\"\uc774 \uc628 \uc138\uacc4\ub97c \"\uc9c0\ubc30\ud560 \uc815\ub3c4\uc758 \uc7a5\ub974\ud654\ub41c \ud558\ub098\uc758 \ud604\uc0c1\"\uc774 \ub418\uc5c8\ub2e4\uace0 \uacb0\ub860\uc9c0\uc5c8\ub2e4.", "answer": "\uc720\uac74\uc2dd", "sentence": "\uc720\uac74\uc2dd \uc740 \"\ud0c4\ud0c4\ud558\uba74\uc11c\ub3c4 \ube60\ub978 \uc2a4\ud1a0\ub9ac \uc804\uac1c\uc640 \uc601\ud654 \uac19\uc740 \uc9c8\uac10, \uc778\ub958\uc560\uc5d0\uc11c \ud53c\uc5b4\ub098\ub294 \ucee4\ud50c\ub4e4\uc758 \uc0ac\ub791\uc774 \uc801\uc808\ud788 \ubc30\ubd84\ub41c \uac83\uc774 \uc778\uae30\uc694\uc778\uc73c\ub85c \uaf3d\ud788\uace0 \uc788\ub2e4.", "paragraph_sentence": "\uc601\uad6d BBC\ub294\u300a\ud0dc\uc591\uc758 \ud6c4\uc608\u300b\uac00 \ud55c\ub958\uc758 \uc815\uc810\uc744 \ucc0d\uc744 \uac83\uc774\ub77c \uc608\uc0c1\ud588\ub2e4. \uc720\uac74\uc2dd \uc740 \"\ud0c4\ud0c4\ud558\uba74\uc11c\ub3c4 \ube60\ub978 \uc2a4\ud1a0\ub9ac \uc804\uac1c\uc640 \uc601\ud654 \uac19\uc740 \uc9c8\uac10, \uc778\ub958\uc560\uc5d0\uc11c \ud53c\uc5b4\ub098\ub294 \ucee4\ud50c\ub4e4\uc758 \uc0ac\ub791\uc774 \uc801\uc808\ud788 \ubc30\ubd84\ub41c \uac83\uc774 \uc778\uae30\uc694\uc778\uc73c\ub85c \uaf3d\ud788\uace0 \uc788\ub2e4. \u201d\uace0 \uc804\ud588\ub2e4. \ub0a8\uad81\uc778\uc740 \ud3c9\ub860\uc5d0\uc11c \u300a\ud0dc\uc591\uc758 \ud6c4\uc608\u300b\uc758 \uc778\uae30\uc694\uc778\uc5d0 \ub300\ud574 \ub2e4\uc74c\uacfc \uac19\uc774 \ubd84\uc11d\ud588\ub2e4. \u300a\ud0dc\uc591\uc758 \ud6c4\uc608\u300b\ub294 \"\uc8fc\uc5b4\uc9c4 \uc2dc\uac04\uc5d0 \uadf9\uc801\uc778 \uc774\uc57c\uae30\ub97c \ubc30\uce58\ud558\ub294 \ud6a8\uc728\uc131\uc73c\ub85c \ud1b1\ub2c8\ubc14\ud034\ucc98\ub7fc \ub9de\ubb3c\ub824 \uc788\ub2e4. \uc774\uc57c\uae30\ud558\uace0\uc790 \ud558\ub294 \ubc14\uc640, \uadf8 \uc774\uc57c\uae30\ub97c \uc2dc\uac01\ud654\ud558\uc5ec \ubcf4\uc5ec\uc8fc\uace0\uc790 \ud558\ub294 \uc695\ub9dd\uc744 \uc5b4\ub514\uae4c\uc9c0 \uc2e4\ud604\ud560 \uc218 \uc788\ub290\ub0d0\ub294 \uc9c8\ubb38\uc5d0 \ub2f5\ud558\ub294, \uadf9\ub2e8\uc801\uc778 \ubc1c\uc804\uc758 \ud55c \uc608\ub2e4. \u300a\ud0dc\uc591\uc758 \ud6c4\uc608\u300b\uc5d0\uc11c \ub300\uc911\uc740 \"\ub2e4\uc774\ub0b4\ubbf9\ud55c \ud074\ub77c\uc774\ub9e5\uc2a4, \uadf8\ub9ac\uace0 \uae30\uc6d0\ud558\ub358 \uac83\uc774 \uc774\ub8e8\uc5b4\uc84c\uc744 \ub54c\uc758 \uce74\ud0c0\ub974\uc2dc\uc2a4\"\ub97c \ub290\ub07c\uac8c \ub41c\ub2e4. \u300a\ud0dc\uc591\uc758 \ud6c4\uc608\u300b\ub294 \"\uc774 \uc695\ub9dd\uc744 \uc2ed\ubd84 \ucda9\uc871\uc2dc\ud0a4\uae30 \uc704\ud574 \ud604\uc2e4\uc774\ub098 \uace0\uc99d\uc740 \uacfc\uac10\ud788 \ubb34\uc2dc\ud558\uace0, \uc77c\ubc18 \ub300\uc911\uc758 \ub208\ub192\uc774\uc5d0 \uc11c\uc11c, \uadf8\ub4e4\uc774 \ubcf4\uace0 \uc2f6\uc740 \uc7a5\uba74\uc774\ub77c\uba74 \ubb34\uc5c7\uc774\ub4e0 \ubcf4\uc5ec\uc900\ub2e4. (\uc911\ub7b5) \ub300\uc911\uc774 \ubcf4\uace0 \uc2f6\uc5b4 \ud558\ub294 \uac83\uc740 \ucd1d\ud0c4\uc774 \ubc30\uc5d0 \ubc15\ud614\ub2e4\uace0 \ubc14\uc774\ud0c8\uc744 \uc7ac\uba74\uc11c \ubcd1\uc6d0\uc73c\ub85c \uc2e0\uc18d\ud788 \ud6c4\uc1a1\ud558\ub294 \uc7a5\uba74\ub3c4 \uc544\ub2c8\uace0, \ub2e4\uc774\uc544\ubaac\ub4dc\ub294 \uc548 \ucc0d\ud614\uc9c0\ub9cc \ud544\ub984\uc744 \ub4e4\uace0 \uc640\uc11c \uc815\ud669\uc0c1 \uc758\uc2ec \uac00\ub2a5\ud560 \uc218 \uc788\ub2e4\uace0 \ub9dd\uc124\uc774\ub294 \uc7a5\uba74\ub3c4, \uccad\uc9c4\uae30\ub97c \uc548 \ub300\uace0 \ubc30\ub97c \uc5f4\uc5b4 \uc870\uc2ec\uc870\uc2ec \ucd09\uc9c4\ud574 \ud658\uc790\uc758 \ubc18\uc751\uc744 \uc0b4\ud53c\ub294 \uac83\ub3c4 \uc544\ub2c8\ub2e4. \ub300\uc911\uacfc TV \uc548\uc5d0\ub294 \uc9c1\uad00\uc801\uc778 \uc138\uc0c1\uc774 \uc788\uc744 \ubfd0, \uc544\ubb34\ub3c4 \uadf8\uac83\uc774 \ud604\uc2e4\uc801\uc778 \uacbd\uacc4\uc5d0 \uc788\uc5b4\uc57c \ud55c\ub2e4\uace0 \uac15\uc694\ud558\uc9c0 \uc54a\ub294\ub2e4. \uadf8\ub798\uc11c, \uc774 \ub4dc\ub77c\ub9c8\ub294 \uacfc\uac10\ud788 \ud604\uc2e4\uc744 \ud0c8\ud53c\ud558\uace0 \ubd95\uad34\uc2dc\ucf1c \uc790\uae30\ub4e4\uc774 \uac00\uc7a5 \uc798 \ud558\ub294 \uc774\uc57c\uae30\ub97c \ud568\uc73c\ub85c\uc368 \ubaa8\ub450\uc758 \uc695\ub9dd\uc744 \ubc1c\ud604\uc2dc\ud0a8\ub2e4. (\uc911\ub7b5) \ucd1d\ud0c4\uc740 \uc989\uc2dc \ud5e4\uc9d1\uc5b4 \ubf51\uc544\uc57c \ud558\uba70, \uc5d1\uc2a4\ub808\uc774\uc5d0\ub294 \ubd84\uba85\ud788 \ud558\uc580 \ub2e4\uc774\uc544\ubaac\ub4dc\uac00 \ucc0d\ud600\uc57c \ud55c\ub2e4. \ud1b5\ub150\uc0c1 \ubc18\uad70 \uc9c0\ub3c4\uc790\ub294 \uc544\ud504\ub9ac\uce74 \uc778\uc774\uace0, \ud30c\ubcd1\ub41c \ub098\ub77c \uc0ac\ub78c\ub4e4\uc740 \uc544\ub78d\uc5b4\ub97c \uc4f0\uace0, \uac31\uc740 \uc601\uc5b4\ub97c \uc4f0\ub294 \ubc31\uc778\uc774\uc5b4\uc57c \ud560 \uac83 \uac19\"\ub2e4\uace0 \ub300\uc911\uc774 \uc0dd\uac01\ud558\uae30 \ub54c\ubb38\uc5d0 \u300a\ud0dc\uc591\uc758 \ud6c4\uc608\u300b\ub294 \"\uadf8\ub7f0 \ub098\ub77c\uc640 \uc2dc\uacf5\uc744 \uad6c\ud604\ud574\ubc84\"\ub838\ub2e4. \"\uc81c\uc791\uc9c4\uc774 \uc758\ub3c4\uc801\uc73c\ub85c \ud3bc\uccd0\ub0b8, \ubb3c\ub9ac\uc801\uc73c\ub85c \ub4a4\uc8fd\ubc15\uc8fd\uc778 \uc774 \uacf5\uac04\uc5d0\uc11c, \uc774\uc57c\uae30\ub294 \ub0a0\uac1c\uac00 \ub3cb\uc544 \uad00\uac1d\uc758 \ub208\ub3d9\uc790\ub97c \ud33d\ud33d \ub3cc\ub9ac\uba70 \ubc1c\ud604\ub41c\ub2e4. \uc5ec\uc8fc\uc778\uacf5\uc758 \ucc28\ub294 \ub9e4\ud68c \uace4\ub450\ubc15\uc9c8\uce58\uace0, \ucd94\ub77d\ud558\uace0, \uc9c0\ub8b0\ubc2d\uc5d0 \ub4e4\uc5b4\uac04\ub2e4. \ub354\ubd88\uc5b4 \uc9c0\uc9c4\ub3c4 \ub098\uace0, \uc801\uad70\ub3c4 \uccd0\ub4e4\uc5b4 \uc624\uace0, \uc804\uc5fc\ubcd1\ub3c4 \ub3cc\uace0, \uac31\uc740 \ub9e4\ubc88 \ub3c4\ub07c\ub208\uc73c\ub85c \ucd1d\uc744 \uc3dc\ub2e4. \uadf8 \uc7a5\uce58\ub85c \uacf5\uace0\ud574\uc9c0\ub294 \uac83\uc740 \uc8fc\uc5b4\uc9c4 \uc721\uc2ed \ubd84\uc5d0\ub3c4 \uba87 \ubc88\uc529 \ud074\ub77c\uc774\ub9e5\uc2a4\ub97c \ub9cc\ub4e4 \uc218 \uc788\ub294 \uc2dc\uac01\uc801 \uad6c\ud604\uacfc, \uba87 \ubc88\uc529 \uba5c\ub85c\uc801 \uce74\ud0c0\ub974\uc2dc\uc2a4\ub97c \ub290\ub084 \uc218 \uc788\ub294 \ub2ec\ub2ec\ud55c \uc7a5\uba74\uc774\ub2e4. \" \ub610\ud55c \ub0a8\uad81\uc778\uc740 \"\uadf8 \uc640\uc911\uc5d0 \uc8fc\uc778\uacf5\ub4e4\uc774 \uc8fc\uace0\ubc1b\ub294 \ub300\uc0ac\ub4e4\uc740 \uc704\ud2b8\ub098 \uae34\ubc15\ud568\uc5d0\uc11c \ud765\ubbf8\ub86d\uace0 \uae30\ubc1c\"\ud558\uace0 \uc7a5\uba74\uc740 \"\ucc38\uc2e0\"\ud558\ub2e4 \ud3c9\ud588\ub2e4. \uadf8\ub798\uc11c \u300a\ud0dc\uc591\uc758 \ud6c4\uc608\u300b\uc5d0\ub294 \"\uc27d\uac8c \ub118\ubcf4\uae30 \ud798\ub4e0 \uba85\ub791\ud568\uc774 \uac00\ub4dd\ud558\ub2e4. \" \ub0a8\uad81\uc778\uc740 \ud0dc\uc591\uc758 \ud6c4\uc608\uac00 \ub300\uc911\uc758 \"\uc695\ub9dd\uc758 \ud604\uc7ac \uc8fc\uc18c\"\ub97c \ud3ec\ucc29\ud55c \ub4a4 \uc2dc\ub3c4\ud55c \"\ub098\ub984\ub300\ub85c\uc758 \uc791\ud488\uc138\uacc4\uc640 \uc791\ud654 \ubc29\ubc95\"\uc774 \uc628 \uc138\uacc4\ub97c \"\uc9c0\ubc30\ud560 \uc815\ub3c4\uc758 \uc7a5\ub974\ud654\ub41c \ud558\ub098\uc758 \ud604\uc0c1\"\uc774 \ub418\uc5c8\ub2e4\uace0 \uacb0\ub860\uc9c0\uc5c8\ub2e4.", "paragraph_answer": "\uc601\uad6d BBC\ub294\u300a\ud0dc\uc591\uc758 \ud6c4\uc608\u300b\uac00 \ud55c\ub958\uc758 \uc815\uc810\uc744 \ucc0d\uc744 \uac83\uc774\ub77c \uc608\uc0c1\ud588\ub2e4. \uc720\uac74\uc2dd \uc740 \"\ud0c4\ud0c4\ud558\uba74\uc11c\ub3c4 \ube60\ub978 \uc2a4\ud1a0\ub9ac \uc804\uac1c\uc640 \uc601\ud654 \uac19\uc740 \uc9c8\uac10, \uc778\ub958\uc560\uc5d0\uc11c \ud53c\uc5b4\ub098\ub294 \ucee4\ud50c\ub4e4\uc758 \uc0ac\ub791\uc774 \uc801\uc808\ud788 \ubc30\ubd84\ub41c \uac83\uc774 \uc778\uae30\uc694\uc778\uc73c\ub85c \uaf3d\ud788\uace0 \uc788\ub2e4.\u201d\uace0 \uc804\ud588\ub2e4. \ub0a8\uad81\uc778\uc740 \ud3c9\ub860\uc5d0\uc11c \u300a\ud0dc\uc591\uc758 \ud6c4\uc608\u300b\uc758 \uc778\uae30\uc694\uc778\uc5d0 \ub300\ud574 \ub2e4\uc74c\uacfc \uac19\uc774 \ubd84\uc11d\ud588\ub2e4. \u300a\ud0dc\uc591\uc758 \ud6c4\uc608\u300b\ub294 \"\uc8fc\uc5b4\uc9c4 \uc2dc\uac04\uc5d0 \uadf9\uc801\uc778 \uc774\uc57c\uae30\ub97c \ubc30\uce58\ud558\ub294 \ud6a8\uc728\uc131\uc73c\ub85c \ud1b1\ub2c8\ubc14\ud034\ucc98\ub7fc \ub9de\ubb3c\ub824 \uc788\ub2e4. \uc774\uc57c\uae30\ud558\uace0\uc790 \ud558\ub294 \ubc14\uc640, \uadf8 \uc774\uc57c\uae30\ub97c \uc2dc\uac01\ud654\ud558\uc5ec \ubcf4\uc5ec\uc8fc\uace0\uc790 \ud558\ub294 \uc695\ub9dd\uc744 \uc5b4\ub514\uae4c\uc9c0 \uc2e4\ud604\ud560 \uc218 \uc788\ub290\ub0d0\ub294 \uc9c8\ubb38\uc5d0 \ub2f5\ud558\ub294, \uadf9\ub2e8\uc801\uc778 \ubc1c\uc804\uc758 \ud55c \uc608\ub2e4. \u300a\ud0dc\uc591\uc758 \ud6c4\uc608\u300b\uc5d0\uc11c \ub300\uc911\uc740 \"\ub2e4\uc774\ub0b4\ubbf9\ud55c \ud074\ub77c\uc774\ub9e5\uc2a4, \uadf8\ub9ac\uace0 \uae30\uc6d0\ud558\ub358 \uac83\uc774 \uc774\ub8e8\uc5b4\uc84c\uc744 \ub54c\uc758 \uce74\ud0c0\ub974\uc2dc\uc2a4\"\ub97c \ub290\ub07c\uac8c \ub41c\ub2e4. \u300a\ud0dc\uc591\uc758 \ud6c4\uc608\u300b\ub294 \"\uc774 \uc695\ub9dd\uc744 \uc2ed\ubd84 \ucda9\uc871\uc2dc\ud0a4\uae30 \uc704\ud574 \ud604\uc2e4\uc774\ub098 \uace0\uc99d\uc740 \uacfc\uac10\ud788 \ubb34\uc2dc\ud558\uace0, \uc77c\ubc18 \ub300\uc911\uc758 \ub208\ub192\uc774\uc5d0 \uc11c\uc11c, \uadf8\ub4e4\uc774 \ubcf4\uace0 \uc2f6\uc740 \uc7a5\uba74\uc774\ub77c\uba74 \ubb34\uc5c7\uc774\ub4e0 \ubcf4\uc5ec\uc900\ub2e4. (\uc911\ub7b5) \ub300\uc911\uc774 \ubcf4\uace0 \uc2f6\uc5b4 \ud558\ub294 \uac83\uc740 \ucd1d\ud0c4\uc774 \ubc30\uc5d0 \ubc15\ud614\ub2e4\uace0 \ubc14\uc774\ud0c8\uc744 \uc7ac\uba74\uc11c \ubcd1\uc6d0\uc73c\ub85c \uc2e0\uc18d\ud788 \ud6c4\uc1a1\ud558\ub294 \uc7a5\uba74\ub3c4 \uc544\ub2c8\uace0, \ub2e4\uc774\uc544\ubaac\ub4dc\ub294 \uc548 \ucc0d\ud614\uc9c0\ub9cc \ud544\ub984\uc744 \ub4e4\uace0 \uc640\uc11c \uc815\ud669\uc0c1 \uc758\uc2ec \uac00\ub2a5\ud560 \uc218 \uc788\ub2e4\uace0 \ub9dd\uc124\uc774\ub294 \uc7a5\uba74\ub3c4, \uccad\uc9c4\uae30\ub97c \uc548 \ub300\uace0 \ubc30\ub97c \uc5f4\uc5b4 \uc870\uc2ec\uc870\uc2ec \ucd09\uc9c4\ud574 \ud658\uc790\uc758 \ubc18\uc751\uc744 \uc0b4\ud53c\ub294 \uac83\ub3c4 \uc544\ub2c8\ub2e4. \ub300\uc911\uacfc TV \uc548\uc5d0\ub294 \uc9c1\uad00\uc801\uc778 \uc138\uc0c1\uc774 \uc788\uc744 \ubfd0, \uc544\ubb34\ub3c4 \uadf8\uac83\uc774 \ud604\uc2e4\uc801\uc778 \uacbd\uacc4\uc5d0 \uc788\uc5b4\uc57c \ud55c\ub2e4\uace0 \uac15\uc694\ud558\uc9c0 \uc54a\ub294\ub2e4. \uadf8\ub798\uc11c, \uc774 \ub4dc\ub77c\ub9c8\ub294 \uacfc\uac10\ud788 \ud604\uc2e4\uc744 \ud0c8\ud53c\ud558\uace0 \ubd95\uad34\uc2dc\ucf1c \uc790\uae30\ub4e4\uc774 \uac00\uc7a5 \uc798 \ud558\ub294 \uc774\uc57c\uae30\ub97c \ud568\uc73c\ub85c\uc368 \ubaa8\ub450\uc758 \uc695\ub9dd\uc744 \ubc1c\ud604\uc2dc\ud0a8\ub2e4. (\uc911\ub7b5) \ucd1d\ud0c4\uc740 \uc989\uc2dc \ud5e4\uc9d1\uc5b4 \ubf51\uc544\uc57c \ud558\uba70, \uc5d1\uc2a4\ub808\uc774\uc5d0\ub294 \ubd84\uba85\ud788 \ud558\uc580 \ub2e4\uc774\uc544\ubaac\ub4dc\uac00 \ucc0d\ud600\uc57c \ud55c\ub2e4. \ud1b5\ub150\uc0c1 \ubc18\uad70 \uc9c0\ub3c4\uc790\ub294 \uc544\ud504\ub9ac\uce74 \uc778\uc774\uace0, \ud30c\ubcd1\ub41c \ub098\ub77c \uc0ac\ub78c\ub4e4\uc740 \uc544\ub78d\uc5b4\ub97c \uc4f0\uace0, \uac31\uc740 \uc601\uc5b4\ub97c \uc4f0\ub294 \ubc31\uc778\uc774\uc5b4\uc57c \ud560 \uac83 \uac19\"\ub2e4\uace0 \ub300\uc911\uc774 \uc0dd\uac01\ud558\uae30 \ub54c\ubb38\uc5d0 \u300a\ud0dc\uc591\uc758 \ud6c4\uc608\u300b\ub294 \"\uadf8\ub7f0 \ub098\ub77c\uc640 \uc2dc\uacf5\uc744 \uad6c\ud604\ud574\ubc84\"\ub838\ub2e4. \"\uc81c\uc791\uc9c4\uc774 \uc758\ub3c4\uc801\uc73c\ub85c \ud3bc\uccd0\ub0b8, \ubb3c\ub9ac\uc801\uc73c\ub85c \ub4a4\uc8fd\ubc15\uc8fd\uc778 \uc774 \uacf5\uac04\uc5d0\uc11c, \uc774\uc57c\uae30\ub294 \ub0a0\uac1c\uac00 \ub3cb\uc544 \uad00\uac1d\uc758 \ub208\ub3d9\uc790\ub97c \ud33d\ud33d \ub3cc\ub9ac\uba70 \ubc1c\ud604\ub41c\ub2e4. \uc5ec\uc8fc\uc778\uacf5\uc758 \ucc28\ub294 \ub9e4\ud68c \uace4\ub450\ubc15\uc9c8\uce58\uace0, \ucd94\ub77d\ud558\uace0, \uc9c0\ub8b0\ubc2d\uc5d0 \ub4e4\uc5b4\uac04\ub2e4. \ub354\ubd88\uc5b4 \uc9c0\uc9c4\ub3c4 \ub098\uace0, \uc801\uad70\ub3c4 \uccd0\ub4e4\uc5b4 \uc624\uace0, \uc804\uc5fc\ubcd1\ub3c4 \ub3cc\uace0, \uac31\uc740 \ub9e4\ubc88 \ub3c4\ub07c\ub208\uc73c\ub85c \ucd1d\uc744 \uc3dc\ub2e4. \uadf8 \uc7a5\uce58\ub85c \uacf5\uace0\ud574\uc9c0\ub294 \uac83\uc740 \uc8fc\uc5b4\uc9c4 \uc721\uc2ed \ubd84\uc5d0\ub3c4 \uba87 \ubc88\uc529 \ud074\ub77c\uc774\ub9e5\uc2a4\ub97c \ub9cc\ub4e4 \uc218 \uc788\ub294 \uc2dc\uac01\uc801 \uad6c\ud604\uacfc, \uba87 \ubc88\uc529 \uba5c\ub85c\uc801 \uce74\ud0c0\ub974\uc2dc\uc2a4\ub97c \ub290\ub084 \uc218 \uc788\ub294 \ub2ec\ub2ec\ud55c \uc7a5\uba74\uc774\ub2e4.\" \ub610\ud55c \ub0a8\uad81\uc778\uc740 \"\uadf8 \uc640\uc911\uc5d0 \uc8fc\uc778\uacf5\ub4e4\uc774 \uc8fc\uace0\ubc1b\ub294 \ub300\uc0ac\ub4e4\uc740 \uc704\ud2b8\ub098 \uae34\ubc15\ud568\uc5d0\uc11c \ud765\ubbf8\ub86d\uace0 \uae30\ubc1c\"\ud558\uace0 \uc7a5\uba74\uc740 \"\ucc38\uc2e0\"\ud558\ub2e4 \ud3c9\ud588\ub2e4. \uadf8\ub798\uc11c \u300a\ud0dc\uc591\uc758 \ud6c4\uc608\u300b\uc5d0\ub294 \"\uc27d\uac8c \ub118\ubcf4\uae30 \ud798\ub4e0 \uba85\ub791\ud568\uc774 \uac00\ub4dd\ud558\ub2e4.\" \ub0a8\uad81\uc778\uc740 \ud0dc\uc591\uc758 \ud6c4\uc608\uac00 \ub300\uc911\uc758 \"\uc695\ub9dd\uc758 \ud604\uc7ac \uc8fc\uc18c\"\ub97c \ud3ec\ucc29\ud55c \ub4a4 \uc2dc\ub3c4\ud55c \"\ub098\ub984\ub300\ub85c\uc758 \uc791\ud488\uc138\uacc4\uc640 \uc791\ud654 \ubc29\ubc95\"\uc774 \uc628 \uc138\uacc4\ub97c \"\uc9c0\ubc30\ud560 \uc815\ub3c4\uc758 \uc7a5\ub974\ud654\ub41c \ud558\ub098\uc758 \ud604\uc0c1\"\uc774 \ub418\uc5c8\ub2e4\uace0 \uacb0\ub860\uc9c0\uc5c8\ub2e4.", "sentence_answer": " \uc720\uac74\uc2dd \uc740 \"\ud0c4\ud0c4\ud558\uba74\uc11c\ub3c4 \ube60\ub978 \uc2a4\ud1a0\ub9ac \uc804\uac1c\uc640 \uc601\ud654 \uac19\uc740 \uc9c8\uac10, \uc778\ub958\uc560\uc5d0\uc11c \ud53c\uc5b4\ub098\ub294 \ucee4\ud50c\ub4e4\uc758 \uc0ac\ub791\uc774 \uc801\uc808\ud788 \ubc30\ubd84\ub41c \uac83\uc774 \uc778\uae30\uc694\uc778\uc73c\ub85c \uaf3d\ud788\uace0 \uc788\ub2e4.", "paragraph_id": "6598975-10-0", "paragraph_question": "question: \ud0dc\uc591\uc758 \ud6c4\uc608\uac00 \ud0c4\ud0c4\ud55c \uc2a4\ud1a0\ub9ac\uc640 \uc778\ub958\uc560 \ubc0f \ucee4\ud50c\ub4e4\uc758 \uc0ac\ub791\uc774\ub77c\ub294 \uc694\uc18c\uac00 \uc801\uc808\ud788 \ubc30\ud569\ub418\uc5b4 \uc778\uae30 \uc788\ub2e4\uace0 \ud55c \uc0ac\ub78c\uc740?, context: \uc601\uad6d BBC\ub294\u300a\ud0dc\uc591\uc758 \ud6c4\uc608\u300b\uac00 \ud55c\ub958\uc758 \uc815\uc810\uc744 \ucc0d\uc744 \uac83\uc774\ub77c \uc608\uc0c1\ud588\ub2e4. \uc720\uac74\uc2dd\uc740 \"\ud0c4\ud0c4\ud558\uba74\uc11c\ub3c4 \ube60\ub978 \uc2a4\ud1a0\ub9ac \uc804\uac1c\uc640 \uc601\ud654 \uac19\uc740 \uc9c8\uac10, \uc778\ub958\uc560\uc5d0\uc11c \ud53c\uc5b4\ub098\ub294 \ucee4\ud50c\ub4e4\uc758 \uc0ac\ub791\uc774 \uc801\uc808\ud788 \ubc30\ubd84\ub41c \uac83\uc774 \uc778\uae30\uc694\uc778\uc73c\ub85c \uaf3d\ud788\uace0 \uc788\ub2e4.\u201d\uace0 \uc804\ud588\ub2e4. \ub0a8\uad81\uc778\uc740 \ud3c9\ub860\uc5d0\uc11c \u300a\ud0dc\uc591\uc758 \ud6c4\uc608\u300b\uc758 \uc778\uae30\uc694\uc778\uc5d0 \ub300\ud574 \ub2e4\uc74c\uacfc \uac19\uc774 \ubd84\uc11d\ud588\ub2e4. \u300a\ud0dc\uc591\uc758 \ud6c4\uc608\u300b\ub294 \"\uc8fc\uc5b4\uc9c4 \uc2dc\uac04\uc5d0 \uadf9\uc801\uc778 \uc774\uc57c\uae30\ub97c \ubc30\uce58\ud558\ub294 \ud6a8\uc728\uc131\uc73c\ub85c \ud1b1\ub2c8\ubc14\ud034\ucc98\ub7fc \ub9de\ubb3c\ub824 \uc788\ub2e4. \uc774\uc57c\uae30\ud558\uace0\uc790 \ud558\ub294 \ubc14\uc640, \uadf8 \uc774\uc57c\uae30\ub97c \uc2dc\uac01\ud654\ud558\uc5ec \ubcf4\uc5ec\uc8fc\uace0\uc790 \ud558\ub294 \uc695\ub9dd\uc744 \uc5b4\ub514\uae4c\uc9c0 \uc2e4\ud604\ud560 \uc218 \uc788\ub290\ub0d0\ub294 \uc9c8\ubb38\uc5d0 \ub2f5\ud558\ub294, \uadf9\ub2e8\uc801\uc778 \ubc1c\uc804\uc758 \ud55c \uc608\ub2e4. \u300a\ud0dc\uc591\uc758 \ud6c4\uc608\u300b\uc5d0\uc11c \ub300\uc911\uc740 \"\ub2e4\uc774\ub0b4\ubbf9\ud55c \ud074\ub77c\uc774\ub9e5\uc2a4, \uadf8\ub9ac\uace0 \uae30\uc6d0\ud558\ub358 \uac83\uc774 \uc774\ub8e8\uc5b4\uc84c\uc744 \ub54c\uc758 \uce74\ud0c0\ub974\uc2dc\uc2a4\"\ub97c \ub290\ub07c\uac8c \ub41c\ub2e4. \u300a\ud0dc\uc591\uc758 \ud6c4\uc608\u300b\ub294 \"\uc774 \uc695\ub9dd\uc744 \uc2ed\ubd84 \ucda9\uc871\uc2dc\ud0a4\uae30 \uc704\ud574 \ud604\uc2e4\uc774\ub098 \uace0\uc99d\uc740 \uacfc\uac10\ud788 \ubb34\uc2dc\ud558\uace0, \uc77c\ubc18 \ub300\uc911\uc758 \ub208\ub192\uc774\uc5d0 \uc11c\uc11c, \uadf8\ub4e4\uc774 \ubcf4\uace0 \uc2f6\uc740 \uc7a5\uba74\uc774\ub77c\uba74 \ubb34\uc5c7\uc774\ub4e0 \ubcf4\uc5ec\uc900\ub2e4. (\uc911\ub7b5) \ub300\uc911\uc774 \ubcf4\uace0 \uc2f6\uc5b4 \ud558\ub294 \uac83\uc740 \ucd1d\ud0c4\uc774 \ubc30\uc5d0 \ubc15\ud614\ub2e4\uace0 \ubc14\uc774\ud0c8\uc744 \uc7ac\uba74\uc11c \ubcd1\uc6d0\uc73c\ub85c \uc2e0\uc18d\ud788 \ud6c4\uc1a1\ud558\ub294 \uc7a5\uba74\ub3c4 \uc544\ub2c8\uace0, \ub2e4\uc774\uc544\ubaac\ub4dc\ub294 \uc548 \ucc0d\ud614\uc9c0\ub9cc \ud544\ub984\uc744 \ub4e4\uace0 \uc640\uc11c \uc815\ud669\uc0c1 \uc758\uc2ec \uac00\ub2a5\ud560 \uc218 \uc788\ub2e4\uace0 \ub9dd\uc124\uc774\ub294 \uc7a5\uba74\ub3c4, \uccad\uc9c4\uae30\ub97c \uc548 \ub300\uace0 \ubc30\ub97c \uc5f4\uc5b4 \uc870\uc2ec\uc870\uc2ec \ucd09\uc9c4\ud574 \ud658\uc790\uc758 \ubc18\uc751\uc744 \uc0b4\ud53c\ub294 \uac83\ub3c4 \uc544\ub2c8\ub2e4. \ub300\uc911\uacfc TV \uc548\uc5d0\ub294 \uc9c1\uad00\uc801\uc778 \uc138\uc0c1\uc774 \uc788\uc744 \ubfd0, \uc544\ubb34\ub3c4 \uadf8\uac83\uc774 \ud604\uc2e4\uc801\uc778 \uacbd\uacc4\uc5d0 \uc788\uc5b4\uc57c \ud55c\ub2e4\uace0 \uac15\uc694\ud558\uc9c0 \uc54a\ub294\ub2e4. \uadf8\ub798\uc11c, \uc774 \ub4dc\ub77c\ub9c8\ub294 \uacfc\uac10\ud788 \ud604\uc2e4\uc744 \ud0c8\ud53c\ud558\uace0 \ubd95\uad34\uc2dc\ucf1c \uc790\uae30\ub4e4\uc774 \uac00\uc7a5 \uc798 \ud558\ub294 \uc774\uc57c\uae30\ub97c \ud568\uc73c\ub85c\uc368 \ubaa8\ub450\uc758 \uc695\ub9dd\uc744 \ubc1c\ud604\uc2dc\ud0a8\ub2e4. (\uc911\ub7b5) \ucd1d\ud0c4\uc740 \uc989\uc2dc \ud5e4\uc9d1\uc5b4 \ubf51\uc544\uc57c \ud558\uba70, \uc5d1\uc2a4\ub808\uc774\uc5d0\ub294 \ubd84\uba85\ud788 \ud558\uc580 \ub2e4\uc774\uc544\ubaac\ub4dc\uac00 \ucc0d\ud600\uc57c \ud55c\ub2e4. \ud1b5\ub150\uc0c1 \ubc18\uad70 \uc9c0\ub3c4\uc790\ub294 \uc544\ud504\ub9ac\uce74 \uc778\uc774\uace0, \ud30c\ubcd1\ub41c \ub098\ub77c \uc0ac\ub78c\ub4e4\uc740 \uc544\ub78d\uc5b4\ub97c \uc4f0\uace0, \uac31\uc740 \uc601\uc5b4\ub97c \uc4f0\ub294 \ubc31\uc778\uc774\uc5b4\uc57c \ud560 \uac83 \uac19\"\ub2e4\uace0 \ub300\uc911\uc774 \uc0dd\uac01\ud558\uae30 \ub54c\ubb38\uc5d0 \u300a\ud0dc\uc591\uc758 \ud6c4\uc608\u300b\ub294 \"\uadf8\ub7f0 \ub098\ub77c\uc640 \uc2dc\uacf5\uc744 \uad6c\ud604\ud574\ubc84\"\ub838\ub2e4. \"\uc81c\uc791\uc9c4\uc774 \uc758\ub3c4\uc801\uc73c\ub85c \ud3bc\uccd0\ub0b8, \ubb3c\ub9ac\uc801\uc73c\ub85c \ub4a4\uc8fd\ubc15\uc8fd\uc778 \uc774 \uacf5\uac04\uc5d0\uc11c, \uc774\uc57c\uae30\ub294 \ub0a0\uac1c\uac00 \ub3cb\uc544 \uad00\uac1d\uc758 \ub208\ub3d9\uc790\ub97c \ud33d\ud33d \ub3cc\ub9ac\uba70 \ubc1c\ud604\ub41c\ub2e4. \uc5ec\uc8fc\uc778\uacf5\uc758 \ucc28\ub294 \ub9e4\ud68c \uace4\ub450\ubc15\uc9c8\uce58\uace0, \ucd94\ub77d\ud558\uace0, \uc9c0\ub8b0\ubc2d\uc5d0 \ub4e4\uc5b4\uac04\ub2e4. \ub354\ubd88\uc5b4 \uc9c0\uc9c4\ub3c4 \ub098\uace0, \uc801\uad70\ub3c4 \uccd0\ub4e4\uc5b4 \uc624\uace0, \uc804\uc5fc\ubcd1\ub3c4 \ub3cc\uace0, \uac31\uc740 \ub9e4\ubc88 \ub3c4\ub07c\ub208\uc73c\ub85c \ucd1d\uc744 \uc3dc\ub2e4. \uadf8 \uc7a5\uce58\ub85c \uacf5\uace0\ud574\uc9c0\ub294 \uac83\uc740 \uc8fc\uc5b4\uc9c4 \uc721\uc2ed \ubd84\uc5d0\ub3c4 \uba87 \ubc88\uc529 \ud074\ub77c\uc774\ub9e5\uc2a4\ub97c \ub9cc\ub4e4 \uc218 \uc788\ub294 \uc2dc\uac01\uc801 \uad6c\ud604\uacfc, \uba87 \ubc88\uc529 \uba5c\ub85c\uc801 \uce74\ud0c0\ub974\uc2dc\uc2a4\ub97c \ub290\ub084 \uc218 \uc788\ub294 \ub2ec\ub2ec\ud55c \uc7a5\uba74\uc774\ub2e4.\" \ub610\ud55c \ub0a8\uad81\uc778\uc740 \"\uadf8 \uc640\uc911\uc5d0 \uc8fc\uc778\uacf5\ub4e4\uc774 \uc8fc\uace0\ubc1b\ub294 \ub300\uc0ac\ub4e4\uc740 \uc704\ud2b8\ub098 \uae34\ubc15\ud568\uc5d0\uc11c \ud765\ubbf8\ub86d\uace0 \uae30\ubc1c\"\ud558\uace0 \uc7a5\uba74\uc740 \"\ucc38\uc2e0\"\ud558\ub2e4 \ud3c9\ud588\ub2e4. \uadf8\ub798\uc11c \u300a\ud0dc\uc591\uc758 \ud6c4\uc608\u300b\uc5d0\ub294 \"\uc27d\uac8c \ub118\ubcf4\uae30 \ud798\ub4e0 \uba85\ub791\ud568\uc774 \uac00\ub4dd\ud558\ub2e4.\" \ub0a8\uad81\uc778\uc740 \ud0dc\uc591\uc758 \ud6c4\uc608\uac00 \ub300\uc911\uc758 \"\uc695\ub9dd\uc758 \ud604\uc7ac \uc8fc\uc18c\"\ub97c \ud3ec\ucc29\ud55c \ub4a4 \uc2dc\ub3c4\ud55c \"\ub098\ub984\ub300\ub85c\uc758 \uc791\ud488\uc138\uacc4\uc640 \uc791\ud654 \ubc29\ubc95\"\uc774 \uc628 \uc138\uacc4\ub97c \"\uc9c0\ubc30\ud560 \uc815\ub3c4\uc758 \uc7a5\ub974\ud654\ub41c \ud558\ub098\uc758 \ud604\uc0c1\"\uc774 \ub418\uc5c8\ub2e4\uace0 \uacb0\ub860\uc9c0\uc5c8\ub2e4."} {"question": "\ub098\uce74\ubaa8\ub9ac \uc544\ud0a4\ub098\ub294 \uc77c\ubcf8\uc758 \uc5b4\ub290\ub9cc\ud654\uc758 \uce90\ub9ad\ud130 \ubaa8\ub378\uc774 \ub418\uc5c8\ub098?", "paragraph": "\ub300\ud55c\ubbfc\uad6d\uc758 \uacbd\uc6b0 \ub2f9\uc2dc \uc77c\ubcf8 \ubb38\ud654\ub97c \ud5c8\uc6a9\ud558\uc9c0 \uc54a\uc558\ub358 \ud130\ub77c \uc9c1\uc811\uc801\uc778 \uc601\ud5a5\uc740 \ub07c\uce5c \uc801\uc774 \uc5c6\uc5c8\uc9c0\ub9cc, 1980\ub144\ub300 \ud6c4\ubc18 \uc74c\uc545\uacc4\uc5d0\ub294 \ub9c8\uce20\ub2e4 \uc138\uc774\ucf54\uc640 \ud568\uaed8 \uac04\uc811\uc801\uc778 \uc601\ud5a5\uc744 \uc8fc\uac8c \ub41c\ub2e4. \uc74c\ubc18 \uc81c\uc791\uc790\ub4e4\uc774 \ub098\uce74\ubaa8\ub9ac\uc640 \ub9c8\uce20\ub2e4 \ub4f1\uc744 \ubca4\uce58\ub9c8\ud0b9\ud558\uc5ec, \ub300\ud55c\ubbfc\uad6d\uc758 \uc8fc\ub958 \uac00\uc694 \uc2dc\uc7a5\uc5d0 \uadf8 \uc774\ubbf8\uc9c0\ub97c \uc815\ucc29\uc2dc\ucf30\ub2e4. \ub85d\uadf8\ub8f9 \ubc31\ub450\uc0b0\uc758 \ub9ac\ub354\uc778 \uc720\ud604\uc0c1\uc740 \uc77c\ubcf8\uc5d0\uc11c \ub098\uce74\ubaa8\ub9ac \uc544\ud0a4\ub098\uac00 \uacf5\uc5f0\ud55c \ub77c\uc774\ube0c \ucf58\uc11c\ud2b8\ub97c \ubcf4\uace0 \uc601\uac10\uc744 \ubc1b\uc544 \uc5f0\uc608\uc0ac\ubb34\uc18c\ub97c \uc124\ub9bd, \ucc98\uc74c\uc73c\ub85c \uc591\uc131\ud55c \uac00\uc218\uac00 \ubc14\ub85c \uc774\uc9c0\uc5f0\uc774\uc5c8\ub2e4\uace0 \uc220\ud68c\ud558\uc600\ub2e4. \uadf8\ub7ec\ub098 \ub300\ud55c\ubbfc\uad6d\uc758 \uac00\uc218 \ubbfc\ud574\uacbd\uc740 \ub098\uce74\ubaa8\ub9ac\uc758 \uace1\uc744 \ud45c\uc808\ud588\ub2e4\ub294 \uc758\ud639\uc744 \ubc1b\uace0 \uc788\ub294\ub370, \ubbfc\ud574\uacbd\uc758 \uace1 \u3008\ubbf8\ub2c8\uc2a4\ucee4\ud2b8\u3009\uc758 \uc804\uc8fc\uc640 \uac04\uc8fc\ub294 \u3008BLONDE\u3009\uc758 \uadf8\uac83\uacfc \ub9e4\uc6b0 \ud761\uc0ac\ud558\uace0, \uc7ac\ud0b7 \uc0ac\uc9c4 \uc5ed\uc2dc \ub098\uce74\ubaa8\ub9ac\uac00 \u3008BLONDE\u3009 \ub54c \uc785\uc5c8\ub358 \uac83\uacfc \ud761\uc0ac\ud55c \uc758\uc0c1\uc744 \uc785\uace0 \ucc0d\uae30\ub3c4 \ud588\ub2e4. \ubbfc\ud574\uacbd\uc758 \u3008\ubc14\ub78c\uc774 \ubd88\uc5b4\uc624\uba74\u3009\uc758 \uac04\uc8fc \uc5ed\uc2dc \u3008\ub09c\ud30c\uc120\u3009\uc758 \uac04\uc8fc\uc640 \ub9e4\uc6b0 \ube44\uc2b7\ud558\uc5ec \ub17c\ub780\uc774 \uc77c\uc5c8\ub2e4. \ub610\ud55c \ub098\uce74\ubaa8\ub9ac \uc544\ud0a4\ub098\ub294 \uc77c\ubcf8\uc758 \ub9cc\ud654 \u300a\uc0c8\ucf64\ub2ec\ucf64 \uc624\ub80c\uc9c0 \ub85c\ub4dc\u300b\uc758 \uc5ec\uc8fc\uc778\uacf5 \uc544\uc720\uce74\uc640 \ub9c8\ub3c4\uce74(\u9b8e\u5ddd\u307e\u3069\u304b)\uc758 \ubaa8\ub378\uc774\uae30\ub3c4 \ud588\ub294\ub370 \uadf8\uc911\uc5d0\uc11c\ub3c4 \ucd08\ucc3d\uae30\uc758 \ubd88\ub7c9\uccad\uc18c\ub144 \ucf58\uc149\ud2b8\uc758 \ubaa8\uc2b5\uc774 \ubaa8\ud2f0\ube0c\uac00 \ub418\ub294 \ub4f1 \uc11c\ube0c\uceec\ucc98\uacc4\uc5d0\uc11c\ub3c4 \ubc18\ud5a5\uc744 \uc77c\uc73c\ucf30\ub2e4.", "answer": "\uc0c8\ucf64\ub2ec\ucf64 \uc624\ub80c\uc9c0 \ub85c\ub4dc", "sentence": "\ub610\ud55c \ub098\uce74\ubaa8\ub9ac \uc544\ud0a4\ub098\ub294 \uc77c\ubcf8\uc758 \ub9cc\ud654 \u300a \uc0c8\ucf64\ub2ec\ucf64 \uc624\ub80c\uc9c0 \ub85c\ub4dc \u300b\uc758 \uc5ec\uc8fc\uc778\uacf5 \uc544\uc720\uce74\uc640 \ub9c8\ub3c4\uce74(\u9b8e\u5ddd\u307e\u3069\u304b)\uc758 \ubaa8\ub378\uc774\uae30\ub3c4 \ud588\ub294\ub370 \uadf8\uc911\uc5d0\uc11c\ub3c4 \ucd08\ucc3d\uae30\uc758 \ubd88\ub7c9\uccad\uc18c\ub144 \ucf58\uc149\ud2b8\uc758 \ubaa8\uc2b5\uc774 \ubaa8\ud2f0\ube0c\uac00 \ub418\ub294 \ub4f1 \uc11c\ube0c\uceec\ucc98\uacc4\uc5d0\uc11c\ub3c4 \ubc18\ud5a5\uc744 \uc77c\uc73c\ucf30\ub2e4.", "paragraph_sentence": "\ub300\ud55c\ubbfc\uad6d\uc758 \uacbd\uc6b0 \ub2f9\uc2dc \uc77c\ubcf8 \ubb38\ud654\ub97c \ud5c8\uc6a9\ud558\uc9c0 \uc54a\uc558\ub358 \ud130\ub77c \uc9c1\uc811\uc801\uc778 \uc601\ud5a5\uc740 \ub07c\uce5c \uc801\uc774 \uc5c6\uc5c8\uc9c0\ub9cc, 1980\ub144\ub300 \ud6c4\ubc18 \uc74c\uc545\uacc4\uc5d0\ub294 \ub9c8\uce20\ub2e4 \uc138\uc774\ucf54\uc640 \ud568\uaed8 \uac04\uc811\uc801\uc778 \uc601\ud5a5\uc744 \uc8fc\uac8c \ub41c\ub2e4. \uc74c\ubc18 \uc81c\uc791\uc790\ub4e4\uc774 \ub098\uce74\ubaa8\ub9ac\uc640 \ub9c8\uce20\ub2e4 \ub4f1\uc744 \ubca4\uce58\ub9c8\ud0b9\ud558\uc5ec, \ub300\ud55c\ubbfc\uad6d\uc758 \uc8fc\ub958 \uac00\uc694 \uc2dc\uc7a5\uc5d0 \uadf8 \uc774\ubbf8\uc9c0\ub97c \uc815\ucc29\uc2dc\ucf30\ub2e4. \ub85d\uadf8\ub8f9 \ubc31\ub450\uc0b0\uc758 \ub9ac\ub354\uc778 \uc720\ud604\uc0c1\uc740 \uc77c\ubcf8\uc5d0\uc11c \ub098\uce74\ubaa8\ub9ac \uc544\ud0a4\ub098\uac00 \uacf5\uc5f0\ud55c \ub77c\uc774\ube0c \ucf58\uc11c\ud2b8\ub97c \ubcf4\uace0 \uc601\uac10\uc744 \ubc1b\uc544 \uc5f0\uc608\uc0ac\ubb34\uc18c\ub97c \uc124\ub9bd, \ucc98\uc74c\uc73c\ub85c \uc591\uc131\ud55c \uac00\uc218\uac00 \ubc14\ub85c \uc774\uc9c0\uc5f0\uc774\uc5c8\ub2e4\uace0 \uc220\ud68c\ud558\uc600\ub2e4. \uadf8\ub7ec\ub098 \ub300\ud55c\ubbfc\uad6d\uc758 \uac00\uc218 \ubbfc\ud574\uacbd\uc740 \ub098\uce74\ubaa8\ub9ac\uc758 \uace1\uc744 \ud45c\uc808\ud588\ub2e4\ub294 \uc758\ud639\uc744 \ubc1b\uace0 \uc788\ub294\ub370, \ubbfc\ud574\uacbd\uc758 \uace1 \u3008\ubbf8\ub2c8\uc2a4\ucee4\ud2b8\u3009\uc758 \uc804\uc8fc\uc640 \uac04\uc8fc\ub294 \u3008BLONDE\u3009\uc758 \uadf8\uac83\uacfc \ub9e4\uc6b0 \ud761\uc0ac\ud558\uace0, \uc7ac\ud0b7 \uc0ac\uc9c4 \uc5ed\uc2dc \ub098\uce74\ubaa8\ub9ac\uac00 \u3008BLONDE\u3009 \ub54c \uc785\uc5c8\ub358 \uac83\uacfc \ud761\uc0ac\ud55c \uc758\uc0c1\uc744 \uc785\uace0 \ucc0d\uae30\ub3c4 \ud588\ub2e4. \ubbfc\ud574\uacbd\uc758 \u3008\ubc14\ub78c\uc774 \ubd88\uc5b4\uc624\uba74\u3009\uc758 \uac04\uc8fc \uc5ed\uc2dc \u3008\ub09c\ud30c\uc120\u3009\uc758 \uac04\uc8fc\uc640 \ub9e4\uc6b0 \ube44\uc2b7\ud558\uc5ec \ub17c\ub780\uc774 \uc77c\uc5c8\ub2e4. \ub610\ud55c \ub098\uce74\ubaa8\ub9ac \uc544\ud0a4\ub098\ub294 \uc77c\ubcf8\uc758 \ub9cc\ud654 \u300a \uc0c8\ucf64\ub2ec\ucf64 \uc624\ub80c\uc9c0 \ub85c\ub4dc \u300b\uc758 \uc5ec\uc8fc\uc778\uacf5 \uc544\uc720\uce74\uc640 \ub9c8\ub3c4\uce74(\u9b8e\u5ddd\u307e\u3069\u304b)\uc758 \ubaa8\ub378\uc774\uae30\ub3c4 \ud588\ub294\ub370 \uadf8\uc911\uc5d0\uc11c\ub3c4 \ucd08\ucc3d\uae30\uc758 \ubd88\ub7c9\uccad\uc18c\ub144 \ucf58\uc149\ud2b8\uc758 \ubaa8\uc2b5\uc774 \ubaa8\ud2f0\ube0c\uac00 \ub418\ub294 \ub4f1 \uc11c\ube0c\uceec\ucc98\uacc4\uc5d0\uc11c\ub3c4 \ubc18\ud5a5\uc744 \uc77c\uc73c\ucf30\ub2e4. ", "paragraph_answer": "\ub300\ud55c\ubbfc\uad6d\uc758 \uacbd\uc6b0 \ub2f9\uc2dc \uc77c\ubcf8 \ubb38\ud654\ub97c \ud5c8\uc6a9\ud558\uc9c0 \uc54a\uc558\ub358 \ud130\ub77c \uc9c1\uc811\uc801\uc778 \uc601\ud5a5\uc740 \ub07c\uce5c \uc801\uc774 \uc5c6\uc5c8\uc9c0\ub9cc, 1980\ub144\ub300 \ud6c4\ubc18 \uc74c\uc545\uacc4\uc5d0\ub294 \ub9c8\uce20\ub2e4 \uc138\uc774\ucf54\uc640 \ud568\uaed8 \uac04\uc811\uc801\uc778 \uc601\ud5a5\uc744 \uc8fc\uac8c \ub41c\ub2e4. \uc74c\ubc18 \uc81c\uc791\uc790\ub4e4\uc774 \ub098\uce74\ubaa8\ub9ac\uc640 \ub9c8\uce20\ub2e4 \ub4f1\uc744 \ubca4\uce58\ub9c8\ud0b9\ud558\uc5ec, \ub300\ud55c\ubbfc\uad6d\uc758 \uc8fc\ub958 \uac00\uc694 \uc2dc\uc7a5\uc5d0 \uadf8 \uc774\ubbf8\uc9c0\ub97c \uc815\ucc29\uc2dc\ucf30\ub2e4. \ub85d\uadf8\ub8f9 \ubc31\ub450\uc0b0\uc758 \ub9ac\ub354\uc778 \uc720\ud604\uc0c1\uc740 \uc77c\ubcf8\uc5d0\uc11c \ub098\uce74\ubaa8\ub9ac \uc544\ud0a4\ub098\uac00 \uacf5\uc5f0\ud55c \ub77c\uc774\ube0c \ucf58\uc11c\ud2b8\ub97c \ubcf4\uace0 \uc601\uac10\uc744 \ubc1b\uc544 \uc5f0\uc608\uc0ac\ubb34\uc18c\ub97c \uc124\ub9bd, \ucc98\uc74c\uc73c\ub85c \uc591\uc131\ud55c \uac00\uc218\uac00 \ubc14\ub85c \uc774\uc9c0\uc5f0\uc774\uc5c8\ub2e4\uace0 \uc220\ud68c\ud558\uc600\ub2e4. \uadf8\ub7ec\ub098 \ub300\ud55c\ubbfc\uad6d\uc758 \uac00\uc218 \ubbfc\ud574\uacbd\uc740 \ub098\uce74\ubaa8\ub9ac\uc758 \uace1\uc744 \ud45c\uc808\ud588\ub2e4\ub294 \uc758\ud639\uc744 \ubc1b\uace0 \uc788\ub294\ub370, \ubbfc\ud574\uacbd\uc758 \uace1 \u3008\ubbf8\ub2c8\uc2a4\ucee4\ud2b8\u3009\uc758 \uc804\uc8fc\uc640 \uac04\uc8fc\ub294 \u3008BLONDE\u3009\uc758 \uadf8\uac83\uacfc \ub9e4\uc6b0 \ud761\uc0ac\ud558\uace0, \uc7ac\ud0b7 \uc0ac\uc9c4 \uc5ed\uc2dc \ub098\uce74\ubaa8\ub9ac\uac00 \u3008BLONDE\u3009 \ub54c \uc785\uc5c8\ub358 \uac83\uacfc \ud761\uc0ac\ud55c \uc758\uc0c1\uc744 \uc785\uace0 \ucc0d\uae30\ub3c4 \ud588\ub2e4. \ubbfc\ud574\uacbd\uc758 \u3008\ubc14\ub78c\uc774 \ubd88\uc5b4\uc624\uba74\u3009\uc758 \uac04\uc8fc \uc5ed\uc2dc \u3008\ub09c\ud30c\uc120\u3009\uc758 \uac04\uc8fc\uc640 \ub9e4\uc6b0 \ube44\uc2b7\ud558\uc5ec \ub17c\ub780\uc774 \uc77c\uc5c8\ub2e4. \ub610\ud55c \ub098\uce74\ubaa8\ub9ac \uc544\ud0a4\ub098\ub294 \uc77c\ubcf8\uc758 \ub9cc\ud654 \u300a \uc0c8\ucf64\ub2ec\ucf64 \uc624\ub80c\uc9c0 \ub85c\ub4dc \u300b\uc758 \uc5ec\uc8fc\uc778\uacf5 \uc544\uc720\uce74\uc640 \ub9c8\ub3c4\uce74(\u9b8e\u5ddd\u307e\u3069\u304b)\uc758 \ubaa8\ub378\uc774\uae30\ub3c4 \ud588\ub294\ub370 \uadf8\uc911\uc5d0\uc11c\ub3c4 \ucd08\ucc3d\uae30\uc758 \ubd88\ub7c9\uccad\uc18c\ub144 \ucf58\uc149\ud2b8\uc758 \ubaa8\uc2b5\uc774 \ubaa8\ud2f0\ube0c\uac00 \ub418\ub294 \ub4f1 \uc11c\ube0c\uceec\ucc98\uacc4\uc5d0\uc11c\ub3c4 \ubc18\ud5a5\uc744 \uc77c\uc73c\ucf30\ub2e4.", "sentence_answer": "\ub610\ud55c \ub098\uce74\ubaa8\ub9ac \uc544\ud0a4\ub098\ub294 \uc77c\ubcf8\uc758 \ub9cc\ud654 \u300a \uc0c8\ucf64\ub2ec\ucf64 \uc624\ub80c\uc9c0 \ub85c\ub4dc \u300b\uc758 \uc5ec\uc8fc\uc778\uacf5 \uc544\uc720\uce74\uc640 \ub9c8\ub3c4\uce74(\u9b8e\u5ddd\u307e\u3069\u304b)\uc758 \ubaa8\ub378\uc774\uae30\ub3c4 \ud588\ub294\ub370 \uadf8\uc911\uc5d0\uc11c\ub3c4 \ucd08\ucc3d\uae30\uc758 \ubd88\ub7c9\uccad\uc18c\ub144 \ucf58\uc149\ud2b8\uc758 \ubaa8\uc2b5\uc774 \ubaa8\ud2f0\ube0c\uac00 \ub418\ub294 \ub4f1 \uc11c\ube0c\uceec\ucc98\uacc4\uc5d0\uc11c\ub3c4 \ubc18\ud5a5\uc744 \uc77c\uc73c\ucf30\ub2e4.", "paragraph_id": "6546040-35-2", "paragraph_question": "question: \ub098\uce74\ubaa8\ub9ac \uc544\ud0a4\ub098\ub294 \uc77c\ubcf8\uc758 \uc5b4\ub290\ub9cc\ud654\uc758 \uce90\ub9ad\ud130 \ubaa8\ub378\uc774 \ub418\uc5c8\ub098?, context: \ub300\ud55c\ubbfc\uad6d\uc758 \uacbd\uc6b0 \ub2f9\uc2dc \uc77c\ubcf8 \ubb38\ud654\ub97c \ud5c8\uc6a9\ud558\uc9c0 \uc54a\uc558\ub358 \ud130\ub77c \uc9c1\uc811\uc801\uc778 \uc601\ud5a5\uc740 \ub07c\uce5c \uc801\uc774 \uc5c6\uc5c8\uc9c0\ub9cc, 1980\ub144\ub300 \ud6c4\ubc18 \uc74c\uc545\uacc4\uc5d0\ub294 \ub9c8\uce20\ub2e4 \uc138\uc774\ucf54\uc640 \ud568\uaed8 \uac04\uc811\uc801\uc778 \uc601\ud5a5\uc744 \uc8fc\uac8c \ub41c\ub2e4. \uc74c\ubc18 \uc81c\uc791\uc790\ub4e4\uc774 \ub098\uce74\ubaa8\ub9ac\uc640 \ub9c8\uce20\ub2e4 \ub4f1\uc744 \ubca4\uce58\ub9c8\ud0b9\ud558\uc5ec, \ub300\ud55c\ubbfc\uad6d\uc758 \uc8fc\ub958 \uac00\uc694 \uc2dc\uc7a5\uc5d0 \uadf8 \uc774\ubbf8\uc9c0\ub97c \uc815\ucc29\uc2dc\ucf30\ub2e4. \ub85d\uadf8\ub8f9 \ubc31\ub450\uc0b0\uc758 \ub9ac\ub354\uc778 \uc720\ud604\uc0c1\uc740 \uc77c\ubcf8\uc5d0\uc11c \ub098\uce74\ubaa8\ub9ac \uc544\ud0a4\ub098\uac00 \uacf5\uc5f0\ud55c \ub77c\uc774\ube0c \ucf58\uc11c\ud2b8\ub97c \ubcf4\uace0 \uc601\uac10\uc744 \ubc1b\uc544 \uc5f0\uc608\uc0ac\ubb34\uc18c\ub97c \uc124\ub9bd, \ucc98\uc74c\uc73c\ub85c \uc591\uc131\ud55c \uac00\uc218\uac00 \ubc14\ub85c \uc774\uc9c0\uc5f0\uc774\uc5c8\ub2e4\uace0 \uc220\ud68c\ud558\uc600\ub2e4. \uadf8\ub7ec\ub098 \ub300\ud55c\ubbfc\uad6d\uc758 \uac00\uc218 \ubbfc\ud574\uacbd\uc740 \ub098\uce74\ubaa8\ub9ac\uc758 \uace1\uc744 \ud45c\uc808\ud588\ub2e4\ub294 \uc758\ud639\uc744 \ubc1b\uace0 \uc788\ub294\ub370, \ubbfc\ud574\uacbd\uc758 \uace1 \u3008\ubbf8\ub2c8\uc2a4\ucee4\ud2b8\u3009\uc758 \uc804\uc8fc\uc640 \uac04\uc8fc\ub294 \u3008BLONDE\u3009\uc758 \uadf8\uac83\uacfc \ub9e4\uc6b0 \ud761\uc0ac\ud558\uace0, \uc7ac\ud0b7 \uc0ac\uc9c4 \uc5ed\uc2dc \ub098\uce74\ubaa8\ub9ac\uac00 \u3008BLONDE\u3009 \ub54c \uc785\uc5c8\ub358 \uac83\uacfc \ud761\uc0ac\ud55c \uc758\uc0c1\uc744 \uc785\uace0 \ucc0d\uae30\ub3c4 \ud588\ub2e4. \ubbfc\ud574\uacbd\uc758 \u3008\ubc14\ub78c\uc774 \ubd88\uc5b4\uc624\uba74\u3009\uc758 \uac04\uc8fc \uc5ed\uc2dc \u3008\ub09c\ud30c\uc120\u3009\uc758 \uac04\uc8fc\uc640 \ub9e4\uc6b0 \ube44\uc2b7\ud558\uc5ec \ub17c\ub780\uc774 \uc77c\uc5c8\ub2e4. \ub610\ud55c \ub098\uce74\ubaa8\ub9ac \uc544\ud0a4\ub098\ub294 \uc77c\ubcf8\uc758 \ub9cc\ud654 \u300a\uc0c8\ucf64\ub2ec\ucf64 \uc624\ub80c\uc9c0 \ub85c\ub4dc\u300b\uc758 \uc5ec\uc8fc\uc778\uacf5 \uc544\uc720\uce74\uc640 \ub9c8\ub3c4\uce74(\u9b8e\u5ddd\u307e\u3069\u304b)\uc758 \ubaa8\ub378\uc774\uae30\ub3c4 \ud588\ub294\ub370 \uadf8\uc911\uc5d0\uc11c\ub3c4 \ucd08\ucc3d\uae30\uc758 \ubd88\ub7c9\uccad\uc18c\ub144 \ucf58\uc149\ud2b8\uc758 \ubaa8\uc2b5\uc774 \ubaa8\ud2f0\ube0c\uac00 \ub418\ub294 \ub4f1 \uc11c\ube0c\uceec\ucc98\uacc4\uc5d0\uc11c\ub3c4 \ubc18\ud5a5\uc744 \uc77c\uc73c\ucf30\ub2e4."} {"question": "\ubcf4\uc704\uac00 \ube0c\ub86c\ub9ac \uae30\uc220 \uace0\ub4f1\ud559\uad50\ub97c \uc785\ud559\ud558\uae30 \uc804 \uce58\ub800\ub358 \uc2dc\ud5d8\uc740?", "paragraph": "\ud504\ub798\uc2ac\ub9ac \ub610\ud55c \uac19\uc740 \ucda9\uaca9\uc744 \uc804\ud588\ub2e4. \uadf8\ub294 \"\uc804 \uc0ac\ucd0c\uc774 \u3008Hound Dog\u3009\uc744 \ud2c0\uba70 \ub9c8\uc778 \ub304\uc2a4\ub97c \ucd94\ub294 \uac83\uc744 \ubaa9\uaca9\ud588\uc2b5\ub2c8\ub2e4. \uc804 \uadf8\ub140\uac00 \uc77c\uc5b4\ub098\uc11c \uadf8\ub7f0 \ucda4\uc744 \ucd94\ub294 \uac83\uc740 \ud55c \ubc88\ub3c4 \ubcf4\uc9c0 \ubabb\ud588\uc5b4\uc694. \uadf8 \uc74c\uc545\uc774 \ub0b4\ud3ec\ud55c \ud798\uc5d0 \uc815\ub9d0 \ucda9\uaca9\uc744 \ubc1b\uc558\uc2b5\ub2c8\ub2e4. \uc774\ud6c4 \ubc14\ub85c \uc74c\ubc18\uc744 \uad6c\uc785\ud588\uc5b4\uc694.\"\ub77c\uace0 \ud68c\uc0c1\ud588\ub2e4. \ub2e4\uc74c\ud574 \ub9d0\uc5d0 \uadf8\ub294 \uc6b0\ucfe0\ub810\ub808\uc640 \ucc28\uc0c1\uc790 \ubca0\uc774\uc2a4\ub97c \uac00\uc838\uc640 \uce5c\uad6c\ub4e4\uacfc \uc2a4\ud0a4\ud50c \uc138\uc158\uc744 \uac00\uc84c\uc73c\uba70, \ud53c\uc544\ub178 \uc5f0\uc8fc\ub97c \uc2dc\uc791\ud588\ub2e4. \ud55c\ud3b8, \ub3d9\ub124 \uc6b8\ud504 \ud074\ub7fd\uc5d0\uc11c \ud504\ub808\uc2ac\ub9ac\uc640 \ucc99 \ubca0\ub9ac\uc5d0\uac8c \ubc14\uce58\ub294 \uc218\ub9ce\uc740 \uacf5\uc5f0\uc744 \ud3bc\uce5c \uadf8\ub97c \ubcf8 \uc0ac\ub78c\ub4e4\uc740 \"\ub10b\uc744 \ube7c\ub193\uc558\ub2e4 ... \ub9c8\uce58 \ub2e4\ub978 \ud589\uc131\uc5d0\uc11c \uc628 \uc0ac\ub78c \uac19\uc558\ub2e4.\"\uace0 \ud45c\ud604\ud588\ub2e4. \uc77c\ub808\ube10 \ud50c\ub7ec\uc2a4 \uc2dc\ud5d8\uc744 \uce5c \ub4a4 \ubcf4\uc704\ub294 \ube0c\ub86c\ub9ac \uae30\uc220 \uace0\ub4f1\ud559\uad50\uc5d0 \uc785\ud559\ud588\ub2e4. \uc804\uae30 \uc791\uac00 \ud06c\ub9ac\uc2a4\ud1a0\ud37c \uc0cc\ub4dc \ud3ec\ub4dc\uc5d0 \uc758\ud558\uba74 \uc774 \ud559\uad50\ub294 \uae30\uc220 \ud559\uad50 \uce58\uace0\ub294 \uc774\uc0c1\ud588\ub2e4.", "answer": "\uc77c\ub808\ube10 \ud50c\ub7ec\uc2a4 \uc2dc\ud5d8", "sentence": "\uc77c\ub808\ube10 \ud50c\ub7ec\uc2a4 \uc2dc\ud5d8 \uc744 \uce5c \ub4a4 \ubcf4\uc704\ub294 \ube0c\ub86c\ub9ac \uae30\uc220 \uace0\ub4f1\ud559\uad50\uc5d0 \uc785\ud559\ud588\ub2e4.", "paragraph_sentence": "\ud504\ub798\uc2ac\ub9ac \ub610\ud55c \uac19\uc740 \ucda9\uaca9\uc744 \uc804\ud588\ub2e4. \uadf8\ub294 \"\uc804 \uc0ac\ucd0c\uc774 \u3008Hound Dog\u3009\uc744 \ud2c0\uba70 \ub9c8\uc778 \ub304\uc2a4\ub97c \ucd94\ub294 \uac83\uc744 \ubaa9\uaca9\ud588\uc2b5\ub2c8\ub2e4. \uc804 \uadf8\ub140\uac00 \uc77c\uc5b4\ub098\uc11c \uadf8\ub7f0 \ucda4\uc744 \ucd94\ub294 \uac83\uc740 \ud55c \ubc88\ub3c4 \ubcf4\uc9c0 \ubabb\ud588\uc5b4\uc694. \uadf8 \uc74c\uc545\uc774 \ub0b4\ud3ec\ud55c \ud798\uc5d0 \uc815\ub9d0 \ucda9\uaca9\uc744 \ubc1b\uc558\uc2b5\ub2c8\ub2e4. \uc774\ud6c4 \ubc14\ub85c \uc74c\ubc18\uc744 \uad6c\uc785\ud588\uc5b4\uc694. \"\ub77c\uace0 \ud68c\uc0c1\ud588\ub2e4. \ub2e4\uc74c\ud574 \ub9d0\uc5d0 \uadf8\ub294 \uc6b0\ucfe0\ub810\ub808\uc640 \ucc28\uc0c1\uc790 \ubca0\uc774\uc2a4\ub97c \uac00\uc838\uc640 \uce5c\uad6c\ub4e4\uacfc \uc2a4\ud0a4\ud50c \uc138\uc158\uc744 \uac00\uc84c\uc73c\uba70, \ud53c\uc544\ub178 \uc5f0\uc8fc\ub97c \uc2dc\uc791\ud588\ub2e4. \ud55c\ud3b8, \ub3d9\ub124 \uc6b8\ud504 \ud074\ub7fd\uc5d0\uc11c \ud504\ub808\uc2ac\ub9ac\uc640 \ucc99 \ubca0\ub9ac\uc5d0\uac8c \ubc14\uce58\ub294 \uc218\ub9ce\uc740 \uacf5\uc5f0\uc744 \ud3bc\uce5c \uadf8\ub97c \ubcf8 \uc0ac\ub78c\ub4e4\uc740 \"\ub10b\uc744 \ube7c\ub193\uc558\ub2e4 ... \ub9c8\uce58 \ub2e4\ub978 \ud589\uc131\uc5d0\uc11c \uc628 \uc0ac\ub78c \uac19\uc558\ub2e4. \"\uace0 \ud45c\ud604\ud588\ub2e4. \uc77c\ub808\ube10 \ud50c\ub7ec\uc2a4 \uc2dc\ud5d8 \uc744 \uce5c \ub4a4 \ubcf4\uc704\ub294 \ube0c\ub86c\ub9ac \uae30\uc220 \uace0\ub4f1\ud559\uad50\uc5d0 \uc785\ud559\ud588\ub2e4. \uc804\uae30 \uc791\uac00 \ud06c\ub9ac\uc2a4\ud1a0\ud37c \uc0cc\ub4dc \ud3ec\ub4dc\uc5d0 \uc758\ud558\uba74 \uc774 \ud559\uad50\ub294 \uae30\uc220 \ud559\uad50 \uce58\uace0\ub294 \uc774\uc0c1\ud588\ub2e4.", "paragraph_answer": "\ud504\ub798\uc2ac\ub9ac \ub610\ud55c \uac19\uc740 \ucda9\uaca9\uc744 \uc804\ud588\ub2e4. \uadf8\ub294 \"\uc804 \uc0ac\ucd0c\uc774 \u3008Hound Dog\u3009\uc744 \ud2c0\uba70 \ub9c8\uc778 \ub304\uc2a4\ub97c \ucd94\ub294 \uac83\uc744 \ubaa9\uaca9\ud588\uc2b5\ub2c8\ub2e4. \uc804 \uadf8\ub140\uac00 \uc77c\uc5b4\ub098\uc11c \uadf8\ub7f0 \ucda4\uc744 \ucd94\ub294 \uac83\uc740 \ud55c \ubc88\ub3c4 \ubcf4\uc9c0 \ubabb\ud588\uc5b4\uc694. \uadf8 \uc74c\uc545\uc774 \ub0b4\ud3ec\ud55c \ud798\uc5d0 \uc815\ub9d0 \ucda9\uaca9\uc744 \ubc1b\uc558\uc2b5\ub2c8\ub2e4. \uc774\ud6c4 \ubc14\ub85c \uc74c\ubc18\uc744 \uad6c\uc785\ud588\uc5b4\uc694.\"\ub77c\uace0 \ud68c\uc0c1\ud588\ub2e4. \ub2e4\uc74c\ud574 \ub9d0\uc5d0 \uadf8\ub294 \uc6b0\ucfe0\ub810\ub808\uc640 \ucc28\uc0c1\uc790 \ubca0\uc774\uc2a4\ub97c \uac00\uc838\uc640 \uce5c\uad6c\ub4e4\uacfc \uc2a4\ud0a4\ud50c \uc138\uc158\uc744 \uac00\uc84c\uc73c\uba70, \ud53c\uc544\ub178 \uc5f0\uc8fc\ub97c \uc2dc\uc791\ud588\ub2e4. \ud55c\ud3b8, \ub3d9\ub124 \uc6b8\ud504 \ud074\ub7fd\uc5d0\uc11c \ud504\ub808\uc2ac\ub9ac\uc640 \ucc99 \ubca0\ub9ac\uc5d0\uac8c \ubc14\uce58\ub294 \uc218\ub9ce\uc740 \uacf5\uc5f0\uc744 \ud3bc\uce5c \uadf8\ub97c \ubcf8 \uc0ac\ub78c\ub4e4\uc740 \"\ub10b\uc744 \ube7c\ub193\uc558\ub2e4 ... \ub9c8\uce58 \ub2e4\ub978 \ud589\uc131\uc5d0\uc11c \uc628 \uc0ac\ub78c \uac19\uc558\ub2e4.\"\uace0 \ud45c\ud604\ud588\ub2e4. \uc77c\ub808\ube10 \ud50c\ub7ec\uc2a4 \uc2dc\ud5d8 \uc744 \uce5c \ub4a4 \ubcf4\uc704\ub294 \ube0c\ub86c\ub9ac \uae30\uc220 \uace0\ub4f1\ud559\uad50\uc5d0 \uc785\ud559\ud588\ub2e4. \uc804\uae30 \uc791\uac00 \ud06c\ub9ac\uc2a4\ud1a0\ud37c \uc0cc\ub4dc \ud3ec\ub4dc\uc5d0 \uc758\ud558\uba74 \uc774 \ud559\uad50\ub294 \uae30\uc220 \ud559\uad50 \uce58\uace0\ub294 \uc774\uc0c1\ud588\ub2e4.", "sentence_answer": " \uc77c\ub808\ube10 \ud50c\ub7ec\uc2a4 \uc2dc\ud5d8 \uc744 \uce5c \ub4a4 \ubcf4\uc704\ub294 \ube0c\ub86c\ub9ac \uae30\uc220 \uace0\ub4f1\ud559\uad50\uc5d0 \uc785\ud559\ud588\ub2e4.", "paragraph_id": "6578410-4-1", "paragraph_question": "question: \ubcf4\uc704\uac00 \ube0c\ub86c\ub9ac \uae30\uc220 \uace0\ub4f1\ud559\uad50\ub97c \uc785\ud559\ud558\uae30 \uc804 \uce58\ub800\ub358 \uc2dc\ud5d8\uc740?, context: \ud504\ub798\uc2ac\ub9ac \ub610\ud55c \uac19\uc740 \ucda9\uaca9\uc744 \uc804\ud588\ub2e4. \uadf8\ub294 \"\uc804 \uc0ac\ucd0c\uc774 \u3008Hound Dog\u3009\uc744 \ud2c0\uba70 \ub9c8\uc778 \ub304\uc2a4\ub97c \ucd94\ub294 \uac83\uc744 \ubaa9\uaca9\ud588\uc2b5\ub2c8\ub2e4. \uc804 \uadf8\ub140\uac00 \uc77c\uc5b4\ub098\uc11c \uadf8\ub7f0 \ucda4\uc744 \ucd94\ub294 \uac83\uc740 \ud55c \ubc88\ub3c4 \ubcf4\uc9c0 \ubabb\ud588\uc5b4\uc694. \uadf8 \uc74c\uc545\uc774 \ub0b4\ud3ec\ud55c \ud798\uc5d0 \uc815\ub9d0 \ucda9\uaca9\uc744 \ubc1b\uc558\uc2b5\ub2c8\ub2e4. \uc774\ud6c4 \ubc14\ub85c \uc74c\ubc18\uc744 \uad6c\uc785\ud588\uc5b4\uc694.\"\ub77c\uace0 \ud68c\uc0c1\ud588\ub2e4. \ub2e4\uc74c\ud574 \ub9d0\uc5d0 \uadf8\ub294 \uc6b0\ucfe0\ub810\ub808\uc640 \ucc28\uc0c1\uc790 \ubca0\uc774\uc2a4\ub97c \uac00\uc838\uc640 \uce5c\uad6c\ub4e4\uacfc \uc2a4\ud0a4\ud50c \uc138\uc158\uc744 \uac00\uc84c\uc73c\uba70, \ud53c\uc544\ub178 \uc5f0\uc8fc\ub97c \uc2dc\uc791\ud588\ub2e4. \ud55c\ud3b8, \ub3d9\ub124 \uc6b8\ud504 \ud074\ub7fd\uc5d0\uc11c \ud504\ub808\uc2ac\ub9ac\uc640 \ucc99 \ubca0\ub9ac\uc5d0\uac8c \ubc14\uce58\ub294 \uc218\ub9ce\uc740 \uacf5\uc5f0\uc744 \ud3bc\uce5c \uadf8\ub97c \ubcf8 \uc0ac\ub78c\ub4e4\uc740 \"\ub10b\uc744 \ube7c\ub193\uc558\ub2e4 ... \ub9c8\uce58 \ub2e4\ub978 \ud589\uc131\uc5d0\uc11c \uc628 \uc0ac\ub78c \uac19\uc558\ub2e4.\"\uace0 \ud45c\ud604\ud588\ub2e4. \uc77c\ub808\ube10 \ud50c\ub7ec\uc2a4 \uc2dc\ud5d8\uc744 \uce5c \ub4a4 \ubcf4\uc704\ub294 \ube0c\ub86c\ub9ac \uae30\uc220 \uace0\ub4f1\ud559\uad50\uc5d0 \uc785\ud559\ud588\ub2e4. \uc804\uae30 \uc791\uac00 \ud06c\ub9ac\uc2a4\ud1a0\ud37c \uc0cc\ub4dc \ud3ec\ub4dc\uc5d0 \uc758\ud558\uba74 \uc774 \ud559\uad50\ub294 \uae30\uc220 \ud559\uad50 \uce58\uace0\ub294 \uc774\uc0c1\ud588\ub2e4."} {"question": "\uc5b8\ub518\uc740 \ud574\uacbd\uc73c\ub85c\ubd80\ud130 \uc5bc\ub9c8\uc758 \ub3c8\uc744 \uc9c0\uae09\ubc1b\uc558\ub294\uac00?", "paragraph": "\ud574\uacbd\uacfc \uc5b8\ub518\uc740 4\ub144 \uc804 \ucc9c\uc548\ud568 \uc0ac\uac74 \ub54c\uc5d0\ub3c4 \uc218\uc0c9\uc791\uc5c5\uc744 \ud568\uaed8 \uc218\ud589\ud55c \uc778\uc5f0\uc774 \uc788\ub2e4. \ud574\uacbd\uc740 2010\ub144 4\uc6d4 \ucc9c\uc548\ud568 \uc2e4\uc885\uc7a5\ubcd1 \uc218\uc0c9\uc791\uc5c5\uc5d0 \ucc38\uc5ec\ud588\ub2e4\uac00 \uce68\ubab0\ud55c \uc30d\ub04c\uc774 \uc5b4\uc120 \uae08\uc59198\ud638\uc758 \uc120\ub0b4 \uc218\uc0c9\uc5c5\uccb4\ub85c \uc5b8\ub518\uc744 \uc120\uc815\ud588\ub2e4. \uc5b8\ub518\uc740 \uc2dc\uc2e0 \uc218\uc0c9\uc791\uc5c5\uc5d0\uc11c \ubcc4\ub2e4\ub978 \uc131\uacfc\ub97c \uac70\ub450\uc9c0 \ubabb\ud588\uc9c0\ub9cc \uacc4\uc57d\uae08\uc561\uc758 10%\ub97c \uc81c\uc678\ud55c 4\uc5b5 3\ucc9c\ub9cc\uc6d0\uc744 \ud574\uacbd\uc73c\ub85c\ubd80\ud130 \uc9c0\uae09\ubc1b\uc558\ub2e4. \uadf8\ub7ec\ub098 \uc2e4\uc885\uc790 \uc2dc\uc2e0\uc744 \ub2e8 1\uad6c\ub3c4 \uc778\uc591\ud558\uc9c0 \ubabb\ud55c \uc5c5\uccb4\uc5d0 \uac70\uc561\uc758 \uc608\uc0b0\uc744 \uc9c0\uae09\ud55c \uac83\uc774 \uc801\uc808\ud588\ub294\uc9c0\uc5d0 \ub300\ud55c \ub17c\ub780\uc740 \uc9c0\uae08\uae4c\uc9c0 \uacc4\uc18d\ub418\uace0 \uc788\ub2e4. \ud574\uacbd\uc740 \uc774\uc5d0 \ub300\ud574 \"\uae08\uc591\ud638 \uc218\uc0c9\uc791\uc5c5 \ub54c\ubb38\uc5d0 \uc5b8\ub518\uc774 \ub2e4\ub978 \ud604\uc7a5\uc758 \uc0ac\uc5c5\uc744 \uc9c4\ud589\ud558\uc9c0 \ubabb\ud55c \uc190\ud574\ub97c \ubd24\ub2e4\uace0 \uc8fc\uc7a5\ud588\uace0 \uacc4\uc57d \uc870\uac74\uc0c1 \uc2dc\uc2e0 \uc778\uc591\uacfc \uc0c1\uad00\uc5c6\uc774 \ub300\uae08\uc744 \uc9c0\uae09\ud560 \uc218\ubc16\uc5d0 \uc5c6\uc5c8\ub2e4\"\uace0 \ubc1d\ud614\ub2e4.", "answer": "4\uc5b5 3\ucc9c\ub9cc\uc6d0", "sentence": "\uc5b8\ub518\uc740 \uc2dc\uc2e0 \uc218\uc0c9\uc791\uc5c5\uc5d0\uc11c \ubcc4\ub2e4\ub978 \uc131\uacfc\ub97c \uac70\ub450\uc9c0 \ubabb\ud588\uc9c0\ub9cc \uacc4\uc57d\uae08\uc561\uc758 10%\ub97c \uc81c\uc678\ud55c 4\uc5b5 3\ucc9c\ub9cc\uc6d0 \uc744 \ud574\uacbd\uc73c\ub85c\ubd80\ud130 \uc9c0\uae09\ubc1b\uc558\ub2e4.", "paragraph_sentence": "\ud574\uacbd\uacfc \uc5b8\ub518\uc740 4\ub144 \uc804 \ucc9c\uc548\ud568 \uc0ac\uac74 \ub54c\uc5d0\ub3c4 \uc218\uc0c9\uc791\uc5c5\uc744 \ud568\uaed8 \uc218\ud589\ud55c \uc778\uc5f0\uc774 \uc788\ub2e4. \ud574\uacbd\uc740 2010\ub144 4\uc6d4 \ucc9c\uc548\ud568 \uc2e4\uc885\uc7a5\ubcd1 \uc218\uc0c9\uc791\uc5c5\uc5d0 \ucc38\uc5ec\ud588\ub2e4\uac00 \uce68\ubab0\ud55c \uc30d\ub04c\uc774 \uc5b4\uc120 \uae08\uc59198\ud638\uc758 \uc120\ub0b4 \uc218\uc0c9\uc5c5\uccb4\ub85c \uc5b8\ub518\uc744 \uc120\uc815\ud588\ub2e4. \uc5b8\ub518\uc740 \uc2dc\uc2e0 \uc218\uc0c9\uc791\uc5c5\uc5d0\uc11c \ubcc4\ub2e4\ub978 \uc131\uacfc\ub97c \uac70\ub450\uc9c0 \ubabb\ud588\uc9c0\ub9cc \uacc4\uc57d\uae08\uc561\uc758 10%\ub97c \uc81c\uc678\ud55c 4\uc5b5 3\ucc9c\ub9cc\uc6d0 \uc744 \ud574\uacbd\uc73c\ub85c\ubd80\ud130 \uc9c0\uae09\ubc1b\uc558\ub2e4. \uadf8\ub7ec\ub098 \uc2e4\uc885\uc790 \uc2dc\uc2e0\uc744 \ub2e8 1\uad6c\ub3c4 \uc778\uc591\ud558\uc9c0 \ubabb\ud55c \uc5c5\uccb4\uc5d0 \uac70\uc561\uc758 \uc608\uc0b0\uc744 \uc9c0\uae09\ud55c \uac83\uc774 \uc801\uc808\ud588\ub294\uc9c0\uc5d0 \ub300\ud55c \ub17c\ub780\uc740 \uc9c0\uae08\uae4c\uc9c0 \uacc4\uc18d\ub418\uace0 \uc788\ub2e4. \ud574\uacbd\uc740 \uc774\uc5d0 \ub300\ud574 \"\uae08\uc591\ud638 \uc218\uc0c9\uc791\uc5c5 \ub54c\ubb38\uc5d0 \uc5b8\ub518\uc774 \ub2e4\ub978 \ud604\uc7a5\uc758 \uc0ac\uc5c5\uc744 \uc9c4\ud589\ud558\uc9c0 \ubabb\ud55c \uc190\ud574\ub97c \ubd24\ub2e4\uace0 \uc8fc\uc7a5\ud588\uace0 \uacc4\uc57d \uc870\uac74\uc0c1 \uc2dc\uc2e0 \uc778\uc591\uacfc \uc0c1\uad00\uc5c6\uc774 \ub300\uae08\uc744 \uc9c0\uae09\ud560 \uc218\ubc16\uc5d0 \uc5c6\uc5c8\ub2e4\"\uace0 \ubc1d\ud614\ub2e4.", "paragraph_answer": "\ud574\uacbd\uacfc \uc5b8\ub518\uc740 4\ub144 \uc804 \ucc9c\uc548\ud568 \uc0ac\uac74 \ub54c\uc5d0\ub3c4 \uc218\uc0c9\uc791\uc5c5\uc744 \ud568\uaed8 \uc218\ud589\ud55c \uc778\uc5f0\uc774 \uc788\ub2e4. \ud574\uacbd\uc740 2010\ub144 4\uc6d4 \ucc9c\uc548\ud568 \uc2e4\uc885\uc7a5\ubcd1 \uc218\uc0c9\uc791\uc5c5\uc5d0 \ucc38\uc5ec\ud588\ub2e4\uac00 \uce68\ubab0\ud55c \uc30d\ub04c\uc774 \uc5b4\uc120 \uae08\uc59198\ud638\uc758 \uc120\ub0b4 \uc218\uc0c9\uc5c5\uccb4\ub85c \uc5b8\ub518\uc744 \uc120\uc815\ud588\ub2e4. \uc5b8\ub518\uc740 \uc2dc\uc2e0 \uc218\uc0c9\uc791\uc5c5\uc5d0\uc11c \ubcc4\ub2e4\ub978 \uc131\uacfc\ub97c \uac70\ub450\uc9c0 \ubabb\ud588\uc9c0\ub9cc \uacc4\uc57d\uae08\uc561\uc758 10%\ub97c \uc81c\uc678\ud55c 4\uc5b5 3\ucc9c\ub9cc\uc6d0 \uc744 \ud574\uacbd\uc73c\ub85c\ubd80\ud130 \uc9c0\uae09\ubc1b\uc558\ub2e4. \uadf8\ub7ec\ub098 \uc2e4\uc885\uc790 \uc2dc\uc2e0\uc744 \ub2e8 1\uad6c\ub3c4 \uc778\uc591\ud558\uc9c0 \ubabb\ud55c \uc5c5\uccb4\uc5d0 \uac70\uc561\uc758 \uc608\uc0b0\uc744 \uc9c0\uae09\ud55c \uac83\uc774 \uc801\uc808\ud588\ub294\uc9c0\uc5d0 \ub300\ud55c \ub17c\ub780\uc740 \uc9c0\uae08\uae4c\uc9c0 \uacc4\uc18d\ub418\uace0 \uc788\ub2e4. \ud574\uacbd\uc740 \uc774\uc5d0 \ub300\ud574 \"\uae08\uc591\ud638 \uc218\uc0c9\uc791\uc5c5 \ub54c\ubb38\uc5d0 \uc5b8\ub518\uc774 \ub2e4\ub978 \ud604\uc7a5\uc758 \uc0ac\uc5c5\uc744 \uc9c4\ud589\ud558\uc9c0 \ubabb\ud55c \uc190\ud574\ub97c \ubd24\ub2e4\uace0 \uc8fc\uc7a5\ud588\uace0 \uacc4\uc57d \uc870\uac74\uc0c1 \uc2dc\uc2e0 \uc778\uc591\uacfc \uc0c1\uad00\uc5c6\uc774 \ub300\uae08\uc744 \uc9c0\uae09\ud560 \uc218\ubc16\uc5d0 \uc5c6\uc5c8\ub2e4\"\uace0 \ubc1d\ud614\ub2e4.", "sentence_answer": "\uc5b8\ub518\uc740 \uc2dc\uc2e0 \uc218\uc0c9\uc791\uc5c5\uc5d0\uc11c \ubcc4\ub2e4\ub978 \uc131\uacfc\ub97c \uac70\ub450\uc9c0 \ubabb\ud588\uc9c0\ub9cc \uacc4\uc57d\uae08\uc561\uc758 10%\ub97c \uc81c\uc678\ud55c 4\uc5b5 3\ucc9c\ub9cc\uc6d0 \uc744 \ud574\uacbd\uc73c\ub85c\ubd80\ud130 \uc9c0\uae09\ubc1b\uc558\ub2e4.", "paragraph_id": "6463490-13-0", "paragraph_question": "question: \uc5b8\ub518\uc740 \ud574\uacbd\uc73c\ub85c\ubd80\ud130 \uc5bc\ub9c8\uc758 \ub3c8\uc744 \uc9c0\uae09\ubc1b\uc558\ub294\uac00?, context: \ud574\uacbd\uacfc \uc5b8\ub518\uc740 4\ub144 \uc804 \ucc9c\uc548\ud568 \uc0ac\uac74 \ub54c\uc5d0\ub3c4 \uc218\uc0c9\uc791\uc5c5\uc744 \ud568\uaed8 \uc218\ud589\ud55c \uc778\uc5f0\uc774 \uc788\ub2e4. \ud574\uacbd\uc740 2010\ub144 4\uc6d4 \ucc9c\uc548\ud568 \uc2e4\uc885\uc7a5\ubcd1 \uc218\uc0c9\uc791\uc5c5\uc5d0 \ucc38\uc5ec\ud588\ub2e4\uac00 \uce68\ubab0\ud55c \uc30d\ub04c\uc774 \uc5b4\uc120 \uae08\uc59198\ud638\uc758 \uc120\ub0b4 \uc218\uc0c9\uc5c5\uccb4\ub85c \uc5b8\ub518\uc744 \uc120\uc815\ud588\ub2e4. \uc5b8\ub518\uc740 \uc2dc\uc2e0 \uc218\uc0c9\uc791\uc5c5\uc5d0\uc11c \ubcc4\ub2e4\ub978 \uc131\uacfc\ub97c \uac70\ub450\uc9c0 \ubabb\ud588\uc9c0\ub9cc \uacc4\uc57d\uae08\uc561\uc758 10%\ub97c \uc81c\uc678\ud55c 4\uc5b5 3\ucc9c\ub9cc\uc6d0\uc744 \ud574\uacbd\uc73c\ub85c\ubd80\ud130 \uc9c0\uae09\ubc1b\uc558\ub2e4. \uadf8\ub7ec\ub098 \uc2e4\uc885\uc790 \uc2dc\uc2e0\uc744 \ub2e8 1\uad6c\ub3c4 \uc778\uc591\ud558\uc9c0 \ubabb\ud55c \uc5c5\uccb4\uc5d0 \uac70\uc561\uc758 \uc608\uc0b0\uc744 \uc9c0\uae09\ud55c \uac83\uc774 \uc801\uc808\ud588\ub294\uc9c0\uc5d0 \ub300\ud55c \ub17c\ub780\uc740 \uc9c0\uae08\uae4c\uc9c0 \uacc4\uc18d\ub418\uace0 \uc788\ub2e4. \ud574\uacbd\uc740 \uc774\uc5d0 \ub300\ud574 \"\uae08\uc591\ud638 \uc218\uc0c9\uc791\uc5c5 \ub54c\ubb38\uc5d0 \uc5b8\ub518\uc774 \ub2e4\ub978 \ud604\uc7a5\uc758 \uc0ac\uc5c5\uc744 \uc9c4\ud589\ud558\uc9c0 \ubabb\ud55c \uc190\ud574\ub97c \ubd24\ub2e4\uace0 \uc8fc\uc7a5\ud588\uace0 \uacc4\uc57d \uc870\uac74\uc0c1 \uc2dc\uc2e0 \uc778\uc591\uacfc \uc0c1\uad00\uc5c6\uc774 \ub300\uae08\uc744 \uc9c0\uae09\ud560 \uc218\ubc16\uc5d0 \uc5c6\uc5c8\ub2e4\"\uace0 \ubc1d\ud614\ub2e4."} {"question": "\uc57c\ub9c8\uc6b0\uce58 \uac00\uc988\ud1a0\uc694\uac00 \uc624\ubbf8 \ub098\uac00\ud558\ub9c8 \uc131\uc758 \uc131\uc8fc\uac00 \ub41c \ub144\ub3c4\ub294?", "paragraph": "\ub178\ubd80\ub098\uac00\uc758 \uc0ac\ud6c4\uc5d0\ub3c4 \uadf8\ub798\ub3c4 \ud788\ub370\uc694\uc2dc\uc758 \uac00\uc2e0\uc73c\ub85c\uc11c \ud65c\uc57d\ud588\ub2e4. \ub374\uc1fc 11\ub144(1583\ub144)\uc758 \uc2dc\uc988\uac00\ud0c0\ucf00 \uc804\ud22c(\u8ce4\u30f6\u5cb3\u306e\u6226\u3044)\uc5d0\uc11c\ub294, \uadf8 \uc804\ucd08\uc804\uc778 \uc774\uc138(\u4f0a\u52e2) \uac00\uba54\uc57c\ub9c8 \uc131(\u4e80\u5c71\u57ce) \uacf5\uaca9\uc5d0\uc11c \uc81c\uc77c\uc758 \uacf5\uc801\uc744 \uc62c\ub838\ub2e4. \ub610 \ub2e4\uc74c \ub144 12\ub144(1584\ub144)\uc758 \uace0\ub9c8\ud0a4\u00b7\ub098\uac00\ucfe0\ud14c \uc804\ud22c(\u5c0f\u7267\u30fb\u9577\u4e45\u624b\u306e\u6226\u3044)\uc758 \ucc38\uac00\uc758 \ub54c\uc5d0\ub294 \ud788\ub370\uc694\uc2dc\ub85c\ubd80\ud130 \uba85\ub839\uc744 \ubc1b\uc544 \uc774\uc5d0\uc57c\uc2a4\ub97c \ud3ec\uc704\ud558\uae30 \uc704\ud55c \ubd80\uc131 \uad6c\ucd95\uc744 \ub2f4\ub2f9\ud558\uc600\ub2e4. \uc774 \ud6c4 \ud788\ub370\uc694\uc2dc\uc758 \uc870\uce74 \u00b7 \ub3c4\uc694\ud1a0\ubbf8 \ud788\ub370\uc4f0\uad6c(\u8c4a\u81e3\u79c0\u6b21)\uc758 \uc219\ub85c\uac00 \ub418\uc5b4 \ub374\uc1fc 13\ub144(1585\ub144)\uc5d0\ub294 \uc640\uce74\uc0ac \uad6d(\u82e5\u72ed\u56fd) \ub2e4\uce74\ud558\ub9c8 \uc131(\u9ad8\u6d5c\u57ce)\uc758 \uc131\uc8fc, \uace7 \uc624\ubbf8 \ub098\uac00\ud558\ub9c8 \uc131(\u9577\u6d5c\u57ce)\uc758 \uc131\uc8fc\uac00 \ub418\uc5b4 2\ub9cc\uc11d\uc744 \ub2e4\uc2a4\ub9b0\ub2e4. \uc774 \uc2dc\uae30\uc5d0 \uac19\uc740 \ud788\ub370\uc4f0\uad6c\uc758 \uc219\ub85c\ub85c \ub098\uc5f4\ub41c \uc0ac\ub78c\uc740 \ub2e4\ub098\uce74 \uc694\uc2dc\ub9c8\uc0ac(\u7530\u4e2d\u5409\u653f)\u00b7\ud638\ub9ac\uc624 \uc694\uc2dc\ud558\ub8e8\u00b7\ub098\uce74\ubb34\ub77c \uac00\uc988\uc6b0\uc9c0\u00b7\ud788\ud1a0\uc4f0\uc57c\ub098\uae30 \ub098\uc624\uc2a4\uc5d0(\u4e00\u67f3\u76f4\u672b) \ub4f1\uc774 \uc788\ub2e4. \ub610, \uac19\uc740 \ub144\uc5d0 \uc77c\uc5b4\ub0ac\ub358 \ub374\uc1fc \ub300\uc9c0\uc9c4\uc73c\ub85c \uc778\ud574 \uc678\ub3d9\ub538\uc778 \uc694\ub124\ud788\uba54(\u4e0e\u7962)\ub97c \uc783\uc5c8\ub2e4. \uc774 \uc988\uc74c\uc5d0 \uc8855\uc704 \uc4f0\uc2dc\ub9c8\ub178\uce74\ubbf8(\u5bfe\u99ac\u5b88)\uc5d0 \uc784\uba85\ub41c\ub2e4.", "answer": "1585\ub144", "sentence": "\uc774 \ud6c4 \ud788\ub370\uc694\uc2dc\uc758 \uc870\uce74 \u00b7 \ub3c4\uc694\ud1a0\ubbf8 \ud788\ub370\uc4f0\uad6c(\u8c4a\u81e3\u79c0\u6b21)\uc758 \uc219\ub85c\uac00 \ub418\uc5b4 \ub374\uc1fc 13\ub144( 1585\ub144 )", "paragraph_sentence": "\ub178\ubd80\ub098\uac00\uc758 \uc0ac\ud6c4\uc5d0\ub3c4 \uadf8\ub798\ub3c4 \ud788\ub370\uc694\uc2dc\uc758 \uac00\uc2e0\uc73c\ub85c\uc11c \ud65c\uc57d\ud588\ub2e4. \ub374\uc1fc 11\ub144(1583\ub144)\uc758 \uc2dc\uc988\uac00\ud0c0\ucf00 \uc804\ud22c(\u8ce4\u30f6\u5cb3\u306e\u6226\u3044)\uc5d0\uc11c\ub294, \uadf8 \uc804\ucd08\uc804\uc778 \uc774\uc138(\u4f0a\u52e2) \uac00\uba54\uc57c\ub9c8 \uc131(\u4e80\u5c71\u57ce) \uacf5\uaca9\uc5d0\uc11c \uc81c\uc77c\uc758 \uacf5\uc801\uc744 \uc62c\ub838\ub2e4. \ub610 \ub2e4\uc74c \ub144 12\ub144(1584\ub144)\uc758 \uace0\ub9c8\ud0a4\u00b7\ub098\uac00\ucfe0\ud14c \uc804\ud22c(\u5c0f\u7267\u30fb\u9577\u4e45\u624b\u306e\u6226\u3044)\uc758 \ucc38\uac00\uc758 \ub54c\uc5d0\ub294 \ud788\ub370\uc694\uc2dc\ub85c\ubd80\ud130 \uba85\ub839\uc744 \ubc1b\uc544 \uc774\uc5d0\uc57c\uc2a4\ub97c \ud3ec\uc704\ud558\uae30 \uc704\ud55c \ubd80\uc131 \uad6c\ucd95\uc744 \ub2f4\ub2f9\ud558\uc600\ub2e4. \uc774 \ud6c4 \ud788\ub370\uc694\uc2dc\uc758 \uc870\uce74 \u00b7 \ub3c4\uc694\ud1a0\ubbf8 \ud788\ub370\uc4f0\uad6c(\u8c4a\u81e3\u79c0\u6b21)\uc758 \uc219\ub85c\uac00 \ub418\uc5b4 \ub374\uc1fc 13\ub144( 1585\ub144 ) \uc5d0\ub294 \uc640\uce74\uc0ac \uad6d(\u82e5\u72ed\u56fd) \ub2e4\uce74\ud558\ub9c8 \uc131(\u9ad8\u6d5c\u57ce)\uc758 \uc131\uc8fc, \uace7 \uc624\ubbf8 \ub098\uac00\ud558\ub9c8 \uc131(\u9577\u6d5c\u57ce)\uc758 \uc131\uc8fc\uac00 \ub418\uc5b4 2\ub9cc\uc11d\uc744 \ub2e4\uc2a4\ub9b0\ub2e4. \uc774 \uc2dc\uae30\uc5d0 \uac19\uc740 \ud788\ub370\uc4f0\uad6c\uc758 \uc219\ub85c\ub85c \ub098\uc5f4\ub41c \uc0ac\ub78c\uc740 \ub2e4\ub098\uce74 \uc694\uc2dc\ub9c8\uc0ac(\u7530\u4e2d\u5409\u653f)\u00b7\ud638\ub9ac\uc624 \uc694\uc2dc\ud558\ub8e8\u00b7\ub098\uce74\ubb34\ub77c \uac00\uc988\uc6b0\uc9c0\u00b7\ud788\ud1a0\uc4f0\uc57c\ub098\uae30 \ub098\uc624\uc2a4\uc5d0(\u4e00\u67f3\u76f4\u672b) \ub4f1\uc774 \uc788\ub2e4. \ub610, \uac19\uc740 \ub144\uc5d0 \uc77c\uc5b4\ub0ac\ub358 \ub374\uc1fc \ub300\uc9c0\uc9c4\uc73c\ub85c \uc778\ud574 \uc678\ub3d9\ub538\uc778 \uc694\ub124\ud788\uba54(\u4e0e\u7962)\ub97c \uc783\uc5c8\ub2e4. \uc774 \uc988\uc74c\uc5d0 \uc8855\uc704 \uc4f0\uc2dc\ub9c8\ub178\uce74\ubbf8(\u5bfe\u99ac\u5b88)\uc5d0 \uc784\uba85\ub41c\ub2e4.", "paragraph_answer": "\ub178\ubd80\ub098\uac00\uc758 \uc0ac\ud6c4\uc5d0\ub3c4 \uadf8\ub798\ub3c4 \ud788\ub370\uc694\uc2dc\uc758 \uac00\uc2e0\uc73c\ub85c\uc11c \ud65c\uc57d\ud588\ub2e4. \ub374\uc1fc 11\ub144(1583\ub144)\uc758 \uc2dc\uc988\uac00\ud0c0\ucf00 \uc804\ud22c(\u8ce4\u30f6\u5cb3\u306e\u6226\u3044)\uc5d0\uc11c\ub294, \uadf8 \uc804\ucd08\uc804\uc778 \uc774\uc138(\u4f0a\u52e2) \uac00\uba54\uc57c\ub9c8 \uc131(\u4e80\u5c71\u57ce) \uacf5\uaca9\uc5d0\uc11c \uc81c\uc77c\uc758 \uacf5\uc801\uc744 \uc62c\ub838\ub2e4. \ub610 \ub2e4\uc74c \ub144 12\ub144(1584\ub144)\uc758 \uace0\ub9c8\ud0a4\u00b7\ub098\uac00\ucfe0\ud14c \uc804\ud22c(\u5c0f\u7267\u30fb\u9577\u4e45\u624b\u306e\u6226\u3044)\uc758 \ucc38\uac00\uc758 \ub54c\uc5d0\ub294 \ud788\ub370\uc694\uc2dc\ub85c\ubd80\ud130 \uba85\ub839\uc744 \ubc1b\uc544 \uc774\uc5d0\uc57c\uc2a4\ub97c \ud3ec\uc704\ud558\uae30 \uc704\ud55c \ubd80\uc131 \uad6c\ucd95\uc744 \ub2f4\ub2f9\ud558\uc600\ub2e4. \uc774 \ud6c4 \ud788\ub370\uc694\uc2dc\uc758 \uc870\uce74 \u00b7 \ub3c4\uc694\ud1a0\ubbf8 \ud788\ub370\uc4f0\uad6c(\u8c4a\u81e3\u79c0\u6b21)\uc758 \uc219\ub85c\uac00 \ub418\uc5b4 \ub374\uc1fc 13\ub144( 1585\ub144 )\uc5d0\ub294 \uc640\uce74\uc0ac \uad6d(\u82e5\u72ed\u56fd) \ub2e4\uce74\ud558\ub9c8 \uc131(\u9ad8\u6d5c\u57ce)\uc758 \uc131\uc8fc, \uace7 \uc624\ubbf8 \ub098\uac00\ud558\ub9c8 \uc131(\u9577\u6d5c\u57ce)\uc758 \uc131\uc8fc\uac00 \ub418\uc5b4 2\ub9cc\uc11d\uc744 \ub2e4\uc2a4\ub9b0\ub2e4. \uc774 \uc2dc\uae30\uc5d0 \uac19\uc740 \ud788\ub370\uc4f0\uad6c\uc758 \uc219\ub85c\ub85c \ub098\uc5f4\ub41c \uc0ac\ub78c\uc740 \ub2e4\ub098\uce74 \uc694\uc2dc\ub9c8\uc0ac(\u7530\u4e2d\u5409\u653f)\u00b7\ud638\ub9ac\uc624 \uc694\uc2dc\ud558\ub8e8\u00b7\ub098\uce74\ubb34\ub77c \uac00\uc988\uc6b0\uc9c0\u00b7\ud788\ud1a0\uc4f0\uc57c\ub098\uae30 \ub098\uc624\uc2a4\uc5d0(\u4e00\u67f3\u76f4\u672b) \ub4f1\uc774 \uc788\ub2e4. \ub610, \uac19\uc740 \ub144\uc5d0 \uc77c\uc5b4\ub0ac\ub358 \ub374\uc1fc \ub300\uc9c0\uc9c4\uc73c\ub85c \uc778\ud574 \uc678\ub3d9\ub538\uc778 \uc694\ub124\ud788\uba54(\u4e0e\u7962)\ub97c \uc783\uc5c8\ub2e4. \uc774 \uc988\uc74c\uc5d0 \uc8855\uc704 \uc4f0\uc2dc\ub9c8\ub178\uce74\ubbf8(\u5bfe\u99ac\u5b88)\uc5d0 \uc784\uba85\ub41c\ub2e4.", "sentence_answer": "\uc774 \ud6c4 \ud788\ub370\uc694\uc2dc\uc758 \uc870\uce74 \u00b7 \ub3c4\uc694\ud1a0\ubbf8 \ud788\ub370\uc4f0\uad6c(\u8c4a\u81e3\u79c0\u6b21)\uc758 \uc219\ub85c\uac00 \ub418\uc5b4 \ub374\uc1fc 13\ub144( 1585\ub144 )", "paragraph_id": "6540324-3-1", "paragraph_question": "question: \uc57c\ub9c8\uc6b0\uce58 \uac00\uc988\ud1a0\uc694\uac00 \uc624\ubbf8 \ub098\uac00\ud558\ub9c8 \uc131\uc758 \uc131\uc8fc\uac00 \ub41c \ub144\ub3c4\ub294?, context: \ub178\ubd80\ub098\uac00\uc758 \uc0ac\ud6c4\uc5d0\ub3c4 \uadf8\ub798\ub3c4 \ud788\ub370\uc694\uc2dc\uc758 \uac00\uc2e0\uc73c\ub85c\uc11c \ud65c\uc57d\ud588\ub2e4. \ub374\uc1fc 11\ub144(1583\ub144)\uc758 \uc2dc\uc988\uac00\ud0c0\ucf00 \uc804\ud22c(\u8ce4\u30f6\u5cb3\u306e\u6226\u3044)\uc5d0\uc11c\ub294, \uadf8 \uc804\ucd08\uc804\uc778 \uc774\uc138(\u4f0a\u52e2) \uac00\uba54\uc57c\ub9c8 \uc131(\u4e80\u5c71\u57ce) \uacf5\uaca9\uc5d0\uc11c \uc81c\uc77c\uc758 \uacf5\uc801\uc744 \uc62c\ub838\ub2e4. \ub610 \ub2e4\uc74c \ub144 12\ub144(1584\ub144)\uc758 \uace0\ub9c8\ud0a4\u00b7\ub098\uac00\ucfe0\ud14c \uc804\ud22c(\u5c0f\u7267\u30fb\u9577\u4e45\u624b\u306e\u6226\u3044)\uc758 \ucc38\uac00\uc758 \ub54c\uc5d0\ub294 \ud788\ub370\uc694\uc2dc\ub85c\ubd80\ud130 \uba85\ub839\uc744 \ubc1b\uc544 \uc774\uc5d0\uc57c\uc2a4\ub97c \ud3ec\uc704\ud558\uae30 \uc704\ud55c \ubd80\uc131 \uad6c\ucd95\uc744 \ub2f4\ub2f9\ud558\uc600\ub2e4. \uc774 \ud6c4 \ud788\ub370\uc694\uc2dc\uc758 \uc870\uce74 \u00b7 \ub3c4\uc694\ud1a0\ubbf8 \ud788\ub370\uc4f0\uad6c(\u8c4a\u81e3\u79c0\u6b21)\uc758 \uc219\ub85c\uac00 \ub418\uc5b4 \ub374\uc1fc 13\ub144(1585\ub144)\uc5d0\ub294 \uc640\uce74\uc0ac \uad6d(\u82e5\u72ed\u56fd) \ub2e4\uce74\ud558\ub9c8 \uc131(\u9ad8\u6d5c\u57ce)\uc758 \uc131\uc8fc, \uace7 \uc624\ubbf8 \ub098\uac00\ud558\ub9c8 \uc131(\u9577\u6d5c\u57ce)\uc758 \uc131\uc8fc\uac00 \ub418\uc5b4 2\ub9cc\uc11d\uc744 \ub2e4\uc2a4\ub9b0\ub2e4. \uc774 \uc2dc\uae30\uc5d0 \uac19\uc740 \ud788\ub370\uc4f0\uad6c\uc758 \uc219\ub85c\ub85c \ub098\uc5f4\ub41c \uc0ac\ub78c\uc740 \ub2e4\ub098\uce74 \uc694\uc2dc\ub9c8\uc0ac(\u7530\u4e2d\u5409\u653f)\u00b7\ud638\ub9ac\uc624 \uc694\uc2dc\ud558\ub8e8\u00b7\ub098\uce74\ubb34\ub77c \uac00\uc988\uc6b0\uc9c0\u00b7\ud788\ud1a0\uc4f0\uc57c\ub098\uae30 \ub098\uc624\uc2a4\uc5d0(\u4e00\u67f3\u76f4\u672b) \ub4f1\uc774 \uc788\ub2e4. \ub610, \uac19\uc740 \ub144\uc5d0 \uc77c\uc5b4\ub0ac\ub358 \ub374\uc1fc \ub300\uc9c0\uc9c4\uc73c\ub85c \uc778\ud574 \uc678\ub3d9\ub538\uc778 \uc694\ub124\ud788\uba54(\u4e0e\u7962)\ub97c \uc783\uc5c8\ub2e4. \uc774 \uc988\uc74c\uc5d0 \uc8855\uc704 \uc4f0\uc2dc\ub9c8\ub178\uce74\ubbf8(\u5bfe\u99ac\u5b88)\uc5d0 \uc784\uba85\ub41c\ub2e4."} {"question": "\uc2dc\ubbfc\uc0ac\ud68c \ubc0f \ud504\ub85c\ud14c\uc2a4\ud0c4\ud2b8\uc801 \ubd84\uc704\uae30\uc758 \ube44\uadf9\uc774 \ud5c8\uc6a9\ud558\uc9c0 \uc54a\uc74c\uc744 \ub73b\ud558\ub294 \uac83\uc740?", "paragraph": "19\uc138\uae30\uac00 \uc9c4\ud589\ub418\ub294 \ub3d9\uc548 \uc74c\uc545\uc5d0\uc11c\ub3c4 \ub2e4\ub978 \uc608\uc220\uc5d0\uc11c\uc640 \ub9c8\ucc2c\uac00\uc9c0\ub85c \"\uc548\ud2f0\uace0\uc804\uc8fc\uc758\" \ud639\uc740 \"\uc548\ud2f0\ub0ad\ub9cc\uc8fc\uc758\"\uc801 \uacbd\ud5a5\uc744 \uc9c0\ub2cc \ub9ac\uc5bc\ub9ac\uc998 \uc74c\uc545\uc758 \ud615\ud0dc\uac00 \ub098\ud0c0\ub0ac\ub2e4. \uc0ac\uc2e4\uc8fc\uc758\uc5d0\uc11c\ub294 \uc544\ub984\ub2e4\uc6c0\uc774 \uc544\ub2cc \ud45c\ud604\uc758 \uc9c4\uc2e4\uc774 \uc911\uc694\ud558\uac8c \ub418\uc5c8\ub2e4. \"\uc74c\uc545\uc801 \ub9ac\uc5bc\ub9ac\uc998\"\uc740 \uc2dc\ubbfc\uc0ac\ud68c \ubc0f \ud504\ub85c\ud14c\uc2a4\ud0c4\ud2b8\uc801 \ubd84\uc704\uae30\uc758 \ube44\uadf9\uc774 \ud5c8\uc6a9\ud558\uc9c0 \uc54a\uc558\ub358 \uc591\uc2dd\uc758 \uc5ec\ub7ec \uaddc\uce59\ub4e4\uc744 \ubb34\ub108\ub728\ub838\uace0 \uc608\uc220\uc758 \uc18c\uc7ac\ub97c \uc2e0\ud654\ub098 \ub3d9\ud654 \ud639\uc740 \uc5ed\uc0ac\uc5d0\uc11c\uac00 \uc544\ub2cc \ud604\uc7ac\uc758 \uc2e4\uc0dd\ud65c\uc5d0\uc11c \ucc3e\uc558\ub2e4. \ube44\uc81c\uc640 \ubca0\ub974\ub514\uc758 \uc624\ud398\ub77c\uc5d0\uc11c\ub294 \"\ucd94\ud558\uace0\" \"\uc800\uc18d\ud558\ub2e4\"\ub294 \uc774\uc720\ub85c \ubc30\uc81c\ub418\uc5c8\ub358 \uc2e4\uc0dd\ud65c \uc8fc\uc778\uacf5\uc758 \uc5ed\ud560 \ubc0f \uc18c\uc7ac\ub3c4 \ud65c\uc6a9\ub418\uc5c8\uace0, \ubb34\uc18c\ub975\uc2a4\ud0a4\uc758 \uc624\ud398\ub77c\uc5d0\uc11c\ub294 \"\uc74c\uc545\uc801 \uc0b0\ubb38\"\uc758 \uacbd\ud5a5\uc744 \ubcf4\uc5ec\uc900\ub2e4. 1830\ub144\ub300 \uc774\ud6c4 \uc774 \uc74c\uc545\uc740 \uc2e0\ub3c5\uc77c\uc545\ud30c\uac00 \ucde8\ud55c \uc785\uc7a5, \uc989 '\uae30\uc545\uc74c\uc545\uc5d0\uc11c \uc808\ub300 \uc74c\uc545\uc801 \uc0ac\uace0\uc5d0 \ubc18\ub300\ud55c\ub2e4'\ub294 \uc785\uc7a5\uc744 \ub300\ubcc0\ud55c\ub2e4. \uadf8\ubfd0\ub9cc \uc544\ub2c8\ub77c \ub0ad\ub9cc\uc801 \uae30\uc545\uc74c\uc545\uc758 \uc218\uc218\uaed8\ub07c\uac19\uc740 \ud3ec\uc5d0\uc9c0\uc5d0 \ub300\uce6d\ub418\uba74\uc11c \"\ud2b9\uc9d5\uc801 \ubbf8\"\ub97c \uace0\uc218\ud558\uace0 \"\uc74c\uc545\uc801 \ud6a8\uacfc\"\uc640 \ud504\ub85c\uadf8\ub7a8\uc801 \uc694\uc18c\ub97c \uc911\uc2dc\ud558\uac8c \ub41c\ub2e4. \uc624\uc624", "answer": "\uc74c\uc545\uc801 \ub9ac\uc5bc\ub9ac\uc998", "sentence": "\" \uc74c\uc545\uc801 \ub9ac\uc5bc\ub9ac\uc998 \"\uc740 \uc2dc\ubbfc\uc0ac\ud68c \ubc0f \ud504\ub85c\ud14c\uc2a4\ud0c4\ud2b8\uc801 \ubd84\uc704\uae30\uc758 \ube44\uadf9\uc774 \ud5c8\uc6a9\ud558\uc9c0 \uc54a\uc558\ub358 \uc591\uc2dd\uc758 \uc5ec\ub7ec \uaddc\uce59\ub4e4\uc744 \ubb34\ub108\ub728\ub838\uace0 \uc608\uc220\uc758 \uc18c\uc7ac\ub97c \uc2e0\ud654\ub098 \ub3d9\ud654 \ud639\uc740 \uc5ed\uc0ac\uc5d0\uc11c\uac00 \uc544\ub2cc \ud604\uc7ac\uc758 \uc2e4\uc0dd\ud65c\uc5d0\uc11c \ucc3e\uc558\ub2e4.", "paragraph_sentence": "19\uc138\uae30\uac00 \uc9c4\ud589\ub418\ub294 \ub3d9\uc548 \uc74c\uc545\uc5d0\uc11c\ub3c4 \ub2e4\ub978 \uc608\uc220\uc5d0\uc11c\uc640 \ub9c8\ucc2c\uac00\uc9c0\ub85c \"\uc548\ud2f0\uace0\uc804\uc8fc\uc758\" \ud639\uc740 \"\uc548\ud2f0\ub0ad\ub9cc\uc8fc\uc758\"\uc801 \uacbd\ud5a5\uc744 \uc9c0\ub2cc \ub9ac\uc5bc\ub9ac\uc998 \uc74c\uc545\uc758 \ud615\ud0dc\uac00 \ub098\ud0c0\ub0ac\ub2e4. \uc0ac\uc2e4\uc8fc\uc758\uc5d0\uc11c\ub294 \uc544\ub984\ub2e4\uc6c0\uc774 \uc544\ub2cc \ud45c\ud604\uc758 \uc9c4\uc2e4\uc774 \uc911\uc694\ud558\uac8c \ub418\uc5c8\ub2e4. \" \uc74c\uc545\uc801 \ub9ac\uc5bc\ub9ac\uc998 \"\uc740 \uc2dc\ubbfc\uc0ac\ud68c \ubc0f \ud504\ub85c\ud14c\uc2a4\ud0c4\ud2b8\uc801 \ubd84\uc704\uae30\uc758 \ube44\uadf9\uc774 \ud5c8\uc6a9\ud558\uc9c0 \uc54a\uc558\ub358 \uc591\uc2dd\uc758 \uc5ec\ub7ec \uaddc\uce59\ub4e4\uc744 \ubb34\ub108\ub728\ub838\uace0 \uc608\uc220\uc758 \uc18c\uc7ac\ub97c \uc2e0\ud654\ub098 \ub3d9\ud654 \ud639\uc740 \uc5ed\uc0ac\uc5d0\uc11c\uac00 \uc544\ub2cc \ud604\uc7ac\uc758 \uc2e4\uc0dd\ud65c\uc5d0\uc11c \ucc3e\uc558\ub2e4. \ube44\uc81c\uc640 \ubca0\ub974\ub514\uc758 \uc624\ud398\ub77c\uc5d0\uc11c\ub294 \"\ucd94\ud558\uace0\" \"\uc800\uc18d\ud558\ub2e4\"\ub294 \uc774\uc720\ub85c \ubc30\uc81c\ub418\uc5c8\ub358 \uc2e4\uc0dd\ud65c \uc8fc\uc778\uacf5\uc758 \uc5ed\ud560 \ubc0f \uc18c\uc7ac\ub3c4 \ud65c\uc6a9\ub418\uc5c8\uace0, \ubb34\uc18c\ub975\uc2a4\ud0a4\uc758 \uc624\ud398\ub77c\uc5d0\uc11c\ub294 \"\uc74c\uc545\uc801 \uc0b0\ubb38\"\uc758 \uacbd\ud5a5\uc744 \ubcf4\uc5ec\uc900\ub2e4. 1830\ub144\ub300 \uc774\ud6c4 \uc774 \uc74c\uc545\uc740 \uc2e0\ub3c5\uc77c\uc545\ud30c\uac00 \ucde8\ud55c \uc785\uc7a5, \uc989 '\uae30\uc545\uc74c\uc545\uc5d0\uc11c \uc808\ub300 \uc74c\uc545\uc801 \uc0ac\uace0\uc5d0 \ubc18\ub300\ud55c\ub2e4'\ub294 \uc785\uc7a5\uc744 \ub300\ubcc0\ud55c\ub2e4. \uadf8\ubfd0\ub9cc \uc544\ub2c8\ub77c \ub0ad\ub9cc\uc801 \uae30\uc545\uc74c\uc545\uc758 \uc218\uc218\uaed8\ub07c\uac19\uc740 \ud3ec\uc5d0\uc9c0\uc5d0 \ub300\uce6d\ub418\uba74\uc11c \"\ud2b9\uc9d5\uc801 \ubbf8\"\ub97c \uace0\uc218\ud558\uace0 \"\uc74c\uc545\uc801 \ud6a8\uacfc\"\uc640 \ud504\ub85c\uadf8\ub7a8\uc801 \uc694\uc18c\ub97c \uc911\uc2dc\ud558\uac8c \ub41c\ub2e4. \uc624\uc624", "paragraph_answer": "19\uc138\uae30\uac00 \uc9c4\ud589\ub418\ub294 \ub3d9\uc548 \uc74c\uc545\uc5d0\uc11c\ub3c4 \ub2e4\ub978 \uc608\uc220\uc5d0\uc11c\uc640 \ub9c8\ucc2c\uac00\uc9c0\ub85c \"\uc548\ud2f0\uace0\uc804\uc8fc\uc758\" \ud639\uc740 \"\uc548\ud2f0\ub0ad\ub9cc\uc8fc\uc758\"\uc801 \uacbd\ud5a5\uc744 \uc9c0\ub2cc \ub9ac\uc5bc\ub9ac\uc998 \uc74c\uc545\uc758 \ud615\ud0dc\uac00 \ub098\ud0c0\ub0ac\ub2e4. \uc0ac\uc2e4\uc8fc\uc758\uc5d0\uc11c\ub294 \uc544\ub984\ub2e4\uc6c0\uc774 \uc544\ub2cc \ud45c\ud604\uc758 \uc9c4\uc2e4\uc774 \uc911\uc694\ud558\uac8c \ub418\uc5c8\ub2e4. \" \uc74c\uc545\uc801 \ub9ac\uc5bc\ub9ac\uc998 \"\uc740 \uc2dc\ubbfc\uc0ac\ud68c \ubc0f \ud504\ub85c\ud14c\uc2a4\ud0c4\ud2b8\uc801 \ubd84\uc704\uae30\uc758 \ube44\uadf9\uc774 \ud5c8\uc6a9\ud558\uc9c0 \uc54a\uc558\ub358 \uc591\uc2dd\uc758 \uc5ec\ub7ec \uaddc\uce59\ub4e4\uc744 \ubb34\ub108\ub728\ub838\uace0 \uc608\uc220\uc758 \uc18c\uc7ac\ub97c \uc2e0\ud654\ub098 \ub3d9\ud654 \ud639\uc740 \uc5ed\uc0ac\uc5d0\uc11c\uac00 \uc544\ub2cc \ud604\uc7ac\uc758 \uc2e4\uc0dd\ud65c\uc5d0\uc11c \ucc3e\uc558\ub2e4. \ube44\uc81c\uc640 \ubca0\ub974\ub514\uc758 \uc624\ud398\ub77c\uc5d0\uc11c\ub294 \"\ucd94\ud558\uace0\" \"\uc800\uc18d\ud558\ub2e4\"\ub294 \uc774\uc720\ub85c \ubc30\uc81c\ub418\uc5c8\ub358 \uc2e4\uc0dd\ud65c \uc8fc\uc778\uacf5\uc758 \uc5ed\ud560 \ubc0f \uc18c\uc7ac\ub3c4 \ud65c\uc6a9\ub418\uc5c8\uace0, \ubb34\uc18c\ub975\uc2a4\ud0a4\uc758 \uc624\ud398\ub77c\uc5d0\uc11c\ub294 \"\uc74c\uc545\uc801 \uc0b0\ubb38\"\uc758 \uacbd\ud5a5\uc744 \ubcf4\uc5ec\uc900\ub2e4. 1830\ub144\ub300 \uc774\ud6c4 \uc774 \uc74c\uc545\uc740 \uc2e0\ub3c5\uc77c\uc545\ud30c\uac00 \ucde8\ud55c \uc785\uc7a5, \uc989 '\uae30\uc545\uc74c\uc545\uc5d0\uc11c \uc808\ub300 \uc74c\uc545\uc801 \uc0ac\uace0\uc5d0 \ubc18\ub300\ud55c\ub2e4'\ub294 \uc785\uc7a5\uc744 \ub300\ubcc0\ud55c\ub2e4. \uadf8\ubfd0\ub9cc \uc544\ub2c8\ub77c \ub0ad\ub9cc\uc801 \uae30\uc545\uc74c\uc545\uc758 \uc218\uc218\uaed8\ub07c\uac19\uc740 \ud3ec\uc5d0\uc9c0\uc5d0 \ub300\uce6d\ub418\uba74\uc11c \"\ud2b9\uc9d5\uc801 \ubbf8\"\ub97c \uace0\uc218\ud558\uace0 \"\uc74c\uc545\uc801 \ud6a8\uacfc\"\uc640 \ud504\ub85c\uadf8\ub7a8\uc801 \uc694\uc18c\ub97c \uc911\uc2dc\ud558\uac8c \ub41c\ub2e4. \uc624\uc624", "sentence_answer": "\" \uc74c\uc545\uc801 \ub9ac\uc5bc\ub9ac\uc998 \"\uc740 \uc2dc\ubbfc\uc0ac\ud68c \ubc0f \ud504\ub85c\ud14c\uc2a4\ud0c4\ud2b8\uc801 \ubd84\uc704\uae30\uc758 \ube44\uadf9\uc774 \ud5c8\uc6a9\ud558\uc9c0 \uc54a\uc558\ub358 \uc591\uc2dd\uc758 \uc5ec\ub7ec \uaddc\uce59\ub4e4\uc744 \ubb34\ub108\ub728\ub838\uace0 \uc608\uc220\uc758 \uc18c\uc7ac\ub97c \uc2e0\ud654\ub098 \ub3d9\ud654 \ud639\uc740 \uc5ed\uc0ac\uc5d0\uc11c\uac00 \uc544\ub2cc \ud604\uc7ac\uc758 \uc2e4\uc0dd\ud65c\uc5d0\uc11c \ucc3e\uc558\ub2e4.", "paragraph_id": "6464953-1-1", "paragraph_question": "question: \uc2dc\ubbfc\uc0ac\ud68c \ubc0f \ud504\ub85c\ud14c\uc2a4\ud0c4\ud2b8\uc801 \ubd84\uc704\uae30\uc758 \ube44\uadf9\uc774 \ud5c8\uc6a9\ud558\uc9c0 \uc54a\uc74c\uc744 \ub73b\ud558\ub294 \uac83\uc740?, context: 19\uc138\uae30\uac00 \uc9c4\ud589\ub418\ub294 \ub3d9\uc548 \uc74c\uc545\uc5d0\uc11c\ub3c4 \ub2e4\ub978 \uc608\uc220\uc5d0\uc11c\uc640 \ub9c8\ucc2c\uac00\uc9c0\ub85c \"\uc548\ud2f0\uace0\uc804\uc8fc\uc758\" \ud639\uc740 \"\uc548\ud2f0\ub0ad\ub9cc\uc8fc\uc758\"\uc801 \uacbd\ud5a5\uc744 \uc9c0\ub2cc \ub9ac\uc5bc\ub9ac\uc998 \uc74c\uc545\uc758 \ud615\ud0dc\uac00 \ub098\ud0c0\ub0ac\ub2e4. \uc0ac\uc2e4\uc8fc\uc758\uc5d0\uc11c\ub294 \uc544\ub984\ub2e4\uc6c0\uc774 \uc544\ub2cc \ud45c\ud604\uc758 \uc9c4\uc2e4\uc774 \uc911\uc694\ud558\uac8c \ub418\uc5c8\ub2e4. \"\uc74c\uc545\uc801 \ub9ac\uc5bc\ub9ac\uc998\"\uc740 \uc2dc\ubbfc\uc0ac\ud68c \ubc0f \ud504\ub85c\ud14c\uc2a4\ud0c4\ud2b8\uc801 \ubd84\uc704\uae30\uc758 \ube44\uadf9\uc774 \ud5c8\uc6a9\ud558\uc9c0 \uc54a\uc558\ub358 \uc591\uc2dd\uc758 \uc5ec\ub7ec \uaddc\uce59\ub4e4\uc744 \ubb34\ub108\ub728\ub838\uace0 \uc608\uc220\uc758 \uc18c\uc7ac\ub97c \uc2e0\ud654\ub098 \ub3d9\ud654 \ud639\uc740 \uc5ed\uc0ac\uc5d0\uc11c\uac00 \uc544\ub2cc \ud604\uc7ac\uc758 \uc2e4\uc0dd\ud65c\uc5d0\uc11c \ucc3e\uc558\ub2e4. \ube44\uc81c\uc640 \ubca0\ub974\ub514\uc758 \uc624\ud398\ub77c\uc5d0\uc11c\ub294 \"\ucd94\ud558\uace0\" \"\uc800\uc18d\ud558\ub2e4\"\ub294 \uc774\uc720\ub85c \ubc30\uc81c\ub418\uc5c8\ub358 \uc2e4\uc0dd\ud65c \uc8fc\uc778\uacf5\uc758 \uc5ed\ud560 \ubc0f \uc18c\uc7ac\ub3c4 \ud65c\uc6a9\ub418\uc5c8\uace0, \ubb34\uc18c\ub975\uc2a4\ud0a4\uc758 \uc624\ud398\ub77c\uc5d0\uc11c\ub294 \"\uc74c\uc545\uc801 \uc0b0\ubb38\"\uc758 \uacbd\ud5a5\uc744 \ubcf4\uc5ec\uc900\ub2e4. 1830\ub144\ub300 \uc774\ud6c4 \uc774 \uc74c\uc545\uc740 \uc2e0\ub3c5\uc77c\uc545\ud30c\uac00 \ucde8\ud55c \uc785\uc7a5, \uc989 '\uae30\uc545\uc74c\uc545\uc5d0\uc11c \uc808\ub300 \uc74c\uc545\uc801 \uc0ac\uace0\uc5d0 \ubc18\ub300\ud55c\ub2e4'\ub294 \uc785\uc7a5\uc744 \ub300\ubcc0\ud55c\ub2e4. \uadf8\ubfd0\ub9cc \uc544\ub2c8\ub77c \ub0ad\ub9cc\uc801 \uae30\uc545\uc74c\uc545\uc758 \uc218\uc218\uaed8\ub07c\uac19\uc740 \ud3ec\uc5d0\uc9c0\uc5d0 \ub300\uce6d\ub418\uba74\uc11c \"\ud2b9\uc9d5\uc801 \ubbf8\"\ub97c \uace0\uc218\ud558\uace0 \"\uc74c\uc545\uc801 \ud6a8\uacfc\"\uc640 \ud504\ub85c\uadf8\ub7a8\uc801 \uc694\uc18c\ub97c \uc911\uc2dc\ud558\uac8c \ub41c\ub2e4. \uc624\uc624"} {"question": "\uac10\uc218 \ubd84\uc5f4\uc774\ub098 DNA \ubcf5\uc81c \uacfc\uc815\uc5d0\uc11c \ub3cc\uc5f0\ubcc0\uc774\uc6d0\uc774 \ub418\ub294 \ubb3c\uc9c8\uc5d0 \uc758\ud574 \ub3cc\uc5f0\ubcc0\uc774\ub97c \uc77c\uc73c\ud0ac \uc218 \uc788\ub294\uac83\uc740?", "paragraph": "\uac8c\ub188\uc5d0\uc11c \ubc1c\uc0dd\ud558\ub294 \ubb34\uc791\uc704\uc801\uc778 \ub3cc\uc5f0\ubcc0\uc774\ub294 \uc720\uc804\ud615\uc9c8\uc758 \uac1c\uccb4\ucc28\ub97c \uc77c\uc73c\ud0a8\ub2e4. \uc138\ud3ec\uc548\uc758 \uac8c\ub188\uc744 \uc774\ub8e8\ub294 DNA \uc5fc\uae30 \uc11c\uc5f4\uc740 \uac10\uc218 \ubd84\uc5f4\uc774\ub098 DNA \ubcf5\uc81c\uc758 \uacfc\uc815\uc5d0\uc11c \ubc29\uc0ac\ub2a5, \ubc14\uc774\ub7ec\uc2a4, \ud2b8\ub80c\uc2a4\ud3ec\uc874, \uadf8\ub9ac\uace0 \ub3cc\uc5f0\ubcc0\uc774\uc6d0\uc774 \ub418\ub294 \ubb3c\uc9c8\uc5d0 \uc758\ud574 \ub3cc\uc5f0\ubcc0\uc774\ub97c \uc77c\uc73c\ud0ac \uc218 \uc788\ub2e4. \uc774\ub807\uac8c \ud558\uc5ec \ubc1c\uc0dd\ud558\ub294 \ub3cc\uc5f0\ubcc0\uc774\ub294 \uc720\uc804\uc790\uc758 \uae30\ub2a5\uc774\ub098 \uc0dd\uc131\ubb3c\uc5d0 \uc601\ud5a5\uc744 \uc904 \uc218 \uc788\ub2e4. \ucd08\ud30c\ub9ac\ub97c \uc774\uc6a9\ud55c \ub3cc\uc5f0\ubcc0\uc774 \uc5f0\uad6c\uc758 \uacb0\uacfc \ub3cc\uc5f0\ubcc0\uc774\uc758 \uc57d 70%\ub294 \uac1c\uccb4\uc5d0\uac8c \ud574\ub85c\uc6b4 \ud615\ud0dc\ub85c \ubc1c\ud604\ub418\uc5c8\ub2e4. \uc774\ub7ec\ud55c \ud574\ub85c\uc6b4 \ub3cc\uc5f0\ubcc0\uc774\uac00 \uac1c\uccb4\uc758 \uc138\ud3ec \ub2e8\uc704\uc5d0\uc11c \ubc1c\uc0dd\ud55c \uacbd\uc6b0 \uac1c\uccb4\ub294 DNA \uc218\ub9ac\ub97c \ud1b5\ud574 \ub3cc\uc5f0\ubcc0\uc774\ub97c \uc81c\uac70\ud558\ub3c4\ub85d \uc9c4\ud654\ub418\uc5c8\ub2e4. \uc774\ub97c \ud1b5\ud574 \uc0dd\ubb3c \uac1c\uccb4\ub294 \ubb3c\uc9c8\ub300\uc0ac\uc758 \uacfc\uc815\uc5d0\uc11c \uc138\ud3ec\ub2e8\uc704\uc758 \ub3cc\uc5f0\ubcc0\uc774\ub97c \uc5b5\uc81c\ud558\uc5ec \uacfc\ub3c4\ud55c \ub3cc\uc5f0\ubcc0\uc774\uc758 \ubc1c\uc0dd\uc5d0 \ub530\ub978 \uc704\ud5d8\uc744 \ud68c\ud53c\ud560 \uc218 \uc788\ub2e4. \ud55c\ud3b8, \ub808\ud2b8\ub85c\ubc14\uc774\ub7ec\uc2a4\uc640 \uac19\uc740 \uba87\uba87 \uc885\uc740 \ub3cc\uc5f0\ubcc0\uc774 \ubc1c\uc0dd \ube44\uc728\uc774 \ub192\uc544 \uba87 \uc138\ub300 \uc9c0\ub098\uc9c0 \uc54a\uc544 \ub2e4\uc591\ud55c \ub3cc\uc5f0\ubcc0\uc774\ub97c \uac16\ub294 \uc790\uc2dd \uc138\ub300\uac00 \ub4f1\uc7a5\ud55c\ub2e4. \uc774\ub7ec\ud55c \ub192\uc740 \ub3cc\uc5f0\ubcc0\uc774 \ubc1c\uc0dd\ub960\uc740 \uc778\uac04\uc758 \uba74\uc5ed\uacc4\ub97c \ud68c\ud53c\ud560 \uc218 \uc788\uc5b4 \ubc14\uc774\ub7ec\uc2a4\uc758 \uc0dd\uc874\uc5d0 \uc720\ub9ac\ud558\uac8c \uc791\uc6a9\ud55c\ub2e4.", "answer": "DNA \uc5fc\uae30 \uc11c\uc5f4", "sentence": "\uc138\ud3ec\uc548\uc758 \uac8c\ub188\uc744 \uc774\ub8e8\ub294 DNA \uc5fc\uae30 \uc11c\uc5f4 \uc740 \uac10\uc218 \ubd84\uc5f4\uc774\ub098 DNA \ubcf5\uc81c\uc758 \uacfc\uc815\uc5d0\uc11c \ubc29\uc0ac\ub2a5, \ubc14\uc774\ub7ec\uc2a4, \ud2b8\ub80c\uc2a4\ud3ec\uc874, \uadf8\ub9ac\uace0 \ub3cc\uc5f0\ubcc0\uc774\uc6d0\uc774 \ub418\ub294 \ubb3c\uc9c8\uc5d0 \uc758\ud574 \ub3cc\uc5f0\ubcc0\uc774\ub97c \uc77c\uc73c\ud0ac \uc218 \uc788\ub2e4.", "paragraph_sentence": "\uac8c\ub188\uc5d0\uc11c \ubc1c\uc0dd\ud558\ub294 \ubb34\uc791\uc704\uc801\uc778 \ub3cc\uc5f0\ubcc0\uc774\ub294 \uc720\uc804\ud615\uc9c8\uc758 \uac1c\uccb4\ucc28\ub97c \uc77c\uc73c\ud0a8\ub2e4. \uc138\ud3ec\uc548\uc758 \uac8c\ub188\uc744 \uc774\ub8e8\ub294 DNA \uc5fc\uae30 \uc11c\uc5f4 \uc740 \uac10\uc218 \ubd84\uc5f4\uc774\ub098 DNA \ubcf5\uc81c\uc758 \uacfc\uc815\uc5d0\uc11c \ubc29\uc0ac\ub2a5, \ubc14\uc774\ub7ec\uc2a4, \ud2b8\ub80c\uc2a4\ud3ec\uc874, \uadf8\ub9ac\uace0 \ub3cc\uc5f0\ubcc0\uc774\uc6d0\uc774 \ub418\ub294 \ubb3c\uc9c8\uc5d0 \uc758\ud574 \ub3cc\uc5f0\ubcc0\uc774\ub97c \uc77c\uc73c\ud0ac \uc218 \uc788\ub2e4. \uc774\ub807\uac8c \ud558\uc5ec \ubc1c\uc0dd\ud558\ub294 \ub3cc\uc5f0\ubcc0\uc774\ub294 \uc720\uc804\uc790\uc758 \uae30\ub2a5\uc774\ub098 \uc0dd\uc131\ubb3c\uc5d0 \uc601\ud5a5\uc744 \uc904 \uc218 \uc788\ub2e4. \ucd08\ud30c\ub9ac\ub97c \uc774\uc6a9\ud55c \ub3cc\uc5f0\ubcc0\uc774 \uc5f0\uad6c\uc758 \uacb0\uacfc \ub3cc\uc5f0\ubcc0\uc774\uc758 \uc57d 70%\ub294 \uac1c\uccb4\uc5d0\uac8c \ud574\ub85c\uc6b4 \ud615\ud0dc\ub85c \ubc1c\ud604\ub418\uc5c8\ub2e4. \uc774\ub7ec\ud55c \ud574\ub85c\uc6b4 \ub3cc\uc5f0\ubcc0\uc774\uac00 \uac1c\uccb4\uc758 \uc138\ud3ec \ub2e8\uc704\uc5d0\uc11c \ubc1c\uc0dd\ud55c \uacbd\uc6b0 \uac1c\uccb4\ub294 DNA \uc218\ub9ac\ub97c \ud1b5\ud574 \ub3cc\uc5f0\ubcc0\uc774\ub97c \uc81c\uac70\ud558\ub3c4\ub85d \uc9c4\ud654\ub418\uc5c8\ub2e4. \uc774\ub97c \ud1b5\ud574 \uc0dd\ubb3c \uac1c\uccb4\ub294 \ubb3c\uc9c8\ub300\uc0ac\uc758 \uacfc\uc815\uc5d0\uc11c \uc138\ud3ec\ub2e8\uc704\uc758 \ub3cc\uc5f0\ubcc0\uc774\ub97c \uc5b5\uc81c\ud558\uc5ec \uacfc\ub3c4\ud55c \ub3cc\uc5f0\ubcc0\uc774\uc758 \ubc1c\uc0dd\uc5d0 \ub530\ub978 \uc704\ud5d8\uc744 \ud68c\ud53c\ud560 \uc218 \uc788\ub2e4. \ud55c\ud3b8, \ub808\ud2b8\ub85c\ubc14\uc774\ub7ec\uc2a4\uc640 \uac19\uc740 \uba87\uba87 \uc885\uc740 \ub3cc\uc5f0\ubcc0\uc774 \ubc1c\uc0dd \ube44\uc728\uc774 \ub192\uc544 \uba87 \uc138\ub300 \uc9c0\ub098\uc9c0 \uc54a\uc544 \ub2e4\uc591\ud55c \ub3cc\uc5f0\ubcc0\uc774\ub97c \uac16\ub294 \uc790\uc2dd \uc138\ub300\uac00 \ub4f1\uc7a5\ud55c\ub2e4. \uc774\ub7ec\ud55c \ub192\uc740 \ub3cc\uc5f0\ubcc0\uc774 \ubc1c\uc0dd\ub960\uc740 \uc778\uac04\uc758 \uba74\uc5ed\uacc4\ub97c \ud68c\ud53c\ud560 \uc218 \uc788\uc5b4 \ubc14\uc774\ub7ec\uc2a4\uc758 \uc0dd\uc874\uc5d0 \uc720\ub9ac\ud558\uac8c \uc791\uc6a9\ud55c\ub2e4.", "paragraph_answer": "\uac8c\ub188\uc5d0\uc11c \ubc1c\uc0dd\ud558\ub294 \ubb34\uc791\uc704\uc801\uc778 \ub3cc\uc5f0\ubcc0\uc774\ub294 \uc720\uc804\ud615\uc9c8\uc758 \uac1c\uccb4\ucc28\ub97c \uc77c\uc73c\ud0a8\ub2e4. \uc138\ud3ec\uc548\uc758 \uac8c\ub188\uc744 \uc774\ub8e8\ub294 DNA \uc5fc\uae30 \uc11c\uc5f4 \uc740 \uac10\uc218 \ubd84\uc5f4\uc774\ub098 DNA \ubcf5\uc81c\uc758 \uacfc\uc815\uc5d0\uc11c \ubc29\uc0ac\ub2a5, \ubc14\uc774\ub7ec\uc2a4, \ud2b8\ub80c\uc2a4\ud3ec\uc874, \uadf8\ub9ac\uace0 \ub3cc\uc5f0\ubcc0\uc774\uc6d0\uc774 \ub418\ub294 \ubb3c\uc9c8\uc5d0 \uc758\ud574 \ub3cc\uc5f0\ubcc0\uc774\ub97c \uc77c\uc73c\ud0ac \uc218 \uc788\ub2e4. \uc774\ub807\uac8c \ud558\uc5ec \ubc1c\uc0dd\ud558\ub294 \ub3cc\uc5f0\ubcc0\uc774\ub294 \uc720\uc804\uc790\uc758 \uae30\ub2a5\uc774\ub098 \uc0dd\uc131\ubb3c\uc5d0 \uc601\ud5a5\uc744 \uc904 \uc218 \uc788\ub2e4. \ucd08\ud30c\ub9ac\ub97c \uc774\uc6a9\ud55c \ub3cc\uc5f0\ubcc0\uc774 \uc5f0\uad6c\uc758 \uacb0\uacfc \ub3cc\uc5f0\ubcc0\uc774\uc758 \uc57d 70%\ub294 \uac1c\uccb4\uc5d0\uac8c \ud574\ub85c\uc6b4 \ud615\ud0dc\ub85c \ubc1c\ud604\ub418\uc5c8\ub2e4. \uc774\ub7ec\ud55c \ud574\ub85c\uc6b4 \ub3cc\uc5f0\ubcc0\uc774\uac00 \uac1c\uccb4\uc758 \uc138\ud3ec \ub2e8\uc704\uc5d0\uc11c \ubc1c\uc0dd\ud55c \uacbd\uc6b0 \uac1c\uccb4\ub294 DNA \uc218\ub9ac\ub97c \ud1b5\ud574 \ub3cc\uc5f0\ubcc0\uc774\ub97c \uc81c\uac70\ud558\ub3c4\ub85d \uc9c4\ud654\ub418\uc5c8\ub2e4. \uc774\ub97c \ud1b5\ud574 \uc0dd\ubb3c \uac1c\uccb4\ub294 \ubb3c\uc9c8\ub300\uc0ac\uc758 \uacfc\uc815\uc5d0\uc11c \uc138\ud3ec\ub2e8\uc704\uc758 \ub3cc\uc5f0\ubcc0\uc774\ub97c \uc5b5\uc81c\ud558\uc5ec \uacfc\ub3c4\ud55c \ub3cc\uc5f0\ubcc0\uc774\uc758 \ubc1c\uc0dd\uc5d0 \ub530\ub978 \uc704\ud5d8\uc744 \ud68c\ud53c\ud560 \uc218 \uc788\ub2e4. \ud55c\ud3b8, \ub808\ud2b8\ub85c\ubc14\uc774\ub7ec\uc2a4\uc640 \uac19\uc740 \uba87\uba87 \uc885\uc740 \ub3cc\uc5f0\ubcc0\uc774 \ubc1c\uc0dd \ube44\uc728\uc774 \ub192\uc544 \uba87 \uc138\ub300 \uc9c0\ub098\uc9c0 \uc54a\uc544 \ub2e4\uc591\ud55c \ub3cc\uc5f0\ubcc0\uc774\ub97c \uac16\ub294 \uc790\uc2dd \uc138\ub300\uac00 \ub4f1\uc7a5\ud55c\ub2e4. \uc774\ub7ec\ud55c \ub192\uc740 \ub3cc\uc5f0\ubcc0\uc774 \ubc1c\uc0dd\ub960\uc740 \uc778\uac04\uc758 \uba74\uc5ed\uacc4\ub97c \ud68c\ud53c\ud560 \uc218 \uc788\uc5b4 \ubc14\uc774\ub7ec\uc2a4\uc758 \uc0dd\uc874\uc5d0 \uc720\ub9ac\ud558\uac8c \uc791\uc6a9\ud55c\ub2e4.", "sentence_answer": "\uc138\ud3ec\uc548\uc758 \uac8c\ub188\uc744 \uc774\ub8e8\ub294 DNA \uc5fc\uae30 \uc11c\uc5f4 \uc740 \uac10\uc218 \ubd84\uc5f4\uc774\ub098 DNA \ubcf5\uc81c\uc758 \uacfc\uc815\uc5d0\uc11c \ubc29\uc0ac\ub2a5, \ubc14\uc774\ub7ec\uc2a4, \ud2b8\ub80c\uc2a4\ud3ec\uc874, \uadf8\ub9ac\uace0 \ub3cc\uc5f0\ubcc0\uc774\uc6d0\uc774 \ub418\ub294 \ubb3c\uc9c8\uc5d0 \uc758\ud574 \ub3cc\uc5f0\ubcc0\uc774\ub97c \uc77c\uc73c\ud0ac \uc218 \uc788\ub2e4.", "paragraph_id": "6557516-4-0", "paragraph_question": "question: \uac10\uc218 \ubd84\uc5f4\uc774\ub098 DNA \ubcf5\uc81c \uacfc\uc815\uc5d0\uc11c \ub3cc\uc5f0\ubcc0\uc774\uc6d0\uc774 \ub418\ub294 \ubb3c\uc9c8\uc5d0 \uc758\ud574 \ub3cc\uc5f0\ubcc0\uc774\ub97c \uc77c\uc73c\ud0ac \uc218 \uc788\ub294\uac83\uc740?, context: \uac8c\ub188\uc5d0\uc11c \ubc1c\uc0dd\ud558\ub294 \ubb34\uc791\uc704\uc801\uc778 \ub3cc\uc5f0\ubcc0\uc774\ub294 \uc720\uc804\ud615\uc9c8\uc758 \uac1c\uccb4\ucc28\ub97c \uc77c\uc73c\ud0a8\ub2e4. \uc138\ud3ec\uc548\uc758 \uac8c\ub188\uc744 \uc774\ub8e8\ub294 DNA \uc5fc\uae30 \uc11c\uc5f4\uc740 \uac10\uc218 \ubd84\uc5f4\uc774\ub098 DNA \ubcf5\uc81c\uc758 \uacfc\uc815\uc5d0\uc11c \ubc29\uc0ac\ub2a5, \ubc14\uc774\ub7ec\uc2a4, \ud2b8\ub80c\uc2a4\ud3ec\uc874, \uadf8\ub9ac\uace0 \ub3cc\uc5f0\ubcc0\uc774\uc6d0\uc774 \ub418\ub294 \ubb3c\uc9c8\uc5d0 \uc758\ud574 \ub3cc\uc5f0\ubcc0\uc774\ub97c \uc77c\uc73c\ud0ac \uc218 \uc788\ub2e4. \uc774\ub807\uac8c \ud558\uc5ec \ubc1c\uc0dd\ud558\ub294 \ub3cc\uc5f0\ubcc0\uc774\ub294 \uc720\uc804\uc790\uc758 \uae30\ub2a5\uc774\ub098 \uc0dd\uc131\ubb3c\uc5d0 \uc601\ud5a5\uc744 \uc904 \uc218 \uc788\ub2e4. \ucd08\ud30c\ub9ac\ub97c \uc774\uc6a9\ud55c \ub3cc\uc5f0\ubcc0\uc774 \uc5f0\uad6c\uc758 \uacb0\uacfc \ub3cc\uc5f0\ubcc0\uc774\uc758 \uc57d 70%\ub294 \uac1c\uccb4\uc5d0\uac8c \ud574\ub85c\uc6b4 \ud615\ud0dc\ub85c \ubc1c\ud604\ub418\uc5c8\ub2e4. \uc774\ub7ec\ud55c \ud574\ub85c\uc6b4 \ub3cc\uc5f0\ubcc0\uc774\uac00 \uac1c\uccb4\uc758 \uc138\ud3ec \ub2e8\uc704\uc5d0\uc11c \ubc1c\uc0dd\ud55c \uacbd\uc6b0 \uac1c\uccb4\ub294 DNA \uc218\ub9ac\ub97c \ud1b5\ud574 \ub3cc\uc5f0\ubcc0\uc774\ub97c \uc81c\uac70\ud558\ub3c4\ub85d \uc9c4\ud654\ub418\uc5c8\ub2e4. \uc774\ub97c \ud1b5\ud574 \uc0dd\ubb3c \uac1c\uccb4\ub294 \ubb3c\uc9c8\ub300\uc0ac\uc758 \uacfc\uc815\uc5d0\uc11c \uc138\ud3ec\ub2e8\uc704\uc758 \ub3cc\uc5f0\ubcc0\uc774\ub97c \uc5b5\uc81c\ud558\uc5ec \uacfc\ub3c4\ud55c \ub3cc\uc5f0\ubcc0\uc774\uc758 \ubc1c\uc0dd\uc5d0 \ub530\ub978 \uc704\ud5d8\uc744 \ud68c\ud53c\ud560 \uc218 \uc788\ub2e4. \ud55c\ud3b8, \ub808\ud2b8\ub85c\ubc14\uc774\ub7ec\uc2a4\uc640 \uac19\uc740 \uba87\uba87 \uc885\uc740 \ub3cc\uc5f0\ubcc0\uc774 \ubc1c\uc0dd \ube44\uc728\uc774 \ub192\uc544 \uba87 \uc138\ub300 \uc9c0\ub098\uc9c0 \uc54a\uc544 \ub2e4\uc591\ud55c \ub3cc\uc5f0\ubcc0\uc774\ub97c \uac16\ub294 \uc790\uc2dd \uc138\ub300\uac00 \ub4f1\uc7a5\ud55c\ub2e4. \uc774\ub7ec\ud55c \ub192\uc740 \ub3cc\uc5f0\ubcc0\uc774 \ubc1c\uc0dd\ub960\uc740 \uc778\uac04\uc758 \uba74\uc5ed\uacc4\ub97c \ud68c\ud53c\ud560 \uc218 \uc788\uc5b4 \ubc14\uc774\ub7ec\uc2a4\uc758 \uc0dd\uc874\uc5d0 \uc720\ub9ac\ud558\uac8c \uc791\uc6a9\ud55c\ub2e4."} {"question": "2014\ub144 8\uc6d4\uc5d0 \ub300\uad6c FC\uc758 \ub2e8\uc7a5\uc73c\ub85c \ubd80\uc784\ud55c \uc0ac\ub78c\uc740 \ub204\uad6c\uc778\uac00?", "paragraph": "K\ub9ac\uadf8 \ucc4c\ub9b0\uc9c0\uc5d0\uc11c\uc758 \uccab \uc2dc\uc98c\uc778 2014 \uc2dc\uc98c\uc774 \ud55c\ucc3d\uc774\ub358 2014\ub144 8\uc6d4\uc5d0 \uc870\uad11\ub798 \u524d \ub300\ud55c\ubbfc\uad6d \uad6d\uac00\ub300\ud45c\ud300 \uac10\ub3c5\uc774 \ub2e8\uc7a5\uc73c\ub85c \ubd80\uc784\ud558\uc600\uace0, 2\ub2ec \ud6c4\uc5d0\ub294 \ub300\ud45c\uc774\uc0ac\ub97c \uacb8\uc784\ud558\uac8c \ub418\uc5c8\ub2e4. \ud55c\ud3b8 \ucc4c\ub9b0\uc9c0\uc5d0\uc11c\ub294 7\uc704\ub77c\ub294 \uc2e4\ub9dd\uc2a4\ub7ec\uc6b4 \uc131\uc801\uc73c\ub85c \uc2dc\uc98c\uc744 \ub9c8\ubb34\ub9ac\ud558\uc600\uace0, \ucd5c\ub355\uc8fc \uac10\ub3c5\uc740 \uc784\uae30\ub97c \ucc44\uc6b0\uc9c0 \ubabb\ud55c \ucc44 \ud574\uc784\ub410\ub2e4. 2015 \uc2dc\uc98c\uc744 \uc55e\ub450\uace0 2010\ub144\ubd80\ud130 2011\ub144\uae4c\uc9c0 \ub300\uad6c\ub97c \uc774\ub04c\uc5c8\ub358 \uc774\uc601\uc9c4 \uac10\ub3c5\uc774 \ubcf5\uadc0\ud558\uc600\ub2e4. 2015 \uc2dc\uc98c \uafb8\uc900\ud788 \uc0c1\uc704\uad8c\uc744 \uc720\uc9c0\ud558\ub2e4 35\ub77c\uc6b4\ub4dc\uc5d0\ub294 \ub9c8\uce68\ub0b4 1\uc704\uc5d0 \uc62c\ub77c\uc130\ub2e4. \ud558\uc9c0\ub9cc \ub9ac\uadf8 2\uacbd\uae30\ub97c \ub0a8\uae30\uace0 2\uc704\ub85c \ub5a8\uc5b4\uc9c0\uba70 \uc6b0\uc2b9\uacfc K\ub9ac\uadf8 \ud074\ub798\uc2dd \uc790\ub3d9 \uc2b9\uaca9\uc758 \uae30\ud68c\ub97c \ub193\uce5c \ucc44 \uc2b9\uac15 \ud50c\ub808\uc774\uc624\ud504\uc5d0 \ucc38\uac00\ud558\uc600\ub2e4. \ud50c\ub808\uc774\uc624\ud504\uc5d0\uc11c \ub2e4\uc2dc \uc2b9\uaca9\uc758 \uae30\ud68c\ub97c \ub178\ub838\uc73c\ub098 \uc218\uc6d0 FC\uc5d0 2-1\ub85c \ud328\ud558\uba70 \uc2dc\uc98c\uc744 \ub9c8\uac10\ud558\uc600\ub2e4.", "answer": "\uc870\uad11\ub798", "sentence": "K\ub9ac\uadf8 \ucc4c\ub9b0\uc9c0\uc5d0\uc11c\uc758 \uccab \uc2dc\uc98c\uc778 2014 \uc2dc\uc98c\uc774 \ud55c\ucc3d\uc774\ub358 2014\ub144 8\uc6d4\uc5d0 \uc870\uad11\ub798 \u524d \ub300\ud55c\ubbfc\uad6d \uad6d\uac00\ub300\ud45c\ud300 \uac10\ub3c5\uc774 \ub2e8\uc7a5\uc73c\ub85c \ubd80\uc784\ud558\uc600\uace0, 2\ub2ec \ud6c4\uc5d0\ub294 \ub300\ud45c\uc774\uc0ac\ub97c \uacb8\uc784\ud558\uac8c \ub418\uc5c8\ub2e4.", "paragraph_sentence": " K\ub9ac\uadf8 \ucc4c\ub9b0\uc9c0\uc5d0\uc11c\uc758 \uccab \uc2dc\uc98c\uc778 2014 \uc2dc\uc98c\uc774 \ud55c\ucc3d\uc774\ub358 2014\ub144 8\uc6d4\uc5d0 \uc870\uad11\ub798 \u524d \ub300\ud55c\ubbfc\uad6d \uad6d\uac00\ub300\ud45c\ud300 \uac10\ub3c5\uc774 \ub2e8\uc7a5\uc73c\ub85c \ubd80\uc784\ud558\uc600\uace0, 2\ub2ec \ud6c4\uc5d0\ub294 \ub300\ud45c\uc774\uc0ac\ub97c \uacb8\uc784\ud558\uac8c \ub418\uc5c8\ub2e4. \ud55c\ud3b8 \ucc4c\ub9b0\uc9c0\uc5d0\uc11c\ub294 7\uc704\ub77c\ub294 \uc2e4\ub9dd\uc2a4\ub7ec\uc6b4 \uc131\uc801\uc73c\ub85c \uc2dc\uc98c\uc744 \ub9c8\ubb34\ub9ac\ud558\uc600\uace0, \ucd5c\ub355\uc8fc \uac10\ub3c5\uc740 \uc784\uae30\ub97c \ucc44\uc6b0\uc9c0 \ubabb\ud55c \ucc44 \ud574\uc784\ub410\ub2e4. 2015 \uc2dc\uc98c\uc744 \uc55e\ub450\uace0 2010\ub144\ubd80\ud130 2011\ub144\uae4c\uc9c0 \ub300\uad6c\ub97c \uc774\ub04c\uc5c8\ub358 \uc774\uc601\uc9c4 \uac10\ub3c5\uc774 \ubcf5\uadc0\ud558\uc600\ub2e4. 2015 \uc2dc\uc98c \uafb8\uc900\ud788 \uc0c1\uc704\uad8c\uc744 \uc720\uc9c0\ud558\ub2e4 35\ub77c\uc6b4\ub4dc\uc5d0\ub294 \ub9c8\uce68\ub0b4 1\uc704\uc5d0 \uc62c\ub77c\uc130\ub2e4. \ud558\uc9c0\ub9cc \ub9ac\uadf8 2\uacbd\uae30\ub97c \ub0a8\uae30\uace0 2\uc704\ub85c \ub5a8\uc5b4\uc9c0\uba70 \uc6b0\uc2b9\uacfc K\ub9ac\uadf8 \ud074\ub798\uc2dd \uc790\ub3d9 \uc2b9\uaca9\uc758 \uae30\ud68c\ub97c \ub193\uce5c \ucc44 \uc2b9\uac15 \ud50c\ub808\uc774\uc624\ud504\uc5d0 \ucc38\uac00\ud558\uc600\ub2e4. \ud50c\ub808\uc774\uc624\ud504\uc5d0\uc11c \ub2e4\uc2dc \uc2b9\uaca9\uc758 \uae30\ud68c\ub97c \ub178\ub838\uc73c\ub098 \uc218\uc6d0 FC\uc5d0 2-1\ub85c \ud328\ud558\uba70 \uc2dc\uc98c\uc744 \ub9c8\uac10\ud558\uc600\ub2e4.", "paragraph_answer": "K\ub9ac\uadf8 \ucc4c\ub9b0\uc9c0\uc5d0\uc11c\uc758 \uccab \uc2dc\uc98c\uc778 2014 \uc2dc\uc98c\uc774 \ud55c\ucc3d\uc774\ub358 2014\ub144 8\uc6d4\uc5d0 \uc870\uad11\ub798 \u524d \ub300\ud55c\ubbfc\uad6d \uad6d\uac00\ub300\ud45c\ud300 \uac10\ub3c5\uc774 \ub2e8\uc7a5\uc73c\ub85c \ubd80\uc784\ud558\uc600\uace0, 2\ub2ec \ud6c4\uc5d0\ub294 \ub300\ud45c\uc774\uc0ac\ub97c \uacb8\uc784\ud558\uac8c \ub418\uc5c8\ub2e4. \ud55c\ud3b8 \ucc4c\ub9b0\uc9c0\uc5d0\uc11c\ub294 7\uc704\ub77c\ub294 \uc2e4\ub9dd\uc2a4\ub7ec\uc6b4 \uc131\uc801\uc73c\ub85c \uc2dc\uc98c\uc744 \ub9c8\ubb34\ub9ac\ud558\uc600\uace0, \ucd5c\ub355\uc8fc \uac10\ub3c5\uc740 \uc784\uae30\ub97c \ucc44\uc6b0\uc9c0 \ubabb\ud55c \ucc44 \ud574\uc784\ub410\ub2e4. 2015 \uc2dc\uc98c\uc744 \uc55e\ub450\uace0 2010\ub144\ubd80\ud130 2011\ub144\uae4c\uc9c0 \ub300\uad6c\ub97c \uc774\ub04c\uc5c8\ub358 \uc774\uc601\uc9c4 \uac10\ub3c5\uc774 \ubcf5\uadc0\ud558\uc600\ub2e4. 2015 \uc2dc\uc98c \uafb8\uc900\ud788 \uc0c1\uc704\uad8c\uc744 \uc720\uc9c0\ud558\ub2e4 35\ub77c\uc6b4\ub4dc\uc5d0\ub294 \ub9c8\uce68\ub0b4 1\uc704\uc5d0 \uc62c\ub77c\uc130\ub2e4. \ud558\uc9c0\ub9cc \ub9ac\uadf8 2\uacbd\uae30\ub97c \ub0a8\uae30\uace0 2\uc704\ub85c \ub5a8\uc5b4\uc9c0\uba70 \uc6b0\uc2b9\uacfc K\ub9ac\uadf8 \ud074\ub798\uc2dd \uc790\ub3d9 \uc2b9\uaca9\uc758 \uae30\ud68c\ub97c \ub193\uce5c \ucc44 \uc2b9\uac15 \ud50c\ub808\uc774\uc624\ud504\uc5d0 \ucc38\uac00\ud558\uc600\ub2e4. \ud50c\ub808\uc774\uc624\ud504\uc5d0\uc11c \ub2e4\uc2dc \uc2b9\uaca9\uc758 \uae30\ud68c\ub97c \ub178\ub838\uc73c\ub098 \uc218\uc6d0 FC\uc5d0 2-1\ub85c \ud328\ud558\uba70 \uc2dc\uc98c\uc744 \ub9c8\uac10\ud558\uc600\ub2e4.", "sentence_answer": "K\ub9ac\uadf8 \ucc4c\ub9b0\uc9c0\uc5d0\uc11c\uc758 \uccab \uc2dc\uc98c\uc778 2014 \uc2dc\uc98c\uc774 \ud55c\ucc3d\uc774\ub358 2014\ub144 8\uc6d4\uc5d0 \uc870\uad11\ub798 \u524d \ub300\ud55c\ubbfc\uad6d \uad6d\uac00\ub300\ud45c\ud300 \uac10\ub3c5\uc774 \ub2e8\uc7a5\uc73c\ub85c \ubd80\uc784\ud558\uc600\uace0, 2\ub2ec \ud6c4\uc5d0\ub294 \ub300\ud45c\uc774\uc0ac\ub97c \uacb8\uc784\ud558\uac8c \ub418\uc5c8\ub2e4.", "paragraph_id": "6546195-5-0", "paragraph_question": "question: 2014\ub144 8\uc6d4\uc5d0 \ub300\uad6c FC\uc758 \ub2e8\uc7a5\uc73c\ub85c \ubd80\uc784\ud55c \uc0ac\ub78c\uc740 \ub204\uad6c\uc778\uac00?, context: K\ub9ac\uadf8 \ucc4c\ub9b0\uc9c0\uc5d0\uc11c\uc758 \uccab \uc2dc\uc98c\uc778 2014 \uc2dc\uc98c\uc774 \ud55c\ucc3d\uc774\ub358 2014\ub144 8\uc6d4\uc5d0 \uc870\uad11\ub798 \u524d \ub300\ud55c\ubbfc\uad6d \uad6d\uac00\ub300\ud45c\ud300 \uac10\ub3c5\uc774 \ub2e8\uc7a5\uc73c\ub85c \ubd80\uc784\ud558\uc600\uace0, 2\ub2ec \ud6c4\uc5d0\ub294 \ub300\ud45c\uc774\uc0ac\ub97c \uacb8\uc784\ud558\uac8c \ub418\uc5c8\ub2e4. \ud55c\ud3b8 \ucc4c\ub9b0\uc9c0\uc5d0\uc11c\ub294 7\uc704\ub77c\ub294 \uc2e4\ub9dd\uc2a4\ub7ec\uc6b4 \uc131\uc801\uc73c\ub85c \uc2dc\uc98c\uc744 \ub9c8\ubb34\ub9ac\ud558\uc600\uace0, \ucd5c\ub355\uc8fc \uac10\ub3c5\uc740 \uc784\uae30\ub97c \ucc44\uc6b0\uc9c0 \ubabb\ud55c \ucc44 \ud574\uc784\ub410\ub2e4. 2015 \uc2dc\uc98c\uc744 \uc55e\ub450\uace0 2010\ub144\ubd80\ud130 2011\ub144\uae4c\uc9c0 \ub300\uad6c\ub97c \uc774\ub04c\uc5c8\ub358 \uc774\uc601\uc9c4 \uac10\ub3c5\uc774 \ubcf5\uadc0\ud558\uc600\ub2e4. 2015 \uc2dc\uc98c \uafb8\uc900\ud788 \uc0c1\uc704\uad8c\uc744 \uc720\uc9c0\ud558\ub2e4 35\ub77c\uc6b4\ub4dc\uc5d0\ub294 \ub9c8\uce68\ub0b4 1\uc704\uc5d0 \uc62c\ub77c\uc130\ub2e4. \ud558\uc9c0\ub9cc \ub9ac\uadf8 2\uacbd\uae30\ub97c \ub0a8\uae30\uace0 2\uc704\ub85c \ub5a8\uc5b4\uc9c0\uba70 \uc6b0\uc2b9\uacfc K\ub9ac\uadf8 \ud074\ub798\uc2dd \uc790\ub3d9 \uc2b9\uaca9\uc758 \uae30\ud68c\ub97c \ub193\uce5c \ucc44 \uc2b9\uac15 \ud50c\ub808\uc774\uc624\ud504\uc5d0 \ucc38\uac00\ud558\uc600\ub2e4. \ud50c\ub808\uc774\uc624\ud504\uc5d0\uc11c \ub2e4\uc2dc \uc2b9\uaca9\uc758 \uae30\ud68c\ub97c \ub178\ub838\uc73c\ub098 \uc218\uc6d0 FC\uc5d0 2-1\ub85c \ud328\ud558\uba70 \uc2dc\uc98c\uc744 \ub9c8\uac10\ud558\uc600\ub2e4."} {"question": "2\ubc14\uc6b8\uc758 \ubb3c\ubc1b\uc774\uac00 \uc811\ud558\ub294 \ub370\ub294 \ubb34\uc5c7\uc73c\ub85c \ub418\uc5b4 \uc788\ub294\uac00?", "paragraph": "\ucc28\ucd95\uc774 \uc218\ud3c9\uacfc \uc5f0\uc9c1(\u925b\u76f4) \ub450 \uac00\uc9c0 \ud615\ud0dc\uac00 \uc788\ub294\ub370, \ubcf4\ud1b5 \uc804\uc790\ub97c \uc0ac\uc6a9\ud55c\ub2e4. \ub610 \ud558\ub098\uc758 \ucc28\ucd95\uc5d0 \ub0a0\uac1c\uc218\ub808\uac00 \ud55c\uac1c\ub85c \ud55c\uc815\ub418\uc9c0 \uc54a\uace0 \ub450\uac1c \uc788\ub294 \uac83\ub3c4 \uc788\ub2e4. \uadf8\ub9ac\uace0, \ud558\ub098\uc758 \ub0a0\uac1c \uc218\ub808\uc5d0 \ubb3c\uc744 \ubd80\ub52a\uce58\uac8c \ud558\ub294 \ub178\uc990\uc758 \uc218\ub3c4 \uac00\uc9c0\uac01\uc0c9\uc774\ub2e4. \uac01 \ubb3c\ubc1b\uc774\ub294 2\ubc14\uc6b8\ud615\uc774\uba70, 2\ubc14\uc6b8\uc774 \uc811\ud558\ub294 \ub370\uac00 \ubb3c\uc790\ub984\uc73c\ub85c \ub418\uc5b4 \uc788\ub2e4. \uc774 \ubb3c\uc790\ub984\uacfc \ubc14\uc6b8 \uc678\uc5f0(\u5916\u7de3)\uc774 \ub9cc\ub098\ub294 \ubd80\ubd84\uc774 \ubfb0\uc871\ud558\uac8c \ub418\uc5b4 \uc788\uace0, \ubd84\ub958\uac00 \uc774 \ubd80\ubd84\uc5d0 \ucda9\ub3cc\ud574\uc11c \uac04\uc12d\ub418\uc9c0 \uc54a\uace0 \ubc14\uc6b8\uc758 \uc548\ucabd\uba74\uc5d0 \ub530\ub77c \ud758\ub7ec\uc11c \ubc29\ud5a5\uc744 \ubc14\uafb8\uc5b4 \ubb3c\ubc1b\uc774\ub97c \ub5a0\ub098\uac8c \ud55c\ub2e4. \ub178\uc990 \uc911\uc559\uc5d0\ub294 \ub2c8\ub4e4\ud310(\u74e3)\uc774 \uc788\ub294\ub370, \uc774 \ud310\uc740 \ub808\ubc84\uc5d0 \uc758\ud574 \ub098\uc624\uace0 \ub4e4\uc5b4\uac00\uac8c \ub418\uc5b4 \uc788\uc73c\uba70, \ub610\ud55c \ub178\uc990\ucd9c\uad6c\uc640 \ubc38\ube0c \uc0ac\uc774\uc758 \uba74\uc801\uc744 \ubcc0\ud654\uc2dc\ud0b4\uc73c\ub85c\uc368 \ubd84\ub958\uc758 \uc218\ub7c9(\u6c34\u91cf)\uc744 \uc870\uc808\ud560 \uc218 \uc788\ub3c4\ub85d \ub418\uc5b4 \uc788\ub2e4. \ub2e4\uc74c\uc73c\ub85c \ud3a0\ud1a4(Pelton) \uc218\ucc28\uc5d0\uc11c\uc758 \ubb3c\uc758 \uc791\uc6a9\uc744 \uc0dd\uac01\ud574 \ubcf4\uc790. \uc18d\ub3c4\uc758 \ubd84\ub958\uac00 \ubb3c\ubc1b\uc774\uc5d0 \ucda9\ub3d9\ud558\uc5ec, \ubb3c\ubc1b\uc774\uc5d0 \uc18d\ub3c4 \ub97c \ubd80\uc5ec\ud558\uace0 \ub9cc\uace1\uba74\uc5d0\uc11c \ubc18\uc804(\u53cd\u8f49)\ud558\uc5ec \ud758\ub7ec\ub098\uac00\ub294 \uac83\uc774\ub77c \ud558\uba74 \ub0a0\uac1c\uc218\ub808\uc758 \ud6a8\uc728(\uc218\ub825\ud6a8\uc728) \uc740 \ub2e4\uc74c \uc2dd\uc73c\ub85c \ub098\ud0c0\ub0b4\uac8c \ub41c\ub2e4.", "answer": "\ubb3c\uc790\ub984", "sentence": "\uac01 \ubb3c\ubc1b\uc774\ub294 2\ubc14\uc6b8\ud615\uc774\uba70, 2\ubc14\uc6b8\uc774 \uc811\ud558\ub294 \ub370\uac00 \ubb3c\uc790\ub984 \uc73c\ub85c \ub418\uc5b4 \uc788\ub2e4.", "paragraph_sentence": "\ucc28\ucd95\uc774 \uc218\ud3c9\uacfc \uc5f0\uc9c1(\u925b\u76f4) \ub450 \uac00\uc9c0 \ud615\ud0dc\uac00 \uc788\ub294\ub370, \ubcf4\ud1b5 \uc804\uc790\ub97c \uc0ac\uc6a9\ud55c\ub2e4. \ub610 \ud558\ub098\uc758 \ucc28\ucd95\uc5d0 \ub0a0\uac1c\uc218\ub808\uac00 \ud55c\uac1c\ub85c \ud55c\uc815\ub418\uc9c0 \uc54a\uace0 \ub450\uac1c \uc788\ub294 \uac83\ub3c4 \uc788\ub2e4. \uadf8\ub9ac\uace0, \ud558\ub098\uc758 \ub0a0\uac1c \uc218\ub808\uc5d0 \ubb3c\uc744 \ubd80\ub52a\uce58\uac8c \ud558\ub294 \ub178\uc990\uc758 \uc218\ub3c4 \uac00\uc9c0\uac01\uc0c9\uc774\ub2e4. \uac01 \ubb3c\ubc1b\uc774\ub294 2\ubc14\uc6b8\ud615\uc774\uba70, 2\ubc14\uc6b8\uc774 \uc811\ud558\ub294 \ub370\uac00 \ubb3c\uc790\ub984 \uc73c\ub85c \ub418\uc5b4 \uc788\ub2e4. \uc774 \ubb3c\uc790\ub984\uacfc \ubc14\uc6b8 \uc678\uc5f0(\u5916\u7de3)\uc774 \ub9cc\ub098\ub294 \ubd80\ubd84\uc774 \ubfb0\uc871\ud558\uac8c \ub418\uc5b4 \uc788\uace0, \ubd84\ub958\uac00 \uc774 \ubd80\ubd84\uc5d0 \ucda9\ub3cc\ud574\uc11c \uac04\uc12d\ub418\uc9c0 \uc54a\uace0 \ubc14\uc6b8\uc758 \uc548\ucabd\uba74\uc5d0 \ub530\ub77c \ud758\ub7ec\uc11c \ubc29\ud5a5\uc744 \ubc14\uafb8\uc5b4 \ubb3c\ubc1b\uc774\ub97c \ub5a0\ub098\uac8c \ud55c\ub2e4. \ub178\uc990 \uc911\uc559\uc5d0\ub294 \ub2c8\ub4e4\ud310(\u74e3)\uc774 \uc788\ub294\ub370, \uc774 \ud310\uc740 \ub808\ubc84\uc5d0 \uc758\ud574 \ub098\uc624\uace0 \ub4e4\uc5b4\uac00\uac8c \ub418\uc5b4 \uc788\uc73c\uba70, \ub610\ud55c \ub178\uc990\ucd9c\uad6c\uc640 \ubc38\ube0c \uc0ac\uc774\uc758 \uba74\uc801\uc744 \ubcc0\ud654\uc2dc\ud0b4\uc73c\ub85c\uc368 \ubd84\ub958\uc758 \uc218\ub7c9(\u6c34\u91cf)\uc744 \uc870\uc808\ud560 \uc218 \uc788\ub3c4\ub85d \ub418\uc5b4 \uc788\ub2e4. \ub2e4\uc74c\uc73c\ub85c \ud3a0\ud1a4(Pelton) \uc218\ucc28\uc5d0\uc11c\uc758 \ubb3c\uc758 \uc791\uc6a9\uc744 \uc0dd\uac01\ud574 \ubcf4\uc790. \uc18d\ub3c4\uc758 \ubd84\ub958\uac00 \ubb3c\ubc1b\uc774\uc5d0 \ucda9\ub3d9\ud558\uc5ec, \ubb3c\ubc1b\uc774\uc5d0 \uc18d\ub3c4 \ub97c \ubd80\uc5ec\ud558\uace0 \ub9cc\uace1\uba74\uc5d0\uc11c \ubc18\uc804(\u53cd\u8f49)\ud558\uc5ec \ud758\ub7ec\ub098\uac00\ub294 \uac83\uc774\ub77c \ud558\uba74 \ub0a0\uac1c\uc218\ub808\uc758 \ud6a8\uc728(\uc218\ub825\ud6a8\uc728) \uc740 \ub2e4\uc74c \uc2dd\uc73c\ub85c \ub098\ud0c0\ub0b4\uac8c \ub41c\ub2e4.", "paragraph_answer": "\ucc28\ucd95\uc774 \uc218\ud3c9\uacfc \uc5f0\uc9c1(\u925b\u76f4) \ub450 \uac00\uc9c0 \ud615\ud0dc\uac00 \uc788\ub294\ub370, \ubcf4\ud1b5 \uc804\uc790\ub97c \uc0ac\uc6a9\ud55c\ub2e4. \ub610 \ud558\ub098\uc758 \ucc28\ucd95\uc5d0 \ub0a0\uac1c\uc218\ub808\uac00 \ud55c\uac1c\ub85c \ud55c\uc815\ub418\uc9c0 \uc54a\uace0 \ub450\uac1c \uc788\ub294 \uac83\ub3c4 \uc788\ub2e4. \uadf8\ub9ac\uace0, \ud558\ub098\uc758 \ub0a0\uac1c \uc218\ub808\uc5d0 \ubb3c\uc744 \ubd80\ub52a\uce58\uac8c \ud558\ub294 \ub178\uc990\uc758 \uc218\ub3c4 \uac00\uc9c0\uac01\uc0c9\uc774\ub2e4. \uac01 \ubb3c\ubc1b\uc774\ub294 2\ubc14\uc6b8\ud615\uc774\uba70, 2\ubc14\uc6b8\uc774 \uc811\ud558\ub294 \ub370\uac00 \ubb3c\uc790\ub984 \uc73c\ub85c \ub418\uc5b4 \uc788\ub2e4. \uc774 \ubb3c\uc790\ub984\uacfc \ubc14\uc6b8 \uc678\uc5f0(\u5916\u7de3)\uc774 \ub9cc\ub098\ub294 \ubd80\ubd84\uc774 \ubfb0\uc871\ud558\uac8c \ub418\uc5b4 \uc788\uace0, \ubd84\ub958\uac00 \uc774 \ubd80\ubd84\uc5d0 \ucda9\ub3cc\ud574\uc11c \uac04\uc12d\ub418\uc9c0 \uc54a\uace0 \ubc14\uc6b8\uc758 \uc548\ucabd\uba74\uc5d0 \ub530\ub77c \ud758\ub7ec\uc11c \ubc29\ud5a5\uc744 \ubc14\uafb8\uc5b4 \ubb3c\ubc1b\uc774\ub97c \ub5a0\ub098\uac8c \ud55c\ub2e4. \ub178\uc990 \uc911\uc559\uc5d0\ub294 \ub2c8\ub4e4\ud310(\u74e3)\uc774 \uc788\ub294\ub370, \uc774 \ud310\uc740 \ub808\ubc84\uc5d0 \uc758\ud574 \ub098\uc624\uace0 \ub4e4\uc5b4\uac00\uac8c \ub418\uc5b4 \uc788\uc73c\uba70, \ub610\ud55c \ub178\uc990\ucd9c\uad6c\uc640 \ubc38\ube0c \uc0ac\uc774\uc758 \uba74\uc801\uc744 \ubcc0\ud654\uc2dc\ud0b4\uc73c\ub85c\uc368 \ubd84\ub958\uc758 \uc218\ub7c9(\u6c34\u91cf)\uc744 \uc870\uc808\ud560 \uc218 \uc788\ub3c4\ub85d \ub418\uc5b4 \uc788\ub2e4. \ub2e4\uc74c\uc73c\ub85c \ud3a0\ud1a4(Pelton) \uc218\ucc28\uc5d0\uc11c\uc758 \ubb3c\uc758 \uc791\uc6a9\uc744 \uc0dd\uac01\ud574 \ubcf4\uc790. \uc18d\ub3c4\uc758 \ubd84\ub958\uac00 \ubb3c\ubc1b\uc774\uc5d0 \ucda9\ub3d9\ud558\uc5ec, \ubb3c\ubc1b\uc774\uc5d0 \uc18d\ub3c4 \ub97c \ubd80\uc5ec\ud558\uace0 \ub9cc\uace1\uba74\uc5d0\uc11c \ubc18\uc804(\u53cd\u8f49)\ud558\uc5ec \ud758\ub7ec\ub098\uac00\ub294 \uac83\uc774\ub77c \ud558\uba74 \ub0a0\uac1c\uc218\ub808\uc758 \ud6a8\uc728(\uc218\ub825\ud6a8\uc728) \uc740 \ub2e4\uc74c \uc2dd\uc73c\ub85c \ub098\ud0c0\ub0b4\uac8c \ub41c\ub2e4.", "sentence_answer": "\uac01 \ubb3c\ubc1b\uc774\ub294 2\ubc14\uc6b8\ud615\uc774\uba70, 2\ubc14\uc6b8\uc774 \uc811\ud558\ub294 \ub370\uac00 \ubb3c\uc790\ub984 \uc73c\ub85c \ub418\uc5b4 \uc788\ub2e4.", "paragraph_id": "6518784-2-1", "paragraph_question": "question: 2\ubc14\uc6b8\uc758 \ubb3c\ubc1b\uc774\uac00 \uc811\ud558\ub294 \ub370\ub294 \ubb34\uc5c7\uc73c\ub85c \ub418\uc5b4 \uc788\ub294\uac00?, context: \ucc28\ucd95\uc774 \uc218\ud3c9\uacfc \uc5f0\uc9c1(\u925b\u76f4) \ub450 \uac00\uc9c0 \ud615\ud0dc\uac00 \uc788\ub294\ub370, \ubcf4\ud1b5 \uc804\uc790\ub97c \uc0ac\uc6a9\ud55c\ub2e4. \ub610 \ud558\ub098\uc758 \ucc28\ucd95\uc5d0 \ub0a0\uac1c\uc218\ub808\uac00 \ud55c\uac1c\ub85c \ud55c\uc815\ub418\uc9c0 \uc54a\uace0 \ub450\uac1c \uc788\ub294 \uac83\ub3c4 \uc788\ub2e4. \uadf8\ub9ac\uace0, \ud558\ub098\uc758 \ub0a0\uac1c \uc218\ub808\uc5d0 \ubb3c\uc744 \ubd80\ub52a\uce58\uac8c \ud558\ub294 \ub178\uc990\uc758 \uc218\ub3c4 \uac00\uc9c0\uac01\uc0c9\uc774\ub2e4. \uac01 \ubb3c\ubc1b\uc774\ub294 2\ubc14\uc6b8\ud615\uc774\uba70, 2\ubc14\uc6b8\uc774 \uc811\ud558\ub294 \ub370\uac00 \ubb3c\uc790\ub984\uc73c\ub85c \ub418\uc5b4 \uc788\ub2e4. \uc774 \ubb3c\uc790\ub984\uacfc \ubc14\uc6b8 \uc678\uc5f0(\u5916\u7de3)\uc774 \ub9cc\ub098\ub294 \ubd80\ubd84\uc774 \ubfb0\uc871\ud558\uac8c \ub418\uc5b4 \uc788\uace0, \ubd84\ub958\uac00 \uc774 \ubd80\ubd84\uc5d0 \ucda9\ub3cc\ud574\uc11c \uac04\uc12d\ub418\uc9c0 \uc54a\uace0 \ubc14\uc6b8\uc758 \uc548\ucabd\uba74\uc5d0 \ub530\ub77c \ud758\ub7ec\uc11c \ubc29\ud5a5\uc744 \ubc14\uafb8\uc5b4 \ubb3c\ubc1b\uc774\ub97c \ub5a0\ub098\uac8c \ud55c\ub2e4. \ub178\uc990 \uc911\uc559\uc5d0\ub294 \ub2c8\ub4e4\ud310(\u74e3)\uc774 \uc788\ub294\ub370, \uc774 \ud310\uc740 \ub808\ubc84\uc5d0 \uc758\ud574 \ub098\uc624\uace0 \ub4e4\uc5b4\uac00\uac8c \ub418\uc5b4 \uc788\uc73c\uba70, \ub610\ud55c \ub178\uc990\ucd9c\uad6c\uc640 \ubc38\ube0c \uc0ac\uc774\uc758 \uba74\uc801\uc744 \ubcc0\ud654\uc2dc\ud0b4\uc73c\ub85c\uc368 \ubd84\ub958\uc758 \uc218\ub7c9(\u6c34\u91cf)\uc744 \uc870\uc808\ud560 \uc218 \uc788\ub3c4\ub85d \ub418\uc5b4 \uc788\ub2e4. \ub2e4\uc74c\uc73c\ub85c \ud3a0\ud1a4(Pelton) \uc218\ucc28\uc5d0\uc11c\uc758 \ubb3c\uc758 \uc791\uc6a9\uc744 \uc0dd\uac01\ud574 \ubcf4\uc790. \uc18d\ub3c4\uc758 \ubd84\ub958\uac00 \ubb3c\ubc1b\uc774\uc5d0 \ucda9\ub3d9\ud558\uc5ec, \ubb3c\ubc1b\uc774\uc5d0 \uc18d\ub3c4 \ub97c \ubd80\uc5ec\ud558\uace0 \ub9cc\uace1\uba74\uc5d0\uc11c \ubc18\uc804(\u53cd\u8f49)\ud558\uc5ec \ud758\ub7ec\ub098\uac00\ub294 \uac83\uc774\ub77c \ud558\uba74 \ub0a0\uac1c\uc218\ub808\uc758 \ud6a8\uc728(\uc218\ub825\ud6a8\uc728) \uc740 \ub2e4\uc74c \uc2dd\uc73c\ub85c \ub098\ud0c0\ub0b4\uac8c \ub41c\ub2e4."} {"question": "\uadf8\ub79c\ub4dc \uce90\ub2c8\uc5b8 \uad6d\ub9bd\uacf5\uc6d0\uc758 \ub178\uc2a4\ub9bc\uacfc \uc0ac\uc6b0\uc2a4\ub9bc\uc758 \uc0ac\uc774 \uac70\ub9ac\ub294 \uba87 \ub9c8\uc77c\uc778\uac00?", "paragraph": "\uacf5\uc6d0\uc740 1904\ud3c9\ubc29\ub9c8\uc77c(4930\ud3c9\ubc29km)\uc758 \ubc29\ub300\ud55c \uba74\uc801\uc744 \ucc28\uc9c0\ud55c\ub2e4. \ucf5c\ub85c\ub77c\ub3c4 \uac15\uc744 \uc0ac\uc774\uc5d0 \ub450\uace0 \ub0a8\uacfc \ubd81\uc758 \ud611\uace1\uc73c\ub85c \ub098\ub258\uc5b4 \uc788\ub2e4. \uacf5\uc6d0\uc9c0\uc5ed \ub0b4\uc5d0\uc11c \ucf5c\ub85c\ub77c\ub3c4 \uac15\uc758 \uae38\uc774\uac00 \ubb34\ub824277\ub9c8\uc77c(443km)\ub85c \ud55c\uad6d\uc758 \ud55c\uac15 \uae38\uc774 (494km)\uc5d0 \uac00\uae5d\ub2e4. \uce90\ub2c8\uc5b8\uc758 \ud3ed\uc740 \uc81c\uc77c \uc881\uc740 \uacf3\uc740 180m\uc5d0\uc11c \uc81c\uc77c \ub113\uc740 \uacf3\uc740 30km(600ft~18\ub9c8\uc77c)\uae4c\uc9c0 \ub418\uba70 \ud611\uace1\uc758 \uae4a\uc774\ub294 \uc57d 1\ub9c8\uc77c(1.6km)\uc774 \ub418\ub294 \ud611\uace1\uc774\ub2e4. \uc0ac\uc6b0\uc2a4\ub9bc(South Rim)\uc740 \ud574\ubc1c 2300m\uc758 \uace0\uc6d0\uc9c0\ub300\uc774\uba70 \uac15\uc6b0\ub7c9\uc774 \uc801\uc740 \ud3b8\uc774\ub098 \ub178\uc2a4\ub9bc(Nouth Rim)\uc740 \ud574\ubc1c 2700m\uac00 \ub418\uc5b4 \ube44\uad50\uc801 \ube44\uc640 \ub208\uc774 \ub9ce\uc774 \ub0b4\ub9b0\ub2e4. \ub530\ub77c\uc11c \ubd81\ucabd \ub178\uc2a4\ub9bc\uc740 \ub0a8\ucabd\uc758 \uc0ac\uc6b0\uc2a4\ub9bc\ubcf4\ub2e4 \ub354 \uc232\uc774 \uc6b8\ucc3d\ud558\uace0 \uae30\uc628\ub3c4 \ub0ae\ub2e4. \ub178\uc2a4\ub9bc\uacfc \uc0ac\uc6b0\uc2a4\ub9bc \uc0ac\uc774\uc758 \uac70\ub9ac\ub294 \uae4c\ub9c8\uadc0\uc5d0\uac8c\ub294 \ubd88\uacfc 10~18\ub9c8\uc77c(16~29km)\ubc16\uc5d0 \uc548 \ub418\uc9c0\ub9cc \uc790\ub3d9\ucc28\ub85c \uac08 \uc218\ubc16\uc5d0 \uc5c6\ub294 \uc0ac\ub78c\uc5d0\uac8c\ub294 5\uc2dc\uac04\uc744 \uc6b4\uc804\ud574\uc57c \ud558\ub294 220\ub9c8\uc77c(354km) \uc7a5\uac70\ub9ac\uac00 \ub41c\ub2e4.", "answer": "220\ub9c8\uc77c", "sentence": "\ub178\uc2a4\ub9bc\uacfc \uc0ac\uc6b0\uc2a4\ub9bc \uc0ac\uc774\uc758 \uac70\ub9ac\ub294 \uae4c\ub9c8\uadc0\uc5d0\uac8c\ub294 \ubd88\uacfc 10~18\ub9c8\uc77c(16~29km)\ubc16\uc5d0 \uc548 \ub418\uc9c0\ub9cc \uc790\ub3d9\ucc28\ub85c \uac08 \uc218\ubc16\uc5d0 \uc5c6\ub294 \uc0ac\ub78c\uc5d0\uac8c\ub294 5\uc2dc\uac04\uc744 \uc6b4\uc804\ud574\uc57c \ud558\ub294 220\ub9c8\uc77c (354km) \uc7a5\uac70\ub9ac\uac00 \ub41c\ub2e4.", "paragraph_sentence": "\uacf5\uc6d0\uc740 1904\ud3c9\ubc29\ub9c8\uc77c(4930\ud3c9\ubc29km)\uc758 \ubc29\ub300\ud55c \uba74\uc801\uc744 \ucc28\uc9c0\ud55c\ub2e4. \ucf5c\ub85c\ub77c\ub3c4 \uac15\uc744 \uc0ac\uc774\uc5d0 \ub450\uace0 \ub0a8\uacfc \ubd81\uc758 \ud611\uace1\uc73c\ub85c \ub098\ub258\uc5b4 \uc788\ub2e4. \uacf5\uc6d0\uc9c0\uc5ed \ub0b4\uc5d0\uc11c \ucf5c\ub85c\ub77c\ub3c4 \uac15\uc758 \uae38\uc774\uac00 \ubb34\ub824277\ub9c8\uc77c(443km)\ub85c \ud55c\uad6d\uc758 \ud55c\uac15 \uae38\uc774 (494km)\uc5d0 \uac00\uae5d\ub2e4. \uce90\ub2c8\uc5b8\uc758 \ud3ed\uc740 \uc81c\uc77c \uc881\uc740 \uacf3\uc740 180m\uc5d0\uc11c \uc81c\uc77c \ub113\uc740 \uacf3\uc740 30km(600ft~18\ub9c8\uc77c)\uae4c\uc9c0 \ub418\uba70 \ud611\uace1\uc758 \uae4a\uc774\ub294 \uc57d 1\ub9c8\uc77c(1.6km)\uc774 \ub418\ub294 \ud611\uace1\uc774\ub2e4. \uc0ac\uc6b0\uc2a4\ub9bc(South Rim)\uc740 \ud574\ubc1c 2300m\uc758 \uace0\uc6d0\uc9c0\ub300\uc774\uba70 \uac15\uc6b0\ub7c9\uc774 \uc801\uc740 \ud3b8\uc774\ub098 \ub178\uc2a4\ub9bc(Nouth Rim)\uc740 \ud574\ubc1c 2700m\uac00 \ub418\uc5b4 \ube44\uad50\uc801 \ube44\uc640 \ub208\uc774 \ub9ce\uc774 \ub0b4\ub9b0\ub2e4. \ub530\ub77c\uc11c \ubd81\ucabd \ub178\uc2a4\ub9bc\uc740 \ub0a8\ucabd\uc758 \uc0ac\uc6b0\uc2a4\ub9bc\ubcf4\ub2e4 \ub354 \uc232\uc774 \uc6b8\ucc3d\ud558\uace0 \uae30\uc628\ub3c4 \ub0ae\ub2e4. \ub178\uc2a4\ub9bc\uacfc \uc0ac\uc6b0\uc2a4\ub9bc \uc0ac\uc774\uc758 \uac70\ub9ac\ub294 \uae4c\ub9c8\uadc0\uc5d0\uac8c\ub294 \ubd88\uacfc 10~18\ub9c8\uc77c(16~29km)\ubc16\uc5d0 \uc548 \ub418\uc9c0\ub9cc \uc790\ub3d9\ucc28\ub85c \uac08 \uc218\ubc16\uc5d0 \uc5c6\ub294 \uc0ac\ub78c\uc5d0\uac8c\ub294 5\uc2dc\uac04\uc744 \uc6b4\uc804\ud574\uc57c \ud558\ub294 220\ub9c8\uc77c (354km) \uc7a5\uac70\ub9ac\uac00 \ub41c\ub2e4. ", "paragraph_answer": "\uacf5\uc6d0\uc740 1904\ud3c9\ubc29\ub9c8\uc77c(4930\ud3c9\ubc29km)\uc758 \ubc29\ub300\ud55c \uba74\uc801\uc744 \ucc28\uc9c0\ud55c\ub2e4. \ucf5c\ub85c\ub77c\ub3c4 \uac15\uc744 \uc0ac\uc774\uc5d0 \ub450\uace0 \ub0a8\uacfc \ubd81\uc758 \ud611\uace1\uc73c\ub85c \ub098\ub258\uc5b4 \uc788\ub2e4. \uacf5\uc6d0\uc9c0\uc5ed \ub0b4\uc5d0\uc11c \ucf5c\ub85c\ub77c\ub3c4 \uac15\uc758 \uae38\uc774\uac00 \ubb34\ub824277\ub9c8\uc77c(443km)\ub85c \ud55c\uad6d\uc758 \ud55c\uac15 \uae38\uc774 (494km)\uc5d0 \uac00\uae5d\ub2e4. \uce90\ub2c8\uc5b8\uc758 \ud3ed\uc740 \uc81c\uc77c \uc881\uc740 \uacf3\uc740 180m\uc5d0\uc11c \uc81c\uc77c \ub113\uc740 \uacf3\uc740 30km(600ft~18\ub9c8\uc77c)\uae4c\uc9c0 \ub418\uba70 \ud611\uace1\uc758 \uae4a\uc774\ub294 \uc57d 1\ub9c8\uc77c(1.6km)\uc774 \ub418\ub294 \ud611\uace1\uc774\ub2e4. \uc0ac\uc6b0\uc2a4\ub9bc(South Rim)\uc740 \ud574\ubc1c 2300m\uc758 \uace0\uc6d0\uc9c0\ub300\uc774\uba70 \uac15\uc6b0\ub7c9\uc774 \uc801\uc740 \ud3b8\uc774\ub098 \ub178\uc2a4\ub9bc(Nouth Rim)\uc740 \ud574\ubc1c 2700m\uac00 \ub418\uc5b4 \ube44\uad50\uc801 \ube44\uc640 \ub208\uc774 \ub9ce\uc774 \ub0b4\ub9b0\ub2e4. \ub530\ub77c\uc11c \ubd81\ucabd \ub178\uc2a4\ub9bc\uc740 \ub0a8\ucabd\uc758 \uc0ac\uc6b0\uc2a4\ub9bc\ubcf4\ub2e4 \ub354 \uc232\uc774 \uc6b8\ucc3d\ud558\uace0 \uae30\uc628\ub3c4 \ub0ae\ub2e4. \ub178\uc2a4\ub9bc\uacfc \uc0ac\uc6b0\uc2a4\ub9bc \uc0ac\uc774\uc758 \uac70\ub9ac\ub294 \uae4c\ub9c8\uadc0\uc5d0\uac8c\ub294 \ubd88\uacfc 10~18\ub9c8\uc77c(16~29km)\ubc16\uc5d0 \uc548 \ub418\uc9c0\ub9cc \uc790\ub3d9\ucc28\ub85c \uac08 \uc218\ubc16\uc5d0 \uc5c6\ub294 \uc0ac\ub78c\uc5d0\uac8c\ub294 5\uc2dc\uac04\uc744 \uc6b4\uc804\ud574\uc57c \ud558\ub294 220\ub9c8\uc77c (354km) \uc7a5\uac70\ub9ac\uac00 \ub41c\ub2e4.", "sentence_answer": "\ub178\uc2a4\ub9bc\uacfc \uc0ac\uc6b0\uc2a4\ub9bc \uc0ac\uc774\uc758 \uac70\ub9ac\ub294 \uae4c\ub9c8\uadc0\uc5d0\uac8c\ub294 \ubd88\uacfc 10~18\ub9c8\uc77c(16~29km)\ubc16\uc5d0 \uc548 \ub418\uc9c0\ub9cc \uc790\ub3d9\ucc28\ub85c \uac08 \uc218\ubc16\uc5d0 \uc5c6\ub294 \uc0ac\ub78c\uc5d0\uac8c\ub294 5\uc2dc\uac04\uc744 \uc6b4\uc804\ud574\uc57c \ud558\ub294 220\ub9c8\uc77c (354km) \uc7a5\uac70\ub9ac\uac00 \ub41c\ub2e4.", "paragraph_id": "6511703-1-1", "paragraph_question": "question: \uadf8\ub79c\ub4dc \uce90\ub2c8\uc5b8 \uad6d\ub9bd\uacf5\uc6d0\uc758 \ub178\uc2a4\ub9bc\uacfc \uc0ac\uc6b0\uc2a4\ub9bc\uc758 \uc0ac\uc774 \uac70\ub9ac\ub294 \uba87 \ub9c8\uc77c\uc778\uac00?, context: \uacf5\uc6d0\uc740 1904\ud3c9\ubc29\ub9c8\uc77c(4930\ud3c9\ubc29km)\uc758 \ubc29\ub300\ud55c \uba74\uc801\uc744 \ucc28\uc9c0\ud55c\ub2e4. \ucf5c\ub85c\ub77c\ub3c4 \uac15\uc744 \uc0ac\uc774\uc5d0 \ub450\uace0 \ub0a8\uacfc \ubd81\uc758 \ud611\uace1\uc73c\ub85c \ub098\ub258\uc5b4 \uc788\ub2e4. \uacf5\uc6d0\uc9c0\uc5ed \ub0b4\uc5d0\uc11c \ucf5c\ub85c\ub77c\ub3c4 \uac15\uc758 \uae38\uc774\uac00 \ubb34\ub824277\ub9c8\uc77c(443km)\ub85c \ud55c\uad6d\uc758 \ud55c\uac15 \uae38\uc774 (494km)\uc5d0 \uac00\uae5d\ub2e4. \uce90\ub2c8\uc5b8\uc758 \ud3ed\uc740 \uc81c\uc77c \uc881\uc740 \uacf3\uc740 180m\uc5d0\uc11c \uc81c\uc77c \ub113\uc740 \uacf3\uc740 30km(600ft~18\ub9c8\uc77c)\uae4c\uc9c0 \ub418\uba70 \ud611\uace1\uc758 \uae4a\uc774\ub294 \uc57d 1\ub9c8\uc77c(1.6km)\uc774 \ub418\ub294 \ud611\uace1\uc774\ub2e4. \uc0ac\uc6b0\uc2a4\ub9bc(South Rim)\uc740 \ud574\ubc1c 2300m\uc758 \uace0\uc6d0\uc9c0\ub300\uc774\uba70 \uac15\uc6b0\ub7c9\uc774 \uc801\uc740 \ud3b8\uc774\ub098 \ub178\uc2a4\ub9bc(Nouth Rim)\uc740 \ud574\ubc1c 2700m\uac00 \ub418\uc5b4 \ube44\uad50\uc801 \ube44\uc640 \ub208\uc774 \ub9ce\uc774 \ub0b4\ub9b0\ub2e4. \ub530\ub77c\uc11c \ubd81\ucabd \ub178\uc2a4\ub9bc\uc740 \ub0a8\ucabd\uc758 \uc0ac\uc6b0\uc2a4\ub9bc\ubcf4\ub2e4 \ub354 \uc232\uc774 \uc6b8\ucc3d\ud558\uace0 \uae30\uc628\ub3c4 \ub0ae\ub2e4. \ub178\uc2a4\ub9bc\uacfc \uc0ac\uc6b0\uc2a4\ub9bc \uc0ac\uc774\uc758 \uac70\ub9ac\ub294 \uae4c\ub9c8\uadc0\uc5d0\uac8c\ub294 \ubd88\uacfc 10~18\ub9c8\uc77c(16~29km)\ubc16\uc5d0 \uc548 \ub418\uc9c0\ub9cc \uc790\ub3d9\ucc28\ub85c \uac08 \uc218\ubc16\uc5d0 \uc5c6\ub294 \uc0ac\ub78c\uc5d0\uac8c\ub294 5\uc2dc\uac04\uc744 \uc6b4\uc804\ud574\uc57c \ud558\ub294 220\ub9c8\uc77c(354km) \uc7a5\uac70\ub9ac\uac00 \ub41c\ub2e4."} {"question": "\uc774\uc0c1\ub2ec\uc740 \uc9d5\uc5ed \uba87\ub144\ud615\uc744 \uc120\uace0\ubc1b\uc558\ub294\uac00?", "paragraph": "\uae30\ud765\ucee8\ud2b8\ub9ac\ud074\ub7fd\uc740 \uc804\ub450\ud658 \uc815\ubd80 \uc2dc\uc808 \ud1f4\uc9c1 \uacbd\ucc30\uad00\uc758 \ubaa8\uc784\uc778 \uacbd\uc6b0\ud68c\uac00 \uc0ac\uc5c5\uad8c\uc744 \ubc1b\uc544 \uc815\uac15\uc911\uae30\uc758 \uc774\uc0c1\ub2ec\uc758 \uc790\ubcf8\uc744 \uac00\uc9c0\uace0 \ucd94\uc9c4\ud55c \uace8\ud504\uc7a5 \uc0ac\uc5c5\uc778\ub370, \uc774 \uc0ac\uc5c5\uc758 \uc9c0\ubd84\uc758 \uacfc\ubc18\uc218 \uc774\uc0c1\uc774 \uc774\uc0c1\ub2ec\uacfc \uadf8 \uc9c0\uc778\uc5d0\uac8c \ub118\uc5b4\uac04 \uac83\uc774 \ub4dc\ub7ec\ub098\uc790 1993\ub144 5\uc6d4 \uacbd\ucc30\uc740 \uc774\uc0c1\ub2ec\uc774 \uc804\uc9c1 \uce58\uc548\ubcf8\ubd80\uc7a5 \ub4f1 \uacbd\uc6b0\ud68c \uac04\ubd80\uc5d0 \ub1cc\ubb3c\uc744 \uacf5\uc5ec\ud588\ub294\uc9c0\ub97c \uc870\uc0ac\ud558\uae30 \uc2dc\uc791\ud588\ub2e4. \uacb0\uad6d 6\uc6d4\uc5d0\ub294 \uc804 \uce58\uc548\uac10\uc774\uc790 \uacbd\uc6b0\ud68c \uc774\uc0ac\uc778 \uc625\uae30\uc9c4\uc774 \uc774\uc0c1\ub2ec\uc5d0\uac8c\uc11c \ub2e4\ubc29\uc6b4\uc601\uad8c\uc744 \ub118\uaca8\ubc1b\uc544 \uc774\uc775\uc744 \ucc59\uacbc\ub2e4\uba70 \ubc30\uc784\uc218\uc7ac \ud610\uc758\ub85c \uc774\uc0c1\ub2ec\uacfc \ud568\uaed8 \uad6c\uc18d\ub418\uc5c8\uace0, 7\uc6d4\uc5d0\ub294 \uc774\uc778\uc12d \uc804 \uacbd\ucc30\uccad\uc7a5\uc774 \uc774\uc0c1\ub2ec\uc5d0\uac8c 1\ucc9c\ub9cc \uc6d0\uc744 \ubc1b\uc558\ub2e4\uba70 \ub1cc\ubb3c \uc218\uc218 \ud610\uc758\ub85c \uad6c\uc18d\ub418\uc5c8\ub2e4. \uacb0\uad6d \uc11c\uc6b8\ud615\uc0ac\uc9c0\ubc95 \ud569\uc75821\ubd80\ub294 \uc774\uc0c1\ub2ec\uc5d0\uac8c \ubc30\uc784 \ubc0f \ub1cc\ubb3c\uacf5\uc5ec\uc8c4\ub85c \uc9d5\uc5ed 3\ub144\ud615\uc744 \uc120\uace0\ud588\ub2e4. \ud558\uc9c0\ub9cc \uc804\uc9c1 \uce58\uc548\ucd1d\uc218\uac00 \uad6c\uc18d\ub418\ub294 \ub4f1 \ud070 \ud30c\ubb38\uc744 \uc77c\uc73c\ucf30\ub358 \uc774 \uc0ac\uac74\uc758 \uc8fc\ubc94\uc778 \uc774\uc0c1\ub2ec\uc774 \uc6b0\ubcd1\uc6b0\ub85c \uc778\ud574 \ud070 \ud654\ub294 \uba74\ud588\ub2e4\ub294 \ub9d0\ub3c4 \uc788\ub2e4.", "answer": "3\ub144\ud615", "sentence": "\uacb0\uad6d \uc11c\uc6b8\ud615\uc0ac\uc9c0\ubc95 \ud569\uc75821\ubd80\ub294 \uc774\uc0c1\ub2ec\uc5d0\uac8c \ubc30\uc784 \ubc0f \ub1cc\ubb3c\uacf5\uc5ec\uc8c4\ub85c \uc9d5\uc5ed 3\ub144\ud615 \uc744 \uc120\uace0\ud588\ub2e4.", "paragraph_sentence": "\uae30\ud765\ucee8\ud2b8\ub9ac\ud074\ub7fd\uc740 \uc804\ub450\ud658 \uc815\ubd80 \uc2dc\uc808 \ud1f4\uc9c1 \uacbd\ucc30\uad00\uc758 \ubaa8\uc784\uc778 \uacbd\uc6b0\ud68c\uac00 \uc0ac\uc5c5\uad8c\uc744 \ubc1b\uc544 \uc815\uac15\uc911\uae30\uc758 \uc774\uc0c1\ub2ec\uc758 \uc790\ubcf8\uc744 \uac00\uc9c0\uace0 \ucd94\uc9c4\ud55c \uace8\ud504\uc7a5 \uc0ac\uc5c5\uc778\ub370, \uc774 \uc0ac\uc5c5\uc758 \uc9c0\ubd84\uc758 \uacfc\ubc18\uc218 \uc774\uc0c1\uc774 \uc774\uc0c1\ub2ec\uacfc \uadf8 \uc9c0\uc778\uc5d0\uac8c \ub118\uc5b4\uac04 \uac83\uc774 \ub4dc\ub7ec\ub098\uc790 1993\ub144 5\uc6d4 \uacbd\ucc30\uc740 \uc774\uc0c1\ub2ec\uc774 \uc804\uc9c1 \uce58\uc548\ubcf8\ubd80\uc7a5 \ub4f1 \uacbd\uc6b0\ud68c \uac04\ubd80\uc5d0 \ub1cc\ubb3c\uc744 \uacf5\uc5ec\ud588\ub294\uc9c0\ub97c \uc870\uc0ac\ud558\uae30 \uc2dc\uc791\ud588\ub2e4. \uacb0\uad6d 6\uc6d4\uc5d0\ub294 \uc804 \uce58\uc548\uac10\uc774\uc790 \uacbd\uc6b0\ud68c \uc774\uc0ac\uc778 \uc625\uae30\uc9c4\uc774 \uc774\uc0c1\ub2ec\uc5d0\uac8c\uc11c \ub2e4\ubc29\uc6b4\uc601\uad8c\uc744 \ub118\uaca8\ubc1b\uc544 \uc774\uc775\uc744 \ucc59\uacbc\ub2e4\uba70 \ubc30\uc784\uc218\uc7ac \ud610\uc758\ub85c \uc774\uc0c1\ub2ec\uacfc \ud568\uaed8 \uad6c\uc18d\ub418\uc5c8\uace0, 7\uc6d4\uc5d0\ub294 \uc774\uc778\uc12d \uc804 \uacbd\ucc30\uccad\uc7a5\uc774 \uc774\uc0c1\ub2ec\uc5d0\uac8c 1\ucc9c\ub9cc \uc6d0\uc744 \ubc1b\uc558\ub2e4\uba70 \ub1cc\ubb3c \uc218\uc218 \ud610\uc758\ub85c \uad6c\uc18d\ub418\uc5c8\ub2e4. \uacb0\uad6d \uc11c\uc6b8\ud615\uc0ac\uc9c0\ubc95 \ud569\uc75821\ubd80\ub294 \uc774\uc0c1\ub2ec\uc5d0\uac8c \ubc30\uc784 \ubc0f \ub1cc\ubb3c\uacf5\uc5ec\uc8c4\ub85c \uc9d5\uc5ed 3\ub144\ud615 \uc744 \uc120\uace0\ud588\ub2e4. \ud558\uc9c0\ub9cc \uc804\uc9c1 \uce58\uc548\ucd1d\uc218\uac00 \uad6c\uc18d\ub418\ub294 \ub4f1 \ud070 \ud30c\ubb38\uc744 \uc77c\uc73c\ucf30\ub358 \uc774 \uc0ac\uac74\uc758 \uc8fc\ubc94\uc778 \uc774\uc0c1\ub2ec\uc774 \uc6b0\ubcd1\uc6b0\ub85c \uc778\ud574 \ud070 \ud654\ub294 \uba74\ud588\ub2e4\ub294 \ub9d0\ub3c4 \uc788\ub2e4.", "paragraph_answer": "\uae30\ud765\ucee8\ud2b8\ub9ac\ud074\ub7fd\uc740 \uc804\ub450\ud658 \uc815\ubd80 \uc2dc\uc808 \ud1f4\uc9c1 \uacbd\ucc30\uad00\uc758 \ubaa8\uc784\uc778 \uacbd\uc6b0\ud68c\uac00 \uc0ac\uc5c5\uad8c\uc744 \ubc1b\uc544 \uc815\uac15\uc911\uae30\uc758 \uc774\uc0c1\ub2ec\uc758 \uc790\ubcf8\uc744 \uac00\uc9c0\uace0 \ucd94\uc9c4\ud55c \uace8\ud504\uc7a5 \uc0ac\uc5c5\uc778\ub370, \uc774 \uc0ac\uc5c5\uc758 \uc9c0\ubd84\uc758 \uacfc\ubc18\uc218 \uc774\uc0c1\uc774 \uc774\uc0c1\ub2ec\uacfc \uadf8 \uc9c0\uc778\uc5d0\uac8c \ub118\uc5b4\uac04 \uac83\uc774 \ub4dc\ub7ec\ub098\uc790 1993\ub144 5\uc6d4 \uacbd\ucc30\uc740 \uc774\uc0c1\ub2ec\uc774 \uc804\uc9c1 \uce58\uc548\ubcf8\ubd80\uc7a5 \ub4f1 \uacbd\uc6b0\ud68c \uac04\ubd80\uc5d0 \ub1cc\ubb3c\uc744 \uacf5\uc5ec\ud588\ub294\uc9c0\ub97c \uc870\uc0ac\ud558\uae30 \uc2dc\uc791\ud588\ub2e4. \uacb0\uad6d 6\uc6d4\uc5d0\ub294 \uc804 \uce58\uc548\uac10\uc774\uc790 \uacbd\uc6b0\ud68c \uc774\uc0ac\uc778 \uc625\uae30\uc9c4\uc774 \uc774\uc0c1\ub2ec\uc5d0\uac8c\uc11c \ub2e4\ubc29\uc6b4\uc601\uad8c\uc744 \ub118\uaca8\ubc1b\uc544 \uc774\uc775\uc744 \ucc59\uacbc\ub2e4\uba70 \ubc30\uc784\uc218\uc7ac \ud610\uc758\ub85c \uc774\uc0c1\ub2ec\uacfc \ud568\uaed8 \uad6c\uc18d\ub418\uc5c8\uace0, 7\uc6d4\uc5d0\ub294 \uc774\uc778\uc12d \uc804 \uacbd\ucc30\uccad\uc7a5\uc774 \uc774\uc0c1\ub2ec\uc5d0\uac8c 1\ucc9c\ub9cc \uc6d0\uc744 \ubc1b\uc558\ub2e4\uba70 \ub1cc\ubb3c \uc218\uc218 \ud610\uc758\ub85c \uad6c\uc18d\ub418\uc5c8\ub2e4. \uacb0\uad6d \uc11c\uc6b8\ud615\uc0ac\uc9c0\ubc95 \ud569\uc75821\ubd80\ub294 \uc774\uc0c1\ub2ec\uc5d0\uac8c \ubc30\uc784 \ubc0f \ub1cc\ubb3c\uacf5\uc5ec\uc8c4\ub85c \uc9d5\uc5ed 3\ub144\ud615 \uc744 \uc120\uace0\ud588\ub2e4. \ud558\uc9c0\ub9cc \uc804\uc9c1 \uce58\uc548\ucd1d\uc218\uac00 \uad6c\uc18d\ub418\ub294 \ub4f1 \ud070 \ud30c\ubb38\uc744 \uc77c\uc73c\ucf30\ub358 \uc774 \uc0ac\uac74\uc758 \uc8fc\ubc94\uc778 \uc774\uc0c1\ub2ec\uc774 \uc6b0\ubcd1\uc6b0\ub85c \uc778\ud574 \ud070 \ud654\ub294 \uba74\ud588\ub2e4\ub294 \ub9d0\ub3c4 \uc788\ub2e4.", "sentence_answer": "\uacb0\uad6d \uc11c\uc6b8\ud615\uc0ac\uc9c0\ubc95 \ud569\uc75821\ubd80\ub294 \uc774\uc0c1\ub2ec\uc5d0\uac8c \ubc30\uc784 \ubc0f \ub1cc\ubb3c\uacf5\uc5ec\uc8c4\ub85c \uc9d5\uc5ed 3\ub144\ud615 \uc744 \uc120\uace0\ud588\ub2e4.", "paragraph_id": "6526268-2-1", "paragraph_question": "question: \uc774\uc0c1\ub2ec\uc740 \uc9d5\uc5ed \uba87\ub144\ud615\uc744 \uc120\uace0\ubc1b\uc558\ub294\uac00?, context: \uae30\ud765\ucee8\ud2b8\ub9ac\ud074\ub7fd\uc740 \uc804\ub450\ud658 \uc815\ubd80 \uc2dc\uc808 \ud1f4\uc9c1 \uacbd\ucc30\uad00\uc758 \ubaa8\uc784\uc778 \uacbd\uc6b0\ud68c\uac00 \uc0ac\uc5c5\uad8c\uc744 \ubc1b\uc544 \uc815\uac15\uc911\uae30\uc758 \uc774\uc0c1\ub2ec\uc758 \uc790\ubcf8\uc744 \uac00\uc9c0\uace0 \ucd94\uc9c4\ud55c \uace8\ud504\uc7a5 \uc0ac\uc5c5\uc778\ub370, \uc774 \uc0ac\uc5c5\uc758 \uc9c0\ubd84\uc758 \uacfc\ubc18\uc218 \uc774\uc0c1\uc774 \uc774\uc0c1\ub2ec\uacfc \uadf8 \uc9c0\uc778\uc5d0\uac8c \ub118\uc5b4\uac04 \uac83\uc774 \ub4dc\ub7ec\ub098\uc790 1993\ub144 5\uc6d4 \uacbd\ucc30\uc740 \uc774\uc0c1\ub2ec\uc774 \uc804\uc9c1 \uce58\uc548\ubcf8\ubd80\uc7a5 \ub4f1 \uacbd\uc6b0\ud68c \uac04\ubd80\uc5d0 \ub1cc\ubb3c\uc744 \uacf5\uc5ec\ud588\ub294\uc9c0\ub97c \uc870\uc0ac\ud558\uae30 \uc2dc\uc791\ud588\ub2e4. \uacb0\uad6d 6\uc6d4\uc5d0\ub294 \uc804 \uce58\uc548\uac10\uc774\uc790 \uacbd\uc6b0\ud68c \uc774\uc0ac\uc778 \uc625\uae30\uc9c4\uc774 \uc774\uc0c1\ub2ec\uc5d0\uac8c\uc11c \ub2e4\ubc29\uc6b4\uc601\uad8c\uc744 \ub118\uaca8\ubc1b\uc544 \uc774\uc775\uc744 \ucc59\uacbc\ub2e4\uba70 \ubc30\uc784\uc218\uc7ac \ud610\uc758\ub85c \uc774\uc0c1\ub2ec\uacfc \ud568\uaed8 \uad6c\uc18d\ub418\uc5c8\uace0, 7\uc6d4\uc5d0\ub294 \uc774\uc778\uc12d \uc804 \uacbd\ucc30\uccad\uc7a5\uc774 \uc774\uc0c1\ub2ec\uc5d0\uac8c 1\ucc9c\ub9cc \uc6d0\uc744 \ubc1b\uc558\ub2e4\uba70 \ub1cc\ubb3c \uc218\uc218 \ud610\uc758\ub85c \uad6c\uc18d\ub418\uc5c8\ub2e4. \uacb0\uad6d \uc11c\uc6b8\ud615\uc0ac\uc9c0\ubc95 \ud569\uc75821\ubd80\ub294 \uc774\uc0c1\ub2ec\uc5d0\uac8c \ubc30\uc784 \ubc0f \ub1cc\ubb3c\uacf5\uc5ec\uc8c4\ub85c \uc9d5\uc5ed 3\ub144\ud615\uc744 \uc120\uace0\ud588\ub2e4. \ud558\uc9c0\ub9cc \uc804\uc9c1 \uce58\uc548\ucd1d\uc218\uac00 \uad6c\uc18d\ub418\ub294 \ub4f1 \ud070 \ud30c\ubb38\uc744 \uc77c\uc73c\ucf30\ub358 \uc774 \uc0ac\uac74\uc758 \uc8fc\ubc94\uc778 \uc774\uc0c1\ub2ec\uc774 \uc6b0\ubcd1\uc6b0\ub85c \uc778\ud574 \ud070 \ud654\ub294 \uba74\ud588\ub2e4\ub294 \ub9d0\ub3c4 \uc788\ub2e4."} {"question": "\uc138\uacc4\uc5d0\uc11c \uac00\uc7a5 \uac15\ud55c \uc694\uc0c8 \uc911 \ud558\ub098\uc778 \uc18c\ub828\uc758 \ud574\uad70\uae30\uc9c0\ub294?", "paragraph": "\uc18c\ub828\uad70\uc758 \ud574\uad70\uae30\uc9c0\uc778 \uc138\ubc14\uc2a4\ud1a0\ud3f4\uc740 \uc138\uacc4\uc5d0\uc11c \uac15\ub825\ud55c \uc694\uc0c8 \uc911 \ud558\ub098\uc600\ub2e4. \ud06c\ub9bc\ubc18\ub3c4 \ub0a8\uc11c\ubd80 \ub05d\uc758 \uc11d\ud68c\uc554\uc9c8 \uacf6\uc5d0\uc11c \uae4a\uc774 \uce68\uc2dd\ud55c \uacf3\uc5d0 \uc788\ub294 \uc138\ubc14\uc2a4\ud1a0\ud3f4\uc740 \uc9c0\uc0c1\uc73c\ub85c \uc811\uadfc\ud558\uae30 \ub418\uac8c \ud798\ub4e4\uc5c8\ub2e4. \uc138\ubca0\ub974\ub0d0 \ub9cc\uc774 \ub0b4\ub824\ub2e4 \ubcf4\uc774\ub294 \uace0\ub3c4\uac00 \ub192\uc740 \uc808\ubcbd \uc9c0\ub300\uac00 \uc815\ubc15\uc9c0\ub97c \ubcf4\ud638\ud558\uc5ec \uc0c1\ub959 \uc704\ud611\uc744 \ub9c9\uc558\ub2e4. \uc18c\ub828 \ud574\uad70\uc740 \ud56d\uad6c\ub97c \ud604\ub300\ud654\ud558\uace0 \ucc9c\ud61c\uc758 \ubc29\uc5b4\uc5d0 \ub354\ud558\uc5ec \ub0b4\ub959\uacfc \ubc14\ub2e4\ub85c \ubc1c\uc0ac\ud560 \uc218 \uc788\ub294 \uacac\uace0\ud55c \ud574\uc548 \ubc29\uc5b4 \uc2dc\uc124\uc744 \uac74\ub9bd\ud588\ub2e4. \ud3ec\ubcd1\uc740 \uac15\ud55c \ucf58\ud06c\ub9ac\ud2b8\uc640 \uc7a5\uac11\ucc28 \ud3ec\ud0d1\uc73c\ub85c \ubcf4\ud638\ubc1b\uc558\ub2e4. \ud56d\uad6c\ub294 \uc8fc\uc694\ud55c \ubaa9\ud45c\ubb3c\uc774\uc5c8\ub2e4. \uc138\ubc14\uc2a4\ud1a0\ud3f4 \ud56d\uad6c\ub294 \ucd94\ucd95\uad6d\uc774 \ub354 \uba3c \uc6d0\ud574\ub85c \uc9c4\ucd9c\ud560 \uc218 \uc788\uac8c \ud574 \uc918 \ucf54\uce74\uc11c\uc2a4 \ud56d\uad6c\uc640 \uc0b0\ub9e5 \uc9c0\uc5ed\uc744 \ud574\uc0c1\uc774\ub098 \uacf5\uc911\uc5d0\uc11c \uacf5\uaca9\uc774 \uac00\ub2a5\ud558\uac8c \ud558\ub294 \uc7a0\uc7ac\ub41c, \uc911\uc694\ud55c \ud574\uad70\uae30\uc9c0\u00b7\uacf5\uad70\uae30\uc9c0 \uc5ed\ud560\uc744 \ud588\ub2e4. \uc18c\ub828 \uacf5\uad70\uc740 1941\ub144 6\uc6d4 \ucd94\ucd95\uad6d\uc758 \uc18c\ub828 \uce68\uacf5 \uc774\ud6c4 \ud06c\ub9bc\ubc18\ub3c4\uc5d0\uc11c \ub8e8\ub9c8\ub2c8\uc544\ub85c \ud3ed\uaca9\ud558\uba74\uc11c \uacf5\uad70\uae30\uc9c0\ub85c\uc11c\uc758 \uac00\uce58\uac00 \uc99d\uba85\ub418\uc5c8\ub2e4.", "answer": "\uc138\ubc14\uc2a4\ud1a0\ud3f4", "sentence": "\uc18c\ub828\uad70\uc758 \ud574\uad70\uae30\uc9c0\uc778 \uc138\ubc14\uc2a4\ud1a0\ud3f4 \uc740 \uc138\uacc4\uc5d0\uc11c \uac15\ub825\ud55c \uc694\uc0c8 \uc911 \ud558\ub098\uc600\ub2e4.", "paragraph_sentence": " \uc18c\ub828\uad70\uc758 \ud574\uad70\uae30\uc9c0\uc778 \uc138\ubc14\uc2a4\ud1a0\ud3f4 \uc740 \uc138\uacc4\uc5d0\uc11c \uac15\ub825\ud55c \uc694\uc0c8 \uc911 \ud558\ub098\uc600\ub2e4. \ud06c\ub9bc\ubc18\ub3c4 \ub0a8\uc11c\ubd80 \ub05d\uc758 \uc11d\ud68c\uc554\uc9c8 \uacf6\uc5d0\uc11c \uae4a\uc774 \uce68\uc2dd\ud55c \uacf3\uc5d0 \uc788\ub294 \uc138\ubc14\uc2a4\ud1a0\ud3f4\uc740 \uc9c0\uc0c1\uc73c\ub85c \uc811\uadfc\ud558\uae30 \ub418\uac8c \ud798\ub4e4\uc5c8\ub2e4. \uc138\ubca0\ub974\ub0d0 \ub9cc\uc774 \ub0b4\ub824\ub2e4 \ubcf4\uc774\ub294 \uace0\ub3c4\uac00 \ub192\uc740 \uc808\ubcbd \uc9c0\ub300\uac00 \uc815\ubc15\uc9c0\ub97c \ubcf4\ud638\ud558\uc5ec \uc0c1\ub959 \uc704\ud611\uc744 \ub9c9\uc558\ub2e4. \uc18c\ub828 \ud574\uad70\uc740 \ud56d\uad6c\ub97c \ud604\ub300\ud654\ud558\uace0 \ucc9c\ud61c\uc758 \ubc29\uc5b4\uc5d0 \ub354\ud558\uc5ec \ub0b4\ub959\uacfc \ubc14\ub2e4\ub85c \ubc1c\uc0ac\ud560 \uc218 \uc788\ub294 \uacac\uace0\ud55c \ud574\uc548 \ubc29\uc5b4 \uc2dc\uc124\uc744 \uac74\ub9bd\ud588\ub2e4. \ud3ec\ubcd1\uc740 \uac15\ud55c \ucf58\ud06c\ub9ac\ud2b8\uc640 \uc7a5\uac11\ucc28 \ud3ec\ud0d1\uc73c\ub85c \ubcf4\ud638\ubc1b\uc558\ub2e4. \ud56d\uad6c\ub294 \uc8fc\uc694\ud55c \ubaa9\ud45c\ubb3c\uc774\uc5c8\ub2e4. \uc138\ubc14\uc2a4\ud1a0\ud3f4 \ud56d\uad6c\ub294 \ucd94\ucd95\uad6d\uc774 \ub354 \uba3c \uc6d0\ud574\ub85c \uc9c4\ucd9c\ud560 \uc218 \uc788\uac8c \ud574 \uc918 \ucf54\uce74\uc11c\uc2a4 \ud56d\uad6c\uc640 \uc0b0\ub9e5 \uc9c0\uc5ed\uc744 \ud574\uc0c1\uc774\ub098 \uacf5\uc911\uc5d0\uc11c \uacf5\uaca9\uc774 \uac00\ub2a5\ud558\uac8c \ud558\ub294 \uc7a0\uc7ac\ub41c, \uc911\uc694\ud55c \ud574\uad70\uae30\uc9c0\u00b7\uacf5\uad70\uae30\uc9c0 \uc5ed\ud560\uc744 \ud588\ub2e4. \uc18c\ub828 \uacf5\uad70\uc740 1941\ub144 6\uc6d4 \ucd94\ucd95\uad6d\uc758 \uc18c\ub828 \uce68\uacf5 \uc774\ud6c4 \ud06c\ub9bc\ubc18\ub3c4\uc5d0\uc11c \ub8e8\ub9c8\ub2c8\uc544\ub85c \ud3ed\uaca9\ud558\uba74\uc11c \uacf5\uad70\uae30\uc9c0\ub85c\uc11c\uc758 \uac00\uce58\uac00 \uc99d\uba85\ub418\uc5c8\ub2e4.", "paragraph_answer": "\uc18c\ub828\uad70\uc758 \ud574\uad70\uae30\uc9c0\uc778 \uc138\ubc14\uc2a4\ud1a0\ud3f4 \uc740 \uc138\uacc4\uc5d0\uc11c \uac15\ub825\ud55c \uc694\uc0c8 \uc911 \ud558\ub098\uc600\ub2e4. \ud06c\ub9bc\ubc18\ub3c4 \ub0a8\uc11c\ubd80 \ub05d\uc758 \uc11d\ud68c\uc554\uc9c8 \uacf6\uc5d0\uc11c \uae4a\uc774 \uce68\uc2dd\ud55c \uacf3\uc5d0 \uc788\ub294 \uc138\ubc14\uc2a4\ud1a0\ud3f4\uc740 \uc9c0\uc0c1\uc73c\ub85c \uc811\uadfc\ud558\uae30 \ub418\uac8c \ud798\ub4e4\uc5c8\ub2e4. \uc138\ubca0\ub974\ub0d0 \ub9cc\uc774 \ub0b4\ub824\ub2e4 \ubcf4\uc774\ub294 \uace0\ub3c4\uac00 \ub192\uc740 \uc808\ubcbd \uc9c0\ub300\uac00 \uc815\ubc15\uc9c0\ub97c \ubcf4\ud638\ud558\uc5ec \uc0c1\ub959 \uc704\ud611\uc744 \ub9c9\uc558\ub2e4. \uc18c\ub828 \ud574\uad70\uc740 \ud56d\uad6c\ub97c \ud604\ub300\ud654\ud558\uace0 \ucc9c\ud61c\uc758 \ubc29\uc5b4\uc5d0 \ub354\ud558\uc5ec \ub0b4\ub959\uacfc \ubc14\ub2e4\ub85c \ubc1c\uc0ac\ud560 \uc218 \uc788\ub294 \uacac\uace0\ud55c \ud574\uc548 \ubc29\uc5b4 \uc2dc\uc124\uc744 \uac74\ub9bd\ud588\ub2e4. \ud3ec\ubcd1\uc740 \uac15\ud55c \ucf58\ud06c\ub9ac\ud2b8\uc640 \uc7a5\uac11\ucc28 \ud3ec\ud0d1\uc73c\ub85c \ubcf4\ud638\ubc1b\uc558\ub2e4. \ud56d\uad6c\ub294 \uc8fc\uc694\ud55c \ubaa9\ud45c\ubb3c\uc774\uc5c8\ub2e4. \uc138\ubc14\uc2a4\ud1a0\ud3f4 \ud56d\uad6c\ub294 \ucd94\ucd95\uad6d\uc774 \ub354 \uba3c \uc6d0\ud574\ub85c \uc9c4\ucd9c\ud560 \uc218 \uc788\uac8c \ud574 \uc918 \ucf54\uce74\uc11c\uc2a4 \ud56d\uad6c\uc640 \uc0b0\ub9e5 \uc9c0\uc5ed\uc744 \ud574\uc0c1\uc774\ub098 \uacf5\uc911\uc5d0\uc11c \uacf5\uaca9\uc774 \uac00\ub2a5\ud558\uac8c \ud558\ub294 \uc7a0\uc7ac\ub41c, \uc911\uc694\ud55c \ud574\uad70\uae30\uc9c0\u00b7\uacf5\uad70\uae30\uc9c0 \uc5ed\ud560\uc744 \ud588\ub2e4. \uc18c\ub828 \uacf5\uad70\uc740 1941\ub144 6\uc6d4 \ucd94\ucd95\uad6d\uc758 \uc18c\ub828 \uce68\uacf5 \uc774\ud6c4 \ud06c\ub9bc\ubc18\ub3c4\uc5d0\uc11c \ub8e8\ub9c8\ub2c8\uc544\ub85c \ud3ed\uaca9\ud558\uba74\uc11c \uacf5\uad70\uae30\uc9c0\ub85c\uc11c\uc758 \uac00\uce58\uac00 \uc99d\uba85\ub418\uc5c8\ub2e4.", "sentence_answer": "\uc18c\ub828\uad70\uc758 \ud574\uad70\uae30\uc9c0\uc778 \uc138\ubc14\uc2a4\ud1a0\ud3f4 \uc740 \uc138\uacc4\uc5d0\uc11c \uac15\ub825\ud55c \uc694\uc0c8 \uc911 \ud558\ub098\uc600\ub2e4.", "paragraph_id": "6532114-1-0", "paragraph_question": "question: \uc138\uacc4\uc5d0\uc11c \uac00\uc7a5 \uac15\ud55c \uc694\uc0c8 \uc911 \ud558\ub098\uc778 \uc18c\ub828\uc758 \ud574\uad70\uae30\uc9c0\ub294?, context: \uc18c\ub828\uad70\uc758 \ud574\uad70\uae30\uc9c0\uc778 \uc138\ubc14\uc2a4\ud1a0\ud3f4\uc740 \uc138\uacc4\uc5d0\uc11c \uac15\ub825\ud55c \uc694\uc0c8 \uc911 \ud558\ub098\uc600\ub2e4. \ud06c\ub9bc\ubc18\ub3c4 \ub0a8\uc11c\ubd80 \ub05d\uc758 \uc11d\ud68c\uc554\uc9c8 \uacf6\uc5d0\uc11c \uae4a\uc774 \uce68\uc2dd\ud55c \uacf3\uc5d0 \uc788\ub294 \uc138\ubc14\uc2a4\ud1a0\ud3f4\uc740 \uc9c0\uc0c1\uc73c\ub85c \uc811\uadfc\ud558\uae30 \ub418\uac8c \ud798\ub4e4\uc5c8\ub2e4. \uc138\ubca0\ub974\ub0d0 \ub9cc\uc774 \ub0b4\ub824\ub2e4 \ubcf4\uc774\ub294 \uace0\ub3c4\uac00 \ub192\uc740 \uc808\ubcbd \uc9c0\ub300\uac00 \uc815\ubc15\uc9c0\ub97c \ubcf4\ud638\ud558\uc5ec \uc0c1\ub959 \uc704\ud611\uc744 \ub9c9\uc558\ub2e4. \uc18c\ub828 \ud574\uad70\uc740 \ud56d\uad6c\ub97c \ud604\ub300\ud654\ud558\uace0 \ucc9c\ud61c\uc758 \ubc29\uc5b4\uc5d0 \ub354\ud558\uc5ec \ub0b4\ub959\uacfc \ubc14\ub2e4\ub85c \ubc1c\uc0ac\ud560 \uc218 \uc788\ub294 \uacac\uace0\ud55c \ud574\uc548 \ubc29\uc5b4 \uc2dc\uc124\uc744 \uac74\ub9bd\ud588\ub2e4. \ud3ec\ubcd1\uc740 \uac15\ud55c \ucf58\ud06c\ub9ac\ud2b8\uc640 \uc7a5\uac11\ucc28 \ud3ec\ud0d1\uc73c\ub85c \ubcf4\ud638\ubc1b\uc558\ub2e4. \ud56d\uad6c\ub294 \uc8fc\uc694\ud55c \ubaa9\ud45c\ubb3c\uc774\uc5c8\ub2e4. \uc138\ubc14\uc2a4\ud1a0\ud3f4 \ud56d\uad6c\ub294 \ucd94\ucd95\uad6d\uc774 \ub354 \uba3c \uc6d0\ud574\ub85c \uc9c4\ucd9c\ud560 \uc218 \uc788\uac8c \ud574 \uc918 \ucf54\uce74\uc11c\uc2a4 \ud56d\uad6c\uc640 \uc0b0\ub9e5 \uc9c0\uc5ed\uc744 \ud574\uc0c1\uc774\ub098 \uacf5\uc911\uc5d0\uc11c \uacf5\uaca9\uc774 \uac00\ub2a5\ud558\uac8c \ud558\ub294 \uc7a0\uc7ac\ub41c, \uc911\uc694\ud55c \ud574\uad70\uae30\uc9c0\u00b7\uacf5\uad70\uae30\uc9c0 \uc5ed\ud560\uc744 \ud588\ub2e4. \uc18c\ub828 \uacf5\uad70\uc740 1941\ub144 6\uc6d4 \ucd94\ucd95\uad6d\uc758 \uc18c\ub828 \uce68\uacf5 \uc774\ud6c4 \ud06c\ub9bc\ubc18\ub3c4\uc5d0\uc11c \ub8e8\ub9c8\ub2c8\uc544\ub85c \ud3ed\uaca9\ud558\uba74\uc11c \uacf5\uad70\uae30\uc9c0\ub85c\uc11c\uc758 \uac00\uce58\uac00 \uc99d\uba85\ub418\uc5c8\ub2e4."} {"question": "\ub300\ud55c\ubbfc\uad6d\uc5d0\uc11c \uc544\uc774\ud3f0\uc774 \ucd9c\uc2dc\ub41c \uc774\ud6c4 \uc0bc\uc131\uacfc \ub9c8\ucc30\uc774 \uc0dd\uae34 \ud1b5\uc2e0\uc5c5\uccb4\ub294?", "paragraph": "\uc544\uc774\ud3f0\uc744 \ub3c4\uc785\ud55c \uc774\ud6c4\ub85c KT\uc640 \uc0bc\uc131\uc804\uc790\uac04\uc758 \ubd88\ud654\uac00 \uc77c\uae30 \uc2dc\uc791\ud558\uc600\ub2e4. \uc544\uc774\ud3f0 \ucd9c\uc2dc\ub85c \uc778\ud574 \uc2a4\ub9c8\ud2b8\ud3f0 \ubd80\ubd84\uc5d0\uc11c \uc0bc\uc131\uc740 \ub9e4\ucd9c\uacfc \uc774\ubbf8\uc9c0\uc5d0 \ud0c0\uaca9\uc744 \uc785\uc5c8\uc73c\uba70, \uc0bc\uc131\uc758 \ub300\ud45c\uc801\uc778 \uc2a4\ub9c8\ud2b8\ud3f0\uc778 \uc634\ub2c8\uc5442\uc758 \ub9cc\uc871\ub3c4 \uc870\uc0ac\uc640, \uc2a4\ub9c8\ud2b8\ud3f0 \uc120\ud638\ub3c4 \uc870\uc0ac\uc5d0\uc11c \uc0bc\uc131\uc740 \uc560\ud50c\uc5d0 \ud06c\uac8c \ub5a8\uc5b4\uc838 \uad74\uc695\uc744 \ub9db\ubcf4\uc544\uc57c \ud588\ub2e4. \uc0bc\uc131\uc740 \uadf8\uc5d0 \ub530\ub77c KT\uc5d0 \ub300\ud55c \ubd88\ud3b8\ud55c \uc2ec\uae30\ub97c \uac10\ucd94\uc9c0 \uc54a\uc558\uc73c\uba70, \uc774\ud6c4 \uc0bc\uc131\uc758 \uac01\uc885 \uc2a4\ub9c8\ud2b8\ud3f0 \ucd9c\uc2dc\uc5d0\uc11c KT\uac00 \uc81c\uc678\ub418\uac70\ub098, \ubcf4\uc870\uae08 \uc9c0\uae09 \ub300\uc0c1\uc5d0\uc11c \uc81c\uc678\ub418\uac70\ub098, \uc5c5\uadf8\ub808\uc774\ub4dc\ub97c \ud574\uc8fc\uc9c0 \uc54a\uac70\ub098, \uc2ec\uc9c0\uc5b4 '\uc1fc\uc634\ub2c8\uc544' \uba85\uce6d \uc870\ucc28 \uc0ac\uc6a9\ud560 \uc218 \uc5c6\uac8c \ud558\ub294 \ub4f1 KT\uc5d0 \ub300\ud55c \uc0bc\uc131\uc758 \ucc28\ubcc4\uc774 \uc2dc\uc791\ub418\uc5c8\ub2e4. KT\ub294 \uc544\uc774\ud3f0\uc744 \ucd9c\uc2dc\ud568\uc73c\ub85c \uc778\ud574 \ud070 \uc601\ud5a5\ub825\uc744 \ud589\uc0ac\ud558\ub294 \uacc4\uae30\uac00 \ub418\uc5c8\uc73c\ub098, \ub300\ud55c\ubbfc\uad6d \ucd5c\ub300 \ud734\ub300 \uc804\ud654 \uc81c\uc870\uc0ac\uc778 \uc0bc\uc131\uacfc\uc758 \ubd88\ud654\ub85c \uc778\ud574 \ub300\ud55c\ubbfc\uad6d\uc758 \uc2a4\ub9c8\ud2b8\ud3f0\uc744 \uc6d0\ud65c\ud558\uac8c \ud655\ubcf4\ud558\uae30\uac00 \uc5b4\ub835\uac8c \ub418\ub294 \uc704\uae30\uc5d0 \ucc98\ud558\uae30\ub3c4 \ud558\uc600\ub2e4. \uc774\ud6c4 SK\ud154\ub808\ucf64\uc744 \ud1b5\ud574\uc11c \ucd9c\uc2dc\ub41c \uc0bc\uc131\uc758 \uc2a4\ub9c8\ud2b8\ud3f0 '\uac24\ub7ed\uc2dc' \uc2dc\ub9ac\uc988\ub3c4 \uba87\uac1c\uc6d4 \uc774\uc0c1 \uc9c0\ub098\uc11c KT\uc5d0\uc11c \ucd9c\uc2dc\ub418\uc5c8\uc73c\uba70 \uc0bc\uc131\uc758 '\ubc14\ub2e4' \uc6b4\uc601 \uccb4\uc81c\ub97c \ud0d1\uc7ac\ud55c '\uc6e8\uc774\ube0c2'\ub294 KT\uc5d0\uc11c \uc544\uc608 \ucd9c\uc2dc \ud560 \uacc4\ud68d\uc774 \uc5c6\ub294 \uac83\uc73c\ub85c \uc54c\ub824\uc84c\ub2e4. \uc774\uc640\ub294 \ubcc4\uac1c\ub85c \uc6e8\uc774\ube0c3\ub294 KT\uc5d0 \uc815\uc0c1\uc801\uc73c\ub85c \ucd9c\uc2dc\ub418\uc5c8\ub2e4.", "answer": "KT", "sentence": "\uc544\uc774\ud3f0\uc744 \ub3c4\uc785\ud55c \uc774\ud6c4\ub85c KT \uc640 \uc0bc\uc131\uc804\uc790\uac04\uc758 \ubd88\ud654\uac00 \uc77c\uae30 \uc2dc\uc791\ud558\uc600\ub2e4.", "paragraph_sentence": " \uc544\uc774\ud3f0\uc744 \ub3c4\uc785\ud55c \uc774\ud6c4\ub85c KT \uc640 \uc0bc\uc131\uc804\uc790\uac04\uc758 \ubd88\ud654\uac00 \uc77c\uae30 \uc2dc\uc791\ud558\uc600\ub2e4. \uc544\uc774\ud3f0 \ucd9c\uc2dc\ub85c \uc778\ud574 \uc2a4\ub9c8\ud2b8\ud3f0 \ubd80\ubd84\uc5d0\uc11c \uc0bc\uc131\uc740 \ub9e4\ucd9c\uacfc \uc774\ubbf8\uc9c0\uc5d0 \ud0c0\uaca9\uc744 \uc785\uc5c8\uc73c\uba70, \uc0bc\uc131\uc758 \ub300\ud45c\uc801\uc778 \uc2a4\ub9c8\ud2b8\ud3f0\uc778 \uc634\ub2c8\uc5442\uc758 \ub9cc\uc871\ub3c4 \uc870\uc0ac\uc640, \uc2a4\ub9c8\ud2b8\ud3f0 \uc120\ud638\ub3c4 \uc870\uc0ac\uc5d0\uc11c \uc0bc\uc131\uc740 \uc560\ud50c\uc5d0 \ud06c\uac8c \ub5a8\uc5b4\uc838 \uad74\uc695\uc744 \ub9db\ubcf4\uc544\uc57c \ud588\ub2e4. \uc0bc\uc131\uc740 \uadf8\uc5d0 \ub530\ub77c KT\uc5d0 \ub300\ud55c \ubd88\ud3b8\ud55c \uc2ec\uae30\ub97c \uac10\ucd94\uc9c0 \uc54a\uc558\uc73c\uba70, \uc774\ud6c4 \uc0bc\uc131\uc758 \uac01\uc885 \uc2a4\ub9c8\ud2b8\ud3f0 \ucd9c\uc2dc\uc5d0\uc11c KT\uac00 \uc81c\uc678\ub418\uac70\ub098, \ubcf4\uc870\uae08 \uc9c0\uae09 \ub300\uc0c1\uc5d0\uc11c \uc81c\uc678\ub418\uac70\ub098, \uc5c5\uadf8\ub808\uc774\ub4dc\ub97c \ud574\uc8fc\uc9c0 \uc54a\uac70\ub098, \uc2ec\uc9c0\uc5b4 '\uc1fc\uc634\ub2c8\uc544' \uba85\uce6d \uc870\ucc28 \uc0ac\uc6a9\ud560 \uc218 \uc5c6\uac8c \ud558\ub294 \ub4f1 KT\uc5d0 \ub300\ud55c \uc0bc\uc131\uc758 \ucc28\ubcc4\uc774 \uc2dc\uc791\ub418\uc5c8\ub2e4. KT\ub294 \uc544\uc774\ud3f0\uc744 \ucd9c\uc2dc\ud568\uc73c\ub85c \uc778\ud574 \ud070 \uc601\ud5a5\ub825\uc744 \ud589\uc0ac\ud558\ub294 \uacc4\uae30\uac00 \ub418\uc5c8\uc73c\ub098, \ub300\ud55c\ubbfc\uad6d \ucd5c\ub300 \ud734\ub300 \uc804\ud654 \uc81c\uc870\uc0ac\uc778 \uc0bc\uc131\uacfc\uc758 \ubd88\ud654\ub85c \uc778\ud574 \ub300\ud55c\ubbfc\uad6d\uc758 \uc2a4\ub9c8\ud2b8\ud3f0\uc744 \uc6d0\ud65c\ud558\uac8c \ud655\ubcf4\ud558\uae30\uac00 \uc5b4\ub835\uac8c \ub418\ub294 \uc704\uae30\uc5d0 \ucc98\ud558\uae30\ub3c4 \ud558\uc600\ub2e4. \uc774\ud6c4 SK\ud154\ub808\ucf64\uc744 \ud1b5\ud574\uc11c \ucd9c\uc2dc\ub41c \uc0bc\uc131\uc758 \uc2a4\ub9c8\ud2b8\ud3f0 '\uac24\ub7ed\uc2dc' \uc2dc\ub9ac\uc988\ub3c4 \uba87\uac1c\uc6d4 \uc774\uc0c1 \uc9c0\ub098\uc11c KT\uc5d0\uc11c \ucd9c\uc2dc\ub418\uc5c8\uc73c\uba70 \uc0bc\uc131\uc758 '\ubc14\ub2e4' \uc6b4\uc601 \uccb4\uc81c\ub97c \ud0d1\uc7ac\ud55c '\uc6e8\uc774\ube0c2'\ub294 KT\uc5d0\uc11c \uc544\uc608 \ucd9c\uc2dc \ud560 \uacc4\ud68d\uc774 \uc5c6\ub294 \uac83\uc73c\ub85c \uc54c\ub824\uc84c\ub2e4. \uc774\uc640\ub294 \ubcc4\uac1c\ub85c \uc6e8\uc774\ube0c3\ub294 KT\uc5d0 \uc815\uc0c1\uc801\uc73c\ub85c \ucd9c\uc2dc\ub418\uc5c8\ub2e4.", "paragraph_answer": "\uc544\uc774\ud3f0\uc744 \ub3c4\uc785\ud55c \uc774\ud6c4\ub85c KT \uc640 \uc0bc\uc131\uc804\uc790\uac04\uc758 \ubd88\ud654\uac00 \uc77c\uae30 \uc2dc\uc791\ud558\uc600\ub2e4. \uc544\uc774\ud3f0 \ucd9c\uc2dc\ub85c \uc778\ud574 \uc2a4\ub9c8\ud2b8\ud3f0 \ubd80\ubd84\uc5d0\uc11c \uc0bc\uc131\uc740 \ub9e4\ucd9c\uacfc \uc774\ubbf8\uc9c0\uc5d0 \ud0c0\uaca9\uc744 \uc785\uc5c8\uc73c\uba70, \uc0bc\uc131\uc758 \ub300\ud45c\uc801\uc778 \uc2a4\ub9c8\ud2b8\ud3f0\uc778 \uc634\ub2c8\uc5442\uc758 \ub9cc\uc871\ub3c4 \uc870\uc0ac\uc640, \uc2a4\ub9c8\ud2b8\ud3f0 \uc120\ud638\ub3c4 \uc870\uc0ac\uc5d0\uc11c \uc0bc\uc131\uc740 \uc560\ud50c\uc5d0 \ud06c\uac8c \ub5a8\uc5b4\uc838 \uad74\uc695\uc744 \ub9db\ubcf4\uc544\uc57c \ud588\ub2e4. \uc0bc\uc131\uc740 \uadf8\uc5d0 \ub530\ub77c KT\uc5d0 \ub300\ud55c \ubd88\ud3b8\ud55c \uc2ec\uae30\ub97c \uac10\ucd94\uc9c0 \uc54a\uc558\uc73c\uba70, \uc774\ud6c4 \uc0bc\uc131\uc758 \uac01\uc885 \uc2a4\ub9c8\ud2b8\ud3f0 \ucd9c\uc2dc\uc5d0\uc11c KT\uac00 \uc81c\uc678\ub418\uac70\ub098, \ubcf4\uc870\uae08 \uc9c0\uae09 \ub300\uc0c1\uc5d0\uc11c \uc81c\uc678\ub418\uac70\ub098, \uc5c5\uadf8\ub808\uc774\ub4dc\ub97c \ud574\uc8fc\uc9c0 \uc54a\uac70\ub098, \uc2ec\uc9c0\uc5b4 '\uc1fc\uc634\ub2c8\uc544' \uba85\uce6d \uc870\ucc28 \uc0ac\uc6a9\ud560 \uc218 \uc5c6\uac8c \ud558\ub294 \ub4f1 KT\uc5d0 \ub300\ud55c \uc0bc\uc131\uc758 \ucc28\ubcc4\uc774 \uc2dc\uc791\ub418\uc5c8\ub2e4. KT\ub294 \uc544\uc774\ud3f0\uc744 \ucd9c\uc2dc\ud568\uc73c\ub85c \uc778\ud574 \ud070 \uc601\ud5a5\ub825\uc744 \ud589\uc0ac\ud558\ub294 \uacc4\uae30\uac00 \ub418\uc5c8\uc73c\ub098, \ub300\ud55c\ubbfc\uad6d \ucd5c\ub300 \ud734\ub300 \uc804\ud654 \uc81c\uc870\uc0ac\uc778 \uc0bc\uc131\uacfc\uc758 \ubd88\ud654\ub85c \uc778\ud574 \ub300\ud55c\ubbfc\uad6d\uc758 \uc2a4\ub9c8\ud2b8\ud3f0\uc744 \uc6d0\ud65c\ud558\uac8c \ud655\ubcf4\ud558\uae30\uac00 \uc5b4\ub835\uac8c \ub418\ub294 \uc704\uae30\uc5d0 \ucc98\ud558\uae30\ub3c4 \ud558\uc600\ub2e4. \uc774\ud6c4 SK\ud154\ub808\ucf64\uc744 \ud1b5\ud574\uc11c \ucd9c\uc2dc\ub41c \uc0bc\uc131\uc758 \uc2a4\ub9c8\ud2b8\ud3f0 '\uac24\ub7ed\uc2dc' \uc2dc\ub9ac\uc988\ub3c4 \uba87\uac1c\uc6d4 \uc774\uc0c1 \uc9c0\ub098\uc11c KT\uc5d0\uc11c \ucd9c\uc2dc\ub418\uc5c8\uc73c\uba70 \uc0bc\uc131\uc758 '\ubc14\ub2e4' \uc6b4\uc601 \uccb4\uc81c\ub97c \ud0d1\uc7ac\ud55c '\uc6e8\uc774\ube0c2'\ub294 KT\uc5d0\uc11c \uc544\uc608 \ucd9c\uc2dc \ud560 \uacc4\ud68d\uc774 \uc5c6\ub294 \uac83\uc73c\ub85c \uc54c\ub824\uc84c\ub2e4. \uc774\uc640\ub294 \ubcc4\uac1c\ub85c \uc6e8\uc774\ube0c3\ub294 KT\uc5d0 \uc815\uc0c1\uc801\uc73c\ub85c \ucd9c\uc2dc\ub418\uc5c8\ub2e4.", "sentence_answer": "\uc544\uc774\ud3f0\uc744 \ub3c4\uc785\ud55c \uc774\ud6c4\ub85c KT \uc640 \uc0bc\uc131\uc804\uc790\uac04\uc758 \ubd88\ud654\uac00 \uc77c\uae30 \uc2dc\uc791\ud558\uc600\ub2e4.", "paragraph_id": "6038335-4-0", "paragraph_question": "question: \ub300\ud55c\ubbfc\uad6d\uc5d0\uc11c \uc544\uc774\ud3f0\uc774 \ucd9c\uc2dc\ub41c \uc774\ud6c4 \uc0bc\uc131\uacfc \ub9c8\ucc30\uc774 \uc0dd\uae34 \ud1b5\uc2e0\uc5c5\uccb4\ub294?, context: \uc544\uc774\ud3f0\uc744 \ub3c4\uc785\ud55c \uc774\ud6c4\ub85c KT\uc640 \uc0bc\uc131\uc804\uc790\uac04\uc758 \ubd88\ud654\uac00 \uc77c\uae30 \uc2dc\uc791\ud558\uc600\ub2e4. \uc544\uc774\ud3f0 \ucd9c\uc2dc\ub85c \uc778\ud574 \uc2a4\ub9c8\ud2b8\ud3f0 \ubd80\ubd84\uc5d0\uc11c \uc0bc\uc131\uc740 \ub9e4\ucd9c\uacfc \uc774\ubbf8\uc9c0\uc5d0 \ud0c0\uaca9\uc744 \uc785\uc5c8\uc73c\uba70, \uc0bc\uc131\uc758 \ub300\ud45c\uc801\uc778 \uc2a4\ub9c8\ud2b8\ud3f0\uc778 \uc634\ub2c8\uc5442\uc758 \ub9cc\uc871\ub3c4 \uc870\uc0ac\uc640, \uc2a4\ub9c8\ud2b8\ud3f0 \uc120\ud638\ub3c4 \uc870\uc0ac\uc5d0\uc11c \uc0bc\uc131\uc740 \uc560\ud50c\uc5d0 \ud06c\uac8c \ub5a8\uc5b4\uc838 \uad74\uc695\uc744 \ub9db\ubcf4\uc544\uc57c \ud588\ub2e4. \uc0bc\uc131\uc740 \uadf8\uc5d0 \ub530\ub77c KT\uc5d0 \ub300\ud55c \ubd88\ud3b8\ud55c \uc2ec\uae30\ub97c \uac10\ucd94\uc9c0 \uc54a\uc558\uc73c\uba70, \uc774\ud6c4 \uc0bc\uc131\uc758 \uac01\uc885 \uc2a4\ub9c8\ud2b8\ud3f0 \ucd9c\uc2dc\uc5d0\uc11c KT\uac00 \uc81c\uc678\ub418\uac70\ub098, \ubcf4\uc870\uae08 \uc9c0\uae09 \ub300\uc0c1\uc5d0\uc11c \uc81c\uc678\ub418\uac70\ub098, \uc5c5\uadf8\ub808\uc774\ub4dc\ub97c \ud574\uc8fc\uc9c0 \uc54a\uac70\ub098, \uc2ec\uc9c0\uc5b4 '\uc1fc\uc634\ub2c8\uc544' \uba85\uce6d \uc870\ucc28 \uc0ac\uc6a9\ud560 \uc218 \uc5c6\uac8c \ud558\ub294 \ub4f1 KT\uc5d0 \ub300\ud55c \uc0bc\uc131\uc758 \ucc28\ubcc4\uc774 \uc2dc\uc791\ub418\uc5c8\ub2e4. KT\ub294 \uc544\uc774\ud3f0\uc744 \ucd9c\uc2dc\ud568\uc73c\ub85c \uc778\ud574 \ud070 \uc601\ud5a5\ub825\uc744 \ud589\uc0ac\ud558\ub294 \uacc4\uae30\uac00 \ub418\uc5c8\uc73c\ub098, \ub300\ud55c\ubbfc\uad6d \ucd5c\ub300 \ud734\ub300 \uc804\ud654 \uc81c\uc870\uc0ac\uc778 \uc0bc\uc131\uacfc\uc758 \ubd88\ud654\ub85c \uc778\ud574 \ub300\ud55c\ubbfc\uad6d\uc758 \uc2a4\ub9c8\ud2b8\ud3f0\uc744 \uc6d0\ud65c\ud558\uac8c \ud655\ubcf4\ud558\uae30\uac00 \uc5b4\ub835\uac8c \ub418\ub294 \uc704\uae30\uc5d0 \ucc98\ud558\uae30\ub3c4 \ud558\uc600\ub2e4. \uc774\ud6c4 SK\ud154\ub808\ucf64\uc744 \ud1b5\ud574\uc11c \ucd9c\uc2dc\ub41c \uc0bc\uc131\uc758 \uc2a4\ub9c8\ud2b8\ud3f0 '\uac24\ub7ed\uc2dc' \uc2dc\ub9ac\uc988\ub3c4 \uba87\uac1c\uc6d4 \uc774\uc0c1 \uc9c0\ub098\uc11c KT\uc5d0\uc11c \ucd9c\uc2dc\ub418\uc5c8\uc73c\uba70 \uc0bc\uc131\uc758 '\ubc14\ub2e4' \uc6b4\uc601 \uccb4\uc81c\ub97c \ud0d1\uc7ac\ud55c '\uc6e8\uc774\ube0c2'\ub294 KT\uc5d0\uc11c \uc544\uc608 \ucd9c\uc2dc \ud560 \uacc4\ud68d\uc774 \uc5c6\ub294 \uac83\uc73c\ub85c \uc54c\ub824\uc84c\ub2e4. \uc774\uc640\ub294 \ubcc4\uac1c\ub85c \uc6e8\uc774\ube0c3\ub294 KT\uc5d0 \uc815\uc0c1\uc801\uc73c\ub85c \ucd9c\uc2dc\ub418\uc5c8\ub2e4."} {"question": "\uac74\ub96d\uc81c\uc758 \uc5ec\ub984 \uc5f4\uc7a5\uc740?", "paragraph": "\ub2f9\uc2dc \uac74\ub96d\uc81c\ub294 \ub9c9 \uc644\uacf5\ub41c \uc5ec\ub984 \ubcc4\uc7a5\uc778 \uc5f4\ud558 \ud53c\uc11c\uc0b0\uc7a5\uc73c\ub85c \uac00\uc11c \uc9c0\ub0b4\uace0 \uc788\uc5c8\ub2e4. \uadf8\ub807\uae30\uc5d0 \ucc9c\uc9c4\uc73c\ub85c \uc628 \ub2e4\uc74c \ubd81\uacbd\uc5d0\uc11c \uc5ec\ub3c5\uc744 \ud480\uace0 \ub2e4\uc2dc \ub9cc\ub9ac\uc7a5\uc131\uc744 \ub118\uc5b4 1793\ub144(\uac74\ub96d 58\ub144) 8\uc6d4\uc5d0\uc57c \ud53c\uc11c\uc0b0\uc7a5\uc5d0 \ub3c4\ucc29\ud574 \uac74\ub96d\uc81c\ub97c \uc54c\ud604\ud558\uc600\ub2e4. \uac74\ub96d\uc81c\ub294 \ud654\uc2e0\uc5d0\uac8c \uc0ac\uc808\ub2e8\uc744 \ub300\uc811\ud558\ub294\ub370 \uc18c\ud640\ud558\uc9c0 \uc54a\ub3c4\ub85d \uba85\uc744 \ub0b4\ub838\uc73c\ub098 \ub3c4\uc911 \uc811\uacac\uc758 \uc608\uc758\ubb38\uc81c\ub85c \ub09c\ud56d\uc744 \ube5a\uac8c \ub418\uc5c8\ub2e4. \ubcf8\ub798 \uc678\uad6d\uc758 \uc0ac\uc2e0\uc740 \ud669\uc81c\ub97c \uc811\uacac\ud560 \ub54c \uc0bc\uada4\uad6c\uace0\ub450(\u4e09\u8dea\u4e5d\u53e9\u982d), \uc989 \uc138 \ubc88 \uc808\ud558\uace0 \uc544\ud649 \ubc88 \uba38\ub9ac \ub97c \ub545\uc5d0 \uc870\uc544\ub9ac\ub294 \uc608\ub97c \ucde8\ud558\ub3c4\ub85d \ud558\uc600\ub2e4. \uadf8\ub7ec\ub098 \ub9e4\uce74\ud2b8\ub2c8\ub294 \uc790\uc2e0\uc740 \uccad\ub098\ub77c\uc758 \uc18d\uad6d\uc758 \uc2e0\ud558\uac00 \uc544\ub2c8\ub77c\uba70 \ub2e8\ud638\ud788 \uac70\uc808\ud558\uc600\uace0 \uacb0\uad6d \ud569\uc758 \ub05d\uc5d0 \uac74\ub96d\uc81c \ub4a4\uc5d0 \uc601\uad6d \uad6d\uc655\uc758 \ucd08\uc0c1\ud654\ub97c \uac78\uc5b4\ub193\uace0 \ud55c\ucabd \ubb34\ub98e\ub9cc\uc744 \uad7d\ud788\uba70 \uc601\uad6d\uc2dd \uc608\ub9cc \ucde8\ud558\ub3c4\ub85d \ud558\uc600\ub2e4. \uac74\ub96d\uc81c\ub294 \ud0c4\uc2e0 \ub9cc\ucc2c\uc5d0\uc11c \ub9e4\uce74\ud2b8\ub2c8\uac00 \uac00\uc838\uc628 \uc870\uc9c0 3\uc138\uc758 \uce5c\uc11c\ub97c \uac74\ub124 \ubc1b\uc558\ub294\ub370 \uac70\uae30\uc5d0\ub294 \ubb34\uc5ed\uc744 \ub298\ub9ac\uace0 \uacf5\uc0ac\ub97c \uc0c1\uc8fc\uc2dc\ud0a4\uace0 \uc2f6\ub2e4\ub294 \uc694\uad6c\uac00 \ub4e4\uc5b4 \uc788\uc5c8\ub2e4. \uac70\uae30\uc5d0\ub2e4 \uc601\uad6d\uc5d0\uac8c \uc8fc\uc0b0 \uadfc\ucc98\uc758 \uc791\uc740 \uc12c\uc744 \ud560\uc591\ud574 \uc601\uad6d\uc778\ub4e4\uc774 \uc0ac\uc6a9\ud558\uac8c \ud574\ub2ec\ub77c\ub294 \ub0b4\uc6a9\ub3c4 \uc4f0\uc5ec\uc788\uc790 \uac74\ub96d\uc81c\ub294 \uc774\ub97c \uac70\ubd80\ud558\uace0 \uc601\uad6d \uc0ac\uc808\ub2e8\uc758 \uc77c\uccb4\uc758 \ud589\ub3d9\uc744 \uae08\ud558\uc600\ub2e4. \uac74\ub96d\uc81c\ub294 \uadf8 \ub2e4\uc74c \ub2ec\uc778 9\uc6d4\uc5d0 \uc601\uad6d \uc0ac\uc808\ub2e8\uc758 \uadc0\uad6d\uc744 \uac15\uc81c \uc870\ucc98\ud558\uc600\ub2e4.", "answer": "\uc5f4\ud558 \ud53c\uc11c\uc0b0\uc7a5", "sentence": "\ub2f9\uc2dc \uac74\ub96d\uc81c\ub294 \ub9c9 \uc644\uacf5\ub41c \uc5ec\ub984 \ubcc4\uc7a5\uc778 \uc5f4\ud558 \ud53c\uc11c\uc0b0\uc7a5 \uc73c\ub85c \uac00\uc11c \uc9c0\ub0b4\uace0 \uc788\uc5c8\ub2e4.", "paragraph_sentence": " \ub2f9\uc2dc \uac74\ub96d\uc81c\ub294 \ub9c9 \uc644\uacf5\ub41c \uc5ec\ub984 \ubcc4\uc7a5\uc778 \uc5f4\ud558 \ud53c\uc11c\uc0b0\uc7a5 \uc73c\ub85c \uac00\uc11c \uc9c0\ub0b4\uace0 \uc788\uc5c8\ub2e4. \uadf8\ub807\uae30\uc5d0 \ucc9c\uc9c4\uc73c\ub85c \uc628 \ub2e4\uc74c \ubd81\uacbd\uc5d0\uc11c \uc5ec\ub3c5\uc744 \ud480\uace0 \ub2e4\uc2dc \ub9cc\ub9ac\uc7a5\uc131\uc744 \ub118\uc5b4 1793\ub144(\uac74\ub96d 58\ub144) 8\uc6d4\uc5d0\uc57c \ud53c\uc11c\uc0b0\uc7a5\uc5d0 \ub3c4\ucc29\ud574 \uac74\ub96d\uc81c\ub97c \uc54c\ud604\ud558\uc600\ub2e4. \uac74\ub96d\uc81c\ub294 \ud654\uc2e0\uc5d0\uac8c \uc0ac\uc808\ub2e8\uc744 \ub300\uc811\ud558\ub294\ub370 \uc18c\ud640\ud558\uc9c0 \uc54a\ub3c4\ub85d \uba85\uc744 \ub0b4\ub838\uc73c\ub098 \ub3c4\uc911 \uc811\uacac\uc758 \uc608\uc758\ubb38\uc81c\ub85c \ub09c\ud56d\uc744 \ube5a\uac8c \ub418\uc5c8\ub2e4. \ubcf8\ub798 \uc678\uad6d\uc758 \uc0ac\uc2e0\uc740 \ud669\uc81c\ub97c \uc811\uacac\ud560 \ub54c \uc0bc\uada4\uad6c\uace0\ub450(\u4e09\u8dea\u4e5d\u53e9\u982d), \uc989 \uc138 \ubc88 \uc808\ud558\uace0 \uc544\ud649 \ubc88 \uba38\ub9ac \ub97c \ub545\uc5d0 \uc870\uc544\ub9ac\ub294 \uc608\ub97c \ucde8\ud558\ub3c4\ub85d \ud558\uc600\ub2e4. \uadf8\ub7ec\ub098 \ub9e4\uce74\ud2b8\ub2c8\ub294 \uc790\uc2e0\uc740 \uccad\ub098\ub77c\uc758 \uc18d\uad6d\uc758 \uc2e0\ud558\uac00 \uc544\ub2c8\ub77c\uba70 \ub2e8\ud638\ud788 \uac70\uc808\ud558\uc600\uace0 \uacb0\uad6d \ud569\uc758 \ub05d\uc5d0 \uac74\ub96d\uc81c \ub4a4\uc5d0 \uc601\uad6d \uad6d\uc655\uc758 \ucd08\uc0c1\ud654\ub97c \uac78\uc5b4\ub193\uace0 \ud55c\ucabd \ubb34\ub98e\ub9cc\uc744 \uad7d\ud788\uba70 \uc601\uad6d\uc2dd \uc608\ub9cc \ucde8\ud558\ub3c4\ub85d \ud558\uc600\ub2e4. \uac74\ub96d\uc81c\ub294 \ud0c4\uc2e0 \ub9cc\ucc2c\uc5d0\uc11c \ub9e4\uce74\ud2b8\ub2c8\uac00 \uac00\uc838\uc628 \uc870\uc9c0 3\uc138\uc758 \uce5c\uc11c\ub97c \uac74\ub124 \ubc1b\uc558\ub294\ub370 \uac70\uae30\uc5d0\ub294 \ubb34\uc5ed\uc744 \ub298\ub9ac\uace0 \uacf5\uc0ac\ub97c \uc0c1\uc8fc\uc2dc\ud0a4\uace0 \uc2f6\ub2e4\ub294 \uc694\uad6c\uac00 \ub4e4\uc5b4 \uc788\uc5c8\ub2e4. \uac70\uae30\uc5d0\ub2e4 \uc601\uad6d\uc5d0\uac8c \uc8fc\uc0b0 \uadfc\ucc98\uc758 \uc791\uc740 \uc12c\uc744 \ud560\uc591\ud574 \uc601\uad6d\uc778\ub4e4\uc774 \uc0ac\uc6a9\ud558\uac8c \ud574\ub2ec\ub77c\ub294 \ub0b4\uc6a9\ub3c4 \uc4f0\uc5ec\uc788\uc790 \uac74\ub96d\uc81c\ub294 \uc774\ub97c \uac70\ubd80\ud558\uace0 \uc601\uad6d \uc0ac\uc808\ub2e8\uc758 \uc77c\uccb4\uc758 \ud589\ub3d9\uc744 \uae08\ud558\uc600\ub2e4. \uac74\ub96d\uc81c\ub294 \uadf8 \ub2e4\uc74c \ub2ec\uc778 9\uc6d4\uc5d0 \uc601\uad6d \uc0ac\uc808\ub2e8\uc758 \uadc0\uad6d\uc744 \uac15\uc81c \uc870\ucc98\ud558\uc600\ub2e4.", "paragraph_answer": "\ub2f9\uc2dc \uac74\ub96d\uc81c\ub294 \ub9c9 \uc644\uacf5\ub41c \uc5ec\ub984 \ubcc4\uc7a5\uc778 \uc5f4\ud558 \ud53c\uc11c\uc0b0\uc7a5 \uc73c\ub85c \uac00\uc11c \uc9c0\ub0b4\uace0 \uc788\uc5c8\ub2e4. \uadf8\ub807\uae30\uc5d0 \ucc9c\uc9c4\uc73c\ub85c \uc628 \ub2e4\uc74c \ubd81\uacbd\uc5d0\uc11c \uc5ec\ub3c5\uc744 \ud480\uace0 \ub2e4\uc2dc \ub9cc\ub9ac\uc7a5\uc131\uc744 \ub118\uc5b4 1793\ub144(\uac74\ub96d 58\ub144) 8\uc6d4\uc5d0\uc57c \ud53c\uc11c\uc0b0\uc7a5\uc5d0 \ub3c4\ucc29\ud574 \uac74\ub96d\uc81c\ub97c \uc54c\ud604\ud558\uc600\ub2e4. \uac74\ub96d\uc81c\ub294 \ud654\uc2e0\uc5d0\uac8c \uc0ac\uc808\ub2e8\uc744 \ub300\uc811\ud558\ub294\ub370 \uc18c\ud640\ud558\uc9c0 \uc54a\ub3c4\ub85d \uba85\uc744 \ub0b4\ub838\uc73c\ub098 \ub3c4\uc911 \uc811\uacac\uc758 \uc608\uc758\ubb38\uc81c\ub85c \ub09c\ud56d\uc744 \ube5a\uac8c \ub418\uc5c8\ub2e4. \ubcf8\ub798 \uc678\uad6d\uc758 \uc0ac\uc2e0\uc740 \ud669\uc81c\ub97c \uc811\uacac\ud560 \ub54c \uc0bc\uada4\uad6c\uace0\ub450(\u4e09\u8dea\u4e5d\u53e9\u982d), \uc989 \uc138 \ubc88 \uc808\ud558\uace0 \uc544\ud649 \ubc88 \uba38\ub9ac \ub97c \ub545\uc5d0 \uc870\uc544\ub9ac\ub294 \uc608\ub97c \ucde8\ud558\ub3c4\ub85d \ud558\uc600\ub2e4. \uadf8\ub7ec\ub098 \ub9e4\uce74\ud2b8\ub2c8\ub294 \uc790\uc2e0\uc740 \uccad\ub098\ub77c\uc758 \uc18d\uad6d\uc758 \uc2e0\ud558\uac00 \uc544\ub2c8\ub77c\uba70 \ub2e8\ud638\ud788 \uac70\uc808\ud558\uc600\uace0 \uacb0\uad6d \ud569\uc758 \ub05d\uc5d0 \uac74\ub96d\uc81c \ub4a4\uc5d0 \uc601\uad6d \uad6d\uc655\uc758 \ucd08\uc0c1\ud654\ub97c \uac78\uc5b4\ub193\uace0 \ud55c\ucabd \ubb34\ub98e\ub9cc\uc744 \uad7d\ud788\uba70 \uc601\uad6d\uc2dd \uc608\ub9cc \ucde8\ud558\ub3c4\ub85d \ud558\uc600\ub2e4. \uac74\ub96d\uc81c\ub294 \ud0c4\uc2e0 \ub9cc\ucc2c\uc5d0\uc11c \ub9e4\uce74\ud2b8\ub2c8\uac00 \uac00\uc838\uc628 \uc870\uc9c0 3\uc138\uc758 \uce5c\uc11c\ub97c \uac74\ub124 \ubc1b\uc558\ub294\ub370 \uac70\uae30\uc5d0\ub294 \ubb34\uc5ed\uc744 \ub298\ub9ac\uace0 \uacf5\uc0ac\ub97c \uc0c1\uc8fc\uc2dc\ud0a4\uace0 \uc2f6\ub2e4\ub294 \uc694\uad6c\uac00 \ub4e4\uc5b4 \uc788\uc5c8\ub2e4. \uac70\uae30\uc5d0\ub2e4 \uc601\uad6d\uc5d0\uac8c \uc8fc\uc0b0 \uadfc\ucc98\uc758 \uc791\uc740 \uc12c\uc744 \ud560\uc591\ud574 \uc601\uad6d\uc778\ub4e4\uc774 \uc0ac\uc6a9\ud558\uac8c \ud574\ub2ec\ub77c\ub294 \ub0b4\uc6a9\ub3c4 \uc4f0\uc5ec\uc788\uc790 \uac74\ub96d\uc81c\ub294 \uc774\ub97c \uac70\ubd80\ud558\uace0 \uc601\uad6d \uc0ac\uc808\ub2e8\uc758 \uc77c\uccb4\uc758 \ud589\ub3d9\uc744 \uae08\ud558\uc600\ub2e4. \uac74\ub96d\uc81c\ub294 \uadf8 \ub2e4\uc74c \ub2ec\uc778 9\uc6d4\uc5d0 \uc601\uad6d \uc0ac\uc808\ub2e8\uc758 \uadc0\uad6d\uc744 \uac15\uc81c \uc870\ucc98\ud558\uc600\ub2e4.", "sentence_answer": "\ub2f9\uc2dc \uac74\ub96d\uc81c\ub294 \ub9c9 \uc644\uacf5\ub41c \uc5ec\ub984 \ubcc4\uc7a5\uc778 \uc5f4\ud558 \ud53c\uc11c\uc0b0\uc7a5 \uc73c\ub85c \uac00\uc11c \uc9c0\ub0b4\uace0 \uc788\uc5c8\ub2e4.", "paragraph_id": "6391190-27-0", "paragraph_question": "question: \uac74\ub96d\uc81c\uc758 \uc5ec\ub984 \uc5f4\uc7a5\uc740?, context: \ub2f9\uc2dc \uac74\ub96d\uc81c\ub294 \ub9c9 \uc644\uacf5\ub41c \uc5ec\ub984 \ubcc4\uc7a5\uc778 \uc5f4\ud558 \ud53c\uc11c\uc0b0\uc7a5\uc73c\ub85c \uac00\uc11c \uc9c0\ub0b4\uace0 \uc788\uc5c8\ub2e4. \uadf8\ub807\uae30\uc5d0 \ucc9c\uc9c4\uc73c\ub85c \uc628 \ub2e4\uc74c \ubd81\uacbd\uc5d0\uc11c \uc5ec\ub3c5\uc744 \ud480\uace0 \ub2e4\uc2dc \ub9cc\ub9ac\uc7a5\uc131\uc744 \ub118\uc5b4 1793\ub144(\uac74\ub96d 58\ub144) 8\uc6d4\uc5d0\uc57c \ud53c\uc11c\uc0b0\uc7a5\uc5d0 \ub3c4\ucc29\ud574 \uac74\ub96d\uc81c\ub97c \uc54c\ud604\ud558\uc600\ub2e4. \uac74\ub96d\uc81c\ub294 \ud654\uc2e0\uc5d0\uac8c \uc0ac\uc808\ub2e8\uc744 \ub300\uc811\ud558\ub294\ub370 \uc18c\ud640\ud558\uc9c0 \uc54a\ub3c4\ub85d \uba85\uc744 \ub0b4\ub838\uc73c\ub098 \ub3c4\uc911 \uc811\uacac\uc758 \uc608\uc758\ubb38\uc81c\ub85c \ub09c\ud56d\uc744 \ube5a\uac8c \ub418\uc5c8\ub2e4. \ubcf8\ub798 \uc678\uad6d\uc758 \uc0ac\uc2e0\uc740 \ud669\uc81c\ub97c \uc811\uacac\ud560 \ub54c \uc0bc\uada4\uad6c\uace0\ub450(\u4e09\u8dea\u4e5d\u53e9\u982d), \uc989 \uc138 \ubc88 \uc808\ud558\uace0 \uc544\ud649 \ubc88 \uba38\ub9ac \ub97c \ub545\uc5d0 \uc870\uc544\ub9ac\ub294 \uc608\ub97c \ucde8\ud558\ub3c4\ub85d \ud558\uc600\ub2e4. \uadf8\ub7ec\ub098 \ub9e4\uce74\ud2b8\ub2c8\ub294 \uc790\uc2e0\uc740 \uccad\ub098\ub77c\uc758 \uc18d\uad6d\uc758 \uc2e0\ud558\uac00 \uc544\ub2c8\ub77c\uba70 \ub2e8\ud638\ud788 \uac70\uc808\ud558\uc600\uace0 \uacb0\uad6d \ud569\uc758 \ub05d\uc5d0 \uac74\ub96d\uc81c \ub4a4\uc5d0 \uc601\uad6d \uad6d\uc655\uc758 \ucd08\uc0c1\ud654\ub97c \uac78\uc5b4\ub193\uace0 \ud55c\ucabd \ubb34\ub98e\ub9cc\uc744 \uad7d\ud788\uba70 \uc601\uad6d\uc2dd \uc608\ub9cc \ucde8\ud558\ub3c4\ub85d \ud558\uc600\ub2e4. \uac74\ub96d\uc81c\ub294 \ud0c4\uc2e0 \ub9cc\ucc2c\uc5d0\uc11c \ub9e4\uce74\ud2b8\ub2c8\uac00 \uac00\uc838\uc628 \uc870\uc9c0 3\uc138\uc758 \uce5c\uc11c\ub97c \uac74\ub124 \ubc1b\uc558\ub294\ub370 \uac70\uae30\uc5d0\ub294 \ubb34\uc5ed\uc744 \ub298\ub9ac\uace0 \uacf5\uc0ac\ub97c \uc0c1\uc8fc\uc2dc\ud0a4\uace0 \uc2f6\ub2e4\ub294 \uc694\uad6c\uac00 \ub4e4\uc5b4 \uc788\uc5c8\ub2e4. \uac70\uae30\uc5d0\ub2e4 \uc601\uad6d\uc5d0\uac8c \uc8fc\uc0b0 \uadfc\ucc98\uc758 \uc791\uc740 \uc12c\uc744 \ud560\uc591\ud574 \uc601\uad6d\uc778\ub4e4\uc774 \uc0ac\uc6a9\ud558\uac8c \ud574\ub2ec\ub77c\ub294 \ub0b4\uc6a9\ub3c4 \uc4f0\uc5ec\uc788\uc790 \uac74\ub96d\uc81c\ub294 \uc774\ub97c \uac70\ubd80\ud558\uace0 \uc601\uad6d \uc0ac\uc808\ub2e8\uc758 \uc77c\uccb4\uc758 \ud589\ub3d9\uc744 \uae08\ud558\uc600\ub2e4. \uac74\ub96d\uc81c\ub294 \uadf8 \ub2e4\uc74c \ub2ec\uc778 9\uc6d4\uc5d0 \uc601\uad6d \uc0ac\uc808\ub2e8\uc758 \uadc0\uad6d\uc744 \uac15\uc81c \uc870\ucc98\ud558\uc600\ub2e4."} {"question": "\uae40\uba85\uc21c\uc528\uc758 \uc544\uba85\uc774\uc790 \ud544\uba85\uc740?", "paragraph": "\uae40\uba85\uc21c\uc740 1896\ub144 1\uc6d4 20\uc77c \ud3c9\uc548\ub0a8\ub3c4 \ud3c9\uc591\uad70 \uc735\ub355\uba74\uc5d0\uc11c \ud3c9\uc591\uc758 \uc9c0\uc8fc\uc774\uc790 \ubb38\uc2e0 \uad00\ub8cc\uc600\ub358 \uae40\ud76c\uacbd(\u91d1\u7fb2\u5e9a)\uc758 \uc18c\uc2e4\uc758 \ub538\ub85c \ud0dc\uc5b4\ub0ac\ub2e4. \uadf8\uc758 \ucd9c\uc0dd \uc5f0\ub300\ub294 \ub2e4\uc18c \ubd88\uba85\ud655\ud558\uc5ec 1897\ub144\uc0dd \uc124\ub3c4 \uc788\ub2e4. \ud0c4\uc2e4(\u5f48\u5be6)\uc740 \uadf8\uc758 \uc544\uba85\uc774\uc790 \ud544\uba85\uc774\uc5c8\ub2e4. \uc5b4\uba38\ub2c8\ub294 \uae30\uc0dd \ucd9c\uc2e0 \uc0b0\uc6d4(\u5c71\u6708)\uc774\ub2e4. \uadf8\ub7ec\ub098 \ud6c4\ub300\uc5d0 \uadf8\ub140\uc758 \uc5b4\uba38\ub2c8\ub294 \uae40\uc778\uc219(\u91d1\u4ec1\u6dd1)\uc774\ub77c\uace0\ub3c4 \ud558\uace0, \uae40\uc778\uc815(\u91d1\u4ec1\u8c9e)\uc774\ub77c\uace0 \uae30\ub85d\ub41c \uacf3\ub3c4 \uc788\ub294\ub370, \uadf8\uc758 \uc544\ubc84\uc9c0\uc5d0\uac8c \ucca9\uc774 \uc5ec\ub7ec \uba85 \uc788\uc5c8\uae30 \ub54c\ubb38\uc5d0 \uc774\ub7ec\ud55c \ud63c\ub780\uc774 \uc0dd\uae34 \uac83\uc774\ub2e4. \uae40\uba85\uc21c\uc774 \ub098\uc911\uc5d0 \uc9c0\uc740 \uc790\uc804\uc801 \uc18c\uc124 <\ud0c4\uc2e4\uc774\uc640 \uc8fc\uc601\uc774>\uc5d0\ub294 \uae30\uc0dd\uc774\uc5c8\ub358 \uadf8\uc758 \uc5b4\uba38\ub2c8 \uc774\ub984\uc774 \uc0b0\uc6d4(\u5c71\u6708)\uc774\ub77c \ud558\uc600\ub2e4. \uc544\ubc84\uc9c0 \uae40\ud76c\uacbd\uc740 \uad6c\ud55c\ub9d0\uc758 \uad00\ub8cc\ub85c \ud3c9\uc548\ub0a8\ub3c4 \ucc38\uc0ac\uad00(\u53c3\u4e8b\u5b98)\uc744 \uc9c0\ub0b4\uae30\ub3c4 \ud588\ub2e4.", "answer": "\ud0c4\uc2e4", "sentence": "\ud0c4\uc2e4 (\u5f48\u5be6)\uc740 \uadf8\uc758 \uc544\uba85\uc774\uc790 \ud544\uba85\uc774\uc5c8\ub2e4.", "paragraph_sentence": "\uae40\uba85\uc21c\uc740 1896\ub144 1\uc6d4 20\uc77c \ud3c9\uc548\ub0a8\ub3c4 \ud3c9\uc591\uad70 \uc735\ub355\uba74\uc5d0\uc11c \ud3c9\uc591\uc758 \uc9c0\uc8fc\uc774\uc790 \ubb38\uc2e0 \uad00\ub8cc\uc600\ub358 \uae40\ud76c\uacbd(\u91d1\u7fb2\u5e9a)\uc758 \uc18c\uc2e4\uc758 \ub538\ub85c \ud0dc\uc5b4\ub0ac\ub2e4. \uadf8\uc758 \ucd9c\uc0dd \uc5f0\ub300\ub294 \ub2e4\uc18c \ubd88\uba85\ud655\ud558\uc5ec 1897\ub144\uc0dd \uc124\ub3c4 \uc788\ub2e4. \ud0c4\uc2e4 (\u5f48\u5be6)\uc740 \uadf8\uc758 \uc544\uba85\uc774\uc790 \ud544\uba85\uc774\uc5c8\ub2e4. \uc5b4\uba38\ub2c8\ub294 \uae30\uc0dd \ucd9c\uc2e0 \uc0b0\uc6d4(\u5c71\u6708)\uc774\ub2e4. \uadf8\ub7ec\ub098 \ud6c4\ub300\uc5d0 \uadf8\ub140\uc758 \uc5b4\uba38\ub2c8\ub294 \uae40\uc778\uc219(\u91d1\u4ec1\u6dd1)\uc774\ub77c\uace0\ub3c4 \ud558\uace0, \uae40\uc778\uc815(\u91d1\u4ec1\u8c9e)\uc774\ub77c\uace0 \uae30\ub85d\ub41c \uacf3\ub3c4 \uc788\ub294\ub370, \uadf8\uc758 \uc544\ubc84\uc9c0\uc5d0\uac8c \ucca9\uc774 \uc5ec\ub7ec \uba85 \uc788\uc5c8\uae30 \ub54c\ubb38\uc5d0 \uc774\ub7ec\ud55c \ud63c\ub780\uc774 \uc0dd\uae34 \uac83\uc774\ub2e4. \uae40\uba85\uc21c\uc774 \ub098\uc911\uc5d0 \uc9c0\uc740 \uc790\uc804\uc801 \uc18c\uc124 <\ud0c4\uc2e4\uc774\uc640 \uc8fc\uc601\uc774>\uc5d0\ub294 \uae30\uc0dd\uc774\uc5c8\ub358 \uadf8\uc758 \uc5b4\uba38\ub2c8 \uc774\ub984\uc774 \uc0b0\uc6d4(\u5c71\u6708)\uc774\ub77c \ud558\uc600\ub2e4. \uc544\ubc84\uc9c0 \uae40\ud76c\uacbd\uc740 \uad6c\ud55c\ub9d0\uc758 \uad00\ub8cc\ub85c \ud3c9\uc548\ub0a8\ub3c4 \ucc38\uc0ac\uad00(\u53c3\u4e8b\u5b98)\uc744 \uc9c0\ub0b4\uae30\ub3c4 \ud588\ub2e4.", "paragraph_answer": "\uae40\uba85\uc21c\uc740 1896\ub144 1\uc6d4 20\uc77c \ud3c9\uc548\ub0a8\ub3c4 \ud3c9\uc591\uad70 \uc735\ub355\uba74\uc5d0\uc11c \ud3c9\uc591\uc758 \uc9c0\uc8fc\uc774\uc790 \ubb38\uc2e0 \uad00\ub8cc\uc600\ub358 \uae40\ud76c\uacbd(\u91d1\u7fb2\u5e9a)\uc758 \uc18c\uc2e4\uc758 \ub538\ub85c \ud0dc\uc5b4\ub0ac\ub2e4. \uadf8\uc758 \ucd9c\uc0dd \uc5f0\ub300\ub294 \ub2e4\uc18c \ubd88\uba85\ud655\ud558\uc5ec 1897\ub144\uc0dd \uc124\ub3c4 \uc788\ub2e4. \ud0c4\uc2e4 (\u5f48\u5be6)\uc740 \uadf8\uc758 \uc544\uba85\uc774\uc790 \ud544\uba85\uc774\uc5c8\ub2e4. \uc5b4\uba38\ub2c8\ub294 \uae30\uc0dd \ucd9c\uc2e0 \uc0b0\uc6d4(\u5c71\u6708)\uc774\ub2e4. \uadf8\ub7ec\ub098 \ud6c4\ub300\uc5d0 \uadf8\ub140\uc758 \uc5b4\uba38\ub2c8\ub294 \uae40\uc778\uc219(\u91d1\u4ec1\u6dd1)\uc774\ub77c\uace0\ub3c4 \ud558\uace0, \uae40\uc778\uc815(\u91d1\u4ec1\u8c9e)\uc774\ub77c\uace0 \uae30\ub85d\ub41c \uacf3\ub3c4 \uc788\ub294\ub370, \uadf8\uc758 \uc544\ubc84\uc9c0\uc5d0\uac8c \ucca9\uc774 \uc5ec\ub7ec \uba85 \uc788\uc5c8\uae30 \ub54c\ubb38\uc5d0 \uc774\ub7ec\ud55c \ud63c\ub780\uc774 \uc0dd\uae34 \uac83\uc774\ub2e4. \uae40\uba85\uc21c\uc774 \ub098\uc911\uc5d0 \uc9c0\uc740 \uc790\uc804\uc801 \uc18c\uc124 <\ud0c4\uc2e4\uc774\uc640 \uc8fc\uc601\uc774>\uc5d0\ub294 \uae30\uc0dd\uc774\uc5c8\ub358 \uadf8\uc758 \uc5b4\uba38\ub2c8 \uc774\ub984\uc774 \uc0b0\uc6d4(\u5c71\u6708)\uc774\ub77c \ud558\uc600\ub2e4. \uc544\ubc84\uc9c0 \uae40\ud76c\uacbd\uc740 \uad6c\ud55c\ub9d0\uc758 \uad00\ub8cc\ub85c \ud3c9\uc548\ub0a8\ub3c4 \ucc38\uc0ac\uad00(\u53c3\u4e8b\u5b98)\uc744 \uc9c0\ub0b4\uae30\ub3c4 \ud588\ub2e4.", "sentence_answer": " \ud0c4\uc2e4 (\u5f48\u5be6)\uc740 \uadf8\uc758 \uc544\uba85\uc774\uc790 \ud544\uba85\uc774\uc5c8\ub2e4.", "paragraph_id": "6481543-2-1", "paragraph_question": "question: \uae40\uba85\uc21c\uc528\uc758 \uc544\uba85\uc774\uc790 \ud544\uba85\uc740?, context: \uae40\uba85\uc21c\uc740 1896\ub144 1\uc6d4 20\uc77c \ud3c9\uc548\ub0a8\ub3c4 \ud3c9\uc591\uad70 \uc735\ub355\uba74\uc5d0\uc11c \ud3c9\uc591\uc758 \uc9c0\uc8fc\uc774\uc790 \ubb38\uc2e0 \uad00\ub8cc\uc600\ub358 \uae40\ud76c\uacbd(\u91d1\u7fb2\u5e9a)\uc758 \uc18c\uc2e4\uc758 \ub538\ub85c \ud0dc\uc5b4\ub0ac\ub2e4. \uadf8\uc758 \ucd9c\uc0dd \uc5f0\ub300\ub294 \ub2e4\uc18c \ubd88\uba85\ud655\ud558\uc5ec 1897\ub144\uc0dd \uc124\ub3c4 \uc788\ub2e4. \ud0c4\uc2e4(\u5f48\u5be6)\uc740 \uadf8\uc758 \uc544\uba85\uc774\uc790 \ud544\uba85\uc774\uc5c8\ub2e4. \uc5b4\uba38\ub2c8\ub294 \uae30\uc0dd \ucd9c\uc2e0 \uc0b0\uc6d4(\u5c71\u6708)\uc774\ub2e4. \uadf8\ub7ec\ub098 \ud6c4\ub300\uc5d0 \uadf8\ub140\uc758 \uc5b4\uba38\ub2c8\ub294 \uae40\uc778\uc219(\u91d1\u4ec1\u6dd1)\uc774\ub77c\uace0\ub3c4 \ud558\uace0, \uae40\uc778\uc815(\u91d1\u4ec1\u8c9e)\uc774\ub77c\uace0 \uae30\ub85d\ub41c \uacf3\ub3c4 \uc788\ub294\ub370, \uadf8\uc758 \uc544\ubc84\uc9c0\uc5d0\uac8c \ucca9\uc774 \uc5ec\ub7ec \uba85 \uc788\uc5c8\uae30 \ub54c\ubb38\uc5d0 \uc774\ub7ec\ud55c \ud63c\ub780\uc774 \uc0dd\uae34 \uac83\uc774\ub2e4. \uae40\uba85\uc21c\uc774 \ub098\uc911\uc5d0 \uc9c0\uc740 \uc790\uc804\uc801 \uc18c\uc124 <\ud0c4\uc2e4\uc774\uc640 \uc8fc\uc601\uc774>\uc5d0\ub294 \uae30\uc0dd\uc774\uc5c8\ub358 \uadf8\uc758 \uc5b4\uba38\ub2c8 \uc774\ub984\uc774 \uc0b0\uc6d4(\u5c71\u6708)\uc774\ub77c \ud558\uc600\ub2e4. \uc544\ubc84\uc9c0 \uae40\ud76c\uacbd\uc740 \uad6c\ud55c\ub9d0\uc758 \uad00\ub8cc\ub85c \ud3c9\uc548\ub0a8\ub3c4 \ucc38\uc0ac\uad00(\u53c3\u4e8b\u5b98)\uc744 \uc9c0\ub0b4\uae30\ub3c4 \ud588\ub2e4."} {"question": "\uc11c\ud0dc\ud6c4\uc758 \uc874\ud638\ub97c \ub534 \uc2e0\uc815\uc758 \uba85\uce6d\uc740?", "paragraph": "\ubcc0\ubc95\uc790\uac15\uc6b4\ub3d9\uc740 \uc11c\ud0dc\ud6c4\uc640 \uc218\uad6c\ud30c \ud0d3\uc5d0 \uc2e4\ud328\ud588\uc73c\ub098 \uc758\ud654\ub2e8 \uc6b4\ub3d9\uc758 \ucc38\ud639\ud55c \uacb0\uacfc\ub85c \uad6d\ub0b4\uc0c1\uc73c\ub85c\ub294 \ub9c9\ub300\ud55c \uc138\uae08 \uc9d5\uc218\ub85c \ubbfc\uc2ec\uc758 \ub3d9\uc694\uc640 \ubd88\ub9cc, \uad6d\uc678\uc0c1\uc73c\ub85c\ub294 \uc5f4\uac15\uc758 \uc774\uad8c \uce68\ud0c8 \uc2ec\ud654\uac00 \uc99d\ud3ed\ud558\uc5ec \uccad \uc870\uc815\uc740 \ub0b4\uc815\uacfc \uc678\uad50 \ucd1d\uccb4\uac00 \uc704\uae30\uc5d0 \ucc98\ud558\uc600\ub2e4. \ud2b9\ud788, \uc0c1\uce35\ub3c4 \uccad\uc870\uc5d0 \ubd88\ub9cc\uc774\uc5b4\uc11c \ub354\ub294 \ubc29\uce58\ud560 \uc218 \uc5c6\ub294 \uc704\uae30\uc600\ub2e4. 1901\ub144 1\uc6d4 \uccad\uc758 \uc2e4\uad8c\uc790 \uc11c\ud0dc\ud6c4\ub294 \u201c\uc678\uad6d\uc758 \uc7a5\uc810\uc744 \ucde8\ud558\uace0 \uc911\uad6d\uc758 \ub2e8\uc810\uc744 \ubc84\ub824 \ubd80\uac15\uc744 \uaf80\ud55c\ub2e4\u201d\ub294 \ubc29\uce68\ud558\uc5d0 \uace0\uc704 \uad00\ub8cc\ub4e4\uc5d0\uac8c \uad6d\uc815\uc5d0 \uad00\ud55c \uc758\uacac\uc744 \u4e0a\u7533 (\uc0c1\uc2e0)\ud558\ub77c\uace0 \uba85\ud588\ub2e4. \uc774\ub97c \uae30\uc810\uc73c\ub85c \ud558\uc5ec \uccad \uc815\ubd80\ub294 \uc0dd\uc874\ud558\ub824\ub294 \u2018\u65b0\u653f (\uc2e0\uc815)\u2019\uc744 \uac1c\uc2dc\ud588\ub294\ub370 \uc774\ub97c \uc11c\ud0dc\ud6c4\uc758 \uc874\ud638\uc778 \u6148\u79a7 (\uc790\ud76c)\ub97c \ub530 \u2018\u6148\u79a7\u65b0\u653f (\uc790\ud76c\uc2e0\uc815)\u2019 \uc774\ub77c\uace0 \ud55c\ub2e4. \uc9c1\ub840\ucd1d\ub3c5 \uc704\uc548\uc2a4\uce74\uc774, \uc591\uac15\ucd1d\ub3c5 \uc720\uace4\uc77c, \ud638\uad11\ucd1d\ub3c5 \uc7a5\uc9c0\ub3d9\uc774 \ucd94\uc9c4\ud55c \uc2e0\uc815\uc758 \uc8fc \ub0b4\uc6a9\uc740 \ub2e4\uc74c\uacfc \uac19\ub2e4.", "answer": "\uc790\ud76c\uc2e0\uc815", "sentence": "\uc774\ub97c \uae30\uc810\uc73c\ub85c \ud558\uc5ec \uccad \uc815\ubd80\ub294 \uc0dd\uc874\ud558\ub824\ub294 \u2018\u65b0\u653f (\uc2e0\uc815)\u2019\uc744 \uac1c\uc2dc\ud588\ub294\ub370 \uc774\ub97c \uc11c\ud0dc\ud6c4\uc758 \uc874\ud638\uc778 \u6148\u79a7 (\uc790\ud76c)\ub97c \ub530 \u2018\u6148\u79a7\u65b0\u653f ( \uc790\ud76c\uc2e0\uc815 )", "paragraph_sentence": "\ubcc0\ubc95\uc790\uac15\uc6b4\ub3d9\uc740 \uc11c\ud0dc\ud6c4\uc640 \uc218\uad6c\ud30c \ud0d3\uc5d0 \uc2e4\ud328\ud588\uc73c\ub098 \uc758\ud654\ub2e8 \uc6b4\ub3d9\uc758 \ucc38\ud639\ud55c \uacb0\uacfc\ub85c \uad6d\ub0b4\uc0c1\uc73c\ub85c\ub294 \ub9c9\ub300\ud55c \uc138\uae08 \uc9d5\uc218\ub85c \ubbfc\uc2ec\uc758 \ub3d9\uc694\uc640 \ubd88\ub9cc, \uad6d\uc678\uc0c1\uc73c\ub85c\ub294 \uc5f4\uac15\uc758 \uc774\uad8c \uce68\ud0c8 \uc2ec\ud654\uac00 \uc99d\ud3ed\ud558\uc5ec \uccad \uc870\uc815\uc740 \ub0b4\uc815\uacfc \uc678\uad50 \ucd1d\uccb4\uac00 \uc704\uae30\uc5d0 \ucc98\ud558\uc600\ub2e4. \ud2b9\ud788, \uc0c1\uce35\ub3c4 \uccad\uc870\uc5d0 \ubd88\ub9cc\uc774\uc5b4\uc11c \ub354\ub294 \ubc29\uce58\ud560 \uc218 \uc5c6\ub294 \uc704\uae30\uc600\ub2e4. 1901\ub144 1\uc6d4 \uccad\uc758 \uc2e4\uad8c\uc790 \uc11c\ud0dc\ud6c4\ub294 \u201c\uc678\uad6d\uc758 \uc7a5\uc810\uc744 \ucde8\ud558\uace0 \uc911\uad6d\uc758 \ub2e8\uc810\uc744 \ubc84\ub824 \ubd80\uac15\uc744 \uaf80\ud55c\ub2e4\u201d\ub294 \ubc29\uce68\ud558\uc5d0 \uace0\uc704 \uad00\ub8cc\ub4e4\uc5d0\uac8c \uad6d\uc815\uc5d0 \uad00\ud55c \uc758\uacac\uc744 \u4e0a\u7533 (\uc0c1\uc2e0)\ud558\ub77c\uace0 \uba85\ud588\ub2e4. \uc774\ub97c \uae30\uc810\uc73c\ub85c \ud558\uc5ec \uccad \uc815\ubd80\ub294 \uc0dd\uc874\ud558\ub824\ub294 \u2018\u65b0\u653f (\uc2e0\uc815)\u2019\uc744 \uac1c\uc2dc\ud588\ub294\ub370 \uc774\ub97c \uc11c\ud0dc\ud6c4\uc758 \uc874\ud638\uc778 \u6148\u79a7 (\uc790\ud76c)\ub97c \ub530 \u2018\u6148\u79a7\u65b0\u653f ( \uc790\ud76c\uc2e0\uc815 ) \u2019 \uc774\ub77c\uace0 \ud55c\ub2e4. \uc9c1\ub840\ucd1d\ub3c5 \uc704\uc548\uc2a4\uce74\uc774, \uc591\uac15\ucd1d\ub3c5 \uc720\uace4\uc77c, \ud638\uad11\ucd1d\ub3c5 \uc7a5\uc9c0\ub3d9\uc774 \ucd94\uc9c4\ud55c \uc2e0\uc815\uc758 \uc8fc \ub0b4\uc6a9\uc740 \ub2e4\uc74c\uacfc \uac19\ub2e4.", "paragraph_answer": "\ubcc0\ubc95\uc790\uac15\uc6b4\ub3d9\uc740 \uc11c\ud0dc\ud6c4\uc640 \uc218\uad6c\ud30c \ud0d3\uc5d0 \uc2e4\ud328\ud588\uc73c\ub098 \uc758\ud654\ub2e8 \uc6b4\ub3d9\uc758 \ucc38\ud639\ud55c \uacb0\uacfc\ub85c \uad6d\ub0b4\uc0c1\uc73c\ub85c\ub294 \ub9c9\ub300\ud55c \uc138\uae08 \uc9d5\uc218\ub85c \ubbfc\uc2ec\uc758 \ub3d9\uc694\uc640 \ubd88\ub9cc, \uad6d\uc678\uc0c1\uc73c\ub85c\ub294 \uc5f4\uac15\uc758 \uc774\uad8c \uce68\ud0c8 \uc2ec\ud654\uac00 \uc99d\ud3ed\ud558\uc5ec \uccad \uc870\uc815\uc740 \ub0b4\uc815\uacfc \uc678\uad50 \ucd1d\uccb4\uac00 \uc704\uae30\uc5d0 \ucc98\ud558\uc600\ub2e4. \ud2b9\ud788, \uc0c1\uce35\ub3c4 \uccad\uc870\uc5d0 \ubd88\ub9cc\uc774\uc5b4\uc11c \ub354\ub294 \ubc29\uce58\ud560 \uc218 \uc5c6\ub294 \uc704\uae30\uc600\ub2e4. 1901\ub144 1\uc6d4 \uccad\uc758 \uc2e4\uad8c\uc790 \uc11c\ud0dc\ud6c4\ub294 \u201c\uc678\uad6d\uc758 \uc7a5\uc810\uc744 \ucde8\ud558\uace0 \uc911\uad6d\uc758 \ub2e8\uc810\uc744 \ubc84\ub824 \ubd80\uac15\uc744 \uaf80\ud55c\ub2e4\u201d\ub294 \ubc29\uce68\ud558\uc5d0 \uace0\uc704 \uad00\ub8cc\ub4e4\uc5d0\uac8c \uad6d\uc815\uc5d0 \uad00\ud55c \uc758\uacac\uc744 \u4e0a\u7533 (\uc0c1\uc2e0)\ud558\ub77c\uace0 \uba85\ud588\ub2e4. \uc774\ub97c \uae30\uc810\uc73c\ub85c \ud558\uc5ec \uccad \uc815\ubd80\ub294 \uc0dd\uc874\ud558\ub824\ub294 \u2018\u65b0\u653f (\uc2e0\uc815)\u2019\uc744 \uac1c\uc2dc\ud588\ub294\ub370 \uc774\ub97c \uc11c\ud0dc\ud6c4\uc758 \uc874\ud638\uc778 \u6148\u79a7 (\uc790\ud76c)\ub97c \ub530 \u2018\u6148\u79a7\u65b0\u653f ( \uc790\ud76c\uc2e0\uc815 )\u2019 \uc774\ub77c\uace0 \ud55c\ub2e4. \uc9c1\ub840\ucd1d\ub3c5 \uc704\uc548\uc2a4\uce74\uc774, \uc591\uac15\ucd1d\ub3c5 \uc720\uace4\uc77c, \ud638\uad11\ucd1d\ub3c5 \uc7a5\uc9c0\ub3d9\uc774 \ucd94\uc9c4\ud55c \uc2e0\uc815\uc758 \uc8fc \ub0b4\uc6a9\uc740 \ub2e4\uc74c\uacfc \uac19\ub2e4.", "sentence_answer": "\uc774\ub97c \uae30\uc810\uc73c\ub85c \ud558\uc5ec \uccad \uc815\ubd80\ub294 \uc0dd\uc874\ud558\ub824\ub294 \u2018\u65b0\u653f (\uc2e0\uc815)\u2019\uc744 \uac1c\uc2dc\ud588\ub294\ub370 \uc774\ub97c \uc11c\ud0dc\ud6c4\uc758 \uc874\ud638\uc778 \u6148\u79a7 (\uc790\ud76c)\ub97c \ub530 \u2018\u6148\u79a7\u65b0\u653f ( \uc790\ud76c\uc2e0\uc815 )", "paragraph_id": "6457494-4-1", "paragraph_question": "question: \uc11c\ud0dc\ud6c4\uc758 \uc874\ud638\ub97c \ub534 \uc2e0\uc815\uc758 \uba85\uce6d\uc740?, context: \ubcc0\ubc95\uc790\uac15\uc6b4\ub3d9\uc740 \uc11c\ud0dc\ud6c4\uc640 \uc218\uad6c\ud30c \ud0d3\uc5d0 \uc2e4\ud328\ud588\uc73c\ub098 \uc758\ud654\ub2e8 \uc6b4\ub3d9\uc758 \ucc38\ud639\ud55c \uacb0\uacfc\ub85c \uad6d\ub0b4\uc0c1\uc73c\ub85c\ub294 \ub9c9\ub300\ud55c \uc138\uae08 \uc9d5\uc218\ub85c \ubbfc\uc2ec\uc758 \ub3d9\uc694\uc640 \ubd88\ub9cc, \uad6d\uc678\uc0c1\uc73c\ub85c\ub294 \uc5f4\uac15\uc758 \uc774\uad8c \uce68\ud0c8 \uc2ec\ud654\uac00 \uc99d\ud3ed\ud558\uc5ec \uccad \uc870\uc815\uc740 \ub0b4\uc815\uacfc \uc678\uad50 \ucd1d\uccb4\uac00 \uc704\uae30\uc5d0 \ucc98\ud558\uc600\ub2e4. \ud2b9\ud788, \uc0c1\uce35\ub3c4 \uccad\uc870\uc5d0 \ubd88\ub9cc\uc774\uc5b4\uc11c \ub354\ub294 \ubc29\uce58\ud560 \uc218 \uc5c6\ub294 \uc704\uae30\uc600\ub2e4. 1901\ub144 1\uc6d4 \uccad\uc758 \uc2e4\uad8c\uc790 \uc11c\ud0dc\ud6c4\ub294 \u201c\uc678\uad6d\uc758 \uc7a5\uc810\uc744 \ucde8\ud558\uace0 \uc911\uad6d\uc758 \ub2e8\uc810\uc744 \ubc84\ub824 \ubd80\uac15\uc744 \uaf80\ud55c\ub2e4\u201d\ub294 \ubc29\uce68\ud558\uc5d0 \uace0\uc704 \uad00\ub8cc\ub4e4\uc5d0\uac8c \uad6d\uc815\uc5d0 \uad00\ud55c \uc758\uacac\uc744 \u4e0a\u7533 (\uc0c1\uc2e0)\ud558\ub77c\uace0 \uba85\ud588\ub2e4. \uc774\ub97c \uae30\uc810\uc73c\ub85c \ud558\uc5ec \uccad \uc815\ubd80\ub294 \uc0dd\uc874\ud558\ub824\ub294 \u2018\u65b0\u653f (\uc2e0\uc815)\u2019\uc744 \uac1c\uc2dc\ud588\ub294\ub370 \uc774\ub97c \uc11c\ud0dc\ud6c4\uc758 \uc874\ud638\uc778 \u6148\u79a7 (\uc790\ud76c)\ub97c \ub530 \u2018\u6148\u79a7\u65b0\u653f (\uc790\ud76c\uc2e0\uc815)\u2019 \uc774\ub77c\uace0 \ud55c\ub2e4. \uc9c1\ub840\ucd1d\ub3c5 \uc704\uc548\uc2a4\uce74\uc774, \uc591\uac15\ucd1d\ub3c5 \uc720\uace4\uc77c, \ud638\uad11\ucd1d\ub3c5 \uc7a5\uc9c0\ub3d9\uc774 \ucd94\uc9c4\ud55c \uc2e0\uc815\uc758 \uc8fc \ub0b4\uc6a9\uc740 \ub2e4\uc74c\uacfc \uac19\ub2e4."} {"question": "\uc601\ud654\uc5d0\uc11c \uc2a4\uc2a4\ub85c \ubc29\ud328\ub9c9\uc774\uac00 \ub3fc \uc678\uacc4\uc778 \uac00\ub9ac\ub9c8\uc2a4\uc758 \uacf5\uaca9\uc744 \ub9c9\uc740 \uc778\ubb3c\uc740?", "paragraph": "\ub54c\ub294 \uac00\ubbf8\ub77c\uc2a4\ub77c\uace0 \ubd88\ub9ac\ub294 \uc678\uacc4\uc778\ub4e4\uc774 \uc9c0\uad6c\ub97c \uacf5\uaca9\ud558\uae30 \uc2dc\uc791\ud55c\uc9c0 5\ub144\uc774 \uc9c0\ub09c 2199\ub144, \uc9c0\uad6c\ub97c \uc9c0\ud0a4\ub294 \uc9c0\uad6c \ubc29\uc704\uad70 (Earth Defense Force, EDF)\ub294 \ubd88\ub9ac\ud55c \uc804\uc138\ub97c \ub4a4\uc9d1\uae30 \uc704\ud574 \ud654\uc131 \uac00\uae4c\uc774\ub85c \uacf5\uc138\uc774\uc804\uc5d0 \ucc29\uc218\ud55c\ub2e4. \uadf8\ub7ec\ub098 \uc778\uac04\uc758 \ubb34\uae30\ub85c\ub294 \uc804\ud600 \uc678\uacc4\uc778 \uac00\ub9ac\ub9c8\uc2a4\uc758 \uc0c1\ub300\uac00 \ub418\uc9c0 \ubabb\ud588\uace0, EDF\uc758 \ud568\ub300\ub4e4\uc740 \uc2ec\uac01\ud55c \ud53c\ud574\ub97c \uc785\uac8c \ub418\uc5b4 \uc808\uccb4\uc808\uba85\uc758 \uc704\uae30\uc5d0 \ucc98\ud558\uac8c \ub41c\ub2e4. \uc774 \ub54c '\uc720\ud0a4\uce74\uc81c'\ub77c\ub294 \uc774\ub984\uc758 \uad6c\ucd95\ud568\uc744 \uc9c0\ud718\ud558\ub358 \ud568\uc7a5 \ucf54\ub2e4\uc774 \ub9c8\ubaa8\ub8e8\uac00 \uc2a4\uc2a4\ub85c \ubc29\ud328\ub85c \ub098\uc11c\uc11c \uac00\ubbf8\ub77c\uc2a4\uc758 \uacf5\uaca9\uc744 \ub9c9\uc544\uc8fc\uc5c8\uace0, \uadf8 \uc0ac\uc774\uc5d0 EDF \uc624\ud0a4\ub2e4 \uc120\uc7a5\uc758 \ud568\ub300\ub4e4\uc740 \ubb34\uc0ac\ud788 \uc704\uae30\uc5d0\uc11c \ube60\uc838\ub098\uc62c \uc218 \uc788\uc5c8\ub2e4. \uadf8\ub7ec\ub098 \uacc4\uc18d \uac00\ubbf8\ub77c\uc2a4\uc758 \uacf5\uaca9\uc744 \ubc1b\uc740 \ub9c8\ubaa8\ub8e8\uc758 \uad6c\ucd95\ud568\uc740 \uacb0\uad6d \ud30c\uad34\ub418\uace0 \ub9cc\ub2e4.", "answer": "\ucf54\ub2e4\uc774 \ub9c8\ubaa8\ub8e8", "sentence": "\uc774 \ub54c '\uc720\ud0a4\uce74\uc81c'\ub77c\ub294 \uc774\ub984\uc758 \uad6c\ucd95\ud568\uc744 \uc9c0\ud718\ud558\ub358 \ud568\uc7a5 \ucf54\ub2e4\uc774 \ub9c8\ubaa8\ub8e8 \uac00 \uc2a4\uc2a4\ub85c \ubc29\ud328\ub85c \ub098\uc11c\uc11c \uac00\ubbf8\ub77c\uc2a4\uc758 \uacf5\uaca9\uc744 \ub9c9\uc544\uc8fc\uc5c8\uace0, \uadf8 \uc0ac\uc774\uc5d0 EDF \uc624\ud0a4\ub2e4 \uc120\uc7a5\uc758 \ud568\ub300\ub4e4\uc740 \ubb34\uc0ac\ud788 \uc704\uae30\uc5d0\uc11c \ube60\uc838\ub098\uc62c \uc218 \uc788\uc5c8\ub2e4.", "paragraph_sentence": "\ub54c\ub294 \uac00\ubbf8\ub77c\uc2a4\ub77c\uace0 \ubd88\ub9ac\ub294 \uc678\uacc4\uc778\ub4e4\uc774 \uc9c0\uad6c\ub97c \uacf5\uaca9\ud558\uae30 \uc2dc\uc791\ud55c\uc9c0 5\ub144\uc774 \uc9c0\ub09c 2199\ub144, \uc9c0\uad6c\ub97c \uc9c0\ud0a4\ub294 \uc9c0\uad6c \ubc29\uc704\uad70 (Earth Defense Force, EDF)\ub294 \ubd88\ub9ac\ud55c \uc804\uc138\ub97c \ub4a4\uc9d1\uae30 \uc704\ud574 \ud654\uc131 \uac00\uae4c\uc774\ub85c \uacf5\uc138\uc774\uc804\uc5d0 \ucc29\uc218\ud55c\ub2e4. \uadf8\ub7ec\ub098 \uc778\uac04\uc758 \ubb34\uae30\ub85c\ub294 \uc804\ud600 \uc678\uacc4\uc778 \uac00\ub9ac\ub9c8\uc2a4\uc758 \uc0c1\ub300\uac00 \ub418\uc9c0 \ubabb\ud588\uace0, EDF\uc758 \ud568\ub300\ub4e4\uc740 \uc2ec\uac01\ud55c \ud53c\ud574\ub97c \uc785\uac8c \ub418\uc5b4 \uc808\uccb4\uc808\uba85\uc758 \uc704\uae30\uc5d0 \ucc98\ud558\uac8c \ub41c\ub2e4. \uc774 \ub54c '\uc720\ud0a4\uce74\uc81c'\ub77c\ub294 \uc774\ub984\uc758 \uad6c\ucd95\ud568\uc744 \uc9c0\ud718\ud558\ub358 \ud568\uc7a5 \ucf54\ub2e4\uc774 \ub9c8\ubaa8\ub8e8 \uac00 \uc2a4\uc2a4\ub85c \ubc29\ud328\ub85c \ub098\uc11c\uc11c \uac00\ubbf8\ub77c\uc2a4\uc758 \uacf5\uaca9\uc744 \ub9c9\uc544\uc8fc\uc5c8\uace0, \uadf8 \uc0ac\uc774\uc5d0 EDF \uc624\ud0a4\ub2e4 \uc120\uc7a5\uc758 \ud568\ub300\ub4e4\uc740 \ubb34\uc0ac\ud788 \uc704\uae30\uc5d0\uc11c \ube60\uc838\ub098\uc62c \uc218 \uc788\uc5c8\ub2e4. \uadf8\ub7ec\ub098 \uacc4\uc18d \uac00\ubbf8\ub77c\uc2a4\uc758 \uacf5\uaca9\uc744 \ubc1b\uc740 \ub9c8\ubaa8\ub8e8\uc758 \uad6c\ucd95\ud568\uc740 \uacb0\uad6d \ud30c\uad34\ub418\uace0 \ub9cc\ub2e4.", "paragraph_answer": "\ub54c\ub294 \uac00\ubbf8\ub77c\uc2a4\ub77c\uace0 \ubd88\ub9ac\ub294 \uc678\uacc4\uc778\ub4e4\uc774 \uc9c0\uad6c\ub97c \uacf5\uaca9\ud558\uae30 \uc2dc\uc791\ud55c\uc9c0 5\ub144\uc774 \uc9c0\ub09c 2199\ub144, \uc9c0\uad6c\ub97c \uc9c0\ud0a4\ub294 \uc9c0\uad6c \ubc29\uc704\uad70 (Earth Defense Force, EDF)\ub294 \ubd88\ub9ac\ud55c \uc804\uc138\ub97c \ub4a4\uc9d1\uae30 \uc704\ud574 \ud654\uc131 \uac00\uae4c\uc774\ub85c \uacf5\uc138\uc774\uc804\uc5d0 \ucc29\uc218\ud55c\ub2e4. \uadf8\ub7ec\ub098 \uc778\uac04\uc758 \ubb34\uae30\ub85c\ub294 \uc804\ud600 \uc678\uacc4\uc778 \uac00\ub9ac\ub9c8\uc2a4\uc758 \uc0c1\ub300\uac00 \ub418\uc9c0 \ubabb\ud588\uace0, EDF\uc758 \ud568\ub300\ub4e4\uc740 \uc2ec\uac01\ud55c \ud53c\ud574\ub97c \uc785\uac8c \ub418\uc5b4 \uc808\uccb4\uc808\uba85\uc758 \uc704\uae30\uc5d0 \ucc98\ud558\uac8c \ub41c\ub2e4. \uc774 \ub54c '\uc720\ud0a4\uce74\uc81c'\ub77c\ub294 \uc774\ub984\uc758 \uad6c\ucd95\ud568\uc744 \uc9c0\ud718\ud558\ub358 \ud568\uc7a5 \ucf54\ub2e4\uc774 \ub9c8\ubaa8\ub8e8 \uac00 \uc2a4\uc2a4\ub85c \ubc29\ud328\ub85c \ub098\uc11c\uc11c \uac00\ubbf8\ub77c\uc2a4\uc758 \uacf5\uaca9\uc744 \ub9c9\uc544\uc8fc\uc5c8\uace0, \uadf8 \uc0ac\uc774\uc5d0 EDF \uc624\ud0a4\ub2e4 \uc120\uc7a5\uc758 \ud568\ub300\ub4e4\uc740 \ubb34\uc0ac\ud788 \uc704\uae30\uc5d0\uc11c \ube60\uc838\ub098\uc62c \uc218 \uc788\uc5c8\ub2e4. \uadf8\ub7ec\ub098 \uacc4\uc18d \uac00\ubbf8\ub77c\uc2a4\uc758 \uacf5\uaca9\uc744 \ubc1b\uc740 \ub9c8\ubaa8\ub8e8\uc758 \uad6c\ucd95\ud568\uc740 \uacb0\uad6d \ud30c\uad34\ub418\uace0 \ub9cc\ub2e4.", "sentence_answer": "\uc774 \ub54c '\uc720\ud0a4\uce74\uc81c'\ub77c\ub294 \uc774\ub984\uc758 \uad6c\ucd95\ud568\uc744 \uc9c0\ud718\ud558\ub358 \ud568\uc7a5 \ucf54\ub2e4\uc774 \ub9c8\ubaa8\ub8e8 \uac00 \uc2a4\uc2a4\ub85c \ubc29\ud328\ub85c \ub098\uc11c\uc11c \uac00\ubbf8\ub77c\uc2a4\uc758 \uacf5\uaca9\uc744 \ub9c9\uc544\uc8fc\uc5c8\uace0, \uadf8 \uc0ac\uc774\uc5d0 EDF \uc624\ud0a4\ub2e4 \uc120\uc7a5\uc758 \ud568\ub300\ub4e4\uc740 \ubb34\uc0ac\ud788 \uc704\uae30\uc5d0\uc11c \ube60\uc838\ub098\uc62c \uc218 \uc788\uc5c8\ub2e4.", "paragraph_id": "6526330-1-0", "paragraph_question": "question: \uc601\ud654\uc5d0\uc11c \uc2a4\uc2a4\ub85c \ubc29\ud328\ub9c9\uc774\uac00 \ub3fc \uc678\uacc4\uc778 \uac00\ub9ac\ub9c8\uc2a4\uc758 \uacf5\uaca9\uc744 \ub9c9\uc740 \uc778\ubb3c\uc740?, context: \ub54c\ub294 \uac00\ubbf8\ub77c\uc2a4\ub77c\uace0 \ubd88\ub9ac\ub294 \uc678\uacc4\uc778\ub4e4\uc774 \uc9c0\uad6c\ub97c \uacf5\uaca9\ud558\uae30 \uc2dc\uc791\ud55c\uc9c0 5\ub144\uc774 \uc9c0\ub09c 2199\ub144, \uc9c0\uad6c\ub97c \uc9c0\ud0a4\ub294 \uc9c0\uad6c \ubc29\uc704\uad70 (Earth Defense Force, EDF)\ub294 \ubd88\ub9ac\ud55c \uc804\uc138\ub97c \ub4a4\uc9d1\uae30 \uc704\ud574 \ud654\uc131 \uac00\uae4c\uc774\ub85c \uacf5\uc138\uc774\uc804\uc5d0 \ucc29\uc218\ud55c\ub2e4. \uadf8\ub7ec\ub098 \uc778\uac04\uc758 \ubb34\uae30\ub85c\ub294 \uc804\ud600 \uc678\uacc4\uc778 \uac00\ub9ac\ub9c8\uc2a4\uc758 \uc0c1\ub300\uac00 \ub418\uc9c0 \ubabb\ud588\uace0, EDF\uc758 \ud568\ub300\ub4e4\uc740 \uc2ec\uac01\ud55c \ud53c\ud574\ub97c \uc785\uac8c \ub418\uc5b4 \uc808\uccb4\uc808\uba85\uc758 \uc704\uae30\uc5d0 \ucc98\ud558\uac8c \ub41c\ub2e4. \uc774 \ub54c '\uc720\ud0a4\uce74\uc81c'\ub77c\ub294 \uc774\ub984\uc758 \uad6c\ucd95\ud568\uc744 \uc9c0\ud718\ud558\ub358 \ud568\uc7a5 \ucf54\ub2e4\uc774 \ub9c8\ubaa8\ub8e8\uac00 \uc2a4\uc2a4\ub85c \ubc29\ud328\ub85c \ub098\uc11c\uc11c \uac00\ubbf8\ub77c\uc2a4\uc758 \uacf5\uaca9\uc744 \ub9c9\uc544\uc8fc\uc5c8\uace0, \uadf8 \uc0ac\uc774\uc5d0 EDF \uc624\ud0a4\ub2e4 \uc120\uc7a5\uc758 \ud568\ub300\ub4e4\uc740 \ubb34\uc0ac\ud788 \uc704\uae30\uc5d0\uc11c \ube60\uc838\ub098\uc62c \uc218 \uc788\uc5c8\ub2e4. \uadf8\ub7ec\ub098 \uacc4\uc18d \uac00\ubbf8\ub77c\uc2a4\uc758 \uacf5\uaca9\uc744 \ubc1b\uc740 \ub9c8\ubaa8\ub8e8\uc758 \uad6c\ucd95\ud568\uc740 \uacb0\uad6d \ud30c\uad34\ub418\uace0 \ub9cc\ub2e4."} {"question": "\uc810\uc131\uc220 \ubb38\ud5cc \uc911 \ud558\ub098\ub85c \uc54c\ube44\ub8e8\ub2c8\uc758 \uc800\uc11c\uc758 \uc774\ub984\uc740 \ubb34\uc5c7\uc778\uac00?", "paragraph": "\uc2b9\uad50\uc810\uc740 \uac8c\uc790\ub9ac\uc758 3\ub3c4\uc5d0\uc11c \uac15\uad50\uc810\uc740 \uc0ac\uc218\uc790\ub9ac\uc758 3\ub3c4\uc5d0\uc11c \uace0\uc591\uc9c0\uc704\ub97c \uac16\ub294\ub2e4. \uadf8 \uc704\uce58\ub4e4\uc740 11\uc138\uae30 \uc54c\ube44\ub8e8\ub2c8\uc758 \uc800\uc11c \u300a\uc810\uc131\uc608\uc220\uc758 \uc694\uc18c\uc5d0 \ub300\ud55c \uc18c\uac1c\uc11c(Book of Instruction in the elements of the art of astrology)\u300b\uc640 \uac19\uc740 \uc911\uc138 \ucd08\uae30 \uc544\ub78d \uc2dc\ub300\uc758 \uc810\uc131\uc220 \ubb38\ud5cc\ub4e4\uc5d0 \uc218\ub85d\ub418\uc5b4 \uc788\ub2e4. \ud604\ub300 \ubca0\ub2e4 \uc810\uc131\uac00\ub4e4\uc740 \uad50\uc810\uc758 \uace0\uc591 \uc9c0\uc704\uc5d0 \uc911\uc694\uc131\uc744 \ubd80\uc5ec\ud558\ub294 \ubc18\uba74, \uc911\uc138 \uc720\ub7fd\uc744 \ud1b5\ud574 \uc804\ud574\uc9c4 \uc11c\uc591 \uc810\uc131\uc220\uc758 \uc804\ud1b5\uc740 \uadf8 \uc2e4\ucc9c\uc5d0 \uc788\uc5b4\uc11c \uc804\ud1b5\uc801\uc73c\ub85c \uadf8\ub9ac\uace0 \ud604\uc7ac\uc5d0 \uadf8\uac83\ub4e4\uc744 \uc57d\uac04\ub9cc \uc0ac\uc6a9\ud55c\ub2e4. \uadf8\ub9ac\uc2a4\uc778\ub4e4\uacfc \ud398\ub974\uc2dc\uc544\uc778\ub4e4\uacfc\ub294 \ubc18\ub300\ub85c \uc54c\ube44\ub8e8\ub2c8\ub294 \uadf8 \uc810\ub4e4\uc758 \uace0\uc591\uc5d0 \ub300\ud574 \uc54c\ub9ac\uba70, \uadf8 \uc6d0\ub9ac\ub97c \"\uc544\uc8fc \uc801\uc808\ud55c\" \uac83\uc73c\ub85c \ubb18\uc0ac\ud588\ub294\ub370 \ubc18\ud574, \uadf8\uc758 \uc2dc\ub300\uc758 \ud78c\ub450 \uc810\uc131\uac00\ub4e4\uc740 \ud0dc\uc591\uacfc \ubaa9\uc131 \uadf8\ub9ac\uace0 \ud1a0\uc131\uc758 \uace0\uc591 \uc704\uce58\uc5d0 \uc77c\uce58\uac00 \uc5c6\uc5c8\uc73c\uba70, \uad50\uc810\uc758 \uace0\uc591\uc740 \uc778\uc815\ud558\uc9c0 \uc54a\uc558\ub2e4.", "answer": "\uc810\uc131\uc608\uc220\uc758 \uc694\uc18c\uc5d0 \ub300\ud55c \uc18c\uac1c\uc11c", "sentence": "\uadf8 \uc704\uce58\ub4e4\uc740 11\uc138\uae30 \uc54c\ube44\ub8e8\ub2c8\uc758 \uc800\uc11c \u300a \uc810\uc131\uc608\uc220\uc758 \uc694\uc18c\uc5d0 \ub300\ud55c \uc18c\uac1c\uc11c (Book of Instruction in the elements of the art of astrology)\u300b\uc640 \uac19\uc740 \uc911\uc138 \ucd08\uae30 \uc544\ub78d \uc2dc\ub300\uc758 \uc810\uc131\uc220 \ubb38\ud5cc\ub4e4\uc5d0 \uc218\ub85d\ub418\uc5b4 \uc788\ub2e4.", "paragraph_sentence": "\uc2b9\uad50\uc810\uc740 \uac8c\uc790\ub9ac\uc758 3\ub3c4\uc5d0\uc11c \uac15\uad50\uc810\uc740 \uc0ac\uc218\uc790\ub9ac\uc758 3\ub3c4\uc5d0\uc11c \uace0\uc591\uc9c0\uc704\ub97c \uac16\ub294\ub2e4. \uadf8 \uc704\uce58\ub4e4\uc740 11\uc138\uae30 \uc54c\ube44\ub8e8\ub2c8\uc758 \uc800\uc11c \u300a \uc810\uc131\uc608\uc220\uc758 \uc694\uc18c\uc5d0 \ub300\ud55c \uc18c\uac1c\uc11c (Book of Instruction in the elements of the art of astrology)\u300b\uc640 \uac19\uc740 \uc911\uc138 \ucd08\uae30 \uc544\ub78d \uc2dc\ub300\uc758 \uc810\uc131\uc220 \ubb38\ud5cc\ub4e4\uc5d0 \uc218\ub85d\ub418\uc5b4 \uc788\ub2e4. \ud604\ub300 \ubca0\ub2e4 \uc810\uc131\uac00\ub4e4\uc740 \uad50\uc810\uc758 \uace0\uc591 \uc9c0\uc704\uc5d0 \uc911\uc694\uc131\uc744 \ubd80\uc5ec\ud558\ub294 \ubc18\uba74, \uc911\uc138 \uc720\ub7fd\uc744 \ud1b5\ud574 \uc804\ud574\uc9c4 \uc11c\uc591 \uc810\uc131\uc220\uc758 \uc804\ud1b5\uc740 \uadf8 \uc2e4\ucc9c\uc5d0 \uc788\uc5b4\uc11c \uc804\ud1b5\uc801\uc73c\ub85c \uadf8\ub9ac\uace0 \ud604\uc7ac\uc5d0 \uadf8\uac83\ub4e4\uc744 \uc57d\uac04\ub9cc \uc0ac\uc6a9\ud55c\ub2e4. \uadf8\ub9ac\uc2a4\uc778\ub4e4\uacfc \ud398\ub974\uc2dc\uc544\uc778\ub4e4\uacfc\ub294 \ubc18\ub300\ub85c \uc54c\ube44\ub8e8\ub2c8\ub294 \uadf8 \uc810\ub4e4\uc758 \uace0\uc591\uc5d0 \ub300\ud574 \uc54c\ub9ac\uba70, \uadf8 \uc6d0\ub9ac\ub97c \"\uc544\uc8fc \uc801\uc808\ud55c\" \uac83\uc73c\ub85c \ubb18\uc0ac\ud588\ub294\ub370 \ubc18\ud574, \uadf8\uc758 \uc2dc\ub300\uc758 \ud78c\ub450 \uc810\uc131\uac00\ub4e4\uc740 \ud0dc\uc591\uacfc \ubaa9\uc131 \uadf8\ub9ac\uace0 \ud1a0\uc131\uc758 \uace0\uc591 \uc704\uce58\uc5d0 \uc77c\uce58\uac00 \uc5c6\uc5c8\uc73c\uba70, \uad50\uc810\uc758 \uace0\uc591\uc740 \uc778\uc815\ud558\uc9c0 \uc54a\uc558\ub2e4.", "paragraph_answer": "\uc2b9\uad50\uc810\uc740 \uac8c\uc790\ub9ac\uc758 3\ub3c4\uc5d0\uc11c \uac15\uad50\uc810\uc740 \uc0ac\uc218\uc790\ub9ac\uc758 3\ub3c4\uc5d0\uc11c \uace0\uc591\uc9c0\uc704\ub97c \uac16\ub294\ub2e4. \uadf8 \uc704\uce58\ub4e4\uc740 11\uc138\uae30 \uc54c\ube44\ub8e8\ub2c8\uc758 \uc800\uc11c \u300a \uc810\uc131\uc608\uc220\uc758 \uc694\uc18c\uc5d0 \ub300\ud55c \uc18c\uac1c\uc11c (Book of Instruction in the elements of the art of astrology)\u300b\uc640 \uac19\uc740 \uc911\uc138 \ucd08\uae30 \uc544\ub78d \uc2dc\ub300\uc758 \uc810\uc131\uc220 \ubb38\ud5cc\ub4e4\uc5d0 \uc218\ub85d\ub418\uc5b4 \uc788\ub2e4. \ud604\ub300 \ubca0\ub2e4 \uc810\uc131\uac00\ub4e4\uc740 \uad50\uc810\uc758 \uace0\uc591 \uc9c0\uc704\uc5d0 \uc911\uc694\uc131\uc744 \ubd80\uc5ec\ud558\ub294 \ubc18\uba74, \uc911\uc138 \uc720\ub7fd\uc744 \ud1b5\ud574 \uc804\ud574\uc9c4 \uc11c\uc591 \uc810\uc131\uc220\uc758 \uc804\ud1b5\uc740 \uadf8 \uc2e4\ucc9c\uc5d0 \uc788\uc5b4\uc11c \uc804\ud1b5\uc801\uc73c\ub85c \uadf8\ub9ac\uace0 \ud604\uc7ac\uc5d0 \uadf8\uac83\ub4e4\uc744 \uc57d\uac04\ub9cc \uc0ac\uc6a9\ud55c\ub2e4. \uadf8\ub9ac\uc2a4\uc778\ub4e4\uacfc \ud398\ub974\uc2dc\uc544\uc778\ub4e4\uacfc\ub294 \ubc18\ub300\ub85c \uc54c\ube44\ub8e8\ub2c8\ub294 \uadf8 \uc810\ub4e4\uc758 \uace0\uc591\uc5d0 \ub300\ud574 \uc54c\ub9ac\uba70, \uadf8 \uc6d0\ub9ac\ub97c \"\uc544\uc8fc \uc801\uc808\ud55c\" \uac83\uc73c\ub85c \ubb18\uc0ac\ud588\ub294\ub370 \ubc18\ud574, \uadf8\uc758 \uc2dc\ub300\uc758 \ud78c\ub450 \uc810\uc131\uac00\ub4e4\uc740 \ud0dc\uc591\uacfc \ubaa9\uc131 \uadf8\ub9ac\uace0 \ud1a0\uc131\uc758 \uace0\uc591 \uc704\uce58\uc5d0 \uc77c\uce58\uac00 \uc5c6\uc5c8\uc73c\uba70, \uad50\uc810\uc758 \uace0\uc591\uc740 \uc778\uc815\ud558\uc9c0 \uc54a\uc558\ub2e4.", "sentence_answer": "\uadf8 \uc704\uce58\ub4e4\uc740 11\uc138\uae30 \uc54c\ube44\ub8e8\ub2c8\uc758 \uc800\uc11c \u300a \uc810\uc131\uc608\uc220\uc758 \uc694\uc18c\uc5d0 \ub300\ud55c \uc18c\uac1c\uc11c (Book of Instruction in the elements of the art of astrology)\u300b\uc640 \uac19\uc740 \uc911\uc138 \ucd08\uae30 \uc544\ub78d \uc2dc\ub300\uc758 \uc810\uc131\uc220 \ubb38\ud5cc\ub4e4\uc5d0 \uc218\ub85d\ub418\uc5b4 \uc788\ub2e4.", "paragraph_id": "6545987-0-0", "paragraph_question": "question: \uc810\uc131\uc220 \ubb38\ud5cc \uc911 \ud558\ub098\ub85c \uc54c\ube44\ub8e8\ub2c8\uc758 \uc800\uc11c\uc758 \uc774\ub984\uc740 \ubb34\uc5c7\uc778\uac00?, context: \uc2b9\uad50\uc810\uc740 \uac8c\uc790\ub9ac\uc758 3\ub3c4\uc5d0\uc11c \uac15\uad50\uc810\uc740 \uc0ac\uc218\uc790\ub9ac\uc758 3\ub3c4\uc5d0\uc11c \uace0\uc591\uc9c0\uc704\ub97c \uac16\ub294\ub2e4. \uadf8 \uc704\uce58\ub4e4\uc740 11\uc138\uae30 \uc54c\ube44\ub8e8\ub2c8\uc758 \uc800\uc11c \u300a\uc810\uc131\uc608\uc220\uc758 \uc694\uc18c\uc5d0 \ub300\ud55c \uc18c\uac1c\uc11c(Book of Instruction in the elements of the art of astrology)\u300b\uc640 \uac19\uc740 \uc911\uc138 \ucd08\uae30 \uc544\ub78d \uc2dc\ub300\uc758 \uc810\uc131\uc220 \ubb38\ud5cc\ub4e4\uc5d0 \uc218\ub85d\ub418\uc5b4 \uc788\ub2e4. \ud604\ub300 \ubca0\ub2e4 \uc810\uc131\uac00\ub4e4\uc740 \uad50\uc810\uc758 \uace0\uc591 \uc9c0\uc704\uc5d0 \uc911\uc694\uc131\uc744 \ubd80\uc5ec\ud558\ub294 \ubc18\uba74, \uc911\uc138 \uc720\ub7fd\uc744 \ud1b5\ud574 \uc804\ud574\uc9c4 \uc11c\uc591 \uc810\uc131\uc220\uc758 \uc804\ud1b5\uc740 \uadf8 \uc2e4\ucc9c\uc5d0 \uc788\uc5b4\uc11c \uc804\ud1b5\uc801\uc73c\ub85c \uadf8\ub9ac\uace0 \ud604\uc7ac\uc5d0 \uadf8\uac83\ub4e4\uc744 \uc57d\uac04\ub9cc \uc0ac\uc6a9\ud55c\ub2e4. \uadf8\ub9ac\uc2a4\uc778\ub4e4\uacfc \ud398\ub974\uc2dc\uc544\uc778\ub4e4\uacfc\ub294 \ubc18\ub300\ub85c \uc54c\ube44\ub8e8\ub2c8\ub294 \uadf8 \uc810\ub4e4\uc758 \uace0\uc591\uc5d0 \ub300\ud574 \uc54c\ub9ac\uba70, \uadf8 \uc6d0\ub9ac\ub97c \"\uc544\uc8fc \uc801\uc808\ud55c\" \uac83\uc73c\ub85c \ubb18\uc0ac\ud588\ub294\ub370 \ubc18\ud574, \uadf8\uc758 \uc2dc\ub300\uc758 \ud78c\ub450 \uc810\uc131\uac00\ub4e4\uc740 \ud0dc\uc591\uacfc \ubaa9\uc131 \uadf8\ub9ac\uace0 \ud1a0\uc131\uc758 \uace0\uc591 \uc704\uce58\uc5d0 \uc77c\uce58\uac00 \uc5c6\uc5c8\uc73c\uba70, \uad50\uc810\uc758 \uace0\uc591\uc740 \uc778\uc815\ud558\uc9c0 \uc54a\uc558\ub2e4."} {"question": "\ubc14\uc608\uc9c0\ub4dc 1\uc138\ub294 \uba87 \ub144\uc5d0 \uc624\uc2a4\ub9cc\uc81c\uad6d\uc758 \uc655\uc704\uc5d0 \uc62c\ub790\ub294\uac00?", "paragraph": "\ub2c8\ucf54\ud3f4\ub9ac\uc2a4 \uc804\ud22c(\ubd88\uac00\ub9ac\uc544\uc5b4: \u0411\u0438\u0442\u043a\u0430 \u043f\u0440\u0438 \u041d\u0438\u043a\u043e\u043f\u043e\u043b, Bitka pri Nikopol;\ud130\ud0a4\uc5b4: Ni\u011fbolu Sava\u015f\u0131;\ud5dd\uac00\ub9ac\uc5b4: Nik\u00e1polyi Csata;\ub8e8\ub9c8\ub2c8\uc544\uc5b4: B\u0103t\u0103lia de la Nicopole)\ub294 1396\ub144 9\uc6d4 25\uc77c(\ud639\uc740 9\uc6d4 28\uc77c\uc774\ub780 \ub9d0\ub3c4 \uc788\ub2e4)\uc5d0 \ub3c4\ub098\uc6b0\uac15\ubcc0\uc758 \ub2c8\ucf54\ud3f4\ub9ac\uc2a4\uc5d0\uc11c \uc624\uc2a4\ub9cc \uc81c\uad6d\uc758 \ubc14\uc608\uc9c0\ub4dc 1\uc138(\uc81c\uc704:1389\ub144~1402\ub144)\uc640 \ud5dd\uac00\ub9ac \uc655 \uc9c0\uae30\uc2a4\ubb38\ud2b8\uac00 \uc774\ub044\ub294 \uc720\ub7fd\uc5f0\ud569(\ucc38\uac00\uc138\ub825:\ud5dd\uac00\ub9ac \uc655\uad6d, \uc2e0\uc131\ub85c\ub9c8\uc81c\uad6d, \ud504\ub791\uc2a4, \uc648\ub77c\ud0a4\uc544, \ud3f4\ub780\ub4dc, \uc789\uae00\ub79c\ub4dc, \uc2a4\ucf54\ud2c0\ub79c\ub4dc \uc655\uad6d, \uad6c \uc2a4\uc704\uc2a4 \uc5f0\ud569, \ud29c\ud2bc \uae30\uc0ac\ub2e8, \ubca0\ub124\uce58\uc544 \uacf5\ud654\uad6d, \uc81c\ub178\ubc14 \uacf5\ud654\uad6d, \uc131 \uc694\ud55c \uae30\uc0ac\ub2e8)\uc0ac\uc774\uc5d0\uc11c \uc77c\uc5b4\ub09c \uc804\ud22c\uc774\ub2e4. \ud754\ud788 \ub2c8\ucf54\ud3f4\ub9ac\uc2a4 \uc2ed\uc790\uad70\uc774\ub77c\uace0 \ubd88\ub9ac\uba70, \uc911\uc138 \ucd5c\ud6c4\uc758 \ub300\uaddc\ubaa8 \uc2ed\uc790\uad70\uc774\uc5c8\ub2e4. \uc804\ud22c \uacb0\uacfc\ub294 \uc624\uc2a4\ub9cc \uc81c\uad6d\uc758 \uacb0\uc815\uc801 \uc2b9\ub9ac\ub85c \ub05d\ub0ac\uace0, \ubc14\uc608\uc9c0\ub4dc 1\uc138\ub294 \uce74\uc774\ub85c\uc758 \ub9d8\ub8e8\ud06c \uc655\uc870 \ubcf4\ud638\ud558\uc5d0 \uc788\ub358 \uce7c\ub9ac\ud30c\ub85c\ubd80\ud130 \ub192\uc740 \ud3c9\uac00\ub97c \ubc1b\uc544 \uc220\ud0c4\uc758 \uce6d\ud638\ub97c \ud558\uc0ac\ubc1b\uc558\ub2e4.", "answer": "1389\ub144", "sentence": "\ub2c8\ucf54\ud3f4\ub9ac\uc2a4 \uc804\ud22c(\ubd88\uac00\ub9ac\uc544\uc5b4: \u0411\u0438\u0442\u043a\u0430 \u043f\u0440\u0438 \u041d\u0438\u043a\u043e\u043f\u043e\u043b, Bitka pri Nikopol;\ud130\ud0a4\uc5b4: Ni\u011fbolu Sava\u015f\u0131;\ud5dd\uac00\ub9ac\uc5b4: Nik\u00e1polyi Csata;\ub8e8\ub9c8\ub2c8\uc544\uc5b4: B\u0103t\u0103lia de la Nicopole)\ub294 1396\ub144 9\uc6d4 25\uc77c(\ud639\uc740 9\uc6d4 28\uc77c\uc774\ub780 \ub9d0\ub3c4 \uc788\ub2e4)\uc5d0 \ub3c4\ub098\uc6b0\uac15\ubcc0\uc758 \ub2c8\ucf54\ud3f4\ub9ac\uc2a4\uc5d0\uc11c \uc624\uc2a4\ub9cc \uc81c\uad6d\uc758 \ubc14\uc608\uc9c0\ub4dc 1\uc138(\uc81c\uc704: 1389\ub144 ~1402\ub144)", "paragraph_sentence": " \ub2c8\ucf54\ud3f4\ub9ac\uc2a4 \uc804\ud22c(\ubd88\uac00\ub9ac\uc544\uc5b4: \u0411\u0438\u0442\u043a\u0430 \u043f\u0440\u0438 \u041d\u0438\u043a\u043e\u043f\u043e\u043b, Bitka pri Nikopol;\ud130\ud0a4\uc5b4: Ni\u011fbolu Sava\u015f\u0131;\ud5dd\uac00\ub9ac\uc5b4: Nik\u00e1polyi Csata;\ub8e8\ub9c8\ub2c8\uc544\uc5b4: B\u0103t\u0103lia de la Nicopole)\ub294 1396\ub144 9\uc6d4 25\uc77c(\ud639\uc740 9\uc6d4 28\uc77c\uc774\ub780 \ub9d0\ub3c4 \uc788\ub2e4)\uc5d0 \ub3c4\ub098\uc6b0\uac15\ubcc0\uc758 \ub2c8\ucf54\ud3f4\ub9ac\uc2a4\uc5d0\uc11c \uc624\uc2a4\ub9cc \uc81c\uad6d\uc758 \ubc14\uc608\uc9c0\ub4dc 1\uc138(\uc81c\uc704: 1389\ub144 ~1402\ub144) \uc640 \ud5dd\uac00\ub9ac \uc655 \uc9c0\uae30\uc2a4\ubb38\ud2b8\uac00 \uc774\ub044\ub294 \uc720\ub7fd\uc5f0\ud569(\ucc38\uac00\uc138\ub825:\ud5dd\uac00\ub9ac \uc655\uad6d, \uc2e0\uc131\ub85c\ub9c8\uc81c\uad6d, \ud504\ub791\uc2a4, \uc648\ub77c\ud0a4\uc544, \ud3f4\ub780\ub4dc, \uc789\uae00\ub79c\ub4dc, \uc2a4\ucf54\ud2c0\ub79c\ub4dc \uc655\uad6d, \uad6c \uc2a4\uc704\uc2a4 \uc5f0\ud569, \ud29c\ud2bc \uae30\uc0ac\ub2e8, \ubca0\ub124\uce58\uc544 \uacf5\ud654\uad6d, \uc81c\ub178\ubc14 \uacf5\ud654\uad6d, \uc131 \uc694\ud55c \uae30\uc0ac\ub2e8)\uc0ac\uc774\uc5d0\uc11c \uc77c\uc5b4\ub09c \uc804\ud22c\uc774\ub2e4. \ud754\ud788 \ub2c8\ucf54\ud3f4\ub9ac\uc2a4 \uc2ed\uc790\uad70\uc774\ub77c\uace0 \ubd88\ub9ac\uba70, \uc911\uc138 \ucd5c\ud6c4\uc758 \ub300\uaddc\ubaa8 \uc2ed\uc790\uad70\uc774\uc5c8\ub2e4. \uc804\ud22c \uacb0\uacfc\ub294 \uc624\uc2a4\ub9cc \uc81c\uad6d\uc758 \uacb0\uc815\uc801 \uc2b9\ub9ac\ub85c \ub05d\ub0ac\uace0, \ubc14\uc608\uc9c0\ub4dc 1\uc138\ub294 \uce74\uc774\ub85c\uc758 \ub9d8\ub8e8\ud06c \uc655\uc870 \ubcf4\ud638\ud558\uc5d0 \uc788\ub358 \uce7c\ub9ac\ud30c\ub85c\ubd80\ud130 \ub192\uc740 \ud3c9\uac00\ub97c \ubc1b\uc544 \uc220\ud0c4\uc758 \uce6d\ud638\ub97c \ud558\uc0ac\ubc1b\uc558\ub2e4.", "paragraph_answer": "\ub2c8\ucf54\ud3f4\ub9ac\uc2a4 \uc804\ud22c(\ubd88\uac00\ub9ac\uc544\uc5b4: \u0411\u0438\u0442\u043a\u0430 \u043f\u0440\u0438 \u041d\u0438\u043a\u043e\u043f\u043e\u043b, Bitka pri Nikopol;\ud130\ud0a4\uc5b4: Ni\u011fbolu Sava\u015f\u0131;\ud5dd\uac00\ub9ac\uc5b4: Nik\u00e1polyi Csata;\ub8e8\ub9c8\ub2c8\uc544\uc5b4: B\u0103t\u0103lia de la Nicopole)\ub294 1396\ub144 9\uc6d4 25\uc77c(\ud639\uc740 9\uc6d4 28\uc77c\uc774\ub780 \ub9d0\ub3c4 \uc788\ub2e4)\uc5d0 \ub3c4\ub098\uc6b0\uac15\ubcc0\uc758 \ub2c8\ucf54\ud3f4\ub9ac\uc2a4\uc5d0\uc11c \uc624\uc2a4\ub9cc \uc81c\uad6d\uc758 \ubc14\uc608\uc9c0\ub4dc 1\uc138(\uc81c\uc704: 1389\ub144 ~1402\ub144)\uc640 \ud5dd\uac00\ub9ac \uc655 \uc9c0\uae30\uc2a4\ubb38\ud2b8\uac00 \uc774\ub044\ub294 \uc720\ub7fd\uc5f0\ud569(\ucc38\uac00\uc138\ub825:\ud5dd\uac00\ub9ac \uc655\uad6d, \uc2e0\uc131\ub85c\ub9c8\uc81c\uad6d, \ud504\ub791\uc2a4, \uc648\ub77c\ud0a4\uc544, \ud3f4\ub780\ub4dc, \uc789\uae00\ub79c\ub4dc, \uc2a4\ucf54\ud2c0\ub79c\ub4dc \uc655\uad6d, \uad6c \uc2a4\uc704\uc2a4 \uc5f0\ud569, \ud29c\ud2bc \uae30\uc0ac\ub2e8, \ubca0\ub124\uce58\uc544 \uacf5\ud654\uad6d, \uc81c\ub178\ubc14 \uacf5\ud654\uad6d, \uc131 \uc694\ud55c \uae30\uc0ac\ub2e8)\uc0ac\uc774\uc5d0\uc11c \uc77c\uc5b4\ub09c \uc804\ud22c\uc774\ub2e4. \ud754\ud788 \ub2c8\ucf54\ud3f4\ub9ac\uc2a4 \uc2ed\uc790\uad70\uc774\ub77c\uace0 \ubd88\ub9ac\uba70, \uc911\uc138 \ucd5c\ud6c4\uc758 \ub300\uaddc\ubaa8 \uc2ed\uc790\uad70\uc774\uc5c8\ub2e4. \uc804\ud22c \uacb0\uacfc\ub294 \uc624\uc2a4\ub9cc \uc81c\uad6d\uc758 \uacb0\uc815\uc801 \uc2b9\ub9ac\ub85c \ub05d\ub0ac\uace0, \ubc14\uc608\uc9c0\ub4dc 1\uc138\ub294 \uce74\uc774\ub85c\uc758 \ub9d8\ub8e8\ud06c \uc655\uc870 \ubcf4\ud638\ud558\uc5d0 \uc788\ub358 \uce7c\ub9ac\ud30c\ub85c\ubd80\ud130 \ub192\uc740 \ud3c9\uac00\ub97c \ubc1b\uc544 \uc220\ud0c4\uc758 \uce6d\ud638\ub97c \ud558\uc0ac\ubc1b\uc558\ub2e4.", "sentence_answer": "\ub2c8\ucf54\ud3f4\ub9ac\uc2a4 \uc804\ud22c(\ubd88\uac00\ub9ac\uc544\uc5b4: \u0411\u0438\u0442\u043a\u0430 \u043f\u0440\u0438 \u041d\u0438\u043a\u043e\u043f\u043e\u043b, Bitka pri Nikopol;\ud130\ud0a4\uc5b4: Ni\u011fbolu Sava\u015f\u0131;\ud5dd\uac00\ub9ac\uc5b4: Nik\u00e1polyi Csata;\ub8e8\ub9c8\ub2c8\uc544\uc5b4: B\u0103t\u0103lia de la Nicopole)\ub294 1396\ub144 9\uc6d4 25\uc77c(\ud639\uc740 9\uc6d4 28\uc77c\uc774\ub780 \ub9d0\ub3c4 \uc788\ub2e4)\uc5d0 \ub3c4\ub098\uc6b0\uac15\ubcc0\uc758 \ub2c8\ucf54\ud3f4\ub9ac\uc2a4\uc5d0\uc11c \uc624\uc2a4\ub9cc \uc81c\uad6d\uc758 \ubc14\uc608\uc9c0\ub4dc 1\uc138(\uc81c\uc704: 1389\ub144 ~1402\ub144)", "paragraph_id": "6484835-0-1", "paragraph_question": "question: \ubc14\uc608\uc9c0\ub4dc 1\uc138\ub294 \uba87 \ub144\uc5d0 \uc624\uc2a4\ub9cc\uc81c\uad6d\uc758 \uc655\uc704\uc5d0 \uc62c\ub790\ub294\uac00?, context: \ub2c8\ucf54\ud3f4\ub9ac\uc2a4 \uc804\ud22c(\ubd88\uac00\ub9ac\uc544\uc5b4: \u0411\u0438\u0442\u043a\u0430 \u043f\u0440\u0438 \u041d\u0438\u043a\u043e\u043f\u043e\u043b, Bitka pri Nikopol;\ud130\ud0a4\uc5b4: Ni\u011fbolu Sava\u015f\u0131;\ud5dd\uac00\ub9ac\uc5b4: Nik\u00e1polyi Csata;\ub8e8\ub9c8\ub2c8\uc544\uc5b4: B\u0103t\u0103lia de la Nicopole)\ub294 1396\ub144 9\uc6d4 25\uc77c(\ud639\uc740 9\uc6d4 28\uc77c\uc774\ub780 \ub9d0\ub3c4 \uc788\ub2e4)\uc5d0 \ub3c4\ub098\uc6b0\uac15\ubcc0\uc758 \ub2c8\ucf54\ud3f4\ub9ac\uc2a4\uc5d0\uc11c \uc624\uc2a4\ub9cc \uc81c\uad6d\uc758 \ubc14\uc608\uc9c0\ub4dc 1\uc138(\uc81c\uc704:1389\ub144~1402\ub144)\uc640 \ud5dd\uac00\ub9ac \uc655 \uc9c0\uae30\uc2a4\ubb38\ud2b8\uac00 \uc774\ub044\ub294 \uc720\ub7fd\uc5f0\ud569(\ucc38\uac00\uc138\ub825:\ud5dd\uac00\ub9ac \uc655\uad6d, \uc2e0\uc131\ub85c\ub9c8\uc81c\uad6d, \ud504\ub791\uc2a4, \uc648\ub77c\ud0a4\uc544, \ud3f4\ub780\ub4dc, \uc789\uae00\ub79c\ub4dc, \uc2a4\ucf54\ud2c0\ub79c\ub4dc \uc655\uad6d, \uad6c \uc2a4\uc704\uc2a4 \uc5f0\ud569, \ud29c\ud2bc \uae30\uc0ac\ub2e8, \ubca0\ub124\uce58\uc544 \uacf5\ud654\uad6d, \uc81c\ub178\ubc14 \uacf5\ud654\uad6d, \uc131 \uc694\ud55c \uae30\uc0ac\ub2e8)\uc0ac\uc774\uc5d0\uc11c \uc77c\uc5b4\ub09c \uc804\ud22c\uc774\ub2e4. \ud754\ud788 \ub2c8\ucf54\ud3f4\ub9ac\uc2a4 \uc2ed\uc790\uad70\uc774\ub77c\uace0 \ubd88\ub9ac\uba70, \uc911\uc138 \ucd5c\ud6c4\uc758 \ub300\uaddc\ubaa8 \uc2ed\uc790\uad70\uc774\uc5c8\ub2e4. \uc804\ud22c \uacb0\uacfc\ub294 \uc624\uc2a4\ub9cc \uc81c\uad6d\uc758 \uacb0\uc815\uc801 \uc2b9\ub9ac\ub85c \ub05d\ub0ac\uace0, \ubc14\uc608\uc9c0\ub4dc 1\uc138\ub294 \uce74\uc774\ub85c\uc758 \ub9d8\ub8e8\ud06c \uc655\uc870 \ubcf4\ud638\ud558\uc5d0 \uc788\ub358 \uce7c\ub9ac\ud30c\ub85c\ubd80\ud130 \ub192\uc740 \ud3c9\uac00\ub97c \ubc1b\uc544 \uc220\ud0c4\uc758 \uce6d\ud638\ub97c \ud558\uc0ac\ubc1b\uc558\ub2e4."} {"question": "\uc7ad \ud5e4\ub354\ub9c1\ud134\uc774 \uc2e4\uc218\ub97c \uace0\uce58\uae30 \uc704\ud574 \ub9cc\ub4e4\uc5b4\ub0b8 \uc81c2\uc800\uc790\uc758 \uc774\ub984\uc740?", "paragraph": "\uc7ad \ud5e4\ub354\ub9c1\ud134\uc740 \uace0\uc591\uc774\ub97c \uc81c2\uc800\uc790\ub85c \uc0bc\uc740 \uc774\uc720\ub97c, \ubcf8\ub798 \ub17c\ubb38\uc758 1\uc778\uce6d \ub300\uba85\uc0ac\ub4e4\uc744 \ub2e8\uc218\uac00 \uc544\ub2cc \ubcf5\uc218\ub85c \uc4f0\ub294 \uc2e4\uc218\ub97c \uc800\uc9c8\ub800\uace0, \u300a\ud53c\uc9c0\uceec \ub9ac\ubdf0\u300b\uc758 \ud3b8\uc9d1\uc790\ub4e4\uc774 \uc774\ub7ec\ud55c \ud615\uc2dd\uc5d0 \ub300\ud558\uc5ec \uc800\uc790\uac00 \ucd5c\uc18c \ub450 \uba85 \uc774\uc0c1\uc77c \uacbd\uc6b0\uc5d0\ub9cc \ud5c8\uc6a9\ud588\uae30 \ub54c\ubb38\uc774\ub77c\uace0 \uc124\uba85\ud588\ub2e4. \ub17c\ubb38\uc758 \ucc98\uc74c\ubd80\ud130 \ub05d\uae4c\uc9c0 \ubcf5\uc218\ub300\uba85\uc0ac\ub4e4\uc744 \ucc3e\uc544\uc11c \ub2e8\uc218\ub300\uba85\uc0ac\ub85c \ubc14\uafb8\ub290\ub2c8 \uadc0\ucc2e\uc558\ub358 \ud5e4\ub354\ub9c1\ud134\uc740 \uc81c2\uc800\uc790\ub97c \"\ub9cc\ub4e4\uc5b4 \ub0b4\uae30\ub85c\" \ud588\ub2e4. \uadf8\ub798\uc11c \uadf8\ub294 \"F.D.C. \uc70c\ub7ec\ub4dc\"\ub77c\ub294 \uc81c2\uc800\uc790\ub97c \ub9cc\ub4e4\uc5b4\ub0c8\uace0, \uc5ec\uae30\uc11c F.D. \ub77c\ub294 \uc774\ub2c8\uc15c\uc740 \uc9d1\uace0\uc591\uc774\uc758 \ud559\uba85\uc778 \"\ud3a0\ub9ac\uc2a4 \ub3c4\uba54\uc2a4\ud2f0\ucfe0\uc2a4\"\ub97c \uc758\ubbf8\ud558\ub294 \uac83\uc774\ub2e4. C. \ub294 \uace0\uc591\uc774 \uc774\ub984 \"\uccb4\uc2a4\ud130\"\uc774\uace0, \"\uc70c\ub7ec\ub4dc\"\ub294 \ucf5c\ub85c\ub77c\ub3c4 \uc8fc \uc560\uc2a4\ud380\uc5d0 \uc0b4\ub358 \uc2dc\uc554\uace0\uc591\uc774\ub85c, \ub2e4\ub984\uc544\ub2cc \uccb4\uc2a4\ud130\uc758 \uc544\ube44\uace0\uc591\uc774\uc600\ub2e4.", "answer": "F.D.C. \uc70c\ub7ec\ub4dc", "sentence": "\uadf8\ub798\uc11c \uadf8\ub294 \" F.D.C. \uc70c\ub7ec\ub4dc \"\ub77c\ub294 \uc81c2\uc800\uc790\ub97c \ub9cc\ub4e4\uc5b4\ub0c8\uace0, \uc5ec\uae30\uc11c F.D. \ub77c\ub294 \uc774\ub2c8\uc15c\uc740 \uc9d1\uace0\uc591\uc774\uc758 \ud559\uba85\uc778 \"\ud3a0\ub9ac\uc2a4 \ub3c4\uba54\uc2a4\ud2f0\ucfe0\uc2a4\"\ub97c \uc758\ubbf8\ud558\ub294 \uac83\uc774\ub2e4.", "paragraph_sentence": "\uc7ad \ud5e4\ub354\ub9c1\ud134\uc740 \uace0\uc591\uc774\ub97c \uc81c2\uc800\uc790\ub85c \uc0bc\uc740 \uc774\uc720\ub97c, \ubcf8\ub798 \ub17c\ubb38\uc758 1\uc778\uce6d \ub300\uba85\uc0ac\ub4e4\uc744 \ub2e8\uc218\uac00 \uc544\ub2cc \ubcf5\uc218\ub85c \uc4f0\ub294 \uc2e4\uc218\ub97c \uc800\uc9c8\ub800\uace0, \u300a\ud53c\uc9c0\uceec \ub9ac\ubdf0\u300b\uc758 \ud3b8\uc9d1\uc790\ub4e4\uc774 \uc774\ub7ec\ud55c \ud615\uc2dd\uc5d0 \ub300\ud558\uc5ec \uc800\uc790\uac00 \ucd5c\uc18c \ub450 \uba85 \uc774\uc0c1\uc77c \uacbd\uc6b0\uc5d0\ub9cc \ud5c8\uc6a9\ud588\uae30 \ub54c\ubb38\uc774\ub77c\uace0 \uc124\uba85\ud588\ub2e4. \ub17c\ubb38\uc758 \ucc98\uc74c\ubd80\ud130 \ub05d\uae4c\uc9c0 \ubcf5\uc218\ub300\uba85\uc0ac\ub4e4\uc744 \ucc3e\uc544\uc11c \ub2e8\uc218\ub300\uba85\uc0ac\ub85c \ubc14\uafb8\ub290\ub2c8 \uadc0\ucc2e\uc558\ub358 \ud5e4\ub354\ub9c1\ud134\uc740 \uc81c2\uc800\uc790\ub97c \"\ub9cc\ub4e4\uc5b4 \ub0b4\uae30\ub85c\" \ud588\ub2e4. \uadf8\ub798\uc11c \uadf8\ub294 \" F.D.C. \uc70c\ub7ec\ub4dc \"\ub77c\ub294 \uc81c2\uc800\uc790\ub97c \ub9cc\ub4e4\uc5b4\ub0c8\uace0, \uc5ec\uae30\uc11c F.D. \ub77c\ub294 \uc774\ub2c8\uc15c\uc740 \uc9d1\uace0\uc591\uc774\uc758 \ud559\uba85\uc778 \"\ud3a0\ub9ac\uc2a4 \ub3c4\uba54\uc2a4\ud2f0\ucfe0\uc2a4\"\ub97c \uc758\ubbf8\ud558\ub294 \uac83\uc774\ub2e4. C. \ub294 \uace0\uc591\uc774 \uc774\ub984 \"\uccb4\uc2a4\ud130\"\uc774\uace0, \"\uc70c\ub7ec\ub4dc\"\ub294 \ucf5c\ub85c\ub77c\ub3c4 \uc8fc \uc560\uc2a4\ud380\uc5d0 \uc0b4\ub358 \uc2dc\uc554\uace0\uc591\uc774\ub85c, \ub2e4\ub984\uc544\ub2cc \uccb4\uc2a4\ud130\uc758 \uc544\ube44\uace0\uc591\uc774\uc600\ub2e4.", "paragraph_answer": "\uc7ad \ud5e4\ub354\ub9c1\ud134\uc740 \uace0\uc591\uc774\ub97c \uc81c2\uc800\uc790\ub85c \uc0bc\uc740 \uc774\uc720\ub97c, \ubcf8\ub798 \ub17c\ubb38\uc758 1\uc778\uce6d \ub300\uba85\uc0ac\ub4e4\uc744 \ub2e8\uc218\uac00 \uc544\ub2cc \ubcf5\uc218\ub85c \uc4f0\ub294 \uc2e4\uc218\ub97c \uc800\uc9c8\ub800\uace0, \u300a\ud53c\uc9c0\uceec \ub9ac\ubdf0\u300b\uc758 \ud3b8\uc9d1\uc790\ub4e4\uc774 \uc774\ub7ec\ud55c \ud615\uc2dd\uc5d0 \ub300\ud558\uc5ec \uc800\uc790\uac00 \ucd5c\uc18c \ub450 \uba85 \uc774\uc0c1\uc77c \uacbd\uc6b0\uc5d0\ub9cc \ud5c8\uc6a9\ud588\uae30 \ub54c\ubb38\uc774\ub77c\uace0 \uc124\uba85\ud588\ub2e4. \ub17c\ubb38\uc758 \ucc98\uc74c\ubd80\ud130 \ub05d\uae4c\uc9c0 \ubcf5\uc218\ub300\uba85\uc0ac\ub4e4\uc744 \ucc3e\uc544\uc11c \ub2e8\uc218\ub300\uba85\uc0ac\ub85c \ubc14\uafb8\ub290\ub2c8 \uadc0\ucc2e\uc558\ub358 \ud5e4\ub354\ub9c1\ud134\uc740 \uc81c2\uc800\uc790\ub97c \"\ub9cc\ub4e4\uc5b4 \ub0b4\uae30\ub85c\" \ud588\ub2e4. \uadf8\ub798\uc11c \uadf8\ub294 \" F.D.C. \uc70c\ub7ec\ub4dc \"\ub77c\ub294 \uc81c2\uc800\uc790\ub97c \ub9cc\ub4e4\uc5b4\ub0c8\uace0, \uc5ec\uae30\uc11c F.D. \ub77c\ub294 \uc774\ub2c8\uc15c\uc740 \uc9d1\uace0\uc591\uc774\uc758 \ud559\uba85\uc778 \"\ud3a0\ub9ac\uc2a4 \ub3c4\uba54\uc2a4\ud2f0\ucfe0\uc2a4\"\ub97c \uc758\ubbf8\ud558\ub294 \uac83\uc774\ub2e4. C. \ub294 \uace0\uc591\uc774 \uc774\ub984 \"\uccb4\uc2a4\ud130\"\uc774\uace0, \"\uc70c\ub7ec\ub4dc\"\ub294 \ucf5c\ub85c\ub77c\ub3c4 \uc8fc \uc560\uc2a4\ud380\uc5d0 \uc0b4\ub358 \uc2dc\uc554\uace0\uc591\uc774\ub85c, \ub2e4\ub984\uc544\ub2cc \uccb4\uc2a4\ud130\uc758 \uc544\ube44\uace0\uc591\uc774\uc600\ub2e4.", "sentence_answer": "\uadf8\ub798\uc11c \uadf8\ub294 \" F.D.C. \uc70c\ub7ec\ub4dc \"\ub77c\ub294 \uc81c2\uc800\uc790\ub97c \ub9cc\ub4e4\uc5b4\ub0c8\uace0, \uc5ec\uae30\uc11c F.D. \ub77c\ub294 \uc774\ub2c8\uc15c\uc740 \uc9d1\uace0\uc591\uc774\uc758 \ud559\uba85\uc778 \"\ud3a0\ub9ac\uc2a4 \ub3c4\uba54\uc2a4\ud2f0\ucfe0\uc2a4\"\ub97c \uc758\ubbf8\ud558\ub294 \uac83\uc774\ub2e4.", "paragraph_id": "6596864-1-1", "paragraph_question": "question: \uc7ad \ud5e4\ub354\ub9c1\ud134\uc774 \uc2e4\uc218\ub97c \uace0\uce58\uae30 \uc704\ud574 \ub9cc\ub4e4\uc5b4\ub0b8 \uc81c2\uc800\uc790\uc758 \uc774\ub984\uc740?, context: \uc7ad \ud5e4\ub354\ub9c1\ud134\uc740 \uace0\uc591\uc774\ub97c \uc81c2\uc800\uc790\ub85c \uc0bc\uc740 \uc774\uc720\ub97c, \ubcf8\ub798 \ub17c\ubb38\uc758 1\uc778\uce6d \ub300\uba85\uc0ac\ub4e4\uc744 \ub2e8\uc218\uac00 \uc544\ub2cc \ubcf5\uc218\ub85c \uc4f0\ub294 \uc2e4\uc218\ub97c \uc800\uc9c8\ub800\uace0, \u300a\ud53c\uc9c0\uceec \ub9ac\ubdf0\u300b\uc758 \ud3b8\uc9d1\uc790\ub4e4\uc774 \uc774\ub7ec\ud55c \ud615\uc2dd\uc5d0 \ub300\ud558\uc5ec \uc800\uc790\uac00 \ucd5c\uc18c \ub450 \uba85 \uc774\uc0c1\uc77c \uacbd\uc6b0\uc5d0\ub9cc \ud5c8\uc6a9\ud588\uae30 \ub54c\ubb38\uc774\ub77c\uace0 \uc124\uba85\ud588\ub2e4. \ub17c\ubb38\uc758 \ucc98\uc74c\ubd80\ud130 \ub05d\uae4c\uc9c0 \ubcf5\uc218\ub300\uba85\uc0ac\ub4e4\uc744 \ucc3e\uc544\uc11c \ub2e8\uc218\ub300\uba85\uc0ac\ub85c \ubc14\uafb8\ub290\ub2c8 \uadc0\ucc2e\uc558\ub358 \ud5e4\ub354\ub9c1\ud134\uc740 \uc81c2\uc800\uc790\ub97c \"\ub9cc\ub4e4\uc5b4 \ub0b4\uae30\ub85c\" \ud588\ub2e4. \uadf8\ub798\uc11c \uadf8\ub294 \"F.D.C. \uc70c\ub7ec\ub4dc\"\ub77c\ub294 \uc81c2\uc800\uc790\ub97c \ub9cc\ub4e4\uc5b4\ub0c8\uace0, \uc5ec\uae30\uc11c F.D. \ub77c\ub294 \uc774\ub2c8\uc15c\uc740 \uc9d1\uace0\uc591\uc774\uc758 \ud559\uba85\uc778 \"\ud3a0\ub9ac\uc2a4 \ub3c4\uba54\uc2a4\ud2f0\ucfe0\uc2a4\"\ub97c \uc758\ubbf8\ud558\ub294 \uac83\uc774\ub2e4. C. \ub294 \uace0\uc591\uc774 \uc774\ub984 \"\uccb4\uc2a4\ud130\"\uc774\uace0, \"\uc70c\ub7ec\ub4dc\"\ub294 \ucf5c\ub85c\ub77c\ub3c4 \uc8fc \uc560\uc2a4\ud380\uc5d0 \uc0b4\ub358 \uc2dc\uc554\uace0\uc591\uc774\ub85c, \ub2e4\ub984\uc544\ub2cc \uccb4\uc2a4\ud130\uc758 \uc544\ube44\uace0\uc591\uc774\uc600\ub2e4."} {"question": "\uad8c\ub825\uae30\uad00 \uac1c\ud3b8\ubc29\uc548\uc5d0 \uc758\ud574 \uce58\uc548, \uacbd\ube44, \uc815\ubcf4\ub97c \ub2f4\ub2f9\ud558\ub294 \uacbd\ucc30\uc740 \ub204\uad6c\uc778\uac00?", "paragraph": "2018\ub144 1\uc6d4 14\uc77c \uccad\uc640\ub300\ub294 \uad6d\uc815\uc6d0\uacfc \uac80\ucc30\uc758 \uad8c\ud55c\uc744 \uacbd\ucc30\ub85c \uc774\uad00\ud558\ub294 \ub0b4\uc6a9\uc744 \ub2f4\uc740 \uad8c\ub825\uae30\uad00 \uac1c\ud3b8\ubc29\uc548\uc744 \ubc1c\ud45c\ud588\ub2e4. \uac80\ucc30\uc740 \uc8fc\uc694 \uc0ac\uac74\uc758 1\ucc28\uc801 \uc218\uc0ac\ub294 \uacbd\ucc30\uccad\uc73c\ub85c, \uace0\uc704\uacf5\uc9c1\uc790 \uc218\uc0ac \ubc0f \uae30\uc18c \uad8c\ud55c\uc740 \uacf5\uc218\ucc98\ub97c \uc2e0\uc124\ud574 \ub9e1\ub3c4\ub85d \ud588\uc73c\uba70 \uc9c1\uc811 \uc218\uc0ac\ub294 \uacbd\uc81c\u00b7\uae08\uc735 \uac19\uc740 \ud2b9\uc218\uc218\uc0ac\ub85c \ud55c\uc815\ud588\ub2e4. \uad6d\uc815\uc6d0\uc740 \ub300\uacf5\uc218\uc0ac\uad8c\uc744 \uacbd\ucc30\uccad \uc0b0\ud558\uc5d0 \ub3c5\ub9bd\ub41c \uc548\ubcf4\uc870\uc9c1\uc744 \ub9cc\ub4e4\uc5b4 \uc774\uad00\ud558\uba70 \uad6d\ub0b4 \uc815\ubcf4 \ud65c\ub3d9\uc744 \ud3d0\uc9c0\ud558\uace0 \ub300\ubd81\u00b7\ud574\uc678 \uc815\ubcf4 \ubd80\uc11c\ub85c\ub9cc \uc5ed\ud560\uc744 \uc904\uc774\uae30\ub85c \ud588\ub2e4. \ud55c\ud3b8, \uac80\ucc30\uacfc \uad6d\uc815\uc6d0\uc758 \ud575\uc2ec \uad8c\ud55c\uc744 \uc774\uad00\ubc1b\ub294 \uacbd\ucc30\uc740 \uacbd\ucc30\uad8c \uacac\uc81c\ub97c \uc704\ud574 \uad6d\uac00\uacbd\ucc30\uacfc \uc790\uce58\uacbd\ucc30\ub85c \ubd84\ub9ac\ud558\uba70 \uce58\uc548\u00b7\uacbd\ube44\u00b7\uc815\ubcf4\ub97c \ub2f4\ub2f9\ud558\ub294 \uc77c\ubc18\uacbd\ucc30\uacfc 1\ucc28\uc801 \uc218\uc0ac\ub97c \ub2f4\ub2f9\ud558\ub294 \uc218\uc0ac\uacbd\ucc30\ub85c \uad6c\ubd84\ud558\uae30\ub85c \ud588\ub2e4. \ud558\uc9c0\ub9cc \uc774\uc5d0 \ub300\ud55c \ubc18\ubc1c\ub3c4 \ub098\uc654\ub2e4. \ud2b9\ud788 \uad6d\uc815\uc6d0\uc758 \ub300\uacf5\uc218\uc0ac\uad8c\uc744 \uacbd\ucc30\uc5d0 \uc774\uad00\ud558\ub294 \uac83\uc5d0 \ub300\ud55c \uc774\uacac\uc774 \ub9ce\uc558\ub294\ub370 \uad6d\uc815\uc6d0\uc740 \ub300\uacf5\uc218\uc0ac\uc5d0 \ub300\ud55c \ubaa8\ub4e0 \uae30\ub2a5, \ub178\ud558\uc6b0, \ucca9\ubcf4\ub9dd\uc744 \uac00\uc9c0\uace0 \uc788\uc9c0\ub9cc \uacbd\ucc30\uc740 \uc774\uac83\uc774 \uc5c6\uc5b4 \ud55c\uacc4\uac00 \uc788\ub2e4\ub294 \uac83\uc774\ub2e4. \uad00\ub828 \uc9c1\uc6d0\uc744 \uacbd\ucc30\uccad\uc774 \ud761\uc218\ud558\uace0 \uae30\uc220\uacfc \ubc29\ubc95\uc744 \uac00\ub974\uccd0 \uc900\ub2e4\uace0 \ud55c\ub4e4 \uc774\uac83\uc774 \uc26c\uc6b4 \uc77c\uc774 \uc544\ub2c8\uba70 \ud574\uc678 \uc815\ubcf4\uae30\uad00\uacfc\uc758 \ud611\ub825\ub3c4 \uc55e\uc73c\ub85c \ubb38\uc81c\uac00 \ub420 \uc218 \uc788\ub2e4. \ub610\ud55c \uc774 \uac1c\ud601\uc548\uc744 \uad00\ucca0\uc2dc\ud0a4\uae30 \uc704\ud574\uc120 \u300c\uad6d\uac00\uc815\ubcf4\uc6d0\ubc95\u300d \ub4f1\uc744 \ube44\ub86f\ud55c \ucd5c\uc18c 6\uac1c\uc758 \ubc95\ub960\uc744 \uac1c\uc815\ud574\uc57c \ud558\ubbc0\ub85c \uad6d\ud68c\uc758 \ud611\uc870\uac00 \ud544\uc218\uc801\uc774\ub2e4. \ud558\uc9c0\ub9cc \uc81c1\uc57c\ub2f9\uc778 \uc790\uc720\ud55c\uad6d\ub2f9\uc740 \"\uad8c\ub825\uae30\uad00\uc744 \uc218\uc871\ucc98\ub7fc \ubd80\ub9ac\uaca0\ub2e4\ub294 \uac1c\uc57d\uc774\uc790 \uad6d\ud68c\uc5d0 \ub300\ud55c \uccad\uc640\ub300\uc758 \uac00\uc774\ub4dc\ub77c\uc778\"\uc774\ub77c\uba70 \uc218\uc6a9\ubd88\uac00\ub97c \ubd84\uba85\ud788 \ud588\ub2e4. \ubc95\uc81c\uc0ac\ubc95\uc704\uc6d0\ud68c \ud55c\uad6d\ub2f9 \uac04\uc0ac\uc778 \uae40\uc9c4\ud0dc \uc758\uc6d0\uc740 \"\uac80\uacbd \uc218\uc0ac\uad8c \uc870\uc815\uc5d0 \ubc18\ub300\ud558\ub294 \uac8c \uc544\ub2c8\uace0, \ud544\uc694\ud558\ub2e4\uba74 \uac80\ucc30 \uac1c\ud601\uc744 \uc704\ud574 \ub354 \uac15\ub3c4 \ub192\uc740 \uc218\uc220\ub3c4 \uc774\ub904\uc9c8 \uc218 \uc788\ub2e4\ub294 \uc785\uc7a5\uc778\ub370 \uccad\uc640\ub300\uac00 \uadf8\ub9bc\uc744 \ub2e4 \uadf8\ub824 \ub193\uace0 \ub530\ub77c\uc624\ub77c\uace0 \ud558\uba74 \uad6d\ud68c\ub294 \ubb50\uac00 \ub418\ub290\ub0d0\"\uba70 \ubc18\ubc1c\ud588\ub2e4. \uae40\uc131\ud0dc \ud55c\uad6d\ub2f9 \uc6d0\ub0b4\ub300\ud45c\ub3c4 \"\ub300\uc758\uae30\uad00\uc778 \uad6d\ud68c\uc758 \ub17c\uc758 \uc790\uccb4\ub97c \uac70\ub4e4\ub5a0\ubcf4\uc9c0 \uc54a\uaca0\ub2e4\ub294 \uc77c\ubc29\ud1b5\ud589\uc2dd \uad6d\uc815\uc6b4\uc601\"\uc774\ub77c\uba70 '\uad6d\ud68c\ud328\uc2f1'\uc774\ub77c\uace0 \ube44\ud310\ud588\ub2e4.", "answer": "\uc77c\ubc18\uacbd\ucc30", "sentence": "\ud55c\ud3b8, \uac80\ucc30\uacfc \uad6d\uc815\uc6d0\uc758 \ud575\uc2ec \uad8c\ud55c\uc744 \uc774\uad00\ubc1b\ub294 \uacbd\ucc30\uc740 \uacbd\ucc30\uad8c \uacac\uc81c\ub97c \uc704\ud574 \uad6d\uac00\uacbd\ucc30\uacfc \uc790\uce58\uacbd\ucc30\ub85c \ubd84\ub9ac\ud558\uba70 \uce58\uc548\u00b7\uacbd\ube44\u00b7\uc815\ubcf4\ub97c \ub2f4\ub2f9\ud558\ub294 \uc77c\ubc18\uacbd\ucc30 \uacfc 1\ucc28\uc801 \uc218\uc0ac\ub97c \ub2f4\ub2f9\ud558\ub294 \uc218\uc0ac\uacbd\ucc30\ub85c \uad6c\ubd84\ud558\uae30\ub85c \ud588\ub2e4.", "paragraph_sentence": "2018\ub144 1\uc6d4 14\uc77c \uccad\uc640\ub300\ub294 \uad6d\uc815\uc6d0\uacfc \uac80\ucc30\uc758 \uad8c\ud55c\uc744 \uacbd\ucc30\ub85c \uc774\uad00\ud558\ub294 \ub0b4\uc6a9\uc744 \ub2f4\uc740 \uad8c\ub825\uae30\uad00 \uac1c\ud3b8\ubc29\uc548\uc744 \ubc1c\ud45c\ud588\ub2e4. \uac80\ucc30\uc740 \uc8fc\uc694 \uc0ac\uac74\uc758 1\ucc28\uc801 \uc218\uc0ac\ub294 \uacbd\ucc30\uccad\uc73c\ub85c, \uace0\uc704\uacf5\uc9c1\uc790 \uc218\uc0ac \ubc0f \uae30\uc18c \uad8c\ud55c\uc740 \uacf5\uc218\ucc98\ub97c \uc2e0\uc124\ud574 \ub9e1\ub3c4\ub85d \ud588\uc73c\uba70 \uc9c1\uc811 \uc218\uc0ac\ub294 \uacbd\uc81c\u00b7\uae08\uc735 \uac19\uc740 \ud2b9\uc218\uc218\uc0ac\ub85c \ud55c\uc815\ud588\ub2e4. \uad6d\uc815\uc6d0\uc740 \ub300\uacf5\uc218\uc0ac\uad8c\uc744 \uacbd\ucc30\uccad \uc0b0\ud558\uc5d0 \ub3c5\ub9bd\ub41c \uc548\ubcf4\uc870\uc9c1\uc744 \ub9cc\ub4e4\uc5b4 \uc774\uad00\ud558\uba70 \uad6d\ub0b4 \uc815\ubcf4 \ud65c\ub3d9\uc744 \ud3d0\uc9c0\ud558\uace0 \ub300\ubd81\u00b7\ud574\uc678 \uc815\ubcf4 \ubd80\uc11c\ub85c\ub9cc \uc5ed\ud560\uc744 \uc904\uc774\uae30\ub85c \ud588\ub2e4. \ud55c\ud3b8, \uac80\ucc30\uacfc \uad6d\uc815\uc6d0\uc758 \ud575\uc2ec \uad8c\ud55c\uc744 \uc774\uad00\ubc1b\ub294 \uacbd\ucc30\uc740 \uacbd\ucc30\uad8c \uacac\uc81c\ub97c \uc704\ud574 \uad6d\uac00\uacbd\ucc30\uacfc \uc790\uce58\uacbd\ucc30\ub85c \ubd84\ub9ac\ud558\uba70 \uce58\uc548\u00b7\uacbd\ube44\u00b7\uc815\ubcf4\ub97c \ub2f4\ub2f9\ud558\ub294 \uc77c\ubc18\uacbd\ucc30 \uacfc 1\ucc28\uc801 \uc218\uc0ac\ub97c \ub2f4\ub2f9\ud558\ub294 \uc218\uc0ac\uacbd\ucc30\ub85c \uad6c\ubd84\ud558\uae30\ub85c \ud588\ub2e4. \ud558\uc9c0\ub9cc \uc774\uc5d0 \ub300\ud55c \ubc18\ubc1c\ub3c4 \ub098\uc654\ub2e4. \ud2b9\ud788 \uad6d\uc815\uc6d0\uc758 \ub300\uacf5\uc218\uc0ac\uad8c\uc744 \uacbd\ucc30\uc5d0 \uc774\uad00\ud558\ub294 \uac83\uc5d0 \ub300\ud55c \uc774\uacac\uc774 \ub9ce\uc558\ub294\ub370 \uad6d\uc815\uc6d0\uc740 \ub300\uacf5\uc218\uc0ac\uc5d0 \ub300\ud55c \ubaa8\ub4e0 \uae30\ub2a5, \ub178\ud558\uc6b0, \ucca9\ubcf4\ub9dd\uc744 \uac00\uc9c0\uace0 \uc788\uc9c0\ub9cc \uacbd\ucc30\uc740 \uc774\uac83\uc774 \uc5c6\uc5b4 \ud55c\uacc4\uac00 \uc788\ub2e4\ub294 \uac83\uc774\ub2e4. \uad00\ub828 \uc9c1\uc6d0\uc744 \uacbd\ucc30\uccad\uc774 \ud761\uc218\ud558\uace0 \uae30\uc220\uacfc \ubc29\ubc95\uc744 \uac00\ub974\uccd0 \uc900\ub2e4\uace0 \ud55c\ub4e4 \uc774\uac83\uc774 \uc26c\uc6b4 \uc77c\uc774 \uc544\ub2c8\uba70 \ud574\uc678 \uc815\ubcf4\uae30\uad00\uacfc\uc758 \ud611\ub825\ub3c4 \uc55e\uc73c\ub85c \ubb38\uc81c\uac00 \ub420 \uc218 \uc788\ub2e4. \ub610\ud55c \uc774 \uac1c\ud601\uc548\uc744 \uad00\ucca0\uc2dc\ud0a4\uae30 \uc704\ud574\uc120 \u300c\uad6d\uac00\uc815\ubcf4\uc6d0\ubc95\u300d \ub4f1\uc744 \ube44\ub86f\ud55c \ucd5c\uc18c 6\uac1c\uc758 \ubc95\ub960\uc744 \uac1c\uc815\ud574\uc57c \ud558\ubbc0\ub85c \uad6d\ud68c\uc758 \ud611\uc870\uac00 \ud544\uc218\uc801\uc774\ub2e4. \ud558\uc9c0\ub9cc \uc81c1\uc57c\ub2f9\uc778 \uc790\uc720\ud55c\uad6d\ub2f9\uc740 \"\uad8c\ub825\uae30\uad00\uc744 \uc218\uc871\ucc98\ub7fc \ubd80\ub9ac\uaca0\ub2e4\ub294 \uac1c\uc57d\uc774\uc790 \uad6d\ud68c\uc5d0 \ub300\ud55c \uccad\uc640\ub300\uc758 \uac00\uc774\ub4dc\ub77c\uc778\"\uc774\ub77c\uba70 \uc218\uc6a9\ubd88\uac00\ub97c \ubd84\uba85\ud788 \ud588\ub2e4. \ubc95\uc81c\uc0ac\ubc95\uc704\uc6d0\ud68c \ud55c\uad6d\ub2f9 \uac04\uc0ac\uc778 \uae40\uc9c4\ud0dc \uc758\uc6d0\uc740 \"\uac80\uacbd \uc218\uc0ac\uad8c \uc870\uc815\uc5d0 \ubc18\ub300\ud558\ub294 \uac8c \uc544\ub2c8\uace0, \ud544\uc694\ud558\ub2e4\uba74 \uac80\ucc30 \uac1c\ud601\uc744 \uc704\ud574 \ub354 \uac15\ub3c4 \ub192\uc740 \uc218\uc220\ub3c4 \uc774\ub904\uc9c8 \uc218 \uc788\ub2e4\ub294 \uc785\uc7a5\uc778\ub370 \uccad\uc640\ub300\uac00 \uadf8\ub9bc\uc744 \ub2e4 \uadf8\ub824 \ub193\uace0 \ub530\ub77c\uc624\ub77c\uace0 \ud558\uba74 \uad6d\ud68c\ub294 \ubb50\uac00 \ub418\ub290\ub0d0\"\uba70 \ubc18\ubc1c\ud588\ub2e4. \uae40\uc131\ud0dc \ud55c\uad6d\ub2f9 \uc6d0\ub0b4\ub300\ud45c\ub3c4 \"\ub300\uc758\uae30\uad00\uc778 \uad6d\ud68c\uc758 \ub17c\uc758 \uc790\uccb4\ub97c \uac70\ub4e4\ub5a0\ubcf4\uc9c0 \uc54a\uaca0\ub2e4\ub294 \uc77c\ubc29\ud1b5\ud589\uc2dd \uad6d\uc815\uc6b4\uc601\"\uc774\ub77c\uba70 '\uad6d\ud68c\ud328\uc2f1'\uc774\ub77c\uace0 \ube44\ud310\ud588\ub2e4.", "paragraph_answer": "2018\ub144 1\uc6d4 14\uc77c \uccad\uc640\ub300\ub294 \uad6d\uc815\uc6d0\uacfc \uac80\ucc30\uc758 \uad8c\ud55c\uc744 \uacbd\ucc30\ub85c \uc774\uad00\ud558\ub294 \ub0b4\uc6a9\uc744 \ub2f4\uc740 \uad8c\ub825\uae30\uad00 \uac1c\ud3b8\ubc29\uc548\uc744 \ubc1c\ud45c\ud588\ub2e4. \uac80\ucc30\uc740 \uc8fc\uc694 \uc0ac\uac74\uc758 1\ucc28\uc801 \uc218\uc0ac\ub294 \uacbd\ucc30\uccad\uc73c\ub85c, \uace0\uc704\uacf5\uc9c1\uc790 \uc218\uc0ac \ubc0f \uae30\uc18c \uad8c\ud55c\uc740 \uacf5\uc218\ucc98\ub97c \uc2e0\uc124\ud574 \ub9e1\ub3c4\ub85d \ud588\uc73c\uba70 \uc9c1\uc811 \uc218\uc0ac\ub294 \uacbd\uc81c\u00b7\uae08\uc735 \uac19\uc740 \ud2b9\uc218\uc218\uc0ac\ub85c \ud55c\uc815\ud588\ub2e4. \uad6d\uc815\uc6d0\uc740 \ub300\uacf5\uc218\uc0ac\uad8c\uc744 \uacbd\ucc30\uccad \uc0b0\ud558\uc5d0 \ub3c5\ub9bd\ub41c \uc548\ubcf4\uc870\uc9c1\uc744 \ub9cc\ub4e4\uc5b4 \uc774\uad00\ud558\uba70 \uad6d\ub0b4 \uc815\ubcf4 \ud65c\ub3d9\uc744 \ud3d0\uc9c0\ud558\uace0 \ub300\ubd81\u00b7\ud574\uc678 \uc815\ubcf4 \ubd80\uc11c\ub85c\ub9cc \uc5ed\ud560\uc744 \uc904\uc774\uae30\ub85c \ud588\ub2e4. \ud55c\ud3b8, \uac80\ucc30\uacfc \uad6d\uc815\uc6d0\uc758 \ud575\uc2ec \uad8c\ud55c\uc744 \uc774\uad00\ubc1b\ub294 \uacbd\ucc30\uc740 \uacbd\ucc30\uad8c \uacac\uc81c\ub97c \uc704\ud574 \uad6d\uac00\uacbd\ucc30\uacfc \uc790\uce58\uacbd\ucc30\ub85c \ubd84\ub9ac\ud558\uba70 \uce58\uc548\u00b7\uacbd\ube44\u00b7\uc815\ubcf4\ub97c \ub2f4\ub2f9\ud558\ub294 \uc77c\ubc18\uacbd\ucc30 \uacfc 1\ucc28\uc801 \uc218\uc0ac\ub97c \ub2f4\ub2f9\ud558\ub294 \uc218\uc0ac\uacbd\ucc30\ub85c \uad6c\ubd84\ud558\uae30\ub85c \ud588\ub2e4. \ud558\uc9c0\ub9cc \uc774\uc5d0 \ub300\ud55c \ubc18\ubc1c\ub3c4 \ub098\uc654\ub2e4. \ud2b9\ud788 \uad6d\uc815\uc6d0\uc758 \ub300\uacf5\uc218\uc0ac\uad8c\uc744 \uacbd\ucc30\uc5d0 \uc774\uad00\ud558\ub294 \uac83\uc5d0 \ub300\ud55c \uc774\uacac\uc774 \ub9ce\uc558\ub294\ub370 \uad6d\uc815\uc6d0\uc740 \ub300\uacf5\uc218\uc0ac\uc5d0 \ub300\ud55c \ubaa8\ub4e0 \uae30\ub2a5, \ub178\ud558\uc6b0, \ucca9\ubcf4\ub9dd\uc744 \uac00\uc9c0\uace0 \uc788\uc9c0\ub9cc \uacbd\ucc30\uc740 \uc774\uac83\uc774 \uc5c6\uc5b4 \ud55c\uacc4\uac00 \uc788\ub2e4\ub294 \uac83\uc774\ub2e4. \uad00\ub828 \uc9c1\uc6d0\uc744 \uacbd\ucc30\uccad\uc774 \ud761\uc218\ud558\uace0 \uae30\uc220\uacfc \ubc29\ubc95\uc744 \uac00\ub974\uccd0 \uc900\ub2e4\uace0 \ud55c\ub4e4 \uc774\uac83\uc774 \uc26c\uc6b4 \uc77c\uc774 \uc544\ub2c8\uba70 \ud574\uc678 \uc815\ubcf4\uae30\uad00\uacfc\uc758 \ud611\ub825\ub3c4 \uc55e\uc73c\ub85c \ubb38\uc81c\uac00 \ub420 \uc218 \uc788\ub2e4. \ub610\ud55c \uc774 \uac1c\ud601\uc548\uc744 \uad00\ucca0\uc2dc\ud0a4\uae30 \uc704\ud574\uc120 \u300c\uad6d\uac00\uc815\ubcf4\uc6d0\ubc95\u300d \ub4f1\uc744 \ube44\ub86f\ud55c \ucd5c\uc18c 6\uac1c\uc758 \ubc95\ub960\uc744 \uac1c\uc815\ud574\uc57c \ud558\ubbc0\ub85c \uad6d\ud68c\uc758 \ud611\uc870\uac00 \ud544\uc218\uc801\uc774\ub2e4. \ud558\uc9c0\ub9cc \uc81c1\uc57c\ub2f9\uc778 \uc790\uc720\ud55c\uad6d\ub2f9\uc740 \"\uad8c\ub825\uae30\uad00\uc744 \uc218\uc871\ucc98\ub7fc \ubd80\ub9ac\uaca0\ub2e4\ub294 \uac1c\uc57d\uc774\uc790 \uad6d\ud68c\uc5d0 \ub300\ud55c \uccad\uc640\ub300\uc758 \uac00\uc774\ub4dc\ub77c\uc778\"\uc774\ub77c\uba70 \uc218\uc6a9\ubd88\uac00\ub97c \ubd84\uba85\ud788 \ud588\ub2e4. \ubc95\uc81c\uc0ac\ubc95\uc704\uc6d0\ud68c \ud55c\uad6d\ub2f9 \uac04\uc0ac\uc778 \uae40\uc9c4\ud0dc \uc758\uc6d0\uc740 \"\uac80\uacbd \uc218\uc0ac\uad8c \uc870\uc815\uc5d0 \ubc18\ub300\ud558\ub294 \uac8c \uc544\ub2c8\uace0, \ud544\uc694\ud558\ub2e4\uba74 \uac80\ucc30 \uac1c\ud601\uc744 \uc704\ud574 \ub354 \uac15\ub3c4 \ub192\uc740 \uc218\uc220\ub3c4 \uc774\ub904\uc9c8 \uc218 \uc788\ub2e4\ub294 \uc785\uc7a5\uc778\ub370 \uccad\uc640\ub300\uac00 \uadf8\ub9bc\uc744 \ub2e4 \uadf8\ub824 \ub193\uace0 \ub530\ub77c\uc624\ub77c\uace0 \ud558\uba74 \uad6d\ud68c\ub294 \ubb50\uac00 \ub418\ub290\ub0d0\"\uba70 \ubc18\ubc1c\ud588\ub2e4. \uae40\uc131\ud0dc \ud55c\uad6d\ub2f9 \uc6d0\ub0b4\ub300\ud45c\ub3c4 \"\ub300\uc758\uae30\uad00\uc778 \uad6d\ud68c\uc758 \ub17c\uc758 \uc790\uccb4\ub97c \uac70\ub4e4\ub5a0\ubcf4\uc9c0 \uc54a\uaca0\ub2e4\ub294 \uc77c\ubc29\ud1b5\ud589\uc2dd \uad6d\uc815\uc6b4\uc601\"\uc774\ub77c\uba70 '\uad6d\ud68c\ud328\uc2f1'\uc774\ub77c\uace0 \ube44\ud310\ud588\ub2e4.", "sentence_answer": "\ud55c\ud3b8, \uac80\ucc30\uacfc \uad6d\uc815\uc6d0\uc758 \ud575\uc2ec \uad8c\ud55c\uc744 \uc774\uad00\ubc1b\ub294 \uacbd\ucc30\uc740 \uacbd\ucc30\uad8c \uacac\uc81c\ub97c \uc704\ud574 \uad6d\uac00\uacbd\ucc30\uacfc \uc790\uce58\uacbd\ucc30\ub85c \ubd84\ub9ac\ud558\uba70 \uce58\uc548\u00b7\uacbd\ube44\u00b7\uc815\ubcf4\ub97c \ub2f4\ub2f9\ud558\ub294 \uc77c\ubc18\uacbd\ucc30 \uacfc 1\ucc28\uc801 \uc218\uc0ac\ub97c \ub2f4\ub2f9\ud558\ub294 \uc218\uc0ac\uacbd\ucc30\ub85c \uad6c\ubd84\ud558\uae30\ub85c \ud588\ub2e4.", "paragraph_id": "6580768-71-1", "paragraph_question": "question: \uad8c\ub825\uae30\uad00 \uac1c\ud3b8\ubc29\uc548\uc5d0 \uc758\ud574 \uce58\uc548, \uacbd\ube44, \uc815\ubcf4\ub97c \ub2f4\ub2f9\ud558\ub294 \uacbd\ucc30\uc740 \ub204\uad6c\uc778\uac00?, context: 2018\ub144 1\uc6d4 14\uc77c \uccad\uc640\ub300\ub294 \uad6d\uc815\uc6d0\uacfc \uac80\ucc30\uc758 \uad8c\ud55c\uc744 \uacbd\ucc30\ub85c \uc774\uad00\ud558\ub294 \ub0b4\uc6a9\uc744 \ub2f4\uc740 \uad8c\ub825\uae30\uad00 \uac1c\ud3b8\ubc29\uc548\uc744 \ubc1c\ud45c\ud588\ub2e4. \uac80\ucc30\uc740 \uc8fc\uc694 \uc0ac\uac74\uc758 1\ucc28\uc801 \uc218\uc0ac\ub294 \uacbd\ucc30\uccad\uc73c\ub85c, \uace0\uc704\uacf5\uc9c1\uc790 \uc218\uc0ac \ubc0f \uae30\uc18c \uad8c\ud55c\uc740 \uacf5\uc218\ucc98\ub97c \uc2e0\uc124\ud574 \ub9e1\ub3c4\ub85d \ud588\uc73c\uba70 \uc9c1\uc811 \uc218\uc0ac\ub294 \uacbd\uc81c\u00b7\uae08\uc735 \uac19\uc740 \ud2b9\uc218\uc218\uc0ac\ub85c \ud55c\uc815\ud588\ub2e4. \uad6d\uc815\uc6d0\uc740 \ub300\uacf5\uc218\uc0ac\uad8c\uc744 \uacbd\ucc30\uccad \uc0b0\ud558\uc5d0 \ub3c5\ub9bd\ub41c \uc548\ubcf4\uc870\uc9c1\uc744 \ub9cc\ub4e4\uc5b4 \uc774\uad00\ud558\uba70 \uad6d\ub0b4 \uc815\ubcf4 \ud65c\ub3d9\uc744 \ud3d0\uc9c0\ud558\uace0 \ub300\ubd81\u00b7\ud574\uc678 \uc815\ubcf4 \ubd80\uc11c\ub85c\ub9cc \uc5ed\ud560\uc744 \uc904\uc774\uae30\ub85c \ud588\ub2e4. \ud55c\ud3b8, \uac80\ucc30\uacfc \uad6d\uc815\uc6d0\uc758 \ud575\uc2ec \uad8c\ud55c\uc744 \uc774\uad00\ubc1b\ub294 \uacbd\ucc30\uc740 \uacbd\ucc30\uad8c \uacac\uc81c\ub97c \uc704\ud574 \uad6d\uac00\uacbd\ucc30\uacfc \uc790\uce58\uacbd\ucc30\ub85c \ubd84\ub9ac\ud558\uba70 \uce58\uc548\u00b7\uacbd\ube44\u00b7\uc815\ubcf4\ub97c \ub2f4\ub2f9\ud558\ub294 \uc77c\ubc18\uacbd\ucc30\uacfc 1\ucc28\uc801 \uc218\uc0ac\ub97c \ub2f4\ub2f9\ud558\ub294 \uc218\uc0ac\uacbd\ucc30\ub85c \uad6c\ubd84\ud558\uae30\ub85c \ud588\ub2e4. \ud558\uc9c0\ub9cc \uc774\uc5d0 \ub300\ud55c \ubc18\ubc1c\ub3c4 \ub098\uc654\ub2e4. \ud2b9\ud788 \uad6d\uc815\uc6d0\uc758 \ub300\uacf5\uc218\uc0ac\uad8c\uc744 \uacbd\ucc30\uc5d0 \uc774\uad00\ud558\ub294 \uac83\uc5d0 \ub300\ud55c \uc774\uacac\uc774 \ub9ce\uc558\ub294\ub370 \uad6d\uc815\uc6d0\uc740 \ub300\uacf5\uc218\uc0ac\uc5d0 \ub300\ud55c \ubaa8\ub4e0 \uae30\ub2a5, \ub178\ud558\uc6b0, \ucca9\ubcf4\ub9dd\uc744 \uac00\uc9c0\uace0 \uc788\uc9c0\ub9cc \uacbd\ucc30\uc740 \uc774\uac83\uc774 \uc5c6\uc5b4 \ud55c\uacc4\uac00 \uc788\ub2e4\ub294 \uac83\uc774\ub2e4. \uad00\ub828 \uc9c1\uc6d0\uc744 \uacbd\ucc30\uccad\uc774 \ud761\uc218\ud558\uace0 \uae30\uc220\uacfc \ubc29\ubc95\uc744 \uac00\ub974\uccd0 \uc900\ub2e4\uace0 \ud55c\ub4e4 \uc774\uac83\uc774 \uc26c\uc6b4 \uc77c\uc774 \uc544\ub2c8\uba70 \ud574\uc678 \uc815\ubcf4\uae30\uad00\uacfc\uc758 \ud611\ub825\ub3c4 \uc55e\uc73c\ub85c \ubb38\uc81c\uac00 \ub420 \uc218 \uc788\ub2e4. \ub610\ud55c \uc774 \uac1c\ud601\uc548\uc744 \uad00\ucca0\uc2dc\ud0a4\uae30 \uc704\ud574\uc120 \u300c\uad6d\uac00\uc815\ubcf4\uc6d0\ubc95\u300d \ub4f1\uc744 \ube44\ub86f\ud55c \ucd5c\uc18c 6\uac1c\uc758 \ubc95\ub960\uc744 \uac1c\uc815\ud574\uc57c \ud558\ubbc0\ub85c \uad6d\ud68c\uc758 \ud611\uc870\uac00 \ud544\uc218\uc801\uc774\ub2e4. \ud558\uc9c0\ub9cc \uc81c1\uc57c\ub2f9\uc778 \uc790\uc720\ud55c\uad6d\ub2f9\uc740 \"\uad8c\ub825\uae30\uad00\uc744 \uc218\uc871\ucc98\ub7fc \ubd80\ub9ac\uaca0\ub2e4\ub294 \uac1c\uc57d\uc774\uc790 \uad6d\ud68c\uc5d0 \ub300\ud55c \uccad\uc640\ub300\uc758 \uac00\uc774\ub4dc\ub77c\uc778\"\uc774\ub77c\uba70 \uc218\uc6a9\ubd88\uac00\ub97c \ubd84\uba85\ud788 \ud588\ub2e4. \ubc95\uc81c\uc0ac\ubc95\uc704\uc6d0\ud68c \ud55c\uad6d\ub2f9 \uac04\uc0ac\uc778 \uae40\uc9c4\ud0dc \uc758\uc6d0\uc740 \"\uac80\uacbd \uc218\uc0ac\uad8c \uc870\uc815\uc5d0 \ubc18\ub300\ud558\ub294 \uac8c \uc544\ub2c8\uace0, \ud544\uc694\ud558\ub2e4\uba74 \uac80\ucc30 \uac1c\ud601\uc744 \uc704\ud574 \ub354 \uac15\ub3c4 \ub192\uc740 \uc218\uc220\ub3c4 \uc774\ub904\uc9c8 \uc218 \uc788\ub2e4\ub294 \uc785\uc7a5\uc778\ub370 \uccad\uc640\ub300\uac00 \uadf8\ub9bc\uc744 \ub2e4 \uadf8\ub824 \ub193\uace0 \ub530\ub77c\uc624\ub77c\uace0 \ud558\uba74 \uad6d\ud68c\ub294 \ubb50\uac00 \ub418\ub290\ub0d0\"\uba70 \ubc18\ubc1c\ud588\ub2e4. \uae40\uc131\ud0dc \ud55c\uad6d\ub2f9 \uc6d0\ub0b4\ub300\ud45c\ub3c4 \"\ub300\uc758\uae30\uad00\uc778 \uad6d\ud68c\uc758 \ub17c\uc758 \uc790\uccb4\ub97c \uac70\ub4e4\ub5a0\ubcf4\uc9c0 \uc54a\uaca0\ub2e4\ub294 \uc77c\ubc29\ud1b5\ud589\uc2dd \uad6d\uc815\uc6b4\uc601\"\uc774\ub77c\uba70 '\uad6d\ud68c\ud328\uc2f1'\uc774\ub77c\uace0 \ube44\ud310\ud588\ub2e4."} {"question": "\uc2dc\uba3c\uc2a4\uc758 \uacf5\uc5f0\uc744 \ubcf4\uace0 \uacf5\ub3d9\uad00\ub9ac\uc5d0 \ub3d9\uc758\ud55c \uc0ac\ub78c\uc740?", "paragraph": "2002\ub144 14\uc0b4\uc758 \ub098\uc774\ub85c \uadf8\uc758 \uba58\ud1a0\uc774\uc790 \uacf5\ub3d9 \uacbd\uc601\uc790\uc778 \ube44. \ub9ac\uce58(B. Rich)\ub97c \ub9cc\ub09c \ud6c4\uc5d0, B.o.B\ub294 \uc804 \uc2ac\ub9bd\uc5d4 \uc2ac\ub77c\uc774\ub4dc \ub9ac\ucf54\ub529(lip-n-Slide recording)\uc758 \uc544\ud2f0\uc2a4\ud2b8 \uc528\ud2f0(Citti)\uc758 \uace1 \"I'm the Cookie Man.\"\uc5d0 \uc4f0\uc778 \ube44\ud2b8\ub97c \ucc98\uc74c\uc73c\ub85c \ud314\uc558\ub2e4. \uadf8\ub54c B.o.B\ub294 \uc790\uc2e0\uc774 \ud574\ub0c8\ub2e4\ub294 \uc131\ucde8\uac10\uc744 \uc774\ub807\uac8c \ud45c\ud604 \ud588\ub2e4. : \u201c\ub098\ub294 \ub098\uac00\uc11c \ub0b4 \ubaa8\ub4e0 \ub3c8\uc744 \uc21c\uc2dd\uac04\uc5d0 \ub2e4 \uc368\ubc84\ub838\uc9c0. \uae08\ubc29 \ub2e4\uc2dc \ub098\ub294 \ube48\ud138\ud130\ub9ac\uac00 \ub410\uc9c0\ub9cc \ub450 \uac00\uc9c0 \uc911\uc694\ud55c \uac83\uc744 \uae68\ub2ec\uc558\uc9c0. \uc6b0\uc120\uc740 \ub3c8\uc744 \uc544\uaef4\uc57c \ud588\uace0 \uc74c\uc545\uc5d0 \ud760\ubed1 \ube60\uc838\uc57c \ud55c\ub2e4\ub294 \uac83\uc774\uc5c8\uc9c0.\u201d \ub2e4\uc2dc \uc6d0\uc704\uce58\ub85c \ub3cc\uc544\uac00, B.o.B\ub294 \uc790\uc2e0\uc758 \uae30\uc220\uc744 \uc644\ubcbd\ud558\uac8c \ud558\uae30 \uc704\ud574 \uacf5\uc5f0\uc758 \uc624\ud504\ub2dd \ubb34\ub300\uc5d0 \uc11c\uac70\ub098 \uc5b8\ub354\uadf8\ub77c\uc6b4\ub4dc\uc5d0\uc11c \ud65c\ub3d9\uc744 \uc9c0\uc18d\ud588\ub2e4. 2006\ub144 B. \ub9ac\uce58\ub294 \ubbf8\uc131\ub144\uc790\uc778 B.o.B\uac00 \uc560\ud2c0\ub79c\ud0c0 \ub7a9\ud37c T.I. \uc18c\uc720\uc778 \ub098\uc774\ud2b8\ud074\ub7fd \ud074\ub7fd \ud06c\ub8e8\uc15c(Club Crucial)\uc5d0 \uc77c\ud560 \uc218 \uc788\uac8c \ub3c4\uc640\uc92c\ub2e4. \uadf8\uacf3\uc5d0\uc11c B.o.B\ub294 \uc790\uc2e0\uc774 \ud504\ub85c\ub4c0\uc2a4\ud55c \uc2a4\ud3ec\ud070 \uc6cc\ub4dc \ud615\ud0dc\uc758 \ub9c8\ub9ac\ud558\ub098\uc758 \uc1a1\uac00\uc778 \"Cloud 9\"\uc73c\ub85c \uacf5\uc5f0\uc744 \ud588\ub2e4. \uadf8\uc758 \uacf5\uc5f0\uc744 \ubcf8 \uad00\uac1d \uc911\uc5d0\ub294 \ud504\ub85c\ub4c0\uc11c\uc774\uc790 \uc0c1\uc5c5 \uc74c\uc545\uc758 \ubca0\ud14c\ub791\uc778 TJ's DJ'S\uc758 CEO\uc778 T.J. \ucc44\ud504\uba3c(T.J. Chapman)\uc774 \uc788\uc5c8\ub2e4. \ucc44\ud504\uba3c\uc740 B.o.B\ub97c \uacf5\ub3d9\uad00\ub9ac\ud558\ub294 \uac83\uc5d0 \ub3d9\uc758\ud588\uace0 \uba87 \ub2ec \ub4a4 B.o.B\ub294 \uc560\ud2c0\ub79c\ud2f1 \ub808\ucf54\ub4dc\uc640 \ud50c\ub85c\ub9ac\ub2e4 \ud504\ub85c\ub4c0\uc11c \uc9d0 \uc874\uc2e0(Jim Jonsin)\uc758 \uc6b4\uc601\ub418\ub294 \ubc1c\ub9e4 \uacc4\uc5f4\uc0ac\uc778 \ub808\ubc8c \ub85d\uacfc \uacc4\uc57d\uc744 \ub9fa\uc5c8\ub2e4. \uc560\ud2c0\ub79c\ud2f1\uc5d0\uc11c \ubc1c\ub9e4\ub41c \uadf8\uc758 2007\ub144\uc758 \uccab \ubc88\uc9f8 \uc2f1\uae00 \"Haterz Everywhere\"\uc740 \ube4c\ubcf4\ub4dc\uc758 \ubc84\ube14\ub9c1 \uc5b8\ub354 \uc54c\uc564\ube44/\ud799\ud569 \uc2f1\uae00 \ucc28\ud2b8(U.S. Billboard Bubbling Under Hot 100 Singles)\uc5d0\uc11c \ud0d1 5\ub97c \uae30\ub85d\ud588\ub2e4. \uadf8\uc758 \uc194\ub85c \ud504\ub85c\ub355\uc158 \uacbd\ub825\uc740 B.o.B\ub97c \ud50c\ub808\uc774\ubcf4\uc774 \ud2b8\ub808(Playboy Tre), TJ \ucc44\ud504\uba3c, B. \ub9ac\uce58\uc640 \ud568\uaed8 \ud584 \uc2a4\ucffc\ub4dc(Ham Squad)\ub77c \ubd88\ub9ac\ub294 \ud504\ub85c\ub355\uc158/\ub7a9 \uadf8\ub8f9\uc758 \uc77c\uc6d0\uc774 \ub418\uac8c \ud588\ub2e4.", "answer": "T.J. \ucc44\ud504\uba3c", "sentence": "\uadf8\uc758 \uacf5\uc5f0\uc744 \ubcf8 \uad00\uac1d \uc911\uc5d0\ub294 \ud504\ub85c\ub4c0\uc11c\uc774\uc790 \uc0c1\uc5c5 \uc74c\uc545\uc758 \ubca0\ud14c\ub791\uc778 TJ's DJ'S\uc758 CEO\uc778 T.J. \ucc44\ud504\uba3c (T.J. Chapman)\uc774 \uc788\uc5c8\ub2e4.", "paragraph_sentence": "2002\ub144 14\uc0b4\uc758 \ub098\uc774\ub85c \uadf8\uc758 \uba58\ud1a0\uc774\uc790 \uacf5\ub3d9 \uacbd\uc601\uc790\uc778 \ube44. \ub9ac\uce58(B. Rich)\ub97c \ub9cc\ub09c \ud6c4\uc5d0, B.o. B\ub294 \uc804 \uc2ac\ub9bd\uc5d4 \uc2ac\ub77c\uc774\ub4dc \ub9ac\ucf54\ub529(lip-n-Slide recording)\uc758 \uc544\ud2f0\uc2a4\ud2b8 \uc528\ud2f0(Citti)\uc758 \uace1 \"I'm the Cookie Man. \"\uc5d0 \uc4f0\uc778 \ube44\ud2b8\ub97c \ucc98\uc74c\uc73c\ub85c \ud314\uc558\ub2e4. \uadf8\ub54c B.o. B\ub294 \uc790\uc2e0\uc774 \ud574\ub0c8\ub2e4\ub294 \uc131\ucde8\uac10\uc744 \uc774\ub807\uac8c \ud45c\ud604 \ud588\ub2e4. : \u201c\ub098\ub294 \ub098\uac00\uc11c \ub0b4 \ubaa8\ub4e0 \ub3c8\uc744 \uc21c\uc2dd\uac04\uc5d0 \ub2e4 \uc368\ubc84\ub838\uc9c0. \uae08\ubc29 \ub2e4\uc2dc \ub098\ub294 \ube48\ud138\ud130\ub9ac\uac00 \ub410\uc9c0\ub9cc \ub450 \uac00\uc9c0 \uc911\uc694\ud55c \uac83\uc744 \uae68\ub2ec\uc558\uc9c0. \uc6b0\uc120\uc740 \ub3c8\uc744 \uc544\uaef4\uc57c \ud588\uace0 \uc74c\uc545\uc5d0 \ud760\ubed1 \ube60\uc838\uc57c \ud55c\ub2e4\ub294 \uac83\uc774\uc5c8\uc9c0. \u201d \ub2e4\uc2dc \uc6d0\uc704\uce58\ub85c \ub3cc\uc544\uac00, B.o. B\ub294 \uc790\uc2e0\uc758 \uae30\uc220\uc744 \uc644\ubcbd\ud558\uac8c \ud558\uae30 \uc704\ud574 \uacf5\uc5f0\uc758 \uc624\ud504\ub2dd \ubb34\ub300\uc5d0 \uc11c\uac70\ub098 \uc5b8\ub354\uadf8\ub77c\uc6b4\ub4dc\uc5d0\uc11c \ud65c\ub3d9\uc744 \uc9c0\uc18d\ud588\ub2e4. 2006\ub144 B. \ub9ac\uce58\ub294 \ubbf8\uc131\ub144\uc790\uc778 B.o. B\uac00 \uc560\ud2c0\ub79c\ud0c0 \ub7a9\ud37c T.I. \uc18c\uc720\uc778 \ub098\uc774\ud2b8\ud074\ub7fd \ud074\ub7fd \ud06c\ub8e8\uc15c(Club Crucial)\uc5d0 \uc77c\ud560 \uc218 \uc788\uac8c \ub3c4\uc640\uc92c\ub2e4. \uadf8\uacf3\uc5d0\uc11c B.o. B\ub294 \uc790\uc2e0\uc774 \ud504\ub85c\ub4c0\uc2a4\ud55c \uc2a4\ud3ec\ud070 \uc6cc\ub4dc \ud615\ud0dc\uc758 \ub9c8\ub9ac\ud558\ub098\uc758 \uc1a1\uac00\uc778 \"Cloud 9\"\uc73c\ub85c \uacf5\uc5f0\uc744 \ud588\ub2e4. \uadf8\uc758 \uacf5\uc5f0\uc744 \ubcf8 \uad00\uac1d \uc911\uc5d0\ub294 \ud504\ub85c\ub4c0\uc11c\uc774\uc790 \uc0c1\uc5c5 \uc74c\uc545\uc758 \ubca0\ud14c\ub791\uc778 TJ's DJ'S\uc758 CEO\uc778 T.J. \ucc44\ud504\uba3c (T.J. Chapman)\uc774 \uc788\uc5c8\ub2e4. \ucc44\ud504\uba3c\uc740 B.o. B\ub97c \uacf5\ub3d9\uad00\ub9ac\ud558\ub294 \uac83\uc5d0 \ub3d9\uc758\ud588\uace0 \uba87 \ub2ec \ub4a4 B.o. B\ub294 \uc560\ud2c0\ub79c\ud2f1 \ub808\ucf54\ub4dc\uc640 \ud50c\ub85c\ub9ac\ub2e4 \ud504\ub85c\ub4c0\uc11c \uc9d0 \uc874\uc2e0(Jim Jonsin)\uc758 \uc6b4\uc601\ub418\ub294 \ubc1c\ub9e4 \uacc4\uc5f4\uc0ac\uc778 \ub808\ubc8c \ub85d\uacfc \uacc4\uc57d\uc744 \ub9fa\uc5c8\ub2e4. \uc560\ud2c0\ub79c\ud2f1\uc5d0\uc11c \ubc1c\ub9e4\ub41c \uadf8\uc758 2007\ub144\uc758 \uccab \ubc88\uc9f8 \uc2f1\uae00 \"Haterz Everywhere\"\uc740 \ube4c\ubcf4\ub4dc\uc758 \ubc84\ube14\ub9c1 \uc5b8\ub354 \uc54c\uc564\ube44/\ud799\ud569 \uc2f1\uae00 \ucc28\ud2b8(U.S. Billboard Bubbling Under Hot 100 Singles)\uc5d0\uc11c \ud0d1 5\ub97c \uae30\ub85d\ud588\ub2e4. \uadf8\uc758 \uc194\ub85c \ud504\ub85c\ub355\uc158 \uacbd\ub825\uc740 B.o. B\ub97c \ud50c\ub808\uc774\ubcf4\uc774 \ud2b8\ub808(Playboy Tre), TJ \ucc44\ud504\uba3c, B. \ub9ac\uce58\uc640 \ud568\uaed8 \ud584 \uc2a4\ucffc\ub4dc(Ham Squad)\ub77c \ubd88\ub9ac\ub294 \ud504\ub85c\ub355\uc158/\ub7a9 \uadf8\ub8f9\uc758 \uc77c\uc6d0\uc774 \ub418\uac8c \ud588\ub2e4.", "paragraph_answer": "2002\ub144 14\uc0b4\uc758 \ub098\uc774\ub85c \uadf8\uc758 \uba58\ud1a0\uc774\uc790 \uacf5\ub3d9 \uacbd\uc601\uc790\uc778 \ube44. \ub9ac\uce58(B. Rich)\ub97c \ub9cc\ub09c \ud6c4\uc5d0, B.o.B\ub294 \uc804 \uc2ac\ub9bd\uc5d4 \uc2ac\ub77c\uc774\ub4dc \ub9ac\ucf54\ub529(lip-n-Slide recording)\uc758 \uc544\ud2f0\uc2a4\ud2b8 \uc528\ud2f0(Citti)\uc758 \uace1 \"I'm the Cookie Man.\"\uc5d0 \uc4f0\uc778 \ube44\ud2b8\ub97c \ucc98\uc74c\uc73c\ub85c \ud314\uc558\ub2e4. \uadf8\ub54c B.o.B\ub294 \uc790\uc2e0\uc774 \ud574\ub0c8\ub2e4\ub294 \uc131\ucde8\uac10\uc744 \uc774\ub807\uac8c \ud45c\ud604 \ud588\ub2e4. : \u201c\ub098\ub294 \ub098\uac00\uc11c \ub0b4 \ubaa8\ub4e0 \ub3c8\uc744 \uc21c\uc2dd\uac04\uc5d0 \ub2e4 \uc368\ubc84\ub838\uc9c0. \uae08\ubc29 \ub2e4\uc2dc \ub098\ub294 \ube48\ud138\ud130\ub9ac\uac00 \ub410\uc9c0\ub9cc \ub450 \uac00\uc9c0 \uc911\uc694\ud55c \uac83\uc744 \uae68\ub2ec\uc558\uc9c0. \uc6b0\uc120\uc740 \ub3c8\uc744 \uc544\uaef4\uc57c \ud588\uace0 \uc74c\uc545\uc5d0 \ud760\ubed1 \ube60\uc838\uc57c \ud55c\ub2e4\ub294 \uac83\uc774\uc5c8\uc9c0.\u201d \ub2e4\uc2dc \uc6d0\uc704\uce58\ub85c \ub3cc\uc544\uac00, B.o.B\ub294 \uc790\uc2e0\uc758 \uae30\uc220\uc744 \uc644\ubcbd\ud558\uac8c \ud558\uae30 \uc704\ud574 \uacf5\uc5f0\uc758 \uc624\ud504\ub2dd \ubb34\ub300\uc5d0 \uc11c\uac70\ub098 \uc5b8\ub354\uadf8\ub77c\uc6b4\ub4dc\uc5d0\uc11c \ud65c\ub3d9\uc744 \uc9c0\uc18d\ud588\ub2e4. 2006\ub144 B. \ub9ac\uce58\ub294 \ubbf8\uc131\ub144\uc790\uc778 B.o.B\uac00 \uc560\ud2c0\ub79c\ud0c0 \ub7a9\ud37c T.I. \uc18c\uc720\uc778 \ub098\uc774\ud2b8\ud074\ub7fd \ud074\ub7fd \ud06c\ub8e8\uc15c(Club Crucial)\uc5d0 \uc77c\ud560 \uc218 \uc788\uac8c \ub3c4\uc640\uc92c\ub2e4. \uadf8\uacf3\uc5d0\uc11c B.o.B\ub294 \uc790\uc2e0\uc774 \ud504\ub85c\ub4c0\uc2a4\ud55c \uc2a4\ud3ec\ud070 \uc6cc\ub4dc \ud615\ud0dc\uc758 \ub9c8\ub9ac\ud558\ub098\uc758 \uc1a1\uac00\uc778 \"Cloud 9\"\uc73c\ub85c \uacf5\uc5f0\uc744 \ud588\ub2e4. \uadf8\uc758 \uacf5\uc5f0\uc744 \ubcf8 \uad00\uac1d \uc911\uc5d0\ub294 \ud504\ub85c\ub4c0\uc11c\uc774\uc790 \uc0c1\uc5c5 \uc74c\uc545\uc758 \ubca0\ud14c\ub791\uc778 TJ's DJ'S\uc758 CEO\uc778 T.J. \ucc44\ud504\uba3c (T.J. Chapman)\uc774 \uc788\uc5c8\ub2e4. \ucc44\ud504\uba3c\uc740 B.o.B\ub97c \uacf5\ub3d9\uad00\ub9ac\ud558\ub294 \uac83\uc5d0 \ub3d9\uc758\ud588\uace0 \uba87 \ub2ec \ub4a4 B.o.B\ub294 \uc560\ud2c0\ub79c\ud2f1 \ub808\ucf54\ub4dc\uc640 \ud50c\ub85c\ub9ac\ub2e4 \ud504\ub85c\ub4c0\uc11c \uc9d0 \uc874\uc2e0(Jim Jonsin)\uc758 \uc6b4\uc601\ub418\ub294 \ubc1c\ub9e4 \uacc4\uc5f4\uc0ac\uc778 \ub808\ubc8c \ub85d\uacfc \uacc4\uc57d\uc744 \ub9fa\uc5c8\ub2e4. \uc560\ud2c0\ub79c\ud2f1\uc5d0\uc11c \ubc1c\ub9e4\ub41c \uadf8\uc758 2007\ub144\uc758 \uccab \ubc88\uc9f8 \uc2f1\uae00 \"Haterz Everywhere\"\uc740 \ube4c\ubcf4\ub4dc\uc758 \ubc84\ube14\ub9c1 \uc5b8\ub354 \uc54c\uc564\ube44/\ud799\ud569 \uc2f1\uae00 \ucc28\ud2b8(U.S. Billboard Bubbling Under Hot 100 Singles)\uc5d0\uc11c \ud0d1 5\ub97c \uae30\ub85d\ud588\ub2e4. \uadf8\uc758 \uc194\ub85c \ud504\ub85c\ub355\uc158 \uacbd\ub825\uc740 B.o.B\ub97c \ud50c\ub808\uc774\ubcf4\uc774 \ud2b8\ub808(Playboy Tre), TJ \ucc44\ud504\uba3c, B. \ub9ac\uce58\uc640 \ud568\uaed8 \ud584 \uc2a4\ucffc\ub4dc(Ham Squad)\ub77c \ubd88\ub9ac\ub294 \ud504\ub85c\ub355\uc158/\ub7a9 \uadf8\ub8f9\uc758 \uc77c\uc6d0\uc774 \ub418\uac8c \ud588\ub2e4.", "sentence_answer": "\uadf8\uc758 \uacf5\uc5f0\uc744 \ubcf8 \uad00\uac1d \uc911\uc5d0\ub294 \ud504\ub85c\ub4c0\uc11c\uc774\uc790 \uc0c1\uc5c5 \uc74c\uc545\uc758 \ubca0\ud14c\ub791\uc778 TJ's DJ'S\uc758 CEO\uc778 T.J. \ucc44\ud504\uba3c (T.J. Chapman)\uc774 \uc788\uc5c8\ub2e4.", "paragraph_id": "6482433-2-1", "paragraph_question": "question: \uc2dc\uba3c\uc2a4\uc758 \uacf5\uc5f0\uc744 \ubcf4\uace0 \uacf5\ub3d9\uad00\ub9ac\uc5d0 \ub3d9\uc758\ud55c \uc0ac\ub78c\uc740?, context: 2002\ub144 14\uc0b4\uc758 \ub098\uc774\ub85c \uadf8\uc758 \uba58\ud1a0\uc774\uc790 \uacf5\ub3d9 \uacbd\uc601\uc790\uc778 \ube44. \ub9ac\uce58(B. Rich)\ub97c \ub9cc\ub09c \ud6c4\uc5d0, B.o.B\ub294 \uc804 \uc2ac\ub9bd\uc5d4 \uc2ac\ub77c\uc774\ub4dc \ub9ac\ucf54\ub529(lip-n-Slide recording)\uc758 \uc544\ud2f0\uc2a4\ud2b8 \uc528\ud2f0(Citti)\uc758 \uace1 \"I'm the Cookie Man.\"\uc5d0 \uc4f0\uc778 \ube44\ud2b8\ub97c \ucc98\uc74c\uc73c\ub85c \ud314\uc558\ub2e4. \uadf8\ub54c B.o.B\ub294 \uc790\uc2e0\uc774 \ud574\ub0c8\ub2e4\ub294 \uc131\ucde8\uac10\uc744 \uc774\ub807\uac8c \ud45c\ud604 \ud588\ub2e4. : \u201c\ub098\ub294 \ub098\uac00\uc11c \ub0b4 \ubaa8\ub4e0 \ub3c8\uc744 \uc21c\uc2dd\uac04\uc5d0 \ub2e4 \uc368\ubc84\ub838\uc9c0. \uae08\ubc29 \ub2e4\uc2dc \ub098\ub294 \ube48\ud138\ud130\ub9ac\uac00 \ub410\uc9c0\ub9cc \ub450 \uac00\uc9c0 \uc911\uc694\ud55c \uac83\uc744 \uae68\ub2ec\uc558\uc9c0. \uc6b0\uc120\uc740 \ub3c8\uc744 \uc544\uaef4\uc57c \ud588\uace0 \uc74c\uc545\uc5d0 \ud760\ubed1 \ube60\uc838\uc57c \ud55c\ub2e4\ub294 \uac83\uc774\uc5c8\uc9c0.\u201d \ub2e4\uc2dc \uc6d0\uc704\uce58\ub85c \ub3cc\uc544\uac00, B.o.B\ub294 \uc790\uc2e0\uc758 \uae30\uc220\uc744 \uc644\ubcbd\ud558\uac8c \ud558\uae30 \uc704\ud574 \uacf5\uc5f0\uc758 \uc624\ud504\ub2dd \ubb34\ub300\uc5d0 \uc11c\uac70\ub098 \uc5b8\ub354\uadf8\ub77c\uc6b4\ub4dc\uc5d0\uc11c \ud65c\ub3d9\uc744 \uc9c0\uc18d\ud588\ub2e4. 2006\ub144 B. \ub9ac\uce58\ub294 \ubbf8\uc131\ub144\uc790\uc778 B.o.B\uac00 \uc560\ud2c0\ub79c\ud0c0 \ub7a9\ud37c T.I. \uc18c\uc720\uc778 \ub098\uc774\ud2b8\ud074\ub7fd \ud074\ub7fd \ud06c\ub8e8\uc15c(Club Crucial)\uc5d0 \uc77c\ud560 \uc218 \uc788\uac8c \ub3c4\uc640\uc92c\ub2e4. \uadf8\uacf3\uc5d0\uc11c B.o.B\ub294 \uc790\uc2e0\uc774 \ud504\ub85c\ub4c0\uc2a4\ud55c \uc2a4\ud3ec\ud070 \uc6cc\ub4dc \ud615\ud0dc\uc758 \ub9c8\ub9ac\ud558\ub098\uc758 \uc1a1\uac00\uc778 \"Cloud 9\"\uc73c\ub85c \uacf5\uc5f0\uc744 \ud588\ub2e4. \uadf8\uc758 \uacf5\uc5f0\uc744 \ubcf8 \uad00\uac1d \uc911\uc5d0\ub294 \ud504\ub85c\ub4c0\uc11c\uc774\uc790 \uc0c1\uc5c5 \uc74c\uc545\uc758 \ubca0\ud14c\ub791\uc778 TJ's DJ'S\uc758 CEO\uc778 T.J. \ucc44\ud504\uba3c(T.J. Chapman)\uc774 \uc788\uc5c8\ub2e4. \ucc44\ud504\uba3c\uc740 B.o.B\ub97c \uacf5\ub3d9\uad00\ub9ac\ud558\ub294 \uac83\uc5d0 \ub3d9\uc758\ud588\uace0 \uba87 \ub2ec \ub4a4 B.o.B\ub294 \uc560\ud2c0\ub79c\ud2f1 \ub808\ucf54\ub4dc\uc640 \ud50c\ub85c\ub9ac\ub2e4 \ud504\ub85c\ub4c0\uc11c \uc9d0 \uc874\uc2e0(Jim Jonsin)\uc758 \uc6b4\uc601\ub418\ub294 \ubc1c\ub9e4 \uacc4\uc5f4\uc0ac\uc778 \ub808\ubc8c \ub85d\uacfc \uacc4\uc57d\uc744 \ub9fa\uc5c8\ub2e4. \uc560\ud2c0\ub79c\ud2f1\uc5d0\uc11c \ubc1c\ub9e4\ub41c \uadf8\uc758 2007\ub144\uc758 \uccab \ubc88\uc9f8 \uc2f1\uae00 \"Haterz Everywhere\"\uc740 \ube4c\ubcf4\ub4dc\uc758 \ubc84\ube14\ub9c1 \uc5b8\ub354 \uc54c\uc564\ube44/\ud799\ud569 \uc2f1\uae00 \ucc28\ud2b8(U.S. Billboard Bubbling Under Hot 100 Singles)\uc5d0\uc11c \ud0d1 5\ub97c \uae30\ub85d\ud588\ub2e4. \uadf8\uc758 \uc194\ub85c \ud504\ub85c\ub355\uc158 \uacbd\ub825\uc740 B.o.B\ub97c \ud50c\ub808\uc774\ubcf4\uc774 \ud2b8\ub808(Playboy Tre), TJ \ucc44\ud504\uba3c, B. \ub9ac\uce58\uc640 \ud568\uaed8 \ud584 \uc2a4\ucffc\ub4dc(Ham Squad)\ub77c \ubd88\ub9ac\ub294 \ud504\ub85c\ub355\uc158/\ub7a9 \uadf8\ub8f9\uc758 \uc77c\uc6d0\uc774 \ub418\uac8c \ud588\ub2e4."} {"question": "\uc9c4\ubcf4\uc2e0\ub2f9\uc774 \ucc3d\ub2f9\ub41c \uc2dc\uae30\ub294?", "paragraph": "\uacb0\uad6d 2008\ub144 2\uc6d4 8\uc77c, \uc13c\ud2b8\ub7f4\uc2dc\ud2f0 9\uce35 \ub300\ud68c\uc758\uc2e4\uc5d0\uc11c \uc9c4\ud589\ub41c \ubbfc\uc8fc\ub178\ub3d9\ub2f9 \uc784\uc2dc\ub2f9\ub300\ud68c\uc5d0\uc11c \uc790\uc8fc\ud30c \ub300\uc758\uc6d0\ub4e4\uc758 \ubc18\ub300\ub85c \ud601\uc2e0\uc548\uc758 \ud575\uc2ec \ub0b4\uc6a9\uc740 \ubd80\uacb0\ub418\uc5c8\uc73c\uba70 \uc790\uc8fc\ud30c\uc758 \uc218\uc815\uc548\ub300\ub85c \ud1b5\uacfc\ub418\uc5c8\ub2e4. \ud601\uc2e0\uc548\uc774 \ubd80\uacb0\ub418\uc790 \uc2ec\uc0c1\uc815 \uc704\uc6d0\uc7a5 \ub4f1 \ube44\ub300\uc704\uc6d0 \uc804\uc6d0\uc774 \uc0ac\ud1f4\ud558\uc600\uace0, \ubbfc\uc8fc\ub178\ub3d9\ub2f9 \uc81c1\uae30\uc640 \uc81c2\uae30 \uc9c0\ub3c4\ubd80\ub97c \uad6c\uc131\ud558\uc600\ub358 \uc778\ubb3c\ub4e4 \uac00\uc6b4\ub370 \uae40\ud61c\uacbd, \uc8fc\ub300\ud658, \uae40\uc885\ucca0, \uc2ec\uc7ac\uc625, \ud64d\uc2b9\ud558, \uae40\uae30\uc218 \ub4f1\uc774 \ud0c8\ub2f9\ud558\uc600\uc73c\uba70, \ubbfc\uc8fc\ub178\ub3d9\ub2f9 \uc18c\uc18d \uad6d\ud68c\uc758\uc6d0 \uc2ec\uc0c1\uc815, \ub178\ud68c\ucc2c, \uc870\uc2b9\uc218, \ub2e8\ubcd1\ud638\ub3c4 \ud0c8\ub2f9\ud558\uc600\ub2e4. \uc784\uc2dc\ub2f9\ub300\ud68c \uc774\ud6c4 2008\ub144 4\uc6d4\uae4c\uc9c0 \ubbfc\uc8fc\ub178\ub3d9\ub2f9\uc744 \ud0c8\ub2f9\ud55c \uc0ac\ub78c\uc740 16,904\uba85\uc5d0 \uc774\ub974\ub800\uace0, 2008\ub144 1\uc6d4 \ub2f9\uc2dc 11\ub9cc\uc5ec \uba85\uc5d0 \uc774\ub974\ub358 \ubbfc\uc8fc\ub178\ub3d9\ub2f9\uc758 \ub2f9\uc6d0 \uc218\ub294 9\ub9cc 4\ucc9c\uc5ec \uba85\uc73c\ub85c \uc904\uc5c8\ub2e4. \ubbfc\uc8fc\ub178\ub3d9\ub2f9\uc744 \ud0c8\ub2f9\ud55c \uc0ac\ub78c\ub4e4 \uc911 \uc77c\ubd80\ub294 2008\ub144 2\uc6d4 21\uc77c '\uc9c4\ubcf4\uc2e0\ub2f9\uc5f0\ub300\ud68c\uc758' \uc900\ube44\uc704\uc6d0\ud68c\ub97c \uacb0\uc131\ud558\uc600\uc73c\uba70, 2008\ub144 3\uc6d4 \uc9c4\ubcf4\uc2e0\ub2f9 \ucc3d\ub2f9\uc774 \uacf5\uc2dd \uc120\uc5b8\ub418\uc5c8\uace0, \ubbfc\uc8fc\ub178\ub3d9\ub2f9\uc740 \ubd84\ub2f9\ub418\uc5c8\ub2e4.", "answer": "2008\ub144 3\uc6d4", "sentence": "\ubbfc\uc8fc\ub178\ub3d9\ub2f9\uc744 \ud0c8\ub2f9\ud55c \uc0ac\ub78c\ub4e4 \uc911 \uc77c\ubd80\ub294 2008\ub144 2\uc6d4 21\uc77c '\uc9c4\ubcf4\uc2e0\ub2f9\uc5f0\ub300\ud68c\uc758' \uc900\ube44\uc704\uc6d0\ud68c\ub97c \uacb0\uc131\ud558\uc600\uc73c\uba70, 2008\ub144 3\uc6d4 \uc9c4\ubcf4\uc2e0\ub2f9 \ucc3d\ub2f9\uc774 \uacf5\uc2dd \uc120\uc5b8\ub418\uc5c8\uace0, \ubbfc\uc8fc\ub178\ub3d9\ub2f9\uc740 \ubd84\ub2f9\ub418\uc5c8\ub2e4.", "paragraph_sentence": "\uacb0\uad6d 2008\ub144 2\uc6d4 8\uc77c, \uc13c\ud2b8\ub7f4\uc2dc\ud2f0 9\uce35 \ub300\ud68c\uc758\uc2e4\uc5d0\uc11c \uc9c4\ud589\ub41c \ubbfc\uc8fc\ub178\ub3d9\ub2f9 \uc784\uc2dc\ub2f9\ub300\ud68c\uc5d0\uc11c \uc790\uc8fc\ud30c \ub300\uc758\uc6d0\ub4e4\uc758 \ubc18\ub300\ub85c \ud601\uc2e0\uc548\uc758 \ud575\uc2ec \ub0b4\uc6a9\uc740 \ubd80\uacb0\ub418\uc5c8\uc73c\uba70 \uc790\uc8fc\ud30c\uc758 \uc218\uc815\uc548\ub300\ub85c \ud1b5\uacfc\ub418\uc5c8\ub2e4. \ud601\uc2e0\uc548\uc774 \ubd80\uacb0\ub418\uc790 \uc2ec\uc0c1\uc815 \uc704\uc6d0\uc7a5 \ub4f1 \ube44\ub300\uc704\uc6d0 \uc804\uc6d0\uc774 \uc0ac\ud1f4\ud558\uc600\uace0, \ubbfc\uc8fc\ub178\ub3d9\ub2f9 \uc81c1\uae30\uc640 \uc81c2\uae30 \uc9c0\ub3c4\ubd80\ub97c \uad6c\uc131\ud558\uc600\ub358 \uc778\ubb3c\ub4e4 \uac00\uc6b4\ub370 \uae40\ud61c\uacbd, \uc8fc\ub300\ud658, \uae40\uc885\ucca0, \uc2ec\uc7ac\uc625, \ud64d\uc2b9\ud558, \uae40\uae30\uc218 \ub4f1\uc774 \ud0c8\ub2f9\ud558\uc600\uc73c\uba70, \ubbfc\uc8fc\ub178\ub3d9\ub2f9 \uc18c\uc18d \uad6d\ud68c\uc758\uc6d0 \uc2ec\uc0c1\uc815, \ub178\ud68c\ucc2c, \uc870\uc2b9\uc218, \ub2e8\ubcd1\ud638\ub3c4 \ud0c8\ub2f9\ud558\uc600\ub2e4. \uc784\uc2dc\ub2f9\ub300\ud68c \uc774\ud6c4 2008\ub144 4\uc6d4\uae4c\uc9c0 \ubbfc\uc8fc\ub178\ub3d9\ub2f9\uc744 \ud0c8\ub2f9\ud55c \uc0ac\ub78c\uc740 16,904\uba85\uc5d0 \uc774\ub974\ub800\uace0, 2008\ub144 1\uc6d4 \ub2f9\uc2dc 11\ub9cc\uc5ec \uba85\uc5d0 \uc774\ub974\ub358 \ubbfc\uc8fc\ub178\ub3d9\ub2f9\uc758 \ub2f9\uc6d0 \uc218\ub294 9\ub9cc 4\ucc9c\uc5ec \uba85\uc73c\ub85c \uc904\uc5c8\ub2e4. \ubbfc\uc8fc\ub178\ub3d9\ub2f9\uc744 \ud0c8\ub2f9\ud55c \uc0ac\ub78c\ub4e4 \uc911 \uc77c\ubd80\ub294 2008\ub144 2\uc6d4 21\uc77c '\uc9c4\ubcf4\uc2e0\ub2f9\uc5f0\ub300\ud68c\uc758' \uc900\ube44\uc704\uc6d0\ud68c\ub97c \uacb0\uc131\ud558\uc600\uc73c\uba70, 2008\ub144 3\uc6d4 \uc9c4\ubcf4\uc2e0\ub2f9 \ucc3d\ub2f9\uc774 \uacf5\uc2dd \uc120\uc5b8\ub418\uc5c8\uace0, \ubbfc\uc8fc\ub178\ub3d9\ub2f9\uc740 \ubd84\ub2f9\ub418\uc5c8\ub2e4. ", "paragraph_answer": "\uacb0\uad6d 2008\ub144 2\uc6d4 8\uc77c, \uc13c\ud2b8\ub7f4\uc2dc\ud2f0 9\uce35 \ub300\ud68c\uc758\uc2e4\uc5d0\uc11c \uc9c4\ud589\ub41c \ubbfc\uc8fc\ub178\ub3d9\ub2f9 \uc784\uc2dc\ub2f9\ub300\ud68c\uc5d0\uc11c \uc790\uc8fc\ud30c \ub300\uc758\uc6d0\ub4e4\uc758 \ubc18\ub300\ub85c \ud601\uc2e0\uc548\uc758 \ud575\uc2ec \ub0b4\uc6a9\uc740 \ubd80\uacb0\ub418\uc5c8\uc73c\uba70 \uc790\uc8fc\ud30c\uc758 \uc218\uc815\uc548\ub300\ub85c \ud1b5\uacfc\ub418\uc5c8\ub2e4. \ud601\uc2e0\uc548\uc774 \ubd80\uacb0\ub418\uc790 \uc2ec\uc0c1\uc815 \uc704\uc6d0\uc7a5 \ub4f1 \ube44\ub300\uc704\uc6d0 \uc804\uc6d0\uc774 \uc0ac\ud1f4\ud558\uc600\uace0, \ubbfc\uc8fc\ub178\ub3d9\ub2f9 \uc81c1\uae30\uc640 \uc81c2\uae30 \uc9c0\ub3c4\ubd80\ub97c \uad6c\uc131\ud558\uc600\ub358 \uc778\ubb3c\ub4e4 \uac00\uc6b4\ub370 \uae40\ud61c\uacbd, \uc8fc\ub300\ud658, \uae40\uc885\ucca0, \uc2ec\uc7ac\uc625, \ud64d\uc2b9\ud558, \uae40\uae30\uc218 \ub4f1\uc774 \ud0c8\ub2f9\ud558\uc600\uc73c\uba70, \ubbfc\uc8fc\ub178\ub3d9\ub2f9 \uc18c\uc18d \uad6d\ud68c\uc758\uc6d0 \uc2ec\uc0c1\uc815, \ub178\ud68c\ucc2c, \uc870\uc2b9\uc218, \ub2e8\ubcd1\ud638\ub3c4 \ud0c8\ub2f9\ud558\uc600\ub2e4. \uc784\uc2dc\ub2f9\ub300\ud68c \uc774\ud6c4 2008\ub144 4\uc6d4\uae4c\uc9c0 \ubbfc\uc8fc\ub178\ub3d9\ub2f9\uc744 \ud0c8\ub2f9\ud55c \uc0ac\ub78c\uc740 16,904\uba85\uc5d0 \uc774\ub974\ub800\uace0, 2008\ub144 1\uc6d4 \ub2f9\uc2dc 11\ub9cc\uc5ec \uba85\uc5d0 \uc774\ub974\ub358 \ubbfc\uc8fc\ub178\ub3d9\ub2f9\uc758 \ub2f9\uc6d0 \uc218\ub294 9\ub9cc 4\ucc9c\uc5ec \uba85\uc73c\ub85c \uc904\uc5c8\ub2e4. \ubbfc\uc8fc\ub178\ub3d9\ub2f9\uc744 \ud0c8\ub2f9\ud55c \uc0ac\ub78c\ub4e4 \uc911 \uc77c\ubd80\ub294 2008\ub144 2\uc6d4 21\uc77c '\uc9c4\ubcf4\uc2e0\ub2f9\uc5f0\ub300\ud68c\uc758' \uc900\ube44\uc704\uc6d0\ud68c\ub97c \uacb0\uc131\ud558\uc600\uc73c\uba70, 2008\ub144 3\uc6d4 \uc9c4\ubcf4\uc2e0\ub2f9 \ucc3d\ub2f9\uc774 \uacf5\uc2dd \uc120\uc5b8\ub418\uc5c8\uace0, \ubbfc\uc8fc\ub178\ub3d9\ub2f9\uc740 \ubd84\ub2f9\ub418\uc5c8\ub2e4.", "sentence_answer": "\ubbfc\uc8fc\ub178\ub3d9\ub2f9\uc744 \ud0c8\ub2f9\ud55c \uc0ac\ub78c\ub4e4 \uc911 \uc77c\ubd80\ub294 2008\ub144 2\uc6d4 21\uc77c '\uc9c4\ubcf4\uc2e0\ub2f9\uc5f0\ub300\ud68c\uc758' \uc900\ube44\uc704\uc6d0\ud68c\ub97c \uacb0\uc131\ud558\uc600\uc73c\uba70, 2008\ub144 3\uc6d4 \uc9c4\ubcf4\uc2e0\ub2f9 \ucc3d\ub2f9\uc774 \uacf5\uc2dd \uc120\uc5b8\ub418\uc5c8\uace0, \ubbfc\uc8fc\ub178\ub3d9\ub2f9\uc740 \ubd84\ub2f9\ub418\uc5c8\ub2e4.", "paragraph_id": "6515322-1-2", "paragraph_question": "question: \uc9c4\ubcf4\uc2e0\ub2f9\uc774 \ucc3d\ub2f9\ub41c \uc2dc\uae30\ub294?, context: \uacb0\uad6d 2008\ub144 2\uc6d4 8\uc77c, \uc13c\ud2b8\ub7f4\uc2dc\ud2f0 9\uce35 \ub300\ud68c\uc758\uc2e4\uc5d0\uc11c \uc9c4\ud589\ub41c \ubbfc\uc8fc\ub178\ub3d9\ub2f9 \uc784\uc2dc\ub2f9\ub300\ud68c\uc5d0\uc11c \uc790\uc8fc\ud30c \ub300\uc758\uc6d0\ub4e4\uc758 \ubc18\ub300\ub85c \ud601\uc2e0\uc548\uc758 \ud575\uc2ec \ub0b4\uc6a9\uc740 \ubd80\uacb0\ub418\uc5c8\uc73c\uba70 \uc790\uc8fc\ud30c\uc758 \uc218\uc815\uc548\ub300\ub85c \ud1b5\uacfc\ub418\uc5c8\ub2e4. \ud601\uc2e0\uc548\uc774 \ubd80\uacb0\ub418\uc790 \uc2ec\uc0c1\uc815 \uc704\uc6d0\uc7a5 \ub4f1 \ube44\ub300\uc704\uc6d0 \uc804\uc6d0\uc774 \uc0ac\ud1f4\ud558\uc600\uace0, \ubbfc\uc8fc\ub178\ub3d9\ub2f9 \uc81c1\uae30\uc640 \uc81c2\uae30 \uc9c0\ub3c4\ubd80\ub97c \uad6c\uc131\ud558\uc600\ub358 \uc778\ubb3c\ub4e4 \uac00\uc6b4\ub370 \uae40\ud61c\uacbd, \uc8fc\ub300\ud658, \uae40\uc885\ucca0, \uc2ec\uc7ac\uc625, \ud64d\uc2b9\ud558, \uae40\uae30\uc218 \ub4f1\uc774 \ud0c8\ub2f9\ud558\uc600\uc73c\uba70, \ubbfc\uc8fc\ub178\ub3d9\ub2f9 \uc18c\uc18d \uad6d\ud68c\uc758\uc6d0 \uc2ec\uc0c1\uc815, \ub178\ud68c\ucc2c, \uc870\uc2b9\uc218, \ub2e8\ubcd1\ud638\ub3c4 \ud0c8\ub2f9\ud558\uc600\ub2e4. \uc784\uc2dc\ub2f9\ub300\ud68c \uc774\ud6c4 2008\ub144 4\uc6d4\uae4c\uc9c0 \ubbfc\uc8fc\ub178\ub3d9\ub2f9\uc744 \ud0c8\ub2f9\ud55c \uc0ac\ub78c\uc740 16,904\uba85\uc5d0 \uc774\ub974\ub800\uace0, 2008\ub144 1\uc6d4 \ub2f9\uc2dc 11\ub9cc\uc5ec \uba85\uc5d0 \uc774\ub974\ub358 \ubbfc\uc8fc\ub178\ub3d9\ub2f9\uc758 \ub2f9\uc6d0 \uc218\ub294 9\ub9cc 4\ucc9c\uc5ec \uba85\uc73c\ub85c \uc904\uc5c8\ub2e4. \ubbfc\uc8fc\ub178\ub3d9\ub2f9\uc744 \ud0c8\ub2f9\ud55c \uc0ac\ub78c\ub4e4 \uc911 \uc77c\ubd80\ub294 2008\ub144 2\uc6d4 21\uc77c '\uc9c4\ubcf4\uc2e0\ub2f9\uc5f0\ub300\ud68c\uc758' \uc900\ube44\uc704\uc6d0\ud68c\ub97c \uacb0\uc131\ud558\uc600\uc73c\uba70, 2008\ub144 3\uc6d4 \uc9c4\ubcf4\uc2e0\ub2f9 \ucc3d\ub2f9\uc774 \uacf5\uc2dd \uc120\uc5b8\ub418\uc5c8\uace0, \ubbfc\uc8fc\ub178\ub3d9\ub2f9\uc740 \ubd84\ub2f9\ub418\uc5c8\ub2e4."} {"question": "19\uc138\uae30 \ucd08\uc5d0 \uc601\uad6d\uc774 \ub178\uc608 \ubb34\uc5ed\uc744 \uae08\uc9c0\ud558\uba74\uc11c \ub9cc\ub4e0 \ubc95\uc740?", "paragraph": "\uc601\uad6d\uc740 \ucca0\ud559\uc801\uc774\uace0 \uacfc\ud559\uc801\uc778 \uc815\ubcf4 \ubc0f \uc720\ub825\ud55c \ubb38\ud559\uacfc \uc5f0\uadf9\uc758 \uc804\ud1b5\uc744 \uac00\uc9c4 \uacc4\ubabd\uc2dc\ub300\uc758 \uc911\uc694\ud55c \uad6d\uac00\uc600\ub2e4. \uadf8 \ub2e4\uc74c \uc138\uae30 \ub0b4\ub0b4 \uc601\uad6d\uc740 \ubb38\ud559, \uc608\uc220 \ubc0f \uacfc\ud559\uc5d0 \ud68d\uae30\uc801\uc73c\ub85c \uacf5\ud5cc\ud55c \uc758\ud68c \ubbfc\uc8fc\uc8fc\uc758\uc640 \uac19\uc740 \uc11c\uc591 \uc0ac\uc0c1\uc758 \ubc1c\uc804\uc5d0\uc11c \uc8fc\uc5ed\uc774 \ub418\uc5c8\ub2e4. \ucd08\uae30 \ub300\uc601 \uc81c\uad6d\uc758 \ubd80\ub294 \ub2e4\ub978 \uac15\uad6d\ucc98\ub7fc \uc5ed\uc2dc 1750\ub144 \uc774\ud6c4\uc5d0 \uc788\uc5c8\ub358 \ub178\uc608\ubb34\uc5ed\uc758 \uc0b0\uc5c5\ud654\ub97c \ud3ec\ud568\ud55c \uc2dd\ubbfc \ucc29\ucde8\uc5d0 \uc758\ud574 \uadf8 \uc77c\ubd80\ub294 \uc0dd\uc131\ub418\uc5c8\ub2e4. 18\uc138\uae30 \uc601\uad6d\uc758 \ub178\uc608 \ub9e4\ub9e4\ub294 \uc138\uacc4\uc5d0\uc11c \uac00\uc7a5 \uac15\ub825\ud588\ub358 \uc601\uad6d \ud568\ub300\uac00 \uc544\ud504\ub9ac\uce74 \ub178\uc608\ub97c \uc544\uba54\ub9ac\uce74 \ub300\ub959\uc73c\ub85c \ubcf4\ub0b4\ub294 \uc545\uba85 \ub192\uc740 \uc0bc\uac01\ubb34\uc5ed\uc758 \uc77c\ubd80\uc600\ub2e4. \uadf8\ub7ec\ub098 19\uc138\uae30 \ucd08\uc5d0 \uc601\uad6d\uc740 \ub178\uc608\ubb34\uc5ed\ubc95\uc744 \ub9cc\ub4e4\uc5b4, \uc601\uc6d0\ud788 \ub178\uc608 \ubb34\uc5ed\uc744 \uae08\uc9c0\ud558\ub294 \uccab \ubc88\uc9f8 \uad6d\uac00\uac00 \ub418\uc5c8\ub2e4. \uc601\uad6d\uc740 \ubbf8\uad6d\uc758 \uc2dc\uce74\uace0, \ud074\ub9ac\ube14\ub79c\ub4dc, \ub514\ud2b8\ub85c\uc774\ud2b8, \ud53c\uce20\ubc84\uadf8, \ud544\ub77c\ub378\ud53c\uc544, \ub274\uc695, \ubcfc\ud2f0\ubaa8\uc5b4, \ubcf4\uc2a4\ud134, \uc6cc\uc2f1\ud134 D.C., \uc560\ud2c0\ub79c\ud0c0 \ub4f1\uc744 \ucc28\uc9c0\ud588\ub2e4\uac00 18\uc138\uae30 \ub9d0\uc5d0 \ub3c5\ub9bd\uc2dc\ucf30\ub2e4.", "answer": "\ub178\uc608\ubb34\uc5ed\ubc95", "sentence": "\uc601\uad6d\uc740 \ub178\uc608\ubb34\uc5ed\ubc95 \uc744 \ub9cc\ub4e4\uc5b4, \uc601\uc6d0\ud788 \ub178\uc608 \ubb34\uc5ed\uc744 \uae08\uc9c0\ud558\ub294 \uccab \ubc88\uc9f8 \uad6d\uac00\uac00 \ub418\uc5c8\ub2e4.", "paragraph_sentence": "\uc601\uad6d\uc740 \ucca0\ud559\uc801\uc774\uace0 \uacfc\ud559\uc801\uc778 \uc815\ubcf4 \ubc0f \uc720\ub825\ud55c \ubb38\ud559\uacfc \uc5f0\uadf9\uc758 \uc804\ud1b5\uc744 \uac00\uc9c4 \uacc4\ubabd\uc2dc\ub300\uc758 \uc911\uc694\ud55c \uad6d\uac00\uc600\ub2e4. \uadf8 \ub2e4\uc74c \uc138\uae30 \ub0b4\ub0b4 \uc601\uad6d\uc740 \ubb38\ud559, \uc608\uc220 \ubc0f \uacfc\ud559\uc5d0 \ud68d\uae30\uc801\uc73c\ub85c \uacf5\ud5cc\ud55c \uc758\ud68c \ubbfc\uc8fc\uc8fc\uc758\uc640 \uac19\uc740 \uc11c\uc591 \uc0ac\uc0c1\uc758 \ubc1c\uc804\uc5d0\uc11c \uc8fc\uc5ed\uc774 \ub418\uc5c8\ub2e4. \ucd08\uae30 \ub300\uc601 \uc81c\uad6d\uc758 \ubd80\ub294 \ub2e4\ub978 \uac15\uad6d\ucc98\ub7fc \uc5ed\uc2dc 1750\ub144 \uc774\ud6c4\uc5d0 \uc788\uc5c8\ub358 \ub178\uc608\ubb34\uc5ed\uc758 \uc0b0\uc5c5\ud654\ub97c \ud3ec\ud568\ud55c \uc2dd\ubbfc \ucc29\ucde8\uc5d0 \uc758\ud574 \uadf8 \uc77c\ubd80\ub294 \uc0dd\uc131\ub418\uc5c8\ub2e4. 18\uc138\uae30 \uc601\uad6d\uc758 \ub178\uc608 \ub9e4\ub9e4\ub294 \uc138\uacc4\uc5d0\uc11c \uac00\uc7a5 \uac15\ub825\ud588\ub358 \uc601\uad6d \ud568\ub300\uac00 \uc544\ud504\ub9ac\uce74 \ub178\uc608\ub97c \uc544\uba54\ub9ac\uce74 \ub300\ub959\uc73c\ub85c \ubcf4\ub0b4\ub294 \uc545\uba85 \ub192\uc740 \uc0bc\uac01\ubb34\uc5ed\uc758 \uc77c\ubd80\uc600\ub2e4. \uadf8\ub7ec\ub098 19\uc138\uae30 \ucd08\uc5d0 \uc601\uad6d\uc740 \ub178\uc608\ubb34\uc5ed\ubc95 \uc744 \ub9cc\ub4e4\uc5b4, \uc601\uc6d0\ud788 \ub178\uc608 \ubb34\uc5ed\uc744 \uae08\uc9c0\ud558\ub294 \uccab \ubc88\uc9f8 \uad6d\uac00\uac00 \ub418\uc5c8\ub2e4. \uc601\uad6d\uc740 \ubbf8\uad6d\uc758 \uc2dc\uce74\uace0, \ud074\ub9ac\ube14\ub79c\ub4dc, \ub514\ud2b8\ub85c\uc774\ud2b8, \ud53c\uce20\ubc84\uadf8, \ud544\ub77c\ub378\ud53c\uc544, \ub274\uc695, \ubcfc\ud2f0\ubaa8\uc5b4, \ubcf4\uc2a4\ud134, \uc6cc\uc2f1\ud134 D.C., \uc560\ud2c0\ub79c\ud0c0 \ub4f1\uc744 \ucc28\uc9c0\ud588\ub2e4\uac00 18\uc138\uae30 \ub9d0\uc5d0 \ub3c5\ub9bd\uc2dc\ucf30\ub2e4.", "paragraph_answer": "\uc601\uad6d\uc740 \ucca0\ud559\uc801\uc774\uace0 \uacfc\ud559\uc801\uc778 \uc815\ubcf4 \ubc0f \uc720\ub825\ud55c \ubb38\ud559\uacfc \uc5f0\uadf9\uc758 \uc804\ud1b5\uc744 \uac00\uc9c4 \uacc4\ubabd\uc2dc\ub300\uc758 \uc911\uc694\ud55c \uad6d\uac00\uc600\ub2e4. \uadf8 \ub2e4\uc74c \uc138\uae30 \ub0b4\ub0b4 \uc601\uad6d\uc740 \ubb38\ud559, \uc608\uc220 \ubc0f \uacfc\ud559\uc5d0 \ud68d\uae30\uc801\uc73c\ub85c \uacf5\ud5cc\ud55c \uc758\ud68c \ubbfc\uc8fc\uc8fc\uc758\uc640 \uac19\uc740 \uc11c\uc591 \uc0ac\uc0c1\uc758 \ubc1c\uc804\uc5d0\uc11c \uc8fc\uc5ed\uc774 \ub418\uc5c8\ub2e4. \ucd08\uae30 \ub300\uc601 \uc81c\uad6d\uc758 \ubd80\ub294 \ub2e4\ub978 \uac15\uad6d\ucc98\ub7fc \uc5ed\uc2dc 1750\ub144 \uc774\ud6c4\uc5d0 \uc788\uc5c8\ub358 \ub178\uc608\ubb34\uc5ed\uc758 \uc0b0\uc5c5\ud654\ub97c \ud3ec\ud568\ud55c \uc2dd\ubbfc \ucc29\ucde8\uc5d0 \uc758\ud574 \uadf8 \uc77c\ubd80\ub294 \uc0dd\uc131\ub418\uc5c8\ub2e4. 18\uc138\uae30 \uc601\uad6d\uc758 \ub178\uc608 \ub9e4\ub9e4\ub294 \uc138\uacc4\uc5d0\uc11c \uac00\uc7a5 \uac15\ub825\ud588\ub358 \uc601\uad6d \ud568\ub300\uac00 \uc544\ud504\ub9ac\uce74 \ub178\uc608\ub97c \uc544\uba54\ub9ac\uce74 \ub300\ub959\uc73c\ub85c \ubcf4\ub0b4\ub294 \uc545\uba85 \ub192\uc740 \uc0bc\uac01\ubb34\uc5ed\uc758 \uc77c\ubd80\uc600\ub2e4. \uadf8\ub7ec\ub098 19\uc138\uae30 \ucd08\uc5d0 \uc601\uad6d\uc740 \ub178\uc608\ubb34\uc5ed\ubc95 \uc744 \ub9cc\ub4e4\uc5b4, \uc601\uc6d0\ud788 \ub178\uc608 \ubb34\uc5ed\uc744 \uae08\uc9c0\ud558\ub294 \uccab \ubc88\uc9f8 \uad6d\uac00\uac00 \ub418\uc5c8\ub2e4. \uc601\uad6d\uc740 \ubbf8\uad6d\uc758 \uc2dc\uce74\uace0, \ud074\ub9ac\ube14\ub79c\ub4dc, \ub514\ud2b8\ub85c\uc774\ud2b8, \ud53c\uce20\ubc84\uadf8, \ud544\ub77c\ub378\ud53c\uc544, \ub274\uc695, \ubcfc\ud2f0\ubaa8\uc5b4, \ubcf4\uc2a4\ud134, \uc6cc\uc2f1\ud134 D.C., \uc560\ud2c0\ub79c\ud0c0 \ub4f1\uc744 \ucc28\uc9c0\ud588\ub2e4\uac00 18\uc138\uae30 \ub9d0\uc5d0 \ub3c5\ub9bd\uc2dc\ucf30\ub2e4.", "sentence_answer": "\uc601\uad6d\uc740 \ub178\uc608\ubb34\uc5ed\ubc95 \uc744 \ub9cc\ub4e4\uc5b4, \uc601\uc6d0\ud788 \ub178\uc608 \ubb34\uc5ed\uc744 \uae08\uc9c0\ud558\ub294 \uccab \ubc88\uc9f8 \uad6d\uac00\uac00 \ub418\uc5c8\ub2e4.", "paragraph_id": "6587522-3-2", "paragraph_question": "question: 19\uc138\uae30 \ucd08\uc5d0 \uc601\uad6d\uc774 \ub178\uc608 \ubb34\uc5ed\uc744 \uae08\uc9c0\ud558\uba74\uc11c \ub9cc\ub4e0 \ubc95\uc740?, context: \uc601\uad6d\uc740 \ucca0\ud559\uc801\uc774\uace0 \uacfc\ud559\uc801\uc778 \uc815\ubcf4 \ubc0f \uc720\ub825\ud55c \ubb38\ud559\uacfc \uc5f0\uadf9\uc758 \uc804\ud1b5\uc744 \uac00\uc9c4 \uacc4\ubabd\uc2dc\ub300\uc758 \uc911\uc694\ud55c \uad6d\uac00\uc600\ub2e4. \uadf8 \ub2e4\uc74c \uc138\uae30 \ub0b4\ub0b4 \uc601\uad6d\uc740 \ubb38\ud559, \uc608\uc220 \ubc0f \uacfc\ud559\uc5d0 \ud68d\uae30\uc801\uc73c\ub85c \uacf5\ud5cc\ud55c \uc758\ud68c \ubbfc\uc8fc\uc8fc\uc758\uc640 \uac19\uc740 \uc11c\uc591 \uc0ac\uc0c1\uc758 \ubc1c\uc804\uc5d0\uc11c \uc8fc\uc5ed\uc774 \ub418\uc5c8\ub2e4. \ucd08\uae30 \ub300\uc601 \uc81c\uad6d\uc758 \ubd80\ub294 \ub2e4\ub978 \uac15\uad6d\ucc98\ub7fc \uc5ed\uc2dc 1750\ub144 \uc774\ud6c4\uc5d0 \uc788\uc5c8\ub358 \ub178\uc608\ubb34\uc5ed\uc758 \uc0b0\uc5c5\ud654\ub97c \ud3ec\ud568\ud55c \uc2dd\ubbfc \ucc29\ucde8\uc5d0 \uc758\ud574 \uadf8 \uc77c\ubd80\ub294 \uc0dd\uc131\ub418\uc5c8\ub2e4. 18\uc138\uae30 \uc601\uad6d\uc758 \ub178\uc608 \ub9e4\ub9e4\ub294 \uc138\uacc4\uc5d0\uc11c \uac00\uc7a5 \uac15\ub825\ud588\ub358 \uc601\uad6d \ud568\ub300\uac00 \uc544\ud504\ub9ac\uce74 \ub178\uc608\ub97c \uc544\uba54\ub9ac\uce74 \ub300\ub959\uc73c\ub85c \ubcf4\ub0b4\ub294 \uc545\uba85 \ub192\uc740 \uc0bc\uac01\ubb34\uc5ed\uc758 \uc77c\ubd80\uc600\ub2e4. \uadf8\ub7ec\ub098 19\uc138\uae30 \ucd08\uc5d0 \uc601\uad6d\uc740 \ub178\uc608\ubb34\uc5ed\ubc95\uc744 \ub9cc\ub4e4\uc5b4, \uc601\uc6d0\ud788 \ub178\uc608 \ubb34\uc5ed\uc744 \uae08\uc9c0\ud558\ub294 \uccab \ubc88\uc9f8 \uad6d\uac00\uac00 \ub418\uc5c8\ub2e4. \uc601\uad6d\uc740 \ubbf8\uad6d\uc758 \uc2dc\uce74\uace0, \ud074\ub9ac\ube14\ub79c\ub4dc, \ub514\ud2b8\ub85c\uc774\ud2b8, \ud53c\uce20\ubc84\uadf8, \ud544\ub77c\ub378\ud53c\uc544, \ub274\uc695, \ubcfc\ud2f0\ubaa8\uc5b4, \ubcf4\uc2a4\ud134, \uc6cc\uc2f1\ud134 D.C., \uc560\ud2c0\ub79c\ud0c0 \ub4f1\uc744 \ucc28\uc9c0\ud588\ub2e4\uac00 18\uc138\uae30 \ub9d0\uc5d0 \ub3c5\ub9bd\uc2dc\ucf30\ub2e4."} {"question": "\ubcd1\ub9c8\uc808\ub3c4\uc0ac\uc601\uc774 \uccad\uc8fc\ub85c \uc62e\uaca8\uc9c4 \ud574\ub294?", "paragraph": "\ud574\ubbf8\ub294 \uc870\uc120 \ucd08\uae30 \ucda9\uccad \ubcd1\ub9c8\uc808\ub3c4\uc0ac\uc601\uc774 \uc704\uce58\ud55c \uacf3\uc774\uc600\ub2e4. \uc870\uc120 \uc911\uae30\uc5d0\ub294 1651\ub144 \ubcd1\ub9c8\uc808\ub3c4\uc0ac\uc601\uc774 \uccad\uc8fc\ub85c \uc62e\uaca8\uc9c0\uace0 \ud574\ubbf8\ub294 \ud604\uc73c\ub85c \ucd95\uc18c\ub418\uc9c0\ub9cc, \uc5ec\uc804\ud788 1,400~1,500\uc5ec \uba85\uc758 \uad70\uc0ac\uac00 \uc8fc\ub454\ud55c \uc9c4\uc601\uc774 \uc874\uc7ac\ud558\ub294 \uad70\uc0ac\uc801 \uc694\ucda9\uc9c0\uc600\ub2e4. \uad70\uc0ac\ub97c \uac70\ub290\ub9b0 \ubb34\uad00 \uc601\uc7a5\uc740 \ud574\ubbf8 \ud604\uac10\uc744 \uacb8\ud558\uc600\uace0, \uc774\uc5d0 \ud574\ubbf8 \ud604\uac10\uc740 \ucda9\uccad\uc88c\ub3c4\uc758 \ub0b4\ud3ec \uc9c0\ubc29 \ud574\uc548\uc218\ube44 \uba85\ubaa9\uc73c\ub85c \uad6d\uc0ac\ubc94\uc744 \ucc98\ud615\ud560 \uc218 \uc788\ub294 \uad8c\ud55c\uc744 \uac16\uac8c \ub41c\ub2e4. \ub2f9\uc2dc \ud574\ubbf8 \uad00\uc544\uc758 \ub2f4\ub2f9 \uc9c0\uc5ed\uc740 \ucda9\uccad\ub3c4\ub294 \ubb3c\ub860\uc774\uace0 \uacbd\uae30\ub3c4 \ud3c9\ud0dd\uc5d0 \uc774\ub974\ub294 \ub113\uc740 \uc9c0\uc5ed\uc744 \uc544\uc6b8\ub800\ub2e4. \uc774\uc5d0 \ub2f4\ub2f9 \uc9c0\uc5ed\uc758 \uc2e0\uc790\ub4e4\uc774 \uccb4\ud3ec\ub418\uba74 \ubaa8\ub450 \ud574\ubbf8\uc74d\uc131\uc73c\ub85c \ub04c\ub824\uc624\uac8c \ub418\uc5c8\ub2e4. \uc774\ub4e4\uc774 \ud574\ubbf8\uc74d\uc131\uc5d0\uc11c \uac16\uc740 \uace0\ubb38\uc744 \ub2f9\ud55c \ud6c4 \ub9c8\uc9c0\ub9c9\uc73c\ub85c \uc8fd\uc74c\uc744 \ub9de\uc73c\ub7ec \uac00\ub294 \uacf3\uc774 \ud574\ubbf8\uc74d\uc131 \uc11c\ubb38 \ubc16\uacfc \ubc14\ub85c \uc774\uacf3 \ud574\ubbf8\uc21c\uad50\uc131\uc9c0\uc758 \uc7a5\uc18c\uc600\ub2e4.", "answer": "1651\ub144", "sentence": "\uc870\uc120 \uc911\uae30\uc5d0\ub294 1651\ub144 \ubcd1\ub9c8\uc808\ub3c4\uc0ac\uc601\uc774 \uccad\uc8fc\ub85c \uc62e\uaca8\uc9c0\uace0 \ud574\ubbf8\ub294 \ud604\uc73c\ub85c \ucd95\uc18c\ub418\uc9c0\ub9cc, \uc5ec\uc804\ud788 1,400~1,500\uc5ec \uba85\uc758 \uad70\uc0ac\uac00 \uc8fc\ub454\ud55c \uc9c4\uc601\uc774 \uc874\uc7ac\ud558\ub294 \uad70\uc0ac\uc801 \uc694\ucda9\uc9c0\uc600\ub2e4.", "paragraph_sentence": "\ud574\ubbf8\ub294 \uc870\uc120 \ucd08\uae30 \ucda9\uccad \ubcd1\ub9c8\uc808\ub3c4\uc0ac\uc601\uc774 \uc704\uce58\ud55c \uacf3\uc774\uc600\ub2e4. \uc870\uc120 \uc911\uae30\uc5d0\ub294 1651\ub144 \ubcd1\ub9c8\uc808\ub3c4\uc0ac\uc601\uc774 \uccad\uc8fc\ub85c \uc62e\uaca8\uc9c0\uace0 \ud574\ubbf8\ub294 \ud604\uc73c\ub85c \ucd95\uc18c\ub418\uc9c0\ub9cc, \uc5ec\uc804\ud788 1,400~1,500\uc5ec \uba85\uc758 \uad70\uc0ac\uac00 \uc8fc\ub454\ud55c \uc9c4\uc601\uc774 \uc874\uc7ac\ud558\ub294 \uad70\uc0ac\uc801 \uc694\ucda9\uc9c0\uc600\ub2e4. \uad70\uc0ac\ub97c \uac70\ub290\ub9b0 \ubb34\uad00 \uc601\uc7a5\uc740 \ud574\ubbf8 \ud604\uac10\uc744 \uacb8\ud558\uc600\uace0, \uc774\uc5d0 \ud574\ubbf8 \ud604\uac10\uc740 \ucda9\uccad\uc88c\ub3c4\uc758 \ub0b4\ud3ec \uc9c0\ubc29 \ud574\uc548\uc218\ube44 \uba85\ubaa9\uc73c\ub85c \uad6d\uc0ac\ubc94\uc744 \ucc98\ud615\ud560 \uc218 \uc788\ub294 \uad8c\ud55c\uc744 \uac16\uac8c \ub41c\ub2e4. \ub2f9\uc2dc \ud574\ubbf8 \uad00\uc544\uc758 \ub2f4\ub2f9 \uc9c0\uc5ed\uc740 \ucda9\uccad\ub3c4\ub294 \ubb3c\ub860\uc774\uace0 \uacbd\uae30\ub3c4 \ud3c9\ud0dd\uc5d0 \uc774\ub974\ub294 \ub113\uc740 \uc9c0\uc5ed\uc744 \uc544\uc6b8\ub800\ub2e4. \uc774\uc5d0 \ub2f4\ub2f9 \uc9c0\uc5ed\uc758 \uc2e0\uc790\ub4e4\uc774 \uccb4\ud3ec\ub418\uba74 \ubaa8\ub450 \ud574\ubbf8\uc74d\uc131\uc73c\ub85c \ub04c\ub824\uc624\uac8c \ub418\uc5c8\ub2e4. \uc774\ub4e4\uc774 \ud574\ubbf8\uc74d\uc131\uc5d0\uc11c \uac16\uc740 \uace0\ubb38\uc744 \ub2f9\ud55c \ud6c4 \ub9c8\uc9c0\ub9c9\uc73c\ub85c \uc8fd\uc74c\uc744 \ub9de\uc73c\ub7ec \uac00\ub294 \uacf3\uc774 \ud574\ubbf8\uc74d\uc131 \uc11c\ubb38 \ubc16\uacfc \ubc14\ub85c \uc774\uacf3 \ud574\ubbf8\uc21c\uad50\uc131\uc9c0\uc758 \uc7a5\uc18c\uc600\ub2e4.", "paragraph_answer": "\ud574\ubbf8\ub294 \uc870\uc120 \ucd08\uae30 \ucda9\uccad \ubcd1\ub9c8\uc808\ub3c4\uc0ac\uc601\uc774 \uc704\uce58\ud55c \uacf3\uc774\uc600\ub2e4. \uc870\uc120 \uc911\uae30\uc5d0\ub294 1651\ub144 \ubcd1\ub9c8\uc808\ub3c4\uc0ac\uc601\uc774 \uccad\uc8fc\ub85c \uc62e\uaca8\uc9c0\uace0 \ud574\ubbf8\ub294 \ud604\uc73c\ub85c \ucd95\uc18c\ub418\uc9c0\ub9cc, \uc5ec\uc804\ud788 1,400~1,500\uc5ec \uba85\uc758 \uad70\uc0ac\uac00 \uc8fc\ub454\ud55c \uc9c4\uc601\uc774 \uc874\uc7ac\ud558\ub294 \uad70\uc0ac\uc801 \uc694\ucda9\uc9c0\uc600\ub2e4. \uad70\uc0ac\ub97c \uac70\ub290\ub9b0 \ubb34\uad00 \uc601\uc7a5\uc740 \ud574\ubbf8 \ud604\uac10\uc744 \uacb8\ud558\uc600\uace0, \uc774\uc5d0 \ud574\ubbf8 \ud604\uac10\uc740 \ucda9\uccad\uc88c\ub3c4\uc758 \ub0b4\ud3ec \uc9c0\ubc29 \ud574\uc548\uc218\ube44 \uba85\ubaa9\uc73c\ub85c \uad6d\uc0ac\ubc94\uc744 \ucc98\ud615\ud560 \uc218 \uc788\ub294 \uad8c\ud55c\uc744 \uac16\uac8c \ub41c\ub2e4. \ub2f9\uc2dc \ud574\ubbf8 \uad00\uc544\uc758 \ub2f4\ub2f9 \uc9c0\uc5ed\uc740 \ucda9\uccad\ub3c4\ub294 \ubb3c\ub860\uc774\uace0 \uacbd\uae30\ub3c4 \ud3c9\ud0dd\uc5d0 \uc774\ub974\ub294 \ub113\uc740 \uc9c0\uc5ed\uc744 \uc544\uc6b8\ub800\ub2e4. \uc774\uc5d0 \ub2f4\ub2f9 \uc9c0\uc5ed\uc758 \uc2e0\uc790\ub4e4\uc774 \uccb4\ud3ec\ub418\uba74 \ubaa8\ub450 \ud574\ubbf8\uc74d\uc131\uc73c\ub85c \ub04c\ub824\uc624\uac8c \ub418\uc5c8\ub2e4. \uc774\ub4e4\uc774 \ud574\ubbf8\uc74d\uc131\uc5d0\uc11c \uac16\uc740 \uace0\ubb38\uc744 \ub2f9\ud55c \ud6c4 \ub9c8\uc9c0\ub9c9\uc73c\ub85c \uc8fd\uc74c\uc744 \ub9de\uc73c\ub7ec \uac00\ub294 \uacf3\uc774 \ud574\ubbf8\uc74d\uc131 \uc11c\ubb38 \ubc16\uacfc \ubc14\ub85c \uc774\uacf3 \ud574\ubbf8\uc21c\uad50\uc131\uc9c0\uc758 \uc7a5\uc18c\uc600\ub2e4.", "sentence_answer": "\uc870\uc120 \uc911\uae30\uc5d0\ub294 1651\ub144 \ubcd1\ub9c8\uc808\ub3c4\uc0ac\uc601\uc774 \uccad\uc8fc\ub85c \uc62e\uaca8\uc9c0\uace0 \ud574\ubbf8\ub294 \ud604\uc73c\ub85c \ucd95\uc18c\ub418\uc9c0\ub9cc, \uc5ec\uc804\ud788 1,400~1,500\uc5ec \uba85\uc758 \uad70\uc0ac\uac00 \uc8fc\ub454\ud55c \uc9c4\uc601\uc774 \uc874\uc7ac\ud558\ub294 \uad70\uc0ac\uc801 \uc694\ucda9\uc9c0\uc600\ub2e4.", "paragraph_id": "5941565-0-0", "paragraph_question": "question: \ubcd1\ub9c8\uc808\ub3c4\uc0ac\uc601\uc774 \uccad\uc8fc\ub85c \uc62e\uaca8\uc9c4 \ud574\ub294?, context: \ud574\ubbf8\ub294 \uc870\uc120 \ucd08\uae30 \ucda9\uccad \ubcd1\ub9c8\uc808\ub3c4\uc0ac\uc601\uc774 \uc704\uce58\ud55c \uacf3\uc774\uc600\ub2e4. \uc870\uc120 \uc911\uae30\uc5d0\ub294 1651\ub144 \ubcd1\ub9c8\uc808\ub3c4\uc0ac\uc601\uc774 \uccad\uc8fc\ub85c \uc62e\uaca8\uc9c0\uace0 \ud574\ubbf8\ub294 \ud604\uc73c\ub85c \ucd95\uc18c\ub418\uc9c0\ub9cc, \uc5ec\uc804\ud788 1,400~1,500\uc5ec \uba85\uc758 \uad70\uc0ac\uac00 \uc8fc\ub454\ud55c \uc9c4\uc601\uc774 \uc874\uc7ac\ud558\ub294 \uad70\uc0ac\uc801 \uc694\ucda9\uc9c0\uc600\ub2e4. \uad70\uc0ac\ub97c \uac70\ub290\ub9b0 \ubb34\uad00 \uc601\uc7a5\uc740 \ud574\ubbf8 \ud604\uac10\uc744 \uacb8\ud558\uc600\uace0, \uc774\uc5d0 \ud574\ubbf8 \ud604\uac10\uc740 \ucda9\uccad\uc88c\ub3c4\uc758 \ub0b4\ud3ec \uc9c0\ubc29 \ud574\uc548\uc218\ube44 \uba85\ubaa9\uc73c\ub85c \uad6d\uc0ac\ubc94\uc744 \ucc98\ud615\ud560 \uc218 \uc788\ub294 \uad8c\ud55c\uc744 \uac16\uac8c \ub41c\ub2e4. \ub2f9\uc2dc \ud574\ubbf8 \uad00\uc544\uc758 \ub2f4\ub2f9 \uc9c0\uc5ed\uc740 \ucda9\uccad\ub3c4\ub294 \ubb3c\ub860\uc774\uace0 \uacbd\uae30\ub3c4 \ud3c9\ud0dd\uc5d0 \uc774\ub974\ub294 \ub113\uc740 \uc9c0\uc5ed\uc744 \uc544\uc6b8\ub800\ub2e4. \uc774\uc5d0 \ub2f4\ub2f9 \uc9c0\uc5ed\uc758 \uc2e0\uc790\ub4e4\uc774 \uccb4\ud3ec\ub418\uba74 \ubaa8\ub450 \ud574\ubbf8\uc74d\uc131\uc73c\ub85c \ub04c\ub824\uc624\uac8c \ub418\uc5c8\ub2e4. \uc774\ub4e4\uc774 \ud574\ubbf8\uc74d\uc131\uc5d0\uc11c \uac16\uc740 \uace0\ubb38\uc744 \ub2f9\ud55c \ud6c4 \ub9c8\uc9c0\ub9c9\uc73c\ub85c \uc8fd\uc74c\uc744 \ub9de\uc73c\ub7ec \uac00\ub294 \uacf3\uc774 \ud574\ubbf8\uc74d\uc131 \uc11c\ubb38 \ubc16\uacfc \ubc14\ub85c \uc774\uacf3 \ud574\ubbf8\uc21c\uad50\uc131\uc9c0\uc758 \uc7a5\uc18c\uc600\ub2e4."} {"question": "\uc624\ub298\ub0a0 \ud55c\uad6d \uc758\ub8cc\ubcf4\ud5d8\uc758 \ud1a0\ub300\uac00 \ub41c \uac83\uc740 \uc5b4\ub290 \uc815\ubd80\ub54c\uc778\uac00?", "paragraph": "\ubc15\uc815\ud76c\uc758 \ub300\ud1b5\ub839 \uc9c1\ubb34 \uc218\ud589\uc5d0 \ub300\ud55c \ud3c9\uac00\ub294 \ub2e4\uc591\ud558\ub2e4. \uacbd\uc81c\uc801\uc778 \uce21\uba74\uc5d0\uc11c\ub294 \uc7a5\uba74 \ub0b4\uac01\uc774 \uc218\ub9bd\ud55c \uad6d\uac00 \uc8fc\ub3c4 \ud575\uc2ec \uacf5\uc5c5 \uac1c\ubc1c\uc744 \uace8\uc790\ub85c \ud558\ub294 '\ubd88\uade0\ud615 \uac1c\ubc1c \ubaa8\ub378-\uc9c0\ub3c4\ubc1b\ub294 \uc790\ubcf8\uc8fc\uc758 \uccb4\uc81c'(\uc81c2\uacf5\ud654\uad6d\uacfc \uc6b8\ud504 \ubc15\uc0ac\uc758 \ud569\uc791\uc778 \uacbd\uc81c \uac1c\ubc1c 5\uac1c\ub144 \uacc4\ud68d), \ubbf8\uad6d\uc774 \uc8fc\ubb38\ud55c \ud658\uc728 \ud604\uc2e4\ud654 \ubc0f \ub178\ub3d9\uc9d1\uc57d\ud615 \uc218\ucd9c \uacbd\uacf5\uc5c5 \uac1c\ubc1c\uc744 \ube44\uad50\uc801 \ucda9\uc2e4\ud558\uac8c \ub530\ub790\ub2e4\ub294 \ud3c9\uac00\uc640 \ubb34\ub9ac\ud558\uace0 \ube44\ud6a8\uc728\uc801\uc778 \uc911\uacf5\uc5c5 \uc911\ubcf5 \ud22c\uc790 \ubc0f \uc9c0\ub098\uce5c \uad00\uce58 \uacbd\uc81c\ub85c \uc778\ud55c \uae08\uc735 \uc2dc\uc7a5\uc758 \ubd80\uc2e4\ud654\ub85c \ud55c\uad6d\uc758 \uacbd\uc81c\ubc1c\uc804\uc744 \ub2a6\ucd94\uc5c8\ub2e4\ub294 \ud3c9\uac00\uac00 \uacf5\uc874\ud558\uace0 \uc788\ub2e4. \uc774\uc678\uc5d0\ub3c4 \uc0c8\ub9c8\uc744 \uc6b4\ub3d9\uc744 \ud1b5\ud574 \ub18d\ucd0c \ubc1c\uc804\uc5d0 \uc131\uacf5\ud588\ub2e4\ub294 \uae0d\uc815\uc801 \ud3c9\uac00\uc640 \uad70\uc815 \uc2dc\uc808 \uc774\ub798\ub85c \uc911\ub18d \uc815\ucc45, \ud1b5\uc77c\ubcbc \ubcf4\uae09, \uc0c8\ub9c8\uc744 \uc6b4\ub3d9\uc5d0\ub3c4 \ubd88\uad6c\ud558\uace0 \ub18d\ucd0c\uacfc \ub18d\uac00 \uacbd\uc81c\ub97c \ud53c\ud3d0\ud558\uac8c \ub9cc\ub4e4\uc5c8\ub2e4\ub294 \ubd80\uc815\uc801 \ud3c9\uac00\uac00 \uacf5\uc874\ud558\uace0 \uc788\uc73c\uba70, \uadf8\ub9b0\ubca8\ud2b8 \uaddc\uc81c\uc5d0\ub3c4 \ubd88\uad6c\ud558\uace0 \uc218\ub3c4\uad8c \uacfc\ubc00\uc744 \uc608\ubc29\ud558\uc9c0 \ubabb\ud588\ub2e4\ub294 \ubd80\uc815\uc801 \ud3c9\uac00, \uc758\ub8cc\ubcf4\ud5d8 \uc81c\ub3c4\uac00 \uc624\ub298\ub0a0 \ud55c\uad6d \uc758\ub8cc\ubcf4\ud5d8\uc758 \ud1a0\ub300\uac00 \ub41c \uc804\ub450\ud658 \uc815\ubd80\uc758 \uc758\ub8cc\ubcf4\ud5d8\ub9cc \ubabb\ud558\ub2e4\ub294 \ubd80\uc815\uc801 \ud3c9\uac00, \uac1c\ubc1c \uc704\uc8fc\uc758 \ud68d\uc77c\ud654\ub41c \uc815\ucc45\uc774\ub77c\ub294 \ubd80\uc815\uc801 \ud3c9\uac00 \ub4f1\uc774 \ub4a4\ub530\ub974\uace0 \uc788\ub2e4. \ub2e4\ub9cc, \uc81c1\uacf5\ud654\uad6d\uacfc \uc81c2\uacf5\ud654\uad6d\uc5d0\uc11c \uafb8\uc900\ud788 \uc2e4\uc2dc\ub418\uc5c8\ub358 \uc0ac\ubc29 \uc0ac\uc5c5, \uc870\ub9bc \uc0ac\uc5c5, \uc0b0\ub9bc \ubcf5\uc6d0 \uc0ac\uc5c5 \ub4f1\uc5d0 \uad00\ud558\uc5ec\ub294 \ud638\ud3c9\uc774 \ub354 \ub9ce\uc740 \ud3b8\uc774\ub2e4. \uac01\uc885 \uc5ec\ub860\uc870\uc0ac\uc5d0\uc11c\ub294 \uc874\uacbd\ud558\ub294 \ub300\ud1b5\ub839\uc744 \ube44\ub86f\ud55c \ub300\ud1b5\ub839 \uc120\ud638\ub3c4\uc640 \uacf5\uc801 \ubd80\ubb38 \ub4f1\uc5d0\uc11c \ucd5c\uc0c1\uc704 \ud3c9\uac00\ub97c \ubc1b\uc544\uc624\uace0 \uc788\ub2e4.", "answer": "\uc804\ub450\ud658 \uc815\ubd80", "sentence": "\uc774\uc678\uc5d0\ub3c4 \uc0c8\ub9c8\uc744 \uc6b4\ub3d9\uc744 \ud1b5\ud574 \ub18d\ucd0c \ubc1c\uc804\uc5d0 \uc131\uacf5\ud588\ub2e4\ub294 \uae0d\uc815\uc801 \ud3c9\uac00\uc640 \uad70\uc815 \uc2dc\uc808 \uc774\ub798\ub85c \uc911\ub18d \uc815\ucc45, \ud1b5\uc77c\ubcbc \ubcf4\uae09, \uc0c8\ub9c8\uc744 \uc6b4\ub3d9\uc5d0\ub3c4 \ubd88\uad6c\ud558\uace0 \ub18d\ucd0c\uacfc \ub18d\uac00 \uacbd\uc81c\ub97c \ud53c\ud3d0\ud558\uac8c \ub9cc\ub4e4\uc5c8\ub2e4\ub294 \ubd80\uc815\uc801 \ud3c9\uac00\uac00 \uacf5\uc874\ud558\uace0 \uc788\uc73c\uba70, \uadf8\ub9b0\ubca8\ud2b8 \uaddc\uc81c\uc5d0\ub3c4 \ubd88\uad6c\ud558\uace0 \uc218\ub3c4\uad8c \uacfc\ubc00\uc744 \uc608\ubc29\ud558\uc9c0 \ubabb\ud588\ub2e4\ub294 \ubd80\uc815\uc801 \ud3c9\uac00, \uc758\ub8cc\ubcf4\ud5d8 \uc81c\ub3c4\uac00 \uc624\ub298\ub0a0 \ud55c\uad6d \uc758\ub8cc\ubcf4\ud5d8\uc758 \ud1a0\ub300\uac00 \ub41c \uc804\ub450\ud658 \uc815\ubd80 \uc758 \uc758\ub8cc\ubcf4\ud5d8\ub9cc \ubabb\ud558\ub2e4\ub294 \ubd80\uc815\uc801 \ud3c9\uac00, \uac1c\ubc1c \uc704\uc8fc\uc758 \ud68d\uc77c\ud654\ub41c \uc815\ucc45\uc774\ub77c\ub294 \ubd80\uc815\uc801 \ud3c9\uac00 \ub4f1\uc774 \ub4a4\ub530\ub974\uace0 \uc788\ub2e4.", "paragraph_sentence": "\ubc15\uc815\ud76c\uc758 \ub300\ud1b5\ub839 \uc9c1\ubb34 \uc218\ud589\uc5d0 \ub300\ud55c \ud3c9\uac00\ub294 \ub2e4\uc591\ud558\ub2e4. \uacbd\uc81c\uc801\uc778 \uce21\uba74\uc5d0\uc11c\ub294 \uc7a5\uba74 \ub0b4\uac01\uc774 \uc218\ub9bd\ud55c \uad6d\uac00 \uc8fc\ub3c4 \ud575\uc2ec \uacf5\uc5c5 \uac1c\ubc1c\uc744 \uace8\uc790\ub85c \ud558\ub294 '\ubd88\uade0\ud615 \uac1c\ubc1c \ubaa8\ub378-\uc9c0\ub3c4\ubc1b\ub294 \uc790\ubcf8\uc8fc\uc758 \uccb4\uc81c'(\uc81c2\uacf5\ud654\uad6d\uacfc \uc6b8\ud504 \ubc15\uc0ac\uc758 \ud569\uc791\uc778 \uacbd\uc81c \uac1c\ubc1c 5\uac1c\ub144 \uacc4\ud68d), \ubbf8\uad6d\uc774 \uc8fc\ubb38\ud55c \ud658\uc728 \ud604\uc2e4\ud654 \ubc0f \ub178\ub3d9\uc9d1\uc57d\ud615 \uc218\ucd9c \uacbd\uacf5\uc5c5 \uac1c\ubc1c\uc744 \ube44\uad50\uc801 \ucda9\uc2e4\ud558\uac8c \ub530\ub790\ub2e4\ub294 \ud3c9\uac00\uc640 \ubb34\ub9ac\ud558\uace0 \ube44\ud6a8\uc728\uc801\uc778 \uc911\uacf5\uc5c5 \uc911\ubcf5 \ud22c\uc790 \ubc0f \uc9c0\ub098\uce5c \uad00\uce58 \uacbd\uc81c\ub85c \uc778\ud55c \uae08\uc735 \uc2dc\uc7a5\uc758 \ubd80\uc2e4\ud654\ub85c \ud55c\uad6d\uc758 \uacbd\uc81c\ubc1c\uc804\uc744 \ub2a6\ucd94\uc5c8\ub2e4\ub294 \ud3c9\uac00\uac00 \uacf5\uc874\ud558\uace0 \uc788\ub2e4. \uc774\uc678\uc5d0\ub3c4 \uc0c8\ub9c8\uc744 \uc6b4\ub3d9\uc744 \ud1b5\ud574 \ub18d\ucd0c \ubc1c\uc804\uc5d0 \uc131\uacf5\ud588\ub2e4\ub294 \uae0d\uc815\uc801 \ud3c9\uac00\uc640 \uad70\uc815 \uc2dc\uc808 \uc774\ub798\ub85c \uc911\ub18d \uc815\ucc45, \ud1b5\uc77c\ubcbc \ubcf4\uae09, \uc0c8\ub9c8\uc744 \uc6b4\ub3d9\uc5d0\ub3c4 \ubd88\uad6c\ud558\uace0 \ub18d\ucd0c\uacfc \ub18d\uac00 \uacbd\uc81c\ub97c \ud53c\ud3d0\ud558\uac8c \ub9cc\ub4e4\uc5c8\ub2e4\ub294 \ubd80\uc815\uc801 \ud3c9\uac00\uac00 \uacf5\uc874\ud558\uace0 \uc788\uc73c\uba70, \uadf8\ub9b0\ubca8\ud2b8 \uaddc\uc81c\uc5d0\ub3c4 \ubd88\uad6c\ud558\uace0 \uc218\ub3c4\uad8c \uacfc\ubc00\uc744 \uc608\ubc29\ud558\uc9c0 \ubabb\ud588\ub2e4\ub294 \ubd80\uc815\uc801 \ud3c9\uac00, \uc758\ub8cc\ubcf4\ud5d8 \uc81c\ub3c4\uac00 \uc624\ub298\ub0a0 \ud55c\uad6d \uc758\ub8cc\ubcf4\ud5d8\uc758 \ud1a0\ub300\uac00 \ub41c \uc804\ub450\ud658 \uc815\ubd80 \uc758 \uc758\ub8cc\ubcf4\ud5d8\ub9cc \ubabb\ud558\ub2e4\ub294 \ubd80\uc815\uc801 \ud3c9\uac00, \uac1c\ubc1c \uc704\uc8fc\uc758 \ud68d\uc77c\ud654\ub41c \uc815\ucc45\uc774\ub77c\ub294 \ubd80\uc815\uc801 \ud3c9\uac00 \ub4f1\uc774 \ub4a4\ub530\ub974\uace0 \uc788\ub2e4. \ub2e4\ub9cc, \uc81c1\uacf5\ud654\uad6d\uacfc \uc81c2\uacf5\ud654\uad6d\uc5d0\uc11c \uafb8\uc900\ud788 \uc2e4\uc2dc\ub418\uc5c8\ub358 \uc0ac\ubc29 \uc0ac\uc5c5, \uc870\ub9bc \uc0ac\uc5c5, \uc0b0\ub9bc \ubcf5\uc6d0 \uc0ac\uc5c5 \ub4f1\uc5d0 \uad00\ud558\uc5ec\ub294 \ud638\ud3c9\uc774 \ub354 \ub9ce\uc740 \ud3b8\uc774\ub2e4. \uac01\uc885 \uc5ec\ub860\uc870\uc0ac\uc5d0\uc11c\ub294 \uc874\uacbd\ud558\ub294 \ub300\ud1b5\ub839\uc744 \ube44\ub86f\ud55c \ub300\ud1b5\ub839 \uc120\ud638\ub3c4\uc640 \uacf5\uc801 \ubd80\ubb38 \ub4f1\uc5d0\uc11c \ucd5c\uc0c1\uc704 \ud3c9\uac00\ub97c \ubc1b\uc544\uc624\uace0 \uc788\ub2e4.", "paragraph_answer": "\ubc15\uc815\ud76c\uc758 \ub300\ud1b5\ub839 \uc9c1\ubb34 \uc218\ud589\uc5d0 \ub300\ud55c \ud3c9\uac00\ub294 \ub2e4\uc591\ud558\ub2e4. \uacbd\uc81c\uc801\uc778 \uce21\uba74\uc5d0\uc11c\ub294 \uc7a5\uba74 \ub0b4\uac01\uc774 \uc218\ub9bd\ud55c \uad6d\uac00 \uc8fc\ub3c4 \ud575\uc2ec \uacf5\uc5c5 \uac1c\ubc1c\uc744 \uace8\uc790\ub85c \ud558\ub294 '\ubd88\uade0\ud615 \uac1c\ubc1c \ubaa8\ub378-\uc9c0\ub3c4\ubc1b\ub294 \uc790\ubcf8\uc8fc\uc758 \uccb4\uc81c'(\uc81c2\uacf5\ud654\uad6d\uacfc \uc6b8\ud504 \ubc15\uc0ac\uc758 \ud569\uc791\uc778 \uacbd\uc81c \uac1c\ubc1c 5\uac1c\ub144 \uacc4\ud68d), \ubbf8\uad6d\uc774 \uc8fc\ubb38\ud55c \ud658\uc728 \ud604\uc2e4\ud654 \ubc0f \ub178\ub3d9\uc9d1\uc57d\ud615 \uc218\ucd9c \uacbd\uacf5\uc5c5 \uac1c\ubc1c\uc744 \ube44\uad50\uc801 \ucda9\uc2e4\ud558\uac8c \ub530\ub790\ub2e4\ub294 \ud3c9\uac00\uc640 \ubb34\ub9ac\ud558\uace0 \ube44\ud6a8\uc728\uc801\uc778 \uc911\uacf5\uc5c5 \uc911\ubcf5 \ud22c\uc790 \ubc0f \uc9c0\ub098\uce5c \uad00\uce58 \uacbd\uc81c\ub85c \uc778\ud55c \uae08\uc735 \uc2dc\uc7a5\uc758 \ubd80\uc2e4\ud654\ub85c \ud55c\uad6d\uc758 \uacbd\uc81c\ubc1c\uc804\uc744 \ub2a6\ucd94\uc5c8\ub2e4\ub294 \ud3c9\uac00\uac00 \uacf5\uc874\ud558\uace0 \uc788\ub2e4. \uc774\uc678\uc5d0\ub3c4 \uc0c8\ub9c8\uc744 \uc6b4\ub3d9\uc744 \ud1b5\ud574 \ub18d\ucd0c \ubc1c\uc804\uc5d0 \uc131\uacf5\ud588\ub2e4\ub294 \uae0d\uc815\uc801 \ud3c9\uac00\uc640 \uad70\uc815 \uc2dc\uc808 \uc774\ub798\ub85c \uc911\ub18d \uc815\ucc45, \ud1b5\uc77c\ubcbc \ubcf4\uae09, \uc0c8\ub9c8\uc744 \uc6b4\ub3d9\uc5d0\ub3c4 \ubd88\uad6c\ud558\uace0 \ub18d\ucd0c\uacfc \ub18d\uac00 \uacbd\uc81c\ub97c \ud53c\ud3d0\ud558\uac8c \ub9cc\ub4e4\uc5c8\ub2e4\ub294 \ubd80\uc815\uc801 \ud3c9\uac00\uac00 \uacf5\uc874\ud558\uace0 \uc788\uc73c\uba70, \uadf8\ub9b0\ubca8\ud2b8 \uaddc\uc81c\uc5d0\ub3c4 \ubd88\uad6c\ud558\uace0 \uc218\ub3c4\uad8c \uacfc\ubc00\uc744 \uc608\ubc29\ud558\uc9c0 \ubabb\ud588\ub2e4\ub294 \ubd80\uc815\uc801 \ud3c9\uac00, \uc758\ub8cc\ubcf4\ud5d8 \uc81c\ub3c4\uac00 \uc624\ub298\ub0a0 \ud55c\uad6d \uc758\ub8cc\ubcf4\ud5d8\uc758 \ud1a0\ub300\uac00 \ub41c \uc804\ub450\ud658 \uc815\ubd80 \uc758 \uc758\ub8cc\ubcf4\ud5d8\ub9cc \ubabb\ud558\ub2e4\ub294 \ubd80\uc815\uc801 \ud3c9\uac00, \uac1c\ubc1c \uc704\uc8fc\uc758 \ud68d\uc77c\ud654\ub41c \uc815\ucc45\uc774\ub77c\ub294 \ubd80\uc815\uc801 \ud3c9\uac00 \ub4f1\uc774 \ub4a4\ub530\ub974\uace0 \uc788\ub2e4. \ub2e4\ub9cc, \uc81c1\uacf5\ud654\uad6d\uacfc \uc81c2\uacf5\ud654\uad6d\uc5d0\uc11c \uafb8\uc900\ud788 \uc2e4\uc2dc\ub418\uc5c8\ub358 \uc0ac\ubc29 \uc0ac\uc5c5, \uc870\ub9bc \uc0ac\uc5c5, \uc0b0\ub9bc \ubcf5\uc6d0 \uc0ac\uc5c5 \ub4f1\uc5d0 \uad00\ud558\uc5ec\ub294 \ud638\ud3c9\uc774 \ub354 \ub9ce\uc740 \ud3b8\uc774\ub2e4. \uac01\uc885 \uc5ec\ub860\uc870\uc0ac\uc5d0\uc11c\ub294 \uc874\uacbd\ud558\ub294 \ub300\ud1b5\ub839\uc744 \ube44\ub86f\ud55c \ub300\ud1b5\ub839 \uc120\ud638\ub3c4\uc640 \uacf5\uc801 \ubd80\ubb38 \ub4f1\uc5d0\uc11c \ucd5c\uc0c1\uc704 \ud3c9\uac00\ub97c \ubc1b\uc544\uc624\uace0 \uc788\ub2e4.", "sentence_answer": "\uc774\uc678\uc5d0\ub3c4 \uc0c8\ub9c8\uc744 \uc6b4\ub3d9\uc744 \ud1b5\ud574 \ub18d\ucd0c \ubc1c\uc804\uc5d0 \uc131\uacf5\ud588\ub2e4\ub294 \uae0d\uc815\uc801 \ud3c9\uac00\uc640 \uad70\uc815 \uc2dc\uc808 \uc774\ub798\ub85c \uc911\ub18d \uc815\ucc45, \ud1b5\uc77c\ubcbc \ubcf4\uae09, \uc0c8\ub9c8\uc744 \uc6b4\ub3d9\uc5d0\ub3c4 \ubd88\uad6c\ud558\uace0 \ub18d\ucd0c\uacfc \ub18d\uac00 \uacbd\uc81c\ub97c \ud53c\ud3d0\ud558\uac8c \ub9cc\ub4e4\uc5c8\ub2e4\ub294 \ubd80\uc815\uc801 \ud3c9\uac00\uac00 \uacf5\uc874\ud558\uace0 \uc788\uc73c\uba70, \uadf8\ub9b0\ubca8\ud2b8 \uaddc\uc81c\uc5d0\ub3c4 \ubd88\uad6c\ud558\uace0 \uc218\ub3c4\uad8c \uacfc\ubc00\uc744 \uc608\ubc29\ud558\uc9c0 \ubabb\ud588\ub2e4\ub294 \ubd80\uc815\uc801 \ud3c9\uac00, \uc758\ub8cc\ubcf4\ud5d8 \uc81c\ub3c4\uac00 \uc624\ub298\ub0a0 \ud55c\uad6d \uc758\ub8cc\ubcf4\ud5d8\uc758 \ud1a0\ub300\uac00 \ub41c \uc804\ub450\ud658 \uc815\ubd80 \uc758 \uc758\ub8cc\ubcf4\ud5d8\ub9cc \ubabb\ud558\ub2e4\ub294 \ubd80\uc815\uc801 \ud3c9\uac00, \uac1c\ubc1c \uc704\uc8fc\uc758 \ud68d\uc77c\ud654\ub41c \uc815\ucc45\uc774\ub77c\ub294 \ubd80\uc815\uc801 \ud3c9\uac00 \ub4f1\uc774 \ub4a4\ub530\ub974\uace0 \uc788\ub2e4.", "paragraph_id": "6516370-2-1", "paragraph_question": "question: \uc624\ub298\ub0a0 \ud55c\uad6d \uc758\ub8cc\ubcf4\ud5d8\uc758 \ud1a0\ub300\uac00 \ub41c \uac83\uc740 \uc5b4\ub290 \uc815\ubd80\ub54c\uc778\uac00?, context: \ubc15\uc815\ud76c\uc758 \ub300\ud1b5\ub839 \uc9c1\ubb34 \uc218\ud589\uc5d0 \ub300\ud55c \ud3c9\uac00\ub294 \ub2e4\uc591\ud558\ub2e4. \uacbd\uc81c\uc801\uc778 \uce21\uba74\uc5d0\uc11c\ub294 \uc7a5\uba74 \ub0b4\uac01\uc774 \uc218\ub9bd\ud55c \uad6d\uac00 \uc8fc\ub3c4 \ud575\uc2ec \uacf5\uc5c5 \uac1c\ubc1c\uc744 \uace8\uc790\ub85c \ud558\ub294 '\ubd88\uade0\ud615 \uac1c\ubc1c \ubaa8\ub378-\uc9c0\ub3c4\ubc1b\ub294 \uc790\ubcf8\uc8fc\uc758 \uccb4\uc81c'(\uc81c2\uacf5\ud654\uad6d\uacfc \uc6b8\ud504 \ubc15\uc0ac\uc758 \ud569\uc791\uc778 \uacbd\uc81c \uac1c\ubc1c 5\uac1c\ub144 \uacc4\ud68d), \ubbf8\uad6d\uc774 \uc8fc\ubb38\ud55c \ud658\uc728 \ud604\uc2e4\ud654 \ubc0f \ub178\ub3d9\uc9d1\uc57d\ud615 \uc218\ucd9c \uacbd\uacf5\uc5c5 \uac1c\ubc1c\uc744 \ube44\uad50\uc801 \ucda9\uc2e4\ud558\uac8c \ub530\ub790\ub2e4\ub294 \ud3c9\uac00\uc640 \ubb34\ub9ac\ud558\uace0 \ube44\ud6a8\uc728\uc801\uc778 \uc911\uacf5\uc5c5 \uc911\ubcf5 \ud22c\uc790 \ubc0f \uc9c0\ub098\uce5c \uad00\uce58 \uacbd\uc81c\ub85c \uc778\ud55c \uae08\uc735 \uc2dc\uc7a5\uc758 \ubd80\uc2e4\ud654\ub85c \ud55c\uad6d\uc758 \uacbd\uc81c\ubc1c\uc804\uc744 \ub2a6\ucd94\uc5c8\ub2e4\ub294 \ud3c9\uac00\uac00 \uacf5\uc874\ud558\uace0 \uc788\ub2e4. \uc774\uc678\uc5d0\ub3c4 \uc0c8\ub9c8\uc744 \uc6b4\ub3d9\uc744 \ud1b5\ud574 \ub18d\ucd0c \ubc1c\uc804\uc5d0 \uc131\uacf5\ud588\ub2e4\ub294 \uae0d\uc815\uc801 \ud3c9\uac00\uc640 \uad70\uc815 \uc2dc\uc808 \uc774\ub798\ub85c \uc911\ub18d \uc815\ucc45, \ud1b5\uc77c\ubcbc \ubcf4\uae09, \uc0c8\ub9c8\uc744 \uc6b4\ub3d9\uc5d0\ub3c4 \ubd88\uad6c\ud558\uace0 \ub18d\ucd0c\uacfc \ub18d\uac00 \uacbd\uc81c\ub97c \ud53c\ud3d0\ud558\uac8c \ub9cc\ub4e4\uc5c8\ub2e4\ub294 \ubd80\uc815\uc801 \ud3c9\uac00\uac00 \uacf5\uc874\ud558\uace0 \uc788\uc73c\uba70, \uadf8\ub9b0\ubca8\ud2b8 \uaddc\uc81c\uc5d0\ub3c4 \ubd88\uad6c\ud558\uace0 \uc218\ub3c4\uad8c \uacfc\ubc00\uc744 \uc608\ubc29\ud558\uc9c0 \ubabb\ud588\ub2e4\ub294 \ubd80\uc815\uc801 \ud3c9\uac00, \uc758\ub8cc\ubcf4\ud5d8 \uc81c\ub3c4\uac00 \uc624\ub298\ub0a0 \ud55c\uad6d \uc758\ub8cc\ubcf4\ud5d8\uc758 \ud1a0\ub300\uac00 \ub41c \uc804\ub450\ud658 \uc815\ubd80\uc758 \uc758\ub8cc\ubcf4\ud5d8\ub9cc \ubabb\ud558\ub2e4\ub294 \ubd80\uc815\uc801 \ud3c9\uac00, \uac1c\ubc1c \uc704\uc8fc\uc758 \ud68d\uc77c\ud654\ub41c \uc815\ucc45\uc774\ub77c\ub294 \ubd80\uc815\uc801 \ud3c9\uac00 \ub4f1\uc774 \ub4a4\ub530\ub974\uace0 \uc788\ub2e4. \ub2e4\ub9cc, \uc81c1\uacf5\ud654\uad6d\uacfc \uc81c2\uacf5\ud654\uad6d\uc5d0\uc11c \uafb8\uc900\ud788 \uc2e4\uc2dc\ub418\uc5c8\ub358 \uc0ac\ubc29 \uc0ac\uc5c5, \uc870\ub9bc \uc0ac\uc5c5, \uc0b0\ub9bc \ubcf5\uc6d0 \uc0ac\uc5c5 \ub4f1\uc5d0 \uad00\ud558\uc5ec\ub294 \ud638\ud3c9\uc774 \ub354 \ub9ce\uc740 \ud3b8\uc774\ub2e4. \uac01\uc885 \uc5ec\ub860\uc870\uc0ac\uc5d0\uc11c\ub294 \uc874\uacbd\ud558\ub294 \ub300\ud1b5\ub839\uc744 \ube44\ub86f\ud55c \ub300\ud1b5\ub839 \uc120\ud638\ub3c4\uc640 \uacf5\uc801 \ubd80\ubb38 \ub4f1\uc5d0\uc11c \ucd5c\uc0c1\uc704 \ud3c9\uac00\ub97c \ubc1b\uc544\uc624\uace0 \uc788\ub2e4."} {"question": "\ud398\uc774\uc2a4\ubd81\uc744 10\ub144 \ucd5c\uace0 \ub9ac\uc2a4\ud2b8\uc5d0 \uc62c\ub824 \uc5b8\uae09\ud55c \uc5d4\ud130\ud14c\uc774\uba3c\ud2b8 \uc5c5\uccb4\ub294?", "paragraph": "\"Compete.com\"\uc5d0 \ub530\ub974\uba74 \ud398\uc774\uc2a4\ubd81\uc740 \ud65c\ub3d9 \uc0ac\uc6a9\uc790\ub4e4 \uac00\uc6b4\ub370 \uac00\uc7a5 \ub9ce\uc774 \uc774\uc6a9\ub418\ub294 \uc18c\uc15c \ub124\ud2b8\uc6cc\ud06c \uc11c\ube44\uc2a4\ub77c\uace0 \ud55c\ub2e4. \uc5d4\ud130\ud14c\uc778\uba3c\ud2b8 \uc704\ud074\ub9ac\uc5d0\uc11c\ub294 \ud398\uc774\uc2a4\ubd81\uc744 \uc774\ubc88 10\ub144 \ucd5c\uace0 \ub9ac\uc2a4\ud2b8\uc5d0 \uc62c\ub9ac\uba74\uc11c \"\ud398\uc774\uc2a4\ubd81\uc774 \uc788\uae30 \uc804\uc5d0\ub294 \uc5b4\ub5bb\uac8c \uc6b0\ub9ac\uac00 \uc804 \uc560\uc778\uc758 \uadfc\ud669\uc744 \uc54c\uc544\ubcf4\uace0, \uc6b0\ub9ac \ub3d9\ub8cc\uc758 \uc0dd\uc77c\uc744 \uae30\uc5b5\ud558\uace0,\uce5c\uad6c\ub97c \ub180\ub838\uc744\uae4c?\"\ub77c\uace0 \uc5b8\uae09\ud588\ub2e4. Quantcast\ub294 \ub9e4\ub2ec\ub9c8\ub2e4 1\uc5b5 3890\ub9cc \uba85\uc758 \uc0c8\ub85c\uc6b4 \ubbf8\uad6d \ubc29\ubb38\uc790\ub4e4\uc774 \uc788\ub2e4\uace0 \ud55c\ub2e4. \uc18c\uc15c \ubbf8\ub514\uc5b4 \ud22c\ub370\uc774\uc5d0 \ub530\ub974\uba74 2010\ub144 4\uc6d4 \uae30\uc900 \ubbf8\uad6d \uc778\uad6c\uc758 41.6%\uac00 \ud398\uc774\uc2a4\ubd81 \uacc4\uc815\uc744 \uac16\uace0 \uc788\ub2e4\uace0 \ud55c\ub2e4. \uadf8\ub7fc\uc5d0\ub3c4 \ubd88\uad6c\ud558\uace0 \ud398\uc774\uc2a4\ubd81\uc758 \uc131\uc7a5\uc740 \uc77c\ubd80 \uc9c0\uc5ed\uc5d0\uc11c \uc81c\ub3d9\uc774 \uac78\ub838\uc73c\uba70, \ubbf8\uad6d\uacfc \uce90\ub098\ub2e4\uc5d0\uc11c\ub9cc 700\ub9cc \uba85\uc758 \ud65c\ub3d9 \uc0ac\uc6a9\uc790\ub4e4\uc774 \uc5c6\uc5b4\uc9c4 \uc0c1\ud0dc\uc774\ub2e4. 2012\ub144 9\uc6d4, \ub9c8\ud06c \uc800\ucee4\ubc84\uadf8\ub294 \ud68c\uc0ac\uc758 \uc2dc\uc7a5\uac00\uce58 \ud558\ub77d\uc5d0 \uad00\ud574 \uc774\uc57c\uae30\ud558\uba74\uc11c \uc774\ub7ec\ud55c \uac10\uc18c\ub97c \"\uc2e4\ub9dd\uc801\"\uc774\ub77c\uace0 \ud45c\ud604\ud588\ub2e4. 2015\ub144 8\uc6d4 \ud604\uc7ac, \ud398\uc774\uc2a4\ubd81\uc758 \uc6d4 \ud65c\ub3d9 \uc0ac\uc6a9\uc790 \ubc0f \uc77c \ud65c\ub3d9 \uc0ac\uc6a9\uc790\ub294 \uc804\ub144 \ub3d9\uae30 \ub300\ube44 13%\uc640 17%\uac00 \uc99d\uac00\ud574 14\uc5b5 9\ucc9c\ub9cc \uba85\uacfc 9\uc5b5 6\ucc9c 8\ubc31\ub9cc \uba85\uc744 \uae30\ub85d\ud588\ub2e4. \uc544\uc6b8\ub7ec, \ubaa8\ubc14\uc77c \uc6d4 \ud65c\ub3d9 \uc0ac\uc6a9\uc790\uc640 \ubaa8\ubc14\uc77c \uc77c \ud65c\ub3d9 \uc0ac\uc6a9\uc790 \uc218\uce58\ub294 13\uc5b5 1\ucc9c\ub9cc \uba85\uacfc 8\uc5b5 4\ucc9c 4\ubc31\ub9cc \uba85\uc73c\ub85c, \uc804\uccb4 \uc6d4 \ud65c\ub3d9 \uc0ac\uc6a9\uc790\uc758 65% \uac00\ub7c9\uc774 \ub9e4\uc77c \ud398\uc774\uc2a4\ubd81\uc744 \uc0ac\uc6a9\ud558\uace0 \uc788\ub2e4. \ub610\ud55c, \uc804 \uc138\uacc4\uc5d0\uc11c \uc778\ud130\ub137\uc5d0 \uc811\uc18d\ud560 \uc218 \uc788\ub294 \uc778\uad6c\uc778 30\uc5b5 \uba85\uc758 \uc808\ubc18 \uac00\ub7c9\uc774 \ud398\uc774\uc2a4\ubd81\uc744 \uc0ac\uc6a9\ud558\uace0 \uc788\ub294 \uac83\uc73c\ub85c \ub098\ud0c0\ub0ac\ub2e4 (2015\ub144 2\ubd84\uae30 \uc6d4 \ud65c\ub3d9 \uc0ac\uc6a9\uc790 \uae30\uc900).", "answer": "\uc704\ud074\ub9ac", "sentence": "\uc5d4\ud130\ud14c\uc778\uba3c\ud2b8 \uc704\ud074\ub9ac \uc5d0\uc11c\ub294 \ud398\uc774\uc2a4\ubd81\uc744 \uc774\ubc88 10\ub144 \ucd5c\uace0 \ub9ac\uc2a4\ud2b8\uc5d0 \uc62c\ub9ac\uba74\uc11c \"\ud398\uc774\uc2a4\ubd81\uc774 \uc788\uae30 \uc804\uc5d0\ub294 \uc5b4\ub5bb\uac8c \uc6b0\ub9ac\uac00 \uc804 \uc560\uc778\uc758 \uadfc\ud669\uc744 \uc54c\uc544\ubcf4\uace0, \uc6b0\ub9ac \ub3d9\ub8cc\uc758 \uc0dd\uc77c\uc744 \uae30\uc5b5\ud558\uace0,\uce5c\uad6c\ub97c \ub180\ub838\uc744\uae4c?\"\ub77c\uace0 \uc5b8\uae09\ud588\ub2e4.", "paragraph_sentence": "\"Compete.com\"\uc5d0 \ub530\ub974\uba74 \ud398\uc774\uc2a4\ubd81\uc740 \ud65c\ub3d9 \uc0ac\uc6a9\uc790\ub4e4 \uac00\uc6b4\ub370 \uac00\uc7a5 \ub9ce\uc774 \uc774\uc6a9\ub418\ub294 \uc18c\uc15c \ub124\ud2b8\uc6cc\ud06c \uc11c\ube44\uc2a4\ub77c\uace0 \ud55c\ub2e4. \uc5d4\ud130\ud14c\uc778\uba3c\ud2b8 \uc704\ud074\ub9ac \uc5d0\uc11c\ub294 \ud398\uc774\uc2a4\ubd81\uc744 \uc774\ubc88 10\ub144 \ucd5c\uace0 \ub9ac\uc2a4\ud2b8\uc5d0 \uc62c\ub9ac\uba74\uc11c \"\ud398\uc774\uc2a4\ubd81\uc774 \uc788\uae30 \uc804\uc5d0\ub294 \uc5b4\ub5bb\uac8c \uc6b0\ub9ac\uac00 \uc804 \uc560\uc778\uc758 \uadfc\ud669\uc744 \uc54c\uc544\ubcf4\uace0, \uc6b0\ub9ac \ub3d9\ub8cc\uc758 \uc0dd\uc77c\uc744 \uae30\uc5b5\ud558\uace0,\uce5c\uad6c\ub97c \ub180\ub838\uc744\uae4c?\"\ub77c\uace0 \uc5b8\uae09\ud588\ub2e4. Quantcast\ub294 \ub9e4\ub2ec\ub9c8\ub2e4 1\uc5b5 3890\ub9cc \uba85\uc758 \uc0c8\ub85c\uc6b4 \ubbf8\uad6d \ubc29\ubb38\uc790\ub4e4\uc774 \uc788\ub2e4\uace0 \ud55c\ub2e4. \uc18c\uc15c \ubbf8\ub514\uc5b4 \ud22c\ub370\uc774\uc5d0 \ub530\ub974\uba74 2010\ub144 4\uc6d4 \uae30\uc900 \ubbf8\uad6d \uc778\uad6c\uc758 41.6%\uac00 \ud398\uc774\uc2a4\ubd81 \uacc4\uc815\uc744 \uac16\uace0 \uc788\ub2e4\uace0 \ud55c\ub2e4. \uadf8\ub7fc\uc5d0\ub3c4 \ubd88\uad6c\ud558\uace0 \ud398\uc774\uc2a4\ubd81\uc758 \uc131\uc7a5\uc740 \uc77c\ubd80 \uc9c0\uc5ed\uc5d0\uc11c \uc81c\ub3d9\uc774 \uac78\ub838\uc73c\uba70, \ubbf8\uad6d\uacfc \uce90\ub098\ub2e4\uc5d0\uc11c\ub9cc 700\ub9cc \uba85\uc758 \ud65c\ub3d9 \uc0ac\uc6a9\uc790\ub4e4\uc774 \uc5c6\uc5b4\uc9c4 \uc0c1\ud0dc\uc774\ub2e4. 2012\ub144 9\uc6d4, \ub9c8\ud06c \uc800\ucee4\ubc84\uadf8\ub294 \ud68c\uc0ac\uc758 \uc2dc\uc7a5\uac00\uce58 \ud558\ub77d\uc5d0 \uad00\ud574 \uc774\uc57c\uae30\ud558\uba74\uc11c \uc774\ub7ec\ud55c \uac10\uc18c\ub97c \"\uc2e4\ub9dd\uc801\"\uc774\ub77c\uace0 \ud45c\ud604\ud588\ub2e4. 2015\ub144 8\uc6d4 \ud604\uc7ac, \ud398\uc774\uc2a4\ubd81\uc758 \uc6d4 \ud65c\ub3d9 \uc0ac\uc6a9\uc790 \ubc0f \uc77c \ud65c\ub3d9 \uc0ac\uc6a9\uc790\ub294 \uc804\ub144 \ub3d9\uae30 \ub300\ube44 13%\uc640 17%\uac00 \uc99d\uac00\ud574 14\uc5b5 9\ucc9c\ub9cc \uba85\uacfc 9\uc5b5 6\ucc9c 8\ubc31\ub9cc \uba85\uc744 \uae30\ub85d\ud588\ub2e4. \uc544\uc6b8\ub7ec, \ubaa8\ubc14\uc77c \uc6d4 \ud65c\ub3d9 \uc0ac\uc6a9\uc790\uc640 \ubaa8\ubc14\uc77c \uc77c \ud65c\ub3d9 \uc0ac\uc6a9\uc790 \uc218\uce58\ub294 13\uc5b5 1\ucc9c\ub9cc \uba85\uacfc 8\uc5b5 4\ucc9c 4\ubc31\ub9cc \uba85\uc73c\ub85c, \uc804\uccb4 \uc6d4 \ud65c\ub3d9 \uc0ac\uc6a9\uc790\uc758 65% \uac00\ub7c9\uc774 \ub9e4\uc77c \ud398\uc774\uc2a4\ubd81\uc744 \uc0ac\uc6a9\ud558\uace0 \uc788\ub2e4. \ub610\ud55c, \uc804 \uc138\uacc4\uc5d0\uc11c \uc778\ud130\ub137\uc5d0 \uc811\uc18d\ud560 \uc218 \uc788\ub294 \uc778\uad6c\uc778 30\uc5b5 \uba85\uc758 \uc808\ubc18 \uac00\ub7c9\uc774 \ud398\uc774\uc2a4\ubd81\uc744 \uc0ac\uc6a9\ud558\uace0 \uc788\ub294 \uac83\uc73c\ub85c \ub098\ud0c0\ub0ac\ub2e4 (2015\ub144 2\ubd84\uae30 \uc6d4 \ud65c\ub3d9 \uc0ac\uc6a9\uc790 \uae30\uc900).", "paragraph_answer": "\"Compete.com\"\uc5d0 \ub530\ub974\uba74 \ud398\uc774\uc2a4\ubd81\uc740 \ud65c\ub3d9 \uc0ac\uc6a9\uc790\ub4e4 \uac00\uc6b4\ub370 \uac00\uc7a5 \ub9ce\uc774 \uc774\uc6a9\ub418\ub294 \uc18c\uc15c \ub124\ud2b8\uc6cc\ud06c \uc11c\ube44\uc2a4\ub77c\uace0 \ud55c\ub2e4. \uc5d4\ud130\ud14c\uc778\uba3c\ud2b8 \uc704\ud074\ub9ac \uc5d0\uc11c\ub294 \ud398\uc774\uc2a4\ubd81\uc744 \uc774\ubc88 10\ub144 \ucd5c\uace0 \ub9ac\uc2a4\ud2b8\uc5d0 \uc62c\ub9ac\uba74\uc11c \"\ud398\uc774\uc2a4\ubd81\uc774 \uc788\uae30 \uc804\uc5d0\ub294 \uc5b4\ub5bb\uac8c \uc6b0\ub9ac\uac00 \uc804 \uc560\uc778\uc758 \uadfc\ud669\uc744 \uc54c\uc544\ubcf4\uace0, \uc6b0\ub9ac \ub3d9\ub8cc\uc758 \uc0dd\uc77c\uc744 \uae30\uc5b5\ud558\uace0,\uce5c\uad6c\ub97c \ub180\ub838\uc744\uae4c?\"\ub77c\uace0 \uc5b8\uae09\ud588\ub2e4. Quantcast\ub294 \ub9e4\ub2ec\ub9c8\ub2e4 1\uc5b5 3890\ub9cc \uba85\uc758 \uc0c8\ub85c\uc6b4 \ubbf8\uad6d \ubc29\ubb38\uc790\ub4e4\uc774 \uc788\ub2e4\uace0 \ud55c\ub2e4. \uc18c\uc15c \ubbf8\ub514\uc5b4 \ud22c\ub370\uc774\uc5d0 \ub530\ub974\uba74 2010\ub144 4\uc6d4 \uae30\uc900 \ubbf8\uad6d \uc778\uad6c\uc758 41.6%\uac00 \ud398\uc774\uc2a4\ubd81 \uacc4\uc815\uc744 \uac16\uace0 \uc788\ub2e4\uace0 \ud55c\ub2e4. \uadf8\ub7fc\uc5d0\ub3c4 \ubd88\uad6c\ud558\uace0 \ud398\uc774\uc2a4\ubd81\uc758 \uc131\uc7a5\uc740 \uc77c\ubd80 \uc9c0\uc5ed\uc5d0\uc11c \uc81c\ub3d9\uc774 \uac78\ub838\uc73c\uba70, \ubbf8\uad6d\uacfc \uce90\ub098\ub2e4\uc5d0\uc11c\ub9cc 700\ub9cc \uba85\uc758 \ud65c\ub3d9 \uc0ac\uc6a9\uc790\ub4e4\uc774 \uc5c6\uc5b4\uc9c4 \uc0c1\ud0dc\uc774\ub2e4. 2012\ub144 9\uc6d4, \ub9c8\ud06c \uc800\ucee4\ubc84\uadf8\ub294 \ud68c\uc0ac\uc758 \uc2dc\uc7a5\uac00\uce58 \ud558\ub77d\uc5d0 \uad00\ud574 \uc774\uc57c\uae30\ud558\uba74\uc11c \uc774\ub7ec\ud55c \uac10\uc18c\ub97c \"\uc2e4\ub9dd\uc801\"\uc774\ub77c\uace0 \ud45c\ud604\ud588\ub2e4. 2015\ub144 8\uc6d4 \ud604\uc7ac, \ud398\uc774\uc2a4\ubd81\uc758 \uc6d4 \ud65c\ub3d9 \uc0ac\uc6a9\uc790 \ubc0f \uc77c \ud65c\ub3d9 \uc0ac\uc6a9\uc790\ub294 \uc804\ub144 \ub3d9\uae30 \ub300\ube44 13%\uc640 17%\uac00 \uc99d\uac00\ud574 14\uc5b5 9\ucc9c\ub9cc \uba85\uacfc 9\uc5b5 6\ucc9c 8\ubc31\ub9cc \uba85\uc744 \uae30\ub85d\ud588\ub2e4. \uc544\uc6b8\ub7ec, \ubaa8\ubc14\uc77c \uc6d4 \ud65c\ub3d9 \uc0ac\uc6a9\uc790\uc640 \ubaa8\ubc14\uc77c \uc77c \ud65c\ub3d9 \uc0ac\uc6a9\uc790 \uc218\uce58\ub294 13\uc5b5 1\ucc9c\ub9cc \uba85\uacfc 8\uc5b5 4\ucc9c 4\ubc31\ub9cc \uba85\uc73c\ub85c, \uc804\uccb4 \uc6d4 \ud65c\ub3d9 \uc0ac\uc6a9\uc790\uc758 65% \uac00\ub7c9\uc774 \ub9e4\uc77c \ud398\uc774\uc2a4\ubd81\uc744 \uc0ac\uc6a9\ud558\uace0 \uc788\ub2e4. \ub610\ud55c, \uc804 \uc138\uacc4\uc5d0\uc11c \uc778\ud130\ub137\uc5d0 \uc811\uc18d\ud560 \uc218 \uc788\ub294 \uc778\uad6c\uc778 30\uc5b5 \uba85\uc758 \uc808\ubc18 \uac00\ub7c9\uc774 \ud398\uc774\uc2a4\ubd81\uc744 \uc0ac\uc6a9\ud558\uace0 \uc788\ub294 \uac83\uc73c\ub85c \ub098\ud0c0\ub0ac\ub2e4 (2015\ub144 2\ubd84\uae30 \uc6d4 \ud65c\ub3d9 \uc0ac\uc6a9\uc790 \uae30\uc900).", "sentence_answer": "\uc5d4\ud130\ud14c\uc778\uba3c\ud2b8 \uc704\ud074\ub9ac \uc5d0\uc11c\ub294 \ud398\uc774\uc2a4\ubd81\uc744 \uc774\ubc88 10\ub144 \ucd5c\uace0 \ub9ac\uc2a4\ud2b8\uc5d0 \uc62c\ub9ac\uba74\uc11c \"\ud398\uc774\uc2a4\ubd81\uc774 \uc788\uae30 \uc804\uc5d0\ub294 \uc5b4\ub5bb\uac8c \uc6b0\ub9ac\uac00 \uc804 \uc560\uc778\uc758 \uadfc\ud669\uc744 \uc54c\uc544\ubcf4\uace0, \uc6b0\ub9ac \ub3d9\ub8cc\uc758 \uc0dd\uc77c\uc744 \uae30\uc5b5\ud558\uace0,\uce5c\uad6c\ub97c \ub180\ub838\uc744\uae4c?\"\ub77c\uace0 \uc5b8\uae09\ud588\ub2e4.", "paragraph_id": "5920606-0-0", "paragraph_question": "question: \ud398\uc774\uc2a4\ubd81\uc744 10\ub144 \ucd5c\uace0 \ub9ac\uc2a4\ud2b8\uc5d0 \uc62c\ub824 \uc5b8\uae09\ud55c \uc5d4\ud130\ud14c\uc774\uba3c\ud2b8 \uc5c5\uccb4\ub294?, context: \"Compete.com\"\uc5d0 \ub530\ub974\uba74 \ud398\uc774\uc2a4\ubd81\uc740 \ud65c\ub3d9 \uc0ac\uc6a9\uc790\ub4e4 \uac00\uc6b4\ub370 \uac00\uc7a5 \ub9ce\uc774 \uc774\uc6a9\ub418\ub294 \uc18c\uc15c \ub124\ud2b8\uc6cc\ud06c \uc11c\ube44\uc2a4\ub77c\uace0 \ud55c\ub2e4. \uc5d4\ud130\ud14c\uc778\uba3c\ud2b8 \uc704\ud074\ub9ac\uc5d0\uc11c\ub294 \ud398\uc774\uc2a4\ubd81\uc744 \uc774\ubc88 10\ub144 \ucd5c\uace0 \ub9ac\uc2a4\ud2b8\uc5d0 \uc62c\ub9ac\uba74\uc11c \"\ud398\uc774\uc2a4\ubd81\uc774 \uc788\uae30 \uc804\uc5d0\ub294 \uc5b4\ub5bb\uac8c \uc6b0\ub9ac\uac00 \uc804 \uc560\uc778\uc758 \uadfc\ud669\uc744 \uc54c\uc544\ubcf4\uace0, \uc6b0\ub9ac \ub3d9\ub8cc\uc758 \uc0dd\uc77c\uc744 \uae30\uc5b5\ud558\uace0,\uce5c\uad6c\ub97c \ub180\ub838\uc744\uae4c?\"\ub77c\uace0 \uc5b8\uae09\ud588\ub2e4. Quantcast\ub294 \ub9e4\ub2ec\ub9c8\ub2e4 1\uc5b5 3890\ub9cc \uba85\uc758 \uc0c8\ub85c\uc6b4 \ubbf8\uad6d \ubc29\ubb38\uc790\ub4e4\uc774 \uc788\ub2e4\uace0 \ud55c\ub2e4. \uc18c\uc15c \ubbf8\ub514\uc5b4 \ud22c\ub370\uc774\uc5d0 \ub530\ub974\uba74 2010\ub144 4\uc6d4 \uae30\uc900 \ubbf8\uad6d \uc778\uad6c\uc758 41.6%\uac00 \ud398\uc774\uc2a4\ubd81 \uacc4\uc815\uc744 \uac16\uace0 \uc788\ub2e4\uace0 \ud55c\ub2e4. \uadf8\ub7fc\uc5d0\ub3c4 \ubd88\uad6c\ud558\uace0 \ud398\uc774\uc2a4\ubd81\uc758 \uc131\uc7a5\uc740 \uc77c\ubd80 \uc9c0\uc5ed\uc5d0\uc11c \uc81c\ub3d9\uc774 \uac78\ub838\uc73c\uba70, \ubbf8\uad6d\uacfc \uce90\ub098\ub2e4\uc5d0\uc11c\ub9cc 700\ub9cc \uba85\uc758 \ud65c\ub3d9 \uc0ac\uc6a9\uc790\ub4e4\uc774 \uc5c6\uc5b4\uc9c4 \uc0c1\ud0dc\uc774\ub2e4. 2012\ub144 9\uc6d4, \ub9c8\ud06c \uc800\ucee4\ubc84\uadf8\ub294 \ud68c\uc0ac\uc758 \uc2dc\uc7a5\uac00\uce58 \ud558\ub77d\uc5d0 \uad00\ud574 \uc774\uc57c\uae30\ud558\uba74\uc11c \uc774\ub7ec\ud55c \uac10\uc18c\ub97c \"\uc2e4\ub9dd\uc801\"\uc774\ub77c\uace0 \ud45c\ud604\ud588\ub2e4. 2015\ub144 8\uc6d4 \ud604\uc7ac, \ud398\uc774\uc2a4\ubd81\uc758 \uc6d4 \ud65c\ub3d9 \uc0ac\uc6a9\uc790 \ubc0f \uc77c \ud65c\ub3d9 \uc0ac\uc6a9\uc790\ub294 \uc804\ub144 \ub3d9\uae30 \ub300\ube44 13%\uc640 17%\uac00 \uc99d\uac00\ud574 14\uc5b5 9\ucc9c\ub9cc \uba85\uacfc 9\uc5b5 6\ucc9c 8\ubc31\ub9cc \uba85\uc744 \uae30\ub85d\ud588\ub2e4. \uc544\uc6b8\ub7ec, \ubaa8\ubc14\uc77c \uc6d4 \ud65c\ub3d9 \uc0ac\uc6a9\uc790\uc640 \ubaa8\ubc14\uc77c \uc77c \ud65c\ub3d9 \uc0ac\uc6a9\uc790 \uc218\uce58\ub294 13\uc5b5 1\ucc9c\ub9cc \uba85\uacfc 8\uc5b5 4\ucc9c 4\ubc31\ub9cc \uba85\uc73c\ub85c, \uc804\uccb4 \uc6d4 \ud65c\ub3d9 \uc0ac\uc6a9\uc790\uc758 65% \uac00\ub7c9\uc774 \ub9e4\uc77c \ud398\uc774\uc2a4\ubd81\uc744 \uc0ac\uc6a9\ud558\uace0 \uc788\ub2e4. \ub610\ud55c, \uc804 \uc138\uacc4\uc5d0\uc11c \uc778\ud130\ub137\uc5d0 \uc811\uc18d\ud560 \uc218 \uc788\ub294 \uc778\uad6c\uc778 30\uc5b5 \uba85\uc758 \uc808\ubc18 \uac00\ub7c9\uc774 \ud398\uc774\uc2a4\ubd81\uc744 \uc0ac\uc6a9\ud558\uace0 \uc788\ub294 \uac83\uc73c\ub85c \ub098\ud0c0\ub0ac\ub2e4 (2015\ub144 2\ubd84\uae30 \uc6d4 \ud65c\ub3d9 \uc0ac\uc6a9\uc790 \uae30\uc900)."} {"question": "\uc8fc\uc138\ud398 \uce74\uc2a4\ud2f8\ub9ac\uc624\ub124\uac00 \uc120\uad50\uc0ac\ub85c \ud30c\uacac\ub41c \ub098\ub77c\ub294?", "paragraph": "\uadf8\ub294 \ub9c8\uce74\uc624\uc5d0\uc11c \uc911\uad6d\uc5b4\ub97c \ud559\uc2b5\ud558\uc600\uace0, \uc911\uad6d\uc5d0\uc11c \ub0ad\uc138\ub155\uc774\ub77c\ub294 \uc774\ub984\uc73c\ub85c \ud65c\ub3d9\ud558\uc600\ub2e4. \uac15\ud76c\uc81c(\uc7ac\uc704: 1661\ub144 ~ 1722\ub144), \uc639\uc815\uc81c(\uc7ac\uc704: 1722\ub144 ~ 1735\ub144), \uac74\ub96d\uc81c(\uc7ac\uc704: 1735\ub144 ~ 1796\ub144)\uc758 \uc8fc\ubaa9\uc744 \ubc1b\uc558\uace0, \ud669\uc2e4 \uc6a9\ud488 \uc81c\uc791 \uae30\uad00\uc774\uc5c8\ub358 \uc870\ud310\ucc98(\u9020\u8fa6\u8655) \ub0b4\uc758 \ud68c\ud654 \uc81c\uc791 \ubd80\uc11c\uc778 \uc5ec\uc758\uad00(\u5982\u610f\u9928)\uc5d0 \uc18c\uc18d\ub41c \uad81\uc815 \ud654\uac00\ub85c 50\uc5ec \ub144\uac04 \ud65c\uc57d\ud558\uc600\ub2e4. \uc774\ud0c8\ub9ac\uc544\uc5d0\uc11c \uae30\ucd08\ub97c \ub2e6\uc740 \uc11c\uc591\ud654 \uae30\ubc95\uc744 \ubc14\ud0d5\uc73c\ub85c \uc911\uad6d\uc5d0 \uc640\uc11c \ub2e4\uc2dc \uc911\uad6d\ud654\ub97c \ubc30\uc6e0\uace0, \ucd5c\ucd08\ub85c \ub450 \uc591\uc2dd\uc744 \ud63c\ud569\ud55c \uc0c8\ub85c\uc6b4 \ud615\ud0dc\uc758 \uadf8\ub9bc\uc744 \ub9cc\ub4e4\uc5b4\ub0c8\ub2e4. \uc911\uad6d\uc5d0 \uc120\uad50\uc0ac\ub85c \ud30c\uacac\ub418\uc5c8\uc9c0\ub9cc \uc120\uad50\uc0ac\ubcf4\ub2e4\ub294 \uad81\uc815 \ud654\uac00\ub85c\uc11c \ud65c\ub3d9\ud558\uc600\uace0, \uc639\uc815\uc81c\u00b7\uac74\ub96d\uc81c \ub54c\uc5d0 \uadf8\ub9ac\uc2a4\ub3c4\uad50\uc5d0 \ub300\ud55c \ubc15\ud574\uc5d0\ub3c4 \ubd88\uad6c\ud558\uace0 \ud669\uc81c\uc758 \ucd1d\uc560 \ub54c\ubb38\uc5d0 \ubaa8\uba74\ud560 \uc218 \uc788\uc5c8\ub2e4.", "answer": "\uc911\uad6d", "sentence": "\uadf8\ub294 \ub9c8\uce74\uc624\uc5d0\uc11c \uc911\uad6d \uc5b4\ub97c \ud559\uc2b5\ud558\uc600\uace0, \uc911\uad6d\uc5d0\uc11c \ub0ad\uc138\ub155\uc774\ub77c\ub294 \uc774\ub984\uc73c\ub85c \ud65c\ub3d9\ud558\uc600\ub2e4.", "paragraph_sentence": " \uadf8\ub294 \ub9c8\uce74\uc624\uc5d0\uc11c \uc911\uad6d \uc5b4\ub97c \ud559\uc2b5\ud558\uc600\uace0, \uc911\uad6d\uc5d0\uc11c \ub0ad\uc138\ub155\uc774\ub77c\ub294 \uc774\ub984\uc73c\ub85c \ud65c\ub3d9\ud558\uc600\ub2e4. \uac15\ud76c\uc81c(\uc7ac\uc704: 1661\ub144 ~ 1722\ub144), \uc639\uc815\uc81c(\uc7ac\uc704: 1722\ub144 ~ 1735\ub144), \uac74\ub96d\uc81c(\uc7ac\uc704: 1735\ub144 ~ 1796\ub144)\uc758 \uc8fc\ubaa9\uc744 \ubc1b\uc558\uace0, \ud669\uc2e4 \uc6a9\ud488 \uc81c\uc791 \uae30\uad00\uc774\uc5c8\ub358 \uc870\ud310\ucc98(\u9020\u8fa6\u8655) \ub0b4\uc758 \ud68c\ud654 \uc81c\uc791 \ubd80\uc11c\uc778 \uc5ec\uc758\uad00(\u5982\u610f\u9928)\uc5d0 \uc18c\uc18d\ub41c \uad81\uc815 \ud654\uac00\ub85c 50\uc5ec \ub144\uac04 \ud65c\uc57d\ud558\uc600\ub2e4. \uc774\ud0c8\ub9ac\uc544\uc5d0\uc11c \uae30\ucd08\ub97c \ub2e6\uc740 \uc11c\uc591\ud654 \uae30\ubc95\uc744 \ubc14\ud0d5\uc73c\ub85c \uc911\uad6d\uc5d0 \uc640\uc11c \ub2e4\uc2dc \uc911\uad6d\ud654\ub97c \ubc30\uc6e0\uace0, \ucd5c\ucd08\ub85c \ub450 \uc591\uc2dd\uc744 \ud63c\ud569\ud55c \uc0c8\ub85c\uc6b4 \ud615\ud0dc\uc758 \uadf8\ub9bc\uc744 \ub9cc\ub4e4\uc5b4\ub0c8\ub2e4. \uc911\uad6d\uc5d0 \uc120\uad50\uc0ac\ub85c \ud30c\uacac\ub418\uc5c8\uc9c0\ub9cc \uc120\uad50\uc0ac\ubcf4\ub2e4\ub294 \uad81\uc815 \ud654\uac00\ub85c\uc11c \ud65c\ub3d9\ud558\uc600\uace0, \uc639\uc815\uc81c\u00b7\uac74\ub96d\uc81c \ub54c\uc5d0 \uadf8\ub9ac\uc2a4\ub3c4\uad50\uc5d0 \ub300\ud55c \ubc15\ud574\uc5d0\ub3c4 \ubd88\uad6c\ud558\uace0 \ud669\uc81c\uc758 \ucd1d\uc560 \ub54c\ubb38\uc5d0 \ubaa8\uba74\ud560 \uc218 \uc788\uc5c8\ub2e4.", "paragraph_answer": "\uadf8\ub294 \ub9c8\uce74\uc624\uc5d0\uc11c \uc911\uad6d \uc5b4\ub97c \ud559\uc2b5\ud558\uc600\uace0, \uc911\uad6d\uc5d0\uc11c \ub0ad\uc138\ub155\uc774\ub77c\ub294 \uc774\ub984\uc73c\ub85c \ud65c\ub3d9\ud558\uc600\ub2e4. \uac15\ud76c\uc81c(\uc7ac\uc704: 1661\ub144 ~ 1722\ub144), \uc639\uc815\uc81c(\uc7ac\uc704: 1722\ub144 ~ 1735\ub144), \uac74\ub96d\uc81c(\uc7ac\uc704: 1735\ub144 ~ 1796\ub144)\uc758 \uc8fc\ubaa9\uc744 \ubc1b\uc558\uace0, \ud669\uc2e4 \uc6a9\ud488 \uc81c\uc791 \uae30\uad00\uc774\uc5c8\ub358 \uc870\ud310\ucc98(\u9020\u8fa6\u8655) \ub0b4\uc758 \ud68c\ud654 \uc81c\uc791 \ubd80\uc11c\uc778 \uc5ec\uc758\uad00(\u5982\u610f\u9928)\uc5d0 \uc18c\uc18d\ub41c \uad81\uc815 \ud654\uac00\ub85c 50\uc5ec \ub144\uac04 \ud65c\uc57d\ud558\uc600\ub2e4. \uc774\ud0c8\ub9ac\uc544\uc5d0\uc11c \uae30\ucd08\ub97c \ub2e6\uc740 \uc11c\uc591\ud654 \uae30\ubc95\uc744 \ubc14\ud0d5\uc73c\ub85c \uc911\uad6d\uc5d0 \uc640\uc11c \ub2e4\uc2dc \uc911\uad6d\ud654\ub97c \ubc30\uc6e0\uace0, \ucd5c\ucd08\ub85c \ub450 \uc591\uc2dd\uc744 \ud63c\ud569\ud55c \uc0c8\ub85c\uc6b4 \ud615\ud0dc\uc758 \uadf8\ub9bc\uc744 \ub9cc\ub4e4\uc5b4\ub0c8\ub2e4. \uc911\uad6d\uc5d0 \uc120\uad50\uc0ac\ub85c \ud30c\uacac\ub418\uc5c8\uc9c0\ub9cc \uc120\uad50\uc0ac\ubcf4\ub2e4\ub294 \uad81\uc815 \ud654\uac00\ub85c\uc11c \ud65c\ub3d9\ud558\uc600\uace0, \uc639\uc815\uc81c\u00b7\uac74\ub96d\uc81c \ub54c\uc5d0 \uadf8\ub9ac\uc2a4\ub3c4\uad50\uc5d0 \ub300\ud55c \ubc15\ud574\uc5d0\ub3c4 \ubd88\uad6c\ud558\uace0 \ud669\uc81c\uc758 \ucd1d\uc560 \ub54c\ubb38\uc5d0 \ubaa8\uba74\ud560 \uc218 \uc788\uc5c8\ub2e4.", "sentence_answer": "\uadf8\ub294 \ub9c8\uce74\uc624\uc5d0\uc11c \uc911\uad6d \uc5b4\ub97c \ud559\uc2b5\ud558\uc600\uace0, \uc911\uad6d\uc5d0\uc11c \ub0ad\uc138\ub155\uc774\ub77c\ub294 \uc774\ub984\uc73c\ub85c \ud65c\ub3d9\ud558\uc600\ub2e4.", "paragraph_id": "5891590-0-0", "paragraph_question": "question: \uc8fc\uc138\ud398 \uce74\uc2a4\ud2f8\ub9ac\uc624\ub124\uac00 \uc120\uad50\uc0ac\ub85c \ud30c\uacac\ub41c \ub098\ub77c\ub294?, context: \uadf8\ub294 \ub9c8\uce74\uc624\uc5d0\uc11c \uc911\uad6d\uc5b4\ub97c \ud559\uc2b5\ud558\uc600\uace0, \uc911\uad6d\uc5d0\uc11c \ub0ad\uc138\ub155\uc774\ub77c\ub294 \uc774\ub984\uc73c\ub85c \ud65c\ub3d9\ud558\uc600\ub2e4. \uac15\ud76c\uc81c(\uc7ac\uc704: 1661\ub144 ~ 1722\ub144), \uc639\uc815\uc81c(\uc7ac\uc704: 1722\ub144 ~ 1735\ub144), \uac74\ub96d\uc81c(\uc7ac\uc704: 1735\ub144 ~ 1796\ub144)\uc758 \uc8fc\ubaa9\uc744 \ubc1b\uc558\uace0, \ud669\uc2e4 \uc6a9\ud488 \uc81c\uc791 \uae30\uad00\uc774\uc5c8\ub358 \uc870\ud310\ucc98(\u9020\u8fa6\u8655) \ub0b4\uc758 \ud68c\ud654 \uc81c\uc791 \ubd80\uc11c\uc778 \uc5ec\uc758\uad00(\u5982\u610f\u9928)\uc5d0 \uc18c\uc18d\ub41c \uad81\uc815 \ud654\uac00\ub85c 50\uc5ec \ub144\uac04 \ud65c\uc57d\ud558\uc600\ub2e4. \uc774\ud0c8\ub9ac\uc544\uc5d0\uc11c \uae30\ucd08\ub97c \ub2e6\uc740 \uc11c\uc591\ud654 \uae30\ubc95\uc744 \ubc14\ud0d5\uc73c\ub85c \uc911\uad6d\uc5d0 \uc640\uc11c \ub2e4\uc2dc \uc911\uad6d\ud654\ub97c \ubc30\uc6e0\uace0, \ucd5c\ucd08\ub85c \ub450 \uc591\uc2dd\uc744 \ud63c\ud569\ud55c \uc0c8\ub85c\uc6b4 \ud615\ud0dc\uc758 \uadf8\ub9bc\uc744 \ub9cc\ub4e4\uc5b4\ub0c8\ub2e4. \uc911\uad6d\uc5d0 \uc120\uad50\uc0ac\ub85c \ud30c\uacac\ub418\uc5c8\uc9c0\ub9cc \uc120\uad50\uc0ac\ubcf4\ub2e4\ub294 \uad81\uc815 \ud654\uac00\ub85c\uc11c \ud65c\ub3d9\ud558\uc600\uace0, \uc639\uc815\uc81c\u00b7\uac74\ub96d\uc81c \ub54c\uc5d0 \uadf8\ub9ac\uc2a4\ub3c4\uad50\uc5d0 \ub300\ud55c \ubc15\ud574\uc5d0\ub3c4 \ubd88\uad6c\ud558\uace0 \ud669\uc81c\uc758 \ucd1d\uc560 \ub54c\ubb38\uc5d0 \ubaa8\uba74\ud560 \uc218 \uc788\uc5c8\ub2e4."} {"question": "\uc804\uc778\ubc94\uc774 2015\ub144 \ub274 \uc6e8\uc2a4\ud2b8\ubbfc\uc2a4\ud130 \ub300\ud559\uc5d0\uc11c \uc120\uc815\ub41c \uac83\uc740?", "paragraph": "2015\ub144 6\uc6d4 \ub274 \uc6e8\uc2a4\ud2b8\ubbfc\uc2a4\ud130 \ub300\ud559(New Westminster College)\uc758 \ud2b9\ubcc4\uc5f0\uad6c\uc6d0(Distinguished Fellow)\uc73c\ub85c \uc120\uc815\ub418\uc5c8\ub2e4. \ub274 \uc6e8\uc2a4\ud2b8\ubbfc\uc2a4\ud130 \ub300\ud559 \ucd1d\uc7a5 \ub85c\ubc84\ud2b8 \uad7f\uc708 3\uc138\ub294 \u201c\uadf8\ub294 \uc724\ub9ac\uc801 \ub9ac\ub354\uc2ed\uc5d0\uc11c \uad04\ubaa9\ud55c \uc131\uacfc\ub97c \ub0b4\uc5c8\uace0 \uad50\uc721\ud6c8\ub828\uc5d0 \ud3c9\uc0dd \ub3d9\uc548 \ud5cc\uc2e0\ud574\uc654\ub2e4\u201d\uba70 \u201c\ud604 \uc138\uacc4\uc758 \uad6d\uac00\uc801 \uc548\ubcf4 \uc704\ud611\uc5d0 \ub300\ud55c \uc774\ud574\ub294 \uadf8\uac00 \uc815\ubd80\uc758 \uad6d\uc81c\uc801, \uad6d\uac00\uc801, \uc9c0\uc5ed \ub2e8\uc704 \uc774\ud574\uad00\uacc4\uc790\ub4e4\uc758 \ubbfc\uad70 \uc0c1\ud638\uc6b4\uc6a9\uc131\uacfc \ubc29\uc704\ud611\uc57d\uc5d0 \ub300\ud55c \uac00\uce58 \uc788\ub294 \ud1b5\ucc30\ub825\uc744 \uac16\ucd94\uac8c \ud558\uc600\ub2e4. \uc774\uac83\uc740 \uc6b0\ub9ac\uc758 \ub9ac\ub354\uc2ed, \uc678\uad50\uc640 \uc0ac\uc774\ubc84 \ubcf4\uc548\uc5d0 \uad00\ud55c \uc5f0\uad6c\uc758 \uc124\uacc4\uc640 \uc804\ub2ec\uc5d0 \uc788\uc5b4 \uac00\uce58\ub97c \ub530\uc9c8 \uc218 \uc5c6\uc744 \uc815\ub3c4\ub85c \uadc0\uc911\ud55c \uac83\uc774\ub2e4. \uc704\uc6d0\ud68c\uc640 \ub098\ub294 \uadf8\uac00 \uc6b0\ub9ac \ub274 \uc6e8\uc2a4\ud2b8\ubbfc\uc2a4\ud130 \ub300\ud559\uc758 \uc0c8\ub85c\uc6b4 \ud2b9\ubcc4\uc5f0\uad6c\uc6d0\uc774 \ub41c \uac83\uc744 \ub9e4\uc6b0 \uc601\uad11\uc2a4\ub7fd\uac8c \uc0dd\uac01\ud55c\ub2e4\u201d\uace0 \ubc1d\ud614\ub2e4. \uc804\uc778\ubc94 \uc911\uc7a5\uc740 \ud2b9\ubcc4\uc5f0\uad6c\uc6d0\uc73c\ub85c\uc11c \ud559\ubb38\uc801 \uc218\uc6d4\uc131\uc758 \ucd5c\uace0\uc704 \uae30\uc900\uc5d0 \ub300\ud55c \uc9c0\uc6d0\uc5d0 \uacf5\ud5cc\ud574\uc654\uace0, \ub274 \uc6e8\uc2a4\ud2b8\ubbfc\uc2a4\ud130 \ub300\ud559\uacfc \uc5f0\uad6c\uc6d0\ub4e4\uc758 \uc138\uacc4\uc801\uc778 \uc870\uc9c1\ub9dd \ubc1c\uc804\uc5d0 \ud5cc\uc2e0\ud574\uc654\ub2e4\uace0 \uc54c\ub824\uc84c\ub2e4. \uc774 \ub300\ud559\uc758 \ud2b9\ubcc4\uc5f0\uad6c\uc6d0\uc73c\ub85c\ub294 \ud544\ub9ac\ud540 \ub300\ud1b5\ub839 \ud53c\ub378 \ub77c\ubaa8\uc2a4, \uccb4\ucf54 \ub300\ud1b5\ub839 \ubc14\uce28\ub77c\ud504 \ud074\ub77c\uc6b0\uc2a4, \uce90\ub098\ub2e4 \uad6d\ubc29\ucc38\ubaa8\ucd1d\uc7a5 \ub300\uc7a5 \ub808\uc774 \ud5e4\ub178 \ub4f1\uc774 \uc788\uc73c\uba70, \ud2b9\ubcc4\uc5f0\uad6c\uc6d0\uc73c\ub85c \uc120\uc815\ub41c \uc778\ubb3c 125\uba85 \uac00\uc6b4\ub370 \ud55c\uad6d\uc778\uc740 \uadf8\uac00 \ucd5c\ucd08\uc774\ub2e4.", "answer": "\ud2b9\ubcc4\uc5f0\uad6c\uc6d0", "sentence": "2015\ub144 6\uc6d4 \ub274 \uc6e8\uc2a4\ud2b8\ubbfc\uc2a4\ud130 \ub300\ud559(New Westminster College)\uc758 \ud2b9\ubcc4\uc5f0\uad6c\uc6d0 (Distinguished Fellow)\uc73c\ub85c \uc120\uc815\ub418\uc5c8\ub2e4.", "paragraph_sentence": " 2015\ub144 6\uc6d4 \ub274 \uc6e8\uc2a4\ud2b8\ubbfc\uc2a4\ud130 \ub300\ud559(New Westminster College)\uc758 \ud2b9\ubcc4\uc5f0\uad6c\uc6d0 (Distinguished Fellow)\uc73c\ub85c \uc120\uc815\ub418\uc5c8\ub2e4. \ub274 \uc6e8\uc2a4\ud2b8\ubbfc\uc2a4\ud130 \ub300\ud559 \ucd1d\uc7a5 \ub85c\ubc84\ud2b8 \uad7f\uc708 3\uc138\ub294 \u201c\uadf8\ub294 \uc724\ub9ac\uc801 \ub9ac\ub354\uc2ed\uc5d0\uc11c \uad04\ubaa9\ud55c \uc131\uacfc\ub97c \ub0b4\uc5c8\uace0 \uad50\uc721\ud6c8\ub828\uc5d0 \ud3c9\uc0dd \ub3d9\uc548 \ud5cc\uc2e0\ud574\uc654\ub2e4\u201d\uba70 \u201c\ud604 \uc138\uacc4\uc758 \uad6d\uac00\uc801 \uc548\ubcf4 \uc704\ud611\uc5d0 \ub300\ud55c \uc774\ud574\ub294 \uadf8\uac00 \uc815\ubd80\uc758 \uad6d\uc81c\uc801, \uad6d\uac00\uc801, \uc9c0\uc5ed \ub2e8\uc704 \uc774\ud574\uad00\uacc4\uc790\ub4e4\uc758 \ubbfc\uad70 \uc0c1\ud638\uc6b4\uc6a9\uc131\uacfc \ubc29\uc704\ud611\uc57d\uc5d0 \ub300\ud55c \uac00\uce58 \uc788\ub294 \ud1b5\ucc30\ub825\uc744 \uac16\ucd94\uac8c \ud558\uc600\ub2e4. \uc774\uac83\uc740 \uc6b0\ub9ac\uc758 \ub9ac\ub354\uc2ed, \uc678\uad50\uc640 \uc0ac\uc774\ubc84 \ubcf4\uc548\uc5d0 \uad00\ud55c \uc5f0\uad6c\uc758 \uc124\uacc4\uc640 \uc804\ub2ec\uc5d0 \uc788\uc5b4 \uac00\uce58\ub97c \ub530\uc9c8 \uc218 \uc5c6\uc744 \uc815\ub3c4\ub85c \uadc0\uc911\ud55c \uac83\uc774\ub2e4. \uc704\uc6d0\ud68c\uc640 \ub098\ub294 \uadf8\uac00 \uc6b0\ub9ac \ub274 \uc6e8\uc2a4\ud2b8\ubbfc\uc2a4\ud130 \ub300\ud559\uc758 \uc0c8\ub85c\uc6b4 \ud2b9\ubcc4\uc5f0\uad6c\uc6d0\uc774 \ub41c \uac83\uc744 \ub9e4\uc6b0 \uc601\uad11\uc2a4\ub7fd\uac8c \uc0dd\uac01\ud55c\ub2e4\u201d\uace0 \ubc1d\ud614\ub2e4. \uc804\uc778\ubc94 \uc911\uc7a5\uc740 \ud2b9\ubcc4\uc5f0\uad6c\uc6d0\uc73c\ub85c\uc11c \ud559\ubb38\uc801 \uc218\uc6d4\uc131\uc758 \ucd5c\uace0\uc704 \uae30\uc900\uc5d0 \ub300\ud55c \uc9c0\uc6d0\uc5d0 \uacf5\ud5cc\ud574\uc654\uace0, \ub274 \uc6e8\uc2a4\ud2b8\ubbfc\uc2a4\ud130 \ub300\ud559\uacfc \uc5f0\uad6c\uc6d0\ub4e4\uc758 \uc138\uacc4\uc801\uc778 \uc870\uc9c1\ub9dd \ubc1c\uc804\uc5d0 \ud5cc\uc2e0\ud574\uc654\ub2e4\uace0 \uc54c\ub824\uc84c\ub2e4. \uc774 \ub300\ud559\uc758 \ud2b9\ubcc4\uc5f0\uad6c\uc6d0\uc73c\ub85c\ub294 \ud544\ub9ac\ud540 \ub300\ud1b5\ub839 \ud53c\ub378 \ub77c\ubaa8\uc2a4, \uccb4\ucf54 \ub300\ud1b5\ub839 \ubc14\uce28\ub77c\ud504 \ud074\ub77c\uc6b0\uc2a4, \uce90\ub098\ub2e4 \uad6d\ubc29\ucc38\ubaa8\ucd1d\uc7a5 \ub300\uc7a5 \ub808\uc774 \ud5e4\ub178 \ub4f1\uc774 \uc788\uc73c\uba70, \ud2b9\ubcc4\uc5f0\uad6c\uc6d0\uc73c\ub85c \uc120\uc815\ub41c \uc778\ubb3c 125\uba85 \uac00\uc6b4\ub370 \ud55c\uad6d\uc778\uc740 \uadf8\uac00 \ucd5c\ucd08\uc774\ub2e4.", "paragraph_answer": "2015\ub144 6\uc6d4 \ub274 \uc6e8\uc2a4\ud2b8\ubbfc\uc2a4\ud130 \ub300\ud559(New Westminster College)\uc758 \ud2b9\ubcc4\uc5f0\uad6c\uc6d0 (Distinguished Fellow)\uc73c\ub85c \uc120\uc815\ub418\uc5c8\ub2e4. \ub274 \uc6e8\uc2a4\ud2b8\ubbfc\uc2a4\ud130 \ub300\ud559 \ucd1d\uc7a5 \ub85c\ubc84\ud2b8 \uad7f\uc708 3\uc138\ub294 \u201c\uadf8\ub294 \uc724\ub9ac\uc801 \ub9ac\ub354\uc2ed\uc5d0\uc11c \uad04\ubaa9\ud55c \uc131\uacfc\ub97c \ub0b4\uc5c8\uace0 \uad50\uc721\ud6c8\ub828\uc5d0 \ud3c9\uc0dd \ub3d9\uc548 \ud5cc\uc2e0\ud574\uc654\ub2e4\u201d\uba70 \u201c\ud604 \uc138\uacc4\uc758 \uad6d\uac00\uc801 \uc548\ubcf4 \uc704\ud611\uc5d0 \ub300\ud55c \uc774\ud574\ub294 \uadf8\uac00 \uc815\ubd80\uc758 \uad6d\uc81c\uc801, \uad6d\uac00\uc801, \uc9c0\uc5ed \ub2e8\uc704 \uc774\ud574\uad00\uacc4\uc790\ub4e4\uc758 \ubbfc\uad70 \uc0c1\ud638\uc6b4\uc6a9\uc131\uacfc \ubc29\uc704\ud611\uc57d\uc5d0 \ub300\ud55c \uac00\uce58 \uc788\ub294 \ud1b5\ucc30\ub825\uc744 \uac16\ucd94\uac8c \ud558\uc600\ub2e4. \uc774\uac83\uc740 \uc6b0\ub9ac\uc758 \ub9ac\ub354\uc2ed, \uc678\uad50\uc640 \uc0ac\uc774\ubc84 \ubcf4\uc548\uc5d0 \uad00\ud55c \uc5f0\uad6c\uc758 \uc124\uacc4\uc640 \uc804\ub2ec\uc5d0 \uc788\uc5b4 \uac00\uce58\ub97c \ub530\uc9c8 \uc218 \uc5c6\uc744 \uc815\ub3c4\ub85c \uadc0\uc911\ud55c \uac83\uc774\ub2e4. \uc704\uc6d0\ud68c\uc640 \ub098\ub294 \uadf8\uac00 \uc6b0\ub9ac \ub274 \uc6e8\uc2a4\ud2b8\ubbfc\uc2a4\ud130 \ub300\ud559\uc758 \uc0c8\ub85c\uc6b4 \ud2b9\ubcc4\uc5f0\uad6c\uc6d0\uc774 \ub41c \uac83\uc744 \ub9e4\uc6b0 \uc601\uad11\uc2a4\ub7fd\uac8c \uc0dd\uac01\ud55c\ub2e4\u201d\uace0 \ubc1d\ud614\ub2e4. \uc804\uc778\ubc94 \uc911\uc7a5\uc740 \ud2b9\ubcc4\uc5f0\uad6c\uc6d0\uc73c\ub85c\uc11c \ud559\ubb38\uc801 \uc218\uc6d4\uc131\uc758 \ucd5c\uace0\uc704 \uae30\uc900\uc5d0 \ub300\ud55c \uc9c0\uc6d0\uc5d0 \uacf5\ud5cc\ud574\uc654\uace0, \ub274 \uc6e8\uc2a4\ud2b8\ubbfc\uc2a4\ud130 \ub300\ud559\uacfc \uc5f0\uad6c\uc6d0\ub4e4\uc758 \uc138\uacc4\uc801\uc778 \uc870\uc9c1\ub9dd \ubc1c\uc804\uc5d0 \ud5cc\uc2e0\ud574\uc654\ub2e4\uace0 \uc54c\ub824\uc84c\ub2e4. \uc774 \ub300\ud559\uc758 \ud2b9\ubcc4\uc5f0\uad6c\uc6d0\uc73c\ub85c\ub294 \ud544\ub9ac\ud540 \ub300\ud1b5\ub839 \ud53c\ub378 \ub77c\ubaa8\uc2a4, \uccb4\ucf54 \ub300\ud1b5\ub839 \ubc14\uce28\ub77c\ud504 \ud074\ub77c\uc6b0\uc2a4, \uce90\ub098\ub2e4 \uad6d\ubc29\ucc38\ubaa8\ucd1d\uc7a5 \ub300\uc7a5 \ub808\uc774 \ud5e4\ub178 \ub4f1\uc774 \uc788\uc73c\uba70, \ud2b9\ubcc4\uc5f0\uad6c\uc6d0\uc73c\ub85c \uc120\uc815\ub41c \uc778\ubb3c 125\uba85 \uac00\uc6b4\ub370 \ud55c\uad6d\uc778\uc740 \uadf8\uac00 \ucd5c\ucd08\uc774\ub2e4.", "sentence_answer": "2015\ub144 6\uc6d4 \ub274 \uc6e8\uc2a4\ud2b8\ubbfc\uc2a4\ud130 \ub300\ud559(New Westminster College)\uc758 \ud2b9\ubcc4\uc5f0\uad6c\uc6d0 (Distinguished Fellow)\uc73c\ub85c \uc120\uc815\ub418\uc5c8\ub2e4.", "paragraph_id": "6593261-8-0", "paragraph_question": "question: \uc804\uc778\ubc94\uc774 2015\ub144 \ub274 \uc6e8\uc2a4\ud2b8\ubbfc\uc2a4\ud130 \ub300\ud559\uc5d0\uc11c \uc120\uc815\ub41c \uac83\uc740?, context: 2015\ub144 6\uc6d4 \ub274 \uc6e8\uc2a4\ud2b8\ubbfc\uc2a4\ud130 \ub300\ud559(New Westminster College)\uc758 \ud2b9\ubcc4\uc5f0\uad6c\uc6d0(Distinguished Fellow)\uc73c\ub85c \uc120\uc815\ub418\uc5c8\ub2e4. \ub274 \uc6e8\uc2a4\ud2b8\ubbfc\uc2a4\ud130 \ub300\ud559 \ucd1d\uc7a5 \ub85c\ubc84\ud2b8 \uad7f\uc708 3\uc138\ub294 \u201c\uadf8\ub294 \uc724\ub9ac\uc801 \ub9ac\ub354\uc2ed\uc5d0\uc11c \uad04\ubaa9\ud55c \uc131\uacfc\ub97c \ub0b4\uc5c8\uace0 \uad50\uc721\ud6c8\ub828\uc5d0 \ud3c9\uc0dd \ub3d9\uc548 \ud5cc\uc2e0\ud574\uc654\ub2e4\u201d\uba70 \u201c\ud604 \uc138\uacc4\uc758 \uad6d\uac00\uc801 \uc548\ubcf4 \uc704\ud611\uc5d0 \ub300\ud55c \uc774\ud574\ub294 \uadf8\uac00 \uc815\ubd80\uc758 \uad6d\uc81c\uc801, \uad6d\uac00\uc801, \uc9c0\uc5ed \ub2e8\uc704 \uc774\ud574\uad00\uacc4\uc790\ub4e4\uc758 \ubbfc\uad70 \uc0c1\ud638\uc6b4\uc6a9\uc131\uacfc \ubc29\uc704\ud611\uc57d\uc5d0 \ub300\ud55c \uac00\uce58 \uc788\ub294 \ud1b5\ucc30\ub825\uc744 \uac16\ucd94\uac8c \ud558\uc600\ub2e4. \uc774\uac83\uc740 \uc6b0\ub9ac\uc758 \ub9ac\ub354\uc2ed, \uc678\uad50\uc640 \uc0ac\uc774\ubc84 \ubcf4\uc548\uc5d0 \uad00\ud55c \uc5f0\uad6c\uc758 \uc124\uacc4\uc640 \uc804\ub2ec\uc5d0 \uc788\uc5b4 \uac00\uce58\ub97c \ub530\uc9c8 \uc218 \uc5c6\uc744 \uc815\ub3c4\ub85c \uadc0\uc911\ud55c \uac83\uc774\ub2e4. \uc704\uc6d0\ud68c\uc640 \ub098\ub294 \uadf8\uac00 \uc6b0\ub9ac \ub274 \uc6e8\uc2a4\ud2b8\ubbfc\uc2a4\ud130 \ub300\ud559\uc758 \uc0c8\ub85c\uc6b4 \ud2b9\ubcc4\uc5f0\uad6c\uc6d0\uc774 \ub41c \uac83\uc744 \ub9e4\uc6b0 \uc601\uad11\uc2a4\ub7fd\uac8c \uc0dd\uac01\ud55c\ub2e4\u201d\uace0 \ubc1d\ud614\ub2e4. \uc804\uc778\ubc94 \uc911\uc7a5\uc740 \ud2b9\ubcc4\uc5f0\uad6c\uc6d0\uc73c\ub85c\uc11c \ud559\ubb38\uc801 \uc218\uc6d4\uc131\uc758 \ucd5c\uace0\uc704 \uae30\uc900\uc5d0 \ub300\ud55c \uc9c0\uc6d0\uc5d0 \uacf5\ud5cc\ud574\uc654\uace0, \ub274 \uc6e8\uc2a4\ud2b8\ubbfc\uc2a4\ud130 \ub300\ud559\uacfc \uc5f0\uad6c\uc6d0\ub4e4\uc758 \uc138\uacc4\uc801\uc778 \uc870\uc9c1\ub9dd \ubc1c\uc804\uc5d0 \ud5cc\uc2e0\ud574\uc654\ub2e4\uace0 \uc54c\ub824\uc84c\ub2e4. \uc774 \ub300\ud559\uc758 \ud2b9\ubcc4\uc5f0\uad6c\uc6d0\uc73c\ub85c\ub294 \ud544\ub9ac\ud540 \ub300\ud1b5\ub839 \ud53c\ub378 \ub77c\ubaa8\uc2a4, \uccb4\ucf54 \ub300\ud1b5\ub839 \ubc14\uce28\ub77c\ud504 \ud074\ub77c\uc6b0\uc2a4, \uce90\ub098\ub2e4 \uad6d\ubc29\ucc38\ubaa8\ucd1d\uc7a5 \ub300\uc7a5 \ub808\uc774 \ud5e4\ub178 \ub4f1\uc774 \uc788\uc73c\uba70, \ud2b9\ubcc4\uc5f0\uad6c\uc6d0\uc73c\ub85c \uc120\uc815\ub41c \uc778\ubb3c 125\uba85 \uac00\uc6b4\ub370 \ud55c\uad6d\uc778\uc740 \uadf8\uac00 \ucd5c\ucd08\uc774\ub2e4."} {"question": "\ud55c\uad6d\uad50\uc6d0\ub300\ud559\uad50\ub294 \ud2b9\uc131\uc0c1 \ub300\ud55c\ubbfc\uad6d \ub300\ud559\uc5d0\uc11c \uc5b4\ub5a4 \uc9c0\uc704\ub77c \ud560 \uc218 \uc788\ub294\uac00?", "paragraph": "\uad50\uc721\uc5f0\uad6c\ub294 \ub300\ud559\uc6d0\uacfc \uad50\uc721\uc5f0\uad6c\uc6d0, \uc720\uc544\uad50\uc721\uc6d0 \ub4f1 \ud559\ub0b4\uc5d0 \uc124\uce58\ub41c \uc5f0\uad6c \uae30\uad00\ub4e4\uc744 \uc911\uc2ec\uc73c\ub85c \uc815\ubd80\u00b7\ub300\ud559\u00b7\ud559\uad50 \ud604\uc7a5 \uac04 \ud611\ub825\uc744 \ud1b5\ud55c \uc2e4\ucc9c \uc911\uc2ec\uc758 \uc5f0\uad6c \ud65c\ub3d9\uc744 \uc9c0\ud5a5\ud558\uace0 \uc788\ub2e4. \ud2b9\ud788 \uad50\uc721\uc5f0\uad6c \ud65c\ub3d9\uc744 \uc804\ubb38\uc801\uc73c\ub85c \uc218\ud589\ud558\ub294 \uad50\uc721\uc5f0\uad6c\uc6d0\uc740 \uad50\uc721\uc758 \uae30\ucd08 \uc774\ub860\uacfc \uad50\uacfc\uad50\uc721, \uad50\uc6d0\uad50\uc721, \uad50\uc721\uacfc\uc815, \ud1b5\uc77c\uad50\uc721 \ub4f1\uc5d0 \uad00\ud55c \uc5f0\uad6c\ub97c \uc218\ud589\ud574 \ud559\uad50\uad50\uc721\uc758 \uc9c8\uc801 \ud5a5\uc0c1\uc5d0 \uc774\ubc14\uc9c0\ud568\uc744 \ubaa9\ud45c\ub85c \ud55c\ub2e4. \ucd5c\uadfc\uc5d0\ub294 \ud55c\uad6d\uc5f0\uad6c\uc7ac\ub2e8\uc73c\ub85c\ubd80\ud130 \u2018\uc2a4\ub9c8\ud2b8\uad50\uc721\uc815\ucc45 \uc911\uc810\uc5f0\uad6c\uc18c\u2019\uc640 '\uc720\uc544\uad50\uc721 \ub300\ud559\uc911\uc810\uc5f0\uad6c\uc18c'\ub85c \uc120\uc815\ub418\ub294 \ud55c\ud3b8, \ud55c\uad6d\uacfc\ud559\ucc3d\uc758\uc7ac\ub2e8\uc73c\ub85c\ubd80\ud130 '\ucc3d\uc758\u00b7\uc778\uc131\uad50\uc721 \uac70\uc810\uc13c\ud130'\ub97c \uc720\uce58\ud558\ub294 \ub4f1 \uad6d\uac00 \uc7ac\uc815\uc774 \ud22c\uc785\ub418\ub294 \uc5f0\uad6c \uc0ac\uc5c5\uc744 \ub2e4\uc218 \uc218\ud589\ud558\uace0 \uc788\ub2e4. \uc9c0\ub9ac\uc801\uc73c\ub85c \ub300\ud55c\ubbfc\uad6d\uc758 \uc911\uc2ec\ubd80\uc5d0 \uc788\ub294 \ud2b9\uc131\uacfc \uc885\ud569\uad50\uc6d0\uc591\uc131\ub300\ud559\uc73c\ub85c\uc11c\uc758 \uc9c0\uc704\ub97c \ubc18\uc601\ud558\uc5ec \uc804\uad6d \ub2e8\uc704\uc758 \uad50\uc721 \uc815\ucc45 \uad00\ub828 \uacf5\uccad\ud68c\uc640 \ud559\uc220 \ud589\uc0ac\ub4e4\uc774 \ub9ce\uc774 \uac1c\ucd5c\ub41c\ub2e4.", "answer": "\uc885\ud569\uad50\uc6d0\uc591\uc131\ub300\ud559", "sentence": "\uc9c0\ub9ac\uc801\uc73c\ub85c \ub300\ud55c\ubbfc\uad6d\uc758 \uc911\uc2ec\ubd80\uc5d0 \uc788\ub294 \ud2b9\uc131\uacfc \uc885\ud569\uad50\uc6d0\uc591\uc131\ub300\ud559 \uc73c\ub85c\uc11c\uc758 \uc9c0\uc704\ub97c \ubc18\uc601\ud558\uc5ec \uc804\uad6d \ub2e8\uc704\uc758 \uad50\uc721 \uc815\ucc45 \uad00\ub828 \uacf5\uccad\ud68c\uc640 \ud559\uc220 \ud589\uc0ac\ub4e4\uc774 \ub9ce\uc774 \uac1c\ucd5c\ub41c\ub2e4.", "paragraph_sentence": "\uad50\uc721\uc5f0\uad6c\ub294 \ub300\ud559\uc6d0\uacfc \uad50\uc721\uc5f0\uad6c\uc6d0, \uc720\uc544\uad50\uc721\uc6d0 \ub4f1 \ud559\ub0b4\uc5d0 \uc124\uce58\ub41c \uc5f0\uad6c \uae30\uad00\ub4e4\uc744 \uc911\uc2ec\uc73c\ub85c \uc815\ubd80\u00b7\ub300\ud559\u00b7\ud559\uad50 \ud604\uc7a5 \uac04 \ud611\ub825\uc744 \ud1b5\ud55c \uc2e4\ucc9c \uc911\uc2ec\uc758 \uc5f0\uad6c \ud65c\ub3d9\uc744 \uc9c0\ud5a5\ud558\uace0 \uc788\ub2e4. \ud2b9\ud788 \uad50\uc721\uc5f0\uad6c \ud65c\ub3d9\uc744 \uc804\ubb38\uc801\uc73c\ub85c \uc218\ud589\ud558\ub294 \uad50\uc721\uc5f0\uad6c\uc6d0\uc740 \uad50\uc721\uc758 \uae30\ucd08 \uc774\ub860\uacfc \uad50\uacfc\uad50\uc721, \uad50\uc6d0\uad50\uc721, \uad50\uc721\uacfc\uc815, \ud1b5\uc77c\uad50\uc721 \ub4f1\uc5d0 \uad00\ud55c \uc5f0\uad6c\ub97c \uc218\ud589\ud574 \ud559\uad50\uad50\uc721\uc758 \uc9c8\uc801 \ud5a5\uc0c1\uc5d0 \uc774\ubc14\uc9c0\ud568\uc744 \ubaa9\ud45c\ub85c \ud55c\ub2e4. \ucd5c\uadfc\uc5d0\ub294 \ud55c\uad6d\uc5f0\uad6c\uc7ac\ub2e8\uc73c\ub85c\ubd80\ud130 \u2018\uc2a4\ub9c8\ud2b8\uad50\uc721\uc815\ucc45 \uc911\uc810\uc5f0\uad6c\uc18c\u2019\uc640 '\uc720\uc544\uad50\uc721 \ub300\ud559\uc911\uc810\uc5f0\uad6c\uc18c'\ub85c \uc120\uc815\ub418\ub294 \ud55c\ud3b8, \ud55c\uad6d\uacfc\ud559\ucc3d\uc758\uc7ac\ub2e8\uc73c\ub85c\ubd80\ud130 '\ucc3d\uc758\u00b7\uc778\uc131\uad50\uc721 \uac70\uc810\uc13c\ud130'\ub97c \uc720\uce58\ud558\ub294 \ub4f1 \uad6d\uac00 \uc7ac\uc815\uc774 \ud22c\uc785\ub418\ub294 \uc5f0\uad6c \uc0ac\uc5c5\uc744 \ub2e4\uc218 \uc218\ud589\ud558\uace0 \uc788\ub2e4. \uc9c0\ub9ac\uc801\uc73c\ub85c \ub300\ud55c\ubbfc\uad6d\uc758 \uc911\uc2ec\ubd80\uc5d0 \uc788\ub294 \ud2b9\uc131\uacfc \uc885\ud569\uad50\uc6d0\uc591\uc131\ub300\ud559 \uc73c\ub85c\uc11c\uc758 \uc9c0\uc704\ub97c \ubc18\uc601\ud558\uc5ec \uc804\uad6d \ub2e8\uc704\uc758 \uad50\uc721 \uc815\ucc45 \uad00\ub828 \uacf5\uccad\ud68c\uc640 \ud559\uc220 \ud589\uc0ac\ub4e4\uc774 \ub9ce\uc774 \uac1c\ucd5c\ub41c\ub2e4. ", "paragraph_answer": "\uad50\uc721\uc5f0\uad6c\ub294 \ub300\ud559\uc6d0\uacfc \uad50\uc721\uc5f0\uad6c\uc6d0, \uc720\uc544\uad50\uc721\uc6d0 \ub4f1 \ud559\ub0b4\uc5d0 \uc124\uce58\ub41c \uc5f0\uad6c \uae30\uad00\ub4e4\uc744 \uc911\uc2ec\uc73c\ub85c \uc815\ubd80\u00b7\ub300\ud559\u00b7\ud559\uad50 \ud604\uc7a5 \uac04 \ud611\ub825\uc744 \ud1b5\ud55c \uc2e4\ucc9c \uc911\uc2ec\uc758 \uc5f0\uad6c \ud65c\ub3d9\uc744 \uc9c0\ud5a5\ud558\uace0 \uc788\ub2e4. \ud2b9\ud788 \uad50\uc721\uc5f0\uad6c \ud65c\ub3d9\uc744 \uc804\ubb38\uc801\uc73c\ub85c \uc218\ud589\ud558\ub294 \uad50\uc721\uc5f0\uad6c\uc6d0\uc740 \uad50\uc721\uc758 \uae30\ucd08 \uc774\ub860\uacfc \uad50\uacfc\uad50\uc721, \uad50\uc6d0\uad50\uc721, \uad50\uc721\uacfc\uc815, \ud1b5\uc77c\uad50\uc721 \ub4f1\uc5d0 \uad00\ud55c \uc5f0\uad6c\ub97c \uc218\ud589\ud574 \ud559\uad50\uad50\uc721\uc758 \uc9c8\uc801 \ud5a5\uc0c1\uc5d0 \uc774\ubc14\uc9c0\ud568\uc744 \ubaa9\ud45c\ub85c \ud55c\ub2e4. \ucd5c\uadfc\uc5d0\ub294 \ud55c\uad6d\uc5f0\uad6c\uc7ac\ub2e8\uc73c\ub85c\ubd80\ud130 \u2018\uc2a4\ub9c8\ud2b8\uad50\uc721\uc815\ucc45 \uc911\uc810\uc5f0\uad6c\uc18c\u2019\uc640 '\uc720\uc544\uad50\uc721 \ub300\ud559\uc911\uc810\uc5f0\uad6c\uc18c'\ub85c \uc120\uc815\ub418\ub294 \ud55c\ud3b8, \ud55c\uad6d\uacfc\ud559\ucc3d\uc758\uc7ac\ub2e8\uc73c\ub85c\ubd80\ud130 '\ucc3d\uc758\u00b7\uc778\uc131\uad50\uc721 \uac70\uc810\uc13c\ud130'\ub97c \uc720\uce58\ud558\ub294 \ub4f1 \uad6d\uac00 \uc7ac\uc815\uc774 \ud22c\uc785\ub418\ub294 \uc5f0\uad6c \uc0ac\uc5c5\uc744 \ub2e4\uc218 \uc218\ud589\ud558\uace0 \uc788\ub2e4. \uc9c0\ub9ac\uc801\uc73c\ub85c \ub300\ud55c\ubbfc\uad6d\uc758 \uc911\uc2ec\ubd80\uc5d0 \uc788\ub294 \ud2b9\uc131\uacfc \uc885\ud569\uad50\uc6d0\uc591\uc131\ub300\ud559 \uc73c\ub85c\uc11c\uc758 \uc9c0\uc704\ub97c \ubc18\uc601\ud558\uc5ec \uc804\uad6d \ub2e8\uc704\uc758 \uad50\uc721 \uc815\ucc45 \uad00\ub828 \uacf5\uccad\ud68c\uc640 \ud559\uc220 \ud589\uc0ac\ub4e4\uc774 \ub9ce\uc774 \uac1c\ucd5c\ub41c\ub2e4.", "sentence_answer": "\uc9c0\ub9ac\uc801\uc73c\ub85c \ub300\ud55c\ubbfc\uad6d\uc758 \uc911\uc2ec\ubd80\uc5d0 \uc788\ub294 \ud2b9\uc131\uacfc \uc885\ud569\uad50\uc6d0\uc591\uc131\ub300\ud559 \uc73c\ub85c\uc11c\uc758 \uc9c0\uc704\ub97c \ubc18\uc601\ud558\uc5ec \uc804\uad6d \ub2e8\uc704\uc758 \uad50\uc721 \uc815\ucc45 \uad00\ub828 \uacf5\uccad\ud68c\uc640 \ud559\uc220 \ud589\uc0ac\ub4e4\uc774 \ub9ce\uc774 \uac1c\ucd5c\ub41c\ub2e4.", "paragraph_id": "6569877-8-1", "paragraph_question": "question: \ud55c\uad6d\uad50\uc6d0\ub300\ud559\uad50\ub294 \ud2b9\uc131\uc0c1 \ub300\ud55c\ubbfc\uad6d \ub300\ud559\uc5d0\uc11c \uc5b4\ub5a4 \uc9c0\uc704\ub77c \ud560 \uc218 \uc788\ub294\uac00?, context: \uad50\uc721\uc5f0\uad6c\ub294 \ub300\ud559\uc6d0\uacfc \uad50\uc721\uc5f0\uad6c\uc6d0, \uc720\uc544\uad50\uc721\uc6d0 \ub4f1 \ud559\ub0b4\uc5d0 \uc124\uce58\ub41c \uc5f0\uad6c \uae30\uad00\ub4e4\uc744 \uc911\uc2ec\uc73c\ub85c \uc815\ubd80\u00b7\ub300\ud559\u00b7\ud559\uad50 \ud604\uc7a5 \uac04 \ud611\ub825\uc744 \ud1b5\ud55c \uc2e4\ucc9c \uc911\uc2ec\uc758 \uc5f0\uad6c \ud65c\ub3d9\uc744 \uc9c0\ud5a5\ud558\uace0 \uc788\ub2e4. \ud2b9\ud788 \uad50\uc721\uc5f0\uad6c \ud65c\ub3d9\uc744 \uc804\ubb38\uc801\uc73c\ub85c \uc218\ud589\ud558\ub294 \uad50\uc721\uc5f0\uad6c\uc6d0\uc740 \uad50\uc721\uc758 \uae30\ucd08 \uc774\ub860\uacfc \uad50\uacfc\uad50\uc721, \uad50\uc6d0\uad50\uc721, \uad50\uc721\uacfc\uc815, \ud1b5\uc77c\uad50\uc721 \ub4f1\uc5d0 \uad00\ud55c \uc5f0\uad6c\ub97c \uc218\ud589\ud574 \ud559\uad50\uad50\uc721\uc758 \uc9c8\uc801 \ud5a5\uc0c1\uc5d0 \uc774\ubc14\uc9c0\ud568\uc744 \ubaa9\ud45c\ub85c \ud55c\ub2e4. \ucd5c\uadfc\uc5d0\ub294 \ud55c\uad6d\uc5f0\uad6c\uc7ac\ub2e8\uc73c\ub85c\ubd80\ud130 \u2018\uc2a4\ub9c8\ud2b8\uad50\uc721\uc815\ucc45 \uc911\uc810\uc5f0\uad6c\uc18c\u2019\uc640 '\uc720\uc544\uad50\uc721 \ub300\ud559\uc911\uc810\uc5f0\uad6c\uc18c'\ub85c \uc120\uc815\ub418\ub294 \ud55c\ud3b8, \ud55c\uad6d\uacfc\ud559\ucc3d\uc758\uc7ac\ub2e8\uc73c\ub85c\ubd80\ud130 '\ucc3d\uc758\u00b7\uc778\uc131\uad50\uc721 \uac70\uc810\uc13c\ud130'\ub97c \uc720\uce58\ud558\ub294 \ub4f1 \uad6d\uac00 \uc7ac\uc815\uc774 \ud22c\uc785\ub418\ub294 \uc5f0\uad6c \uc0ac\uc5c5\uc744 \ub2e4\uc218 \uc218\ud589\ud558\uace0 \uc788\ub2e4. \uc9c0\ub9ac\uc801\uc73c\ub85c \ub300\ud55c\ubbfc\uad6d\uc758 \uc911\uc2ec\ubd80\uc5d0 \uc788\ub294 \ud2b9\uc131\uacfc \uc885\ud569\uad50\uc6d0\uc591\uc131\ub300\ud559\uc73c\ub85c\uc11c\uc758 \uc9c0\uc704\ub97c \ubc18\uc601\ud558\uc5ec \uc804\uad6d \ub2e8\uc704\uc758 \uad50\uc721 \uc815\ucc45 \uad00\ub828 \uacf5\uccad\ud68c\uc640 \ud559\uc220 \ud589\uc0ac\ub4e4\uc774 \ub9ce\uc774 \uac1c\ucd5c\ub41c\ub2e4."} {"question": "\uc694\ud22c\ub098\ub974\uc758 \uc870\uc0c1\uc740 \ubb34\uc5c7\uc73c\ub85c \ub9cc\ub4e4\uc5b4\uc84c\ub294\uac00?", "paragraph": "\ub9d0\uc2f8\uc6c0\uc740 \uacc4\uc18d\ub418\uace0, \uc704\ubbf8\ub974\uc5d0 \uad00\ud55c \uc774\uc57c\uae30\uac00 \uba87 \uac1c \ub354 \ub098\uc628\ub2e4. \uc624\ub518\uc740 \"\uc704\ubbf8\ub974\uc758 \uc871\uc18d\"\ub4e4 \uc911 \uac00\uc7a5 \ub098\uc774\uac00 \ub9ce\uc740 \uace0\ub300\uc758 \uc694\ud230\uc740 \ub204\uad6c\ub0d0\uace0 \ubb3b\ub294\ub2e4. \ubc14\ud504\uc2a4\ub8e8\ub4dc\ub2c8\ub974\ub294 \uc61c\ub0a0 \uc61b\uc801\uc5d0 \uc0b4\uc558\ub358 \ubca0\ub974\uac94\ubbf8\ub974\uac00 \ubc14\ub85c \uadf8 \uc694\ud230\uc774\uba70 \uadf8\ub294 \uc2a4\ub8e8\ub4dc\uac94\ubbf8\ub974\uc758 \uc544\ub4e4\uc774\uace0 \uc544\uc6b0\uac94\ubbf8\ub974\uc758 \uc190\uc790\ub77c\uace0 \ub300\ub2f5\ud55c\ub2e4. \uadf8 \ub2e4\uc74c \uc808\uc5d0\uc11c \uc624\ub518\uc740 \uc544\uc6b0\uac94\ubbf8\ub974\ub294 \uc5b4\ub514\uc11c \uc654\ub0d0\uace0 \ubb3b\ub294\ub2e4. \ubc14\ud504\uc2a4\ub8e8\ub4dc\ub2c8\ub974\ub294 \uc5d8\ub9ac\ubc14\uac00\ub974\uc5d0\uc11c \ub3c5\uc561(\uc5d0\uc774\ud2b8)\uc774 \ubc29\uc6b8\uc838 \ub5a8\uc5b4\uc84c\uace0, \uadf8 \ub3c5\uc561\ubc29\uc6b8\uc774 \uacc4\uc18d \ucee4\uc838\uc11c \ud55c \uba85\uc758 \uc694\ud230\uc774 \ub418\uc5c8\uc73c\ub2c8 \uadf8\uac00 \ubaa8\ub4e0 \uc694\ud22c\ub098\ub974\uc758 \uc870\uc0c1\uc774\ub77c\uace0 \ub9d0\ud55c\ub2e4. \ub9c8\uc9c0\ub9c9\uc73c\ub85c \uc624\ub518\uc740 \uadf8 \uc874\uc7ac\uac00 \uc5ec\uc131 \uc694\ud230\uacfc \uc9dd\uc744 \uc9d3\uc9c0 \uc54a\uace0 \uc5b4\ub5bb\uac8c \uc544\uc774\ub97c \uac00\uc84c\ub0d0\uace0 \ubb3b\ub294\ub2e4. \ubc14\ud504\uc2a4\ub8e8\ub4dc\ub2c8\ub974\ub294 \uace0\ub300\uc758 \uc694\ud230\uc758 \uaca8\ub4dc\ub791\uc774\uc5d0\uc11c \uc5ec\uc790\uc640 \ub0a8\uc790\uac00 \ub098\uc654\uc73c\uba70, \uadf8\uc758 \ubc1c\uc5d0\uc11c\ub294 \uba38\ub9ac\uac00 \uc5ec\uc12f\uac1c\uc778 \uc694\ud230\uc774 \uc0dd\uaca8\ub0ac\ub2e4\uace0 \ub9d0\ud55c\ub2e4.", "answer": "\ub3c5\uc561\ubc29\uc6b8", "sentence": "\uadf8 \ub3c5\uc561\ubc29\uc6b8 \uc774 \uacc4\uc18d \ucee4\uc838\uc11c \ud55c \uba85\uc758 \uc694\ud230\uc774 \ub418\uc5c8\uc73c\ub2c8 \uadf8\uac00 \ubaa8\ub4e0 \uc694\ud22c\ub098\ub974\uc758 \uc870\uc0c1\uc774\ub77c\uace0 \ub9d0\ud55c\ub2e4.", "paragraph_sentence": "\ub9d0\uc2f8\uc6c0\uc740 \uacc4\uc18d\ub418\uace0, \uc704\ubbf8\ub974\uc5d0 \uad00\ud55c \uc774\uc57c\uae30\uac00 \uba87 \uac1c \ub354 \ub098\uc628\ub2e4. \uc624\ub518\uc740 \"\uc704\ubbf8\ub974\uc758 \uc871\uc18d\"\ub4e4 \uc911 \uac00\uc7a5 \ub098\uc774\uac00 \ub9ce\uc740 \uace0\ub300\uc758 \uc694\ud230\uc740 \ub204\uad6c\ub0d0\uace0 \ubb3b\ub294\ub2e4. \ubc14\ud504\uc2a4\ub8e8\ub4dc\ub2c8\ub974\ub294 \uc61c\ub0a0 \uc61b\uc801\uc5d0 \uc0b4\uc558\ub358 \ubca0\ub974\uac94\ubbf8\ub974\uac00 \ubc14\ub85c \uadf8 \uc694\ud230\uc774\uba70 \uadf8\ub294 \uc2a4\ub8e8\ub4dc\uac94\ubbf8\ub974\uc758 \uc544\ub4e4\uc774\uace0 \uc544\uc6b0\uac94\ubbf8\ub974\uc758 \uc190\uc790\ub77c\uace0 \ub300\ub2f5\ud55c\ub2e4. \uadf8 \ub2e4\uc74c \uc808\uc5d0\uc11c \uc624\ub518\uc740 \uc544\uc6b0\uac94\ubbf8\ub974\ub294 \uc5b4\ub514\uc11c \uc654\ub0d0\uace0 \ubb3b\ub294\ub2e4. \ubc14\ud504\uc2a4\ub8e8\ub4dc\ub2c8\ub974\ub294 \uc5d8\ub9ac\ubc14\uac00\ub974\uc5d0\uc11c \ub3c5\uc561(\uc5d0\uc774\ud2b8)\uc774 \ubc29\uc6b8\uc838 \ub5a8\uc5b4\uc84c\uace0, \uadf8 \ub3c5\uc561\ubc29\uc6b8 \uc774 \uacc4\uc18d \ucee4\uc838\uc11c \ud55c \uba85\uc758 \uc694\ud230\uc774 \ub418\uc5c8\uc73c\ub2c8 \uadf8\uac00 \ubaa8\ub4e0 \uc694\ud22c\ub098\ub974\uc758 \uc870\uc0c1\uc774\ub77c\uace0 \ub9d0\ud55c\ub2e4. \ub9c8\uc9c0\ub9c9\uc73c\ub85c \uc624\ub518\uc740 \uadf8 \uc874\uc7ac\uac00 \uc5ec\uc131 \uc694\ud230\uacfc \uc9dd\uc744 \uc9d3\uc9c0 \uc54a\uace0 \uc5b4\ub5bb\uac8c \uc544\uc774\ub97c \uac00\uc84c\ub0d0\uace0 \ubb3b\ub294\ub2e4. \ubc14\ud504\uc2a4\ub8e8\ub4dc\ub2c8\ub974\ub294 \uace0\ub300\uc758 \uc694\ud230\uc758 \uaca8\ub4dc\ub791\uc774\uc5d0\uc11c \uc5ec\uc790\uc640 \ub0a8\uc790\uac00 \ub098\uc654\uc73c\uba70, \uadf8\uc758 \ubc1c\uc5d0\uc11c\ub294 \uba38\ub9ac\uac00 \uc5ec\uc12f\uac1c\uc778 \uc694\ud230\uc774 \uc0dd\uaca8\ub0ac\ub2e4\uace0 \ub9d0\ud55c\ub2e4.", "paragraph_answer": "\ub9d0\uc2f8\uc6c0\uc740 \uacc4\uc18d\ub418\uace0, \uc704\ubbf8\ub974\uc5d0 \uad00\ud55c \uc774\uc57c\uae30\uac00 \uba87 \uac1c \ub354 \ub098\uc628\ub2e4. \uc624\ub518\uc740 \"\uc704\ubbf8\ub974\uc758 \uc871\uc18d\"\ub4e4 \uc911 \uac00\uc7a5 \ub098\uc774\uac00 \ub9ce\uc740 \uace0\ub300\uc758 \uc694\ud230\uc740 \ub204\uad6c\ub0d0\uace0 \ubb3b\ub294\ub2e4. \ubc14\ud504\uc2a4\ub8e8\ub4dc\ub2c8\ub974\ub294 \uc61c\ub0a0 \uc61b\uc801\uc5d0 \uc0b4\uc558\ub358 \ubca0\ub974\uac94\ubbf8\ub974\uac00 \ubc14\ub85c \uadf8 \uc694\ud230\uc774\uba70 \uadf8\ub294 \uc2a4\ub8e8\ub4dc\uac94\ubbf8\ub974\uc758 \uc544\ub4e4\uc774\uace0 \uc544\uc6b0\uac94\ubbf8\ub974\uc758 \uc190\uc790\ub77c\uace0 \ub300\ub2f5\ud55c\ub2e4. \uadf8 \ub2e4\uc74c \uc808\uc5d0\uc11c \uc624\ub518\uc740 \uc544\uc6b0\uac94\ubbf8\ub974\ub294 \uc5b4\ub514\uc11c \uc654\ub0d0\uace0 \ubb3b\ub294\ub2e4. \ubc14\ud504\uc2a4\ub8e8\ub4dc\ub2c8\ub974\ub294 \uc5d8\ub9ac\ubc14\uac00\ub974\uc5d0\uc11c \ub3c5\uc561(\uc5d0\uc774\ud2b8)\uc774 \ubc29\uc6b8\uc838 \ub5a8\uc5b4\uc84c\uace0, \uadf8 \ub3c5\uc561\ubc29\uc6b8 \uc774 \uacc4\uc18d \ucee4\uc838\uc11c \ud55c \uba85\uc758 \uc694\ud230\uc774 \ub418\uc5c8\uc73c\ub2c8 \uadf8\uac00 \ubaa8\ub4e0 \uc694\ud22c\ub098\ub974\uc758 \uc870\uc0c1\uc774\ub77c\uace0 \ub9d0\ud55c\ub2e4. \ub9c8\uc9c0\ub9c9\uc73c\ub85c \uc624\ub518\uc740 \uadf8 \uc874\uc7ac\uac00 \uc5ec\uc131 \uc694\ud230\uacfc \uc9dd\uc744 \uc9d3\uc9c0 \uc54a\uace0 \uc5b4\ub5bb\uac8c \uc544\uc774\ub97c \uac00\uc84c\ub0d0\uace0 \ubb3b\ub294\ub2e4. \ubc14\ud504\uc2a4\ub8e8\ub4dc\ub2c8\ub974\ub294 \uace0\ub300\uc758 \uc694\ud230\uc758 \uaca8\ub4dc\ub791\uc774\uc5d0\uc11c \uc5ec\uc790\uc640 \ub0a8\uc790\uac00 \ub098\uc654\uc73c\uba70, \uadf8\uc758 \ubc1c\uc5d0\uc11c\ub294 \uba38\ub9ac\uac00 \uc5ec\uc12f\uac1c\uc778 \uc694\ud230\uc774 \uc0dd\uaca8\ub0ac\ub2e4\uace0 \ub9d0\ud55c\ub2e4.", "sentence_answer": "\uadf8 \ub3c5\uc561\ubc29\uc6b8 \uc774 \uacc4\uc18d \ucee4\uc838\uc11c \ud55c \uba85\uc758 \uc694\ud230\uc774 \ub418\uc5c8\uc73c\ub2c8 \uadf8\uac00 \ubaa8\ub4e0 \uc694\ud22c\ub098\ub974\uc758 \uc870\uc0c1\uc774\ub77c\uace0 \ub9d0\ud55c\ub2e4.", "paragraph_id": "6542646-1-1", "paragraph_question": "question: \uc694\ud22c\ub098\ub974\uc758 \uc870\uc0c1\uc740 \ubb34\uc5c7\uc73c\ub85c \ub9cc\ub4e4\uc5b4\uc84c\ub294\uac00?, context: \ub9d0\uc2f8\uc6c0\uc740 \uacc4\uc18d\ub418\uace0, \uc704\ubbf8\ub974\uc5d0 \uad00\ud55c \uc774\uc57c\uae30\uac00 \uba87 \uac1c \ub354 \ub098\uc628\ub2e4. \uc624\ub518\uc740 \"\uc704\ubbf8\ub974\uc758 \uc871\uc18d\"\ub4e4 \uc911 \uac00\uc7a5 \ub098\uc774\uac00 \ub9ce\uc740 \uace0\ub300\uc758 \uc694\ud230\uc740 \ub204\uad6c\ub0d0\uace0 \ubb3b\ub294\ub2e4. \ubc14\ud504\uc2a4\ub8e8\ub4dc\ub2c8\ub974\ub294 \uc61c\ub0a0 \uc61b\uc801\uc5d0 \uc0b4\uc558\ub358 \ubca0\ub974\uac94\ubbf8\ub974\uac00 \ubc14\ub85c \uadf8 \uc694\ud230\uc774\uba70 \uadf8\ub294 \uc2a4\ub8e8\ub4dc\uac94\ubbf8\ub974\uc758 \uc544\ub4e4\uc774\uace0 \uc544\uc6b0\uac94\ubbf8\ub974\uc758 \uc190\uc790\ub77c\uace0 \ub300\ub2f5\ud55c\ub2e4. \uadf8 \ub2e4\uc74c \uc808\uc5d0\uc11c \uc624\ub518\uc740 \uc544\uc6b0\uac94\ubbf8\ub974\ub294 \uc5b4\ub514\uc11c \uc654\ub0d0\uace0 \ubb3b\ub294\ub2e4. \ubc14\ud504\uc2a4\ub8e8\ub4dc\ub2c8\ub974\ub294 \uc5d8\ub9ac\ubc14\uac00\ub974\uc5d0\uc11c \ub3c5\uc561(\uc5d0\uc774\ud2b8)\uc774 \ubc29\uc6b8\uc838 \ub5a8\uc5b4\uc84c\uace0, \uadf8 \ub3c5\uc561\ubc29\uc6b8\uc774 \uacc4\uc18d \ucee4\uc838\uc11c \ud55c \uba85\uc758 \uc694\ud230\uc774 \ub418\uc5c8\uc73c\ub2c8 \uadf8\uac00 \ubaa8\ub4e0 \uc694\ud22c\ub098\ub974\uc758 \uc870\uc0c1\uc774\ub77c\uace0 \ub9d0\ud55c\ub2e4. \ub9c8\uc9c0\ub9c9\uc73c\ub85c \uc624\ub518\uc740 \uadf8 \uc874\uc7ac\uac00 \uc5ec\uc131 \uc694\ud230\uacfc \uc9dd\uc744 \uc9d3\uc9c0 \uc54a\uace0 \uc5b4\ub5bb\uac8c \uc544\uc774\ub97c \uac00\uc84c\ub0d0\uace0 \ubb3b\ub294\ub2e4. \ubc14\ud504\uc2a4\ub8e8\ub4dc\ub2c8\ub974\ub294 \uace0\ub300\uc758 \uc694\ud230\uc758 \uaca8\ub4dc\ub791\uc774\uc5d0\uc11c \uc5ec\uc790\uc640 \ub0a8\uc790\uac00 \ub098\uc654\uc73c\uba70, \uadf8\uc758 \ubc1c\uc5d0\uc11c\ub294 \uba38\ub9ac\uac00 \uc5ec\uc12f\uac1c\uc778 \uc694\ud230\uc774 \uc0dd\uaca8\ub0ac\ub2e4\uace0 \ub9d0\ud55c\ub2e4."} {"question": "\ub2e5\ud130 \ub85c\ubcf4\ud2b8\ub2c9\uc774\ub77c\uace0\ub3c4 \ubd88\ub9ac\ub294 \uc18c\ub2c9\uc758 \uc601\uc6d0\ud55c \uc801\uc740?", "paragraph": "\uc18c\ub2c9\uc758 \uc601\uc6d0\ud55c \uc801\uc740 \ub2e5\ud130 \uc5d0\uadf8\ub9e8\uc774\uba70, \ub2e5\ud130 \ub85c\ubcf4\ud2b8\ub2c9 \uc774\ub77c\uace0\ub3c4 \uc54c\ub824\uc838 \uc788\ub2e4. IQ 300\uc758 \uc9c0\ub2a5\uc744 \uac00\uc9c4 \uc5d0\uadf8\ub9e8\uc740 \ucc9c\uc7ac\uc801\uc778 \uc545\uc758 \uacfc\ud559\uc790\ub85c \uadf8\uc758 \uc5c4\uccad\ub09c \uae30\uacc4\ub97c \uc774\uc6a9\ud574\uc11c \uc138\uacc4\ub97c \uc815\ubcf5\ud558\ub824\uace0 \ud558\uc9c0\ub9cc, \uadf8\ub7f4 \ub54c\ub9c8\ub2e4 \uc18c\ub2c9\uc758 \ubc29\ud574\ub85c \ubc88\ubc88\uc774 \uc2e4\ud328\ud55c\ub2e4. \uc18c\ub2c9\uc740 \ucc98\uc74c\uc5d0\ub294 \ub2e5\ud130 \uc5d0\uadf8\ub9e8\uc774 \uadf8\uc758 \ub3d9\ubb3c \uce5c\uad6c\ub4e4\uc744 \ubc30\ub4dc\ub2c9\uc774\ub77c\uace0 \ubd88\ub9ac\ub294 \ub85c\ubd07 \ub178\uc608\ub85c \ubc14\uafb8\ub294 \uac83 \ub54c\ubb38\uc5d0 \uc2f8\uc6b0\uac8c \ub418\uc5c8\ub2e4. \uc18c\ub2c9\uc740 \uce5c\uad6c\ub4e4\uc744 \uad6c\ucd9c\ud588\uace0, \uba87\ubc88\uc774\ub098 \uce74\uc624\uc2a4 \uc5d0\uba54\ub784\ub4dc, \ud0c0\uc784 \uc2a4\ud1a4, \uadf8\ub9ac\uace0 \ub9c8\uc2a4\ud130 \uc5d0\uba54\ub784\ub4dc\ub97c \uc774\uc6a9\ud574\uc11c \uc138\uacc4\ub97c \uc815\ubcf5\ud558\ub824\ub294 \uc2dc\ub3c4\ub97c \ub9c9\uc558\ub2e4. \uc885\uc885 \uc774\uac83\ub4e4\uc740 \uc5d0\uadf8\ub9e8\uc758 \uac15\ub825\ud55c \ubaa8\uc2b5\uc744 \ubcf8\ub534 \uba54\uce74 \uc288\ud2b8\ub97c \uc791\ub3d9\uc2dc\ud0a4\ub294 \ub370 \uc0ac\uc6a9\ub418\uc5c8\ub2e4. \ud558\uc9c0\ub9cc, \ucd5c\uadfc \uac8c\uc784\uc5d0\uc11c\ub294 \ub2e5\ud130 \uc5d0\uadf8\ub9e8\uc740 \uc804\ud568\uc774\ub098 \ud575 \ubbf8\uc0ac\uc77c\uc744 \uc774\uc6a9\ud574\uc11c \uc138\uacc4\ub97c \uc815\ubcf5\ud558\ub824\ub294 \ubaa8\uc2b5\uc744 \ubcf4\uc5ec\uc8fc\uace0 \uc788\ub2e4. \uc544\uc639\ub2e4\uc639 \ub2e4\ud22c\ub294 \uc0ac\uc774\uc774\uc9c0\ub9cc \ub2e5\ud130 \uc5d0\uadf8\ub9e8\ub3c4 \uc18c\ub2c9\uc744 \ub192\uc774 \ud3c9\uac00\ud558\uace0 \uc788\uc73c\uba70, \uac1c\uc778\uc801\uc73c\ub85c\ub3c4 \uc798 \uc544\ub294 \uc0ac\uc774\uc778 \uc560\uc99d\uc758 \uad00\uacc4\uc774\uae30\ub3c4 \ud558\ub2e4.", "answer": "\ub2e5\ud130 \uc5d0\uadf8\ub9e8", "sentence": "\uc18c\ub2c9\uc758 \uc601\uc6d0\ud55c \uc801\uc740 \ub2e5\ud130 \uc5d0\uadf8\ub9e8 \uc774\uba70, \ub2e5\ud130 \ub85c\ubcf4\ud2b8\ub2c9 \uc774\ub77c\uace0\ub3c4 \uc54c\ub824\uc838 \uc788\ub2e4.", "paragraph_sentence": " \uc18c\ub2c9\uc758 \uc601\uc6d0\ud55c \uc801\uc740 \ub2e5\ud130 \uc5d0\uadf8\ub9e8 \uc774\uba70, \ub2e5\ud130 \ub85c\ubcf4\ud2b8\ub2c9 \uc774\ub77c\uace0\ub3c4 \uc54c\ub824\uc838 \uc788\ub2e4. IQ 300\uc758 \uc9c0\ub2a5\uc744 \uac00\uc9c4 \uc5d0\uadf8\ub9e8\uc740 \ucc9c\uc7ac\uc801\uc778 \uc545\uc758 \uacfc\ud559\uc790\ub85c \uadf8\uc758 \uc5c4\uccad\ub09c \uae30\uacc4\ub97c \uc774\uc6a9\ud574\uc11c \uc138\uacc4\ub97c \uc815\ubcf5\ud558\ub824\uace0 \ud558\uc9c0\ub9cc, \uadf8\ub7f4 \ub54c\ub9c8\ub2e4 \uc18c\ub2c9\uc758 \ubc29\ud574\ub85c \ubc88\ubc88\uc774 \uc2e4\ud328\ud55c\ub2e4. \uc18c\ub2c9\uc740 \ucc98\uc74c\uc5d0\ub294 \ub2e5\ud130 \uc5d0\uadf8\ub9e8\uc774 \uadf8\uc758 \ub3d9\ubb3c \uce5c\uad6c\ub4e4\uc744 \ubc30\ub4dc\ub2c9\uc774\ub77c\uace0 \ubd88\ub9ac\ub294 \ub85c\ubd07 \ub178\uc608\ub85c \ubc14\uafb8\ub294 \uac83 \ub54c\ubb38\uc5d0 \uc2f8\uc6b0\uac8c \ub418\uc5c8\ub2e4. \uc18c\ub2c9\uc740 \uce5c\uad6c\ub4e4\uc744 \uad6c\ucd9c\ud588\uace0, \uba87\ubc88\uc774\ub098 \uce74\uc624\uc2a4 \uc5d0\uba54\ub784\ub4dc, \ud0c0\uc784 \uc2a4\ud1a4, \uadf8\ub9ac\uace0 \ub9c8\uc2a4\ud130 \uc5d0\uba54\ub784\ub4dc\ub97c \uc774\uc6a9\ud574\uc11c \uc138\uacc4\ub97c \uc815\ubcf5\ud558\ub824\ub294 \uc2dc\ub3c4\ub97c \ub9c9\uc558\ub2e4. \uc885\uc885 \uc774\uac83\ub4e4\uc740 \uc5d0\uadf8\ub9e8\uc758 \uac15\ub825\ud55c \ubaa8\uc2b5\uc744 \ubcf8\ub534 \uba54\uce74 \uc288\ud2b8\ub97c \uc791\ub3d9\uc2dc\ud0a4\ub294 \ub370 \uc0ac\uc6a9\ub418\uc5c8\ub2e4. \ud558\uc9c0\ub9cc, \ucd5c\uadfc \uac8c\uc784\uc5d0\uc11c\ub294 \ub2e5\ud130 \uc5d0\uadf8\ub9e8\uc740 \uc804\ud568\uc774\ub098 \ud575 \ubbf8\uc0ac\uc77c\uc744 \uc774\uc6a9\ud574\uc11c \uc138\uacc4\ub97c \uc815\ubcf5\ud558\ub824\ub294 \ubaa8\uc2b5\uc744 \ubcf4\uc5ec\uc8fc\uace0 \uc788\ub2e4. \uc544\uc639\ub2e4\uc639 \ub2e4\ud22c\ub294 \uc0ac\uc774\uc774\uc9c0\ub9cc \ub2e5\ud130 \uc5d0\uadf8\ub9e8\ub3c4 \uc18c\ub2c9\uc744 \ub192\uc774 \ud3c9\uac00\ud558\uace0 \uc788\uc73c\uba70, \uac1c\uc778\uc801\uc73c\ub85c\ub3c4 \uc798 \uc544\ub294 \uc0ac\uc774\uc778 \uc560\uc99d\uc758 \uad00\uacc4\uc774\uae30\ub3c4 \ud558\ub2e4.", "paragraph_answer": "\uc18c\ub2c9\uc758 \uc601\uc6d0\ud55c \uc801\uc740 \ub2e5\ud130 \uc5d0\uadf8\ub9e8 \uc774\uba70, \ub2e5\ud130 \ub85c\ubcf4\ud2b8\ub2c9 \uc774\ub77c\uace0\ub3c4 \uc54c\ub824\uc838 \uc788\ub2e4. IQ 300\uc758 \uc9c0\ub2a5\uc744 \uac00\uc9c4 \uc5d0\uadf8\ub9e8\uc740 \ucc9c\uc7ac\uc801\uc778 \uc545\uc758 \uacfc\ud559\uc790\ub85c \uadf8\uc758 \uc5c4\uccad\ub09c \uae30\uacc4\ub97c \uc774\uc6a9\ud574\uc11c \uc138\uacc4\ub97c \uc815\ubcf5\ud558\ub824\uace0 \ud558\uc9c0\ub9cc, \uadf8\ub7f4 \ub54c\ub9c8\ub2e4 \uc18c\ub2c9\uc758 \ubc29\ud574\ub85c \ubc88\ubc88\uc774 \uc2e4\ud328\ud55c\ub2e4. \uc18c\ub2c9\uc740 \ucc98\uc74c\uc5d0\ub294 \ub2e5\ud130 \uc5d0\uadf8\ub9e8\uc774 \uadf8\uc758 \ub3d9\ubb3c \uce5c\uad6c\ub4e4\uc744 \ubc30\ub4dc\ub2c9\uc774\ub77c\uace0 \ubd88\ub9ac\ub294 \ub85c\ubd07 \ub178\uc608\ub85c \ubc14\uafb8\ub294 \uac83 \ub54c\ubb38\uc5d0 \uc2f8\uc6b0\uac8c \ub418\uc5c8\ub2e4. \uc18c\ub2c9\uc740 \uce5c\uad6c\ub4e4\uc744 \uad6c\ucd9c\ud588\uace0, \uba87\ubc88\uc774\ub098 \uce74\uc624\uc2a4 \uc5d0\uba54\ub784\ub4dc, \ud0c0\uc784 \uc2a4\ud1a4, \uadf8\ub9ac\uace0 \ub9c8\uc2a4\ud130 \uc5d0\uba54\ub784\ub4dc\ub97c \uc774\uc6a9\ud574\uc11c \uc138\uacc4\ub97c \uc815\ubcf5\ud558\ub824\ub294 \uc2dc\ub3c4\ub97c \ub9c9\uc558\ub2e4. \uc885\uc885 \uc774\uac83\ub4e4\uc740 \uc5d0\uadf8\ub9e8\uc758 \uac15\ub825\ud55c \ubaa8\uc2b5\uc744 \ubcf8\ub534 \uba54\uce74 \uc288\ud2b8\ub97c \uc791\ub3d9\uc2dc\ud0a4\ub294 \ub370 \uc0ac\uc6a9\ub418\uc5c8\ub2e4. \ud558\uc9c0\ub9cc, \ucd5c\uadfc \uac8c\uc784\uc5d0\uc11c\ub294 \ub2e5\ud130 \uc5d0\uadf8\ub9e8\uc740 \uc804\ud568\uc774\ub098 \ud575 \ubbf8\uc0ac\uc77c\uc744 \uc774\uc6a9\ud574\uc11c \uc138\uacc4\ub97c \uc815\ubcf5\ud558\ub824\ub294 \ubaa8\uc2b5\uc744 \ubcf4\uc5ec\uc8fc\uace0 \uc788\ub2e4. \uc544\uc639\ub2e4\uc639 \ub2e4\ud22c\ub294 \uc0ac\uc774\uc774\uc9c0\ub9cc \ub2e5\ud130 \uc5d0\uadf8\ub9e8\ub3c4 \uc18c\ub2c9\uc744 \ub192\uc774 \ud3c9\uac00\ud558\uace0 \uc788\uc73c\uba70, \uac1c\uc778\uc801\uc73c\ub85c\ub3c4 \uc798 \uc544\ub294 \uc0ac\uc774\uc778 \uc560\uc99d\uc758 \uad00\uacc4\uc774\uae30\ub3c4 \ud558\ub2e4.", "sentence_answer": "\uc18c\ub2c9\uc758 \uc601\uc6d0\ud55c \uc801\uc740 \ub2e5\ud130 \uc5d0\uadf8\ub9e8 \uc774\uba70, \ub2e5\ud130 \ub85c\ubcf4\ud2b8\ub2c9 \uc774\ub77c\uace0\ub3c4 \uc54c\ub824\uc838 \uc788\ub2e4.", "paragraph_id": "6584154-7-0", "paragraph_question": "question: \ub2e5\ud130 \ub85c\ubcf4\ud2b8\ub2c9\uc774\ub77c\uace0\ub3c4 \ubd88\ub9ac\ub294 \uc18c\ub2c9\uc758 \uc601\uc6d0\ud55c \uc801\uc740?, context: \uc18c\ub2c9\uc758 \uc601\uc6d0\ud55c \uc801\uc740 \ub2e5\ud130 \uc5d0\uadf8\ub9e8\uc774\uba70, \ub2e5\ud130 \ub85c\ubcf4\ud2b8\ub2c9 \uc774\ub77c\uace0\ub3c4 \uc54c\ub824\uc838 \uc788\ub2e4. IQ 300\uc758 \uc9c0\ub2a5\uc744 \uac00\uc9c4 \uc5d0\uadf8\ub9e8\uc740 \ucc9c\uc7ac\uc801\uc778 \uc545\uc758 \uacfc\ud559\uc790\ub85c \uadf8\uc758 \uc5c4\uccad\ub09c \uae30\uacc4\ub97c \uc774\uc6a9\ud574\uc11c \uc138\uacc4\ub97c \uc815\ubcf5\ud558\ub824\uace0 \ud558\uc9c0\ub9cc, \uadf8\ub7f4 \ub54c\ub9c8\ub2e4 \uc18c\ub2c9\uc758 \ubc29\ud574\ub85c \ubc88\ubc88\uc774 \uc2e4\ud328\ud55c\ub2e4. \uc18c\ub2c9\uc740 \ucc98\uc74c\uc5d0\ub294 \ub2e5\ud130 \uc5d0\uadf8\ub9e8\uc774 \uadf8\uc758 \ub3d9\ubb3c \uce5c\uad6c\ub4e4\uc744 \ubc30\ub4dc\ub2c9\uc774\ub77c\uace0 \ubd88\ub9ac\ub294 \ub85c\ubd07 \ub178\uc608\ub85c \ubc14\uafb8\ub294 \uac83 \ub54c\ubb38\uc5d0 \uc2f8\uc6b0\uac8c \ub418\uc5c8\ub2e4. \uc18c\ub2c9\uc740 \uce5c\uad6c\ub4e4\uc744 \uad6c\ucd9c\ud588\uace0, \uba87\ubc88\uc774\ub098 \uce74\uc624\uc2a4 \uc5d0\uba54\ub784\ub4dc, \ud0c0\uc784 \uc2a4\ud1a4, \uadf8\ub9ac\uace0 \ub9c8\uc2a4\ud130 \uc5d0\uba54\ub784\ub4dc\ub97c \uc774\uc6a9\ud574\uc11c \uc138\uacc4\ub97c \uc815\ubcf5\ud558\ub824\ub294 \uc2dc\ub3c4\ub97c \ub9c9\uc558\ub2e4. \uc885\uc885 \uc774\uac83\ub4e4\uc740 \uc5d0\uadf8\ub9e8\uc758 \uac15\ub825\ud55c \ubaa8\uc2b5\uc744 \ubcf8\ub534 \uba54\uce74 \uc288\ud2b8\ub97c \uc791\ub3d9\uc2dc\ud0a4\ub294 \ub370 \uc0ac\uc6a9\ub418\uc5c8\ub2e4. \ud558\uc9c0\ub9cc, \ucd5c\uadfc \uac8c\uc784\uc5d0\uc11c\ub294 \ub2e5\ud130 \uc5d0\uadf8\ub9e8\uc740 \uc804\ud568\uc774\ub098 \ud575 \ubbf8\uc0ac\uc77c\uc744 \uc774\uc6a9\ud574\uc11c \uc138\uacc4\ub97c \uc815\ubcf5\ud558\ub824\ub294 \ubaa8\uc2b5\uc744 \ubcf4\uc5ec\uc8fc\uace0 \uc788\ub2e4. \uc544\uc639\ub2e4\uc639 \ub2e4\ud22c\ub294 \uc0ac\uc774\uc774\uc9c0\ub9cc \ub2e5\ud130 \uc5d0\uadf8\ub9e8\ub3c4 \uc18c\ub2c9\uc744 \ub192\uc774 \ud3c9\uac00\ud558\uace0 \uc788\uc73c\uba70, \uac1c\uc778\uc801\uc73c\ub85c\ub3c4 \uc798 \uc544\ub294 \uc0ac\uc774\uc778 \uc560\uc99d\uc758 \uad00\uacc4\uc774\uae30\ub3c4 \ud558\ub2e4."} {"question": "7\uc6d4 26\uc77c \ud1b5\uc0c1\uc870\uc9c1 \ubb38\uc81c\ub294 \uc5b4\ub290 \ubc95\uc548\uc758 \uac1c\uc815\uc548\uc774 \ud1b5\uacfc\ub418\uba74\uc11c \uc77c\ub2e8\ub77d \ub418\uc5c8\ub294\uac00?", "paragraph": "\ubb38\uc7ac\uc778 \uc815\ubd80\uac00 \ucd9c\ubc94\ud560 \ub2f9\uc2dc \ud1b5\uc0c1 \ubb38\uc81c\ub294 \ub2f9\uc7a5 \ud574\uacb0\ud574\uc57c \ud560 \ub2f9\uba74 \uacfc\uc81c\ub85c \ubd80\uc0c1\ud574 \uc788\uc5c8\ub2e4. \ub3c4\ub110\ub4dc \ud2b8\ub7fc\ud504\uac00 \ubbf8\uad6d\uc758 \ub300\ud1b5\ub839\uc73c\ub85c \ucde8\uc784\ud558\uace0 \ud55c\ubbf8 FTA\uc5d0 \ub300\ud55c \ubc1c\uc5b8\uc744 \uc3df\uc544\ub0b4\uc790 \uc7ac\ud611\uc0c1\uc774 \uac00\uc2dc\ud654\ub418\uace0 \uc788\uc73c\uba70 \uc0ac\ub4dc \uc124\uce58\uc5d0 \ub530\ub978 \uc911\uad6d\uc758 \uacbd\uc81c\ubcf4\ubcf5\ub3c4 \uc704\ud5d8\uc218\uc704\ub97c \ub118\ub098\ub4e4\uace0 \uc788\uc5b4 \ubbf8\uc911 \uc591\uad6d\uc758 \"\ubcf4\ud638\ubb34\uc5ed\uc8fc\uc758\uc801\uc778 \ud1b5\uc0c1\ucd94\uc9c4\uc804\ub7b5\uc5d0 \uacf5\uc138\uc801 \ub300\uc751\ubc29\uc548\uc744 \ub9c8\ub828\ud560 \ud544\uc694\"\uc131\uc774 \uc81c\uae30\ub418\uc5c8\ub2e4. \ud558\uc9c0\ub9cc \ud1b5\uc0c1 \uc870\uc9c1\uc744 \uc5b4\ub5bb\uac8c \ud560 \uac83\uc778\uc9c0\ub85c \ub0b4\ubd80\uac00 \ub4a4\uc22d\uc22d\ud558\uace0 \uc778\uc120\ub3c4 \uac78\uc74c\ub9c8 \ub2e8\uacc4\ub77c \uad00\ub828 \ub300\uc751\uc774 \uc18d\ub3c4\ub97c \ub0b4\uc9c0 \ubabb\ud558\uace0 \uc788\ub2e4. \ud1b5\uc0c1 \uc0ac\ubb34\ub294 1998\ub144 \uc774\ub798 \uc678\uad50\ubd80\uc5d0\uc11c \ub9e1\uc544\uc624\ub358 \uac83\uc744 2013\ub144 \uc0b0\uc5c5\ubd80\ub85c \uc774\uad00\ud558\uc600\ub294\ub370 \ubb38\uc7ac\uc778\uc740 \ub300\uc120 \ud6c4\ubcf4 \ub2f9\uc2dc \uc774\ub97c \ube44\ud310\ud558\uba74\uc11c \uc678\uad50\ubd80\ub85c \ud658\uc6d0\ud560 \uac83\uc744 \uc8fc\uc7a5\ud55c \uac83\uc774\ub2e4. \ud558\uc9c0\ub9cc \ud1b5\uc0c1\uc870\uc9c1\uc774 \"\uc678\uad50\ubd80\uc5d0\uc11c \uc0b0\uc5c5\ubd80\ub85c \ub118\uc5b4\uc62c \ub54c \uc7a5\uad00\uae09\uc5d0\uc11c \ucc28\uad00\ubcf4\uae09\uc73c\ub85c \uc904\uc5ec\uc11c \ub118\uc5b4\uc628\" \uac83\uc774 \ubb38\uc81c\uac00 \ub418\uc5c8\ub2e4\uba70 \ud1b5\uc0c1 \uc815\ucc45\uc758 \uacb0\uacfc\ub97c \ubb34\uc870\uac74 \uc0b0\uc5c5\ubd80\uc5d0\uac8c\ub9cc \ub36e\uc5b4\uc50c\uc6b0\ub294 \uac83\uc740 \ubd80\ub2f9\ud558\ub2e4\ub294 \uc8fc\uc7a5\uc774 \uc81c\uae30\ub418\uc5c8\ub2e4. \uc774\ud6c4 7\uc6d4 26\uc77c \uc0b0\uc5c5\ubd80 \ub0b4\uc5d0 \ud1b5\uc0c1\uad50\uc12d\ubcf8\ubd80\ub97c \uc124\uce58\ud558\ub294 \u300c\uc815\ubd80\uc870\uc9c1\ubc95\u300d \uac1c\uc815\uc548\uc774 \ud1b5\uacfc\ub418\uba74\uc11c \ud1b5\uc0c1\uc870\uc9c1 \ubb38\uc81c\ub294 \uc77c\ub2e8\ub77d\ub418\uc5c8\ub2e4.", "answer": "\uc815\ubd80\uc870\uc9c1\ubc95", "sentence": "\uc774\ud6c4 7\uc6d4 26\uc77c \uc0b0\uc5c5\ubd80 \ub0b4\uc5d0 \ud1b5\uc0c1\uad50\uc12d\ubcf8\ubd80\ub97c \uc124\uce58\ud558\ub294 \u300c \uc815\ubd80\uc870\uc9c1\ubc95 \u300d \uac1c\uc815\uc548\uc774 \ud1b5\uacfc\ub418\uba74\uc11c \ud1b5\uc0c1\uc870\uc9c1 \ubb38\uc81c\ub294 \uc77c\ub2e8\ub77d\ub418\uc5c8\ub2e4.", "paragraph_sentence": "\ubb38\uc7ac\uc778 \uc815\ubd80\uac00 \ucd9c\ubc94\ud560 \ub2f9\uc2dc \ud1b5\uc0c1 \ubb38\uc81c\ub294 \ub2f9\uc7a5 \ud574\uacb0\ud574\uc57c \ud560 \ub2f9\uba74 \uacfc\uc81c\ub85c \ubd80\uc0c1\ud574 \uc788\uc5c8\ub2e4. \ub3c4\ub110\ub4dc \ud2b8\ub7fc\ud504\uac00 \ubbf8\uad6d\uc758 \ub300\ud1b5\ub839\uc73c\ub85c \ucde8\uc784\ud558\uace0 \ud55c\ubbf8 FTA\uc5d0 \ub300\ud55c \ubc1c\uc5b8\uc744 \uc3df\uc544\ub0b4\uc790 \uc7ac\ud611\uc0c1\uc774 \uac00\uc2dc\ud654\ub418\uace0 \uc788\uc73c\uba70 \uc0ac\ub4dc \uc124\uce58\uc5d0 \ub530\ub978 \uc911\uad6d\uc758 \uacbd\uc81c\ubcf4\ubcf5\ub3c4 \uc704\ud5d8\uc218\uc704\ub97c \ub118\ub098\ub4e4\uace0 \uc788\uc5b4 \ubbf8\uc911 \uc591\uad6d\uc758 \"\ubcf4\ud638\ubb34\uc5ed\uc8fc\uc758\uc801\uc778 \ud1b5\uc0c1\ucd94\uc9c4\uc804\ub7b5\uc5d0 \uacf5\uc138\uc801 \ub300\uc751\ubc29\uc548\uc744 \ub9c8\ub828\ud560 \ud544\uc694\"\uc131\uc774 \uc81c\uae30\ub418\uc5c8\ub2e4. \ud558\uc9c0\ub9cc \ud1b5\uc0c1 \uc870\uc9c1\uc744 \uc5b4\ub5bb\uac8c \ud560 \uac83\uc778\uc9c0\ub85c \ub0b4\ubd80\uac00 \ub4a4\uc22d\uc22d\ud558\uace0 \uc778\uc120\ub3c4 \uac78\uc74c\ub9c8 \ub2e8\uacc4\ub77c \uad00\ub828 \ub300\uc751\uc774 \uc18d\ub3c4\ub97c \ub0b4\uc9c0 \ubabb\ud558\uace0 \uc788\ub2e4. \ud1b5\uc0c1 \uc0ac\ubb34\ub294 1998\ub144 \uc774\ub798 \uc678\uad50\ubd80\uc5d0\uc11c \ub9e1\uc544\uc624\ub358 \uac83\uc744 2013\ub144 \uc0b0\uc5c5\ubd80\ub85c \uc774\uad00\ud558\uc600\ub294\ub370 \ubb38\uc7ac\uc778\uc740 \ub300\uc120 \ud6c4\ubcf4 \ub2f9\uc2dc \uc774\ub97c \ube44\ud310\ud558\uba74\uc11c \uc678\uad50\ubd80\ub85c \ud658\uc6d0\ud560 \uac83\uc744 \uc8fc\uc7a5\ud55c \uac83\uc774\ub2e4. \ud558\uc9c0\ub9cc \ud1b5\uc0c1\uc870\uc9c1\uc774 \"\uc678\uad50\ubd80\uc5d0\uc11c \uc0b0\uc5c5\ubd80\ub85c \ub118\uc5b4\uc62c \ub54c \uc7a5\uad00\uae09\uc5d0\uc11c \ucc28\uad00\ubcf4\uae09\uc73c\ub85c \uc904\uc5ec\uc11c \ub118\uc5b4\uc628\" \uac83\uc774 \ubb38\uc81c\uac00 \ub418\uc5c8\ub2e4\uba70 \ud1b5\uc0c1 \uc815\ucc45\uc758 \uacb0\uacfc\ub97c \ubb34\uc870\uac74 \uc0b0\uc5c5\ubd80\uc5d0\uac8c\ub9cc \ub36e\uc5b4\uc50c\uc6b0\ub294 \uac83\uc740 \ubd80\ub2f9\ud558\ub2e4\ub294 \uc8fc\uc7a5\uc774 \uc81c\uae30\ub418\uc5c8\ub2e4. \uc774\ud6c4 7\uc6d4 26\uc77c \uc0b0\uc5c5\ubd80 \ub0b4\uc5d0 \ud1b5\uc0c1\uad50\uc12d\ubcf8\ubd80\ub97c \uc124\uce58\ud558\ub294 \u300c \uc815\ubd80\uc870\uc9c1\ubc95 \u300d \uac1c\uc815\uc548\uc774 \ud1b5\uacfc\ub418\uba74\uc11c \ud1b5\uc0c1\uc870\uc9c1 \ubb38\uc81c\ub294 \uc77c\ub2e8\ub77d\ub418\uc5c8\ub2e4. ", "paragraph_answer": "\ubb38\uc7ac\uc778 \uc815\ubd80\uac00 \ucd9c\ubc94\ud560 \ub2f9\uc2dc \ud1b5\uc0c1 \ubb38\uc81c\ub294 \ub2f9\uc7a5 \ud574\uacb0\ud574\uc57c \ud560 \ub2f9\uba74 \uacfc\uc81c\ub85c \ubd80\uc0c1\ud574 \uc788\uc5c8\ub2e4. \ub3c4\ub110\ub4dc \ud2b8\ub7fc\ud504\uac00 \ubbf8\uad6d\uc758 \ub300\ud1b5\ub839\uc73c\ub85c \ucde8\uc784\ud558\uace0 \ud55c\ubbf8 FTA\uc5d0 \ub300\ud55c \ubc1c\uc5b8\uc744 \uc3df\uc544\ub0b4\uc790 \uc7ac\ud611\uc0c1\uc774 \uac00\uc2dc\ud654\ub418\uace0 \uc788\uc73c\uba70 \uc0ac\ub4dc \uc124\uce58\uc5d0 \ub530\ub978 \uc911\uad6d\uc758 \uacbd\uc81c\ubcf4\ubcf5\ub3c4 \uc704\ud5d8\uc218\uc704\ub97c \ub118\ub098\ub4e4\uace0 \uc788\uc5b4 \ubbf8\uc911 \uc591\uad6d\uc758 \"\ubcf4\ud638\ubb34\uc5ed\uc8fc\uc758\uc801\uc778 \ud1b5\uc0c1\ucd94\uc9c4\uc804\ub7b5\uc5d0 \uacf5\uc138\uc801 \ub300\uc751\ubc29\uc548\uc744 \ub9c8\ub828\ud560 \ud544\uc694\"\uc131\uc774 \uc81c\uae30\ub418\uc5c8\ub2e4. \ud558\uc9c0\ub9cc \ud1b5\uc0c1 \uc870\uc9c1\uc744 \uc5b4\ub5bb\uac8c \ud560 \uac83\uc778\uc9c0\ub85c \ub0b4\ubd80\uac00 \ub4a4\uc22d\uc22d\ud558\uace0 \uc778\uc120\ub3c4 \uac78\uc74c\ub9c8 \ub2e8\uacc4\ub77c \uad00\ub828 \ub300\uc751\uc774 \uc18d\ub3c4\ub97c \ub0b4\uc9c0 \ubabb\ud558\uace0 \uc788\ub2e4. \ud1b5\uc0c1 \uc0ac\ubb34\ub294 1998\ub144 \uc774\ub798 \uc678\uad50\ubd80\uc5d0\uc11c \ub9e1\uc544\uc624\ub358 \uac83\uc744 2013\ub144 \uc0b0\uc5c5\ubd80\ub85c \uc774\uad00\ud558\uc600\ub294\ub370 \ubb38\uc7ac\uc778\uc740 \ub300\uc120 \ud6c4\ubcf4 \ub2f9\uc2dc \uc774\ub97c \ube44\ud310\ud558\uba74\uc11c \uc678\uad50\ubd80\ub85c \ud658\uc6d0\ud560 \uac83\uc744 \uc8fc\uc7a5\ud55c \uac83\uc774\ub2e4. \ud558\uc9c0\ub9cc \ud1b5\uc0c1\uc870\uc9c1\uc774 \"\uc678\uad50\ubd80\uc5d0\uc11c \uc0b0\uc5c5\ubd80\ub85c \ub118\uc5b4\uc62c \ub54c \uc7a5\uad00\uae09\uc5d0\uc11c \ucc28\uad00\ubcf4\uae09\uc73c\ub85c \uc904\uc5ec\uc11c \ub118\uc5b4\uc628\" \uac83\uc774 \ubb38\uc81c\uac00 \ub418\uc5c8\ub2e4\uba70 \ud1b5\uc0c1 \uc815\ucc45\uc758 \uacb0\uacfc\ub97c \ubb34\uc870\uac74 \uc0b0\uc5c5\ubd80\uc5d0\uac8c\ub9cc \ub36e\uc5b4\uc50c\uc6b0\ub294 \uac83\uc740 \ubd80\ub2f9\ud558\ub2e4\ub294 \uc8fc\uc7a5\uc774 \uc81c\uae30\ub418\uc5c8\ub2e4. \uc774\ud6c4 7\uc6d4 26\uc77c \uc0b0\uc5c5\ubd80 \ub0b4\uc5d0 \ud1b5\uc0c1\uad50\uc12d\ubcf8\ubd80\ub97c \uc124\uce58\ud558\ub294 \u300c \uc815\ubd80\uc870\uc9c1\ubc95 \u300d \uac1c\uc815\uc548\uc774 \ud1b5\uacfc\ub418\uba74\uc11c \ud1b5\uc0c1\uc870\uc9c1 \ubb38\uc81c\ub294 \uc77c\ub2e8\ub77d\ub418\uc5c8\ub2e4.", "sentence_answer": "\uc774\ud6c4 7\uc6d4 26\uc77c \uc0b0\uc5c5\ubd80 \ub0b4\uc5d0 \ud1b5\uc0c1\uad50\uc12d\ubcf8\ubd80\ub97c \uc124\uce58\ud558\ub294 \u300c \uc815\ubd80\uc870\uc9c1\ubc95 \u300d \uac1c\uc815\uc548\uc774 \ud1b5\uacfc\ub418\uba74\uc11c \ud1b5\uc0c1\uc870\uc9c1 \ubb38\uc81c\ub294 \uc77c\ub2e8\ub77d\ub418\uc5c8\ub2e4.", "paragraph_id": "6580768-77-1", "paragraph_question": "question: 7\uc6d4 26\uc77c \ud1b5\uc0c1\uc870\uc9c1 \ubb38\uc81c\ub294 \uc5b4\ub290 \ubc95\uc548\uc758 \uac1c\uc815\uc548\uc774 \ud1b5\uacfc\ub418\uba74\uc11c \uc77c\ub2e8\ub77d \ub418\uc5c8\ub294\uac00?, context: \ubb38\uc7ac\uc778 \uc815\ubd80\uac00 \ucd9c\ubc94\ud560 \ub2f9\uc2dc \ud1b5\uc0c1 \ubb38\uc81c\ub294 \ub2f9\uc7a5 \ud574\uacb0\ud574\uc57c \ud560 \ub2f9\uba74 \uacfc\uc81c\ub85c \ubd80\uc0c1\ud574 \uc788\uc5c8\ub2e4. \ub3c4\ub110\ub4dc \ud2b8\ub7fc\ud504\uac00 \ubbf8\uad6d\uc758 \ub300\ud1b5\ub839\uc73c\ub85c \ucde8\uc784\ud558\uace0 \ud55c\ubbf8 FTA\uc5d0 \ub300\ud55c \ubc1c\uc5b8\uc744 \uc3df\uc544\ub0b4\uc790 \uc7ac\ud611\uc0c1\uc774 \uac00\uc2dc\ud654\ub418\uace0 \uc788\uc73c\uba70 \uc0ac\ub4dc \uc124\uce58\uc5d0 \ub530\ub978 \uc911\uad6d\uc758 \uacbd\uc81c\ubcf4\ubcf5\ub3c4 \uc704\ud5d8\uc218\uc704\ub97c \ub118\ub098\ub4e4\uace0 \uc788\uc5b4 \ubbf8\uc911 \uc591\uad6d\uc758 \"\ubcf4\ud638\ubb34\uc5ed\uc8fc\uc758\uc801\uc778 \ud1b5\uc0c1\ucd94\uc9c4\uc804\ub7b5\uc5d0 \uacf5\uc138\uc801 \ub300\uc751\ubc29\uc548\uc744 \ub9c8\ub828\ud560 \ud544\uc694\"\uc131\uc774 \uc81c\uae30\ub418\uc5c8\ub2e4. \ud558\uc9c0\ub9cc \ud1b5\uc0c1 \uc870\uc9c1\uc744 \uc5b4\ub5bb\uac8c \ud560 \uac83\uc778\uc9c0\ub85c \ub0b4\ubd80\uac00 \ub4a4\uc22d\uc22d\ud558\uace0 \uc778\uc120\ub3c4 \uac78\uc74c\ub9c8 \ub2e8\uacc4\ub77c \uad00\ub828 \ub300\uc751\uc774 \uc18d\ub3c4\ub97c \ub0b4\uc9c0 \ubabb\ud558\uace0 \uc788\ub2e4. \ud1b5\uc0c1 \uc0ac\ubb34\ub294 1998\ub144 \uc774\ub798 \uc678\uad50\ubd80\uc5d0\uc11c \ub9e1\uc544\uc624\ub358 \uac83\uc744 2013\ub144 \uc0b0\uc5c5\ubd80\ub85c \uc774\uad00\ud558\uc600\ub294\ub370 \ubb38\uc7ac\uc778\uc740 \ub300\uc120 \ud6c4\ubcf4 \ub2f9\uc2dc \uc774\ub97c \ube44\ud310\ud558\uba74\uc11c \uc678\uad50\ubd80\ub85c \ud658\uc6d0\ud560 \uac83\uc744 \uc8fc\uc7a5\ud55c \uac83\uc774\ub2e4. \ud558\uc9c0\ub9cc \ud1b5\uc0c1\uc870\uc9c1\uc774 \"\uc678\uad50\ubd80\uc5d0\uc11c \uc0b0\uc5c5\ubd80\ub85c \ub118\uc5b4\uc62c \ub54c \uc7a5\uad00\uae09\uc5d0\uc11c \ucc28\uad00\ubcf4\uae09\uc73c\ub85c \uc904\uc5ec\uc11c \ub118\uc5b4\uc628\" \uac83\uc774 \ubb38\uc81c\uac00 \ub418\uc5c8\ub2e4\uba70 \ud1b5\uc0c1 \uc815\ucc45\uc758 \uacb0\uacfc\ub97c \ubb34\uc870\uac74 \uc0b0\uc5c5\ubd80\uc5d0\uac8c\ub9cc \ub36e\uc5b4\uc50c\uc6b0\ub294 \uac83\uc740 \ubd80\ub2f9\ud558\ub2e4\ub294 \uc8fc\uc7a5\uc774 \uc81c\uae30\ub418\uc5c8\ub2e4. \uc774\ud6c4 7\uc6d4 26\uc77c \uc0b0\uc5c5\ubd80 \ub0b4\uc5d0 \ud1b5\uc0c1\uad50\uc12d\ubcf8\ubd80\ub97c \uc124\uce58\ud558\ub294 \u300c\uc815\ubd80\uc870\uc9c1\ubc95\u300d \uac1c\uc815\uc548\uc774 \ud1b5\uacfc\ub418\uba74\uc11c \ud1b5\uc0c1\uc870\uc9c1 \ubb38\uc81c\ub294 \uc77c\ub2e8\ub77d\ub418\uc5c8\ub2e4."} {"question": "1981\ub144 \uc804\ub450\ud658\uc774 \ub300\ud1b5\ub839\uc774 \ub41c \uc120\uac70 \ubc29\ubc95\uc740?", "paragraph": "1981\ub144 3\uc6d4 3\uc77c \uc5ed\uc2dc \uccb4\uc721\uad00\uc5d0\uc11c\uc758 \uac04\uc811 \uc120\uac70\ub97c \ud1b5\ud574 \uc2a4\uc2a4\ub85c \ub300\ud55c\ubbfc\uad6d\uc758 \uc81c12\ub300 \ub300\ud1b5\ub839\uc5d0 \ucde8\uc784\ud558\uc600\uace0, \uadf8\ud574 5\uc6d4 \uad6d\ud48d 81 \ucd95\uc81c\ub97c \uac1c\ucd5c\ud558\uc5ec \uad11\uc8fc \ubbfc\uc8fc\ud654 \uc6b4\ub3d9 1\uc8fc\uae30\uc5d0 \ub300\ud55c \uad00\uc2ec\uacfc \ubd84\uc704\uae30\ub97c \ub2e4\ub978 \uacf3\uc73c\ub85c \ub3cc\ub9ac\ub824 \uc2dc\ub3c4\ud588\ub2e4. \uc804\ub450\ud658 \uc815\uad8c\uc740 \uc774\ud6c4 1982\ub144 \ud55c\uad6d\ud504\ub85c\uc57c\uad6c\ub97c \ucc3d\uc124\ud558\uace0 \uc57c\uac04\ud1b5\ud589\uae08\uc9c0 \uc870\uce58\ub97c \ud574\uc81c\ud558\uc600\uc73c\uba70, \ud559\uc6d0 \ub450\ubc1c/\ubcf5\uc7a5 \uc790\uc728\ud654 \uc815\ucc45\uc744 \uc2dc\ub3c4\ud558\ub294 \ub4f1 \uad6d\ubbfc\ub4e4\uc758 \ud658\uc2ec\uc744 \uc0ac\uae30 \uc704\ud55c \uc720\ud654\ucc45\uc744 \ub0b4\uc138\uc6e0\ub2e4. \uceec\ub7ec \ud154\ub808\ube44\uc804 \ubcf4\uae09, \ud3ec\ub974\ub178 \uc601\ud654 \uc7a5\ub824\ub85c \ub300\ud45c\ub418\ub294 3S \uc815\ucc45 \ub610\ud55c \uc774\ub7ec\ud55c \uc720\ud654\ucc45\uc758 \uc77c\ud658\uc774\uc5c8\ub2e4. \uadf8\ub7ec\ub098 \ud55c\ud3b8\uc73c\ub85c\ub294 \uc0bc\uccad\uad50\uc721\ub300\ub97c \ucc3d\uc124\ud558\uc5ec \uc720\uc8c4 \ud310\uacb0\uc744 \ub0b4\ub9b0 \ubc94\ubc95\uc790\ub4e4\uc758 \uc778\uad8c\uc744 \uc720\ub9b0\ud558\uc600\uc73c\uba70, \ubbfc\uc8fc\ud654 \uc6b4\ub3d9\uc5d0 \ub300\ud55c \ud0c4\uc555 \ub610\ud55c \uc9c0\uc18d\ud558\uc600\ub2e4.", "answer": "\uac04\uc811 \uc120\uac70", "sentence": "1981\ub144 3\uc6d4 3\uc77c \uc5ed\uc2dc \uccb4\uc721\uad00\uc5d0\uc11c\uc758 \uac04\uc811 \uc120\uac70 \ub97c \ud1b5\ud574 \uc2a4\uc2a4\ub85c \ub300\ud55c\ubbfc\uad6d\uc758 \uc81c12\ub300 \ub300\ud1b5\ub839\uc5d0 \ucde8\uc784\ud558\uc600\uace0, \uadf8\ud574 5\uc6d4 \uad6d\ud48d 81 \ucd95\uc81c\ub97c \uac1c\ucd5c\ud558\uc5ec \uad11\uc8fc \ubbfc\uc8fc\ud654 \uc6b4\ub3d9 1\uc8fc\uae30\uc5d0 \ub300\ud55c \uad00\uc2ec\uacfc \ubd84\uc704\uae30\ub97c \ub2e4\ub978 \uacf3\uc73c\ub85c \ub3cc\ub9ac\ub824 \uc2dc\ub3c4\ud588\ub2e4.", "paragraph_sentence": " 1981\ub144 3\uc6d4 3\uc77c \uc5ed\uc2dc \uccb4\uc721\uad00\uc5d0\uc11c\uc758 \uac04\uc811 \uc120\uac70 \ub97c \ud1b5\ud574 \uc2a4\uc2a4\ub85c \ub300\ud55c\ubbfc\uad6d\uc758 \uc81c12\ub300 \ub300\ud1b5\ub839\uc5d0 \ucde8\uc784\ud558\uc600\uace0, \uadf8\ud574 5\uc6d4 \uad6d\ud48d 81 \ucd95\uc81c\ub97c \uac1c\ucd5c\ud558\uc5ec \uad11\uc8fc \ubbfc\uc8fc\ud654 \uc6b4\ub3d9 1\uc8fc\uae30\uc5d0 \ub300\ud55c \uad00\uc2ec\uacfc \ubd84\uc704\uae30\ub97c \ub2e4\ub978 \uacf3\uc73c\ub85c \ub3cc\ub9ac\ub824 \uc2dc\ub3c4\ud588\ub2e4. \uc804\ub450\ud658 \uc815\uad8c\uc740 \uc774\ud6c4 1982\ub144 \ud55c\uad6d\ud504\ub85c\uc57c\uad6c\ub97c \ucc3d\uc124\ud558\uace0 \uc57c\uac04\ud1b5\ud589\uae08\uc9c0 \uc870\uce58\ub97c \ud574\uc81c\ud558\uc600\uc73c\uba70, \ud559\uc6d0 \ub450\ubc1c/\ubcf5\uc7a5 \uc790\uc728\ud654 \uc815\ucc45\uc744 \uc2dc\ub3c4\ud558\ub294 \ub4f1 \uad6d\ubbfc\ub4e4\uc758 \ud658\uc2ec\uc744 \uc0ac\uae30 \uc704\ud55c \uc720\ud654\ucc45\uc744 \ub0b4\uc138\uc6e0\ub2e4. \uceec\ub7ec \ud154\ub808\ube44\uc804 \ubcf4\uae09, \ud3ec\ub974\ub178 \uc601\ud654 \uc7a5\ub824\ub85c \ub300\ud45c\ub418\ub294 3S \uc815\ucc45 \ub610\ud55c \uc774\ub7ec\ud55c \uc720\ud654\ucc45\uc758 \uc77c\ud658\uc774\uc5c8\ub2e4. \uadf8\ub7ec\ub098 \ud55c\ud3b8\uc73c\ub85c\ub294 \uc0bc\uccad\uad50\uc721\ub300\ub97c \ucc3d\uc124\ud558\uc5ec \uc720\uc8c4 \ud310\uacb0\uc744 \ub0b4\ub9b0 \ubc94\ubc95\uc790\ub4e4\uc758 \uc778\uad8c\uc744 \uc720\ub9b0\ud558\uc600\uc73c\uba70, \ubbfc\uc8fc\ud654 \uc6b4\ub3d9\uc5d0 \ub300\ud55c \ud0c4\uc555 \ub610\ud55c \uc9c0\uc18d\ud558\uc600\ub2e4.", "paragraph_answer": "1981\ub144 3\uc6d4 3\uc77c \uc5ed\uc2dc \uccb4\uc721\uad00\uc5d0\uc11c\uc758 \uac04\uc811 \uc120\uac70 \ub97c \ud1b5\ud574 \uc2a4\uc2a4\ub85c \ub300\ud55c\ubbfc\uad6d\uc758 \uc81c12\ub300 \ub300\ud1b5\ub839\uc5d0 \ucde8\uc784\ud558\uc600\uace0, \uadf8\ud574 5\uc6d4 \uad6d\ud48d 81 \ucd95\uc81c\ub97c \uac1c\ucd5c\ud558\uc5ec \uad11\uc8fc \ubbfc\uc8fc\ud654 \uc6b4\ub3d9 1\uc8fc\uae30\uc5d0 \ub300\ud55c \uad00\uc2ec\uacfc \ubd84\uc704\uae30\ub97c \ub2e4\ub978 \uacf3\uc73c\ub85c \ub3cc\ub9ac\ub824 \uc2dc\ub3c4\ud588\ub2e4. \uc804\ub450\ud658 \uc815\uad8c\uc740 \uc774\ud6c4 1982\ub144 \ud55c\uad6d\ud504\ub85c\uc57c\uad6c\ub97c \ucc3d\uc124\ud558\uace0 \uc57c\uac04\ud1b5\ud589\uae08\uc9c0 \uc870\uce58\ub97c \ud574\uc81c\ud558\uc600\uc73c\uba70, \ud559\uc6d0 \ub450\ubc1c/\ubcf5\uc7a5 \uc790\uc728\ud654 \uc815\ucc45\uc744 \uc2dc\ub3c4\ud558\ub294 \ub4f1 \uad6d\ubbfc\ub4e4\uc758 \ud658\uc2ec\uc744 \uc0ac\uae30 \uc704\ud55c \uc720\ud654\ucc45\uc744 \ub0b4\uc138\uc6e0\ub2e4. \uceec\ub7ec \ud154\ub808\ube44\uc804 \ubcf4\uae09, \ud3ec\ub974\ub178 \uc601\ud654 \uc7a5\ub824\ub85c \ub300\ud45c\ub418\ub294 3S \uc815\ucc45 \ub610\ud55c \uc774\ub7ec\ud55c \uc720\ud654\ucc45\uc758 \uc77c\ud658\uc774\uc5c8\ub2e4. \uadf8\ub7ec\ub098 \ud55c\ud3b8\uc73c\ub85c\ub294 \uc0bc\uccad\uad50\uc721\ub300\ub97c \ucc3d\uc124\ud558\uc5ec \uc720\uc8c4 \ud310\uacb0\uc744 \ub0b4\ub9b0 \ubc94\ubc95\uc790\ub4e4\uc758 \uc778\uad8c\uc744 \uc720\ub9b0\ud558\uc600\uc73c\uba70, \ubbfc\uc8fc\ud654 \uc6b4\ub3d9\uc5d0 \ub300\ud55c \ud0c4\uc555 \ub610\ud55c \uc9c0\uc18d\ud558\uc600\ub2e4.", "sentence_answer": "1981\ub144 3\uc6d4 3\uc77c \uc5ed\uc2dc \uccb4\uc721\uad00\uc5d0\uc11c\uc758 \uac04\uc811 \uc120\uac70 \ub97c \ud1b5\ud574 \uc2a4\uc2a4\ub85c \ub300\ud55c\ubbfc\uad6d\uc758 \uc81c12\ub300 \ub300\ud1b5\ub839\uc5d0 \ucde8\uc784\ud558\uc600\uace0, \uadf8\ud574 5\uc6d4 \uad6d\ud48d 81 \ucd95\uc81c\ub97c \uac1c\ucd5c\ud558\uc5ec \uad11\uc8fc \ubbfc\uc8fc\ud654 \uc6b4\ub3d9 1\uc8fc\uae30\uc5d0 \ub300\ud55c \uad00\uc2ec\uacfc \ubd84\uc704\uae30\ub97c \ub2e4\ub978 \uacf3\uc73c\ub85c \ub3cc\ub9ac\ub824 \uc2dc\ub3c4\ud588\ub2e4.", "paragraph_id": "6390080-1-2", "paragraph_question": "question: 1981\ub144 \uc804\ub450\ud658\uc774 \ub300\ud1b5\ub839\uc774 \ub41c \uc120\uac70 \ubc29\ubc95\uc740?, context: 1981\ub144 3\uc6d4 3\uc77c \uc5ed\uc2dc \uccb4\uc721\uad00\uc5d0\uc11c\uc758 \uac04\uc811 \uc120\uac70\ub97c \ud1b5\ud574 \uc2a4\uc2a4\ub85c \ub300\ud55c\ubbfc\uad6d\uc758 \uc81c12\ub300 \ub300\ud1b5\ub839\uc5d0 \ucde8\uc784\ud558\uc600\uace0, \uadf8\ud574 5\uc6d4 \uad6d\ud48d 81 \ucd95\uc81c\ub97c \uac1c\ucd5c\ud558\uc5ec \uad11\uc8fc \ubbfc\uc8fc\ud654 \uc6b4\ub3d9 1\uc8fc\uae30\uc5d0 \ub300\ud55c \uad00\uc2ec\uacfc \ubd84\uc704\uae30\ub97c \ub2e4\ub978 \uacf3\uc73c\ub85c \ub3cc\ub9ac\ub824 \uc2dc\ub3c4\ud588\ub2e4. \uc804\ub450\ud658 \uc815\uad8c\uc740 \uc774\ud6c4 1982\ub144 \ud55c\uad6d\ud504\ub85c\uc57c\uad6c\ub97c \ucc3d\uc124\ud558\uace0 \uc57c\uac04\ud1b5\ud589\uae08\uc9c0 \uc870\uce58\ub97c \ud574\uc81c\ud558\uc600\uc73c\uba70, \ud559\uc6d0 \ub450\ubc1c/\ubcf5\uc7a5 \uc790\uc728\ud654 \uc815\ucc45\uc744 \uc2dc\ub3c4\ud558\ub294 \ub4f1 \uad6d\ubbfc\ub4e4\uc758 \ud658\uc2ec\uc744 \uc0ac\uae30 \uc704\ud55c \uc720\ud654\ucc45\uc744 \ub0b4\uc138\uc6e0\ub2e4. \uceec\ub7ec \ud154\ub808\ube44\uc804 \ubcf4\uae09, \ud3ec\ub974\ub178 \uc601\ud654 \uc7a5\ub824\ub85c \ub300\ud45c\ub418\ub294 3S \uc815\ucc45 \ub610\ud55c \uc774\ub7ec\ud55c \uc720\ud654\ucc45\uc758 \uc77c\ud658\uc774\uc5c8\ub2e4. \uadf8\ub7ec\ub098 \ud55c\ud3b8\uc73c\ub85c\ub294 \uc0bc\uccad\uad50\uc721\ub300\ub97c \ucc3d\uc124\ud558\uc5ec \uc720\uc8c4 \ud310\uacb0\uc744 \ub0b4\ub9b0 \ubc94\ubc95\uc790\ub4e4\uc758 \uc778\uad8c\uc744 \uc720\ub9b0\ud558\uc600\uc73c\uba70, \ubbfc\uc8fc\ud654 \uc6b4\ub3d9\uc5d0 \ub300\ud55c \ud0c4\uc555 \ub610\ud55c \uc9c0\uc18d\ud558\uc600\ub2e4."} {"question": "\ubd81\ud55c\uc774 \uc7a0\uc218\ud568\ubc1c\uc0ac\ud0c4\ub3c4\ubbf8\uc0ac\uc77c\uc744 \uc120\ubcf4\uc778 \ud589\uc0ac\ub294?", "paragraph": "\ud575 \ubb3c\uc790 \ubfd0\ub9cc \uc544\ub2c8\ub77c \ubd81\ud55c\uc774 \uc5f4\ubcd1\uc2dd\uc5d0 \uc120\ubcf4\uc778 \uc911\uad6d\uc81c \uc7a0\uc218\ud568\ubc1c\uc0ac\ud0c4\ub3c4\ubbf8\uc0ac\uc77c(SLBM)\uc5d0 \ub300\ud574 \uc911\uad6d \uc815\ubd80\ub294 \uc815\uc0c1\uc801\uc778 \ubb34\uc5ed \uad00\uacc4\uc5d0 \ub530\ub978 \uac83\uc774\ub77c\ub294 \uc8fc\uc7a5\uc744 \ud588\ub2e4. \uadf8\ub7ec\ub098 \uc720\uc5d4 \uc548\ubcf4\ub9ac \uacb0\uc758\uc5d0 \ub530\ub974\uba74 \ubd81\ud55c\uc5d0 \uad70\uc0ac \uc7a5\ube44\ub97c \uc218\ucd9c\ud558\uba74 \uc81c\uc7ac \uc704\ubc18\uc774 \ub41c\ub2e4. \ubd81\ud55c\uc774 \uc3dc \ud0c4\ub3c4\ubbf8\uc0ac\uc77c \uc794\ud574\ub97c \ubbf8\uad6d\uc774 \uc785\uc218\ud574 \ubd84\uc11d\ud55c \uacb0\uacfc, \ubd80\ud488 \uc81c\uc791\uc5d0 \uc911\uad6d\uc774 \uad00\uc5ec\ud55c \uc815\ud669\uc774 \ub4dc\ub7ec\ub0ac\ub2e4. \ubd81\ud55c\uc740 2012\ub144 \uc804\ud6c4\ub85c \ub300\ub959\uac04\ud0c4\ub3c4\ubbf8\uc0ac\uc77c(ICBM) \uc81c\uc870\uc5d0 \ud544\uc694\ud55c \ud575\uc2ec \uae30\uc220\uc744 \uc911\uad6d\uacfc\uc758 \uacf5\ub3d9 \uc5f0\uad6c \ud615\ud0dc\ub97c \ud1b5\ud574\uc11c \ud655\ubcf4\ud588\ub2e4. \uc911\uad6d\uc740 \ubd81\ud55c\uc5d0\uac8c \uac01\uc885 \ud0c4\ub3c4\ubbf8\uc0ac\uc77c\uc744 \uc81c\uacf5\ud574\uc92c\uc73c\uba74\uc11c \ud55c\uad6d\uc5d0\uac8c\ub294 \ubc29\uc5b4\uc801\uc778 \ubb34\uae30\uc870\ucc28\ub3c4 \ubc30\uce58\ud558\uc9c0 \ub9d0\ub77c\uba70 \ubc18\ub300\ud558\uace0 \uc788\ub2e4. \ubd81\ud55c\uc758 \ud575\ud0c4\ub450\uc640 \ud0c4\ub3c4\ubbf8\uc0ac\uc77c\uc740 '\ub85c\ub9e8\uc2a4'\uc774\uace0 \ud55c\uad6d\uc758 \uc0ac\ub4dc\ub294 '\ubd88\ub95c'\uc774\ub77c\ub294 \ub17c\ub9ac\uc778 \uac83\uc774\ub2e4. \ub610\ud55c \uc911\uad6d\uc740 \ubd81\ud55c\uc758 \uc0ac\uac70\ub9ac 4,000 km \ud0c4\ub3c4\ubbf8\uc0ac\uc77c\uc740 \uc804\ud600 \ubc18\ub300\ud558\uc9c0 \uc54a\uc73c\uba74\uc11c \ud55c\uad6d\uc758 \uc0ac\uac70\ub9ac 800 km \ud0c4\ub3c4\ubbf8\uc0ac\uc77c\uc740 \uac15\ub825\ud558\uac8c \ubc18\ub300\ud558\uace0 \uc788\ub2e4. \uc911\uad6d\uc740 \ud55c\ubc18\ub3c4 \ub0a8\ubd81\ud1b5\uc77c\uc744 \uc911\uad6d\uc5d0 \ub300\ud55c \uc704\ud611\uc73c\ub85c \ubcf4\uace0 \uc788\uace0 \ud55c\ubc18\ub3c4 \ub0a8\ubd81\ud1b5\uc77c\uc5d0 \ub300\ud574 \ubd80\uc815\uc801\uc774\ub2e4.", "answer": "\uc5f4\ubcd1\uc2dd", "sentence": "\ud575 \ubb3c\uc790 \ubfd0\ub9cc \uc544\ub2c8\ub77c \ubd81\ud55c\uc774 \uc5f4\ubcd1\uc2dd \uc5d0 \uc120\ubcf4\uc778 \uc911\uad6d\uc81c \uc7a0\uc218\ud568\ubc1c\uc0ac\ud0c4\ub3c4\ubbf8\uc0ac\uc77c(SLBM)\uc5d0 \ub300\ud574 \uc911\uad6d \uc815\ubd80\ub294 \uc815\uc0c1\uc801\uc778 \ubb34\uc5ed \uad00\uacc4\uc5d0 \ub530\ub978 \uac83\uc774\ub77c\ub294 \uc8fc\uc7a5\uc744 \ud588\ub2e4.", "paragraph_sentence": " \ud575 \ubb3c\uc790 \ubfd0\ub9cc \uc544\ub2c8\ub77c \ubd81\ud55c\uc774 \uc5f4\ubcd1\uc2dd \uc5d0 \uc120\ubcf4\uc778 \uc911\uad6d\uc81c \uc7a0\uc218\ud568\ubc1c\uc0ac\ud0c4\ub3c4\ubbf8\uc0ac\uc77c(SLBM)\uc5d0 \ub300\ud574 \uc911\uad6d \uc815\ubd80\ub294 \uc815\uc0c1\uc801\uc778 \ubb34\uc5ed \uad00\uacc4\uc5d0 \ub530\ub978 \uac83\uc774\ub77c\ub294 \uc8fc\uc7a5\uc744 \ud588\ub2e4. \uadf8\ub7ec\ub098 \uc720\uc5d4 \uc548\ubcf4\ub9ac \uacb0\uc758\uc5d0 \ub530\ub974\uba74 \ubd81\ud55c\uc5d0 \uad70\uc0ac \uc7a5\ube44\ub97c \uc218\ucd9c\ud558\uba74 \uc81c\uc7ac \uc704\ubc18\uc774 \ub41c\ub2e4. \ubd81\ud55c\uc774 \uc3dc \ud0c4\ub3c4\ubbf8\uc0ac\uc77c \uc794\ud574\ub97c \ubbf8\uad6d\uc774 \uc785\uc218\ud574 \ubd84\uc11d\ud55c \uacb0\uacfc, \ubd80\ud488 \uc81c\uc791\uc5d0 \uc911\uad6d\uc774 \uad00\uc5ec\ud55c \uc815\ud669\uc774 \ub4dc\ub7ec\ub0ac\ub2e4. \ubd81\ud55c\uc740 2012\ub144 \uc804\ud6c4\ub85c \ub300\ub959\uac04\ud0c4\ub3c4\ubbf8\uc0ac\uc77c(ICBM) \uc81c\uc870\uc5d0 \ud544\uc694\ud55c \ud575\uc2ec \uae30\uc220\uc744 \uc911\uad6d\uacfc\uc758 \uacf5\ub3d9 \uc5f0\uad6c \ud615\ud0dc\ub97c \ud1b5\ud574\uc11c \ud655\ubcf4\ud588\ub2e4. \uc911\uad6d\uc740 \ubd81\ud55c\uc5d0\uac8c \uac01\uc885 \ud0c4\ub3c4\ubbf8\uc0ac\uc77c\uc744 \uc81c\uacf5\ud574\uc92c\uc73c\uba74\uc11c \ud55c\uad6d\uc5d0\uac8c\ub294 \ubc29\uc5b4\uc801\uc778 \ubb34\uae30\uc870\ucc28\ub3c4 \ubc30\uce58\ud558\uc9c0 \ub9d0\ub77c\uba70 \ubc18\ub300\ud558\uace0 \uc788\ub2e4. \ubd81\ud55c\uc758 \ud575\ud0c4\ub450\uc640 \ud0c4\ub3c4\ubbf8\uc0ac\uc77c\uc740 '\ub85c\ub9e8\uc2a4'\uc774\uace0 \ud55c\uad6d\uc758 \uc0ac\ub4dc\ub294 '\ubd88\ub95c'\uc774\ub77c\ub294 \ub17c\ub9ac\uc778 \uac83\uc774\ub2e4. \ub610\ud55c \uc911\uad6d\uc740 \ubd81\ud55c\uc758 \uc0ac\uac70\ub9ac 4,000 km \ud0c4\ub3c4\ubbf8\uc0ac\uc77c\uc740 \uc804\ud600 \ubc18\ub300\ud558\uc9c0 \uc54a\uc73c\uba74\uc11c \ud55c\uad6d\uc758 \uc0ac\uac70\ub9ac 800 km \ud0c4\ub3c4\ubbf8\uc0ac\uc77c\uc740 \uac15\ub825\ud558\uac8c \ubc18\ub300\ud558\uace0 \uc788\ub2e4. \uc911\uad6d\uc740 \ud55c\ubc18\ub3c4 \ub0a8\ubd81\ud1b5\uc77c\uc744 \uc911\uad6d\uc5d0 \ub300\ud55c \uc704\ud611\uc73c\ub85c \ubcf4\uace0 \uc788\uace0 \ud55c\ubc18\ub3c4 \ub0a8\ubd81\ud1b5\uc77c\uc5d0 \ub300\ud574 \ubd80\uc815\uc801\uc774\ub2e4.", "paragraph_answer": "\ud575 \ubb3c\uc790 \ubfd0\ub9cc \uc544\ub2c8\ub77c \ubd81\ud55c\uc774 \uc5f4\ubcd1\uc2dd \uc5d0 \uc120\ubcf4\uc778 \uc911\uad6d\uc81c \uc7a0\uc218\ud568\ubc1c\uc0ac\ud0c4\ub3c4\ubbf8\uc0ac\uc77c(SLBM)\uc5d0 \ub300\ud574 \uc911\uad6d \uc815\ubd80\ub294 \uc815\uc0c1\uc801\uc778 \ubb34\uc5ed \uad00\uacc4\uc5d0 \ub530\ub978 \uac83\uc774\ub77c\ub294 \uc8fc\uc7a5\uc744 \ud588\ub2e4. \uadf8\ub7ec\ub098 \uc720\uc5d4 \uc548\ubcf4\ub9ac \uacb0\uc758\uc5d0 \ub530\ub974\uba74 \ubd81\ud55c\uc5d0 \uad70\uc0ac \uc7a5\ube44\ub97c \uc218\ucd9c\ud558\uba74 \uc81c\uc7ac \uc704\ubc18\uc774 \ub41c\ub2e4. \ubd81\ud55c\uc774 \uc3dc \ud0c4\ub3c4\ubbf8\uc0ac\uc77c \uc794\ud574\ub97c \ubbf8\uad6d\uc774 \uc785\uc218\ud574 \ubd84\uc11d\ud55c \uacb0\uacfc, \ubd80\ud488 \uc81c\uc791\uc5d0 \uc911\uad6d\uc774 \uad00\uc5ec\ud55c \uc815\ud669\uc774 \ub4dc\ub7ec\ub0ac\ub2e4. \ubd81\ud55c\uc740 2012\ub144 \uc804\ud6c4\ub85c \ub300\ub959\uac04\ud0c4\ub3c4\ubbf8\uc0ac\uc77c(ICBM) \uc81c\uc870\uc5d0 \ud544\uc694\ud55c \ud575\uc2ec \uae30\uc220\uc744 \uc911\uad6d\uacfc\uc758 \uacf5\ub3d9 \uc5f0\uad6c \ud615\ud0dc\ub97c \ud1b5\ud574\uc11c \ud655\ubcf4\ud588\ub2e4. \uc911\uad6d\uc740 \ubd81\ud55c\uc5d0\uac8c \uac01\uc885 \ud0c4\ub3c4\ubbf8\uc0ac\uc77c\uc744 \uc81c\uacf5\ud574\uc92c\uc73c\uba74\uc11c \ud55c\uad6d\uc5d0\uac8c\ub294 \ubc29\uc5b4\uc801\uc778 \ubb34\uae30\uc870\ucc28\ub3c4 \ubc30\uce58\ud558\uc9c0 \ub9d0\ub77c\uba70 \ubc18\ub300\ud558\uace0 \uc788\ub2e4. \ubd81\ud55c\uc758 \ud575\ud0c4\ub450\uc640 \ud0c4\ub3c4\ubbf8\uc0ac\uc77c\uc740 '\ub85c\ub9e8\uc2a4'\uc774\uace0 \ud55c\uad6d\uc758 \uc0ac\ub4dc\ub294 '\ubd88\ub95c'\uc774\ub77c\ub294 \ub17c\ub9ac\uc778 \uac83\uc774\ub2e4. \ub610\ud55c \uc911\uad6d\uc740 \ubd81\ud55c\uc758 \uc0ac\uac70\ub9ac 4,000 km \ud0c4\ub3c4\ubbf8\uc0ac\uc77c\uc740 \uc804\ud600 \ubc18\ub300\ud558\uc9c0 \uc54a\uc73c\uba74\uc11c \ud55c\uad6d\uc758 \uc0ac\uac70\ub9ac 800 km \ud0c4\ub3c4\ubbf8\uc0ac\uc77c\uc740 \uac15\ub825\ud558\uac8c \ubc18\ub300\ud558\uace0 \uc788\ub2e4. \uc911\uad6d\uc740 \ud55c\ubc18\ub3c4 \ub0a8\ubd81\ud1b5\uc77c\uc744 \uc911\uad6d\uc5d0 \ub300\ud55c \uc704\ud611\uc73c\ub85c \ubcf4\uace0 \uc788\uace0 \ud55c\ubc18\ub3c4 \ub0a8\ubd81\ud1b5\uc77c\uc5d0 \ub300\ud574 \ubd80\uc815\uc801\uc774\ub2e4.", "sentence_answer": "\ud575 \ubb3c\uc790 \ubfd0\ub9cc \uc544\ub2c8\ub77c \ubd81\ud55c\uc774 \uc5f4\ubcd1\uc2dd \uc5d0 \uc120\ubcf4\uc778 \uc911\uad6d\uc81c \uc7a0\uc218\ud568\ubc1c\uc0ac\ud0c4\ub3c4\ubbf8\uc0ac\uc77c(SLBM)\uc5d0 \ub300\ud574 \uc911\uad6d \uc815\ubd80\ub294 \uc815\uc0c1\uc801\uc778 \ubb34\uc5ed \uad00\uacc4\uc5d0 \ub530\ub978 \uac83\uc774\ub77c\ub294 \uc8fc\uc7a5\uc744 \ud588\ub2e4.", "paragraph_id": "6512357-15-0", "paragraph_question": "question: \ubd81\ud55c\uc774 \uc7a0\uc218\ud568\ubc1c\uc0ac\ud0c4\ub3c4\ubbf8\uc0ac\uc77c\uc744 \uc120\ubcf4\uc778 \ud589\uc0ac\ub294?, context: \ud575 \ubb3c\uc790 \ubfd0\ub9cc \uc544\ub2c8\ub77c \ubd81\ud55c\uc774 \uc5f4\ubcd1\uc2dd\uc5d0 \uc120\ubcf4\uc778 \uc911\uad6d\uc81c \uc7a0\uc218\ud568\ubc1c\uc0ac\ud0c4\ub3c4\ubbf8\uc0ac\uc77c(SLBM)\uc5d0 \ub300\ud574 \uc911\uad6d \uc815\ubd80\ub294 \uc815\uc0c1\uc801\uc778 \ubb34\uc5ed \uad00\uacc4\uc5d0 \ub530\ub978 \uac83\uc774\ub77c\ub294 \uc8fc\uc7a5\uc744 \ud588\ub2e4. \uadf8\ub7ec\ub098 \uc720\uc5d4 \uc548\ubcf4\ub9ac \uacb0\uc758\uc5d0 \ub530\ub974\uba74 \ubd81\ud55c\uc5d0 \uad70\uc0ac \uc7a5\ube44\ub97c \uc218\ucd9c\ud558\uba74 \uc81c\uc7ac \uc704\ubc18\uc774 \ub41c\ub2e4. \ubd81\ud55c\uc774 \uc3dc \ud0c4\ub3c4\ubbf8\uc0ac\uc77c \uc794\ud574\ub97c \ubbf8\uad6d\uc774 \uc785\uc218\ud574 \ubd84\uc11d\ud55c \uacb0\uacfc, \ubd80\ud488 \uc81c\uc791\uc5d0 \uc911\uad6d\uc774 \uad00\uc5ec\ud55c \uc815\ud669\uc774 \ub4dc\ub7ec\ub0ac\ub2e4. \ubd81\ud55c\uc740 2012\ub144 \uc804\ud6c4\ub85c \ub300\ub959\uac04\ud0c4\ub3c4\ubbf8\uc0ac\uc77c(ICBM) \uc81c\uc870\uc5d0 \ud544\uc694\ud55c \ud575\uc2ec \uae30\uc220\uc744 \uc911\uad6d\uacfc\uc758 \uacf5\ub3d9 \uc5f0\uad6c \ud615\ud0dc\ub97c \ud1b5\ud574\uc11c \ud655\ubcf4\ud588\ub2e4. \uc911\uad6d\uc740 \ubd81\ud55c\uc5d0\uac8c \uac01\uc885 \ud0c4\ub3c4\ubbf8\uc0ac\uc77c\uc744 \uc81c\uacf5\ud574\uc92c\uc73c\uba74\uc11c \ud55c\uad6d\uc5d0\uac8c\ub294 \ubc29\uc5b4\uc801\uc778 \ubb34\uae30\uc870\ucc28\ub3c4 \ubc30\uce58\ud558\uc9c0 \ub9d0\ub77c\uba70 \ubc18\ub300\ud558\uace0 \uc788\ub2e4. \ubd81\ud55c\uc758 \ud575\ud0c4\ub450\uc640 \ud0c4\ub3c4\ubbf8\uc0ac\uc77c\uc740 '\ub85c\ub9e8\uc2a4'\uc774\uace0 \ud55c\uad6d\uc758 \uc0ac\ub4dc\ub294 '\ubd88\ub95c'\uc774\ub77c\ub294 \ub17c\ub9ac\uc778 \uac83\uc774\ub2e4. \ub610\ud55c \uc911\uad6d\uc740 \ubd81\ud55c\uc758 \uc0ac\uac70\ub9ac 4,000 km \ud0c4\ub3c4\ubbf8\uc0ac\uc77c\uc740 \uc804\ud600 \ubc18\ub300\ud558\uc9c0 \uc54a\uc73c\uba74\uc11c \ud55c\uad6d\uc758 \uc0ac\uac70\ub9ac 800 km \ud0c4\ub3c4\ubbf8\uc0ac\uc77c\uc740 \uac15\ub825\ud558\uac8c \ubc18\ub300\ud558\uace0 \uc788\ub2e4. \uc911\uad6d\uc740 \ud55c\ubc18\ub3c4 \ub0a8\ubd81\ud1b5\uc77c\uc744 \uc911\uad6d\uc5d0 \ub300\ud55c \uc704\ud611\uc73c\ub85c \ubcf4\uace0 \uc788\uace0 \ud55c\ubc18\ub3c4 \ub0a8\ubd81\ud1b5\uc77c\uc5d0 \ub300\ud574 \ubd80\uc815\uc801\uc774\ub2e4."} {"question": "\ubc15\uc7ac\uc601\uc774 10-11 \uc2a4\ud0c0\ub9ac\uadf8\uc5d0\uc11c \uccab \uc2b9\uc744 \uac70\ub454 \uc120\uc218\ub294?", "paragraph": "10-11 \ud504\ub85c\ub9ac\uadf8\uc5d0\uc11c\ub294 \uc2b9\ub9ac \uc5c6\uc774 7\uc5f0\ud328\ub77c\ub294 \uadf9\ub3c4\uc758 \ubd80\uc9c4\uc5d0 \ube60\uc84c\ub2e4. 2011\ub144 1\uc6d4 8\uc77c\uc5d0\ub294 \uc0c1\ub300\uc804\uc801\uc5d0\uc11c \uc6b0\uc138\ud55c \uc815\uba85\ud6c8\uc744 \ud504\ub85c\ud1a0\uc2a4\uac00 \ud14c\ub780\uc5d0\uac8c \uc6b0\uc138\ud55c \ub9f5 \uc544\uc988\ud14d\uc5d0\uc11c \uc0c1\ub300\ud588\uc73c\ub098 \uc5f0\ud328\ub97c \ub04a\uc5b4\ub0b4\uc9c0 \ubabb\ud588\ub2e4. 5\uc6d4 23\uc77c \ub9c8\ucc2c\uac00\uc9c0\ub85c \uc2b9\ub9ac\uac00 \uc5c6\ub294 \uc624\uc138\uae30\uc640 \ub9de\ubd99\uc5b4 \uc2b9\uae30\ub97c \uc7a1\uc558\uc73c\ub098 \uc624\uc138\uae30\uc758 \ub4dc\ub86d\uc5d0 \ubcf8\uc9c4 \ub125\uc11c\uc2a4\uac00 \ud30c\uad34\ub2f9\ud588\uc73c\uba70 \uc790\uc2e0\uc740 \uc624\uc138\uae30\uac00 \uacf5\uc911\uc73c\ub85c \ub744\uc6cc \ud53c\uc2e0\uc2dc\ud0a8 \uac74\ubb3c\uc744 \ud30c\uad34\ud560 \uc218 \uc5c6\uc5b4 \uc2dc\uc98c \uccab \ubb34\uc2b9\ubd80\ub97c \uae30\ub85d\ud558\ub294 \ud574\ud504\ub2dd\uc744 \uacaa\uc5c8\uace0, \uc7ac\uacbd\uae30\uc5d0\uc11c\ub294 \uc624\uc138\uae30\ub97c \uc18d\uc5ec\ub0b4\uba70 \ub4dc\ub514\uc5b4 \uccab \uc2b9\ub9ac\ub97c \uac70\ub450\uc5c8\ub294\ub370 \uacf5\uaca9\ub825\u00b7\ubc29\uc5b4\ub825\u00b7\uc2e4\ub4dc \uc5c5\uadf8\ub808\uc774\ub4dc\uac00 \uc804\ubd80 \ub41c \uce90\ub9ac\uc5b4\ub77c\ub294 \uc9c4\uae30\ud55c \uc138\ub9ac\uba38\ub2c8\ub97c \ubcf4\uc5ec\uc92c\ub2e4. \uc774 \uc2b9\ub9ac \ud55c \ub2ec \uc804\ubd80\ud130 \ucef4\ud4e8\ud130\uc5d0\uc11c \uc2a4\ud0c0\ud06c\ub798\ud504\ud2b8\ub97c \uc0ad\uc81c\ud558\uace0 \uac8c\uc784\uc5d0\ub9cc \uc9d1\uc911\ud558\ub294 \uc0dd\ud65c\uc744 \ud558\uace0 \uc788\uc5c8\ub2e4.", "answer": "\uc624\uc138\uae30", "sentence": "5\uc6d4 23\uc77c \ub9c8\ucc2c\uac00\uc9c0\ub85c \uc2b9\ub9ac\uac00 \uc5c6\ub294 \uc624\uc138\uae30 \uc640 \ub9de\ubd99\uc5b4 \uc2b9\uae30\ub97c \uc7a1\uc558\uc73c\ub098 \uc624\uc138\uae30\uc758 \ub4dc\ub86d\uc5d0 \ubcf8\uc9c4 \ub125\uc11c\uc2a4\uac00 \ud30c\uad34\ub2f9\ud588\uc73c\uba70 \uc790\uc2e0\uc740 \uc624\uc138\uae30\uac00 \uacf5\uc911\uc73c\ub85c \ub744\uc6cc \ud53c\uc2e0\uc2dc\ud0a8 \uac74\ubb3c\uc744 \ud30c\uad34\ud560 \uc218 \uc5c6\uc5b4 \uc2dc\uc98c \uccab \ubb34\uc2b9\ubd80\ub97c \uae30\ub85d\ud558\ub294 \ud574\ud504\ub2dd\uc744 \uacaa\uc5c8\uace0, \uc7ac\uacbd\uae30\uc5d0\uc11c\ub294 \uc624\uc138\uae30\ub97c \uc18d\uc5ec\ub0b4\uba70 \ub4dc\ub514\uc5b4 \uccab \uc2b9\ub9ac\ub97c \uac70\ub450\uc5c8\ub294\ub370 \uacf5\uaca9\ub825\u00b7\ubc29\uc5b4\ub825\u00b7\uc2e4\ub4dc \uc5c5\uadf8\ub808\uc774\ub4dc\uac00 \uc804\ubd80 \ub41c \uce90\ub9ac\uc5b4\ub77c\ub294 \uc9c4\uae30\ud55c \uc138\ub9ac\uba38\ub2c8\ub97c \ubcf4\uc5ec\uc92c\ub2e4.", "paragraph_sentence": "10-11 \ud504\ub85c\ub9ac\uadf8\uc5d0\uc11c\ub294 \uc2b9\ub9ac \uc5c6\uc774 7\uc5f0\ud328\ub77c\ub294 \uadf9\ub3c4\uc758 \ubd80\uc9c4\uc5d0 \ube60\uc84c\ub2e4. 2011\ub144 1\uc6d4 8\uc77c\uc5d0\ub294 \uc0c1\ub300\uc804\uc801\uc5d0\uc11c \uc6b0\uc138\ud55c \uc815\uba85\ud6c8\uc744 \ud504\ub85c\ud1a0\uc2a4\uac00 \ud14c\ub780\uc5d0\uac8c \uc6b0\uc138\ud55c \ub9f5 \uc544\uc988\ud14d\uc5d0\uc11c \uc0c1\ub300\ud588\uc73c\ub098 \uc5f0\ud328\ub97c \ub04a\uc5b4\ub0b4\uc9c0 \ubabb\ud588\ub2e4. 5\uc6d4 23\uc77c \ub9c8\ucc2c\uac00\uc9c0\ub85c \uc2b9\ub9ac\uac00 \uc5c6\ub294 \uc624\uc138\uae30 \uc640 \ub9de\ubd99\uc5b4 \uc2b9\uae30\ub97c \uc7a1\uc558\uc73c\ub098 \uc624\uc138\uae30\uc758 \ub4dc\ub86d\uc5d0 \ubcf8\uc9c4 \ub125\uc11c\uc2a4\uac00 \ud30c\uad34\ub2f9\ud588\uc73c\uba70 \uc790\uc2e0\uc740 \uc624\uc138\uae30\uac00 \uacf5\uc911\uc73c\ub85c \ub744\uc6cc \ud53c\uc2e0\uc2dc\ud0a8 \uac74\ubb3c\uc744 \ud30c\uad34\ud560 \uc218 \uc5c6\uc5b4 \uc2dc\uc98c \uccab \ubb34\uc2b9\ubd80\ub97c \uae30\ub85d\ud558\ub294 \ud574\ud504\ub2dd\uc744 \uacaa\uc5c8\uace0, \uc7ac\uacbd\uae30\uc5d0\uc11c\ub294 \uc624\uc138\uae30\ub97c \uc18d\uc5ec\ub0b4\uba70 \ub4dc\ub514\uc5b4 \uccab \uc2b9\ub9ac\ub97c \uac70\ub450\uc5c8\ub294\ub370 \uacf5\uaca9\ub825\u00b7\ubc29\uc5b4\ub825\u00b7\uc2e4\ub4dc \uc5c5\uadf8\ub808\uc774\ub4dc\uac00 \uc804\ubd80 \ub41c \uce90\ub9ac\uc5b4\ub77c\ub294 \uc9c4\uae30\ud55c \uc138\ub9ac\uba38\ub2c8\ub97c \ubcf4\uc5ec\uc92c\ub2e4. \uc774 \uc2b9\ub9ac \ud55c \ub2ec \uc804\ubd80\ud130 \ucef4\ud4e8\ud130\uc5d0\uc11c \uc2a4\ud0c0\ud06c\ub798\ud504\ud2b8\ub97c \uc0ad\uc81c\ud558\uace0 \uac8c\uc784\uc5d0\ub9cc \uc9d1\uc911\ud558\ub294 \uc0dd\ud65c\uc744 \ud558\uace0 \uc788\uc5c8\ub2e4.", "paragraph_answer": "10-11 \ud504\ub85c\ub9ac\uadf8\uc5d0\uc11c\ub294 \uc2b9\ub9ac \uc5c6\uc774 7\uc5f0\ud328\ub77c\ub294 \uadf9\ub3c4\uc758 \ubd80\uc9c4\uc5d0 \ube60\uc84c\ub2e4. 2011\ub144 1\uc6d4 8\uc77c\uc5d0\ub294 \uc0c1\ub300\uc804\uc801\uc5d0\uc11c \uc6b0\uc138\ud55c \uc815\uba85\ud6c8\uc744 \ud504\ub85c\ud1a0\uc2a4\uac00 \ud14c\ub780\uc5d0\uac8c \uc6b0\uc138\ud55c \ub9f5 \uc544\uc988\ud14d\uc5d0\uc11c \uc0c1\ub300\ud588\uc73c\ub098 \uc5f0\ud328\ub97c \ub04a\uc5b4\ub0b4\uc9c0 \ubabb\ud588\ub2e4. 5\uc6d4 23\uc77c \ub9c8\ucc2c\uac00\uc9c0\ub85c \uc2b9\ub9ac\uac00 \uc5c6\ub294 \uc624\uc138\uae30 \uc640 \ub9de\ubd99\uc5b4 \uc2b9\uae30\ub97c \uc7a1\uc558\uc73c\ub098 \uc624\uc138\uae30\uc758 \ub4dc\ub86d\uc5d0 \ubcf8\uc9c4 \ub125\uc11c\uc2a4\uac00 \ud30c\uad34\ub2f9\ud588\uc73c\uba70 \uc790\uc2e0\uc740 \uc624\uc138\uae30\uac00 \uacf5\uc911\uc73c\ub85c \ub744\uc6cc \ud53c\uc2e0\uc2dc\ud0a8 \uac74\ubb3c\uc744 \ud30c\uad34\ud560 \uc218 \uc5c6\uc5b4 \uc2dc\uc98c \uccab \ubb34\uc2b9\ubd80\ub97c \uae30\ub85d\ud558\ub294 \ud574\ud504\ub2dd\uc744 \uacaa\uc5c8\uace0, \uc7ac\uacbd\uae30\uc5d0\uc11c\ub294 \uc624\uc138\uae30\ub97c \uc18d\uc5ec\ub0b4\uba70 \ub4dc\ub514\uc5b4 \uccab \uc2b9\ub9ac\ub97c \uac70\ub450\uc5c8\ub294\ub370 \uacf5\uaca9\ub825\u00b7\ubc29\uc5b4\ub825\u00b7\uc2e4\ub4dc \uc5c5\uadf8\ub808\uc774\ub4dc\uac00 \uc804\ubd80 \ub41c \uce90\ub9ac\uc5b4\ub77c\ub294 \uc9c4\uae30\ud55c \uc138\ub9ac\uba38\ub2c8\ub97c \ubcf4\uc5ec\uc92c\ub2e4. \uc774 \uc2b9\ub9ac \ud55c \ub2ec \uc804\ubd80\ud130 \ucef4\ud4e8\ud130\uc5d0\uc11c \uc2a4\ud0c0\ud06c\ub798\ud504\ud2b8\ub97c \uc0ad\uc81c\ud558\uace0 \uac8c\uc784\uc5d0\ub9cc \uc9d1\uc911\ud558\ub294 \uc0dd\ud65c\uc744 \ud558\uace0 \uc788\uc5c8\ub2e4.", "sentence_answer": "5\uc6d4 23\uc77c \ub9c8\ucc2c\uac00\uc9c0\ub85c \uc2b9\ub9ac\uac00 \uc5c6\ub294 \uc624\uc138\uae30 \uc640 \ub9de\ubd99\uc5b4 \uc2b9\uae30\ub97c \uc7a1\uc558\uc73c\ub098 \uc624\uc138\uae30\uc758 \ub4dc\ub86d\uc5d0 \ubcf8\uc9c4 \ub125\uc11c\uc2a4\uac00 \ud30c\uad34\ub2f9\ud588\uc73c\uba70 \uc790\uc2e0\uc740 \uc624\uc138\uae30\uac00 \uacf5\uc911\uc73c\ub85c \ub744\uc6cc \ud53c\uc2e0\uc2dc\ud0a8 \uac74\ubb3c\uc744 \ud30c\uad34\ud560 \uc218 \uc5c6\uc5b4 \uc2dc\uc98c \uccab \ubb34\uc2b9\ubd80\ub97c \uae30\ub85d\ud558\ub294 \ud574\ud504\ub2dd\uc744 \uacaa\uc5c8\uace0, \uc7ac\uacbd\uae30\uc5d0\uc11c\ub294 \uc624\uc138\uae30\ub97c \uc18d\uc5ec\ub0b4\uba70 \ub4dc\ub514\uc5b4 \uccab \uc2b9\ub9ac\ub97c \uac70\ub450\uc5c8\ub294\ub370 \uacf5\uaca9\ub825\u00b7\ubc29\uc5b4\ub825\u00b7\uc2e4\ub4dc \uc5c5\uadf8\ub808\uc774\ub4dc\uac00 \uc804\ubd80 \ub41c \uce90\ub9ac\uc5b4\ub77c\ub294 \uc9c4\uae30\ud55c \uc138\ub9ac\uba38\ub2c8\ub97c \ubcf4\uc5ec\uc92c\ub2e4.", "paragraph_id": "6507645-0-1", "paragraph_question": "question: \ubc15\uc7ac\uc601\uc774 10-11 \uc2a4\ud0c0\ub9ac\uadf8\uc5d0\uc11c \uccab \uc2b9\uc744 \uac70\ub454 \uc120\uc218\ub294?, context: 10-11 \ud504\ub85c\ub9ac\uadf8\uc5d0\uc11c\ub294 \uc2b9\ub9ac \uc5c6\uc774 7\uc5f0\ud328\ub77c\ub294 \uadf9\ub3c4\uc758 \ubd80\uc9c4\uc5d0 \ube60\uc84c\ub2e4. 2011\ub144 1\uc6d4 8\uc77c\uc5d0\ub294 \uc0c1\ub300\uc804\uc801\uc5d0\uc11c \uc6b0\uc138\ud55c \uc815\uba85\ud6c8\uc744 \ud504\ub85c\ud1a0\uc2a4\uac00 \ud14c\ub780\uc5d0\uac8c \uc6b0\uc138\ud55c \ub9f5 \uc544\uc988\ud14d\uc5d0\uc11c \uc0c1\ub300\ud588\uc73c\ub098 \uc5f0\ud328\ub97c \ub04a\uc5b4\ub0b4\uc9c0 \ubabb\ud588\ub2e4. 5\uc6d4 23\uc77c \ub9c8\ucc2c\uac00\uc9c0\ub85c \uc2b9\ub9ac\uac00 \uc5c6\ub294 \uc624\uc138\uae30\uc640 \ub9de\ubd99\uc5b4 \uc2b9\uae30\ub97c \uc7a1\uc558\uc73c\ub098 \uc624\uc138\uae30\uc758 \ub4dc\ub86d\uc5d0 \ubcf8\uc9c4 \ub125\uc11c\uc2a4\uac00 \ud30c\uad34\ub2f9\ud588\uc73c\uba70 \uc790\uc2e0\uc740 \uc624\uc138\uae30\uac00 \uacf5\uc911\uc73c\ub85c \ub744\uc6cc \ud53c\uc2e0\uc2dc\ud0a8 \uac74\ubb3c\uc744 \ud30c\uad34\ud560 \uc218 \uc5c6\uc5b4 \uc2dc\uc98c \uccab \ubb34\uc2b9\ubd80\ub97c \uae30\ub85d\ud558\ub294 \ud574\ud504\ub2dd\uc744 \uacaa\uc5c8\uace0, \uc7ac\uacbd\uae30\uc5d0\uc11c\ub294 \uc624\uc138\uae30\ub97c \uc18d\uc5ec\ub0b4\uba70 \ub4dc\ub514\uc5b4 \uccab \uc2b9\ub9ac\ub97c \uac70\ub450\uc5c8\ub294\ub370 \uacf5\uaca9\ub825\u00b7\ubc29\uc5b4\ub825\u00b7\uc2e4\ub4dc \uc5c5\uadf8\ub808\uc774\ub4dc\uac00 \uc804\ubd80 \ub41c \uce90\ub9ac\uc5b4\ub77c\ub294 \uc9c4\uae30\ud55c \uc138\ub9ac\uba38\ub2c8\ub97c \ubcf4\uc5ec\uc92c\ub2e4. \uc774 \uc2b9\ub9ac \ud55c \ub2ec \uc804\ubd80\ud130 \ucef4\ud4e8\ud130\uc5d0\uc11c \uc2a4\ud0c0\ud06c\ub798\ud504\ud2b8\ub97c \uc0ad\uc81c\ud558\uace0 \uac8c\uc784\uc5d0\ub9cc \uc9d1\uc911\ud558\ub294 \uc0dd\ud65c\uc744 \ud558\uace0 \uc788\uc5c8\ub2e4."} {"question": "\ucd08\ub4f1\ud559\uad50\ub97c \ub2e4 \ub9c8\uce58\uc9c0 \ubabb\ud55c \uc804\ud0dc\uc77c\uc774 \uc785\ud559\ud55c \ud559\uad50\ub294?", "paragraph": "\uc11c\uc6b8\uc5d0\uc11c \ub2e4\uc2dc \uc774\ub9ac \uc800\ub9ac \ub5a0\ub3c4\ub294 \uc0dd\ud65c\uc744 \ud558\ub358 \uac00\uc871\uc740 \uc2dc\ub3d9\uc0dd\uc758 \uad8c\uc720\uc5d0 \ubabb\uc774\uaca8 \ub300\uad6c\uc758 \uc2dc\ub301\uc73c\ub85c \ub0b4\ub824\uac14\ub2e4. \uc804\ud0dc\uc77c\uc740 \ubd80\uc0b0\uae4c\uc9c0 \ub5a0\ub3cc\ub2e4 \ube44\uca4d \ub9c8\ub978 \ubaa8\uc2b5\uc73c\ub85c \ub300\uad6c\uc5d0 \uc654\ub2e4. \ub300\uad6c\uc5d0\uc11c \ub2e4\uc2dc \ubaa8\uc778 \uac00\uc871\uc740 \ub2e4\uc2dc \uc7ac\ubd09\uc77c\uc744 \ud558\uba70 \uc0b4\uc558\uace0 \uc815\uc2dd\uc73c\ub85c \ucd08\ub4f1\ud559\uad50 \uc878\uc5c5\uc744 \ud558\uc9c0 \ubabb\ud55c \uc804\ud0dc\uc77c\uc740 \uccad\uc625\uace0\ub4f1\uacf5\ubbfc\ud559\uad50\uc5d0 \uc785\ud559\ud558\uc600\ub2e4. \uc804\ud0dc\uc77c\uc740 \uc2a4\uc2a4\ub85c \uc4f4 \uc218\uae30\uc5d0\uc11c \uc774 \ud559\uad50\uc5d0 \ub2e4\ub2c8\ub358 \uc2dc\uc808\uc744 \uac00\uc7a5 \ud589\ubcf5\ud588\ub358 \uc2dc\uae30\ub85c \uae30\uc5b5\ud588\ub2e4. \uadf8\ub7ec\ub098 \uc804\uc0c1\uc625\uc758 \uc0ac\uc5c5\uc740 \uacb0\uad6d \ub9dd\ud588\uace0 \ub2e4\uc2dc \uc220\uc5d0 \uc758\uc9c0\ud558\uac8c \ub418\uc5c8\ub2e4. \uc774\uc18c\uc120\uc740 \uc220\uc5d0 \ucde8\ud574 \uc544\uc774\ub4e4\uc744 \ub54c\ub9ac\ub294 \uc804\uc0c1\uc625\uc744 \ubcf4\uba70 \uc808\ub9dd\uc744 \ub290\uaf08\ub2e4. \uc804\uc0c1\uc625\uc774 \uc9d1\uc744 \ub098\uac00 \ubc84\ub9ac\uc790 \ub0a8\uc740 \uac00\uc871\ub4e4\uc740 \uaf3c\uc9dd\uc5c6\uc774 \uad76\uac8c \ub418\uc5c8\ub2e4. 1964\ub144 \uc774\uc18c\uc120\uc740 \uc77c\uc744 \uad6c\ud558\uae30 \uc704\ud574 \ud640\ub85c \uc11c\uc6b8\ub85c \uc0c1\uacbd\ud558\uc600\ub2e4.", "answer": "\uccad\uc625\uace0\ub4f1\uacf5\ubbfc\ud559\uad50", "sentence": "\ub300\uad6c\uc5d0\uc11c \ub2e4\uc2dc \ubaa8\uc778 \uac00\uc871\uc740 \ub2e4\uc2dc \uc7ac\ubd09\uc77c\uc744 \ud558\uba70 \uc0b4\uc558\uace0 \uc815\uc2dd\uc73c\ub85c \ucd08\ub4f1\ud559\uad50 \uc878\uc5c5\uc744 \ud558\uc9c0 \ubabb\ud55c \uc804\ud0dc\uc77c\uc740 \uccad\uc625\uace0\ub4f1\uacf5\ubbfc\ud559\uad50 \uc5d0 \uc785\ud559\ud558\uc600\ub2e4.", "paragraph_sentence": "\uc11c\uc6b8\uc5d0\uc11c \ub2e4\uc2dc \uc774\ub9ac \uc800\ub9ac \ub5a0\ub3c4\ub294 \uc0dd\ud65c\uc744 \ud558\ub358 \uac00\uc871\uc740 \uc2dc\ub3d9\uc0dd\uc758 \uad8c\uc720\uc5d0 \ubabb\uc774\uaca8 \ub300\uad6c\uc758 \uc2dc\ub301\uc73c\ub85c \ub0b4\ub824\uac14\ub2e4. \uc804\ud0dc\uc77c\uc740 \ubd80\uc0b0\uae4c\uc9c0 \ub5a0\ub3cc\ub2e4 \ube44\uca4d \ub9c8\ub978 \ubaa8\uc2b5\uc73c\ub85c \ub300\uad6c\uc5d0 \uc654\ub2e4. \ub300\uad6c\uc5d0\uc11c \ub2e4\uc2dc \ubaa8\uc778 \uac00\uc871\uc740 \ub2e4\uc2dc \uc7ac\ubd09\uc77c\uc744 \ud558\uba70 \uc0b4\uc558\uace0 \uc815\uc2dd\uc73c\ub85c \ucd08\ub4f1\ud559\uad50 \uc878\uc5c5\uc744 \ud558\uc9c0 \ubabb\ud55c \uc804\ud0dc\uc77c\uc740 \uccad\uc625\uace0\ub4f1\uacf5\ubbfc\ud559\uad50 \uc5d0 \uc785\ud559\ud558\uc600\ub2e4. \uc804\ud0dc\uc77c\uc740 \uc2a4\uc2a4\ub85c \uc4f4 \uc218\uae30\uc5d0\uc11c \uc774 \ud559\uad50\uc5d0 \ub2e4\ub2c8\ub358 \uc2dc\uc808\uc744 \uac00\uc7a5 \ud589\ubcf5\ud588\ub358 \uc2dc\uae30\ub85c \uae30\uc5b5\ud588\ub2e4. \uadf8\ub7ec\ub098 \uc804\uc0c1\uc625\uc758 \uc0ac\uc5c5\uc740 \uacb0\uad6d \ub9dd\ud588\uace0 \ub2e4\uc2dc \uc220\uc5d0 \uc758\uc9c0\ud558\uac8c \ub418\uc5c8\ub2e4. \uc774\uc18c\uc120\uc740 \uc220\uc5d0 \ucde8\ud574 \uc544\uc774\ub4e4\uc744 \ub54c\ub9ac\ub294 \uc804\uc0c1\uc625\uc744 \ubcf4\uba70 \uc808\ub9dd\uc744 \ub290\uaf08\ub2e4. \uc804\uc0c1\uc625\uc774 \uc9d1\uc744 \ub098\uac00 \ubc84\ub9ac\uc790 \ub0a8\uc740 \uac00\uc871\ub4e4\uc740 \uaf3c\uc9dd\uc5c6\uc774 \uad76\uac8c \ub418\uc5c8\ub2e4. 1964\ub144 \uc774\uc18c\uc120\uc740 \uc77c\uc744 \uad6c\ud558\uae30 \uc704\ud574 \ud640\ub85c \uc11c\uc6b8\ub85c \uc0c1\uacbd\ud558\uc600\ub2e4.", "paragraph_answer": "\uc11c\uc6b8\uc5d0\uc11c \ub2e4\uc2dc \uc774\ub9ac \uc800\ub9ac \ub5a0\ub3c4\ub294 \uc0dd\ud65c\uc744 \ud558\ub358 \uac00\uc871\uc740 \uc2dc\ub3d9\uc0dd\uc758 \uad8c\uc720\uc5d0 \ubabb\uc774\uaca8 \ub300\uad6c\uc758 \uc2dc\ub301\uc73c\ub85c \ub0b4\ub824\uac14\ub2e4. \uc804\ud0dc\uc77c\uc740 \ubd80\uc0b0\uae4c\uc9c0 \ub5a0\ub3cc\ub2e4 \ube44\uca4d \ub9c8\ub978 \ubaa8\uc2b5\uc73c\ub85c \ub300\uad6c\uc5d0 \uc654\ub2e4. \ub300\uad6c\uc5d0\uc11c \ub2e4\uc2dc \ubaa8\uc778 \uac00\uc871\uc740 \ub2e4\uc2dc \uc7ac\ubd09\uc77c\uc744 \ud558\uba70 \uc0b4\uc558\uace0 \uc815\uc2dd\uc73c\ub85c \ucd08\ub4f1\ud559\uad50 \uc878\uc5c5\uc744 \ud558\uc9c0 \ubabb\ud55c \uc804\ud0dc\uc77c\uc740 \uccad\uc625\uace0\ub4f1\uacf5\ubbfc\ud559\uad50 \uc5d0 \uc785\ud559\ud558\uc600\ub2e4. \uc804\ud0dc\uc77c\uc740 \uc2a4\uc2a4\ub85c \uc4f4 \uc218\uae30\uc5d0\uc11c \uc774 \ud559\uad50\uc5d0 \ub2e4\ub2c8\ub358 \uc2dc\uc808\uc744 \uac00\uc7a5 \ud589\ubcf5\ud588\ub358 \uc2dc\uae30\ub85c \uae30\uc5b5\ud588\ub2e4. \uadf8\ub7ec\ub098 \uc804\uc0c1\uc625\uc758 \uc0ac\uc5c5\uc740 \uacb0\uad6d \ub9dd\ud588\uace0 \ub2e4\uc2dc \uc220\uc5d0 \uc758\uc9c0\ud558\uac8c \ub418\uc5c8\ub2e4. \uc774\uc18c\uc120\uc740 \uc220\uc5d0 \ucde8\ud574 \uc544\uc774\ub4e4\uc744 \ub54c\ub9ac\ub294 \uc804\uc0c1\uc625\uc744 \ubcf4\uba70 \uc808\ub9dd\uc744 \ub290\uaf08\ub2e4. \uc804\uc0c1\uc625\uc774 \uc9d1\uc744 \ub098\uac00 \ubc84\ub9ac\uc790 \ub0a8\uc740 \uac00\uc871\ub4e4\uc740 \uaf3c\uc9dd\uc5c6\uc774 \uad76\uac8c \ub418\uc5c8\ub2e4. 1964\ub144 \uc774\uc18c\uc120\uc740 \uc77c\uc744 \uad6c\ud558\uae30 \uc704\ud574 \ud640\ub85c \uc11c\uc6b8\ub85c \uc0c1\uacbd\ud558\uc600\ub2e4.", "sentence_answer": "\ub300\uad6c\uc5d0\uc11c \ub2e4\uc2dc \ubaa8\uc778 \uac00\uc871\uc740 \ub2e4\uc2dc \uc7ac\ubd09\uc77c\uc744 \ud558\uba70 \uc0b4\uc558\uace0 \uc815\uc2dd\uc73c\ub85c \ucd08\ub4f1\ud559\uad50 \uc878\uc5c5\uc744 \ud558\uc9c0 \ubabb\ud55c \uc804\ud0dc\uc77c\uc740 \uccad\uc625\uace0\ub4f1\uacf5\ubbfc\ud559\uad50 \uc5d0 \uc785\ud559\ud558\uc600\ub2e4.", "paragraph_id": "6464150-2-0", "paragraph_question": "question: \ucd08\ub4f1\ud559\uad50\ub97c \ub2e4 \ub9c8\uce58\uc9c0 \ubabb\ud55c \uc804\ud0dc\uc77c\uc774 \uc785\ud559\ud55c \ud559\uad50\ub294?, context: \uc11c\uc6b8\uc5d0\uc11c \ub2e4\uc2dc \uc774\ub9ac \uc800\ub9ac \ub5a0\ub3c4\ub294 \uc0dd\ud65c\uc744 \ud558\ub358 \uac00\uc871\uc740 \uc2dc\ub3d9\uc0dd\uc758 \uad8c\uc720\uc5d0 \ubabb\uc774\uaca8 \ub300\uad6c\uc758 \uc2dc\ub301\uc73c\ub85c \ub0b4\ub824\uac14\ub2e4. \uc804\ud0dc\uc77c\uc740 \ubd80\uc0b0\uae4c\uc9c0 \ub5a0\ub3cc\ub2e4 \ube44\uca4d \ub9c8\ub978 \ubaa8\uc2b5\uc73c\ub85c \ub300\uad6c\uc5d0 \uc654\ub2e4. \ub300\uad6c\uc5d0\uc11c \ub2e4\uc2dc \ubaa8\uc778 \uac00\uc871\uc740 \ub2e4\uc2dc \uc7ac\ubd09\uc77c\uc744 \ud558\uba70 \uc0b4\uc558\uace0 \uc815\uc2dd\uc73c\ub85c \ucd08\ub4f1\ud559\uad50 \uc878\uc5c5\uc744 \ud558\uc9c0 \ubabb\ud55c \uc804\ud0dc\uc77c\uc740 \uccad\uc625\uace0\ub4f1\uacf5\ubbfc\ud559\uad50\uc5d0 \uc785\ud559\ud558\uc600\ub2e4. \uc804\ud0dc\uc77c\uc740 \uc2a4\uc2a4\ub85c \uc4f4 \uc218\uae30\uc5d0\uc11c \uc774 \ud559\uad50\uc5d0 \ub2e4\ub2c8\ub358 \uc2dc\uc808\uc744 \uac00\uc7a5 \ud589\ubcf5\ud588\ub358 \uc2dc\uae30\ub85c \uae30\uc5b5\ud588\ub2e4. \uadf8\ub7ec\ub098 \uc804\uc0c1\uc625\uc758 \uc0ac\uc5c5\uc740 \uacb0\uad6d \ub9dd\ud588\uace0 \ub2e4\uc2dc \uc220\uc5d0 \uc758\uc9c0\ud558\uac8c \ub418\uc5c8\ub2e4. \uc774\uc18c\uc120\uc740 \uc220\uc5d0 \ucde8\ud574 \uc544\uc774\ub4e4\uc744 \ub54c\ub9ac\ub294 \uc804\uc0c1\uc625\uc744 \ubcf4\uba70 \uc808\ub9dd\uc744 \ub290\uaf08\ub2e4. \uc804\uc0c1\uc625\uc774 \uc9d1\uc744 \ub098\uac00 \ubc84\ub9ac\uc790 \ub0a8\uc740 \uac00\uc871\ub4e4\uc740 \uaf3c\uc9dd\uc5c6\uc774 \uad76\uac8c \ub418\uc5c8\ub2e4. 1964\ub144 \uc774\uc18c\uc120\uc740 \uc77c\uc744 \uad6c\ud558\uae30 \uc704\ud574 \ud640\ub85c \uc11c\uc6b8\ub85c \uc0c1\uacbd\ud558\uc600\ub2e4."} {"question": "\uc11c\uc6b8\uc804\ucc28 \uc6b4\ud589\uc5d0 \uad00\ud55c \ud611\uc0c1 \ub09c\ud669\uc73c\ub85c \uc6b4\ud589\uc774 \uc911\uc9c0\ub41c \uae30\uac04\uc740?", "paragraph": "\uadf8\ub7ec\ub098 \uac1c\ud1b5 \ud6c4 10\uc77c\uc9f8 \ub418\ub294 \ub0a0 \uc885\ub85c2\uac00\ucbe4\uc5d0\uc11c \ub2e4\uc12f \uc0b4\ub41c \uc544\uc774\uac00 \uc804\ucc28\uac00 \uce58\uc5b4 \uc8fd\ub294 \uc77c\uc774 \ubc8c\uc5b4\uc84c\ub2e4. \uc544\uc774\uc758 \uc544\ubc84\uc9c0\uac00 \uc774\ub97c \ubcf4\uace0 \ub3c4\ub07c\ub97c \ube7c\ub4e4\uace0 \uc804\ucc28\uc5d0 \ub2ec\ub824\ub4e4\uc790, \uc2dc\ubbfc\ub4e4\uc774 \uc5ec\uae30\uc5d0 \ud569\uc138\ud558\uc5ec \ucc28\ub7c9\uc5d0 \ub2ec\ub824\ub4e4\uc5c8\ub2e4. \ub2f9\uc2dc \uc2dc\ub0b4\uc5d0 \uc804\ucc28\ub85c \uc778\ud574 \uac00\ubb44\uc774 \uc77c\uc5b4\ub09c\ub2e4\ub294 \ub465 \uc720\uc5b8\ube44\uc5b4\uac00 \uc720\ud3ec\ub418\uc5c8\ub294\ub370, \uc774\ub85c \uc778\ud574 \uc2dc\ubbfc\ub4e4\uc758 \ubc18\uc751\uc774 \uacfc\uaca9\ud574\uc9c4 \uac83\uc774 \uc544\ub2cc\uac00 \uc0dd\uac01\ub41c\ub2e4. \ucc28\uc7a5\uacfc \uc6b4\uc804\uc218\ub294 \uc694\ud589\uc774 \ube60\uc838\ub098\uc62c \uc218 \uc788\uc5c8\uc73c\ub098, \uc804\ucc28\ub294 \ud30c\uad34, \uc804\uc18c\ub418\uc5c8\ub2e4. \uc774\ud6c4 \ub2e4\ub978 \uc804\ucc28 \uc5ed\uc2dc \uc218\ub09c\uc744 \ub2f9\ud558\uace0, \ubc1c\uc804\uc18c\uae4c\uc9c0 \uc704\ud611\ub2f9\ud558\ub294 \uc0c1\ud669\uc774 \ubc1c\uc0dd\ud558\uc600\ub2e4. \uc18c\ub3d9\uc740 \uace7 \uc9c4\uc815\ub418\uace0, \uc774\ub7ec\ud55c \ud3ed\ub3d9 \ubc1c\uc0dd\uc5d0 \ub300\ud574 \uc870\uc57c\uc5d0\uc11c\ub294 \ucc45\uc784\uc790\ub97c \ubb38\ucc45\ud558\uace0 \uc0ac\uc0c1\uc790\uc5d0 \ub300\ud55c \ubc30\uc0c1\uc744 \uc2e4\uc2dc\ud558\uc600\ub2e4. \uadf8\ub7ec\ub098, \uc77c\ubcf8\uc778 \uc804\ucc28 \uc885\uc5c5\uc6d0\uc740 \uc6b4\ud589 \uc790\uccb4\ub97c \uac70\ubd80\ud558\uace0, \ud638\uc2e0\uc6a9 \uad8c\ucd1d\uc758 \uc18c\uc9c0, \uacbd\ucc30 1\uc778\uc758 \ucca8\uc2b9, \uadf8\ub9ac\uace0 \uc720\uc871 \ubcf4\ud638\ub97c \uc704\ud55c \uacf5\ud0c1\uae08 \uc81c\ub3c4 \uc2e4\uc2dc \ub4f1\uc744 \ub0b4\uac78\uc5c8\ub2e4. \uadf8\ub7ec\ub098 \ud611\uc0c1\uc740 \ub09c\ud56d\uc744 \uacaa\uc5b4, \uc77c\ubcf8\uc778 \uc885\uc5c5\uc6d0\uc740 \uc804\uc6d0 \ucca0\uc218\ud558\uc600\uc73c\uba70, \uc774\ub85c \uc778\ud574 5\uac1c\uc6d4\uac04 \uc6b4\ud734\ud558\uac8c \ub418\uc5c8\ub2e4. \uc774\uc5d0 \ud55c\uc131\uc804\uae30\ud68c\uc0ac\uce21\uc740 \ubbf8\uad6d\uc778 \uc6b4\uc804\uc218 8\uba85\uacfc \uae30\uacc4\uacf5 2\uba85\uc744 \uae09\uac70 \ubaa8\uc9d1\ud558\uc5ec 9\uc6d4 \ub9d0 \uacbd\uc5d0 \uc6b4\ud589\uc744 \uc7ac\uac1c\ud558\uc600\ub2e4.", "answer": "5\uac1c\uc6d4", "sentence": "\uadf8\ub7ec\ub098 \ud611\uc0c1\uc740 \ub09c\ud56d\uc744 \uacaa\uc5b4, \uc77c\ubcf8\uc778 \uc885\uc5c5\uc6d0\uc740 \uc804\uc6d0 \ucca0\uc218\ud558\uc600\uc73c\uba70, \uc774\ub85c \uc778\ud574 5\uac1c\uc6d4 \uac04 \uc6b4\ud734\ud558\uac8c \ub418\uc5c8\ub2e4.", "paragraph_sentence": "\uadf8\ub7ec\ub098 \uac1c\ud1b5 \ud6c4 10\uc77c\uc9f8 \ub418\ub294 \ub0a0 \uc885\ub85c2\uac00\ucbe4\uc5d0\uc11c \ub2e4\uc12f \uc0b4\ub41c \uc544\uc774\uac00 \uc804\ucc28\uac00 \uce58\uc5b4 \uc8fd\ub294 \uc77c\uc774 \ubc8c\uc5b4\uc84c\ub2e4. \uc544\uc774\uc758 \uc544\ubc84\uc9c0\uac00 \uc774\ub97c \ubcf4\uace0 \ub3c4\ub07c\ub97c \ube7c\ub4e4\uace0 \uc804\ucc28\uc5d0 \ub2ec\ub824\ub4e4\uc790, \uc2dc\ubbfc\ub4e4\uc774 \uc5ec\uae30\uc5d0 \ud569\uc138\ud558\uc5ec \ucc28\ub7c9\uc5d0 \ub2ec\ub824\ub4e4\uc5c8\ub2e4. \ub2f9\uc2dc \uc2dc\ub0b4\uc5d0 \uc804\ucc28\ub85c \uc778\ud574 \uac00\ubb44\uc774 \uc77c\uc5b4\ub09c\ub2e4\ub294 \ub465 \uc720\uc5b8\ube44\uc5b4\uac00 \uc720\ud3ec\ub418\uc5c8\ub294\ub370, \uc774\ub85c \uc778\ud574 \uc2dc\ubbfc\ub4e4\uc758 \ubc18\uc751\uc774 \uacfc\uaca9\ud574\uc9c4 \uac83\uc774 \uc544\ub2cc\uac00 \uc0dd\uac01\ub41c\ub2e4. \ucc28\uc7a5\uacfc \uc6b4\uc804\uc218\ub294 \uc694\ud589\uc774 \ube60\uc838\ub098\uc62c \uc218 \uc788\uc5c8\uc73c\ub098, \uc804\ucc28\ub294 \ud30c\uad34, \uc804\uc18c\ub418\uc5c8\ub2e4. \uc774\ud6c4 \ub2e4\ub978 \uc804\ucc28 \uc5ed\uc2dc \uc218\ub09c\uc744 \ub2f9\ud558\uace0, \ubc1c\uc804\uc18c\uae4c\uc9c0 \uc704\ud611\ub2f9\ud558\ub294 \uc0c1\ud669\uc774 \ubc1c\uc0dd\ud558\uc600\ub2e4. \uc18c\ub3d9\uc740 \uace7 \uc9c4\uc815\ub418\uace0, \uc774\ub7ec\ud55c \ud3ed\ub3d9 \ubc1c\uc0dd\uc5d0 \ub300\ud574 \uc870\uc57c\uc5d0\uc11c\ub294 \ucc45\uc784\uc790\ub97c \ubb38\ucc45\ud558\uace0 \uc0ac\uc0c1\uc790\uc5d0 \ub300\ud55c \ubc30\uc0c1\uc744 \uc2e4\uc2dc\ud558\uc600\ub2e4. \uadf8\ub7ec\ub098, \uc77c\ubcf8\uc778 \uc804\ucc28 \uc885\uc5c5\uc6d0\uc740 \uc6b4\ud589 \uc790\uccb4\ub97c \uac70\ubd80\ud558\uace0, \ud638\uc2e0\uc6a9 \uad8c\ucd1d\uc758 \uc18c\uc9c0, \uacbd\ucc30 1\uc778\uc758 \ucca8\uc2b9, \uadf8\ub9ac\uace0 \uc720\uc871 \ubcf4\ud638\ub97c \uc704\ud55c \uacf5\ud0c1\uae08 \uc81c\ub3c4 \uc2e4\uc2dc \ub4f1\uc744 \ub0b4\uac78\uc5c8\ub2e4. \uadf8\ub7ec\ub098 \ud611\uc0c1\uc740 \ub09c\ud56d\uc744 \uacaa\uc5b4, \uc77c\ubcf8\uc778 \uc885\uc5c5\uc6d0\uc740 \uc804\uc6d0 \ucca0\uc218\ud558\uc600\uc73c\uba70, \uc774\ub85c \uc778\ud574 5\uac1c\uc6d4 \uac04 \uc6b4\ud734\ud558\uac8c \ub418\uc5c8\ub2e4. \uc774\uc5d0 \ud55c\uc131\uc804\uae30\ud68c\uc0ac\uce21\uc740 \ubbf8\uad6d\uc778 \uc6b4\uc804\uc218 8\uba85\uacfc \uae30\uacc4\uacf5 2\uba85\uc744 \uae09\uac70 \ubaa8\uc9d1\ud558\uc5ec 9\uc6d4 \ub9d0 \uacbd\uc5d0 \uc6b4\ud589\uc744 \uc7ac\uac1c\ud558\uc600\ub2e4.", "paragraph_answer": "\uadf8\ub7ec\ub098 \uac1c\ud1b5 \ud6c4 10\uc77c\uc9f8 \ub418\ub294 \ub0a0 \uc885\ub85c2\uac00\ucbe4\uc5d0\uc11c \ub2e4\uc12f \uc0b4\ub41c \uc544\uc774\uac00 \uc804\ucc28\uac00 \uce58\uc5b4 \uc8fd\ub294 \uc77c\uc774 \ubc8c\uc5b4\uc84c\ub2e4. \uc544\uc774\uc758 \uc544\ubc84\uc9c0\uac00 \uc774\ub97c \ubcf4\uace0 \ub3c4\ub07c\ub97c \ube7c\ub4e4\uace0 \uc804\ucc28\uc5d0 \ub2ec\ub824\ub4e4\uc790, \uc2dc\ubbfc\ub4e4\uc774 \uc5ec\uae30\uc5d0 \ud569\uc138\ud558\uc5ec \ucc28\ub7c9\uc5d0 \ub2ec\ub824\ub4e4\uc5c8\ub2e4. \ub2f9\uc2dc \uc2dc\ub0b4\uc5d0 \uc804\ucc28\ub85c \uc778\ud574 \uac00\ubb44\uc774 \uc77c\uc5b4\ub09c\ub2e4\ub294 \ub465 \uc720\uc5b8\ube44\uc5b4\uac00 \uc720\ud3ec\ub418\uc5c8\ub294\ub370, \uc774\ub85c \uc778\ud574 \uc2dc\ubbfc\ub4e4\uc758 \ubc18\uc751\uc774 \uacfc\uaca9\ud574\uc9c4 \uac83\uc774 \uc544\ub2cc\uac00 \uc0dd\uac01\ub41c\ub2e4. \ucc28\uc7a5\uacfc \uc6b4\uc804\uc218\ub294 \uc694\ud589\uc774 \ube60\uc838\ub098\uc62c \uc218 \uc788\uc5c8\uc73c\ub098, \uc804\ucc28\ub294 \ud30c\uad34, \uc804\uc18c\ub418\uc5c8\ub2e4. \uc774\ud6c4 \ub2e4\ub978 \uc804\ucc28 \uc5ed\uc2dc \uc218\ub09c\uc744 \ub2f9\ud558\uace0, \ubc1c\uc804\uc18c\uae4c\uc9c0 \uc704\ud611\ub2f9\ud558\ub294 \uc0c1\ud669\uc774 \ubc1c\uc0dd\ud558\uc600\ub2e4. \uc18c\ub3d9\uc740 \uace7 \uc9c4\uc815\ub418\uace0, \uc774\ub7ec\ud55c \ud3ed\ub3d9 \ubc1c\uc0dd\uc5d0 \ub300\ud574 \uc870\uc57c\uc5d0\uc11c\ub294 \ucc45\uc784\uc790\ub97c \ubb38\ucc45\ud558\uace0 \uc0ac\uc0c1\uc790\uc5d0 \ub300\ud55c \ubc30\uc0c1\uc744 \uc2e4\uc2dc\ud558\uc600\ub2e4. \uadf8\ub7ec\ub098, \uc77c\ubcf8\uc778 \uc804\ucc28 \uc885\uc5c5\uc6d0\uc740 \uc6b4\ud589 \uc790\uccb4\ub97c \uac70\ubd80\ud558\uace0, \ud638\uc2e0\uc6a9 \uad8c\ucd1d\uc758 \uc18c\uc9c0, \uacbd\ucc30 1\uc778\uc758 \ucca8\uc2b9, \uadf8\ub9ac\uace0 \uc720\uc871 \ubcf4\ud638\ub97c \uc704\ud55c \uacf5\ud0c1\uae08 \uc81c\ub3c4 \uc2e4\uc2dc \ub4f1\uc744 \ub0b4\uac78\uc5c8\ub2e4. \uadf8\ub7ec\ub098 \ud611\uc0c1\uc740 \ub09c\ud56d\uc744 \uacaa\uc5b4, \uc77c\ubcf8\uc778 \uc885\uc5c5\uc6d0\uc740 \uc804\uc6d0 \ucca0\uc218\ud558\uc600\uc73c\uba70, \uc774\ub85c \uc778\ud574 5\uac1c\uc6d4 \uac04 \uc6b4\ud734\ud558\uac8c \ub418\uc5c8\ub2e4. \uc774\uc5d0 \ud55c\uc131\uc804\uae30\ud68c\uc0ac\uce21\uc740 \ubbf8\uad6d\uc778 \uc6b4\uc804\uc218 8\uba85\uacfc \uae30\uacc4\uacf5 2\uba85\uc744 \uae09\uac70 \ubaa8\uc9d1\ud558\uc5ec 9\uc6d4 \ub9d0 \uacbd\uc5d0 \uc6b4\ud589\uc744 \uc7ac\uac1c\ud558\uc600\ub2e4.", "sentence_answer": "\uadf8\ub7ec\ub098 \ud611\uc0c1\uc740 \ub09c\ud56d\uc744 \uacaa\uc5b4, \uc77c\ubcf8\uc778 \uc885\uc5c5\uc6d0\uc740 \uc804\uc6d0 \ucca0\uc218\ud558\uc600\uc73c\uba70, \uc774\ub85c \uc778\ud574 5\uac1c\uc6d4 \uac04 \uc6b4\ud734\ud558\uac8c \ub418\uc5c8\ub2e4.", "paragraph_id": "6517594-1-1", "paragraph_question": "question: \uc11c\uc6b8\uc804\ucc28 \uc6b4\ud589\uc5d0 \uad00\ud55c \ud611\uc0c1 \ub09c\ud669\uc73c\ub85c \uc6b4\ud589\uc774 \uc911\uc9c0\ub41c \uae30\uac04\uc740?, context: \uadf8\ub7ec\ub098 \uac1c\ud1b5 \ud6c4 10\uc77c\uc9f8 \ub418\ub294 \ub0a0 \uc885\ub85c2\uac00\ucbe4\uc5d0\uc11c \ub2e4\uc12f \uc0b4\ub41c \uc544\uc774\uac00 \uc804\ucc28\uac00 \uce58\uc5b4 \uc8fd\ub294 \uc77c\uc774 \ubc8c\uc5b4\uc84c\ub2e4. \uc544\uc774\uc758 \uc544\ubc84\uc9c0\uac00 \uc774\ub97c \ubcf4\uace0 \ub3c4\ub07c\ub97c \ube7c\ub4e4\uace0 \uc804\ucc28\uc5d0 \ub2ec\ub824\ub4e4\uc790, \uc2dc\ubbfc\ub4e4\uc774 \uc5ec\uae30\uc5d0 \ud569\uc138\ud558\uc5ec \ucc28\ub7c9\uc5d0 \ub2ec\ub824\ub4e4\uc5c8\ub2e4. \ub2f9\uc2dc \uc2dc\ub0b4\uc5d0 \uc804\ucc28\ub85c \uc778\ud574 \uac00\ubb44\uc774 \uc77c\uc5b4\ub09c\ub2e4\ub294 \ub465 \uc720\uc5b8\ube44\uc5b4\uac00 \uc720\ud3ec\ub418\uc5c8\ub294\ub370, \uc774\ub85c \uc778\ud574 \uc2dc\ubbfc\ub4e4\uc758 \ubc18\uc751\uc774 \uacfc\uaca9\ud574\uc9c4 \uac83\uc774 \uc544\ub2cc\uac00 \uc0dd\uac01\ub41c\ub2e4. \ucc28\uc7a5\uacfc \uc6b4\uc804\uc218\ub294 \uc694\ud589\uc774 \ube60\uc838\ub098\uc62c \uc218 \uc788\uc5c8\uc73c\ub098, \uc804\ucc28\ub294 \ud30c\uad34, \uc804\uc18c\ub418\uc5c8\ub2e4. \uc774\ud6c4 \ub2e4\ub978 \uc804\ucc28 \uc5ed\uc2dc \uc218\ub09c\uc744 \ub2f9\ud558\uace0, \ubc1c\uc804\uc18c\uae4c\uc9c0 \uc704\ud611\ub2f9\ud558\ub294 \uc0c1\ud669\uc774 \ubc1c\uc0dd\ud558\uc600\ub2e4. \uc18c\ub3d9\uc740 \uace7 \uc9c4\uc815\ub418\uace0, \uc774\ub7ec\ud55c \ud3ed\ub3d9 \ubc1c\uc0dd\uc5d0 \ub300\ud574 \uc870\uc57c\uc5d0\uc11c\ub294 \ucc45\uc784\uc790\ub97c \ubb38\ucc45\ud558\uace0 \uc0ac\uc0c1\uc790\uc5d0 \ub300\ud55c \ubc30\uc0c1\uc744 \uc2e4\uc2dc\ud558\uc600\ub2e4. \uadf8\ub7ec\ub098, \uc77c\ubcf8\uc778 \uc804\ucc28 \uc885\uc5c5\uc6d0\uc740 \uc6b4\ud589 \uc790\uccb4\ub97c \uac70\ubd80\ud558\uace0, \ud638\uc2e0\uc6a9 \uad8c\ucd1d\uc758 \uc18c\uc9c0, \uacbd\ucc30 1\uc778\uc758 \ucca8\uc2b9, \uadf8\ub9ac\uace0 \uc720\uc871 \ubcf4\ud638\ub97c \uc704\ud55c \uacf5\ud0c1\uae08 \uc81c\ub3c4 \uc2e4\uc2dc \ub4f1\uc744 \ub0b4\uac78\uc5c8\ub2e4. \uadf8\ub7ec\ub098 \ud611\uc0c1\uc740 \ub09c\ud56d\uc744 \uacaa\uc5b4, \uc77c\ubcf8\uc778 \uc885\uc5c5\uc6d0\uc740 \uc804\uc6d0 \ucca0\uc218\ud558\uc600\uc73c\uba70, \uc774\ub85c \uc778\ud574 5\uac1c\uc6d4\uac04 \uc6b4\ud734\ud558\uac8c \ub418\uc5c8\ub2e4. \uc774\uc5d0 \ud55c\uc131\uc804\uae30\ud68c\uc0ac\uce21\uc740 \ubbf8\uad6d\uc778 \uc6b4\uc804\uc218 8\uba85\uacfc \uae30\uacc4\uacf5 2\uba85\uc744 \uae09\uac70 \ubaa8\uc9d1\ud558\uc5ec 9\uc6d4 \ub9d0 \uacbd\uc5d0 \uc6b4\ud589\uc744 \uc7ac\uac1c\ud558\uc600\ub2e4."} {"question": "\uc7ac\uc0dd\uc5d0\ub108\uc9c0 \ubc1c\uc804\uc0ac\uc5c5\uc790\ub4e4\uc774 \ud65c\uc131\ud654\uc5d0 \ud070 \uae30\uc5ec\ub97c \ud55c \ud310\ub9e4 \uc815\ucc45\uc740?", "paragraph": "\uc7ac\uc0dd\uc5d0\ub108\uc9c0\ubc95\uc740 1991\ub144\uc5d0 \uc81c\uc815\ub41c \uc804\ub825\ub9e4\uc785\ubc95\uc744 \ud1a0\ub300\ub85c \uc81c\uc815\ub418\uc5b4 2000\ub144\ubd80\ud130 \ubcf8\uaca9\uc801\uc73c\ub85c \uc2dc\ud589\ub418\uc5c8\ub2e4. \uc7ac\uc0dd\uc5d0\ub108\uc9c0\ubc95\uc740 \uc804\ub825\ub9dd \uc5f0\uacb0\uc5d0 \ub300\ud55c \ubcf4\uc7a5, \uac00\uaca9\uc5d0 \ub300\ud55c \ubcf4\uc7a5, \uc0dd\uc0b0\uc804\ub825 \ub9e4\uc785\uc5d0 \ub300\ud55c \ubcf4\uc7a5\uc744 \uc8fc\ucd95\uc73c\ub85c \uc2dc\ud589\ub418\uc5c8\ub2e4. \uc774 \ubc95\uc758 \uc8fc\ub41c \ubaa9\uc801\uc740 \uae30\ud6c4\uc640 \ud658\uacbd \ubcf4\ud638\ub97c \uc704\ud574 \uc5d0\ub108\uc9c0\ub97c \uacf5\uae09\ud558\ub294\ub370 \uc788\uc5b4\uc11c \uc9c0\uc18d\uac00\ub2a5\ud55c \ubc1c\uc804\uc744 \uac00\ub2a5\ud558\uac8c \ud558\ub294 \uac83\uc774\ub2e4. \ub610\ud55c \uc5d0\ub108\uc9c0 \uacf5\uae09 \ube44\uc6a9\uc744 \uc808\uac10\ud574 \uad6d\uac00 \uacbd\uc81c\uc5d0 \uc77c\uc870\ud558\uace0, \ud654\uc11d \uc5f0\ub8cc\ub97c \uc808\uc57d\ud558\ub294 \ub3d9\uc2dc\uc5d0 \uc0c8\ub85c\uc6b4 \uc7ac\uc0dd\uac00\ub2a5\ud55c \uc5d0\ub108\uc9c0\ub97c \uac1c\ubc1c\ud558\uac8c \ud574 \uc7a5\uae30\uc801\uc73c\ub85c \ud070 \ud6a8\uacfc\ub97c \ubcfc \uc218 \uc788\ub294 \uc81c\ub3c4\uc774\ub2e4. \uc7ac\uc0dd\uc5d0\ub108\uc9c0\uc77c\uc218\ub85d \ub192\uc740 \uac00\uaca9\uc5d0 \ub9e4\uc785\ub418\uba70, \ubaa8\ub4e0 \uc7ac\uc0dd\uc5d0\ub108\uc9c0 \ubc1c\uc804 \uc0ac\uc5c5\uc790\ub4e4\uc740 \uc7ac\uc0dd\uc5d0\ub108\uc9c0 \uc804\uae30\ub97c \uc7ac\uc0dd\uc5d0\ub108\uc9c0\ubc95\uc5d0 \ub530\ub77c \ub2e4\ub978 \uc5d0\ub108\uc9c0\uc6d0\ubcf4\ub2e4 \uc6b0\uc120\uc801\uc778 \uac00\uaca9\uc5d0 \ud55c\ub3c4 \uc81c\ud55c \uc5c6\uc774 \ud310\ub9e4\ud560 \uc218 \uc788\ub2e4\ub294 \ud61c\ud0dd\uc744 \uc8fc\uc5b4 \uc7ac\uc0dd\uc5d0\ub108\uc9c0\uc2dc\uc7a5 \ud65c\uc131\ud654\uc5d0 \ud070 \uae30\uc5ec\ub97c \ud588\ub2e4.", "answer": "\uac00\uaca9\uc5d0 \ud55c\ub3c4 \uc81c\ud55c \uc5c6\uc774 \ud310\ub9e4", "sentence": "\uc7ac\uc0dd\uc5d0\ub108\uc9c0\uc77c\uc218\ub85d \ub192\uc740 \uac00\uaca9\uc5d0 \ub9e4\uc785\ub418\uba70, \ubaa8\ub4e0 \uc7ac\uc0dd\uc5d0\ub108\uc9c0 \ubc1c\uc804 \uc0ac\uc5c5\uc790\ub4e4\uc740 \uc7ac\uc0dd\uc5d0\ub108\uc9c0 \uc804\uae30\ub97c \uc7ac\uc0dd\uc5d0\ub108\uc9c0\ubc95\uc5d0 \ub530\ub77c \ub2e4\ub978 \uc5d0\ub108\uc9c0\uc6d0\ubcf4\ub2e4 \uc6b0\uc120\uc801\uc778 \uac00\uaca9\uc5d0 \ud55c\ub3c4 \uc81c\ud55c \uc5c6\uc774 \ud310\ub9e4 \ud560 \uc218 \uc788\ub2e4\ub294 \ud61c\ud0dd\uc744 \uc8fc\uc5b4 \uc7ac\uc0dd\uc5d0\ub108\uc9c0\uc2dc\uc7a5 \ud65c\uc131\ud654\uc5d0 \ud070 \uae30\uc5ec\ub97c \ud588\ub2e4.", "paragraph_sentence": "\uc7ac\uc0dd\uc5d0\ub108\uc9c0\ubc95\uc740 1991\ub144\uc5d0 \uc81c\uc815\ub41c \uc804\ub825\ub9e4\uc785\ubc95\uc744 \ud1a0\ub300\ub85c \uc81c\uc815\ub418\uc5b4 2000\ub144\ubd80\ud130 \ubcf8\uaca9\uc801\uc73c\ub85c \uc2dc\ud589\ub418\uc5c8\ub2e4. \uc7ac\uc0dd\uc5d0\ub108\uc9c0\ubc95\uc740 \uc804\ub825\ub9dd \uc5f0\uacb0\uc5d0 \ub300\ud55c \ubcf4\uc7a5, \uac00\uaca9\uc5d0 \ub300\ud55c \ubcf4\uc7a5, \uc0dd\uc0b0\uc804\ub825 \ub9e4\uc785\uc5d0 \ub300\ud55c \ubcf4\uc7a5\uc744 \uc8fc\ucd95\uc73c\ub85c \uc2dc\ud589\ub418\uc5c8\ub2e4. \uc774 \ubc95\uc758 \uc8fc\ub41c \ubaa9\uc801\uc740 \uae30\ud6c4\uc640 \ud658\uacbd \ubcf4\ud638\ub97c \uc704\ud574 \uc5d0\ub108\uc9c0\ub97c \uacf5\uae09\ud558\ub294\ub370 \uc788\uc5b4\uc11c \uc9c0\uc18d\uac00\ub2a5\ud55c \ubc1c\uc804\uc744 \uac00\ub2a5\ud558\uac8c \ud558\ub294 \uac83\uc774\ub2e4. \ub610\ud55c \uc5d0\ub108\uc9c0 \uacf5\uae09 \ube44\uc6a9\uc744 \uc808\uac10\ud574 \uad6d\uac00 \uacbd\uc81c\uc5d0 \uc77c\uc870\ud558\uace0, \ud654\uc11d \uc5f0\ub8cc\ub97c \uc808\uc57d\ud558\ub294 \ub3d9\uc2dc\uc5d0 \uc0c8\ub85c\uc6b4 \uc7ac\uc0dd\uac00\ub2a5\ud55c \uc5d0\ub108\uc9c0\ub97c \uac1c\ubc1c\ud558\uac8c \ud574 \uc7a5\uae30\uc801\uc73c\ub85c \ud070 \ud6a8\uacfc\ub97c \ubcfc \uc218 \uc788\ub294 \uc81c\ub3c4\uc774\ub2e4. \uc7ac\uc0dd\uc5d0\ub108\uc9c0\uc77c\uc218\ub85d \ub192\uc740 \uac00\uaca9\uc5d0 \ub9e4\uc785\ub418\uba70, \ubaa8\ub4e0 \uc7ac\uc0dd\uc5d0\ub108\uc9c0 \ubc1c\uc804 \uc0ac\uc5c5\uc790\ub4e4\uc740 \uc7ac\uc0dd\uc5d0\ub108\uc9c0 \uc804\uae30\ub97c \uc7ac\uc0dd\uc5d0\ub108\uc9c0\ubc95\uc5d0 \ub530\ub77c \ub2e4\ub978 \uc5d0\ub108\uc9c0\uc6d0\ubcf4\ub2e4 \uc6b0\uc120\uc801\uc778 \uac00\uaca9\uc5d0 \ud55c\ub3c4 \uc81c\ud55c \uc5c6\uc774 \ud310\ub9e4 \ud560 \uc218 \uc788\ub2e4\ub294 \ud61c\ud0dd\uc744 \uc8fc\uc5b4 \uc7ac\uc0dd\uc5d0\ub108\uc9c0\uc2dc\uc7a5 \ud65c\uc131\ud654\uc5d0 \ud070 \uae30\uc5ec\ub97c \ud588\ub2e4. ", "paragraph_answer": "\uc7ac\uc0dd\uc5d0\ub108\uc9c0\ubc95\uc740 1991\ub144\uc5d0 \uc81c\uc815\ub41c \uc804\ub825\ub9e4\uc785\ubc95\uc744 \ud1a0\ub300\ub85c \uc81c\uc815\ub418\uc5b4 2000\ub144\ubd80\ud130 \ubcf8\uaca9\uc801\uc73c\ub85c \uc2dc\ud589\ub418\uc5c8\ub2e4. \uc7ac\uc0dd\uc5d0\ub108\uc9c0\ubc95\uc740 \uc804\ub825\ub9dd \uc5f0\uacb0\uc5d0 \ub300\ud55c \ubcf4\uc7a5, \uac00\uaca9\uc5d0 \ub300\ud55c \ubcf4\uc7a5, \uc0dd\uc0b0\uc804\ub825 \ub9e4\uc785\uc5d0 \ub300\ud55c \ubcf4\uc7a5\uc744 \uc8fc\ucd95\uc73c\ub85c \uc2dc\ud589\ub418\uc5c8\ub2e4. \uc774 \ubc95\uc758 \uc8fc\ub41c \ubaa9\uc801\uc740 \uae30\ud6c4\uc640 \ud658\uacbd \ubcf4\ud638\ub97c \uc704\ud574 \uc5d0\ub108\uc9c0\ub97c \uacf5\uae09\ud558\ub294\ub370 \uc788\uc5b4\uc11c \uc9c0\uc18d\uac00\ub2a5\ud55c \ubc1c\uc804\uc744 \uac00\ub2a5\ud558\uac8c \ud558\ub294 \uac83\uc774\ub2e4. \ub610\ud55c \uc5d0\ub108\uc9c0 \uacf5\uae09 \ube44\uc6a9\uc744 \uc808\uac10\ud574 \uad6d\uac00 \uacbd\uc81c\uc5d0 \uc77c\uc870\ud558\uace0, \ud654\uc11d \uc5f0\ub8cc\ub97c \uc808\uc57d\ud558\ub294 \ub3d9\uc2dc\uc5d0 \uc0c8\ub85c\uc6b4 \uc7ac\uc0dd\uac00\ub2a5\ud55c \uc5d0\ub108\uc9c0\ub97c \uac1c\ubc1c\ud558\uac8c \ud574 \uc7a5\uae30\uc801\uc73c\ub85c \ud070 \ud6a8\uacfc\ub97c \ubcfc \uc218 \uc788\ub294 \uc81c\ub3c4\uc774\ub2e4. \uc7ac\uc0dd\uc5d0\ub108\uc9c0\uc77c\uc218\ub85d \ub192\uc740 \uac00\uaca9\uc5d0 \ub9e4\uc785\ub418\uba70, \ubaa8\ub4e0 \uc7ac\uc0dd\uc5d0\ub108\uc9c0 \ubc1c\uc804 \uc0ac\uc5c5\uc790\ub4e4\uc740 \uc7ac\uc0dd\uc5d0\ub108\uc9c0 \uc804\uae30\ub97c \uc7ac\uc0dd\uc5d0\ub108\uc9c0\ubc95\uc5d0 \ub530\ub77c \ub2e4\ub978 \uc5d0\ub108\uc9c0\uc6d0\ubcf4\ub2e4 \uc6b0\uc120\uc801\uc778 \uac00\uaca9\uc5d0 \ud55c\ub3c4 \uc81c\ud55c \uc5c6\uc774 \ud310\ub9e4 \ud560 \uc218 \uc788\ub2e4\ub294 \ud61c\ud0dd\uc744 \uc8fc\uc5b4 \uc7ac\uc0dd\uc5d0\ub108\uc9c0\uc2dc\uc7a5 \ud65c\uc131\ud654\uc5d0 \ud070 \uae30\uc5ec\ub97c \ud588\ub2e4.", "sentence_answer": "\uc7ac\uc0dd\uc5d0\ub108\uc9c0\uc77c\uc218\ub85d \ub192\uc740 \uac00\uaca9\uc5d0 \ub9e4\uc785\ub418\uba70, \ubaa8\ub4e0 \uc7ac\uc0dd\uc5d0\ub108\uc9c0 \ubc1c\uc804 \uc0ac\uc5c5\uc790\ub4e4\uc740 \uc7ac\uc0dd\uc5d0\ub108\uc9c0 \uc804\uae30\ub97c \uc7ac\uc0dd\uc5d0\ub108\uc9c0\ubc95\uc5d0 \ub530\ub77c \ub2e4\ub978 \uc5d0\ub108\uc9c0\uc6d0\ubcf4\ub2e4 \uc6b0\uc120\uc801\uc778 \uac00\uaca9\uc5d0 \ud55c\ub3c4 \uc81c\ud55c \uc5c6\uc774 \ud310\ub9e4 \ud560 \uc218 \uc788\ub2e4\ub294 \ud61c\ud0dd\uc744 \uc8fc\uc5b4 \uc7ac\uc0dd\uc5d0\ub108\uc9c0\uc2dc\uc7a5 \ud65c\uc131\ud654\uc5d0 \ud070 \uae30\uc5ec\ub97c \ud588\ub2e4.", "paragraph_id": "6656928-2-2", "paragraph_question": "question: \uc7ac\uc0dd\uc5d0\ub108\uc9c0 \ubc1c\uc804\uc0ac\uc5c5\uc790\ub4e4\uc774 \ud65c\uc131\ud654\uc5d0 \ud070 \uae30\uc5ec\ub97c \ud55c \ud310\ub9e4 \uc815\ucc45\uc740?, context: \uc7ac\uc0dd\uc5d0\ub108\uc9c0\ubc95\uc740 1991\ub144\uc5d0 \uc81c\uc815\ub41c \uc804\ub825\ub9e4\uc785\ubc95\uc744 \ud1a0\ub300\ub85c \uc81c\uc815\ub418\uc5b4 2000\ub144\ubd80\ud130 \ubcf8\uaca9\uc801\uc73c\ub85c \uc2dc\ud589\ub418\uc5c8\ub2e4. \uc7ac\uc0dd\uc5d0\ub108\uc9c0\ubc95\uc740 \uc804\ub825\ub9dd \uc5f0\uacb0\uc5d0 \ub300\ud55c \ubcf4\uc7a5, \uac00\uaca9\uc5d0 \ub300\ud55c \ubcf4\uc7a5, \uc0dd\uc0b0\uc804\ub825 \ub9e4\uc785\uc5d0 \ub300\ud55c \ubcf4\uc7a5\uc744 \uc8fc\ucd95\uc73c\ub85c \uc2dc\ud589\ub418\uc5c8\ub2e4. \uc774 \ubc95\uc758 \uc8fc\ub41c \ubaa9\uc801\uc740 \uae30\ud6c4\uc640 \ud658\uacbd \ubcf4\ud638\ub97c \uc704\ud574 \uc5d0\ub108\uc9c0\ub97c \uacf5\uae09\ud558\ub294\ub370 \uc788\uc5b4\uc11c \uc9c0\uc18d\uac00\ub2a5\ud55c \ubc1c\uc804\uc744 \uac00\ub2a5\ud558\uac8c \ud558\ub294 \uac83\uc774\ub2e4. \ub610\ud55c \uc5d0\ub108\uc9c0 \uacf5\uae09 \ube44\uc6a9\uc744 \uc808\uac10\ud574 \uad6d\uac00 \uacbd\uc81c\uc5d0 \uc77c\uc870\ud558\uace0, \ud654\uc11d \uc5f0\ub8cc\ub97c \uc808\uc57d\ud558\ub294 \ub3d9\uc2dc\uc5d0 \uc0c8\ub85c\uc6b4 \uc7ac\uc0dd\uac00\ub2a5\ud55c \uc5d0\ub108\uc9c0\ub97c \uac1c\ubc1c\ud558\uac8c \ud574 \uc7a5\uae30\uc801\uc73c\ub85c \ud070 \ud6a8\uacfc\ub97c \ubcfc \uc218 \uc788\ub294 \uc81c\ub3c4\uc774\ub2e4. \uc7ac\uc0dd\uc5d0\ub108\uc9c0\uc77c\uc218\ub85d \ub192\uc740 \uac00\uaca9\uc5d0 \ub9e4\uc785\ub418\uba70, \ubaa8\ub4e0 \uc7ac\uc0dd\uc5d0\ub108\uc9c0 \ubc1c\uc804 \uc0ac\uc5c5\uc790\ub4e4\uc740 \uc7ac\uc0dd\uc5d0\ub108\uc9c0 \uc804\uae30\ub97c \uc7ac\uc0dd\uc5d0\ub108\uc9c0\ubc95\uc5d0 \ub530\ub77c \ub2e4\ub978 \uc5d0\ub108\uc9c0\uc6d0\ubcf4\ub2e4 \uc6b0\uc120\uc801\uc778 \uac00\uaca9\uc5d0 \ud55c\ub3c4 \uc81c\ud55c \uc5c6\uc774 \ud310\ub9e4\ud560 \uc218 \uc788\ub2e4\ub294 \ud61c\ud0dd\uc744 \uc8fc\uc5b4 \uc7ac\uc0dd\uc5d0\ub108\uc9c0\uc2dc\uc7a5 \ud65c\uc131\ud654\uc5d0 \ud070 \uae30\uc5ec\ub97c \ud588\ub2e4."} {"question": "1918\ub144 \uacbd\uc131\uc720\uce58\uc6d0\uc744 \ubc29\ubb38\ud55c \uc601\uce5c\uc655 \uc774\uc740 \uc55e\uc5d0\uc11c \uc774\uc6b0\uac00 \ubd80\ub978 \ucc3d\uac00\ub294?", "paragraph": "1915\ub144\uc5d0 \uacbd\uc131\uc720\uce58\uc6d0\uc5d0 \uc785\ud559\ud558\uc5ec 1918\ub144 3\uc6d4 28\uc77c\uc5d0 \uc878\uc5c5\ud558\uc600\ub2e4. \uc77c\ubcf8\uc5d0\uc11c \uc77c\uc2dc \uadc0\uad6d\ud55c \uc601\uce5c\uc655 \uc774\uc740\uc774 1918\ub144 1\uc6d4 25\uc77c\uc5d0 \uacbd\uc131\uc720\uce58\uc6d0\uc744 \ubc29\ubb38\ud558\uc600\uc744 \ub54c \uc774\uc6b0\ub294 \uc601\uce5c\uc655 \uc55e\uc5d0\uc11c \u2018\ub9e4\ud654\uc640 \uaf80\uaf2c\ub9ac\u2019\ub77c\ub294 \ucc3d\uac00\ub97c \ubd88\ub800\ub2e4. 1919\ub144 1\uc6d4 21\uc77c\uc5d0 \uace0\uc885\uc774 \uc2b9\ud558\ud558\uc790 150\uc77c \ub3d9\uc548 \ubcf5\uc0c1(\u670d\u55aa)\uc744 \ud558\uc600\ub2e4. \uac19\uc740 \ud574 4\uc6d4\uc5d0 \uacbd\uc131\uc885\ub85c\uacf5\ub9bd\uc2ec\uc0c1\uace0\ub4f1\uc18c\ud559\uad50\uc5d0 \uc785\ud559\ud558\uc600\ub2e4. 1922\ub144 \ubd04\uc5d0 \uc885\ub85c\uc18c\ud559\uad50\uc5d0\uc11c \uc2ec\uc0c1\uacfc 3\ud559\ub144 \uacfc\uc815\uc744 \ub9c8\uce58\uace0, \uac19\uc740 \ud574 7\uc6d4 5\uc77c\uc5d0 \uc720\ud559 \uba85\ubd84\uc73c\ub85c \uc77c\ubcf8\uc73c\ub85c \ubcf4\ub0b4\uc838 \ud559\uc2b5\uc6d0 \ucd08\ub4f1\uacfc 4\ud559\ub144\uc5d0 \ud3b8\uc785\ud558\uc600\ub2e4. \uac19\uc740 \ud574 7\uc6d4 22\uc77c\uc5d0 \uc5ec\ub984 \ud734\uac00\ub97c \uc774\uc720\ub85c \uc870\uc120\uc73c\ub85c \uadc0\ud658\ud558\uba74\uc11c \uc624\uc0ac\uce74 \uc5ed\uc744 \ud1b5\uacfc\ud558\uac8c \ub418\uc5c8\ub294\ub370 \uc624\uc0ac\uce74\uc758 \uc870\uc120\uc778\ub4e4\uc774 \uc774\uc6b0\ub97c \ub458\ub7ec\uc2f8\uace0 \uc77c\uc744 \ubaa8\uc758\ud55c\ub2e4\ub294 \uc18c\ubb38\uc774 \ub3cc\uc544 \uc624\uc0ac\uce74 \uc18c\ub124\uc790\ud0a4(\u66fe\u6839\u5d0e) \uacbd\ucc30\uc11c\uc5d0\uc11c \ub2e4\uc218\uc758 \uc870\uc120\uc778\uc744 \uac80\uc18d\ud558\uc600\ub2e4.", "answer": "\ub9e4\ud654\uc640 \uaf80\uaf2c\ub9ac", "sentence": "\uc77c\ubcf8\uc5d0\uc11c \uc77c\uc2dc \uadc0\uad6d\ud55c \uc601\uce5c\uc655 \uc774\uc740\uc774 1918\ub144 1\uc6d4 25\uc77c\uc5d0 \uacbd\uc131\uc720\uce58\uc6d0\uc744 \ubc29\ubb38\ud558\uc600\uc744 \ub54c \uc774\uc6b0\ub294 \uc601\uce5c\uc655 \uc55e\uc5d0\uc11c \u2018 \ub9e4\ud654\uc640 \uaf80\uaf2c\ub9ac \u2019\ub77c\ub294 \ucc3d\uac00\ub97c \ubd88\ub800\ub2e4.", "paragraph_sentence": "1915\ub144\uc5d0 \uacbd\uc131\uc720\uce58\uc6d0\uc5d0 \uc785\ud559\ud558\uc5ec 1918\ub144 3\uc6d4 28\uc77c\uc5d0 \uc878\uc5c5\ud558\uc600\ub2e4. \uc77c\ubcf8\uc5d0\uc11c \uc77c\uc2dc \uadc0\uad6d\ud55c \uc601\uce5c\uc655 \uc774\uc740\uc774 1918\ub144 1\uc6d4 25\uc77c\uc5d0 \uacbd\uc131\uc720\uce58\uc6d0\uc744 \ubc29\ubb38\ud558\uc600\uc744 \ub54c \uc774\uc6b0\ub294 \uc601\uce5c\uc655 \uc55e\uc5d0\uc11c \u2018 \ub9e4\ud654\uc640 \uaf80\uaf2c\ub9ac \u2019\ub77c\ub294 \ucc3d\uac00\ub97c \ubd88\ub800\ub2e4. 1919\ub144 1\uc6d4 21\uc77c\uc5d0 \uace0\uc885\uc774 \uc2b9\ud558\ud558\uc790 150\uc77c \ub3d9\uc548 \ubcf5\uc0c1(\u670d\u55aa)\uc744 \ud558\uc600\ub2e4. \uac19\uc740 \ud574 4\uc6d4\uc5d0 \uacbd\uc131\uc885\ub85c\uacf5\ub9bd\uc2ec\uc0c1\uace0\ub4f1\uc18c\ud559\uad50\uc5d0 \uc785\ud559\ud558\uc600\ub2e4. 1922\ub144 \ubd04\uc5d0 \uc885\ub85c\uc18c\ud559\uad50\uc5d0\uc11c \uc2ec\uc0c1\uacfc 3\ud559\ub144 \uacfc\uc815\uc744 \ub9c8\uce58\uace0, \uac19\uc740 \ud574 7\uc6d4 5\uc77c\uc5d0 \uc720\ud559 \uba85\ubd84\uc73c\ub85c \uc77c\ubcf8\uc73c\ub85c \ubcf4\ub0b4\uc838 \ud559\uc2b5\uc6d0 \ucd08\ub4f1\uacfc 4\ud559\ub144\uc5d0 \ud3b8\uc785\ud558\uc600\ub2e4. \uac19\uc740 \ud574 7\uc6d4 22\uc77c\uc5d0 \uc5ec\ub984 \ud734\uac00\ub97c \uc774\uc720\ub85c \uc870\uc120\uc73c\ub85c \uadc0\ud658\ud558\uba74\uc11c \uc624\uc0ac\uce74 \uc5ed\uc744 \ud1b5\uacfc\ud558\uac8c \ub418\uc5c8\ub294\ub370 \uc624\uc0ac\uce74\uc758 \uc870\uc120\uc778\ub4e4\uc774 \uc774\uc6b0\ub97c \ub458\ub7ec\uc2f8\uace0 \uc77c\uc744 \ubaa8\uc758\ud55c\ub2e4\ub294 \uc18c\ubb38\uc774 \ub3cc\uc544 \uc624\uc0ac\uce74 \uc18c\ub124\uc790\ud0a4(\u66fe\u6839\u5d0e) \uacbd\ucc30\uc11c\uc5d0\uc11c \ub2e4\uc218\uc758 \uc870\uc120\uc778\uc744 \uac80\uc18d\ud558\uc600\ub2e4.", "paragraph_answer": "1915\ub144\uc5d0 \uacbd\uc131\uc720\uce58\uc6d0\uc5d0 \uc785\ud559\ud558\uc5ec 1918\ub144 3\uc6d4 28\uc77c\uc5d0 \uc878\uc5c5\ud558\uc600\ub2e4. \uc77c\ubcf8\uc5d0\uc11c \uc77c\uc2dc \uadc0\uad6d\ud55c \uc601\uce5c\uc655 \uc774\uc740\uc774 1918\ub144 1\uc6d4 25\uc77c\uc5d0 \uacbd\uc131\uc720\uce58\uc6d0\uc744 \ubc29\ubb38\ud558\uc600\uc744 \ub54c \uc774\uc6b0\ub294 \uc601\uce5c\uc655 \uc55e\uc5d0\uc11c \u2018 \ub9e4\ud654\uc640 \uaf80\uaf2c\ub9ac \u2019\ub77c\ub294 \ucc3d\uac00\ub97c \ubd88\ub800\ub2e4. 1919\ub144 1\uc6d4 21\uc77c\uc5d0 \uace0\uc885\uc774 \uc2b9\ud558\ud558\uc790 150\uc77c \ub3d9\uc548 \ubcf5\uc0c1(\u670d\u55aa)\uc744 \ud558\uc600\ub2e4. \uac19\uc740 \ud574 4\uc6d4\uc5d0 \uacbd\uc131\uc885\ub85c\uacf5\ub9bd\uc2ec\uc0c1\uace0\ub4f1\uc18c\ud559\uad50\uc5d0 \uc785\ud559\ud558\uc600\ub2e4. 1922\ub144 \ubd04\uc5d0 \uc885\ub85c\uc18c\ud559\uad50\uc5d0\uc11c \uc2ec\uc0c1\uacfc 3\ud559\ub144 \uacfc\uc815\uc744 \ub9c8\uce58\uace0, \uac19\uc740 \ud574 7\uc6d4 5\uc77c\uc5d0 \uc720\ud559 \uba85\ubd84\uc73c\ub85c \uc77c\ubcf8\uc73c\ub85c \ubcf4\ub0b4\uc838 \ud559\uc2b5\uc6d0 \ucd08\ub4f1\uacfc 4\ud559\ub144\uc5d0 \ud3b8\uc785\ud558\uc600\ub2e4. \uac19\uc740 \ud574 7\uc6d4 22\uc77c\uc5d0 \uc5ec\ub984 \ud734\uac00\ub97c \uc774\uc720\ub85c \uc870\uc120\uc73c\ub85c \uadc0\ud658\ud558\uba74\uc11c \uc624\uc0ac\uce74 \uc5ed\uc744 \ud1b5\uacfc\ud558\uac8c \ub418\uc5c8\ub294\ub370 \uc624\uc0ac\uce74\uc758 \uc870\uc120\uc778\ub4e4\uc774 \uc774\uc6b0\ub97c \ub458\ub7ec\uc2f8\uace0 \uc77c\uc744 \ubaa8\uc758\ud55c\ub2e4\ub294 \uc18c\ubb38\uc774 \ub3cc\uc544 \uc624\uc0ac\uce74 \uc18c\ub124\uc790\ud0a4(\u66fe\u6839\u5d0e) \uacbd\ucc30\uc11c\uc5d0\uc11c \ub2e4\uc218\uc758 \uc870\uc120\uc778\uc744 \uac80\uc18d\ud558\uc600\ub2e4.", "sentence_answer": "\uc77c\ubcf8\uc5d0\uc11c \uc77c\uc2dc \uadc0\uad6d\ud55c \uc601\uce5c\uc655 \uc774\uc740\uc774 1918\ub144 1\uc6d4 25\uc77c\uc5d0 \uacbd\uc131\uc720\uce58\uc6d0\uc744 \ubc29\ubb38\ud558\uc600\uc744 \ub54c \uc774\uc6b0\ub294 \uc601\uce5c\uc655 \uc55e\uc5d0\uc11c \u2018 \ub9e4\ud654\uc640 \uaf80\uaf2c\ub9ac \u2019\ub77c\ub294 \ucc3d\uac00\ub97c \ubd88\ub800\ub2e4.", "paragraph_id": "6545881-2-0", "paragraph_question": "question: 1918\ub144 \uacbd\uc131\uc720\uce58\uc6d0\uc744 \ubc29\ubb38\ud55c \uc601\uce5c\uc655 \uc774\uc740 \uc55e\uc5d0\uc11c \uc774\uc6b0\uac00 \ubd80\ub978 \ucc3d\uac00\ub294?, context: 1915\ub144\uc5d0 \uacbd\uc131\uc720\uce58\uc6d0\uc5d0 \uc785\ud559\ud558\uc5ec 1918\ub144 3\uc6d4 28\uc77c\uc5d0 \uc878\uc5c5\ud558\uc600\ub2e4. \uc77c\ubcf8\uc5d0\uc11c \uc77c\uc2dc \uadc0\uad6d\ud55c \uc601\uce5c\uc655 \uc774\uc740\uc774 1918\ub144 1\uc6d4 25\uc77c\uc5d0 \uacbd\uc131\uc720\uce58\uc6d0\uc744 \ubc29\ubb38\ud558\uc600\uc744 \ub54c \uc774\uc6b0\ub294 \uc601\uce5c\uc655 \uc55e\uc5d0\uc11c \u2018\ub9e4\ud654\uc640 \uaf80\uaf2c\ub9ac\u2019\ub77c\ub294 \ucc3d\uac00\ub97c \ubd88\ub800\ub2e4. 1919\ub144 1\uc6d4 21\uc77c\uc5d0 \uace0\uc885\uc774 \uc2b9\ud558\ud558\uc790 150\uc77c \ub3d9\uc548 \ubcf5\uc0c1(\u670d\u55aa)\uc744 \ud558\uc600\ub2e4. \uac19\uc740 \ud574 4\uc6d4\uc5d0 \uacbd\uc131\uc885\ub85c\uacf5\ub9bd\uc2ec\uc0c1\uace0\ub4f1\uc18c\ud559\uad50\uc5d0 \uc785\ud559\ud558\uc600\ub2e4. 1922\ub144 \ubd04\uc5d0 \uc885\ub85c\uc18c\ud559\uad50\uc5d0\uc11c \uc2ec\uc0c1\uacfc 3\ud559\ub144 \uacfc\uc815\uc744 \ub9c8\uce58\uace0, \uac19\uc740 \ud574 7\uc6d4 5\uc77c\uc5d0 \uc720\ud559 \uba85\ubd84\uc73c\ub85c \uc77c\ubcf8\uc73c\ub85c \ubcf4\ub0b4\uc838 \ud559\uc2b5\uc6d0 \ucd08\ub4f1\uacfc 4\ud559\ub144\uc5d0 \ud3b8\uc785\ud558\uc600\ub2e4. \uac19\uc740 \ud574 7\uc6d4 22\uc77c\uc5d0 \uc5ec\ub984 \ud734\uac00\ub97c \uc774\uc720\ub85c \uc870\uc120\uc73c\ub85c \uadc0\ud658\ud558\uba74\uc11c \uc624\uc0ac\uce74 \uc5ed\uc744 \ud1b5\uacfc\ud558\uac8c \ub418\uc5c8\ub294\ub370 \uc624\uc0ac\uce74\uc758 \uc870\uc120\uc778\ub4e4\uc774 \uc774\uc6b0\ub97c \ub458\ub7ec\uc2f8\uace0 \uc77c\uc744 \ubaa8\uc758\ud55c\ub2e4\ub294 \uc18c\ubb38\uc774 \ub3cc\uc544 \uc624\uc0ac\uce74 \uc18c\ub124\uc790\ud0a4(\u66fe\u6839\u5d0e) \uacbd\ucc30\uc11c\uc5d0\uc11c \ub2e4\uc218\uc758 \uc870\uc120\uc778\uc744 \uac80\uc18d\ud558\uc600\ub2e4."} {"question": "\ud06c\ub9ac\uc2a4\ud2f4 \uc81c\uc784\uc2a4 \uc2a4\ud29c\uc5b4\ud2b8\uac00 2004\ub144\ub3c4\uc5d0 \uc5f0\uae30\ud55c \uc601\ud654 \uc774\ub984\uc740?", "paragraph": "\ud06c\ub9ac\uc2a4\ud2f4 \uc81c\uc784\uc2a4 \uc2a4\ud29c\uc5b4\ud2b8 (\uc601\uc5b4: Kristen Jaymes Stewart, 1990\ub144 4\uc6d4 9\uc77c ~ )\ub294 \ubbf8\uad6d\uc758 \ubc30\uc6b0, \ubaa8\ub378\uc774\ub2e4. \ub85c\uc2a4 \uc564\uc824\ub808\uc2a4\uc5d0\uc11c \uc5f0\uc608\uacc4\uc5d0\uc11c \uc885\uc0ac\ud558\ub294 \uc9d1\uc548\uc5d0\uc11c \ud0dc\uc5b4\ub09c \uc2a4\ud29c\uc5b4\ud2b8\ub294 1999\ub144\uc5d0 \uc5ec\ub7ec \uc601\ud654\uc5d0\uc11c \ub2e8\uc5ed\uacfc \uc791\uc740 \uc5ed\ud560\ub85c \uc5f0\uae30 \uacbd\ub825\uc744 \uc2dc\uc791\ud588\ub2e4. \uadf8\ub140\ub294 2002\ub144 \uc870\ub514 \ud3ec\uc2a4\ud130\uc758 \ub538\uc744 \uc5f0\uae30\ud55c \uc2a4\ub9b4\ub7ec \uc601\ud654 \u300a\ud328\ub2c9 \ub8f8\u300b\uc73c\ub85c \uc8fc\ubaa9\uc744 \ubc1b\uc558\ub294\ub370, \uadf8\ub140\ub294 \uc80a\uc740 \uc608\uc220\uac00\uc0c1(\uc601 \uc544\ud2f0\uc2a4\ud2b8 \uc5b4\uc6cc\ub4dc)\uc5d0\uc11c \uc7a5\ud3b8 \uc601\ud654 \ubd80\ubd84 \ucd5c\uc6b0\uc218 \uc5ec\uc6b0\uc8fc\uc5f0\uc0c1\uc744 \uc218\uc0c1\ud588\ub2e4. \uadf8\ub140\ub294 \u300a\uc2a4\ud53c\ud06c\u300b(2004), \u300a\uce90\uce58 \ub313 \ud0a4\ub4dc\u300b(2004), \u300a\uc790\ud22c\ub77c - \uc2a4\ud398\uc774\uc2a4 \uc5b4\ub4dc\ubca4\uccd0\u300b(2005), \u300a\uc778 \ub354 \ub79c\ub4dc \uc624\ube0c \uc6b0\uba3c\u300b(2007)\uc5d0 \ucd9c\uc5f0\ud558\uc5ec \ubbf8\uad6d \ubc30\uc6b0 \uc870\ud569\uc0c1(SAG) \ud6c4\ubcf4\ub85c \uc9c0\uba85\ub418\uc5c8\ub2e4. \uadf8\ub140\ub294 \uc804 \uc138\uacc4\uc5d0\uc11c 33\uc5b5 \ub2ec\ub7ec \uc774\uc0c1\uc744 \ubc8c\uc5b4 \ub4e4\uc778 \u300a\ud2b8\uc640\uc77c\ub77c\uc787 \uc601\ud654 \uc2dc\ub9ac\uc988\u300b(2008-12)\uc5d0\uc11c \ubca8\ub77c \uc2a4\uc644\uc744 \uc5f0\uae30\ud558\uc5ec \uad6d\uc81c\uc801\uc73c\ub85c \ub110\ub9ac \uba85\uc131\uc744 \uc5bb\uc5c8\ub2e4.", "answer": "\uc2a4\ud53c\ud06c", "sentence": "\uadf8\ub140\ub294 \u300a \uc2a4\ud53c\ud06c \u300b(2004), \u300a\uce90\uce58 \ub313 \ud0a4\ub4dc\u300b(2004), \u300a\uc790\ud22c\ub77c - \uc2a4\ud398\uc774\uc2a4 \uc5b4\ub4dc\ubca4\uccd0\u300b(2005), \u300a\uc778 \ub354 \ub79c\ub4dc \uc624\ube0c \uc6b0\uba3c\u300b(2007)\uc5d0 \ucd9c\uc5f0\ud558\uc5ec \ubbf8\uad6d \ubc30\uc6b0 \uc870\ud569\uc0c1(SAG) \ud6c4\ubcf4\ub85c \uc9c0\uba85\ub418\uc5c8\ub2e4.", "paragraph_sentence": "\ud06c\ub9ac\uc2a4\ud2f4 \uc81c\uc784\uc2a4 \uc2a4\ud29c\uc5b4\ud2b8 (\uc601\uc5b4: Kristen Jaymes Stewart, 1990\ub144 4\uc6d4 9\uc77c ~ )\ub294 \ubbf8\uad6d\uc758 \ubc30\uc6b0, \ubaa8\ub378\uc774\ub2e4. \ub85c\uc2a4 \uc564\uc824\ub808\uc2a4\uc5d0\uc11c \uc5f0\uc608\uacc4\uc5d0\uc11c \uc885\uc0ac\ud558\ub294 \uc9d1\uc548\uc5d0\uc11c \ud0dc\uc5b4\ub09c \uc2a4\ud29c\uc5b4\ud2b8\ub294 1999\ub144\uc5d0 \uc5ec\ub7ec \uc601\ud654\uc5d0\uc11c \ub2e8\uc5ed\uacfc \uc791\uc740 \uc5ed\ud560\ub85c \uc5f0\uae30 \uacbd\ub825\uc744 \uc2dc\uc791\ud588\ub2e4. \uadf8\ub140\ub294 2002\ub144 \uc870\ub514 \ud3ec\uc2a4\ud130\uc758 \ub538\uc744 \uc5f0\uae30\ud55c \uc2a4\ub9b4\ub7ec \uc601\ud654 \u300a\ud328\ub2c9 \ub8f8\u300b\uc73c\ub85c \uc8fc\ubaa9\uc744 \ubc1b\uc558\ub294\ub370, \uadf8\ub140\ub294 \uc80a\uc740 \uc608\uc220\uac00\uc0c1(\uc601 \uc544\ud2f0\uc2a4\ud2b8 \uc5b4\uc6cc\ub4dc)\uc5d0\uc11c \uc7a5\ud3b8 \uc601\ud654 \ubd80\ubd84 \ucd5c\uc6b0\uc218 \uc5ec\uc6b0\uc8fc\uc5f0\uc0c1\uc744 \uc218\uc0c1\ud588\ub2e4. \uadf8\ub140\ub294 \u300a \uc2a4\ud53c\ud06c \u300b(2004), \u300a\uce90\uce58 \ub313 \ud0a4\ub4dc\u300b(2004), \u300a\uc790\ud22c\ub77c - \uc2a4\ud398\uc774\uc2a4 \uc5b4\ub4dc\ubca4\uccd0\u300b(2005), \u300a\uc778 \ub354 \ub79c\ub4dc \uc624\ube0c \uc6b0\uba3c\u300b(2007)\uc5d0 \ucd9c\uc5f0\ud558\uc5ec \ubbf8\uad6d \ubc30\uc6b0 \uc870\ud569\uc0c1(SAG) \ud6c4\ubcf4\ub85c \uc9c0\uba85\ub418\uc5c8\ub2e4. \uadf8\ub140\ub294 \uc804 \uc138\uacc4\uc5d0\uc11c 33\uc5b5 \ub2ec\ub7ec \uc774\uc0c1\uc744 \ubc8c\uc5b4 \ub4e4\uc778 \u300a\ud2b8\uc640\uc77c\ub77c\uc787 \uc601\ud654 \uc2dc\ub9ac\uc988\u300b(2008-12)\uc5d0\uc11c \ubca8\ub77c \uc2a4\uc644\uc744 \uc5f0\uae30\ud558\uc5ec \uad6d\uc81c\uc801\uc73c\ub85c \ub110\ub9ac \uba85\uc131\uc744 \uc5bb\uc5c8\ub2e4.", "paragraph_answer": "\ud06c\ub9ac\uc2a4\ud2f4 \uc81c\uc784\uc2a4 \uc2a4\ud29c\uc5b4\ud2b8 (\uc601\uc5b4: Kristen Jaymes Stewart, 1990\ub144 4\uc6d4 9\uc77c ~ )\ub294 \ubbf8\uad6d\uc758 \ubc30\uc6b0, \ubaa8\ub378\uc774\ub2e4. \ub85c\uc2a4 \uc564\uc824\ub808\uc2a4\uc5d0\uc11c \uc5f0\uc608\uacc4\uc5d0\uc11c \uc885\uc0ac\ud558\ub294 \uc9d1\uc548\uc5d0\uc11c \ud0dc\uc5b4\ub09c \uc2a4\ud29c\uc5b4\ud2b8\ub294 1999\ub144\uc5d0 \uc5ec\ub7ec \uc601\ud654\uc5d0\uc11c \ub2e8\uc5ed\uacfc \uc791\uc740 \uc5ed\ud560\ub85c \uc5f0\uae30 \uacbd\ub825\uc744 \uc2dc\uc791\ud588\ub2e4. \uadf8\ub140\ub294 2002\ub144 \uc870\ub514 \ud3ec\uc2a4\ud130\uc758 \ub538\uc744 \uc5f0\uae30\ud55c \uc2a4\ub9b4\ub7ec \uc601\ud654 \u300a\ud328\ub2c9 \ub8f8\u300b\uc73c\ub85c \uc8fc\ubaa9\uc744 \ubc1b\uc558\ub294\ub370, \uadf8\ub140\ub294 \uc80a\uc740 \uc608\uc220\uac00\uc0c1(\uc601 \uc544\ud2f0\uc2a4\ud2b8 \uc5b4\uc6cc\ub4dc)\uc5d0\uc11c \uc7a5\ud3b8 \uc601\ud654 \ubd80\ubd84 \ucd5c\uc6b0\uc218 \uc5ec\uc6b0\uc8fc\uc5f0\uc0c1\uc744 \uc218\uc0c1\ud588\ub2e4. \uadf8\ub140\ub294 \u300a \uc2a4\ud53c\ud06c \u300b(2004), \u300a\uce90\uce58 \ub313 \ud0a4\ub4dc\u300b(2004), \u300a\uc790\ud22c\ub77c - \uc2a4\ud398\uc774\uc2a4 \uc5b4\ub4dc\ubca4\uccd0\u300b(2005), \u300a\uc778 \ub354 \ub79c\ub4dc \uc624\ube0c \uc6b0\uba3c\u300b(2007)\uc5d0 \ucd9c\uc5f0\ud558\uc5ec \ubbf8\uad6d \ubc30\uc6b0 \uc870\ud569\uc0c1(SAG) \ud6c4\ubcf4\ub85c \uc9c0\uba85\ub418\uc5c8\ub2e4. \uadf8\ub140\ub294 \uc804 \uc138\uacc4\uc5d0\uc11c 33\uc5b5 \ub2ec\ub7ec \uc774\uc0c1\uc744 \ubc8c\uc5b4 \ub4e4\uc778 \u300a\ud2b8\uc640\uc77c\ub77c\uc787 \uc601\ud654 \uc2dc\ub9ac\uc988\u300b(2008-12)\uc5d0\uc11c \ubca8\ub77c \uc2a4\uc644\uc744 \uc5f0\uae30\ud558\uc5ec \uad6d\uc81c\uc801\uc73c\ub85c \ub110\ub9ac \uba85\uc131\uc744 \uc5bb\uc5c8\ub2e4.", "sentence_answer": "\uadf8\ub140\ub294 \u300a \uc2a4\ud53c\ud06c \u300b(2004), \u300a\uce90\uce58 \ub313 \ud0a4\ub4dc\u300b(2004), \u300a\uc790\ud22c\ub77c - \uc2a4\ud398\uc774\uc2a4 \uc5b4\ub4dc\ubca4\uccd0\u300b(2005), \u300a\uc778 \ub354 \ub79c\ub4dc \uc624\ube0c \uc6b0\uba3c\u300b(2007)\uc5d0 \ucd9c\uc5f0\ud558\uc5ec \ubbf8\uad6d \ubc30\uc6b0 \uc870\ud569\uc0c1(SAG) \ud6c4\ubcf4\ub85c \uc9c0\uba85\ub418\uc5c8\ub2e4.", "paragraph_id": "6212768-0-2", "paragraph_question": "question: \ud06c\ub9ac\uc2a4\ud2f4 \uc81c\uc784\uc2a4 \uc2a4\ud29c\uc5b4\ud2b8\uac00 2004\ub144\ub3c4\uc5d0 \uc5f0\uae30\ud55c \uc601\ud654 \uc774\ub984\uc740?, context: \ud06c\ub9ac\uc2a4\ud2f4 \uc81c\uc784\uc2a4 \uc2a4\ud29c\uc5b4\ud2b8 (\uc601\uc5b4: Kristen Jaymes Stewart, 1990\ub144 4\uc6d4 9\uc77c ~ )\ub294 \ubbf8\uad6d\uc758 \ubc30\uc6b0, \ubaa8\ub378\uc774\ub2e4. \ub85c\uc2a4 \uc564\uc824\ub808\uc2a4\uc5d0\uc11c \uc5f0\uc608\uacc4\uc5d0\uc11c \uc885\uc0ac\ud558\ub294 \uc9d1\uc548\uc5d0\uc11c \ud0dc\uc5b4\ub09c \uc2a4\ud29c\uc5b4\ud2b8\ub294 1999\ub144\uc5d0 \uc5ec\ub7ec \uc601\ud654\uc5d0\uc11c \ub2e8\uc5ed\uacfc \uc791\uc740 \uc5ed\ud560\ub85c \uc5f0\uae30 \uacbd\ub825\uc744 \uc2dc\uc791\ud588\ub2e4. \uadf8\ub140\ub294 2002\ub144 \uc870\ub514 \ud3ec\uc2a4\ud130\uc758 \ub538\uc744 \uc5f0\uae30\ud55c \uc2a4\ub9b4\ub7ec \uc601\ud654 \u300a\ud328\ub2c9 \ub8f8\u300b\uc73c\ub85c \uc8fc\ubaa9\uc744 \ubc1b\uc558\ub294\ub370, \uadf8\ub140\ub294 \uc80a\uc740 \uc608\uc220\uac00\uc0c1(\uc601 \uc544\ud2f0\uc2a4\ud2b8 \uc5b4\uc6cc\ub4dc)\uc5d0\uc11c \uc7a5\ud3b8 \uc601\ud654 \ubd80\ubd84 \ucd5c\uc6b0\uc218 \uc5ec\uc6b0\uc8fc\uc5f0\uc0c1\uc744 \uc218\uc0c1\ud588\ub2e4. \uadf8\ub140\ub294 \u300a\uc2a4\ud53c\ud06c\u300b(2004), \u300a\uce90\uce58 \ub313 \ud0a4\ub4dc\u300b(2004), \u300a\uc790\ud22c\ub77c - \uc2a4\ud398\uc774\uc2a4 \uc5b4\ub4dc\ubca4\uccd0\u300b(2005), \u300a\uc778 \ub354 \ub79c\ub4dc \uc624\ube0c \uc6b0\uba3c\u300b(2007)\uc5d0 \ucd9c\uc5f0\ud558\uc5ec \ubbf8\uad6d \ubc30\uc6b0 \uc870\ud569\uc0c1(SAG) \ud6c4\ubcf4\ub85c \uc9c0\uba85\ub418\uc5c8\ub2e4. \uadf8\ub140\ub294 \uc804 \uc138\uacc4\uc5d0\uc11c 33\uc5b5 \ub2ec\ub7ec \uc774\uc0c1\uc744 \ubc8c\uc5b4 \ub4e4\uc778 \u300a\ud2b8\uc640\uc77c\ub77c\uc787 \uc601\ud654 \uc2dc\ub9ac\uc988\u300b(2008-12)\uc5d0\uc11c \ubca8\ub77c \uc2a4\uc644\uc744 \uc5f0\uae30\ud558\uc5ec \uad6d\uc81c\uc801\uc73c\ub85c \ub110\ub9ac \uba85\uc131\uc744 \uc5bb\uc5c8\ub2e4."} {"question": "10\uc6d4 4\uc77c\uc5d0 \uc5f4\ub9b0 \ud55c\ubbf8FTA\uac1c\uc815 \ud611\uc0c1 \ub17c\uc758\ub97c \uc704\ud55c \uacf5\ub3d9\uc704\uc6d0\ud68c \uc81c2\ucc28 \ud2b9\ubcc4\ud68c\uae30\ub294 \uc5b4\ub514\uc5d0\uc11c \uc5f4\ub838\ub294\uac00?", "paragraph": "\ubbf8\uad6d\uc758 \ud1b5\uc0c1 \uc555\ubc15\uc774 \uac70\uc138\uc9c0\uba74\uc11c 8\uc6d4 22\uc77c\uc5d0\ub294 \uc11c\uc6b8\uc5d0\uc11c \ud55c\ubbf8 FTA \uac1c\uc815\ud611\uc0c1 \ub17c\uc758\ub97c \uc704\ud55c \uacf5\ub3d9\uc704\uc6d0\ud68c \uccab \ud2b9\ubcc4\ud68c\uae30\uac00 \uc5f4\ub838\ub2e4. \ud558\uc9c0\ub9cc \ubbf8\uad6d\uc774 \uac1c\uc815\ud611\uc0c1\uc744 \uacf5\uc2dd \uc694\uad6c\ud55c \ub370 \ub300\ud574 \ud55c\uad6d\uc740 \uc218\uc6a9\ud560 \uc218 \uc5c6\ub2e4\uba70 \uc6b0\uc120 FTA\uc758 \uacbd\uc81c\uc801 \ud6a8\uacfc\ub97c \uba3c\uc800 \ubd84\uc11d\ud574\uc57c \ud55c\ub2e4\uace0 \uc5ed\uc81c\uc548\ud558\ub294 \ub4f1 \uc544\ubb34\ub7f0 \ud611\uc758\ub3c4 \ubabb\ud55c \ucc44 \ubb3c\ub7ec\ub0ac\ub2e4. \uc774\ud6c4 10\uc6d4 4\uc77c \uc6cc\uc2f1\ud134\uc5d0\uc11c 2\ucc28 \ud2b9\ubcc4\ud68c\uae30\ub97c \uc5f4\uc5c8\ub294\ub370 \ud55c\uad6d\uc740 \ud55c\ubbf8 FTA\uc640 \ubbf8\uad6d \ubb34\uc5ed\uc801\uc790\uc758 \uad00\uacc4 \ub4f1\uc744 \ubd84\uc11d\ud55c \uc785\uc7a5 \uc790\ub8cc\uae4c\uc9c0 \ub0b4\ub193\uc73c\uba70 \ubbf8\uad6d\uc744 \uc124\ub4dd\ud588\uc9c0\ub9cc \ubbf8\uad6d\uc758 \uc694\uad6c\ub294 \ubc14\ub00c\uc9c0 \uc54a\uc558\uace0 \uacb0\uad6d \uac1c\uc815\ud611\uc0c1\uc744 \uc2dc\uc791\ud558\uae30\ub85c \uc808\ucc28\uac00 \uc9c4\ud589\ub410\ub2e4. \uc774\ub294 \ud2b8\ub7fc\ud504\uac00 \uc9c0\uc18d\uc801\uc73c\ub85c \ud3d0\uae30 \uac00\ub2a5\uc131\uae4c\uc9c0 \uac70\ub860\ud558\uba74\uc11c \uc555\ubc15\uc758 \uc218\uc704\ub97c \ub192\uc600\uae30 \ub54c\ubb38\uc5d0 2\ucc28 \ud68c\uae30\uac00 \uc5f4\ub9ac\uae30 \uc804\ubd80\ud130 \uac1c\uc815 \uc694\uad6c\ub97c \uc218\uc6a9\ud560 \uc218\ubc16\uc5d0 \uc5c6\uc744 \uac83\uc774\ub780 \uc804\ub9dd\uc774 \ub098\uc624\uae30\ub3c4 \ud588\ub2e4. \uc774\ud6c4 '\ud2b8\ub7fc\ud504\uc758 FTA \ud3d0\uae30 \ud611\ubc15\uc5d0 \ubc00\ub824 \ubc31\uae30\ud22c\ud56d\uc744 \ud588\ub2e4'\ub294 \ubcf4\ub3c4\uac00 \ub098\uc624\uc790 \uccad\uc640\ub300\ub294 \uc801\uadf9\uc801\uc73c\ub85c \ud574\uba85\ud558\uba70 \"\uc9c0\uae08\uc740 \uac1c\uc815\ud611\uc0c1\uc744 \ud558\uae30 \uc704\ud55c \uc808\ucc28 \ud569\uc758\" \uc911\uc774\ub77c\uace0 \ub9d0\ud588\ub2e4.", "answer": "\uc6cc\uc2f1\ud134", "sentence": "\uc774\ud6c4 10\uc6d4 4\uc77c \uc6cc\uc2f1\ud134 \uc5d0\uc11c 2\ucc28 \ud2b9\ubcc4\ud68c\uae30\ub97c \uc5f4\uc5c8\ub294\ub370 \ud55c\uad6d\uc740 \ud55c\ubbf8 FTA\uc640 \ubbf8\uad6d \ubb34\uc5ed\uc801\uc790\uc758 \uad00\uacc4 \ub4f1\uc744 \ubd84\uc11d\ud55c \uc785\uc7a5 \uc790\ub8cc\uae4c\uc9c0 \ub0b4\ub193\uc73c\uba70 \ubbf8\uad6d\uc744 \uc124\ub4dd\ud588\uc9c0\ub9cc \ubbf8\uad6d\uc758 \uc694\uad6c\ub294 \ubc14\ub00c\uc9c0 \uc54a\uc558\uace0 \uacb0\uad6d \uac1c\uc815\ud611\uc0c1\uc744 \uc2dc\uc791\ud558\uae30\ub85c \uc808\ucc28\uac00 \uc9c4\ud589\ub410\ub2e4.", "paragraph_sentence": "\ubbf8\uad6d\uc758 \ud1b5\uc0c1 \uc555\ubc15\uc774 \uac70\uc138\uc9c0\uba74\uc11c 8\uc6d4 22\uc77c\uc5d0\ub294 \uc11c\uc6b8\uc5d0\uc11c \ud55c\ubbf8 FTA \uac1c\uc815\ud611\uc0c1 \ub17c\uc758\ub97c \uc704\ud55c \uacf5\ub3d9\uc704\uc6d0\ud68c \uccab \ud2b9\ubcc4\ud68c\uae30\uac00 \uc5f4\ub838\ub2e4. \ud558\uc9c0\ub9cc \ubbf8\uad6d\uc774 \uac1c\uc815\ud611\uc0c1\uc744 \uacf5\uc2dd \uc694\uad6c\ud55c \ub370 \ub300\ud574 \ud55c\uad6d\uc740 \uc218\uc6a9\ud560 \uc218 \uc5c6\ub2e4\uba70 \uc6b0\uc120 FTA\uc758 \uacbd\uc81c\uc801 \ud6a8\uacfc\ub97c \uba3c\uc800 \ubd84\uc11d\ud574\uc57c \ud55c\ub2e4\uace0 \uc5ed\uc81c\uc548\ud558\ub294 \ub4f1 \uc544\ubb34\ub7f0 \ud611\uc758\ub3c4 \ubabb\ud55c \ucc44 \ubb3c\ub7ec\ub0ac\ub2e4. \uc774\ud6c4 10\uc6d4 4\uc77c \uc6cc\uc2f1\ud134 \uc5d0\uc11c 2\ucc28 \ud2b9\ubcc4\ud68c\uae30\ub97c \uc5f4\uc5c8\ub294\ub370 \ud55c\uad6d\uc740 \ud55c\ubbf8 FTA\uc640 \ubbf8\uad6d \ubb34\uc5ed\uc801\uc790\uc758 \uad00\uacc4 \ub4f1\uc744 \ubd84\uc11d\ud55c \uc785\uc7a5 \uc790\ub8cc\uae4c\uc9c0 \ub0b4\ub193\uc73c\uba70 \ubbf8\uad6d\uc744 \uc124\ub4dd\ud588\uc9c0\ub9cc \ubbf8\uad6d\uc758 \uc694\uad6c\ub294 \ubc14\ub00c\uc9c0 \uc54a\uc558\uace0 \uacb0\uad6d \uac1c\uc815\ud611\uc0c1\uc744 \uc2dc\uc791\ud558\uae30\ub85c \uc808\ucc28\uac00 \uc9c4\ud589\ub410\ub2e4. \uc774\ub294 \ud2b8\ub7fc\ud504\uac00 \uc9c0\uc18d\uc801\uc73c\ub85c \ud3d0\uae30 \uac00\ub2a5\uc131\uae4c\uc9c0 \uac70\ub860\ud558\uba74\uc11c \uc555\ubc15\uc758 \uc218\uc704\ub97c \ub192\uc600\uae30 \ub54c\ubb38\uc5d0 2\ucc28 \ud68c\uae30\uac00 \uc5f4\ub9ac\uae30 \uc804\ubd80\ud130 \uac1c\uc815 \uc694\uad6c\ub97c \uc218\uc6a9\ud560 \uc218\ubc16\uc5d0 \uc5c6\uc744 \uac83\uc774\ub780 \uc804\ub9dd\uc774 \ub098\uc624\uae30\ub3c4 \ud588\ub2e4. \uc774\ud6c4 '\ud2b8\ub7fc\ud504\uc758 FTA \ud3d0\uae30 \ud611\ubc15\uc5d0 \ubc00\ub824 \ubc31\uae30\ud22c\ud56d\uc744 \ud588\ub2e4'\ub294 \ubcf4\ub3c4\uac00 \ub098\uc624\uc790 \uccad\uc640\ub300\ub294 \uc801\uadf9\uc801\uc73c\ub85c \ud574\uba85\ud558\uba70 \"\uc9c0\uae08\uc740 \uac1c\uc815\ud611\uc0c1\uc744 \ud558\uae30 \uc704\ud55c \uc808\ucc28 \ud569\uc758\" \uc911\uc774\ub77c\uace0 \ub9d0\ud588\ub2e4.", "paragraph_answer": "\ubbf8\uad6d\uc758 \ud1b5\uc0c1 \uc555\ubc15\uc774 \uac70\uc138\uc9c0\uba74\uc11c 8\uc6d4 22\uc77c\uc5d0\ub294 \uc11c\uc6b8\uc5d0\uc11c \ud55c\ubbf8 FTA \uac1c\uc815\ud611\uc0c1 \ub17c\uc758\ub97c \uc704\ud55c \uacf5\ub3d9\uc704\uc6d0\ud68c \uccab \ud2b9\ubcc4\ud68c\uae30\uac00 \uc5f4\ub838\ub2e4. \ud558\uc9c0\ub9cc \ubbf8\uad6d\uc774 \uac1c\uc815\ud611\uc0c1\uc744 \uacf5\uc2dd \uc694\uad6c\ud55c \ub370 \ub300\ud574 \ud55c\uad6d\uc740 \uc218\uc6a9\ud560 \uc218 \uc5c6\ub2e4\uba70 \uc6b0\uc120 FTA\uc758 \uacbd\uc81c\uc801 \ud6a8\uacfc\ub97c \uba3c\uc800 \ubd84\uc11d\ud574\uc57c \ud55c\ub2e4\uace0 \uc5ed\uc81c\uc548\ud558\ub294 \ub4f1 \uc544\ubb34\ub7f0 \ud611\uc758\ub3c4 \ubabb\ud55c \ucc44 \ubb3c\ub7ec\ub0ac\ub2e4. \uc774\ud6c4 10\uc6d4 4\uc77c \uc6cc\uc2f1\ud134 \uc5d0\uc11c 2\ucc28 \ud2b9\ubcc4\ud68c\uae30\ub97c \uc5f4\uc5c8\ub294\ub370 \ud55c\uad6d\uc740 \ud55c\ubbf8 FTA\uc640 \ubbf8\uad6d \ubb34\uc5ed\uc801\uc790\uc758 \uad00\uacc4 \ub4f1\uc744 \ubd84\uc11d\ud55c \uc785\uc7a5 \uc790\ub8cc\uae4c\uc9c0 \ub0b4\ub193\uc73c\uba70 \ubbf8\uad6d\uc744 \uc124\ub4dd\ud588\uc9c0\ub9cc \ubbf8\uad6d\uc758 \uc694\uad6c\ub294 \ubc14\ub00c\uc9c0 \uc54a\uc558\uace0 \uacb0\uad6d \uac1c\uc815\ud611\uc0c1\uc744 \uc2dc\uc791\ud558\uae30\ub85c \uc808\ucc28\uac00 \uc9c4\ud589\ub410\ub2e4. \uc774\ub294 \ud2b8\ub7fc\ud504\uac00 \uc9c0\uc18d\uc801\uc73c\ub85c \ud3d0\uae30 \uac00\ub2a5\uc131\uae4c\uc9c0 \uac70\ub860\ud558\uba74\uc11c \uc555\ubc15\uc758 \uc218\uc704\ub97c \ub192\uc600\uae30 \ub54c\ubb38\uc5d0 2\ucc28 \ud68c\uae30\uac00 \uc5f4\ub9ac\uae30 \uc804\ubd80\ud130 \uac1c\uc815 \uc694\uad6c\ub97c \uc218\uc6a9\ud560 \uc218\ubc16\uc5d0 \uc5c6\uc744 \uac83\uc774\ub780 \uc804\ub9dd\uc774 \ub098\uc624\uae30\ub3c4 \ud588\ub2e4. \uc774\ud6c4 '\ud2b8\ub7fc\ud504\uc758 FTA \ud3d0\uae30 \ud611\ubc15\uc5d0 \ubc00\ub824 \ubc31\uae30\ud22c\ud56d\uc744 \ud588\ub2e4'\ub294 \ubcf4\ub3c4\uac00 \ub098\uc624\uc790 \uccad\uc640\ub300\ub294 \uc801\uadf9\uc801\uc73c\ub85c \ud574\uba85\ud558\uba70 \"\uc9c0\uae08\uc740 \uac1c\uc815\ud611\uc0c1\uc744 \ud558\uae30 \uc704\ud55c \uc808\ucc28 \ud569\uc758\" \uc911\uc774\ub77c\uace0 \ub9d0\ud588\ub2e4.", "sentence_answer": "\uc774\ud6c4 10\uc6d4 4\uc77c \uc6cc\uc2f1\ud134 \uc5d0\uc11c 2\ucc28 \ud2b9\ubcc4\ud68c\uae30\ub97c \uc5f4\uc5c8\ub294\ub370 \ud55c\uad6d\uc740 \ud55c\ubbf8 FTA\uc640 \ubbf8\uad6d \ubb34\uc5ed\uc801\uc790\uc758 \uad00\uacc4 \ub4f1\uc744 \ubd84\uc11d\ud55c \uc785\uc7a5 \uc790\ub8cc\uae4c\uc9c0 \ub0b4\ub193\uc73c\uba70 \ubbf8\uad6d\uc744 \uc124\ub4dd\ud588\uc9c0\ub9cc \ubbf8\uad6d\uc758 \uc694\uad6c\ub294 \ubc14\ub00c\uc9c0 \uc54a\uc558\uace0 \uacb0\uad6d \uac1c\uc815\ud611\uc0c1\uc744 \uc2dc\uc791\ud558\uae30\ub85c \uc808\ucc28\uac00 \uc9c4\ud589\ub410\ub2e4.", "paragraph_id": "6580768-83-1", "paragraph_question": "question: 10\uc6d4 4\uc77c\uc5d0 \uc5f4\ub9b0 \ud55c\ubbf8FTA\uac1c\uc815 \ud611\uc0c1 \ub17c\uc758\ub97c \uc704\ud55c \uacf5\ub3d9\uc704\uc6d0\ud68c \uc81c2\ucc28 \ud2b9\ubcc4\ud68c\uae30\ub294 \uc5b4\ub514\uc5d0\uc11c \uc5f4\ub838\ub294\uac00?, context: \ubbf8\uad6d\uc758 \ud1b5\uc0c1 \uc555\ubc15\uc774 \uac70\uc138\uc9c0\uba74\uc11c 8\uc6d4 22\uc77c\uc5d0\ub294 \uc11c\uc6b8\uc5d0\uc11c \ud55c\ubbf8 FTA \uac1c\uc815\ud611\uc0c1 \ub17c\uc758\ub97c \uc704\ud55c \uacf5\ub3d9\uc704\uc6d0\ud68c \uccab \ud2b9\ubcc4\ud68c\uae30\uac00 \uc5f4\ub838\ub2e4. \ud558\uc9c0\ub9cc \ubbf8\uad6d\uc774 \uac1c\uc815\ud611\uc0c1\uc744 \uacf5\uc2dd \uc694\uad6c\ud55c \ub370 \ub300\ud574 \ud55c\uad6d\uc740 \uc218\uc6a9\ud560 \uc218 \uc5c6\ub2e4\uba70 \uc6b0\uc120 FTA\uc758 \uacbd\uc81c\uc801 \ud6a8\uacfc\ub97c \uba3c\uc800 \ubd84\uc11d\ud574\uc57c \ud55c\ub2e4\uace0 \uc5ed\uc81c\uc548\ud558\ub294 \ub4f1 \uc544\ubb34\ub7f0 \ud611\uc758\ub3c4 \ubabb\ud55c \ucc44 \ubb3c\ub7ec\ub0ac\ub2e4. \uc774\ud6c4 10\uc6d4 4\uc77c \uc6cc\uc2f1\ud134\uc5d0\uc11c 2\ucc28 \ud2b9\ubcc4\ud68c\uae30\ub97c \uc5f4\uc5c8\ub294\ub370 \ud55c\uad6d\uc740 \ud55c\ubbf8 FTA\uc640 \ubbf8\uad6d \ubb34\uc5ed\uc801\uc790\uc758 \uad00\uacc4 \ub4f1\uc744 \ubd84\uc11d\ud55c \uc785\uc7a5 \uc790\ub8cc\uae4c\uc9c0 \ub0b4\ub193\uc73c\uba70 \ubbf8\uad6d\uc744 \uc124\ub4dd\ud588\uc9c0\ub9cc \ubbf8\uad6d\uc758 \uc694\uad6c\ub294 \ubc14\ub00c\uc9c0 \uc54a\uc558\uace0 \uacb0\uad6d \uac1c\uc815\ud611\uc0c1\uc744 \uc2dc\uc791\ud558\uae30\ub85c \uc808\ucc28\uac00 \uc9c4\ud589\ub410\ub2e4. \uc774\ub294 \ud2b8\ub7fc\ud504\uac00 \uc9c0\uc18d\uc801\uc73c\ub85c \ud3d0\uae30 \uac00\ub2a5\uc131\uae4c\uc9c0 \uac70\ub860\ud558\uba74\uc11c \uc555\ubc15\uc758 \uc218\uc704\ub97c \ub192\uc600\uae30 \ub54c\ubb38\uc5d0 2\ucc28 \ud68c\uae30\uac00 \uc5f4\ub9ac\uae30 \uc804\ubd80\ud130 \uac1c\uc815 \uc694\uad6c\ub97c \uc218\uc6a9\ud560 \uc218\ubc16\uc5d0 \uc5c6\uc744 \uac83\uc774\ub780 \uc804\ub9dd\uc774 \ub098\uc624\uae30\ub3c4 \ud588\ub2e4. \uc774\ud6c4 '\ud2b8\ub7fc\ud504\uc758 FTA \ud3d0\uae30 \ud611\ubc15\uc5d0 \ubc00\ub824 \ubc31\uae30\ud22c\ud56d\uc744 \ud588\ub2e4'\ub294 \ubcf4\ub3c4\uac00 \ub098\uc624\uc790 \uccad\uc640\ub300\ub294 \uc801\uadf9\uc801\uc73c\ub85c \ud574\uba85\ud558\uba70 \"\uc9c0\uae08\uc740 \uac1c\uc815\ud611\uc0c1\uc744 \ud558\uae30 \uc704\ud55c \uc808\ucc28 \ud569\uc758\" \uc911\uc774\ub77c\uace0 \ub9d0\ud588\ub2e4."} {"question": "\uc774\uc774\ub294 \ub9cc\uc5b8\ubd09\uc0ac\uc5d0\uc11c \uc784\uc9c4\uc65c\ub780\uc744 \uc608\uacac\ud558\uba70 \uc774\uac83\uc5d0 \ub300\ube44\ud558\uc5ec \ubb34\uc5c7\uc744 \ud574\uc57c\ud55c\ub2e4\uace0 \uc8fc\uc7a5\ud558\uc600\ub098?", "paragraph": "\uc774 \ubc16\uc5d0 \uc815\uce58\uc801 \ubd80\ud328\uc758 \ud0c0\uac1c\uc640 \ubc31\uc131\uc758 \uad6c\uc81c\uc5d0 \ub300\ud55c \ubc29\ucc45\uc5d0 \uad00\ud574\uc11c\ub294 \ud55c\uce35 \uad6c\uccb4\uc801\uc778 \ud3ec\ubd80\ub97c \uac00\uc9c0\uace0 \uc788\uc5c8\ub2e4. \u300a\ub9cc\uc5b8\ubd09\uc0ac(\u842c\u8a00\u5c01\u4e8b)\u300b\uc5d0\uc11c \uc728\uace1\uc740 \ubd80\ud328\uc758 \uc2dc\uc815\ucc45 7\uac1c\ud56d\uc744 \uc81c\uc2dc\ud558\uc600\ub294\ub370 \ud2b9\ud788 \uadf8 \uc911 \uc2ed\ub9cc\uc591\ubcd1\uc124\uc744 \uc8fc\uc7a5\ud558\uc5ec \uc784\uc9c4\uc65c\ub780\uc744 \uc608\uc5b8\ud55c \uac83\uc740 \uc720\uba85\ud55c \uc0ac\uc2e4\uc774\ub2e4. \uc774 \ubc16\uc5d0\ub3c4 \ub300\ub3d9\ubc95\uc758 \uc2e4\uc2dc\uc640 \uc0ac\ucc3d\uc758 \uc124\uce58 \ub4f1\uc744 \uc81c\uc758\ud55c \uc77c\uc740 \uc870\uc120 \uc0ac\ud68c \uc815\ucc45\uc5d0 \ud68d\uae30\uc801\uc778 \ud601\uc2e0\uc744 \uac00\uc838\uc624\uac8c \ud558\uc600\uc73c\uba70, \uc77c\ubc18 \ubbfc\uc911\uc758 \uacc4\ubabd\uc744 \uc704\ud558\uc5ec \u300a\uc11c\uc6d0\ud5a5\uc57d(\u897f\u539f\u9115\u7d04)\u300b, \u300a\ud574\uc8fc\ud5a5\uc57d(\u6d77\u5dde\u9115\u85e5)\u300b, \u300a\uc0ac\ucc3d\uacc4\uc57d\uc18d(\u793e\u5009\u5951\u7d04\u675f)\u300b, \u300a\ub3d9\uac70\uacc4\uc0ac(\u540c\u5c45\u6212\u8fad)\u300b, \u300a\ud559\uad50\ubaa8\ubc94(\u5b78\u654e\u6a21\u7bc4)\u300b, \u300a\ud574\uc8fc\uc740\ubcd1\uc815\uc0ac\ud559\uaddc(\u6d77\u5dde\u96b1\u5c5b\u7cbe\u820d\u5b78\u898f)\u300b, \u300a\uc57d\uc18d(\u7d04\u675f)\u300b, \u300a\ubb38\ud5cc\uc11c\uc6d0\ud559\uaddc(\u6587\u737b\u66f8\u9662\u5b78\u898f)\u300b \ub4f1\uc758 \uaddc\ub840\ub97c \ub9ce\uc774 \ub9cc\ub4e4\uc5c8\ub2e4.", "answer": "\uc2ed\ub9cc\uc591\ubcd1\uc124", "sentence": "\u300a\ub9cc\uc5b8\ubd09\uc0ac(\u842c\u8a00\u5c01\u4e8b)\u300b\uc5d0\uc11c \uc728\uace1\uc740 \ubd80\ud328\uc758 \uc2dc\uc815\ucc45 7\uac1c\ud56d\uc744 \uc81c\uc2dc\ud558\uc600\ub294\ub370 \ud2b9\ud788 \uadf8 \uc911 \uc2ed\ub9cc\uc591\ubcd1\uc124 \uc744 \uc8fc\uc7a5\ud558\uc5ec \uc784\uc9c4\uc65c\ub780\uc744 \uc608\uc5b8\ud55c \uac83\uc740 \uc720\uba85\ud55c \uc0ac\uc2e4\uc774\ub2e4.", "paragraph_sentence": "\uc774 \ubc16\uc5d0 \uc815\uce58\uc801 \ubd80\ud328\uc758 \ud0c0\uac1c\uc640 \ubc31\uc131\uc758 \uad6c\uc81c\uc5d0 \ub300\ud55c \ubc29\ucc45\uc5d0 \uad00\ud574\uc11c\ub294 \ud55c\uce35 \uad6c\uccb4\uc801\uc778 \ud3ec\ubd80\ub97c \uac00\uc9c0\uace0 \uc788\uc5c8\ub2e4. \u300a\ub9cc\uc5b8\ubd09\uc0ac(\u842c\u8a00\u5c01\u4e8b)\u300b\uc5d0\uc11c \uc728\uace1\uc740 \ubd80\ud328\uc758 \uc2dc\uc815\ucc45 7\uac1c\ud56d\uc744 \uc81c\uc2dc\ud558\uc600\ub294\ub370 \ud2b9\ud788 \uadf8 \uc911 \uc2ed\ub9cc\uc591\ubcd1\uc124 \uc744 \uc8fc\uc7a5\ud558\uc5ec \uc784\uc9c4\uc65c\ub780\uc744 \uc608\uc5b8\ud55c \uac83\uc740 \uc720\uba85\ud55c \uc0ac\uc2e4\uc774\ub2e4. \uc774 \ubc16\uc5d0\ub3c4 \ub300\ub3d9\ubc95\uc758 \uc2e4\uc2dc\uc640 \uc0ac\ucc3d\uc758 \uc124\uce58 \ub4f1\uc744 \uc81c\uc758\ud55c \uc77c\uc740 \uc870\uc120 \uc0ac\ud68c \uc815\ucc45\uc5d0 \ud68d\uae30\uc801\uc778 \ud601\uc2e0\uc744 \uac00\uc838\uc624\uac8c \ud558\uc600\uc73c\uba70, \uc77c\ubc18 \ubbfc\uc911\uc758 \uacc4\ubabd\uc744 \uc704\ud558\uc5ec \u300a\uc11c\uc6d0\ud5a5\uc57d(\u897f\u539f\u9115\u7d04)\u300b, \u300a\ud574\uc8fc\ud5a5\uc57d(\u6d77\u5dde\u9115\u85e5)\u300b, \u300a\uc0ac\ucc3d\uacc4\uc57d\uc18d(\u793e\u5009\u5951\u7d04\u675f)\u300b, \u300a\ub3d9\uac70\uacc4\uc0ac(\u540c\u5c45\u6212\u8fad)\u300b, \u300a\ud559\uad50\ubaa8\ubc94(\u5b78\u654e\u6a21\u7bc4)\u300b, \u300a\ud574\uc8fc\uc740\ubcd1\uc815\uc0ac\ud559\uaddc(\u6d77\u5dde\u96b1\u5c5b\u7cbe\u820d\u5b78\u898f)\u300b, \u300a\uc57d\uc18d(\u7d04\u675f)\u300b, \u300a\ubb38\ud5cc\uc11c\uc6d0\ud559\uaddc(\u6587\u737b\u66f8\u9662\u5b78\u898f)\u300b \ub4f1\uc758 \uaddc\ub840\ub97c \ub9ce\uc774 \ub9cc\ub4e4\uc5c8\ub2e4.", "paragraph_answer": "\uc774 \ubc16\uc5d0 \uc815\uce58\uc801 \ubd80\ud328\uc758 \ud0c0\uac1c\uc640 \ubc31\uc131\uc758 \uad6c\uc81c\uc5d0 \ub300\ud55c \ubc29\ucc45\uc5d0 \uad00\ud574\uc11c\ub294 \ud55c\uce35 \uad6c\uccb4\uc801\uc778 \ud3ec\ubd80\ub97c \uac00\uc9c0\uace0 \uc788\uc5c8\ub2e4. \u300a\ub9cc\uc5b8\ubd09\uc0ac(\u842c\u8a00\u5c01\u4e8b)\u300b\uc5d0\uc11c \uc728\uace1\uc740 \ubd80\ud328\uc758 \uc2dc\uc815\ucc45 7\uac1c\ud56d\uc744 \uc81c\uc2dc\ud558\uc600\ub294\ub370 \ud2b9\ud788 \uadf8 \uc911 \uc2ed\ub9cc\uc591\ubcd1\uc124 \uc744 \uc8fc\uc7a5\ud558\uc5ec \uc784\uc9c4\uc65c\ub780\uc744 \uc608\uc5b8\ud55c \uac83\uc740 \uc720\uba85\ud55c \uc0ac\uc2e4\uc774\ub2e4. \uc774 \ubc16\uc5d0\ub3c4 \ub300\ub3d9\ubc95\uc758 \uc2e4\uc2dc\uc640 \uc0ac\ucc3d\uc758 \uc124\uce58 \ub4f1\uc744 \uc81c\uc758\ud55c \uc77c\uc740 \uc870\uc120 \uc0ac\ud68c \uc815\ucc45\uc5d0 \ud68d\uae30\uc801\uc778 \ud601\uc2e0\uc744 \uac00\uc838\uc624\uac8c \ud558\uc600\uc73c\uba70, \uc77c\ubc18 \ubbfc\uc911\uc758 \uacc4\ubabd\uc744 \uc704\ud558\uc5ec \u300a\uc11c\uc6d0\ud5a5\uc57d(\u897f\u539f\u9115\u7d04)\u300b, \u300a\ud574\uc8fc\ud5a5\uc57d(\u6d77\u5dde\u9115\u85e5)\u300b, \u300a\uc0ac\ucc3d\uacc4\uc57d\uc18d(\u793e\u5009\u5951\u7d04\u675f)\u300b, \u300a\ub3d9\uac70\uacc4\uc0ac(\u540c\u5c45\u6212\u8fad)\u300b, \u300a\ud559\uad50\ubaa8\ubc94(\u5b78\u654e\u6a21\u7bc4)\u300b, \u300a\ud574\uc8fc\uc740\ubcd1\uc815\uc0ac\ud559\uaddc(\u6d77\u5dde\u96b1\u5c5b\u7cbe\u820d\u5b78\u898f)\u300b, \u300a\uc57d\uc18d(\u7d04\u675f)\u300b, \u300a\ubb38\ud5cc\uc11c\uc6d0\ud559\uaddc(\u6587\u737b\u66f8\u9662\u5b78\u898f)\u300b \ub4f1\uc758 \uaddc\ub840\ub97c \ub9ce\uc774 \ub9cc\ub4e4\uc5c8\ub2e4.", "sentence_answer": "\u300a\ub9cc\uc5b8\ubd09\uc0ac(\u842c\u8a00\u5c01\u4e8b)\u300b\uc5d0\uc11c \uc728\uace1\uc740 \ubd80\ud328\uc758 \uc2dc\uc815\ucc45 7\uac1c\ud56d\uc744 \uc81c\uc2dc\ud558\uc600\ub294\ub370 \ud2b9\ud788 \uadf8 \uc911 \uc2ed\ub9cc\uc591\ubcd1\uc124 \uc744 \uc8fc\uc7a5\ud558\uc5ec \uc784\uc9c4\uc65c\ub780\uc744 \uc608\uc5b8\ud55c \uac83\uc740 \uc720\uba85\ud55c \uc0ac\uc2e4\uc774\ub2e4.", "paragraph_id": "6545887-13-1", "paragraph_question": "question: \uc774\uc774\ub294 \ub9cc\uc5b8\ubd09\uc0ac\uc5d0\uc11c \uc784\uc9c4\uc65c\ub780\uc744 \uc608\uacac\ud558\uba70 \uc774\uac83\uc5d0 \ub300\ube44\ud558\uc5ec \ubb34\uc5c7\uc744 \ud574\uc57c\ud55c\ub2e4\uace0 \uc8fc\uc7a5\ud558\uc600\ub098?, context: \uc774 \ubc16\uc5d0 \uc815\uce58\uc801 \ubd80\ud328\uc758 \ud0c0\uac1c\uc640 \ubc31\uc131\uc758 \uad6c\uc81c\uc5d0 \ub300\ud55c \ubc29\ucc45\uc5d0 \uad00\ud574\uc11c\ub294 \ud55c\uce35 \uad6c\uccb4\uc801\uc778 \ud3ec\ubd80\ub97c \uac00\uc9c0\uace0 \uc788\uc5c8\ub2e4. \u300a\ub9cc\uc5b8\ubd09\uc0ac(\u842c\u8a00\u5c01\u4e8b)\u300b\uc5d0\uc11c \uc728\uace1\uc740 \ubd80\ud328\uc758 \uc2dc\uc815\ucc45 7\uac1c\ud56d\uc744 \uc81c\uc2dc\ud558\uc600\ub294\ub370 \ud2b9\ud788 \uadf8 \uc911 \uc2ed\ub9cc\uc591\ubcd1\uc124\uc744 \uc8fc\uc7a5\ud558\uc5ec \uc784\uc9c4\uc65c\ub780\uc744 \uc608\uc5b8\ud55c \uac83\uc740 \uc720\uba85\ud55c \uc0ac\uc2e4\uc774\ub2e4. \uc774 \ubc16\uc5d0\ub3c4 \ub300\ub3d9\ubc95\uc758 \uc2e4\uc2dc\uc640 \uc0ac\ucc3d\uc758 \uc124\uce58 \ub4f1\uc744 \uc81c\uc758\ud55c \uc77c\uc740 \uc870\uc120 \uc0ac\ud68c \uc815\ucc45\uc5d0 \ud68d\uae30\uc801\uc778 \ud601\uc2e0\uc744 \uac00\uc838\uc624\uac8c \ud558\uc600\uc73c\uba70, \uc77c\ubc18 \ubbfc\uc911\uc758 \uacc4\ubabd\uc744 \uc704\ud558\uc5ec \u300a\uc11c\uc6d0\ud5a5\uc57d(\u897f\u539f\u9115\u7d04)\u300b, \u300a\ud574\uc8fc\ud5a5\uc57d(\u6d77\u5dde\u9115\u85e5)\u300b, \u300a\uc0ac\ucc3d\uacc4\uc57d\uc18d(\u793e\u5009\u5951\u7d04\u675f)\u300b, \u300a\ub3d9\uac70\uacc4\uc0ac(\u540c\u5c45\u6212\u8fad)\u300b, \u300a\ud559\uad50\ubaa8\ubc94(\u5b78\u654e\u6a21\u7bc4)\u300b, \u300a\ud574\uc8fc\uc740\ubcd1\uc815\uc0ac\ud559\uaddc(\u6d77\u5dde\u96b1\u5c5b\u7cbe\u820d\u5b78\u898f)\u300b, \u300a\uc57d\uc18d(\u7d04\u675f)\u300b, \u300a\ubb38\ud5cc\uc11c\uc6d0\ud559\uaddc(\u6587\u737b\u66f8\u9662\u5b78\u898f)\u300b \ub4f1\uc758 \uaddc\ub840\ub97c \ub9ce\uc774 \ub9cc\ub4e4\uc5c8\ub2e4."} {"question": "2014\ub144 IOC \ucd1d\ud68c\ub2f9\uc2dc \uba87\uc77c\uac04 \ucd1d\ud68c\uac00 \uc5f4\ub838\ub098?", "paragraph": "2014\ub144 12\uc6d4 7\uc77c \ud1a0\ub9c8\uc2a4 \ubc14\ud750 IOC \uc704\uc6d0\uc7a5\uc740 \u201c8\uc77c\ubd80\ud130 \uc774\ud2c0\uac04 \uc5f4\ub9ac\ub294 IOC \ucd1d\ud68c\uc5d0\uc11c \u2018\uc5b4\uc820\ub2e4 2020\u2019\uc774 \ud655\uc815\ub418\uba74 2018\ub144\uacfc 2020\ub144 \ub3d9\u00b7\ud558\uacc4 \uc62c\ub9bc\ud53d\uc744 \uce58\ub974\ub294 \ud55c\uad6d\uacfc \uc77c\ubcf8\uc774 \uc77c\ubd80 \uc885\ubaa9\uc744 \ubd84\uc0b0 \uac1c\ucd5c\ud560 \uc218 \uc788\ub2e4\u201d\uace0 \ub9d0\ud588\ub2e4. IOC(\uad6d\uc81c\uc62c\ub9bc\ud53d\uc704\uc6d0\ud68c)\ub294 2014\ub144 12\uc6d4 8\uc77c(\ud55c\uad6d \uc2dc\uac04) \uc81c 127\ud68c \ucd1d\ud68c\uc5d0\uc11c \uc0c8\ub85c\uc6b4 \uac1c\ud601\uc548\uc774 \ub2f4\uae34 'Olympics Agenda 2020(\uc5b4\uc820\ub2e4 2020)'\uc744 \ucc44\ud0dd\ud558\uc600\ub2e4. \uc0c8\ub85c\uc6b4 \uac1c\ud601\uc548\uc740 \uae30\uc874\uc758 \uc62c\ub9bc\ud53d \uac1c\ucd5c \ubc29\uc2dd\uacfc \ub2ec\ub9ac \uad6d\ub0b4 \uc5ec\ub7ec \ub3c4\uc2dc\ub4e4\uacfc \ubd84\uc0b0 \uac1c\ucd5c\uac00 \uac00\ub2a5\ud558\uba70, \ub2e4\ub978 \uad6d\uac00\uc758 \ub3c4\uc2dc\uc5d0\uc11c\ub3c4 \ubd84\uc0b0 \uac1c\ucd5c\ub97c \ud560 \uc218 \uc788\ub2e4\ub294 \uac83\uc774\ub2e4. \ub610 \uc2e0\ucd95 \uacbd\uae30\uc7a5\uc774 \uc544\ub2cc \uae30\uc874\uc758 \uc2dc\uc124\ubb3c\uc744 \ubcf4\uc644\ud558\uac70\ub098 \ucca0\uac70 \uc608\uc815\uc778 \uc2dc\uc124\ubb3c\ub4e4\uc5d0\uc11c\ub3c4 \uacbd\uae30\ub97c \uce58\ub97c \uc218 \uc788\ub2e4.", "answer": "\uc774\ud2c0", "sentence": "2014\ub144 12\uc6d4 7\uc77c \ud1a0\ub9c8\uc2a4 \ubc14\ud750 IOC \uc704\uc6d0\uc7a5\uc740 \u201c8\uc77c\ubd80\ud130 \uc774\ud2c0 \uac04 \uc5f4\ub9ac\ub294 IOC \ucd1d\ud68c\uc5d0\uc11c \u2018\uc5b4\uc820\ub2e4 2020\u2019\uc774 \ud655\uc815\ub418\uba74 2018\ub144\uacfc 2020\ub144 \ub3d9\u00b7\ud558\uacc4 \uc62c\ub9bc\ud53d\uc744 \uce58\ub974\ub294 \ud55c\uad6d\uacfc \uc77c\ubcf8\uc774 \uc77c\ubd80 \uc885\ubaa9\uc744 \ubd84\uc0b0 \uac1c\ucd5c\ud560 \uc218 \uc788\ub2e4\u201d\uace0 \ub9d0\ud588\ub2e4.", "paragraph_sentence": " 2014\ub144 12\uc6d4 7\uc77c \ud1a0\ub9c8\uc2a4 \ubc14\ud750 IOC \uc704\uc6d0\uc7a5\uc740 \u201c8\uc77c\ubd80\ud130 \uc774\ud2c0 \uac04 \uc5f4\ub9ac\ub294 IOC \ucd1d\ud68c\uc5d0\uc11c \u2018\uc5b4\uc820\ub2e4 2020\u2019\uc774 \ud655\uc815\ub418\uba74 2018\ub144\uacfc 2020\ub144 \ub3d9\u00b7\ud558\uacc4 \uc62c\ub9bc\ud53d\uc744 \uce58\ub974\ub294 \ud55c\uad6d\uacfc \uc77c\ubcf8\uc774 \uc77c\ubd80 \uc885\ubaa9\uc744 \ubd84\uc0b0 \uac1c\ucd5c\ud560 \uc218 \uc788\ub2e4\u201d\uace0 \ub9d0\ud588\ub2e4. IOC(\uad6d\uc81c\uc62c\ub9bc\ud53d\uc704\uc6d0\ud68c)\ub294 2014\ub144 12\uc6d4 8\uc77c(\ud55c\uad6d \uc2dc\uac04) \uc81c 127\ud68c \ucd1d\ud68c\uc5d0\uc11c \uc0c8\ub85c\uc6b4 \uac1c\ud601\uc548\uc774 \ub2f4\uae34 'Olympics Agenda 2020(\uc5b4\uc820\ub2e4 2020)'\uc744 \ucc44\ud0dd\ud558\uc600\ub2e4. \uc0c8\ub85c\uc6b4 \uac1c\ud601\uc548\uc740 \uae30\uc874\uc758 \uc62c\ub9bc\ud53d \uac1c\ucd5c \ubc29\uc2dd\uacfc \ub2ec\ub9ac \uad6d\ub0b4 \uc5ec\ub7ec \ub3c4\uc2dc\ub4e4\uacfc \ubd84\uc0b0 \uac1c\ucd5c\uac00 \uac00\ub2a5\ud558\uba70, \ub2e4\ub978 \uad6d\uac00\uc758 \ub3c4\uc2dc\uc5d0\uc11c\ub3c4 \ubd84\uc0b0 \uac1c\ucd5c\ub97c \ud560 \uc218 \uc788\ub2e4\ub294 \uac83\uc774\ub2e4. \ub610 \uc2e0\ucd95 \uacbd\uae30\uc7a5\uc774 \uc544\ub2cc \uae30\uc874\uc758 \uc2dc\uc124\ubb3c\uc744 \ubcf4\uc644\ud558\uac70\ub098 \ucca0\uac70 \uc608\uc815\uc778 \uc2dc\uc124\ubb3c\ub4e4\uc5d0\uc11c\ub3c4 \uacbd\uae30\ub97c \uce58\ub97c \uc218 \uc788\ub2e4.", "paragraph_answer": "2014\ub144 12\uc6d4 7\uc77c \ud1a0\ub9c8\uc2a4 \ubc14\ud750 IOC \uc704\uc6d0\uc7a5\uc740 \u201c8\uc77c\ubd80\ud130 \uc774\ud2c0 \uac04 \uc5f4\ub9ac\ub294 IOC \ucd1d\ud68c\uc5d0\uc11c \u2018\uc5b4\uc820\ub2e4 2020\u2019\uc774 \ud655\uc815\ub418\uba74 2018\ub144\uacfc 2020\ub144 \ub3d9\u00b7\ud558\uacc4 \uc62c\ub9bc\ud53d\uc744 \uce58\ub974\ub294 \ud55c\uad6d\uacfc \uc77c\ubcf8\uc774 \uc77c\ubd80 \uc885\ubaa9\uc744 \ubd84\uc0b0 \uac1c\ucd5c\ud560 \uc218 \uc788\ub2e4\u201d\uace0 \ub9d0\ud588\ub2e4. IOC(\uad6d\uc81c\uc62c\ub9bc\ud53d\uc704\uc6d0\ud68c)\ub294 2014\ub144 12\uc6d4 8\uc77c(\ud55c\uad6d \uc2dc\uac04) \uc81c 127\ud68c \ucd1d\ud68c\uc5d0\uc11c \uc0c8\ub85c\uc6b4 \uac1c\ud601\uc548\uc774 \ub2f4\uae34 'Olympics Agenda 2020(\uc5b4\uc820\ub2e4 2020)'\uc744 \ucc44\ud0dd\ud558\uc600\ub2e4. \uc0c8\ub85c\uc6b4 \uac1c\ud601\uc548\uc740 \uae30\uc874\uc758 \uc62c\ub9bc\ud53d \uac1c\ucd5c \ubc29\uc2dd\uacfc \ub2ec\ub9ac \uad6d\ub0b4 \uc5ec\ub7ec \ub3c4\uc2dc\ub4e4\uacfc \ubd84\uc0b0 \uac1c\ucd5c\uac00 \uac00\ub2a5\ud558\uba70, \ub2e4\ub978 \uad6d\uac00\uc758 \ub3c4\uc2dc\uc5d0\uc11c\ub3c4 \ubd84\uc0b0 \uac1c\ucd5c\ub97c \ud560 \uc218 \uc788\ub2e4\ub294 \uac83\uc774\ub2e4. \ub610 \uc2e0\ucd95 \uacbd\uae30\uc7a5\uc774 \uc544\ub2cc \uae30\uc874\uc758 \uc2dc\uc124\ubb3c\uc744 \ubcf4\uc644\ud558\uac70\ub098 \ucca0\uac70 \uc608\uc815\uc778 \uc2dc\uc124\ubb3c\ub4e4\uc5d0\uc11c\ub3c4 \uacbd\uae30\ub97c \uce58\ub97c \uc218 \uc788\ub2e4.", "sentence_answer": "2014\ub144 12\uc6d4 7\uc77c \ud1a0\ub9c8\uc2a4 \ubc14\ud750 IOC \uc704\uc6d0\uc7a5\uc740 \u201c8\uc77c\ubd80\ud130 \uc774\ud2c0 \uac04 \uc5f4\ub9ac\ub294 IOC \ucd1d\ud68c\uc5d0\uc11c \u2018\uc5b4\uc820\ub2e4 2020\u2019\uc774 \ud655\uc815\ub418\uba74 2018\ub144\uacfc 2020\ub144 \ub3d9\u00b7\ud558\uacc4 \uc62c\ub9bc\ud53d\uc744 \uce58\ub974\ub294 \ud55c\uad6d\uacfc \uc77c\ubcf8\uc774 \uc77c\ubd80 \uc885\ubaa9\uc744 \ubd84\uc0b0 \uac1c\ucd5c\ud560 \uc218 \uc788\ub2e4\u201d\uace0 \ub9d0\ud588\ub2e4.", "paragraph_id": "6469594-2-1", "paragraph_question": "question: 2014\ub144 IOC \ucd1d\ud68c\ub2f9\uc2dc \uba87\uc77c\uac04 \ucd1d\ud68c\uac00 \uc5f4\ub838\ub098?, context: 2014\ub144 12\uc6d4 7\uc77c \ud1a0\ub9c8\uc2a4 \ubc14\ud750 IOC \uc704\uc6d0\uc7a5\uc740 \u201c8\uc77c\ubd80\ud130 \uc774\ud2c0\uac04 \uc5f4\ub9ac\ub294 IOC \ucd1d\ud68c\uc5d0\uc11c \u2018\uc5b4\uc820\ub2e4 2020\u2019\uc774 \ud655\uc815\ub418\uba74 2018\ub144\uacfc 2020\ub144 \ub3d9\u00b7\ud558\uacc4 \uc62c\ub9bc\ud53d\uc744 \uce58\ub974\ub294 \ud55c\uad6d\uacfc \uc77c\ubcf8\uc774 \uc77c\ubd80 \uc885\ubaa9\uc744 \ubd84\uc0b0 \uac1c\ucd5c\ud560 \uc218 \uc788\ub2e4\u201d\uace0 \ub9d0\ud588\ub2e4. IOC(\uad6d\uc81c\uc62c\ub9bc\ud53d\uc704\uc6d0\ud68c)\ub294 2014\ub144 12\uc6d4 8\uc77c(\ud55c\uad6d \uc2dc\uac04) \uc81c 127\ud68c \ucd1d\ud68c\uc5d0\uc11c \uc0c8\ub85c\uc6b4 \uac1c\ud601\uc548\uc774 \ub2f4\uae34 'Olympics Agenda 2020(\uc5b4\uc820\ub2e4 2020)'\uc744 \ucc44\ud0dd\ud558\uc600\ub2e4. \uc0c8\ub85c\uc6b4 \uac1c\ud601\uc548\uc740 \uae30\uc874\uc758 \uc62c\ub9bc\ud53d \uac1c\ucd5c \ubc29\uc2dd\uacfc \ub2ec\ub9ac \uad6d\ub0b4 \uc5ec\ub7ec \ub3c4\uc2dc\ub4e4\uacfc \ubd84\uc0b0 \uac1c\ucd5c\uac00 \uac00\ub2a5\ud558\uba70, \ub2e4\ub978 \uad6d\uac00\uc758 \ub3c4\uc2dc\uc5d0\uc11c\ub3c4 \ubd84\uc0b0 \uac1c\ucd5c\ub97c \ud560 \uc218 \uc788\ub2e4\ub294 \uac83\uc774\ub2e4. \ub610 \uc2e0\ucd95 \uacbd\uae30\uc7a5\uc774 \uc544\ub2cc \uae30\uc874\uc758 \uc2dc\uc124\ubb3c\uc744 \ubcf4\uc644\ud558\uac70\ub098 \ucca0\uac70 \uc608\uc815\uc778 \uc2dc\uc124\ubb3c\ub4e4\uc5d0\uc11c\ub3c4 \uacbd\uae30\ub97c \uce58\ub97c \uc218 \uc788\ub2e4."} {"question": "\uc6d0\ub354\uac78\uc2a4\uac00 \uccab \uc815\uaddc \uc568\ubc94 \ud65c\ub3d9\uc744 \ub9c8\uce5c \uc2dc\uae30\ub294?", "paragraph": "\uc6d0\ub354\uac78\uc2a4\ub294 2008\ub144 1\uc6d4 \ucd08 \uccab \uc815\uaddc \uc568\ubc94 \ud65c\ub3d9\uc744 \uc131\uacf5\uc801\uc73c\ub85c \ub9c8\uce58\uace0 \uacf5\ubc31\uae30\uc5d0 \ub4e4\uc5b4\uac14\ub2e4. \uc774 \uae30\uac04\uc5d0 \uc6d0\ub354\uac78\uc2a4\ub294 \ubbf8\uad6d\uc5d0\uc11c MTV \u300a\uc6d0\ub354\uac78\uc2a4 \uc2dc\uc98c3\u300b\ub97c \ucd2c\uc601\ud558\uba74\uc11c \ub3d9\uc2dc\uc5d0 \ubc15\uc9c4\uc601\uacfc JYP \uc5d4\ud130\ud14c\uc778\uba3c\ud2b8 \uc18c\uc18d \uac00\uc218\ub4e4\uc774 \ud568\uaed8\ud55c \u2018The JYP Tour\u2019 \ubb34\ub300\uc5d0 \uc624\ub974\uace0, \ud0c0\uc774\uc5d0\uc11c 1\ub9cc\uc5ec \uba85\uc758 \ud32c\ub4e4\uacfc \ud32c \ubbf8\ud305\uc744 \ud558\ub294 \ub4f1 \uad6d\uc678 \ud65c\ub3d9\uc5d0 \uc8fc\ub825\ud558\ub294 \ud55c\ud3b8, \uad6d\ub0b4\uc758 \uac01\uc885 \ucd95\uc81c\uc640 \ud589\uc0ac \ud65c\ub3d9 \ub610\ud55c \uc26c\uc9c0 \uc54a\uc558\ub2e4. 4\uac1c\uc6d4\uac04\uc758 \uacf5\ubc31\uae30\ub97c \uac70\uce5c \ud6c4, \uc18c\uc18d\uc0ac\ub294 \uc0c8 \uace1 \uacf5\uac1c\ub97c \uc55e\ub450\uace0 \ud558\ub8e8\uc5d0 \ud55c \uc7a5\uc529 \uba64\ubc84\ub4e4\uc758 \ucf58\uc149\ud2b8 \uc0ac\uc9c4\uc744 \uacf5\uac1c\ud558\uba70 \ub300\uc911\ub4e4\uc758 \uad00\uc2ec\uc744 \ub04c\uc5c8\ub2e4. \ub9c8\uc9c0\ub9c9 \ub2e8\uccb4 \uc0ac\uc9c4\uc774 \uacf5\uac1c\ub41c \ub2e4\uc74c\ub0a0\uc778 2008\ub144 5\uc6d4 22\uc77c, \ub450 \ubc88\uc9f8 \uc2f1\uae00 So Hot\uc758 \ud0c0\uc774\ud2c0\uace1 \"So Hot\"\uacfc \ubba4\uc9c1\ube44\ub514\uc624\ub97c \uc628\ub77c\uc778\uc73c\ub85c \uacf5\uac1c\ud588\ub2e4. \ud0c0\uc774\ud2c0 \uace1 \"So Hot\"\uc740 \uc790\uc2e0\uc758 \uc608\uc05c \uc678\ubaa8\uc640 \uc778\uae30\uac00 \ud53c\uace4\ud558\ub2e4\ub294 \ub0b4\uc6a9\uc758 \u201821\uc138\uae30\ud615 \uacf5\uc8fc\ubcd1\u2019\uc744 \ud45c\ubc29\ud55c \uac00\uc0ac\uc640 \ud638\ud53c\ubb34\ub2ac \uc758\uc0c1, \uadf8\ub9ac\uace0 \u2018V \ub77c\uc778 \ucda4\u2019, \u2018\uae30\uc9c0\uac1c \ucda4\u2019 \ub4f1\uc758 \uc548\ubb34\uac00 \ud2b9\uc9d5\uc801\uc778 \uace1\uc73c\ub85c, \uc568\ubc94\uc740 6\uc6d4 3\uc77c \uc815\uc2dd \ubc1c\ub9e4\ub418\uc5c8\ub2e4. \uc628\ub77c\uc778 \uacf5\uac1c \uc774\ud2c0 \ub9cc\uc5d0 \uba5c\ub860, \ub3c4\uc2dc\ub77d, \uc96c\ud06c\uc628, \ubba4\uc988, \uc2f8\uc774\uc6d4\ub4dc, \uc5e0\ub137\ub2f7\ucef4 \ub4f1 \uc628\ub77c\uc778 \ucc28\ud2b8\uc5d0\uc11c 1\uc704\ub97c \uc11d\uad8c\ud558\uace0, \ud154\ub808\ube44\uc804 \uac00\uc694 \ud504\ub85c\uadf8\ub7a8\uc5d0\uc11c\ub294 \uccab \ubb34\ub300\ubd80\ud130 1\uc704\ub97c \ucc28\uc9c0\ud558\uc5ec KBS \u300a\ubba4\uc9c1\ubc45\ud06c\u300b\uc5d0\uc11c\ub294 5\uc8fc \uc5f0\uc18d 1\uc704, SBS \u300a\uc778\uae30\uac00\uc694\u300b\uc640 Mnet \u300aM! Countdown\u300b\uc5d0\uc11c\ub294 3\uc8fc \uc5f0\uc18d 1\uc704\ub97c \ucc28\uc9c0\ud558\ub294 \ub4f1 \ud65c\ub3d9 \ucd08\ubc18\ubd80\ud130 \ud3ed\ubc1c\uc801\uc778 \ubc18\uc751\uc744 \ubd88\ub7ec\uc77c\uc73c\ucf30\uace0, \"Tell Me\" \ud65c\ub3d9 \uc2dc\uae30\uc640 \ub9c8\ucc2c\uac00\uc9c0\ub85c \uc218\ub9ce\uc740 \uc5f0\uc608\uc778\uc774 \uc774 \ucda4\uc744 \ub530\ub77c\ud558\uc600\ub2e4. \ub3d9\uc601\uc0c1 \uc81c\uacf5 \uc6f9\uc0ac\uc774\ud2b8 \uc720\ud29c\ube0c\uc5d0\uc11c\ub294 \ubba4\uc9c1\ube44\ub514\uc624\uac00 \ud55c\uad6d\ub0b4 \ub2e8\uc77c \uc601\uc0c1\uc73c\ub85c\ub294 \uc774\ub840\uc801\uc73c\ub85c \uc870\ud68c\uc218 100\ub9cc\uac74\uc744 \ub118\uae30\uba70 \ub178\ub798\ub97c \uac01\uad6d\uc5d0 \uc54c\ub838\ub2e4.", "answer": "2008\ub144 1\uc6d4 \ucd08", "sentence": "\uc6d0\ub354\uac78\uc2a4\ub294 2008\ub144 1\uc6d4 \ucd08 \uccab \uc815\uaddc \uc568\ubc94 \ud65c\ub3d9\uc744 \uc131\uacf5\uc801\uc73c\ub85c \ub9c8\uce58\uace0 \uacf5\ubc31\uae30\uc5d0 \ub4e4\uc5b4\uac14\ub2e4.", "paragraph_sentence": " \uc6d0\ub354\uac78\uc2a4\ub294 2008\ub144 1\uc6d4 \ucd08 \uccab \uc815\uaddc \uc568\ubc94 \ud65c\ub3d9\uc744 \uc131\uacf5\uc801\uc73c\ub85c \ub9c8\uce58\uace0 \uacf5\ubc31\uae30\uc5d0 \ub4e4\uc5b4\uac14\ub2e4. \uc774 \uae30\uac04\uc5d0 \uc6d0\ub354\uac78\uc2a4\ub294 \ubbf8\uad6d\uc5d0\uc11c MTV \u300a\uc6d0\ub354\uac78\uc2a4 \uc2dc\uc98c3\u300b\ub97c \ucd2c\uc601\ud558\uba74\uc11c \ub3d9\uc2dc\uc5d0 \ubc15\uc9c4\uc601\uacfc JYP \uc5d4\ud130\ud14c\uc778\uba3c\ud2b8 \uc18c\uc18d \uac00\uc218\ub4e4\uc774 \ud568\uaed8\ud55c \u2018The JYP Tour\u2019 \ubb34\ub300\uc5d0 \uc624\ub974\uace0, \ud0c0\uc774\uc5d0\uc11c 1\ub9cc\uc5ec \uba85\uc758 \ud32c\ub4e4\uacfc \ud32c \ubbf8\ud305\uc744 \ud558\ub294 \ub4f1 \uad6d\uc678 \ud65c\ub3d9\uc5d0 \uc8fc\ub825\ud558\ub294 \ud55c\ud3b8, \uad6d\ub0b4\uc758 \uac01\uc885 \ucd95\uc81c\uc640 \ud589\uc0ac \ud65c\ub3d9 \ub610\ud55c \uc26c\uc9c0 \uc54a\uc558\ub2e4. 4\uac1c\uc6d4\uac04\uc758 \uacf5\ubc31\uae30\ub97c \uac70\uce5c \ud6c4, \uc18c\uc18d\uc0ac\ub294 \uc0c8 \uace1 \uacf5\uac1c\ub97c \uc55e\ub450\uace0 \ud558\ub8e8\uc5d0 \ud55c \uc7a5\uc529 \uba64\ubc84\ub4e4\uc758 \ucf58\uc149\ud2b8 \uc0ac\uc9c4\uc744 \uacf5\uac1c\ud558\uba70 \ub300\uc911\ub4e4\uc758 \uad00\uc2ec\uc744 \ub04c\uc5c8\ub2e4. \ub9c8\uc9c0\ub9c9 \ub2e8\uccb4 \uc0ac\uc9c4\uc774 \uacf5\uac1c\ub41c \ub2e4\uc74c\ub0a0\uc778 2008\ub144 5\uc6d4 22\uc77c, \ub450 \ubc88\uc9f8 \uc2f1\uae00 So Hot\uc758 \ud0c0\uc774\ud2c0\uace1 \"So Hot\"\uacfc \ubba4\uc9c1\ube44\ub514\uc624\ub97c \uc628\ub77c\uc778\uc73c\ub85c \uacf5\uac1c\ud588\ub2e4. \ud0c0\uc774\ud2c0 \uace1 \"So Hot\"\uc740 \uc790\uc2e0\uc758 \uc608\uc05c \uc678\ubaa8\uc640 \uc778\uae30\uac00 \ud53c\uace4\ud558\ub2e4\ub294 \ub0b4\uc6a9\uc758 \u201821\uc138\uae30\ud615 \uacf5\uc8fc\ubcd1\u2019\uc744 \ud45c\ubc29\ud55c \uac00\uc0ac\uc640 \ud638\ud53c\ubb34\ub2ac \uc758\uc0c1, \uadf8\ub9ac\uace0 \u2018V \ub77c\uc778 \ucda4\u2019, \u2018\uae30\uc9c0\uac1c \ucda4\u2019 \ub4f1\uc758 \uc548\ubb34\uac00 \ud2b9\uc9d5\uc801\uc778 \uace1\uc73c\ub85c, \uc568\ubc94\uc740 6\uc6d4 3\uc77c \uc815\uc2dd \ubc1c\ub9e4\ub418\uc5c8\ub2e4. \uc628\ub77c\uc778 \uacf5\uac1c \uc774\ud2c0 \ub9cc\uc5d0 \uba5c\ub860, \ub3c4\uc2dc\ub77d, \uc96c\ud06c\uc628, \ubba4\uc988, \uc2f8\uc774\uc6d4\ub4dc, \uc5e0\ub137\ub2f7\ucef4 \ub4f1 \uc628\ub77c\uc778 \ucc28\ud2b8\uc5d0\uc11c 1\uc704\ub97c \uc11d\uad8c\ud558\uace0, \ud154\ub808\ube44\uc804 \uac00\uc694 \ud504\ub85c\uadf8\ub7a8\uc5d0\uc11c\ub294 \uccab \ubb34\ub300\ubd80\ud130 1\uc704\ub97c \ucc28\uc9c0\ud558\uc5ec KBS \u300a\ubba4\uc9c1\ubc45\ud06c\u300b\uc5d0\uc11c\ub294 5\uc8fc \uc5f0\uc18d 1\uc704, SBS \u300a\uc778\uae30\uac00\uc694\u300b\uc640 Mnet \u300aM! Countdown\u300b\uc5d0\uc11c\ub294 3\uc8fc \uc5f0\uc18d 1\uc704\ub97c \ucc28\uc9c0\ud558\ub294 \ub4f1 \ud65c\ub3d9 \ucd08\ubc18\ubd80\ud130 \ud3ed\ubc1c\uc801\uc778 \ubc18\uc751\uc744 \ubd88\ub7ec\uc77c\uc73c\ucf30\uace0, \"Tell Me\" \ud65c\ub3d9 \uc2dc\uae30\uc640 \ub9c8\ucc2c\uac00\uc9c0\ub85c \uc218\ub9ce\uc740 \uc5f0\uc608\uc778\uc774 \uc774 \ucda4\uc744 \ub530\ub77c\ud558\uc600\ub2e4. \ub3d9\uc601\uc0c1 \uc81c\uacf5 \uc6f9\uc0ac\uc774\ud2b8 \uc720\ud29c\ube0c\uc5d0\uc11c\ub294 \ubba4\uc9c1\ube44\ub514\uc624\uac00 \ud55c\uad6d\ub0b4 \ub2e8\uc77c \uc601\uc0c1\uc73c\ub85c\ub294 \uc774\ub840\uc801\uc73c\ub85c \uc870\ud68c\uc218 100\ub9cc\uac74\uc744 \ub118\uae30\uba70 \ub178\ub798\ub97c \uac01\uad6d\uc5d0 \uc54c\ub838\ub2e4.", "paragraph_answer": "\uc6d0\ub354\uac78\uc2a4\ub294 2008\ub144 1\uc6d4 \ucd08 \uccab \uc815\uaddc \uc568\ubc94 \ud65c\ub3d9\uc744 \uc131\uacf5\uc801\uc73c\ub85c \ub9c8\uce58\uace0 \uacf5\ubc31\uae30\uc5d0 \ub4e4\uc5b4\uac14\ub2e4. \uc774 \uae30\uac04\uc5d0 \uc6d0\ub354\uac78\uc2a4\ub294 \ubbf8\uad6d\uc5d0\uc11c MTV \u300a\uc6d0\ub354\uac78\uc2a4 \uc2dc\uc98c3\u300b\ub97c \ucd2c\uc601\ud558\uba74\uc11c \ub3d9\uc2dc\uc5d0 \ubc15\uc9c4\uc601\uacfc JYP \uc5d4\ud130\ud14c\uc778\uba3c\ud2b8 \uc18c\uc18d \uac00\uc218\ub4e4\uc774 \ud568\uaed8\ud55c \u2018The JYP Tour\u2019 \ubb34\ub300\uc5d0 \uc624\ub974\uace0, \ud0c0\uc774\uc5d0\uc11c 1\ub9cc\uc5ec \uba85\uc758 \ud32c\ub4e4\uacfc \ud32c \ubbf8\ud305\uc744 \ud558\ub294 \ub4f1 \uad6d\uc678 \ud65c\ub3d9\uc5d0 \uc8fc\ub825\ud558\ub294 \ud55c\ud3b8, \uad6d\ub0b4\uc758 \uac01\uc885 \ucd95\uc81c\uc640 \ud589\uc0ac \ud65c\ub3d9 \ub610\ud55c \uc26c\uc9c0 \uc54a\uc558\ub2e4. 4\uac1c\uc6d4\uac04\uc758 \uacf5\ubc31\uae30\ub97c \uac70\uce5c \ud6c4, \uc18c\uc18d\uc0ac\ub294 \uc0c8 \uace1 \uacf5\uac1c\ub97c \uc55e\ub450\uace0 \ud558\ub8e8\uc5d0 \ud55c \uc7a5\uc529 \uba64\ubc84\ub4e4\uc758 \ucf58\uc149\ud2b8 \uc0ac\uc9c4\uc744 \uacf5\uac1c\ud558\uba70 \ub300\uc911\ub4e4\uc758 \uad00\uc2ec\uc744 \ub04c\uc5c8\ub2e4. \ub9c8\uc9c0\ub9c9 \ub2e8\uccb4 \uc0ac\uc9c4\uc774 \uacf5\uac1c\ub41c \ub2e4\uc74c\ub0a0\uc778 2008\ub144 5\uc6d4 22\uc77c, \ub450 \ubc88\uc9f8 \uc2f1\uae00 So Hot\uc758 \ud0c0\uc774\ud2c0\uace1 \"So Hot\"\uacfc \ubba4\uc9c1\ube44\ub514\uc624\ub97c \uc628\ub77c\uc778\uc73c\ub85c \uacf5\uac1c\ud588\ub2e4. \ud0c0\uc774\ud2c0 \uace1 \"So Hot\"\uc740 \uc790\uc2e0\uc758 \uc608\uc05c \uc678\ubaa8\uc640 \uc778\uae30\uac00 \ud53c\uace4\ud558\ub2e4\ub294 \ub0b4\uc6a9\uc758 \u201821\uc138\uae30\ud615 \uacf5\uc8fc\ubcd1\u2019\uc744 \ud45c\ubc29\ud55c \uac00\uc0ac\uc640 \ud638\ud53c\ubb34\ub2ac \uc758\uc0c1, \uadf8\ub9ac\uace0 \u2018V \ub77c\uc778 \ucda4\u2019, \u2018\uae30\uc9c0\uac1c \ucda4\u2019 \ub4f1\uc758 \uc548\ubb34\uac00 \ud2b9\uc9d5\uc801\uc778 \uace1\uc73c\ub85c, \uc568\ubc94\uc740 6\uc6d4 3\uc77c \uc815\uc2dd \ubc1c\ub9e4\ub418\uc5c8\ub2e4. \uc628\ub77c\uc778 \uacf5\uac1c \uc774\ud2c0 \ub9cc\uc5d0 \uba5c\ub860, \ub3c4\uc2dc\ub77d, \uc96c\ud06c\uc628, \ubba4\uc988, \uc2f8\uc774\uc6d4\ub4dc, \uc5e0\ub137\ub2f7\ucef4 \ub4f1 \uc628\ub77c\uc778 \ucc28\ud2b8\uc5d0\uc11c 1\uc704\ub97c \uc11d\uad8c\ud558\uace0, \ud154\ub808\ube44\uc804 \uac00\uc694 \ud504\ub85c\uadf8\ub7a8\uc5d0\uc11c\ub294 \uccab \ubb34\ub300\ubd80\ud130 1\uc704\ub97c \ucc28\uc9c0\ud558\uc5ec KBS \u300a\ubba4\uc9c1\ubc45\ud06c\u300b\uc5d0\uc11c\ub294 5\uc8fc \uc5f0\uc18d 1\uc704, SBS \u300a\uc778\uae30\uac00\uc694\u300b\uc640 Mnet \u300aM! Countdown\u300b\uc5d0\uc11c\ub294 3\uc8fc \uc5f0\uc18d 1\uc704\ub97c \ucc28\uc9c0\ud558\ub294 \ub4f1 \ud65c\ub3d9 \ucd08\ubc18\ubd80\ud130 \ud3ed\ubc1c\uc801\uc778 \ubc18\uc751\uc744 \ubd88\ub7ec\uc77c\uc73c\ucf30\uace0, \"Tell Me\" \ud65c\ub3d9 \uc2dc\uae30\uc640 \ub9c8\ucc2c\uac00\uc9c0\ub85c \uc218\ub9ce\uc740 \uc5f0\uc608\uc778\uc774 \uc774 \ucda4\uc744 \ub530\ub77c\ud558\uc600\ub2e4. \ub3d9\uc601\uc0c1 \uc81c\uacf5 \uc6f9\uc0ac\uc774\ud2b8 \uc720\ud29c\ube0c\uc5d0\uc11c\ub294 \ubba4\uc9c1\ube44\ub514\uc624\uac00 \ud55c\uad6d\ub0b4 \ub2e8\uc77c \uc601\uc0c1\uc73c\ub85c\ub294 \uc774\ub840\uc801\uc73c\ub85c \uc870\ud68c\uc218 100\ub9cc\uac74\uc744 \ub118\uae30\uba70 \ub178\ub798\ub97c \uac01\uad6d\uc5d0 \uc54c\ub838\ub2e4.", "sentence_answer": "\uc6d0\ub354\uac78\uc2a4\ub294 2008\ub144 1\uc6d4 \ucd08 \uccab \uc815\uaddc \uc568\ubc94 \ud65c\ub3d9\uc744 \uc131\uacf5\uc801\uc73c\ub85c \ub9c8\uce58\uace0 \uacf5\ubc31\uae30\uc5d0 \ub4e4\uc5b4\uac14\ub2e4.", "paragraph_id": "6542307-8-0", "paragraph_question": "question: \uc6d0\ub354\uac78\uc2a4\uac00 \uccab \uc815\uaddc \uc568\ubc94 \ud65c\ub3d9\uc744 \ub9c8\uce5c \uc2dc\uae30\ub294?, context: \uc6d0\ub354\uac78\uc2a4\ub294 2008\ub144 1\uc6d4 \ucd08 \uccab \uc815\uaddc \uc568\ubc94 \ud65c\ub3d9\uc744 \uc131\uacf5\uc801\uc73c\ub85c \ub9c8\uce58\uace0 \uacf5\ubc31\uae30\uc5d0 \ub4e4\uc5b4\uac14\ub2e4. \uc774 \uae30\uac04\uc5d0 \uc6d0\ub354\uac78\uc2a4\ub294 \ubbf8\uad6d\uc5d0\uc11c MTV \u300a\uc6d0\ub354\uac78\uc2a4 \uc2dc\uc98c3\u300b\ub97c \ucd2c\uc601\ud558\uba74\uc11c \ub3d9\uc2dc\uc5d0 \ubc15\uc9c4\uc601\uacfc JYP \uc5d4\ud130\ud14c\uc778\uba3c\ud2b8 \uc18c\uc18d \uac00\uc218\ub4e4\uc774 \ud568\uaed8\ud55c \u2018The JYP Tour\u2019 \ubb34\ub300\uc5d0 \uc624\ub974\uace0, \ud0c0\uc774\uc5d0\uc11c 1\ub9cc\uc5ec \uba85\uc758 \ud32c\ub4e4\uacfc \ud32c \ubbf8\ud305\uc744 \ud558\ub294 \ub4f1 \uad6d\uc678 \ud65c\ub3d9\uc5d0 \uc8fc\ub825\ud558\ub294 \ud55c\ud3b8, \uad6d\ub0b4\uc758 \uac01\uc885 \ucd95\uc81c\uc640 \ud589\uc0ac \ud65c\ub3d9 \ub610\ud55c \uc26c\uc9c0 \uc54a\uc558\ub2e4. 4\uac1c\uc6d4\uac04\uc758 \uacf5\ubc31\uae30\ub97c \uac70\uce5c \ud6c4, \uc18c\uc18d\uc0ac\ub294 \uc0c8 \uace1 \uacf5\uac1c\ub97c \uc55e\ub450\uace0 \ud558\ub8e8\uc5d0 \ud55c \uc7a5\uc529 \uba64\ubc84\ub4e4\uc758 \ucf58\uc149\ud2b8 \uc0ac\uc9c4\uc744 \uacf5\uac1c\ud558\uba70 \ub300\uc911\ub4e4\uc758 \uad00\uc2ec\uc744 \ub04c\uc5c8\ub2e4. \ub9c8\uc9c0\ub9c9 \ub2e8\uccb4 \uc0ac\uc9c4\uc774 \uacf5\uac1c\ub41c \ub2e4\uc74c\ub0a0\uc778 2008\ub144 5\uc6d4 22\uc77c, \ub450 \ubc88\uc9f8 \uc2f1\uae00 So Hot\uc758 \ud0c0\uc774\ud2c0\uace1 \"So Hot\"\uacfc \ubba4\uc9c1\ube44\ub514\uc624\ub97c \uc628\ub77c\uc778\uc73c\ub85c \uacf5\uac1c\ud588\ub2e4. \ud0c0\uc774\ud2c0 \uace1 \"So Hot\"\uc740 \uc790\uc2e0\uc758 \uc608\uc05c \uc678\ubaa8\uc640 \uc778\uae30\uac00 \ud53c\uace4\ud558\ub2e4\ub294 \ub0b4\uc6a9\uc758 \u201821\uc138\uae30\ud615 \uacf5\uc8fc\ubcd1\u2019\uc744 \ud45c\ubc29\ud55c \uac00\uc0ac\uc640 \ud638\ud53c\ubb34\ub2ac \uc758\uc0c1, \uadf8\ub9ac\uace0 \u2018V \ub77c\uc778 \ucda4\u2019, \u2018\uae30\uc9c0\uac1c \ucda4\u2019 \ub4f1\uc758 \uc548\ubb34\uac00 \ud2b9\uc9d5\uc801\uc778 \uace1\uc73c\ub85c, \uc568\ubc94\uc740 6\uc6d4 3\uc77c \uc815\uc2dd \ubc1c\ub9e4\ub418\uc5c8\ub2e4. \uc628\ub77c\uc778 \uacf5\uac1c \uc774\ud2c0 \ub9cc\uc5d0 \uba5c\ub860, \ub3c4\uc2dc\ub77d, \uc96c\ud06c\uc628, \ubba4\uc988, \uc2f8\uc774\uc6d4\ub4dc, \uc5e0\ub137\ub2f7\ucef4 \ub4f1 \uc628\ub77c\uc778 \ucc28\ud2b8\uc5d0\uc11c 1\uc704\ub97c \uc11d\uad8c\ud558\uace0, \ud154\ub808\ube44\uc804 \uac00\uc694 \ud504\ub85c\uadf8\ub7a8\uc5d0\uc11c\ub294 \uccab \ubb34\ub300\ubd80\ud130 1\uc704\ub97c \ucc28\uc9c0\ud558\uc5ec KBS \u300a\ubba4\uc9c1\ubc45\ud06c\u300b\uc5d0\uc11c\ub294 5\uc8fc \uc5f0\uc18d 1\uc704, SBS \u300a\uc778\uae30\uac00\uc694\u300b\uc640 Mnet \u300aM! Countdown\u300b\uc5d0\uc11c\ub294 3\uc8fc \uc5f0\uc18d 1\uc704\ub97c \ucc28\uc9c0\ud558\ub294 \ub4f1 \ud65c\ub3d9 \ucd08\ubc18\ubd80\ud130 \ud3ed\ubc1c\uc801\uc778 \ubc18\uc751\uc744 \ubd88\ub7ec\uc77c\uc73c\ucf30\uace0, \"Tell Me\" \ud65c\ub3d9 \uc2dc\uae30\uc640 \ub9c8\ucc2c\uac00\uc9c0\ub85c \uc218\ub9ce\uc740 \uc5f0\uc608\uc778\uc774 \uc774 \ucda4\uc744 \ub530\ub77c\ud558\uc600\ub2e4. \ub3d9\uc601\uc0c1 \uc81c\uacf5 \uc6f9\uc0ac\uc774\ud2b8 \uc720\ud29c\ube0c\uc5d0\uc11c\ub294 \ubba4\uc9c1\ube44\ub514\uc624\uac00 \ud55c\uad6d\ub0b4 \ub2e8\uc77c \uc601\uc0c1\uc73c\ub85c\ub294 \uc774\ub840\uc801\uc73c\ub85c \uc870\ud68c\uc218 100\ub9cc\uac74\uc744 \ub118\uae30\uba70 \ub178\ub798\ub97c \uac01\uad6d\uc5d0 \uc54c\ub838\ub2e4."} {"question": "\uc5ec\ud638\uc640\uc640 \ube44\uc2b7\ud55c \ubaa8\uc74c\ud654\ub97c \uc0ac\uc6a9\ud55c \ucd5c\ucd08\uc758 \ub77c\ud2f4\uc5b4 \ubcf8\ubb38\uc774 \ub098\uc628 \ub54c\ub294?", "paragraph": "\uc5ec\ud638\uc640(\uc601\uc5b4: Jehovah, /d\u0292\u0268\u02c8ho\u028av\u0259/ \ub85c \ubc1c\uc74c)\ub77c\ub294 \ub9d0\uc758 \uc5b4\uc6d0\uc740 \ud558\ub098\ub2d8\uc758 \ud638\uce6d\uc744 \uac00\ub9ac\ud0a4\ub294 \uace0\uc720 \uba85\uc0ac\ub85c \uc601\uad6d\uc2dd \uc5b8\uc5b4\uc758 \ud45c\uae30 \ubc29\uc2dd\uc744 \ub530\ub978 \uac83\uc774\ub2e4. \uc774\uac83\uc740 \ud558\ub098\ub2d8\uc758 \uc774\ub984 4\ubb38\uc790 \ud788\ube0c\ub9ac\uc5b4 \u05d9\u05d4\u05d5\u05d4\ub97c \ubaa8\uc74c\ud654 \uc74c\uc5ed\ud55c \uac83\uc73c\ub85c, \ud574\ub2f9 4\uac1c \ubb38\uc790\ub294 \uc131\uacbd\uc5d0 \ub530\ub974\uba74 \ud558\ub098\ub2d8\uaed8\uc11c \uc0ac\ub78c\ub4e4\uc5d0\uac8c \ubc1d\ud788\uc2e0 \uc790\uc2e0\uc758 \uc774\ub984\uc774\ub2e4. \uc804\ud1b5\uc801\uc778 \ub9c8\uc18c\ub77c \ubcf8\ubb38\uc5d0\ub294 305\ubc88\uc758 \u05d9\u05b1\u05d4\u05b9\u05d5\u05b4\u05d4(Jehovih)\ubfd0 \uc544\ub2c8\ub77c \u05d9\u05b0\u05d4\u05b9\u05d5\u05b8\u05d4\uac00 6,518\ubc88 \ub098\ud0c0\ub09c\ub2e4. \"\uc5ec\ud638\uc640\"\uc640 \ube44\uc2b7\ud55c \ubaa8\uc74c\ud654\ub97c \uc0ac\uc6a9\ud55c \ucd5c\ucd08\uc758 \ub77c\ud2f4\uc5b4 \ubcf8\ubb38\uc740 13\uc138\uae30\ub85c \uac70\uc2ac\ub7ec \uc62c\ub77c\uac04\ub2e4. \uadf8\ub798\uc11c \uc131\uacbd\uc744 \uc77d\uc744 \ub54c \uc720\ub300\uc778\ub4e4\uc740 \uc774\ub97c \u2018\uc544\ub3c4\ub098\uc774(\ub098\uc758 \uc8fc\ub2d8\uc774\ub780 \ub73b)\u2019\ub85c \uc77d\uace0, \ucc9c\uc8fc\uad50\uc5d0\uc11c\ub294 \u2018\uc57c\ud6fc\u2019, \uae30\ub3c5\uad50\uc5d0\uc11c\ub294 \u2018\uc5ec\ud638\uc640\u2019\ub77c\uace0 \uc77d\ub294\ub2e4.", "answer": "13\uc138\uae30", "sentence": "\"\uc5ec\ud638\uc640\"\uc640 \ube44\uc2b7\ud55c \ubaa8\uc74c\ud654\ub97c \uc0ac\uc6a9\ud55c \ucd5c\ucd08\uc758 \ub77c\ud2f4\uc5b4 \ubcf8\ubb38\uc740 13\uc138\uae30 \ub85c \uac70\uc2ac\ub7ec \uc62c\ub77c\uac04\ub2e4.", "paragraph_sentence": "\uc5ec\ud638\uc640(\uc601\uc5b4: Jehovah, /d\u0292\u0268\u02c8ho\u028av\u0259/ \ub85c \ubc1c\uc74c)\ub77c\ub294 \ub9d0\uc758 \uc5b4\uc6d0\uc740 \ud558\ub098\ub2d8\uc758 \ud638\uce6d\uc744 \uac00\ub9ac\ud0a4\ub294 \uace0\uc720 \uba85\uc0ac\ub85c \uc601\uad6d\uc2dd \uc5b8\uc5b4\uc758 \ud45c\uae30 \ubc29\uc2dd\uc744 \ub530\ub978 \uac83\uc774\ub2e4. \uc774\uac83\uc740 \ud558\ub098\ub2d8\uc758 \uc774\ub984 4\ubb38\uc790 \ud788\ube0c\ub9ac\uc5b4 \u05d9\u05d4\u05d5\u05d4\ub97c \ubaa8\uc74c\ud654 \uc74c\uc5ed\ud55c \uac83\uc73c\ub85c, \ud574\ub2f9 4\uac1c \ubb38\uc790\ub294 \uc131\uacbd\uc5d0 \ub530\ub974\uba74 \ud558\ub098\ub2d8\uaed8\uc11c \uc0ac\ub78c\ub4e4\uc5d0\uac8c \ubc1d\ud788\uc2e0 \uc790\uc2e0\uc758 \uc774\ub984\uc774\ub2e4. \uc804\ud1b5\uc801\uc778 \ub9c8\uc18c\ub77c \ubcf8\ubb38\uc5d0\ub294 305\ubc88\uc758 \u05d9\u05b1\u05d4\u05b9\u05d5\u05b4\u05d4(Jehovih)\ubfd0 \uc544\ub2c8\ub77c \u05d9\u05b0\u05d4\u05b9\u05d5\u05b8\u05d4\uac00 6,518\ubc88 \ub098\ud0c0\ub09c\ub2e4. \"\uc5ec\ud638\uc640\"\uc640 \ube44\uc2b7\ud55c \ubaa8\uc74c\ud654\ub97c \uc0ac\uc6a9\ud55c \ucd5c\ucd08\uc758 \ub77c\ud2f4\uc5b4 \ubcf8\ubb38\uc740 13\uc138\uae30 \ub85c \uac70\uc2ac\ub7ec \uc62c\ub77c\uac04\ub2e4. \uadf8\ub798\uc11c \uc131\uacbd\uc744 \uc77d\uc744 \ub54c \uc720\ub300\uc778\ub4e4\uc740 \uc774\ub97c \u2018\uc544\ub3c4\ub098\uc774(\ub098\uc758 \uc8fc\ub2d8\uc774\ub780 \ub73b)\u2019\ub85c \uc77d\uace0, \ucc9c\uc8fc\uad50\uc5d0\uc11c\ub294 \u2018\uc57c\ud6fc\u2019, \uae30\ub3c5\uad50\uc5d0\uc11c\ub294 \u2018\uc5ec\ud638\uc640\u2019\ub77c\uace0 \uc77d\ub294\ub2e4.", "paragraph_answer": "\uc5ec\ud638\uc640(\uc601\uc5b4: Jehovah, /d\u0292\u0268\u02c8ho\u028av\u0259/ \ub85c \ubc1c\uc74c)\ub77c\ub294 \ub9d0\uc758 \uc5b4\uc6d0\uc740 \ud558\ub098\ub2d8\uc758 \ud638\uce6d\uc744 \uac00\ub9ac\ud0a4\ub294 \uace0\uc720 \uba85\uc0ac\ub85c \uc601\uad6d\uc2dd \uc5b8\uc5b4\uc758 \ud45c\uae30 \ubc29\uc2dd\uc744 \ub530\ub978 \uac83\uc774\ub2e4. \uc774\uac83\uc740 \ud558\ub098\ub2d8\uc758 \uc774\ub984 4\ubb38\uc790 \ud788\ube0c\ub9ac\uc5b4 \u05d9\u05d4\u05d5\u05d4\ub97c \ubaa8\uc74c\ud654 \uc74c\uc5ed\ud55c \uac83\uc73c\ub85c, \ud574\ub2f9 4\uac1c \ubb38\uc790\ub294 \uc131\uacbd\uc5d0 \ub530\ub974\uba74 \ud558\ub098\ub2d8\uaed8\uc11c \uc0ac\ub78c\ub4e4\uc5d0\uac8c \ubc1d\ud788\uc2e0 \uc790\uc2e0\uc758 \uc774\ub984\uc774\ub2e4. \uc804\ud1b5\uc801\uc778 \ub9c8\uc18c\ub77c \ubcf8\ubb38\uc5d0\ub294 305\ubc88\uc758 \u05d9\u05b1\u05d4\u05b9\u05d5\u05b4\u05d4(Jehovih)\ubfd0 \uc544\ub2c8\ub77c \u05d9\u05b0\u05d4\u05b9\u05d5\u05b8\u05d4\uac00 6,518\ubc88 \ub098\ud0c0\ub09c\ub2e4. \"\uc5ec\ud638\uc640\"\uc640 \ube44\uc2b7\ud55c \ubaa8\uc74c\ud654\ub97c \uc0ac\uc6a9\ud55c \ucd5c\ucd08\uc758 \ub77c\ud2f4\uc5b4 \ubcf8\ubb38\uc740 13\uc138\uae30 \ub85c \uac70\uc2ac\ub7ec \uc62c\ub77c\uac04\ub2e4. \uadf8\ub798\uc11c \uc131\uacbd\uc744 \uc77d\uc744 \ub54c \uc720\ub300\uc778\ub4e4\uc740 \uc774\ub97c \u2018\uc544\ub3c4\ub098\uc774(\ub098\uc758 \uc8fc\ub2d8\uc774\ub780 \ub73b)\u2019\ub85c \uc77d\uace0, \ucc9c\uc8fc\uad50\uc5d0\uc11c\ub294 \u2018\uc57c\ud6fc\u2019, \uae30\ub3c5\uad50\uc5d0\uc11c\ub294 \u2018\uc5ec\ud638\uc640\u2019\ub77c\uace0 \uc77d\ub294\ub2e4.", "sentence_answer": "\"\uc5ec\ud638\uc640\"\uc640 \ube44\uc2b7\ud55c \ubaa8\uc74c\ud654\ub97c \uc0ac\uc6a9\ud55c \ucd5c\ucd08\uc758 \ub77c\ud2f4\uc5b4 \ubcf8\ubb38\uc740 13\uc138\uae30 \ub85c \uac70\uc2ac\ub7ec \uc62c\ub77c\uac04\ub2e4.", "paragraph_id": "6524016-0-0", "paragraph_question": "question: \uc5ec\ud638\uc640\uc640 \ube44\uc2b7\ud55c \ubaa8\uc74c\ud654\ub97c \uc0ac\uc6a9\ud55c \ucd5c\ucd08\uc758 \ub77c\ud2f4\uc5b4 \ubcf8\ubb38\uc774 \ub098\uc628 \ub54c\ub294?, context: \uc5ec\ud638\uc640(\uc601\uc5b4: Jehovah, /d\u0292\u0268\u02c8ho\u028av\u0259/ \ub85c \ubc1c\uc74c)\ub77c\ub294 \ub9d0\uc758 \uc5b4\uc6d0\uc740 \ud558\ub098\ub2d8\uc758 \ud638\uce6d\uc744 \uac00\ub9ac\ud0a4\ub294 \uace0\uc720 \uba85\uc0ac\ub85c \uc601\uad6d\uc2dd \uc5b8\uc5b4\uc758 \ud45c\uae30 \ubc29\uc2dd\uc744 \ub530\ub978 \uac83\uc774\ub2e4. \uc774\uac83\uc740 \ud558\ub098\ub2d8\uc758 \uc774\ub984 4\ubb38\uc790 \ud788\ube0c\ub9ac\uc5b4 \u05d9\u05d4\u05d5\u05d4\ub97c \ubaa8\uc74c\ud654 \uc74c\uc5ed\ud55c \uac83\uc73c\ub85c, \ud574\ub2f9 4\uac1c \ubb38\uc790\ub294 \uc131\uacbd\uc5d0 \ub530\ub974\uba74 \ud558\ub098\ub2d8\uaed8\uc11c \uc0ac\ub78c\ub4e4\uc5d0\uac8c \ubc1d\ud788\uc2e0 \uc790\uc2e0\uc758 \uc774\ub984\uc774\ub2e4. \uc804\ud1b5\uc801\uc778 \ub9c8\uc18c\ub77c \ubcf8\ubb38\uc5d0\ub294 305\ubc88\uc758 \u05d9\u05b1\u05d4\u05b9\u05d5\u05b4\u05d4(Jehovih)\ubfd0 \uc544\ub2c8\ub77c \u05d9\u05b0\u05d4\u05b9\u05d5\u05b8\u05d4\uac00 6,518\ubc88 \ub098\ud0c0\ub09c\ub2e4. \"\uc5ec\ud638\uc640\"\uc640 \ube44\uc2b7\ud55c \ubaa8\uc74c\ud654\ub97c \uc0ac\uc6a9\ud55c \ucd5c\ucd08\uc758 \ub77c\ud2f4\uc5b4 \ubcf8\ubb38\uc740 13\uc138\uae30\ub85c \uac70\uc2ac\ub7ec \uc62c\ub77c\uac04\ub2e4. \uadf8\ub798\uc11c \uc131\uacbd\uc744 \uc77d\uc744 \ub54c \uc720\ub300\uc778\ub4e4\uc740 \uc774\ub97c \u2018\uc544\ub3c4\ub098\uc774(\ub098\uc758 \uc8fc\ub2d8\uc774\ub780 \ub73b)\u2019\ub85c \uc77d\uace0, \ucc9c\uc8fc\uad50\uc5d0\uc11c\ub294 \u2018\uc57c\ud6fc\u2019, \uae30\ub3c5\uad50\uc5d0\uc11c\ub294 \u2018\uc5ec\ud638\uc640\u2019\ub77c\uace0 \uc77d\ub294\ub2e4."} {"question": "\ud50c\ub77c\uc988\ub9c8\ub294 \ub3c4\uccb4\uc774\uae30\uc5d0 \uc800\ud56d\uc774 \uc5bc\ub9c8 \uc548\ub418\uc9c0\ub9cc \uc5f4\uc744 \ubc1c\uc0dd\uc2dc\ucf1c \uc628\ub3c4\ub97c \ub192\uc774\uac8c \ub418\ub294 \ud604\uc0c1\uc744 \ubb34\uc5c7\uc774\ub77c \ud558\ub294\uac00?", "paragraph": "\uc55e\uc5d0\uc11c \uc774\uc57c\uae30\ud558\uc600\ub4ef\uc774 \ud1a0\uce74\ub9c9 \ub0b4\ubd80 \ud50c\ub77c\uc988\ub9c8\uc5d0\ub294 \uc804\ub958\uac00 \ud750\ub978\ub2e4. \ud50c\ub77c\uc988\ub9c8 \uc0c1\ud0dc\ub294 \ub3c4\uccb4 \uc0c1\ud0dc\uc774\ubbc0\ub85c \uc804\ub958\uac00 \ud750\ub974\ub294 \uac83\uc5d0\ub294 \ubb38\uc81c\uac00 \uc5c6\ub2e4. \ud558\uc9c0\ub9cc \ub3c4\uccb4 \uc0c1\ud0dc\uc640 \ub9c8\ucc2c\uac00\uc9c0\ub85c \ud50c\ub77c\uc988\ub9c8\uc5d0\ub294 \uc800\ud56d\uc774 \uc788\ub2e4. \ub9cc\uc57d \uc800\ud56d\uc774 \uc788\ub2e4\uba74 \uadf8\ub85c \uc778\ud574 \uc5f4\uc774 \ubc1c\uc0dd\ud558\uac8c \ub420 \uac83\uc774\uace0 \uc774\uac83\uc774 \ud50c\ub77c\uc988\ub9c8\ub97c \uac00\uc5f4\uc2dc\ud0a4\ub294 \uc5ed\ud560\uc744 \ud558\uac8c \ub41c\ub2e4. \ud50c\ub77c\uc988\ub9c8\ub294 \ub3c4\uccb4\uc774\uae30 \ub54c\ubb38\uc5d0 \uc800\ud56d\uc774 \uc5bc\ub9c8 \uc548 \ub418\uc9c0\ub9cc \uc804\ub958\uac00 \uc0c1\ub2f9\ud788 \ud06c\uae30 \ub54c\ubb38\uc5d0 \uc5f4\uc744 \ubc1c\uc0dd\uc2dc\ucf1c \uc628\ub3c4\ub97c \ub192\uc774\uac8c \ub41c\ub2e4. \ud558\uc9c0\ub9cc \uacc4\uc18d \uc628\ub3c4\uac00 \uc62c\ub77c\uac08\uc218\ub85d \ud50c\ub77c\uc988\ub9c8\uc758 \uc800\ud56d\uc774 \uc904\uc5b4\ub4e4\uac8c \ub418\uace0 \uc774\ub85c \uc778\ud574 \ub192\uc77c \uc218 \uc788\ub294 \uc628\ub3c4\uc5d0 \ud55c\uacc4\uac00 \uc624\uac8c \ub41c\ub2e4. \ud558\uc9c0\ub9cc \ud1a0\uce74\ub9c9\uc740 \ucd08\uace0\uc628 \ud50c\ub77c\uc988\ub9c8\ub97c \ub9cc\ub4e4\uc5b4\uc57c \ud558\uae30 \ub54c\ubb38\uc5d0 \uc800\ud56d\uc5d0 \uc758\ud55c \uac00\uc5f4\ub85c\ub294 \ub9cc\uc871\uc2dc\ud0ac \uc218 \uc5c6\ub2e4. \uadf8\ub7ec\ubbc0\ub85c \uc678\ubd80\uc5d0\uc11c \uac00\uc5f4\uc744 \uc704\ud55c \ub2e4\ub978 \ubc29\ubc95\ub4e4\uc774 \ud544\uc694\ud558\ub2e4.", "answer": "\uc804\ub958", "sentence": "\uc55e\uc5d0\uc11c \uc774\uc57c\uae30\ud558\uc600\ub4ef\uc774 \ud1a0\uce74\ub9c9 \ub0b4\ubd80 \ud50c\ub77c\uc988\ub9c8\uc5d0\ub294 \uc804\ub958 \uac00 \ud750\ub978\ub2e4.", "paragraph_sentence": " \uc55e\uc5d0\uc11c \uc774\uc57c\uae30\ud558\uc600\ub4ef\uc774 \ud1a0\uce74\ub9c9 \ub0b4\ubd80 \ud50c\ub77c\uc988\ub9c8\uc5d0\ub294 \uc804\ub958 \uac00 \ud750\ub978\ub2e4. \ud50c\ub77c\uc988\ub9c8 \uc0c1\ud0dc\ub294 \ub3c4\uccb4 \uc0c1\ud0dc\uc774\ubbc0\ub85c \uc804\ub958\uac00 \ud750\ub974\ub294 \uac83\uc5d0\ub294 \ubb38\uc81c\uac00 \uc5c6\ub2e4. \ud558\uc9c0\ub9cc \ub3c4\uccb4 \uc0c1\ud0dc\uc640 \ub9c8\ucc2c\uac00\uc9c0\ub85c \ud50c\ub77c\uc988\ub9c8\uc5d0\ub294 \uc800\ud56d\uc774 \uc788\ub2e4. \ub9cc\uc57d \uc800\ud56d\uc774 \uc788\ub2e4\uba74 \uadf8\ub85c \uc778\ud574 \uc5f4\uc774 \ubc1c\uc0dd\ud558\uac8c \ub420 \uac83\uc774\uace0 \uc774\uac83\uc774 \ud50c\ub77c\uc988\ub9c8\ub97c \uac00\uc5f4\uc2dc\ud0a4\ub294 \uc5ed\ud560\uc744 \ud558\uac8c \ub41c\ub2e4. \ud50c\ub77c\uc988\ub9c8\ub294 \ub3c4\uccb4\uc774\uae30 \ub54c\ubb38\uc5d0 \uc800\ud56d\uc774 \uc5bc\ub9c8 \uc548 \ub418\uc9c0\ub9cc \uc804\ub958\uac00 \uc0c1\ub2f9\ud788 \ud06c\uae30 \ub54c\ubb38\uc5d0 \uc5f4\uc744 \ubc1c\uc0dd\uc2dc\ucf1c \uc628\ub3c4\ub97c \ub192\uc774\uac8c \ub41c\ub2e4. \ud558\uc9c0\ub9cc \uacc4\uc18d \uc628\ub3c4\uac00 \uc62c\ub77c\uac08\uc218\ub85d \ud50c\ub77c\uc988\ub9c8\uc758 \uc800\ud56d\uc774 \uc904\uc5b4\ub4e4\uac8c \ub418\uace0 \uc774\ub85c \uc778\ud574 \ub192\uc77c \uc218 \uc788\ub294 \uc628\ub3c4\uc5d0 \ud55c\uacc4\uac00 \uc624\uac8c \ub41c\ub2e4. \ud558\uc9c0\ub9cc \ud1a0\uce74\ub9c9\uc740 \ucd08\uace0\uc628 \ud50c\ub77c\uc988\ub9c8\ub97c \ub9cc\ub4e4\uc5b4\uc57c \ud558\uae30 \ub54c\ubb38\uc5d0 \uc800\ud56d\uc5d0 \uc758\ud55c \uac00\uc5f4\ub85c\ub294 \ub9cc\uc871\uc2dc\ud0ac \uc218 \uc5c6\ub2e4. \uadf8\ub7ec\ubbc0\ub85c \uc678\ubd80\uc5d0\uc11c \uac00\uc5f4\uc744 \uc704\ud55c \ub2e4\ub978 \ubc29\ubc95\ub4e4\uc774 \ud544\uc694\ud558\ub2e4.", "paragraph_answer": "\uc55e\uc5d0\uc11c \uc774\uc57c\uae30\ud558\uc600\ub4ef\uc774 \ud1a0\uce74\ub9c9 \ub0b4\ubd80 \ud50c\ub77c\uc988\ub9c8\uc5d0\ub294 \uc804\ub958 \uac00 \ud750\ub978\ub2e4. \ud50c\ub77c\uc988\ub9c8 \uc0c1\ud0dc\ub294 \ub3c4\uccb4 \uc0c1\ud0dc\uc774\ubbc0\ub85c \uc804\ub958\uac00 \ud750\ub974\ub294 \uac83\uc5d0\ub294 \ubb38\uc81c\uac00 \uc5c6\ub2e4. \ud558\uc9c0\ub9cc \ub3c4\uccb4 \uc0c1\ud0dc\uc640 \ub9c8\ucc2c\uac00\uc9c0\ub85c \ud50c\ub77c\uc988\ub9c8\uc5d0\ub294 \uc800\ud56d\uc774 \uc788\ub2e4. \ub9cc\uc57d \uc800\ud56d\uc774 \uc788\ub2e4\uba74 \uadf8\ub85c \uc778\ud574 \uc5f4\uc774 \ubc1c\uc0dd\ud558\uac8c \ub420 \uac83\uc774\uace0 \uc774\uac83\uc774 \ud50c\ub77c\uc988\ub9c8\ub97c \uac00\uc5f4\uc2dc\ud0a4\ub294 \uc5ed\ud560\uc744 \ud558\uac8c \ub41c\ub2e4. \ud50c\ub77c\uc988\ub9c8\ub294 \ub3c4\uccb4\uc774\uae30 \ub54c\ubb38\uc5d0 \uc800\ud56d\uc774 \uc5bc\ub9c8 \uc548 \ub418\uc9c0\ub9cc \uc804\ub958\uac00 \uc0c1\ub2f9\ud788 \ud06c\uae30 \ub54c\ubb38\uc5d0 \uc5f4\uc744 \ubc1c\uc0dd\uc2dc\ucf1c \uc628\ub3c4\ub97c \ub192\uc774\uac8c \ub41c\ub2e4. \ud558\uc9c0\ub9cc \uacc4\uc18d \uc628\ub3c4\uac00 \uc62c\ub77c\uac08\uc218\ub85d \ud50c\ub77c\uc988\ub9c8\uc758 \uc800\ud56d\uc774 \uc904\uc5b4\ub4e4\uac8c \ub418\uace0 \uc774\ub85c \uc778\ud574 \ub192\uc77c \uc218 \uc788\ub294 \uc628\ub3c4\uc5d0 \ud55c\uacc4\uac00 \uc624\uac8c \ub41c\ub2e4. \ud558\uc9c0\ub9cc \ud1a0\uce74\ub9c9\uc740 \ucd08\uace0\uc628 \ud50c\ub77c\uc988\ub9c8\ub97c \ub9cc\ub4e4\uc5b4\uc57c \ud558\uae30 \ub54c\ubb38\uc5d0 \uc800\ud56d\uc5d0 \uc758\ud55c \uac00\uc5f4\ub85c\ub294 \ub9cc\uc871\uc2dc\ud0ac \uc218 \uc5c6\ub2e4. \uadf8\ub7ec\ubbc0\ub85c \uc678\ubd80\uc5d0\uc11c \uac00\uc5f4\uc744 \uc704\ud55c \ub2e4\ub978 \ubc29\ubc95\ub4e4\uc774 \ud544\uc694\ud558\ub2e4.", "sentence_answer": "\uc55e\uc5d0\uc11c \uc774\uc57c\uae30\ud558\uc600\ub4ef\uc774 \ud1a0\uce74\ub9c9 \ub0b4\ubd80 \ud50c\ub77c\uc988\ub9c8\uc5d0\ub294 \uc804\ub958 \uac00 \ud750\ub978\ub2e4.", "paragraph_id": "5915496-3-1", "paragraph_question": "question: \ud50c\ub77c\uc988\ub9c8\ub294 \ub3c4\uccb4\uc774\uae30\uc5d0 \uc800\ud56d\uc774 \uc5bc\ub9c8 \uc548\ub418\uc9c0\ub9cc \uc5f4\uc744 \ubc1c\uc0dd\uc2dc\ucf1c \uc628\ub3c4\ub97c \ub192\uc774\uac8c \ub418\ub294 \ud604\uc0c1\uc744 \ubb34\uc5c7\uc774\ub77c \ud558\ub294\uac00?, context: \uc55e\uc5d0\uc11c \uc774\uc57c\uae30\ud558\uc600\ub4ef\uc774 \ud1a0\uce74\ub9c9 \ub0b4\ubd80 \ud50c\ub77c\uc988\ub9c8\uc5d0\ub294 \uc804\ub958\uac00 \ud750\ub978\ub2e4. \ud50c\ub77c\uc988\ub9c8 \uc0c1\ud0dc\ub294 \ub3c4\uccb4 \uc0c1\ud0dc\uc774\ubbc0\ub85c \uc804\ub958\uac00 \ud750\ub974\ub294 \uac83\uc5d0\ub294 \ubb38\uc81c\uac00 \uc5c6\ub2e4. \ud558\uc9c0\ub9cc \ub3c4\uccb4 \uc0c1\ud0dc\uc640 \ub9c8\ucc2c\uac00\uc9c0\ub85c \ud50c\ub77c\uc988\ub9c8\uc5d0\ub294 \uc800\ud56d\uc774 \uc788\ub2e4. \ub9cc\uc57d \uc800\ud56d\uc774 \uc788\ub2e4\uba74 \uadf8\ub85c \uc778\ud574 \uc5f4\uc774 \ubc1c\uc0dd\ud558\uac8c \ub420 \uac83\uc774\uace0 \uc774\uac83\uc774 \ud50c\ub77c\uc988\ub9c8\ub97c \uac00\uc5f4\uc2dc\ud0a4\ub294 \uc5ed\ud560\uc744 \ud558\uac8c \ub41c\ub2e4. \ud50c\ub77c\uc988\ub9c8\ub294 \ub3c4\uccb4\uc774\uae30 \ub54c\ubb38\uc5d0 \uc800\ud56d\uc774 \uc5bc\ub9c8 \uc548 \ub418\uc9c0\ub9cc \uc804\ub958\uac00 \uc0c1\ub2f9\ud788 \ud06c\uae30 \ub54c\ubb38\uc5d0 \uc5f4\uc744 \ubc1c\uc0dd\uc2dc\ucf1c \uc628\ub3c4\ub97c \ub192\uc774\uac8c \ub41c\ub2e4. \ud558\uc9c0\ub9cc \uacc4\uc18d \uc628\ub3c4\uac00 \uc62c\ub77c\uac08\uc218\ub85d \ud50c\ub77c\uc988\ub9c8\uc758 \uc800\ud56d\uc774 \uc904\uc5b4\ub4e4\uac8c \ub418\uace0 \uc774\ub85c \uc778\ud574 \ub192\uc77c \uc218 \uc788\ub294 \uc628\ub3c4\uc5d0 \ud55c\uacc4\uac00 \uc624\uac8c \ub41c\ub2e4. \ud558\uc9c0\ub9cc \ud1a0\uce74\ub9c9\uc740 \ucd08\uace0\uc628 \ud50c\ub77c\uc988\ub9c8\ub97c \ub9cc\ub4e4\uc5b4\uc57c \ud558\uae30 \ub54c\ubb38\uc5d0 \uc800\ud56d\uc5d0 \uc758\ud55c \uac00\uc5f4\ub85c\ub294 \ub9cc\uc871\uc2dc\ud0ac \uc218 \uc5c6\ub2e4. \uadf8\ub7ec\ubbc0\ub85c \uc678\ubd80\uc5d0\uc11c \uac00\uc5f4\uc744 \uc704\ud55c \ub2e4\ub978 \ubc29\ubc95\ub4e4\uc774 \ud544\uc694\ud558\ub2e4."} {"question": "1\ucc28 \ub17c\uc7c1\uc5d0\uc11c\ub294 \uc11c\uc778\uc758 \uc8fc\uc7a5\uc774 \uc77c\uc5b4\ub09c \ud574\ub294?", "paragraph": "\ub0a8\uc778\ub4e4\uc740 \uc11c\uc778 \uc815\uad8c\uc774 \ucd94\uad6c\ud55c \uac1c\ud601\uc758 \ubd80\ub2f9\uc131\uacfc \ubd81\ubc8c \uc6b4\ub3d9\uc758 \ubb34\ubaa8\ud568\uc744 \uc9c0\uc801\ud558\uba74\uc11c \uc608\uc1a1\ub17c\uc7c1(\u79ae\u8a1f\u8ad6\u722d)\uc744 \uc77c\uc73c\ucf1c \uc11c\uc778\ub4e4\uacfc \uc815\uce58\uc801\uc73c\ub85c \ub300\ub9bd\ud558\uc600\ub2e4. \ub354\uad6c\ub098 \uc608\uc1a1\ub17c\uc7c1\uc774 \uc815\uccb4(\u653f\u9ad4)\ubb38\uc81c\uc640 \uad00\ub828\ub418\uba74\uc11c \ub450 \uc815\ud30c \uac04\uc5d0 \uc2ec\uac01\ud55c \uac08\ub4f1\uc744 \uc790\uc544\ub0c8\ub2e4. \uc608\uc1a1\ub17c\uc7c1\uc740 \ud6a8\uc885\uacfc \uc778\uc120\uc655\ud6c4\uc758 \uad6d\uc0c1(\u570b\u55aa)\uc5d0\uc11c \uc790\uc758\uc655\ub300\ube44\uc758 \ubcf5\uc81c(\u670d\u5236)\ubb38\uc81c\ub97c \uacc4\uae30\ub85c \uc77c\uc5b4\ub0ac\ub294\ub370, \ucc28\uc790\ub85c\uc11c \uc655\ud1b5\uc744 \uc774\uc740 \ud6a8\uc885\uc744 \uc801\ud1b5\uc73c\ub85c \ubcf4\ub290\ub0d0 \uc548 \ubcf4\ub290\ub0d0\uc758 \uc2dc\ube44\uc600\ub2e4. \uc11c\uc778\uc758 \uc8fc\uc7a5\uc740 \uc784\uae08\ub3c4 \uc784\uae08 \uc774\uc804\uc5d0 \uc0ac\ub300\ubd80\ub85c\uc11c\uc758 \uc608\ub97c \uc9c0\ucf1c\uc57c \ud55c\ub2e4\ub294 \uc758\ubbf8\uc774\uace0, \ub0a8\uc778\uc758 \uc8fc\uc7a5\uc740 \uc784\uae08\uc740 \uc0ac\ub300\ubd80\uc758 \uc608\uac00 \uc544\ub2cc \ub2e4\ub978 \uc608\uac00 \uac00\ub2a5\ud558\ub2e4\ub294 \uc8fc\uc7a5\uc73c\ub85c\uc11c \uacb0\uad6d \uc11c\uc778\uc740 \uc2e0\uad8c\uc911\uc2dc \uc0ac\uc0c1\uc744, \ub0a8\uc778\uc740 \uc655\uad8c\uc911\uc2dc \uc0ac\uc0c1\uc744 \ub300\ud45c\ud55c\ub2e4. 1659\ub144(\ud604\uc885 \uc6d0\ub144)\uc758 1\ucc28 \ub17c\uc7c1\uc5d0\uc11c\ub294 \uc11c\uc778\uc758 \uc8fc\uc7a5\uc774, 1674\ub144(\uc219\uc885 \uc6d0\ub144)\uc758 2\ucc28 \ub17c\uc7c1\uc5d0\uc11c\ub294 \ub0a8\uc778\uc758 \uc8fc\uc7a5\uc774 \ubc1b\uc544\ub4e4\uc5ec\uc9d0\uc73c\ub85c\uc368 \ub0a8\uc778\uc758 \uc815\uce58\uc801 \uc9c0\uc704\uac00 \uc2e0\uc7a5\ud558\uc600\ub2e4.", "answer": "1659\ub144", "sentence": "1659\ub144 (\ud604\uc885 \uc6d0\ub144)", "paragraph_sentence": "\ub0a8\uc778\ub4e4\uc740 \uc11c\uc778 \uc815\uad8c\uc774 \ucd94\uad6c\ud55c \uac1c\ud601\uc758 \ubd80\ub2f9\uc131\uacfc \ubd81\ubc8c \uc6b4\ub3d9\uc758 \ubb34\ubaa8\ud568\uc744 \uc9c0\uc801\ud558\uba74\uc11c \uc608\uc1a1\ub17c\uc7c1(\u79ae\u8a1f\u8ad6\u722d)\uc744 \uc77c\uc73c\ucf1c \uc11c\uc778\ub4e4\uacfc \uc815\uce58\uc801\uc73c\ub85c \ub300\ub9bd\ud558\uc600\ub2e4. \ub354\uad6c\ub098 \uc608\uc1a1\ub17c\uc7c1\uc774 \uc815\uccb4(\u653f\u9ad4)\ubb38\uc81c\uc640 \uad00\ub828\ub418\uba74\uc11c \ub450 \uc815\ud30c \uac04\uc5d0 \uc2ec\uac01\ud55c \uac08\ub4f1\uc744 \uc790\uc544\ub0c8\ub2e4. \uc608\uc1a1\ub17c\uc7c1\uc740 \ud6a8\uc885\uacfc \uc778\uc120\uc655\ud6c4\uc758 \uad6d\uc0c1(\u570b\u55aa)\uc5d0\uc11c \uc790\uc758\uc655\ub300\ube44\uc758 \ubcf5\uc81c(\u670d\u5236)\ubb38\uc81c\ub97c \uacc4\uae30\ub85c \uc77c\uc5b4\ub0ac\ub294\ub370, \ucc28\uc790\ub85c\uc11c \uc655\ud1b5\uc744 \uc774\uc740 \ud6a8\uc885\uc744 \uc801\ud1b5\uc73c\ub85c \ubcf4\ub290\ub0d0 \uc548 \ubcf4\ub290\ub0d0\uc758 \uc2dc\ube44\uc600\ub2e4. \uc11c\uc778\uc758 \uc8fc\uc7a5\uc740 \uc784\uae08\ub3c4 \uc784\uae08 \uc774\uc804\uc5d0 \uc0ac\ub300\ubd80\ub85c\uc11c\uc758 \uc608\ub97c \uc9c0\ucf1c\uc57c \ud55c\ub2e4\ub294 \uc758\ubbf8\uc774\uace0, \ub0a8\uc778\uc758 \uc8fc\uc7a5\uc740 \uc784\uae08\uc740 \uc0ac\ub300\ubd80\uc758 \uc608\uac00 \uc544\ub2cc \ub2e4\ub978 \uc608\uac00 \uac00\ub2a5\ud558\ub2e4\ub294 \uc8fc\uc7a5\uc73c\ub85c\uc11c \uacb0\uad6d \uc11c\uc778\uc740 \uc2e0\uad8c\uc911\uc2dc \uc0ac\uc0c1\uc744, \ub0a8\uc778\uc740 \uc655\uad8c\uc911\uc2dc \uc0ac\uc0c1\uc744 \ub300\ud45c\ud55c\ub2e4. 1659\ub144 (\ud604\uc885 \uc6d0\ub144) \uc758 1\ucc28 \ub17c\uc7c1\uc5d0\uc11c\ub294 \uc11c\uc778\uc758 \uc8fc\uc7a5\uc774, 1674\ub144(\uc219\uc885 \uc6d0\ub144)\uc758 2\ucc28 \ub17c\uc7c1\uc5d0\uc11c\ub294 \ub0a8\uc778\uc758 \uc8fc\uc7a5\uc774 \ubc1b\uc544\ub4e4\uc5ec\uc9d0\uc73c\ub85c\uc368 \ub0a8\uc778\uc758 \uc815\uce58\uc801 \uc9c0\uc704\uac00 \uc2e0\uc7a5\ud558\uc600\ub2e4.", "paragraph_answer": "\ub0a8\uc778\ub4e4\uc740 \uc11c\uc778 \uc815\uad8c\uc774 \ucd94\uad6c\ud55c \uac1c\ud601\uc758 \ubd80\ub2f9\uc131\uacfc \ubd81\ubc8c \uc6b4\ub3d9\uc758 \ubb34\ubaa8\ud568\uc744 \uc9c0\uc801\ud558\uba74\uc11c \uc608\uc1a1\ub17c\uc7c1(\u79ae\u8a1f\u8ad6\u722d)\uc744 \uc77c\uc73c\ucf1c \uc11c\uc778\ub4e4\uacfc \uc815\uce58\uc801\uc73c\ub85c \ub300\ub9bd\ud558\uc600\ub2e4. \ub354\uad6c\ub098 \uc608\uc1a1\ub17c\uc7c1\uc774 \uc815\uccb4(\u653f\u9ad4)\ubb38\uc81c\uc640 \uad00\ub828\ub418\uba74\uc11c \ub450 \uc815\ud30c \uac04\uc5d0 \uc2ec\uac01\ud55c \uac08\ub4f1\uc744 \uc790\uc544\ub0c8\ub2e4. \uc608\uc1a1\ub17c\uc7c1\uc740 \ud6a8\uc885\uacfc \uc778\uc120\uc655\ud6c4\uc758 \uad6d\uc0c1(\u570b\u55aa)\uc5d0\uc11c \uc790\uc758\uc655\ub300\ube44\uc758 \ubcf5\uc81c(\u670d\u5236)\ubb38\uc81c\ub97c \uacc4\uae30\ub85c \uc77c\uc5b4\ub0ac\ub294\ub370, \ucc28\uc790\ub85c\uc11c \uc655\ud1b5\uc744 \uc774\uc740 \ud6a8\uc885\uc744 \uc801\ud1b5\uc73c\ub85c \ubcf4\ub290\ub0d0 \uc548 \ubcf4\ub290\ub0d0\uc758 \uc2dc\ube44\uc600\ub2e4. \uc11c\uc778\uc758 \uc8fc\uc7a5\uc740 \uc784\uae08\ub3c4 \uc784\uae08 \uc774\uc804\uc5d0 \uc0ac\ub300\ubd80\ub85c\uc11c\uc758 \uc608\ub97c \uc9c0\ucf1c\uc57c \ud55c\ub2e4\ub294 \uc758\ubbf8\uc774\uace0, \ub0a8\uc778\uc758 \uc8fc\uc7a5\uc740 \uc784\uae08\uc740 \uc0ac\ub300\ubd80\uc758 \uc608\uac00 \uc544\ub2cc \ub2e4\ub978 \uc608\uac00 \uac00\ub2a5\ud558\ub2e4\ub294 \uc8fc\uc7a5\uc73c\ub85c\uc11c \uacb0\uad6d \uc11c\uc778\uc740 \uc2e0\uad8c\uc911\uc2dc \uc0ac\uc0c1\uc744, \ub0a8\uc778\uc740 \uc655\uad8c\uc911\uc2dc \uc0ac\uc0c1\uc744 \ub300\ud45c\ud55c\ub2e4. 1659\ub144 (\ud604\uc885 \uc6d0\ub144)\uc758 1\ucc28 \ub17c\uc7c1\uc5d0\uc11c\ub294 \uc11c\uc778\uc758 \uc8fc\uc7a5\uc774, 1674\ub144(\uc219\uc885 \uc6d0\ub144)\uc758 2\ucc28 \ub17c\uc7c1\uc5d0\uc11c\ub294 \ub0a8\uc778\uc758 \uc8fc\uc7a5\uc774 \ubc1b\uc544\ub4e4\uc5ec\uc9d0\uc73c\ub85c\uc368 \ub0a8\uc778\uc758 \uc815\uce58\uc801 \uc9c0\uc704\uac00 \uc2e0\uc7a5\ud558\uc600\ub2e4.", "sentence_answer": " 1659\ub144 (\ud604\uc885 \uc6d0\ub144)", "paragraph_id": "6187890-9-1", "paragraph_question": "question: 1\ucc28 \ub17c\uc7c1\uc5d0\uc11c\ub294 \uc11c\uc778\uc758 \uc8fc\uc7a5\uc774 \uc77c\uc5b4\ub09c \ud574\ub294?, context: \ub0a8\uc778\ub4e4\uc740 \uc11c\uc778 \uc815\uad8c\uc774 \ucd94\uad6c\ud55c \uac1c\ud601\uc758 \ubd80\ub2f9\uc131\uacfc \ubd81\ubc8c \uc6b4\ub3d9\uc758 \ubb34\ubaa8\ud568\uc744 \uc9c0\uc801\ud558\uba74\uc11c \uc608\uc1a1\ub17c\uc7c1(\u79ae\u8a1f\u8ad6\u722d)\uc744 \uc77c\uc73c\ucf1c \uc11c\uc778\ub4e4\uacfc \uc815\uce58\uc801\uc73c\ub85c \ub300\ub9bd\ud558\uc600\ub2e4. \ub354\uad6c\ub098 \uc608\uc1a1\ub17c\uc7c1\uc774 \uc815\uccb4(\u653f\u9ad4)\ubb38\uc81c\uc640 \uad00\ub828\ub418\uba74\uc11c \ub450 \uc815\ud30c \uac04\uc5d0 \uc2ec\uac01\ud55c \uac08\ub4f1\uc744 \uc790\uc544\ub0c8\ub2e4. \uc608\uc1a1\ub17c\uc7c1\uc740 \ud6a8\uc885\uacfc \uc778\uc120\uc655\ud6c4\uc758 \uad6d\uc0c1(\u570b\u55aa)\uc5d0\uc11c \uc790\uc758\uc655\ub300\ube44\uc758 \ubcf5\uc81c(\u670d\u5236)\ubb38\uc81c\ub97c \uacc4\uae30\ub85c \uc77c\uc5b4\ub0ac\ub294\ub370, \ucc28\uc790\ub85c\uc11c \uc655\ud1b5\uc744 \uc774\uc740 \ud6a8\uc885\uc744 \uc801\ud1b5\uc73c\ub85c \ubcf4\ub290\ub0d0 \uc548 \ubcf4\ub290\ub0d0\uc758 \uc2dc\ube44\uc600\ub2e4. \uc11c\uc778\uc758 \uc8fc\uc7a5\uc740 \uc784\uae08\ub3c4 \uc784\uae08 \uc774\uc804\uc5d0 \uc0ac\ub300\ubd80\ub85c\uc11c\uc758 \uc608\ub97c \uc9c0\ucf1c\uc57c \ud55c\ub2e4\ub294 \uc758\ubbf8\uc774\uace0, \ub0a8\uc778\uc758 \uc8fc\uc7a5\uc740 \uc784\uae08\uc740 \uc0ac\ub300\ubd80\uc758 \uc608\uac00 \uc544\ub2cc \ub2e4\ub978 \uc608\uac00 \uac00\ub2a5\ud558\ub2e4\ub294 \uc8fc\uc7a5\uc73c\ub85c\uc11c \uacb0\uad6d \uc11c\uc778\uc740 \uc2e0\uad8c\uc911\uc2dc \uc0ac\uc0c1\uc744, \ub0a8\uc778\uc740 \uc655\uad8c\uc911\uc2dc \uc0ac\uc0c1\uc744 \ub300\ud45c\ud55c\ub2e4. 1659\ub144(\ud604\uc885 \uc6d0\ub144)\uc758 1\ucc28 \ub17c\uc7c1\uc5d0\uc11c\ub294 \uc11c\uc778\uc758 \uc8fc\uc7a5\uc774, 1674\ub144(\uc219\uc885 \uc6d0\ub144)\uc758 2\ucc28 \ub17c\uc7c1\uc5d0\uc11c\ub294 \ub0a8\uc778\uc758 \uc8fc\uc7a5\uc774 \ubc1b\uc544\ub4e4\uc5ec\uc9d0\uc73c\ub85c\uc368 \ub0a8\uc778\uc758 \uc815\uce58\uc801 \uc9c0\uc704\uac00 \uc2e0\uc7a5\ud558\uc600\ub2e4."} {"question": "\ub3c5\uc77c\ubcf4\ub2e4 \uc62c\ub9bc\ud53d \ubcf8\uc120\uc5d0 \ub354 \uc790\uc8fc \uc9c4\ucd9c\ud560 \uc218 \uc788\uc5c8\ub358 \uad6d\uac00\ub294?", "paragraph": "1930\ub144 FIFA \uc6d4\ub4dc\ucef5\ubd80\ud130 2014\ub144 FIFA \uc6d4\ub4dc\ucef5\uae4c\uc9c0 \uc2a4\ubb34 \ubc88\uc758 \uc6d4\ub4dc\ucef5\uc5d0\uc11c \uac01 \uad6d\uac00\uac00 \uae30\ub85d\ud55c \uc131\uc801\uc744 \uc21c\uc704\ubcc4\ub85c \ub098\uc5f4\ud55c \ud1b5\uacc4\ud45c\ub2e4. \ucd1d 77\uac1c\uad6d\uc774 \uc6d4\ub4dc\ucef5 \ubcf8\uc120\uc5d0 \uc9c4\ucd9c\ud588\uc73c\uba70, \uc774 \uc911 \ub3d9\ub3c5\uc740 \ud1b5\uc77c \uc774\ud6c4 \uad6d\uac00\uac00 \uc0ac\ub77c\uc84c\uace0 \uc774\uc2a4\ub77c\uc5d8\uc740 \uc544\uc2dc\uc544 \uc18c\uc18d\uc77c\ub54c\uc758 \uc131\uc801, \ud638\uc8fc\ub294 2006\ub144\uae4c\uc9c0\ub294 \uc624\uc138\uc544\ub2c8\uc544 \uc131\uc801\uc774\ub098 \uadf8 \uc774\ud6c4\ub85c\ub294 \uc544\uc2dc\uc544 \uc18c\uc18d\uc73c\ub85c \uac70\ub454 \uc131\uc801\uc774\ub2e4. \ucd5c\ub2e4 \uc9c4\ucd9c\uad6d\uc740 \ube0c\ub77c\uc9c8\ub85c\uc11c \ub2e8 \ud55c\ubc88\ub3c4 \ube60\uc9c0\uc9c0 \uc54a\uace0 \ubaa8\ub4e0 \ub300\ud68c(20\ud68c)\uc5d0 \uc9c4\ucd9c\ud588\uc73c\uba70, \ub3c5\uc77c\uacfc \uc774\ud0c8\ub9ac\uc544\uac00 18\ud68c\ub85c \uadf8 \ub2e4\uc74c\uc744 \uc787\uace0 \uc788\ub2e4. \ub300\ud55c\ubbfc\uad6d\uc740 \ucd1d 9\ud68c \uc9c4\ucd9c\ud574 \uc544\uc2dc\uc544 \ucd5c\ub2e4\uc9c4\ucd9c\uad6d \uae30\ub85d\uc744 \uac00\uc9c0\uace0 \uc788\uc744 \ubfd0\ub354\ub7ec \uc720\ub7fd\uacfc \ub0a8\ubbf8 \ub300\ub959\uc744 \uc81c\uc678\ud55c \ubaa8\ub4e0 \uad6d\uac00\ub4e4 \uc911 \uac00\uc7a5 \ub192\uc740 \uc131\uc801(4\uc704)\uc744 \uac00\uc9c0\uace0 \uc788\ub2e4.", "answer": "\ube0c\ub77c\uc9c8", "sentence": "\ucd5c\ub2e4 \uc9c4\ucd9c\uad6d\uc740 \ube0c\ub77c\uc9c8 \ub85c\uc11c \ub2e8 \ud55c\ubc88\ub3c4 \ube60\uc9c0\uc9c0 \uc54a\uace0 \ubaa8\ub4e0 \ub300\ud68c(20\ud68c)\uc5d0 \uc9c4\ucd9c\ud588\uc73c\uba70, \ub3c5\uc77c\uacfc \uc774\ud0c8\ub9ac\uc544\uac00 18\ud68c\ub85c \uadf8 \ub2e4\uc74c\uc744 \uc787\uace0 \uc788\ub2e4.", "paragraph_sentence": "1930\ub144 FIFA \uc6d4\ub4dc\ucef5\ubd80\ud130 2014\ub144 FIFA \uc6d4\ub4dc\ucef5\uae4c\uc9c0 \uc2a4\ubb34 \ubc88\uc758 \uc6d4\ub4dc\ucef5\uc5d0\uc11c \uac01 \uad6d\uac00\uac00 \uae30\ub85d\ud55c \uc131\uc801\uc744 \uc21c\uc704\ubcc4\ub85c \ub098\uc5f4\ud55c \ud1b5\uacc4\ud45c\ub2e4. \ucd1d 77\uac1c\uad6d\uc774 \uc6d4\ub4dc\ucef5 \ubcf8\uc120\uc5d0 \uc9c4\ucd9c\ud588\uc73c\uba70, \uc774 \uc911 \ub3d9\ub3c5\uc740 \ud1b5\uc77c \uc774\ud6c4 \uad6d\uac00\uac00 \uc0ac\ub77c\uc84c\uace0 \uc774\uc2a4\ub77c\uc5d8\uc740 \uc544\uc2dc\uc544 \uc18c\uc18d\uc77c\ub54c\uc758 \uc131\uc801, \ud638\uc8fc\ub294 2006\ub144\uae4c\uc9c0\ub294 \uc624\uc138\uc544\ub2c8\uc544 \uc131\uc801\uc774\ub098 \uadf8 \uc774\ud6c4\ub85c\ub294 \uc544\uc2dc\uc544 \uc18c\uc18d\uc73c\ub85c \uac70\ub454 \uc131\uc801\uc774\ub2e4. \ucd5c\ub2e4 \uc9c4\ucd9c\uad6d\uc740 \ube0c\ub77c\uc9c8 \ub85c\uc11c \ub2e8 \ud55c\ubc88\ub3c4 \ube60\uc9c0\uc9c0 \uc54a\uace0 \ubaa8\ub4e0 \ub300\ud68c(20\ud68c)\uc5d0 \uc9c4\ucd9c\ud588\uc73c\uba70, \ub3c5\uc77c\uacfc \uc774\ud0c8\ub9ac\uc544\uac00 18\ud68c\ub85c \uadf8 \ub2e4\uc74c\uc744 \uc787\uace0 \uc788\ub2e4. \ub300\ud55c\ubbfc\uad6d\uc740 \ucd1d 9\ud68c \uc9c4\ucd9c\ud574 \uc544\uc2dc\uc544 \ucd5c\ub2e4\uc9c4\ucd9c\uad6d \uae30\ub85d\uc744 \uac00\uc9c0\uace0 \uc788\uc744 \ubfd0\ub354\ub7ec \uc720\ub7fd\uacfc \ub0a8\ubbf8 \ub300\ub959\uc744 \uc81c\uc678\ud55c \ubaa8\ub4e0 \uad6d\uac00\ub4e4 \uc911 \uac00\uc7a5 \ub192\uc740 \uc131\uc801(4\uc704)\uc744 \uac00\uc9c0\uace0 \uc788\ub2e4.", "paragraph_answer": "1930\ub144 FIFA \uc6d4\ub4dc\ucef5\ubd80\ud130 2014\ub144 FIFA \uc6d4\ub4dc\ucef5\uae4c\uc9c0 \uc2a4\ubb34 \ubc88\uc758 \uc6d4\ub4dc\ucef5\uc5d0\uc11c \uac01 \uad6d\uac00\uac00 \uae30\ub85d\ud55c \uc131\uc801\uc744 \uc21c\uc704\ubcc4\ub85c \ub098\uc5f4\ud55c \ud1b5\uacc4\ud45c\ub2e4. \ucd1d 77\uac1c\uad6d\uc774 \uc6d4\ub4dc\ucef5 \ubcf8\uc120\uc5d0 \uc9c4\ucd9c\ud588\uc73c\uba70, \uc774 \uc911 \ub3d9\ub3c5\uc740 \ud1b5\uc77c \uc774\ud6c4 \uad6d\uac00\uac00 \uc0ac\ub77c\uc84c\uace0 \uc774\uc2a4\ub77c\uc5d8\uc740 \uc544\uc2dc\uc544 \uc18c\uc18d\uc77c\ub54c\uc758 \uc131\uc801, \ud638\uc8fc\ub294 2006\ub144\uae4c\uc9c0\ub294 \uc624\uc138\uc544\ub2c8\uc544 \uc131\uc801\uc774\ub098 \uadf8 \uc774\ud6c4\ub85c\ub294 \uc544\uc2dc\uc544 \uc18c\uc18d\uc73c\ub85c \uac70\ub454 \uc131\uc801\uc774\ub2e4. \ucd5c\ub2e4 \uc9c4\ucd9c\uad6d\uc740 \ube0c\ub77c\uc9c8 \ub85c\uc11c \ub2e8 \ud55c\ubc88\ub3c4 \ube60\uc9c0\uc9c0 \uc54a\uace0 \ubaa8\ub4e0 \ub300\ud68c(20\ud68c)\uc5d0 \uc9c4\ucd9c\ud588\uc73c\uba70, \ub3c5\uc77c\uacfc \uc774\ud0c8\ub9ac\uc544\uac00 18\ud68c\ub85c \uadf8 \ub2e4\uc74c\uc744 \uc787\uace0 \uc788\ub2e4. \ub300\ud55c\ubbfc\uad6d\uc740 \ucd1d 9\ud68c \uc9c4\ucd9c\ud574 \uc544\uc2dc\uc544 \ucd5c\ub2e4\uc9c4\ucd9c\uad6d \uae30\ub85d\uc744 \uac00\uc9c0\uace0 \uc788\uc744 \ubfd0\ub354\ub7ec \uc720\ub7fd\uacfc \ub0a8\ubbf8 \ub300\ub959\uc744 \uc81c\uc678\ud55c \ubaa8\ub4e0 \uad6d\uac00\ub4e4 \uc911 \uac00\uc7a5 \ub192\uc740 \uc131\uc801(4\uc704)\uc744 \uac00\uc9c0\uace0 \uc788\ub2e4.", "sentence_answer": "\ucd5c\ub2e4 \uc9c4\ucd9c\uad6d\uc740 \ube0c\ub77c\uc9c8 \ub85c\uc11c \ub2e8 \ud55c\ubc88\ub3c4 \ube60\uc9c0\uc9c0 \uc54a\uace0 \ubaa8\ub4e0 \ub300\ud68c(20\ud68c)\uc5d0 \uc9c4\ucd9c\ud588\uc73c\uba70, \ub3c5\uc77c\uacfc \uc774\ud0c8\ub9ac\uc544\uac00 18\ud68c\ub85c \uadf8 \ub2e4\uc74c\uc744 \uc787\uace0 \uc788\ub2e4.", "paragraph_id": "6523258-11-2", "paragraph_question": "question: \ub3c5\uc77c\ubcf4\ub2e4 \uc62c\ub9bc\ud53d \ubcf8\uc120\uc5d0 \ub354 \uc790\uc8fc \uc9c4\ucd9c\ud560 \uc218 \uc788\uc5c8\ub358 \uad6d\uac00\ub294?, context: 1930\ub144 FIFA \uc6d4\ub4dc\ucef5\ubd80\ud130 2014\ub144 FIFA \uc6d4\ub4dc\ucef5\uae4c\uc9c0 \uc2a4\ubb34 \ubc88\uc758 \uc6d4\ub4dc\ucef5\uc5d0\uc11c \uac01 \uad6d\uac00\uac00 \uae30\ub85d\ud55c \uc131\uc801\uc744 \uc21c\uc704\ubcc4\ub85c \ub098\uc5f4\ud55c \ud1b5\uacc4\ud45c\ub2e4. \ucd1d 77\uac1c\uad6d\uc774 \uc6d4\ub4dc\ucef5 \ubcf8\uc120\uc5d0 \uc9c4\ucd9c\ud588\uc73c\uba70, \uc774 \uc911 \ub3d9\ub3c5\uc740 \ud1b5\uc77c \uc774\ud6c4 \uad6d\uac00\uac00 \uc0ac\ub77c\uc84c\uace0 \uc774\uc2a4\ub77c\uc5d8\uc740 \uc544\uc2dc\uc544 \uc18c\uc18d\uc77c\ub54c\uc758 \uc131\uc801, \ud638\uc8fc\ub294 2006\ub144\uae4c\uc9c0\ub294 \uc624\uc138\uc544\ub2c8\uc544 \uc131\uc801\uc774\ub098 \uadf8 \uc774\ud6c4\ub85c\ub294 \uc544\uc2dc\uc544 \uc18c\uc18d\uc73c\ub85c \uac70\ub454 \uc131\uc801\uc774\ub2e4. \ucd5c\ub2e4 \uc9c4\ucd9c\uad6d\uc740 \ube0c\ub77c\uc9c8\ub85c\uc11c \ub2e8 \ud55c\ubc88\ub3c4 \ube60\uc9c0\uc9c0 \uc54a\uace0 \ubaa8\ub4e0 \ub300\ud68c(20\ud68c)\uc5d0 \uc9c4\ucd9c\ud588\uc73c\uba70, \ub3c5\uc77c\uacfc \uc774\ud0c8\ub9ac\uc544\uac00 18\ud68c\ub85c \uadf8 \ub2e4\uc74c\uc744 \uc787\uace0 \uc788\ub2e4. \ub300\ud55c\ubbfc\uad6d\uc740 \ucd1d 9\ud68c \uc9c4\ucd9c\ud574 \uc544\uc2dc\uc544 \ucd5c\ub2e4\uc9c4\ucd9c\uad6d \uae30\ub85d\uc744 \uac00\uc9c0\uace0 \uc788\uc744 \ubfd0\ub354\ub7ec \uc720\ub7fd\uacfc \ub0a8\ubbf8 \ub300\ub959\uc744 \uc81c\uc678\ud55c \ubaa8\ub4e0 \uad6d\uac00\ub4e4 \uc911 \uac00\uc7a5 \ub192\uc740 \uc131\uc801(4\uc704)\uc744 \uac00\uc9c0\uace0 \uc788\ub2e4."} {"question": "\ubc15\ubcf4\uc601\uc774 \ubc30\uc6b0\uc758 \uc9c1\uc5c5\uc801 \uc7a5\uc810\uc774\ub77c\uace0 \ub9d0\ud55c \uac83\uc740?", "paragraph": "2015\ub144 10\uc6d4 \uc601\ud654 \u300a\ub3cc\uc5f0\ubcc0\uc774\u300b\uac00 \uac1c\ubd09\ub410\ub2e4. \ubc15\ubcf4\uc601\uc740 \uc0dd\uc120\uc778\uac04\uc774 \ub41c \uc804 \ub0a8\uc790\uce5c\uad6c \ubc15\uad6c(\uc774\uad11\uc218)\uc758 \uc774\uc57c\uae30\ub97c \uc778\ud130\ub137\uc5d0 \ud37c\ub098\ub974\uba70 \u2018\uc774\uc288\ub140\u2019\uac00 \ub418\uae38 \ubc14\ub77c\ub294 \u2018\ud0a4\ubcf4\ub4dc \uc6cc\ub9ac\uc5b4\u2019 \uc8fc\uc9c4 \uc5ed\uc744 \ub9e1\uc558\ub2e4. \ucc38\uace0\ub85c \uc774 \uce90\ub9ad\ud130\uc758 \ubaa8\ud2f0\ube0c\ub294 \uad6d\uac00\uc815\ubcf4\uc6d0 \uc5ec\ub860 \uc870\uc791 \uc0ac\uac74\uc5d0\uc11c \uc545\ud50c\uc744 \ub2ec\uc558\ub358 \uad6d\uc815\uc6d0 \uc5ec\uc9c1\uc6d0\uc774\ub2e4. \uacb0\ub9d0\uc5d0\uc11c \uc790\uc2e0\uc758 \ud2b9\uc131\uc778 \u2018\ud0a4\ubcf4\ub4dc \uc6cc\ub9ac\uc5b4\u2019\ub97c \uc0b4\ub824 \uacf5\ubb34\uc6d0\uc774 \ub418\ub294\ub370 \uad6d\uc815\uc6d0 \ub313\uae00\ub140\ub97c \uc5f0\uc0c1\ucf00 \ud558\ub294 \ub178\uace8\uc801\uc778 \uc7a5\uba74\uc774 \uc788\uc5c8\uc73c\ub098 \ub108\ubb34 \uc9c1\uc811\uc801\uc778 \ud45c\ud604\uc774\ub77c \ud3b8\uc9d1 \ub2e8\uacc4\uc5d0\uc11c \ube60\uc84c\ub2e4\uace0 \ud55c\ub2e4. \uc5ed\ub7c9 \uc788\ub294 \uc2e0\uc778 \uac10\ub3c5\uc744 \ubc1c\uad74\ud558\ub294 \ubaa9\uc801\uc73c\ub85c \ud0c4\uc0dd\ud55c CJ E&M \ubc84\ud130\ud50c\ub77c\uc774 \ud504\ub85c\uc81d\ud2b8 \uccab \ubc88\uc9f8 \uc791\ud488\uc778 \uc774 \uc601\ud654\ub294 \uc81c40\ud68c \ud1a0\ub860\ud1a0 \uad6d\uc81c \uc601\ud654\uc81c \ubc45\uac00\ub4dc \uc139\uc158 \ubd80\ubb38\uc5d0 \uacf5\uc2dd \ucd08\uccad\ub418\uae30\ub3c4 \ud588\ub2e4. \uc800\uc608\uc0b0\uc5d0 \ucd9c\uc5f0\ub8cc 1/10\ub3c4 \uc548\ubc1b\uc740 \uc870\uc5f0 \ubd84\ub7c9\uc758 \uc601\ud654\uc600\uc9c0\ub9cc \ubc15\ubcf4\uc601\uc740 \uc774 \uc791\ud488\uc744 \ud1b5\ud574 \uc5f0\uae30\ud558\ub294 \uc7ac\ubbf8\ub97c \ub2e4\uc2dc \uae68\ub2ec\uc558\ub2e4\uace0 \ud55c\ub2e4. \uadf8\ub294 \uc81c\uc791\ubc1c\ud45c\ud68c\uc5d0\uc11c \uccad\ub144 \uc2e4\uc5c5\uc744 \uacaa\uc5b4\ubcf8 \uc801\uc740 \uc5c6\uc9c0\ub9cc \uc8fc\uc704\uc5d0 \uacaa\uace0 \uc788\ub294 \uce5c\uad6c\ub4e4\uc774 \uc788\uae30\uc5d0 \uc0ac\ub78c\ub4e4\uc774 \uc0ac\ud68c\uc801\uc778 \ubb38\uc81c\ub97c \uc0dd\uac01\ud574\ubd24\uc73c\uba74 \uc88b\uaca0\ub2e4\ub294 \ub9c8\uc74c\uc5d0\uc11c \uc5f0\uae30\ud588\ub2e4\u201d\uba70 \ubc1d\ud614\uace0 \ubc30\uc6b0\ub85c\uc11c \uc791\ud488\uc5d0 \ucc38\uc5ec\ud558\uace0 \uce90\ub9ad\ud130\ub97c \uc5f0\uae30\ud574 \uc0ac\ud68c \ubb38\uc81c\ub97c \ub9d0\ud558\ub294 \uac83\uc774 \uc790\uc2e0\uc758 \uc9c1\uc5c5\uc758 \uc7a5\uc810\uc774\ub77c \ub73b\uae4a\uac8c \ucd9c\uc5f0\ud588\ub2e4\uace0 \ud558\uc5ec \ubc30\uc6b0\ub85c\uc11c\uc758 \uc0ac\ud68c\uc801 \ucc45\uc784\uc5d0 \ub300\ud574 \ub9d0\ud574 \ubc15\uc218\ub97c \ubc1b\uae30\ub3c4 \ud588\ub2e4.", "answer": "\uc0ac\ud68c \ubb38\uc81c\ub97c \ub9d0\ud558\ub294 \uac83", "sentence": "\uadf8\ub294 \uc81c\uc791\ubc1c\ud45c\ud68c\uc5d0\uc11c \uccad\ub144 \uc2e4\uc5c5\uc744 \uacaa\uc5b4\ubcf8 \uc801\uc740 \uc5c6\uc9c0\ub9cc \uc8fc\uc704\uc5d0 \uacaa\uace0 \uc788\ub294 \uce5c\uad6c\ub4e4\uc774 \uc788\uae30\uc5d0 \uc0ac\ub78c\ub4e4\uc774 \uc0ac\ud68c\uc801\uc778 \ubb38\uc81c\ub97c \uc0dd\uac01\ud574\ubd24\uc73c\uba74 \uc88b\uaca0\ub2e4\ub294 \ub9c8\uc74c\uc5d0\uc11c \uc5f0\uae30\ud588\ub2e4\u201d\uba70 \ubc1d\ud614\uace0 \ubc30\uc6b0\ub85c\uc11c \uc791\ud488\uc5d0 \ucc38\uc5ec\ud558\uace0 \uce90\ub9ad\ud130\ub97c \uc5f0\uae30\ud574 \uc0ac\ud68c \ubb38\uc81c\ub97c \ub9d0\ud558\ub294 \uac83 \uc774 \uc790\uc2e0\uc758 \uc9c1\uc5c5\uc758 \uc7a5\uc810\uc774\ub77c \ub73b\uae4a\uac8c \ucd9c\uc5f0\ud588\ub2e4\uace0 \ud558\uc5ec \ubc30\uc6b0\ub85c\uc11c\uc758 \uc0ac\ud68c\uc801 \ucc45\uc784\uc5d0 \ub300\ud574 \ub9d0\ud574 \ubc15\uc218\ub97c \ubc1b\uae30\ub3c4 \ud588\ub2e4.", "paragraph_sentence": "2015\ub144 10\uc6d4 \uc601\ud654 \u300a\ub3cc\uc5f0\ubcc0\uc774\u300b\uac00 \uac1c\ubd09\ub410\ub2e4. \ubc15\ubcf4\uc601\uc740 \uc0dd\uc120\uc778\uac04\uc774 \ub41c \uc804 \ub0a8\uc790\uce5c\uad6c \ubc15\uad6c(\uc774\uad11\uc218)\uc758 \uc774\uc57c\uae30\ub97c \uc778\ud130\ub137\uc5d0 \ud37c\ub098\ub974\uba70 \u2018\uc774\uc288\ub140\u2019\uac00 \ub418\uae38 \ubc14\ub77c\ub294 \u2018\ud0a4\ubcf4\ub4dc \uc6cc\ub9ac\uc5b4\u2019 \uc8fc\uc9c4 \uc5ed\uc744 \ub9e1\uc558\ub2e4. \ucc38\uace0\ub85c \uc774 \uce90\ub9ad\ud130\uc758 \ubaa8\ud2f0\ube0c\ub294 \uad6d\uac00\uc815\ubcf4\uc6d0 \uc5ec\ub860 \uc870\uc791 \uc0ac\uac74\uc5d0\uc11c \uc545\ud50c\uc744 \ub2ec\uc558\ub358 \uad6d\uc815\uc6d0 \uc5ec\uc9c1\uc6d0\uc774\ub2e4. \uacb0\ub9d0\uc5d0\uc11c \uc790\uc2e0\uc758 \ud2b9\uc131\uc778 \u2018\ud0a4\ubcf4\ub4dc \uc6cc\ub9ac\uc5b4\u2019\ub97c \uc0b4\ub824 \uacf5\ubb34\uc6d0\uc774 \ub418\ub294\ub370 \uad6d\uc815\uc6d0 \ub313\uae00\ub140\ub97c \uc5f0\uc0c1\ucf00 \ud558\ub294 \ub178\uace8\uc801\uc778 \uc7a5\uba74\uc774 \uc788\uc5c8\uc73c\ub098 \ub108\ubb34 \uc9c1\uc811\uc801\uc778 \ud45c\ud604\uc774\ub77c \ud3b8\uc9d1 \ub2e8\uacc4\uc5d0\uc11c \ube60\uc84c\ub2e4\uace0 \ud55c\ub2e4. \uc5ed\ub7c9 \uc788\ub294 \uc2e0\uc778 \uac10\ub3c5\uc744 \ubc1c\uad74\ud558\ub294 \ubaa9\uc801\uc73c\ub85c \ud0c4\uc0dd\ud55c CJ E&M \ubc84\ud130\ud50c\ub77c\uc774 \ud504\ub85c\uc81d\ud2b8 \uccab \ubc88\uc9f8 \uc791\ud488\uc778 \uc774 \uc601\ud654\ub294 \uc81c40\ud68c \ud1a0\ub860\ud1a0 \uad6d\uc81c \uc601\ud654\uc81c \ubc45\uac00\ub4dc \uc139\uc158 \ubd80\ubb38\uc5d0 \uacf5\uc2dd \ucd08\uccad\ub418\uae30\ub3c4 \ud588\ub2e4. \uc800\uc608\uc0b0\uc5d0 \ucd9c\uc5f0\ub8cc 1/10\ub3c4 \uc548\ubc1b\uc740 \uc870\uc5f0 \ubd84\ub7c9\uc758 \uc601\ud654\uc600\uc9c0\ub9cc \ubc15\ubcf4\uc601\uc740 \uc774 \uc791\ud488\uc744 \ud1b5\ud574 \uc5f0\uae30\ud558\ub294 \uc7ac\ubbf8\ub97c \ub2e4\uc2dc \uae68\ub2ec\uc558\ub2e4\uace0 \ud55c\ub2e4. \uadf8\ub294 \uc81c\uc791\ubc1c\ud45c\ud68c\uc5d0\uc11c \uccad\ub144 \uc2e4\uc5c5\uc744 \uacaa\uc5b4\ubcf8 \uc801\uc740 \uc5c6\uc9c0\ub9cc \uc8fc\uc704\uc5d0 \uacaa\uace0 \uc788\ub294 \uce5c\uad6c\ub4e4\uc774 \uc788\uae30\uc5d0 \uc0ac\ub78c\ub4e4\uc774 \uc0ac\ud68c\uc801\uc778 \ubb38\uc81c\ub97c \uc0dd\uac01\ud574\ubd24\uc73c\uba74 \uc88b\uaca0\ub2e4\ub294 \ub9c8\uc74c\uc5d0\uc11c \uc5f0\uae30\ud588\ub2e4\u201d\uba70 \ubc1d\ud614\uace0 \ubc30\uc6b0\ub85c\uc11c \uc791\ud488\uc5d0 \ucc38\uc5ec\ud558\uace0 \uce90\ub9ad\ud130\ub97c \uc5f0\uae30\ud574 \uc0ac\ud68c \ubb38\uc81c\ub97c \ub9d0\ud558\ub294 \uac83 \uc774 \uc790\uc2e0\uc758 \uc9c1\uc5c5\uc758 \uc7a5\uc810\uc774\ub77c \ub73b\uae4a\uac8c \ucd9c\uc5f0\ud588\ub2e4\uace0 \ud558\uc5ec \ubc30\uc6b0\ub85c\uc11c\uc758 \uc0ac\ud68c\uc801 \ucc45\uc784\uc5d0 \ub300\ud574 \ub9d0\ud574 \ubc15\uc218\ub97c \ubc1b\uae30\ub3c4 \ud588\ub2e4. ", "paragraph_answer": "2015\ub144 10\uc6d4 \uc601\ud654 \u300a\ub3cc\uc5f0\ubcc0\uc774\u300b\uac00 \uac1c\ubd09\ub410\ub2e4. \ubc15\ubcf4\uc601\uc740 \uc0dd\uc120\uc778\uac04\uc774 \ub41c \uc804 \ub0a8\uc790\uce5c\uad6c \ubc15\uad6c(\uc774\uad11\uc218)\uc758 \uc774\uc57c\uae30\ub97c \uc778\ud130\ub137\uc5d0 \ud37c\ub098\ub974\uba70 \u2018\uc774\uc288\ub140\u2019\uac00 \ub418\uae38 \ubc14\ub77c\ub294 \u2018\ud0a4\ubcf4\ub4dc \uc6cc\ub9ac\uc5b4\u2019 \uc8fc\uc9c4 \uc5ed\uc744 \ub9e1\uc558\ub2e4. \ucc38\uace0\ub85c \uc774 \uce90\ub9ad\ud130\uc758 \ubaa8\ud2f0\ube0c\ub294 \uad6d\uac00\uc815\ubcf4\uc6d0 \uc5ec\ub860 \uc870\uc791 \uc0ac\uac74\uc5d0\uc11c \uc545\ud50c\uc744 \ub2ec\uc558\ub358 \uad6d\uc815\uc6d0 \uc5ec\uc9c1\uc6d0\uc774\ub2e4. \uacb0\ub9d0\uc5d0\uc11c \uc790\uc2e0\uc758 \ud2b9\uc131\uc778 \u2018\ud0a4\ubcf4\ub4dc \uc6cc\ub9ac\uc5b4\u2019\ub97c \uc0b4\ub824 \uacf5\ubb34\uc6d0\uc774 \ub418\ub294\ub370 \uad6d\uc815\uc6d0 \ub313\uae00\ub140\ub97c \uc5f0\uc0c1\ucf00 \ud558\ub294 \ub178\uace8\uc801\uc778 \uc7a5\uba74\uc774 \uc788\uc5c8\uc73c\ub098 \ub108\ubb34 \uc9c1\uc811\uc801\uc778 \ud45c\ud604\uc774\ub77c \ud3b8\uc9d1 \ub2e8\uacc4\uc5d0\uc11c \ube60\uc84c\ub2e4\uace0 \ud55c\ub2e4. \uc5ed\ub7c9 \uc788\ub294 \uc2e0\uc778 \uac10\ub3c5\uc744 \ubc1c\uad74\ud558\ub294 \ubaa9\uc801\uc73c\ub85c \ud0c4\uc0dd\ud55c CJ E&M \ubc84\ud130\ud50c\ub77c\uc774 \ud504\ub85c\uc81d\ud2b8 \uccab \ubc88\uc9f8 \uc791\ud488\uc778 \uc774 \uc601\ud654\ub294 \uc81c40\ud68c \ud1a0\ub860\ud1a0 \uad6d\uc81c \uc601\ud654\uc81c \ubc45\uac00\ub4dc \uc139\uc158 \ubd80\ubb38\uc5d0 \uacf5\uc2dd \ucd08\uccad\ub418\uae30\ub3c4 \ud588\ub2e4. \uc800\uc608\uc0b0\uc5d0 \ucd9c\uc5f0\ub8cc 1/10\ub3c4 \uc548\ubc1b\uc740 \uc870\uc5f0 \ubd84\ub7c9\uc758 \uc601\ud654\uc600\uc9c0\ub9cc \ubc15\ubcf4\uc601\uc740 \uc774 \uc791\ud488\uc744 \ud1b5\ud574 \uc5f0\uae30\ud558\ub294 \uc7ac\ubbf8\ub97c \ub2e4\uc2dc \uae68\ub2ec\uc558\ub2e4\uace0 \ud55c\ub2e4. \uadf8\ub294 \uc81c\uc791\ubc1c\ud45c\ud68c\uc5d0\uc11c \uccad\ub144 \uc2e4\uc5c5\uc744 \uacaa\uc5b4\ubcf8 \uc801\uc740 \uc5c6\uc9c0\ub9cc \uc8fc\uc704\uc5d0 \uacaa\uace0 \uc788\ub294 \uce5c\uad6c\ub4e4\uc774 \uc788\uae30\uc5d0 \uc0ac\ub78c\ub4e4\uc774 \uc0ac\ud68c\uc801\uc778 \ubb38\uc81c\ub97c \uc0dd\uac01\ud574\ubd24\uc73c\uba74 \uc88b\uaca0\ub2e4\ub294 \ub9c8\uc74c\uc5d0\uc11c \uc5f0\uae30\ud588\ub2e4\u201d\uba70 \ubc1d\ud614\uace0 \ubc30\uc6b0\ub85c\uc11c \uc791\ud488\uc5d0 \ucc38\uc5ec\ud558\uace0 \uce90\ub9ad\ud130\ub97c \uc5f0\uae30\ud574 \uc0ac\ud68c \ubb38\uc81c\ub97c \ub9d0\ud558\ub294 \uac83 \uc774 \uc790\uc2e0\uc758 \uc9c1\uc5c5\uc758 \uc7a5\uc810\uc774\ub77c \ub73b\uae4a\uac8c \ucd9c\uc5f0\ud588\ub2e4\uace0 \ud558\uc5ec \ubc30\uc6b0\ub85c\uc11c\uc758 \uc0ac\ud68c\uc801 \ucc45\uc784\uc5d0 \ub300\ud574 \ub9d0\ud574 \ubc15\uc218\ub97c \ubc1b\uae30\ub3c4 \ud588\ub2e4.", "sentence_answer": "\uadf8\ub294 \uc81c\uc791\ubc1c\ud45c\ud68c\uc5d0\uc11c \uccad\ub144 \uc2e4\uc5c5\uc744 \uacaa\uc5b4\ubcf8 \uc801\uc740 \uc5c6\uc9c0\ub9cc \uc8fc\uc704\uc5d0 \uacaa\uace0 \uc788\ub294 \uce5c\uad6c\ub4e4\uc774 \uc788\uae30\uc5d0 \uc0ac\ub78c\ub4e4\uc774 \uc0ac\ud68c\uc801\uc778 \ubb38\uc81c\ub97c \uc0dd\uac01\ud574\ubd24\uc73c\uba74 \uc88b\uaca0\ub2e4\ub294 \ub9c8\uc74c\uc5d0\uc11c \uc5f0\uae30\ud588\ub2e4\u201d\uba70 \ubc1d\ud614\uace0 \ubc30\uc6b0\ub85c\uc11c \uc791\ud488\uc5d0 \ucc38\uc5ec\ud558\uace0 \uce90\ub9ad\ud130\ub97c \uc5f0\uae30\ud574 \uc0ac\ud68c \ubb38\uc81c\ub97c \ub9d0\ud558\ub294 \uac83 \uc774 \uc790\uc2e0\uc758 \uc9c1\uc5c5\uc758 \uc7a5\uc810\uc774\ub77c \ub73b\uae4a\uac8c \ucd9c\uc5f0\ud588\ub2e4\uace0 \ud558\uc5ec \ubc30\uc6b0\ub85c\uc11c\uc758 \uc0ac\ud68c\uc801 \ucc45\uc784\uc5d0 \ub300\ud574 \ub9d0\ud574 \ubc15\uc218\ub97c \ubc1b\uae30\ub3c4 \ud588\ub2e4.", "paragraph_id": "6416570-13-1", "paragraph_question": "question: \ubc15\ubcf4\uc601\uc774 \ubc30\uc6b0\uc758 \uc9c1\uc5c5\uc801 \uc7a5\uc810\uc774\ub77c\uace0 \ub9d0\ud55c \uac83\uc740?, context: 2015\ub144 10\uc6d4 \uc601\ud654 \u300a\ub3cc\uc5f0\ubcc0\uc774\u300b\uac00 \uac1c\ubd09\ub410\ub2e4. \ubc15\ubcf4\uc601\uc740 \uc0dd\uc120\uc778\uac04\uc774 \ub41c \uc804 \ub0a8\uc790\uce5c\uad6c \ubc15\uad6c(\uc774\uad11\uc218)\uc758 \uc774\uc57c\uae30\ub97c \uc778\ud130\ub137\uc5d0 \ud37c\ub098\ub974\uba70 \u2018\uc774\uc288\ub140\u2019\uac00 \ub418\uae38 \ubc14\ub77c\ub294 \u2018\ud0a4\ubcf4\ub4dc \uc6cc\ub9ac\uc5b4\u2019 \uc8fc\uc9c4 \uc5ed\uc744 \ub9e1\uc558\ub2e4. \ucc38\uace0\ub85c \uc774 \uce90\ub9ad\ud130\uc758 \ubaa8\ud2f0\ube0c\ub294 \uad6d\uac00\uc815\ubcf4\uc6d0 \uc5ec\ub860 \uc870\uc791 \uc0ac\uac74\uc5d0\uc11c \uc545\ud50c\uc744 \ub2ec\uc558\ub358 \uad6d\uc815\uc6d0 \uc5ec\uc9c1\uc6d0\uc774\ub2e4. \uacb0\ub9d0\uc5d0\uc11c \uc790\uc2e0\uc758 \ud2b9\uc131\uc778 \u2018\ud0a4\ubcf4\ub4dc \uc6cc\ub9ac\uc5b4\u2019\ub97c \uc0b4\ub824 \uacf5\ubb34\uc6d0\uc774 \ub418\ub294\ub370 \uad6d\uc815\uc6d0 \ub313\uae00\ub140\ub97c \uc5f0\uc0c1\ucf00 \ud558\ub294 \ub178\uace8\uc801\uc778 \uc7a5\uba74\uc774 \uc788\uc5c8\uc73c\ub098 \ub108\ubb34 \uc9c1\uc811\uc801\uc778 \ud45c\ud604\uc774\ub77c \ud3b8\uc9d1 \ub2e8\uacc4\uc5d0\uc11c \ube60\uc84c\ub2e4\uace0 \ud55c\ub2e4. \uc5ed\ub7c9 \uc788\ub294 \uc2e0\uc778 \uac10\ub3c5\uc744 \ubc1c\uad74\ud558\ub294 \ubaa9\uc801\uc73c\ub85c \ud0c4\uc0dd\ud55c CJ E&M \ubc84\ud130\ud50c\ub77c\uc774 \ud504\ub85c\uc81d\ud2b8 \uccab \ubc88\uc9f8 \uc791\ud488\uc778 \uc774 \uc601\ud654\ub294 \uc81c40\ud68c \ud1a0\ub860\ud1a0 \uad6d\uc81c \uc601\ud654\uc81c \ubc45\uac00\ub4dc \uc139\uc158 \ubd80\ubb38\uc5d0 \uacf5\uc2dd \ucd08\uccad\ub418\uae30\ub3c4 \ud588\ub2e4. \uc800\uc608\uc0b0\uc5d0 \ucd9c\uc5f0\ub8cc 1/10\ub3c4 \uc548\ubc1b\uc740 \uc870\uc5f0 \ubd84\ub7c9\uc758 \uc601\ud654\uc600\uc9c0\ub9cc \ubc15\ubcf4\uc601\uc740 \uc774 \uc791\ud488\uc744 \ud1b5\ud574 \uc5f0\uae30\ud558\ub294 \uc7ac\ubbf8\ub97c \ub2e4\uc2dc \uae68\ub2ec\uc558\ub2e4\uace0 \ud55c\ub2e4. \uadf8\ub294 \uc81c\uc791\ubc1c\ud45c\ud68c\uc5d0\uc11c \uccad\ub144 \uc2e4\uc5c5\uc744 \uacaa\uc5b4\ubcf8 \uc801\uc740 \uc5c6\uc9c0\ub9cc \uc8fc\uc704\uc5d0 \uacaa\uace0 \uc788\ub294 \uce5c\uad6c\ub4e4\uc774 \uc788\uae30\uc5d0 \uc0ac\ub78c\ub4e4\uc774 \uc0ac\ud68c\uc801\uc778 \ubb38\uc81c\ub97c \uc0dd\uac01\ud574\ubd24\uc73c\uba74 \uc88b\uaca0\ub2e4\ub294 \ub9c8\uc74c\uc5d0\uc11c \uc5f0\uae30\ud588\ub2e4\u201d\uba70 \ubc1d\ud614\uace0 \ubc30\uc6b0\ub85c\uc11c \uc791\ud488\uc5d0 \ucc38\uc5ec\ud558\uace0 \uce90\ub9ad\ud130\ub97c \uc5f0\uae30\ud574 \uc0ac\ud68c \ubb38\uc81c\ub97c \ub9d0\ud558\ub294 \uac83\uc774 \uc790\uc2e0\uc758 \uc9c1\uc5c5\uc758 \uc7a5\uc810\uc774\ub77c \ub73b\uae4a\uac8c \ucd9c\uc5f0\ud588\ub2e4\uace0 \ud558\uc5ec \ubc30\uc6b0\ub85c\uc11c\uc758 \uc0ac\ud68c\uc801 \ucc45\uc784\uc5d0 \ub300\ud574 \ub9d0\ud574 \ubc15\uc218\ub97c \ubc1b\uae30\ub3c4 \ud588\ub2e4."} {"question": "\uc794\uc790\uc704\ub4dc\ub97c \uc7a0\uc7ac\uc801\uc73c\ub85c \uc9c0\uc6d0\ud558\ub294 \uacf3\uc740 \uc5b4\ub514\uc778\uac00?", "paragraph": "\ub2e4\ub974\ud478\ub974 \ubd84\uc7c1\uc740 \uc218\ub2e8\uc758 \uc11c\ubd80 \uc9c0\uc5ed\uc778 \ub2e4\ub974\ud478\ub974\uc5d0\uc11c \ubc8c\uc5b4\uc9c4 \uc0ac\uac74\uc774\uc5c8\ub2e4. \uc774 \ubd84\uc7c1\uc740 \uc544\ud504\ub9ac\uce74 \ud751\uc778\uacc4 \ubc18\uad70\uacfc \uc794\uc790\uc704\ub4dc (\uc601\uc5b4: Janjaweed, \"\ub9d0\ub4f1\uc5d0 \ud0c4 \uc545\ub9c8\" devil on horseback \ub97c \uc758\ubbf8)\ub77c \ubd88\ub9ac\ub294 \ubd81\ubd80 \uc544\ub78d\uacc4 \uc774\uc2ac\ub78c \ubbfc\ubcd1\ub300\uac04\uc758 \ubb34\ub825 \ubd84\uc7c1\uc774\ub2e4. \uc774 \uc804\uc7c1\uc758 \ud55c\ucabd\uc5d0\ub294 \uc794\uc790\uc704\ub4dc\uc640 \uc774\ub97c \uc7a0\uc7ac\uc801\uc73c\ub85c \uc9c0\uc6d0\ud558\ub294 \uc218\ub2e8 \uc815\ubd80\uac00 \uc788\ub2e4. \uc794\uc790\uc704\ub4dc\ub294 \ub300\ubd80\ubd84 \ub099\ud0c0\ub85c \uc774\ub3d9\ud558\ub294 \uc720\ubaa9\ubbfc\uc778 \ub9ac\uc9c0\uac00\ud2b8\uc758 \uc544\ub78d \ubc14\uac00\ub77c\uc871\uc73c\ub85c \uad6c\uc131\ub418\uc5b4 \uc788\ub2e4. \ub2e4\ub978 \ud55c\ucabd\uc740 \ub2e4\ub974\ud478\ub974 \uc9c0\uc5ed\uc758 \uc544\ud504\ub9ac\uce74\uacc4\ub85c \uacbd\uc791\uc744 \uc8fc\ub85c \ud558\ub294 \ud478\ub974(Fur), \uc790\uac00\uc640, \ub9c8\uc0b4\ub77c\uc774\ud2b8 \uc885\uc871\uc774 \uad6c\uc131\ud55c \ubc18\uad70\uc774\ub2e4. \uc218\ub2e8\ud574\ubc29\uad70(Sudan Liberation Army : SLA)\uacfc \uc815\uc758\ud3c9\ub4f1\uc6b4\ub3d9(Justice and Equality Movement : JEM)\uc774 \uc8fc\uc694 \uc138\ub825\uc774\ub2e4. \uc218\ub2e8 \uc815\ubd80\ub294 (\uacf5\uc2dd\uc801\uc73c\ub85c \uc794\uc790\uc704\ub4dc\ub97c \uc9c0\uc6d0\ud55c\ub2e4\ub294 \uac83\uc744 \ubd80\uc815\ud558\uc9c0\ub9cc) \uc794\uc790\uc704\ub4dc\uc5d0 \uc790\uae08\uc744 \uc81c\uacf5\ud558\uace0 \ub3c4\uc6c0\uc744 \uc8fc\uba70 \ubc18\uad70\uc774 \uc790\uc6d0\uc744 \uc870\ub2ec\ud558\ub294 \uc885\uc871\uc744 \uacf5\uaca9\ud560 \ub54c \ud569\ub3d9\uc791\uc804\uc744 \ud55c\ub2e4. 2003\ub144 2\uc6d4\ubd80\ud130 \ub0b4\uc804\uc774 \uc2dc\uc791\ub418\uc5c8\ub2e4. \ubd81\ucabd \ubb34\uc2ac\ub9bc\uacfc \ub0a8\ucabd \uae30\ub3c5\uad50\ub3c4, \uc560\ub2c8\ubbf8\uc998\uc744 \ubbff\ub294 \uc0ac\ub78c\ub4e4\uc774 \uc2f8\uc6e0\ub358 \uc81c2\ucc28 \uc218\ub2e8 \ub0b4\uc804\uacfc\ub294 \ub2ec\ub9ac \ub2e4\ub974\ud478\ub974\uc758 \uc804\ud22c \ub2f9\uc0ac\uc790\uc640 \ud76c\uc0dd\uc790\ub294 \uac70\uc758 \ubb34\uc2ac\ub9bc\uc774\ub2e4.", "answer": "\uc218\ub2e8 \uc815\ubd80", "sentence": "\uc774 \uc804\uc7c1\uc758 \ud55c\ucabd\uc5d0\ub294 \uc794\uc790\uc704\ub4dc\uc640 \uc774\ub97c \uc7a0\uc7ac\uc801\uc73c\ub85c \uc9c0\uc6d0\ud558\ub294 \uc218\ub2e8 \uc815\ubd80 \uac00 \uc788\ub2e4.", "paragraph_sentence": "\ub2e4\ub974\ud478\ub974 \ubd84\uc7c1\uc740 \uc218\ub2e8\uc758 \uc11c\ubd80 \uc9c0\uc5ed\uc778 \ub2e4\ub974\ud478\ub974\uc5d0\uc11c \ubc8c\uc5b4\uc9c4 \uc0ac\uac74\uc774\uc5c8\ub2e4. \uc774 \ubd84\uc7c1\uc740 \uc544\ud504\ub9ac\uce74 \ud751\uc778\uacc4 \ubc18\uad70\uacfc \uc794\uc790\uc704\ub4dc (\uc601\uc5b4: Janjaweed, \"\ub9d0\ub4f1\uc5d0 \ud0c4 \uc545\ub9c8\" devil on horseback \ub97c \uc758\ubbf8)\ub77c \ubd88\ub9ac\ub294 \ubd81\ubd80 \uc544\ub78d\uacc4 \uc774\uc2ac\ub78c \ubbfc\ubcd1\ub300\uac04\uc758 \ubb34\ub825 \ubd84\uc7c1\uc774\ub2e4. \uc774 \uc804\uc7c1\uc758 \ud55c\ucabd\uc5d0\ub294 \uc794\uc790\uc704\ub4dc\uc640 \uc774\ub97c \uc7a0\uc7ac\uc801\uc73c\ub85c \uc9c0\uc6d0\ud558\ub294 \uc218\ub2e8 \uc815\ubd80 \uac00 \uc788\ub2e4. \uc794\uc790\uc704\ub4dc\ub294 \ub300\ubd80\ubd84 \ub099\ud0c0\ub85c \uc774\ub3d9\ud558\ub294 \uc720\ubaa9\ubbfc\uc778 \ub9ac\uc9c0\uac00\ud2b8\uc758 \uc544\ub78d \ubc14\uac00\ub77c\uc871\uc73c\ub85c \uad6c\uc131\ub418\uc5b4 \uc788\ub2e4. \ub2e4\ub978 \ud55c\ucabd\uc740 \ub2e4\ub974\ud478\ub974 \uc9c0\uc5ed\uc758 \uc544\ud504\ub9ac\uce74\uacc4\ub85c \uacbd\uc791\uc744 \uc8fc\ub85c \ud558\ub294 \ud478\ub974(Fur), \uc790\uac00\uc640, \ub9c8\uc0b4\ub77c\uc774\ud2b8 \uc885\uc871\uc774 \uad6c\uc131\ud55c \ubc18\uad70\uc774\ub2e4. \uc218\ub2e8\ud574\ubc29\uad70(Sudan Liberation Army : SLA)\uacfc \uc815\uc758\ud3c9\ub4f1\uc6b4\ub3d9(Justice and Equality Movement : JEM)\uc774 \uc8fc\uc694 \uc138\ub825\uc774\ub2e4. \uc218\ub2e8 \uc815\ubd80\ub294 (\uacf5\uc2dd\uc801\uc73c\ub85c \uc794\uc790\uc704\ub4dc\ub97c \uc9c0\uc6d0\ud55c\ub2e4\ub294 \uac83\uc744 \ubd80\uc815\ud558\uc9c0\ub9cc) \uc794\uc790\uc704\ub4dc\uc5d0 \uc790\uae08\uc744 \uc81c\uacf5\ud558\uace0 \ub3c4\uc6c0\uc744 \uc8fc\uba70 \ubc18\uad70\uc774 \uc790\uc6d0\uc744 \uc870\ub2ec\ud558\ub294 \uc885\uc871\uc744 \uacf5\uaca9\ud560 \ub54c \ud569\ub3d9\uc791\uc804\uc744 \ud55c\ub2e4. 2003\ub144 2\uc6d4\ubd80\ud130 \ub0b4\uc804\uc774 \uc2dc\uc791\ub418\uc5c8\ub2e4. \ubd81\ucabd \ubb34\uc2ac\ub9bc\uacfc \ub0a8\ucabd \uae30\ub3c5\uad50\ub3c4, \uc560\ub2c8\ubbf8\uc998\uc744 \ubbff\ub294 \uc0ac\ub78c\ub4e4\uc774 \uc2f8\uc6e0\ub358 \uc81c2\ucc28 \uc218\ub2e8 \ub0b4\uc804\uacfc\ub294 \ub2ec\ub9ac \ub2e4\ub974\ud478\ub974\uc758 \uc804\ud22c \ub2f9\uc0ac\uc790\uc640 \ud76c\uc0dd\uc790\ub294 \uac70\uc758 \ubb34\uc2ac\ub9bc\uc774\ub2e4.", "paragraph_answer": "\ub2e4\ub974\ud478\ub974 \ubd84\uc7c1\uc740 \uc218\ub2e8\uc758 \uc11c\ubd80 \uc9c0\uc5ed\uc778 \ub2e4\ub974\ud478\ub974\uc5d0\uc11c \ubc8c\uc5b4\uc9c4 \uc0ac\uac74\uc774\uc5c8\ub2e4. \uc774 \ubd84\uc7c1\uc740 \uc544\ud504\ub9ac\uce74 \ud751\uc778\uacc4 \ubc18\uad70\uacfc \uc794\uc790\uc704\ub4dc (\uc601\uc5b4: Janjaweed, \"\ub9d0\ub4f1\uc5d0 \ud0c4 \uc545\ub9c8\" devil on horseback \ub97c \uc758\ubbf8)\ub77c \ubd88\ub9ac\ub294 \ubd81\ubd80 \uc544\ub78d\uacc4 \uc774\uc2ac\ub78c \ubbfc\ubcd1\ub300\uac04\uc758 \ubb34\ub825 \ubd84\uc7c1\uc774\ub2e4. \uc774 \uc804\uc7c1\uc758 \ud55c\ucabd\uc5d0\ub294 \uc794\uc790\uc704\ub4dc\uc640 \uc774\ub97c \uc7a0\uc7ac\uc801\uc73c\ub85c \uc9c0\uc6d0\ud558\ub294 \uc218\ub2e8 \uc815\ubd80 \uac00 \uc788\ub2e4. \uc794\uc790\uc704\ub4dc\ub294 \ub300\ubd80\ubd84 \ub099\ud0c0\ub85c \uc774\ub3d9\ud558\ub294 \uc720\ubaa9\ubbfc\uc778 \ub9ac\uc9c0\uac00\ud2b8\uc758 \uc544\ub78d \ubc14\uac00\ub77c\uc871\uc73c\ub85c \uad6c\uc131\ub418\uc5b4 \uc788\ub2e4. \ub2e4\ub978 \ud55c\ucabd\uc740 \ub2e4\ub974\ud478\ub974 \uc9c0\uc5ed\uc758 \uc544\ud504\ub9ac\uce74\uacc4\ub85c \uacbd\uc791\uc744 \uc8fc\ub85c \ud558\ub294 \ud478\ub974(Fur), \uc790\uac00\uc640, \ub9c8\uc0b4\ub77c\uc774\ud2b8 \uc885\uc871\uc774 \uad6c\uc131\ud55c \ubc18\uad70\uc774\ub2e4. \uc218\ub2e8\ud574\ubc29\uad70(Sudan Liberation Army : SLA)\uacfc \uc815\uc758\ud3c9\ub4f1\uc6b4\ub3d9(Justice and Equality Movement : JEM)\uc774 \uc8fc\uc694 \uc138\ub825\uc774\ub2e4. \uc218\ub2e8 \uc815\ubd80\ub294 (\uacf5\uc2dd\uc801\uc73c\ub85c \uc794\uc790\uc704\ub4dc\ub97c \uc9c0\uc6d0\ud55c\ub2e4\ub294 \uac83\uc744 \ubd80\uc815\ud558\uc9c0\ub9cc) \uc794\uc790\uc704\ub4dc\uc5d0 \uc790\uae08\uc744 \uc81c\uacf5\ud558\uace0 \ub3c4\uc6c0\uc744 \uc8fc\uba70 \ubc18\uad70\uc774 \uc790\uc6d0\uc744 \uc870\ub2ec\ud558\ub294 \uc885\uc871\uc744 \uacf5\uaca9\ud560 \ub54c \ud569\ub3d9\uc791\uc804\uc744 \ud55c\ub2e4. 2003\ub144 2\uc6d4\ubd80\ud130 \ub0b4\uc804\uc774 \uc2dc\uc791\ub418\uc5c8\ub2e4. \ubd81\ucabd \ubb34\uc2ac\ub9bc\uacfc \ub0a8\ucabd \uae30\ub3c5\uad50\ub3c4, \uc560\ub2c8\ubbf8\uc998\uc744 \ubbff\ub294 \uc0ac\ub78c\ub4e4\uc774 \uc2f8\uc6e0\ub358 \uc81c2\ucc28 \uc218\ub2e8 \ub0b4\uc804\uacfc\ub294 \ub2ec\ub9ac \ub2e4\ub974\ud478\ub974\uc758 \uc804\ud22c \ub2f9\uc0ac\uc790\uc640 \ud76c\uc0dd\uc790\ub294 \uac70\uc758 \ubb34\uc2ac\ub9bc\uc774\ub2e4.", "sentence_answer": "\uc774 \uc804\uc7c1\uc758 \ud55c\ucabd\uc5d0\ub294 \uc794\uc790\uc704\ub4dc\uc640 \uc774\ub97c \uc7a0\uc7ac\uc801\uc73c\ub85c \uc9c0\uc6d0\ud558\ub294 \uc218\ub2e8 \uc815\ubd80 \uac00 \uc788\ub2e4.", "paragraph_id": "6029197-0-1", "paragraph_question": "question: \uc794\uc790\uc704\ub4dc\ub97c \uc7a0\uc7ac\uc801\uc73c\ub85c \uc9c0\uc6d0\ud558\ub294 \uacf3\uc740 \uc5b4\ub514\uc778\uac00?, context: \ub2e4\ub974\ud478\ub974 \ubd84\uc7c1\uc740 \uc218\ub2e8\uc758 \uc11c\ubd80 \uc9c0\uc5ed\uc778 \ub2e4\ub974\ud478\ub974\uc5d0\uc11c \ubc8c\uc5b4\uc9c4 \uc0ac\uac74\uc774\uc5c8\ub2e4. \uc774 \ubd84\uc7c1\uc740 \uc544\ud504\ub9ac\uce74 \ud751\uc778\uacc4 \ubc18\uad70\uacfc \uc794\uc790\uc704\ub4dc (\uc601\uc5b4: Janjaweed, \"\ub9d0\ub4f1\uc5d0 \ud0c4 \uc545\ub9c8\" devil on horseback \ub97c \uc758\ubbf8)\ub77c \ubd88\ub9ac\ub294 \ubd81\ubd80 \uc544\ub78d\uacc4 \uc774\uc2ac\ub78c \ubbfc\ubcd1\ub300\uac04\uc758 \ubb34\ub825 \ubd84\uc7c1\uc774\ub2e4. \uc774 \uc804\uc7c1\uc758 \ud55c\ucabd\uc5d0\ub294 \uc794\uc790\uc704\ub4dc\uc640 \uc774\ub97c \uc7a0\uc7ac\uc801\uc73c\ub85c \uc9c0\uc6d0\ud558\ub294 \uc218\ub2e8 \uc815\ubd80\uac00 \uc788\ub2e4. \uc794\uc790\uc704\ub4dc\ub294 \ub300\ubd80\ubd84 \ub099\ud0c0\ub85c \uc774\ub3d9\ud558\ub294 \uc720\ubaa9\ubbfc\uc778 \ub9ac\uc9c0\uac00\ud2b8\uc758 \uc544\ub78d \ubc14\uac00\ub77c\uc871\uc73c\ub85c \uad6c\uc131\ub418\uc5b4 \uc788\ub2e4. \ub2e4\ub978 \ud55c\ucabd\uc740 \ub2e4\ub974\ud478\ub974 \uc9c0\uc5ed\uc758 \uc544\ud504\ub9ac\uce74\uacc4\ub85c \uacbd\uc791\uc744 \uc8fc\ub85c \ud558\ub294 \ud478\ub974(Fur), \uc790\uac00\uc640, \ub9c8\uc0b4\ub77c\uc774\ud2b8 \uc885\uc871\uc774 \uad6c\uc131\ud55c \ubc18\uad70\uc774\ub2e4. \uc218\ub2e8\ud574\ubc29\uad70(Sudan Liberation Army : SLA)\uacfc \uc815\uc758\ud3c9\ub4f1\uc6b4\ub3d9(Justice and Equality Movement : JEM)\uc774 \uc8fc\uc694 \uc138\ub825\uc774\ub2e4. \uc218\ub2e8 \uc815\ubd80\ub294 (\uacf5\uc2dd\uc801\uc73c\ub85c \uc794\uc790\uc704\ub4dc\ub97c \uc9c0\uc6d0\ud55c\ub2e4\ub294 \uac83\uc744 \ubd80\uc815\ud558\uc9c0\ub9cc) \uc794\uc790\uc704\ub4dc\uc5d0 \uc790\uae08\uc744 \uc81c\uacf5\ud558\uace0 \ub3c4\uc6c0\uc744 \uc8fc\uba70 \ubc18\uad70\uc774 \uc790\uc6d0\uc744 \uc870\ub2ec\ud558\ub294 \uc885\uc871\uc744 \uacf5\uaca9\ud560 \ub54c \ud569\ub3d9\uc791\uc804\uc744 \ud55c\ub2e4. 2003\ub144 2\uc6d4\ubd80\ud130 \ub0b4\uc804\uc774 \uc2dc\uc791\ub418\uc5c8\ub2e4. \ubd81\ucabd \ubb34\uc2ac\ub9bc\uacfc \ub0a8\ucabd \uae30\ub3c5\uad50\ub3c4, \uc560\ub2c8\ubbf8\uc998\uc744 \ubbff\ub294 \uc0ac\ub78c\ub4e4\uc774 \uc2f8\uc6e0\ub358 \uc81c2\ucc28 \uc218\ub2e8 \ub0b4\uc804\uacfc\ub294 \ub2ec\ub9ac \ub2e4\ub974\ud478\ub974\uc758 \uc804\ud22c \ub2f9\uc0ac\uc790\uc640 \ud76c\uc0dd\uc790\ub294 \uac70\uc758 \ubb34\uc2ac\ub9bc\uc774\ub2e4."} {"question": "\uac74\ub96d\uc81c\uac00 \uc8fc\ub85c \uc21c\ud589\ud55c 4\uacf3\uc740 \ub0a8\uacbd, \uc591\uc8fc, \uc18c\uc8fc, \ub610 \uc5b4\ub514\uc778\uac00?", "paragraph": "\uac74\ub96d\uc81c\uc758 \uc21c\ud589\uc758 \uc8fc\uc694 \ub3d9\uc120\uc740 \ub0a8\uacbd, \uc591\uc8fc, \ud56d\uc8fc, \uadf8\ub9ac\uace0 \uc18c\uc8fc\uc600\ub2e4. \uc774 \ub124 \ub3c4\uc2dc\uc5d0\uc11c \uac74\ub96d\uc81c\ub294 \ub300\uc6b4\ud558\ub098 \ub2e4\ub978 \uba85\uc2b9\uace0\uc801\uc744 \ub3cc\uc544\ub2e4\ub2c8\uba70 \ub2f4\ub860\uc744 \uc990\uacbc\uace0 \uc800\ub141\uc5d0\ub294 \ud638\ud654\ub85c\uc6b4 \ud589\uad81\uc5d0\uc11c \uc9c0\uc5ed \uc720\uc9c0\ub4e4\uacfc \uad00\ub9ac\ub4e4\uc774 \uc8fc\ucd5c\ud558\ub294 \ud638\ud654\ub85c\uc6b4 \ub9cc\ucc2c\uc744 \uc990\uacbc\ub2e4. \uadf8\ub9ac\uace0 \ub9e4\uc77c \uac70\ub974\uc9c0 \uc54a\uace0 \uc0ac\uce58\uc2a4\ub7fd\uace0 \ud638\ud654\ub85c\uc6b4 \ub9cc\ucc2c\uc744 \ubc8c\ub824 \uc790\uc2e0\uc774 \uac00\uc838\uc628 \ub0b4\ud0d5\uae08\uc774\ub098 \uc0c1\uc778\ub4e4\uacfc \uc9c0\uc5ed \uad00\ub9ac\ub4e4\uc774 \ubc14\uce5c \ub3c8\uae4c\uc9c0 \ubaa8\ub450 \ub5a8\uc5b4\uc9c0\uc790 \uc9c0\uc5ed \uad00\ub9ac\ub4e4\uacfc \uc0c1\uc778\ub4e4\uc758 \ud558\uc218\uc778\ub4e4\uc774 \uc790\uc6d0\ud558\uc5ec \ubc31\uc131\ub4e4\ub85c\ubd80\ud130 \ub3c8\uc744 \ub72f\uc5b4\ub0b4\uc5b4 \ucda9\ub2f9\uc744 \ud558\uc600\uace0 \uba87 \uac1c\uc758 \ud589\uad81\uc744 \uc9d3\ub294 \ub370\uc5d0\ub3c4 \ubc31\uc131\ub4e4\uc744 \uac15\uc81c\ub85c \ub3d9\uc6d0\ud558\uc5ec \ud070 \uc6d0\uc131\uc744 \uc0c0\ub2e4. \uc870\ubd80\uc778 \uac15\ud76c\uc81c\uac00 \uc21c\ud589 \ub54c \uac00\uc838\uac04 \ub0b4\ud0d5\uae08\ub9cc\uc73c\ub85c \ubaa8\ub4e0 \uac78 \ucda9\ub2f9\ud558\uace0 \ubd81\uacbd\uc73c\ub85c \ub3cc\uc544\uc62c \ub54c\ub3c4 \ub2e4 \uc4f0\uc774\uc9c0 \uc54a\uc558\ub358 \uac83\uacfc \uc0c1\ubc18\ub41c\ub2e4. \uc2e4\uc81c\ub85c \uac74\ub96d\uc81c\ub294 \uc21c\ud589 \ub54c \uac15\ud76c\uc81c\uac00 \uc4f0\ub358 \ube44\uc6a9\uc758 \ud3c9\uade0 \uc5f4 \ubc30 \uc774\uc0c1\uc744 \uc368\uc11c \uad6d\uace0\uc758 \ub3c8\uc744 \uc9c0\ub098\uce58\uac8c \ub0ad\ube44\ud558\uc600\ub2e4.", "answer": "\ud56d\uc8fc", "sentence": "\uac74\ub96d\uc81c\uc758 \uc21c\ud589\uc758 \uc8fc\uc694 \ub3d9\uc120\uc740 \ub0a8\uacbd, \uc591\uc8fc, \ud56d\uc8fc ,", "paragraph_sentence": " \uac74\ub96d\uc81c\uc758 \uc21c\ud589\uc758 \uc8fc\uc694 \ub3d9\uc120\uc740 \ub0a8\uacbd, \uc591\uc8fc, \ud56d\uc8fc , \uadf8\ub9ac\uace0 \uc18c\uc8fc\uc600\ub2e4. \uc774 \ub124 \ub3c4\uc2dc\uc5d0\uc11c \uac74\ub96d\uc81c\ub294 \ub300\uc6b4\ud558\ub098 \ub2e4\ub978 \uba85\uc2b9\uace0\uc801\uc744 \ub3cc\uc544\ub2e4\ub2c8\uba70 \ub2f4\ub860\uc744 \uc990\uacbc\uace0 \uc800\ub141\uc5d0\ub294 \ud638\ud654\ub85c\uc6b4 \ud589\uad81\uc5d0\uc11c \uc9c0\uc5ed \uc720\uc9c0\ub4e4\uacfc \uad00\ub9ac\ub4e4\uc774 \uc8fc\ucd5c\ud558\ub294 \ud638\ud654\ub85c\uc6b4 \ub9cc\ucc2c\uc744 \uc990\uacbc\ub2e4. \uadf8\ub9ac\uace0 \ub9e4\uc77c \uac70\ub974\uc9c0 \uc54a\uace0 \uc0ac\uce58\uc2a4\ub7fd\uace0 \ud638\ud654\ub85c\uc6b4 \ub9cc\ucc2c\uc744 \ubc8c\ub824 \uc790\uc2e0\uc774 \uac00\uc838\uc628 \ub0b4\ud0d5\uae08\uc774\ub098 \uc0c1\uc778\ub4e4\uacfc \uc9c0\uc5ed \uad00\ub9ac\ub4e4\uc774 \ubc14\uce5c \ub3c8\uae4c\uc9c0 \ubaa8\ub450 \ub5a8\uc5b4\uc9c0\uc790 \uc9c0\uc5ed \uad00\ub9ac\ub4e4\uacfc \uc0c1\uc778\ub4e4\uc758 \ud558\uc218\uc778\ub4e4\uc774 \uc790\uc6d0\ud558\uc5ec \ubc31\uc131\ub4e4\ub85c\ubd80\ud130 \ub3c8\uc744 \ub72f\uc5b4\ub0b4\uc5b4 \ucda9\ub2f9\uc744 \ud558\uc600\uace0 \uba87 \uac1c\uc758 \ud589\uad81\uc744 \uc9d3\ub294 \ub370\uc5d0\ub3c4 \ubc31\uc131\ub4e4\uc744 \uac15\uc81c\ub85c \ub3d9\uc6d0\ud558\uc5ec \ud070 \uc6d0\uc131\uc744 \uc0c0\ub2e4. \uc870\ubd80\uc778 \uac15\ud76c\uc81c\uac00 \uc21c\ud589 \ub54c \uac00\uc838\uac04 \ub0b4\ud0d5\uae08\ub9cc\uc73c\ub85c \ubaa8\ub4e0 \uac78 \ucda9\ub2f9\ud558\uace0 \ubd81\uacbd\uc73c\ub85c \ub3cc\uc544\uc62c \ub54c\ub3c4 \ub2e4 \uc4f0\uc774\uc9c0 \uc54a\uc558\ub358 \uac83\uacfc \uc0c1\ubc18\ub41c\ub2e4. \uc2e4\uc81c\ub85c \uac74\ub96d\uc81c\ub294 \uc21c\ud589 \ub54c \uac15\ud76c\uc81c\uac00 \uc4f0\ub358 \ube44\uc6a9\uc758 \ud3c9\uade0 \uc5f4 \ubc30 \uc774\uc0c1\uc744 \uc368\uc11c \uad6d\uace0\uc758 \ub3c8\uc744 \uc9c0\ub098\uce58\uac8c \ub0ad\ube44\ud558\uc600\ub2e4.", "paragraph_answer": "\uac74\ub96d\uc81c\uc758 \uc21c\ud589\uc758 \uc8fc\uc694 \ub3d9\uc120\uc740 \ub0a8\uacbd, \uc591\uc8fc, \ud56d\uc8fc , \uadf8\ub9ac\uace0 \uc18c\uc8fc\uc600\ub2e4. \uc774 \ub124 \ub3c4\uc2dc\uc5d0\uc11c \uac74\ub96d\uc81c\ub294 \ub300\uc6b4\ud558\ub098 \ub2e4\ub978 \uba85\uc2b9\uace0\uc801\uc744 \ub3cc\uc544\ub2e4\ub2c8\uba70 \ub2f4\ub860\uc744 \uc990\uacbc\uace0 \uc800\ub141\uc5d0\ub294 \ud638\ud654\ub85c\uc6b4 \ud589\uad81\uc5d0\uc11c \uc9c0\uc5ed \uc720\uc9c0\ub4e4\uacfc \uad00\ub9ac\ub4e4\uc774 \uc8fc\ucd5c\ud558\ub294 \ud638\ud654\ub85c\uc6b4 \ub9cc\ucc2c\uc744 \uc990\uacbc\ub2e4. \uadf8\ub9ac\uace0 \ub9e4\uc77c \uac70\ub974\uc9c0 \uc54a\uace0 \uc0ac\uce58\uc2a4\ub7fd\uace0 \ud638\ud654\ub85c\uc6b4 \ub9cc\ucc2c\uc744 \ubc8c\ub824 \uc790\uc2e0\uc774 \uac00\uc838\uc628 \ub0b4\ud0d5\uae08\uc774\ub098 \uc0c1\uc778\ub4e4\uacfc \uc9c0\uc5ed \uad00\ub9ac\ub4e4\uc774 \ubc14\uce5c \ub3c8\uae4c\uc9c0 \ubaa8\ub450 \ub5a8\uc5b4\uc9c0\uc790 \uc9c0\uc5ed \uad00\ub9ac\ub4e4\uacfc \uc0c1\uc778\ub4e4\uc758 \ud558\uc218\uc778\ub4e4\uc774 \uc790\uc6d0\ud558\uc5ec \ubc31\uc131\ub4e4\ub85c\ubd80\ud130 \ub3c8\uc744 \ub72f\uc5b4\ub0b4\uc5b4 \ucda9\ub2f9\uc744 \ud558\uc600\uace0 \uba87 \uac1c\uc758 \ud589\uad81\uc744 \uc9d3\ub294 \ub370\uc5d0\ub3c4 \ubc31\uc131\ub4e4\uc744 \uac15\uc81c\ub85c \ub3d9\uc6d0\ud558\uc5ec \ud070 \uc6d0\uc131\uc744 \uc0c0\ub2e4. \uc870\ubd80\uc778 \uac15\ud76c\uc81c\uac00 \uc21c\ud589 \ub54c \uac00\uc838\uac04 \ub0b4\ud0d5\uae08\ub9cc\uc73c\ub85c \ubaa8\ub4e0 \uac78 \ucda9\ub2f9\ud558\uace0 \ubd81\uacbd\uc73c\ub85c \ub3cc\uc544\uc62c \ub54c\ub3c4 \ub2e4 \uc4f0\uc774\uc9c0 \uc54a\uc558\ub358 \uac83\uacfc \uc0c1\ubc18\ub41c\ub2e4. \uc2e4\uc81c\ub85c \uac74\ub96d\uc81c\ub294 \uc21c\ud589 \ub54c \uac15\ud76c\uc81c\uac00 \uc4f0\ub358 \ube44\uc6a9\uc758 \ud3c9\uade0 \uc5f4 \ubc30 \uc774\uc0c1\uc744 \uc368\uc11c \uad6d\uace0\uc758 \ub3c8\uc744 \uc9c0\ub098\uce58\uac8c \ub0ad\ube44\ud558\uc600\ub2e4.", "sentence_answer": "\uac74\ub96d\uc81c\uc758 \uc21c\ud589\uc758 \uc8fc\uc694 \ub3d9\uc120\uc740 \ub0a8\uacbd, \uc591\uc8fc, \ud56d\uc8fc ,", "paragraph_id": "6486904-14-0", "paragraph_question": "question: \uac74\ub96d\uc81c\uac00 \uc8fc\ub85c \uc21c\ud589\ud55c 4\uacf3\uc740 \ub0a8\uacbd, \uc591\uc8fc, \uc18c\uc8fc, \ub610 \uc5b4\ub514\uc778\uac00?, context: \uac74\ub96d\uc81c\uc758 \uc21c\ud589\uc758 \uc8fc\uc694 \ub3d9\uc120\uc740 \ub0a8\uacbd, \uc591\uc8fc, \ud56d\uc8fc, \uadf8\ub9ac\uace0 \uc18c\uc8fc\uc600\ub2e4. \uc774 \ub124 \ub3c4\uc2dc\uc5d0\uc11c \uac74\ub96d\uc81c\ub294 \ub300\uc6b4\ud558\ub098 \ub2e4\ub978 \uba85\uc2b9\uace0\uc801\uc744 \ub3cc\uc544\ub2e4\ub2c8\uba70 \ub2f4\ub860\uc744 \uc990\uacbc\uace0 \uc800\ub141\uc5d0\ub294 \ud638\ud654\ub85c\uc6b4 \ud589\uad81\uc5d0\uc11c \uc9c0\uc5ed \uc720\uc9c0\ub4e4\uacfc \uad00\ub9ac\ub4e4\uc774 \uc8fc\ucd5c\ud558\ub294 \ud638\ud654\ub85c\uc6b4 \ub9cc\ucc2c\uc744 \uc990\uacbc\ub2e4. \uadf8\ub9ac\uace0 \ub9e4\uc77c \uac70\ub974\uc9c0 \uc54a\uace0 \uc0ac\uce58\uc2a4\ub7fd\uace0 \ud638\ud654\ub85c\uc6b4 \ub9cc\ucc2c\uc744 \ubc8c\ub824 \uc790\uc2e0\uc774 \uac00\uc838\uc628 \ub0b4\ud0d5\uae08\uc774\ub098 \uc0c1\uc778\ub4e4\uacfc \uc9c0\uc5ed \uad00\ub9ac\ub4e4\uc774 \ubc14\uce5c \ub3c8\uae4c\uc9c0 \ubaa8\ub450 \ub5a8\uc5b4\uc9c0\uc790 \uc9c0\uc5ed \uad00\ub9ac\ub4e4\uacfc \uc0c1\uc778\ub4e4\uc758 \ud558\uc218\uc778\ub4e4\uc774 \uc790\uc6d0\ud558\uc5ec \ubc31\uc131\ub4e4\ub85c\ubd80\ud130 \ub3c8\uc744 \ub72f\uc5b4\ub0b4\uc5b4 \ucda9\ub2f9\uc744 \ud558\uc600\uace0 \uba87 \uac1c\uc758 \ud589\uad81\uc744 \uc9d3\ub294 \ub370\uc5d0\ub3c4 \ubc31\uc131\ub4e4\uc744 \uac15\uc81c\ub85c \ub3d9\uc6d0\ud558\uc5ec \ud070 \uc6d0\uc131\uc744 \uc0c0\ub2e4. \uc870\ubd80\uc778 \uac15\ud76c\uc81c\uac00 \uc21c\ud589 \ub54c \uac00\uc838\uac04 \ub0b4\ud0d5\uae08\ub9cc\uc73c\ub85c \ubaa8\ub4e0 \uac78 \ucda9\ub2f9\ud558\uace0 \ubd81\uacbd\uc73c\ub85c \ub3cc\uc544\uc62c \ub54c\ub3c4 \ub2e4 \uc4f0\uc774\uc9c0 \uc54a\uc558\ub358 \uac83\uacfc \uc0c1\ubc18\ub41c\ub2e4. \uc2e4\uc81c\ub85c \uac74\ub96d\uc81c\ub294 \uc21c\ud589 \ub54c \uac15\ud76c\uc81c\uac00 \uc4f0\ub358 \ube44\uc6a9\uc758 \ud3c9\uade0 \uc5f4 \ubc30 \uc774\uc0c1\uc744 \uc368\uc11c \uad6d\uace0\uc758 \ub3c8\uc744 \uc9c0\ub098\uce58\uac8c \ub0ad\ube44\ud558\uc600\ub2e4."} {"question": "\uc218\ud3c9\uc73c\ub85c \ub098\ub220\uc9c4 \uc804\ubb38\uc801 \ud65c\ub3d9\uc744 \ud1b5\uc81c\ud558\ub294 \uc804\ud615\uc801\uc778 \ubc29\ubc95\uc740 \ubb34\uc5c7\uc778\uac00?", "paragraph": "\uc218\ud3c9\uc801 \ubd84\ud654\ub294 \uc870\uc9c1\uc774 \uc218\ud589\ud558\ub294 \uc5c5\ubb34\ub97c \uc870\uc9c1\uc758 \uad6c\uc131\uc6d0\ub4e4\uc774 \ubd84\ud560, \uc138\ubd84\ud558\uc5ec \uc218\ud589\ud558\ub294 \uac83\uc744 \uc758\ubbf8\ud558\uba70 \uc9c1\ubb34\ub97c \uae30\uc900\uc73c\ub85c \uadf8 \uc9c1\ubb34\ub97c \uc138\ubd84\ud654\ud558\uc5ec \uc804\ubb38\ud654 \uc2dc\ud0a4\ub294 \ubc29\ubc95\uacfc \uc720\uc0ac\ud55c \uc9c1\ubb34\ub97c \uc218\ud589\ud558\ub294 \uc804\ubb38\uac00\ub4e4\uc744 \uc9d1\ub2e8\ud654\uc2dc\ucf1c \uadf8 \uc804\ubb38\uac00 \uc9d1\ub2e8\uc5d0 \uc758\ud558\uc5ec \uc77c\uc744 \uc218\ud589\ud558\ub3c4\ub85d \ud558\ub294 \ubd80\uc11c\ud654\uc758 \ubc29\ubc95\uc774 \uc788\ub2e4. \uc804\ubb38\ud654\ub780 \uc5c5\ubb34\ub098 \ubaa9\uc801\uc5d0 \ub530\ub77c \uc5ed\ud560\uc774 \uc138\ubd84\ud654\ub41c \uc815\ub3c4\ub85c\uc11c, \uc870\uc9c1\uc5c5\ubb34\uac00 \ud558\uc704 \uc5c5\ubb34\ub85c \ub098\ub204\uc5b4\uc9c0\ub294 \uc815\ub3c4\uc640 \ub098\ub204\uc5b4\uc9c4 \ud558\uc704 \uc5c5\ubb34\uc5d0 \uc624\uc9c1 \ud558\ub098\uc758 \uc5ed\ud560\ub9cc\uc744 \uc774\ud589\ud558\ub3c4\ub85d \ubd84\ubc30\ub418\ub294 \uc815\ub3c4\ub97c \uc758\ubbf8\ud55c\ub2e4. \uc774\uc5d0 \ub530\ub77c \uc804\ubb38\ud654\ub294 \uc870\uc9c1 \ub0b4\uc5d0\uc11c \ucd94\uad6c\ub418\ub294 \ub2e4\uc591\ud55c \uae30\ub2a5\uc801 \ud65c\ub3d9\uc774\ub098 \uc5ed\ud560\uc758 \uc218\ub97c \ub098\ud0c0\ub0b4\ub294\ub370, \uc81c\ud55c\ub41c \ud558\ub098\uc758 \uc5c5\ubb34\ub9cc\uc744 \uc218\ud589\ud560 \ub54c \ub192\uc740 \uc218\uc900\uc758 \uc804\ubb38\ud654\ub97c \ubcf4\uc774\uac8c \ub418\uace0 \ubc18\uba74\uc5d0 \ub2e4\uc591\ud55c \ubc94\uc704\uc758 \uc77c\uc744 \uc218\ud589\ud558\uba70 \uc5c5\ubb34\ub97c \ube48\ubc88\ud558\uac8c \ubcc0\ud654\ud558\uac8c \ub418\uba74 \ub0ae\uc740 \uc218\uc900\uc758 \uc804\ubb38\ud654\ub97c \ubcf4\uc774\uac8c \ub41c\ub2e4. \ubd80\uc11c\ud654\ub294 \uc218\ud3c9\uc801\uc73c\ub85c \ubd84\ud654\ub41c \uc804\ubb38\uc801 \ud65c\ub3d9\uc744 \ud1b5\uc81c\ud558\ub294 \uc804\ud615\uc801\uc778 \ubc29\ubc95\uc73c\ub85c \uc804\ubb38\uac00\ub4e4\uc744 \uc9d1\ub2e8\ud654\uc2dc\ucf1c \uc77c\uc744 \uc218\ud589\ud558\ub3c4\ub85d \ud558\ub294 \uac83\uc744 \ub9d0\ud55c\ub2e4.", "answer": "\ubd80\uc11c\ud654", "sentence": "\uc218\ud3c9\uc801 \ubd84\ud654\ub294 \uc870\uc9c1\uc774 \uc218\ud589\ud558\ub294 \uc5c5\ubb34\ub97c \uc870\uc9c1\uc758 \uad6c\uc131\uc6d0\ub4e4\uc774 \ubd84\ud560, \uc138\ubd84\ud558\uc5ec \uc218\ud589\ud558\ub294 \uac83\uc744 \uc758\ubbf8\ud558\uba70 \uc9c1\ubb34\ub97c \uae30\uc900\uc73c\ub85c \uadf8 \uc9c1\ubb34\ub97c \uc138\ubd84\ud654\ud558\uc5ec \uc804\ubb38\ud654 \uc2dc\ud0a4\ub294 \ubc29\ubc95\uacfc \uc720\uc0ac\ud55c \uc9c1\ubb34\ub97c \uc218\ud589\ud558\ub294 \uc804\ubb38\uac00\ub4e4\uc744 \uc9d1\ub2e8\ud654\uc2dc\ucf1c \uadf8 \uc804\ubb38\uac00 \uc9d1\ub2e8\uc5d0 \uc758\ud558\uc5ec \uc77c\uc744 \uc218\ud589\ud558\ub3c4\ub85d \ud558\ub294 \ubd80\uc11c\ud654 \uc758 \ubc29\ubc95\uc774 \uc788\ub2e4.", "paragraph_sentence": " \uc218\ud3c9\uc801 \ubd84\ud654\ub294 \uc870\uc9c1\uc774 \uc218\ud589\ud558\ub294 \uc5c5\ubb34\ub97c \uc870\uc9c1\uc758 \uad6c\uc131\uc6d0\ub4e4\uc774 \ubd84\ud560, \uc138\ubd84\ud558\uc5ec \uc218\ud589\ud558\ub294 \uac83\uc744 \uc758\ubbf8\ud558\uba70 \uc9c1\ubb34\ub97c \uae30\uc900\uc73c\ub85c \uadf8 \uc9c1\ubb34\ub97c \uc138\ubd84\ud654\ud558\uc5ec \uc804\ubb38\ud654 \uc2dc\ud0a4\ub294 \ubc29\ubc95\uacfc \uc720\uc0ac\ud55c \uc9c1\ubb34\ub97c \uc218\ud589\ud558\ub294 \uc804\ubb38\uac00\ub4e4\uc744 \uc9d1\ub2e8\ud654\uc2dc\ucf1c \uadf8 \uc804\ubb38\uac00 \uc9d1\ub2e8\uc5d0 \uc758\ud558\uc5ec \uc77c\uc744 \uc218\ud589\ud558\ub3c4\ub85d \ud558\ub294 \ubd80\uc11c\ud654 \uc758 \ubc29\ubc95\uc774 \uc788\ub2e4. \uc804\ubb38\ud654\ub780 \uc5c5\ubb34\ub098 \ubaa9\uc801\uc5d0 \ub530\ub77c \uc5ed\ud560\uc774 \uc138\ubd84\ud654\ub41c \uc815\ub3c4\ub85c\uc11c, \uc870\uc9c1\uc5c5\ubb34\uac00 \ud558\uc704 \uc5c5\ubb34\ub85c \ub098\ub204\uc5b4\uc9c0\ub294 \uc815\ub3c4\uc640 \ub098\ub204\uc5b4\uc9c4 \ud558\uc704 \uc5c5\ubb34\uc5d0 \uc624\uc9c1 \ud558\ub098\uc758 \uc5ed\ud560\ub9cc\uc744 \uc774\ud589\ud558\ub3c4\ub85d \ubd84\ubc30\ub418\ub294 \uc815\ub3c4\ub97c \uc758\ubbf8\ud55c\ub2e4. \uc774\uc5d0 \ub530\ub77c \uc804\ubb38\ud654\ub294 \uc870\uc9c1 \ub0b4\uc5d0\uc11c \ucd94\uad6c\ub418\ub294 \ub2e4\uc591\ud55c \uae30\ub2a5\uc801 \ud65c\ub3d9\uc774\ub098 \uc5ed\ud560\uc758 \uc218\ub97c \ub098\ud0c0\ub0b4\ub294\ub370, \uc81c\ud55c\ub41c \ud558\ub098\uc758 \uc5c5\ubb34\ub9cc\uc744 \uc218\ud589\ud560 \ub54c \ub192\uc740 \uc218\uc900\uc758 \uc804\ubb38\ud654\ub97c \ubcf4\uc774\uac8c \ub418\uace0 \ubc18\uba74\uc5d0 \ub2e4\uc591\ud55c \ubc94\uc704\uc758 \uc77c\uc744 \uc218\ud589\ud558\uba70 \uc5c5\ubb34\ub97c \ube48\ubc88\ud558\uac8c \ubcc0\ud654\ud558\uac8c \ub418\uba74 \ub0ae\uc740 \uc218\uc900\uc758 \uc804\ubb38\ud654\ub97c \ubcf4\uc774\uac8c \ub41c\ub2e4. \ubd80\uc11c\ud654\ub294 \uc218\ud3c9\uc801\uc73c\ub85c \ubd84\ud654\ub41c \uc804\ubb38\uc801 \ud65c\ub3d9\uc744 \ud1b5\uc81c\ud558\ub294 \uc804\ud615\uc801\uc778 \ubc29\ubc95\uc73c\ub85c \uc804\ubb38\uac00\ub4e4\uc744 \uc9d1\ub2e8\ud654\uc2dc\ucf1c \uc77c\uc744 \uc218\ud589\ud558\ub3c4\ub85d \ud558\ub294 \uac83\uc744 \ub9d0\ud55c\ub2e4.", "paragraph_answer": "\uc218\ud3c9\uc801 \ubd84\ud654\ub294 \uc870\uc9c1\uc774 \uc218\ud589\ud558\ub294 \uc5c5\ubb34\ub97c \uc870\uc9c1\uc758 \uad6c\uc131\uc6d0\ub4e4\uc774 \ubd84\ud560, \uc138\ubd84\ud558\uc5ec \uc218\ud589\ud558\ub294 \uac83\uc744 \uc758\ubbf8\ud558\uba70 \uc9c1\ubb34\ub97c \uae30\uc900\uc73c\ub85c \uadf8 \uc9c1\ubb34\ub97c \uc138\ubd84\ud654\ud558\uc5ec \uc804\ubb38\ud654 \uc2dc\ud0a4\ub294 \ubc29\ubc95\uacfc \uc720\uc0ac\ud55c \uc9c1\ubb34\ub97c \uc218\ud589\ud558\ub294 \uc804\ubb38\uac00\ub4e4\uc744 \uc9d1\ub2e8\ud654\uc2dc\ucf1c \uadf8 \uc804\ubb38\uac00 \uc9d1\ub2e8\uc5d0 \uc758\ud558\uc5ec \uc77c\uc744 \uc218\ud589\ud558\ub3c4\ub85d \ud558\ub294 \ubd80\uc11c\ud654 \uc758 \ubc29\ubc95\uc774 \uc788\ub2e4. \uc804\ubb38\ud654\ub780 \uc5c5\ubb34\ub098 \ubaa9\uc801\uc5d0 \ub530\ub77c \uc5ed\ud560\uc774 \uc138\ubd84\ud654\ub41c \uc815\ub3c4\ub85c\uc11c, \uc870\uc9c1\uc5c5\ubb34\uac00 \ud558\uc704 \uc5c5\ubb34\ub85c \ub098\ub204\uc5b4\uc9c0\ub294 \uc815\ub3c4\uc640 \ub098\ub204\uc5b4\uc9c4 \ud558\uc704 \uc5c5\ubb34\uc5d0 \uc624\uc9c1 \ud558\ub098\uc758 \uc5ed\ud560\ub9cc\uc744 \uc774\ud589\ud558\ub3c4\ub85d \ubd84\ubc30\ub418\ub294 \uc815\ub3c4\ub97c \uc758\ubbf8\ud55c\ub2e4. \uc774\uc5d0 \ub530\ub77c \uc804\ubb38\ud654\ub294 \uc870\uc9c1 \ub0b4\uc5d0\uc11c \ucd94\uad6c\ub418\ub294 \ub2e4\uc591\ud55c \uae30\ub2a5\uc801 \ud65c\ub3d9\uc774\ub098 \uc5ed\ud560\uc758 \uc218\ub97c \ub098\ud0c0\ub0b4\ub294\ub370, \uc81c\ud55c\ub41c \ud558\ub098\uc758 \uc5c5\ubb34\ub9cc\uc744 \uc218\ud589\ud560 \ub54c \ub192\uc740 \uc218\uc900\uc758 \uc804\ubb38\ud654\ub97c \ubcf4\uc774\uac8c \ub418\uace0 \ubc18\uba74\uc5d0 \ub2e4\uc591\ud55c \ubc94\uc704\uc758 \uc77c\uc744 \uc218\ud589\ud558\uba70 \uc5c5\ubb34\ub97c \ube48\ubc88\ud558\uac8c \ubcc0\ud654\ud558\uac8c \ub418\uba74 \ub0ae\uc740 \uc218\uc900\uc758 \uc804\ubb38\ud654\ub97c \ubcf4\uc774\uac8c \ub41c\ub2e4. \ubd80\uc11c\ud654\ub294 \uc218\ud3c9\uc801\uc73c\ub85c \ubd84\ud654\ub41c \uc804\ubb38\uc801 \ud65c\ub3d9\uc744 \ud1b5\uc81c\ud558\ub294 \uc804\ud615\uc801\uc778 \ubc29\ubc95\uc73c\ub85c \uc804\ubb38\uac00\ub4e4\uc744 \uc9d1\ub2e8\ud654\uc2dc\ucf1c \uc77c\uc744 \uc218\ud589\ud558\ub3c4\ub85d \ud558\ub294 \uac83\uc744 \ub9d0\ud55c\ub2e4.", "sentence_answer": "\uc218\ud3c9\uc801 \ubd84\ud654\ub294 \uc870\uc9c1\uc774 \uc218\ud589\ud558\ub294 \uc5c5\ubb34\ub97c \uc870\uc9c1\uc758 \uad6c\uc131\uc6d0\ub4e4\uc774 \ubd84\ud560, \uc138\ubd84\ud558\uc5ec \uc218\ud589\ud558\ub294 \uac83\uc744 \uc758\ubbf8\ud558\uba70 \uc9c1\ubb34\ub97c \uae30\uc900\uc73c\ub85c \uadf8 \uc9c1\ubb34\ub97c \uc138\ubd84\ud654\ud558\uc5ec \uc804\ubb38\ud654 \uc2dc\ud0a4\ub294 \ubc29\ubc95\uacfc \uc720\uc0ac\ud55c \uc9c1\ubb34\ub97c \uc218\ud589\ud558\ub294 \uc804\ubb38\uac00\ub4e4\uc744 \uc9d1\ub2e8\ud654\uc2dc\ucf1c \uadf8 \uc804\ubb38\uac00 \uc9d1\ub2e8\uc5d0 \uc758\ud558\uc5ec \uc77c\uc744 \uc218\ud589\ud558\ub3c4\ub85d \ud558\ub294 \ubd80\uc11c\ud654 \uc758 \ubc29\ubc95\uc774 \uc788\ub2e4.", "paragraph_id": "6554793-2-1", "paragraph_question": "question: \uc218\ud3c9\uc73c\ub85c \ub098\ub220\uc9c4 \uc804\ubb38\uc801 \ud65c\ub3d9\uc744 \ud1b5\uc81c\ud558\ub294 \uc804\ud615\uc801\uc778 \ubc29\ubc95\uc740 \ubb34\uc5c7\uc778\uac00?, context: \uc218\ud3c9\uc801 \ubd84\ud654\ub294 \uc870\uc9c1\uc774 \uc218\ud589\ud558\ub294 \uc5c5\ubb34\ub97c \uc870\uc9c1\uc758 \uad6c\uc131\uc6d0\ub4e4\uc774 \ubd84\ud560, \uc138\ubd84\ud558\uc5ec \uc218\ud589\ud558\ub294 \uac83\uc744 \uc758\ubbf8\ud558\uba70 \uc9c1\ubb34\ub97c \uae30\uc900\uc73c\ub85c \uadf8 \uc9c1\ubb34\ub97c \uc138\ubd84\ud654\ud558\uc5ec \uc804\ubb38\ud654 \uc2dc\ud0a4\ub294 \ubc29\ubc95\uacfc \uc720\uc0ac\ud55c \uc9c1\ubb34\ub97c \uc218\ud589\ud558\ub294 \uc804\ubb38\uac00\ub4e4\uc744 \uc9d1\ub2e8\ud654\uc2dc\ucf1c \uadf8 \uc804\ubb38\uac00 \uc9d1\ub2e8\uc5d0 \uc758\ud558\uc5ec \uc77c\uc744 \uc218\ud589\ud558\ub3c4\ub85d \ud558\ub294 \ubd80\uc11c\ud654\uc758 \ubc29\ubc95\uc774 \uc788\ub2e4. \uc804\ubb38\ud654\ub780 \uc5c5\ubb34\ub098 \ubaa9\uc801\uc5d0 \ub530\ub77c \uc5ed\ud560\uc774 \uc138\ubd84\ud654\ub41c \uc815\ub3c4\ub85c\uc11c, \uc870\uc9c1\uc5c5\ubb34\uac00 \ud558\uc704 \uc5c5\ubb34\ub85c \ub098\ub204\uc5b4\uc9c0\ub294 \uc815\ub3c4\uc640 \ub098\ub204\uc5b4\uc9c4 \ud558\uc704 \uc5c5\ubb34\uc5d0 \uc624\uc9c1 \ud558\ub098\uc758 \uc5ed\ud560\ub9cc\uc744 \uc774\ud589\ud558\ub3c4\ub85d \ubd84\ubc30\ub418\ub294 \uc815\ub3c4\ub97c \uc758\ubbf8\ud55c\ub2e4. \uc774\uc5d0 \ub530\ub77c \uc804\ubb38\ud654\ub294 \uc870\uc9c1 \ub0b4\uc5d0\uc11c \ucd94\uad6c\ub418\ub294 \ub2e4\uc591\ud55c \uae30\ub2a5\uc801 \ud65c\ub3d9\uc774\ub098 \uc5ed\ud560\uc758 \uc218\ub97c \ub098\ud0c0\ub0b4\ub294\ub370, \uc81c\ud55c\ub41c \ud558\ub098\uc758 \uc5c5\ubb34\ub9cc\uc744 \uc218\ud589\ud560 \ub54c \ub192\uc740 \uc218\uc900\uc758 \uc804\ubb38\ud654\ub97c \ubcf4\uc774\uac8c \ub418\uace0 \ubc18\uba74\uc5d0 \ub2e4\uc591\ud55c \ubc94\uc704\uc758 \uc77c\uc744 \uc218\ud589\ud558\uba70 \uc5c5\ubb34\ub97c \ube48\ubc88\ud558\uac8c \ubcc0\ud654\ud558\uac8c \ub418\uba74 \ub0ae\uc740 \uc218\uc900\uc758 \uc804\ubb38\ud654\ub97c \ubcf4\uc774\uac8c \ub41c\ub2e4. \ubd80\uc11c\ud654\ub294 \uc218\ud3c9\uc801\uc73c\ub85c \ubd84\ud654\ub41c \uc804\ubb38\uc801 \ud65c\ub3d9\uc744 \ud1b5\uc81c\ud558\ub294 \uc804\ud615\uc801\uc778 \ubc29\ubc95\uc73c\ub85c \uc804\ubb38\uac00\ub4e4\uc744 \uc9d1\ub2e8\ud654\uc2dc\ucf1c \uc77c\uc744 \uc218\ud589\ud558\ub3c4\ub85d \ud558\ub294 \uac83\uc744 \ub9d0\ud55c\ub2e4."} {"question": "\ub124\uc774\ubc84 \ubba4\uc9c1 2014 10\uc6d4 3\uc8fc, \uc774\uc8fc\uc758 \ubc1c\uacac\uc5d0 \uc624\ub978 \uc7a5\uae30\ud558\uc640 \uc5bc\uad74\ub4e4\uc758 \uc568\ubc94 \uc774\ub984\uc740?", "paragraph": "\u300a\uc0ac\ub78c\uc758 \ub9c8\uc74c\u300b\uc740 \uc74c\uc545 \ud3c9\ub860\uac00\ub85c\ubd80\ud130 \ud638\ud3c9\uc744 \ubc1b\uc558\ub2e4. \uc74c\ubc18\uc740 \ub124\uc774\ubc84 \ubba4\uc9c1 2014 10\uc6d4 3\uc8fc, \uc774\uc8fc\uc758 \ubc1c\uacac\uc5d0 \uc62c\ub790\uace0 \uc774\uacbd\uc900 \uc74c\uc545 \ud3c9\ub860\uac00\ub294 \"\uc728\ub3d9\uc774\ub098 \uc190\uc9d3, \uc720\uba38 \ub4f1 \uc74c\uc545 \uc678\uc801\uc778 \ucf54\ub4dc\ub97c \ubaa8\uc870\ub9ac \uc81c\uac70\ud574\ub3c4 \uadf8 \uc790\uccb4\ub85c \ub9e4\ud639\uc801\uc778 \uc74c\ubc18\uc774 \ub418\uc5c8\ub2e4.\"\uace0 \ud638\ud3c9\ud588\ub2e4. \u300a\uc774\uc998\u300b\uc758 \uae40\ub3c4\ud5cc\uc740 \"\uc138 \ubc88\uc9f8 \uc815\uaddc \uc568\ubc94\uc5d0\uc11c\uc57c \ube44\ub85c\uc18c \uc7a5\uae30\ud558\uc640 \uc5bc\uad74\ub4e4\uc740 '\uc7a5\uae30\ud558'\ub77c\ub294 \uc774\ub984 \ub300\uc2e0 '\uc7a5\uae30\ud558\uc640 \uc5bc\uad74\ub4e4'\uc774\ub77c\ub294 \ud504\ub808\uc784\uc5d0 \uc9d1\uc911\ud55c\ub2e4. ... \uba64\ubc84 \uc804\uc6d0\uc774 \uace0\ub974\uac8c \uc568\ubc94 \uc791\uc5c5\uc5d0 \ucc38\uc5ec\ud588\ub2e4\ub294 \uc0ac\uc2e4\uc740 \uadf8 \ud2b9\uc815 \ub204\uad6c\uc758 \uc774\uc57c\uae30\ub3c4 \uc544\ub2cc \ubcf4\ud3b8\uc801 '\uc0ac\ub78c\uc758 \ub9c8\uc74c'\uc744 \ub2e4\ub8e8\uba70 \ub4dc\ub7ec\ub09c\ub2e4. \uc7a5\uae30\ud558 \ubcf8\uc778\uc758 \uc2a4\ud1a0\ub9ac\ud154\ub9c1\uc774 \uc8fc\uac00 \ub418\uc5c8\ub358 \uacfc\uac70\uc5d0 \ube44\ud574 \uc0c8 \uc568\ubc94\uc758 \uc8fc\uc81c\ub294 \ubcf4\ub2e4 \ud3ed\ub113\uc740 \ubcf4\ud3b8\uc801 \uac10\uc815\uc5d0 \uae30\uc778\ud558\uace0 \uc788\uace0, \uc774\ub97c \uac04\uacb0\ud55c \ud45c\ud604\uc73c\ub85c \uc54c\ub9de\uac8c \uc804\ub2ec\ud558\ub824\ub294 \ub178\ub825\uc774 \ub3cb\ubcf4\uc778\ub2e4\"\uc9c0\ub9cc, \"\uc544\uc9c1\uae4c\uc9c0 \uc644\uc804\ud55c \uc131\ucde8\ub97c \ub17c\ud558\uae30\uc5d0\ub294 \uc5b4\uc0c9\ud55c \ubd80\ubd84\uc774 \ub9ce\ub2e4. \ud655\uc2e4\ud55c \ud6c4\ub834\uad6c\uac00 \ubd84\uba85 \uc778\uc0c1\uc801\uc774\ub098 \uadf8\uacf3\uae4c\uc9c0 \ub2ff\ub294 \ubc29\uc2dd\uc5d0\uc11c \uc5ec\uc804\ud788 \uacfc\uac70\uc758 '\uc2a4\ud1a0\ub9ac\ud154\ub9c1' \ucc3d\ubc95\uc774 \ubc1c\ubaa9\uc744 \uc7a1\ub294 \ud0d3\uc774\ub2e4.\"\ub77c\uace0 \uc9c0\uc801\ud588\uace0 \"\uc74c\uc545 \uc790\uccb4\uc5d0\uc11c\uc758 \ub2e8\uc810\uc744 \ub300\uac70 \ubcf4\uc644\ud558\uc5ec \ubc34\ub4dc\uc5d0\uac8c \uc815\uc791 \ubd80\uc871\ud588\ub358 \uc0ac\uc6b4\ub4dc\uc758 \uacf5\ub3d9(\u7a7a\u6d1e)\ud654\ub97c \ucc44\uc6cc\ub098\uac00\ub294 \ubaa8\uc2b5\uc740 \ubd84\uba85\ud55c \uac1c\uc120\uc758 \ub178\ub825\uc774\ub2e4.\"\ub77c\uba70 \uae30\ud68c\ub97c \ub0a8\uaca8\ub454 \ucc44 5\uc810\uc5d0 3\uc810\uc744 \uc92c\ub2e4.", "answer": "\uc0ac\ub78c\uc758 \ub9c8\uc74c", "sentence": "\u300a \uc0ac\ub78c\uc758 \ub9c8\uc74c \u300b\uc740 \uc74c\uc545 \ud3c9\ub860\uac00\ub85c\ubd80\ud130 \ud638\ud3c9\uc744 \ubc1b\uc558\ub2e4.", "paragraph_sentence": " \u300a \uc0ac\ub78c\uc758 \ub9c8\uc74c \u300b\uc740 \uc74c\uc545 \ud3c9\ub860\uac00\ub85c\ubd80\ud130 \ud638\ud3c9\uc744 \ubc1b\uc558\ub2e4. \uc74c\ubc18\uc740 \ub124\uc774\ubc84 \ubba4\uc9c1 2014 10\uc6d4 3\uc8fc, \uc774\uc8fc\uc758 \ubc1c\uacac\uc5d0 \uc62c\ub790\uace0 \uc774\uacbd\uc900 \uc74c\uc545 \ud3c9\ub860\uac00\ub294 \"\uc728\ub3d9\uc774\ub098 \uc190\uc9d3, \uc720\uba38 \ub4f1 \uc74c\uc545 \uc678\uc801\uc778 \ucf54\ub4dc\ub97c \ubaa8\uc870\ub9ac \uc81c\uac70\ud574\ub3c4 \uadf8 \uc790\uccb4\ub85c \ub9e4\ud639\uc801\uc778 \uc74c\ubc18\uc774 \ub418\uc5c8\ub2e4. \"\uace0 \ud638\ud3c9\ud588\ub2e4. \u300a\uc774\uc998\u300b\uc758 \uae40\ub3c4\ud5cc\uc740 \"\uc138 \ubc88\uc9f8 \uc815\uaddc \uc568\ubc94\uc5d0\uc11c\uc57c \ube44\ub85c\uc18c \uc7a5\uae30\ud558\uc640 \uc5bc\uad74\ub4e4\uc740 '\uc7a5\uae30\ud558'\ub77c\ub294 \uc774\ub984 \ub300\uc2e0 '\uc7a5\uae30\ud558\uc640 \uc5bc\uad74\ub4e4'\uc774\ub77c\ub294 \ud504\ub808\uc784\uc5d0 \uc9d1\uc911\ud55c\ub2e4. ... \uba64\ubc84 \uc804\uc6d0\uc774 \uace0\ub974\uac8c \uc568\ubc94 \uc791\uc5c5\uc5d0 \ucc38\uc5ec\ud588\ub2e4\ub294 \uc0ac\uc2e4\uc740 \uadf8 \ud2b9\uc815 \ub204\uad6c\uc758 \uc774\uc57c\uae30\ub3c4 \uc544\ub2cc \ubcf4\ud3b8\uc801 '\uc0ac\ub78c\uc758 \ub9c8\uc74c'\uc744 \ub2e4\ub8e8\uba70 \ub4dc\ub7ec\ub09c\ub2e4. \uc7a5\uae30\ud558 \ubcf8\uc778\uc758 \uc2a4\ud1a0\ub9ac\ud154\ub9c1\uc774 \uc8fc\uac00 \ub418\uc5c8\ub358 \uacfc\uac70\uc5d0 \ube44\ud574 \uc0c8 \uc568\ubc94\uc758 \uc8fc\uc81c\ub294 \ubcf4\ub2e4 \ud3ed\ub113\uc740 \ubcf4\ud3b8\uc801 \uac10\uc815\uc5d0 \uae30\uc778\ud558\uace0 \uc788\uace0, \uc774\ub97c \uac04\uacb0\ud55c \ud45c\ud604\uc73c\ub85c \uc54c\ub9de\uac8c \uc804\ub2ec\ud558\ub824\ub294 \ub178\ub825\uc774 \ub3cb\ubcf4\uc778\ub2e4\"\uc9c0\ub9cc, \"\uc544\uc9c1\uae4c\uc9c0 \uc644\uc804\ud55c \uc131\ucde8\ub97c \ub17c\ud558\uae30\uc5d0\ub294 \uc5b4\uc0c9\ud55c \ubd80\ubd84\uc774 \ub9ce\ub2e4. \ud655\uc2e4\ud55c \ud6c4\ub834\uad6c\uac00 \ubd84\uba85 \uc778\uc0c1\uc801\uc774\ub098 \uadf8\uacf3\uae4c\uc9c0 \ub2ff\ub294 \ubc29\uc2dd\uc5d0\uc11c \uc5ec\uc804\ud788 \uacfc\uac70\uc758 '\uc2a4\ud1a0\ub9ac\ud154\ub9c1' \ucc3d\ubc95\uc774 \ubc1c\ubaa9\uc744 \uc7a1\ub294 \ud0d3\uc774\ub2e4. \"\ub77c\uace0 \uc9c0\uc801\ud588\uace0 \"\uc74c\uc545 \uc790\uccb4\uc5d0\uc11c\uc758 \ub2e8\uc810\uc744 \ub300\uac70 \ubcf4\uc644\ud558\uc5ec \ubc34\ub4dc\uc5d0\uac8c \uc815\uc791 \ubd80\uc871\ud588\ub358 \uc0ac\uc6b4\ub4dc\uc758 \uacf5\ub3d9(\u7a7a\u6d1e)\ud654\ub97c \ucc44\uc6cc\ub098\uac00\ub294 \ubaa8\uc2b5\uc740 \ubd84\uba85\ud55c \uac1c\uc120\uc758 \ub178\ub825\uc774\ub2e4. \"\ub77c\uba70 \uae30\ud68c\ub97c \ub0a8\uaca8\ub454 \ucc44 5\uc810\uc5d0 3\uc810\uc744 \uc92c\ub2e4.", "paragraph_answer": "\u300a \uc0ac\ub78c\uc758 \ub9c8\uc74c \u300b\uc740 \uc74c\uc545 \ud3c9\ub860\uac00\ub85c\ubd80\ud130 \ud638\ud3c9\uc744 \ubc1b\uc558\ub2e4. \uc74c\ubc18\uc740 \ub124\uc774\ubc84 \ubba4\uc9c1 2014 10\uc6d4 3\uc8fc, \uc774\uc8fc\uc758 \ubc1c\uacac\uc5d0 \uc62c\ub790\uace0 \uc774\uacbd\uc900 \uc74c\uc545 \ud3c9\ub860\uac00\ub294 \"\uc728\ub3d9\uc774\ub098 \uc190\uc9d3, \uc720\uba38 \ub4f1 \uc74c\uc545 \uc678\uc801\uc778 \ucf54\ub4dc\ub97c \ubaa8\uc870\ub9ac \uc81c\uac70\ud574\ub3c4 \uadf8 \uc790\uccb4\ub85c \ub9e4\ud639\uc801\uc778 \uc74c\ubc18\uc774 \ub418\uc5c8\ub2e4.\"\uace0 \ud638\ud3c9\ud588\ub2e4. \u300a\uc774\uc998\u300b\uc758 \uae40\ub3c4\ud5cc\uc740 \"\uc138 \ubc88\uc9f8 \uc815\uaddc \uc568\ubc94\uc5d0\uc11c\uc57c \ube44\ub85c\uc18c \uc7a5\uae30\ud558\uc640 \uc5bc\uad74\ub4e4\uc740 '\uc7a5\uae30\ud558'\ub77c\ub294 \uc774\ub984 \ub300\uc2e0 '\uc7a5\uae30\ud558\uc640 \uc5bc\uad74\ub4e4'\uc774\ub77c\ub294 \ud504\ub808\uc784\uc5d0 \uc9d1\uc911\ud55c\ub2e4. ... \uba64\ubc84 \uc804\uc6d0\uc774 \uace0\ub974\uac8c \uc568\ubc94 \uc791\uc5c5\uc5d0 \ucc38\uc5ec\ud588\ub2e4\ub294 \uc0ac\uc2e4\uc740 \uadf8 \ud2b9\uc815 \ub204\uad6c\uc758 \uc774\uc57c\uae30\ub3c4 \uc544\ub2cc \ubcf4\ud3b8\uc801 '\uc0ac\ub78c\uc758 \ub9c8\uc74c'\uc744 \ub2e4\ub8e8\uba70 \ub4dc\ub7ec\ub09c\ub2e4. \uc7a5\uae30\ud558 \ubcf8\uc778\uc758 \uc2a4\ud1a0\ub9ac\ud154\ub9c1\uc774 \uc8fc\uac00 \ub418\uc5c8\ub358 \uacfc\uac70\uc5d0 \ube44\ud574 \uc0c8 \uc568\ubc94\uc758 \uc8fc\uc81c\ub294 \ubcf4\ub2e4 \ud3ed\ub113\uc740 \ubcf4\ud3b8\uc801 \uac10\uc815\uc5d0 \uae30\uc778\ud558\uace0 \uc788\uace0, \uc774\ub97c \uac04\uacb0\ud55c \ud45c\ud604\uc73c\ub85c \uc54c\ub9de\uac8c \uc804\ub2ec\ud558\ub824\ub294 \ub178\ub825\uc774 \ub3cb\ubcf4\uc778\ub2e4\"\uc9c0\ub9cc, \"\uc544\uc9c1\uae4c\uc9c0 \uc644\uc804\ud55c \uc131\ucde8\ub97c \ub17c\ud558\uae30\uc5d0\ub294 \uc5b4\uc0c9\ud55c \ubd80\ubd84\uc774 \ub9ce\ub2e4. \ud655\uc2e4\ud55c \ud6c4\ub834\uad6c\uac00 \ubd84\uba85 \uc778\uc0c1\uc801\uc774\ub098 \uadf8\uacf3\uae4c\uc9c0 \ub2ff\ub294 \ubc29\uc2dd\uc5d0\uc11c \uc5ec\uc804\ud788 \uacfc\uac70\uc758 '\uc2a4\ud1a0\ub9ac\ud154\ub9c1' \ucc3d\ubc95\uc774 \ubc1c\ubaa9\uc744 \uc7a1\ub294 \ud0d3\uc774\ub2e4.\"\ub77c\uace0 \uc9c0\uc801\ud588\uace0 \"\uc74c\uc545 \uc790\uccb4\uc5d0\uc11c\uc758 \ub2e8\uc810\uc744 \ub300\uac70 \ubcf4\uc644\ud558\uc5ec \ubc34\ub4dc\uc5d0\uac8c \uc815\uc791 \ubd80\uc871\ud588\ub358 \uc0ac\uc6b4\ub4dc\uc758 \uacf5\ub3d9(\u7a7a\u6d1e)\ud654\ub97c \ucc44\uc6cc\ub098\uac00\ub294 \ubaa8\uc2b5\uc740 \ubd84\uba85\ud55c \uac1c\uc120\uc758 \ub178\ub825\uc774\ub2e4.\"\ub77c\uba70 \uae30\ud68c\ub97c \ub0a8\uaca8\ub454 \ucc44 5\uc810\uc5d0 3\uc810\uc744 \uc92c\ub2e4.", "sentence_answer": "\u300a \uc0ac\ub78c\uc758 \ub9c8\uc74c \u300b\uc740 \uc74c\uc545 \ud3c9\ub860\uac00\ub85c\ubd80\ud130 \ud638\ud3c9\uc744 \ubc1b\uc558\ub2e4.", "paragraph_id": "6535354-14-1", "paragraph_question": "question: \ub124\uc774\ubc84 \ubba4\uc9c1 2014 10\uc6d4 3\uc8fc, \uc774\uc8fc\uc758 \ubc1c\uacac\uc5d0 \uc624\ub978 \uc7a5\uae30\ud558\uc640 \uc5bc\uad74\ub4e4\uc758 \uc568\ubc94 \uc774\ub984\uc740?, context: \u300a\uc0ac\ub78c\uc758 \ub9c8\uc74c\u300b\uc740 \uc74c\uc545 \ud3c9\ub860\uac00\ub85c\ubd80\ud130 \ud638\ud3c9\uc744 \ubc1b\uc558\ub2e4. \uc74c\ubc18\uc740 \ub124\uc774\ubc84 \ubba4\uc9c1 2014 10\uc6d4 3\uc8fc, \uc774\uc8fc\uc758 \ubc1c\uacac\uc5d0 \uc62c\ub790\uace0 \uc774\uacbd\uc900 \uc74c\uc545 \ud3c9\ub860\uac00\ub294 \"\uc728\ub3d9\uc774\ub098 \uc190\uc9d3, \uc720\uba38 \ub4f1 \uc74c\uc545 \uc678\uc801\uc778 \ucf54\ub4dc\ub97c \ubaa8\uc870\ub9ac \uc81c\uac70\ud574\ub3c4 \uadf8 \uc790\uccb4\ub85c \ub9e4\ud639\uc801\uc778 \uc74c\ubc18\uc774 \ub418\uc5c8\ub2e4.\"\uace0 \ud638\ud3c9\ud588\ub2e4. \u300a\uc774\uc998\u300b\uc758 \uae40\ub3c4\ud5cc\uc740 \"\uc138 \ubc88\uc9f8 \uc815\uaddc \uc568\ubc94\uc5d0\uc11c\uc57c \ube44\ub85c\uc18c \uc7a5\uae30\ud558\uc640 \uc5bc\uad74\ub4e4\uc740 '\uc7a5\uae30\ud558'\ub77c\ub294 \uc774\ub984 \ub300\uc2e0 '\uc7a5\uae30\ud558\uc640 \uc5bc\uad74\ub4e4'\uc774\ub77c\ub294 \ud504\ub808\uc784\uc5d0 \uc9d1\uc911\ud55c\ub2e4. ... \uba64\ubc84 \uc804\uc6d0\uc774 \uace0\ub974\uac8c \uc568\ubc94 \uc791\uc5c5\uc5d0 \ucc38\uc5ec\ud588\ub2e4\ub294 \uc0ac\uc2e4\uc740 \uadf8 \ud2b9\uc815 \ub204\uad6c\uc758 \uc774\uc57c\uae30\ub3c4 \uc544\ub2cc \ubcf4\ud3b8\uc801 '\uc0ac\ub78c\uc758 \ub9c8\uc74c'\uc744 \ub2e4\ub8e8\uba70 \ub4dc\ub7ec\ub09c\ub2e4. \uc7a5\uae30\ud558 \ubcf8\uc778\uc758 \uc2a4\ud1a0\ub9ac\ud154\ub9c1\uc774 \uc8fc\uac00 \ub418\uc5c8\ub358 \uacfc\uac70\uc5d0 \ube44\ud574 \uc0c8 \uc568\ubc94\uc758 \uc8fc\uc81c\ub294 \ubcf4\ub2e4 \ud3ed\ub113\uc740 \ubcf4\ud3b8\uc801 \uac10\uc815\uc5d0 \uae30\uc778\ud558\uace0 \uc788\uace0, \uc774\ub97c \uac04\uacb0\ud55c \ud45c\ud604\uc73c\ub85c \uc54c\ub9de\uac8c \uc804\ub2ec\ud558\ub824\ub294 \ub178\ub825\uc774 \ub3cb\ubcf4\uc778\ub2e4\"\uc9c0\ub9cc, \"\uc544\uc9c1\uae4c\uc9c0 \uc644\uc804\ud55c \uc131\ucde8\ub97c \ub17c\ud558\uae30\uc5d0\ub294 \uc5b4\uc0c9\ud55c \ubd80\ubd84\uc774 \ub9ce\ub2e4. \ud655\uc2e4\ud55c \ud6c4\ub834\uad6c\uac00 \ubd84\uba85 \uc778\uc0c1\uc801\uc774\ub098 \uadf8\uacf3\uae4c\uc9c0 \ub2ff\ub294 \ubc29\uc2dd\uc5d0\uc11c \uc5ec\uc804\ud788 \uacfc\uac70\uc758 '\uc2a4\ud1a0\ub9ac\ud154\ub9c1' \ucc3d\ubc95\uc774 \ubc1c\ubaa9\uc744 \uc7a1\ub294 \ud0d3\uc774\ub2e4.\"\ub77c\uace0 \uc9c0\uc801\ud588\uace0 \"\uc74c\uc545 \uc790\uccb4\uc5d0\uc11c\uc758 \ub2e8\uc810\uc744 \ub300\uac70 \ubcf4\uc644\ud558\uc5ec \ubc34\ub4dc\uc5d0\uac8c \uc815\uc791 \ubd80\uc871\ud588\ub358 \uc0ac\uc6b4\ub4dc\uc758 \uacf5\ub3d9(\u7a7a\u6d1e)\ud654\ub97c \ucc44\uc6cc\ub098\uac00\ub294 \ubaa8\uc2b5\uc740 \ubd84\uba85\ud55c \uac1c\uc120\uc758 \ub178\ub825\uc774\ub2e4.\"\ub77c\uba70 \uae30\ud68c\ub97c \ub0a8\uaca8\ub454 \ucc44 5\uc810\uc5d0 3\uc810\uc744 \uc92c\ub2e4."} {"question": "\uc0bc\uc5fd\ucda9\uc740 \ud5c8\ubb3c\uc744 \ud55c\ubc88 \ubc97\uc744 \ub54c \ub9c8\ub2e4 \uba87\uac1c\uc758 \ub9c8\ub514\uac00 \ucd94\uac00\ub418\ub294\uac00?", "paragraph": "(\uc5ed\uc2dc \uc544\ub098\ubab0\ud53d \ud2b9\uc131\uc744 \uac00\uc9c4) \"\uba54\ub77c\uc2a4\ud53c\ub4dc\" \ub2e8\uacc4\ub294 \uba38\ub9ac\uc640 \ubab8\ud1b5 \uc0ac\uc774\uc758 \ubd84\uc808\uc774 \ub098\ud0c0\ub098\ub294 \uac83\uc73c\ub85c \uc2dc\uc791\ub41c\ub2e4. \uccab\ubc88\uc9f8 \uba54\ub77c\uc2a4\ud53c\ub4dc \ub2e8\uacc4\uac00 \uc2dc\uc791\ub418\uae30 \uc804\uc5d0 \uc0bc\uc5fd\ucda9\uc740 \ub450 \ubd80\ubd84 - \uba38\ub9ac, \uadf8\ub9ac\uace0 \ubab8\ud1b5 \ub9c8\ub514\uac00 \ud569\uccd0\uc9c4 \uaf2c\ub9ac - \uc73c\ub85c \uad6c\uc131\ub418\uc5b4 \uc788\ub2e4. \uba54\ub77c\uc2a4\ud53c\ub4dc \ub2e8\uacc4\uc5d0\uc11c\ub294 \uc0c8\ub85c\uc6b4 \ub9c8\ub514\uac00 \uaf2c\ub9ac\uc758 \ub4b7\ubd80\ubd84\uc5d0\uc11c \uc0dd\uaca8\ub098\ub294 \ub3d9\uc2dc\uc5d0 \uaf2c\ub9ac\uc758 \uc55e\ucabd\uc5d0\uc11c\ub294 \ucd94\uac00\uc801\uc778 \ubd84\uc808\uc774 \uc0dd\uaca8\ub098\uba74\uc11c \ub9c8\ub514\uac00 \uaf2c\ub9ac\uc5d0\uc11c \ub5a8\uc5b4\uc838 \ub098\uc640 \uac00\uc2b4\uc73c\ub85c \uc774\ub3d9\ud55c\ub2e4. \ubcf4\ud1b5 \ud55c \ubc88 \ud5c8\ubb3c\uc744 \ubc97\uc744 \ub54c \ub9c8\ub514\uac00 \ud558\ub098\uc529 \ucd94\uac00\ub418\uc5b4 (\ud55c \ubc88 \ud5c8\ubb3c \ubc97\uc744 \ub54c \ub450 \uac1c\uac00 \uc0dd\uaca8\ub098\ub294 \uacbd\uc6b0\ub3c4 \uc788\uace0 \ud55c \ubc88 \uac78\ub7ec \ud558\ub098\uc529 \uc0dd\uaca8\ub098\ub294 \uacbd\uc6b0\ub3c4 \uc788\uae34 \ud558\ub2e4) \ud5c8\ubb3c\uc744 \ubc97\ub294 \ud68c\uc218\uc640 \uac00\uc2b4 \ub9c8\ub514\uc758 \uc218\uac00 \uac19\uac8c \ub41c\ub2e4. \uc804\uccb4 \ub300\ube44 25% \ubbf8\ub9cc\uc5d0\uc11c 30%-40% \uc815\ub3c4\uae4c\uc9c0 \uc0c1\ub2f9\ud55c \ube44\uc728\uc758 \uc131\uc7a5\uc774 \uba54\ub77c\uc2a4\ud53c\ub4dc \ub2e8\uacc4\uc5d0\uc11c \uc774\ub8e8\uc5b4\uc9c4\ub2e4.", "answer": "\ud558\ub098\uc529", "sentence": "\ubcf4\ud1b5 \ud55c \ubc88 \ud5c8\ubb3c\uc744 \ubc97\uc744 \ub54c \ub9c8\ub514\uac00 \ud558\ub098\uc529 \ucd94\uac00\ub418\uc5b4 (\ud55c \ubc88 \ud5c8\ubb3c \ubc97\uc744 \ub54c \ub450 \uac1c\uac00 \uc0dd\uaca8\ub098\ub294 \uacbd\uc6b0\ub3c4 \uc788\uace0 \ud55c \ubc88 \uac78\ub7ec \ud558\ub098\uc529 \uc0dd\uaca8\ub098\ub294 \uacbd\uc6b0\ub3c4 \uc788\uae34 \ud558\ub2e4) \ud5c8\ubb3c\uc744 \ubc97\ub294 \ud68c\uc218\uc640 \uac00\uc2b4 \ub9c8\ub514\uc758 \uc218\uac00 \uac19\uac8c \ub41c\ub2e4.", "paragraph_sentence": "(\uc5ed\uc2dc \uc544\ub098\ubab0\ud53d \ud2b9\uc131\uc744 \uac00\uc9c4) \"\uba54\ub77c\uc2a4\ud53c\ub4dc\" \ub2e8\uacc4\ub294 \uba38\ub9ac\uc640 \ubab8\ud1b5 \uc0ac\uc774\uc758 \ubd84\uc808\uc774 \ub098\ud0c0\ub098\ub294 \uac83\uc73c\ub85c \uc2dc\uc791\ub41c\ub2e4. \uccab\ubc88\uc9f8 \uba54\ub77c\uc2a4\ud53c\ub4dc \ub2e8\uacc4\uac00 \uc2dc\uc791\ub418\uae30 \uc804\uc5d0 \uc0bc\uc5fd\ucda9\uc740 \ub450 \ubd80\ubd84 - \uba38\ub9ac, \uadf8\ub9ac\uace0 \ubab8\ud1b5 \ub9c8\ub514\uac00 \ud569\uccd0\uc9c4 \uaf2c\ub9ac - \uc73c\ub85c \uad6c\uc131\ub418\uc5b4 \uc788\ub2e4. \uba54\ub77c\uc2a4\ud53c\ub4dc \ub2e8\uacc4\uc5d0\uc11c\ub294 \uc0c8\ub85c\uc6b4 \ub9c8\ub514\uac00 \uaf2c\ub9ac\uc758 \ub4b7\ubd80\ubd84\uc5d0\uc11c \uc0dd\uaca8\ub098\ub294 \ub3d9\uc2dc\uc5d0 \uaf2c\ub9ac\uc758 \uc55e\ucabd\uc5d0\uc11c\ub294 \ucd94\uac00\uc801\uc778 \ubd84\uc808\uc774 \uc0dd\uaca8\ub098\uba74\uc11c \ub9c8\ub514\uac00 \uaf2c\ub9ac\uc5d0\uc11c \ub5a8\uc5b4\uc838 \ub098\uc640 \uac00\uc2b4\uc73c\ub85c \uc774\ub3d9\ud55c\ub2e4. \ubcf4\ud1b5 \ud55c \ubc88 \ud5c8\ubb3c\uc744 \ubc97\uc744 \ub54c \ub9c8\ub514\uac00 \ud558\ub098\uc529 \ucd94\uac00\ub418\uc5b4 (\ud55c \ubc88 \ud5c8\ubb3c \ubc97\uc744 \ub54c \ub450 \uac1c\uac00 \uc0dd\uaca8\ub098\ub294 \uacbd\uc6b0\ub3c4 \uc788\uace0 \ud55c \ubc88 \uac78\ub7ec \ud558\ub098\uc529 \uc0dd\uaca8\ub098\ub294 \uacbd\uc6b0\ub3c4 \uc788\uae34 \ud558\ub2e4) \ud5c8\ubb3c\uc744 \ubc97\ub294 \ud68c\uc218\uc640 \uac00\uc2b4 \ub9c8\ub514\uc758 \uc218\uac00 \uac19\uac8c \ub41c\ub2e4. \uc804\uccb4 \ub300\ube44 25% \ubbf8\ub9cc\uc5d0\uc11c 30%-40% \uc815\ub3c4\uae4c\uc9c0 \uc0c1\ub2f9\ud55c \ube44\uc728\uc758 \uc131\uc7a5\uc774 \uba54\ub77c\uc2a4\ud53c\ub4dc \ub2e8\uacc4\uc5d0\uc11c \uc774\ub8e8\uc5b4\uc9c4\ub2e4.", "paragraph_answer": "(\uc5ed\uc2dc \uc544\ub098\ubab0\ud53d \ud2b9\uc131\uc744 \uac00\uc9c4) \"\uba54\ub77c\uc2a4\ud53c\ub4dc\" \ub2e8\uacc4\ub294 \uba38\ub9ac\uc640 \ubab8\ud1b5 \uc0ac\uc774\uc758 \ubd84\uc808\uc774 \ub098\ud0c0\ub098\ub294 \uac83\uc73c\ub85c \uc2dc\uc791\ub41c\ub2e4. \uccab\ubc88\uc9f8 \uba54\ub77c\uc2a4\ud53c\ub4dc \ub2e8\uacc4\uac00 \uc2dc\uc791\ub418\uae30 \uc804\uc5d0 \uc0bc\uc5fd\ucda9\uc740 \ub450 \ubd80\ubd84 - \uba38\ub9ac, \uadf8\ub9ac\uace0 \ubab8\ud1b5 \ub9c8\ub514\uac00 \ud569\uccd0\uc9c4 \uaf2c\ub9ac - \uc73c\ub85c \uad6c\uc131\ub418\uc5b4 \uc788\ub2e4. \uba54\ub77c\uc2a4\ud53c\ub4dc \ub2e8\uacc4\uc5d0\uc11c\ub294 \uc0c8\ub85c\uc6b4 \ub9c8\ub514\uac00 \uaf2c\ub9ac\uc758 \ub4b7\ubd80\ubd84\uc5d0\uc11c \uc0dd\uaca8\ub098\ub294 \ub3d9\uc2dc\uc5d0 \uaf2c\ub9ac\uc758 \uc55e\ucabd\uc5d0\uc11c\ub294 \ucd94\uac00\uc801\uc778 \ubd84\uc808\uc774 \uc0dd\uaca8\ub098\uba74\uc11c \ub9c8\ub514\uac00 \uaf2c\ub9ac\uc5d0\uc11c \ub5a8\uc5b4\uc838 \ub098\uc640 \uac00\uc2b4\uc73c\ub85c \uc774\ub3d9\ud55c\ub2e4. \ubcf4\ud1b5 \ud55c \ubc88 \ud5c8\ubb3c\uc744 \ubc97\uc744 \ub54c \ub9c8\ub514\uac00 \ud558\ub098\uc529 \ucd94\uac00\ub418\uc5b4 (\ud55c \ubc88 \ud5c8\ubb3c \ubc97\uc744 \ub54c \ub450 \uac1c\uac00 \uc0dd\uaca8\ub098\ub294 \uacbd\uc6b0\ub3c4 \uc788\uace0 \ud55c \ubc88 \uac78\ub7ec \ud558\ub098\uc529 \uc0dd\uaca8\ub098\ub294 \uacbd\uc6b0\ub3c4 \uc788\uae34 \ud558\ub2e4) \ud5c8\ubb3c\uc744 \ubc97\ub294 \ud68c\uc218\uc640 \uac00\uc2b4 \ub9c8\ub514\uc758 \uc218\uac00 \uac19\uac8c \ub41c\ub2e4. \uc804\uccb4 \ub300\ube44 25% \ubbf8\ub9cc\uc5d0\uc11c 30%-40% \uc815\ub3c4\uae4c\uc9c0 \uc0c1\ub2f9\ud55c \ube44\uc728\uc758 \uc131\uc7a5\uc774 \uba54\ub77c\uc2a4\ud53c\ub4dc \ub2e8\uacc4\uc5d0\uc11c \uc774\ub8e8\uc5b4\uc9c4\ub2e4.", "sentence_answer": "\ubcf4\ud1b5 \ud55c \ubc88 \ud5c8\ubb3c\uc744 \ubc97\uc744 \ub54c \ub9c8\ub514\uac00 \ud558\ub098\uc529 \ucd94\uac00\ub418\uc5b4 (\ud55c \ubc88 \ud5c8\ubb3c \ubc97\uc744 \ub54c \ub450 \uac1c\uac00 \uc0dd\uaca8\ub098\ub294 \uacbd\uc6b0\ub3c4 \uc788\uace0 \ud55c \ubc88 \uac78\ub7ec \ud558\ub098\uc529 \uc0dd\uaca8\ub098\ub294 \uacbd\uc6b0\ub3c4 \uc788\uae34 \ud558\ub2e4) \ud5c8\ubb3c\uc744 \ubc97\ub294 \ud68c\uc218\uc640 \uac00\uc2b4 \ub9c8\ub514\uc758 \uc218\uac00 \uac19\uac8c \ub41c\ub2e4.", "paragraph_id": "6583774-14-2", "paragraph_question": "question: \uc0bc\uc5fd\ucda9\uc740 \ud5c8\ubb3c\uc744 \ud55c\ubc88 \ubc97\uc744 \ub54c \ub9c8\ub2e4 \uba87\uac1c\uc758 \ub9c8\ub514\uac00 \ucd94\uac00\ub418\ub294\uac00?, context: (\uc5ed\uc2dc \uc544\ub098\ubab0\ud53d \ud2b9\uc131\uc744 \uac00\uc9c4) \"\uba54\ub77c\uc2a4\ud53c\ub4dc\" \ub2e8\uacc4\ub294 \uba38\ub9ac\uc640 \ubab8\ud1b5 \uc0ac\uc774\uc758 \ubd84\uc808\uc774 \ub098\ud0c0\ub098\ub294 \uac83\uc73c\ub85c \uc2dc\uc791\ub41c\ub2e4. \uccab\ubc88\uc9f8 \uba54\ub77c\uc2a4\ud53c\ub4dc \ub2e8\uacc4\uac00 \uc2dc\uc791\ub418\uae30 \uc804\uc5d0 \uc0bc\uc5fd\ucda9\uc740 \ub450 \ubd80\ubd84 - \uba38\ub9ac, \uadf8\ub9ac\uace0 \ubab8\ud1b5 \ub9c8\ub514\uac00 \ud569\uccd0\uc9c4 \uaf2c\ub9ac - \uc73c\ub85c \uad6c\uc131\ub418\uc5b4 \uc788\ub2e4. \uba54\ub77c\uc2a4\ud53c\ub4dc \ub2e8\uacc4\uc5d0\uc11c\ub294 \uc0c8\ub85c\uc6b4 \ub9c8\ub514\uac00 \uaf2c\ub9ac\uc758 \ub4b7\ubd80\ubd84\uc5d0\uc11c \uc0dd\uaca8\ub098\ub294 \ub3d9\uc2dc\uc5d0 \uaf2c\ub9ac\uc758 \uc55e\ucabd\uc5d0\uc11c\ub294 \ucd94\uac00\uc801\uc778 \ubd84\uc808\uc774 \uc0dd\uaca8\ub098\uba74\uc11c \ub9c8\ub514\uac00 \uaf2c\ub9ac\uc5d0\uc11c \ub5a8\uc5b4\uc838 \ub098\uc640 \uac00\uc2b4\uc73c\ub85c \uc774\ub3d9\ud55c\ub2e4. \ubcf4\ud1b5 \ud55c \ubc88 \ud5c8\ubb3c\uc744 \ubc97\uc744 \ub54c \ub9c8\ub514\uac00 \ud558\ub098\uc529 \ucd94\uac00\ub418\uc5b4 (\ud55c \ubc88 \ud5c8\ubb3c \ubc97\uc744 \ub54c \ub450 \uac1c\uac00 \uc0dd\uaca8\ub098\ub294 \uacbd\uc6b0\ub3c4 \uc788\uace0 \ud55c \ubc88 \uac78\ub7ec \ud558\ub098\uc529 \uc0dd\uaca8\ub098\ub294 \uacbd\uc6b0\ub3c4 \uc788\uae34 \ud558\ub2e4) \ud5c8\ubb3c\uc744 \ubc97\ub294 \ud68c\uc218\uc640 \uac00\uc2b4 \ub9c8\ub514\uc758 \uc218\uac00 \uac19\uac8c \ub41c\ub2e4. \uc804\uccb4 \ub300\ube44 25% \ubbf8\ub9cc\uc5d0\uc11c 30%-40% \uc815\ub3c4\uae4c\uc9c0 \uc0c1\ub2f9\ud55c \ube44\uc728\uc758 \uc131\uc7a5\uc774 \uba54\ub77c\uc2a4\ud53c\ub4dc \ub2e8\uacc4\uc5d0\uc11c \uc774\ub8e8\uc5b4\uc9c4\ub2e4."} {"question": "\ud56d\uacf5\uae30 \uc0ac\uace0 \uad6c\uc870, \uc751\uae09\ucc98\uce58, \ud658\uc790 \uc774\uc1a1\uc744 \ub9e1\uace0 \uc788\ub294 \ubd80\ub300\ub294?", "paragraph": "\ucd5c\uace0\uc758 \uc218\uc911 \ud0d0\uc0c9\u00b7\uad6c\uc870 \ub2a5\ub825\uc744 \uac16\ucd98 \uacf5\uad70 \ud2b9\uc218\ubd80\ub300 \uc694\uc6d0\ub4e4\uc774 \uc138\uc6d4\ud638 \uce68\ubab0\uc0ac\uace0 \ub2f9\uc77c \ud604\uc7a5\uc5d0 \ucd9c\ub3d9\ud588\uc9c0\ub9cc \ud604\uc7a5 \uc9c0\ud718\ud1b5\uc81c \uae30\ub2a5 \ub9c8\ube44\ub85c \uc778\uba85\uad6c\uc870 \uc791\uc5c5\uc5d0 \ub2e8 1\uba85\ub3c4 \ud22c\uc785\ub418\uc9c0 \ubabb\ud55c \uac83\uc73c\ub85c 6\uc6d4 9\uc77c \ud655\uc778\ub410\ub2e4. \uc694\uc6d0\ub4e4\uc740 \uc2b9\uac1d\ub4e4\uc758 \uc0dd\uc874 \uac00\ub2a5\uc131\uc774 \ub192\uc740 \uace8\ub4e0\ud0c0\uc784 \ub0b4\ub0b4 \ud5ec\uae30 \uc548\uc5d0\uc11c \uc2e4\uc885\uc790\ub4e4\uc5d0 \ub300\ud55c \uc721\uc548\uc218\uc0c9\ub9cc \ubc8c\uc600\ub2e4. \uacf5\uad70 \ud56d\uacf5\uad6c\uc870\ub300\ub294 \ud3c9\uc2dc \ud56d\uacf5\uae30 \uc0ac\uace0\uad6c\uc870, \uc751\uae09\ucc98\uce58, \ud658\uc790 \uc774\uc1a1 \ub4f1\uc758 \uc784\ubb34\ub97c \ub9e1\ub294\ub2e4. \uc804\uc2dc\uc5d0\ub294 \uc801\uc9c4\uc5d0 \ud22c\uc785\ub3fc \uac15\uc774\ub098 \ubc14\ub2e4\uc5d0 \ub5a8\uc5b4\uc9c4 \uc870\uc885\uc0ac\ub97c \uad6c\ucd9c\ud558\ub294 \uc784\ubb34\ub97c \uc218\ud589\ud55c\ub2e4. \uc694\uc6d0\ub4e4\uc740 \uacf5\uc911\uce68\ud22c\ubfd0 \uc544\ub2c8\ub77c \ud574\uc0c1\uad6c\uc870\uc640 \uc751\uae09\uc758\ub8cc \ub4f1 \ud2b9\uc218\uc791\uc804\uc5d0 \ud544\uc694\ud55c \ubaa8\ub4e0 \ub2a5\ub825\uc744 \uac16\ucd94\uace0 \uc788\ub2e4. \uae40\uad11\uc9c4 \uc758\uc6d0\uc740 \u201c\uc138\uc6d4\ud638 \uce68\ubab0 \ub2f9\uc77c \ub300\ud55c\ubbfc\uad6d \uad6d\uad70 \ud2b9\uc218\ubd80\ub300 \uc694\uc6d0\ub4e4\uc740 \ub208\ub728\uace0 \ucc38\uc0ac\ub97c \uc9c0\ucf1c\ubcf4\uba74\uc11c \ud669\ub9dd\ud55c \uae30\ubd84\uc744 \ub290\uaf08\uc744 \uac83\u201d\uc774\ub77c\uba70 \u201c\ub2f9\uc2dc \ud604\uc7a5\uc744 \ud1b5\uc81c\ud588\ub358 \uc9c0\ud718\uad00\uc740 \ubc18\ub4dc\uc2dc \ubb38\ucc45\ub418\uc5b4\uc57c \ud55c\ub2e4\u201d\uace0 \ub9d0\ud588\ub2e4.", "answer": "\uacf5\uad70 \ud56d\uacf5\uad6c\uc870\ub300", "sentence": "\uacf5\uad70 \ud56d\uacf5\uad6c\uc870\ub300 \ub294 \ud3c9\uc2dc \ud56d\uacf5\uae30 \uc0ac\uace0\uad6c\uc870, \uc751\uae09\ucc98\uce58, \ud658\uc790 \uc774\uc1a1 \ub4f1\uc758 \uc784\ubb34\ub97c \ub9e1\ub294\ub2e4.", "paragraph_sentence": "\ucd5c\uace0\uc758 \uc218\uc911 \ud0d0\uc0c9\u00b7\uad6c\uc870 \ub2a5\ub825\uc744 \uac16\ucd98 \uacf5\uad70 \ud2b9\uc218\ubd80\ub300 \uc694\uc6d0\ub4e4\uc774 \uc138\uc6d4\ud638 \uce68\ubab0\uc0ac\uace0 \ub2f9\uc77c \ud604\uc7a5\uc5d0 \ucd9c\ub3d9\ud588\uc9c0\ub9cc \ud604\uc7a5 \uc9c0\ud718\ud1b5\uc81c \uae30\ub2a5 \ub9c8\ube44\ub85c \uc778\uba85\uad6c\uc870 \uc791\uc5c5\uc5d0 \ub2e8 1\uba85\ub3c4 \ud22c\uc785\ub418\uc9c0 \ubabb\ud55c \uac83\uc73c\ub85c 6\uc6d4 9\uc77c \ud655\uc778\ub410\ub2e4. \uc694\uc6d0\ub4e4\uc740 \uc2b9\uac1d\ub4e4\uc758 \uc0dd\uc874 \uac00\ub2a5\uc131\uc774 \ub192\uc740 \uace8\ub4e0\ud0c0\uc784 \ub0b4\ub0b4 \ud5ec\uae30 \uc548\uc5d0\uc11c \uc2e4\uc885\uc790\ub4e4\uc5d0 \ub300\ud55c \uc721\uc548\uc218\uc0c9\ub9cc \ubc8c\uc600\ub2e4. \uacf5\uad70 \ud56d\uacf5\uad6c\uc870\ub300 \ub294 \ud3c9\uc2dc \ud56d\uacf5\uae30 \uc0ac\uace0\uad6c\uc870, \uc751\uae09\ucc98\uce58, \ud658\uc790 \uc774\uc1a1 \ub4f1\uc758 \uc784\ubb34\ub97c \ub9e1\ub294\ub2e4. \uc804\uc2dc\uc5d0\ub294 \uc801\uc9c4\uc5d0 \ud22c\uc785\ub3fc \uac15\uc774\ub098 \ubc14\ub2e4\uc5d0 \ub5a8\uc5b4\uc9c4 \uc870\uc885\uc0ac\ub97c \uad6c\ucd9c\ud558\ub294 \uc784\ubb34\ub97c \uc218\ud589\ud55c\ub2e4. \uc694\uc6d0\ub4e4\uc740 \uacf5\uc911\uce68\ud22c\ubfd0 \uc544\ub2c8\ub77c \ud574\uc0c1\uad6c\uc870\uc640 \uc751\uae09\uc758\ub8cc \ub4f1 \ud2b9\uc218\uc791\uc804\uc5d0 \ud544\uc694\ud55c \ubaa8\ub4e0 \ub2a5\ub825\uc744 \uac16\ucd94\uace0 \uc788\ub2e4. \uae40\uad11\uc9c4 \uc758\uc6d0\uc740 \u201c\uc138\uc6d4\ud638 \uce68\ubab0 \ub2f9\uc77c \ub300\ud55c\ubbfc\uad6d \uad6d\uad70 \ud2b9\uc218\ubd80\ub300 \uc694\uc6d0\ub4e4\uc740 \ub208\ub728\uace0 \ucc38\uc0ac\ub97c \uc9c0\ucf1c\ubcf4\uba74\uc11c \ud669\ub9dd\ud55c \uae30\ubd84\uc744 \ub290\uaf08\uc744 \uac83\u201d\uc774\ub77c\uba70 \u201c\ub2f9\uc2dc \ud604\uc7a5\uc744 \ud1b5\uc81c\ud588\ub358 \uc9c0\ud718\uad00\uc740 \ubc18\ub4dc\uc2dc \ubb38\ucc45\ub418\uc5b4\uc57c \ud55c\ub2e4\u201d\uace0 \ub9d0\ud588\ub2e4.", "paragraph_answer": "\ucd5c\uace0\uc758 \uc218\uc911 \ud0d0\uc0c9\u00b7\uad6c\uc870 \ub2a5\ub825\uc744 \uac16\ucd98 \uacf5\uad70 \ud2b9\uc218\ubd80\ub300 \uc694\uc6d0\ub4e4\uc774 \uc138\uc6d4\ud638 \uce68\ubab0\uc0ac\uace0 \ub2f9\uc77c \ud604\uc7a5\uc5d0 \ucd9c\ub3d9\ud588\uc9c0\ub9cc \ud604\uc7a5 \uc9c0\ud718\ud1b5\uc81c \uae30\ub2a5 \ub9c8\ube44\ub85c \uc778\uba85\uad6c\uc870 \uc791\uc5c5\uc5d0 \ub2e8 1\uba85\ub3c4 \ud22c\uc785\ub418\uc9c0 \ubabb\ud55c \uac83\uc73c\ub85c 6\uc6d4 9\uc77c \ud655\uc778\ub410\ub2e4. \uc694\uc6d0\ub4e4\uc740 \uc2b9\uac1d\ub4e4\uc758 \uc0dd\uc874 \uac00\ub2a5\uc131\uc774 \ub192\uc740 \uace8\ub4e0\ud0c0\uc784 \ub0b4\ub0b4 \ud5ec\uae30 \uc548\uc5d0\uc11c \uc2e4\uc885\uc790\ub4e4\uc5d0 \ub300\ud55c \uc721\uc548\uc218\uc0c9\ub9cc \ubc8c\uc600\ub2e4. \uacf5\uad70 \ud56d\uacf5\uad6c\uc870\ub300 \ub294 \ud3c9\uc2dc \ud56d\uacf5\uae30 \uc0ac\uace0\uad6c\uc870, \uc751\uae09\ucc98\uce58, \ud658\uc790 \uc774\uc1a1 \ub4f1\uc758 \uc784\ubb34\ub97c \ub9e1\ub294\ub2e4. \uc804\uc2dc\uc5d0\ub294 \uc801\uc9c4\uc5d0 \ud22c\uc785\ub3fc \uac15\uc774\ub098 \ubc14\ub2e4\uc5d0 \ub5a8\uc5b4\uc9c4 \uc870\uc885\uc0ac\ub97c \uad6c\ucd9c\ud558\ub294 \uc784\ubb34\ub97c \uc218\ud589\ud55c\ub2e4. \uc694\uc6d0\ub4e4\uc740 \uacf5\uc911\uce68\ud22c\ubfd0 \uc544\ub2c8\ub77c \ud574\uc0c1\uad6c\uc870\uc640 \uc751\uae09\uc758\ub8cc \ub4f1 \ud2b9\uc218\uc791\uc804\uc5d0 \ud544\uc694\ud55c \ubaa8\ub4e0 \ub2a5\ub825\uc744 \uac16\ucd94\uace0 \uc788\ub2e4. \uae40\uad11\uc9c4 \uc758\uc6d0\uc740 \u201c\uc138\uc6d4\ud638 \uce68\ubab0 \ub2f9\uc77c \ub300\ud55c\ubbfc\uad6d \uad6d\uad70 \ud2b9\uc218\ubd80\ub300 \uc694\uc6d0\ub4e4\uc740 \ub208\ub728\uace0 \ucc38\uc0ac\ub97c \uc9c0\ucf1c\ubcf4\uba74\uc11c \ud669\ub9dd\ud55c \uae30\ubd84\uc744 \ub290\uaf08\uc744 \uac83\u201d\uc774\ub77c\uba70 \u201c\ub2f9\uc2dc \ud604\uc7a5\uc744 \ud1b5\uc81c\ud588\ub358 \uc9c0\ud718\uad00\uc740 \ubc18\ub4dc\uc2dc \ubb38\ucc45\ub418\uc5b4\uc57c \ud55c\ub2e4\u201d\uace0 \ub9d0\ud588\ub2e4.", "sentence_answer": " \uacf5\uad70 \ud56d\uacf5\uad6c\uc870\ub300 \ub294 \ud3c9\uc2dc \ud56d\uacf5\uae30 \uc0ac\uace0\uad6c\uc870, \uc751\uae09\ucc98\uce58, \ud658\uc790 \uc774\uc1a1 \ub4f1\uc758 \uc784\ubb34\ub97c \ub9e1\ub294\ub2e4.", "paragraph_id": "6536361-11-0", "paragraph_question": "question: \ud56d\uacf5\uae30 \uc0ac\uace0 \uad6c\uc870, \uc751\uae09\ucc98\uce58, \ud658\uc790 \uc774\uc1a1\uc744 \ub9e1\uace0 \uc788\ub294 \ubd80\ub300\ub294?, context: \ucd5c\uace0\uc758 \uc218\uc911 \ud0d0\uc0c9\u00b7\uad6c\uc870 \ub2a5\ub825\uc744 \uac16\ucd98 \uacf5\uad70 \ud2b9\uc218\ubd80\ub300 \uc694\uc6d0\ub4e4\uc774 \uc138\uc6d4\ud638 \uce68\ubab0\uc0ac\uace0 \ub2f9\uc77c \ud604\uc7a5\uc5d0 \ucd9c\ub3d9\ud588\uc9c0\ub9cc \ud604\uc7a5 \uc9c0\ud718\ud1b5\uc81c \uae30\ub2a5 \ub9c8\ube44\ub85c \uc778\uba85\uad6c\uc870 \uc791\uc5c5\uc5d0 \ub2e8 1\uba85\ub3c4 \ud22c\uc785\ub418\uc9c0 \ubabb\ud55c \uac83\uc73c\ub85c 6\uc6d4 9\uc77c \ud655\uc778\ub410\ub2e4. \uc694\uc6d0\ub4e4\uc740 \uc2b9\uac1d\ub4e4\uc758 \uc0dd\uc874 \uac00\ub2a5\uc131\uc774 \ub192\uc740 \uace8\ub4e0\ud0c0\uc784 \ub0b4\ub0b4 \ud5ec\uae30 \uc548\uc5d0\uc11c \uc2e4\uc885\uc790\ub4e4\uc5d0 \ub300\ud55c \uc721\uc548\uc218\uc0c9\ub9cc \ubc8c\uc600\ub2e4. \uacf5\uad70 \ud56d\uacf5\uad6c\uc870\ub300\ub294 \ud3c9\uc2dc \ud56d\uacf5\uae30 \uc0ac\uace0\uad6c\uc870, \uc751\uae09\ucc98\uce58, \ud658\uc790 \uc774\uc1a1 \ub4f1\uc758 \uc784\ubb34\ub97c \ub9e1\ub294\ub2e4. \uc804\uc2dc\uc5d0\ub294 \uc801\uc9c4\uc5d0 \ud22c\uc785\ub3fc \uac15\uc774\ub098 \ubc14\ub2e4\uc5d0 \ub5a8\uc5b4\uc9c4 \uc870\uc885\uc0ac\ub97c \uad6c\ucd9c\ud558\ub294 \uc784\ubb34\ub97c \uc218\ud589\ud55c\ub2e4. \uc694\uc6d0\ub4e4\uc740 \uacf5\uc911\uce68\ud22c\ubfd0 \uc544\ub2c8\ub77c \ud574\uc0c1\uad6c\uc870\uc640 \uc751\uae09\uc758\ub8cc \ub4f1 \ud2b9\uc218\uc791\uc804\uc5d0 \ud544\uc694\ud55c \ubaa8\ub4e0 \ub2a5\ub825\uc744 \uac16\ucd94\uace0 \uc788\ub2e4. \uae40\uad11\uc9c4 \uc758\uc6d0\uc740 \u201c\uc138\uc6d4\ud638 \uce68\ubab0 \ub2f9\uc77c \ub300\ud55c\ubbfc\uad6d \uad6d\uad70 \ud2b9\uc218\ubd80\ub300 \uc694\uc6d0\ub4e4\uc740 \ub208\ub728\uace0 \ucc38\uc0ac\ub97c \uc9c0\ucf1c\ubcf4\uba74\uc11c \ud669\ub9dd\ud55c \uae30\ubd84\uc744 \ub290\uaf08\uc744 \uac83\u201d\uc774\ub77c\uba70 \u201c\ub2f9\uc2dc \ud604\uc7a5\uc744 \ud1b5\uc81c\ud588\ub358 \uc9c0\ud718\uad00\uc740 \ubc18\ub4dc\uc2dc \ubb38\ucc45\ub418\uc5b4\uc57c \ud55c\ub2e4\u201d\uace0 \ub9d0\ud588\ub2e4."} {"question": "\uc548\ucca0\uc218\ub294 \ubc15\uc6d0\uc21c\ubcf4\ub2e4 \uba87 \ubc30\uc758 \uc9c0\uc9c0\ub3c4\ub97c \uac00\uc9c0\uace0 \uc788\uc5c8\ub294\uac00?", "paragraph": "\ubc15\uc6d0\uc21c\uc740 \ub2e8\uc77c\ud654\uc5d0 \ub300\ud574 \u201c\ub450 \uc0ac\ub78c \ubaa8\ub450 \uc2dc\uc7a5\uc9c1 \uc790\ub9ac\ub97c \uc6d0\ud55c \uac8c \uc544\ub2c8\ub2e4. \uc9c4\uc815 \uc0c8\ub85c\uc6b4 \uc138\uc0c1\uc744 \ub9cc\ub4dc\ub294 \ub370 \uad00\uc2ec\uc774 \uc788\uc5c8\uae30 \ub54c\ubb38\uc5d0 \uc774\ub807\uac8c \uc0c1\uc2dd\uc801\uc73c\ub85c \uc774\ud574 \uc548 \ub418\ub294 \uacb0\ub860\uc774 \ub098\uc628 \uac83\u201d\ub77c\uace0 \ub9d0\ud588\ub2e4. \ubc15\uc6d0\uc21c\uc740 \ub610 \uc548\ucca0\uc218\uc5d0 \ub300\ud574 \u201c\uc544\ubb34\ub9ac \uc2e0\ub8b0\uad00\uacc4\uac00 \uc788\ub2e4\ud574\ub3c4 \uc800\ubcf4\ub2e4 10\ubc30\ub098 \ub354 \ub418\ub294 \uc9c0\uc9c0\ub3c4\ub97c \uac16\uace0 \uc788\ub358 \ubd84\uc774 \uc815\ub9d0 \uc544\ubb34 \uc870\uac74 \uc5c6\uc774 \u2018\ub354 \uc798 \ud560 \uc218 \uc788\ub2e4\u2019\uace0 \ud558\ub294 (\ub0b4 \ub9d0) \ud55c\ub9c8\ub514\ub85c \uc591\ubcf4\ud55c\ub2e4\ub294 \uac8c \uc0ac\uc2e4 \ub610 \ubbff\uae30 \uc5b4\ub824\uc6b4 \uadf8\ub7f0 \uc77c\u201d\uc774\ub77c\uba70 \u201c\uc548 \uad50\uc218\uac00 \uac1c\uc778\uc758 \uc774\uc775\ubcf4\ub2e4 \uc0ac\ud68c\uc758 \uc5b4\ub5a4 \uacf5\uacf5\uc801\uc778 \uc774\uc775\uc744 \uc704\ud574\uc11c \ud574\uc654\ub358 \ubd84\uc774\uc5c8\uae30 \ub54c\ubb38\uc5d0 \uac00\ub2a5\ud588\ub358 \ud0dc\ub3c4\uc600\ub2e4\uace0 \ubcf8\ub2e4\u201d\ub77c\uace0 \ub9d0\ud588\ub2e4. \uc774\ud6c4 \ubc15\uc6d0\uc21c, \ud55c\uba85\uc219, \ubb38\uc7ac\uc778 \ub4f1\uc740 \u201c\uc120\uac70 \uc2b9\ub9ac\ub97c \uc704\ud574\uc11c\ub294 \ubc94\uc2dc\ubbfc \uc57c\uad8c \ub2e8\uc77c\ud6c4\ubcf4\ub97c \ud1b5\ud574 \ud55c\ub098\ub77c\ub2f9\uacfc 1:1 \uad6c\ub3c4\ub97c \ub9cc\ub4e4\uc5b4\uc57c \ud55c\ub2e4. (\uc11c\uc6b8\uc2dc\uc7a5 \ud6c4\ubcf4\ub85c \uac70\ub860\ub418\ub294)\ubc15\uc6d0\uc21c-\ud55c\uba85\uc219 \ub450 \uc0ac\ub78c\uc740 \ubc94\uc2dc\ubbfc \uc57c\uad8c \ub2e8\uc77c \ud6c4\ubcf4 \uc120\ucd9c\uc744 \uc704\ud574 \uc0c1\ud638 \ud611\ub825\ud558\uace0, \uc774\ud6c4\uc5d4 \uc120\uac70 \uc2b9\ub9ac\ub97c \uc704\ud574 \ubaa8\ub4e0 \ud798\uc744 \uae30\uc6b8\uc778\ub2e4\u201d\ub77c\uba70 \uacb0\uc758\ub97c \ub2e4\uc84c\ub2e4.", "answer": "10\ubc30", "sentence": "\ubc15\uc6d0\uc21c\uc740 \ub610 \uc548\ucca0\uc218\uc5d0 \ub300\ud574 \u201c\uc544\ubb34\ub9ac \uc2e0\ub8b0\uad00\uacc4\uac00 \uc788\ub2e4\ud574\ub3c4 \uc800\ubcf4\ub2e4 10\ubc30 \ub098 \ub354 \ub418\ub294 \uc9c0\uc9c0\ub3c4\ub97c \uac16\uace0 \uc788\ub358 \ubd84\uc774 \uc815\ub9d0 \uc544\ubb34 \uc870\uac74 \uc5c6\uc774 \u2018\ub354 \uc798 \ud560 \uc218 \uc788\ub2e4\u2019\uace0 \ud558\ub294 (\ub0b4 \ub9d0) \ud55c\ub9c8\ub514\ub85c \uc591\ubcf4\ud55c\ub2e4\ub294 \uac8c \uc0ac\uc2e4 \ub610 \ubbff\uae30 \uc5b4\ub824\uc6b4 \uadf8\ub7f0 \uc77c\u201d\uc774\ub77c\uba70 \u201c\uc548 \uad50\uc218\uac00 \uac1c\uc778\uc758 \uc774\uc775\ubcf4\ub2e4 \uc0ac\ud68c\uc758 \uc5b4\ub5a4 \uacf5\uacf5\uc801\uc778 \uc774\uc775\uc744 \uc704\ud574\uc11c \ud574\uc654\ub358 \ubd84\uc774\uc5c8\uae30 \ub54c\ubb38\uc5d0 \uac00\ub2a5\ud588\ub358 \ud0dc\ub3c4\uc600\ub2e4\uace0 \ubcf8\ub2e4\u201d\ub77c\uace0 \ub9d0\ud588\ub2e4.", "paragraph_sentence": "\ubc15\uc6d0\uc21c\uc740 \ub2e8\uc77c\ud654\uc5d0 \ub300\ud574 \u201c\ub450 \uc0ac\ub78c \ubaa8\ub450 \uc2dc\uc7a5\uc9c1 \uc790\ub9ac\ub97c \uc6d0\ud55c \uac8c \uc544\ub2c8\ub2e4. \uc9c4\uc815 \uc0c8\ub85c\uc6b4 \uc138\uc0c1\uc744 \ub9cc\ub4dc\ub294 \ub370 \uad00\uc2ec\uc774 \uc788\uc5c8\uae30 \ub54c\ubb38\uc5d0 \uc774\ub807\uac8c \uc0c1\uc2dd\uc801\uc73c\ub85c \uc774\ud574 \uc548 \ub418\ub294 \uacb0\ub860\uc774 \ub098\uc628 \uac83\u201d\ub77c\uace0 \ub9d0\ud588\ub2e4. \ubc15\uc6d0\uc21c\uc740 \ub610 \uc548\ucca0\uc218\uc5d0 \ub300\ud574 \u201c\uc544\ubb34\ub9ac \uc2e0\ub8b0\uad00\uacc4\uac00 \uc788\ub2e4\ud574\ub3c4 \uc800\ubcf4\ub2e4 10\ubc30 \ub098 \ub354 \ub418\ub294 \uc9c0\uc9c0\ub3c4\ub97c \uac16\uace0 \uc788\ub358 \ubd84\uc774 \uc815\ub9d0 \uc544\ubb34 \uc870\uac74 \uc5c6\uc774 \u2018\ub354 \uc798 \ud560 \uc218 \uc788\ub2e4\u2019\uace0 \ud558\ub294 (\ub0b4 \ub9d0) \ud55c\ub9c8\ub514\ub85c \uc591\ubcf4\ud55c\ub2e4\ub294 \uac8c \uc0ac\uc2e4 \ub610 \ubbff\uae30 \uc5b4\ub824\uc6b4 \uadf8\ub7f0 \uc77c\u201d\uc774\ub77c\uba70 \u201c\uc548 \uad50\uc218\uac00 \uac1c\uc778\uc758 \uc774\uc775\ubcf4\ub2e4 \uc0ac\ud68c\uc758 \uc5b4\ub5a4 \uacf5\uacf5\uc801\uc778 \uc774\uc775\uc744 \uc704\ud574\uc11c \ud574\uc654\ub358 \ubd84\uc774\uc5c8\uae30 \ub54c\ubb38\uc5d0 \uac00\ub2a5\ud588\ub358 \ud0dc\ub3c4\uc600\ub2e4\uace0 \ubcf8\ub2e4\u201d\ub77c\uace0 \ub9d0\ud588\ub2e4. \uc774\ud6c4 \ubc15\uc6d0\uc21c, \ud55c\uba85\uc219, \ubb38\uc7ac\uc778 \ub4f1\uc740 \u201c\uc120\uac70 \uc2b9\ub9ac\ub97c \uc704\ud574\uc11c\ub294 \ubc94\uc2dc\ubbfc \uc57c\uad8c \ub2e8\uc77c\ud6c4\ubcf4\ub97c \ud1b5\ud574 \ud55c\ub098\ub77c\ub2f9\uacfc 1:1 \uad6c\ub3c4\ub97c \ub9cc\ub4e4\uc5b4\uc57c \ud55c\ub2e4. (\uc11c\uc6b8\uc2dc\uc7a5 \ud6c4\ubcf4\ub85c \uac70\ub860\ub418\ub294)\ubc15\uc6d0\uc21c-\ud55c\uba85\uc219 \ub450 \uc0ac\ub78c\uc740 \ubc94\uc2dc\ubbfc \uc57c\uad8c \ub2e8\uc77c \ud6c4\ubcf4 \uc120\ucd9c\uc744 \uc704\ud574 \uc0c1\ud638 \ud611\ub825\ud558\uace0, \uc774\ud6c4\uc5d4 \uc120\uac70 \uc2b9\ub9ac\ub97c \uc704\ud574 \ubaa8\ub4e0 \ud798\uc744 \uae30\uc6b8\uc778\ub2e4\u201d\ub77c\uba70 \uacb0\uc758\ub97c \ub2e4\uc84c\ub2e4.", "paragraph_answer": "\ubc15\uc6d0\uc21c\uc740 \ub2e8\uc77c\ud654\uc5d0 \ub300\ud574 \u201c\ub450 \uc0ac\ub78c \ubaa8\ub450 \uc2dc\uc7a5\uc9c1 \uc790\ub9ac\ub97c \uc6d0\ud55c \uac8c \uc544\ub2c8\ub2e4. \uc9c4\uc815 \uc0c8\ub85c\uc6b4 \uc138\uc0c1\uc744 \ub9cc\ub4dc\ub294 \ub370 \uad00\uc2ec\uc774 \uc788\uc5c8\uae30 \ub54c\ubb38\uc5d0 \uc774\ub807\uac8c \uc0c1\uc2dd\uc801\uc73c\ub85c \uc774\ud574 \uc548 \ub418\ub294 \uacb0\ub860\uc774 \ub098\uc628 \uac83\u201d\ub77c\uace0 \ub9d0\ud588\ub2e4. \ubc15\uc6d0\uc21c\uc740 \ub610 \uc548\ucca0\uc218\uc5d0 \ub300\ud574 \u201c\uc544\ubb34\ub9ac \uc2e0\ub8b0\uad00\uacc4\uac00 \uc788\ub2e4\ud574\ub3c4 \uc800\ubcf4\ub2e4 10\ubc30 \ub098 \ub354 \ub418\ub294 \uc9c0\uc9c0\ub3c4\ub97c \uac16\uace0 \uc788\ub358 \ubd84\uc774 \uc815\ub9d0 \uc544\ubb34 \uc870\uac74 \uc5c6\uc774 \u2018\ub354 \uc798 \ud560 \uc218 \uc788\ub2e4\u2019\uace0 \ud558\ub294 (\ub0b4 \ub9d0) \ud55c\ub9c8\ub514\ub85c \uc591\ubcf4\ud55c\ub2e4\ub294 \uac8c \uc0ac\uc2e4 \ub610 \ubbff\uae30 \uc5b4\ub824\uc6b4 \uadf8\ub7f0 \uc77c\u201d\uc774\ub77c\uba70 \u201c\uc548 \uad50\uc218\uac00 \uac1c\uc778\uc758 \uc774\uc775\ubcf4\ub2e4 \uc0ac\ud68c\uc758 \uc5b4\ub5a4 \uacf5\uacf5\uc801\uc778 \uc774\uc775\uc744 \uc704\ud574\uc11c \ud574\uc654\ub358 \ubd84\uc774\uc5c8\uae30 \ub54c\ubb38\uc5d0 \uac00\ub2a5\ud588\ub358 \ud0dc\ub3c4\uc600\ub2e4\uace0 \ubcf8\ub2e4\u201d\ub77c\uace0 \ub9d0\ud588\ub2e4. \uc774\ud6c4 \ubc15\uc6d0\uc21c, \ud55c\uba85\uc219, \ubb38\uc7ac\uc778 \ub4f1\uc740 \u201c\uc120\uac70 \uc2b9\ub9ac\ub97c \uc704\ud574\uc11c\ub294 \ubc94\uc2dc\ubbfc \uc57c\uad8c \ub2e8\uc77c\ud6c4\ubcf4\ub97c \ud1b5\ud574 \ud55c\ub098\ub77c\ub2f9\uacfc 1:1 \uad6c\ub3c4\ub97c \ub9cc\ub4e4\uc5b4\uc57c \ud55c\ub2e4. (\uc11c\uc6b8\uc2dc\uc7a5 \ud6c4\ubcf4\ub85c \uac70\ub860\ub418\ub294)\ubc15\uc6d0\uc21c-\ud55c\uba85\uc219 \ub450 \uc0ac\ub78c\uc740 \ubc94\uc2dc\ubbfc \uc57c\uad8c \ub2e8\uc77c \ud6c4\ubcf4 \uc120\ucd9c\uc744 \uc704\ud574 \uc0c1\ud638 \ud611\ub825\ud558\uace0, \uc774\ud6c4\uc5d4 \uc120\uac70 \uc2b9\ub9ac\ub97c \uc704\ud574 \ubaa8\ub4e0 \ud798\uc744 \uae30\uc6b8\uc778\ub2e4\u201d\ub77c\uba70 \uacb0\uc758\ub97c \ub2e4\uc84c\ub2e4.", "sentence_answer": "\ubc15\uc6d0\uc21c\uc740 \ub610 \uc548\ucca0\uc218\uc5d0 \ub300\ud574 \u201c\uc544\ubb34\ub9ac \uc2e0\ub8b0\uad00\uacc4\uac00 \uc788\ub2e4\ud574\ub3c4 \uc800\ubcf4\ub2e4 10\ubc30 \ub098 \ub354 \ub418\ub294 \uc9c0\uc9c0\ub3c4\ub97c \uac16\uace0 \uc788\ub358 \ubd84\uc774 \uc815\ub9d0 \uc544\ubb34 \uc870\uac74 \uc5c6\uc774 \u2018\ub354 \uc798 \ud560 \uc218 \uc788\ub2e4\u2019\uace0 \ud558\ub294 (\ub0b4 \ub9d0) \ud55c\ub9c8\ub514\ub85c \uc591\ubcf4\ud55c\ub2e4\ub294 \uac8c \uc0ac\uc2e4 \ub610 \ubbff\uae30 \uc5b4\ub824\uc6b4 \uadf8\ub7f0 \uc77c\u201d\uc774\ub77c\uba70 \u201c\uc548 \uad50\uc218\uac00 \uac1c\uc778\uc758 \uc774\uc775\ubcf4\ub2e4 \uc0ac\ud68c\uc758 \uc5b4\ub5a4 \uacf5\uacf5\uc801\uc778 \uc774\uc775\uc744 \uc704\ud574\uc11c \ud574\uc654\ub358 \ubd84\uc774\uc5c8\uae30 \ub54c\ubb38\uc5d0 \uac00\ub2a5\ud588\ub358 \ud0dc\ub3c4\uc600\ub2e4\uace0 \ubcf8\ub2e4\u201d\ub77c\uace0 \ub9d0\ud588\ub2e4.", "paragraph_id": "6547507-10-1", "paragraph_question": "question: \uc548\ucca0\uc218\ub294 \ubc15\uc6d0\uc21c\ubcf4\ub2e4 \uba87 \ubc30\uc758 \uc9c0\uc9c0\ub3c4\ub97c \uac00\uc9c0\uace0 \uc788\uc5c8\ub294\uac00?, context: \ubc15\uc6d0\uc21c\uc740 \ub2e8\uc77c\ud654\uc5d0 \ub300\ud574 \u201c\ub450 \uc0ac\ub78c \ubaa8\ub450 \uc2dc\uc7a5\uc9c1 \uc790\ub9ac\ub97c \uc6d0\ud55c \uac8c \uc544\ub2c8\ub2e4. \uc9c4\uc815 \uc0c8\ub85c\uc6b4 \uc138\uc0c1\uc744 \ub9cc\ub4dc\ub294 \ub370 \uad00\uc2ec\uc774 \uc788\uc5c8\uae30 \ub54c\ubb38\uc5d0 \uc774\ub807\uac8c \uc0c1\uc2dd\uc801\uc73c\ub85c \uc774\ud574 \uc548 \ub418\ub294 \uacb0\ub860\uc774 \ub098\uc628 \uac83\u201d\ub77c\uace0 \ub9d0\ud588\ub2e4. \ubc15\uc6d0\uc21c\uc740 \ub610 \uc548\ucca0\uc218\uc5d0 \ub300\ud574 \u201c\uc544\ubb34\ub9ac \uc2e0\ub8b0\uad00\uacc4\uac00 \uc788\ub2e4\ud574\ub3c4 \uc800\ubcf4\ub2e4 10\ubc30\ub098 \ub354 \ub418\ub294 \uc9c0\uc9c0\ub3c4\ub97c \uac16\uace0 \uc788\ub358 \ubd84\uc774 \uc815\ub9d0 \uc544\ubb34 \uc870\uac74 \uc5c6\uc774 \u2018\ub354 \uc798 \ud560 \uc218 \uc788\ub2e4\u2019\uace0 \ud558\ub294 (\ub0b4 \ub9d0) \ud55c\ub9c8\ub514\ub85c \uc591\ubcf4\ud55c\ub2e4\ub294 \uac8c \uc0ac\uc2e4 \ub610 \ubbff\uae30 \uc5b4\ub824\uc6b4 \uadf8\ub7f0 \uc77c\u201d\uc774\ub77c\uba70 \u201c\uc548 \uad50\uc218\uac00 \uac1c\uc778\uc758 \uc774\uc775\ubcf4\ub2e4 \uc0ac\ud68c\uc758 \uc5b4\ub5a4 \uacf5\uacf5\uc801\uc778 \uc774\uc775\uc744 \uc704\ud574\uc11c \ud574\uc654\ub358 \ubd84\uc774\uc5c8\uae30 \ub54c\ubb38\uc5d0 \uac00\ub2a5\ud588\ub358 \ud0dc\ub3c4\uc600\ub2e4\uace0 \ubcf8\ub2e4\u201d\ub77c\uace0 \ub9d0\ud588\ub2e4. \uc774\ud6c4 \ubc15\uc6d0\uc21c, \ud55c\uba85\uc219, \ubb38\uc7ac\uc778 \ub4f1\uc740 \u201c\uc120\uac70 \uc2b9\ub9ac\ub97c \uc704\ud574\uc11c\ub294 \ubc94\uc2dc\ubbfc \uc57c\uad8c \ub2e8\uc77c\ud6c4\ubcf4\ub97c \ud1b5\ud574 \ud55c\ub098\ub77c\ub2f9\uacfc 1:1 \uad6c\ub3c4\ub97c \ub9cc\ub4e4\uc5b4\uc57c \ud55c\ub2e4. (\uc11c\uc6b8\uc2dc\uc7a5 \ud6c4\ubcf4\ub85c \uac70\ub860\ub418\ub294)\ubc15\uc6d0\uc21c-\ud55c\uba85\uc219 \ub450 \uc0ac\ub78c\uc740 \ubc94\uc2dc\ubbfc \uc57c\uad8c \ub2e8\uc77c \ud6c4\ubcf4 \uc120\ucd9c\uc744 \uc704\ud574 \uc0c1\ud638 \ud611\ub825\ud558\uace0, \uc774\ud6c4\uc5d4 \uc120\uac70 \uc2b9\ub9ac\ub97c \uc704\ud574 \ubaa8\ub4e0 \ud798\uc744 \uae30\uc6b8\uc778\ub2e4\u201d\ub77c\uba70 \uacb0\uc758\ub97c \ub2e4\uc84c\ub2e4."} {"question": "\ud544\ub85c\ud3ec\ub2c8\uc544\uac00 \uc544\uc774\ud1a8\ub9ac\uc544,\uc5d8\ub9ac\uc2a4 \uc5f0\ud569\uad70\uacfc \uc2f8\uc6cc \uc2b9\ub9ac\ud55c \uc804\ud22c\ub294 \uc5b4\ub5a4 \uc804\uc7c1\uacfc \uc5f0\uad00\ub41c \uc804\ud22c\uc778\uac00?", "paragraph": "\uae30\uc6d0\uc804 221\ub144, \ud544\ub85c\ud3ec\uc774\uba58\uc740 \uc804\uc7c1 \uc911\uc778 \ud06c\ub808\ud0c0\uc5d0\uc11c \uc6a9\ubcd1 \ub300\uc7a5\uc73c\ub85c \uc2f8\uc6e0\ub2e4. 10\ub144 \ud6c4 \uadc0\uad6d\uc5d0 \uc988\uc74c\ud558\uc5ec \uadf8\ub294 \uc544\uce74\uc774\uc544 \ub3d9\ub9f9\uc758 \uae30\ubcd1\ub300 \uc9c0\ud718\uad00\uc73c\ub85c \uc784\uba85\ub418\uc5c8\ub2e4. \ub2f9\uc2dc \uc544\uce74\uc774\uc544\uc758 \uae30\ubcd1\ub4e4\uc740 \uc57d\ud588\uae30 \ub54c\ubb38\uc5d0 \uadf8\ub294 \uae30\ubcd1\uc744 \uc6a9\ub9f9\ud558\uac8c \ub2e8\ub828\ud588\ub2e4. \uac19\uc740 \ud574\uc5d0 \uc81c1\ucc28 \ub9c8\ucf00\ub3c4\ub2c8\uc544 \uc804\uc7c1\uacfc \uc5f0\uad00\ub41c \uc804\ud22c\uc5d0\uc11c \ud544\ub85c\ud3ec\uc774\uba58\uc774 \uc774\ub044\ub294 \uc544\uce74\uc774\uc544 \uad70\uc740 \uc5d8\ub9ac\uc2a4\uc758 \uacbd\uacc4\uc5d0 \uc788\ub294 \ub77c\ub9ac\uc0ac \uac15\uc5d0\uc11c \uc544\uc774\ud1a8\ub9ac\uc544, \uc5d8\ub9ac\uc2a4 \uc5f0\ud569\uad70\uacfc \uc2f8\uc6cc \ubb3c\ub9ac\uce58\uace0, \uc801\uc7a5 \ub2e4\ubaa8\ud310\ud22c\uc2a4\uc640 \uc9c1\uc811 \ub9de\uc11c\uc11c \ubb3c\ub9ac\uce58\uace0 \uc2b9\ub9ac\ub97c \uac70\ub450\uc5c8\ub2e4. \uc774 \uc2b9\ub9ac\ub85c \ud544\ub85c\ud3ec\uc774\uba58\uc758 \uba85\uc131\uc740 \uadf8\ub9ac\uc2a4\uc5d0 \ub110\ub9ac \ud37c\uc84c\uc73c\uba70, \ub610\ud55c \uadf8\ub294 \uc544\uce74\uc774\uc544 \ub3d9\ub9f9\uc758 \ubcf4\ubcd1 \uc7a5\ube44\ub97c \uac1c\ud601\ud588\ub2e4. \uadf8\ub54c\uae4c\uc9c0 \uc544\uce74\uc774\uc544 \uad70\uc758 \uc7a5\ube44\ub294 \uc9e7\uc740 \ucc3d\uacfc \uac00\ubccd\uac8c \uc587\uc740 \ud0c0\uc6d0\ud615 \ubc29\ud328\uc600\uc73c\uba70, \uc774\uac83\uc740 \uc6d0\uac70\ub9ac \uc804\ud22c\uc5d0\uc11c\ub294 \uc720\ub9ac\ud558\uc9c0\ub9cc, \ubc31\ubcd1\uc804\uc5d0\uc11c\ub294 \ubd88\ub9ac\ud558\uc5ec \uc27d\uac8c \uc804\uc5f4\uc774 \ub3cc\ud30c\ub418\uace0 \uc788\uc5c8\ub2e4. \uadf8\ub294 \ud314\ub791\ud06c\uc2a4\uc758 \ubc00\uc9d1\ub300\ud615\uc744 \ud615\uc131\ud560 \ud615\ud3b8\uc774 \uc88b\uc740 \uc7a5\ucc3d\uacfc \ub465\uadfc\ubc29\ud328\ub85c \ubc14\uafb8\uc5c8\ub2e4.", "answer": "\uc81c1\ucc28 \ub9c8\ucf00\ub3c4\ub2c8\uc544 \uc804\uc7c1", "sentence": "\uac19\uc740 \ud574\uc5d0 \uc81c1\ucc28 \ub9c8\ucf00\ub3c4\ub2c8\uc544 \uc804\uc7c1 \uacfc \uc5f0\uad00\ub41c \uc804\ud22c\uc5d0\uc11c \ud544\ub85c\ud3ec\uc774\uba58\uc774 \uc774\ub044\ub294 \uc544\uce74\uc774\uc544 \uad70\uc740 \uc5d8\ub9ac\uc2a4\uc758 \uacbd\uacc4\uc5d0 \uc788\ub294 \ub77c\ub9ac\uc0ac \uac15\uc5d0\uc11c \uc544\uc774\ud1a8\ub9ac\uc544, \uc5d8\ub9ac\uc2a4 \uc5f0\ud569\uad70\uacfc \uc2f8\uc6cc \ubb3c\ub9ac\uce58\uace0, \uc801\uc7a5 \ub2e4\ubaa8\ud310\ud22c\uc2a4\uc640 \uc9c1\uc811 \ub9de\uc11c\uc11c \ubb3c\ub9ac\uce58\uace0 \uc2b9\ub9ac\ub97c \uac70\ub450\uc5c8\ub2e4.", "paragraph_sentence": "\uae30\uc6d0\uc804 221\ub144, \ud544\ub85c\ud3ec\uc774\uba58\uc740 \uc804\uc7c1 \uc911\uc778 \ud06c\ub808\ud0c0\uc5d0\uc11c \uc6a9\ubcd1 \ub300\uc7a5\uc73c\ub85c \uc2f8\uc6e0\ub2e4. 10\ub144 \ud6c4 \uadc0\uad6d\uc5d0 \uc988\uc74c\ud558\uc5ec \uadf8\ub294 \uc544\uce74\uc774\uc544 \ub3d9\ub9f9\uc758 \uae30\ubcd1\ub300 \uc9c0\ud718\uad00\uc73c\ub85c \uc784\uba85\ub418\uc5c8\ub2e4. \ub2f9\uc2dc \uc544\uce74\uc774\uc544\uc758 \uae30\ubcd1\ub4e4\uc740 \uc57d\ud588\uae30 \ub54c\ubb38\uc5d0 \uadf8\ub294 \uae30\ubcd1\uc744 \uc6a9\ub9f9\ud558\uac8c \ub2e8\ub828\ud588\ub2e4. \uac19\uc740 \ud574\uc5d0 \uc81c1\ucc28 \ub9c8\ucf00\ub3c4\ub2c8\uc544 \uc804\uc7c1 \uacfc \uc5f0\uad00\ub41c \uc804\ud22c\uc5d0\uc11c \ud544\ub85c\ud3ec\uc774\uba58\uc774 \uc774\ub044\ub294 \uc544\uce74\uc774\uc544 \uad70\uc740 \uc5d8\ub9ac\uc2a4\uc758 \uacbd\uacc4\uc5d0 \uc788\ub294 \ub77c\ub9ac\uc0ac \uac15\uc5d0\uc11c \uc544\uc774\ud1a8\ub9ac\uc544, \uc5d8\ub9ac\uc2a4 \uc5f0\ud569\uad70\uacfc \uc2f8\uc6cc \ubb3c\ub9ac\uce58\uace0, \uc801\uc7a5 \ub2e4\ubaa8\ud310\ud22c\uc2a4\uc640 \uc9c1\uc811 \ub9de\uc11c\uc11c \ubb3c\ub9ac\uce58\uace0 \uc2b9\ub9ac\ub97c \uac70\ub450\uc5c8\ub2e4. \uc774 \uc2b9\ub9ac\ub85c \ud544\ub85c\ud3ec\uc774\uba58\uc758 \uba85\uc131\uc740 \uadf8\ub9ac\uc2a4\uc5d0 \ub110\ub9ac \ud37c\uc84c\uc73c\uba70, \ub610\ud55c \uadf8\ub294 \uc544\uce74\uc774\uc544 \ub3d9\ub9f9\uc758 \ubcf4\ubcd1 \uc7a5\ube44\ub97c \uac1c\ud601\ud588\ub2e4. \uadf8\ub54c\uae4c\uc9c0 \uc544\uce74\uc774\uc544 \uad70\uc758 \uc7a5\ube44\ub294 \uc9e7\uc740 \ucc3d\uacfc \uac00\ubccd\uac8c \uc587\uc740 \ud0c0\uc6d0\ud615 \ubc29\ud328\uc600\uc73c\uba70, \uc774\uac83\uc740 \uc6d0\uac70\ub9ac \uc804\ud22c\uc5d0\uc11c\ub294 \uc720\ub9ac\ud558\uc9c0\ub9cc, \ubc31\ubcd1\uc804\uc5d0\uc11c\ub294 \ubd88\ub9ac\ud558\uc5ec \uc27d\uac8c \uc804\uc5f4\uc774 \ub3cc\ud30c\ub418\uace0 \uc788\uc5c8\ub2e4. \uadf8\ub294 \ud314\ub791\ud06c\uc2a4\uc758 \ubc00\uc9d1\ub300\ud615\uc744 \ud615\uc131\ud560 \ud615\ud3b8\uc774 \uc88b\uc740 \uc7a5\ucc3d\uacfc \ub465\uadfc\ubc29\ud328\ub85c \ubc14\uafb8\uc5c8\ub2e4.", "paragraph_answer": "\uae30\uc6d0\uc804 221\ub144, \ud544\ub85c\ud3ec\uc774\uba58\uc740 \uc804\uc7c1 \uc911\uc778 \ud06c\ub808\ud0c0\uc5d0\uc11c \uc6a9\ubcd1 \ub300\uc7a5\uc73c\ub85c \uc2f8\uc6e0\ub2e4. 10\ub144 \ud6c4 \uadc0\uad6d\uc5d0 \uc988\uc74c\ud558\uc5ec \uadf8\ub294 \uc544\uce74\uc774\uc544 \ub3d9\ub9f9\uc758 \uae30\ubcd1\ub300 \uc9c0\ud718\uad00\uc73c\ub85c \uc784\uba85\ub418\uc5c8\ub2e4. \ub2f9\uc2dc \uc544\uce74\uc774\uc544\uc758 \uae30\ubcd1\ub4e4\uc740 \uc57d\ud588\uae30 \ub54c\ubb38\uc5d0 \uadf8\ub294 \uae30\ubcd1\uc744 \uc6a9\ub9f9\ud558\uac8c \ub2e8\ub828\ud588\ub2e4. \uac19\uc740 \ud574\uc5d0 \uc81c1\ucc28 \ub9c8\ucf00\ub3c4\ub2c8\uc544 \uc804\uc7c1 \uacfc \uc5f0\uad00\ub41c \uc804\ud22c\uc5d0\uc11c \ud544\ub85c\ud3ec\uc774\uba58\uc774 \uc774\ub044\ub294 \uc544\uce74\uc774\uc544 \uad70\uc740 \uc5d8\ub9ac\uc2a4\uc758 \uacbd\uacc4\uc5d0 \uc788\ub294 \ub77c\ub9ac\uc0ac \uac15\uc5d0\uc11c \uc544\uc774\ud1a8\ub9ac\uc544, \uc5d8\ub9ac\uc2a4 \uc5f0\ud569\uad70\uacfc \uc2f8\uc6cc \ubb3c\ub9ac\uce58\uace0, \uc801\uc7a5 \ub2e4\ubaa8\ud310\ud22c\uc2a4\uc640 \uc9c1\uc811 \ub9de\uc11c\uc11c \ubb3c\ub9ac\uce58\uace0 \uc2b9\ub9ac\ub97c \uac70\ub450\uc5c8\ub2e4. \uc774 \uc2b9\ub9ac\ub85c \ud544\ub85c\ud3ec\uc774\uba58\uc758 \uba85\uc131\uc740 \uadf8\ub9ac\uc2a4\uc5d0 \ub110\ub9ac \ud37c\uc84c\uc73c\uba70, \ub610\ud55c \uadf8\ub294 \uc544\uce74\uc774\uc544 \ub3d9\ub9f9\uc758 \ubcf4\ubcd1 \uc7a5\ube44\ub97c \uac1c\ud601\ud588\ub2e4. \uadf8\ub54c\uae4c\uc9c0 \uc544\uce74\uc774\uc544 \uad70\uc758 \uc7a5\ube44\ub294 \uc9e7\uc740 \ucc3d\uacfc \uac00\ubccd\uac8c \uc587\uc740 \ud0c0\uc6d0\ud615 \ubc29\ud328\uc600\uc73c\uba70, \uc774\uac83\uc740 \uc6d0\uac70\ub9ac \uc804\ud22c\uc5d0\uc11c\ub294 \uc720\ub9ac\ud558\uc9c0\ub9cc, \ubc31\ubcd1\uc804\uc5d0\uc11c\ub294 \ubd88\ub9ac\ud558\uc5ec \uc27d\uac8c \uc804\uc5f4\uc774 \ub3cc\ud30c\ub418\uace0 \uc788\uc5c8\ub2e4. \uadf8\ub294 \ud314\ub791\ud06c\uc2a4\uc758 \ubc00\uc9d1\ub300\ud615\uc744 \ud615\uc131\ud560 \ud615\ud3b8\uc774 \uc88b\uc740 \uc7a5\ucc3d\uacfc \ub465\uadfc\ubc29\ud328\ub85c \ubc14\uafb8\uc5c8\ub2e4.", "sentence_answer": "\uac19\uc740 \ud574\uc5d0 \uc81c1\ucc28 \ub9c8\ucf00\ub3c4\ub2c8\uc544 \uc804\uc7c1 \uacfc \uc5f0\uad00\ub41c \uc804\ud22c\uc5d0\uc11c \ud544\ub85c\ud3ec\uc774\uba58\uc774 \uc774\ub044\ub294 \uc544\uce74\uc774\uc544 \uad70\uc740 \uc5d8\ub9ac\uc2a4\uc758 \uacbd\uacc4\uc5d0 \uc788\ub294 \ub77c\ub9ac\uc0ac \uac15\uc5d0\uc11c \uc544\uc774\ud1a8\ub9ac\uc544, \uc5d8\ub9ac\uc2a4 \uc5f0\ud569\uad70\uacfc \uc2f8\uc6cc \ubb3c\ub9ac\uce58\uace0, \uc801\uc7a5 \ub2e4\ubaa8\ud310\ud22c\uc2a4\uc640 \uc9c1\uc811 \ub9de\uc11c\uc11c \ubb3c\ub9ac\uce58\uace0 \uc2b9\ub9ac\ub97c \uac70\ub450\uc5c8\ub2e4.", "paragraph_id": "6569448-3-1", "paragraph_question": "question: \ud544\ub85c\ud3ec\ub2c8\uc544\uac00 \uc544\uc774\ud1a8\ub9ac\uc544,\uc5d8\ub9ac\uc2a4 \uc5f0\ud569\uad70\uacfc \uc2f8\uc6cc \uc2b9\ub9ac\ud55c \uc804\ud22c\ub294 \uc5b4\ub5a4 \uc804\uc7c1\uacfc \uc5f0\uad00\ub41c \uc804\ud22c\uc778\uac00?, context: \uae30\uc6d0\uc804 221\ub144, \ud544\ub85c\ud3ec\uc774\uba58\uc740 \uc804\uc7c1 \uc911\uc778 \ud06c\ub808\ud0c0\uc5d0\uc11c \uc6a9\ubcd1 \ub300\uc7a5\uc73c\ub85c \uc2f8\uc6e0\ub2e4. 10\ub144 \ud6c4 \uadc0\uad6d\uc5d0 \uc988\uc74c\ud558\uc5ec \uadf8\ub294 \uc544\uce74\uc774\uc544 \ub3d9\ub9f9\uc758 \uae30\ubcd1\ub300 \uc9c0\ud718\uad00\uc73c\ub85c \uc784\uba85\ub418\uc5c8\ub2e4. \ub2f9\uc2dc \uc544\uce74\uc774\uc544\uc758 \uae30\ubcd1\ub4e4\uc740 \uc57d\ud588\uae30 \ub54c\ubb38\uc5d0 \uadf8\ub294 \uae30\ubcd1\uc744 \uc6a9\ub9f9\ud558\uac8c \ub2e8\ub828\ud588\ub2e4. \uac19\uc740 \ud574\uc5d0 \uc81c1\ucc28 \ub9c8\ucf00\ub3c4\ub2c8\uc544 \uc804\uc7c1\uacfc \uc5f0\uad00\ub41c \uc804\ud22c\uc5d0\uc11c \ud544\ub85c\ud3ec\uc774\uba58\uc774 \uc774\ub044\ub294 \uc544\uce74\uc774\uc544 \uad70\uc740 \uc5d8\ub9ac\uc2a4\uc758 \uacbd\uacc4\uc5d0 \uc788\ub294 \ub77c\ub9ac\uc0ac \uac15\uc5d0\uc11c \uc544\uc774\ud1a8\ub9ac\uc544, \uc5d8\ub9ac\uc2a4 \uc5f0\ud569\uad70\uacfc \uc2f8\uc6cc \ubb3c\ub9ac\uce58\uace0, \uc801\uc7a5 \ub2e4\ubaa8\ud310\ud22c\uc2a4\uc640 \uc9c1\uc811 \ub9de\uc11c\uc11c \ubb3c\ub9ac\uce58\uace0 \uc2b9\ub9ac\ub97c \uac70\ub450\uc5c8\ub2e4. \uc774 \uc2b9\ub9ac\ub85c \ud544\ub85c\ud3ec\uc774\uba58\uc758 \uba85\uc131\uc740 \uadf8\ub9ac\uc2a4\uc5d0 \ub110\ub9ac \ud37c\uc84c\uc73c\uba70, \ub610\ud55c \uadf8\ub294 \uc544\uce74\uc774\uc544 \ub3d9\ub9f9\uc758 \ubcf4\ubcd1 \uc7a5\ube44\ub97c \uac1c\ud601\ud588\ub2e4. \uadf8\ub54c\uae4c\uc9c0 \uc544\uce74\uc774\uc544 \uad70\uc758 \uc7a5\ube44\ub294 \uc9e7\uc740 \ucc3d\uacfc \uac00\ubccd\uac8c \uc587\uc740 \ud0c0\uc6d0\ud615 \ubc29\ud328\uc600\uc73c\uba70, \uc774\uac83\uc740 \uc6d0\uac70\ub9ac \uc804\ud22c\uc5d0\uc11c\ub294 \uc720\ub9ac\ud558\uc9c0\ub9cc, \ubc31\ubcd1\uc804\uc5d0\uc11c\ub294 \ubd88\ub9ac\ud558\uc5ec \uc27d\uac8c \uc804\uc5f4\uc774 \ub3cc\ud30c\ub418\uace0 \uc788\uc5c8\ub2e4. \uadf8\ub294 \ud314\ub791\ud06c\uc2a4\uc758 \ubc00\uc9d1\ub300\ud615\uc744 \ud615\uc131\ud560 \ud615\ud3b8\uc774 \uc88b\uc740 \uc7a5\ucc3d\uacfc \ub465\uadfc\ubc29\ud328\ub85c \ubc14\uafb8\uc5c8\ub2e4."} {"question": "\uac10\uac01 \uc815\ubcf4\ub294 \uc5b4\ub514\ub97c \ud1b5\ud574 \uc911\ucd94\uc2e0\uacbd\uacc4\ub85c \uc804\ub2ec\ub418\ub098?", "paragraph": "\ub9d0\ucd08 \ubd80\uc704\uc5d0\uc11c, \ubab8\uac10\uac01 \uae30\uad00\uc740 \ucd09\uac01\uc744 \ubc1b\uc544\ub4e4\uc774\ub294 \uae30\uacc4\uc218\uc6a9\uccb4\ub098 \ud1b5\uac01\uc744 \ubc1b\uc544\ub4e4\uc774\ub294 \ud1b5\uac01\uc218\uc6a9\uccb4 (nociceptor) \ub4f1\uc744 \ud1b5\ud574 \ub2e4\uc591\ud55c \uc790\uadf9\uc744 \ubc1b\uc544\ub4e4\uc778\ub2e4. \uc774\ud6c4 \uac10\uac01 \uc815\ubcf4 (\ucd09\uac01, \ud1b5\uac01, \uc628\uac01, \ub0c9\uac01 \ub4f1\ub4f1)\ub294 \uad6c\uc2ec\uc131 \uc2e0\uacbd (afferent neuron)\uc744 \ud1b5\ud574 \uc911\ucd94\uc2e0\uacbd\uacc4\ub85c \uc804\ub2ec\ub41c\ub2e4. \uad6c\uc2ec\uc131 \uc2e0\uacbd\uc740 \uc5ec\ub7ec \uc885\ub958\uac00 \uc788\uc5b4 \ud06c\uae30, \uad6c\uc870, \ud2b9\uc131\uc774 \uc81c\uac01\uac01\uc774\ub2e4. \uc77c\ubc18\uc801\uc73c\ub85c\ub294 \ubc1b\uc544\ub4e4\uc774\ub294 \uac10\uac01\uc758 \uc885\ub958\uc5d0 \ub530\ub77c \uc2e0\uacbd\uc774 \ub2e4\ub974\uac8c \uc874\uc7ac\ud55c\ub2e4. \uc608\ub97c \ub4e4\uc5b4, \uace0\ud1b5\uc744 \ubc1b\uc544\ub4e4\uc774\ub294 \uc2e0\uacbd\uc758 \uacbd\uc6b0 \uac00\ub290\ub2e4\ub780 \ubb34\uc218 \uc2e0\uacbd (unmyelinated neuron)\uc73c\ub85c \ucc9c\ucc9c\ud788 \uac10\uac01\uc744 \uc804\ub2ec\ud558\uba70, \ucd09\uac01\uc740 \uad75\uc740 \uc720\uc218 \uc2e0\uacbd (mylienated neuron)\uc73c\ub85c \ube60\ub974\uac8c \uac10\uac01\uc744 \uc804\ub2ec\ud55c\ub2e4.", "answer": "\uad6c\uc2ec\uc131 \uc2e0\uacbd", "sentence": "\uc774\ud6c4 \uac10\uac01 \uc815\ubcf4 (\ucd09\uac01, \ud1b5\uac01, \uc628\uac01, \ub0c9\uac01 \ub4f1\ub4f1)\ub294 \uad6c\uc2ec\uc131 \uc2e0\uacbd (afferent neuron)\uc744 \ud1b5\ud574 \uc911\ucd94\uc2e0\uacbd\uacc4\ub85c \uc804\ub2ec\ub41c\ub2e4.", "paragraph_sentence": "\ub9d0\ucd08 \ubd80\uc704\uc5d0\uc11c, \ubab8\uac10\uac01 \uae30\uad00\uc740 \ucd09\uac01\uc744 \ubc1b\uc544\ub4e4\uc774\ub294 \uae30\uacc4\uc218\uc6a9\uccb4\ub098 \ud1b5\uac01\uc744 \ubc1b\uc544\ub4e4\uc774\ub294 \ud1b5\uac01\uc218\uc6a9\uccb4 (nociceptor) \ub4f1\uc744 \ud1b5\ud574 \ub2e4\uc591\ud55c \uc790\uadf9\uc744 \ubc1b\uc544\ub4e4\uc778\ub2e4. \uc774\ud6c4 \uac10\uac01 \uc815\ubcf4 (\ucd09\uac01, \ud1b5\uac01, \uc628\uac01, \ub0c9\uac01 \ub4f1\ub4f1)\ub294 \uad6c\uc2ec\uc131 \uc2e0\uacbd (afferent neuron)\uc744 \ud1b5\ud574 \uc911\ucd94\uc2e0\uacbd\uacc4\ub85c \uc804\ub2ec\ub41c\ub2e4. \uad6c\uc2ec\uc131 \uc2e0\uacbd\uc740 \uc5ec\ub7ec \uc885\ub958\uac00 \uc788\uc5b4 \ud06c\uae30, \uad6c\uc870, \ud2b9\uc131\uc774 \uc81c\uac01\uac01\uc774\ub2e4. \uc77c\ubc18\uc801\uc73c\ub85c\ub294 \ubc1b\uc544\ub4e4\uc774\ub294 \uac10\uac01\uc758 \uc885\ub958\uc5d0 \ub530\ub77c \uc2e0\uacbd\uc774 \ub2e4\ub974\uac8c \uc874\uc7ac\ud55c\ub2e4. \uc608\ub97c \ub4e4\uc5b4, \uace0\ud1b5\uc744 \ubc1b\uc544\ub4e4\uc774\ub294 \uc2e0\uacbd\uc758 \uacbd\uc6b0 \uac00\ub290\ub2e4\ub780 \ubb34\uc218 \uc2e0\uacbd (unmyelinated neuron)\uc73c\ub85c \ucc9c\ucc9c\ud788 \uac10\uac01\uc744 \uc804\ub2ec\ud558\uba70, \ucd09\uac01\uc740 \uad75\uc740 \uc720\uc218 \uc2e0\uacbd (mylienated neuron)\uc73c\ub85c \ube60\ub974\uac8c \uac10\uac01\uc744 \uc804\ub2ec\ud55c\ub2e4.", "paragraph_answer": "\ub9d0\ucd08 \ubd80\uc704\uc5d0\uc11c, \ubab8\uac10\uac01 \uae30\uad00\uc740 \ucd09\uac01\uc744 \ubc1b\uc544\ub4e4\uc774\ub294 \uae30\uacc4\uc218\uc6a9\uccb4\ub098 \ud1b5\uac01\uc744 \ubc1b\uc544\ub4e4\uc774\ub294 \ud1b5\uac01\uc218\uc6a9\uccb4 (nociceptor) \ub4f1\uc744 \ud1b5\ud574 \ub2e4\uc591\ud55c \uc790\uadf9\uc744 \ubc1b\uc544\ub4e4\uc778\ub2e4. \uc774\ud6c4 \uac10\uac01 \uc815\ubcf4 (\ucd09\uac01, \ud1b5\uac01, \uc628\uac01, \ub0c9\uac01 \ub4f1\ub4f1)\ub294 \uad6c\uc2ec\uc131 \uc2e0\uacbd (afferent neuron)\uc744 \ud1b5\ud574 \uc911\ucd94\uc2e0\uacbd\uacc4\ub85c \uc804\ub2ec\ub41c\ub2e4. \uad6c\uc2ec\uc131 \uc2e0\uacbd\uc740 \uc5ec\ub7ec \uc885\ub958\uac00 \uc788\uc5b4 \ud06c\uae30, \uad6c\uc870, \ud2b9\uc131\uc774 \uc81c\uac01\uac01\uc774\ub2e4. \uc77c\ubc18\uc801\uc73c\ub85c\ub294 \ubc1b\uc544\ub4e4\uc774\ub294 \uac10\uac01\uc758 \uc885\ub958\uc5d0 \ub530\ub77c \uc2e0\uacbd\uc774 \ub2e4\ub974\uac8c \uc874\uc7ac\ud55c\ub2e4. \uc608\ub97c \ub4e4\uc5b4, \uace0\ud1b5\uc744 \ubc1b\uc544\ub4e4\uc774\ub294 \uc2e0\uacbd\uc758 \uacbd\uc6b0 \uac00\ub290\ub2e4\ub780 \ubb34\uc218 \uc2e0\uacbd (unmyelinated neuron)\uc73c\ub85c \ucc9c\ucc9c\ud788 \uac10\uac01\uc744 \uc804\ub2ec\ud558\uba70, \ucd09\uac01\uc740 \uad75\uc740 \uc720\uc218 \uc2e0\uacbd (mylienated neuron)\uc73c\ub85c \ube60\ub974\uac8c \uac10\uac01\uc744 \uc804\ub2ec\ud55c\ub2e4.", "sentence_answer": "\uc774\ud6c4 \uac10\uac01 \uc815\ubcf4 (\ucd09\uac01, \ud1b5\uac01, \uc628\uac01, \ub0c9\uac01 \ub4f1\ub4f1)\ub294 \uad6c\uc2ec\uc131 \uc2e0\uacbd (afferent neuron)\uc744 \ud1b5\ud574 \uc911\ucd94\uc2e0\uacbd\uacc4\ub85c \uc804\ub2ec\ub41c\ub2e4.", "paragraph_id": "6085377-0-2", "paragraph_question": "question: \uac10\uac01 \uc815\ubcf4\ub294 \uc5b4\ub514\ub97c \ud1b5\ud574 \uc911\ucd94\uc2e0\uacbd\uacc4\ub85c \uc804\ub2ec\ub418\ub098?, context: \ub9d0\ucd08 \ubd80\uc704\uc5d0\uc11c, \ubab8\uac10\uac01 \uae30\uad00\uc740 \ucd09\uac01\uc744 \ubc1b\uc544\ub4e4\uc774\ub294 \uae30\uacc4\uc218\uc6a9\uccb4\ub098 \ud1b5\uac01\uc744 \ubc1b\uc544\ub4e4\uc774\ub294 \ud1b5\uac01\uc218\uc6a9\uccb4 (nociceptor) \ub4f1\uc744 \ud1b5\ud574 \ub2e4\uc591\ud55c \uc790\uadf9\uc744 \ubc1b\uc544\ub4e4\uc778\ub2e4. \uc774\ud6c4 \uac10\uac01 \uc815\ubcf4 (\ucd09\uac01, \ud1b5\uac01, \uc628\uac01, \ub0c9\uac01 \ub4f1\ub4f1)\ub294 \uad6c\uc2ec\uc131 \uc2e0\uacbd (afferent neuron)\uc744 \ud1b5\ud574 \uc911\ucd94\uc2e0\uacbd\uacc4\ub85c \uc804\ub2ec\ub41c\ub2e4. \uad6c\uc2ec\uc131 \uc2e0\uacbd\uc740 \uc5ec\ub7ec \uc885\ub958\uac00 \uc788\uc5b4 \ud06c\uae30, \uad6c\uc870, \ud2b9\uc131\uc774 \uc81c\uac01\uac01\uc774\ub2e4. \uc77c\ubc18\uc801\uc73c\ub85c\ub294 \ubc1b\uc544\ub4e4\uc774\ub294 \uac10\uac01\uc758 \uc885\ub958\uc5d0 \ub530\ub77c \uc2e0\uacbd\uc774 \ub2e4\ub974\uac8c \uc874\uc7ac\ud55c\ub2e4. \uc608\ub97c \ub4e4\uc5b4, \uace0\ud1b5\uc744 \ubc1b\uc544\ub4e4\uc774\ub294 \uc2e0\uacbd\uc758 \uacbd\uc6b0 \uac00\ub290\ub2e4\ub780 \ubb34\uc218 \uc2e0\uacbd (unmyelinated neuron)\uc73c\ub85c \ucc9c\ucc9c\ud788 \uac10\uac01\uc744 \uc804\ub2ec\ud558\uba70, \ucd09\uac01\uc740 \uad75\uc740 \uc720\uc218 \uc2e0\uacbd (mylienated neuron)\uc73c\ub85c \ube60\ub974\uac8c \uac10\uac01\uc744 \uc804\ub2ec\ud55c\ub2e4."} {"question": "\uac10\uc219\uc131\uc5d0\uc11c \uc77c\uc5b4\ub09c \uccad\ub098\ub77c \ucd5c\ub300 \ub1cc\ubb3c\uc218\uc218 \uc0ac\uac74 \uc774\ub984\uc740 \ubb34\uc5c7\uc778\uac00?", "paragraph": "\uac74\ub96d\uc81c\uac00 \uccad\ub144, \uc911\ub144 \ub54c\uc5d0\ub294 \ubd80\ud669 \uc639\uc815\uc81c\ucc98\ub7fc \uc8fc\ud544(\u6731\u7b46)\uc744 \ud558\uc5ec \uad00\ub9ac\ub4e4\uc744 \ud56d\uc0c1 \uac10\ucc30\ud558\ub294 \ub4f1 \uae30\uac15\uc744 \uc5c4\uc815\ud788 \ud558\uc5ec \uad00\ub9ac\ub4e4\uc758 \ud6a1\ub839\uc774\ub098 \ud0c8\uc138\uac19\uc740 \uc77c\uc774 \uac70\uc758 \uc77c\uc5b4\ub098\uc9c0 \uc54a\uc558\uace0 \uc77c\uc5b4\ub098\ub354\ub77c\ub3c4 \uadf8 \uae08\uc561\uc740 \uc801\uc5c8\ub2e4. \uadf8\ub7ec\ub098 \uac74\ub96d\uc81c\uac00 \ub098\uc774\uac00 \ub4e4\uba74\uc11c \uc911\uc559\uacfc \ubd81\uacbd \uc8fc\ubcc0\uc758 \uc815\ubb34\ub9cc \ucc98\ub9ac\ud558\uace0 \uc8fc\ud544\uc744 \uc4f0\ub294\ub370 \uc18c\ud640\ud574\uc84c\uc73c\uba70 \uac01 \uc131\uc758 \ud0c4\uc6d0\uc774\ub098 \uc815\ucc45\uc740 \uac01 \uc131\uc758 \ucd1d\ub3c5, \uc21c\ubb34\uac00 \uc7ac\ub7c9\uaecf \ucc98\ub9ac\ud558\uac8c \ub0b4\ubc84\ub824 \ub450\ub294 \ub4f1 \ub290\uc2a8\ud558\uac8c \ub300\ud558\uc600\ub2e4. \uc774\uc5d0 \ub530\ub77c \uac74\ub96d\uc81c\uc758 \uac10\uc2dc\ub9dd\uc5d0\uc11c \ubc97\uc5b4\ub09c \uc9c0\ubc29 \uad00\ub9ac\ub4e4\uc774 \ubc31\uc131\ub4e4\uacfc \uc0c1\uc778\ub4e4\uc744 \uc0c1\ub300\ub85c \ucc29\ubcf5\uacfc \ud6a1\ub839, \ud0c8\uc138\ub97c \uc790\uc8fc \ud558\uc600\uace0 \uadf8 \uc5f0\uad00\ub41c \uae08\uc561\ub3c4 \ub9e4 \uac74\ub2f9 \uc218\uc2ed\ub9cc \ub0e5\uc744 \uc6c3\ub3cc\uc558\ub2e4. \uac74\ub96d\uc81c\uc758 \uce58\uc138 \ub3d9\uc548 \ud669\ud558\uc5d0\uc11c 7\uac74\uc758 \ud64d\uc218 \ubc94\ub78c \uc0ac\ud0dc\uac00 \ubc1c\uc0dd\ud558\uc5ec \uac74\ub96d\uc81c\uac00 \uad6c\ud638 \uc790\uae08\uc744 \ubcf4\ub0b4\uc8fc\uc5c8\uc73c\ub098 \uc911\uac04\uc5d0\uc11c \uad00\ub9ac\ub4e4\uc774 \ubaa8\ub450 \ud6a1\ub839\ud55c \ubc14\ub78c\uc5d0 \uce58\uc218 \uacf5\uc0ac\uac00 \uc9c4\ucc99\uc744 \uc774\ub8e8\uc9c0 \ubabb\ud55c \uc801\ub3c4 \uc788\uc5c8\ub2e4. \uadf8 \uc774\ud6c4\ub85c\ub294 \uc810\uc810 \uc2ec\ud574\uc838 \uc911\uc559\uc5d0\uc11c \ucc98\ub9ac\ud574\uc904 \uac15\ub825\ud558\uace0 \uccad\ub834\ud55c \uace0\uc704 \uad00\ub9ac\ub4e4\uc774 \ub098\ud0c0\ub098\uc9c0 \uc54a\uc544 \uc9c0\ubc29\uc758 \ub9d0\ub2e8\uad00\ub9ac\ubd80\ud130 \uc7ac\uc0c1\uae09\uc778 \uc218\uc11d\uad70\uae30\ub300\uc2e0(\u9996\u5e2d\u8ecd\u6a5f\u5927\u81e3)\uae4c\uc9c0 \uc5f0\uad00\ub418\uc5b4\uc11c \uc9d1\ub2e8 \ub1cc\ubb3c\uc218\uc218 \uc0ac\uac74\uae4c\uc9c0 \ubc1c\uc0dd\ud558\ub294 \ub4f1 \uc2ec\uac01\ud574\uc84c\ub2e4. \uac74\ub96d\uc81c \uce58\uc138 \uc911\uae30 \ub54c\ubd80\ud130 \uad70\uae30\ub300\uc2e0 \uc6b0\ubbfc\uc911\uc740 \uccad\ub834\uacfc\ub294 \uac70\ub9ac\uac00 \uba40\uace0 \uc2a4\uc2a4\ub85c \ub9c9\ub300\ud55c \uc0ac\uc775\uc744 \ucde8\ud558\uba70 \ub300\uc2e0\ub4e4 \uc0ac\uc774\uc5d0\uc11c\ub3c4 \ub1cc\ubb3c\uc774 \uc624\uac00\uae30 \uc2dc\uc791\ud558\uc600\ub2e4. \uc774\uc5d0 \ud558\uae09 \uad00\ub9ac\ub4e4 \uc5ed\uc2dc \uc11c\ub85c \ub1cc\ubb3c\uc218\uc218\ub97c \ub208\uac10\uc544\uc8fc\uace0 \uacfc\uac70\uc2dc\ud5d8\uc5d0\uc11c\ub3c4 \uc77c\uc815\ud55c \ub3c8\uc744 \uc9c0\ubd88\ud55c \ub300\uac00\ub85c \uadf8 \uc751\uc2dc\uc790\uc5d0\uac8c \ud569\uaca9\uc2dc\ucf1c \uc8fc\ub294 \ub4f1 \uadf8 \ube44\ub9ac\uac00 \uacc4\uc18d\ub418\uba74\uc11c \uac10\uc219\uc131\uc5d0\uc11c \uccad\ub098\ub77c \ucd5c\ub300\uc758 \ub1cc\ubb3c\uc218\uc218 \uc0ac\uac74\uc778 \uac10\uc219\ubaa8\uc9c4\uc548(\u7518\u8085\u5192\u8cd1\u6848)\uc774 \ubc1c\uc0dd\ud558\uc600\ub2e4.", "answer": "\uac10\uc219\ubaa8\uc9c4\uc548", "sentence": "\uc774\uc5d0 \ud558\uae09 \uad00\ub9ac\ub4e4 \uc5ed\uc2dc \uc11c\ub85c \ub1cc\ubb3c\uc218\uc218\ub97c \ub208\uac10\uc544\uc8fc\uace0 \uacfc\uac70\uc2dc\ud5d8\uc5d0\uc11c\ub3c4 \uc77c\uc815\ud55c \ub3c8\uc744 \uc9c0\ubd88\ud55c \ub300\uac00\ub85c \uadf8 \uc751\uc2dc\uc790\uc5d0\uac8c \ud569\uaca9\uc2dc\ucf1c \uc8fc\ub294 \ub4f1 \uadf8 \ube44\ub9ac\uac00 \uacc4\uc18d\ub418\uba74\uc11c \uac10\uc219\uc131\uc5d0\uc11c \uccad\ub098\ub77c \ucd5c\ub300\uc758 \ub1cc\ubb3c\uc218\uc218 \uc0ac\uac74\uc778 \uac10\uc219\ubaa8\uc9c4\uc548 (\u7518\u8085\u5192\u8cd1\u6848)\uc774 \ubc1c\uc0dd\ud558\uc600\ub2e4.", "paragraph_sentence": "\uac74\ub96d\uc81c\uac00 \uccad\ub144, \uc911\ub144 \ub54c\uc5d0\ub294 \ubd80\ud669 \uc639\uc815\uc81c\ucc98\ub7fc \uc8fc\ud544(\u6731\u7b46)\uc744 \ud558\uc5ec \uad00\ub9ac\ub4e4\uc744 \ud56d\uc0c1 \uac10\ucc30\ud558\ub294 \ub4f1 \uae30\uac15\uc744 \uc5c4\uc815\ud788 \ud558\uc5ec \uad00\ub9ac\ub4e4\uc758 \ud6a1\ub839\uc774\ub098 \ud0c8\uc138\uac19\uc740 \uc77c\uc774 \uac70\uc758 \uc77c\uc5b4\ub098\uc9c0 \uc54a\uc558\uace0 \uc77c\uc5b4\ub098\ub354\ub77c\ub3c4 \uadf8 \uae08\uc561\uc740 \uc801\uc5c8\ub2e4. \uadf8\ub7ec\ub098 \uac74\ub96d\uc81c\uac00 \ub098\uc774\uac00 \ub4e4\uba74\uc11c \uc911\uc559\uacfc \ubd81\uacbd \uc8fc\ubcc0\uc758 \uc815\ubb34\ub9cc \ucc98\ub9ac\ud558\uace0 \uc8fc\ud544\uc744 \uc4f0\ub294\ub370 \uc18c\ud640\ud574\uc84c\uc73c\uba70 \uac01 \uc131\uc758 \ud0c4\uc6d0\uc774\ub098 \uc815\ucc45\uc740 \uac01 \uc131\uc758 \ucd1d\ub3c5, \uc21c\ubb34\uac00 \uc7ac\ub7c9\uaecf \ucc98\ub9ac\ud558\uac8c \ub0b4\ubc84\ub824 \ub450\ub294 \ub4f1 \ub290\uc2a8\ud558\uac8c \ub300\ud558\uc600\ub2e4. \uc774\uc5d0 \ub530\ub77c \uac74\ub96d\uc81c\uc758 \uac10\uc2dc\ub9dd\uc5d0\uc11c \ubc97\uc5b4\ub09c \uc9c0\ubc29 \uad00\ub9ac\ub4e4\uc774 \ubc31\uc131\ub4e4\uacfc \uc0c1\uc778\ub4e4\uc744 \uc0c1\ub300\ub85c \ucc29\ubcf5\uacfc \ud6a1\ub839, \ud0c8\uc138\ub97c \uc790\uc8fc \ud558\uc600\uace0 \uadf8 \uc5f0\uad00\ub41c \uae08\uc561\ub3c4 \ub9e4 \uac74\ub2f9 \uc218\uc2ed\ub9cc \ub0e5\uc744 \uc6c3\ub3cc\uc558\ub2e4. \uac74\ub96d\uc81c\uc758 \uce58\uc138 \ub3d9\uc548 \ud669\ud558\uc5d0\uc11c 7\uac74\uc758 \ud64d\uc218 \ubc94\ub78c \uc0ac\ud0dc\uac00 \ubc1c\uc0dd\ud558\uc5ec \uac74\ub96d\uc81c\uac00 \uad6c\ud638 \uc790\uae08\uc744 \ubcf4\ub0b4\uc8fc\uc5c8\uc73c\ub098 \uc911\uac04\uc5d0\uc11c \uad00\ub9ac\ub4e4\uc774 \ubaa8\ub450 \ud6a1\ub839\ud55c \ubc14\ub78c\uc5d0 \uce58\uc218 \uacf5\uc0ac\uac00 \uc9c4\ucc99\uc744 \uc774\ub8e8\uc9c0 \ubabb\ud55c \uc801\ub3c4 \uc788\uc5c8\ub2e4. \uadf8 \uc774\ud6c4\ub85c\ub294 \uc810\uc810 \uc2ec\ud574\uc838 \uc911\uc559\uc5d0\uc11c \ucc98\ub9ac\ud574\uc904 \uac15\ub825\ud558\uace0 \uccad\ub834\ud55c \uace0\uc704 \uad00\ub9ac\ub4e4\uc774 \ub098\ud0c0\ub098\uc9c0 \uc54a\uc544 \uc9c0\ubc29\uc758 \ub9d0\ub2e8\uad00\ub9ac\ubd80\ud130 \uc7ac\uc0c1\uae09\uc778 \uc218\uc11d\uad70\uae30\ub300\uc2e0(\u9996\u5e2d\u8ecd\u6a5f\u5927\u81e3)\uae4c\uc9c0 \uc5f0\uad00\ub418\uc5b4\uc11c \uc9d1\ub2e8 \ub1cc\ubb3c\uc218\uc218 \uc0ac\uac74\uae4c\uc9c0 \ubc1c\uc0dd\ud558\ub294 \ub4f1 \uc2ec\uac01\ud574\uc84c\ub2e4. \uac74\ub96d\uc81c \uce58\uc138 \uc911\uae30 \ub54c\ubd80\ud130 \uad70\uae30\ub300\uc2e0 \uc6b0\ubbfc\uc911\uc740 \uccad\ub834\uacfc\ub294 \uac70\ub9ac\uac00 \uba40\uace0 \uc2a4\uc2a4\ub85c \ub9c9\ub300\ud55c \uc0ac\uc775\uc744 \ucde8\ud558\uba70 \ub300\uc2e0\ub4e4 \uc0ac\uc774\uc5d0\uc11c\ub3c4 \ub1cc\ubb3c\uc774 \uc624\uac00\uae30 \uc2dc\uc791\ud558\uc600\ub2e4. \uc774\uc5d0 \ud558\uae09 \uad00\ub9ac\ub4e4 \uc5ed\uc2dc \uc11c\ub85c \ub1cc\ubb3c\uc218\uc218\ub97c \ub208\uac10\uc544\uc8fc\uace0 \uacfc\uac70\uc2dc\ud5d8\uc5d0\uc11c\ub3c4 \uc77c\uc815\ud55c \ub3c8\uc744 \uc9c0\ubd88\ud55c \ub300\uac00\ub85c \uadf8 \uc751\uc2dc\uc790\uc5d0\uac8c \ud569\uaca9\uc2dc\ucf1c \uc8fc\ub294 \ub4f1 \uadf8 \ube44\ub9ac\uac00 \uacc4\uc18d\ub418\uba74\uc11c \uac10\uc219\uc131\uc5d0\uc11c \uccad\ub098\ub77c \ucd5c\ub300\uc758 \ub1cc\ubb3c\uc218\uc218 \uc0ac\uac74\uc778 \uac10\uc219\ubaa8\uc9c4\uc548 (\u7518\u8085\u5192\u8cd1\u6848)\uc774 \ubc1c\uc0dd\ud558\uc600\ub2e4. ", "paragraph_answer": "\uac74\ub96d\uc81c\uac00 \uccad\ub144, \uc911\ub144 \ub54c\uc5d0\ub294 \ubd80\ud669 \uc639\uc815\uc81c\ucc98\ub7fc \uc8fc\ud544(\u6731\u7b46)\uc744 \ud558\uc5ec \uad00\ub9ac\ub4e4\uc744 \ud56d\uc0c1 \uac10\ucc30\ud558\ub294 \ub4f1 \uae30\uac15\uc744 \uc5c4\uc815\ud788 \ud558\uc5ec \uad00\ub9ac\ub4e4\uc758 \ud6a1\ub839\uc774\ub098 \ud0c8\uc138\uac19\uc740 \uc77c\uc774 \uac70\uc758 \uc77c\uc5b4\ub098\uc9c0 \uc54a\uc558\uace0 \uc77c\uc5b4\ub098\ub354\ub77c\ub3c4 \uadf8 \uae08\uc561\uc740 \uc801\uc5c8\ub2e4. \uadf8\ub7ec\ub098 \uac74\ub96d\uc81c\uac00 \ub098\uc774\uac00 \ub4e4\uba74\uc11c \uc911\uc559\uacfc \ubd81\uacbd \uc8fc\ubcc0\uc758 \uc815\ubb34\ub9cc \ucc98\ub9ac\ud558\uace0 \uc8fc\ud544\uc744 \uc4f0\ub294\ub370 \uc18c\ud640\ud574\uc84c\uc73c\uba70 \uac01 \uc131\uc758 \ud0c4\uc6d0\uc774\ub098 \uc815\ucc45\uc740 \uac01 \uc131\uc758 \ucd1d\ub3c5, \uc21c\ubb34\uac00 \uc7ac\ub7c9\uaecf \ucc98\ub9ac\ud558\uac8c \ub0b4\ubc84\ub824 \ub450\ub294 \ub4f1 \ub290\uc2a8\ud558\uac8c \ub300\ud558\uc600\ub2e4. \uc774\uc5d0 \ub530\ub77c \uac74\ub96d\uc81c\uc758 \uac10\uc2dc\ub9dd\uc5d0\uc11c \ubc97\uc5b4\ub09c \uc9c0\ubc29 \uad00\ub9ac\ub4e4\uc774 \ubc31\uc131\ub4e4\uacfc \uc0c1\uc778\ub4e4\uc744 \uc0c1\ub300\ub85c \ucc29\ubcf5\uacfc \ud6a1\ub839, \ud0c8\uc138\ub97c \uc790\uc8fc \ud558\uc600\uace0 \uadf8 \uc5f0\uad00\ub41c \uae08\uc561\ub3c4 \ub9e4 \uac74\ub2f9 \uc218\uc2ed\ub9cc \ub0e5\uc744 \uc6c3\ub3cc\uc558\ub2e4. \uac74\ub96d\uc81c\uc758 \uce58\uc138 \ub3d9\uc548 \ud669\ud558\uc5d0\uc11c 7\uac74\uc758 \ud64d\uc218 \ubc94\ub78c \uc0ac\ud0dc\uac00 \ubc1c\uc0dd\ud558\uc5ec \uac74\ub96d\uc81c\uac00 \uad6c\ud638 \uc790\uae08\uc744 \ubcf4\ub0b4\uc8fc\uc5c8\uc73c\ub098 \uc911\uac04\uc5d0\uc11c \uad00\ub9ac\ub4e4\uc774 \ubaa8\ub450 \ud6a1\ub839\ud55c \ubc14\ub78c\uc5d0 \uce58\uc218 \uacf5\uc0ac\uac00 \uc9c4\ucc99\uc744 \uc774\ub8e8\uc9c0 \ubabb\ud55c \uc801\ub3c4 \uc788\uc5c8\ub2e4. \uadf8 \uc774\ud6c4\ub85c\ub294 \uc810\uc810 \uc2ec\ud574\uc838 \uc911\uc559\uc5d0\uc11c \ucc98\ub9ac\ud574\uc904 \uac15\ub825\ud558\uace0 \uccad\ub834\ud55c \uace0\uc704 \uad00\ub9ac\ub4e4\uc774 \ub098\ud0c0\ub098\uc9c0 \uc54a\uc544 \uc9c0\ubc29\uc758 \ub9d0\ub2e8\uad00\ub9ac\ubd80\ud130 \uc7ac\uc0c1\uae09\uc778 \uc218\uc11d\uad70\uae30\ub300\uc2e0(\u9996\u5e2d\u8ecd\u6a5f\u5927\u81e3)\uae4c\uc9c0 \uc5f0\uad00\ub418\uc5b4\uc11c \uc9d1\ub2e8 \ub1cc\ubb3c\uc218\uc218 \uc0ac\uac74\uae4c\uc9c0 \ubc1c\uc0dd\ud558\ub294 \ub4f1 \uc2ec\uac01\ud574\uc84c\ub2e4. \uac74\ub96d\uc81c \uce58\uc138 \uc911\uae30 \ub54c\ubd80\ud130 \uad70\uae30\ub300\uc2e0 \uc6b0\ubbfc\uc911\uc740 \uccad\ub834\uacfc\ub294 \uac70\ub9ac\uac00 \uba40\uace0 \uc2a4\uc2a4\ub85c \ub9c9\ub300\ud55c \uc0ac\uc775\uc744 \ucde8\ud558\uba70 \ub300\uc2e0\ub4e4 \uc0ac\uc774\uc5d0\uc11c\ub3c4 \ub1cc\ubb3c\uc774 \uc624\uac00\uae30 \uc2dc\uc791\ud558\uc600\ub2e4. \uc774\uc5d0 \ud558\uae09 \uad00\ub9ac\ub4e4 \uc5ed\uc2dc \uc11c\ub85c \ub1cc\ubb3c\uc218\uc218\ub97c \ub208\uac10\uc544\uc8fc\uace0 \uacfc\uac70\uc2dc\ud5d8\uc5d0\uc11c\ub3c4 \uc77c\uc815\ud55c \ub3c8\uc744 \uc9c0\ubd88\ud55c \ub300\uac00\ub85c \uadf8 \uc751\uc2dc\uc790\uc5d0\uac8c \ud569\uaca9\uc2dc\ucf1c \uc8fc\ub294 \ub4f1 \uadf8 \ube44\ub9ac\uac00 \uacc4\uc18d\ub418\uba74\uc11c \uac10\uc219\uc131\uc5d0\uc11c \uccad\ub098\ub77c \ucd5c\ub300\uc758 \ub1cc\ubb3c\uc218\uc218 \uc0ac\uac74\uc778 \uac10\uc219\ubaa8\uc9c4\uc548 (\u7518\u8085\u5192\u8cd1\u6848)\uc774 \ubc1c\uc0dd\ud558\uc600\ub2e4.", "sentence_answer": "\uc774\uc5d0 \ud558\uae09 \uad00\ub9ac\ub4e4 \uc5ed\uc2dc \uc11c\ub85c \ub1cc\ubb3c\uc218\uc218\ub97c \ub208\uac10\uc544\uc8fc\uace0 \uacfc\uac70\uc2dc\ud5d8\uc5d0\uc11c\ub3c4 \uc77c\uc815\ud55c \ub3c8\uc744 \uc9c0\ubd88\ud55c \ub300\uac00\ub85c \uadf8 \uc751\uc2dc\uc790\uc5d0\uac8c \ud569\uaca9\uc2dc\ucf1c \uc8fc\ub294 \ub4f1 \uadf8 \ube44\ub9ac\uac00 \uacc4\uc18d\ub418\uba74\uc11c \uac10\uc219\uc131\uc5d0\uc11c \uccad\ub098\ub77c \ucd5c\ub300\uc758 \ub1cc\ubb3c\uc218\uc218 \uc0ac\uac74\uc778 \uac10\uc219\ubaa8\uc9c4\uc548 (\u7518\u8085\u5192\u8cd1\u6848)\uc774 \ubc1c\uc0dd\ud558\uc600\ub2e4.", "paragraph_id": "6486904-21-0", "paragraph_question": "question: \uac10\uc219\uc131\uc5d0\uc11c \uc77c\uc5b4\ub09c \uccad\ub098\ub77c \ucd5c\ub300 \ub1cc\ubb3c\uc218\uc218 \uc0ac\uac74 \uc774\ub984\uc740 \ubb34\uc5c7\uc778\uac00?, context: \uac74\ub96d\uc81c\uac00 \uccad\ub144, \uc911\ub144 \ub54c\uc5d0\ub294 \ubd80\ud669 \uc639\uc815\uc81c\ucc98\ub7fc \uc8fc\ud544(\u6731\u7b46)\uc744 \ud558\uc5ec \uad00\ub9ac\ub4e4\uc744 \ud56d\uc0c1 \uac10\ucc30\ud558\ub294 \ub4f1 \uae30\uac15\uc744 \uc5c4\uc815\ud788 \ud558\uc5ec \uad00\ub9ac\ub4e4\uc758 \ud6a1\ub839\uc774\ub098 \ud0c8\uc138\uac19\uc740 \uc77c\uc774 \uac70\uc758 \uc77c\uc5b4\ub098\uc9c0 \uc54a\uc558\uace0 \uc77c\uc5b4\ub098\ub354\ub77c\ub3c4 \uadf8 \uae08\uc561\uc740 \uc801\uc5c8\ub2e4. \uadf8\ub7ec\ub098 \uac74\ub96d\uc81c\uac00 \ub098\uc774\uac00 \ub4e4\uba74\uc11c \uc911\uc559\uacfc \ubd81\uacbd \uc8fc\ubcc0\uc758 \uc815\ubb34\ub9cc \ucc98\ub9ac\ud558\uace0 \uc8fc\ud544\uc744 \uc4f0\ub294\ub370 \uc18c\ud640\ud574\uc84c\uc73c\uba70 \uac01 \uc131\uc758 \ud0c4\uc6d0\uc774\ub098 \uc815\ucc45\uc740 \uac01 \uc131\uc758 \ucd1d\ub3c5, \uc21c\ubb34\uac00 \uc7ac\ub7c9\uaecf \ucc98\ub9ac\ud558\uac8c \ub0b4\ubc84\ub824 \ub450\ub294 \ub4f1 \ub290\uc2a8\ud558\uac8c \ub300\ud558\uc600\ub2e4. \uc774\uc5d0 \ub530\ub77c \uac74\ub96d\uc81c\uc758 \uac10\uc2dc\ub9dd\uc5d0\uc11c \ubc97\uc5b4\ub09c \uc9c0\ubc29 \uad00\ub9ac\ub4e4\uc774 \ubc31\uc131\ub4e4\uacfc \uc0c1\uc778\ub4e4\uc744 \uc0c1\ub300\ub85c \ucc29\ubcf5\uacfc \ud6a1\ub839, \ud0c8\uc138\ub97c \uc790\uc8fc \ud558\uc600\uace0 \uadf8 \uc5f0\uad00\ub41c \uae08\uc561\ub3c4 \ub9e4 \uac74\ub2f9 \uc218\uc2ed\ub9cc \ub0e5\uc744 \uc6c3\ub3cc\uc558\ub2e4. \uac74\ub96d\uc81c\uc758 \uce58\uc138 \ub3d9\uc548 \ud669\ud558\uc5d0\uc11c 7\uac74\uc758 \ud64d\uc218 \ubc94\ub78c \uc0ac\ud0dc\uac00 \ubc1c\uc0dd\ud558\uc5ec \uac74\ub96d\uc81c\uac00 \uad6c\ud638 \uc790\uae08\uc744 \ubcf4\ub0b4\uc8fc\uc5c8\uc73c\ub098 \uc911\uac04\uc5d0\uc11c \uad00\ub9ac\ub4e4\uc774 \ubaa8\ub450 \ud6a1\ub839\ud55c \ubc14\ub78c\uc5d0 \uce58\uc218 \uacf5\uc0ac\uac00 \uc9c4\ucc99\uc744 \uc774\ub8e8\uc9c0 \ubabb\ud55c \uc801\ub3c4 \uc788\uc5c8\ub2e4. \uadf8 \uc774\ud6c4\ub85c\ub294 \uc810\uc810 \uc2ec\ud574\uc838 \uc911\uc559\uc5d0\uc11c \ucc98\ub9ac\ud574\uc904 \uac15\ub825\ud558\uace0 \uccad\ub834\ud55c \uace0\uc704 \uad00\ub9ac\ub4e4\uc774 \ub098\ud0c0\ub098\uc9c0 \uc54a\uc544 \uc9c0\ubc29\uc758 \ub9d0\ub2e8\uad00\ub9ac\ubd80\ud130 \uc7ac\uc0c1\uae09\uc778 \uc218\uc11d\uad70\uae30\ub300\uc2e0(\u9996\u5e2d\u8ecd\u6a5f\u5927\u81e3)\uae4c\uc9c0 \uc5f0\uad00\ub418\uc5b4\uc11c \uc9d1\ub2e8 \ub1cc\ubb3c\uc218\uc218 \uc0ac\uac74\uae4c\uc9c0 \ubc1c\uc0dd\ud558\ub294 \ub4f1 \uc2ec\uac01\ud574\uc84c\ub2e4. \uac74\ub96d\uc81c \uce58\uc138 \uc911\uae30 \ub54c\ubd80\ud130 \uad70\uae30\ub300\uc2e0 \uc6b0\ubbfc\uc911\uc740 \uccad\ub834\uacfc\ub294 \uac70\ub9ac\uac00 \uba40\uace0 \uc2a4\uc2a4\ub85c \ub9c9\ub300\ud55c \uc0ac\uc775\uc744 \ucde8\ud558\uba70 \ub300\uc2e0\ub4e4 \uc0ac\uc774\uc5d0\uc11c\ub3c4 \ub1cc\ubb3c\uc774 \uc624\uac00\uae30 \uc2dc\uc791\ud558\uc600\ub2e4. \uc774\uc5d0 \ud558\uae09 \uad00\ub9ac\ub4e4 \uc5ed\uc2dc \uc11c\ub85c \ub1cc\ubb3c\uc218\uc218\ub97c \ub208\uac10\uc544\uc8fc\uace0 \uacfc\uac70\uc2dc\ud5d8\uc5d0\uc11c\ub3c4 \uc77c\uc815\ud55c \ub3c8\uc744 \uc9c0\ubd88\ud55c \ub300\uac00\ub85c \uadf8 \uc751\uc2dc\uc790\uc5d0\uac8c \ud569\uaca9\uc2dc\ucf1c \uc8fc\ub294 \ub4f1 \uadf8 \ube44\ub9ac\uac00 \uacc4\uc18d\ub418\uba74\uc11c \uac10\uc219\uc131\uc5d0\uc11c \uccad\ub098\ub77c \ucd5c\ub300\uc758 \ub1cc\ubb3c\uc218\uc218 \uc0ac\uac74\uc778 \uac10\uc219\ubaa8\uc9c4\uc548(\u7518\u8085\u5192\u8cd1\u6848)\uc774 \ubc1c\uc0dd\ud558\uc600\ub2e4."} {"question": "\uc911\uad6d \ucd5c\ucd08\uc758 \uc778\uacf5 \ubd84\uc218\uac00 \uc124\uce58\ub41c \uacf3\uc740?", "paragraph": "\uac74\ub96d\uc81c\ub294 \ub8e8\uc774 14\uc138\uac00 \ubcf4\ub0b4\uc628 \uac00\ube0c\ub9ac\uc5d8 \ud398\ub810(Gabriel P\u00e9relle, 1602\ub144 ~ 1677\ub144)\uc758 \u300a\ud504\ub791\uc2a4\uc758 \uc544\ub984\ub2e4\uc6b4 \uac74\ubb3c\ub4e4\u300b\uacfc \u300a\ud504\ub791\uc2a4 \ub3c4\uc2dc\uc640 \ubcf4\uc2a4\ucf00 \ubc0f \ubd84\uc218\ub97c \ud3ec\ud568\ud55c \ubca0\ub974\uc0ac\uc774\uc720 \uad81\uc804\uc758 \uc124\uacc4\ub3c4\u00b7\ub2e8\uba74\ub3c4\u00b7\uc785\uba74\ub3c4\u300b\uc758 \ub3d9\ud310 \uc0bd\ud654\ub97c \ubcf4\uace0 \ud765\ubbf8\ub97c \ub290\uaf08\uace0, 1747\ub144(\uac74\ub96d 12\ub144)\ubd80\ud130 1759\ub144(\uac74\ub96d 24\ub144)\uae4c\uc9c0 \uc11c\uc591 \uc120\uad50\uc0ac\ub97c \ub3d9\uc6d0\ud574 \ubca0\ub974\uc0ac\uc720 \uad81\uc804\uc744 \ubaa8\ubc29\ud55c \uac74\ubb3c\ub4e4\uc744 \uc9c0\uc5c8\ub2e4. \uc774 \uac74\ubb3c\ub4e4\uc740 \ubcf8\ub798 \ubc14\ub85c\ud06c \uc591\uc2dd\uc758 \uac74\ucd95\uc778 \ubca0\ub974\uc0ac\uc720 \uad81\uc804\uc744 \ubaa8\ubc29\ud558\uc5ec \uc9c0\uc5b4\uc9c4 \uac83\uc774\uc9c0\ub9cc \uace1\uc120\uc801 \uc678\uc591\uacfc \uac74\ubb3c\uc5d0 \uac00\ud574\uc9c4 \uc7a5\uc2dd\uc131\uc740 \ub85c\ucf54\ucf54 \uc591\uc2dd\uc774\ub77c \ud560 \uc218 \uc788\ub2e4. \uce74\uc2a4\ud2f8\ub9ac\uc624\ub124\ub294 \ub2e4\ub978 \uc608\uc218\ud68c \uc120\uad50\uc0ac\ub4e4\uacfc \ud568\uaed8 \uc124\uacc4\uc640 \uac74\ub9bd \uc791\uc5c5\uc5d0 \uc8fc\ub3c4\uc801\uc73c\ub85c \ucc38\uc5ec\ud558\uc600\ub2e4. \uadf8\uac00 \uc124\uacc4\u00b7\uac10\ub3c5\ud55c \uc11c\uc591\ub8e8(\u897f\u6d0b\u6a13)\ub294 \ubc14\ub85c\ud06c\uc2dd \ub300\ub9ac\uc11d\uc8fc\ub85c \ub41c \ub204\uac01 \uc704\uc5d0 \uc911\uad6d \uc804\ud1b5 \uc804\uac01\uc744 \uc62c\ub9ac\uace0 \uc624\uc0c9 \uc720\ub9ac \uae30\uc640\ub97c \uc5b9\uc5c8\ub294\ub370 \ubcf4\uae30\uc5d0 \uc6c5\uc7a5\ud558\uace0 \ub9e4\uc6b0 \ub3c5\ucc3d\uc801\uc774\uc5c8\ub2e4. \uc11c\uc591\ub8e8\ub294 \uc911\uad6d \ucd5c\ucd08\uc758 \uc778\uacf5 \ubd84\uc218\uac00 \uc124\uce58\ub41c \uacf3\uc774\uae30\ub3c4 \ud558\ub2e4. \uc608\uc218\ud68c \uc120\uad50\uc0ac \ube0c\ub178\uc544\uac00 \uace0\uc548\ud55c \ubd84\uc218 \uc911 \uac00\uc7a5 \uc778\uc0c1\uc801\uc778 \uac83\uc740 \ud574\uc548\ub2f9(\u6d77\u664f\u5802)\uc758 \uc2dc\uacc4 \ubd84\uc218\uc600\ub294\ub370 \uc911\uad6d \uc804\ud1b5\uc5d0\uc11c \uc2dc\uac04\uacfc \ubc29\uc704\ub97c \uc0c1\uc9d5\ud558\ub294 12\ub9c8\ub9ac\uc758 \ub3d9\ubb3c\uc0c1\uc774 \ucc28\ub840\ub85c \uc785\uc5d0\uc11c \ubb3c\uc744 \ubfdc\uc73c\uba74\uc11c \ub9e4\uc2dc \uc2dc\uac04\uc744 \uc54c\ub838\ub2e4\uace0 \ud55c\ub2e4. \uac74\ub96d\uc81c\ub294 \uce74\uc2a4\ud2f8\ub9ac\uc624\ub124\uc5d0\uac8c \uc6d0\uba85\uc6d0 \uc548\uc5d0 \uc5ec\uc758\uad00(\u5982\u610f\u9928)\uc774\ub77c\ub294 \ud654\uc2e4\uc744 \uc9c0\uc5b4 \uc8fc\uc5c8\ub2e4. \uadf8\ub7ec\ub098 \uc6d0\uba85\uc6d0\uc740 1856\ub144(\ud568\ud48d 6\ub144)\uc5d0 \ubc1c\ubc1c\ud55c \uc81c2\ucc28 \uc544\ud3b8\uc804\uc7c1 \ub54c \ud504\ub791\uc2a4\u00b7\uc601\uad6d \uc5f0\ud569\uad70\uc774 \uce68\uc785\ud558\uc5ec \ucca0\uc800\ud558\uac8c \uc57d\ud0c8\ub2f9\ud558\uace0 \ud30c\uad34\ub418\uc5b4 \ud3d0\ud5c8\uac00 \ub418\uc5c8\ub2e4.", "answer": "\uc11c\uc591\ub8e8", "sentence": "\uadf8\uac00 \uc124\uacc4\u00b7\uac10\ub3c5\ud55c \uc11c\uc591\ub8e8 (\u897f\u6d0b\u6a13)", "paragraph_sentence": "\uac74\ub96d\uc81c\ub294 \ub8e8\uc774 14\uc138\uac00 \ubcf4\ub0b4\uc628 \uac00\ube0c\ub9ac\uc5d8 \ud398\ub810(Gabriel P\u00e9relle, 1602\ub144 ~ 1677\ub144)\uc758 \u300a\ud504\ub791\uc2a4\uc758 \uc544\ub984\ub2e4\uc6b4 \uac74\ubb3c\ub4e4\u300b\uacfc \u300a\ud504\ub791\uc2a4 \ub3c4\uc2dc\uc640 \ubcf4\uc2a4\ucf00 \ubc0f \ubd84\uc218\ub97c \ud3ec\ud568\ud55c \ubca0\ub974\uc0ac\uc774\uc720 \uad81\uc804\uc758 \uc124\uacc4\ub3c4\u00b7\ub2e8\uba74\ub3c4\u00b7\uc785\uba74\ub3c4\u300b\uc758 \ub3d9\ud310 \uc0bd\ud654\ub97c \ubcf4\uace0 \ud765\ubbf8\ub97c \ub290\uaf08\uace0, 1747\ub144(\uac74\ub96d 12\ub144)\ubd80\ud130 1759\ub144(\uac74\ub96d 24\ub144)\uae4c\uc9c0 \uc11c\uc591 \uc120\uad50\uc0ac\ub97c \ub3d9\uc6d0\ud574 \ubca0\ub974\uc0ac\uc720 \uad81\uc804\uc744 \ubaa8\ubc29\ud55c \uac74\ubb3c\ub4e4\uc744 \uc9c0\uc5c8\ub2e4. \uc774 \uac74\ubb3c\ub4e4\uc740 \ubcf8\ub798 \ubc14\ub85c\ud06c \uc591\uc2dd\uc758 \uac74\ucd95\uc778 \ubca0\ub974\uc0ac\uc720 \uad81\uc804\uc744 \ubaa8\ubc29\ud558\uc5ec \uc9c0\uc5b4\uc9c4 \uac83\uc774\uc9c0\ub9cc \uace1\uc120\uc801 \uc678\uc591\uacfc \uac74\ubb3c\uc5d0 \uac00\ud574\uc9c4 \uc7a5\uc2dd\uc131\uc740 \ub85c\ucf54\ucf54 \uc591\uc2dd\uc774\ub77c \ud560 \uc218 \uc788\ub2e4. \uce74\uc2a4\ud2f8\ub9ac\uc624\ub124\ub294 \ub2e4\ub978 \uc608\uc218\ud68c \uc120\uad50\uc0ac\ub4e4\uacfc \ud568\uaed8 \uc124\uacc4\uc640 \uac74\ub9bd \uc791\uc5c5\uc5d0 \uc8fc\ub3c4\uc801\uc73c\ub85c \ucc38\uc5ec\ud558\uc600\ub2e4. \uadf8\uac00 \uc124\uacc4\u00b7\uac10\ub3c5\ud55c \uc11c\uc591\ub8e8 (\u897f\u6d0b\u6a13) \ub294 \ubc14\ub85c\ud06c\uc2dd \ub300\ub9ac\uc11d\uc8fc\ub85c \ub41c \ub204\uac01 \uc704\uc5d0 \uc911\uad6d \uc804\ud1b5 \uc804\uac01\uc744 \uc62c\ub9ac\uace0 \uc624\uc0c9 \uc720\ub9ac \uae30\uc640\ub97c \uc5b9\uc5c8\ub294\ub370 \ubcf4\uae30\uc5d0 \uc6c5\uc7a5\ud558\uace0 \ub9e4\uc6b0 \ub3c5\ucc3d\uc801\uc774\uc5c8\ub2e4. \uc11c\uc591\ub8e8\ub294 \uc911\uad6d \ucd5c\ucd08\uc758 \uc778\uacf5 \ubd84\uc218\uac00 \uc124\uce58\ub41c \uacf3\uc774\uae30\ub3c4 \ud558\ub2e4. \uc608\uc218\ud68c \uc120\uad50\uc0ac \ube0c\ub178\uc544\uac00 \uace0\uc548\ud55c \ubd84\uc218 \uc911 \uac00\uc7a5 \uc778\uc0c1\uc801\uc778 \uac83\uc740 \ud574\uc548\ub2f9(\u6d77\u664f\u5802)\uc758 \uc2dc\uacc4 \ubd84\uc218\uc600\ub294\ub370 \uc911\uad6d \uc804\ud1b5\uc5d0\uc11c \uc2dc\uac04\uacfc \ubc29\uc704\ub97c \uc0c1\uc9d5\ud558\ub294 12\ub9c8\ub9ac\uc758 \ub3d9\ubb3c\uc0c1\uc774 \ucc28\ub840\ub85c \uc785\uc5d0\uc11c \ubb3c\uc744 \ubfdc\uc73c\uba74\uc11c \ub9e4\uc2dc \uc2dc\uac04\uc744 \uc54c\ub838\ub2e4\uace0 \ud55c\ub2e4. \uac74\ub96d\uc81c\ub294 \uce74\uc2a4\ud2f8\ub9ac\uc624\ub124\uc5d0\uac8c \uc6d0\uba85\uc6d0 \uc548\uc5d0 \uc5ec\uc758\uad00(\u5982\u610f\u9928)\uc774\ub77c\ub294 \ud654\uc2e4\uc744 \uc9c0\uc5b4 \uc8fc\uc5c8\ub2e4. \uadf8\ub7ec\ub098 \uc6d0\uba85\uc6d0\uc740 1856\ub144(\ud568\ud48d 6\ub144)\uc5d0 \ubc1c\ubc1c\ud55c \uc81c2\ucc28 \uc544\ud3b8\uc804\uc7c1 \ub54c \ud504\ub791\uc2a4\u00b7\uc601\uad6d \uc5f0\ud569\uad70\uc774 \uce68\uc785\ud558\uc5ec \ucca0\uc800\ud558\uac8c \uc57d\ud0c8\ub2f9\ud558\uace0 \ud30c\uad34\ub418\uc5b4 \ud3d0\ud5c8\uac00 \ub418\uc5c8\ub2e4.", "paragraph_answer": "\uac74\ub96d\uc81c\ub294 \ub8e8\uc774 14\uc138\uac00 \ubcf4\ub0b4\uc628 \uac00\ube0c\ub9ac\uc5d8 \ud398\ub810(Gabriel P\u00e9relle, 1602\ub144 ~ 1677\ub144)\uc758 \u300a\ud504\ub791\uc2a4\uc758 \uc544\ub984\ub2e4\uc6b4 \uac74\ubb3c\ub4e4\u300b\uacfc \u300a\ud504\ub791\uc2a4 \ub3c4\uc2dc\uc640 \ubcf4\uc2a4\ucf00 \ubc0f \ubd84\uc218\ub97c \ud3ec\ud568\ud55c \ubca0\ub974\uc0ac\uc774\uc720 \uad81\uc804\uc758 \uc124\uacc4\ub3c4\u00b7\ub2e8\uba74\ub3c4\u00b7\uc785\uba74\ub3c4\u300b\uc758 \ub3d9\ud310 \uc0bd\ud654\ub97c \ubcf4\uace0 \ud765\ubbf8\ub97c \ub290\uaf08\uace0, 1747\ub144(\uac74\ub96d 12\ub144)\ubd80\ud130 1759\ub144(\uac74\ub96d 24\ub144)\uae4c\uc9c0 \uc11c\uc591 \uc120\uad50\uc0ac\ub97c \ub3d9\uc6d0\ud574 \ubca0\ub974\uc0ac\uc720 \uad81\uc804\uc744 \ubaa8\ubc29\ud55c \uac74\ubb3c\ub4e4\uc744 \uc9c0\uc5c8\ub2e4. \uc774 \uac74\ubb3c\ub4e4\uc740 \ubcf8\ub798 \ubc14\ub85c\ud06c \uc591\uc2dd\uc758 \uac74\ucd95\uc778 \ubca0\ub974\uc0ac\uc720 \uad81\uc804\uc744 \ubaa8\ubc29\ud558\uc5ec \uc9c0\uc5b4\uc9c4 \uac83\uc774\uc9c0\ub9cc \uace1\uc120\uc801 \uc678\uc591\uacfc \uac74\ubb3c\uc5d0 \uac00\ud574\uc9c4 \uc7a5\uc2dd\uc131\uc740 \ub85c\ucf54\ucf54 \uc591\uc2dd\uc774\ub77c \ud560 \uc218 \uc788\ub2e4. \uce74\uc2a4\ud2f8\ub9ac\uc624\ub124\ub294 \ub2e4\ub978 \uc608\uc218\ud68c \uc120\uad50\uc0ac\ub4e4\uacfc \ud568\uaed8 \uc124\uacc4\uc640 \uac74\ub9bd \uc791\uc5c5\uc5d0 \uc8fc\ub3c4\uc801\uc73c\ub85c \ucc38\uc5ec\ud558\uc600\ub2e4. \uadf8\uac00 \uc124\uacc4\u00b7\uac10\ub3c5\ud55c \uc11c\uc591\ub8e8 (\u897f\u6d0b\u6a13)\ub294 \ubc14\ub85c\ud06c\uc2dd \ub300\ub9ac\uc11d\uc8fc\ub85c \ub41c \ub204\uac01 \uc704\uc5d0 \uc911\uad6d \uc804\ud1b5 \uc804\uac01\uc744 \uc62c\ub9ac\uace0 \uc624\uc0c9 \uc720\ub9ac \uae30\uc640\ub97c \uc5b9\uc5c8\ub294\ub370 \ubcf4\uae30\uc5d0 \uc6c5\uc7a5\ud558\uace0 \ub9e4\uc6b0 \ub3c5\ucc3d\uc801\uc774\uc5c8\ub2e4. \uc11c\uc591\ub8e8\ub294 \uc911\uad6d \ucd5c\ucd08\uc758 \uc778\uacf5 \ubd84\uc218\uac00 \uc124\uce58\ub41c \uacf3\uc774\uae30\ub3c4 \ud558\ub2e4. \uc608\uc218\ud68c \uc120\uad50\uc0ac \ube0c\ub178\uc544\uac00 \uace0\uc548\ud55c \ubd84\uc218 \uc911 \uac00\uc7a5 \uc778\uc0c1\uc801\uc778 \uac83\uc740 \ud574\uc548\ub2f9(\u6d77\u664f\u5802)\uc758 \uc2dc\uacc4 \ubd84\uc218\uc600\ub294\ub370 \uc911\uad6d \uc804\ud1b5\uc5d0\uc11c \uc2dc\uac04\uacfc \ubc29\uc704\ub97c \uc0c1\uc9d5\ud558\ub294 12\ub9c8\ub9ac\uc758 \ub3d9\ubb3c\uc0c1\uc774 \ucc28\ub840\ub85c \uc785\uc5d0\uc11c \ubb3c\uc744 \ubfdc\uc73c\uba74\uc11c \ub9e4\uc2dc \uc2dc\uac04\uc744 \uc54c\ub838\ub2e4\uace0 \ud55c\ub2e4. \uac74\ub96d\uc81c\ub294 \uce74\uc2a4\ud2f8\ub9ac\uc624\ub124\uc5d0\uac8c \uc6d0\uba85\uc6d0 \uc548\uc5d0 \uc5ec\uc758\uad00(\u5982\u610f\u9928)\uc774\ub77c\ub294 \ud654\uc2e4\uc744 \uc9c0\uc5b4 \uc8fc\uc5c8\ub2e4. \uadf8\ub7ec\ub098 \uc6d0\uba85\uc6d0\uc740 1856\ub144(\ud568\ud48d 6\ub144)\uc5d0 \ubc1c\ubc1c\ud55c \uc81c2\ucc28 \uc544\ud3b8\uc804\uc7c1 \ub54c \ud504\ub791\uc2a4\u00b7\uc601\uad6d \uc5f0\ud569\uad70\uc774 \uce68\uc785\ud558\uc5ec \ucca0\uc800\ud558\uac8c \uc57d\ud0c8\ub2f9\ud558\uace0 \ud30c\uad34\ub418\uc5b4 \ud3d0\ud5c8\uac00 \ub418\uc5c8\ub2e4.", "sentence_answer": "\uadf8\uac00 \uc124\uacc4\u00b7\uac10\ub3c5\ud55c \uc11c\uc591\ub8e8 (\u897f\u6d0b\u6a13)", "paragraph_id": "6555980-6-0", "paragraph_question": "question: \uc911\uad6d \ucd5c\ucd08\uc758 \uc778\uacf5 \ubd84\uc218\uac00 \uc124\uce58\ub41c \uacf3\uc740?, context: \uac74\ub96d\uc81c\ub294 \ub8e8\uc774 14\uc138\uac00 \ubcf4\ub0b4\uc628 \uac00\ube0c\ub9ac\uc5d8 \ud398\ub810(Gabriel P\u00e9relle, 1602\ub144 ~ 1677\ub144)\uc758 \u300a\ud504\ub791\uc2a4\uc758 \uc544\ub984\ub2e4\uc6b4 \uac74\ubb3c\ub4e4\u300b\uacfc \u300a\ud504\ub791\uc2a4 \ub3c4\uc2dc\uc640 \ubcf4\uc2a4\ucf00 \ubc0f \ubd84\uc218\ub97c \ud3ec\ud568\ud55c \ubca0\ub974\uc0ac\uc774\uc720 \uad81\uc804\uc758 \uc124\uacc4\ub3c4\u00b7\ub2e8\uba74\ub3c4\u00b7\uc785\uba74\ub3c4\u300b\uc758 \ub3d9\ud310 \uc0bd\ud654\ub97c \ubcf4\uace0 \ud765\ubbf8\ub97c \ub290\uaf08\uace0, 1747\ub144(\uac74\ub96d 12\ub144)\ubd80\ud130 1759\ub144(\uac74\ub96d 24\ub144)\uae4c\uc9c0 \uc11c\uc591 \uc120\uad50\uc0ac\ub97c \ub3d9\uc6d0\ud574 \ubca0\ub974\uc0ac\uc720 \uad81\uc804\uc744 \ubaa8\ubc29\ud55c \uac74\ubb3c\ub4e4\uc744 \uc9c0\uc5c8\ub2e4. \uc774 \uac74\ubb3c\ub4e4\uc740 \ubcf8\ub798 \ubc14\ub85c\ud06c \uc591\uc2dd\uc758 \uac74\ucd95\uc778 \ubca0\ub974\uc0ac\uc720 \uad81\uc804\uc744 \ubaa8\ubc29\ud558\uc5ec \uc9c0\uc5b4\uc9c4 \uac83\uc774\uc9c0\ub9cc \uace1\uc120\uc801 \uc678\uc591\uacfc \uac74\ubb3c\uc5d0 \uac00\ud574\uc9c4 \uc7a5\uc2dd\uc131\uc740 \ub85c\ucf54\ucf54 \uc591\uc2dd\uc774\ub77c \ud560 \uc218 \uc788\ub2e4. \uce74\uc2a4\ud2f8\ub9ac\uc624\ub124\ub294 \ub2e4\ub978 \uc608\uc218\ud68c \uc120\uad50\uc0ac\ub4e4\uacfc \ud568\uaed8 \uc124\uacc4\uc640 \uac74\ub9bd \uc791\uc5c5\uc5d0 \uc8fc\ub3c4\uc801\uc73c\ub85c \ucc38\uc5ec\ud558\uc600\ub2e4. \uadf8\uac00 \uc124\uacc4\u00b7\uac10\ub3c5\ud55c \uc11c\uc591\ub8e8(\u897f\u6d0b\u6a13)\ub294 \ubc14\ub85c\ud06c\uc2dd \ub300\ub9ac\uc11d\uc8fc\ub85c \ub41c \ub204\uac01 \uc704\uc5d0 \uc911\uad6d \uc804\ud1b5 \uc804\uac01\uc744 \uc62c\ub9ac\uace0 \uc624\uc0c9 \uc720\ub9ac \uae30\uc640\ub97c \uc5b9\uc5c8\ub294\ub370 \ubcf4\uae30\uc5d0 \uc6c5\uc7a5\ud558\uace0 \ub9e4\uc6b0 \ub3c5\ucc3d\uc801\uc774\uc5c8\ub2e4. \uc11c\uc591\ub8e8\ub294 \uc911\uad6d \ucd5c\ucd08\uc758 \uc778\uacf5 \ubd84\uc218\uac00 \uc124\uce58\ub41c \uacf3\uc774\uae30\ub3c4 \ud558\ub2e4. \uc608\uc218\ud68c \uc120\uad50\uc0ac \ube0c\ub178\uc544\uac00 \uace0\uc548\ud55c \ubd84\uc218 \uc911 \uac00\uc7a5 \uc778\uc0c1\uc801\uc778 \uac83\uc740 \ud574\uc548\ub2f9(\u6d77\u664f\u5802)\uc758 \uc2dc\uacc4 \ubd84\uc218\uc600\ub294\ub370 \uc911\uad6d \uc804\ud1b5\uc5d0\uc11c \uc2dc\uac04\uacfc \ubc29\uc704\ub97c \uc0c1\uc9d5\ud558\ub294 12\ub9c8\ub9ac\uc758 \ub3d9\ubb3c\uc0c1\uc774 \ucc28\ub840\ub85c \uc785\uc5d0\uc11c \ubb3c\uc744 \ubfdc\uc73c\uba74\uc11c \ub9e4\uc2dc \uc2dc\uac04\uc744 \uc54c\ub838\ub2e4\uace0 \ud55c\ub2e4. \uac74\ub96d\uc81c\ub294 \uce74\uc2a4\ud2f8\ub9ac\uc624\ub124\uc5d0\uac8c \uc6d0\uba85\uc6d0 \uc548\uc5d0 \uc5ec\uc758\uad00(\u5982\u610f\u9928)\uc774\ub77c\ub294 \ud654\uc2e4\uc744 \uc9c0\uc5b4 \uc8fc\uc5c8\ub2e4. \uadf8\ub7ec\ub098 \uc6d0\uba85\uc6d0\uc740 1856\ub144(\ud568\ud48d 6\ub144)\uc5d0 \ubc1c\ubc1c\ud55c \uc81c2\ucc28 \uc544\ud3b8\uc804\uc7c1 \ub54c \ud504\ub791\uc2a4\u00b7\uc601\uad6d \uc5f0\ud569\uad70\uc774 \uce68\uc785\ud558\uc5ec \ucca0\uc800\ud558\uac8c \uc57d\ud0c8\ub2f9\ud558\uace0 \ud30c\uad34\ub418\uc5b4 \ud3d0\ud5c8\uac00 \ub418\uc5c8\ub2e4."} {"question": "\ubc15\ubcf4\uc601\uc774 \ud0dc\uc5b4\ub09c \uc5f0\ub3c4\ub294?", "paragraph": "\ubc15\ubcf4\uc601\uc740 1990\ub144 2\uc6d4 12\uc77c\uc5d0 \ucda9\uccad\ubd81\ub3c4 \uc99d\ud3c9\uc5d0\uc11c \u524d \uc721\uad70 \ud2b9\uc218\uc804\uc0ac\ub839\ubd80 \ubd80\uc0ac\uad00 \ubc15\uc644\uc218 \uc608\ube44\uc5ed \ub300\ud55c\ubbfc\uad6d \uc721\uad70 \uc6d0\uc0ac\uc758 3\ub140 \uc911 \ucc28\ub140\ub85c \ud0dc\uc5b4\ub0ac\ub2e4. \uadf8\ub294 \uc99d\ud3c9\ucd08\ub4f1\ud559\uad50, \uc99d\ud3c9\uc5ec\uc790\uc911\ud559\uad50, \ub300\uc131\uc5ec\uc790\uc0c1\uc5c5\uace0\ub4f1\ud559\uad50\ub97c \uc878\uc5c5\ud558\uace0, \ub2e8\uad6d\ub300\ud559\uad50 \uc608\uc220\uc870\ud615\ub300\ud559 \uacf5\uc5f0\uc601\ud654\ud559\ubd80 \uc5f0\uadf9\uc804\uacf5 2008\ub144 \uc815\uc2dc \ubaa8\uc9d1 \uc804\ud615\uc5d0 \uc9c0\uc6d0\ud558\uc5ec \ud569\uaca9\ud588\ub2e4. \uad70\uc778\uc778 \uc544\ubc84\uc9c0 \uc601\ud5a5\uc73c\ub85c \uc2dc\uac04\uad00\ub150\uc774 \ucca0\uc800\ud55c \ud3b8\uc774\uba70 \uc2dc\uace8\uc5d0\uc11c \ub098\uace0 \uc790\ub790\ub2e4. \uce5c\uad6c\ub4e4\uc774 \uac70\uc758 \uad70\uc778 \uc790\ub140\uc778 \ud0d3\uc5d0, \uc544\ubc84\uc9c0\ub4e4 \ubc1c\ub839\uc5d0 \ub530\ub77c \ubfd4\ubfd4\uc774 \ud769\uc5b4\uc9c0\uba74 \uc54c\uac8c \ub41c \uc8fc\uc18c\ub97c \ud1b5\ud574 \uc11c\ub85c \uc190\ud3b8\uc9c0\ub97c \uc790\uc8fc \uc8fc\uace0 \ubc1b\uc558\ub2e4\uace0 \ud55c\ub2e4. \uadf8\ub54c \ud39c\ud314\uc744 \uc37c\ub358 \uacbd\ud5d8 \ub4f1\uc744 \ud1b5\ud574\uc11c \uac00\uc9c0\uac8c \ub41c \uc544\ub0a0\ub85c\uadf8 \uac10\uc131\uc774 \uc5f0\uae30\uc5d0 \ud070 \uc790\uc591\ubd84\uc774 \ub410\ub2e4\uba70 \uc2a4\uc2a4\ub85c \ucd95\ubcf5\ubc1b\uc558\ub2e4\uace0 \uc5ec\uae34\ub2e4\uace0 \ud55c\ub2e4.", "answer": "1990\ub144", "sentence": "\ubc15\ubcf4\uc601\uc740 1990\ub144 2\uc6d4 12\uc77c\uc5d0 \ucda9\uccad\ubd81\ub3c4 \uc99d\ud3c9\uc5d0\uc11c \u524d \uc721\uad70 \ud2b9\uc218\uc804\uc0ac\ub839\ubd80 \ubd80\uc0ac\uad00 \ubc15\uc644\uc218 \uc608\ube44\uc5ed \ub300\ud55c\ubbfc\uad6d \uc721\uad70 \uc6d0\uc0ac\uc758 3\ub140 \uc911 \ucc28\ub140\ub85c \ud0dc\uc5b4\ub0ac\ub2e4.", "paragraph_sentence": " \ubc15\ubcf4\uc601\uc740 1990\ub144 2\uc6d4 12\uc77c\uc5d0 \ucda9\uccad\ubd81\ub3c4 \uc99d\ud3c9\uc5d0\uc11c \u524d \uc721\uad70 \ud2b9\uc218\uc804\uc0ac\ub839\ubd80 \ubd80\uc0ac\uad00 \ubc15\uc644\uc218 \uc608\ube44\uc5ed \ub300\ud55c\ubbfc\uad6d \uc721\uad70 \uc6d0\uc0ac\uc758 3\ub140 \uc911 \ucc28\ub140\ub85c \ud0dc\uc5b4\ub0ac\ub2e4. \uadf8\ub294 \uc99d\ud3c9\ucd08\ub4f1\ud559\uad50, \uc99d\ud3c9\uc5ec\uc790\uc911\ud559\uad50, \ub300\uc131\uc5ec\uc790\uc0c1\uc5c5\uace0\ub4f1\ud559\uad50\ub97c \uc878\uc5c5\ud558\uace0, \ub2e8\uad6d\ub300\ud559\uad50 \uc608\uc220\uc870\ud615\ub300\ud559 \uacf5\uc5f0\uc601\ud654\ud559\ubd80 \uc5f0\uadf9\uc804\uacf5 2008\ub144 \uc815\uc2dc \ubaa8\uc9d1 \uc804\ud615\uc5d0 \uc9c0\uc6d0\ud558\uc5ec \ud569\uaca9\ud588\ub2e4. \uad70\uc778\uc778 \uc544\ubc84\uc9c0 \uc601\ud5a5\uc73c\ub85c \uc2dc\uac04\uad00\ub150\uc774 \ucca0\uc800\ud55c \ud3b8\uc774\uba70 \uc2dc\uace8\uc5d0\uc11c \ub098\uace0 \uc790\ub790\ub2e4. \uce5c\uad6c\ub4e4\uc774 \uac70\uc758 \uad70\uc778 \uc790\ub140\uc778 \ud0d3\uc5d0, \uc544\ubc84\uc9c0\ub4e4 \ubc1c\ub839\uc5d0 \ub530\ub77c \ubfd4\ubfd4\uc774 \ud769\uc5b4\uc9c0\uba74 \uc54c\uac8c \ub41c \uc8fc\uc18c\ub97c \ud1b5\ud574 \uc11c\ub85c \uc190\ud3b8\uc9c0\ub97c \uc790\uc8fc \uc8fc\uace0 \ubc1b\uc558\ub2e4\uace0 \ud55c\ub2e4. \uadf8\ub54c \ud39c\ud314\uc744 \uc37c\ub358 \uacbd\ud5d8 \ub4f1\uc744 \ud1b5\ud574\uc11c \uac00\uc9c0\uac8c \ub41c \uc544\ub0a0\ub85c\uadf8 \uac10\uc131\uc774 \uc5f0\uae30\uc5d0 \ud070 \uc790\uc591\ubd84\uc774 \ub410\ub2e4\uba70 \uc2a4\uc2a4\ub85c \ucd95\ubcf5\ubc1b\uc558\ub2e4\uace0 \uc5ec\uae34\ub2e4\uace0 \ud55c\ub2e4.", "paragraph_answer": "\ubc15\ubcf4\uc601\uc740 1990\ub144 2\uc6d4 12\uc77c\uc5d0 \ucda9\uccad\ubd81\ub3c4 \uc99d\ud3c9\uc5d0\uc11c \u524d \uc721\uad70 \ud2b9\uc218\uc804\uc0ac\ub839\ubd80 \ubd80\uc0ac\uad00 \ubc15\uc644\uc218 \uc608\ube44\uc5ed \ub300\ud55c\ubbfc\uad6d \uc721\uad70 \uc6d0\uc0ac\uc758 3\ub140 \uc911 \ucc28\ub140\ub85c \ud0dc\uc5b4\ub0ac\ub2e4. \uadf8\ub294 \uc99d\ud3c9\ucd08\ub4f1\ud559\uad50, \uc99d\ud3c9\uc5ec\uc790\uc911\ud559\uad50, \ub300\uc131\uc5ec\uc790\uc0c1\uc5c5\uace0\ub4f1\ud559\uad50\ub97c \uc878\uc5c5\ud558\uace0, \ub2e8\uad6d\ub300\ud559\uad50 \uc608\uc220\uc870\ud615\ub300\ud559 \uacf5\uc5f0\uc601\ud654\ud559\ubd80 \uc5f0\uadf9\uc804\uacf5 2008\ub144 \uc815\uc2dc \ubaa8\uc9d1 \uc804\ud615\uc5d0 \uc9c0\uc6d0\ud558\uc5ec \ud569\uaca9\ud588\ub2e4. \uad70\uc778\uc778 \uc544\ubc84\uc9c0 \uc601\ud5a5\uc73c\ub85c \uc2dc\uac04\uad00\ub150\uc774 \ucca0\uc800\ud55c \ud3b8\uc774\uba70 \uc2dc\uace8\uc5d0\uc11c \ub098\uace0 \uc790\ub790\ub2e4. \uce5c\uad6c\ub4e4\uc774 \uac70\uc758 \uad70\uc778 \uc790\ub140\uc778 \ud0d3\uc5d0, \uc544\ubc84\uc9c0\ub4e4 \ubc1c\ub839\uc5d0 \ub530\ub77c \ubfd4\ubfd4\uc774 \ud769\uc5b4\uc9c0\uba74 \uc54c\uac8c \ub41c \uc8fc\uc18c\ub97c \ud1b5\ud574 \uc11c\ub85c \uc190\ud3b8\uc9c0\ub97c \uc790\uc8fc \uc8fc\uace0 \ubc1b\uc558\ub2e4\uace0 \ud55c\ub2e4. \uadf8\ub54c \ud39c\ud314\uc744 \uc37c\ub358 \uacbd\ud5d8 \ub4f1\uc744 \ud1b5\ud574\uc11c \uac00\uc9c0\uac8c \ub41c \uc544\ub0a0\ub85c\uadf8 \uac10\uc131\uc774 \uc5f0\uae30\uc5d0 \ud070 \uc790\uc591\ubd84\uc774 \ub410\ub2e4\uba70 \uc2a4\uc2a4\ub85c \ucd95\ubcf5\ubc1b\uc558\ub2e4\uace0 \uc5ec\uae34\ub2e4\uace0 \ud55c\ub2e4.", "sentence_answer": "\ubc15\ubcf4\uc601\uc740 1990\ub144 2\uc6d4 12\uc77c\uc5d0 \ucda9\uccad\ubd81\ub3c4 \uc99d\ud3c9\uc5d0\uc11c \u524d \uc721\uad70 \ud2b9\uc218\uc804\uc0ac\ub839\ubd80 \ubd80\uc0ac\uad00 \ubc15\uc644\uc218 \uc608\ube44\uc5ed \ub300\ud55c\ubbfc\uad6d \uc721\uad70 \uc6d0\uc0ac\uc758 3\ub140 \uc911 \ucc28\ub140\ub85c \ud0dc\uc5b4\ub0ac\ub2e4.", "paragraph_id": "6534780-1-0", "paragraph_question": "question: \ubc15\ubcf4\uc601\uc774 \ud0dc\uc5b4\ub09c \uc5f0\ub3c4\ub294?, context: \ubc15\ubcf4\uc601\uc740 1990\ub144 2\uc6d4 12\uc77c\uc5d0 \ucda9\uccad\ubd81\ub3c4 \uc99d\ud3c9\uc5d0\uc11c \u524d \uc721\uad70 \ud2b9\uc218\uc804\uc0ac\ub839\ubd80 \ubd80\uc0ac\uad00 \ubc15\uc644\uc218 \uc608\ube44\uc5ed \ub300\ud55c\ubbfc\uad6d \uc721\uad70 \uc6d0\uc0ac\uc758 3\ub140 \uc911 \ucc28\ub140\ub85c \ud0dc\uc5b4\ub0ac\ub2e4. \uadf8\ub294 \uc99d\ud3c9\ucd08\ub4f1\ud559\uad50, \uc99d\ud3c9\uc5ec\uc790\uc911\ud559\uad50, \ub300\uc131\uc5ec\uc790\uc0c1\uc5c5\uace0\ub4f1\ud559\uad50\ub97c \uc878\uc5c5\ud558\uace0, \ub2e8\uad6d\ub300\ud559\uad50 \uc608\uc220\uc870\ud615\ub300\ud559 \uacf5\uc5f0\uc601\ud654\ud559\ubd80 \uc5f0\uadf9\uc804\uacf5 2008\ub144 \uc815\uc2dc \ubaa8\uc9d1 \uc804\ud615\uc5d0 \uc9c0\uc6d0\ud558\uc5ec \ud569\uaca9\ud588\ub2e4. \uad70\uc778\uc778 \uc544\ubc84\uc9c0 \uc601\ud5a5\uc73c\ub85c \uc2dc\uac04\uad00\ub150\uc774 \ucca0\uc800\ud55c \ud3b8\uc774\uba70 \uc2dc\uace8\uc5d0\uc11c \ub098\uace0 \uc790\ub790\ub2e4. \uce5c\uad6c\ub4e4\uc774 \uac70\uc758 \uad70\uc778 \uc790\ub140\uc778 \ud0d3\uc5d0, \uc544\ubc84\uc9c0\ub4e4 \ubc1c\ub839\uc5d0 \ub530\ub77c \ubfd4\ubfd4\uc774 \ud769\uc5b4\uc9c0\uba74 \uc54c\uac8c \ub41c \uc8fc\uc18c\ub97c \ud1b5\ud574 \uc11c\ub85c \uc190\ud3b8\uc9c0\ub97c \uc790\uc8fc \uc8fc\uace0 \ubc1b\uc558\ub2e4\uace0 \ud55c\ub2e4. \uadf8\ub54c \ud39c\ud314\uc744 \uc37c\ub358 \uacbd\ud5d8 \ub4f1\uc744 \ud1b5\ud574\uc11c \uac00\uc9c0\uac8c \ub41c \uc544\ub0a0\ub85c\uadf8 \uac10\uc131\uc774 \uc5f0\uae30\uc5d0 \ud070 \uc790\uc591\ubd84\uc774 \ub410\ub2e4\uba70 \uc2a4\uc2a4\ub85c \ucd95\ubcf5\ubc1b\uc558\ub2e4\uace0 \uc5ec\uae34\ub2e4\uace0 \ud55c\ub2e4."} {"question": "\uc9c4\uc740 \ubb34\uc5c7\uc744 \ubaa9\ud45c\ub85c \ud558\uace0 \uc788\uc5c8\ub098?", "paragraph": "\ub2f9\uc2dc \uc591\uad6d \uc870\uc815\uc740 \ub098\ub984\ub300\ub85c \ucd08\uc870\ud588\ub2e4. \ub2f9\uc2dc \uc0c1\ud669\uc774 \uc870\uc5d0\uac8c \uc720\ub9ac\ud558\uac8c \ub3cc\uc544\uac00\uace0 \uc788\uc5c8\uc74c\uc5d0\ub3c4 \ubd88\uad6c\ud558\uace0 \ud6a8\uc131\uc655\uc740 \ub2e4\uae09\ud588\ub2e4. \ubc29\uc5b4\ub9cc \ud558\uace0 \uc788\ub294 \uc5fc\ud30c\uac00 \uc790\uc2e0\uc758 \uc790\uc874\uc2ec\uc744 \ub5a8\uc5b4\ub728\ub9ac\uace0 \uc788\ub2e4\uace0 \ubbff\uc5c8\uae30 \ub54c\ubb38\uc774\uc5c8\ub2e4. \ub610\ud55c \ucc9c\ud558\ud1b5\uc77c\uc744 \ubaa9\ud45c\ub85c \ud558\uace0 \uc788\ub294 \uc9c4\uc740 \uc774 \uc2f8\uc6c0\uc5d0\uc11c \ubc00\ub9ac\uba74 6\uad6d\uc5d0 \uc704\uc2e0\uc774 \ub5a8\uc5b4\uc9c0\uac8c \ub418\uba70, \ud1b5\uc77c\uc744 \ud5a5\ud55c \uae38\uc740 \ub354\uc6b1 \ud5d8\ub09c\ud574\uc9c8 \uc218 \uc788\uc5c8\uae30 \ub54c\ubb38\uc5d0 \ubc18\ub4dc\uc2dc \uc774\uaca8\uc57c\ub9cc \ud588\ub2e4. \uac8c\ub2e4\uac00 \ub2f9\uc2dc \uc7ac\uc0c1\uc774\uc5c8\ub358 \ubc94\uc218\ub294 \uc774\ubbf8 \uba87 \ucc28\ub840\uc758 \uc2e4\ucc45\uc73c\ub85c \uc9d5\uacc4\ub97c \ubc1b\uc558\ub2e4. \uc774\ubc88 \uc804\ud22c\uc5d0\uc11c \ud328\ubc30\ud55c\ub2e4\uba74 \uadf8\uc758 \ubab0\ub77d\uc740 \uc790\uba85\ud55c \uc77c\uc774\uc5c8\ub2e4. \uc9c4\uc740 \ubc1c\ube60\ub974\uac8c \uc6c0\uc9c1\uc600\ub2e4. \ubc94\uc218\ub294 \ub450 \uac00\uc9c0 \uc2b9\ubd80\uc218\ub97c \ub744\uc6e0\ub2e4. \uc6b0\uc120 \ub2f9\uc2dc \uc9c0\ud718\uad00\uc774\uc5c8\ub358 \uc655\ud758\uc744 \ubab0\ub798 \ubc31\uc804\ub178\uc7a5 \ubc31\uae30\ub85c \uad50\uccb4\ud558\uc600\ub2e4. \uadf8\ub9ac\uace0 \uc774\ub97c \ubc1c\uc124\ud558\ub294 \uc790\ub294 \uac00\ucc28\uc5c6\uc774 \ucc98\ud615\ud558\uc600\ub2e4. \uc774\ub97c \uc870\uc5d0\uc11c \uc54c\uba74 \uc548 \ub418\uae30 \ub54c\ubb38\uc774\uc5c8\ub2e4. \ub610\ud55c \uc5fc\ud30c\uac00 \uc804\uc7a5\uc744 \uc9c0\ud718\ud558\uba74 \uc2b9\ub9ac\ud560 \uc218 \uc5c6\uc74c\uc744 \uc54c\uc558\uae30\uc5d0 \uc774\uac04\ucc45\uc744 \uc368\uc11c \uc9c0\ud718\uad00\uc744 \ubc14\uafb8\uae30\ub85c \ud558\uace0 \uc870\ub098\ub77c \ub0b4\ubd80\uc5d0 \ucca9\uc790\ub4e4\uc744 \ud22c\uc785\ud558\uc5ec \uc774\ub7ec\ud55c \uc18c\ubb38\uc744 \ud37c\ub728\ub9ac\uac8c \ud558\uc600\ub2e4. \"\uc5fc\ud30c\uac00 \uc9c4\uc744 \uacf5\uaca9\ud558\uc9c0 \uc54a\ub294 \uac83\uc740 \ub4a4\uc5d0\uc11c \uc9c4\uacfc \ub0b4\ud1b5\ud558\uace0 \uc788\uae30 \ub54c\ubb38\uc774\ub2e4\" \"\uc9c4\uc774 \ub450\ub824\uc6cc\ud558\ub294 \uac83\uc740 \uc870\uad04\uc774 \uc7a5\uad70\uc774 \ub418\ub294 \uac83\uc774\ub2e4\" \uc774\ub7ec\ud55c \uc18c\ubb38\uc744 \ub4e4\uc740 \ud6a8\uc131\uc655\uc740 \uc5fc\ud30c\ub97c \uc9c0\ud718\uad00 \uc790\ub9ac\uc5d0\uc11c \ub0b4\uce58\uace0 \ub300\uc2e0 \uc870\uad04\uc744 \uc784\uba85\ud558\uc600\ub2e4.", "answer": "\ucc9c\ud558\ud1b5\uc77c", "sentence": "\ub610\ud55c \ucc9c\ud558\ud1b5\uc77c \uc744 \ubaa9\ud45c\ub85c \ud558\uace0 \uc788\ub294 \uc9c4\uc740 \uc774 \uc2f8\uc6c0\uc5d0\uc11c \ubc00\ub9ac\uba74 6\uad6d\uc5d0 \uc704\uc2e0\uc774 \ub5a8\uc5b4\uc9c0\uac8c \ub418\uba70, \ud1b5\uc77c\uc744 \ud5a5\ud55c \uae38\uc740 \ub354\uc6b1 \ud5d8\ub09c\ud574\uc9c8 \uc218 \uc788\uc5c8\uae30 \ub54c\ubb38\uc5d0 \ubc18\ub4dc\uc2dc \uc774\uaca8\uc57c\ub9cc \ud588\ub2e4.", "paragraph_sentence": "\ub2f9\uc2dc \uc591\uad6d \uc870\uc815\uc740 \ub098\ub984\ub300\ub85c \ucd08\uc870\ud588\ub2e4. \ub2f9\uc2dc \uc0c1\ud669\uc774 \uc870\uc5d0\uac8c \uc720\ub9ac\ud558\uac8c \ub3cc\uc544\uac00\uace0 \uc788\uc5c8\uc74c\uc5d0\ub3c4 \ubd88\uad6c\ud558\uace0 \ud6a8\uc131\uc655\uc740 \ub2e4\uae09\ud588\ub2e4. \ubc29\uc5b4\ub9cc \ud558\uace0 \uc788\ub294 \uc5fc\ud30c\uac00 \uc790\uc2e0\uc758 \uc790\uc874\uc2ec\uc744 \ub5a8\uc5b4\ub728\ub9ac\uace0 \uc788\ub2e4\uace0 \ubbff\uc5c8\uae30 \ub54c\ubb38\uc774\uc5c8\ub2e4. \ub610\ud55c \ucc9c\ud558\ud1b5\uc77c \uc744 \ubaa9\ud45c\ub85c \ud558\uace0 \uc788\ub294 \uc9c4\uc740 \uc774 \uc2f8\uc6c0\uc5d0\uc11c \ubc00\ub9ac\uba74 6\uad6d\uc5d0 \uc704\uc2e0\uc774 \ub5a8\uc5b4\uc9c0\uac8c \ub418\uba70, \ud1b5\uc77c\uc744 \ud5a5\ud55c \uae38\uc740 \ub354\uc6b1 \ud5d8\ub09c\ud574\uc9c8 \uc218 \uc788\uc5c8\uae30 \ub54c\ubb38\uc5d0 \ubc18\ub4dc\uc2dc \uc774\uaca8\uc57c\ub9cc \ud588\ub2e4. \uac8c\ub2e4\uac00 \ub2f9\uc2dc \uc7ac\uc0c1\uc774\uc5c8\ub358 \ubc94\uc218\ub294 \uc774\ubbf8 \uba87 \ucc28\ub840\uc758 \uc2e4\ucc45\uc73c\ub85c \uc9d5\uacc4\ub97c \ubc1b\uc558\ub2e4. \uc774\ubc88 \uc804\ud22c\uc5d0\uc11c \ud328\ubc30\ud55c\ub2e4\uba74 \uadf8\uc758 \ubab0\ub77d\uc740 \uc790\uba85\ud55c \uc77c\uc774\uc5c8\ub2e4. \uc9c4\uc740 \ubc1c\ube60\ub974\uac8c \uc6c0\uc9c1\uc600\ub2e4. \ubc94\uc218\ub294 \ub450 \uac00\uc9c0 \uc2b9\ubd80\uc218\ub97c \ub744\uc6e0\ub2e4. \uc6b0\uc120 \ub2f9\uc2dc \uc9c0\ud718\uad00\uc774\uc5c8\ub358 \uc655\ud758\uc744 \ubab0\ub798 \ubc31\uc804\ub178\uc7a5 \ubc31\uae30\ub85c \uad50\uccb4\ud558\uc600\ub2e4. \uadf8\ub9ac\uace0 \uc774\ub97c \ubc1c\uc124\ud558\ub294 \uc790\ub294 \uac00\ucc28\uc5c6\uc774 \ucc98\ud615\ud558\uc600\ub2e4. \uc774\ub97c \uc870\uc5d0\uc11c \uc54c\uba74 \uc548 \ub418\uae30 \ub54c\ubb38\uc774\uc5c8\ub2e4. \ub610\ud55c \uc5fc\ud30c\uac00 \uc804\uc7a5\uc744 \uc9c0\ud718\ud558\uba74 \uc2b9\ub9ac\ud560 \uc218 \uc5c6\uc74c\uc744 \uc54c\uc558\uae30\uc5d0 \uc774\uac04\ucc45\uc744 \uc368\uc11c \uc9c0\ud718\uad00\uc744 \ubc14\uafb8\uae30\ub85c \ud558\uace0 \uc870\ub098\ub77c \ub0b4\ubd80\uc5d0 \ucca9\uc790\ub4e4\uc744 \ud22c\uc785\ud558\uc5ec \uc774\ub7ec\ud55c \uc18c\ubb38\uc744 \ud37c\ub728\ub9ac\uac8c \ud558\uc600\ub2e4. \"\uc5fc\ud30c\uac00 \uc9c4\uc744 \uacf5\uaca9\ud558\uc9c0 \uc54a\ub294 \uac83\uc740 \ub4a4\uc5d0\uc11c \uc9c4\uacfc \ub0b4\ud1b5\ud558\uace0 \uc788\uae30 \ub54c\ubb38\uc774\ub2e4\" \"\uc9c4\uc774 \ub450\ub824\uc6cc\ud558\ub294 \uac83\uc740 \uc870\uad04\uc774 \uc7a5\uad70\uc774 \ub418\ub294 \uac83\uc774\ub2e4\" \uc774\ub7ec\ud55c \uc18c\ubb38\uc744 \ub4e4\uc740 \ud6a8\uc131\uc655\uc740 \uc5fc\ud30c\ub97c \uc9c0\ud718\uad00 \uc790\ub9ac\uc5d0\uc11c \ub0b4\uce58\uace0 \ub300\uc2e0 \uc870\uad04\uc744 \uc784\uba85\ud558\uc600\ub2e4.", "paragraph_answer": "\ub2f9\uc2dc \uc591\uad6d \uc870\uc815\uc740 \ub098\ub984\ub300\ub85c \ucd08\uc870\ud588\ub2e4. \ub2f9\uc2dc \uc0c1\ud669\uc774 \uc870\uc5d0\uac8c \uc720\ub9ac\ud558\uac8c \ub3cc\uc544\uac00\uace0 \uc788\uc5c8\uc74c\uc5d0\ub3c4 \ubd88\uad6c\ud558\uace0 \ud6a8\uc131\uc655\uc740 \ub2e4\uae09\ud588\ub2e4. \ubc29\uc5b4\ub9cc \ud558\uace0 \uc788\ub294 \uc5fc\ud30c\uac00 \uc790\uc2e0\uc758 \uc790\uc874\uc2ec\uc744 \ub5a8\uc5b4\ub728\ub9ac\uace0 \uc788\ub2e4\uace0 \ubbff\uc5c8\uae30 \ub54c\ubb38\uc774\uc5c8\ub2e4. \ub610\ud55c \ucc9c\ud558\ud1b5\uc77c \uc744 \ubaa9\ud45c\ub85c \ud558\uace0 \uc788\ub294 \uc9c4\uc740 \uc774 \uc2f8\uc6c0\uc5d0\uc11c \ubc00\ub9ac\uba74 6\uad6d\uc5d0 \uc704\uc2e0\uc774 \ub5a8\uc5b4\uc9c0\uac8c \ub418\uba70, \ud1b5\uc77c\uc744 \ud5a5\ud55c \uae38\uc740 \ub354\uc6b1 \ud5d8\ub09c\ud574\uc9c8 \uc218 \uc788\uc5c8\uae30 \ub54c\ubb38\uc5d0 \ubc18\ub4dc\uc2dc \uc774\uaca8\uc57c\ub9cc \ud588\ub2e4. \uac8c\ub2e4\uac00 \ub2f9\uc2dc \uc7ac\uc0c1\uc774\uc5c8\ub358 \ubc94\uc218\ub294 \uc774\ubbf8 \uba87 \ucc28\ub840\uc758 \uc2e4\ucc45\uc73c\ub85c \uc9d5\uacc4\ub97c \ubc1b\uc558\ub2e4. \uc774\ubc88 \uc804\ud22c\uc5d0\uc11c \ud328\ubc30\ud55c\ub2e4\uba74 \uadf8\uc758 \ubab0\ub77d\uc740 \uc790\uba85\ud55c \uc77c\uc774\uc5c8\ub2e4. \uc9c4\uc740 \ubc1c\ube60\ub974\uac8c \uc6c0\uc9c1\uc600\ub2e4. \ubc94\uc218\ub294 \ub450 \uac00\uc9c0 \uc2b9\ubd80\uc218\ub97c \ub744\uc6e0\ub2e4. \uc6b0\uc120 \ub2f9\uc2dc \uc9c0\ud718\uad00\uc774\uc5c8\ub358 \uc655\ud758\uc744 \ubab0\ub798 \ubc31\uc804\ub178\uc7a5 \ubc31\uae30\ub85c \uad50\uccb4\ud558\uc600\ub2e4. \uadf8\ub9ac\uace0 \uc774\ub97c \ubc1c\uc124\ud558\ub294 \uc790\ub294 \uac00\ucc28\uc5c6\uc774 \ucc98\ud615\ud558\uc600\ub2e4. \uc774\ub97c \uc870\uc5d0\uc11c \uc54c\uba74 \uc548 \ub418\uae30 \ub54c\ubb38\uc774\uc5c8\ub2e4. \ub610\ud55c \uc5fc\ud30c\uac00 \uc804\uc7a5\uc744 \uc9c0\ud718\ud558\uba74 \uc2b9\ub9ac\ud560 \uc218 \uc5c6\uc74c\uc744 \uc54c\uc558\uae30\uc5d0 \uc774\uac04\ucc45\uc744 \uc368\uc11c \uc9c0\ud718\uad00\uc744 \ubc14\uafb8\uae30\ub85c \ud558\uace0 \uc870\ub098\ub77c \ub0b4\ubd80\uc5d0 \ucca9\uc790\ub4e4\uc744 \ud22c\uc785\ud558\uc5ec \uc774\ub7ec\ud55c \uc18c\ubb38\uc744 \ud37c\ub728\ub9ac\uac8c \ud558\uc600\ub2e4. \"\uc5fc\ud30c\uac00 \uc9c4\uc744 \uacf5\uaca9\ud558\uc9c0 \uc54a\ub294 \uac83\uc740 \ub4a4\uc5d0\uc11c \uc9c4\uacfc \ub0b4\ud1b5\ud558\uace0 \uc788\uae30 \ub54c\ubb38\uc774\ub2e4\" \"\uc9c4\uc774 \ub450\ub824\uc6cc\ud558\ub294 \uac83\uc740 \uc870\uad04\uc774 \uc7a5\uad70\uc774 \ub418\ub294 \uac83\uc774\ub2e4\" \uc774\ub7ec\ud55c \uc18c\ubb38\uc744 \ub4e4\uc740 \ud6a8\uc131\uc655\uc740 \uc5fc\ud30c\ub97c \uc9c0\ud718\uad00 \uc790\ub9ac\uc5d0\uc11c \ub0b4\uce58\uace0 \ub300\uc2e0 \uc870\uad04\uc744 \uc784\uba85\ud558\uc600\ub2e4.", "sentence_answer": "\ub610\ud55c \ucc9c\ud558\ud1b5\uc77c \uc744 \ubaa9\ud45c\ub85c \ud558\uace0 \uc788\ub294 \uc9c4\uc740 \uc774 \uc2f8\uc6c0\uc5d0\uc11c \ubc00\ub9ac\uba74 6\uad6d\uc5d0 \uc704\uc2e0\uc774 \ub5a8\uc5b4\uc9c0\uac8c \ub418\uba70, \ud1b5\uc77c\uc744 \ud5a5\ud55c \uae38\uc740 \ub354\uc6b1 \ud5d8\ub09c\ud574\uc9c8 \uc218 \uc788\uc5c8\uae30 \ub54c\ubb38\uc5d0 \ubc18\ub4dc\uc2dc \uc774\uaca8\uc57c\ub9cc \ud588\ub2e4.", "paragraph_id": "5859475-1-1", "paragraph_question": "question: \uc9c4\uc740 \ubb34\uc5c7\uc744 \ubaa9\ud45c\ub85c \ud558\uace0 \uc788\uc5c8\ub098?, context: \ub2f9\uc2dc \uc591\uad6d \uc870\uc815\uc740 \ub098\ub984\ub300\ub85c \ucd08\uc870\ud588\ub2e4. \ub2f9\uc2dc \uc0c1\ud669\uc774 \uc870\uc5d0\uac8c \uc720\ub9ac\ud558\uac8c \ub3cc\uc544\uac00\uace0 \uc788\uc5c8\uc74c\uc5d0\ub3c4 \ubd88\uad6c\ud558\uace0 \ud6a8\uc131\uc655\uc740 \ub2e4\uae09\ud588\ub2e4. \ubc29\uc5b4\ub9cc \ud558\uace0 \uc788\ub294 \uc5fc\ud30c\uac00 \uc790\uc2e0\uc758 \uc790\uc874\uc2ec\uc744 \ub5a8\uc5b4\ub728\ub9ac\uace0 \uc788\ub2e4\uace0 \ubbff\uc5c8\uae30 \ub54c\ubb38\uc774\uc5c8\ub2e4. \ub610\ud55c \ucc9c\ud558\ud1b5\uc77c\uc744 \ubaa9\ud45c\ub85c \ud558\uace0 \uc788\ub294 \uc9c4\uc740 \uc774 \uc2f8\uc6c0\uc5d0\uc11c \ubc00\ub9ac\uba74 6\uad6d\uc5d0 \uc704\uc2e0\uc774 \ub5a8\uc5b4\uc9c0\uac8c \ub418\uba70, \ud1b5\uc77c\uc744 \ud5a5\ud55c \uae38\uc740 \ub354\uc6b1 \ud5d8\ub09c\ud574\uc9c8 \uc218 \uc788\uc5c8\uae30 \ub54c\ubb38\uc5d0 \ubc18\ub4dc\uc2dc \uc774\uaca8\uc57c\ub9cc \ud588\ub2e4. \uac8c\ub2e4\uac00 \ub2f9\uc2dc \uc7ac\uc0c1\uc774\uc5c8\ub358 \ubc94\uc218\ub294 \uc774\ubbf8 \uba87 \ucc28\ub840\uc758 \uc2e4\ucc45\uc73c\ub85c \uc9d5\uacc4\ub97c \ubc1b\uc558\ub2e4. \uc774\ubc88 \uc804\ud22c\uc5d0\uc11c \ud328\ubc30\ud55c\ub2e4\uba74 \uadf8\uc758 \ubab0\ub77d\uc740 \uc790\uba85\ud55c \uc77c\uc774\uc5c8\ub2e4. \uc9c4\uc740 \ubc1c\ube60\ub974\uac8c \uc6c0\uc9c1\uc600\ub2e4. \ubc94\uc218\ub294 \ub450 \uac00\uc9c0 \uc2b9\ubd80\uc218\ub97c \ub744\uc6e0\ub2e4. \uc6b0\uc120 \ub2f9\uc2dc \uc9c0\ud718\uad00\uc774\uc5c8\ub358 \uc655\ud758\uc744 \ubab0\ub798 \ubc31\uc804\ub178\uc7a5 \ubc31\uae30\ub85c \uad50\uccb4\ud558\uc600\ub2e4. \uadf8\ub9ac\uace0 \uc774\ub97c \ubc1c\uc124\ud558\ub294 \uc790\ub294 \uac00\ucc28\uc5c6\uc774 \ucc98\ud615\ud558\uc600\ub2e4. \uc774\ub97c \uc870\uc5d0\uc11c \uc54c\uba74 \uc548 \ub418\uae30 \ub54c\ubb38\uc774\uc5c8\ub2e4. \ub610\ud55c \uc5fc\ud30c\uac00 \uc804\uc7a5\uc744 \uc9c0\ud718\ud558\uba74 \uc2b9\ub9ac\ud560 \uc218 \uc5c6\uc74c\uc744 \uc54c\uc558\uae30\uc5d0 \uc774\uac04\ucc45\uc744 \uc368\uc11c \uc9c0\ud718\uad00\uc744 \ubc14\uafb8\uae30\ub85c \ud558\uace0 \uc870\ub098\ub77c \ub0b4\ubd80\uc5d0 \ucca9\uc790\ub4e4\uc744 \ud22c\uc785\ud558\uc5ec \uc774\ub7ec\ud55c \uc18c\ubb38\uc744 \ud37c\ub728\ub9ac\uac8c \ud558\uc600\ub2e4. \"\uc5fc\ud30c\uac00 \uc9c4\uc744 \uacf5\uaca9\ud558\uc9c0 \uc54a\ub294 \uac83\uc740 \ub4a4\uc5d0\uc11c \uc9c4\uacfc \ub0b4\ud1b5\ud558\uace0 \uc788\uae30 \ub54c\ubb38\uc774\ub2e4\" \"\uc9c4\uc774 \ub450\ub824\uc6cc\ud558\ub294 \uac83\uc740 \uc870\uad04\uc774 \uc7a5\uad70\uc774 \ub418\ub294 \uac83\uc774\ub2e4\" \uc774\ub7ec\ud55c \uc18c\ubb38\uc744 \ub4e4\uc740 \ud6a8\uc131\uc655\uc740 \uc5fc\ud30c\ub97c \uc9c0\ud718\uad00 \uc790\ub9ac\uc5d0\uc11c \ub0b4\uce58\uace0 \ub300\uc2e0 \uc870\uad04\uc744 \uc784\uba85\ud558\uc600\ub2e4."} {"question": "\uc0bc\uad6d\uc0ac\uae30\uc758 \ubc31\uac15\uacfc \uae08\uac15 \ud558\uad6c\uc758 \uc6c5\uc9c4\uad6c\ub294 \uac19\uc740 \uac15\uc774 \uc544\ub2c8\ub77c\uace0 \uc8fc\uc7a5\ud55c \ud559\uc790\ub294?", "paragraph": "\ubd80\uc548 \uc9c0\uc5ed\uc744 \ubc31\uc81c \uc8fc\ub958\uc131\uc73c\ub85c \ube44\uc815\ud55c \uac83\uc740 1924\ub144 \uc77c\ubcf8\uc778 \ud559\uc790 \uc624\ud0c0 \uc1fc\uace0\uac00 \u300c\uc870\uc120\uc0c1\uc138\uc0ac\u300d\uc5d0\uc11c \ucc98\uc74c \uc8fc\ucc3d\ud55c \uc774\ub798, \uc9c1\uc811\uc801\uc73c\ub85c \uc704\uae08\uc554\uc0b0\uc131\uc744 \ubc31\uc81c \uc8fc\ub958\uc131\uc73c\ub85c \uc9c0\ubaa9\ud55c \uac83\uc740 \uc774\ub9c8\ub2c8\uc2dc \ub958\uac00 \ucd5c\ucd08\uc600\uc73c\uba70 \uc870\uc120\uc758 \uc548\uc7ac\ud64d \ub4f1\uc774 \ub3d9\uc870\ud558\uc5ec \uadf8 \uc124\uc744 \ubcf4\uac15\ud558\uc600\ub2e4. \uc774\ud6c4 \uc804\uc601\ub798, \uc774\ub3c4\ud559 \ub4f1\uc774 \uc774 \uc124\uc744 \ub530\ub974\uace0 \uc788\ub2e4. \uc624\ud0c0 \uc1fc\uace0\ub294 \u300a\uc0bc\uad6d\uc0ac\uae30\u300b \ub4f1\uc5d0 \ub4f1\uc7a5\ud558\ub294 \ubc31\uac15\uc774\ub77c\ub294 \uc9c0\uba85\uc5d0 \ub300\ud574 \uae08\uac15 \ud558\uad6c\ub294 \uc6c5\uc9c4\uac15\uad6c \ub610\ub294 \uc6c5\uc9c4\uad6c\ub85c \ud45c\uae30\ub418\uace0 \ubc31\uac15 \ud558\uad6c\ub294 \ubcc4\ub3c4\ub85c \ubc31\uac15\uad6c\ub85c \uae30\uc7ac\ub418\uc5b4 \uad6c\ubcc4\ud558\uace0 \uc788\ub2e4\uba70 \uacb0\ucf54 \uac19\uc740 \uac15\uc73c\ub85c \ubcfc\uc218 \uc5c6\ub2e4\uace0 \uc8fc\uc7a5\ud588\ub2e4. \uc804\uc601\ub798 \uc5ed\uc2dc \uc6c5\uc9c4\uac15\uacfc \ubc31\uac15\uc774 \uc11c\ub85c \ub2e4\ub978 \uac15\uc774\ub77c\ub294 \uc804\uc81c \uc544\ub798\uc11c \ubc31\uac15\uc758 \ub2e4\ub978 \uc774\ub984\uc778 '\uae30\ubc8c\ud3ec(\u4f0e\u4f10\u6d66)'\ub294 \uc9c0\uae08\uc758 \ub3d9\uc9c4\uac15 \ud558\uad6c \ud574\uc0c1\uc758 '\uacc4\ud654\ub3c4(\u754c\u706b\u5cf6)' \ubc0f \ubd80\uc548\uc74d\uc758 \ubc31\uc81c \uc9c0\uba85\uc778 '\uac1c\ud654(\u7686\u706b)'\ub294 \uc74c\uc6b4\uc0c1 \ud1b5\ud558\ubbc0\ub85c \ubc31\uac15\uc740 \ub3d9\uc9c4\uac15\uc774\uace0 \uc8fc\ub958\uc131\uc740 \ubd80\uc548\uc774\ub77c\uace0 \ud30c\uc545\ud55c \uc624\ud0c0 \uc1fc\uace0\uc758 \uc124\uc5d0 \ub3d9\uc870\ud558\uc600\ub2e4.", "answer": "\uc624\ud0c0 \uc1fc\uace0", "sentence": "\ubd80\uc548 \uc9c0\uc5ed\uc744 \ubc31\uc81c \uc8fc\ub958\uc131\uc73c\ub85c \ube44\uc815\ud55c \uac83\uc740 1924\ub144 \uc77c\ubcf8\uc778 \ud559\uc790 \uc624\ud0c0 \uc1fc\uace0 \uac00 \u300c\uc870\uc120\uc0c1\uc138\uc0ac\u300d\uc5d0\uc11c \ucc98\uc74c \uc8fc\ucc3d\ud55c \uc774\ub798, \uc9c1\uc811\uc801\uc73c\ub85c \uc704\uae08\uc554\uc0b0\uc131\uc744 \ubc31\uc81c \uc8fc\ub958\uc131\uc73c\ub85c \uc9c0\ubaa9\ud55c \uac83\uc740 \uc774\ub9c8\ub2c8\uc2dc \ub958\uac00 \ucd5c\ucd08\uc600\uc73c\uba70 \uc870\uc120\uc758 \uc548\uc7ac\ud64d \ub4f1\uc774 \ub3d9\uc870\ud558\uc5ec \uadf8 \uc124\uc744 \ubcf4\uac15\ud558\uc600\ub2e4.", "paragraph_sentence": " \ubd80\uc548 \uc9c0\uc5ed\uc744 \ubc31\uc81c \uc8fc\ub958\uc131\uc73c\ub85c \ube44\uc815\ud55c \uac83\uc740 1924\ub144 \uc77c\ubcf8\uc778 \ud559\uc790 \uc624\ud0c0 \uc1fc\uace0 \uac00 \u300c\uc870\uc120\uc0c1\uc138\uc0ac\u300d\uc5d0\uc11c \ucc98\uc74c \uc8fc\ucc3d\ud55c \uc774\ub798, \uc9c1\uc811\uc801\uc73c\ub85c \uc704\uae08\uc554\uc0b0\uc131\uc744 \ubc31\uc81c \uc8fc\ub958\uc131\uc73c\ub85c \uc9c0\ubaa9\ud55c \uac83\uc740 \uc774\ub9c8\ub2c8\uc2dc \ub958\uac00 \ucd5c\ucd08\uc600\uc73c\uba70 \uc870\uc120\uc758 \uc548\uc7ac\ud64d \ub4f1\uc774 \ub3d9\uc870\ud558\uc5ec \uadf8 \uc124\uc744 \ubcf4\uac15\ud558\uc600\ub2e4. \uc774\ud6c4 \uc804\uc601\ub798, \uc774\ub3c4\ud559 \ub4f1\uc774 \uc774 \uc124\uc744 \ub530\ub974\uace0 \uc788\ub2e4. \uc624\ud0c0 \uc1fc\uace0\ub294 \u300a\uc0bc\uad6d\uc0ac\uae30\u300b \ub4f1\uc5d0 \ub4f1\uc7a5\ud558\ub294 \ubc31\uac15\uc774\ub77c\ub294 \uc9c0\uba85\uc5d0 \ub300\ud574 \uae08\uac15 \ud558\uad6c\ub294 \uc6c5\uc9c4\uac15\uad6c \ub610\ub294 \uc6c5\uc9c4\uad6c\ub85c \ud45c\uae30\ub418\uace0 \ubc31\uac15 \ud558\uad6c\ub294 \ubcc4\ub3c4\ub85c \ubc31\uac15\uad6c\ub85c \uae30\uc7ac\ub418\uc5b4 \uad6c\ubcc4\ud558\uace0 \uc788\ub2e4\uba70 \uacb0\ucf54 \uac19\uc740 \uac15\uc73c\ub85c \ubcfc\uc218 \uc5c6\ub2e4\uace0 \uc8fc\uc7a5\ud588\ub2e4. \uc804\uc601\ub798 \uc5ed\uc2dc \uc6c5\uc9c4\uac15\uacfc \ubc31\uac15\uc774 \uc11c\ub85c \ub2e4\ub978 \uac15\uc774\ub77c\ub294 \uc804\uc81c \uc544\ub798\uc11c \ubc31\uac15\uc758 \ub2e4\ub978 \uc774\ub984\uc778 '\uae30\ubc8c\ud3ec(\u4f0e\u4f10\u6d66)'\ub294 \uc9c0\uae08\uc758 \ub3d9\uc9c4\uac15 \ud558\uad6c \ud574\uc0c1\uc758 '\uacc4\ud654\ub3c4(\u754c\u706b\u5cf6)' \ubc0f \ubd80\uc548\uc74d\uc758 \ubc31\uc81c \uc9c0\uba85\uc778 '\uac1c\ud654(\u7686\u706b)'\ub294 \uc74c\uc6b4\uc0c1 \ud1b5\ud558\ubbc0\ub85c \ubc31\uac15\uc740 \ub3d9\uc9c4\uac15\uc774\uace0 \uc8fc\ub958\uc131\uc740 \ubd80\uc548\uc774\ub77c\uace0 \ud30c\uc545\ud55c \uc624\ud0c0 \uc1fc\uace0\uc758 \uc124\uc5d0 \ub3d9\uc870\ud558\uc600\ub2e4.", "paragraph_answer": "\ubd80\uc548 \uc9c0\uc5ed\uc744 \ubc31\uc81c \uc8fc\ub958\uc131\uc73c\ub85c \ube44\uc815\ud55c \uac83\uc740 1924\ub144 \uc77c\ubcf8\uc778 \ud559\uc790 \uc624\ud0c0 \uc1fc\uace0 \uac00 \u300c\uc870\uc120\uc0c1\uc138\uc0ac\u300d\uc5d0\uc11c \ucc98\uc74c \uc8fc\ucc3d\ud55c \uc774\ub798, \uc9c1\uc811\uc801\uc73c\ub85c \uc704\uae08\uc554\uc0b0\uc131\uc744 \ubc31\uc81c \uc8fc\ub958\uc131\uc73c\ub85c \uc9c0\ubaa9\ud55c \uac83\uc740 \uc774\ub9c8\ub2c8\uc2dc \ub958\uac00 \ucd5c\ucd08\uc600\uc73c\uba70 \uc870\uc120\uc758 \uc548\uc7ac\ud64d \ub4f1\uc774 \ub3d9\uc870\ud558\uc5ec \uadf8 \uc124\uc744 \ubcf4\uac15\ud558\uc600\ub2e4. \uc774\ud6c4 \uc804\uc601\ub798, \uc774\ub3c4\ud559 \ub4f1\uc774 \uc774 \uc124\uc744 \ub530\ub974\uace0 \uc788\ub2e4. \uc624\ud0c0 \uc1fc\uace0\ub294 \u300a\uc0bc\uad6d\uc0ac\uae30\u300b \ub4f1\uc5d0 \ub4f1\uc7a5\ud558\ub294 \ubc31\uac15\uc774\ub77c\ub294 \uc9c0\uba85\uc5d0 \ub300\ud574 \uae08\uac15 \ud558\uad6c\ub294 \uc6c5\uc9c4\uac15\uad6c \ub610\ub294 \uc6c5\uc9c4\uad6c\ub85c \ud45c\uae30\ub418\uace0 \ubc31\uac15 \ud558\uad6c\ub294 \ubcc4\ub3c4\ub85c \ubc31\uac15\uad6c\ub85c \uae30\uc7ac\ub418\uc5b4 \uad6c\ubcc4\ud558\uace0 \uc788\ub2e4\uba70 \uacb0\ucf54 \uac19\uc740 \uac15\uc73c\ub85c \ubcfc\uc218 \uc5c6\ub2e4\uace0 \uc8fc\uc7a5\ud588\ub2e4. \uc804\uc601\ub798 \uc5ed\uc2dc \uc6c5\uc9c4\uac15\uacfc \ubc31\uac15\uc774 \uc11c\ub85c \ub2e4\ub978 \uac15\uc774\ub77c\ub294 \uc804\uc81c \uc544\ub798\uc11c \ubc31\uac15\uc758 \ub2e4\ub978 \uc774\ub984\uc778 '\uae30\ubc8c\ud3ec(\u4f0e\u4f10\u6d66)'\ub294 \uc9c0\uae08\uc758 \ub3d9\uc9c4\uac15 \ud558\uad6c \ud574\uc0c1\uc758 '\uacc4\ud654\ub3c4(\u754c\u706b\u5cf6)' \ubc0f \ubd80\uc548\uc74d\uc758 \ubc31\uc81c \uc9c0\uba85\uc778 '\uac1c\ud654(\u7686\u706b)'\ub294 \uc74c\uc6b4\uc0c1 \ud1b5\ud558\ubbc0\ub85c \ubc31\uac15\uc740 \ub3d9\uc9c4\uac15\uc774\uace0 \uc8fc\ub958\uc131\uc740 \ubd80\uc548\uc774\ub77c\uace0 \ud30c\uc545\ud55c \uc624\ud0c0 \uc1fc\uace0\uc758 \uc124\uc5d0 \ub3d9\uc870\ud558\uc600\ub2e4.", "sentence_answer": "\ubd80\uc548 \uc9c0\uc5ed\uc744 \ubc31\uc81c \uc8fc\ub958\uc131\uc73c\ub85c \ube44\uc815\ud55c \uac83\uc740 1924\ub144 \uc77c\ubcf8\uc778 \ud559\uc790 \uc624\ud0c0 \uc1fc\uace0 \uac00 \u300c\uc870\uc120\uc0c1\uc138\uc0ac\u300d\uc5d0\uc11c \ucc98\uc74c \uc8fc\ucc3d\ud55c \uc774\ub798, \uc9c1\uc811\uc801\uc73c\ub85c \uc704\uae08\uc554\uc0b0\uc131\uc744 \ubc31\uc81c \uc8fc\ub958\uc131\uc73c\ub85c \uc9c0\ubaa9\ud55c \uac83\uc740 \uc774\ub9c8\ub2c8\uc2dc \ub958\uac00 \ucd5c\ucd08\uc600\uc73c\uba70 \uc870\uc120\uc758 \uc548\uc7ac\ud64d \ub4f1\uc774 \ub3d9\uc870\ud558\uc5ec \uadf8 \uc124\uc744 \ubcf4\uac15\ud558\uc600\ub2e4.", "paragraph_id": "5890164-9-1", "paragraph_question": "question: \uc0bc\uad6d\uc0ac\uae30\uc758 \ubc31\uac15\uacfc \uae08\uac15 \ud558\uad6c\uc758 \uc6c5\uc9c4\uad6c\ub294 \uac19\uc740 \uac15\uc774 \uc544\ub2c8\ub77c\uace0 \uc8fc\uc7a5\ud55c \ud559\uc790\ub294?, context: \ubd80\uc548 \uc9c0\uc5ed\uc744 \ubc31\uc81c \uc8fc\ub958\uc131\uc73c\ub85c \ube44\uc815\ud55c \uac83\uc740 1924\ub144 \uc77c\ubcf8\uc778 \ud559\uc790 \uc624\ud0c0 \uc1fc\uace0\uac00 \u300c\uc870\uc120\uc0c1\uc138\uc0ac\u300d\uc5d0\uc11c \ucc98\uc74c \uc8fc\ucc3d\ud55c \uc774\ub798, \uc9c1\uc811\uc801\uc73c\ub85c \uc704\uae08\uc554\uc0b0\uc131\uc744 \ubc31\uc81c \uc8fc\ub958\uc131\uc73c\ub85c \uc9c0\ubaa9\ud55c \uac83\uc740 \uc774\ub9c8\ub2c8\uc2dc \ub958\uac00 \ucd5c\ucd08\uc600\uc73c\uba70 \uc870\uc120\uc758 \uc548\uc7ac\ud64d \ub4f1\uc774 \ub3d9\uc870\ud558\uc5ec \uadf8 \uc124\uc744 \ubcf4\uac15\ud558\uc600\ub2e4. \uc774\ud6c4 \uc804\uc601\ub798, \uc774\ub3c4\ud559 \ub4f1\uc774 \uc774 \uc124\uc744 \ub530\ub974\uace0 \uc788\ub2e4. \uc624\ud0c0 \uc1fc\uace0\ub294 \u300a\uc0bc\uad6d\uc0ac\uae30\u300b \ub4f1\uc5d0 \ub4f1\uc7a5\ud558\ub294 \ubc31\uac15\uc774\ub77c\ub294 \uc9c0\uba85\uc5d0 \ub300\ud574 \uae08\uac15 \ud558\uad6c\ub294 \uc6c5\uc9c4\uac15\uad6c \ub610\ub294 \uc6c5\uc9c4\uad6c\ub85c \ud45c\uae30\ub418\uace0 \ubc31\uac15 \ud558\uad6c\ub294 \ubcc4\ub3c4\ub85c \ubc31\uac15\uad6c\ub85c \uae30\uc7ac\ub418\uc5b4 \uad6c\ubcc4\ud558\uace0 \uc788\ub2e4\uba70 \uacb0\ucf54 \uac19\uc740 \uac15\uc73c\ub85c \ubcfc\uc218 \uc5c6\ub2e4\uace0 \uc8fc\uc7a5\ud588\ub2e4. \uc804\uc601\ub798 \uc5ed\uc2dc \uc6c5\uc9c4\uac15\uacfc \ubc31\uac15\uc774 \uc11c\ub85c \ub2e4\ub978 \uac15\uc774\ub77c\ub294 \uc804\uc81c \uc544\ub798\uc11c \ubc31\uac15\uc758 \ub2e4\ub978 \uc774\ub984\uc778 '\uae30\ubc8c\ud3ec(\u4f0e\u4f10\u6d66)'\ub294 \uc9c0\uae08\uc758 \ub3d9\uc9c4\uac15 \ud558\uad6c \ud574\uc0c1\uc758 '\uacc4\ud654\ub3c4(\u754c\u706b\u5cf6)' \ubc0f \ubd80\uc548\uc74d\uc758 \ubc31\uc81c \uc9c0\uba85\uc778 '\uac1c\ud654(\u7686\u706b)'\ub294 \uc74c\uc6b4\uc0c1 \ud1b5\ud558\ubbc0\ub85c \ubc31\uac15\uc740 \ub3d9\uc9c4\uac15\uc774\uace0 \uc8fc\ub958\uc131\uc740 \ubd80\uc548\uc774\ub77c\uace0 \ud30c\uc545\ud55c \uc624\ud0c0 \uc1fc\uace0\uc758 \uc124\uc5d0 \ub3d9\uc870\ud558\uc600\ub2e4."} {"question": "\ubd81\ud55c\uc774 2006\ub144 10\uc6d4 9\uc77c \uc9c0\ud558 \ud575\uc2e4\ud5d8\uc744 \uac15\ud589\ud55c \ub4a4 10\uc6d4 15\uc77c \uc720\uc5d4 \uc548\ubcf4\ub9ac\uac00 \ub9cc\uc7a5\uc77c\uce58\ub85c \ucc44\ud0dd\ud55c \uacb0\uc758\uc548\uc740?", "paragraph": "\ubd81\ud55c\uc774 2006\ub144 7\uc6d4 5\uc77c \ub2e8.\uc911.\uc7a5\uac70\ub9ac \ubbf8\uc0ac\uc77c 6\ubc1c\uc744 \ubc1c\uc0ac\ud55c \ub4a4 \uc720\uc5d4 \uc548\ubcf4\ub9ac\ub294 2006\ub144 7\uc6d4 16\uc77c \uacb0\uc758\uc548 1695\ud638\ub97c \ucc44\ud0dd\ud588\ub2e4. \uc774\uc5b4 \ubd81\ud55c\uc740 10\uc6d49\uc77c \uc9c0\ud558 \ud575\uc2e4\ud5d8\uc744 \uac15\ud589\ud588\uace0 \uc77c\uc8fc\uc77c \ub9cc\uc778 10\uc6d415\uc77c \uc548\ubcf4\ub9ac\ub294 \uacb0\uc758 1718\uc744 \ub9cc\uc7a5\uc77c\uce58\ub85c \ucc44\ud0dd\ud588\ub2e4. \uc774\ub807\uac8c \ub418\uba74\uc11c \ud55c\ubc18\ub3c4 \uc0c1\ud669\uc758 \ud2b9\uc218\uc131\uc744 \uac70\ub860\ud558\uba70 PSI\ucc38\uc5ec \ud655\ub300 \uc694\uccad\uc744 \ubc1b\uc544\ub4e4\uc774\uc9c0 \uc54a\uc558\ub358 \ud55c\uad6d \uc815\ubd80\uc758 \uc785\uc7a5\uc740 \ub09c\uac10\ud574 \uc84c\uace0 \uc774\ub54c\ub97c \ub193\uce58\uc9c0 \uc54a\uace0 \ubbf8\uad6d\uc740 \ud55c-\ubbf8 \uc678\ubb34\uc7a5\uad00 \ud68c\ub2f4(10\uc6d4 19\uc77c)\uc744 \ud3ec\ud568\ud574 \uc5ec\ub7ec \uacbd\ub85c\ub97c \ud1b5\ud574 PSI\ucc38\uc5ec \ud655\ub300\ub97c \uc555\ubc15\ud588\ub2e4. \ud2b9\ud788 \ubbf8\uad6d\uc740 \uc774 \uacfc\uc815\uc5d0\uc11c \uc774\ub978\ubc14 \ud587\ubcd5\uc815\ucc45 \ud639\uc740 \ub300\ubd81\ud3ec\uc6a9\uc815\ucc45\uc758 \uc0c1\uc9d5\uc801\uc778 \uacb0\uacfc\ubb3c\uc778 \uae08\uac15\uc0b0\uad00\uad11\uacfc \uac1c\uc131\uacf5\ub2e8 \uc0ac\uc5c5\uc744 \ubb38\uc81c\uc0bc\uace0 \ub098\uc11c\uae30\uae4c\uc9c0 \ud588\ub2e4. \ub2f9\uc2dc \uc788\uc5c8\ub358 \uc678\uad50\ud1b5\uc0c1\ubd80 \uad6d\uc815\uac10\uc0ac(10\uc6d4 27\uc77c)\ub294 \uadf8\uc57c\ub9d0\ub85c \u2018PSI \uad6d\uc815\uac10\uc0ac\u2019\ub77c\uace0 \ud574\ub3c4 \uacfc\uc5b8\uc774 \uc544\ub2d0 \uc815\ub3c4\ub85c PSI \ucc38\uc5ec\ud655\ub300\ub97c \ub458\ub7ec\uc2fc \uce58\uc5f4\ud55c \ub17c\uc7c1\uc774 \ubc8c\uc5b4\uc84c\ub2e4.", "answer": "1718", "sentence": "\uc774\uc5b4 \ubd81\ud55c\uc740 10\uc6d49\uc77c \uc9c0\ud558 \ud575\uc2e4\ud5d8\uc744 \uac15\ud589\ud588\uace0 \uc77c\uc8fc\uc77c \ub9cc\uc778 10\uc6d415\uc77c \uc548\ubcf4\ub9ac\ub294 \uacb0\uc758 1718 \uc744 \ub9cc\uc7a5\uc77c\uce58\ub85c \ucc44\ud0dd\ud588\ub2e4.", "paragraph_sentence": "\ubd81\ud55c\uc774 2006\ub144 7\uc6d4 5\uc77c \ub2e8.\uc911.\uc7a5\uac70\ub9ac \ubbf8\uc0ac\uc77c 6\ubc1c\uc744 \ubc1c\uc0ac\ud55c \ub4a4 \uc720\uc5d4 \uc548\ubcf4\ub9ac\ub294 2006\ub144 7\uc6d4 16\uc77c \uacb0\uc758\uc548 1695\ud638\ub97c \ucc44\ud0dd\ud588\ub2e4. \uc774\uc5b4 \ubd81\ud55c\uc740 10\uc6d49\uc77c \uc9c0\ud558 \ud575\uc2e4\ud5d8\uc744 \uac15\ud589\ud588\uace0 \uc77c\uc8fc\uc77c \ub9cc\uc778 10\uc6d415\uc77c \uc548\ubcf4\ub9ac\ub294 \uacb0\uc758 1718 \uc744 \ub9cc\uc7a5\uc77c\uce58\ub85c \ucc44\ud0dd\ud588\ub2e4. \uc774\ub807\uac8c \ub418\uba74\uc11c \ud55c\ubc18\ub3c4 \uc0c1\ud669\uc758 \ud2b9\uc218\uc131\uc744 \uac70\ub860\ud558\uba70 PSI\ucc38\uc5ec \ud655\ub300 \uc694\uccad\uc744 \ubc1b\uc544\ub4e4\uc774\uc9c0 \uc54a\uc558\ub358 \ud55c\uad6d \uc815\ubd80\uc758 \uc785\uc7a5\uc740 \ub09c\uac10\ud574 \uc84c\uace0 \uc774\ub54c\ub97c \ub193\uce58\uc9c0 \uc54a\uace0 \ubbf8\uad6d\uc740 \ud55c-\ubbf8 \uc678\ubb34\uc7a5\uad00 \ud68c\ub2f4(10\uc6d4 19\uc77c)\uc744 \ud3ec\ud568\ud574 \uc5ec\ub7ec \uacbd\ub85c\ub97c \ud1b5\ud574 PSI\ucc38\uc5ec \ud655\ub300\ub97c \uc555\ubc15\ud588\ub2e4. \ud2b9\ud788 \ubbf8\uad6d\uc740 \uc774 \uacfc\uc815\uc5d0\uc11c \uc774\ub978\ubc14 \ud587\ubcd5\uc815\ucc45 \ud639\uc740 \ub300\ubd81\ud3ec\uc6a9\uc815\ucc45\uc758 \uc0c1\uc9d5\uc801\uc778 \uacb0\uacfc\ubb3c\uc778 \uae08\uac15\uc0b0\uad00\uad11\uacfc \uac1c\uc131\uacf5\ub2e8 \uc0ac\uc5c5\uc744 \ubb38\uc81c\uc0bc\uace0 \ub098\uc11c\uae30\uae4c\uc9c0 \ud588\ub2e4. \ub2f9\uc2dc \uc788\uc5c8\ub358 \uc678\uad50\ud1b5\uc0c1\ubd80 \uad6d\uc815\uac10\uc0ac(10\uc6d4 27\uc77c)\ub294 \uadf8\uc57c\ub9d0\ub85c \u2018PSI \uad6d\uc815\uac10\uc0ac\u2019\ub77c\uace0 \ud574\ub3c4 \uacfc\uc5b8\uc774 \uc544\ub2d0 \uc815\ub3c4\ub85c PSI \ucc38\uc5ec\ud655\ub300\ub97c \ub458\ub7ec\uc2fc \uce58\uc5f4\ud55c \ub17c\uc7c1\uc774 \ubc8c\uc5b4\uc84c\ub2e4.", "paragraph_answer": "\ubd81\ud55c\uc774 2006\ub144 7\uc6d4 5\uc77c \ub2e8.\uc911.\uc7a5\uac70\ub9ac \ubbf8\uc0ac\uc77c 6\ubc1c\uc744 \ubc1c\uc0ac\ud55c \ub4a4 \uc720\uc5d4 \uc548\ubcf4\ub9ac\ub294 2006\ub144 7\uc6d4 16\uc77c \uacb0\uc758\uc548 1695\ud638\ub97c \ucc44\ud0dd\ud588\ub2e4. \uc774\uc5b4 \ubd81\ud55c\uc740 10\uc6d49\uc77c \uc9c0\ud558 \ud575\uc2e4\ud5d8\uc744 \uac15\ud589\ud588\uace0 \uc77c\uc8fc\uc77c \ub9cc\uc778 10\uc6d415\uc77c \uc548\ubcf4\ub9ac\ub294 \uacb0\uc758 1718 \uc744 \ub9cc\uc7a5\uc77c\uce58\ub85c \ucc44\ud0dd\ud588\ub2e4. \uc774\ub807\uac8c \ub418\uba74\uc11c \ud55c\ubc18\ub3c4 \uc0c1\ud669\uc758 \ud2b9\uc218\uc131\uc744 \uac70\ub860\ud558\uba70 PSI\ucc38\uc5ec \ud655\ub300 \uc694\uccad\uc744 \ubc1b\uc544\ub4e4\uc774\uc9c0 \uc54a\uc558\ub358 \ud55c\uad6d \uc815\ubd80\uc758 \uc785\uc7a5\uc740 \ub09c\uac10\ud574 \uc84c\uace0 \uc774\ub54c\ub97c \ub193\uce58\uc9c0 \uc54a\uace0 \ubbf8\uad6d\uc740 \ud55c-\ubbf8 \uc678\ubb34\uc7a5\uad00 \ud68c\ub2f4(10\uc6d4 19\uc77c)\uc744 \ud3ec\ud568\ud574 \uc5ec\ub7ec \uacbd\ub85c\ub97c \ud1b5\ud574 PSI\ucc38\uc5ec \ud655\ub300\ub97c \uc555\ubc15\ud588\ub2e4. \ud2b9\ud788 \ubbf8\uad6d\uc740 \uc774 \uacfc\uc815\uc5d0\uc11c \uc774\ub978\ubc14 \ud587\ubcd5\uc815\ucc45 \ud639\uc740 \ub300\ubd81\ud3ec\uc6a9\uc815\ucc45\uc758 \uc0c1\uc9d5\uc801\uc778 \uacb0\uacfc\ubb3c\uc778 \uae08\uac15\uc0b0\uad00\uad11\uacfc \uac1c\uc131\uacf5\ub2e8 \uc0ac\uc5c5\uc744 \ubb38\uc81c\uc0bc\uace0 \ub098\uc11c\uae30\uae4c\uc9c0 \ud588\ub2e4. \ub2f9\uc2dc \uc788\uc5c8\ub358 \uc678\uad50\ud1b5\uc0c1\ubd80 \uad6d\uc815\uac10\uc0ac(10\uc6d4 27\uc77c)\ub294 \uadf8\uc57c\ub9d0\ub85c \u2018PSI \uad6d\uc815\uac10\uc0ac\u2019\ub77c\uace0 \ud574\ub3c4 \uacfc\uc5b8\uc774 \uc544\ub2d0 \uc815\ub3c4\ub85c PSI \ucc38\uc5ec\ud655\ub300\ub97c \ub458\ub7ec\uc2fc \uce58\uc5f4\ud55c \ub17c\uc7c1\uc774 \ubc8c\uc5b4\uc84c\ub2e4.", "sentence_answer": "\uc774\uc5b4 \ubd81\ud55c\uc740 10\uc6d49\uc77c \uc9c0\ud558 \ud575\uc2e4\ud5d8\uc744 \uac15\ud589\ud588\uace0 \uc77c\uc8fc\uc77c \ub9cc\uc778 10\uc6d415\uc77c \uc548\ubcf4\ub9ac\ub294 \uacb0\uc758 1718 \uc744 \ub9cc\uc7a5\uc77c\uce58\ub85c \ucc44\ud0dd\ud588\ub2e4.", "paragraph_id": "6490854-7-1", "paragraph_question": "question: \ubd81\ud55c\uc774 2006\ub144 10\uc6d4 9\uc77c \uc9c0\ud558 \ud575\uc2e4\ud5d8\uc744 \uac15\ud589\ud55c \ub4a4 10\uc6d4 15\uc77c \uc720\uc5d4 \uc548\ubcf4\ub9ac\uac00 \ub9cc\uc7a5\uc77c\uce58\ub85c \ucc44\ud0dd\ud55c \uacb0\uc758\uc548\uc740?, context: \ubd81\ud55c\uc774 2006\ub144 7\uc6d4 5\uc77c \ub2e8.\uc911.\uc7a5\uac70\ub9ac \ubbf8\uc0ac\uc77c 6\ubc1c\uc744 \ubc1c\uc0ac\ud55c \ub4a4 \uc720\uc5d4 \uc548\ubcf4\ub9ac\ub294 2006\ub144 7\uc6d4 16\uc77c \uacb0\uc758\uc548 1695\ud638\ub97c \ucc44\ud0dd\ud588\ub2e4. \uc774\uc5b4 \ubd81\ud55c\uc740 10\uc6d49\uc77c \uc9c0\ud558 \ud575\uc2e4\ud5d8\uc744 \uac15\ud589\ud588\uace0 \uc77c\uc8fc\uc77c \ub9cc\uc778 10\uc6d415\uc77c \uc548\ubcf4\ub9ac\ub294 \uacb0\uc758 1718\uc744 \ub9cc\uc7a5\uc77c\uce58\ub85c \ucc44\ud0dd\ud588\ub2e4. \uc774\ub807\uac8c \ub418\uba74\uc11c \ud55c\ubc18\ub3c4 \uc0c1\ud669\uc758 \ud2b9\uc218\uc131\uc744 \uac70\ub860\ud558\uba70 PSI\ucc38\uc5ec \ud655\ub300 \uc694\uccad\uc744 \ubc1b\uc544\ub4e4\uc774\uc9c0 \uc54a\uc558\ub358 \ud55c\uad6d \uc815\ubd80\uc758 \uc785\uc7a5\uc740 \ub09c\uac10\ud574 \uc84c\uace0 \uc774\ub54c\ub97c \ub193\uce58\uc9c0 \uc54a\uace0 \ubbf8\uad6d\uc740 \ud55c-\ubbf8 \uc678\ubb34\uc7a5\uad00 \ud68c\ub2f4(10\uc6d4 19\uc77c)\uc744 \ud3ec\ud568\ud574 \uc5ec\ub7ec \uacbd\ub85c\ub97c \ud1b5\ud574 PSI\ucc38\uc5ec \ud655\ub300\ub97c \uc555\ubc15\ud588\ub2e4. \ud2b9\ud788 \ubbf8\uad6d\uc740 \uc774 \uacfc\uc815\uc5d0\uc11c \uc774\ub978\ubc14 \ud587\ubcd5\uc815\ucc45 \ud639\uc740 \ub300\ubd81\ud3ec\uc6a9\uc815\ucc45\uc758 \uc0c1\uc9d5\uc801\uc778 \uacb0\uacfc\ubb3c\uc778 \uae08\uac15\uc0b0\uad00\uad11\uacfc \uac1c\uc131\uacf5\ub2e8 \uc0ac\uc5c5\uc744 \ubb38\uc81c\uc0bc\uace0 \ub098\uc11c\uae30\uae4c\uc9c0 \ud588\ub2e4. \ub2f9\uc2dc \uc788\uc5c8\ub358 \uc678\uad50\ud1b5\uc0c1\ubd80 \uad6d\uc815\uac10\uc0ac(10\uc6d4 27\uc77c)\ub294 \uadf8\uc57c\ub9d0\ub85c \u2018PSI \uad6d\uc815\uac10\uc0ac\u2019\ub77c\uace0 \ud574\ub3c4 \uacfc\uc5b8\uc774 \uc544\ub2d0 \uc815\ub3c4\ub85c PSI \ucc38\uc5ec\ud655\ub300\ub97c \ub458\ub7ec\uc2fc \uce58\uc5f4\ud55c \ub17c\uc7c1\uc774 \ubc8c\uc5b4\uc84c\ub2e4."} {"question": "\ub3d9\uc77c\ubcf8\ub300\uc9c0\uc9c4\uc758 \uc6d0\uc778\uc778 \ub290\ub9b0 \ubbf8\ub044\ub7ec\uc9d0 \ud604\uc0c1\uc73c\ub85c \uc778\ud55c \uc9c0\uac01\ubcc0\ub3d9\uc73c\ub85c \uc778\ud55c \ub3c4\ud638\ucfe0 \uc9c0\ubc29\uc758 \uc774\ub3d9 \uac70\ub9ac\ub294?", "paragraph": "\ubcf8\uc9c4\uacfc \uc5ec\uc9c4\uc744 \ud3ec\ud568\ud55c \ub290\ub9b0 \ubbf8\ub044\ub7ec\uc9d0 \ud604\uc0c1\uc73c\ub85c \ud070 \uc9c0\uac01\ubcc0\ub3d9\uc774 \uc77c\uc5b4\ub0ac\ub2e4. \ub3c4\ud638\ucfe0 \uc9c0\ubc29 \uc804\uc5ed\uc774 \ub3d9\ucabd\uc73c\ub85c 10cm \uc774\uc0c1 \uc774\ub3d9 \ud558\ub294 \ub4f1 \uadf8 \uc601\ud5a5\uc740 \ub9e4\uc6b0 \ucef8\ub2e4. \ub3c4\ud638\ucfe0 \uc9c0\ubc29\ubd80\ud130 \uad00\ub3d9\uae4c\uc9c0 \ub3d9\ubd81\uc77c\ubcf8 \uc9c0\uac01\uc740 \uc6d0\ub798\ub294 \ud0dc\ud3c9\uc591\ud310\uc774 \uc544\ub798\ub85c \uc12d\uc785\ud558\uba74\uc11c \uc11c\ucabd\uc73c\ub85c \uc9c0\uac01\ubcc0\ub3d9\uc744 \ubc1b\uace0 \uc788\uc5c8\uc73c\ub098, \ub3d9\uc77c\ubcf8\ub300\uc9c0\uc9c4 \uc774\ud6c4\uc5d0\ub294 \uc9c0\uac01\uc774 \ubc18\ub300\ub85c \ub3d9\ucabd\uc73c\ub85c \uc751\ub825\uc744 \ubc1b\uace0 \uc788\uc73c\uba70, \uc774\ub85c \uc778\ud574 \uc9c0\uc9c4 \ud65c\ub3d9\uc774 \ubc14\ub014 \uc218 \uc788\ub2e4\ub294 \uc6b0\ub824\uac00 \ub098\uc624\uace0 \uc788\ub2e4. \ub610\ud55c, \ud0dc\ud3c9\uc591\ud310\uc774 \uac00\ub77c\uc549\uc544 \uc788\ub294 \uacf3 \uc704\uc758 \uc721\uc9c0\uc5d0\uc11c \uc9c4\uc6d0\uc5ed \ubd80\uadfc\uc740 \uc735\uae30\ud558\uace0 \uc788\uc73c\uba70, \uc9c4\uc6d0\uc5ed \uc11c\ucabd\uc5d0\uc11c\ub294 \uce68\uac15\ud558\uace0 \uc788\uc5b4 \ub3c4\ud638\ucfe0\uc5d0\uc11c \uad00\ub3d9\uc5d0 \uac78\uce5c \ud0dc\ud3c9\uc591 \uc5f0\uc548 \uc804 \uc9c0\uc5ed\uc774 \uc77c\uc81c\ud788 \uc9c0\ubc18 \uce68\ud558 \ud604\uc0c1\uc774 \uc77c\uc5b4\ub098 2\ucc28 \ud53c\ud574\ub97c \uc77c\uc73c\ud0a4\uace0 \uc788\ub2e4. \uc9c4\uc6d0\uc5d0 \uac00\uae4c\uc6b4 \uc9c0\uc5ed\uc77c\uc218\ub85d \uc774 \uc9c0\uac01\ubcc0\ub3d9 \ud604\uc0c1\uc774 \ud06c\uac8c \uc77c\uc5b4\ub098\uace0 \uc788\ub294\ub370, \uc5f0\uad6c\uc5d0 \ub530\ub77c\uc11c \uc9c4\uc6d0 \uadfc\ucc98\uc758 \ud574\uc800 \uc9c0\uc5ed\uc740 \ub3d9\ucabd\uc73c\ub85c 50m \uc774\uc0c1 \uc774\ub3d9\ud588\ub2e4\ub294, \uc138\uacc4 \uc9c0\uc9c4 \uad00\uce21 \uc0ac\uc0c1 \uc720\ub840\uc5c6\ub294 \uae09\uaca9\ud55c \uc774\ub3d9\ub3c4 \uc77c\uc5b4\ub0ac\ub2e4.", "answer": "10cm \uc774\uc0c1", "sentence": "\ub3c4\ud638\ucfe0 \uc9c0\ubc29 \uc804\uc5ed\uc774 \ub3d9\ucabd\uc73c\ub85c 10cm \uc774\uc0c1 \uc774\ub3d9 \ud558\ub294 \ub4f1 \uadf8 \uc601\ud5a5\uc740 \ub9e4\uc6b0 \ucef8\ub2e4.", "paragraph_sentence": "\ubcf8\uc9c4\uacfc \uc5ec\uc9c4\uc744 \ud3ec\ud568\ud55c \ub290\ub9b0 \ubbf8\ub044\ub7ec\uc9d0 \ud604\uc0c1\uc73c\ub85c \ud070 \uc9c0\uac01\ubcc0\ub3d9\uc774 \uc77c\uc5b4\ub0ac\ub2e4. \ub3c4\ud638\ucfe0 \uc9c0\ubc29 \uc804\uc5ed\uc774 \ub3d9\ucabd\uc73c\ub85c 10cm \uc774\uc0c1 \uc774\ub3d9 \ud558\ub294 \ub4f1 \uadf8 \uc601\ud5a5\uc740 \ub9e4\uc6b0 \ucef8\ub2e4. \ub3c4\ud638\ucfe0 \uc9c0\ubc29\ubd80\ud130 \uad00\ub3d9\uae4c\uc9c0 \ub3d9\ubd81\uc77c\ubcf8 \uc9c0\uac01\uc740 \uc6d0\ub798\ub294 \ud0dc\ud3c9\uc591\ud310\uc774 \uc544\ub798\ub85c \uc12d\uc785\ud558\uba74\uc11c \uc11c\ucabd\uc73c\ub85c \uc9c0\uac01\ubcc0\ub3d9\uc744 \ubc1b\uace0 \uc788\uc5c8\uc73c\ub098, \ub3d9\uc77c\ubcf8\ub300\uc9c0\uc9c4 \uc774\ud6c4\uc5d0\ub294 \uc9c0\uac01\uc774 \ubc18\ub300\ub85c \ub3d9\ucabd\uc73c\ub85c \uc751\ub825\uc744 \ubc1b\uace0 \uc788\uc73c\uba70, \uc774\ub85c \uc778\ud574 \uc9c0\uc9c4 \ud65c\ub3d9\uc774 \ubc14\ub014 \uc218 \uc788\ub2e4\ub294 \uc6b0\ub824\uac00 \ub098\uc624\uace0 \uc788\ub2e4. \ub610\ud55c, \ud0dc\ud3c9\uc591\ud310\uc774 \uac00\ub77c\uc549\uc544 \uc788\ub294 \uacf3 \uc704\uc758 \uc721\uc9c0\uc5d0\uc11c \uc9c4\uc6d0\uc5ed \ubd80\uadfc\uc740 \uc735\uae30\ud558\uace0 \uc788\uc73c\uba70, \uc9c4\uc6d0\uc5ed \uc11c\ucabd\uc5d0\uc11c\ub294 \uce68\uac15\ud558\uace0 \uc788\uc5b4 \ub3c4\ud638\ucfe0\uc5d0\uc11c \uad00\ub3d9\uc5d0 \uac78\uce5c \ud0dc\ud3c9\uc591 \uc5f0\uc548 \uc804 \uc9c0\uc5ed\uc774 \uc77c\uc81c\ud788 \uc9c0\ubc18 \uce68\ud558 \ud604\uc0c1\uc774 \uc77c\uc5b4\ub098 2\ucc28 \ud53c\ud574\ub97c \uc77c\uc73c\ud0a4\uace0 \uc788\ub2e4. \uc9c4\uc6d0\uc5d0 \uac00\uae4c\uc6b4 \uc9c0\uc5ed\uc77c\uc218\ub85d \uc774 \uc9c0\uac01\ubcc0\ub3d9 \ud604\uc0c1\uc774 \ud06c\uac8c \uc77c\uc5b4\ub098\uace0 \uc788\ub294\ub370, \uc5f0\uad6c\uc5d0 \ub530\ub77c\uc11c \uc9c4\uc6d0 \uadfc\ucc98\uc758 \ud574\uc800 \uc9c0\uc5ed\uc740 \ub3d9\ucabd\uc73c\ub85c 50m \uc774\uc0c1 \uc774\ub3d9\ud588\ub2e4\ub294, \uc138\uacc4 \uc9c0\uc9c4 \uad00\uce21 \uc0ac\uc0c1 \uc720\ub840\uc5c6\ub294 \uae09\uaca9\ud55c \uc774\ub3d9\ub3c4 \uc77c\uc5b4\ub0ac\ub2e4.", "paragraph_answer": "\ubcf8\uc9c4\uacfc \uc5ec\uc9c4\uc744 \ud3ec\ud568\ud55c \ub290\ub9b0 \ubbf8\ub044\ub7ec\uc9d0 \ud604\uc0c1\uc73c\ub85c \ud070 \uc9c0\uac01\ubcc0\ub3d9\uc774 \uc77c\uc5b4\ub0ac\ub2e4. \ub3c4\ud638\ucfe0 \uc9c0\ubc29 \uc804\uc5ed\uc774 \ub3d9\ucabd\uc73c\ub85c 10cm \uc774\uc0c1 \uc774\ub3d9 \ud558\ub294 \ub4f1 \uadf8 \uc601\ud5a5\uc740 \ub9e4\uc6b0 \ucef8\ub2e4. \ub3c4\ud638\ucfe0 \uc9c0\ubc29\ubd80\ud130 \uad00\ub3d9\uae4c\uc9c0 \ub3d9\ubd81\uc77c\ubcf8 \uc9c0\uac01\uc740 \uc6d0\ub798\ub294 \ud0dc\ud3c9\uc591\ud310\uc774 \uc544\ub798\ub85c \uc12d\uc785\ud558\uba74\uc11c \uc11c\ucabd\uc73c\ub85c \uc9c0\uac01\ubcc0\ub3d9\uc744 \ubc1b\uace0 \uc788\uc5c8\uc73c\ub098, \ub3d9\uc77c\ubcf8\ub300\uc9c0\uc9c4 \uc774\ud6c4\uc5d0\ub294 \uc9c0\uac01\uc774 \ubc18\ub300\ub85c \ub3d9\ucabd\uc73c\ub85c \uc751\ub825\uc744 \ubc1b\uace0 \uc788\uc73c\uba70, \uc774\ub85c \uc778\ud574 \uc9c0\uc9c4 \ud65c\ub3d9\uc774 \ubc14\ub014 \uc218 \uc788\ub2e4\ub294 \uc6b0\ub824\uac00 \ub098\uc624\uace0 \uc788\ub2e4. \ub610\ud55c, \ud0dc\ud3c9\uc591\ud310\uc774 \uac00\ub77c\uc549\uc544 \uc788\ub294 \uacf3 \uc704\uc758 \uc721\uc9c0\uc5d0\uc11c \uc9c4\uc6d0\uc5ed \ubd80\uadfc\uc740 \uc735\uae30\ud558\uace0 \uc788\uc73c\uba70, \uc9c4\uc6d0\uc5ed \uc11c\ucabd\uc5d0\uc11c\ub294 \uce68\uac15\ud558\uace0 \uc788\uc5b4 \ub3c4\ud638\ucfe0\uc5d0\uc11c \uad00\ub3d9\uc5d0 \uac78\uce5c \ud0dc\ud3c9\uc591 \uc5f0\uc548 \uc804 \uc9c0\uc5ed\uc774 \uc77c\uc81c\ud788 \uc9c0\ubc18 \uce68\ud558 \ud604\uc0c1\uc774 \uc77c\uc5b4\ub098 2\ucc28 \ud53c\ud574\ub97c \uc77c\uc73c\ud0a4\uace0 \uc788\ub2e4. \uc9c4\uc6d0\uc5d0 \uac00\uae4c\uc6b4 \uc9c0\uc5ed\uc77c\uc218\ub85d \uc774 \uc9c0\uac01\ubcc0\ub3d9 \ud604\uc0c1\uc774 \ud06c\uac8c \uc77c\uc5b4\ub098\uace0 \uc788\ub294\ub370, \uc5f0\uad6c\uc5d0 \ub530\ub77c\uc11c \uc9c4\uc6d0 \uadfc\ucc98\uc758 \ud574\uc800 \uc9c0\uc5ed\uc740 \ub3d9\ucabd\uc73c\ub85c 50m \uc774\uc0c1 \uc774\ub3d9\ud588\ub2e4\ub294, \uc138\uacc4 \uc9c0\uc9c4 \uad00\uce21 \uc0ac\uc0c1 \uc720\ub840\uc5c6\ub294 \uae09\uaca9\ud55c \uc774\ub3d9\ub3c4 \uc77c\uc5b4\ub0ac\ub2e4.", "sentence_answer": "\ub3c4\ud638\ucfe0 \uc9c0\ubc29 \uc804\uc5ed\uc774 \ub3d9\ucabd\uc73c\ub85c 10cm \uc774\uc0c1 \uc774\ub3d9 \ud558\ub294 \ub4f1 \uadf8 \uc601\ud5a5\uc740 \ub9e4\uc6b0 \ucef8\ub2e4.", "paragraph_id": "6528699-27-0", "paragraph_question": "question: \ub3d9\uc77c\ubcf8\ub300\uc9c0\uc9c4\uc758 \uc6d0\uc778\uc778 \ub290\ub9b0 \ubbf8\ub044\ub7ec\uc9d0 \ud604\uc0c1\uc73c\ub85c \uc778\ud55c \uc9c0\uac01\ubcc0\ub3d9\uc73c\ub85c \uc778\ud55c \ub3c4\ud638\ucfe0 \uc9c0\ubc29\uc758 \uc774\ub3d9 \uac70\ub9ac\ub294?, context: \ubcf8\uc9c4\uacfc \uc5ec\uc9c4\uc744 \ud3ec\ud568\ud55c \ub290\ub9b0 \ubbf8\ub044\ub7ec\uc9d0 \ud604\uc0c1\uc73c\ub85c \ud070 \uc9c0\uac01\ubcc0\ub3d9\uc774 \uc77c\uc5b4\ub0ac\ub2e4. \ub3c4\ud638\ucfe0 \uc9c0\ubc29 \uc804\uc5ed\uc774 \ub3d9\ucabd\uc73c\ub85c 10cm \uc774\uc0c1 \uc774\ub3d9 \ud558\ub294 \ub4f1 \uadf8 \uc601\ud5a5\uc740 \ub9e4\uc6b0 \ucef8\ub2e4. \ub3c4\ud638\ucfe0 \uc9c0\ubc29\ubd80\ud130 \uad00\ub3d9\uae4c\uc9c0 \ub3d9\ubd81\uc77c\ubcf8 \uc9c0\uac01\uc740 \uc6d0\ub798\ub294 \ud0dc\ud3c9\uc591\ud310\uc774 \uc544\ub798\ub85c \uc12d\uc785\ud558\uba74\uc11c \uc11c\ucabd\uc73c\ub85c \uc9c0\uac01\ubcc0\ub3d9\uc744 \ubc1b\uace0 \uc788\uc5c8\uc73c\ub098, \ub3d9\uc77c\ubcf8\ub300\uc9c0\uc9c4 \uc774\ud6c4\uc5d0\ub294 \uc9c0\uac01\uc774 \ubc18\ub300\ub85c \ub3d9\ucabd\uc73c\ub85c \uc751\ub825\uc744 \ubc1b\uace0 \uc788\uc73c\uba70, \uc774\ub85c \uc778\ud574 \uc9c0\uc9c4 \ud65c\ub3d9\uc774 \ubc14\ub014 \uc218 \uc788\ub2e4\ub294 \uc6b0\ub824\uac00 \ub098\uc624\uace0 \uc788\ub2e4. \ub610\ud55c, \ud0dc\ud3c9\uc591\ud310\uc774 \uac00\ub77c\uc549\uc544 \uc788\ub294 \uacf3 \uc704\uc758 \uc721\uc9c0\uc5d0\uc11c \uc9c4\uc6d0\uc5ed \ubd80\uadfc\uc740 \uc735\uae30\ud558\uace0 \uc788\uc73c\uba70, \uc9c4\uc6d0\uc5ed \uc11c\ucabd\uc5d0\uc11c\ub294 \uce68\uac15\ud558\uace0 \uc788\uc5b4 \ub3c4\ud638\ucfe0\uc5d0\uc11c \uad00\ub3d9\uc5d0 \uac78\uce5c \ud0dc\ud3c9\uc591 \uc5f0\uc548 \uc804 \uc9c0\uc5ed\uc774 \uc77c\uc81c\ud788 \uc9c0\ubc18 \uce68\ud558 \ud604\uc0c1\uc774 \uc77c\uc5b4\ub098 2\ucc28 \ud53c\ud574\ub97c \uc77c\uc73c\ud0a4\uace0 \uc788\ub2e4. \uc9c4\uc6d0\uc5d0 \uac00\uae4c\uc6b4 \uc9c0\uc5ed\uc77c\uc218\ub85d \uc774 \uc9c0\uac01\ubcc0\ub3d9 \ud604\uc0c1\uc774 \ud06c\uac8c \uc77c\uc5b4\ub098\uace0 \uc788\ub294\ub370, \uc5f0\uad6c\uc5d0 \ub530\ub77c\uc11c \uc9c4\uc6d0 \uadfc\ucc98\uc758 \ud574\uc800 \uc9c0\uc5ed\uc740 \ub3d9\ucabd\uc73c\ub85c 50m \uc774\uc0c1 \uc774\ub3d9\ud588\ub2e4\ub294, \uc138\uacc4 \uc9c0\uc9c4 \uad00\uce21 \uc0ac\uc0c1 \uc720\ub840\uc5c6\ub294 \uae09\uaca9\ud55c \uc774\ub3d9\ub3c4 \uc77c\uc5b4\ub0ac\ub2e4."} {"question": "\ud55c\uc591\ub300\ud559\uad50 \uc11c\uc6b8\ucea0\ud37c\uc2a4\ub294 \uc5b4\ub290 \uc740\ud589\uc758 ATM\uc774 \uc788\ub294\uac00?", "paragraph": "\ud55c\uc591\ub300\ud559\uad50 \uc11c\uc6b8\ucea0\ud37c\uc2a4\uc5d0\ub294 \ud559\uc0dd\ubcf5\uc9c0\uc640 \uad00\ub828\ud55c \uc5ec\ub7ec \uac00\uc9c0 \ud3b8\uc758 \uc2dc\uc124\ub4e4\uc774 \uc788\ub2e4. \uae08\uc735\uad00\ub828 \uc2dc\uc124\ub85c\ub294 \uc2e0\ud55c\uc740\ud589\uacfc \uad6d\ubbfc\uc740\ud589 ATM\uc774 \uc77c\uacf1 \uacf3\uc5d0 \uc788\uc73c\uba70, \uc2e0\ud55c\uc740\ud589 \ub450 \uac1c \uc9c0\uc810\uc774 \uc788\ub2e4. \ubcf5\uc9c0 \uc2dc\uc124 \uc911\uc2ec\uc758 \uac74\ubb3c\uc740 \ud559\uc0dd\ud68c\uad00, \ud55c\uc591\ud50c\ub77c\uc790, \ud589\uc6d0\ud30c\ud06c\uac00 \uc788\ub2e4. \ub610\ud55c \uc870\uacbd \uad00\ub828 \uc2dc\uc124\ub85c\ub294 \ubd84\uc218 \uc138 \uac1c, \uc778\uacf5 \ud3ed\ud3ec \ud55c \uac1c\uac00 \uc788\uc73c\uba70, \uc18c\uaddc\ubaa8\uc758 \ub179\uc9c0\uc2dc\uc124\ub4e4\uc774 \ucea0\ud37c\uc2a4 \uacf3\uacf3\uc5d0 \ub4e4\uc5b4\uc11c \uc788\ub2e4. \ub610\ud55c 158\uacc4\ub2e8 \uc27c\ud130\uc5d0\ub294 \ubc15\ubaa9\uc6d4 \uc2dc\ube44\uc640 \ud568\uaed8 \ubbfc\uc8fc\uc5f4\uc0ac \uae30\ub150\ube44\uac00 \uc788\ub2e4. \uac04\uc2dd\uad00\ub828 \ubcf5\uc9c0 \uc2dc\uc124\ub85c\ub294 \ucee4\ud53c\uc20d\uc774 \uc5f4\ud55c \uacf3\uc774 \uc788\uc73c\uba70, 24\uc2dc\uac04 \ud3b8\uc758\uc810 \ub124 \uacf3\uc744 \ud3ec\ud568\ud574 \ucd1d \uc5f4 \uac1c\uc758 \ub9e4\uc810\uc744 \uc6b4\uc601\ud558\uace0 \uc788\ub2e4. \uadf8 \uc678\uc5d0 \uc704\uc0dd \uc2dc\uc124\uc778 \uc628\uc218\uac00\ub3d9 \uc0e4\uc6cc\uc2e4\uc774 \uac74\ubb3c \uc5ec\uc12f \uacf3\uc5d0 \uc6b4\uc601\ub418\uace0 \uc788\uc73c\uba70 \uc11c\uc810 \ub450 \uacf3, \ubb38\uad6c\uc810 \uc138 \uacf3 , \ubcf5\uc0ac\uc2e4 / \uc5ec\ud589\uc0ac / \uc0ac\uc9c4\uad00 / \uae30\ub150\ud488\uc810 \ub4f1\uc774 \uc788\ub2e4.", "answer": "\uc2e0\ud55c\uc740\ud589\uacfc \uad6d\ubbfc\uc740\ud589", "sentence": "\uae08\uc735\uad00\ub828 \uc2dc\uc124\ub85c\ub294 \uc2e0\ud55c\uc740\ud589\uacfc \uad6d\ubbfc\uc740\ud589 ATM\uc774 \uc77c\uacf1 \uacf3\uc5d0 \uc788\uc73c\uba70, \uc2e0\ud55c\uc740\ud589 \ub450 \uac1c \uc9c0\uc810\uc774 \uc788\ub2e4.", "paragraph_sentence": "\ud55c\uc591\ub300\ud559\uad50 \uc11c\uc6b8\ucea0\ud37c\uc2a4\uc5d0\ub294 \ud559\uc0dd\ubcf5\uc9c0\uc640 \uad00\ub828\ud55c \uc5ec\ub7ec \uac00\uc9c0 \ud3b8\uc758 \uc2dc\uc124\ub4e4\uc774 \uc788\ub2e4. \uae08\uc735\uad00\ub828 \uc2dc\uc124\ub85c\ub294 \uc2e0\ud55c\uc740\ud589\uacfc \uad6d\ubbfc\uc740\ud589 ATM\uc774 \uc77c\uacf1 \uacf3\uc5d0 \uc788\uc73c\uba70, \uc2e0\ud55c\uc740\ud589 \ub450 \uac1c \uc9c0\uc810\uc774 \uc788\ub2e4. \ubcf5\uc9c0 \uc2dc\uc124 \uc911\uc2ec\uc758 \uac74\ubb3c\uc740 \ud559\uc0dd\ud68c\uad00, \ud55c\uc591\ud50c\ub77c\uc790, \ud589\uc6d0\ud30c\ud06c\uac00 \uc788\ub2e4. \ub610\ud55c \uc870\uacbd \uad00\ub828 \uc2dc\uc124\ub85c\ub294 \ubd84\uc218 \uc138 \uac1c, \uc778\uacf5 \ud3ed\ud3ec \ud55c \uac1c\uac00 \uc788\uc73c\uba70, \uc18c\uaddc\ubaa8\uc758 \ub179\uc9c0\uc2dc\uc124\ub4e4\uc774 \ucea0\ud37c\uc2a4 \uacf3\uacf3\uc5d0 \ub4e4\uc5b4\uc11c \uc788\ub2e4. \ub610\ud55c 158\uacc4\ub2e8 \uc27c\ud130\uc5d0\ub294 \ubc15\ubaa9\uc6d4 \uc2dc\ube44\uc640 \ud568\uaed8 \ubbfc\uc8fc\uc5f4\uc0ac \uae30\ub150\ube44\uac00 \uc788\ub2e4. \uac04\uc2dd\uad00\ub828 \ubcf5\uc9c0 \uc2dc\uc124\ub85c\ub294 \ucee4\ud53c\uc20d\uc774 \uc5f4\ud55c \uacf3\uc774 \uc788\uc73c\uba70, 24\uc2dc\uac04 \ud3b8\uc758\uc810 \ub124 \uacf3\uc744 \ud3ec\ud568\ud574 \ucd1d \uc5f4 \uac1c\uc758 \ub9e4\uc810\uc744 \uc6b4\uc601\ud558\uace0 \uc788\ub2e4. \uadf8 \uc678\uc5d0 \uc704\uc0dd \uc2dc\uc124\uc778 \uc628\uc218\uac00\ub3d9 \uc0e4\uc6cc\uc2e4\uc774 \uac74\ubb3c \uc5ec\uc12f \uacf3\uc5d0 \uc6b4\uc601\ub418\uace0 \uc788\uc73c\uba70 \uc11c\uc810 \ub450 \uacf3, \ubb38\uad6c\uc810 \uc138 \uacf3 , \ubcf5\uc0ac\uc2e4 / \uc5ec\ud589\uc0ac / \uc0ac\uc9c4\uad00 / \uae30\ub150\ud488\uc810 \ub4f1\uc774 \uc788\ub2e4.", "paragraph_answer": "\ud55c\uc591\ub300\ud559\uad50 \uc11c\uc6b8\ucea0\ud37c\uc2a4\uc5d0\ub294 \ud559\uc0dd\ubcf5\uc9c0\uc640 \uad00\ub828\ud55c \uc5ec\ub7ec \uac00\uc9c0 \ud3b8\uc758 \uc2dc\uc124\ub4e4\uc774 \uc788\ub2e4. \uae08\uc735\uad00\ub828 \uc2dc\uc124\ub85c\ub294 \uc2e0\ud55c\uc740\ud589\uacfc \uad6d\ubbfc\uc740\ud589 ATM\uc774 \uc77c\uacf1 \uacf3\uc5d0 \uc788\uc73c\uba70, \uc2e0\ud55c\uc740\ud589 \ub450 \uac1c \uc9c0\uc810\uc774 \uc788\ub2e4. \ubcf5\uc9c0 \uc2dc\uc124 \uc911\uc2ec\uc758 \uac74\ubb3c\uc740 \ud559\uc0dd\ud68c\uad00, \ud55c\uc591\ud50c\ub77c\uc790, \ud589\uc6d0\ud30c\ud06c\uac00 \uc788\ub2e4. \ub610\ud55c \uc870\uacbd \uad00\ub828 \uc2dc\uc124\ub85c\ub294 \ubd84\uc218 \uc138 \uac1c, \uc778\uacf5 \ud3ed\ud3ec \ud55c \uac1c\uac00 \uc788\uc73c\uba70, \uc18c\uaddc\ubaa8\uc758 \ub179\uc9c0\uc2dc\uc124\ub4e4\uc774 \ucea0\ud37c\uc2a4 \uacf3\uacf3\uc5d0 \ub4e4\uc5b4\uc11c \uc788\ub2e4. \ub610\ud55c 158\uacc4\ub2e8 \uc27c\ud130\uc5d0\ub294 \ubc15\ubaa9\uc6d4 \uc2dc\ube44\uc640 \ud568\uaed8 \ubbfc\uc8fc\uc5f4\uc0ac \uae30\ub150\ube44\uac00 \uc788\ub2e4. \uac04\uc2dd\uad00\ub828 \ubcf5\uc9c0 \uc2dc\uc124\ub85c\ub294 \ucee4\ud53c\uc20d\uc774 \uc5f4\ud55c \uacf3\uc774 \uc788\uc73c\uba70, 24\uc2dc\uac04 \ud3b8\uc758\uc810 \ub124 \uacf3\uc744 \ud3ec\ud568\ud574 \ucd1d \uc5f4 \uac1c\uc758 \ub9e4\uc810\uc744 \uc6b4\uc601\ud558\uace0 \uc788\ub2e4. \uadf8 \uc678\uc5d0 \uc704\uc0dd \uc2dc\uc124\uc778 \uc628\uc218\uac00\ub3d9 \uc0e4\uc6cc\uc2e4\uc774 \uac74\ubb3c \uc5ec\uc12f \uacf3\uc5d0 \uc6b4\uc601\ub418\uace0 \uc788\uc73c\uba70 \uc11c\uc810 \ub450 \uacf3, \ubb38\uad6c\uc810 \uc138 \uacf3 , \ubcf5\uc0ac\uc2e4 / \uc5ec\ud589\uc0ac / \uc0ac\uc9c4\uad00 / \uae30\ub150\ud488\uc810 \ub4f1\uc774 \uc788\ub2e4.", "sentence_answer": "\uae08\uc735\uad00\ub828 \uc2dc\uc124\ub85c\ub294 \uc2e0\ud55c\uc740\ud589\uacfc \uad6d\ubbfc\uc740\ud589 ATM\uc774 \uc77c\uacf1 \uacf3\uc5d0 \uc788\uc73c\uba70, \uc2e0\ud55c\uc740\ud589 \ub450 \uac1c \uc9c0\uc810\uc774 \uc788\ub2e4.", "paragraph_id": "6570118-11-2", "paragraph_question": "question: \ud55c\uc591\ub300\ud559\uad50 \uc11c\uc6b8\ucea0\ud37c\uc2a4\ub294 \uc5b4\ub290 \uc740\ud589\uc758 ATM\uc774 \uc788\ub294\uac00?, context: \ud55c\uc591\ub300\ud559\uad50 \uc11c\uc6b8\ucea0\ud37c\uc2a4\uc5d0\ub294 \ud559\uc0dd\ubcf5\uc9c0\uc640 \uad00\ub828\ud55c \uc5ec\ub7ec \uac00\uc9c0 \ud3b8\uc758 \uc2dc\uc124\ub4e4\uc774 \uc788\ub2e4. \uae08\uc735\uad00\ub828 \uc2dc\uc124\ub85c\ub294 \uc2e0\ud55c\uc740\ud589\uacfc \uad6d\ubbfc\uc740\ud589 ATM\uc774 \uc77c\uacf1 \uacf3\uc5d0 \uc788\uc73c\uba70, \uc2e0\ud55c\uc740\ud589 \ub450 \uac1c \uc9c0\uc810\uc774 \uc788\ub2e4. \ubcf5\uc9c0 \uc2dc\uc124 \uc911\uc2ec\uc758 \uac74\ubb3c\uc740 \ud559\uc0dd\ud68c\uad00, \ud55c\uc591\ud50c\ub77c\uc790, \ud589\uc6d0\ud30c\ud06c\uac00 \uc788\ub2e4. \ub610\ud55c \uc870\uacbd \uad00\ub828 \uc2dc\uc124\ub85c\ub294 \ubd84\uc218 \uc138 \uac1c, \uc778\uacf5 \ud3ed\ud3ec \ud55c \uac1c\uac00 \uc788\uc73c\uba70, \uc18c\uaddc\ubaa8\uc758 \ub179\uc9c0\uc2dc\uc124\ub4e4\uc774 \ucea0\ud37c\uc2a4 \uacf3\uacf3\uc5d0 \ub4e4\uc5b4\uc11c \uc788\ub2e4. \ub610\ud55c 158\uacc4\ub2e8 \uc27c\ud130\uc5d0\ub294 \ubc15\ubaa9\uc6d4 \uc2dc\ube44\uc640 \ud568\uaed8 \ubbfc\uc8fc\uc5f4\uc0ac \uae30\ub150\ube44\uac00 \uc788\ub2e4. \uac04\uc2dd\uad00\ub828 \ubcf5\uc9c0 \uc2dc\uc124\ub85c\ub294 \ucee4\ud53c\uc20d\uc774 \uc5f4\ud55c \uacf3\uc774 \uc788\uc73c\uba70, 24\uc2dc\uac04 \ud3b8\uc758\uc810 \ub124 \uacf3\uc744 \ud3ec\ud568\ud574 \ucd1d \uc5f4 \uac1c\uc758 \ub9e4\uc810\uc744 \uc6b4\uc601\ud558\uace0 \uc788\ub2e4. \uadf8 \uc678\uc5d0 \uc704\uc0dd \uc2dc\uc124\uc778 \uc628\uc218\uac00\ub3d9 \uc0e4\uc6cc\uc2e4\uc774 \uac74\ubb3c \uc5ec\uc12f \uacf3\uc5d0 \uc6b4\uc601\ub418\uace0 \uc788\uc73c\uba70 \uc11c\uc810 \ub450 \uacf3, \ubb38\uad6c\uc810 \uc138 \uacf3 , \ubcf5\uc0ac\uc2e4 / \uc5ec\ud589\uc0ac / \uc0ac\uc9c4\uad00 / \uae30\ub150\ud488\uc810 \ub4f1\uc774 \uc788\ub2e4."} {"question": "\uc544\uc77c\ub79c\ub4dc \ubbfc\uc871\uc8fc\uc758\uc790\ub4e4\uc740 \ucfe0 \ud6cc\ub9b0\uc744 \ubb50\ub77c\uace0 \ud574\uc11d\ud558\ub294\uac00?", "paragraph": "\uc544\uc77c\ub79c\ub4dc \ubbfc\uc871\uc8fc\uc758\uc640 \uc5bc\uc2a4\ud130 \uc5f0\ud569\uc8fc\uc758\ub77c\ub294 \uc11c\ub85c \uc0c1\ubc18\ub41c \ub450 \uc774\ub150\uc774 \ubaa8\ub450 \ucfe0 \ud6cc\ub9b0\uc758 \uc774\ubbf8\uc9c0\ub97c \uc774\uc6a9\ud558\uace4 \ud55c\ub2e4. \ubbfc\uc871\uc8fc\uc758\uc790\ub4e4\uc740 \ucfe0 \ud6cc\ub9b0\uc744 \uac00\uc7a5 \uc720\uba85\ud55c \ucf08\ud2b8 \uc601\uc6c5\uc73c\ub85c \ud574\uc11d\ud558\uace0, \uadf8\uac00 \uc544\uc77c\ub79c\ub4dc\uc758 \ucf08\ud2b8 \ubb38\ud654\uc5d0 \uc788\uc5b4 \uc911\uc694\ud55c \uc874\uc7ac\ub77c\uace0 \ubcf8\ub2e4. \ub354\ube14\ub9b0 \uc911\uc559\uc6b0\uccb4\uad6d(GPO)\uc5d0\ub294 \uc62c\ub9ac\ubc84 \uc170\ud37c\ub4dc\uac00 \ub9cc\ub4e0 \uc8fd\uc5b4\uac00\ub294 \ucfe0 \ud6cc\ub9b0 \uccad\ub3d9\uc0c1\uc774 \uc788\ub294\ub370, 1911\ub144 \uc81c\uc791\ud588\ub2e4\uac00 \ub098\uc911\uc5d0 1916\ub144 \ubd80\ud65c\uc808 \ubd09\uae30\ub97c \uae30\ub150\ud558\uba74\uc11c \ud604\uc7ac \uc911\uc559\uc6b0\uccb4\uad6d\uc73c\ub85c \uc62e\uaca8\uc628 \uac83\uc774\ub2e4. \ud55c\ud3b8 \uc5f0\ud569\uc8fc\uc758\uc790\ub4e4\uc740 \ucfe0 \ud6cc\ub9b0\uc744 \ub0a8\ubd80 \uc544\uc77c\ub79c\ub4dc(\ucf54\ub178\ud2b8)\uc758 \uce68\ub7b5\uc790\ub4e4\ub85c\ubd80\ud130 \uc5bc\uc2a4\ud130(\uc624\ub298\ub0a0\uc758 \ubd81\uc544\uc77c\ub79c\ub4dc)\ub97c \uc218\ud638\ud55c \uc601\uc6c5\uc774\ub77c\uace0 \ud574\uc11d\ud55c\ub2e4. \uc608\ucee8\ub300 \ubca8\ud30c\uc2a4\ud2b8\uc5d0\ub294 \ucfe0 \ud6cc\ub9b0\uc744 \u201c\uc544\uc77c\ub79c\ub4dc\uc778\uc758 \uce68\ub7b5\uc5d0 \ub9de\uc120 \uc5bc\uc2a4\ud130\uc758 \ubc29\uc5b4\uc790\u201d\ub77c\uace0 \ubb18\uc0ac\ud55c \ubcbd\ud654\uac00 \ub450 \uc810 \uc788\ub2e4. \ub354\uc6b1 \uc544\uc774\ub7ec\ub2c8\ud55c \uac83\uc740 \uc774 \ub450 \ubcbd\ud654 \ubaa8\ub450 \uc170\ud37c\ub4dc\uc758 \uccad\ub3d9\uc0c1\uc744 \uae30\ubc18\uc73c\ub85c \uadf8\ub824\uc84c\ub2e4\ub294 \uac83\uc774\ub2e4. \ubb3c\ub860 \ubd81\uc544\uc77c\ub79c\ub4dc\uc758 \ubbfc\uc871\uc8fc\uc758\uc790\ub4e4\ub3c4 \ucfe0 \ud6cc\ub9b0\uc744 \uc790\uc2e0\ub4e4\uc758 \uc0c1\uc9d5\uc73c\ub85c \uc4f0\uace0 \uc788\ub2e4. 1966\ub144 \uc544\uc77c\ub79c\ub4dc\uc5d0\uc11c \ubc1c\ud589\ub41c 10\uc2e4\ub9c1 \uc8fc\ud654\uc5d0\ub3c4 \uc8fd\uc5b4\uac00\ub294 \ucfe0 \ud6cc\ub9b0 \ub3d9\uc0c1\uc774 \uc0c8\uaca8\uc838 \uc788\ub2e4. \ub610\ud55c \uc544\uc77c\ub79c\ub4dc \uad6d\ubc29\uad70\uc758 \ud6c8\uc7a5\uc5d0\ub3c4 \ucfe0 \ud6cc\ub9b0\uc774 \ub3cb\uc744\uc0c8\uae40\ub418\uc5b4 \uc788\ub294\ub370, \uc774\ub294 \ucfe0 \ud6cc\ub9b0\uc774 \uadf8\ub4e4\uc744 \uad7d\uc5b4 \uc0b4\ud3b4\ubcf8\ub2e4\ub294 \uc758\ubbf8\ub97c \uac16\uace0 \uc788\ub2e4.", "answer": "\uac00\uc7a5 \uc720\uba85\ud55c \ucf08\ud2b8 \uc601\uc6c5", "sentence": "\ubbfc\uc871\uc8fc\uc758\uc790\ub4e4\uc740 \ucfe0 \ud6cc\ub9b0\uc744 \uac00\uc7a5 \uc720\uba85\ud55c \ucf08\ud2b8 \uc601\uc6c5 \uc73c\ub85c \ud574\uc11d\ud558\uace0, \uadf8\uac00 \uc544\uc77c\ub79c\ub4dc\uc758 \ucf08\ud2b8 \ubb38\ud654\uc5d0 \uc788\uc5b4 \uc911\uc694\ud55c \uc874\uc7ac\ub77c\uace0 \ubcf8\ub2e4.", "paragraph_sentence": "\uc544\uc77c\ub79c\ub4dc \ubbfc\uc871\uc8fc\uc758\uc640 \uc5bc\uc2a4\ud130 \uc5f0\ud569\uc8fc\uc758\ub77c\ub294 \uc11c\ub85c \uc0c1\ubc18\ub41c \ub450 \uc774\ub150\uc774 \ubaa8\ub450 \ucfe0 \ud6cc\ub9b0\uc758 \uc774\ubbf8\uc9c0\ub97c \uc774\uc6a9\ud558\uace4 \ud55c\ub2e4. \ubbfc\uc871\uc8fc\uc758\uc790\ub4e4\uc740 \ucfe0 \ud6cc\ub9b0\uc744 \uac00\uc7a5 \uc720\uba85\ud55c \ucf08\ud2b8 \uc601\uc6c5 \uc73c\ub85c \ud574\uc11d\ud558\uace0, \uadf8\uac00 \uc544\uc77c\ub79c\ub4dc\uc758 \ucf08\ud2b8 \ubb38\ud654\uc5d0 \uc788\uc5b4 \uc911\uc694\ud55c \uc874\uc7ac\ub77c\uace0 \ubcf8\ub2e4. \ub354\ube14\ub9b0 \uc911\uc559\uc6b0\uccb4\uad6d(GPO)\uc5d0\ub294 \uc62c\ub9ac\ubc84 \uc170\ud37c\ub4dc\uac00 \ub9cc\ub4e0 \uc8fd\uc5b4\uac00\ub294 \ucfe0 \ud6cc\ub9b0 \uccad\ub3d9\uc0c1\uc774 \uc788\ub294\ub370, 1911\ub144 \uc81c\uc791\ud588\ub2e4\uac00 \ub098\uc911\uc5d0 1916\ub144 \ubd80\ud65c\uc808 \ubd09\uae30\ub97c \uae30\ub150\ud558\uba74\uc11c \ud604\uc7ac \uc911\uc559\uc6b0\uccb4\uad6d\uc73c\ub85c \uc62e\uaca8\uc628 \uac83\uc774\ub2e4. \ud55c\ud3b8 \uc5f0\ud569\uc8fc\uc758\uc790\ub4e4\uc740 \ucfe0 \ud6cc\ub9b0\uc744 \ub0a8\ubd80 \uc544\uc77c\ub79c\ub4dc(\ucf54\ub178\ud2b8)\uc758 \uce68\ub7b5\uc790\ub4e4\ub85c\ubd80\ud130 \uc5bc\uc2a4\ud130(\uc624\ub298\ub0a0\uc758 \ubd81\uc544\uc77c\ub79c\ub4dc)\ub97c \uc218\ud638\ud55c \uc601\uc6c5\uc774\ub77c\uace0 \ud574\uc11d\ud55c\ub2e4. \uc608\ucee8\ub300 \ubca8\ud30c\uc2a4\ud2b8\uc5d0\ub294 \ucfe0 \ud6cc\ub9b0\uc744 \u201c\uc544\uc77c\ub79c\ub4dc\uc778\uc758 \uce68\ub7b5\uc5d0 \ub9de\uc120 \uc5bc\uc2a4\ud130\uc758 \ubc29\uc5b4\uc790\u201d\ub77c\uace0 \ubb18\uc0ac\ud55c \ubcbd\ud654\uac00 \ub450 \uc810 \uc788\ub2e4. \ub354\uc6b1 \uc544\uc774\ub7ec\ub2c8\ud55c \uac83\uc740 \uc774 \ub450 \ubcbd\ud654 \ubaa8\ub450 \uc170\ud37c\ub4dc\uc758 \uccad\ub3d9\uc0c1\uc744 \uae30\ubc18\uc73c\ub85c \uadf8\ub824\uc84c\ub2e4\ub294 \uac83\uc774\ub2e4. \ubb3c\ub860 \ubd81\uc544\uc77c\ub79c\ub4dc\uc758 \ubbfc\uc871\uc8fc\uc758\uc790\ub4e4\ub3c4 \ucfe0 \ud6cc\ub9b0\uc744 \uc790\uc2e0\ub4e4\uc758 \uc0c1\uc9d5\uc73c\ub85c \uc4f0\uace0 \uc788\ub2e4. 1966\ub144 \uc544\uc77c\ub79c\ub4dc\uc5d0\uc11c \ubc1c\ud589\ub41c 10\uc2e4\ub9c1 \uc8fc\ud654\uc5d0\ub3c4 \uc8fd\uc5b4\uac00\ub294 \ucfe0 \ud6cc\ub9b0 \ub3d9\uc0c1\uc774 \uc0c8\uaca8\uc838 \uc788\ub2e4. \ub610\ud55c \uc544\uc77c\ub79c\ub4dc \uad6d\ubc29\uad70\uc758 \ud6c8\uc7a5\uc5d0\ub3c4 \ucfe0 \ud6cc\ub9b0\uc774 \ub3cb\uc744\uc0c8\uae40\ub418\uc5b4 \uc788\ub294\ub370, \uc774\ub294 \ucfe0 \ud6cc\ub9b0\uc774 \uadf8\ub4e4\uc744 \uad7d\uc5b4 \uc0b4\ud3b4\ubcf8\ub2e4\ub294 \uc758\ubbf8\ub97c \uac16\uace0 \uc788\ub2e4.", "paragraph_answer": "\uc544\uc77c\ub79c\ub4dc \ubbfc\uc871\uc8fc\uc758\uc640 \uc5bc\uc2a4\ud130 \uc5f0\ud569\uc8fc\uc758\ub77c\ub294 \uc11c\ub85c \uc0c1\ubc18\ub41c \ub450 \uc774\ub150\uc774 \ubaa8\ub450 \ucfe0 \ud6cc\ub9b0\uc758 \uc774\ubbf8\uc9c0\ub97c \uc774\uc6a9\ud558\uace4 \ud55c\ub2e4. \ubbfc\uc871\uc8fc\uc758\uc790\ub4e4\uc740 \ucfe0 \ud6cc\ub9b0\uc744 \uac00\uc7a5 \uc720\uba85\ud55c \ucf08\ud2b8 \uc601\uc6c5 \uc73c\ub85c \ud574\uc11d\ud558\uace0, \uadf8\uac00 \uc544\uc77c\ub79c\ub4dc\uc758 \ucf08\ud2b8 \ubb38\ud654\uc5d0 \uc788\uc5b4 \uc911\uc694\ud55c \uc874\uc7ac\ub77c\uace0 \ubcf8\ub2e4. \ub354\ube14\ub9b0 \uc911\uc559\uc6b0\uccb4\uad6d(GPO)\uc5d0\ub294 \uc62c\ub9ac\ubc84 \uc170\ud37c\ub4dc\uac00 \ub9cc\ub4e0 \uc8fd\uc5b4\uac00\ub294 \ucfe0 \ud6cc\ub9b0 \uccad\ub3d9\uc0c1\uc774 \uc788\ub294\ub370, 1911\ub144 \uc81c\uc791\ud588\ub2e4\uac00 \ub098\uc911\uc5d0 1916\ub144 \ubd80\ud65c\uc808 \ubd09\uae30\ub97c \uae30\ub150\ud558\uba74\uc11c \ud604\uc7ac \uc911\uc559\uc6b0\uccb4\uad6d\uc73c\ub85c \uc62e\uaca8\uc628 \uac83\uc774\ub2e4. \ud55c\ud3b8 \uc5f0\ud569\uc8fc\uc758\uc790\ub4e4\uc740 \ucfe0 \ud6cc\ub9b0\uc744 \ub0a8\ubd80 \uc544\uc77c\ub79c\ub4dc(\ucf54\ub178\ud2b8)\uc758 \uce68\ub7b5\uc790\ub4e4\ub85c\ubd80\ud130 \uc5bc\uc2a4\ud130(\uc624\ub298\ub0a0\uc758 \ubd81\uc544\uc77c\ub79c\ub4dc)\ub97c \uc218\ud638\ud55c \uc601\uc6c5\uc774\ub77c\uace0 \ud574\uc11d\ud55c\ub2e4. \uc608\ucee8\ub300 \ubca8\ud30c\uc2a4\ud2b8\uc5d0\ub294 \ucfe0 \ud6cc\ub9b0\uc744 \u201c\uc544\uc77c\ub79c\ub4dc\uc778\uc758 \uce68\ub7b5\uc5d0 \ub9de\uc120 \uc5bc\uc2a4\ud130\uc758 \ubc29\uc5b4\uc790\u201d\ub77c\uace0 \ubb18\uc0ac\ud55c \ubcbd\ud654\uac00 \ub450 \uc810 \uc788\ub2e4. \ub354\uc6b1 \uc544\uc774\ub7ec\ub2c8\ud55c \uac83\uc740 \uc774 \ub450 \ubcbd\ud654 \ubaa8\ub450 \uc170\ud37c\ub4dc\uc758 \uccad\ub3d9\uc0c1\uc744 \uae30\ubc18\uc73c\ub85c \uadf8\ub824\uc84c\ub2e4\ub294 \uac83\uc774\ub2e4. \ubb3c\ub860 \ubd81\uc544\uc77c\ub79c\ub4dc\uc758 \ubbfc\uc871\uc8fc\uc758\uc790\ub4e4\ub3c4 \ucfe0 \ud6cc\ub9b0\uc744 \uc790\uc2e0\ub4e4\uc758 \uc0c1\uc9d5\uc73c\ub85c \uc4f0\uace0 \uc788\ub2e4. 1966\ub144 \uc544\uc77c\ub79c\ub4dc\uc5d0\uc11c \ubc1c\ud589\ub41c 10\uc2e4\ub9c1 \uc8fc\ud654\uc5d0\ub3c4 \uc8fd\uc5b4\uac00\ub294 \ucfe0 \ud6cc\ub9b0 \ub3d9\uc0c1\uc774 \uc0c8\uaca8\uc838 \uc788\ub2e4. \ub610\ud55c \uc544\uc77c\ub79c\ub4dc \uad6d\ubc29\uad70\uc758 \ud6c8\uc7a5\uc5d0\ub3c4 \ucfe0 \ud6cc\ub9b0\uc774 \ub3cb\uc744\uc0c8\uae40\ub418\uc5b4 \uc788\ub294\ub370, \uc774\ub294 \ucfe0 \ud6cc\ub9b0\uc774 \uadf8\ub4e4\uc744 \uad7d\uc5b4 \uc0b4\ud3b4\ubcf8\ub2e4\ub294 \uc758\ubbf8\ub97c \uac16\uace0 \uc788\ub2e4.", "sentence_answer": "\ubbfc\uc871\uc8fc\uc758\uc790\ub4e4\uc740 \ucfe0 \ud6cc\ub9b0\uc744 \uac00\uc7a5 \uc720\uba85\ud55c \ucf08\ud2b8 \uc601\uc6c5 \uc73c\ub85c \ud574\uc11d\ud558\uace0, \uadf8\uac00 \uc544\uc77c\ub79c\ub4dc\uc758 \ucf08\ud2b8 \ubb38\ud654\uc5d0 \uc788\uc5b4 \uc911\uc694\ud55c \uc874\uc7ac\ub77c\uace0 \ubcf8\ub2e4.", "paragraph_id": "6397689-18-0", "paragraph_question": "question: \uc544\uc77c\ub79c\ub4dc \ubbfc\uc871\uc8fc\uc758\uc790\ub4e4\uc740 \ucfe0 \ud6cc\ub9b0\uc744 \ubb50\ub77c\uace0 \ud574\uc11d\ud558\ub294\uac00?, context: \uc544\uc77c\ub79c\ub4dc \ubbfc\uc871\uc8fc\uc758\uc640 \uc5bc\uc2a4\ud130 \uc5f0\ud569\uc8fc\uc758\ub77c\ub294 \uc11c\ub85c \uc0c1\ubc18\ub41c \ub450 \uc774\ub150\uc774 \ubaa8\ub450 \ucfe0 \ud6cc\ub9b0\uc758 \uc774\ubbf8\uc9c0\ub97c \uc774\uc6a9\ud558\uace4 \ud55c\ub2e4. \ubbfc\uc871\uc8fc\uc758\uc790\ub4e4\uc740 \ucfe0 \ud6cc\ub9b0\uc744 \uac00\uc7a5 \uc720\uba85\ud55c \ucf08\ud2b8 \uc601\uc6c5\uc73c\ub85c \ud574\uc11d\ud558\uace0, \uadf8\uac00 \uc544\uc77c\ub79c\ub4dc\uc758 \ucf08\ud2b8 \ubb38\ud654\uc5d0 \uc788\uc5b4 \uc911\uc694\ud55c \uc874\uc7ac\ub77c\uace0 \ubcf8\ub2e4. \ub354\ube14\ub9b0 \uc911\uc559\uc6b0\uccb4\uad6d(GPO)\uc5d0\ub294 \uc62c\ub9ac\ubc84 \uc170\ud37c\ub4dc\uac00 \ub9cc\ub4e0 \uc8fd\uc5b4\uac00\ub294 \ucfe0 \ud6cc\ub9b0 \uccad\ub3d9\uc0c1\uc774 \uc788\ub294\ub370, 1911\ub144 \uc81c\uc791\ud588\ub2e4\uac00 \ub098\uc911\uc5d0 1916\ub144 \ubd80\ud65c\uc808 \ubd09\uae30\ub97c \uae30\ub150\ud558\uba74\uc11c \ud604\uc7ac \uc911\uc559\uc6b0\uccb4\uad6d\uc73c\ub85c \uc62e\uaca8\uc628 \uac83\uc774\ub2e4. \ud55c\ud3b8 \uc5f0\ud569\uc8fc\uc758\uc790\ub4e4\uc740 \ucfe0 \ud6cc\ub9b0\uc744 \ub0a8\ubd80 \uc544\uc77c\ub79c\ub4dc(\ucf54\ub178\ud2b8)\uc758 \uce68\ub7b5\uc790\ub4e4\ub85c\ubd80\ud130 \uc5bc\uc2a4\ud130(\uc624\ub298\ub0a0\uc758 \ubd81\uc544\uc77c\ub79c\ub4dc)\ub97c \uc218\ud638\ud55c \uc601\uc6c5\uc774\ub77c\uace0 \ud574\uc11d\ud55c\ub2e4. \uc608\ucee8\ub300 \ubca8\ud30c\uc2a4\ud2b8\uc5d0\ub294 \ucfe0 \ud6cc\ub9b0\uc744 \u201c\uc544\uc77c\ub79c\ub4dc\uc778\uc758 \uce68\ub7b5\uc5d0 \ub9de\uc120 \uc5bc\uc2a4\ud130\uc758 \ubc29\uc5b4\uc790\u201d\ub77c\uace0 \ubb18\uc0ac\ud55c \ubcbd\ud654\uac00 \ub450 \uc810 \uc788\ub2e4. \ub354\uc6b1 \uc544\uc774\ub7ec\ub2c8\ud55c \uac83\uc740 \uc774 \ub450 \ubcbd\ud654 \ubaa8\ub450 \uc170\ud37c\ub4dc\uc758 \uccad\ub3d9\uc0c1\uc744 \uae30\ubc18\uc73c\ub85c \uadf8\ub824\uc84c\ub2e4\ub294 \uac83\uc774\ub2e4. \ubb3c\ub860 \ubd81\uc544\uc77c\ub79c\ub4dc\uc758 \ubbfc\uc871\uc8fc\uc758\uc790\ub4e4\ub3c4 \ucfe0 \ud6cc\ub9b0\uc744 \uc790\uc2e0\ub4e4\uc758 \uc0c1\uc9d5\uc73c\ub85c \uc4f0\uace0 \uc788\ub2e4. 1966\ub144 \uc544\uc77c\ub79c\ub4dc\uc5d0\uc11c \ubc1c\ud589\ub41c 10\uc2e4\ub9c1 \uc8fc\ud654\uc5d0\ub3c4 \uc8fd\uc5b4\uac00\ub294 \ucfe0 \ud6cc\ub9b0 \ub3d9\uc0c1\uc774 \uc0c8\uaca8\uc838 \uc788\ub2e4. \ub610\ud55c \uc544\uc77c\ub79c\ub4dc \uad6d\ubc29\uad70\uc758 \ud6c8\uc7a5\uc5d0\ub3c4 \ucfe0 \ud6cc\ub9b0\uc774 \ub3cb\uc744\uc0c8\uae40\ub418\uc5b4 \uc788\ub294\ub370, \uc774\ub294 \ucfe0 \ud6cc\ub9b0\uc774 \uadf8\ub4e4\uc744 \uad7d\uc5b4 \uc0b4\ud3b4\ubcf8\ub2e4\ub294 \uc758\ubbf8\ub97c \uac16\uace0 \uc788\ub2e4."} {"question": "\ubb38\uc7ac\uc778 \ub300\ud1b5\ub839\uc758 \ub178\ub825\uc740 \uc911\ub300\ud55c \uc774\uc815\ud45c\ub97c \uc138\uc6e0\ub2e4\uace0 \ubcf4\ub3c4\ud55c \ubbf8\uad6d\uc758 \uc5b8\ub860\uc740 \uc5b4\ub514\uc778\uac00?", "paragraph": "\ub0a8\ubd81 \uc815\uc0c1\ud68c\ub2f4 \ud569\uc758\uc640 \ube44\ud575\ud654 \uc758\uc9c0 \ud45c\uba85\uc5d0 \ub300\ud574 \ud2b8\ub7fc\ud504\ub294 \"\ub9ce\uc740 \uc0ac\ub78c\uc774 \uc774\ub7f0 \ub0a0\uc774 \uc624\ub9ac\ub77c\uace0 \uc0dd\uac01\ud558\uc9c0 \ubabb\ud588\ub2e4\"\uace0 \ub9d0\ud558\uba70 \ud658\uc601 \uc785\uc7a5\uc744 \ubcf4\uc600\ub2e4. \ud2b8\ub7fc\ud504\ub294 \ud2b8\uc704\ud130\ub97c \ud1b5\ud574 \"\uc218\ub144 \ub9cc\uc5d0 \ucc98\uc74c\uc73c\ub85c \uc9c4\uc9c0\ud55c \ub178\ub825\uc774 \ubaa8\ub4e0 \uad00\ub828 \ub2f9\uc0ac\uc790\ub4e4\uc5d0 \uc758\ud574 \ud3bc\uccd0\uc9c0\uace0 \uc788\ub2e4\"\uba70 \"\ud5db\ub41c \ud76c\ub9dd\uc77c\uc9c0\ub3c4 \ubaa8\ub978\ub2e4. \uadf8\ub7ec\ub098 \ubbf8\uad6d\uc740 \uc5b4\ub290 \ubc29\ud5a5\uc774 \ub410\ub4e0 \uc5f4\uc2ec\ud788 \uac08 \uc900\ube44\uac00 \ub3fc \uc788\ub2e4\"\uace0 \ud588\ub2e4. \ub610 \"\uadf8\uac83\uc740 \uc804 \uc138\uacc4\ub97c \uc704\ud574 \uc704\ub300\ud55c \uc77c\uc774 \ub420 \uac83\uc774\uace0 \ubd81\ud55c\uc744 \uc704\ud574 \uc704\ub300\ud55c \uc77c\uc774 \ub420 \uac83\uc774\uba70, \ud55c\ubc18\ub3c4\ub97c \uc704\ud574 \uc704\ub300\ud55c \uc77c\uc774 \ub420 \uac83\uc774\uc9c0\ub9cc, \uc6b0\ub9ac\ub294 \ubb34\uc2a8 \uc77c\uc774 \uc77c\uc5b4\ub0a0\uc9c0 \ub450\uace0 \ubcfc \uac83\"\uc774\ub77c\uace0 \ud588\ub2e4. \uc9c0\ub09c \uba87 \ub2ec \uac04 \ubd81\ubbf8 \uc0ac\uc774\uc5d0 '\ub9d0\uc758 \uc804\uc7c1'\uc73c\ub85c \uc704\uae30\uac00 \uace0\uc870\ub410\ub358 \ud55c\ubc18\ub3c4 \uc815\uc138\uac00 \uc911\ub300\ud55c \ubd84\uc218\ub839\uc744 \ub9de\uac8c \ub418\uc5c8\ub2e4\ub294 \ubd84\uc11d\ub3c4 \uc788\ub2e4. \uad6d\ubb34\ubd80\ub3c4 \ub300\ubcc0\uc778 \ube0c\ub9ac\ud551\uc744 \ud1b5\ud574 \ube44\ud575\ud654 \uc758\uc9c0 \ud45c\uba85\uc5d0 \ub300\ud574 \"\uc633\uc740 \ubc29\ud5a5\uc5d0\uc11c \uc774\ub904\uc9c4 \uc870\uce58\ub77c\uace0 \ud655\uc2e0\ud55c\ub2e4\"\uba70 \ub2e4\uc74c \uc870\uce58\ub97c \ub17c\uc758\ud558\uaca0\ub2e4\uace0 \ubc1d\ud614\ub2e4. \u300a\uc6cc\uc2f1\ud134 \ud3ec\uc2a4\ud2b8\u300b\ub294 \"\uae40\uc815\uc740 \ub178\ub3d9\ub2f9 \uc704\uc6d0\uc7a5 \uc2a4\uc2a4\ub85c\uac00 \uba85\ubc31\ud788 \ubcf4\uc99d\ud55c \uadf8 \uc81c\uc548\uc740 \ubbf8\uad6d \ubcf8\ud1a0\ub97c \uc0ac\uac70\ub9ac\uc5d0 \ub450\uc5c8\ub358 \uc218\ub144\uac04\uc758 \ud575\uc2e4\ud5d8\uacfc \ubbf8\uc0ac\uc77c \uae30\uc220\uc758 \uc9c4\uc804 \uc774\ud6c4 \uc911\ub300\ud55c \ubc18\uc804\"\uc774\ub77c\uace0 \uc804\ud588\uc73c\uba70, \u300a\ub274\uc695 \ud0c0\uc784\uc2a4\u300b\ub294 \"\ubb38\uc7ac\uc778 \ub300\ud1b5\ub839\uc758 \ub178\ub825\uc740 \uc911\ub300\ud55c \uc774\uc815\ud45c\ub97c \uc138\uc6e0\ub2e4\"\uace0 \ubcf4\ub3c4\ud588\uc73c\uba70, CNN\uc740 \"\uc62c\ub9bc\ud53d\uc744 \uad00\uacc4 \ud574\ube59\uc758 \uacc4\uae30\ub85c \uc0bc\uc740 \ubb38 \ub300\ud1b5\ub839\uc758 \uc758\ubbf8 \uc788\ub294 \uc678\uad50\uc801 \uc131\uacfc\"\ub77c\uace0 \ud3c9\uac00\ud588\ub2e4. \ud558\uc9c0\ub9cc \ubbf8\uad6d\uc758 \uc815\ubcf4\uae30\uad00 \uc218\uc7a5\ub4e4\uc744 \ud68c\uc758\uc801\uc778 \ubc18\uc751\ub3c4 \ubcf4\uc600\ub2e4. \ub304 \ucf54\uce20 \uad6d\uac00\uc815\ubcf4\uad6d\uc7a5\uc740 \"\uc774 \ubaa8\ub4e0 \uac83\uc5d0 \ub300\ud574 \uc0c1\ub2f9\ud788 \ud68c\uc758\uc801\"\uc774\ub77c\uba70 \"\ub3cc\ud30c\uad6c\uac00 \ub9cc\ub4e4\uc5b4\uc9c4 \uac83\ucc98\ub7fc \ubcf4\uc77c \uc218 \uc788\uc73c\ub098, \uc0c1\ub2f9\ud788 \uc758\uc2ec\uc774 \ub4e0\ub2e4\"\uace0 \ud588\ub2e4.\ub85c\ubc84\ud2b8 \uc560\uc290\ub9ac \uad6d\ubc29\uc815\ubcf4\uad6d\uc7a5 \uc5ed\uc2dc \"\uae40\uc815\uc740\uc740 \ud575\u00b7\ud0c4\ub3c4\ubbf8\uc0ac\uc77c \ud504\ub85c\uadf8\ub7a8\uc744 \ud3ec\uae30\ud560 \uc758\ud5a5\uc744 \ubcf4\uc774\uc9c0 \uc54a\uace0 \uc788\ub2e4\"\uba70 \"\uae40\uc815\uc740\uc740 (\ubb34\ub825) \ucda9\ub3cc\uc5d0 \ub300\ube44\ud55c \uc900\ube44\ud0dc\uc138\ub97c \ub9e4\uc6b0 \uc9c4\uc9c0\ud558\uac8c \uc0dd\uac01\ud558\uace0 \uc788\ub2e4\"\uace0 \ub9d0\ud588\ub2e4.", "answer": "\ub274\uc695 \ud0c0\uc784\uc2a4", "sentence": "\u300a\uc6cc\uc2f1\ud134 \ud3ec\uc2a4\ud2b8\u300b\ub294 \"\uae40\uc815\uc740 \ub178\ub3d9\ub2f9 \uc704\uc6d0\uc7a5 \uc2a4\uc2a4\ub85c\uac00 \uba85\ubc31\ud788 \ubcf4\uc99d\ud55c \uadf8 \uc81c\uc548\uc740 \ubbf8\uad6d \ubcf8\ud1a0\ub97c \uc0ac\uac70\ub9ac\uc5d0 \ub450\uc5c8\ub358 \uc218\ub144\uac04\uc758 \ud575\uc2e4\ud5d8\uacfc \ubbf8\uc0ac\uc77c \uae30\uc220\uc758 \uc9c4\uc804 \uc774\ud6c4 \uc911\ub300\ud55c \ubc18\uc804\"\uc774\ub77c\uace0 \uc804\ud588\uc73c\uba70, \u300a \ub274\uc695 \ud0c0\uc784\uc2a4 \u300b\ub294 \"\ubb38\uc7ac\uc778 \ub300\ud1b5\ub839\uc758 \ub178\ub825\uc740 \uc911\ub300\ud55c \uc774\uc815\ud45c\ub97c \uc138\uc6e0\ub2e4\"\uace0 \ubcf4\ub3c4\ud588\uc73c\uba70, CNN\uc740 \"\uc62c\ub9bc\ud53d\uc744 \uad00\uacc4 \ud574\ube59\uc758 \uacc4\uae30\ub85c \uc0bc\uc740 \ubb38 \ub300\ud1b5\ub839\uc758 \uc758\ubbf8 \uc788\ub294 \uc678\uad50\uc801 \uc131\uacfc\"\ub77c\uace0 \ud3c9\uac00\ud588\ub2e4.", "paragraph_sentence": "\ub0a8\ubd81 \uc815\uc0c1\ud68c\ub2f4 \ud569\uc758\uc640 \ube44\ud575\ud654 \uc758\uc9c0 \ud45c\uba85\uc5d0 \ub300\ud574 \ud2b8\ub7fc\ud504\ub294 \"\ub9ce\uc740 \uc0ac\ub78c\uc774 \uc774\ub7f0 \ub0a0\uc774 \uc624\ub9ac\ub77c\uace0 \uc0dd\uac01\ud558\uc9c0 \ubabb\ud588\ub2e4\"\uace0 \ub9d0\ud558\uba70 \ud658\uc601 \uc785\uc7a5\uc744 \ubcf4\uc600\ub2e4. \ud2b8\ub7fc\ud504\ub294 \ud2b8\uc704\ud130\ub97c \ud1b5\ud574 \"\uc218\ub144 \ub9cc\uc5d0 \ucc98\uc74c\uc73c\ub85c \uc9c4\uc9c0\ud55c \ub178\ub825\uc774 \ubaa8\ub4e0 \uad00\ub828 \ub2f9\uc0ac\uc790\ub4e4\uc5d0 \uc758\ud574 \ud3bc\uccd0\uc9c0\uace0 \uc788\ub2e4\"\uba70 \"\ud5db\ub41c \ud76c\ub9dd\uc77c\uc9c0\ub3c4 \ubaa8\ub978\ub2e4. \uadf8\ub7ec\ub098 \ubbf8\uad6d\uc740 \uc5b4\ub290 \ubc29\ud5a5\uc774 \ub410\ub4e0 \uc5f4\uc2ec\ud788 \uac08 \uc900\ube44\uac00 \ub3fc \uc788\ub2e4\"\uace0 \ud588\ub2e4. \ub610 \"\uadf8\uac83\uc740 \uc804 \uc138\uacc4\ub97c \uc704\ud574 \uc704\ub300\ud55c \uc77c\uc774 \ub420 \uac83\uc774\uace0 \ubd81\ud55c\uc744 \uc704\ud574 \uc704\ub300\ud55c \uc77c\uc774 \ub420 \uac83\uc774\uba70, \ud55c\ubc18\ub3c4\ub97c \uc704\ud574 \uc704\ub300\ud55c \uc77c\uc774 \ub420 \uac83\uc774\uc9c0\ub9cc, \uc6b0\ub9ac\ub294 \ubb34\uc2a8 \uc77c\uc774 \uc77c\uc5b4\ub0a0\uc9c0 \ub450\uace0 \ubcfc \uac83\"\uc774\ub77c\uace0 \ud588\ub2e4. \uc9c0\ub09c \uba87 \ub2ec \uac04 \ubd81\ubbf8 \uc0ac\uc774\uc5d0 '\ub9d0\uc758 \uc804\uc7c1'\uc73c\ub85c \uc704\uae30\uac00 \uace0\uc870\ub410\ub358 \ud55c\ubc18\ub3c4 \uc815\uc138\uac00 \uc911\ub300\ud55c \ubd84\uc218\ub839\uc744 \ub9de\uac8c \ub418\uc5c8\ub2e4\ub294 \ubd84\uc11d\ub3c4 \uc788\ub2e4. \uad6d\ubb34\ubd80\ub3c4 \ub300\ubcc0\uc778 \ube0c\ub9ac\ud551\uc744 \ud1b5\ud574 \ube44\ud575\ud654 \uc758\uc9c0 \ud45c\uba85\uc5d0 \ub300\ud574 \"\uc633\uc740 \ubc29\ud5a5\uc5d0\uc11c \uc774\ub904\uc9c4 \uc870\uce58\ub77c\uace0 \ud655\uc2e0\ud55c\ub2e4\"\uba70 \ub2e4\uc74c \uc870\uce58\ub97c \ub17c\uc758\ud558\uaca0\ub2e4\uace0 \ubc1d\ud614\ub2e4. \u300a\uc6cc\uc2f1\ud134 \ud3ec\uc2a4\ud2b8\u300b\ub294 \"\uae40\uc815\uc740 \ub178\ub3d9\ub2f9 \uc704\uc6d0\uc7a5 \uc2a4\uc2a4\ub85c\uac00 \uba85\ubc31\ud788 \ubcf4\uc99d\ud55c \uadf8 \uc81c\uc548\uc740 \ubbf8\uad6d \ubcf8\ud1a0\ub97c \uc0ac\uac70\ub9ac\uc5d0 \ub450\uc5c8\ub358 \uc218\ub144\uac04\uc758 \ud575\uc2e4\ud5d8\uacfc \ubbf8\uc0ac\uc77c \uae30\uc220\uc758 \uc9c4\uc804 \uc774\ud6c4 \uc911\ub300\ud55c \ubc18\uc804\"\uc774\ub77c\uace0 \uc804\ud588\uc73c\uba70, \u300a \ub274\uc695 \ud0c0\uc784\uc2a4 \u300b\ub294 \"\ubb38\uc7ac\uc778 \ub300\ud1b5\ub839\uc758 \ub178\ub825\uc740 \uc911\ub300\ud55c \uc774\uc815\ud45c\ub97c \uc138\uc6e0\ub2e4\"\uace0 \ubcf4\ub3c4\ud588\uc73c\uba70, CNN\uc740 \"\uc62c\ub9bc\ud53d\uc744 \uad00\uacc4 \ud574\ube59\uc758 \uacc4\uae30\ub85c \uc0bc\uc740 \ubb38 \ub300\ud1b5\ub839\uc758 \uc758\ubbf8 \uc788\ub294 \uc678\uad50\uc801 \uc131\uacfc\"\ub77c\uace0 \ud3c9\uac00\ud588\ub2e4. \ud558\uc9c0\ub9cc \ubbf8\uad6d\uc758 \uc815\ubcf4\uae30\uad00 \uc218\uc7a5\ub4e4\uc744 \ud68c\uc758\uc801\uc778 \ubc18\uc751\ub3c4 \ubcf4\uc600\ub2e4. \ub304 \ucf54\uce20 \uad6d\uac00\uc815\ubcf4\uad6d\uc7a5\uc740 \"\uc774 \ubaa8\ub4e0 \uac83\uc5d0 \ub300\ud574 \uc0c1\ub2f9\ud788 \ud68c\uc758\uc801\"\uc774\ub77c\uba70 \"\ub3cc\ud30c\uad6c\uac00 \ub9cc\ub4e4\uc5b4\uc9c4 \uac83\ucc98\ub7fc \ubcf4\uc77c \uc218 \uc788\uc73c\ub098, \uc0c1\ub2f9\ud788 \uc758\uc2ec\uc774 \ub4e0\ub2e4\"\uace0 \ud588\ub2e4.\ub85c\ubc84\ud2b8 \uc560\uc290\ub9ac \uad6d\ubc29\uc815\ubcf4\uad6d\uc7a5 \uc5ed\uc2dc \"\uae40\uc815\uc740\uc740 \ud575\u00b7\ud0c4\ub3c4\ubbf8\uc0ac\uc77c \ud504\ub85c\uadf8\ub7a8\uc744 \ud3ec\uae30\ud560 \uc758\ud5a5\uc744 \ubcf4\uc774\uc9c0 \uc54a\uace0 \uc788\ub2e4\"\uba70 \"\uae40\uc815\uc740\uc740 (\ubb34\ub825) \ucda9\ub3cc\uc5d0 \ub300\ube44\ud55c \uc900\ube44\ud0dc\uc138\ub97c \ub9e4\uc6b0 \uc9c4\uc9c0\ud558\uac8c \uc0dd\uac01\ud558\uace0 \uc788\ub2e4\"\uace0 \ub9d0\ud588\ub2e4.", "paragraph_answer": "\ub0a8\ubd81 \uc815\uc0c1\ud68c\ub2f4 \ud569\uc758\uc640 \ube44\ud575\ud654 \uc758\uc9c0 \ud45c\uba85\uc5d0 \ub300\ud574 \ud2b8\ub7fc\ud504\ub294 \"\ub9ce\uc740 \uc0ac\ub78c\uc774 \uc774\ub7f0 \ub0a0\uc774 \uc624\ub9ac\ub77c\uace0 \uc0dd\uac01\ud558\uc9c0 \ubabb\ud588\ub2e4\"\uace0 \ub9d0\ud558\uba70 \ud658\uc601 \uc785\uc7a5\uc744 \ubcf4\uc600\ub2e4. \ud2b8\ub7fc\ud504\ub294 \ud2b8\uc704\ud130\ub97c \ud1b5\ud574 \"\uc218\ub144 \ub9cc\uc5d0 \ucc98\uc74c\uc73c\ub85c \uc9c4\uc9c0\ud55c \ub178\ub825\uc774 \ubaa8\ub4e0 \uad00\ub828 \ub2f9\uc0ac\uc790\ub4e4\uc5d0 \uc758\ud574 \ud3bc\uccd0\uc9c0\uace0 \uc788\ub2e4\"\uba70 \"\ud5db\ub41c \ud76c\ub9dd\uc77c\uc9c0\ub3c4 \ubaa8\ub978\ub2e4. \uadf8\ub7ec\ub098 \ubbf8\uad6d\uc740 \uc5b4\ub290 \ubc29\ud5a5\uc774 \ub410\ub4e0 \uc5f4\uc2ec\ud788 \uac08 \uc900\ube44\uac00 \ub3fc \uc788\ub2e4\"\uace0 \ud588\ub2e4. \ub610 \"\uadf8\uac83\uc740 \uc804 \uc138\uacc4\ub97c \uc704\ud574 \uc704\ub300\ud55c \uc77c\uc774 \ub420 \uac83\uc774\uace0 \ubd81\ud55c\uc744 \uc704\ud574 \uc704\ub300\ud55c \uc77c\uc774 \ub420 \uac83\uc774\uba70, \ud55c\ubc18\ub3c4\ub97c \uc704\ud574 \uc704\ub300\ud55c \uc77c\uc774 \ub420 \uac83\uc774\uc9c0\ub9cc, \uc6b0\ub9ac\ub294 \ubb34\uc2a8 \uc77c\uc774 \uc77c\uc5b4\ub0a0\uc9c0 \ub450\uace0 \ubcfc \uac83\"\uc774\ub77c\uace0 \ud588\ub2e4. \uc9c0\ub09c \uba87 \ub2ec \uac04 \ubd81\ubbf8 \uc0ac\uc774\uc5d0 '\ub9d0\uc758 \uc804\uc7c1'\uc73c\ub85c \uc704\uae30\uac00 \uace0\uc870\ub410\ub358 \ud55c\ubc18\ub3c4 \uc815\uc138\uac00 \uc911\ub300\ud55c \ubd84\uc218\ub839\uc744 \ub9de\uac8c \ub418\uc5c8\ub2e4\ub294 \ubd84\uc11d\ub3c4 \uc788\ub2e4. \uad6d\ubb34\ubd80\ub3c4 \ub300\ubcc0\uc778 \ube0c\ub9ac\ud551\uc744 \ud1b5\ud574 \ube44\ud575\ud654 \uc758\uc9c0 \ud45c\uba85\uc5d0 \ub300\ud574 \"\uc633\uc740 \ubc29\ud5a5\uc5d0\uc11c \uc774\ub904\uc9c4 \uc870\uce58\ub77c\uace0 \ud655\uc2e0\ud55c\ub2e4\"\uba70 \ub2e4\uc74c \uc870\uce58\ub97c \ub17c\uc758\ud558\uaca0\ub2e4\uace0 \ubc1d\ud614\ub2e4. \u300a\uc6cc\uc2f1\ud134 \ud3ec\uc2a4\ud2b8\u300b\ub294 \"\uae40\uc815\uc740 \ub178\ub3d9\ub2f9 \uc704\uc6d0\uc7a5 \uc2a4\uc2a4\ub85c\uac00 \uba85\ubc31\ud788 \ubcf4\uc99d\ud55c \uadf8 \uc81c\uc548\uc740 \ubbf8\uad6d \ubcf8\ud1a0\ub97c \uc0ac\uac70\ub9ac\uc5d0 \ub450\uc5c8\ub358 \uc218\ub144\uac04\uc758 \ud575\uc2e4\ud5d8\uacfc \ubbf8\uc0ac\uc77c \uae30\uc220\uc758 \uc9c4\uc804 \uc774\ud6c4 \uc911\ub300\ud55c \ubc18\uc804\"\uc774\ub77c\uace0 \uc804\ud588\uc73c\uba70, \u300a \ub274\uc695 \ud0c0\uc784\uc2a4 \u300b\ub294 \"\ubb38\uc7ac\uc778 \ub300\ud1b5\ub839\uc758 \ub178\ub825\uc740 \uc911\ub300\ud55c \uc774\uc815\ud45c\ub97c \uc138\uc6e0\ub2e4\"\uace0 \ubcf4\ub3c4\ud588\uc73c\uba70, CNN\uc740 \"\uc62c\ub9bc\ud53d\uc744 \uad00\uacc4 \ud574\ube59\uc758 \uacc4\uae30\ub85c \uc0bc\uc740 \ubb38 \ub300\ud1b5\ub839\uc758 \uc758\ubbf8 \uc788\ub294 \uc678\uad50\uc801 \uc131\uacfc\"\ub77c\uace0 \ud3c9\uac00\ud588\ub2e4. \ud558\uc9c0\ub9cc \ubbf8\uad6d\uc758 \uc815\ubcf4\uae30\uad00 \uc218\uc7a5\ub4e4\uc744 \ud68c\uc758\uc801\uc778 \ubc18\uc751\ub3c4 \ubcf4\uc600\ub2e4. \ub304 \ucf54\uce20 \uad6d\uac00\uc815\ubcf4\uad6d\uc7a5\uc740 \"\uc774 \ubaa8\ub4e0 \uac83\uc5d0 \ub300\ud574 \uc0c1\ub2f9\ud788 \ud68c\uc758\uc801\"\uc774\ub77c\uba70 \"\ub3cc\ud30c\uad6c\uac00 \ub9cc\ub4e4\uc5b4\uc9c4 \uac83\ucc98\ub7fc \ubcf4\uc77c \uc218 \uc788\uc73c\ub098, \uc0c1\ub2f9\ud788 \uc758\uc2ec\uc774 \ub4e0\ub2e4\"\uace0 \ud588\ub2e4.\ub85c\ubc84\ud2b8 \uc560\uc290\ub9ac \uad6d\ubc29\uc815\ubcf4\uad6d\uc7a5 \uc5ed\uc2dc \"\uae40\uc815\uc740\uc740 \ud575\u00b7\ud0c4\ub3c4\ubbf8\uc0ac\uc77c \ud504\ub85c\uadf8\ub7a8\uc744 \ud3ec\uae30\ud560 \uc758\ud5a5\uc744 \ubcf4\uc774\uc9c0 \uc54a\uace0 \uc788\ub2e4\"\uba70 \"\uae40\uc815\uc740\uc740 (\ubb34\ub825) \ucda9\ub3cc\uc5d0 \ub300\ube44\ud55c \uc900\ube44\ud0dc\uc138\ub97c \ub9e4\uc6b0 \uc9c4\uc9c0\ud558\uac8c \uc0dd\uac01\ud558\uace0 \uc788\ub2e4\"\uace0 \ub9d0\ud588\ub2e4.", "sentence_answer": "\u300a\uc6cc\uc2f1\ud134 \ud3ec\uc2a4\ud2b8\u300b\ub294 \"\uae40\uc815\uc740 \ub178\ub3d9\ub2f9 \uc704\uc6d0\uc7a5 \uc2a4\uc2a4\ub85c\uac00 \uba85\ubc31\ud788 \ubcf4\uc99d\ud55c \uadf8 \uc81c\uc548\uc740 \ubbf8\uad6d \ubcf8\ud1a0\ub97c \uc0ac\uac70\ub9ac\uc5d0 \ub450\uc5c8\ub358 \uc218\ub144\uac04\uc758 \ud575\uc2e4\ud5d8\uacfc \ubbf8\uc0ac\uc77c \uae30\uc220\uc758 \uc9c4\uc804 \uc774\ud6c4 \uc911\ub300\ud55c \ubc18\uc804\"\uc774\ub77c\uace0 \uc804\ud588\uc73c\uba70, \u300a \ub274\uc695 \ud0c0\uc784\uc2a4 \u300b\ub294 \"\ubb38\uc7ac\uc778 \ub300\ud1b5\ub839\uc758 \ub178\ub825\uc740 \uc911\ub300\ud55c \uc774\uc815\ud45c\ub97c \uc138\uc6e0\ub2e4\"\uace0 \ubcf4\ub3c4\ud588\uc73c\uba70, CNN\uc740 \"\uc62c\ub9bc\ud53d\uc744 \uad00\uacc4 \ud574\ube59\uc758 \uacc4\uae30\ub85c \uc0bc\uc740 \ubb38 \ub300\ud1b5\ub839\uc758 \uc758\ubbf8 \uc788\ub294 \uc678\uad50\uc801 \uc131\uacfc\"\ub77c\uace0 \ud3c9\uac00\ud588\ub2e4.", "paragraph_id": "6580768-44-0", "paragraph_question": "question: \ubb38\uc7ac\uc778 \ub300\ud1b5\ub839\uc758 \ub178\ub825\uc740 \uc911\ub300\ud55c \uc774\uc815\ud45c\ub97c \uc138\uc6e0\ub2e4\uace0 \ubcf4\ub3c4\ud55c \ubbf8\uad6d\uc758 \uc5b8\ub860\uc740 \uc5b4\ub514\uc778\uac00?, context: \ub0a8\ubd81 \uc815\uc0c1\ud68c\ub2f4 \ud569\uc758\uc640 \ube44\ud575\ud654 \uc758\uc9c0 \ud45c\uba85\uc5d0 \ub300\ud574 \ud2b8\ub7fc\ud504\ub294 \"\ub9ce\uc740 \uc0ac\ub78c\uc774 \uc774\ub7f0 \ub0a0\uc774 \uc624\ub9ac\ub77c\uace0 \uc0dd\uac01\ud558\uc9c0 \ubabb\ud588\ub2e4\"\uace0 \ub9d0\ud558\uba70 \ud658\uc601 \uc785\uc7a5\uc744 \ubcf4\uc600\ub2e4. \ud2b8\ub7fc\ud504\ub294 \ud2b8\uc704\ud130\ub97c \ud1b5\ud574 \"\uc218\ub144 \ub9cc\uc5d0 \ucc98\uc74c\uc73c\ub85c \uc9c4\uc9c0\ud55c \ub178\ub825\uc774 \ubaa8\ub4e0 \uad00\ub828 \ub2f9\uc0ac\uc790\ub4e4\uc5d0 \uc758\ud574 \ud3bc\uccd0\uc9c0\uace0 \uc788\ub2e4\"\uba70 \"\ud5db\ub41c \ud76c\ub9dd\uc77c\uc9c0\ub3c4 \ubaa8\ub978\ub2e4. \uadf8\ub7ec\ub098 \ubbf8\uad6d\uc740 \uc5b4\ub290 \ubc29\ud5a5\uc774 \ub410\ub4e0 \uc5f4\uc2ec\ud788 \uac08 \uc900\ube44\uac00 \ub3fc \uc788\ub2e4\"\uace0 \ud588\ub2e4. \ub610 \"\uadf8\uac83\uc740 \uc804 \uc138\uacc4\ub97c \uc704\ud574 \uc704\ub300\ud55c \uc77c\uc774 \ub420 \uac83\uc774\uace0 \ubd81\ud55c\uc744 \uc704\ud574 \uc704\ub300\ud55c \uc77c\uc774 \ub420 \uac83\uc774\uba70, \ud55c\ubc18\ub3c4\ub97c \uc704\ud574 \uc704\ub300\ud55c \uc77c\uc774 \ub420 \uac83\uc774\uc9c0\ub9cc, \uc6b0\ub9ac\ub294 \ubb34\uc2a8 \uc77c\uc774 \uc77c\uc5b4\ub0a0\uc9c0 \ub450\uace0 \ubcfc \uac83\"\uc774\ub77c\uace0 \ud588\ub2e4. \uc9c0\ub09c \uba87 \ub2ec \uac04 \ubd81\ubbf8 \uc0ac\uc774\uc5d0 '\ub9d0\uc758 \uc804\uc7c1'\uc73c\ub85c \uc704\uae30\uac00 \uace0\uc870\ub410\ub358 \ud55c\ubc18\ub3c4 \uc815\uc138\uac00 \uc911\ub300\ud55c \ubd84\uc218\ub839\uc744 \ub9de\uac8c \ub418\uc5c8\ub2e4\ub294 \ubd84\uc11d\ub3c4 \uc788\ub2e4. \uad6d\ubb34\ubd80\ub3c4 \ub300\ubcc0\uc778 \ube0c\ub9ac\ud551\uc744 \ud1b5\ud574 \ube44\ud575\ud654 \uc758\uc9c0 \ud45c\uba85\uc5d0 \ub300\ud574 \"\uc633\uc740 \ubc29\ud5a5\uc5d0\uc11c \uc774\ub904\uc9c4 \uc870\uce58\ub77c\uace0 \ud655\uc2e0\ud55c\ub2e4\"\uba70 \ub2e4\uc74c \uc870\uce58\ub97c \ub17c\uc758\ud558\uaca0\ub2e4\uace0 \ubc1d\ud614\ub2e4. \u300a\uc6cc\uc2f1\ud134 \ud3ec\uc2a4\ud2b8\u300b\ub294 \"\uae40\uc815\uc740 \ub178\ub3d9\ub2f9 \uc704\uc6d0\uc7a5 \uc2a4\uc2a4\ub85c\uac00 \uba85\ubc31\ud788 \ubcf4\uc99d\ud55c \uadf8 \uc81c\uc548\uc740 \ubbf8\uad6d \ubcf8\ud1a0\ub97c \uc0ac\uac70\ub9ac\uc5d0 \ub450\uc5c8\ub358 \uc218\ub144\uac04\uc758 \ud575\uc2e4\ud5d8\uacfc \ubbf8\uc0ac\uc77c \uae30\uc220\uc758 \uc9c4\uc804 \uc774\ud6c4 \uc911\ub300\ud55c \ubc18\uc804\"\uc774\ub77c\uace0 \uc804\ud588\uc73c\uba70, \u300a\ub274\uc695 \ud0c0\uc784\uc2a4\u300b\ub294 \"\ubb38\uc7ac\uc778 \ub300\ud1b5\ub839\uc758 \ub178\ub825\uc740 \uc911\ub300\ud55c \uc774\uc815\ud45c\ub97c \uc138\uc6e0\ub2e4\"\uace0 \ubcf4\ub3c4\ud588\uc73c\uba70, CNN\uc740 \"\uc62c\ub9bc\ud53d\uc744 \uad00\uacc4 \ud574\ube59\uc758 \uacc4\uae30\ub85c \uc0bc\uc740 \ubb38 \ub300\ud1b5\ub839\uc758 \uc758\ubbf8 \uc788\ub294 \uc678\uad50\uc801 \uc131\uacfc\"\ub77c\uace0 \ud3c9\uac00\ud588\ub2e4. \ud558\uc9c0\ub9cc \ubbf8\uad6d\uc758 \uc815\ubcf4\uae30\uad00 \uc218\uc7a5\ub4e4\uc744 \ud68c\uc758\uc801\uc778 \ubc18\uc751\ub3c4 \ubcf4\uc600\ub2e4. \ub304 \ucf54\uce20 \uad6d\uac00\uc815\ubcf4\uad6d\uc7a5\uc740 \"\uc774 \ubaa8\ub4e0 \uac83\uc5d0 \ub300\ud574 \uc0c1\ub2f9\ud788 \ud68c\uc758\uc801\"\uc774\ub77c\uba70 \"\ub3cc\ud30c\uad6c\uac00 \ub9cc\ub4e4\uc5b4\uc9c4 \uac83\ucc98\ub7fc \ubcf4\uc77c \uc218 \uc788\uc73c\ub098, \uc0c1\ub2f9\ud788 \uc758\uc2ec\uc774 \ub4e0\ub2e4\"\uace0 \ud588\ub2e4.\ub85c\ubc84\ud2b8 \uc560\uc290\ub9ac \uad6d\ubc29\uc815\ubcf4\uad6d\uc7a5 \uc5ed\uc2dc \"\uae40\uc815\uc740\uc740 \ud575\u00b7\ud0c4\ub3c4\ubbf8\uc0ac\uc77c \ud504\ub85c\uadf8\ub7a8\uc744 \ud3ec\uae30\ud560 \uc758\ud5a5\uc744 \ubcf4\uc774\uc9c0 \uc54a\uace0 \uc788\ub2e4\"\uba70 \"\uae40\uc815\uc740\uc740 (\ubb34\ub825) \ucda9\ub3cc\uc5d0 \ub300\ube44\ud55c \uc900\ube44\ud0dc\uc138\ub97c \ub9e4\uc6b0 \uc9c4\uc9c0\ud558\uac8c \uc0dd\uac01\ud558\uace0 \uc788\ub2e4\"\uace0 \ub9d0\ud588\ub2e4."} {"question": "\uace0\ub300 \ub85c\ub9c8\uc778\uc774 \uc0ac\uc6a9\ud558\ub358 \ub77c\ud2f4\uc5b4 \ubc0f \uadf8 \ubc29\uc5b8\uc5d0\uc11c \ubc1c\uc0dd\ud55c \uc5b8\uc5b4\uad70\uc744 \uac00\ub9ac\ucf1c \ubb34\uc5c7\uc774\ub77c\uace0 \ud558\ub294\uac00?", "paragraph": "\uc5b8\uc5b4 \ubd84\ub958\ud559\uc0c1 \ub85c\ub9dd\uc2a4\uc5b4\uc5d0 \uc18d\ud558\ub294 \uc5b8\uc5b4\uc774\ub2e4. \ub85c\ub9dd\uc2a4\uc5b4\ub294 \uadf8 \uc774\ub984\uc774 \ub73b\ud558\ub4ef\uc774 \uace0\ub300 \ub85c\ub9c8\uc778\uc774 \uc4f0\ub358 \ub77c\ud2f4\uc5b4\uc5d0\uc11c, \ub610\ub294 \uc774\uc758 \ubc29\uc5b8\uc5d0\uc11c \ubc1c\uc0dd\ud55c \uc5b8\uc5b4\uad70\uc744 \uc77c\uceeb\ub294\ub2e4. \ud558\uc9c0\ub9cc \ud504\ub791\uc2a4\uc5b4\ub294 \uace0\uc804 \ub77c\ud2f4\uc5b4(Latin classique)\uc5d0\uc11c \uc720\ub798\ud55c \uac83\uc774 \uc544\ub2c8\ub77c \uc138\uc18d \ub77c\ud2f4\uc5b4(Latin vulgaire)\uc5d0\uc11c \uc720\ub798\ud55c \uac83\uc73c\ub85c \ube44\uad50\uc801 \uac04\ub2e8\ud55c \ud1b5\uc0ac\uad6c\uc870\uc640 \uad74\uc808\uc774 \uac70\uc758 \uc0c1\uc2e4\ub41c \ud615\ud0dc\ub97c \uac16\ucd94\uace0 \uc788\ub2e4. \uae30\uc6d0\uc804 1\uc138\uae30 \ud6c4\ubc18\uc5d0 \ud604\uc7ac\uc758 \ud504\ub791\uc2a4 \uc9c0\uc5ed\uc774 \ub85c\ub9c8 \uc81c\uad6d\uc758 \uc9c0\ubc30\ud558\uc5d0 \ub4e4\uc5b4\uac04 \ud6c4, \ub85c\ub9c8\uc758 \uad70\uc778, \uc0c1\uc778, \uc774\uc8fc\ubbfc \ub4f1\uc758 \uc7a6\uc740 \uc655\ub798, \uadf8\ub9ac\uace0 \uadf8\ub4e4\uacfc \uac08\ub9ac\uc544(\ud504\ub791\uc2a4\uc758 \ub77c\ud2f4\uc5b4 \uc774\ub984)\uc758 \uc6d0\uc8fc\ubbfc\ub4e4\uacfc\uc758 \uad50\uc12d\uc744 \ud1b5\ud558\uc5ec \ud504\ub791\uc2a4\uc5b4\uc758 \ubaa8\uccb4\uac00 \ub418\ub294 \uc0c8\ub85c\uc6b4 \uc5b8\uc5b4 \u2018\uac08\ub9ac\uc544 \uc11c\ubbfc \ub77c\ud2f4\uc5b4(Latin populaire des Gaules)\u2019\uac00 \uc0dd\uaca8\ub0ac\ub2e4. \uc774 \uac08\ub9ac\uc544 \uc11c\ubbfc \ub77c\ud2f4\uc5b4\ub294 \uadf8 \ub2f9\uc2dc\uc5d0 \uc0c8\uaca8\uc9c4 \ub610\ub294 \uc791\uc131\ub41c \ube44\ubb38\uacfc \uae30\ub85d\ubb38\uc11c\uc5d0\uc11c \uc0b4\ud3b4\ubcfc \uc218 \uc788\ub4ef\uc774 \ubb38\ubc95, \ubc1c\uc74c, \uc5b4\ud718 \ub4f1\uc5d0\uc11c \ub77c\ud2f4\uc5b4\uc640 \ucee4\ub2e4\ub780 \ucc28\uc774\uac00 \uc788\ub2e4.", "answer": "\ub85c\ub9dd\uc2a4\uc5b4", "sentence": "\uc5b8\uc5b4 \ubd84\ub958\ud559\uc0c1 \ub85c\ub9dd\uc2a4\uc5b4 \uc5d0 \uc18d\ud558\ub294 \uc5b8\uc5b4\uc774\ub2e4.", "paragraph_sentence": " \uc5b8\uc5b4 \ubd84\ub958\ud559\uc0c1 \ub85c\ub9dd\uc2a4\uc5b4 \uc5d0 \uc18d\ud558\ub294 \uc5b8\uc5b4\uc774\ub2e4. \ub85c\ub9dd\uc2a4\uc5b4\ub294 \uadf8 \uc774\ub984\uc774 \ub73b\ud558\ub4ef\uc774 \uace0\ub300 \ub85c\ub9c8\uc778\uc774 \uc4f0\ub358 \ub77c\ud2f4\uc5b4\uc5d0\uc11c, \ub610\ub294 \uc774\uc758 \ubc29\uc5b8\uc5d0\uc11c \ubc1c\uc0dd\ud55c \uc5b8\uc5b4\uad70\uc744 \uc77c\uceeb\ub294\ub2e4. \ud558\uc9c0\ub9cc \ud504\ub791\uc2a4\uc5b4\ub294 \uace0\uc804 \ub77c\ud2f4\uc5b4(Latin classique)\uc5d0\uc11c \uc720\ub798\ud55c \uac83\uc774 \uc544\ub2c8\ub77c \uc138\uc18d \ub77c\ud2f4\uc5b4(Latin vulgaire)\uc5d0\uc11c \uc720\ub798\ud55c \uac83\uc73c\ub85c \ube44\uad50\uc801 \uac04\ub2e8\ud55c \ud1b5\uc0ac\uad6c\uc870\uc640 \uad74\uc808\uc774 \uac70\uc758 \uc0c1\uc2e4\ub41c \ud615\ud0dc\ub97c \uac16\ucd94\uace0 \uc788\ub2e4. \uae30\uc6d0\uc804 1\uc138\uae30 \ud6c4\ubc18\uc5d0 \ud604\uc7ac\uc758 \ud504\ub791\uc2a4 \uc9c0\uc5ed\uc774 \ub85c\ub9c8 \uc81c\uad6d\uc758 \uc9c0\ubc30\ud558\uc5d0 \ub4e4\uc5b4\uac04 \ud6c4, \ub85c\ub9c8\uc758 \uad70\uc778, \uc0c1\uc778, \uc774\uc8fc\ubbfc \ub4f1\uc758 \uc7a6\uc740 \uc655\ub798, \uadf8\ub9ac\uace0 \uadf8\ub4e4\uacfc \uac08\ub9ac\uc544(\ud504\ub791\uc2a4\uc758 \ub77c\ud2f4\uc5b4 \uc774\ub984)\uc758 \uc6d0\uc8fc\ubbfc\ub4e4\uacfc\uc758 \uad50\uc12d\uc744 \ud1b5\ud558\uc5ec \ud504\ub791\uc2a4\uc5b4\uc758 \ubaa8\uccb4\uac00 \ub418\ub294 \uc0c8\ub85c\uc6b4 \uc5b8\uc5b4 \u2018\uac08\ub9ac\uc544 \uc11c\ubbfc \ub77c\ud2f4\uc5b4(Latin populaire des Gaules)\u2019\uac00 \uc0dd\uaca8\ub0ac\ub2e4. \uc774 \uac08\ub9ac\uc544 \uc11c\ubbfc \ub77c\ud2f4\uc5b4\ub294 \uadf8 \ub2f9\uc2dc\uc5d0 \uc0c8\uaca8\uc9c4 \ub610\ub294 \uc791\uc131\ub41c \ube44\ubb38\uacfc \uae30\ub85d\ubb38\uc11c\uc5d0\uc11c \uc0b4\ud3b4\ubcfc \uc218 \uc788\ub4ef\uc774 \ubb38\ubc95, \ubc1c\uc74c, \uc5b4\ud718 \ub4f1\uc5d0\uc11c \ub77c\ud2f4\uc5b4\uc640 \ucee4\ub2e4\ub780 \ucc28\uc774\uac00 \uc788\ub2e4.", "paragraph_answer": "\uc5b8\uc5b4 \ubd84\ub958\ud559\uc0c1 \ub85c\ub9dd\uc2a4\uc5b4 \uc5d0 \uc18d\ud558\ub294 \uc5b8\uc5b4\uc774\ub2e4. \ub85c\ub9dd\uc2a4\uc5b4\ub294 \uadf8 \uc774\ub984\uc774 \ub73b\ud558\ub4ef\uc774 \uace0\ub300 \ub85c\ub9c8\uc778\uc774 \uc4f0\ub358 \ub77c\ud2f4\uc5b4\uc5d0\uc11c, \ub610\ub294 \uc774\uc758 \ubc29\uc5b8\uc5d0\uc11c \ubc1c\uc0dd\ud55c \uc5b8\uc5b4\uad70\uc744 \uc77c\uceeb\ub294\ub2e4. \ud558\uc9c0\ub9cc \ud504\ub791\uc2a4\uc5b4\ub294 \uace0\uc804 \ub77c\ud2f4\uc5b4(Latin classique)\uc5d0\uc11c \uc720\ub798\ud55c \uac83\uc774 \uc544\ub2c8\ub77c \uc138\uc18d \ub77c\ud2f4\uc5b4(Latin vulgaire)\uc5d0\uc11c \uc720\ub798\ud55c \uac83\uc73c\ub85c \ube44\uad50\uc801 \uac04\ub2e8\ud55c \ud1b5\uc0ac\uad6c\uc870\uc640 \uad74\uc808\uc774 \uac70\uc758 \uc0c1\uc2e4\ub41c \ud615\ud0dc\ub97c \uac16\ucd94\uace0 \uc788\ub2e4. \uae30\uc6d0\uc804 1\uc138\uae30 \ud6c4\ubc18\uc5d0 \ud604\uc7ac\uc758 \ud504\ub791\uc2a4 \uc9c0\uc5ed\uc774 \ub85c\ub9c8 \uc81c\uad6d\uc758 \uc9c0\ubc30\ud558\uc5d0 \ub4e4\uc5b4\uac04 \ud6c4, \ub85c\ub9c8\uc758 \uad70\uc778, \uc0c1\uc778, \uc774\uc8fc\ubbfc \ub4f1\uc758 \uc7a6\uc740 \uc655\ub798, \uadf8\ub9ac\uace0 \uadf8\ub4e4\uacfc \uac08\ub9ac\uc544(\ud504\ub791\uc2a4\uc758 \ub77c\ud2f4\uc5b4 \uc774\ub984)\uc758 \uc6d0\uc8fc\ubbfc\ub4e4\uacfc\uc758 \uad50\uc12d\uc744 \ud1b5\ud558\uc5ec \ud504\ub791\uc2a4\uc5b4\uc758 \ubaa8\uccb4\uac00 \ub418\ub294 \uc0c8\ub85c\uc6b4 \uc5b8\uc5b4 \u2018\uac08\ub9ac\uc544 \uc11c\ubbfc \ub77c\ud2f4\uc5b4(Latin populaire des Gaules)\u2019\uac00 \uc0dd\uaca8\ub0ac\ub2e4. \uc774 \uac08\ub9ac\uc544 \uc11c\ubbfc \ub77c\ud2f4\uc5b4\ub294 \uadf8 \ub2f9\uc2dc\uc5d0 \uc0c8\uaca8\uc9c4 \ub610\ub294 \uc791\uc131\ub41c \ube44\ubb38\uacfc \uae30\ub85d\ubb38\uc11c\uc5d0\uc11c \uc0b4\ud3b4\ubcfc \uc218 \uc788\ub4ef\uc774 \ubb38\ubc95, \ubc1c\uc74c, \uc5b4\ud718 \ub4f1\uc5d0\uc11c \ub77c\ud2f4\uc5b4\uc640 \ucee4\ub2e4\ub780 \ucc28\uc774\uac00 \uc788\ub2e4.", "sentence_answer": "\uc5b8\uc5b4 \ubd84\ub958\ud559\uc0c1 \ub85c\ub9dd\uc2a4\uc5b4 \uc5d0 \uc18d\ud558\ub294 \uc5b8\uc5b4\uc774\ub2e4.", "paragraph_id": "6568592-0-0", "paragraph_question": "question: \uace0\ub300 \ub85c\ub9c8\uc778\uc774 \uc0ac\uc6a9\ud558\ub358 \ub77c\ud2f4\uc5b4 \ubc0f \uadf8 \ubc29\uc5b8\uc5d0\uc11c \ubc1c\uc0dd\ud55c \uc5b8\uc5b4\uad70\uc744 \uac00\ub9ac\ucf1c \ubb34\uc5c7\uc774\ub77c\uace0 \ud558\ub294\uac00?, context: \uc5b8\uc5b4 \ubd84\ub958\ud559\uc0c1 \ub85c\ub9dd\uc2a4\uc5b4\uc5d0 \uc18d\ud558\ub294 \uc5b8\uc5b4\uc774\ub2e4. \ub85c\ub9dd\uc2a4\uc5b4\ub294 \uadf8 \uc774\ub984\uc774 \ub73b\ud558\ub4ef\uc774 \uace0\ub300 \ub85c\ub9c8\uc778\uc774 \uc4f0\ub358 \ub77c\ud2f4\uc5b4\uc5d0\uc11c, \ub610\ub294 \uc774\uc758 \ubc29\uc5b8\uc5d0\uc11c \ubc1c\uc0dd\ud55c \uc5b8\uc5b4\uad70\uc744 \uc77c\uceeb\ub294\ub2e4. \ud558\uc9c0\ub9cc \ud504\ub791\uc2a4\uc5b4\ub294 \uace0\uc804 \ub77c\ud2f4\uc5b4(Latin classique)\uc5d0\uc11c \uc720\ub798\ud55c \uac83\uc774 \uc544\ub2c8\ub77c \uc138\uc18d \ub77c\ud2f4\uc5b4(Latin vulgaire)\uc5d0\uc11c \uc720\ub798\ud55c \uac83\uc73c\ub85c \ube44\uad50\uc801 \uac04\ub2e8\ud55c \ud1b5\uc0ac\uad6c\uc870\uc640 \uad74\uc808\uc774 \uac70\uc758 \uc0c1\uc2e4\ub41c \ud615\ud0dc\ub97c \uac16\ucd94\uace0 \uc788\ub2e4. \uae30\uc6d0\uc804 1\uc138\uae30 \ud6c4\ubc18\uc5d0 \ud604\uc7ac\uc758 \ud504\ub791\uc2a4 \uc9c0\uc5ed\uc774 \ub85c\ub9c8 \uc81c\uad6d\uc758 \uc9c0\ubc30\ud558\uc5d0 \ub4e4\uc5b4\uac04 \ud6c4, \ub85c\ub9c8\uc758 \uad70\uc778, \uc0c1\uc778, \uc774\uc8fc\ubbfc \ub4f1\uc758 \uc7a6\uc740 \uc655\ub798, \uadf8\ub9ac\uace0 \uadf8\ub4e4\uacfc \uac08\ub9ac\uc544(\ud504\ub791\uc2a4\uc758 \ub77c\ud2f4\uc5b4 \uc774\ub984)\uc758 \uc6d0\uc8fc\ubbfc\ub4e4\uacfc\uc758 \uad50\uc12d\uc744 \ud1b5\ud558\uc5ec \ud504\ub791\uc2a4\uc5b4\uc758 \ubaa8\uccb4\uac00 \ub418\ub294 \uc0c8\ub85c\uc6b4 \uc5b8\uc5b4 \u2018\uac08\ub9ac\uc544 \uc11c\ubbfc \ub77c\ud2f4\uc5b4(Latin populaire des Gaules)\u2019\uac00 \uc0dd\uaca8\ub0ac\ub2e4. \uc774 \uac08\ub9ac\uc544 \uc11c\ubbfc \ub77c\ud2f4\uc5b4\ub294 \uadf8 \ub2f9\uc2dc\uc5d0 \uc0c8\uaca8\uc9c4 \ub610\ub294 \uc791\uc131\ub41c \ube44\ubb38\uacfc \uae30\ub85d\ubb38\uc11c\uc5d0\uc11c \uc0b4\ud3b4\ubcfc \uc218 \uc788\ub4ef\uc774 \ubb38\ubc95, \ubc1c\uc74c, \uc5b4\ud718 \ub4f1\uc5d0\uc11c \ub77c\ud2f4\uc5b4\uc640 \ucee4\ub2e4\ub780 \ucc28\uc774\uac00 \uc788\ub2e4."} {"question": "1990\ub144\uc5d0 \uc774\uc2dc\ubbf8\ub124 \uac00\uc988\ud788\ucf54\uac00 \uc88c\uc775\uc218\ub85c \uc120\ubc1c \ucd9c\uc804\ud55c \uacbd\uae30\uc758 \uc22b\uc790\ub294?", "paragraph": "\uc774\ub4ec\ud574 1990\ub144\uc5d0\ub294 \uc791\ub144 \uc2dc\uc98c\uc758 \ubc18\uc131\uc744 \uacc4\uae30\ub85c \uac00\ub3c4\ud0c0\uc640 \uc9c0\uba85 \ud0c0\uc790 \uc790\ub9ac\ub97c \uacbd\uc7c1\ud558\uc9c0 \uc54a\uace0 \uc88c\uc775\uc218\ub85c \ucd9c\uc804\ud558\uaca0\ub2e4\ub294 \uc758\uc0ac\ub97c \uad73\ud600 \uccb4\uc911\ub3c4 80kg\uc73c\ub85c \ub418\ub3cc\ub838\ub2e4. \uc2dc\uc98c\uc5d0 \ub4e4\uc5b4\uac00\uba74\uc11c 4\uc6d4 26\uc77c \uc138\uc774\ubd80\uc640\uc758 \uacbd\uae30\uc5d0\uc11c\ubd80\ud130 4\uc6d4 30\uc77c \uae34\ud14c\uc4f0\uc640\uc758 \uacbd\uae30\uc5d0 \uac78\uccd0 \ud37c\uc2dc\ud53d \ub9ac\uadf8 \uc5ed\ub300 2\uc704 \ud0c0\uc774 \uae30\ub85d\uc778 4\uacbd\uae30 \uc5f0\uc18d \ubcd1\uc0b4\ud0c0\ub97c \uae30\ub85d\ud588\uc9c0\ub9cc \uc624\ub808\uc2a4\ud14c\uc2a4 \ub370\uc2a4\ud2b8\ub77c\ub370\ub098 \uae30\uc694\ud558\ub77c \uac00\uc988\ud788\ub85c \ub4f1\uacfc \uc2dc\uc98c \uc885\ubc18\uae4c\uc9c0 \ud648\ub7f0\uc655 \ud0c0\uc774\ud2c0\uc744 \ub193\uace0 \uacbd\uc7c1\ud558\ub294 \ud65c\uc57d\uc744 \ubcf4\uc600\ub2e4. \ub610 3\ub144 \ub9cc\uc5d0 \ucd9c\uc804\ud55c \uc62c\uc2a4\ud0c0\uc804\uc5d0\uc11c\ub294 2\uacbd\uae30 \ubaa8\ub450 \ud37c\uc2dc\ud53d \ub9ac\uadf8 \ud300\uc758 5\ubc88 \ud0c0\uc790\ub85c\uc11c \uc120\ubc1c \ucd9c\uc804\ud574 2\ucc28\uc804\uc5d0\uc11c\ub294 \ud648\ub7f0\uc744 \ud3ec\ud568\ud55c 2\uc548\ud0c0 2\ud0c0\uc810\uc758 \uc131\uc801\uc744 \ub0a8\uacbc\ub2e4. \ucd5c\uc885\uc801\uc73c\ub85c\ub294 \uc2dc\uc98c \ucd5c\ub2e4\uc778 106\ud0c0\uc810\uc744 \ub0a8\uae30\uba74\uc11c \uccab \ud0c0\uaca9 \ubd80\ubb38 \ud0c0\uc774\ud2c0\uc774 \ub418\ub294 \ud0c0\uc810\uc655\uc744 \uc11d\uad8c, \uc678\uc57c\uc218\ub85c\uc11c \ubca0\uc2a4\ud2b8 \ub098\uc778\uc5d0 \uc120\uc815\ub410\ub2e4. \ub610, 123\uacbd\uae30\uc5d0 \uc88c\uc775\uc218\ub85c\uc11c \uc120\ubc1c \ucd9c\uc804\ud55c \uc218\ube44\uc5d0\uc11c\ub294 \ub9ac\uadf8 \ucd5c\ub2e4 \ubcf4\uc0b4 14\uac1c\ub97c \uae30\ub85d\ud588\ub2e4.", "answer": "123\uacbd\uae30", "sentence": "\ub610, 123\uacbd\uae30 \uc5d0 \uc88c\uc775\uc218\ub85c\uc11c \uc120\ubc1c \ucd9c\uc804\ud55c \uc218\ube44\uc5d0\uc11c\ub294 \ub9ac\uadf8 \ucd5c\ub2e4 \ubcf4\uc0b4 14\uac1c\ub97c \uae30\ub85d\ud588\ub2e4.", "paragraph_sentence": "\uc774\ub4ec\ud574 1990\ub144\uc5d0\ub294 \uc791\ub144 \uc2dc\uc98c\uc758 \ubc18\uc131\uc744 \uacc4\uae30\ub85c \uac00\ub3c4\ud0c0\uc640 \uc9c0\uba85 \ud0c0\uc790 \uc790\ub9ac\ub97c \uacbd\uc7c1\ud558\uc9c0 \uc54a\uace0 \uc88c\uc775\uc218\ub85c \ucd9c\uc804\ud558\uaca0\ub2e4\ub294 \uc758\uc0ac\ub97c \uad73\ud600 \uccb4\uc911\ub3c4 80kg\uc73c\ub85c \ub418\ub3cc\ub838\ub2e4. \uc2dc\uc98c\uc5d0 \ub4e4\uc5b4\uac00\uba74\uc11c 4\uc6d4 26\uc77c \uc138\uc774\ubd80\uc640\uc758 \uacbd\uae30\uc5d0\uc11c\ubd80\ud130 4\uc6d4 30\uc77c \uae34\ud14c\uc4f0\uc640\uc758 \uacbd\uae30\uc5d0 \uac78\uccd0 \ud37c\uc2dc\ud53d \ub9ac\uadf8 \uc5ed\ub300 2\uc704 \ud0c0\uc774 \uae30\ub85d\uc778 4\uacbd\uae30 \uc5f0\uc18d \ubcd1\uc0b4\ud0c0\ub97c \uae30\ub85d\ud588\uc9c0\ub9cc \uc624\ub808\uc2a4\ud14c\uc2a4 \ub370\uc2a4\ud2b8\ub77c\ub370\ub098 \uae30\uc694\ud558\ub77c \uac00\uc988\ud788\ub85c \ub4f1\uacfc \uc2dc\uc98c \uc885\ubc18\uae4c\uc9c0 \ud648\ub7f0\uc655 \ud0c0\uc774\ud2c0\uc744 \ub193\uace0 \uacbd\uc7c1\ud558\ub294 \ud65c\uc57d\uc744 \ubcf4\uc600\ub2e4. \ub610 3\ub144 \ub9cc\uc5d0 \ucd9c\uc804\ud55c \uc62c\uc2a4\ud0c0\uc804\uc5d0\uc11c\ub294 2\uacbd\uae30 \ubaa8\ub450 \ud37c\uc2dc\ud53d \ub9ac\uadf8 \ud300\uc758 5\ubc88 \ud0c0\uc790\ub85c\uc11c \uc120\ubc1c \ucd9c\uc804\ud574 2\ucc28\uc804\uc5d0\uc11c\ub294 \ud648\ub7f0\uc744 \ud3ec\ud568\ud55c 2\uc548\ud0c0 2\ud0c0\uc810\uc758 \uc131\uc801\uc744 \ub0a8\uacbc\ub2e4. \ucd5c\uc885\uc801\uc73c\ub85c\ub294 \uc2dc\uc98c \ucd5c\ub2e4\uc778 106\ud0c0\uc810\uc744 \ub0a8\uae30\uba74\uc11c \uccab \ud0c0\uaca9 \ubd80\ubb38 \ud0c0\uc774\ud2c0\uc774 \ub418\ub294 \ud0c0\uc810\uc655\uc744 \uc11d\uad8c, \uc678\uc57c\uc218\ub85c\uc11c \ubca0\uc2a4\ud2b8 \ub098\uc778\uc5d0 \uc120\uc815\ub410\ub2e4. \ub610, 123\uacbd\uae30 \uc5d0 \uc88c\uc775\uc218\ub85c\uc11c \uc120\ubc1c \ucd9c\uc804\ud55c \uc218\ube44\uc5d0\uc11c\ub294 \ub9ac\uadf8 \ucd5c\ub2e4 \ubcf4\uc0b4 14\uac1c\ub97c \uae30\ub85d\ud588\ub2e4. ", "paragraph_answer": "\uc774\ub4ec\ud574 1990\ub144\uc5d0\ub294 \uc791\ub144 \uc2dc\uc98c\uc758 \ubc18\uc131\uc744 \uacc4\uae30\ub85c \uac00\ub3c4\ud0c0\uc640 \uc9c0\uba85 \ud0c0\uc790 \uc790\ub9ac\ub97c \uacbd\uc7c1\ud558\uc9c0 \uc54a\uace0 \uc88c\uc775\uc218\ub85c \ucd9c\uc804\ud558\uaca0\ub2e4\ub294 \uc758\uc0ac\ub97c \uad73\ud600 \uccb4\uc911\ub3c4 80kg\uc73c\ub85c \ub418\ub3cc\ub838\ub2e4. \uc2dc\uc98c\uc5d0 \ub4e4\uc5b4\uac00\uba74\uc11c 4\uc6d4 26\uc77c \uc138\uc774\ubd80\uc640\uc758 \uacbd\uae30\uc5d0\uc11c\ubd80\ud130 4\uc6d4 30\uc77c \uae34\ud14c\uc4f0\uc640\uc758 \uacbd\uae30\uc5d0 \uac78\uccd0 \ud37c\uc2dc\ud53d \ub9ac\uadf8 \uc5ed\ub300 2\uc704 \ud0c0\uc774 \uae30\ub85d\uc778 4\uacbd\uae30 \uc5f0\uc18d \ubcd1\uc0b4\ud0c0\ub97c \uae30\ub85d\ud588\uc9c0\ub9cc \uc624\ub808\uc2a4\ud14c\uc2a4 \ub370\uc2a4\ud2b8\ub77c\ub370\ub098 \uae30\uc694\ud558\ub77c \uac00\uc988\ud788\ub85c \ub4f1\uacfc \uc2dc\uc98c \uc885\ubc18\uae4c\uc9c0 \ud648\ub7f0\uc655 \ud0c0\uc774\ud2c0\uc744 \ub193\uace0 \uacbd\uc7c1\ud558\ub294 \ud65c\uc57d\uc744 \ubcf4\uc600\ub2e4. \ub610 3\ub144 \ub9cc\uc5d0 \ucd9c\uc804\ud55c \uc62c\uc2a4\ud0c0\uc804\uc5d0\uc11c\ub294 2\uacbd\uae30 \ubaa8\ub450 \ud37c\uc2dc\ud53d \ub9ac\uadf8 \ud300\uc758 5\ubc88 \ud0c0\uc790\ub85c\uc11c \uc120\ubc1c \ucd9c\uc804\ud574 2\ucc28\uc804\uc5d0\uc11c\ub294 \ud648\ub7f0\uc744 \ud3ec\ud568\ud55c 2\uc548\ud0c0 2\ud0c0\uc810\uc758 \uc131\uc801\uc744 \ub0a8\uacbc\ub2e4. \ucd5c\uc885\uc801\uc73c\ub85c\ub294 \uc2dc\uc98c \ucd5c\ub2e4\uc778 106\ud0c0\uc810\uc744 \ub0a8\uae30\uba74\uc11c \uccab \ud0c0\uaca9 \ubd80\ubb38 \ud0c0\uc774\ud2c0\uc774 \ub418\ub294 \ud0c0\uc810\uc655\uc744 \uc11d\uad8c, \uc678\uc57c\uc218\ub85c\uc11c \ubca0\uc2a4\ud2b8 \ub098\uc778\uc5d0 \uc120\uc815\ub410\ub2e4. \ub610, 123\uacbd\uae30 \uc5d0 \uc88c\uc775\uc218\ub85c\uc11c \uc120\ubc1c \ucd9c\uc804\ud55c \uc218\ube44\uc5d0\uc11c\ub294 \ub9ac\uadf8 \ucd5c\ub2e4 \ubcf4\uc0b4 14\uac1c\ub97c \uae30\ub85d\ud588\ub2e4.", "sentence_answer": "\ub610, 123\uacbd\uae30 \uc5d0 \uc88c\uc775\uc218\ub85c\uc11c \uc120\ubc1c \ucd9c\uc804\ud55c \uc218\ube44\uc5d0\uc11c\ub294 \ub9ac\uadf8 \ucd5c\ub2e4 \ubcf4\uc0b4 14\uac1c\ub97c \uae30\ub85d\ud588\ub2e4.", "paragraph_id": "6162112-8-1", "paragraph_question": "question: 1990\ub144\uc5d0 \uc774\uc2dc\ubbf8\ub124 \uac00\uc988\ud788\ucf54\uac00 \uc88c\uc775\uc218\ub85c \uc120\ubc1c \ucd9c\uc804\ud55c \uacbd\uae30\uc758 \uc22b\uc790\ub294?, context: \uc774\ub4ec\ud574 1990\ub144\uc5d0\ub294 \uc791\ub144 \uc2dc\uc98c\uc758 \ubc18\uc131\uc744 \uacc4\uae30\ub85c \uac00\ub3c4\ud0c0\uc640 \uc9c0\uba85 \ud0c0\uc790 \uc790\ub9ac\ub97c \uacbd\uc7c1\ud558\uc9c0 \uc54a\uace0 \uc88c\uc775\uc218\ub85c \ucd9c\uc804\ud558\uaca0\ub2e4\ub294 \uc758\uc0ac\ub97c \uad73\ud600 \uccb4\uc911\ub3c4 80kg\uc73c\ub85c \ub418\ub3cc\ub838\ub2e4. \uc2dc\uc98c\uc5d0 \ub4e4\uc5b4\uac00\uba74\uc11c 4\uc6d4 26\uc77c \uc138\uc774\ubd80\uc640\uc758 \uacbd\uae30\uc5d0\uc11c\ubd80\ud130 4\uc6d4 30\uc77c \uae34\ud14c\uc4f0\uc640\uc758 \uacbd\uae30\uc5d0 \uac78\uccd0 \ud37c\uc2dc\ud53d \ub9ac\uadf8 \uc5ed\ub300 2\uc704 \ud0c0\uc774 \uae30\ub85d\uc778 4\uacbd\uae30 \uc5f0\uc18d \ubcd1\uc0b4\ud0c0\ub97c \uae30\ub85d\ud588\uc9c0\ub9cc \uc624\ub808\uc2a4\ud14c\uc2a4 \ub370\uc2a4\ud2b8\ub77c\ub370\ub098 \uae30\uc694\ud558\ub77c \uac00\uc988\ud788\ub85c \ub4f1\uacfc \uc2dc\uc98c \uc885\ubc18\uae4c\uc9c0 \ud648\ub7f0\uc655 \ud0c0\uc774\ud2c0\uc744 \ub193\uace0 \uacbd\uc7c1\ud558\ub294 \ud65c\uc57d\uc744 \ubcf4\uc600\ub2e4. \ub610 3\ub144 \ub9cc\uc5d0 \ucd9c\uc804\ud55c \uc62c\uc2a4\ud0c0\uc804\uc5d0\uc11c\ub294 2\uacbd\uae30 \ubaa8\ub450 \ud37c\uc2dc\ud53d \ub9ac\uadf8 \ud300\uc758 5\ubc88 \ud0c0\uc790\ub85c\uc11c \uc120\ubc1c \ucd9c\uc804\ud574 2\ucc28\uc804\uc5d0\uc11c\ub294 \ud648\ub7f0\uc744 \ud3ec\ud568\ud55c 2\uc548\ud0c0 2\ud0c0\uc810\uc758 \uc131\uc801\uc744 \ub0a8\uacbc\ub2e4. \ucd5c\uc885\uc801\uc73c\ub85c\ub294 \uc2dc\uc98c \ucd5c\ub2e4\uc778 106\ud0c0\uc810\uc744 \ub0a8\uae30\uba74\uc11c \uccab \ud0c0\uaca9 \ubd80\ubb38 \ud0c0\uc774\ud2c0\uc774 \ub418\ub294 \ud0c0\uc810\uc655\uc744 \uc11d\uad8c, \uc678\uc57c\uc218\ub85c\uc11c \ubca0\uc2a4\ud2b8 \ub098\uc778\uc5d0 \uc120\uc815\ub410\ub2e4. \ub610, 123\uacbd\uae30\uc5d0 \uc88c\uc775\uc218\ub85c\uc11c \uc120\ubc1c \ucd9c\uc804\ud55c \uc218\ube44\uc5d0\uc11c\ub294 \ub9ac\uadf8 \ucd5c\ub2e4 \ubcf4\uc0b4 14\uac1c\ub97c \uae30\ub85d\ud588\ub2e4."} {"question": "\uc758\uc0ac\uc18c\ud1b5\uacfc \ubcf4\uace0\uccb4\uacc4\uac00 \ud655\uc2e4\ud574 \uac04\uacb0\ud558\uace0 \ube60\ub978 \uc758\uc0ac\uacb0\uc815\uacfc \uc9c4\ud589\uc774 \uac00\ub2a5\ud55c \uad6c\uc870\ub294 \ubb34\uc5c7\uc778\uac00?", "paragraph": "\ud504\ub85c\uc81d\ud2b8 \uad6c\uc870\ub294 \uc758\uc0ac\uc18c\ud1b5\uacfc \ubcf4\uace0\uccb4\uacc4\uac00 \uba85\ud655\ud558\uc5ec \uac04\ub2e8\ud558\uace0 \uc2e0\uc18d\ud55c \uc758\uc0ac\uacb0\uc815 \ubc0f \uc9c4\ud589\uc774 \uac00\ub2a5\ud558\ub2e4. \ub610\ud55c \ud504\ub85c\uc81d\ud2b8 \uad00\ub9ac\uc790\uc758 \uad8c\ud55c\uc774 \ub192\uc544 \ud504\ub85c\uc81d\ud2b8\uc5d0 \uc9c0\uc2dd\uc744 \uac16\ucd98 \uc778\ub825\uc744 \ubc30\uce58\ud558\uace0 \ud504\ub85c\uc81d\ud2b8 \uc218\ud589\uc5d0 \uc9d1\uc911\uc2dc\ud0b4\uc73c\ub85c\uc368 \ud504\ub85c\uc81d\ud2b8 \ucd94\uc9c4\ub825\uc774 \ub192\uc77c \uc218 \uc788\ub2e4\ub294 \uc7a5\uc810\uc774 \uc788\ub2e4. \ud558\uc9c0\ub9cc \uac1c\uac1c\uc778\uc758 \ub178\ud558\uc6b0\uac00 \uae30\uc220\uc758 \uac1c\uc778\uc758\uc874\uc73c\ub85c \uae30\uc220 \uc804\ubb38\uc131\uc5d0 \uc81c\uc57d\uc744 \ubc1b\uc744 \uc218 \uc788\uace0 \uae30\uc874\uc758 \ubd80\uc11c\uc5d0\uc11c \uc6b0\uc218\ud55c \uc778\ub825\uc744 \ud504\ub85c\uc81d\ud2b8 \ud300\uc73c\ub85c \uc644\uc804\ud788 \ubcf4\ub0b4\uc57c \ud558\uae30 \ub54c\ubb38\uc5d0 \uae30\uc874 \uae30\ub2a5\ubd80\uc11c\uc758 \uc5c5\ubb34\uc218\ud589\uc5d0 \ucc28\uc9c8\uc744 \uc904 \uc218 \uc788\ub2e4. \uc870\uc9c1\uc774 \uc5ec\ub7ec \uac1c\uc758 \ud504\ub85c\uc81d\ud2b8\ub97c \ub3d9\uc2dc\uc5d0 \uc218\ud589\ud558\uac8c \ub420 \ub54c\uc5d0\ub294 \ub2e8\uc21c\ud788 \uadf8 \uae30\ub2a5\ubd80\uc11c\ubfd0\ub9cc \uc544\ub2c8\ub77c \uc870\uc9c1 \uc804\uccb4\uc5d0 \ubd80\ub2f4\uc744 \uc904 \uc218 \uc788\uc73c\uba70 \ud504\ub85c\uc81d\ud2b8\ub97c \uc644\ub8cc\ud55c \uc774\ud6c4 \ud300\uc6d0\ub4e4\uc758 \uc18c\uc18d\ud300\uc73c\ub85c\uc758 \ubcf5\uadc0\uac00 \ubd88\uc548\uc815\ud558\ub2e4\ub294 \ub2e8\uc810\uc774 \uc788\ub2e4.", "answer": "\ud504\ub85c\uc81d\ud2b8 \uad6c\uc870", "sentence": "\ud504\ub85c\uc81d\ud2b8 \uad6c\uc870 \ub294 \uc758\uc0ac\uc18c\ud1b5\uacfc \ubcf4\uace0\uccb4\uacc4\uac00 \uba85\ud655\ud558\uc5ec \uac04\ub2e8\ud558\uace0 \uc2e0\uc18d\ud55c \uc758\uc0ac\uacb0\uc815 \ubc0f \uc9c4\ud589\uc774 \uac00\ub2a5\ud558\ub2e4.", "paragraph_sentence": " \ud504\ub85c\uc81d\ud2b8 \uad6c\uc870 \ub294 \uc758\uc0ac\uc18c\ud1b5\uacfc \ubcf4\uace0\uccb4\uacc4\uac00 \uba85\ud655\ud558\uc5ec \uac04\ub2e8\ud558\uace0 \uc2e0\uc18d\ud55c \uc758\uc0ac\uacb0\uc815 \ubc0f \uc9c4\ud589\uc774 \uac00\ub2a5\ud558\ub2e4. \ub610\ud55c \ud504\ub85c\uc81d\ud2b8 \uad00\ub9ac\uc790\uc758 \uad8c\ud55c\uc774 \ub192\uc544 \ud504\ub85c\uc81d\ud2b8\uc5d0 \uc9c0\uc2dd\uc744 \uac16\ucd98 \uc778\ub825\uc744 \ubc30\uce58\ud558\uace0 \ud504\ub85c\uc81d\ud2b8 \uc218\ud589\uc5d0 \uc9d1\uc911\uc2dc\ud0b4\uc73c\ub85c\uc368 \ud504\ub85c\uc81d\ud2b8 \ucd94\uc9c4\ub825\uc774 \ub192\uc77c \uc218 \uc788\ub2e4\ub294 \uc7a5\uc810\uc774 \uc788\ub2e4. \ud558\uc9c0\ub9cc \uac1c\uac1c\uc778\uc758 \ub178\ud558\uc6b0\uac00 \uae30\uc220\uc758 \uac1c\uc778\uc758\uc874\uc73c\ub85c \uae30\uc220 \uc804\ubb38\uc131\uc5d0 \uc81c\uc57d\uc744 \ubc1b\uc744 \uc218 \uc788\uace0 \uae30\uc874\uc758 \ubd80\uc11c\uc5d0\uc11c \uc6b0\uc218\ud55c \uc778\ub825\uc744 \ud504\ub85c\uc81d\ud2b8 \ud300\uc73c\ub85c \uc644\uc804\ud788 \ubcf4\ub0b4\uc57c \ud558\uae30 \ub54c\ubb38\uc5d0 \uae30\uc874 \uae30\ub2a5\ubd80\uc11c\uc758 \uc5c5\ubb34\uc218\ud589\uc5d0 \ucc28\uc9c8\uc744 \uc904 \uc218 \uc788\ub2e4. \uc870\uc9c1\uc774 \uc5ec\ub7ec \uac1c\uc758 \ud504\ub85c\uc81d\ud2b8\ub97c \ub3d9\uc2dc\uc5d0 \uc218\ud589\ud558\uac8c \ub420 \ub54c\uc5d0\ub294 \ub2e8\uc21c\ud788 \uadf8 \uae30\ub2a5\ubd80\uc11c\ubfd0\ub9cc \uc544\ub2c8\ub77c \uc870\uc9c1 \uc804\uccb4\uc5d0 \ubd80\ub2f4\uc744 \uc904 \uc218 \uc788\uc73c\uba70 \ud504\ub85c\uc81d\ud2b8\ub97c \uc644\ub8cc\ud55c \uc774\ud6c4 \ud300\uc6d0\ub4e4\uc758 \uc18c\uc18d\ud300\uc73c\ub85c\uc758 \ubcf5\uadc0\uac00 \ubd88\uc548\uc815\ud558\ub2e4\ub294 \ub2e8\uc810\uc774 \uc788\ub2e4.", "paragraph_answer": " \ud504\ub85c\uc81d\ud2b8 \uad6c\uc870 \ub294 \uc758\uc0ac\uc18c\ud1b5\uacfc \ubcf4\uace0\uccb4\uacc4\uac00 \uba85\ud655\ud558\uc5ec \uac04\ub2e8\ud558\uace0 \uc2e0\uc18d\ud55c \uc758\uc0ac\uacb0\uc815 \ubc0f \uc9c4\ud589\uc774 \uac00\ub2a5\ud558\ub2e4. \ub610\ud55c \ud504\ub85c\uc81d\ud2b8 \uad00\ub9ac\uc790\uc758 \uad8c\ud55c\uc774 \ub192\uc544 \ud504\ub85c\uc81d\ud2b8\uc5d0 \uc9c0\uc2dd\uc744 \uac16\ucd98 \uc778\ub825\uc744 \ubc30\uce58\ud558\uace0 \ud504\ub85c\uc81d\ud2b8 \uc218\ud589\uc5d0 \uc9d1\uc911\uc2dc\ud0b4\uc73c\ub85c\uc368 \ud504\ub85c\uc81d\ud2b8 \ucd94\uc9c4\ub825\uc774 \ub192\uc77c \uc218 \uc788\ub2e4\ub294 \uc7a5\uc810\uc774 \uc788\ub2e4. \ud558\uc9c0\ub9cc \uac1c\uac1c\uc778\uc758 \ub178\ud558\uc6b0\uac00 \uae30\uc220\uc758 \uac1c\uc778\uc758\uc874\uc73c\ub85c \uae30\uc220 \uc804\ubb38\uc131\uc5d0 \uc81c\uc57d\uc744 \ubc1b\uc744 \uc218 \uc788\uace0 \uae30\uc874\uc758 \ubd80\uc11c\uc5d0\uc11c \uc6b0\uc218\ud55c \uc778\ub825\uc744 \ud504\ub85c\uc81d\ud2b8 \ud300\uc73c\ub85c \uc644\uc804\ud788 \ubcf4\ub0b4\uc57c \ud558\uae30 \ub54c\ubb38\uc5d0 \uae30\uc874 \uae30\ub2a5\ubd80\uc11c\uc758 \uc5c5\ubb34\uc218\ud589\uc5d0 \ucc28\uc9c8\uc744 \uc904 \uc218 \uc788\ub2e4. \uc870\uc9c1\uc774 \uc5ec\ub7ec \uac1c\uc758 \ud504\ub85c\uc81d\ud2b8\ub97c \ub3d9\uc2dc\uc5d0 \uc218\ud589\ud558\uac8c \ub420 \ub54c\uc5d0\ub294 \ub2e8\uc21c\ud788 \uadf8 \uae30\ub2a5\ubd80\uc11c\ubfd0\ub9cc \uc544\ub2c8\ub77c \uc870\uc9c1 \uc804\uccb4\uc5d0 \ubd80\ub2f4\uc744 \uc904 \uc218 \uc788\uc73c\uba70 \ud504\ub85c\uc81d\ud2b8\ub97c \uc644\ub8cc\ud55c \uc774\ud6c4 \ud300\uc6d0\ub4e4\uc758 \uc18c\uc18d\ud300\uc73c\ub85c\uc758 \ubcf5\uadc0\uac00 \ubd88\uc548\uc815\ud558\ub2e4\ub294 \ub2e8\uc810\uc774 \uc788\ub2e4.", "sentence_answer": " \ud504\ub85c\uc81d\ud2b8 \uad6c\uc870 \ub294 \uc758\uc0ac\uc18c\ud1b5\uacfc \ubcf4\uace0\uccb4\uacc4\uac00 \uba85\ud655\ud558\uc5ec \uac04\ub2e8\ud558\uace0 \uc2e0\uc18d\ud55c \uc758\uc0ac\uacb0\uc815 \ubc0f \uc9c4\ud589\uc774 \uac00\ub2a5\ud558\ub2e4.", "paragraph_id": "6554793-17-0", "paragraph_question": "question: \uc758\uc0ac\uc18c\ud1b5\uacfc \ubcf4\uace0\uccb4\uacc4\uac00 \ud655\uc2e4\ud574 \uac04\uacb0\ud558\uace0 \ube60\ub978 \uc758\uc0ac\uacb0\uc815\uacfc \uc9c4\ud589\uc774 \uac00\ub2a5\ud55c \uad6c\uc870\ub294 \ubb34\uc5c7\uc778\uac00?, context: \ud504\ub85c\uc81d\ud2b8 \uad6c\uc870\ub294 \uc758\uc0ac\uc18c\ud1b5\uacfc \ubcf4\uace0\uccb4\uacc4\uac00 \uba85\ud655\ud558\uc5ec \uac04\ub2e8\ud558\uace0 \uc2e0\uc18d\ud55c \uc758\uc0ac\uacb0\uc815 \ubc0f \uc9c4\ud589\uc774 \uac00\ub2a5\ud558\ub2e4. \ub610\ud55c \ud504\ub85c\uc81d\ud2b8 \uad00\ub9ac\uc790\uc758 \uad8c\ud55c\uc774 \ub192\uc544 \ud504\ub85c\uc81d\ud2b8\uc5d0 \uc9c0\uc2dd\uc744 \uac16\ucd98 \uc778\ub825\uc744 \ubc30\uce58\ud558\uace0 \ud504\ub85c\uc81d\ud2b8 \uc218\ud589\uc5d0 \uc9d1\uc911\uc2dc\ud0b4\uc73c\ub85c\uc368 \ud504\ub85c\uc81d\ud2b8 \ucd94\uc9c4\ub825\uc774 \ub192\uc77c \uc218 \uc788\ub2e4\ub294 \uc7a5\uc810\uc774 \uc788\ub2e4. \ud558\uc9c0\ub9cc \uac1c\uac1c\uc778\uc758 \ub178\ud558\uc6b0\uac00 \uae30\uc220\uc758 \uac1c\uc778\uc758\uc874\uc73c\ub85c \uae30\uc220 \uc804\ubb38\uc131\uc5d0 \uc81c\uc57d\uc744 \ubc1b\uc744 \uc218 \uc788\uace0 \uae30\uc874\uc758 \ubd80\uc11c\uc5d0\uc11c \uc6b0\uc218\ud55c \uc778\ub825\uc744 \ud504\ub85c\uc81d\ud2b8 \ud300\uc73c\ub85c \uc644\uc804\ud788 \ubcf4\ub0b4\uc57c \ud558\uae30 \ub54c\ubb38\uc5d0 \uae30\uc874 \uae30\ub2a5\ubd80\uc11c\uc758 \uc5c5\ubb34\uc218\ud589\uc5d0 \ucc28\uc9c8\uc744 \uc904 \uc218 \uc788\ub2e4. \uc870\uc9c1\uc774 \uc5ec\ub7ec \uac1c\uc758 \ud504\ub85c\uc81d\ud2b8\ub97c \ub3d9\uc2dc\uc5d0 \uc218\ud589\ud558\uac8c \ub420 \ub54c\uc5d0\ub294 \ub2e8\uc21c\ud788 \uadf8 \uae30\ub2a5\ubd80\uc11c\ubfd0\ub9cc \uc544\ub2c8\ub77c \uc870\uc9c1 \uc804\uccb4\uc5d0 \ubd80\ub2f4\uc744 \uc904 \uc218 \uc788\uc73c\uba70 \ud504\ub85c\uc81d\ud2b8\ub97c \uc644\ub8cc\ud55c \uc774\ud6c4 \ud300\uc6d0\ub4e4\uc758 \uc18c\uc18d\ud300\uc73c\ub85c\uc758 \ubcf5\uadc0\uac00 \ubd88\uc548\uc815\ud558\ub2e4\ub294 \ub2e8\uc810\uc774 \uc788\ub2e4."} {"question": "B.O.B\uc758 \ucd9c\uc0dd\uc77c\uc740?", "paragraph": "\uc2dc\uba3c\uc2a4\ub294 \ub178\uc2a4\uce90\ub864\ub77c\uc774\ub098 \uc8fc \uc708\uc2a4\ud134\uc138\uc77c\ub7fc\uc5d0\uc11c 1988\ub144 11\uc6d4 15\uc77c\uc5d0 \ud0dc\uc5b4\ub0ac\ub2e4. \uadf8\ub294 \ucd08\ub4f1\ud559\uad50 \ub54c\ubd80\ud130 \uace0\ub4f1\ud559\uad50 \ub54c\uae4c\uc9c0 \uc2a4\ucfe8 \ubc34\ub4dc\uc5d0\uc11c \ud2b8\ub7fc\ud3ab\uc744 \uc5f0\uc8fc\ud588\ub2e4. \ubd80\ubaa8\ub2d8\uc740 \ud559\uc5c5\uc744 \uc9c0\uc18d\ud558\uae38 \ubc14\ub790\uc9c0\ub9cc 6\ud559\ub144(sixth grade) \ub54c \uc774\ubbf8 \uc2dc\uba3c\uc2a4\ub294 \ubba4\uc9c0\uc158\uc758 \uae38\uc744 \uac00\uace0 \uc2f6\uc5b4 \ud588\ub2e4. \ubaa9\uc0ac\uc600\ub358 \uc544\ubc84\uc9c0\ub294 \uc2dc\uba3c\uc2a4\uc5d0\uac8c \uc74c\uc545\uc740 \uce58\uc720\uc774\uc790 \ucc3d\uc758\ub825\uc758 \ubc1c\ud718\ub77c\ub294 \uc810\uc744 \uae68\ub2eb\uac8c \ub41c \ud6c4\ub860 \ubc18\ub300\ub97c \uba48\ucdc4\ub2e4. \ud6d7\ub0a0 \uc2dc\uba3c\uc2a4\ub294 \uacfc\uac70\ub97c \ud68c\uc0c1\ud558\uba70 \u201c\ubd80\ubaa8\ub2d8\uc740 \uc5b8\uc81c\ub098 \ub098\ub97c \uc9c0\uc9c0\ud574 \uc92c\uc5b4. \uadf8\ub4e4\uc740 \ube44\ud2b8\ub97c \ub9cc\ub4dc\ub294 \ub370 \ud544\uc694\ud55c \ub0b4 \uccab \ubc88\uc9f8 \ud0a4\ubcf4\ub4dc\ub97c \ub9c8\ub828\ud574\uc92c\uace0 \uc774\uacf3\uc800\uacf3\uc5d0\uc11c \ub0b4 \uc7a5\ube44\ub97c \ub9c8\ub828\ud574 \uc92c\uc5b4. \ud558\uc9c0\ub9cc \uadf8\ubd84\ub4e4\uc774 \ub0b4\uac00 \uc9c4\uc9dc\ub85c \uc5b4\ub5a4 \uac78 \uc5bb\uace0\uc790 \ud558\ub294\uc9c0\ub97c \uc54c\uc9c0 \ubabb\ud588\uc744 \uac70\uc57c.\u201d \uc774\ud6c4 B.o.B\ub294 \uc870\uc9c0\uc544 \uc8fc \ub514\ucf00\uc774\ud130 \uc9c0\uc5ed\uc758 \uceec\ub7fc\ube44\uc544 \uace0\ub4f1\ud559\uad50\uc5d0 \uc9c4\ud559\ud588\ub2e4.", "answer": "1988\ub144 11\uc6d4 15\uc77c", "sentence": "\uc2dc\uba3c\uc2a4\ub294 \ub178\uc2a4\uce90\ub864\ub77c\uc774\ub098 \uc8fc \uc708\uc2a4\ud134\uc138\uc77c\ub7fc\uc5d0\uc11c 1988\ub144 11\uc6d4 15\uc77c \uc5d0 \ud0dc\uc5b4\ub0ac\ub2e4.", "paragraph_sentence": " \uc2dc\uba3c\uc2a4\ub294 \ub178\uc2a4\uce90\ub864\ub77c\uc774\ub098 \uc8fc \uc708\uc2a4\ud134\uc138\uc77c\ub7fc\uc5d0\uc11c 1988\ub144 11\uc6d4 15\uc77c \uc5d0 \ud0dc\uc5b4\ub0ac\ub2e4. \uadf8\ub294 \ucd08\ub4f1\ud559\uad50 \ub54c\ubd80\ud130 \uace0\ub4f1\ud559\uad50 \ub54c\uae4c\uc9c0 \uc2a4\ucfe8 \ubc34\ub4dc\uc5d0\uc11c \ud2b8\ub7fc\ud3ab\uc744 \uc5f0\uc8fc\ud588\ub2e4. \ubd80\ubaa8\ub2d8\uc740 \ud559\uc5c5\uc744 \uc9c0\uc18d\ud558\uae38 \ubc14\ub790\uc9c0\ub9cc 6\ud559\ub144(sixth grade) \ub54c \uc774\ubbf8 \uc2dc\uba3c\uc2a4\ub294 \ubba4\uc9c0\uc158\uc758 \uae38\uc744 \uac00\uace0 \uc2f6\uc5b4 \ud588\ub2e4. \ubaa9\uc0ac\uc600\ub358 \uc544\ubc84\uc9c0\ub294 \uc2dc\uba3c\uc2a4\uc5d0\uac8c \uc74c\uc545\uc740 \uce58\uc720\uc774\uc790 \ucc3d\uc758\ub825\uc758 \ubc1c\ud718\ub77c\ub294 \uc810\uc744 \uae68\ub2eb\uac8c \ub41c \ud6c4\ub860 \ubc18\ub300\ub97c \uba48\ucdc4\ub2e4. \ud6d7\ub0a0 \uc2dc\uba3c\uc2a4\ub294 \uacfc\uac70\ub97c \ud68c\uc0c1\ud558\uba70 \u201c\ubd80\ubaa8\ub2d8\uc740 \uc5b8\uc81c\ub098 \ub098\ub97c \uc9c0\uc9c0\ud574 \uc92c\uc5b4. \uadf8\ub4e4\uc740 \ube44\ud2b8\ub97c \ub9cc\ub4dc\ub294 \ub370 \ud544\uc694\ud55c \ub0b4 \uccab \ubc88\uc9f8 \ud0a4\ubcf4\ub4dc\ub97c \ub9c8\ub828\ud574\uc92c\uace0 \uc774\uacf3\uc800\uacf3\uc5d0\uc11c \ub0b4 \uc7a5\ube44\ub97c \ub9c8\ub828\ud574 \uc92c\uc5b4. \ud558\uc9c0\ub9cc \uadf8\ubd84\ub4e4\uc774 \ub0b4\uac00 \uc9c4\uc9dc\ub85c \uc5b4\ub5a4 \uac78 \uc5bb\uace0\uc790 \ud558\ub294\uc9c0\ub97c \uc54c\uc9c0 \ubabb\ud588\uc744 \uac70\uc57c. \u201d \uc774\ud6c4 B.o. B\ub294 \uc870\uc9c0\uc544 \uc8fc \ub514\ucf00\uc774\ud130 \uc9c0\uc5ed\uc758 \uceec\ub7fc\ube44\uc544 \uace0\ub4f1\ud559\uad50\uc5d0 \uc9c4\ud559\ud588\ub2e4.", "paragraph_answer": "\uc2dc\uba3c\uc2a4\ub294 \ub178\uc2a4\uce90\ub864\ub77c\uc774\ub098 \uc8fc \uc708\uc2a4\ud134\uc138\uc77c\ub7fc\uc5d0\uc11c 1988\ub144 11\uc6d4 15\uc77c \uc5d0 \ud0dc\uc5b4\ub0ac\ub2e4. \uadf8\ub294 \ucd08\ub4f1\ud559\uad50 \ub54c\ubd80\ud130 \uace0\ub4f1\ud559\uad50 \ub54c\uae4c\uc9c0 \uc2a4\ucfe8 \ubc34\ub4dc\uc5d0\uc11c \ud2b8\ub7fc\ud3ab\uc744 \uc5f0\uc8fc\ud588\ub2e4. \ubd80\ubaa8\ub2d8\uc740 \ud559\uc5c5\uc744 \uc9c0\uc18d\ud558\uae38 \ubc14\ub790\uc9c0\ub9cc 6\ud559\ub144(sixth grade) \ub54c \uc774\ubbf8 \uc2dc\uba3c\uc2a4\ub294 \ubba4\uc9c0\uc158\uc758 \uae38\uc744 \uac00\uace0 \uc2f6\uc5b4 \ud588\ub2e4. \ubaa9\uc0ac\uc600\ub358 \uc544\ubc84\uc9c0\ub294 \uc2dc\uba3c\uc2a4\uc5d0\uac8c \uc74c\uc545\uc740 \uce58\uc720\uc774\uc790 \ucc3d\uc758\ub825\uc758 \ubc1c\ud718\ub77c\ub294 \uc810\uc744 \uae68\ub2eb\uac8c \ub41c \ud6c4\ub860 \ubc18\ub300\ub97c \uba48\ucdc4\ub2e4. \ud6d7\ub0a0 \uc2dc\uba3c\uc2a4\ub294 \uacfc\uac70\ub97c \ud68c\uc0c1\ud558\uba70 \u201c\ubd80\ubaa8\ub2d8\uc740 \uc5b8\uc81c\ub098 \ub098\ub97c \uc9c0\uc9c0\ud574 \uc92c\uc5b4. \uadf8\ub4e4\uc740 \ube44\ud2b8\ub97c \ub9cc\ub4dc\ub294 \ub370 \ud544\uc694\ud55c \ub0b4 \uccab \ubc88\uc9f8 \ud0a4\ubcf4\ub4dc\ub97c \ub9c8\ub828\ud574\uc92c\uace0 \uc774\uacf3\uc800\uacf3\uc5d0\uc11c \ub0b4 \uc7a5\ube44\ub97c \ub9c8\ub828\ud574 \uc92c\uc5b4. \ud558\uc9c0\ub9cc \uadf8\ubd84\ub4e4\uc774 \ub0b4\uac00 \uc9c4\uc9dc\ub85c \uc5b4\ub5a4 \uac78 \uc5bb\uace0\uc790 \ud558\ub294\uc9c0\ub97c \uc54c\uc9c0 \ubabb\ud588\uc744 \uac70\uc57c.\u201d \uc774\ud6c4 B.o.B\ub294 \uc870\uc9c0\uc544 \uc8fc \ub514\ucf00\uc774\ud130 \uc9c0\uc5ed\uc758 \uceec\ub7fc\ube44\uc544 \uace0\ub4f1\ud559\uad50\uc5d0 \uc9c4\ud559\ud588\ub2e4.", "sentence_answer": "\uc2dc\uba3c\uc2a4\ub294 \ub178\uc2a4\uce90\ub864\ub77c\uc774\ub098 \uc8fc \uc708\uc2a4\ud134\uc138\uc77c\ub7fc\uc5d0\uc11c 1988\ub144 11\uc6d4 15\uc77c \uc5d0 \ud0dc\uc5b4\ub0ac\ub2e4.", "paragraph_id": "6491044-1-1", "paragraph_question": "question: B.O.B\uc758 \ucd9c\uc0dd\uc77c\uc740?, context: \uc2dc\uba3c\uc2a4\ub294 \ub178\uc2a4\uce90\ub864\ub77c\uc774\ub098 \uc8fc \uc708\uc2a4\ud134\uc138\uc77c\ub7fc\uc5d0\uc11c 1988\ub144 11\uc6d4 15\uc77c\uc5d0 \ud0dc\uc5b4\ub0ac\ub2e4. \uadf8\ub294 \ucd08\ub4f1\ud559\uad50 \ub54c\ubd80\ud130 \uace0\ub4f1\ud559\uad50 \ub54c\uae4c\uc9c0 \uc2a4\ucfe8 \ubc34\ub4dc\uc5d0\uc11c \ud2b8\ub7fc\ud3ab\uc744 \uc5f0\uc8fc\ud588\ub2e4. \ubd80\ubaa8\ub2d8\uc740 \ud559\uc5c5\uc744 \uc9c0\uc18d\ud558\uae38 \ubc14\ub790\uc9c0\ub9cc 6\ud559\ub144(sixth grade) \ub54c \uc774\ubbf8 \uc2dc\uba3c\uc2a4\ub294 \ubba4\uc9c0\uc158\uc758 \uae38\uc744 \uac00\uace0 \uc2f6\uc5b4 \ud588\ub2e4. \ubaa9\uc0ac\uc600\ub358 \uc544\ubc84\uc9c0\ub294 \uc2dc\uba3c\uc2a4\uc5d0\uac8c \uc74c\uc545\uc740 \uce58\uc720\uc774\uc790 \ucc3d\uc758\ub825\uc758 \ubc1c\ud718\ub77c\ub294 \uc810\uc744 \uae68\ub2eb\uac8c \ub41c \ud6c4\ub860 \ubc18\ub300\ub97c \uba48\ucdc4\ub2e4. \ud6d7\ub0a0 \uc2dc\uba3c\uc2a4\ub294 \uacfc\uac70\ub97c \ud68c\uc0c1\ud558\uba70 \u201c\ubd80\ubaa8\ub2d8\uc740 \uc5b8\uc81c\ub098 \ub098\ub97c \uc9c0\uc9c0\ud574 \uc92c\uc5b4. \uadf8\ub4e4\uc740 \ube44\ud2b8\ub97c \ub9cc\ub4dc\ub294 \ub370 \ud544\uc694\ud55c \ub0b4 \uccab \ubc88\uc9f8 \ud0a4\ubcf4\ub4dc\ub97c \ub9c8\ub828\ud574\uc92c\uace0 \uc774\uacf3\uc800\uacf3\uc5d0\uc11c \ub0b4 \uc7a5\ube44\ub97c \ub9c8\ub828\ud574 \uc92c\uc5b4. \ud558\uc9c0\ub9cc \uadf8\ubd84\ub4e4\uc774 \ub0b4\uac00 \uc9c4\uc9dc\ub85c \uc5b4\ub5a4 \uac78 \uc5bb\uace0\uc790 \ud558\ub294\uc9c0\ub97c \uc54c\uc9c0 \ubabb\ud588\uc744 \uac70\uc57c.\u201d \uc774\ud6c4 B.o.B\ub294 \uc870\uc9c0\uc544 \uc8fc \ub514\ucf00\uc774\ud130 \uc9c0\uc5ed\uc758 \uceec\ub7fc\ube44\uc544 \uace0\ub4f1\ud559\uad50\uc5d0 \uc9c4\ud559\ud588\ub2e4."} {"question": "\ub300\ubcf8\uc5d0 \ub098\ud0c0\ub098 \ucc3d \ubc0f \ub3c4\ucc3d \ubd80\ubd84\uc744 \uc9c0\ub3c4\ud558\ub294 \uac83\uc740 \ub204\uad6c\uc778\uac00?", "paragraph": "\ub300\ubcf8\uc5d0 \ub098\ud0c0\ub09c \ucc3d \ubc0f \ub3c4\ucc3d \ubd80\ubd84\uc740 \ucc3d\uadf9 \uc791\uace1\uc790\uc758 \ucc3d\uc5d0 \ub300\ud55c \uc9c0\ub3c4\ub85c\ubd80\ud130 \uc2dc\uc791\ub41c\ub2e4. \uc624\ud398\ub77c\ucc98\ub7fc \uc545\ubcf4\uc5d0 \uc758\ud55c \uac1c\uc778 \uc5f0\uc2b5\uc774 \uc120\ud589\ub418\ub294 \uac83\uc774 \uc544\ub2c8\uace0, \uc791\uace1\uc790\uac00 \uc7a5\ub2e8\uacfc \uc870\ub97c \uc9c0\uc2dc\ud558\uace0 \uc9c1\uc811 \ucc3d\uc744 \ubd88\ub7ec\uc11c \uad6c\uc804, \uc2ec\uc218(\u53e3\u50b3\u5fc3\u53d7)\ud558\ub294 \uac83\uc774\ub2e4. \ud310\uc18c\ub9ac\uc77c \uacbd\uc6b0\uc5d0\ub294 \uc5b4\ub290 \uc815\ub3c4 \uace0\uc815\uc120\uc728(\u56fa\u5b9a\u65cb\u5f8b)\uc774\uae30 \ub54c\ubb38\uc5d0 \uc804\uc218\uc0dd\uc5d0\uac8c \ub9ac\ub4ec\uacfc \uac00\ub77d\ubfd0\ub9cc \uc544\ub2c8\ub77c \ucc3d\ubc95\uae4c\uc9c0\ub3c4 \ud558\ub098\ud558\ub098 \uad50\uc218\ud55c\ub2e4. \uadf8\ub7ec\ub098 \ucc3d\uadf9\uc758 \uacbd\uc6b0\uc5d0\ub294 \ucc3d\uc790\ub4e4\uc774 \uc5b4\ub290 \uc815\ub3c4 \ud310\uc18c\ub9ac\uc758 \ucc3d\uc744 \uad6c\uc0ac\ud560 \uc218 \uc788\ub294 \ub2a5\ub825\uc774 \uc788\uc73c\ubbc0\ub85c \uc7a5\ub2e8\uacfc \uc870 \uadf8\ub9ac\uace0 \uac00\ub77d\uc758 \uace8\uaca9\uc744 \uc9c0\uc2dc\ud558\uc9c0\ub9cc \uc138\ubd80 \ucc3d\ubc95\uacfc \uac19\uc740 \uac83\uc740 \ucc3d\uc790\uc758 \uae30\ub7c9\uc5d0 \ub9e1\uae30\uae30 \ub54c\ubb38\uc5d0 \ucc3d\uc790\uc758 \ucc3d\ubc95\uc5d0 \uc758\ud55c \uac1c\uc131\uc774 \ubc30\uc5ed\ub9c8\ub2e4 \uc2ec\ud558\uac8c \ub178\ucd9c\ub418\uae30\ub3c4 \ud55c\ub2e4. \ub3c4\ucc3d\uc740 \uc77c\ubc18\uc801\uc73c\ub85c \uc6d0\ub85c\uae09 \ucc3d\uc790\uac00 \ub9e1\uac8c \ub418\ubbc0\ub85c \uc7a5\ub2e8\uacfc \uc870(\u8abf)\ub9cc \uc9c0\uc2dc\ud558\uace0 \ub098\uba38\uc9c0\ub294 \ucc3d\uc790\uc758 \uc74c\uc545\uc131\uacfc \ud45c\ucd9c\ub825\uc5d0 \ub9e1\uae34\ub2e4. \uc81c\ucc3d\uc758 \uacbd\uc6b0\uc5d0\ub294 \ub2e4\uc218\uc758 \ucd9c\uc5f0\uc790\uac00 \uac19\uc740 \uac00\ub77d\uc73c\ub85c \ubd88\ub7ec\uc57c \ud558\uae30 \ub54c\ubb38\uc5d0 \ud569\ub3d9 \ucc3d \uc5f0\uc2b5\uc774 \ud544\uc694\ud558\ub2e4.", "answer": "\ucc3d\uadf9 \uc791\uace1\uc790", "sentence": "\ub300\ubcf8\uc5d0 \ub098\ud0c0\ub09c \ucc3d \ubc0f \ub3c4\ucc3d \ubd80\ubd84\uc740 \ucc3d\uadf9 \uc791\uace1\uc790 \uc758 \ucc3d\uc5d0 \ub300\ud55c \uc9c0\ub3c4\ub85c\ubd80\ud130 \uc2dc\uc791\ub41c\ub2e4.", "paragraph_sentence": " \ub300\ubcf8\uc5d0 \ub098\ud0c0\ub09c \ucc3d \ubc0f \ub3c4\ucc3d \ubd80\ubd84\uc740 \ucc3d\uadf9 \uc791\uace1\uc790 \uc758 \ucc3d\uc5d0 \ub300\ud55c \uc9c0\ub3c4\ub85c\ubd80\ud130 \uc2dc\uc791\ub41c\ub2e4. \uc624\ud398\ub77c\ucc98\ub7fc \uc545\ubcf4\uc5d0 \uc758\ud55c \uac1c\uc778 \uc5f0\uc2b5\uc774 \uc120\ud589\ub418\ub294 \uac83\uc774 \uc544\ub2c8\uace0, \uc791\uace1\uc790\uac00 \uc7a5\ub2e8\uacfc \uc870\ub97c \uc9c0\uc2dc\ud558\uace0 \uc9c1\uc811 \ucc3d\uc744 \ubd88\ub7ec\uc11c \uad6c\uc804, \uc2ec\uc218(\u53e3\u50b3\u5fc3\u53d7)\ud558\ub294 \uac83\uc774\ub2e4. \ud310\uc18c\ub9ac\uc77c \uacbd\uc6b0\uc5d0\ub294 \uc5b4\ub290 \uc815\ub3c4 \uace0\uc815\uc120\uc728(\u56fa\u5b9a\u65cb\u5f8b)\uc774\uae30 \ub54c\ubb38\uc5d0 \uc804\uc218\uc0dd\uc5d0\uac8c \ub9ac\ub4ec\uacfc \uac00\ub77d\ubfd0\ub9cc \uc544\ub2c8\ub77c \ucc3d\ubc95\uae4c\uc9c0\ub3c4 \ud558\ub098\ud558\ub098 \uad50\uc218\ud55c\ub2e4. \uadf8\ub7ec\ub098 \ucc3d\uadf9\uc758 \uacbd\uc6b0\uc5d0\ub294 \ucc3d\uc790\ub4e4\uc774 \uc5b4\ub290 \uc815\ub3c4 \ud310\uc18c\ub9ac\uc758 \ucc3d\uc744 \uad6c\uc0ac\ud560 \uc218 \uc788\ub294 \ub2a5\ub825\uc774 \uc788\uc73c\ubbc0\ub85c \uc7a5\ub2e8\uacfc \uc870 \uadf8\ub9ac\uace0 \uac00\ub77d\uc758 \uace8\uaca9\uc744 \uc9c0\uc2dc\ud558\uc9c0\ub9cc \uc138\ubd80 \ucc3d\ubc95\uacfc \uac19\uc740 \uac83\uc740 \ucc3d\uc790\uc758 \uae30\ub7c9\uc5d0 \ub9e1\uae30\uae30 \ub54c\ubb38\uc5d0 \ucc3d\uc790\uc758 \ucc3d\ubc95\uc5d0 \uc758\ud55c \uac1c\uc131\uc774 \ubc30\uc5ed\ub9c8\ub2e4 \uc2ec\ud558\uac8c \ub178\ucd9c\ub418\uae30\ub3c4 \ud55c\ub2e4. \ub3c4\ucc3d\uc740 \uc77c\ubc18\uc801\uc73c\ub85c \uc6d0\ub85c\uae09 \ucc3d\uc790\uac00 \ub9e1\uac8c \ub418\ubbc0\ub85c \uc7a5\ub2e8\uacfc \uc870(\u8abf)\ub9cc \uc9c0\uc2dc\ud558\uace0 \ub098\uba38\uc9c0\ub294 \ucc3d\uc790\uc758 \uc74c\uc545\uc131\uacfc \ud45c\ucd9c\ub825\uc5d0 \ub9e1\uae34\ub2e4. \uc81c\ucc3d\uc758 \uacbd\uc6b0\uc5d0\ub294 \ub2e4\uc218\uc758 \ucd9c\uc5f0\uc790\uac00 \uac19\uc740 \uac00\ub77d\uc73c\ub85c \ubd88\ub7ec\uc57c \ud558\uae30 \ub54c\ubb38\uc5d0 \ud569\ub3d9 \ucc3d \uc5f0\uc2b5\uc774 \ud544\uc694\ud558\ub2e4.", "paragraph_answer": "\ub300\ubcf8\uc5d0 \ub098\ud0c0\ub09c \ucc3d \ubc0f \ub3c4\ucc3d \ubd80\ubd84\uc740 \ucc3d\uadf9 \uc791\uace1\uc790 \uc758 \ucc3d\uc5d0 \ub300\ud55c \uc9c0\ub3c4\ub85c\ubd80\ud130 \uc2dc\uc791\ub41c\ub2e4. \uc624\ud398\ub77c\ucc98\ub7fc \uc545\ubcf4\uc5d0 \uc758\ud55c \uac1c\uc778 \uc5f0\uc2b5\uc774 \uc120\ud589\ub418\ub294 \uac83\uc774 \uc544\ub2c8\uace0, \uc791\uace1\uc790\uac00 \uc7a5\ub2e8\uacfc \uc870\ub97c \uc9c0\uc2dc\ud558\uace0 \uc9c1\uc811 \ucc3d\uc744 \ubd88\ub7ec\uc11c \uad6c\uc804, \uc2ec\uc218(\u53e3\u50b3\u5fc3\u53d7)\ud558\ub294 \uac83\uc774\ub2e4. \ud310\uc18c\ub9ac\uc77c \uacbd\uc6b0\uc5d0\ub294 \uc5b4\ub290 \uc815\ub3c4 \uace0\uc815\uc120\uc728(\u56fa\u5b9a\u65cb\u5f8b)\uc774\uae30 \ub54c\ubb38\uc5d0 \uc804\uc218\uc0dd\uc5d0\uac8c \ub9ac\ub4ec\uacfc \uac00\ub77d\ubfd0\ub9cc \uc544\ub2c8\ub77c \ucc3d\ubc95\uae4c\uc9c0\ub3c4 \ud558\ub098\ud558\ub098 \uad50\uc218\ud55c\ub2e4. \uadf8\ub7ec\ub098 \ucc3d\uadf9\uc758 \uacbd\uc6b0\uc5d0\ub294 \ucc3d\uc790\ub4e4\uc774 \uc5b4\ub290 \uc815\ub3c4 \ud310\uc18c\ub9ac\uc758 \ucc3d\uc744 \uad6c\uc0ac\ud560 \uc218 \uc788\ub294 \ub2a5\ub825\uc774 \uc788\uc73c\ubbc0\ub85c \uc7a5\ub2e8\uacfc \uc870 \uadf8\ub9ac\uace0 \uac00\ub77d\uc758 \uace8\uaca9\uc744 \uc9c0\uc2dc\ud558\uc9c0\ub9cc \uc138\ubd80 \ucc3d\ubc95\uacfc \uac19\uc740 \uac83\uc740 \ucc3d\uc790\uc758 \uae30\ub7c9\uc5d0 \ub9e1\uae30\uae30 \ub54c\ubb38\uc5d0 \ucc3d\uc790\uc758 \ucc3d\ubc95\uc5d0 \uc758\ud55c \uac1c\uc131\uc774 \ubc30\uc5ed\ub9c8\ub2e4 \uc2ec\ud558\uac8c \ub178\ucd9c\ub418\uae30\ub3c4 \ud55c\ub2e4. \ub3c4\ucc3d\uc740 \uc77c\ubc18\uc801\uc73c\ub85c \uc6d0\ub85c\uae09 \ucc3d\uc790\uac00 \ub9e1\uac8c \ub418\ubbc0\ub85c \uc7a5\ub2e8\uacfc \uc870(\u8abf)\ub9cc \uc9c0\uc2dc\ud558\uace0 \ub098\uba38\uc9c0\ub294 \ucc3d\uc790\uc758 \uc74c\uc545\uc131\uacfc \ud45c\ucd9c\ub825\uc5d0 \ub9e1\uae34\ub2e4. \uc81c\ucc3d\uc758 \uacbd\uc6b0\uc5d0\ub294 \ub2e4\uc218\uc758 \ucd9c\uc5f0\uc790\uac00 \uac19\uc740 \uac00\ub77d\uc73c\ub85c \ubd88\ub7ec\uc57c \ud558\uae30 \ub54c\ubb38\uc5d0 \ud569\ub3d9 \ucc3d \uc5f0\uc2b5\uc774 \ud544\uc694\ud558\ub2e4.", "sentence_answer": "\ub300\ubcf8\uc5d0 \ub098\ud0c0\ub09c \ucc3d \ubc0f \ub3c4\ucc3d \ubd80\ubd84\uc740 \ucc3d\uadf9 \uc791\uace1\uc790 \uc758 \ucc3d\uc5d0 \ub300\ud55c \uc9c0\ub3c4\ub85c\ubd80\ud130 \uc2dc\uc791\ub41c\ub2e4.", "paragraph_id": "6557875-7-0", "paragraph_question": "question: \ub300\ubcf8\uc5d0 \ub098\ud0c0\ub098 \ucc3d \ubc0f \ub3c4\ucc3d \ubd80\ubd84\uc744 \uc9c0\ub3c4\ud558\ub294 \uac83\uc740 \ub204\uad6c\uc778\uac00?, context: \ub300\ubcf8\uc5d0 \ub098\ud0c0\ub09c \ucc3d \ubc0f \ub3c4\ucc3d \ubd80\ubd84\uc740 \ucc3d\uadf9 \uc791\uace1\uc790\uc758 \ucc3d\uc5d0 \ub300\ud55c \uc9c0\ub3c4\ub85c\ubd80\ud130 \uc2dc\uc791\ub41c\ub2e4. \uc624\ud398\ub77c\ucc98\ub7fc \uc545\ubcf4\uc5d0 \uc758\ud55c \uac1c\uc778 \uc5f0\uc2b5\uc774 \uc120\ud589\ub418\ub294 \uac83\uc774 \uc544\ub2c8\uace0, \uc791\uace1\uc790\uac00 \uc7a5\ub2e8\uacfc \uc870\ub97c \uc9c0\uc2dc\ud558\uace0 \uc9c1\uc811 \ucc3d\uc744 \ubd88\ub7ec\uc11c \uad6c\uc804, \uc2ec\uc218(\u53e3\u50b3\u5fc3\u53d7)\ud558\ub294 \uac83\uc774\ub2e4. \ud310\uc18c\ub9ac\uc77c \uacbd\uc6b0\uc5d0\ub294 \uc5b4\ub290 \uc815\ub3c4 \uace0\uc815\uc120\uc728(\u56fa\u5b9a\u65cb\u5f8b)\uc774\uae30 \ub54c\ubb38\uc5d0 \uc804\uc218\uc0dd\uc5d0\uac8c \ub9ac\ub4ec\uacfc \uac00\ub77d\ubfd0\ub9cc \uc544\ub2c8\ub77c \ucc3d\ubc95\uae4c\uc9c0\ub3c4 \ud558\ub098\ud558\ub098 \uad50\uc218\ud55c\ub2e4. \uadf8\ub7ec\ub098 \ucc3d\uadf9\uc758 \uacbd\uc6b0\uc5d0\ub294 \ucc3d\uc790\ub4e4\uc774 \uc5b4\ub290 \uc815\ub3c4 \ud310\uc18c\ub9ac\uc758 \ucc3d\uc744 \uad6c\uc0ac\ud560 \uc218 \uc788\ub294 \ub2a5\ub825\uc774 \uc788\uc73c\ubbc0\ub85c \uc7a5\ub2e8\uacfc \uc870 \uadf8\ub9ac\uace0 \uac00\ub77d\uc758 \uace8\uaca9\uc744 \uc9c0\uc2dc\ud558\uc9c0\ub9cc \uc138\ubd80 \ucc3d\ubc95\uacfc \uac19\uc740 \uac83\uc740 \ucc3d\uc790\uc758 \uae30\ub7c9\uc5d0 \ub9e1\uae30\uae30 \ub54c\ubb38\uc5d0 \ucc3d\uc790\uc758 \ucc3d\ubc95\uc5d0 \uc758\ud55c \uac1c\uc131\uc774 \ubc30\uc5ed\ub9c8\ub2e4 \uc2ec\ud558\uac8c \ub178\ucd9c\ub418\uae30\ub3c4 \ud55c\ub2e4. \ub3c4\ucc3d\uc740 \uc77c\ubc18\uc801\uc73c\ub85c \uc6d0\ub85c\uae09 \ucc3d\uc790\uac00 \ub9e1\uac8c \ub418\ubbc0\ub85c \uc7a5\ub2e8\uacfc \uc870(\u8abf)\ub9cc \uc9c0\uc2dc\ud558\uace0 \ub098\uba38\uc9c0\ub294 \ucc3d\uc790\uc758 \uc74c\uc545\uc131\uacfc \ud45c\ucd9c\ub825\uc5d0 \ub9e1\uae34\ub2e4. \uc81c\ucc3d\uc758 \uacbd\uc6b0\uc5d0\ub294 \ub2e4\uc218\uc758 \ucd9c\uc5f0\uc790\uac00 \uac19\uc740 \uac00\ub77d\uc73c\ub85c \ubd88\ub7ec\uc57c \ud558\uae30 \ub54c\ubb38\uc5d0 \ud569\ub3d9 \ucc3d \uc5f0\uc2b5\uc774 \ud544\uc694\ud558\ub2e4."} {"question": "\ud55c\uad6d\uc5ec\uc131\ub2e8\uccb4\ud611\uc758\ud68c\uac00 \ud654\ud3d0 \uc778\ubb3c\ub85c \uc720\uad00\uc21c\uc744 \uc9c0\uc9c0\ud558\ub294 \uc131\uba85\uc744 \ubc1c\ud45c\ud55c \ub0a0\uc9dc\ub294?", "paragraph": "\uc77c\ubd80 \uc5ec\uc131\ub2e8\uccb4\uc640 \ubb38\ud654\ubbf8\ub798 \uc774\ud504 \ub4f1\uc758 \ubc18\ub300 \uc774\ud6c4 \uc5ec\ub7ec \uc5ec\uc131\ub2e8\uccb4\ub4e4\uc774 \uc2e0\uc0ac\uc784\ub2f9 \uc120\uc815\uc744 \ubc18\ub300\ud558\uac70\ub098 \ubd80\uc815\uc801\uc73c\ub85c \ud3c9\uac00\ud558\uc600\ub2e4. \ud55c\uad6d\uc5ec\uc131\ub2e8\uccb4\ud611\uc758\ud68c(\ud68c\uc7a5 \uae40\ud654\uc911)\ub294 2007\ub144 10\uc6d4 23\uc77c \uc131\uba85\uc744 \ub0b4\uc5b4 \u201c\uac1c\uc778\uc758 \uc131\ucde8\uac00 \uacf5\ub3d9\uccb4\uc758 \ubc1c\uc804, \uc989 \ud0c0\uc778\uc758 \uc0b6\uc5d0\ub3c4 \ub3c4\uc6c0\uc744 \uc8fc\uc5b4\uc57c \ud55c\ub2e4\ub294 \uac00\ub974\uce68\uc744 \uc774 \uc2dc\ub300\uc5d0 \ub358\uc9c0\uace0 \uc788\ub2e4\u201d\uba70 \uc720\uad00\uc21c \uc9c0\uc9c0 \ub73b\uc744 \ubc1d\ud614\ub2e4. \ud55c\uad6d\uc5ec\uc131\ub2e8\uccb4\uc5f0\ud569, \ud55c\uad6d\uc5ec\uc131\ub178\ub3d9\uc790\ud68c, \ud55c\uad6d\uc5ec\uc131\uc758\uc804\ud654\uc5f0\ud569, \ud55c\uad6d\uc131\ud3ed\ub825\uc0c1\ub2f4\uc18c, \uc131\ub9e4\ub9e4\ubb38\uc81c\ud574\uacb0\uc744\uc704\ud55c\uc804\uad6d\uc5f0\ub300 \ub4f1 5\uac1c \ub2e8\uccb4\ub3c4 \uc774\ub0a0 \uc131\uba85\uc744 \ub0b4\uc5b4 \u201c\ud604\uc7ac \uac70\ub860\ub418\ub294 \ud654\ud3d0 \uc778\ubb3c \uc911\uc5d0\ub294 \uc720\uad00\uc21c\uc774 \uc801\ud569\ud558\ub2e4\u201d\uace0 \uc8fc\uc7a5\ud588\ub2e4. \ub610 \uc5ec\uc131\ub2e8\uccb4\uc5f0\ud569 \ub4f1\uc740 \uad6d\ubbfc\uc5ec\ub860 \ubc18\uc601 \ubd80\uc871\uacfc \uc120\uc815\uacfc\uc815 \ubbf8\uacf5\uac1c\uc758 \ubb38\uc81c\uc810 \ub610\ud55c \uc9c0\uc801\ud588\ub2e4. \ud55c\uad6d\uc5ec\uc131\ub2e8\uccb4\uc5f0\ud569 \uac04\uc0ac \ud55c\ud669\uc8fc\uc5f0 \ub4f1\uc740 \u201c\ud55c\uad6d\uc740\ud589\uc774 \uc9c0\ub09c 5\uc6d4\ubd80\ud130 7\uc6d4\ub9d0\uae4c\uc9c0 \uc9c4\ud589\ud55c \uc124\ubb38\uc870\uc0ac\uc5d0 \ub530\ub974\uba74 \uc5ec\uc131\uc778\ubb3c \uc911\uc5d0\ub294 \uc720\uad00\uc21c\uc5d0 \ub300\ud55c \uc9c0\uc9c0\ub3c4\uac00 \uac00\uc7a5 \ub192\uc558\ub2e4\u201d\uace0 \ubc1d\ud614\ub2e4.", "answer": "2007\ub144 10\uc6d4 23\uc77c", "sentence": "\ud55c\uad6d\uc5ec\uc131\ub2e8\uccb4\ud611\uc758\ud68c(\ud68c\uc7a5 \uae40\ud654\uc911)\ub294 2007\ub144 10\uc6d4 23\uc77c \uc131\uba85\uc744 \ub0b4\uc5b4 \u201c\uac1c\uc778\uc758 \uc131\ucde8\uac00 \uacf5\ub3d9\uccb4\uc758 \ubc1c\uc804, \uc989 \ud0c0\uc778\uc758 \uc0b6\uc5d0\ub3c4 \ub3c4\uc6c0\uc744 \uc8fc\uc5b4\uc57c \ud55c\ub2e4\ub294 \uac00\ub974\uce68\uc744 \uc774 \uc2dc\ub300\uc5d0 \ub358\uc9c0\uace0 \uc788\ub2e4\u201d\uba70 \uc720\uad00\uc21c \uc9c0\uc9c0 \ub73b\uc744 \ubc1d\ud614\ub2e4.", "paragraph_sentence": "\uc77c\ubd80 \uc5ec\uc131\ub2e8\uccb4\uc640 \ubb38\ud654\ubbf8\ub798 \uc774\ud504 \ub4f1\uc758 \ubc18\ub300 \uc774\ud6c4 \uc5ec\ub7ec \uc5ec\uc131\ub2e8\uccb4\ub4e4\uc774 \uc2e0\uc0ac\uc784\ub2f9 \uc120\uc815\uc744 \ubc18\ub300\ud558\uac70\ub098 \ubd80\uc815\uc801\uc73c\ub85c \ud3c9\uac00\ud558\uc600\ub2e4. \ud55c\uad6d\uc5ec\uc131\ub2e8\uccb4\ud611\uc758\ud68c(\ud68c\uc7a5 \uae40\ud654\uc911)\ub294 2007\ub144 10\uc6d4 23\uc77c \uc131\uba85\uc744 \ub0b4\uc5b4 \u201c\uac1c\uc778\uc758 \uc131\ucde8\uac00 \uacf5\ub3d9\uccb4\uc758 \ubc1c\uc804, \uc989 \ud0c0\uc778\uc758 \uc0b6\uc5d0\ub3c4 \ub3c4\uc6c0\uc744 \uc8fc\uc5b4\uc57c \ud55c\ub2e4\ub294 \uac00\ub974\uce68\uc744 \uc774 \uc2dc\ub300\uc5d0 \ub358\uc9c0\uace0 \uc788\ub2e4\u201d\uba70 \uc720\uad00\uc21c \uc9c0\uc9c0 \ub73b\uc744 \ubc1d\ud614\ub2e4. \ud55c\uad6d\uc5ec\uc131\ub2e8\uccb4\uc5f0\ud569, \ud55c\uad6d\uc5ec\uc131\ub178\ub3d9\uc790\ud68c, \ud55c\uad6d\uc5ec\uc131\uc758\uc804\ud654\uc5f0\ud569, \ud55c\uad6d\uc131\ud3ed\ub825\uc0c1\ub2f4\uc18c, \uc131\ub9e4\ub9e4\ubb38\uc81c\ud574\uacb0\uc744\uc704\ud55c\uc804\uad6d\uc5f0\ub300 \ub4f1 5\uac1c \ub2e8\uccb4\ub3c4 \uc774\ub0a0 \uc131\uba85\uc744 \ub0b4\uc5b4 \u201c\ud604\uc7ac \uac70\ub860\ub418\ub294 \ud654\ud3d0 \uc778\ubb3c \uc911\uc5d0\ub294 \uc720\uad00\uc21c\uc774 \uc801\ud569\ud558\ub2e4\u201d\uace0 \uc8fc\uc7a5\ud588\ub2e4. \ub610 \uc5ec\uc131\ub2e8\uccb4\uc5f0\ud569 \ub4f1\uc740 \uad6d\ubbfc\uc5ec\ub860 \ubc18\uc601 \ubd80\uc871\uacfc \uc120\uc815\uacfc\uc815 \ubbf8\uacf5\uac1c\uc758 \ubb38\uc81c\uc810 \ub610\ud55c \uc9c0\uc801\ud588\ub2e4. \ud55c\uad6d\uc5ec\uc131\ub2e8\uccb4\uc5f0\ud569 \uac04\uc0ac \ud55c\ud669\uc8fc\uc5f0 \ub4f1\uc740 \u201c\ud55c\uad6d\uc740\ud589\uc774 \uc9c0\ub09c 5\uc6d4\ubd80\ud130 7\uc6d4\ub9d0\uae4c\uc9c0 \uc9c4\ud589\ud55c \uc124\ubb38\uc870\uc0ac\uc5d0 \ub530\ub974\uba74 \uc5ec\uc131\uc778\ubb3c \uc911\uc5d0\ub294 \uc720\uad00\uc21c\uc5d0 \ub300\ud55c \uc9c0\uc9c0\ub3c4\uac00 \uac00\uc7a5 \ub192\uc558\ub2e4\u201d\uace0 \ubc1d\ud614\ub2e4.", "paragraph_answer": "\uc77c\ubd80 \uc5ec\uc131\ub2e8\uccb4\uc640 \ubb38\ud654\ubbf8\ub798 \uc774\ud504 \ub4f1\uc758 \ubc18\ub300 \uc774\ud6c4 \uc5ec\ub7ec \uc5ec\uc131\ub2e8\uccb4\ub4e4\uc774 \uc2e0\uc0ac\uc784\ub2f9 \uc120\uc815\uc744 \ubc18\ub300\ud558\uac70\ub098 \ubd80\uc815\uc801\uc73c\ub85c \ud3c9\uac00\ud558\uc600\ub2e4. \ud55c\uad6d\uc5ec\uc131\ub2e8\uccb4\ud611\uc758\ud68c(\ud68c\uc7a5 \uae40\ud654\uc911)\ub294 2007\ub144 10\uc6d4 23\uc77c \uc131\uba85\uc744 \ub0b4\uc5b4 \u201c\uac1c\uc778\uc758 \uc131\ucde8\uac00 \uacf5\ub3d9\uccb4\uc758 \ubc1c\uc804, \uc989 \ud0c0\uc778\uc758 \uc0b6\uc5d0\ub3c4 \ub3c4\uc6c0\uc744 \uc8fc\uc5b4\uc57c \ud55c\ub2e4\ub294 \uac00\ub974\uce68\uc744 \uc774 \uc2dc\ub300\uc5d0 \ub358\uc9c0\uace0 \uc788\ub2e4\u201d\uba70 \uc720\uad00\uc21c \uc9c0\uc9c0 \ub73b\uc744 \ubc1d\ud614\ub2e4. \ud55c\uad6d\uc5ec\uc131\ub2e8\uccb4\uc5f0\ud569, \ud55c\uad6d\uc5ec\uc131\ub178\ub3d9\uc790\ud68c, \ud55c\uad6d\uc5ec\uc131\uc758\uc804\ud654\uc5f0\ud569, \ud55c\uad6d\uc131\ud3ed\ub825\uc0c1\ub2f4\uc18c, \uc131\ub9e4\ub9e4\ubb38\uc81c\ud574\uacb0\uc744\uc704\ud55c\uc804\uad6d\uc5f0\ub300 \ub4f1 5\uac1c \ub2e8\uccb4\ub3c4 \uc774\ub0a0 \uc131\uba85\uc744 \ub0b4\uc5b4 \u201c\ud604\uc7ac \uac70\ub860\ub418\ub294 \ud654\ud3d0 \uc778\ubb3c \uc911\uc5d0\ub294 \uc720\uad00\uc21c\uc774 \uc801\ud569\ud558\ub2e4\u201d\uace0 \uc8fc\uc7a5\ud588\ub2e4. \ub610 \uc5ec\uc131\ub2e8\uccb4\uc5f0\ud569 \ub4f1\uc740 \uad6d\ubbfc\uc5ec\ub860 \ubc18\uc601 \ubd80\uc871\uacfc \uc120\uc815\uacfc\uc815 \ubbf8\uacf5\uac1c\uc758 \ubb38\uc81c\uc810 \ub610\ud55c \uc9c0\uc801\ud588\ub2e4. \ud55c\uad6d\uc5ec\uc131\ub2e8\uccb4\uc5f0\ud569 \uac04\uc0ac \ud55c\ud669\uc8fc\uc5f0 \ub4f1\uc740 \u201c\ud55c\uad6d\uc740\ud589\uc774 \uc9c0\ub09c 5\uc6d4\ubd80\ud130 7\uc6d4\ub9d0\uae4c\uc9c0 \uc9c4\ud589\ud55c \uc124\ubb38\uc870\uc0ac\uc5d0 \ub530\ub974\uba74 \uc5ec\uc131\uc778\ubb3c \uc911\uc5d0\ub294 \uc720\uad00\uc21c\uc5d0 \ub300\ud55c \uc9c0\uc9c0\ub3c4\uac00 \uac00\uc7a5 \ub192\uc558\ub2e4\u201d\uace0 \ubc1d\ud614\ub2e4.", "sentence_answer": "\ud55c\uad6d\uc5ec\uc131\ub2e8\uccb4\ud611\uc758\ud68c(\ud68c\uc7a5 \uae40\ud654\uc911)\ub294 2007\ub144 10\uc6d4 23\uc77c \uc131\uba85\uc744 \ub0b4\uc5b4 \u201c\uac1c\uc778\uc758 \uc131\ucde8\uac00 \uacf5\ub3d9\uccb4\uc758 \ubc1c\uc804, \uc989 \ud0c0\uc778\uc758 \uc0b6\uc5d0\ub3c4 \ub3c4\uc6c0\uc744 \uc8fc\uc5b4\uc57c \ud55c\ub2e4\ub294 \uac00\ub974\uce68\uc744 \uc774 \uc2dc\ub300\uc5d0 \ub358\uc9c0\uace0 \uc788\ub2e4\u201d\uba70 \uc720\uad00\uc21c \uc9c0\uc9c0 \ub73b\uc744 \ubc1d\ud614\ub2e4.", "paragraph_id": "6657247-5-1", "paragraph_question": "question: \ud55c\uad6d\uc5ec\uc131\ub2e8\uccb4\ud611\uc758\ud68c\uac00 \ud654\ud3d0 \uc778\ubb3c\ub85c \uc720\uad00\uc21c\uc744 \uc9c0\uc9c0\ud558\ub294 \uc131\uba85\uc744 \ubc1c\ud45c\ud55c \ub0a0\uc9dc\ub294?, context: \uc77c\ubd80 \uc5ec\uc131\ub2e8\uccb4\uc640 \ubb38\ud654\ubbf8\ub798 \uc774\ud504 \ub4f1\uc758 \ubc18\ub300 \uc774\ud6c4 \uc5ec\ub7ec \uc5ec\uc131\ub2e8\uccb4\ub4e4\uc774 \uc2e0\uc0ac\uc784\ub2f9 \uc120\uc815\uc744 \ubc18\ub300\ud558\uac70\ub098 \ubd80\uc815\uc801\uc73c\ub85c \ud3c9\uac00\ud558\uc600\ub2e4. \ud55c\uad6d\uc5ec\uc131\ub2e8\uccb4\ud611\uc758\ud68c(\ud68c\uc7a5 \uae40\ud654\uc911)\ub294 2007\ub144 10\uc6d4 23\uc77c \uc131\uba85\uc744 \ub0b4\uc5b4 \u201c\uac1c\uc778\uc758 \uc131\ucde8\uac00 \uacf5\ub3d9\uccb4\uc758 \ubc1c\uc804, \uc989 \ud0c0\uc778\uc758 \uc0b6\uc5d0\ub3c4 \ub3c4\uc6c0\uc744 \uc8fc\uc5b4\uc57c \ud55c\ub2e4\ub294 \uac00\ub974\uce68\uc744 \uc774 \uc2dc\ub300\uc5d0 \ub358\uc9c0\uace0 \uc788\ub2e4\u201d\uba70 \uc720\uad00\uc21c \uc9c0\uc9c0 \ub73b\uc744 \ubc1d\ud614\ub2e4. \ud55c\uad6d\uc5ec\uc131\ub2e8\uccb4\uc5f0\ud569, \ud55c\uad6d\uc5ec\uc131\ub178\ub3d9\uc790\ud68c, \ud55c\uad6d\uc5ec\uc131\uc758\uc804\ud654\uc5f0\ud569, \ud55c\uad6d\uc131\ud3ed\ub825\uc0c1\ub2f4\uc18c, \uc131\ub9e4\ub9e4\ubb38\uc81c\ud574\uacb0\uc744\uc704\ud55c\uc804\uad6d\uc5f0\ub300 \ub4f1 5\uac1c \ub2e8\uccb4\ub3c4 \uc774\ub0a0 \uc131\uba85\uc744 \ub0b4\uc5b4 \u201c\ud604\uc7ac \uac70\ub860\ub418\ub294 \ud654\ud3d0 \uc778\ubb3c \uc911\uc5d0\ub294 \uc720\uad00\uc21c\uc774 \uc801\ud569\ud558\ub2e4\u201d\uace0 \uc8fc\uc7a5\ud588\ub2e4. \ub610 \uc5ec\uc131\ub2e8\uccb4\uc5f0\ud569 \ub4f1\uc740 \uad6d\ubbfc\uc5ec\ub860 \ubc18\uc601 \ubd80\uc871\uacfc \uc120\uc815\uacfc\uc815 \ubbf8\uacf5\uac1c\uc758 \ubb38\uc81c\uc810 \ub610\ud55c \uc9c0\uc801\ud588\ub2e4. \ud55c\uad6d\uc5ec\uc131\ub2e8\uccb4\uc5f0\ud569 \uac04\uc0ac \ud55c\ud669\uc8fc\uc5f0 \ub4f1\uc740 \u201c\ud55c\uad6d\uc740\ud589\uc774 \uc9c0\ub09c 5\uc6d4\ubd80\ud130 7\uc6d4\ub9d0\uae4c\uc9c0 \uc9c4\ud589\ud55c \uc124\ubb38\uc870\uc0ac\uc5d0 \ub530\ub974\uba74 \uc5ec\uc131\uc778\ubb3c \uc911\uc5d0\ub294 \uc720\uad00\uc21c\uc5d0 \ub300\ud55c \uc9c0\uc9c0\ub3c4\uac00 \uac00\uc7a5 \ub192\uc558\ub2e4\u201d\uace0 \ubc1d\ud614\ub2e4."} {"question": "\uba54\uac74 \ud3ed\uc2a4\uac00 2010\ub144\uc5d0 \uc553\uace0 \uc788\ub2e4\uace0 \ubc1d\ud78c \uc7a5\uc560\ub294 \ubb34\uc5c7\uc778\uac00?", "paragraph": "\ud3ed\uc2a4\ub294 \ub610\ud55c \uc624\ub978\uc190 \uc5c4\uc9c0 \uc190\uac00\ub77d\uc774 \ub2e8\uc9c0\uc99d\uc744 \uc553\uace0 \uc788\ub2e4\uace0\ud55c\ub2e4. 2010\ub144\uc5d0\ub294 \uac15\ubc15 \uc7a5\uc560\ub97c \uc553\uace0\uc788\ub2e4 \ubc1d\ud788\uba70 \"\ub2e4\ub978 \uc0ac\ub78c\uc774 \uc4f0\ub358 \ud654\uc7a5\uc2e4\uc774\ub098 \uc695\uc2e4\uc744 \ubcf4\uba74 \ubc15\ud14c\ub9ac\uc544\uac00 \uc0ac\ubc29\uc5d0 \ub4dd\uc2e4\uac70\ub9ac\ub294 \uac83 \uac19\ub2e4.\"\ub77c\uace0 \ub9d0\ud588\uace0 \uc774\uc5b4 \"\uc2dd\ub2f9 \ud3ec\ud06c \ub610\ud55c \ub2e4\ub978 \uc0ac\ub78c\ub4e4\uc774 \uc0ac\uc6a9\ud588\uc744 \ud150\ub370 \ub0b4 \uc785\uc18d\uc73c\ub85c \ubc15\ud14c\ub9ac\uc544\uac00 \ub4e4\uc5b4\uac04\ub2e4\ub294 \uc0dd\uac01\uc744 \ud558\uba74 \ub054\ucc0d\ud558\ub2e4.\"\ub77c\uba70 \uc2ec\uacbd\uc744 \ud1a0\ud588\ub2e4. \ub610\ud55c \uc601\uad6d\uc758 \uc7a1\uc9c0 \u300a\uc6d0\ub354\ub79c\ub4dc\u300b\uc640\uc758 \uc778\ud130\ubdf0\uc5d0\uc11c \"\ub098\ub294 \uc815\uc2e0 \uc9c8\ud658\uc774 \uc788\ub2e4. \uacbd\uacc4\uc131 \uc778\uaca9\uc7a5\uc560\ub098 \uacbd\ubbf8\ud55c \uc815\uc2e0 \ubd84\uc5f4\uc99d \ubc1c\uc791 \uc99d\uc138\uac00 \uc788\ub2e4\ub294 \uc0dd\uac01\uacfc \ub04a\uc784\uc5c6\uc774 \uc2f8\uc6b0\uace0 \uc788\ub2e4\"\ub77c\uace0 \ubc1d\ud614\uace0, \uc774\uc5b4 \ub9e4\ub9b4\ub9b0 \uba3c\ub85c\ub97c \ub9e4\uc6b0 \uc88b\uc544\ud55c\ub2e4\uace0 \ub9d0\ud588\ub2e4. 2009\ub144\uc5d0\ub294 NBC \u300a\uc9c0\ubbf8 \ud330\ub7f0 \uc1fc\u300b\uc5d0 \ucd9c\uc5f0\ud574 \"\ub300\ubcf8\uc774\ub4e0 \uc2e0\ubb38\uc774\ub4e0 \uac74\uc870\ud55c \uc885\uc774\ub97c \ub9cc\uc9c0\ub294 \uac83\uc744 \ucc38\uc9c0 \ubabb\ud558\uaca0\uc5b4\uc694. \uadf8\ub798\uc11c \ud56d\uc0c1 \uc190\uc5d0 \uce68\uc744 \ubc1c\ub77c\uc57c \ud574\uc694\"\ub77c\uace0 \ubc1d\ud614\uace0 \uc774\uc5b4 \"\ucc45\uc744 \uc77d\uace0 \uc2f6\uc73c\uba74 \uc606\uc5d0 \ubb3c \ud55c \ucef5\uc744 \ub5a0\ub2e4 \ub193\uace0 \uc190\uac00\ub77d\uc744 \ubb3c\uc5d0 \uacc4\uc18d \ub2f4\uac00 \uc900\ub2e4\"\uace0 \ub9d0\ud588\ub2e4. \ub610\ud55c \"\uc800\ub294 \uc9d1\uc5d0 \ubd88\uc744 \ud56d\uc0c1 \ucf1c\ub450\ub294\ub370, \ubd88\uc774 \uaebc\uc9c0\uba74 \uc11c\ub458\ub7ec \uc2a4\uc704\uce58\uac00 \uc788\ub294 \uacf3\uc73c\ub85c \ub2ec\ub824\uac04\ub2e4. \uc548 \ubcf4\uc774\uba74 \ubb34\uc12d\ub2e4\"\uace0 \ub9d0\ud558\uba70 \uc720\ub839\uc744 \ubbff\uc5b4 \uc5b4\ub450\uc6b4 \uacf3\uc744 \uc2eb\uc5b4\ud55c\ub2e4.", "answer": "\uac15\ubc15 \uc7a5\uc560", "sentence": "2010\ub144\uc5d0\ub294 \uac15\ubc15 \uc7a5\uc560 \ub97c \uc553\uace0\uc788\ub2e4 \ubc1d\ud788\uba70 \"\ub2e4\ub978 \uc0ac\ub78c\uc774 \uc4f0\ub358 \ud654\uc7a5\uc2e4\uc774\ub098 \uc695\uc2e4\uc744 \ubcf4\uba74 \ubc15\ud14c\ub9ac\uc544\uac00 \uc0ac\ubc29\uc5d0 \ub4dd\uc2e4\uac70\ub9ac\ub294 \uac83 \uac19\ub2e4.", "paragraph_sentence": "\ud3ed\uc2a4\ub294 \ub610\ud55c \uc624\ub978\uc190 \uc5c4\uc9c0 \uc190\uac00\ub77d\uc774 \ub2e8\uc9c0\uc99d\uc744 \uc553\uace0 \uc788\ub2e4\uace0\ud55c\ub2e4. 2010\ub144\uc5d0\ub294 \uac15\ubc15 \uc7a5\uc560 \ub97c \uc553\uace0\uc788\ub2e4 \ubc1d\ud788\uba70 \"\ub2e4\ub978 \uc0ac\ub78c\uc774 \uc4f0\ub358 \ud654\uc7a5\uc2e4\uc774\ub098 \uc695\uc2e4\uc744 \ubcf4\uba74 \ubc15\ud14c\ub9ac\uc544\uac00 \uc0ac\ubc29\uc5d0 \ub4dd\uc2e4\uac70\ub9ac\ub294 \uac83 \uac19\ub2e4. \"\ub77c\uace0 \ub9d0\ud588\uace0 \uc774\uc5b4 \"\uc2dd\ub2f9 \ud3ec\ud06c \ub610\ud55c \ub2e4\ub978 \uc0ac\ub78c\ub4e4\uc774 \uc0ac\uc6a9\ud588\uc744 \ud150\ub370 \ub0b4 \uc785\uc18d\uc73c\ub85c \ubc15\ud14c\ub9ac\uc544\uac00 \ub4e4\uc5b4\uac04\ub2e4\ub294 \uc0dd\uac01\uc744 \ud558\uba74 \ub054\ucc0d\ud558\ub2e4. \"\ub77c\uba70 \uc2ec\uacbd\uc744 \ud1a0\ud588\ub2e4. \ub610\ud55c \uc601\uad6d\uc758 \uc7a1\uc9c0 \u300a\uc6d0\ub354\ub79c\ub4dc\u300b\uc640\uc758 \uc778\ud130\ubdf0\uc5d0\uc11c \"\ub098\ub294 \uc815\uc2e0 \uc9c8\ud658\uc774 \uc788\ub2e4. \uacbd\uacc4\uc131 \uc778\uaca9\uc7a5\uc560\ub098 \uacbd\ubbf8\ud55c \uc815\uc2e0 \ubd84\uc5f4\uc99d \ubc1c\uc791 \uc99d\uc138\uac00 \uc788\ub2e4\ub294 \uc0dd\uac01\uacfc \ub04a\uc784\uc5c6\uc774 \uc2f8\uc6b0\uace0 \uc788\ub2e4\"\ub77c\uace0 \ubc1d\ud614\uace0, \uc774\uc5b4 \ub9e4\ub9b4\ub9b0 \uba3c\ub85c\ub97c \ub9e4\uc6b0 \uc88b\uc544\ud55c\ub2e4\uace0 \ub9d0\ud588\ub2e4. 2009\ub144\uc5d0\ub294 NBC \u300a\uc9c0\ubbf8 \ud330\ub7f0 \uc1fc\u300b\uc5d0 \ucd9c\uc5f0\ud574 \"\ub300\ubcf8\uc774\ub4e0 \uc2e0\ubb38\uc774\ub4e0 \uac74\uc870\ud55c \uc885\uc774\ub97c \ub9cc\uc9c0\ub294 \uac83\uc744 \ucc38\uc9c0 \ubabb\ud558\uaca0\uc5b4\uc694. \uadf8\ub798\uc11c \ud56d\uc0c1 \uc190\uc5d0 \uce68\uc744 \ubc1c\ub77c\uc57c \ud574\uc694\"\ub77c\uace0 \ubc1d\ud614\uace0 \uc774\uc5b4 \"\ucc45\uc744 \uc77d\uace0 \uc2f6\uc73c\uba74 \uc606\uc5d0 \ubb3c \ud55c \ucef5\uc744 \ub5a0\ub2e4 \ub193\uace0 \uc190\uac00\ub77d\uc744 \ubb3c\uc5d0 \uacc4\uc18d \ub2f4\uac00 \uc900\ub2e4\"\uace0 \ub9d0\ud588\ub2e4. \ub610\ud55c \"\uc800\ub294 \uc9d1\uc5d0 \ubd88\uc744 \ud56d\uc0c1 \ucf1c\ub450\ub294\ub370, \ubd88\uc774 \uaebc\uc9c0\uba74 \uc11c\ub458\ub7ec \uc2a4\uc704\uce58\uac00 \uc788\ub294 \uacf3\uc73c\ub85c \ub2ec\ub824\uac04\ub2e4. \uc548 \ubcf4\uc774\uba74 \ubb34\uc12d\ub2e4\"\uace0 \ub9d0\ud558\uba70 \uc720\ub839\uc744 \ubbff\uc5b4 \uc5b4\ub450\uc6b4 \uacf3\uc744 \uc2eb\uc5b4\ud55c\ub2e4.", "paragraph_answer": "\ud3ed\uc2a4\ub294 \ub610\ud55c \uc624\ub978\uc190 \uc5c4\uc9c0 \uc190\uac00\ub77d\uc774 \ub2e8\uc9c0\uc99d\uc744 \uc553\uace0 \uc788\ub2e4\uace0\ud55c\ub2e4. 2010\ub144\uc5d0\ub294 \uac15\ubc15 \uc7a5\uc560 \ub97c \uc553\uace0\uc788\ub2e4 \ubc1d\ud788\uba70 \"\ub2e4\ub978 \uc0ac\ub78c\uc774 \uc4f0\ub358 \ud654\uc7a5\uc2e4\uc774\ub098 \uc695\uc2e4\uc744 \ubcf4\uba74 \ubc15\ud14c\ub9ac\uc544\uac00 \uc0ac\ubc29\uc5d0 \ub4dd\uc2e4\uac70\ub9ac\ub294 \uac83 \uac19\ub2e4.\"\ub77c\uace0 \ub9d0\ud588\uace0 \uc774\uc5b4 \"\uc2dd\ub2f9 \ud3ec\ud06c \ub610\ud55c \ub2e4\ub978 \uc0ac\ub78c\ub4e4\uc774 \uc0ac\uc6a9\ud588\uc744 \ud150\ub370 \ub0b4 \uc785\uc18d\uc73c\ub85c \ubc15\ud14c\ub9ac\uc544\uac00 \ub4e4\uc5b4\uac04\ub2e4\ub294 \uc0dd\uac01\uc744 \ud558\uba74 \ub054\ucc0d\ud558\ub2e4.\"\ub77c\uba70 \uc2ec\uacbd\uc744 \ud1a0\ud588\ub2e4. \ub610\ud55c \uc601\uad6d\uc758 \uc7a1\uc9c0 \u300a\uc6d0\ub354\ub79c\ub4dc\u300b\uc640\uc758 \uc778\ud130\ubdf0\uc5d0\uc11c \"\ub098\ub294 \uc815\uc2e0 \uc9c8\ud658\uc774 \uc788\ub2e4. \uacbd\uacc4\uc131 \uc778\uaca9\uc7a5\uc560\ub098 \uacbd\ubbf8\ud55c \uc815\uc2e0 \ubd84\uc5f4\uc99d \ubc1c\uc791 \uc99d\uc138\uac00 \uc788\ub2e4\ub294 \uc0dd\uac01\uacfc \ub04a\uc784\uc5c6\uc774 \uc2f8\uc6b0\uace0 \uc788\ub2e4\"\ub77c\uace0 \ubc1d\ud614\uace0, \uc774\uc5b4 \ub9e4\ub9b4\ub9b0 \uba3c\ub85c\ub97c \ub9e4\uc6b0 \uc88b\uc544\ud55c\ub2e4\uace0 \ub9d0\ud588\ub2e4. 2009\ub144\uc5d0\ub294 NBC \u300a\uc9c0\ubbf8 \ud330\ub7f0 \uc1fc\u300b\uc5d0 \ucd9c\uc5f0\ud574 \"\ub300\ubcf8\uc774\ub4e0 \uc2e0\ubb38\uc774\ub4e0 \uac74\uc870\ud55c \uc885\uc774\ub97c \ub9cc\uc9c0\ub294 \uac83\uc744 \ucc38\uc9c0 \ubabb\ud558\uaca0\uc5b4\uc694. \uadf8\ub798\uc11c \ud56d\uc0c1 \uc190\uc5d0 \uce68\uc744 \ubc1c\ub77c\uc57c \ud574\uc694\"\ub77c\uace0 \ubc1d\ud614\uace0 \uc774\uc5b4 \"\ucc45\uc744 \uc77d\uace0 \uc2f6\uc73c\uba74 \uc606\uc5d0 \ubb3c \ud55c \ucef5\uc744 \ub5a0\ub2e4 \ub193\uace0 \uc190\uac00\ub77d\uc744 \ubb3c\uc5d0 \uacc4\uc18d \ub2f4\uac00 \uc900\ub2e4\"\uace0 \ub9d0\ud588\ub2e4. \ub610\ud55c \"\uc800\ub294 \uc9d1\uc5d0 \ubd88\uc744 \ud56d\uc0c1 \ucf1c\ub450\ub294\ub370, \ubd88\uc774 \uaebc\uc9c0\uba74 \uc11c\ub458\ub7ec \uc2a4\uc704\uce58\uac00 \uc788\ub294 \uacf3\uc73c\ub85c \ub2ec\ub824\uac04\ub2e4. \uc548 \ubcf4\uc774\uba74 \ubb34\uc12d\ub2e4\"\uace0 \ub9d0\ud558\uba70 \uc720\ub839\uc744 \ubbff\uc5b4 \uc5b4\ub450\uc6b4 \uacf3\uc744 \uc2eb\uc5b4\ud55c\ub2e4.", "sentence_answer": "2010\ub144\uc5d0\ub294 \uac15\ubc15 \uc7a5\uc560 \ub97c \uc553\uace0\uc788\ub2e4 \ubc1d\ud788\uba70 \"\ub2e4\ub978 \uc0ac\ub78c\uc774 \uc4f0\ub358 \ud654\uc7a5\uc2e4\uc774\ub098 \uc695\uc2e4\uc744 \ubcf4\uba74 \ubc15\ud14c\ub9ac\uc544\uac00 \uc0ac\ubc29\uc5d0 \ub4dd\uc2e4\uac70\ub9ac\ub294 \uac83 \uac19\ub2e4.", "paragraph_id": "6502694-7-1", "paragraph_question": "question: \uba54\uac74 \ud3ed\uc2a4\uac00 2010\ub144\uc5d0 \uc553\uace0 \uc788\ub2e4\uace0 \ubc1d\ud78c \uc7a5\uc560\ub294 \ubb34\uc5c7\uc778\uac00?, context: \ud3ed\uc2a4\ub294 \ub610\ud55c \uc624\ub978\uc190 \uc5c4\uc9c0 \uc190\uac00\ub77d\uc774 \ub2e8\uc9c0\uc99d\uc744 \uc553\uace0 \uc788\ub2e4\uace0\ud55c\ub2e4. 2010\ub144\uc5d0\ub294 \uac15\ubc15 \uc7a5\uc560\ub97c \uc553\uace0\uc788\ub2e4 \ubc1d\ud788\uba70 \"\ub2e4\ub978 \uc0ac\ub78c\uc774 \uc4f0\ub358 \ud654\uc7a5\uc2e4\uc774\ub098 \uc695\uc2e4\uc744 \ubcf4\uba74 \ubc15\ud14c\ub9ac\uc544\uac00 \uc0ac\ubc29\uc5d0 \ub4dd\uc2e4\uac70\ub9ac\ub294 \uac83 \uac19\ub2e4.\"\ub77c\uace0 \ub9d0\ud588\uace0 \uc774\uc5b4 \"\uc2dd\ub2f9 \ud3ec\ud06c \ub610\ud55c \ub2e4\ub978 \uc0ac\ub78c\ub4e4\uc774 \uc0ac\uc6a9\ud588\uc744 \ud150\ub370 \ub0b4 \uc785\uc18d\uc73c\ub85c \ubc15\ud14c\ub9ac\uc544\uac00 \ub4e4\uc5b4\uac04\ub2e4\ub294 \uc0dd\uac01\uc744 \ud558\uba74 \ub054\ucc0d\ud558\ub2e4.\"\ub77c\uba70 \uc2ec\uacbd\uc744 \ud1a0\ud588\ub2e4. \ub610\ud55c \uc601\uad6d\uc758 \uc7a1\uc9c0 \u300a\uc6d0\ub354\ub79c\ub4dc\u300b\uc640\uc758 \uc778\ud130\ubdf0\uc5d0\uc11c \"\ub098\ub294 \uc815\uc2e0 \uc9c8\ud658\uc774 \uc788\ub2e4. \uacbd\uacc4\uc131 \uc778\uaca9\uc7a5\uc560\ub098 \uacbd\ubbf8\ud55c \uc815\uc2e0 \ubd84\uc5f4\uc99d \ubc1c\uc791 \uc99d\uc138\uac00 \uc788\ub2e4\ub294 \uc0dd\uac01\uacfc \ub04a\uc784\uc5c6\uc774 \uc2f8\uc6b0\uace0 \uc788\ub2e4\"\ub77c\uace0 \ubc1d\ud614\uace0, \uc774\uc5b4 \ub9e4\ub9b4\ub9b0 \uba3c\ub85c\ub97c \ub9e4\uc6b0 \uc88b\uc544\ud55c\ub2e4\uace0 \ub9d0\ud588\ub2e4. 2009\ub144\uc5d0\ub294 NBC \u300a\uc9c0\ubbf8 \ud330\ub7f0 \uc1fc\u300b\uc5d0 \ucd9c\uc5f0\ud574 \"\ub300\ubcf8\uc774\ub4e0 \uc2e0\ubb38\uc774\ub4e0 \uac74\uc870\ud55c \uc885\uc774\ub97c \ub9cc\uc9c0\ub294 \uac83\uc744 \ucc38\uc9c0 \ubabb\ud558\uaca0\uc5b4\uc694. \uadf8\ub798\uc11c \ud56d\uc0c1 \uc190\uc5d0 \uce68\uc744 \ubc1c\ub77c\uc57c \ud574\uc694\"\ub77c\uace0 \ubc1d\ud614\uace0 \uc774\uc5b4 \"\ucc45\uc744 \uc77d\uace0 \uc2f6\uc73c\uba74 \uc606\uc5d0 \ubb3c \ud55c \ucef5\uc744 \ub5a0\ub2e4 \ub193\uace0 \uc190\uac00\ub77d\uc744 \ubb3c\uc5d0 \uacc4\uc18d \ub2f4\uac00 \uc900\ub2e4\"\uace0 \ub9d0\ud588\ub2e4. \ub610\ud55c \"\uc800\ub294 \uc9d1\uc5d0 \ubd88\uc744 \ud56d\uc0c1 \ucf1c\ub450\ub294\ub370, \ubd88\uc774 \uaebc\uc9c0\uba74 \uc11c\ub458\ub7ec \uc2a4\uc704\uce58\uac00 \uc788\ub294 \uacf3\uc73c\ub85c \ub2ec\ub824\uac04\ub2e4. \uc548 \ubcf4\uc774\uba74 \ubb34\uc12d\ub2e4\"\uace0 \ub9d0\ud558\uba70 \uc720\ub839\uc744 \ubbff\uc5b4 \uc5b4\ub450\uc6b4 \uacf3\uc744 \uc2eb\uc5b4\ud55c\ub2e4."} {"question": "12.12 \uad70\uc0ac\ubc18\ub780\uc73c\ub85c \uc2e0\uad70\ubd80\uac00 \uc2e4\uad8c\uc744 \uc7a5\uc545\ud55c \uc5f0\ub3c4\ub294?", "paragraph": "1980\ub144, 12\u00b712 \uad70\uc0ac \ubc18\ub780\uc73c\ub85c \uc2e4\uad8c\uc744 \uc7a5\uc545\ud55c \uc2e0\uad70\ubd80\ub294 \uc9d1\uad8c \uc2dc\ub098\ub9ac\uc624\uc5d0 \ub530\ub77c 80\ub144 5\uc6d4 17\uc77c \ube44\uc0c1\uacc4\uc5c4\uc744 \uc804\uad6d\uc73c\ub85c \ud655\ub300\ud558\uba74\uc11c \uc815\uce58\ud65c\ub3d9 \uae08\uc9c0\ub97c \uc8fc\uc694\ub0b4\uc6a9\uc73c\ub85c \ud55c \ud3ec\uace0\ub83910\ud638\ub97c \ubc1c\ud45c\ud568\uacfc \ud568\uaed8 \uae40\ub300\uc911\uc744 \ud3ec\ud568\ud55c \uc7ac\uc57c\uc778\uc0ac 20\uc5ec\uba85\uc744 \uc0ac\ud68c\ud63c\ub780 \ubc0f \ud559\uc0dd, \ub178\uc870 \ubc30\ud6c4\uc870\uc885 \ud610\uc758\ub85c \uc804\uaca9 \uc5f0\ud589\ud558\uc600\ub2e4. \uae40\ub300\uc911\uc740 \uc2e0\uad70\ubd80\uac00 \uc870\uc791\ud55c \uae40\ub300\uc911 \ub0b4\ub780\uc74c\ubaa8\uc0ac\uac74\uc73c\ub85c \uad70\uc0ac\uc7ac\ud310\uc5d0\uc11c \uc0ac\ud615\uc744 \uc120\uace0\ubc1b\uc558\ub2e4. \ub2f9\uc2dc \uae40\ub300\uc911\uc740 \"\uc774 \ub545\uc758 \ubbfc\uc8fc\uc8fc\uc758\uac00 \ud68c\ubcf5\ub418\uba74 \uba3c\uc800 \uc8fd\uc5b4\uac04 \ub098\ub97c \uc704\ud574\uc11c \uc815\uce58\ubcf4\ubcf5\uc774 \ub2e4\uc2dc\ub294 \ud589\ud574\uc9c0\uc9c0 \uc54a\ub3c4\ub85d \ud574\ub2ec\ub77c\"\ub77c\uace0 \ud55c \ubc95\uc815 \ucd5c\ud6c4 \uc9c4\uc220\uc774 \uad6d\uc81c\uc0ac\ud68c\uc5d0 \uc54c\ub824\uc9c0\uba74\uc11c \ud070 \ubc18\ud5a5\uc744 \ubd88\ub7ec\uc77c\uc73c\ud0a4\uace0 \uc9c0\ubbf8 \uce74\ud130 \uc804 \ubbf8\uad6d \ub300\ud1b5\ub839, \ub3c4\ub110\ub4dc \uadf8\ub808\uadf8 \uc804 \uc8fc\ud55c\ub300\uc0ac\ub97c \ube44\ub86f\ud55c \ub808\uc774\uac74 \ud589\uc815\ubd80, \uc138\uacc4 \uac01\uad6d \uc9c0\ub3c4\uc790\uc640 \uc778\uad8c\ub2e8\uccb4\ub4e4\uc774 \uad6c\uba85\uc6b4\ub3d9\uc5d0 \ub098\uc11c\uac8c \ub41c\ub2e4. \ub808\uc774\uac74 \uc815\uad8c\uc758 \ub9ac\ucc98\ub4dc \uc568\ub7f0 \uc548\ubcf4\ubcf4\uc88c\uad00\uc740 \uc720\ubcd1\ud604 \ud569\ucc39\uc758\uc7a5\uacfc \ub9cc\ub098 \uae40\ub300\uc911\uc744 \uc0ac\ud615\ub420 \uacbd\uc6b0 \uc591\uad6d\uad00\uacc4\ub294 \ud68c\ubcf5\ub418\uae30 \uc5b4\ub824\uc6b4 \uc0c1\ucc98\ub97c \uc785\uac8c \ub420 \uac83\uc774\ub77c\uba74\uc11c \uc555\ubc15\uc744 \ud558\uac8c \ub41c\ub2e4.", "answer": "1980\ub144, 12\u00b712", "sentence": "1980\ub144, 12\u00b712 \uad70\uc0ac \ubc18\ub780\uc73c\ub85c \uc2e4\uad8c\uc744 \uc7a5\uc545\ud55c \uc2e0\uad70\ubd80\ub294 \uc9d1\uad8c \uc2dc\ub098\ub9ac\uc624\uc5d0 \ub530\ub77c 80\ub144 5\uc6d4 17\uc77c \ube44\uc0c1\uacc4\uc5c4\uc744 \uc804\uad6d\uc73c\ub85c \ud655\ub300\ud558\uba74\uc11c \uc815\uce58\ud65c\ub3d9 \uae08\uc9c0\ub97c \uc8fc\uc694\ub0b4\uc6a9\uc73c\ub85c \ud55c \ud3ec\uace0\ub83910\ud638\ub97c \ubc1c\ud45c\ud568\uacfc \ud568\uaed8 \uae40\ub300\uc911\uc744 \ud3ec\ud568\ud55c \uc7ac\uc57c\uc778\uc0ac 20\uc5ec\uba85\uc744 \uc0ac\ud68c\ud63c\ub780 \ubc0f \ud559\uc0dd, \ub178\uc870 \ubc30\ud6c4\uc870\uc885 \ud610\uc758\ub85c \uc804\uaca9 \uc5f0\ud589\ud558\uc600\ub2e4.", "paragraph_sentence": " 1980\ub144, 12\u00b712 \uad70\uc0ac \ubc18\ub780\uc73c\ub85c \uc2e4\uad8c\uc744 \uc7a5\uc545\ud55c \uc2e0\uad70\ubd80\ub294 \uc9d1\uad8c \uc2dc\ub098\ub9ac\uc624\uc5d0 \ub530\ub77c 80\ub144 5\uc6d4 17\uc77c \ube44\uc0c1\uacc4\uc5c4\uc744 \uc804\uad6d\uc73c\ub85c \ud655\ub300\ud558\uba74\uc11c \uc815\uce58\ud65c\ub3d9 \uae08\uc9c0\ub97c \uc8fc\uc694\ub0b4\uc6a9\uc73c\ub85c \ud55c \ud3ec\uace0\ub83910\ud638\ub97c \ubc1c\ud45c\ud568\uacfc \ud568\uaed8 \uae40\ub300\uc911\uc744 \ud3ec\ud568\ud55c \uc7ac\uc57c\uc778\uc0ac 20\uc5ec\uba85\uc744 \uc0ac\ud68c\ud63c\ub780 \ubc0f \ud559\uc0dd, \ub178\uc870 \ubc30\ud6c4\uc870\uc885 \ud610\uc758\ub85c \uc804\uaca9 \uc5f0\ud589\ud558\uc600\ub2e4. \uae40\ub300\uc911\uc740 \uc2e0\uad70\ubd80\uac00 \uc870\uc791\ud55c \uae40\ub300\uc911 \ub0b4\ub780\uc74c\ubaa8\uc0ac\uac74\uc73c\ub85c \uad70\uc0ac\uc7ac\ud310\uc5d0\uc11c \uc0ac\ud615\uc744 \uc120\uace0\ubc1b\uc558\ub2e4. \ub2f9\uc2dc \uae40\ub300\uc911\uc740 \"\uc774 \ub545\uc758 \ubbfc\uc8fc\uc8fc\uc758\uac00 \ud68c\ubcf5\ub418\uba74 \uba3c\uc800 \uc8fd\uc5b4\uac04 \ub098\ub97c \uc704\ud574\uc11c \uc815\uce58\ubcf4\ubcf5\uc774 \ub2e4\uc2dc\ub294 \ud589\ud574\uc9c0\uc9c0 \uc54a\ub3c4\ub85d \ud574\ub2ec\ub77c\"\ub77c\uace0 \ud55c \ubc95\uc815 \ucd5c\ud6c4 \uc9c4\uc220\uc774 \uad6d\uc81c\uc0ac\ud68c\uc5d0 \uc54c\ub824\uc9c0\uba74\uc11c \ud070 \ubc18\ud5a5\uc744 \ubd88\ub7ec\uc77c\uc73c\ud0a4\uace0 \uc9c0\ubbf8 \uce74\ud130 \uc804 \ubbf8\uad6d \ub300\ud1b5\ub839, \ub3c4\ub110\ub4dc \uadf8\ub808\uadf8 \uc804 \uc8fc\ud55c\ub300\uc0ac\ub97c \ube44\ub86f\ud55c \ub808\uc774\uac74 \ud589\uc815\ubd80, \uc138\uacc4 \uac01\uad6d \uc9c0\ub3c4\uc790\uc640 \uc778\uad8c\ub2e8\uccb4\ub4e4\uc774 \uad6c\uba85\uc6b4\ub3d9\uc5d0 \ub098\uc11c\uac8c \ub41c\ub2e4. \ub808\uc774\uac74 \uc815\uad8c\uc758 \ub9ac\ucc98\ub4dc \uc568\ub7f0 \uc548\ubcf4\ubcf4\uc88c\uad00\uc740 \uc720\ubcd1\ud604 \ud569\ucc39\uc758\uc7a5\uacfc \ub9cc\ub098 \uae40\ub300\uc911\uc744 \uc0ac\ud615\ub420 \uacbd\uc6b0 \uc591\uad6d\uad00\uacc4\ub294 \ud68c\ubcf5\ub418\uae30 \uc5b4\ub824\uc6b4 \uc0c1\ucc98\ub97c \uc785\uac8c \ub420 \uac83\uc774\ub77c\uba74\uc11c \uc555\ubc15\uc744 \ud558\uac8c \ub41c\ub2e4.", "paragraph_answer": " 1980\ub144, 12\u00b712 \uad70\uc0ac \ubc18\ub780\uc73c\ub85c \uc2e4\uad8c\uc744 \uc7a5\uc545\ud55c \uc2e0\uad70\ubd80\ub294 \uc9d1\uad8c \uc2dc\ub098\ub9ac\uc624\uc5d0 \ub530\ub77c 80\ub144 5\uc6d4 17\uc77c \ube44\uc0c1\uacc4\uc5c4\uc744 \uc804\uad6d\uc73c\ub85c \ud655\ub300\ud558\uba74\uc11c \uc815\uce58\ud65c\ub3d9 \uae08\uc9c0\ub97c \uc8fc\uc694\ub0b4\uc6a9\uc73c\ub85c \ud55c \ud3ec\uace0\ub83910\ud638\ub97c \ubc1c\ud45c\ud568\uacfc \ud568\uaed8 \uae40\ub300\uc911\uc744 \ud3ec\ud568\ud55c \uc7ac\uc57c\uc778\uc0ac 20\uc5ec\uba85\uc744 \uc0ac\ud68c\ud63c\ub780 \ubc0f \ud559\uc0dd, \ub178\uc870 \ubc30\ud6c4\uc870\uc885 \ud610\uc758\ub85c \uc804\uaca9 \uc5f0\ud589\ud558\uc600\ub2e4. \uae40\ub300\uc911\uc740 \uc2e0\uad70\ubd80\uac00 \uc870\uc791\ud55c \uae40\ub300\uc911 \ub0b4\ub780\uc74c\ubaa8\uc0ac\uac74\uc73c\ub85c \uad70\uc0ac\uc7ac\ud310\uc5d0\uc11c \uc0ac\ud615\uc744 \uc120\uace0\ubc1b\uc558\ub2e4. \ub2f9\uc2dc \uae40\ub300\uc911\uc740 \"\uc774 \ub545\uc758 \ubbfc\uc8fc\uc8fc\uc758\uac00 \ud68c\ubcf5\ub418\uba74 \uba3c\uc800 \uc8fd\uc5b4\uac04 \ub098\ub97c \uc704\ud574\uc11c \uc815\uce58\ubcf4\ubcf5\uc774 \ub2e4\uc2dc\ub294 \ud589\ud574\uc9c0\uc9c0 \uc54a\ub3c4\ub85d \ud574\ub2ec\ub77c\"\ub77c\uace0 \ud55c \ubc95\uc815 \ucd5c\ud6c4 \uc9c4\uc220\uc774 \uad6d\uc81c\uc0ac\ud68c\uc5d0 \uc54c\ub824\uc9c0\uba74\uc11c \ud070 \ubc18\ud5a5\uc744 \ubd88\ub7ec\uc77c\uc73c\ud0a4\uace0 \uc9c0\ubbf8 \uce74\ud130 \uc804 \ubbf8\uad6d \ub300\ud1b5\ub839, \ub3c4\ub110\ub4dc \uadf8\ub808\uadf8 \uc804 \uc8fc\ud55c\ub300\uc0ac\ub97c \ube44\ub86f\ud55c \ub808\uc774\uac74 \ud589\uc815\ubd80, \uc138\uacc4 \uac01\uad6d \uc9c0\ub3c4\uc790\uc640 \uc778\uad8c\ub2e8\uccb4\ub4e4\uc774 \uad6c\uba85\uc6b4\ub3d9\uc5d0 \ub098\uc11c\uac8c \ub41c\ub2e4. \ub808\uc774\uac74 \uc815\uad8c\uc758 \ub9ac\ucc98\ub4dc \uc568\ub7f0 \uc548\ubcf4\ubcf4\uc88c\uad00\uc740 \uc720\ubcd1\ud604 \ud569\ucc39\uc758\uc7a5\uacfc \ub9cc\ub098 \uae40\ub300\uc911\uc744 \uc0ac\ud615\ub420 \uacbd\uc6b0 \uc591\uad6d\uad00\uacc4\ub294 \ud68c\ubcf5\ub418\uae30 \uc5b4\ub824\uc6b4 \uc0c1\ucc98\ub97c \uc785\uac8c \ub420 \uac83\uc774\ub77c\uba74\uc11c \uc555\ubc15\uc744 \ud558\uac8c \ub41c\ub2e4.", "sentence_answer": " 1980\ub144, 12\u00b712 \uad70\uc0ac \ubc18\ub780\uc73c\ub85c \uc2e4\uad8c\uc744 \uc7a5\uc545\ud55c \uc2e0\uad70\ubd80\ub294 \uc9d1\uad8c \uc2dc\ub098\ub9ac\uc624\uc5d0 \ub530\ub77c 80\ub144 5\uc6d4 17\uc77c \ube44\uc0c1\uacc4\uc5c4\uc744 \uc804\uad6d\uc73c\ub85c \ud655\ub300\ud558\uba74\uc11c \uc815\uce58\ud65c\ub3d9 \uae08\uc9c0\ub97c \uc8fc\uc694\ub0b4\uc6a9\uc73c\ub85c \ud55c \ud3ec\uace0\ub83910\ud638\ub97c \ubc1c\ud45c\ud568\uacfc \ud568\uaed8 \uae40\ub300\uc911\uc744 \ud3ec\ud568\ud55c \uc7ac\uc57c\uc778\uc0ac 20\uc5ec\uba85\uc744 \uc0ac\ud68c\ud63c\ub780 \ubc0f \ud559\uc0dd, \ub178\uc870 \ubc30\ud6c4\uc870\uc885 \ud610\uc758\ub85c \uc804\uaca9 \uc5f0\ud589\ud558\uc600\ub2e4.", "paragraph_id": "6656766-16-0", "paragraph_question": "question: 12.12 \uad70\uc0ac\ubc18\ub780\uc73c\ub85c \uc2e0\uad70\ubd80\uac00 \uc2e4\uad8c\uc744 \uc7a5\uc545\ud55c \uc5f0\ub3c4\ub294?, context: 1980\ub144, 12\u00b712 \uad70\uc0ac \ubc18\ub780\uc73c\ub85c \uc2e4\uad8c\uc744 \uc7a5\uc545\ud55c \uc2e0\uad70\ubd80\ub294 \uc9d1\uad8c \uc2dc\ub098\ub9ac\uc624\uc5d0 \ub530\ub77c 80\ub144 5\uc6d4 17\uc77c \ube44\uc0c1\uacc4\uc5c4\uc744 \uc804\uad6d\uc73c\ub85c \ud655\ub300\ud558\uba74\uc11c \uc815\uce58\ud65c\ub3d9 \uae08\uc9c0\ub97c \uc8fc\uc694\ub0b4\uc6a9\uc73c\ub85c \ud55c \ud3ec\uace0\ub83910\ud638\ub97c \ubc1c\ud45c\ud568\uacfc \ud568\uaed8 \uae40\ub300\uc911\uc744 \ud3ec\ud568\ud55c \uc7ac\uc57c\uc778\uc0ac 20\uc5ec\uba85\uc744 \uc0ac\ud68c\ud63c\ub780 \ubc0f \ud559\uc0dd, \ub178\uc870 \ubc30\ud6c4\uc870\uc885 \ud610\uc758\ub85c \uc804\uaca9 \uc5f0\ud589\ud558\uc600\ub2e4. \uae40\ub300\uc911\uc740 \uc2e0\uad70\ubd80\uac00 \uc870\uc791\ud55c \uae40\ub300\uc911 \ub0b4\ub780\uc74c\ubaa8\uc0ac\uac74\uc73c\ub85c \uad70\uc0ac\uc7ac\ud310\uc5d0\uc11c \uc0ac\ud615\uc744 \uc120\uace0\ubc1b\uc558\ub2e4. \ub2f9\uc2dc \uae40\ub300\uc911\uc740 \"\uc774 \ub545\uc758 \ubbfc\uc8fc\uc8fc\uc758\uac00 \ud68c\ubcf5\ub418\uba74 \uba3c\uc800 \uc8fd\uc5b4\uac04 \ub098\ub97c \uc704\ud574\uc11c \uc815\uce58\ubcf4\ubcf5\uc774 \ub2e4\uc2dc\ub294 \ud589\ud574\uc9c0\uc9c0 \uc54a\ub3c4\ub85d \ud574\ub2ec\ub77c\"\ub77c\uace0 \ud55c \ubc95\uc815 \ucd5c\ud6c4 \uc9c4\uc220\uc774 \uad6d\uc81c\uc0ac\ud68c\uc5d0 \uc54c\ub824\uc9c0\uba74\uc11c \ud070 \ubc18\ud5a5\uc744 \ubd88\ub7ec\uc77c\uc73c\ud0a4\uace0 \uc9c0\ubbf8 \uce74\ud130 \uc804 \ubbf8\uad6d \ub300\ud1b5\ub839, \ub3c4\ub110\ub4dc \uadf8\ub808\uadf8 \uc804 \uc8fc\ud55c\ub300\uc0ac\ub97c \ube44\ub86f\ud55c \ub808\uc774\uac74 \ud589\uc815\ubd80, \uc138\uacc4 \uac01\uad6d \uc9c0\ub3c4\uc790\uc640 \uc778\uad8c\ub2e8\uccb4\ub4e4\uc774 \uad6c\uba85\uc6b4\ub3d9\uc5d0 \ub098\uc11c\uac8c \ub41c\ub2e4. \ub808\uc774\uac74 \uc815\uad8c\uc758 \ub9ac\ucc98\ub4dc \uc568\ub7f0 \uc548\ubcf4\ubcf4\uc88c\uad00\uc740 \uc720\ubcd1\ud604 \ud569\ucc39\uc758\uc7a5\uacfc \ub9cc\ub098 \uae40\ub300\uc911\uc744 \uc0ac\ud615\ub420 \uacbd\uc6b0 \uc591\uad6d\uad00\uacc4\ub294 \ud68c\ubcf5\ub418\uae30 \uc5b4\ub824\uc6b4 \uc0c1\ucc98\ub97c \uc785\uac8c \ub420 \uac83\uc774\ub77c\uba74\uc11c \uc555\ubc15\uc744 \ud558\uac8c \ub41c\ub2e4."} {"question": "\ub098\uce74\ubaa8\ub9ac \uc544\ud0a4\ub098\uac00 \ubc24\uc758 \ud788\ud2b8 \uc2a4\ud29c\ub514\uc624 DELUXE\uc5d0 \ucd9c\uc5f0\ud55c \ub0a0\uc9dc\ub294?", "paragraph": "\uc74c\ubc18\uc774 \ubc1c\ub9e4\ub418\uae30 \uc774\uc804, \uc774\ubbf8 \ubaa8\ub4e0 \ud6c4\ubc18 \uc791\uc5c5\uc744 \ub9c8\uce5c \ub4a4\uc778 1986\ub144 7\uc6d4 12\uc77c\uc5d0\ub294 \uc804\uad6d \ucf58\uc11c\ud2b8 \ud22c\uc5b4 LIGHT & SHADE\ub97c \uac1c\ucd5c\ud558\uc5ec \ubcf8 \uc74c\ubc18\uc758 \uace1\ub4e4\uc744 \ubbf8\ub9ac \ubd88\ub800\ub2e4. 1987\ub144 2\uc6d4 3\uc77c, \uadf8\ub140\uc758 \uc815\uc2dd \uc74c\ubc18\uc5d0 \uc218\ub85d\ub41c \uace1\uc73c\ub85c\uc11c\ub294 \uc774\ub840\uc801\uc73c\ub85c \ud6c4\uc9c0 \ud14c\ub808\ube44\uc758 \uc74c\uc545\ubc29\uc1a1 \ud504\ub85c\uadf8\ub7a8 \u300a\ubc24\uc758 \ud788\ud2b8 \uc2a4\ud29c\ub514\uc624 DELUXE\u300b\uc5d0\ub3c4 \ucd9c\uc5f0\ud558\uc5ec \u3008Back door night\u3009\uacfc \u3008\ub9c8\ub9ac\uc624\ub124\ud2b8\u3009\ub97c \uc120\ubcf4\uc600\ub2e4. \ub2f9\ucd08 \ub098\uce74\ubaa8\ub9ac \uc544\ud0a4\ub098\ub294 \uc774 \uc74c\ubc18\uc758 \uc774\ubbf8\uc9c0\ub300\ub85c \ubc29\uc1a1\uc744 \ud558\ub294\ub370 \ub09c\uc0c9\uc744 \ud45c\ud558\uba70 \ub450 \uace1\uc758 \ubc29\uc1a1\uc744 \uac70\uc808\ud558\uc600\uc73c\ub098, \ud6c4\uc9c0 \ud14c\ub808\ube44\uc758 \uad00\uacc4\uc790\ub4e4\uc758 \uac04\uace1\ud55c \uc694\uccad\uc5d0 \ub9c8\uc74c\uc744 \ubc14\uafb8\uc5b4 \ucd9c\uc5f0\uc744 \uc2b9\ub099\ud588\ub2e4. \uc774\uc5b4 \ubb34\ub300\ub97c \ucd5c\ub300\ud55c \uc74c\ubc18\uacfc \uace1\uc758 \uc774\ubbf8\uc9c0\uc5d0 \uc811\uadfc\ud558\uae30\ub97c \uc6d0\ud558\uc5ec \ubb34\ub300\uc758 \uc138\ud305\uacfc \uc870\uba85, \uadf8\ub9ac\uace0 \uce74\uba54\ub77c \uc6cc\ud0b9\uae4c\uc9c0\ub3c4 \uc790\uc2e0\uc774 \uc6d0\ud558\ub294 \ubc29\ud5a5\uc73c\ub85c \ud560 \uac83\uc744 \uace0\uc9d1\ud558\uc600\ub2e4. \uc774 \ubc29\uc1a1\uc5d0\uc11c \ub098\uce74\ubaa8\ub9ac\ub294 \ubb34\ub300 \uc758\uc0c1\uc744 \ub9c8\ub140\ucc98\ub7fc \uc785\uace0, \ubb34\ub300\ub3c4 \uac70\ubbf8\uc904\ub85c \ub4a4\ub36e\uc778 \uc74c\uc0b0\ud55c \ubd84\uc704\uae30\uc5d0\uc11c \uace1\ub4e4\uc744 \uc120\ubcf4\uc600\ub2e4.", "answer": "1987\ub144 2\uc6d4 3\uc77c", "sentence": "1987\ub144 2\uc6d4 3\uc77c ,", "paragraph_sentence": "\uc74c\ubc18\uc774 \ubc1c\ub9e4\ub418\uae30 \uc774\uc804, \uc774\ubbf8 \ubaa8\ub4e0 \ud6c4\ubc18 \uc791\uc5c5\uc744 \ub9c8\uce5c \ub4a4\uc778 1986\ub144 7\uc6d4 12\uc77c\uc5d0\ub294 \uc804\uad6d \ucf58\uc11c\ud2b8 \ud22c\uc5b4 LIGHT & SHADE\ub97c \uac1c\ucd5c\ud558\uc5ec \ubcf8 \uc74c\ubc18\uc758 \uace1\ub4e4\uc744 \ubbf8\ub9ac \ubd88\ub800\ub2e4. 1987\ub144 2\uc6d4 3\uc77c , \uadf8\ub140\uc758 \uc815\uc2dd \uc74c\ubc18\uc5d0 \uc218\ub85d\ub41c \uace1\uc73c\ub85c\uc11c\ub294 \uc774\ub840\uc801\uc73c\ub85c \ud6c4\uc9c0 \ud14c\ub808\ube44\uc758 \uc74c\uc545\ubc29\uc1a1 \ud504\ub85c\uadf8\ub7a8 \u300a\ubc24\uc758 \ud788\ud2b8 \uc2a4\ud29c\ub514\uc624 DELUXE\u300b\uc5d0\ub3c4 \ucd9c\uc5f0\ud558\uc5ec \u3008Back door night\u3009\uacfc \u3008\ub9c8\ub9ac\uc624\ub124\ud2b8\u3009\ub97c \uc120\ubcf4\uc600\ub2e4. \ub2f9\ucd08 \ub098\uce74\ubaa8\ub9ac \uc544\ud0a4\ub098\ub294 \uc774 \uc74c\ubc18\uc758 \uc774\ubbf8\uc9c0\ub300\ub85c \ubc29\uc1a1\uc744 \ud558\ub294\ub370 \ub09c\uc0c9\uc744 \ud45c\ud558\uba70 \ub450 \uace1\uc758 \ubc29\uc1a1\uc744 \uac70\uc808\ud558\uc600\uc73c\ub098, \ud6c4\uc9c0 \ud14c\ub808\ube44\uc758 \uad00\uacc4\uc790\ub4e4\uc758 \uac04\uace1\ud55c \uc694\uccad\uc5d0 \ub9c8\uc74c\uc744 \ubc14\uafb8\uc5b4 \ucd9c\uc5f0\uc744 \uc2b9\ub099\ud588\ub2e4. \uc774\uc5b4 \ubb34\ub300\ub97c \ucd5c\ub300\ud55c \uc74c\ubc18\uacfc \uace1\uc758 \uc774\ubbf8\uc9c0\uc5d0 \uc811\uadfc\ud558\uae30\ub97c \uc6d0\ud558\uc5ec \ubb34\ub300\uc758 \uc138\ud305\uacfc \uc870\uba85, \uadf8\ub9ac\uace0 \uce74\uba54\ub77c \uc6cc\ud0b9\uae4c\uc9c0\ub3c4 \uc790\uc2e0\uc774 \uc6d0\ud558\ub294 \ubc29\ud5a5\uc73c\ub85c \ud560 \uac83\uc744 \uace0\uc9d1\ud558\uc600\ub2e4. \uc774 \ubc29\uc1a1\uc5d0\uc11c \ub098\uce74\ubaa8\ub9ac\ub294 \ubb34\ub300 \uc758\uc0c1\uc744 \ub9c8\ub140\ucc98\ub7fc \uc785\uace0, \ubb34\ub300\ub3c4 \uac70\ubbf8\uc904\ub85c \ub4a4\ub36e\uc778 \uc74c\uc0b0\ud55c \ubd84\uc704\uae30\uc5d0\uc11c \uace1\ub4e4\uc744 \uc120\ubcf4\uc600\ub2e4.", "paragraph_answer": "\uc74c\ubc18\uc774 \ubc1c\ub9e4\ub418\uae30 \uc774\uc804, \uc774\ubbf8 \ubaa8\ub4e0 \ud6c4\ubc18 \uc791\uc5c5\uc744 \ub9c8\uce5c \ub4a4\uc778 1986\ub144 7\uc6d4 12\uc77c\uc5d0\ub294 \uc804\uad6d \ucf58\uc11c\ud2b8 \ud22c\uc5b4 LIGHT & SHADE\ub97c \uac1c\ucd5c\ud558\uc5ec \ubcf8 \uc74c\ubc18\uc758 \uace1\ub4e4\uc744 \ubbf8\ub9ac \ubd88\ub800\ub2e4. 1987\ub144 2\uc6d4 3\uc77c , \uadf8\ub140\uc758 \uc815\uc2dd \uc74c\ubc18\uc5d0 \uc218\ub85d\ub41c \uace1\uc73c\ub85c\uc11c\ub294 \uc774\ub840\uc801\uc73c\ub85c \ud6c4\uc9c0 \ud14c\ub808\ube44\uc758 \uc74c\uc545\ubc29\uc1a1 \ud504\ub85c\uadf8\ub7a8 \u300a\ubc24\uc758 \ud788\ud2b8 \uc2a4\ud29c\ub514\uc624 DELUXE\u300b\uc5d0\ub3c4 \ucd9c\uc5f0\ud558\uc5ec \u3008Back door night\u3009\uacfc \u3008\ub9c8\ub9ac\uc624\ub124\ud2b8\u3009\ub97c \uc120\ubcf4\uc600\ub2e4. \ub2f9\ucd08 \ub098\uce74\ubaa8\ub9ac \uc544\ud0a4\ub098\ub294 \uc774 \uc74c\ubc18\uc758 \uc774\ubbf8\uc9c0\ub300\ub85c \ubc29\uc1a1\uc744 \ud558\ub294\ub370 \ub09c\uc0c9\uc744 \ud45c\ud558\uba70 \ub450 \uace1\uc758 \ubc29\uc1a1\uc744 \uac70\uc808\ud558\uc600\uc73c\ub098, \ud6c4\uc9c0 \ud14c\ub808\ube44\uc758 \uad00\uacc4\uc790\ub4e4\uc758 \uac04\uace1\ud55c \uc694\uccad\uc5d0 \ub9c8\uc74c\uc744 \ubc14\uafb8\uc5b4 \ucd9c\uc5f0\uc744 \uc2b9\ub099\ud588\ub2e4. \uc774\uc5b4 \ubb34\ub300\ub97c \ucd5c\ub300\ud55c \uc74c\ubc18\uacfc \uace1\uc758 \uc774\ubbf8\uc9c0\uc5d0 \uc811\uadfc\ud558\uae30\ub97c \uc6d0\ud558\uc5ec \ubb34\ub300\uc758 \uc138\ud305\uacfc \uc870\uba85, \uadf8\ub9ac\uace0 \uce74\uba54\ub77c \uc6cc\ud0b9\uae4c\uc9c0\ub3c4 \uc790\uc2e0\uc774 \uc6d0\ud558\ub294 \ubc29\ud5a5\uc73c\ub85c \ud560 \uac83\uc744 \uace0\uc9d1\ud558\uc600\ub2e4. \uc774 \ubc29\uc1a1\uc5d0\uc11c \ub098\uce74\ubaa8\ub9ac\ub294 \ubb34\ub300 \uc758\uc0c1\uc744 \ub9c8\ub140\ucc98\ub7fc \uc785\uace0, \ubb34\ub300\ub3c4 \uac70\ubbf8\uc904\ub85c \ub4a4\ub36e\uc778 \uc74c\uc0b0\ud55c \ubd84\uc704\uae30\uc5d0\uc11c \uace1\ub4e4\uc744 \uc120\ubcf4\uc600\ub2e4.", "sentence_answer": " 1987\ub144 2\uc6d4 3\uc77c ,", "paragraph_id": "6509185-3-1", "paragraph_question": "question: \ub098\uce74\ubaa8\ub9ac \uc544\ud0a4\ub098\uac00 \ubc24\uc758 \ud788\ud2b8 \uc2a4\ud29c\ub514\uc624 DELUXE\uc5d0 \ucd9c\uc5f0\ud55c \ub0a0\uc9dc\ub294?, context: \uc74c\ubc18\uc774 \ubc1c\ub9e4\ub418\uae30 \uc774\uc804, \uc774\ubbf8 \ubaa8\ub4e0 \ud6c4\ubc18 \uc791\uc5c5\uc744 \ub9c8\uce5c \ub4a4\uc778 1986\ub144 7\uc6d4 12\uc77c\uc5d0\ub294 \uc804\uad6d \ucf58\uc11c\ud2b8 \ud22c\uc5b4 LIGHT & SHADE\ub97c \uac1c\ucd5c\ud558\uc5ec \ubcf8 \uc74c\ubc18\uc758 \uace1\ub4e4\uc744 \ubbf8\ub9ac \ubd88\ub800\ub2e4. 1987\ub144 2\uc6d4 3\uc77c, \uadf8\ub140\uc758 \uc815\uc2dd \uc74c\ubc18\uc5d0 \uc218\ub85d\ub41c \uace1\uc73c\ub85c\uc11c\ub294 \uc774\ub840\uc801\uc73c\ub85c \ud6c4\uc9c0 \ud14c\ub808\ube44\uc758 \uc74c\uc545\ubc29\uc1a1 \ud504\ub85c\uadf8\ub7a8 \u300a\ubc24\uc758 \ud788\ud2b8 \uc2a4\ud29c\ub514\uc624 DELUXE\u300b\uc5d0\ub3c4 \ucd9c\uc5f0\ud558\uc5ec \u3008Back door night\u3009\uacfc \u3008\ub9c8\ub9ac\uc624\ub124\ud2b8\u3009\ub97c \uc120\ubcf4\uc600\ub2e4. \ub2f9\ucd08 \ub098\uce74\ubaa8\ub9ac \uc544\ud0a4\ub098\ub294 \uc774 \uc74c\ubc18\uc758 \uc774\ubbf8\uc9c0\ub300\ub85c \ubc29\uc1a1\uc744 \ud558\ub294\ub370 \ub09c\uc0c9\uc744 \ud45c\ud558\uba70 \ub450 \uace1\uc758 \ubc29\uc1a1\uc744 \uac70\uc808\ud558\uc600\uc73c\ub098, \ud6c4\uc9c0 \ud14c\ub808\ube44\uc758 \uad00\uacc4\uc790\ub4e4\uc758 \uac04\uace1\ud55c \uc694\uccad\uc5d0 \ub9c8\uc74c\uc744 \ubc14\uafb8\uc5b4 \ucd9c\uc5f0\uc744 \uc2b9\ub099\ud588\ub2e4. \uc774\uc5b4 \ubb34\ub300\ub97c \ucd5c\ub300\ud55c \uc74c\ubc18\uacfc \uace1\uc758 \uc774\ubbf8\uc9c0\uc5d0 \uc811\uadfc\ud558\uae30\ub97c \uc6d0\ud558\uc5ec \ubb34\ub300\uc758 \uc138\ud305\uacfc \uc870\uba85, \uadf8\ub9ac\uace0 \uce74\uba54\ub77c \uc6cc\ud0b9\uae4c\uc9c0\ub3c4 \uc790\uc2e0\uc774 \uc6d0\ud558\ub294 \ubc29\ud5a5\uc73c\ub85c \ud560 \uac83\uc744 \uace0\uc9d1\ud558\uc600\ub2e4. \uc774 \ubc29\uc1a1\uc5d0\uc11c \ub098\uce74\ubaa8\ub9ac\ub294 \ubb34\ub300 \uc758\uc0c1\uc744 \ub9c8\ub140\ucc98\ub7fc \uc785\uace0, \ubb34\ub300\ub3c4 \uac70\ubbf8\uc904\ub85c \ub4a4\ub36e\uc778 \uc74c\uc0b0\ud55c \ubd84\uc704\uae30\uc5d0\uc11c \uace1\ub4e4\uc744 \uc120\ubcf4\uc600\ub2e4."} {"question": "1998\ub144 \uad00\uce21 \uc774\ub798, \ud56d\uc131\ub4e4 \uc911 \ud558\ub098(S2)\uac00 \uba87\ud68c \uacf5\uc804\uc744 \uc644\ub8cc\ud588\ub098\uc694?", "paragraph": "\uc6b0\ub9ac\uc740\ud558 \uc911\uc2ec \uadfc\ucc98\uc758 \ud56d\uc131\ub4e4\uc758 \uace0\uc720\uc6b4\ub3d9\uc740 \uadf8 \ud56d\uc131\ub4e4\uc774 \ucd08\ub300\uc9c8\ub7c9 \ube14\ub799\ud640\uc744 \uacf5\uc804\ud558\uace0 \uc788\uc744 \uac83\uc774\ub77c\ub294 \uac15\ud55c \uad00\uce21\uc801 \uc99d\uac70\ub97c \uc81c\uacf5\ud55c\ub2e4. 1995\ub144 \uc774\ub798\ub85c \ucc9c\ubb38\ud559\uc790\ub4e4\uc740 \uc804\ud30c\uc6d0 \uad81\uc218\uc790\ub9ac A*\uc640 \uac19\uc740 \uc704\uce58\uc758 \ubcf4\uc774\uc9c0 \uc54a\ub294 \ucc9c\uccb4\ub97c \uacf5\uc804\ud558\ub294 90\uac1c\uc758 \ubcc4\ub4e4\uc758 \uc6b4\ub3d9\uc744 \ucd94\uc801\ud574\uc654\uc73c\uba70, \uadf8 \uc6b4\ub3d9\uc744 \ucf00\ud50c\ub7ec \uada4\ub3c4\uc5d0 \ub300\uc785\ud568\uc73c\ub85c\uc368 1998\ub144, \uc774 \ud56d\uc131\ub4e4\uc758 \uc6b4\ub3d9\uc744 \uc720\ubc1c\uc2dc\ud0a4\uae30 \uc704\ud574\uc11c\ub294 \ubc18\uacbd 0.02 \uad11\ub144\uc758 \ubd80\ud53c \uc18d\uc5d0 \uc9c8\ub7c9 2\ubc31 6\uc2ed\ub9cc M\u2609\uc774 \ub4e4\uc5b4 \uc788\uc5b4\uc57c \ud55c\ub2e4\ub294 \uacb0\ub860\uc744 \uc5bb\uc5b4\ub0c8\ub2e4. \uadf8 \uc774\ub798\ub85c \ud56d\uc131\ub4e4 \uc911 \ud558\ub098(S2)\uac00 1\ud68c \uacf5\uc804\uc744 \uc644\ub8cc\ud588\ub2e4. \ucc9c\ubb38\ud559\uc790\ub4e4\uc740 \uada4\ub3c4 \ub370\uc774\ud130\ub85c\ubd80\ud130 \uc7ac\uacc4\uc0b0\ud558\uc5ec \ud56d\uc131\ub4e4\uc758 \uc6b4\ub3d9\uc744 \uc720\ubc1c\uc2dc\ud0a4\uae30 \uc704\ud574\uc11c\ub294 \ubc18\uacbd 0.002 \uad11\ub144 \uc758 \ubd80\ud53c \uc18d\uc5d0 \uc9c8\ub7c9 4\ubc31 3\uc2ed\ub9cc M\u2609\uc774 \ub4e4\uc5b4 \uc788\uc5b4\uc57c \ud55c\ub2e4\uace0 \uacb0\ub860\ub0b4\ub838\ub2e4. \ud06c\uae30 \uc0c1\ud55c\uc774 \uc5ec\uc804\ud788 \ub108\ubb34 \ucee4\uc11c \uc774\uac83\uc774 \uc288\ubc14\ub974\uce20\uc2e4\ud2b8 \ubc18\uacbd \uc774\ud558\uc778\uc9c0 \uc54c\uc544\ub0b4\ub294 \ub370 \uc5b4\ub824\uc6c0\uc774 \uc788\ub2e4. \uadf8\ub7fc\uc5d0\ub3c4 \ubd88\uad6c\ud558\uace0 \uad00\uce21 \uacb0\uacfc\ub294 \uc911\uc2ec\uc758 \ucc9c\uccb4\uac00 \ucd08\ub300\uc9c8\ub7c9 \ube14\ub799\ud640\uc784\uc744 \uac15\ud558\uac8c \uc2dc\uc0ac\ud558\uace0 \uc788\ub2e4. \uadf8 \uc774\uc678\uc5d0 \uc774\ub807\uac8c \uc791\uc740 \ubd80\ud53c \uc18d\uc5d0 \uc5c4\uccad\ub09c \uc591\uc758 \ubcf4\uc774\uc9c0 \uc54a\ub294 \uc9c8\ub7c9\uc774 \uc874\uc7ac\ud558\ub294 \uacbd\uc6b0\uc5d0 \ub300\ud55c \ub2e4\ub978 \ud0c0\ub2f9\ud55c \uc124\uba85\uc744 \uc81c\uacf5\ud560 \uc218 \uc5c6\ub2e4. \ub610\ud55c \uc774 \ucc9c\uccb4\uac00 \ube14\ub799\ud640 \ud2b9\uc720\uc758 \ud2b9\uc9d5\uc778 \uc0ac\uac74\uc758 \uc9c0\ud3c9\uc120\uc744 \uac00\uc9c0\uace0 \uc788\ub2e4\ub294 \uad00\uce21\uc801 \uc99d\uac70\ub3c4 \uc874\uc7ac\ud55c\ub2e4.", "answer": "1\ud68c", "sentence": "\uadf8 \uc774\ub798\ub85c \ud56d\uc131\ub4e4 \uc911 \ud558\ub098(S2)\uac00 1\ud68c \uacf5\uc804\uc744 \uc644\ub8cc\ud588\ub2e4.", "paragraph_sentence": "\uc6b0\ub9ac\uc740\ud558 \uc911\uc2ec \uadfc\ucc98\uc758 \ud56d\uc131\ub4e4\uc758 \uace0\uc720\uc6b4\ub3d9\uc740 \uadf8 \ud56d\uc131\ub4e4\uc774 \ucd08\ub300\uc9c8\ub7c9 \ube14\ub799\ud640\uc744 \uacf5\uc804\ud558\uace0 \uc788\uc744 \uac83\uc774\ub77c\ub294 \uac15\ud55c \uad00\uce21\uc801 \uc99d\uac70\ub97c \uc81c\uacf5\ud55c\ub2e4. 1995\ub144 \uc774\ub798\ub85c \ucc9c\ubb38\ud559\uc790\ub4e4\uc740 \uc804\ud30c\uc6d0 \uad81\uc218\uc790\ub9ac A*\uc640 \uac19\uc740 \uc704\uce58\uc758 \ubcf4\uc774\uc9c0 \uc54a\ub294 \ucc9c\uccb4\ub97c \uacf5\uc804\ud558\ub294 90\uac1c\uc758 \ubcc4\ub4e4\uc758 \uc6b4\ub3d9\uc744 \ucd94\uc801\ud574\uc654\uc73c\uba70, \uadf8 \uc6b4\ub3d9\uc744 \ucf00\ud50c\ub7ec \uada4\ub3c4\uc5d0 \ub300\uc785\ud568\uc73c\ub85c\uc368 1998\ub144, \uc774 \ud56d\uc131\ub4e4\uc758 \uc6b4\ub3d9\uc744 \uc720\ubc1c\uc2dc\ud0a4\uae30 \uc704\ud574\uc11c\ub294 \ubc18\uacbd 0.02 \uad11\ub144\uc758 \ubd80\ud53c \uc18d\uc5d0 \uc9c8\ub7c9 2\ubc31 6\uc2ed\ub9cc M\u2609\uc774 \ub4e4\uc5b4 \uc788\uc5b4\uc57c \ud55c\ub2e4\ub294 \uacb0\ub860\uc744 \uc5bb\uc5b4\ub0c8\ub2e4. \uadf8 \uc774\ub798\ub85c \ud56d\uc131\ub4e4 \uc911 \ud558\ub098(S2)\uac00 1\ud68c \uacf5\uc804\uc744 \uc644\ub8cc\ud588\ub2e4. \ucc9c\ubb38\ud559\uc790\ub4e4\uc740 \uada4\ub3c4 \ub370\uc774\ud130\ub85c\ubd80\ud130 \uc7ac\uacc4\uc0b0\ud558\uc5ec \ud56d\uc131\ub4e4\uc758 \uc6b4\ub3d9\uc744 \uc720\ubc1c\uc2dc\ud0a4\uae30 \uc704\ud574\uc11c\ub294 \ubc18\uacbd 0.002 \uad11\ub144 \uc758 \ubd80\ud53c \uc18d\uc5d0 \uc9c8\ub7c9 4\ubc31 3\uc2ed\ub9cc M\u2609\uc774 \ub4e4\uc5b4 \uc788\uc5b4\uc57c \ud55c\ub2e4\uace0 \uacb0\ub860\ub0b4\ub838\ub2e4. \ud06c\uae30 \uc0c1\ud55c\uc774 \uc5ec\uc804\ud788 \ub108\ubb34 \ucee4\uc11c \uc774\uac83\uc774 \uc288\ubc14\ub974\uce20\uc2e4\ud2b8 \ubc18\uacbd \uc774\ud558\uc778\uc9c0 \uc54c\uc544\ub0b4\ub294 \ub370 \uc5b4\ub824\uc6c0\uc774 \uc788\ub2e4. \uadf8\ub7fc\uc5d0\ub3c4 \ubd88\uad6c\ud558\uace0 \uad00\uce21 \uacb0\uacfc\ub294 \uc911\uc2ec\uc758 \ucc9c\uccb4\uac00 \ucd08\ub300\uc9c8\ub7c9 \ube14\ub799\ud640\uc784\uc744 \uac15\ud558\uac8c \uc2dc\uc0ac\ud558\uace0 \uc788\ub2e4. \uadf8 \uc774\uc678\uc5d0 \uc774\ub807\uac8c \uc791\uc740 \ubd80\ud53c \uc18d\uc5d0 \uc5c4\uccad\ub09c \uc591\uc758 \ubcf4\uc774\uc9c0 \uc54a\ub294 \uc9c8\ub7c9\uc774 \uc874\uc7ac\ud558\ub294 \uacbd\uc6b0\uc5d0 \ub300\ud55c \ub2e4\ub978 \ud0c0\ub2f9\ud55c \uc124\uba85\uc744 \uc81c\uacf5\ud560 \uc218 \uc5c6\ub2e4. \ub610\ud55c \uc774 \ucc9c\uccb4\uac00 \ube14\ub799\ud640 \ud2b9\uc720\uc758 \ud2b9\uc9d5\uc778 \uc0ac\uac74\uc758 \uc9c0\ud3c9\uc120\uc744 \uac00\uc9c0\uace0 \uc788\ub2e4\ub294 \uad00\uce21\uc801 \uc99d\uac70\ub3c4 \uc874\uc7ac\ud55c\ub2e4.", "paragraph_answer": "\uc6b0\ub9ac\uc740\ud558 \uc911\uc2ec \uadfc\ucc98\uc758 \ud56d\uc131\ub4e4\uc758 \uace0\uc720\uc6b4\ub3d9\uc740 \uadf8 \ud56d\uc131\ub4e4\uc774 \ucd08\ub300\uc9c8\ub7c9 \ube14\ub799\ud640\uc744 \uacf5\uc804\ud558\uace0 \uc788\uc744 \uac83\uc774\ub77c\ub294 \uac15\ud55c \uad00\uce21\uc801 \uc99d\uac70\ub97c \uc81c\uacf5\ud55c\ub2e4. 1995\ub144 \uc774\ub798\ub85c \ucc9c\ubb38\ud559\uc790\ub4e4\uc740 \uc804\ud30c\uc6d0 \uad81\uc218\uc790\ub9ac A*\uc640 \uac19\uc740 \uc704\uce58\uc758 \ubcf4\uc774\uc9c0 \uc54a\ub294 \ucc9c\uccb4\ub97c \uacf5\uc804\ud558\ub294 90\uac1c\uc758 \ubcc4\ub4e4\uc758 \uc6b4\ub3d9\uc744 \ucd94\uc801\ud574\uc654\uc73c\uba70, \uadf8 \uc6b4\ub3d9\uc744 \ucf00\ud50c\ub7ec \uada4\ub3c4\uc5d0 \ub300\uc785\ud568\uc73c\ub85c\uc368 1998\ub144, \uc774 \ud56d\uc131\ub4e4\uc758 \uc6b4\ub3d9\uc744 \uc720\ubc1c\uc2dc\ud0a4\uae30 \uc704\ud574\uc11c\ub294 \ubc18\uacbd 0.02 \uad11\ub144\uc758 \ubd80\ud53c \uc18d\uc5d0 \uc9c8\ub7c9 2\ubc31 6\uc2ed\ub9cc M\u2609\uc774 \ub4e4\uc5b4 \uc788\uc5b4\uc57c \ud55c\ub2e4\ub294 \uacb0\ub860\uc744 \uc5bb\uc5b4\ub0c8\ub2e4. \uadf8 \uc774\ub798\ub85c \ud56d\uc131\ub4e4 \uc911 \ud558\ub098(S2)\uac00 1\ud68c \uacf5\uc804\uc744 \uc644\ub8cc\ud588\ub2e4. \ucc9c\ubb38\ud559\uc790\ub4e4\uc740 \uada4\ub3c4 \ub370\uc774\ud130\ub85c\ubd80\ud130 \uc7ac\uacc4\uc0b0\ud558\uc5ec \ud56d\uc131\ub4e4\uc758 \uc6b4\ub3d9\uc744 \uc720\ubc1c\uc2dc\ud0a4\uae30 \uc704\ud574\uc11c\ub294 \ubc18\uacbd 0.002 \uad11\ub144 \uc758 \ubd80\ud53c \uc18d\uc5d0 \uc9c8\ub7c9 4\ubc31 3\uc2ed\ub9cc M\u2609\uc774 \ub4e4\uc5b4 \uc788\uc5b4\uc57c \ud55c\ub2e4\uace0 \uacb0\ub860\ub0b4\ub838\ub2e4. \ud06c\uae30 \uc0c1\ud55c\uc774 \uc5ec\uc804\ud788 \ub108\ubb34 \ucee4\uc11c \uc774\uac83\uc774 \uc288\ubc14\ub974\uce20\uc2e4\ud2b8 \ubc18\uacbd \uc774\ud558\uc778\uc9c0 \uc54c\uc544\ub0b4\ub294 \ub370 \uc5b4\ub824\uc6c0\uc774 \uc788\ub2e4. \uadf8\ub7fc\uc5d0\ub3c4 \ubd88\uad6c\ud558\uace0 \uad00\uce21 \uacb0\uacfc\ub294 \uc911\uc2ec\uc758 \ucc9c\uccb4\uac00 \ucd08\ub300\uc9c8\ub7c9 \ube14\ub799\ud640\uc784\uc744 \uac15\ud558\uac8c \uc2dc\uc0ac\ud558\uace0 \uc788\ub2e4. \uadf8 \uc774\uc678\uc5d0 \uc774\ub807\uac8c \uc791\uc740 \ubd80\ud53c \uc18d\uc5d0 \uc5c4\uccad\ub09c \uc591\uc758 \ubcf4\uc774\uc9c0 \uc54a\ub294 \uc9c8\ub7c9\uc774 \uc874\uc7ac\ud558\ub294 \uacbd\uc6b0\uc5d0 \ub300\ud55c \ub2e4\ub978 \ud0c0\ub2f9\ud55c \uc124\uba85\uc744 \uc81c\uacf5\ud560 \uc218 \uc5c6\ub2e4. \ub610\ud55c \uc774 \ucc9c\uccb4\uac00 \ube14\ub799\ud640 \ud2b9\uc720\uc758 \ud2b9\uc9d5\uc778 \uc0ac\uac74\uc758 \uc9c0\ud3c9\uc120\uc744 \uac00\uc9c0\uace0 \uc788\ub2e4\ub294 \uad00\uce21\uc801 \uc99d\uac70\ub3c4 \uc874\uc7ac\ud55c\ub2e4.", "sentence_answer": "\uadf8 \uc774\ub798\ub85c \ud56d\uc131\ub4e4 \uc911 \ud558\ub098(S2)\uac00 1\ud68c \uacf5\uc804\uc744 \uc644\ub8cc\ud588\ub2e4.", "paragraph_id": "6583541-13-1", "paragraph_question": "question: 1998\ub144 \uad00\uce21 \uc774\ub798, \ud56d\uc131\ub4e4 \uc911 \ud558\ub098(S2)\uac00 \uba87\ud68c \uacf5\uc804\uc744 \uc644\ub8cc\ud588\ub098\uc694?, context: \uc6b0\ub9ac\uc740\ud558 \uc911\uc2ec \uadfc\ucc98\uc758 \ud56d\uc131\ub4e4\uc758 \uace0\uc720\uc6b4\ub3d9\uc740 \uadf8 \ud56d\uc131\ub4e4\uc774 \ucd08\ub300\uc9c8\ub7c9 \ube14\ub799\ud640\uc744 \uacf5\uc804\ud558\uace0 \uc788\uc744 \uac83\uc774\ub77c\ub294 \uac15\ud55c \uad00\uce21\uc801 \uc99d\uac70\ub97c \uc81c\uacf5\ud55c\ub2e4. 1995\ub144 \uc774\ub798\ub85c \ucc9c\ubb38\ud559\uc790\ub4e4\uc740 \uc804\ud30c\uc6d0 \uad81\uc218\uc790\ub9ac A*\uc640 \uac19\uc740 \uc704\uce58\uc758 \ubcf4\uc774\uc9c0 \uc54a\ub294 \ucc9c\uccb4\ub97c \uacf5\uc804\ud558\ub294 90\uac1c\uc758 \ubcc4\ub4e4\uc758 \uc6b4\ub3d9\uc744 \ucd94\uc801\ud574\uc654\uc73c\uba70, \uadf8 \uc6b4\ub3d9\uc744 \ucf00\ud50c\ub7ec \uada4\ub3c4\uc5d0 \ub300\uc785\ud568\uc73c\ub85c\uc368 1998\ub144, \uc774 \ud56d\uc131\ub4e4\uc758 \uc6b4\ub3d9\uc744 \uc720\ubc1c\uc2dc\ud0a4\uae30 \uc704\ud574\uc11c\ub294 \ubc18\uacbd 0.02 \uad11\ub144\uc758 \ubd80\ud53c \uc18d\uc5d0 \uc9c8\ub7c9 2\ubc31 6\uc2ed\ub9cc M\u2609\uc774 \ub4e4\uc5b4 \uc788\uc5b4\uc57c \ud55c\ub2e4\ub294 \uacb0\ub860\uc744 \uc5bb\uc5b4\ub0c8\ub2e4. \uadf8 \uc774\ub798\ub85c \ud56d\uc131\ub4e4 \uc911 \ud558\ub098(S2)\uac00 1\ud68c \uacf5\uc804\uc744 \uc644\ub8cc\ud588\ub2e4. \ucc9c\ubb38\ud559\uc790\ub4e4\uc740 \uada4\ub3c4 \ub370\uc774\ud130\ub85c\ubd80\ud130 \uc7ac\uacc4\uc0b0\ud558\uc5ec \ud56d\uc131\ub4e4\uc758 \uc6b4\ub3d9\uc744 \uc720\ubc1c\uc2dc\ud0a4\uae30 \uc704\ud574\uc11c\ub294 \ubc18\uacbd 0.002 \uad11\ub144 \uc758 \ubd80\ud53c \uc18d\uc5d0 \uc9c8\ub7c9 4\ubc31 3\uc2ed\ub9cc M\u2609\uc774 \ub4e4\uc5b4 \uc788\uc5b4\uc57c \ud55c\ub2e4\uace0 \uacb0\ub860\ub0b4\ub838\ub2e4. \ud06c\uae30 \uc0c1\ud55c\uc774 \uc5ec\uc804\ud788 \ub108\ubb34 \ucee4\uc11c \uc774\uac83\uc774 \uc288\ubc14\ub974\uce20\uc2e4\ud2b8 \ubc18\uacbd \uc774\ud558\uc778\uc9c0 \uc54c\uc544\ub0b4\ub294 \ub370 \uc5b4\ub824\uc6c0\uc774 \uc788\ub2e4. \uadf8\ub7fc\uc5d0\ub3c4 \ubd88\uad6c\ud558\uace0 \uad00\uce21 \uacb0\uacfc\ub294 \uc911\uc2ec\uc758 \ucc9c\uccb4\uac00 \ucd08\ub300\uc9c8\ub7c9 \ube14\ub799\ud640\uc784\uc744 \uac15\ud558\uac8c \uc2dc\uc0ac\ud558\uace0 \uc788\ub2e4. \uadf8 \uc774\uc678\uc5d0 \uc774\ub807\uac8c \uc791\uc740 \ubd80\ud53c \uc18d\uc5d0 \uc5c4\uccad\ub09c \uc591\uc758 \ubcf4\uc774\uc9c0 \uc54a\ub294 \uc9c8\ub7c9\uc774 \uc874\uc7ac\ud558\ub294 \uacbd\uc6b0\uc5d0 \ub300\ud55c \ub2e4\ub978 \ud0c0\ub2f9\ud55c \uc124\uba85\uc744 \uc81c\uacf5\ud560 \uc218 \uc5c6\ub2e4. \ub610\ud55c \uc774 \ucc9c\uccb4\uac00 \ube14\ub799\ud640 \ud2b9\uc720\uc758 \ud2b9\uc9d5\uc778 \uc0ac\uac74\uc758 \uc9c0\ud3c9\uc120\uc744 \uac00\uc9c0\uace0 \uc788\ub2e4\ub294 \uad00\uce21\uc801 \uc99d\uac70\ub3c4 \uc874\uc7ac\ud55c\ub2e4."} {"question": "\ub178\uc608 \uc81c\ub3c4 \ud3d0\uc9c0 \uc774\ud6c4, \ud130\ube0c\uba3c\uc774 \uce5c\uad6c\uc640 \uc9c0\uc9c0\uc790\ub4e4\uc774 \uc6b4\uc601\ud55c\uac83\uc740?", "paragraph": "\ub178\uc608 \uc81c\ub3c4\uac00 \ud3d0\uc9c0\ub41c \uc774\ud6c4\ub85c \ud130\ube0c\uba3c\uc758 \uce5c\uad6c\uc640 \uc9c0\uc9c0\uc790\ub4e4\uc740 \ud55c\ud3b8\uc73c\ub85c, \uadf8\ub97c \ub3d5\uae30 \uc704\ud574 \uae30\uae08\uc744 \uc6b4\uc601\ud558\uc600\ub2e4. \uc9c0\uc9c0\uc790 \uc911 \ud55c \uba85\uc778 \uc0ac\ub77c \ube0c\ub798\ub4dc\ud3ec\ub4dc\ub294 \u300a\ud574\ub9ac\uc5c7 \ud130\ube0c\uba3c\uc758 \uc0b6 \uc18d\uc758 \ud48d\uacbd\ub4e4\u300b \uc774\ub780 \uc81c\ubaa9\uc758 \uc804\uae30\ub97c \ucd9c\uac04\ud558\uc600\ub2e4. \uc774 132\ucabd \uc9dc\ub9ac \ucc45\uc740 1869\ub144\uc5d0 \ub098\uc654\uc73c\uba70, \ud130\ube0c\uba3c\uc5d0\uac8c $1200 \uc815\ub3c4\ub97c \uc81c\uacf5\ud558\uc600\ub2e4. \ube44\ub85d \uc5ec\ub7ec \ubbf8\uc801 \uc218\uc0ac\uc640 \uc9c0\uadf9\ud788 \uc8fc\uad00\uc801\uc778 \uad00\uc810\uc73c\ub85c \ud604\ub300\uc758 \uc804\uae30 \uc791\uac00\ub4e4\uc5d0\uac8c \ube44\ud310\ubc1b\uace0\ub294 \uc788\uc9c0\ub9cc, \uc774 \ucc45\uc740 \uc5ec\uc804\ud788 \ud130\ube0c\uba3c\uc758 \uc0b6\uc5d0 \ub300\ud55c \uc815\ubcf4\uc640 \uad00\uc810\uc758 \uc911\uc694\ud55c \uc6d0\ucc9c\uc73c\ub85c \ub0a8\uc544\uc788\ub2e4. \ube0c\ub798\ub4dc\ud3ec\ub4dc\ub294 1886\ub144 \u300a\ud574\ub9ac\uc5c7, \uadf8\uc758 \uce5c\uad6c\ub4e4\uc758 \ubaa8\uc138\u300b \ub77c\ub294 \uc81c\ubaa9\uc758 \ucc45\uc744 \ucd9c\uac04\ud558\uc600\ub294\ub370, \uc774 \ucc45\uc5d0\uc11c\ub294 \ub178\uc608 \uc81c\ub3c4\uc640 \ub0a8\ubd80\uc5d0 \ub300\ud55c \uad00\uc810\uc774 \uc880 \ub204\uadf8\ub7ec\uc838\uc788\ub294 \uc0c1\ud0dc\uc600\ub2e4. \uc774 \ucc45 \uc5ed\uc2dc \ud130\ube0c\uba3c\uc758 \uac00\ub09c \uad6c\uc81c\ub97c \uc704\ud55c \uc218\ub2e8\uc73c\ub85c \uc4f0\uc600\ub2e4.", "answer": "\uae30\uae08", "sentence": "\ub178\uc608 \uc81c\ub3c4\uac00 \ud3d0\uc9c0\ub41c \uc774\ud6c4\ub85c \ud130\ube0c\uba3c\uc758 \uce5c\uad6c\uc640 \uc9c0\uc9c0\uc790\ub4e4\uc740 \ud55c\ud3b8\uc73c\ub85c, \uadf8\ub97c \ub3d5\uae30 \uc704\ud574 \uae30\uae08 \uc744 \uc6b4\uc601\ud558\uc600\ub2e4.", "paragraph_sentence": " \ub178\uc608 \uc81c\ub3c4\uac00 \ud3d0\uc9c0\ub41c \uc774\ud6c4\ub85c \ud130\ube0c\uba3c\uc758 \uce5c\uad6c\uc640 \uc9c0\uc9c0\uc790\ub4e4\uc740 \ud55c\ud3b8\uc73c\ub85c, \uadf8\ub97c \ub3d5\uae30 \uc704\ud574 \uae30\uae08 \uc744 \uc6b4\uc601\ud558\uc600\ub2e4. \uc9c0\uc9c0\uc790 \uc911 \ud55c \uba85\uc778 \uc0ac\ub77c \ube0c\ub798\ub4dc\ud3ec\ub4dc\ub294 \u300a\ud574\ub9ac\uc5c7 \ud130\ube0c\uba3c\uc758 \uc0b6 \uc18d\uc758 \ud48d\uacbd\ub4e4\u300b \uc774\ub780 \uc81c\ubaa9\uc758 \uc804\uae30\ub97c \ucd9c\uac04\ud558\uc600\ub2e4. \uc774 132\ucabd \uc9dc\ub9ac \ucc45\uc740 1869\ub144\uc5d0 \ub098\uc654\uc73c\uba70, \ud130\ube0c\uba3c\uc5d0\uac8c $1200 \uc815\ub3c4\ub97c \uc81c\uacf5\ud558\uc600\ub2e4. \ube44\ub85d \uc5ec\ub7ec \ubbf8\uc801 \uc218\uc0ac\uc640 \uc9c0\uadf9\ud788 \uc8fc\uad00\uc801\uc778 \uad00\uc810\uc73c\ub85c \ud604\ub300\uc758 \uc804\uae30 \uc791\uac00\ub4e4\uc5d0\uac8c \ube44\ud310\ubc1b\uace0\ub294 \uc788\uc9c0\ub9cc, \uc774 \ucc45\uc740 \uc5ec\uc804\ud788 \ud130\ube0c\uba3c\uc758 \uc0b6\uc5d0 \ub300\ud55c \uc815\ubcf4\uc640 \uad00\uc810\uc758 \uc911\uc694\ud55c \uc6d0\ucc9c\uc73c\ub85c \ub0a8\uc544\uc788\ub2e4. \ube0c\ub798\ub4dc\ud3ec\ub4dc\ub294 1886\ub144 \u300a\ud574\ub9ac\uc5c7, \uadf8\uc758 \uce5c\uad6c\ub4e4\uc758 \ubaa8\uc138\u300b \ub77c\ub294 \uc81c\ubaa9\uc758 \ucc45\uc744 \ucd9c\uac04\ud558\uc600\ub294\ub370, \uc774 \ucc45\uc5d0\uc11c\ub294 \ub178\uc608 \uc81c\ub3c4\uc640 \ub0a8\ubd80\uc5d0 \ub300\ud55c \uad00\uc810\uc774 \uc880 \ub204\uadf8\ub7ec\uc838\uc788\ub294 \uc0c1\ud0dc\uc600\ub2e4. \uc774 \ucc45 \uc5ed\uc2dc \ud130\ube0c\uba3c\uc758 \uac00\ub09c \uad6c\uc81c\ub97c \uc704\ud55c \uc218\ub2e8\uc73c\ub85c \uc4f0\uc600\ub2e4.", "paragraph_answer": "\ub178\uc608 \uc81c\ub3c4\uac00 \ud3d0\uc9c0\ub41c \uc774\ud6c4\ub85c \ud130\ube0c\uba3c\uc758 \uce5c\uad6c\uc640 \uc9c0\uc9c0\uc790\ub4e4\uc740 \ud55c\ud3b8\uc73c\ub85c, \uadf8\ub97c \ub3d5\uae30 \uc704\ud574 \uae30\uae08 \uc744 \uc6b4\uc601\ud558\uc600\ub2e4. \uc9c0\uc9c0\uc790 \uc911 \ud55c \uba85\uc778 \uc0ac\ub77c \ube0c\ub798\ub4dc\ud3ec\ub4dc\ub294 \u300a\ud574\ub9ac\uc5c7 \ud130\ube0c\uba3c\uc758 \uc0b6 \uc18d\uc758 \ud48d\uacbd\ub4e4\u300b \uc774\ub780 \uc81c\ubaa9\uc758 \uc804\uae30\ub97c \ucd9c\uac04\ud558\uc600\ub2e4. \uc774 132\ucabd \uc9dc\ub9ac \ucc45\uc740 1869\ub144\uc5d0 \ub098\uc654\uc73c\uba70, \ud130\ube0c\uba3c\uc5d0\uac8c $1200 \uc815\ub3c4\ub97c \uc81c\uacf5\ud558\uc600\ub2e4. \ube44\ub85d \uc5ec\ub7ec \ubbf8\uc801 \uc218\uc0ac\uc640 \uc9c0\uadf9\ud788 \uc8fc\uad00\uc801\uc778 \uad00\uc810\uc73c\ub85c \ud604\ub300\uc758 \uc804\uae30 \uc791\uac00\ub4e4\uc5d0\uac8c \ube44\ud310\ubc1b\uace0\ub294 \uc788\uc9c0\ub9cc, \uc774 \ucc45\uc740 \uc5ec\uc804\ud788 \ud130\ube0c\uba3c\uc758 \uc0b6\uc5d0 \ub300\ud55c \uc815\ubcf4\uc640 \uad00\uc810\uc758 \uc911\uc694\ud55c \uc6d0\ucc9c\uc73c\ub85c \ub0a8\uc544\uc788\ub2e4. \ube0c\ub798\ub4dc\ud3ec\ub4dc\ub294 1886\ub144 \u300a\ud574\ub9ac\uc5c7, \uadf8\uc758 \uce5c\uad6c\ub4e4\uc758 \ubaa8\uc138\u300b \ub77c\ub294 \uc81c\ubaa9\uc758 \ucc45\uc744 \ucd9c\uac04\ud558\uc600\ub294\ub370, \uc774 \ucc45\uc5d0\uc11c\ub294 \ub178\uc608 \uc81c\ub3c4\uc640 \ub0a8\ubd80\uc5d0 \ub300\ud55c \uad00\uc810\uc774 \uc880 \ub204\uadf8\ub7ec\uc838\uc788\ub294 \uc0c1\ud0dc\uc600\ub2e4. \uc774 \ucc45 \uc5ed\uc2dc \ud130\ube0c\uba3c\uc758 \uac00\ub09c \uad6c\uc81c\ub97c \uc704\ud55c \uc218\ub2e8\uc73c\ub85c \uc4f0\uc600\ub2e4.", "sentence_answer": "\ub178\uc608 \uc81c\ub3c4\uac00 \ud3d0\uc9c0\ub41c \uc774\ud6c4\ub85c \ud130\ube0c\uba3c\uc758 \uce5c\uad6c\uc640 \uc9c0\uc9c0\uc790\ub4e4\uc740 \ud55c\ud3b8\uc73c\ub85c, \uadf8\ub97c \ub3d5\uae30 \uc704\ud574 \uae30\uae08 \uc744 \uc6b4\uc601\ud558\uc600\ub2e4.", "paragraph_id": "6570493-23-0", "paragraph_question": "question: \ub178\uc608 \uc81c\ub3c4 \ud3d0\uc9c0 \uc774\ud6c4, \ud130\ube0c\uba3c\uc774 \uce5c\uad6c\uc640 \uc9c0\uc9c0\uc790\ub4e4\uc774 \uc6b4\uc601\ud55c\uac83\uc740?, context: \ub178\uc608 \uc81c\ub3c4\uac00 \ud3d0\uc9c0\ub41c \uc774\ud6c4\ub85c \ud130\ube0c\uba3c\uc758 \uce5c\uad6c\uc640 \uc9c0\uc9c0\uc790\ub4e4\uc740 \ud55c\ud3b8\uc73c\ub85c, \uadf8\ub97c \ub3d5\uae30 \uc704\ud574 \uae30\uae08\uc744 \uc6b4\uc601\ud558\uc600\ub2e4. \uc9c0\uc9c0\uc790 \uc911 \ud55c \uba85\uc778 \uc0ac\ub77c \ube0c\ub798\ub4dc\ud3ec\ub4dc\ub294 \u300a\ud574\ub9ac\uc5c7 \ud130\ube0c\uba3c\uc758 \uc0b6 \uc18d\uc758 \ud48d\uacbd\ub4e4\u300b \uc774\ub780 \uc81c\ubaa9\uc758 \uc804\uae30\ub97c \ucd9c\uac04\ud558\uc600\ub2e4. \uc774 132\ucabd \uc9dc\ub9ac \ucc45\uc740 1869\ub144\uc5d0 \ub098\uc654\uc73c\uba70, \ud130\ube0c\uba3c\uc5d0\uac8c $1200 \uc815\ub3c4\ub97c \uc81c\uacf5\ud558\uc600\ub2e4. \ube44\ub85d \uc5ec\ub7ec \ubbf8\uc801 \uc218\uc0ac\uc640 \uc9c0\uadf9\ud788 \uc8fc\uad00\uc801\uc778 \uad00\uc810\uc73c\ub85c \ud604\ub300\uc758 \uc804\uae30 \uc791\uac00\ub4e4\uc5d0\uac8c \ube44\ud310\ubc1b\uace0\ub294 \uc788\uc9c0\ub9cc, \uc774 \ucc45\uc740 \uc5ec\uc804\ud788 \ud130\ube0c\uba3c\uc758 \uc0b6\uc5d0 \ub300\ud55c \uc815\ubcf4\uc640 \uad00\uc810\uc758 \uc911\uc694\ud55c \uc6d0\ucc9c\uc73c\ub85c \ub0a8\uc544\uc788\ub2e4. \ube0c\ub798\ub4dc\ud3ec\ub4dc\ub294 1886\ub144 \u300a\ud574\ub9ac\uc5c7, \uadf8\uc758 \uce5c\uad6c\ub4e4\uc758 \ubaa8\uc138\u300b \ub77c\ub294 \uc81c\ubaa9\uc758 \ucc45\uc744 \ucd9c\uac04\ud558\uc600\ub294\ub370, \uc774 \ucc45\uc5d0\uc11c\ub294 \ub178\uc608 \uc81c\ub3c4\uc640 \ub0a8\ubd80\uc5d0 \ub300\ud55c \uad00\uc810\uc774 \uc880 \ub204\uadf8\ub7ec\uc838\uc788\ub294 \uc0c1\ud0dc\uc600\ub2e4. \uc774 \ucc45 \uc5ed\uc2dc \ud130\ube0c\uba3c\uc758 \uac00\ub09c \uad6c\uc81c\ub97c \uc704\ud55c \uc218\ub2e8\uc73c\ub85c \uc4f0\uc600\ub2e4."} {"question": "\uc7a1\uc9c0 \ub864\ub9c1 \uc2a4\ud1a4\uc9c0\uc5d0\uc11c Teenage Dream \uc2dc\uae30\uc758 \ud398\ub9ac \ubcf4\uceec\uc744 \ud3c9\uac00\ud55c \uc0ac\ub78c\uc740?", "paragraph": "\ud398\ub9ac\ub294 \ucf58\ud2b8\ub784\ud1a0 \ubcf4\uceec \ubc94\uc704\ub97c \uac00\uc9c0\uace0 \uc788\ub2e4. \ud398\ub9ac\uc758 \uac00\ucc3d\ub825\uc740 \uce6d\ucc2c\uacfc \ube44\ud310 \ubaa8\ub450 \ubc1b\uace0 \uc788\ub2e4. \u300a\uac00\ub514\uc5b8\u300b\uc758 \ubca0\ud2f0 \ud074\ub77c\ud06c\ub294 \"\ud30c\uc6cc\ud480\ud55c \ubaa9\uc18c\ub9ac\ub294 \ucca0\uc800\ud558\uac8c \ud604\uc2e4\uc5d0 \uc785\uac01\ud55c\ub2e4\"\ub77c\uace0 \ub9d0\ud55c \ubc18\uba74, \u300a\ub864\ub9c1 \uc2a4\ud1a4\u300b\uc758 \ub86d \uc170\ud544\ub4dc\ub294 Teenage Dream \uc2dc\uae30 \ud398\ub9ac\uc758 \ubcf4\uceec\uc744 \"\uc2a4\ud0c0\uce74\ud1a0\ub85c \uae5c\ubc15\uc774\ub294 \uc2e0\ud638\ub97c \uac00\uacf5\ucc98\ub9ac \ud55c \uac83 \uac19\ub2e4\"\uace0 \ud45c\ud604\ud588\ub2e4. \u300amusicOMH\u300b\uc758 \ub300\ub7f0 \ud558\ube44\ub294 One of the Boys \uc2dc\uae30 \ud398\ub9ac\uc758 \ubcf4\uceec\uc5d0\uc11c \uc568\ub7ec\ub2c8\uc2a4 \ubaa8\ub9ac\uc138\ud2b8\uc758 \"\uac00\uc6b4\ub370 \uc74c\uc808\uc5d0\uc11c \uc625\ud0c0\ube0c\uac00 \ubcc0\ud558\ub294 \ud65c\uae30\ucc2c \ubaa9\uc18c\ub9ac\"\uc640 \uac19\uc740 \ud2b9\uc9d5\uc744 \ub290\uaf08\ub2e4\uace0 \uc37c\ub2e4. \u300aNME\u300b\uc758 \uc568\ub9ad\uc2a4 \ubc00\ub7ec\ub294 One of the Boys\uc5d0\uc11c \"\ud398\ub9ac\uc758 \ubb38\uc81c\uc810\uc740 \ubaa9\uc18c\ub9ac\"\uc784\uc744 \ub290\uaf08\ub2e4\uace0 \ub9ac\ubdf0\ud588\ub2e4. \uc774\uc640 \ubc18\ub300\ub85c \u300a\ub370\uc77c\ub9ac \ud154\ub808\uadf8\ub798\ud504\u300b\uc758 \ubc84\ub098\ub383 \ub9e5\ub110\ud2f0\ub294 Prism \ucf58\uc11c\ud2b8 \ud64d\ubcf4 \ub9ac\ubdf0\uc5d0\uc11c \"rock chick voice\"\ub77c\uba70 \uce6d\ucc2c\ud588\ub2e4.", "answer": "\ub86d \uc170\ud544\ub4dc", "sentence": "\u300a\uac00\ub514\uc5b8\u300b\uc758 \ubca0\ud2f0 \ud074\ub77c\ud06c\ub294 \"\ud30c\uc6cc\ud480\ud55c \ubaa9\uc18c\ub9ac\ub294 \ucca0\uc800\ud558\uac8c \ud604\uc2e4\uc5d0 \uc785\uac01\ud55c\ub2e4\"\ub77c\uace0 \ub9d0\ud55c \ubc18\uba74, \u300a\ub864\ub9c1 \uc2a4\ud1a4\u300b\uc758 \ub86d \uc170\ud544\ub4dc \ub294 Teenage Dream \uc2dc\uae30 \ud398\ub9ac\uc758 \ubcf4\uceec\uc744 \"\uc2a4\ud0c0\uce74\ud1a0\ub85c \uae5c\ubc15\uc774\ub294 \uc2e0\ud638\ub97c \uac00\uacf5\ucc98\ub9ac \ud55c \uac83 \uac19\ub2e4\"\uace0 \ud45c\ud604\ud588\ub2e4.", "paragraph_sentence": "\ud398\ub9ac\ub294 \ucf58\ud2b8\ub784\ud1a0 \ubcf4\uceec \ubc94\uc704\ub97c \uac00\uc9c0\uace0 \uc788\ub2e4. \ud398\ub9ac\uc758 \uac00\ucc3d\ub825\uc740 \uce6d\ucc2c\uacfc \ube44\ud310 \ubaa8\ub450 \ubc1b\uace0 \uc788\ub2e4. \u300a\uac00\ub514\uc5b8\u300b\uc758 \ubca0\ud2f0 \ud074\ub77c\ud06c\ub294 \"\ud30c\uc6cc\ud480\ud55c \ubaa9\uc18c\ub9ac\ub294 \ucca0\uc800\ud558\uac8c \ud604\uc2e4\uc5d0 \uc785\uac01\ud55c\ub2e4\"\ub77c\uace0 \ub9d0\ud55c \ubc18\uba74, \u300a\ub864\ub9c1 \uc2a4\ud1a4\u300b\uc758 \ub86d \uc170\ud544\ub4dc \ub294 Teenage Dream \uc2dc\uae30 \ud398\ub9ac\uc758 \ubcf4\uceec\uc744 \"\uc2a4\ud0c0\uce74\ud1a0\ub85c \uae5c\ubc15\uc774\ub294 \uc2e0\ud638\ub97c \uac00\uacf5\ucc98\ub9ac \ud55c \uac83 \uac19\ub2e4\"\uace0 \ud45c\ud604\ud588\ub2e4. \u300amusicOMH\u300b\uc758 \ub300\ub7f0 \ud558\ube44\ub294 One of the Boys \uc2dc\uae30 \ud398\ub9ac\uc758 \ubcf4\uceec\uc5d0\uc11c \uc568\ub7ec\ub2c8\uc2a4 \ubaa8\ub9ac\uc138\ud2b8\uc758 \"\uac00\uc6b4\ub370 \uc74c\uc808\uc5d0\uc11c \uc625\ud0c0\ube0c\uac00 \ubcc0\ud558\ub294 \ud65c\uae30\ucc2c \ubaa9\uc18c\ub9ac\"\uc640 \uac19\uc740 \ud2b9\uc9d5\uc744 \ub290\uaf08\ub2e4\uace0 \uc37c\ub2e4. \u300aNME\u300b\uc758 \uc568\ub9ad\uc2a4 \ubc00\ub7ec\ub294 One of the Boys\uc5d0\uc11c \"\ud398\ub9ac\uc758 \ubb38\uc81c\uc810\uc740 \ubaa9\uc18c\ub9ac\"\uc784\uc744 \ub290\uaf08\ub2e4\uace0 \ub9ac\ubdf0\ud588\ub2e4. \uc774\uc640 \ubc18\ub300\ub85c \u300a\ub370\uc77c\ub9ac \ud154\ub808\uadf8\ub798\ud504\u300b\uc758 \ubc84\ub098\ub383 \ub9e5\ub110\ud2f0\ub294 Prism \ucf58\uc11c\ud2b8 \ud64d\ubcf4 \ub9ac\ubdf0\uc5d0\uc11c \"rock chick voice\"\ub77c\uba70 \uce6d\ucc2c\ud588\ub2e4.", "paragraph_answer": "\ud398\ub9ac\ub294 \ucf58\ud2b8\ub784\ud1a0 \ubcf4\uceec \ubc94\uc704\ub97c \uac00\uc9c0\uace0 \uc788\ub2e4. \ud398\ub9ac\uc758 \uac00\ucc3d\ub825\uc740 \uce6d\ucc2c\uacfc \ube44\ud310 \ubaa8\ub450 \ubc1b\uace0 \uc788\ub2e4. \u300a\uac00\ub514\uc5b8\u300b\uc758 \ubca0\ud2f0 \ud074\ub77c\ud06c\ub294 \"\ud30c\uc6cc\ud480\ud55c \ubaa9\uc18c\ub9ac\ub294 \ucca0\uc800\ud558\uac8c \ud604\uc2e4\uc5d0 \uc785\uac01\ud55c\ub2e4\"\ub77c\uace0 \ub9d0\ud55c \ubc18\uba74, \u300a\ub864\ub9c1 \uc2a4\ud1a4\u300b\uc758 \ub86d \uc170\ud544\ub4dc \ub294 Teenage Dream \uc2dc\uae30 \ud398\ub9ac\uc758 \ubcf4\uceec\uc744 \"\uc2a4\ud0c0\uce74\ud1a0\ub85c \uae5c\ubc15\uc774\ub294 \uc2e0\ud638\ub97c \uac00\uacf5\ucc98\ub9ac \ud55c \uac83 \uac19\ub2e4\"\uace0 \ud45c\ud604\ud588\ub2e4. \u300amusicOMH\u300b\uc758 \ub300\ub7f0 \ud558\ube44\ub294 One of the Boys \uc2dc\uae30 \ud398\ub9ac\uc758 \ubcf4\uceec\uc5d0\uc11c \uc568\ub7ec\ub2c8\uc2a4 \ubaa8\ub9ac\uc138\ud2b8\uc758 \"\uac00\uc6b4\ub370 \uc74c\uc808\uc5d0\uc11c \uc625\ud0c0\ube0c\uac00 \ubcc0\ud558\ub294 \ud65c\uae30\ucc2c \ubaa9\uc18c\ub9ac\"\uc640 \uac19\uc740 \ud2b9\uc9d5\uc744 \ub290\uaf08\ub2e4\uace0 \uc37c\ub2e4. \u300aNME\u300b\uc758 \uc568\ub9ad\uc2a4 \ubc00\ub7ec\ub294 One of the Boys\uc5d0\uc11c \"\ud398\ub9ac\uc758 \ubb38\uc81c\uc810\uc740 \ubaa9\uc18c\ub9ac\"\uc784\uc744 \ub290\uaf08\ub2e4\uace0 \ub9ac\ubdf0\ud588\ub2e4. \uc774\uc640 \ubc18\ub300\ub85c \u300a\ub370\uc77c\ub9ac \ud154\ub808\uadf8\ub798\ud504\u300b\uc758 \ubc84\ub098\ub383 \ub9e5\ub110\ud2f0\ub294 Prism \ucf58\uc11c\ud2b8 \ud64d\ubcf4 \ub9ac\ubdf0\uc5d0\uc11c \"rock chick voice\"\ub77c\uba70 \uce6d\ucc2c\ud588\ub2e4.", "sentence_answer": "\u300a\uac00\ub514\uc5b8\u300b\uc758 \ubca0\ud2f0 \ud074\ub77c\ud06c\ub294 \"\ud30c\uc6cc\ud480\ud55c \ubaa9\uc18c\ub9ac\ub294 \ucca0\uc800\ud558\uac8c \ud604\uc2e4\uc5d0 \uc785\uac01\ud55c\ub2e4\"\ub77c\uace0 \ub9d0\ud55c \ubc18\uba74, \u300a\ub864\ub9c1 \uc2a4\ud1a4\u300b\uc758 \ub86d \uc170\ud544\ub4dc \ub294 Teenage Dream \uc2dc\uae30 \ud398\ub9ac\uc758 \ubcf4\uceec\uc744 \"\uc2a4\ud0c0\uce74\ud1a0\ub85c \uae5c\ubc15\uc774\ub294 \uc2e0\ud638\ub97c \uac00\uacf5\ucc98\ub9ac \ud55c \uac83 \uac19\ub2e4\"\uace0 \ud45c\ud604\ud588\ub2e4.", "paragraph_id": "6597628-22-0", "paragraph_question": "question: \uc7a1\uc9c0 \ub864\ub9c1 \uc2a4\ud1a4\uc9c0\uc5d0\uc11c Teenage Dream \uc2dc\uae30\uc758 \ud398\ub9ac \ubcf4\uceec\uc744 \ud3c9\uac00\ud55c \uc0ac\ub78c\uc740?, context: \ud398\ub9ac\ub294 \ucf58\ud2b8\ub784\ud1a0 \ubcf4\uceec \ubc94\uc704\ub97c \uac00\uc9c0\uace0 \uc788\ub2e4. \ud398\ub9ac\uc758 \uac00\ucc3d\ub825\uc740 \uce6d\ucc2c\uacfc \ube44\ud310 \ubaa8\ub450 \ubc1b\uace0 \uc788\ub2e4. \u300a\uac00\ub514\uc5b8\u300b\uc758 \ubca0\ud2f0 \ud074\ub77c\ud06c\ub294 \"\ud30c\uc6cc\ud480\ud55c \ubaa9\uc18c\ub9ac\ub294 \ucca0\uc800\ud558\uac8c \ud604\uc2e4\uc5d0 \uc785\uac01\ud55c\ub2e4\"\ub77c\uace0 \ub9d0\ud55c \ubc18\uba74, \u300a\ub864\ub9c1 \uc2a4\ud1a4\u300b\uc758 \ub86d \uc170\ud544\ub4dc\ub294 Teenage Dream \uc2dc\uae30 \ud398\ub9ac\uc758 \ubcf4\uceec\uc744 \"\uc2a4\ud0c0\uce74\ud1a0\ub85c \uae5c\ubc15\uc774\ub294 \uc2e0\ud638\ub97c \uac00\uacf5\ucc98\ub9ac \ud55c \uac83 \uac19\ub2e4\"\uace0 \ud45c\ud604\ud588\ub2e4. \u300amusicOMH\u300b\uc758 \ub300\ub7f0 \ud558\ube44\ub294 One of the Boys \uc2dc\uae30 \ud398\ub9ac\uc758 \ubcf4\uceec\uc5d0\uc11c \uc568\ub7ec\ub2c8\uc2a4 \ubaa8\ub9ac\uc138\ud2b8\uc758 \"\uac00\uc6b4\ub370 \uc74c\uc808\uc5d0\uc11c \uc625\ud0c0\ube0c\uac00 \ubcc0\ud558\ub294 \ud65c\uae30\ucc2c \ubaa9\uc18c\ub9ac\"\uc640 \uac19\uc740 \ud2b9\uc9d5\uc744 \ub290\uaf08\ub2e4\uace0 \uc37c\ub2e4. \u300aNME\u300b\uc758 \uc568\ub9ad\uc2a4 \ubc00\ub7ec\ub294 One of the Boys\uc5d0\uc11c \"\ud398\ub9ac\uc758 \ubb38\uc81c\uc810\uc740 \ubaa9\uc18c\ub9ac\"\uc784\uc744 \ub290\uaf08\ub2e4\uace0 \ub9ac\ubdf0\ud588\ub2e4. \uc774\uc640 \ubc18\ub300\ub85c \u300a\ub370\uc77c\ub9ac \ud154\ub808\uadf8\ub798\ud504\u300b\uc758 \ubc84\ub098\ub383 \ub9e5\ub110\ud2f0\ub294 Prism \ucf58\uc11c\ud2b8 \ud64d\ubcf4 \ub9ac\ubdf0\uc5d0\uc11c \"rock chick voice\"\ub77c\uba70 \uce6d\ucc2c\ud588\ub2e4."} {"question": "\ud5e8\ub9ac\uc5d0\ud0c0 \uc5d4\uc774 \ud0dc\uc5b4\ub09c \ud574\ub294?", "paragraph": "\ud5e8\ub9ac\uc5d0\ud0c0 \uc564\uc740 \uc789\uae00\ub79c\ub4dc \ub0b4\uc804 \ub3c4\uc911\uc778 1644\ub144 6\uc6d4 26\uc77c, \uc5d1\uc11c\ud130\uc758 \ubca0\ub4dc\ud3ec\ub4dc \ud558\uc6b0\uc2a4\uc5d0\uc11c \ucc30\uc2a4 1\uc138\uc640 \uc655\ube44 \ud5e8\ub9ac\uc5d0\ud0c0 \ub9c8\ub9ac\uc544 \uc0ac\uc774\uc5d0\uc11c \ud0dc\uc5b4\ub0ac\ub2e4. \uadf8\ub140\uac00 \ud0dc\uc5b4\ub0a0 \ub2f9\uc2dc \uc5b4\uba38\ub2c8 \ud5e8\ub9ac\uc5d0\ud0c0 \ub9c8\ub9ac\uc544\ub294 \ub0a8\ud3b8\uacfc \ud568\uaed8 \uc788\ub358 \uc625\uc2a4\ud37c\ub4dc\ub97c \ub5a0\ub098 \ucd9c\uc0b0\uc744 \uc704\ud574 \uc5d1\uc11c\ud130\ub85c \uc628 \uc0c1\ud0dc\uc600\uace0, \uac13 \ud0dc\uc5b4\ub09c \uc544\uae30\uc640 \ub2e4\ub978 \uc790\ub140\ub4e4\uc758 \uc548\uc804\uc744 \uc704\ud574 \ud504\ub791\uc2a4\ub85c \ud5a5\ud558\uae30\ub85c \ub9c8\uc74c\uba39\uc5c8\ub2e4. \ud5e8\ub9ac\uc5d0\ud0c0 \ub9c8\ub9ac\uc544\ub294 \uc559\ub9ac 4\uc138\uc640 \ub9c8\ub9ac \ub4dc \uba54\ub514\uc2dc\uc2a4\uc758 \ub538\ub85c, \ub8e8\uc774 13\uc138\uc758 \ub204\uc774\uc600\uae30 \ub54c\ubb38\uc5d0 \uadf8 \ub538\uc778 \ud5e8\ub9ac\uc5d0\ud0c0 \uc564\uc5d0\uac8c\ub3c4 \ud504\ub791\uc2a4 \uc655\uc2e4\uc758 \ud53c\uac00 \ud750\ub974\uace0 \uc788\uc5c8\ub2e4. 1646\ub144 \uce5c\uc815\uc73c\ub85c \ub3cc\uc544\uc628 \ud5e8\ub9ac\uc5d0\ud0c0 \ub9c8\ub9ac\uc544\uc640 \uadf8\ub140\uc758 \uc544\uc774\ub4e4\uc740 \ub8e8\ube0c\ub974 \uad81\uc5d0\uc11c \uac70\ucc98\ud558\uac8c \ub418\uc5c8\uace0, 3\ub9cc \ub9ac\ube0c\ub974\uc758 \uc5f0\uae08\uacfc \uc0dd\uc81c\ub974\ub9f9\uc559\ub808\uc758 \uc131\uc774 \uc8fc\uc5b4\uc84c\uc9c0\ub9cc \uc774 \ub9c9\ub300\ud55c \ub3c8\uc740 \uc601\uad6d\uc758 \ucc30\uc2a4 1\uc138 \ubc0f \uc601\uad6d\uc5d0\uc11c \ud504\ub791\uc2a4\ub85c \ub3c4\ub9dd\uccd0 \uc628 \ub9dd\uba85 \uadc0\uc871\ub4e4\uc5d0\uac8c\ub85c \ub3cc\uc544\uac14\ub2e4. 1649\ub144 2\uc6d4, \uc544\ubc84\uc9c0 \ucc30\uc2a4 1\uc138\uac00 \ucc98\ud615\ub418\uc5c8\ub2e4\ub294 \uc18c\uc2dd\uc774 \uc804\ud574\uc84c\uc744 \ubb34\ub835 \ud5e8\ub9ac\uc5d0\ud0c0 \uc564\uacfc \uac00\uc871\ub4e4\uc740 \ub8e8\uc774 14\uc138 \ud615\uc81c\ub4e4\uc744 \ube44\ub86f\ud55c \uc655\uc2e4 \uc2dd\uad6c\ub4e4\uacfc \ud568\uaed8 \ud504\ub871\ub4dc\uc758 \ub09c\uc744 \ud53c\ud574 \ud314\ub808 \ub8e8\uc544\uc584\uc5d0 \uc788\uc5c8\ub2e4. \uadf8\ub85c\ubd80\ud130 10\uc5ec\ub144\uc774 \uc9c0\ub09c 1660\ub144, \uc789\uae00\ub79c\ub4dc \uc655\uc815\ubcf5\uace0\ub85c \ud5e8\ub9ac\uc5d0\ud0c0 \uc564\uc758 \uc624\ube60 \ucc30\uc2a4 2\uc138\uac00 \uad6d\uc655\uc73c\ub85c \ucd94\ub300\ub418\uc5c8\ub2e4. \uadf8\ub140\uac00 \uc601\uad6d\uc758 \uacf5\uc8fc \uc9c0\uc704\ub97c \ub418\ucc3e\uc790 \ub8e8\uc774 14\uc138\uc758 \ub3d9\uc0dd\uc774\uc790 \ud5e8\ub9ac\uc5d0\ud0c0\uc758 \uc774\uc885\uc0ac\ucd0c \uc624\ube60\uc778 \uc624\ub97c\ub808\uc559 \uacf5 \ud544\ub9ac\ud504\ub294 \ud5e8\ub9ac\uc5d0\ud0c0\uc5d0\uac8c \uad6c\ud63c\ud588\uc9c0\ub9cc \uadf8\ub140\ub294 \uc5b4\uba38\ub2c8\uc640 \ud568\uaed8 \uc77c\ub2e8 \uc601\uad6d\uc73c\ub85c \ub3cc\uc544\uac14\ub2e4. \uadf8 \ud574 11\uc6d4 22\uc77c \ud504\ub791\uc2a4 \uc655\uc2e4\uc740 \uc815\uc2dd\uc73c\ub85c \uacb0\ud63c \uc2e0\uccad\uc744 \ud588\uace0, \ucc30\uc2a4 2\uc138\ub294 \uc5ec\ub3d9\uc0dd\uc744 \uc704\ud574 \uc9c0\ucc38\uae08\uacfc \ubabd\ud0c0\ub974\uc9c0 \uc131\uc744 \uc8fc\uc5c8\ub2e4.", "answer": "1644\ub144", "sentence": "\ud5e8\ub9ac\uc5d0\ud0c0 \uc564\uc740 \uc789\uae00\ub79c\ub4dc \ub0b4\uc804 \ub3c4\uc911\uc778 1644\ub144 6\uc6d4 26\uc77c, \uc5d1\uc11c\ud130\uc758 \ubca0\ub4dc\ud3ec\ub4dc \ud558\uc6b0\uc2a4\uc5d0\uc11c \ucc30\uc2a4 1\uc138\uc640 \uc655\ube44 \ud5e8\ub9ac\uc5d0\ud0c0 \ub9c8\ub9ac\uc544 \uc0ac\uc774\uc5d0\uc11c \ud0dc\uc5b4\ub0ac\ub2e4.", "paragraph_sentence": " \ud5e8\ub9ac\uc5d0\ud0c0 \uc564\uc740 \uc789\uae00\ub79c\ub4dc \ub0b4\uc804 \ub3c4\uc911\uc778 1644\ub144 6\uc6d4 26\uc77c, \uc5d1\uc11c\ud130\uc758 \ubca0\ub4dc\ud3ec\ub4dc \ud558\uc6b0\uc2a4\uc5d0\uc11c \ucc30\uc2a4 1\uc138\uc640 \uc655\ube44 \ud5e8\ub9ac\uc5d0\ud0c0 \ub9c8\ub9ac\uc544 \uc0ac\uc774\uc5d0\uc11c \ud0dc\uc5b4\ub0ac\ub2e4. \uadf8\ub140\uac00 \ud0dc\uc5b4\ub0a0 \ub2f9\uc2dc \uc5b4\uba38\ub2c8 \ud5e8\ub9ac\uc5d0\ud0c0 \ub9c8\ub9ac\uc544\ub294 \ub0a8\ud3b8\uacfc \ud568\uaed8 \uc788\ub358 \uc625\uc2a4\ud37c\ub4dc\ub97c \ub5a0\ub098 \ucd9c\uc0b0\uc744 \uc704\ud574 \uc5d1\uc11c\ud130\ub85c \uc628 \uc0c1\ud0dc\uc600\uace0, \uac13 \ud0dc\uc5b4\ub09c \uc544\uae30\uc640 \ub2e4\ub978 \uc790\ub140\ub4e4\uc758 \uc548\uc804\uc744 \uc704\ud574 \ud504\ub791\uc2a4\ub85c \ud5a5\ud558\uae30\ub85c \ub9c8\uc74c\uba39\uc5c8\ub2e4. \ud5e8\ub9ac\uc5d0\ud0c0 \ub9c8\ub9ac\uc544\ub294 \uc559\ub9ac 4\uc138\uc640 \ub9c8\ub9ac \ub4dc \uba54\ub514\uc2dc\uc2a4\uc758 \ub538\ub85c, \ub8e8\uc774 13\uc138\uc758 \ub204\uc774\uc600\uae30 \ub54c\ubb38\uc5d0 \uadf8 \ub538\uc778 \ud5e8\ub9ac\uc5d0\ud0c0 \uc564\uc5d0\uac8c\ub3c4 \ud504\ub791\uc2a4 \uc655\uc2e4\uc758 \ud53c\uac00 \ud750\ub974\uace0 \uc788\uc5c8\ub2e4. 1646\ub144 \uce5c\uc815\uc73c\ub85c \ub3cc\uc544\uc628 \ud5e8\ub9ac\uc5d0\ud0c0 \ub9c8\ub9ac\uc544\uc640 \uadf8\ub140\uc758 \uc544\uc774\ub4e4\uc740 \ub8e8\ube0c\ub974 \uad81\uc5d0\uc11c \uac70\ucc98\ud558\uac8c \ub418\uc5c8\uace0, 3\ub9cc \ub9ac\ube0c\ub974\uc758 \uc5f0\uae08\uacfc \uc0dd\uc81c\ub974\ub9f9\uc559\ub808\uc758 \uc131\uc774 \uc8fc\uc5b4\uc84c\uc9c0\ub9cc \uc774 \ub9c9\ub300\ud55c \ub3c8\uc740 \uc601\uad6d\uc758 \ucc30\uc2a4 1\uc138 \ubc0f \uc601\uad6d\uc5d0\uc11c \ud504\ub791\uc2a4\ub85c \ub3c4\ub9dd\uccd0 \uc628 \ub9dd\uba85 \uadc0\uc871\ub4e4\uc5d0\uac8c\ub85c \ub3cc\uc544\uac14\ub2e4. 1649\ub144 2\uc6d4, \uc544\ubc84\uc9c0 \ucc30\uc2a4 1\uc138\uac00 \ucc98\ud615\ub418\uc5c8\ub2e4\ub294 \uc18c\uc2dd\uc774 \uc804\ud574\uc84c\uc744 \ubb34\ub835 \ud5e8\ub9ac\uc5d0\ud0c0 \uc564\uacfc \uac00\uc871\ub4e4\uc740 \ub8e8\uc774 14\uc138 \ud615\uc81c\ub4e4\uc744 \ube44\ub86f\ud55c \uc655\uc2e4 \uc2dd\uad6c\ub4e4\uacfc \ud568\uaed8 \ud504\ub871\ub4dc\uc758 \ub09c\uc744 \ud53c\ud574 \ud314\ub808 \ub8e8\uc544\uc584\uc5d0 \uc788\uc5c8\ub2e4. \uadf8\ub85c\ubd80\ud130 10\uc5ec\ub144\uc774 \uc9c0\ub09c 1660\ub144, \uc789\uae00\ub79c\ub4dc \uc655\uc815\ubcf5\uace0\ub85c \ud5e8\ub9ac\uc5d0\ud0c0 \uc564\uc758 \uc624\ube60 \ucc30\uc2a4 2\uc138\uac00 \uad6d\uc655\uc73c\ub85c \ucd94\ub300\ub418\uc5c8\ub2e4. \uadf8\ub140\uac00 \uc601\uad6d\uc758 \uacf5\uc8fc \uc9c0\uc704\ub97c \ub418\ucc3e\uc790 \ub8e8\uc774 14\uc138\uc758 \ub3d9\uc0dd\uc774\uc790 \ud5e8\ub9ac\uc5d0\ud0c0\uc758 \uc774\uc885\uc0ac\ucd0c \uc624\ube60\uc778 \uc624\ub97c\ub808\uc559 \uacf5 \ud544\ub9ac\ud504\ub294 \ud5e8\ub9ac\uc5d0\ud0c0\uc5d0\uac8c \uad6c\ud63c\ud588\uc9c0\ub9cc \uadf8\ub140\ub294 \uc5b4\uba38\ub2c8\uc640 \ud568\uaed8 \uc77c\ub2e8 \uc601\uad6d\uc73c\ub85c \ub3cc\uc544\uac14\ub2e4. \uadf8 \ud574 11\uc6d4 22\uc77c \ud504\ub791\uc2a4 \uc655\uc2e4\uc740 \uc815\uc2dd\uc73c\ub85c \uacb0\ud63c \uc2e0\uccad\uc744 \ud588\uace0, \ucc30\uc2a4 2\uc138\ub294 \uc5ec\ub3d9\uc0dd\uc744 \uc704\ud574 \uc9c0\ucc38\uae08\uacfc \ubabd\ud0c0\ub974\uc9c0 \uc131\uc744 \uc8fc\uc5c8\ub2e4.", "paragraph_answer": "\ud5e8\ub9ac\uc5d0\ud0c0 \uc564\uc740 \uc789\uae00\ub79c\ub4dc \ub0b4\uc804 \ub3c4\uc911\uc778 1644\ub144 6\uc6d4 26\uc77c, \uc5d1\uc11c\ud130\uc758 \ubca0\ub4dc\ud3ec\ub4dc \ud558\uc6b0\uc2a4\uc5d0\uc11c \ucc30\uc2a4 1\uc138\uc640 \uc655\ube44 \ud5e8\ub9ac\uc5d0\ud0c0 \ub9c8\ub9ac\uc544 \uc0ac\uc774\uc5d0\uc11c \ud0dc\uc5b4\ub0ac\ub2e4. \uadf8\ub140\uac00 \ud0dc\uc5b4\ub0a0 \ub2f9\uc2dc \uc5b4\uba38\ub2c8 \ud5e8\ub9ac\uc5d0\ud0c0 \ub9c8\ub9ac\uc544\ub294 \ub0a8\ud3b8\uacfc \ud568\uaed8 \uc788\ub358 \uc625\uc2a4\ud37c\ub4dc\ub97c \ub5a0\ub098 \ucd9c\uc0b0\uc744 \uc704\ud574 \uc5d1\uc11c\ud130\ub85c \uc628 \uc0c1\ud0dc\uc600\uace0, \uac13 \ud0dc\uc5b4\ub09c \uc544\uae30\uc640 \ub2e4\ub978 \uc790\ub140\ub4e4\uc758 \uc548\uc804\uc744 \uc704\ud574 \ud504\ub791\uc2a4\ub85c \ud5a5\ud558\uae30\ub85c \ub9c8\uc74c\uba39\uc5c8\ub2e4. \ud5e8\ub9ac\uc5d0\ud0c0 \ub9c8\ub9ac\uc544\ub294 \uc559\ub9ac 4\uc138\uc640 \ub9c8\ub9ac \ub4dc \uba54\ub514\uc2dc\uc2a4\uc758 \ub538\ub85c, \ub8e8\uc774 13\uc138\uc758 \ub204\uc774\uc600\uae30 \ub54c\ubb38\uc5d0 \uadf8 \ub538\uc778 \ud5e8\ub9ac\uc5d0\ud0c0 \uc564\uc5d0\uac8c\ub3c4 \ud504\ub791\uc2a4 \uc655\uc2e4\uc758 \ud53c\uac00 \ud750\ub974\uace0 \uc788\uc5c8\ub2e4. 1646\ub144 \uce5c\uc815\uc73c\ub85c \ub3cc\uc544\uc628 \ud5e8\ub9ac\uc5d0\ud0c0 \ub9c8\ub9ac\uc544\uc640 \uadf8\ub140\uc758 \uc544\uc774\ub4e4\uc740 \ub8e8\ube0c\ub974 \uad81\uc5d0\uc11c \uac70\ucc98\ud558\uac8c \ub418\uc5c8\uace0, 3\ub9cc \ub9ac\ube0c\ub974\uc758 \uc5f0\uae08\uacfc \uc0dd\uc81c\ub974\ub9f9\uc559\ub808\uc758 \uc131\uc774 \uc8fc\uc5b4\uc84c\uc9c0\ub9cc \uc774 \ub9c9\ub300\ud55c \ub3c8\uc740 \uc601\uad6d\uc758 \ucc30\uc2a4 1\uc138 \ubc0f \uc601\uad6d\uc5d0\uc11c \ud504\ub791\uc2a4\ub85c \ub3c4\ub9dd\uccd0 \uc628 \ub9dd\uba85 \uadc0\uc871\ub4e4\uc5d0\uac8c\ub85c \ub3cc\uc544\uac14\ub2e4. 1649\ub144 2\uc6d4, \uc544\ubc84\uc9c0 \ucc30\uc2a4 1\uc138\uac00 \ucc98\ud615\ub418\uc5c8\ub2e4\ub294 \uc18c\uc2dd\uc774 \uc804\ud574\uc84c\uc744 \ubb34\ub835 \ud5e8\ub9ac\uc5d0\ud0c0 \uc564\uacfc \uac00\uc871\ub4e4\uc740 \ub8e8\uc774 14\uc138 \ud615\uc81c\ub4e4\uc744 \ube44\ub86f\ud55c \uc655\uc2e4 \uc2dd\uad6c\ub4e4\uacfc \ud568\uaed8 \ud504\ub871\ub4dc\uc758 \ub09c\uc744 \ud53c\ud574 \ud314\ub808 \ub8e8\uc544\uc584\uc5d0 \uc788\uc5c8\ub2e4. \uadf8\ub85c\ubd80\ud130 10\uc5ec\ub144\uc774 \uc9c0\ub09c 1660\ub144, \uc789\uae00\ub79c\ub4dc \uc655\uc815\ubcf5\uace0\ub85c \ud5e8\ub9ac\uc5d0\ud0c0 \uc564\uc758 \uc624\ube60 \ucc30\uc2a4 2\uc138\uac00 \uad6d\uc655\uc73c\ub85c \ucd94\ub300\ub418\uc5c8\ub2e4. \uadf8\ub140\uac00 \uc601\uad6d\uc758 \uacf5\uc8fc \uc9c0\uc704\ub97c \ub418\ucc3e\uc790 \ub8e8\uc774 14\uc138\uc758 \ub3d9\uc0dd\uc774\uc790 \ud5e8\ub9ac\uc5d0\ud0c0\uc758 \uc774\uc885\uc0ac\ucd0c \uc624\ube60\uc778 \uc624\ub97c\ub808\uc559 \uacf5 \ud544\ub9ac\ud504\ub294 \ud5e8\ub9ac\uc5d0\ud0c0\uc5d0\uac8c \uad6c\ud63c\ud588\uc9c0\ub9cc \uadf8\ub140\ub294 \uc5b4\uba38\ub2c8\uc640 \ud568\uaed8 \uc77c\ub2e8 \uc601\uad6d\uc73c\ub85c \ub3cc\uc544\uac14\ub2e4. \uadf8 \ud574 11\uc6d4 22\uc77c \ud504\ub791\uc2a4 \uc655\uc2e4\uc740 \uc815\uc2dd\uc73c\ub85c \uacb0\ud63c \uc2e0\uccad\uc744 \ud588\uace0, \ucc30\uc2a4 2\uc138\ub294 \uc5ec\ub3d9\uc0dd\uc744 \uc704\ud574 \uc9c0\ucc38\uae08\uacfc \ubabd\ud0c0\ub974\uc9c0 \uc131\uc744 \uc8fc\uc5c8\ub2e4.", "sentence_answer": "\ud5e8\ub9ac\uc5d0\ud0c0 \uc564\uc740 \uc789\uae00\ub79c\ub4dc \ub0b4\uc804 \ub3c4\uc911\uc778 1644\ub144 6\uc6d4 26\uc77c, \uc5d1\uc11c\ud130\uc758 \ubca0\ub4dc\ud3ec\ub4dc \ud558\uc6b0\uc2a4\uc5d0\uc11c \ucc30\uc2a4 1\uc138\uc640 \uc655\ube44 \ud5e8\ub9ac\uc5d0\ud0c0 \ub9c8\ub9ac\uc544 \uc0ac\uc774\uc5d0\uc11c \ud0dc\uc5b4\ub0ac\ub2e4.", "paragraph_id": "6522500-0-0", "paragraph_question": "question: \ud5e8\ub9ac\uc5d0\ud0c0 \uc5d4\uc774 \ud0dc\uc5b4\ub09c \ud574\ub294?, context: \ud5e8\ub9ac\uc5d0\ud0c0 \uc564\uc740 \uc789\uae00\ub79c\ub4dc \ub0b4\uc804 \ub3c4\uc911\uc778 1644\ub144 6\uc6d4 26\uc77c, \uc5d1\uc11c\ud130\uc758 \ubca0\ub4dc\ud3ec\ub4dc \ud558\uc6b0\uc2a4\uc5d0\uc11c \ucc30\uc2a4 1\uc138\uc640 \uc655\ube44 \ud5e8\ub9ac\uc5d0\ud0c0 \ub9c8\ub9ac\uc544 \uc0ac\uc774\uc5d0\uc11c \ud0dc\uc5b4\ub0ac\ub2e4. \uadf8\ub140\uac00 \ud0dc\uc5b4\ub0a0 \ub2f9\uc2dc \uc5b4\uba38\ub2c8 \ud5e8\ub9ac\uc5d0\ud0c0 \ub9c8\ub9ac\uc544\ub294 \ub0a8\ud3b8\uacfc \ud568\uaed8 \uc788\ub358 \uc625\uc2a4\ud37c\ub4dc\ub97c \ub5a0\ub098 \ucd9c\uc0b0\uc744 \uc704\ud574 \uc5d1\uc11c\ud130\ub85c \uc628 \uc0c1\ud0dc\uc600\uace0, \uac13 \ud0dc\uc5b4\ub09c \uc544\uae30\uc640 \ub2e4\ub978 \uc790\ub140\ub4e4\uc758 \uc548\uc804\uc744 \uc704\ud574 \ud504\ub791\uc2a4\ub85c \ud5a5\ud558\uae30\ub85c \ub9c8\uc74c\uba39\uc5c8\ub2e4. \ud5e8\ub9ac\uc5d0\ud0c0 \ub9c8\ub9ac\uc544\ub294 \uc559\ub9ac 4\uc138\uc640 \ub9c8\ub9ac \ub4dc \uba54\ub514\uc2dc\uc2a4\uc758 \ub538\ub85c, \ub8e8\uc774 13\uc138\uc758 \ub204\uc774\uc600\uae30 \ub54c\ubb38\uc5d0 \uadf8 \ub538\uc778 \ud5e8\ub9ac\uc5d0\ud0c0 \uc564\uc5d0\uac8c\ub3c4 \ud504\ub791\uc2a4 \uc655\uc2e4\uc758 \ud53c\uac00 \ud750\ub974\uace0 \uc788\uc5c8\ub2e4. 1646\ub144 \uce5c\uc815\uc73c\ub85c \ub3cc\uc544\uc628 \ud5e8\ub9ac\uc5d0\ud0c0 \ub9c8\ub9ac\uc544\uc640 \uadf8\ub140\uc758 \uc544\uc774\ub4e4\uc740 \ub8e8\ube0c\ub974 \uad81\uc5d0\uc11c \uac70\ucc98\ud558\uac8c \ub418\uc5c8\uace0, 3\ub9cc \ub9ac\ube0c\ub974\uc758 \uc5f0\uae08\uacfc \uc0dd\uc81c\ub974\ub9f9\uc559\ub808\uc758 \uc131\uc774 \uc8fc\uc5b4\uc84c\uc9c0\ub9cc \uc774 \ub9c9\ub300\ud55c \ub3c8\uc740 \uc601\uad6d\uc758 \ucc30\uc2a4 1\uc138 \ubc0f \uc601\uad6d\uc5d0\uc11c \ud504\ub791\uc2a4\ub85c \ub3c4\ub9dd\uccd0 \uc628 \ub9dd\uba85 \uadc0\uc871\ub4e4\uc5d0\uac8c\ub85c \ub3cc\uc544\uac14\ub2e4. 1649\ub144 2\uc6d4, \uc544\ubc84\uc9c0 \ucc30\uc2a4 1\uc138\uac00 \ucc98\ud615\ub418\uc5c8\ub2e4\ub294 \uc18c\uc2dd\uc774 \uc804\ud574\uc84c\uc744 \ubb34\ub835 \ud5e8\ub9ac\uc5d0\ud0c0 \uc564\uacfc \uac00\uc871\ub4e4\uc740 \ub8e8\uc774 14\uc138 \ud615\uc81c\ub4e4\uc744 \ube44\ub86f\ud55c \uc655\uc2e4 \uc2dd\uad6c\ub4e4\uacfc \ud568\uaed8 \ud504\ub871\ub4dc\uc758 \ub09c\uc744 \ud53c\ud574 \ud314\ub808 \ub8e8\uc544\uc584\uc5d0 \uc788\uc5c8\ub2e4. \uadf8\ub85c\ubd80\ud130 10\uc5ec\ub144\uc774 \uc9c0\ub09c 1660\ub144, \uc789\uae00\ub79c\ub4dc \uc655\uc815\ubcf5\uace0\ub85c \ud5e8\ub9ac\uc5d0\ud0c0 \uc564\uc758 \uc624\ube60 \ucc30\uc2a4 2\uc138\uac00 \uad6d\uc655\uc73c\ub85c \ucd94\ub300\ub418\uc5c8\ub2e4. \uadf8\ub140\uac00 \uc601\uad6d\uc758 \uacf5\uc8fc \uc9c0\uc704\ub97c \ub418\ucc3e\uc790 \ub8e8\uc774 14\uc138\uc758 \ub3d9\uc0dd\uc774\uc790 \ud5e8\ub9ac\uc5d0\ud0c0\uc758 \uc774\uc885\uc0ac\ucd0c \uc624\ube60\uc778 \uc624\ub97c\ub808\uc559 \uacf5 \ud544\ub9ac\ud504\ub294 \ud5e8\ub9ac\uc5d0\ud0c0\uc5d0\uac8c \uad6c\ud63c\ud588\uc9c0\ub9cc \uadf8\ub140\ub294 \uc5b4\uba38\ub2c8\uc640 \ud568\uaed8 \uc77c\ub2e8 \uc601\uad6d\uc73c\ub85c \ub3cc\uc544\uac14\ub2e4. \uadf8 \ud574 11\uc6d4 22\uc77c \ud504\ub791\uc2a4 \uc655\uc2e4\uc740 \uc815\uc2dd\uc73c\ub85c \uacb0\ud63c \uc2e0\uccad\uc744 \ud588\uace0, \ucc30\uc2a4 2\uc138\ub294 \uc5ec\ub3d9\uc0dd\uc744 \uc704\ud574 \uc9c0\ucc38\uae08\uacfc \ubabd\ud0c0\ub974\uc9c0 \uc131\uc744 \uc8fc\uc5c8\ub2e4."} {"question": "\uc88c\ud0c0\uc790\ub85c\uc11c\ub3c4 \ub2ec\uc131\ud55c \uc5ed\ub300 \ucd5c\ucd08\uc758 \uae30\ub85d\uc740 3\ubc88\uc9f8 \uc2dc\uc98c\uc758 \uba87 \ud648\ub7f0\uc778\uac00?", "paragraph": "5\uc6d4 21\uc77c\uc758 \ub77c\ucfe0\ud150\uc804\uc5d0\uc11c\ub294 \ub098\uac00\uc774 \uc0ac\ud1a0\uc2dc\ub85c\ubd80\ud130 \ud648\ub7f0\uc744 \ub54c\ub824\ub0b4 10\ub144 \uc5f0\uc18d \ub450 \uc790\ub9bf\uc218 \ud648\ub7f0 \uae30\ub85d\uc744 \ub2ec\uc131, \uc694\ubbf8\uc6b0\ub9ac\uc5d0\uc11c\ub294 \ub098\uac00\uc2dc\ub9c8 \uc2dc\uac8c\uc624, \uc624 \uc0ac\ub2e4\ud558\ub8e8, \ud558\ub77c \ub2e4\uc4f0\ub178\ub9ac, \ub9c8\uc4f0\uc774 \ud788\ub370\ud0a4, \ub2e4\uce74\ud558\uc2dc \uc694\uc2dc\ub178\ubd80\uc5d0 \ub4a4\ub97c \uc774\uc740 6\ubc88\uc9f8\ub2e4. 6\uc6d4 19\uc77c\uc758 \uc8fc\ub2c8\uce58\uc804\uc5d0\uc11c\ub294 \uc544\uc0ac\uc624 \ub2e4\ucfe0\uc57c\ub85c\ubd80\ud130 \uc774 \uacbd\uae30\uc5d0\uc11c\uc758 2\uac1c \ud648\ub7f0\uc744 \uae30\ub85d\ud558\uba74\uc11c \ud1b5\uc0b0 6\ubc88\uc9f8\uc758 \uc5f0\uac04 20\ud648\ub7f0\uc744 \ub2ec\uc131\ud588\ub2e4. \uadf8\ub9ac\uace0 2004\ub144 \uc774\ud6c4 \uc591\ub300 \ub9ac\uadf8\uc5d0\uc11c \uac00\uc7a5 \ube60\ub978 \uae30\ub85d\uc73c\ub85c 20\ud648\ub7f0\uc744 \uae30\ub85d(\uac19\uc740 \ub0a0\uc5d0\ub294 \ud06c\ub808\uc774\uadf8 \ube0c\ub77c\uc824\ub3c4 \uae30\ub85d)\ud558\uc600\uace0 6\uc6d4\uc5d0\ub294 \ud0c0\uc728 3\ud560 7\ud47c 5\ub9ac, 14\ud648\ub7f0, 21\ud0c0\uc810\uc774\ub77c\ub294 \uc88b\uc740 \uc131\uc801\uc73c\ub85c \uc6d4\uac04 MVP\uc5d0 \uc120\uc815\ub418\uc5c8\ub2e4. 9\uc6d4 11\uc77c\uc758 \ud788\ub85c\uc2dc\ub9c8\uc804\uc5d0\uc11c \ud3ec\uc218\ub85c\uc11c\ub294 \ub178\ubb34\ub77c \uac00\uc4f0\uc57c, \ub2e4\ubd80\uce58 \uace0\uc774\uce58\uc5d0 \ub4a4\ub97c \uc787\ub294 \uc5ed\ub300 \uc138 \ubc88\uc9f8\uc758 \uc2dc\uc98c 40\ud648\ub7f0\uc744 \ub2ec\uc131\ud588\ub2e4. \uc774 \uae30\ub85d\uc740 \uad6c\ub2e8 \uc5ed\uc0ac\uc0c1 \ucc98\uc74c\uc788\ub294 \uc77c\uc774\uc790 \uc88c\ud0c0\uc790\ub85c\uc11c\ub3c4 \ub2ec\uc131\ud55c \ucd5c\ucd08\uc758 \uae30\ub85d\uc774\ub2e4. \uc2dc\uc98c \ud6c4\uc5d0 \uac00\uc9c4 \uc7ac\uacc4\uc57d\uacfc \uad00\ub828\ud574\uc11c\ub294 \ucd5c\uace0 \uc5f0\ubd09\uc778 4\uc5b5\uc5d4\uc744 \uad6c\ub2e8\uc5d0\uc11c \uc81c\uc2dc\ub418\uc5c8\ub2e4.", "answer": "40\ud648\ub7f0", "sentence": "9\uc6d4 11\uc77c\uc758 \ud788\ub85c\uc2dc\ub9c8\uc804\uc5d0\uc11c \ud3ec\uc218\ub85c\uc11c\ub294 \ub178\ubb34\ub77c \uac00\uc4f0\uc57c, \ub2e4\ubd80\uce58 \uace0\uc774\uce58\uc5d0 \ub4a4\ub97c \uc787\ub294 \uc5ed\ub300 \uc138 \ubc88\uc9f8\uc758 \uc2dc\uc98c 40\ud648\ub7f0 \uc744 \ub2ec\uc131\ud588\ub2e4.", "paragraph_sentence": "5\uc6d4 21\uc77c\uc758 \ub77c\ucfe0\ud150\uc804\uc5d0\uc11c\ub294 \ub098\uac00\uc774 \uc0ac\ud1a0\uc2dc\ub85c\ubd80\ud130 \ud648\ub7f0\uc744 \ub54c\ub824\ub0b4 10\ub144 \uc5f0\uc18d \ub450 \uc790\ub9bf\uc218 \ud648\ub7f0 \uae30\ub85d\uc744 \ub2ec\uc131, \uc694\ubbf8\uc6b0\ub9ac\uc5d0\uc11c\ub294 \ub098\uac00\uc2dc\ub9c8 \uc2dc\uac8c\uc624, \uc624 \uc0ac\ub2e4\ud558\ub8e8, \ud558\ub77c \ub2e4\uc4f0\ub178\ub9ac, \ub9c8\uc4f0\uc774 \ud788\ub370\ud0a4, \ub2e4\uce74\ud558\uc2dc \uc694\uc2dc\ub178\ubd80\uc5d0 \ub4a4\ub97c \uc774\uc740 6\ubc88\uc9f8\ub2e4. 6\uc6d4 19\uc77c\uc758 \uc8fc\ub2c8\uce58\uc804\uc5d0\uc11c\ub294 \uc544\uc0ac\uc624 \ub2e4\ucfe0\uc57c\ub85c\ubd80\ud130 \uc774 \uacbd\uae30\uc5d0\uc11c\uc758 2\uac1c \ud648\ub7f0\uc744 \uae30\ub85d\ud558\uba74\uc11c \ud1b5\uc0b0 6\ubc88\uc9f8\uc758 \uc5f0\uac04 20\ud648\ub7f0\uc744 \ub2ec\uc131\ud588\ub2e4. \uadf8\ub9ac\uace0 2004\ub144 \uc774\ud6c4 \uc591\ub300 \ub9ac\uadf8\uc5d0\uc11c \uac00\uc7a5 \ube60\ub978 \uae30\ub85d\uc73c\ub85c 20\ud648\ub7f0\uc744 \uae30\ub85d(\uac19\uc740 \ub0a0\uc5d0\ub294 \ud06c\ub808\uc774\uadf8 \ube0c\ub77c\uc824\ub3c4 \uae30\ub85d)\ud558\uc600\uace0 6\uc6d4\uc5d0\ub294 \ud0c0\uc728 3\ud560 7\ud47c 5\ub9ac, 14\ud648\ub7f0, 21\ud0c0\uc810\uc774\ub77c\ub294 \uc88b\uc740 \uc131\uc801\uc73c\ub85c \uc6d4\uac04 MVP\uc5d0 \uc120\uc815\ub418\uc5c8\ub2e4. 9\uc6d4 11\uc77c\uc758 \ud788\ub85c\uc2dc\ub9c8\uc804\uc5d0\uc11c \ud3ec\uc218\ub85c\uc11c\ub294 \ub178\ubb34\ub77c \uac00\uc4f0\uc57c, \ub2e4\ubd80\uce58 \uace0\uc774\uce58\uc5d0 \ub4a4\ub97c \uc787\ub294 \uc5ed\ub300 \uc138 \ubc88\uc9f8\uc758 \uc2dc\uc98c 40\ud648\ub7f0 \uc744 \ub2ec\uc131\ud588\ub2e4. \uc774 \uae30\ub85d\uc740 \uad6c\ub2e8 \uc5ed\uc0ac\uc0c1 \ucc98\uc74c\uc788\ub294 \uc77c\uc774\uc790 \uc88c\ud0c0\uc790\ub85c\uc11c\ub3c4 \ub2ec\uc131\ud55c \ucd5c\ucd08\uc758 \uae30\ub85d\uc774\ub2e4. \uc2dc\uc98c \ud6c4\uc5d0 \uac00\uc9c4 \uc7ac\uacc4\uc57d\uacfc \uad00\ub828\ud574\uc11c\ub294 \ucd5c\uace0 \uc5f0\ubd09\uc778 4\uc5b5\uc5d4\uc744 \uad6c\ub2e8\uc5d0\uc11c \uc81c\uc2dc\ub418\uc5c8\ub2e4.", "paragraph_answer": "5\uc6d4 21\uc77c\uc758 \ub77c\ucfe0\ud150\uc804\uc5d0\uc11c\ub294 \ub098\uac00\uc774 \uc0ac\ud1a0\uc2dc\ub85c\ubd80\ud130 \ud648\ub7f0\uc744 \ub54c\ub824\ub0b4 10\ub144 \uc5f0\uc18d \ub450 \uc790\ub9bf\uc218 \ud648\ub7f0 \uae30\ub85d\uc744 \ub2ec\uc131, \uc694\ubbf8\uc6b0\ub9ac\uc5d0\uc11c\ub294 \ub098\uac00\uc2dc\ub9c8 \uc2dc\uac8c\uc624, \uc624 \uc0ac\ub2e4\ud558\ub8e8, \ud558\ub77c \ub2e4\uc4f0\ub178\ub9ac, \ub9c8\uc4f0\uc774 \ud788\ub370\ud0a4, \ub2e4\uce74\ud558\uc2dc \uc694\uc2dc\ub178\ubd80\uc5d0 \ub4a4\ub97c \uc774\uc740 6\ubc88\uc9f8\ub2e4. 6\uc6d4 19\uc77c\uc758 \uc8fc\ub2c8\uce58\uc804\uc5d0\uc11c\ub294 \uc544\uc0ac\uc624 \ub2e4\ucfe0\uc57c\ub85c\ubd80\ud130 \uc774 \uacbd\uae30\uc5d0\uc11c\uc758 2\uac1c \ud648\ub7f0\uc744 \uae30\ub85d\ud558\uba74\uc11c \ud1b5\uc0b0 6\ubc88\uc9f8\uc758 \uc5f0\uac04 20\ud648\ub7f0\uc744 \ub2ec\uc131\ud588\ub2e4. \uadf8\ub9ac\uace0 2004\ub144 \uc774\ud6c4 \uc591\ub300 \ub9ac\uadf8\uc5d0\uc11c \uac00\uc7a5 \ube60\ub978 \uae30\ub85d\uc73c\ub85c 20\ud648\ub7f0\uc744 \uae30\ub85d(\uac19\uc740 \ub0a0\uc5d0\ub294 \ud06c\ub808\uc774\uadf8 \ube0c\ub77c\uc824\ub3c4 \uae30\ub85d)\ud558\uc600\uace0 6\uc6d4\uc5d0\ub294 \ud0c0\uc728 3\ud560 7\ud47c 5\ub9ac, 14\ud648\ub7f0, 21\ud0c0\uc810\uc774\ub77c\ub294 \uc88b\uc740 \uc131\uc801\uc73c\ub85c \uc6d4\uac04 MVP\uc5d0 \uc120\uc815\ub418\uc5c8\ub2e4. 9\uc6d4 11\uc77c\uc758 \ud788\ub85c\uc2dc\ub9c8\uc804\uc5d0\uc11c \ud3ec\uc218\ub85c\uc11c\ub294 \ub178\ubb34\ub77c \uac00\uc4f0\uc57c, \ub2e4\ubd80\uce58 \uace0\uc774\uce58\uc5d0 \ub4a4\ub97c \uc787\ub294 \uc5ed\ub300 \uc138 \ubc88\uc9f8\uc758 \uc2dc\uc98c 40\ud648\ub7f0 \uc744 \ub2ec\uc131\ud588\ub2e4. \uc774 \uae30\ub85d\uc740 \uad6c\ub2e8 \uc5ed\uc0ac\uc0c1 \ucc98\uc74c\uc788\ub294 \uc77c\uc774\uc790 \uc88c\ud0c0\uc790\ub85c\uc11c\ub3c4 \ub2ec\uc131\ud55c \ucd5c\ucd08\uc758 \uae30\ub85d\uc774\ub2e4. \uc2dc\uc98c \ud6c4\uc5d0 \uac00\uc9c4 \uc7ac\uacc4\uc57d\uacfc \uad00\ub828\ud574\uc11c\ub294 \ucd5c\uace0 \uc5f0\ubd09\uc778 4\uc5b5\uc5d4\uc744 \uad6c\ub2e8\uc5d0\uc11c \uc81c\uc2dc\ub418\uc5c8\ub2e4.", "sentence_answer": "9\uc6d4 11\uc77c\uc758 \ud788\ub85c\uc2dc\ub9c8\uc804\uc5d0\uc11c \ud3ec\uc218\ub85c\uc11c\ub294 \ub178\ubb34\ub77c \uac00\uc4f0\uc57c, \ub2e4\ubd80\uce58 \uace0\uc774\uce58\uc5d0 \ub4a4\ub97c \uc787\ub294 \uc5ed\ub300 \uc138 \ubc88\uc9f8\uc758 \uc2dc\uc98c 40\ud648\ub7f0 \uc744 \ub2ec\uc131\ud588\ub2e4.", "paragraph_id": "6533668-3-1", "paragraph_question": "question: \uc88c\ud0c0\uc790\ub85c\uc11c\ub3c4 \ub2ec\uc131\ud55c \uc5ed\ub300 \ucd5c\ucd08\uc758 \uae30\ub85d\uc740 3\ubc88\uc9f8 \uc2dc\uc98c\uc758 \uba87 \ud648\ub7f0\uc778\uac00?, context: 5\uc6d4 21\uc77c\uc758 \ub77c\ucfe0\ud150\uc804\uc5d0\uc11c\ub294 \ub098\uac00\uc774 \uc0ac\ud1a0\uc2dc\ub85c\ubd80\ud130 \ud648\ub7f0\uc744 \ub54c\ub824\ub0b4 10\ub144 \uc5f0\uc18d \ub450 \uc790\ub9bf\uc218 \ud648\ub7f0 \uae30\ub85d\uc744 \ub2ec\uc131, \uc694\ubbf8\uc6b0\ub9ac\uc5d0\uc11c\ub294 \ub098\uac00\uc2dc\ub9c8 \uc2dc\uac8c\uc624, \uc624 \uc0ac\ub2e4\ud558\ub8e8, \ud558\ub77c \ub2e4\uc4f0\ub178\ub9ac, \ub9c8\uc4f0\uc774 \ud788\ub370\ud0a4, \ub2e4\uce74\ud558\uc2dc \uc694\uc2dc\ub178\ubd80\uc5d0 \ub4a4\ub97c \uc774\uc740 6\ubc88\uc9f8\ub2e4. 6\uc6d4 19\uc77c\uc758 \uc8fc\ub2c8\uce58\uc804\uc5d0\uc11c\ub294 \uc544\uc0ac\uc624 \ub2e4\ucfe0\uc57c\ub85c\ubd80\ud130 \uc774 \uacbd\uae30\uc5d0\uc11c\uc758 2\uac1c \ud648\ub7f0\uc744 \uae30\ub85d\ud558\uba74\uc11c \ud1b5\uc0b0 6\ubc88\uc9f8\uc758 \uc5f0\uac04 20\ud648\ub7f0\uc744 \ub2ec\uc131\ud588\ub2e4. \uadf8\ub9ac\uace0 2004\ub144 \uc774\ud6c4 \uc591\ub300 \ub9ac\uadf8\uc5d0\uc11c \uac00\uc7a5 \ube60\ub978 \uae30\ub85d\uc73c\ub85c 20\ud648\ub7f0\uc744 \uae30\ub85d(\uac19\uc740 \ub0a0\uc5d0\ub294 \ud06c\ub808\uc774\uadf8 \ube0c\ub77c\uc824\ub3c4 \uae30\ub85d)\ud558\uc600\uace0 6\uc6d4\uc5d0\ub294 \ud0c0\uc728 3\ud560 7\ud47c 5\ub9ac, 14\ud648\ub7f0, 21\ud0c0\uc810\uc774\ub77c\ub294 \uc88b\uc740 \uc131\uc801\uc73c\ub85c \uc6d4\uac04 MVP\uc5d0 \uc120\uc815\ub418\uc5c8\ub2e4. 9\uc6d4 11\uc77c\uc758 \ud788\ub85c\uc2dc\ub9c8\uc804\uc5d0\uc11c \ud3ec\uc218\ub85c\uc11c\ub294 \ub178\ubb34\ub77c \uac00\uc4f0\uc57c, \ub2e4\ubd80\uce58 \uace0\uc774\uce58\uc5d0 \ub4a4\ub97c \uc787\ub294 \uc5ed\ub300 \uc138 \ubc88\uc9f8\uc758 \uc2dc\uc98c 40\ud648\ub7f0\uc744 \ub2ec\uc131\ud588\ub2e4. \uc774 \uae30\ub85d\uc740 \uad6c\ub2e8 \uc5ed\uc0ac\uc0c1 \ucc98\uc74c\uc788\ub294 \uc77c\uc774\uc790 \uc88c\ud0c0\uc790\ub85c\uc11c\ub3c4 \ub2ec\uc131\ud55c \ucd5c\ucd08\uc758 \uae30\ub85d\uc774\ub2e4. \uc2dc\uc98c \ud6c4\uc5d0 \uac00\uc9c4 \uc7ac\uacc4\uc57d\uacfc \uad00\ub828\ud574\uc11c\ub294 \ucd5c\uace0 \uc5f0\ubd09\uc778 4\uc5b5\uc5d4\uc744 \uad6c\ub2e8\uc5d0\uc11c \uc81c\uc2dc\ub418\uc5c8\ub2e4."} {"question": "\uadf8\ub9ac\uc2a4\uc758 \uc804\uccb4 \ud56d\uacf5 \ud3b8\uc218 \uc911 \uc774\ub77c\ud074\ub9ac\uc624\ub85c \uc624\ub294 \uac83\uc758 \ube44\uc728\uc740 \uc5bc\ub9c8\uc778\uac00?", "paragraph": "\ud06c\ub808\ud0c0 \uc12c\uc740 \uadf8\ub9ac\uc2a4\uc5d0\uc11c \ud734\uac00\uc9c0\ub85c \uc778\uae30\uac00 \ub192\ub2e4. \uadf8\ub9ac\uc2a4\uc5d0\uc11c \ud06c\ub808\ud0c0\ub85c \uc624\ub294 \uc0ac\ub78c \uc911 15%\uac00 \uc774\ub77c\ud074\ub9ac\uc624\uc758 \ud56d\uad6c\uc640 \uacf5\ud56d\uc744 \ud1b5\ud574 \uc624\uba70, \uadf8\ub9ac\uc2a4\uc758 \ubaa8\ub4e0 \ud56d\uacf5 \ud3b8\uc218 \uc911 20%\uac00 \uc774\ub77c\ud074\ub9ac\uc624\ub85c \uc624\ub294 \uac83\uc774\ub2e4. \uc804\uccb4\uc801\uc73c\ub85c 2\ubc31\ub9cc\uba85 \uc774\uc0c1\uc758 \uad00\uad11\uac1d\uc774 \ud06c\ub808\ud0c0\ub97c \ubc29\ubb38\ud588\uc73c\uba70, 1986\ub144\uc5d0\uc11c 1991\ub144\uae4c\uc9c0 \ud638\ud154 \uc219\ubc15\uc774 \uadf8\ub9ac\uc2a4 \ub2e4\ub978 \uc9c0\uc5ed\uc5d0\uc11c 25% \uc815\ub3c4 \ub298\uc5c8\uc744\ub54c \ud06c\ub808\ud0c0\ub294 53% \uc99d\uac00\ud588\ub2e4. \uc624\ub298\ub0a0 \uc774 \uc12c\uc758 \uad00\uad11 \uc778\ud504\ub77c\ub294 \ubaa8\ub4e0 \ucde8\ud5a5\uc5d0 \ub9de\uac8c \uc900\ube44\ub418\uc5b4 \uc788\ub2e4. \uc774 \uc12c\uc5d0\ub294 \uc644\ubcbd\ud55c \uc2dc\uc124\uc744 \uac16\ucd98 \ud070 \ud638\ud654 \ud638\ud154, \uc218\uc601\uc7a5, \uc2a4\ud3ec\uce20\uc640 \ub808\ud06c\ub9ac\uc5d0\uc774\uc158, \uc791\uc740 \uac00\uc871 \uc18c\uc720 \uc544\ud30c\ud2b8, \ucea0\ud551 \uc2dc\uc124 \ub4f1\uc774 \uc788\ub2e4. \ubc29\ubb38\uc790\ub4e4\uc740 \uc774\ub77c\ud074\ub9ac\uc624\uc640 \ud558\ub2c8\uc544\uc758 \ub450 \uad6d\uc81c \uacf5\ud56d\uc774\ub098 \uc774\ub77c\ud074\ub9ac\uc624, \ud558\ub2c8\uc544, \ub808\ud300\ub178, \uc544\uc694\uc2a4 \ub2c8\ucf5c\ub77c\uc624\uc2a4\uc758 \ud56d\uad6c\ub97c \ud1b5\ud574 \uc12c\uc5d0 \uc624\uac08 \uc218 \uc788\ub2e4.", "answer": "20%", "sentence": "\uadf8\ub9ac\uc2a4\uc5d0\uc11c \ud06c\ub808\ud0c0\ub85c \uc624\ub294 \uc0ac\ub78c \uc911 15%\uac00 \uc774\ub77c\ud074\ub9ac\uc624\uc758 \ud56d\uad6c\uc640 \uacf5\ud56d\uc744 \ud1b5\ud574 \uc624\uba70, \uadf8\ub9ac\uc2a4\uc758 \ubaa8\ub4e0 \ud56d\uacf5 \ud3b8\uc218 \uc911 20% \uac00 \uc774\ub77c\ud074\ub9ac\uc624\ub85c \uc624\ub294 \uac83\uc774\ub2e4.", "paragraph_sentence": "\ud06c\ub808\ud0c0 \uc12c\uc740 \uadf8\ub9ac\uc2a4\uc5d0\uc11c \ud734\uac00\uc9c0\ub85c \uc778\uae30\uac00 \ub192\ub2e4. \uadf8\ub9ac\uc2a4\uc5d0\uc11c \ud06c\ub808\ud0c0\ub85c \uc624\ub294 \uc0ac\ub78c \uc911 15%\uac00 \uc774\ub77c\ud074\ub9ac\uc624\uc758 \ud56d\uad6c\uc640 \uacf5\ud56d\uc744 \ud1b5\ud574 \uc624\uba70, \uadf8\ub9ac\uc2a4\uc758 \ubaa8\ub4e0 \ud56d\uacf5 \ud3b8\uc218 \uc911 20% \uac00 \uc774\ub77c\ud074\ub9ac\uc624\ub85c \uc624\ub294 \uac83\uc774\ub2e4. \uc804\uccb4\uc801\uc73c\ub85c 2\ubc31\ub9cc\uba85 \uc774\uc0c1\uc758 \uad00\uad11\uac1d\uc774 \ud06c\ub808\ud0c0\ub97c \ubc29\ubb38\ud588\uc73c\uba70, 1986\ub144\uc5d0\uc11c 1991\ub144\uae4c\uc9c0 \ud638\ud154 \uc219\ubc15\uc774 \uadf8\ub9ac\uc2a4 \ub2e4\ub978 \uc9c0\uc5ed\uc5d0\uc11c 25% \uc815\ub3c4 \ub298\uc5c8\uc744\ub54c \ud06c\ub808\ud0c0\ub294 53% \uc99d\uac00\ud588\ub2e4. \uc624\ub298\ub0a0 \uc774 \uc12c\uc758 \uad00\uad11 \uc778\ud504\ub77c\ub294 \ubaa8\ub4e0 \ucde8\ud5a5\uc5d0 \ub9de\uac8c \uc900\ube44\ub418\uc5b4 \uc788\ub2e4. \uc774 \uc12c\uc5d0\ub294 \uc644\ubcbd\ud55c \uc2dc\uc124\uc744 \uac16\ucd98 \ud070 \ud638\ud654 \ud638\ud154, \uc218\uc601\uc7a5, \uc2a4\ud3ec\uce20\uc640 \ub808\ud06c\ub9ac\uc5d0\uc774\uc158, \uc791\uc740 \uac00\uc871 \uc18c\uc720 \uc544\ud30c\ud2b8, \ucea0\ud551 \uc2dc\uc124 \ub4f1\uc774 \uc788\ub2e4. \ubc29\ubb38\uc790\ub4e4\uc740 \uc774\ub77c\ud074\ub9ac\uc624\uc640 \ud558\ub2c8\uc544\uc758 \ub450 \uad6d\uc81c \uacf5\ud56d\uc774\ub098 \uc774\ub77c\ud074\ub9ac\uc624, \ud558\ub2c8\uc544, \ub808\ud300\ub178, \uc544\uc694\uc2a4 \ub2c8\ucf5c\ub77c\uc624\uc2a4\uc758 \ud56d\uad6c\ub97c \ud1b5\ud574 \uc12c\uc5d0 \uc624\uac08 \uc218 \uc788\ub2e4.", "paragraph_answer": "\ud06c\ub808\ud0c0 \uc12c\uc740 \uadf8\ub9ac\uc2a4\uc5d0\uc11c \ud734\uac00\uc9c0\ub85c \uc778\uae30\uac00 \ub192\ub2e4. \uadf8\ub9ac\uc2a4\uc5d0\uc11c \ud06c\ub808\ud0c0\ub85c \uc624\ub294 \uc0ac\ub78c \uc911 15%\uac00 \uc774\ub77c\ud074\ub9ac\uc624\uc758 \ud56d\uad6c\uc640 \uacf5\ud56d\uc744 \ud1b5\ud574 \uc624\uba70, \uadf8\ub9ac\uc2a4\uc758 \ubaa8\ub4e0 \ud56d\uacf5 \ud3b8\uc218 \uc911 20% \uac00 \uc774\ub77c\ud074\ub9ac\uc624\ub85c \uc624\ub294 \uac83\uc774\ub2e4. \uc804\uccb4\uc801\uc73c\ub85c 2\ubc31\ub9cc\uba85 \uc774\uc0c1\uc758 \uad00\uad11\uac1d\uc774 \ud06c\ub808\ud0c0\ub97c \ubc29\ubb38\ud588\uc73c\uba70, 1986\ub144\uc5d0\uc11c 1991\ub144\uae4c\uc9c0 \ud638\ud154 \uc219\ubc15\uc774 \uadf8\ub9ac\uc2a4 \ub2e4\ub978 \uc9c0\uc5ed\uc5d0\uc11c 25% \uc815\ub3c4 \ub298\uc5c8\uc744\ub54c \ud06c\ub808\ud0c0\ub294 53% \uc99d\uac00\ud588\ub2e4. \uc624\ub298\ub0a0 \uc774 \uc12c\uc758 \uad00\uad11 \uc778\ud504\ub77c\ub294 \ubaa8\ub4e0 \ucde8\ud5a5\uc5d0 \ub9de\uac8c \uc900\ube44\ub418\uc5b4 \uc788\ub2e4. \uc774 \uc12c\uc5d0\ub294 \uc644\ubcbd\ud55c \uc2dc\uc124\uc744 \uac16\ucd98 \ud070 \ud638\ud654 \ud638\ud154, \uc218\uc601\uc7a5, \uc2a4\ud3ec\uce20\uc640 \ub808\ud06c\ub9ac\uc5d0\uc774\uc158, \uc791\uc740 \uac00\uc871 \uc18c\uc720 \uc544\ud30c\ud2b8, \ucea0\ud551 \uc2dc\uc124 \ub4f1\uc774 \uc788\ub2e4. \ubc29\ubb38\uc790\ub4e4\uc740 \uc774\ub77c\ud074\ub9ac\uc624\uc640 \ud558\ub2c8\uc544\uc758 \ub450 \uad6d\uc81c \uacf5\ud56d\uc774\ub098 \uc774\ub77c\ud074\ub9ac\uc624, \ud558\ub2c8\uc544, \ub808\ud300\ub178, \uc544\uc694\uc2a4 \ub2c8\ucf5c\ub77c\uc624\uc2a4\uc758 \ud56d\uad6c\ub97c \ud1b5\ud574 \uc12c\uc5d0 \uc624\uac08 \uc218 \uc788\ub2e4.", "sentence_answer": "\uadf8\ub9ac\uc2a4\uc5d0\uc11c \ud06c\ub808\ud0c0\ub85c \uc624\ub294 \uc0ac\ub78c \uc911 15%\uac00 \uc774\ub77c\ud074\ub9ac\uc624\uc758 \ud56d\uad6c\uc640 \uacf5\ud56d\uc744 \ud1b5\ud574 \uc624\uba70, \uadf8\ub9ac\uc2a4\uc758 \ubaa8\ub4e0 \ud56d\uacf5 \ud3b8\uc218 \uc911 20% \uac00 \uc774\ub77c\ud074\ub9ac\uc624\ub85c \uc624\ub294 \uac83\uc774\ub2e4.", "paragraph_id": "6397843-3-0", "paragraph_question": "question: \uadf8\ub9ac\uc2a4\uc758 \uc804\uccb4 \ud56d\uacf5 \ud3b8\uc218 \uc911 \uc774\ub77c\ud074\ub9ac\uc624\ub85c \uc624\ub294 \uac83\uc758 \ube44\uc728\uc740 \uc5bc\ub9c8\uc778\uac00?, context: \ud06c\ub808\ud0c0 \uc12c\uc740 \uadf8\ub9ac\uc2a4\uc5d0\uc11c \ud734\uac00\uc9c0\ub85c \uc778\uae30\uac00 \ub192\ub2e4. \uadf8\ub9ac\uc2a4\uc5d0\uc11c \ud06c\ub808\ud0c0\ub85c \uc624\ub294 \uc0ac\ub78c \uc911 15%\uac00 \uc774\ub77c\ud074\ub9ac\uc624\uc758 \ud56d\uad6c\uc640 \uacf5\ud56d\uc744 \ud1b5\ud574 \uc624\uba70, \uadf8\ub9ac\uc2a4\uc758 \ubaa8\ub4e0 \ud56d\uacf5 \ud3b8\uc218 \uc911 20%\uac00 \uc774\ub77c\ud074\ub9ac\uc624\ub85c \uc624\ub294 \uac83\uc774\ub2e4. \uc804\uccb4\uc801\uc73c\ub85c 2\ubc31\ub9cc\uba85 \uc774\uc0c1\uc758 \uad00\uad11\uac1d\uc774 \ud06c\ub808\ud0c0\ub97c \ubc29\ubb38\ud588\uc73c\uba70, 1986\ub144\uc5d0\uc11c 1991\ub144\uae4c\uc9c0 \ud638\ud154 \uc219\ubc15\uc774 \uadf8\ub9ac\uc2a4 \ub2e4\ub978 \uc9c0\uc5ed\uc5d0\uc11c 25% \uc815\ub3c4 \ub298\uc5c8\uc744\ub54c \ud06c\ub808\ud0c0\ub294 53% \uc99d\uac00\ud588\ub2e4. \uc624\ub298\ub0a0 \uc774 \uc12c\uc758 \uad00\uad11 \uc778\ud504\ub77c\ub294 \ubaa8\ub4e0 \ucde8\ud5a5\uc5d0 \ub9de\uac8c \uc900\ube44\ub418\uc5b4 \uc788\ub2e4. \uc774 \uc12c\uc5d0\ub294 \uc644\ubcbd\ud55c \uc2dc\uc124\uc744 \uac16\ucd98 \ud070 \ud638\ud654 \ud638\ud154, \uc218\uc601\uc7a5, \uc2a4\ud3ec\uce20\uc640 \ub808\ud06c\ub9ac\uc5d0\uc774\uc158, \uc791\uc740 \uac00\uc871 \uc18c\uc720 \uc544\ud30c\ud2b8, \ucea0\ud551 \uc2dc\uc124 \ub4f1\uc774 \uc788\ub2e4. \ubc29\ubb38\uc790\ub4e4\uc740 \uc774\ub77c\ud074\ub9ac\uc624\uc640 \ud558\ub2c8\uc544\uc758 \ub450 \uad6d\uc81c \uacf5\ud56d\uc774\ub098 \uc774\ub77c\ud074\ub9ac\uc624, \ud558\ub2c8\uc544, \ub808\ud300\ub178, \uc544\uc694\uc2a4 \ub2c8\ucf5c\ub77c\uc624\uc2a4\uc758 \ud56d\uad6c\ub97c \ud1b5\ud574 \uc12c\uc5d0 \uc624\uac08 \uc218 \uc788\ub2e4."} {"question": "1981\ub144 \ub3d9\uc548 \uce58\uc548\uc720\uc9c0 \uad70\uacbd\uc774 \ubc1c\uc0ac\ud55c \uace0\ubb34\ud0c4\uc758 \uac2f\uc218\ub294?", "paragraph": "\uc601\uad6d \uc5b8\ub860\ub4e4\uc740 \ub2e8\uc2dd\ud22c\uc7c1 \uacb0\uacfc \ub300\ucc98\uac00 \uc2b9\ub9ac\ud55c \uac83\ucc98\ub7fc \ub300\uc11c\ud2b9\ud544\ud588\ub2e4. \u300a\uac00\ub514\uc5b8\u300b\uc740 \u201c\uc815\ubd80\uac00 \uce68\ud574\ub418\uc9c0 \uc54a\uc744 \ub2e8\ud638\ud55c \uc758\uc9c0\ub97c \ud45c\ud604\ud568\uc73c\ub85c\uc368 \ub2e8\uc2dd\ud22c\uc7c1\uc744 \uadf9\ubcf5\ud588\ub2e4.\u201d\ub77c\uace0 \uc37c\ub2e4. \uadf8\ub7ec\ub098 \ub2e8\uc2dd\ud22c\uc7c1\uc740 \ub300\ucc98\uc640 \uc601\uad6d \uc815\ubd80\uc758 \uc785\uc7a5\uc5d0\uc11c\ub294 \ud53c\ub85c\uc2a4\uc758 \uc2b9\ub9ac\uc5d0 \ubd88\uacfc\ud588\ub2e4. \ub300\ucc98\ub294 \uc544\uc77c\ub79c\ub4dc \uacf5\ud654\uc8fc\uc758\uc790\ub4e4\uc5d0\uac8c \uc544\uc77c\ub79c\ub4dc\ub97c \uc815\ubcf5\ud55c \uc62c\ub9ac\ubc84 \ud06c\ub86c\uc6f0\uc5d0 \ub9de\uba39\ub294 \ub300\uc545\ub2f9\uc73c\ub85c \ub0a8\uac8c \ub418\uc5c8\ub2e4. \ub300\ub2c8 \ubaa8\ub9ac\uc2a8\uc740 \ub300\ub193\uace0 \uc721\ub450\ubb38\uc790\ub97c \uc368\uc11c \ub300\ucc98\ub97c \u201c\uc6b0\ub9ac\uac00 \uc544\ub294 \ucd5c\uc545\uc758 \uc30d\ub144bastard\u201d\uc774\ub77c\uace0 \ubd88\ub800\ub2e4. \ub2e8\uc2dd\ud22c\uc7c1\uc774 \uc9c4\ud589\ub418\ub358 \ub2f9\uc2dc\uc5d0\ub294 \ud22c\uc7c1\uc774 \uacf5\ud654\uc8fc\uc758\uc790\ub4e4\uc758 \uc644\ud328\ub85c \uc5ec\uaca8\uc84c\uace0, \uc774\uac83\uc774 IRA\uc640 \uc2e0\ud398\uc778 \ub0b4\ubd80\uc5d0\uc11c \uacf5\uc720\ub41c \uc2dc\uac01\uc774\uae30\ub3c4 \ud588\ub2e4. \uadf8\ub7ec\ub098 \uc0cc\uc988\uc758 \ubcf4\uad90\uc120\uac70 \uc625\uc911 \ucd9c\ub9c8\uc640 \ub2f9\uc120\uc740 \uc120\ub3d9\uc801 \uc2b9\ub9ac\uc600\ub2e4. 1971\ub144\uc758 \ub4dc\ubbf8\ud2b8\ub9ac\uc6b0\uc2a4 \uc791\uc804\uacfc 1972\ub144\uc758 \ud53c\uc758 \uc77c\uc694\uc77c \ub54c\uc640 \ub9c8\ucc2c\uac00\uc9c0\ub85c IRA\uc5d0 \uac00\uc785\ud558\ub294 \uc0ac\ub78c\uc758 \uc218\uac00 \ub298\uc5b4\ub0ac\uc73c\uba70, IRA\uc758 \uc900\uad70\uc0ac \uc900\ub3d9\uc774 \uc7ac\ucc28 \uae09\uc99d\ud558\ub294 \uacb0\uacfc\ub97c \ub0b3\uc558\ub2e4. \ube44\uad50\uc801 \uc870\uc6a9\ud588\ub358 1970\ub144\ub300 \ub9d0\ubcf4\ub2e4 \ud3ed\ub825\uc758 \ube48\ub3c4\uac00 \ub300\ud3ed \uc99d\uac00\ud588\uc73c\uba70, \ubd81\uc544\uc77c\ub79c\ub4dc \uacf3\uacf3\uc5d0\uc11c \uad11\ubc94\ud55c \uc18c\uc694\uac00 \uc77c\uc5b4\ub098\uace0 \ub354\ube14\ub9b0\uc758 \uc601\uad6d \ub300\uc0ac\uad00 \uc55e\uc5d0\uc11c\ub294 \ud3ed\ub3d9\uc774 \ub4a4\ub530\ub790\ub2e4. 1981\ub144 \ud55c \ud574 \ub3d9\uc548 \uce58\uc548\uc720\uc9c0 \uad70\uacbd\uc740 29,695\ubc1c\uc758 \uace0\ubb34\ud0c4\uc744 \ubc1c\uc0ac\ud588\uace0, \uadf8 \uacb0\uacfc 7\uba85\uc774 \uc0ac\ub9dd\ud588\ub2e4. \uadf8\ub9ac\uace0 \uadf8 \ub4a4\ub85c\ub3c4 8\ub144 \ub3d9\uc548 16,000\uc5ec \ubc1c\uc758 \uace0\ubb34\ud0c4\uc774 \ubc1c\uc0ac\ub418\uc5b4 4\uba85\uc744 \uc8fd\uac8c \ud558\uc600\ub2e4. IRA\ub294 \ub2e8\uc2dd\ud22c\uc7c1\uc774 \uc9c4\ud589\uc911\uc774\ub358 7\uac1c\uc6d4 \ub3d9\uc548 \ubb34\uc7a5\ud22c\uc7c1\uc744 \uacc4\uc18d\ud588\uace0, \uc601\uad6d \uacbd\ucc30 13\uba85, \uad70\uc778 13\uba85(\uadf8\uc911 \uc5bc\uc2a4\ud130 \ubc29\uc704\uc5f0\ub300 \uc18c\uc18d\uc774 5\uba85), \ubbfc\uac04\uc778 5\uba85\uc744 \uc8fd\uc600\ub2e4. \uc774 7\uac1c\uc6d4\uc740 \ubd81\uc544\uc77c\ub79c\ub4dc \ubd84\uc7c1 30\ub144 \ub3d9\uc548 \uac00\uc7a5 \uc720\ud608\uc774 \ub0ad\uc790\ud55c \uae30\uac04 \uc911 \ud558\ub098\ub85c \ucd1d 61\uba85\uc774 \uc0ac\ub9dd\ud588\uace0 \uadf8\uc911 34\uba85\uc774 \ubbfc\uac04\uc778\uc774\uc5c8\ub2e4. 3\ub144 \ub4a4 IRA\ub294 \ub300\ucc98\uc5d0\uac8c \ubcf5\uc218\ud558\uae30 \uc704\ud574 \ubcf4\uc218\ub2f9 \ub300\ud68c\uac00 \uc5f4\ub9ac\ub358 \uadf8\ub79c\ub4dc\ud638\ud154\uc5d0 \uc2dc\ud55c\ud3ed\ud0c4\uc744 \uc7a5\uce58\ud574 \ud638\ud154\uc744 \ub0a0\ub824 \ubc84\ub838\uace0(\ube0c\ub77c\uc774\ud2bc \ud638\ud154 \ud3ed\ud30c) \uadf8 \uacb0\uacfc 5\uba85\uc774 \uc0ac\ub9dd\ud588\ub2e4. \uc774\ub54c \ub300\ucc98 \ubcf8\uc778\ub3c4 \uac04\uc2e0\ud788 \ubaa9\uc228\ub9cc \uac74\uc838 \ub2ec\uc544\ub0ac\ub2e4.", "answer": "29,695\ubc1c", "sentence": "1981\ub144 \ud55c \ud574 \ub3d9\uc548 \uce58\uc548\uc720\uc9c0 \uad70\uacbd\uc740 29,695\ubc1c \uc758 \uace0\ubb34\ud0c4\uc744 \ubc1c\uc0ac\ud588\uace0, \uadf8 \uacb0\uacfc 7\uba85\uc774 \uc0ac\ub9dd\ud588\ub2e4.", "paragraph_sentence": "\uc601\uad6d \uc5b8\ub860\ub4e4\uc740 \ub2e8\uc2dd\ud22c\uc7c1 \uacb0\uacfc \ub300\ucc98\uac00 \uc2b9\ub9ac\ud55c \uac83\ucc98\ub7fc \ub300\uc11c\ud2b9\ud544\ud588\ub2e4. \u300a\uac00\ub514\uc5b8\u300b\uc740 \u201c\uc815\ubd80\uac00 \uce68\ud574\ub418\uc9c0 \uc54a\uc744 \ub2e8\ud638\ud55c \uc758\uc9c0\ub97c \ud45c\ud604\ud568\uc73c\ub85c\uc368 \ub2e8\uc2dd\ud22c\uc7c1\uc744 \uadf9\ubcf5\ud588\ub2e4. \u201d\ub77c\uace0 \uc37c\ub2e4. \uadf8\ub7ec\ub098 \ub2e8\uc2dd\ud22c\uc7c1\uc740 \ub300\ucc98\uc640 \uc601\uad6d \uc815\ubd80\uc758 \uc785\uc7a5\uc5d0\uc11c\ub294 \ud53c\ub85c\uc2a4\uc758 \uc2b9\ub9ac\uc5d0 \ubd88\uacfc\ud588\ub2e4. \ub300\ucc98\ub294 \uc544\uc77c\ub79c\ub4dc \uacf5\ud654\uc8fc\uc758\uc790\ub4e4\uc5d0\uac8c \uc544\uc77c\ub79c\ub4dc\ub97c \uc815\ubcf5\ud55c \uc62c\ub9ac\ubc84 \ud06c\ub86c\uc6f0\uc5d0 \ub9de\uba39\ub294 \ub300\uc545\ub2f9\uc73c\ub85c \ub0a8\uac8c \ub418\uc5c8\ub2e4. \ub300\ub2c8 \ubaa8\ub9ac\uc2a8\uc740 \ub300\ub193\uace0 \uc721\ub450\ubb38\uc790\ub97c \uc368\uc11c \ub300\ucc98\ub97c \u201c\uc6b0\ub9ac\uac00 \uc544\ub294 \ucd5c\uc545\uc758 \uc30d\ub144bastard\u201d\uc774\ub77c\uace0 \ubd88\ub800\ub2e4. \ub2e8\uc2dd\ud22c\uc7c1\uc774 \uc9c4\ud589\ub418\ub358 \ub2f9\uc2dc\uc5d0\ub294 \ud22c\uc7c1\uc774 \uacf5\ud654\uc8fc\uc758\uc790\ub4e4\uc758 \uc644\ud328\ub85c \uc5ec\uaca8\uc84c\uace0, \uc774\uac83\uc774 IRA\uc640 \uc2e0\ud398\uc778 \ub0b4\ubd80\uc5d0\uc11c \uacf5\uc720\ub41c \uc2dc\uac01\uc774\uae30\ub3c4 \ud588\ub2e4. \uadf8\ub7ec\ub098 \uc0cc\uc988\uc758 \ubcf4\uad90\uc120\uac70 \uc625\uc911 \ucd9c\ub9c8\uc640 \ub2f9\uc120\uc740 \uc120\ub3d9\uc801 \uc2b9\ub9ac\uc600\ub2e4. 1971\ub144\uc758 \ub4dc\ubbf8\ud2b8\ub9ac\uc6b0\uc2a4 \uc791\uc804\uacfc 1972\ub144\uc758 \ud53c\uc758 \uc77c\uc694\uc77c \ub54c\uc640 \ub9c8\ucc2c\uac00\uc9c0\ub85c IRA\uc5d0 \uac00\uc785\ud558\ub294 \uc0ac\ub78c\uc758 \uc218\uac00 \ub298\uc5b4\ub0ac\uc73c\uba70, IRA\uc758 \uc900\uad70\uc0ac \uc900\ub3d9\uc774 \uc7ac\ucc28 \uae09\uc99d\ud558\ub294 \uacb0\uacfc\ub97c \ub0b3\uc558\ub2e4. \ube44\uad50\uc801 \uc870\uc6a9\ud588\ub358 1970\ub144\ub300 \ub9d0\ubcf4\ub2e4 \ud3ed\ub825\uc758 \ube48\ub3c4\uac00 \ub300\ud3ed \uc99d\uac00\ud588\uc73c\uba70, \ubd81\uc544\uc77c\ub79c\ub4dc \uacf3\uacf3\uc5d0\uc11c \uad11\ubc94\ud55c \uc18c\uc694\uac00 \uc77c\uc5b4\ub098\uace0 \ub354\ube14\ub9b0\uc758 \uc601\uad6d \ub300\uc0ac\uad00 \uc55e\uc5d0\uc11c\ub294 \ud3ed\ub3d9\uc774 \ub4a4\ub530\ub790\ub2e4. 1981\ub144 \ud55c \ud574 \ub3d9\uc548 \uce58\uc548\uc720\uc9c0 \uad70\uacbd\uc740 29,695\ubc1c \uc758 \uace0\ubb34\ud0c4\uc744 \ubc1c\uc0ac\ud588\uace0, \uadf8 \uacb0\uacfc 7\uba85\uc774 \uc0ac\ub9dd\ud588\ub2e4. \uadf8\ub9ac\uace0 \uadf8 \ub4a4\ub85c\ub3c4 8\ub144 \ub3d9\uc548 16,000\uc5ec \ubc1c\uc758 \uace0\ubb34\ud0c4\uc774 \ubc1c\uc0ac\ub418\uc5b4 4\uba85\uc744 \uc8fd\uac8c \ud558\uc600\ub2e4. IRA\ub294 \ub2e8\uc2dd\ud22c\uc7c1\uc774 \uc9c4\ud589\uc911\uc774\ub358 7\uac1c\uc6d4 \ub3d9\uc548 \ubb34\uc7a5\ud22c\uc7c1\uc744 \uacc4\uc18d\ud588\uace0, \uc601\uad6d \uacbd\ucc30 13\uba85, \uad70\uc778 13\uba85(\uadf8\uc911 \uc5bc\uc2a4\ud130 \ubc29\uc704\uc5f0\ub300 \uc18c\uc18d\uc774 5\uba85), \ubbfc\uac04\uc778 5\uba85\uc744 \uc8fd\uc600\ub2e4. \uc774 7\uac1c\uc6d4\uc740 \ubd81\uc544\uc77c\ub79c\ub4dc \ubd84\uc7c1 30\ub144 \ub3d9\uc548 \uac00\uc7a5 \uc720\ud608\uc774 \ub0ad\uc790\ud55c \uae30\uac04 \uc911 \ud558\ub098\ub85c \ucd1d 61\uba85\uc774 \uc0ac\ub9dd\ud588\uace0 \uadf8\uc911 34\uba85\uc774 \ubbfc\uac04\uc778\uc774\uc5c8\ub2e4. 3\ub144 \ub4a4 IRA\ub294 \ub300\ucc98\uc5d0\uac8c \ubcf5\uc218\ud558\uae30 \uc704\ud574 \ubcf4\uc218\ub2f9 \ub300\ud68c\uac00 \uc5f4\ub9ac\ub358 \uadf8\ub79c\ub4dc\ud638\ud154\uc5d0 \uc2dc\ud55c\ud3ed\ud0c4\uc744 \uc7a5\uce58\ud574 \ud638\ud154\uc744 \ub0a0\ub824 \ubc84\ub838\uace0(\ube0c\ub77c\uc774\ud2bc \ud638\ud154 \ud3ed\ud30c) \uadf8 \uacb0\uacfc 5\uba85\uc774 \uc0ac\ub9dd\ud588\ub2e4. \uc774\ub54c \ub300\ucc98 \ubcf8\uc778\ub3c4 \uac04\uc2e0\ud788 \ubaa9\uc228\ub9cc \uac74\uc838 \ub2ec\uc544\ub0ac\ub2e4.", "paragraph_answer": "\uc601\uad6d \uc5b8\ub860\ub4e4\uc740 \ub2e8\uc2dd\ud22c\uc7c1 \uacb0\uacfc \ub300\ucc98\uac00 \uc2b9\ub9ac\ud55c \uac83\ucc98\ub7fc \ub300\uc11c\ud2b9\ud544\ud588\ub2e4. \u300a\uac00\ub514\uc5b8\u300b\uc740 \u201c\uc815\ubd80\uac00 \uce68\ud574\ub418\uc9c0 \uc54a\uc744 \ub2e8\ud638\ud55c \uc758\uc9c0\ub97c \ud45c\ud604\ud568\uc73c\ub85c\uc368 \ub2e8\uc2dd\ud22c\uc7c1\uc744 \uadf9\ubcf5\ud588\ub2e4.\u201d\ub77c\uace0 \uc37c\ub2e4. \uadf8\ub7ec\ub098 \ub2e8\uc2dd\ud22c\uc7c1\uc740 \ub300\ucc98\uc640 \uc601\uad6d \uc815\ubd80\uc758 \uc785\uc7a5\uc5d0\uc11c\ub294 \ud53c\ub85c\uc2a4\uc758 \uc2b9\ub9ac\uc5d0 \ubd88\uacfc\ud588\ub2e4. \ub300\ucc98\ub294 \uc544\uc77c\ub79c\ub4dc \uacf5\ud654\uc8fc\uc758\uc790\ub4e4\uc5d0\uac8c \uc544\uc77c\ub79c\ub4dc\ub97c \uc815\ubcf5\ud55c \uc62c\ub9ac\ubc84 \ud06c\ub86c\uc6f0\uc5d0 \ub9de\uba39\ub294 \ub300\uc545\ub2f9\uc73c\ub85c \ub0a8\uac8c \ub418\uc5c8\ub2e4. \ub300\ub2c8 \ubaa8\ub9ac\uc2a8\uc740 \ub300\ub193\uace0 \uc721\ub450\ubb38\uc790\ub97c \uc368\uc11c \ub300\ucc98\ub97c \u201c\uc6b0\ub9ac\uac00 \uc544\ub294 \ucd5c\uc545\uc758 \uc30d\ub144bastard\u201d\uc774\ub77c\uace0 \ubd88\ub800\ub2e4. \ub2e8\uc2dd\ud22c\uc7c1\uc774 \uc9c4\ud589\ub418\ub358 \ub2f9\uc2dc\uc5d0\ub294 \ud22c\uc7c1\uc774 \uacf5\ud654\uc8fc\uc758\uc790\ub4e4\uc758 \uc644\ud328\ub85c \uc5ec\uaca8\uc84c\uace0, \uc774\uac83\uc774 IRA\uc640 \uc2e0\ud398\uc778 \ub0b4\ubd80\uc5d0\uc11c \uacf5\uc720\ub41c \uc2dc\uac01\uc774\uae30\ub3c4 \ud588\ub2e4. \uadf8\ub7ec\ub098 \uc0cc\uc988\uc758 \ubcf4\uad90\uc120\uac70 \uc625\uc911 \ucd9c\ub9c8\uc640 \ub2f9\uc120\uc740 \uc120\ub3d9\uc801 \uc2b9\ub9ac\uc600\ub2e4. 1971\ub144\uc758 \ub4dc\ubbf8\ud2b8\ub9ac\uc6b0\uc2a4 \uc791\uc804\uacfc 1972\ub144\uc758 \ud53c\uc758 \uc77c\uc694\uc77c \ub54c\uc640 \ub9c8\ucc2c\uac00\uc9c0\ub85c IRA\uc5d0 \uac00\uc785\ud558\ub294 \uc0ac\ub78c\uc758 \uc218\uac00 \ub298\uc5b4\ub0ac\uc73c\uba70, IRA\uc758 \uc900\uad70\uc0ac \uc900\ub3d9\uc774 \uc7ac\ucc28 \uae09\uc99d\ud558\ub294 \uacb0\uacfc\ub97c \ub0b3\uc558\ub2e4. \ube44\uad50\uc801 \uc870\uc6a9\ud588\ub358 1970\ub144\ub300 \ub9d0\ubcf4\ub2e4 \ud3ed\ub825\uc758 \ube48\ub3c4\uac00 \ub300\ud3ed \uc99d\uac00\ud588\uc73c\uba70, \ubd81\uc544\uc77c\ub79c\ub4dc \uacf3\uacf3\uc5d0\uc11c \uad11\ubc94\ud55c \uc18c\uc694\uac00 \uc77c\uc5b4\ub098\uace0 \ub354\ube14\ub9b0\uc758 \uc601\uad6d \ub300\uc0ac\uad00 \uc55e\uc5d0\uc11c\ub294 \ud3ed\ub3d9\uc774 \ub4a4\ub530\ub790\ub2e4. 1981\ub144 \ud55c \ud574 \ub3d9\uc548 \uce58\uc548\uc720\uc9c0 \uad70\uacbd\uc740 29,695\ubc1c \uc758 \uace0\ubb34\ud0c4\uc744 \ubc1c\uc0ac\ud588\uace0, \uadf8 \uacb0\uacfc 7\uba85\uc774 \uc0ac\ub9dd\ud588\ub2e4. \uadf8\ub9ac\uace0 \uadf8 \ub4a4\ub85c\ub3c4 8\ub144 \ub3d9\uc548 16,000\uc5ec \ubc1c\uc758 \uace0\ubb34\ud0c4\uc774 \ubc1c\uc0ac\ub418\uc5b4 4\uba85\uc744 \uc8fd\uac8c \ud558\uc600\ub2e4. IRA\ub294 \ub2e8\uc2dd\ud22c\uc7c1\uc774 \uc9c4\ud589\uc911\uc774\ub358 7\uac1c\uc6d4 \ub3d9\uc548 \ubb34\uc7a5\ud22c\uc7c1\uc744 \uacc4\uc18d\ud588\uace0, \uc601\uad6d \uacbd\ucc30 13\uba85, \uad70\uc778 13\uba85(\uadf8\uc911 \uc5bc\uc2a4\ud130 \ubc29\uc704\uc5f0\ub300 \uc18c\uc18d\uc774 5\uba85), \ubbfc\uac04\uc778 5\uba85\uc744 \uc8fd\uc600\ub2e4. \uc774 7\uac1c\uc6d4\uc740 \ubd81\uc544\uc77c\ub79c\ub4dc \ubd84\uc7c1 30\ub144 \ub3d9\uc548 \uac00\uc7a5 \uc720\ud608\uc774 \ub0ad\uc790\ud55c \uae30\uac04 \uc911 \ud558\ub098\ub85c \ucd1d 61\uba85\uc774 \uc0ac\ub9dd\ud588\uace0 \uadf8\uc911 34\uba85\uc774 \ubbfc\uac04\uc778\uc774\uc5c8\ub2e4. 3\ub144 \ub4a4 IRA\ub294 \ub300\ucc98\uc5d0\uac8c \ubcf5\uc218\ud558\uae30 \uc704\ud574 \ubcf4\uc218\ub2f9 \ub300\ud68c\uac00 \uc5f4\ub9ac\ub358 \uadf8\ub79c\ub4dc\ud638\ud154\uc5d0 \uc2dc\ud55c\ud3ed\ud0c4\uc744 \uc7a5\uce58\ud574 \ud638\ud154\uc744 \ub0a0\ub824 \ubc84\ub838\uace0(\ube0c\ub77c\uc774\ud2bc \ud638\ud154 \ud3ed\ud30c) \uadf8 \uacb0\uacfc 5\uba85\uc774 \uc0ac\ub9dd\ud588\ub2e4. \uc774\ub54c \ub300\ucc98 \ubcf8\uc778\ub3c4 \uac04\uc2e0\ud788 \ubaa9\uc228\ub9cc \uac74\uc838 \ub2ec\uc544\ub0ac\ub2e4.", "sentence_answer": "1981\ub144 \ud55c \ud574 \ub3d9\uc548 \uce58\uc548\uc720\uc9c0 \uad70\uacbd\uc740 29,695\ubc1c \uc758 \uace0\ubb34\ud0c4\uc744 \ubc1c\uc0ac\ud588\uace0, \uadf8 \uacb0\uacfc 7\uba85\uc774 \uc0ac\ub9dd\ud588\ub2e4.", "paragraph_id": "6585474-11-1", "paragraph_question": "question: 1981\ub144 \ub3d9\uc548 \uce58\uc548\uc720\uc9c0 \uad70\uacbd\uc774 \ubc1c\uc0ac\ud55c \uace0\ubb34\ud0c4\uc758 \uac2f\uc218\ub294?, context: \uc601\uad6d \uc5b8\ub860\ub4e4\uc740 \ub2e8\uc2dd\ud22c\uc7c1 \uacb0\uacfc \ub300\ucc98\uac00 \uc2b9\ub9ac\ud55c \uac83\ucc98\ub7fc \ub300\uc11c\ud2b9\ud544\ud588\ub2e4. \u300a\uac00\ub514\uc5b8\u300b\uc740 \u201c\uc815\ubd80\uac00 \uce68\ud574\ub418\uc9c0 \uc54a\uc744 \ub2e8\ud638\ud55c \uc758\uc9c0\ub97c \ud45c\ud604\ud568\uc73c\ub85c\uc368 \ub2e8\uc2dd\ud22c\uc7c1\uc744 \uadf9\ubcf5\ud588\ub2e4.\u201d\ub77c\uace0 \uc37c\ub2e4. \uadf8\ub7ec\ub098 \ub2e8\uc2dd\ud22c\uc7c1\uc740 \ub300\ucc98\uc640 \uc601\uad6d \uc815\ubd80\uc758 \uc785\uc7a5\uc5d0\uc11c\ub294 \ud53c\ub85c\uc2a4\uc758 \uc2b9\ub9ac\uc5d0 \ubd88\uacfc\ud588\ub2e4. \ub300\ucc98\ub294 \uc544\uc77c\ub79c\ub4dc \uacf5\ud654\uc8fc\uc758\uc790\ub4e4\uc5d0\uac8c \uc544\uc77c\ub79c\ub4dc\ub97c \uc815\ubcf5\ud55c \uc62c\ub9ac\ubc84 \ud06c\ub86c\uc6f0\uc5d0 \ub9de\uba39\ub294 \ub300\uc545\ub2f9\uc73c\ub85c \ub0a8\uac8c \ub418\uc5c8\ub2e4. \ub300\ub2c8 \ubaa8\ub9ac\uc2a8\uc740 \ub300\ub193\uace0 \uc721\ub450\ubb38\uc790\ub97c \uc368\uc11c \ub300\ucc98\ub97c \u201c\uc6b0\ub9ac\uac00 \uc544\ub294 \ucd5c\uc545\uc758 \uc30d\ub144bastard\u201d\uc774\ub77c\uace0 \ubd88\ub800\ub2e4. \ub2e8\uc2dd\ud22c\uc7c1\uc774 \uc9c4\ud589\ub418\ub358 \ub2f9\uc2dc\uc5d0\ub294 \ud22c\uc7c1\uc774 \uacf5\ud654\uc8fc\uc758\uc790\ub4e4\uc758 \uc644\ud328\ub85c \uc5ec\uaca8\uc84c\uace0, \uc774\uac83\uc774 IRA\uc640 \uc2e0\ud398\uc778 \ub0b4\ubd80\uc5d0\uc11c \uacf5\uc720\ub41c \uc2dc\uac01\uc774\uae30\ub3c4 \ud588\ub2e4. \uadf8\ub7ec\ub098 \uc0cc\uc988\uc758 \ubcf4\uad90\uc120\uac70 \uc625\uc911 \ucd9c\ub9c8\uc640 \ub2f9\uc120\uc740 \uc120\ub3d9\uc801 \uc2b9\ub9ac\uc600\ub2e4. 1971\ub144\uc758 \ub4dc\ubbf8\ud2b8\ub9ac\uc6b0\uc2a4 \uc791\uc804\uacfc 1972\ub144\uc758 \ud53c\uc758 \uc77c\uc694\uc77c \ub54c\uc640 \ub9c8\ucc2c\uac00\uc9c0\ub85c IRA\uc5d0 \uac00\uc785\ud558\ub294 \uc0ac\ub78c\uc758 \uc218\uac00 \ub298\uc5b4\ub0ac\uc73c\uba70, IRA\uc758 \uc900\uad70\uc0ac \uc900\ub3d9\uc774 \uc7ac\ucc28 \uae09\uc99d\ud558\ub294 \uacb0\uacfc\ub97c \ub0b3\uc558\ub2e4. \ube44\uad50\uc801 \uc870\uc6a9\ud588\ub358 1970\ub144\ub300 \ub9d0\ubcf4\ub2e4 \ud3ed\ub825\uc758 \ube48\ub3c4\uac00 \ub300\ud3ed \uc99d\uac00\ud588\uc73c\uba70, \ubd81\uc544\uc77c\ub79c\ub4dc \uacf3\uacf3\uc5d0\uc11c \uad11\ubc94\ud55c \uc18c\uc694\uac00 \uc77c\uc5b4\ub098\uace0 \ub354\ube14\ub9b0\uc758 \uc601\uad6d \ub300\uc0ac\uad00 \uc55e\uc5d0\uc11c\ub294 \ud3ed\ub3d9\uc774 \ub4a4\ub530\ub790\ub2e4. 1981\ub144 \ud55c \ud574 \ub3d9\uc548 \uce58\uc548\uc720\uc9c0 \uad70\uacbd\uc740 29,695\ubc1c\uc758 \uace0\ubb34\ud0c4\uc744 \ubc1c\uc0ac\ud588\uace0, \uadf8 \uacb0\uacfc 7\uba85\uc774 \uc0ac\ub9dd\ud588\ub2e4. \uadf8\ub9ac\uace0 \uadf8 \ub4a4\ub85c\ub3c4 8\ub144 \ub3d9\uc548 16,000\uc5ec \ubc1c\uc758 \uace0\ubb34\ud0c4\uc774 \ubc1c\uc0ac\ub418\uc5b4 4\uba85\uc744 \uc8fd\uac8c \ud558\uc600\ub2e4. IRA\ub294 \ub2e8\uc2dd\ud22c\uc7c1\uc774 \uc9c4\ud589\uc911\uc774\ub358 7\uac1c\uc6d4 \ub3d9\uc548 \ubb34\uc7a5\ud22c\uc7c1\uc744 \uacc4\uc18d\ud588\uace0, \uc601\uad6d \uacbd\ucc30 13\uba85, \uad70\uc778 13\uba85(\uadf8\uc911 \uc5bc\uc2a4\ud130 \ubc29\uc704\uc5f0\ub300 \uc18c\uc18d\uc774 5\uba85), \ubbfc\uac04\uc778 5\uba85\uc744 \uc8fd\uc600\ub2e4. \uc774 7\uac1c\uc6d4\uc740 \ubd81\uc544\uc77c\ub79c\ub4dc \ubd84\uc7c1 30\ub144 \ub3d9\uc548 \uac00\uc7a5 \uc720\ud608\uc774 \ub0ad\uc790\ud55c \uae30\uac04 \uc911 \ud558\ub098\ub85c \ucd1d 61\uba85\uc774 \uc0ac\ub9dd\ud588\uace0 \uadf8\uc911 34\uba85\uc774 \ubbfc\uac04\uc778\uc774\uc5c8\ub2e4. 3\ub144 \ub4a4 IRA\ub294 \ub300\ucc98\uc5d0\uac8c \ubcf5\uc218\ud558\uae30 \uc704\ud574 \ubcf4\uc218\ub2f9 \ub300\ud68c\uac00 \uc5f4\ub9ac\ub358 \uadf8\ub79c\ub4dc\ud638\ud154\uc5d0 \uc2dc\ud55c\ud3ed\ud0c4\uc744 \uc7a5\uce58\ud574 \ud638\ud154\uc744 \ub0a0\ub824 \ubc84\ub838\uace0(\ube0c\ub77c\uc774\ud2bc \ud638\ud154 \ud3ed\ud30c) \uadf8 \uacb0\uacfc 5\uba85\uc774 \uc0ac\ub9dd\ud588\ub2e4. \uc774\ub54c \ub300\ucc98 \ubcf8\uc778\ub3c4 \uac04\uc2e0\ud788 \ubaa9\uc228\ub9cc \uac74\uc838 \ub2ec\uc544\ub0ac\ub2e4."} {"question": "2017\ub144 8\uc6d4 30\uc77c \ud56d\uc18c\uc2ec\uc5d0\uc11c \uadf8\uac00 \ubc1b\uc740 \ud615\ub7c9\uc740 \uc5bc\ub9c8\uc778\uac00?", "paragraph": "2014\ub144 7\uc6d4 14\uc77c\uc5d0\ub294 \uac80\ucc30\uc774 \uc9d5\uc5ed 4\ub144\uacfc \uc790\uaca9\uc815\uc9c0 4\ub144\uc774 \uad6c\ud615\ub410\ub2e4. \uadf8\ub7ec\ub098 9\uc6d4 11\uc77c \uc11c\uc6b8\uc9c0\ubc95 21\uc7ac\ud310\ubd80\ub294 \uadf8\uc758 \uad6d\uac00\uc815\ubcf4\uc6d0\ubc95 \uc704\ubc18 \ud610\uc758\ub9cc \uc720\uc8c4\ub85c \uc778\uc815\ud558\uace0 \ub098\uba38\uc9c0\ub294 \ubb34\uc8c4\ub85c \ud310\uacb0, \uc9d5\uc5ed 2\ub1446\uc6d4\uc5d0 \uc9d1\ud589\uc720\uc608 4\ub144, \uc790\uaca9\uc815\uc9c0 3\ub144\uc744 \uc120\uace0\ud588\ub2e4. \uc7ac\ud310\ubd80\ub294 \uc6d0 \uc804 \uc6d0\uc7a5\uc758 \uad6d\uc815\uc6d0\ubc95 \uc704\ubc18\uc744 \uc778\uc815\ud558\uc600\uc73c\ub098, \uacf5\uc9c1\uc120\uac70\ubc95\uc704\ubc18 \ud610\uc758\uc5d0 \ub300\ud574\uc11c\ub294 \ubb34\uc8c4\ub97c \uc120\uace0\ud558\uc600\ub2e4. \uc11c\uc6b8\uace0\ub4f1\ubc95\uc6d0 \ud615\uc0ac6\ubd80\ub294 2015\ub144 2\uc6d4 9\uc77c \ud56d\uc18c\uc2ec \uc120\uace0 \uacf5\ud310\uc5d0\uc11c \uacf5\uc9c1\uc120\uac70\ubc95 \uc704\ubc18 \ud610\uc758\ub3c4 \uc720\uc8c4\ub85c \uc778\uc815\ud558\uc5ec \uc9d5\uc5ed 3\ub144, \uc790\uaca9\uc815\uc9c0 3\ub144\uc744 \uc120\uace0\ud558\uace0 \ubc95\uc815\uad6c\uc18d\ud558\uc600\ub2e4. \ub300\ubc95\uc6d0 \uc804\uc6d0\ud569\uc758\uccb4\ub294 2015\ub144 7\uc6d4 16\uc77c \uc6d0\uc2ec\uc774 \uc99d\uac70\ub2a5\ub825\uc5d0 \ub300\ud55c \ubc95\ub9ac\ub97c \uc624\ud574\ud558\uace0 \uc0ac\uc2e4\uad00\uacc4\ub97c \uc798\ubabb \ud310\ub2e8\ud55c \uc624\ub958\uac00 \uc788\ub2e4\uba70 \uc720\ubb34\uc8c4 \ud310\ub2e8 \uc5c6\uc774 \uc0ac\uac74\uc744 \ud30c\uae30\ud658\uc1a1\ud558\uc600\ub2e4. 2017\ub144 8\uc6d4 30\uc77c \ud56d\uc18c\uc2ec\uc5d0\uc11c \uad6d\uac00\uc815\ubcf4\uc6d0\ubc95 \uc704\ubc18\uacfc \uacf5\uc9c1\uc120\uac70\ubc95 \uc704\ubc18 \ud610\uc758\uac00 \uc778\uc815\ub418\uc5b4 \uc9d5\uc5ed 4\ub144\ud615\uc774 \uc120\uace0\ub418\uc5c8\ub2e4.", "answer": "\uc9d5\uc5ed 4\ub144\ud615", "sentence": "2017\ub144 8\uc6d4 30\uc77c \ud56d\uc18c\uc2ec\uc5d0\uc11c \uad6d\uac00\uc815\ubcf4\uc6d0\ubc95 \uc704\ubc18\uacfc \uacf5\uc9c1\uc120\uac70\ubc95 \uc704\ubc18 \ud610\uc758\uac00 \uc778\uc815\ub418\uc5b4 \uc9d5\uc5ed 4\ub144\ud615 \uc774 \uc120\uace0\ub418\uc5c8\ub2e4.", "paragraph_sentence": "2014\ub144 7\uc6d4 14\uc77c\uc5d0\ub294 \uac80\ucc30\uc774 \uc9d5\uc5ed 4\ub144\uacfc \uc790\uaca9\uc815\uc9c0 4\ub144\uc774 \uad6c\ud615\ub410\ub2e4. \uadf8\ub7ec\ub098 9\uc6d4 11\uc77c \uc11c\uc6b8\uc9c0\ubc95 21\uc7ac\ud310\ubd80\ub294 \uadf8\uc758 \uad6d\uac00\uc815\ubcf4\uc6d0\ubc95 \uc704\ubc18 \ud610\uc758\ub9cc \uc720\uc8c4\ub85c \uc778\uc815\ud558\uace0 \ub098\uba38\uc9c0\ub294 \ubb34\uc8c4\ub85c \ud310\uacb0, \uc9d5\uc5ed 2\ub1446\uc6d4\uc5d0 \uc9d1\ud589\uc720\uc608 4\ub144, \uc790\uaca9\uc815\uc9c0 3\ub144\uc744 \uc120\uace0\ud588\ub2e4. \uc7ac\ud310\ubd80\ub294 \uc6d0 \uc804 \uc6d0\uc7a5\uc758 \uad6d\uc815\uc6d0\ubc95 \uc704\ubc18\uc744 \uc778\uc815\ud558\uc600\uc73c\ub098, \uacf5\uc9c1\uc120\uac70\ubc95\uc704\ubc18 \ud610\uc758\uc5d0 \ub300\ud574\uc11c\ub294 \ubb34\uc8c4\ub97c \uc120\uace0\ud558\uc600\ub2e4. \uc11c\uc6b8\uace0\ub4f1\ubc95\uc6d0 \ud615\uc0ac6\ubd80\ub294 2015\ub144 2\uc6d4 9\uc77c \ud56d\uc18c\uc2ec \uc120\uace0 \uacf5\ud310\uc5d0\uc11c \uacf5\uc9c1\uc120\uac70\ubc95 \uc704\ubc18 \ud610\uc758\ub3c4 \uc720\uc8c4\ub85c \uc778\uc815\ud558\uc5ec \uc9d5\uc5ed 3\ub144, \uc790\uaca9\uc815\uc9c0 3\ub144\uc744 \uc120\uace0\ud558\uace0 \ubc95\uc815\uad6c\uc18d\ud558\uc600\ub2e4. \ub300\ubc95\uc6d0 \uc804\uc6d0\ud569\uc758\uccb4\ub294 2015\ub144 7\uc6d4 16\uc77c \uc6d0\uc2ec\uc774 \uc99d\uac70\ub2a5\ub825\uc5d0 \ub300\ud55c \ubc95\ub9ac\ub97c \uc624\ud574\ud558\uace0 \uc0ac\uc2e4\uad00\uacc4\ub97c \uc798\ubabb \ud310\ub2e8\ud55c \uc624\ub958\uac00 \uc788\ub2e4\uba70 \uc720\ubb34\uc8c4 \ud310\ub2e8 \uc5c6\uc774 \uc0ac\uac74\uc744 \ud30c\uae30\ud658\uc1a1\ud558\uc600\ub2e4. 2017\ub144 8\uc6d4 30\uc77c \ud56d\uc18c\uc2ec\uc5d0\uc11c \uad6d\uac00\uc815\ubcf4\uc6d0\ubc95 \uc704\ubc18\uacfc \uacf5\uc9c1\uc120\uac70\ubc95 \uc704\ubc18 \ud610\uc758\uac00 \uc778\uc815\ub418\uc5b4 \uc9d5\uc5ed 4\ub144\ud615 \uc774 \uc120\uace0\ub418\uc5c8\ub2e4. ", "paragraph_answer": "2014\ub144 7\uc6d4 14\uc77c\uc5d0\ub294 \uac80\ucc30\uc774 \uc9d5\uc5ed 4\ub144\uacfc \uc790\uaca9\uc815\uc9c0 4\ub144\uc774 \uad6c\ud615\ub410\ub2e4. \uadf8\ub7ec\ub098 9\uc6d4 11\uc77c \uc11c\uc6b8\uc9c0\ubc95 21\uc7ac\ud310\ubd80\ub294 \uadf8\uc758 \uad6d\uac00\uc815\ubcf4\uc6d0\ubc95 \uc704\ubc18 \ud610\uc758\ub9cc \uc720\uc8c4\ub85c \uc778\uc815\ud558\uace0 \ub098\uba38\uc9c0\ub294 \ubb34\uc8c4\ub85c \ud310\uacb0, \uc9d5\uc5ed 2\ub1446\uc6d4\uc5d0 \uc9d1\ud589\uc720\uc608 4\ub144, \uc790\uaca9\uc815\uc9c0 3\ub144\uc744 \uc120\uace0\ud588\ub2e4. \uc7ac\ud310\ubd80\ub294 \uc6d0 \uc804 \uc6d0\uc7a5\uc758 \uad6d\uc815\uc6d0\ubc95 \uc704\ubc18\uc744 \uc778\uc815\ud558\uc600\uc73c\ub098, \uacf5\uc9c1\uc120\uac70\ubc95\uc704\ubc18 \ud610\uc758\uc5d0 \ub300\ud574\uc11c\ub294 \ubb34\uc8c4\ub97c \uc120\uace0\ud558\uc600\ub2e4. \uc11c\uc6b8\uace0\ub4f1\ubc95\uc6d0 \ud615\uc0ac6\ubd80\ub294 2015\ub144 2\uc6d4 9\uc77c \ud56d\uc18c\uc2ec \uc120\uace0 \uacf5\ud310\uc5d0\uc11c \uacf5\uc9c1\uc120\uac70\ubc95 \uc704\ubc18 \ud610\uc758\ub3c4 \uc720\uc8c4\ub85c \uc778\uc815\ud558\uc5ec \uc9d5\uc5ed 3\ub144, \uc790\uaca9\uc815\uc9c0 3\ub144\uc744 \uc120\uace0\ud558\uace0 \ubc95\uc815\uad6c\uc18d\ud558\uc600\ub2e4. \ub300\ubc95\uc6d0 \uc804\uc6d0\ud569\uc758\uccb4\ub294 2015\ub144 7\uc6d4 16\uc77c \uc6d0\uc2ec\uc774 \uc99d\uac70\ub2a5\ub825\uc5d0 \ub300\ud55c \ubc95\ub9ac\ub97c \uc624\ud574\ud558\uace0 \uc0ac\uc2e4\uad00\uacc4\ub97c \uc798\ubabb \ud310\ub2e8\ud55c \uc624\ub958\uac00 \uc788\ub2e4\uba70 \uc720\ubb34\uc8c4 \ud310\ub2e8 \uc5c6\uc774 \uc0ac\uac74\uc744 \ud30c\uae30\ud658\uc1a1\ud558\uc600\ub2e4. 2017\ub144 8\uc6d4 30\uc77c \ud56d\uc18c\uc2ec\uc5d0\uc11c \uad6d\uac00\uc815\ubcf4\uc6d0\ubc95 \uc704\ubc18\uacfc \uacf5\uc9c1\uc120\uac70\ubc95 \uc704\ubc18 \ud610\uc758\uac00 \uc778\uc815\ub418\uc5b4 \uc9d5\uc5ed 4\ub144\ud615 \uc774 \uc120\uace0\ub418\uc5c8\ub2e4.", "sentence_answer": "2017\ub144 8\uc6d4 30\uc77c \ud56d\uc18c\uc2ec\uc5d0\uc11c \uad6d\uac00\uc815\ubcf4\uc6d0\ubc95 \uc704\ubc18\uacfc \uacf5\uc9c1\uc120\uac70\ubc95 \uc704\ubc18 \ud610\uc758\uac00 \uc778\uc815\ub418\uc5b4 \uc9d5\uc5ed 4\ub144\ud615 \uc774 \uc120\uace0\ub418\uc5c8\ub2e4.", "paragraph_id": "6588864-4-0", "paragraph_question": "question: 2017\ub144 8\uc6d4 30\uc77c \ud56d\uc18c\uc2ec\uc5d0\uc11c \uadf8\uac00 \ubc1b\uc740 \ud615\ub7c9\uc740 \uc5bc\ub9c8\uc778\uac00?, context: 2014\ub144 7\uc6d4 14\uc77c\uc5d0\ub294 \uac80\ucc30\uc774 \uc9d5\uc5ed 4\ub144\uacfc \uc790\uaca9\uc815\uc9c0 4\ub144\uc774 \uad6c\ud615\ub410\ub2e4. \uadf8\ub7ec\ub098 9\uc6d4 11\uc77c \uc11c\uc6b8\uc9c0\ubc95 21\uc7ac\ud310\ubd80\ub294 \uadf8\uc758 \uad6d\uac00\uc815\ubcf4\uc6d0\ubc95 \uc704\ubc18 \ud610\uc758\ub9cc \uc720\uc8c4\ub85c \uc778\uc815\ud558\uace0 \ub098\uba38\uc9c0\ub294 \ubb34\uc8c4\ub85c \ud310\uacb0, \uc9d5\uc5ed 2\ub1446\uc6d4\uc5d0 \uc9d1\ud589\uc720\uc608 4\ub144, \uc790\uaca9\uc815\uc9c0 3\ub144\uc744 \uc120\uace0\ud588\ub2e4. \uc7ac\ud310\ubd80\ub294 \uc6d0 \uc804 \uc6d0\uc7a5\uc758 \uad6d\uc815\uc6d0\ubc95 \uc704\ubc18\uc744 \uc778\uc815\ud558\uc600\uc73c\ub098, \uacf5\uc9c1\uc120\uac70\ubc95\uc704\ubc18 \ud610\uc758\uc5d0 \ub300\ud574\uc11c\ub294 \ubb34\uc8c4\ub97c \uc120\uace0\ud558\uc600\ub2e4. \uc11c\uc6b8\uace0\ub4f1\ubc95\uc6d0 \ud615\uc0ac6\ubd80\ub294 2015\ub144 2\uc6d4 9\uc77c \ud56d\uc18c\uc2ec \uc120\uace0 \uacf5\ud310\uc5d0\uc11c \uacf5\uc9c1\uc120\uac70\ubc95 \uc704\ubc18 \ud610\uc758\ub3c4 \uc720\uc8c4\ub85c \uc778\uc815\ud558\uc5ec \uc9d5\uc5ed 3\ub144, \uc790\uaca9\uc815\uc9c0 3\ub144\uc744 \uc120\uace0\ud558\uace0 \ubc95\uc815\uad6c\uc18d\ud558\uc600\ub2e4. \ub300\ubc95\uc6d0 \uc804\uc6d0\ud569\uc758\uccb4\ub294 2015\ub144 7\uc6d4 16\uc77c \uc6d0\uc2ec\uc774 \uc99d\uac70\ub2a5\ub825\uc5d0 \ub300\ud55c \ubc95\ub9ac\ub97c \uc624\ud574\ud558\uace0 \uc0ac\uc2e4\uad00\uacc4\ub97c \uc798\ubabb \ud310\ub2e8\ud55c \uc624\ub958\uac00 \uc788\ub2e4\uba70 \uc720\ubb34\uc8c4 \ud310\ub2e8 \uc5c6\uc774 \uc0ac\uac74\uc744 \ud30c\uae30\ud658\uc1a1\ud558\uc600\ub2e4. 2017\ub144 8\uc6d4 30\uc77c \ud56d\uc18c\uc2ec\uc5d0\uc11c \uad6d\uac00\uc815\ubcf4\uc6d0\ubc95 \uc704\ubc18\uacfc \uacf5\uc9c1\uc120\uac70\ubc95 \uc704\ubc18 \ud610\uc758\uac00 \uc778\uc815\ub418\uc5b4 \uc9d5\uc5ed 4\ub144\ud615\uc774 \uc120\uace0\ub418\uc5c8\ub2e4."} {"question": "BBK \uc218\uc0ac\uac80\uc0ac\ub4e4\uc774 \uc815\ubd09\uc8fc\ub97c \uc0c1\ub300\ub85c \ub0b8 \uc190\ud574\ubc30\uc0c1 \uccad\uad6c\uc18c\uc1a1\uc758 \uae08\uc561\uc740?", "paragraph": "2007\ub144 12\uc6d4, \ub300\ud1b5\ud569\ubbfc\uc8fc\uc2e0\ub2f9 \uc18c\uc18d\uc758 \uad6d\ud68c\uc758\uc6d0 \uc815\ubd09\uc8fc\ub294 \uae30\uc790\ud68c\uacac \ub4f1\uc744 \ud1b5\ud574, \"\uac80\ucc30\uc774 BBK\uac00 \uc774\uba85\ubc15 \ud6c4\ubcf4\uc758 \uac83\uc774\ub77c\ub294 \uae40\uc528\uc758 \uc790\ud544 \uba54\ubaa8\ub97c \uc218\uc0ac\uacfc\uc815\uc5d0 \ub204\ub77d\ud588\ub2e4. \uc9dc\ub9de\ucd94\uae30\uc2dd \ubd80\uc2e4 \uc218\uc0ac\ub2e4\"\ub77c\uace0 \ube44\ud310\ud588\ub2e4. \uc774\uc5d0 2007\ub144 \ub300\uc120 \ubb34\ub835 `BBK \uc0ac\uac74'\uc744 \uc218\uc0ac\ud588\ub358 \uac80\uc0ac\ub4e4\uc740 \uc815\ubd09\uc8fc\ub97c \uc0c1\ub300\ub85c 2\uc5b5 8000\ub9cc \uc6d0\uc758 \uc190\ud574\ubc30\uc0c1 \uccad\uad6c\uc18c\uc1a1\uc744 \ud55c\ud3b8 \ud5c8\uc704\uc0ac\uc2e4\uc744 \uc720\ud3ec\ud55c \ud610\uc758(\uacf5\uc9c1\uc120\uac70\ubc95 \uc704\ubc18 \ub4f1)\ub85c \uae30\uc18c\ud558\uc600\ub2e4. \uc190\ud574\ubc30\uc0c1 \uccad\uad6c\uc5d0 \ub300\ud574\uc11c\ub294 1\uc2ec\uc740 1600\ub9cc \uc6d0\uc744 \ubc30\uc0c1\ud558\ub3c4\ub85d \uc6d0\uace0 \uc77c\ubd80 \uc2b9\uc18c \ud310\uacb0\uc744 \ub0b4\ub838\uace0, \ud56d\uc18c\uc2ec\uc740 \uc6d0\uace0 \ud328\uc18c \ud310\uacb0\uc744 \ub0b4\ub838\ub2e4. 2011\ub144 4\uc6d4 26\uc77c, \ud56d\uc18c\uc2ec \uc7ac\ud310\ubd80\ub294 \"\uad6d\ud68c\uc758\uc6d0\uc73c\ub85c\uc11c \uac80\ucc30 \uc218\uc0ac \uacb0\uacfc\ub97c \uac10\uc2dc\u00b7\ube44\ud310\ud558\ub294 \uac83\uc740 \uc815\ub2f9\ud55c \uc5c5\ubb34 \ud65c\ub3d9\uc774\ub2e4. \uae40\uc528\uc758 \uba54\ubaa8 \ub4f1 \uc5b4\ub290 \uc815\ub3c4 \uadfc\uac70\ub97c \uac00\uc9c0\uace0 \uc758\ubb38\uc744 \uc81c\uae30\ud588\ub358 \ub9cc\ud07c \ud604\uc800\ud788 \uc0c1\ub2f9\uc131\uc744 \uc783\uc740 \uac83\uc73c\ub85c \ud3c9\uac00\ub418\uc9c0 \uc54a\ub294\ub2e4\"\ub77c\uace0 \ud310\ub2e8\ud588\ub2e4. \uadf8\ub7ec\ub098 \uacf5\uc9c1\uc120\uac70\ubc95 \uc704\ubc18 \ubc0f \ud5c8\uc704\uc0ac\uc2e4 \uc720\ud3ec \ub4f1\uc758 \ud610\uc758\uc5d0 \ub300\ud558\uc5ec\uc11c\ub294 1\uc2ec \ubc0f \ud56d\uc18c\uc2ec\uc5d0\uc11c \uc9d5\uc5ed 1\ub144\uc758 \uc2e4\ud615\uc774 \uc120\uace0\ub418\uc5c8\ub2e4. 2011\ub144 12\uc6d4 22\uc77c, \ub300\ubc95\uc6d0 2\ubd80(\uc8fc\uc2ec \uc774\uc0c1\ud6c8 \ub300\ubc95\uad00)\ub294 \uc815\ubd09\uc8fc\uc5d0\uac8c \uc9d5\uc5ed 1\ub144 \uc2e4\ud615\uc744 \uc120\uace0\ud55c \uc6d0\uc2ec\uc744 \ud655\uc815\ud558\uc600\uc73c\uba70 \uc774\ub85c \uc778\ud574, \uc815\ubd09\uc8fc\ub294 [\uacf5\uc9c1\uc120\uac70\ubc95]\uc81c18\uc870(\uc120\uac70\uad8c\uc774 \uc5c6\ub294\uc790) \ubc0f \uc81c19\uc870(\ud53c\uc120\uac70\uad8c\uc774 \uc5c6\ub294\uc790)\uc5d0 \uc758\uac70 10\ub144\uac04 \ud53c\uc120\uac70\uad8c\uc744 \ubc15\ud0c8\ub2f9\ud588\ub2e4.", "answer": "2\uc5b5 8000\ub9cc \uc6d0", "sentence": "\uc774\uc5d0 2007\ub144 \ub300\uc120 \ubb34\ub835 `BBK \uc0ac\uac74'\uc744 \uc218\uc0ac\ud588\ub358 \uac80\uc0ac\ub4e4\uc740 \uc815\ubd09\uc8fc\ub97c \uc0c1\ub300\ub85c 2\uc5b5 8000\ub9cc \uc6d0 \uc758 \uc190\ud574\ubc30\uc0c1 \uccad\uad6c\uc18c\uc1a1\uc744 \ud55c\ud3b8 \ud5c8\uc704\uc0ac\uc2e4\uc744 \uc720\ud3ec\ud55c \ud610\uc758(\uacf5\uc9c1\uc120\uac70\ubc95 \uc704\ubc18 \ub4f1)\ub85c \uae30\uc18c\ud558\uc600\ub2e4.", "paragraph_sentence": "2007\ub144 12\uc6d4, \ub300\ud1b5\ud569\ubbfc\uc8fc\uc2e0\ub2f9 \uc18c\uc18d\uc758 \uad6d\ud68c\uc758\uc6d0 \uc815\ubd09\uc8fc\ub294 \uae30\uc790\ud68c\uacac \ub4f1\uc744 \ud1b5\ud574, \"\uac80\ucc30\uc774 BBK\uac00 \uc774\uba85\ubc15 \ud6c4\ubcf4\uc758 \uac83\uc774\ub77c\ub294 \uae40\uc528\uc758 \uc790\ud544 \uba54\ubaa8\ub97c \uc218\uc0ac\uacfc\uc815\uc5d0 \ub204\ub77d\ud588\ub2e4. \uc9dc\ub9de\ucd94\uae30\uc2dd \ubd80\uc2e4 \uc218\uc0ac\ub2e4\"\ub77c\uace0 \ube44\ud310\ud588\ub2e4. \uc774\uc5d0 2007\ub144 \ub300\uc120 \ubb34\ub835 `BBK \uc0ac\uac74'\uc744 \uc218\uc0ac\ud588\ub358 \uac80\uc0ac\ub4e4\uc740 \uc815\ubd09\uc8fc\ub97c \uc0c1\ub300\ub85c 2\uc5b5 8000\ub9cc \uc6d0 \uc758 \uc190\ud574\ubc30\uc0c1 \uccad\uad6c\uc18c\uc1a1\uc744 \ud55c\ud3b8 \ud5c8\uc704\uc0ac\uc2e4\uc744 \uc720\ud3ec\ud55c \ud610\uc758(\uacf5\uc9c1\uc120\uac70\ubc95 \uc704\ubc18 \ub4f1)\ub85c \uae30\uc18c\ud558\uc600\ub2e4. \uc190\ud574\ubc30\uc0c1 \uccad\uad6c\uc5d0 \ub300\ud574\uc11c\ub294 1\uc2ec\uc740 1600\ub9cc \uc6d0\uc744 \ubc30\uc0c1\ud558\ub3c4\ub85d \uc6d0\uace0 \uc77c\ubd80 \uc2b9\uc18c \ud310\uacb0\uc744 \ub0b4\ub838\uace0, \ud56d\uc18c\uc2ec\uc740 \uc6d0\uace0 \ud328\uc18c \ud310\uacb0\uc744 \ub0b4\ub838\ub2e4. 2011\ub144 4\uc6d4 26\uc77c, \ud56d\uc18c\uc2ec \uc7ac\ud310\ubd80\ub294 \"\uad6d\ud68c\uc758\uc6d0\uc73c\ub85c\uc11c \uac80\ucc30 \uc218\uc0ac \uacb0\uacfc\ub97c \uac10\uc2dc\u00b7\ube44\ud310\ud558\ub294 \uac83\uc740 \uc815\ub2f9\ud55c \uc5c5\ubb34 \ud65c\ub3d9\uc774\ub2e4. \uae40\uc528\uc758 \uba54\ubaa8 \ub4f1 \uc5b4\ub290 \uc815\ub3c4 \uadfc\uac70\ub97c \uac00\uc9c0\uace0 \uc758\ubb38\uc744 \uc81c\uae30\ud588\ub358 \ub9cc\ud07c \ud604\uc800\ud788 \uc0c1\ub2f9\uc131\uc744 \uc783\uc740 \uac83\uc73c\ub85c \ud3c9\uac00\ub418\uc9c0 \uc54a\ub294\ub2e4\"\ub77c\uace0 \ud310\ub2e8\ud588\ub2e4. \uadf8\ub7ec\ub098 \uacf5\uc9c1\uc120\uac70\ubc95 \uc704\ubc18 \ubc0f \ud5c8\uc704\uc0ac\uc2e4 \uc720\ud3ec \ub4f1\uc758 \ud610\uc758\uc5d0 \ub300\ud558\uc5ec\uc11c\ub294 1\uc2ec \ubc0f \ud56d\uc18c\uc2ec\uc5d0\uc11c \uc9d5\uc5ed 1\ub144\uc758 \uc2e4\ud615\uc774 \uc120\uace0\ub418\uc5c8\ub2e4. 2011\ub144 12\uc6d4 22\uc77c, \ub300\ubc95\uc6d0 2\ubd80(\uc8fc\uc2ec \uc774\uc0c1\ud6c8 \ub300\ubc95\uad00)\ub294 \uc815\ubd09\uc8fc\uc5d0\uac8c \uc9d5\uc5ed 1\ub144 \uc2e4\ud615\uc744 \uc120\uace0\ud55c \uc6d0\uc2ec\uc744 \ud655\uc815\ud558\uc600\uc73c\uba70 \uc774\ub85c \uc778\ud574, \uc815\ubd09\uc8fc\ub294 [\uacf5\uc9c1\uc120\uac70\ubc95]\uc81c18\uc870(\uc120\uac70\uad8c\uc774 \uc5c6\ub294\uc790) \ubc0f \uc81c19\uc870(\ud53c\uc120\uac70\uad8c\uc774 \uc5c6\ub294\uc790)\uc5d0 \uc758\uac70 10\ub144\uac04 \ud53c\uc120\uac70\uad8c\uc744 \ubc15\ud0c8\ub2f9\ud588\ub2e4.", "paragraph_answer": "2007\ub144 12\uc6d4, \ub300\ud1b5\ud569\ubbfc\uc8fc\uc2e0\ub2f9 \uc18c\uc18d\uc758 \uad6d\ud68c\uc758\uc6d0 \uc815\ubd09\uc8fc\ub294 \uae30\uc790\ud68c\uacac \ub4f1\uc744 \ud1b5\ud574, \"\uac80\ucc30\uc774 BBK\uac00 \uc774\uba85\ubc15 \ud6c4\ubcf4\uc758 \uac83\uc774\ub77c\ub294 \uae40\uc528\uc758 \uc790\ud544 \uba54\ubaa8\ub97c \uc218\uc0ac\uacfc\uc815\uc5d0 \ub204\ub77d\ud588\ub2e4. \uc9dc\ub9de\ucd94\uae30\uc2dd \ubd80\uc2e4 \uc218\uc0ac\ub2e4\"\ub77c\uace0 \ube44\ud310\ud588\ub2e4. \uc774\uc5d0 2007\ub144 \ub300\uc120 \ubb34\ub835 `BBK \uc0ac\uac74'\uc744 \uc218\uc0ac\ud588\ub358 \uac80\uc0ac\ub4e4\uc740 \uc815\ubd09\uc8fc\ub97c \uc0c1\ub300\ub85c 2\uc5b5 8000\ub9cc \uc6d0 \uc758 \uc190\ud574\ubc30\uc0c1 \uccad\uad6c\uc18c\uc1a1\uc744 \ud55c\ud3b8 \ud5c8\uc704\uc0ac\uc2e4\uc744 \uc720\ud3ec\ud55c \ud610\uc758(\uacf5\uc9c1\uc120\uac70\ubc95 \uc704\ubc18 \ub4f1)\ub85c \uae30\uc18c\ud558\uc600\ub2e4. \uc190\ud574\ubc30\uc0c1 \uccad\uad6c\uc5d0 \ub300\ud574\uc11c\ub294 1\uc2ec\uc740 1600\ub9cc \uc6d0\uc744 \ubc30\uc0c1\ud558\ub3c4\ub85d \uc6d0\uace0 \uc77c\ubd80 \uc2b9\uc18c \ud310\uacb0\uc744 \ub0b4\ub838\uace0, \ud56d\uc18c\uc2ec\uc740 \uc6d0\uace0 \ud328\uc18c \ud310\uacb0\uc744 \ub0b4\ub838\ub2e4. 2011\ub144 4\uc6d4 26\uc77c, \ud56d\uc18c\uc2ec \uc7ac\ud310\ubd80\ub294 \"\uad6d\ud68c\uc758\uc6d0\uc73c\ub85c\uc11c \uac80\ucc30 \uc218\uc0ac \uacb0\uacfc\ub97c \uac10\uc2dc\u00b7\ube44\ud310\ud558\ub294 \uac83\uc740 \uc815\ub2f9\ud55c \uc5c5\ubb34 \ud65c\ub3d9\uc774\ub2e4. \uae40\uc528\uc758 \uba54\ubaa8 \ub4f1 \uc5b4\ub290 \uc815\ub3c4 \uadfc\uac70\ub97c \uac00\uc9c0\uace0 \uc758\ubb38\uc744 \uc81c\uae30\ud588\ub358 \ub9cc\ud07c \ud604\uc800\ud788 \uc0c1\ub2f9\uc131\uc744 \uc783\uc740 \uac83\uc73c\ub85c \ud3c9\uac00\ub418\uc9c0 \uc54a\ub294\ub2e4\"\ub77c\uace0 \ud310\ub2e8\ud588\ub2e4. \uadf8\ub7ec\ub098 \uacf5\uc9c1\uc120\uac70\ubc95 \uc704\ubc18 \ubc0f \ud5c8\uc704\uc0ac\uc2e4 \uc720\ud3ec \ub4f1\uc758 \ud610\uc758\uc5d0 \ub300\ud558\uc5ec\uc11c\ub294 1\uc2ec \ubc0f \ud56d\uc18c\uc2ec\uc5d0\uc11c \uc9d5\uc5ed 1\ub144\uc758 \uc2e4\ud615\uc774 \uc120\uace0\ub418\uc5c8\ub2e4. 2011\ub144 12\uc6d4 22\uc77c, \ub300\ubc95\uc6d0 2\ubd80(\uc8fc\uc2ec \uc774\uc0c1\ud6c8 \ub300\ubc95\uad00)\ub294 \uc815\ubd09\uc8fc\uc5d0\uac8c \uc9d5\uc5ed 1\ub144 \uc2e4\ud615\uc744 \uc120\uace0\ud55c \uc6d0\uc2ec\uc744 \ud655\uc815\ud558\uc600\uc73c\uba70 \uc774\ub85c \uc778\ud574, \uc815\ubd09\uc8fc\ub294 [\uacf5\uc9c1\uc120\uac70\ubc95]\uc81c18\uc870(\uc120\uac70\uad8c\uc774 \uc5c6\ub294\uc790) \ubc0f \uc81c19\uc870(\ud53c\uc120\uac70\uad8c\uc774 \uc5c6\ub294\uc790)\uc5d0 \uc758\uac70 10\ub144\uac04 \ud53c\uc120\uac70\uad8c\uc744 \ubc15\ud0c8\ub2f9\ud588\ub2e4.", "sentence_answer": "\uc774\uc5d0 2007\ub144 \ub300\uc120 \ubb34\ub835 `BBK \uc0ac\uac74'\uc744 \uc218\uc0ac\ud588\ub358 \uac80\uc0ac\ub4e4\uc740 \uc815\ubd09\uc8fc\ub97c \uc0c1\ub300\ub85c 2\uc5b5 8000\ub9cc \uc6d0 \uc758 \uc190\ud574\ubc30\uc0c1 \uccad\uad6c\uc18c\uc1a1\uc744 \ud55c\ud3b8 \ud5c8\uc704\uc0ac\uc2e4\uc744 \uc720\ud3ec\ud55c \ud610\uc758(\uacf5\uc9c1\uc120\uac70\ubc95 \uc704\ubc18 \ub4f1)\ub85c \uae30\uc18c\ud558\uc600\ub2e4.", "paragraph_id": "6500461-5-1", "paragraph_question": "question: BBK \uc218\uc0ac\uac80\uc0ac\ub4e4\uc774 \uc815\ubd09\uc8fc\ub97c \uc0c1\ub300\ub85c \ub0b8 \uc190\ud574\ubc30\uc0c1 \uccad\uad6c\uc18c\uc1a1\uc758 \uae08\uc561\uc740?, context: 2007\ub144 12\uc6d4, \ub300\ud1b5\ud569\ubbfc\uc8fc\uc2e0\ub2f9 \uc18c\uc18d\uc758 \uad6d\ud68c\uc758\uc6d0 \uc815\ubd09\uc8fc\ub294 \uae30\uc790\ud68c\uacac \ub4f1\uc744 \ud1b5\ud574, \"\uac80\ucc30\uc774 BBK\uac00 \uc774\uba85\ubc15 \ud6c4\ubcf4\uc758 \uac83\uc774\ub77c\ub294 \uae40\uc528\uc758 \uc790\ud544 \uba54\ubaa8\ub97c \uc218\uc0ac\uacfc\uc815\uc5d0 \ub204\ub77d\ud588\ub2e4. \uc9dc\ub9de\ucd94\uae30\uc2dd \ubd80\uc2e4 \uc218\uc0ac\ub2e4\"\ub77c\uace0 \ube44\ud310\ud588\ub2e4. \uc774\uc5d0 2007\ub144 \ub300\uc120 \ubb34\ub835 `BBK \uc0ac\uac74'\uc744 \uc218\uc0ac\ud588\ub358 \uac80\uc0ac\ub4e4\uc740 \uc815\ubd09\uc8fc\ub97c \uc0c1\ub300\ub85c 2\uc5b5 8000\ub9cc \uc6d0\uc758 \uc190\ud574\ubc30\uc0c1 \uccad\uad6c\uc18c\uc1a1\uc744 \ud55c\ud3b8 \ud5c8\uc704\uc0ac\uc2e4\uc744 \uc720\ud3ec\ud55c \ud610\uc758(\uacf5\uc9c1\uc120\uac70\ubc95 \uc704\ubc18 \ub4f1)\ub85c \uae30\uc18c\ud558\uc600\ub2e4. \uc190\ud574\ubc30\uc0c1 \uccad\uad6c\uc5d0 \ub300\ud574\uc11c\ub294 1\uc2ec\uc740 1600\ub9cc \uc6d0\uc744 \ubc30\uc0c1\ud558\ub3c4\ub85d \uc6d0\uace0 \uc77c\ubd80 \uc2b9\uc18c \ud310\uacb0\uc744 \ub0b4\ub838\uace0, \ud56d\uc18c\uc2ec\uc740 \uc6d0\uace0 \ud328\uc18c \ud310\uacb0\uc744 \ub0b4\ub838\ub2e4. 2011\ub144 4\uc6d4 26\uc77c, \ud56d\uc18c\uc2ec \uc7ac\ud310\ubd80\ub294 \"\uad6d\ud68c\uc758\uc6d0\uc73c\ub85c\uc11c \uac80\ucc30 \uc218\uc0ac \uacb0\uacfc\ub97c \uac10\uc2dc\u00b7\ube44\ud310\ud558\ub294 \uac83\uc740 \uc815\ub2f9\ud55c \uc5c5\ubb34 \ud65c\ub3d9\uc774\ub2e4. \uae40\uc528\uc758 \uba54\ubaa8 \ub4f1 \uc5b4\ub290 \uc815\ub3c4 \uadfc\uac70\ub97c \uac00\uc9c0\uace0 \uc758\ubb38\uc744 \uc81c\uae30\ud588\ub358 \ub9cc\ud07c \ud604\uc800\ud788 \uc0c1\ub2f9\uc131\uc744 \uc783\uc740 \uac83\uc73c\ub85c \ud3c9\uac00\ub418\uc9c0 \uc54a\ub294\ub2e4\"\ub77c\uace0 \ud310\ub2e8\ud588\ub2e4. \uadf8\ub7ec\ub098 \uacf5\uc9c1\uc120\uac70\ubc95 \uc704\ubc18 \ubc0f \ud5c8\uc704\uc0ac\uc2e4 \uc720\ud3ec \ub4f1\uc758 \ud610\uc758\uc5d0 \ub300\ud558\uc5ec\uc11c\ub294 1\uc2ec \ubc0f \ud56d\uc18c\uc2ec\uc5d0\uc11c \uc9d5\uc5ed 1\ub144\uc758 \uc2e4\ud615\uc774 \uc120\uace0\ub418\uc5c8\ub2e4. 2011\ub144 12\uc6d4 22\uc77c, \ub300\ubc95\uc6d0 2\ubd80(\uc8fc\uc2ec \uc774\uc0c1\ud6c8 \ub300\ubc95\uad00)\ub294 \uc815\ubd09\uc8fc\uc5d0\uac8c \uc9d5\uc5ed 1\ub144 \uc2e4\ud615\uc744 \uc120\uace0\ud55c \uc6d0\uc2ec\uc744 \ud655\uc815\ud558\uc600\uc73c\uba70 \uc774\ub85c \uc778\ud574, \uc815\ubd09\uc8fc\ub294 [\uacf5\uc9c1\uc120\uac70\ubc95]\uc81c18\uc870(\uc120\uac70\uad8c\uc774 \uc5c6\ub294\uc790) \ubc0f \uc81c19\uc870(\ud53c\uc120\uac70\uad8c\uc774 \uc5c6\ub294\uc790)\uc5d0 \uc758\uac70 10\ub144\uac04 \ud53c\uc120\uac70\uad8c\uc744 \ubc15\ud0c8\ub2f9\ud588\ub2e4."} {"question": "\uae40\uc720\uc815\uc774 \ub2a5\uccad\uc2a4\ub7ec\uc6b4 \uc804\ub77c\ub3c4 \uc0ac\ud22c\ub9ac \uc5f0\uae30\ub97c \uc120\ubcf4\uc5ec\uc900 \uc791\ud488 \uc774\ub984\uc740?", "paragraph": "\uae40\uc720\uc815\uc740 2011\ub144\ubd80\ud130 2012\ub144\uae4c\uc9c0 \ud22c\ub2c8\ubc84\uc2a4 \u300a\ub9c9\uc774\ub798\uc1fc\u300b\uacfc \u300a\ub9c9\uc774\ub798\uc1fc 2: \ubb34\uc791\uc815\ud0d0\uc815\ub300\u300b\uc758 MC\ub97c \ub9e1\uc73c\uba70 \uc9c4\ud589\uc790\ub85c\ub3c4 \ud65c\uc57d\ud558\uc600\ub2e4. 2012\ub144 1\uc6d4\ubd80\ud130 \ubc29\uc1a1 \ub41c \ud4e8\uc804 \uc0ac\uadf9 \u300a\ud574\ub97c \ud488\uc740 \ub2ec\u300b\uc5d0\uc11c \uc0c1\ub300\uc5ed\uc758 \uc5ec\uc9c4\uad6c\uc640 \uadf9\uc911 \ucd08\ubc18\uc744 \uc774\ub04c\uba70 \ud48b\ud48b\ud558\uace0 \uc124\ub818 \uac00\ub4dd\ud55c \ub85c\ub9e8\uc2a4\ub97c \uadf8\ub824\ub0b4\uba70 \uc131\uc778 \uc5f0\uae30\uc790\ub4e4 \ubabb\uc9c0\uc54a\uc740 \ud654\uc81c\ub97c \ub0b3\uc73c\uba70 \ub9ce\uc740 \uc778\uae30\ub97c \ub204\ub838\ub2e4. \ud2b9\ud788 \uc774 \uc791\ud488\uc740 \ubc29\uc601 \ub2f9\uc2dc 40%\uac00 \ub118\ub294 \ub192\uc740 \uc2dc\uccad\ub960\uc744 \uae30\ub85d\ud558\uc600\ub2e4. \uadf8\ub140\ub294 \uadf8\ud574 \uc5f0\uc774\uc5b4 \u300a\uba54\uc774\ud038\u300b\uc5d0 \ucd9c\uc5f0\ud558\uba70 \ub370\ubdd4 \ucc98\uc74c\uc73c\ub85c \ub2a5\uccad\uc2a4\ub7ec\uc6b4 \uc804\ub77c\ub3c4 \uc0ac\ud22c\ub9ac \uc5f0\uae30\ub97c \uc120\ubcf4\uc774\uae30\ub3c4 \ud588\ub2e4. \uae40\uc720\uc815\uc740 \ubaa8\ub378\ub85c\uc11c 2012 F/W \uc11c\uc6b8\ud328\uc158\uc704\ud06c \uc11c\uc6b8\ucf5c\ub809\uc158\uacfc 2012\ub144 \u2018Salon Du Chocolat in Paris\u2019\uc640 2013\ub144 \u2018Salon Du Chocolat in Seoul\u2019 \ubb34\ub300\uc5d0 \uc624\ub974\uae30\ub3c4 \ud588\ub2e4.", "answer": "\uba54\uc774\ud038", "sentence": "\uadf8\ub140\ub294 \uadf8\ud574 \uc5f0\uc774\uc5b4 \u300a \uba54\uc774\ud038 \u300b\uc5d0 \ucd9c\uc5f0\ud558\uba70 \ub370\ubdd4 \ucc98\uc74c\uc73c\ub85c \ub2a5\uccad\uc2a4\ub7ec\uc6b4 \uc804\ub77c\ub3c4 \uc0ac\ud22c\ub9ac \uc5f0\uae30\ub97c \uc120\ubcf4\uc774\uae30\ub3c4 \ud588\ub2e4.", "paragraph_sentence": "\uae40\uc720\uc815\uc740 2011\ub144\ubd80\ud130 2012\ub144\uae4c\uc9c0 \ud22c\ub2c8\ubc84\uc2a4 \u300a\ub9c9\uc774\ub798\uc1fc\u300b\uacfc \u300a\ub9c9\uc774\ub798\uc1fc 2: \ubb34\uc791\uc815\ud0d0\uc815\ub300\u300b\uc758 MC\ub97c \ub9e1\uc73c\uba70 \uc9c4\ud589\uc790\ub85c\ub3c4 \ud65c\uc57d\ud558\uc600\ub2e4. 2012\ub144 1\uc6d4\ubd80\ud130 \ubc29\uc1a1 \ub41c \ud4e8\uc804 \uc0ac\uadf9 \u300a\ud574\ub97c \ud488\uc740 \ub2ec\u300b\uc5d0\uc11c \uc0c1\ub300\uc5ed\uc758 \uc5ec\uc9c4\uad6c\uc640 \uadf9\uc911 \ucd08\ubc18\uc744 \uc774\ub04c\uba70 \ud48b\ud48b\ud558\uace0 \uc124\ub818 \uac00\ub4dd\ud55c \ub85c\ub9e8\uc2a4\ub97c \uadf8\ub824\ub0b4\uba70 \uc131\uc778 \uc5f0\uae30\uc790\ub4e4 \ubabb\uc9c0\uc54a\uc740 \ud654\uc81c\ub97c \ub0b3\uc73c\uba70 \ub9ce\uc740 \uc778\uae30\ub97c \ub204\ub838\ub2e4. \ud2b9\ud788 \uc774 \uc791\ud488\uc740 \ubc29\uc601 \ub2f9\uc2dc 40%\uac00 \ub118\ub294 \ub192\uc740 \uc2dc\uccad\ub960\uc744 \uae30\ub85d\ud558\uc600\ub2e4. \uadf8\ub140\ub294 \uadf8\ud574 \uc5f0\uc774\uc5b4 \u300a \uba54\uc774\ud038 \u300b\uc5d0 \ucd9c\uc5f0\ud558\uba70 \ub370\ubdd4 \ucc98\uc74c\uc73c\ub85c \ub2a5\uccad\uc2a4\ub7ec\uc6b4 \uc804\ub77c\ub3c4 \uc0ac\ud22c\ub9ac \uc5f0\uae30\ub97c \uc120\ubcf4\uc774\uae30\ub3c4 \ud588\ub2e4. \uae40\uc720\uc815\uc740 \ubaa8\ub378\ub85c\uc11c 2012 F/W \uc11c\uc6b8\ud328\uc158\uc704\ud06c \uc11c\uc6b8\ucf5c\ub809\uc158\uacfc 2012\ub144 \u2018Salon Du Chocolat in Paris\u2019\uc640 2013\ub144 \u2018Salon Du Chocolat in Seoul\u2019 \ubb34\ub300\uc5d0 \uc624\ub974\uae30\ub3c4 \ud588\ub2e4.", "paragraph_answer": "\uae40\uc720\uc815\uc740 2011\ub144\ubd80\ud130 2012\ub144\uae4c\uc9c0 \ud22c\ub2c8\ubc84\uc2a4 \u300a\ub9c9\uc774\ub798\uc1fc\u300b\uacfc \u300a\ub9c9\uc774\ub798\uc1fc 2: \ubb34\uc791\uc815\ud0d0\uc815\ub300\u300b\uc758 MC\ub97c \ub9e1\uc73c\uba70 \uc9c4\ud589\uc790\ub85c\ub3c4 \ud65c\uc57d\ud558\uc600\ub2e4. 2012\ub144 1\uc6d4\ubd80\ud130 \ubc29\uc1a1 \ub41c \ud4e8\uc804 \uc0ac\uadf9 \u300a\ud574\ub97c \ud488\uc740 \ub2ec\u300b\uc5d0\uc11c \uc0c1\ub300\uc5ed\uc758 \uc5ec\uc9c4\uad6c\uc640 \uadf9\uc911 \ucd08\ubc18\uc744 \uc774\ub04c\uba70 \ud48b\ud48b\ud558\uace0 \uc124\ub818 \uac00\ub4dd\ud55c \ub85c\ub9e8\uc2a4\ub97c \uadf8\ub824\ub0b4\uba70 \uc131\uc778 \uc5f0\uae30\uc790\ub4e4 \ubabb\uc9c0\uc54a\uc740 \ud654\uc81c\ub97c \ub0b3\uc73c\uba70 \ub9ce\uc740 \uc778\uae30\ub97c \ub204\ub838\ub2e4. \ud2b9\ud788 \uc774 \uc791\ud488\uc740 \ubc29\uc601 \ub2f9\uc2dc 40%\uac00 \ub118\ub294 \ub192\uc740 \uc2dc\uccad\ub960\uc744 \uae30\ub85d\ud558\uc600\ub2e4. \uadf8\ub140\ub294 \uadf8\ud574 \uc5f0\uc774\uc5b4 \u300a \uba54\uc774\ud038 \u300b\uc5d0 \ucd9c\uc5f0\ud558\uba70 \ub370\ubdd4 \ucc98\uc74c\uc73c\ub85c \ub2a5\uccad\uc2a4\ub7ec\uc6b4 \uc804\ub77c\ub3c4 \uc0ac\ud22c\ub9ac \uc5f0\uae30\ub97c \uc120\ubcf4\uc774\uae30\ub3c4 \ud588\ub2e4. \uae40\uc720\uc815\uc740 \ubaa8\ub378\ub85c\uc11c 2012 F/W \uc11c\uc6b8\ud328\uc158\uc704\ud06c \uc11c\uc6b8\ucf5c\ub809\uc158\uacfc 2012\ub144 \u2018Salon Du Chocolat in Paris\u2019\uc640 2013\ub144 \u2018Salon Du Chocolat in Seoul\u2019 \ubb34\ub300\uc5d0 \uc624\ub974\uae30\ub3c4 \ud588\ub2e4.", "sentence_answer": "\uadf8\ub140\ub294 \uadf8\ud574 \uc5f0\uc774\uc5b4 \u300a \uba54\uc774\ud038 \u300b\uc5d0 \ucd9c\uc5f0\ud558\uba70 \ub370\ubdd4 \ucc98\uc74c\uc73c\ub85c \ub2a5\uccad\uc2a4\ub7ec\uc6b4 \uc804\ub77c\ub3c4 \uc0ac\ud22c\ub9ac \uc5f0\uae30\ub97c \uc120\ubcf4\uc774\uae30\ub3c4 \ud588\ub2e4.", "paragraph_id": "5999753-2-2", "paragraph_question": "question: \uae40\uc720\uc815\uc774 \ub2a5\uccad\uc2a4\ub7ec\uc6b4 \uc804\ub77c\ub3c4 \uc0ac\ud22c\ub9ac \uc5f0\uae30\ub97c \uc120\ubcf4\uc5ec\uc900 \uc791\ud488 \uc774\ub984\uc740?, context: \uae40\uc720\uc815\uc740 2011\ub144\ubd80\ud130 2012\ub144\uae4c\uc9c0 \ud22c\ub2c8\ubc84\uc2a4 \u300a\ub9c9\uc774\ub798\uc1fc\u300b\uacfc \u300a\ub9c9\uc774\ub798\uc1fc 2: \ubb34\uc791\uc815\ud0d0\uc815\ub300\u300b\uc758 MC\ub97c \ub9e1\uc73c\uba70 \uc9c4\ud589\uc790\ub85c\ub3c4 \ud65c\uc57d\ud558\uc600\ub2e4. 2012\ub144 1\uc6d4\ubd80\ud130 \ubc29\uc1a1 \ub41c \ud4e8\uc804 \uc0ac\uadf9 \u300a\ud574\ub97c \ud488\uc740 \ub2ec\u300b\uc5d0\uc11c \uc0c1\ub300\uc5ed\uc758 \uc5ec\uc9c4\uad6c\uc640 \uadf9\uc911 \ucd08\ubc18\uc744 \uc774\ub04c\uba70 \ud48b\ud48b\ud558\uace0 \uc124\ub818 \uac00\ub4dd\ud55c \ub85c\ub9e8\uc2a4\ub97c \uadf8\ub824\ub0b4\uba70 \uc131\uc778 \uc5f0\uae30\uc790\ub4e4 \ubabb\uc9c0\uc54a\uc740 \ud654\uc81c\ub97c \ub0b3\uc73c\uba70 \ub9ce\uc740 \uc778\uae30\ub97c \ub204\ub838\ub2e4. \ud2b9\ud788 \uc774 \uc791\ud488\uc740 \ubc29\uc601 \ub2f9\uc2dc 40%\uac00 \ub118\ub294 \ub192\uc740 \uc2dc\uccad\ub960\uc744 \uae30\ub85d\ud558\uc600\ub2e4. \uadf8\ub140\ub294 \uadf8\ud574 \uc5f0\uc774\uc5b4 \u300a\uba54\uc774\ud038\u300b\uc5d0 \ucd9c\uc5f0\ud558\uba70 \ub370\ubdd4 \ucc98\uc74c\uc73c\ub85c \ub2a5\uccad\uc2a4\ub7ec\uc6b4 \uc804\ub77c\ub3c4 \uc0ac\ud22c\ub9ac \uc5f0\uae30\ub97c \uc120\ubcf4\uc774\uae30\ub3c4 \ud588\ub2e4. \uae40\uc720\uc815\uc740 \ubaa8\ub378\ub85c\uc11c 2012 F/W \uc11c\uc6b8\ud328\uc158\uc704\ud06c \uc11c\uc6b8\ucf5c\ub809\uc158\uacfc 2012\ub144 \u2018Salon Du Chocolat in Paris\u2019\uc640 2013\ub144 \u2018Salon Du Chocolat in Seoul\u2019 \ubb34\ub300\uc5d0 \uc624\ub974\uae30\ub3c4 \ud588\ub2e4."} {"question": "2008\ub144 \uc2a4\ucf00\uc774\ud2b8 \uc544\uba54\ub9ac\uce74\uc5d0\uc11c \uc0c8\ub86d\uac8c \uc120\ubcf4\uc778 \ud504\ub9ac\uc2a4\ucf00\uc774\ud305\uc740 \ub2c8\ucf5c\ub77c\uc774 \ub9bc\uc2a4\ud0a4\ucf54\ub974\uc0ac\ucf54\ud504\uc758 \uc5b4\ub5a4 \uace1\uc774\uc5c8\ub098?", "paragraph": "2008\ub144 10\uc6d4, 1\ucc28 \uadf8\ub791\ud504\ub9ac \ub300\ud68c\uc778 \uc2a4\ucf00\uc774\ud2b8 \uc544\uba54\ub9ac\uce74\uc5d0 \ucd9c\uc804\ud588\ub2e4. \uc2dc\uc98c\uc2dc\uc791 \ud6c4 \ucc98\uc74c \uc120\ubcf4\uc778 \uc1fc\ud2b8\ud504\ub85c\uadf8\ub7a8\uc740 \uce74\ubbf8\uc720 \uc0dd\uc0c1\uc2a4\uc758 \u3008\uc8fd\uc74c\uc758 \ubb34\ub3c4\u3009\uc600\uc73c\uba70 \uacbd\uae30\uc758 \uacb0\uacfc\ub294 69.50\uc810\uc774\ub77c\ub294 \ub192\uc740 \uc810\uc218\ub85c \uc1fc\ud2b8 2\uc704\uc778 \uc548\ub3c4 \ubbf8\ud0a4\uc640 11.70\uc810\uc758 \ucc28\uc774\ub97c \ub0b4\uba70 1\uc704\ub97c \uae30\ub85d\ud55c\ub2e4. \uc2dc\uc98c \uc2dc\uc791 \ud6c4 \uc0c8\ub86d\uac8c \uc120\ubcf4\uc778 \ud504\ub9ac\uc2a4\ucf00\uc774\ud305\uc740 \ub2c8\ucf5c\ub77c\uc774 \ub9bc\uc2a4\ud0a4\ucf54\ub974\uc0ac\ucf54\ud504\uc758 \u3008\uc138\ud5e4\ub77c\uc790\ub370\u3009\uc600\ub2e4. \ud504\ub9ac\ud504\ub85c\uadf8\ub7a8\uc5d0\uc11c\ub294 3\ud68c\uc804 \ub8f9 \uc810\ud504\ub97c 1\ud68c\uc804\uc73c\ub85c \ucc98\ub9ac, \ud50c\ub77c\uc789 \uccb4\uc778\uc9c0\ud48b \ucef4\ube44\ub124\uc774\uc158 \uc2a4\ud540\uc5d0\uc11c \ud68c\uc804\uc218 \ubd80\uc871\uc758 \uc2e4\uc218\ub97c \uc81c\uc678\ud558\uace0\ub294 \ub2e4\ub978 \uc694\uc18c\uc5d0\uc11c \uc88b\uc740 \uc5f0\uae30\ub97c \ud3bc\uccd0 193.45\uc810\uc774\ub77c\ub294 \uc555\ub3c4\uc801\uc778 \uc810\uc218\ub85c \uc6b0\uc2b9\uc744 \ucc28\uc9c0\ud588\ub2e4. \uc774 \uc810\uc218\ub294 2\uc704\uc778 \ub098\uce74\ub178 \uc720\uce74\ub9ac\ubcf4\ub2e4 21\uc810\uc774\ub098 \ub192\uc740 \uc810\uc218\uc600\ub2e4. \uc138\uacc4\uc120\uc218\uad8c 4\ud68c \uc6b0\uc2b9\uc790\uc774\uba70 \ud604 \ubbf8\uad6d \ud53c\uaca8\uc2a4\ucf00\uc774\ud305 \ud574\uc124\uc790\uc778 \ucee4\ud2b8 \ube0c\ub77c\uc6b0\ub2dd\uc740 \uae40\uc5f0\uc544\uc758 \uc5f0\uae30\ub97c \ubcf4\uace0 \uc5b8\ub860\uacfc\uc758 \uc778\ud130\ubdf0\uc5d0\uc11c \"\uc624\ub298 \uae40\uc5f0\uc544 \uc120\uc218\uc758 \uc5f0\uae30\ub294 \uac70\uc758 '\uc4f0\ub098\ubbf8'\uc640 \uac19\uc740 \uac10\ub3d9\uc744 \uc92c\uc2b5\ub2c8\ub2e4.\"\uace0 \ub9d0\ud588\ub2e4.", "answer": "\uc138\ud5e4\ub77c\uc790\ub370", "sentence": "\uc2dc\uc98c \uc2dc\uc791 \ud6c4 \uc0c8\ub86d\uac8c \uc120\ubcf4\uc778 \ud504\ub9ac\uc2a4\ucf00\uc774\ud305\uc740 \ub2c8\ucf5c\ub77c\uc774 \ub9bc\uc2a4\ud0a4\ucf54\ub974\uc0ac\ucf54\ud504\uc758 \u3008 \uc138\ud5e4\ub77c\uc790\ub370 \u3009\uc600\ub2e4.", "paragraph_sentence": "2008\ub144 10\uc6d4, 1\ucc28 \uadf8\ub791\ud504\ub9ac \ub300\ud68c\uc778 \uc2a4\ucf00\uc774\ud2b8 \uc544\uba54\ub9ac\uce74\uc5d0 \ucd9c\uc804\ud588\ub2e4. \uc2dc\uc98c\uc2dc\uc791 \ud6c4 \ucc98\uc74c \uc120\ubcf4\uc778 \uc1fc\ud2b8\ud504\ub85c\uadf8\ub7a8\uc740 \uce74\ubbf8\uc720 \uc0dd\uc0c1\uc2a4\uc758 \u3008\uc8fd\uc74c\uc758 \ubb34\ub3c4\u3009\uc600\uc73c\uba70 \uacbd\uae30\uc758 \uacb0\uacfc\ub294 69.50\uc810\uc774\ub77c\ub294 \ub192\uc740 \uc810\uc218\ub85c \uc1fc\ud2b8 2\uc704\uc778 \uc548\ub3c4 \ubbf8\ud0a4\uc640 11.70\uc810\uc758 \ucc28\uc774\ub97c \ub0b4\uba70 1\uc704\ub97c \uae30\ub85d\ud55c\ub2e4. \uc2dc\uc98c \uc2dc\uc791 \ud6c4 \uc0c8\ub86d\uac8c \uc120\ubcf4\uc778 \ud504\ub9ac\uc2a4\ucf00\uc774\ud305\uc740 \ub2c8\ucf5c\ub77c\uc774 \ub9bc\uc2a4\ud0a4\ucf54\ub974\uc0ac\ucf54\ud504\uc758 \u3008 \uc138\ud5e4\ub77c\uc790\ub370 \u3009\uc600\ub2e4. \ud504\ub9ac\ud504\ub85c\uadf8\ub7a8\uc5d0\uc11c\ub294 3\ud68c\uc804 \ub8f9 \uc810\ud504\ub97c 1\ud68c\uc804\uc73c\ub85c \ucc98\ub9ac, \ud50c\ub77c\uc789 \uccb4\uc778\uc9c0\ud48b \ucef4\ube44\ub124\uc774\uc158 \uc2a4\ud540\uc5d0\uc11c \ud68c\uc804\uc218 \ubd80\uc871\uc758 \uc2e4\uc218\ub97c \uc81c\uc678\ud558\uace0\ub294 \ub2e4\ub978 \uc694\uc18c\uc5d0\uc11c \uc88b\uc740 \uc5f0\uae30\ub97c \ud3bc\uccd0 193.45\uc810\uc774\ub77c\ub294 \uc555\ub3c4\uc801\uc778 \uc810\uc218\ub85c \uc6b0\uc2b9\uc744 \ucc28\uc9c0\ud588\ub2e4. \uc774 \uc810\uc218\ub294 2\uc704\uc778 \ub098\uce74\ub178 \uc720\uce74\ub9ac\ubcf4\ub2e4 21\uc810\uc774\ub098 \ub192\uc740 \uc810\uc218\uc600\ub2e4. \uc138\uacc4\uc120\uc218\uad8c 4\ud68c \uc6b0\uc2b9\uc790\uc774\uba70 \ud604 \ubbf8\uad6d \ud53c\uaca8\uc2a4\ucf00\uc774\ud305 \ud574\uc124\uc790\uc778 \ucee4\ud2b8 \ube0c\ub77c\uc6b0\ub2dd\uc740 \uae40\uc5f0\uc544\uc758 \uc5f0\uae30\ub97c \ubcf4\uace0 \uc5b8\ub860\uacfc\uc758 \uc778\ud130\ubdf0\uc5d0\uc11c \"\uc624\ub298 \uae40\uc5f0\uc544 \uc120\uc218\uc758 \uc5f0\uae30\ub294 \uac70\uc758 '\uc4f0\ub098\ubbf8'\uc640 \uac19\uc740 \uac10\ub3d9\uc744 \uc92c\uc2b5\ub2c8\ub2e4. \"\uace0 \ub9d0\ud588\ub2e4.", "paragraph_answer": "2008\ub144 10\uc6d4, 1\ucc28 \uadf8\ub791\ud504\ub9ac \ub300\ud68c\uc778 \uc2a4\ucf00\uc774\ud2b8 \uc544\uba54\ub9ac\uce74\uc5d0 \ucd9c\uc804\ud588\ub2e4. \uc2dc\uc98c\uc2dc\uc791 \ud6c4 \ucc98\uc74c \uc120\ubcf4\uc778 \uc1fc\ud2b8\ud504\ub85c\uadf8\ub7a8\uc740 \uce74\ubbf8\uc720 \uc0dd\uc0c1\uc2a4\uc758 \u3008\uc8fd\uc74c\uc758 \ubb34\ub3c4\u3009\uc600\uc73c\uba70 \uacbd\uae30\uc758 \uacb0\uacfc\ub294 69.50\uc810\uc774\ub77c\ub294 \ub192\uc740 \uc810\uc218\ub85c \uc1fc\ud2b8 2\uc704\uc778 \uc548\ub3c4 \ubbf8\ud0a4\uc640 11.70\uc810\uc758 \ucc28\uc774\ub97c \ub0b4\uba70 1\uc704\ub97c \uae30\ub85d\ud55c\ub2e4. \uc2dc\uc98c \uc2dc\uc791 \ud6c4 \uc0c8\ub86d\uac8c \uc120\ubcf4\uc778 \ud504\ub9ac\uc2a4\ucf00\uc774\ud305\uc740 \ub2c8\ucf5c\ub77c\uc774 \ub9bc\uc2a4\ud0a4\ucf54\ub974\uc0ac\ucf54\ud504\uc758 \u3008 \uc138\ud5e4\ub77c\uc790\ub370 \u3009\uc600\ub2e4. \ud504\ub9ac\ud504\ub85c\uadf8\ub7a8\uc5d0\uc11c\ub294 3\ud68c\uc804 \ub8f9 \uc810\ud504\ub97c 1\ud68c\uc804\uc73c\ub85c \ucc98\ub9ac, \ud50c\ub77c\uc789 \uccb4\uc778\uc9c0\ud48b \ucef4\ube44\ub124\uc774\uc158 \uc2a4\ud540\uc5d0\uc11c \ud68c\uc804\uc218 \ubd80\uc871\uc758 \uc2e4\uc218\ub97c \uc81c\uc678\ud558\uace0\ub294 \ub2e4\ub978 \uc694\uc18c\uc5d0\uc11c \uc88b\uc740 \uc5f0\uae30\ub97c \ud3bc\uccd0 193.45\uc810\uc774\ub77c\ub294 \uc555\ub3c4\uc801\uc778 \uc810\uc218\ub85c \uc6b0\uc2b9\uc744 \ucc28\uc9c0\ud588\ub2e4. \uc774 \uc810\uc218\ub294 2\uc704\uc778 \ub098\uce74\ub178 \uc720\uce74\ub9ac\ubcf4\ub2e4 21\uc810\uc774\ub098 \ub192\uc740 \uc810\uc218\uc600\ub2e4. \uc138\uacc4\uc120\uc218\uad8c 4\ud68c \uc6b0\uc2b9\uc790\uc774\uba70 \ud604 \ubbf8\uad6d \ud53c\uaca8\uc2a4\ucf00\uc774\ud305 \ud574\uc124\uc790\uc778 \ucee4\ud2b8 \ube0c\ub77c\uc6b0\ub2dd\uc740 \uae40\uc5f0\uc544\uc758 \uc5f0\uae30\ub97c \ubcf4\uace0 \uc5b8\ub860\uacfc\uc758 \uc778\ud130\ubdf0\uc5d0\uc11c \"\uc624\ub298 \uae40\uc5f0\uc544 \uc120\uc218\uc758 \uc5f0\uae30\ub294 \uac70\uc758 '\uc4f0\ub098\ubbf8'\uc640 \uac19\uc740 \uac10\ub3d9\uc744 \uc92c\uc2b5\ub2c8\ub2e4.\"\uace0 \ub9d0\ud588\ub2e4.", "sentence_answer": "\uc2dc\uc98c \uc2dc\uc791 \ud6c4 \uc0c8\ub86d\uac8c \uc120\ubcf4\uc778 \ud504\ub9ac\uc2a4\ucf00\uc774\ud305\uc740 \ub2c8\ucf5c\ub77c\uc774 \ub9bc\uc2a4\ud0a4\ucf54\ub974\uc0ac\ucf54\ud504\uc758 \u3008 \uc138\ud5e4\ub77c\uc790\ub370 \u3009\uc600\ub2e4.", "paragraph_id": "6511758-5-2", "paragraph_question": "question: 2008\ub144 \uc2a4\ucf00\uc774\ud2b8 \uc544\uba54\ub9ac\uce74\uc5d0\uc11c \uc0c8\ub86d\uac8c \uc120\ubcf4\uc778 \ud504\ub9ac\uc2a4\ucf00\uc774\ud305\uc740 \ub2c8\ucf5c\ub77c\uc774 \ub9bc\uc2a4\ud0a4\ucf54\ub974\uc0ac\ucf54\ud504\uc758 \uc5b4\ub5a4 \uace1\uc774\uc5c8\ub098?, context: 2008\ub144 10\uc6d4, 1\ucc28 \uadf8\ub791\ud504\ub9ac \ub300\ud68c\uc778 \uc2a4\ucf00\uc774\ud2b8 \uc544\uba54\ub9ac\uce74\uc5d0 \ucd9c\uc804\ud588\ub2e4. \uc2dc\uc98c\uc2dc\uc791 \ud6c4 \ucc98\uc74c \uc120\ubcf4\uc778 \uc1fc\ud2b8\ud504\ub85c\uadf8\ub7a8\uc740 \uce74\ubbf8\uc720 \uc0dd\uc0c1\uc2a4\uc758 \u3008\uc8fd\uc74c\uc758 \ubb34\ub3c4\u3009\uc600\uc73c\uba70 \uacbd\uae30\uc758 \uacb0\uacfc\ub294 69.50\uc810\uc774\ub77c\ub294 \ub192\uc740 \uc810\uc218\ub85c \uc1fc\ud2b8 2\uc704\uc778 \uc548\ub3c4 \ubbf8\ud0a4\uc640 11.70\uc810\uc758 \ucc28\uc774\ub97c \ub0b4\uba70 1\uc704\ub97c \uae30\ub85d\ud55c\ub2e4. \uc2dc\uc98c \uc2dc\uc791 \ud6c4 \uc0c8\ub86d\uac8c \uc120\ubcf4\uc778 \ud504\ub9ac\uc2a4\ucf00\uc774\ud305\uc740 \ub2c8\ucf5c\ub77c\uc774 \ub9bc\uc2a4\ud0a4\ucf54\ub974\uc0ac\ucf54\ud504\uc758 \u3008\uc138\ud5e4\ub77c\uc790\ub370\u3009\uc600\ub2e4. \ud504\ub9ac\ud504\ub85c\uadf8\ub7a8\uc5d0\uc11c\ub294 3\ud68c\uc804 \ub8f9 \uc810\ud504\ub97c 1\ud68c\uc804\uc73c\ub85c \ucc98\ub9ac, \ud50c\ub77c\uc789 \uccb4\uc778\uc9c0\ud48b \ucef4\ube44\ub124\uc774\uc158 \uc2a4\ud540\uc5d0\uc11c \ud68c\uc804\uc218 \ubd80\uc871\uc758 \uc2e4\uc218\ub97c \uc81c\uc678\ud558\uace0\ub294 \ub2e4\ub978 \uc694\uc18c\uc5d0\uc11c \uc88b\uc740 \uc5f0\uae30\ub97c \ud3bc\uccd0 193.45\uc810\uc774\ub77c\ub294 \uc555\ub3c4\uc801\uc778 \uc810\uc218\ub85c \uc6b0\uc2b9\uc744 \ucc28\uc9c0\ud588\ub2e4. \uc774 \uc810\uc218\ub294 2\uc704\uc778 \ub098\uce74\ub178 \uc720\uce74\ub9ac\ubcf4\ub2e4 21\uc810\uc774\ub098 \ub192\uc740 \uc810\uc218\uc600\ub2e4. \uc138\uacc4\uc120\uc218\uad8c 4\ud68c \uc6b0\uc2b9\uc790\uc774\uba70 \ud604 \ubbf8\uad6d \ud53c\uaca8\uc2a4\ucf00\uc774\ud305 \ud574\uc124\uc790\uc778 \ucee4\ud2b8 \ube0c\ub77c\uc6b0\ub2dd\uc740 \uae40\uc5f0\uc544\uc758 \uc5f0\uae30\ub97c \ubcf4\uace0 \uc5b8\ub860\uacfc\uc758 \uc778\ud130\ubdf0\uc5d0\uc11c \"\uc624\ub298 \uae40\uc5f0\uc544 \uc120\uc218\uc758 \uc5f0\uae30\ub294 \uac70\uc758 '\uc4f0\ub098\ubbf8'\uc640 \uac19\uc740 \uac10\ub3d9\uc744 \uc92c\uc2b5\ub2c8\ub2e4.\"\uace0 \ub9d0\ud588\ub2e4."} {"question": "\ub2e4\ud558\uc6b0, \uc624\ub77c\ub2c8\ubd80\ub974\ud06c \ub4f1 \uac15\uc81c \uc218\uc6a9\uc18c\uc5d0 \uc218\uac10\ub41c \uc0ac\ub78c\ub4e4\uc740 \uba87 \uba85\uc778\uac00?", "paragraph": "\uacf5\uc2dd\uc801\uc73c\ub85c \uc54c\ub824\uc9c4 \uc22b\uc790\uc778 91\uba85\uc758 \uc0ac\uc0c1\uc790\ubcf4\ub2e4 \ub354 \ub9ce\uc740 \uc0ac\uc0c1\uc790\uac00 \ubc1c\uc0dd\ud588\uc744 \uac83\uc73c\ub85c \uc608\uc0c1\ub41c\ub2e4. \uc57d 30,000\uba85\uc774 \ub2e4\ud558\uc6b0(Dachau), \uc791\uc13c\ud558\uc6b0\uc820(Sachsenhausen), \ubd80\ud5e8\ubc1c\ud2b8(Buchenwald), \uc624\ub77c\ub2c8\ubd80\ub974\ud06c(Oranieburg), \ub4f1\uc5d0 \uc788\ub294 \uac15\uc81c \uc218\uc6a9\uc18c\uc5d0 \uc218\uac10\ub418\uc5c8\ub2e4. \uadf8\ub4e4\uc740 \uba87 \uc8fc\uac04 \uc218\uac10\ub418\uc5c8\ub294\ub370 \uace7 \uadf8\ub4e4\uc774 \ub2e4\ub978 \uacf3\uc73c\ub85c \uc774\uc8fc\ub420 \uac83\uc774\ub77c \ud655\uc2e4\ud558\uac70\ub098 \uadf8\ub4e4\uc758 \uc7ac\uc0b0\uc744 \ub098\uce58\uc5d0 \uc591\ub3c4\ud560 \uacbd\uc6b0 \ud480\ub824\ub0a0 \uc218\ub3c4 \uc788\uc5c8\ub2e4. \ub3c5\uc77c \uc720\ub300\uc778\ub4e4\uc740 \ub300\ud559\uc0b4\uc744 \ud53c\ud558\uae30 \uc704\ud574 \ubb3c\uc9c8\uc801\uc73c\ub85c \ubcf4\uc0c1\ud558\uae30\ub3c4 \ud588\ub294\ub370, \uc774\ub294 \uc218 \ubc31, \uc218 \ucc9c \ub9c8\ub974\ud06c\uc600\uc73c\uba70 \uc774 \ubfd0\ub9cc \uc544\ub2c8\ub77c \u201c\uc18d\uc8c4 \uc138\uae08\u201d\uc73c\ub85c 10\uc5b5 \ub9c8\ub974\ud06c\uac00 \ub118\ub294 \ub3c8\uc744 \uc9c0\ubd88\ud574\uc57c \ud588\ub2e4. \uc774\ub7ec\ud55c \uc9d1\ub2e8\ud559\uc0b4 \uc774\ud6c4 \ub3c5\uc77c\ub85c\ubd80\ud130 \uc720\ub300\uc778\ub4e4\uc758 \uc774\uc8fc\ub294 \uac00\uc18d\ud654\ub418\uc5c8\uace0, \ub3c5\uc77c\uc5d0\uc11c \uacf5\uac1c\uc801\uc778 \uc720\ub300\uc778\uc758 \uc0b6\uc740 \ub05d\uc7a5\uc774 \ub0ac\ub2e4.", "answer": "\uc57d 30,000\uba85", "sentence": "\uc57d 30,000\uba85 \uc774 \ub2e4\ud558\uc6b0(Dachau), \uc791\uc13c\ud558\uc6b0\uc820(Sachsenhausen), \ubd80\ud5e8\ubc1c\ud2b8(Buchenwald), \uc624\ub77c\ub2c8\ubd80\ub974\ud06c(Oranieburg), \ub4f1\uc5d0 \uc788\ub294 \uac15\uc81c \uc218\uc6a9\uc18c\uc5d0 \uc218\uac10\ub418\uc5c8\ub2e4.", "paragraph_sentence": "\uacf5\uc2dd\uc801\uc73c\ub85c \uc54c\ub824\uc9c4 \uc22b\uc790\uc778 91\uba85\uc758 \uc0ac\uc0c1\uc790\ubcf4\ub2e4 \ub354 \ub9ce\uc740 \uc0ac\uc0c1\uc790\uac00 \ubc1c\uc0dd\ud588\uc744 \uac83\uc73c\ub85c \uc608\uc0c1\ub41c\ub2e4. \uc57d 30,000\uba85 \uc774 \ub2e4\ud558\uc6b0(Dachau), \uc791\uc13c\ud558\uc6b0\uc820(Sachsenhausen), \ubd80\ud5e8\ubc1c\ud2b8(Buchenwald), \uc624\ub77c\ub2c8\ubd80\ub974\ud06c(Oranieburg), \ub4f1\uc5d0 \uc788\ub294 \uac15\uc81c \uc218\uc6a9\uc18c\uc5d0 \uc218\uac10\ub418\uc5c8\ub2e4. \uadf8\ub4e4\uc740 \uba87 \uc8fc\uac04 \uc218\uac10\ub418\uc5c8\ub294\ub370 \uace7 \uadf8\ub4e4\uc774 \ub2e4\ub978 \uacf3\uc73c\ub85c \uc774\uc8fc\ub420 \uac83\uc774\ub77c \ud655\uc2e4\ud558\uac70\ub098 \uadf8\ub4e4\uc758 \uc7ac\uc0b0\uc744 \ub098\uce58\uc5d0 \uc591\ub3c4\ud560 \uacbd\uc6b0 \ud480\ub824\ub0a0 \uc218\ub3c4 \uc788\uc5c8\ub2e4. \ub3c5\uc77c \uc720\ub300\uc778\ub4e4\uc740 \ub300\ud559\uc0b4\uc744 \ud53c\ud558\uae30 \uc704\ud574 \ubb3c\uc9c8\uc801\uc73c\ub85c \ubcf4\uc0c1\ud558\uae30\ub3c4 \ud588\ub294\ub370, \uc774\ub294 \uc218 \ubc31, \uc218 \ucc9c \ub9c8\ub974\ud06c\uc600\uc73c\uba70 \uc774 \ubfd0\ub9cc \uc544\ub2c8\ub77c \u201c\uc18d\uc8c4 \uc138\uae08\u201d\uc73c\ub85c 10\uc5b5 \ub9c8\ub974\ud06c\uac00 \ub118\ub294 \ub3c8\uc744 \uc9c0\ubd88\ud574\uc57c \ud588\ub2e4. \uc774\ub7ec\ud55c \uc9d1\ub2e8\ud559\uc0b4 \uc774\ud6c4 \ub3c5\uc77c\ub85c\ubd80\ud130 \uc720\ub300\uc778\ub4e4\uc758 \uc774\uc8fc\ub294 \uac00\uc18d\ud654\ub418\uc5c8\uace0, \ub3c5\uc77c\uc5d0\uc11c \uacf5\uac1c\uc801\uc778 \uc720\ub300\uc778\uc758 \uc0b6\uc740 \ub05d\uc7a5\uc774 \ub0ac\ub2e4.", "paragraph_answer": "\uacf5\uc2dd\uc801\uc73c\ub85c \uc54c\ub824\uc9c4 \uc22b\uc790\uc778 91\uba85\uc758 \uc0ac\uc0c1\uc790\ubcf4\ub2e4 \ub354 \ub9ce\uc740 \uc0ac\uc0c1\uc790\uac00 \ubc1c\uc0dd\ud588\uc744 \uac83\uc73c\ub85c \uc608\uc0c1\ub41c\ub2e4. \uc57d 30,000\uba85 \uc774 \ub2e4\ud558\uc6b0(Dachau), \uc791\uc13c\ud558\uc6b0\uc820(Sachsenhausen), \ubd80\ud5e8\ubc1c\ud2b8(Buchenwald), \uc624\ub77c\ub2c8\ubd80\ub974\ud06c(Oranieburg), \ub4f1\uc5d0 \uc788\ub294 \uac15\uc81c \uc218\uc6a9\uc18c\uc5d0 \uc218\uac10\ub418\uc5c8\ub2e4. \uadf8\ub4e4\uc740 \uba87 \uc8fc\uac04 \uc218\uac10\ub418\uc5c8\ub294\ub370 \uace7 \uadf8\ub4e4\uc774 \ub2e4\ub978 \uacf3\uc73c\ub85c \uc774\uc8fc\ub420 \uac83\uc774\ub77c \ud655\uc2e4\ud558\uac70\ub098 \uadf8\ub4e4\uc758 \uc7ac\uc0b0\uc744 \ub098\uce58\uc5d0 \uc591\ub3c4\ud560 \uacbd\uc6b0 \ud480\ub824\ub0a0 \uc218\ub3c4 \uc788\uc5c8\ub2e4. \ub3c5\uc77c \uc720\ub300\uc778\ub4e4\uc740 \ub300\ud559\uc0b4\uc744 \ud53c\ud558\uae30 \uc704\ud574 \ubb3c\uc9c8\uc801\uc73c\ub85c \ubcf4\uc0c1\ud558\uae30\ub3c4 \ud588\ub294\ub370, \uc774\ub294 \uc218 \ubc31, \uc218 \ucc9c \ub9c8\ub974\ud06c\uc600\uc73c\uba70 \uc774 \ubfd0\ub9cc \uc544\ub2c8\ub77c \u201c\uc18d\uc8c4 \uc138\uae08\u201d\uc73c\ub85c 10\uc5b5 \ub9c8\ub974\ud06c\uac00 \ub118\ub294 \ub3c8\uc744 \uc9c0\ubd88\ud574\uc57c \ud588\ub2e4. \uc774\ub7ec\ud55c \uc9d1\ub2e8\ud559\uc0b4 \uc774\ud6c4 \ub3c5\uc77c\ub85c\ubd80\ud130 \uc720\ub300\uc778\ub4e4\uc758 \uc774\uc8fc\ub294 \uac00\uc18d\ud654\ub418\uc5c8\uace0, \ub3c5\uc77c\uc5d0\uc11c \uacf5\uac1c\uc801\uc778 \uc720\ub300\uc778\uc758 \uc0b6\uc740 \ub05d\uc7a5\uc774 \ub0ac\ub2e4.", "sentence_answer": " \uc57d 30,000\uba85 \uc774 \ub2e4\ud558\uc6b0(Dachau), \uc791\uc13c\ud558\uc6b0\uc820(Sachsenhausen), \ubd80\ud5e8\ubc1c\ud2b8(Buchenwald), \uc624\ub77c\ub2c8\ubd80\ub974\ud06c(Oranieburg), \ub4f1\uc5d0 \uc788\ub294 \uac15\uc81c \uc218\uc6a9\uc18c\uc5d0 \uc218\uac10\ub418\uc5c8\ub2e4.", "paragraph_id": "6571960-10-1", "paragraph_question": "question: \ub2e4\ud558\uc6b0, \uc624\ub77c\ub2c8\ubd80\ub974\ud06c \ub4f1 \uac15\uc81c \uc218\uc6a9\uc18c\uc5d0 \uc218\uac10\ub41c \uc0ac\ub78c\ub4e4\uc740 \uba87 \uba85\uc778\uac00?, context: \uacf5\uc2dd\uc801\uc73c\ub85c \uc54c\ub824\uc9c4 \uc22b\uc790\uc778 91\uba85\uc758 \uc0ac\uc0c1\uc790\ubcf4\ub2e4 \ub354 \ub9ce\uc740 \uc0ac\uc0c1\uc790\uac00 \ubc1c\uc0dd\ud588\uc744 \uac83\uc73c\ub85c \uc608\uc0c1\ub41c\ub2e4. \uc57d 30,000\uba85\uc774 \ub2e4\ud558\uc6b0(Dachau), \uc791\uc13c\ud558\uc6b0\uc820(Sachsenhausen), \ubd80\ud5e8\ubc1c\ud2b8(Buchenwald), \uc624\ub77c\ub2c8\ubd80\ub974\ud06c(Oranieburg), \ub4f1\uc5d0 \uc788\ub294 \uac15\uc81c \uc218\uc6a9\uc18c\uc5d0 \uc218\uac10\ub418\uc5c8\ub2e4. \uadf8\ub4e4\uc740 \uba87 \uc8fc\uac04 \uc218\uac10\ub418\uc5c8\ub294\ub370 \uace7 \uadf8\ub4e4\uc774 \ub2e4\ub978 \uacf3\uc73c\ub85c \uc774\uc8fc\ub420 \uac83\uc774\ub77c \ud655\uc2e4\ud558\uac70\ub098 \uadf8\ub4e4\uc758 \uc7ac\uc0b0\uc744 \ub098\uce58\uc5d0 \uc591\ub3c4\ud560 \uacbd\uc6b0 \ud480\ub824\ub0a0 \uc218\ub3c4 \uc788\uc5c8\ub2e4. \ub3c5\uc77c \uc720\ub300\uc778\ub4e4\uc740 \ub300\ud559\uc0b4\uc744 \ud53c\ud558\uae30 \uc704\ud574 \ubb3c\uc9c8\uc801\uc73c\ub85c \ubcf4\uc0c1\ud558\uae30\ub3c4 \ud588\ub294\ub370, \uc774\ub294 \uc218 \ubc31, \uc218 \ucc9c \ub9c8\ub974\ud06c\uc600\uc73c\uba70 \uc774 \ubfd0\ub9cc \uc544\ub2c8\ub77c \u201c\uc18d\uc8c4 \uc138\uae08\u201d\uc73c\ub85c 10\uc5b5 \ub9c8\ub974\ud06c\uac00 \ub118\ub294 \ub3c8\uc744 \uc9c0\ubd88\ud574\uc57c \ud588\ub2e4. \uc774\ub7ec\ud55c \uc9d1\ub2e8\ud559\uc0b4 \uc774\ud6c4 \ub3c5\uc77c\ub85c\ubd80\ud130 \uc720\ub300\uc778\ub4e4\uc758 \uc774\uc8fc\ub294 \uac00\uc18d\ud654\ub418\uc5c8\uace0, \ub3c5\uc77c\uc5d0\uc11c \uacf5\uac1c\uc801\uc778 \uc720\ub300\uc778\uc758 \uc0b6\uc740 \ub05d\uc7a5\uc774 \ub0ac\ub2e4."} {"question": "DESIRE-\uc815\uc5f4-\uc774 \uc77c\ubcf8\ud48d\uc73c\ub85c \ubbf9\uc2f1\ub41c \uace1\uc778 \uac83\uacfc \ub2ec\ub9ac LA BOHEME\uc740 \uc5b4\ub5a4 \uc2a4\ud0c0\uc77c\uc758 \ubbf9\uc2f1 \uace1\uc778\uac00?", "paragraph": "A\uc0ac\uc774\ub4dc \uace1\uc778 \u3008DESIRE -\uc815\uc5f4-\u3009\uc740 \uc77c\ubcf8 \ud48d\uc73c\ub85c \ubbf9\uc2f1\ub418\uc5b4 \ud765\uaca8\uc6b4 \ubd84\uc704\uae30\ub97c \ub0b4\ub294 \uac83\uacfc \ub2ec\ub9ac B\uc0ac\uc774\ub4dc \uace1\uc778\u3008LA BOH\u00c8ME\u3009\uc740 \ud558\ub4dc \ub85d \uc2a4\ud0c0\uc77c\ub85c \ubbf9\uc2f1\ub41c \uace1\uc774\uc5c8\ub2e4. NHK\uc758 \uc74c\uc545\ubc29\uc1a1 \ud504\ub85c\uadf8\ub7a8 \u300a\uc601 \uc2a4\ud29c\ub514\uc624 101\u300b(\u30e4\u30f3\u30b0\u30b9\u30bf\u30b8\u30aa101)\uc5d0\uc11c \ucc98\uc74c \uc120\ubcf4\uc778 \uc774\ud6c4 \uc5ec\ub7ec \ubc29\uc1a1\uc0ac\uc758 \uc74c\uc545 \ud504\ub85c\uadf8\ub7a8\uc5d0\uc11c \uc790\uc8fc \ubd88\ub800\uc73c\uba70 \ucc28\ud2b8\uc758 \uc0c1\uc704\uad8c\uc5d0\ub3c4 \uc904\uace7 \uc62c\ub77c\uac14\ub2e4. \uc774\uc804 \uc2f1\uae00\ub4e4\uc5d0\uc120 B\uc0ac\uc774\ub4dc \uace1\uc744 \ubc29\uc1a1\uc5d0\uc11c \uc120\ubcf4\uc774\uc9c0 \uc54a\uc558\ub358 \uac83\uacfc\ub294 \ub2ec\ub9ac \ub098\uce74\ubaa8\ub9ac \uc544\ud0a4\ub098\ub294 \u3008DESIRE -\uc815\uc5f4-\u3009\uc758 \ud65c\ub3d9 \uc774\ud6c4 \ub530\ub85c \ud734\uc2dd \uae30\uac04\uc744 \ub450\uc9c0 \uc54a\uace0 \uacc4\uc18d \ubc29\uc1a1\uc5d0 \ucd9c\uc5f0\ud558\uc600\ub2e4. \uc774\ud6c4 \uc790\uc2e0\uc758 \ucf58\uc11c\ud2b8 \ud22c\uc5b4\uc5d0\uc11c\ub3c4 \uc5d4\ub529 \uace1\uc744 \uc7a5\uc2dd\ud558\ub294 \ub4f1 \ube44\ub85d B\uc0ac\uc774\ub4dc \uace1\uc774\uc5c8\uc9c0\ub9cc \ud32c\ub4e4\ub85c\ubd80\ud130 \ud070 \uc9c0\uc9c0\ub97c \uc5bb\uc740 \uace1\uc774\uc5c8\ub2e4.", "answer": "\ud558\ub4dc \ub85d \uc2a4\ud0c0\uc77c", "sentence": "A\uc0ac\uc774\ub4dc \uace1\uc778 \u3008DESIRE -\uc815\uc5f4-\u3009\uc740 \uc77c\ubcf8 \ud48d\uc73c\ub85c \ubbf9\uc2f1\ub418\uc5b4 \ud765\uaca8\uc6b4 \ubd84\uc704\uae30\ub97c \ub0b4\ub294 \uac83\uacfc \ub2ec\ub9ac B\uc0ac\uc774\ub4dc \uace1\uc778\u3008LA BOH\u00c8ME\u3009\uc740 \ud558\ub4dc \ub85d \uc2a4\ud0c0\uc77c \ub85c \ubbf9\uc2f1\ub41c \uace1\uc774\uc5c8\ub2e4.", "paragraph_sentence": " A\uc0ac\uc774\ub4dc \uace1\uc778 \u3008DESIRE -\uc815\uc5f4-\u3009\uc740 \uc77c\ubcf8 \ud48d\uc73c\ub85c \ubbf9\uc2f1\ub418\uc5b4 \ud765\uaca8\uc6b4 \ubd84\uc704\uae30\ub97c \ub0b4\ub294 \uac83\uacfc \ub2ec\ub9ac B\uc0ac\uc774\ub4dc \uace1\uc778\u3008LA BOH\u00c8ME\u3009\uc740 \ud558\ub4dc \ub85d \uc2a4\ud0c0\uc77c \ub85c \ubbf9\uc2f1\ub41c \uace1\uc774\uc5c8\ub2e4. NHK\uc758 \uc74c\uc545\ubc29\uc1a1 \ud504\ub85c\uadf8\ub7a8 \u300a\uc601 \uc2a4\ud29c\ub514\uc624 101\u300b(\u30e4\u30f3\u30b0\u30b9\u30bf\u30b8\u30aa101)\uc5d0\uc11c \ucc98\uc74c \uc120\ubcf4\uc778 \uc774\ud6c4 \uc5ec\ub7ec \ubc29\uc1a1\uc0ac\uc758 \uc74c\uc545 \ud504\ub85c\uadf8\ub7a8\uc5d0\uc11c \uc790\uc8fc \ubd88\ub800\uc73c\uba70 \ucc28\ud2b8\uc758 \uc0c1\uc704\uad8c\uc5d0\ub3c4 \uc904\uace7 \uc62c\ub77c\uac14\ub2e4. \uc774\uc804 \uc2f1\uae00\ub4e4\uc5d0\uc120 B\uc0ac\uc774\ub4dc \uace1\uc744 \ubc29\uc1a1\uc5d0\uc11c \uc120\ubcf4\uc774\uc9c0 \uc54a\uc558\ub358 \uac83\uacfc\ub294 \ub2ec\ub9ac \ub098\uce74\ubaa8\ub9ac \uc544\ud0a4\ub098\ub294 \u3008DESIRE -\uc815\uc5f4-\u3009\uc758 \ud65c\ub3d9 \uc774\ud6c4 \ub530\ub85c \ud734\uc2dd \uae30\uac04\uc744 \ub450\uc9c0 \uc54a\uace0 \uacc4\uc18d \ubc29\uc1a1\uc5d0 \ucd9c\uc5f0\ud558\uc600\ub2e4. \uc774\ud6c4 \uc790\uc2e0\uc758 \ucf58\uc11c\ud2b8 \ud22c\uc5b4\uc5d0\uc11c\ub3c4 \uc5d4\ub529 \uace1\uc744 \uc7a5\uc2dd\ud558\ub294 \ub4f1 \ube44\ub85d B\uc0ac\uc774\ub4dc \uace1\uc774\uc5c8\uc9c0\ub9cc \ud32c\ub4e4\ub85c\ubd80\ud130 \ud070 \uc9c0\uc9c0\ub97c \uc5bb\uc740 \uace1\uc774\uc5c8\ub2e4.", "paragraph_answer": "A\uc0ac\uc774\ub4dc \uace1\uc778 \u3008DESIRE -\uc815\uc5f4-\u3009\uc740 \uc77c\ubcf8 \ud48d\uc73c\ub85c \ubbf9\uc2f1\ub418\uc5b4 \ud765\uaca8\uc6b4 \ubd84\uc704\uae30\ub97c \ub0b4\ub294 \uac83\uacfc \ub2ec\ub9ac B\uc0ac\uc774\ub4dc \uace1\uc778\u3008LA BOH\u00c8ME\u3009\uc740 \ud558\ub4dc \ub85d \uc2a4\ud0c0\uc77c \ub85c \ubbf9\uc2f1\ub41c \uace1\uc774\uc5c8\ub2e4. NHK\uc758 \uc74c\uc545\ubc29\uc1a1 \ud504\ub85c\uadf8\ub7a8 \u300a\uc601 \uc2a4\ud29c\ub514\uc624 101\u300b(\u30e4\u30f3\u30b0\u30b9\u30bf\u30b8\u30aa101)\uc5d0\uc11c \ucc98\uc74c \uc120\ubcf4\uc778 \uc774\ud6c4 \uc5ec\ub7ec \ubc29\uc1a1\uc0ac\uc758 \uc74c\uc545 \ud504\ub85c\uadf8\ub7a8\uc5d0\uc11c \uc790\uc8fc \ubd88\ub800\uc73c\uba70 \ucc28\ud2b8\uc758 \uc0c1\uc704\uad8c\uc5d0\ub3c4 \uc904\uace7 \uc62c\ub77c\uac14\ub2e4. \uc774\uc804 \uc2f1\uae00\ub4e4\uc5d0\uc120 B\uc0ac\uc774\ub4dc \uace1\uc744 \ubc29\uc1a1\uc5d0\uc11c \uc120\ubcf4\uc774\uc9c0 \uc54a\uc558\ub358 \uac83\uacfc\ub294 \ub2ec\ub9ac \ub098\uce74\ubaa8\ub9ac \uc544\ud0a4\ub098\ub294 \u3008DESIRE -\uc815\uc5f4-\u3009\uc758 \ud65c\ub3d9 \uc774\ud6c4 \ub530\ub85c \ud734\uc2dd \uae30\uac04\uc744 \ub450\uc9c0 \uc54a\uace0 \uacc4\uc18d \ubc29\uc1a1\uc5d0 \ucd9c\uc5f0\ud558\uc600\ub2e4. \uc774\ud6c4 \uc790\uc2e0\uc758 \ucf58\uc11c\ud2b8 \ud22c\uc5b4\uc5d0\uc11c\ub3c4 \uc5d4\ub529 \uace1\uc744 \uc7a5\uc2dd\ud558\ub294 \ub4f1 \ube44\ub85d B\uc0ac\uc774\ub4dc \uace1\uc774\uc5c8\uc9c0\ub9cc \ud32c\ub4e4\ub85c\ubd80\ud130 \ud070 \uc9c0\uc9c0\ub97c \uc5bb\uc740 \uace1\uc774\uc5c8\ub2e4.", "sentence_answer": "A\uc0ac\uc774\ub4dc \uace1\uc778 \u3008DESIRE -\uc815\uc5f4-\u3009\uc740 \uc77c\ubcf8 \ud48d\uc73c\ub85c \ubbf9\uc2f1\ub418\uc5b4 \ud765\uaca8\uc6b4 \ubd84\uc704\uae30\ub97c \ub0b4\ub294 \uac83\uacfc \ub2ec\ub9ac B\uc0ac\uc774\ub4dc \uace1\uc778\u3008LA BOH\u00c8ME\u3009\uc740 \ud558\ub4dc \ub85d \uc2a4\ud0c0\uc77c \ub85c \ubbf9\uc2f1\ub41c \uace1\uc774\uc5c8\ub2e4.", "paragraph_id": "6564696-2-1", "paragraph_question": "question: DESIRE-\uc815\uc5f4-\uc774 \uc77c\ubcf8\ud48d\uc73c\ub85c \ubbf9\uc2f1\ub41c \uace1\uc778 \uac83\uacfc \ub2ec\ub9ac LA BOHEME\uc740 \uc5b4\ub5a4 \uc2a4\ud0c0\uc77c\uc758 \ubbf9\uc2f1 \uace1\uc778\uac00?, context: A\uc0ac\uc774\ub4dc \uace1\uc778 \u3008DESIRE -\uc815\uc5f4-\u3009\uc740 \uc77c\ubcf8 \ud48d\uc73c\ub85c \ubbf9\uc2f1\ub418\uc5b4 \ud765\uaca8\uc6b4 \ubd84\uc704\uae30\ub97c \ub0b4\ub294 \uac83\uacfc \ub2ec\ub9ac B\uc0ac\uc774\ub4dc \uace1\uc778\u3008LA BOH\u00c8ME\u3009\uc740 \ud558\ub4dc \ub85d \uc2a4\ud0c0\uc77c\ub85c \ubbf9\uc2f1\ub41c \uace1\uc774\uc5c8\ub2e4. NHK\uc758 \uc74c\uc545\ubc29\uc1a1 \ud504\ub85c\uadf8\ub7a8 \u300a\uc601 \uc2a4\ud29c\ub514\uc624 101\u300b(\u30e4\u30f3\u30b0\u30b9\u30bf\u30b8\u30aa101)\uc5d0\uc11c \ucc98\uc74c \uc120\ubcf4\uc778 \uc774\ud6c4 \uc5ec\ub7ec \ubc29\uc1a1\uc0ac\uc758 \uc74c\uc545 \ud504\ub85c\uadf8\ub7a8\uc5d0\uc11c \uc790\uc8fc \ubd88\ub800\uc73c\uba70 \ucc28\ud2b8\uc758 \uc0c1\uc704\uad8c\uc5d0\ub3c4 \uc904\uace7 \uc62c\ub77c\uac14\ub2e4. \uc774\uc804 \uc2f1\uae00\ub4e4\uc5d0\uc120 B\uc0ac\uc774\ub4dc \uace1\uc744 \ubc29\uc1a1\uc5d0\uc11c \uc120\ubcf4\uc774\uc9c0 \uc54a\uc558\ub358 \uac83\uacfc\ub294 \ub2ec\ub9ac \ub098\uce74\ubaa8\ub9ac \uc544\ud0a4\ub098\ub294 \u3008DESIRE -\uc815\uc5f4-\u3009\uc758 \ud65c\ub3d9 \uc774\ud6c4 \ub530\ub85c \ud734\uc2dd \uae30\uac04\uc744 \ub450\uc9c0 \uc54a\uace0 \uacc4\uc18d \ubc29\uc1a1\uc5d0 \ucd9c\uc5f0\ud558\uc600\ub2e4. \uc774\ud6c4 \uc790\uc2e0\uc758 \ucf58\uc11c\ud2b8 \ud22c\uc5b4\uc5d0\uc11c\ub3c4 \uc5d4\ub529 \uace1\uc744 \uc7a5\uc2dd\ud558\ub294 \ub4f1 \ube44\ub85d B\uc0ac\uc774\ub4dc \uace1\uc774\uc5c8\uc9c0\ub9cc \ud32c\ub4e4\ub85c\ubd80\ud130 \ud070 \uc9c0\uc9c0\ub97c \uc5bb\uc740 \uace1\uc774\uc5c8\ub2e4."} {"question": "\ud0c0\uc774\uc82c\uc740 \uc5b4\ub5a4 \uc0ac\uc774\ud2b8\uc778\uac00?", "paragraph": "\ud55c\uad6d\uc758 \uc628\ub77c\uc778 \ubc14\ub451\uc804\ubb38 \uc0ac\uc774\ud2b8\uc778 \ud0c0\uc774\uc82c\uc5d0 \ub4f1\uc7a5\ud55c Magister\ub77c\ub294 \uc544\uc774\ub514\ub294 2016\ub144 12\uc6d4 29\uc77c \ubd80\ud130 31\uc77c \uae4c\uc9c0 30\uc804 30\uc2b9\uc744 \uae30\ub85d\ud558\uc600\uace0, \uc911\uad6d\uc758 \ud150\uc13c\ud2b8\uac00 \uc11c\ube44\uc2a4 \uc911\uc778 \ud55c\ud050\ubc14\ub451\uc5d0 \ub4f1\uc7a5\ud55c Master\ub77c\ub294 \uc544\uc774\ub514\ub3c4 2017\ub144 1\uc6d4 2\uc77c \ubd80\ud130 4\uc77c\uae4c\uc9c0 30\uc804 30\uc2b9\uc744 \uae30\ub85d\ud558\uba74\uc11c \ubc14\ub451\uacc4\ub294 \ube44\uc0c1\ud55c \uad00\uc2ec\uc744 \ubcf4\uc600\ub2e4. \uc5f0\uc2b9\uc5d0 \ub180\ub780 \ud55c \uc911\uad6d \uae30\uc0ac\ub294 10\ub9cc \uc704\uc548\uc758 \ud604\uc0c1\uae08\uc744 \uac78\uae30\ub3c4 \ud588\ub294\ub370, Magister\uc640 Master\ub294 \ubc24\ub0ae\uc5c6\uc774 \ub300\uad6d\uc744 \ud558\uba74\uc11c\ub3c4 \ub300\ubd80\ubd84 170\uc218 \uadfc\ucc98\uc5d0\uc11c \ubd88\uacc4\uc2b9\uc73c\ub85c \ub05d\ub0b4\ub294 \ub4f1 \uc555\ub3c4\uc801\uc778 \uae30\ub825\uc744 \uc120\ubcf4\uc5ec \uc0ac\ub78c\uc774 \uc544\ub2c8\ub77c \ub3d9\uc77c\ud55c \uc778\uacf5\uc9c0\ub2a5 \uc989 \uc54c\ud30c\uace0\ub77c\ub294 \ucd94\uce21\uc774 \ub098\ub3cc\uc558\ub2e4. 2017\ub144 1\uc6d4 4\uc77c \ub370\ubbf8\uc2a4 \ud558\uc0ac\ube44\uc2a4 CEO\ub294 \ud2b8\uc704\ud130\ub97c \ud1b5\ud574 \uc218\uc218\uaed8\ub07c\uc758 \ubc14\ub451\uacc4\uc815 Magister\uc640 Master\ub294 \uc54c\ud30c\uace0\uc758 \uc0c8\ub85c\uc6b4 \ubc84\uc804\uc774\ub77c\uace0 \ubc1d\ud614\ub2e4.", "answer": "\ubc14\ub451\uc804\ubb38 \uc0ac\uc774\ud2b8", "sentence": "\ud55c\uad6d\uc758 \uc628\ub77c\uc778 \ubc14\ub451\uc804\ubb38 \uc0ac\uc774\ud2b8 \uc778 \ud0c0\uc774\uc82c\uc5d0 \ub4f1\uc7a5\ud55c Magister\ub77c\ub294 \uc544\uc774\ub514\ub294 2016\ub144 12\uc6d4 29\uc77c \ubd80\ud130 31\uc77c \uae4c\uc9c0 30\uc804 30\uc2b9\uc744 \uae30\ub85d\ud558\uc600\uace0, \uc911\uad6d\uc758 \ud150\uc13c\ud2b8\uac00 \uc11c\ube44\uc2a4 \uc911\uc778 \ud55c\ud050\ubc14\ub451\uc5d0 \ub4f1\uc7a5\ud55c Master\ub77c\ub294 \uc544\uc774\ub514\ub3c4 2017\ub144 1\uc6d4 2\uc77c \ubd80\ud130 4\uc77c\uae4c\uc9c0 30\uc804 30\uc2b9\uc744 \uae30\ub85d\ud558\uba74\uc11c \ubc14\ub451\uacc4\ub294 \ube44\uc0c1\ud55c \uad00\uc2ec\uc744 \ubcf4\uc600\ub2e4.", "paragraph_sentence": " \ud55c\uad6d\uc758 \uc628\ub77c\uc778 \ubc14\ub451\uc804\ubb38 \uc0ac\uc774\ud2b8 \uc778 \ud0c0\uc774\uc82c\uc5d0 \ub4f1\uc7a5\ud55c Magister\ub77c\ub294 \uc544\uc774\ub514\ub294 2016\ub144 12\uc6d4 29\uc77c \ubd80\ud130 31\uc77c \uae4c\uc9c0 30\uc804 30\uc2b9\uc744 \uae30\ub85d\ud558\uc600\uace0, \uc911\uad6d\uc758 \ud150\uc13c\ud2b8\uac00 \uc11c\ube44\uc2a4 \uc911\uc778 \ud55c\ud050\ubc14\ub451\uc5d0 \ub4f1\uc7a5\ud55c Master\ub77c\ub294 \uc544\uc774\ub514\ub3c4 2017\ub144 1\uc6d4 2\uc77c \ubd80\ud130 4\uc77c\uae4c\uc9c0 30\uc804 30\uc2b9\uc744 \uae30\ub85d\ud558\uba74\uc11c \ubc14\ub451\uacc4\ub294 \ube44\uc0c1\ud55c \uad00\uc2ec\uc744 \ubcf4\uc600\ub2e4. \uc5f0\uc2b9\uc5d0 \ub180\ub780 \ud55c \uc911\uad6d \uae30\uc0ac\ub294 10\ub9cc \uc704\uc548\uc758 \ud604\uc0c1\uae08\uc744 \uac78\uae30\ub3c4 \ud588\ub294\ub370, Magister\uc640 Master\ub294 \ubc24\ub0ae\uc5c6\uc774 \ub300\uad6d\uc744 \ud558\uba74\uc11c\ub3c4 \ub300\ubd80\ubd84 170\uc218 \uadfc\ucc98\uc5d0\uc11c \ubd88\uacc4\uc2b9\uc73c\ub85c \ub05d\ub0b4\ub294 \ub4f1 \uc555\ub3c4\uc801\uc778 \uae30\ub825\uc744 \uc120\ubcf4\uc5ec \uc0ac\ub78c\uc774 \uc544\ub2c8\ub77c \ub3d9\uc77c\ud55c \uc778\uacf5\uc9c0\ub2a5 \uc989 \uc54c\ud30c\uace0\ub77c\ub294 \ucd94\uce21\uc774 \ub098\ub3cc\uc558\ub2e4. 2017\ub144 1\uc6d4 4\uc77c \ub370\ubbf8\uc2a4 \ud558\uc0ac\ube44\uc2a4 CEO\ub294 \ud2b8\uc704\ud130\ub97c \ud1b5\ud574 \uc218\uc218\uaed8\ub07c\uc758 \ubc14\ub451\uacc4\uc815 Magister\uc640 Master\ub294 \uc54c\ud30c\uace0\uc758 \uc0c8\ub85c\uc6b4 \ubc84\uc804\uc774\ub77c\uace0 \ubc1d\ud614\ub2e4.", "paragraph_answer": "\ud55c\uad6d\uc758 \uc628\ub77c\uc778 \ubc14\ub451\uc804\ubb38 \uc0ac\uc774\ud2b8 \uc778 \ud0c0\uc774\uc82c\uc5d0 \ub4f1\uc7a5\ud55c Magister\ub77c\ub294 \uc544\uc774\ub514\ub294 2016\ub144 12\uc6d4 29\uc77c \ubd80\ud130 31\uc77c \uae4c\uc9c0 30\uc804 30\uc2b9\uc744 \uae30\ub85d\ud558\uc600\uace0, \uc911\uad6d\uc758 \ud150\uc13c\ud2b8\uac00 \uc11c\ube44\uc2a4 \uc911\uc778 \ud55c\ud050\ubc14\ub451\uc5d0 \ub4f1\uc7a5\ud55c Master\ub77c\ub294 \uc544\uc774\ub514\ub3c4 2017\ub144 1\uc6d4 2\uc77c \ubd80\ud130 4\uc77c\uae4c\uc9c0 30\uc804 30\uc2b9\uc744 \uae30\ub85d\ud558\uba74\uc11c \ubc14\ub451\uacc4\ub294 \ube44\uc0c1\ud55c \uad00\uc2ec\uc744 \ubcf4\uc600\ub2e4. \uc5f0\uc2b9\uc5d0 \ub180\ub780 \ud55c \uc911\uad6d \uae30\uc0ac\ub294 10\ub9cc \uc704\uc548\uc758 \ud604\uc0c1\uae08\uc744 \uac78\uae30\ub3c4 \ud588\ub294\ub370, Magister\uc640 Master\ub294 \ubc24\ub0ae\uc5c6\uc774 \ub300\uad6d\uc744 \ud558\uba74\uc11c\ub3c4 \ub300\ubd80\ubd84 170\uc218 \uadfc\ucc98\uc5d0\uc11c \ubd88\uacc4\uc2b9\uc73c\ub85c \ub05d\ub0b4\ub294 \ub4f1 \uc555\ub3c4\uc801\uc778 \uae30\ub825\uc744 \uc120\ubcf4\uc5ec \uc0ac\ub78c\uc774 \uc544\ub2c8\ub77c \ub3d9\uc77c\ud55c \uc778\uacf5\uc9c0\ub2a5 \uc989 \uc54c\ud30c\uace0\ub77c\ub294 \ucd94\uce21\uc774 \ub098\ub3cc\uc558\ub2e4. 2017\ub144 1\uc6d4 4\uc77c \ub370\ubbf8\uc2a4 \ud558\uc0ac\ube44\uc2a4 CEO\ub294 \ud2b8\uc704\ud130\ub97c \ud1b5\ud574 \uc218\uc218\uaed8\ub07c\uc758 \ubc14\ub451\uacc4\uc815 Magister\uc640 Master\ub294 \uc54c\ud30c\uace0\uc758 \uc0c8\ub85c\uc6b4 \ubc84\uc804\uc774\ub77c\uace0 \ubc1d\ud614\ub2e4.", "sentence_answer": "\ud55c\uad6d\uc758 \uc628\ub77c\uc778 \ubc14\ub451\uc804\ubb38 \uc0ac\uc774\ud2b8 \uc778 \ud0c0\uc774\uc82c\uc5d0 \ub4f1\uc7a5\ud55c Magister\ub77c\ub294 \uc544\uc774\ub514\ub294 2016\ub144 12\uc6d4 29\uc77c \ubd80\ud130 31\uc77c \uae4c\uc9c0 30\uc804 30\uc2b9\uc744 \uae30\ub85d\ud558\uc600\uace0, \uc911\uad6d\uc758 \ud150\uc13c\ud2b8\uac00 \uc11c\ube44\uc2a4 \uc911\uc778 \ud55c\ud050\ubc14\ub451\uc5d0 \ub4f1\uc7a5\ud55c Master\ub77c\ub294 \uc544\uc774\ub514\ub3c4 2017\ub144 1\uc6d4 2\uc77c \ubd80\ud130 4\uc77c\uae4c\uc9c0 30\uc804 30\uc2b9\uc744 \uae30\ub85d\ud558\uba74\uc11c \ubc14\ub451\uacc4\ub294 \ube44\uc0c1\ud55c \uad00\uc2ec\uc744 \ubcf4\uc600\ub2e4.", "paragraph_id": "6604916-3-0", "paragraph_question": "question: \ud0c0\uc774\uc82c\uc740 \uc5b4\ub5a4 \uc0ac\uc774\ud2b8\uc778\uac00?, context: \ud55c\uad6d\uc758 \uc628\ub77c\uc778 \ubc14\ub451\uc804\ubb38 \uc0ac\uc774\ud2b8\uc778 \ud0c0\uc774\uc82c\uc5d0 \ub4f1\uc7a5\ud55c Magister\ub77c\ub294 \uc544\uc774\ub514\ub294 2016\ub144 12\uc6d4 29\uc77c \ubd80\ud130 31\uc77c \uae4c\uc9c0 30\uc804 30\uc2b9\uc744 \uae30\ub85d\ud558\uc600\uace0, \uc911\uad6d\uc758 \ud150\uc13c\ud2b8\uac00 \uc11c\ube44\uc2a4 \uc911\uc778 \ud55c\ud050\ubc14\ub451\uc5d0 \ub4f1\uc7a5\ud55c Master\ub77c\ub294 \uc544\uc774\ub514\ub3c4 2017\ub144 1\uc6d4 2\uc77c \ubd80\ud130 4\uc77c\uae4c\uc9c0 30\uc804 30\uc2b9\uc744 \uae30\ub85d\ud558\uba74\uc11c \ubc14\ub451\uacc4\ub294 \ube44\uc0c1\ud55c \uad00\uc2ec\uc744 \ubcf4\uc600\ub2e4. \uc5f0\uc2b9\uc5d0 \ub180\ub780 \ud55c \uc911\uad6d \uae30\uc0ac\ub294 10\ub9cc \uc704\uc548\uc758 \ud604\uc0c1\uae08\uc744 \uac78\uae30\ub3c4 \ud588\ub294\ub370, Magister\uc640 Master\ub294 \ubc24\ub0ae\uc5c6\uc774 \ub300\uad6d\uc744 \ud558\uba74\uc11c\ub3c4 \ub300\ubd80\ubd84 170\uc218 \uadfc\ucc98\uc5d0\uc11c \ubd88\uacc4\uc2b9\uc73c\ub85c \ub05d\ub0b4\ub294 \ub4f1 \uc555\ub3c4\uc801\uc778 \uae30\ub825\uc744 \uc120\ubcf4\uc5ec \uc0ac\ub78c\uc774 \uc544\ub2c8\ub77c \ub3d9\uc77c\ud55c \uc778\uacf5\uc9c0\ub2a5 \uc989 \uc54c\ud30c\uace0\ub77c\ub294 \ucd94\uce21\uc774 \ub098\ub3cc\uc558\ub2e4. 2017\ub144 1\uc6d4 4\uc77c \ub370\ubbf8\uc2a4 \ud558\uc0ac\ube44\uc2a4 CEO\ub294 \ud2b8\uc704\ud130\ub97c \ud1b5\ud574 \uc218\uc218\uaed8\ub07c\uc758 \ubc14\ub451\uacc4\uc815 Magister\uc640 Master\ub294 \uc54c\ud30c\uace0\uc758 \uc0c8\ub85c\uc6b4 \ubc84\uc804\uc774\ub77c\uace0 \ubc1d\ud614\ub2e4."} {"question": "\ubc15\uaca9\ud3ec\uc18c\ub300\uc7a5 \uc911\uc704\uc758 \uc774\ub984\uc740 \ubb34\uc5c7\uc785\ub2c8\uae4c?", "paragraph": "\uc0dd\uc874\uc790 \uc99d\uc5b8\uc5d0 \ub530\ub974\uba74, 8\uc6d4 15\uc77c \uc0c8\ubcbd\uc774 \ub418\uae30 \uc804\uc5d0 H\uc911\ub300 \ubc15\uaca9\ud3ec\uc18c\ub300\uc6d0\ub4e4\uc774 303\uace0\uc9c0 \uc8fc\ubcc0\uc758 \uc801\uc758 \uc6c0\uc9c1\uc784\uc744 \uc54c\uc544\ucc58\ub2e4\uace0 \ud55c\ub2e4. \uc18c\ub300\uc7a5\uc740 G\uc911\ub300\uc640 5\uae30\ubcd1\uc0ac\ub2e8\uc5d0 \uc5f0\ub77d\ud558\uc5ec \ub0a8\ud55c \uad6d\uad70 1\uac1c \uc18c\ub300 60\uba85\uc774 \ubc15\uaca9\ud3ec\uc18c\ub300\ub97c \uc9c0\uc6d0\ud558\uae30 \uc704\ud574 \ub3c4\ucc29\ud588\ub2e4\uace0 \uc54c\ub838\ub2e4. \uadf8\ub7ec\ub098 \ud574\uac00 \ub728\uc790 \uadf8\ub4e4\uc774 \ubc1c\uacac\ud55c \uac83\uc740 \uc778\ubbfc\uad70\uc758 T-34 \ub450 \ub300\uc640 \uadf8 \ub4a4\ub97c \ub530\ub974\ub294 200 \uba85 \uc774\uc0c1\uc758 \uc778\ubbfc\uad70\uc774\uc5c8\ub2e4. \uc7a0\uc2dc \ub4a4 \ud55c\uad6d\uc778 \uba87 \uba85\uc774 \uc57c\uc0b0 \uc0ac\uba74\uc744 \ud0c0\uace0 \ub098\ud0c0\ub0ac\ub2e4. \ucd08\ubcd1\uc774 \uadf8\ub4e4\uc744 \ubd88\ub800\uc73c\ub098 \ud55c\uad6d\uc778\ub4e4\uc740 \uc790\ub3d9\uc18c\ucd1d \ub09c\uc0ac\ub85c \ub300\ub2f5\ud588\ub2e4. \ub2f9\uc2dc \ubc15\uaca9\ud3ec\uc18c\ub300\uc7a5 \uc7ad \ud5c8\ub4dc\uc2a4\ud398\uc2a4(Jack Hudspeth) \uc911\uc704\ub294 \uadf8\ub4e4\uc774 \uc544\uad70\uc774\ub77c\uace0 \uc0dd\uac01\ud588\ub2e4. \ud504\ub808\ub4dc \ub77c\uc774\uc5b8(Fred Ryan) \uc774\ubcd1\uacfc \ub85c\uc774 \ub9e8\ub9c1(Roy Manring) \uc774\ubcd1\uc774 1999\ub144 \ud55c\uad6d\uc744 \uc7ac\ubc29\ubb38\ud574 \uc99d\uc5b8\ud55c \ubc14\uc5d0 \ub530\ub974\uba74, \uc77c\ubd80 \ubbf8\uad70 \ubcd1\uc0ac\ub4e4\uc740 \uc811\uadfc\ud558\ub294 \ubcd1\ub825\uc774 \uc778\ubbfc\uad70\uc774\ub77c\ub294 \uac83\uc744 \uc54c\uc544\ucc58\uace0 \ubc1c\ud3ec\ub97c \uc900\ube44\ud588\ub2e4. \uadf8\ub7ec\ub098 \ud5c8\ub4dc\uc2a4\ud398\uc2a4 \uc18c\ub300\uc7a5\uc774 \ubc1c\ud3ec \uc911\uc9c0\ub97c \uba85\ub839\ud558\uba70 \ubc1c\ud3ec\ud55c\ub2e4\uba74 \uad70\uc0ac\ubc95\uc815\uc5d0 \ud68c\ubd80\uc2dc\ud0ac \uac83\uc774\ub77c\uace0 \uc704\ud611\ud588\ub2e4. \ub098\uba38\uc9c0 \ubbf8\uad70\ub4e4\uc740 \uc811\uadfc\ud558\ub294 \ud55c\uad6d\uc778\ub4e4\uc774 \ucda9\ubd84\ud788 \uac00\uae4c\uc6cc\uc838 \uadf8\ub4e4 \uad70\ubaa8\uc758 \ubd89\uc740 \ubcc4\uc774 \ubcf4\uc77c \ub54c\uae4c\uc9c0 \uc801\uc778\uc9c0 \uc544\uad70\uc778\uc9c0 \uc54c\uc9c0 \ubabb\ud588\ub2e4. \uc774 \uc2dc\uc810\uc5d0\uc11c \uc778\ubbfc\uad70\uc740 \uc774\ubbf8 \ubbf8\uad70\ub4e4\uc758 \uc704\uce58\uc5d0 \ubc14\uc9dd \ub2e4\uac00\uc628 \uc0c1\ud0dc\uc600\ub2e4. \uc778\ubbfc\uad70\uc740 \ucd1d \ud55c \ubc1c \ub9de\uc9c0 \uc54a\uace0 \ucc38\ud638\uae4c\uc9c0 \ub3c4\ucc29\ud588\ub2e4. \uc18c\ub300\uc7a5\uc740 \ubcd1\ub825\uacfc \ud654\ub825\uc758 \uc5f4\uc138\ub97c \uc774\uae38 \uc218 \uc5c6\ub2e4\uace0 \ud310\ub2e8\ud558\uace0 \ud56d\ubcf5\ud588\ub2e4. \uc778\ubbfc\uad70\uc740 \uc18c\ub300\uc6d0 31\uba85\uc744 \ubaa8\ub450 \ud3ec\ub85c\ub85c \uc7a1\uc558\ub2e4. \uadf8\ub7ec\ub098 \uc99d\uc5b8\uc5d0 \ub530\ub77c\uc11c\ub294 \ud3ec\ub85c\ub85c \uc7a1\ud78c \ubcd1\uc0ac\uac00 42\uba85\uc774\uc5c8\ub2e4\uace0\ub3c4 \ud55c\ub2e4.", "answer": "\uc7ad \ud5c8\ub4dc\uc2a4\ud398\uc2a4", "sentence": "\ub2f9\uc2dc \ubc15\uaca9\ud3ec\uc18c\ub300\uc7a5 \uc7ad \ud5c8\ub4dc\uc2a4\ud398\uc2a4 (Jack Hudspeth) \uc911\uc704\ub294 \uadf8\ub4e4\uc774 \uc544\uad70\uc774\ub77c\uace0 \uc0dd\uac01\ud588\ub2e4.", "paragraph_sentence": "\uc0dd\uc874\uc790 \uc99d\uc5b8\uc5d0 \ub530\ub974\uba74, 8\uc6d4 15\uc77c \uc0c8\ubcbd\uc774 \ub418\uae30 \uc804\uc5d0 H\uc911\ub300 \ubc15\uaca9\ud3ec\uc18c\ub300\uc6d0\ub4e4\uc774 303\uace0\uc9c0 \uc8fc\ubcc0\uc758 \uc801\uc758 \uc6c0\uc9c1\uc784\uc744 \uc54c\uc544\ucc58\ub2e4\uace0 \ud55c\ub2e4. \uc18c\ub300\uc7a5\uc740 G\uc911\ub300\uc640 5\uae30\ubcd1\uc0ac\ub2e8\uc5d0 \uc5f0\ub77d\ud558\uc5ec \ub0a8\ud55c \uad6d\uad70 1\uac1c \uc18c\ub300 60\uba85\uc774 \ubc15\uaca9\ud3ec\uc18c\ub300\ub97c \uc9c0\uc6d0\ud558\uae30 \uc704\ud574 \ub3c4\ucc29\ud588\ub2e4\uace0 \uc54c\ub838\ub2e4. \uadf8\ub7ec\ub098 \ud574\uac00 \ub728\uc790 \uadf8\ub4e4\uc774 \ubc1c\uacac\ud55c \uac83\uc740 \uc778\ubbfc\uad70\uc758 T-34 \ub450 \ub300\uc640 \uadf8 \ub4a4\ub97c \ub530\ub974\ub294 200 \uba85 \uc774\uc0c1\uc758 \uc778\ubbfc\uad70\uc774\uc5c8\ub2e4. \uc7a0\uc2dc \ub4a4 \ud55c\uad6d\uc778 \uba87 \uba85\uc774 \uc57c\uc0b0 \uc0ac\uba74\uc744 \ud0c0\uace0 \ub098\ud0c0\ub0ac\ub2e4. \ucd08\ubcd1\uc774 \uadf8\ub4e4\uc744 \ubd88\ub800\uc73c\ub098 \ud55c\uad6d\uc778\ub4e4\uc740 \uc790\ub3d9\uc18c\ucd1d \ub09c\uc0ac\ub85c \ub300\ub2f5\ud588\ub2e4. \ub2f9\uc2dc \ubc15\uaca9\ud3ec\uc18c\ub300\uc7a5 \uc7ad \ud5c8\ub4dc\uc2a4\ud398\uc2a4 (Jack Hudspeth) \uc911\uc704\ub294 \uadf8\ub4e4\uc774 \uc544\uad70\uc774\ub77c\uace0 \uc0dd\uac01\ud588\ub2e4. \ud504\ub808\ub4dc \ub77c\uc774\uc5b8(Fred Ryan) \uc774\ubcd1\uacfc \ub85c\uc774 \ub9e8\ub9c1(Roy Manring) \uc774\ubcd1\uc774 1999\ub144 \ud55c\uad6d\uc744 \uc7ac\ubc29\ubb38\ud574 \uc99d\uc5b8\ud55c \ubc14\uc5d0 \ub530\ub974\uba74, \uc77c\ubd80 \ubbf8\uad70 \ubcd1\uc0ac\ub4e4\uc740 \uc811\uadfc\ud558\ub294 \ubcd1\ub825\uc774 \uc778\ubbfc\uad70\uc774\ub77c\ub294 \uac83\uc744 \uc54c\uc544\ucc58\uace0 \ubc1c\ud3ec\ub97c \uc900\ube44\ud588\ub2e4. \uadf8\ub7ec\ub098 \ud5c8\ub4dc\uc2a4\ud398\uc2a4 \uc18c\ub300\uc7a5\uc774 \ubc1c\ud3ec \uc911\uc9c0\ub97c \uba85\ub839\ud558\uba70 \ubc1c\ud3ec\ud55c\ub2e4\uba74 \uad70\uc0ac\ubc95\uc815\uc5d0 \ud68c\ubd80\uc2dc\ud0ac \uac83\uc774\ub77c\uace0 \uc704\ud611\ud588\ub2e4. \ub098\uba38\uc9c0 \ubbf8\uad70\ub4e4\uc740 \uc811\uadfc\ud558\ub294 \ud55c\uad6d\uc778\ub4e4\uc774 \ucda9\ubd84\ud788 \uac00\uae4c\uc6cc\uc838 \uadf8\ub4e4 \uad70\ubaa8\uc758 \ubd89\uc740 \ubcc4\uc774 \ubcf4\uc77c \ub54c\uae4c\uc9c0 \uc801\uc778\uc9c0 \uc544\uad70\uc778\uc9c0 \uc54c\uc9c0 \ubabb\ud588\ub2e4. \uc774 \uc2dc\uc810\uc5d0\uc11c \uc778\ubbfc\uad70\uc740 \uc774\ubbf8 \ubbf8\uad70\ub4e4\uc758 \uc704\uce58\uc5d0 \ubc14\uc9dd \ub2e4\uac00\uc628 \uc0c1\ud0dc\uc600\ub2e4. \uc778\ubbfc\uad70\uc740 \ucd1d \ud55c \ubc1c \ub9de\uc9c0 \uc54a\uace0 \ucc38\ud638\uae4c\uc9c0 \ub3c4\ucc29\ud588\ub2e4. \uc18c\ub300\uc7a5\uc740 \ubcd1\ub825\uacfc \ud654\ub825\uc758 \uc5f4\uc138\ub97c \uc774\uae38 \uc218 \uc5c6\ub2e4\uace0 \ud310\ub2e8\ud558\uace0 \ud56d\ubcf5\ud588\ub2e4. \uc778\ubbfc\uad70\uc740 \uc18c\ub300\uc6d0 31\uba85\uc744 \ubaa8\ub450 \ud3ec\ub85c\ub85c \uc7a1\uc558\ub2e4. \uadf8\ub7ec\ub098 \uc99d\uc5b8\uc5d0 \ub530\ub77c\uc11c\ub294 \ud3ec\ub85c\ub85c \uc7a1\ud78c \ubcd1\uc0ac\uac00 42\uba85\uc774\uc5c8\ub2e4\uace0\ub3c4 \ud55c\ub2e4.", "paragraph_answer": "\uc0dd\uc874\uc790 \uc99d\uc5b8\uc5d0 \ub530\ub974\uba74, 8\uc6d4 15\uc77c \uc0c8\ubcbd\uc774 \ub418\uae30 \uc804\uc5d0 H\uc911\ub300 \ubc15\uaca9\ud3ec\uc18c\ub300\uc6d0\ub4e4\uc774 303\uace0\uc9c0 \uc8fc\ubcc0\uc758 \uc801\uc758 \uc6c0\uc9c1\uc784\uc744 \uc54c\uc544\ucc58\ub2e4\uace0 \ud55c\ub2e4. \uc18c\ub300\uc7a5\uc740 G\uc911\ub300\uc640 5\uae30\ubcd1\uc0ac\ub2e8\uc5d0 \uc5f0\ub77d\ud558\uc5ec \ub0a8\ud55c \uad6d\uad70 1\uac1c \uc18c\ub300 60\uba85\uc774 \ubc15\uaca9\ud3ec\uc18c\ub300\ub97c \uc9c0\uc6d0\ud558\uae30 \uc704\ud574 \ub3c4\ucc29\ud588\ub2e4\uace0 \uc54c\ub838\ub2e4. \uadf8\ub7ec\ub098 \ud574\uac00 \ub728\uc790 \uadf8\ub4e4\uc774 \ubc1c\uacac\ud55c \uac83\uc740 \uc778\ubbfc\uad70\uc758 T-34 \ub450 \ub300\uc640 \uadf8 \ub4a4\ub97c \ub530\ub974\ub294 200 \uba85 \uc774\uc0c1\uc758 \uc778\ubbfc\uad70\uc774\uc5c8\ub2e4. \uc7a0\uc2dc \ub4a4 \ud55c\uad6d\uc778 \uba87 \uba85\uc774 \uc57c\uc0b0 \uc0ac\uba74\uc744 \ud0c0\uace0 \ub098\ud0c0\ub0ac\ub2e4. \ucd08\ubcd1\uc774 \uadf8\ub4e4\uc744 \ubd88\ub800\uc73c\ub098 \ud55c\uad6d\uc778\ub4e4\uc740 \uc790\ub3d9\uc18c\ucd1d \ub09c\uc0ac\ub85c \ub300\ub2f5\ud588\ub2e4. \ub2f9\uc2dc \ubc15\uaca9\ud3ec\uc18c\ub300\uc7a5 \uc7ad \ud5c8\ub4dc\uc2a4\ud398\uc2a4 (Jack Hudspeth) \uc911\uc704\ub294 \uadf8\ub4e4\uc774 \uc544\uad70\uc774\ub77c\uace0 \uc0dd\uac01\ud588\ub2e4. \ud504\ub808\ub4dc \ub77c\uc774\uc5b8(Fred Ryan) \uc774\ubcd1\uacfc \ub85c\uc774 \ub9e8\ub9c1(Roy Manring) \uc774\ubcd1\uc774 1999\ub144 \ud55c\uad6d\uc744 \uc7ac\ubc29\ubb38\ud574 \uc99d\uc5b8\ud55c \ubc14\uc5d0 \ub530\ub974\uba74, \uc77c\ubd80 \ubbf8\uad70 \ubcd1\uc0ac\ub4e4\uc740 \uc811\uadfc\ud558\ub294 \ubcd1\ub825\uc774 \uc778\ubbfc\uad70\uc774\ub77c\ub294 \uac83\uc744 \uc54c\uc544\ucc58\uace0 \ubc1c\ud3ec\ub97c \uc900\ube44\ud588\ub2e4. \uadf8\ub7ec\ub098 \ud5c8\ub4dc\uc2a4\ud398\uc2a4 \uc18c\ub300\uc7a5\uc774 \ubc1c\ud3ec \uc911\uc9c0\ub97c \uba85\ub839\ud558\uba70 \ubc1c\ud3ec\ud55c\ub2e4\uba74 \uad70\uc0ac\ubc95\uc815\uc5d0 \ud68c\ubd80\uc2dc\ud0ac \uac83\uc774\ub77c\uace0 \uc704\ud611\ud588\ub2e4. \ub098\uba38\uc9c0 \ubbf8\uad70\ub4e4\uc740 \uc811\uadfc\ud558\ub294 \ud55c\uad6d\uc778\ub4e4\uc774 \ucda9\ubd84\ud788 \uac00\uae4c\uc6cc\uc838 \uadf8\ub4e4 \uad70\ubaa8\uc758 \ubd89\uc740 \ubcc4\uc774 \ubcf4\uc77c \ub54c\uae4c\uc9c0 \uc801\uc778\uc9c0 \uc544\uad70\uc778\uc9c0 \uc54c\uc9c0 \ubabb\ud588\ub2e4. \uc774 \uc2dc\uc810\uc5d0\uc11c \uc778\ubbfc\uad70\uc740 \uc774\ubbf8 \ubbf8\uad70\ub4e4\uc758 \uc704\uce58\uc5d0 \ubc14\uc9dd \ub2e4\uac00\uc628 \uc0c1\ud0dc\uc600\ub2e4. \uc778\ubbfc\uad70\uc740 \ucd1d \ud55c \ubc1c \ub9de\uc9c0 \uc54a\uace0 \ucc38\ud638\uae4c\uc9c0 \ub3c4\ucc29\ud588\ub2e4. \uc18c\ub300\uc7a5\uc740 \ubcd1\ub825\uacfc \ud654\ub825\uc758 \uc5f4\uc138\ub97c \uc774\uae38 \uc218 \uc5c6\ub2e4\uace0 \ud310\ub2e8\ud558\uace0 \ud56d\ubcf5\ud588\ub2e4. \uc778\ubbfc\uad70\uc740 \uc18c\ub300\uc6d0 31\uba85\uc744 \ubaa8\ub450 \ud3ec\ub85c\ub85c \uc7a1\uc558\ub2e4. \uadf8\ub7ec\ub098 \uc99d\uc5b8\uc5d0 \ub530\ub77c\uc11c\ub294 \ud3ec\ub85c\ub85c \uc7a1\ud78c \ubcd1\uc0ac\uac00 42\uba85\uc774\uc5c8\ub2e4\uace0\ub3c4 \ud55c\ub2e4.", "sentence_answer": "\ub2f9\uc2dc \ubc15\uaca9\ud3ec\uc18c\ub300\uc7a5 \uc7ad \ud5c8\ub4dc\uc2a4\ud398\uc2a4 (Jack Hudspeth) \uc911\uc704\ub294 \uadf8\ub4e4\uc774 \uc544\uad70\uc774\ub77c\uace0 \uc0dd\uac01\ud588\ub2e4.", "paragraph_id": "6498876-4-0", "paragraph_question": "question: \ubc15\uaca9\ud3ec\uc18c\ub300\uc7a5 \uc911\uc704\uc758 \uc774\ub984\uc740 \ubb34\uc5c7\uc785\ub2c8\uae4c?, context: \uc0dd\uc874\uc790 \uc99d\uc5b8\uc5d0 \ub530\ub974\uba74, 8\uc6d4 15\uc77c \uc0c8\ubcbd\uc774 \ub418\uae30 \uc804\uc5d0 H\uc911\ub300 \ubc15\uaca9\ud3ec\uc18c\ub300\uc6d0\ub4e4\uc774 303\uace0\uc9c0 \uc8fc\ubcc0\uc758 \uc801\uc758 \uc6c0\uc9c1\uc784\uc744 \uc54c\uc544\ucc58\ub2e4\uace0 \ud55c\ub2e4. \uc18c\ub300\uc7a5\uc740 G\uc911\ub300\uc640 5\uae30\ubcd1\uc0ac\ub2e8\uc5d0 \uc5f0\ub77d\ud558\uc5ec \ub0a8\ud55c \uad6d\uad70 1\uac1c \uc18c\ub300 60\uba85\uc774 \ubc15\uaca9\ud3ec\uc18c\ub300\ub97c \uc9c0\uc6d0\ud558\uae30 \uc704\ud574 \ub3c4\ucc29\ud588\ub2e4\uace0 \uc54c\ub838\ub2e4. \uadf8\ub7ec\ub098 \ud574\uac00 \ub728\uc790 \uadf8\ub4e4\uc774 \ubc1c\uacac\ud55c \uac83\uc740 \uc778\ubbfc\uad70\uc758 T-34 \ub450 \ub300\uc640 \uadf8 \ub4a4\ub97c \ub530\ub974\ub294 200 \uba85 \uc774\uc0c1\uc758 \uc778\ubbfc\uad70\uc774\uc5c8\ub2e4. \uc7a0\uc2dc \ub4a4 \ud55c\uad6d\uc778 \uba87 \uba85\uc774 \uc57c\uc0b0 \uc0ac\uba74\uc744 \ud0c0\uace0 \ub098\ud0c0\ub0ac\ub2e4. \ucd08\ubcd1\uc774 \uadf8\ub4e4\uc744 \ubd88\ub800\uc73c\ub098 \ud55c\uad6d\uc778\ub4e4\uc740 \uc790\ub3d9\uc18c\ucd1d \ub09c\uc0ac\ub85c \ub300\ub2f5\ud588\ub2e4. \ub2f9\uc2dc \ubc15\uaca9\ud3ec\uc18c\ub300\uc7a5 \uc7ad \ud5c8\ub4dc\uc2a4\ud398\uc2a4(Jack Hudspeth) \uc911\uc704\ub294 \uadf8\ub4e4\uc774 \uc544\uad70\uc774\ub77c\uace0 \uc0dd\uac01\ud588\ub2e4. \ud504\ub808\ub4dc \ub77c\uc774\uc5b8(Fred Ryan) \uc774\ubcd1\uacfc \ub85c\uc774 \ub9e8\ub9c1(Roy Manring) \uc774\ubcd1\uc774 1999\ub144 \ud55c\uad6d\uc744 \uc7ac\ubc29\ubb38\ud574 \uc99d\uc5b8\ud55c \ubc14\uc5d0 \ub530\ub974\uba74, \uc77c\ubd80 \ubbf8\uad70 \ubcd1\uc0ac\ub4e4\uc740 \uc811\uadfc\ud558\ub294 \ubcd1\ub825\uc774 \uc778\ubbfc\uad70\uc774\ub77c\ub294 \uac83\uc744 \uc54c\uc544\ucc58\uace0 \ubc1c\ud3ec\ub97c \uc900\ube44\ud588\ub2e4. \uadf8\ub7ec\ub098 \ud5c8\ub4dc\uc2a4\ud398\uc2a4 \uc18c\ub300\uc7a5\uc774 \ubc1c\ud3ec \uc911\uc9c0\ub97c \uba85\ub839\ud558\uba70 \ubc1c\ud3ec\ud55c\ub2e4\uba74 \uad70\uc0ac\ubc95\uc815\uc5d0 \ud68c\ubd80\uc2dc\ud0ac \uac83\uc774\ub77c\uace0 \uc704\ud611\ud588\ub2e4. \ub098\uba38\uc9c0 \ubbf8\uad70\ub4e4\uc740 \uc811\uadfc\ud558\ub294 \ud55c\uad6d\uc778\ub4e4\uc774 \ucda9\ubd84\ud788 \uac00\uae4c\uc6cc\uc838 \uadf8\ub4e4 \uad70\ubaa8\uc758 \ubd89\uc740 \ubcc4\uc774 \ubcf4\uc77c \ub54c\uae4c\uc9c0 \uc801\uc778\uc9c0 \uc544\uad70\uc778\uc9c0 \uc54c\uc9c0 \ubabb\ud588\ub2e4. \uc774 \uc2dc\uc810\uc5d0\uc11c \uc778\ubbfc\uad70\uc740 \uc774\ubbf8 \ubbf8\uad70\ub4e4\uc758 \uc704\uce58\uc5d0 \ubc14\uc9dd \ub2e4\uac00\uc628 \uc0c1\ud0dc\uc600\ub2e4. \uc778\ubbfc\uad70\uc740 \ucd1d \ud55c \ubc1c \ub9de\uc9c0 \uc54a\uace0 \ucc38\ud638\uae4c\uc9c0 \ub3c4\ucc29\ud588\ub2e4. \uc18c\ub300\uc7a5\uc740 \ubcd1\ub825\uacfc \ud654\ub825\uc758 \uc5f4\uc138\ub97c \uc774\uae38 \uc218 \uc5c6\ub2e4\uace0 \ud310\ub2e8\ud558\uace0 \ud56d\ubcf5\ud588\ub2e4. \uc778\ubbfc\uad70\uc740 \uc18c\ub300\uc6d0 31\uba85\uc744 \ubaa8\ub450 \ud3ec\ub85c\ub85c \uc7a1\uc558\ub2e4. \uadf8\ub7ec\ub098 \uc99d\uc5b8\uc5d0 \ub530\ub77c\uc11c\ub294 \ud3ec\ub85c\ub85c \uc7a1\ud78c \ubcd1\uc0ac\uac00 42\uba85\uc774\uc5c8\ub2e4\uace0\ub3c4 \ud55c\ub2e4."} {"question": "41\ub9c8\ub514\ubd80\ud130 \uc544\ud3ec\uc9c0\ub77c\ud22c\ub77c\uac00 \uc787\ub294 2\uc74c \ub3d9\uae30\ub97c \uc720\uc9c0\ud558\ub294 \uc545\uae30\uad70\uc740 \ubb34\uc5c7\uc778\uac00?", "paragraph": "40\ub9c8\ub514\uc5d0 \ubc18\ubcf5\uae30\ud638\uac00 \uc788\uc5b4 5\ub9c8\ub514\ub85c \ub418\ub3cc\uc544\uc628\ub2e4. 41\ub9c8\ub514\ubd80\ud130\ub294 \uc544\ud3ec\uc9c0\ub77c\ud22c\ub77c\uac00 \uc787\ub294 2\uc74c \ub3d9\uae30\ub97c \uad00\uc545\uae30\uac00 \uacc4\uc18d \uc720\uc9c0\ud558\ub294 \uac00\uc6b4\ub370, \ud604\uc545\uae30\uac00 \uc0c8\ub86d\uac8c \ubd80\ub4dc\ub7ec\uc6b4 \uc18c\uc7ac\ub97c \uc81c\uc2dc\ud558\uae30 \uc2dc\uc791\ud55c\ub2e4. \uc774\uc640 \uac19\uc740 \uc18c\uc7ac\ub85c \uacc4\uc18d \ubc1c\uc804\ud558\uc5ec 124\ub9c8\ub514\uc758 \ubc18\ubcf5\uae30\ud638\uc5d0 \uc774\ub978\ub2e4. \uc138 \ub9c8\ub514\uc758 \uc774\ud589\uad6c \ud6c4\uc5d0 \uc81c2\ubd80\uac00 \uc2dc\uc791\ub41c\ub2e4. \uc6b4 \ud3ec\ucf54 \uba54\ub178 \uc54c\ub808\uadf8\ub85c(\uc54c\ub808\uadf8\ub85c\ubcf4\ub2e4 \uc57d\uac04 \ub290\ub9ac\uac8c)\ub85c \uc870\uae08 \uc18d\ub3c4\ub989 \ub2a6\ucd94\ub294 128\ub9c8\ub514\ubd80\ud130\ub294 \ud2b8\ub9ac\uc624\uac00 \ub41c\ub2e4. \uc2a4\ucf00\ub974\ucd08\uc758 \uc8fc\uc694\ubd80\uc640\ub294 \uc83c\ud600 \ub2e4\ub97c, \uc21c\ucc28\uc9c4\ud589\ud558\ub294 \ubd80\ub4dc\ub7ec\uc6b4 \uc8fc\uc81c\uac00 \uad00\uc545\uae30\uc758 \ud48d\ubd80\ud55c \uc6b8\ub9bc\uc73c\ub85c \uc5f0\uc8fc\ub41c\ub2e4. \uc774 \uc8fc\uc81c\uc758 \ud615\ud0dc\ub294 \uac70\uc758 \ubc14\ub00c\uc9c0 \uc54a\uace0, \ud48d\ubd80\ud55c \uc870\ubc14\uafc8\uc73c\ub85c \uc74c\uc0c9\uc774 \ubc14\ub00c\uba74\uc11c \uc5ec\ub7ec \ubc88 \ub418\ud480\uc774\ub418\uc5b4 212\ub9c8\ub514\uae4c\uc9c0 \uc81c2\ubd80\ub97c \ub9cc\ub4e4\uc5b4 \uac04\ub2e4. \ud15c\ud3ec \ud504\ub9ac\ubaa8(\ucc98\uc74c \ube60\ub974\uae30\ub85c)\ub85c 213\ub9c8\ub514\ubd80\ud130 \uc81c1\ubd80\uac00 \ub2e4\uc2dc \ub098\ud0c0\ub098\ub294\ub370, \ub0b4\uc6a9 \ubcc0\ud654\uac00 \uac70\uc758 \uc5c6\uc73c\ubbc0\ub85c \ub2e4 \uce74\ud3ec\uc801\uc778 \ud615\ud0dc\ub77c\uace0 \ud574\ub3c4 \uc88b\uc744 \uac83\uc774\ub2e4. 234\ub9c8\ub514\ubd80\ud130\uc758 \uba54\ub178 \uc54c\ub808\uadf8\ub85c(\uc57d\uac04 \ub290\ub9b0 \uc54c\ub808\uadf8\ub85c) \ubd80\ubd84\uc740 \uc81c2\ubd80, \uc989 \ud2b8\ub9ac\uc624\uc758 \uc7ac\ud604\uc774\ub2e4. \uc5ec\uae30\uc5d0\uc11c\ub3c4 \ubcc4\ub85c\ubcc0\ud654\ub294 \ub098\ud0c0\ub098\uc9c0 \uc54a\uc544, \uacb0\uad6d \uc2a4\ucf00\ub974\ucd08\uc640 \ud2b8\ub9ac\uc624 \uc804\uccb4\uac00 \ub2e4 \uce74\ud3ec\ub418\ub294 \uac83\uacfc \uac70\uc758 \ub3d9\uc77c\ud55c \ud615\ud0dc\uac00 \ub41c\ub2e4. 417\ub9c8\ub514\ubd80\ud130\ub294 \ud15c\ud3ec\uac00 \ud504\ub808\uc2a4\ud1a0\ub85c \ubcc0\ud558\uc5ec \ubc18\ubcf5\ubd80\ub85c\uc11c\uc758 \uae30\ub2a5\uc744 \ubc1c\ud718\ud558\ub294\ub370, \ud300\ud30c\ub2c8\uac00 C\uc7a5\uc870\uc758 \uc74c\ub738\uc74c\uacfc \ub538\ub9bc\uc74c\uc744 \uc2a4\ucf00\ub974\ucd08 \uc8fc\uc81c\ub3d9\uae30\uc758 \ub9ac\ub4ec\uc744 \uc774\uc6a9\ud558\uc5ec \uc5f0\uc8fc\ud558\ub294 \uac00\uc6b4\ub370 \uad00\uc545\uae30\uac00 \uae34 \ud654\uc74c\uc744 \uc5f0\uc8fc\ud558\uba74\uc11c \ud06c\ub808\uc174\ub3c4\ub85c \uc9c4\ud589\ub41c\ub2e4. \ud300\ud30c\ub2c8\uc758 \ub9ac\ub4ec\uc5d0 \uc774\ub044\ub824\uc11c \uad00\ud604\uc545 \uc804\uccb4\uc5d0 \uc2a4\ucf00\ub974\ucd08 \uc8fc\uc81c\uc758 \ub3d9\uae30\uac00 \ub2e4\uc2dc \ub098\ud0c0\ub098 \uc555\ub3c4\uc801\uc778 \ud074\ub77c\uc774\ub9e5\uc2a4\ub97c \uc774\ub8e8\uace0 \uc545\uc7a5\uc744 \ub9c8\uce5c\ub2e4.", "answer": "\uad00\uc545\uae30", "sentence": "41\ub9c8\ub514\ubd80\ud130\ub294 \uc544\ud3ec\uc9c0\ub77c\ud22c\ub77c\uac00 \uc787\ub294 2\uc74c \ub3d9\uae30\ub97c \uad00\uc545\uae30 \uac00 \uacc4\uc18d \uc720\uc9c0\ud558\ub294 \uac00\uc6b4\ub370, \ud604\uc545\uae30\uac00 \uc0c8\ub86d\uac8c \ubd80\ub4dc\ub7ec\uc6b4 \uc18c\uc7ac\ub97c \uc81c\uc2dc\ud558\uae30 \uc2dc\uc791\ud55c\ub2e4.", "paragraph_sentence": "40\ub9c8\ub514\uc5d0 \ubc18\ubcf5\uae30\ud638\uac00 \uc788\uc5b4 5\ub9c8\ub514\ub85c \ub418\ub3cc\uc544\uc628\ub2e4. 41\ub9c8\ub514\ubd80\ud130\ub294 \uc544\ud3ec\uc9c0\ub77c\ud22c\ub77c\uac00 \uc787\ub294 2\uc74c \ub3d9\uae30\ub97c \uad00\uc545\uae30 \uac00 \uacc4\uc18d \uc720\uc9c0\ud558\ub294 \uac00\uc6b4\ub370, \ud604\uc545\uae30\uac00 \uc0c8\ub86d\uac8c \ubd80\ub4dc\ub7ec\uc6b4 \uc18c\uc7ac\ub97c \uc81c\uc2dc\ud558\uae30 \uc2dc\uc791\ud55c\ub2e4. \uc774\uc640 \uac19\uc740 \uc18c\uc7ac\ub85c \uacc4\uc18d \ubc1c\uc804\ud558\uc5ec 124\ub9c8\ub514\uc758 \ubc18\ubcf5\uae30\ud638\uc5d0 \uc774\ub978\ub2e4. \uc138 \ub9c8\ub514\uc758 \uc774\ud589\uad6c \ud6c4\uc5d0 \uc81c2\ubd80\uac00 \uc2dc\uc791\ub41c\ub2e4. \uc6b4 \ud3ec\ucf54 \uba54\ub178 \uc54c\ub808\uadf8\ub85c(\uc54c\ub808\uadf8\ub85c\ubcf4\ub2e4 \uc57d\uac04 \ub290\ub9ac\uac8c)\ub85c \uc870\uae08 \uc18d\ub3c4\ub989 \ub2a6\ucd94\ub294 128\ub9c8\ub514\ubd80\ud130\ub294 \ud2b8\ub9ac\uc624\uac00 \ub41c\ub2e4. \uc2a4\ucf00\ub974\ucd08\uc758 \uc8fc\uc694\ubd80\uc640\ub294 \uc83c\ud600 \ub2e4\ub97c, \uc21c\ucc28\uc9c4\ud589\ud558\ub294 \ubd80\ub4dc\ub7ec\uc6b4 \uc8fc\uc81c\uac00 \uad00\uc545\uae30\uc758 \ud48d\ubd80\ud55c \uc6b8\ub9bc\uc73c\ub85c \uc5f0\uc8fc\ub41c\ub2e4. \uc774 \uc8fc\uc81c\uc758 \ud615\ud0dc\ub294 \uac70\uc758 \ubc14\ub00c\uc9c0 \uc54a\uace0, \ud48d\ubd80\ud55c \uc870\ubc14\uafc8\uc73c\ub85c \uc74c\uc0c9\uc774 \ubc14\ub00c\uba74\uc11c \uc5ec\ub7ec \ubc88 \ub418\ud480\uc774\ub418\uc5b4 212\ub9c8\ub514\uae4c\uc9c0 \uc81c2\ubd80\ub97c \ub9cc\ub4e4\uc5b4 \uac04\ub2e4. \ud15c\ud3ec \ud504\ub9ac\ubaa8(\ucc98\uc74c \ube60\ub974\uae30\ub85c)\ub85c 213\ub9c8\ub514\ubd80\ud130 \uc81c1\ubd80\uac00 \ub2e4\uc2dc \ub098\ud0c0\ub098\ub294\ub370, \ub0b4\uc6a9 \ubcc0\ud654\uac00 \uac70\uc758 \uc5c6\uc73c\ubbc0\ub85c \ub2e4 \uce74\ud3ec\uc801\uc778 \ud615\ud0dc\ub77c\uace0 \ud574\ub3c4 \uc88b\uc744 \uac83\uc774\ub2e4. 234\ub9c8\ub514\ubd80\ud130\uc758 \uba54\ub178 \uc54c\ub808\uadf8\ub85c(\uc57d\uac04 \ub290\ub9b0 \uc54c\ub808\uadf8\ub85c) \ubd80\ubd84\uc740 \uc81c2\ubd80, \uc989 \ud2b8\ub9ac\uc624\uc758 \uc7ac\ud604\uc774\ub2e4. \uc5ec\uae30\uc5d0\uc11c\ub3c4 \ubcc4\ub85c\ubcc0\ud654\ub294 \ub098\ud0c0\ub098\uc9c0 \uc54a\uc544, \uacb0\uad6d \uc2a4\ucf00\ub974\ucd08\uc640 \ud2b8\ub9ac\uc624 \uc804\uccb4\uac00 \ub2e4 \uce74\ud3ec\ub418\ub294 \uac83\uacfc \uac70\uc758 \ub3d9\uc77c\ud55c \ud615\ud0dc\uac00 \ub41c\ub2e4. 417\ub9c8\ub514\ubd80\ud130\ub294 \ud15c\ud3ec\uac00 \ud504\ub808\uc2a4\ud1a0\ub85c \ubcc0\ud558\uc5ec \ubc18\ubcf5\ubd80\ub85c\uc11c\uc758 \uae30\ub2a5\uc744 \ubc1c\ud718\ud558\ub294\ub370, \ud300\ud30c\ub2c8\uac00 C\uc7a5\uc870\uc758 \uc74c\ub738\uc74c\uacfc \ub538\ub9bc\uc74c\uc744 \uc2a4\ucf00\ub974\ucd08 \uc8fc\uc81c\ub3d9\uae30\uc758 \ub9ac\ub4ec\uc744 \uc774\uc6a9\ud558\uc5ec \uc5f0\uc8fc\ud558\ub294 \uac00\uc6b4\ub370 \uad00\uc545\uae30\uac00 \uae34 \ud654\uc74c\uc744 \uc5f0\uc8fc\ud558\uba74\uc11c \ud06c\ub808\uc174\ub3c4\ub85c \uc9c4\ud589\ub41c\ub2e4. \ud300\ud30c\ub2c8\uc758 \ub9ac\ub4ec\uc5d0 \uc774\ub044\ub824\uc11c \uad00\ud604\uc545 \uc804\uccb4\uc5d0 \uc2a4\ucf00\ub974\ucd08 \uc8fc\uc81c\uc758 \ub3d9\uae30\uac00 \ub2e4\uc2dc \ub098\ud0c0\ub098 \uc555\ub3c4\uc801\uc778 \ud074\ub77c\uc774\ub9e5\uc2a4\ub97c \uc774\ub8e8\uace0 \uc545\uc7a5\uc744 \ub9c8\uce5c\ub2e4.", "paragraph_answer": "40\ub9c8\ub514\uc5d0 \ubc18\ubcf5\uae30\ud638\uac00 \uc788\uc5b4 5\ub9c8\ub514\ub85c \ub418\ub3cc\uc544\uc628\ub2e4. 41\ub9c8\ub514\ubd80\ud130\ub294 \uc544\ud3ec\uc9c0\ub77c\ud22c\ub77c\uac00 \uc787\ub294 2\uc74c \ub3d9\uae30\ub97c \uad00\uc545\uae30 \uac00 \uacc4\uc18d \uc720\uc9c0\ud558\ub294 \uac00\uc6b4\ub370, \ud604\uc545\uae30\uac00 \uc0c8\ub86d\uac8c \ubd80\ub4dc\ub7ec\uc6b4 \uc18c\uc7ac\ub97c \uc81c\uc2dc\ud558\uae30 \uc2dc\uc791\ud55c\ub2e4. \uc774\uc640 \uac19\uc740 \uc18c\uc7ac\ub85c \uacc4\uc18d \ubc1c\uc804\ud558\uc5ec 124\ub9c8\ub514\uc758 \ubc18\ubcf5\uae30\ud638\uc5d0 \uc774\ub978\ub2e4. \uc138 \ub9c8\ub514\uc758 \uc774\ud589\uad6c \ud6c4\uc5d0 \uc81c2\ubd80\uac00 \uc2dc\uc791\ub41c\ub2e4. \uc6b4 \ud3ec\ucf54 \uba54\ub178 \uc54c\ub808\uadf8\ub85c(\uc54c\ub808\uadf8\ub85c\ubcf4\ub2e4 \uc57d\uac04 \ub290\ub9ac\uac8c)\ub85c \uc870\uae08 \uc18d\ub3c4\ub989 \ub2a6\ucd94\ub294 128\ub9c8\ub514\ubd80\ud130\ub294 \ud2b8\ub9ac\uc624\uac00 \ub41c\ub2e4. \uc2a4\ucf00\ub974\ucd08\uc758 \uc8fc\uc694\ubd80\uc640\ub294 \uc83c\ud600 \ub2e4\ub97c, \uc21c\ucc28\uc9c4\ud589\ud558\ub294 \ubd80\ub4dc\ub7ec\uc6b4 \uc8fc\uc81c\uac00 \uad00\uc545\uae30\uc758 \ud48d\ubd80\ud55c \uc6b8\ub9bc\uc73c\ub85c \uc5f0\uc8fc\ub41c\ub2e4. \uc774 \uc8fc\uc81c\uc758 \ud615\ud0dc\ub294 \uac70\uc758 \ubc14\ub00c\uc9c0 \uc54a\uace0, \ud48d\ubd80\ud55c \uc870\ubc14\uafc8\uc73c\ub85c \uc74c\uc0c9\uc774 \ubc14\ub00c\uba74\uc11c \uc5ec\ub7ec \ubc88 \ub418\ud480\uc774\ub418\uc5b4 212\ub9c8\ub514\uae4c\uc9c0 \uc81c2\ubd80\ub97c \ub9cc\ub4e4\uc5b4 \uac04\ub2e4. \ud15c\ud3ec \ud504\ub9ac\ubaa8(\ucc98\uc74c \ube60\ub974\uae30\ub85c)\ub85c 213\ub9c8\ub514\ubd80\ud130 \uc81c1\ubd80\uac00 \ub2e4\uc2dc \ub098\ud0c0\ub098\ub294\ub370, \ub0b4\uc6a9 \ubcc0\ud654\uac00 \uac70\uc758 \uc5c6\uc73c\ubbc0\ub85c \ub2e4 \uce74\ud3ec\uc801\uc778 \ud615\ud0dc\ub77c\uace0 \ud574\ub3c4 \uc88b\uc744 \uac83\uc774\ub2e4. 234\ub9c8\ub514\ubd80\ud130\uc758 \uba54\ub178 \uc54c\ub808\uadf8\ub85c(\uc57d\uac04 \ub290\ub9b0 \uc54c\ub808\uadf8\ub85c) \ubd80\ubd84\uc740 \uc81c2\ubd80, \uc989 \ud2b8\ub9ac\uc624\uc758 \uc7ac\ud604\uc774\ub2e4. \uc5ec\uae30\uc5d0\uc11c\ub3c4 \ubcc4\ub85c\ubcc0\ud654\ub294 \ub098\ud0c0\ub098\uc9c0 \uc54a\uc544, \uacb0\uad6d \uc2a4\ucf00\ub974\ucd08\uc640 \ud2b8\ub9ac\uc624 \uc804\uccb4\uac00 \ub2e4 \uce74\ud3ec\ub418\ub294 \uac83\uacfc \uac70\uc758 \ub3d9\uc77c\ud55c \ud615\ud0dc\uac00 \ub41c\ub2e4. 417\ub9c8\ub514\ubd80\ud130\ub294 \ud15c\ud3ec\uac00 \ud504\ub808\uc2a4\ud1a0\ub85c \ubcc0\ud558\uc5ec \ubc18\ubcf5\ubd80\ub85c\uc11c\uc758 \uae30\ub2a5\uc744 \ubc1c\ud718\ud558\ub294\ub370, \ud300\ud30c\ub2c8\uac00 C\uc7a5\uc870\uc758 \uc74c\ub738\uc74c\uacfc \ub538\ub9bc\uc74c\uc744 \uc2a4\ucf00\ub974\ucd08 \uc8fc\uc81c\ub3d9\uae30\uc758 \ub9ac\ub4ec\uc744 \uc774\uc6a9\ud558\uc5ec \uc5f0\uc8fc\ud558\ub294 \uac00\uc6b4\ub370 \uad00\uc545\uae30\uac00 \uae34 \ud654\uc74c\uc744 \uc5f0\uc8fc\ud558\uba74\uc11c \ud06c\ub808\uc174\ub3c4\ub85c \uc9c4\ud589\ub41c\ub2e4. \ud300\ud30c\ub2c8\uc758 \ub9ac\ub4ec\uc5d0 \uc774\ub044\ub824\uc11c \uad00\ud604\uc545 \uc804\uccb4\uc5d0 \uc2a4\ucf00\ub974\ucd08 \uc8fc\uc81c\uc758 \ub3d9\uae30\uac00 \ub2e4\uc2dc \ub098\ud0c0\ub098 \uc555\ub3c4\uc801\uc778 \ud074\ub77c\uc774\ub9e5\uc2a4\ub97c \uc774\ub8e8\uace0 \uc545\uc7a5\uc744 \ub9c8\uce5c\ub2e4.", "sentence_answer": "41\ub9c8\ub514\ubd80\ud130\ub294 \uc544\ud3ec\uc9c0\ub77c\ud22c\ub77c\uac00 \uc787\ub294 2\uc74c \ub3d9\uae30\ub97c \uad00\uc545\uae30 \uac00 \uacc4\uc18d \uc720\uc9c0\ud558\ub294 \uac00\uc6b4\ub370, \ud604\uc545\uae30\uac00 \uc0c8\ub86d\uac8c \ubd80\ub4dc\ub7ec\uc6b4 \uc18c\uc7ac\ub97c \uc81c\uc2dc\ud558\uae30 \uc2dc\uc791\ud55c\ub2e4.", "paragraph_id": "6487516-11-0", "paragraph_question": "question: 41\ub9c8\ub514\ubd80\ud130 \uc544\ud3ec\uc9c0\ub77c\ud22c\ub77c\uac00 \uc787\ub294 2\uc74c \ub3d9\uae30\ub97c \uc720\uc9c0\ud558\ub294 \uc545\uae30\uad70\uc740 \ubb34\uc5c7\uc778\uac00?, context: 40\ub9c8\ub514\uc5d0 \ubc18\ubcf5\uae30\ud638\uac00 \uc788\uc5b4 5\ub9c8\ub514\ub85c \ub418\ub3cc\uc544\uc628\ub2e4. 41\ub9c8\ub514\ubd80\ud130\ub294 \uc544\ud3ec\uc9c0\ub77c\ud22c\ub77c\uac00 \uc787\ub294 2\uc74c \ub3d9\uae30\ub97c \uad00\uc545\uae30\uac00 \uacc4\uc18d \uc720\uc9c0\ud558\ub294 \uac00\uc6b4\ub370, \ud604\uc545\uae30\uac00 \uc0c8\ub86d\uac8c \ubd80\ub4dc\ub7ec\uc6b4 \uc18c\uc7ac\ub97c \uc81c\uc2dc\ud558\uae30 \uc2dc\uc791\ud55c\ub2e4. \uc774\uc640 \uac19\uc740 \uc18c\uc7ac\ub85c \uacc4\uc18d \ubc1c\uc804\ud558\uc5ec 124\ub9c8\ub514\uc758 \ubc18\ubcf5\uae30\ud638\uc5d0 \uc774\ub978\ub2e4. \uc138 \ub9c8\ub514\uc758 \uc774\ud589\uad6c \ud6c4\uc5d0 \uc81c2\ubd80\uac00 \uc2dc\uc791\ub41c\ub2e4. \uc6b4 \ud3ec\ucf54 \uba54\ub178 \uc54c\ub808\uadf8\ub85c(\uc54c\ub808\uadf8\ub85c\ubcf4\ub2e4 \uc57d\uac04 \ub290\ub9ac\uac8c)\ub85c \uc870\uae08 \uc18d\ub3c4\ub989 \ub2a6\ucd94\ub294 128\ub9c8\ub514\ubd80\ud130\ub294 \ud2b8\ub9ac\uc624\uac00 \ub41c\ub2e4. \uc2a4\ucf00\ub974\ucd08\uc758 \uc8fc\uc694\ubd80\uc640\ub294 \uc83c\ud600 \ub2e4\ub97c, \uc21c\ucc28\uc9c4\ud589\ud558\ub294 \ubd80\ub4dc\ub7ec\uc6b4 \uc8fc\uc81c\uac00 \uad00\uc545\uae30\uc758 \ud48d\ubd80\ud55c \uc6b8\ub9bc\uc73c\ub85c \uc5f0\uc8fc\ub41c\ub2e4. \uc774 \uc8fc\uc81c\uc758 \ud615\ud0dc\ub294 \uac70\uc758 \ubc14\ub00c\uc9c0 \uc54a\uace0, \ud48d\ubd80\ud55c \uc870\ubc14\uafc8\uc73c\ub85c \uc74c\uc0c9\uc774 \ubc14\ub00c\uba74\uc11c \uc5ec\ub7ec \ubc88 \ub418\ud480\uc774\ub418\uc5b4 212\ub9c8\ub514\uae4c\uc9c0 \uc81c2\ubd80\ub97c \ub9cc\ub4e4\uc5b4 \uac04\ub2e4. \ud15c\ud3ec \ud504\ub9ac\ubaa8(\ucc98\uc74c \ube60\ub974\uae30\ub85c)\ub85c 213\ub9c8\ub514\ubd80\ud130 \uc81c1\ubd80\uac00 \ub2e4\uc2dc \ub098\ud0c0\ub098\ub294\ub370, \ub0b4\uc6a9 \ubcc0\ud654\uac00 \uac70\uc758 \uc5c6\uc73c\ubbc0\ub85c \ub2e4 \uce74\ud3ec\uc801\uc778 \ud615\ud0dc\ub77c\uace0 \ud574\ub3c4 \uc88b\uc744 \uac83\uc774\ub2e4. 234\ub9c8\ub514\ubd80\ud130\uc758 \uba54\ub178 \uc54c\ub808\uadf8\ub85c(\uc57d\uac04 \ub290\ub9b0 \uc54c\ub808\uadf8\ub85c) \ubd80\ubd84\uc740 \uc81c2\ubd80, \uc989 \ud2b8\ub9ac\uc624\uc758 \uc7ac\ud604\uc774\ub2e4. \uc5ec\uae30\uc5d0\uc11c\ub3c4 \ubcc4\ub85c\ubcc0\ud654\ub294 \ub098\ud0c0\ub098\uc9c0 \uc54a\uc544, \uacb0\uad6d \uc2a4\ucf00\ub974\ucd08\uc640 \ud2b8\ub9ac\uc624 \uc804\uccb4\uac00 \ub2e4 \uce74\ud3ec\ub418\ub294 \uac83\uacfc \uac70\uc758 \ub3d9\uc77c\ud55c \ud615\ud0dc\uac00 \ub41c\ub2e4. 417\ub9c8\ub514\ubd80\ud130\ub294 \ud15c\ud3ec\uac00 \ud504\ub808\uc2a4\ud1a0\ub85c \ubcc0\ud558\uc5ec \ubc18\ubcf5\ubd80\ub85c\uc11c\uc758 \uae30\ub2a5\uc744 \ubc1c\ud718\ud558\ub294\ub370, \ud300\ud30c\ub2c8\uac00 C\uc7a5\uc870\uc758 \uc74c\ub738\uc74c\uacfc \ub538\ub9bc\uc74c\uc744 \uc2a4\ucf00\ub974\ucd08 \uc8fc\uc81c\ub3d9\uae30\uc758 \ub9ac\ub4ec\uc744 \uc774\uc6a9\ud558\uc5ec \uc5f0\uc8fc\ud558\ub294 \uac00\uc6b4\ub370 \uad00\uc545\uae30\uac00 \uae34 \ud654\uc74c\uc744 \uc5f0\uc8fc\ud558\uba74\uc11c \ud06c\ub808\uc174\ub3c4\ub85c \uc9c4\ud589\ub41c\ub2e4. \ud300\ud30c\ub2c8\uc758 \ub9ac\ub4ec\uc5d0 \uc774\ub044\ub824\uc11c \uad00\ud604\uc545 \uc804\uccb4\uc5d0 \uc2a4\ucf00\ub974\ucd08 \uc8fc\uc81c\uc758 \ub3d9\uae30\uac00 \ub2e4\uc2dc \ub098\ud0c0\ub098 \uc555\ub3c4\uc801\uc778 \ud074\ub77c\uc774\ub9e5\uc2a4\ub97c \uc774\ub8e8\uace0 \uc545\uc7a5\uc744 \ub9c8\uce5c\ub2e4."} {"question": "RFC 3339\ub97c \uc0ac\uc6a9\ud558\uba74\uc11c \ud0c0\uc784\uc2a4\ud0ec\ud504 \ud615\uc2dd\uc758 \uc5b4\ub5a4 \ubd80\ubd84\uc774\ub4e0 ISO 8601\uc5d0 \uc77c\uce58\ud558\ub294 \uac83\uc744 \ubcf4\uc7a5\ud558\uc9c0 \uc54a\uc544, \ucda9\ub3cc\uc5c6\uc774 \uc790\uc720\ub86d\uac8c \uc0ac\uc6a9\ud560 \uc218 \uc788\ub294 \uac83\uc740 \ubb34\uc5c7\uc778\uac00?", "paragraph": "RFC 3339\ub294 ISO 8601\uac00 \uae08\ud55c, \"-00:00\"\ub85c \uc9c0\uc815\ub418\ub294 0 \uc2dc\uac04\ub300 \uc624\ud504\uc14b \ud5c8\uc6a9\uc5d0 \uc788\uc5b4 \uad6c\ubd84\ub41c\ub2e4. \"+00:00\" \ud639\uc740 0\uc774 \uc544\ub2cc \uc624\ud504\uc14b\uc744 \ub530\ub974\ub294 \uac83\uc774 \uc624\ud504\uc14b \uc0ac\uc6a9 \uc2dc \uc120\ud638\ub428\uc5d0\ub3c4 \ubd88\uad6c\ud558\uace0, RFC 3339\ub294 \uc120\ud638\ub418\ub294 \uc2dc\uac04\ub300\ub97c \ub9d0\ud558\ub294 \uac83\uc774 \uc544\ub2c8\ub77c\ub294 \ud568\ucd95\ub41c \uc758\ubbf8\ub97c \uc804\ub2ec\ud558\uae30 \uc704\ud574 \"-00:00\"\uc744 \uc9c0\ud5a5\ud55c\ub2e4. \uc774\ub7ec\ud55c \"-00:00\"\uc744 \ub530\ub974\ub294 \uad00\ub840\ub294 \uc774\uba54\uc77c \ud5e4\ub354\uc5d0\uc11c\uc758 \ud0c0\uc784\uc2a4\ud0ec\ud504\ub97c \uc704\ud574 RFC 3339\ub97c \uc0ac\uc6a9\ud558\ub294 RFC 2822\uc640 \uac19\uc740, \ucd08\uae30 RFC\ub4e4\ub85c\ubd80\ud130 \ud30c\uc0dd\ub418\uc5c8\ub2e4. RFC 2822\ub294 \ud0c0\uc784\uc2a4\ud0ec\ud504 \ud615\uc2dd\uc758 \uc5b4\ub5a4 \ubd80\ubd84\uc774\ub4e0 ISO 8601\uc5d0 \uc77c\uce58\ud55c\ub2e4\ub294 \uac83\uc744 \ubcf4\uc7a5\ud558\uc9c0 \uc54a\uc73c\ubbc0\ub85c, \ucda9\ub3cc\uc5c6\uc774 \uc774\ub7f0 \uad00\ub840\ub97c \uc0ac\uc6a9\ud568\uc5d0 \uc788\uc5b4 \uc790\uc720\ub86d\ub2e4.", "answer": "RFC 2822", "sentence": "\uc774\ub7ec\ud55c \"-00:00\"\uc744 \ub530\ub974\ub294 \uad00\ub840\ub294 \uc774\uba54\uc77c \ud5e4\ub354\uc5d0\uc11c\uc758 \ud0c0\uc784\uc2a4\ud0ec\ud504\ub97c \uc704\ud574 RFC 3339\ub97c \uc0ac\uc6a9\ud558\ub294 RFC 2822 \uc640 \uac19\uc740, \ucd08\uae30 RFC\ub4e4\ub85c\ubd80\ud130 \ud30c\uc0dd\ub418\uc5c8\ub2e4.", "paragraph_sentence": "RFC 3339\ub294 ISO 8601\uac00 \uae08\ud55c, \"-00:00\"\ub85c \uc9c0\uc815\ub418\ub294 0 \uc2dc\uac04\ub300 \uc624\ud504\uc14b \ud5c8\uc6a9\uc5d0 \uc788\uc5b4 \uad6c\ubd84\ub41c\ub2e4. \"+00:00\" \ud639\uc740 0\uc774 \uc544\ub2cc \uc624\ud504\uc14b\uc744 \ub530\ub974\ub294 \uac83\uc774 \uc624\ud504\uc14b \uc0ac\uc6a9 \uc2dc \uc120\ud638\ub428\uc5d0\ub3c4 \ubd88\uad6c\ud558\uace0, RFC 3339\ub294 \uc120\ud638\ub418\ub294 \uc2dc\uac04\ub300\ub97c \ub9d0\ud558\ub294 \uac83\uc774 \uc544\ub2c8\ub77c\ub294 \ud568\ucd95\ub41c \uc758\ubbf8\ub97c \uc804\ub2ec\ud558\uae30 \uc704\ud574 \"-00:00\"\uc744 \uc9c0\ud5a5\ud55c\ub2e4. \uc774\ub7ec\ud55c \"-00:00\"\uc744 \ub530\ub974\ub294 \uad00\ub840\ub294 \uc774\uba54\uc77c \ud5e4\ub354\uc5d0\uc11c\uc758 \ud0c0\uc784\uc2a4\ud0ec\ud504\ub97c \uc704\ud574 RFC 3339\ub97c \uc0ac\uc6a9\ud558\ub294 RFC 2822 \uc640 \uac19\uc740, \ucd08\uae30 RFC\ub4e4\ub85c\ubd80\ud130 \ud30c\uc0dd\ub418\uc5c8\ub2e4. RFC 2822\ub294 \ud0c0\uc784\uc2a4\ud0ec\ud504 \ud615\uc2dd\uc758 \uc5b4\ub5a4 \ubd80\ubd84\uc774\ub4e0 ISO 8601\uc5d0 \uc77c\uce58\ud55c\ub2e4\ub294 \uac83\uc744 \ubcf4\uc7a5\ud558\uc9c0 \uc54a\uc73c\ubbc0\ub85c, \ucda9\ub3cc\uc5c6\uc774 \uc774\ub7f0 \uad00\ub840\ub97c \uc0ac\uc6a9\ud568\uc5d0 \uc788\uc5b4 \uc790\uc720\ub86d\ub2e4.", "paragraph_answer": "RFC 3339\ub294 ISO 8601\uac00 \uae08\ud55c, \"-00:00\"\ub85c \uc9c0\uc815\ub418\ub294 0 \uc2dc\uac04\ub300 \uc624\ud504\uc14b \ud5c8\uc6a9\uc5d0 \uc788\uc5b4 \uad6c\ubd84\ub41c\ub2e4. \"+00:00\" \ud639\uc740 0\uc774 \uc544\ub2cc \uc624\ud504\uc14b\uc744 \ub530\ub974\ub294 \uac83\uc774 \uc624\ud504\uc14b \uc0ac\uc6a9 \uc2dc \uc120\ud638\ub428\uc5d0\ub3c4 \ubd88\uad6c\ud558\uace0, RFC 3339\ub294 \uc120\ud638\ub418\ub294 \uc2dc\uac04\ub300\ub97c \ub9d0\ud558\ub294 \uac83\uc774 \uc544\ub2c8\ub77c\ub294 \ud568\ucd95\ub41c \uc758\ubbf8\ub97c \uc804\ub2ec\ud558\uae30 \uc704\ud574 \"-00:00\"\uc744 \uc9c0\ud5a5\ud55c\ub2e4. \uc774\ub7ec\ud55c \"-00:00\"\uc744 \ub530\ub974\ub294 \uad00\ub840\ub294 \uc774\uba54\uc77c \ud5e4\ub354\uc5d0\uc11c\uc758 \ud0c0\uc784\uc2a4\ud0ec\ud504\ub97c \uc704\ud574 RFC 3339\ub97c \uc0ac\uc6a9\ud558\ub294 RFC 2822 \uc640 \uac19\uc740, \ucd08\uae30 RFC\ub4e4\ub85c\ubd80\ud130 \ud30c\uc0dd\ub418\uc5c8\ub2e4. RFC 2822\ub294 \ud0c0\uc784\uc2a4\ud0ec\ud504 \ud615\uc2dd\uc758 \uc5b4\ub5a4 \ubd80\ubd84\uc774\ub4e0 ISO 8601\uc5d0 \uc77c\uce58\ud55c\ub2e4\ub294 \uac83\uc744 \ubcf4\uc7a5\ud558\uc9c0 \uc54a\uc73c\ubbc0\ub85c, \ucda9\ub3cc\uc5c6\uc774 \uc774\ub7f0 \uad00\ub840\ub97c \uc0ac\uc6a9\ud568\uc5d0 \uc788\uc5b4 \uc790\uc720\ub86d\ub2e4.", "sentence_answer": "\uc774\ub7ec\ud55c \"-00:00\"\uc744 \ub530\ub974\ub294 \uad00\ub840\ub294 \uc774\uba54\uc77c \ud5e4\ub354\uc5d0\uc11c\uc758 \ud0c0\uc784\uc2a4\ud0ec\ud504\ub97c \uc704\ud574 RFC 3339\ub97c \uc0ac\uc6a9\ud558\ub294 RFC 2822 \uc640 \uac19\uc740, \ucd08\uae30 RFC\ub4e4\ub85c\ubd80\ud130 \ud30c\uc0dd\ub418\uc5c8\ub2e4.", "paragraph_id": "6494055-5-1", "paragraph_question": "question: RFC 3339\ub97c \uc0ac\uc6a9\ud558\uba74\uc11c \ud0c0\uc784\uc2a4\ud0ec\ud504 \ud615\uc2dd\uc758 \uc5b4\ub5a4 \ubd80\ubd84\uc774\ub4e0 ISO 8601\uc5d0 \uc77c\uce58\ud558\ub294 \uac83\uc744 \ubcf4\uc7a5\ud558\uc9c0 \uc54a\uc544, \ucda9\ub3cc\uc5c6\uc774 \uc790\uc720\ub86d\uac8c \uc0ac\uc6a9\ud560 \uc218 \uc788\ub294 \uac83\uc740 \ubb34\uc5c7\uc778\uac00?, context: RFC 3339\ub294 ISO 8601\uac00 \uae08\ud55c, \"-00:00\"\ub85c \uc9c0\uc815\ub418\ub294 0 \uc2dc\uac04\ub300 \uc624\ud504\uc14b \ud5c8\uc6a9\uc5d0 \uc788\uc5b4 \uad6c\ubd84\ub41c\ub2e4. \"+00:00\" \ud639\uc740 0\uc774 \uc544\ub2cc \uc624\ud504\uc14b\uc744 \ub530\ub974\ub294 \uac83\uc774 \uc624\ud504\uc14b \uc0ac\uc6a9 \uc2dc \uc120\ud638\ub428\uc5d0\ub3c4 \ubd88\uad6c\ud558\uace0, RFC 3339\ub294 \uc120\ud638\ub418\ub294 \uc2dc\uac04\ub300\ub97c \ub9d0\ud558\ub294 \uac83\uc774 \uc544\ub2c8\ub77c\ub294 \ud568\ucd95\ub41c \uc758\ubbf8\ub97c \uc804\ub2ec\ud558\uae30 \uc704\ud574 \"-00:00\"\uc744 \uc9c0\ud5a5\ud55c\ub2e4. \uc774\ub7ec\ud55c \"-00:00\"\uc744 \ub530\ub974\ub294 \uad00\ub840\ub294 \uc774\uba54\uc77c \ud5e4\ub354\uc5d0\uc11c\uc758 \ud0c0\uc784\uc2a4\ud0ec\ud504\ub97c \uc704\ud574 RFC 3339\ub97c \uc0ac\uc6a9\ud558\ub294 RFC 2822\uc640 \uac19\uc740, \ucd08\uae30 RFC\ub4e4\ub85c\ubd80\ud130 \ud30c\uc0dd\ub418\uc5c8\ub2e4. RFC 2822\ub294 \ud0c0\uc784\uc2a4\ud0ec\ud504 \ud615\uc2dd\uc758 \uc5b4\ub5a4 \ubd80\ubd84\uc774\ub4e0 ISO 8601\uc5d0 \uc77c\uce58\ud55c\ub2e4\ub294 \uac83\uc744 \ubcf4\uc7a5\ud558\uc9c0 \uc54a\uc73c\ubbc0\ub85c, \ucda9\ub3cc\uc5c6\uc774 \uc774\ub7f0 \uad00\ub840\ub97c \uc0ac\uc6a9\ud568\uc5d0 \uc788\uc5b4 \uc790\uc720\ub86d\ub2e4."} {"question": "\ud504\ub9ac\ub4dc\ub9ac\ud788 \ube4c\ud5ec\ub984 4\uc138\ub294 \uc5b4\ub5a4 \uc6d0\uce59\uc73c\ub85c \uad6d\ubbfc\uc758\ud68c\uac00 \ucd94\ub300\ud55c \ud669\uc81c\uc704\ub97c \uac70\ubd80\ud558\uc600\ub294\uac00?", "paragraph": "\uc774\uc5d0 \ub530\ub77c 1849\ub144 4\uc6d4 3\uc77c \uad6d\ubbfc\uc758\ud68c\ub294 \uc5d0\ub450\uc544\ub974\ud2b8 \ud3f0 \uc9d0\uc190\uc774 \uc774\ub044\ub294 \ub300\ud45c\ub2e8(\ud669\uc81c\ub300\ud45c\ub2e8, Kaiserdeputation)\uc744 \ud30c\uacac\ud558\uc5ec \ud504\ub9ac\ub4dc\ub9ac\ud788 \ube4c\ud5ec\ub984 4\uc138\uc5d0\uac8c \ud669\uc81c\uac00 \ub420 \uac83\uc744 \uc81c\uc758\ud588\ub2e4. \uc774\uac83\uc740 \uadf8\ub7ec\ub098 \uc9c0\uc704\ub098 \uad00\uc9c1\uc740 \uc81c\ud6c4\ub4e4\uacfc \uc790\uc720\uc2dc\ub4e4\uc758 \ucc2c\uc131 \uc5c6\uc774\ub294 \ubc1b\uc744 \uc218 \uc5c6\ub2e4\ub294 \ud45c\uba74\uc801\uc778 \uadfc\uac70\uc5d0 \ub530\ub77c \uac70\uc808\ub418\uc5c8\ub2e4. \uadf8\ub807\uc9c0\ub9cc \uc0ac\uc2e4 \ud504\ub9ac\ub4dc\ub9ac\ud788 \ube4c\ud5ec\ub984\uc740 \uc655\uad8c\uc2e0\uc218\uc124\uc758 \uc6d0\uce59\uc744 \uace0\uc218\ud588\uace0, \u2018\ud601\uba85\uc758 \uc369\uc740\ub0b4\u2019(Ludergeruch der Revolution)\uac00 \ub098\ub294 \uc5b4\ub5a0\ud55c \uc655\uad00\ub3c4 \ubc1b\uc744 \uc218 \uc5c6\ub2e4\uace0 \uc0dd\uac01\ud588\ub2e4. \uc774\ub85c\uc368 \uad6d\ubbfc\uc758\ud68c\uc758 \ud5cc\ubc95\uc791\uc5c5\uacfc, \ub354 \ub098\uc544\uac00 \ub3c5\uc77c\uc758 3\uc6d4 \ud601\uba85\uae4c\uc9c0 \uc644\uc804\ud788 \uc88c\uc808\ub418\uc5c8\ub2e4. \ub610\ud55c \ud669\uc81c \uc9c0\uc704\uc758 \uac70\uc808\uc740 \ub2e4\ub978 \ub3c5\uc77c \uc81c\ud6c4\ub4e4\uc5d0\uac8c\ub294 \uc815\uce58\uc801 \ud798\uc758 \uad00\uacc4\uac00 \uc790\uc720\uc8fc\uc758\uc790\ub4e4\uc5d0\uac8c \ubc18\ub300\ub85c \uc120\ud68c\ud588\ub2e4\ub294 \uc2e0\ud638\uc600\ub2e4. \uc791\uc740 \uad6d\uac00\ub4e4\uc740 \ud2b9\ud788 \uc81c\uad6d\ud5cc\ubc95\uc744 \ubcc4\ub85c \ub0b4\ud0a4\uc9c0 \uc54a\uac8c \ubc1b\uc544\ub4e4\uc600\uc73c\uba70, \ubdd4\ub974\ud15c\ubca0\ub974\ud06c\ub9cc\uc774 \uc624\ub79c \ub9dd\uc124\uc784 \ub05d\uc5d0 \ud5cc\ubc95\uc791\uc5c5\uc744 \ubc1b\uc544\ub4e4\uc778 \uc720\uc77c\ud55c \uc655\uad6d\uc73c\ub85c \ub0a8\uc558\ub2e4.", "answer": "\uc655\uad8c\uc2e0\uc218\uc124", "sentence": "\uadf8\ub807\uc9c0\ub9cc \uc0ac\uc2e4 \ud504\ub9ac\ub4dc\ub9ac\ud788 \ube4c\ud5ec\ub984\uc740 \uc655\uad8c\uc2e0\uc218\uc124 \uc758 \uc6d0\uce59\uc744 \uace0\uc218\ud588\uace0, \u2018\ud601\uba85\uc758 \uc369\uc740\ub0b4\u2019(Ludergeruch der Revolution)\uac00 \ub098\ub294 \uc5b4\ub5a0\ud55c \uc655\uad00\ub3c4 \ubc1b\uc744 \uc218 \uc5c6\ub2e4\uace0 \uc0dd\uac01\ud588\ub2e4.", "paragraph_sentence": "\uc774\uc5d0 \ub530\ub77c 1849\ub144 4\uc6d4 3\uc77c \uad6d\ubbfc\uc758\ud68c\ub294 \uc5d0\ub450\uc544\ub974\ud2b8 \ud3f0 \uc9d0\uc190\uc774 \uc774\ub044\ub294 \ub300\ud45c\ub2e8(\ud669\uc81c\ub300\ud45c\ub2e8, Kaiserdeputation)\uc744 \ud30c\uacac\ud558\uc5ec \ud504\ub9ac\ub4dc\ub9ac\ud788 \ube4c\ud5ec\ub984 4\uc138\uc5d0\uac8c \ud669\uc81c\uac00 \ub420 \uac83\uc744 \uc81c\uc758\ud588\ub2e4. \uc774\uac83\uc740 \uadf8\ub7ec\ub098 \uc9c0\uc704\ub098 \uad00\uc9c1\uc740 \uc81c\ud6c4\ub4e4\uacfc \uc790\uc720\uc2dc\ub4e4\uc758 \ucc2c\uc131 \uc5c6\uc774\ub294 \ubc1b\uc744 \uc218 \uc5c6\ub2e4\ub294 \ud45c\uba74\uc801\uc778 \uadfc\uac70\uc5d0 \ub530\ub77c \uac70\uc808\ub418\uc5c8\ub2e4. \uadf8\ub807\uc9c0\ub9cc \uc0ac\uc2e4 \ud504\ub9ac\ub4dc\ub9ac\ud788 \ube4c\ud5ec\ub984\uc740 \uc655\uad8c\uc2e0\uc218\uc124 \uc758 \uc6d0\uce59\uc744 \uace0\uc218\ud588\uace0, \u2018\ud601\uba85\uc758 \uc369\uc740\ub0b4\u2019(Ludergeruch der Revolution)\uac00 \ub098\ub294 \uc5b4\ub5a0\ud55c \uc655\uad00\ub3c4 \ubc1b\uc744 \uc218 \uc5c6\ub2e4\uace0 \uc0dd\uac01\ud588\ub2e4. \uc774\ub85c\uc368 \uad6d\ubbfc\uc758\ud68c\uc758 \ud5cc\ubc95\uc791\uc5c5\uacfc, \ub354 \ub098\uc544\uac00 \ub3c5\uc77c\uc758 3\uc6d4 \ud601\uba85\uae4c\uc9c0 \uc644\uc804\ud788 \uc88c\uc808\ub418\uc5c8\ub2e4. \ub610\ud55c \ud669\uc81c \uc9c0\uc704\uc758 \uac70\uc808\uc740 \ub2e4\ub978 \ub3c5\uc77c \uc81c\ud6c4\ub4e4\uc5d0\uac8c\ub294 \uc815\uce58\uc801 \ud798\uc758 \uad00\uacc4\uac00 \uc790\uc720\uc8fc\uc758\uc790\ub4e4\uc5d0\uac8c \ubc18\ub300\ub85c \uc120\ud68c\ud588\ub2e4\ub294 \uc2e0\ud638\uc600\ub2e4. \uc791\uc740 \uad6d\uac00\ub4e4\uc740 \ud2b9\ud788 \uc81c\uad6d\ud5cc\ubc95\uc744 \ubcc4\ub85c \ub0b4\ud0a4\uc9c0 \uc54a\uac8c \ubc1b\uc544\ub4e4\uc600\uc73c\uba70, \ubdd4\ub974\ud15c\ubca0\ub974\ud06c\ub9cc\uc774 \uc624\ub79c \ub9dd\uc124\uc784 \ub05d\uc5d0 \ud5cc\ubc95\uc791\uc5c5\uc744 \ubc1b\uc544\ub4e4\uc778 \uc720\uc77c\ud55c \uc655\uad6d\uc73c\ub85c \ub0a8\uc558\ub2e4.", "paragraph_answer": "\uc774\uc5d0 \ub530\ub77c 1849\ub144 4\uc6d4 3\uc77c \uad6d\ubbfc\uc758\ud68c\ub294 \uc5d0\ub450\uc544\ub974\ud2b8 \ud3f0 \uc9d0\uc190\uc774 \uc774\ub044\ub294 \ub300\ud45c\ub2e8(\ud669\uc81c\ub300\ud45c\ub2e8, Kaiserdeputation)\uc744 \ud30c\uacac\ud558\uc5ec \ud504\ub9ac\ub4dc\ub9ac\ud788 \ube4c\ud5ec\ub984 4\uc138\uc5d0\uac8c \ud669\uc81c\uac00 \ub420 \uac83\uc744 \uc81c\uc758\ud588\ub2e4. \uc774\uac83\uc740 \uadf8\ub7ec\ub098 \uc9c0\uc704\ub098 \uad00\uc9c1\uc740 \uc81c\ud6c4\ub4e4\uacfc \uc790\uc720\uc2dc\ub4e4\uc758 \ucc2c\uc131 \uc5c6\uc774\ub294 \ubc1b\uc744 \uc218 \uc5c6\ub2e4\ub294 \ud45c\uba74\uc801\uc778 \uadfc\uac70\uc5d0 \ub530\ub77c \uac70\uc808\ub418\uc5c8\ub2e4. \uadf8\ub807\uc9c0\ub9cc \uc0ac\uc2e4 \ud504\ub9ac\ub4dc\ub9ac\ud788 \ube4c\ud5ec\ub984\uc740 \uc655\uad8c\uc2e0\uc218\uc124 \uc758 \uc6d0\uce59\uc744 \uace0\uc218\ud588\uace0, \u2018\ud601\uba85\uc758 \uc369\uc740\ub0b4\u2019(Ludergeruch der Revolution)\uac00 \ub098\ub294 \uc5b4\ub5a0\ud55c \uc655\uad00\ub3c4 \ubc1b\uc744 \uc218 \uc5c6\ub2e4\uace0 \uc0dd\uac01\ud588\ub2e4. \uc774\ub85c\uc368 \uad6d\ubbfc\uc758\ud68c\uc758 \ud5cc\ubc95\uc791\uc5c5\uacfc, \ub354 \ub098\uc544\uac00 \ub3c5\uc77c\uc758 3\uc6d4 \ud601\uba85\uae4c\uc9c0 \uc644\uc804\ud788 \uc88c\uc808\ub418\uc5c8\ub2e4. \ub610\ud55c \ud669\uc81c \uc9c0\uc704\uc758 \uac70\uc808\uc740 \ub2e4\ub978 \ub3c5\uc77c \uc81c\ud6c4\ub4e4\uc5d0\uac8c\ub294 \uc815\uce58\uc801 \ud798\uc758 \uad00\uacc4\uac00 \uc790\uc720\uc8fc\uc758\uc790\ub4e4\uc5d0\uac8c \ubc18\ub300\ub85c \uc120\ud68c\ud588\ub2e4\ub294 \uc2e0\ud638\uc600\ub2e4. \uc791\uc740 \uad6d\uac00\ub4e4\uc740 \ud2b9\ud788 \uc81c\uad6d\ud5cc\ubc95\uc744 \ubcc4\ub85c \ub0b4\ud0a4\uc9c0 \uc54a\uac8c \ubc1b\uc544\ub4e4\uc600\uc73c\uba70, \ubdd4\ub974\ud15c\ubca0\ub974\ud06c\ub9cc\uc774 \uc624\ub79c \ub9dd\uc124\uc784 \ub05d\uc5d0 \ud5cc\ubc95\uc791\uc5c5\uc744 \ubc1b\uc544\ub4e4\uc778 \uc720\uc77c\ud55c \uc655\uad6d\uc73c\ub85c \ub0a8\uc558\ub2e4.", "sentence_answer": "\uadf8\ub807\uc9c0\ub9cc \uc0ac\uc2e4 \ud504\ub9ac\ub4dc\ub9ac\ud788 \ube4c\ud5ec\ub984\uc740 \uc655\uad8c\uc2e0\uc218\uc124 \uc758 \uc6d0\uce59\uc744 \uace0\uc218\ud588\uace0, \u2018\ud601\uba85\uc758 \uc369\uc740\ub0b4\u2019(Ludergeruch der Revolution)\uac00 \ub098\ub294 \uc5b4\ub5a0\ud55c \uc655\uad00\ub3c4 \ubc1b\uc744 \uc218 \uc5c6\ub2e4\uace0 \uc0dd\uac01\ud588\ub2e4.", "paragraph_id": "6465718-25-1", "paragraph_question": "question: \ud504\ub9ac\ub4dc\ub9ac\ud788 \ube4c\ud5ec\ub984 4\uc138\ub294 \uc5b4\ub5a4 \uc6d0\uce59\uc73c\ub85c \uad6d\ubbfc\uc758\ud68c\uac00 \ucd94\ub300\ud55c \ud669\uc81c\uc704\ub97c \uac70\ubd80\ud558\uc600\ub294\uac00?, context: \uc774\uc5d0 \ub530\ub77c 1849\ub144 4\uc6d4 3\uc77c \uad6d\ubbfc\uc758\ud68c\ub294 \uc5d0\ub450\uc544\ub974\ud2b8 \ud3f0 \uc9d0\uc190\uc774 \uc774\ub044\ub294 \ub300\ud45c\ub2e8(\ud669\uc81c\ub300\ud45c\ub2e8, Kaiserdeputation)\uc744 \ud30c\uacac\ud558\uc5ec \ud504\ub9ac\ub4dc\ub9ac\ud788 \ube4c\ud5ec\ub984 4\uc138\uc5d0\uac8c \ud669\uc81c\uac00 \ub420 \uac83\uc744 \uc81c\uc758\ud588\ub2e4. \uc774\uac83\uc740 \uadf8\ub7ec\ub098 \uc9c0\uc704\ub098 \uad00\uc9c1\uc740 \uc81c\ud6c4\ub4e4\uacfc \uc790\uc720\uc2dc\ub4e4\uc758 \ucc2c\uc131 \uc5c6\uc774\ub294 \ubc1b\uc744 \uc218 \uc5c6\ub2e4\ub294 \ud45c\uba74\uc801\uc778 \uadfc\uac70\uc5d0 \ub530\ub77c \uac70\uc808\ub418\uc5c8\ub2e4. \uadf8\ub807\uc9c0\ub9cc \uc0ac\uc2e4 \ud504\ub9ac\ub4dc\ub9ac\ud788 \ube4c\ud5ec\ub984\uc740 \uc655\uad8c\uc2e0\uc218\uc124\uc758 \uc6d0\uce59\uc744 \uace0\uc218\ud588\uace0, \u2018\ud601\uba85\uc758 \uc369\uc740\ub0b4\u2019(Ludergeruch der Revolution)\uac00 \ub098\ub294 \uc5b4\ub5a0\ud55c \uc655\uad00\ub3c4 \ubc1b\uc744 \uc218 \uc5c6\ub2e4\uace0 \uc0dd\uac01\ud588\ub2e4. \uc774\ub85c\uc368 \uad6d\ubbfc\uc758\ud68c\uc758 \ud5cc\ubc95\uc791\uc5c5\uacfc, \ub354 \ub098\uc544\uac00 \ub3c5\uc77c\uc758 3\uc6d4 \ud601\uba85\uae4c\uc9c0 \uc644\uc804\ud788 \uc88c\uc808\ub418\uc5c8\ub2e4. \ub610\ud55c \ud669\uc81c \uc9c0\uc704\uc758 \uac70\uc808\uc740 \ub2e4\ub978 \ub3c5\uc77c \uc81c\ud6c4\ub4e4\uc5d0\uac8c\ub294 \uc815\uce58\uc801 \ud798\uc758 \uad00\uacc4\uac00 \uc790\uc720\uc8fc\uc758\uc790\ub4e4\uc5d0\uac8c \ubc18\ub300\ub85c \uc120\ud68c\ud588\ub2e4\ub294 \uc2e0\ud638\uc600\ub2e4. \uc791\uc740 \uad6d\uac00\ub4e4\uc740 \ud2b9\ud788 \uc81c\uad6d\ud5cc\ubc95\uc744 \ubcc4\ub85c \ub0b4\ud0a4\uc9c0 \uc54a\uac8c \ubc1b\uc544\ub4e4\uc600\uc73c\uba70, \ubdd4\ub974\ud15c\ubca0\ub974\ud06c\ub9cc\uc774 \uc624\ub79c \ub9dd\uc124\uc784 \ub05d\uc5d0 \ud5cc\ubc95\uc791\uc5c5\uc744 \ubc1b\uc544\ub4e4\uc778 \uc720\uc77c\ud55c \uc655\uad6d\uc73c\ub85c \ub0a8\uc558\ub2e4."} {"question": "\uc2a4\ud29c\uc5b4\ud2b8\uac00 \uc778\ub354 \ub79c\ub4dc\uc624\ube0c \uc6b0\uba3c\uc5d0\uc11c \ub9e1\uc740 \uc5ed\ud560\uc740?", "paragraph": "2007\ub144, \uc2a4\ud29c\uc5b4\ud2b8\ub294 \uba54\uadf8 \ub77c\uc774\uc5b8\uacfc \uc544\ub2f4 \ube0c\ub85c\ub514\uac00 \ucd9c\uc5f0\ud55c \ub85c\ub9e8\ud2f1 \ucf54\ubbf8\ub514 \uc601\ud654 \u300a\uc778 \ub354 \ub79c\ub4dc \uc624\ube0c \uc6b0\uba3c\u300b\uc5d0\uc11c 10\ub300 \ub8e8\uc2dc \ud558\ub4dc\uc705\uc73c\ub85c \ucd9c\uc5f0\ud588\ub2e4. \uc601\ud654\ub294 \uc2a4\ud29c\uc5b4\ud2b8\uc758 \uc5f0\uae30\uc640 \ud568\uaed8 \ud638\ud3c9\uacfc \uc545\ud3c9\uc73c\ub85c \ub098\ub258\uc5c8\ub2e4. \uac19\uc740 \ud574\uc5d0, \uadf8\ub294 \ube44\ud3c9\uac00\ub4e4\uc5d0\uac8c \uac08\ucc44\ub97c \ubc1b\uc740 \uc200 \ud39c\uc758 \ubc88\uc548 \uc601\ud654 \u300a\uc778\ud22c \ub354 \uc640\uc77c\ub4dc\u300b\uc5d0 \ucd9c\uc5f0\ud588\ub2e4. \uc80a\uc740 \ubaa8\ud5d8\uac00 \ud06c\ub9ac\uc2a4\ud1a0\ud37c \ub9e5\uce94\ub4e4\ub808\uc2a4\uc5d0\uac8c \ubc18\ud55c 10\ub300 \uac00\uc218 \ud2b8\ub808\uc774\uc2dc \uc5ed\uc744 \uc5f0\uae30\ud558\uba70 \ub300\uccb4\ub85c \ud638\ud3c9\uc744 \ubc1b\uc558\ub2e4. \uc0b4\ub871\ub2f7\ucef4\uc740 \u201c\ub2e8\ub2e8\ud558\uace0 \uc12c\uc138\ud55c \uc5f0\uae30\u201d\ub77c\uace0 \ud3c9\uac00\ud588\uace0, \uc2dc\uce74\uace0 \ud2b8\ub9ac\ubdf4\uc740 \uadf8\uac00 \u201c\uc5ed\ud560\uc758 \uc2a4\ucf00\uce58\ub97c \uc120\uba85\ud558\uac8c \uc798 \ud574\ub0b4\uc5c8\ub2e4\u201d\uace0 \uc5b8\uae09\ud588\ub2e4. \uadf8\uc758 \uc5f0\uae30\uc5d0 \uacb0\uc810\uc774 \uc5c6\ub294 \uac83\uc740 \uc544\ub2c8\uc5c8\ub2e4. \ubc84\ub77c\uc774\uc5b4\ud2f0 \uc9c0\uc758 \ube44\ud3c9\uac00 \ub370\ub2c8\uc2a4 \ud558\ube44\ub294 \"\uc2a4\ud29c\uc5b4\ud2b8\uac00 \uc0dd\uae30 \uc5c6\ub294 \ud788\ud53c \uc18c\ub140 \ud2b8\ub808\uc774\uc2dc\ub97c \ud45c\ud604\ud558\ub824\uace0 \ud588\ub294\uc9c0 \uc544\ub2c8\uba74 \uadf8\ub0e5 \uadf8\ub807\uac8c \ub41c \uac83\uc778\uc9c0\ub294 \ud655\uc2e4\ud558\uc9c0 \uc54a\ub2e4.\"\ub77c\uace0 \ud3c9\ud588\ub2e4.", "answer": "\ub8e8\uc2dc \ud558\ub4dc\uc705", "sentence": "2007\ub144, \uc2a4\ud29c\uc5b4\ud2b8\ub294 \uba54\uadf8 \ub77c\uc774\uc5b8\uacfc \uc544\ub2f4 \ube0c\ub85c\ub514\uac00 \ucd9c\uc5f0\ud55c \ub85c\ub9e8\ud2f1 \ucf54\ubbf8\ub514 \uc601\ud654 \u300a\uc778 \ub354 \ub79c\ub4dc \uc624\ube0c \uc6b0\uba3c\u300b\uc5d0\uc11c 10\ub300 \ub8e8\uc2dc \ud558\ub4dc\uc705 \uc73c\ub85c \ucd9c\uc5f0\ud588\ub2e4.", "paragraph_sentence": " 2007\ub144, \uc2a4\ud29c\uc5b4\ud2b8\ub294 \uba54\uadf8 \ub77c\uc774\uc5b8\uacfc \uc544\ub2f4 \ube0c\ub85c\ub514\uac00 \ucd9c\uc5f0\ud55c \ub85c\ub9e8\ud2f1 \ucf54\ubbf8\ub514 \uc601\ud654 \u300a\uc778 \ub354 \ub79c\ub4dc \uc624\ube0c \uc6b0\uba3c\u300b\uc5d0\uc11c 10\ub300 \ub8e8\uc2dc \ud558\ub4dc\uc705 \uc73c\ub85c \ucd9c\uc5f0\ud588\ub2e4. \uc601\ud654\ub294 \uc2a4\ud29c\uc5b4\ud2b8\uc758 \uc5f0\uae30\uc640 \ud568\uaed8 \ud638\ud3c9\uacfc \uc545\ud3c9\uc73c\ub85c \ub098\ub258\uc5c8\ub2e4. \uac19\uc740 \ud574\uc5d0, \uadf8\ub294 \ube44\ud3c9\uac00\ub4e4\uc5d0\uac8c \uac08\ucc44\ub97c \ubc1b\uc740 \uc200 \ud39c\uc758 \ubc88\uc548 \uc601\ud654 \u300a\uc778\ud22c \ub354 \uc640\uc77c\ub4dc\u300b\uc5d0 \ucd9c\uc5f0\ud588\ub2e4. \uc80a\uc740 \ubaa8\ud5d8\uac00 \ud06c\ub9ac\uc2a4\ud1a0\ud37c \ub9e5\uce94\ub4e4\ub808\uc2a4\uc5d0\uac8c \ubc18\ud55c 10\ub300 \uac00\uc218 \ud2b8\ub808\uc774\uc2dc \uc5ed\uc744 \uc5f0\uae30\ud558\uba70 \ub300\uccb4\ub85c \ud638\ud3c9\uc744 \ubc1b\uc558\ub2e4. \uc0b4\ub871\ub2f7\ucef4\uc740 \u201c\ub2e8\ub2e8\ud558\uace0 \uc12c\uc138\ud55c \uc5f0\uae30\u201d\ub77c\uace0 \ud3c9\uac00\ud588\uace0, \uc2dc\uce74\uace0 \ud2b8\ub9ac\ubdf4\uc740 \uadf8\uac00 \u201c\uc5ed\ud560\uc758 \uc2a4\ucf00\uce58\ub97c \uc120\uba85\ud558\uac8c \uc798 \ud574\ub0b4\uc5c8\ub2e4\u201d\uace0 \uc5b8\uae09\ud588\ub2e4. \uadf8\uc758 \uc5f0\uae30\uc5d0 \uacb0\uc810\uc774 \uc5c6\ub294 \uac83\uc740 \uc544\ub2c8\uc5c8\ub2e4. \ubc84\ub77c\uc774\uc5b4\ud2f0 \uc9c0\uc758 \ube44\ud3c9\uac00 \ub370\ub2c8\uc2a4 \ud558\ube44\ub294 \"\uc2a4\ud29c\uc5b4\ud2b8\uac00 \uc0dd\uae30 \uc5c6\ub294 \ud788\ud53c \uc18c\ub140 \ud2b8\ub808\uc774\uc2dc\ub97c \ud45c\ud604\ud558\ub824\uace0 \ud588\ub294\uc9c0 \uc544\ub2c8\uba74 \uadf8\ub0e5 \uadf8\ub807\uac8c \ub41c \uac83\uc778\uc9c0\ub294 \ud655\uc2e4\ud558\uc9c0 \uc54a\ub2e4. \"\ub77c\uace0 \ud3c9\ud588\ub2e4.", "paragraph_answer": "2007\ub144, \uc2a4\ud29c\uc5b4\ud2b8\ub294 \uba54\uadf8 \ub77c\uc774\uc5b8\uacfc \uc544\ub2f4 \ube0c\ub85c\ub514\uac00 \ucd9c\uc5f0\ud55c \ub85c\ub9e8\ud2f1 \ucf54\ubbf8\ub514 \uc601\ud654 \u300a\uc778 \ub354 \ub79c\ub4dc \uc624\ube0c \uc6b0\uba3c\u300b\uc5d0\uc11c 10\ub300 \ub8e8\uc2dc \ud558\ub4dc\uc705 \uc73c\ub85c \ucd9c\uc5f0\ud588\ub2e4. \uc601\ud654\ub294 \uc2a4\ud29c\uc5b4\ud2b8\uc758 \uc5f0\uae30\uc640 \ud568\uaed8 \ud638\ud3c9\uacfc \uc545\ud3c9\uc73c\ub85c \ub098\ub258\uc5c8\ub2e4. \uac19\uc740 \ud574\uc5d0, \uadf8\ub294 \ube44\ud3c9\uac00\ub4e4\uc5d0\uac8c \uac08\ucc44\ub97c \ubc1b\uc740 \uc200 \ud39c\uc758 \ubc88\uc548 \uc601\ud654 \u300a\uc778\ud22c \ub354 \uc640\uc77c\ub4dc\u300b\uc5d0 \ucd9c\uc5f0\ud588\ub2e4. \uc80a\uc740 \ubaa8\ud5d8\uac00 \ud06c\ub9ac\uc2a4\ud1a0\ud37c \ub9e5\uce94\ub4e4\ub808\uc2a4\uc5d0\uac8c \ubc18\ud55c 10\ub300 \uac00\uc218 \ud2b8\ub808\uc774\uc2dc \uc5ed\uc744 \uc5f0\uae30\ud558\uba70 \ub300\uccb4\ub85c \ud638\ud3c9\uc744 \ubc1b\uc558\ub2e4. \uc0b4\ub871\ub2f7\ucef4\uc740 \u201c\ub2e8\ub2e8\ud558\uace0 \uc12c\uc138\ud55c \uc5f0\uae30\u201d\ub77c\uace0 \ud3c9\uac00\ud588\uace0, \uc2dc\uce74\uace0 \ud2b8\ub9ac\ubdf4\uc740 \uadf8\uac00 \u201c\uc5ed\ud560\uc758 \uc2a4\ucf00\uce58\ub97c \uc120\uba85\ud558\uac8c \uc798 \ud574\ub0b4\uc5c8\ub2e4\u201d\uace0 \uc5b8\uae09\ud588\ub2e4. \uadf8\uc758 \uc5f0\uae30\uc5d0 \uacb0\uc810\uc774 \uc5c6\ub294 \uac83\uc740 \uc544\ub2c8\uc5c8\ub2e4. \ubc84\ub77c\uc774\uc5b4\ud2f0 \uc9c0\uc758 \ube44\ud3c9\uac00 \ub370\ub2c8\uc2a4 \ud558\ube44\ub294 \"\uc2a4\ud29c\uc5b4\ud2b8\uac00 \uc0dd\uae30 \uc5c6\ub294 \ud788\ud53c \uc18c\ub140 \ud2b8\ub808\uc774\uc2dc\ub97c \ud45c\ud604\ud558\ub824\uace0 \ud588\ub294\uc9c0 \uc544\ub2c8\uba74 \uadf8\ub0e5 \uadf8\ub807\uac8c \ub41c \uac83\uc778\uc9c0\ub294 \ud655\uc2e4\ud558\uc9c0 \uc54a\ub2e4.\"\ub77c\uace0 \ud3c9\ud588\ub2e4.", "sentence_answer": "2007\ub144, \uc2a4\ud29c\uc5b4\ud2b8\ub294 \uba54\uadf8 \ub77c\uc774\uc5b8\uacfc \uc544\ub2f4 \ube0c\ub85c\ub514\uac00 \ucd9c\uc5f0\ud55c \ub85c\ub9e8\ud2f1 \ucf54\ubbf8\ub514 \uc601\ud654 \u300a\uc778 \ub354 \ub79c\ub4dc \uc624\ube0c \uc6b0\uba3c\u300b\uc5d0\uc11c 10\ub300 \ub8e8\uc2dc \ud558\ub4dc\uc705 \uc73c\ub85c \ucd9c\uc5f0\ud588\ub2e4.", "paragraph_id": "6564059-3-1", "paragraph_question": "question: \uc2a4\ud29c\uc5b4\ud2b8\uac00 \uc778\ub354 \ub79c\ub4dc\uc624\ube0c \uc6b0\uba3c\uc5d0\uc11c \ub9e1\uc740 \uc5ed\ud560\uc740?, context: 2007\ub144, \uc2a4\ud29c\uc5b4\ud2b8\ub294 \uba54\uadf8 \ub77c\uc774\uc5b8\uacfc \uc544\ub2f4 \ube0c\ub85c\ub514\uac00 \ucd9c\uc5f0\ud55c \ub85c\ub9e8\ud2f1 \ucf54\ubbf8\ub514 \uc601\ud654 \u300a\uc778 \ub354 \ub79c\ub4dc \uc624\ube0c \uc6b0\uba3c\u300b\uc5d0\uc11c 10\ub300 \ub8e8\uc2dc \ud558\ub4dc\uc705\uc73c\ub85c \ucd9c\uc5f0\ud588\ub2e4. \uc601\ud654\ub294 \uc2a4\ud29c\uc5b4\ud2b8\uc758 \uc5f0\uae30\uc640 \ud568\uaed8 \ud638\ud3c9\uacfc \uc545\ud3c9\uc73c\ub85c \ub098\ub258\uc5c8\ub2e4. \uac19\uc740 \ud574\uc5d0, \uadf8\ub294 \ube44\ud3c9\uac00\ub4e4\uc5d0\uac8c \uac08\ucc44\ub97c \ubc1b\uc740 \uc200 \ud39c\uc758 \ubc88\uc548 \uc601\ud654 \u300a\uc778\ud22c \ub354 \uc640\uc77c\ub4dc\u300b\uc5d0 \ucd9c\uc5f0\ud588\ub2e4. \uc80a\uc740 \ubaa8\ud5d8\uac00 \ud06c\ub9ac\uc2a4\ud1a0\ud37c \ub9e5\uce94\ub4e4\ub808\uc2a4\uc5d0\uac8c \ubc18\ud55c 10\ub300 \uac00\uc218 \ud2b8\ub808\uc774\uc2dc \uc5ed\uc744 \uc5f0\uae30\ud558\uba70 \ub300\uccb4\ub85c \ud638\ud3c9\uc744 \ubc1b\uc558\ub2e4. \uc0b4\ub871\ub2f7\ucef4\uc740 \u201c\ub2e8\ub2e8\ud558\uace0 \uc12c\uc138\ud55c \uc5f0\uae30\u201d\ub77c\uace0 \ud3c9\uac00\ud588\uace0, \uc2dc\uce74\uace0 \ud2b8\ub9ac\ubdf4\uc740 \uadf8\uac00 \u201c\uc5ed\ud560\uc758 \uc2a4\ucf00\uce58\ub97c \uc120\uba85\ud558\uac8c \uc798 \ud574\ub0b4\uc5c8\ub2e4\u201d\uace0 \uc5b8\uae09\ud588\ub2e4. \uadf8\uc758 \uc5f0\uae30\uc5d0 \uacb0\uc810\uc774 \uc5c6\ub294 \uac83\uc740 \uc544\ub2c8\uc5c8\ub2e4. \ubc84\ub77c\uc774\uc5b4\ud2f0 \uc9c0\uc758 \ube44\ud3c9\uac00 \ub370\ub2c8\uc2a4 \ud558\ube44\ub294 \"\uc2a4\ud29c\uc5b4\ud2b8\uac00 \uc0dd\uae30 \uc5c6\ub294 \ud788\ud53c \uc18c\ub140 \ud2b8\ub808\uc774\uc2dc\ub97c \ud45c\ud604\ud558\ub824\uace0 \ud588\ub294\uc9c0 \uc544\ub2c8\uba74 \uadf8\ub0e5 \uadf8\ub807\uac8c \ub41c \uac83\uc778\uc9c0\ub294 \ud655\uc2e4\ud558\uc9c0 \uc54a\ub2e4.\"\ub77c\uace0 \ud3c9\ud588\ub2e4."} {"question": "\uc11c\uc6d0\uc774 \ucd9c\uc5f0\ud55c \uc2e0\ub144\ud2b9\uc9d1\uadf9\uc740?", "paragraph": "\uc11c\uc6d0\uc740 \uc5f0\ud76c\uc5ec\uc911 2\ud559\ub144 \ub54c\uc778 1993\ub144 \ubbf8\uc2a4\ube59\uadf8\ub808 \uc120\ubc1c\ub300\ud68c\uc5d0\uc11c 2\ub4f1\uc73c\ub85c \uc785\uc0c1\ud55c \ud6c4, \uc911\ud559\uad50 3\ud559\ub144\uc5d0 \uc62c\ub77c\uac00\uba70 \uc9c0\uc778\uc758 \uc18c\uac1c\ub85c MBC \uccad\uc18c\ub144 \ub4dc\ub77c\ub9c8 \u300a\uc0ac\ucd98\uae30\u300b \uc624\ub514\uc158\uc744 \ubd10\uc11c \ud569\uaca9\ud558\uba74\uc11c \uc5f0\uae30 \ud65c\ub3d9\uc744 \uc2dc\uc791\ud588\ub2e4. \uc11c\uc6d0\uc740 \u300a\uc0ac\ucd98\uae30\u300b\uc5d0 1994\ub144 3\uc6d4\ubd80\ud130 1\ub144\uac04 \ucd9c\uc5f0\ud588\ub294\ub370, \uadf8\ub140\uac00 \ub9e1\uc740 \uc5ed\ud560\uc740 \uc774\ub3d9\ubbfc(\uc815\uc900 \ubd84)\uc758 \uc0c8 \uc5ec\uc790\uce5c\uad6c\uc778 \ubc15\uc131\ud76c \uc5ed\uc774\uc5c8\ub2e4. \uac19\uc740 \ud574 SBS \uc218\ubaa9\ub4dc\ub77c\ub9c8 \u300a\uc0ac\ub791\uc740 \uc5c6\ub2e4\u300b\uc5d0\ub3c4 \uc5b4\ub9b0 \uc774\ud654\uc601(\ud669\uc2e0\ud61c \ubd84) \uc5ed\uc73c\ub85c \ucd9c\uc5f0\ud558\uc600\ub2e4. 1995\ub144\uc5d0\ub294 SBS\uc758 \ucd94\ub9ac\uadf9 \u300a\u674e\uac00\uc0ac \ud06c\ub9ac\uc2a4\ud2f0\u300b\uc5d0 \uae40\ud601(\uae40\uc815\ud604 \ubd84)\uc758 \uc5ec\ub3d9\uc0dd \uc5ed\uc73c\ub85c \ub2e8\uc5ed \ucd9c\uc5f0\ud588\ub2e4. \uc774\ud6c4 1996\ub144 \ubaa8\ubc94\uc0dd\uacfc \ubd88\ub7c9\ud559\uc0dd\uc758 \uc6b0\uc815\uc744 \uadf8\ub9b0 \ub2e8\ub9c9\uadf9 \u300a\ub4dc\ub77c\ub9c8\uac8c\uc784\u300b \u3008\ucef4\ubc31\ud648\u3009 \ud3b8\uc5d0 \uae40\uc18c\uc5f0\uacfc \uac19\uc774 \ucd9c\uc5f0\ud574 \ubaa8\ubc94\uc0dd \uc120\uc560 \uc5ed\uc744, KBS \uccad\uc18c\ub144 \ub4dc\ub77c\ub9c8 \u300a\uc0ac\ub791\uc774 \uaf43\ud53c\ub294 \uacc4\uc808\u300b\uc5d0\uc11c\ub294 \ubc15\uc815\uc6b0(\uc774\uc601\ud638 \ubd84)\uc758 \uc5ec\uc790\uce5c\uad6c\uc778 \uc11c\ucc44\uc601 \uc5ed\uc744 \uc5f0\uae30\ud588\ub2e4. \uc774\ub4ec\ud574\uc778 1997\ub144\uc5d0\ub294 \uc2e0\ub144\ud2b9\uc9d1\uadf9 \u300a\ud669\uae08\uae43\ud138\u300b\uc5d0 \ubc15\uc900\uad6c(\ubc31\uc77c\uc12d \ubd84)\uc758 \ub538 \ubc15\uc218\uc815 \uc5ed\uc73c\ub85c \ucd9c\uc5f0\ud588\ub2e4.", "answer": "\ud669\uae08\uae43\ud138", "sentence": "\uc774\ub4ec\ud574\uc778 1997\ub144\uc5d0\ub294 \uc2e0\ub144\ud2b9\uc9d1\uadf9 \u300a \ud669\uae08\uae43\ud138 \u300b\uc5d0 \ubc15\uc900\uad6c(\ubc31\uc77c\uc12d \ubd84)\uc758 \ub538 \ubc15\uc218\uc815 \uc5ed\uc73c\ub85c \ucd9c\uc5f0\ud588\ub2e4.", "paragraph_sentence": "\uc11c\uc6d0\uc740 \uc5f0\ud76c\uc5ec\uc911 2\ud559\ub144 \ub54c\uc778 1993\ub144 \ubbf8\uc2a4\ube59\uadf8\ub808 \uc120\ubc1c\ub300\ud68c\uc5d0\uc11c 2\ub4f1\uc73c\ub85c \uc785\uc0c1\ud55c \ud6c4, \uc911\ud559\uad50 3\ud559\ub144\uc5d0 \uc62c\ub77c\uac00\uba70 \uc9c0\uc778\uc758 \uc18c\uac1c\ub85c MBC \uccad\uc18c\ub144 \ub4dc\ub77c\ub9c8 \u300a\uc0ac\ucd98\uae30\u300b \uc624\ub514\uc158\uc744 \ubd10\uc11c \ud569\uaca9\ud558\uba74\uc11c \uc5f0\uae30 \ud65c\ub3d9\uc744 \uc2dc\uc791\ud588\ub2e4. \uc11c\uc6d0\uc740 \u300a\uc0ac\ucd98\uae30\u300b\uc5d0 1994\ub144 3\uc6d4\ubd80\ud130 1\ub144\uac04 \ucd9c\uc5f0\ud588\ub294\ub370, \uadf8\ub140\uac00 \ub9e1\uc740 \uc5ed\ud560\uc740 \uc774\ub3d9\ubbfc(\uc815\uc900 \ubd84)\uc758 \uc0c8 \uc5ec\uc790\uce5c\uad6c\uc778 \ubc15\uc131\ud76c \uc5ed\uc774\uc5c8\ub2e4. \uac19\uc740 \ud574 SBS \uc218\ubaa9\ub4dc\ub77c\ub9c8 \u300a\uc0ac\ub791\uc740 \uc5c6\ub2e4\u300b\uc5d0\ub3c4 \uc5b4\ub9b0 \uc774\ud654\uc601(\ud669\uc2e0\ud61c \ubd84) \uc5ed\uc73c\ub85c \ucd9c\uc5f0\ud558\uc600\ub2e4. 1995\ub144\uc5d0\ub294 SBS\uc758 \ucd94\ub9ac\uadf9 \u300a\u674e\uac00\uc0ac \ud06c\ub9ac\uc2a4\ud2f0\u300b\uc5d0 \uae40\ud601(\uae40\uc815\ud604 \ubd84)\uc758 \uc5ec\ub3d9\uc0dd \uc5ed\uc73c\ub85c \ub2e8\uc5ed \ucd9c\uc5f0\ud588\ub2e4. \uc774\ud6c4 1996\ub144 \ubaa8\ubc94\uc0dd\uacfc \ubd88\ub7c9\ud559\uc0dd\uc758 \uc6b0\uc815\uc744 \uadf8\ub9b0 \ub2e8\ub9c9\uadf9 \u300a\ub4dc\ub77c\ub9c8\uac8c\uc784\u300b \u3008\ucef4\ubc31\ud648\u3009 \ud3b8\uc5d0 \uae40\uc18c\uc5f0\uacfc \uac19\uc774 \ucd9c\uc5f0\ud574 \ubaa8\ubc94\uc0dd \uc120\uc560 \uc5ed\uc744, KBS \uccad\uc18c\ub144 \ub4dc\ub77c\ub9c8 \u300a\uc0ac\ub791\uc774 \uaf43\ud53c\ub294 \uacc4\uc808\u300b\uc5d0\uc11c\ub294 \ubc15\uc815\uc6b0(\uc774\uc601\ud638 \ubd84)\uc758 \uc5ec\uc790\uce5c\uad6c\uc778 \uc11c\ucc44\uc601 \uc5ed\uc744 \uc5f0\uae30\ud588\ub2e4. \uc774\ub4ec\ud574\uc778 1997\ub144\uc5d0\ub294 \uc2e0\ub144\ud2b9\uc9d1\uadf9 \u300a \ud669\uae08\uae43\ud138 \u300b\uc5d0 \ubc15\uc900\uad6c(\ubc31\uc77c\uc12d \ubd84)\uc758 \ub538 \ubc15\uc218\uc815 \uc5ed\uc73c\ub85c \ucd9c\uc5f0\ud588\ub2e4. ", "paragraph_answer": "\uc11c\uc6d0\uc740 \uc5f0\ud76c\uc5ec\uc911 2\ud559\ub144 \ub54c\uc778 1993\ub144 \ubbf8\uc2a4\ube59\uadf8\ub808 \uc120\ubc1c\ub300\ud68c\uc5d0\uc11c 2\ub4f1\uc73c\ub85c \uc785\uc0c1\ud55c \ud6c4, \uc911\ud559\uad50 3\ud559\ub144\uc5d0 \uc62c\ub77c\uac00\uba70 \uc9c0\uc778\uc758 \uc18c\uac1c\ub85c MBC \uccad\uc18c\ub144 \ub4dc\ub77c\ub9c8 \u300a\uc0ac\ucd98\uae30\u300b \uc624\ub514\uc158\uc744 \ubd10\uc11c \ud569\uaca9\ud558\uba74\uc11c \uc5f0\uae30 \ud65c\ub3d9\uc744 \uc2dc\uc791\ud588\ub2e4. \uc11c\uc6d0\uc740 \u300a\uc0ac\ucd98\uae30\u300b\uc5d0 1994\ub144 3\uc6d4\ubd80\ud130 1\ub144\uac04 \ucd9c\uc5f0\ud588\ub294\ub370, \uadf8\ub140\uac00 \ub9e1\uc740 \uc5ed\ud560\uc740 \uc774\ub3d9\ubbfc(\uc815\uc900 \ubd84)\uc758 \uc0c8 \uc5ec\uc790\uce5c\uad6c\uc778 \ubc15\uc131\ud76c \uc5ed\uc774\uc5c8\ub2e4. \uac19\uc740 \ud574 SBS \uc218\ubaa9\ub4dc\ub77c\ub9c8 \u300a\uc0ac\ub791\uc740 \uc5c6\ub2e4\u300b\uc5d0\ub3c4 \uc5b4\ub9b0 \uc774\ud654\uc601(\ud669\uc2e0\ud61c \ubd84) \uc5ed\uc73c\ub85c \ucd9c\uc5f0\ud558\uc600\ub2e4. 1995\ub144\uc5d0\ub294 SBS\uc758 \ucd94\ub9ac\uadf9 \u300a\u674e\uac00\uc0ac \ud06c\ub9ac\uc2a4\ud2f0\u300b\uc5d0 \uae40\ud601(\uae40\uc815\ud604 \ubd84)\uc758 \uc5ec\ub3d9\uc0dd \uc5ed\uc73c\ub85c \ub2e8\uc5ed \ucd9c\uc5f0\ud588\ub2e4. \uc774\ud6c4 1996\ub144 \ubaa8\ubc94\uc0dd\uacfc \ubd88\ub7c9\ud559\uc0dd\uc758 \uc6b0\uc815\uc744 \uadf8\ub9b0 \ub2e8\ub9c9\uadf9 \u300a\ub4dc\ub77c\ub9c8\uac8c\uc784\u300b \u3008\ucef4\ubc31\ud648\u3009 \ud3b8\uc5d0 \uae40\uc18c\uc5f0\uacfc \uac19\uc774 \ucd9c\uc5f0\ud574 \ubaa8\ubc94\uc0dd \uc120\uc560 \uc5ed\uc744, KBS \uccad\uc18c\ub144 \ub4dc\ub77c\ub9c8 \u300a\uc0ac\ub791\uc774 \uaf43\ud53c\ub294 \uacc4\uc808\u300b\uc5d0\uc11c\ub294 \ubc15\uc815\uc6b0(\uc774\uc601\ud638 \ubd84)\uc758 \uc5ec\uc790\uce5c\uad6c\uc778 \uc11c\ucc44\uc601 \uc5ed\uc744 \uc5f0\uae30\ud588\ub2e4. \uc774\ub4ec\ud574\uc778 1997\ub144\uc5d0\ub294 \uc2e0\ub144\ud2b9\uc9d1\uadf9 \u300a \ud669\uae08\uae43\ud138 \u300b\uc5d0 \ubc15\uc900\uad6c(\ubc31\uc77c\uc12d \ubd84)\uc758 \ub538 \ubc15\uc218\uc815 \uc5ed\uc73c\ub85c \ucd9c\uc5f0\ud588\ub2e4.", "sentence_answer": "\uc774\ub4ec\ud574\uc778 1997\ub144\uc5d0\ub294 \uc2e0\ub144\ud2b9\uc9d1\uadf9 \u300a \ud669\uae08\uae43\ud138 \u300b\uc5d0 \ubc15\uc900\uad6c(\ubc31\uc77c\uc12d \ubd84)\uc758 \ub538 \ubc15\uc218\uc815 \uc5ed\uc73c\ub85c \ucd9c\uc5f0\ud588\ub2e4.", "paragraph_id": "6531862-0-1", "paragraph_question": "question: \uc11c\uc6d0\uc774 \ucd9c\uc5f0\ud55c \uc2e0\ub144\ud2b9\uc9d1\uadf9\uc740?, context: \uc11c\uc6d0\uc740 \uc5f0\ud76c\uc5ec\uc911 2\ud559\ub144 \ub54c\uc778 1993\ub144 \ubbf8\uc2a4\ube59\uadf8\ub808 \uc120\ubc1c\ub300\ud68c\uc5d0\uc11c 2\ub4f1\uc73c\ub85c \uc785\uc0c1\ud55c \ud6c4, \uc911\ud559\uad50 3\ud559\ub144\uc5d0 \uc62c\ub77c\uac00\uba70 \uc9c0\uc778\uc758 \uc18c\uac1c\ub85c MBC \uccad\uc18c\ub144 \ub4dc\ub77c\ub9c8 \u300a\uc0ac\ucd98\uae30\u300b \uc624\ub514\uc158\uc744 \ubd10\uc11c \ud569\uaca9\ud558\uba74\uc11c \uc5f0\uae30 \ud65c\ub3d9\uc744 \uc2dc\uc791\ud588\ub2e4. \uc11c\uc6d0\uc740 \u300a\uc0ac\ucd98\uae30\u300b\uc5d0 1994\ub144 3\uc6d4\ubd80\ud130 1\ub144\uac04 \ucd9c\uc5f0\ud588\ub294\ub370, \uadf8\ub140\uac00 \ub9e1\uc740 \uc5ed\ud560\uc740 \uc774\ub3d9\ubbfc(\uc815\uc900 \ubd84)\uc758 \uc0c8 \uc5ec\uc790\uce5c\uad6c\uc778 \ubc15\uc131\ud76c \uc5ed\uc774\uc5c8\ub2e4. \uac19\uc740 \ud574 SBS \uc218\ubaa9\ub4dc\ub77c\ub9c8 \u300a\uc0ac\ub791\uc740 \uc5c6\ub2e4\u300b\uc5d0\ub3c4 \uc5b4\ub9b0 \uc774\ud654\uc601(\ud669\uc2e0\ud61c \ubd84) \uc5ed\uc73c\ub85c \ucd9c\uc5f0\ud558\uc600\ub2e4. 1995\ub144\uc5d0\ub294 SBS\uc758 \ucd94\ub9ac\uadf9 \u300a\u674e\uac00\uc0ac \ud06c\ub9ac\uc2a4\ud2f0\u300b\uc5d0 \uae40\ud601(\uae40\uc815\ud604 \ubd84)\uc758 \uc5ec\ub3d9\uc0dd \uc5ed\uc73c\ub85c \ub2e8\uc5ed \ucd9c\uc5f0\ud588\ub2e4. \uc774\ud6c4 1996\ub144 \ubaa8\ubc94\uc0dd\uacfc \ubd88\ub7c9\ud559\uc0dd\uc758 \uc6b0\uc815\uc744 \uadf8\ub9b0 \ub2e8\ub9c9\uadf9 \u300a\ub4dc\ub77c\ub9c8\uac8c\uc784\u300b \u3008\ucef4\ubc31\ud648\u3009 \ud3b8\uc5d0 \uae40\uc18c\uc5f0\uacfc \uac19\uc774 \ucd9c\uc5f0\ud574 \ubaa8\ubc94\uc0dd \uc120\uc560 \uc5ed\uc744, KBS \uccad\uc18c\ub144 \ub4dc\ub77c\ub9c8 \u300a\uc0ac\ub791\uc774 \uaf43\ud53c\ub294 \uacc4\uc808\u300b\uc5d0\uc11c\ub294 \ubc15\uc815\uc6b0(\uc774\uc601\ud638 \ubd84)\uc758 \uc5ec\uc790\uce5c\uad6c\uc778 \uc11c\ucc44\uc601 \uc5ed\uc744 \uc5f0\uae30\ud588\ub2e4. \uc774\ub4ec\ud574\uc778 1997\ub144\uc5d0\ub294 \uc2e0\ub144\ud2b9\uc9d1\uadf9 \u300a\ud669\uae08\uae43\ud138\u300b\uc5d0 \ubc15\uc900\uad6c(\ubc31\uc77c\uc12d \ubd84)\uc758 \ub538 \ubc15\uc218\uc815 \uc5ed\uc73c\ub85c \ucd9c\uc5f0\ud588\ub2e4."} {"question": "\uc2dc\ud638\ub294 \ubb34\uc5c7\uc778\uac00?", "paragraph": "1455\ub144 \uc138\uc870 \ubc18\uc815 \uc9c1\ud6c4 \uc88c\uc775\uacf5\uc2e0 3\ub4f1\uc774 \ub418\uace0 \uc608\uc870\ud310\uc11c \ub4f1\uc744 \uac70\uccd0 \ubaa8\ub9b0\uc704 \uc5ec\uc9c4\uc871\uc758 \ub09c\uacfc \uac74\uc8fc\uc704 \uc5ec\uc9c4\uc871\uc758 \ub09c\uc744 \ud1a0\ubc8c\ud558\uace0 \uc6b0\uc758\uc815\uc744 \uc9c0\ub0c8\ub2e4. \uc608\uc885 \uc989\uc704 \ud6c4\uc5d0\ub294 \uc88c\uc758\uc815\uacfc \uc601\uc758\uc815\uc744 \uc9c0\ub0c8\ub2e4. \uc608\uc885 \uc0ac\ud6c4\uc5d0\ub294 \uc131\uc885\uc744 \uc9c0\uc9c0\ud55c \uacf5\ub85c\ub85c \ubd80\uc6d0\uad70\uc5d0 \uc9c4\ubd09\ub418\uace0 \uc88c\ub9ac\uacf5\uc2e0 1\ub4f1\uc774 \ub418\uc5c8\ub2e4. \uc131\uc9c8\uc774 \uc0ac\ub098\uc6cc \uad8c\uc138\ub97c \uc5bb\uc740 \ub4a4\uc5d0\ub294 \uc8fc\ubcc0\uc744 \ub3cc\uc544\ubcf4\uc9c0 \uc54a\uace0 \uc624\ub85c\uc9c0 \uae30\uc138(\u6c23\u52e2)\ub85c\uc368 \ub2e4\ub978 \uc0ac\ub78c\uc744 \ub2a5\uba78\ud558\uace0, \uac00\ub178(\u5bb6\u5974)\ub97c \ub193\uc544 \uc0ac\ub78c\uc744 \uc8fd\uc774\uae30\ub3c4 \ud558\uc600\uc73c\uba70 \uc790\uc2e0\uc758 \uc219\ubd80\uae4c\uc9c0 \uc0b4\ud574\ud588\uc73c\ub098, \uc138\uc870\ub294 \uac00\ubccd\uac8c \ucc98\ubc8c\ud558\uc600\uace0, \uadf8\uac00 \uc815\ub09c\uacf5\uc2e0\uacfc \uc88c\uc775\uacf5\uc2e0\uc73c\ub85c \uc138\uc870\ubc18\uc815\uc758 \uc6d0\ud6c8(\u5143\u52f3)\uc778 \uc774\uc720\ub85c\uc368 \ub2e8\uc9c0 \ucc45\ub9dd\ub9cc \ud558\uace0 \ucc98\ubc8c\uc740 \uac00\ud558\uc9c0 \uc54a\uc558\ub2e4. \uc2dc\ud638(\u8ae1\u865f)\ub294 \uc704\ud3c9(\u5a01\u5e73)\uc774\ub2e4. \uadf8\uc758 \uc720\ud574\ub294 \ucda9\uccad\ub0a8\ub3c4 \ubd80\uc5ec\uad70 \uc740\uc0b0\uba74 \uacbd\ub454\ub9ac\uc5d0 \uc788\ub2e4.", "answer": "\uc704\ud3c9", "sentence": "\uc2dc\ud638(\u8ae1\u865f)\ub294 \uc704\ud3c9 (\u5a01\u5e73)\uc774\ub2e4.", "paragraph_sentence": "1455\ub144 \uc138\uc870 \ubc18\uc815 \uc9c1\ud6c4 \uc88c\uc775\uacf5\uc2e0 3\ub4f1\uc774 \ub418\uace0 \uc608\uc870\ud310\uc11c \ub4f1\uc744 \uac70\uccd0 \ubaa8\ub9b0\uc704 \uc5ec\uc9c4\uc871\uc758 \ub09c\uacfc \uac74\uc8fc\uc704 \uc5ec\uc9c4\uc871\uc758 \ub09c\uc744 \ud1a0\ubc8c\ud558\uace0 \uc6b0\uc758\uc815\uc744 \uc9c0\ub0c8\ub2e4. \uc608\uc885 \uc989\uc704 \ud6c4\uc5d0\ub294 \uc88c\uc758\uc815\uacfc \uc601\uc758\uc815\uc744 \uc9c0\ub0c8\ub2e4. \uc608\uc885 \uc0ac\ud6c4\uc5d0\ub294 \uc131\uc885\uc744 \uc9c0\uc9c0\ud55c \uacf5\ub85c\ub85c \ubd80\uc6d0\uad70\uc5d0 \uc9c4\ubd09\ub418\uace0 \uc88c\ub9ac\uacf5\uc2e0 1\ub4f1\uc774 \ub418\uc5c8\ub2e4. \uc131\uc9c8\uc774 \uc0ac\ub098\uc6cc \uad8c\uc138\ub97c \uc5bb\uc740 \ub4a4\uc5d0\ub294 \uc8fc\ubcc0\uc744 \ub3cc\uc544\ubcf4\uc9c0 \uc54a\uace0 \uc624\ub85c\uc9c0 \uae30\uc138(\u6c23\u52e2)\ub85c\uc368 \ub2e4\ub978 \uc0ac\ub78c\uc744 \ub2a5\uba78\ud558\uace0, \uac00\ub178(\u5bb6\u5974)\ub97c \ub193\uc544 \uc0ac\ub78c\uc744 \uc8fd\uc774\uae30\ub3c4 \ud558\uc600\uc73c\uba70 \uc790\uc2e0\uc758 \uc219\ubd80\uae4c\uc9c0 \uc0b4\ud574\ud588\uc73c\ub098, \uc138\uc870\ub294 \uac00\ubccd\uac8c \ucc98\ubc8c\ud558\uc600\uace0, \uadf8\uac00 \uc815\ub09c\uacf5\uc2e0\uacfc \uc88c\uc775\uacf5\uc2e0\uc73c\ub85c \uc138\uc870\ubc18\uc815\uc758 \uc6d0\ud6c8(\u5143\u52f3)\uc778 \uc774\uc720\ub85c\uc368 \ub2e8\uc9c0 \ucc45\ub9dd\ub9cc \ud558\uace0 \ucc98\ubc8c\uc740 \uac00\ud558\uc9c0 \uc54a\uc558\ub2e4. \uc2dc\ud638(\u8ae1\u865f)\ub294 \uc704\ud3c9 (\u5a01\u5e73)\uc774\ub2e4. \uadf8\uc758 \uc720\ud574\ub294 \ucda9\uccad\ub0a8\ub3c4 \ubd80\uc5ec\uad70 \uc740\uc0b0\uba74 \uacbd\ub454\ub9ac\uc5d0 \uc788\ub2e4.", "paragraph_answer": "1455\ub144 \uc138\uc870 \ubc18\uc815 \uc9c1\ud6c4 \uc88c\uc775\uacf5\uc2e0 3\ub4f1\uc774 \ub418\uace0 \uc608\uc870\ud310\uc11c \ub4f1\uc744 \uac70\uccd0 \ubaa8\ub9b0\uc704 \uc5ec\uc9c4\uc871\uc758 \ub09c\uacfc \uac74\uc8fc\uc704 \uc5ec\uc9c4\uc871\uc758 \ub09c\uc744 \ud1a0\ubc8c\ud558\uace0 \uc6b0\uc758\uc815\uc744 \uc9c0\ub0c8\ub2e4. \uc608\uc885 \uc989\uc704 \ud6c4\uc5d0\ub294 \uc88c\uc758\uc815\uacfc \uc601\uc758\uc815\uc744 \uc9c0\ub0c8\ub2e4. \uc608\uc885 \uc0ac\ud6c4\uc5d0\ub294 \uc131\uc885\uc744 \uc9c0\uc9c0\ud55c \uacf5\ub85c\ub85c \ubd80\uc6d0\uad70\uc5d0 \uc9c4\ubd09\ub418\uace0 \uc88c\ub9ac\uacf5\uc2e0 1\ub4f1\uc774 \ub418\uc5c8\ub2e4. \uc131\uc9c8\uc774 \uc0ac\ub098\uc6cc \uad8c\uc138\ub97c \uc5bb\uc740 \ub4a4\uc5d0\ub294 \uc8fc\ubcc0\uc744 \ub3cc\uc544\ubcf4\uc9c0 \uc54a\uace0 \uc624\ub85c\uc9c0 \uae30\uc138(\u6c23\u52e2)\ub85c\uc368 \ub2e4\ub978 \uc0ac\ub78c\uc744 \ub2a5\uba78\ud558\uace0, \uac00\ub178(\u5bb6\u5974)\ub97c \ub193\uc544 \uc0ac\ub78c\uc744 \uc8fd\uc774\uae30\ub3c4 \ud558\uc600\uc73c\uba70 \uc790\uc2e0\uc758 \uc219\ubd80\uae4c\uc9c0 \uc0b4\ud574\ud588\uc73c\ub098, \uc138\uc870\ub294 \uac00\ubccd\uac8c \ucc98\ubc8c\ud558\uc600\uace0, \uadf8\uac00 \uc815\ub09c\uacf5\uc2e0\uacfc \uc88c\uc775\uacf5\uc2e0\uc73c\ub85c \uc138\uc870\ubc18\uc815\uc758 \uc6d0\ud6c8(\u5143\u52f3)\uc778 \uc774\uc720\ub85c\uc368 \ub2e8\uc9c0 \ucc45\ub9dd\ub9cc \ud558\uace0 \ucc98\ubc8c\uc740 \uac00\ud558\uc9c0 \uc54a\uc558\ub2e4. \uc2dc\ud638(\u8ae1\u865f)\ub294 \uc704\ud3c9 (\u5a01\u5e73)\uc774\ub2e4. \uadf8\uc758 \uc720\ud574\ub294 \ucda9\uccad\ub0a8\ub3c4 \ubd80\uc5ec\uad70 \uc740\uc0b0\uba74 \uacbd\ub454\ub9ac\uc5d0 \uc788\ub2e4.", "sentence_answer": "\uc2dc\ud638(\u8ae1\u865f)\ub294 \uc704\ud3c9 (\u5a01\u5e73)\uc774\ub2e4.", "paragraph_id": "6572059-0-2", "paragraph_question": "question: \uc2dc\ud638\ub294 \ubb34\uc5c7\uc778\uac00?, context: 1455\ub144 \uc138\uc870 \ubc18\uc815 \uc9c1\ud6c4 \uc88c\uc775\uacf5\uc2e0 3\ub4f1\uc774 \ub418\uace0 \uc608\uc870\ud310\uc11c \ub4f1\uc744 \uac70\uccd0 \ubaa8\ub9b0\uc704 \uc5ec\uc9c4\uc871\uc758 \ub09c\uacfc \uac74\uc8fc\uc704 \uc5ec\uc9c4\uc871\uc758 \ub09c\uc744 \ud1a0\ubc8c\ud558\uace0 \uc6b0\uc758\uc815\uc744 \uc9c0\ub0c8\ub2e4. \uc608\uc885 \uc989\uc704 \ud6c4\uc5d0\ub294 \uc88c\uc758\uc815\uacfc \uc601\uc758\uc815\uc744 \uc9c0\ub0c8\ub2e4. \uc608\uc885 \uc0ac\ud6c4\uc5d0\ub294 \uc131\uc885\uc744 \uc9c0\uc9c0\ud55c \uacf5\ub85c\ub85c \ubd80\uc6d0\uad70\uc5d0 \uc9c4\ubd09\ub418\uace0 \uc88c\ub9ac\uacf5\uc2e0 1\ub4f1\uc774 \ub418\uc5c8\ub2e4. \uc131\uc9c8\uc774 \uc0ac\ub098\uc6cc \uad8c\uc138\ub97c \uc5bb\uc740 \ub4a4\uc5d0\ub294 \uc8fc\ubcc0\uc744 \ub3cc\uc544\ubcf4\uc9c0 \uc54a\uace0 \uc624\ub85c\uc9c0 \uae30\uc138(\u6c23\u52e2)\ub85c\uc368 \ub2e4\ub978 \uc0ac\ub78c\uc744 \ub2a5\uba78\ud558\uace0, \uac00\ub178(\u5bb6\u5974)\ub97c \ub193\uc544 \uc0ac\ub78c\uc744 \uc8fd\uc774\uae30\ub3c4 \ud558\uc600\uc73c\uba70 \uc790\uc2e0\uc758 \uc219\ubd80\uae4c\uc9c0 \uc0b4\ud574\ud588\uc73c\ub098, \uc138\uc870\ub294 \uac00\ubccd\uac8c \ucc98\ubc8c\ud558\uc600\uace0, \uadf8\uac00 \uc815\ub09c\uacf5\uc2e0\uacfc \uc88c\uc775\uacf5\uc2e0\uc73c\ub85c \uc138\uc870\ubc18\uc815\uc758 \uc6d0\ud6c8(\u5143\u52f3)\uc778 \uc774\uc720\ub85c\uc368 \ub2e8\uc9c0 \ucc45\ub9dd\ub9cc \ud558\uace0 \ucc98\ubc8c\uc740 \uac00\ud558\uc9c0 \uc54a\uc558\ub2e4. \uc2dc\ud638(\u8ae1\u865f)\ub294 \uc704\ud3c9(\u5a01\u5e73)\uc774\ub2e4. \uadf8\uc758 \uc720\ud574\ub294 \ucda9\uccad\ub0a8\ub3c4 \ubd80\uc5ec\uad70 \uc740\uc0b0\uba74 \uacbd\ub454\ub9ac\uc5d0 \uc788\ub2e4."} {"question": "\uc0dd\uba85\uc774 \uc0b4 \uc218 \uc788\ub294 \uadfc\uac70\ub9ac \ud56d\uc131\uacc4 \ubaa9\ub85d\uc744 \ub9cc\ub4e0 \uc0ac\ub78c \ub450\uba85\uc758 \uc774\ub984\uc740?", "paragraph": "\uadf8\ub7ec\ub098 \uc774\ub7ec\ud55c \ube44\uad00\uc801\uc778 \uacb0\uacfc\uc5d0\ub3c4 \uace0\ub798\uc790\ub9ac \ud0c0\uc6b0\uc5d0 \ub300\ud55c \uc5f0\uad6c\uc758 \uc5f4\uae30\ub294 \uc2dd\uc9c0 \uc54a\uc558\ub2e4. 2002\ub144 \ub9c8\uac00\ub81b \ud134\ubd88\uacfc \uc9c8 \ud0c0\ud130\ub294 \u2018\uc0dd\uba85\uc774 \uc0b4 \uc218 \uc788\ub294 \uadfc\uac70\ub9ac \ud56d\uc131\uacc4 \ubaa9\ub85d\u2019(HabCat)\uc744 \ub9cc\ub4e4\uc5c8\ub2e4. \uc774 \ubaa9\ub85d\uc5d0\ub294 1\ub9cc 7\ucc9c \uac1c \uc774\uc0c1\uc758 \uc774\ub860\uc801\uc778 \uc0dd\uba85\uccb4 \uac70\uc8fc \uac00\ub2a5 \ud589\uc131\uacc4\uac00 \uc2e4\ub824 \uc788\ub2e4. 2003\ub144 \ud134\ubd88\uc740 \ud0dc\uc591 \uc8fc\ubcc0 1\ucc9c \uad11\ub144 \uc774\ub0b4 5,000\uac1c\uc758 \ud6c4\ubcf4\ub4e4 \uc911 \uac00\uc7a5 \uc720\ub825\ud55c 30\uac1c \ud56d\uc131\uc744 \ucd94\ub824\ub0c8\uc73c\uba70 \uc5ec\uae30\uc5d0 \uace0\ub798\uc790\ub9ac \ud0c0\uc6b0\uac00 \ud3ec\ud568\ub418\uc5b4 \uc788\ub2e4. \ud134\ubd88\uc740 \uace0\ub798\uc790\ub9ac \ud0c0\uc6b0\ub97c \uc9c0\uad6c\ud615 \ud589\uc131 \ud0d0\uc0ac\uae30\ub97c \ud1b5\ud55c \uc5f0\uad6c\uc5d0 \uc801\ud569\ud55c \ub2e4\uc12f \ubcc4 \uc911 \ud558\ub098\ub85c \uaf3d\uc558\uc73c\uba70, \u201c\uc774\ub4e4 \ub2e4\uc12f \ubcc4\uc740 \ub9cc\uc57d \uc2e0\uc774 \uc6b0\ub9ac \ud589\uc131\uc744 \ub2e4\ub978 \ubcc4 \uc8fc\uc704\uc5d0 \uac00\uc838\ub2e4 \ub193\ub294\ub2e4\uba74 \uac00\uc7a5 \uc0b4\uace0 \uc2f6\uc740 \uacf3\ub4e4\uc774\ub2e4.\u201d\ub77c\uace0 \ub9d0\ud558\uae30\ub3c4 \ud588\ub2e4. \uadf8\ub7ec\ub098 NASA\uc758 \uc9c0\uad6c\ud615 \ud589\uc131 \ud0d0\uc0ac\uae30 \ubbf8\uc158\uc740 \ubbf8 \uc758\ud68c\uac00 \uc608\uc0b0\uc744 \uc0ad\uac10\ud558\uc5ec \ubb34\uae30\ud55c \uc5f0\uae30\ub418\uc5c8\uc73c\uba70, \ud604\uc7ac\ub294 \uc720\ub7fd \uc6b0\uc8fc\uad6d\uc758 \ub2e4\uc708 \uacc4\ud68d\uc774 \uace0\ub798\uc790\ub9ac \ud0c0\uc6b0\ub97c \ud0d0\uc0ac \ub300\uc0c1\uc73c\ub85c \uc9c0\ubaa9\ud558\uace0 \uc788\ub2e4.", "answer": "\ub9c8\uac00\ub81b \ud134\ubd88\uacfc \uc9c8 \ud0c0\ud130", "sentence": "2002\ub144 \ub9c8\uac00\ub81b \ud134\ubd88\uacfc \uc9c8 \ud0c0\ud130 \ub294 \u2018\uc0dd\uba85\uc774 \uc0b4 \uc218 \uc788\ub294 \uadfc\uac70\ub9ac \ud56d\uc131\uacc4 \ubaa9\ub85d\u2019(HabCat)\uc744 \ub9cc\ub4e4\uc5c8\ub2e4.", "paragraph_sentence": "\uadf8\ub7ec\ub098 \uc774\ub7ec\ud55c \ube44\uad00\uc801\uc778 \uacb0\uacfc\uc5d0\ub3c4 \uace0\ub798\uc790\ub9ac \ud0c0\uc6b0\uc5d0 \ub300\ud55c \uc5f0\uad6c\uc758 \uc5f4\uae30\ub294 \uc2dd\uc9c0 \uc54a\uc558\ub2e4. 2002\ub144 \ub9c8\uac00\ub81b \ud134\ubd88\uacfc \uc9c8 \ud0c0\ud130 \ub294 \u2018\uc0dd\uba85\uc774 \uc0b4 \uc218 \uc788\ub294 \uadfc\uac70\ub9ac \ud56d\uc131\uacc4 \ubaa9\ub85d\u2019(HabCat)\uc744 \ub9cc\ub4e4\uc5c8\ub2e4. \uc774 \ubaa9\ub85d\uc5d0\ub294 1\ub9cc 7\ucc9c \uac1c \uc774\uc0c1\uc758 \uc774\ub860\uc801\uc778 \uc0dd\uba85\uccb4 \uac70\uc8fc \uac00\ub2a5 \ud589\uc131\uacc4\uac00 \uc2e4\ub824 \uc788\ub2e4. 2003\ub144 \ud134\ubd88\uc740 \ud0dc\uc591 \uc8fc\ubcc0 1\ucc9c \uad11\ub144 \uc774\ub0b4 5,000\uac1c\uc758 \ud6c4\ubcf4\ub4e4 \uc911 \uac00\uc7a5 \uc720\ub825\ud55c 30\uac1c \ud56d\uc131\uc744 \ucd94\ub824\ub0c8\uc73c\uba70 \uc5ec\uae30\uc5d0 \uace0\ub798\uc790\ub9ac \ud0c0\uc6b0\uac00 \ud3ec\ud568\ub418\uc5b4 \uc788\ub2e4. \ud134\ubd88\uc740 \uace0\ub798\uc790\ub9ac \ud0c0\uc6b0\ub97c \uc9c0\uad6c\ud615 \ud589\uc131 \ud0d0\uc0ac\uae30\ub97c \ud1b5\ud55c \uc5f0\uad6c\uc5d0 \uc801\ud569\ud55c \ub2e4\uc12f \ubcc4 \uc911 \ud558\ub098\ub85c \uaf3d\uc558\uc73c\uba70, \u201c\uc774\ub4e4 \ub2e4\uc12f \ubcc4\uc740 \ub9cc\uc57d \uc2e0\uc774 \uc6b0\ub9ac \ud589\uc131\uc744 \ub2e4\ub978 \ubcc4 \uc8fc\uc704\uc5d0 \uac00\uc838\ub2e4 \ub193\ub294\ub2e4\uba74 \uac00\uc7a5 \uc0b4\uace0 \uc2f6\uc740 \uacf3\ub4e4\uc774\ub2e4. \u201d\ub77c\uace0 \ub9d0\ud558\uae30\ub3c4 \ud588\ub2e4. \uadf8\ub7ec\ub098 NASA\uc758 \uc9c0\uad6c\ud615 \ud589\uc131 \ud0d0\uc0ac\uae30 \ubbf8\uc158\uc740 \ubbf8 \uc758\ud68c\uac00 \uc608\uc0b0\uc744 \uc0ad\uac10\ud558\uc5ec \ubb34\uae30\ud55c \uc5f0\uae30\ub418\uc5c8\uc73c\uba70, \ud604\uc7ac\ub294 \uc720\ub7fd \uc6b0\uc8fc\uad6d\uc758 \ub2e4\uc708 \uacc4\ud68d\uc774 \uace0\ub798\uc790\ub9ac \ud0c0\uc6b0\ub97c \ud0d0\uc0ac \ub300\uc0c1\uc73c\ub85c \uc9c0\ubaa9\ud558\uace0 \uc788\ub2e4.", "paragraph_answer": "\uadf8\ub7ec\ub098 \uc774\ub7ec\ud55c \ube44\uad00\uc801\uc778 \uacb0\uacfc\uc5d0\ub3c4 \uace0\ub798\uc790\ub9ac \ud0c0\uc6b0\uc5d0 \ub300\ud55c \uc5f0\uad6c\uc758 \uc5f4\uae30\ub294 \uc2dd\uc9c0 \uc54a\uc558\ub2e4. 2002\ub144 \ub9c8\uac00\ub81b \ud134\ubd88\uacfc \uc9c8 \ud0c0\ud130 \ub294 \u2018\uc0dd\uba85\uc774 \uc0b4 \uc218 \uc788\ub294 \uadfc\uac70\ub9ac \ud56d\uc131\uacc4 \ubaa9\ub85d\u2019(HabCat)\uc744 \ub9cc\ub4e4\uc5c8\ub2e4. \uc774 \ubaa9\ub85d\uc5d0\ub294 1\ub9cc 7\ucc9c \uac1c \uc774\uc0c1\uc758 \uc774\ub860\uc801\uc778 \uc0dd\uba85\uccb4 \uac70\uc8fc \uac00\ub2a5 \ud589\uc131\uacc4\uac00 \uc2e4\ub824 \uc788\ub2e4. 2003\ub144 \ud134\ubd88\uc740 \ud0dc\uc591 \uc8fc\ubcc0 1\ucc9c \uad11\ub144 \uc774\ub0b4 5,000\uac1c\uc758 \ud6c4\ubcf4\ub4e4 \uc911 \uac00\uc7a5 \uc720\ub825\ud55c 30\uac1c \ud56d\uc131\uc744 \ucd94\ub824\ub0c8\uc73c\uba70 \uc5ec\uae30\uc5d0 \uace0\ub798\uc790\ub9ac \ud0c0\uc6b0\uac00 \ud3ec\ud568\ub418\uc5b4 \uc788\ub2e4. \ud134\ubd88\uc740 \uace0\ub798\uc790\ub9ac \ud0c0\uc6b0\ub97c \uc9c0\uad6c\ud615 \ud589\uc131 \ud0d0\uc0ac\uae30\ub97c \ud1b5\ud55c \uc5f0\uad6c\uc5d0 \uc801\ud569\ud55c \ub2e4\uc12f \ubcc4 \uc911 \ud558\ub098\ub85c \uaf3d\uc558\uc73c\uba70, \u201c\uc774\ub4e4 \ub2e4\uc12f \ubcc4\uc740 \ub9cc\uc57d \uc2e0\uc774 \uc6b0\ub9ac \ud589\uc131\uc744 \ub2e4\ub978 \ubcc4 \uc8fc\uc704\uc5d0 \uac00\uc838\ub2e4 \ub193\ub294\ub2e4\uba74 \uac00\uc7a5 \uc0b4\uace0 \uc2f6\uc740 \uacf3\ub4e4\uc774\ub2e4.\u201d\ub77c\uace0 \ub9d0\ud558\uae30\ub3c4 \ud588\ub2e4. \uadf8\ub7ec\ub098 NASA\uc758 \uc9c0\uad6c\ud615 \ud589\uc131 \ud0d0\uc0ac\uae30 \ubbf8\uc158\uc740 \ubbf8 \uc758\ud68c\uac00 \uc608\uc0b0\uc744 \uc0ad\uac10\ud558\uc5ec \ubb34\uae30\ud55c \uc5f0\uae30\ub418\uc5c8\uc73c\uba70, \ud604\uc7ac\ub294 \uc720\ub7fd \uc6b0\uc8fc\uad6d\uc758 \ub2e4\uc708 \uacc4\ud68d\uc774 \uace0\ub798\uc790\ub9ac \ud0c0\uc6b0\ub97c \ud0d0\uc0ac \ub300\uc0c1\uc73c\ub85c \uc9c0\ubaa9\ud558\uace0 \uc788\ub2e4.", "sentence_answer": "2002\ub144 \ub9c8\uac00\ub81b \ud134\ubd88\uacfc \uc9c8 \ud0c0\ud130 \ub294 \u2018\uc0dd\uba85\uc774 \uc0b4 \uc218 \uc788\ub294 \uadfc\uac70\ub9ac \ud56d\uc131\uacc4 \ubaa9\ub85d\u2019(HabCat)\uc744 \ub9cc\ub4e4\uc5c8\ub2e4.", "paragraph_id": "6487118-3-0", "paragraph_question": "question: \uc0dd\uba85\uc774 \uc0b4 \uc218 \uc788\ub294 \uadfc\uac70\ub9ac \ud56d\uc131\uacc4 \ubaa9\ub85d\uc744 \ub9cc\ub4e0 \uc0ac\ub78c \ub450\uba85\uc758 \uc774\ub984\uc740?, context: \uadf8\ub7ec\ub098 \uc774\ub7ec\ud55c \ube44\uad00\uc801\uc778 \uacb0\uacfc\uc5d0\ub3c4 \uace0\ub798\uc790\ub9ac \ud0c0\uc6b0\uc5d0 \ub300\ud55c \uc5f0\uad6c\uc758 \uc5f4\uae30\ub294 \uc2dd\uc9c0 \uc54a\uc558\ub2e4. 2002\ub144 \ub9c8\uac00\ub81b \ud134\ubd88\uacfc \uc9c8 \ud0c0\ud130\ub294 \u2018\uc0dd\uba85\uc774 \uc0b4 \uc218 \uc788\ub294 \uadfc\uac70\ub9ac \ud56d\uc131\uacc4 \ubaa9\ub85d\u2019(HabCat)\uc744 \ub9cc\ub4e4\uc5c8\ub2e4. \uc774 \ubaa9\ub85d\uc5d0\ub294 1\ub9cc 7\ucc9c \uac1c \uc774\uc0c1\uc758 \uc774\ub860\uc801\uc778 \uc0dd\uba85\uccb4 \uac70\uc8fc \uac00\ub2a5 \ud589\uc131\uacc4\uac00 \uc2e4\ub824 \uc788\ub2e4. 2003\ub144 \ud134\ubd88\uc740 \ud0dc\uc591 \uc8fc\ubcc0 1\ucc9c \uad11\ub144 \uc774\ub0b4 5,000\uac1c\uc758 \ud6c4\ubcf4\ub4e4 \uc911 \uac00\uc7a5 \uc720\ub825\ud55c 30\uac1c \ud56d\uc131\uc744 \ucd94\ub824\ub0c8\uc73c\uba70 \uc5ec\uae30\uc5d0 \uace0\ub798\uc790\ub9ac \ud0c0\uc6b0\uac00 \ud3ec\ud568\ub418\uc5b4 \uc788\ub2e4. \ud134\ubd88\uc740 \uace0\ub798\uc790\ub9ac \ud0c0\uc6b0\ub97c \uc9c0\uad6c\ud615 \ud589\uc131 \ud0d0\uc0ac\uae30\ub97c \ud1b5\ud55c \uc5f0\uad6c\uc5d0 \uc801\ud569\ud55c \ub2e4\uc12f \ubcc4 \uc911 \ud558\ub098\ub85c \uaf3d\uc558\uc73c\uba70, \u201c\uc774\ub4e4 \ub2e4\uc12f \ubcc4\uc740 \ub9cc\uc57d \uc2e0\uc774 \uc6b0\ub9ac \ud589\uc131\uc744 \ub2e4\ub978 \ubcc4 \uc8fc\uc704\uc5d0 \uac00\uc838\ub2e4 \ub193\ub294\ub2e4\uba74 \uac00\uc7a5 \uc0b4\uace0 \uc2f6\uc740 \uacf3\ub4e4\uc774\ub2e4.\u201d\ub77c\uace0 \ub9d0\ud558\uae30\ub3c4 \ud588\ub2e4. \uadf8\ub7ec\ub098 NASA\uc758 \uc9c0\uad6c\ud615 \ud589\uc131 \ud0d0\uc0ac\uae30 \ubbf8\uc158\uc740 \ubbf8 \uc758\ud68c\uac00 \uc608\uc0b0\uc744 \uc0ad\uac10\ud558\uc5ec \ubb34\uae30\ud55c \uc5f0\uae30\ub418\uc5c8\uc73c\uba70, \ud604\uc7ac\ub294 \uc720\ub7fd \uc6b0\uc8fc\uad6d\uc758 \ub2e4\uc708 \uacc4\ud68d\uc774 \uace0\ub798\uc790\ub9ac \ud0c0\uc6b0\ub97c \ud0d0\uc0ac \ub300\uc0c1\uc73c\ub85c \uc9c0\ubaa9\ud558\uace0 \uc788\ub2e4."} {"question": "\ubc15\uc815\ud76c\ub294 1969\ub144 11\uc6d4\uc5d0 \uacf5\uc7a5 \uadfc\ub85c\uc790\ub4e4\uc744 \uc704\ud574 7\uac1c \ub300\uae30\uc5c5\uccb4\uc758 \ud68c\uc0ac \ub0b4\uc5d0 \uc5b4\ub5a4 \ubd80\uc124\uc744 \uc9c0\uc2dc\ud558\uc600\ub294\uac00?", "paragraph": "\uadf8\ub7f0\uac00 \ud558\uba74 64\ub144 1\uc6d4 4\uc77c \uc2dc\ub3c4 \ub2e8\uc704 \uad50\uc721\uc790\uce58\uc81c\ub97c \uc2e4\uc2dc\ud558\uc5ec \uc2dc\ub3c4 \uad50\uc721\uccad\uc5d0 \uad50\uc721\ud589\uc815\uad8c\uc744 \uc704\uc784\ud558\uae30\ub3c4 \ud588\ub2e4. 1968\ub144 7\uc6d4 15\uc77c 71\ub144\uae4c\uc9c0 \uc911\ud559\uc785\uc2dc\uc2dc\ud5d8\uc744 \ud3d0\uc9c0\ud558\ub294 \ub4f1 \uc785\uc2dc\uac1c\ud601\uc548\uc744 \ubc1c\ud45c\ud55c \ubc18\uba74 10\uc6d4 14\uc77c \ub300\ud559\uad50 \uc785\uc2dc \uc608\ube44\uace0\uc0ac\uc81c\ub97c 69\ub144\ubd80\ud130 \uc2e4\uc2dc\ud558\uac8c \ud558\uc600\ub2e4. \ub610\ud55c \uad6d\uacf5\ub9bd\uc911\ud559\uad50\uc99d\uc124 7\uac1c\ub144 \uacc4\ud68d\uacfc \uace0\ub4f1\ud559\uad50\uae30\uad00\ud655\ucda9\uacc4\ud68d\uc744 \ucd94\uc9c4\ud558\uc600\uace0 1969\ub144 11\uc6d4\uc5d0\ub294 \uacf5\uc7a5 \uadfc\ub85c\uc790\ub4e4\uc744 \uc704\ud55c \uae08\uc131\uc0ac \ub4f1 7\uac1c \ub300\uae30\uc5c5\uccb4\uc5d0 \ud68c\uc0ac \ub0b4\uc5d0 \uc774\uacf5\uacc4 \uc2e4\uc5c5\ud559\uad50 \ubd80\uc124\uc744 \uc9c0\uc2dc\ud558\uc600\ub2e4. 1976\ub144 5\uc6d4 20\uc77c\uc5d0\ub294 \uad6d\ube44 \uc7a5\ud559\uc0dd\uc744 \uc120\ubc1c\ud558\uc5ec \uc720\ud559\ubcf4\ub0b4\ub294 \uc81c\ub3c4\ub97c \uc2e0\uc124\ud558\uc600\uace0, 1976\ub144 7\uc6d4\uc5d0\ub294 \uc77c\ubc18 \uc601\uc138 \uae30\uc5c5\uccb4\ub4e4\uc774 \uc0b0\uc9d1\ud574 \uc788\ub294 \uacf5\ub2e8 \uadfc\ub85c\uc790\ub97c \uc704\ud574 \uc57c\uac04\uc911\ud559 \uac1c\uc124\uc744 \uc9c0\uc2dc\ud558\uc600\ub2e4.", "answer": "\uc774\uacf5\uacc4 \uc2e4\uc5c5\ud559\uad50", "sentence": "\uacf5\uc7a5 \uadfc\ub85c\uc790\ub4e4\uc744 \uc704\ud55c \uae08\uc131\uc0ac \ub4f1 7\uac1c \ub300\uae30\uc5c5\uccb4\uc5d0 \ud68c\uc0ac \ub0b4\uc5d0 \uc774\uacf5\uacc4 \uc2e4\uc5c5\ud559\uad50 \ubd80\uc124\uc744 \uc9c0\uc2dc\ud558\uc600\ub2e4.", "paragraph_sentence": "\uadf8\ub7f0\uac00 \ud558\uba74 64\ub144 1\uc6d4 4\uc77c \uc2dc\ub3c4 \ub2e8\uc704 \uad50\uc721\uc790\uce58\uc81c\ub97c \uc2e4\uc2dc\ud558\uc5ec \uc2dc\ub3c4 \uad50\uc721\uccad\uc5d0 \uad50\uc721\ud589\uc815\uad8c\uc744 \uc704\uc784\ud558\uae30\ub3c4 \ud588\ub2e4. 1968\ub144 7\uc6d4 15\uc77c 71\ub144\uae4c\uc9c0 \uc911\ud559\uc785\uc2dc\uc2dc\ud5d8\uc744 \ud3d0\uc9c0\ud558\ub294 \ub4f1 \uc785\uc2dc\uac1c\ud601\uc548\uc744 \ubc1c\ud45c\ud55c \ubc18\uba74 10\uc6d4 14\uc77c \ub300\ud559\uad50 \uc785\uc2dc \uc608\ube44\uace0\uc0ac\uc81c\ub97c 69\ub144\ubd80\ud130 \uc2e4\uc2dc\ud558\uac8c \ud558\uc600\ub2e4. \ub610\ud55c \uad6d\uacf5\ub9bd\uc911\ud559\uad50\uc99d\uc124 7\uac1c\ub144 \uacc4\ud68d\uacfc \uace0\ub4f1\ud559\uad50\uae30\uad00\ud655\ucda9\uacc4\ud68d\uc744 \ucd94\uc9c4\ud558\uc600\uace0 1969\ub144 11\uc6d4\uc5d0\ub294 \uacf5\uc7a5 \uadfc\ub85c\uc790\ub4e4\uc744 \uc704\ud55c \uae08\uc131\uc0ac \ub4f1 7\uac1c \ub300\uae30\uc5c5\uccb4\uc5d0 \ud68c\uc0ac \ub0b4\uc5d0 \uc774\uacf5\uacc4 \uc2e4\uc5c5\ud559\uad50 \ubd80\uc124\uc744 \uc9c0\uc2dc\ud558\uc600\ub2e4. 1976\ub144 5\uc6d4 20\uc77c\uc5d0\ub294 \uad6d\ube44 \uc7a5\ud559\uc0dd\uc744 \uc120\ubc1c\ud558\uc5ec \uc720\ud559\ubcf4\ub0b4\ub294 \uc81c\ub3c4\ub97c \uc2e0\uc124\ud558\uc600\uace0, 1976\ub144 7\uc6d4\uc5d0\ub294 \uc77c\ubc18 \uc601\uc138 \uae30\uc5c5\uccb4\ub4e4\uc774 \uc0b0\uc9d1\ud574 \uc788\ub294 \uacf5\ub2e8 \uadfc\ub85c\uc790\ub97c \uc704\ud574 \uc57c\uac04\uc911\ud559 \uac1c\uc124\uc744 \uc9c0\uc2dc\ud558\uc600\ub2e4.", "paragraph_answer": "\uadf8\ub7f0\uac00 \ud558\uba74 64\ub144 1\uc6d4 4\uc77c \uc2dc\ub3c4 \ub2e8\uc704 \uad50\uc721\uc790\uce58\uc81c\ub97c \uc2e4\uc2dc\ud558\uc5ec \uc2dc\ub3c4 \uad50\uc721\uccad\uc5d0 \uad50\uc721\ud589\uc815\uad8c\uc744 \uc704\uc784\ud558\uae30\ub3c4 \ud588\ub2e4. 1968\ub144 7\uc6d4 15\uc77c 71\ub144\uae4c\uc9c0 \uc911\ud559\uc785\uc2dc\uc2dc\ud5d8\uc744 \ud3d0\uc9c0\ud558\ub294 \ub4f1 \uc785\uc2dc\uac1c\ud601\uc548\uc744 \ubc1c\ud45c\ud55c \ubc18\uba74 10\uc6d4 14\uc77c \ub300\ud559\uad50 \uc785\uc2dc \uc608\ube44\uace0\uc0ac\uc81c\ub97c 69\ub144\ubd80\ud130 \uc2e4\uc2dc\ud558\uac8c \ud558\uc600\ub2e4. \ub610\ud55c \uad6d\uacf5\ub9bd\uc911\ud559\uad50\uc99d\uc124 7\uac1c\ub144 \uacc4\ud68d\uacfc \uace0\ub4f1\ud559\uad50\uae30\uad00\ud655\ucda9\uacc4\ud68d\uc744 \ucd94\uc9c4\ud558\uc600\uace0 1969\ub144 11\uc6d4\uc5d0\ub294 \uacf5\uc7a5 \uadfc\ub85c\uc790\ub4e4\uc744 \uc704\ud55c \uae08\uc131\uc0ac \ub4f1 7\uac1c \ub300\uae30\uc5c5\uccb4\uc5d0 \ud68c\uc0ac \ub0b4\uc5d0 \uc774\uacf5\uacc4 \uc2e4\uc5c5\ud559\uad50 \ubd80\uc124\uc744 \uc9c0\uc2dc\ud558\uc600\ub2e4. 1976\ub144 5\uc6d4 20\uc77c\uc5d0\ub294 \uad6d\ube44 \uc7a5\ud559\uc0dd\uc744 \uc120\ubc1c\ud558\uc5ec \uc720\ud559\ubcf4\ub0b4\ub294 \uc81c\ub3c4\ub97c \uc2e0\uc124\ud558\uc600\uace0, 1976\ub144 7\uc6d4\uc5d0\ub294 \uc77c\ubc18 \uc601\uc138 \uae30\uc5c5\uccb4\ub4e4\uc774 \uc0b0\uc9d1\ud574 \uc788\ub294 \uacf5\ub2e8 \uadfc\ub85c\uc790\ub97c \uc704\ud574 \uc57c\uac04\uc911\ud559 \uac1c\uc124\uc744 \uc9c0\uc2dc\ud558\uc600\ub2e4.", "sentence_answer": "\uacf5\uc7a5 \uadfc\ub85c\uc790\ub4e4\uc744 \uc704\ud55c \uae08\uc131\uc0ac \ub4f1 7\uac1c \ub300\uae30\uc5c5\uccb4\uc5d0 \ud68c\uc0ac \ub0b4\uc5d0 \uc774\uacf5\uacc4 \uc2e4\uc5c5\ud559\uad50 \ubd80\uc124\uc744 \uc9c0\uc2dc\ud558\uc600\ub2e4.", "paragraph_id": "6516370-35-1", "paragraph_question": "question: \ubc15\uc815\ud76c\ub294 1969\ub144 11\uc6d4\uc5d0 \uacf5\uc7a5 \uadfc\ub85c\uc790\ub4e4\uc744 \uc704\ud574 7\uac1c \ub300\uae30\uc5c5\uccb4\uc758 \ud68c\uc0ac \ub0b4\uc5d0 \uc5b4\ub5a4 \ubd80\uc124\uc744 \uc9c0\uc2dc\ud558\uc600\ub294\uac00?, context: \uadf8\ub7f0\uac00 \ud558\uba74 64\ub144 1\uc6d4 4\uc77c \uc2dc\ub3c4 \ub2e8\uc704 \uad50\uc721\uc790\uce58\uc81c\ub97c \uc2e4\uc2dc\ud558\uc5ec \uc2dc\ub3c4 \uad50\uc721\uccad\uc5d0 \uad50\uc721\ud589\uc815\uad8c\uc744 \uc704\uc784\ud558\uae30\ub3c4 \ud588\ub2e4. 1968\ub144 7\uc6d4 15\uc77c 71\ub144\uae4c\uc9c0 \uc911\ud559\uc785\uc2dc\uc2dc\ud5d8\uc744 \ud3d0\uc9c0\ud558\ub294 \ub4f1 \uc785\uc2dc\uac1c\ud601\uc548\uc744 \ubc1c\ud45c\ud55c \ubc18\uba74 10\uc6d4 14\uc77c \ub300\ud559\uad50 \uc785\uc2dc \uc608\ube44\uace0\uc0ac\uc81c\ub97c 69\ub144\ubd80\ud130 \uc2e4\uc2dc\ud558\uac8c \ud558\uc600\ub2e4. \ub610\ud55c \uad6d\uacf5\ub9bd\uc911\ud559\uad50\uc99d\uc124 7\uac1c\ub144 \uacc4\ud68d\uacfc \uace0\ub4f1\ud559\uad50\uae30\uad00\ud655\ucda9\uacc4\ud68d\uc744 \ucd94\uc9c4\ud558\uc600\uace0 1969\ub144 11\uc6d4\uc5d0\ub294 \uacf5\uc7a5 \uadfc\ub85c\uc790\ub4e4\uc744 \uc704\ud55c \uae08\uc131\uc0ac \ub4f1 7\uac1c \ub300\uae30\uc5c5\uccb4\uc5d0 \ud68c\uc0ac \ub0b4\uc5d0 \uc774\uacf5\uacc4 \uc2e4\uc5c5\ud559\uad50 \ubd80\uc124\uc744 \uc9c0\uc2dc\ud558\uc600\ub2e4. 1976\ub144 5\uc6d4 20\uc77c\uc5d0\ub294 \uad6d\ube44 \uc7a5\ud559\uc0dd\uc744 \uc120\ubc1c\ud558\uc5ec \uc720\ud559\ubcf4\ub0b4\ub294 \uc81c\ub3c4\ub97c \uc2e0\uc124\ud558\uc600\uace0, 1976\ub144 7\uc6d4\uc5d0\ub294 \uc77c\ubc18 \uc601\uc138 \uae30\uc5c5\uccb4\ub4e4\uc774 \uc0b0\uc9d1\ud574 \uc788\ub294 \uacf5\ub2e8 \uadfc\ub85c\uc790\ub97c \uc704\ud574 \uc57c\uac04\uc911\ud559 \uac1c\uc124\uc744 \uc9c0\uc2dc\ud558\uc600\ub2e4."} {"question": "ISO 8601\uc5d0\uc11c \uac04\uaca9 \ubc18\ubcf5\ud558\uae30\ub294 \uc5b4\ub5a4 \uc808\uc5d0\uc11c \uba85\uc2dc\ub418\uace0 \uc788\ub294\uac00?", "paragraph": "\uac04\uaca9 \ubc18\ubcf5\ud558\uae30\ub294 \"4.5 \uc2dc\uac04 \uac04\uaca9 \ubc18\ubcf5\" \uc808\uc5d0\uc11c \uba85\uc2dc\ub418\uace0 \uc788\ub2e4. \uc774\ub294 \uac04\uaca9 \ud45c\ud604\uc758 \uc2dc\uc791\uc5d0 \"R[n]/\"\uc744 \ucd94\uac00\ud558\uc5ec \ub9cc\ub4e4\uc5b4\uc9c8 \uc218 \uc788\ub294\ub370 , R\uc740 \uadf8 \uc790\uccb4\uac00 \uc758\ubbf8\ud558\ub294 \ubb38\uc790\ub85c\uc368 \uc0ac\uc6a9\ub418\uba70 [n]\uc740 \ubc18\ubcf5 \ud69f\uc218\ub85c \ub300\uccb4\ub41c\ub2e4. [n] \uac12\uc758 \uc0dd\ub7b5\uc740 \ubb34\ud55c \ubc18\ubcf5\uc744 \uc758\ubbf8\ud55c\ub2e4. \uac04\uaca9\uc774 (\ud615\uc2dd 1\ubc88\uacfc 2\ubc88\ucc98\ub7fc) \uc2dc\uc791\uc744 \uc9c0\uc815\ud558\uace0 \uc788\ub2e4\uba74, \uac04\uaca9 \ubc18\ubcf5\uc744 \uc704\ud55c \uc2dc\uc791\uc774\ub2e4. \uac04\uaca9\uc774 \uc2dc\uc791\uc774 \uc544\ub2cc \ub05d(3\ubc88 \ud615\uc2dd\ucc98\ub7fc)\uc744 \uc9c0\uc815\ud558\uace0 \uc788\ub2e4\uba74, \uac04\uaca9 \ubc18\ubcf5\uc758 \ub05d\uc744 \ub9d0\ud55c\ub2e4. \uc608\ub97c \ub4e4\uc5b4, \"2008-03-01T13:00:00Z\"\uc744 \uc2dc\uc791\uc73c\ub85c \"P1Y2M10DT2H30M\" \uac04\uaca9\uc744 5\ubc88 \ubc18\ubcf5\ud558\ub824\uba74, \"R5/2008-03-01T13:00:00Z/P1Y2M10DT2H30M\"\uc744 \uc0ac\uc6a9\ud55c\ub2e4.", "answer": "\"4.5 \uc2dc\uac04 \uac04\uaca9 \ubc18\ubcf5\"", "sentence": "\uac04\uaca9 \ubc18\ubcf5\ud558\uae30\ub294 \"4.5 \uc2dc\uac04 \uac04\uaca9 \ubc18\ubcf5\" \uc808\uc5d0\uc11c \uba85\uc2dc\ub418\uace0 \uc788\ub2e4.", "paragraph_sentence": " \uac04\uaca9 \ubc18\ubcf5\ud558\uae30\ub294 \"4.5 \uc2dc\uac04 \uac04\uaca9 \ubc18\ubcf5\" \uc808\uc5d0\uc11c \uba85\uc2dc\ub418\uace0 \uc788\ub2e4. \uc774\ub294 \uac04\uaca9 \ud45c\ud604\uc758 \uc2dc\uc791\uc5d0 \"R[n]/\"\uc744 \ucd94\uac00\ud558\uc5ec \ub9cc\ub4e4\uc5b4\uc9c8 \uc218 \uc788\ub294\ub370 , R\uc740 \uadf8 \uc790\uccb4\uac00 \uc758\ubbf8\ud558\ub294 \ubb38\uc790\ub85c\uc368 \uc0ac\uc6a9\ub418\uba70 [n]\uc740 \ubc18\ubcf5 \ud69f\uc218\ub85c \ub300\uccb4\ub41c\ub2e4. [n] \uac12\uc758 \uc0dd\ub7b5\uc740 \ubb34\ud55c \ubc18\ubcf5\uc744 \uc758\ubbf8\ud55c\ub2e4. \uac04\uaca9\uc774 (\ud615\uc2dd 1\ubc88\uacfc 2\ubc88\ucc98\ub7fc) \uc2dc\uc791\uc744 \uc9c0\uc815\ud558\uace0 \uc788\ub2e4\uba74, \uac04\uaca9 \ubc18\ubcf5\uc744 \uc704\ud55c \uc2dc\uc791\uc774\ub2e4. \uac04\uaca9\uc774 \uc2dc\uc791\uc774 \uc544\ub2cc \ub05d(3\ubc88 \ud615\uc2dd\ucc98\ub7fc)\uc744 \uc9c0\uc815\ud558\uace0 \uc788\ub2e4\uba74, \uac04\uaca9 \ubc18\ubcf5\uc758 \ub05d\uc744 \ub9d0\ud55c\ub2e4. \uc608\ub97c \ub4e4\uc5b4, \"2008-03-01T13:00:00Z\"\uc744 \uc2dc\uc791\uc73c\ub85c \"P1Y2M10DT2H30M\" \uac04\uaca9\uc744 5\ubc88 \ubc18\ubcf5\ud558\ub824\uba74, \"R5/2008-03-01T13:00:00Z/P1Y2M10DT2H30M\"\uc744 \uc0ac\uc6a9\ud55c\ub2e4.", "paragraph_answer": "\uac04\uaca9 \ubc18\ubcf5\ud558\uae30\ub294 \"4.5 \uc2dc\uac04 \uac04\uaca9 \ubc18\ubcf5\" \uc808\uc5d0\uc11c \uba85\uc2dc\ub418\uace0 \uc788\ub2e4. \uc774\ub294 \uac04\uaca9 \ud45c\ud604\uc758 \uc2dc\uc791\uc5d0 \"R[n]/\"\uc744 \ucd94\uac00\ud558\uc5ec \ub9cc\ub4e4\uc5b4\uc9c8 \uc218 \uc788\ub294\ub370 , R\uc740 \uadf8 \uc790\uccb4\uac00 \uc758\ubbf8\ud558\ub294 \ubb38\uc790\ub85c\uc368 \uc0ac\uc6a9\ub418\uba70 [n]\uc740 \ubc18\ubcf5 \ud69f\uc218\ub85c \ub300\uccb4\ub41c\ub2e4. [n] \uac12\uc758 \uc0dd\ub7b5\uc740 \ubb34\ud55c \ubc18\ubcf5\uc744 \uc758\ubbf8\ud55c\ub2e4. \uac04\uaca9\uc774 (\ud615\uc2dd 1\ubc88\uacfc 2\ubc88\ucc98\ub7fc) \uc2dc\uc791\uc744 \uc9c0\uc815\ud558\uace0 \uc788\ub2e4\uba74, \uac04\uaca9 \ubc18\ubcf5\uc744 \uc704\ud55c \uc2dc\uc791\uc774\ub2e4. \uac04\uaca9\uc774 \uc2dc\uc791\uc774 \uc544\ub2cc \ub05d(3\ubc88 \ud615\uc2dd\ucc98\ub7fc)\uc744 \uc9c0\uc815\ud558\uace0 \uc788\ub2e4\uba74, \uac04\uaca9 \ubc18\ubcf5\uc758 \ub05d\uc744 \ub9d0\ud55c\ub2e4. \uc608\ub97c \ub4e4\uc5b4, \"2008-03-01T13:00:00Z\"\uc744 \uc2dc\uc791\uc73c\ub85c \"P1Y2M10DT2H30M\" \uac04\uaca9\uc744 5\ubc88 \ubc18\ubcf5\ud558\ub824\uba74, \"R5/2008-03-01T13:00:00Z/P1Y2M10DT2H30M\"\uc744 \uc0ac\uc6a9\ud55c\ub2e4.", "sentence_answer": "\uac04\uaca9 \ubc18\ubcf5\ud558\uae30\ub294 \"4.5 \uc2dc\uac04 \uac04\uaca9 \ubc18\ubcf5\" \uc808\uc5d0\uc11c \uba85\uc2dc\ub418\uace0 \uc788\ub2e4.", "paragraph_id": "5782912-4-0", "paragraph_question": "question: ISO 8601\uc5d0\uc11c \uac04\uaca9 \ubc18\ubcf5\ud558\uae30\ub294 \uc5b4\ub5a4 \uc808\uc5d0\uc11c \uba85\uc2dc\ub418\uace0 \uc788\ub294\uac00?, context: \uac04\uaca9 \ubc18\ubcf5\ud558\uae30\ub294 \"4.5 \uc2dc\uac04 \uac04\uaca9 \ubc18\ubcf5\" \uc808\uc5d0\uc11c \uba85\uc2dc\ub418\uace0 \uc788\ub2e4. \uc774\ub294 \uac04\uaca9 \ud45c\ud604\uc758 \uc2dc\uc791\uc5d0 \"R[n]/\"\uc744 \ucd94\uac00\ud558\uc5ec \ub9cc\ub4e4\uc5b4\uc9c8 \uc218 \uc788\ub294\ub370 , R\uc740 \uadf8 \uc790\uccb4\uac00 \uc758\ubbf8\ud558\ub294 \ubb38\uc790\ub85c\uc368 \uc0ac\uc6a9\ub418\uba70 [n]\uc740 \ubc18\ubcf5 \ud69f\uc218\ub85c \ub300\uccb4\ub41c\ub2e4. [n] \uac12\uc758 \uc0dd\ub7b5\uc740 \ubb34\ud55c \ubc18\ubcf5\uc744 \uc758\ubbf8\ud55c\ub2e4. \uac04\uaca9\uc774 (\ud615\uc2dd 1\ubc88\uacfc 2\ubc88\ucc98\ub7fc) \uc2dc\uc791\uc744 \uc9c0\uc815\ud558\uace0 \uc788\ub2e4\uba74, \uac04\uaca9 \ubc18\ubcf5\uc744 \uc704\ud55c \uc2dc\uc791\uc774\ub2e4. \uac04\uaca9\uc774 \uc2dc\uc791\uc774 \uc544\ub2cc \ub05d(3\ubc88 \ud615\uc2dd\ucc98\ub7fc)\uc744 \uc9c0\uc815\ud558\uace0 \uc788\ub2e4\uba74, \uac04\uaca9 \ubc18\ubcf5\uc758 \ub05d\uc744 \ub9d0\ud55c\ub2e4. \uc608\ub97c \ub4e4\uc5b4, \"2008-03-01T13:00:00Z\"\uc744 \uc2dc\uc791\uc73c\ub85c \"P1Y2M10DT2H30M\" \uac04\uaca9\uc744 5\ubc88 \ubc18\ubcf5\ud558\ub824\uba74, \"R5/2008-03-01T13:00:00Z/P1Y2M10DT2H30M\"\uc744 \uc0ac\uc6a9\ud55c\ub2e4."} {"question": "\uc624\uc2a4\ud2b8\ub9ac\uc544-\uc2e0\uc131\ub85c\ub9c8\uc81c\uad6d\uad70\uc774 \uc783\uc740 \ubcd1\ub825\uc758 \uc218\ub294?", "paragraph": "\uacb0\ucf54 \ubb34\uc775\ud558\uac8c \uac00\ub9cc\ud788 \uc788\ub294 \uc0ac\ub78c\uc774 \uc544\ub2cc \ub300(\u5927) \uc288\ud130\ud558\uc784\uc740 \uc81c\uad6d\uad70\uc774 \ub0a8\ucabd\uc73c\ub85c \uc774\ub3d9\ud558\ub294 \uac83\uc744 \ubc1c\uacac\ud558\uc600\uace0, \uc790\uc2e0\uc774 \uc9c0\ud718\ud558\ub294 \ubd80\ub300\uc758 \uc55e\uc5d0 \uc788\ub294 \ubc29\uc5b4\ubd80\ub300\uac00 \uc774 \uac19\uc740 \uc6c0\uc9c1\uc784\uc73c\ub85c \uc778\ud558\uc5ec \uc57d\ud654\ub418\uc5c8\ub2e4\ub294 \uac83\uc744 \uae68\ub2ec\uc558\ub2e4. \uadf8\ub294 \ud718\ud558\uc758 \uad70\ub300\uc5d0\uac8c \uc989\uac01\uc801\uc73c\ub85c \uacf5\uaca9\uc744 \uac1c\uc2dc\ud558\ub3c4\ub85d \uba85\ub839\uc744 \ub0b4\ub838\uace0, \uc624\uc2a4\ud2b8\ub9ac\uc544 \ubcf4\ubcd1\ub300\ub294 \ubd95\uad34\ub418\uc5c8\ub2e4. \ud504\ub85c\uc774\uc13c\uc758 \ubca8\ub9c1 \ud6c4\uc0ac\ub974(The Prussian Belling Hussars)\uc640 KR4\ub294 \ubd95\uad34\ub41c \uc624\uc2a4\ud2b8\ub9ac\uc544\uad70\uc744 \ub3cc\ud30c\ud558\ub294 \ub3cc\uaca9\uc744 \uac10\ud589\ud558\uc600\uace0, \uc774\ub85c \uc778\ud574 \uc624\uc2a4\ud2b8\ub9ac\uc544\uad70\uc774 \ubd84\uc0b0\ub428\uc5d0 \ub530\ub77c \ud504\ub85c\uc774\uc13c\uad70\uc740 \uc2b9\uae30\ub97c \uc7a5\uc545\ud560 \uc218 \uc788\uc5c8\ub2e4. \uc2a4\ud1a8\ubca0\ub974\ud06c\ub294 \ub3d9\ub9f9\uad70\uc774 \ud1f4\uac01\ud568\uc5d0 \ub530\ub77c \uc790\uc2e0\uc774 \uc9c0\ud718\ud558\ub294 \ubd80\ub300\uc758 \uce21\uba74\uc774 \uc801\uc758 \uacf5\uaca9\uc5d0 \ub178\ucd9c\ub41c \uac83\uc744 \uae68\ub2eb\uace0 \uc989\uac01\uc801\uc73c\ub85c \ud1f4\uac01\ud560 \uac83\uc744 \uba85\ud588\ub2e4. \uc81c\uad6d\uad70\uc774 \uc790\uc2e0\uc758 \uc704\uce58\uc5d0\uc11c \ubb3c\ub7ec\ub0a8\uc5d0 \ub530\ub77c \uce84\ud53c\ud154\ub9ac\uc758 \ub098\uba38\uc9c0 \ubd80\ub300\ub3c4 \ub4a4\ub530\ub77c \ud1f4\uac01\uc744 \uc2dc\uc791\ud588\ub2e4. \ubbf8\ub9ac \uc608\uace0\ub418\uc5c8\ub4ef \uba54\uc774\uc5d0\ub974\uc758 \ubd80\ub300\ub294 \uc794\uc5ec\ubd80\ub300\ub97c \ub3d5\uae30 \uc704\ud574 \uc790\uc2e0\ub4e4\uc758 \uc704\uce58\uc5d0\uc11c \ubb3c\ub7ec\ub098\uc9c0 \uc54a\uc558\ub2e4. \uba54\uc774\uc5d0\ub974\ub294 \ud6c4\uc5d0 \uc790\uc2e0\uc774 \ub9e1\uc740 \uc704\uce58\ub97c \uc0ac\uc218\ud558\uae30 \uc704\ud558\uc5ec \ucd5c\ud6c4\uae4c\uc9c0 \uc2f8\uc6b0\ub77c\ub294 \uba85\ub839\uc744 \ub0b4\ub838\ub2e4\uace0 \uc8fc\uc7a5\ud558\uc600\ub2e4. \ud504\ub85c\uc774\uc13c\uad70\uc740 1,400\uba85\uc744 \uc783\uc5c8\uace0, \uc624\uc2a4\ud2b8\ub9ac\uc544-\uc2e0\uc131\ub85c\ub9c8\uc81c\uad6d\uad70\uc740 7,000\uba85\uc758 \ubcd1\ub825\uacfc 9\uac1c\uc758 \uad70\uae30, 28\ubb38\uc758 \ud3ec\ub97c \uc0c1\uc2e4\ud588\ub2e4.", "answer": "7,000\uba85", "sentence": "\ud504\ub85c\uc774\uc13c\uad70\uc740 1,400\uba85\uc744 \uc783\uc5c8\uace0, \uc624\uc2a4\ud2b8\ub9ac\uc544-\uc2e0\uc131\ub85c\ub9c8\uc81c\uad6d\uad70\uc740 7,000\uba85 \uc758 \ubcd1\ub825\uacfc 9\uac1c\uc758 \uad70\uae30, 28\ubb38\uc758 \ud3ec\ub97c \uc0c1\uc2e4\ud588\ub2e4.", "paragraph_sentence": "\uacb0\ucf54 \ubb34\uc775\ud558\uac8c \uac00\ub9cc\ud788 \uc788\ub294 \uc0ac\ub78c\uc774 \uc544\ub2cc \ub300(\u5927) \uc288\ud130\ud558\uc784\uc740 \uc81c\uad6d\uad70\uc774 \ub0a8\ucabd\uc73c\ub85c \uc774\ub3d9\ud558\ub294 \uac83\uc744 \ubc1c\uacac\ud558\uc600\uace0, \uc790\uc2e0\uc774 \uc9c0\ud718\ud558\ub294 \ubd80\ub300\uc758 \uc55e\uc5d0 \uc788\ub294 \ubc29\uc5b4\ubd80\ub300\uac00 \uc774 \uac19\uc740 \uc6c0\uc9c1\uc784\uc73c\ub85c \uc778\ud558\uc5ec \uc57d\ud654\ub418\uc5c8\ub2e4\ub294 \uac83\uc744 \uae68\ub2ec\uc558\ub2e4. \uadf8\ub294 \ud718\ud558\uc758 \uad70\ub300\uc5d0\uac8c \uc989\uac01\uc801\uc73c\ub85c \uacf5\uaca9\uc744 \uac1c\uc2dc\ud558\ub3c4\ub85d \uba85\ub839\uc744 \ub0b4\ub838\uace0, \uc624\uc2a4\ud2b8\ub9ac\uc544 \ubcf4\ubcd1\ub300\ub294 \ubd95\uad34\ub418\uc5c8\ub2e4. \ud504\ub85c\uc774\uc13c\uc758 \ubca8\ub9c1 \ud6c4\uc0ac\ub974(The Prussian Belling Hussars)\uc640 KR4\ub294 \ubd95\uad34\ub41c \uc624\uc2a4\ud2b8\ub9ac\uc544\uad70\uc744 \ub3cc\ud30c\ud558\ub294 \ub3cc\uaca9\uc744 \uac10\ud589\ud558\uc600\uace0, \uc774\ub85c \uc778\ud574 \uc624\uc2a4\ud2b8\ub9ac\uc544\uad70\uc774 \ubd84\uc0b0\ub428\uc5d0 \ub530\ub77c \ud504\ub85c\uc774\uc13c\uad70\uc740 \uc2b9\uae30\ub97c \uc7a5\uc545\ud560 \uc218 \uc788\uc5c8\ub2e4. \uc2a4\ud1a8\ubca0\ub974\ud06c\ub294 \ub3d9\ub9f9\uad70\uc774 \ud1f4\uac01\ud568\uc5d0 \ub530\ub77c \uc790\uc2e0\uc774 \uc9c0\ud718\ud558\ub294 \ubd80\ub300\uc758 \uce21\uba74\uc774 \uc801\uc758 \uacf5\uaca9\uc5d0 \ub178\ucd9c\ub41c \uac83\uc744 \uae68\ub2eb\uace0 \uc989\uac01\uc801\uc73c\ub85c \ud1f4\uac01\ud560 \uac83\uc744 \uba85\ud588\ub2e4. \uc81c\uad6d\uad70\uc774 \uc790\uc2e0\uc758 \uc704\uce58\uc5d0\uc11c \ubb3c\ub7ec\ub0a8\uc5d0 \ub530\ub77c \uce84\ud53c\ud154\ub9ac\uc758 \ub098\uba38\uc9c0 \ubd80\ub300\ub3c4 \ub4a4\ub530\ub77c \ud1f4\uac01\uc744 \uc2dc\uc791\ud588\ub2e4. \ubbf8\ub9ac \uc608\uace0\ub418\uc5c8\ub4ef \uba54\uc774\uc5d0\ub974\uc758 \ubd80\ub300\ub294 \uc794\uc5ec\ubd80\ub300\ub97c \ub3d5\uae30 \uc704\ud574 \uc790\uc2e0\ub4e4\uc758 \uc704\uce58\uc5d0\uc11c \ubb3c\ub7ec\ub098\uc9c0 \uc54a\uc558\ub2e4. \uba54\uc774\uc5d0\ub974\ub294 \ud6c4\uc5d0 \uc790\uc2e0\uc774 \ub9e1\uc740 \uc704\uce58\ub97c \uc0ac\uc218\ud558\uae30 \uc704\ud558\uc5ec \ucd5c\ud6c4\uae4c\uc9c0 \uc2f8\uc6b0\ub77c\ub294 \uba85\ub839\uc744 \ub0b4\ub838\ub2e4\uace0 \uc8fc\uc7a5\ud558\uc600\ub2e4. \ud504\ub85c\uc774\uc13c\uad70\uc740 1,400\uba85\uc744 \uc783\uc5c8\uace0, \uc624\uc2a4\ud2b8\ub9ac\uc544-\uc2e0\uc131\ub85c\ub9c8\uc81c\uad6d\uad70\uc740 7,000\uba85 \uc758 \ubcd1\ub825\uacfc 9\uac1c\uc758 \uad70\uae30, 28\ubb38\uc758 \ud3ec\ub97c \uc0c1\uc2e4\ud588\ub2e4. ", "paragraph_answer": "\uacb0\ucf54 \ubb34\uc775\ud558\uac8c \uac00\ub9cc\ud788 \uc788\ub294 \uc0ac\ub78c\uc774 \uc544\ub2cc \ub300(\u5927) \uc288\ud130\ud558\uc784\uc740 \uc81c\uad6d\uad70\uc774 \ub0a8\ucabd\uc73c\ub85c \uc774\ub3d9\ud558\ub294 \uac83\uc744 \ubc1c\uacac\ud558\uc600\uace0, \uc790\uc2e0\uc774 \uc9c0\ud718\ud558\ub294 \ubd80\ub300\uc758 \uc55e\uc5d0 \uc788\ub294 \ubc29\uc5b4\ubd80\ub300\uac00 \uc774 \uac19\uc740 \uc6c0\uc9c1\uc784\uc73c\ub85c \uc778\ud558\uc5ec \uc57d\ud654\ub418\uc5c8\ub2e4\ub294 \uac83\uc744 \uae68\ub2ec\uc558\ub2e4. \uadf8\ub294 \ud718\ud558\uc758 \uad70\ub300\uc5d0\uac8c \uc989\uac01\uc801\uc73c\ub85c \uacf5\uaca9\uc744 \uac1c\uc2dc\ud558\ub3c4\ub85d \uba85\ub839\uc744 \ub0b4\ub838\uace0, \uc624\uc2a4\ud2b8\ub9ac\uc544 \ubcf4\ubcd1\ub300\ub294 \ubd95\uad34\ub418\uc5c8\ub2e4. \ud504\ub85c\uc774\uc13c\uc758 \ubca8\ub9c1 \ud6c4\uc0ac\ub974(The Prussian Belling Hussars)\uc640 KR4\ub294 \ubd95\uad34\ub41c \uc624\uc2a4\ud2b8\ub9ac\uc544\uad70\uc744 \ub3cc\ud30c\ud558\ub294 \ub3cc\uaca9\uc744 \uac10\ud589\ud558\uc600\uace0, \uc774\ub85c \uc778\ud574 \uc624\uc2a4\ud2b8\ub9ac\uc544\uad70\uc774 \ubd84\uc0b0\ub428\uc5d0 \ub530\ub77c \ud504\ub85c\uc774\uc13c\uad70\uc740 \uc2b9\uae30\ub97c \uc7a5\uc545\ud560 \uc218 \uc788\uc5c8\ub2e4. \uc2a4\ud1a8\ubca0\ub974\ud06c\ub294 \ub3d9\ub9f9\uad70\uc774 \ud1f4\uac01\ud568\uc5d0 \ub530\ub77c \uc790\uc2e0\uc774 \uc9c0\ud718\ud558\ub294 \ubd80\ub300\uc758 \uce21\uba74\uc774 \uc801\uc758 \uacf5\uaca9\uc5d0 \ub178\ucd9c\ub41c \uac83\uc744 \uae68\ub2eb\uace0 \uc989\uac01\uc801\uc73c\ub85c \ud1f4\uac01\ud560 \uac83\uc744 \uba85\ud588\ub2e4. \uc81c\uad6d\uad70\uc774 \uc790\uc2e0\uc758 \uc704\uce58\uc5d0\uc11c \ubb3c\ub7ec\ub0a8\uc5d0 \ub530\ub77c \uce84\ud53c\ud154\ub9ac\uc758 \ub098\uba38\uc9c0 \ubd80\ub300\ub3c4 \ub4a4\ub530\ub77c \ud1f4\uac01\uc744 \uc2dc\uc791\ud588\ub2e4. \ubbf8\ub9ac \uc608\uace0\ub418\uc5c8\ub4ef \uba54\uc774\uc5d0\ub974\uc758 \ubd80\ub300\ub294 \uc794\uc5ec\ubd80\ub300\ub97c \ub3d5\uae30 \uc704\ud574 \uc790\uc2e0\ub4e4\uc758 \uc704\uce58\uc5d0\uc11c \ubb3c\ub7ec\ub098\uc9c0 \uc54a\uc558\ub2e4. \uba54\uc774\uc5d0\ub974\ub294 \ud6c4\uc5d0 \uc790\uc2e0\uc774 \ub9e1\uc740 \uc704\uce58\ub97c \uc0ac\uc218\ud558\uae30 \uc704\ud558\uc5ec \ucd5c\ud6c4\uae4c\uc9c0 \uc2f8\uc6b0\ub77c\ub294 \uba85\ub839\uc744 \ub0b4\ub838\ub2e4\uace0 \uc8fc\uc7a5\ud558\uc600\ub2e4. \ud504\ub85c\uc774\uc13c\uad70\uc740 1,400\uba85\uc744 \uc783\uc5c8\uace0, \uc624\uc2a4\ud2b8\ub9ac\uc544-\uc2e0\uc131\ub85c\ub9c8\uc81c\uad6d\uad70\uc740 7,000\uba85 \uc758 \ubcd1\ub825\uacfc 9\uac1c\uc758 \uad70\uae30, 28\ubb38\uc758 \ud3ec\ub97c \uc0c1\uc2e4\ud588\ub2e4.", "sentence_answer": "\ud504\ub85c\uc774\uc13c\uad70\uc740 1,400\uba85\uc744 \uc783\uc5c8\uace0, \uc624\uc2a4\ud2b8\ub9ac\uc544-\uc2e0\uc131\ub85c\ub9c8\uc81c\uad6d\uad70\uc740 7,000\uba85 \uc758 \ubcd1\ub825\uacfc 9\uac1c\uc758 \uad70\uae30, 28\ubb38\uc758 \ud3ec\ub97c \uc0c1\uc2e4\ud588\ub2e4.", "paragraph_id": "6568515-3-1", "paragraph_question": "question: \uc624\uc2a4\ud2b8\ub9ac\uc544-\uc2e0\uc131\ub85c\ub9c8\uc81c\uad6d\uad70\uc774 \uc783\uc740 \ubcd1\ub825\uc758 \uc218\ub294?, context: \uacb0\ucf54 \ubb34\uc775\ud558\uac8c \uac00\ub9cc\ud788 \uc788\ub294 \uc0ac\ub78c\uc774 \uc544\ub2cc \ub300(\u5927) \uc288\ud130\ud558\uc784\uc740 \uc81c\uad6d\uad70\uc774 \ub0a8\ucabd\uc73c\ub85c \uc774\ub3d9\ud558\ub294 \uac83\uc744 \ubc1c\uacac\ud558\uc600\uace0, \uc790\uc2e0\uc774 \uc9c0\ud718\ud558\ub294 \ubd80\ub300\uc758 \uc55e\uc5d0 \uc788\ub294 \ubc29\uc5b4\ubd80\ub300\uac00 \uc774 \uac19\uc740 \uc6c0\uc9c1\uc784\uc73c\ub85c \uc778\ud558\uc5ec \uc57d\ud654\ub418\uc5c8\ub2e4\ub294 \uac83\uc744 \uae68\ub2ec\uc558\ub2e4. \uadf8\ub294 \ud718\ud558\uc758 \uad70\ub300\uc5d0\uac8c \uc989\uac01\uc801\uc73c\ub85c \uacf5\uaca9\uc744 \uac1c\uc2dc\ud558\ub3c4\ub85d \uba85\ub839\uc744 \ub0b4\ub838\uace0, \uc624\uc2a4\ud2b8\ub9ac\uc544 \ubcf4\ubcd1\ub300\ub294 \ubd95\uad34\ub418\uc5c8\ub2e4. \ud504\ub85c\uc774\uc13c\uc758 \ubca8\ub9c1 \ud6c4\uc0ac\ub974(The Prussian Belling Hussars)\uc640 KR4\ub294 \ubd95\uad34\ub41c \uc624\uc2a4\ud2b8\ub9ac\uc544\uad70\uc744 \ub3cc\ud30c\ud558\ub294 \ub3cc\uaca9\uc744 \uac10\ud589\ud558\uc600\uace0, \uc774\ub85c \uc778\ud574 \uc624\uc2a4\ud2b8\ub9ac\uc544\uad70\uc774 \ubd84\uc0b0\ub428\uc5d0 \ub530\ub77c \ud504\ub85c\uc774\uc13c\uad70\uc740 \uc2b9\uae30\ub97c \uc7a5\uc545\ud560 \uc218 \uc788\uc5c8\ub2e4. \uc2a4\ud1a8\ubca0\ub974\ud06c\ub294 \ub3d9\ub9f9\uad70\uc774 \ud1f4\uac01\ud568\uc5d0 \ub530\ub77c \uc790\uc2e0\uc774 \uc9c0\ud718\ud558\ub294 \ubd80\ub300\uc758 \uce21\uba74\uc774 \uc801\uc758 \uacf5\uaca9\uc5d0 \ub178\ucd9c\ub41c \uac83\uc744 \uae68\ub2eb\uace0 \uc989\uac01\uc801\uc73c\ub85c \ud1f4\uac01\ud560 \uac83\uc744 \uba85\ud588\ub2e4. \uc81c\uad6d\uad70\uc774 \uc790\uc2e0\uc758 \uc704\uce58\uc5d0\uc11c \ubb3c\ub7ec\ub0a8\uc5d0 \ub530\ub77c \uce84\ud53c\ud154\ub9ac\uc758 \ub098\uba38\uc9c0 \ubd80\ub300\ub3c4 \ub4a4\ub530\ub77c \ud1f4\uac01\uc744 \uc2dc\uc791\ud588\ub2e4. \ubbf8\ub9ac \uc608\uace0\ub418\uc5c8\ub4ef \uba54\uc774\uc5d0\ub974\uc758 \ubd80\ub300\ub294 \uc794\uc5ec\ubd80\ub300\ub97c \ub3d5\uae30 \uc704\ud574 \uc790\uc2e0\ub4e4\uc758 \uc704\uce58\uc5d0\uc11c \ubb3c\ub7ec\ub098\uc9c0 \uc54a\uc558\ub2e4. \uba54\uc774\uc5d0\ub974\ub294 \ud6c4\uc5d0 \uc790\uc2e0\uc774 \ub9e1\uc740 \uc704\uce58\ub97c \uc0ac\uc218\ud558\uae30 \uc704\ud558\uc5ec \ucd5c\ud6c4\uae4c\uc9c0 \uc2f8\uc6b0\ub77c\ub294 \uba85\ub839\uc744 \ub0b4\ub838\ub2e4\uace0 \uc8fc\uc7a5\ud558\uc600\ub2e4. \ud504\ub85c\uc774\uc13c\uad70\uc740 1,400\uba85\uc744 \uc783\uc5c8\uace0, \uc624\uc2a4\ud2b8\ub9ac\uc544-\uc2e0\uc131\ub85c\ub9c8\uc81c\uad6d\uad70\uc740 7,000\uba85\uc758 \ubcd1\ub825\uacfc 9\uac1c\uc758 \uad70\uae30, 28\ubb38\uc758 \ud3ec\ub97c \uc0c1\uc2e4\ud588\ub2e4."} {"question": "\ube14\ub799\ud640\uc740 \ubb34\uc5c7\uc73c\ub85c \uc778\ud574 \ube60\uc838\ub098\uc62c \uc218 \uc5c6\ub294 \uc2dc\uacf5\uac04 \uc601\uc5ed\uc778\uac00?", "paragraph": "\ube14\ub799\ud640(black hole)\uc740 \uac15\ub825\ud55c \ubc00\ub3c4\uc640 \uc911\ub825\uc73c\ub85c \uc778\ud574 \uc785\uc790\ub098 \uc804\uc790\uae30 \ubcf5\uc0ac, \ube5b\uc744 \ud3ec\ud568\ud55c \uadf8 \ubb34\uc5c7\ub3c4 \ube60\uc838\ub098\uc62c \uc218 \uc5c6\ub294 \uc2dc\uacf5\uac04 \uc601\uc5ed\uc774\ub2e4. \uc77c\ubc18\uc0c1\ub300\ub860\uc740 \ucda9\ubd84\ud788 \ubc00\uc9d1\ub41c \uc9c8\ub7c9\uc774 \uc2dc\uacf5\uc744 \ub4a4\ud2c0\uc5b4 \ube14\ub799\ud640\uc744 \ud615\uc131\ud560 \uc218 \uc788\uc74c\uc744 \uc608\uce21\ud55c\ub2e4. \ube14\ub799\ud640\ub85c\ubd80\ud130\uc758 \ud0c8\ucd9c\uc774 \ubd88\uac00\ub2a5\ud574\uc9c0\ub294 \uacbd\uacc4\ub97c \uc0ac\uac74\uc758 \uc9c0\ud3c9\uc120\uc774\ub77c\uace0 \ud55c\ub2e4. \uc5b4\ub5a4 \ubb3c\uccb4\uac00 \uc0ac\uac74\uc758 \uc9c0\ud3c9\uc120\uc744 \ub118\uc5b4\uac08 \uacbd\uc6b0, \uadf8 \ubb3c\uccb4\uc5d0\uac8c\ub294 \ud30c\uba78\uc801 \uc601\ud5a5\uc774 \uac00\ud574\uc9c0\uaca0\uc9c0\ub9cc, \ubc14\uae65 \uad00\ucc30\uc790\uc5d0\uac8c\ub294 \uc18d\ub3c4\uac00 \uc810\uc810 \ub290\ub824\uc838 \uadf8 \uacbd\uacc4\uc5d0 \uc601\uc6d0\ud788 \ub2ff\uc9c0 \uc54a\ub294 \uac83\ucc98\ub7fc \ubcf4\uc778\ub2e4. \ube14\ub799\ud640\uc740 \ube5b\uc744 \ubc18\uc0ac\ud558\uc9c0 \uc54a\uae30\uc5d0 \uc774\uc0c1\uc801 \ud751\uccb4\ucc98\ub7fc \ud589\ub3d9\ud55c\ub2e4. \ub610\ud55c \ud718\uc5b4\uc9c4 \uc2dc\uacf5\uac04\uc758 \uc591\uc790\uc7a5\ub860\uc5d0 \ub530\ub974\uba74 \uc0ac\uac74\uc758 \uc9c0\ud3c9\uc120\uc740 \ube14\ub799\ud640\uc758 \uc9c8\ub7c9\uc5d0 \ubc18\ube44\ub840\ud558\ub294 \uc628\ub3c4\ub97c \uac00\uc9c4 \ud751\uccb4\uc640 \uac19\uc740 \uc2a4\ud399\ud2b8\ub7fc\uc758 \uc5f4\ubcf5\uc0ac\ub97c \ubc29\ucd9c\ud558\uba70, \uc774\ub97c \ud638\ud0b9 \ubcf5\uc0ac\ub77c\uace0 \ud55c\ub2e4. \ud56d\uc131\uc9c8\ub7c9 \ube14\ub799\ud640\uc758 \uacbd\uc6b0 \uc774 \uc628\ub3c4\uac00 \uc218\uc2ed\uc5b5\ubd84\uc758 1 \ucf08\ube48 \uc218\uc900\uc774\uae30\uc5d0 \uadf8 \uc5f4\ubcf5\uc0ac\ub97c \uad00\uce21\ud558\ub294 \uac83\uc740 \ubcf8\uc9c8\uc801\uc73c\ub85c \ubd88\uac00\ub2a5\ud558\ub2e4.", "answer": "\uac15\ub825\ud55c \ubc00\ub3c4\uc640 \uc911\ub825", "sentence": "\ube14\ub799\ud640(black hole)\uc740 \uac15\ub825\ud55c \ubc00\ub3c4\uc640 \uc911\ub825 \uc73c\ub85c \uc778\ud574 \uc785\uc790\ub098 \uc804\uc790\uae30 \ubcf5\uc0ac, \ube5b\uc744 \ud3ec\ud568\ud55c \uadf8 \ubb34\uc5c7\ub3c4 \ube60\uc838\ub098\uc62c \uc218 \uc5c6\ub294 \uc2dc\uacf5\uac04 \uc601\uc5ed\uc774\ub2e4.", "paragraph_sentence": " \ube14\ub799\ud640(black hole)\uc740 \uac15\ub825\ud55c \ubc00\ub3c4\uc640 \uc911\ub825 \uc73c\ub85c \uc778\ud574 \uc785\uc790\ub098 \uc804\uc790\uae30 \ubcf5\uc0ac, \ube5b\uc744 \ud3ec\ud568\ud55c \uadf8 \ubb34\uc5c7\ub3c4 \ube60\uc838\ub098\uc62c \uc218 \uc5c6\ub294 \uc2dc\uacf5\uac04 \uc601\uc5ed\uc774\ub2e4. \uc77c\ubc18\uc0c1\ub300\ub860\uc740 \ucda9\ubd84\ud788 \ubc00\uc9d1\ub41c \uc9c8\ub7c9\uc774 \uc2dc\uacf5\uc744 \ub4a4\ud2c0\uc5b4 \ube14\ub799\ud640\uc744 \ud615\uc131\ud560 \uc218 \uc788\uc74c\uc744 \uc608\uce21\ud55c\ub2e4. \ube14\ub799\ud640\ub85c\ubd80\ud130\uc758 \ud0c8\ucd9c\uc774 \ubd88\uac00\ub2a5\ud574\uc9c0\ub294 \uacbd\uacc4\ub97c \uc0ac\uac74\uc758 \uc9c0\ud3c9\uc120\uc774\ub77c\uace0 \ud55c\ub2e4. \uc5b4\ub5a4 \ubb3c\uccb4\uac00 \uc0ac\uac74\uc758 \uc9c0\ud3c9\uc120\uc744 \ub118\uc5b4\uac08 \uacbd\uc6b0, \uadf8 \ubb3c\uccb4\uc5d0\uac8c\ub294 \ud30c\uba78\uc801 \uc601\ud5a5\uc774 \uac00\ud574\uc9c0\uaca0\uc9c0\ub9cc, \ubc14\uae65 \uad00\ucc30\uc790\uc5d0\uac8c\ub294 \uc18d\ub3c4\uac00 \uc810\uc810 \ub290\ub824\uc838 \uadf8 \uacbd\uacc4\uc5d0 \uc601\uc6d0\ud788 \ub2ff\uc9c0 \uc54a\ub294 \uac83\ucc98\ub7fc \ubcf4\uc778\ub2e4. \ube14\ub799\ud640\uc740 \ube5b\uc744 \ubc18\uc0ac\ud558\uc9c0 \uc54a\uae30\uc5d0 \uc774\uc0c1\uc801 \ud751\uccb4\ucc98\ub7fc \ud589\ub3d9\ud55c\ub2e4. \ub610\ud55c \ud718\uc5b4\uc9c4 \uc2dc\uacf5\uac04\uc758 \uc591\uc790\uc7a5\ub860\uc5d0 \ub530\ub974\uba74 \uc0ac\uac74\uc758 \uc9c0\ud3c9\uc120\uc740 \ube14\ub799\ud640\uc758 \uc9c8\ub7c9\uc5d0 \ubc18\ube44\ub840\ud558\ub294 \uc628\ub3c4\ub97c \uac00\uc9c4 \ud751\uccb4\uc640 \uac19\uc740 \uc2a4\ud399\ud2b8\ub7fc\uc758 \uc5f4\ubcf5\uc0ac\ub97c \ubc29\ucd9c\ud558\uba70, \uc774\ub97c \ud638\ud0b9 \ubcf5\uc0ac\ub77c\uace0 \ud55c\ub2e4. \ud56d\uc131\uc9c8\ub7c9 \ube14\ub799\ud640\uc758 \uacbd\uc6b0 \uc774 \uc628\ub3c4\uac00 \uc218\uc2ed\uc5b5\ubd84\uc758 1 \ucf08\ube48 \uc218\uc900\uc774\uae30\uc5d0 \uadf8 \uc5f4\ubcf5\uc0ac\ub97c \uad00\uce21\ud558\ub294 \uac83\uc740 \ubcf8\uc9c8\uc801\uc73c\ub85c \ubd88\uac00\ub2a5\ud558\ub2e4.", "paragraph_answer": "\ube14\ub799\ud640(black hole)\uc740 \uac15\ub825\ud55c \ubc00\ub3c4\uc640 \uc911\ub825 \uc73c\ub85c \uc778\ud574 \uc785\uc790\ub098 \uc804\uc790\uae30 \ubcf5\uc0ac, \ube5b\uc744 \ud3ec\ud568\ud55c \uadf8 \ubb34\uc5c7\ub3c4 \ube60\uc838\ub098\uc62c \uc218 \uc5c6\ub294 \uc2dc\uacf5\uac04 \uc601\uc5ed\uc774\ub2e4. \uc77c\ubc18\uc0c1\ub300\ub860\uc740 \ucda9\ubd84\ud788 \ubc00\uc9d1\ub41c \uc9c8\ub7c9\uc774 \uc2dc\uacf5\uc744 \ub4a4\ud2c0\uc5b4 \ube14\ub799\ud640\uc744 \ud615\uc131\ud560 \uc218 \uc788\uc74c\uc744 \uc608\uce21\ud55c\ub2e4. \ube14\ub799\ud640\ub85c\ubd80\ud130\uc758 \ud0c8\ucd9c\uc774 \ubd88\uac00\ub2a5\ud574\uc9c0\ub294 \uacbd\uacc4\ub97c \uc0ac\uac74\uc758 \uc9c0\ud3c9\uc120\uc774\ub77c\uace0 \ud55c\ub2e4. \uc5b4\ub5a4 \ubb3c\uccb4\uac00 \uc0ac\uac74\uc758 \uc9c0\ud3c9\uc120\uc744 \ub118\uc5b4\uac08 \uacbd\uc6b0, \uadf8 \ubb3c\uccb4\uc5d0\uac8c\ub294 \ud30c\uba78\uc801 \uc601\ud5a5\uc774 \uac00\ud574\uc9c0\uaca0\uc9c0\ub9cc, \ubc14\uae65 \uad00\ucc30\uc790\uc5d0\uac8c\ub294 \uc18d\ub3c4\uac00 \uc810\uc810 \ub290\ub824\uc838 \uadf8 \uacbd\uacc4\uc5d0 \uc601\uc6d0\ud788 \ub2ff\uc9c0 \uc54a\ub294 \uac83\ucc98\ub7fc \ubcf4\uc778\ub2e4. \ube14\ub799\ud640\uc740 \ube5b\uc744 \ubc18\uc0ac\ud558\uc9c0 \uc54a\uae30\uc5d0 \uc774\uc0c1\uc801 \ud751\uccb4\ucc98\ub7fc \ud589\ub3d9\ud55c\ub2e4. \ub610\ud55c \ud718\uc5b4\uc9c4 \uc2dc\uacf5\uac04\uc758 \uc591\uc790\uc7a5\ub860\uc5d0 \ub530\ub974\uba74 \uc0ac\uac74\uc758 \uc9c0\ud3c9\uc120\uc740 \ube14\ub799\ud640\uc758 \uc9c8\ub7c9\uc5d0 \ubc18\ube44\ub840\ud558\ub294 \uc628\ub3c4\ub97c \uac00\uc9c4 \ud751\uccb4\uc640 \uac19\uc740 \uc2a4\ud399\ud2b8\ub7fc\uc758 \uc5f4\ubcf5\uc0ac\ub97c \ubc29\ucd9c\ud558\uba70, \uc774\ub97c \ud638\ud0b9 \ubcf5\uc0ac\ub77c\uace0 \ud55c\ub2e4. \ud56d\uc131\uc9c8\ub7c9 \ube14\ub799\ud640\uc758 \uacbd\uc6b0 \uc774 \uc628\ub3c4\uac00 \uc218\uc2ed\uc5b5\ubd84\uc758 1 \ucf08\ube48 \uc218\uc900\uc774\uae30\uc5d0 \uadf8 \uc5f4\ubcf5\uc0ac\ub97c \uad00\uce21\ud558\ub294 \uac83\uc740 \ubcf8\uc9c8\uc801\uc73c\ub85c \ubd88\uac00\ub2a5\ud558\ub2e4.", "sentence_answer": "\ube14\ub799\ud640(black hole)\uc740 \uac15\ub825\ud55c \ubc00\ub3c4\uc640 \uc911\ub825 \uc73c\ub85c \uc778\ud574 \uc785\uc790\ub098 \uc804\uc790\uae30 \ubcf5\uc0ac, \ube5b\uc744 \ud3ec\ud568\ud55c \uadf8 \ubb34\uc5c7\ub3c4 \ube60\uc838\ub098\uc62c \uc218 \uc5c6\ub294 \uc2dc\uacf5\uac04 \uc601\uc5ed\uc774\ub2e4.", "paragraph_id": "6583522-0-2", "paragraph_question": "question: \ube14\ub799\ud640\uc740 \ubb34\uc5c7\uc73c\ub85c \uc778\ud574 \ube60\uc838\ub098\uc62c \uc218 \uc5c6\ub294 \uc2dc\uacf5\uac04 \uc601\uc5ed\uc778\uac00?, context: \ube14\ub799\ud640(black hole)\uc740 \uac15\ub825\ud55c \ubc00\ub3c4\uc640 \uc911\ub825\uc73c\ub85c \uc778\ud574 \uc785\uc790\ub098 \uc804\uc790\uae30 \ubcf5\uc0ac, \ube5b\uc744 \ud3ec\ud568\ud55c \uadf8 \ubb34\uc5c7\ub3c4 \ube60\uc838\ub098\uc62c \uc218 \uc5c6\ub294 \uc2dc\uacf5\uac04 \uc601\uc5ed\uc774\ub2e4. \uc77c\ubc18\uc0c1\ub300\ub860\uc740 \ucda9\ubd84\ud788 \ubc00\uc9d1\ub41c \uc9c8\ub7c9\uc774 \uc2dc\uacf5\uc744 \ub4a4\ud2c0\uc5b4 \ube14\ub799\ud640\uc744 \ud615\uc131\ud560 \uc218 \uc788\uc74c\uc744 \uc608\uce21\ud55c\ub2e4. \ube14\ub799\ud640\ub85c\ubd80\ud130\uc758 \ud0c8\ucd9c\uc774 \ubd88\uac00\ub2a5\ud574\uc9c0\ub294 \uacbd\uacc4\ub97c \uc0ac\uac74\uc758 \uc9c0\ud3c9\uc120\uc774\ub77c\uace0 \ud55c\ub2e4. \uc5b4\ub5a4 \ubb3c\uccb4\uac00 \uc0ac\uac74\uc758 \uc9c0\ud3c9\uc120\uc744 \ub118\uc5b4\uac08 \uacbd\uc6b0, \uadf8 \ubb3c\uccb4\uc5d0\uac8c\ub294 \ud30c\uba78\uc801 \uc601\ud5a5\uc774 \uac00\ud574\uc9c0\uaca0\uc9c0\ub9cc, \ubc14\uae65 \uad00\ucc30\uc790\uc5d0\uac8c\ub294 \uc18d\ub3c4\uac00 \uc810\uc810 \ub290\ub824\uc838 \uadf8 \uacbd\uacc4\uc5d0 \uc601\uc6d0\ud788 \ub2ff\uc9c0 \uc54a\ub294 \uac83\ucc98\ub7fc \ubcf4\uc778\ub2e4. \ube14\ub799\ud640\uc740 \ube5b\uc744 \ubc18\uc0ac\ud558\uc9c0 \uc54a\uae30\uc5d0 \uc774\uc0c1\uc801 \ud751\uccb4\ucc98\ub7fc \ud589\ub3d9\ud55c\ub2e4. \ub610\ud55c \ud718\uc5b4\uc9c4 \uc2dc\uacf5\uac04\uc758 \uc591\uc790\uc7a5\ub860\uc5d0 \ub530\ub974\uba74 \uc0ac\uac74\uc758 \uc9c0\ud3c9\uc120\uc740 \ube14\ub799\ud640\uc758 \uc9c8\ub7c9\uc5d0 \ubc18\ube44\ub840\ud558\ub294 \uc628\ub3c4\ub97c \uac00\uc9c4 \ud751\uccb4\uc640 \uac19\uc740 \uc2a4\ud399\ud2b8\ub7fc\uc758 \uc5f4\ubcf5\uc0ac\ub97c \ubc29\ucd9c\ud558\uba70, \uc774\ub97c \ud638\ud0b9 \ubcf5\uc0ac\ub77c\uace0 \ud55c\ub2e4. \ud56d\uc131\uc9c8\ub7c9 \ube14\ub799\ud640\uc758 \uacbd\uc6b0 \uc774 \uc628\ub3c4\uac00 \uc218\uc2ed\uc5b5\ubd84\uc758 1 \ucf08\ube48 \uc218\uc900\uc774\uae30\uc5d0 \uadf8 \uc5f4\ubcf5\uc0ac\ub97c \uad00\uce21\ud558\ub294 \uac83\uc740 \ubcf8\uc9c8\uc801\uc73c\ub85c \ubd88\uac00\ub2a5\ud558\ub2e4."} {"question": "\uad6d\uac00\uc7ac\uac74\ucd5c\uace0\ud68c\uc758 \uc758\uc7a5\uc774\uba70 \uc774\uc740\uc5d0\uac8c \uc0dd\ud65c\ube44\ub97c \uc9c0\uc6d0\ud574\uc900 \uc0ac\ub78c\uc740?", "paragraph": "\ub300\ud55c\ubbfc\uad6d\uc5d0\uc11c 4\u00b719 \ud601\uba85\uc73c\ub85c \uc81c2\uacf5\ud654\uad6d\uc774 \ucd9c\ubc94\ud558\uc790 \uad6d\ubb34\ucd1d\ub9ac \uc7a5\uba74\uc740 \uc774\uc740\uc5d0\uac8c \uc8fc\uc601 \ub300\uc0ac\uc9c1\uc744 \uc81c\uc548\ud558\uc600\uc9c0\ub9cc \uc774\uc740\uc740 \uac74\uac15\uc0c1 \uc774\uc720\ub85c \uac70\uc808\ud558\uc600\ub2e4. \uc774\uc740\uc774\ub77c\ub294 \uc0c1\uc9d5\uc801 \uc778\ubb3c\uc744 \ud1b5\ud574\uc11c \ubbfc\uc8fc\ub2f9 \uc815\uad8c\uc758 \ubd88\uc548\ud55c \uc785\uc9c0\ub97c \ud655\uace0\ud788 \ub2e4\uc9c0\uaca0\ub2e4\ub294 \uc815\uce58\uc801 \ud310\ub2e8\ub3c4 \uc788\uc5c8\ub2e4. \uc0c8\ub85c\uc6b4 \ub300\ud55c\ubbfc\uad6d \uc815\ubd80\ub294 \uc774\uc740\uc758 \uadc0\uad6d\uc5d0 \ud638\uc758\uc801\uc774\uc5c8\uc73c\uba70, \uc774\uc740\ub3c4 \ud658\uad6d\ud558\uaca0\ub2e4\ub294 \uc758\uc0ac\ub97c \uc804\ub2ec\ud558\uc600\ub2e4. 1961\ub144 5\uc6d4\uc5d0 \uc774\uc740\uc740 \uc774\uad6c\uac00 \uba38\ubb3c\uace0 \uc788\ub294 \ubbf8\uad6d \ud558\uc640\uc774 \uc8fc\ub97c \ubc29\ubb38\ud55c \ud6c4\uc5d0 \uc77c\ubcf8\uc73c\ub85c \ub3cc\uc544\uc654\ub294\ub370 \ubcd1\uc774 \uc7ac\ubc1c\ud558\uc5ec \uc911\ud0dc\uc5d0 \ube60\uc838 \uc131 \ub204\uac00 \ubcd1\uc6d0\uc5d0 \uc785\uc6d0\ud558\uc600\ub2e4. 5.16 \uad70\uc0ac \uc815\ubcc0\uc73c\ub85c \uc9d1\uad8c\ud55c \uad6d\uac00\uc7ac\uac74\ucd5c\uace0\ud68c\uc758 \uc758\uc7a5 \ubc15\uc815\ud76c\ub294 \uc774\uc740\uc758 \uce58\ub8cc\ube44\uc640 \uc0dd\ud65c\ube44\ub97c \uc9c0\uc6d0\ud558\uc600\ub2e4. 1963\ub144 5\uc6d4\uc5d0 \uc0c1\ud0dc\uac00 \uc545\ud654\ub418\uc5b4 \uc544\uce74\uc0ac\uce74\uc758 \uc0b0\ub178(\u5c71\u738b) \ubcd1\uc6d0\uc5d0 \uc785\uc6d0\ud558\uc600\ub2e4.", "answer": "\ubc15\uc815\ud76c", "sentence": "5.16 \uad70\uc0ac \uc815\ubcc0\uc73c\ub85c \uc9d1\uad8c\ud55c \uad6d\uac00\uc7ac\uac74\ucd5c\uace0\ud68c\uc758 \uc758\uc7a5 \ubc15\uc815\ud76c \ub294 \uc774\uc740\uc758 \uce58\ub8cc\ube44\uc640 \uc0dd\ud65c\ube44\ub97c \uc9c0\uc6d0\ud558\uc600\ub2e4.", "paragraph_sentence": "\ub300\ud55c\ubbfc\uad6d\uc5d0\uc11c 4\u00b719 \ud601\uba85\uc73c\ub85c \uc81c2\uacf5\ud654\uad6d\uc774 \ucd9c\ubc94\ud558\uc790 \uad6d\ubb34\ucd1d\ub9ac \uc7a5\uba74\uc740 \uc774\uc740\uc5d0\uac8c \uc8fc\uc601 \ub300\uc0ac\uc9c1\uc744 \uc81c\uc548\ud558\uc600\uc9c0\ub9cc \uc774\uc740\uc740 \uac74\uac15\uc0c1 \uc774\uc720\ub85c \uac70\uc808\ud558\uc600\ub2e4. \uc774\uc740\uc774\ub77c\ub294 \uc0c1\uc9d5\uc801 \uc778\ubb3c\uc744 \ud1b5\ud574\uc11c \ubbfc\uc8fc\ub2f9 \uc815\uad8c\uc758 \ubd88\uc548\ud55c \uc785\uc9c0\ub97c \ud655\uace0\ud788 \ub2e4\uc9c0\uaca0\ub2e4\ub294 \uc815\uce58\uc801 \ud310\ub2e8\ub3c4 \uc788\uc5c8\ub2e4. \uc0c8\ub85c\uc6b4 \ub300\ud55c\ubbfc\uad6d \uc815\ubd80\ub294 \uc774\uc740\uc758 \uadc0\uad6d\uc5d0 \ud638\uc758\uc801\uc774\uc5c8\uc73c\uba70, \uc774\uc740\ub3c4 \ud658\uad6d\ud558\uaca0\ub2e4\ub294 \uc758\uc0ac\ub97c \uc804\ub2ec\ud558\uc600\ub2e4. 1961\ub144 5\uc6d4\uc5d0 \uc774\uc740\uc740 \uc774\uad6c\uac00 \uba38\ubb3c\uace0 \uc788\ub294 \ubbf8\uad6d \ud558\uc640\uc774 \uc8fc\ub97c \ubc29\ubb38\ud55c \ud6c4\uc5d0 \uc77c\ubcf8\uc73c\ub85c \ub3cc\uc544\uc654\ub294\ub370 \ubcd1\uc774 \uc7ac\ubc1c\ud558\uc5ec \uc911\ud0dc\uc5d0 \ube60\uc838 \uc131 \ub204\uac00 \ubcd1\uc6d0\uc5d0 \uc785\uc6d0\ud558\uc600\ub2e4. 5.16 \uad70\uc0ac \uc815\ubcc0\uc73c\ub85c \uc9d1\uad8c\ud55c \uad6d\uac00\uc7ac\uac74\ucd5c\uace0\ud68c\uc758 \uc758\uc7a5 \ubc15\uc815\ud76c \ub294 \uc774\uc740\uc758 \uce58\ub8cc\ube44\uc640 \uc0dd\ud65c\ube44\ub97c \uc9c0\uc6d0\ud558\uc600\ub2e4. 1963\ub144 5\uc6d4\uc5d0 \uc0c1\ud0dc\uac00 \uc545\ud654\ub418\uc5b4 \uc544\uce74\uc0ac\uce74\uc758 \uc0b0\ub178(\u5c71\u738b) \ubcd1\uc6d0\uc5d0 \uc785\uc6d0\ud558\uc600\ub2e4.", "paragraph_answer": "\ub300\ud55c\ubbfc\uad6d\uc5d0\uc11c 4\u00b719 \ud601\uba85\uc73c\ub85c \uc81c2\uacf5\ud654\uad6d\uc774 \ucd9c\ubc94\ud558\uc790 \uad6d\ubb34\ucd1d\ub9ac \uc7a5\uba74\uc740 \uc774\uc740\uc5d0\uac8c \uc8fc\uc601 \ub300\uc0ac\uc9c1\uc744 \uc81c\uc548\ud558\uc600\uc9c0\ub9cc \uc774\uc740\uc740 \uac74\uac15\uc0c1 \uc774\uc720\ub85c \uac70\uc808\ud558\uc600\ub2e4. \uc774\uc740\uc774\ub77c\ub294 \uc0c1\uc9d5\uc801 \uc778\ubb3c\uc744 \ud1b5\ud574\uc11c \ubbfc\uc8fc\ub2f9 \uc815\uad8c\uc758 \ubd88\uc548\ud55c \uc785\uc9c0\ub97c \ud655\uace0\ud788 \ub2e4\uc9c0\uaca0\ub2e4\ub294 \uc815\uce58\uc801 \ud310\ub2e8\ub3c4 \uc788\uc5c8\ub2e4. \uc0c8\ub85c\uc6b4 \ub300\ud55c\ubbfc\uad6d \uc815\ubd80\ub294 \uc774\uc740\uc758 \uadc0\uad6d\uc5d0 \ud638\uc758\uc801\uc774\uc5c8\uc73c\uba70, \uc774\uc740\ub3c4 \ud658\uad6d\ud558\uaca0\ub2e4\ub294 \uc758\uc0ac\ub97c \uc804\ub2ec\ud558\uc600\ub2e4. 1961\ub144 5\uc6d4\uc5d0 \uc774\uc740\uc740 \uc774\uad6c\uac00 \uba38\ubb3c\uace0 \uc788\ub294 \ubbf8\uad6d \ud558\uc640\uc774 \uc8fc\ub97c \ubc29\ubb38\ud55c \ud6c4\uc5d0 \uc77c\ubcf8\uc73c\ub85c \ub3cc\uc544\uc654\ub294\ub370 \ubcd1\uc774 \uc7ac\ubc1c\ud558\uc5ec \uc911\ud0dc\uc5d0 \ube60\uc838 \uc131 \ub204\uac00 \ubcd1\uc6d0\uc5d0 \uc785\uc6d0\ud558\uc600\ub2e4. 5.16 \uad70\uc0ac \uc815\ubcc0\uc73c\ub85c \uc9d1\uad8c\ud55c \uad6d\uac00\uc7ac\uac74\ucd5c\uace0\ud68c\uc758 \uc758\uc7a5 \ubc15\uc815\ud76c \ub294 \uc774\uc740\uc758 \uce58\ub8cc\ube44\uc640 \uc0dd\ud65c\ube44\ub97c \uc9c0\uc6d0\ud558\uc600\ub2e4. 1963\ub144 5\uc6d4\uc5d0 \uc0c1\ud0dc\uac00 \uc545\ud654\ub418\uc5b4 \uc544\uce74\uc0ac\uce74\uc758 \uc0b0\ub178(\u5c71\u738b) \ubcd1\uc6d0\uc5d0 \uc785\uc6d0\ud558\uc600\ub2e4.", "sentence_answer": "5.16 \uad70\uc0ac \uc815\ubcc0\uc73c\ub85c \uc9d1\uad8c\ud55c \uad6d\uac00\uc7ac\uac74\ucd5c\uace0\ud68c\uc758 \uc758\uc7a5 \ubc15\uc815\ud76c \ub294 \uc774\uc740\uc758 \uce58\ub8cc\ube44\uc640 \uc0dd\ud65c\ube44\ub97c \uc9c0\uc6d0\ud558\uc600\ub2e4.", "paragraph_id": "6589822-15-1", "paragraph_question": "question: \uad6d\uac00\uc7ac\uac74\ucd5c\uace0\ud68c\uc758 \uc758\uc7a5\uc774\uba70 \uc774\uc740\uc5d0\uac8c \uc0dd\ud65c\ube44\ub97c \uc9c0\uc6d0\ud574\uc900 \uc0ac\ub78c\uc740?, context: \ub300\ud55c\ubbfc\uad6d\uc5d0\uc11c 4\u00b719 \ud601\uba85\uc73c\ub85c \uc81c2\uacf5\ud654\uad6d\uc774 \ucd9c\ubc94\ud558\uc790 \uad6d\ubb34\ucd1d\ub9ac \uc7a5\uba74\uc740 \uc774\uc740\uc5d0\uac8c \uc8fc\uc601 \ub300\uc0ac\uc9c1\uc744 \uc81c\uc548\ud558\uc600\uc9c0\ub9cc \uc774\uc740\uc740 \uac74\uac15\uc0c1 \uc774\uc720\ub85c \uac70\uc808\ud558\uc600\ub2e4. \uc774\uc740\uc774\ub77c\ub294 \uc0c1\uc9d5\uc801 \uc778\ubb3c\uc744 \ud1b5\ud574\uc11c \ubbfc\uc8fc\ub2f9 \uc815\uad8c\uc758 \ubd88\uc548\ud55c \uc785\uc9c0\ub97c \ud655\uace0\ud788 \ub2e4\uc9c0\uaca0\ub2e4\ub294 \uc815\uce58\uc801 \ud310\ub2e8\ub3c4 \uc788\uc5c8\ub2e4. \uc0c8\ub85c\uc6b4 \ub300\ud55c\ubbfc\uad6d \uc815\ubd80\ub294 \uc774\uc740\uc758 \uadc0\uad6d\uc5d0 \ud638\uc758\uc801\uc774\uc5c8\uc73c\uba70, \uc774\uc740\ub3c4 \ud658\uad6d\ud558\uaca0\ub2e4\ub294 \uc758\uc0ac\ub97c \uc804\ub2ec\ud558\uc600\ub2e4. 1961\ub144 5\uc6d4\uc5d0 \uc774\uc740\uc740 \uc774\uad6c\uac00 \uba38\ubb3c\uace0 \uc788\ub294 \ubbf8\uad6d \ud558\uc640\uc774 \uc8fc\ub97c \ubc29\ubb38\ud55c \ud6c4\uc5d0 \uc77c\ubcf8\uc73c\ub85c \ub3cc\uc544\uc654\ub294\ub370 \ubcd1\uc774 \uc7ac\ubc1c\ud558\uc5ec \uc911\ud0dc\uc5d0 \ube60\uc838 \uc131 \ub204\uac00 \ubcd1\uc6d0\uc5d0 \uc785\uc6d0\ud558\uc600\ub2e4. 5.16 \uad70\uc0ac \uc815\ubcc0\uc73c\ub85c \uc9d1\uad8c\ud55c \uad6d\uac00\uc7ac\uac74\ucd5c\uace0\ud68c\uc758 \uc758\uc7a5 \ubc15\uc815\ud76c\ub294 \uc774\uc740\uc758 \uce58\ub8cc\ube44\uc640 \uc0dd\ud65c\ube44\ub97c \uc9c0\uc6d0\ud558\uc600\ub2e4. 1963\ub144 5\uc6d4\uc5d0 \uc0c1\ud0dc\uac00 \uc545\ud654\ub418\uc5b4 \uc544\uce74\uc0ac\uce74\uc758 \uc0b0\ub178(\u5c71\u738b) \ubcd1\uc6d0\uc5d0 \uc785\uc6d0\ud558\uc600\ub2e4."} {"question": "24\uac1c\uc6d4 \uc774\ud558\uc758 \uc544\uc774\uc5d0\uac8c \ub0b4\uc131\uade0 \uac00\ub2a5\uc131\uc774 \ub192\ub2e4\uba74 \uc5b4\ub5a4 \ud56d\uc0dd\uc81c\ub97c \uc0ac\uc6a9\ud574\uc57c \ud558\ub294\uac00?", "paragraph": "\ud56d\uc0dd\uc81c \ub0b4\uc131\uade0\uc740 \uc5b4\ub9b0\uc774\uc9d1\uc5d0 \ub2e4\ub2c8\ub294 \uacbd\uc6b0, \ucd5c\uadfc 30\uc77c \uc774\ub0b4 \ud56d\uc0dd\uc81c\ub97c \uc0ac\uc6a9\ud55c \uacbd\uc6b0, 24\uac1c\uc6d4 \uc774\ud558\uc778 \uacbd\uc6b0 \ub0b4\uc131\uade0\uc758 \uac00\ub2a5\uc131\uc774 \ub192\uae30 \ub54c\ubb38\uc5d0 \uc774\ub7f0 \uacbd\uc6b0 \uace0\uc6a9\ub7c9 \uc544\ubaa9\uc2dc\uc2e4\ub9b0 \uc0ac\uc6a9\uc774 \uc88b\ub2e4. \ubd80\ube44\ub3d9\uc5fc \ub4f1\uc5d0\uc11c \ud1b5\uc0c1 \uc6a9\ub7c9\uc778 20~40 mg/kg \uacfc \ube44\uad50\ud558\uba74 \uae09\uc131 \uc911\uc774\uc5fc\uc758 \uace0\uc6a9\ub7c9 \ud56d\uc0dd\uc81c \ubcf5\uc6a9 \uc2dc \ucca0\uc800\ud55c \ubcf5\uc6a9\uc744 \ud574\uc57c \ub0b4\uc131\uade0\uc744 \uc5c6\uc568 \uc218 \uc788\uc74c\uc5d0\ub3c4 \ud56d\uc0dd\uc81c\ub97c \uac70\ub974\ub294 \uac78 \uc88b\uc544\ud558\ub294 \ud55c\uad6d\uc778\uc758 \uc2b5\uad00\uc740 \ud56d\uc0dd\uc81c \ub0b4\uc131\uc744 \ub354\uc6b1 \uc545\ud654\uc2dc\ud0b4\uc744 \uace0\ub824\ud574\uc57c \ud55c\ub2e4. \ud56d\uc0dd\uc81c \ubcf5\uc6a9 \uae30\uac04\uc740 \ud1b5\uc0c1 5\uc77c\uc774\ub098 \uc758\uc0ac\uc758 \ud310\ub2e8\uc5d0 \ub530\ub77c \uacb0\uc815\ud558\uba70, 24\uac1c\uc6d4 \uc774\ud558, \uace0\ub9c9 \ucc9c\uacf5\uc774 \uc788\ub294 \uacbd\uc6b0, \uc7ac\ubc1c\uc131 \uae09\uc131 \uc911\uc774\uc5fc\uc778 \uacbd\uc6b0 10\uc77c\uae4c\uc9c0 \uc5f0\uc7a5 \uc0ac\uc6a9\uc744 \uad8c\uc7a5\ud558\uace0 \uc788\ub2e4. \uc7a5\uc5fc, \uad6c\ud1a0 \ub4f1\uc73c\ub85c \uc778\ud574 \uacbd\uad6c \ud56d\uc0dd\uc81c \ud22c\uc5ec\uac00 \ubd88\uac00\ub2a5\ud560 \uacbd\uc6b0 Ceftriaxone (50 mg/kg) \uc8fc\uc0ac\ub97c 1\ud68c 1~3\uc77c \uc0ac\uc6a9 \uc2dc \uacbd\uad6c \ud56d\uc0dd\uc81c\uc640 \ub3d9\uc77c\ud55c \ud6a8\uacfc\ub97c \ubcfc \uc218 \uc788\ub2e4.", "answer": "\uace0\uc6a9\ub7c9 \uc544\ubaa9\uc2dc\uc2e4\ub9b0", "sentence": "\ud56d\uc0dd\uc81c \ub0b4\uc131\uade0\uc740 \uc5b4\ub9b0\uc774\uc9d1\uc5d0 \ub2e4\ub2c8\ub294 \uacbd\uc6b0, \ucd5c\uadfc 30\uc77c \uc774\ub0b4 \ud56d\uc0dd\uc81c\ub97c \uc0ac\uc6a9\ud55c \uacbd\uc6b0, 24\uac1c\uc6d4 \uc774\ud558\uc778 \uacbd\uc6b0 \ub0b4\uc131\uade0\uc758 \uac00\ub2a5\uc131\uc774 \ub192\uae30 \ub54c\ubb38\uc5d0 \uc774\ub7f0 \uacbd\uc6b0 \uace0\uc6a9\ub7c9 \uc544\ubaa9\uc2dc\uc2e4\ub9b0 \uc0ac\uc6a9\uc774 \uc88b\ub2e4.", "paragraph_sentence": " \ud56d\uc0dd\uc81c \ub0b4\uc131\uade0\uc740 \uc5b4\ub9b0\uc774\uc9d1\uc5d0 \ub2e4\ub2c8\ub294 \uacbd\uc6b0, \ucd5c\uadfc 30\uc77c \uc774\ub0b4 \ud56d\uc0dd\uc81c\ub97c \uc0ac\uc6a9\ud55c \uacbd\uc6b0, 24\uac1c\uc6d4 \uc774\ud558\uc778 \uacbd\uc6b0 \ub0b4\uc131\uade0\uc758 \uac00\ub2a5\uc131\uc774 \ub192\uae30 \ub54c\ubb38\uc5d0 \uc774\ub7f0 \uacbd\uc6b0 \uace0\uc6a9\ub7c9 \uc544\ubaa9\uc2dc\uc2e4\ub9b0 \uc0ac\uc6a9\uc774 \uc88b\ub2e4. \ubd80\ube44\ub3d9\uc5fc \ub4f1\uc5d0\uc11c \ud1b5\uc0c1 \uc6a9\ub7c9\uc778 20~40 mg/kg \uacfc \ube44\uad50\ud558\uba74 \uae09\uc131 \uc911\uc774\uc5fc\uc758 \uace0\uc6a9\ub7c9 \ud56d\uc0dd\uc81c \ubcf5\uc6a9 \uc2dc \ucca0\uc800\ud55c \ubcf5\uc6a9\uc744 \ud574\uc57c \ub0b4\uc131\uade0\uc744 \uc5c6\uc568 \uc218 \uc788\uc74c\uc5d0\ub3c4 \ud56d\uc0dd\uc81c\ub97c \uac70\ub974\ub294 \uac78 \uc88b\uc544\ud558\ub294 \ud55c\uad6d\uc778\uc758 \uc2b5\uad00\uc740 \ud56d\uc0dd\uc81c \ub0b4\uc131\uc744 \ub354\uc6b1 \uc545\ud654\uc2dc\ud0b4\uc744 \uace0\ub824\ud574\uc57c \ud55c\ub2e4. \ud56d\uc0dd\uc81c \ubcf5\uc6a9 \uae30\uac04\uc740 \ud1b5\uc0c1 5\uc77c\uc774\ub098 \uc758\uc0ac\uc758 \ud310\ub2e8\uc5d0 \ub530\ub77c \uacb0\uc815\ud558\uba70, 24\uac1c\uc6d4 \uc774\ud558, \uace0\ub9c9 \ucc9c\uacf5\uc774 \uc788\ub294 \uacbd\uc6b0, \uc7ac\ubc1c\uc131 \uae09\uc131 \uc911\uc774\uc5fc\uc778 \uacbd\uc6b0 10\uc77c\uae4c\uc9c0 \uc5f0\uc7a5 \uc0ac\uc6a9\uc744 \uad8c\uc7a5\ud558\uace0 \uc788\ub2e4. \uc7a5\uc5fc, \uad6c\ud1a0 \ub4f1\uc73c\ub85c \uc778\ud574 \uacbd\uad6c \ud56d\uc0dd\uc81c \ud22c\uc5ec\uac00 \ubd88\uac00\ub2a5\ud560 \uacbd\uc6b0 Ceftriaxone (50 mg/kg) \uc8fc\uc0ac\ub97c 1\ud68c 1~3\uc77c \uc0ac\uc6a9 \uc2dc \uacbd\uad6c \ud56d\uc0dd\uc81c\uc640 \ub3d9\uc77c\ud55c \ud6a8\uacfc\ub97c \ubcfc \uc218 \uc788\ub2e4.", "paragraph_answer": "\ud56d\uc0dd\uc81c \ub0b4\uc131\uade0\uc740 \uc5b4\ub9b0\uc774\uc9d1\uc5d0 \ub2e4\ub2c8\ub294 \uacbd\uc6b0, \ucd5c\uadfc 30\uc77c \uc774\ub0b4 \ud56d\uc0dd\uc81c\ub97c \uc0ac\uc6a9\ud55c \uacbd\uc6b0, 24\uac1c\uc6d4 \uc774\ud558\uc778 \uacbd\uc6b0 \ub0b4\uc131\uade0\uc758 \uac00\ub2a5\uc131\uc774 \ub192\uae30 \ub54c\ubb38\uc5d0 \uc774\ub7f0 \uacbd\uc6b0 \uace0\uc6a9\ub7c9 \uc544\ubaa9\uc2dc\uc2e4\ub9b0 \uc0ac\uc6a9\uc774 \uc88b\ub2e4. \ubd80\ube44\ub3d9\uc5fc \ub4f1\uc5d0\uc11c \ud1b5\uc0c1 \uc6a9\ub7c9\uc778 20~40 mg/kg \uacfc \ube44\uad50\ud558\uba74 \uae09\uc131 \uc911\uc774\uc5fc\uc758 \uace0\uc6a9\ub7c9 \ud56d\uc0dd\uc81c \ubcf5\uc6a9 \uc2dc \ucca0\uc800\ud55c \ubcf5\uc6a9\uc744 \ud574\uc57c \ub0b4\uc131\uade0\uc744 \uc5c6\uc568 \uc218 \uc788\uc74c\uc5d0\ub3c4 \ud56d\uc0dd\uc81c\ub97c \uac70\ub974\ub294 \uac78 \uc88b\uc544\ud558\ub294 \ud55c\uad6d\uc778\uc758 \uc2b5\uad00\uc740 \ud56d\uc0dd\uc81c \ub0b4\uc131\uc744 \ub354\uc6b1 \uc545\ud654\uc2dc\ud0b4\uc744 \uace0\ub824\ud574\uc57c \ud55c\ub2e4. \ud56d\uc0dd\uc81c \ubcf5\uc6a9 \uae30\uac04\uc740 \ud1b5\uc0c1 5\uc77c\uc774\ub098 \uc758\uc0ac\uc758 \ud310\ub2e8\uc5d0 \ub530\ub77c \uacb0\uc815\ud558\uba70, 24\uac1c\uc6d4 \uc774\ud558, \uace0\ub9c9 \ucc9c\uacf5\uc774 \uc788\ub294 \uacbd\uc6b0, \uc7ac\ubc1c\uc131 \uae09\uc131 \uc911\uc774\uc5fc\uc778 \uacbd\uc6b0 10\uc77c\uae4c\uc9c0 \uc5f0\uc7a5 \uc0ac\uc6a9\uc744 \uad8c\uc7a5\ud558\uace0 \uc788\ub2e4. \uc7a5\uc5fc, \uad6c\ud1a0 \ub4f1\uc73c\ub85c \uc778\ud574 \uacbd\uad6c \ud56d\uc0dd\uc81c \ud22c\uc5ec\uac00 \ubd88\uac00\ub2a5\ud560 \uacbd\uc6b0 Ceftriaxone (50 mg/kg) \uc8fc\uc0ac\ub97c 1\ud68c 1~3\uc77c \uc0ac\uc6a9 \uc2dc \uacbd\uad6c \ud56d\uc0dd\uc81c\uc640 \ub3d9\uc77c\ud55c \ud6a8\uacfc\ub97c \ubcfc \uc218 \uc788\ub2e4.", "sentence_answer": "\ud56d\uc0dd\uc81c \ub0b4\uc131\uade0\uc740 \uc5b4\ub9b0\uc774\uc9d1\uc5d0 \ub2e4\ub2c8\ub294 \uacbd\uc6b0, \ucd5c\uadfc 30\uc77c \uc774\ub0b4 \ud56d\uc0dd\uc81c\ub97c \uc0ac\uc6a9\ud55c \uacbd\uc6b0, 24\uac1c\uc6d4 \uc774\ud558\uc778 \uacbd\uc6b0 \ub0b4\uc131\uade0\uc758 \uac00\ub2a5\uc131\uc774 \ub192\uae30 \ub54c\ubb38\uc5d0 \uc774\ub7f0 \uacbd\uc6b0 \uace0\uc6a9\ub7c9 \uc544\ubaa9\uc2dc\uc2e4\ub9b0 \uc0ac\uc6a9\uc774 \uc88b\ub2e4.", "paragraph_id": "6490200-1-1", "paragraph_question": "question: 24\uac1c\uc6d4 \uc774\ud558\uc758 \uc544\uc774\uc5d0\uac8c \ub0b4\uc131\uade0 \uac00\ub2a5\uc131\uc774 \ub192\ub2e4\uba74 \uc5b4\ub5a4 \ud56d\uc0dd\uc81c\ub97c \uc0ac\uc6a9\ud574\uc57c \ud558\ub294\uac00?, context: \ud56d\uc0dd\uc81c \ub0b4\uc131\uade0\uc740 \uc5b4\ub9b0\uc774\uc9d1\uc5d0 \ub2e4\ub2c8\ub294 \uacbd\uc6b0, \ucd5c\uadfc 30\uc77c \uc774\ub0b4 \ud56d\uc0dd\uc81c\ub97c \uc0ac\uc6a9\ud55c \uacbd\uc6b0, 24\uac1c\uc6d4 \uc774\ud558\uc778 \uacbd\uc6b0 \ub0b4\uc131\uade0\uc758 \uac00\ub2a5\uc131\uc774 \ub192\uae30 \ub54c\ubb38\uc5d0 \uc774\ub7f0 \uacbd\uc6b0 \uace0\uc6a9\ub7c9 \uc544\ubaa9\uc2dc\uc2e4\ub9b0 \uc0ac\uc6a9\uc774 \uc88b\ub2e4. \ubd80\ube44\ub3d9\uc5fc \ub4f1\uc5d0\uc11c \ud1b5\uc0c1 \uc6a9\ub7c9\uc778 20~40 mg/kg \uacfc \ube44\uad50\ud558\uba74 \uae09\uc131 \uc911\uc774\uc5fc\uc758 \uace0\uc6a9\ub7c9 \ud56d\uc0dd\uc81c \ubcf5\uc6a9 \uc2dc \ucca0\uc800\ud55c \ubcf5\uc6a9\uc744 \ud574\uc57c \ub0b4\uc131\uade0\uc744 \uc5c6\uc568 \uc218 \uc788\uc74c\uc5d0\ub3c4 \ud56d\uc0dd\uc81c\ub97c \uac70\ub974\ub294 \uac78 \uc88b\uc544\ud558\ub294 \ud55c\uad6d\uc778\uc758 \uc2b5\uad00\uc740 \ud56d\uc0dd\uc81c \ub0b4\uc131\uc744 \ub354\uc6b1 \uc545\ud654\uc2dc\ud0b4\uc744 \uace0\ub824\ud574\uc57c \ud55c\ub2e4. \ud56d\uc0dd\uc81c \ubcf5\uc6a9 \uae30\uac04\uc740 \ud1b5\uc0c1 5\uc77c\uc774\ub098 \uc758\uc0ac\uc758 \ud310\ub2e8\uc5d0 \ub530\ub77c \uacb0\uc815\ud558\uba70, 24\uac1c\uc6d4 \uc774\ud558, \uace0\ub9c9 \ucc9c\uacf5\uc774 \uc788\ub294 \uacbd\uc6b0, \uc7ac\ubc1c\uc131 \uae09\uc131 \uc911\uc774\uc5fc\uc778 \uacbd\uc6b0 10\uc77c\uae4c\uc9c0 \uc5f0\uc7a5 \uc0ac\uc6a9\uc744 \uad8c\uc7a5\ud558\uace0 \uc788\ub2e4. \uc7a5\uc5fc, \uad6c\ud1a0 \ub4f1\uc73c\ub85c \uc778\ud574 \uacbd\uad6c \ud56d\uc0dd\uc81c \ud22c\uc5ec\uac00 \ubd88\uac00\ub2a5\ud560 \uacbd\uc6b0 Ceftriaxone (50 mg/kg) \uc8fc\uc0ac\ub97c 1\ud68c 1~3\uc77c \uc0ac\uc6a9 \uc2dc \uacbd\uad6c \ud56d\uc0dd\uc81c\uc640 \ub3d9\uc77c\ud55c \ud6a8\uacfc\ub97c \ubcfc \uc218 \uc788\ub2e4."} {"question": "\ubc15\uc815\ud76c\uc758 516\ucfe0\ud14c\ud0c0\ub97c \uc77c\uc73c\ud0a8 \uac83\uc740 \uc8fc\ub85c \ub204\uad6c\uc600\ub294\uac00?", "paragraph": "\ubaa9\uc0ac \uac15\uc6d0\ub8e1\uc758 \uc99d\uc5b8\uc5d0 \uc758\ud558\uba74 \uc815\ubcc0 \uc9c1\ud6c4 \ubc15\uc815\ud76c\uc758 \uad70\uc0ac \ud601\uba85\uc744 \uc774\ub370\uc62c\ub85c\uae30\ub85c\uc11c \uc88c\uc775\uc774\ub77c\uace0 \ubcf8 \uc0ac\ub78c\uc740 \uac70\uc758 \uc5c6\uc5c8\uace0 \uad70\uc778\ub4e4\uc774 \uc77c\uc73c\ud0a8 \ud601\uba85\uc778 \ub370\ub2e4, 6\uac1c \ud601\uba85\uacf5\uc57d\uc758 \uc81c1\ud56d\uc5d0 \u2018\ubc18\uacf5\uc744 \uad6d\uc2dc\uc758 \uc81c\uc77c\ub85c \uc0bc\uace0 \ubc18\uacf5\ud0dc\uc138\ub97c \uc7ac\uc815\ube44 \uac15\ud654\ud560 \uac83\u2019\uc774\ub77c\uace0 \ubabb\ubc15\uc558\uc73c\ub2c8 \uc544\ubb34\ub3c4 \uc758\uc2ec\ud558\uc9c0 \uc54a\uc558\ub294\ub370, \ucc28\uce30 \uadf8\uc758 \uacfc\uac70\uac00 \ub4dc\ub7ec\ub098\uae30 \uc2dc\uc791\ud558\uba74\uc11c \uc5b8\ub860\uc5d0 \ubcf4\ub3c4\ub410\uace0 \uc724\ubcf4\uc120\uc774 \uc120\uac70\uc5d0\uc11c \uc774 \uc810\uc744 \ubcf8\uaca9\uc801\uc73c\ub85c \ubd80\uac01\uc2dc\ud0a8 \uac83\uc774\ub77c\uace0 \ud55c\ub2e4. \ub2f9\uc2dc 5\u00b716 \uc815\ubcc0\uc774 \uc77c\uc5b4\ub0a0 \ubb34\ub835 \uc870\uc120\ubbfc\uc8fc\uc8fc\uc758\uc778\ubbfc\uacf5\ud654\uad6d\uc740 \uad70\uc0ac\u00b7\uacbd\uc81c\uc801\uc73c\ub85c \uc0c1\ub2f9\ud55c \uc5ed\ub7c9\uc744 \uac16\ucd94\uace0 \uc788\uc5c8\ub2e4. \uc18c\ub828, \uc911\uacf5\uacfc \uad70\uc0ac\ub3d9\ub9f9\ub3c4 \ub9fa\uace0 \uc788\uc5c8\uace0, 4\u00b719 \ud601\uba85 \uc774\ud6c4 \uc870\uc120\ubbfc\uc8fc\uc8fc\uc758\uc778\ubbfc\uacf5\ud654\uad6d\uc5d0\uc11c\ub294 \u2018\ub0a8\uc870\uc120 \uc778\ubbfc\ub4e4\uc774 \ubd09\uae30\ud588\uc73c\ub2c8 \uc6b0\ub9ac\uac00 \ub3c4\uc640\uc57c \ud55c\ub2e4\u2019\ub294 \ub9d0\uae4c\uc9c0 \ub098\uc654\uae30\uc5d0 \uacf5\uc0b0\uc8fc\uc758\ub77c\uace0 \ud558\uba74 \ub2e4\ub4e4 \ubb34\ucc99 \uc608\ubbfc\ud574\uc9c8 \uc218\ubc16\uc5d0 \uc5c6\uc5c8\ub358 \ubc30\uacbd\uc744 \ub4e4\uc5b4 \ubc15\uc815\ud76c\uc758 \uc88c\uc775 \uc804\ub825\uc774 \ucee4\ub2e4\ub780 \ud30c\ubb38\uc744 \uc77c\uc73c\ucf30\ub2e4\uace0 \ubcf4\uc558\ub2e4.", "answer": "\uad70\uc778\ub4e4", "sentence": "\ubaa9\uc0ac \uac15\uc6d0\ub8e1\uc758 \uc99d\uc5b8\uc5d0 \uc758\ud558\uba74 \uc815\ubcc0 \uc9c1\ud6c4 \ubc15\uc815\ud76c\uc758 \uad70\uc0ac \ud601\uba85\uc744 \uc774\ub370\uc62c\ub85c\uae30\ub85c\uc11c \uc88c\uc775\uc774\ub77c\uace0 \ubcf8 \uc0ac\ub78c\uc740 \uac70\uc758 \uc5c6\uc5c8\uace0 \uad70\uc778\ub4e4 \uc774 \uc77c\uc73c\ud0a8 \ud601\uba85\uc778 \ub370\ub2e4, 6\uac1c \ud601\uba85\uacf5\uc57d\uc758 \uc81c1\ud56d\uc5d0 \u2018\ubc18\uacf5\uc744 \uad6d\uc2dc\uc758 \uc81c\uc77c\ub85c \uc0bc\uace0 \ubc18\uacf5\ud0dc\uc138\ub97c \uc7ac\uc815\ube44 \uac15\ud654\ud560 \uac83\u2019\uc774\ub77c\uace0 \ubabb\ubc15\uc558\uc73c\ub2c8 \uc544\ubb34\ub3c4 \uc758\uc2ec\ud558\uc9c0 \uc54a\uc558\ub294\ub370, \ucc28\uce30 \uadf8\uc758 \uacfc\uac70\uac00 \ub4dc\ub7ec\ub098\uae30 \uc2dc\uc791\ud558\uba74\uc11c \uc5b8\ub860\uc5d0 \ubcf4\ub3c4\ub410\uace0 \uc724\ubcf4\uc120\uc774 \uc120\uac70\uc5d0\uc11c \uc774 \uc810\uc744 \ubcf8\uaca9\uc801\uc73c\ub85c \ubd80\uac01\uc2dc\ud0a8 \uac83\uc774\ub77c\uace0 \ud55c\ub2e4.", "paragraph_sentence": " \ubaa9\uc0ac \uac15\uc6d0\ub8e1\uc758 \uc99d\uc5b8\uc5d0 \uc758\ud558\uba74 \uc815\ubcc0 \uc9c1\ud6c4 \ubc15\uc815\ud76c\uc758 \uad70\uc0ac \ud601\uba85\uc744 \uc774\ub370\uc62c\ub85c\uae30\ub85c\uc11c \uc88c\uc775\uc774\ub77c\uace0 \ubcf8 \uc0ac\ub78c\uc740 \uac70\uc758 \uc5c6\uc5c8\uace0 \uad70\uc778\ub4e4 \uc774 \uc77c\uc73c\ud0a8 \ud601\uba85\uc778 \ub370\ub2e4, 6\uac1c \ud601\uba85\uacf5\uc57d\uc758 \uc81c1\ud56d\uc5d0 \u2018\ubc18\uacf5\uc744 \uad6d\uc2dc\uc758 \uc81c\uc77c\ub85c \uc0bc\uace0 \ubc18\uacf5\ud0dc\uc138\ub97c \uc7ac\uc815\ube44 \uac15\ud654\ud560 \uac83\u2019\uc774\ub77c\uace0 \ubabb\ubc15\uc558\uc73c\ub2c8 \uc544\ubb34\ub3c4 \uc758\uc2ec\ud558\uc9c0 \uc54a\uc558\ub294\ub370, \ucc28\uce30 \uadf8\uc758 \uacfc\uac70\uac00 \ub4dc\ub7ec\ub098\uae30 \uc2dc\uc791\ud558\uba74\uc11c \uc5b8\ub860\uc5d0 \ubcf4\ub3c4\ub410\uace0 \uc724\ubcf4\uc120\uc774 \uc120\uac70\uc5d0\uc11c \uc774 \uc810\uc744 \ubcf8\uaca9\uc801\uc73c\ub85c \ubd80\uac01\uc2dc\ud0a8 \uac83\uc774\ub77c\uace0 \ud55c\ub2e4. \ub2f9\uc2dc 5\u00b716 \uc815\ubcc0\uc774 \uc77c\uc5b4\ub0a0 \ubb34\ub835 \uc870\uc120\ubbfc\uc8fc\uc8fc\uc758\uc778\ubbfc\uacf5\ud654\uad6d\uc740 \uad70\uc0ac\u00b7\uacbd\uc81c\uc801\uc73c\ub85c \uc0c1\ub2f9\ud55c \uc5ed\ub7c9\uc744 \uac16\ucd94\uace0 \uc788\uc5c8\ub2e4. \uc18c\ub828, \uc911\uacf5\uacfc \uad70\uc0ac\ub3d9\ub9f9\ub3c4 \ub9fa\uace0 \uc788\uc5c8\uace0, 4\u00b719 \ud601\uba85 \uc774\ud6c4 \uc870\uc120\ubbfc\uc8fc\uc8fc\uc758\uc778\ubbfc\uacf5\ud654\uad6d\uc5d0\uc11c\ub294 \u2018\ub0a8\uc870\uc120 \uc778\ubbfc\ub4e4\uc774 \ubd09\uae30\ud588\uc73c\ub2c8 \uc6b0\ub9ac\uac00 \ub3c4\uc640\uc57c \ud55c\ub2e4\u2019\ub294 \ub9d0\uae4c\uc9c0 \ub098\uc654\uae30\uc5d0 \uacf5\uc0b0\uc8fc\uc758\ub77c\uace0 \ud558\uba74 \ub2e4\ub4e4 \ubb34\ucc99 \uc608\ubbfc\ud574\uc9c8 \uc218\ubc16\uc5d0 \uc5c6\uc5c8\ub358 \ubc30\uacbd\uc744 \ub4e4\uc5b4 \ubc15\uc815\ud76c\uc758 \uc88c\uc775 \uc804\ub825\uc774 \ucee4\ub2e4\ub780 \ud30c\ubb38\uc744 \uc77c\uc73c\ucf30\ub2e4\uace0 \ubcf4\uc558\ub2e4.", "paragraph_answer": "\ubaa9\uc0ac \uac15\uc6d0\ub8e1\uc758 \uc99d\uc5b8\uc5d0 \uc758\ud558\uba74 \uc815\ubcc0 \uc9c1\ud6c4 \ubc15\uc815\ud76c\uc758 \uad70\uc0ac \ud601\uba85\uc744 \uc774\ub370\uc62c\ub85c\uae30\ub85c\uc11c \uc88c\uc775\uc774\ub77c\uace0 \ubcf8 \uc0ac\ub78c\uc740 \uac70\uc758 \uc5c6\uc5c8\uace0 \uad70\uc778\ub4e4 \uc774 \uc77c\uc73c\ud0a8 \ud601\uba85\uc778 \ub370\ub2e4, 6\uac1c \ud601\uba85\uacf5\uc57d\uc758 \uc81c1\ud56d\uc5d0 \u2018\ubc18\uacf5\uc744 \uad6d\uc2dc\uc758 \uc81c\uc77c\ub85c \uc0bc\uace0 \ubc18\uacf5\ud0dc\uc138\ub97c \uc7ac\uc815\ube44 \uac15\ud654\ud560 \uac83\u2019\uc774\ub77c\uace0 \ubabb\ubc15\uc558\uc73c\ub2c8 \uc544\ubb34\ub3c4 \uc758\uc2ec\ud558\uc9c0 \uc54a\uc558\ub294\ub370, \ucc28\uce30 \uadf8\uc758 \uacfc\uac70\uac00 \ub4dc\ub7ec\ub098\uae30 \uc2dc\uc791\ud558\uba74\uc11c \uc5b8\ub860\uc5d0 \ubcf4\ub3c4\ub410\uace0 \uc724\ubcf4\uc120\uc774 \uc120\uac70\uc5d0\uc11c \uc774 \uc810\uc744 \ubcf8\uaca9\uc801\uc73c\ub85c \ubd80\uac01\uc2dc\ud0a8 \uac83\uc774\ub77c\uace0 \ud55c\ub2e4. \ub2f9\uc2dc 5\u00b716 \uc815\ubcc0\uc774 \uc77c\uc5b4\ub0a0 \ubb34\ub835 \uc870\uc120\ubbfc\uc8fc\uc8fc\uc758\uc778\ubbfc\uacf5\ud654\uad6d\uc740 \uad70\uc0ac\u00b7\uacbd\uc81c\uc801\uc73c\ub85c \uc0c1\ub2f9\ud55c \uc5ed\ub7c9\uc744 \uac16\ucd94\uace0 \uc788\uc5c8\ub2e4. \uc18c\ub828, \uc911\uacf5\uacfc \uad70\uc0ac\ub3d9\ub9f9\ub3c4 \ub9fa\uace0 \uc788\uc5c8\uace0, 4\u00b719 \ud601\uba85 \uc774\ud6c4 \uc870\uc120\ubbfc\uc8fc\uc8fc\uc758\uc778\ubbfc\uacf5\ud654\uad6d\uc5d0\uc11c\ub294 \u2018\ub0a8\uc870\uc120 \uc778\ubbfc\ub4e4\uc774 \ubd09\uae30\ud588\uc73c\ub2c8 \uc6b0\ub9ac\uac00 \ub3c4\uc640\uc57c \ud55c\ub2e4\u2019\ub294 \ub9d0\uae4c\uc9c0 \ub098\uc654\uae30\uc5d0 \uacf5\uc0b0\uc8fc\uc758\ub77c\uace0 \ud558\uba74 \ub2e4\ub4e4 \ubb34\ucc99 \uc608\ubbfc\ud574\uc9c8 \uc218\ubc16\uc5d0 \uc5c6\uc5c8\ub358 \ubc30\uacbd\uc744 \ub4e4\uc5b4 \ubc15\uc815\ud76c\uc758 \uc88c\uc775 \uc804\ub825\uc774 \ucee4\ub2e4\ub780 \ud30c\ubb38\uc744 \uc77c\uc73c\ucf30\ub2e4\uace0 \ubcf4\uc558\ub2e4.", "sentence_answer": "\ubaa9\uc0ac \uac15\uc6d0\ub8e1\uc758 \uc99d\uc5b8\uc5d0 \uc758\ud558\uba74 \uc815\ubcc0 \uc9c1\ud6c4 \ubc15\uc815\ud76c\uc758 \uad70\uc0ac \ud601\uba85\uc744 \uc774\ub370\uc62c\ub85c\uae30\ub85c\uc11c \uc88c\uc775\uc774\ub77c\uace0 \ubcf8 \uc0ac\ub78c\uc740 \uac70\uc758 \uc5c6\uc5c8\uace0 \uad70\uc778\ub4e4 \uc774 \uc77c\uc73c\ud0a8 \ud601\uba85\uc778 \ub370\ub2e4, 6\uac1c \ud601\uba85\uacf5\uc57d\uc758 \uc81c1\ud56d\uc5d0 \u2018\ubc18\uacf5\uc744 \uad6d\uc2dc\uc758 \uc81c\uc77c\ub85c \uc0bc\uace0 \ubc18\uacf5\ud0dc\uc138\ub97c \uc7ac\uc815\ube44 \uac15\ud654\ud560 \uac83\u2019\uc774\ub77c\uace0 \ubabb\ubc15\uc558\uc73c\ub2c8 \uc544\ubb34\ub3c4 \uc758\uc2ec\ud558\uc9c0 \uc54a\uc558\ub294\ub370, \ucc28\uce30 \uadf8\uc758 \uacfc\uac70\uac00 \ub4dc\ub7ec\ub098\uae30 \uc2dc\uc791\ud558\uba74\uc11c \uc5b8\ub860\uc5d0 \ubcf4\ub3c4\ub410\uace0 \uc724\ubcf4\uc120\uc774 \uc120\uac70\uc5d0\uc11c \uc774 \uc810\uc744 \ubcf8\uaca9\uc801\uc73c\ub85c \ubd80\uac01\uc2dc\ud0a8 \uac83\uc774\ub77c\uace0 \ud55c\ub2e4.", "paragraph_id": "6417222-45-1", "paragraph_question": "question: \ubc15\uc815\ud76c\uc758 516\ucfe0\ud14c\ud0c0\ub97c \uc77c\uc73c\ud0a8 \uac83\uc740 \uc8fc\ub85c \ub204\uad6c\uc600\ub294\uac00?, context: \ubaa9\uc0ac \uac15\uc6d0\ub8e1\uc758 \uc99d\uc5b8\uc5d0 \uc758\ud558\uba74 \uc815\ubcc0 \uc9c1\ud6c4 \ubc15\uc815\ud76c\uc758 \uad70\uc0ac \ud601\uba85\uc744 \uc774\ub370\uc62c\ub85c\uae30\ub85c\uc11c \uc88c\uc775\uc774\ub77c\uace0 \ubcf8 \uc0ac\ub78c\uc740 \uac70\uc758 \uc5c6\uc5c8\uace0 \uad70\uc778\ub4e4\uc774 \uc77c\uc73c\ud0a8 \ud601\uba85\uc778 \ub370\ub2e4, 6\uac1c \ud601\uba85\uacf5\uc57d\uc758 \uc81c1\ud56d\uc5d0 \u2018\ubc18\uacf5\uc744 \uad6d\uc2dc\uc758 \uc81c\uc77c\ub85c \uc0bc\uace0 \ubc18\uacf5\ud0dc\uc138\ub97c \uc7ac\uc815\ube44 \uac15\ud654\ud560 \uac83\u2019\uc774\ub77c\uace0 \ubabb\ubc15\uc558\uc73c\ub2c8 \uc544\ubb34\ub3c4 \uc758\uc2ec\ud558\uc9c0 \uc54a\uc558\ub294\ub370, \ucc28\uce30 \uadf8\uc758 \uacfc\uac70\uac00 \ub4dc\ub7ec\ub098\uae30 \uc2dc\uc791\ud558\uba74\uc11c \uc5b8\ub860\uc5d0 \ubcf4\ub3c4\ub410\uace0 \uc724\ubcf4\uc120\uc774 \uc120\uac70\uc5d0\uc11c \uc774 \uc810\uc744 \ubcf8\uaca9\uc801\uc73c\ub85c \ubd80\uac01\uc2dc\ud0a8 \uac83\uc774\ub77c\uace0 \ud55c\ub2e4. \ub2f9\uc2dc 5\u00b716 \uc815\ubcc0\uc774 \uc77c\uc5b4\ub0a0 \ubb34\ub835 \uc870\uc120\ubbfc\uc8fc\uc8fc\uc758\uc778\ubbfc\uacf5\ud654\uad6d\uc740 \uad70\uc0ac\u00b7\uacbd\uc81c\uc801\uc73c\ub85c \uc0c1\ub2f9\ud55c \uc5ed\ub7c9\uc744 \uac16\ucd94\uace0 \uc788\uc5c8\ub2e4. \uc18c\ub828, \uc911\uacf5\uacfc \uad70\uc0ac\ub3d9\ub9f9\ub3c4 \ub9fa\uace0 \uc788\uc5c8\uace0, 4\u00b719 \ud601\uba85 \uc774\ud6c4 \uc870\uc120\ubbfc\uc8fc\uc8fc\uc758\uc778\ubbfc\uacf5\ud654\uad6d\uc5d0\uc11c\ub294 \u2018\ub0a8\uc870\uc120 \uc778\ubbfc\ub4e4\uc774 \ubd09\uae30\ud588\uc73c\ub2c8 \uc6b0\ub9ac\uac00 \ub3c4\uc640\uc57c \ud55c\ub2e4\u2019\ub294 \ub9d0\uae4c\uc9c0 \ub098\uc654\uae30\uc5d0 \uacf5\uc0b0\uc8fc\uc758\ub77c\uace0 \ud558\uba74 \ub2e4\ub4e4 \ubb34\ucc99 \uc608\ubbfc\ud574\uc9c8 \uc218\ubc16\uc5d0 \uc5c6\uc5c8\ub358 \ubc30\uacbd\uc744 \ub4e4\uc5b4 \ubc15\uc815\ud76c\uc758 \uc88c\uc775 \uc804\ub825\uc774 \ucee4\ub2e4\ub780 \ud30c\ubb38\uc744 \uc77c\uc73c\ucf30\ub2e4\uace0 \ubcf4\uc558\ub2e4."}